{
"data": [
{
"id": 53988,
"tgt": "Suggest treatment to lower sgpt level",
"src": "Patient: how to lower my sgpt, i,ve done several test bcuz i was sufferring og hep A last march 28 i done my latest test my sgpt is normal my total billirubin is now getting normal but my indirect is still not in normal range but near on normal range. pls help me.. Doctor: FATTY LIVERHi and welcome to Healthcaremagic. Thank you for your query. I am Dr. Rommstein, I understand your concerns and I will try to help you as much as I can.This is possble fatty liver. it can help to avoid drinking. If you are overweight or obese, do what you can to gradually lose weight -- no more than 1 or 2 pounds a week.Eat a balanced and healthy diet and get regular exercise. Limit high-carb foods such as bread, grits, rice, potatoes, and corn. And cut down on drinks with lots of sugar like sports drinks and juice.I hope I have answered you query. If you have any further questions you can contact us in every time.Kindly regards. Wish you a good health.DR. Ivan Rommstein"
},
{
"id": 52006,
"tgt": "Is it because of kidney problem that I have burning sensation in my urethra ?",
"src": "Patient: sir, iam 38y old female.i got uti before 1mon.dr prescribe culture. e-coli org seen.i lltake 10days antibiotic course.but no improvement.atpresent i consult MD DGO,she advise citralkasyp,nitrofurantion 100 10days.atpresent i ll taking 5 days.but burning sensation in my urethra .is it kidney problem sir .i am worried ,pls help. Doctor: Hi dear Welcome to HealthcareMagic. Your doctor has prescribed you a good combination of drugs in treatment of UTI.If you can get an ultrasound done to see kidneys , it can be told about the kidney involvement.Its not suggestive that infection is in kidney just because its taking lung to be treated.Symptoms like back pain , intermittent abdominal pain suggest of kidney involvement.Please drink lots of water(upto 5 liters) to fasten the improvement.Please follow the same treatment. Take care."
},
{
"id": 61013,
"tgt": "What does a lump at an injection site indicate?",
"src": "Patient: hi..um.. okay so i am a dumb person and to start off I want to say I have no intentions of killing or harming myself or being a drug addict but um.. I watch greys anatomy and I wanted to practice poking a needle so I did it on myself and I accidentally injected lidocaine hci 2% into the top of my hand not very much but im scared because there\u2019s a small thick lump and it\u2019s more pale then the rest of my hand please help.. Doctor: Hello,It is possible due to\u00a0-\u00a0The anesthetic\u00a0effect of the\u00a0lidocaine\u00a0- developing\u00a0cellulitis\u00a0or abscess.\u00a0-Doppler\u00a0ultrasound\u00a0recommended for further management guidelines.\u00a0Hope I have answered your query. Let me know if I can assist you further.Regards,\u00a0Dr. Bhagyesh V. Patel"
},
{
"id": 49248,
"tgt": "What is the reason for loss of concentration, forgetfulness after having dialysis?",
"src": "Patient: I do dialysis by way of a cycler machine every night for about 8.5 hours....... I have noticed a huge difference in my mind and how it works now. sounds silly but very true. i am very forgetful now and i jumble my letters up when i type or text almost like I have dislexia . sometimes I catch it and sometimes i dont. others have noticed this change in me too. I have troubles concentrating also. I NEVER had these issues before dialysis. could this be a cause ? im not on meds for depression ect ... just dont know what else it could be. any suggestions would be great ! Thanks for your time . Val Doctor: **1. Progressive dementia [dialysis dementia] due to aluminium toxicity has been described from certain parts of Europe where the levels of aluminium were high in the water used for dialysis, although this is rarely seen now.2. since patient on dialysis suffer from secondary hyper parathyroidism [excess secretion of parathormone PTH], thus effect of this is '\"Psychiatric disturbances\" [trouble concentrating] you need to check with 24 hours urinary excretion of calcium and phosphorus along with relevant Imaging [if needed]PS. if you feel trouble typing letter, make sure to check with your Blood urea level and LFT [liver function tests] beside Neurological examination."
},
{
"id": 122212,
"tgt": "What is causing the acute pain after the surgery of spiral fracture?",
"src": "Patient: On 1st July 2011 I had a spiral fracture to my 5th metatarsal they waited 4 months insisting it will grow as I am a healthy 30 year old non smoker, non diabetic, highly active female. Deciding on surgery they discovered a muscle stuck in the brake which they removed and pinned the fracture with a plate. Once anesthetic had worn off I was in acute pain, with nerve shocks firing through my foot. 3 months on following the prescriptions, physio and pain clinic I have been diagnosed with CRPS and am still in acute pain. Can the plate be removed, will it help me please advise as no treatment is helping and this simple brake 7 months ago is debilitating and ruining my life? Doctor: Hello, The acute pain that you are having can be due to the plate. Getting the plate out can help but you need to get an X-ray done first to find out the extent of healing. Hope I have answered your query. Let me know if I can assist you further. Take care Regards, Dr Praveen Tayal, Orthopaedic Surgeon"
},
{
"id": 216161,
"tgt": "Suggest remedy for swelling and pain in knee",
"src": "Patient: I am a female age 50 my left knee has been hurting for the last couple of weeks, past two days knee has been swollen and severe pain, at night pain gets worse and is through out my entire leg, I ve tried ice and take ibuprophen for pain. Just last night I noticed a ball on my calve area about the size of a quarter. having a very hard time walking. I am diabetic and do have high blood pressure. I am not overweight and I try to keep active (walk 3-5 miles a day). Not sure if this is something I should go to urgent care to see what is going on. I do not have health insurance so they tend to make you feel like you are not worth the time to check out thouroghly. If I go is there any questions or test I should ask them to run on me. Doctor: Dear patient you.may be having early osteoarthritis. This means age related degenerative changes in knee. Overweight persons tend to get it early. There will be difficulty in squatting. Diagnosis can be confirmed by xray of the left knee anteroposterior lateral.and skyline views. Please get it done from radiology centre nearby you. Meanwhile start tab ultracet twice a day for pain relief. Avoid squatting and crossed leg sitting. If xray is abnormal please visit orthopedic surgeon nearby you."
},
{
"id": 155585,
"tgt": "What can cause a prolonged breast pain and a sore nipple?",
"src": "Patient: hello i am 53 years old, had a partial hysterectomy and had breast pain for about 3 months now my nipple is real sore to touch and scaly looking and where my shoulder blade is is hurts too all the time i have put off going to the dr. i m real worried about cancer i really don t feel a knot it is just real painful near my armpit so you think i have cancer i made a appointment for next week Doctor: Hi and welcome to HCM. THanks for the query. you need to do ultrasound and mamography to evaluae this condition. in most cases this is benign conditions but since you had cancer then you must be more catious.Wish you good health. Regards"
},
{
"id": 101133,
"tgt": "What causes a ammonia smell while eating ?",
"src": "Patient: After a week long cold (bronchial and sinus), I have a very strong ammonia smell when attempting to eat. I have been taking over the counter chest congestion med with guaifenesin and dextromethorphan. What would cause the smell? Only I can smell it by the way. Doctor: HIThank for asking to HCMI really appreciate your concern and looking to the history given here I could say that if you have the ammonia like smell then this could be due to the sinus infection and in my opinion better to have Tab Loratidine with pseudoephedrine three times in day, drink lot of water and have a steam inhalation, take care and have a nice day."
},
{
"id": 150717,
"tgt": "Blood clot in head, treated, tasteless tongue. What is the remedy for this?",
"src": "Patient: Hello Doctor, Last month, My mother-in-law had blood clot in her head. She was treated in the hospital. She is fine now. But She is having tasteless tongue . Whatever food she takes in, she feels like vomiting . At present, she is just having bread, biscuits, liquids, fruits & puddings. Kindly suggest remedy for this. Thanks. Doctor: Hi, Thank you for posting your query. First of all, it is good to note that your mother in law is recovering well after a brain stroke. Tasteless tongue is unlikely to be related to blood clot in brain. However, if you can upload the CT or MRI brain report, it would be helpful. Taste involvement can also be due to side effects of certain medications, such as antibiotics (metronidazole), iron, etc. So, if you can send a list of medications your mother in law is taking currently, it would help. Best wishes, Dr Sudhir Kumar MD DM (Neurology) Senior Consultant Neurologist Apollo Hospitals, Hyderabad"
},
{
"id": 187262,
"tgt": "Is weakness considered normal after tooth extraction?",
"src": "Patient: Hi, just had my second biggest tooth removed, it was a root canal as well...i did not take painkiller afterwards, and now its the 3rd day, the pain is almost gone, but i still feel very weak at times, is this considered normal reaction after tooth extraction? Doctor: Hello, Thanks for your query.In the event of nausea and/or vomiting following surgery, do not take anything by mouth for at least an hour including the prescribed medicine. You should then sip on Coke, Spritete, Gatorade, or ginger ale. You should sip slowly over a fifteen-minute period. When the nausea subsides you can begin taking solid foods and the prescribed medicine.I do hope that you have found something helpful and I will be glad to answer any further query.Take care"
},
{
"id": 176484,
"tgt": "Suggest treatment for obesity in kids",
"src": "Patient: My bestie Louise is 11 years old, 160 cm (5 3) tall, and 164-180 pounds ( 70-80 Kg ) , and she is in the obese percentile when i checked on internet... \u0130m really worried about her becuase she always starves herself to 350-500 calories a day and then binges all day for the most 1 week later and then she starts again.She is not a bulumic because she doesnt puke after for sure. We have a lot of sleepovers like nearly 3-5 times a week that may sound to much but its becuase we are neighbours and her mum works all the time and we go to the same school. she gained 10 kilos by doing this for 1and a half years, that is really freatining. She diesnt believe she can lose weight by eating healthy becuase about 2 years before she starved herself and lost alot of weight , that made her try to do it more and more but she always failed at it. \u0130m really worried about her i dont want. to lose her and i dont know ehat her condition is called... please help she was crying yesterday on the phone becuase she was to fat... Please help \u00f6e so i can help her... Doctor: For all kind of conditionFirst of all you make a proper diet chart for week...Start early morning honey with lukewarm water.no heavy diet or junk food...Plenty of water should be start (3-4 lit/day)In diets you adds more fiber diet.In homoeopathy you can go with Tablets Phyto berry 2 tablet with lukewarm water three times a day for 3-4 months.....before start treatment 1stly check u r weight."
},
{
"id": 72654,
"tgt": "Suggest treatment for lungs and breathing problem",
"src": "Patient: sir, my son vasanth age-17 , he is suffering from some lungs problem. some times he affect heavy breathing problem. its comes one or two times in every year. (in this time no cold ). but he bathing daily in cold water, goes in very cold weather condition without wearing any masks.drinking only normal coald water. i want best treatment for him . what can i do sir? Doctor: Thanks for your question on Healthcare Magic.I can understand your concern. Recurrent breathing difficulty in young patient (17 years) is mostly due to allergic asthma.So better to consult pulmonologist and get done clinical examination of respiratory system and PFT (Pulmonary Function Test).PFT will not only diagnose asthma but it will also tell you about severity of the disease and treatment is based on severity only. He will need inhaled bronchodilators (formoterol or salmeterol) and inhaled corticosteroid (ICS) (budesonide or fluticasone).Don't worry, he will be alright with all these. Tell him not to drink cold water. Hope I have solved your query. I will be happy to help you further. Wishing good health to your son. Thanks."
},
{
"id": 42893,
"tgt": "Suggest treatment for infertility",
"src": "Patient: Hi, im married for 2 year and im 28 yrs. i have been daigonised PCOD 5 yrs ago, i was on medication for some time, i was taking metapage for 6 mnths and 3 cycles of daine 35 and took tertab once, but din work, i was 62 kilos, ow i have lost 5 kilos, so will my fertab drug work on me if i take it from 3rd day onwards ? Doctor: HAIWELCOME TO HCMYou need TO CHECK DAY 2 FSH,LH,PROLACTIN,THYROID ,SERUM FASTING AND POSTPRANDIAL INSULIN . FOLLICULAR STUDY WITH OVULATION INDUCTION will reveal your ovulation status.try 6 cycles of ovulation induction with IUI,IF IT FAILS you need ivf treatment."
},
{
"id": 178290,
"tgt": "Which milk is the best for babies from 6 to 12 months?",
"src": "Patient: HI DOCTOR, would like to ask if what is the best milk for babies 6-12 months? I was using mixed....because im working for now i feed her with nan pro one...do i need to continue the milk?doc is it true that much prefer to use milk which can help to develop the brain? Doctor: At 6 months of age, it is already time for weaning. It is basically a transition to the adult diet. At this age milk is simply not adequate to meet the nutritive requirements of the body for which semi solids need to be introduced. If you give milk, it is just an optional element and you can give any milk that the child can tolerate. NAN pro 1 is for children below 6 months. You may give NAN 2. Rather, give any sort of adult food low on spices in a mashed up form. Give a boiled rice based diet. Add pulses and vegetables to it. You can also add chicken or eggs in mashed form. You can give fruits in grated form. No need to boil fruits if they can be otherwise converted to semi solid form. Instead of formula feeds, these are more nutritive and should be given. I hope that these will help the baby gain weight. For brain development, omega 3 fatty acids play an important role. These are found in sea fishes. I would prefer those rather than opting for supplements."
},
{
"id": 205424,
"tgt": "Suggest remedy for anxiety when diagnosed with somatic-type delusional disorder",
"src": "Patient: hi sir i am doctor and i have a person not a patient in reality, and he use to come to me daily and tells lot of complains but in reality he does not have those things, his sleep is also disturbed and he is having also anxiety, and i have come to the conclusion to reffer that person to psychiatrist as i have made diaganosis of SOMATIC DELUSIONAL DISORDER, sir should i refer him to psychiatrist and what precisely i should write on his refferal please Doctor: Hi sir , thanks for your query.You can write complaint of person and do routine investigations and write no obvious organic cause found.refer to psychiatrist for proper evaluation and management."
},
{
"id": 35013,
"tgt": "Suggest treatment for painful, smelly infection on chin",
"src": "Patient: I have a painfull, smelly infection on my chin. It started off to appear as a pimple. I popped it and that I was done with it. However, a week later it has doubled in size and now smells. I no longer have any discharge from it other that what appears to be water. Doctor: Hi,It seems that there might be having bacterial infection giving this problem.Go for one antibiotic medicine course for 5 days like Amoxicilin or Doxycyclin.Do not try to prick it as it might aggravate the condition.Ok and take care."
},
{
"id": 88258,
"tgt": "Suggest treatment for stomach pain and fever",
"src": "Patient: I was in Mexico last week with a friend. We both woke up with an upset stomach one day. Her's kept her in bed all day. My stomach felt off but I was able to get out of bed. Since then, about a week, I have had what feels sort of like a stitch in my abdomen on the left side. I've been in bed with a fever twice. Doctor: Hi.Thanks for your query. This looks to be due to gastroenteritis. Both of you should have taken a course of an antibiotic and metronidazole and probiotic. If there is a complete relief no need to worry. If the problem remains then I would suggest you the following:get blood, urine and stool tests and ultrasonography done to see the results and get treatment according to the reports."
},
{
"id": 129203,
"tgt": "Is there any recent article about new bone density medication trials?",
"src": "Patient: hi. a few days ago there was information about a new bone density medication still in trials. can you point me to the article? my mother is suffering from bone degeneration side effect currently and thought this might be of interest to her as she is still taking med that caused problem. thank you. Doctor: hi youI'm Dr RohitOrthopaedicianI'm pleased to answer you regarding the new drug for the bone problemI think what you are talking about is bond is losing its mineralisation and it is becoming weakI would recommended you to give your mominj Teriparatide subcutaneous injection dailyit increases bone density and reduces resorption of boneit is a recombinant PTH (PARARTHYROID HORMONE)"
},
{
"id": 129400,
"tgt": "What is the cause of tight feeling from knee cap to shin?",
"src": "Patient: I had ACL reconstruction 6 months ago.I have been working out daily to strengthen muscles, lose weight etc.. I now have pain on the front of my knee..from my knee cap to shin ..it feels very tight when I bend my knee and walk stairs.. and my knee pops. Doctor: Hello and Welcome to \u2018Ask A Doctor\u2019 service.I have reviewed your query and here is my advice.I guess you had the ACL reconstruction with patellar tendon that means a piece of bone from your knee cup was taken as a graft. That has left a hole on you knee cup that takes some time to fill out and sometimes has disturbing effects. Another complication that may arise from this surgery is knee cup fractures as the knee cup weakens after taking some bone from it.I suggest you to do an x-ray of the knee to exclude the fracture of the knee cup and then see the orthopedic doctor.Wish you a quick improvement.Hope I have answered your query. Let me know if I can assist you further.Regards,Dr. Edvin Selmani"
},
{
"id": 225972,
"tgt": "Switched to Sprintec from Lo Loestrin Fe. When will period start?",
"src": "Patient: Im switching from Lo Loestrin Fe to Sprintec 28.From what I have researched i have had \"withdrawal bleeding\" for 5 days. I took that last pill in my Lo Loestrin Fe today, and should start my actual period tomorrow. However, I have not yet. So should i start my new pack of Sprintec 28 tomorrow or wait till i have started my actual period? Doctor: Thanks for using HCMYou can start taking tablet regularly, withdrawal bleeding should have started by this time, if not started yet get UPT done just for your satisfaction. Take tablets regularly. Have a good health.RegardsDr. Vidya"
},
{
"id": 19902,
"tgt": "Why does the heart rate go up when jogging?",
"src": "Patient: My heart rate goes above 170 when jogging at 6.0 with 1.5% incline on the treadmill. Seems high and way above my \"target zone\" If I went just by my target zone, I would have to walk the entire time. I don't feel too stressed when I am running, but feel like I need to slow down. If I do sprint, I am sure it would be off the charts. I have been jogging for about 2 months now ever other day and was hoping it would maybe go down. Also, this is interval training. Walk for lap, jog for 2 walk a lap, jog a mile, walk a lap, jog a half mile, walk a lap job a lap. So a total 2.25 miles and running 1.5 miles. When I walk in between it goes down about about 137-144 which is still over what my target zone is supposed to be. Suggestions? Doctor: HelloIts physiologically normal for the heart rate to go high during work outs. Its normal to achieve your target heart rate and there is nothing wrong even if your rate is going above THR. Some people do have a higher sympathetic drive due to which there rate goes beyond there THR.Dont worry.Regards"
},
{
"id": 111720,
"tgt": "What are the medicines to get relief from chronic back pain?",
"src": "Patient: I have a chronic back pain I have no discs left in my lower back and several other discs in my neck and back are bad I have had all kinds of injections and they just seem to make the pain worse my doctor only wants to give me morphine but I can`t take it and they know that so how can I get pain relief Doctor: HiPlease tell me,what is the cause ofyour suffering and age?Do you have difficulty in walking and pain/numbness in lower limbs?My advice would be regular spine excercises(under guidance).You should have diet rich in calcium and vitamin D.Regarding medicine for backache my advice is to take nasal spray of calcitonin after discussion with your treating doctor.Take care and wish you good health"
},
{
"id": 53575,
"tgt": "Suggest treatment for cholangitis",
"src": "Patient: I have recently been diagnosed with cholangitis by my gastro dr but was not prescribed antibiotics. My doctor said ERCP would be the treatment, however, I was hospitalized for 34 days last year with pancreatitis and an ERCP at the time inflamed that condition consideralby. I was wondering if I should ask my dr if I should be on antibiotics, he set me up for a follow up in 4 months to have a blood test done at that time. The cholangitis diagnoses came up as a result of a liver biospsy I had done last month. Thanx p.s. He diagnosed me with crhonic pancreatitis last year. Doctor: Hi, with diagnose Cholangitis, you need to visit your doctor as soon as and take antibiotics, ERCP is most popular diagnosing and treatment method on Cholangitis. Did he diagnosed you after USG ?"
},
{
"id": 86236,
"tgt": "Suggest remedy for abdominal twitches while having IBS",
"src": "Patient: hi there, over 5 months ago i had to take the morning pill through fear that my contraception pill hadnt worked. i had sex on the 5th day of my period and took the emergency pill 13 hours later. since then i have had all periods, no morning sickness no cravings etc. however 13 weeks later i started feeling twitching in my tummy. i took a pregnancy test which came back negative. 6 weeks later i was still getting the twitching. took another test which was still negative. i am still getting the twitching feeling along with pain and bloating. i suffer from ibs but it has never felt like this before. as i have said i have not missed a period but have read that this can happen. i am a tiny uk size 6 with no bump but am really worried. thanks in advance Doctor: Hi.Thanks for your query.Read and understood that you had taken morning after pill to avoid pregnancy, have normal periods since then, still worried about pregnancy as you have fluttering in abdomen.Since you are a known case of IBS. This can present like this, although this is for the first time for you.I would advise you to go for tests of blood, urine and stool. Ultrasonography will help to rule out many of the problems and a clinical diagnosis will give a plan for the treatment."
},
{
"id": 134290,
"tgt": "Suggest treatment for sprained ankle",
"src": "Patient: I severely sprained my ankle and fractured ankle bone in foot five weeks ago . I was in non weight bearing plaster cast for three weeks. I was not very good about using crutches and was walking tip toe which didn t hurt. Cast has been off for one week now and I can t bend my foot to walk. It is also very painful to put pressure on heel. I am in a brace now but still can t walk on foot besides on my toes. I start pt Thursday. What are your thoughts? Doctor: Hi thereAfter 4 or 5 weeks of a plaster there is always some stiffness that set in at the ankle joint simply because the ankle joint had been imobilized for a while. This is an expected Complication. now that you are in a brace you need to work towards regaining movement at your ankle joint. physiotherapists can help you with this. Do take it easy and slowly. After a significant ankle injury it can take you up to 2 years for a full recovery."
},
{
"id": 111093,
"tgt": "Suggest treatment for consistent pain at the back",
"src": "Patient: dear sir, i am suffering from back pain for the last 8 months. I have cosulted 5 doctors till now. but there is no improvements. it started in the month of may on a sudden point. then it got worst over the time. i am working as a territory service manager in Maruti Suzuki India ltd. My job includes sitting in front of laptops and driving. Use to drive at leat 2000-3000km on monthly basis and use to sit atlest 8-9 hours in office. in my xrays there was no issue with the spines. kinldy help in resolving the issue. Doctor: Hello,I had gone through the case and advised you to go for MRI of spine and Vitamin D3 test.Take calcium rich food and Vitamin D3 once a week.Do back and leg exercise for strethening the muscles.Hope my answer will be effective for you.Thanks"
},
{
"id": 89856,
"tgt": "What causes abdominal pain with congestion in chest and throat?",
"src": "Patient: My daughter has severe pain in abdomen, which travels upwards and makes her feel congestion in chest and throat and finally giddiness and blackout. Pain is severe and the attacks come 2-3 times a day and lasts for 30-45 minutes, mostly an hour after lunch and dinner. Doctors think it is GERD and have ruled out most other likely causes like appendicitis, gall bladder stone, pancreatitis, kidney, ovarian cyst, heart related, lung infection etc. she is aged 9 +. And is not improving even after being on Omeprazole, Debridat and Domperidone for past 4 days. Really worried whether we should wait as the docs suggest this takes 7-10 days to get well or if we should do any additional thing. Doctor: The symptoms you mention fit into gerd. You have to continue the medications as advised by the doctor. Try to give small and frequent meals. Let her sleep in elevated position. If the symptoms persist even after 3 to 4 days do a repeat chest radiograph and ultrasound scan of the abdomen."
},
{
"id": 112352,
"tgt": "Experiencing sharp pains in middle right side of back. What should i do?",
"src": "Patient: I HAVE BEEN HAVING SHARP PAINS IN THE MIDDLE RIGHT SIDE OF MY BACK FOR 2 WEEKS. IT FEELS LIKE A MUSCLE SPASM AT TIMES, HARD TO INHALE TOO DEEP AND IF I SNEEZE OR BEND DOWN STRETCH OR TURN THE WRONG WAY. NO INJURIES OR CAUSE THAT I CAN THINK OF UNLESS I LIFTED SOMETHING TOO HEAVY. I HAVE BEEN USING IP ABOUT 600 MG 3 TIMES DAILY. I DO NOT DO WELL WITH MUSCLE RELAXERS SO I HAVE NOT USED ANY. IT HAS EASED UP BUT AFRAID TO AGGREVATE IT AGAIN. SPASMS HURT VERY MUCH WHEN THEY OCCUR. Doctor: hello dearthanks for your query at HCMsharp pains and even on inhalation might be due to muscle spasm only.anti inflammatory drugs like Ip are usefulapply hot compression in area of cramps.if still pain not releaved you can take tab myoril 8 milligrams if pain is untolarable and disturbing your daily activities.do not take it regularly , might cause sleepy feeling.hope this will help you.regards"
},
{
"id": 23857,
"tgt": "Suggest treatment for blood pressure level 98/58",
"src": "Patient: i just came down off my roof, I am doing roofing. My blood pressure is 98/58 my hart rate was 88. my blood pressure has been in doubel digets, should I worry. I am 57 years old my weight is 280 I have dibetes which I am controling at the present. I take a blood pressure pill which is enepreil. Doctor: Typically this blood pressure is not a concern if you are without symptoms and if you are not on any blood pressure medications. See your provider about reducing the dose your medication, your blood pressure does NOT need to be this low. Good luck."
},
{
"id": 200248,
"tgt": "What cause very bad itch around perineum and base of penis?",
"src": "Patient: I have a very very bad itch that comes and goes around my perineum, and base of my penis. With loss of pigment in the most itchy areas. It feels amazing when I scratch, until I break the skin and cause pain. Any of ideas what it could be? I shower everyday, dry it very good, but it doesn t seem to help. Doctor: HiThank you for asking HCM. I have gone through your query. Your problem can be most likely due to dryness and fungal infection. Avoid using soaps and irritants to wash the genital area and instead you can use saline or warm water. Using ointments with combination of antifungal like miconazole and corticosteroid like flucinolone will be helpful to get rid off this . Also avoid using tight underwear. Hope this may help you. Let me know if anything not clear. Thanks."
},
{
"id": 90184,
"tgt": "Suggest treatment for lower abdominal pain",
"src": "Patient: A few months ago I had a pain attack in my lower right abdomen, it was umbearable... dr's couldnt even apply any pressure to the area without me screaming in pain, i went to hospital the got rushed to another hospital as the thought it was accute apendicitis. I was given morphine and after about 2 hours the pain went away completly. Ever since this attack i have had many small bouts of pain in the same are... no where near as bad but uncomfortable, along with the pain it also feels like there is some preasure in the area aswell. 2 GP's i have seen have just said to take pain releif and rest, without doing any tests... should I be concerned.? Doctor: hello ! Welcome to health care magic.1. You have not mentioned if you have ultrasound abdomen done o not? 2.If you have not gone through, then i would recommend as soon as possible. 3.Your pain looks like for me as kidney stones, They do give similar symptoms.4.Other thing like cholicystitis/colilithiasis (Gallbladder infection/stones)5.Request your GP for above test if not done.Any questions ? do not hesitate to ask. Thank you"
},
{
"id": 159805,
"tgt": "Can cancerous cyst break during operation and the cancerous cells spread to other organs of the body?",
"src": "Patient: Hello Doctors. I have a very confusing question to ask. Hope you can give me a good answer. There was a cysts of about 10 cm in my mum s uterus and she just removed it last week. The doctor in charge of my mum s case told her that the cysts is actually a cancerous one. It is about stage 2 going to 3 and he advice my mum to go for 6 chemo . My mum was really afraid and we went to several doctors and they gave the same advice too. Yesterday, we met another doctor, and he said that, during the operation, the cysts is supposely remove together with the uterus but the doctor in charge of the operation broke the cysts and causing the content of the cysts flows out. That s why the operation extended for about an hour because of the cleansing inside the stomach. The doctor now advice her to go for the treatments is to prevent the cancerous cells from growing and spread to another organ . I just browse the web and found that there is a Dr.Simoncini in youtube, he taught the people to take sodium bicarbonate instead of going for the chemo. Now, can this be trusted? Doctor: If we believe your doctor that your mom has stage two cancer she should have undergone total abdominal hysterectomy and bilateral salpingo oophorectomy which mens complete removal of uterous and both ovaries and tubes. After she recovers from surgery she needs external pelvic irradiation because you have to sterlize the pelvic lymh nodes whose possiblity of positivety is 25 to 50 percent. it is around 45 to 50 GY in 4 to 4.1/2 weeks time followed by brachy therapy to treat the vaginal vault. This is the domain of a Radiation Oncologist. after this according to grade of the tumor she can be put on Adj. chemotherapy or simply hormone therapy. The concerned Oncologist will decide what should be the actual management of the disease.The disease can by monitored by doing marker assays before treatment and after treatment which is the level of CEA and Ca 125. To me the stage two cancer of the body uterous is completely curable after complete treatment."
},
{
"id": 40238,
"tgt": "What could cause persistent Strep throat despite taking antibiotics repeatedly?",
"src": "Patient: I've had strep throat since Christmas eve and I'm on the fourth round of antibiotics now (30 erythro-base 250mg). The test came back positive everytime. I don't have mono. Now I'm feeling numbness throughout my body, along with weakness and some dizziness. Is this a result of taking antibiotics for too long? Also, why won't the strep go away? Doctor: HelloIn my clinic I have noted that few strain of streptococcus become resistant to erythromycin , so this may be in your case.As you mentioned that strepto is positive , so get in drug sensitivity test and take another most sensitive antibiotics .In such type of cases I prescribe Peniciline ( benzathine ) 12 lac unit intramuscular on every 15 days .Culture and drug sensitivity test from throat swab is the best measurement in this type of cases . So consult an E N T and get his opinion ( 2nd one)."
},
{
"id": 178541,
"tgt": "Suggest treatment for shoulder pain and fever",
"src": "Patient: My son, age 7, got up at 2 am this morning crying with shoulder pain. I questioned him to see if maybe he had fell at school or bumped it in any way during the day. He said no and said it had hurt all day at school. This is a child that does not complain about being sick or hurting. His body felt really warm but his face was cool. I gave him some advil for pain and laid down with him until he fell back asleep. I then woke him up for school at 6:30 and he didn t mention his shoulder until we were on our way to school and he said his shoulder really hurt. The school nurse called me at 11 and said Cooper had a fever of 102.2. When I picked him up his color was off, he said his side and tummy hurt and his head hurt. The nurse said there was a fever virus going around and that was probably it. He has not had fever since about 1 but I have a concern about his shoulder hurting all of a sudden. Thank you for your time. Paula Doctor: Hi....this looks like a reactive arthritis or a transient synovitis. With fever going on along with so intense shoulder pain in a child who you say is very tolerant and never complains.... I would also like to rule out infectious synovitis.I suggest you consult your pediatrician or pediatric orthopedic surgeon.Regards - Dr. Sumanth"
},
{
"id": 36134,
"tgt": "What causes red pimple like rash on my neck, shoulders, back?",
"src": "Patient: red pimple like rash on my neck, shoulders, back, and moving up onto my face... doesn't itch, had it before, but this time it seems more persistent... had it for a couple of weeks now... I've tried corticosteroid cream with urea and that seemed to have some initial success but it's losing its effectiveness Doctor: IT MAY BE INSECT BITE.KEEP IT CLEAN WITH SOAP & WATER.YOU CAN APPLY CALAMINE LOTION.CONSULT YOUR DOCTOR FOR PHYSICAL EXAMINATION.TREATMENT DEPEND ON CAUSE."
},
{
"id": 139483,
"tgt": "What causes flares in the lumbar and spine?",
"src": "Patient: Hi, after my hysterectomy over a year ago my lower lumbar area to the left of the spine flares up , gets tender to the touch and excruciating pain which runs down hip leg and outter foot. MRI was read normal. When this happens, I can t sit or stand for too long and my bowel movements are greatly affected. Xrays are normal as well I need answers! I have been getting trigger point injections in the area which helps for a period of time. Tylenol, Ibuprofen, heat and Ice don t touch it either. Doctor: Hi, The symptoms are suggestive of pinched nerves in the lower back. This causes low back pain and radiating pain to the legs. MRI is reported normal, however please upload it, so that I can personally double check. The initial treatment is medicines and physiotherapy. Drugs like pregabalin are very helpful. Hope I have answered your question. Let me know if I can assist you further. Regards, Dr. Sudhir Kumar, Neurologist"
},
{
"id": 143391,
"tgt": "What causes memory impaired for middle back tenderness?",
"src": "Patient: Hi, I have middle back tenderness, and pain on gentle pressing of the spine specifically on one vertabrae. Also it feels that this vertabrae is very slightly enlarged, although not visibly. I am obviously concerned about this, but more so as lately my memory seems a bit impaired, and I am concerned it is related. What are the possibilities please? Doctor: Hello!Welcome and thank you for asking on HCM!Regarding your concern, you should know that the back pain and the tenderness while touching could be related to chronic degenerations of the spinal column. As the pain does not radiate and is localised only in the back and at one point, it seems to be just inflammation and not a herniated disc. Coming to this point, I would recommend a thoraco-lumbar column X ray study and inflammation tests (PCR, sedimentation rate, complete blood count). Regarding the memory problems they are not related to the back pain. Working too much or lack of a balanced diet could be the causes. I would recommend regulating your daily regimen (alternate work with relax pauses, try to have a good sleeping hygiene) and take a balanced diet (full of vitamins, proteins, lipids and carbohydrates). Anxiety could also trigger all this situation. Hope you will find this answer helpful!Best wishes, Dr. Aida"
},
{
"id": 158305,
"tgt": "History of mastectomy due to adenocarcinoma. Bone scintigraphy shows increase in osteoblastic activity. Meaning?",
"src": "Patient: Greetings in Trinitian Spirit! Last 2000 , I had rt breast mastectomy , it was stage 2-3 adeno carcinomma and I had chemotheraphy for 6 sessions.I was free from cancer up to 2008. Had remission last July 2009 , it metastasized to the lungs and bones. This is my latest bone scintigraphy , it revealed mild increase in osteoblastic activity in the posterior 6th right rib, T8-T9 and both hips. Stable osteoblastic activity in the anterior 5th right rib, T10. L2 and L 4 vertebrae. What does it mean Sir? Thank you for your time in aswering my inquiry. Doctor: Hi and thanks s much for using our panel to get answers to your medical questions. I am so sorry to hear about this battle with breast cancer and the recent scan and all the unanswered question this has raised in your mind.This reports shows that there is synthesis of new bone in these areas. This is highly suggestive of breast cancers metastasizes. I am sorry about this. But then we solve our problems by identifying them and not shying away. So, we can now tackle this problem.You will most likely get some more testing eventually another round of chemotherapy, hormonatherapy and ay be radiotherapy to address these lesions. I think meeting and talking with your oncologist at this time is the best way to go.I know you must be devastated by this but I am advising that you stay even stronger in the face of adversity if you must win and i know you can win.I wish you a complete resolution of symptoms.Chobufo, MD"
},
{
"id": 199232,
"tgt": "Suggest medication for schitzo effective",
"src": "Patient: Hi , My son has schitzo effective, for 12 yrs he has been on resperdone, recently he has started hearing voices again, which was gone until his doctor put him on additional meds. they are latuda and zyprexodone(spelling might be wrong) his symptums are getting worse, not better Doctor: DearWe understand your concernsI went through your details. Schizo affective disorder is usually treated with a combination of psychiatric medicines, psychotherapy, life style changes, meditation and exercise. if you feel the present medicines are not working as required, please approach another psychiatrist.If you require more of my help in this aspect, please use this URL. http://goo.gl/aYW2pR.Make sure that you include every minute details possible. Hope this answers your query. Available for further clarifications.Good luck."
},
{
"id": 81182,
"tgt": "What causes shortness of breath?",
"src": "Patient: Hi, my husband has been having trouble with shortness of breath. It usually occurs after he s eaten breakfast but will happen off an on throughout the day. He has had all kinds of allergy testing through the VA but they haven t been able to come up with anything. It is wearing on his health but we don t know what route to take from here. Doctor: Thanks for your question on HCM.I can understand your situation and problem.In my opinion you should first consult pulmonologist and get done1. Chest x ray2. PFT (pulmonary function test).Shortness of breath can be due to bronchitis and lung infection.So PFT is needed to rule out bronchitis.Chest x ray is needed to rule out lung infection.If both are negative than cardiac cause shoukd be rule out for breathlessness.So get done ECG and 2d echo to rule out cardiac cause. If everything is normal than no need to worry much. Shortness of breath can be due to anxiety. So ask him to avoid stress and tension. He will be alright."
},
{
"id": 192652,
"tgt": "Is sperm count of 65 million/ml normal?",
"src": "Patient: my husband has a sperm count of 65 milliom/ml with active sperms 30%, sluggish 10% and non motile 60%. the viscosity is normal and pH 8 with liquefaction time 20 minutes. abnormal forms < 15% and pus cells 4-6 /HPF. can u please tell me if it's a normal sperm count? Doctor: Hello, The count is normal but the motility (ability to move and fuse with eggs) is slightly on the lower side. Consult a urologist and get evaluated. Possible causes like varicocele must be ruled out. If your partner fails to conceive, you can opt for newer techniques like IVF or ICSI. Hope I have answered your query. Let me know if I can assist you further. Take care Regards, Dr Shinas Hussain, General & Family Physician"
},
{
"id": 55649,
"tgt": "Is SGPT of 108 a cause for concern?",
"src": "Patient: I'm taking methotrexate for RA. My hepatic panel shows SGPT of 108. I understand this is higher than the norm but is this level cause for concern? I have a MD appointment on 11/29. Can I wait until then or should I schedule for an earlier appointmenrt? Doctor: HelloIncreased SGPT may indicate liver injury and it may be due to methotrexate therapy.You may need dose adjustment of methotrexate.You may need tablet ursodeoxycholic acid tablet 300 mg twice daily for three months.Ursodeoxycholic helps in regeneration of liver cells.You may also need ultrasonography of abdomen to exclude other causes.Get well soon.Take CareDr.Indu Bhushan"
},
{
"id": 75145,
"tgt": "Why does my body go into painful spasms after coughing?",
"src": "Patient: I have had 2 Trachaes and have trachael stenosis! There are times when I cough so hard as it is difficult to expel the phlegm that my entire body goes into spasm and gets the most painful pins and needles. It can incapacitate me for up to ten minutes. what causes this and is there anyway I can prevent these attacks. I have 40 % tracheal capacity left Doctor: there is a chance to go with tracheal stents which makes airway expanded for that you should consult the best pulmonologist... presently you need to use mucolytic and expectorants along with chest physio and nebulization with 3%ns week dive your problem"
},
{
"id": 2484,
"tgt": "Will taking tarana tablets help me in becoming pregnant?",
"src": "Patient: Dear doctor,am 25 yrs old women married 1 year ago.i got more fat and irregular of mensuration also.Dr.padma ravi from hyderabad did the scan and she said that more follicles are formed to me..for that she suggested to use TARANA.by using this tablets i will get pregnant or not?..am in lots of confusion.. Doctor: Hello dearI understand containTarana is combined birth control pill.It will help to regularize period by correcting hormonal imbalanceBut when you are on tarana, pregnancy is not possible.Once your period will become regular, you have to stop to take it to become pregnant.I would also suggest to undergo reproductive hormone analysis, USG scan and ovarian follicle study regularly for better management.Drugs like metfromin, progesterone pill in second half also useful.Avoid stress, take healthy diet with lots of fresh fruits and green leafy vegetables. do regular exercise for 45-60 minutes in morning and reduce 10-12% body weight, avoid excessive sweet in diet, take small frequent meal rather than large mealHope this may help youContact further if follow up neededBest regardsDr. Sagar"
},
{
"id": 23673,
"tgt": "How to reduce blood pressure level?",
"src": "Patient: Hi doctor, i would like to consult about my mom. She is 43 years of age. We have the blood pressure machine at home, her blood pressure when i check is between 150-175 systolic over 90-100 diastolic. she doesnt have any symptoms when i ask her! i'm really worried about this. what do you recommend? should she start taking medications. by the way, she doesnt take any medications at the moment. She is not a smoker nor a drinker, she has a family history of hypertension (her mom). she drinks coffee.looking forward for your answer.Many thanks,Shamsa Doctor: Dear ShamsaMany patient with high blood pressure doesn't feel when they have high blood pressure. Advise your mother to restrict the amount of salt intake. And if her blood pressure is around 150-170, there is a need to see a doctor to prescribe a medication. She should take medications to control blood pressure to avoid complications. Take careCome back if you have any further questions"
},
{
"id": 20724,
"tgt": "What causes weight gain with dyspnea on moderate exertion?",
"src": "Patient: im gaining weight continiously since 8-10 yrs,im non diabetic,mildly hypertensive ,taking tab telmasartan 40mg,lopresor 50 mg daily,i wiegh 120kg,and has become dyspnoic on moderate exertion,my stress test is normal done 2 mths back,pl advise,what to do,i ocassionally smoke also. Doctor: Hello!Welcome on HCM!Your symptoms seem to be related to fluid retention. From the other hand the difficulty in breathing could be related to fluid retention or heart failure. Coming to this point, I would recommend consulting with your attending physician for a careful physical exam and some tests: - a cardiac ultrasound and a resting ECG to examine your heart- NT-proBNP levels for heart failure- kidney and liver function tests- blood electrolytes- a urine analysis. A diuretic (like furasemid) may be needed. You should discuss with your doctor on the above issues. Hope to have been helpful!Wishing good health, Dr. Iliri"
},
{
"id": 25750,
"tgt": "What causes rapid heart rate after using implanted heart monitor?",
"src": "Patient: For at least nine months i have been trying to resolve this problem.... I m on my third cardolgist and have an implanted heart monotor.... I m fine during the day, but when i go to sleep my heart starts feeling like a vibrator is running and I wake up every 45 -60 minutes ....this goes on all night and I don t ever get enough sleep to make me feel alive.....i Have sleep aphnea but it still happens wearing the mask....it s been getting worse lately.. all tests don t show anything but they are done while I m awake.....heart rate goes up to 160 showing on a holter monitor....again this only happens at night ....any ideas ? Doctor: hello,I have gone through your query.Thanks for using HCM.Actually I need to know exact report of your Holter Monitoring especially at night when you get fast heart rate.Depending upon the type of rhythm disturbances we may plan medication or Implatable device.My best wishesDr.Rajesh Teli,MD."
},
{
"id": 213809,
"tgt": "I am suffering from depression, need advice and guidance",
"src": "Patient: I am a 30 year old woman.I am suffering from depression .I have been on Prasilex 10mg and Rivotril .5mg for 2 months.I feel i am not recovering as I hoped.Kindly help. Doctor: , depression can be eliminated by usibg some natural homeo remedy. R14,15 RECEWEG'S GERMAN MADE---- 15 DROPS IN LITTLE WATER FROM EACH REMEDY ALTERNATE DOSE THRICE A DAY,THESE REMEDIEDS CAN BE HAD FROM YOUR NEAREST PHARMACY,YOU WILL FEEL GOOD RELIED AND COMFORT , DR.MOHSIN MADNI madni66@saudia.com"
},
{
"id": 133973,
"tgt": "What medication is suggested for pain in the upper part of the buttocks?",
"src": "Patient: Hello, I m having really bad pain on the upper part of my butt crack. It started like 2 days ago and me and my mom thought it could just be soreness from a car accident I was in last month. I looked at in the mirror and I noticed a bump, I couldn t really see it and I haven t been able to tell my mom about it but I don t know what it is and I d really like to at least have an idea of what it could be. If you could please answer my question I d appreciate it Doctor: hi,thank you for providing the brief history of you.A thorough neuromuscular assessment is guided for details.As by going through your history, i could understand that due to the accident in the past you may have damaged the soft tissue. As initially any soft tissue do not give symptoms and as time passes by the slow symptoms arises. For which a Thorough neuromuscular assessment is needed. Also, the symptoms are in the upper part of the but, post assessment, there may be a need of the MRI of the lumbar spine to assess the nerve entrapment, which in most cases is ignored by just taking medication.Post the thorough assessment, if medicine are needed to prescribe they will be done and physical therapy will be advised. Usually people recover with 99% success rate with controlling the symptoms through pain and physical therapy.In my clinical practice after a thorough clinical assessment, the next treatment plan is initiated with physical therapy and medication. We are able to manage 99% success in their symptoms and recovery,I wish the same from you.RegardsJay Indravadan Patel"
},
{
"id": 165897,
"tgt": "What causes baby to experience twitching in muscles?",
"src": "Patient: my 8 month old son had been exhibiting some strange muscle twitches, mostly at the end of his feeding. His right shoulder goes up to his ear and his are extends in front of him for 2 - 3 seconds. he doesn t cry, he continues to eat, and he doesn t stare into space . I am waiting for his pedi to ring me back. Can this be something other than infantile spasms? this happens maybe 2-3 times a day. Doctor: it looks like infantile spasmsbut calcium deficiency cant be ruled out Plz start calcium suoplementation startedget EEG test done to know whether It is a seizure or not"
},
{
"id": 50141,
"tgt": "Stomach pain. Blood work shows high WBC. Swollen lymph nodes in abdomen. CT scan shows hydronephrosis",
"src": "Patient: I was recently in the ER with stomach pain in my right side. bloodwork showed high wbc and the dr said ct showed lymphnodes in abdomen swollen. I went and picked up my cat scan report and it didnt say anything about lymphnodes but it did say i had a right extrarenal pelvis in borderline mild right hydronephrosis without a clear etiology. I was told i had this about ten years ago and since then I have had this re-occur several times. Should I be concered about this being ongoing for so long? Doctor: you should get a radionuclide scan like DTPA Renography before and after lasix ( diuretic injection to know if the enlarged renal pelvis is due to obstruction or is just an extra renal pelvis without obstruction which doesn't require any treatment."
},
{
"id": 130525,
"tgt": "Suggest treatment for persisting pain in leg after hamstring strain",
"src": "Patient: I tore or strained my hamstring and the pain has been so severe that I nearly passed out a few time. My leg swelled up hugh and is black and blue. It has been 1 week and the swelling is going down but the pain is still severe even on the surface of my skin. What sould I do. Doctor: Hi,Hamstring sprain will take about 3/4 weeks to heal(grade 1/2/3). You need to be patient. Till then need to avoid the movement of hamstring to aid in repair(avoid knee extension)You can were a knee support or crape bandage for stability. Kinesiotaping will help in this. Also use of contrast bath (hot and cold packs alternatively) will help in pain reduction Can take acetaminophen twice a day for pain relief.Hope this helps. Let me know if I can assist you further.Regards,Dr. Jenis Bhalavat"
},
{
"id": 79681,
"tgt": "What causes occasional sudden pins and needles with tightness in left chest?",
"src": "Patient: I get occasional sudden pins and needles with some tightness in my left chest. Usually only lasts 5 minutes, but today it lasted 30 mins . It comes out of nowhere and its happened in and off for at least 10 years. Not sure what it is or why it happens. Doctor: HelloTightness in left side of chest need proper evaluation.It may be due to cardio pulmonary causes,reflux etc.You may need clinical examination and investigations like routine hemogram,random blood sugar,liver function test,renal function test,lipid profile,urine RE/ME,upper GI endoscopy,ECG,ECHO,TMT,ultrasound of abdomen.Proper treatment depend upon findings.Get well soon.Take CareDr.Indu Bhushan"
},
{
"id": 192098,
"tgt": "Suggest permanent remedy for diabetes mellitus and hypertension",
"src": "Patient: sir i am dr amar physician at mandya.my cousin 19yrs has DM AND HTN poorly controled with medications.would you like to rule out any secondary causes in him and get a permanent remedy.ALREDY SHOWN TO MANIPAL HOSP ENDOCRINOLOGIST 2 YRS BACK.HE DID 24 HR URINE CORTISOL FOUND MARGINAL ELEVATION HE SENT BACK MY COUSIN TELLING NOTHING.WERE CAN I BRING HIM TO SHOW YOU Doctor: Dear sir.. Greetings from HCM ... As you rightly pointed out YES it could be a secondary hypertension with diabetes ... Cortisol could be responsible for both .. Pls get properly evaluated with serum cortisol levels at morning time .. In adduction ultrasound abdoman even CT for pituitary ... for ruling out Cushing s disease or Cushing syndrome ... If the cause is treated he will be alright .. Do contact us with reports .. Thank you"
},
{
"id": 186363,
"tgt": "Will wisdom tooth cause pain and headache?",
"src": "Patient: My wisdom teeth are currently \"trying\" to come in but I do not have enough room in my mouth for them to come in. My gum over the bottom left wisdom tooth is split. In between my cheek and my wisdom tooth, I have a hard knot, and it hurts to close my mouth tightly. Its causing me pain and head aches. What could this be? Should I go back to the dentist or just wait until my consultation to get my wisdom teeth taken out? Doctor: Hello, thank you for consulting with healthcaremagic. This all problems are associated with the wisdom tooth only, for relief from them you have to get it removed. But before that you should start with antibiotics and painkiller medications two days before the procedure. If dentist has not prescribed you the medications, then visit him tomorrow and get it done.Hope it will help you."
},
{
"id": 107691,
"tgt": "Suggest medication for severe back pain",
"src": "Patient: i m 23 yrs old.m suffering from severe back pain from last 2 years.there is no abnormality in xray reports.Dr. says my muscles are weak but no medicine relive my pain.i cant even sit under fan even in summer without covering my back with blanket.please advise me which med i shud take? Doctor: For treatment of severe back pain you can take some pain killer like biozobid-plus or osteonac-th along with some calcium supplements like shalcal-d or toscal-gem once daily. You can use volidup gel or powergesic-plus gel for local application. Do some physiotherapy exercises after an expert opinion of physiotherapist. I will also advice you to sleep on a plain surface."
},
{
"id": 42514,
"tgt": "Is the sperm count normal?",
"src": "Patient: LAB REPORT Patient's Name : Mr. AHMAD IBRAHIM ALHAMAD Age/Sex : 33 Years /M Permanent : AD0000 Centre : WALK IN Visit No : 1112AD0000 Reg. date : 01-Oct-2011 Ref Doctor : Dr.IBTESAM . Test Result Units Reference Range Semen Analysis Method of Collection Masturbation Spillage Nil Time of collection 11:15 AM Time of arrival 11:15 AM Time of examination 12:15 PM Liquefaction time > 1 HR Min Appearance Grey White Grey White Viscosity High Volume 2.5 mL = or > 2.0 PH 8.0 7.2 - 8.0 The number of sperms /mL 4.0 Million/mL = or > 20 The number of sperms per 10.00 Million = or > 40 ejaculate Motility at 1/2 hour Motility at 1 hour Rapid Linear Progressive 5 % Sluggish Linear Progressive 25 % Total Forward Progressive 30 % Dr.Saravana N.Chetty Specialist Pathologist RePrinted on : 01-Oct-2011 5:44PM : Printed on : 01-Oct-2011 04:46PM Authenticated on : Requested on : 0 1-Oct-2011 11:07AM 01-Oct-2011 04:45PM Copy of Original Page 1 of 3 LAB REPORT Patient's Name : Mr. AHMAD IBRAHIM AL HAMAD Age/Sex : 33 Years /M Permanent : AD0000 Centre : WALK IN Visit No : 1112AD0000 Reg. date : 01-Oct-2011 Ref Doctor : Dr.IBTESAM . Test Result Units Reference Range Non Progressive 5 % Non Motile 65 % Total number of active Sperms in 3.0 Million the ejaculate MORPHOLOGY Abnormal forms Pin Head 2 % Microsperms 1 % Deformed 1 % Bent neck 2 % Curled Tail 1 % Miscellaneous Immature 6 % Total no. of abnormal Sperms 13.0 % Cellular Elements Pus Cells 4-5 hpf Erythrocytes 0-1 hpf Epithelial cells OCC hpf Specimen : Semen Dr.Saravana N.Chetty Specialist Pathologist RePrinted on : 01-Oct-2011 5:44PM : Printed on : 01-Oct-2011 04:46PM Authenticated on : Requested on : 0 1-Oct-2011 11:07AM 01-Oct-2011 04:45PM Copy of Original Page 2 of 3 LAB REPORT Patient's Name : Mr. AHMAD IBRAHIM AL HAMAD Age/Sex : 33 Years /M Permanent : AD0000 Centre : WALK IN Visit No : 1112AD0000 Reg. date : 01-Oct-2011 Ref Doctor : Dr.IBTESAM . Test Result Units Reference Range W.H.O MANUAL Motility : 25 % or more with Rapid Linear Progressive Activity (A) Motility : OR 50 % or more with Forward Progressive Motilitty (A+B)CORELATION OF SPERMS AS PER W.H.O. CRITERIA & MACLEOD GRADES (A) Rapid Linear Progressive Motilitty Includes Macleod' Grades IV & III Grade IV : Excellent Swift Progressive Motility Grade III : Moderate Progressive Motility (B) Sluggish Linear Progressive Motility corresponds to Macleod's Grade II Grade II: Struggling Progressive Motility (C) Non Progressive Motility corresponds to Macleod's Grade I Grade I : Poor Sluggish Non Progressive Motility **** End of Report **** Dr.Saravana N.Chetty Specialist Pathologist RePrinted on : 01-Oct-2011 5:44PM : Printed on : 01-Oct-2011 04:46PM Authenticated on : Requested on : 0 1-Oct-2011 11:07AM 01-Oct-2011 04:45PM please can you give me evident opinion about the above mentioned Doctor: HELLO AHMAD IBRAHIM ALHAMAD ,I will comment on your semen analysis report.in the first page liquefaction time is mentioned as less than one hour.ideally should be within 30 minutes.but still less than one hour is considerable.Ph is normal -alkaline -8.0.volume should be 2to 5ml -yours 2ml.-normal.total count should be around 20 million/ml-yours 4million/ml-less than normal.-abnormal.Rapid forward progressive motility of sperm after one hour - 50 % or more than that is normal- yours is 30%only.morphology- More than 30% of the sperm cells should have normal shape.(Kruger criteria: More than 14% of the sperm have a normal shape). your report says abnormal sperm in less count- it is normal.pus cells will not present in semen. still it is non significant as of your report.finally it is a abnormal report, because it contain problem in motility.thank youhope i answered your query."
},
{
"id": 147287,
"tgt": "Vocal cord paralysis, unable to swallow, fever. Taken tylenol. Elevated proteins. What is the cause for symptoms?",
"src": "Patient: My cousin developed vocal chord paralysis and is unable to swallow following a procedure to remove dead tissue from his leg (he has diabetes and sores that do not heal easily). He was given general anesthesia for the procedure. They performed a cat scan but did not see signs of a stroke. An MRI was not done because he has a pacemaker. He is now in this state, with a feeding tube , for two weeks. He had a fever of 104 last night and given Tylenol. A spinal tap showed elevated levels of protein. What are possible causes for his symptoms? Doctor: Damage to the vocal cords may have occurred if your cousin had a tube placed in his throat for the general anesthesia. Elevated protein in the spinal fluid indicates inflammation that could be caused by an infection affecting his brain. Diabetes lowers the body's ability to fight infection. The dead tissue most likely resulted from an infection, which could have spread from there into the bloodstream and then into the brain. The fever also suggests some kind of infection. If an infection affected the lower part of his brain, it could damage the nerves that go to his vocal cords and cause that paralysis. Hope I have answered your query. If you have any further questions I will be happy to help,"
},
{
"id": 191896,
"tgt": "Suggest treatment to control blood sugar",
"src": "Patient: Hlo Doctor...my mother is suffering from diabetes from the past 19-20 yrs...recently we got her sugar level tested and results were shocking...with fasting it was 197 and without fasting it was 210... She is taking glyz-m ....plzzz suggest something for her Doctor: HiSince her fasting sugar and Random sugar are on higher levers suggest that you get her HBA1C value at the earliestHBA1c test to be done once in three months to check the sugar value consistency in the body for three months. It is very good test to see how well the sugar is controlled in diabetic patientsif I were her treating doctor would have tried glimipride 1mg once daily and monitor for 6-7 weeks (however hypoglycemia need to be monitored regularly)She can do exercise/walking for 45 minutes daily atleast 5 days a weekShe can take sufficient vegetables and limit the rice content in the dietDiet and exercise plays a vital role in controlling diabetesWish Her Good HealthRegardsDr Sridhar K"
},
{
"id": 24383,
"tgt": "Suggest remedy for persistent cough post heart surgery",
"src": "Patient: My husband has had open heart surgery, pacemaker/defib, on all kinds of meds including amirodione plus he had a heart ablation due to A-fib after the pacemaker. He has stopped all his meds except coumadin and lasix. He has stopped his meds for about 5 mos. now. He is starting to cough a lot and can t sleep at night. I know the ultimate consequenses. What should I be looking for? Doctor: Hello... Stop himself medicine or by doctor??? For him I will examine him thoroughly and do basic investigation like blood test v to see parameter OK n... Ecg and echo x ray... And complete with previous one... Treatment depend on worsening heart condition or due to LUNG condition... Take care"
},
{
"id": 8700,
"tgt": "Want to whiten skin. Is gluthathione injection safe?",
"src": "Patient: Hi Doctors, I\u2019m really confused, i want to whiting my skin and i heard that the glutathione injection using for this purpose, to be Injected IM, I\u2019m in Bangladesh and there is a lot of doctors using it for whiting the skin, and I\u2019m going to try it but I\u2019m afraid if it may causes Cancers or a harmful diseases, please I need to know if that glutathione injection with a FDA permission is safe to take or not? Doctor: Hi, Thanks for query, The color of the skin is due to its pigment.no medicine can change it. Glutathione is an anti oxident. How anti oxident resulted less melanin production is still debatable. Anti oxident in oral form does not harm you. You can use natural methods like Drink lot of water that will hydrate your skin & glow it. Take healthy nourshing vitamin rich diet. Avoid junk foods. if needed protect your skin from direct sun light. Thanks."
},
{
"id": 168387,
"tgt": "What causes a keloid at the back of ear in a 2 year old?",
"src": "Patient: My daughter is 2.8 years and we noticed the Keloid on her ear lobe back side. I think it was developed after ear piercing, It is not much big it is something like a pimple and on one ear it is big and on the othe rit is very very small. What is the best way to get rid of this. Doctor: Keloid are known to reoccurring.injection of steroid and hyuronidase will helpn in reducing .all other measures are useless"
},
{
"id": 35391,
"tgt": "Suggest remedy for wheezing and stomach infection",
"src": "Patient: My 2 year old daughter has got wizzing and stomach infection. She is running temperature after efevry 8 hours. she is on Bricanyl ( 2.5 ml 4 times a day ) and sporadix ( 4 ml 3 times a day). Pyregesic on a SoS basis. No signs of relief for the last 2 days and the stomach problems have increased. Please suggest Doctor: Brief answerpneumoniaDetailed answerI have gone through your query I think I have a better answer to help you outIn my opinion it could be a case of LRTI like pneumoniawheezing even after treatment with bricanyl is a matter of concern please visit your doctor for a battery of tests like complete blood count, ESR, X ray chest for definitive diagnosisa course of antibiotics like augmentin is strongly recommended under supervision of your doctorask for steam inhalation if possibleHope I have answered your question. If you have any further questions I will be happy to help wish you good health Dr BR Hudda"
},
{
"id": 158030,
"tgt": "History of lung cancer. Feeling of malaise. Taking Omeprazole for acid reflux. Any ideas?",
"src": "Patient: Several times a day I feel lousy (a general yucky feeling). This has been going on for several months. I went through radiation and chemo two years ago. I had radiation in upper right lobe of lung as well as several lymph nodes near the lung and one at base of neck on right side. All pet-scans since (there have been several) have been negative - no reucrriing cancer; everything remains in remission. I just had a colonoscopy and and an upper endoscopy . I now have acid reflux. I am taking omeprazole (20 mg), levothroxine (175 mcg), and trilipix (135 mg). I also take Metimucil for additional fiber. The general yucky feeling only started about four/five months ago. I was on cisplatin, carboplatin, and aliminta plus Tarceva for 16 months. I ve been off Tarceva for about a year. My last blood test showed a wbc count of 12.6. Any ideas why I feel this way at times during the day. I ve already had two episodes today - one last about 2 minutes the other about 30 seconds. Any ideas? I m having more tests done over the next couple of weeks. Doctor: Hi! Thanks for the query. The nauseating feeling is unrelated to your cancer. It is most likely the residual effect of chemotherapy. If your latest reports also show no evidence of cancer, you can rest assured. Ondansetron thrice a day might help."
},
{
"id": 98939,
"tgt": "Suggest treatment for allergic reaction to sea food",
"src": "Patient: Hello Doctor, My mom is a resident here in Hyderabad, she is 55years & in good health. She used to have allergic reactions to sea-food & some other food items earlier but I write to you about a recent incident which has caused concern. She was travelling Uk & USA & had eaten out....for first time this reaction was so bad that her face & body got swollen & didn't subside too much. 2 days back she came back to hyderbad & her swelling got better. But last night she had food outside & now developed terrible allergic rashes on her body & face...its itchy but not painful. Where can we show her in Hyderabad (hi-tech city area) for this allergic reactions? Please help us. thank you Doctor: In my opinion you should see nearby doctor.As such no definitive cure for allergic reaction.We can prescribe medicine & she will be fine but if she eats sea food again surely allergic reaction will develop hence AVOIDANCE of allergic food is only the definitive treatment.Secondly watch out for breathlessness,wheeze,flushed face,swelling of tongue n neck if this symptoms occurs you must have to rush to emergency department as early as possible.Please go to nearby doctor for present allergic reaction treatment.Regards"
},
{
"id": 211836,
"tgt": "Refill medicines for anxiety over, Doctor changed location. Tab Alprazolam. Suggestion?",
"src": "Patient: my psyciatrist recently moved locations and i cant find him online. i called my last refill in for my anxiety meds a few days ago and i called to see if it was ready and the pharmacy said it had already been picked up and i didnt pick it up and i didnt send anyone to pick it up. the pharmacy told me to call my doctor and have him call them. His name is Dr. woodrow w. coppedge. the pharmacy didnt get an id number of the person who picked it up so i cant press charges. i am starting to have withdrawl symptoms. the medicine is alprazolam. Can you help me find dr. coppedge or tell me what to do. I cant afford to go to the e.r Doctor: Hi,Thanks for writing in.You may enquire about Dr Woodrow W Coppedge at the following location.18333 Egret Bay Blvd Ste 305Houston, TX 77058(281) 333-5740If he is unavailable, please ask any representative to help you out with the search.Hope this helps"
},
{
"id": 159667,
"tgt": "Diagnosed with colon cancer. Red patch on stomach leaked pus. Fitted catheters. Is it due to cancer?",
"src": "Patient: hi my mother was diagnosed with colon can in november 2011... in january she wasadmitted to hospital because she kept on getting temperatures..they told us there wasnt anything they coulddo because it had spread to the liver .. she had the colostamy bag fitted an the night after she got really poorly she couldnt eat or drink... they never put her on a drip we asked to speak to a specialist and they said she was being handed over for pallative care... she went till the monday and luckily she pulled through and they put her on a drip she built up some strenght and was discharged from hospital... my mother kept on getting temperatures and would start to hallusinate.. a lump formed on her left side and we were made to believe it was the cancer .. this went on for weeks then on a thursday night on the right side of my mothers stomach a red patch formed 2 days later it started to leak pus ... she was admitted to hospital and where it was leaking they removed it via surgery but not the lump that keep forming on her left side so it keeps filling with pus... we are worried that once the whole has healed it wont have anywhere to drain and we will haveto go through all this agian they gave my mom 3 months to live in january but she is a fighter and still here.. she was admitted to hospital again on monday... we were hoping for them to investigate because on a home visit from the doctor gave her an internal a pus leeked out she has had a lot of discomfort for a few weeks after having a cathetar fitted.. we asked the doctors if they could give my mother a scan but they told us no her problems are due to her cancer... she has put weight on since at home and my father has been looking after her.. if it wasnt for the absess i dont think she would be as bad as they think... they want her to go into a hospice but she isnt on her last legs yet.. please can you give me some advice on what we could do please. thanks very much Doctor: hello i think hospice care is best for end stage patients they will give her pain free time till she lasts"
},
{
"id": 211660,
"tgt": "Have difficulty with emotions. Taking celexa. Will it alter personality?",
"src": "Patient: i am currently taking celexa and would like too know something good about it . so far i have only heard bad and read bad makes me scared too continue taking it . i dont want too feel the way i do today about life but i dont want too take something that might alter my personality , i like me !!! sometimes have difficulty dealing with my emotions and feelings towards others. Doctor: Hi,Whoever has informed you that Celexa alters the personality was incorrect. It simply corrects the symptoms occurring from altered serotonin levels in the brain. It alleviates the symptoms of anxiety, depression and some other mental illnesses. Please take it as prescribed by your doctor and you have nothing to worry.Best wishes."
},
{
"id": 41989,
"tgt": "Suggest treatment for secondary infertility",
"src": "Patient: i am 36 yrs old female , i have taken anti -tuberculous treatment 8 yrs back for pulmonary t.b , now i am having secondary infertility ,what test should i do , both my tubes are blocked .which test is reliable to confirm uterine tuberculosis & treatment for tubal block . Doctor: As you have secondary infertility and got treatment for tuberculosis 8 years back, had you done with HSG (histo salpingo graphy) ? If not then first do this test.HSG is therapeutic procedure rather than diagnostic.It can help to reopen your fallopian tubes. 2 test to do ultrasound of pelvis.After hsg tubes found block then you have to go for laproscopy to diagnose the blockage in path. So consult a gynecologist and take his/her advises."
},
{
"id": 80826,
"tgt": "How to treat chest pain caused due to an injury making it difficult to cough and sneeze?",
"src": "Patient: i jumped into water that was much shallower that i thought & experienced sharp pain in upper chest. almost 2 weeks ago. since then when i cough or sneeze or laugh or try to turn over in bed or get out of bed i get a sharp pain in upper chest. any idea of what i did to myself? (i had a vius about 4 weeks ago and another 1 started the day after i hurt myself, so i need to cough but hold it back because of the pain) Doctor: Thanks for your question on HCM. I can understand your situation and problem. In my opinion you are having blunt chest trauma. And pain on coughing, sneezing is seen in following possible complications of blunt chest trauma. 1. Rib fracture2. Pulmonary contusions3. Pneumothorax4. Muscular injury. So get done chest x ray. If chest x ray is normal then no need to worry much. You are having muscular injury mostly.So try to follow these steps for better symptomatic relief. 1. Avoid heavyweight lifting. 2. Avoid strenuous exercise. 3. Avoid bad posture and movements causing pain. 4. Start painkiller and muscle relaxant. 5. Apply warm water pad on affected site. 6. Start antihistamine drug for cough and sneezing. Don't worry, you will be alright."
},
{
"id": 73123,
"tgt": "How to treat costochondritis?",
"src": "Patient: hi, i have costochondritis and i'm 16 years old, i have a few questions about it. first i want to know if it will go away? and how long will it take until it goes away? after it goes away, if i do a certain things like run or play sports will it come back? Doctor: Thanks for your question on Healthcare Magic.I can understand your concern. Costochondritis is inflammation of costal cartilage (rib cage).It is 100% reversible condition.It usually takes 2-3 weeks for complete recovery from costochondritis.It can recur. You should follow these steps for recovery in costochondritis. 1. Avoid movements causing pain. Avoid heavyweight lifting and strenuous exercise.2. Avoid sudden jerky movements of chest.3. Apply warm water pad on affected areas of chest.5. Take painkiller and anti inflammatory drugs like ibuprofen or acetaminophen.Do all these for 2 weeks and you will be alright.Hope I have solved your query. I will be happy to help you further. Wish you good health. Thanks."
},
{
"id": 76069,
"tgt": "What causes pain in the chest while breathing?",
"src": "Patient: Sir, When I am taking breath I am having pain in the lungs. And full tight in the lungs also. Please advise remedy. Further my right shoulder if very tight inside and when lifting the right hand upside there is some pain. My Age is 56, weight 70, height 5.3 Doctor: Thanks for your question on Healthcare Magic. I can understand your concern. Chest pain on breathing is known as pleuritic pain. And it is commonly seen with bronchitis and lung infection. You are also having chest tightness. So better to consult pulmonologist and get done 1. Clinical examination of respiratory system2. Chest x ray 3. PFT (pulmonary function test). Chest x ray is needed to rule out lung infection. PFT is must for the diagnosis of bronchitis. You may need antibiotic, inhaled bronchodilator and inhaled corticosteroids (ICS) on the basis of these reports. Your shoulder symptom is mostly due to frozen shoulder. So start physiotherapy for this. Don't worry, you will be alright with all these. Hope I have solved your query. I will be happy to help you further. Wish you good health. Thanks."
},
{
"id": 151774,
"tgt": "Can I undergo a spinal fusion surgery at the age of 75 ?",
"src": "Patient: I am 75, female, in good health, I have some scoliosis , and degeneration of the discs and a breakdown of the spine at T12/L1 and L4/L5. This has been causing me quite a lot of pain in my low back and left hip area for 1 1/2 years. I had physical therapy a year ago which helped very little. I am quite active and hate the restrictions this is causing. The doctor says he can do minimally invasive fusion surgery . So many people do everthing they can to avoid surgery. What is the advantage in waiting? I feel that eventually it will have to be done anyway. Right or Wrong? Doctor: Welcome to Healthcare Magic Good Day Yes, In such painful conditions of the spine it is best to do the spinal fusion surgery as your Doctor ordered, but I doubt the chance of relief in the restriction of movements. Do speak to your Doctor regarding this and the pros and cons of the Surgery."
},
{
"id": 104239,
"tgt": "Does a burnt plastic in the microwave cause any harm to a small baby?",
"src": "Patient: My husband accidentally burned some plastic in the microwave this evening. We don t know what type of plastic was burned, but the smell permeated our small house. We opened a window and turned on a fan to air out the kitchen. We have a 3-year-old and an 18-month-old. How likely is it that either child was harmed by inhaling these plastic fumes? Thank you. Doctor: it shu;d not have any effect as per ypur history you can yourself see if the children are breathing normally no cough no iritatio on skin or other extra symptomps it means n efect if there was some effect childrem shuold have symptomp like itcing iritation snezing coughing etc"
},
{
"id": 164869,
"tgt": "What causes anal pain in children?",
"src": "Patient: my 4year old has been complaining for the last month of pain inside his bum that seems to be getting worse,he screeches in pain and cant sit still when it happens.is goin to the toilet reguarly so unsure to what it could be, have also checked for worms and cant see anything. Doctor: hi,thanks for the question. most common cause for anal pain in children especially if the child is passing hard stools is injury to the sensitive anal skin caused by the hard stools. if your child is passing hard stools give her some laxative like lactulose. intestinal worms are less likely to cause anal pain."
},
{
"id": 135373,
"tgt": "Suggest remedy for muscle & joint pain",
"src": "Patient: I am active, healthy and strong, just turned 51, female, exercise class twice a week, pretty busy, no allergies or medications. For the past two weeks my muscles and joints are achy and actually hurt. They feel stiff like I did a new exercise and then did not stretch. Have a big red bump that I thought was a black fly bite, it is sore and hard on my shin like I bumped it. Does not look like a tick bite. Any ideas why the aches? Doctor: Hi Dear,Welcome to HCM.Understanding your concern. As per your query you have muscle & joint pain. Well there can be many reasons for symptoms you mention in query like bursitis , gout , infectious diseases , tendinitis or fibromyalgia . I would suggest you to consult orthopedic surgeon for proper examination . Doctor may order certain test like blood test , check vitals , take history and physical examination . Doctor may prescribe muscle relaxant , anti inflammatory along with vitamin supplement and physical therapy .For now take proper rest , apply warm compresses and drink plenty of water . Hope your concern has been resolved.Get Well Soon.Best Wishes,Dr. Harry Maheshwari"
},
{
"id": 95179,
"tgt": "13 year old vomits weekly for past 2-3 months",
"src": "Patient: my 13 year old daughter has been vomiting weekly for the past 2-3 monthe she also get a red face and feels sleepy when this happens she is fine one min then red face then vomiting. i have had her to the family dac several times she has seen an alergest. we went to the hospital they said it was like a migrane headache in the abdomin but i am not ok with this they gave her some antivomit meds that they use for people on cemo theripy they are not helping. what could this be and how long does a 13 year old girl go on like this? the hosp made her an appointment for a gi as i have severe chrones myself but it is not until sept what should i do? Doctor: 13 yr girlwith recurrant vomiting ,if colour of vomitus is greenish to startwith likely cause woulld be malrotation of gut requiring BMFT investigation to rule out this problem. if nnogreenish vomiting,in b/w bcild is ok having headache also,and parents have h/o headache likely cause would be abdominalmigraine or cyclicalvomiting syndrome.consult pediatric GI doctor."
},
{
"id": 108733,
"tgt": "What causes back pain, dizziness and headaches?",
"src": "Patient: Hello, I have an 19 y.o son who has been sick since Tuesday 7/6 running really high fevers, body aches spinal pain, dizzines and headaches. My husband took him to the clinic on Wednesday 7/7 they said it would be best to take to ER which we did twice now. the first day was on Wednesday 7/7 and on Thursday 7/8, they did saliva, xray (2), urine, blood and spinal tap test. His test came back fine. He is still running fevers plus all the symptons mentioned above and as of yesterday he has started vomitting and diarrhea. He doesnt eat, but is taking in fluids. When he went to hospital the second time his fever at time of arrival which was around 6:00 p.m. was 104.9 at discharge which was 12:30 m.n. it was 103.2. What can I do Doctor: you may give plenty of fluids orally, acetaminophen 500mg Thrice a day,vitamin c500 mg once daily.bodyache and fever may be due to viral infection.you may add levocetrizine 5mg if sore throat and also warm saline gargles.Keep him well nutrited."
},
{
"id": 142954,
"tgt": "What are the symptoms of meningitis?",
"src": "Patient: My 8 month old son has a low grade fever (99-100 degrees). However, last night we noticed that his feet were cold (not ice cold but cold) even with pajamas to cover his feet. He slept a lot today. All the online forums bring up meningitus. I don't think that is it but want to be sure. Doctor: In an 8 month patient there is no such thing as \"classical\" signs or symptoms of meningitis. It is true that many cases of meningitis have higher fevers, are accompanied by stiff neck, nausea, vomiting, etc. However, at that young age many of these symptoms may be absent. If I were in your position I would default to take the baby for a checkup with the pediatrician. They may be able to get some bloodwork and the pediatrician can do some examination maneuvers to see whether meningitis is likely or not. It may turn out to just be a viral syndrome but if you're suspicion is that something else could be going on then, you wouldn't be wrong to act on it and have the baby examined. Please rate this a 5 STAR RESPONSE & write me at: www.bit.ly/drdariushsaghafi with more questions on this or other topics."
},
{
"id": 207610,
"tgt": "Could headache and mild dyspnea be due to stress?",
"src": "Patient: hi i have been feeling unwell for 10 days but before i was totally healthy i have dypsnea lightheadness and pulse of 90bpm my blpr was at first 140/85 then it returned to be 110/75 over the next few days i feel tired exhausted very depressed i m 32 years old my hb is 10 and i m taking ferroglobin caps for that i also take procoralan for tachycardia but still have taht light head and headache and mild dypsnea my doctor says i m stressed that s it and that this is anxiety but i really doubt that pls help me i m very afraid Doctor: hiI understand your concern.Yes it may due to stress and anxiety.tachycardia, palpitation, difficulty in breathing etc might indicate anxiety.Anxiety is unconcerned fear which produce physical symptoms.This condition can be treated with SSRI like sertraline and paroxetine.Anxiolytic can be beneficial like etizolam and clonazepam.Do take doctors advise before taking medicines.Regular exercise, yoga, meditations, relaxation exercise will help.Take proper sleep and food regularly.I hope i have answered your query.Happy to help you.Take care."
},
{
"id": 222531,
"tgt": "What causes bleeding when you are pregnant?",
"src": "Patient: I m 34 weeks pregnant today and when i went pee and wiped there was a little dot of red blood on the toilet paper so i wiped two more times and the same thing. I put a panty liner on to see if anything would get on it to assure my to call the doctor. This is my second child and I was just wondering If there were any suggestions. Doctor: Hi dear, I have gone through your question and understand your concerns.Bleeding or spotting at 34 weeks of pregnancy can be due to low lying placenta ( placenta previa) or due to hemorrhage beneath the placenta( abruptio placenta) or can be due to any local lesions in the cervix or vagina.I will suggest you to consult an OBGYN specialist to get properly examined, investigated including ultrasound and get treated accordingly.Hope you found the answer helpful.RegardsDr Deepti Verma"
},
{
"id": 14352,
"tgt": "Suggest remedy for itchy red rashes due to bug bites",
"src": "Patient: I had 18 bug bites on my right arm two weeks ago. They looked like a mosquito bites. They itch immensely. Now they have turned into red rashes. It is a raised rough looking surface. Some days there are tiny little bumps present in the red rash. Other days you can t even notice the rash. I m thinking that the rash has been caused by my constant scratching. Doctor: Hi...Welcome to HEALTHCARE MAGIC..I have gone through your query and can understand your concerns...As per your complain due to allergic reaction to the toxin chemical that is injected in your body and as the number of stings are high the toxin concentration is more and you have got itching and rashes due to it..As of now you can take anti allergics like Levocetrizine and fexofenadine [allegra], apply a steroid ointment over the rashes,Doing cold compresses and application of calamine lotion can also soothe the skin and reduce itching..In case if still you do not find improvement consult an Allergist and he can also give to injectable anti allergics and steroids.Hope this information helps..Thanks and regards.Dr.Honey Nandwani Arora."
},
{
"id": 66184,
"tgt": "What can be the immovable lump on side of sternum?",
"src": "Patient: My son has an almond shape immovable lump on side of sternum. I am so worried (getting crazy) since as a 8 months old he was diagnosed with bilateral Willms. He is alright, but all previous treatments posibly could cause another type of cancer. I am taking him to doctor this afternoon but right now I found your website and want from you something for reassurance. Thank you so much. Doctor: Hi! as per your description and the History of Wilm's tumor of your son, there is high possibility that the immovable lump is kind of inclusion cyst, keloid or some abscess but in worse case it could also be metastatic Wilm's tumor or some metaplastic tumor due to therapy-related effects!A simple needle biopsy takes just 5 minutes and it confirms the diagnosis.Therefore, please see your doctor and write to us with feedback.regards and all the best!"
},
{
"id": 61812,
"tgt": "Suggest treatment for painless lump below knee",
"src": "Patient: hi, i have developed since just last night a lump below my knee that is painless no matter what i do to it, and at first sight looks like it could be a bruise (red on the outside. white on inside) and sticks out more or less depending on which way i turn or the wieght i put on it. it feels like it s my bone sticking out and i have not injured myself anytime recently. please... tell me what i should do and what it could be... Doctor: Hello and welcome to healthcaremagic.I have gone through your query and I can understand your concerns.If I were your treating physician i would have palpated it to look for fluctuation and skin pinch in order to rule out bakers cyst which usually doesnt cause significant problems and can be excised by surgery.I would have examined its mobility in order to rule out lipoma which is a benign mass arising from fats and can be excised if it is causing problems.I would have auscultated with stethoscope to look for any bruit which is heard in popliteal artery aneurysm.I would have done trendelenburg test to ascertain vericose veins.i would have done xray to look for any bony pathology arising from bone.I would suggest you to meet an orthopaedic surgeon and a general surgeon for second opinion.in the mean time remain cool and brace that area lightly.Thanks.Hope the answer is helpful.Feel free to ask further questions.Regards"
},
{
"id": 196314,
"tgt": "What causes the testicles to be of unequal sizes?",
"src": "Patient: Hi, I am 24 years old. My problem is, I am having one big and one small pennis balls instead of two equal sizes and at morning when i woke up the skin around the balls is tight and after that it will become loose. Is there any problem,Y one of the pennis ball are un unequal size. Doctor: HiGREETINGS Testis can be of different size .If it is grossly small you need to consult an urologist .Scrotum skin will contract and relax according to the temperature. In cold weather it will shrink and in warm weather it hangs .So if it is uncomfortable consult the doctor.Hope my answer helps you. Regards"
},
{
"id": 61454,
"tgt": "How can a lump on the shoulder be treated?",
"src": "Patient: hello, I've been having a somewhat large football shape lump on top of my shoulder near the neck. Can't think of an object that would come close to the size. It's bigger than a golf ball but smaller than a pear. At first, I thought nothing of it since I do carry heavy bags and my purse so I assumed that it's just a knots in my muscle that needed to be kneaded out. But I just got out of my massues appointment and she was worried about that huge lump in my shoulder and ask if I've seen a doctor. I didn't really see the need to see one since as I've said before, just carrying around 20 pound or more bags almost every single day but now I'm not so sure. Should I scheduel an appt with my doctor to get an x-ray of it or should I wait and just recorded the lump for the next month or so and go to the doctor later if it get bigger. Doctor: Respected user, hi I evaluated your query thoroughly.* Seems large soft tissue tumor .* Strongly recommend to consult with clinical photograph of it to do proper evaluation , judgement & further assistance accordingly.Thanks for using Health care magic.Welcome for any further assistance.Regards dear take care."
},
{
"id": 64733,
"tgt": "What causes a big black lump in the throat?",
"src": "Patient: hi my partner has been felling really rough for a few days now with sore throat and cold and flu like simptoms and today he asked me to look down his throat and ther is a big black lump with white little spots on what could this b we r quit worried pls help Doctor: Hi,Dear,Thanks for the query to HCM .I studied your query in depth.I feel concerned about the worry and fear expressed by YOu.In my opinion your black lump -is due to -acute heamorrhagic-tonsillitis with white-pus papules on it.I would advise -simple saline gargles/warm liquids orally to soothen it, Antibiotics,Tb-NSAIDs, under cover of your doctor.Promptly act on these health tips,with help of your doctor at primary care unit.This will resolve your problem in a day or two.Hope you would recover with these health tips.Please contact again and I would love to attend your queries soon.Have a good time."
},
{
"id": 40602,
"tgt": "How can infertility be treated?",
"src": "Patient: Hallo dr I have been married for 8 months... My inlaws are very eager for me to conceive After marriage I had regular periods for 5 months After that i missed period for one month Since then i am having irregular bleeding For 15 to 16 days and then after a week or two again i am having bleeding I got an ultrasound done..and it says pco And horseshoe kidneys Actually after 3 months of my marriage i had taken duphaston for 3 months and clomid for five days My husband stays abroad So he was coming for 15 days I took clomid right after my periods stopped. then i wasnt feeling much well So after medical examination i came to know that i dint have proper intercourse Now i have stomach cramps and headaches all the time And my periods wont stop Its 17th day today I was undergoing unani treatment from a certain dr But it wasnt effective Then last week i met a gynacologist and shes giving me herbal treatment..still periods dint stop So she is suggesting me duoleton Which i am scared of taking after all this What should i do Please help Doctor: Hello,First of all, I would like to reassure you that you should not feel disheartened about it. It is important for you to understand that at the peak of her fertility, any healthy woman around thirty years of age has approximately 1 in 5 to 6 chance of becoming pregnant during the fertile period of her cycle. This chance may become less due to a number of factors - increasing age, timing, and frequency of intercourse etcetera. In your case, the fact that you are not staying with your husband regularly and he is based abroad reduces your chance of becoming pregnant. Besides your marriage is very recent. A guiding principle for starting to investigate for sub-fertility is when a woman has not conceived for twelve months of having regular sex with her partner. This criterion has not been met in your case. However, given your health concerns - diagnosis of horseshoe kidney and PCO, it is important to start an investigation and plan for the conception. Horseshoe kidney is a condition that is there from birth and can occur together with malformations and problems linked to the reproductive parts. It may be worth getting imaging done for the kidneys, ureters, bladders as well as for the uterus, tubes and varied. The blood tests for a renal function should be done and review of results with a Urologist or Gynaecologist may be useful. Please get your blood pressure checked as sometimes this can occur with this condition.PCO or polycystic ovaries is a condition that is common, occurring in one out of three women. Sometimes it resolves with correction of weight, lifestyle changes, Sometimes medications may be necessary. Giving combined pills for 3-6 months is sometimes offered to manage irregular periods if fertility is not much of a concern. Doulton is a contraceptive pill, so it might not be the preferred method in your case. Clomid and Duphaston might be a better alternative, however, the intercourse has to be properly consummated, timed to the fertile period of your cycle and occur every two days to maximize the chances of conception.I would like to emphasize that for any investigation and management to deal with conception and pregnancy, it is important to include and involve the partner as well.Hope I have answered your query. Let me know if I can assist you further.Regards,Dr. Sabina Hussain"
},
{
"id": 83047,
"tgt": "Lump on shin above ankle, knee hurting severely, causing a knot. Is it due to lupus?",
"src": "Patient: Yes i have a lump on my shin right above my ankle idk if it might have something to do with why my ankle and knee are hurting severely or not i already have a bad knee and now this knot has appeared and i think its causing the pain in my ankle and knee but i dont know what it is. I do know i have lupus so any advice would be great thank you Doctor: it could be related to the knee pain which sometimes may radiate to the leg."
},
{
"id": 175767,
"tgt": "Why does my son have mucous and blood in his stools?",
"src": "Patient: My 3 year old grandson has mucous with a small amount of blood in his stool. He has also been c/o his tummy hurting and his knees and back hurting over the past 3 days. No lethargy or vomiting. He has an appt with his pediatrician this afternoon. What things can cause these symptoms and how worried should I be? Doctor: Hi DEarWelcome the HCM,Sir, the baby has developed stomach infection. DYSENTRY.REQUIRES STOOL CULTURE AND SENSITIVITY TO GET THE EXACT BACTERIA CAUSING THE PROBLEM AND EXACT DRUG TO BE USED.IMPERICALLY ONE CAN START OFLOXACIN TO GET THE EARLY RELIEF.hOPE THE QUERY IS ANSWERED.THANKS"
},
{
"id": 197719,
"tgt": "What causes irritation on my scrotum and penis?",
"src": "Patient: for the third time in the last 2 mths i hav an irritation on my scrotum and penis . it begins with an unbearable itch that only intensifies and skin sensitivity around tip of penis . the top of scrotum on the sides are white and the penis jus gets red and almost like blue coloration in some places . the only thing coincidental ios that i also hav a n itchy blister n the exact same spot on my wrist that came up at same time lasat time and more importantly i took bactrim for another problem both times and within the day problems began Doctor: Hi..Welcome to HEALTHCARE MAGIC..I have gone through your query and can understand your concerns..As per your complain it seems that you have a fungal infection in the scrotum and penis and it seems to be secondary to antibiotic intake that is bactrim that you have taken every time you experienced the symptoms..It seems to be Mucocutaneous Thrush leading to the symptoms of burning, redness and itching severely..I would suggest you to consult an Internal Medicine specialist and get evaluated and a thorough clinical evaluation and investigations like urethral swab culture can help in diagnosis and treatment can be done accordingly..You can be advised to apply antifungal ointment over the infected areas and along with it an oral antifungal medication course like Fluconazole can help..In case if you are sexually active or married the same treatment can be given to your wife/partner..Along with it take probiotics like Yoghurt and also keep the area clean and dry..Take Levocetrizine for itching and redness.Hope this information helps..Thanks and regards.Dr.Honey Nandwani Arora."
},
{
"id": 166815,
"tgt": "Suggest treatment for cough and cold in a toddler",
"src": "Patient: hi my 2 and half year old son has a bad cold and cough for 2 weeks, i took him doctors in the end and they gave me a prescription but told me not to get it unless he doesnt improve in few days as he had had it for few weeks should be on its way out. he has got rid of it now but passed it on to my 10month old baby, she has a snotty nose and a cough i have given her calpol and neurofen for kids is there anything else i can do to help? or should i just take her to the doctors? the cough is hurting her as after she coughs she cries for a bit. she is drinking plenty and eating food still Doctor: Hi,I understand your concerns but snotty nose requires an antihistaminic which is not recommended at the age if 10 month. It is better that you give her salty water nasal drops that will clear the secretions. You should add an over the counter cough suppressant.Hope I have answered your query. Let me know if I can assist you further.Regards,Dr. Salah Saad Shoman"
},
{
"id": 57090,
"tgt": "How to reduce the SGOT and SGPT?",
"src": "Patient: Hii Doc, Pease help me : 1. How to reduce the SGOT and SGPT? I means with medicine and without medicine, or there are any special treatment to do ? 2. How the procedure if we would like to do the operation of lipoma under the feet ? This operation need totally anesthesia or just local (its call spinal anesthesia?) Thanks. Doctor: Hello Increased SGPT indicates liver injury.Increase in SGPT may be due to many reasons like hepatitis,alcohol intake,altered lipid profile,medicines,auto immune causes etc.SGOT is non specific and it increases in many conditions.You need few other investigations like routine hemogram,Random blood sugar(RBS),Complete liver function test(LFT),Viral markers,ultrasound of abdomen.Further investigations can be done if needed.Proper treatment depend upon findings.You may need few medicines after investigations.Lipoma under feet should be ideally removed under spinal anaesthesia.Lipoma may have internal extension,so only local block should be avoided.Get well soon.Hope I have answered your question.Take CareDr.Indu Bhushan"
},
{
"id": 189110,
"tgt": "Discomfort in jaws, feel fatigue and tired. Cause? Cure?",
"src": "Patient: Hello, I got a discomfort under my jaws, on both sides for the past the few days now. I did notice that it really comes up when I drink alcohol (colder or room temperature, it doesn't matter). I have no fever, but I do feel fatigue and tired. For the past few months I only get max 6 hours of sleep and eat very irregularly, I also smoke. That along with being stressed often makes me think that the tiredness is a result from there. While I am aware that I should start sleeping/eating regularly and quit smoking, I wonder if the discomfort I now have under my jaws could be related, I did have a cold and a flu 2 months ago, swollen tonsils and such, I took antibiotics for it and I felt I treated it. My tonsils are not swollen now, I can swallow food OK, but why the discomfort ? ... any ideas? thanks. Doctor: it seems to be chronic allergic sinusitis and it looks like allergy to yeast the post nasal drip of sinuses causes recurent infection in tonsils and throat and off and on cold and fatigueyou get xray pns waters view you will get answer this time you take montair fx bd(combination of montelucast and fexoenadine)put sea water drops in each nstril 2 drops nightthis will clear your back of throat and eustachian tubesthis will clear your jaw discomfortsyp gelusil 2 tsf tdstake for 3 wek and must consult doctor after xray"
},
{
"id": 30849,
"tgt": "Suggest treatment for fever with muscle pain",
"src": "Patient: I am a 37 year old female. I had a fever last weekend for 2 days with no other syptoms except extreme muscle aches in my back; it went away but now almost 6 days later I have developed a red rash on my arms, legs, buttocks and stomach. The rash is not raised and doesn't itch for the most part (my skin feels slightly tight). Could this be related to the fever. I have used no new products etc that could cause an allergic reaction. Is there a virus or such going around that cause a high fever and muscle aches followed by a rash a few days after the fever? Doctor: Thanks for posting your query to HCM .As I undersand you are concern about your rash . but befor suggesting treatment I want to know what medication have you had during fever episode , are you still on some medicaion or not .rashes are painful or painless , rashes are present on both side of body or only on one side .Present complaint could be allergic or may be due to viral illness. But as there is no associated fever and body ache so chances of infection are least but we canno ignore it .So I suggest you to do only one test 1.Complete blood count .and if possible send me picure of your rash .and meanwhile you can take antiallergic medicine Loraadine 10mg once at night for syymptomatic releif .review me afeter required detail .thank you ."
},
{
"id": 157019,
"tgt": "Can gastroscopy be cancer?",
"src": "Patient: Hi Doctor, my boyfriend had a gastroscopy performed last week. His physician called today to tell him that an amount WBCs were found in the sample that he took. He told him to continue to take his Protonix for the next 4 weeks and then they will do another gastroscopy. My boyfriend is only 27 and his father died from brain cancer 14 years ago. Should we be concerned that this could be some preemptive sign of cancer? Doctor: Hi and welcome to HCM. Thanks for the query. Stomach cancer is extremely rare in this age and this is the last thing to think of. WBc in sample isnt indciating anything maligannt. it is indicating inflammation of stomach mucosa which is seen in gastritis, gerb or h.pyloriinfection. so you should not be worried at all. He should continue with antiacid mediciens and change his dietary habits.Wish you good health. Regrads"
},
{
"id": 110517,
"tgt": "What should I do for increase low back pain, fatigue, wgt gain?",
"src": "Patient: I have recently been experiencing increased fatigue, severe low back pain (last 6 months), increased weight gain (30 lb in 6months), and most recent, abdominal distention/swelling and fatigue. Recent MRI, CT negative for back pain...therefore on Gabapentin to manage. CT showed enlarged abdominal lymph node. No treatment yet as doctor on holiday for two weeks. Recent blood work all WNL, except low Alkaline phosphatase flagged. Previous health issues: kidney stone formation, melanoma March 2011, undergoing menopause (9 months) Doctor: Hello, I have studied your case. There is possibility of malignancy leading to pain.Pain on swelling indicates inflammatory process or nerve compression.I will advise you to do PET SCAN You may need further investigation to rule out malignancy.Hope this answers your query. If you have additional questions or follow up queries then please do not hesitate in writing to us. I will be happy to answer your queries. Wishing you good health.Take care."
},
{
"id": 119071,
"tgt": "Can an online doctor explain the histopathology report ?",
"src": "Patient: Dear Sir, MY Histopathology report says : Gross Description: Multiple cores of tissuesand Microscopic Description says; Metastatic carcinoma consistent with primary origin in an andrenocortical neoplasm The Tumor cells express synaptophysin focally.They are immunonegative for CK, Melan A, chromogranins, HSA and inhibin,The Mib-1 Prolieferation index is 5%Are these reports sensitive Doctor: Hello, You have not mentioned site of biopsy .Whatever may be site of biopsy this report shows malignancy there which has most probably come from adrenocortical area .it means that cancer has spread and has reached the site of biopsy. Adrenocortical area is situated just above kidney . Consult your oncologist for further guidance."
},
{
"id": 138896,
"tgt": "What causes neuropathy having numbness in the ankles after giving birth?",
"src": "Patient: I am a 33 year old female 8 weeks and 2 days postpartum. This was my second child and they are 15 years apart. During pregnancy I had occasional tachycardia and a rug burn sensation behind my right knee and on my right calf when I rubbed or scratched. Around the end of my second trimester to my third trimester my thumb on my right hand developed a numbness (right is my dominate side). Since 1 1/2 weeks postpartum I have had peripheral neuropathy. I have been taking neurontin for close to 3 weeks now. I started with 300 mg then 600 mg now taking 900 mg and my neuropathy has went from the balls of my feet and toes to having numbness in my ankles. I am a RN and will have to be going back to work soon. Will this subside? If so how much longer until it does? Doctor: hi,your hand symptoms could be due to Carpal tunnel syndrome, and the leg and ankle numbness could be due to the stress and deficiency the body has gone through due to pregnancy, one thing I need to know, have you felt some improvement in symptoms due to the drug you are taking or its just ineffective. All these pregnancy stress symptoms usually settle with time and that can vary form few days to months .I would recommend you to continue treatment, which you are going through and see if things settle before making any decision on consultation, moreover watch for diabetes, if you have had or would have developed during pregnancy.I hope I have been successful in answering your concern."
},
{
"id": 161067,
"tgt": "What causes frequent bowel movements which is stringy?",
"src": "Patient: My 3 week old baby wasnt pooping for a day my dr had said to give her prune juice because she always strains like she has to poop, that was yesterday morning and she has been pooping since. However ive noticed it getting more yellowish and this morning it was like stringy. is that normal? Doctor: Hello,It could be an indigestion and will settle by itself. No definite treatment is required. If she is developing fever or consult a pediatrician and get evaluated. In that case conditions like gastroenteritis has to be considered.Hope I have answered your query. Let me know if I can assist you further. Regards, Dr. Shinas Hussain, General & Family Physician"
},
{
"id": 99657,
"tgt": "What is the treatment for asthma and lump in the back of the head?",
"src": "Patient: i have had a chest infection and bad bout of asthma i have been back tomy gp as i am constantly dizzy and feel drunk i have noticed a lump at the back of my head right at the base of my skull on the left hand side is moves about is not completely round does not feel sore but is large enough for me to seee if i holda mirror at the backof my head Doctor: A lump over back of neck might be just a boil,lipoma or sebaceous cyst,neuroma etc,it can be removed easily by simple excision by general surgeon which will take hardly 10-15 minutes under local anesthesia.For that u go to a surgeon and get removed it.For your asthma there are many inhalers and nebulizers,tablets like steroids,alpha agonists,anticholinergics are available in market,u can easily go to pulmonologist the get prescribed it."
},
{
"id": 202069,
"tgt": "What causes difficulty in pulling back penis foreskin?",
"src": "Patient: Hello,I am 20 years old and I am uncircumcised. I have no problem with my genitals but I feel when it is in un-erect state it is very small. But when in erected stage I have no issues...My foreskin when pulled back is slightly attached to the back part of the head,Is that why there seems to be a problem? Doctor: welcome to Health care magic.1.If it is causing pain on retraction of the foreskin on erection, it may try gradually it will enlarges and you will be able to retract.(try with lubricants )2.If it doesn't work then needs circumcised.3.Its seen in case of phimosis.4.Get the needful things done, maintain hygiene as it may cause infections.Anything to ask ? do not hesitate. Thank you."
},
{
"id": 190436,
"tgt": "Treated for tooth abscess twice, just behind the wisdom tooth. Area painful, yellow pus with nasty odor. Further treatment ?",
"src": "Patient: About a year and a half ago I had a tooth abscess . It was my very back tooth on the bottom right. This was just before my wisdom tooth came in behind it. I ve been treated for the abscess twice since then but haven t had the money to get my tooth pulled. Now my wisdom tooth is almost completely grown in behind the bad tooth and it s partially covered by my gums. But only covered a little bit on the top and completely covered on the back. The area is painful most of the time and now I when I put pressure on my gums by the tooth, yellow pus comes from the back of my gums. And it has a nasty odor. What s wrong with me? And what type of treatment do I need? Doctor: Hi, Thanks for posting your query. First, the pus and discomfort you are feeling are because of the infected wisdom tooth. Some times the food also gets lodged in between the flap and the tooth resulting in infection due to degradation of food there leading to pain and bad odor (halitosis). Getting the pus removed again and again may not resolve your problem permanently if the tooth is infected. Ideally you need to get the offending tooth extracted and the pus drained. Your dentist may take an xray of the area to confirm the diagnosis. Meanwhile you can start with antibiotics and anti inflammatory drugs to reduce the infection. You should also do warm saline gargles 3-4 times/day to help in dissolution of the stuck food and removal of pus from the area. Mouthwash rinses twice daily after brushing are also recommended. Regards"
},
{
"id": 125531,
"tgt": "How to treat leg bruising?",
"src": "Patient: I have strange blackish bruises on my legs that appear every now and then, they re big but are not painful to touch. They are kind of hard to see but if you look closely you can see them very well, so they re not dark black, but still they re black, do you understand? Doctor: Hello, These are nothing but blood collection underneath the skin. Nothing much to worry and it will settle by itself. You can apply ice packs for symptomatic relief. If symptoms persist, it is better to consult a physician and get evaluated. Hope I have answered your query. Let me know if I can assist you further. Regards, Dr. Shinas Hussain, General & Family Physician"
},
{
"id": 34497,
"tgt": "Suggest treatment for difficulty walking caused post cellulitis treatment",
"src": "Patient: My dad has a bacterial infection that has settled in his foot at first it was said to be cellulitis and after antibiotics the cellulitis subsided however he s had the swelling and redness and heat for 9 months to a year..he has been put on clindamycin now and has been on cephalexin ,bacterim and indomethacin,,how long is to long for this infection to be continuing he struggles to walk he says its painful and kind of tight feeling and then every now and then he just get sharp pain\u2026he tries to walk on it a bit every day but is this good or bad to walk on his food if the infections seems to have settled there and what is the best way to get it taken care of? Doctor: My concern is that the infection has gotten into the bone. This is difficult to treat and does require long-term, often IV antibiotics, and possibly surgery. He needs to find out if this is the case and get the appropriate treatment. They may need to surgically remove the infected areas to get a cure. Hope this helps."
},
{
"id": 182509,
"tgt": "How to treat a bump at back of mouth?",
"src": "Patient: hi on the tooth all the way in the back of my mouth i have a bump the size of a #2 earaser and now my whole mouth hurts and my tonsol hurts please help me Doctor: Thanks for your query, I have gone through your query.The pain and bump over the tooth can be a pericoronal abscess, that is collection of the pus below the gums over a wisdom tooth. Consult a oral physician and get a radiograph done to rule the amount of space present for the tooth to erupt if space is there for the tooth to erupt then the gum covering over the wisdom tooth can be removed.If the space is not available then you have to get the wisdom tooth removed. Mean while you can take a course of antibiotics like amoxicillin 500mg and metronidazole 400mg tid for 5 days (if you are not allergic). Do saline gargling. i hope my answer will help you, take care.I hope my answer will help you, take care."
},
{
"id": 200279,
"tgt": "What causes erectile dysfunction after using Amway nutrilite men s pack?",
"src": "Patient: Since my 40 year old husband started taking the Amway nutrilite men s pack with the 4 vit supplements our sex sessions r going down hill. He can t keep an erection. I told him to stop taking them but now I want to know what will help get his mojo back quickly? Doctor: Thanks for asking in healthcaremagic forum Amway nutrilite contains vitamins and mineral supplements which is unlikely to cause your husband erection problem. So, it may be due to normal age related factor, if not has to be investigated for the cause. Kegel exercise where perineal muscles are put to work can help him a lot. All the best."
},
{
"id": 6779,
"tgt": "Does a negative urine test and delayed periods indicate pregnancy ?",
"src": "Patient: Im vary scared i havnt had my period since october 18th i been vomitting i took 4 home pregnancy test they were all positive when i went to the docotrs today around 9am i took a urine test it came back negative. i drank alot of water before i went to help myself urinate im confused no w wondering if im pregnant or not does anyone have anything that may help me Doctor: Hi, Thanks for your query, If urine tests are doubtful go for blood hcg test for confirmation. Now ultra sound also will tell about pregnancy. Ok and bye."
},
{
"id": 215436,
"tgt": "Suggest treatment for pain in fore head and behind the eyebrows due to hit on head",
"src": "Patient: Hello! I pulled my shredder down off the shelf and it hit on the right side of my head and neck a few weeks ago. It really hurt so I sat with ice only for about 15 minutes. A day after I did it, when I would touch the top of my head, it felt cool inside, like fluid? I thought maybe it was just bruised and blood rushing to heal that spot? My forehead and behind my eye brows were hurting too, and now my sinuses also hurt. When I bend down to do something, I feel the bulk of the pain in my forehead and the top of my head but also sinus pain too. Still have that fluid feeling, especially when I day or get up from bed. Do I need to go in for tests or something? :( Doctor: Hi, I suggest you to do hot water massage over that area. Yes, sinus will b filled with fluid. I suggest you do to steam inhalation. Do take antibiotic and pain killer for a week. If still pain persists do a scan and meet a physician. Hope I have answered your query. Let me know if I can assist you further. Regards, Dr. Rohit S Menon, Dentist"
},
{
"id": 3981,
"tgt": "What is causing trouble in conceiving even with normal HSG?",
"src": "Patient: Hi I am swati 32 years old and married before 2 years trying for pregnancy but not happen i meet the doctor also they told me to take HSG test i did it but the report are also normal no blockage now from last 3 months i am taking fertyl 50mg tablet plz guide me what will be the problem Doctor: Hi, thanks for writing..Is your husband's semen analysis normal. Along with increased dose of clomiphene get a follicular study on day 11 to see for follicle development and continue study tillit reaches 20mm. You can take inj hcg and get IUI done after 36 hrs. It increses the chances od pregnancy. Hope I have answered your queries.. Good day"
},
{
"id": 162946,
"tgt": "What causes fever blister on lips and stomach pain in a 5 years old?",
"src": "Patient: My 5 year daughter has told me her stomach hurts once or twice a day for two weeks. It hasn t stop her from doing any physical activity she still attends tumbling class, and play normally. She has said a few time it hurts to go to the bathroom (bowel movement). She also fever blister on lips and in mouth. This morning she threw up several times and started to wet the bed last night. Doctor: Hello and Welcome to \u2018Ask A Doctor\u2019 service. I have reviewed your query and here is my advice. Does your daughter have a fever? Has she ever had blisters like these before? Has she been exposed to anyone with fever blisters? Since these might be contagious, avoid kissing and sharing drinks and cutlery. Please see that she drinks adequate amounts of fluids and takes acetaminophen or ibuprofen for pain. Blisters persist for about a week. I wonder if it hurts her to have a bowel movement because she is impacted. Children who don't have a BM regularly can have accumulation of large amounts of fecal matter in the rectum and colon and this part of the intestines can become distended. This may be giving her the abdominal pain. You may wish to take a look at the toilet bowl before she flushes to see how large the stool is. Children who become impacted can pass stool so large it can block the toilet. See if the bed wetting stops. Fecal impaction(and we don't know she has it) can be associated with wetting bed. I hope this helps you. Please return if we can assist you in the future. Arnold Zedd, MD, FAAP"
},
{
"id": 9126,
"tgt": "How can I get rid of hairs on my face and neck ? What ointment can I apply to take marks of chicken pox from my face ?",
"src": "Patient: actualy doc i have very much hairs on my face and neck ....... and also one more prob that i hav chicken pox markz also...which look very ugly.. Doctor: Hi greetings from health care magic this is called hirsutism. It is due to hormonal causes. too much androgen in the system can cause hirsutism. the skin's increased conversion of testosterone to a more potent androgen called DHT (dihydrotestosterone) is also one cause."
},
{
"id": 161247,
"tgt": "Can giving Similac be safe instead of Lactodex?",
"src": "Patient: HI, when my baby was 2 weeks, I gave her Lactogen which caused her extreme allergy. After that I gave Lactodex which suited her. Unfortunately Lactodex is not available in US. My baby is 4 and half months now. I have bought Similac here but I am afraid whther it will suit her. Can similac cause allergy? Doctor: Hi, Similac can be given safely. However, it contains soy extracts. Out of both, similac is better tolerated. Hope I have answered your query. Let me know if I can assist you further. Regards, Dr. Prasanna Lakshmi, Pediatrician"
},
{
"id": 42512,
"tgt": "Is undergoing IVI safe and will it increase pregnancy chances?",
"src": "Patient: Good evening doctor, I am 36yrs old now and trying to concieve my 2nd baby. As per my doctors advice I have to undergo IVI to concieve as my husbands sperm count is very low. Please advise is this procedure safe and will it increase my chances of pregnancy and also if I will deliver a healthy child. thanks and regards Doctor: Hi,Welcome to HealthcareMagic. IVF is quite safe procedure so don't worry. Since your husband sperm count is low IVF increases chances of pregnancy. Here healthy sperms are fertilised with egg and put into uterus. So problem of low sperm count is reduced as hardly they need 4 to 5 healthy sperms. Once embryo is transferred and you conceive it is same as like normal pregnancy and you will complete it as normal pregnancy only. You will give birth to healthy baby as they screen embryo for many diseases and put only healthy embryo. The chances of abortion , normal delivery and any pregnancy complications are same for IVF and normal conception. Hope I have been helpful .RegardsDr.Deepika Patil"
},
{
"id": 137010,
"tgt": "Does a surgery cure accessory navicular completely?",
"src": "Patient: I have an accessory navicular and I have three questions 1. Will surgery on the foot by removing the bone completely fix my problem if all goes well/ will I be able to ski, play soccer, etc.? 2. How much does surgery cost 3. What are the risks of the surgery Doctor: HelloYes surgery will reduce your problem by 90 percent.Yes you can join sports later on.Cost may vary according to centre and country.Arthritis and tendon weakness can occur after surgery.Hope this answers your query. If you have additional questions or follow up queries then please do not hesitate in writing to us. I will be happy to answer your queries. Wishing you good health.Take care"
},
{
"id": 185458,
"tgt": "Suggest treatment for illness caused after rear molar extraction",
"src": "Patient: Hi, I m not one to be concerned about colds and minor illnesses, but on Friday I had a rear molar extracted that had been causing me some trouble,(I had a filling put in 2 weeks before which had caused pain and tingling to the side of my face that travelled to my arm just on the right side) and as I could not afford root canal and a crown I opted for extraction. Not 3 hours after the extraction I came down with cold type symptoms, blocked nose, slight earache, cough, phlegm, etc. This seemed to have peaked yesterday as I began to feel a bit better. However, today, I have been feeling sick, vomiting, and a slight pain in my ear is now complete hearing loss in my right ear, combined with tingling and numbness in the right side of my face from my eye, across my cheek to my chin and ear. The sudden onset of these symptoms have me a bit worried, what is the best course of action, should I ride it out until I can get to my GP on Thursday, or do something sooner? Kind regards Sarah Doctor: Hello, Thank you for consulting with HCM.Actually these third molars are in very close contact to the maxillary sinus. the symptoms which you are mentioning it looks that there is a possibility that the infection of molar has extended to the sinus or the dentist might have perforated the sinus.Better that once you should visit an ENT specialist and get a skull x-ray done which will show you the infection.Hope it will help you."
},
{
"id": 147438,
"tgt": "Is it safe to drink with sleep apnea and history of stroke ?",
"src": "Patient: Hello, I m 40 yrs old and just had a stroke. I due smoke and drink, but all the test came back negative, kidneys, liver, and all other blood work. I did a sleep study and found out I have clinical sleep apnea. Is it safe for me to continue to drink? I know I must quit smoking. Doctor: HIThank for asking to HCMI really appreciate your concern, if you have history of sleep apnea and stroke then this is better to stop drinking the alcohol and quit the smoking even if you do not have such medical condition then also it would be nice to away from the alcohol and smoking particularly prior to bed, hope this information helps you, have nice day."
},
{
"id": 87655,
"tgt": "What causes sinking and bubbling feeling in lower stomach?",
"src": "Patient: I have experienced a 'sinking' and a 'bubbling' feeling in my lower-stomach: This is the first ever, time that I have had such a problem: Notwithstanding stress and anxiety, what could be the reason: I have begun to use Contrloc 40mg twice a,day: It has alleviated the intensity to a large extent with freedom of sleep and activity - No history of GERD or anxiety/depression I do not really, want to do Berium X-ray - or, endoscopy for expense and risk reasons in the latter - especially, being, overseas as an American Two questions: What and, why this condition has arisen? What else, can I supplement the Controloc 40mg with (antibiotic etc;)? for a greater thrust? Please assist Aftab - with thanks Doctor: Hi Aftab.Thanks for your query.The feeling of sinking and bubbling in the lower stomach can be due to an intestinal infection of mild form.I would advise you to consult a local Doctor to get you a 5-day course of an antibiotic and metronidazole with probiotics.This may solve your problem and you may not have to do anything more. IF there is no relief you can go for the investigations of the blood, urine and stool.As also ultrasonography of the whole abdomen and endoscopies if required."
},
{
"id": 92796,
"tgt": "Severe stomach pain, stomach feels bloated, nausea, vomiting, diarrhea, blood in stool. On pepcid and zofran. Sufficient?",
"src": "Patient: Tonight I have severe stomach pain . My stomach feels bloated and hard. I ve had stomach problems since last Tuesday, consisting of nausea , vomiting , and diarrhea . I mostly took Pepto Bismol to deal with it. When it started there wasn t much pain but the pain has gotten terrible. Sunday night I was in such pain I went to a 24 hour CVS where the pharmacist recommended Zantac , it helped and I was able to sleep. However, it doesn t seem to help with the pain anymore. I went to an urgent care facility yesterday since my doctor s office was closed, at first my blood pressure was through the roof--the nurses words--so the doctor gave me IV fluids as well as IV pepcid and zofran as well as a prescription for zofran. This morning I unexpectedly had diarrhea--I was in the shower, felt like I was passing gas but instead a large amount of diarrhea came out. When I was cleaning it up, most was brown, but a small amount was red. Between the pain and the possibility of blood in my stools, I m wondering if I should go to the ER tonight. I have an appt with my Dr. tomorrow but the pain is really bad right now and I m wondering if I ll be able to sleep. Thank you for your advice. Edit: Went to ER, then to my regular Dr next day. I have gastritis, and I'm on 2 different antibiotics now. Starting to feel a little better. Can't eat anything but toast and gatorade though, so I miss real food. Doctor: there are hances of stomach infection as you have gastritis and acidityyou only took antacids if your antibiotics are working u can continue with antacids liquid tdsantispasmodic sos for painlot of warm waternon greesy non oily non fatty food for few daysthe infection can also cause blood in stoolif the infectons are over you can get reliefbut you must consult doctor to findthe condition physically and treat accordingly"
},
{
"id": 101286,
"tgt": "Any suggestion for hypothyroidism, bp, asthma, swollen leg veins/abdomen, 16.21 TSH, premenstrual symptoms?",
"src": "Patient: I was very ill with untreated hypothyroidism 2 yrs ago obese, bp 165/120, asthma, swollen leg veins, swollen abdomen, wheezing, fatigued, depressed and period abruptly stopped. Now TSH is 16.21 and new doc just started treating with Eltroxin. Have premenstrual symptoms. Will period return Doctor: HI, thanks for using healthcare magicHypothyroidism would be associated with delayed periods, fatigue, depressed mood and weight gain.Treatment with the hormone replacement will help these issues. Your periods may return as your thyroid hormone levels return to normal.In some women with increased weight, there can be problems with menstruation due to other hormonal changes. If the period does not recur despite normal thyroid hormone levels, you should consider visiting your gynaecologist for assessment.I hope this helps"
},
{
"id": 151092,
"tgt": "Dizziness, pins and needles in arms and legs. Brain CT and chest x-ray clear. Have PCOS. Due to being overweight and smoking?",
"src": "Patient: Hello, I m a 26yr old female. I am over weight and a smoker. I THINK I have an anxiety problem because I get so paranoid I ll get a migraine (I have not ever had one) that I don t want to leave the house. Anyway, for almost three years I have had dizzy spells. I suddenly get dizzy mostly in the afternoons but can occur anytime. I do not feel like the room is spinning, just my head. I get lightheaded, or a very heavy headed feel. I get pins and needles in arms and legs very quickly and sometimes get a tingly feeling in bizarre places like one cheek. I constantly yawn and have had a brain ct and chest X-ray which were both clear. I have been diagnosed with pcos and am not taking anything for that. Is it my weight and smoking? Or could there e something more sinister? Doctor: Hi, Welcome to HealthcareMagic Public Forum From your description, it appear that you probably have generalized anxiety disorder and agoraphobia. In anxiety tests reports are usually normal. Anxious patient frequently have dizziness, tingling/numbness and headache. Considering your symptom severity, it is better to meet psychiatrist early. I hope this information has been both informative and helpful for you. Wish you Good Health. Regards, Dr. Ashish Mittal www.99doctor.com"
},
{
"id": 41869,
"tgt": "Suggest a solution for infertility inspite of using medicines",
"src": "Patient: Hi v r trying for baby since two yrs . V married in 2007. And started to plan since two yrs Took clomid 6 cycles.. no use And took sisten and estrogen with hcg trigger and failed again Now doc adviced me to take deviry to get my period. My husband sperm count is 25mil. Could u pl suggeste me any solution Doctor: Hi welcome to healthcaremagic.I have gone through your query.All medicines you have taken is either for increase follicle size or to make uterine wall favourable for implantation.Your husband is count is near normal. But it sholud have normal motility and morphology.I would suggest continue with deviry for 3/4 months and wait to conceive. If couldn't get success then go for iui intrauterine insemination, is good option for you.Hope i answered your question.Would be happy to help you further.Take care."
},
{
"id": 60850,
"tgt": "What causes severe pain in the pelvic area after drainage of a lump?",
"src": "Patient: HI I have a large lump that has tripled in size since being put on antibiotics. We drained it this morning and there was yellowish-green pus followed by blood and a bad odor. The size has not gone down. Now my pelvic area hurts and there is a large internal lump on my let side. I was told it was a staph infection. What is this really? Doctor: Hello and Welcome to \u2018Ask A Doctor\u2019 service.I have reviewed your query and here is my advice.As per my surgical experience, the lump in the pelvis suggests an infected boil or sebaceous cyst. After clinical confirmation, it requires definite intervention. Hope I have answered your query. Let me know if I can assist you further.Regards, \u00a0\u00a0\u00a0\u00a0\u00a0Dr. Bhagyesh V. Patel"
},
{
"id": 160747,
"tgt": "What causes red bumps and swelling in legs with fever in toddler?",
"src": "Patient: a friend of mine has a 1 year old toddler and he has red bumps on his legs along with swelling of his feet and bruising on his feet and legs...she said he was sleeping a lot so she had to wake him up and he is also running a fever with all the symptoms i have described can you please help me identify what it is Doctor: Hi,From what you described, he is having fever, red bumps on legs, along with erythema ans swelling - all these suggest that he is having a skin infection that is now progressing to cellulitis. Kindly take him to doctor without delay as this need immediate treatment with antibiotics and anti-inflammatory drugs. Meanwhile give him paracetamol for pain relief and keep the affected limb elevated.Hope I have answered your question. Let me know if I can assist you further. Regards, Dr. Muhammed Aslam TK, Pediatrician"
},
{
"id": 49108,
"tgt": "What s the difference between primary and secondary parathyroid disease?",
"src": "Patient: My daughter had acute renal failure but last June her kidneys recovered to approx. 35%. She has a calcium level of 10.5 and I am wondering if she may have primary or secondary parathyroid disease? However, what is the difference between primary and secondary parathyroid disease...that I don t understand. Please explain. Doctor: Primary hyperparathyroidism is primarily a parathyroid pathology where the kidneys might be affected secondarily due to high calcium levels and renal stones. Hence it is important to known what was the cause of acute renal failure in your daughter's case. on the other hand a long standing kidney problem itself can lead to a parathyroiid pathology which is known as secondary hyperparathyroidism."
},
{
"id": 54122,
"tgt": "What is the prognosis for liver cirrhosis with enlarged spleen?",
"src": "Patient: my friend was diagnosed ten years ago with portal vein thrombosis and just found out he has cirrohsis of the liver. the doctor also said he is not a candidate for operation as he has an enlarged spleen and numerous variocies (not sure of spelling)....what are his options and how much quality time does he have left? Doctor: 1) your friend may have cirrhosis (liver damage) with portal hypertension (spleen enlarge) associated with portal vein thrombosis.2) liver transplantation is treatment for this but portal vein thrombosis is relative contraindication for this depending upon severity of thrombosis.3) his quality of life depends on his current medication status and current investigation details. in case of extended thrombosis, without possibility of transplantation, he need to continue medication support for better life."
},
{
"id": 72234,
"tgt": "What causes night sweats and shortness of breath?",
"src": "Patient: My husband has copd. Abuteral 2 times a day and advair as needed. For the last couple of nights he wakes up during the night with night sweats and a shorness of breath. He het up does his advair and goes back to sleep. Is the copd getting worse or something else? Doctor: Thanks for your question on Healthcare Magic.I can understand your concern. Night time sweating and breathlessness in COPD patient is suggestive of worsening of disease. In my opinion, he needs higher and longer acting inhaled treatment for better control of COPD. So get done PFT (Pulmonary Function Test).PFT will tell you about severity of the disease and treatment is based on severity only. He will mostly improve with long acting inhaled bronchodilators (formoterol or salmeterol) and inhaled corticosteroid (ICS) (budesonide or fluticasone).Don't worry, he will be alright. Hope I have solved your query. I will be happy to help you further. Wishing good health to your husband. Thanks."
},
{
"id": 10122,
"tgt": "Suggest medication for hair fall",
"src": "Patient: Hi iam 27 yrs old and i have only very sparse hair .it happened before 5 years wen i went to hostel due to salt water.but no improvement after i changed place also .i hav tried minoxidil for more than 6 months but no change .i badly want to improve my hair please tell a suggestion Doctor: Hello, I suggest you to also take an oral biotin supplement. You may continue with Minoxidil solution, 1ml, twice daily. Hope I have answered your query. Let me know if I can assist you further. Take care Regards, Dr Kakkar S., Dermatologist"
},
{
"id": 118103,
"tgt": "How to treat elevated blood pressure?",
"src": "Patient: my daughter who is 15 weighing about 120 lbs just started adderall today about 2 hrs ago was her first dose. she says she feels really dizzy her head hurts and she feels shaky. i am thinking her blood pressure is elevated and wanted to know what i could do at home to help her or should i thake her to the er Doctor: Thanks for contacting HCM. You have not mentioned why your daughter has started adderall. if she is taking it to reduce weight then it should be discontinued immediately as it has no proven effect in reducing weight but has potential for really serious side effects and even addiction. Adderall has known side effects of causing dizziness and high blood pressure. Take her to Er get her Blood pressure checked and get her treated for dizziness and do not give her any products and medication including adderall with out consulting a doctor. regards"
},
{
"id": 95412,
"tgt": "I am not exercising but still feel pain in abdomen. Why is it ?",
"src": "Patient: My abdominal muscles hurt as though I did 200 crunches or some other physical activity, but I haven t done any exercising in at least two months. Why do they hurt? I m a 19 year old female with a healthy weight, and healthy eating habits. I d hate to make an appointment with my doctor if this is something trivial, so I thought I d post my concern here. Just 10-15 minutes prior to the pain, I decided to try a diet Coke (I usually stick to water, milk, or 100% juices). I doubt there is any relevance, but I can t think of anything else I could have done that would have caused any pain. Thank you for your time. Doctor: the problem is due to sudden stop of exercise.dont do like this that is sometime ovre exercise &sometimes no exercise .do exercise in level.for your problem you can take balarist liquid 2 teaspoon 2time after meal for 1wk ."
},
{
"id": 165767,
"tgt": "What causes sudden weight gain in face?",
"src": "Patient: hi, my 4 year old daughter has all of a sudden in the last week got fatter in the face. she goes to kindy 3 days a week and relatively active and doesnt eat huge portions of junk, though could possibly eat a little more healthy. can you advise why she may have all of a sudden gotten fatter? it mostly only noticed in the face the weight gain though. Doctor: Hello, I have read your question and understand your concern.Swollen face may be due to various reasons. The most worrisome is Nephrotic Syndrome which can result in peri orbital puffiness, especially in mornings after getting up from sleep. You should send his urine in Laboratory for proteinuria.Hope your query has been answered."
},
{
"id": 202738,
"tgt": "Varicocele in both testicles, testicle twists after sitting for a long time",
"src": "Patient: I m a 21 Male about 140 lbs. I have varicocele in both testicles, After i sit in a chair in one position for a little while, My left testicle will turn sideways when I stand up. I then have to twist it back into place. why does this happen and how can I prevent it? I went in to see my GP and he examined my testicles, He suggested I go for an ultrasound just to be on the safe side. So I did, that s how they found the varicocele in both testes. The testicle only twists sideways when I sit for a long time. Does it have to do with my Varicocele? Doctor: Good Day and thank you for being with healthcare magic!Varicocoele is dilation of the veins that drains the testes and doesnt have to do with the twisting of your testis. Varicocoele may or may not cause scrotal pain and discomfort. You may have a congenital condition called bell-clapper deformity where in your testis is not firmly attached to the scrotum. This may cause severe twisting of your testis leading to testicular torsion which is an emergency. I would suggest seeing a Urologist and the testis can be fixed to the scrotum to avoid torsion thereby avoiding emergency removal of the testis that usually happens after testicular torsion.Thank You.Regards,Manuel See IV, MDUrologist"
},
{
"id": 4630,
"tgt": "Trying to conceive with subserosa fibroids in uterus. Will it interfere with pregnancy?",
"src": "Patient: my name is Mhah Rose, i have four subserosa fibriods ontop of my uterus, since i dont yet have issues i decided to have a child before going for my surgery. The largst is 7cm the smallest is 2cm. I just had two miscarrages i five months. I dont know the cause. Dr can fundal subserosa fibroid interfere with pregnantcy Doctor: Hello!The fibroids do not interfere with conception but can interfere with pregnancy.You say that you have two miscarrages in five months and this is enough to say that you should remove the fibroids .After you remove you can try to conceive after 6 months.Dont be worried and take care for your self."
},
{
"id": 93943,
"tgt": "Suffering from GERD, infected with bacteria, severe abdominal pain, on gentamicin, got relief. Will it start again after stopping?",
"src": "Patient: Hello Doctor, I have been suffering from GERD for over 5 years now. 3 weeks ago , I was infected with a bacteria resulting in severe abdominal pain. Subsequently , I was put on gentamicin for 10 days and doxy 100 mg for another 10 days alongwith pantocid 40. The doxy course comes to an end today. Doctor has advised to stop all medications. I feel ok now, if not completely alright. My problem is : 1) I believe if I stop all the medications , the abdominal pain will return. 2) more than the pain, my problem is the bowel. Every morning my stomach never used to empty completely. I had to use the was room more than twice every day to clear my stomach. The stools used to be semi solid in nature .3) all of the above come alongwith elevated heart rate. - by being on doxy 100 mg alongwith pantocid 40 mg , the above symptoms are getting better if not completely cured. Do advise what should I do ? Doctor: Hi and welcome to HCM! Thanks for posting your query here. The bacterial infection would have been subsided now after using the antibiotics for 3 weeks. Hence after stopping the medication your problem will not return for sure. The difficulty in passing the stools can be referred as constipation. If there is a difficulty a diet rich in fibre, having lots of water and exercise will help in reducing the problem. All these mentioned will also help you in reducing the GERD problem. The increased heart rate could be because of your anxiety. Pantocid 40mg will give relief in GERD. You are feeling better because of pantocid. The only affect of using pantop for longer duration-more than 3 months- is rebound acidity. Please continue Pantocid for 2 more weeks. Then you can slowly stop the drug. Hope this is helpful to you. Regards"
},
{
"id": 186241,
"tgt": "Is very low sugar level a cause of concern?",
"src": "Patient: Usually it's a challenge to keep my blood sugar at a good # but recently i was to the dentist and they have prescribed an antibiotic called Metronidazole 250mg 8 x a day in prep for dental surgery as i had 2 absesses. I have just taken my blood sugar and my metre reads 3.3 which i think is dangerously low.What are your thoughts. I don't feel disoriented or anything. Doctor: Hello, Welcome to HCM . I have gone through your query, yes during dental treatment , sugar level should be maintained as you have abscess in tooth ,so treatment is Root canal and for this you should be given anaesthesia during treatment and level should be maintained to avoid hypoglycemic shock or syncope. Consult physician and go for RBS and go for treatment and then consult dentist for treatment. Hope this will help you."
},
{
"id": 132787,
"tgt": "What causes pain at the top of stomach,chest and neck?",
"src": "Patient: I have been getting woke up in the middle if the night with pains from the top of my stomach over my chest up to my neck and round the back,then I would take very short pin and neddles but the come and go my doctor seems to thing I have had a heart attack,but I have argafobia and can t go out to Hospatil can you please tell me what this could be Doctor: hihope this msg finds u in good health.i have gone through your complaints and under stand your concern. ur complaints have nothing to do with heart attack.its coz of compression of neck and chest nerves that might be causing d symptoms i suggest u get a scanning done to find out whats wrong. nothing to worry about. hope your question has been answered. if u have any follow up queries, feel free to consult me anytime. thanks Take care god bless"
},
{
"id": 146666,
"tgt": "Does nerve root compression need non steroidal anti inflammatory medications to heal?",
"src": "Patient: I have been diagnosed with L4 nerve root compression causing pain in my thigh and outer shin. I have been prescribed Tramadol for the severe pain. Is it important to take non steroidal anti inflammatory medications as well to speed the healing process? Doctor: No. Analgesics only give symptomatic relief by reducing pain. If neuropathic pain is troublesome then start tab pregabaline. Mention your MRI findings. If significant neuro deficit is present then go ahead with surgery.Hope my advice will help you. Take care. Don't forget to rate me."
},
{
"id": 42045,
"tgt": "How can male infertility be treated?",
"src": "Patient: I am an indian, working in Qatar for last 5 years. My marriage was three years before and for last 2 and half years we were trying for child.. and unfrotunately not yet. As per doctor.. my wife is ok.. only me needed treatment since my sperm count and motility is very low. my last sperm analysis as follows, Total Sperm count : 44 Total Ejaculate Sperm count: 88 Motility (1/2 Hr) Active 25 Sluggish 30 Non Motile 45 Motility (1 Hr) Active 20 Sluggish 30 Non Motile 50 Motility (2 Hrs) Active 15 Sluggish 25 Non Motile 60 RBC 6-7 Pus Cells 2-4. This was two months before.. and before this analysis i have done the same three more time and the result was poor.. Pls advice wht will be the best treatment for me.. Continue with medicine or... Doctor: Hi, Welcome to HealthcareMagic . Since you are taking treatment and the parameters are not responding that means you are not responding to the treatment. It would be good if you had written what treatment you are taking. I suggest you to discuss with your doctor regarding IUI. It is intrauterine ingestion of sperms . Here sperms are washed and processed and healthy sperms are placed in uterus . So though your count is less healthy sperms may be taken and also since they are placed in uterus motility problem also tackled. So your problem is bypassed and increase chances of pregnancy. You may try 5-6 cycles of IUI before proceeding to IVF(in vitro fertilization) .Hope I have answered your query. RegardsDr.Deepika Patil"
},
{
"id": 105414,
"tgt": "Cough, headache, dizziness, eye pain, nasal discharge. Due to allergies?",
"src": "Patient: my 7 yr old daughter has had a cough for over a week with no ther symptoms except runny nose . The doctor checked her out and said she was fine except the usual allergy stuff. she has no fever but now shes been feeling dizzy, has a bad headache , and eyes hurt all within several hours. this has to be more than allergies, correct? but no panic because no fever, correct? what could be going on? take her back to doctor? blood work? thank you in advance!!! Doctor: Your daughter is suffering from allergy and that may be cause of all her symptoms. She could even be having sinusitis which may be causing headache. You may give her montair lc kid and tyelenol to tackle her symptoms. If possible, make her do some steam inhalation."
},
{
"id": 15840,
"tgt": "Red rash under nose, chest tightness, swollen tonsils, white hair on tongue. Any ideas?",
"src": "Patient: Hi I have a red rash that is under my nose and is pretty bright red my other symptoms are chest tightness, my chest is burning, my tonsils are swollen, I have what it looks like white hair on my tongue when I brush good they go away but 5 mins later my tongue is as white as before the rash is also on my chin but not as bad as under my nose and when I wake up in the morning where the redness is under my nose is really dry and flaky Doctor: Hello.Rash under your nose,swelling tonsils,burning chest;ares signs of infection(upper airway infection/lower airway infection). You must go to your doctor,he will examine you and,if he deems it necessary,will request additional test to stablish correct diagnose and treatment.Now you can take paracetamol or ibuprofen to improve your symptoms. I wish you good health. (In case the answer would have been useful please indicate this)"
},
{
"id": 127921,
"tgt": "What causes intense burning sensation in the inner leg?",
"src": "Patient: Hi, Periodically and randomly I get a very intense, burning with sharp pain on my right inner leg in the crease of my leg and the outter part if my majors labia. The pain cons fast and will leave fast but it s enough to know the wind out of you. What could this be? Doctor: Hello,It sounds like a pain that is coming from nerve impingement in the low back. Try gentle stretches of the back and warm compresses. Also, some ibuprofen may help. If it gets worse or is not resolving, then see a doctor. If you develop any incontinence of urine, or stool go to emergency.Hope I have answered your query. Let me know if I can assist you further.Regards,\u00a0\u00a0Dr. Kathy Robinson"
},
{
"id": 69559,
"tgt": "What causes a hard painful lump on the skull?",
"src": "Patient: I awoke with a hard lump on the left side of my skull. Approximately 2 1/2 inches from my left ear, and right at the base of my skull.Very hard to the touch. Pain when touched is very sharp, like an electrical shock.The mass doesn't move or drift--also causing some irritating neck sensations.I ABSOLUTELY do not attribute this to a spider bite, or muscle pull of any kind.Has been 2 days now with no signs of any improvement. I'm not sick in any way, so an infected lymph node also seems doubtful, but I understand still a possibility.Wondering if a tumor can appear that fast?Any help or suggestions would be GREATLY appreciated. I've done as much research and diagnosing as I could. Next step obviously is med attention. Thx 4 taking time to read this Doctor: Hello!Thank you for the query.From your description you have an inflammation in this area. An abscess has probably appeared and as long as it wont be incised, the symptoms will be present. This is not a cancer for sure. It does not grow that fast and rarely grows in mentioned area.Most likely you have had there a sebaceus cyst which has gotten infected.Please consult general surgeon. Incision should be done.Hope this will help.Regards."
},
{
"id": 216477,
"tgt": "Suggest remedy for pain in ankle",
"src": "Patient: My daughter plays basketball. She has broken her foot, her back, and her arm in four places. Her bones are not real strong. She is six feet tall and 15 years old. She has been complaining about both her ankles hurting. Because of her past, you know what I am thinking..... When I press on her ankles they hurt. If they are sprained would it hurt more when I let go of the pressure? The funny thing is there is not much swelling! Doctor: Yeah, this has to be seen by an expert there are a lot of possibilities. They are all serious in a growing, tall 15 yr old. Even the most minor (muscle strain) needs expert couching on stretching, prevention, and how much to rest versus train. And, can be a stress fracture, a minor fracture, a fracture in the growing part of the foot/ankle that can cause deformity and probably other possibilities."
},
{
"id": 116133,
"tgt": "Can low WBC result in excessive sleep?",
"src": "Patient: Hi, may I answer your health queries right now ? Please type your query here...MY 13 YEAR OLD NIECE HAS HAD 4 BLOOD TESTS DONE THE FIRST ONE WAS BACK TODAY AND SHOWED SHE HAS A LOW WHITE BLOOD CELLS COUNT SHE HAS BEEN SLEEPING ALOT LATELY WHAT COULD THIS MEAN? Doctor: Hi,Thanks for asking.Based on your query, my opinion is as follows.1. No, low WBC cannot cause excessive sleep.2. Excessive sleep can be due to more fatigue secondary to anemia or hypothyroidism commonly.3. Check for hemoglobin and thyroid hormone levels.4. If normal, a sleep study maybe necessary to rule out sleep apnea syndrome.Hope it helps.Any further queries, happy to help again."
},
{
"id": 131863,
"tgt": "What is the treatment for swelling of joints?",
"src": "Patient: Hi Doctor, I m from Pakistan, My mother is suffering from high & low blood pressure, body ach and frequent swallowing on joints and other musculler parts of body. We are visisting doctors for last 8 years but no medicine is effecting andhelping to overcome her diseases, we are in a great worry that how to overcome the situation as her cholestrol level tests show that she is fine. Can you help me out to get her out of trouble? Regards, Adnan Doctor: Hi normally blood pressure does not cause any swelling in the joints and muscles. In my opinion your mother could be suffering from rheumatoid arthritis or deficiency of vitamin D. I suggest start with Vit D3 60 K once a week for 6-8 weeks along with calcium and if there is no improvement then go for R A Factor and Anti CCP antibody blood tests."
},
{
"id": 21157,
"tgt": "Suggest treatment for rapid heart beat while having emphysema",
"src": "Patient: Hi, may I answer your health queries right now ? Please type your query here...I have emphysema and I am having a rapid heart rate of 135 after walking a block. I am 55 yrs old and am fit. Should I see a doctor and is there some meds I should be taking? Doctor: hellothanks for posting here.rapid heart rate in emphysema is usually caused due to the inhaled bronchodilators you are using. but they are very essential for your emphysema. another group of medications commonly prescribed in emphysema can cause rapid heart rate. these are aminophylline geoup of medications (deriphylline, doxophylline, acebrophylline). you can try stop aminophylline group of medications for some 5 days. or alternatively you can start tab ivabrad or tab coralan 5mg two times a day.in addition to this i would advice you to take a cardiology evaluation. thank you"
},
{
"id": 124404,
"tgt": "How would one confirm for having Cellulitis?",
"src": "Patient: Hi,My father inlaw broke his forearm 2 months ago and had his cast removed after 6 weeks.He is now experiencing pain and swelling in the inner part of his elbow.He is unable to bend at the elbow,and he has redness.Is it possible that he could have a cellulitis? Doctor: Hi, Consult a general surgeon and evaluated. An ultrasound scan is required. A short course of antibiotics might be required preferably Amoxclav. Hope I have answered your query. Let me know if I can assist you further. Regards, Dr. Shinas Hussain, General & Family Physician"
},
{
"id": 197869,
"tgt": "What causes foul smell and cloudy discharge from penis?",
"src": "Patient: Hi my name is Chuck and for the past week my penis has been stinging and releasing a cloudy discharge, I haven't had sex in two weeks but recently I had oral,I googled it and it could be irritation,infection,caused from masturbation constantly,or stds I have no insurance and I need help please. Doctor: Dear, We understand your concernsI went through your details. The symptoms of cloudy urine with strong odor, discharge from penis, frequent urge to urinate and frequent urination indicates many physical problems such as Urinary tract infection (UTI), Diabetes, type 2, Vesicoureteral reflux etc.As there are many conditions associated with cloudy urine with strong odor, discharge from penis, frequent urge to urinate and frequent urination, I suggest you to consult an urologist without wasting time. Early treatment cures well and early. If you require more of my help in this aspect, please use this URL. http://goo.gl/aYW2pR. Make sure that you include every minute detail possible. Hope this answers your query. Further clarifications are welcome.Good luck. Take care."
},
{
"id": 91523,
"tgt": "What causes mild stinging in my lower abdomen?",
"src": "Patient: I have a very mild stinging in my lower abdomen, almost like being poked by a needle. I had sex about two weeks ago and i am late by seven days. I'm not ruling out pregnancy but would this be normal for menstrual pain? I am 23 and have never suffered from mebstrual cramps. My breasts are also very sensitive. Doctor: Hi ! Good morning. i am Dr Shareef answering your query.If I were your doctor, I would advise you for a serum HCG test to rule out pregnancy. In case of its persistence and your menstrual period getting more delayed, I would consider an ultrasound of abdomen. In the meantime, if you get your normal periods, then nothing to worry, as then you are sure that it was pre-menstrual pain.I hope this information would help you in discussing with your family physician/treating doctor in further management of your problem if it persits. Please do not hesitate to ask in case of any further doubts.Thanks for choosing health care magic to clear doubts on your health problems. Wishing you an early recovery. Dr Shareef."
},
{
"id": 148244,
"tgt": "What could be the reason for passing out, numbness in hands, weakness, lethargy?",
"src": "Patient: My wife fainted for 10-20 seconds in the middle of he exercise after she walked to me and told me to hold her which I did and he she past out. When she got out of her faint (syncope) she stated her hands are numb but went away after 3 minutes. It has been two days. And she still feels weak and very lathergic. Should I take her th ER and ave her examined/run some tests. She can talk and walk but she is always tired. BTW: she is los in her men s trial period. Doctor: Hi, Welcome to Health care magic forum. It appears that she is suffering with postural hypotension, may be due to anemia, pain some where like back ache, gastric irritation,or infection of the internal ear. I advise you to consult a physician for diagnosis and treatment. She may need to have MRI of the ear, besides other tests. Ask her to take green leafy vegetables, pulses, sprouts, and protein rich foods. When ever she feels giddy, she should lie down, where ever she is. Wishing for a quick and complete recovery. Thank you. Wishing for a quick and complete recovery. Thank you."
},
{
"id": 184558,
"tgt": "Can Moxikind CV cause eye swelling?",
"src": "Patient: after a tooth infection doctor prescribed Moxikind cv 625. After 3 doses I got severe swelling around both the eyes which has still not disappeared. Is that a side effect of this drug? My age is 65 yrs. Hieght is 179 cm and weight is 95 kg. I am otherwise in very good health. Doctor: Hi,Thanks for posting the query, I would like to ask what type of infection you where having and which is the involved tooth?Are you having bp problem ? have you discussed this with your Dentist?I would suggest you to get a re checkup done, stop the use of the medication for sometime, instead take tab oflox 200mg along with tab aceclofenac TDS.Get your bp checked.Take care!"
},
{
"id": 189693,
"tgt": "Root canal done, tooth pain, swollen face. Taking cifron. Cure for pain?",
"src": "Patient: sir actually my mom went through the treatment of root canaling some days before she is given medicines like cifron,etc..... but there is no improvement in her health....... she is suffering a severe pain in her tooth .......and one half of her face is sweeled.... sir i dont have account to pay u online...... is der any other option ?? Doctor: Dear friend. Thanks for sharing your concern. In root canal treatment ,once the tooth is cleaned properly(that is bio mechanically prepared) then there should be no pain. Therefore i would suggest you to revisit your dentist and get the canal preparation completed. once this is done she will have no pain,as the nerves and infected pulp will be removed during this process. In the same visit you can request your dentist for changing her antibiotic and analgesic,so that she can have faster relief ,but please remember that in dental treatment just antibiotic and analgesic wont help much. They are given to localize and minimize the infection,temporarily,where as treatment like root canal gives you complete cure for that infected tooth. meanwhile ask her to do frequent warm saline rinses. and maintain very good oral hygiene. Hope it helps. Thanks."
},
{
"id": 111099,
"tgt": "How to treat back pain with fever?",
"src": "Patient: Hi i am having fever from last 6 days and. Back pain also. I am taking medications but its not improving. I have done test for dengue snd malaria and the results are negative. Typhoid result still awaited . Please advice. Fever vsries from 100-103 farhnt Doctor: Hello,I had gone through the case and found that back pain might be due to weakness.So wait for Typhoid result.Meanwhile take tab Crocin and multivitamin A to Z twice a day.Keep hydrated and maintain hygiene.Hope my answer will be effective for you.Thanks"
},
{
"id": 197400,
"tgt": "Will stopping masturbation put you at a higher risk of prostate cancer?",
"src": "Patient: I am 17, I am 5 8`, weight around 150 lbs and had asthma when I was young but other than that nothing. I wanted to know \"Are wet dreams going to stay permanent in your life if you stop masturbating at a young age, or is semen going to be stopped being produced at a certain point?\" 2. I heard that if you stop masturbating you have a higher risk of Prostate cancer, is that true or false? Because if I don't want to masturbate then why do I have to? Doctor: Hi thanks for contacting HCM...Mastirbation pleasure giving procedure alternate to sex....But overmasturbation avoided....If you don't masturbate then it is not like semen not produce ....Its secretion is not affected by masturbation...And prostate cancer not related to whether masturbation done or not ...Avoid smoking and alcohol like bad habit ..Avoid stress.Nothing actively done in your case.Don't worry...Take care"
},
{
"id": 33601,
"tgt": "What went wrong with my voice after long term cold?",
"src": "Patient: I am a singer and I had a very long-term cold last year. My singing voice don't seem to heal though I have been healthy for many months now. I went to a doctor and they did not see anything at my vocal cords.. But still I cant sing like i use to. My voice can crack anytime. What should I do? Should I train my voice? Or be queit for some days? Thanks Doctor: HI, thanks for using healthcare magicIt may be best to limit the use of your voice for a few days.You may also want to consider visiting an ENT specialist for assessment and treatment if any issues are found.I hope this helps"
},
{
"id": 44131,
"tgt": "Trying to conceive, abdominal myomectomy. Taking humog, fertigyn injections, hemorrhagic cyst. Treatment?",
"src": "Patient: Hi Doc, I am trying to concieve since 6 months. I recently underwent an abdominal myomectomy -about 3 months ago. This cycle my doc advised me to go for an HSG and follicle study. The tubes are patent. I was put on progynova, Humog ( 3 injections) and 1 shot of Fertigyn . Now on day 20 the maturing follicle has turned in to a hemmorhagic cyst. How may I proceed doc. Kindly advice? Doctor: hello thank you for writing in to HCM usually a hemorraghic cyst will resolve in 1 or 2 cycles It will be advisable not to stimulate the ovary with medication for a short while. rescan after a couple months and then resume treatment hope to have answered your query Dr Nandita Thakkar"
},
{
"id": 113597,
"tgt": "Lower back pain, spreading to stomach, unable to walk, severe pain. Causes for symptoms?",
"src": "Patient: for over a year iv been getting back pain which started lower back pain. since then the pain has spread to shooting pains to my stomach . now im getting pains on the right side of my back shooting through to my stomach. im in so much pain cant even walk for 2 minutes without beeing in cripling pain. please tell me whats wrong with me???? Doctor: Back pain radiating to stomach could be due to issues of the abdominal organs. I would suggest to get a USG abdomen. Rarely a thoracic disc herniation can cause similar problem. But before exploring that I would rule out any tummy problem. Thanks."
},
{
"id": 73500,
"tgt": "What do hurting ribs radiating to back and shoulder after being treated for pneumonia mean?",
"src": "Patient: Hi, may I answer your health queries right now ? Please type your query here...I have been to the doctor 2 times. First time, it was a severe sinus infection, week later had to go back, turned into pneumonia, had to go back third time. All 3 times I was given a shot, antibotics & c-pack first time, steroids, antibotics & shot second and antibotics & shot third. I have been hurting under my ribs and has gone to my back and shoulder. Do you think the pneumonia has gotten worse? Do I need to go back to the doctor? Doctor: Hello dearWarm welcome to Healthcaremagic.comI have evaluated your concern in depth .* This indicates lung pathology giving nerve endings pain with hurting ribs radiating to back and shoulder .* If not carried out x-ray chest with or without CT thorax is a must for proper diagnosis and management guidelines .Hope this clears your query .Regards ."
},
{
"id": 111691,
"tgt": "Is the back and hip pain normal with spina bifida occult?",
"src": "Patient: Hi, my names Aimee, I was diagnosed with fibermyalga, now I read a recent x-ray and it says spina bifida occult a s1. I kinda knew that I didn't have fibermyalga Because the pain is always in my back and hips. Is bad back and hip pain normal with sbo?, and learning disabilities? And mood disorders? Is all that normal with sbo? Doctor: Hello and welcome to HCM,Spinal bifida is a congenital condition in which the posterior parts of some of the vertebrae do not close completely.There are various types of spina bifida depending on the degree of non-closure of the vertebrae. When the defect is large enough the spinal cord can protrude from the gap.Spina bifida occulta is a condition in which the gap is very small and it does not allow the spinal cord to protrude.It is asymptomatic in majority of cases however, it can be associated with back pain not responding to conventional treatment for back ache.Surgical closure of the defect is possible however, the functional abnormalities (if present) cannot be corrected.However, functional abnormalities are rare with spina bifida occulta.Thanks and take careDr Shailja P Wahal"
},
{
"id": 29541,
"tgt": "What causes persistent fever along with ear infection and sore throat?",
"src": "Patient: i have a five year old boy, who is on day four of fever between 99.7 to 104.00 pink eye symptoms, ear infection, sour throat. he has also had loose stools for two days, he also has headaches. i have taken him to Dr twice . He is on amoxicillin 400 mg/5 ml s he is taking 9.5 ml twice daily for ten days. He is also taking Tobramycin 0.3% in eyes one drop every 4 hours. also he is taking ibuprofen 7.5 ml every 4 hours , he has been on these meds for four days why is he not better? Doctor: sore throat, conjunctivitis, fever with headache, and according to the age of patient three diagnosis can be possible, measles , malaria or simple upper respiratory tract infection. 1. check for any rash , if present it indicates measles2. do blood test thick slide test for malariaeven after 4 days of antibiotic therapy symptoms are not subsiding, I recommend above mentioned tests."
},
{
"id": 205721,
"tgt": "Suggest treatment for insomnia and depression",
"src": "Patient: Hi I have had insomnia for about 18 years now, no I am not depressed, yes I have tried melatonin and I think all other supplements to try and find a good solution. If you could help me with a good solution for this you would then become my new best friend. Doctor: DearWe understand your concernsI went through your details. Insomnia definitely will be having an underlying basic factor which needed to be identified. You did not provide many details about the factors and other investigations carried out by previous doctors. You should provide those information. Secondly, provide previous treatment options. Provide emphasis on psychological aspect too. Psychologic origin insomnia is totally curable.If you require more of my help in this aspect, please use this URL. http://goo.gl/aYW2pR. Make sure that you include every minute details possible. Hope this answers your query. Available for further clarifications.Good luck."
},
{
"id": 26048,
"tgt": "Is 112 heart rate in a mitral valve regurgitation patient abnormal?",
"src": "Patient: I have been sitting quietly for about an hour and about 30 minutes ago I felt my heart racing. I checked it and for the last 15 minutes its been 112 beats per minute. I have been having these episodes for the last two weeks . I was diagnosed several years ago with mitral valve regurgitation. I am 44 years old and a female. Should I be concerned? Doctor: Hello!Welcome and thank you for asking on HCM!Regarding your concern, I would explain that your symptoms could be related to a cardiac arrhythmia. I recommend consulting with your attending physician for a careful physical examination and some tests: -resting ECG and cardiac ultrasound to examine your cardiac function-chest X ray study-blood lab tests (complete blood count, thyroid hormone levels, fasting glucose, PCR, sedimentation rate, blood electrolytes). An ambulatory 24-48 hours ECG monitoring would be needed to examine your heart rhythm trends during the day. Hope to have been of help!Feel free to ask any other questions whenever you need!Best wishes!Dr. Iliri"
},
{
"id": 23043,
"tgt": "Is it safe to take Losartan for hypertension?",
"src": "Patient: Hello I am 76 kg 56 yrs old professor working in a university. I am taking losartan 50 daily for hypertension daily in the morning. Recently i had developed some kind of discomfort or say pain like situation in upper chest or right or left shoulder . But I continue to go to office . Then I consulted my physician he prescribed me Aztor Asp75 in the night and Dolo 650 in Morning and Evening alongwith .25 Alprazolam. I am feeling slightly improved but quite weak in the evening. My lipid profile is colesterol 152,Triglyceride 186 HDL 34, LDL 80 and VLDL 37.32 in May 2011. I am worried. It is affecting my work. I have not taken any rest and keep going to office. Please suggest me in what way I should go. Regards Prof P.D.Juyal Ludhiana Doctor: DEAR USER,THANKS FOR CONSULTING WITH HCMI UNDERSTAND YOUR CONCERN.. YES LOSARTAN IS SAFE FOR HYPERTENSIONSEEING YOUR TREATMENT, YOUR PHYSICIAN HAS STARTED TREATMENT SUSPECTING YOU TO HAVE ANGINAI THINK YOU NEED TO CONSULT A CARDIOLOGIST URGENTLE.. GET A ECG AND IF POSSIBLE A ECHOCARDIOGRAPHY DONEI DID NOT UNDERSTAND THE POINT WHY YOU WERE STARTED ON DOLO 650I SUGGEST YOU TALK TO YOUR PHYSICIAN WHY YOU WERE STARTED ON ITYOU NEED TO DESCRIBE YOUR SYMPTOM IF YOUR SUFFERING FROM DIFFICULTY IN BREATHING, BREATHLESSNESS, ANY SWELLING IN LEGS, ANY HISTORY OF CHEST PAIN.. SO THAT I CAN HELP YOU IN A MORE DETAILED WAYHOPE I ANSWERED YOUR QUERY. YOU CAN MESSAGE ME FOR ANY FURTHER CONCERNS"
},
{
"id": 14229,
"tgt": "Why am I suffering from rashes and pimples in underarms?",
"src": "Patient: I have a rash on my inner arms and inner foreaarm that look similar to pimples but with no head... They appear randomly and usually dont itsch.. However I was recently on vacation in Arizona and noticed they came back and got worse on inner arm and upper ribcage and started to itch after I had been in a hot tub. Doctor: hi there.1. by your description, your concern may be pityrosporum folliculitis.2.it's due to candidial yeast infection of hair follicles.3. you can try lulifin lotion once daily bed time for 30 consecutive days along with any clotrimazole dusting powder twice daily as maintenance.4. avoid hot and humid conditions 5. have 2-3 litres of water daily.if there is no response within couple of weeks, then have a consultation with a dermatologist near by you for physical examination.this will help you."
},
{
"id": 94535,
"tgt": "Child extremely lethargic, sleepy, stomach pain. What can I do?",
"src": "Patient: My almost 2 year old barfed yesterday around 3 p.m. and she was very tired around 830 p.m. went to sleep for an hour and a half, woke up, went back to sleep around 11ish woke up at 7 went back to sleep woke up at 930 and has been extremely lethargic, got up for some fruit and milk then laid back down and has been just laying around. It doesn t feel like she has a fever , but she occasionally has been saying ouch and pointing to her tummy. Doctor: Hi, This may be due to constipation if didnot passed stool for 01 day or 02 days or is passing hard stools the stool impaction is causing this problem. May have acute gastroenteritis if having symptoms of diarrhea and vommiting consult your physician for appropriate antibiotics and antiemetics. I hope I am sucessful in solving your query If u have further you can ask me Take care Regards, Dr. Azhar Sattar"
},
{
"id": 68157,
"tgt": "What causes lumps all over the body?",
"src": "Patient: I had a baby 8 months ago and have lost all my bay weight plus an extra 15. Now I am noticing all sorts of lumps in varying sizes around my body, by my c sec scar, around my upper biceps and a fairly large limp similar to a marble aim my lower back. I am 5'8\" and 135 lbs. Any advice? Doctor: Hi, dearI have gone through your question. I can understand your concern. You may have some tumour pathology. You may have multiple neurofibromatosis or lipomatosis or some other lesion. You should go for fine needle aspiration cytology or biopsy of one of the lump.Then you should take treatment accordingly. Hope I have answered your question, if you have doubt then I will be happy to answer. Thanks for using health care magic. Wish you a very good health."
},
{
"id": 108764,
"tgt": "What causes severe upper back pain and burping?",
"src": "Patient: I had a sudden severe upper back pain (between the shoulder blades) that felt like a stabbing pain. After a few minutes, I burped and the pain was gone. Was this indigestion? I also have a herniated disc mid back, although i was sitting at the time. I am worried it could be my heart. Doctor: hellowelcome to HCMthere are many causes of thee pain in the upper back,related to stomach,heart,lung,musculoskeletal,neurological.but from your history it seems to be indigestion.there is gas formation and distension of gut that causes pressure over the diaphragm and causes pain in shoulder.you can avoid hot spicy food,take small frequent feed.take ppi tab od.pancreatic enzyme syrup.also do an USG whole abdomen because it is associated with gb stone sometimes.consult your physician if recurrence.hope this is helpful.regards."
},
{
"id": 3736,
"tgt": "Is pregnancy the reason for delayed periods?",
"src": "Patient: Hi, my name is Imari. Me and my husband have been trying to get pregnant for 3 months now, but haven t had any luck. Actually it s been frustrating. My monthly was suppose to come on but it hasn t at all, and today is the last day of the month. I have been cramping for almost a week now, but no signs of it coming on. Iv e even been having clear discharge. I took a pregnancy test a few days ago, but it came out to be negative. So what should I do? Is it a chance I could still be pregnant? Or may it be to early? Doctor: Hello, thanks for writing in.Your pregnancy test is reliable if you have done the test at least 5-6 days after your missed period date, following proper instructions on the kit.But if its negative/ doubtful and your periods have been irregular, I would ask for an ultrasound scan.There are many causes of delayed/irregular periods- polycystic ovarian disease, thyroid abnormality etc. If pregnancy is ruled out, I would suggest you consult your doctor and discuss this.Do not worry, even stress or sudden weight gain/loss can delay periods. If you have tested early, wait for a few days and do it again.Hope this has answered your queries. Write in if you have any doubts.All the best."
},
{
"id": 34529,
"tgt": "How to treat infected wound?",
"src": "Patient: I was running and my rolled, my right knee and elbow caught me and got pretty scraped up. I went to the doctor for my ankle, and they \"cleaned\" out my wounds. This happened Monday, and I still don't have scabs on either wound, they are just covered in a few layers of yellowy pus like goo and I got it mostly off my knee, but can't get it off my elbow...what can I do? I can tell they are infected, they are causing redness around the wounds... I have been putting neosporin on them and using peroxide, but I can't get the goo off my elbow! Help!!! Doctor: Hello,For an infected wound if the local antibiotics are not helping its better to use systemic antibiotics like Amoxyclav 625MCG twice daily for 3 days and can extend it by two more days if there is no response.Take a multivitamin once daily for better healing.However continue cleaning the wound with normal saline.Hope my advise will help you.Do no hesitate to ask any further questions."
},
{
"id": 187146,
"tgt": "Experiencing nasal drainage, pressure in right sinus , under eye and allergies",
"src": "Patient: I had a root canal done today and he said that he hit the lingual (sp) nerve that caused a sharp pain. I started sneezing soon after that. Now I have severe nasal drainage, pressure in my right sinus and pressure under my right eye. I have seasonal allergies and this almost feels the same but just on that right side. I am 29 years old, 5' 2\". I have had many dental injections and this has never happened before. Doctor: Hello, Thank you for consulting with HCM.It looks that while performing root canal treatment on the tooth the dentist has punctured the wall of maxillary sinus and now it is causing drainage.Better you visit an oral surgeon and get a complete examination done. and if necessary get the tooth removed.Hope it will help you."
},
{
"id": 143307,
"tgt": "What does Hyperintense foci and flair in MRI brain plain mean?",
"src": "Patient: On the advise of my doctor I got a MRI (Brain Plain) done. The result shows 'Hyperintense foci on T2W and FLAIR in bifrontal deep medullary white matter'. I am 62 and am on asprin, nitrates and medicine for BP for last 18 years. What does the finding mean? and what more I should do to treat this? Dalip Mehra Doctor: I read your question carefully and understand your concern. 'Hyperintense foci on T2W and FLAIR in bifrontal deep medullary white matter' is most likely referring to microangiopathic changes. These occur in every patient and get more with each year of life. They are harmless and you should not worry about it. Hope you will find this answer helpful!Kind regards,Dr. Wu"
},
{
"id": 109006,
"tgt": "What causes lower back pain, pain and blood while urinating?",
"src": "Patient: Hello, I have a pain in my lower back, and when i urinate it really hurts and is becoming more frequent. There is sometimes blood when i urinate, and my vagina is irritating very much inside. i'v been to the doctors many times with the same symptoms as they are reoccurring and they just say its a urine infection, they give me antibiotics which don't get rid of whatever it is. Could you help me please? Doctor: Hi,From history it seems that there might be having some problem like stones in kidneys, chronic urinary tract infection.Go for ultra sound of kidneys.Consult urologist and get examined and investigated.After all reports go for treatment accordingly.Take plenty of water.Ok and take care."
},
{
"id": 196552,
"tgt": "What causes microscopic blood in urine ?",
"src": "Patient: Hi, I am 28. For the past month I have had frequent urination. The doctors told me that I have microscopic blood in my urine. They said that there was no bacteria. I have felt no pain. My urination increased the day that they told me that I had blood in my urine. I also have felt that my penis retracts to a smaller size. Doctor: Welcome to healthcaremagic.com and myself Dr. Atishay Bukharia would be answering your questions.. microscopic hematuria can be because of trauma to urethra or some kind of kidney problem so I would suggest you to go for ultrasonography of abdomen in order to rule out any kidney Pathology.. Hope it helpsIf you have any other question please do not hesitate to contact usRegards."
},
{
"id": 6298,
"tgt": "Endometriosis, operated for ovarian cyst, trying to conceive, in vitro fertilisation",
"src": "Patient: Hi dr.This is Sridevi.I am 28years old.married since 2.11 yrs.trying to get pregnant . endometriosis surgery was done on 15th aug 2011.after that i was in decapeptyl for 3 month.i got my period on jan 2012.then diagnosis with one ovarian cyst .got operated.then every organ tested incluuding ovaries,tubes and uterus all are working fing.m ovulating regularly(got to know from follicular study)my husbands simen report also good.trying 2months (feb,march) with unsucessfull result..my dr suggest me to go for ivf.can u suggest me which way i should go..naturally i will try or i will go for ivf..thanks Doctor: hi sridevi, thanks for the query.please do not loose heart.since all reports are fine,you should conceive.Pregnancy itself will improve endometriosis.your ovulation period will be exactly 14 days before your next period.Chances of conceiving in one cycle for a normal couple is 8-10%.So it will take few more months.Try for IUI(intra uterine insemination) after 3-4 months,where your husbands semen is processed and inserted directly inside your uterus.IUI should be tried for 3-4 cycles.any eexcessive vaginal discharge,or foul smelling discharge?Infections should be treated for both husband and wife"
},
{
"id": 63894,
"tgt": "How should a mons pubis lump be treated?",
"src": "Patient: hi,i have a pea sized lump under the skin of my pubic region from lasr two months,i'm worried like hell,consulted my gynaec but she said,itz a fat lobule...itz not increasing in size and iz not at all painful...plz tell me what r the possibilities..:(i forgot to mention that i'm 32 f.and am hypertensive on ,medicine...plz tell me hw can i get the answer!?? Doctor: Hi,Dear,Good Morning and Thanks for your query to my virtual Clinic At HCM.I studied your query in depth and understood your concerns .Causes for lump in pubic region as follows-Mostly they are -Lipoma ? or could be Fatty globule-as its painless.Treatment-Check with ER Surgeon and check your fbs and pps,and plan for Excision Biopsy of the lump to resolve the worry.Don't worry of them,as they could be relieved with proper treatment.Hope above reply would help you to relieve of the issue you have.Welcome for any further query to ME and HCM.Wishing you a Fast Recovery.Write Good review for the benefit of my new patients.Click thanks ASAP after this.Have A Good Day.With RegardsDr.Savaskar"
},
{
"id": 124755,
"tgt": "Is deformed hand in a child with wrist and thumb bone with no other nails and bones a physical disability?",
"src": "Patient: Hi my baby girl was born with a deformed left hand she has a wrist and the bottom bone in her thumb the rest are stubs with nails & no bones. She is 4 weeks old what should i expect also is this classed as a physical disability. We are really worried x thanks carol Doctor: Hello, Unfortunately, she is having some form of congenital deformity and in lead to some sort of disability. You can seek help from a physiatrist to overcome the disability by means of exercises. Hope I have answered your query. Let me know if I can assist you further. Take care Regards, Dr Shinas Hussain, General & Family Physician"
},
{
"id": 21355,
"tgt": "Should I be worried about my blood pressure being 143/72?",
"src": "Patient: i am about to turn 40 years old in June. I am male 200lbs and 5'7\" with a muscular build. I am currently taking Adderal for ADD and was recently prescribed .25mg of Alprazolam to take as needed, I just used a home blood pressure test and it is 143/72 and pulse is 93. I am just feeling stressed and believe it is simply anxiety, Should I be concerned or just take a short break to calm my nerves down? Doctor: Hello, I understand your worry, but there is no need for that.Adderal elevates blood pressure, this is one of its side effects. Although your blood pressure is slightly increased it isn't much of a worry. I advise you to continue medication and your normal exercise, but if your blood pressure continues to increase please consult your attending physician to review your medication to see if it should be replaced or dosage needs to be modified. Warm regards"
},
{
"id": 128874,
"tgt": "How to treat fibromyalgia?",
"src": "Patient: Have severe fibromyalgia symptoms... I know it causes weight gain in which ive struggled with my whole life. Need info on prescribed weight loss pills to assist in reducing my pain. I fired my doctor as she refused to help me with weight loss medication... Doctor: Hi. I understanded your question and I will help YouThe best treatment for fibromyalgia is to reduce physical and mental stress and I suggest Indomethacin 50 mg BID... Metformin 500 mg twice daily can help you to lose weight."
},
{
"id": 60406,
"tgt": "I am suffering from Hepatitis B. Suggest me the treatment",
"src": "Patient: Dear Doctor , I am suffering from Hepatitis B , what is the treatment for Hepatitis B and I want to come and visit you in Mumbai if you can send me your E-mail address and your personnel address in Mumbai My E-mail is Anwaraap@gmail.com I will appreciate your valuable opinion as I am in critical conduction Thanks & Regards Doctor: Hello.welcome.Treatment is supportive and specific.In supportive you have to avoid alcohol.In specific treatment interferon and antivirals are given.Consult a gastoenterologist for management and treatment.good luck."
},
{
"id": 134043,
"tgt": "Will the gap after cyst removal in my buttock fill in?",
"src": "Patient: I had a cyst in my left butt cheek the size of maybe an tennis ball or bigger. I had surgery two years ago in which the cyst was removed but also a huge chuck of my butt muscle in which has left a pretty big size dent.. The doctor that performed the surgery told me it would fill in in time it has already been two years and I really feel like it s not going to fill in at least not to the normal size it once was so my question is.. Is it realistic to think it will never fill back in or will it fill in and how long would it take Doctor: hi,thank you for providing the brief history of you. As mentioned by you about the cyst and the surgery you have undergone. The surgery was performed to help ease of the cyst. Now since the space you are feeling empty there is not filled than you need to perfrom some exercises to strengthen the muscles and eliminate the space. Usually due to the cyst the muscle is overstretched and than after removal there is empty space which can be filled by exercises. Also , to not allow that space to be occupied by anything else in future you need to have a good tone of the muscles. I am sure you will be helped and your complains will be eliminated by toning of muscles.RegardsJay Indravadan Patel"
},
{
"id": 218985,
"tgt": "What causes nausea, breast heaviness and cramps despite having negative UPT?",
"src": "Patient: My basal body temp. is 99.9, the past two weeks I wake up soaked in between my thighs and lower back. I am on day 46 today of my cycle and still no period. Today I got a negative home preg. test but Im not convinced. I feel like my period is coming but it's felt like that the past three weeks. I am over emotional, nausea, tender fuller breasts, bloated comes and goes but for the most part always feeling cramps and tight uterus. I feel starving even still after I eat, major exhaustion more than like before your period. Hot flashes like crazy and my face hasnt broken out yet which hasnt happened in the last year ever. Feeling of UTI sometimes too and I have had this head cold for a month now, so much snot and mucus still not all the way gone yet. My cervix is still high, soft, and always extra wet lately. I am 21 and not convinced that I am not pregnant right now? please help Doctor: Hi, I think you should go for a blood test for pregnancy and a ultrasound also. It will clear your doubt. If you are not pregnant, you can take some medicines for inducing periods. Consult a gynecologist. Hope I have answered your question. Regards Dr khushboo"
},
{
"id": 129477,
"tgt": "What causes intermittent shoulder pain?",
"src": "Patient: I have a shoulder pain that is extremely intermittent, I cannot repeat the pain on demand, but it feels like a tiny shooting right shoulder pain not too far from my rotator cuff, deltoid muscle, etc......what do you think it might be and what may have caused it? Doctor: It may be an acute subacromial bursitis caused from inflammation of bursa with or without supraspinatus tendonitis."
},
{
"id": 46258,
"tgt": "Suggest remedy for \"mild to moderate pelvicalyceal system dilation\"",
"src": "Patient: sir, iam 32 years healthy male, diagnosis shows mild to moderate pelvicalyceal system dilatation in left . 9.4 mm at the pelvic ureteric calculus. ( left pelviureteric junction calculus causing hydroneprhrosis (grade II - III) . sir what is the remedy. kindly guide me severe pain at intervals Doctor: Good day and thank you for being with healthcare magic!!!You need procedure for stone removal either endoscopic or shockwave lithotripsy. If the stone persists then you will continue to have pain and dilatation in your kidneys May lead to kidney damage. I hope I have answered your question satisfactorily and please consider a 5 Star rating."
},
{
"id": 112539,
"tgt": "Have back and pelvic area pain. Had lumbar fracture, stable fracture. How long for healing process?",
"src": "Patient: Hi My name is Jane, I had a lumbar fracture 2wks ago, ER doctor said it is a stable fracture. I have tried to be in bed as much as possible.But I can still walk around in my house with mild to moderate pain on my low back and pelvic area.ER doctor told me to walk around every hour as long as the pain is tolerable. I'm worried if I am moving around too much and the healing process is going to be prolonged. Can you give me clear definition of immobilization to promote healing? I am taking Morphine5mg and Ketorolac 1tab once daily. Thank you! Doctor: Hi, thanks for writing to HCM.A stable lumbar vertebral fracture may heal uneventfully within 2-3 months with adequate rest.A Lumbosacral corset is usually sufficient to provide support to the injured area.Absolute bed rest is not necessary after initial few days. You should always sit with a straight back and avoid bending forwards as this may cause the fracture to collapse further.Regular use of calcium supplements is mandatory till the fracture consolidates.Hope this information is helpful. Good day"
},
{
"id": 46767,
"tgt": "What is the life expectancy with high creatinine level and low functioning kidney?",
"src": "Patient: Hi my father has a creatinine reading over 227 - not sure what this means except that it is way too high and his kidneys are working at 24% - he is 76 and the hospital will not do dialysis as his heart is failing too - what does this mean now for my father - what is his life expectancy and what can I expect to happen Doctor: At the age of 76 with 24% kidney function dialysis is not required. When kidney function decrease to 10% or low then he needs dialysis. At the age of 76,before starting dialysis many factors shoukd be considered like burden on care givers his cardiac status etc. With 24% function I would expect at least 2 more years he can survive without dialysis. The average life span on dialysis at this age is 1-1.5 yrears and if you still willing to go for at when it is necessary peritoneal dialysis is good option. But maintain on low protein diet and avoid potassium rich foods. All the best. if you have any questions feel free to contact me."
},
{
"id": 77965,
"tgt": "What causes pressure and pain at the center of the chest?",
"src": "Patient: Yesterday I took one advil with some filtered water when I felt this incredible pressure in the middle of my chest. It felt tight and I couldn't stand up straight because it hurt so bad. Then after five minutes it subsided. I could barely catch my breath. About a couple of hours later, I had mashed potatoes with cheese with my lunch and because the potatoes were a little warmer than usual when I swallowed, I felt the heat of the potatoes become amplified as it settled in what felt like a hole just below my breastbone in my chest. The spot or hole made the potatoes feel as if they were hotter than they were before I ingested them as if they were on fire. I never had the heat from food cause this before, usually I will get a severe chest freeze when I consume anything cold like ice cream or ice water that causes the pressure thing where I am doubled over with excrutiating pain but this is the first time with something that was heated. What is going on? It actually feels as if I have a hole somewhere in the middle of my chest just above the zyphoid bone. Autumn from Spokane Doctor: Hi. I can understand your concern. Ideally ecg and 2d echo should be done first in your case. If both are normal than no need to worry for heart diseases. GERD (gastroesophageal reflux disease) can cause similar kind of chest pain. GERD is due to laxity of gastroesophageal sphincter. Because of this the acid of the stomach tends to come up in the esophagus and cause central chest pain and nausea. You can take proton pump inhibitors. But along with drugs you need to follow certain below mentioned lifestyle modifications for better symptomatic relief. Avoid stress and tension. Avoid hot and spicy food. Avoid junk food. Avoid large meals, instead take frequent small meals. Quit smoking and alcohol if you have these habits. Go for walk after meals. Keep 2-3 pillows under head in the bed to prevent reflux. Loose weight if you are obeseDon't worry, you will be alright. Hope I have solved your query. Wish you good health. Thanks."
},
{
"id": 1286,
"tgt": "Can tab siphene and metformin with fol 123 help in conceiving?",
"src": "Patient: hi, i am 26yrs old and known pcos for past 4 yrs. i am now planning for a baby. My doctor put me on tab.siphene 150mg (twice daily) + metformin sr 500 mg (twicw daily)+ fol 123 (noon time). I want to know does this treatment work for me..?? and does my periods be regular for 28 or 30 days.. Doctor: Hi, I think you should consult your doctor regarding the dose of siphene. I think 100 mg once a day will be fine for you. 150 mg 2 times a day is a bit higher. After taking it, track your follicles growth by repeated ultrasound and when your follicles is more than 17 to 18 mm, take injection for rupturing the follicles. Be in contact with your husband for 2 to 3 days. Take progesterone for next 2 weeks. Do a urine pregnancy test at home after that. You can try like that for 3 to 6 months. Metformin also helps in increasing the chance to get pregnant. Hope I have answered your question. Regards Dr khushboo"
},
{
"id": 134078,
"tgt": "What causes edema?",
"src": "Patient: My father, age 90, has severe edema of his left foot. This condition has been ongoing for the last three weeks. At times, the swelling decreases. He says he has no pain. However, his usual response to any question regarding his health is that he is fine. Doctor: hi,thank you for providing the brief history of your father.As you mentioned the edema in ankles and no pain, it is assumed to me that the cardiac rehabilitation is needed. Since his age is 90, what happens in this age is the slowing of the venous return due to lack of sufficient mobility. For reducing the swelling, what can be helped in this age is to perform ankle toe movements to Improve the venous return. As also, exercises are highly recommended in geriatrics because what has to be understand here is the slowing for the metabolism and this leads to abnormal reactions in the body. Exercise, mild also will help improve the metabolism and cardio respiratory endurance. which will help in future to avoid small issues like this.With the grace of God I wish your dad a long healthy life.RegardsJay Indravadan Patel"
},
{
"id": 121204,
"tgt": "How will arthritis affect in future?",
"src": "Patient: I am 19 year old healthy young man but I ve always had a achey back. When I went to the doctor I talked to him about this I got Xray s done and he got back to me 2 days later saying that I have Arthritis in my back. How will this effect me later on in my life considering how young I am now. Doctor: Hello,I read carefully your query and understand your concern. Arthritis is a chronic disease which can be maintained under control through medication and physical activity.If you take care of yourself and do regular check ups it should not affect your future. Hope my answer was helpful.If you have further queries feel free to contact me again.Kind regards! Dr.Dorina Gurabardhi General &Family Physician"
},
{
"id": 149886,
"tgt": "Diagnosed with Ankylosing spondylitis. Started Saaz, homeopathy, ayurvedic medication. Side effects?",
"src": "Patient: diagonsed with AS in year 2006.Started saaz 500 took for 2 or 3 years then discontinued and tried altenative homeopathy and ayurvedic but then from year 2010 iritis started .It occured almost every year and in year 2013 its degree was high .Hence Doc recomeneded flitrax and iron and sazo together .What do u suggest ?does this medicines have side effects also? Doctor: Thanks for using HCM.Sazo is a medication to treat your disease. every medication do have some side effects, its necessary to treat condition. fwe side effects are Stomach upset, nausea, vomiting, loss of appetite, mouth sores, headache, dizziness, or unusual tiredness may occur. your skin and urine to turn orange-yellow.For your AS few suggestions are:A program of daily exercise helps reduce stiffness, strengthen the muscles around the joints and prevent or minimize the risk of disability. Deep breathing exercises may help keep the chest cage flexible. Swimming is an excellent form of exercise .sleep on a firm mattress with the back straight. Placing large pillows under the head is not preferred ,Hope i have answered your query, have a good health.RegardsDr. Vidya"
},
{
"id": 89994,
"tgt": "Suggest medication to stop recurring stomach pain",
"src": "Patient: hi,my husband age 32,height ,5 feet 10 inches and medical history includes eye retina problem with a catarct opertion is having stomach ache which is arising in regular intervals and then settles down.pls suggest some immediate drug which can be purchased from walmart over the counter Doctor: Hi ! Good morning. I am Dr Shareef answering your query.For the present, you could give him an anti spasmodic, a proton pump inhibitor and a probiotic drug as an immediate measure. However, I would advise you to get him assessed clinically by your family physician for a proper diagnosis, and appropriate treatment.I hope this information would help you in discussing with your family physician/treating doctor in further management of your problem. Please do not hesitate to ask in case of any further doubts.Thanks for choosing health care magic to clear doubts on your health problems. Wishing you an early recovery. Dr Shareef."
},
{
"id": 203095,
"tgt": "How many months should fertisure tablets be taken to increase motility?",
"src": "Patient: Dr.My husband age 44, i am 29 .We have a second marriage. But I have a child. My second marriage is done in one year. But we have no child.We meet one doctor, and prescribed fertisure medicine. my husband,s motilility is very low. Dr. how many months this medicine is used, which time this medicine is effected. Doctor: Hello,I have gone through your query and understand your concern.Fertisure M is a supplements in the form of anti oxidants which would help him to increase sperm motility.You should rule out any other cause causing infertility like infection. If there is any sign of infection, he may need broad spectrum antibiotics.Further we cannot surely how long he have to take fertisure M. Because this may works in many and not in some and the effectiveness vary from person and person and also on other factors like age, previous infections, etc.Recommend him to take natural foods to increase the sperm count like garlic, ginseng, pumpkin seeds, walnuts, asparagus, bananas,, dark chocolate, oysters etc. Further if he is a smoker, he should stop smoking and taking alcohol too.My recommendation is to recheck the sperm analysis after 3 months of taking this Fertisure M to know about the status and discuss with your treating physician.Hope this helps.Please write back for further queries.Wishing you good health."
},
{
"id": 43533,
"tgt": "Trying to conceive, have PCOS. Done laparoscopy. Taking Metformin. Should IVF be done?",
"src": "Patient: Hi,Im 27 years old and my hubby is 30years.... we r trying to conceive from 4 years but not successed yet.... my husband is fine... i was diagonosed PCOS in 2011.. my gyn suggest me go through for laproscopy or drilling.. in april 2011 my laproscopy done.. and then doc gave me clomid CD2 to 6... and also metformin 500mg twice a day... but no results...then i changed my consultant last month....march 2013...he gave me Femara 2.5mg from day 2 to day 6th of my cycle.... and menogon injections from day 7..8..9..and 10th and advise to intercourse for 5 days regular... but still no result...now im asking should i continue this natural process or go through for IVF? as my age concern? plz give me answer im very worried and hopeless now... bcz my husband is seamen and he is on vactions now thats why im in bit hurry.Mrs Shah Doctor: Hello Mrs Shah, I can understand your worries and concern. Reading your history I understand that you have taken ovulation induction drugs that is clomid and famara just for two cycles if I am not wrong? According to the protocol what we follow you need to undergo ovulation induction cycles atleast for 5-6 times before we switch over to other mode of treatment, but anyways that is absolutely subjecive and can be changed according to the patient profile and needs. The next level would be to go for IUI that is intra uterine insemenation, rather than directly jumping over t IVF. So I would advice you to go for IUI for atleast 2 cycles before taking up IVF. But still you can consult your doctor and take a call on what you want to do next. Simple advice would be to be stress free. Don't worry All the best,take care. Dr. Om Lakhani"
},
{
"id": 46746,
"tgt": "Can dialysis technician tell about the kidney progress?",
"src": "Patient: hi im a 22 year old women who recently just got on dialysis its been three months since i been on here. and the question i have is do the dialysis tech post to tell you about your kidneys progress? because they don t tell me nothing i have high blood pressure and we still trying get that down when i first went to the dialysis center my blood pressure was good then it turn around and shot high my doctor gave me some more medicine but the top number still high. how can i get a high risk doctor or a better doctor that can help me? Doctor: Relying on dialysis technician for blood pressure control is not a good idea. Experienced technician might have idea but better to consult in charge nephrologist. If you are not happy with your doctor consult other nephrologist. common causes for uncontrolled BP are excessive salt and water intake. try to take bp medicines after dialysis finished. Regrading survival on HD is on average is 8 years. If possible go for transplant with which survival will be more than 30 years. All the best. if you have any questions feel free to consult me."
},
{
"id": 202327,
"tgt": "What can be done for itch and burn on penis?",
"src": "Patient: Hi My Self Is AYAN. From last few days i found whenever I am doing intercourse with my wife,just after intercourse My penis heads come up with a redish circle which itch and burn for upto 1 hours max then gets normal.pls do the needful reply. With Best Regards Ayan Doctor: Hello,Thank You for consulting HealthcareMagic!May be due to trauma, because of the act itself or it can also be due to infections like viral or bacterial, or may be due to allergy if you are usinf any condoms or lubricants.Stay healthy and thank you for consulting! You can contact me again anytime, if you need clarifications or further assistance in future. Kindly rate the answer if you are satisfied."
},
{
"id": 109894,
"tgt": "Suggest exercise for back pain",
"src": "Patient: Hello doctor, my back, more specifically middle and upper, started hurting today. The pain became sharper and more intense. I think it may be gas build up. What is the best way to relieve this pain? Are there any exercises or medicines I should take? Doctor: Hi,Welcome to healthcare magic. After going through your query I think You are suffering from acute backache. Treatment of it is rest and analgesics(diclofenac) . If not relieved by it you can take narcotic analgesic such as tramadol after prescription from your doctor. Exercises are not advisable in case of acute pain.You need to done MRI TO RULE OUT DISC PROLAPSE.You can discuss with your treating Doctor about it. I do hope that you have found something helpful. If you have additional questions or follow up queries then please do not hesitate in writing to us. I will be happy to answer your queries. Wishing you good health. Take care."
},
{
"id": 134720,
"tgt": "What causes swelling in ankles and feet with numbness and tingling feeling?",
"src": "Patient: I m 34yrs old 207lbs & 5ft 7in I m swelling in my ankles/feet/& legs again however feeling numbness/tingling/pins & needles in my left leg more then my right. Firstly is that bad - ie the pin & needle feeling in such a case or not - Doctor: you need to undergo various blood investigations 1) sugar profile2)vitamin b12 assay3) thyroid profile.these symptoms are most probably due to derangements in any of the above systems"
},
{
"id": 88126,
"tgt": "Suggest cure for a persistent stomach pain",
"src": "Patient: Hi, may I answer your health queries right now ? Please type your query here...My 10 year old daughter continues to suffer from stomach pain and nausea and our Doctor has suggested that she may have Gastroparesis. We are concerned that this illness has continued for four months and now wondering if there may be other issues. Do you have any suggestions/recommendations as to what our daughter may have? Doctor: HI.Thanks for your query.The causes of pain in abdomen in a child for 4 months can not be gastroparesis and there are no great reasons to have gastroparesis in such a young age. The causes of nausea are anything that is not normal in stomach.The causes of pain in a child can be as follows::-Tummy tonsils- meaning enlargement of the lymph nodes of the abdomen.-Appendicitis- by default this has to be ruled out in every case of pain in -abdomen of a child.-Worms.The best diagnosis is a high resolution ultrasonography. some blood, urine and stool tests.A therapeutic trial with a course of an antibiotic, metronidazole and probiotics under the guidance of a pediatrician / Surgeon always helps to solve the problem and to get a diagnosis.Also given is a dose of medicines active against worms in intestine.I hope a consultation with a Pediatric Surgeon, investigations and a course of an antibiotic and metronidazole should solve the problem.If this is indicative of appendicitis , please go for the surgery of appendectomy."
},
{
"id": 208926,
"tgt": "Suggest treatment for anxiety and depression",
"src": "Patient: Hello, I m 22, I feel like my life isn t where I want it to be. I m in college, but I feel stupid going. I fail most of the classes or I have to drop them because they consist of presentations. I can t even hold a simple conversation with my own mother. Ive isolated myself form everyone including my family. Im always in my room. Most of the time i feel like i don t belong in this world and i want to take my own life away. sometimes i also get urges to kill people. the thing with that is that i hate the site of blood. i don t know who i am. i feel like a failure i want to work for law enforcement , but im too fat to even do that. and im not smart enough to do anything else. i don t know what to do. i already over dosed and i had to go the hospital but i didn t tell them i over dosed and they just thought it was my body reacting to the medication. I feel so lost. Doctor: HIThanks for using healthcare magicI think, you are in depression and in that case, you need antidepressant with low dose benzodiazepine. These drugs would help you to come out of that phase. You can also try some relaxation exercise that would help you to come out of that phase. Better to consult a psychiatrist for proper treatment. In case, you need further help, you can ask.Thanks"
},
{
"id": 21818,
"tgt": "Is high BP and urine albumin curable?",
"src": "Patient: Hi, may I answer your health queries right now ? Please type your query here... My son who is 34 yrs detected with high blood pressure & urine albumine but his functions are normal according to other tests. Is this curable? what food should be avoided. Doctor: Thanks for contacting For dietary advice it is necessary to know BMI (height and weight ratio) of your child . As you told that he has High BP and Urinary albumin so that in general please restrict salt intake wich is in average 5 grams in 24 hours for blood pressure and restric protien diet till albumin will normal (light protein allowed) . Avoid full cream milk (use low fat contains milk which is also known as toned milk) and it is also advice to you that you must be contact to dietician from your treating hospital who will help you to make diet chart from according to BMI and investigation which you have.Thanks"
},
{
"id": 57191,
"tgt": "Is high level of bilirubin an indication of cancer?",
"src": "Patient: Hi I am a 43 year old women, I had a blood test done last week because I was feeling nauseous. My bilirubin level was very high at 25, my doctor told me that was very high and I have to go for a scan later today. I am worried sick, I can't eat or sleep. Is it a sign of liver cancer? Doctor: Hi,I am so sorry to hear about your distress. A bilirubin level of 25 is definitely on the higher side and would have clinically presented with jaundice. I presume the bilirubin level you are stating is the \"Direct\" or the \"Conjugated\" type of bilirubin? There are various causes for such a high level of bilirubin and it might be good to know that cancer is not the commonest cause. A stone in the bile duct or a stricture in the duct is usually the main culprit. But unfortunately cancer of the bile ducts or the pancreas or \"Ampulla\" (where the bile ducts open into the intestine) also presents with jaundice. So the only way to diagnose the cause is to get a scan. Irrespective of what shows up on the scan, it might be heartening to know that there are excellent therapeutic options available for treating your condition.So I would suggest you to relax and wait for the results of your scan.Please do not hesitate to contact me fir further clarifications - rxsuresh@gmail.com"
},
{
"id": 94504,
"tgt": "Abdominal pain. Taken Amoxicillin and Metrogyl. Normal ultrasound",
"src": "Patient: HI PAIN ABDOMEN LAST 25 DAYS IN EPIGASTRIUM ,(STRATED AFTER TAKING OUTSIDE FOOD AND ALCOHOL FOR A FEW DAYS)TAKEN AMOXYCILLIN500AND METROGYL400 TDSX8DAYS AND PANTOCID 40MG IBD LAST 20DAYS AND ALBENDAZOLE ITAB STAT( PREVIOUS HISTORY OF HIATAL HERNIA 4 YEARS BACK WITH HEALED DUODENAL ULCER)STILL HAVING DULL ACHING IN UPPER ABDOMEN BUT NO ACID REFLUX OR HEART BURN.WITHOUT ALCOHOL FOR LAST 25 DAYS AND NORMAL HOME FOOD.ULTRA SOUND IS NORMAL BLOOD REPORTS :LFT,KFT,AMYLASE ALL NORMAL Doctor: Hi, Thanks for posting your query. You are most likely suffering from acute gastroenteritis. Absence of watery diarrhea doesn't rule out possibility of gastroenteritis. You should consult with your treating doctor/ internal medicine specialist and should go for thorough check up. You should take complete antibiotic course (cefixime and combination of oflaxacin & tinidazole), probiotics, antispasmodics, and continue Pantocid DSR . Avoid milk and all milk products such as cheese, butter. Take soft diet with curd. Take care, Dr. Mayank Bhragava"
},
{
"id": 208572,
"tgt": "What causes sensitive nature and difficulty in remembering things?",
"src": "Patient: i am really sorry to contact you but since you are a doctor i need your advice, my friend, actually my best friend , she is normal all the time , but sometimes she forget all her friends , and me as well , but she remember her parents , her sister, but she forgot her servant . and none of schizophrenia cases is like her . she love to be alone , she don t talk much with her mother , she talks much with me and her dad , she have no cold feelings , she is so sensetive and so lovely and nice, Today she forgot all her friends and me , for now it have been 4 hours since she is like that , please tell me what she have . Doctor: DearWe understand your concernsI went through your details. I suggest you not to worry much. From the symptoms you provide, it seems that your friend is amnesiac. Amnesia is a deficit in memory caused by brain damage, disease, or psychological trauma. There are several types of amnesia. This could be Prosopamnesia which is the inability to recognize or remember faces or names. Still perfect diagnosis is required. Please consult a neurologist.If you require more of my help in this aspect, Please post a direct question to me in this website. Make sure that you include every minute details possible. I shall prescribe the needed psychotherapy techniques which should help you cure your condition further.Hope this answers your query. Available for further clarifications.Good luck."
},
{
"id": 117816,
"tgt": "What does spgt result of 43.9 u/1 suggest?",
"src": "Patient: hello sir , i have take a report for sgpt twice in a gap period of in which i have given result accordingly (1) 88.9u/l date:11th jan 2014 (2) 43.9u/l respectely. date:5th feb 2014 so ,wat does it means is it checking error. or other Doctor: hi, your first report is slightly high, but your second report is normal. so no need to worry. sgpt is an enzyme. its level increase in liver dysfunction. but at preseent your level is normal. if you dont have any symptoms then no need of any intervention required. if you have any symptoms then investigate further and take treatment accordingly.thanks for using health care magic."
},
{
"id": 196248,
"tgt": "What causes the feeling of passing urine at the edge of the penis?",
"src": "Patient: In the past few weeks, I've been getting this feeling this weird feeling at the tip of my penis like pee or semen is going to come out. After i'm done peeing when i wiggle my penis the feeling is as if a semen is going to come out. I've checked and nothing comes out. I'm concerned about the fact that it keeps happening. Any ideas? Thanks! Doctor: Hello and thanks for your question.It's my first priority to help you.Well possible causes may include Balanitis,Urethritis.And you should hydrate yourself with plenty of fluids.Do cool water sitz bath with betadine solution for 10-15 minutes.You should wear loose fitted cotton clothes to prevent friction.Try OTC AZO pills and cranberry juice.You should avoid scented soaps for washing.If your symptoms'll persist then urinalysis should be done.Hope this helps."
},
{
"id": 167661,
"tgt": "Suggest remedy for dizziness",
"src": "Patient: My 22 month old granddaughter has over the last couple of months been having dizzy spells, where her head will wobble, like she s trying to catch herself and then she will fall. This phenomena also happens while she s sitting on your lap, etc. it happens mostly in the morning, roughly every half an hour or so. She has been congested and had bronchilitus (about a month ago) and had 2 antibiotics to combat a virus. She is still congested, and pokes at her ears. Doctor: few things that causes dizziness / vertigo in a child : the likely cause in this child is middle ear infection which is usually follow a viral respiratory tract infection, and can cause vertigo . and diagnosis is made by otoscope by her pediatrician at the clinic. the second one is acute cerebellar ataxia which usually follow viral infections and this condition is post inflammatory and self limited will wean off by 2- 3 weeks .I hope this helps"
},
{
"id": 196440,
"tgt": "How to quit the habit of masturbation?",
"src": "Patient: i am feeling so much regret about telling my story to you.My age is 22 years and 6 month.I have been masturbating excessively since 12 years old.I started masturbation unintentionally from childhood.When I came to know about masturbation,I was addicted to masturbation.Now I try my level best to quiet this habbit but I cannot because when I use computer and internet I go toward sexy videos that's why I have to masturbate.Now I am suffering from some urinary problem,premature ejaculation problem and my body has become so much thin.Please guide me,what should I do to solve my urinary problems,premature ejaculation and to recover my physical fitness.I shall be very thank full to you. Doctor: Hi, Thanks for posting in HCM. I understand your concern. Masturbation is a healthy and normal physiological phenomenon. Excess practice is not recommended. When practiced properly, it doesn't cause any harm like physical weakness, urinary problem, premature ejaculation, decrease in sexual performance or fertility. It is all there in you and with will power, you can decrease the frequency gradually. Usually, doing it once a week or thrice a month should be alright. You can get engaged in other activities, physical activities and meditation to get diversion from masturbation. Hope the information provided would be helpful. All the best. Regards, Dr. Ashakiran.S., MBBS., MD."
},
{
"id": 788,
"tgt": "What are the chances of pregnancy by ejaculating outside the vagina?",
"src": "Patient: Good Morning Doc i had my mens on the 3rd of january and stopped bleeding on the 6th. then had sex on the 10th of january. my husband ejaculated outside but we didnt use condom. i am afraid he might have pre cum insideas we not planning for a child.but for sure he did urinate before making love. what is the possibility of falling pregnant. i have a regular cycle with around 25 to 28 days thanks for advising on the probability Doctor: Hi,I understand your concerns.Following is my reply:1)\u00a0\u00a0\u00a0\u00a0\u00a0There are no chances of pregnancy. Dont worryYou can contact me anytime directly to ask question by pasting following link in your browser:XXXX"
},
{
"id": 100508,
"tgt": "Suggest treatment for dust allergy",
"src": "Patient: hello sir,this ram... actually i am suffering with nose problem since 15 days onwards by the cause of one day i used bud to remove dust. then after i am suffering with some problem in nose but actually i am using 'neospirin-h' drop which is prescibed by one of the doctor... so let me inform some good hospitals for ent those are proving low cost service or free service....Thanks & Regardsram Doctor: Hello Ram,Thank you for asking at HCM.I went through your query and would like to make suggestions as follows:1. Were I treating you, I would suggest you regular montelukast and levocetirizine for at least 1 month.2. I would also suggest you allergy testing which will help you identify the substances you may be allergic to and also to know the measures to avoid them.3. In general, I would suggest you to avoid exposure to dusts, smokes and air pollution.Hope above suggestions will be helpful to you.Should you have any further query, please feel free to ask at HCM.Wish you best of the health.Thank you & Regards."
},
{
"id": 37295,
"tgt": "What are the effects of ant bites?",
"src": "Patient: yes, we have a 21 month old that was bitten severely by fire ants. he was taken to the hospital, given a shot and sent home. within a few days, he was having pain in his legs and now he can t walk. the doctors sent him home again. nobody can figure what is going on with him. Doctor: The bite of fire ants injects formalin and certain other chemicals which are locally damaging to the tissue. This gives rise to pain and also may give rise to an allergic reaction. The shot that the doctor gave, would probably take care of the allergic component. But the pain is likely to have persisted and is responsible for his inability to walk properly. Give him some anti allergic like levocetirizine for a few days and also apply some emollient over the area. It should become ok in a week or so."
},
{
"id": 192409,
"tgt": "Could the pain in spine be due to excessive masturbation?",
"src": "Patient: hi my pinus is not straight and not erct straigh and it took only 20 sec to ejeculate and i mastrubate 4 times in a day some times and twice a day and daily thats why i became so weak my spinal cord pains every tim plz doc help m i`m very upset plz plz plz Doctor: Hello, The pain at your lower back (not spine) could be due to the strain you give to your back during masturbation. Reduce the masturbation frequency to three times a week and take rest. Therefore I suggest consulting a psychologist for counseling. Hope I have answered your query. Let me know if I can assist you further. Take care Regards, Dr. K. V. Anand"
},
{
"id": 172065,
"tgt": "What causes a lump under the chin in a child?",
"src": "Patient: Hello, My son is 13months old and he has had a lump under his chis for about a month now. Its gotten bigger in the past couple of weeks. I have taken him to the doctor and they feel that it is a swollen lymph node due to his being ill. He has had cough and cold for about a month and for the past two weeks a double ear infection that I have been treating with antibiotics. I am starting to worry that he has something seriously wrong like (cancer) or something. Should I be really concerned here? Doctor: Hi dear,I understand your concernIt seems he has reactive lymphadenopathy due viral infection.You should be worried,but not too much.Observe for lymph.node, if it will decrease till 1sm,it is normal. If it will more,then 1 am,then perform blood test to exclude mononucleosis, other viral infection as herpes, for this you can give Ig M,G to HHV 6 type,Cytomegalovirus, EBV. It is also important to perform throat culture and consult child at ENT doctor to exclude chronic infection:adenoids,tonsillitis,ethmoiditis.If everything is Ok.You should wait at least 1month,it will decrease.For speedy recovery you should put on compress with Camphor every night(crash 2tablets,mix with water and apply to node,it will resolve, Camphor is strong antibiotics inflammed medicine).Please contact me before visiting your primary doctor.Wishing your baby good health"
},
{
"id": 191393,
"tgt": "How can gestational diabetes be treated?",
"src": "Patient: Hi i am 12.5wks preg and I have gestational diabetes I take insulin my sugars are usually between 70 &120 but today after. Lunch it spiked to 159 then down to 137 i know. I didn t. Eat anything. I wasnt. Suppose. To is this normal. Just to have. One random spike. Doctor: Hello,The sugar levels you have mentioned are in normal range. Even the spike of 159 after lunch is normal. Do not worry.Hope I have answered your query. Let me know if I can assist you further.Regards, Dr. Phanindra Dulipala"
},
{
"id": 47591,
"tgt": "What causes burning while urination while on treatment for kidney stones?",
"src": "Patient: 9 MONTHS back i got Stone in right Kidney which is 6 mm,It reduced to 3 mm with the help of doctors advice,tablets.From day before yesterday,It starts burning at the time of urine outflow.left hip is paining,doctor advised to take Neeri tablet for Stone,how this helps me... pls reply me Sir. Doctor: Hi, welcome to HCM. Kidney stone can precipitate urine infection. Urine infection can cause burning in urine.Sometime stone can stuck into urinary passage and causes this type of pain.it is necessary to do basic urine report with culture, S.creatinine and sonography to rule out both these possibilities. Maintain adequate hydration. You can get back to me with above reports.I think this would be helpful to you. Best wishes. tc. Dr Jay Patel."
},
{
"id": 189712,
"tgt": "On amoxycillin for tooth infection. Swelling on cheek. Feeling nauseous, bloated, tired. Caused by antibiotic?",
"src": "Patient: I am on 2000 mg of amoxycillin for tooth infection. I have had a root canal , another procedure scheduled next week to extend the 3rd root the dentist had difficulty with, and will see an oral surgeon for swelling on my cheek if the anitbiotic doesn t resolve the issue. It looks like the infection has gone into the jawbone . I am feeling nauseous, have a very bloated stomach, and am really tired. Can the antibiotic cause those symptoms. Diarrhea, too. Doctor: Hello, Thanks for posting your query. During root canal procedures,the pain and swelling are usual. Nausea,tiredness,,diarrhoea abd bloated stomach are the symptoms associated with heavy dose of antibiotics administered. Please do inform your dentist regarding this and replace the antibiotic into lower dose. Keep yourself nourished and take plenty of water as well as fluids. Antigastric tablets has to be taken along the antibiotic to relieve the gastric acidity. hope this helps."
},
{
"id": 70914,
"tgt": "What causes recurrent pain in the chest?",
"src": "Patient: Hi. My name is Kim and I have been experiencing pain that comes and goes right in the middle of the chest area below my breasts. It was so bad two nights ago which caused me to go to the emergency room. And I m having the pains again. They come and go and sometimes are pretty sharp! What is going on? Doctor: Hello and Welcome to \u2018Ask A Doctor\u2019 service. I have reviewed your query and here is my advice. If cardiac diseases have not been ruled out then it should be done first because sharp chest pain at night is commonly seen with heart diseases. So get done ECG, 2D echo, and stress test. If all these are normal then no need to worry for heart diseases. Sometimes stress and anxiety can also cause similar symptoms. So better to consult psychiatrist and get done counselling sessions. Try to identify stressor in your life and start working on its solution. You may need anxiolytic drugs (Propranolol and Flunarizine combination) too. Don't worry, you will be alright with all these. Avoid stress and tension, be relax and calm. Hope I have answered your query. Let me know if I can assist you further. Regards, Dr. Kaushal Bhavsar"
},
{
"id": 196304,
"tgt": "Suggest treatment for dry and irritated tip of shaft of penis",
"src": "Patient: 24 year old male , i recently had a surgery to remove the \"bridge\" between the shaft and the forskin and now (4 months) after i still feel abit unconftrable , it feels dry and the tip of the shaft looks irretated and inside the pee hole is a small cyst , but i dont feel any discunfort Doctor: HiGREETINGS Very difficult to comment without seeing pictures. If frenuloplasty was done properly there shouldn't be any issues.I suggest you to consult the same surgeon and get it evaluated. Hope you are convinced. Regards"
},
{
"id": 111806,
"tgt": "Treatment for lower back pain and testicle and leg numbness?",
"src": "Patient: Lately I have been having little pain on my lower back and the left testacle (could the pain in the lower back also involve the testical pain)?Sometimes my left leg goes numb like it has fallen asleep. Someone said to me that this could be a pinched nerve as the pain mostly happens when I sit down, if I lay down or stand the pain is little to none.I am not sure what to do, shall I go and visit neurologist for a scan? A friend of mine recommended me if this is just a pinched nerve to just exercise alittle more and maybe get a massage.Would appreciate if you could give me some more advice on this.Kind RegardsMitja Doctor: Dear mitjaAfter reading your study, chances are that either you have a kidney stone or a nerve compression. Kidney stone pain can go from back to testicle. Very rarely nerve compression can cause pain in testis. But as you are saying that you also feel numbness, so this thing also has a possibility. So my advice to you is to have MRI of your spine and ultrasound of your kidney. This will give us complete information.Exercises are helpful if mild nerve compression is there but that has to be supervised by a physiotherapist.I hope this information will help you."
},
{
"id": 19695,
"tgt": "What causes palpitations and irregular heartbeat?",
"src": "Patient: As I check my pulse, I can tell my heart is skipping a beat intermittently while at rest. There is no pain and I feel fine, but there has also been an occasional palpitation that seems to correspond with the missed beat. I have not had a heart attack and I had a routine check-up with my internist yesterday. Doctor: Hello there. Thank you for choosing Healthcare magic . First of all I would like to ask you what is your age and sex. There can be many causes of palpitations like stress, thyroid disorders, fever, too much caffeine. Have you got an EKG done during your routine check up ? if not I would advice you to get it done to rule out any cardiac causes. Also a 24 hour holter monitoring to detect any irregular heartbeats. Get a thyroid profile done. Get a haemoglobin test done cause sometimes anemia may also lead to palpitations. I hope that answers your question. i f you ahve any other queries please feel free to write."
},
{
"id": 170645,
"tgt": "What causes rashes on body after treatment for fever in a child?",
"src": "Patient: my 15 month old son has been ill for 4 weeks on and off, had a a fever for a week 2 weeks ago which he was treated with antibiotics as they THOUGHT it may be to do with his ears, he hasn t been 100% since, itching rashes one of which was dry and looked yellowish on his face the last 3 days which seems to be clearing with nappy rash cream on saturday he had 1 small white spot which was red around the outside, the next day it looked deeper yellowish and was twice the size, today it is twice the size again yellow and weeps watery fluid he also had a high temp this afternoon so gave him calpol and was sick twice, cant keep anything down. eyes also watery and gunky. getting worried as he doesn t seem to be getting any better but is up and down. i dont feel confident in my doctors as the 3 who saw him 2 weeks ago didn t really know what was wrong (at GP s and hospital) and this sore thing is concerning me...i need so decisive answers..what do i do and what could it be? Doctor: Your child most likely has chicken poXThey usually start with initial fever when the virus is in the blood and multiples and after improvement starts with fever and rash as what you describe.Please keep your child comfortable .have a doctor check his throat and lungs since your baby is only 15 months and can apply calamine lotion.If fever continues and baby becomes drowsy take him to nearby hospital for infdection ."
},
{
"id": 85458,
"tgt": "What are side effects of Duoluton L?",
"src": "Patient: hello sir..... am 28 yrs old married woman with 4 nd hf yr old son. during check up we found that my left ovary has enlarged with cysts which got ruptured. my doctor prescribed Duoluton L. what are the side effects of this medicine?? Are there any chances of gaining weight, as am am already overweight. Doctor: Hello, Some of the side effects of Duoluton L are: - Nausea, vomiting - Headache, abdominal cramps / bloating - Breast tenderness, swelling of the ankles / feet (retaining fluid) If any of these effects persist or worsen, you should consult your doctor. Hope I have answered your query. Let me know if I can assist you further. Regards, Dr. Dorina Gurabardhi, General & Family Physician"
},
{
"id": 113178,
"tgt": "Constant lower back pain. Can an online doctor explain the MRI report ?",
"src": "Patient: i have lower back pain from 6 year bcz of damge somting in back during my duty lifit weight . i feel that time some pain after 3 day it littile normal, but during 5 when lifit weight mor than 10 to 20 kg it start agin like first time pain, MY MRI report 1. Dessicted L4-L5 disc noted. 2.The vertebral bodies are grossly normal in morphology and signal intensity. 3.right paracentral disc bulge noted at D11-D12 level causing thecal sac indentation on the right side leading to compromise of ritht trvaversing D12 nerve root Normal conus medullaris . Nomal posterior elements. Normal paravertebral soft tissues Doctor: hi first thing you need to do is stop lifting heavy weights which have cause damage to your disc and causing backpain consult your orthopedic surgeon , if you do not have any neurodificit and otherwise bearable pain then can be managed conservatively without surgeryand good physiotherapyand medications. but, you need to be very carefull because if it progresses then you may have to undergo the surgery"
},
{
"id": 188102,
"tgt": "Could Amoxicillin allergy be causing numbness and tingling sensation on my tongue?",
"src": "Patient: Hi Doc, First went to see my surgeon for my wisdom teeth because I had an infection to the gum area and it as swelling and hurting. The dentist/surgeon gave me amoxicillin for 2 week. My surgery was schedule right after my medication ended. My wisdom teeth were extraxted and I was prescibed again amoxicillin and porcacet. After the 3rd day taken those medication, I felt my tongue numb and no taste. I was old that can be a lingual nerve. However, I stopped taken Amoxicillin for 2 days and I was feeling to get better, less numb and getting a little more taste to food. Today, I took the amoxicillin again, and my tongue feels more numb, and no taste, for tingling and burning cold hot feeling that was fading away. You think that instead of having a lingual nerve damage because of the surgery, I can be allergic to the Amoxicillin and if so, would I be able to get my taste and normal tongue back??? Doctor: Hello, thank you for consulting with healthcaremagic. I don't think that allergy of amoxicillin causes these kind of symptoms like numbness of the tongue, because before the surgery also you were taking it if it did not caused numbness at that time, then it should not happen this time. It is some complication of the surgery only.I think you should visit your dentist again for the problem. Because damage to lingual nerve can be diagnosed in X-ray. He will prescribe you the medications and you will be fine. Hope it will help you."
},
{
"id": 9743,
"tgt": "Suggest remedy for hair loss, loss of concentration and memory loss",
"src": "Patient: i gone operation for mitral valve replacement in the year 1996 year.i am using regularly acitrom 3mg, lanoxin .25. presently i am facing hair loss and loss of concentration , memory loss, and severly heart beating.........please suggest what to do Doctor: Hello, For hair loss I suggest you to use either minoxidil or Procapil solution. Hope I have answered your query. Let me know if I can assist you further. Take care Regards, Dr Kakkar S., Dermatologist"
},
{
"id": 160476,
"tgt": "What causes runny nose and watery eyes in a child?",
"src": "Patient: My 8 month old has had a runny nose (clear mucous) and watery eyes (no redness) for the last three days. She doesn t appear to be fussy or uncomfortable. I do hear the congestion in her nose when she is breathing. Does this sound like a cold, or an allergy? Doctor: Hello, This sounds like a reaction to pollen and changing weather. But in general, we don't consider babies to have true seasonal allergies until they are a bit older. You have to live through a few seasons of pollen before you can have a true immune allergy response. But, pollen is an irritating molecule and babies little noses and little eyes can get irritated and make mucous just upon exposure to varying levels of pollen (or dust, or animal dander, smoke). If there is no fever or cough then she probably doesn't have a cold. Hope I have answered your question. Let me know if I can assist you further. Regards, Dr. Lisa Baker, Pediatrician"
},
{
"id": 146157,
"tgt": "What causes viral spinal meningitis?",
"src": "Patient: my son age 47 has been diagnosed with viral spinal meningitis what can you tell me about this disease and its prognosis my email address is YYYY@YYYY please I am handicapped and living on a very low income. Im 1200 miles away from my son and now I dont know if i ll ever see him again please help me. Doctor: Dear sir,I understand your concern about your son's health.Spinal meningitis , when caused by a viral infection can have variable prognosis depending on the time how early treatment is started.If diagnosed early , and antivirals like acyclovir given early in the course of the disease, patient may recover upto a larger extent, however delay in treatment may leave fixed residual deficits.My advice is to immediately take him to a neurologist."
},
{
"id": 108384,
"tgt": "Suggest treatment for back pain",
"src": "Patient: My lower back hurts. The pain runs down into my hip then turns into my shine and along the side of my foot. Sometimes my foot swells. My doctor said I have lost some reflects in my left leg. I can do just a little work and I get really tired. What can this be Doctor: 1.Since you have radiating pain with loss of reflex , you should consult a neuro/spine specialist and get investigated possibly with MRI. 2.Your definitive treatment to decided thereafter. 3.This possibly is nerve root compression in LS spine."
},
{
"id": 99181,
"tgt": "What causes cough,loss of voice and phlegm in throat?",
"src": "Patient: Hi can you please tell me what you think the following might be.. Well I have allergies.. pollen, ragweed, dust and mold.. also I have asthma. My question is .. what could be causing my coughing with losing my voice (like it cuts in and out) and then I feel that there is so much phlegm in the back of my throat that it seems like no amount of coughing will clear it out and Im having trouble breathing.. like Im breathing on the surface if that make any sense.. please help Doctor: HI, thanks for using healthcare magicAllergies would cause post nasal drip which would cause coughing and hoarseness.It would be best to reduce exposure to these allergens if possible. In addition the use of oral anti histamines and topical steroid nasal sprays would be helpful.This includes sprays such as nasonex, nasocort, flonase, rhinocortI hope this helps"
},
{
"id": 208575,
"tgt": "Suggest therapy for split personality",
"src": "Patient: My name is haley, I feel like someone in my family is going to die soon and I try to warn them but they get mad at me, I had a dream three nights ago where my moms car was flipped upside down on a country road on her way to work I m scared now because some of my dreams have been coming true usually the bad ones. nobody believes me and I don t want to scare anybody. I ve also been feeling lately like I can t control my myself like when I get angry I m a different person when I look in my mirror I see someone else I ve been having Deja vu everyday for a couple months and I m very scared about everything. everyone thinks I m doing it for attention please help Doctor: DearWe understand your concernsI went through your details. I suggest you not to worry much. From psychologist's point of view, you do not have any extra human capabilities. You just see dreams. They are simply dreams. What is there to be alarmed about it? Everybody have dreams. Everybody dreams about a future event. Most people dreams about a negative future event. But 99.9 percent of such dream events never happen. In your case also, the event you saw in your dream shall not happen. What ever the case may be. Just ignore it. For your better future.If you require more of my help in this aspect, Please post a direct question to me in this website. Make sure that you include every minute details possible. I shall prescribe the needed psychotherapy techniques which should help you cure your condition further.Hope this answers your query. Available for further clarifications.Good luck."
},
{
"id": 138853,
"tgt": "Suggest treatment for severe ankle pain",
"src": "Patient: I have had a stabilization surgery on my ankle 4 years ago. I now have been experiencing Cramp Like symptoms that stem from where the surgery took place and shoot up the side of my leg. It is intense pain and lasts from 2-15 minutes. During this time the ankle often freezes or is unmovable. Sometimes the pain subsides for a minute and movement returns, but if I move it again it freezes and the pain returns again. This happens mainly early in the morning and wakes me from sleep. I find that moist heat can SOMETIMES relieve the pain. After one of these episodes the area of the ankle/leg are often sore. Should I consult a Dr about this and if so, what kind of doctor should I consult? a Pediatrist? Doctor: Hi,since your ankle was stabilized 4 years back, it means the implant would be still in your body, these symptoms could be either due to infection or implant irritation. since the implant has now solved its purpose, i would recommend you to get an X Ray done and get it removed. I hope this will solve your problem"
},
{
"id": 72934,
"tgt": "Suggest treatment for chest pain and shortness of breath",
"src": "Patient: I have been having chest pain along with shortness of breath for 5 days, i have had 2 ekgs and lots of labs that came back normal. The doctor put me on xanxa thinking it was anxiety and she said if it was i would feel some relieve within 2-3 hours. its been 3 days know with no relieve and its getting worse. Doctor: Thanks for your question on Healthcare Magic.I can understand your concern. Since all your blood reports and ecg s are normal, no need to worry for heart diseases. Stress and anxiety can cause similar symptoms. It is better to consult psychiatrist and get done counselling sessions. Try to identify stressor in your life and start working on its solution. Newer anxiolytic drugs, propranolol and flunarizine combination at night time before going to sleep is very effective than Xanax. Counselling sessions and anxiolytic drugs together work well really well to suppress symptoms. Don't worry, you will be alright. Avoid stress and tension, be relax and calm. Hope I have solved your query. I will be happy to help you further. Wish you good health. Thanks."
},
{
"id": 28346,
"tgt": "What causes hollow feeling like sensation developing in my chest?",
"src": "Patient: Hi, I ve had a cold for three weeks, nasal drip, cough, and sore throat. I m taking zinc, musninex, and cold calm along with using a neti pot. A hollow feeling seems to be developing in my chest. I m hoping I can get rid of this without having to the doctor (sorry). What do you think is causing it? Doctor: Hi welcome to hcmI understand your query and concern.Your symptoms are suggestive of Ongoing Myocardial ischemia leading to evolving stroke in the brain.I advise you to get an ECG,2D Echo of your heart,lipid profile,hsCRP,CT brain immediately to confirm the diagnosis.Monitor your blood pressure,heart rate immediately.Drugs like clopilet,ecosprin,atorvas,betaloc,pantop,Strocit should be taken to prevent the progression of stroke.Avoid stress and anxiety.Avoid fatty foods,smoking and alcohol.I also advise you to get your BMI measured and reduce weight if you are obese.Also Exercise daily for 30 min.Coronary angiogram with or without stenting will be the procedure of choice.Consult your cardiologist for expert management and follow up.Post your further queries if any,Thank you."
},
{
"id": 101966,
"tgt": "What is causing allergic rashes on palms, neck and ears?",
"src": "Patient: looking for information after I visited my doctor today about a (probable) allergic rash, mostly on my torso but some on my neck and ears. She gave me a kenalog shot and a round of prednisone. I have had this before, about 5 years ago, and the same treatment. What is different about this time around is that I also have soar, red blotches on my palms. When I visited the doctor, these were the most painful and after several hours since the injection, they are still painful (like bruises) but less red. It is also the most itchy. What is causing the reactions on my palms? Is it related to the other rash which I had before or something different? Doctor: these are allergies and prednisone decrease it not cureyou can start reacting to any protein at any timemay be soaps oils cooking oils application oils 'hena dyes body application powders creams shampoosometime food and pollenyou can control by application of anti allergic ointments and tabs but get correlation with history or get tests to find the cause for preventing future allergies"
},
{
"id": 168016,
"tgt": "What causes nausea in a 10 year old while drinking milk?",
"src": "Patient: my 10 years old son feels vomiting when given milk in the morning befor school......infact he feels vomiting in the school also but everything remains fine in the evening.what should i give him to eat or drink?i have been giving him milk from the age of 3 before going to school......but suddenly why this kind of problem have arrived? Doctor: HI...by what you quote I feel that the child is having mild form of GER (gastroesophageal reflux). Your son will need - 1. Antireflux therapy - Proton pump inhibitors + antiemetics2. Always make him lie down in a propped up position - as in the care seat. He should not be sleeping completely flatly.3. Avoid heavy feeding and feeding more times and in small aliquots.He needs expert gastroenterology consultation.Regards - Dr. Sumanth"
},
{
"id": 175458,
"tgt": "Why is my child passing smelly gas?",
"src": "Patient: My eight year old has had very smelly gas the last couple weeks. He is prone to constipation, and is on a regular Mira ax and senna regimen. But this symptom is new for him and concerning in that it continues though he seems to be having relatively regular bms. Doctor: Hi Dear Welcome to the HCM,Smelly stools with gas is the sign of some infection in the stomach.antibiotic like ciprofloxacin with meterogyl combination will releive the symptoms.Hope the query is answered.Thank"
},
{
"id": 6072,
"tgt": "Can I keep the baby if I'm pregnant as I was on antibiotics for loose stools ?",
"src": "Patient: Hi, I am 30 years old. I am trying to conceive. Two weeks back I had to do a antibiotic course of Satrogyl for a week due to loose stools . I had to travel last week due to which the stomach problem relapsed this week adn I am taking Cifran right now. My period is due tomorrow, If I do get pregnant, will I be able to keep this baby? Doctor: Hello. Thanks for writing to us. Sincxe you have taken antibiotics like fluoroquinolones which are not recommended in the first trimester of pregnancy, ideally speaking, you should not keep the baby in case you become pregnant. But if your pregnancy test comes positive, then it is best to discuss this in detail with your gynecologist. I hope this information has been both informative and helpful for you. Regards, Dr. Rakhi Tayal drrakhitayal@gmail.com"
},
{
"id": 223373,
"tgt": "Can pills cause cramps with constipation?",
"src": "Patient: Hi I ve had weird cramps for the past 3/4 days it s my 3rd week into my cycle. I also had bad heartburn last night and feel like I m constipated along with lower back pain. I m on birth control junel fe, had my period last month and have not missed any pills this month Doctor: hello user,side effect of pills are inter menstural bleeding,breast tenderness,nausea,weight gain,mood changes,missed period,decreased lipido etc...it does not cause constipation...you can take hot water with lemon,fibrous diet...and consult your physician if constipation persist..."
},
{
"id": 19816,
"tgt": "What causes rise in BP with pain in chest and left arm?",
"src": "Patient: Hi, may I answer your health queries right now ? Please type your query here... My husband 31 yrs old is experiencing uneasiness, discomfort and his BP is high 140 - 150. 2 days ago, he had shooting pain in his chest area and pain in the left arm, i took him to a doc, the ecg was normal and his enzyme test is normal as well.He is still having uneasiness and discomfort and bp is still high. He refuses to take tablets as he feels he is too young to start taking tablets.Kindly advise if this discomfort continues, is it something to be worried abt or is it just some digestion problem and it will be alrite in time Doctor: Hi, Thanks for the query. According to the history, your husband seems to be hypertensive and is having chest pain. Even though the ECG was normal, I would suggest getting an Echocardiography done. The enzyme test sometimes comes normal if done too early in a heart disease. As far as age is concerned, yes he is young but it is quite common these days to see young hypertensives to have heart disease. I would suggest a repeat visit to your cardiologist, get an Echocardiography done and take medications as advised by your doctor. Hope I have answered your query. Let me know if I can assist you further."
},
{
"id": 49416,
"tgt": "What are the risks of undergoing baratric sleeve surgery while having sodium deficiency in kidneys?",
"src": "Patient: Hi Dr. Grief, My name is Mary Harper, I was just diagnosed with sodium deficiency in my kidneys. What are the risks of undergoing baratric sleeve surgery if I have this condition if it is to be monitored closely with my I.V. fluids after surgery? Am I a high risk for this procedure? Doctor: Hi...if it is closely monitored by iv fluids then you need not worry about it...no it dosent makes you high risk patient for this procedure...Dr. Ashish Verma"
},
{
"id": 74922,
"tgt": "What causes chest pain, fatigue and metallic taste in mouth?",
"src": "Patient: I have aching in my chest when i breath in and have had a metallic taste in my mouth for the last three days, and am so tired, exhausted - could sleep anywhere, but can't, and even my teeth started to hurt. Am i going crazy? I have an ppointment tomorrow, but I was hoping you might know something Doctor: Hey there,Definitely there is some problem with you.Get done serum calcium, phosphorus levels, rule out any tooth problem leading to this.Have good sleep for same."
},
{
"id": 9021,
"tgt": "Ayurvedic cream for lightening lips",
"src": "Patient: hello docter i m teena.......i have a big problem that my lips are very dark.........nighter i do smoking.........nor use lipstik........i never used local lipbalm or lipglows..........before 2years ago my lips are not too much dark........my lips were ok bt now it is so dark........please tell me what should i do........please suggest me the best ayurvedic cream s name that can be lighten my lips soon......... Doctor: hi read your prob the cause of dark lip n coffee lip r using excess coffee, sun burn, anemia, micro minrals deficency. so dont keep dry your lip, for avoiding their dryness use excess liquid intake, use any mostirizer, use excess amount of vita-C riched fruits, n high protine diet. aseemadhuri@gmail.com"
},
{
"id": 189468,
"tgt": "Gums filled with puss and blood",
"src": "Patient: Dr one of my aunt,who is 43 yr old often get a boil in her gums filled with puss and blood, if she prick that boil all puss and blood will come out bt after some time again it will be filled. It is nt at all paining and ter is only one boil at the extreme end of the gum in right side with past one yr.She also took some tablets bt nothing happened. Doctor: Hello, Thanks for posting your query. The pus is developed due to periodontal infection of the associated tooth. A thorough clinical as well as x-ray evaluation is required to diagnose the disease. Pricking the pus is not a permanent solution . The pus has to be drained thro;drainage and curettage of abscess. Get your teeth cleaned. Maintain good oral hygiene. Brush your teeth twice daily and rinse your mouth after every meals. Also monitor your blood-sugar level to monitor diabetes mellitus. Include vitamin-c rich fruits into your diet. Take plenty of water and fluids too. Hope this helps."
},
{
"id": 2789,
"tgt": "What causes abdominal pain after embryo transfer?",
"src": "Patient: hi i had embryo transfer before 9 days & since that time I experiece lower abdominal pain crampy in nature (like the premenstrual pain ) & i think it is uterine contractions , but day by day it is decreasing. does that means I am not pregnant & I lost the embryos ? Doctor: Hi,Decreasing pain doesn't indicate if you are harbouring a viable embryo or not. Just wait for your B Hcg results while continuing your folic acid and progesterone support.Hope I have answered your query. Let me know if I can assist you further. Regards,Dr. Anjali Jain"
},
{
"id": 56775,
"tgt": "Suggest treatment for fatty liver syndrome",
"src": "Patient: Sir, I have fatty lever stage -I , used Udibon for about six month. After leaving the medicine, I feel heaviness on lever side of abdomen. kindly advise me about whether medicinal cure is enough to cure the disease fully. How long I should take the medicine? Doctor: Hello Underlying cause of fatty liver should be investigated.It may be due to many reasons like altered lipid profile,sedentary life style,obesity etc.Fatty liver is a reversible stage.It is due to excessive deposition of fat within the liver cells.Your condition need few more investigations lie routine hemogram,random blood sugar,lipid profile,liver function test.You should increase your physical activity and avoid fried and fatty food.You may need few medicines after full investigations.It may take 6-8 months for complete reversal of fatty liver.Get well soon.Take CareDr.Indu Bhushan"
},
{
"id": 172091,
"tgt": "Suggest treatment for cough & pain in right arm",
"src": "Patient: My 13-year old son has had a hacky non productive cough for two weeks. He started complaining of right arm pain yesterday between his elbow and shoulder. He is having a lot of trouble writing and it even hurt him to hold the hair dryer this morning. The cough is still present as well. Doctor: There is a general rule that one should try and unify a single diagnosis from apparently unrelated symptoms. I tried, but I could not find a single reason for both, his cough and his arms. I guess he might have slept over his arm at night, since only one arm is hurting. For the dry cough, you can try any DRY-cough syrup like Actifed DM or Dilo DX."
},
{
"id": 172334,
"tgt": "What causes severe head jerking in a child?",
"src": "Patient: hi my child was in hos last month they said he suffering with anxiety nbut i think its much more she is head jerking really bad and its getting worse by the day she falls asleep during the day a lot to im sure she has conntacted something after having pandemrix swine flu jab can i be right Doctor: Hi dear,I had gone through your question and understand your concern, jerking of head can be sign of epilepsy after infection. I suggest- perform EE G;-MRI to exclude changes in brain tissue;-contact to neurologist.Hope it is answer to your question.Wishing your baby good health"
},
{
"id": 85212,
"tgt": "What causes dizziness after stopping Serlift intake?",
"src": "Patient: Hi..I have slowly decreased Serlift 50 to 25 mg and now gradually stopped. I am having dizzyness and increased appetite...so is this normal? I am trying alternate heialing for depression and a physician recently told me that Serlift was a possible cause of not being able to lose wight. Pls advise. Doctor: Hello,It is good that you have tapered serlift gradually before stopping it but you should also know that withdrawal symptoms last for almost 3 weeks. As dizziness is one of the withdrawal symptoms common for most of the antidepressant drugs, you can experience for a few more days.Hope I have answered your query. Let me know if I can assist you further. Regards, Dr. Prabhash Verma, General & Family Physician"
},
{
"id": 192930,
"tgt": "Suggest remedy for low sperm count and motility",
"src": "Patient: Dear Dr, I was put on Siphene M (Clomphene Citrate) and Lycopene tablets after diagnosing with low sperm motility(20%) and total count (20 miilion). After 6 months the motility was 48% and total count 80 million and i was asked to stop Siphene. Later after 2 months the count and motility is showing the initial low values,Please advice what to do? Doctor: Hi, The low motility can be due to hormonal imbalance, stress and smoking. based on the cause only the treatment can be given. you can initially take antioxidants. Hope I have answered your query. Let me know if I can assist you further. Take care Regards, Dr S.R.Raveendran, Sexologist"
},
{
"id": 33476,
"tgt": "What causes yeast infection after depo shot?",
"src": "Patient: I have been taking the depo shot for about a year now. About 4 days after getting the shot I will get symptoms that mimic a yeast infection: white clumpy discharge, itching and burning. However, this isn't a yeast infection. About 1-5% of women suffer these side effects from the depo. My question is is there something I can do or a medication that I can take that will alleviate these symptoms? They will last a very long, excruciating few weeks. Doctor: HI, thanks for using healthcare magicSome women are more prone to have yeast infections.The following may help: (1)determine if there is underlying immune deficiency such as diabetes, steroids, other immune suppressing medications, HIV(2) probiotics-these may help to reduce the frequency of acute infection(3)using cotton underwear, cool clothes(4)you may need a prolonged course of antifungalsI hope this helps"
},
{
"id": 186394,
"tgt": "What treatment is suggested for tiny painful lump on the roof of the mouth?",
"src": "Patient: Hey there I am 20 years old and I have had a tiny bump/lump in the roof of my mouth now for maybe 2 months, there has been no pain and it hasnt changed shape or size since i felt it first with my tongue. I used mirror angles to get a closer look at it today as I have started to worry because it hasn't disappeared. It is white and is slightly raised out from the roof of my mouth and there is no red rings/patches near or around it. I've done some searching around the internet for photos but haven't come up with anything similar, would be great if you could help! Doctor: good evening dear the problem which you are describing seems to be a palatal tori, is that lump hard??? if it then it is most likely to be a tori, in this case you have nothing to worry, it is like an extra piece of bone in your roof of mouth, if it irritating you while speaking or eating, you can get it excised by a oral and maxillofacial surgeon nearby your locality."
},
{
"id": 30418,
"tgt": "Suggest remedy for patient dying due to colitis",
"src": "Patient: my sister-in-law is laying in the hospital dieing of what they think is Colitis, but the anti-biotics they have tried are not working. She also has head and neck pain. She is 45 years old and has been dealing with this for over 5 weeks and been in the hospital for a week. Her sister went to see her last night and said shes just a shell. What are they missing and why arnt they doing more? Doctor: HiColitis is inflammation of the colon which could be caused by an infection, inflammatory bowel disease,ischemia or allergy.Its treatment depends on the cause.Antibiotics alone will not solve the problem if it is not of infectious origin.A stool exam/culture/sensitivity test, a colonoscopy and other blood tests will be needed to bring out the cause .I suggest you request the opinion of a gastroenterologist.Best wishes"
},
{
"id": 206681,
"tgt": "What causes chronic panic attacks?",
"src": "Patient: i am 19 year old male, for many years i have exsperienced aprehension or panic attacks (usally something im looking forward to) over the smallest of things genually! the physical symptoms come and go genually in the sequence, hand trembeling , then leg tremeling, tightness in stomache growing very tight, then causing nausia and then throwing up in some cases (in some cases muscel twiches usally small ones around the eye). which due to the smallness of the sitmulus eg. seeing friends, catching up with friends, before an interveiw, at work if im asked to look after a friends child for a shot period of time. they usally last around 10minuts start to finish sometimes as a cycle, then go and im fine, i have tryed controlling my breathing, cutting out caffine, sugars, saying to myself this is rediculus. starving myself, but the no or little effect. they seem to be getting worse and harder to hide them, so i would like any help or serjestions. its like my body is over reacting like mad and dumping enough adrenerlin to last bloody years. any help? or ideas Doctor: DearWe understand your concernsI went through your details. I suggest you not to worry much. Panic attacks always happen due to underlying anxiety. Therefore you must address the underlying anxiety first. Please understand anxiety and related problems. Once you understand your anxiety, you will be able to keep yourself away from panic creating situations. Therapy always starts with avoiding such situations along with coping techniques when forced to be in such situations. Start slowly, progress gradually, attain neatly.If you require more of my help in this aspect, Please post a direct question to me in this URL. http://goo.gl/aYW2pR. Make sure that you include every minute details possible. I shall prescribe the needed psychotherapy techniques.Hope this answers your query. Available for further clarifications.Good luck."
},
{
"id": 155834,
"tgt": "Can my daughter take nutrilite products along with her chemotherapy?",
"src": "Patient: My 9-year-old daughter is suffering from blood cancer(Pre B-cell ALL). Her treatment was started in Aug-21 2012 under BFM protocol and now she is in remission and undergoing her maintainance phase.Can she take nutrilite products alongwith her chemotherapy?Does it really help cancer patients?If yes, then plz suggest which products to take? Doctor: Hi,Thanks for asking.Based on your query, my opinion is as follows.1. Yes, she can take.2. Nutrilite products would contain protein importantly along with essential nutrients. These help in buildup of immunity and also helps in the building the cells damaged due to chemotherapy.3. She can also add fish, eggs, pulses, legumes and green leafy vegetables to her diet. It will help her in fighting cancer. Hope it helps.Any further queries, happy to help again."
},
{
"id": 222689,
"tgt": "Suggest possible tests to confirm pregnancy",
"src": "Patient: The first day of my last menstrual cycle was September 21. I had sex on September 30, October 3 and several time after. I have a 32 day cycle. Could I have gotten pregnant on September 30 or October 3rd? I am trying to figure out the closest possible due date. Doctor: Hello, and I hope I can help you today.Contrary to what many women think- ovulation does not occur on day 14 of a women's cycle... it happens 14 days before the start of your period.So if you have a 32 day cycle and your last period was September 21, you likely ovulated 14 days before your period would be due for October (the 23rd) which means you likely ovulated around Oct. 9th.The bst way to know for sure, however, is with an ultrasound (you must wait until at least 5-6 weeks of pregnancy) which can measure the fetus and make the most accurate estimate of your due date.I hope this information was helpful, and best wishes for the pregnancy,Dr. Brown"
},
{
"id": 6029,
"tgt": "On clomid tab. How many follicles one should normally produce each month?",
"src": "Patient: hello, last month i had one follicle on my right side that grew to 27mm i took oviderel as it didn t look like i was going to spike on my own/release wondered what are the chances that it actually ruptured and was mature? How many follicles are normal to have each month ...i seem to only produce one and I am on clomid 50mg day 3-7 how many follices are normal to produce? Doctor: Hello. Thanks for writing to us. Even if you are producing a single follicle every month, it should be sufficient enough to cause conception. If you are doubtful regarding the rupture of the follicle then you can ask for HCG injections from your doctor which will help in the rupture. I hope this information has been both informative and helpful for you. Regards, Dr. Rakhi Tayal drrakhitayal@gmail.com"
},
{
"id": 116398,
"tgt": "Suggest treatment for high value of rubella antibody",
"src": "Patient: hello doctor, recently i did torch results . they are showing rubella IgG antibodies 146 positive, CMV Igg antibodies positive 114, and HSV IgG antibodies positive 30 . so i am asking that the test results can be this much more and also or can be the test results false also. at present i am not pregnant so i can i get the therapy and evn i have put rubella vaccine then also the test result can how come be positive Doctor: Hi dear.I have gonr through your question. I can understand your concern. You can take treatment for rubella. You can take spiramycin. Since it is a priscription based medicine you should consult your doctoe and take it according to his advise.Thanks for using health care magic.Wish you a very good health."
},
{
"id": 184953,
"tgt": "What causes tightening at the tip of the tongue like muscle spasm?",
"src": "Patient: Hi. I'm female, age 40, 5'6\" and 128 lb. I moved from a northern region of the U.S. to a southern region 1 1/2 weeks ago, and yesterday began having a strange symptom -- the tip of my tongue keeps tightening, like a muscle spasm, but no pain. Makes speaking very difficult! It happens several times throughout the day -- is this something I should be worried about? Thanks and kindest regards, KK Doctor: Thanks for your query, I have gone through your query. The tightening of the tongue can occur following healing process of the ulcer or any wound or in case of oral submucous fibrosis. consult a oral physician and get these things ruled out. if it is a healing process then nothin to worry, if it is OSMF then it has to be treated.i hope my answer will help you. take care."
},
{
"id": 192011,
"tgt": "Suggest treatment for high blood sugar level",
"src": "Patient: I just got a surprise reading of a blood sugar of 441, I am a diabetic, and I have ran out of my Januvia meds, which I have not taken in several days, because I get that medicine from CVS mail order. Is this cause for alarm , and how can I lower it fast? Doctor: Dear Sir, greeting s from HCM.. I understand your concern.. NEVER STOP MEDICATIONS FOR BP AND DIABETES ... it could lead to lots of complications in future ... Try to get the medications from some were and start taking soon ... In time .. Please check your lipid profile , microalbuminuria, HbA1c levels and do contact plus with the reports .... Anything else please feel free to contact us ... THANK YOU"
},
{
"id": 63879,
"tgt": "What causes hard lump on sternocleidomastoid muscle and behind my ear?",
"src": "Patient: hi im 22 years old and I have a lump on my sternocleidomastoid muscle, its golfball sized and seems hard. I have a few others, one that s behind my ear and the area around my clavicle feels very swollen. This all came on within days. I don t know what to do. Doctor: Hi, dearI have gone through your question. I can understand your concern.You may have enlarged lymphnodes. It can be due to reactive hyperplasia, tuberculosis, lymphoma or metastatic carcinoma. You should go for fine needle aspiration cytology or biopsy of that lump. It will give you exact diagnosis. Then you should take treatment accordingly. Hope I have answered your question, if you have any doubts then contact me at bit.ly/Drsanghvihardik, I will be happy to answer you.Thanks for using health care magic.Wish you a very good health"
},
{
"id": 191531,
"tgt": "What causes reduced blood pressure level while on Metformin 1000?",
"src": "Patient: 74 year old black male np 98/74 pulse 72 6 feet 2inches weight 190 non drinker/smoker eat mostly veggies little or no meat no salt little or no sweets exercise cardio walk one hour five days weekly, stretch pushups seven days weekly worked 56 years retired two years ago 22 yrs. Military Viet Nam vet feel great. Problem too low, no no meds take metformin 1000 mg dailyzocor20 mg daily aspirin 81 mg problem no too low Doctor: Hello, Thanks for the query.I have gone through the details given. Your BP reading of 98/74 mm Hg, shows a slightly lower systolic BP. But it can not be balled as as low BP. It is at the lower end of the normal BP range.Metformin 1000 mg or higher has no effect on BP. Its major side effects are related to gastrointestinal disturbances, that too when taken on empty stomach. You should consume more of liquids and plenty of water. Unless you have any specific symptoms due to so called low systolic BP, there is no reason to worry. Thanks"
},
{
"id": 73121,
"tgt": "Suggest remedy for dry cough",
"src": "Patient: I have had a dry tickly cough for over a week. It is getting more productive. But about 5 days ago a experienced sudden pain in my lower ribs on the right side. Now it has travelled up to below my breast still on my right side and still painful in the original spot. Should I worry? Or did I just strain myself? Doctor: Thanks for your question on Healthcare Magic.I can understand your concern. In my opinion, we should definitely rule out lung infection in your case because you're have developed chest pain and worsening cough. So get done chest x ray. If this is normal then no need to worry for lung infection. Possibility of musculoskeletal pain is more likely due to coughing. So take levocetrizine 5 mg twice daily for coughing. Apply warm water pad on affected areas of chest. Take simple painkiller like paracetamol or ibuprofen. Avoid movements causing pain. You will mostly improve with all these in 5 days.Hope I have solved your query. I will be happy to help you further. Wish you good health. Thanks."
},
{
"id": 12463,
"tgt": "How to treat psoriasis?",
"src": "Patient: hello Sir..i have been diagonised to have psoriosis and i have been given a tablet Melanocyl. when i browsed in the internet, there are few sites that says it has side effects. is it advisable to take that tablet or should i go for another?kindly advice.Thanks Doctor: Hello and welcome to healthcaremagicPsoriasis is an inflammatory skin disease characterised by well defined erythematous scaly plaques with silvery white or candle wax type of scaling. Chronic plaque psoriasis is the most common type. It can affect sites like scalp, arms, and legs, (specifically elbows and knees), palms and soles etcThe disease has a chronic course and there topical creams/ointments/gels/lotions etc as well as oral and injectable medicines which can treat psoriasis but there is no permanent cure for psoriasis.Melanocyl is an oral Psoralen, 8-Methoxypsoralen, which is taken at least 2 hours prior to sun-exposure OR UV A exposure in a chamber. It is a very effective drug for treatment of psoriasis along with UV A exposure known as photchemotherapy and has been used for years. I would suggest that you continue with it.However, it can cause minor side effects mostly related to GI System: Nausea, vomiting, Elevated Liver enzymes etc,Ocular side effects like: Cataract (can be avoided by covering the eyes during UV A exposure)Cutaneous side effects like: Pruritus, Solar lentigines and rarely serious side effects Skin cancer (Squamous cell carcinoma) etc. Skin cancer is rare and depends on the total cummulative UV A exposure (with more than 200Joules lifetime exposure increasing the chances of cancer development). The risk of skin cancer is reduced by simultaneous use of oral retinoid:acitretin.regards"
},
{
"id": 136201,
"tgt": "Suggest treatment for the crooked toes",
"src": "Patient: I need to find a very great surgeon for my crooked toes, meaning bunions hammer toes on both feet. the toes overlap each other, very crooked. They are very painful and it is getting very difficult to wear shoes. I live in Birmingham, Al. I am 69 years old and I have lived with this condition for years. I am using my daughters email address. Sorry did not expect to pay for an answer. Forget I ask Doctor: hiVisit a hospital for podiatrist or orthopedic surgeon specialized in foot corrective surgery.It has alredy been neglected for long, but some corrective surgery may be helpful.Best wishes"
},
{
"id": 47301,
"tgt": "Suggest treatment for kidney stone",
"src": "Patient: my mother in law is suffering from lots of pain in her kid,vomitings, and motions.... when we went to doctr he sadi that problm iz beecause of the stone which is struck at down part of the kidny......the pain un brble.plz sugst smthing for pain relif Doctor: HelloThanks for query .Your mother in law has severe pain in abdomen ,vomiting due to impacted stone in lower ureter .Please consult qualified Urologist for clinical examination and get following basic tests done to confirm the diagnosis.1) Urine routine and culture.2) Ultrasound scanning of abdomen and pelvis.Give her following treatment for a week 1) Antibiotics to prevent infection 2) Tab Tomsulosin once daily which helps to dilate the ureter and thereby helps to pass down the stone in to bladder .3) Dratovarine twice daily has similar action like Tomsulosin 4) Pain killers like Cyclopam twice daily .5) Plenty of fluids orally or I.V fluids to produce more urine If these measure fail to pass down the stone within a week ,she may need to undergo an endoscopic surgery of URS to remove the stone from ureter Dr.Patil...Further treatment will depend upon result of these tests and final diagnosis"
},
{
"id": 173116,
"tgt": "What is the cause of dark circles around the eyes and red pimples on the face?",
"src": "Patient: My 1 yr old girl has very dark circles around her eyes and red pimples around her eyes,mouth and on her cheeks. The pimples don t have a head and seem to disappear after a day and reappear somewhere else. She sleeps for approx 12/13 hrs a night and has 2 x2hr naps in the day. Should I take her to see a doctor? Doctor: Hi...Skin conditions are best diagnosed only after seeing directly. I suggest you to upload photographs of the same on this website, so that I can guide you scientifically. Hope my answer was helpful for you. I am happy to help any time. Further clarifications and consultations on Health care magic are welcome. If you do not have any clarifications, you can close the discussion and rate the answer. Wish your kid good health.Dr. Sumanth MBBS., DCH., DNB (Paed).,"
},
{
"id": 40390,
"tgt": "Is penicillin recommended after a positive result on strep test?",
"src": "Patient: Hi, I did a strep test on Saturday and it is positive. Do I absolutely need to take the penicillin that was prescribed? I've already got celiac disease and am on synthroid and Lamictal so not feeling like adding another medication to the mix. thanks. Shelley Doctor: HIThank for asking to HCMThe penicillin still is golden therapy for certain bacterial infection like streptococcus, if this is the real situation then you have to take the penicillin for better result, hope this information helps you take care and have good time."
},
{
"id": 165441,
"tgt": "What causes stuffy nose and red eyes in child after having Azithromycin?",
"src": "Patient: 6 yr old son had fever 103. bad headaches & strep throat. Went to doctor put him on 5 days of azithromycin which he did finish. Now he has a stuffy nose & blood shot eyes. Called doctor doc said sounds like allergies. Child acting fine but I don t like the blood shot eyes. Doctor: Hello,Stuffy nose and red eyes are unusual adverse effects of Azithromycin. As you said 5 days course is complete you should now stop Azithromycin. For stuffy nose you may use antiallergic decongestant combination. For red eyes, you may use lubricant eye drops.Hope I have answered your query. Let me know if I can assist you further.Regards,Dr. Khan Shoeb Mohammad Sher Mohammad"
},
{
"id": 164555,
"tgt": "Suggest treatment for lower back pain and joints popping in kids",
"src": "Patient: My 9 year old daughter has lower back pain every night. We took her to the dr. in October 2010 and was told that her lower lumbar curved in just a little, but not enough to be worried about. Her back, knees, and ankles have started popping quite often, and are painful to her. Why is her back hurting so much, and why are her joints popping? Doctor: do thyroid profile and start physiotherapy for the back ache.. mostly in hyperthyroidism or hypothyroidism ,you will get like this symptoms.. go to your pediatrician and have a thyroid profile done"
},
{
"id": 197281,
"tgt": "Why CCQ 50 and Quantus capsules are prescribed for men?",
"src": "Patient: In my husb semen analysis report sperm count is 22.4 mil/Ml & in sperm class,Class A its 12.62%, 2.83Million/Ml.So doc advised him to take Tab CCQ 50 & Cap Quantus.But my concern is when i serached the net its content is clomiphene which is usually given to females to induce Ovulation.So can males too can take the above mentioned tab? Doctor: HelloThanks for query .You are right in saying that the Clomiphene is prescribed to females to induce ovulation .But it has been observed clinically that it does help to increase sperm count and motility of sperms hence being prescribed to males to increase sperm count .Dr.Patil."
},
{
"id": 39012,
"tgt": "What does \"bilateral hilar and basal bronchial congestion seen\" mean?",
"src": "Patient: I m ramesh, age 39 years, ht. 167 cm, wt 50 kg - in my x-ray of chest PA it is mentioned that 'there are bilateral hilar and basal bronchial congestion seen' what is it? Also in my CBC report, lymphocytes and eosinophils are shown high (47 and 7 respectively) any relation between these reports? Please reply. Doctor: Hello Ramesh There are bilateral hilar and basal bronchial congestion i.e. bronchoconstriction or bronchospasm in the portion of lungs ( bilateral or both lungs ) from where blood vessels, lymph nodes enter and leaves i.e mid portion of lungs .There is also congestion of bronchi in the base of lungs . This type of congestion occurs due to allergy to certain allergens , these are ,pollen , mite , dust , fine dusting powder , hay , fodder, sudden increase or decrease of temperature , certain drugs.These allergen produces bronchoconstriction ( as in your case on both lungs and in the base of lungs ( bilateral hilar and basal congestion ) and this bronchoconstriction causes bronchospasm , hence there develop cough , bronchitis etc.Now about the C B C report ) clearly mentioned that eosinophils count is high ( normal 2-5 %) , and this is due to allergy and bronchial congestion.Lymphocytes ( normal 20-40 -45 ) . This is non-specific , so don't bother about that . In my opinion you are having allergic bronchitis , so consult your physician and get his opinion.Good luck."
},
{
"id": 164167,
"tgt": "Suggest cure for constant chest pains",
"src": "Patient: My son is 8 years old and has been experiencing chest pain for 1 month not related to exercise. His ecg shows inverted twaves - 1 doctor said this is ok,the other was concerned. Is this growing pains or should I take him to a paediatrician. Thanks Karen Doctor: Hi.... Inverted T waves are very common in paediatric age group and are called Juvenile T wave pattern. This is usually seen in Leads V1 to V3.If the child is not having any loss of consciousness or palpitations or acute shortness of breath and breathlessness you need not worry about them at all.Regards - Dr. Sumanth"
},
{
"id": 60114,
"tgt": "Continuously positive HbsAg, want to know measures to overcome this. Advice?",
"src": "Patient: hi doctor my hbsag is positive since 3 years but i need it negative as soon as possible wat can i do an since 1 year i had no blood tests plzz help me i cant travel bcoz of this.... plzz doctor plzz help me wat are the precautions an wat should i eat ... i am very much upset how much more will i live .... will i die sooner becoz of this please doctor tel me Doctor: are you talking for HIV"
},
{
"id": 180385,
"tgt": "How can oral thrush be treated despite taking Nystatin suspension?",
"src": "Patient: I have thrush and have been taking nystatin oral suspension 5 ml 4 times a day. I am on my second dose of the medication and still have thrush. It is better but not gone. I have one more day of the dose. Should I be getting a different medication to clear up the tongue area? Doctor: Hello dear. Welcome to Healthcaremagic. I read your query and I understand your concern. You should finish the medication as prescribed by the doctor. Use it even the next day then wait for the result. The most probably it will be cured. If not ask again your doctor. Take care. Hope my answer was helpful, if you have any other questions I'll be glad to help you. Best regards. Dr.Olgeta Xhufka ( general and family physician )."
},
{
"id": 151850,
"tgt": "Can a osteoporosis patient having severe asthma undergo a surgery for multilevel degenerative disc disease after a bad fall ?",
"src": "Patient: MY MOTHER IS 52 YEARS OLD WITH SEVER OSTEOPROSIS. SHE IS ON STEROIDS FOR ABOUT 5 YEARS FOR ASTHMA . HER ASTHMA IS SEVERE. A MONTH AGO SHE FELL AND HAVE SEVERE PAIN DUE TO DISC COMPRESSION. SHE HAVE CONSTANT BURNING SENSATION IN BOTH HER FEET AND SHE IS ON HEAVY PAIN MEDICATION. NEURO SURGEONS WE HAVE CONSULTED SAY THAT SHE IS UNFIT FOR SURGERY AND ONLY PHYSIO THERAPY AND TIME WILL HELP HER RECOVER. THE MRI REPORT SAYS MULTILEVEL DEGENERATIVE DISC DISEASE INVOLVING L3-L4 AND L4-L5 LEVEL. AT L3-L4 LEVEL, RIGHT PARACENTRAL DISC PROTRUSION NOTED CAUSING RIGHT LATERAL RECESS NARROWING. AT L4-L5 LEVEL, RIGHT LATERAL DISC PROTRUSION NOTED CAUSING RIGHT LATERAL RECESS NARROWING AND RIGHT RADICULAR COMPRESSION. ON THE LEFT SIDE, THERE IS ENCROACHMENT OF LEFT NEURAL FORAMINA WITHOUT ANY RADICULAR COMPRESSION. IMPORTANT. I AM I PAKISTAN SO CONSIDERING THAT A NEURO SURGEON MIGHT SEE 20 TO 30 PATIENTS IN THE 2 HOURS CONSULTING I AM NOT SURE IF THEY ARE TAKING THEIR TIME TO EVALUATE MY MOTHERS CONDITION PROPERLY. SHE IS TAKING MULTIPLE PAIN MANAGEMENT MEDICATION INCLUDING OPIOIDS AND STILL CANT WALK MORE THEN A FEW STEPS AT A TIME AND CANT SIT. THE BURNING SENSATION IS CONSTANT AND I AM WORRIED THAT MIGHT LEAD TO SOMETHING WORSE IN THE FUTURE. I NEED SOME ADVICE AND HELP. THANKYOU Doctor: Hi Noman, Your Mother seems to be suffering from complications of degenerative disc disease. Since she in on steroids do make sure she is taking Calcium and Vitamin D3 supplements to help prevent progression of Osteoporosis at her age. The pain she experiences is due to nerve getting pressed by the degenerated and herniated discs from her spine. She can try using a Lumbo-sacral corset belt to attain better posture while walking and reduce her symptoms. Usually, the main exercise you will be asked to do is that of extension which is leaning backwards by propping yourself up on your elbows when lying on your front. This mobilises the joints but more importantly closes the area of the prolapse and reduces the disc prolapse. If the disc prolapse is reduced the tear in the disc can heal."
},
{
"id": 118517,
"tgt": "Blood report shows cholesterol 235, HDL 36, LDL 139, SGOT 50, SGPT 67. Advice to control cholesterol and liver disorder",
"src": "Patient: My blood reports are asUric Acid 7.1Cholesterol 235HDL 36LDL 139Triglyceride 522Total Billionaire 0.9SGOT (AST) 50SGPT (ALT) 67Alkaline Phosphates 67Kindle advise diet and medicines how to control Cholesterol and Lever , plz also tell me either lever readings are harm full Doctor: Hello.Elevations of liver enzymes(SGOT and SGPT) and cholesterol are not worrisome,but triglyceride levels probably should be treated with medication by mouth (fenofibrate is the most commonly used drug for this condition).About the diet you must avoid intake of alcohol and fat products (bakery products, fried foods,pastries, red meat).Please consult your local doctor for a prescription.I wish you good health.(If the answer has helped you, please indicate this)"
},
{
"id": 89857,
"tgt": "Should i be worried about pain in abdomen after an injury?",
"src": "Patient: my daughter was in art 9 days ago and was sanding a board...it did a kick back and hit her in the lower left side of her abdomin....she is still complaining of pain. But she was walking all day yesterday and then had a 5 year old head butt her in that area today and now she is complaining...should I be overly concerned Doctor: I advise you take her to a hospital and get an simple ultrasound scan done which should ruleout any injury to the internal organs. That should help alleviate your concern."
},
{
"id": 97730,
"tgt": "What is the remedy for the fluid from the anus when on homeopathy treatment and antacid?",
"src": "Patient: I have acute acidity problem for last two three years, previously I also suffering from acidity but not so acute. I have also suffering from insomonia but not regularly, some type of fluid is comming out from anus for last two years. Somewhat it has been control by the Homeopathy treatment.Kindly advise me what should I do? I have to take antacid tablet off and on to reduce the burning sensation in the chest. Regards, Debasis Sarkar Burdwan West bengal India Doctor: **1. since type of fluid is not mentioned [mucous/purulent/watery/fatty] thus it is recommended to go for an expert opinion with a General Surgeon to rule out any Rectal Pathology [irrespective of cause and outcome]2. Reduce anxiety,eliminate sugar from diet,no caffeine, plenty [2 litres or more] water per day, fiber rich diet,Increased fibre rich diet, probiotics [medicine and yogurt].3. regarding your Acidity problem:i. Avoid food which take more time to digest or irritate stomach mucosa like: cabbage/ sprouts, onions, asparagus, pears, apples, peaches,chewing gum,milk products,fruit juice,vinegar,alcohol, caffeine,greasy-spicy-refined foods,Avoid oily food, aerated drinks, and over eating, instead take regular moderate meals.Avoid swallowing of air i.e. sipping tea, chewing gum, aerated drinks.ii. Eat slowly, avoid haste, chew properly.iii. Avoid onions, peas, beans.iv. Close the mouth while belchingv.. Abdominal exercises to improve the tone of abdominal muscles: straight leg raising, sit ups, walking, joggingvi..eating a little less than demanded by appetite is important."
},
{
"id": 86842,
"tgt": "What does the following test result indicate?",
"src": "Patient: I have sever lower adominal pain/pelvic pain. I was in the ER for it about two weeks ago. They did a CT Scan and an ultrasound... I have the results, but I can't get my doctor to call me back to explain them. It say very heterogeneous soft tissue mass posterior to the bladder, which measured about 5.5cm, and in long axis 5.2x5.8cm. What does this mean? Doctor: Hello Heterogeneous soft tissue mass posterior to the bladder may be due to many reasons like pelvic pathology,bowel origin mass etc.It is important to know your age and sex.You condition need contrast CT scan of abdomen.CT is needed to assess origin,extent,invasion to surrounding structures.You may need further investigations lie tumour markers,FNAC depending upon CT scan findings.Proper treatment depend upon clinical findings and investigations reports. Get well soon.Take CareDr.Indu Bhushan"
},
{
"id": 69868,
"tgt": "What could cause the lump on the back of the head?",
"src": "Patient: hi i lump on the back of my head about egg sized its appered out of nowhere about a week ago ihave server headachs but have a consteant pain like someone is pushing down onto my head with force it is cause trouble with my eyesight speech and balance .i haveno idea whatit is Doctor: Hi,welcome to HCM. i understand your query as well the concern.Your history looks like that the lump might be attached with the brain ,because of that you are having speech, vision & balance problem. But it needs to be confirm by USG of the lump to rule out whether it is attached to the brain or just superficial. Then consult any doctor/surgeon near by for further guidance.Good luck,Thanks."
},
{
"id": 215580,
"tgt": "Are Hydrocodone and Clonazepam safe to be used to relieve pain?",
"src": "Patient: Can hydrocodone 5mg 1 or 2 as need for pain and clonazepam 1mg up to 3 a day be used together. I am 74 and have had two major back surgeries after a car wreck in 2009. One was a fusion of L4 and L5 in which two of the four screws are loose and I have to wear AFOs on both legs for foot drop as a result. I have had anxiety all my life. I do not abuse either drug. I have a pacemaker and deliberator that is monitored. I also have a back stimulator. Doctor: Hello, You can safely take clonazepam and hydrocodone together. It will not interact with each other or does not affect the efficacy of one another. Keep in mind that combining these two can cause excessive sleepiness. Hope I have answered your query. Let me know if I can assist you further. Regards, Dr. Shinas Hussain, General & Family Physician"
},
{
"id": 211818,
"tgt": "Taking Venex. Had depression after drinking alcohol. Reason?",
"src": "Patient: Hi, I am on venex xl 75mg for 3 months now and last night i was at a party and had alcohol for the first time since taking this tablet and i had a bit of a melt down and couldn't stop crying. I've been on other antidepressants for the past 10 years and never had any problems like this. I only drink occasionally, maybe 3 or 4 glasses. Why did i react like that?Mary Doctor: Hi, it appears that there is no relation with the alcohol and aurvedic product venex, to stop rectal bleeding, and vericose veins. So you would have taken cocktail, or the brand you have taken dindn't suit you. Take bland foods for few days. Thank you."
},
{
"id": 140117,
"tgt": "What could cause abdominal bloating, diarrhea and vomiting in a 36-year-old schizophrenic patient?",
"src": "Patient: My 36 year old son has constant diahhrea vomits daily, his stomach is hugh. his doctor office called and said his alkaline, neutrophils, are high. My son has schizophrenia and takes 4 medications. We can t get into the doctor for 2 more weeks, should we wait that long? He only eats once a day and its a very small amount he gets full fast so I can t see how his belly got so big??? Doctor: Hello, This enlarged belly seems to be something quite new and abnormal in terms of USUAL body shape and size. It sounds like he is producing an awful lot of gas resulting in ABDOMINAL DISTENTION. One reason could be the presence of H.Pylori which is a common infection in the stomach and is easily tested for and treated with certain antibiotics. The elevated neutrophil count may be signaling some type of infection. The elevated ALP (alkaline phosphatase) levels could indicate some type of liver or gall bladder issue and this could also account for the increased amount of gas being produced in the GI tract. If he has diarrhea and vomits daily and doesn't eat very well then, my concern is for dehydration that can set in quickly. If the doctor's office is not willing to some how get an appointment for him then, I would recommend 2 choices. An URGENT CARE CENTER which may be able to at least make sure he's well hydrated and stays that way until the doctor's appointment OR ER where again they can make a decision whether he needs some immediate treatment with antibiotics, IV's, or some other diagnostic testing to tell what's going on with the liver and gall bladder to start with. Is your son by any chance having trouble with excessive use of alcohol because some of these symptoms could also suggest a form of hepatitis-related to alcohol use? Hope I have answered your query. Let me know if I can assist you further. Take care Regards, Dr Dariush Saghafi, Neurologist"
},
{
"id": 21553,
"tgt": "Suggest opinion on high BP and side effects of Loprin+Extor",
"src": "Patient: Why do i have hi bp? Sir, i am 25 (male) and from last year my doc has prescribed me to take Loprin 75mg + Extorr and Monitor once daily day - Night. My bp was a high i.e. 150/110 etc. My all medical reports are clear at last dr. says that u got Hi B.P. as an inherit....i am totally confused and now iam fadeup of these tablets...as he say that u have to take these whole life. :( is their any side effects of these tablets? Doctor: Hi There I have read the medical details provided by you and I understand your concern for your health. Familial Hypertension is quite common now days and it usually affect young age group. Yes your Blood pressure is high and it needs to be managed. Your doctor has put you on a blood thinner and a combination of two anti hypertensive drugs. I personally don't think there's a need to start you on so many meds so it's advisable that you should personally consult a cardiologist and this regarding this issue.Things I would like to recommend you are1. Start going for daily morning walks 45 mins a day for 5 days a week.2. Low dietary salt intake.3. Avoid junk, canned and deep fried food items.4. Stop smoking completely ( if you do ). 5. Include good amount of fruits and vegetables in your daily routine diet.6. Keep yourself stress free7. Avoid drinking too much of coffee or drinks that contains caffeine as it also increases bp.Follow this routine strictly for 3 months and hopefully you will not need anymore meds to lower your BP. All medicines do have their side effects on long term use.I wish you good health"
},
{
"id": 75566,
"tgt": "What causes chest pain and sweating?",
"src": "Patient: My mother had a massive heart attack 3 years ago, and because of the severity of the damage and other complications she was not able to have surgery and was told that medication was the only way to help her now. For about a week and half now she has been getting pain in her chest and she tells us that it is heartburn. She is now admitted into the hospital, she is getting bad headaches and sweats. What do you think could be the problem and what could we do? Doctor: Thanks for your question on Healthcare Magic. I can understand your concern. In my opinion, we should rule out heart diseases in her case. She was already having heart disease in past and current symptoms like chest pain, sweating etc favour heart diseases more. So get done blood pressure monitoring, ecg, 2d echo and stress test (trade mill test). She may need Coronary angiography if required to confirm heart diseases. On the basis of these reports, she will need cardiac supportive drugs. Don't worry, she will be alright. Hope I have solved your query. I will be happy to help you further. Wish you good health. Thanks."
},
{
"id": 70023,
"tgt": "What is the hard lump on the head?",
"src": "Patient: hello, i have just discovered a small lump on my head at the top of my forehead on the right-hand side. its approximately 1-2 cms in diameter, if i rub it with my finger fairly hard it feels tender. Any ideas ? I'm nearly 44 years of age and dont normally or cuurently experiencing and syptoms of headaches ? thanks for the help if you can .. Doctor: Hi. The most probable reason is usually a sebaceous cyst.This is the commonest on the scalp.Better to get this removed for two reasons One - the disease is removed Two- you get the perfect diagnosis."
},
{
"id": 182698,
"tgt": "Suggest remedy for bad breath",
"src": "Patient: Dear Doctor , i am 18 years old and i am suffering from too much bad breath for last few years.i have consult many doctors regarding the matter but no one able to help me.i have read about bad breath on internet where many people post that one does not no that he has bad breath.but my breath is so bad that i clearly smell it.its too bad.Dear Doctor,i am so much tensed and worried about this extreme bad condition.i am from pakistan.kindly help me and recommend me something so that i could relieve from this problem.regards Doctor: Thanks for your query, I have gone through your query.The possible causes for the bad breath could be the gum infection or deposits, pus discharge. Consult a oral physician and take a opinion to rule out these causes. these causes can be treated by getting your teeth cleaned and maintaining oral hygiene.If these causes are not there then, it can be secondary to respiratory disorders like sinusitis or gastrointestinal disorders. Consult a ENT and Gastroenterologist to rule out these causes.Also get your blood investigations done particularly, blood sugar level, because even in diabetes you can have halitosis.I hope my answer will help you, take care."
},
{
"id": 66118,
"tgt": "What could be the ball of tissue on the side of my cheeks?",
"src": "Patient: I have a small ball of tissue on the side of my cheek, near the vertex of my upper and lower lip. It hurts to stretch my mouth or to touch the ball of tissue - been like this for about a week. There is no sign of the lump from looking inside, but when I stretch my lips taut, to the side, the ball of tissue is outlined. Doctor: Hi, dearI have gone through your question. I can understand your concern. You may have some growth. You should go for biopsy to search for thr cause and rule out malignancy. Consult your doctor and take treatment according to yoir diagnosis. Hope I have answered your question, if you have doubt then I will be happy to answer. Thanks for using health care magic. Wish you a very good health."
},
{
"id": 37933,
"tgt": "What causes fever,raised heart beat and weakness after getting a tattoo?",
"src": "Patient: I just had a 5 1/2 tattoo completed on my right shinbone.I am no stranger to tattoos, I have plenty. However, I feel very weak, shakey, and very sleepy. I am also running a fever. I don t remember the last time I ran a fever. I feel very weak and space. My heart is beating out of my chest. But I can t move.. what is happening? YYYY@YYYY (no R in southengirl1975@yahoo) Please help, Amy Weiss Doctor: HI, thanks for using healthcare magicFever, fatigue and weakness are symptoms of infection. This may or may not be related to your recent tattoo.If the tattoo area appears infected- warm, tender, increased redness, pain- then it is possible that there is a connected and antibiotic treatment may be neededIf there is no indication of infection in this area then the symptoms have another source.You may need to visit your doctor for an assessment and treatmentI hope this helps"
},
{
"id": 51063,
"tgt": "Kidney malrotated, blood in urine and had gallbladder removed. Treatment?",
"src": "Patient: Hello My wife age 33 years gone through surgery of Gall Balder which was removed about more then 10 years, three children all of them through surgery got some blood in urine and go for US the report of us speaks that everything is normal except the right kidney slightly malrotated , no hydronephrosis or nepherlithiasis on either side, I want to ask you is it some how dangerous for her remaining life and what is the treatment available for this abnormality. thanks Muhammad Tayyub +0000 YYYY@YYYY Doctor: Hi Thanks for the query. If your wife has evidence of blood in the urine we need to see the urine microscopy to look for white cells or casts. We also need to need to know whether there is evidence of protein in urine. If any of these are present she needs to see a nephrologist. A malrotated kidney is harmless and should not be the cause of blood in urine. Hope this helps Good luck."
},
{
"id": 30442,
"tgt": "Suggest cure for filaria",
"src": "Patient: a female 62years old,suferring from filaria since 6 to 7 years.chief complains are-swelling whole body,pain, oozing is stop now ,dyspnoea, ,unable to walk,but when oozing stop the condition of patient worse with fever,n etc. now she is not taking any treatment .plz suggest us what should we do? Doctor: Hello, Thanks for asking the question. You will need to get a test for microfilarial worm, which is done at night after 12 am as it enters the blood during that time. You might need to use Hetrazan drug which treats filarial worm for about 3 weeks. Please consult your doctor for more information."
},
{
"id": 146438,
"tgt": "How can a pilonidal cyst be treated?",
"src": "Patient: I have been diagnosed with neuropathy, Parkinson s and after seven MRI s my neurologist tells me that I have something wrong, but he does not know what it is. Now I have a sympton that I think may be a clue. For years I have had a cyst or fistula on my lower spine that opens and drains periodically. A few days ago, it opened and my symptoms began to subside and improved. The doctors so not see a connection here because I guess nothing has shown upon the MRI s. It as been called a palodinal (sp) cyst, but there is no hair or other debree noticeable -- just clear or pink fluid. I have moved w/in the past couple of years and have not found the right doctors. There seems to be no team effort w/in the system that I am using. I do not think the doctors talk to each other at all! The new internist told me that I need to see a surgeon, but suggested that I go back to the one I used who is 130 miles away. I am not able to drive that far because I can hardly walk w/out a cane or walker. Doctor: Hi, I had gone through your question and understand your concerns. A pilonidal cyst or sinus is a cyst in the lowest part of column or in tailbone area. It may contain inside hair or skin remnants. Treatment is surgery under local anesthesia. Usualy there is no connection between the cyst and neurological symptoms. However there are isolated cases reported of sacral or lumbar osteomyelitis and epidural abscess as a complication of pilonidal cyst. I suggest you to discus with your Doctor the possibility of plain radiographs of your lumbosacral column. I also suggest to treat the cyst surgically, it is a simple procedure under local anesthesia. Hope this answers your question. If you have additional questions or follow up questions then please do not hesitate in writing to us. I will be happy to answer your questions."
},
{
"id": 190316,
"tgt": "Pimple in the mouth, hurting. How can it be cured effectively?",
"src": "Patient: Hello I woke this morning with a big pimple on my mouth and its really hurting me I have a huge dinner today is there any possibilty that I can make it smaller or get rid of it please its an emergency Doctor: Hello Welcome to HCM, I just went through your problem & it is known as Apthous ulcers & they are really painful. Common Causes can be:- -Improper food habits like taking much of spicy food -Gastric upset -Stress problem -Hormonal changes -Nutritional deficency -Improper Oral hygiene -Lastly if its reccuring then it can be due to some sharp cusp. What all you need to do is improve her dietary habits like include more of green vegetables & less oily & spicy food. Also start with Tenvit-Plus capsules along with application of some anaesthetic gel like dologel or Mucopain gel. You can also go for tetracycline mouthwash. All these will definitely help you out. Take Care Regards Dr.Neha"
},
{
"id": 210194,
"tgt": "Does Xanax have any side effects?",
"src": "Patient: Is Xanax being taken off the market and why are MD's changing pts to clonaphen also I don't take Xanax everyday. I currently have several prescriptions that my MD has given me that I never filled. They are even out of date now. I'm concerned with the side effects with the other med. I have many problems with allergic reactions to different medications. Xanax does not cause me any problems. What is your opinion on this. Thanks in advance. Doctor: HIThanks for using healthcare magicXanax and klonopin contain same medicine that is clonazepam, so there is nothing to worry about it. Both drugs are available in market, but sometime, psychiatrist or physician prescribed drug according to the availability in stock. That may be the reason of changing brand. If you are facing problem with klonopin, though it has same medicine, you can consult your physician.Thanks"
},
{
"id": 56482,
"tgt": "What does the liver profile with SGPT 240 indicate?",
"src": "Patient: liver profile sgpt/alt at 204, six month ago last checkup was at 25 sgot/ast at 80, last checkup was at 23 I am 46. Should i be concerned or what could be wrong to have this spike, lately I have ate lot of durians(a hot tropical fruit). Thank you. Doctor: Welcome to HCM!I appreciate your concern for the isolated increase in SGPT.Incomplete data like do you have any problem or symptoms that made you to undergo this investigations? Because elevated SGPT level has to be co related to your illness if you have.Durian is a topical fruit cultivated in most of the South Asian countries, it is found to have several benefits. Eaten alone it is unlikely to be harmful.If you have underlying problem like fatty liver or if you take alcohol then you must stop taking alcohol, eat healthy food containing fresh fruits and vegetables as they contain good amount of anti-oxidants which are required to preventing the fatty liver.Take care of your health."
},
{
"id": 90934,
"tgt": "What causes low abdominal pains with all the tests being normal?",
"src": "Patient: I have low abdominal pains. They are mild-medium about a 4 on a scale of 1-10. I have a high tolerance for pain. The pain is not constant, it occurs when I get up from a prone or sitting position and when I lean forward. I have had a Colonostopy, a GED, an x-ray and a CT of the pelvic and abdomen with and without contrast. My blood test have been normal. Doctor: Hi.Thanks for your query and an elucidate history.If all the tests are normal , means there is least chance of the problems which could have been detected by the tests you have undergone through. Your pain is:Intermittent, it occurs when you get up from a prone or sitting position and when you lean forward.A good clinical examination by a Surgeon with proper physical examination can definitely get the diagnosis. From your history , I think you have a problem with abdominal wall , which can be sprained muscles, hernia or pinched nerves."
},
{
"id": 81524,
"tgt": "What causes spitting up blood in the early morning post alcohol intake?",
"src": "Patient: I have been throwing/spitting up blood for few month in the morning specially after long night of drinking beer, I have heat flashs during sleep, after I drink fluid in the morning it sitts in my stomach and after I smoke a cig if I get up I get dizzy and will spit acid with mucous, acud and followed by spitting blood..and throwing up whatever fluid I consumed, and if stay up it will be fine if I go to bed and wake up in few hours it will reoccur... Doctor: Thanks for your question on HCM.Since you are alcoholic and smoker, spitting of blood can originated from either lungs or stomach.Smokers damage their lungs and can develop bronchitis or infection or malignancy. And coughing of blood (hemoptysis) can be seen in all.Similarly alcohol damage liver and stomach. Can cause gastric ulcer and cirrhosis. And vomiting of blood (hemetemesis) can be seen in both.So better to consult doctor and get done1. Chest x ray2. Ultrasound examination of abdomen3. Upper GI Scopy.And quit smoking and alcohol as soon as possible. Both are not good for health. They will only harm you."
},
{
"id": 67749,
"tgt": "Suggest treatment for lump on the neck after treated melanoma cancer",
"src": "Patient: Good Day Dr Krueger, I am a bit concerned about my husband he had melanome cancer about 5 years ago. Had operation, mule removed and had chemo in the form of an injection he got 3 times a week for 18 months. He went for test SB100 and it was said he is fine last followed up a year ago. About 3 weeks ago he became aware of a lump at the side of his neck in the vicinity of the lower jaw. We live in a very small town and going to see the GP tomorrow but how can we make sure that the necessary test is done to make sure there is no gland or any other serious problem. His oncologist was in Pretoria but he is not registered as a oncology patient at the medical aid. Please give me an idea what is the next step to follow. Is a SB100 test reliable to be sure. In fact we told the GP this was the test done and according to that he did the test. Thanks I am a very worried wife and do appreciate your assistance. Rina Swart Doctor: Hi, dearI have gone through your question. I can understand your concern. Your husband had melanoma in past. Now there is a lumpIin his neck. Melanoma is highly malignant tumor and known for recurrence. He has high chance of recurrence of melanoma. You should go for biopsy of that lump. Go for histopathological examination and if needed go for marker study like HMB 45 and S100.Consult your doctor and plan accordingly. Hope I have answered your question, if you have doubt then I will be happy to answer. Thanks for using health care magic. Wish you a very good health."
},
{
"id": 32970,
"tgt": "What is the treatment for severe cold and cough?",
"src": "Patient: First about 3 days ago I had a super stuffy nose and thought a simple head cold was my problem. a day or so later my nose cleared up and breathing was no issue. Then, a day later, my throat began to hurt so bad. I have no cough, or sneeze, but I am full of multicolored mucus. When it comes from my nasal passage it is yellow, sometimes with a small amount of redness, and from my throat it is usually a light brown, sometimes a bit bloody as well. this is only the second day, and it seems to be getting worse. whats up with me? Doctor: Hi,The multicolored mucus especially yellow indicates that your bodys defense has taken well care about the infection. You need not worry as I find no fever in your history. I presume you are violently clearing your nose and throat secretions which might have injured some mucosa resulting in a twinge of mixed blood which is self limiting. However, if the bleeding persists, you need to seek a personal medical attention. Steam Inhalation is the best home remedy and the viral infection shall subside on its own. Hope this helps."
},
{
"id": 188177,
"tgt": "Rabies injection taken along with antibiotics after bitten by dog. Any need to take supplementary injections?",
"src": "Patient: Doctor, my boyfriend was bitten by a dog a month ago. Upon the accident, I took him to the clinic where the wound was disinfected and he was give the rabbies injection. I do not have the name of the injection. The doctor also prescribed antibiotic which he took correctly and finished it. We are wondering, does he need supplementary injections to avoid rabbies? Are there follow up treatments that we should follow. Thank you for your answer. Doctor: Hello, thank you for sharing you problem with us.It is good that you visited the doctor immediately where the wound was disinfected this is the first thing that should be done post exposure of the bite.After this there are 4 doses of vaccine-First on the same day-Second on the 3rd day-Third on the 7th day-Fourth on the 14 dayI think it will be better that you should visit that doctor and get confirmed with the doses."
},
{
"id": 199705,
"tgt": "Any suggestion to increase active motile sperms and reduce pus cells?",
"src": "Patient: COLOUR- greyish white, volume-2.0 ml, PH- 7.8, REACTION- ALKALINE, VISCOSITY-NORMAL, TOTAL SPERM COUNT - 76.0 MOTILITYACTIVE MOTILE - 0.5%SLUGGISH MOTILE - 30%NON MOTILE - 65%PUS CELLS -3-4 WE HAVE NO CHILD PLEASE SAY HOW TO INCREASE ACTIVE MOTILE AND HOW TO REDUCE PUS CELLS AND PLEASE HELP ME Doctor: Hello Your semen analysis findings may indicate semen infection with low sperm motility.Infection is indicated by presence of few pus cells in semen.You may need investigations like routine hemogram,random blood sugar,semen culture and sensitivity.You may need to take antibiotics based on culture and sensitivity report.Motility should be at least 55 %.Your findings suggests over all 30.5% overall motility,so it is low.Sperm count is normal.It should be at least 20 millions/ml.Your findings suggests 76 millions/ml,so it is normal.Fertility is expected to increase with control of infection.If findings persists then you may need additional fertilization techniques like IUI.You should take nutritious diet and do exercise.Get well soon. Take Care Dr.Indu Bhushan"
},
{
"id": 70131,
"tgt": "What is the treatment for a lump in the thigh?",
"src": "Patient: hello, im 15 and i have this internal bump on the top of my thigh a little lower than my bikini line and its invisible but when you touch it and rub it its painful. ive only had it for about a week or so but theres no irritation or redness. please help! Doctor: Hi! Good evening. I am Dr Shareef answering your query.If I were your doctor I would have gone for an ultrasound and an MRI if needed t see in which plane exactly the swelling is located. The most likely diagnosis would be a femoral hernia, or an inguinal lymphadenitis or a simple sebaceous cyst. However these diagnosis is more of a physical examination diagnosis rather than by investigations. Therefore, I would advise you to please consult a general surgeon and get it reviewed by him for a definitive diagnosis and treatment.I hope this information would help you in discussing with your family physician/treating doctor in further management of your problem. Please do not hesitate to ask in case of any further doubts.Thanks for choosing health care magic to clear doubts on your health problems. Wishing you an early recovery. Dr Shareef."
},
{
"id": 220365,
"tgt": "What are the early signs and symptoms of pregnancy?",
"src": "Patient: I am 29 yrs old last period 6 wks ago, blocked tubes not by choice, never been more than 5 days late, neg hpt, nausea, bloating, lowers abdomen distended and hard, uterus and cervix are very low in vaginal cana, very fatigued, sore breasts. what could cause this? Doctor: Hi, Dr Purushottam Neurgaonkar here. I welcome you to HCM VIRTUAL CLINIC. I have gone through your question, and I think I have understood your concern. I will suggest you the best possible treatment options. As there is history of tying up of tubes, and urine test is negative.;please do not consider possibility of pregnancy. It can be a delayed period and you can wait until the cycle starts or you can induce the periods with Tab Regestrone 5 mg twice a day for 5 days. Most of the symptoms that you are getting can be due to pre menstrual syndrome .I will suggest you to indulge in a healthy diet and regular exercise regimen. Include more portion of fruits, salads and vegetables in daily diet. Drink plenty of water in a day. Avoid deep fried foods, bakery products and refined sugars. Use of Tablet Vitamin E, Long acting form of Vitamin B 6 -B LONG F and Capsule PRIMOSA each one daily for 3 months will be of help. I hope this answer helps you. Thanks Dr Purushottam Neurgaonkar."
},
{
"id": 86038,
"tgt": "What causes lower abdominal cramps during a bowel movement?",
"src": "Patient: Hello, I have recently had lower stomach cramping while having a bowel movement and losing all color in my face, becoming very hot, extremely light headed and almost passing out. Can you please tell me what this might be? There was no diarrhea or vomiting. Thank you. Doctor: Hello and Welcome to \u2018Ask A Doctor\u2019 service. I have reviewed your query and here is my advice. * The presentation of narrated symptoms suggest that there can be various reasons for the same as: - Straining due to hard stool - Inadequate evacuation - Associated heart problem which show up particularly during this type of activity as inducible ischemia or others - Bleeding in stool - Other systemic causes. * I recommend to visit an ER without delay for the diagnosis of the same and prevent future such episodes. Hope I have answered your query. Let me know if I can assist you further."
},
{
"id": 113751,
"tgt": "Back pain, suspicion of nerve issue and muscle damage. No relief with hot fomentation, painkillers, physiotherapy. How can I relieve pain?",
"src": "Patient: Hi, I have a query about weakness of nerves. I have been suffering from back pain for the past 2 weeks. I met an ortho. He diagnosed and prescribed medication for pain relief and a 5 day physiotherapy session. I am done with my physiotherapy sessions. The physio initially said, the pain might be because I might have damaged my muscle badly. I did not get hurt. Neither did I involve in some straining physical activity. I did not have any relief from the physiotherapy sessions. Later on the 5 day of my session, the physio said, it might be some nerve issue. He wasn t very sure of it. I was asked to use hot bag for pain relief. I follow it, but I do not find any relief. I have pain in multiple areas. Starting from my neck (backside, sides), till the shoulder blade. Can you suggest me what I am supposed to be doing to get relieved from this pain. Thanks a lot in advance Doctor: Thanks for using the HCM. I am happy to address your questions, from your history there could be a nerve involvement, usualy muscle spasm 'l be reduced by few days and with physiotherapy treatment. but your symptoms are not improved with that, so its better to consult a neuro physician for complete neurological examination and further management. Hope I have answered your query. If you have any further questions I will be happy to help. Thanks get well soon."
},
{
"id": 89000,
"tgt": "Suggest medication for abdominal pain,gas and liquid stools",
"src": "Patient: Hi, I'm a 55 year old woman and quite heavy at 155 lbs. I experienced abdominal and gas pains so I take Buscopan, later I develop fever and my stool become liquid. So i take Loperamide Lormide but still am experiencing gas pains. Should I continue to take that medicine? Doctor: Hi! Good evening. I am Dr Shareef answering your query.If you eat from outside, please try to avoid it as intestinal infections could creep into your stomach which might have caused such symptoms of loose stools with fever. If I were your doctor, I would advise you for a routine stool test for ova and cyst, and culture sensitivity test and treat it if positive with proper antibiotics. Loperamide might stop diarrhea, but it might not be able to eradicate the persisting infection in your intestines. Till then you could go for a proton pump inhibitor drug, along with a probiotic.I hope this information would help you in discussing with your family physician/treating doctor in further management of your problem. Please do not hesitate to ask in case of any further doubts.Thanks for choosing health care magic to clear doubts on your health problems. I wish you an early recovery. Dr Shareef."
},
{
"id": 53272,
"tgt": "Could constant diarrhea, pain in abdomen, fatigue be due to diagnosed focal nodular hyperplasia?",
"src": "Patient: I m 20 years old and I have focal nodular hyperplasia. It s located on the left side of my liver about 13cm. I ve been having diarrhea all day and pain on the left side or my stomach. I feel tired and weak. Does it have to do with my fnh? Or could it possibly be something else. Doctor: Hello! Thanks for putting your query in HCM. I am a Gastroenterologist.These symptoms are not related to focal nodular hyperplasia. If these symptoms are persisting for long time then I will suggest you to visit a Gastroenterologist for evaluation and management I hope I have answered your query and this will help you . Wish you a good health"
},
{
"id": 54184,
"tgt": "Will elevated ALT level result in cirrhosis?",
"src": "Patient: I am a 45 y/o female in good health with an ALT that has gone from 36 in 2009 to 47 in 2011. I understand it is due to a fatty liver. What does having an elevated ALT mean to my overall health and likelihood of it developing into cirrhosis? Thanks. Doctor: Hi, dearI have gone through your question. I can understand your concern.You have slightly high alanine transaminase level. It is a liver enzymes in its level increase in liver damage. Your level is changed slightly which is not significant clinically. It is common in fatty liver. Risk of cirrhosis depends on alcohol consumption and many other factor. You should avoid high fat diet and go for regular exercise. Avoid smoking and alcohol. Nothing to worry much about fatty changes in liver. It is very common condition. Just be relaxed. Hope I have answered your question. If you have any doubts then feel free to ask me. I will be happy to answer.Thanks for using health care magic. Wish you a very good health."
},
{
"id": 119799,
"tgt": "What causes swelling of knuckles after a back spin surgery?",
"src": "Patient: my wife had recent back spin fusion surgery , recently she has been having pain in the heels of her feet and now her hands are swelling at the knuckles; she recently received a call frm kidney doc. who said her calcium levels are extremly high , and to quit calcium supp. and blood press pills at onehalf till it can be to zero. Doctor: Hello,His symptoms could be related to calcium and anti-hypertensive drugs adverse effects. If the swelling persists, I recommend taking a diuretic (furosemide) to help reduce the swelling.Hope I have answered your question. Let me know if I can assist you further. Regards, Dr. Ilir Sharka, Cardiologist"
},
{
"id": 138885,
"tgt": "What does the lateral and oblique views of cervical spine indicate?",
"src": "Patient: Six views of the cervical spine were obtained to include bilateral oblique views. On the lateral view, the cervical lordosis is straight. There is mild degenerative disc and degenerative joint disease at C4-5, C5-6 and C6-7 levels, being most significant at C5-6 level. No compression fracture or spondylolisthesis. The prevertebral soft tissue thickness is within normal range. The odontoid is unremarkable. On the oblique views, there is no \u00a0\u00a0\u00a0\u00a0\u00a0 significant bony foraminal encroachment. This is the test result of my husband. I want to know what is this? Doctor: Dear patient Your husband has got loss of cervical lordosis means straightening of cervical spine which is due to muscle spasm. Normal cervical spine is curved anteriorly. If muscle spasm occurs it becomes straight. Also he has got disc degeneration which means reduction of disc space and formation of osteophytes. This is most significant at c5c6 level. Disc degeneration leads to compression of spinal cord and tingling and numbness in both upper limbs. Anterior soft tissue to cervical spine is normal. There is no fracture or displacement of vertebra. This all findings indicates cervical spondylitis. You need to be investigated further with mri of cervical spine with screening of whole spine. You need to consult expert orthopaedic surgeon with report. All the best."
},
{
"id": 194455,
"tgt": "Suggest treatment to grow penis",
"src": "Patient: ive been trying to figure out what really truly can stimulate penis growth. ive tried different pills and stuff but they never worked and im just wondering if theres anything that really does work weather its food or pills or whatever it may be...i need help Doctor: Hello, Please let us know your age and physical status. Usually, there are some medicines and massage techniques which can help improve your penis length. You can contact me for treatment options. Hope I have answered your query. Let me know if I can assist you further. Take care Regards, Dr K. V. Anand, Psychologist"
},
{
"id": 189194,
"tgt": "Small indentation/hole in gums below bottom front teeth. What could this be?",
"src": "Patient: Hi, I have just noticved a small indentation/hole in my gums immediately below one of my bottom front teeth, my teeth are in decent condtion although I have noticed a slight change in the health of my gums which im not sure is the start of receeding gums, at the moment I'm more worried about this indentation/hole. Would you have any adive as to what it could be besides goign to see a dentist, by the way it doesnt hurt. Doctor: Hi, Thanks for writing, A hole in the gingiva can be a sinus opening, but for that infection has to be present nearby. As you have stated the condition of your teeth are decent so it can be a reason due to your gums. Oral prophylaxis (scaling) will be required to improve your gums condition. A diagnostic x-ray and a clinical examination will be required to reach to any confirmed conclusion. Maintain your oral hygiene well, brush twice daily, use a mouthwash along. Stay healthy, Thank you."
},
{
"id": 193045,
"tgt": "Is masturbation really necessary for men's health?",
"src": "Patient: I am just wondering. How long can a male really go on without mastrubation and at all fantasising about sex?? I recently met some mormon people and they told me about their anti sex laws. especially the missionaries. do men at the age of 19-20 really survive two whole years without mastrubating even once? Doctor: Hello,Masturbation is necessary or not. Your body is made for a purpose of reproduction. As we are an animal just like any other animal, our main purpose is to reproduce. Hence, reproduction organs are essential. Reproduction is essential. In that sense, your body will have the capacity to produce sperm for reproduction. If it is not removed from your body, your body will either absorb it or throw it away with your urine or night ejaculations. that means even if you do not masturbate, sperm will be produced and will be ejaculated. Nothing unhealthy about it. Now it is just your decision whether to masturbate or not.Hope I have answered your query. Let me know if I can assist you further. Regards, Dr. K. V. Anand, Psychologist"
},
{
"id": 28862,
"tgt": "How can chest congestion and laryngitis be treated?",
"src": "Patient: I've had a bad cold the past 3 days, starting as a head cold and now has gone to my chest with a lot of mucus. I now have laryngitis and can only whisper. I can breath and swallow ok, but am concerned about sleeping. I also started gagging and almost vomited a few hours ago. I know it will get worse when I lie down. I've taken a hot shower and drunk herbal tea with honey but have not taken cough medication yet. The cough is calm right now;. Doctor: Hello,These problems are usually caused by a virus and will resolve on their own in time. Hot showers, drinking hot tea and using nasal saline can help.If the mucus is a big problem, then antihistamines may also help. Do not take decongestants since they may keep you awake.Hope I have answered your query. Let me know if I can assist you further.Regards,Dr. Kathy Robinson"
},
{
"id": 147152,
"tgt": "When should anti coagulant medicines started after cervical spine surgery?",
"src": "Patient: if a patient is immoblie after cervical spine surgery for cerviclal spinal stenosis, when should anti coagulant or clot preventive meds be started. While wating for emergency corrective surgery his condition , deteriorated tovery quickly being bed ridden Doctor: Hi,Thank you for posting your query.I have noted the patient's condition.I agree that there is a risk of deep vein thrombosis (clotting of blood in leg veins) in this patient, as he is immobilised after surgery and due to weakness in the legs.Anti-coagulation can be started after 48-72 hours of surgery in most cases. However, the decision should be taken in consultation with the surgeon.The best anti-coagulant to use is low molecular weight heparin injection- enoxaparin 40 mg once daily subcutaneous route.I hope my reply has helped you.I would be pleased to answer, if you have any follow up queries or if you require any further information.\u00a0\u00a0\u00a0\u00a0\u00a0Best wishes,Dr Sudhir Kumar MD (Internal Medicine), DM (Neurology)Senior Consultant NeurologistApollo Hospitals, Hyderabad,For DIRECT QUERY to me: http://bit.ly/Dr-Sudhir-kumar My blog: http://bestneurodoctor.blogspot.com/"
},
{
"id": 207253,
"tgt": "What causes lack of hearing and hearing scary voices intermittently?",
"src": "Patient: Sometimes I stop hearing everything around me...i can t understand them and can t hear myself talking or make sense of it..when that happens i hear voices..scary voices...it makes me so scares i panic, cover my ears, crawl into a small space and scream...is something wrong with me? Doctor: DearWe understand your concernsI went through your details. I suggest you not to worry much. If the symptoms you gave here are true to the best of your knowledge, you may need psychiatric intervention for paranoid schizophrenia. You are possibly having illusions. Please understand, timely diagnosis and treatment can cure your problem.If you require more of my help in this aspect, Please post a direct question to me in this URL. http://goo.gl/aYW2pR. Make sure that you include every minute details possible. I shall prescribe the needed psychotherapy techniques.Hope this answers your query. Available for further clarifications.Good luck."
},
{
"id": 168511,
"tgt": "What causes spots on tongue of a child while having dermatomyositis?",
"src": "Patient: My 3 year old daughter has been diagonsed with dermatomyosis and is under the care of a pediatric rhuemetologist. Tonight, we noticed spots on her tongue - I know these disease can present ulcers in the mouth - how do I know if this is an ulcer related to her condition? Doctor: Hello,I can understand your concern. As your daughter is suffering from Dermatomyositis, chances are more that the spots you have found are related to this condition only. To differentiate them from other ulcers, while other kids of oral ulcers are very painful, the ulcers associated with dermatomyositis are not painful. However, some cases present with painful ulcers, too. They are more like the spots and not ulcers and I assume that is what you have observed. However, it is better to get it confirmed by the treating doctor rather than deciding for yourself as doctors are the experts in identifying one type of lesions from others based on their appearance.I hope this information helps you. Thank you for choosing HealthcareMagic. Take care.Best,Dr. Viraj Shah"
},
{
"id": 205038,
"tgt": "Is dementia indicative with the memory loss?",
"src": "Patient: I am 71 years old and have memory problems and get irritated easily. My wife suspects that it is the onset of dementia. I had a cousin who died from Pick's Disease and am worried that I have it. I am going to the VA in two weeks and want to know what I am up against. Doctor: Hello thanks for asking from HCMDementia is an organic brain disorder with progressive loss of cognitive abilities. If you have issues related to memory, irritation and in other cognitive functions then you can think of visiting a neuropsychiatrist. MMSE or mini mental status examination is a small test that can be administered and will help you to screen for dementia. If the scores come in normal range then no need to worry. Since you are 71 so please increase the intake of foods rich in vitamin B12 and E. This will help to maintain your cognitive functions.Thanks and Take care"
},
{
"id": 56965,
"tgt": "What is meaning of hbsab positive in test report?",
"src": "Patient: initially tested positive for Hepatitis B Core Antibody. other results are summarized below: HBSAG Panel HBSAG Result \u2013 Negative Hep B Surface AG \u2013 0.07 (normal 0 \u2013 0.99) HBSAB Panel HBSAB Result \u2013 Positive Hep B surface Ab (Quant) - >1,000 (doctor\u2019s notes indicate this reflects immunity) Hepatic Panel Total Protein 7.1 (normal 6.3 \u2013 8.2) Albumin 4.3 (normal 3.5 \u2013 5.0) Bilirubin Total 0.5 (normal 0.1 \u2013 1.4) Bilirubin Indirect 0.1 (normal 0.0 \u2013 5.7) SGOT/AST 76 (normal 14-42) High SGPT/ALT 89 (normal 9-52) High Alkaline Phosphatase 59 (normal 38-126) Bilirubin Direct 0.40 (0.0 \u20130.3) High HBV DNA Detect/Quant S \u2013 Undetected HBV DNA not detected. Range is 20 to 170,000,000 My mother is not a carrier and received the Hep B. vaccine in 1997, as did I. I have no other symptoms. I am a 25 year old female Doctor: Hello,Your tests say that you are not carrying any Hepatitis B virus mostly ( as you test says Hbsag negative) and your body have developed antibodies ( as your tests says HbcAb positive ) , your virus is undetectable( below the level that is not detectable under a certain limit), but if your Anti HbC is positive, sometimes very rarely you can have an occult infection ( that means though you have negative hbsag you might be carrying the virus), since your HBV D A is below the detectable levels , you need not worry , you please consult your Gastroenterologist you will have to be doing few other tetas and please be under continues follow up and keep repeating tests every 3 months , as advised by your doctor, also get an USG abdomen to see the liver and Spleen also. Summary: you need to be under regular follow up with regular investigations.Hope I could help you.Thank you."
},
{
"id": 218940,
"tgt": "What does this pelvic ultrasound test result during the early pregnancy indicate?",
"src": "Patient: HI MAM, I AM 17 WEEKS WITH HEALTHY PREGNANCY.IN MY TIFFA SCAN MY CERVIX LEGNTH 4.2CM.IS IT SAFE LENGTH??...IN MY FIRST PREGNANCY 3 YEARS BACK THEY GIVEN CERVIX CERCLAGE FOR 3.2CM....NOW IN MY SECOND PREGNANCY THEY WILL AGAIN GIVE CERCLAGE STICHES FOR 4.2CM IN NORMAL RANGE??...PLZ REPLY ME..THANK U Doctor: Hi, I think your cervical length is fine. The thing is that if you had a history of preterm labour or cervical incompetence, it is better to go for a cervical encerclage. If you don't have such history, there is no need of encerclage. Discuss with your doctor regarding this. Hope it helps."
},
{
"id": 217829,
"tgt": "What could cause pain in knees?",
"src": "Patient: i am a cab driver. i sit in the car about 7 to 10 hour every day, atleast 5 or 6 days a week. i am 28 years old. i been driving cab about 2 years now. since last 2 to 3 weeks, i am having a pain in my knees.can you suggest something. do you think its a grease problem in the knees Doctor: Hai, this is due to the constant strain due to ur longstanding profession. Due to the constant position. ! Pls keep doing on and off knee movements,and physiotherapy. Use knee caps, during the day, topicsl pain gel application, painkillers like brufen, diclofenac., keep ur both limbs elevated at rest.if stoll not reduced pls visit an orthopaedic surgeon.dont worry."
},
{
"id": 142388,
"tgt": "Suggest treatment for balance problem after spinal fusion surgery",
"src": "Patient: On Oct 10th 2013 I underwent spinal fusion surgery at the l3,l4,l5, levels. Post op was good, except eversince I woke up I have had Balance issues. I canot walk independently. I must ambulate with a walker of not walk. I have been told I am improving, but I see little inprovment in my walking. Is this a fairly commom post op issue or should I be better informed. My surgeon has told me tne surgery is no way responsible for this problem. I was not nlike this pre-op. I did walk with a cain for added safety as my legs were weak and I had had a few falls, which is why I decided to go forth with the operation,being told it would stabilize my spin. My pain is much improved post-op. can you help me withj an answer? Doctor: Hello there, welcome to HCM , It would have been nice if yo had given more information like your age. However, balance problem may or may not be related to your surgery. Balance issues will need evaluation by a neurologist. Balance issues can be due to compression in some other part of your spinal cord - that is thoracic spine( balance issues only in legs) or cervical spine( balance issues in both upper or lower limbs) or occasionally in the part of the spine which was operated due to weakness in the muscles of the leg. It can also be related to issues of brain( cerebellum apart mostly). With this background I would suggest you to kindly visit a neurologist . Also it would be in the best interest to use aid while walking and avoid further events like fall.Many thanks With Kind regardsDr.Shashidhar B K Spine surgeon Bangalore."
},
{
"id": 151928,
"tgt": "I have back pain and fatigue and cannot focus on anything i forget and loose control of my body. What could be the reason ?",
"src": "Patient: hello. I am 28 years old female.I was a very bright student at school,college and university.Now i am married and have a kid.I feel like I cant focus on anything at all.I cant drive because i feel like even though i am looking ahead,i cant see anything.so i refuse to drive at all now.I feel that my head is very heavy and i cant function properly like every body else around me.I am so tired when i wake up in the morning and it gets worse as the day ends.I have my whole body acheing and fatiged.I cant be focused or punctual so i cant go to work.I have really bad back pain and neck pain ,i forget things and loosing control of my body.I am wondering what is happening to me,why i am so confused all the time.It is effecting my life badly and i dont know how to fix it.I dont drink or smoke and i am not on any medication.please answer my question in detail.thanks. Doctor: The symptoms you have described hint at the possibility of you being depressed or anxious. It is important to understand that there may or may not be any reason for you to get depressed/anxious. It is advisable to consult a psychiatrist, who will be able to help you after detailed evaluation. With proper treatment, you can regain the \"brightness\" which you had during your college days."
},
{
"id": 146360,
"tgt": "What causes pulsating pain on right side of spine?",
"src": "Patient: Today Ive noticed a pulsating pain on the right side of my spine right about where the top of my shoulder blades start. It is very sensitive to touch. The pain comes and goes but can happen anytime; while sitting, standing, running, etc. Should I be concerned? My back often hurts but never before like this. Doctor: Hi, I had gone through your question and understand your concerns. If you are experiencing pain from a long time and it is worsening, then I suggest you to consult your primary care Doctor and to discus your concerns. From what you say I think there is a alignment problem of your thoracic column, maybe scoliosis or kyphosis progressing, so you need imaging study of your column as well ( plain radiographs). Treatment options include posture correction, NSAID drugs and physical therapy. Hope this answers your question. If you have additional questions or follow up questions then please do not hesitate in writing to us. I will be happy to answer your questions."
},
{
"id": 18613,
"tgt": "Can a sinus infection cause the blood pressure to rise?",
"src": "Patient: I am an 18 year old male. I have a resting heart rate of around 45 beats per minute and am a long distance runner. I eat very healthy and am fit. I am 124 pounds and am 5 ft 8 inches tall. I have some kind of upper respiratory tract infection / sinus infection. My blood pressure was just measured at 130/90. I had a second reading taken just to be sure and they got the same result the second time. This number seems very alarming to me. Could my number be so elevated because of my infection or do I have a reason to be worried? Thanks for taking the time to help me. Doctor: Hello and Welcome to \u2018Ask A Doctor\u2019 service. I have reviewed your query and here is my advice. I would explain that this elevation in blood pressure values could be related to inflammation or the infection. Isolated values of high blood pressure do not always indicate arterial hypertension. For this reason, I would recommend you to closely monitor your blood pressure values frequently during the day for a week and refer those values to your doctor. In the meantime, it is necessary to treat the upper respiratory tract infection (antibiotics may be needed). Hope I have answered your query. Let me know if I can assist you further. Regards, Dr. Iliri"
},
{
"id": 176520,
"tgt": "Suggest treatment for wheezing and coughing in a 1 year old",
"src": "Patient: My 1 year old has the croup, woke up with it. my other children used to get this when they were young too. But she is more weezy then I remember and her skin seems a little blotchy colored, if that makes sense. Is that a sign of her not getting enough oxygen? She is not running a fever and acting fairly normal. Doctor: Hello. I just read through your question.Croup is a very common childhood viral illness that usually resolves on it's own . Sometimes it is accompanied by wheezing. When this happens, your doctor may choose to treat the wheezing with medication. I recommend consulting with your doctor so that the lungs can be examined and treatment, if warranted, may be prescribed."
},
{
"id": 105188,
"tgt": "Exercise induced asthma, constipation, back pain, urinary incontinence, headaches, tiredness. Causes for symptoms?",
"src": "Patient: I m really worried. I;m a 15 year old female. I have exercise induced asthma . Recently I ve haven;t been able to go to the toilet properly, I can pee fine but I think I may be constipated. However I worried because I also have lower back pain and stomach pain. I also have Urinary incontinence . I get headaches more often even though I drink quite a lot of water and I feel extremely tired, all the time. I m wondering what is wrong with me :) Doctor: Hi, Thanks for your query I think your asthma control has been compromised and exercise induced asthma is itself an indicator of poorly controlled asthma. Therefore your asthma treatment should be stepped up. Please go through a spirometry testing and record the FEV1. You should start your treatment with a formoterol-budesonide combination inhaler 200 at 2 puffs twice daily along with montelukast 10 mg daily at night. You should have to use them on regular basis. Buy a peak flow meter and record your peak flow daily in a chart to self-monitor your asthma. Consult a good pulmonologist for that and treat under his guidence and make a periodical visit to his clinic for proper monitoring. Thank you."
},
{
"id": 211885,
"tgt": "Lack of sex drive, sweating, fatigue, mood swings, sadness, incomplete feeling due to hysterectomy. On estradiol i mg. Help",
"src": "Patient: I had a hysterectomy when I was 38 I am now 43 year old I have no sex drive is there any I can take to improve my sex life? I am on estradiol 1 mg don't seem to help I have sweats all the time I feel so tard and I am miserable I cant stand it. mood swings sadness I have strange smell when I urinate .I feel by far this was the worst thing I ever done .I was bleeding . and wouldn't stop so that's why I hysterectomy complete no orvaries everthing gone can I ever feel some what normal I feel so bad ever day is a struggle for me I need some can of help ? Doctor: Your symptoms will resemble post-menopausal symptoms. However, they will also be more severe than post-menopausal symptoms because of the sudden withdrawal of hormone levels after the surgery. In post-menopausal state, the transition is more gradual and hence bearable in many women. These symptoms can be in various domains:1) Physical: Fatigue/ tiredness; low libido/ sex drive; sweating episodes, flushing2) Mental: Mood swings, panic attacks, sadness of mood, occasionally psychosisIt is ideal to replace with hormones if the symptoms are severe and unbearable as they are more likely to address all symptoms. This benefit has to be weighed against the increased risk of cardiovascular disease and certain cancers. You may take your Gynecologists advice for this decision. As I see you are not able to tolerate hormone replacement therapy. You may consult a psychiatrist for the mental symptoms which can be treated and handled well with the use of Selective Serotonin Reuptake Inhibitors (SSRIs) like sertraline. They will help you regardless of your taking estrogen pills or not. Alternative Medicines like phytoestrogen supplements/ soya based supplements may also be tried from a practitioner of this system. They may have the advantage of having less side-effects but may work as effectively as hormone replacement. The phytoestrogens are compounds which are similar in action to estrogens, but are derived from plants. They may still have similar side-effects as estrogens, but at a lower risk probably. You may decide on which of these options suits you the best. You have to consult a Gynecologist or Alternative Medicine expert for the first and the third options."
},
{
"id": 197996,
"tgt": "What causes burning sensation during urination and red rash on penis tip?",
"src": "Patient: For the past month or so i ve been experiencing slightly burning urination. It s not every time but it s noticeable when it does occur (nothing intollerable though). I also have experienced a small red ring around the very tip of my penis. It might be scabbing but its too small to recognize. It s deffinentely red though. I am curcumsized and have been sexually active in the past six months Doctor: Dear, We understand your concernsI went through your details. Burning sensation during urination and red rash on your penis explains the inflammation of your urethra. The reason could be some agitative sexual practices or allergic reaction. You should consult an urologist or physician for treatment.If you require more of my help in this aspect, please use this URL. http://goo.gl/aYW2pR. Make sure that you include every minute detail possible. Hope this answers your query. Further clarifications are welcome.Good luck. Happy New Year. Take care."
},
{
"id": 174552,
"tgt": "What causes increased heart rate and low hemoglobin level in a 3 year old?",
"src": "Patient: Hi, I have a 3.5 month old baby boy his heart rate is 150 we took x ray of baby's chest is normal, again blood test for knowing any infection, thyroid or any pottasium , calcium are in high or low, finally our baby has only 8.9gm hemaglobin, and doctor gave a iron syrup.We are worried too much Doctor: HI low hemoglobin results from decrease intake of foods containing iron(green leaf vegetables,carrot, fruits, dates etc) or due to loss of blood or eating pica(sand,paint etc). This results in decreased oxygen to all body parts-heart resulting in increased heart rate.Hope this answers your query and ready to help you further."
},
{
"id": 171273,
"tgt": "What could cause changed behaviour while being off zyrtec?",
"src": "Patient: my grandson is on zyrtec when we accidentally skip a dose he has a horrible day at school acting out, etc another child at school experiences same issues. why would this happen. he has no visible symptoms of allergies that would be causing discomfort. Doctor: Welcome at HCM I have gone through your query and being your physician i completely understand your health concerns.what is his age? For how long he is suffering from? Is he running fever? A Is he asthmatic? why he is using zyrtec? As far as ZYRTEC is concerned it use for in allergic conditions , as well as to prevent the allergic attack. The abnormal change in behavior which u are talking about , i don't see any association between these two. You better meet PEDIATRICIAN so that he can be properly assessed. Zyrtec normally should not be use for prolonged periods since it has got its own side effects . Meanwhile stay calm May he get well soon Hope your query is adequately addressed if you still have any feel free to ask RegardsDr Saad Sultan"
},
{
"id": 40098,
"tgt": "Will there be chances of getting infected due to a pin inserted into arm?",
"src": "Patient: Yesterday I was drinking alcohol and I got involved with sum friends that were shooting drugs and so I join in and it wasn't my first time. Here as I was pulling out the needle the entire tip came off and its still in my arm I was so scared that I didn't go in yet to get it out. Is it serious and can I die if it isn't removed? Doctor: yes this can cause infection better go to a surgeon get it removed followed by a course of antibiotics."
},
{
"id": 169134,
"tgt": "What causes stuffed nose and bloody nasal discharge in a child?",
"src": "Patient: My 2 year old (3 next month) was given morphine in error (Dr had prescribed an antibiotic for tonsilitus and pharmacy read prescription wrong). He was taken to hospital once we realised our mistake in administering. He had a fever of 103 and began vomiting so hospital kept him in overnight for observation as they were unsure if vomiting was morphine induced. He was discharged the following day.I noticed a slight discharge containing blood (amount of blood was too minimal to say he had a nose bleed) the following day. His nose has since been stuffed and he is unable to blow it at all. His nasal discharge when it does expel is hard and contains dry blood. Is this a result form the Morphine Sulphate and should we be concerned? He has been tired (Had to have a nap every day this wk - not like him at all) and a little tearful this week but generally is running around as normal. He is drinking loads and eating very well. Please advise, I am worrying despite the hospital assuring me that there would be no long term side effects from the 5ml dose he was given. Thank you in anticipation. Doctor: Hi..one episode of blood stained discharge might not be due to morphine and the crusts in the nose might b=have been the reason for this.I too agree with the doctors you have visited. You can be rest assured.Regards - Dr. Sumanth"
},
{
"id": 184901,
"tgt": "Why does everything get stuck on the roof of her my mouth?",
"src": "Patient: something in the roof of my mouth i noticed earlier that i have a piece of food or something lodged in the roof of my mouth its not very painful but i can feel it when i run my tounge across. i dont know if this is an emergency or if it can hold off for at least 16 hours. so can should i seek medical attention asap or can i wait this out? i really have no clue. Doctor: hello thanks for consulting at hcm..some food material might have stuck in the palate region,, i suggest you do salt water saline,,also apply topical anesthetic in that area,,then again recheck whether you feel the food is still stuck there,, hydrate yourself well,, drink plenty of juices and warm water,,if still bothers you then you can take a consult with a oral physician,,nothing to very since it is not painful,,hope it helps,,take care"
},
{
"id": 192649,
"tgt": "What causes red spots and irritation on shaft after masturbation?",
"src": "Patient: My girlfriend was a little rough with my penis last night when she engaged in masturbation with me. I woke up this morning and my skin had tiny red spots on it and the skin on the shaft looked a little irritated. we had genital contact but not intercourse and we are both clean. are the red spots and irritation from the masturbation and how long will it take to heal. Doctor: Hello,It may be due to allergic reaction. For that you can apply hydrocortisone ointment locally. For itching you can take tablet loratidine. Avoid harsh cosmetic products. Maintain proper hygiene. Avoid sweating Hope I have answered your query...Let me know if I can assist you further.Regards,Dr Shyam kaleFamily and general physician"
},
{
"id": 75986,
"tgt": "What causes continuous hiccups burning the ?chest",
"src": "Patient: Hi, my partner had a surgery yesterday on his last 2 fingers of his right hand. he sliced them while trying to catch a broken mug. the thing is, after the surgery they prescribed him 'Co-amoxiclav 500/125mg' and 'Co-codamol 8mg/500mg' soon after leaving the hospital he began to have hiccups that are burning his chest and its been continous for the past 20 hours. he is vomitting but explains its not from the stomach, he feels the strain in his chest. im not sure if this is usual or what i should do, could you possibly help? Doctor: Thanks for your question on Healthcare Magic. I can understand your concern. By your history and description, possibility of acute gastritis and GERD (gastroesophageal reflux disease) is more in your partner's case. Antibiotic like Co amoxiclav and Coumadin are known too cause gastritis and GERD. GERD is due to laxity of gastroesophageal sphincter. Because of this the acid of the stomach tends to come up in the esophagus and cause symptoms of hiccups, burning chest pain, vomiting etc. So give him combination of antacid (pantoprazole) and anti Emetic (domperidone). Stop all these drugs. Tell him to avoid hot and spicy food. Don't worry, he will be alright with all these. Hope I have solved your query. I will be happy to help you further. Wishing good health to your partner. Thanks."
},
{
"id": 125672,
"tgt": "How to treat recurring stabbing pain on rib area?",
"src": "Patient: Hi- wonder if you can help. I have had, for the past 3 or 4 days, a recurring stabbing pain, which moves around. Worst when at lower left rib area, towards side, around 3 rib up from bottom, but also moves to pectoral area (less stabbing) either directly above or below nipple area - am male, 38 in good general health, bp & heart ok Doctor: Hi, As you are in a good general health and you did not mention a history of trauma, this is almost certainly a benign and self-limiting problem. The most common cause of your problem is a local spasm or a trigger point in the muscles between the ribs (intercostal muscles). This is not a serious problem and usually resolves after 1-2 weeks or less. If the problem persists for more than 2 weeks, I advise to take a plain x-ray and look at the rib bones (most probably will not be necessary). Meanwhile, you can look for a possible source of the pain by palpating all the areas at lower rib region. If you found a tender spot, you can use local heat as well as gentle massaging (better with a relaxing gel such as Mineral Ice gel (or any gel that has Menthol in it)). Again, this could be self-limiting but it persists for more than 2 weeks a more detailed exam, as well as an X-ray, could be necessary. Hope I have answered your query. Let me know if I can assist you further. Take care Regards, Dr Hooman Mahmoudi, Neurologist"
},
{
"id": 123602,
"tgt": "Suggest treatment for muscle spasms and lower back pain",
"src": "Patient: iam suffred from spam muscles of bacl with pivd-l5-ls1 please tell me the treatment.please reply my problem.i have sever lower back pain n also loss of normal lordotic curvature of l/s spine with straightening of spine Doctor: Hello, The loss of lordotic curvature is due to muscle spasm. So first thing need is to relax the spasmodic muscles by using the hot water fermentation. Do exercise like core stability, spinal muscle strengthening, hip strengthening, lower limb strengthening. We can get the best results if a genuine exercise program is followed for 8-12 weeks of time. In my clinical practice of over 12 years, most cases with PVID responds well to exercise and Physiotherapy. Hope I have answered your query. Let me know if I can assist you further. Regards, Jay Indravadan Patel, Physical Therapist or Physiotherapist"
},
{
"id": 2014,
"tgt": "Is it good to have ethinyl estradiol while preparing for pregnancy?",
"src": "Patient: i want to get pregnant and my doctor priscribed ethinyl estradiol after I have finished using primolut-N just last week before my mesturation period,and this morning,i just took one tablet before contacting you,please i want to know if i am in the right direction? Doctor: Hi I think ethinyl estradiol will not help you in getting pregnant. You need to take some medicines for ovulation. Talk to your doctor regarding this. You should also get yourself evaluated first. Do a thyroid profile and prolactin levels and a ultrasound for your ovaries.Do a semen analysis of your partner."
},
{
"id": 195977,
"tgt": "Is pain in belly & hip common post vasectomy?",
"src": "Patient: I had a vasectomy about 8 months ago...I had pain ever since in that area...and in the belly area right at the hips...sometimes in the kidney area all at the same time...the constant pain is right next to my left hip bone. and mostly where the operation was..is this serious? Doctor: Hello and Welcome to \u2018Ask A Doctor\u2019 service.I have reviewed your query and here is my advice.After vasectomy, 5% to 33% of men experience some form of pain, whether it is acute or chronic, mild or debilitating. I agree that there is a chance for pain when flow in any organ system is blocked, but these numbers seem awfully high.This may be due to procedural pain, granuloma pain, epididymitis pain, congestive pain, neurogenic pain.Blockage of any tube can cause pain that leads to pressure in epididimus which eventually leads to pain in abdomen. This is because both testes and abdomen have same dermatomal distribution. But, pain in the hip is uncommon. You have to consult orthopedician for further evaluation.Hope I have answered your query. Let me know if I can assist you further.Regards,Dr. Penchila Prasad Kandikattu"
},
{
"id": 68620,
"tgt": "What causes sore lump below the belly button?",
"src": "Patient: A few days ago I found a small lump about 3 or 4 inches below my belly button. The first day it was painful but now it is not it is also a bit smaller. It does not move and is about the size of a bb. There is also some discoloration like bruising. What could this be. Should I see a doctor right away? Doctor: Hi.This may be due to cellulitis of the abdominal wall. Get checked by the Surgeon to confirm the diagnosis, get a proper course of antibiotic treatment and incision and drainage operation if need be,. You can get this confirmed by high resolution ultrasonography too."
},
{
"id": 198668,
"tgt": "What is the treatment for erectile dysfunction and swollen lymph nodes in the neck and ear?",
"src": "Patient: I am 27 in excellent shape(about 190 lbs), workout daily, I am on no medications, alcohol maybe once a week. I have recently been experiencing erectile dysfunction, and my right lymph node in my neck by my ear has been swollen for a few months(seems to shrink then come back). I am not feeling sick I have plenty of energy. I was identified with high uric acid and gout but I have not had a flare up in 6 months after changing my diet and losing weight. Are any of these symptoms related. Doctor: DearWe understand your concernsI went through your details. The swollen lymph nodes are almost related to high uric acid and gout but erectile dysfunction is usually not. The primary cause of erectile dysfunction is psychological and therefore, recollect all those incidents happened prior to the first incident of ED. There should be something like stress, anxiety, physical discomfort etc. Then with that information you should consult a psychologist for diagnosis and counseling. If you require more of my help in this aspect, please use this URL. http://goo.gl/aYW2pR. Make sure that you include every minute details possible. Hope this answers your query. Available for further clarifications.Good luck. Take care."
},
{
"id": 130814,
"tgt": "What is the treatment for swelling in the legs?",
"src": "Patient: I was prescribed ibuprofen 600 with an anti-inflammatory med for sciatic pain. After 4 dosages, I started noticing that my legs were swollen and felt heavier. I checked side effects on ibuprofen and this is one of them. I stopped both Meds 2 days ago and legs still feel swollen. Should I be concern or would swelling go back to normal soon. Also what can I do for swelling to go down? Doctor: Hi i am Dr Ahmed Aly thanks for using helthcaremagic site , Ibuprofen is an analgesic referred to as a nonsteroidal anti-inflammatory drug, or NSAID , so it will remove the pain rather than the edema or the swelling , antiedematous drugs ( alpha chemotrypsin tablets or intramuscular injections) will be more effective ,you may be in a bit of concern because swelling of both legs means that the cause is a systemic cause cardiovascular or pulmonary problems ,most commonly thrombophlebitis or DVT are the main cause so i reccommend vascular investigations on your lower limb vessels , try to ask your physician for some investigations to know exactly the main cause of the edema in both of your legs, elevating your legs, r putting them on a pillow or even wearing long socls is helpfull,avoid salty high cholesterol meals Hope the above information helps you,Thanks for writing into healthcaremagic. .healthcaremagic site doctors will be ready to assist you whenever needed ."
},
{
"id": 154050,
"tgt": "Suggest treatment for prostrate cancer",
"src": "Patient: My father is a patient of cancer prostate with bone met. Now has as underwent TURP surgery with this surgery his front urine has stop. He is on SPC. One month back has UTI Infection and bleeding in urine, doctor suggested Radiation so that his bleeding will stop. But unfortunately it is still bleeding now his Hb is 6.0. Can Orofer FCM injection 500 ml is advisable for his . If yes how many days ? How much his Hb level raised. Please advice ? Doctor: Hi,Thanks for writing in.Since you have said that he is on supra pubic catheter, I understand that he is unable to pass urine through the normal urinary passage after surgery. Unfortunately he has developed complications causing bleeding and his hemoglobin is low if it is 6.0 gm/ dl. This cannot be treated by self medication and in my opinion, after consulting your doctor blood transfusion might have to be given to increase hemoglobin to 10 gm/ dl. Giving orofer only may not increase hemoglobin levels sufficiently therefore it can be given after the hemoglobin increases following blood transfusion.With all measures the target raise in hemoglobin must be at least 10 gm/ dl. The bleeding area also has to be checked and treated if there is no relief after radiation therapy. Please do not worry."
},
{
"id": 40127,
"tgt": "Will accidental contact with insulin could cause an infection?",
"src": "Patient: Hi, today while changing the needle from a novapen insulin for my mother who's a diabetic, I accidentally scratched my finger with it, to see if it was deep i squeezed it. first nothing happened but then i did it further more and a little blood came out, i cleaned the area with a sanitizer. Im a little worried if it may cause any sort of infection? please help Doctor: Hi, You had needle stick injury from your mother's used needle. You need to clean the wound with anti septic solution like sprite and betadine get a dressing for one day. Now you need to check for your mother infection status and accordingly tested for that ( your mother infected with) with other viral marker ( Hiv ,hbsag , hcv ). Generally there are least chance of needle stick infection but still there. There is no need to take any oral antibiotic untill infected .Hope it ll help Thanks n regards"
},
{
"id": 180540,
"tgt": "How can pain and tenderness around a tooth extraction site be treated?",
"src": "Patient: Had my wisdom teeth taken out about a week ago and the right bottom extraction sight where the sutures are doesn\u2019t ache but still hurts and is tender also I have a brownish saliva output not sure if that is normal also where the sutures are is kinda white idk if that\u2019s from scabbing or what maybe I\u2019m just paranoid any advice would help Doctor: Hello and Welcome to \u2018Ask A Doctor\u2019 service. I have reviewed your query and here is my advice. It looks like your extraction site is healing fine and if the pain has subsided then slight tenderness and discomfort is going to get better in next few days, so do not worry about it. Also brownish saliva can be due to old blood from extraction site. White stuff at the extraction site looks like the healing tissue and it is normal appearance of the extraction site. So my suggestion is to follow the post extraction instructions carefully and if one week is completed then a follow up checkup with your dentist should be done. Hope I have answered your query. Let me know if I can assist you further."
},
{
"id": 172258,
"tgt": "Suggest medication for lump noticed on the rid cage and diarrhea",
"src": "Patient: My neice is 5 months old i think there is something wrong with her....shes not taking her bottle, she has a lump on her rib cage, she would be asleep and wake up 15 min later in a Blood screaming cry shes poopin and peeing every 5 min, her cheecks are blood red and shes really warm...im not her parent but i am her uncle im also a paramedic but can you please tell me whats wrong...shes been sick sence christmas and her mom and dad has not takin her to the doctor and i told them to so please Doctor: The information provided by you is not very specific. Where on her rib cage is the lump? Is it painful or inflammed or is it cold and painless? What about the crying? Is it like a colic? Because some infants may continue to have the colic beyond 4 months, which is the usual age after which colics tend to disappear. By warm, do you mean fever? Did you measure the temperature? Do revert with those answers in your rebuttal and we will explore the whole issue together. Best wishes,Dr. Taher"
},
{
"id": 148074,
"tgt": "Will Ginkgo Biloba improve blood circulation to brain and eliminate my speech impediment?",
"src": "Patient: I was diagnosed with a mild Speech Impediment disorder when I was a child, and still sustain its effects to this day. There are certain days where I can verbally express my thoughts and ideas freely, with little to no error. However, on other days, my speech reverts back to how it s consistently been with a constant stammer and incoherent speech. I have decided not to consult with an Audiologist or Speech Pathology Specialist, for the associated Therapy isn t always an effective approach to the extent of curing the disorder. I ve compiled extensive research, and came across an herbal supplement known as Ginkgo Biloba . Considering the fact that it significantly improves blood flow to various sections of the Brain, I confidently assume that this could potentially eliminate my Speech Impediment. Is my observation correct? Doctor: Hello,Ginkgo is a very good herbal medicine used more commonly to cure brain degenerative changes. There is however no proven therapeutic claim, some conditions show significant improvement but the study is still limited to memory related problems. In your case, speech problems are usually traced back to childhood development, such that, any management done currently might not have high success rates. While it is not a hopeless case, speech therapy is still the main management of choice in your case.Thank you.Dr. Eric"
},
{
"id": 219806,
"tgt": "What causes brown vaginal discharge during pregnancy?",
"src": "Patient: Hello Doctor, I m 5.2 weeks pregnant and having brown discharge since 5 days and noticed small clots since yesterday. I got 2 blood work HCG results done on alternate days. 1st one -450 and 2nd one is 824. I m goging for the 3rd one tomorrow. I had constipation 2 days back and when I cleared my bowels, I have lot of pain. Even when I coughed. That time, I had pain in my left abdomen too. All this lasted till I cleared my bowels. After that I was back to normal. It s been 2 days and I don t have any problems like that but having the brown discharge. It is not like a period but when I wipe I see brown color discharge and also lately more on the pad. Am I having a miscarriage or an ectopic pregnancy or is everything ok. I have not had any scans till now. Please advise. Thanks Lakshmi Doctor: you should get scan done to determine the status of intrauterine pathology. Sometime brown discharge can occur due to disruption of pregnancy sac ."
},
{
"id": 90955,
"tgt": "Suggest treatments for pain in upper abdomen and gastric trouble",
"src": "Patient: i have regular continuous sweet pain in upper abdomen left side , just below the chest or u can say just below the rib cage , i have already gone through the ECH , Echo test , Endoscopy all are normal , presently taking some medicine for gasetrology , but still not comfortable feeling same problem of having too much gas , any one can help me plz Doctor: HiThanks for writing in HCMI need further information regarding 1.)your age 2.)how long you are suffering with this problem 3.)do you have symptoms of nausea /vomiting 4.) any complaintsdifficulty in passing stool 4.) factors aggravating your symptom5.)any chronic analgesic intake history/alcoholic 6.)are u under any cardiac or anti hypertensive medications 7.)recent trauma history ,As you mentioned your ECG , Echo test , Endoscopy all are normal ,cardiac and gastric diseases unlikely .can you mention that some medication prescribed by your gastroenterologist.my advise is to drink plenty of water,regularly timed food with every 3 hours but all in small quantity for eg 2-4 biscuits with adequate water and to avoid spicy and oily food,no alcohol,no coffee,no junk foods because you mentioned too much gas.i would be glad to provide answers to any further queries. Thanks for using HMC Have a good day! Regards Dr. V.KUMARAGURU general surgeon"
},
{
"id": 81153,
"tgt": "How to treat pneumonia with fever and coughing up phylum and mucus?",
"src": "Patient: Hello, my son has been sick with pneumonia since last Thursday. He s been to the clinic and was given antibiotics. He hasn t really improved - still has a fever, is coughing up phylum and mucus, goes from feeling really hot to having chills, and tonight has thrown up from coughing so much. He said that it gave a new meaning to coughing up a lung . It was stringy pink stuff with black chunks, is how he described it. We believe the black chunks may have been the black grapes that he had just before getting sick. Shouldn t he be feeling better by now (5 days in) or am I just being an impatient, worried mom? Doctor: Thanks for your query. 5 days antibiotic treatment if fair enough to get symptomatic benefit. In your case it may be possible to have infection with resistant organisms or something else. Get a sputum culture sensitivity, AFB testing done. Guided antibiotic therapy will help . till then supportive care with antipyretic, lot of fluids, cough syrups should continue. Best wishes"
},
{
"id": 170526,
"tgt": "What causes frequent perirectal abscess?",
"src": "Patient: I have an 9 year old son who is having recurrent perirectal abscesses. He has had one i&d with hospitalization and vancomycin. He is now on his fourth one in four months and his fifth round of bactrim. The doctor did second culture because first showed nothing. Doctor: Hello, thank you for posting your query here. Recurrent perirectal abscesses could be due to colonisation with a bacteria called staph aureus in his body, which may be a resistant strain. After the current abscess settles, I suggest you do a decolonisation with mupirocin ointment applied in both his nostrils twice a day for 5 days, with chlorhexidine body washes for 10 days. Give special attention to scrubbing his armpits and groin are well as the bacteria thrives in these areas. if he has had frequent abscesses in the past also, he should get evaluated for immunodeficiency as well/Hope this helps you, take care.Regards,Dr. Vaishnavi"
},
{
"id": 72667,
"tgt": "Suggest remedy for sharp pain in upper left chest area",
"src": "Patient: For the past 10 days I have been greatly bothered by a sharp pain in my upper left chest area, which has been accompanied with extreme fatigue. The area that hurts is just under my left breast in the left upper rib area. My doctor dismissed it as chest wall pain and suggested I see a chiropractor. After 2 appointments with a chiropracter 5 days ago I feel ten times worse. I feel as though it is something major, because it does not feel like a bone issue, it feels like it is an organ pain to me. (it hurts underneath the rib bones) The pain is increasingly worse: when I take a deep breath, upon waking up from laying down, and when doing just mild physical activity. It has also come along with dizzyness and frustration with having no energy and not feeling well. It started off as mild pain about 25 days ago, and I assumed it would go away but it has progressively gotten extremely unbearable. I have been sleeping 18 hours a day, and the pain is awful. I am a healthy 24 year old female, im 5 11 , i weigh 155. I am currently prescribed by a psychiatrist 60mg cymbalta daily for mild depression, and 50mg vyvanse daily for ADD. I have never had major surgery or major medical problems. I had blood work done one week ago, and everything came out fairly normal. About 15 days prior to the chest pain starting I had great pain in my neck. I had x-rays and was told that I had tortecollis/dystonia due to no curve in my neck. After 5 days of muscle relaxers the neck pain felt better and has not bothered me since. I am now concerned with the upper rib pain. I have negatively tested for mono, which is what they thought it was initially. I also have no skin rash, because they thought it might be shingles. In the past 10 days I have been prescribed multiple types pain medication, anti-inflammatory, & muscle relaxers. Which have not helped. Doctor: Hello dear , hiWelcome to Healthcaremagic.comI have evaluated your query thoroughly .* This seems in relation with intercostal muscle or ligament related issue .* Guidelines for better recovery- Local application of heating pad followed by analgesic spray or ointment 3 times a day .- Lavender oil gives excellent relief .- Deep breathing gradual exercises , YOGA , walking in fresh air helps .- MRI work up is mandatory later on if required .Hope this clears your query .Wishing you fine recovery .Regards ."
},
{
"id": 219925,
"tgt": "Could over dosage of hormones during pregnancy affect the baby?",
"src": "Patient: I am in the 9th week of pregnancy. We did an ultrasound last week and it was said that there is a choriodecidual seperation of 25% and some bleeding around the pregnancy. so i was asked to take Susten 200g injections once every week. And also naturogest 200 mg twice daily. Its not only making me very sick, I am worried that its too much of hormones that is being given to me. Is this dosage right? I was also given an Anti D injection as my blood group is O Rh Negative. I am also on 100mcg Eltroxin and 500mg Diamet (metformin) from before. I am really concerned as i have conceived after 6 years of marriage. Please advice. Doctor: Hello dear,I understand your concern.In my opinion the bleeding around the conceptus might result in miscarriage rarely.So the progesterone hormones like susten are given to calm down the uterus and prevent miscarriage.The susten is safe and doesn't harm the fetus.Metformin also is safe .Eltroxin is essential for hypothyroidism to keep thyroid under control.Anti D is essential in case of Rh negative pregnancy.So dont worry as all the medicines you are taking are essential and doesn't harm your pregnancy.So relax.Avoid stress.Hope this helps.Best regards..."
},
{
"id": 218023,
"tgt": "What is the remedy to reduce pain in mid-forearm caused due to a bruising?",
"src": "Patient: I slammed my hand/wrist on counter in a argument today and now have bruis from pinkie down to mid forearm, the bruise is red stripes and white down middle. Close to pinkie is black/blue. Move fingers fully with pain, can t move wrist in circle without excruciating pain or up and down and side to side Doctor: It appears that you have sustained a soft tissue injury from the force of the trauma. I would recommend rest, ice and pain control. For pain, You can try Ibuprofen (400-800 mg three times a day with food)- it has an anti inflammatory effect. Do not take Ibuprofen if you have a history of chronic kidney disease, bleeding from your gut or severe reflux. You can also try Acetaminophen (Tylenol) 325-650 mg every 4-6 hours as needed. Do not take more than 3,000 mg of Acetaminophen over a 24 hour period. You can take both Acetaminophen and Ibuprofen together. If your symptoms do not improve or you have increasing pain, swelling or redness or numbness or tingling thats not improving, then please see your primary care doctor or go to the ER to have it evaluated. Please let me know if that was helpful. Thanks!"
},
{
"id": 49664,
"tgt": "Kidney stones, calculus obstruction, renal calculi. Urine Test showed sugar. Prescribed cystine and calcury. Help",
"src": "Patient: hi..i am 30 yrs old male , taking treatment for kidney stone..while taking scan the impressions are :1) left hydroureteronephrosis due to VUJ calculus obstruction(4.3mm) , 2) left renal calculi ( 4.0mm..) In my urine test report i got sugar (R) is +ve ...my consultant prescribed me CYSTONE tablets and CALCURY tablet for me will it cure...i am worry about that ..will the kidney stone leads to any kidney failure....and one more thing this kidney stone issues will effect any sexual issues.. Doctor: Hello Friend ..Thanks for your query to Health Care Magic ..I read your concern about kidney stone and there is nothing to be worried as Small Kidney stone will not damage your kidney permanently , moreover it does not have issues with sexual life , Kidney stones once detected it will be cured but chances of recurrence is more as stones tries to develop if not on Diet and medications .I suggest you to continue same medications as prescribed by your Doctor for another 3 to 6 months and Drink 3liters of water per day to prevent urinary stone deposition and to neutralise urine .Hope this information is helpful.Thanks & Take Care"
},
{
"id": 85005,
"tgt": "Should I take Progynova 2mg?",
"src": "Patient: hi... im married for 4 yrs, trying to conceive, doc advised ccq50 along with progynova 2mg and duphaston10mg. But i hv found that progynova drugs have side effec ts like breast cancer, it also suggests not to take them during pregnancy .... im confused, plz help. Doctor: Hello, Progynova replaces the hormone oestradiol that the body no longer makes and prevents or relieves symptoms such as hot flushes, sweats, sleep disturbances, depressive moods, irritability, dizziness, headaches as well as vaginal dryness and burning. If considered necessary, Progynova 2 mg should be used. Once treatment is established the lowest effective dose necessary for relief of symptoms should be used. Hope I have answered your query. Let me know if I can assist you further. Take care Regards, Dr. AJEET SINGH, General & Family Physician"
},
{
"id": 120060,
"tgt": "What causes restlessness in legs?",
"src": "Patient: hi! Last few weeks I have been experiencing very uncomfortable sensaton in my legs, mostly in one at a time. I feel so restless, and any position will not feel comfortable.It stays for long, after walking I feel better., but the moment I sit, the problem starts. I suffer from high B.p , on medication and is now in control. What could be the reason for this? Doctor: Hello, Get your vitamin D and vitamin B12 checked. It could be a cause of your pain and if deficient take proper supplements in proper doses. Hope I have answered your query. Let me know if I can assist you further. Take care Regards, Dr. Jaideep Gaver"
},
{
"id": 213801,
"tgt": "Can any online doctor refer a good psychiatrist in Calicut - Kerala ?",
"src": "Patient: nowadays my brother (17yrs) behaves as a violent character for sometime and after that he will be very cool.. he dnt knw wat he doing while he is abnormal.. So we are planning to take him to a good psychiatric doctor today itself(morning).. can I get some best doctor s contact since i m from calicut-kerala.??? shed me some light...plz Dr. Doctor: Hello, Thanks for posting your query. First it would be good if you recognize the cause. Like whether he has problem with his studies or friends or love matter you try to ask him when he is calm. Also teach him odd consequencs of such behaviour. Also make him read good books like teachings of vivekananda which would teach them many things. Initially try to solve it at home and if not later you can consult a doctor. Hope I have answered your query. Wishing you good health. Regards,"
},
{
"id": 34726,
"tgt": "Suggest remedy for swelling and soreness in finger due to wound",
"src": "Patient: I cut my finger about a week ago and it has been feeling okay; that is until 2:00am this morning. When I woke up my finger was throbbing. I soaked it in hydrogen peroxide, but it still hurts. It is also swallen around the wound. I am wondering if my hobby, gardening, has caused an infection that I should be worried about? Doctor: If you had the wound a week ago and if it pains and is swollen now, it is most probably an infection of the wound. You will need to get a doctor's prescription for antibiotics against the infection. If you have an infected wound which has turned into an abscess, your doctor might have to perform a small surgical procedure called incision and drainage to drain any pus collected inside to heal the wound faster. Good day and take care."
},
{
"id": 136504,
"tgt": "Suggest treatment for numbness on the thigh",
"src": "Patient: Hello I have had numbness on my left anterior thigh so I went to a chiropractor and she worked on me twice. Afterwards I got pretty bad bruises on my legs so the chiropractor referred me to a medical doctor. He ordered blood work for my platelets etc. and the blood work all came back normal. If I had some sort of std like HIV or something would that have shown up in the bloodwork? Doctor: hi i appreciate your concern.numbness in the thigh is most probably due to nerve pressed at groin level.it is seen more in fatty bulky abdomen people,usually doing sitting jobs.it is advised to get yourself checked by your ortho expert,at the same time reduction in weight,and abdominal wall toning up may help along with neuro vitamins.thanks,i will be happy to answer any further quary."
},
{
"id": 68510,
"tgt": "What could the swelling in armpit be?",
"src": "Patient: Hi - about 4 months ago i noticed swelling under my right armpit. I went to see my doctor and had mammogram and ultra sound of the area. They found an extremely small lump from the ultra sound. When I press down on the lump it stings (kind of like a bee sting). It gets a little more irritated during my period but outside of that it doesn't bother me. I am 35 years old and there is no history of breast cancer in my family. The doctor told me to come back in 6 months for another ultra sound just to make sure it hasn't grown. The doctor does not believe it is breast cancer...what do you think it can be? Doctor: Welcome to Health care magic.1.The history suggest that there is a small lump with in the breast ( as mentioned in history) and a swelling in the armpit area / axillary region.2.This features suggest a benign lesion on the breast and adjacent lymph nodal enlargement in axillary region.3.Doctor call you after 6 months because - the lesion what he noted during the scan is not coming under criteria of cancer / malignancy.4.In this condition any doctor according to the studies does the same way.5.If there is any difference in lesion - growth - then an FNAC - fine needle aspiration cytology to evaluate the cells.6.Further treatment will depend on the findings. Good luck.Hope it helps you. Wish you a good health.Anything to ask ? do not hesitate. Thank you."
},
{
"id": 132218,
"tgt": "What causes red discoloration and bruising on shin?",
"src": "Patient: My right shin has some red discoloration, feels bruised to the touch and when I stretch and some swelling. Has been this way for about three months. I have not hit my shin on anything. I am 53 years old and a female. I am currently on a 1200 calorie diet prescribed by my physician and have a trainer at the gym that has me on a workout program. The only meds I take are for my migraines. I have not brought this to my physician s attention at this time, but am starting to worry more about it. My son, who is a paramedic, says I should have it x-rayed or an MRI done. What say you? Doctor: Hi Persistent red discoloration on shin , of recent origin with hypersensitivity and swelling needs proper workup. Your paramedic son is right -you should undergo investigations like Blood counts , Sugar and an MRI to investigate the red discoloration on your shin."
},
{
"id": 19313,
"tgt": "What causes fainting while treating fluctuations in the BP level?",
"src": "Patient: Hi, my name kimberley just took my blood pressure reading first reading was 71/41 took second reading came back saying 82/50. Have had my blood pressure bottom out about 5 or 6 times. Was sent to ICU. I call it plan old passing out. Maybe about a week and a half ago, I had two more episodes. Both times was at night around this time. Did not feel dizzy was out first time for 4 1/2 hours. Second time it was 5 hours. What should I do? Lay back down? I am not alone in the house, but my son sleeps up stairs. Thank you Doctor: take plenty of fluid with added salt.....hve routine ix of na n k .......n b12 n complete blood cout with liver n kidney profile.....if all come normal....just hve more fluid at nyt ...vitamin supplement may b adviceble"
},
{
"id": 194598,
"tgt": "How to diagnose gynaecomastia?",
"src": "Patient: i am having gynaecomastia i dont knw really bt my friends say my age s 22 and they say its bilateral my one side of chest is slightly protruding out and my left side a bit more than right side can u help me in how to diagnose weather it s gynaecomastia or some thing else Doctor: Hi, You can get a direct check with a plastic surgeon. He will help you. Hope I have answered your query. Let me know if I can assist you further. Take care Regards, Dr B. Radhakrishnan. Nair, OBGYN"
},
{
"id": 19428,
"tgt": "What causes pain in leg after having a heart failure?",
"src": "Patient: A while back ago i suffered from heart failure... i was wondering because then as well as now my right leg gives out on me it feels weak and the moment i put pressure on it... it gives out... can you tell me whats going on? Or atleast help me in figureing it out? Doctor: Hello!Welcome and thank you for asking on HCM!I passed carefully through your question and would explain that your right leg symptoms do not seem to be related to heart failure, unless your feet are swollen. Only in case of swollen feet, the discomfort could be related to heart failure (considering the fact that it leads to increase fluid retention).If the problem persists, I recommend consulting with your attending physician and performing some tests: - a Doppler ultrasound of the leg vessels to exclude possible peripheral arteriopathy or chronic venous insufficiency- complete blood count, PCR, ESR for inflammation. Hope you will find this answer helpful!Kind regards, Dr. Iliri"
},
{
"id": 16039,
"tgt": "Rashes on skin, bumps on legs, itching, bleeding on scratching. Treatment options?",
"src": "Patient: I have some kind of rash or open wound. It is red around & inside it is white inside. Some of it is bumpy but not all of it. I have two on my left side on my right leg and some on my right side of the leg. It itches sometimes and when I scratch the inside bleeds . What can I do to get rid of it? I have been putting cortisone , but it isn t going away. Doctor: hello, the way you are describing the rash it seems like a fungal infection try using antifungal cream if there is no improvement then you need to see a dermatologist to rule out other skin conditions like psoriasis or dermatitis. hope have answered your query get well soon."
},
{
"id": 145708,
"tgt": "What does the MRI result indicate?",
"src": "Patient: I had an MRI, and these are the results.... do I have a problem long term? 1.Severe bilateral foraminal stenosis at C5-6 and on the left at C6-7 with moderate to severe foraminal stenosis on the right at C6-7. 2. C3-4 minimal anterolisthesis with left-sided facet arthropathy and periarticular edema indicative of acute inflammation. Uncinate spurring on the left with severe left foraminal stenosis. 3. C4-5 moderate to severe degenerative disc disease and disc bulge which abuts the ventral cord with mild central spinal canal stenosis. Left-sided facet arthropathy with periarticular edema and uncinate spurring with moderate left-sided foraminal stenosis. Doctor: Hi, I have gone through your question and understand your concerns.You have a long term problem mainly due to degenative cervical column and disc disease.MRI report shows there are inflammatory processes on your cervical column (arthritis) that gradually resulted in foraminal neural stenosis ( exit nerve roots narrowed) and medular cervical canal narrowing.MRI findings need to be correlated with clinical findings in order to plan an appropriate treatment.I suggest you to consult a Neurosurgeon and discuss your concerns and MRI reports.Hope this answers your question. If you have additional questions or follow up questions then please do not hesitate in writing to us. I will be happy to answer your questions."
},
{
"id": 12330,
"tgt": "Suggest treatment for psoriasis?",
"src": "Patient: I've been on neotigason 2 daily capsules 25 mg for 6 months as a treatment for psoriasis. I came off them actually couple of weeks ago because my doctor prescribed antibiotics for a toe nail infection. In the meantime my psoriasis has returned. Can I re-continue taking neotigason. I have periodical blood tests whose results up to now have been ok to continue with the treatment over the past 6 months. I'm 67 years old and I take high blood pressure pills which control the pressure at 120/ 80 usually. Doctor: Hi,As you said,you suffer from psoriasis vulgaris. You must be aware about psoriasis that it is a controllable disease and there is no complete cure. So, some factors like stress,disease anywhere in body,trauma,dry weather...etc might precipitate the the disease. In your case, it has relapsed. Do not worry. You just restart the medicines you were taking...neotigasone 25 mg twice a day. After control of disease, you may reduce the dose to 25 mg per day. And also apply steroid with salicylic acid oint on the lesions. I hope you got my answer.Thanks.Dr. Ilyas Patel MD"
},
{
"id": 11335,
"tgt": "How to improve hair growth and reduce thinning?",
"src": "Patient: I m 26 years old.Sufferig from female pattern thinning.My dad is bald.Thinning of my hair started about 7-8 years back.it has been progessive since then gradually.But now it is more or less stable.but my centre partition is visibly wide and thinnig is obvious.it there a safe and permament way to have new growths and improve the condition. Also how safe is mintop forte 5%.as prescribed by a doctor to me. Doctor: Hello. Thanks for writing to us at healthcaremagicMinoxidil 2% liquid is currently approved for use in Female Pattern Androgenetic Alopecia. It is very safe. However some dermatologist may also choose to prescribe a higher strength of Minoxidil (5%).Other treatment options include Oral treatment with antiandrogens like Spironolactone and Finasteride. Oral treatment is however to be cautiously used in females of child bearing age because it can cause harmful effects in the fetus.Treatment is long term and effects are usually noticeable after 6-8 months of regular use.Newer treatment options are Microneedling and Platelet Rich Plasma Therapy.The mechanism of these being stimulation of hair follicle by growth factor released from activated platelets. These platelets are derived from the patients own blood.I would suggest that you continue with Minoxidil for the time being for the next few months and if the response is not as desired, your doctor may then either add oral antiandrogens or opt for PRP therapy.Regards"
},
{
"id": 64260,
"tgt": "What is the treatment for sebaceous cyst in the head?",
"src": "Patient: I have had a sebaceous cyst on my head for quite some time. It has gotten bigger this past year, and about a week ago started leaking when I am always at it. The fluid is clear with traces of very little blood,and the cyst area is sore...is this normal? Doctor: Hi,Good Evening.Thanks for the query to HCM.I studied it in depth and I understood your health concerns.-Treatment-for Sebaceous Cyst of the head-1- In my opinion -the clear fluid appears to be due to the cellulitic fluid due to the handling of it by you and hence little of fluid -indicating that the infection is not advanced and could be controlled with proper treatment with ER Surgeon.2-As rightly said by you the cyst is not painful and red ,but is sore due to excessive handling by you with less of fluid discharge.3-So I would advise you to Consult a Surgeon and who would treat it accordingly as told above and with proper antibiotics and anti-inflammatory and proper drainage by surgical means.4-Dont get scared and uneasy,but be cautious and act fast.Hope this would help you a lot to relieve you.Wish you fast recovery-and healthy life.Wellcome to HCM with more queries till you are satisfied.Have a Good Day...!!Dr.SAVASKAR M.N.M.S.GENL-CVTS,Super specialist and Senior Consultant-and Expert in Non-Curable-Disease therapy for Cancer,Asthma,etc,Rejuvenation therapy and Tissue failure -reversal therapies."
},
{
"id": 218866,
"tgt": "What causes delay in periods despite having negative UPT?",
"src": "Patient: I've been having unpredictable periods lately. I had my last period for a month straight and it finally stopped on June 17th. I havn't had a period since and i'm having symptoms like im pregnant but i took a test today and it's negative. Should I wait another week or so and check again if I still dont get my period? Doctor: delay in periods with negative upt can be due to stress,pcod,hormonal imbalance,overweight etc..as upt is negative ...u can wait for a week...then visit your gynecologist"
},
{
"id": 88724,
"tgt": "What causes stomach pains after appendix operation?",
"src": "Patient: about a year and a half ago i got my appendix taking out, from then on i have stomach pains normally by my rib cage, pains behind my life eye, my stomach gets hard after i eat, sometimes i feel like im going to puke in the middle if the day,. i don't know what to do. Doctor: Hi.Your symptoms are suggestive of Irritable Bowel syndrome.For us to say this as a IBS i.e. Irritable Bowel Syndrome, all the investigations should be normal.I would advise the following investigations and treatment in a such a case:Investigations:*Routine blood investigations like Complete blood picture, blood sugar, urea, creatinine, liver function tests, thyroid function tests.*Urine -routine and microscopy*Stool: routine, microscopy, occult blood, culture and sensitivity.*Ultrasonography of abdomen,*Colonoscopy All these investigations should be within normal limits for one to say you have IBS.Treatment:''STRESS'' = stress is the most important cause of IBS. Please remember and you must have observed that when you do not have stress your problem with IBS is reducedChange of lifestyle.I think 'may be' you are not coping with something. Vacation outside of your area, mobiles phones to be off.You must have noticed already that these changes help a lot. Try to avoid anything which you are / have to do against your will and wish. The investigations will lead to a diagnosis and proper treatment."
},
{
"id": 185015,
"tgt": "What causes dental root coming out from the gums after tooth extraction?",
"src": "Patient: Hi all and one,I just had a tooth extraction done Monday just gone. Was glad to find that I don't have a dry socket but several things are worrying me: Firstly it looks like 2 roots have been left in there, can this be right? Will it have complications later on? As the tooth was suppose to be taken out due to pus being developed on the gum and was infecting everything around it,..?Secondly, the tooth next to it it's gum split...one side is flopping around? Will it grow back together?Shall I be rebooking to go and see the dentist? Thank you in advance Doctor: hello thanks for the consult at hcm..Was extracted tooth -molar or premolar and was it very difficult to extract?? Since you think 2 roots are left behind,, firstly let the infection in the region come down..hope you have taken antibiotics and analgesics,,for the gum split --you can review back with your treating dentist for compression of buccal and lingual cortical plates,,and also a suture could be placed at gum split region for faster healing,,hope it helps,,take care,,"
},
{
"id": 17013,
"tgt": "Suggest treatment to normalize the BP",
"src": "Patient: (1) Last week, i went for a health checkup as part of admission to university in August this year. When i took my blood pressure, it was recorded as 133/76. The nurse took a second reading and it was the same as well. When i saw the doctor, she told me that my blood pressure was slightly high, and she took a third time, but this time round, i wasn t told of my pressure. I was wondering if this blood pressure of 133/76 - isit considered as hypertension. My dad has history of hypertension and is currently under medication. And also, what are some things that I can do to keep my pressure normal if I do not want to see a doctor as of now, i.e. food to eat and abstain? Doctor: Hi, Your present blood pressure is at a pre-hypertension level. Although you are not yet ''hypertensive'', that level, along with your family history of having hypertension, puts you at a higher risk of being an over hypertensive. If I may only know more details about you, I would be able to help you more. But on top of these, one important aspect in the prevention of hypertension is the avoidance of salty foods, weight loss to appropriate levels, as well as aerobic exercise, all of which maintains optimal blood pressure. Hope I have answered your query. Let me know if I can assist you further. Regards, Dr. Daryl Derick B. Dizon, General & Family Physician"
},
{
"id": 66249,
"tgt": "Suggest medication for red lumps on the temple",
"src": "Patient: Good morning..My name is Carolyn I am 50 years old and woke up with 2 red lumps by my temple. I went to the DR who said it might be an allergic reaction, I am on prednizone and a topical cortizone cream and its not working. Any ideas what it could be? Doctor: Hiu appreciate your concernLooking at your symptoms it seems to be a case of allergic reaction or giant cell arteritisplease make a visit to your doctor for the battery of tests like ESR complete blood countgiant cell arteritis which can occur in your age, needs further treatmentPlease take some anti inflammatory drugs like acetaminophen or ibuprofen.Hope this helps youthanks for using our sitehave a nice day"
},
{
"id": 76262,
"tgt": "Suggest remedy for extreme pressure in the chest",
"src": "Patient: I am 5'1, 125 lbs 36 yr old female training for a 40 lb, 50 km ruck/portage/canoe event. I have recently been training with a 13 km ruck followed by a short portage with a 90 lb canoe. During the short portage at the end of training I am experiencing extreme tightness/pressure in the chest that goes away as soon as I remove the canoe. Doctor: Thanks for your question on Healthcare Magic. I can understand your concern. You are having chest tightness and pressure after heavy exertion (exercise). So we should first rule out heart diseases in your case. So get done ecg and 2d echo first. If both these are normal then no need to worry for heart diseases. Sometimes, musculoskeletal pain due to exercise can also cause similar chest discomfort. So avoid heavyweight lifting and strenuous exercise. Avoid movements causing pain. Avoid bad postures in sleep. Apply warm water pad on affected areas. Take painkiller like ibuprofen. Don't worry, you will be alright with all these. But first rule out heart diseases. Hope I have solved your query. I will be happy to help you further. Wish you good health. Thanks."
},
{
"id": 161630,
"tgt": "What causes shivering with fever in the night?",
"src": "Patient: my nephiew is 5 year old, from last 10 days he is suffering from fever i mean first shivering is coming after than fever comes 102 degree at night time. we have gone from some tests like malaria, typhoid. malaria report is clean. i want to know what is actually? Doctor: Hi, Fever with shivering may be due to malaria and urinary tract infection. If the malaria test is negative you can do investigation for urine infection. If we treat the urinary infection properly then the fever will stop. You can give paracetamol every 6th hourly. Hope I have answered your query. Let me know if I can assist you further. Regards, Dr. Rajmohan, Pediatrician"
},
{
"id": 120935,
"tgt": "How to treat grinding in the shoulder?",
"src": "Patient: My right shoulder grinds very bad. I always thought I just slept wrong, but I do not think this is the case. I have had 2 cortisone shots in my left shoulder due to sports and my bursa sac. I am only 24, yet have already had arthoscopic knee surgery, too. I guess you can say I am prone. I am just wondering if I should go and get a cortisone shot in my right shoulder. I feel like it is bone on bone, though or maybe loose cartilege. Please help! Doctor: Hello,It is very uncommon to have arthritis in such young age.If you are having such problem again and again in multiple joints then you should have to rule out your problem by having some blood tests like:CBC RA FACTOR ANTI CCP ANTIBODIES TESTS FOR RHEUMATIC DISEASES LIKE ANTI Sm ANTIBODY TEST LIVER AND KIDNEY FUNCTION TESTS Till then you can have following to have relief:1. Avoid excessive strain during work or play.2. Take help of a physiotherapist.3. Frequent steroid shots in joint is also harmful so please take care.4. Take some joint supplements like glucosamine and chondroitin to help your joint cartilage. Hope I have answered your question. Let me know if I can assist you further. Regards, Dr. Mukesh Tiwari, Orthopaedic Surgeon"
},
{
"id": 181242,
"tgt": "What causes receding gums and dry mouth?",
"src": "Patient: I have suddenly notice receding gums in two places in my mouth. I've also been experiencing dry mouth for some time. I do suffer serious sinus issues, but have been under my doctors care for that with no apparent ty to the dry mouth. Do you have a theory? Doctor: Hi,As per your complain receding gums seems to be due to infection in gums and when the infection proceeds deeper there is loose of attachment of the gums to the teeth leading to gum recession. However dry mouth can also cause gum infection as in case of dry mouth there is loss of flushing/cleaning action of saliva leading to debris accumulation inside mouth.Dry mouth taste can be due to causes like severe dehydration, side effects of medications if you take any, salivary gland dysfunction, nerve damage supplying salivary glands, oral infections like thrush, and Sjogren's syndrome.As of now for dry mouth chew sugar free chewing gums.- Drink plenty of water.- Avoid regular mouthwash and use specialized mouthwash that are formulated for dry mouth patients like Biotene.- You can also instill artificial saliva drops.- Suck tart candies as it also stimulates saliva flow.For receding gums you should consult a Periodontist and get evaluated and deep scaling and curettage can help.Hope I have answered your query. Let me know if I can assist you further.Regards, Dr. Honey Arora"
},
{
"id": 102886,
"tgt": "Positive result after retesting for peanut allergy. History of having gastro problems. Possible missed diagnosis of allergy earlier?",
"src": "Patient: My 14 year old daughter just got re tested after 2 yrs for a peanut/tree nut allergy. She is still testing positive. She developed this allergy at age 12, but had gastro problems at a younger age. Could this allergy have been missed early on, and at a later age when she developed throat swelling and eyelid swelling at age 12? Doctor: Hi,Thanks for writing in.Your daughter is suffering from hypersensitivity to peanut. This might have gone unnoticed in earlier years because of limited exposure to the allergy causing agent. When a patient is exposed to the allergy causing agent the first time, it usually doesnt cause a significant reaction. When there is the second exposure, there will be severe reaction because by that time antibodies have developed against the allergy causing agent. That is why she is testing positive subsequently while it might have been missed in early days.Hope this helps"
},
{
"id": 129682,
"tgt": "What is the treatment for knee pain?",
"src": "Patient: Dear sir, i am taking zenretard 400 from last 12 years. from last 2 years i have no fit ( seizure) at all. But from the last 4 days i feel chronic pain in my left knee & after consulting my nearby doctor declared that there has been a deficiency of fluid in the knee joint called sinovial fluid that caused pain in your knee. Now, it is requested to your good self kindly sugest me the treatment and the medicine that i have to take for the said disease & further advice me if it happened with the drug that i have been taking from 12 years. Looking for your reply.....!!!! Peerzada Shafat Ahmad. Doctor: Hi, there!Glucosamine/chondroitin using 3-6month programm can cure joint cartilage condition.Hyaluronic acid gel injection in knee joint can give instant effect by oiling bone surfaces.I hope this helps"
},
{
"id": 129021,
"tgt": "Can Polio leg be cured by surgery?",
"src": "Patient: good morning sir,my sister was attacked by polio one leg. the polio leg had light bent. If she takes surgery can it work like normal leg? or just bend correct? is useful with surgery? but now she walks well and she polio leg is like normal leg in length . her age is 22 Doctor: Hello,Thank you for using healthcaremagic.I read your question and understood your concern.I think the polio has damaged the nerves of the leg and it can not be normal any more. Surgeries can help some but the leg will never be normal as the muscles are weak and not developed.Sorry for this unpleasant answer.Dr. Selmani"
},
{
"id": 47049,
"tgt": "How to confirm kidney disease?",
"src": "Patient: Lately, my daughter is producing many soggy diapers a day, but her stool is firm, small fecal balls. She drinks throughout the day. When she had a fever a month ago, the doctor said that she had mildly elevated kidney values indication mild dehydration.How likely is this that she could have kidney disease.? Doctor: Hi thanks for asking question.You gave not mentioned age of your child.During fever as less water drinking she might have dehydration and kidney function test comes abnormal..To look for kidney disease like pyelonephritis , stones or glomerulonephritis USG can be done....If kidney disease there causative treatment done...If cause not clear and urine show proteinuria then for further work up if needed biopsy might needed.Take care...If she developed any unusual sign and symptom consult doctor for it..."
},
{
"id": 198302,
"tgt": "What causes swollen left testicle in a child?",
"src": "Patient: Hi doctor, my son is 6 yrs old. Recently, we've noticed his left side testicle size is bigger than right, looks swollen. When I examine, I find both testicle size same, except left side swollen . Pls advise what it could be and any home first aid remedy for this Doctor: Hello!Usually the left testis is located at a lower level than right testis . You might have an illusion about its big size. Ask for associated symptoms of pain , redness of scrotum and swelling in inguinal region.Ask for any diurnal variation in size of scrotum . It may be associated with congenital hydrocele.Regards"
},
{
"id": 217450,
"tgt": "How to relieve pain in the site of blood withdrawal?",
"src": "Patient: Hi, I had 4 vials of blood drawn 6 days ago, felt nothing from it until yesterday, when it bruised and started hurting. Uncomfortable, constant sensation and mild pain up my bicep from the puncture site, especially with arm bent. I haven't exercised or done anything strenuous. How can I get rid of this? Doctor: hiit could be a local haematoma or extravastation of the blood in the local tissues surrounding the puncture.cold compression on that area will help.application of thrombophobe ointment wil prevent further complcations of local infection and painkeep the limb elevated to increase the arterial blood flow to help in healing earlier.all the best, take care"
},
{
"id": 35616,
"tgt": "Suggest vaccination to be followed as my pet licked on my wound",
"src": "Patient: my pet dog licked my wound on 1 st April 2012 around 10 : 30 pm i have taken 1st dose of Rabipur on 2 nd April 9:00 pm 2nd dose on 3 rd day and 3rd dose on 7th day and 4 th dose on 14 day due to miss calculation i had taken my 5th dose on 21 st day not in 28 th day.. is there any problem with vaccination ... my pet is healthy having no change in behavior.... one more thing last week on 30 th April 2012 again i was encounter with street dog whom i accidently i walked on his tail dog ran away i am not sure whether he bite or licked me . do i need to get rabipur vaccination once again . Please help i am very much worried Doctor: Hello and thank you for your question. You are probably not at much risk. If you are concerned there are two things you can do. You can ask your doctor to get a rabies titer. This is to make sure that you still have antibodies against rabies in your system from the last set of vaccinations. You could also get a booster of the rabies vaccine. You would not need to whole series, just a one shot booster. People who are at high risk of contacting rabies will get a booster vaccine about every two years.Thank you for using HCM"
},
{
"id": 167433,
"tgt": "Suggest treatment for accidental intake of toothpaste by a 8 month old",
"src": "Patient: i am brushing my teeth with sensodyne whitening toothpaste{ with potassium nitrate ans sodium flouride as the min ingredients and my 8 month old swallowed a bit of it... now she has a husky voice and it gets worse everyday... will she be okay.? she is cranky the whole day after that and lasts for 2 days... her voice is not back yet.? am i killing my baby.? Doctor: hello there,Thank you for posting your query with healthcare magic,first of all, I would like you to see a pediatric/general dentist who can show you the amount of tooth paste we should use for kids under 3 years of age as they cannot spit into the sink while tooth brushing.Tooth pastes contain fluoride in them which can cause upset stomach if swallowed in large amounts, also it may affect the tooth formation in babies, rarely though but we just have to make sure we are not overdosing them with fluoride.A size of a grain rice is the amount of toothpaste which we as dentist recommend on the toothbrush for babies teeth .If you have any further questions, feel free to post them here.I hope I answered your query,thank you"
},
{
"id": 125863,
"tgt": "How can leg cramps be treated?",
"src": "Patient: I have been having a lot of leg cramps. Mostly in my thighs and feet. I have been on Cymbalta for about 2 years for nerve pain from a abrasion on my left inside ankle due to cirrculation problems. I have had vein strippedin my right leg and a vein cauterized in my left leg. Tried bananas and drinking more water. Not working. Do wear a compression sock on left leg. Doctor: Hello, It is due to dehydration. As of now, drink plenty of water and take bed rest. Generally, the symptoms will settle in a couple of days. If symptoms persist better to consult an orthopedician and get evaluated. Hope I have answered your query. Let me know if I can assist you further. Regards, Dr. Shinas Hussain, General & Family Physician"
},
{
"id": 21191,
"tgt": "What causes pain and redness around entry point are of catheterization?",
"src": "Patient: I had a heart catheterization four days ago. Yesterday I must have strained too much and now I have redness and firmness around the entry point but it is not affecting the entry point which appears fine. I want to wait to see my PCP tomorrow as soon as possible. Is that a reasonable decision? It hurts but the pain is where the redness and swelling are, not at the catheter entry point. Doctor: helloi have gone through your query. the thing you are having is an hematoma- a blood collection between the tissues under the skin. it happened because of straining, the artery which was cannulated had a small leak when you strained. you must watch out for increase in sizeof the swelling. please contact your doctor. allow rest to the limb affected including weight bearing. also if possible keep a cold compress. it may be red now. and the color may vary from red to blue to green to yellow before disspaearing. it will take around 15-20 days to dissapear. however increase in the swelling should be check for. you may also be adviced to undergo a sonography of local area to see the extent of the hematoma if its quite large or your doctor thinks so. thank you."
},
{
"id": 93421,
"tgt": "Really bad abdomen cramping, getting nauseous, dizzy. What's wrong with me ?",
"src": "Patient: My abdomen was cramping really bad. To where I couldn't think straight. I thought maybe I needed to use the bathroom or was about to start my period. When to the bathroom only thing that happen was cramps getting worse and slowly started getting nauseous. Got up to go to the living room, took bathroom trash can just in case I threw up and started getting dizzy and cold sweating. I grabbed my phone and hurried back to the bathroom and hurried to the floor. I set there for a couple mins then got back on the toilet and ended up using the bathroom. What is wrong with me? Doctor: Hi ! From your history it seems that you have got into a food poisoning either by eating unhygienic food from outside or by consuming some stored food at home which without your knowledge has become stale. This is why you had cramp in your abdomen followed by nausea and sweating with dizziness which possibly was vaso vagal reaction to the sensation of pain due to cramping. It got settled when you went to the floor. At the end you went to bathroom although you did not mention if you had loose stool or not. Although there are several other causes of cramping abdomen with nausea, I will suggest you to visit your family physician to have on the spot clinical examination of your abdomen and possible antibiotics and antispasmodics if he agrees with my provisional diagnosis. Wishing you an early recovery."
},
{
"id": 188020,
"tgt": "What could be the chances of getting jaw bone affected while pulling off cracked molar?",
"src": "Patient: I had Reclast on September 18, 2003. On November I realized I had a small sore between my molars on the right side of my face. After going many times to the dentist and having taken different antibiotics, I learned today that my molar ((#32) is cracked all the way down! Now my dentist will have to pull the tooth out. I am wondering what are the chances of me getting my jaw bone affected by this procedure? and who should remove this tooth?thank you so much Doctor: hello and welcome to HCM forum,I would like you to know that as the wisdom molar is cracked , it needs to be extracted by an experienced dentist, it can be a specialist or a general dentist.You do not have to worry as it is not something too serious.Extraction of a molar does not affect your jaw in any way ,just follow the instruction given by your dentist post-extraction.I hope I answered your query,i wish you good health,take care."
},
{
"id": 109283,
"tgt": "Suggest treatment for lower back pain",
"src": "Patient: My husband has has sever lower back pain ever since we have been together. He has had a lower spinal fusion and numerous test's done and every doctor has a different answer. He is only 30 and feels like 80 year old man. He is now having pain in his wrists, which sweel up, sholders and sometimes his legs and arms get the tingly sensation. Lately he has been getting extremly bad migraines. We have been to the hospital for chest pain's and there are still no answers. Before the doctors were just giving him scripts for pain killers and depression. He has stopped taking it all because it wasn't doing anything. We need help.. Doctor: Hello, I have studied your case with diligence.As per your symptoms there is possibility of pain due to bad posture as your back is involved,.If not relieved then possibility of polyarthritis needs to be consideredRheumatoid arthritis may involve small joints and early morning stiffness in joints, do you have such complain?Ankylosing spondylitis typically involve spine and hips with other large joint also. There is reduced chest function in this disease. Stiffness in spine is increased do you have such stiffness in spine? When such patients come to our hospital I advise them blood test like RA factor, ACCP, ESR, complete blood count, HLA B27Hope this answers your query. If you have additional questions or follow up queries then please do not hesitate in writing to us. I will be happy to answer your queries. Wishing you good health.Take care"
},
{
"id": 167662,
"tgt": "What causes child to wake up in terror from sleep?",
"src": "Patient: Hi my 19 month old daughter will sleep through from 7.30pm until 1.30am and then wakes up in terror, she has all her teeth through. however she can go up to 4 days with out going for a pooh. could this be the reason she wakes up in the night for up to 3 hours? Doctor: night terrors in children is usually caused by stress , either psychological stress or physiologic stress .try to look for any thing that bothers her or frightened her , if there is a social issues try not let her involved in that .constipation might cause a physiologic stress but rarely if ever can cause a night terror , for this condition give her miralax sachets once a day or lactulose syrup 5 ml twice daily and that will loose her stool . I hope this helps"
},
{
"id": 15452,
"tgt": "Sore on knee, spreading into more pimply blister, keep oozing, skin red and inflamed. What is this ?",
"src": "Patient: I am curious about this sore i have on my knee. It started as a small water blister after i went swimming one day, and then about a week later, two more pimply blisters popped up on my knee, and now they are growing. The sores keep oozing, and the skin is red and inflamed and peeling away. this has been going on for about a week. Doctor: Hello,Thanks for posting on HCM,It seems to me that you are having a soft tissue infection probably due to a bacterial infection. You will need to have that examined by a doctor who can then confirm or exclude my suspicion. You might require a course of antibiotics such as amoxicillin or cloxacillin associated to antiinflammatory orally. Some topical steroids could be applied if your doctor deems it necessary. Hope you feel better"
},
{
"id": 281,
"tgt": "Can non-penetrative sex lead to pregnancy?",
"src": "Patient: I had a very weird period from May 2-May 6 or 7. I took a test a few days after my period was over just to make sure, and it came out negative. Even though my boyfriend and I have never gone past foreplay I thought I was pregnant so I'm still very traumatized about that pregnancy scare. We 'foreplayed' again a few days after that test. He placed his hand in his pants to fix his you know what and then he wiped it on his jeans, he wiped it again on my shorts before inserting his fingers.He said there was no pre-***. I took a test two days after that. Still negative. I'm thinking this is all just the effects of the pregnancy scare a week ago and I'm just traumatized but I just want your opinions. I think I'm supposed to be ovulating around now.. but I'm still feeling dry and my CM isn't clear or a lot.. it's just some drops of white when I wipe with my finger. Usually, there's a lot of CM after my period.. but this time around there's barely any. It feels warm down there and a little dry..Is there a possibility I could STILL be pregnant? I can't be, right? I mean, we've never had actual sex. Please help me ease my mind. I really don't want to be thinking of these things anymore. It's really hard to concentrate on anything else. :( Doctor: Hello and Welcome to \u2018Ask A Doctor\u2019 service.I have reviewed your query and here is my advice.Sperm needs a liquid media to get transported. If he has not ejaculated inside then you cannot get pregnant. Pregnancy cannot occur on foreplay.Hope I have answered your query. Let me know if I can assist you further.Regards,Dr. Sheetal Agarwal"
},
{
"id": 57201,
"tgt": "How long do SGOT and SGPT levels take to return back to normal levels?",
"src": "Patient: hiI had dengue fever 2 weeks back. now i have recovered completely and is dengue non reactive. however my sgot and sgpt readings are very high at 300 and 150 respectively. i did a sonar of my liver and it is normal . how long does it take for the sgot and sgpt readings to be back to normal levels? Doctor: Hello Thanks for writing to HCMIncreased SGPT indicates liver injury.SGOT is non specific and it increases in many conditionsIncreased SGPT may be due to hepatitis,altered lipid profile,autoimmune causes,medicines etc.You need clinical examination and few other investigations.You need few more investigations like random blood sugar(RBS),renal function test(RFT),Lipid profile,viral markers,complete liver function test,urine RE/ME.You should avoid red meat and high fat milk products.You should avoid fried and junk food and avoid alcohol completely if you take it.Medicines should be prescribed after complete investigations.I suggest my patients to take tablet ursodeoxycholic acid 300 mg twice daily after food for three months.This help in regeneration of liver cells.It may take 2-3 months for complete recovery.Get well soon.Hope I have answered your question.Take CareDr.Indu Bhushan"
},
{
"id": 197386,
"tgt": "Does phimosis affect sex life of a person?",
"src": "Patient: Hi, I am having phimosis since my birth, I come to know about this disorder about a year ago, since then I am so tensed about my sex life ahead. I have completed 19 last week, still my foreskin is too tight that even when it is completely erected there is no sign of outcome of the penis from foreskin. Please suggest me the best way to get rid of this disorder. Thank you. Doctor: Hi thanks for contacting HCM...Let me know you phimosis does not need emergency treatment ....You have to consult surgeon for your examination...Phimosis create problem in doing sex...Because of phimosis fore skin can't retract and bleed while doing sex with pain....Few surgeon advise topical steroid for mild phimosis .But you have it since birth and moderate degree .So you have to do cicumsicion procedure with release of frenulum for that....It is minor operation ....Get treatment within 2 to 3 month as per your convinince and after operation your sex life will be good...Take care ..."
},
{
"id": 160065,
"tgt": "Right testicle harder & a little bigger, please help",
"src": "Patient: My rt testicle feels harder than my left & a little bigger. Friday, I went to my Urologist & she said she felt no difference. A sonogram of my rt testicle was negative. Should I seek another opinion ? Doctor: Hi, Welcome to Health Care Magic. Thanks for your query. when the clinical examination is correlated with the reports and even you don't feel pain then you need not to be worried about this.If you feel pain then get the sonography of scrotum done at some other place or center. Thanks and take care"
},
{
"id": 169591,
"tgt": "Could axillary and vagina hair in 6-8 year old kids be related to puberty?",
"src": "Patient: i am a deeply concerned mom..our two kids have seen a pediatric endocrinologist . our 8 yr old son( will be 9 this month) has some mild axillary hair. our 6 yr old dtg has some mild vagina hair. both are very tall, bone scans close to 2 yrs ahead. we were told the labs were all normal and the physician assures us that the hair is not puberty related. w are just are deeply concerned and have considered a second opinion but that takes 6 months... any suggestions?? Doctor: could be signs of precocious buberty and other endocrine problems and should be evaluated further with an endocrinologist."
},
{
"id": 97889,
"tgt": "Lung Scar medicication",
"src": "Patient: My daughter,who is 21,is under medication for PTB. I would like to know if her lung scar can be healed. Doctor: Hi It will heal with time. She can lead a normal life, without worry. Good nutrition and exercise to keep her healthy , fit and build immunity is must. Takecare for further queries- dranshita.rathore@yahoo.co.in"
},
{
"id": 140054,
"tgt": "How to deal with the rapid weight loss after TBI?",
"src": "Patient: i suffered a tbi brain injury in 2005 and have seizures from time to time, and no insurance currently over the last month they were pretty bad i lost 30 pounds in like 3 weeks i had hundreds i am feeling pretty good now and have not had any seizures in a couple of weeks now ... i was pretty week for a bit but i am eating again but my hands started swelling my blood pressure is good... i know the rapid weight loss is not good what is the best treatment for this ice, maybe a little salt elivate let me know Doctor: Hello, In your case is necessary to control seizures with medication. For the weight loss and for better control of seizures it is necessary to follow a healthy lifestyle in the direction of food, exercising, getting enough sleep, avoiding alcohol, etc. Hope I have answered your query. Let me know if I can assist you further. Take care Regards, Dr Erion Spaho, Neurologist, Surgical"
},
{
"id": 9251,
"tgt": "What causes itchy dry skin and eyes?",
"src": "Patient: I m afraid to go see my doctor, but I d like to have an idea of why I feel the way I hVe been. Itchy crawly dry skin, yellow/red glossy/itchy eyes. Night sweats also lots thru out the day even in A/C atmosphere.always irritated and ice lost the patience I ve always had with everyone. Doctor: Hello,I can understand your concern for itchy skin and dry eyes along with yellow glossy look of eyes. In case if you are on any medications then it is better to get checked with your treating doctor. I suggest my patients to apply a lot of moisturizing creams along with emollients. It is better to apply it just after bath so that the hydrating property of the water can be retained after that. It would be better if it could be applied for multiple times in a day. For the part of dry and yellow glossy look of eyes I suggest you to get yourself examined by a eye specialist.All the best."
},
{
"id": 66708,
"tgt": "What causes a lump above the collar bone?",
"src": "Patient: I found an olive sized lump above my collar bone. It is loose and sort of moving around. My neck has been stiff for weeks and I get very tired each day. I thought it was mono (I had glandular fever a number of years ago) but I m not sure about the lump Doctor: This one is a lymph node or could be some lipoma / neurofibroma; not to worry much about it!Please go for an FNAC test to relieve your tension as a lymph node in this location might be a sign of some serious disease elsewhere!All the best and regards,"
},
{
"id": 10751,
"tgt": "How can hair fall be treated?",
"src": "Patient: sir.....my name is mahesh from last 8 months iam suffring with hair fall problem.iam mentally deactivated with this problm.mainly iam losing hair frm my front scalp.iam using nuzen herbal hair oil...but i think its working poor on my head....sir pls help me......my age is 21... Doctor: HiAs per your complains u r having androgenic alopecia.It is very common in your age-group. Nothing to worryStart using Minoxidil 2% solution twice daily (Tugain) and take biotin tablets (follihair) daily once for 3 monthsResults will be gradual and appreciable changes will be seen around 3 months.Thanks"
},
{
"id": 139058,
"tgt": "Suggest treatment for pain in the shoulder and arm",
"src": "Patient: I have a pain in the right arm this pain runs along the outside of the arm from below the elbow to the shoulder, the head, the neck and below the bone under my shoulder. What are the causes? I am diabetic. I have to hold my arm behind my head to get relief or lie down with my arm outstretched. Do I have diabetic nerve pain? The doctor put me on naproxen but it is not helping. I think it elevates my bp. How can you help. Thank you. Doctor: Dear patient You are having typical signs of cervical nerve root compression. Pain that is bit relieved by elevation of arm behind head is typical of cervical disc prolapse with nerve root compression. I would advise mri of cervical spine with screening of whole spine to confirm diagnosis. MRI gives detail of level of compression and cause for it so that treatment can be planned. Visit radiology center nearby you and get it done. Meanwhile start tab vitamin B12 plus pregabalin 75mg at bedtime for 10 days. Need to take visit to spine surgeon with mri report. All the best."
},
{
"id": 122144,
"tgt": "Suggest cause & treatment for pinging pain with a crack in shoulder",
"src": "Patient: Hi, 4 months ago I had shoulder surgery, SAD and ac joint excision, several weeks after this I was in bed and there was an almighty crack with severe pain, and has been really bad since, I feel a pinging pain from my shoulder to half way down my arm, this is also swollen, please help Doctor: Hello, Your symptoms could be related to a ruptured ligament. For this reason, I recommend consulting with your orthopedist and performing an X-ray study of the shoulder joint. Inflammation tests (complete blood count, PCR, ESR) should be checked too. Hope I have answered your query. Let me know if I can assist you further. Take care Regards, Dr Ilir Sharka, Cardiologist"
},
{
"id": 118183,
"tgt": "What is the cause of high ferritin levels?",
"src": "Patient: I have a low mpv count of 6.5 and have had recent problems with my blood possibly associated with tumors that were removed.. I am also having frequent urination, fatigue and dizziness, aches.. prior to the removal of my tumors I had extreme high ferriten levels 9.1 hgb and a raised ca 125... the doctors believed the tumors were ovarian cancer but upon removal none were cancerous..any ideas Doctor: Although extremely elevated ferritin levels may be associated with rheumatologic diseases, more often they are found in patients with other conditions such as malignancy or infection. In addition, extremely high ferritin levels can be found in patients with seemingly indolent disease or levels of chronic inflammation.RegardsDR De"
},
{
"id": 152431,
"tgt": "Are prominent lymph nodes at the sternum a sign of cancer?",
"src": "Patient: I was diagnosed with lung cancer in 2006 and the surgeon removed my right lung and now about 1 and a half months ago I had a heart attack and spent two weeks in cardiac ward.. While there they discovered several limp nods through my sternum.. they are very concerned .. did biopsy .. now talking about radiation treatment... does this mean cancer or a concern it may turn cancerus Doctor: Hi, The most common symptom, if cancer has spread to the lymph nodes, is that they feel hard or swollen. You might have any of the following symptoms if your cancer has spread to the lymph nodes: a lump or swelling under your armpit. Else, a lump or swelling in your breast bone or collar bone area. Hope I have answered your query. Let me know if I can assist you further. Regards, Dr. Esmeralda Sera, Oncologist"
},
{
"id": 120691,
"tgt": "Putting off carpel tunnel surgery will cause any risk?",
"src": "Patient: i just got a job doing alot of hard work with my hands and i have carpul tunnel and my knuckles are swollen and th ey hurt and its hard and sometimes impossible to make a fist on both hands and my left hand is always numb but when i get to work they are numb for a little while, then i work them in and i can use them better but when i get home it comes back full force i wear my wrist splints at night but it never gets better i cant take time off of work because i just got the job and there is a six month probationary period. is there anything i can do to putoff the surgery for a while and what are some of the risks of puting the surgery off Doctor: Hello,I read carefully your query and understand your concern. Your symptoms seem to be related to carpal tunnel syndrome. I suggest to try conservative treatment before surgery.I suggest:-Wrist splinting. A splint that holds your wrist still while you sleep can help relieve nighttime\u00a0symptoms\u00a0of tingling and numbness-Nonsteroidal anti-inflammatory drugs such as Tylenol to treat the pain.-Corticosteroids which should be taken on prescription. Hope my answer was helpful.If you have further queries feel free to contact me again.Kind regards! Dr.Dorina Gurabardhi General &Family Physician"
},
{
"id": 54925,
"tgt": "How to control high SGOT and ESR level?",
"src": "Patient: I have reported with very high SGOT level with my recent liver enzyme test (115), and my ESR level is also increased up 55 mm, what does this indicate, and how should I control this condition? ( I'm a 52 yrs old High blood pressure patient taking medicines over 15 yrs.) Doctor: Hi thanks for asking question....Whenever there is inflammation or infection or cancer in body ESR can be high....You have elevated liver enzyme also.So hepatitis like condition has to be ruled out.USG done if show finding of hepatitis or abscess or any tumour.You can have drug induce hepatitis also.You have not mentioned drug you are using for hypertension...Take low fat diet.Fruits more.One tsp licorice with honey taken.If you have heart problems or celiac disease or muscular problem then also it can elevated.If needed viral marker also can be done as further work up.Dr.Parth Goswami"
},
{
"id": 177515,
"tgt": "What could cause fever, red gums, mouth ulcers, foul breath and fatigue?",
"src": "Patient: 2 year old, fever, red gums, mouth ulcers, white tongue, bad breath, tired, won t eat much- gets worse at night, having screaming fits every hour but asleep but obviously in pain, have been giving cal pol and treating the fever, this is day 4- been to GP, no infections present, what is it? Doctor: Thanks for your query, I have gone through your query.The fever, ulcers, bad breath can be a herpetic virus infection resulting in herpetic gingivostomatitis. Nothing to be panic, its a self limiting condition. You can take symptomatic treatment like paracetamol for fever, topical anesthetic like lignocaine and topical analgesic like choline salicylate for ulcers. Do saline gargling. If the lesions does not heal in a week, consult a oral physician and get it evaluated.I hope my answer will help you, take care."
},
{
"id": 97659,
"tgt": "Are there any homeopathy cure for JME?",
"src": "Patient: Is there a homeopathy cure for JME, my son had a seizure and the EEG says \"an abnormal record with several generalized epileptiform abnormalities in keeping with idiopathic generalized epilepsy\" He had been having hand jerks for some time and yesterday had the first seizure in bathroom with his hand shaking Doctor: **1. JME [juvenile mycoclonic epilepsy] is shock like momentary contraction of muscles of a limb or the whole body, and since cause was 'idiopathic' even then secondary causes like: intracranial tumor, tuberculoma,cysticerosis should be ruled out with relevant investigationsPS. treatment is symptomatic and the same whether epilepsy is primary or secondary.2. since the person is able to think clearly even during the attack, thus no first aid is usually required.3. Because myoclonic seizures are often associated with certain syndromes, thus I won't advise any alternate cure, till he is seen by a doctor to make a diagnosis and then begin the treatment4. Common instructions which should be kept in mind/are:. do not miss the treatment even for one day [when started].. avoid places where convulsions can be dangerous. e.g. swimming,driving, climbing heights,work near moving machinery, standing in train or bus doors, work near fire.. avoid fat rich diet [since fat is known to precipitate attacks of epilepsy]"
},
{
"id": 37109,
"tgt": "Can steroids intake lead to recurring reflux infections?",
"src": "Patient: I was admitted to the hospital last Thursday with chest pain. I told them I had had quite a bit of steroids this summer and fall for sinus infections, bronchitis, etc., and they said that sometimes too many steroids can lead to reflux returning, even though I had surgery for reflux 10 years ago. What can you tell me about that? Doctor: Hi,It seems that you might be having hyper acidity or gastritis mostly due to taken plenty of steroids.Cut the dose of steroid after consulting your treating doctor or take medicine with omeprazole type of medicine.Take Omeprazole or Rabiprazole for few days.Take milk.Avoid fried, chillies and junk food.Ok and take care."
},
{
"id": 77792,
"tgt": "Suggest medication for frequent chest pain",
"src": "Patient: My husband has been getting what he thinks is chest pain for the past 5 days. He is 60 years old. What he does to help is lay down with a heating pad, take Gas X pills and this usually helps. This happens the same time every day. Between 3:00 in the afternoon until maybe 5:00 PM. After he does his routine, he is better. He did make an appointment with his physician for tomorrow. His doctor said on the phone it doesn't sound like something to really worry about. He has been seeing this physician for many years and has faith in him. What do you think? If there are any other questions you need answered, feel free to ask. Thank you Doctor: Thanks for your question on Health Care Magic. I can understand your concern. In my opinion, his chest pain is mostly due to GERD (gastroesophageal reflux disease) because it is improved with antacid (gas x pills). But better to rule out heart diseases because he is 60 years old and heart diseases are common at this age. So consult doctor and get done blood pressure monitoring, ecg and 2d echo. If all these are normal then no need to worry for heart diseases. His chest pain is more likely due to GERD. So ask him to avoid stress and tension, avoid hot and spicy food. Avoid large meals, instead take frequent small meals. Take proton pump inhibitors and prokinetic drugs regularly. Quit smoking and alcohol if he have these habits. Don't worry, he will be alright. Hope I have solved your query. I will be happy to help you further. Wishing good health to your husband. Thanks."
},
{
"id": 14828,
"tgt": "What to do for rashes on chin?",
"src": "Patient: i recently developed a rash on the sides of my chin. Over the last 3 weeks it has spread to my whole face, behind my ears, and starting on my chest, neck and upper back. Its small red spots, most of which are raised and scaly feeling. Sometimes some of them itch but there s no pain Doctor: Hi,According to your history, looks like you have seborrheic eczema.. ie dandruff over scalp .\u00a0Seborrhoeic dermatitis\u00a0(also known as \"seborrheic eczema\" )is an inflammatory\u00a0skin disorder affecting the scalp, face, and torso. Typically,\u00a0seborrheic dermatitis\u00a0presents with scaly, flaky, itchy, and red skin. It particularly affects the sebaceous-gland-rich areas of skin. In adolescents and adults, seborrhoeic dermatitis usually presents as scalp scaling similar to dandruff or as mild to marked erythema\u00a0of the nasolabial fold.Dermatologists recommend topical treatments such as shampoos, cleansers or creams/lotions that contain antifungal, anti-inflammatory, sebo-suppressive or keratolytic\u00a0ingredients:One combination approach combines a dandruff shampoo, antifungal agent and topical steroid\u00a0.Hope it helpsDr Geetika PaulMD ( Dermatology)"
},
{
"id": 92286,
"tgt": "What treatment do you suggest for severe abdominal pain just like spasm ?",
"src": "Patient: I am having severe abdominal pain into left side. And in my lower left back. So bad that sometimes I'm doubled over. Sometimes in comes in like spasms and feels like someone just poked a knife in me. What is it? I had a cystoscopy done on Tuesday, would that have any affect on me? Doctor: Hello!Thank you for the query.Abdominal pain (lower left or right) and back pain can be caused by urinary tract infection. Such infection can appear after a cystoscopy. You may also have frequent urinating, burning while urinating or blood in urine with an infection.Lower left abdominal pain can be also caused by diverticulitis of the large intestine. This condition is caused by consitpation. Other reasons like ovarian cyst or even pregnancy should be also considered.I suggest you to visit your doctor and start with urine analysis and abdominal ultrasound.Hope this will help.Regards."
},
{
"id": 135214,
"tgt": "What causes the leg to be cold and blueish colour?",
"src": "Patient: Hello. I rolled my right ankle about 10 days ago in the park, on the uneven grounds/grass. I did not have any bruising, however, it swelled up slightly right around the bone. I didn t see a doctor because it didn t seem serious as the pain was manageable. I am on crutches now because it is painful to walk on it or put any weight. I have used the RICE method. My concern is that my leg is always cold, whether compressed or not. It changes colours ie blueish, purple from time to time and the it is very tender around the bone. Are these normal symptoms? Doctor: Hello,I have studied your case and I think you have ligaments injury .There is collection of blood in the tissue and this is getting dissolve so you are seeing colour changes. I would suggest you to take a x ray and make sure that it is not a hairline fracture. You will need splinting. Walking cast can be applied so that you can walk also. I hope this answer will be useful for you.Let me know if there is any other followup questions.thanks"
},
{
"id": 173665,
"tgt": "What could cause red welps on thighs in a 8 years old child?",
"src": "Patient: My 8 year old daughter has been waking up (not every day but more regularly lately) with large, red welps on her thighs, front and back and sometimes on her buttocks. We washed her sheets and I ran through another rinse cycle and took her comforter off her bed and just used a regular cotton quilt. Do you have any idea what could be causing this. It always goes away within and hour of her getting up Doctor: Hi...Thank you for consulting in Health Care magic.By what you quote it should be an urticarial or a simple skin allergy. You can use Hydroxyzine at 1-2mg/kg/dose every 6th to 8th hourly for 7-10 days. Most important thing to be remembered is that it has a propensity to recur (called as second crop) within 10-14 days. If this happens, you can start using the same medicine but I suggest you get the kid evaluated with your paediatrician.Hope my answer was helpful for you. I am happy to help any time. Further clarifications and consultations on Health care magic are welcome. If you do not have any clarifications, you can close the discussion and rate the answer. Wish your kid good health.Dr. Sumanth MBBS., DCH., DNB (Paed).,"
},
{
"id": 43469,
"tgt": "Semen analysis done, showed spermatozoa count, motility, pus cells and epithelial cells. What does that indicate?",
"src": "Patient: Hi,Please find my semen analysis report below and let me know if any issue is there:-time of collection:12:00 pmlab receiving time:3:00pmDays of abstinence: 4daysPHYSICAL EXAMINATIONvolume: 5mlcolour:grey-whiteliquefaction time: 30minreaction:alkalineviscosity: normalMICROSCOPIC EXAMINATION:Spermatozoa count: 40 millions/mlMOTILITY: (after 1/2 hour incubation)Actively motile:50%Sluggishly motile:20%Non motile:30%MOTILITY:(after 1 hour incubation)Actively motile: 40%Sluggishly motile:25%Non motile:35%MOTILITY:(after 2 hour incubation)Actively motile:25%Sluggishly motile:30%Non motile:45%MORPHOLOGY:Normal forms:70%abnormal forms:30%OTHERS:Pus Cell: 4-5/H.P.F.Epithelial Cell: AfewR.B.C: Nil Doctor: Hello! I believe your sperm analysis is within the normal range set by the WHO. I dont see any problems with your analysis. Best of luck."
},
{
"id": 183441,
"tgt": "What are the dark areas around my teeth?",
"src": "Patient: My dentist found a couple of areas that showed dark around a couple of my teeth and suspicious areas around the mandible. He sent me to an oral surgeon and he said I would need a CT scan as the X-Ray does not give them enough information. Is this an unusual occurrence? YYYY@YYYY Thanks,Frances Stein Doctor: hellooo...frances stein..it can be some small cysts in the root of the teeth..nothing to be worried in that..some cases how much ever good xrays v take cant get proper information of the problem...so surgeons ask for ct..nothing to be scared in that..infact its good and its can diagnose the problem more clearly and can be treated properly...so be cool....nothing to be worried at all...be calm and meet ur oral surgeon and get it treated..hope ur benefitted something from this reply..have a healthy day!!"
},
{
"id": 26883,
"tgt": "Is heart palpitations and drinking alcohol related?",
"src": "Patient: I have been having heart palipitations every day for a couple weeks straight now. I want to know of it has anything to do with drinking alcohol. i am a daily drinker, like 6-8 beers a day but have been trying to cut down and am wondering if the palipitations are a result Doctor: Hello!Thank you for asking on HCM!I read your question carefully and understand your concern. Alcohol can cause arrhythmia in people with normal hearts and can cause heart conditions that result in arrhythmia. People who drink heavily can develop a weak heart (alcoholic cardiomyopathy). Alcohol can directly injure heart cells and cause extra heartbeats and supraventricular tachycardias. Usually the cardiac function comes back to normality after stopping alcohol intake. But for some days, anxiety and withdrawal symptoms may be related to temporary cardiac arrhythmia. I recommend consulting with the GP and performing a routine blood test, kidney and liver function tests, resting ECG and blood electrolytes, to exclude other causes of this symptomatology. Hope to have been of help!Best wishes, Dr. Iliri"
},
{
"id": 74661,
"tgt": "What is the treatment for chest pain and blurry vision?",
"src": "Patient: I'm 24Was waring a patch and now my chest hurts vision is blurry and my whole left arm is numbI ate nothing but coffee And two nubs atop my neck really hurt I don't have any inurence and have a kid and no jobIs it nessasary to call a abulance because I cannot drive now Doctor: Hello,Yes it's an emergency go to ER immediately. It may be due to heart failure. You need to immediately contact cardiologist.Hope I have answered your query. Let me know if I can assist you further.Regards,Dr. Siva Kumar Reddy"
},
{
"id": 76059,
"tgt": "What causes dry cough mass in lungs?",
"src": "Patient: Hi I am a female 66 years of age had chest xray because of persistent dry cough mass was seen and CT scan was recomended results was mass in rt. lung. pulmonologist did a bronchoscopy could not reach the mass to get sample but also saw a blackish obstruction in one of the airways now I am referred to a surgeon for further evaluation what could this be? Doctor: Thanks for your question on Healthcare Magic. I can understand your concern. At your age of 66 years, mass lesion in lung is most likely due to malignancy (cancer). And constant coughing, blackish looking obstructive thing in airways etc favour lung cancer more. Since bronchoscopic Biopsy from the mass is not possible, you need open surgical lung Biopsy from the mass for confirmation of the diagnosis. This is the reason why you have been referred to surgeon. So consult surgeon and get done Biopsy frome this the mass. Possibility of malignant mass in more. Hope I have solved your query. I will be happy to help you further. Wish you good health. Thanks."
},
{
"id": 184678,
"tgt": "Suggest remedy for dental problem",
"src": "Patient: Hi im 20 im 5'8 190 lbs . . . i have been suffering from toothaches on a chipped moler for about 6 months now but have been putting off the dentist due to lack of health insurance , i had consulted a dentist a couple months ago and she told me i had to have a root canal or pull it out but like i said i put it off . . .about 4 days ago i had a toothache come on again in the same spot i always get them in but a little more painful this time .. . i had a tiny small abscess on my gum that went away , i took some penicilin ( still taking ) and 600 mg motrin for the pain ....now the pain is deff gone but yesterday morning i noticed i have some sort of lump or raised area under my jaw like under my sking but under the side where the tooth hurts . .like i said the pain is gone now but i still have like a raised area that does not really hurt but if i open my mouth to wide it aches just a bit . . i have no idea what it could be ? idk if it is related to the tooth infection or something else . . because i am really in no pain at all just the raised area under my jaw line undr my skin . . . i dont know if it is just swollen or something else i am really worried..pleas help, thanks alot! Doctor: Hello,You describe symptoms of an infection not fully treated and going through different stages. Realize that the source of the infection has not been eliminated since you have not removed or treated the tooth. You may be developing a resistant infection. Infections spread and can cause a generalized swelling or pool into an area with a pocket of pus. This may need to be drained and require additional antibiotic treatment. Lymph nodes can become swollen with dental infections present. Infections flare up and calm down with antibiotics and anti-inflammatory medications, but you risk a more serious ,long term infection being more difficult to control. Stay on the anti-inflammatory medication Motrin. Make an appointment with your dentist before the condition gets worse. Once you treat the tooth, I expect your dental health to improve, since this is most likely related to the abcessed tooth. Consider having a dental exam and cleaning for an overall evaluation of your dental health.I understand your concern with cost and not having dental insurance, but dental conditions untreated tend to get worse. This means additional dental treatment and more costs. Many dental offices offer payment plans or work with health credit cards that offer a payments plans with a period of no interest .Thank you for your inquiry. I hope you seek treatment and feel better soon. Please ask any additional questions and I will be glad to offer assistance."
},
{
"id": 73574,
"tgt": "What causes chest pain and dizziness?",
"src": "Patient: Hi i have been having pains in my chest for several weeks now. Starting a few months ago the pain (like stitch) under both breasts now its predominatley on the left side but also on the right side. I Have had an ECG which showed nothing and i have had blood tests. I don't know what this rules out for a diagnosis. I was also having spells when i thought i was going to faint but never did and feel very heavy like i was being weighed down.The funny spells would go quite quickly but it would result in an upset stomach. Its mainly the chest pains now which seem to be all the time. Doctor: Hello dearWarm welcome to Healthcaremagic.comI have evaluated your query thoroughly .* This seems in relation with different issues possible as - low levels of iron , vitamins , trace elements - inadequate hydration - underlying stress or anxiety - reflux stomach acid - others .Hope this will help you for sure .Regards ."
},
{
"id": 49067,
"tgt": "Why am I having kidney pain?",
"src": "Patient: I have a cyst on my liver it is called a simple cyst but has grown from 6x5x5mm to 10x7x8 mm since 2012 should I be concerned about this ? Also have developed a 5mm hyperechhoic focus without shadowing in my left Kidney, I have been experiencing kidney pain and very yellow smelly urine at times. Not sure what I should do about this it seems to clear up when I increase my water intake. Doctor: HIThank for asking to HCMIf you do have liver cyst and you are asymptomatic then nothing to worry right now but this should never be neglected further investigation need to be done to manage this cyst, now for the kidney it seems a renal infection better to get done the urine culture sensitivity test this would rule out the possibility of infection, hope this information helps you, have nice day."
},
{
"id": 92566,
"tgt": "Pain in lower abdomen. Is this ligament pain?",
"src": "Patient: My 21 yr daughter is having her first baby and she is experiencing pain in her lower abdomen right side. I had her lay down with her knee up with a pillow underneath. The pain has lessened. I told her I thought it was ligament pain. She is married and does not live with me. I am on the phone with her. She she be concerned? Doctor: Hi,Welcome to HCM.If the pain is very severe, then she should get in touch with her gynecologist for check up. I doubt she is having ligament pains. If she is more than 28 weeks pregnant, then she is supposed to lie down in left lateral position. She might have stretching pain due to the growing uterus.Examination by a doctor is necessary to find out the cause.Thanks."
},
{
"id": 16696,
"tgt": "What causes heart fluttering and dizziness while having mechanical aortic valve?",
"src": "Patient: I have experienced on and off heart fluttering over the last couple of weeks. Had some dizziness once or twice. I think it may be caused by caffeine and/or stress, perhaps low blood sugar. I reduced my caffeine intake. This occurrence has happened twice before in my lifetime and it eventually went away. The first Cardiologist did check me out and prescribed a beta blocker and the second Dr. checked me out with a monitor and told me that it will probably would go away and it did. I have a mechanical aortic valve and take warfarin. Doctor: Hello, I would explain that your symptoms could be related to ectopic heartbeats. As the Holter monitoring has not shown any serious cardiac arrhythmia, there is nothing to worry about. Hope I have answered your query. Let me know if I can assist you further. Regards, Dr. Ilir Sharka, Cardiologist"
},
{
"id": 35828,
"tgt": "Suggest remedy for ring worm like infection on body",
"src": "Patient: Morning Doctor? Dr,i have sort ring worm like infection around my neck,arm pits,waist lower abdomen,mostly parts which sweat,it itches me alot and i scratch them so hard,it gets warse when its asunny day because of sweating,i haved tried to apply candiderm cream and whenever i apply it,kind of heals,when i stop smearing myself.it re occurs what could it be Doctor,need your help Doctor: Hello,Better stop applying candiderm cream as it contains steroids.Apply plain antigpfungal cream like CLOTRIMAZOLE.Take oral antifungal tabket like FLUCANAZOLE 150 mg every week for 6 weeks for complete eradication.Apply antifungal dusting powder in the daytime.Dry your body well after taking bath and don't share your clother with anybody.Wear cotton clothes and loose cotton undergarments.Thanks"
},
{
"id": 22811,
"tgt": "Are irregular heart beats a serious problem?",
"src": "Patient: My husband who is 72 felt dizzy and fainted yesterday morning. He was nauseated and after talking to my doctor we went to ER. They said he had an irregular heartbeat, they gave him a blood thinner and a medication to even the rhythm. They kept him for evaluation and further tests today. Is this something that could be very serious? Doctor: Hias he was given blood thinners the irregular heart beats can be atrial fibrillation,as blood thinners are usually given in atrial fibrillation to prevent the formation of clot .trial fibrillation is an abnormal heart rhythm characterized by rapid and irregular beating. Often it starts as brief periods of abnormal beating which become longer and possibly constant over time.A 24 hour holter would be done , this will keep a record of his beats for next 24 hours , as palpitations can be of a number of types atrial ventriculr or sinus tachycardia.High blood pressure and coronary artery disease are the most common cause of it hence a detailed check up would be done to rule out the cause advice2d echotread mill test24 hours holter monitoring.yes initially it should be taken seriously."
},
{
"id": 44015,
"tgt": "PCOD, advised Evanac. Discontinued as TSH went low, started Overal, husband's low sperm count, on Maxazol. Suggestions ?",
"src": "Patient: I have been diagnosed with PCOD , i was asked to take Evanac tabs for the same. I did it for 45 days. Then did T3, T4, TSH ,LH tests below are the results. LH-20.1 TSH- 3.43 My doctor told me to discontinue Evanac tabs and asked me to start with Overal tabs for 21 days n 7 days gap n start it for same cycle. Actually my husband is having low sperm count he advised to take Maxazol powder. Sperm count is 2.5ml/ml Doctor: Hi, Just reconfirm your husband's sperm count with another repeat semen analysis. If that also is so low, its better to go for IVF/ICSI(Test tube baby). Don't waste time in taking medicines to improve sperm count. Wish you good health."
},
{
"id": 131494,
"tgt": "What causes tingle and numbness in arms along with dizziness?",
"src": "Patient: Hello, I am a 58 yr old male, 195lbs, and feeling great until recently. Several times a day, my arms might tingle and feel numb, sometimes get dizzy getting up too fast, vision might blur, swelling and pain in right thumb and fore finger (joints), some discomfort in left thumb at joints. BP is 107/65 pulse 81. Twice I have lost motor control below my waist. Legs wobble and feel useless. Coordination has been off a couple of times too. I am in need of an alignment, as my back has familiar pains upper and lower. What do you think? Doctor: HiYou may consult a neurologist for work up.cause of dizziness, numbness,tingling, motor weakness in lower limbs need explorationIn meantime take methycobalamin and lyrica tabs 75 mg twice a day.Take physical therapy of back and legs.Some weight reduction with modified diet and exercises shall be helpful.Get serum Lipid profile also done.Bad cholesterol deposits does some impaired blood circulation often, with weight, it's possible..must rule out tooBest wishes"
},
{
"id": 221243,
"tgt": "Suggest risk for pregnancy in smoking females",
"src": "Patient: Hi my name is Nat and im 26! missed my period this month i ve taken pregnancy test and im positive but what im scared of is that i have been smoking mathamhetamine for quite some time and 2 years ago i had an ectopic pregnancy im very worried of what could happen because i want too keep the baby please help Doctor: HiDr. Purushottam welcomes you to HCM virtual clinic!Thanks for consulting at my virtual clinic. I have carefully gone through your case, and I think I have understood your concern. I will try to address your medical concerns and would suggest you the best of the available treatment options.It is good to note that you are thinking of parenting.I will suggest to stop use of the substance straight way.It has potential serious health hazards, it is better to stop it's use now.Considering history of ectopic pregnancy ; it is advisable to get USG done to confirm normal location of pregnancy.I will suggest indulging in a healthy diet and regular exercise regime.Include plenty of fruits, salads, vegetables in the diet. Have adequate water intake , say 8 \u2013 10 glasses per day. Avoid deep fried foods, bakery products, and refined sugars.Start on FOLIC ACID, B12 supplements if plan to continue with pregnancy.I hope my answer helps you.Thanks.Wish you great health."
},
{
"id": 173784,
"tgt": "What causes stomach ache in a toddler?",
"src": "Patient: my two year old son has woke up at 2 a.m. screaming his tummy hurts the last two nights, nothing seems to help the problem. yesterday during the day he seemed happy and fine not a word about his tummy .. could this be serious or just a tummy ache. first night he threw up & second night just held his stomach Doctor: Hi,Thanks and welcome to HCM .Stomach in a toddler may be due to urinary tract infection ,worm infestation or simle colicky pain due to indigestion.Cyclopam syrup 3times daily will give temporary relief.Better consult pediatrician to arrive at tha correct diagnosis .If you like you may give a deworming agent like Albendazole empirically .Hope this reply serves your purpose.Please feel free to ask further queries if any.Dr.M.V.Subrahmanyam."
},
{
"id": 3011,
"tgt": "Does fluid in the pouch of Douglas affect chances of pregnancy?",
"src": "Patient: Greetings doc. When a 27years old girl has fluid in her pouch of Douglas, will it affect her chances of getting pregnant?? I had a general abdominal scan about a week ago and the results states; The liver, gallbladder, pancreas, spleen and the Kidneys are grossly normal. No features of cholelithiasis, nephrolithiasis, acute appendicitis or ascites. An anteverted non-gravid normal sized uterus with normal myometrium is demonstrable. There are neither myometrial nor adnexa masses. The ovaries are grossly normal. No features of extra-uterine gestation or retained products of conception. There is significant amount of flood in the pouch of Douglas. The urinary bladder is grossly normal and there are no vesical calculi. The conclusion states; Acute pelvic inflammatory disease. Doctor: Hello and Welcome to \u2018Ask A Doctor\u2019 service. I have reviewed your query and here is my advice. The fluid in the pouch of douglas is usually indicative of conditions like ruptured ectopic pregnancy, pelvic inflammatory diseases (that is chlamydia). As your report suggests a PID, I recommend a more elaborate evaluation along with treatment of the exact cause which can then be followed by the normal process of conception, it would be best to put trying to conceive on hold until then.Hope I have answered your query. Let me know if I can assist you further.Best wishes,Dr. Shoaib Khan"
},
{
"id": 18926,
"tgt": "How to treat pain in left shoulder and chest after stents in arteries?",
"src": "Patient: I received two stents in the arteries around my heart six months ago. Eversince then I had this pain in my left shoulder and on the left side of my chest. There are still some times that I feel out of breath and pressure on my chest but I have stopped smoking and am eating healthy. Can it be my medicine. I use Aspavor 4mg, Plaxim 2mg, Disprin 100mg, Clopidogrel-winthrop 75mg and bilicor 5mg. Doctor: Hello,If you were my patient, I would suggest you to go to a cardiologist and do a cardiac sonography and a stress test to see what the problem is.It is not normal to feel pain after placing a stent, unless there was a small artery that has stenosis and a stent cannot be placed because of the small caliber.The doctor will see what the problem is and do the right thing (one possibility is changing your medications).Hope I have answered your query. Let me know if I can assist you further.Regards,Dr. Anila Skenderi"
},
{
"id": 146958,
"tgt": "How to confirm occurrence of seizure with normal test results?",
"src": "Patient: i blacked out while driving. i have no memory from after getting on the road up until, an officer tapping on my window. then i was in the hospital, with the worst headache i have ever had and i was very nautious. someone said they thought i was having a seizure. anyway to know for sure? all tests were clear. Doctor: the problem with seizure is that the tests may all come out to be normal and yet you may have the condition. there is no absolute way of knowing positively whether you have had a seizure or not, unless it was observed by someone. the description of the seizure is the best way to diagnose it.alternate way of testing for seizure, which you may have not done cause it is infrequently found , is video eeg. here you are monitored in a room for 24 hours or as per the protocol of the testing centre. during this period, your eeg is recorded continuously. the chances of getting a finding are raised. however this may also be negative.i hope that answers youbest of luckDr Mittal"
},
{
"id": 179981,
"tgt": "How to administer Darolac sachet for loose motions and vomiting?",
"src": "Patient: Hi Dr. Santosh. My 16 months old son (~10 kgs) is having loose motions since yesterday night (3 till now). He had vomited too (thrice) yesterday night, we gave domstal, he has not vomited today. But he passed loose motion now again. How can we adminster darolac sachet? Any other advice please? Doctor: Hi,Thank you for asking question on health care magic.Darolac can be given for 5-6 days daily one sachet.You may give ORS solution about 50-100 ml after each motion or vomiting to prevent dehydration.Zn 20 mg/day can be given for fast recovery.Continue feeding as usual in spite of loose motions.Hope this answer will serve your purposePlease feel free to ask any more queries if requiredTake careDr.M.V.Subrahmanyam MD;DCHAssociate professor of pediatrics"
},
{
"id": 82280,
"tgt": "How to cure burning sensation on right side of the body?",
"src": "Patient: Hi can you tell me. i have a burning pain in the right side of my body. It started one week ago in the upper outer arm area. Than i felt in the right side of my around the area of my nipple line to my illac crest area. In the last couple of days it has migrated down my arm all the way to my fingers and is now felt in my leg. What could this be? I have felt none in my face other than for the last 2 weeks my around the area of my moler (which is removed) has been burning. I am on no medication. do not smoke no birth control pills. age 46. Doctor: Thanks for your question on HCM.In my opinion you are having sensory neuropathy.This can be seen in1. Diabetes2. Vit b 12 deficiency3. Zinc and magnesium deficiency. 4. Nerve root compression at spinal cord level.So better to get done1. HbA1c to rule out Diabetes.2. Get done vit b 12 level.3. Get done megnasium level.4. Get done MRI of whole spine.You can start multivitamin having zinc and b 12. Start magnesium supplement."
},
{
"id": 93655,
"tgt": "Bloated, discomfort, burning sensation in anal area. Been passing stools. Is it due to spicy food?",
"src": "Patient: I was passing stools well . 2 nights ago i ate a very spicy dish, ever since i have been experiencing bloating and discomfort in my stomch and also have slight burning sensation in my anal area ( like i have rubbed mint oil ). I pass stools well in the morning but the bloating and burping does not stop. Please help me. Doctor: Hi, thanks for writing in. Spicy foods are known to cause gastrointestinal upsets. You seem to be having two issues 1. Upper abdominal discomfort. For this I would suggest you to take sucralfate suspension in the dose of one tablespoon after each meal three times a day for at least 5 days . Avoid spicy foods. Increase the consumption of bananas and cucumber which are natural antacids. 2. Since the passage of stools is smooth you need not worry. Just take hot water fomentations for the perianal problem. That will relieve the symptoms. You can add half a sachet of magnesium sulphate powder to the tub of water in which you are going to sit for applying hot water fomentations. This remedy will give you instantaneous relief. Regards"
},
{
"id": 119313,
"tgt": "Low hemoglobin levels, wanted to donate plasma. Suggestions to maintain normal levels?",
"src": "Patient: Hi I went to donate plasma today and they told me that my haemoglobin level was 116 and they were concerned it was too low to donate but as it was only plasma they took the donation. What recommendations do you have to raise it to a healthier level. As I don t have any other symptoms such as lethargy do you have any suggestions as to how it has dropped to this level. Thanks Doctor: hello. i guess your Hb (haemoglobin) is 11.6 and not 116. if it is so it is borderline for an indian girl and definitely low for an indian male.for menstruating female it is common to have low Hb and most of the times they are asymptomatic.there is no reason to worry for such Hb.if you want to maintain a good hemoglobin level i advice you to have a balanced diet containing mainly green leafy vegetables, fruits like apple, dates etc. (if you prefer nonveg food-liver, fish etc)these will help you increase iron content of your body stores and prevent iron deficiency anemia which is common.nonveg food and milk also helps to restore your vit B12 levels and prevent or B12 deficiency anemia (megaloblastic anemia).hope i have helped you."
},
{
"id": 115429,
"tgt": "How to increase haemoglobin level?",
"src": "Patient: I am addicted of eating raw rice and my haemoglobin level has reached to 5.6 plz help me to leave this habit and suggest me to recover my haemoglobin level Doctor: Hello,eating rice won't lower your hematocrit. Eating ONLY rice will. You need to have a balanced diet. You can include rice in most meals but you need a variety in your diet. You'll also need investigation for the reason of your anemia. Some basic tests would include ferritin levels, B12 vitamin levels and folic acid levels. Of course a complete blood count is essential.Depending on what's missing (if any) you'll need supplementation and a change in your diet. I hope I've helped!Kind Regards!"
},
{
"id": 191965,
"tgt": "Suggest treatment to control blood sugar",
"src": "Patient: I just got diagnosed with high sugar levels in my urine during a UTI visit with my urologist. I have an appt. with my family doctor tomorrow but is bought an Accu-check-nano machine and just tested myself and it registered 254. Is this what is causing my headache and can I do anything before I get into the doctor on Monday Doctor: Hi Thanks for contacting HCM.You have not mentioned about your age . Dont get stressed about ur sugarlevels . You need not to do anything in one night . Be relax and go to ur family physician . Tomorrow he will help u insorting out appropriate investigations and medications to control ur sugarlevels .Hope I have helped u .With regardsDr ponnarasu"
},
{
"id": 176898,
"tgt": "What causes fever and swelling on thigh post vaccination?",
"src": "Patient: hello doc, myself dr puneet sir i vaccinated my girl child with DPT,ORAL POLIO,ROTAVIRUS AND PNEUMOCCOCAL VACCINE ON 1 1/2 MONTHS AND AFTER 1 DAY, SHE EXPERIENCED FEVER ,SWELLING ON THIGH AND LOOSE SRTOOLS AND VOMITING AND NW ON 2 AND HALF MONTH WAT I HAVE TO DO AS MAY I OMIT ROTA VIRUS AND PNEMOCCOCUS VACCINE AND GO FOR AS SCHEDULE. SHE IS TO ADMITTED AT THAT TIME AFTER VACCINATION PL TELL THANX Doctor: Thanks for your query on health care magic.From history it seems your child develop minor vaccine associated side effect after vaccination for first time. But non of them are serious enough to contraindicate the next vaccination. these side effect are common and not going to adversely affect health of your child. benefits from vaccination is far more then these minor reaction. I advice you to compete the course of all vaccination to take their maximum benefits. regards- dr deepak"
},
{
"id": 96752,
"tgt": "How to cure allergic reaction to the dissolving stitches in foot?",
"src": "Patient: I've had an allergic reaction to the dissolving stitches the surgeon used on my foot surgury 6th dec 2010. Why when I went back to see and he told me this did he decided to wait another six weeks until the end of feb before removing the troublesome stitch. I'm still in pain and my foot has swollen again. Doctor: Hello,I'm dr.Klarida and I thank you for posting your concern here.Looks like it has been some time from your surgery. If It was an allergic reaction it should have not been never better, but being always worse.Swollen foot can have various causes. You will need to see again a doctor, so by asking more information to you, and examining the foot, he will determine what is the cause of your swollen foot.Feel free to ask, if you have more questions.Take care."
},
{
"id": 92578,
"tgt": "Have abdominal pain and lower back pain, light headedness, shallow breathing, feeling faint. Normal blood test",
"src": "Patient: I experienced abdominal pain and middle to lower back pain that would peak while I slept. These pains lasted about three weeks and have since died down, but the light headed feeling, shallow breathing and faint feeling have not gone away. My chest sometimes feels tight and my throat constricted. When I move my head side to side it doesn't feel like I'm in control. My reactions are either slower or faster than my control. My blood was tested and is normal as well as a screening for anemia, diabetes, thyroid, and bladder infection (for other symptoms) and all came back normal. My arms feel numb and I feel like my body temperature is irregular. Last night I shook uncontrollably as if I were freezing, but I wasn't. These feelings always get worse at bedtime. Doctor: food protein allergiesas some of food proteins suddenly start reacting with body proteins get clinical immunologist conultation to get blood serum tests for specific antibodies for milk wheat potato ormajor food you eat withdraw those proteins and you can have benefit in all diseasesi have treated many cases like this"
},
{
"id": 155267,
"tgt": "What causes urge to urinate all the time when scheduled for brachytherapy?",
"src": "Patient: I was diagnosed with prostate cancer 6 months ago. I am 1tc, Gleason of 7, stage 1, 5 percent of one of 14 core samples. I'm scheduled for brachytherapy 3/6/2014. I'm on vacation now and since yesterday I've felt the urge to urinate all the time. What's going on? Doctor: The symptoms that you are having are due to enlargement of the prostate gland which sits around the urinary outflow system. It can thus lead to urgency, increased frequency, burning, blood in urine etc. These symptoms will improve after your treatment starts and the gland decreases in size."
},
{
"id": 21159,
"tgt": "What can cause rapid heart beat?",
"src": "Patient: My 21 year old girl has a rapid heart beat, which started recently. she is taking Contraceptive pill, 5 days ago we rushed her to A&E, her heart beat was very fast, they said she had inner ear infection and her tonsils were inflamed, so she was given Antibiotics but we had to stop as the tablets were making her worse.(heart beat). while we were in hospital they recommend that she should take Magnesuim tablets. can you please advise why all of a sudden she is having this problem? Doctor: hello thank you for posting here.by what you have explained, lt looks like she is having sinus tachycaror supraventricular tachycardia (SVT). sinus tachycardia is a condition when we havr a heart beat due to running, exercising, stress and during infection in the body. so if it is sinus tachycardia then it is due to the infection. the other variant, SVT may also be triggered by infection in the body or due to abberant connections in the heart conduction system. it is a rarely harmful for the patient but required medications to settle down. only an EKG will help to distinguish between them. magnesium also helps in stabilising a fast heart rate. but treating the cause - infection with a different antibiotic which doesnot worsen fast heart rate is advisable.it is usually a bening problem and after she recovers her normal heart rate she may probably not get it. however if she gets it on and off then there is a conduction pathway problem for which she should be on rate control medications. thank you"
},
{
"id": 110835,
"tgt": "What causes severe lower back pain?",
"src": "Patient: My daughter 12 years old and her lower back has been hurting. she went to the doctor a while ago and they told her to rest it healed they said it was a pulled muscle. But now she says it hurts even more. I dont know what it is or is it seroius. She said it hurts alot when she runs or does certian things. Doctor: HELLO, PLZ GET HER U/S ABDOMEN PELVIS , URINE R/E, CBC , DONE TO SCREEN FOR ANY ANOMALY LET ME KNOW THEN, GIVE TAB BUSCOPAN PLUS FOR NOW. KINDLY DESCRIBE WHETHER THESE PAINS HAVE BEEN CHRONIC OR EPISODIC, THANX"
},
{
"id": 176385,
"tgt": "Suggest treatment for cough and sinus drainage in child",
"src": "Patient: Hello- My one year old has no fever but a very deep cough and sinus drainage. He will cough so hard that when he has a full belly he throws up. We have not been to his pediatrician yet. I have been giving him breathing treatments up to this point. Should I be concerned if he has the flu? Doctor: Hi,Looking to the history it seems that he might be having viral bronchiolitis giving deep coughing and mucous drainage.Even if we suspect viral origin, antibiotic medicine is advisable as there might be having mixed infection.If bacterial infection is present there might be having risk of developing pneumonia so antibiotic cover is safe.Consult your pediatrician and get examined.Ok and take care.Meanwhile give some cough drops to get symptomatic relief.Ok and take care."
},
{
"id": 154040,
"tgt": "What causes pain/bleeding in ear while having prostate cancer?",
"src": "Patient: My Husband is a Prostate Cancer Patient. Early Stage and will begin radiationTherapy in about a week. All of a sudden he started having pain in his ear, and some Bleeding. Could it be related to the cancer? and should this be checked out before Radiation begin? Thank You. Doctor: Hi, dearI have gone through your question.I can understand your concern. Your husband has ear pain and bleeding but mostly it has no relation with prostate cancer. He may have some infection in ear like otitis media or otitis externa. He should go for examination once. Then he should take treatment accordingly. Hope I have answered your question, if you have doubt then I will be happy to answer. Thanks for using health care magic. Wish you a very good health."
},
{
"id": 171621,
"tgt": "What causes faster heartbeat to a 9 month baby while doing something?",
"src": "Patient: hi i have a baby boy 9 months , and evry tame he plays or when he is doing something he breathe verry farst , he gets verry tierd , is that normal ?? i have mention to the doctor and the health visitor but they all ways say the same thing ..has anyone smoks on the house ? can you please help me ? what should i do to find out if is ok ? Doctor: Hi,Welcome to HCM,Faster heart beat and a faster respiration during activity is normal in any child. I would be interested to know if he can do little bit of activity which normally any child of his age group can do without any tiredness? I mean to say, does he gets really tired to have to rest on doing normal activities like breast feeding or drinking from bottle etc. If that's the case, I would suggest you to get him evaluated by a pediatric cardiologist and make sure his heart is normal. And if that's normal, starting iron syrup for him would help. Happy to clear any further doubts. Take care."
},
{
"id": 212843,
"tgt": "Taking adderall bid and effexor for depression. No schizophrenia symptoms. Suggestion?",
"src": "Patient: Dear sir; I am a 45 y/o female RN with above average intelligence. In the last 6 months i was recently prescribed 30 mg adderall bid and effexor 325mg QD for depression, motivation and attention. While I don t have a real substantial hx of any physical nor mental dx s, my gene pool has several. ADHD , Tourette s, Asperger s, ODD, OCD, Depression, Bi-polar, Anxiety, Anger, IVDA, Polysub, ETOH. There is no schizophrenia , no suicide nor homocidal behavior. I am seeking answers to my behaviors that don t seem to be quite in-step with the general population, because I have always (mostly) brought out the worst in people, make them angry, resentful, assaultive (both physically and mentally), without meaning to at all. I am straight-forward, and am surprised when it s not appreciated and am perceived as mean. Doctor: Hello...... Thanks for your query. You have a family history of multiple psychiatric disorders while the medication prescribed appear to be for for ADHD (attention deficit hyperactivity disorder)/depressive disorder. However, the symptoms you report fit into cluster \"B\" personality disorder. The facts do not fit in together. I suggest you undergo evaluation of your personality profile and get back to us with the complete symptom profile and treatment records for a better understanding. Regards Dr Sundar Gnanavel Psychiatrist"
},
{
"id": 132615,
"tgt": "What causes disability to bend left thumb?",
"src": "Patient: I have never been able to bend my left thumb since birth and it s weird. Doctors say everything that needs to be in my thumb is there, but can t explain why It wont bend. It s irritating and sometimes painful to try to bend it alone, but I can manually bend it with no complications other than slight irritation. Please help! I need to know. Doctor: Hi Hope this message finds you in good health.I have gone through your complaints and understand your concern.some of a nerve supplying the muscles of your thumb might be damaged since birth..that might be the reason.other possibility is mucle power loss.get EMG,NCV studies done.Nothing to worry about.\u00a0\u00a0\u00a0\u00a0\u00a0I hope your question has been answered.If you have any follow-up queries,feel free to consult me anytime.Thanks,Take care,God bless."
},
{
"id": 97217,
"tgt": "Suggest treatment for bump on head after fall",
"src": "Patient: This morning I slipped and fell backward bumping my head on my granite countertop. I now have a bump behind my right ear. It is tender. I have not felt any dizziness or disorientation What should I do. I am a 45 yo female. I have never had a head injury before Doctor: Hello dear,The bump formation after fall is due to accumulation of fluid & blood and it is called Hematoma.The blood cells accumulated are eventually broken down and absorbed in the body.But this is a slow process & takes times to disappear completely.Till that time, symptomatic relief is obtained by analgesics, ice compression & adequate rest.If it still persists or there are any other associated symptoms like fever, vomiting, dizziness, loss of consciousness, convulsions, etc...then it will be better to consult your Physician.Under such circumstances, a CT scan of brain will be needed to rule out any pathological cause for symptoms.Wishing you a good health.Take care."
},
{
"id": 72133,
"tgt": "What causes shortness of breath, pain and tightness in chest?",
"src": "Patient: I had Mitral Valve Repair almost one year ago as well as 1 Bypass. I have returned to relatively noithrmal life. I am 62 years old. Recently I have been experiencing shortness of breath with very little exertion, ie walking the stairs up to our bedroom. I also feel a sharp pain in my lower left chest cavity when I take a deep breath. I feel a tightness in my chest when I lift my arm up over my head. Not a tightness light heart attack but it feels like I may have adhesions in or near the surgical area. Are these reasons for alarm? Doctor: Thanks for your question on Healthcare Magic.I can understand your concern. Yes, you should definitely consult doctor for these symptoms. Your symptoms are more suggestive heart diseases like congestive heart failure (CHF) and coronary artery disease (CAD).So better to consult cardiologist and get done ecg, 2d echo and stress test. You may need coronary angiography if any of these tests is showing abnormality. So consult cardiologist and discuss all these. Hope I have solved your query. I will be happy to help you further. Wish you good health. Thanks."
},
{
"id": 22682,
"tgt": "What causes stabbing chest and body pains?",
"src": "Patient: I am a women 57 years old, 5'6', weight 130 ,father died of diabetes ,cardial vascular disease age 54.I have been having mild stabbing pain in my left chast area. I have had EKG ,ECG,blood work ,chest xray and stress test. All came back normal. For no apparent reason I recently experienced pain that started between my should blades in my back, crushing pain radiated to the center of my chest up my jaw,my whole upper body . Lasted for about 3 min,took an 325 mg asprin,after asprin it took about 2-3 min to ease up. Could this have been a heart attack? Doctor: Hello and welcome to \u2018Ask A Doctor\u2019 service. I have reviewed your query and here is my advice. If you had all those tests recently with good results, then heart attack is not likely. Also it lasts longer, it is usually relieved only with a strong painkiller and your description is not typical for heart attack, it seems more like a spine problem.Hope I have answered your query. Let me know if I can assist you further.Regards,Dr. Lilit Baghdasaryan"
},
{
"id": 47282,
"tgt": "Suggest treatment for hypo dense area in the kidney",
"src": "Patient: Hi,My 31 yo son went to the ED today for crampy pain in his left flank, blood in his stool (he says only with straining); they checked labs and urine which were fine and did a CT scan which showed a hypodense area in upper pole of left kidney with some extension into perinephric fat; differential dx was infectious etiology vs ischemia. Can you comment? Doctor: HelloThanks for query .Your 31 years old son had a pain in left flank and his CT scan of abdomen has revealed Hypo dense mass in upper pole of left kidney .The hypo dense lesion detected needs further evaluation to rule out its nature whether benign or malignant or infective .Please consult qualified Urologist for clinical examination and get following basic tests done to confirm the diagnosis.1) C.T Scan with I.V contrast 2) C.T angiography 3) M.R.I Further treatment will depend upon result of these tests and final diagnosis.diagnosis. Dr.Patil."
},
{
"id": 216631,
"tgt": "What causes knee and foot pain?",
"src": "Patient: i have knee and foot pain. the bone doctor said that I needed therapy because the leg muscles have tightened up. I caN T RECALL the name of my problem. I had gout a few years ago also my right foot has been swollen for 6 days now. have been taking extra strength acetometaphin for a few days. No asperin. It feels like gout. I had been drinking cheap vodka for a few months but stopped all alcholol consumption 4 days ago. the big toe is bright red the others are swollen. Any thoughts? john Doctor: hithank you for providing the brief history of you.as you have pain in the knee and foot along with the past history of gout, I will Advice you to undergo physical therapy by which your pain levels will be reduced and also the strength in the muscles will be improved too.In my clinical practice most patients respond well to physical therapy for knee pain , foot pain and gout.regards Jay Indravadan Patel"
},
{
"id": 70695,
"tgt": "Experiencing eye twitching, breathing problem, chest tightness after flu vaccination. How long will it continue?",
"src": "Patient: My son had a flu vaccination on Jan. 20th and has had multiple symptoms since then. He has had eye twitching, shortness of breath, tightness in the chest, headaches, dizziness (disoriented feeling), ringing in the ears (only once), blurry vision, and muscle soreness. This is just over 4 weeks from the time of the vaccination and after a round of z-Pack, and zyrtec, and aleave, or Tylenol for the muscle soreness and headaches, he is improving, however, he still begins feeling sort of bad around 9:30pm and the disorientation comes back. Also, the eye twitching still comes and goes. We feel he is having side effects from the flu vaccination and were also wondering if Guillain Barre Syndrome might be a factor here. If so, we don t think it is a severe case. Wondering how long his symptoms might continue and is there anything we should be doing that might help other than the ibuprofen or aleave for muscle soreness? Doctor: Hello, As you explain the history these may be symptoms of a viral situation due to vaccination or not. However, his doctor should examine him definitely. Hope I have answered your query. Let me know if I can assist you further. Regards, Dr. Jnikolla, Pulmonologist"
},
{
"id": 100967,
"tgt": "What causes rashes all over body after intercourse?",
"src": "Patient: My husband eats peanut butter daily and has started adding it to his protien shakes. I am 44 and have been allergic to peanuts since I was 19. I abstain from eating peanuts but lately have been breaking out in rashes that last weeks and usually right after sex. Can peanut allergan show up in semen? Doctor: It's not peanut allergy from semen.The place, clothing ,bed-sheets etc. at the site of sex need to be evaluated as potential allergens."
},
{
"id": 73564,
"tgt": "What causes dry cough despite taking Toplexil?",
"src": "Patient: hi, I suffer from very dry cough. It started first from chest , now it is from throat. I don t breath without coughing . Taking toplexil syrup 3x/day and histamed f 3x/days from 3days ago plus gargling with salt, drinking hot infusions, having steamed bath. Cough getting worse . Can t take any antibiotics coz I have IBS and my stomach will kill me from pain. Doctor: Hello dearWarm welcome to Healthcaremagic.comI have gone through your concern in depth .* Dry cough despite taking the above medicines indicates irritation of underlying bronchi or alveoli through infection of bacteria or virus . - Additional factor can be exposure to pollen / dust . - Smoking .* Needs minimum broncho dilator ( if not antibiotics ) after consultation with the doctor .Hope this will help you for sure .Regards ."
},
{
"id": 5118,
"tgt": "26 years old, undergone abortion. What is the minimum gap to plan for the next pregnancy?",
"src": "Patient: Hi, I am 26 year old. I had a missed abortion , due to no cardaic activity of the fetus at 6weeks. I had repeated scan at 10 weeks. Still the age 6 weeks. I had abortion ( cleaned up my uterus ) as per doctor's suggestion. Now when can i plan for my next pregnancy. what is the minimum time gap i need to maintain to plan for next pregnancy. Doctor: Dear user, welcome to healthcare magic.There is no recommended time gap to be maintained before going through the next pregnancy, providing that you are young, healthy, having no medical disorders and not anemic.You may even get pregnant without experiencing menstruation. If you are willing to get pregnant then don't use contraception. You may use contraception for a while if you and your partner want time it get ready (from the psychological point of view).I hope I have answered your question and I wish you good lick in the next pregnancy.Dr. Ahmed Bahaa."
},
{
"id": 79910,
"tgt": "Suffering from shortness of breath due to slight water in the lung",
"src": "Patient: I am having very bad shortness of breath. Was diagnosed with slight water on the lung almost yr ago and put on Lasix by my Cardiologist. I had a heart attack 18 yrs ago months after my husband died. I had to recover over with my daughter and family at the home I own in Milton, MA. by November 1st I felt well enough to go home with a girlfriend who would stay with me for 6 months. I was under treatment by Dr Bevivino in Norwood, and I soon changed to Dr Cadigan Jr. at Norwood Medical. I moved to Milton and was under care of Dr. Delaney and he went into concierge medicine and I have been with Dr Mark Hodgeman at South Shore Internal Medicine- Beth Israell Deaconess Medical Center at Milton for the past 9 years. Doctor: Thanks for your question on Health Care Magic. I can understand your concern. In my opinion, you should again consult cardiologist and get done x ray to rule out pleural effusion. Because cardiac patients tend to have ttansudative pleural effusion. Post heart attack, cardiac functions especially pumping is decreased. So fluid tends to accumulate in the pleural cavity. This causes pleural effusion and breathlessness. So you need diuretic and inotropic drugs. Diuretics increase urine output and so fluid overload can be avoided. Inotropic drugs will increase the force of contraction, so circulation of fluids can be improved. So consult your cardiologist and discuss all these. Hope I have solved your query. Wish you good health. Thanks."
},
{
"id": 141073,
"tgt": "Can lower back pain caused by a herniated disk affect the nervous system?",
"src": "Patient: Six years Ago I had surgery for a herniated disk between L1 $ L2. Four months later I had a second surgery on the same disk to correct the same problem. Since that time I have had chronic lower back pain. I have had x-rays, MRI s (Twice with no structure problems) and 17 physical sessions with out results. My question is can lower back pain affect the nervous system creating a nervous condition to the point where my whole body shakes? Doctor: Hi, Lower back pain may not affect the whole nervous system. At the most, it may affect the lower limbs or bladder and bowel function etc. If the whole body shivering present then it may be due to electrolyte imbalance or generalized weakness or anxiety or tremors due to parkinsonism or essential tremor etc. Until examination is done it is difficult to say what it is. Please continue physiotherapy, once again check for your lower back by MRI spine. If symptoms not improved please consult your neuro physician, he will examine and treat you accordingly. Hope I have answered your query. Let me know if I can assist you further. Regards, Dr. Penchila Prasad Kandikattu, Internal Medicine Specialist"
},
{
"id": 145740,
"tgt": "Why am I fainting and having blackouts?",
"src": "Patient: a certain feeling comes in/on my leg/thigh and I know I m going to faint. I do not feel it in my head, but my leg/thigh only 1. then I need to lay down and wait for it to pass, that s when I know it s in my head also, the fainting.. but I feel it in my leg/thigh. only one. If I am standing I must immediately sit/lay down or I will fall/pass out. Usually I make it to the bed/sofa/chair. that s when I notice the blacking out would be in my head. Doctor: Hi,Thanks for writing in.Moments of dizziness can happen due to decreased blood flow from heart to brain. When it happens, the blood flowing through the vessels from heart to brain might have a reduced flow. These are by the carotid and vertebral arteries on either sides. This is more significant when you are standing upright and doing any moderate to heavy physical activity.Since he has got scans for inner ear and brain done, it is confirmed that there is no abnormality in the vestibular system in the ear or any serious conditions in the brain.After clinical examination, your doctors might recommend CT scan or MRI scan brain to know any problems within the brain and Electronystagmography (ENG) or videonystagmography (VNG) to record abnormal eye movements while the head is placed in different positions or your balance organs are stimulated with water or air. For this he might consult the ENT specialist.The best way to analyze if the reduced blood flow to the brain is causing your dizziness is by doing a carotid and vertebral arteries Doppler ultrasound. It will show the flow of blood from the heart to the brain and if it is adequate.Dizziness can also occur due to stress related anxiety or overflow of emotions."
},
{
"id": 58088,
"tgt": "Done biopsy, colonoscopy and endoscopy due to bile vomiting, showing inflammation. Is it result of sinus infection medication?",
"src": "Patient: Bile vomitting from past 1.5 months quite often. On Nov 01 2013 colonoscopy and endoscopy done. Colonoscopy is quite normal. Biopsy followed by Endoscopy results some inflammation that might have been caused by 4-5 months of antibiotics for my sinus infection.Doctor prescribed: Omeprazole+sodium carbonate - daiy one at night. sucralfate tablets daily 4 -- each 1 hr before meal.XIFAXAN (RIFAXIMIN)- 550 MG - DAILY 2.Could you please give us online consultation as I am quite resistant to take the above antibiotic medicine.ThanksRam Doctor: Hi and welcome to HCM. Thanks for the query.I don thtink it is reasonable to take an yantibiotics for 5 months. even 5 weeks is eay to loong and this could definitely be cause of your colitis. you should stop with medicines for some time and see will your condition improve. You should eat easier food and drink plenty of fluids till symptoms subside.YOu should continue with omeprasole.Wish you good health. Regards"
},
{
"id": 64958,
"tgt": "Suggest remedy for lumps in rib cage",
"src": "Patient: ONE YEAR OLD GRANSON WITH SMALL EGG SIZE,HARD NON-MOVEABLE LUMP BELOW RIB CAGE ON BILAT SIDES,GREATER ON LT. RECOVERING FROM VIRAL PNEUMONIA. ALSO,HAS FINE,RAISED RASH DIFFUSELY ON TRUNK TO BILAT AXILLAS. RASH IS WHITE,NOT RED AND DOES NOT LOOK LIKE ALLERGY. Doctor: Hi,Dear I went through your query.1-By the facts placed by your query,I feel that this lump below the rib cage bilaterally - appears to be viral- chondritis.2-Plz check your your ER- doctor for second opinion. 3-This may resolve with anti-inflammatory syrup only - as its mostly due to viral infection and no antibiotics are needed.4-Hope this would solve your query,5-Thanks for your query to the Health Care magic. 6-You are wellcome for any further treatments if required.7-I would love to help you out of YOur grandson out of the ribcage lumps. .Thnks.Wellcome."
},
{
"id": 43565,
"tgt": "Have irregular period. On contraceptives. Is this a sign of infertilit?y",
"src": "Patient: I got married on 15th Jan 2012. My gynac adviced a 21 day contraceptive course which i took for around 18 days. but since then.. my period has never been regular. sometimes it comes on the 35 day of the cycle sometimes 38. some times 45. I have never taken any other pill since jan 2012. Now that it is more than a year, this irregular cycle worries me. Is it a sign of infertility? before marriage my period was regular - it would always come on the 27/28th day of the cycle. Doctor: Hello, ,welcome to Hcm. Yes , irre regular period may be the sign of infertility. So, you have to do these following test. These are thyroid profile, LH, FSH, DHEA and ultra sound of whole abdomen. You have to take combind oral contraceptive pill for six menstrual cycle. So, do these things as early as possible and report. Take care."
},
{
"id": 206429,
"tgt": "Should antidepressants be continued for persistent sexual arousal syndrome?",
"src": "Patient: i have Persistent Sexual Arousal Syndrome. i stopped taking my anti depressants about 4 wks ago and 5 or so days later, i experienced exactly those symptoms. i just refilled my script, how soon will this go away if I start my antidepressants again? Doctor: Hi , thank you for using health care magic.I can understand your concern.Antidepressants work in and around 15 to 20 days but reports are there for some relief of symptoms in first week also.Antidepressants should be used accordance with the advise of psychiatrist and should not be stopped abruptly without doctor's advise.Use of antidepressants should be reviewed after 3 to 6 months post recovery, and further decision about to continue drugs or not , should be taken after careful review of persistence symptoms, relapse , family history , any other co existing disorder etc. Hope this will help you.Regards."
},
{
"id": 107040,
"tgt": "How can severe backache be treated?",
"src": "Patient: I invite several epidural injections for my back and the RFC things for my back is well I been out of Pain Care Now for 7 months I m not looking for pain meds but I m looking for something that will help with the pain just not steady walking and kind of fall over into stuff so I just need something that will help stabilize my walking I don t know if there s more damage done in there or not I just need some kind of help so I m not falling into everything Doctor: hi sir/madam,Let me advice you for your problem.Low back ache may be a pre-monitory (prodromal) symptom in fistula in ano, sciatica, gouty arthritis, hernia, emaciation disorder etc. Pain is confined to lower part of the spine (back bone) especially lumbar region or lumbo-sacral area (rarely sacro-iliac region also). If it is secondary, earlier history of fall or injury may be associated. Rarely, radiating pain may be complained by the patient towards lower limbs. But it is quite common in low back aches if the defect is in the discs between vertebrae (back bones). Often the movements of lumbo-sacral region like flexion and rotation are hampered either partially or completely.Low back pain may be found in mild form in case of anemia, sciatica, rheumatoid arthritis, hemorrhoids, urinary calculi, uterine disorders etc also.Line of treatment as per Ayurveda:-The general principles of treatment of vata dosha are adopted in case of katishoola (low back pain). It includes various measures to suit its varied clinical entities, stages and associated complaints.1. Snehana (oleation) \u2013 by sneha dhara (pouring oil), abhyanga (oil massage), avagaha (tub bath with oil or oleaginous medicaments), kati basti (retaining medicaments on the back) etc.2. Swedana (sudation) \u2013avagaha sweda (sweating treatment with tub bath), pizichil (kayaseka), nadi sweda (sudation through a tubular device \u2013 local sudation), panda sweda (sudation through medicated paste or powder) etc.3. Mridu Samshodhana -mild purgation.4. Basti (medicated enema) like eranda basti, vaitarana basti, pippalyadianuvasana basti etc.Formulations indicated in Low back ache as per Ayurveda:-1. Dashamoola kwatha.2.Maharasnadi kashaya.3. Rasnaerandadi kashaya.4. Sahacharadi kashaya.5. Gandharvahastadi kashaya.6. Trayodashanga guggulu.And for local application these oils will help:-1. Ksheerabala tail.2. Mahanarayana tail.3. Dhanwantaram tail.4. Maha narayan tail.Avoid these for better results:-1.Bitter, astringent and pungent food2.Cold water3.Fear4.Exhaustion5.Standing6.Driving7.Cold food and beverages etc.Hope i was helpful.Have a healthy day."
},
{
"id": 32620,
"tgt": "Is cough, fever, swollen neck with headache concerning?",
"src": "Patient: Saturday morning I woke up with a cough. Over the course of the day it escalated to a fever, my neck felt swollen, I had a horrible headache, it hurt to inhale deeply. I took some ibuprofen around 8:30pm and that didn't really help, but an hour later I took some nyquil and my fever broke and my head stopped throbbing. I woke up today, Sunday morning, and felt fine, no fever, headaches, nothing. Should I go see a doctor Monday anyway or is whatever I have cleared up? Doctor: Hi,It seems that you might be having viral infection giving all these problem.This infection is self limiting and as you are feeling better with symptomatic treatment, continue as and e when required.Take plenty of water and take rest.avoid cold drinks, spicy and junk food.Ok and take care."
},
{
"id": 157261,
"tgt": "How can pain and nausea be cured?",
"src": "Patient: Thank you. I had uterine cancer two years ago. I have had chronic pain , very sharp at times bilaterally, where my ovaries used to be. I often feel bloated, and have started to feel lightheaded and nauseous lately. I told my surgical oncologist when I saw him in December; however, he did not seem concerned. I am very concerned. This pain and nausea is not going away. Doctor: The medical treatment will come in bundle/package, not in singles. It means most of post surgical problems , which you mentioned ,will be relieved by simple medications. But remember one point: severe pain in the abdomen, severe vomiting , constipation and not able to pass even gas, abdominal distension - these symptoms show Intestinal Obstruction - can be relieved by medication and simple surgical procedure, if neglected, otherwise can lead to threatening problem.At any time if you face all these problems together, my sincere advise is, to consult the doctor immediately."
},
{
"id": 218893,
"tgt": "Is pregnancy possible when diagnosed with a bicornuate uterus?",
"src": "Patient: i have a split uterus, after i had my period in march i went 7 weeks without a period and did have unprotected sex back in the middle of april, i've taken tests and they have came up negative could i still be pregnant? i haven't taken a test in awhile and i did bleed a little may 12 but just that one day. Doctor: yeah..pregnancy is possible with bicornuate uterus...but as u bled...dis was ur periods probably..u do urine pregnancy test...n check.."
},
{
"id": 115558,
"tgt": "What does RBC count of 3-10 indicate?",
"src": "Patient: Hello i m a male 21 years old. I do 3 times a week significan weight lifting and do drink protein and sometimes pre-workout. I got a blood test and a urine test and it says that my RBC is 3-10, Everything else is in perfect condition shoudl i be worried? What the doctor says is i should drink more water. Doctor: Hi, dearI have gone through your question. I can understand your concern. 2 to 3 rbcs in urine is normal finding. You have 3 to 10 rbcs so you should drink more water. Repeat urine analysis after 15 days. If still rbcs are there then you should go for complete kidney function test with ultrasound abdomen. Right now no treatment is required. Hope I have answered your question, if you have doubt then I will be happy to answer. Thanks for using health care magic. Wish you a very good health."
},
{
"id": 197503,
"tgt": "What causes stinging sensation in my anus and lower abdomen pain?",
"src": "Patient: I had a painful anal rash 2 weeks ago that the doctor thought was herpes. Test results came back negative an the rash mostly is gone now, but I still feel a pulsating and sometimes stinging sensation in my anus. Not sure if it's related, but I have also felt a dull pain in my lower right abdomen/kidney area. Kinda scared, please help. Thanks! Doctor: HiWelcome.You have to get it examined by a local doctor to rule out fissure in ano.Hope this helps.Take care"
},
{
"id": 34108,
"tgt": "Suggest treatment for E.coli infection in surgical wound",
"src": "Patient: Hi I am a 32 yr old woman 5'8 180lbs, history of MRSA, osteomyelitis and had a recent bkn amputation. I recently found out that the surgrical wound is infected with E-Coli. I'm a little scared. I have been taking Cleocin with little improvement. Is this because I have had chronic MRSA infections? What can I do? Doctor: Hello dear,Thank you for your contact to health care magic.I read and understand your concern. I am Dr Arun Tank answering your concern.There is no need to much worry.E.coli is isolated from the infection and not the MRSA.If the isolate is MRSA than we have to worry much. Because it is resistant to multiple antibiotics which we routinely prescribes. So taking the drug as per the sensitivity report is necessary. This will only cure this notorious bug.E.coli is relatively less resistant to routine antibiotics. Even in this case if you take the drug as per the report you can be free from the infection readily.Please maintian good local hygiene this is very important to treat the infections.Please adopt good hand washing protocol. This will help in current condition and also help ful in prevention of future infections.I will be happy to answer your further concern on bit.ly/DrArun.Thank you,Dr Arun TankInfectious diseases specialist,HCM."
},
{
"id": 142746,
"tgt": "How to cure brain TB?",
"src": "Patient: 1. how much time it takes to cure from Brain TB ? 2. how long fever & headache & body pain last in Brain TB ? 3. Is it a contagious disease yes or no ? 4. After how many months a lady can breast feed her baby ? 5. Wht r the home remedies to take care or for cure for Brain TB ? Doctor: It take from 12 to 18 months to cure It is contagious You cannot feed the baby untill you have not confirmed that it is negative Fever take two to three weeks after start of treatment to cover"
},
{
"id": 142787,
"tgt": "What causes dizziness, metallic taste in the mouth, right hand spasms and apnea?",
"src": "Patient: a recent history of unexplained small seizures, most during or prior to medical procedures, MRI & CT scan show no sign of epilepsy, or answer. Last night, similar symptoms without seizure, dizziness, metallic taste in mouth, pain/spasming of right hand, some apnea lasting up to 20 seconds Doctor: hi thanks for using HCM.it can be related to seizure, get a EEG done , as it can detect seizure activity.start antiepileptic after consulting your neurologist.thanks"
},
{
"id": 78320,
"tgt": "What causes chest tightness and burning sensation in the throat?",
"src": "Patient: A fluorescent bulb had formed a crack and blew out sometime yesterday with a strong odor of ozone, I also made sure I opened up the windows as these bulbs do have mercury in them. Shortly thereafter, I started getting uncomfortable, with a mild burning sensation at the back of my throat, and at the far reaches of my sinuses. I also felt as if I had some kind of fluid forming in those locations, especially in my sinuses, as well as tightness. My chest also feels tight and breathing feels off, but not difficult. I also have a mild burning sensation/discomfort in the area where my lungs ought to be. Doctor: Thanks for your question on Health Care Magic. I can understand your concern. By your history and description, possibility inhalation lung injury (bronchitis) is more. But no need to worry much for this because this kind of disease is usually self limiting. You just need to inhale fresh air. Inhale deeply and continuously. Do gargles 5-6 times a day. Drink plenty of fluids orally and keep yourself hydrated. You will mostly improve in 1-2 days. Of not improving or worsening in the form of vigorous coughing, breathlessness etc then immediately consult doctor and get done clinical examination of respiratory system. You may need inhaled bronchodilators and inhaled corticosteroid (ICS). Hope I have solved your query. I will be happy to help you further. Wish you good health. Thanks."
},
{
"id": 186376,
"tgt": "Is mouth rinse effective in treating problems below the gum line?",
"src": "Patient: About a month ago, I bit down on an apple, and experienced pain in my lower jaw. It radiated up to my ear. The pain subsided. My dentist took x-rays, saw no infection, but put me on Keflex 500 mg, which I took till finished. FYI--my sinuses flared up (headaches) & my ears ached, for a while....both have since subsided. In the last week, my teeth have (overall) were sensitive. I've also developed a lump, below the gum line, in the same area, that I originally felt pain. I called the dentist again, and he has me rinsing 2x daily with Chlorhexidine 0.12% He did not renew the Keflex. My tooth sensitivity has subsided. The lump is still there, but seems to be improving. Is the mouth rinse effective in treating problems BELOW the gum line? i.e. lumps & is more keflex recommended? Thanks! Nancy Doctor: thanks for your query, i have gone through your query, the lump below the gum line could be a periodontal abscess or periapical abscess or an healed sinus tract. mouth wash will maintain oral hygiene, but the cause has to be traeted. consult a oral physician and get it diagnosed. if its tooth infection better to take a course of antibiotics like metronidazole, do saline gargling. i hope my answer will help you, take care."
},
{
"id": 59801,
"tgt": "Sonography shows moderate hepatomegaly with grade 1 fatty filtration, mild gallbladder wall thickening, mild splenomegaly. Conclusion?",
"src": "Patient: hello doctor, my brother is having done sonography in that following symptoms shown- moderate hepatomegaly with grade 1 fatty filtration,mild gallbladder wall thickening, mild splenomegaly ,grade 1 medical renal disease, and minimal free fluid in abdomen and pelvis.what does that indicate.please guide us .my email YYYY@YYYY Doctor: Hi, Welcome to HCM Every investigation is to be corelated clinically,first we should know why this sonography was ordered and what is the clinical background of the patient. Symptoms are complaints for which seeks medical advise. This sonography indicate Fatty changes in liver with gall bladder thick walls which is commonly seen in Diabetes,Lipid abnormalities,Alcohol liver disease. Spleen is also enlarged may be due to fever? or alcohol induced.Echogenic changes are to be corelated to kidney function. Free fluid in abdomen may point out Ascitis which corelates with Liver disease. Hence please visit your treating physician he will take care and advise accordingly. Good luck to you"
},
{
"id": 170961,
"tgt": "Suggest treatment for hyper-activeness in children",
"src": "Patient: I have an 11 year old grandson that is always tired and grouchy. He acts childish at times and almost hyperactive. He can calm down when he is involved in a project such as building a Lego kit. Today about 5:00 he fell asleep and when he woke about an hour and 1/2 later, he angrily pounded his footsteps as he went downstair and demanded to know when dinner would be ready. What is going on with him? Doctor: DearWelcome to HCMWe understand your concernsI went through your details. The given symptoms do not warrant hyper activity. Tiredness and grouchy symptoms are not regarded to be part of hyperactivity. Being hyperactive at times is also not hyperactivity. Another thing to note is he is attentive, calm and involved when doing some work / project which is quite engrossing or requires attention. Therefore Attention deficit can also be almost set aside. You need to observe his general behavior for any behavior problems such as aggressiveness or attention seeking. Please consult a child psychologist for diagnosis or you may provide detailed symptom list here. If you require more of my help in this aspect, please use this URL. http://goo.gl/aYW2pR. Make sure that you include every minute details possible. Hope this answers your query. Available for further clarifications.Good luck. Take care."
},
{
"id": 171564,
"tgt": "What causes itching on scalp?",
"src": "Patient: how can you tell if a child has hair cast vs. lice. She has some itching but no readness and I have not seen any bugs. There are these things in her hair but they are not at the base of ths scalp and can be removed very easly unlike nits. We have battled lice two times in her 9 years. Doctor: Hi,Welcome to Hcm,Its hard to identify the type of insect on the scalp. But if its itching that badly then there must be some lice or some fungal infection on the scalp. Best is to use a good shampoo which is mostly antifungal shampoos . these will help in relieving infection and most of times itching resolves. However if there is no adequate response inspite of trying all this , its better to consult a dermatologist. Take care and happy to help with further queries."
},
{
"id": 50926,
"tgt": "Have back pain and blood in urine. Right kidney is enlarged. Advice?",
"src": "Patient: I have been having some mild lower back pain and have blood in my urine. I had an ultrasound of my kidneys and the right kidney is enlarged. I m very concerned and would like to know what this could be. I have had some soreness to my right back for a few months but it wasn t all the time just if I pressed on it a certain way sometimes it would feel sore in that area. I don t have an infection from my urine culture done. Doctor: Hi friend, Welcome to Health Care Magic Stone causing obstruction is to be considered. CT scan may be needed for further evaluation. Acute Nephritis is another / Renal function is normal? / What are the casts? Proteinuria? You should see a nephrologist. Take care Wishing speedy recovery God bless Good luck"
},
{
"id": 34544,
"tgt": "What does a body temperature of 98.9 suggest?",
"src": "Patient: Dear Drs. I am a 50 year old woman who was just diagnosed with Breast Cancer. I have had bilateral mascectomies and reconstruction (DIEP) almost 2 months ago. I began Chemotherapy TC last week on Thursday, and had a Neulasta shot. Day 6, and my WBC is 1.7 and my Neutraphils are .36, very low. I was given Cipro and told to watch for fever (says 100.5 in Neulasta guide). I normally run about 97.3, but am up to 98.9. Is this a fever? Should I go to ER? or call? Doctor: Thanks for posting you query to health care magic.you dont need to worry about your temperature as 98.4 degree Fis normal temperature and mild increse could be due to diurnal variation hence presently no need for medication or investigation . Chemotherapy itself causes tissue damage and produce liberation of inflammatory mediator may be espnsible for rise in temperature .If it peresist for long time or increases above 100 degree or cause discomfort to you consult to your clinician .hope you are satisfied with my answer . feel free to communicate if any query .regards,Dr.Manish PurohitInfectious disease specialist"
},
{
"id": 83385,
"tgt": "Is there any side effect after consuming expired medicine?",
"src": "Patient: My 7 year-old son had a fever 4 days ago, I gave him an ibufropen 3 times before discovering that the medicine has already expired. Today, he has fever again. I am worried that it might be the effect of the expired medicine. What is the effect of taking in expired medicine. Do I need to take my son to the hospital? Doctor: HiExpired medicines lose their potency and will not be efficacious when used. Otherwise, they do not cause any side effects. Certain preservatives used may cause any allergic reactions or vomiting. In case of such symptoms, the child requires medical examination. You should also use new ibuprofen since fever may not be reduced with an expired drug.Hope I have answered your question. Let me know if I can assist you further. Regards, Dr. Saranya Ramadoss, General & Family Physician"
},
{
"id": 135398,
"tgt": "What causes pain and bruise on the thigh?",
"src": "Patient: I am a very active 34 y/o male but I can not recall having any trauma to the posterior side of my left thigh. I awoke today to pain in said area, I looked and saw a cluster of about 10 bruises, none bigger than a quarter size and some as small as a dime. Doctor: Alteration in the coagulation cascade /clotting factors in blood can cause such conditions even without significant trauma. Certain diseases, drugs such as anticoagulants,antiplatlets,NSAIDS,etc can cause so. U may need to get a blood work up to ascertain the cause. Most of the time it may be of no underlying pathology and left alone.."
},
{
"id": 54145,
"tgt": "Why do I feel jamming in the middle of my chest while walking?",
"src": "Patient: I am 51 yeras old and having good health. Last year I had undergone gall bladder surgery in which my gall bladder is removed. I am fine now and there is no pain. However, some time while walking I get feeling of jamming in the middle of my chest? Also please tell me why there is itching on body and red rashes afer every 2-3 days? Doctor: hello Sir, your present problem does not seem to be related to your gall bladder surgery. following exercise if you develop jamming feeling it most probably your fast heart beat that you feel after exercise .can you tell me what's your blood pressure in normal days? please provide me the details about your rashes when they develop? timings? type of rash? it's distribution. over body? did you notice jaundice or yellowish discolouration of your body? will help me sort your actual problem... thankyou"
},
{
"id": 73496,
"tgt": "How to treat severe nasal congestion while having the cough?",
"src": "Patient: Hi I have had post nasal drip now for a week. At night I cough so much I can t sleep well. now I have a fever. The symptoms are worse at home alleviating during the day.I have been checked for allergies and am alergic to mites cats dogs grass. We have cats which haven t bothered me much in the past. I also get full quickly when eating. Doctor: Hello dearWarm welcome to Healthcaremagic.comI have evaluated your query thoroughly .* Suggestions for better recovery- Inhalation of vapors of boiled water with added tincture benzoin or karvol plus capsules 5 minutes each 3 times a day with blanket covered over the head .- Gargles with salted lukewarm water added peppermint oil 3 times a day- Avoid exposure to pollen , dust , mites , grass , cats , dogs .- Maintain hydration with plenty of liquids .- As there is fever , need decongestants , antibiotics , antipyretics after physician check up .Hope this clears your query .Regards ."
},
{
"id": 211112,
"tgt": "How to treat anxiety problem and attention disorder in a 5 years old child?",
"src": "Patient: ok i have a son ho present a ansaety problem and atention disorder what i k do to help him he is only 5 years old and he is or redy be treted by an a terapist b she is no helping ad oll did my son need medication cos she are be inparcial in dis situation and i dont no what to do please help me Doctor: hithanks for choosing healthcare magicyour son is just 5 years old and according to guideline there is no need to give medicine in case of ADHD if child age is less than 6 years. This is normally seen in children and with time he could improve. For attention u can try non-pharmacological method like coloring, putting beads in a thread etc. For anxiety, u can give low dose antidepressant if need. rest u can consult psychiatrist.Thanks"
},
{
"id": 93341,
"tgt": "Constant abdominal pain. Had plastic surgery, was on Keflex. Why ?",
"src": "Patient: Good Morning: I have had abdominal pain for about 1 month now. It began in the area from my navel up to my ribcage (in center, not to either side) and now, about 3 weeks later the pain is in the area from my navel down to my pubic bone. The pain is like a biting or pinching sensation. About 10 weeks ago I had plastic surgery and was on Keflex antibiotic for one week. One week ago I had to have more surgery for the wound dehisence and I am on Keflex antibiotic once again. This pain is a 6 to 8 on a scale of 1 -10 with 1 being no pain and 10 being unbearable pain. The pain occurs all day, after eating, after not eating for hours - continually. Do you have any thoughts on what this might be? Thank you very much for your answer. Doctor: Hi, how are you feeling now? I hink this pain can be due to infection in the wound.As your pain is continuous , it is less likely due to any problem in the intestine. You need to b examined by your surgeon to look for the local cause of pain.Hope my answer helps u"
},
{
"id": 62232,
"tgt": "What could lumps with bruises on forehead suggest?",
"src": "Patient: my 5 year old daughter was bouncing on her bed and fell and hit her forehead on the corner of the radiator 2 weeks ago. a large lump came up straight away and bruising. the lump went down a lot 2 days later. she was not sick or showed any signed of drowsiness. in fact she was fine. the lump is still there and very hard now 2 weeks later. should i be concerned? Doctor: Hi, dearI have gone through your question. I can understand your concern. She has history of trauma. So she may have some hematoma formation due to extravasation of blood. It will heal gradually. Nothing to worry about that. If she has pain yhen analgesic plus anti inflammatory drugs like ibuprofen is helpful. Otherwise no treatment is required. Apply ice massage bon it. Nothing to worry about that. Just be relaxed. Hope I have answered your question, if you have doubt then I will be happy to answer. Thanks for using health care magic. Wish you a very good health."
},
{
"id": 19312,
"tgt": "What causes an elevation in the BP and pulse rate despite taking Losartan?",
"src": "Patient: My blood pressure is usually 127/77 pulse between 60-100. I started to drink coffee and just in the last day. My body is now 147/87 pulse between 80 to 130. I have woking up hot cold seats last two nights after I wake up I feel better just do no how the dramatic change in 3 days. I am on lartsartan 50/25 hrtc Doctor: dontvworry...just take enough sleep.....chng in biological cycle may cause such isddue .......dont take much coffee.....usually not not more than 1 or 2 times a day is advced"
},
{
"id": 107603,
"tgt": "Suggest treatment for stomach bloating , lower back pain and shoulder pain",
"src": "Patient: female, bloating, pain in solar plexis. lower back, back right shoulder, pain level 8. no stones, no blockage of bowels, x rays, ultra sound and ct scan showed nothing, no blood in urine or stools. slight irritation of esophogus but camera showed stomach ok. ??? Doctor: Dear-thanks for using our service, I understand your concern and will help you with my medical advise. You have had multiples studies that have eliminated other possibilities.However, you might need to have MRI of lumbosacral to see if there herniated disc that is pinching sciatic nerve which has roots going to the pelvic area and can be causing the pain. Other possibility is irritable bowel syndrome .This condition gives the bloating and abdominal pain. Shoulder pain could be secondary to bursitis or osteoarthritis. I recommend a shoulder xray for better evaluation. I hope that my advise has been helpful, please hit thanks if it was.Dr.JC"
},
{
"id": 214891,
"tgt": "Ringworm infection on lower buttocks, diagnosed with HIV. Home remedies?",
"src": "Patient: Hi I was recently diagnose to have HIV and am still awaiting for my first apointment with the CDC. I am however now having a ringworm problem at my lower butt area. As I am still awaiting for my first appointment with the CDC I am currently having no medications in regards to my HIV. Is there any ways I could keep my ringworm problem in control at home at the mOment ? Doctor: Hello, As per ayurveda you have to follow some diet regulation for ringworm infection. Like, avoid curd, pickle and other sour food, fermented food, pungent food. Do not wear skin-tight cloths For local application use HALDI(turmeric powder) and PALASH BEEJ(Butea seeds) and make paste with Luke warm water and apply it on infected part, for 3- 4times in a day, till the infection is cured. Do not take any internal medicines yourself right now, consult your doctor for HIV. If you follow diet and apply this herbal paste you can definitely get good result. If you have any query regarding Ayurveda please feel free to ask. Thanks. Dr. Pradip Hira (B.A.M.S) [Ayurveda Consultant & Panchkarma Specialit-Ahmedabad]"
},
{
"id": 144245,
"tgt": "Suggest treatment for Parkinsonism and cervical spinal stenosis",
"src": "Patient: What kind of treatment regimen do you recommend for someone dually diagnosed with Parkinsonism and severe central cervical spinal stenosis? I was diagnosed with PD about the same time as I had a posterior laminectomy for the stenosis about five years ago. Now I am having some movement problems with initiating movements to the side and backwards, some significant balance problems when moving backwards and some voice problems (my neck has been stiff since the surgery). Doctor: Hi, I am Dr.Bruno and Let me answer your query.All the symptoms you have said 1. movement problems with initiating movements to the side and backwards, 2. balance problems when moving backwards and 3. voice problems 4. Neck has been stiff since the surgery.Can be due to PD itself So, I would suggest you to first get your PD Treated. Please take the necessary drugs for at least six months and see whether the symptoms are decreasing. If there are any problems persisting after six months, we can attribute them to Cervical Spine If you need any clarification / have doubts / have additional questions / have follow up questions, then please do not hesitate in asking again. I will be happy to answer your questions."
},
{
"id": 123799,
"tgt": "What causes heating sensation in knee caps?",
"src": "Patient: I am 31,female..I have been having this strange heating sensation in my knee caps for the past three days. It occurs about every 5 to 10 minutes. It literally feels like a miniature hot flash on my knee caps. Do you have any idea what this could be? Doctor: Hello, As you feel hot above the knee cap I think you should check your uric acid levels. As due to increase in the uric acid levels there might be burning sensation, Also, if there is any nerve root compression in the lumbar spine then also the burning symptom can come. Need a thorough clinical examination and assessment of the knee and lumbar spine along with lab tests to figure out what should be the problem which is causing such burning sensation. Hope I have answered your query. Let me know if I can assist you further. Regards, Jay Indravadan Patel, Physical Therapist or Physiotherapist"
},
{
"id": 118549,
"tgt": "Had mouth ulcer, fever. Lab results shows low white blood cell count. Now doing test for lupus",
"src": "Patient: hello, I recently broke out in hives 2moro days after going to the doctor for an ulcer in my mouth. I ran a fever ond and off of about 102 for three days. this is the second time I have had an allergic reaction like this. Lab results show I have a low white blood cell count. They have ran the obvious tests. Are now doing a test for lupus. On a normal day the only symptoms I have is that I'm always tired, even after being well rested and bout don't too much physical activity. Any suggestions? Doctor: Hello,Mouth ulcers can occur due to aphthous stomatitis.I would advice you to take oral corticosteroids.Avoid spicy,allergic foods.Maintain oral hygiene well.Tobacco or pan chewing has to be discontinued if you practice any.Vitamin b complex as well as iron supplements has to be administered.Use chlorhexidine mouthrinses to gargle.Take care."
},
{
"id": 132995,
"tgt": "Suggest treatment for numbness and painful weight bearing after injuring ankle",
"src": "Patient: Fell at football tonight and fell onto my ankle with body weight think iv rolled my ankle and heard it crunch. Not very much swelling, no bruising but limping, pain, some numbness and tender to touch. Pain goes up into my leg. Foot goes numb. Can t stand and put weight on it. Doctor: hihope this msg finds u in good health u have sprained ur ankle.i suggest u get an xray or mri done to find out extent of injury before weight bearing thanks Take care god bless"
},
{
"id": 126311,
"tgt": "Can Ibuprofen be taken for recurrent pain in the hips when scheduled for a hip replacement?",
"src": "Patient: I am scheduled for hip replacement in April 2018. I had a cortisone shot in November but the pain has returned. I have a left over 800 mg ibuprophen from my daughters gall bladder surgery and am thinking of taking it. My doctor is not in today so what do you think? Doctor: Hello, If there is more pain yes you can definitely take Ibuprofen. You can try other painkillers like Diclofenac, Tramadol, Pethidine, etcetera. Since you might have an obvious reason for hip replacement and it's not good to tolerate more pain before surgery. Hope I have answered your query. Let me know if I can assist you further.Regards,Dr. Archana"
},
{
"id": 68345,
"tgt": "Suggest treatment for the lump on tailbone",
"src": "Patient: I have a very swollen tailbone- near the bottom of the tailbone there is a lump/ a swell. It is very uncomfortable and hurts to walk/lie down/sit down and is painful when i put any pressure on it. I have been taking painkillers which do take some of the pain away, but is this something that will go away on it's own or should i see a doctor? Doctor: Welcome to health care magic. 1.Suggest you to see a doctor for this issue - the reasons is there are quite a few complications for the lumps on the tail bone / perianal region.2.Mostly possible cause will be hair follicular infection / infective aetiology which is causing pain and tenderness.3.The treatment in this case after examination a course of antibiotic ( if small one)4.If not responding to the treatment or larger in size then a small incision and drainage will be necessary.5.The complications are pilonidal sinus, fistulas ( which needs operative treatments) so treat it as soon as you can - take good post procedural care. Good luck.Hope i have answered your query.Any thing to ask do not hesitate. Thank you."
},
{
"id": 50850,
"tgt": "Stage 5 renal patient, catheter removal done, have strong urge to urinate. When will I feel normal?",
"src": "Patient: The sensation to pee is strong, but can't--saw a urologists today and she says ttha my bladder is empty. She had a cathether placed and removed because there was no need to keep it in--when does the sensation ends? I am also stage 5 renal patient expecting to stop, so no surprises, but the sensation/urge--how long til I feel normal? Doctor: hi as you are stage 5 CKD patient, the urine output is going to be low and will cease as the disease progresses. The sensation of urge to pass urine when you have no urine in the bladder could be due to bladder spasms. the bladder spasms could be due to cystitis or due to neurological problem with the bladder. First to rule out bladder infection (cystitis) by doing urine routine test and treat with antibiotics. If no infection contact your Doc , to prescribe some anti spasmodics for bladder spasms. Take care"
},
{
"id": 190288,
"tgt": "cheek bites due to improper fitting of dental bridge. How long will it take to heal?",
"src": "Patient: My dental bridge was not fitting properly and my teeth alignment got disturbed because of that. It caused a lot of cheekbites and my dentist kept saying it will go. But now it is more than six months So I changed my dentist, who diagnosied that my teeth alignment is not Ok and he removed the bridge. How long will it take for the chek bites to heal completely? I am applying kErocort cream twice a day for the past 5 days. Doctor: how long it will take to heal depends how badly the cheeks have been bitten. you can yourself estimate the time of healing by comparing the response since you started applying the cream after removing your bridge. have a healthy diet along with multivitamins."
},
{
"id": 39795,
"tgt": "What is the treatment to prevent typhoid fever and what does normocytic normochromic RBC mean?",
"src": "Patient: 1. From the last 4 or 5 years i was had a fever(not typhoid fever )frequently after 4 or 5 months. Now i have had a typhoid fever every year from d last 2 years , not a severe typhoid only 1:120. I have never had any typhoid fever before. plz tell me what should i do to prevent it.my all test reports and chest x-rays r normal. 2. In my test report there is a red blood cells shows mild aniso- cytosis. predominantly normocytic normochromic cells r present.What does this line means and tell me d cause and remedy of d RBC problem. Doctor: The WIDAL test reports both the flagellar (H) and the somatic (O) antigen titres. Significant titres to diagnose a patient as having typhoid are:Salmonella typhi H: 1:200Salmonella typhi O: 1:100Your titre is 1:120; if this is the O antigen titre then it is not significant.Typhoid infection can be prevented by simple measures such as consuming boiled water, thoroughly washing vegetables, fruits before consumption. Typhoid mainly spreads through contaminated food and water.As far as your red cell count is concerned a normal RBC count would be about 4.5 million/mm3. Mild anisocytosis, normochromic cormocytic cells are normal findings as long as your hemoglobin and hematocrit is within the normal range."
},
{
"id": 109967,
"tgt": "Is coumadin recommended after undergoing hip surgery?",
"src": "Patient: my mother recently had hip surgery and is taking coumadin and pain relievers. she is very neatious and cannot keep anything down. i was wondering if she possibly allergic to the coumadin. i think her mother was and i can't seem to find anyone who remembers. her mother has been deceased for 17 years. any thoughts? Doctor: Hello, I have studied your case.Coumadin is given to prevent venous thrombosis after major surgery.Ultrasonography/colour Doppler leg will help I will advise you to do PT/INR.If you suspect allergy to coumadin then you can consult your doctor and change it to say rivaroxaban which is newer.If required you may need to do blood investigation for prognosis and recovery.I will advise to check your vit B12 and vit D3 level.Hope this answers your query. If you have additional questions or follow up queries then please do not hesitate in writing to us. I will be happy to answer your queries. Wishing you good health.Take care."
},
{
"id": 108991,
"tgt": "Suggest cure for severe lower back pain",
"src": "Patient: I have severe lower back pain due to a bulging disc and a pinched sciatic nerve.I am currently on 50mic fentanyl patches every three days and 10-325 norco (60 per month).Every Dr. I have seen will not put me on anything strong enough to work.What the heck am I to do? This is really paying a toll on my depression by loosing my job,can't hardly do anything or go anywhere,on disability and now I can't get any Dr. to help me where I need it.My depression is bad enough that I need medication but all the ones I have been on made me worse.I need some serious help with my pain. Doctor: Hi,Start hot water fomentation thrice a day.once pain reduces start core stability exercises. Do not bend forward or lift heavy objects. Consult a physiotherapist."
},
{
"id": 46774,
"tgt": "Suggest treatment for high CRE level",
"src": "Patient: YYYY@YYYY sir DHARMENDRA SPEAKING IAM SUFFERING KIDNEY PROBLEM FROM LOST 5 YEARS OF 5.5 SR CRE I HAVE BOTH SUGAR AND BP I HAVE MAINTAIN MY DAIT NOW ONE WEEK BACK I GOT FEVER AND COLD MY CRE HAS JUMPED TO 10.5 PLEASE GIVE ME YOUR OPAION Doctor: Being a diabetic and hypertensive you are more prone for Renal failure. Initial high creatinine could be due to these factors. Jumping of creatinine could be due to infection. First control your infection with antibiotics and then you may require few sessions of dialysis if indicated. Hopefully your creatinine would come back with these measures. All the best. If you have any questions feel free to contact me."
},
{
"id": 35977,
"tgt": "Suggest treatment for shingles, fever and chills",
"src": "Patient: I was recently diagnosed with Shingles/autoimmune disease (October 1, 2014). I was very ill for almost two months, sometimes in bed for a day or two, here and there and always itchy with lesions on the right side of my lower spine...not so much pain, but itchy. I also had hives. I was weak and fatigued, had headaches, low grade fever and chills and lost 15 lbs. Around Halloween I began to feel a little stronger, the hives were gone and it still took a couple of wks. to begin to feel normal again. I began to gain some of the weight back. Last week I got two small lesions under my nostrils and yesterday I was in bed most of the day with low grade fever and chills again. Does shingles reoccur or does it have more to do with the autoimmune disease? Will I have to be sick for two months again? I moved here from Iowa and do not have a doctor who treats this disease. I have seen a Pulmonologist. Doctor: Hello,This is Dr. Papaqako answering your question.Shingles occurs when the virus that causes chickenpox starts up again in your body. After you get better from chickenpox, the virus \"sleeps\" in your nerve roots. In some people, it stays sleeps forever. In others, the virus \"wakes up\" when disease, stress, or aging weakens the immune system. Most people who get shingles will get better and will not get it again. But it is possible to get shingles more than once.If you have shingles, symptoms happen in stages. At first you may have a headache or be sensitive to light. You may also feel like you have the flu but not have a fever.Later, you may feel itching, tingling, or pain in a certain area. That's where a band, strip, or small area of rash may occur a few days later.I don't think the fever is connected with shingles, but it may be another virus, that because of low immunity have reactivated a simple Herpes lesion. That because the time difference between the lesions and fever is large.Treat it as a common cold. But if in 3-5 days, is not getting better, or is worse, then I would recommend to see a doctor.Hope this helps, Take care."
},
{
"id": 172491,
"tgt": "Are Pediavit and Fenistil good for 6 months old baby?",
"src": "Patient: My baby is 6 months and her weight in birth was 1.7k . Now her weight 5 k and Dr give her pediavit select to increase her weight. Also she itshing her ear and Dr say she has congestion on her ear and they give her fenistil 5drops twice Are this two drops good for her in her age ? Doctor: Thanks for askingI gone through your question, your 6month old baby whose was low birth weight is now 5 kg. and she having itching in ear. She is on pediavit and fenistil. First I want to say you that she is gaining weight as per expected, so her care giver is giving good care. her weight is triple from her birth weight. you can give her multivitamin drop pediavit, . As her age is now 6 month , now start giving complimentary feeding like mashed banana, gruel of rice or weight or pulses. there is harm to give drop fenistil to relieve her congestion , hope i answer your query"
},
{
"id": 57689,
"tgt": "What could be the reason for having elevated liver enzymes, fatigue and delayed healing process?",
"src": "Patient: My son has elevated liver enzymes, his CT shows a fatty infiltration on his liver/an enlarged spleen, and his VitD levels are falling, despite taking a VitD supplement. he has fatigue, and has noticed delayed healing process. any thots on what might be going on? Doctor: Dear Friend.Hi , I am Dr Anshul Varshney , I have read your query in detail , I understand your concern.it is very difficult to comment on with these details. Also need to know his age , alcohol status , and need to see the reports.Vitamin D production in body involves liver also , if his liver is not functioning well , levels will fall down. If he is an adult possibility of Chronic Liver disease (Cause can be Alcohol , Viral Hepatitis) is high since he has enlarged spleen.Let us know ,if he has jaundice , esophageal varices , ascitis.If you could give us detail history and also if you could share the reports , we would be able to guide you better.This is my personal opinion based on details available here. If you want to discuss your issues further, you may please ask usStay Healthy. Dr Anshul Varshney , MD"
},
{
"id": 132946,
"tgt": "Does lymphedema pump help to reduce pain and swelling in pelvic bone?",
"src": "Patient: I was involved in a motorcycle accident back in march 15, 2014 I broke my pelvic bone had to put plate in front and back and screws ... I am having a lot of pain I can t walk for another 6 weeks. I was looking to get a lymphedema pump to help pain, swelling and circulation.....help please Doctor: Hello, I have studied your case history.You will need X ray of bone where implant is placed.There is possibility of impingement of implant along soft tissue.As bone union is complete and implant is producing pain you can remove implant consulting your doctorClinical examination is important to know site of pain.You can also send x ray photo on my profile.So implant removal is best solution for pain if loosened or infected.Lymphedema pump may not help for pain reduction.Hope this answers your query. If you have additional questions or follow up queries then please do not hesitate in writing to us. I will be happy to answer your queries. Wishing you good health.Take care."
},
{
"id": 169059,
"tgt": "What causes rashes on penis and testicles?",
"src": "Patient: Hi, my son has a rash on his penis and testicles. He has had it sometime now (over 3 weeks) and has been using a cream from the Doctor for thrush (jock itch) twice a day. He was complaining tonight, so I checked it out. The testicles are weeping, swollen and raw looking. What could this be? Doctor: you didin't mention the age of his son; a weeping rash is common on kids with diapers and avoiding diapers soaked with urine and apply daktarin with hydrocort will sooth and heal them faster."
},
{
"id": 32482,
"tgt": "What is the dosage of rabies injection after dog bite?",
"src": "Patient: hello One house dog bite me 0n 23 rd Jan. I was refered to a doctor in nearby hospital. He gave me VERORAB vaccine but did not gave rebies immunoglobine. After that i took one more shot on third day. if i dont take human immunoglobuline with rebies vaccine is it ok? Because it's not available in saudi arabia. Doctor: Hi & Welcome.I can understand your concern.Incomplete data like site & the type of the wound. Whether tetanus toxoid received or not?As per the WHO-recommended guidelines for optimal post dog bite rabies prevention, the five-dose regimen is administered on days 0, 3, 7, 14 & 28 into the deltoid muscle.Anti rabies immunoglobluin is usually administered in case of single or multiple transdermal (deep) bites or scratches, & the contamination of mucous membrane with saliva from licks.If you have dog bite suggestive of the above mentioned wounds, then anti rabies immunoglobulin should be administered just before or shortly after administration of the first dose of vaccine given. If it is not immediately available, it can be administered up until the seventh day after initiation of the primary series of anti rabies vaccination schedule.Hope this answers your query.Wishing you best of your health.A feed back is appreciated."
},
{
"id": 181469,
"tgt": "What causes painful mass in between the cheek and jawbone post Lidocaine injection?",
"src": "Patient: 1 week after having a lidocaine injection for cavity fill I have a yellow/purple bruise on 3/4 of my cheek with a very hard painful mass between my cheek and jawbone. I was given ammoxocillin last week after the procedure because my face became swollen and bruised immediately. Should I be alarmed do I need to see a different dentist? Doctor: Hi..Welcome to HEALTHCARE MAGIC..I have gone through your query and can understand your concerns..As per your complain it seems that the anaesthesia injection needle have been penetrated too deep into the soft tissues and it has reached the venous plexus located deep in the upper back region and it has caused Hematoma formation that is leading to bruising and swelling..I would suggest you to consult an Oral Physician and get evaluated and he can do a thorough clinical evaluation and advise you investigations like x Ray to rule out the exact cause of swelling..As Hematoma is a self limiting condition, it will resolve gradually but can take a longer time to resolve..You can do alternate warm and cool compresses over the swelling atleast 3 to 4 times a day..Take anti-inflammatory painkillers like Ibuprofen..Hope this information helps..Thanks and regards.Dr.Honey Nandwani Arora."
},
{
"id": 98258,
"tgt": "Had tonsillectomy four months ago. How to cure from throat pain ?",
"src": "Patient: Hi now it was four month now after tonsillectomy but these days aim feel painful and headache i don t know what to do hi,i have already made tonsillectomy four month ago,but these days when i drink coca-cola i feel painful,and these days i have pain in a throats i don t know what to do?gender female and 36 years old Doctor: Hi ! Welcome to Healthcare Magic Tonsils acts as gatekeeper preventing any infection to directly enter into the lungs, but as your tonsillectomy has been done, you need to be more careful henceforth. Avoid taking cold and sour things like cold drinks,oranges,grapes,lemon very frequently. Whenever you get such kind of pain you can start salt water gargles and take Tablet THROAT AID , 2 Tab. thrice daily. Thanks and take care"
},
{
"id": 143253,
"tgt": "What causes headache, facial twitching on the right side of my face and tearing of the right eye inspite of an injury?",
"src": "Patient: I had a serious fall in October causing cuts and abrasions to my face and head. I was seen in ER and had xrays of head, face and neck No breaks. but, since have experienced headaches and facial twitching on the right side of my face and tearing of the right eye. Doctor: Hi, Welcome to HealthCareMagic.com I am Dr.J.Mariano Anto Bruno Mascarenhas. I have gone through your query with diligence and would like you to know that I am here to help you.Question : but, since have experienced headaches and facial twitching on the right side of my face and tearing of the right eye.Answer : The Seventh Cranial Nerve on Right Side is injured. This can be probably due to fracture of temporal bone. Please consult a Neurosurgeon for further evaluation. The headache can be due to post traumatic sequelaeHope you found the answer helpful.If you need any clarification / have doubts / have additional questions / have follow up questions, then please do not hesitate in asking again. I will be happy to answer your questions. In the future, for continuity of care, I encourage you to contact me directly in HealthCareMagic at http://bit.ly/askdrbruno Best Wishes for Speedy Recovery Let me know if I can assist you further.Take care."
},
{
"id": 156478,
"tgt": "What precaution can I take to prevent cancer as I use gutka?",
"src": "Patient: I was taking gutaka for last three years. Now I have the problem in mouth. I cannot open mouth fully. My mouth became whitish. What is the problem? Dentist says the first stage of cancer. Is it true. Now what can I do to prevent from occuring cancer. Help me Doctor: Please immediately quit gutka (tobacco) use.You need to have a biopsy of the whitish patch to rule out cancer (malignancy).Please discuss it with your doctor."
},
{
"id": 75952,
"tgt": "What is the treatment for stress and chest pain?",
"src": "Patient: hi I am 25, I was very upset and yelling yesterday with a lot of straining and now I have pain in the center of my chest. It is constant no matter if I stand or sit or shift positions. I have no known history of heart problems. I am not overweight but don't exercise much. Doctor: Thanks for your question on Healthcare Magic. I can understand your concern. Since your chest pain is started after straining, possibility of musculoskeletal pain is more. This pain is worsening by your stress and anxiety. So we should first control your stress and anxiety. So consult psychiatrist and get done counselling sessions. Try to identify stressor in your life and start working on it's solution. You may need anxiolytic drugs too. For chest pain, follow these steps. Avoid heavyweight lifting and strenuous exercise. Avoid movements causing pain. Avoid bad postures in sleep. Take painkiller like ibuprofen. Apply warm water pad on affected areas. Don't worry, you will be alright with all these. Hope I have solved your query. I will be happy to help you further. Wish you good health. Thanks."
},
{
"id": 193376,
"tgt": "Suggest remedies for excess masturbation leading to weakness",
"src": "Patient: hi guys there....i been doing hand job or mast****** for 4 or more years. now feeling pain in knees and cant stop it and i don't know how i may stop it. after every time i did it , i planned not to do it again..i cant even run now, cant work usually in house general tasks, if i do i start feeling pain in my arm and they start vibrating itself and cant stop it...help me thnx plz Doctor: Hello, Please understand masturbation is safe and never unhealthy if it is done in moderation. You can masturbate 3 or 4 times in a week. If you have good stamina it can be more. Even though excess masturbation can cause tiredness, it will not last if you discontinue masturbation for some days. In your case, please provide more details about your masturbation habit. Hope I have answered your query. Let me know if I can assist you further. Take care Regards, Dr K. V. Anand, Psychologist"
},
{
"id": 205234,
"tgt": "What causes difficulty in swallowing food while suffering from anxiety?",
"src": "Patient: Hi, I have for a long to,me eaten very quickly and never chewed my food enough, swallowing large pieces of food. After eating, of which I've noticed more so over the past few months, on occasion the food seems to get stuck or go down slowly producing a tightening uncomfortable pain in my chest. I have a glass of water and each gulp hurts but then five to ten seconds later the pain suddenly goes as though the water has helped lubricate the esophagus. However tonight, it hapened while i was eating steak and rice, I felt the pain build after a few mouthfulls. I stood up, the pain seemed to get worse at doing this and I an unusual sensation, as though I was having trouble breathing along with this tight lodged pain! I had water and within a few seconds the pain had gone. I am worried I might have a growth in my esophagus that is catching the food as it goes down. I also suffer from bad anxiety and have had panic attacks in the past. I definitely eat way way to fast and don't chew enough but am worried its been gradually getting a little worse over the last few weeks/months. It doesn't happen all the time - I had breakfast and lunch and no issues - just occasionally - three or four times a week. I'm male, 38 years ols with no knows medical conditions. Thank you. Doctor: Hello, You are well aware of your wrong eating style.. All the symptoms you have mentioned are attributed to that only.. First improve these habits.. and see... if still these symptoms persists then visit ENT surgeon.. to rule out any growth near oesophagus.."
},
{
"id": 166604,
"tgt": "Suggest treatment for delayed speech in a child",
"src": "Patient: Hello Doctor. I have an 18 month old boy. I checked the milestones for a baby at 18 months and my son is able to do all things other than communication. He did not started calling me Mummy yet. He does not pay attention to me but now once in a while he do. Even though his weight is correct for his age but he looks thin. We live in UK. He does holds his playing toys , balls but he does not hold the feeding bottle or a bowl of food. Please help me in improving his communication and vocabulary. Doctor: Hi...Thank you for consulting in Health Care magic.I understand your concern.I have few questions and suggestions for you -1. What is his birth weight and current weight?2. When did he start walking?3. Did he cry immediately after birth or did he require any special assistance to take the first breath?4. How long does he see television or tablet or cartoons per day?5. Did any one of the parents' had a speech delay as children?6. How many word s does he speak as of now- I mean his vocabulary consists of how many words?Please get back to me with answers to the above question so that I can guide you better.You can approach me at the following link.Once the page opens there will be an option below my image as \u2013 ASK ME A QUESTION \u2013 click on it.Please find the link below -www.healthcaremagic.com/doctors/dr-sumanth-amperayani/67696Regards - Dr. Sumanth MBBS., DCH., DNB (Paed).,"
},
{
"id": 168804,
"tgt": "How to make sure baby if unharmed after falling?",
"src": "Patient: My 6 week old fell off the couch. After a few seconds of crying he started falling back asleep. I started feeling parts of him to see if he d react like he was injured but he didn t. He just went back to sleep. What else can I do to make sure he s ok? Is it ok for him to be sleeping now? Doctor: hi, in cases of head trauma there is high chances of brain injury. There are some symptoms which are found When brain injury occurs like persistent crying, seizues, persistent vomiting, bleeding from mouth or nose. If above symptoms are not present, then most of the times child does not have brain injury. However, it is better to get the child examined by a Neuro surgeon, and if required a CT scan of head should be done. I hope this will help you. Take care."
},
{
"id": 143350,
"tgt": "Suggest medication for dizziness and lightheaded due to seizures",
"src": "Patient: My mother, who is 78 experiences lightheadedness and dizziness. She says she then hears a loud noise in her ears sometimes getting frighteningly loud. At one point she experienced seizures right afterwards but no longer has the seizures. These episodes start off mild and and get progressively worse. Doctor: Hello!Welcome on HCM!I understand your concern and would explain that her symptoms could be related to the seizure aura or prodrome, which means that they are part of the seizures. Is she taking any antiepileptic drug now? I would like to know her current therapy and past medical history. I would recommend consulting with a neurologist for a careful physical check up and some tests: - a brain MRI- an EEG- thyroid hormone levelsIn my opinion antiepileptic drugs like carbamazepine, gapabentine or levetiracetam would be helpful to relieve her symptoms. Hope you will find this answer helpful!Best wishes, Dr. Aida"
},
{
"id": 71583,
"tgt": "What causes chest pain and persistent cough?",
"src": "Patient: I have a pain in left lung for a couple of months , also a persistent cough and only recently coughing up a brownish substance which has me concerned. I don't feel unwell but get tired quite easily. I I am a 47 yr old female. My GP said he couldn't hear anything going on , and it might be muscular ?Kind regardsL. Mc Doctor: Hello,You have brown sputum expectoration in form of cough since many months. So, it should not be neglected. Your work up is needed in form of chest x-ray.If chest x-ray normal, then no need to worry and bronchitis can be the possibility for which antibiotic course prescribed and symptomatic management done. Provide your reports for giving more comment.Hope I have answered your query. Let me know if I can assist you further.Regards,Dr. Parth Goswami"
},
{
"id": 90463,
"tgt": "What could cause abdominal area pain?",
"src": "Patient: my 13 yr old daughter has had pain in her right side for over a week. when she eats seems to worsen, and when walking alot pain gets worse. they had ran test to check appendix, kidneys gulbladder and tons of bloodwork everything came back good but she is still in alot of pain. what can be the cause of it? Doctor: Hi.Thanks for your query.The diagnosis of a pain in a girl of 13 is sometimes difficult and not easy to be accepted by the parents. < test to check appendix, kidneys gallbladder and tons of blood-work everything came back good> indicates there may not be an organic / bodily problem . This may be IBS= irritable bowel syndrome.Also get colonoscopy, CT scan of the abdomen done. A trial of anti-psychotic medicines is worth if all the tests are negative."
},
{
"id": 143908,
"tgt": "Suggest remedy for Parkinson s syndrome with facial tightness",
"src": "Patient: My husband has been diagnosed with Parkinson s. He is on Sinemet and Azilect and rigidity, balance, cramped handwriting, shuffling walk are all improved, But his most distressing symptom is a feeling of tightness in the face, right hand and foot that is with him 24/7 except when the affected area is in use (such as when he is eating or talking). This is driving him crazy--it is not painful, but so maddingly annoying and difficult to ignore. Also since he started Sinemet he is constantly queasy, as though he is about to be sea-sick. The doctor increased the dosage of Sinemet thinking the tightness would react to a larger dosage but there was no change. Doctor: Parkinsonism can cause such symptoms due to rigidity.....but predominantly on face is bit unusual...I would suggest MRI brain 3T with cis sequence done to rule out other causes. Try tab pregaline at night. Continue other medications. .regards."
},
{
"id": 215330,
"tgt": "Suggest treatment for intermittent pain above left breast",
"src": "Patient: I have been experiencing some intermittent pain in a weird location. It is a diagonal pain, above left breast, and heads toward left shoulder. Pain extends about 6 inches. I ve had some chiropractic because I thought it might be related to my cervical spine, and it goes away for a short period of time but then always returns. I had an adjustment today, and the pain was gone for about 5 hours, but it is back again now. It is not sharp, or constant, but feels kind of like a muscle pulling. If I turn my head tot the right, it definitely exacerbates the feeling. I haven t had any injuries. I m 53, good blood pressure, pulse, no diseases other than degenerative disc disease which is causing me lots of pain in my lower back and SI joint areas. Doctor: Hello, The location and movement of pain give clues as to what is going on. Localized and outside of the chest would iimply a small area. While in theory breast cancer could cause that, the more common cause of breast local trauma, pulling down on nearby structures with age and sagging, bra fitting wrongly is of course more common. Never-the-less, every over 50 women should have a yearly mammogram and that would also show infection/tumors other than cancer/cysts/scar. Hope I have answered your query. Let me know if I can assist you further. Take care Regards, Dr Matt Wachsman, Addiction Medicine Specialist"
},
{
"id": 14364,
"tgt": "How long does pityriasis rosea take to resolve?",
"src": "Patient: hi have been diagnosed ( just visually, no biopsy) with pityriasis rosea. How can you tell when it is healing. I have had it for 2 months. I think it is fading but seems to come back. Gets worse after exercise and showers. Read somewhere that it should get flaky and dry out the creams were stopping this should I stop using. Doctor: Lukewarm bath, garlic consumption, sitting in sun, calamine lotion application, Oatmeal bath, tea tree oil + almond oil may be applied over rashes and avoid synthetic cloths. These will help you naturally to get rid of pityriasis rosea"
},
{
"id": 183233,
"tgt": "How can front teeth positions be corrected?",
"src": "Patient: hello, I am an 18 yr old male. I got my braces off 2 years back. I wore my retainers for about 10 months and then i discontinued wearing them. Now i feel that my front teeth are shifting forward from the root! What is the best way to correct them. I dont want braces again. Thanks Doctor: Hello Read your query as you have undergone orthodontic treatment now you feel of forwadly shifting of tooth dont worry you have to wear retainer for year as it was advisved by your orthodontist , I will suggest you to consult your orthodontist again and go for oral examination and discuss about retainer . Hope this will help you."
},
{
"id": 25051,
"tgt": "What causes severe palpitations?",
"src": "Patient: I have been having moments lately where my heart feels like it's trying to jump out of my chest all of a sudden that come and go. Today I noticed that after wards I couldn't laugh because it would happen again. When I was younger I had a heart murmur. Is this it coming back? Doctor: Thanks for your question on Healthcare Magic. I can understand your concern. Yes, worsening office heart murmur can cause similar kind of palpitations. Actually heart murmur can cause arrhythmia (rhythm disturbances in heart). Some tachyarrhythmia like supra ventricular tachycardia (SVT), atrial fibrillation, atrial flutter etc can cause pounding of heart. So get done ecg, 2d echo and Holter monitoring (24 hours continuous recording of ecg). On the basis of these reports, you will need anti arrhythmia drug. Don't worry, you will be alright with appropriate treatment.. Hope I have solved your query. I will be happy to help you further. Wish you good health. Thanks."
},
{
"id": 32845,
"tgt": "Suggest treatment for sinus infection and blurred vision",
"src": "Patient: Why is it that the day after I take my last antibiotic for sinus infection, my sinus infection is right back and my right eye vision is real blurry, so blurry that I can t see very well out of that eye. This sinus problem in now going on for four years. I ve seen an ENT with no real answers. Doctor: Hi, Thanks for posting in HCM.Sinus infection affecting the eye leading to blurred vision could be due to oedematous reaction of eyes. Since you have already taken antibiotics to treat infection, kindly continue to take nasal decongestant tablet or syrup till the symptoms completely subsides. You need to do steam inhalation thrice a day so that all the mucous could be drained out completely. If you are having this problem repeatedly, it might be due to deviated nasal septum, which would need surgical correction. Wish you good health."
},
{
"id": 19968,
"tgt": "What does indigestion hypertension with chills suggest?",
"src": "Patient: My husband has been experiencing a feeling of indigestion(uneasy stomach) and then gets chills really bad. He is 63 yrs old 6 4 (285 lbs)with hypertension(controlled), overweight, IBS, borderline diabetic, takes lisinopril. avodart and Vitamin D. These symptoms don t come at any given time, such as after eating or activity. They have occurred even while he is in bed resting or awakens him from sleeping, but there is no rhythm or reason surrounding the symptoms. What do you think this is related to? Doctor: Hello!Welcome and thank you for asking on HCM!His symptoms could be relate to different possible causes (coronary artery disease, cardiac arrhythmia, thyroid dysfunction, electrolyte imbalance, IBS, etc.). For this reason, I would recommend consulting with his attending physician for a physical exam and some tests to investigate for the possible causes: - a chest X ray study and pulmonary function tests- a resting ECG and cardiac ultrasound- complete blood count, PCR, ESR for inflammation- thyroid hormone levels - blood electrolytes- an ambulatory 24-48 hours ECG monitoring would help investigate for possible cardiac arrhythmia. An exercise cardiac stress test would help investigate for possible coronary artery disease, especially when considering his cardio-vascular risk factor (hypertension, diabetes, overweight, age). You should discuss with your doctor on the above issues. Hope you will find this answer helpful!Wishing good health, Dr. Iliri"
},
{
"id": 77008,
"tgt": "Does hormonal imbalance cause chest pain in left side?",
"src": "Patient: Hi, I have a squeezing chest pain in my upper left side, feels like indigestion but in the wrong place. I have recently had a surge of estrogen imbalance I presume, my breast have swelled and are very tender, my periods have come every other week for a few weeks now. I am not pregnant. I have chronic IBS.I am 40 yrs old and very health concious. Not over weight, eat all organic foods, excercise regularily. But this chest discomfort worries me, It is intermittant and has no rhyme or reason. Happens when I am sitting at the kitchen table, driving, bending over to pick something up. Not excercise induced. What can this be casued by? Is their a link between hormonal imbalance and chest pain?Thanks for your time. Doctor: Thanks for your question on Healthcare Magic. I can understand your concern. Yes, hormonal imbalance and such chest discomfort can be connected. Actually, hormonal imbalance can cause stress, anxiety and mood swings. These stress and anxiety can cause chest pressure, tightness, pain etc. So first of all treat hormonal imbalance. So consult gynecologist and start hormone replacement therapy (HRT). Also consult psychiatrist and get done counselling sessions. You may also need anxiolytic drugs too. Don't worry, with proper treatment, your chest symptoms will also improve. Hope I have solved your query. I will be happy to help you further. Wish you good health. Thanks."
},
{
"id": 61060,
"tgt": "What does a painful lump on the breast indicate?",
"src": "Patient: I had a diagnostic mammogram and ultrasound last week. Calcification was diagnoses on first mammogram. Right breast. She said 99% cancer. Didn't do a biopsy immediately though and scheduled that in week and a half. I am 51 and no breast cancer in family. How concerned should I be? Large \"knot\" in breast and it hurts. Should I be panicked? Doctor: Hello Dear Warm welcome to HealthcareMagic.com I have evaluated your query in details * There are many reasons for the same as - inflammatory conditions - benign tumors - malignancy . * Should not panic till the biopsy results come . Hope this clears your doubt Wishing you fine recovery Welcome for any further assistance . Regards Take Care Dr. Bhagyesh ( MS , FMAS - consultant surgeon )"
},
{
"id": 114812,
"tgt": "Suggest remedy to raise my HDL level",
"src": "Patient: I am a 46 year old AA female that is 5 8 and weight 170 lbs. I also have sickle cell disease that was diagnosed at birth. I have my lab results:CHHD 2.7 50 L LDLC 39 100 TRIG 117 mg/dL 150 VLDL 23 mg/dL 5-40 How do I raise my HDL, or do I need to? Doctor: Hi, dearI have gone through your question. I can understand your concern. Low level of HDL cholesterol is the individual risk factor for coronary heart disease. You should take low fat diet with high amount of polyunsaturated fatty acids. Drugs like atorvastatin is useful. But it is prescription based medicine so consult your doctor and take treatment accordingly. Hope I have answered your question, if you have doubt then I will be happy to answer. Thanks for using health care magic. Wish you a very good health."
},
{
"id": 109212,
"tgt": "Suggest treatment for back pain",
"src": "Patient: Hi Doctor, Yesterday I had a sudden back pain started when playing table tennis, i could not walk or lift my left leg , there severe pain . I managed to comeback home with someone s help but i have severe back pain ( lower left side of spine) and cannot walk or move around in the home, it s very painful to wake up from bed or chair without help . I have taken combiflam 400mg - 2 tab per day but it s give very less temporary help. Please suggest if there any other suitable medicine i should take atlease to make me move or any other suggestion. Pls also advice for long tern cure. Regards, Ramanuj Doctor: Hi,from history it seems that there might be heavy back muscle strain while playing table tennis leading to spasm of back muscle giving this problem.Take analgesic with muscle relaxant like Dan-MR twice a day for 3 days.Apply analgesic, muscle relaxant cream over the part.Apply hot fomentation.Take rest for 2-3 days.Ok and take care."
},
{
"id": 184925,
"tgt": "What to do for gum disease?",
"src": "Patient: i have gum disease started about 2 weeks ago my gums are bleeding really bad i seen my doctor while im waiting to see my dentist in 3 days my doctor gave me amoxil and said not to go to the dentist yet cause he will do is give me antiboics anyway is tis true or should i go to dentist too thank tou Doctor: Hello!Thank you for posting here.Gum bleeding suggests bad oral hygiene.You did not mention when you had your last scaling session.Get a professional scaling done with your dentist.Bleeding gums will not reduce with amoxicillin.It requires scaling and tetracycline antibiotic course.You must see your dentist regarding this.Take doxy-1 once daily for three days.Avoid milk with this medicine.Brush gently over the gums.Use warm saline gargle thrice daily.Scaling must be done every 6 months.Gargle after every meal or snack.Use floss daily.Hope this helps."
},
{
"id": 159293,
"tgt": "Child has loose stools. Passed MRI, Xray and blood work. Is it cancer related?",
"src": "Patient: My 5yo son has been having very loose stools every time he poops. He was in to see his pediatrician last Thursday & again this Monday. Xray revealed foreign object 2mm in size in his stomach last Thursday. On Monday, they did a follw up xray & saw that it passed. Bloodwork done & came back normal, one level indicating he s getting over a virus. Doctor has him off dairy, he s drinking lactaid for about a week to see if that helps w/ his belly aches. But he still has a white Puss, & yellow, loose stool every time he goes. If his bloodwork & xray are normal, this rules out anything cancer related? What should I keep watch for & report to doctor? Should I get sample? Doctor: Hi, why are you worried about cancer it is not clear to me. in pediatric age group common cancers are hematological and in some case neurological. your symptoms not indicating any of this. frequent loose stool very common in childhood most commonly due to rota virus. if not controlled then total biochemical and stool exam to be done to rule out any enzyme deficiency or some other benign cause. consult your pediatrician."
},
{
"id": 69695,
"tgt": "What could be the recurring itchy lumps on the crown of my head?",
"src": "Patient: HI, I am 35 years old and just under 6ft tall and I am 13.5 stone, I have some lumps on my head (crown) that come and go, they get ichy and if I push on them they bleed watery blood and then bright red blood and but will still bleed for about 2 to 3 hours after i have done this, Any advice would be helpful, Cheers. Doctor: Hi.Thanks for your query and an elucidate history.This can be some sort of dermatitis, it bleeds as the scalp has very good blood supply. Consult a Dematologist for a proper diagnosis in such cases can be made by visual inspection .The treatment will definitely depend upon a correct diagnosis."
},
{
"id": 201224,
"tgt": "What is the treatment for varicocele problem in the scrotum?",
"src": "Patient: doctor two and half yrs before i got married but upto still we dont have child because im have varicocele peoblem in my left side of scrotum. last month i have taken semen analyze but only 10 million count was present. I consulted many doctors they suggested many drugs but there was no use.Now im consulting siddha doctor.please suggest me how to cure this varicocele problem. Doctor: Hi.I had gone through your query.Do not panic and relax.Varicocele is associated with male infertility and it is treatable condition.No medicine can help.Only one treatment exist and that is surgical correction.Only indication for surgery is infertility.10 million count still enough to be a father. If you tried and not able to do then go for surgeon opinion.There are reports that sperm count will increase after surgery.Pain in scrotum can be treated with analgesic and wear tight undergarment or scrotal bike to reduce discomfort.I hope i have answered your query.Thank you."
},
{
"id": 123394,
"tgt": "What is the cure for Parkinson s disease?",
"src": "Patient: My cousin, aged 39, is suffering from Parkinsons disease, for the past six years. This has restricted movement and control of his right hand and legs. Who would be the best doctor in India? He has become very depressive and also suffers pain in the limbs, neck and head. Is there a cure? What should we do? Doctor: Hello, Was your cousin's, diagnosed from Parkinson's disease by a neurologist. What are the symptoms - because rigidity and tremors are the two significant symptoms which are notable in Parkinson's disease. For having a diagnosis we need the help of the clinical examination under a neurologist and also some brain scan, blood test and urine test. Post which if he is diagnosed with Parkinson's disease then oral dopamine supplement will be given and the dosage will be monitored as per the clinical findings. Exercise will help to improve the motor control, improve strength, the flexibility of the joints etc. Exercise along with dopamine supplement has a good response in patients with Parkinson's disease. Hope I have answered your query. Let me know if I can assist you further. Take care Regards, Jay Indravadan Patel, Physical Therapist or Physiotherapist"
},
{
"id": 149854,
"tgt": "Dizziness, confused speech, off balance movements, put on Welbutrin. CT, EEG normal, lefts die weaker than right. Any ideas ?",
"src": "Patient: I have been having trouble for about 7 months. It started out that I would get confused a little very rarely. I thought that it was just dehydration . Then about a month later I started getting dizzy and confused a few times a month. Then all of a sudden it hit hard. I could not make sense of things. When I spoke I was saying the wrong words. Finally my mind seemed to give me trouble constantly. My speech was confused, my movements were off balance, I would forget things that had either just happened or I d forget where I was, and when people would talk to me I had trouble understanding what was being said to me. I have had a ct scan which came back fine, an EEG that came back fine, and a balance test that showed that my left side was weaker than my right. I ve lso lost little bit of my hearing . The neruologist that I ve been seeing has put me on Welbutrin and just said to keep coming back every three months. But I keeps getting worse. Doctor: You must have a repeat consultation with your neurologist and possibly get an MRI scan even if your CT is normal. This is especially because you have hearing loss and weakness in one side of the body. Your symptoms suggest a possible temporal lobe epilepsy or CP angle pathology which requires investigation. If the MRI is normal, ruling out cerebello-pontine (CP) angle pathology, I would even suggest a video-EEG telemetry which, if lucky, could record one such episode of confusion. Sometimes complex partial seizures can be missed on routine EEG if the source of seizure in deep in the brain or in the temporal lobe. Video-EEG may be more useful."
},
{
"id": 94384,
"tgt": "Have abdominal pain with dark yellow urine. Had gallbladder surgery. Treatment?",
"src": "Patient: I had gallbladder surgery 2 weeks ago and i ve continued to have severe abdominal pain , both times ending up in the ER. They take my blood and things seem fine, give me IV pain med and send me home. My surgeon has not returned my phone calls and I see him this Tuesday. I m completely frusterated and want to get this pain managed and NOT take a 4th trip to the ER. The last couple of days i ve noticed my urine is very dark yellow. I ve been drinking water by the gallon as i m also breastfeeding my 8 week old. Why isn t anyone concerned? I m so tired of the pain, it s so depressing. Doctor: Hi Jenperna. Pain associated with dark urine can be serious complication. It can be caused by bile duct obstruction which isnt very rare after such procedures. Your blood tests should show raised bilirubin level if i am right, but I think that repeated lab and ultrasound is at least what needs to be done. Second cause can be abscessus formation or intraabdominal colelction. Especially if you have fever and feel fatigue. Just go to ER if you feel that it is neccessary. If i were your doctor I would do CT scan and complete lab tests to reule this out. Wish you good health."
},
{
"id": 31737,
"tgt": "What causes severe pain, nausea and vomiting?",
"src": "Patient: Hi, i'm a 21 year old college student and i was diagnosed with Chron's when I was bout 11. I was recently hospitalized for about a week due to severe left side pain, nausea, and vomiting. My GI doctor perscribed me prednisone but i haven't seen any change. I can barely keep any foods down and my stomach is very swollen to where i can't button my pants. Its hard to be in college and to constantly throw up all day. For some reason, the only thing that tends to stay down is crackers, but i cant see myself surviving off crackers for the rest of my life. Any advice? Please help Doctor: HI, thanks for using healthcare magicThe prednisone was to try to reduce the inflammation in the bowel by suppressing the immune response but if it is not helping there are other immune suppressant medications that can be used.You should go to your doctor for a review so he or she can change to medication to another that may be able to help you.There are over the counter agents that may help with the swelling (if linked to bloating) and the vomiting eg dramamine, pepcidI hope this helps"
},
{
"id": 10881,
"tgt": "Suggest treatment for hair loss due to androgenic alopecia",
"src": "Patient: My Age is 28My height is 5 feets 10 inchesMy weight is 77 KgsI am having Androgenic Alopecia and my hair have gone thin, would like to the expert opinion regarding the treatement of this Androgenic Alopecia and would like to know is there any treatment for this?I have consulted all Alopathic, Homeopathic doctors but didn't received any positive response.I would like to know if there is any treatement other than hair transplantation for this Doctor: Hello, welcomeHere i explain everything about A.A (Androgenic Alopecia) point wise1. Androgenic alopecia or Male pattern baldness is due testosteron hormone which cause hair loss and baldness of scalp except back of scalp2. There few treatment options which can delay the the process but final and ultimate cure is hair transplant only.3. Being a hair transplant surgeon i always recommend to go hair transplant rather than spending the money on medicine and other things.4. As you said you want to know other treatment plans so i mention belowA) Use minoxidile 5-10% over bald or hair thinning area once in nightB) Also take some multivitamine tablets once in a dayC) Finasteride or dutasteride are the tablets which you can take once but it need proper guidance as higher and long duration doses of these tablets may affect sexual desire ( which is reversible after stopping the tablets)d) PRP ( platelet rich plasma) this is the small procedure in which serum is taken out from blood by centrifusing it and inject in to the scalp or dermaroller.Although as i said all above treatments can delay the process but cant give 100 % results but hair transplant is full proof method to get good and proper results."
},
{
"id": 199294,
"tgt": "What causes burning sensation on the tip of penis and frequent urination?",
"src": "Patient: Burning sensation on the tip of my penis and frequent urination1.tab flagentyl(secnidazole) 4 at once.2.tab sparfloxacin 200 mg twice daily for 10days.3.cap fluconazole 50mg twice daily for 10days.I have taken the above drugs but I still feels a burning sensation in my penis Doctor: HelloI appreciate yourconcernLooking at your description I would advise you followingGet few tests done like Urine culture and sensitivity test and blood sugar fasting and postprandial This can be related to passage of concentrated urine so have plenty of fluids dailyI would like further information.to help you better likeYour age and relevant medical history like diabetes or kidney disease If it doesn't improve with all these measures please consult a urologist for further examination and investigations like Ultrasound Maintain good hygieneHope this answers your questionWish you good healthregards"
},
{
"id": 66463,
"tgt": "What is the medium sized,inflamed lump on the buttock?",
"src": "Patient: I have a medium sized bump on the lower part of my buttcheek. Almost near the crease in my upper thigh and cheek. It is very inflamed but not pus filled. It lies right on my underwear line and is painful. When I feel it, it does not feel like there is anything in it, just swollen. Doctor: Hi, thanks for sharing your health concerns with HCM! If I were your treating Doctor for this case of painful lump on lower buttock, I would come up with three possibilities, these include: 1.\u00a0\u00a0\u00a0\u00a0\u00a0skin or hair follicle infection due to some friction injury 2.\u00a0\u00a0\u00a0\u00a0\u00a0The second possibility is of a dermatofibroma or scab \u00a0\u00a0\u00a0\u00a0\u00a03.\u00a0\u00a0\u00a0\u00a0\u00a0The last possibility is of some infected sebaceous cyst or parasitic cyst! \u00a0\u00a0\u00a0\u00a0\u00a0Overall, it is inflammatory and not worry about this!I suggest you to go for ice/cold compress in the region with local hygiene and take some topical antibiotics and wait for 2/3 day; if not relieved please don't forget to write to us again...or go for an FNAC test for confirmation...Hope this answers your question. If you have additional questions or follow up questions then please do not hesitate in writing to us. I will be happy to answer your questions. Wishing you good health."
},
{
"id": 150944,
"tgt": "CT scan for paralysis assumption shows lacunar infarct, symmetrical white matter hypodensities suggestive of age related ischemic changes. Suggestions?",
"src": "Patient: Age of patent: 85 Symptomes: right cheeck and right hand stop sensing for 4 days. paralysis assumption done. CT Scan done. CT Scan done and report is given below:Hypodense area seen in left thalamus (HU 16) suggestive of suacute/old lacunar infarct involving the left middle cerebral artery territory small lacunar infarct in the right lentiform nucleus and left insular cortex. Ni basal ganglia calcification foci. Symmetrical white matter hypodensities seen in bilateral centrum semi ovule and periventricular area suggestive of age related athroscierotic sub cortical white matter ischemic changes. Prominant cisterna magna. Deviated nasal septum to right with septal spur. Mucosal thickening in bilateral ethmidal sinuses. Doctor: Hello, The lacunar infarct in the the left thalamus explains the loss of sensation in right cheek & right hand. Thalamus is a organ through which sensations are relayed to the higher centres in the brain. Loss of sensation is a common symptom in infarct in thalamus. The treatment involves prevention of future attacks by giving antiplatelet drugs like aspirin. Also risk factors have to be treated like hypertension, diabetes mellitus and high cholesterol. The other changes reported on CT scan do not need any specific treatment. Good luck."
},
{
"id": 202542,
"tgt": "What causes erection problem after using Lexapro?",
"src": "Patient: Hi I have been taking lexapro for about 2 years and I have been experiencing problems with sex drive, erections and reaching ejaculation. I have been advised to switch to duloxetine as it may help with these problems. Am I less likely to have these problems with this antidepressant, Wayne Doctor: HelloThanks for your query,based on the facts that you have posted it appears that you have been taking Lexapro for your mental illness and now facing problems related to sexual activities .Lexapro is antidepressant drug that elongs to SSR Inhibitors group of drugs .It is known to have side effects like decreased sex drive, impotence, or difficulty having an orgasm; .Duloxetine also belongs to same group of drug hence unlikely to resolve your problems Please consult your treating Psychiatrist to explore possibility of changing for altrnate drug.Dr.Patil."
},
{
"id": 170289,
"tgt": "Suggest treatment for indentation in head after skull fracture in a child",
"src": "Patient: My 12 month old son suffered a skull fracture when he was just a 11 days old. He went to a neuorologist had all his follow up visits. I was told he was fine and I should have nothing to further worry about. Yesterday, I noticed an long indentation on the side where the fracture was. I do not recall feeling this before. He doesn t act like he s in pain or anything but now Im worried something is wrong. Can you help? Doctor: Hi, since your child has fracture skull at 11 days of life and you have neurological visits at that time this doesn't seems alarming. It may be an old impression of the fracture. If the child has achieved all the milestones normally and there is no history of recent trauma, it will not be a matter of concern. However, an MRI should be done to see if there is an intracranial abnormality and an visit to neurologist should be done so that we don't miss any findings. I hope this will help you. Take care."
},
{
"id": 161856,
"tgt": "What causes frequent sneezing,nasal drainage and nausea in a child?",
"src": "Patient: 10 yr old daughter s symptoms are sneezing that clears her sinus cavities several times a day even though she has no congestion, drainage, burping, feeling of food sitting in throat or esophagus, nausea, hardness in stomach. Cat scan of sinuses showed very slight inflammation; upper endoscopy w/ biopsies & ph probe were all clear; scratch test for food & environment all clear. Allergy meds, Nexium, Prilosec failed to resolve symptoms. What might cause these symptoms? Doctor: Hi, By what you say I feel that your child might be having moderate to severe persistent allergic rhinitis along with an upper airway resistance syndrome. Previously the only treatment for upper airway resistance syndrome used to be surgery or adenotonsillectomy. But now advanced the modalities of treatment have come up, like intranasal steroids which can be given up to a period of 4 to 8 weeks and after which many kids are recovering without the need for surgery. This treatment will take care of the moderate to severe persistent allergic rhinitis also. But this technique of administration of intranasal steroids is very important and should be talking to you by your pediatrician properly. Hope I have answered your query. Let me know if I can assist you further. Regards, Dr. Sumanth Amperayani, Pediatrician, Pulmonology"
},
{
"id": 148189,
"tgt": "What could cause mouth twisting while talking?",
"src": "Patient: Last night my boyfriend came home and my mother was talking to him and asked him what was wrong with his mouth. She asked him how long has you mouth been twisyed when you talked and he told her two days. I didn't noticed cause we barely see each other by working different times of the day. Everytime he talks its like he is talking out one side of his mouth can you tell me have he had a mini stroke or what please help Doctor: Hi,Thank you for posting your query.Based on the description of his symptoms, your boyfriend seems to be suffering from facial paralysis, also called as Bell's palsy.In this condition, one half of the face gets paralysed, without affecting the arm or the leg.There may be difficulty in closing the eye on the affected side, difficulty in chewing on the affected side, and the face deviates to the normal side on talking or smiling.The diagnosis can be confirmed by doing a neurological examination and facial nerve conductions.Treatment consists of physiotherapy, acyclovir and steroids.I hope my answer helps. Please get back if you have any follow up queries or if you require any additional information.Wishing you good health,Dr Sudhir Kumar MD (Internal Medicine), DM (Neurology)Senior Consultant NeurologistApollo Hospitals, Hyderabad, IndiaClick on this link to ask me a DIRECT QUERY: http://bit.ly/Dr-Sudhir-kumarMy BLOG: http://bestneurodoctor.blogspot.in"
},
{
"id": 94098,
"tgt": "Headaches, strange sensation in abdomen. What is wrong?",
"src": "Patient: Hi, I ve been feeling different for the past month or more. I just finished my period, but this month it was way different. It only lasted 3 days and it usually lasts 5-7 and it was light which I m heavy. I ve been having headaches for the past week too. & now I m feeling this sensation in my lower abdomen I can t even explain$ I just feel down and weird. As I m speaking to u, I feel hot and I have this headache n my lower abdomen just above my pubic bone feels weird. I m tired too, but I m fighting it. Lol Doctor: Hi welcome to Health care magic forum. Thanks for choosing H.C.M.F. You have just finished 3 days period instead of 5 days heavy period. You had head ache weakness, hot flushes, and supra pubic pain. This is all suggestive of pelvic inflamatory disease,or just a urinary tract infection. I advise you to consult a gynaecologist for diagnosis and treatment. You may have to undergo M.R.I. besides other Other routine tests for confermation. Wishing for a quick recovery. Best regards."
},
{
"id": 170770,
"tgt": "Suggest medication for diarrhea",
"src": "Patient: My six month old son has had diarrhoe for about four days . No vomiting but seems to have cramps and seems very uncomfortable and cross . First i thought it was teething but now i think it is a tummy bug. Nappies are wet . I have stopped his solid food and am just giving him formula . Is this ok? Doctor: Hi...you need not stop solid food and give only formula. This seems to be a viral diarrhoea. Once this starts it will be there for 4 to 5 days. Most importantly you should prevent dehydration. Unless the kid is having blood in the motion or green or bilious vomiting are extremely lethargic and having low urine output you need not worry. Regarding diet - only thing which you need to avoid is fruit juices as this might cause osmotic diarrhoea.Regards - Dr. Sumanth"
},
{
"id": 215204,
"tgt": "What to do for soreness in the back from the lung area and soreness in lymph nodes?",
"src": "Patient: I was diagnosed with a flu like virus and went into upper resp infection. They gave me z pak and one week worth of protozone and then another week s worth. I still have flem, trying to get my energy back and now I notice that I have I guess lymph nodes very sore all down my right side and left side of my trunk. It is also sore on my back from the lung area down. What should I do? Doctor: Hello, Cannot say in your particular case, but generally, there is one type of uncommon pneumonia in which the Zpak is 100% effective--mycoplasma. Otherwise it fails about a third of the time maybe slightly more. With phlegm, fever, chills, etc. Those are quite serious signs and starting over with another DIFFERENT treatment would generally be done. Pain in the lung is a sign of pneumonia That would be bad, too. Hope I have answered your query. Let me know if I can assist you further. Take care Regards, Dr Matt Wachsman, Addiction Medicine Specialist"
},
{
"id": 138996,
"tgt": "What is the treatment for pain in the hip joints?",
"src": "Patient: Gud morning sir, i an from jammu. I am suffering from pain in both hip joints l am unable to sleep in either of the position doctor advised me to take Tizen 2 mg and Nurokind lc I od. Pl.suggest me is it right treatment and how long it will take to cover the issue. My age is 52. Doctor: HiWelcome to healthcaremagicI have gone through your query and understand your concern. You are likely to be having bilateral osteoarthritic of hip joints. It will be confirmed by x ray of both hips AP view. Glucosamine is helpful in arthritis. You can take analgesic such as ibuprofen for pain relief. You can take Mecobalamine to reduce tingling sensation s. You can discuss with your doctor about it. Hope your query get answered. If you have any clarification then don't hesitate to write to us. I will be happy to help you.Wishing you a good health.Take care."
},
{
"id": 49350,
"tgt": "Can Morphine and dye can cause more damage to the kidneys, having a allergy towards the dye and could this cause to go to the ER?",
"src": "Patient: My aunt had went into the hospital with chest pains and was diagnosed with kidney failure. Was put on dialysis and had other test ran with dye. Would this cause her to go in the ICU. With kidney failure I thought that the morphine and dye would hurt the kidneys more. And she had also had a allergic reaction to the dye. She's 73 years old. Doctor: Hi,I had read your query and according to me this is the most common age for acute renal failure or shut down of kidney due to any allergic reaction and dye is a most common agent for hypersensitivity reaction. Morphine does not damage the kidneys."
},
{
"id": 157726,
"tgt": "Sore pea size lump above left kidney. History of bladder cancer. Cystoscope came clear",
"src": "Patient: Hi I have a small pea size lump in my side above left kidney area, it's generating a soreness type pain and aches. I have had it for 5 wks and when first appeared I didn't feel lump my first thoughts were I had probably pulled a muscle, but now I am a little worried, I have also had bladder cancer and had treatment in left tube to kidney, last check up by cystoscope was January and clear as it has been for the last 6 years. Doctor: Hi and welcome to HCM,thank you for your query.if you can feel it under the skin then this is not originating from kidney probably and this would be only benign subcutaneous lesion. It can be surgically removed under local anasthesia and then sent to hystologic analysis. Also you can do ultrasound and needle biopsy to evaluate this lump.In every case i dont think this is related to your malignant previous disease.Wish you good health. Regards."
},
{
"id": 217957,
"tgt": "What causes sharp pain uptil fingertips in arm after lifting weights?",
"src": "Patient: pain in left armI'm a 23 year old healthy man, I worked out 3 days ago including lifting weights. A day after I noticed my left arm hurting and unable to put my arm straight out. when I try to straighten my arm a sharp pain goes all the way to my finger tips?? any idea what I did and if it will go away soon? Doctor: this type of pain usually occours due cervical spodylosis. You have probqbly caused imbalance of the neck muscles while working out. You will have to take medicine, apply analgesic ointment to the neck and give hot fomentation to the neck. It may take any thing between 3 days to 3 weeks"
},
{
"id": 101480,
"tgt": "Suggest treatment for asthma",
"src": "Patient: my mother 77 years old is a chronic asthmatic patient. Almost every month she gets asthma attacks in which she is coughing almost non stop.Every time she is given antibiotics.This time she again had the attack of coughthe doctor has given her antibiotic and defcort 24 for three days bd. I want to know is that the right way to keep her asthma attacks in control?She daily takes doulin nebulizer.and other puffs but still gets cough attacks .She has no vision due to Glucoma hence life is very difficult.Mere going for a bath makes her breathless. Doctor: Hello dear,Asthma is caused due to broncho-constriction (obstruction of smaller airway passages) which is indicative of Hyper-responsiveness of air passages.Management consists of:1. Asthalin+ Seroflo inhaler- provide symptomatic relief by causing broncho-dilation (dilating the smaller airway passages, relieving the obstruction & increasing airflow to lungs)So, it can be used whenever there is an acute attack.2. Montelukast preparations- used as a maintenance therapy to relieve symptoms of asthma.3. Antihistamines like Cetrizine & Anti tussive can also be used to provide symptomatic relief.4. Need for anti biotics arise if there is a secondary infection. But this should be taken only under the guidance of your Physician.5. Steam inhalation & deep breathing exercises.6. Maintain adequate hydration & a healthy balance diet.7. Protection from exposure to cold, dust or other allergens.If symptoms still persist, kindly consult a Pulmonologist for complete clinical examination including Pulmonary Function Tests.Wishing her a good health.Take care."
},
{
"id": 4602,
"tgt": "Will Siphene tablets aid conception?",
"src": "Patient: hi i have to ask about my conciving last month i hv take fertomid of 50 mg and yet i hvnt concive and this time doctor told me to take siphene of 100mg .from third day of my periods...can you plz tell me about this ....will i concive this month or not ???? Doctor: Hi,Thank you for choosing Healthcaremagic. Siphene contains CLOMIPHENE , which aids in ovulation i.e formation of eggs. But for getting pregnant you need to have NORMAL PATENT FALLOPIAN TUBES , NORMAL OVULATION, NORMAL SEMEN ANALYSIS AND NORMAL SEXUAL LIFE.Please ask if you have more questions. If you are satisfied, please make sure that your ACCEPT my answer so that I receive credit. Good luck!!"
},
{
"id": 123075,
"tgt": "What cause loss of control on feet after sitting on toilet?",
"src": "Patient: my wife sits on the toilet for 15-to 20 minutes. When she get up she loses control of her legs to the point she cannot stand to her feet. I have to lift her, hold her up until she can stabd on her own. What can be the cause of this and how to prevent it? Doctor: Hello, As she is sitting for a long time there is a numbness due to compression of the nerve. If she continues to sit for 15-20 minutes using a small stool and keeping the feet on it will release the pressure and she won't feel the loss of control. Also, she can do on a regular basis simple lower limb strengthening exercises. Hope I have answered your query. Let me know if I can assist you further. Regards, Jay Indravadan Patel, Physical Therapist or Physiotherapist"
},
{
"id": 163862,
"tgt": "Suggest medication for fever,vomiting and diarrhea",
"src": "Patient: Hi Doctor, My kid has a slight fever accompanied with vomitting and diarrhoea for the last 2 days. Is there any special diet/medication that I can give him to stop this condition. Please help. Right now he has been having light diet and fluids. Thanks, Anitha Doctor: Hi...Thank you for consulting in Health Care magic.It seems your kid is having viral diarrhoea. Once it starts it will take 5-7 days to completely get better. Unless the kid's having low urine output or very dull or excessively sleepy or blood in motion or green bilious vomiting...you need not worry.There is no need to use antibiotics unless there is blood in the motion.I suggest you use zinc supplements (Z&D drops 1ml once daily for 14 days) & ORS (Each small packet mixed in 200ml of potable water and keep giving sip by sip) as hydration is very important and crucial part of treatment.Regards - Dr. Sumanth"
},
{
"id": 93611,
"tgt": "Severe pain in right side of the body along with nausea,fatigue and appetiteloss.Could it be appendicitis?",
"src": "Patient: I have been having right side pain for 2 months now along with nausea, fatigue and appetite loss. The pains range from sharp and stabbing, tearing, burning and stinging, mild to severe, My temperature has gone all the way down to 94 degrees and up to a fever. Some days the pains are not so bad but constant and other days the pain is so severe and I can't stand up straight and can hardly move. The pain gets worse with deep breathing, a lot of movement, being hungry, eating, some days I do nothing unusual and the pain gets super severe. I have had an upper abdominal Ultra Sound, An upper UGI with barium and they both came out normal. The doctor said those were to test my gallbladder but they did not find gallstones. They have not mentioned appendicitis. I am wondering what is causing the pain. Thanks Doctor: Hello, Thanks for the query to H.C.M. Forum. This is a very peculiar history. Nausea , fatigue, loss of appetite are due to PAIN. Now the nature of pain . Pain threshold is different in different persons so at present keep it a side. As temperature goes down i.e. 94 F i.e. HYPOTHERMIA. Hypothermia obscures most of the symptoms and signs of INFECTION, notably fever and leukocytosis. Excessive ,extensive cultures and repeated physical examinations are essential. As I mentioned above Infection may obscures so we have too consider for abscess in Liver in initial phase, tuberculosis of intestine, Kidney functions ( MAIN EMPHASIS over this). Once again get in U/S whole of abdomen at least from 3 ( three ) different centers and than consult a doctor who will match these reports and adjudge what is the disease. Once again I would advice you please get in Blood for E S R ( C B C). Good luck Dr. HET"
},
{
"id": 211726,
"tgt": "Taking Maxgalin, Naxito, Zolfresh. Developed head reeling, sweating, uneasiness about train, plane journey. Suggestions?",
"src": "Patient: Respected Doctor,I am a psychiatry patent of National Institute of Mental Health and Neuro Sciences, BANGALORE and i take Maxgalin 150mg twice daily, Naxito 10mg twice daily and Zolfresh 10mg in night since last 2009 but from last few days i feel my fear factor is devloped and head reeling sweating and feel very uneasy , when i thought about train or plane journey uneasyness is too much. Doctor: HelloYou are currently on Pregabalin, Escitalopram and Zolpidem since 2009. For last few days you have developed sweating, headache, uneasiness and restlessness. The problem increases on thinking about train or plane journey. Most likely the problem is due to anxiety. Non specific fear can develop in anxiety. I would advise you to visit your psychiatrist again for any increase in doses of medicines if required. For quick relief of symptoms you can try relaxation methods like Breathing Exercises or Progressive Muscle Relaxation Exercises or yoga. These methods will help to increase your confidence.Thanks, hope this helps you."
},
{
"id": 169199,
"tgt": "What is the cause and treatment for vaginal infection in a child",
"src": "Patient: Hello: My two daughter came home today without any indication of an infection in her vaginal area. But this evening she started complaining hurt. I asked her to show me where and she touched her vagina. I bathed her and checked her out before and after. She appears to have a yeast infection (which I did not notice earlier this evening) and a fishy order. She has never had a yeast infection before so I m not sure what s going on. She is potty training, please help. Thank you in advance. Doctor: Hi,Having fishy odor with pain might be due to some irritation produced by itching on the part.This can be due to local unhealthy hygiene.Clean local part with water and apply antibiotic cream.Make a habit of cleaning local part with running water after passing bowel and urination.Do not allow her to play in clay or dusty play ground.Ok and take care."
},
{
"id": 29458,
"tgt": "What causes pneumonia?",
"src": "Patient: Yes my name is melinda, I m in the hospital with pneumonia. They have done cultures for fungal but so far it is neg. Before I was admitted I was giving 6 Rocephin shots , two steroids shots and po Meds also. A round of levaquin 750mg , z pack X 2 . Did chest X-rays x 3 first X-ray it was in bottom left lobe then move to mid lobe then full blown consolation in lung acrossing in right lung. Then was admitted and started on zosyn. It has helped a lot but they say it not bacteria and checking for fungal . So far neg. If it not viral, bacteria, or infection, what could made the pneumonia? I did have bronchitis before that wasn t clear up before it supposely turn into pneumonia. I never had any fever at any time. Sputum looks like club soda. I m just trying to upstanding this . Can you suggest anything? Thank you Doctor: Hello,There are many things that can cause pneumonia, viruses, bacteria, funguses are the most common. If all testing for bacteria and funguses are negative it may be viral. Tuberculosis should also be ruled out if no other more common cause is found.I hope you are better soon,Regards"
},
{
"id": 140590,
"tgt": "What causes numbness in middle finger during early morning hours?",
"src": "Patient: I have a problem of numbness in my right middle finger in the early morning hours. My Doctor prescribed Neuro kind gold capsules one at bed time for 10 days.I do not have B.P or sugar.What is the function of the above Vitamin? please let me know.what could be the cause of this problem? Doctor: Hello, Well, since the recent news of how poorly efficacious most multivitamin/multi-mineral preparations do their jobs has become publicized it grounds my feelings that I've had for many years which is anything of a MULTI-anything is likely, not worth the capsule covering it comes in. And Nurokin Gold is no exception in my opinion. If your right middle finger is numb early morning only then, I wouldn't necessarily be suspicious of a nutritional deficiency so much as I might be concerned about a VASCULAR deficiency or a compression phenomenon going on at least over part of the MEDIAN NERVE that sub-serves sensory function of that finger. I would also recommend measuring serum levels of whichever B vitamin your doctor is apparently concerned about that would cause such an ISOLATED presentation of 1 single middle finger on 1 hand (not a likely presence of a B12, B6, folate, or niacin deficiency in my opinion). Then, based on those lab results I would prescribe the direct and single entity to the patient as necessary. Otherwise, I might consider doing vascular Doppler studies and possibly an EMG/NCV study and maybe even an Ultrasound of the carpal tunnel which could avoid having to stick needles into the arm/hand. Hope I have answered your query. Let me know if I can assist you further. Take care Regards, Dr Dariush Saghafi, Neurologist"
},
{
"id": 210058,
"tgt": "Why is my mother unable to walk?",
"src": "Patient: my mom is 59 years old she has been having mild signs of altzeihmers like repeating herself and forgetting newly given information .....yesterday i took her to the hospital because she couldn't walk and her legs kept giving out on her they gave her and ekg and said she seemed fine which she did because she began to walk again... however the same thing happened again this morning any suggestions on what is causing this problem Doctor: Hello,Thanks for choosing health care magic for posting your query.I have gone through your question in detail and I can understand what you are going through.Alzheimers cannot lead to this kind of symptom although presence of alzheimers as an independent diagnosis cannot be negated as she has significant forgetfullness. Her forgetfullness needs to be evaluated in detail. Regarding her inability to walk, there could be a problem in the brain which could be leading to such a problem of having inability to walk. She should go ahead with an MRI of the brain. Hope I am able to answer your concerns.If you have any further query, I would be glad to help you.In future if you wish to contact me directly, you can use the below mentioned link:bit.ly/dr-srikanth-reddy\u00a0\u00a0\u00a0\u00a0\u00a0\u00a0\u00a0\u00a0\u00a0\u00a0\u00a0\u00a0\u00a0\u00a0\u00a0\u00a0\u00a0\u00a0\u00a0\u00a0\u00a0\u00a0\u00a0\u00a0\u00a0\u00a0\u00a0\u00a0\u00a0\u00a0\u00a0\u00a0\u00a0\u00a0\u00a0\u00a0\u00a0\u00a0\u00a0\u00a0"
},
{
"id": 21972,
"tgt": "Does stress cause rise in blood pressure level?",
"src": "Patient: Hi Doc, I had gone for preemployment check up and was under severe stress that what would happen in the same. My pressure recorded that day was 140/100 and now i am worried waht happens next, Can you help me in this regards as what to do and is it so bad. I am 42 years ol. Doctor: don't worry 140/80 is perfect for your age....keep a check on it.....don't make it exceed 140....do regular exercise and low salt diet"
},
{
"id": 220812,
"tgt": "What are the early signs and symptoms of pregnancy?",
"src": "Patient: i think i might be pregnant, i had my period 3 weeks ago, and have had sex recently... for the past week i have had increased amount of discharge, more frequent urination, a feeling of bubbles in my lower abdomen, slight back ache at times, and have also gained weight. Does it sound like i am pregnant? Doctor: Hello dear, understand your concern.In my opinion the symptoms of pregnancy doesn't start this early.The first sign of pregnancy ismissed period.Other signs are nausea, vomiting,increased urination,sore breasts start 2 weeks after missed period.A urine pregnancy test confirms the pregnancy.It needs to be done a week after missed period for accurate results.It is too early to feel any pregnancy symptoms.So relax. Avoid stress.Nothing to worry.3 weeks after periods comes under safe period in case of regular cycles.The weight gain also doesn't occur this early due to pregnancy.The stress regarding the thought of pregnancy also makes one to feel pregnancy symptoms.So avoid stress and check for the period.Best regards..."
},
{
"id": 98519,
"tgt": "Are itching and sneezing after touching hair symptoms of an allergy?",
"src": "Patient: I think I m allergic to human hair and pet hair. Every time i bathe dogs or cats my arms and legs itch (i get in the tub with them for easier access). I also cut my boyfriend s hair every now and then and where the cut hair touches me i start to itch. I don t know if I sneeze or get stuffy because we have 2 cats and im allergic to cats definitely. Would i be allergic to hair in general or human hair or pet hair? Doctor: hi sir/madam,Thanks for your question on Healthcare Magic.these allergic reactions can be easily treated in Ayurveda.Allergy is a hyperactive response of the immune system to particular substances called allergens. These allergens may be food, pollen, dust, drugs, molds, air-borne, dog, cat, mushroom, soy, gluten, etc. Sometimes, it may be for short times or continue for longer periods.before going for treatment we should focus on to prevention of this problem.The main causes of allergy are:-1. Infectious diseases at childhood2 .Pollution3 .Food allergies to peanuts, pecans, pistachios, pine nuts, and walnuts etc.4. Latex sensitivity5. Hereditary6 .Hives7. Eczema8. Redness9. Swelling 10 Abdominal crampsYou can go for these test for your allergies:-1. Upper endoscopy.2. Skin Test for allergy.3. Blood test for allergies-RAST and ELISA.4. Food allergy testing.Doing yoga will also help you in this issue,the yoga's like:-1.Matsyasana:- It is a good aasana for those people who suffer from asthma, bronchitis and other lungs ailment thereby relieves congestion.2. Sarvangasana:- It helps in clearing of exchange of gases through the nasal passage.3. Tadasana:- It improves concentration of mind and develop sense of balance.4. Padhastasana:- It improves concentration and helps to decongest the nose.5. Trikonasana:- It systematically revitalizes the entire body.6. Padmasana:- It induces mental calmness and helps to bring about physical health and mental equilibrium on permanent basis.7. Bhujangasana:- It expands the chest more than usual, helping to improve one\u2019s breathing.8. Paschitmoasana:- It tones all the abdominal organs, including the liver, pancreas, spleen, kidney and adrenal glands thus helps to mental calmness.9. Pavan Muktasana:- It relieves from abdominal ailments associated with malfunctioning of inefficient organs.You can see these yoga aasan in the internet for better understandings.Simple home remedies for allergy which you can do yourself at home itself:-1. Warm water gargling with salt helps to treat it.2. For inflamed eyes, use the mixture of lime+ mallow + chamomile tea.3. Quince + Honey + Lemon show a good impact on the throat.4. Eye irritation may be reduced washing it through cold water.5.Keep yourself away from allergens.6. Hot shower is effective to ward off allergen and may be used as skin.7. Use sunglasses to protect your eyes from pollen, mites, dust etc.8. Dust mites favour humid place that may spur the condition.9. Hay, straw and grass burning may make allergic patient vulnerable to the disease.10. For congested and inflammatory nose, use peppermint tea.11. Paste of sandalwood + lime juice, a best bet for the affected area.diet advice:- 1. Eat at least 2 hours before going to bed.2. Drinking warm water or herbal teas throughout the day is beneficial.3. Heavy foods including dairy products, fried foods, cold foods, rice, beans and legumes should be avoided.4. The use of spices, pickles and chilies in cooking should also be minimized Honey, nuts and seeds can be taken in moderate amount.5. Avoid excessive or physically demanding exercise.6.Bitter substance should be taken regularly.Ayurvedic medicines are having best results in allergic reactions take any of this will give good results:-1. Mahasudarshan Ghan Vati :- 1-0-1 AFTER FOOD.2. Panchnimba Churna:- 1 - 2 teaspoon with water twice a day.3. Raktashodhak Syrup:- 2 Spoon daily before bed.4. Haridrakhandam:- 10-15 grams twice daily with lukewarm milk.Children: \u00bd of the adult quantity.Hope this info was helpful.stay healthy,stay safe."
},
{
"id": 57432,
"tgt": "Gall bladder polyps detected in ultrasound. On weight watchers diet, lost and gained weight, retaining water. Relation to gall bladder?",
"src": "Patient: I went to ER june 3rd with abominal pain Ultrasound showed 3mm gallbladder polyp . I was told to leave it alone to keep an eye on it , Now I have been doing Weight Watchers Since Feb and lost 7lbs Thus to the 3rd of June and now have gained a 1lb and retaining alot of water without changing my diet habits following WW. Is there any relation to gallbladder? Doctor: hi there is no relation between your symptoms and gallbladder problems wishing you all the best dr.Klerida"
},
{
"id": 202389,
"tgt": "What causes irritation and red bumps on penis head?",
"src": "Patient: Hi how are you doing? Well my boyfriend of almost 5 years has had a weird irritation on his penis for a couple of months now. His pee hole is really irritated and red, and about an inch below the head he has a big red bump that is very noticeble...what could it be? Doctor: HIThank for asking to HCMI really appreciate your concern looking to history given here, this could be because of the some irritating substance in the urine and for that his urine should be sent for the test and the test should be routine and Microscopic, if this is normal then no need to worry about this, for the symptomatic relief \"Povidone can be tied\", have a nice day."
},
{
"id": 60519,
"tgt": "Laser treatment of gall bladder stone",
"src": "Patient: is there any laser treatment of gall bladder stone without removing gall bladder present in noida/ncr Doctor: Hello. Welcome to heathcaremagic. The laser treatment for gall stones is present in Fortis hospital, noida. A doctor would have to look at condition of your gall bladder to ascertain wether it can be carried on you or a laproscopic removal of gall bladder is required. Wish you good health."
},
{
"id": 119522,
"tgt": "What causes pain in back of thighs, buttocks and upper arms?",
"src": "Patient: This morning I woke up with pain in the back part of my thighs and buttocks as well as pain in the back of my upper arms. I just turned 70 yesterday and was marveling that I have no aches and pains. And today I received a reminder. Any idea what would cause this? Doctor: Hello, I appreciate your positiveness and your work to maintain your body.At this stage this pain in absence of any other disease can be due to strain due to work or due to improper rest.If you find any of these factors then please rectify those.If such pain continue to persist then i shall advice you to have a complete routine check up. Take care. Hope I have answered your question. Let me know if I can assist you further. Regards, Dr. Mukesh Tiwari, Orthopedic Surgeon"
},
{
"id": 110908,
"tgt": "What causes severe back and neck pain?",
"src": "Patient: hi.am 24yrs old girl.I am married for 3 years now. I have been suffering with severe back problem since 2009.Its been gradually increasing day-by-day. I have consulted many doctors, but no one has told me what exactly is the problem.And am also suffering with severe neck pain. these are some of my test results :Sacralization of L5 with narrowing of L5-S1 disc space and exit foramina (taken in feb 2010)-scoliosis with concavity to left side due to muscle spasm.(feb 2010)-discal bulge at L4-L5. (feb 2010)My bone scan result (taken in november 2011)Physiological distribution of traces noted in the axial and appendicular skeleton.FDG uptake noted in the iliac side of both SI joints, left more than right with sclerosis noted on corresponding CT sections.Focal uptake noted in the petrous temporal bone lateral to mastoid aircells - s/o infection. Please tell me what is the problem? my neck problem is also in a very bad condition.i cant be in one position for more than 1/2 n hour.And my back problem is worse before and during periods. We haven't planned for a baby yet.And will there be any complications during pregnancy because of my back problem. please reply. thank you. Doctor: Hello I have studied your case As per your symptoms there is possibility of sacro ileitis which is common after pregnancy.I will advise to check your B 12 and Liver function test, CBC,ESR,CRP via blood.As you are losing your weight there is possibility of malignancy.Till time continue your medication.Postural yogic exercises may help you You can contact directly on my profile for further assistance.http://doctor.healthcaremagic.com/doctors/dr-vaibhav-gandhi/63503Hope this answers your query. If you have additional questions or follow up queries then please do not hesitate in writing to us. I will be happy to answer your queries. Wishing you good health.Take care"
},
{
"id": 209615,
"tgt": "Suggest treatment for hallucinations",
"src": "Patient: Hallo Good morning, I am Ramesh and I want to ask about my 80yrs old mother. She is feeling that one emginary family is shouting and demanding to vacant the house and trying to beet also. She is observing this since last six month.Kindly suggest what to do. Doctor: Hi,I can understand your concern regarding the symptoms of your mother. The symptoms that you have mentioned are suggestive of a psychotic spectrum which at this age occur usually due to some underlying neurological condition, like dementia, etc.I would suggest you seek a psychiatric consultation. Some blood investigations like complete blood counts, renal function, liver function, serum electrolytes and brain scanning would be required to determine any underlying condition. Detailed evaluation of her cognitive functions like memory will be required. Accordingly, her primary condition will have to be managed. For example, medicines like donepizil for dementia. In addition, she will need antipsychotics like quetiapine which will help in controlling the behavioral symptoms.Hope this information was helpful. Best wishes."
},
{
"id": 169527,
"tgt": "What causes lump below rib cage?",
"src": "Patient: Hi. My 3 yr old has a small lump below her right rib cage. Her Dr. examined it & said it seems like a lump of fat. He can move it around & it seems that it is not attached to anything. He asked me to monitor it & alert him it it gets bigger. I m still uneasy about it. Is this common in children? Should I be worried that it is a tumor? Doctor: it's unlikely to be tumour as it is mobile .looks like lipoma means fat collection. need to observe if its decreasing or increasing number in size. it can be removed by minor procedure and sent for pathology dept for examination."
},
{
"id": 110851,
"tgt": "Suggest treatment for back pain after injury",
"src": "Patient: My husband fell off the roof and he hurt his left ankle and his lower back. His back is severe pain, is this ok or should he go to the hospital. He does not want to go. His ankle is sprained bad but not sure if broken. It s his back that is more painful, he landed on his foot then down on his butt. Doctor: if i were you i would go to hospital. there can be minor fracture which can only be ruled out after xray for details examination. even if there is sprain in ankle he must need immediate medical attention. dont delay in trial and error procedure which can lead to further complication. just to rule out problem visit your GP or hospital ASAP"
},
{
"id": 218639,
"tgt": "Is a 2.4cm cervix length during 33 weeks of pregnancy a concern?",
"src": "Patient: I was 33 weeks 4 days wen my scan was done n found my cervix only 8mm. Got betnesol injections for lung growth. I have gdm also but under control with 500mg sr glycomet. On bedrest since 2 mnths wen my cervix was 2.4cm. Is it possible to maintain cervix length for another one month. Taking susten 200 n progesterone injections weekly. Does dilation happen when cervix becomes zero ? Doctor: Hello,Have fully trust in your treating doctor. He has suggested right treatment for you. Please, do continue with the treatment.Secondly, I will suggest you to take bed rest in head low position. Please use blocks or bricks beneath the bed towards your leg end. This will help to create gentle slope and resting in this position will help to reduce pressure on the cervix.You can seek prescription support from your treating Gynecologist by judging your clinical status. One of the following medications to relax the uterus can be taken:1. Tablet Isoxsuprine 10 mg 3 times a day2. Tablet Nicardia Retard 10/20 mg 2 times a day3. Tablet Terbutaline 2 times a day.Bed rest and uterus relaxing agents should be taken till completing 36 weeks of pregnancy. It is safe for the baby to deliver after 36 and preferably after 38 weeks of pregnancy. Injection Betnesol 12 mg can be repeated weekly till 36 weeks.Hope I have answered your query. Let me know if I can assist you further.Regards,Dr. Purushottam Neurgaonkar"
},
{
"id": 6162,
"tgt": "Trying to conceive, delayed ovulation. Taken choriomon 5000 shot. Will taking duphaston tablets enable pregnancy?",
"src": "Patient: hello , I m trying to get pregnant and my doctor said i ovulate late , on day 20 . she also gave me a choriomon 5000 shot . i just want to make sure what is it exactly. and she said after 2 day i should start taking Dupbaston 2 tablets twice a day for 14 days and then i should be pregnant . do u think his will work ? Doctor: hi, choriomon 5000 is injection of beta HCG which will help in release of ovum, duphaston is progesterone which will help in implantation and also in maintenance of pregnancy, so as your doctor recommended you can use them, you have to plan intercourse during the time of ovulation which is important in increasing the chance of pregnancy. take care."
},
{
"id": 141177,
"tgt": "What causes tingling sensation in the left hand?",
"src": "Patient: I have strong tingling sensation inmy left hand in lower right quadrant of the palm. Not sure what this could be, ulnar nerve compression. Have a spot of bone where I had hair-line fracture on the back of my hand back in 1969 in the center, not sure it this could be contributor. Doctor: Hello!Welcome on Ask a Doctor service!I understand your concern and would explain that your symptoms could be related to a pinched nerve. The region is not very typical of the ulnar nerve. Anyway, I would recommend performing a nerve conduction study and a hand X ray study. Hope to have been helpful!Kind regards, Dr. Iliri"
},
{
"id": 56276,
"tgt": "Is is safe to take Mycophenolate 500mg for nephrotic syndrome?",
"src": "Patient: I have nephrotic syndrome, and my doctor has me taking mycophenolate 500mg 2 times daily, and I want to know if it is alright to use products like sunflower lecithin and relishi liver guard herb tea. Or will either cause a reaction because of my disease and medications. Doctor: Hi, I had gone through your question and understand your concerns.Mycophenolic acid is an immunosuppressant drug used to prevent rejection in organ transplantation. It inhibits an enzyme needed for the growth of T cells and B cellsMycophenolate mofetil is metabolised in the liver to the active moiety mycophenolic acid. It reversibly inhibits inosine monophosphate dehydrogenase, the enzyme that controls the rate of synthesis of guanine monophosphate in the de novo pathway of purine synthesis used in the proliferation of B and T lymphocytes.It is definitely used for nephrotic syndrome.AVoid the sunflower lecithin .Herb tea can be taken safely.Hope this answers your question. If you have additional questions or follow up questions then please do not hesitate in writing to us. I will be happy to answer your questions. Wishing you good health."
},
{
"id": 100364,
"tgt": "What causes runny nose, congestion, headache and cough?",
"src": "Patient: My husband and I just returned from France and have contracted a cold virus - runny nose, congestion, cough, headache, ear ache, and frequent urination, sometimes uncontrolled - for both of us. Could a virus give us these symptoms? We were not on a farm but in the city in France. Doctor: Hi, thanks for using healthcare magicThe common cold is a viral illness. It would cause the runny nose, nasal congestion, cough,headache and ear pain ( though ear pain may sometimes occur with bacterial infection as well).Frequent urination can occur with urinary tract infection which is a bacterial infection that would require antibiotic therapy.A urine test at your doctor's office would determine if a urinary tract infection is present.I hope this helps"
},
{
"id": 196671,
"tgt": "How to cure burning sensation during urination?",
"src": "Patient: hi, im 19 this year.. i just started my gym not even a month.. the last i went gyming was on the monday and i did bench press ( 25 lbs ea arm) and other few workouts .. i felt nothing wrong with it after lifting but in the midnight of the day, when i actually woke up to go pee i felt my penis had burning sensation after pee. and the next day my left groin area started to feel a weird feeling which prolonged until today which is already the third day for having such problem.. i tried to press my testicles and i dint feel any pain, what would be the problem? that kind of feeling is really making me down.. could it be serious? thanks for the advise!! Doctor: Hi,Burning sensation in urine may occur after the workout due to passing of concentrated urine owing to loss of lots of water in sweating during work out. So you need not to worry, have lots of fluids before, during and after the workout to maintain good hydration. If it doesn't subside with this than I would advise you for urine culture to rule out urine infection.In my opinion it's not a serious problem and you should take lots of fluids.Hope this answers your question. Please feel free to ask for follow up question and I will gladly answer you.Best wishes,Dr. B R Hudda"
},
{
"id": 140208,
"tgt": "Should I continue the medicines Zoloft and Bupropion for ADD?",
"src": "Patient: I am 27 years ord with ADD and limit depression at times. I am currently taking 150 mg bubropion/voxra and 100 mg setralin/zoloft. I feel sometimes tired and dosed, sometimes it feels great and some not. Should I just consult with my doctor to take one of those medications or do I possible need both of those? Doctor: Hello, I think that your description of how you feel with your medication is very good to carry with you to your next appointment with your doctor. I always like to encourage patients to speak frankly with their physicians about medicines they've been prescribed and to tell the doctor without fear or hesitation if something seems to be making a difference or not. He/she is the best one to determine your need for different doses, dosing schedules, or the inclusion of adjunct medications. It is especially important to realize that some of these medications used for depression and ADD can have dramatic effects on both the body and the mind if they are suddenly stopped, changed, or combined with other drugs unknowingly. Never stop or change a prescription you've been given without checking in with the prescriber. In this case, I highly recommend keeping track of how you feel in a daily diary and to also jot down if you took your medications or not that day (hopefully, you take your medications EVERY DAY!) AND very important. What time you took them AND also very important, did you take them with food, on an empty stomach, or in the company of other medications? This information will help your doctor quickly assess the need to make changes. Hope I have answered your query. Let me know if I can assist you further. Take care Regards, Dr Dariush Saghafi, Neurologist"
},
{
"id": 87649,
"tgt": "How to cure adhesion attached to the bowel causing abdominal pain?",
"src": "Patient: GOOD MORNING, I HAVE A 27 YEAR OLD DAUGHTER THAT SUFFERS FROM CHRONIC LOWER RIGHT ABDOMINAL PAIN, SHE HAD A LAPAROSCOPY SEVERAL YEARS AGO AND FOUND THAT ADHESIONS HAD ATTACHED THE BOWEL AND APPENDICTS TOGETHER, SHE HAD HER APPENDIX REMOVED BUT THE PAIN PERSISTED. SHE HAS HAD 29 ULTRA-SOUNDS ALL CLEAR BUT THEY FOUND ALOT OF ADHESIONS. SHE IS ON ENDONE FOR PAIN RELIEF, BUT CANNOT GET ANY ANSWERS AS TO WHY THIS CONDITION IS GETTING WORSE . SHE HAS A CYST ON EACH OVARY BUT THIS WAS ONLY RECENTLY FOUND. THE PAIN CAN COME ON AT ANYTIME AND TRAVELS DOWN HER LEG CAUSES NUMBNESS. SHE ALSO HAS A SMALL HERNIA. SHE HAS BEEN TO SO MANY DOCTORS AND THEY SAY THEY CAN T HELP HER, SHE IS DEPRESSED. I DO NOT KNOW WHERE TO GO FROM HERE, CAN YOU HELP? KIND REGARDS Doctor: Hi. Thanks for your query for your daughter and an elucidate history.Read the whole history about surgery, adhesions and now recently cysts on ovary.As you have explained that the pain can come anytime and travels down the leg, a suspicion about the involvement of the Prolapsed Interval Disc arises and can be diagnosed only by the MRI of the Spine. So go for the MRI of the spine-CT scan of the abdomen-Enteroclysis for the possible finding of the intestinal obstruction. The adhesion has a natural tendency to develop as a natures way of protecting the intra-abdominal organs , but in this case it over-reacts to cause a problem. If a specific problem is found on any of the investigations , your daughter has to go for the surgery for the said cause. In the meantime take Doctors opinion and treatment to improve her nutritional status, proteins and hemoglobin as deficiency of any of these give bad surgical results."
},
{
"id": 918,
"tgt": "Suggest treatment for infertility with irregular periods",
"src": "Patient: Dear sir ,iam indira ,iam regularly watch your programme .verry verry use ful and learn lot of information.Igot marriage 2009 june but till now i dont have child.my age is 32 weight is 74.5kg hight is 152 cm i have irregular perioud poroblem i took treatment past one and half year. .Regular Period problem last 10 year i take tablet Regestron 5mg tablet 3days after 3 or 4days i get period. i try to conceive but still now get this same problem....... my husband seman test its normal, scan reports all of normal.... but why this problem come to with me i confused. please ,Past three months i take sidda medicine. within three months i spend more than 40,000 theay identify problem through pullse but no improvement now iam in full diet and regular walking .i take only fruits and vegetables.in my friend more than me but he took treatment only six months now she is had baby.Kindly i need your valuable opinnion thankiung you sir. Doctor: Hi, I think you should take some medicines like clomiphene for the growth of your follicles and track your follicles growth by repeated ultrasound and when your follicles is more than 17 to 18 mm, take injection for rupturing the follicles. Be in contact with your husband for 2 to 3 days after injection. Take progesterone for next 2 weeks. Do a urine pregnancy test at home after that. You can try like that for 3 to 6 months. If it doesn't work then you can go for IUI. Discuss with your doctor regarding this. Hope I have answered your question. Regards Dr khushboo"
},
{
"id": 20855,
"tgt": "Suggest treatment for heart disease",
"src": "Patient: hello, my name is Trent and i am a 34-year-old, slightly overweight (around 50lbs over BMI scale for my height) male police officer. for the last 2 years, ive been dealing with ventricular bigeminy/trigeminy as well as extremely frequent PVC s, tachycardia and the occassionally supraventricular tachycardia. at times the chest pains have been so bad i have passed out and hit the floor. i have not been able to work since january 2010. i have been hospitalized for these symptoms 6 times in the last year and half. i have been through 2 cardiac ablations and have been tried on metoprolol, soltalol, amiodarone, dronedarone, and 3 other antiarrythmics that i cannot remember the names of. neither ablation worked, and the second one caused more damage then good. prior to the second ablation (march 2011), my ejection fraction was 45% and there was mild diffuse hypokinesia in the left ventricle. after the second, ejection fraction dropped to 38% and there was moderate global hypokinesia of the left ventricular segments. structural disease was ruled out in 2010 by angiography and echocardiograms as well as a cardiac MRI. so far, none of the 5 cardiologists i have seen have been able to isolate the cause of all of my symptoms. my biological father had a pacemaker implanted at age 35 after he suffered an MI. i believe he was diagnosed with sick sinus syndrome. i have inquired into the possibilty of having SSS myself, but again, no one has confirmed or disproved this. since the failure of the ablations as well as the failure of antiarrythmics, the cardiologists that i have seen have told me there are no further treatment optins other then a 3rd ablation. having been through 2 already, i am definetly not eager to go for a 3rd. am i candidate for a pacemaker or ICD type device? and what is causing all of this? i shoud mention that cardiomyopathy was originally suspected, but again, not ruled out or confirmed. i was drinking alcohol on a regular basis up until november, and have not touched a drop since. any advice you could give me would greatly appreciated! i am extremely eager to have my condition treated and get back to work!! thank you for your time, trent a c wright. Doctor: hello thanks for posting hereYou seem to have refractory arrythmias which despite medical and repeated ablations are not regressing. I think you must go ahead with a CRT -D implantation. I will resynchronize your heart contractions, and will correct your rhythm when required. It also has a back up pacemaker activity which will pace the heart of rhythm is too low. It so give you a small shock when the heart rate becomes irregular and life threatening events can be avoided. You will only feel a small jerk during the episode. Sametime a notification will go to your cardiologist regarding the event so that he can get a tab on your health.Also it is a very safe procedure and will not lead to decrease in your ejection fraction.Regards."
},
{
"id": 145215,
"tgt": "Suggest treatment for weakness and tremors in legs",
"src": "Patient: I just woke up now, It s 7pm. It was a bit of an offsleep, but when i woke up i felt really weak, shaky, almost lighter. I m still shaking now. I tried just waking up a bit, Drinking some water and milk, Went to the bathroom. But i still fell weak shaky and light, Doctor: Hello ! I read your question and understand your concern. In my opinion the symptoms you have had may be related to a drop in your blood pressure. You should measure your blood pressure in sitting position and than in standing up position . A decrease more than 20 mm Hg in your blood pressure would be indicative of orthostatic hypotension. Low blood pressure can cause similar symptoms. I would also recommend a full blood work for anemia , a thyroid function test to rule out hypothyroidism or hyperthyroidism. Hope to have been helpful! Greetings!Dr. Abaz Quka"
},
{
"id": 46148,
"tgt": "What does the kidney and liver spots indicate?",
"src": "Patient: What do red and blue spots/markers/splotches indicate on a kidney and liver ultra sound? Is it an ominous sign? I am 35 years old female, 68kgs with a strong family history of bowel cancer with both my father and grandfather dying from the disease - my father had secondary cancer in his liver. Doctor: Dear patient,you are probably observing color Doppler ultrasound. Blue spots indicate a blood flow that goes away from the probe, while red dots indicates when blood is coming to the probe. This patterns need interpretation, and this is what radiologists does."
},
{
"id": 94008,
"tgt": "Abdominal Lpo done, periods over, resurfacing bleeding with blood clots. Should I be worried?",
"src": "Patient: Had abdominal lpo two weeks ago and had already had my period now it's back but different bleeding for three days blood is bright bright red and clots hat would fill a Dixie cup. About four a day of thoseShould I be worried or is this my body getting rid of things from surgeryHow much is too much?HelpEmail. YYYY@YYYY Doctor: Hi and welcome to HCM. If you want to be sure this isnt serious bleeding then the best way to evaluate your bleeding is to do hemoglobin levels. But also this is usual after such surgical procedures and you should be worried. Few weeks are necessary till your periods become balanced. If you notice bleeding beyond time of periods, lower abdominal sharp pain or fever you should visit doctor. If not, just be patient and everything will be fine. Wish you good health."
},
{
"id": 87418,
"tgt": "What causes severe recurring pain in the upper part of the body?",
"src": "Patient: I am getting severe pain in the middle of the night that runs down my right side and a bit towards my stomach and sometimes slightly into my mid/low back.it will last for a couple hours, then fade off.for a few days after that area is tender and has an achiness. It was happening a couple years ago and stopped.then the other night it happened. I have had every test around and they always come back normal.... I am so frustrated!!! Doctor: Hi,From history it seems that you might be having some problem like,1, chronic urinary tract infection,2, renal stones in right kidney giving renal colic,3, chronic appendicitis.Consult your doctor and get examined.After examination go for treatment accordingly.Take plenty of water.Ok and take care."
},
{
"id": 68325,
"tgt": "Suggest remedy for lumps in throat with difficulty in breathing",
"src": "Patient: Hi. I find it very hard to swallow, feels like a lump in my thoat. Very had to swallow water at this time. This has been going on for about 2 weeks now. I've been taking Prevacid 24hr for 12 days with little relief. Tonight it feels worse than ever. I'm concerned about my breathing. I also have been very sweaty. Any suggestions Please help. Thank You MG Doctor: Hi.This is all suggestive of an actual swelling in the throat. I would advise you to consult an ENT Surgeon to undergo urgent Endoscopy to find out the actual reason. It can be die to enlarged tonsils, a mass in the hypopharynx or anywhere in the pharyngeal region. If required a CT scan is also needed. Also go for :Blood- CBC, liver, kidney function, blood sugar.Throat swab from the infected area fir gram staining and culture and sensitivity.Blood culture helps a lot. The treatment is :Get admitted as you need intra-venous antibiotics and supportive medicinesIntra-venous fluids to tackle dehydration.Observation and needed procedures to be done."
},
{
"id": 32896,
"tgt": "Suggest treatment for scratch by puppy",
"src": "Patient: Hi,I have had a complete course of rabies vaccinations in my teen ages, now i have a puppy at home, and got scratched by her teeth while playing, i have already take a shot last week for the same reason that her teeth scratched me while playing, do i need to take another shot? Doctor: Welcome to HCM.Scratch by teeth nails by animals which bleeds needs immunisation by rabies vaccination.Since you have mentioned that it was not a stray dog i hope that it has been vaccinated.I would like to know the exact site where teeth had scratch you.If it is located near face or palm of hands would advice you to have a vaccination injection.Otherwise in other site last week vaccination is adequate.Thanks.regards"
},
{
"id": 54979,
"tgt": "What causes cirrhosis with loss of appetite?",
"src": "Patient: my sister has cirrohsis of the liver stage 4 shes swollen in her belly she has no appitite yellowing of the skin red and purple spots all over the body and limbs no muscel tone bone deficiency weighs about 75 pounds pain in belly and needs help walking and has no control of bowels how much longer has she got here with us Doctor: Hi thanks for asking question.Here condition of your sister is very critical.You have not mentioned whether she is taking any drug or not.Here she has ascites with erythema and diarrhea.For fluid accumulation and portal hypertension diuretic and beta blocker can be taken respectively.Meanwhile give her oil free diet.Fruits and green leafy salad given.Refined food not given.Grind carrot and spinach leaves and take this juice daily.Ultimate treatment for cirrhosis is liver transplantation.If here no care taken then survival for one year will also be difficult....Take care..Dr.Parth"
},
{
"id": 15176,
"tgt": "Rash on head for months. Taken antibiotics, antihistamines, no improvement. Suggestions?",
"src": "Patient: Hi, I have a rash(?) on my head with some pimples. This has been going on for several months. I am seeing a dermatologist who has tried antibiotics and various antihistamines. She has taken biopsies, and cultures. Nothing has shown up. In the meantime I am scratching most of the day and night and this has spread down my forehead and behind my ears. I think I should consult an allergist. Any suggestions? Doctor: Hi i think it is seborrhic dermatitis.rule out seborrhic capitis first then treat the condition with atarax 25 mg at night.antibiotic twice daily and flucanazole weekly once for 4weeks.and application of momate in the morning and keto b ointment can cure the condition"
},
{
"id": 112565,
"tgt": "Pain on back, bump in lower stomach, bruising, painful when touched. What do you think?",
"src": "Patient: i m a 16 year old female and i cannot remember when this even started happening but for a while now ive gotten these random pains where i cannot move at all, i cant stand i have to lay on my back for over 20 minutes each time for this pain to finally go away, i recently found lumps in my lower stomach left hip area and now there is bruising. the bruise is very painful when touched, i do not know what to think Doctor: Hello. Thanks for writing to us. The appearance of spontaneous bruises with multiple painful lumps is a worrisome finding. This could be related to autoimmune or a hematological disorder. Proper investigations are needed to detect the cause.I hope this information has been both informative and helpful for you. You can consult me again directly through my profile URL http://bit.ly/Dr-Praveen-Tayal Regards, Dr. Praveen Tayaldrtayal72@gmail.com"
},
{
"id": 55770,
"tgt": "Suggest treatment for liver cirrhosis",
"src": "Patient: My mother in law was told she had 30 days to live with cirrhosis. Two months later her amonia levels are normal and she has not had a paracentisis since then. Mentally she has declined- not wanting to take medication and thinks she has to pee all the time. Her communication has declined, she cries all the time and hits her nurses. Doctor: hi,Whatever symptoms you have described ,it looks like she is in encephalopathy ,for which she might be hospitalized for further management and check her blood parameters like renal function and electrolytes which are usually deranged and there are chances of getting infection too.If there is any abnormality better to correct to regain her conscious levels.And about pee ,that will be because of diuretics which your doctor might have prescribed.So starting with encephalopathy measures like adding duphalac syp 30 ml twice or thrice a day so that she should pass stools atleast 2 to 3 times per day if not consider to give her duphalac enema .Tab RIFAGUT 550 MG twice a day for atleast 15 days should be given as this is a intestinal antibiotic with no side effects.Avoid NSAIDS.Proper nutrition specially protein content is necessary as in cirrhosis there will be low protein.AVoid pantop injection as its not recommended in cirrhotics.Avoid oily band spicy foodHope this information helps youThank you"
},
{
"id": 129144,
"tgt": "What causes persistent burning sensation in the feet?",
"src": "Patient: Hi, at night time, when I finally sit down, the underside of my feet burn, like they are on fire. I have had these symptoms for over a year. I did see a doctor, can remember what he called it, but I remember he sad it was nerve damage where the tendons meet in the heel, he put me on anti inflaming pills, and told me to stand on steps and stretch excercise my tendons.This do not work, although it feels nice when I stretch my feet back, I do not believe this is going to cure the problem.I do have hard skin on my heel, and the skin peels off now and again, I am beginning to think there is a more underline serious problem. I am on my feet most of the day as I work in a diy shop. Doctor: Hello,Thank you for using healthcaremagic.I read your question and understood your concern.From the signs that you explain it looks like you have tarsal tunnel syndrome which is a nerve impingement in the ankle area.The second probable diagnosis is plantar fascitis which is chronic inflamation of the fascia in the heel.The treatment for this includes stretching exercises of the plantar fascia and Achilles tendon, use shoe inserts, night splints to sleep with and in severe cases cortisone injection in the heel and shock wafe therapyDr. Selmani"
},
{
"id": 60104,
"tgt": "Tremendous head pain with high BP post hiking. History of NASH with cirrhosis, portal hypertension. Taking quinapril, HCTZ, prilosec, advised increase in nadolol dosage. Suggestion?",
"src": "Patient: Have a history of NASH (dx 3 yrs ago--called fatty liver disease at age of 25, said nothing could be done about it then, Never drank alcohol until age of 53--wine only) with cirrhosis and portal hypertension . Meds are nadolol , quinapril, HCTZ, prilosec . Went hiking the other day and after climbing a hill, was winded (haven t hiked in 2 yrsdue to surgery on both feet and ankle), and started having tremendous pain in my head that lasted about 1 hr, but head sensitive for 24 hrs. BP was 160/93 and one hr later,140/88. Increased my nadolol from 20mg to40mg a day. Doctor: Hi there... Your headache most possibly can be because of raised blood pressure,and in your case Nadolol is good choice,in addition to that if you have cirrhosis you can add Aldactone 25mg once a day at morning time that is aldosterone anatgonist useful both in cirrhosis and hypertension. Regrads."
},
{
"id": 140437,
"tgt": "Suggest treatment for parkinson syndrome",
"src": "Patient: my dad is suffering from Parkinson syndrome. he had been taking t syndopa plus125 and now his physician has told him take syndopa CR 125.Is there any difference in the dosage. can i if there s any treatment to cure this disease. kindly help. he has one side shivering of hands and leg. stiffness, atomach upset frequently, lose of weight. Doctor: Hello, Unfortunately, there is no cure for PARKINSON'S DISEASE (not referred to as syndrome if it the classic form of PARKINSON'S DISEASE you are actually referring to). The CR version of the Syndopa is merely a designation that the medication is CONTROLLED RELEASE and therefore, theoretically it has to be dosed LESS FREQUENTLY. If your father's tremors are not being quieted with this medication then, you should take him to see a NEUROLOGIST or a PARKINSONOLOGIST for the most updated treatments available. There is also a SURGICAL approach called DEEP BRAIN STIMULATION. However, more knowledge on how well your father responds to regular doses of his syndopa are necessary. Hope I have answered your query. Let me know if I can assist you further. Take care Regards, Dr Dariush Saghafi, Neurologist"
},
{
"id": 208959,
"tgt": "Suggest tips to create a better understanding between parents",
"src": "Patient: Hello Doctor, my query is about my parents. They live in Dharwad. For no reason they create problems in home everyday, finding reasons to quarrel with eachother and with others in the family, no patience, no confidance or trust in anything or anyone, not happy with anything in life. This is affecting our personal life and career very badly. We 3 kids and some of close relatives spoke to them several times, but could not figure out the reason for their weird behavior. Could you please suggest what we can do to comeout of this situation? Doctor: DearWe understand your concernsI went through your details. I suggest you not to worry much. Definitely there are solutions. You should take your parents to an expert family psychologist. Not to a simple counselor. Psychologist should be able to know the intricacies of human behavior and relationship. If you require more of my help in this aspect, Please post a direct question to me in this website. Make sure that you include every minute details possible. I shall prescribe some psychotherapy techniques which should help you cure your condition further.Hope this answers your query. Available for further clarifications.Good luck."
},
{
"id": 54960,
"tgt": "Can gallbladder removal be the cause of stabbing pain while taking a deep breath?",
"src": "Patient: Had my gallbladder removed 28 August. When I take a deep breath I get a stabbing pain that interrupts this breath on my right side that radiates and feels worse at the front part of my right chest up and into the right side of my neck. Do I need to go to the er? Doctor: hi. noted history of cholecystectomy and abdominal pain, stabbing in character, over the operative site. post-operative pain can happen even for those who have been operated on years back. the most important thing to note is that it is of tolerable character, resolving spontaneously, with or without intake of analgesics. if the symptoms are persistent, it is best if you consult with a doctor, for physical examination and clinical evaluation. diagnostics and management will be directed accordingly.hope this helps.good day!! ~dr.kaye"
},
{
"id": 193954,
"tgt": "What causes hot flashes and lightheadedness?",
"src": "Patient: im a 21 year old male 6 foot 220 pounds, ive been having hot flashes i think an lightheaded for some reason an i cant figure out why. I dont have any health problems or anything ive always been an active person played spors since i was 5 an this started about a year ago do you have any idea why? Doctor: Hello, If you are sure that you do not have any health problems and if you are still feeling lightheadedness, it has to be due to anxiety. Do talk to your doctor. Hope I have answered your query. Let me know if I can assist you further. Regards, Dr. K. V. Anand, Psychologist"
},
{
"id": 92178,
"tgt": "What could be the reasons for pain in left side of stomach and sore underarms while trying to conceive?",
"src": "Patient: i have pain in the left side of my stomach and it goes partially into my back also on the left side. its like a stretching sensation. also my breasts underneith are sore. but i am trying to get pregnant, could this be early signs maybe or....? because i am not due on my period for another 12days. what could this be? Doctor: HI. This looks unrelated to pregnancy. Pain in abdomen can be due to UTI as or a ovarian cyst. Soreness under the breasts can have another reason. Early pregnancy do not cause pain unless there is spontaneous abortion. I would advise to go for a few blood tests , an ultrasonography examination of abdomen to see any organic lesions."
},
{
"id": 49688,
"tgt": "Pregnancy, have bulky placenta, valementous cord insertion for kidney stones. Anemic, nausea, back pain. Help?",
"src": "Patient: During my pregnancy I was told I have a bulky placenta, a valementous cord insertion as well as being hospitalized 3 times for kidney stones. I just found out I'm anemic as well. I have also been monitored due to a leep procedure from almost 2 years ago. I have alot of fatigue nausea and severe back pain. Should I be concerned? Just seems like so much been severely stressed and want some answers please help. Scared mom Doctor: Hello Scared Mom! you dont need to be scared. anemia is treatable easily. all you need is some good multi vitamins andfolic acids associated with a good balanced diet , green leafy vegetables and meat you gotta be a carnivore literally to get back all your iron you lost. so cheers, and no you should not be concerned. the moment your anemia gets treated your fatigue, palpitations, back pain will all vanish.use iron folic acid and multivitamins pack.get a jar pack for 6 months and share them with your kids even. and wait for the magic to happen.stress is part of being mom. try to have a routine. get a sound 8 hours daily sleep. dont over burden yourself. Trust me you will be fine I will be here for you if you need me.RegardsDr Shafi Ullah KhanMy Patient is my family"
},
{
"id": 59895,
"tgt": "Abdominal pain, chronic condition. CT scan shows nephrolithiasis with umbilical hernia, fatty liver. Treatment?",
"src": "Patient: since 6-7 years i had pain in my whole abdomen.i had many test but all reports are normal.no medicines cured me.yesturday i had a ct scan . impression are-non-obstructive left sided nephrolithiasis with a umbilical hernia with fatty infiltration of liver . non specific subcentimeter sized indeterminate mesennteric nodes noyed in mesentery Doctor: Hi, Thanks for writing in HCM. The results of Ct scanning explains the cause for symptoms. Each disease of individual organ system needs to be dealt with seperately to relieve you of symptoms. The priorities would be 1. Umbilical hernia, 2. fatty liver and 3. Nephrolithiasis assuming that there is no outflow obstruction. With regards"
},
{
"id": 172680,
"tgt": "Is my 3 month's old bowel movement normal?",
"src": "Patient: My three month old baby, who is solely breast feed did not have bowel movement for seven days. when she did have one it was more dark yellow in nature, and the consistancy of soft iced cream. It was also very fowl smelling.I know that her movements have been more watery and light yellow before with a smell of i would say buttered popcorn. Is this normal to have this king of movement with the consistancy and smell. Doctor: He dear, I understand your concernSuch stool and smell are signs of dysbiosis.I recommend :-stool culture and sensitivity to antibiotics;- Econorm 1/2 sachet twice a day for 5 days,then babylac-probiotics for 1 month.-Dufalac 5 ml can be used for stool stimulation.Hope it helps"
},
{
"id": 93735,
"tgt": "Have a pulse in stomach. Have stress and anxiety. No medication till now. What to do?",
"src": "Patient: hi Ive been having a pulse in my stomach for about 2 weeks doesn't really hurt just bugs me I don't really notice it during the day except when I sit and lay I am 18 I have stress and anxiety I am 5ft 5in male I weighed 125 a couple of months ago and stopped working out due to a concussion and gained about 20 pounds recently Doctor: Hello and welcome to Healthcare Magic. Thanks for your query. Your symptoms could be anxiety-related. Anxiety can manifest itself with not only psychological symptoms, but with physical symptoms also. If you are having recurrent such symptoms or having difficulty managing your stress, then I would advise you consult a psychiatrist for a detailed psychological assessment and further treatment. There are effective treatment options - in the form of medication or counselling / psychotherapy which will help you overcome your problems. Regular physical exercise, diet control and adopting a healthy lifestyle are also essential in your situation. Wish you all the best. Regards, Dr. Jonas Sundarakumar Consultant Psychiatrist"
},
{
"id": 225548,
"tgt": "Spotting, cramps after coming off microgynon pill. Taking vitamin tablets. Advice?",
"src": "Patient: Hi,I'm 20yrs old 11st 5ft7. I came off the microgynon pill about a month ago. I had a normal withdrawal bleed which lasted about 4 days. I have had the normal symptoms with coming of the pill. However since discontinuing it I have 2 episodes of spotting. For the past two days I have had abdominal cramps, like I would with a period. I started taking vitamins 3 days ago, but today when I went to the toilet my urine was neon yellow, and the same on the tissue. This has only happened once, but after that it's been like a brown bloody discharge. Can you help? Thanks. Doctor: Welcome to HCM.Yellow color urine usually occurs due to vitamin tablets.Irregular bleeding could be due to hormonal disturbances after withdrawal of microgynon pill.Take any nsaid with antispasmodic to subside cramps.Otherwise go for hormonal study,thyroid profile,s.b12, urine test,USG to rule out further diagnosis and proper medical management under guidance of gynecologist."
},
{
"id": 12253,
"tgt": "How to treat Psoriasis and Oncocyclus caused itching?",
"src": "Patient: I TAKE NEUROMEDICINE / PHYSICRATIC MEDICINE I HAVE PHOSORIASIS , ONCOYCYLIS SKIN PROBLEM . BEFORE LOWER PORTION OF KNEE USED TO SCRACH , NOW THAT IS CURED BUT NOW MY ITCHING IN MY HEARTH ( BREAST ) GOES ON SCRATHING . MAY BE DUE TO SEARCHING SOMETHING I REQUIRED . HOW CAN THIS SCRATHING BE STOPED ? I USE COALTAR / GENAMICIN D WITH LACTOCLAMINE TO GET RELIF . I ALSO GOT ALLERGY TEST AND TAKEN 2/3 YEARS EACH MONTH ALLERGY VACCINATION OF EACH FOR TEN MONTHS . Doctor: HIWell come to HCMI really appreciate your concern, if this is the functional problem you have then it may be the cause of itching because emotional stress is the reason for this try to come out of this, no need to apply thing but you can try Tab Loratadine 10 mg three times in day and try to come out of the stress hope this information helps."
},
{
"id": 42252,
"tgt": "What is the treatment for low sperm count and mobility?",
"src": "Patient: Hi, My name is Abdullahi Omar i am a kenya married for 1 year and 4 months with no kids. when i went for medical check up i was told i have low sperm count and sperm mobility problem which is almost 0%. and i was told the only chance i have is ICSI which is expensive and rare in Nairobi. I need your advise regards Doctor: Hi,I read your query and I understand your concerns.Following is my reply:1) If sperm count is good and motility is less you can try anti oxidant medicines.2) ICSI would be option if even sperm count is low.Let me know if you have anymore questions.Regards,Dr. Mahesh Koregol"
},
{
"id": 120735,
"tgt": "What causes tingling sensation in hands, lightheadedness and fever?",
"src": "Patient: Hello, About an hour ago i started getting a tingling sensation in both of my hands. A little bit later I started to get lightheaded and now i have a fever of 100.5. I also drank a lot last night but I have been eating and drinking a lot of water today. Doctor: Hello, Present symptoms are associate symptoms of fever. These occurs due to increase in blood flow in fever.You should not worry. You have acquired an infection which can be viral or bacterial. You may have paracetamol three times a day, drinking water is good in this period. If your other symptoms bothers you then ou may ask me for further help. Hope I have answered your query. Let me know if I can assist you further. Take care Regards, Dr. Mukesh Tiwari"
},
{
"id": 75411,
"tgt": "What causes shortness of breath and high BP?",
"src": "Patient: hi im 38 weeks pregnant i was discharged from hospital yesterday after bein admitted with high blood pressure and apogestric pains since bein discharged i have been getting sharp pains in my chest and shortness of breath i also have a sharp pain in my lower back and im losing a cloudy white substance from my vagina what could it be and is it serious Doctor: Hi , the symptoms that you describe can be due to a syndrome called cardiac failure when the heart does not function well and so fluid accumulates in the lung . I suggest you to report to emergency services for cardiology assessment ."
},
{
"id": 77038,
"tgt": "Could black mold lead to costochondritis?",
"src": "Patient: Hi, I was wondering if you could tell me if black mold could cause costochondritis? My fiancee and I are currently living in a house with black mold in the bathroom and other rooms in the house. We took her in to the ER twice in the past week and a half because of chest pain/tightness and numbness in her extremities. She's 22, ~180 lbs, and has a history of kidney stones/infections. The ER doctor diagnosed her with costochondritis but had no idea what caused it and it doesn't seem to be getting better even with Antiinflammatories (Relafin) and painkillers (only work for a SHORT time, IE <20 min, when she's limited to 2/day). Doctor: Hi I can understand your concern...Costochondritis caused mainly by trauma or physical strain....It may also be occur as part of osteoarthritis...Sometime viral or fungal infection of rib joint can lead it...So yes mold can affect it sometime....It is inflammation of costal cartilage....Apply hot pads over affected area.Take brufen for few days....Avoid heavy weight lift and strenous work.Proper sleep posture.Chest muscle stretching exercise useful..Finally if available TENS like treatment that is transcutaneus electrical nerve stimulation useful....Take care...You will gradually recover...Dr.Parth"
},
{
"id": 133192,
"tgt": "What cause muscles feel like constant charlie horse?",
"src": "Patient: Hi my name is Anita. I have left lower leg pitting edema, and the muscles feel like they are in a constant charlie horse. The only way to get relief is to lie down with leg up above my heart. I have/had cellulites in the le daysft lower leg. I have been on Keflex for 4 days then my doctor changed it to Doxycyline for 10 days. It is some better, but I don t understand why my left leg looks and feels like it is in a charlie horse. Doctor: HiYou need Doppler venous flow studies of the leg since this may be a deep venous thrombosis--a clot in a deep leg vein"
},
{
"id": 169971,
"tgt": "Suggest treatment for vomiting and watery stool in kid",
"src": "Patient: hi my daughter is almost 2 years old... she had a frequent vomitn', lose of apetite and has a yellowish fluid stool...and as for now she experienced influenza and very weak...I'm really worried doc.. i hate what the doctors advice for my daughter they said they can't determine the virus they can't even say a thing for me... I hope u could help me doc... does my daughter have amoeba??? how can this be treated??? thanks Doctor: Kindly give biff lac sachet half sachet 3 times a day and syrup ofloxacin 100mg per 5 ml at dose of 3 ml twice a day mam..avoid milk and milk products for a short duration mam..give idly, I'd iyappan and curd rice and plenty of ors fluids mam."
},
{
"id": 34834,
"tgt": "Can MRSA lead to other complications which are fatal?",
"src": "Patient: Hello, my name is Mary, Four years ago my fiance had a hernia repair, in and out the same day. The following day he was taken by ambulance back to hospital with MRSA He was put on antibiotic drip, with a compress on the wound, and putb on the ear nose and throat ward.On about the fourth day of visiting he was dilerious etc. I checked his infection, someone had put a marker round it , the infection was now round his back. I alerted a nurse Richard was not well to get someone to check him. Within 5mins 4 people appeared, they operated on the bed and repacked the wound. He lost over a stone in weight, and had stomach problems for the rest of his life. He felt ill went to see his doctor in Shirley on the 30th May, he was given paracetamol and a throat spray and told he had a kidney virus, he was found dead on his own on the 10th of June this year.Is it possible the MRSA contributed to his death. The inquest result was unaccertainable natural causes, enlarged speen was mentioned but not on the outcome????????thanks. YYYY@YYYY Doctor: Hi XXXXThanks for your query at HCM!I am Dr. Sheetal Verma an Infectious Disease Specialist answering to your query! I went through your query and understand your concern.Yes sometimes MRSA can cause complicated infections which can be fatal. But this will not be sudden. Patient will have infection for long not responding to antibiotics and develop sepsis. If patient died suddenly it could be other cardiac related problems.Hope I have answered your question. If you would like some more information I will be happy to provide. You can also write a review for me. Wish you good healthTake care! Wish you best of Health.Dr. Sheetal VermaInfectious Disease Specialist"
},
{
"id": 180687,
"tgt": "What are the symptoms of sublingual papilla disorder?",
"src": "Patient: Hi doctors .... i just noticed that the sublingual papillae taking the shape of thorn, very thin on the edges and a bit redish than normal look .... anything i should be worried about ....taking into consideration that last week i was suffering from intestinal catarrh- infection...thanks Doctor: Hello, You have symptoms of thin edges at sublingual papilla which is mainly due to dehydrated ora cavity, acid reflux due to systemic disease and could be due to an allergic reaction. Need not to worry. I would suggest you drink plenty of water and chew on sugar-free gums to stimulate salivary flow. Avoid intake of sharp and spicy food. You should gargle with a numbing solution containing Lignocaine for relief on tongue and pain. You can take an anti-inflammatory painkiller like Ibuprofen. It will start improving within a week but in case if the condition does not resolve to consult oral physician once and get evaluated and start treatment accordingly. Go for blood tests as per prescription.Hope I have answered your query. Let me know if I can assist you further.Regard, Dr. Harry Maheshwari"
},
{
"id": 175113,
"tgt": "What can cause a big puss filled boil on the chin of a child?",
"src": "Patient: My son had a big boil on the chin. After it burst and fluid came out, over a period of time there is a small pit / dimple chin formed (a fine hole can be seen). Occassionally fluid from the same oozes out..esp after bath with hot water. What is the treatment for the same. We have done an x-ray of the chin and there is nothing found. Dr. has indicated sinus of the chin. can u help Doctor: Thank you for following up, I will advice local therapy, wash sinus with hydrogenium peroxide, around apply iodine and inside put antibiotic -1% chloramphenicol 2 times 5 days.In addition general antibiotic-amoksiclav or ofloxacin 5days in dosage according to age.Visilak 1 capsule 1 time for pofilaxis of disbacteriosis during antibiotic therapy 14 days.For increasing immunity zincovit 5ml 1 time 1 month and of cause give him natural antibiotics-garlic and onion every day. For increasing of phagocytosis add vitamin C 500 mg 1 time 10 days Recovery soonDr.Svetlana"
},
{
"id": 202117,
"tgt": "Is there a way due to regain the sexual power lost due to masturbation?",
"src": "Patient: i spoiled my sexual power, its been more than 10 years i am doing the hand practice, now i have left this habit but i am sexually finished, my parents want me to get married and i dont feel myself sexually fit for marriage. please suggest me something. is there any way i can regain what i have lost due to that dammed habit.... i really would be obliged, i am 33 now. please help save my life........ Doctor: Hello.welcome to Health care magic.1.Not to worry, almost every individual come across this phase.2.The thing is, it look like you have not came across the relationship thats why you feel so.3.This is all in your head/thinking which making you to think this way. You are fit and you will be fine.4.As long as you have no erectile dysfunction, you will get on.Hope helped. Anything to ask ? do not hesitate. Thank you."
},
{
"id": 100144,
"tgt": "Suggest treatment for an allergic reaction on skin",
"src": "Patient: I think I may have had an allergic reaction to lavendar oil. My eyes are swollen and my eyelids feel and look as if they are burned. I took Claritn because it was the only thing I had while on vacation. Will this make the symptoms go away or do I need to do something else as well? I am unable to mix Claritin and Bendadryl at this time correct? Doctor: Claritin (Loratadine) is an effective antiallergic agent and should help in reducing the reactions. You may apply some emollient or moisturiser or calamine lotion over the area for soothing effect. No need to take Benadryl. In a few days time it should be fine."
},
{
"id": 137847,
"tgt": "Suggest remedy for pain in the back of the neck to the shoulder",
"src": "Patient: Hi, I am 42 year old female, I have been experiencing pain from the back of the neck unto the shoulder blade. I thought I probably just slept in wrong position or pulled a muscle and that was the result, its now day 3 and the pain is still there. My husband noticed there is a small pea sized lump on my left shoulder, when he presses on it the pain seems to be less for that time. Wondering if that is something minor like me pulling a muscle? Doctor: Dear patient,your symptoms are acute ie of 3 days duration, severe, and if the pain gets relieved in one position and increases in other, then the chances of its being muscular is more. it could be trapezius spasm, also be clear that you do not have any radiation of pain into your arm and hand, if its so than chances of neural origin is more. 1 Take rest in position, which relieves the pain,2 take analgesics like ibuprofen with muscle relaxant medication,3 Apply infrared heat from lamp, or heat from water bottle, and see if pain soothens.4 Local analgesic spray can be used for faster effect.I hope that answers your question.Thanks"
},
{
"id": 113022,
"tgt": "Suffer from back pain due to slipped disc. Pain reduced after Ayurvedic treatment. Can I play badminton?",
"src": "Patient: age 23, weight 59i had disc slip 3months back in the L4/L5section. There was back pain and pain in my legs too and i adopted ayurvedic treatment. now the pain is cured and now i am able to do my daily activities with out any pain. there is only a mild pain at the back when i am sitting for a long time without back support. i used to play badminton before this disc slip. now can i start playing again??? Doctor: Hello, as your mild pain is persisting and you are confirmed about the prolapse then you should start back and trunk strengthening exercises first along with hamstring stretching. Slowly after your pain resolves, start placing using a lumber support first and slowly wean off but mind you as the disc has prolapsed it can strike back again because the disc never goes back into its original space. All we have to do is certain life style modifications. How much you can play will be decided upon the magnitude of problem you have. Just go as per the protocol and you will be fit to resume your daily play. Thank you. Feel free to contact at drgautamsipani@gmail.com for further review."
},
{
"id": 177837,
"tgt": "Suggest treatment for frequent cough and cold in a child",
"src": "Patient: my daughter is 2 years and 5 months old, she is getting frequent cough, cold and very high fever. Every month sge is getting this. This time my dr gave her calpol 250, bricarex a, and enzocart. but no improvement. Getting blood test CBC, MP and Urine test done. Earlier also she has taken lots of time to recover and got blood test also done those were normal. Two queries i have 1. How she will be recovered this time 2. Once she recoveres what can i give her so that the frequest fever cough and cold can be avoided. Doctor: Hi dearWelcome to HCMExposure from the people suffering from the flu and RTI should be avoided.Improve immunity with high protein diet.Iron syrup andultivamins to be added.Antihistaminic and decongestant medi to be added.Steam inhalation to be done.Hope the query is answered. Thanks"
},
{
"id": 158032,
"tgt": "Should I take hormonal therapy to prevent recurrence of low grade stage 2a endometrioid adenocarcinoma?",
"src": "Patient: I want to know if I should be taking any hormonal therapy to prevent recurrence of low grade stage 2a (3 cm. tumour) NO M0 early breast cancer treated in 2011 while I wait for a total hysterectomy scheduled for October 2013 to treat well-differentiated endometrioid adenocarcinoma with squamous differentiation diagnosed by endometrial biopsy. I am a white postmenopausal woman aged 55. My oncologist told me to stop taking Tamoxifen 20 mg daily since my diagnosis for uterine cancer in July 2013. I would appreciate medical literature references, too. Doctor: Hormonal therapy is normally recommended for 5 years. one of the side effects of Tamoxifen is uterine carcinoma which has happened in your case. In such cases Tamoxifen is stopped and a total hysterectomy with bilateral oophorectomy is done at the earliest and Tamoxifen restarted. It is not advisable either to stop Tamoxifen for 4 months or to continue taking it in the presence of uterine cancer. So my advise to you would be to get the hysterectomy done at the earliest rather than waiting till October and then restart Tamoxifen. All the references can be had from nccn.org"
},
{
"id": 163930,
"tgt": "How long will cough and wheezing take to subside?",
"src": "Patient: my son had a nissen fundoplication three months ago due to severe reflux and esophagitis. He still has a barking cough and wheezing. How long will it take for the wheezing and cough to dissapear? It has improved and he is eating and sleeping much much better. He is 18 months old Doctor: Hi... coughing and wheezing in this season could be because of the ambient temperature and seasonal changes. Depending upon the severity the cough and wheeze, it may take a week or 2 weeks to settle completely.But if your kid is having recurrent symptoms, probably, he will require inhaled corticosteroids in the form of metered dose inhalers to keep these symptoms completely subdued.Regards - Dr. Sumanth"
},
{
"id": 171764,
"tgt": "Suggest treatment for nasal congestion and cough in a child",
"src": "Patient: My daughter is 3.9 years old she is having congetsted nose and productive cough with fever upto 99 f doctor has suggeted to give meftal p 5 ml 02 times and cefpodoxile cv 3.5 ml 02 times fever is coming down after taking medicine and coming back pls suggest this treatment is ok with her Doctor: Hi,Continue with medicine, antibiotic medicine takes some time to give its effect.Treatment is ok.Once antibiotic starts acting fever will be alright.Give her plenty of water and liquids and keep her well hydrated.Ok and take care."
},
{
"id": 122647,
"tgt": "Suggest treatment for stiffness and numbness in finger joints",
"src": "Patient: My fingers in both hands are very stiff & when in bed/at night completely frozen. My left thumb at both joints have complete Triger Finger & painful. Also, three other fingers (left hand) & two right hand have Triger fingers. fingers. All my doctors blood testings indicate there is nothing wrong with me. Meanwhile, due to numbness & pain at night I am suffering at during the daytime I can t open even a bottle with my hands. Doctor: Hi, Well, there can be many reasons for symptoms you mention in queries like a sign of nerve damage, repetitive stress, injuries, bacterial or viral infections, toxic exposures, and systemic diseases such as diabetes and peripheral neuropathy. There can be other reasons also like nerve entrapment, carpal tunnel syndrome, ulnar nerve palsy, hormonal imbalances and vitamin deficiencies (vitamin B1). I would suggest you consult an orthopedic surgeon for proper examination. The doctor may order a CT scan, ask for certain movements and complete blood tests. The doctor may prescribe muscle relaxants, recommend physical therapy of arm and hand along with stretching exercises and recommend particular stretching exercises. Hope I have answered your query. Let me know if I can assist you further. Regards, Dr. Ivan R. Rommstein, General Surgeon"
},
{
"id": 117339,
"tgt": "What does these blood test reports indicate?",
"src": "Patient: Hi Dr,I am 54 this year and I just receive my blood test report which states the following :1. Total wbc = 3300/cmm (below the normal range of 4000 - 11000)2. polymorphs = 47% (below the normal range of 50 - 70)3. lymphocytes = 44% (above the normal range of 20 - 40)5. Polymorphs = 1551/cmm (below the normal range of 2000 - 7500)6. Monocytes = 165 / cmm (below the normal range of 200 - 1000)The film states :RBC - Normochromic normocyticWBC - Mildly reduced with mild neutropenia but morphologically normal.Impression - Mild leucopenia with mild neutropeniaIs there any concern to my health? I have not experience any health issue for the past years and am rather careful with my diet. Appreciate your advise. Doctor: Hi, I have gone through your all reports. I can understand your concern. Your rbc is normal. Your wbc count is slightly reduced. Your neutrophil count is almost normal. If you don't have any health problem then no need to worry at all. Many people have this low count without any disease. It will become normal. You just need to repeat once after few timejust be relaxed. If you have any symptoms then consult your physician. Hope I have answered your question, if you have doubt then I will be happy to answer. Thanks for using health care magic. wish you a very good health."
},
{
"id": 197867,
"tgt": "Will taking MX3 increase sperm count?",
"src": "Patient: Greetings!May i ask if MX3 can give a male good sperm count?? Because i am now 36 years old and my wife is 33, but until now we dont have a baby yet. I am working here in Kingdom of Saudi Arabia, and i just go home and spend my vacation with my wife 1 month every year. i just do hope you can give us good advice in order my wife will conceive i go home this coming December. Thanks and more power to you all Doctor: Dear, We understand your concernsI went through your details. If your wife is unable to conceive, the problem could yours or hers. I hope you had gone through investigations by your urologist and your wife's gynecologist. You should not come into conclusions yourself without having urologist checked you for your sperm count. You should take sperm count improving supplements only on prescription of your urologist.MX3 is a supplement which the makers claim, useful. You can review the world wide web for the feedbacks and reviews. I still, strongly suggest you to consult an urologist or infertility specialist.If you require more of my help in this aspect, please use this URL. http://goo.gl/aYW2pR. Make sure that you include every minute detail possible. Hope this answers your query. Further clarifications are welcome.Good luck. Take care."
},
{
"id": 34086,
"tgt": "Suggest treatment for klebsiella spp",
"src": "Patient: hi, i have recently discovered that i have klebsiella spp, after having a complete digestive stool analysis done. i have suffered badly with diaharreh for 2 years and 6 doctors later i have been given cefaclor monohydrate 375mg twice a day to treat it. after taking it for 10days i was again sick so went back and saw another doctor (as mine was away on hollies) this doctor then told me that i would need to take it a lot longer than 10 days. I'm confused and not sure how long i should be on it. I'm scared if i dont have them long enough i wont kill this bacteria bug off. Help! Kim Doctor: Hello dear,Thank you for your contact to health care magic.I read and understand your concern. I am Dr Arun Tank answering your concern.Usually Klebsiella bacteria are pathogenic in the respiratory tract and urinary tract but not on the GIT or diarrhea.We can count this bacteria as a commensal flora of the intestine and can ignore its presence in the stool.So in my opinion you should not take the drugs just because Klebsiella is iaolated from the stool.Pathology for diarrhea may be some thing different and so can be diagnosed and treated accordingly.May be you have taken the long lasting antibiotics that has washed of your commensal flora. I advice you to take the probiotics like lactobacillus spore. This help replace the flora and can help in prevention of diarrhea.I will be happy to answer your further concern on bit.ly/DrArun.Thank you,Dr Arun TankInfectious diseases specialist,HCM."
},
{
"id": 98594,
"tgt": "What causes swelling in the ears and neck after eating a sandwich?",
"src": "Patient: Yesterday, after eating a sandwich, I had an allergic reaction where the left side of my neck from my ear down to my neck started to swell. I took a benedril and it went down. Everytime I eat anything, the swelling comes back. There is pain from the pressure on my ear. I have expereienced this 4 times in the last 12 months. I don't know what I am allergic to which triggers the reaction. Doctor: Hi i do care for your concern. Your history suggest more of allergic reaction occurring in parotid gland. More details such as your age and whether any allergy present will help a lot to arrive at diagnosis. I would advise To consult your dermatologist and have tested for the substance causing allergy. Benadrly may help for short time only, it is advisable that you have de-sensitization procedure done. Hope everything gets well soon. Hope i have answered your question, if you have more feel free to ask. Thank you."
},
{
"id": 39111,
"tgt": "What causes inflammation around the dog bite site?",
"src": "Patient: Sir, I was bitten by a dog in July of 2008.I took injection of tetanus 26 or 27 hrs latter then i was injected with anti rabis about 27 days latter but i always feel inflation around bitten area.Sir, am i needed further treatment? please suggest me sir i live in tension. Doctor: Hi,I had gone through your query and understand your concern.Please do not worry.You were treated with proper medications earlier.The cause of feeling inflation around the bitten area may be due to fibrosis and scarring during wound healing.It will not cause any health problems and you do not need any further treatment for this.If you find signs of inflammation like redness,swelling,raised local temperature,pain etc,then please contact with your local doctor because it needs physicians attention to confirm the diagnosis and for proper management.Hope this relieves your tension.If you have further queries,please ask us any time.Wish you peaceful life."
},
{
"id": 66260,
"tgt": "Suggest remedy for lump on forearm",
"src": "Patient: I was at work and fell, landing mostly on my elbow almost 2 years ago. I went to a Doctor (work comp) who sent me for an MRI and said there is nothing wrong with my elbow. Immediately after I fell there was a lump on my forearm, right next to my elbow that is still there. It s hard, sometimes painful, and when I pull the skin on my arm back about the size of a strawberry. Any idea what it might be? Doctor: Since it has occurred after a fall on the arm, it could be a condition known as myositis ossificans which is common at the elbow. It is just calcium deposits and nothing to worry about. If pain increases, it will need removal"
},
{
"id": 212424,
"tgt": "Diagnosed with ADD since childhood. Trouble in focusing. Suggestions?",
"src": "Patient: Hi, I m summer and I m 24 years old. Ever since i was a young child I ve been diagnosed with ADD/ ADHD . As a child i was able to get medication through medicaid, now that I m older medicaid no longer treats my disability . I work at O reilly auto repair in Hammond La. I m also having trouble focusing, and getting the job done correctly. Sometimes i find myself rushing too quickly while doing something, not noticing how fast i go. My boss recommends me to find a doctor that can help my disability. However, i have no insurance nor medicaid. Someone please point me in the right direction on finding a doctor for a reasonable price. Doctor: Hi there ~ I am sorry that you are having to go through this condition. Please look up the local social services in your area. To help I have looked up some on the net for you: http://www.yellowpages.com/hammond-la/social-service-organizations Also the city of Hammond LA should have a social services department that can help. I hope this helps. Take care and have a lovely day !"
},
{
"id": 33474,
"tgt": "What could green oozing spot on chest suggest?",
"src": "Patient: There is a small sort of green spot on my chest (it's been there for years) and every so often it will pretrude slightly and I will squeeze it to make it flat or less notable but the gook that shoots out smells aweful. What is that and can it be removed? Doctor: HI, thanks for using healthcare magicIt may be a cyst that is recollecting . Effective treatment would involve removal of the cyst. If it remains then it would continue to recollect intermittently.Depending on its size and location, your GP may be able to remove it. If he or she cannot then you would be referred to a general surgeon.It should be a simple in office procedure if it is a small lesion.I hope this helps"
},
{
"id": 58573,
"tgt": "Cirrhosis of liver, swelling of legs, swelling in stomach. Had gallbladder removed 2 years ago. Suggestions?",
"src": "Patient: I' m 59, female, had gallbladder removed 2 summers ago. My liver started with hep c now I have cirhosis of the liver. My edema of , lately left me swelling of my legs and feet more obvious on my left and continues. I had a home injury which is leading to a rotary cuff repair soon. Since my last visit about July 31 I have been swollen even a distending stomach. I don' feel like eating much but I try. Is there any comfort I should try before the clinic opens? Done. Doctor: Hello,Thanks for posting on HCM,There is not much you can do at home if you are having complications of cirrhosis (swelling of legs and abdomen/ascitis) besides avoiding to take in too much fluids (there is need to reduce fluid intake) and also reduce salt intake. These are potential sources of fluids and since you are suffering from body fluid retention, it is best you avoid taking more until medical intervention can be done. Taking care of your liver cirrhosis can prove to be tough and you will have to get through a lot of treatment sessions just for temporary relief. You will have to exercise patience and acceptance of your condition. If you are having poor urination, then you might also be having an associated kidney diseases. Hope you stay well until the clinics do open.Regards"
},
{
"id": 34355,
"tgt": "Could typhoid cause loss of appetite and vomiting?",
"src": "Patient: Hi I am Ripu Daman I have positive widal test 20 days ago and doctor told me it was typhoid about 5 days ago I was feeling alright then again I was having fever again I consulted doctor and he gave some medicines and now I am having Vomiting also is this due to Typhoid still I am unable to eat anything due to vomiting Doctor: HI, thanks for using healthcare magicYes, the typhoid can cause vomiting, fever and loss of appetite.There are over the counter medications that can be used to help reduce the vomiting.You should concentrate on using small amount of fluids.If you use large quantities then you may continue to vomit. If the small quantities (1 tbs every 5 mins) is tolerated , then you can increase this.When the vomiting stops then you can restart solid food.I hope this helps"
},
{
"id": 156780,
"tgt": "After being operated for liposarcoma, why do I have tenderness?",
"src": "Patient: Hello Dr, I had under operation for liposarcoma 4-5yrs ago, on my left thigh region, now I feel some amount of tenderness in the area, what should I do,? are there chance of growth again? should I go for consultation? what are the test I need to do? again? please reply to YYYY@YYYY Doctor: pain in the same region of primary cancer, definitely calls for attention to look for recurrence of cancer. It is better to undergo MRI of thigh bone with contrast and biopsy of the area, if anything is seen on the MRI, to confirm the cancer. If it is normal, then, nothing to worry. I think, u must be undergoing followup once in 6 month with chest x ray, to also see the recurrence of tumor in lung."
},
{
"id": 50544,
"tgt": "Metastatic melanoma. Lymph node dissection done. Had RCC and have one kidney. Treatment?",
"src": "Patient: what type of treatment would be offered to my 83 yr father who has metastatic melanoma, the primary site was on the upper, outer outer arm, lymp node disection done only one node involved but about ping pong ball size, now its returned next to the incision site. its nodular melanoma and a clark level four ? he has had rcc and only one kidney which is not functioning well. Doctor: Hi, most probably no treatment can be recommended, as he is of advanced age, the metastasis is in stage four, and the kidney left is also not functioning well. Radiation and chemotherapy he can't with stand. you have sort pain killers and antibiotics with less renal side effects should be sought for symptomatic treatment. Thank you."
},
{
"id": 16250,
"tgt": "Red rash on leg, smelly pus. Applied caniston. Any ideas?",
"src": "Patient: Hello, I am 24 years old male. All the time I get this problem. I have a bath everyday and sometimes twice a day. I scrub well as I do not like dirt. Always on my right side of my leg... A red rash sometimes little gunky and saw if touched smelly appears between my leg and arch of testical. It is annoying because I ve had and applied for a week caniston and yet this problem is still there. Any ideas? Doctor: hello and welcome to HCM dear Kyles, the area between the thighs are a bit sweaty and ahs a lot of sebaceous glands. secretions do take place and bath alone will not decrease it. you may try applying cotrimazole powder after your bath and before wearing your dress. pleasee do try these and and let me know if you need more help. thank you and take care. dr satya"
},
{
"id": 78515,
"tgt": "What causes left chest pain after eating or drinking?",
"src": "Patient: Hi my name is misty I have left chest pain directly after I eat or drink something including water. Sometimes after eating I feel sweaty dizzy chest pains start and then I throw up. After I excercise I also get the chest pain including just going for a walk. Doctor: Thanks for your question on Health Care Magic. I can understand your concern. Since you are having left chest pain and nausea which are associated with eating, possibility of gastritis and GERD is more. But better to rule out heart diseases first. So get done ecg and 2d echo. If both are normal then no need to worry for heart diseases. For symptomatic relief, do following things. 1. Avoid hot and spicy food. Avoid junk food. Avoid large meals, instead take frequent small meals. 2. Avoid stress and tension. 3. Start proton pump inhibitors and prokinetic drugs. 4. Keep 2-3 pillows under head in the bed. Go for walk after meals. Loose weight if you are obese. Don't worry, you will be alright with all these. Hope I have solved your query. Wish you good health. Thanks."
},
{
"id": 137499,
"tgt": "Suggest treatment for dystonia",
"src": "Patient: I have distionia , right hemi Arm, leg, hand). My hand seems to be closing-I need something to streetch my hand. I have had several gingers broken from falls. I am now going to o.t., but he doesn t seem to know how to handle my problem. Can you suggest something? Doctor: Weight bearing in right alignment is the first and main key of succsess in dystonia.You should do as much as possible weight bearing and second most important thing ask your therapist to make you exercise in close kinematic chain And add rotation component in each exercise your upper extrimity also should be supported and should b extended in all exercises.Some exercise for examplePush off in prone lyingQuadripod means all four in this position start with sustain then after some days add reaching forword and side waysThen sunstain half kneel here also firat suatain then add reachingStep standing diagonal reachingStep up and downI hope these will help you As far as medication concern you can start pacitane after counsulting your doctor."
},
{
"id": 159680,
"tgt": "Severe neck pain radiating to shoulders. Lump on the neck. Could this be cancer?",
"src": "Patient: I have been experiencing neck pain for about a month now. Whenever I move my neck the right side is very pain full from the top of my neck to the middle of my shoulder blades. It s very uncomfortable and nothing seems to ease the pain. Just recently I notice a small little bump right where my shoulder and neck meets. It doesn t move very much and it is the size of a dime, maybe a little bit smaller. Could this be cancer? Doctor: Hi, Thanks for query. It seems that you might have enlarge cervical lymph node pressing upon neck muscles giving rise to this problem. Nothing to worry about cancer. There are few conditions where there is enlargement of lymph nodes. Consult surgeon and get examined. If require he may go for lymph node biopsy for diagnosis. Ok and bye."
},
{
"id": 25402,
"tgt": "What could cause high BP?",
"src": "Patient: 56 Yrs,163 CM / 5' 3 \" , 70 Kgs. Everything normal except By 130 / 90 now.During check up it was 150 /100 and doctor advised me to have Nebistar daily one for 30 days.If i take it , should I have continue life long or i can discontinue after a dose. Doctor: Hello and thank you for using HCM.I carefully read your question and I understand your concern. I will try to explain something and give you my opinion.We talk about hypertension if we have mean values that exceeds 140/90 mmHg.The hypertension is divided in two groups : the essential hypertension without an identifiable cause, usually in older ages and secondary hypertension meaning that is a secondary cause of it. This is diagnosed in young ages and the cause might be renal, endocrine ect.As you are 56 years old i think we are dealing with essential hypertension. This is a chronic disease, it has not a definite cure but it should be treated all life long. This means that you don't have to take the medicine just for one month, because if you stop it, the blood pressure will rise up again.The fact that the doctors prescribe it just for one month is that you have to monitor your values during this time and maybe after one month you can discus with the doctor about the dosage. If you have to continue with the same dosage, arise it or add another medicament or not, but not discontinue it.Along with your medical treatment, i advise you to have physical activity, walking 40 minutes a day and reduce your salt intake because it rises blood pressure.Hope I was helpful. Wish you good health.Best regards."
},
{
"id": 144559,
"tgt": "Suggest treatment for losing balance resulting concussions",
"src": "Patient: I have had several episodes of sudden loss of balance which results in an uncontrollable fall. I have had 3 concussions in the past, one of which was a double concussion as a result of a horse accident. All concussions happened in about an 8 month period of time. Doctor: Hello. Thanks for asking from HCM.I can understand your concern. If you are having repeated episodes of loss of balance along with history of multiple episodes of concussion, then your complete neurological examination is needed. Your cerebellar and vestibular functions will be evaluated by neurosurgeon. After that you may be advised CT scan or MRI of head to find out any lesion in brain. You might be prescribed medicines like beta-histidine to control vertigo, avoid going in dark, full hydration to rule out low BP.At home, you may follow few tips to evaluate yourself and get relief: Check your hands whether you are moving tremor like movements, try walking with close eyes to find frequent falls, look for any weakness in limbs. If any of above symptoms are present, you should consult doctor as soon as possible: Check your BP if you have manometer at home to find out low BP.: Avoid walking in darkness: Find out in your family whether anyone had episode of stroke, diabetes mellitus, hypertension. If present, there is possibility that you might be having episodes of \"transient ischemic attacks\". It is an alarming sign which later on, can lead to ischemic stroke. Thanks. Hope it will help you. Take care."
},
{
"id": 135588,
"tgt": "Suggest remedy for muscular pain",
"src": "Patient: A few days ago my right knee and thigh swoll up and pain in that leg also noticed a lot of stuffiness in my sinus and flem. I don t have any insurance because I lost my job but wondering if there is something over the counter I can use. I am taking diurex right now Doctor: Hi Dear,Welcome to HCM.Understanding your concern. As per your query you have muscular pain along with sinusitis . Well there can be many reasons for symptoms you mention in query like rheumatic fever , haemarthrosis or seronegative arthritis . Joint swelling can also occur due to injury cyst like baker's cyst . I would suggest you to consult ENT specialist as well as orthopedic surgeon for proper examination . Doctor may order CT scan for sinusitis and also for knee . ENT specialist may prescribe antibiotics like amoxicillin or penicillin along with decongestant or expectorant . Orthopedic surgeon may prescribe anti inflammatory , muscle relaxant or may recommend surgical removal of cyst . Hope your concern has been resolved.Get Well Soon.Best Wishes,Dr. Harry Maheshwari"
},
{
"id": 173251,
"tgt": "Suggest treatment for hard bowel movement in child",
"src": "Patient: hi my son is 8 month old he is passing very hard stools for the past four days .i tried castor oil(yesterday night three drops of castor oil mixed with mother's milk and today morning extra one drop mixed in hot water )but still the condition is same.Pls help Doctor: Hi...Thank you for consulting in Health Care magic. I think your kid is having habitual constipation. I have certain questions and suggestions for you.Questions:1. Did your kid pass motion or meconium on day one of life?2. Since how long is the kid constipated?3. Does the kid have any bleeding along with hard stools?4. How much milk does the kid consume per day?5. Does the kid eat fruits and vegetables (fibre diet) appropriately?You can get back with answers at the following link - www.healthcaremagic.com/doctors/dr-sumanth-amperayani/67696Suggestions:1. Natural methods are the best to relieve constipation.2. Constipation is a risk factor for UTI3. Maximum milk consumption per day should not exceed 300-400ml4. Minimum 3-4 cups of fruits and vegetables to be consumed per dayHope my answer was helpful for you. I am happy to help any time. Further clarifications and consultations on Health care magic are welcome. If you do not have any clarifications, you can close the discussion and rate the answer. Wish your kid good health.Dr. Sumanth MBBS., DCH., DNB (Paed).,.."
},
{
"id": 161372,
"tgt": "What causes child s stool to be acidic and green?",
"src": "Patient: My daughter has been suffering with her bowels pretty much since birth. She was breast fed until 8 weeks then switched onto aptamil 1. when she was about 12 weeks old, her stools turned acidic, green, foul smelling and loose. they are still like this now and she is 10 months old. she was 9lb 20z born and is now about 20lb 10oz. her weight did drop off when the problem started but she is now up to her centile line again. she is now on apatmil 3 3 x 7oz with 3 meals as well. these stools burn her skin and stain her clothes and cause her pain. queried lactose intolerance but nutramigen didnt work so been ruled out. Doctor: Hi, As her weight gain is appropriate I do not think there is anything for you to worry about. The color of the stools being green and mildly acidic is very much normal in this age group as long as the baby's weight gain and development are appropriate. Hope I have answered your query. Let me know if I can assist you further. Take care Regards, Dr. Sumanth Amperayani, Pediatrician, Pulmonology"
},
{
"id": 208825,
"tgt": "How to have a positive outlook in life?",
"src": "Patient: Hello, my name is Shannon and I have recently developed a phobia of bad thoughts and feelings. I'm always afraid that I will start feeling negative for no reason and it will ruin my outlook on life and ruin my day. Sometimes I just look through my eyes at things and it feels like i'm testing myself, like i'm seeing if I will start feeling bad or scared for no reason. Also the thought that I can feel bad at any time, it doesn't have to take experiemces frightens me which makes it worse. Sometimes it feels like a cycle that will never go away and I dread being left alone. Please help me!!! Doctor: DearWe understand your concerns. I went through your details.I suggest you not to worry much. You must understand \"IDLE MIND IS DEVIL'S WORKSHP\". You are obsessing these unnecessary negative thoughts because you arre idle, or whenever you are idle. Therefore o not be idle. Second is your attitude. Even if you fear, what is the necessity that the future shall become as what you think? I think the future is uncertain. You cannot predict it. Over and above, even if you cry, be emotional, do what ever you want, you can't alter your future.But by positive thinking, what ever may happen, you shall have the attitude to take everything positive. Let the obsessive thoughts be there. Ignore them. You can post a direct query to me in case you further need my services in this case. Hope this answers your query. Available for further clarifications.Good luck."
},
{
"id": 172643,
"tgt": "Suggest dosage of Meftal for fever in 11.5 months old",
"src": "Patient: Hi Doc, My baby is having fever since morning and he is 11.5 months old and his weight is 8.7kg. We aregiving him meftal p 2.5ml after every 6 hours, is it the right dose ? Also how much time does it take for meftal to show the results ? If fever comes back can we give him metal after 4-5 hours gap also ? Doctor: thanks for asking i gone through your question, your 11.5 month baby having since one day and you giving meftal p. and and you want to ask about correct dose and effect of it.correct dose of mefenic acid is 6.5mg per kg per dose, thrice a day. as you said weight is 8.7 kg , correct dose is about 25mg per dose(1.25ml of meftal syp), it can be given 8 hourly. so correct your dose.Meftal brings down fever in generally 1-2 hours and its effect to control fever remain for about 8 hours, after that fever may reappear. it not treat cause of fever. fever is geneally viral illness, that is mostly cured by body it self in 3- 5 days. if fever occur before next due dose of meftal , you can give syp paracetamoe 15mg per kg per dose , it can be given up to four times a day. hope i able to answer your query. have healthy family life."
},
{
"id": 222372,
"tgt": "Is Vicodin safe during pregnancy?",
"src": "Patient: hello i am five months pregnant and my doctor says its okay to have one energy drink a day and i have really bad headaches during my pregnancy to the point of needing vicoden and since i started drinking energy drinks my headaches have stopped when not even coffee or soda has been able to do that so am i harming my baby? Doctor: Hello, and I hope I can help you today.Many women have headaches during pregnancy, because as your blood volume increases to prepare for the birth, the amount of blood flowing to your head increases in intensity, which commonly can cause severe pounding headaches. Caffeine is a vasoconstrictor, which means that it makes the blood vessels slightly smaller and as a consequence it actually is a very effective treatment for headaches. It is actually used as an ingredient in prescription migraine medications.The warning against not using caffeine during pregnancy actually has no real scientific basis. There never been any studies in humans that actually shows an increase risk of any pregnancy-related complication.So, if caffeinated drinks help your headaches, there is no evidence that any harm will come to you or your baby. Furthermore, Vicodin is safe to use during pregnancy for pain as well.I would advise you to try to find the least amount of caffeinated beverage you need to drink to help your headaches. Tea and coffee are much healthier than soda or energy drinks because they usually have a lot of artificial ingredients.So in summary, it is okay if you need to increase your caffeine consumption a little bit. Neither doing that, or taking Vicodin occasionally for pain, will do any harm to your baby.I hope I was able to adequately answer your question today, and that my advice was helpful.Best wishes for the rest of the pregnancy,Dr. Brown"
},
{
"id": 209467,
"tgt": "Suggest causes for not realizing actual things and memory loss",
"src": "Patient: my self pankaj age 32, I am feeling that I dont realize actual things and I feeling. that I m in sleepy. state of mind. memory loss also, sometimes I feel darkness against my eyes. lots of stress should be apply to my mind to realize the things, at the evening my body and my words are like that I am taken drugs, suggest me what is the problem Doctor: Hello,Thanks for choosing health care magic for posting your query.I have gone through your question in detail and I can understand what you are going through.It could be a symptom of depression and will require treatment with drugs like escitalopram or fluoxetine or mirtazapine etc.Hope I am able to answer your concerns.If you have any further query, I would be glad to help you.In future if you wish to contact me directly, you can use the below mentioned link:bit.ly/dr-srikanth-reddy\u00a0\u00a0\u00a0\u00a0\u00a0\u00a0\u00a0\u00a0\u00a0\u00a0\u00a0\u00a0\u00a0\u00a0\u00a0\u00a0\u00a0\u00a0\u00a0\u00a0\u00a0\u00a0\u00a0\u00a0\u00a0\u00a0\u00a0\u00a0\u00a0\u00a0\u00a0\u00a0\u00a0\u00a0\u00a0\u00a0\u00a0\u00a0\u00a0\u00a0"
},
{
"id": 187229,
"tgt": "Is tooth extraction and caps for the teeth in a 4 year old necessary?",
"src": "Patient: Hello, my daughter just went to her dentist and I was told she had to have 2 back teeth pulled and 6 caps. She is only four years old. I knew it wasn't god when I brought her in. She has experienced no pain what so ever. I know it was my fault from lack of care but it is my job to make her feel the most comfortable as possible. Her molar on her right side is bothering her gums because it is infected and is starting to puss. She was prescribed amoxicillin. They scheduled four separate appointments to do the dental work. The appointments seem fairly close together. I don't believe it is enough time for her to recover before going back. I had asked for her to be put to sleep when the teeth were pulled and the dentist told me she recommended her to be on laughing gas because she was very cooperative. So my question to you is, how long does it take a four year old to recover from teeth being pulled and caps? Also, do you think the teeth should be pulled? Do you think I should demand her to be put to sleep? Doctor: Hello, thank you for consulting with healthcaremagic. As you are mentioning that your daughter molar teeth are infected, and even there is pus discharge , then you should get them removed as these are milk teeth not permanent and if you will leave them they will infect the erupting permanent tooth also.These capping which the dentist has mentioned will maintain the space for permanent teeth, so they are necessary. It will take only 1 to 2 days in recovery from pain otherwise she will be ok after surgery only. So please do not take tension and believe on the dentist. Hope it will help you."
},
{
"id": 110174,
"tgt": "Suggest remedy for severe pain in lower back",
"src": "Patient: I have some pretty intense pain in my lower back mostly at night. it's in my lower back. i've had this pain for almost 6 years. it was after my second daughter was born. i had flinched when i was getting an epidural and felt it grind against my bones. was worse when i became pregnant with my last child because all my bones felt lax and my lower back felt as if it were shifting when i moved in some ways. now i can't bend too far(can't reach my feet nor my shin) because of pain when doing so and the night pains are terrible cause it feels as if being squeezed with rocks. this pain is like this half of a month not daily though. Doctor: Hi welcome to health care magic. After going through your query I think you require diclofenac sodium gel application locally, hot fomentation, analgesics and physiotherapy exercises . Hope your query has been answered . Thanks ."
},
{
"id": 210733,
"tgt": "How to treat chronic Post-traumatic stress disorder (PTSD)?",
"src": "Patient: I have chronic ptsd and have had for 15 years, spent 7 years in hospitals as regressed to child and kept trying to kill myself, over that now but have night terrors and have stable times then something triggers me and I fall apart again I hate this disease no one in my family will recognise it they just think I do it myself , I don't it just happens, I cant work anymore as diagnosed totally permanently disabled, just want to talk to someone else who has it Doctor: helloYou can find people with PTSD on internet in self help groups. But I seriously doubt PTSD leads to regression to child . Night terrors are common so may be you talk to a mental health professional.DR SAATIISH JHUNTRRAA"
},
{
"id": 117145,
"tgt": "What causes blood clot in both legs and difficulty in breathing?",
"src": "Patient: My husband has a blood clot in both legs. We don t know why he got them. No surgeries or anything like that. He can t sleep and he can t breath very well at all. He goes up the stairs and its very hard for him to breath. His legs and feet are very swollen and now his tummy is swelling. He now also has pain when he pee s and his urine is kind of brownish. Can you help us. We have gone to the hospital and that s where they found the blood clots but now I m really getting worried that he has something else more serious. Help! Doctor: Hi, dear. I have gone through your question. I can understand your concern. You may have deep vein thrombosis with pulmonary embolism. Don't wait. You should take urgent treatment to dissolve blood clots. So consult your physician and take treatment as soon as possible.Hope I have answered your question, if you have doubt then I will be happy to answer you again. Thanks for using health care magic. Wish you a very good health."
},
{
"id": 140752,
"tgt": "What causes a shooting pain on the side of the head?",
"src": "Patient: I have chronic headaches from work, because I sit behind a computer all day, but recently within the last 5 months or so I get sharp shooting pains on the side of my head that are debilitating at times. about 3 inches or so long and will last from a 1 second to 5 mins. What could it be. Doctor: Hi, The sharp pains you describe are consistent with an entity known as either \"jabs and jolts\" or \"ice pick\" headaches. These are 2 names for the same disorder which comes under the more general rubric of the trigeminal autonomic cephalgias. In other words, these are not migraines or tension types headaches although it is possible that eyestrain could be a contributory and even prior type of headaches from looking at the computer screen or not using proper prescription glasses. Your best bet to combat these headaches is to find a headache specialist or neurologist in order to review options. Indomethacin is the drug of 1st choice but in my experience with this drug in my headache clinic there is a poor overall compliance rate due to its GI side effect profile. Another possible treatment for this condition is high dose melatonin. However, I would not go out and simply do this type of treatment (even though it's OTC medication and may seemingly be safe). Melatonin in high doses can react with other medications you may be taking causing unsafe symptoms and even harm. You'll need a doctor to guide your use of either the melatonin and certainly the Indomethacin. Hope I have answered your query. Let me know if I can assist you further. Regards, Dr. Dariush Saghafi, Neurologist"
},
{
"id": 4989,
"tgt": "Did follicular sonography. Egg is ruptured. What is the chance of getting pregnant in this case?",
"src": "Patient: Hi Doctor,My wife just do a follicular sonography in that right sight egg is rapture but left one is still not its showing 13.5 12.5 11.5 sizeAlso we had relationship on yesterday and day before yesterday also.I just want to know that still we need to do a relationship and what is the chance of getting pregnant in this case?Thanks,Vaibhav Doctor: Hello,Conception is usually possible when intercourse occurs around the day of ovulation as the ovum has a very short life of 12-24 hours. It is safer to continue intercourse for a couple of days after evidence of follicular rupture; since there is a chance that the other follicles may rupture even if one chance of fertilization is missed, there would be other. Further sonographic monitoring also would be needed. Hope you find this information useful. Take care."
},
{
"id": 59125,
"tgt": "Feeling tired and weak, nausea. High bilirubin levels. Had an open heart surgery in the past. Any suggestions?",
"src": "Patient: I am a heart patient underwent open heart surgery 7 years back. I am a male aged 63 years. I am very tired and feel very weak. Unable to take my food and have vomiting sensation . My liver function test indicates my Bilirubin total is 2.1 Mg/dl and Bilirubin direct is 0.6 Mg/dl. Kindly advice me what should I do now ? Thanks Doctor: hello welcome to health care magic Elevated bilirubin levels indicates that your liver is not functioning properly or it may elevate because of increased breakdown of red blood cells.. You didn't mention about other symptoms like pain in the abdomen.. it is unrelated to your previous heart surgery.. kindly consult physician and do all the liver function tests..and according to him do an ultrasound.. hope you get satisfies with my answer. thank you Dr.siddartha"
},
{
"id": 133413,
"tgt": "What causes nerve pain on the feet?",
"src": "Patient: I have neorapity of my feet and was wondering if lyrica would be benifical for me to try.I have cold, burning and numb feet and it is getting so I cant fall asleep nites without taking a pain pill. Have had this condition for several years. & had many tests performed.Had douplar test etc saying circulation is OK & should not cuase the foot pain o.t than this I have lower back pain. which they sAY MAY BE A CAUSE SO THIS LEAVES NERVE PAIN...THANK YOU Doctor: Hello,The type of complaint u give appears to be in spine either due to nerve irritation,might be due to disc prolapse. Consult an orthopaedician for thorough clinical evaluation ,might require xray spine and mri of spine to look for lesion.Follow the instructions doctor will advice. I hope your ur problem will be solved and get well soon.thank you."
},
{
"id": 54989,
"tgt": "Can low potassium be a sign of gallbladder disease?",
"src": "Patient: Can low potassium be a sign of gallbladder disease. I have been through a nightmare of gi tests and fianlly I asked for a Hida Scan which was performed and I find my ejection level at 7%. For months I've has low potassium levels, about a year ago I had elevated liver levels. Is it possible that all along it's been my gallbladder causing my stomach issues and chemical inbalances? Doctor: hi.noted history of hypokalemia. are you experiencing other symptoms like abdominal pain, fever, vomiting, jaundice, dizziness? low potassium level or hypokalemia is not the usual presentation of a gallbladder disease. electrolyte imbalances of the body usually happens when you are experiencing gastric upsets, heart problems, malnourishment. it is best if you consult with a doctor for physical examination and clinical evaluation. other diagnostics and management will be directed accordingly. inclusion of banana in your diet is also recommended.hope this helps.good day!!~dr.kaye"
},
{
"id": 131582,
"tgt": "What could cause pain behind lower ribs?",
"src": "Patient: I have pain behind my lower ribs which is worse when I breathe deeply. When I lie on my left side, the pain comes in waves and is constant. It has happened in the past three days. Seems less acute now, but the pain when lying on the left side is pretty severe. Doctor: seems like your discribing a broken rib , did you recently have any direct trauma to your ribs ? you should so a normal x ray to confirm wish i could be of more help Good Luck"
},
{
"id": 13322,
"tgt": "What causes scratch marks on the back?",
"src": "Patient: My boyfriend plays baseball and has gotten marks on his back from being scratched up from that. But I ve also noticed more intense marks, ones that look old and more scarred. I am curious as to if it s possible that he had been whipped in some way as a child that he hasn t told me about, or if it is just really scratches? They are raised off the skin and almost look like incision scars, but he has not had back surgery. None of them look recent. But when I ask about them he simply says he doesn t know. He s out of the house now and in college but I am just wondering if he has a past he s trying to hide. Doctor: Hi,It may be keloid most probably.Consult the dermatologist for the perfect diagnosis and proper treatment. After thorough examination and investigations your doctor might decide the strategy of management. Intralesional steroid injection would help to reduce the lesions. Hyaluronic acid gel application may be done. Oral vitamin C and E may be taken.Hope I have answered your query. Let me know if I can assist you further. Take care Regards, Dr Ilyas Patel, Dermatologist"
},
{
"id": 181647,
"tgt": "What causes hole in the gum with white material inside?",
"src": "Patient: The gum behind my last tooth on the left side, the gum is swollen & there's a hole there & inside of the hole is something white. Its painful & I keep brushing it to make sure nothings in it. I also use mouthwash to clean it but nothing is making it go away. What is the hole & the white in it?? Doctor: Hi..Can understand your concern..As per your complain the swollen gum and the white material inside it is a result of some inflammation..There can be a few reasons depending upon whether it is the second molar or the wisdom tooth..In case it is a wisdom tooth which is erupting or is impacted then the gum above it can get infected and inflamed due to either food deposition between the gum and the wisdom tooth or it can be due to gum flap bitten by the opposite jaw tooth and the condition is known as PERICORONITIS.The white material in it is Pus.In case it is second molar the swollen gum with white material in it can be as a result of infection in the gums due to deposits on the teeth like plaque or calculus or due to foreign body impaction in the gums which carries infection along with it deeper into the gums causing pus formation.It can also be due to deeply infected tooth causing pus formation below the tips of the roots of the tooth causing swelling on the gums with pus inside..I would suggest you to consult a dentist and get evaluated for the exact cause of the swelling and get treated accordingly..In case if it is wisdom tooth either excision of gum flap or extraction if the tooth is impacted followed by antibiotics and painkillers can be done..In case of gum infection professional cleaning of teeth followed by antibiotics and gum paints can help..In case of tooth infection root canal treatment or extraction depending on the condition of the tooth can be done.Hope your query is solved..If you find the answer helpful please write a positive review and click on find this answer helpful as a token of appreciation..Thanks and regards..Dr.Honey Nandwani Arora."
},
{
"id": 189326,
"tgt": "Diagnosed with sinus tachycardia, no treatment, gave up smoking, stone overweight, jaw pain. Cause and cure?",
"src": "Patient: I am a 37year old women, diagnosed a couple of years ago with sinus tachycardia , no treatment, gave up smoking two years ago, 3 stone overweight. Past week have woken up with jaw pain on my left side, which abates during the day. No history of clenching teeth nor grinding when asleep, slight discomfort in left elbow and shoulder, but not sure if I would have noticed if not for the jaw pain, concerned it could be heart related. Father had heart attack at 51. Or is this jaw pain dental related? Doctor: Hi, Thanks for asking the query, Pain in the jaws is not related to heart diseases. It may occur due to infectious lesions like tooth abscess gingival lesions draining in the oral cavity, there could be presence of cysts and beningn and malignant lesions in the jaw, dislocation of the jaw, impacted wisdom tooth. I would suggest you to visit the Dentist and get the checkup done. Exact diagnosis can be made on clinical and radiological examination . For tachycardia you need to consult a Physician and get the checkup done. Hope this helps out. Regards.."
},
{
"id": 164597,
"tgt": "Can breast milk be poisonous leading to death in a new born child?",
"src": "Patient: My grandmother had 11 children but one after the other were dying within months of their birth. It was only my aunt who had survived. My father was born (number 12th child in the house). At his birth the midwife advised my grandparents that -the milk of my grandmother might be poisonous for baby and my grandparents sought help from friends to find a mother who can breastfeed their son (my father). a mother from other faith was found and she nursed my father like her own son. My father and grandparents are all gone but during funeral of my sister in January, 2011, my cousin (the eldest son of my late aunt) mentioned this in one of the family gatherings. We all know about this but since it came back down the memory lane, I was wondering to know, what could have been the cause for those deaths? Can mother milk be poisonous? Also, I would now like to visit Pakistan, the ancestral town of my parents/grandparents/greatgrandparents.. to find the family that helped our grandparents. We are Hindus and had to leave Pakistan at the time of partition in 1947, and the lady that breastfed my father was a muslim. Both aspects are important to me now. Regards Doctor: HiBe sure about one thing , mother's milk is best possible food in world for babies. It is advised not to be given only in very rare genetic condition called galactosemia ( but in that case any other person's milk will also be harmful) and some drugs if mother is taking like cancer drugs and others.Yes what you are mentioning is quite bad obstetric history. Reason can be many but not poisonous mother's milk. It is just a coincidence that your father did not received his mothers milk and survived.Myths involved, but the story is really touching about selfless humanity.Best wishes and regards to the lady"
},
{
"id": 65486,
"tgt": "Suggest medication for painful lump inside the right lower leg",
"src": "Patient: Hello. Great that you take questions. I am a beginning runner and have a small lump on the inside of my right lower leg, under my calve. When I run it becomes painful after a while, directly under my calve. The left side is painless. On the whole my right leg gives aches, while my left doesn t. I have scoliosis pretty severe. Could that be of relevance. What can I do to continue following my running schedule for beginners? Doctor: HelloI had gone through your question and I can understand your concernthis could be very well related to scoliosis if it is a nerve related painanother possibilitis are deep vein thrombosis muscle cramps varicose vein or cellulitis I would like further information to help you better likesince how long you are having this problemyour age and complete medical historyin my opinion you should consult a general surgeon for clinical examination and a battery of tests like Blood profile X ray or doppler studyI would advise you to take ibuprofen for pain and multivitamin and calcium supplementshope this helps you best wishes"
},
{
"id": 217953,
"tgt": "What to do if there is pain, heat, numbness in the knee after falling?",
"src": "Patient: I fell of my bike knees first my rt leg is healing but my left knee is not doing so well my bottom leg is still numb in some areas and i elevate my leg and take pain meds and the pain is 4/10 but once i step on my leg pain can be 8/10 with heat on the numb area please advise Doctor: Tat paim is due to nerve ending injury due to trauma.u can go for ultrasonic physiotherapy treatment and oral methylcobalamine500 tablets daily once 4 one month"
},
{
"id": 100809,
"tgt": "What is causing the swelling and hives?",
"src": "Patient: My daughter-in-law was diagnosed with Celiac in December. She has been diagnosed with MS since August 2008. She is having major problems with hives and swelling. Her face swells up almost every day. Today, she woke up with her eyes almost swollen shut. She is sticking very strictly to her gluten free diet. She has been allergy tested for common food allergies and had no significant reactions. They hives and swelling have been getting worse lately and started in the summer 2010. We are at a loss. What are your thoughts? Doctor: Hello.Thank you for asking at HCM.I went through your daughter-in-law's history and would like to make following suggestions for her:1. I would like to mention here that most of long standing hives and swellings (angioedema) are non-allergic. So if her all allergy testing is negative, I would not suggest her to think of \"allergy to something\", rather than they may be related to abnormal functioning of immune system. As both celiac and multiple sclerosis are non-allergic immune-related disorders, chances are very high that her hives are non-allergic.2. I usually suggest such patients daily antihistamines like cetirizine/hydroxyzine regularly. The dose of them can be adjusted according to response of her symptoms. 3. In general, I would suggest her avoidance of too spicy/too hot diet. Also please activities causing excessive sweating and hot water baths.4. Personally, I would also suggest her tight-fitting, woolen or synthetic type of clothes. In my opinion, loose-fitting, cotton clothes will be the best.5. I would suggest her a diet rich in vitamins and minerals (plenty of green leafy vegetables, fruits, sprouts, etc) which will help her immunity in a long run.6. A stress-free life is very important for a healthy immune system. Regular relaxation, meditation and exercises will also help her in a long run.Hope above suggestions will e helpful to her.Should you have any further query, please feel free to ask at HCM.Wish your daughter the best of the health.Thank you & Regards."
},
{
"id": 175056,
"tgt": "What are the signs of having a Enterocolotis?",
"src": "Patient: My son was born with Hirschprungs he had a pull through surgery at 5 weeks old, he is now 5 years old, he is continent but has a porrige like poo and poops for 20-30 minutes about 2 times a day, he has food issues refuses most foods, and low in iron. last week he got severe stomach pain , exessive sweating, fatigue close to loss of counciousness, rushed him to emergency, they got him on a drip he was like arug doll, fainting. To me he was in shock, the thow up and later blood in the poo. His temperature dropped to 35.5, was released from hospital 2 days later, x ray and ultrasound did not show any abnormalities, blood test are still pending they had difficuties taking his blood, my question is, could this be enterocolitis? And this has been 8 days ago, should have been a blood test done for this specific diagnosis Enterocolitis? Doctor: Thank you for following up. The symptoms of enterocolitis include:1.Syndrome of intoxication:nausea, vomiting, weakness,dizziness2.Pain syndrome:cramps,inflation of abdomen3.Fever It can be enterocolitis, foodborne disease. For confirmation of enterocolitis stool culture for flora and sensitivity to antibiotic and stool analysis should have been done. Best regards Dr.Svetlana"
},
{
"id": 97967,
"tgt": "MRI report, poor result, on physiotherapy, no relief, rest or yoga ?",
"src": "Patient: hi sir. i m sending u my two MRI reports ..sir i am going for phsiothrephy for last 2 mnth ..n i finally took mri report ..which was nt upto mark..n i m not getting any relief from phsiotheaphy ..sir now what should i do..should i stop goinf there..as i have to go for my joing on 10 march ..what should i do ..should i take rest or do some yoga at my home..sir please kindly help.. Doctor: 1. You haven't mentioned about the MRI report, neither you have told about the specific area for which this investigation was done. 2. You haven't mentioned about the symptoms for which it was advised? 3. If report says, there is STENOSIS, physiotherapy or any massage is Contraindicated,as it will result in more accumulation of fluid,resulting in edema and aggravation of symptoms. 4. Rest or yoga depends upon your Physician, that too after careful consideration of your symptoms,MRI report. 5. If cause is structural. initially Surgical Consultation will be of help!"
},
{
"id": 34827,
"tgt": "Is amyloidosis lichen curable ?",
"src": "Patient: HiI am male/30/170 pounds/Asian.. I have platular lichen amyloidosis in my Shins(congo red papules) in both legs.. Is this a curable ? I have been having this for many years.. to be frank i dont even remember...I am coming from a very host sunny place.Looks like this disease is seen more common in asians..More details.. have this hereditary wise as well.. so i dont understand the resons for getting it not see any decrease after taking many medicines.. IS this curable? I dont want to do cryosurgery as explain in internet as i see the skin goes white after the surgery.. please explain me if this issue has been resolved for any patients? Doctor: Hi XXXXThanks for your query at HCM!I am Dr. Sheetal Verma an Infectious Disease Specialist answering to your query!I went through your query and understand your concern.Amyloidosis is the term for diseases caused by the extracellular deposition of insoluble polymeric protein fibrils in tissues and organs. These diseases are a subset of a growing group of disorders attributed to misfolding of proteins. Deposits may respond to surgical intervention or radiation therapy; systemic treatment is generally not appropriate. Patients should be referred to a center familiar with management of these rare manifestations. Prolonged treatment is needed. Hope I have answered your question. If you would like some more information I will be happy to provide. You can also write a review for me. Wish you good healthTake care! Wish you best of Health.Dr. Sheetal VermaInfectious Disease Specialist"
},
{
"id": 104850,
"tgt": "How do you test for histamine intolerance? Have allergic rhinitis. Had nasal polyps",
"src": "Patient: Age 54, weight 195, type 2 diabetic, menopause , - How do you test for histamine intolerance? Allergic rhinitis my whole life, recently had nasal polyps, allergic to aspirin, ibprofen, sulfa drugs and mizicorium (an anastethic) Allergist finds very little: slight allergy to cats, goose feathers, tomatoes, dust mites. Loratadine and air conditioning helps. Also saline and a lower histamine diet. Any advice would be helpful. I believe the histamine problem is in my family for 4 generations. Thanks. Doctor: hi, you seem to be allergic to many thing also its severe.. i suggest you visit immunologist /allergy specialist and you have to under go a test called skin prick test..which will identify most of the allergens you are allergic to..once this is done then there is a treatment called immunotherapy. it is highly effective treatment.. you have to take it for arround 3-4yrs and your allergy will reduce by 80-90 percent. it is done for drugs too. histamine is a substance produced by the body itself once you come in contact with the allergen.. which causes symptoms immunotherapy is currently best treament and also very effective. thank you"
},
{
"id": 118202,
"tgt": "Is my blood pressure 150/72 okay?",
"src": "Patient: I m told that the guidelines have changed regarding acceptable blood pressure numbers for me. I m 79 and my pressure is 150 over 72. I just got off lisinopril with my doctor s permission. I exercise 30 minutes or more daily, and am on a salt-free, low-fat diet. One website says I m okay at this pressure and another says I need to lower bp tro 140. Doctor: Hi Thanks for your question.At 79 a BP of 150/72 without medicine is a treat. It is normal for this age. If, you can manage to lower your BP by another 10 degree that is very good for you. Continue with exercise and low salt diet hope fully you will be able to lower it further by 10 degree.Hope this answers your question"
},
{
"id": 208815,
"tgt": "How to treat the breathing problem with fast heartbeat?",
"src": "Patient: Hi doc.Im 28 years old. Im male. Two weeks before i have and panic attack (which can say a snake just cross my leg).So from the day i have this problem which my heart suddenly beat very fast and i feel difficulty to breath.I went to check doctor and the ECG test result is normal.What should i do or consider?Please advice Doctor: DearWe understand your concernsI went through your details. I suggest you not to worry much. The basic condition of snake crossing and having panic attack is actually called \"fight or flight syndrome\". you need to know it well and understand it. No problem. Such reflex actions happen with everyone and it takes some two or three weeks to have a complete cure. Read about \"fight or flight syndrome\" in the web and you shall understand more about it.If you require more of my help in this aspect, Please post a direct question to me in this website. Make sure that you include every minute details possible. I shall prescribe the needed psychotherapy techniques which should help you cure your condition further.Hope this answers your query. Available for further clarifications.Good luck."
},
{
"id": 149620,
"tgt": "Tremor, holding arm close to body, hand in fist. Taking medicine for seizures. Parkinson's disease?",
"src": "Patient: I noticed my mother in law over the holidays holding her arm close to her body at her side and her hand would be in a fist, and always a slight tremor , she is on medication for seizures and in her early 60 s is this something related to her neurological problems or could this be something new like Parkinsons? She does not have seizures on a regular basis Doctor: Hi,Thank you for posting your query.I must congratulate you for your keen observation.Tremors along with rigidity (as evidenced by a closed fist and arm held close to body) could be suggestive of Parkinson's disease. However, this needs to be confirmed by a detailed neurological examination by a neurologist.These are unlikely to be due to her seizures or her antiepileptic drugs.Please get back if you require any additional information.Best wishes,Dr Sudhir Kumar MD (Internal Medicine), DM (Neurology)Senior Consultant NeurologistApollo Hospitals, Hyderabad,My personal URL on this website: http://bit.ly/Dr-Sudhir-kumar My email: drsudhirkumar@yahoo.comMy blog: http://bestneurodoctor.blogspot.com/"
},
{
"id": 160846,
"tgt": "Suggest medical care for delayed motor development in children",
"src": "Patient: My husband and I are considering adopting a child who ulna and radius is missing but she is able to use her hands to feed herself. She has delayed motor development. She also had a successful surgery due to a congential heart defect. What possible medical care and therapy would she need? Thanks so much!! Doctor: Hi, As long as she is having good health and she is able to feed herself, able to move and play, there is no concern about anything. Hope I have answered your question. Let me know if I can assist you further. Regards, Dr. Salah Saad Shoman, Internal Medicine Specialist"
},
{
"id": 120990,
"tgt": "What causes numbness in hands and legs with shortness of breath?",
"src": "Patient: straightening of normal cercic lordosis sir i have numbness in my hands & legs often & shortness of breath racy heart beats & fiziness from last 3 months finally i got my xray for neck resulting for straightening of normal cercical lordosis due to paraspinal spasm are these symptoms are because of spasm pls reply Doctor: Hello, To comment anything else,I need to examine clinically and need additional information like presence of morning stiffness,other joint involvement,Fever.Back stiffness/pain.In presence of inflammatory arthritis, breathing problems are possible.Giddiness would considered in patients with neck pain.Palpitations needs further evaluation with cardiologist/Internist. Hope I have answered your query. Let me know if I can assist you further. Take care Regards, Dr. Rajesh Gayakwad"
},
{
"id": 182951,
"tgt": "What causes swelling in gums around the wisdom tooth?",
"src": "Patient: My teeth-ridge on the left side, the deepest side of my mouth, where my wisdom teeth is located on my left, has swelled up. It really hurts, and as it is almost covering my bottom left wisdom tooth, I can't close my mouth. Also, whenever I try to swallow my saliva, it hurts too. It also feels like it's paralyzed. Doctor: Thanks for using health Care magic.Read your query.Acute pericoronitis or infection in the erupting last molar will be the probable cause for this signs and symptoms.For the pain , ibuprofen can be taken (if you are not allergic to any medicine).Do salt water gargling and avoid application of hot bag externally.I would advice you to visit a oral surgeon and have it evaluated. Radiograph will help to know the exact position of the erupting tooth. If impacted , extraction is adviced. Antibiotics like amoxicillin and metrolag on prescription will be needed.Hope this was helpful.Thanks and regards."
},
{
"id": 154051,
"tgt": "How can Creon be given through feeding tube to pancreatic cancer patient?",
"src": "Patient: Hi. I am caring for my friend who has pancreatic cancer and is on a PEJ feeding tube. She has a lot of gas and her oncologist recommended putting creon down the feeding tube. Should I crush the contents on the gel cap and mix them with her water flushes or just put the little micropellets into the feeding tube mixed with water. They do not dissolve in water as I have tried. I am concerned about blocking her feeding tube. Doctor: Hi,Thanks for writing in.Creon is pancrelipase and is given to patients who have abnormal pancreas functioning. This enzyme helps in maintaining digestive functions of the pancreas. The usual recommendation is to mix the pellets with the soft liquid food and push them through the syringe. Following this you can flush the gastric tube and then clear any clogged part of the tube with water.If you mix the pellets with the liquid food then it will form a homogeneous distribution and pass through and even if there is little amount sticking to the walls of the nasogastric tube then it will be washed through the tube when you flush it with water. Please do not worry."
},
{
"id": 24127,
"tgt": "What is the total time frame of healing from VAD?",
"src": "Patient: I had a VAD 6 months ago linked to a chiropractic neck adjustment. I had a MRA on saturday, but no follow-up till 2 weeks from now. I have a family trip next week and called my Neuro to check on any restrictions. He was on rounds, but nurse called me back and said no high-velocity rides (including even then disneyland teacups). Scan was said to be stable. If you have seen VADs what is the timeframe that total healing can be seen? Stable just doesn t make me feel too confident. Doctor: Hi Welcome to HCM,I can understand your concerns.It takes minimum 6-8 weeks.Post your further queries if any,Thank you."
},
{
"id": 40454,
"tgt": "Is Induz effective in the treatment of infertility?",
"src": "Patient: hi, I am trying for conception since last 1yr, I already have a baby of 6yr old. last 2 to3 months I was suffering from cyst prob but it is cleared this month. today on second day of mine periods mine doctor has suggested me Induz and ovabless . shall I continue me too a malpani s daughter. Doctor: Hi, Indus is a brand name of Letrazole and it used for ovulation induction. You can continue Induz and nothing much to worry and your treatment is going on the right track. Hope I have answered your query. Let me know if I can assist you further."
},
{
"id": 152558,
"tgt": "Is Methadone a steroid?",
"src": "Patient: My brother has stage 4 liver and adrenal cancer. He has had a side effect from the infusions he has been given. It has drastically affected his lungs , nerves and some muscles. The doctors at UNC are giving him Messadon ( not sure of spelling). It is helping , is this a steroid? Doctor: Hello,Methadone is not a steroid drug, it an opioid used to reduce pain because of malignancy. The side effects which you are telling is because of chemotherapy induced side effects.Hope I have answered your query. Let me know if I can assist you further.Regards,Dr. Siddartha"
},
{
"id": 177523,
"tgt": "Is Fenistil drop safe for cold in an infant?",
"src": "Patient: My daughter is 8 months for cold i had given her fenistil drop 12 -14 drops at night but she took a nap for 1/2 hr n whole night she was awake taking turns n was not able to sleep is this because of overdosage of fenistil drops pls advise as she sleep nicely everyday Doctor: Hi...even cold and nasal block can cause disturbed sleep. Giving Fenistil is OK, but make sure you follow the physician's instructions or the manufacturer's on label instructions.Regards - Dr. Sumanth"
},
{
"id": 194804,
"tgt": "Does precum contain sperm from previous ejaculation?",
"src": "Patient: Ok doc, so i ejaculated when me and my girlfriend were messing around but this ejaculation was not in the vagina. I then went pee, awhile later we were messing around again and i had pre cum. I stuck it in about twice. Did my pee kill the sperm from a previous ejaculation and did my precum contain sperm from the previous ejaculation or was it flushed out? Doctor: Hi,Do not worry. Your precum did not contain sperms,because urination would have flushed out your semen. Take precautions..like condome during next meeting with your girlfriend.Hope this helps.Regards.Dr.Ilyas Patel MDDermatologist"
},
{
"id": 96103,
"tgt": "What is the diagnosis of Amoeba histolytica ?",
"src": "Patient: a child complain from abdominal pain above the umbilicus no vomiting no diarrhea ,there is constipation and diagnosed as amoeba histolytica what is the right diagnosis? Doctor: Hello. Thanks for choosing HealthcareMagic forum. The commonest cause of pain abdomen in children is abdominal colic and if they have diagnosed it as entamoeba then they must have done a stool culture to reach this diagnosis.A course of antibiotic will cure the condition. Dr. Rakhi Tayal"
},
{
"id": 62914,
"tgt": "Suggest treatment for white bumps on tongue",
"src": "Patient: Yes, I had two white bumps in the middle of my tongue last week that really hurt and wouldn't go away even after several days of using listerine. They were white and raised up quite high above the surface and hurt. so a few days ago I took a pair of tweezers and plucked them off. one of them bleed and the other one didn't. it stayed sore for a few days but seems to be OK now. Your thoughts on what it was and my solution to the matter. good or bad?? Larry Doctor: Hi Dear, Welcome to HCM.Understanding your concern. As per query you have white bump on your tongue . Well the symptoms you mention in query are pointing towards a condition known as lie bump also known as ' transient Lingual Papillitis ' . This occur when papillae on the tongue get irritated due to various reasons like hot and spicy food , acid drinks or food , and any other minor injury . This condition can also occur due to stress and hormonal changes . Although they can be uncomfortable, lie bumps aren\u2019t serious and usually clear up without treatment and within a few days. But your should not pluck then by your own as then can get infected and cause further complications . I would suggest you to consult oral pathologist for proper examination . Doctor will examine carefully and rule out conditions like scarlet fever , syphilis and traumatic fibroma . Doctor may prescribe antibiotics along with anti inflammatory . For now do warm saline rinses several times a day and maintain proper oral hygiene .Get Well Soon.Best Wishes,Dr. Harry Maheshwari"
},
{
"id": 156408,
"tgt": "Best medicine for increased TSH levels after radiotherapy for squamous cell carcinoma of tongue",
"src": "Patient: I had gone through Radiotherapy after surgery of tongue for squamous cell carcinoma about 20 months ago, Now I my TSH level has gone up and it is 7.69, I have been advised Eltroxin 50 . OD. Another doctor has advised Thyronorm 50 OD.. Could you guide me which is a better medicine. YYYY@YYYY Doctor: You have hypothyroidism secondary to radiation therapy to neck. It requires replacement of thyroid hormones. Both Eltroxin and Thyronorm contain levothyroxine. Eltroxin is more expensive than thyronorm. If you do not have affordability issues I would suggest Eltroxin as I have found it to be very effective in my patients and is well known to be a quality brand."
},
{
"id": 222845,
"tgt": "What causes numbness in arm during pregnancy?",
"src": "Patient: hi i am 20weeks pregnant with my 4th child and it is the first time i have ever experience cold,tingly numbness in my right arm is this anything to be concerned about,it has been like it for approx 10hrs,thanks. i am 38yrs 4ft 11 inches 50kg prepregnancy and now 54kg medical history all normal apart from all past pregnancies were c/sec. Doctor: Hi.It may be indicative of a nutritional deficiency or even possibly something to do with your heart or blood pressure. I cannot really say without more supportive investigations, so I would recommend a visit to your doctor.Best wishes."
},
{
"id": 55438,
"tgt": "Can medicines cause a fatty liver?",
"src": "Patient: I was just told i have fatty liver after a ct scan and blood tests Im going in for an upper and lower on may 12th,,,,my question is my meds i take? I take 60 mg ms contin 3/day,,,,10,325 vicodin 5/day in between for back pain for many years now....I also take klonopin 2mg 3/day and Nexium 40 mg 1/day and Amlodopin 5 mg 1/day for blood pressure...........Do you think it was these meds that gave me the fatty liver??? I DO NOT DRINK ! Doctor: Hi,Thanks for writing in.Medicines can cause a fatty liver but it is important to know that many other causes can contribute to it.There are many causes of liver showing fatty changes (i) Increased intake of fatty foods and alcohol(ii) Over weight and obese patients.(iii) Those who have fatty liver running in family members.(iv) Any liver condition that is changing appearance of liver.(v) Mild fatty liver can also be there in many normal peopleYou can get your liver function test and lipid profile done to know if the liver enzymes are raised and cholesterol numbers are increased. Please discuss results with your doctor and take required treatment if your tests results are higher than normal. Please do not worry."
},
{
"id": 168873,
"tgt": "What causes loose stools in a child?",
"src": "Patient: Hello, I have a 2 month old baby and he is passing motion 4 to 5 times a day and we have already consulted doctor and he has advised to give the baby C tax, O but I don t see any improvement. Is there anything to worry about now or wait for the full course of 5 days to get completed. His puss count is 26 to 30, no blood trace found in the stool.... Doctor: Hi...Thank you for consulting in Health Care magic.What your kid is having is not a diarrhea and is only a gastro-colic reflux. It is quite common for babies of this age group to pass small amount of diarrhea or loose stools soon after feeds. This need not be treated as diarrhea and especially antibiotics are not indicated.When the baby takes milk , the stomach expands - then when it is contracting it sets off a wave form which moves down the intestines and when that wave reaches the lower down rectum, a small quantity of the stool is evacuated out. This is the basis for gastro-colic reflex. Do not worry. Unless the kid's having low urine output or very dull or excessively sleepy or blood in motion or green bilious vomiting...you need not worry.Hope my answer was helpful for you. I am happy to help any time. Further clarifications and consultations on Health care magic are welcome. If you do not have any clarifications, you can close the discussion and rate the answer. Wish your kid good health.Dr. Sumanth MBBS., DCH., DNB (Paed).,"
},
{
"id": 206741,
"tgt": "Could increased pulse rate be due to tension or stress?",
"src": "Patient: hi, my age is 20 and my pulse rate usually gets upto 151 or so mostly it is near 140. i don't do any excercise or any other activity that may result in the increament of pulse rate. and alond with high pulse rate my bp also gets bit increased just around 140/ 110. I wouls also like to mention that am a migraine patient. so the question is that isit just due t the tension and strees?, i mean that the pulse rate comes back to normal after 1 or 2 days and after that am normal again. Would this pulse ate cause any serious problem or it's ok? Doctor: DearWe understand your concernsI went through your details. I suggest you not to worry much. From the description itself it is clear that you are worried about your health, and that is counted as stress or anxiety. You also has obsession about your pulse rate. You had that disturbing sensation once. You expected it to happen again and tried. Then you started experiencing it as you were expecting it. Then your body brought the familiar symptom because you are expecting it. Then you started worrying that the symptom is coming again and again and expect it. Body brings it again as you are expecting it. You become worried and so on. That is obsession in simple terms. Ignore it and the problem vanishes. If you require more of my help in this aspect, Please post a direct question to me in this URL. http://goo.gl/aYW2pR. Make sure that you include every minute details possible. I shall prescribe the needed psychotherapy techniques.Hope this answers your query. Available for further clarifications.Good luck."
},
{
"id": 131404,
"tgt": "What relieves the stiffness and pain from the ankles to the legs?",
"src": "Patient: Hi, I have constant chronic pain all the time. From my ankles, up the sides of the legs. Also in the arms in the middle joint area. Alot of stiffness, sharp pains like stabbing sometimes when i walk. Most times whwn getting up from sitting or laying down i feell like i weight a ton. When getting out of bed in the mornings i am bent over taking baby steps from extreme pain. Do you know what my problem is? Doctor: dear sir,from what i read you may have arthritis. you have to consult a rheumatologist because there are same kind of arthritis and the terapy varies from antiinfalmation agent to korticosteroid."
},
{
"id": 122866,
"tgt": "What causes severe elbow pain?",
"src": "Patient: I am currently being highly annoyed by what I can only call a sternum cramp. It feels like a joint cramp like one would get in a need or elbow right before a rather loud pop. To my knowledge a sternum should not have such a joint but all the same the feeling is there being highly annoying and rather painful at times. Doctor: Hello, It could be a tennis elbow.As a first line management you can take analgesics like paracetamol or aceclofenac for pain relief. If symptoms persist, it is better to consult a physician and get evaluated. In severe cases steroid injection to the joint cavity might be required. Hope I have answered your query. Let me know if I can assist you further. Regards, Dr. Shinas Hussain, General & Family Physician"
},
{
"id": 677,
"tgt": "Could nausea and back pain be due to pregnancy?",
"src": "Patient: Hi my n my husband are tying for a baby! My average cycle is 29 days since I have been monitoring about 4-5 mths now! This mth I started A day early and I think it may have been lighter than normal! I was sick the 5th day of my period which is the day I finish. The day after I felt really sick the last couple of days I feel a bit sick and been having lower and upper back pains! Two days a go we both started to take fertility supplement tabs, todAy I noticed that my stool is darker in colour. I've also noticed that I haven't been able to go to the toilet properly. Although bloaded this has been otherwise causing any pain. Could I be pregnant? Doctor: No , its not sign of pregnancy , if you have missed your periods this month ,then only you can go for preg test. problem you are facing is just because of these medicines , its an infection of urine. Test suggested: Urine testtake plenty of fluids and add green vegetable like spinach as it contain lycopene which is helpful in pregnancy. for your husnand - please suggest him to intake of cow milk."
},
{
"id": 76116,
"tgt": "What treatment is suggested for bruise on the chest region?",
"src": "Patient: Hello - in an attempt to pick me up and throw me over his shoulder like a jokester barbarian, my husband accidentally put too much pressure directly in the center of my chest and it now is very tender. I don t have any bruising or swelling, but I can t lie down flat, cough or do certain movements, like push down the trunk of my car, without pain. I ve looked up common symptoms for bruised/broken ribs/sternum but everything says I would have swelling and/or bruising, but I don t. It s extremely sensitive in the center of my chest and outward to the left. I can breathe deeply with some sensitivity, but not unbearable, but it does hurt when I put my shoulders back to sit up straight. Any thoughts? Doctor: Thanks for your question on Healthcare Magic. I can understand your concern. You are having blunt chest trauma. And because of this, you are facing musculoskeletal injury. No need to worry much for this as it will gradually improve over the time of 1-2 weeks. Follow these steps for better symptomatic relief in musculoskeletal pain. 1. Avoid movements causing pain. 2. Avoid bad postures in sleep. 3. Avoid heavyweight lifting and strenuous exercise. 4. Apply warm water pad on affected areas. 5. Take painkiller and anti inflammatory drugs. Don't worry, you will be fine with all these. If not improving after 1-2 weeks then consult doctor and get done chest x ray to rule out internal lung injury. Hope I have solved your query. I will be happy to help you further. Wish you good health. Thanks."
},
{
"id": 25895,
"tgt": "Can i do physical exercise after having stent on LAD?",
"src": "Patient: Im only 33 years old man , I have stent on my LAD. Doctor gave me Plavix and Aspirin 300 for one year . Is Aspirins 300 too much or should be lowered after 3 month. my other question , can I do physical exercise such running or playing football and climbing mountain Doctor: hello,I have gone through your query.Thanks for using HCM.yes you can and you should reduce the dose of aspirin to 75 mg per day.If you did not have significant damage to heart and your LVEF is normal you can surely do sport activity of your choice,My best wishesDr.Rajesh Teli,MD."
},
{
"id": 214789,
"tgt": "What is the natural or herbal remedy for treating cough due to acid reflux?",
"src": "Patient: Hi, I suffer severe cough due to my acid reflux. My GP has put my dose of Omperazole to 80mg until my cough calms down. Is there anything more natural or herbal i can take. I was thinking of buying some Probiotic pills from a health shop. I take Manuka Honey it doesnt help. Doctor: I h v read ur problem n there are effective herbal remedies for ur case ....1) drink cold milk with a 2 gm of mulethi powder in it empty stomach2) eat 5 munnacha dipped in water for 12 hrs every nit sleeping time3) Take 2 gm aanvala ( emblica ) with 2 grm of mishri powder 5 min before every meal4) HIMCOCID SUSPENSION 2 TSPOON AFTER every meal .Avoid curd rice beans pulses like Rajma urad, kulath, red spices junk food .non veg 5) dont drink too much water during mealSleep at proper time at nit but keep a gap of 1/2 hr btween meal n sleepBest of luck ..come back to me if u feel anything to tell...i will be happy to help u thnkssss"
},
{
"id": 193643,
"tgt": "Is libid crystal shots placed in tongue safe used for instant erection?",
"src": "Patient: Hello, I have received mail an unsolicited mailing offering 'Libid Crystal Shots' that are placed on the tongue and will give me an almost instant erection. I am an older person in good health and if these work and are safe, they would be very advantageous for me being in a relatively new relationship. Doctor: Hi, No its NOT SAFE. There are chances of cardiac issue. Hope I have answered your query. Let me know if I can assist you further. Take care Regards, Dr S.R.Raveendran, Sexologist"
},
{
"id": 195876,
"tgt": "How to regain libido and penis erection?",
"src": "Patient: i am sixty. years old having height of 5ft-11inches with 58 kg of weight.two grown -up and healthy children.recently,i have started losing libido and stiffness in penis during ejaculation. i would like to know the best tested oil or cream which can create stiffness in penis. Doctor: Hello,As per your history to increase libido and stiffness during ejaculation that is for erectile dysfunction and premature ejaculation, I will suggest you to take tablet Uphold 10 mg after sexologist opinion.Before starting this medication, you have to confirm that there is no history of liver, Kidney, heart Disease or stroke recently. You have confirmed it by blood tests after consultation.You can take this tablet generally one hour before sex. So, I will suggest you to consult with the sexologist.Hope I have answered your query. Let me know if I can assist you further."
},
{
"id": 39481,
"tgt": "Can a tattoo cause staph infection?",
"src": "Patient: I had a staph infection incident almost two years ago, i didnt get it treated by a doctor but havent experienced anymore occurrences, I recently got a tattoo over one my my scars that was left behind and i am afraid that it may be infected again. The tattoo artist used proper sanitation methods but the area appears to be slightly raised. Doctor: Hi and thank you so much for this query.I am so sorry to hear about these worries about a possible infection. Each tie there is a break in the skin, the chances of having a staph infection are increased. During and after the procedure, enough care should be taken to avoid any possible infection. if these measures fail, then the infection has to be adequately addressed. if you think it is infected(red, swollen, tender, drains), please get it evaluated for appropriate diagnosis and treatment.I hope this helps. I wish you well. Feel free to ask for more information and clarification if need be and I will gladly respond to."
},
{
"id": 169163,
"tgt": "Can heart problem with inability to urinate in infant can be rectified?",
"src": "Patient: Hello, my cousin has a new born, that was 2 days late of 9 months, so they proceeded with a C-section. The Doctor says the bay may have some sort of heart problem, but the baby cannot urinate as well. I read something about mothers milk and something called cal something. Can somebody please hel? Doctor: HiWelcome to the HCMI understand your concerns but do not worry. You may discuss with us the detailed condition of the baby including his heart condition. Many babies take upto 48 hours to pass first urine. In case he doesn't pass urine even after 48 hours then urinary obstruction such as posterior urethral valve or pelviureteric obstruction needs to be ruled out.Take care"
},
{
"id": 67084,
"tgt": "Suggest treatment for lump near the rectal area",
"src": "Patient: I have a bump on my bottom next to my rectal area . its been about 5 days & its very painful . I took a needle and burned the end of it , after that I poked where I seen the white head and it popped . there was blood , black stuff & pus that came out ! It also smells really bad . I just popped what I could for now but there a lot more infection in there . its so tender its hard for me to wear underwear . what should I do now ? Doctor: Hi,From history it seems that there might be having anal abscess giving this problem.Due to cut or soreness due to constipation there is infection leading to abscess formation.Go for one antibiotic, anti-inflammatory medicine course for 5 days.Clean the local part with antiseptic lotion and dress it with antibiotic cream.Keep local hygiene clean and dry.Ok and take care."
},
{
"id": 151907,
"tgt": "What treatment should my child undergo for gray matter heterotopia ?",
"src": "Patient: hi i have a child with gray matter heterotopia what can u advice me to do? is there any surgery or medication or whatever ? Doctor: Hi Noha! welcome to healthcaremagic.com.grey matter heterotopia is a developmental anomaly in which grey matter of the brain is situated wrongly.Treatment is aimed at reducing the seizures associated with the abnormality.n general, gray matter heterotopia is fixed in both its occurrence and symptoms; that is, once symptoms occur, it does not tend to progress. Varying results from surgical resection of the affected area have been reported. Although such surgery cannot reverse developmental disabilities, it may provides full or partial relief from seizures.Take care."
},
{
"id": 115235,
"tgt": "What is the treatment for anemia?",
"src": "Patient: I had a natural miscarriage 4 weeks ago, my baby was 16 weeks. I had heavy bleeding since month two. Before i delivered my baby he was alive. I beleive it was placental abruption. this was my 5th pregnancy. I have 4 children alive with no prior related complications. Following miscarriage, I had on/off heavy bleeding. I was in the ER twice. I was told I did not need a D & C and the first ER visit no retained tissue. The second US indicated alot of debris? where did this come from if week prior i was told my uterus was clear? After vaginal US I bled out. I passed out was in hypervolmic shcok, BP 70s/30s, I recived 2 bags of blood and over 6 bags of fluids, I had a spinal and recived an emergency D & C. I am 33 years old, married one partner my whole life. No previous gynecological issues, very healthy. Why did i bleed out and my body not clot? Where did all blood come from? Did the D & C correct the bleeding issue? One hour after surgery my BP was normal and my H & H was 29/9 before surgery my H & H was 22/7. How will the anemia correct itself now in recovery? How long until my body recovers? DO i need hormones? Did i hemorrhage? The bleeding stopped within 10mins but, I lost so much? Why did this happen? What will happen in the future if I should become pregnant? Now i fear miscarriage and hemhorrage?Thank You Doctor: Hi, Sorry to hear about your baby and all that you went through. After a major blood loss like this you need to take iron supplements for a minimum of six months and multivitamin supplements. You anemia will surely get better with this."
},
{
"id": 65645,
"tgt": "What is the cause of painful fatty lump under the knee?",
"src": "Patient: I have what appears to be a fatty lunch under my knee to the right - it's about 3\" x 3 inches and pretty thick - it's semi firm and the area around it is so sore to the touch. Past few months it hurts almost all the time and the w hole region has a dull ache to it. 24/7. Doctor: Hi! thanks for your post and giving us an opportunity to discuss on the knee lumps!I must, again, describe your lump as painful, fatty, firm and thick; and I must consider followings as possibilities, according to my routine clinical experience:1. a bursa or inflamed degenerative cystic tumor like condition: 50% possibility2. benign lipomatous tumor like angiolipoma 30% possibility3. atypical fatty tumors like dermatofibroma, liposarcoma, fibrosarcoma etc 10% possibility4. other infective or inflammatory lesions 10% possibilityTherefore, you must not neglect this and better see your doctor today for some basic tests to confirm the nature of this lesion and thereby relieve your tension...!Wishing you good health,"
},
{
"id": 50374,
"tgt": "Frequent ammonia smell when breathing, stage four of chronic kidney failure. Advise?",
"src": "Patient: i have chronic kidney failure and am currently at stage 4, and expecting to soon need dialysis or hopefully a transplant. My question relates to an amonia smell I get evey now and again. I had it a phew months back but only when breathing hard, my consultant said at the time that he didnt think I should be noticing that given my stage of failure. Past couple of days have noticed it again but only when breathing normally this time? Do you think I should be raising this with my consultant at this time or wait til my next scheduled appointment in mid May 2013? Doctor: HiThanks for the query.Ammoniacal smell to breath is seen in advanced stages of kidney failure.So you need to check your urea and creatinine levels and discuss the results of the same with your physician.Depending on the results and after examining you the physician will decide if it is due to kidney failure or some other cause.Hope this helpsGood luck."
},
{
"id": 157041,
"tgt": "Can shaving off head during chemo lead to ingrown hair/ infection and how can the scalp be moisturized and soothed?",
"src": "Patient: I am currently undergoing chemotherapy for stage 3C ovarian cancer. I was advised not to completely shave my head, but the short stubble I now have is irritating. I was told if I shave my head, I might get ingrown hairs and risk infection. Is this true? Also, I would like to know what I can use on my scalp to moisturize and soothe the bumpy and very irritated skin. Thank you Doctor: Hi my dear, I am sorry for the situation you are in. I understand your impatience of getting your hair shaved. What your doctors told you was right: shaving hair (especially during chemotherapy) increases the chances of ingrown hairs and infection. 1. You should wash your hair with warm water and hypoallergic soap.2. apply otc antibiotic cream with hydrocorticose (gentamycine + hydrocortisone). If not OTC available in your country, should consult your doctor for a prescription. 3. You might also need oral prescription antibiotics, for which you can discuss with your doctor. God bless you my dear!Dr.Albana"
},
{
"id": 23823,
"tgt": "Does smoking marijuana affect the stent in the artery?",
"src": "Patient: I recently had a stent inserted in LAD artery and have totally quit smoking after 47 years of 1/2 pack a day. I am on blood pressure meds, blood thinners now, and lipitore, and janumet for sugar(borderline), not overweight . I smoke marijuana recreationally , a few joints on weekend. Unlike cigarettes , how will this affect my stent and arteries. I was a heavy smoker all my life of marijuana and the rest of my heart and arteries are good. Do you think occasional weekend usage will affect the stent and that area? Doctor: Hi ! Thank you for using HCM Yes Marjuana may affect your arteries and stand condition. This happens cause mariuna affect your nervous system in result you may have : increase heart rate, high systolic and diastolic blood pressure ( that your medication may not manage ) ;.These 3 conditions makes your heart muscles to need more oxygen (so more blood ) and if you have problem with coronary arteries ( especially When you have stand ) you artery can not effort and in the end you may have Angina ( heart infarct ) Also Marjuana may cause vasospasm ( constriction of you coronary artery ) .Some studies shows that Marijuana may increse risk for heart atack with 4.8 after 60 minute you take it .Hope these help Wish you good health"
},
{
"id": 120059,
"tgt": "Suggest treatment for pain in foot after an injury",
"src": "Patient: yes my Achilles or the top of my heel hurts like crazy. i was wearing wedges for a party and fell to the side. during that day i was at the mall and it didn t hurt at all but days later the pain came in and i have to run in gym and it hurts a lot. can you tell me what i have or how can i heal it? Doctor: Hello, It look likes to be a tendo Achilles sprain. give hot fomentation, take over the counter painkillers and take rest for at least one week. Hope I have answered your query. Let me know if I can assist you further. Take care Regards, Dr. Jaideep Gaver"
},
{
"id": 17931,
"tgt": "What should be done to manage low blood pressure levels after taking Amlodipine?",
"src": "Patient: Yes I went to the my pressure was very high 187 I was taking hydrochlorothiazide 25mg also Lisinopril 40mg. And the doctor add another pill name amlopine 10 mg ok today I went to Walgreens to check my pressure it was 101over 68 so now it s to low now what? Doctor: Hi, I would explain that low blood pressure could be related to Amlodipine. For this reason, I would recommend taking half of the dose (5 mg) of Amlodipine, in order to avoid a further drop in blood pressure values. Hope I have answered your query. Let me know if I can assist you further. Regards, Dr. Ilir Sharka, Cardiologist"
},
{
"id": 5576,
"tgt": "On regular dose of contraceptive. Having unprotected sex. Chance of getting pregnant?",
"src": "Patient: Had sex on 4th and 5th day and on regular dose of contraceptive pills -Will I get pregnant Hi, I am on regular dosage of contraceptive pills (Overall L). My periods begun this month on 17th March. Now I had an unprotected sex on 20th and 21st (I.e. my 4th and 5th day). The periods ended by end of the 5th day. On 7th day, I.e. 23rd March, I started consuming my regular dose of contraceptive pills. What is the chance of getting pregnant. Please reply. Doctor: Hello Thanks for your query. If you have used the birth control pills as prescribed, regularly, without missing a single dose, they offer you almost 95 % chances of protection against a pregnancy. Also, having sex on the 4th and 5th day of the period itself has very less chances of resulting in a pregnancy. However, to be absolutely sure, continue taking your birth control pills as prescribed, and be very sincere with them In the rare event that you DO miss a period, please take a home urine pregnancy test at the earliest. Take care"
},
{
"id": 111325,
"tgt": "Can back be massaged when it has a benign tumor?",
"src": "Patient: My boyfriend has a benign tumor that is wrapped around his spinal cord around c-6 or 7. Iam a massage therapist. Can his back be massaged. Can I manipulate the tissue around the spinal cord in this area. He has had it since childhood he said and the doctors told him to just stay away from it. I feel bad because he needs a back massage and I'm afraid to do anything. YYYY@YYYY Doctor: Hi and thanks for the query,A benign tumor would not decrease in size with massage. In case it is already causing pain or symptoms, depending on the size, removing it could be an option. Kind regards"
},
{
"id": 182279,
"tgt": "Suggest treatment for toothache",
"src": "Patient: hello doc. i m 4 weeks pregnant. my wt is 66 kg and my ht is 5.1. ihaving a very bad toothache in upper rightjaws wisdom tooth. my doc has prescribed mucopain ointment and novaclox 500 mg three times a day for 4 days. are both the medicines safe? this is my second pregnancy. i hv a girl child. i dnt have any other health problem. doc will extract the tooth in the 4th month of my pregnancy. Doctor: Thanks for your query, I have gone through your query.The novaclox is a combination of amoxicillin and cloxacillin, you can take this during pregnancy, these drugs will come under category B(NO PROVEN RISK IN HUMANS). You can take topical mucopain ointment also. But If i am your treating doctor, i would have advised you to take plain amoxicillin and paracetamol. Once you complete your first trimester, get the tooth removed under local anesthesia. Do saline gargling and do not use mouth wash.I hope my answer will help you, take care."
},
{
"id": 20532,
"tgt": "Can the groin lump be due to cardiac catheterization?",
"src": "Patient: had heart cath 9 days ago and it came back normal as far as blockage. the Dr.said had mycardio bridging and was born with it. 2 days ago developed and lump in grion area were cath was done. had a ultrasound and came back ok there was no signs of blood clot. my question is could this lump be from heart cath procedure Doctor: Hi,Welcome.Cardiac cath or any other cath done from leg(femoral artery) can cause leakage & collection of blood in the subcutaneous tissue causing a swelling called as hematoma.As you said ultrasound of leg is fine than you need\u00a0not to worry. It will resolve gradually.Thanks"
},
{
"id": 36259,
"tgt": "Suggest treatment for painful, pimple-like spot on labia majora",
"src": "Patient: Hi, I noticed a small, slightly painful, pimple-like spot on my labia majora yesterday. I had shaved off the pubic hair almost completely (I found out that I m not supposed to shave the hair completely or wax it). I left the boil, thinking it must be because I shaved. I ve had small, painless boils before, but they were always only on the mons pubis. And today it s become larger. I also feel slight discomfort on my genital area, like a burning sensation. What do I do? Doctor: Hello,I understand your concern.I am Dr. Arun Tank, infectious diseases specialist, answering your query.In my opinion you should get your self examined by doctors and start the antibiotic Doxycycline under your doctors guidance.You can also apply polysporin ointment over the wound area.Maintaining hygiene by frequent cleanliness and dressing is also important, as it also cures you equally as do antibiotics do.Avoid sexual intercourse during this period as it can worsen the condition and it can be spread to partner also.Please use cotton undergarments as they prevent reaction with infection.Please wash the garments under antiseptic solution like dettol.I will be happy to answer your further concern, you can ask me on bit.ly/DrArun. Thank you.Dr Arun TankInfectious diseases specialist."
},
{
"id": 78003,
"tgt": "What could cause expansion in chest?",
"src": "Patient: Hello Doctor,I am 30 yrs old and I am an asthmatic patient from last 17 yrs And I am under formonide 400 medication but still I feel discomfort while breathing. Wanted to ask you regarding chest tightening. Is there any possibilty that the rib cage increase( chest increase) due to asthma as I observed my shirt size increased in last 6 months. And sometimes also getting pain while breathing Doctor: Thanks for your question on Health Care Magic. I can understand your concern. Yes, uncontrolled asthma can cause hyperinflation and air trapping. Both these cause larger lungs. And larger lungs when chronic can cause increase in chest wall size. All these are markers of uncontrolled asthma. Your pain is also due to this only. So better to consult pulmonologist and get done 1. Clinical examination of respiratory system. 2. PFT (Pulmonary Function Test) 3. Chest x ray. PFT will diagnose severity of asthma and air trapping. Chest x ray will diagnose hyperinflation. You may need additional inhaled bronchodilators and inhaled corticosteroid (ICS). Oral combination of antihistamine and anti allergic drugs are also indicated in asthma. So consult pulmonologist and discuss all these. Hope I have solved your query. I will be happy to help you further. Wish you good health. Thanks."
},
{
"id": 109494,
"tgt": "Is oxycodone help to reduce back pain?",
"src": "Patient: yes.I am taking morphine sulf 30 mg. twice a day for horrible back pain but it does not last long before the pain comes back,my doctor wants to change my medicine to oxycodone I am not sure about this because I do not want to take something that will make me tired or feel like I am not quite with it but I do want something that will help with the pain over a longer period of time do you think that oxycondone is a good choice?\\ Doctor: HiThank you for asking HCM. I have gone through your query. Oxycodone is better than morphine for long term management of pain. Side effects are also lesser compare to morphine. So you can follow your doctor's advise for changing to Oxycodone. Hope this may help you. Let me know if you have any further queries. Thanks"
},
{
"id": 56377,
"tgt": "Suggest treatment for pancreatitis whose lipase levels are too high",
"src": "Patient: Dr.. My husband is suffering from Pancreatitis for past 8 yrs. He is a non alcholic & non Smoker. His Lipase levels are very high. Can you suggest some foos which is rich in lipase level that could rest his pancrease. (he is taking Creaon 25K everytime after the meal Doctor: The best diet would be a very low fat diet. Also his doctor should make sure his blood triglyceride is not high and also make sure he is not diabetic. These would need to be addressed to make sure they are not stimulating his pancreas"
},
{
"id": 72680,
"tgt": "Suggest treatment for left-sided mid zone calcified opacity on a chest X-ray",
"src": "Patient: Hi doctor, this is Anfar i am from kerala. I got a job in gulf country and for that I went throurgh a medical test in they found me unfit just because of calcific opacity at left mid zone but rest all test was normal.Sir what is the remedy for this issue Doctor: Thanks for your question on Healthcare Magic.I can understand your concern. Calcified opacity on chest x ray is suggestive of old healed Scar lesion.Any lung infection heal by fibrosis or calcification. No treatment is required for these as they are inactive fibrotic scars. These scars are remain throughout the life. First fitness purpose, you should consult pulmonologist and get done CT thorax and bronchoscopy. If both these are negative for active infections then you can argue with Visa authority for fitness. So consult pulmonologist and discuss all these. Hope I have solved your query. I will be happy to help you further. Wish you good health. Thanks."
},
{
"id": 212167,
"tgt": "Have stressed out. Herpes simplex II is negative. Have itchy white bumps. Have endometrosis. Is it STD or stress?",
"src": "Patient: I have been really stressed out. I've been checked for herpes simplex II But, it always came back neg. Now I have these white bumps multible. And they itch. I have endometrosis, i've been stressed. I have herpes simplex 1 and my husband had cheated on me with a women yrs. ago but swears he wore a condom and she was medication for herpes. I'm petrifified that I have it. Its sore but not terrible. It's more mental then anything. I have had gential stds. Could this be stress related? Doctor: Hi, Thanks for using healthcare magic. I have gone through your available history. Except in case of psychogenic itching and hypochondriasis, it is very less likely that your symptoms could be explained by stress. You should consult gynecologist first to rule out the causes. If your gynecologist could not found any reason, You may need to consult psychiatrist to rule out hypochondriasis. Thanks"
},
{
"id": 129735,
"tgt": "What is the treatment for a knuckle fracture?",
"src": "Patient: i broke my hand i went too er they said i fractured 2 bones in my hand my pinky my other dr done some tests said one bone healed and the other didnt and my knuckle still hurts and looks swollen this happend 6 weeks ago i have a app with a orthopedic surgeon too look at it next week how will they fix my knuckle thanks Doctor: Hello,If a broken bone does not fuse then they usually need to do a surgery to repair it. Sometimes this involves putting a pin or screw in the bone and sometimes it involves a bone graft.Regards"
},
{
"id": 23642,
"tgt": "What is the next step after a cardiac injection?",
"src": "Patient: hi My mother was having a severe pain in hand and she went to the docto and he took her ecg and referred her to a cardiac hospital. she was immediately admitted and given an injection and asked to wait for 12 hours. will she be ok? or what can be the next step? Doctor: Hello ,Most probably she had an acute heart attack . Timely diagnosis by the doctors , she must have been able to reach the hospital with en 6 hours , so a clot bursting injection would have been given .This would reestablish the flow to the blocked artery preventing major attack . The first 24 hours are important to watch for complications . Considering she was given the right treatment , she should do well . Maybe later they may do an angiography to look for the blockages and treat them if required . Regards Dr. Mody"
},
{
"id": 100266,
"tgt": "How to completely stop Asthma medicine through diet, exercise and alternative medicine?",
"src": "Patient: I am M - 48 and have been suffering from severe Asthama since 30 years. However since 2008 I have followed a strict lifestyle change and have got rid of it 95%. Meditation, Prananayam, (not much Yoga) Strict fitness routine (i have a personal trainer), Strict vegetarian food, I sleep 9:30PM to 4:30AM. My Height is 163 cm, in 2008 my weight was around 50 and since last two years it has stabilised at 63. Now it does not bother me and does not effect my work. Apart from this i don't have any other health issues. Some times i still have to take one Puff of Foracort -200, once in morning and once before sleeping. This is for about 6 months in a year, when I get cough in the morning. At one time 2 puffs every three hours also wont work. I want to get rid of it completely. I am not looking for advice advocating that it is safe etc. to take inhales. Can some doctor advice me on this. With a strategy to get rid of medication completely, I am open to any stream ( alopathy, Ayurved, Homeopathy or anything else) Doctor: Hello,Thank you for asking at HCM.I went through your history and would like to make suggestions for you as follows:1. It is really great that you have been able to improve your asthma significantly by meditation, pranayam, exercise, diet and life-style changes.I would like to know more about you like - Do you have any medical condition other than asthma? Are you taking any other drugs frequently/regularly? Do you have any other allergies - nose allergies, eye allergies, skin allergies, food allergies? Do you have symptoms like headaches, nose obstruction, facial heaviness, ticking in throat, repeated throat clearing, etc? Do you have symptoms like burning in chest, nausea, coming of fluid into mouth especially after heavy/spicy meals or upon lying down? Are you exposed to air pollution, dusts or smokes? Is stress a part of your life?Above details would help me to understand your complaints better and make suggestions more specifically.2. I would suggest you allergy testing, especially to common air borne allergens like - house dust mites, indoor molds, regional pollens, insect proteins and pet dander (if you have pet).This will help you identify the substances you may be allergic to and also to know the measures to avoid them.3. Based on allergy testing, an Allergist-Immunologist may prescribe you allergen specific immunotherapy which works like vaccination to reduce allergy symptoms over a long time gradually.4. Personally, I would suggest you to have a Yoga trainer, who can teach you specific Yogasanas which would be helpful in asthma. (as you have mentioned you have not had much of Yoga)5. I would also suggest you a good multivitamin-multimineral with antioxidant as a diet supplement which improves your immunity. It is especially more important as such deficiencies are more common among those who follow a restricted diet.6. If stress is an important factor for you, I would suggest you stress reduction strategies like meditation and relaxation as stress can affect allergies & asthma adversely.Hope above suggestions will be helpful to you.Should you have any further query, please feel free to ask at HCM.Wish you a very good health ahead.Thank you & Regards."
},
{
"id": 11759,
"tgt": "My skin pigmentation have reduced, but is Panderm+ cream safe for prolonged use?",
"src": "Patient: Dear Sir, I had marks on my face and due to pigmentation my skin colour turned black. A known pharmacist has suggested me to use Panderm+ cream and i am using it from past 3 months. My skin colour has been changed to normal and marks has been reduced to 75%. CAn I continue using it as the pimples are still visible on my face. Kindly suggest Doctor: Hi, thanks for posting your query Panderm plus contains a potent steroid clobetasole , which should not be used without doctor's advise especially on face for more than 2 weeks as it may lead to thinning of skin , visible blood vessels and pimples You need to discontinue it and use clindamycin and adapalene combination for pimples and get it evaluated by a doctor for the type of acne and appropriate treatment . Once acne is under control you may use depigmentation creams as per doctor's advice Hope his solves your query"
},
{
"id": 166165,
"tgt": "What causes white chunky vomit in infants?",
"src": "Patient: Hello I have a 15 and a half month baby girl. We are slowly taking away her bottle and she only drinks one at night and one in the morning. After her last bottle last night she got very fussy and started crying, my husband picked her up and she threw up very white and chunky. Now this morning she did it again. She has no known food allergies or a fever or nothing else and she only seem to do it after she drank milk. What could this be??? Doctor: hi, welcome to this forum. Can understand your concern.- Vomiting in 15 month old child can be due to stomach infection, overfeeding or inadequate burping.- Stomach infection can occur in vomiting because the child usually takes everything in mouth and it can also occur due to teething. Due to teething, there is irritation in mouth and as a result child usually takes everything in mouth. This results in infection and vomiting in children.- Overfeading can be a cause of vomiting. If child is not taking milk then the child should not be over fed. The child usually vomits the extra milk. - Inadequate burping can also be a cause of vomiting. Child should be burped for at least 10 minute after every feed.- An examination by doctor should also be done so that we may not miss any important finding.I hope this will help you. Wishing your child good health. Take care."
},
{
"id": 182855,
"tgt": "Could an upper jaw black hole causing pain be a cancer?",
"src": "Patient: My husband has a abscess or cystis on his upper front jaw. He had his 2 front teeth fixed about 2yrs ago and already had a little round ball on his upper jaw which the doctor said he had a black hole there . Now it\u00b4s really starting to hurt him a lot , he thinks it might be cancer.Is that possible Doctor: Thanks for your query, I have gone through your query.The black hole in the upper jaw could be a pus discharging sinus tract. Consult a oral physician and get a radiograph done to rule out the pathologies like periapical cyst or granuloma or residual cyst. If i am your treating doctor, I would suggest you to get radiograph done like OPG, occlusal radiograph done. If there is any cyst, then we can do biopsy and confirm and later enucleate the cyst. If the cyst is secondary to a non vital or infected tooth, then tooth has to be treated along with enucleation and retrograde filling. Nothing to be panic, it looks more like a benign lesion. Consult a oral physician and get it ruled out.I hope my answer will help you, take care."
},
{
"id": 194659,
"tgt": "What is the treatment to get rid of masturbation?",
"src": "Patient: Hi,I've been trying to stop the habit of masturbation for almost two years but I could stop it for at most 5 days then I do it again. I think that the problem is with the thoughts that I cannot stop for more that a few days.I read online that some medicine like \"Prozac\" will be useful in my case but I'm not sure. I think that it would be embarrassing if I visited a physician for that purpose. I hope to find an answer here.Thank you. Doctor: Hello, Masturbation habit is a behavioural addiction and therefore you should go for behaviour therapy to reduce the frequency of masturbation. Hope I have answered your query. Let me know if I can assist you further. Take care Regards, Dr K. V. Anand, Psychologist"
},
{
"id": 163530,
"tgt": "How to interpret a baby s urine report?",
"src": "Patient: please intepret my 8 months old baby s urine full report. colour;pale yellow, appearance- slightly turbid, S.G.(Refractometer)=1028, ph-6, protein-nil, glucose-nil, ketone bodies-nil,bilirubin-nil,urobilinogen-normal amounts, pus cells-occasional/H.P.F,red cells NIl/H.P.F,Epithrlial cells +, casts- nil,crystals+uric acid Doctor: Hello,It is a normal urine report. Please do not worry about it.Hope I have answered your query. Let me know if I can assist you further.Regards,Dr. Sumanth Amperayani"
},
{
"id": 208569,
"tgt": "Suggest medication for panic attacks",
"src": "Patient: my 17 year old daughter was diagnosed with panic attack 3 months ago. she had strong palpitations, chest tightnes and discomfort and upper left chest pain. All seems to be gone except the pain. How long it will last? she is not under stress anymore; she s having pychotherapy sessions but geting so frustrated because she can t get back to her normal routine. Doctor: DearWe understand your concernsI went through your details. I suggest you not to worry much. In anxiety, the relief you are mentioning here is always superficial. They tend to come back if life style is not changed and learning atmosphere is not created. Your psychotherapist should take care of these. I hope he is doing a good job. He should also advice you about meditation and how to do it. If not ask him. Meditation also comes under psychotherapy care. Get it from your therapist. Don't change therapist. Your therapist know you better than anyone here. He shall do you good. Believe him.If you require more of my help in this aspect, Please post a direct question to me in this website. Make sure that you include every minute details possible. I shall prescribe the needed psychotherapy techniques which should help you cure your condition further.Hope this answers your query. Available for further clarifications.Good luck."
},
{
"id": 200431,
"tgt": "What causes purplish vein on the scrotum?",
"src": "Patient: Hello Dr. Rynne! I noticed two days ago, purple, very dark purple, but a thin vein at the skin of the scrotum - the left side at the top of the scrotum (not on the testicle). This vein has a length of one inch (max). Is this clearly the varicocele, or I should not worry? It does not hurt, does not itch, does not swell - I do not feel discomfort, apart from a slight stress. Please reply. Thank you and best regards! Doctor: Hello dear,Thank you for your contact to health care magic.I read and understand your concern. I am Dr Arun Tank answering your concern.No, its nothing to worry. Its not varicocele.If you tense the skin of the scrotum than vein becomes visible on the skin. It's normal phenomenon. Everyone has the same vein on scrotum.The reason you are not having any problem like itching, burning, swelling etc is because your vein is normal.In varicocele people feel dragging pain at the base of scrotum. Pain and swelling of which can increase with weight lifting. All this symptoms are absent in you so feel free you are not having varicocele.I will be happy to answer your further concern on bit.ly/DrArun.Thank you,Dr Arun TankInfectious diseases specialist,HCM"
},
{
"id": 101865,
"tgt": "Suggest treatment for pain and swollen abdomen when coughing",
"src": "Patient: i have a bad cold which aggravated my asthma. i have taken a lot of prednisolone and inhalers and still feel bad. i have a lot of pain across my abdomen now and it is swolen - i need to cough but its so painful that erverytim i cry. i dont know what else to do Doctor: HI, thanks for using healthcare magicPersistent coughing can cause chest or abdominal discomfort.If the coughing is persisting despite the use of your inhalors, it is possible that you need to see your doctor because you may need to be nebulised. This would be more effective than the use of your inhalors, sometimes these are not enough to resolve acute asthmatic event.You should consider visiting your doctor as soon as possible for an evaluation.I hope this helps"
},
{
"id": 70708,
"tgt": "Is sleep test required for breathing difficulty while walking?",
"src": "Patient: I have had breathing problems come up in the past month & 1/2 when doing much walking & small things. I have shortness of breath where I have to breath through my mouth to get enough air. I went to a heart Dr. & he said all sounds good then he had me go to a lung Dr. that now wants to have me do a sleep test. I only had about 5 of the sleep questions in minimal & that s only because I have a bad hip that wakes me up at night. I told her this but she still wants the sleep study. I want a second opinion, am I wrong ? Doctor: Hi, In my opinion, you should not get done sleep study first. Better to first rule out lung related pathologies for your symptoms. So consult pulmonologist and get done clinical examination of the respiratory system and PFT (Pulmonary Function Test). PFT will not only diagnose bronchitis but it will also tell you about the severity of the disease and treatment is based on severity only. You may need inhaled bronchodilators (formoterol or salmeterol) and inhaled corticosteroid (ICS) (budesonide or fluticasone). Hope I have answered your query. Let me know if I can assist you further. Regards, Dr. Kaushal Bhavsar, Pulmonologist"
},
{
"id": 151004,
"tgt": "Can an online doctor help understand the meaning of the spinal MRI report ?",
"src": "Patient: Plz tell me the meaning of mild posterior disc bulge at l2 l3level indenting the anterior thecal sac ,diffuse disc bulge with posters central disc postrusion and bilateral face gal anthropathy at l3 l4 level effacing the anterior thecal sac and compressing the bilateral existing nerve root ,diffuse disc bulge with posterocentral and right para central disc protrusion along with bilateral facetal anthropathy at l4 l5 level effacing anterior thecal sac with right lateral recess narrowing compressing the bilateral existing nerve roots R L This is MRI report of my mother 62 yrs of YYYY@YYYY Doctor: Hi, Thank you for posting your query. The MRI reports of your mother suggests disc bulges at the lower back (lumbar region) at multiple levels. The bulged discs are pressing on the nerves at the back. These nerves supply sensations and strength to the legs. As a result, your mother may experience pain in the lower back and legs. Tingling sensations, numbness and weakness of legs may also be present. Treatment is with physiotherapy and medications such as pregabalin. Best wishes, Dr Sudhir Kumar MD DM (Neurology) Senior Consultant Neurologist"
},
{
"id": 183667,
"tgt": "Was leaving the wisdom tooth root correct?",
"src": "Patient: my doc left a piece of my wisdom tooth root piece in my jaw after surgical extraction, she said it was difficult to retrieve and the process will be more harmful tham beneficial. the process itself was painful, i had pain in the opp side during the entire thing. now i have residual pain in my jaw, was she correct in leaving it behind? Doctor: Thanks for your query, I have gone through your query.The root piece can be left if it is sterile or not infected. If it is not near or in approximation to important anatomical structures like maxillary sinus or inferior alveolar nerve canal, then it can be left out like that. Consult a oral maxillofacial surgeon and take a radiograph to rule out the above said conditions and take opinion from the surgeon.I hope my answer will help you, take care."
},
{
"id": 81212,
"tgt": "What causes sharp and intermittent chest pains?",
"src": "Patient: I have a sharp pain in left chest at bottom of breast line that some times is on right side. It feels like gas Ihave had a ekg and that was good. I have a henita hernia small. I also takenexium for years. If I don t take it heartburn bad.this has sent me to emergency more than a few times over the years. This comes and goes I work out buy running an d riding my bike I am 52 years old and not over weight. Doctor: Thanks for your question on HCM.I can understand your problem.In my opinion you are having GERD (gastroesophageal reflux disease) associated chest pain mostly. No need to worry much for cardiac cause as your ECG is normal.For better control of GERD, along with drugs you need to follow these lifestyle modifications.1. Avoid hot and spicy food.2. Avoid stress and anxiety.3. Avoid large meals, instead take frequent small meals.4. Go for walk after meals.5. Keep 2-3 pillows under head in bed to avoid acid reflux. Small hiatal hernia can also cause poor control of GERD symptoms. So consult gastrosurgeon and if needed get done operation for hernia."
},
{
"id": 56410,
"tgt": "Suggest medication for a liver condition",
"src": "Patient: My liver profile (3rd one taken), my ast/sgt went from 60 u/l to 100 u/l , my alt/sgpt went from 100 u/l to 197 u/l, my alkaline phosphate 536 to 1015. What exactly does ast, alt and alkaline levels this high indicate? Im negative for hepatitis dr ran test for all of them. Levels are rising. Doctor: HelloHigh SGPT may indicate liver injury.It may be due to many reasons like hepatitis,alcohol intake,altered lipid profile,medicines,auto immune causes etc.Findings also suggest raised alkaline phosphatase.It is important to know bilirubin level.Raised alkaline phosphatse may be due to hepatobiliary obstruction.CT scan of abdomen can be done if needed.You may need few investigations like routine hemogram,random blood sugar,ultrasound of abdomen.I suggest tablet ursodeoxycholic acid 300 mg twice daily for three months to my patients.It helps in regeneration of liver cells.You may need to take few more medicines after investigations.Increased SGOT is non specific and it increases in many conditions.Get well soon.Take CareDr.Indu Bhushan"
},
{
"id": 223973,
"tgt": "Will taking nordette prevent pregnancy?",
"src": "Patient: Good morning, I had an unprotected sex last saturday at 1 pm and then i take 4 pills (nordette) at 8 pm and after 12 hours i follow it with another 4 pills. is is really effective in preventing pregnancy?the first day of my last period was May 26, 2014. Hope you can help me. Thank u Doctor: Hi,Welcome to Healthcare Magic.I am Dr Ramadevi Wani. I will be answering your concerns today.Nordette can be used as emergency contraception. You have taken them correctly. When used within 5 days of unprotected sex it reduces your chances of conception by 75%. There is a slim possibility of pregnancy due to method failure. Therefore get a pregnancy test done if you miss your period.In order to avoid such unnecessary stress, I suggest that you consult doctor and start regular contraception like birth control pills. They are more effective in preventing pregnancy than emergency contraception.If you feel that you are at risk of STDs, always use condoms during sex.I hope this is helpful.If you have further concerns, contact me through Healthcare Magic.Best wishes,Dr Rama"
},
{
"id": 224071,
"tgt": "What are the chances of pregnancy without ejaculating inside and what pill should be given?",
"src": "Patient: i and my gf had unprotected sex a day before, although i didnt ejaculated inside her nor she bleeded. so what are the chances of her getting pregnent. and what pill i should offer her to prevent it.? and how much pills i need to give her.. plz help asap. Doctor: Hello dearUnderstand your concernIf you had insert the penis in the vagina and preejaculation fluid can cause pregnancy as it contain sperm. But if you did not insert the penis in the vagina and ejaculate outside then no chance of pregnancy. In unprotected intercourse always there is possibility of pregnancy.There is no relation between bleeding after sex and pregnancy.For safer side, you can use I pill to prevent the pregnancy up to 72 hour after sexual intercourse. It should be take as early as possible to prevent the fertilization and ovulation.Only one pill is required to prevent pregnancy.In future use safe sex practice like condoms to prevent unwanted pregnancy.Hope this may help youBest regardsDr. Sagar"
},
{
"id": 141222,
"tgt": "What causes dizziness and restlessness?",
"src": "Patient: Hello? I am 36 years old female in ok health. I have cardiogenic neuropathic syncopy and a the largest pfo my cardiologist has seen. My cardiologist has dismissed me and let me go back to my family dr. I have not seen a pcp in about 4 years. I take no mess. I have been experiencing dizziness and lightheaded like I m going to pass out for about 48 hours now. I unfortunately am not able to just rest. I have been taking in a lot of fluids and feel better for about 1 hour then worse again. What is this and how do I get it to stop. I have also been having what I would describe as gallops. With my heart beats. And when I get excited or worked up it becomes much more pronounced. Doctor: Hello and Welcome to \u2018Ask A Doctor\u2019 service. I have reviewed your query and here is my advice. I must point out that if you have that large of a PFO you really must get that closed. You are at a significant risk for embolic stroke in the future. Other symptoms likely due to the mixing and shunting of deoxygenated blood into your arterial system which means your brain (and other organs) are running at suboptimal oxygenation. Something doesn't quite make sense in terms of the cardiologist telling you to just \"go back to your PCP.\" Whatever the other details are in your case. Your focus should be on getting that PFO repaired. Do that and you can look forward to many of your symptoms extinguishing. If not, then, as you get older things will likely accelerate and other complications may begin to happen. Such as stroke as I said before. Hope I have answered your query. Let me know if I can assist you further."
},
{
"id": 212512,
"tgt": "ECG showing inverted t-wave. Is there anything to be concerned? Severe panic attacks. Took radioactive iodine treatment for hyperthyroidism",
"src": "Patient: I ve been to the hospital twice in the past three days and all ecg s have shown an inverted t-wave...None of the doctors seem to be concerned (Although family doctor didn t even mention the inverted wave) and said the test was perfect. All doctors didnt seem to be too concerned??? I am a 30 year old female, dont smoke or drink heavily. 140 lbs 5 7 so I would say healthy weight. I ve been having horrible panic attacks the past few days which I what brought me there in the first place... I have been reading that they could be a sign on an upcoming heart attack which is making me panic further....Anyone have any thoughts on this? All my blood results eg, cardiac enzymes were all normal... Also, 4 months ago I received radioactive iodine treatment for hyperthyroidism so should be going hypo by now, could this affect the results? Now I feel like my life is ending at 30 :( I can t fathom leaving my baby girl! Doctor: Hello and welcome to Healthcare Magic. Thanks for your query. Non-specific T wave changes do not indicate a cardiac abnormality and you need not worry about this. You have mentioned that you have had multiple ECG, cardiac enzymes and evaluations by multiple doctors - who have all told you that you don't have a heart problem. Having treatment for hyperthyroidsm doesn't mean that you would become hypo-thyroid and so, this is unlikely to be affecting your results. I think that these excessive worries and fear of dying are due to your anxiety problem. Since your symptoms seem to be quite prominent and causing you this much of distress, I would advise you to seek professional help. Please see a psychiatrist for a detailed assessment and further treatment. Wish you all the best. Regards, Dr. Jonas Sundarakumar Consultant Psychiatrist"
},
{
"id": 67927,
"tgt": "What causes lump on chin following head collision?",
"src": "Patient: A couple months ago i had a head collision in my footy match. i ended up with a nasty bruise on my chin and on the inside of my lip. I went to the hospital and they said it was fine just bruising for a few weeks. However the bruising has gone i now have a lump on my chin that makes my smile crooked ! will it go away ? ? Doctor: Hi...thanks for asking question here in this forum....Most of the time after collision or trauma forigen bodies remains in the tissue...but in your case i think it was not an open injury....so its seems like there is accumulation of blood in the soft tissue(haematoma) which is hardend to give the feelings of lump...mostly this resolve in some time...even then if you want to consult the surgeon it will be good...thanksDr ihsan"
},
{
"id": 130083,
"tgt": "Why are my hips, buttocks and thigh areas hard to touch?",
"src": "Patient: The soft tissue in my hips, buttucks and thigh areas is hard to the touch. When i message it, it softens a bit. However, it feels extremely sore. Like a very bad bruise for days after. This has been going on for months. Not sure what it is but it's starting to worry me Doctor: Hi...Your stuffeness and soreness in your ower body is due to reduced stability and increased Stein on the lower body...Nothing to worry...It could be mainly because of improper recovery in the muscles...less hydration...less flexibility...I suggest you to kindly...Roll your lower body with a foam roller regularly..Do proper warming up and cool down when you are engaging in some physical activity..If possible do swimming once in a week..Drink ORS incase if you happen to engage in any physical activity...Started working on your lower body stability exercises...Do spine extensions to reduce load on the low back...Hope this is helpful for you..Revert back incase if you need further clarification..."
},
{
"id": 132526,
"tgt": "Suggest treatment to cure face swelling",
"src": "Patient: I was slapped on my cheek hardly which led to swelling..i applied ice and took combiflam to reduce pain..by the end of the day my swelling is reaching up my eye..i slept for about 2 hrs after taking pill..will it be ok by tmrw mrng..or do i need to consult a doctor in person.. Doctor: Hi..Welcome to HEALTHCARE MAGIC..I have gone through your query and can understand your concerns..As per your complain it seems that due to hard slap there has been traumatic injury to the soft tissues of face and it is leading to inflamed and swollen tissues and swelling is also due to accumulation of inflammatory exudate in the area..You need not to worry as the swelling will resolve gradually..You can continue taking anti-inflammatory painkiller like Ibuprofen or Naproxen to reduce inflammation and swelling..You can also continue doing cool compresses over the face and eye..In case if there is severe pain you can also apply numbing gel containing Lignocaine..If there is bruising that is a sigh of ruptured blood vessels you can apply Thrombophob gel over the face but keep away from eye..In case if the swelling does not resolve or you get increase in symptoms consult a General Physician and get evaluated and he can also advise you an x ray to check for any bony fracture and treat you accordingly..Hope this information helps..Thanks and regards.Dr.Honey Nandwani Arora."
},
{
"id": 85756,
"tgt": "How long must cerazette be taken to avoid pregnancy?",
"src": "Patient: hi i have been taking cerazette for 14 days , i am 52 yrs old and have not taken any other contraceptive pills for few years partner and i been using condoms, how long do i take this pop pill before i can stop using condoms as i do not want to chance pregnancy Doctor: Hello, After taking Cerazette regularly for seven days, you can stop using condoms as you can rely on Cerazette only. Hope I have answered your query. Let me know if I can assist you further. Take care. All the best. Regards, Dr. Prabhash Verma, General & Family Physician"
},
{
"id": 96877,
"tgt": "How to treat numbness and pain in leg after i fall down from stairs?",
"src": "Patient: Aloha, 50 year old female, 5ft 3inches tall, 140 lbs. I fell down the stairs in December 2010, couldn t walk - severe pain. I went to emergency and they did CAT Scan - Scan showed slight bulge in disc. Pain still persisted after 2 months - finally insurance approved MRI. MRI showed no bulge. Pain has gotten worse and my left leg is numb down to my toes. Should I go to a neurosurgeon? Not sure what to do? Doctor: Hello....you need to do repeat MRI scanning coz symptoms told by you are closely matching with disc prolapse....better if you could consult a ortho or a neurosurgeon"
},
{
"id": 22605,
"tgt": "What causes fast heartbeat and dizziness?",
"src": "Patient: over the last 3 months, every 3to 4 days my heart will beat very quickly and sometimes I feel as if I am dizzy. This lasts for maybe 10 seconds It has happened at night when I am sleeping and sometimes during the day. there is no pain involved, but I get flushed and walking around helps it to stop Doctor: Hello and welcome to \u2018Ask A Doctor\u2019 service. I have reviewed your query and here is my advice. The possibility of cardiac arrhythmia is there, most probably psvt or some other arrhythmia. You need evaluation for that, one ECG and holter monitoring will be needed. Depending upon the report, further treatment can be suggested. Also get your hemoglobin tested. Try to avoid activities like driving and other high risk activity till evaluation is complete.Hope I have answered your query. Let me know if I can assist you further.Regards, Dr. Sagar Makode"
},
{
"id": 34122,
"tgt": "What could a swollen, itchy lump on elbows with fever indicate?",
"src": "Patient: Hey there I have a red swollen lump on elbow and there is inflammation around the area it's very hot swollen and itcy I don't have any other anywhere else in my body I think it may be a bite of some sort , do u know what may be causing this , I have a very sick feeling and loss in appetite the past 2 days and this just appeared today can u help ? Doctor: Hello.You described typical clinical presentation of cellulitis (skin infection). You need antibiotics as soon as possiable. Untill you get propor treatment you can use ibuprofen for fever and pain, and put some cold (icey) towel on your elbow and lift your hand.Sooner treatment with antibiotics will help you to shorten pain and quicker recovery will be.Visit your doctor. All the best."
},
{
"id": 196797,
"tgt": "What could dark patch on scrotum be?",
"src": "Patient: I was with my boyfriend last night and noticed a small dark patch on the front of his scrotum. He didn't seem to notice it and I didn't ask him about it, it's never been there before. It's not red, more of a very dark ash color. Any ideas? Hes a healthy 25 year old if that helps. Thanks Doctor: Hi and welcome to Healthcaremagic. Thank you for your query. I am Dr. Rommstein, I understand your concerns and I will try to help you as much as I can.I dont think this is related to testicles and it sounds more likely to be dermatologic condition. So he should do dermatologic examination to see if there is dysplastic nevus or hiperpigmentation due to other skin disorders.I hope I have answered you query. If you have any further questions you can contact us in every time.Kindly regards. Wish you a good health."
},
{
"id": 82831,
"tgt": "Is fatigue normal with lupus?",
"src": "Patient: My husband is experiencing extreme fatigue that just started 3 weeks ago. Also, his blood pressure drops 30-40 points when he stands up. He does not have high blood pressure, diabetes, high Cholesterol, or high blood sugar. All cardiac tests have come back normal. Chest X-ray is normal. He does have lupus and celiac disease with non-diabetic peripheral neurothorpy and possible Sjorgen s Syndrome. (his mother had lupus and Sjorgen s Syndrome. He has been so weak and dizzy for the last three weeks that he just has to stay in bed all day. Any ideas? Doctor: Fatigue is one of the constitutional features of SLE .IT CAN BE THE PRESENTING SYMPTOM IN 80 % OF PATIENTS."
},
{
"id": 50823,
"tgt": "Undergone major operation, having only one kidney, unbearable pain during mensuration, irregular, uterus thick, bulky. Reason?",
"src": "Patient: Hi, my mother is aged 47 now. she has undergone a major operation and now having only one kidney.She often gets unbearable pain during her mensuration which is irregular and almost present for 15 days a month.Her uterus scan says her layer inside the uterus is thick and the uterus is bulky. What is the reason for this Doctor: Hello, and welcome to HCM! More details on your mother's medical history (what kind of major surgery, why was a kidney removed, has she ever had cancer before) would be necessary to truly answer your question. In general, heavy vaginal bleeding in a woman your mother's age is concerning for a few problems. With the uterus appearing \"thick\" on imaging, endometrial cancer and fibroids are certainly possibilities. Both of these problems can cause heavy bleeding. She should be evaluated as soon as possible by her doctor, who may decide to do further imaging or blood tests. I wish you good health, and thank you for posting on HCM!"
},
{
"id": 52983,
"tgt": "Suggest treatment to bring SGPT 161 iu/L and SGOT 163 iu/L to normal",
"src": "Patient: Hello Doctor, After my recent medical check up i foumd that my SGPT is161 iu/L and SGOT is163 iu/L.I used to drink till 1 and 1/2 months back.I would like to know about the treatment or precautions for faster recovery to normal levels. Regards SKR Doctor: Hi, I had gone through your question and understand your concerns.SGOT and SGPT are sensitive indicators of liver damage or injury from different types of diseases or conditions, and before any treatment the exact cause should be found. So the next step is to look for certain liver damage and most common cause is fatty liver disease. It should be confirmed by ultrasound. Other causes are viral hepatitis,cirrhosis, medications, alcohol intake or autoimmune diseases. To verify exact cause, US, bilirubin levels and tumor markers should be additionaly done.In every case, till you do these tests you should follow next:1. Avoid alcohol, smoking and hepatotoxic medications2. Avoid fatty, fried and high sugar diet3. Regulate your body mass4. Take Silimarin suppplementThese measures should protect your liver from further damage.If reversible cause if found then liver condition may be healed completely.Other causes should be treated regarding the exact pathology.Hope this answers your question. If you have additional questions or follow up questions then please do not hesitate in writing to us.Wishing you good health."
},
{
"id": 26461,
"tgt": "What are the side effects of having an open heart surgery?",
"src": "Patient: I was having chest pain periodically and my doctor wanted to do an angiogram but I asked him for a noninvasive procedure-- a Cat Scan of the heart. My doctor said that the results were inconclusive. Then he recommended an angiogram. He explained to me the possibility of having stents and if necessary, a few days later, open heart surgery. He is a 41 year old cardiologist at North Shore LIJ. in Manhasset, NY. He has also practiced as an internist. How do I find out how successful has he been with this type of procedure. If I need open heart, I think I want someone more experienced. How should I proceed with this problem? Should I get a second reading from a different radiologist? etc. Doctor: Hi,I would advise you to undergo an angiography. Only after that one can decide if there is a need for any intervention.If there is a single or two blocked arteries, then stent is recommended, if there are many blockages, surgery is indicated.After you have the results of angiogram, you can show it to another doctor and have a second opinion.So I would advise you not to worry, you will have the opportunity to receive the best treatment.Hope I could help youWishing you good healthIn case of further questions don't hesitate to askRegards,"
},
{
"id": 141410,
"tgt": "What causes head discomfort and dizziness?",
"src": "Patient: For a few months I\u2019ve been experiencing the feeling of something wriggling in my head/brain like a worm sometimes it hurts but since it started I\u2019ve had nothing but dizziness and waking up and feeling sick all day I wondered if you could help me figure out what it could be Doctor: Hello and Welcome to \u2018Ask A Doctor\u2019 service. I have reviewed your query and here is my advice. Several things would have to be known about your condition...your ORTHOSTATIC BLOOD PRESSURES, ELECTROLYTE LEVELS, HORMONES, LIVER and KIDNEY function tests, and CBC with differential. You may have an infectious process, a condition known as BPPV, or there could be a spontaneous CSF leak (common in people who wake up with severe headaches with and without spinning sensations). Hope I have answered your query. Let me know if I can assist you further."
},
{
"id": 119170,
"tgt": "Diagnosed with VDRL positive, took pencilin with same results. Should I take a higher dosage?",
"src": "Patient: sir, during blood test i was diagonised VDRL positive 1:16 later i have taken Pencilin4L for ten days and after that i have taken pencilin 12L now after 10 days when i have undergone same blood test the result is same as 1:16 on the same day after blood test i have taken one more 12L pencilin , wheather i need to take medicne agiain ? can u plz guide me i am confused Doctor: Hi and thanks for the query, VDRL measures antibodies and they do not get cleared out of the system immediately with reception of anti biotics. These antibiotics take time to be cleared and its but normal that the test result should be almost the same. Its advisable for you to repeat the test in 3 months, together with the TPHA test. Thanks and hope this helps, Best regards, Luchuo, MD."
},
{
"id": 5035,
"tgt": "Had tubular pregnancy, small fibroids. What are chances of getting pregnant? Recommendations?",
"src": "Patient: i am married 26 yrs..Its been one year i got married...Recently i was diagnosed with tubular pregnancy got treated with methotrexate and now After HSG test my tubes are normal but i have small fibroid which doctor and radiologist told it s normal and i can plan for baby and i can get pregnant....I am worried alot...To try for the second one please please advice me what has to be done......I am going for india alone next month for 3 months again i will be back after 3 months.... Doctor: Hi,Thanks for the query.As now your tubes are patent and normal you can plan for pregnancy now.Usually the effect of fibroids on fertility depends on the size and position of the fibroid.As you are having small sized fibroid that may not affect the fertility and pregnancy as told by your doctor.So you track your ovulation and plan unprotected intercourse around that period, which will increase the possibility of pregnancy.Regular antenatal checkups and ultrasound examinations are needed during pregnancy.For more details you can ask me through: http://www.healthcaremagic.com/doctors/dr-sree-gouri-sr/63429Take care."
},
{
"id": 161791,
"tgt": "Is Banocide syrup right medicine to treat allergic cough in a child?",
"src": "Patient: Hello Doctor, My daughter is 4 years old and is having cold .Doctor suggested to give her an antibiotic saying that she is having slight throat infection. Now her cold is better but has started slight cough not frequently from today. Again when i took her to the doctor he has given her Banocide syrup. Just wanted to confirm if Banocide syrup is used for the treatment of Allergic cough? Doctor: Hello, Banocide is an antiparasitic drug used for the prevent ion and treatment of worm infections. It may also be used for the prevention and treatment of worm infections which may cause breathing disorders. Hope I have answered your query. Let me know if I can assist you further. Take care Regards, Dr Gjustina Loloci, Allergist and Immunologist"
},
{
"id": 211091,
"tgt": "How to keep a manic depressive person unaffected from a depressed partner?",
"src": "Patient: I have a partner who I recently meant who has a drinking problem (former AAA recovered and stared again) who also has been seeing therapist for 20 yrs and suffers from manic depression constantly. They need to be alone daily and seem to deal with past trama from childhood and present . I also suffer from depression, and am wondering what do I need to do to not trigger there mood swings Doctor: hithanks for using healthcare magicI think in ur case best thing is, u know each other problems. First u should help in maintaining the treatment. That would prevent from aggravation of symptoms. Second try to maintain ur social life. Go for outing on weekend and try to spend time with each other as maximum as possible. Third any time u and ur partner feel comfortable, make physical relationship. try to follow all these stuff. can connect us whenever need help.Thanks"
},
{
"id": 106519,
"tgt": "How can chronic backache due to osteoporosis be treated?",
"src": "Patient: Hi Doc, my mom is 78 years old and she has chronic Osteoporosis back Pain. She cannot bend forward even for a few seconds. It has been over 15 years since she has this pain. She only gets some relief when she uses a back-pillow and sits down. She has constipation problem as well. She is allergic to penicillin. Is there any treatment? Doctor: Hi, You can consult an orthopedics and get an MRI scan to assess the extent and location of the disease. As of now she can take drugs like Tramadol for pain relief. Hope I have answered your query. Let me know if I can assist you further. Regards, Dr. Shinas Hussain"
},
{
"id": 77220,
"tgt": "Is replacing ceftum to Augmentin a good idea?",
"src": "Patient: My mother aged about 70 is a patient of chronic hypertension.She has now developed severe chest infection.In earlier years he responded well to ceftum 500.Now the dotctor has advised Augmentin1000 duo every 12 hours.Will it suit her or ceftum is safer and better? Doctor: Thanks for your question on Healthcare Magic.I can understand your concern. No, augmentin is having better coverage and superior than ceftum. Augmentin is having combination of Amoxicillin and clavulinic acid. Ceftum is having cefpofoxime. Combination with clavulinic acid makes augmentin better antibiotic. It is having higher bacterial coverage. It acts not only on gram positive bacteria but also on gram negative and anaerobic bacterias. So don't replace augmentin with ceftum. Augmentin will mostly suit her and treat her lung infection. Hope I have solved your query. I will be happy to help you further. Wishing good health to your mother. Thanks."
},
{
"id": 124836,
"tgt": "Need treatment for severe pain in the ribs and joints while breathing",
"src": "Patient: I am 48 years old,. I feel severe pain some time at ribs joint when breath in and out deeply. I am HLA B27 positive. I also feel pain back bone. I am residing at Ambala Haryana India. Please advise me how can I get the best and cheap treatment and which doctor should be consulted. Doctor: Hi, HLA B27 positive with pain over the backbone and on repiration is suggestive of an autoimmune disease called Ankylosing spondylitis. It is characterised by restriction in movement of ribs and backbones. The treatment is best given by a rheumatologist or a medical physician as well if you do not have access to a rheumatologist. The treatment is started once the diagnosis is confirmed based on a medical checkup and blood investigations and the medications have to be taken lifelong. It is important to take medications to prevent later complications like arthritis of the joint and to increase life expectancy. So better to start treatment for this early. Hope I have answered your query. Let me know if I can assist you further. Take care Regards, Dr Raunak, Orthopaedic Surgeon"
},
{
"id": 57782,
"tgt": "What to do for continuous stomach upset after removing gall bladder?",
"src": "Patient: I had my gallbladder removed over a year ago now but ailments I had before seem to have worsened.i can t socialize and eating is a big issue everything seems to upset my stomach and make me poorly over a 3-4 day period..also I have terrible acid on a daily basis..help! Doctor: GALL BLADDER DISEASE AND PEPTIC ULCER DISEASES ARE KNOWN TO CO- EXIST. SINCE YOU REMAIN SYMPTOMATIC EVEN AFTER GALL BLADDER REMOVAL CONSIDER UPPER GI ENDOSCOPY TO RULE OUT STOMACH/DUODENAL ULCERS. OTHERWISE IT COULD BE A SIMPLE CASE OF BILE REFLUX IN STOMACH CAUSING GASTRITIS WHICH CAN BE RELIEVED BY AVOIDING SPICY AND FATTY MEALS AND ADDITION OF ANTACIDS/PPI GP OF DRUGS IN CONSULTATION WITH YOUR DOCTOR."
},
{
"id": 222028,
"tgt": "What is the treatment for deceased fetus in twin pregnancy?",
"src": "Patient: My girlfriend is 37 Weeks pregnant with twins, and she told me this morning that she was in the hospital and that one of them has passed away, which is heartbreaking news to hear. However I feel as though I'm not getting the truth, because she claims they sent her home from the hospital just a short time later - still carrying a deceased fetus, and a living baby. Is that realistic that she would be sent home? Everything I've seen indicated that after a death of a twin at the very least she would remain under close medical supervision. Doctor: Yes You are right Its full term pregnancy so better is to terminate pregnancy otherwise it may affect another fertus."
},
{
"id": 52334,
"tgt": "What causes pain under the breasts while suffering from liver cirrhosis?",
"src": "Patient: my husband was dxd with Liver Cirrhosis 2 years ago. His liver is unchanged, Thank God, but he s having pain and a pulling/tightening sensation under his breasts that sometimes radiates to his sides. Endoscopies have been performed and at this time he has no varicies or ascites. He s taking Nexium but its not working...also has a cobble-stone appearing stomach as his Gastro dr has seen. What is the pulling and pain? Doctor: Hi, Most causes of pain in the breast are benign and not serious. There are a number of harmless causes for breast pain and tenderness that may primarily be related to changes in hormone levels. These can include menstruation and premenstrual syndrome (PMS), mastitis, menopause and fibrocystic breast tissue which is most common. Fibrocystic breasts are not necessarily linked to cancer, and the lumps are fluid filled cysts rather than a mass of cells. However, it is advisable to do breast ultrasound and mammography just to be sure and start regular follow up of possible cysts. In case of redness, fever and progressive pain this may indicate abscess and then it should be surgically drained or evacuated by needle under control of ultrasound. Hope I have answered your query. Let me know if I can assist you further. Regards, Dr. Ivan R. Rommstein, General Surgeon"
},
{
"id": 13002,
"tgt": "What is the treatment of burn marks ?",
"src": "Patient: I burnt my forehead a couple of years ago on holiday and a brown round raised bump appeared that I cannot seem to get rid off. I have been to my GP who prescribed hydrocortisone cream and although reluctant to use it I did as instructed. There has been no change and occasionally it bleeds when I dry my face. Doctor: Hello, I have gone through your query and would recommend you to apply the silicone-based gel on the affected area twice daily for 6 weeks. This should improve your mark. Hope I have answered your query. Let me know if I can assist you further. Take care Regards, Dr Asmeet Kaur Sawhney, Dermatologist"
},
{
"id": 157733,
"tgt": "Colon cancer, removed cancer down low close to rectum, cannot hold stools, loose bowels",
"src": "Patient: Hello Got a question. My wife had colon cancer, they removed the cancer down low close to the rectum . She worn a bag for almost a year, then then hooked her colon back up. The problem is because it was so close to her rectum she can t hold her stools and has a lot of very loose stools. Sometimes she goes to the bathroom about 10 times in a row. And after eating it gets bad, how long wills this continue and will it get better. Is there any advice you can give me, to correct her lose stools. She had her colon reversed and hooked back up a little over 6 weeks ago (Sep 5, 2013). Please help. Paul Doctor: Hi and welcome to HCM. thanks for the query. Certain abnoramlities in stool foramtion and defecation are expected after such surgeries and it should stabilize in next 4-6 months. All she can do at this point is to drink plenty of fluids and eat high fibre food,milk products,probiotics, fruit and vegetables. It is not matter if anastomosis was to close to rectum. It is just important than anals sphincter is saved. If yes, then she shouldnt have serious problem with stool incontinence. If symptoms persist she should do colonoscopy to rule out some other possible causes. Wish you good health. Regards"
},
{
"id": 154261,
"tgt": "Is cone shaped holes and multiple timorese sign of mesothelioma?",
"src": "Patient: My husband was a contractor most of his life and knows he was exposed to asbestos. He now has developed what the doctor is describing as cone shaped holes in his lungs as well as multiple Timorese the largest being 7 mm in the lower left lobe. It is too dangerous to biopsy. He has been labeled terminal. Could this be mesothelioma? Doctor: Hi, dearI have gone through your question. I can understand your concern. He has asbestos exposure and chances of asbestos related diseases are high. He may have mesothelioma or other asbestos related lung disease or carcinoma. He should go for biopsy of that lump. If needed go for immunohistochemistry study. It will give you exact diagnosis. Then you should take treatment accordingly. Hope I have answered your question, if you have doubt then I will be happy to answer. Thanks for using health care magic. Wish you a very good health."
},
{
"id": 12543,
"tgt": "Suffering from Guttache psoriasis, using cream, no relief. Does taking protein shake is the cause?",
"src": "Patient: hi stephen i am wanting some advice on how to treat guttache psoriasis ,my 20 year old son has had it now for three months and is using prescribed creams that have failed to work and the condition is getting worse. he is into fitness and works out all the time taking protine shakes to build muscle could this cause his condition? Doctor: Hi, Protein shakes do not cause or exacerbate guttate psoriasis. If its guttate psoriasis, a course of oral antibiotic erythromycin along with application of steroid creams and emollients will help. If these do not help much, discuss with your dermatologist the option of starting your son on methotrexate for better control of disease. Hope that helps! Take care."
},
{
"id": 210035,
"tgt": "Suggest treatment for depression",
"src": "Patient: Feeling very low at the moment I have been to c a doctor and has put me on venlafaxine plus go am taking mirtazapine also zopiclone to help me sleep .Today has been my worst day live even looked for a mental health hospital nearby .I hv been depressed before but that was over 22 years ago when I hv hd postnatal depression but I hd to hv electric shock treatment .Please can anyone help me Doctor: DearWe understand your concernsI went through your details. I suggest you not to worry much. Depression is a state of low mood and aversion to activity that can affect a person's thoughts, behavior, feelings and sense of well-being. Depressed mood is not always a psychiatric disorder. It may also be a normal reaction to certain life events, a symptom of some medical conditions, or a side effect of some drugs or medical treatments. This is what you should know. Depression need not be treated with medicines, in the first hand. It requires life style changes so that the mind acquires the capacity of coming out of the current negative feelings. These things are practiced in psychotherapy. Plenty of oxygen through deep breathing relaxation techniques, plenty of water and exercise are essential part of therapy which surely results in total cure.Please post a direct question to me in this website. Make sure that you include every minute details possible. I shall prescribe some psychotherapy techniques which should help you cure your condition further.Hope this answers your query. Available for further clarifications.Good luck."
},
{
"id": 4918,
"tgt": "Trying to conceive. Have regular period. Taking ova shield tablets. What is the cause?",
"src": "Patient: hi, I am 35, 5'5 and 60kgs, I am having one 10 years old daughter, earlier we are on planning but since 8 months we are trying to conceive but unluckily not. I am taking ova shield tablets from my 3rd day of period.i had regular period every month and my doctor had advised scanning and it comes normal. but then why I am unable to conceive plz help me.thanks. Doctor: Hello, Thanks for the query to H.C.M. Forum. No doubt you are a mother of 10 year child ,but at present your age is 35 years and so many ovarian follicles has completed their life . Now your are taking clomiphene + nutritional supplement ( ova shield) for stimulation of ovulation and in spite of 8 months you are not getting pregnant . In my opinion please take this as a fresh case. First of all get in endometrium test whether its thickness is able to receive fertilized ovum or not.Get in blood for L H /FSH ratio,Ultrasound of ovaries. If all tests normal then get in study of ovarian follicle ( whether they mature or not mature , if mature than they attain full size for ovulation and burst or not for release of ovum) . Hope I have answered your question and you understand all things. If further any question I will reply. Good luck. Dr. HET"
},
{
"id": 107033,
"tgt": "How can severe backache be treated?",
"src": "Patient: I have been prescribed Norco 7.5/325 for a few years. I have numerous back pain issues and this dosage does not help my pain. I m embarrassed and feel guilty for asking my pain management staff to increase my dosage as they didn t want to increase it from the 5 mg. As a pain patient, do I have a right to tell the Dr this dosage isn t strong enough, or do I tolerate what they want to prescribe? I often double up on this. Or is there a better pain med I could take with relief results? Doctor: hi sir/madam,Let me advice you for your problem.Low back ache may be a pre-monitory (prodromal) symptom in fistula in ano, sciatica, gouty arthritis, hernia, emaciation disorder etc. Pain is confined to lower part of the spine (back bone) especially lumbar region or lumbo-sacral area (rarely sacro-iliac region also). If it is secondary, earlier history of fall or injury may be associated. Rarely, radiating pain may be complained by the patient towards lower limbs. But it is quite common in low back aches if the defect is in the discs between vertebrae (back bones). Often the movements of lumbo-sacral region like flexion and rotation are hampered either partially or completely.Low back pain may be found in mild form in case of anemia, sciatica, rheumatoid arthritis, hemorrhoids, urinary calculi, uterine disorders etc also.Line of treatment as per Ayurveda:-The general principles of treatment of vata dosha are adopted in case of katishoola (low back pain). It includes various measures to suit its varied clinical entities, stages and associated complaints.1. Snehana (oleation) \u2013 by sneha dhara (pouring oil), abhyanga (oil massage), avagaha (tub bath with oil or oleaginous medicaments), kati basti (retaining medicaments on the back) etc.2. Swedana (sudation) \u2013avagaha sweda (sweating treatment with tub bath), pizichil (kayaseka), nadi sweda (sudation through a tubular device \u2013 local sudation), panda sweda (sudation through medicated paste or powder) etc.3. Mridu Samshodhana -mild purgation.4. Basti (medicated enema) like eranda basti, vaitarana basti, pippalyadianuvasana basti etc.Formulations indicated in Low back ache as per Ayurveda:-1. Dashamoola kwatha.2.Maharasnadi kashaya.3. Rasnaerandadi kashaya.4. Sahacharadi kashaya.5. Gandharvahastadi kashaya.6. Trayodashanga guggulu.And for local application these oils will help:-1. Ksheerabala tail.2. Mahanarayana tail.3. Dhanwantaram tail.4. Maha narayan tail.Avoid these for better results:-1.Bitter, astringent and pungent food2.Cold water3.Fear4.Exhaustion5.Standing6.Driving7.Cold food and beverages etc.Hope i was helpful.Have a healthy day."
},
{
"id": 118007,
"tgt": "What could cause a feeling of blowing off and tiredness?",
"src": "Patient: hi,I have had a bout of the runs on wednesday morning al day.Now its saturday and im blowing off really bad and still feel very tired,have eaten a spud and a tin of baby pears for tea.I am interested to know if i have a bug still in my gut? or is it just a tummmy bug still.I have never been this unwell for years?just lying around? Doctor: HIThank for asking to HCMI really appreciate your concern looking to the history given here I could say that this could be due tot he general weakness, you have not mentioned your age and gender this information was necessary here without this nothing can be said perfectly, if you have doubt for worm infestation then go get done the stool examination, take care and have a nice day."
},
{
"id": 124796,
"tgt": "What to do for the swelling and pain in my leg shin?",
"src": "Patient: Hi I feel of a caravan step and it hit me in the middle of my leg shin and side got massive bruise on it and swelled up and the step put dint in side of leg this was about 5 weeks ago but still swollen and still hurts me it has like lumps on it and still looks bruised Doctor: Hello, It could be a contusion probably. As of now, you can use analgesics/anti-inflammatory combination like aceclofenac/serratiopeptidase for symptomatic relief. You can also apply ice packs for faster recovery. Hope I have answered your query. Let me know if I can assist you further. Regards, Dr. Shinas Hussain, General & Family Physician"
},
{
"id": 60474,
"tgt": "My ggt level was 154, AST 39, ALT 64 and bilirubin 12. What went wrong with my result ?",
"src": "Patient: hi i did my blood test last december, end January and last month. in my last december lipid profile, my ggt level was 154, AST 39, ALT 64 and bilirubin 12, then after consuming essensial forte- liver supplement, for two months, my ggt level was 111, alt 64, ast 32 and bilirubin 21. then I kept taking essensial forte and lipitol as my cholesterol was also high - 6.1, the last month my ggt was 116, alt 62, ast 33, and bilirubin 18. what went wrong with my result. Doctor: Hi Welcome to HCM Read your prob n see your report. your report shows you has beeen sufferening from chronic liver disorder. so first you have to rule out the cause of it by getting USG, CT abdomen n some other test as per advise of your doc. so discuss it with your doc n start proper treatment. you should start syp liv-liv 10 ml three times a day for 5 month. Dr Aseem 9982583020 aseemadhuri@gmail.com"
},
{
"id": 101528,
"tgt": "Suggest methods to reduce asthmatic attacks",
"src": "Patient: Hi I am an asthma patient i have asthma since 23 years it is climatic and dust allergy i get severe attacks from june to march i have been inhalers i started taking nebulizers from 3 years earlier i used to have control for aweek once i take nebulization now i have to take everyday i take combimist for nebulization i use asthalin and budocort as inhalers please let me know what do i need to do to reduce the frequency of attacks My age is 55yrs Doctor: HelloAs you are an asthmatic patient and prone to allergy . I think you know very well to avoid , dust , mite , pollen , hay , fodder, fine dusting powder , humidity , sudden fall & rise of temperature because all these are predisposing ( triggering ) factors of asthma. Asthlin is a potent bronchodilator and safe and effective but budocort is a cortcosteroid ( many doctor like it but I never prescribe or advise except in emergency because of side effects).When such type of patient visit in my clinic I prescribe them to take monteleukast+ bambuterole tablet ( combo pack ) once or twice in day till complete relief .Hot coffee and steam inhalation are very safe and effective bronchodilator , so also try.For first few days you will require asthalin+ budocort , but after few days you will feel relaxed and dosage of asthalin+ budecort will subside ( I have tried in so many cases ).Hope this information will help you.Good luck."
},
{
"id": 216170,
"tgt": "Suggest remedy for pain at the base on my thumb",
"src": "Patient: hi ... i have pain in my left hand on the base of my thumb i belive it is called thenor ... i even have a hard time to pick anything up or open a bottle ... it started 4-5 month ago but i thought it is maybe a pinched nerve and will go away ,... but last couple pf days got more pain.... i work 7 days a week using my hand every day .. i have a cleaning business scrubb all day long... anything i can do ? i have put my hand in paraffin wax looks like the heat helps... Doctor: Dear patient what is your age? Pain at the base of thumb if happens in elderly patients above 60 years of age likely cause is 1st carpometacarpal joint arthritis. This is age related degenerative arthritis. If you are younger age likely cause of pain is dequervains tenosynovitis. In this condition there is inflammation of the synovial layer of first dorsal wrist compartment. Hot wax bath relieves pain in both conditions. Diagnosis can be confirmed on xray of the hand. If there is arthritis joint space reduction will be seen on xray. You need to consult hand specialist for detailed assessment .Meanwhile start tab ultracet twice a day for pain relief. continue hot wax bath."
},
{
"id": 30688,
"tgt": "What could vomiting and dizziness during strep infection suggest?",
"src": "Patient: My 7 year old daughter went into urgent care about 10 days ago and was diagnosed with strep and an ear infection. They gave her a 10 day course of amoxicilin and clavulnate potassium. She was supposed to finish her dosing today, but woke up at 1am vomiting. She has a temp of 102.3 and says she's dizzy when she gets up. Doctor: Thanks for posting your query to HCM>if child is not getting releif after antibiotic treatment it is suggestive of presence of resistant organism or some viras responsible for infection .you need to undergo microbiological examination to find out cause of infection under guidence of your clinician .presently symptomatic treatment using :1.Paracetamol 250ng two times a day.2.Poividine Iodine gurgle two times a day .medication taken under guidence of clinician .hope this information will be sufficient for your query .rate me if you are satisfied with my answer .feel free to ask."
},
{
"id": 171321,
"tgt": "Could breast feeding after consuming Nutrilite fiber effect the baby?",
"src": "Patient: My daughter is 2 and 1/2 years old but still she prefers my breast feeding always try to avoid solid foods.nowadays i am taking NUTRILITE FIBER for constipation. I wamt to know from your side that whether taking the nutrilite fiber will give some unwanted effects to my daughter since she most of the time consumes my breast milk ...? Thanks Doctor: Hi, it will not affect the child. Consumtion of nutrilite fiber in mother cannot affect the baby as the chemicals are not passed through milk. I hope this has helped you. Wishing your child good health. Take care."
},
{
"id": 34854,
"tgt": "What causes bumps in armpit with constant fever?",
"src": "Patient: Hi I was diagnosed with acute pharyngitis, since then I've been on augementin( for abt 2days) yet the fever doesn't subside, I just discovered 2 bumps under my armpit, I want to know if this is a reason to be concerned, or is it from the constant fever? Like I said I have been on augmentin for 2 days, 2x daily, and motrin 800mg 2x daily, what can I do? Doctor: Hi,Augmentin is good antibiotic and continue it.Any antibiotic takes sometime to work so wait for two days more and you will feel better.Having bumps in armpit also suggests some infection near by.This also will be cured with antibiotic.Take plenty of water.Avoid cold drinks and junk food.Do listerine gargles.Ok and take care."
},
{
"id": 207050,
"tgt": "Suggest remedy for severe mood swings and on and off depression",
"src": "Patient: Hi my Mother has been pretty depressed off and on for the past couple of years. Nearly two years ago, she lost her job at an advertising firm due to the economy. She found a new job as a hostess for a restaurant. She clearly is unhappy with her new job and feels a sense of worthlessness from this. Unfortunately, this also comes with major mood swings and extreme agitation over the smallest of issues. The only way to get someone help is to make them understand that they do need help. How do I get my mother to understand that she needs to seek help? Doctor: DearWe understand your concernsI went through your details. I suggest you not to worry much. Mood swings and depression are words which are supposed to be used clinically. Here the words are denoting just irritable mood / altering mood and disappointment which resulted in general irritable feeling. These common scenarios are easy to tackle. Your mother knows how to deal with those. Just tell her that her behavior is a bit weird. Also ask her friends to tell her the same. Once she realizes that her behavior is not correct, she will correct.If you require more of my help in this aspect, Please post a direct question to me in this URL. http://goo.gl/aYW2pR. Make sure that you include every minute details possible. I shall prescribe the needed psychotherapy techniques.Hope this answers your query. Available for further clarifications.Good luck."
},
{
"id": 216703,
"tgt": "Suggest treatment for pain in spine",
"src": "Patient: My Doc has informed me that Hydrocod Will now require a Triplicate RX, for my severe Spine pain. He has recommended a Morhone Pain Patch, yet currently has prescribed Tramadol, which I am not tolerating well. I am on Neurontin for nerve end pain and Cymbalta. I had no probs with Hydrocod, yet am experiencing side effects with the Tramadol. I am female, 68 years of age. What do you recommend? Doctor: hi,thank-you for providing the brief history of you.As the medicine are symptomatic relief for most spine cases and also medicines will be not good on long run as seen by me in the clinical practice, I recommend you to undergo physical therapy and regain the muscle strength and flexibility. By which the pain and aches will be reduced.Also, your age is 68, so you need a thorough body conditioning for which a Geritric rehabilitation will help to avoid much of the medicines due to aches and pains.RegardsJay Indravadan Patel"
},
{
"id": 91950,
"tgt": "What causes persistent stomach pain?",
"src": "Patient: i have pain my tummy all the time. and funny feeling. doctor suggested me to take painkilles , omeperasol and gaviscone. but painkilles make my pain worse. doctor mention it is golstone but he needs magnified scan. i do not know how long i shall wait? could you pls advise me what i should do? Doctor: Hi. If the painkillers are increasing the pain in abdomen, this can be due to acid-peptic disease. Continue with Omeprazole and gaviscone. Go for a bland diet. Avoid oily and spicy foods. Have frequent small meals to cover up the empty stomach , which increases the acidity. No painkillers at all. You do need an Ultrasonography to diagnose or rule out certain diseases . And more important you need is an Endoscopy of upper GI tract. You should not wait, do all these tests as soon as possible."
},
{
"id": 205341,
"tgt": "What causes echo in the head and jingling in the ears?",
"src": "Patient: What causes echoing in the head and jingling in the ears?.. I went to an Ear Nose and Throat doctor and he informed me that I had a middle ear thing going on and that I needed hearing aids ( I had informed him at the beginning that I wore hearing aids) Doctor: byou need. to change to change your ENT specialist...because tinnitus is usually due to middle ear pathologies"
},
{
"id": 39713,
"tgt": "Why am i getting yellowish stuff from private part?",
"src": "Patient: hi i pulled out a hair (i don't wanna say where yet) but it had this hard yellowish stuff all around it it from the tip to the root. it really creeped me out. could you tell me what that could be? I mean is it just a allergic reaction to soap or something? Doctor: Hello,Welcome to HCM,The history and symptoms suggest me that you are having the fungal infection of the genitals, which are the normal comensals of the genitals which will be kept under control by the helpful organisms whenever there is a imbalance in the environment it will lead to multiplication and growth of the fungus.I would suggest you to follow1.Keep the area clean and dry2.Topical antifungal cream3.Oral antihistaminesThese measures will help you to control the symptoms and improve the condition.Thank you."
},
{
"id": 172132,
"tgt": "What causes vision problems after being in pool?",
"src": "Patient: Our 5.5 years old boy was swimming in a pool today and suddenly got scared by not being able to see for a moment, saying: I can see only shapes...Sometimes he blinks a lot and rubs his eyes, saying that he can see everyting just fine and has no pain... Doctor: Hello. I just read through your question.Chlorine contained in pools can often cause visual disturbances when it comes in contact with the surface of the eyes. this effect resolves on it's own after a few hours and is nothing to be concerned about. It can be prevented by either wearing goggles or not opening the eyes while under water."
},
{
"id": 19873,
"tgt": "Suggest remedy for rapid heart heart beat,tiredness and lightheadedness",
"src": "Patient: Hey:) im 15 years old, female - im not overweight, i m about 5ft 4 - a little unfit but i ve never been the sporty type and this is very recent - my heart s been going all crazy in the past week or so - i first noticed it last friday. It felt kinda like adrenaline - i was just sitting there - nothing had happened to make me scared and i hadn t been running or anything - it doesn t hurt when my heart does this - it may not even b my heart ,sometimes it feels a little more to the left, like a lung or something - it ll just start going all fast and beating really hard for a couple of seconds. It s really scary. I get lightheaded a lot and im tired all the time (i ve been like that for a couple of years) and sometimes get sick when i m on my period - it s really freaking me out - hope you can help!! Thanks:) Doctor: Hi ThereAfter going through your query I understand your concern.I would like like to tell you that your rapid heart rate, lightheartedness & tiredness are not related to any heart diseases. It is your anxiety behind your symptoms. Its advisable for you to take axiolytic drugs for some specific time and for that personally consult with a Psychiatrist to get written prescription.Hope this will helpKind Regards"
},
{
"id": 109306,
"tgt": "Suggest medication for a large bruise on the back",
"src": "Patient: yound adolescent boy dress in soccer uniform. Complaining of pain in lower back. When examined he has a large bruise in the painful region of his back. His urine sample is tinged red. He has no fever, no other pain, and is otherwise quite healthy and normal. Doctor: HIWell come to HCMIf no other related symptoms are there then this could be due to some compression injury might have happened un-noticed, this would come around on its own if this dos not then this need to be investigated like blood test, x-ray would be advisable with clinical examination till then this need to be watch for any changes, take care have a nice day."
},
{
"id": 204221,
"tgt": "How can difficulty in focusing on a task be managed?",
"src": "Patient: I am inquiring about some medical help in regards to a problem that I ve had my entire life. I m not a fan of taking drugs but I seem to keep being told by my friends and family that I have always had trouble focussing and maintaining a conversation that I might need some medical help. I also started a new job that requires me to focus and I never really realized how much of a problem that I had until I was really put in a situation where I was forced to do it is there anything you can recommend? Doctor: Hello and Welcome to \u2018Ask A Doctor\u2019 service. I have reviewed your query and here is my advice. I can understand your problem. Start doing this simple exercise- take interval of fixed period after working for time period which is increased gradually. For example, work for 10 min then take a gap for 3 min and then again work for 10 min. On next day increase it to 15 min but keep the break time same. This will help you in improving your concentration. Hope I have answered your query. Let me know if I can assist you further."
},
{
"id": 144347,
"tgt": "Suggest treatment for back pain and numbness in feet",
"src": "Patient: I have sacrlaization and I have lower back pain with numbness in the soles of the feet, and I ve worked therapy sessions, such as electric, Altura Sound and interferential and some exercises like strength and streching , but the pain and numbness is still ongoing I want the relieve pain and numbeness? Doctor: hi,thanks for giving a brief history of you.kindly let us know how many days you have undergone the therapy session. also have you taken any MRI done or no? if not taking one will be advisable as based on that the focus of the physical therapist will be made.once the pain starts it's a build up over a period of time and at one time it starts giving numbrness and tingling.I have seen cases with the same issues, working out properly for 12 weeks I usually recommend to my patients and 100% of them recover. Exercise demands time and patience. It is the non invasive way of getting fixed.also you can consult your spine specialist for more discussion on the same, as along with medicine and physical therapy the recovery will be experienced.with the grace of God I wish you a speedy recovery"
},
{
"id": 205302,
"tgt": "What causes uncomfortable feelings?",
"src": "Patient: I m so confused all the time, I feel like each time things start going well it all turns to shit. At first I felt sad for a while(a month or two), then I started feeling angry but now I feel numb. I don t physically hurt myself in any way but I do mentally. When I have loads to do such as homework I choose to procrastinate not only because I m lazy but in my mind I think when I finish this I ll be back on track and things will be okay and I ll be happy but something about just stops me and says no you can t. Doctor: Hello, Your symptoms need more elaboration but the things you have put here if considered together then the inference is - you are suffering from moderate depression, which gradually affecting your working capacity. Being lazy - it might be your life style or a symptom of depression called Loss of energy and loss of concentration. Again when ever you decide to become happy after some time , your poor confidence (because of depression) you can't. Depression is a clinical condition because of neuro-chemical problems in the brain, and needs medicines to correct the biological changes in the brain. So, don't waste your crucial time in sufferings.. either go for treatment or see counselor at least. Set life goals and try to achieve them.All the best."
},
{
"id": 63695,
"tgt": "What causes fleshy colored bumps on my hands?",
"src": "Patient: I have many very small slightly raised flesh colored bumps on my hands. There s no itching. I have only noticed them recently so I do not know when they began but it was in the last year that I either got them or that they became much more numerous... Doctor: Hi,Dear,Thanks for the query to HCM. I studied your problem in depth and I understood your concerns.Cause of multiple fleshy red lumps on hands-These lumps seem to be skin papillomas due to herpes papilloma virus.History of 1 yrs to spread and develop in number corroborates HPV papilloma of the skin hands.I would suggest consultation with ER Surgeon.So don't build up wrong concepts and create more psychic complications in you which would increase risks and costs to you, but just ask a query to HCM and be comfortable to resolve your health issues.Welcome for any more query in this regard to HCM.Write good resume and Click thanks if you feel satisfied with my advise.Have a Good Day.Dr.Savaskar M.N."
},
{
"id": 32438,
"tgt": "Why does my streptococcus keep coming back?",
"src": "Patient: I got streptococcus after having a nissen fundaplacation and two hernias fixed. I also at the same time had to have both ovaries removed. My doctors can not get rid of it. i have been on all type of antibotics along with weekly B12 shots but it keeps coming back. Do you have any advise for me? Doctor: Hi, dearI have gone through your question. I can understand your concern.You have history of hernia surgery and now post operative infection. You are suffering from streptococcal infection. Your streptococcal infection may be resistant to routine antibiotics. You should go for culture and sensitivity testing. It will help to choose antibiotics. Consult your doctor and take treatment accordingly. Hope I have answered your question, if you have any doubts then contact me at bit.ly/Drsanghvihardik, I will be happy to answer you.Thanks for using health care magic.Wish you a very good health."
},
{
"id": 193424,
"tgt": "What causes erectile dysfunction at the age of 30?",
"src": "Patient: I've always felt that I was healthy sexual wise. But, over the past 2 years or so, I've come across a situation 3 times when I was about to have sex and couldn't maintain a erection. I've noticed that this has happen in the midst of me waking up in the middle of the night to have sex like around 3am. Is there something I can do about this? And I'm 30 years old by the way. Doctor: Hi, It can be due to stress full night work, improper or excess fatty food. If your issue persists, you can think of investigation for hormones. Regular exercise and proper diet can help you. Hope I have answered your query. Let me know if I can assist you further. Regards, Dr. S. R. Raveendran, Sexologist"
},
{
"id": 37287,
"tgt": "What is the treatment for phlegm in the nose and chest?",
"src": "Patient: On Thursday morning I started with a sore throat that got progressively worse all day. By night I had 101 fever, a running nose and the throat was so painful I couldn t speak. I treated myself with Alkazelter Plus (which usually dries me up) and Nasacort (which usually alleviates sore throats cause by my allergies) and Advil for fever and aches. By yesterday morning the sore throat was gone, but replaced by thick phlegm in my nose and chest. The fever was gone. Since yesterday morning I have been treating with Musinex, hot compresses over Vicks vapor rub on my chest and bed rest. My phlegm is yellow & darkish green, from nose and chest and is so hard to bring up that I ve irritated my throat again. I don t want to take antibiotics if I don t need to. What do I have and what should I take? Doctor: Hi, I understand your concern. You had self treated your symptoms of allergic sinusitis with lower respiratory tract infection with the usual drugs you take. But it seems it was a severe allergic episode this time,which was secondarily infected by bacterias/viruses. The occurrence of fever/thick green phlegm indicate deeper lung involvement. and in such condition you do need antibiotic . Ideally antibiotic is selected after physical examination, blood counts & sputum culture & sensitivity test( if needful). If you are raely taking antibiotics- you can try with- Tab Azithral 400 mg twice a day, along with tab. levocetrizine at bed time& cough syrup containing bromexin. For minimum 5 days. the duration of medication & antibiotics are better selected after assessing physical condition. So I would advise for consulting your doctor before you start with medicines. With it you can continue with rest, hot saline gargles, vicks vapor inhalation, light nourishing food, avoiding allergens / cold/sour eatables. Thanks."
},
{
"id": 104236,
"tgt": "Have sinusitis, blocked nose, painful throat, green phlegm. Should I take crestor?",
"src": "Patient: I am a 63 year old male.I ve been using CRESTOR for many years. Been truly suffering of sinusitus in a bad way. Two courses of antibiotics plus four cortizone tablets for four days, made NO difference. I still have a waterfall at back of throat during the day and a blocked nose at night with green phlegm when I try to blow my nose. I am pretty desperate to know the cause of my allergy . Awaits tests on dairy and gluten products, but what about CRESTOR? Doctor: THINK TO BE MILK ALLERGY AFTER TESTS YOU WILL FIND THE LINE TO FOLLOW GO FOR ELIMINATION BUT BEFORE TEST REPORT TAKE METROGYL 200 MG BD TAB FEXOFENADINE 120 MG BD SYP TOSSEX 1 TSF BD APPLY NEOSPORIN H EYE OINTMENT IN NOSE BD PUT SEA WATER 2 DROPS AT NIGHT EACH NOSTERL YOU CAN CNTINUE CRESTOR IF YOU FEEL RELIEF"
},
{
"id": 170257,
"tgt": "What causes recurring cold and sneezing in a child?",
"src": "Patient: My child is 4 years old .He frequently gets cold , sneezing with some less or more mucus.what is reason for it?He is taking bresol syrup now,iron,calcium and magnovit syrup.The problem gets severe in winter.Please tell the solution.Age:4 year,Height :38\" ,Weight:13.8 kg,Medical History;good, takes antibiotics treatment when the problem is severe, sometimes becomes healthy with some normal treatment and light medicine.He is healthy , active and has sharp brain. Doctor: HiWelcome to the HCMI understand your concerns but don't worry. Your son seems to be suffering from allergic tendency. I would recommend you to start him on montelukast once daily for 2-3 months. This will improve his allergic tendency.Hold the calcium, iron and magnovit syrups if your child is not having deficiency. A healthy diet and lifestyle is the best long-term health secret.Hopefully it will be of help.Take care"
},
{
"id": 66432,
"tgt": "What causes leg cramps and lumps at the back of leg?",
"src": "Patient: An 89 year old lady was taking 20mg of diutric, one-half 5 mg of potassium a couple of months ago. She stayed in a nursing home in Kansas for 6 weeks and had to switch to a Kansas doctor. She is now home but is still taking the same diuetric but a whole potassium. She is experiencing leg cramps and the back of her legs feel very lumpy. What can I do at home to help her? Doctor: HiThanks for your queryBased on your query, my opinion is as follows.1. Leg cramps is mostly related to electrolyte imbalance mainly low potassium.2. Since she is already on potassium supplements, you can get serum potassium levels checked to see if she is getting right dose.3. Lumps and cramps in elderly may also be because of varicose veins, which is quite common with sedentary life style.4. If there are too many prominent veins, then she needs to get a Doppler study for varicose veins.5. You can try using elastic stockings to reduce the swelling and improve venous drainage. Foot raise will also allow to decrease the swelling.Hope this helpsRegards"
},
{
"id": 50379,
"tgt": "Unprotected sex, nausea, back and pelvic pain. Family history of kidney conditions. Treatment?",
"src": "Patient: i have this throw up feeling when i wake up and sometime through out the day. i get sharp pain down in my lower abdomen and it sharp and it hurt really bad it low like by my pelvic . i have had unprotected sex about two weeks ago and we used the pull out method my period came it was a couple day late but it was regular should i be worried my family is know for having something wrong with there kidneys and because i have been having back pains. and im 19 if that means any thing Doctor: Hi, many thanks for the query!You need to do certain investigations- USG (Abdo-Pelvis), X-ray KUB,Sr. creat., Sr. BUN, Sr. Uric acid, Urine (Routine & Microscopy).You haven't mentioned about exactly what kidney conditions arethere in your family.For time being, you may take anti-spasmodics, pain killers, diureticswith your doctor's opinion.Drink plenty of water so that at least 2 litres of urine is voided in 24 hrs.Wish you a good health.Take care.Regards."
},
{
"id": 201326,
"tgt": "Suggest remedy for bleeding,itching and foul smell in groin area",
"src": "Patient: Hello, I am experiencing itching and burning pain in my groin area. My testicles and penis was once burring and itching. I thought I had jock itch so I purchased a cream for jock itch. Later, my testicles became scabbed and now they are bleeding and itching and there is a odor as well. Doctor: Hi,From history it seems that you might be having yeast infection and now super added with bacterial infection.Go for one antibiotic medicine course for 3-5 days.Take some antihistamine for itching. Clean local part with antiseptic lotion and apply anti-fungal cream locally.keep local hygiene clean, dry and airy.Ok and take care."
},
{
"id": 172821,
"tgt": "Is dijex sufficient after having swallowed dettol follwed by vomiting?",
"src": "Patient: Hi my son drank dettol and then he vomited out immediately. He also vomited out his meal which he ate 30 mins before that. Doctor gave dijex and he also have some swelling in mouth so he is not eating. He just took milk and now asleep. Is there anything to worry? Doctor: Hi, I had gone through your question and understand your concerns. You should be worried but not too much. With such presentations in my clinic, I would recommend absorbent Charcoal 1 tablet/10 kg for absorption of toxins during 3 days. Hope this answers your question. If you have additional questions or follow up questions then please do not hesitate in writing to us. I will be happy to answer your questions. Wishing your son good health."
},
{
"id": 178378,
"tgt": "How should dry cough with ear pain be treated?",
"src": "Patient: My 5.5yr daughter is having cough which sounds dry, for last 3-4 days. She doesnt have fever, she sometimes complains of pain in her right ear when she chews. In past she has been given relent or montain , somtiems montain LC. Can we give Relent 2.5ml twice a day for these symptoms? Doctor: HiThanks for writing to healthcare magic.Ears need to be examined to look for ear infection or bulging tympanic membrane.You can give relent but she may need more, including antibiotics an decongestantsGet her checked soon without delaying furtherWishing your child speedy recoveryRegardsDr Arun"
},
{
"id": 109308,
"tgt": "Suggest remedy for severe back pain after having c section",
"src": "Patient: Hello Doctor,Good Evening. My problem is Back ache. I am working as a Software Engineer and i need to work for 8 hrs in a day. Also i has a kid of 18 months . i had delivered the baby thru C-Section. When i lay on the bed after finishing all my works i feel lot of back ache also while waking up in the mrng i feel more.Again after an hour as usual i dont feel the pain. Please help me how to reduce this pain Doctor: First u have to take proper rest , take calcium diet , heat fermentation for reduced the pain and stiffness.. after relieve from pain and stiffness go for light exercises prescribed by physiotherapist..."
},
{
"id": 34368,
"tgt": "Can HCV be transmitted through a needle prick?",
"src": "Patient: Dear Sir, my mother is HCV + . She was given an IM injection and after 16 hours my wife has got a very little prick from IM injection. There was no blood on that injection. Can it is possible that my wife also get HCV from that . Moreover,after how much time , we should got her lab test on ICT Kite ? Doctor: HI, thanks for using healthcare magicIf the syringe was previously used then there could be small amount of blood on it that may not be easily seen.Since the syringe was accidentally stuck hours after, however this would significantly reduce the chance of any transmission.The window period of hepatitis c is 8 to 12 weeks.This means that a test can definitively detect the virus by this time and give an accurate result.I hope this helps"
},
{
"id": 169214,
"tgt": "What causes itching feeling in an infants scrotum?",
"src": "Patient: My son is 11 months old and has moderate eczema. Since my other children had it when they were young, we know how to keep it under control. But for the past 4 weeks, he has been frantically trying to itch his scrotum. There doesn t appear to be a rash there, so I initially tried an anti-fungal cream for a week, but his symptoms didn t seem to improve. I just changed him and I noticed blood in his diaper from all the scratching. I put aquaphor on it to provide a barrier between his scratches and urine for the night. But, this whole situation has me baffled! Any suggestions? Doctor: Hi...Thank you for consulting in Health Care magic. Skin conditions are best diagnosed only after seeing directly. I suggest you to upload photographs of the same on this website, so that I can guide you scientifically. Hope my answer was helpful for you. I am happy to help any time. Further clarifications and consultations on Health care magic are welcome. If you do not have any clarifications, you can close the discussion and rate the answer. Wish your kid good health.Dr. Sumanth MBBS., DCH., DNB (Paed).,"
},
{
"id": 159937,
"tgt": "What is the cause of pain after undergoing triple bypass for Carcinoma in pancreas ?",
"src": "Patient: hi Sir, My mummy having the problem in Carcinoma Pancreas and undergone triple bypass(HJ+GJ+JJ) at . After this operation, she is getting severe pain in the same area daily 3 times. Doctors suggested to take the Liv.52 and ultracet whenever there is pain. What should be the cause of this pain? Reply soon. Doctor: Hi Bharath, Carcinoma of pancreas can itself cause pain. In addition she has undergone triple by-pas, suggesting she had obstruction also. All these conditions including post surgical complications can cause pain. As suggested by her doctor, giving painkillers ( mild or strong depending on the severity) to take care of her pain is the only remedy available."
},
{
"id": 88043,
"tgt": "How can abdominal pain and soreness with bloating and flatulence?",
"src": "Patient: Hello, I have been experiencing abdominal pain and soreness all over my abdomen. Some days my upper left side is sore and other days it mimics appendicitis with lower right side pain and pressure in my upper right thigh. I also have alot of bloating and gas. I have two CT scans and several blood and urine test done and the only thing found was constipation. I increased my fiber and have been havinv regular bowel movements but the pain and bloating continue. My last colonoscopy was 5 years ago and nothing was found then. I do take prilosec for acid reflux. I recently spent some time at wilderness school where we drank mountain water that we treated with iodine and also I even eaten a lot of sushi in the past. Could my problem be parasitic? Any advice? Doctor: Hi.Thanks for your query and an elucidate history.Your history of drinking mountain water and lots of sushi may be indirectly related to the pain which you say is on left upper and right lower abdomen. These are sites classically involved in pain when one gets colitis. This can be due to waterborne diseases . May not be parasite. Since all the tests are normal except the colonoscopy not yet done ( last one was 5 years ago.).I would advise you to first take a course of an antibiotic, metronidazole, probiotic, activated charcoal tablets and anti-spasmodic whenever there is pain, continue Raniditidine..See the results, if you get well, nothing to be done further . If no relief you can go for colonoscopy and to see what exactly is the problem."
},
{
"id": 212095,
"tgt": "Tripping feeling after watching a bright colored fairy batting wings on TV. Help",
"src": "Patient: Hey, please help no one knows the answer to this, but i was laying in bed watching TV when a scene came on with a bright colored fairy/pixie batting its wings very fast and it didnt send me into an epileptic seizure or anything i dont think but i literally started \"tripping\" as you would on acid though I was completely sober. Help is appreciated, Thank you. Doctor: Hi there ~I understand your concern, however I wonder if you have been taking any drugs lately or have recently quit substances. Withdrawals can sometimes make you see things that are not there - visual hallucinations. Also your electrolytes may be the reason you are having these problems. A visit to a psychiatrist should help clear your problems. I hope you do so as soon as you can. Take care and have a lovely day!"
},
{
"id": 35727,
"tgt": "Is bruising with little bleeding after fall on rusty pipe serious?",
"src": "Patient: I fell on my butt, about waist height on a round rusty pipe, it was not sharp, but it bruised me thru my shorts. I saw a little blood on the bruise, will that cause blood poisoning? I ONLY had apple cider vinegar, so I washed it with warm water, will that help? What should I do? Doctor: Hello dear,Thank you for your contact to health care magic.I read and understand your concern. I am Dr Arun Tank answering your concern.Rusty metals can contains the spores of teatanus bacilli. This can cause a serious tetanus.Though the chance is very minimal because the wound is not closed one it is actually a open one.I advice you to take tetanus toxoid injection which will protect you against the tetanus.I suggest you to take the due care of the wound. Please do regular washing and dressing of the same. You can use betadine and polysporin ointment over it.I will be happy to answer your further concern on bit.ly/DrArun.Thank you,Dr Arun TankInfectious diseases specialist,HCM."
},
{
"id": 163041,
"tgt": "Suggest treatment for split toenail of an 8 months old",
"src": "Patient: Hello, My 8 month old son s big toenail is split pretty far down toward the middle. I clipped some of it off so it wouldn t get caught on anything. I also applied Neosporin to it before putting his sock on. Is there anything else I should be doing? He is a big time crawler and climber. I think he must of snagged his nail doing one of these things. Thanks for the help! Doctor: Hello and Welcome to \u2018Ask A Doctor\u2019 service. I have reviewed your query and here is my advice. Fortunately healing of infant is very quick. If it is not infected then put a band aid and soc, so that he doesn't pull it or get hit on it again. Change band-aid daily or if needed more time s as child may get it wet. Keep a watch for signs of infection, if it occurs go to a doctor. Otherwise it should heal in a few days. Keep cutting growing nail. Hope I have answered your query. Let me know if I can assist you further."
},
{
"id": 26764,
"tgt": "In stressful situations, what is the average heart rate?",
"src": "Patient: In stressful situations, what is the average heart rate. I had to do an activity where I put myself in a stressful situation (rush hour traffic) and my heart rate fluctuated a lot, is this normal? It began at 92, peaked at 141 after 27 minutes, and then reached 116 at 51 minutues. Doctor: Hello!Welcome and thank you for asking on HCM!I understand your concern, and would like to explain that your heart rate seems to be perfectly normal for physical exercise. You should know that during physical exercise the levels of catecholamines (adrenaline and noradrenaline) raise in your blood, leading to tachycardia and even ectopic heart beats. These levels fluctuate during physical exercise (the balance between the production and metabolisation) , leading to modifications of the heart rate. But in your it seems to be everything within the normal limits. So there is nothing to worry about. Hope to have been helpful!Best wishes, Dr. Iliri"
},
{
"id": 105268,
"tgt": "Allergic dry cough, relapse while playing with pet dog. Treatment?",
"src": "Patient: My son is 5 years old and suffering from allergic dry cough. some times before, his doctor advised him inhaler and we continued it for almost two years. But after 2 long swimming sessions, his problem subdued. Two days back, he played for 5 minutes with a pet dog near our house. and since then his cough relapsed. today, he is coughing continuously. Though we tried a cough syrup but in no use. please suggest something that can be useful. Doctor: child is allergic and allergy increses with dust pets you can start tab montair 5 mgmchewable tab bd syp allegra 30 mgm daily twice a day syp tossex 1/2 tsf bd and syp ventrolin 1/2 t6sf bd do 2 week get allery tests done and go for cure"
},
{
"id": 170296,
"tgt": "What causes stomach ache and flu?",
"src": "Patient: Hello,My name is izwin.Me n my 3 boys has been down with flu n cough for 3weeks now. we have been going on the second set antibiotics since yesterday.My main concern is that my 8 yr old and 6 yr old has been having stomach ache vomiting and they wont eat. what do i do? from what i gather is that they r having too much meds to cope with. Doctor: HiWelcome to the HCMI understand your concerns but don't worry. The most likely reason for these symptoms is a viral illness which easily spread amongst children. They do not need antibiotics unless and until blood or culture reports suggest a bacterial infection.Repeated antibiotics should be avoided. The need to development of gastritis, indigestion, antibiotic resistance. I would recommend a symptomatic management including antihistamine such as Chlorpheniramine or fexofenadine along with antipyretics.Give Lansoprazole for stomachache and associated vomiting. Hopefully this will help you.Take care"
},
{
"id": 89543,
"tgt": "What causes pain and bloating of stomach?",
"src": "Patient: hey, i m a 16 year old girl and for about a year i have been experiencing a lot of discomfort. My lower abdomen area is very tight and i feel bloated constantly, that area makes a loud rumberling noise and also a whining noise and it feels sometimes that my bum makes these noises aswell. its like a feeling that i need to fart but i cannot. i m also constipated nearly all the time and also have lower back pain near the kindneys on both sides this is at its worse in the mornings when i have just gotten up and then goes, but sometimes it can be a light ache throughout the day. i ve also noticed i ve constantly got bad breath i will brush my teeth and about 5 minutes later my bad breath will come back(this is ruining my confidence and i m becoming more and more seclusive because i m scared to talk to people in case they smell it.) I also have itches on my thighs and arms usally late at night. it would be really great if you could help me because this is taking over my life and i m getting really depressed about it. I can t concentrate in class because i am contantly worring that my stomache/abdomen is going to make noises. Doctor: HI.Thanks for an elucidate history. All these symptoms are suggestive of Entero-colitis . For the causative reasons to be found undergo tests like colonoscopy, Contrast enhanced CT scan of the abdomen , routine blood , urine and stool tests. Consult a Gastroenterologist to get a proper clinical examination and medical treatment . In such conditions at your age anxiety and stress have a larger part to play , and it has to be well controlled by you at any cost and see the difference ."
},
{
"id": 4535,
"tgt": "Is it possible that the kicking sensation in stomach is pregnancy after having a partial hysterectomy?",
"src": "Patient: I had a partial hysterectomy many years ago.....I started feeling kicking in my stomach 3 days ago.....it is pretty consistent, I took a pregnancy test and it showed negative....I'm having a small burning sensation right now with kicking....is there a possibility I could be pregnant....what should my next move be? Doctor: Hello,Without a uterus, pregnancy isnt possible. Its unlikely you are pregnant. Get an ultrasound done asap."
},
{
"id": 193316,
"tgt": "What are the side effects of masturbation?",
"src": "Patient: Hi, may I answer your health queries right now ? Please type your query here...I used to shake pennies daily. I started this habit at the age of 14 now I am 24 years old. I need to know will I get any problem in sex and getting baby. please help me. Doctor: Hello, There are no side effects for masturbation if you are doing it moderately. Moderate frequency is three or four times a week. Excess masturbation and rough masturbation can damage the skin receptors and thereby could cause numbness and thereby deficit in sense. It can also cause anxiety and performance anxiety and related depression. The problems can be cured. I suggest sex therapy for you. Hope I have answered your query. Let me know if I can assist you further. Regards, Dr. K. V. Anand, Psychologist"
},
{
"id": 123912,
"tgt": "What causes tightness above the collar bone?",
"src": "Patient: Yes I have had a continual tightness above my collar bone for almost two months now....it does not hurt but is just an uncomfortable feeling and the lymph nodes in that same area have been inflamed. This is only taking place on my left side and I am not sure what step I should take. Doctor: Hello, Pain above the collar bone may be due to neck muscle spasm or Infections or pleurisy of lung or malignancy of stomach with trossiers sign etc. Get examined by surgeon for lung infection or lymph node biopsy then get treated accordingly. Hope I have answered your query. Let me know if I can assist you further. Take care Regards, Dr Penchila Prasad Kandikattu , Internal Medicine Specialist"
},
{
"id": 163520,
"tgt": "What causes rash all over body?",
"src": "Patient: my two year old daughter has a very itchy rash that started on her legs - medium sized red raised bumps - over 24 hours it spread all over her body and some soars on the roof of her mouth - no soars on bottoms of feet or palms - the day before the rash she had a fever of 104 and the day she got the rash still fever of 104 - now its the third day and no more fever but very fussy and wont stop itching - doesn' t want to drink anything - her mouth hurts - the rash is getting redder and keeps spreading - now they have what looks to be black dots in the middle of them - just wondering if anyone had any ideas Doctor: Hello,Skin conditions are best diagnosed only after seeing directly. I suggest you upload photographs of the same on this website so that I can guide you scientifically. Hope I have answered your query. Let me know if I can assist you further.Regards, Dr. Sumanth Amperayani"
},
{
"id": 207764,
"tgt": "How to get rid of mental stress?",
"src": "Patient: I am a male. Had a protected vaginal sex but unprotected insertive oral sex with female SW. Tested negative on hiv combo test (antigen antibody) , equipment Hiv duo ultra by Vidas and CMIA by Abbott, on post 3 week exposour, 4 week, 5 week, 6 week. All came negative. What to do now. Is there any further kind of test to remove severe mental stress? Doctor: HiI understand your concern and stress.Relax dear.You have done enough test to rule out HIV.These test negative means nothing to worry.If still have thought then go for HIV DNA PCR study which has high confirmation value and if it will come negative then relax.Do deep breath exercise.Relaxation and meditations.Try to motivate yourself and do pleasurable activities.Proper sleep and diet.See positively and be confident.Avoid such high risk sex to avoid such stress.Be careful and be safe.Thank you."
},
{
"id": 18772,
"tgt": "Suggest remedy for high BP",
"src": "Patient: Yes I am a 38 ye old female who was on Corey for 6 months after a heart event( doctor wont call it attack cause hearts normal and no blockages I basically overdosed on a pain killer accidentally ) I was taken off Corey 10 days ago and now my blood pressure reads 148 over 50 will it get better? I exercise and eat vegetarian. Doctor: Hello and Welcome to \u2018Ask A Doctor\u2019 service. I have reviewed your query and here is my advice. I understand your concern and would explain that your blood pressure values are above the normal ranges (the systoli blood pressure ). For this reason, I would recommend reducing salt and caffeine intake and closely monitor your blood pressure, frequently during the day. If these blood pressure values persist, I would recommend restarting Coreg. Some blood lab tests are necessary too: - Complete blood count for anemia - Thyroid hormone levels for thyroid gland dysfunction - Kidney and liver function tests - Fasting glucose - Blood electrolytes. Hope you will find this answer helpful! Wishing all the best, Dr. Iliri"
},
{
"id": 117705,
"tgt": "What can i do for bleeding of haemorrhoid after dropping of platelet count due to intake of nplate?",
"src": "Patient: I have had ITP chronic for almost two years. i also have Prostate Cancer and had 27 external beam radiation treatments. I was treated with predsione which did not work ,then given rubituxen . \"Forgive my spelling\" it work only a short time. I am now on Nplate which seems to work. My question is Nplate will bring my count up to at times over 300,000 then in 3or 4 days drop to 150 then in one or two days drop to the 30's. The trouble is i have internal Hemerroids which bleed when count drops. I went to emergency and they gave me platelets. which stoped the bleeding then got a nplate shot . My Hematologist told me not to go to hte emergency anymore and get Platelets because it does more harm then good in the long run. my HGB stays normal from 13.3 to 14.3 and all others are with in accable range. Is there anything else I can try. My dr. wnts me to have my spleen removed ,I am 69 yrs. and not confortable with doing that except as a last reasort. what is you take on my situation? Doctor: Hi,Thanks for asking.Based on your clinical history and query, my opinion is as follows:1. Internal bleeding around hemorrhoids is OK. But if it were CNS bleeding, it can cause severe complications.2. Platelet count, if it falling due to spleen induced platelet destruction, then spleen needs to be removed, as spleen function is reduced due to continuous ongoing damage. 3. If you get the spleen removed, get yourself vaccinated.I would also advise spleen removal, if it is the main cause of low platelets. Antibody testing will confirm ITP. Any further queries, happy to help again."
},
{
"id": 20649,
"tgt": "Can exercises be performed in chronic A-fib?",
"src": "Patient: I am 70 years old and suffered a stroke aprox 18 months ago after being diagnosed with chronic afib. I am taking Coumadin and Toprol XL as medications at present. My question is related to exercise. Can I exercise with this condition. My at rest heartrate is usually between 95 and 110 beats per min. I have been reluctant to exercise because I fear my heart rate will increase to a very high rate. I cannot seem to get a definitive answer to this question. Can you help? Doctor: Hello Welcome to HCM In chronic atrial fibrillation important thing is to keep the ventricular rate under control. You heart rate seems to be well under control with the toprolol. Exercise can increase the heart rate. So strwnous exercises should be avoided. You can do do daily brisk walks and can walk on treadmill. You can keep a pulse oxymeter to monitor the heart rate. It should not go above 120 per minute. You can do short duration exercise with five minute intervals to allow heart rate to settle in between . Wishing you good health. Regards."
},
{
"id": 117841,
"tgt": "What causes low blood platelets?",
"src": "Patient: My stepsons daughter was just born, she has the horseshoe kidney anomaly and low blood platelets, the Dr. said that he thinks it is Turner syndrome, but she doesn't physically have the symptoms of that, I have read many disorders via the web, and I'm wondering how long it will take a lab to diagnose a chomosome disorder and I am wondering if the low platelets could just be due to the kidney anomaly? I am afraid they are jumping to conclusions and are terrifying the parents. Please help... Doctor: Hi, she may have turner syndrom but pletelets has no any relation with horseshoe kidney. Platelet count is low in many cases in the neonate. It the common condition. Don't worry regarding that. However go for the diagnosis of chromosomal abnormality. Consult youe doctor and diagnose that.and then take treatment accordingly. No need to worry."
},
{
"id": 208762,
"tgt": "How does one improve self esteem and confidence?",
"src": "Patient: Im not sure whats wrong with me. Hello there, firstly I am just wondering what could be stopping my voice from breaking. Im a 20 year old male. Moving on, Some symptoms I have are. 1.I cant feel my adams apple 2.argueing with parents for no reason 3.Always feeling low about myself/my impression 4. Lost my sense of humour 5. No self confidence Any idea on what could be causing all ofu this? Thank you. Doctor: DearWe understand your concernsI went through your details. I suggest you not to worry much. The answer to all these questions are just the same. Your negative thinking. You are just 20. At this age you are supposed to be thinking about your education and career and future. instead you are thinking about health, family, state of mind etc. This means you are not doing anything on your education, career or future side. Concentrate on your career and future. all these problems shall be solved.If you require more of my help in this aspect, Please post a direct question to me in this website. Make sure that you include every minute details possible. I shall prescribe the needed psychotherapy techniques which should help you cure your condition further.Hope this answers your query. Available for further clarifications.Good luck."
},
{
"id": 217863,
"tgt": "Suggest treatment for pain in neck and blurred vision after an injury",
"src": "Patient: GOT HIT IN BACK OF NECK WITH STEEL PIPE AT WORK 15PDS FELL ABOUT 5 FT SORT OF BRUISED UP DOC TOOK X RAY SAYS STRAIN AND BRUISE KEPT ME OUT OF WORK FOR A WEEK RELEASED ON TUESDAY HOWEVER AM SEEING DOUBLE VISION ALOT OF PAIN STILL AND AM ONLY 38 MALE WHO LOVES TO WORK BUT THIS IS HURTING SO BAD DOWN MY BACK Doctor: The pipe could have struck you on the back of the head also, which could have resulted in a concussion in addition to the neck strain and bruise (contusion). You should seek a course of physical therapy from your occupational medicine doctor, as well as pain medications (such as hydrocodone, tramadol, or nucynta) and a muscle relaxant (flexeril, tizanidine, or orphenadrine)."
},
{
"id": 170863,
"tgt": "What is the treatment for rhinovirus infection in a baby?",
"src": "Patient: my 8 month old son was diagnosed with Rhinovirus 2 weeks ago. He started getting a very nasty cough and then after a few days vomiting very large amounts and loosing weight, loss of appetite and not wanting to drink anything. he was admitted to hospital 3 days ago and yesterday morning i found a few little lumps on the back of his ears. i asked the dr and he said, its nothing don t worry. They left, half hour later i changed his nappy and it was all over his torso, it has now gone down his left and arms and have raised circular lumps, which is aggrevated by heat. he had a temp but its staying at 37.6 now. They said it is just a viral rash from his rhinovirus but it is getting worse and its now gone round his mouth. Doctor: Hello, I understand your concern. I would like to know a few more details before coming to a conclusion.1. On which day of fever did the rash appear?2. How is he feeding now?3. Is he irritable or playful and happy?4.Has he taken vaccine shots for measles and rubella?A picture of the rash would be very useful in diagnosing his infection.Rhinoviruses produce runny nose and cough, but are not known to produce a rash.Do get back with the above information and we can come to a conclusion."
},
{
"id": 187686,
"tgt": "What is the cause of dry mouth, excessive thirst, and difficulty taking deep breaths?",
"src": "Patient: hi, just wanting to know what these symptoms may mean, they have just started over the last week or so..dry mouth/very thirsty, feeling of not being able to take a deep enough breath in & needing too yawn to get one in, especially after eating, also a feeling of when bretahing in like there is resistance..just had blood test, my fasting bsl was 4.5, everything else was fine except low ferritin of 13, but this is usual for me & have had low levels for at least the last 5 yrs. I am 36, also have mild asthma, but have taken inhalers for it thinking it may be asthma but doesn't seem to a difference..I am also am a RN so have been trying to rack my brain over it Doctor: Hi,Thanks for asking the query, Dry mouth occurs due to various reasons few are neurological ,GIT disturbances, nutritional deficiency, Less water intake, salivary gland disorder.A thorough checkup by the Dentist is required.Take plenty of water to keep yourself hydrated.At home take lukewarm saline rinses.Take care!"
},
{
"id": 97712,
"tgt": "Is valium and valerian root the same thing?",
"src": "Patient: Yes. Can you tell me is valium and valerian root the same thing? I was recently told by a Doctor thatit is the same thing, that really shocked me because in my limited knowledge of herbal substances Idid not think these two were anywhere nearly related. Doctor: hello.i read your query.valium (diazepam) is and valerian are two different substances.although they are used for same indication of anxiety and insomnia.and valerian is thought to share the same mechanism of action with valium i.e via GABA receptors.hence drug interaction.hope my answer is helpful to you in understanding the difference and similirity between the two."
},
{
"id": 151579,
"tgt": "Suffering from spinal spondylolisthesis. Feels painful. Should I go for surgery?",
"src": "Patient: Dear Doctor, I am suffering from spinal spondylolisthesis of lumbar L5 over S1 from last ten years but I was running fine. from 15days back I feel very painful and contact with doctors in our country Bangladesh. some of one said, it is going to static position, so you need not to go for surgery and other doctor said you go for surgery. It is so risky operation, may be successful otherwise your position will deteriorate. so,please give me suggestion what to do. Best regards. Monzur Elahi khan Dhaka Bangladesh Doctor: Dear Monzur, If the pain is localised or with/without radiation to some extent then this can be managed conservatively with Physiotherapy including some electrotherapy modalities , pelvic traction, manual therapy , corrective exercises along with some nerve vitamin tablets..... One assistive devise such as \"lumbo sacral brace\" will be mor useful in this case. Once pain is reduced you can join core strengthening classes only under a supervision of a certified physiotherapist who can design exercises specifically for you as per your condition..... If there is persisting weakness in your legs, increasing gradually and showing sluggish reflexes (Deep tendon reflexes) then this represents motor weakness and you need to be ready for a surgical procedure....Although this surgery is a risky and expensive one these days latest technologies in an operation theater (OT) have already introduced to reduce the risk factors..... So wish you a good health soon....."
},
{
"id": 90259,
"tgt": "How to cure pain on the lower side?",
"src": "Patient: I have been have pain in my lower right side and it's been hurting sometime now. I had noticed when I used the bathroom I had a spot on my stool. Im hurting now and my doctor don't want to take no pain pill cause I may start bleeding and not knowing it..... I need to stop this pain,,,, what shall I do Doctor: Hi.Thanks for your query.First of all you have to get a proper diagnosis by having a clinical/ physical examination done by a Surgeon and a Gynecologist. Undergo blood, urine and stool examination. Ultrasonography will tell whether you have appendicitis or such problems on the right side. The diagnosis will tell us the treatment plan.In the meantime get started on an antibiotic and supportive medicines,. There are medicines who have no chance to start any bleeding like dicyclomin."
},
{
"id": 19703,
"tgt": "What causes a rapid heart beat?",
"src": "Patient: Hi I'm 15 and I'm a girl. A few months ago during a flu that I've got my heart started beating really fast and continued for a day or two. I've been told that it's normal. But after that these episodes started coming back for no reason and sometimes I get cramps around the area of my heart for no reason as well. Just today my heart started beating fast and hard again and I've checked my blood pressure but it's fine. Right now I'm a bit lightheaded/dizzy and I'm starting to worry. I play basketball and practise regularly, but my normal heart rate is a bit high (84bpm)...is there anything wrong? Should I go to the doc? Doctor: Hello!Welcome and thank you for asking on HCM!I understand your concern and would explain that your symptoms could be related to anxiety. Anyway, before concluding that it is just pure anxiety, I would recommend consulting with your attending physician for a careful physical exam and some tests to investigate for other possible causes that may trigger this clinical scenario: - a chest X ray study- complete blood count, PCR, ESR for inflammation- thyroid hormone levels for thyroid dysfunction- blood electrolytes for possible electrolyte imbalance - an ambulatory 24-48 hours ECG monitoring to investigate for cardiac arryhtmia. If all the above tests result normal, you should consider anxiety as the main cause of your complaints. Hope you will find this answer helpful!Wishing good health, Dr. Iliri"
},
{
"id": 174073,
"tgt": "Suggest treatment for recurring fever in a child",
"src": "Patient: when my child was suffering from fever and doctor was advised fevanan plus- 6.5 ml trice in a day for 3 days cepodem 100 - 3 ml trice in a day for 7 days nirocil - 5 ml trice in a day for 7 days now after 1 month again my daughter suffering from little fever from today morning. Please advice can we continue with above to observing for next 2 days? Doctor: HiYes you can safely continue the medication. I would generally not recommend addition of antibiotic like Cepodem. Just giving Fevanan plus will be sufficient for control of fever. If fever still persist even after 48 hours then do consult Physician for further evaluation"
},
{
"id": 116297,
"tgt": "What causes fever, aching muscles, red rash on chest, low platelets and high WBCs?",
"src": "Patient: Yes, sir... My 16 yr old nephew is healthy, but since Thursday he has had temp. spikes of 102. , aching in his chest and most muscles. A fine red rash on his chest. He was seen in a walk in clinic today and was told it was a virus of some kind...? His platelets are low and WBC s high. I don t have a print out, just a concerned call from my sister in law. He is complaining of shortness of breathe....I m a concerned aunt. Carol Wood, RN, MS Doctor: Hello,fever, muscle aches and rash probably points to a viral infection. This is not 100% certain diagnosis though. A thorough clinical examination is needed and perhaps more diagnostic tests. What's concerns me more is his shortness of breath. This symptom calls for immediate medical attention. I would have asked for X-ray, oxygen saturation (SpO2) and electrocardiogram right away, if I were his doctor. Another important test is C-reactive protein. I couldn't suggest a specific diagnosis with the provided information but the above tests can be very useful to exclude more serious conditions.I hope I've been helpful!If you need further clarifications, please ask and I'll be glad to provide them for you.Kind Regards!"
},
{
"id": 206684,
"tgt": "What does on and off bloating, swollen body with confusion and inability to think mean?",
"src": "Patient: Doctor me and my mom are suffering from unknow. when new moon arrives or full moon arrives. My brain stops working and somatch swell hughe and my mom whole body swells and when those bads go away and me and mom are alright as if nothing happen. I brain start working like geniunus after four days after new moon or full moon. This problem lasts till 4 days. Worried both for my mom and me and my dad. I guess don t understand that what we are suffering from it is really pain full. Can you help? Doctor: DearWe understand your concernsI went through your details. I suggest you not to worry much. These are ancient beliefs that point towards abnormal behaviors of people who are affected with psychoses. In the modern science view, these are just superstitions. You need to talk to a psychiatrist and get treatment for this.If you require more of my help in this aspect, Please post a direct question to me in this URL. http://goo.gl/aYW2pR. Make sure that you include every minute details possible. I shall prescribe the needed psychotherapy techniques.Hope this answers your query. Available for further clarifications.Good luck."
},
{
"id": 160471,
"tgt": "What causes a child to be scared of TV?",
"src": "Patient: My 3 years old daughter is scared of the TV. We have not been able to watch any of the 2 TVs in the house for 2 weeks now. She will insist that all TVs must be off. If we put it on, she will run to her room and stay close to the window, crying on top of her voice, really distressed. Even her favourite cartoon channel is now a no go area. Please what is going on? Should I be worried. Doctor: Hello, In general, children develop all sorts of strange fears. Most don't usually last very long. If she does not have any hearing issues in general, then I should say it is not a hearing/sound problem. I would try to slowly reintroduce her to the television, in tiny baby steps. You and your daughter should come up with ways to slowly, slowly get back to normal with the TV. First, just have her sit with the TV not on - try talking about it, describing it, talking about things you used to watch, talking about what you parents like to watch. Then, talk with her about putting pictures on the TV, without sound. Or vice versa- put music on the TV without a picture. You could start with you putting on the picture and watching it without her - she can be brave in another room. Keep moving in very small steps over a long time, like 1 month to get her back to tolerating it. I wouldn't push, but I would stand firm on moving through little steps together. One other suggestion - ask her what she needs to feel calm - could it be a flashlight she shines on the TV, or a blanket to put over her head when it is on? Always leave that (hopefully beloved) object near the TV so that she only has it when she's making progress on getting closer to how she used to be. Don't spend a lot of time discussing her feelings or fears or rationalizations or cause, just keep moving forwards in small steps. Take care. Hope I have answered your question. Let me know if I can assist you further. Regards, Dr. Lisa Baker, Pediatrician"
},
{
"id": 222902,
"tgt": "Suggest precautions before getting pregnant to avoid rubella virus",
"src": "Patient: , i am 27 years old.i have one daughter.she has 5 years old now.my daughter has severe hearing loss.in that result my daughter and me and my husband also affected by rubella virus and CMV virus.is it possible if i go for next baby to affect the virus.what should i do before getting pregnant?shall i need any vaccination?is it possible shall i get a next baby without any problems?pls guide me Doctor: Hi,Yes Rubella infection in pregnancy can cause hearing loss. By affected do you mean that you have Antibodies to rubella ,which was checked by a blood test.Usually if you have already had a infection /vaccination of rubella it is very unlikely for you to get it again. And so you can safely go for the next pregnancy.I would like to know if your 1 child has any other developmental problems or delayed milestones. As that could change the diagnosis and cause of hearing loss .As, if it an inherited or a genetic cause and not rubella which caused the hearing loss ,there could be a recurrence.It would be best to get yourself and kid seen by a fetal medicine specialist to know the cause of hearing loss.Hope this helps.Regards."
},
{
"id": 23822,
"tgt": "Are my blood pressure and pulse rate in normal range?",
"src": "Patient: My age is 84 years, male person. My Blood Pressure is Systolic- 160, Diastolic- 82, and Pulse Rate - 69. Is it within normal range if not what should be my upper limit of normal range Doctor: Hi,Thanks for writing to Health Care Magic, I am Dr Asad Riaz, I have closely read your question and I understand your concerns, I will hereby guide you regarding your health related problem.This is a high systolic blood pressure. Any 2 blood pressure readings of more than 140/90 mmHg (two seperate ocassions) is labelled as hypertension. That above 160 justifies medical treatment i.e drugs need to be given.Before starting any such treatment, you will need a daily or twice weekly blood pressure record, an EKG, lipid profile and renal function tests to ensure safe prescribing.You also need to make certain dietary and lifestyle modifications. Brisk walk gradually increased ti 35 minutes daily, salt intake restricted to 1-1.5 grams a day and avoidance of fatty foods is recommendedI hope this answered your question, if you have more feel free to ask.Regards.Dr.Asad RiazGeneral and Family Physician.."
},
{
"id": 209373,
"tgt": "What causes dizziness, memory loss, and fatigue?",
"src": "Patient: For the past 6 months I have been having episodes of being dizzy, weak, foggy Brian feeling, disconnected from my feelings, inability to concentrate,poor memory, Extreme fatigue,hives after hot shower. driving has become a problem for me because that's when the dizziness is more frequent, sometimes I feel as tho I am going to pass out at any moment and it always seems to be at night and around my period the symptoms are the worst. Sometimes I can't even get the energy to take my son out. I had every test that dr could think of I've had tsh which was low 0.051 and they don't seem to think that's the problem but I really feel it is, I'm really desperate for some opinions as to why I'm feeling this way at such a young age this is so debilitating my life has been a living he'll for 6months and no one can find out anything. Oh and I forgot to say that I have extremely low blood pressure and very high total cholesterol and low hdl along with ebv at 8.0 Doctor: HiThanks for using healthcare magicLow mood, decrease concentration, decrease sleep or poor memory etc are symptoms of hypothyroid induced depression and in that case, you need thyroxine hormone. With medication, these symptoms would decrease. Better to consult a physician for proper diagnosis and management. In case, you need further help, you can ask.Thanks"
},
{
"id": 148612,
"tgt": "Is it necessary to do ACDF surgery as there is limited spinal fluid?",
"src": "Patient: Hello, I just met with a neurosurgeon today and he is strongly recommending a ACDF on C5/6 and C6/7. He said it is a surgery I will not be able to avoid as my spinal fluid is extremely limited in those areas and he is afraid I could fall and end up with my spinal cord severed. Is this true? Thank you. Doctor: Dear Mr. Sorry for your health problems.If the diagnosis is made with magnetic resonance it is quite sure that the problem is and that way surgery is the only solution.I could be much help if you provide me MR answer and tell me about your symptoms.Wish you all the best"
},
{
"id": 157720,
"tgt": "Had co;orectal cancer. Done radiation, bracytherapy. Started bleeding, severe anemia. Does hemoroids hurt?",
"src": "Patient: I had a co;orectal cancer 2 yrs ago and underwent radiation and bracytherapy in th rectal area. I was good for the first 6 months and then I started to bleed more and more. last week I did my CEA reading and no more cancer was apparent. I have however severe anemia and bleed heavily every day. What can that be. Even my dr seems to be confused as we doublted colon cancer but thankfully it is not that.can hemoroids hurt and blled to the point of me having sever anemia. My diet is strickly vegetarian. Thanks Victoria Doctor: For hemorroids if not prolapsed yet proctoscopy may reveal them. For higher bleeding sites sigmoidoscopy is used. Your bleeding time, prothrombin time, clotting time, platelet count, liver function tests should also be done. Bleeding should be arrested earliest by detecting the site."
},
{
"id": 221761,
"tgt": "What is the best treatment to get pregnant?",
"src": "Patient: hye my name is maheen salman ,im 22 year old last year i got married my husband age is 27 year now..first year of my married life we were not going for family.. after feb 2010 we left precautions then problem of irregularance in periods started..every month i visited doc..doc diagnosed me thyroid problem then i used to taking tablets of thyroxin 50mg 2tablets daily...after using it my thyroid level gets normal but still my period process is upset like..you believe that last time i got periods on 3rd june...now 4th month and now i have pains in my legs and lower abdomen..on 30th august doc gave to me parimoulte N for 7 days ..one tablet daily means total 7 tablets..i used to take this tablet frome 31st aug till 6th sep...and now 7 days has gone..no signs of periods and no pregnancy ...i dont understand whats my problem im so confused and depressed.. Doctor: HI, I understand your concern. You had no periods since 4 months. pregnancy was confirmed to be absent. & the you were advised the Promolut N.. 7 tablets & you have recently completed the dose.. worried for not getting bleeding. After stopping the pills like Primolut N, bleeding starts within 8 - 10 days. So , wait for your periods for a weekfrom now, If you have periods, visit your doctor for further medocation on %th day. If you do not get bleeding, still consult a gynecologist for further assessment. The investigations advised in such cases are - CBC, hormonal assay( ovarian, thyroid & pituitary hormones ), Blood sugar, serum prolactin, USG abdomen & ovulation study. Your physical examination & investigations reports would pin point the cause of your irregular periods. Doctor would offer you specfic treatment. * You should plan a pregnancy with you having normal ovulating menstrual cycle & your husband's semen normal .- This would definitely give +ve results. Thanks."
},
{
"id": 198860,
"tgt": "What could it be if having discolouration, itching and flakiness around groin area?",
"src": "Patient: I am having a problem with discolouration, itching and flakiness around my groin area. I am a black male. There is a patch right around the groin with the inside much lighter than my skin tone and the outer edge has some little cuts. Must be from scrating since it itches all the time. Doctor: HelloI appreciate your concern Looking at your description it seems to be related to infection or allergic reaction.I see patients with similar complaints in my clinic and I would advise you to apply clotrimazole with betamethasone ointment twice daily for a week or 10 days If it doesn't subside or decreases than You should consult a dermatologist for further evaluation and definitive treatmentMeanwhile wash your privet parts with antiseptic solution or soap and keep it clean and moist.Replace your undergarment with new one or change it frequently.Hope this answers your questionPlease feel free to ask follow up questions I will be happy to help youBest wishes"
},
{
"id": 204664,
"tgt": "How can severe stress be treated?",
"src": "Patient: my wife of 28 years lost her father in august 17 has a lot of stress and has had problems with depression before seems to be in a very bad place understandably and I do not know how to help anymore she is a very true and loving person who just is not seemingly able to get ahead of life at this point any help is appreciated YYYY@YYYY this is Gregory her husband and friend she just turned 47 Doctor: Hello, I don't see the reason for your wife's depression is her father's demise. There has to other determinant factors. Talking to your wife in person with wholehearted approach may reveal more. Talk to a psychologist. Hope I have answered your query. Let me know if I can assist you further.Regards, Dr. K. V. Anand"
},
{
"id": 163721,
"tgt": "How can vomiting be prevented?",
"src": "Patient: My daughter 13years old is getting vomiting in the morning time before going to scholl (like reddish brown) and all the reports(CT,Ultrasound,Endoscopy) is normal and could not feel the vomiting sensation and suffering alot.All the doctors could not able to find any thing wrong.Can you suggest solution for not getting the vomitings . Doctor: Hello! Welcome to HealthcareMagic! Does your daughter have recurrent Epistaxis?In case of posterior epistaxis reddish brown vomiting can happen.And also you should observe your child for any kind of induced vomiting. Take care Regards, Dr. Priyank Ghanchi, Pediatrician, Neonatology"
},
{
"id": 193879,
"tgt": "Suggest treatment for nightfall",
"src": "Patient: Hi Doc , I have night falls problem & that's so bad for me , cause I'm an Pro Athlete and it's so important to keep my sperm in my body & not to let it out , anyway . one of my friend said that , try sour things before bed / what's your solution then ? Doctor: Hello, The sperm production regularly occurs in testes and it is continuously getting replaced after the discharge of old sperm. So nightfall is not the problem. It usually occur in person who doesn't masturbate regularly. So nightfall will not decrease sperm count. So you need not to worry about night fall. Hope I have answered your query. Let me know if I can assist you further. Take care Regards, Dr Parth Goswami, General & Family Physician"
},
{
"id": 172185,
"tgt": "What causes high lymphocyte count and stomach pain in a child?",
"src": "Patient: my 10 year old son has been have diffuse stomache pain and had a workup things that were out of range are as follow: high absolute lymphs- 3.7 low: alkaline phosphatase 144 high wbs- 9.6 My son is well nutritioned and in the 85th percentile for height and weight. What could the issue be? He has had some constipation which is not normal. Doctor: Hi...THE MOST COMMON REASON FOR PAIN ABDOMEN IN KIDS IS CONSTIPATION.The other parameter you have quoted like high counts and low alkaline phosphatase are not related to pain abdomen. It could be constipation.I have certain questions and suggestions for you.Questions:1. Did your kid pass motion or meconium on day one of life?2. Since how long is the kid constipated?3. Does the kid have any bleeding along with hard stools?4. How much milk does the kid consume per day?5. Does the kid eat fruits and vegetables (fibre diet) appropriately?You can get back with answers at the following link - www.healthcaremagic.com/doctors/dr-sumanth-amperayani/67696Suggestions:1. Natural methods are the best to relieve constipation.2. Constipation is a risk factor for UTI3. Maximum milk consumption per day should not exceed 300-400ml4. Minimum 3-4 cups of fruits and vegetables to be consumed per day5. Toilet training - that is - sitting in Indian type of lavatory daily at the same time will help a lot.Hope my answer was helpful for you. I am happy to help any time. Further clarifications and consultations on Health care magic are welcome. If you do not have any clarifications, you can close the discussion and rate the answer. Wish your kid good health.Dr. Sumanth MBBS., DCH., DNB (Paed).,"
},
{
"id": 213522,
"tgt": "Getting treated for depression, pretty good health. Some microorganism has taken over foods that have not even opened. It is there in all unused food. How to get rid of this?",
"src": "Patient: I am a 57 year old female who is in pretty good health except for major depression for which I am being treated. My problem has been going on for at least a couple of months. Out of nowhere this thing that I guess you would call it a micro organism has taken over my home. It just shows up in almost anything especially foods that have not even been opened. This thing can get inside just about anything that hasn t been used. It is getting inside of my cat s unopened cat food cans and altering the content of the food in a gross way. The food turns into a totally different product which I have been observing with a magnifying glass. I have even double bagged stuff in ziplock bags and this stuff still gets into it. Do you have any suggestions as to what I can do to get rid of this stuff? If I hadn t been witnessing it I wouldn t believe it could be true. Help!!! I can hardly eat anything at home because of this and I am losing quite a bit of weight. Doctor: Hi Lynda, Welcome to Healthcare Magic and thanks for your question... I understand your concern about the strange things that seem to be happening at home. I know how distressing and difficult it can be if you are feeling constantly concerned and afraid about this. Lynda, firstly I want you to understand that as doctors, we are here to help you get out of your distress and I am here to do precisely that by giving you the most appropriate advice. Now, having that in mind, I also want you to understand that sometimes, when we are suffering from a psychological disorder, we may tend to have excessive fears, beliefs or concerns which may actually be a result of the psychological disorder itself. You have mentioned have been suffering form depression and are on treatment currently. Based on my experience of having seen many patients presenting with similar problems like yours, I feel that these are part of the psychological disorder only. I know and understand very well that to you these things may appear absolutely real and it may be difficult to accept the fact that these may not be entirely real. But, sometimes, the psychological disorder can influence our minds and cloud our thinking so much that we may end up believing things which may not be entirely real. This can then result in excessive and irrational fears. Now, again, I want you to be very clear that I am in no way trying to say that the distress and suffering that you are going through is unimportant... I fully agree with you that it is very upsetting and difficult. But, it is also important to be aware and open to the fact that your beliefs / fears may be as a result of the psychological problem you are going through... I would strongly suggest to get a review appointment with your psychiatrist. I'm sure he would be able to assess you more in detail and adjust your medication accordingly to control these fears. Wish you all the best. - Dr. Jonas Sundarakumar Consultant Psychiatrist"
},
{
"id": 217272,
"tgt": "Suggest remedy for pain in toes",
"src": "Patient: At night when I lie down my toes on both feet feel as though I have rubber bands tied around them and they hurt. Morton s Neuroma was ruled out. I was told it may be coming from my back and take 900 mg of Gabapentin at night. If I don t take it a few hours before I go to bet I am in a lot of pain. Doctor: As per your explanation if there is nothing related to neuroma then you need to check the vascular it of you lower limb. I suggest to do dappled test for both entire lower limb and check the blood circulation in legs. And based on that take few medication as per the suggestion of vascular surgeon. Along with this keep your feet warm by using socks in night as well do toe stretch exercise frequently before you go to sleep. Hope this was useful for you. Take care."
},
{
"id": 134635,
"tgt": "What causes burning sensation on the end of the right shoulder near spine?",
"src": "Patient: For the past six weeks I have been feeling a burning sensation on the end of my right shoulder near my spine, and the skin feels numb to the touch and itchy. Keep stretching it out but does not help. I am a builder, and I have started bodybuilding over the past 7 months. I am use to getting pain between shoulder blades from exesive work and repetitive movements, but this is a different pain. I feel it more when I am rested. Doctor: you need to figure out what exercise you perform and what is the dosage for it. the pattern of exercise performed may need a modification. pain in the iterscapular region and shoulder blades are close to trapezitis. you need to undergo a pain management for the release of muscle fiber . Myofascial release should help. mean time the burning sensation needs to be assessed by checking with the specialist. it may be a muscle inflammation or may be high uric acid or lactic acid or may be a nerve entrapment. kindly take a close visit to the specialist for accurate diagnosis and treatment plan."
},
{
"id": 61005,
"tgt": "What does a lump below the sternum indicate?",
"src": "Patient: I have a bony protrusion below my sternum. it is approximately 1\" in length and extends down toward stomach. It is connected to my sternum, but is flexible, almost like a cartilage. Possibly my xiphoid process? I don't really have pain there, but after I eat I feel full and/or bloated all of the time. I am also getting heartburn quite a bit, it doesnt seem to matter what i eat or drink. Thoughts? Doctor: Hello,This is most probably xiphoid process , rather than lump. Burning issue seems to be gastritis or duodenitis with or without underlying ulcer condition.Hope I have answered your query. Let me know if I can assist you further.Regards,Dr. Bhagyesh V. Patel"
},
{
"id": 9631,
"tgt": "Anus area having burning & itching problem. What can i do ?",
"src": "Patient: I am 36 year old male. 2 & 3 months I have bleeding at my anus I have taken treatment now its ok but now at anus area there is burning & itching problem what can i do doctor . Not I taking treatment ie. oxerute cream for anus age 36 male weight 75 kg height 5.3 Doctor: Hi...Sunil..., thanks and welcome to HCM.., Bleeding per anus previously...Now suffering itchinand., burning,,,No doubt it could be ..Hemorrhoids..., First asses the degree of hemorrhoids,. According your history, it is only bleeding and itchin.., So it could be First degree pile.....hence go for Medical treatment... 1) Every day night times...use Laxatives.. 2) Take plenty of water..., 3) Take plenty of fresh fruits and fibre content vegetables.., 4) Tab Ofloxacin and Ornidazole 2 times daily for 5 days..., 5) Tab Levocetrizine night times for 5 days.., 6) Apply Proctocedle cream for every time after passing stool.., 7) Lukewarm and salt water (Sitz bath) dipping..., So follow get good result..good luck..,"
},
{
"id": 192485,
"tgt": "Suggest remedy for sperm leakage during urination",
"src": "Patient: hi sir, i am 23 years old, i have facing problem from past 7 years. 1. sperm leaking daily in my sleep and while urion. 2. if i stop one week i am geting pain in back bone. 3. my penis and tesits are not developed properly, and i have marriage in next year, i feel very shame because my penis and tesits are not developed, one time my fiance saw my testis and penis she told me that you are a child you need to improve, she want little bit more. one time i met one doctor (urologist) and he allso tlod me that you have small penis and testis but he didn't tell solution. now i want improve. pls slove my problem. Doctor: Hello, Possible causes like retrograde ejaculation must be ruled out. Consult a urologist and he will direct you accordingly. Hope I have answered your queries. Let me know if I can assist you further. Take care Regards, Dr. Shinas Hussain"
},
{
"id": 131667,
"tgt": "Does the mild pain in hip usually with weather changes is due to Arthritis?",
"src": "Patient: My father was diagnosed with what he calls Anglos Spondylitis (I m sure that s not how you spell it) But he got diagnosed at the age of 12, with his shoulder. I have a dull irritating pain in my left hip regularly, usually with weather change (no surgery/ breaks). I would like to know if this could be arthritis and if I need to get checked by a doctor? Doctor: try stretching your hip flexors and strengthening your hip extensorsif pain is relifed then it is probably arthritisany way arthritis always get worse with cold weather"
},
{
"id": 210331,
"tgt": "Suggest remedy for chronic boredom",
"src": "Patient: my boyfriend is experiencing something called chronic boredom. he's had to become the \"man\" of the house when he was very young..his family was dysfunctional. he had a very short lived childhood. but needless to say he's 21 now and lives a pretty normal life except he's extremely bored...with reality. with everything as a matter of fact. he said its not that he isnt happy, its just everything is the same day after day, go to work, eat the same food, come home, go to bed..do it all over again the next day. we do go out on the weekends and have fun but then he just dreads the upcomming monday. he does enjoy his job but he just cant seem to get unbored. i know he has had some episodes in the past starting when he was 16 where he pretty much rebelled. the second time this happened he started drinking heavely and just did things that he wouldnt normally do like get into fights he doesnt remember.. after that episode was over he would be \"normal\" again for a while. i dont know what to do to help him.. i hate seeing him in a rut. please please do you have any suggestions? Doctor: Hello,The fact that he had this similar episode in past points to a possibility of mild depressiin in.which sadness is not overt but coloured by feelung of boredom and lethargy...but at the same time the person dosent seem to enjoy life. Moreover, males donot come out with their exact and complete feelings and thoughts that easily.So, please ask him to take.help from a psychiatrist.( preffrably a male).Had i beem treating him, i would have put him on escitalopram 5-10mg. I feel that a low dose would suffice him well.Hope this helps.Plesse ask more questiosn for any clarification.Dr. Manisha GopalMD psychiatry"
},
{
"id": 125406,
"tgt": "Can heel swelling be caused by eating raw rice ?",
"src": "Patient: Hello Doctor, i m urmila 38 years old and i have 3 kids. i am badly addicted to raw rice. i eat approx. 2-3 tbsp daily.Now i m suffering from heel swelling and burning. Recently i have taken vit b-com, evening primrose oil capsule and orofer(iron). What could be the cause? Doctor: Hi, First of all, let me assure you that heel swelling and burning is not related to your habit of eating raw rice. Most likely you are having Plantar Fasciitis. Start painkillers, local heat therapy and soft footwear. If you do not feel any improvement in 2-3 weeks, go for an orthopaedic consultation. Hope I have answered your query. Let me know if I can assist you further. Take care Regards, Dr Gopal Goel, Orthopaedic Surgeon"
},
{
"id": 42710,
"tgt": "How to increase linear progression?",
"src": "Patient: my husband had a zero rapid linear progression..we got a test done after 2 months it came up to 2% and now my doc have advice him to have fertilox.hes been having it since two months n im still not pregnant.is there really a way to increase the linear progression? Doctor: Hi,Thanks for writing to HCM .Sadly medicine has not come with such a drug which have dramatic effect on sperm motility .Multivitamins with zinc combination drugs are used which have shown some improvements .I would suggest you to discuss with your doctor regarding going for IUI. Here healthy sperms are selected and placed in uterus. Thus solving motility problem to some extent. You can try 4 to 5 cycles before proceeding further. Some life style modifications to be done which helps in production of healthy sperms are- reduce weight and ware loose under garment. -stop alcohol and smoking . Avoid caffeine. -healthy diet.Hope I have been helpful .RegardsDr.Deepika Patil"
},
{
"id": 67704,
"tgt": "Suggest treatment for red, painful bumps all over the body",
"src": "Patient: I have slightly bigger than pimple size, red, very painful bumps on multiple areas of body (both sides). The skin within an inch around them is also very sore. It feels like a stinging pain, but it also hurts like a deep muscle ache. I haven t changed anything in my daily routine as far as soaps, detergents, etc. I was recently at the beach in North Carolina and spent most of my time in the ocean. I also used a pool and a hot tub for a minimal amount of time. Some of these bumps almost look shiny like a blister and have a head to them. They have nearly doubled in size since discovering them this morning. I have 5 on my shoulder all within 1/2-1 inch apart, 5 scattered on my abdomen, 2 on both sides about 2 inches under my armpit and on my rib cage, one on my bicep and another on my forearm on my inside elbow, and two close together on my hip. What can they be? Doctor: Hi, dear. I have gone through your question. I can understand your concern. You may have some acne on your body. You should take doxycyclin tablet. You can go for local application of benzoyle peroxide & clindamycin. Drink plenty of water. Take citrus fruits. Consult your doctor and take treatment accordingly. Hope I have answered your question, if you have doubt then I will be happy to answer. Thanks for using health care magic. Wish you a very good health."
},
{
"id": 211372,
"tgt": "Believe myself to be healthy but feeling depressed and abnormal",
"src": "Patient: Hello. I am 17 and I believe myself to be very healthy, but since I believe when I was 15, I noticed there to be a depression in my head. I don't think it has changed size or anything since. I just want to know what is wrong with me, because I feel this is abnormal. Doctor: HelloThis the age when personality development is at peack and nearing completion .So no treatment is required at present Report if this feeling persist and that time consult a psychistrist for complete evaluation do not worry Dr Lal Psychiatrist"
},
{
"id": 86436,
"tgt": "What causes severe pain in lower right abdomen and under rib cage?",
"src": "Patient: Hi. I am experiencing severe pain in my lower right abdomen and higher under rib cage on right side. I have recently been trying to recover from Lyme disease and mono. I have been told that my Lyme has become dormant, but the mono is still active. I was helping my husband move some heavy furniture about an hour ago and now the pain is unimaginable. I have had a hysterectomy, 2 years ago. They removed my uterus, cervix, both tubes, and right ovarie, and my appendix. They also had to remove a large amount of scar tissue and cysts, on nearly every aspect of my abdomen. I have severe endometriosis, and had scar tissue removal, and cysts removed from my abdomen, 4 separate times. I have been doctors so much in the past, I don't want to go if not necessary. Do you think the pain I am experiencing is something serious? Doctor: Welcome to healthcare magic for your health concernsI am Dr Fahim, and I will be helping you with your query . . . \u00a0\u00a0\u00a0\u00a0\u00a0I have gone through your question and understood your concerns. It appears to be pain of the gall bladder because of gall bladder stones called Cholelithiasis. Symptomatic gall stones is very common in middle age women, with presenting complaint of pain in the right upper quadrant. To confirm our diagnosis we need Ultrasonography Abdomen along with blood complete picture and liver function tests to rule out infection and gall stones effects on liver functions. If such a patient walk into my clinic I start treatment with tab paracetamol 500mg 8hourly, tab Nospa 40mg 12hourly, tab metoclopramide 10mg 12hourly, cap omeprazole 40mg 24hourly. Mean while counselling and preparing patient for cholecystectomy, preferably laparascopic cholecystectomy. I hope my answer will help you. Please do rate my answer if you like itRegards"
},
{
"id": 29947,
"tgt": "What causes chronic burning pain with itching inside the vagina?",
"src": "Patient: I have shingles, the rash is gone, I went to the dr and took the antiviral medicine for 7days. Now for the last two weeks I have terrible burn inning, itchy pain where the rash was. Is there any thing I can take to relieve the pain? Should I go back to the dr? Doctor: The pain after shingles is called post herpetic neuralgia(PHN) Antivirals may not reduce pain completely.hence u need to apply some smoothening creams to relieve pain..there are many preperations available..Consult your doctor again he may give suitable preperation for you."
},
{
"id": 5957,
"tgt": "Trying to conceive. Taking progyluton for regularizing periods. What is the action of this medication?",
"src": "Patient: hello doctor,i am a married lady of 23 years old..my wedding been 2 years already.we have been trying for a baby since 1 year already..but still no negative report..i get my mense irregularly and the doctor prescribed progyluton..which i continue to take even when the doctor told me to take pills to become fertile.. i want to know if really progyluton helps to conceive or just make period regular and if the doctor has been fooling us by now taking huge fees.. please help me out thnks Doctor: Hello and Welcome to \u2018Ask A Doctor\u2019 service.I have reviewed your query and here is my advice. Progyluton helps in regularizing your periods. For conception, ovulation-inducing drugs are needed but they are helpful only in regular periods.Hope I have answered your query. Let me know if I can assist you further.Regards, Dr. Rakhi Tayal"
},
{
"id": 59375,
"tgt": "Pregnant. Have severe abdominal pain, vomiting, leg cramps. Is it due to gallstones?",
"src": "Patient: I m 25 weeks pregnant with my 4th baby. No complications with other pregnancies, all were full term. I am a healthy vegetarian regular exerciser, 34 years old. I m experiencing for the 2nd time in a month severe abdominal pain with vomiting . It lasts for 24 hours then when those symptoms subside I have almost unbearable leg pain. Mostly in my calves. Seeing my midwife tomorrow - wondering if it could be gallstones and if the leg cramps could be a result of that as well? Doctor: Hello Welcome to health care magic Dear patient, abdominal pain during pregnancy can be due to many reasons which may be related to pregnancy or unrelated to pregnancy. Pain related to pregnancy include, abruptio placenta, endometritis, salpingitis ectopic pregnancy, red degeneration of fibroid uterus,fatty liver of pregnancy, labour pains etc and pain unrelated to pregnancy include gastritis, cholelethiasis and chloecystitis, urinary tract infection, renal stones, appendicitis , low back pain due to disc disease,etc. Leg pain following abdominal pain and vomiting can be due to electrolyte disturbances like hypokalemia. You consult your gynecologist who will get some tests done like USG abdomen, urine routine and microscopic examination,LFT ,RFT and other necessary test to find out the cause of pain and will then give you the treatement accordingly thanks Good luck Dr Arshad"
},
{
"id": 137217,
"tgt": "Suggest treatment for neck lymph nodes reactive change condition",
"src": "Patient: Hi Doctor i am from Ethiopia i have one problem that is my 19 years son have in his neck and near his ear the glands became swollen and i took him to a doctor and he took a pathology test and in his result it says Lymph nodes reactive change .please tell me what does it mean and what is the medicine to be cure .Thank you Doctor: Hi there.There is infection in his mouth, throat or ear which is causing the lymph nodes in the neck to enlarge and become painful. H needs treatment for that infection. Once the infection is treated, he will get relief and the lymph nodes will quieten and be fine."
},
{
"id": 113470,
"tgt": "Consistent lower back pain, excessive sleepiness, frequent urination at night. X-ray showed disc compression. Treatment?",
"src": "Patient: Hi, i have some problems and they are as follows: 1.) Its been almost 8 weeks now that my lower back or lumber spine area is paining alot that i cant even bend or run or even stand alot. I went to see an orthopaedic surgeon and he made take an x-ray which was very normal with the exception that one of my discs had been compressed. He gave me the following medication namely Gabapentin tabs, Neurotone tabs and Zeradol tabs but unfortunately the pain is still there. 2.) Secondly, i tend to feel sleepy more then normal i.e i cant stay a wake for a long time as i feel sleepy through out the day and i sleep well at night despite having frequent urination. But during the daytime it is very difficult to stay awake as i feel sleepy all the time. Please let me know what treatment to undergo in order to sort out my ilnesses. Hope to hear from you very soon. Thanks, With best regards. Doctor: Thanks for the query. Please try local measures such as physio modalities. Gabapentin causes sleepiness. Please stop it. Continue antiinflammatory with muscle relaxants. Take absolute rest for few days. It usually takes few weeks to subside the pain. Thanks."
},
{
"id": 139593,
"tgt": "Suggest treatment for brain TB",
"src": "Patient: hi, my sister 4 years old. she suffering from brain t.b.& takin medicin for 6 month & was getting treatment lata mangeshkar hospital in pune but shes health not improved.back bone & head is pain.my sister had in starting of t.b. 12 month yesterday MRI report in brain water improved. Doctor: Hello, ATT (anti tuberculosis treatment) for seven or nine months will be the treatment of choice. Most cases respond well to ATT. In your case, MRI also shows good response.Hope I have answered your question. Let me know if I can assist you further. Regards, Dr. Shinas Hussain, General & Family Physician"
},
{
"id": 174417,
"tgt": "Whats the best medication for sinus in children?",
"src": "Patient: dear doctor my son age of 2yrs we have consulting a near by doctor for sinus problem he advised us to give montairlckid tablet daily and told us without his knowledge we should not stop the tablet.past 1yr we are giving the tablet but the sinus is not cured yet. now my son age is 3 and half yrs now.please guide me the right way to cure sinus in tablets or a better way. Doctor: Hi...Thank you for consulting in Health Care magic. Greetings from Chennai.By what you quote I feel what your kid could be having viral associated wheeze or multi triggered wheeze. I have a few questions for you -Questions:1. How many days per month does she cough or feel breathless?2. How many nights per month does her sleep get disturbed due to above symptoms?3. Does she feel breathless when she runs around or plays with other kids?4. Are the symptoms when there are seasonal changes?5. Is there any family history of asthma or any other sort of allergies like skin allergy etc.?6. Is the cough always associated with fever?If your answer is yes for any of the above questions, your kid might be having viral associated wheezing or multi triggered wheezing and I suggest you meet a paediatric pulmonologist who is near your place.Regards - Dr. Sumanth"
},
{
"id": 138731,
"tgt": "What causes red bumps on calf with tingling and pain in leg?",
"src": "Patient: Hi Doc. For the past few days every time I go for a run, when I put pressure on my left leg, these red bumps rise up on my calf and I feel tingling and pain in the leg. When I stop putting pressure, the bumps subside. It almost feels as though my veins are popping out, but I am only 20 years old so I m not sure that can be right. I spend about 10 hours on my feet every day as I am a tennis instructor. What do you think this indicates?? Thanks Doctor: These are signs of varicose veins and are due to your long standing job. If its painful and causing swelling or in some cases ulcers they need treatment consult a general surgeon at the earliest."
},
{
"id": 204238,
"tgt": "How can severe anxiety and panic attacks be treated?",
"src": "Patient: Hi, I was just wondering if you could help me or give me any advice to do with anxiety, especially in school. Whenever I\u2019m in school and I\u2019m asked to read or know that I\u2019ll have to at some stage, my heart starts pounding and I can\u2019t stop shaking. Sometimes when I\u2019m actually reading I hear my voice and it\u2019s so shaky it\u2019s so embarrassing and it often just goes completely or I feel like I\u2019m choking . I get small panic attacks whenever I have to read and I don\u2019t know why so it just frustrates me so much and makes me so mad at myself . I don\u2019t want to be like this so is there anything I can do to help this problem? Then when I\u2019m finished reading aloud and if I try to write I physically can\u2019t because I\u2019m still shaking for ages later and everyone\u2019s staring at me in confusion . Please please help me Doctor: Hello and Welcome to \u2018Ask A Doctor\u2019 service. I have reviewed your query and here is my advice. Some degree of social anxiety is common in the general population. Such anxiety only becomes social anxiety disorder when the anxiety either prevents you from participating in desired activities or causes marked distress during such activities. Both psychotherapy and medicine are useful in treating social anxiety disorder. Consult nearby psychiatric clinic or you can consult me for any further help required. Hope I have answered your query. Let me know if I can assist you further."
},
{
"id": 41492,
"tgt": "Is the sperm count normal?",
"src": "Patient: hi i am manoranjan patra my semen volume-2.0,viscosity-normal,colored-white total sperm count-110 million, normal spermatozoa75%,abnormal spermatozoa 25%, total modility- 90%,active motile 80%,sluggishly motile 10%, non motile ( dead)10 %,pus cell- 1-2, redblood cells - nill, Testicularcells- nill Please answar me is this normal? can i be a fathher Doctor: Hi Mr Patra,Yes you can be a father. The semen analysis result is normal. So you should not worry."
},
{
"id": 13310,
"tgt": "Suggest permanent remedy for skin rash",
"src": "Patient: Sir I am diabetic person, FBS IS 150mg/dcl I have skin allergy having reddish skin and rashes. Where allergy appears I will take two 5mg citizine tablets, it will not appear for two days, again after two days it will appear. once again if I take two 5mg citizine tablets it won t come for two days. It is happening for me from the past six months. What is the permanent remedy Doctor: Hi,It may be chronic urticaria. Consult the dermatologist for the perfect diagnosis and proper treatment. You may go for allergy test. The triggers may be found out. It may be either contactant or dietary factor. That should be avoided. And continue taking cetirizine for more than 6 months even after control. Hope this helps.Dr.Ilyas Patel,Dermatologist"
},
{
"id": 115803,
"tgt": "Can I have an offspring of AB blood group if parents are O+ and B+?",
"src": "Patient: hi i want ur help to find the answer of the following. \"the blood group of a man is O+ and his wife's group is B+, but the group of their child is AB+. is there any ambiguity about the child's legitimacy? is it possible? plz reply with logic and solid reason.....\" Doctor: Hi, dearI have gone through your question. I can understand your concern. If husband's blood group is o positive and Wife's blood group is B positive then their child has no chance of having blood group AB positive. Child can have blood group eitherO or B. Hope I have answered your question, if you have doubt then I will be happy to answer. Thanks for using health care magic. Wish you a very good health."
},
{
"id": 35006,
"tgt": "How to cure itching in legs due to severe alpha1-antitrypsin deficiency?",
"src": "Patient: My husband has severe alpha1-antitrypsin deficiency, type PiZZ with very low values og A1AT. He has developed empghyzema in the lower lungs. He is doing quite well, but now he is bothered by a severe itching on his legs. This was very markant before he got the diagnosis, but has mostly been gone the last year due to improved general health. Does the itching has something to do with cirrhoses of the liver? And if it can, how can we test it? We live in Norway, and there is almost no interest in anything but the lungs here. Doctor: Thanks for posting your question on HCM!I appreciate your concern towards your husband's illness.Hopefully the cause will be detected and treated accordingly.Pruritus is a common complaint in liver disease (including cirrhosis) in patients with alpha1 antitrypsin deficiency.Mild itching may respond to treatment with antihistamines and topical ammonium lactate. Cholestyramine is the mainstay of therapy for the pruritus of liver disease.Since your husband has PiZZ variant which has only 15 to 20 per cent of normal blood alpha-1 antitrypsin levels, which is linked to severe disease.Your husband needs to be consulted for proper evaluation like complications of cirrhosis if any should be detected and managed early.I hope your query has been answered.Wishing your husband a speedy recovery."
},
{
"id": 124840,
"tgt": "What causes weakness in joints?",
"src": "Patient: Dr Sahab my wife is 15 years plus she is 4th month pregnant she has pain legs joint and bones also she fell lowering in blood pressure ,she has no power in hands and body ,feels powerless kindly advise us some medications and diets which overcome these problems .Thanks Doctor: Hello, The feeling of weakness and pain in the joints can be due to iron and calcium deficiency. Proper supplements can be started after getting her blood tests done. Hope I have answered your query. Let me know if I can assist you further. Take care Regards, Dr Praveen Tayal, Orthopaedic Surgeon"
},
{
"id": 122373,
"tgt": "Suggest remedy for pain in the lower leg",
"src": "Patient: I have been having this pain in my lower leg. It started as a shooting pain in below my calf but above my achilles. It is a dull nagging ache. It comes and goes and even goes to my foot at times. I have no swelling, no reddness, and it is not tender to the touch. Doctor: Hello, Most of the lower leg pain occurs due to overuse, wear & tear, injuries in the ligaments, tendons or soft tissues/muscle strain. You can take pain relievers like Ibuprofen or Naproxen, one sos when needed. Take plenty of water, exercise & get a massage done. Physical therapy can help. Maintain a low BMI. You are suggested to see a GP, get clinically examined to rule out Varicose veins, PAD, sciatica, diabetic neuropathy, etc. Get blood & Imaging tests done. Your treatment will depend upon the severity of the pathology detected. Hope I have answered your query. Let me know if I can assist you further. Take care Regards, Dr Nupur K, General & Family Physician"
},
{
"id": 97223,
"tgt": "Should i take my mother to emergency as complaining room spinning?",
"src": "Patient: My mother will be 68 in november she is complaining of the room spinning she had a flu shot on the 8th of this month and a car accident last year that caused her head to have internal bleeding should i take her to the emergency room even though she is afraid to go. Doctor: in my opinion ,it is unlikely last year's internal bleeding or flu shot can cause spinning sensation.my suggession is visit an ent surgeon to check her ears for bppv , blood pressure and sugar test."
},
{
"id": 26748,
"tgt": "Should I be concerned for the high BP and tightening in the chest?",
"src": "Patient: H I am 37 years old femaie had routine smear test and tested my blood pressure which was 140/79 so I also mentioned some other problems with tightening of my chest. It is really getting me down as they said it is anxiety but I dont feel anxious. Should I be concerned. Doctor: Hello!Welcome and thank you for asking on HCM!Regarding your concern, I would explain that your blood pressure is in the highest limits of normal values. Have you measured your blood pressure during the episodes of tightening in your chest?You should know that high blood pressure can lead to chest discomfort. But I recommend consulting with your GP for a careful physical examination, a resting ECG, a chest x ray and some blood lab tests like : a complete blood count, thyroid hormone levels, fasting glucose, kidney and liver function tests, blood electrolytes, lipid profile, to exclude other possible causes of secondary hypertension. A treadmill test should be performed to exclude possible coronary artery disease if suspicions are raised. Meanwhile I advise making some modifications to your lifestyle, which can help reduce your blood pressure: - Lose weight if you are overweight- Make a lot of physical activity- Reduce salt intake and take a lot of vegetables- Reduce caffeine intakeHope to have been helpful!Greetings!Dr. Iliri"
},
{
"id": 219142,
"tgt": "Will pregnancy be affect due to steroids?",
"src": "Patient: I am, 28 years old patient of Myasthenia Gravis since Jan 2006. I got married in Nov 2008.I have a query regarding Pregnancy. I have been on huge doses of steroids. Now I am on 25 mg after tapering (the doses have been high and low depending on my condition.) My thymectomy has also been done in Feb 2008. I am mow in remission since July 2007. Strength is not an issue but my weight is 64 kilos height 5 1 . i am trying to reduce it. Now i am in a dilemma whether to plan my pregnancy or not. Will my child get the same or some other disease through me? If the probability is YES then i may wanna adopt a child. i have pcos as well since 2002. Please suggest something. Is IVF the best option for me? I am being treated at AIIMS but now i m in japiur after marriage Doctor: Hi.Probability is always present, but the choice is left up to you entirely. If having your own child is really important to you, then you can give it a go. You could have a perfectly healthy and normal child, or it could go the other way (although in my opinion, less likely).But if adoption will give you equally as much joy and happiness, then go ahead with that.Best wishes."
},
{
"id": 103232,
"tgt": "Suffering from continuous cold, suspect allergic asthma. No help with inhaler, Winolep. Any suggestions ?",
"src": "Patient: Respected doctor,My mom is suferring from continuous cold,doctor said she is suffering from allergic asthma but I didnt get any remarkable improvement .she is using inhaler ,but I need a good consultancyDoctor prescribe winolep and deflalone6 she use tis medicine for last two years ,but no improvement ,so plz give guidance to me ,and plz say me which specialist doctor I have to consultThank you. Doctor: you need to consult allery specialist as you are sufferin from alleric rhinitis and bronchitisyou are alreadt on steroidsyou need to et blood serum tests to know the cause of allergy and treat it by immunotherapy which can help you lon rather than takin routine medicines for aSTHMA"
},
{
"id": 39075,
"tgt": "Suggest treatment for bone infection in the leg",
"src": "Patient: Yes, this is an urgent request...help us please! 34 yr. old son has bone infection in leg. Smells like sour cheese. Is sev. inches long and depth of approx. 1/2 inch. Has been battling mrsa. Has been almost impossible for him to get med. care wo/health ins. Doctor: A bone infection especially with MRSA is a very serious condition. It requires extended treatment with specific antibiotics and possible surgery to decrease the load of infection. Many communities have medical care available for patients who cannot afford it. Check to see if there is a community health clinic in your area. Some states have programs for people without medical insurance. He may be eligible for Medicaid. Check on all these things for coverage. In the meantime, he can go to an emergency room and start treatment. They also might be able to refer him to places that provide coverage for indigent people. An ER by law is not allowed to turn away a sick person because of lack of insurance, so they should be able to get things started at least. Hope this helps."
},
{
"id": 175363,
"tgt": "Suggest treatment for small white dots on the tongue of a child",
"src": "Patient: I have a one year old and she has been crying and pointing at her tongue for the last two days now. She has white spots but there s one spot that s not white. Its like a clear little spot off to the right of her tongue. What does that mean and what do I need to do? Doctor: Candibene 1% or clotrimazole solution 1% drop 2-3 drops on the tongue. Before this you should clean tongue with roller- bandage with soda solution- 1 teaspoon of soda for 1 glass of water. If it will not help give fluconazole 50 mg 1 time daily 5 days"
},
{
"id": 127182,
"tgt": "What can cause pain and a lump below the knee after an injury?",
"src": "Patient: I fell and hit right below my left knee about a month ago and dr said I had bad contusion and an infection so they placed me on kelflex. it is still very very sensitive to the point if I graze it or put any pressure on that spot it will shoots a massive pain stinging feeling. there is also a lump/bruise ( size of half dollar ) what is going on Doctor: Hello and Welcome to \u2018Ask A Doctor\u2019 service. I have reviewed your query and here is my advice. The pain and lump can be due to hematoma or a mild cortical fracture. MRI scan will help in proper diagnosis. Hope I have answered your query. Let me know if I can assist you further."
},
{
"id": 23615,
"tgt": "Should a resting heart rate over 100 be a cause for concern?",
"src": "Patient: Hi. I have an artificial aortic valve. I tried painting my car in my garage. I used a mask but it didn't end up working very well. I breathed the fumes quite a bit. My throat was irritated and I had a cough, Six days later, I feel short of breath. My regular workout was too hard for me and my resting heart rate is over 100 and it is usually about 80. Should I be concerned at this point? Doctor: our lungs have a tendency to respond to the allergans from outside atmosphere, in a general any pollutant from outside which we inhale is going to irritate the lungsmost likely your lungs responded to allergans (paint ) in your casein response to allergans from outside lungs produces large amount of mucuous , special cells called histamines are released in blood most likely it is an issue with the lungs and not your valvestaketab augmentin duo for 3 dayssyrup bron q 2 tsf twicetab cetrizine 1 hsget a chest x ray doneand tlc/dlc doneif not relieved in 3 days plan for a 2 d echo cardiographythank you"
},
{
"id": 167449,
"tgt": "How to cure burns caused by boiling water in a baby?",
"src": "Patient: Hi, My friend s 17 month old baby has suffered mild burns from hot boiling water 390 Mins back. Plz tell me (A). What first-aid should be administered to him, (B). Which is the nearest Pedriatic Burns specialist - nearest to Indirapuram where my friend can take him. Doctor: Apply cream which has silver sulfadiazine like silverex and go to any nearest hospital. All major hospital have facility for this type of burns."
},
{
"id": 125936,
"tgt": "What causes shortness of breath and muscle pain after taking an injection?",
"src": "Patient: i received the shot on april 19th because i was unable to take any of the pill forms of medication. After the shot I started having the same side affects i got with the other meds. bone pain,muscle pain, shortness of breath, pain that moves from one place to the next. Thought at one point I was having a kidney stone attack, then it moved to my left side. Could`nt get ahold of my DR. so I went to ER. They said it sounded like a heart attack, kept me for three days. Found nothing, said my heart , lungs, bones, everything was fine. All i wanted and ask for was what they had to counteract the shot. Is there something? Doctor: Hi, It might be due to an allergic reaction following the injection. Consult a physician and get evaluated. Hope I have answered your query. Let me know if I can assist you further. Regards, Dr. Shinas Hussain, General & Family Physician"
},
{
"id": 47573,
"tgt": "What causes pressure in lower back with frequent urination and bloating?",
"src": "Patient: A couple days ago I started feeling pressure in lower abdomen and had to pee every 2 seconds now that has stopped and I feel very bloated and my lower back is killing me plus I am on my period which is abnormal went from very heavy to none to heavy and back to none again what is wrong with me Doctor: HelloThanks for query .Severe pain in lower back with frequent urination and bloating is mostly due to presence of stone either in kidney or ureter .Please consult qualified Urologist for clinical examination and get following basic tests done to confirm the diagnosis.1) Urine routine and culture.2) Ultrasound scanning of abdomen and pelvisFurther treatment will depend upon result of these tests and final diagnosis.Dr.Patil."
},
{
"id": 171125,
"tgt": "What causes dark rashes under the eyes with dry cough and low fevers?",
"src": "Patient: My 3\u00bd yr. old daughter is on day six of illness. First, she had a fever of 40-40.5 for three days straight. It was non-responsive to acetaminophen. She had enough water and fluids and urinated frequently. Our doctor said to be sure that there was no respiratory involvement, and that she d be fine. On day 5, she developed a dark red-brown rash under her eyes. It s more of a solid blotch. There are some other areas, on her face, that are non-raised and blanchable, small reddish circular. They seem to fade and move a bit. But the area under her eyes is constant and deeper in colour. She has been developing a persistent dry cough and has remained anorexic. She s been running low-grade fevers and remains lethargic. Should I be more concerned... or is this just a rash following high fever? Our Dr. hasn t seen her, as this has been a busy influenza season. Doctor: These are normally settled by anti allergic medicines but are temporary measures. Repeated anti allergics are not advisable.As per Ayurveda principles, its 'Pittaj' jawar i.e. fever caused by vitiation of Pitta humor of Vata, Pitta, kafa).Allergy, urticaria, low grade fever, acidity, headache are few symptoms.If you agree a basic precipitant of Tinospora cordifolia herb called Giloy Satva or Guduchi Satva may be given to such children. You may get it world wide in the form of Guduchi capsules. Pour the contents and give to child twice a day with honey. Else get powder from India or online.Amritarishta liquid is also made of Tinospora cordifolia as a main content which, if available, may also be added 1 tsp bd + equal water.Results are slow but its 100% safe. About 4 to 8 weeks of treatment with it may ensure immunity against such manifestations for months ahead. It doesn't interact with any other drug too, so you can continue current medicine also (though no need).For any other questions pl write direct.Hope it helps."
},
{
"id": 113931,
"tgt": "What treatment should I take for the stabbing pain in the back which may be a result of shingles or after effect of scoliosis operation ?",
"src": "Patient: Hi In 2004 i had a scoliosis operation where i had 2 metal rods put into my spine. I was fine till about 2 years ago where i discovered shingles which was cured, it had recently come back so the doctor i went to said but was internal and nothing can be done about it. For the past month now i have been experiencing such pain in the lower right side just above my hip. i am not sure if this is shingles or if it could do with my back operation i had but i cant bare the pain anymore and dont know what to do. it actually feels like someone is stabbing me in the back with a knife. Doctor: Thanks for using Healthcaremagic. The symptoms described by you looks like a T12-L1 radiculopathy (pain in the distribution of the radicals). This can be due to various causes like early shingles before the vesicles appear, compression of the nerve either related to progressive scoliosis or surgery. T12-L1 radiculopathy is usually not caused by degenerative lumbar spondylosis. Consult a Neurologist or a Neurosurgeon and get started on medicines for reducing the neuropathic pain. If vesicles appear get treated for shingles again with antiviral medicines. Steroids should not be used as it increases the risk of shingles. I think this will guide you for further treatment. Bye"
},
{
"id": 82132,
"tgt": "What does electrical pain in elbows and chest with breathlessness mean?",
"src": "Patient: Hello, my symptom is electrical pain suddenly starting in my elbows, then radiating up to my chest, then my chest gets tight feeling and i feel breathing deeply helps it go away. This usually last around 30 seconds then goes away. This sometimes does not happen for months, but then sometimes it happens only a few days apart. This has been happening periodically over the past year to year in a half. Doctor: Thanks for your question on HCM.In my opinion your this symptoms are suggestive of either calcium or vitamin B 12 deficiency. But since it is associated with chest symptoms we need to rule out cardiac cause first. So get done ECG to rule out this.If ECG is normal than get done ionised calcium and vit b 12 level. And if deficiency is present we have to treat it.Oral supplements are usually sufficient but in severe deficiency, injectable supplements are needed."
},
{
"id": 179324,
"tgt": "Is valparin, frisium and lamictal right medicine for seizure and abnormal egg report?",
"src": "Patient: sir, my three yr old son can not speak a word till the age of 2.25 yrs but than started to say papa only not any other word ,at the age of 2.5 yrs he was suffered from his first siezer , he was suffered it for atleast 30 minutes b coz he was in school ,we consult the neurophysician. he started vlparin 200 syrup -5ml (two times), his mri report was normal but eeg was abnormal after one month he can not balanced him self. he started to fall down frequently when walking so, doctor added frisium 5(one time) but still not controlled. after one month he has one more episode of siezer but frquency is lower than previous. one more eeg done which was also abnormal. after that doctor prscribed one more drug lamictal syrup(two times), which we have to increase gradually. but after one month he has three minor attacks in three days. than as we gradually increase dose of lamictal to 25 mg(two time) ,he is now better and can balance him self but very clumsy. so today we are giving him valparin 5ml(two times), frisium 5(one time) and lamictal 25 mg(two times). sir, now when medicine are adjust but he can not speak more than five to six words and he is very impulsive , stubborn . his third eeg report which done recently is 80% better than previous reports. we also giving him occupational therapy and speech therapy from one month .so as per your view any other medicine which effect beeter than above and what is the side effects of above drugs? Doctor: Very sorry for your child. He is suffering from a seizure disorder with poor prognosis. His overall treatment is in right path.. He will require all this anticonvulsant for long duration."
},
{
"id": 31010,
"tgt": "Suggest treatment for frequently occurring cold deep in the lungs",
"src": "Patient: My daughter is 14. She has been treated for asthma since she has been 4. She is also on allergy shots. The problem is, this time of the year she seams to get a cold deep in her lungs that comes and goes. She has been on 2 rounds of antibiotics and prednisone. I fear the prednisone give her really bad anxiety problems along with shaking and bloating. Can you recommend anything that would help her that isn't so hard on her heart? I fear long term damage. Doctor: Hi thanks for contacting HCM...Noted she has asthma....and need frequent steroid shot....Here as she has repeated attack maintainance inhaler with low dose steroid and beta 2 agonist can be started....Allergen exposure avoided like dust, pollen , cold etc.If needed allergen test can be done...Leukotrien antagonist like montelukast can be given....Take care..When acute attack of asthma rather then steroid shot nebulization can given with budesonide....Dr.Parth"
},
{
"id": 57817,
"tgt": "Can elevated level of bilirubin s and gilbert s syndrome lead to liver disease?",
"src": "Patient: My 24 year old daughter has elevated bilirubins of 1.9. She is in the Air Force and they continue to check her on a monthly basis and have told her she has Gilbert s Syndrome. Her bilirubin s have been raised since she had mono about 5 years ago. Should she seek a specialist- she is very worried and scared of liver disease. Thank you Doctor: Hi You are saying that your daughter has high bilirubin 1.9 but you are not mentioned if she has high liver enzymes like ALT and AST. if she has only slightly elevated bilirubin without high liver enzymes, the most prabably diagnosis is Gilbert syndrome. Gilbert syndrome has nothing to worry about. No treatment. Regards Dr.Klerida"
},
{
"id": 70403,
"tgt": "What is the soft lump on the hip having bulging discs in the lower back?",
"src": "Patient: I have a bulge above my left hip only that is soft and painless to the touch, that forms an obvious roll when standing. It is approximately 8 cm long (groin to hip) and 2 cm wide. It has appeared over the last 2 days. I have severe pain from bulging discs in my lower back but otherwise healthy. Doctor: Hi.This looks to be a hernia. Consult a Surgeon for confirmation of the diagnosis and if it is hernia get operated.But tell the Doctor about your disc problem."
},
{
"id": 112635,
"tgt": "Back pain, leg pain, neck pain, shoulder blades pain, restlessness, lack of sleep. What can treat the pain?",
"src": "Patient: Could I be tested for ms if I asked my doctor ? Due to the pain I have suffered for almost 6 years . What test would they do? I could cry every day with the pain to my legs feet , left side a quarter the way up my back, pain running across the way. My neck and across the left shoulder also aches. Iam not a person that complains very much and I have been to the doctor several times over the years. I don't sleep very well and the pain gets me up. Iam more restless lying down. I walk round the walls in the morning to get my legs moving then I just have to get on with it. Would you please help me ? I would be so grateful. Iam in a great deal of pain this morning and have had very little sleep and have been crying. Doctor: Hello and Welcome to HCM, Thanks for writing to us. Your symptoms are likely to be due to nerve root compression due to disc protrusion. A follow up MRI scan will help in confirming the findings. Such symptoms are usually taken care of by Methylcobalamin supplements which you can take after consulting your orthopedician For this condition you have to do stretching & strengthening exercises in which you have to stretch tight muscle & strengthen weakened ones which will alleviate stress on facet joint and disc. Meanwhile for pain control take anti-inflammatory drugs like ibuprofen Hope this helps you. Wishing you good health... Regards. Dr Saurabh Gupta."
},
{
"id": 1766,
"tgt": "Is it safe to have IVF to conceive?",
"src": "Patient: I'm 29yrs old. My husband is 30 yrs. His SA is fine. I have a highest FSH of 13.96 and an AMH of 1.1ng. Doctor has suggested to go directly to IVF. She is quoting a cost of 85,000 of basic pack + cost of hormone injections. Please suggest if it is advisable to go for IVF Doctor: Hi, I think you should go for IVF. IVF has the highest chances of pregnancy amongst all the options like Timed intercourse, IUI and IVF. Also you have a high FSH and low AMH, so your follicles are less in number. Cost is fine not very high. Hope I have answered your question. If you have any other query I will be happy to help.Regards Dr khushboo"
},
{
"id": 213140,
"tgt": "Suicidal thoughts, starting with anger, wanting to shoot followed by numb feeling settles with cuts on the body. What treatment is advisable?",
"src": "Patient: My boyfriend is having problems that wont go away. He said it starts with anger , like pure rage, and then i feel like i want to just put a gun to my head and pull the trigger. then i feel so numb, so i cut myself can you help? i dont know what to do and hes scared to tell his mom about it. An he s also had suicidal thoughts. Doctor: hi..it may be impulse-control disorder or something like personality trait..so please visit a psychiatrist..also an EEG is to be done..this disorder will remain in control by pills as-well-as some positive counseling..so have a healthy life"
},
{
"id": 148443,
"tgt": "Have memory loss, cannot remember short term events. Heart patient, diabetic",
"src": "Patient: hi my father has some sort of problem sometime he just goes blank and does not respond to what we ask him after 15 mins or so he becomes normal and then he cannot remember what had happened. Recently he slept while shaving form on his face, when he woke up he could not remember how he got in bed. He is heart patient and he is diabetic as well. Kindly suggest Doctor: Dear it seems that your father suffer from vascular dementia.Dont worry on 21-th century there are a lot of way to treat these symptoms.You should consult a neurologist, do an brain examination and than start treatment.You should be carefull from hypoglicemia.Always see the level of glicemia when your father seems lost.Wish you all the best"
},
{
"id": 6584,
"tgt": "What should I do as I am not able to conceive after my miscarriage and also having stomach pain and irregular periods ?",
"src": "Patient: i had a miscarriage last year march and since then am not be able to get pregnant what can i do pls and i do ve stomach pain and my periods not regular Doctor: Hi! I understand your situation.Since it is year past ur miscarriage,u are not able to conceive,also ur menses are irregular,you need thorough examination for that. Consult gynaecologist."
},
{
"id": 36542,
"tgt": "What causes fever, chilling and cold sweats?",
"src": "Patient: I've had a high temperature for nearly 2 weeks, it is worse during the night! If I haven't got a high temp I have cold chills and have excessive cold sweats! I ache all over and have a real tight chest worse with the high temp and also have a bad headache which again is worse when temp is high? Any ideas? Doctor: Thanks for your query at HCM!Your fever has been persisting beyond 2 weeks. You must visit a doctor.You need thorough examination and set of investigations to come to diagnosis whether viral, bacterial or tubercular in nature.Take paracetamol and nasal decongestants. Do saline gargles 6-8 times/day.You probably have had a secondary infection over your primary viral infection.You will need antibiotic prescription.Take care!Dr. Sheetal VermaInfectious Disease Specialsit"
},
{
"id": 45196,
"tgt": "What treatment should I take for follicular ovarian cyst ?",
"src": "Patient: hello doctor ,I m 30 years old and presently have one problem that i have follicular ovarian cyst .Apart from that everything is normal for me. 1.What is the criticality of that problem? 2.Can you suggest any best treatment in Chennai? Doctor: . About one-fourth of women with this type of cyst experience pain. Usually, these cysts produce no symptoms and disappear by themselves within a few months. Functional ovarian cysts are the most common type of ovarian cyst. They usually disappear by themselves and seldom require treatment. Growths that become abnormally large or last longer than a few months should be removed or examined to determine if they are a sign of a more serious condition."
},
{
"id": 11332,
"tgt": "How to use mintop 2% for hair loss?",
"src": "Patient: Hi, I have small frontal hairloss on my forehead. After consulting a dermatologist, have been advised to use mintop 2%. can you please let me know how should i be using it. I need the exact steps to be followed to apply head. My patch is the size of 1 rupee coin in forehead Doctor: Hello. Thanks for writing to us at healthcaremagicMinoxidil is a hair growth promoter.It can be used in various hair loss conditions affecting the scalp like Androgenetic Alopecia, Alopecia Areata and Telogen Effluvium.Minoxidil should be applied on thoroughly clean scalp preferably after a shampoo.It should be used twice daily.Only the recommended amount should be used. The quantity should never exceed 1 ml in a single application.The hair should not be applied on wet hair, hair should be air dried before applying minoxidil.Hair oils should be avoided while on therapy with Minoxidil because Minoxidil solution is available in an alcohol base and hair oils would prevent penetration of Minoxidil.Regards"
},
{
"id": 43420,
"tgt": "History of miscarriage. Now taking Bigomet and Eltroxin on empty stomach. How to conceive?",
"src": "Patient: hi doctor i just want to know that last august 2012 i met with miscarriage but after that i am taking bigomet sr 500 mg twice a day and in early morning i am taking eltroxin 25mg with empty stomach these two medication i am taking now my question is from last three months we are trying to concieve a child but it dosen t so please can you tell me what to do mentioned two medicine is continouing from august 2012 to till date.... i am so much frustrated as my marriage is of 2.5 years and my age is 27 and height is 5 5 inches and weight is 80.....please reply as soon as possible..... Doctor: Hi,Thanks for your query. I read your query and I understand your concerns. Following is my reply:1) You are overweight which is a problem for conception. Please start diet and excercise to loose weight and increase conception chances and decreasing abortion chances.2) Bigomet might be given to you for PCOS by your doctor i guess. Continue if so.3) Eltroxin has been prescribed for hypothyroidism. Get level of TSH tested to see if medicine dose needs to be adjusted.I hope I answered your query. I will be available for any follow up query you have.Regards,Dr. Mahesh KoregolIVF & Infertility Specialist"
},
{
"id": 41927,
"tgt": "Suggest treatment for brown color semen",
"src": "Patient: from few days i am in great trouble, because of my sperm of brown color, i am worried about this, is this normal? what is the cause and symptom for this..i dont have any pain in penis, i even had urine test ..reports were normal, just worried for brown colored sperm. Doctor: HelloWelcome to healthcare magicAs per your query regarding brown color semen.There is no pain and not any problem shows in urine examination.In this type of case you need to do Semen Analysis report. If this report shows any abnormalities; we can do as per the report.If everything is normal, Ayurveda medicines for purifying semen treatment will make this problem solve.But, in first of all needed to have semen analysis report.Regards,Dr. Nikul PatelAyurveda Infertility ExpertAhmedabad, Indialifecareayurveda@gmail.com"
},
{
"id": 3046,
"tgt": "What causes sore breast and stomach pain with faint second line in pregnancy test?",
"src": "Patient: ive had unprotected sex last monday and saturday not sure if he ejaculated , but my boobs hurt and my stomachs really sore , i took a pregnancy test and it showed up one line and one faint line but the faint line kept disapearing , i also have my period every 3 months because it gets quite painful not sure when i get it next as i dont count , i also have forgotten the pill Doctor: Hallow Dear, I would have appreciated had you mentioned the date of your last period. The fact that your pregnancy test is positive suggests that you are pregnant. However, faint second line is an issue of concern. The main causes of such weak positive test are:1. Ectopic pregnancy: The pregnancy in such cases is implanted somewhere outside the uterine cavity, commonly the tube. Such pregnancy does not grow and subsequently ruptures to cause profuse bleeding in the abdomen. It requires emergency surgical intervention with many units of blood transfusion. Such condition is associated with extreme pain, which I believe you are having. Ultrasonography will help you. If in spite of positive pregnancy test, the uterine cavity is empty, it is most likely ectopic pregnancy. 2. Missed abortion of recent origin when the test is on the way of becoming negative. Ultrasonography can help detecting such condition. If detected, it needs immediate evacuation of the uterus. 3. Test performed too early in normal pregnancy. Subsequently it becomes strong positive. Weak positive pregnancy test, particularly with pain in abdomen should not be taken lightly and should always be investigated with ultrasonography. Please report to your Gynaecologist ASAP. I hope you got the message. Dr. Nishikant Shrotri"
},
{
"id": 221646,
"tgt": "Is increased tiredness and sleep during pregnancy normal?",
"src": "Patient: I am 37 weeks pregnant, my plug has been coming away in bits for a week now, baby is 3/5 engaged and I am really struggling to stay away. This last week I have been overcome with tiredness, I will sleep through the night and within 3 hours of waking will need a nap. I am healthy and pregnancy has been fine. What could this mean? Doctor: Hallow Mumma,Expulsion of mucous plug (show) and the head engaged 3/5 are the signs that you are in imminent labour. You have finished 37 weeks of pregnancy. By this time, the baby is well matured. So you need not worry about the start of labour. During last few weeks, the woman is indeed having awkward physical position and hence does not get proper sleep which may give some fatigue. However, now that you are about to deliver, you will be OK after delivery. Bear down for short period now. Till that you may try relaxing in easy chair. Hope this helps you."
},
{
"id": 186101,
"tgt": "How can i get rid of the gaps in my teeth?",
"src": "Patient: Hello, I am currently using Orthofill bands that are used to close the gaps in my teeth. My gap is completely closed with the band on, but whenever I take the band off the gap slowly comes back. Is there any way that I can keep my gap closed, so that I can finally stop wearing the bands? Doctor: Hi! Welcome to Healthcaremagic.I read your query. Orthofill bands have good reviews regarding closing gap between teeth. However, it does not rectify the underlying condition of mal occlusion of teeth.I suggest you visit an orthodontist and get your dentition checked. You may be required to get x-rays done. After studying your x-rays and dental casts, a treatment plan will be made which may require removal of one or more teeth to make space and then proper teeth allignment with braces.Even after that, retainers are put on lingual side of teeth to prevent relapse.So even after orthofill, You need to wear retainer.Best is to consult an orthodontist.Hope the answer helps you. Thank you!"
},
{
"id": 170421,
"tgt": "What causes elevate SGOT and SGPT levels?",
"src": "Patient: Dear Dr, my daughter is 2years 9 months old she is taking EPIVAL for the last 7 months as per the Doctor advice , last week she is suffering from fever given Crocin DX 7ml every 5 hours for 3 days . now she is feeling very tired for the last 2days so we have taken to our doctor as per his advice we have tested the sgot and sGPT level it showing S.G.O.T - 42.9 S.G.P.T -21.7 last report taken 4 months earlier S.G.O.T - 33.3 S.G.P.T - 13.2 the rise in the S.G.O.T and S.G.P.T is due to continous medicine give or because of the fever ? is new level of S.G.O.T and S.G.P.T is indication of any liver problem ?if so how to control it and get things normal ? Please help me . thanking you in advance. Doctor: Hi...coming to your queries directly - 1. SGOT and SGPT levels of your kid are not alarming - do not worry.2. The raise could be either due to fever or due to paracetamol which was given to control temperature.3. it could also be due to the Epival which is being administered.4. But whatever is the cause - as the levels are not alarmingly high - you need not worry about them.5. They will come down automatically. Regards - Dr. Sumanth"
},
{
"id": 31355,
"tgt": "What causes itchy clitoris with swelling?",
"src": "Patient: Hi, may I answer your health queries right now ? Please type your query here..in july2009 a obgyn told me to use cetaphil soap stating that my wash was causing me yeast infections.i got terrible clitoris pain senstivity and swelling.swelling was bad and took a week to go awaythe pain never went away and is constant.its an awful striking pain.been prescribed steriod creams and antiinflammatory but no relief what could this be? Doctor: Hi, Thanks for posting in HCM. I understand your concern. What problems you are facing like itchy clitoris with swelling could be related to superficial skin infection. Maintain good genital hygiene by washing the area everyday twice with warm water and mild antiseptic like Savlon. Kindly apply Neomycin with Hydrocortisone cream after drying. For itching, you need to take Tablet. Levocetrizine once a day. Gradually, your symptoms should subside.Hope the information provided would be helpful.All the best."
},
{
"id": 112421,
"tgt": "Pain in lower back left side. Taking medication, still painful. Referred to neurologist. Opinion?",
"src": "Patient: i went to my doctor complain ing of back pain in lowetleft side around mywaist he wrote arthopathy 716,90 and 724 as my diagnosis to my short term insurance .i was taken off work for 4 weeks i still feel the pain but i originally thought i pulledamuscle now i have BEEN REFERD TO A NUROLOGISt want to know why would this get claim denied Doctor: hi i had gone through your query and understand your concerns. i would come up with two most possibilities to fit your condition .these include1.first possibility is,RENAL STONES;ultrasound scanning for KUB may confirm the diagnosis.2.second possibility is,DIVERTICULITIS;inflammation of balloon-like protuberance due to excess Gas pressure in intestines.often caused by additives in food,or low blood sugar.x-ray with barium enema may confirm the diagnosis. diet to follow;-high fibre diet which produces large volume of faeces requiring wider diameter of colon,and gas. -refined food food lead to hard stool.softer stools do not strain the gut.I hope this is helpful for you, thank you"
},
{
"id": 90719,
"tgt": "Does ovarian cyst causes abdominal pain?",
"src": "Patient: I m having cyst in my right ovary had been given Elogen and gluciphage.Got ultrasound done in Sep'11,rest is normal.Got CA 125 test done ,it was negative.Since 20days i m having pain in right side in lower abdomen and it moves to right side also.I can feel something gets stiff there and sumtimes in right also.What it could be??? Doctor: HI.Certainly this can be a twisted cyst.Get an urgent ultrasonography and color doppler done to get a real diagnosis and treatment as may be required. Be ready for surgical intervention if need be."
},
{
"id": 5011,
"tgt": "TTC. Regular periods after treatment. Prescribed crisanta tablet, apcod with sipen tablet. Right medication?",
"src": "Patient: SIR I AM 26 YEAR AND GOT MARRIED SINCE 15 MONTHS.I WAS NOT HAVING MY PERIOD REGULARLY,BUT AFTER MY DOCTORS`S TREATMENT NOW I HAVING MY PERIODS ON TIME.I M UNDER TREATMENT FROM LAST 4 MONTHS..IN BETWEEN SHE PRESCRIBED ME CRISANTA TAB,BUT I DON'T THINK THAT THERE IS ANY NEED OF CRISANTA TAB.PLZ TELL ME IS IT ?AND NOW SHE PRESCRIBE ME APCOD WITH SIPEN TAB.....I M CONFUSED WHETHER SHE DOING THE TREATMENT OR NOT? PLZ ANSWER .ME AND MY HUSBAND BOTH ARE MUCH WORRIED AND WANT A BABY SOON.PLZ HELP ME.....I M GETTING VERY STRESS. Doctor: hi, if you are tring to conceive, then may be you have to stop crisanta. generally it will take 3-6 months to regularize cycles. once it is stabalized, then you can stop if you are ttc. Siphene (ovilation stimulating) is given once you stop crisanta. APCOD looks like vitamin tablets. should not take both crisanta and siphene together, take one more month crisanta, then , stop it, start siphene later. be cool, if you are not happy with the doctor, take gynecologist opinion, be relaxedwishing you pregnancy soon."
},
{
"id": 92425,
"tgt": "Lower abdominal pain after falling from stairs. Matter of concern?",
"src": "Patient: I was walking downstairs with a bunch of empty water bottles to put in a recycling bin and being my clumsy self, i slipped and fell. I landed on my lower back and whats strange is that my back didn't hurt. It seemed as though all the shock had been transferred to my lower abdomen. What I'm worried about is that after an hour or so, it still hurts. Should i be worried? Doctor: HIThank for asking to HCMWait for some time at least 8 to 10 hours if your pain becomes more severe then it need to be examined, if it start declining gradually then nothing to worry about it but you will have to treat the pain too, Ibuprofen 400 mg is the best drug and try this, and I am sure this will come around soon take care and have nice day."
},
{
"id": 29454,
"tgt": "Does recovery from yeast infection take long?",
"src": "Patient: I a,m taking my second round of Fluconazole 150mg. I was on heavy antibiotics, 3 shots of Gentamicin and was left with a severe yeast infection.My Dr. sent me 3 more pills to day, to take daily for 3 days, would I be better to wait a few days between? In the past I took one a week. Doctor: Hi i do care for your concern. Yes the anti fungal tablet can be taken as once a day for three days, you have not mentioned about the yeast infection and i do guess it might be less severe so that your doctor prescribed three day course. Sure it will help you to get rid of yeast infection. You have not mentioned your age and also the reason for which the antibiotics were given. If the yeast infection is recurrent it is advisable to get a blood examination done along with blood glucose levels. Hope everything gets well soon. Hope i have answered your question, if you have more feel free to ask. Thank you."
},
{
"id": 181126,
"tgt": "How can a chipped tooth be repaired?",
"src": "Patient: I chipped a tooth in such a way that the bottom of it is very sharp and skinny, it looks fine from the front as it is chipped in the back, but the chip goes up about 1/4-1/3 of the tooth before it returns to normal thickness. Will i need a veneer or crown for this? or just bonding? how much do you think it will cost? Doctor: Hi..Thanks for the query.The exact treatment will depend on whether the nerve is exposed or not..So, first of all you should get an x ray of your chipped tooth done to see that the loss of tooth structure is superficial or it has reached deep to the nerves..If you have symptoms like sensitivity to hot and cold or pain in tooth then it can also be a sign of nerve exposure..If the nerve is not exposed then simply filling the tooth filled by capping of the tooth will help in saving the tooth..If the nerve is exposed then root canal treatment of tooth followed by capping will help in saving the tooth..Hope this helps..Regards.."
},
{
"id": 195393,
"tgt": "What causes low grade fever and face flushing?",
"src": "Patient: Hi. I seem to run a steady low-grade fever and I often have a great deal of face flushing. It creates redness and actual heat coming off my face. I stay tired and when this happens regularly, i often develop a few small sores on my back. My dad called it jungle rot when he got them. He was in vietnam and korea. Doctor: Hello and Welcome to \u2018Ask A Doctor\u2019 service. I have reviewed your query and here is my advice. Fever with facial rash may be seen in mums or measles or Acute viral pharyngotonsillitis or thrombocytopenia or associated with acne or boils etc..Until examination is done it is difficult to say what it is.Drink plenty of water, take rest. Eat soft diet .Please use tablet Pantoprazole thrice a day for five days, if symptoms not improved please consult your doctor he will examine do blood investigation and treat you accordingly.Take care"
},
{
"id": 149499,
"tgt": "Suffering from neuropathy symptoms, taking medicine for vitamin deficiency. When will I become normal?",
"src": "Patient: Hi Doctor, How many months are required to become normal ( once you start taking medicines) to get rid of vitamine deficiencey ? Especially B12, B6 etc ? I have just started suffering from aweful periferal neuropathy symptoms and I suspect its cause of vitamine deficiency. And how quickly can I get relief frm this neuropathy problem ? IT just started a week ago, but its unbearable. Your views would be greatly appriciated. Thanks. Doctor: Dear sir/madamIf neuropathy is caused due to diabetes and it is of long term, then most probably it will be not reversible.If you dont have diabetes and it is exactly happening due to vitamin deficiency the it will become normal.consult your neurologist and do one nerve conduction studies once..You will reveal lot of thingsthanks."
},
{
"id": 97490,
"tgt": "Suggest alternate medicine to cure deviated nasal septum",
"src": "Patient: i suffer from Deviated Nasal Septum and therefore have nasal blockage every night and this has started from last 3 yrs especially after my dad's demise which made me sad and worrysome and in trauma. My question is if dns causes nasal blockage why did it not occur before 3 yrs and i will not go for surgery so please advice allopath or homeopath medicine to cure it. thanks for your time sir. Doctor: Hello and thanks for using this forum .I am very sorry for your loss .A deviated Nasal septum (DNS) is either post traumatic like after an accident or congenital that since birth .In either the patient gives a history of obstructed breathing through the nose since birth or since the trauma .The other most likely cause for the nose block that you are experiencing is allergic rhinitis .The allergies in the human body can be due to altered immunity in response to allergens in the environment or mental stress .In your case I feel that the demise of your father could be trigger for the hypersensitivity causing the allergic rhinitis .In such a case surgery will not really be of great help .I shall be able to suggest the best Hom med for you if yu can answer the following questions What is your body type -lean / overweight /flabby /Muscular ?What are your favourite foods ? How much salt and sour do you like ?Do you also get discharge from nose ,sneezing at any time of the day ?Any relation to weather ?Thanks and take care .Dr Preeti Chauhanhttp://www.healthcaremagic.com/doctors/dr-preeti-chauhan/67212"
},
{
"id": 37628,
"tgt": "Do entamoeba coli, andolimax nana and blasticystis hominis parasites complicate pregnancy?",
"src": "Patient: I have been living in Africa and upon my return had lab work that said I had entamoeba coli, endolimax nana, and blastocystis hominis parasites. I have left them untreated as I understand they can go away on their own. I am yet to do the 6 month check to see if they have gone. What I want to confirm is if any of these can cause pregnancy complications? Doctor: Hi and thank you so much for this query.I am so sorry to hear about these parasites and all the disturbing thoughts this has caused you. There is no reason to get worried about these germs possibly complicating your pregnancy. They have not caused any infection and inmost cases would only do if the immune system were to be greatly weakened. This is simply not the case now. When you live in milieu with such germs, they form harmless relationships and there is no reason to be afraid of their mere presence in the absence of symptoms.I hope this addresses your query fully. I wish you well. Feel free to ask for more information and clarifications if need be."
},
{
"id": 169936,
"tgt": "What causes red lump on back of leg?",
"src": "Patient: I have a 10 yr old daughter that has been on methotrexate for a few years for RA. She now has a red lump on the back of her leg that is warm to the touch. She does not remember being bit by something and because she scratched it, it is hard to see if there is a bug bite at all. Doctor: Hi...keeping her history of RA in mind we cannot ignore this symptom. I feel that this could be a reactivation of the primary disease. I suggest that you get back to her pediatrician or pediatric rheumatologist. She will need evaluation of estimation of acute phase reactants before considering bug bite or allergy.Regards - Dr. Sumanth"
},
{
"id": 216465,
"tgt": "How to cure the pain in natal cleft?",
"src": "Patient: Last night when I showered I washed my rear end in the crack area. I don t know what happened but it is so sore and feels like its raw. It is above the anal area. It is sore when I sit. There are no bumps or open sores. Is there something that I can put on it that will take the pain away? Doctor: Hi, you seem to have hurt it while washing. Please use any anal cream available for piles with lidocaine and steroid in it three times a day. This will soothe the area. Regards"
},
{
"id": 99070,
"tgt": "Suggest treatment for bronchial asthma",
"src": "Patient: hello dr,my sin is 4 yr old he has severe allergy in nose and bronchail asthma he fall sick many times from august to march ..his problem starts from flu and ends in pnumunia now he is takinf sertide spray and nasonax but still he is geting sick every 15 days ..all start with sneezing and flu then cough starts and fever..plz send me your address i livd in khober ...please help me i m hell worried Doctor: HI, thanks for using healthcare magicIf he is frequently getting coughs/cold then you may want to consider the use of foods and vitamins that can boost his immune system.This would include fruits and vegetables , probiotics.Cough and cold is spread by contact with other persons with the cold through sneezing, coughing etc. The use of hygiene wipes and sanitizing may help as well.I hope this helps"
},
{
"id": 38459,
"tgt": "Can strep strain cause UTI and sore throat?",
"src": "Patient: I have been dating a guy for about a month now and we have been sexually active. He just told me that he has the B strain of strep throat, which his doctor told him was caused from oral sex. Does this mean I have strep as well? I ve been having UTI symptoms for the past year now and itching on the outer part of my vagina, would this be caused by the strep strain? Doctor: HelloUrinary tract infections can be caused by streptococci infection. It is possible for these infections to coexist and one causing the other. Do consult your doctor for examination for confirmation of diagnosis and treatment by the use of appropriate antibiotics.Thank you for writing to health care magic."
},
{
"id": 136509,
"tgt": "Suggest treatment for knee pain after an injury",
"src": "Patient: Yesterday morning I was playing volleyball with a partner and we collided. Her knee hit my outer thigh, right above my knee. The muscle tensed up making it difficult to bend/squat and is occasionally painful to put weight on (an example would be walking up stairs). I ve been icing/compressing/elevating it, but it s still quite painful even with pain medication. Is there anything else I should be doing? The pain is sometimes sharp, and causes my knee to buckle. If I m not moving then it s only somewhat sore. Doctor: hi i appreciate ur concern.it appears injury to your thigh muscles and ?strain onknee ligaments.it would be better if u do a x-ray of knee to rule out any significant damage.along with painkiller medicines,andicing,i always advice my patients to immobilize knee in a splint,which helps in healing of injured tissues.consulting ur orthopaedic doctor will help u much. thanks. for any further quaries ,i will be happy to answer."
},
{
"id": 7138,
"tgt": "What should i do to become pregnant ?",
"src": "Patient: I am 25 years old female.I wanna be pregnent and trying last 2 years but still not success,as per the doctor s suggestion I am taking medicine step by step.Lastly dr s prescribe to take pregnil 2 ampl 5000UI injection.I already took 3 cycle but still not success..what are the next please answer... Doctor: Hello Thanks for your query. Three cycles are not sufficient generally we give it for 6 cycles.Has any cause been detected or not in either of two.Avoid tension it also plays role in delaying pregnancy.Continue efforts & trying result will come sooner. \u2018Hope I have answered your query, I will be available to answer your follow up queries, \u201cWish you Good Health and trouble free speedy recovery\u201d"
},
{
"id": 34738,
"tgt": "Is the E-coli in urine and staph infection in throat contagious?",
"src": "Patient: Hi, i'm 42years old and am 31 weeks pregnant. My boyfriend has just been diagnosed with e-coli in his urine and a staf infection in his nose, he is on antibiotics and i want to know if this affects me also since we have a active sex life, i have a horrible smell from my urine like his and thats why i sent him to get tested in the first place, i had a urine sample taking this morning but have to wait a couple of days for the results, he also has been given referal for a scan for further research that might be an underlying problem, can any1 confirm if i could or can contract this from him sexually even though its not classified a std? THOUGHT THIS WAS A FREE SERVICE!!!! Doctor: hi,yes E.coli is transmitted sexually..its good if you can abstain from sex for a few days..but body resistence to E.coli varies from person to person..it is normal inhabitant of gut, causes disease in rare occassions..let the reports come..if your reports are positive then you have to take a course of antibiotics..hope my answer helps youif you have any more queries i would be happy to answer.."
},
{
"id": 95465,
"tgt": "Is there any cure for ulcer ?",
"src": "Patient: Hi I have been suffering from ulcer since three years. Is there any cure which remove ulcer from roots. Thanks and Best regards. Doctor: Hi, Drink Carrot mix with cabbage juice to cure ulcer. Take triphala to maintain good health."
},
{
"id": 76818,
"tgt": "Suggest medication for cough with phegm",
"src": "Patient: My almost 3 year old daughter is sick, she s clammy and has a bad cough with phlegm that she can t seem to cough up/spit out. She was a preemie weighing barely 2lbs, I ve heard respiratory infections are a huge cause for preemie hospitalization...but my question is, since she s passed the 2 year mark, does that still apply to her or can I wait till the morning to see her doctor? Doctor: Thanks for your question on Healthcare Magic. I can understand your concern. Yes, you are right. Your child was having low birth weight at the time of birth. These children are at increased risk of infections, particularly lung infection because of lower immunity. And this kind of risk is there until age of 5-6 years. Her current symptoms are more suggestive of lung infection. So better not to wait at home and consult pediatrician immediately. She will need chest x ray and blood reports for infection. She might need higher antibiotic and other supportive drugs. Don't worry, with appropriate treatment, she will be alright. Hope I have solved your query. I will be happy to help you further. Wishing good health to your daughter. Thanks."
},
{
"id": 144098,
"tgt": "Suggest treatment for arnold chiari malformation",
"src": "Patient: I have been diagnosed w arnold chiari malformation and am suffering from symptoms how is this treated and will it progress as i get older right now i have dizzy spells, pain in back arms and legs, speech problems and also loose my voice (horse alot) Doctor: Hi, I am Dr.Bruno. I have read your question with care and understand your concerns. Let me try to help you Question : Suggest treatment for arnold chair malformationAnswer : This is treated by a Surgery Called Foramen Magnum Decompression. Question : will it progress as i get olderAnswer : Yes. In Most people, unless Foramen Magnum Decompression is done, this will progressHope you found the answer helpful.If you need any clarification / have doubts / have additional questions / have follow up questions, then please do not hesitate in asking again. I will be happy to answer your questions.Let me know if I can assist you further.Take care."
},
{
"id": 160382,
"tgt": "Suggest treatment for severe headache in a child",
"src": "Patient: my 8yr old son was fine all day sunday, then came in @ 4pm and took a nap until 6pm and woke up with severe headache and then started throwing up. He complained all that night of his head and stomach hurting. Yesterday the vomitting stopped but the headache and stomach pain was still there but not as intense. Today he is fine and eating again. What do you think this was? Doctor: Hi,He had any similar episodes or any history of migraine in the past? Headache, abdominal discomfort and vomiting can suggest a viral illness or early gastroenteritis. Sometimes migraine also can have a similar presentation, although here abdominal pain is less common. Anyhow, as he is getting better now, we need not worry now. Give paracetamol or ibuprofen for pain if needed. If symptoms does not clear out in 2-3 days, kindly take him to a pediatrician.Take care. Hope I have answered your question. Let me know if I can assist you further. Regards, Dr. Muhammed Aslam T. K., Pediatrician"
},
{
"id": 204336,
"tgt": "Is Zanaprin an effective alternative for Xanax?",
"src": "Patient: Hi I am in the process of getting off Xanax is Zanaprin the best alternitive. I have heard of another alternative that sounds like Zanaprin but Im not sure if I am correct. Plse let me know if their is another medicine that sounds like Zanaprin I really looking forward to getting of xanax . Doctor: Hello and Welcome to \u2018Ask A Doctor\u2019 service. I have reviewed your query and here is my advice. First of all I like to make it clear that there is no best or worst medication for anxiety. There are multiple options which can help one person but may not help other person with similar symptoms. Secondly Zanaprin is one alternative and can be used as an alternative to xanax as it is less habit forming in compare to xanax. Hope I have answered your query. Let me know if I can assist you further."
},
{
"id": 55005,
"tgt": "What is the life expectancy in a case of advanced Cirrhosis with portal hypertension?",
"src": "Patient: Hi I have cirrhosis of the liver in advanced stages and was recently told i have portal hypertension and i am wondering if this is very serious. What is an estimated prognosis guesstimate of someone who has these health issues as life span? I know its only a guess and not concrete i just want to know an approx number of years Doctor: Hi thanks for asking question.As you have portal hypertension it doesn't mean you will die early..Life expectancy depend on many factors like episodes of variceal bleeding, hepatic vein pressure , life style care you are taking etc...Patient have very bad prognosis and die within 1 year only if hepatic vein pressure above 20 and more then one episode of variceal bleed.Other wise 5 to 10 year survival can be there.To reduce death rate treat portal hypertension by beta blocker ..And trans jugular intra hepatic shunt can be put for lowering portal hypertension.For ascites take diuretic and low salt diet.Death in portal hypertension mainly due to variceal bleeding so whenever you have blood in vomit immediately consult surgeon for endoscopic treatment.Cutt off oil in diet.I hope your concern solved.Take care..."
},
{
"id": 110346,
"tgt": "Suggest treatment for severe lower back pain",
"src": "Patient: Hi. I have been having severe lower back pain to the point i couldnt walk. When i do walk, i walk cooked and my right shoulder pushes out over my pelvis. Now, the muscles along my spine on the left side, lower back, are hard as a rock. My physical therapist said she has never seen it before and i still have pain in my back and even when i stand up, my left hip is higher than my righr..she said the muscles are causing my spine to curve. The back pain started 8 days ago and finally got better 2 days ago..just enough to where i can walk. Doctor: Hi Welcome to healthcare magic.YOUR MUSCLES ARE in sever spasm It can be due to disc prolapse .Treatment of disc prolapse is rest in posture of comfort.Analgesics(Tramadol 50mg three times a day after meals along with omeprazole20mg empty stomach daily). MRI is done to confirm it. I think your query is answered"
},
{
"id": 124424,
"tgt": "When can walking be resumed after surgery?",
"src": "Patient: Hi, i broke my tibia bone while sprinting, it got operated and they kept 2 screws inside, this is about 5 weeks back, how long does it take to resume walking ? And is it true that, surgical procedure of tibia fracture leads to arthritis in future ? Thanks Joe Doctor: Hi, Two to months immobilization is required generally. Physiotherapy is required to regain full range of motion. Hope I have answered your query. Let me know if I can assist you further. Regards, Dr. Shinas Hussain, General & Family Physician"
},
{
"id": 110402,
"tgt": "Could chornic back pain be related to kidney dysfunction at childhood?",
"src": "Patient: I am a fit 56 year old male who was hospitalized for 6 weeks for the treatment of Nephritus (when I was a toddler). I suffer from chronic lower back pain, and have been taking NSAID s to battle this, as well as a good exercise regimen. Could my back pain be related to kydney dysfunction, and what would be the red-flags on a blood test (that was recently performed). Thanks. Doctor: Hello,WELCOME TO HEALTHCARE MAGIC.Your back pain can be very well related to kidney dysfunction as vitamin D get activated in kidney.So in kidney disease there is deficiency of active vitamin D leading to weak bones and backache.So regular vitamin D 3 test is required. and it is deficient then it can be supplemented.Wishing this advice will be of some use to you."
},
{
"id": 79153,
"tgt": "What should I do for the tightness and pressure in my chest?",
"src": "Patient: tightness or pressure in chest areaI'm on Atenelol, they say I have A fast heart rate caused by stress because all my test have come back normal. My vitals have been fine but am feeling occasional tightness, pressure and pain in the chest. Should i be worried and what should I do? I've tried to limit my caffeine. Thanks Doctor: thanks for your questiontightness or chest pain can be due to cardiac issues like MI , angina ,or anxiety and others or lung issues or gastritisyou need to visit a physician who can request for a chest X-ray and an ecg and a 2 d echo if required to evaluate and rule out the organic diseases or causesin my patients I usually give them aspirin and a proton pump inhibitor and then evaluate them thanksfeel free to ask more questions"
},
{
"id": 129530,
"tgt": "How to treat pain from left hip down to toe?",
"src": "Patient: I am diabetic. recently have changed/improved my diet, lost a lot of weight. Seem to have wrenched my back on one side. Now have pain from the left hip down to that toe. Can sit and walk pretty OK, but it s hard to sleep despite having just bought a new firm mattress. Am taking Advil and using a heating pad in bed, but seem to need something stronger. What to do? Doctor: Hi,Its due to disc problem or hip problem in your case.Get xray of lumbosacral spine in AP and Lateral view, xray of pelvis with both hips in AP and send me the report.Till then have tab aceclofenac+thiocolchicoside combination by prescription of your family doctor.Thanks."
},
{
"id": 182516,
"tgt": "Should Coumadin be discontinued prior to dental surgery?",
"src": "Patient: My mom has AFib. She went to the dentist and has 3 infected teeth. She is on Coumadin.. Vicodin..she had double knee replacement surgery this April...both knees done at once. She is 83. Heart surgeon wants her off Coumadin fir 5 days before dental surgery..PCP says no, stay on it. Any thoughts on stay on or off Coumadin Doctor: Hello,A very thorough medical history is absolutely necessary before any dental treatment. A blood thinner such as Coumadin affects the body's ability to clot. This is the initial stage of healing required after an extraction. If the clot is disturbed, delayed healing may occur. A dry socket or infection after the extraction is a risk. I routinely take my patient's off blood thinners including aspirin. If there are high risk conditions, I refer to the oral surgeon specialist. All doctors and specialists should consult, discuss risks and agree to treatment.Premedication for dental treatment, especially extractions, is necessary after knee replacement surgery. An antibiotic such as Amoxicillin should be prescribed. I suggest a consultation with an oral surgeon. Factors such as the amount of tooth in bone and risks need to be considered. Surgical complication risks will also need to be considered when choosing a location. I hope this is helpful and thank you for your inquiry."
},
{
"id": 60043,
"tgt": "Suffering from hepatitis B, have hepatitis B virus DNA quantitative within normal range. What does this indicate?",
"src": "Patient: HELLO doctor my husband has been suffering from Hepetitis B since 2007.His HBV REAL TIME QUANTITAVE are H 0000 copies/mL in 2008. Now his HEPATITIS B VIRUS DNA QUANTITAVE(HBV VIRAL LOAD BY TAQ MAN) are 180IU/ML.What does it indicates? Is this is satisfactory? He is not taking any medicine yet . Shall I give him LIV52 HB or not? Please give me a satisfactory answer Doctor: Hello Regular estimation of HBV VIRAL LOAD gives an indication to your physician that how fast is virus multiplying on you. Treatment depends upon tne viral load levels . As your husbands viral load is less,he doesn't require any treatment and needs only regular monitoring of VIRAL LOAD. there is no role of medicines like LIV 52 in tnetreatment. Thanks"
},
{
"id": 143558,
"tgt": "Could the falling down and hurting the neck be due the Seizure?",
"src": "Patient: I think I Had a seizure yesterday. I feel on my left side hit my head and really hurt my kneck. When i came to i was extremly disorientated.My Name is: Suzanne Rutto Lake. I am 53 a smoker but not a heavy one. I don t drink. I am Hep c positive but that went into remission. Doctor: Hi yes it is possible that after seizure you had fall and head and neck injury was caused due to it .It is advisable to you to consult neurosurgeon to r/o any severe injury to head and neck.Hope my advice will be helpful to you.Thanks."
},
{
"id": 128454,
"tgt": "What are the symptoms of Gout?",
"src": "Patient: Hi, it started yesterday around midday. I got a throbbing pain in my left thumb. It got worse, later my whole thumb was sore. It felt as it it was in the joint then, during the course of the afternoon the pain travelled to my wrist and then to my elbow. Finally to my shoulder. I don t live close to a good hospital and called a cousin who is in the medical field. He said it sounds like gout. Last night I felt a deep throbbing pain in the back of my shoulder/chest and read on the internet this morning that gout doesn t travel from joint to joint . My thumb is also still sore. Can this be gout? I m am a woman of 49, my weight is 63kg I don t smoke or drink and my blood pressure is low. I don t have cholesterol problems either. Doctor: Dear patient Gout usually affects metatarsophalangeal joint of great toe first and if chronic may spread to other joints. It is not migrating joint pain. You amy be having Inflammatory joint pain. You should start tab zerodol sp twice a day for 5 days. If not relieved please consult orthopaedic surgeon nearby your area."
},
{
"id": 95232,
"tgt": "What could be the reason for the dull ache on the left side of the abdomen ?",
"src": "Patient: Hi Doc,plz i have an enquiry.My testicles are functioning optimally and all is well. I do sometimes have a dull ache on the left side of my abdomen ,itz mild and doesnt really disturb,but it gets me wondering,plz what could be responsible? Also,whenever my scrotum is relaxed,i see a vein on the side of my left testicle,it appears to be on it,itz towards the back where the epydidymis is,plz is it normal? So also,i do feel the tubes connected to the left testes more prominently than that of the right,i used to freak out,but i am over it now,is it normal,sir or ma? I am 19 yrs old,i 1.98m tall and i have no record of medical disability . Doctor: Hello.Welcome to HCM forum.Kindly get an ultrasound abdomen along with scrotal Doppler done.Also get hemogram and urine microscopy done.Consult a surgeon.Every thing will become clear.Thanks and good luck."
},
{
"id": 180037,
"tgt": "Is it safe to give clarithromycin to a child for sinus infection?",
"src": "Patient: Hi, my 4yrs old has been diagnosed to have an acute sinus after a continuous experience of on-and-off cough, running nose and sometimes fever for months. I thought it was a regular flu until i feel that it is too often. The doctor said it was originally caused by an allergy, i haven t check it yet. So now, the doctor gave my kid 2 bottle of abbotic clarithromycin 3/4 tbsp twice a day. Is it safe? He also suggest to get him vaporized and shine with a micro wave or something daily for about 2 weeks. Is it the way to cure sinus? Thank you. Cheers, Angel Doctor: Thanks for putting up your query at HealthCareMagic. I understand your concern for your 4 year child. Do not worry about the antibiotic clarithromycin. It is safe and effective. Unfortunately, sinuses are air pockets deep inside our face and their blood supply is poor and often the antibiotics cannot reach there to make an effect. So, at times no antibiotic is successful in providing good relief and hence it is difficult to cure sinusitis. But generally the ailment comes down with age. I guess that you have to be a bit patient and wait to see if they give your son relief. Steam inhalation is a very effective method where water vapour goes deep inside and dilutes the cough and eases the situation.I hope that helps. Feel free to revert back in case of further queries if any."
},
{
"id": 24117,
"tgt": "Is surgery required for atrial secundum defect of the heart ?",
"src": "Patient: Good day, my baby was diagnosed with Atrial Secundum Defect of the Heart when he was only 10 months old. We have tried to consult a doctor and was adviced for a immediate operation of my son since the hole is already big. As we consult another doctor, she adviced that before school age- a heart operation must be done. My question is : is there any chances of not getting a operation for my son ; the thought that his disease is not curable but for operation but as we have gathered few informations from few of our friends,that there were cases like his that have not undergone an operation and was then unholed as time passes. We need your advice on this since we have been thinking of not taking him for operation because we are really afraid of any risk. As parent, we want to be with him for a long time. Doctor: Hi,The hole should be closed. Patients with hole on heart will develop some pathological changes in their hearts with time depending the location and size of the hole. Surgery is done to prevent those complications and heart failure.I would advise you to perform surgery sooner, as sometimes patients refer to physician when there are already such complications, which are contraindication for the surgery.Take care"
},
{
"id": 129002,
"tgt": "What causes injury in the shoulder post an accident?",
"src": "Patient: I fell 8 days ago and injured shoulder. I grabbed a limb as I was falling then dropped 8 ft. Pain has decreased, but it is still painful to reach up and up/ out. I am confident there was not a break, but not sure how long to expect this to resolve or if I need to see a doctor...suggestions? Doctor: Hello,Thank you for using Healthcaremagic.I read your question and understood your concern.I think you need to see a doctor definitely. It may be a fracture too.Dr. Selmani"
},
{
"id": 208958,
"tgt": "Suggest treatment for panic disorder,and how can it be avoided?",
"src": "Patient: hi there been suffering for panic disorder for years now...always feels like having a heart attack...is it possible to be cured?i heard this is for lifetime?is it true?what are the factors to be avoided for me not to panic?and foods to be avoided?tn in advance i have no budget to go to a private doctor thats why im looking for alternative meds by the way im only 25 im only 21 when i was diagnosed with it...im 5 2 ive got allegic conjunctivitis and i am acidic..im always making such sounds like ehem just to clear my throat coz it feels like i have phlegm but im not Doctor: DearWe understand your concernsI went through your details. I suggest you not to worry much. Panic disorder could be cured completely with the help of medicines and psychotherapy. The approach depends on the severity of the case and pathogenic. To diagnose the severity, I need a detailed description. Once the detailed description is handy, I shall advice you about the psychotherapy methods to be used. Meanwhile you can do the following:Step 1 - Practice Deep and full breathing whenever possibleStep 2 - Drink plenty of water and fruit juices. Step 3 - A Tomato a day keeps anxiety at bayStep 4 - Exercise regularlyStep 5 - Avoid red meat and spicesIf you require more of my help in this aspect, Please post a direct question to me in this website. Make sure that you include every minute details possible. I shall prescribe some psychotherapy techniques which should help you cure your condition further.Hope this answers your query. Available for further clarifications.Good luck."
},
{
"id": 224416,
"tgt": "Can i get pregnant during the time of missing few Falmina pills?",
"src": "Patient: I missed 5 days in a row in the first week of falmina. I had unprotected sex during the time the pills were missed. No other forms on contraceptive was used. I have been on the pill for 6 years. What are the chances I could have become pregnant during the time the pills were missed? Doctor: Hello there,Thank you for the question.Oral contraceptive pills like Falmina work efficiently when taken correctly. If you miss more than 2 doses of these pills in a month then their efficiency goes down considerably. Now that you have missed 5 doses of the pill that too in the first week they are completely inefficient and there is a good chance that you could get pregnant from the intercourse during that period. Although in the first week after a period it is the safe period and so it is less likely but the chance cannot be denied. So if you miss your period by 2-3 days then please get a urine pregnancy test to confirm that you are not pregnant. Try not to miss more than 2 doses in a month and if you do always use alternative forms of contraception. If you miss a dose take it immediately when you remember or take it next day with that day's pill. Hope this helps.Take care."
},
{
"id": 177064,
"tgt": "Suggest ways to improve immune system in children",
"src": "Patient: My kids are very skinny and one is prone to infection. How can i help his immunity fast and then help him to gain some fat. He has always been so skinny since he started school was fine up till 5-6 age. But has always caught strange infection. No docs r helping and am stressed out. Hes always ill. Doctor: their are no medicines to improve immunity. It is Innate and hereditary.proper nutrition is the best way to keep the immunity going. Help your child with a lot of anti oxidant like fresh fruits and fruit juices. Also give a lot of fresh milk and other dairy products. Certain immunity boosters and provitamin preparations are available in the market which u can try(although not of much help).probiotics for proper digestion, good amount of proteins in form of meat and cereals, proper carb diet will help him gro strong internally. your child will acquire weight and height later on which we sometimes call as Catch up growth."
},
{
"id": 34642,
"tgt": "Is herpes 1 a contagious disease?",
"src": "Patient: i will like to know if herpes 1 is something serious, i am a 29 year old female, my practioner called with my blood test results and said it came back positive. I will like to know if this type is contagious and if its an std, she said its due to sun, dehydration and low blood count. So is this true? Doctor: Hello,herpes virus type 1 (HSV-1) is a virus which means that it is contagious. It's been transmitted to you by somebody. Most people have HSV-1 and they may experience exacerbations from time to time (cold sores on the lips). The virus lies dormant inside your nervous system. This is nothing very serious to worry about unless the exacerbations are very frequent (many times per year). It's not due to the sun or dehydration or the low blood count but these conditions (except the sun) may cause reactivation of the virus.I'll be glad to answer any further questions you may have on this subject.Kind Regards!"
},
{
"id": 180102,
"tgt": "Suggest medication for constipation",
"src": "Patient: My daughter likes to hold her poop in for days for one then when she does go her poop is very light color . and when she pushes hard to strain a little blood appears with it. Now her bottom between the two cheeks close by the hole of the but is really red and raw. what can I do or is it more serious to see her doctor. Doctor: Thank you for asking querry.Your child is suffering from constipation.consult your physician.First stool disimpaction is important,give either glycerine suppository per rectal or she may need enema.Change diet,include more fibre in deit like fruits ,green vegetables.15 days or 1 month course of lactulose 5-10 ml dialy.Hope you find this helpful.Rate the above answer if you like it.Regards,Dr rushikesh kute"
},
{
"id": 123819,
"tgt": "What Are the symptoms of a bruised tailbone?",
"src": "Patient: I am a figure skater and I fell a lot on my tailbone. My stomach had been hurting and I feel like I am going to throw up. I also feel pain in between my ribs. My back also hurts. Can you explain where the pain is coming from and what should I do about it? Doctor: Hello, Possible symptoms of tail bone injury are: 1. Severe localized pain and tenderness may be felt in the tailbone area. 2. If the injury is traumatic, a bruise may be visible in this area. 3. The pain is generally worse when sitting for prolonged periods of time or with direct pressure to the tailbone area. 4. Bowel movements and straining are often painful. 5. Some women may experience pain during sexual intercourse. Your symptoms may not specific to the tail bone problem. It seems you may be having cervical or thoracic spondylosis with compressive neuropathy it seems. But until the examination is done it is difficult to say what it is. Use tablet tramadol with paracetamol. If symptoms not improved please consult your doctor he will examine and treat you accordingly. Hope I have answered your query. Let me know if I can assist you further. Regards, Dr. Penchila Prasad Kandikattu, Internal Medicine Specialist"
},
{
"id": 165869,
"tgt": "Can walking cause tiredness and exhaustion in a child?",
"src": "Patient: hia my daughter is 3 and has just done a full first week of morning nursery and when i picked her up today she has been whinging ever since about odd little things not painsjust genral moaning she has been nursery before but no more than 2 days a week she has a little bit of a tempreature she does a 15-20min walk there and back everyday she hasnt done anythng since she came home just laid and sat down could the temreature be caused by being over exausted she waS ALSO UP EARLY this morning 6:30 and she isnt used to walking that far she had 30mins sleep wen she came back but still reacted the same when she woke up Doctor: it looks like ur child is tired but neverthless viral infection cannot be ruled out plz give her paracetomol 15mg/kg consult ur pediatrician"
},
{
"id": 182962,
"tgt": "Suggest medication for swollen gums",
"src": "Patient: my daughter is 8 years old her lips are swollen and have swores: the skin is falling off. Her gums are swollen and red. for the past two days she's had a high fever and has vomitted once. she has trouble eating and it hurts to open her mouth. please help Doctor: Thanks for using Health care Magic.Read your query.Viral fever is usually the cause of this symptoms in the oral cavity.I would advice you to control the fever and consult your pediatrician if it is not in control.For the swollen gums, it will reduce gradually once the fever comes down. Avoid anything spicy and hard food. Increase the fluid intake and keep the child hydrated.Mucopain gel/dentogel application will provide relief for the sores.If still persisting once the fever comes down, consult the dentist.Hope this was useful.Thanks and regards."
},
{
"id": 68747,
"tgt": "What causes lump in the spinal column?",
"src": "Patient: My 14 year old daughter had Chiari Malformation decompression surgery when she was two. About a year ago she developed a \"lump\", hard to the touch, at the top of her spinal column (base of her neck). Her former physician suggested warm compresses for the discomfort, but never seemed too concerned.We saw a new doctor this a.m. and now we are being immediately referred back to her neurosurgeon. Should I be scared? Doctor: welcome to Health care magic.1.Its post operative changes seen at the site of operation.2.Initially an ultrasound scan to find out what it is consists of and where it is extending from and the nature of the lesion ( cystic , solid mixed)3.If not helpful then an MRI will help in evaluation of the neural elements.4.Yes the cad has to be seen by neurology specialist.5.Depending in the situation a repeat operation may needed.Hope it helps you. Wish you a good health.Anything to ask ? do not hesitate. Thank you."
},
{
"id": 185425,
"tgt": "What causes pain in jaw opening?",
"src": "Patient: SIR THIS IS ABHAY KARNAVAT AGE 38YRS, HEIGHT5.7INCHES, WEIGHT 69KGS, I HAVE PAIN IN JAW OPENING RECENTLY, USUALLY DISLOCATION AFTER YAWNING, I HAVE PAIN/HEATED TOE ON LONG TIME STANDING, NO EXCESS BONE GROWN, DIFFICIENCY OF VIT B12 DECTECTED, B12 TABLETS GIVEN, IS IT ENOUGH? PLS ADVICE. Doctor: Thanks for using Health Care Magic.Read your query.Usually the TMJ dislocation is because of chronic strain or excess load on your jaw joint and also sometimes keeping your mouth open for longer period or opening too wide can cause dislocation.Avoid opening your mouth too wide.Intake of soft diet is recommended.You can continue with the vitamins Consult a oral surgeon and get the joint reduced and apply bandage around the head stabilizing the joint.If the dislocation is frequent take a advice of oral surgeon for surgery.Hope this was useful.Thanks and regards."
},
{
"id": 181253,
"tgt": "Suggest treatment for a stuck syringe in the gums",
"src": "Patient: i got a xray today,to check for a dentist syringe that the dentist said he broke off 1974 ,i took a stroke 2001 just make sure .the clot 4.5 by5.5.the xray was neg.but mri say different.by the way the dental syringe is 4.57.should i get another mri? Doctor: Hi..Welcome to HEALTHCARE MAGIC..I have gone through your query and can understand your concerns..As per your complain it seems that you have broken needle struck into your gums due to breakage while anaesthesia injection that is being shown in your MRI..You should not go for MRI Scan again and consult an Oral Surgeon and get evaluated along with a close examination of your MRI..The removal of the broken tip of needle is a a must from the tissues as it can act as a foreign body and can cause severe inflammation that can cause swelling and pain..Surgical removal of the same has to be done followed by a course of antibiotics and painkillers..Hope this information helps..Thanks and regards.Dr.Honey Nandwani Arora."
},
{
"id": 46692,
"tgt": "What causes tingling sensation on the lips and fingers while suffering from renal failure?",
"src": "Patient: You are caring for a 65-year old patient who is in renal failure. during your assesment the patient complains of tingling in her lips and fingers whenever anyone takes her blood pressure and painfuls spasms in her wrist and hand. what do you suspect? Hypophosphatemia, hypocalcemia, hypermagnesemia or hyperkalemia? Doctor: Obviously hypocalcemia is the underlying cause and renal failure is the culprit. Vitamin D deficiency due to renal failure is the cause of hypocalcemia."
},
{
"id": 131371,
"tgt": "Is pain while sneezing due to hematoma after sternum injury?",
"src": "Patient: I received a Sternum contusion 6 weeks ago during an elbow to the chest collision playing indoor soccer and it still hurts when I laugh, cough, or sneeze. It hurts most when I sneeze. I got an X-ray the day after the injury and it was not a sternum fracture. Is the pain I feel during a sneeze possibly a hematoma that is situated somewhere behind the sternum? And if it is possibly a hematoma, how long before the hematoma goes away? Thank you! Doctor: Hi ..thank you for posting your question on HCM.Yes it is a possibility that a haematoma has had formed at the costostrenal junctions ..it takes 2-3weeks for this site contusions to resolve.I would treat it with ice packs and then hot packs .Palpable contusions can be managed by antibruising ointments application.Wishing you speedy recovery"
},
{
"id": 102576,
"tgt": "What to do for allergy to fish, shrimp, tomato, coconut, beans, banana, ginseng, aloevera, grapes, raisin, rice, spaghetti and more?",
"src": "Patient: hi I m 23 and i have food allergy to the list bellow, its been a year now and I tried homeopathy 6month from now but it makes me worst. plz help me ps. I can send you photos if necessary coldcuts & hotdogs sea food such az fish & shrimp tomato coconut beans banana ginseng aloe vera grape & raisin rice spaghetti kiwi colored foods like not natural juice & jelly bean Doctor: Hi, Welcome to Health care magic forum. It appears that you are allergic to the cold cuts, hot dogs, az fish, shrimp tomato,coconut, beans,banana, ginseng, aloevera,grapes, and raisin rice. Allergic symptoms you have will decide the doctor you have to consult. If skin problems like itching, consult a dermatologist, if it is cough and cold consult an E.N.T .surgeon I usually prescribe to my patient with such symptoms any type, levocitrizine, montelekast, and triamcinolone 40mg injection every forte night. Wishing for a quick and complete recovery. Thank you."
},
{
"id": 221787,
"tgt": "What causes lower abdominal pain during pregnancy?",
"src": "Patient: I am seven weeks pregnant and I am having acute very lower stomache pains. At first I thought it may be my bladder thus bladder infection but I also feel as though the pain is in my cervix too and there is preasure.. I have drank some oarnge juice and water but can t overdue it due to my congestive heart failure. I have misscarried once before at six weeks.. What do you think it is or what should I do??? Please can you talk with me???? Thank you. Doctor: Hi dear, I have gone through your question and understand your concerns. Pain in lower abdomen in early pregnancy can be due to urinary tract infection or can be due to threatened abortion.I will suggest you to consult an OBGYN specialist to get properly examined, investigated including ultrasound and urine examination and get appropriate treatment.Hope you found the answer helpful.Wishing you good health.Dr Deepti Verma"
},
{
"id": 210901,
"tgt": "What causes severe pain in chest and back, anxiety, frequent panic attacks?",
"src": "Patient: I have an 18 years old daughter that has sever pain in her chest and back, she has anxieity and panic attacks often and she has had the following test done can you please tell me what might be wrong with her she is going crazy tring to figure it out? EKG Stress halter CT scan upper GI Tread mill stress test Blood work for lupas, lukiemia, HIV, Hepititis She is Iron deficient Her B-12 and D is very low What can be wrong with her since the doctor and hospital said all the test came back normal? Doctor: Hello,Thanks for choosing health care magic for posting your query.I have gone through your question in detail and I can understand what you are going through.If allthe tests have come back normal then it is very likely that she may be having panic attacks. This is a disorder of anxiety and occurs cause of imbalance of certain neurotransmitters in the brain. especially serotonin, nor adrenaline, and dopamine to some extent. Medicines are the best options to take care of these symptoms. Paroxetine is the the most popular medicine and my drug of choice. I generally prescribe my patients 12.5 mg per day at night which takes care of anxiety symptoms. Also certain medicines like propranolol and clonazepam can be give as required when there is a panic episode or expected situation where panic attack can appear.Hope I am able to answer your concerns.If you have any further query, I would be glad to help you.In future if you wish to contact me directly, you can use the below mentioned link:bit.ly/dr-srikanth-reddy\u00a0\u00a0\u00a0\u00a0\u00a0\u00a0\u00a0\u00a0\u00a0\u00a0\u00a0\u00a0\u00a0\u00a0\u00a0\u00a0\u00a0\u00a0\u00a0\u00a0\u00a0\u00a0\u00a0\u00a0\u00a0\u00a0\u00a0\u00a0\u00a0\u00a0\u00a0\u00a0\u00a0\u00a0\u00a0\u00a0\u00a0\u00a0\u00a0\u00a0"
},
{
"id": 173744,
"tgt": "What causes spots on face and pain in finger tips in a child?",
"src": "Patient: Hi my child has unexplained spots that appeared in her face about 9 of them all at once , then after a day the spots turned out to look like bruises what could cause that now she is feeling pain in her finger tips please what could be the cause of this Doctor: Hi...I feel by what you quote he should be having a - Hand Foot Mouth disease. This is one viral illness among all other exanthemas which can cause fever followed by rash over palms and soles. It is a self-limiting disorder and itching can be really worrisome. I suggest you use any over the counter antihistamine if you have one with you now. You can use Hydroxyzine at 1-2mg/kg/dose (Maximum 10mg) every 6th to 8th hourly for 7 days. This can even cause some peeling of skin in the next 4-6 weeks and do not worry about it.Regards - Dr. Sumanth"
},
{
"id": 193394,
"tgt": "What causes pain and blood after sex in men?",
"src": "Patient: I'm a gay male who is usually on the receiving end of sex. I recently had some blood/pain when I wiped and had sex and thought I might have had sex that was too rough. I went to the doctor and he found no signs of tears or hemorrhoids. I wast told to use an anti-itch cream, avoid scrubbing in the shower, increase fiber intake and avoid anal sex. After 2 months of following all the advice, the pain/blood is still present (sometimes better than others).Should my next step be going back to the doctor? I really don't mind going back to the doctor, but I don't want to waste my time if they cant really offer me anything more. Doctor: Hi, Yes, you need to review the doctor and go for colonoscopy for further diagnosis. It can be injury or it can be ulcer too. Also, check for stool routine for worm infestation and blood sugar. Hope I have answered your query. Let me know if I can assist you further. Regards, Dr. S. R. Raveendran, Sexologist"
},
{
"id": 63564,
"tgt": "What causes lump on nose with swelling?",
"src": "Patient: Hi..work up 2 mornings ago with a painful lump more so than a pimple inside my instead behind the skin that seperatrs the nostrils so its hard to see what it looks like. Anyway this pain is ridiculous. My nose is very red and swollen. Whenever i bend forward they blood rushes to my nose causing a thumping beating pain. What is this from and when will it go away??? Doctor: Hi,Dear,Thanks for the query to HCM. I studied your problem in depth and I understood your concerns. Cause and plan of Treatment-In my opinion on the given data,you seem to have boil in the nostril at the tip of the nose internally.It would go if you take proper treatment with Er Surgeon or Ent-SurgeonConsult Er Ent-Surgeon who would treat accordingly.So don't build up wrong concepts and self medicate your self which would increase risks and costs to you.Instead just ask a query to HCM and be comfortable to resolve your health issues.Welcome for any more query in this regard to HCM.Write good reviews and Click thanks if you feel satisfied with my advise.Have a Good Day.Dr.Savaskar M.N."
},
{
"id": 30407,
"tgt": "Should i be concerned about the bruised spots due to insect bites?",
"src": "Patient: I have insects bites on my ankles, which were swollen for 2days. Now the bite on my legs and some on my waist are looking like bruised spots. At beginning looked like aunt/mosquito bites, also my legs are aching. Is this serious? Do I need to go to hospital? This is my 3rd day since 1st symptom. Doctor: HiThe lesions are getting serious and might become superinfected.I suggest you go to the hospital and get examined.You might need a course of antibiotics and pain/antiinflammatory medication to get over it.This will be decided by the Doctor after examining the lesions.Best regards"
},
{
"id": 151848,
"tgt": "How can my brother's condition who has T5T6 compress and mild degenerative changes of forek colember spine improve ?",
"src": "Patient: Dear sir,my brother is 47 years old he has suffering fro NERVOUS PROBLEM since from 1999. he can not walk without support,due to wekness in right leg, as per MRI test spinal cord compress in L4 L5 and bulge , as per 2008 MRI report T5 T6 compress right side and Mind degenerative changes of forek colember spine, calcium is 67%,mussule test is normal,docter has given solumeprol 1gm.but sir there is no changes in healh but problem is increase day by day.please sir advice me what will be the problem,and give me solution. Doctor: Hi Welcome to Healthcare magic forum You can visit a psychiatric conselor , if you think the problem is in his mind. (you havnt mentioned why you feel that). You can see a homeopathic physician the orthopedic problem . Takcare.Takecare"
},
{
"id": 83522,
"tgt": "What are the side effects of pepcid complete tablets?",
"src": "Patient: In oct 2011, I had a severe heartburn incident in which I took 4 Pepcid complete tablets in 12 hours. Over the next several days, I have developed a severe lower left intestinal pain that radiates in the side and back area. The painful spot is permanent and will not go away. Antibiotics have helped, but only to a point. I cannot sleep on my left side, the pain that arises is terrible. Is this necrotising entercolitis? I am 52 yrs old with a history of IBS and excessive acid, Pepcid usage has been higher in the prior 6 months. Doctor: Hi,The cause of abdominal pain should be identified. Upper gastro intestinal endoscopy and abdominal ultrasound should be done. Your symptoms mostly suggest peptic ulcer complications.Helicobacter pylori is a bacteria that can cause gastritis. A course of multi-drug regimen of antibiotics will be required to reduce the infection.Injectable proton pump inhibitors may be required to reduce complications and heal the ulcers. Also any history of fever, blood in stools, renal stones or diarrhoea should be ruled out.Hope I have answered your question. Let me know if I can assist you further. Regards, Dr. Saranya Ramadoss, General & Family Physician"
},
{
"id": 79792,
"tgt": "Does PRIALT cause diarrhea, confusion, shortness of breath and respiratory failure?",
"src": "Patient: I had neck surgeries (2) in 3 days June 2013. In Dec I had a pain pump Implanted for PRIALT. I started getting very sick in Dec w/diarrehea, confusion, shortness of breath nearly got me. I Jan I was life flighted to a hospital and the doctors told us they lost me twice w/respiratory failure. They thought it was some medication I had been taking. But I did not get better only a little better at handling it. \\Last week I come up with the idea that it was the PRIALT. Every side effect I had was listed w/them.. My dr decreased the pump 94% for the next 3 wks. Is that possible or probable? Sandy Trahan Doctor: thanks for asking your questionI completely understood the queryprialt can cause confusion and bresthlessness and these are alarming symptoms ,you should consult your treating physician who can helo you adjust the doses.thanksif you have more queries feel free to askmay god bless you with good health\\"
},
{
"id": 77639,
"tgt": "Suggest treatment for pneumonia",
"src": "Patient: Yesterday went to the er and was diagnosed with mild pneumonia. One questions I failed to ask-I have been having heart palpations since my back and shortness of breath start 1 weeks ago. Cou;d heart palpatations be from the pneumonia? I am 61 yo and in good health. Doctor: Thanks for your question on Health Care Magic. I can understand your concern. Pneumonia is acute infection of lungs. As a rule acute infection anywhere in the body cause sympathetic stimulation. And this cause rapid heart rate (tachycardia), palpitation. Another reason is infection induced hypotension. And hypotension can cause tachycardia and palpitations. You are also having breathlessness so this can also contribute to palpitation. Improvement in palpitation is a marker of improvement in pneumonia. So take your drugs timely and wait for improvement. Usual time of recovery is 1-2 weeks. Drink plenty of fluids orally and keep yourself hydrated. This will also help in palpitation. Hope I have solved your query. I will be happy to help you further. Wish you good health. Thanks."
},
{
"id": 409,
"tgt": "Can ovarian cyst cause a false positive pregnancy test?",
"src": "Patient: My wifes LMP is 06/01/2010.Yesterday we did home pregnancy test morning and afternoon and both are positive.We went to appolo BGS.There they did blood test and routine urine test and transvaginal US.But they didnt test for HCG.US result is no gestational sac seen and cyst in rt Ovary.Dr ask us to take another Ultrasound next week.We are not satisfied with Dr's dcecsion.Just they want to make money out of us.Does home pregnancy test may get false?can we see G sac at this early time in US.Does the cyst is the reason for Positive Home Pregnancy test.Please advice. Doctor: Hi welcome to the health care magic I will solve your query and will give my opinion in this answer..... Let me explain you false positive urine card test causes...... _abortion history in last 6 week _Test not done as per kit insert _reading not taken in prescribed time in kit insert _expired kit _urine contamination with blood _trophoblastic tumour etc... I advise you to repeat urine card(at home) or blood hcg estimation done (at hospital) I need to know USG done by Transvaginal route or transabdominal route?? Tvs sonography able to visualize gestational sac 5 week after LMP if conception occured And for Transabdominal time period is 6 week after LMP Urine card positive 5 week after LMP And remember USG confirmation needed to rule out pregnancy Hope I have solved your query Take care"
},
{
"id": 32385,
"tgt": "Suggest medication for permanent cure of fungal infection in inner thigh",
"src": "Patient: Hi... I am 48 yrs old female, I got permanent fungal infection on my inner thighs near reproductive area on skin and during summer it increases a lot , I tried many antifungal creams but no positive result . Pls. suggest some good medicated cream for this. Doctor: HiI can understand your concern for the suspected fungal infection of inner thighs near the private parts.It seems you have a fungal infection of the groin (inner thighs and private parts) referred to as tinea cruris.Antifungal creams containing Terbinafine or Clotrimazole or Miconazole are very effective in the treatment of tinea cruris.Apply the cream twice daily to the surrounding 4-6 cm of normal skin in addition to the rash. The following measures may prevent fungal groin infection (tinea cruris) from recurring:1. Wash your groin daily; then dry thoroughly. The damp or moist groin is an ideal site for fungal germs to multiply. 2. Change your underwear daily. Fungi may multiply in flakes of skin in unwashed underwear.3. Check for the fungal infection of the toes. The fungi from athlete's foot may spread to the groin. The same creams are used to treat fungal infection of the toes and the groin.4. Do not share towels with people in communal changing rooms. Wash towels frequently &5. Keep your own towel when you have a fungal skin infection to reduce the chance of passing on the fungus to others.Hope the suggestions given above could be useful in total eradication of the fungus from the goin (inner thighs and private parts)."
},
{
"id": 125579,
"tgt": "What causes ring like formation on the ankle?",
"src": "Patient: Hello, My name is Sam. My friends baby has bright red rings around her ankle. We thought they were from her socks but we have removed them and there still there so I felt them. They are bumped out and really hurting her I was wondering what they are. (Her socks have been off for 6hrs) Doctor: Hello, It could be due to minor contusion. Apply ice packs for symptomatic relief. If symptoms persist better to consult an orthopaedician and get evaluated. Hope I have answered your query. Let me know if I can assist you further. Take care Regards, Dr Shinas Hussain, General & Family Physician"
},
{
"id": 190050,
"tgt": "Infected tooth, pus taste in mouth, swelling and pain around cheek and top of neck. What should be done?",
"src": "Patient: I have a bad tooth and the taste of pus in my mouth. There is some swelling and pain around my cheek and the top of my neck . I think I also have a wisdom tooth coming through, but I cannot be too sure. I am taking painkillers, applying ice packs and swilling my mouth out with salty water. I went to see a dentist but with Christmas coming up, I will have to wait a few days before I can be seen unless you think I should see someone sooner. What should I do? Thank you for your time. Doctor: hello & welcome most probably u are suffering from pericornitis. This is a type of infection caused by partially erupted wisdom tooth. in such cases only painkillers are not sufficient you have to do an antibiotic course including Amoxycilline 500mg TDS, Metronidazole 400 mg TDS , Ibuprofen 400 mg TDS, & warm saline rinses. Than visit to your dentist as soon as possible and go for a check X-ray. take care."
},
{
"id": 108230,
"tgt": "Suggest treatment for lower back pain",
"src": "Patient: I have had a history of slip disk 3 years back. Lately i have been suffering from lower back pain that to on the left side which shoots up till my legs And travels to abdominal area And something like a click or something happens in the vaginal area and then the pain stop immediately What is it? Doctor: HiThank you for asking HCMI have gone through your query. Your problem of back pain radiating to legs can be most likely (Sciatica) nerve compression syndrome from lumbar disc herniation. A physical examination by an orthopedician will give clue for that. MRI will needed to confirm in case of positive signs. In such case muscle relaxants like cyclobenzaprine , NSAIDs like aceclofenac and neurotropic drugs will be helpful. Physiotherapy and muscle strengthening exercises along with this will make you better. Wearing lumbar supporting belt also will be helpful.Hope this may help you. Let me know if anything not clear.Thanks."
},
{
"id": 27003,
"tgt": "Blood test shows an elevated IGE of 2617 & slight tingling in hands & feet",
"src": "Patient: I have had constant, dull lower left quadrant pain for almost 4 months now. I recently had a blood test that showed an elevated IgE of 2617 with the normal range being <114. I also have had tingling in my hands and feet for over a month now. Any ideas? Doctor: Hello and thanks for writing.I can understand your concern and would try to help you in the best possible way.The elevated IgE levels mean that you are allergic to some thing however it cannot tell you what you are allergic to. For that a skin test or allergen specific IgE test has to be done. Your abdominal pain and tingling in hands or feet may be related or even be two separate entities. A complete history and physical examination would be required for getting to a diagnosis. Some relevant examination like CBC, stool examination, USG or even scopy may be required to get to a diagnosis.I suggest you consult a physician who would help you with a diagnostic workup and appropriate management plan."
},
{
"id": 77836,
"tgt": "What causes chest pain, shortness of breath and difficulty in breathing?",
"src": "Patient: chest pain I'm a 26 year old female 5'7\"and for the past two weeks I have experiencing cf chest pains, at first I thought it was heartburn because it came with constipation but now it's chest pains that go straight to my back and it makes me short of breath and sometimes hard to breath and if I get stressed or upset it gets worse Doctor: Whether the pain is relieved by leaning forward and rolled over lying in bed sir?? If it is more at night and radiated to back and severe in intensity then it may be pancreatitis sir.so kindly do an ultrasonogram and serum amylase level..it will give the diagnosis,"
},
{
"id": 48199,
"tgt": "What causes swollen toes and foot with drop in kidney level?",
"src": "Patient: went to my doctor today with swollen toes and top of my foot,couldnt find anything significant but my kidney levels have dropped really low,tired all the time and difficulty walking all showing signs of kidney disease, I am not the doctor what do u suggest I should do Doctor: HelloThanks for query.You have swelling of the feet and toes with tiredness and generalized weakness This may be due to Kidney failure or Cardiac problem .Consult qualified Physician for clinical examination and get following basic tests done to arrive at probable cause of your symptoms .1) Haemogram 2) Routine Urine test 3) Renal function Test 4) ECG (Electro -Cardiogram)5)Liver Function TestsFurther treatment will be decided by him based on results of these tests and final diagnosis.Dr.Patil."
},
{
"id": 108074,
"tgt": "What causes flank pain from my ribs to my glute?",
"src": "Patient: I have a dull warm throbbing ache in my right back flank radiates from my last rib to my glute. This has been constant for 7 hours. I have just finished taking progesterone for 11 days after trying iui for fertility. The hpt I have taken have all said negative. Do you think it could be causing this Doctor: Hi i did review your concern.The pain you have been describing can be due to either due to your kidneys or due to ovary.I would recommend you to get an ultrasound done and find the root cause of your painI hope this helpswish you all the best."
},
{
"id": 217448,
"tgt": "What causes severe pains in the top of stomach?",
"src": "Patient: hi, My boyfriend brock has been getting bad pains in the top of his belly undernieth the sternam, he has had it off and on through the days for over a week now. At first we thought it could of been gastro but that is not the case. just wondering weather it is something to worry about and what it could be? Emma. Doctor: Dear Emma,Thank you for your questin on HCM.The characteristic feature of gastritis is pain in the epigastric region which is like you described ,below the sternum.But since you have not mentioned how you confirmed that it is not due to gastritis,I would suggest that you advise your boyfriend to take antacids for a few days and see if he gets relief from that.He acn take tab,Pantoprazole 40 mg for a week half an hour before breakfast,avoid spicy food,drink lots of water and have an early dinner .This will definitely bring him some relief,but if you have already tried this,do get back to me and let me know if he has any other symptoms like nausea and vomiting,and if the pain increases on intake of food or reduces with intake of food and if his bowel movements are regular.hope this information was helpgul.regards,DrRadhika"
},
{
"id": 35108,
"tgt": "Suggest treatment for infected leg caused by an insect bite",
"src": "Patient: I have had an infected leg caused by a bite(possibly a spider bite according to Doc) and am on antibiotics. I had cellulitis 5 months ago in the same leg. I am now getting a stabbing pain in my calf when I bend over in the same leg. Is this anything to worry about? Doctor: HelloWelcome to HCM,I can understand your concern for bruising after suffering insect bites. There can be multiple possibilities of the blisters in your condition. It can be a part of simple folliculitis by bacterial proliferation or may be part of some autoimmune problem which is causing blistering.A detailed history with proper work up is required to reach to the exact diagnosis. You should meet dermatologist at the earliest so that exact diagnosis can be made.Do not try to pinch or pop the blister as can get secondarily infected and further complications.Thank you."
},
{
"id": 215782,
"tgt": "What causes constant body pain?",
"src": "Patient: Hi, my wife is 36 year old and suffering from body pain last two weeks. she has below 100 temperature and her widal test is positive. Her temperature can t cross above temperature any time. After taken antibiotic she feel good but her body pain and temperature come back after one or two month. Kindly suggest me what can i do and prescribe some antibiotic whose good for her. Doctor: Hi, She can start on antibiotics preferably Ciprofloxacin or Cefixime after consulting a physician. She can take analgesics like Acetaminophen for symptomatic relief. Drink plenty of water to keep hydrated. Hope I have answered your query. Let me know if I can assist you further."
},
{
"id": 99410,
"tgt": "What causes cold inside lungs which hurts and back pain?",
"src": "Patient: Hi. I ve been in and out of my doctors office 9 times in the last 3 months. The majority of it was every 2 weeks over a 6 week period. Every time he said it was allergies. Gave me an antibiotic and steriod shot along with Rx. This week, I have developed back pain which is keeping me from sleeping. (I also have a 23 month old who doesn t sleep through the night). Every time I go in, he takes my blood and says my WBC is high. (The passed few times he didn t know what was wrong with me. Just that my counts were high and it s a sign of infection). Yesterday I went in with feeling exhausted, back pain, and naucious. He just said I had an infection somewhere and gave me more antibiotics. Then this morning, I woke up with a coldness in my chest. My lungs hurt. And the back pain is significantly worse. Could this be pneumonia? Should I go to an internist instead of a gp? Doctor: Hi,Yes,it could be because of pneumonia,bronchitis(allergic or infective),pertussis, seasonal asthma etc.You may undergo routine blood count,ESR with X- ray chest and pulmonary function test.You might require antibiotics (quinolones or macrolides or amoxillin with clavulanic acid) with cough suppressants (antiallergic antihistamines or mast cell stabilizers) and antiinflammatories or analgesics on as and when required basis.You must consult your pulmonologist instead of general practiotionar to get diagnosed first and then treatment.Thanks."
},
{
"id": 59439,
"tgt": "Cyst on liver, ovaries. Treatment?",
"src": "Patient: Went into the ER for symptoms of apendicidus, they did CT scan ruled out apendendidus, found a cyst on liver 5.8 cm and cyst on overaies. I followed u[ with primary care dr and ob dr. Primary care Dr sent me Gastrologist to check live, he also informed me the cyst had bben there and has grown, no one bothered to tell me.. What do you think the Dr will do if anything? Doctor: Hello, rimanu8, The cysts can occur without any symptoms unless they are very big and put pressure on the surrounding structures or bleed or even get infected. Most of them are found during routine sonogram or CT-scan done for some other reason. Liver cysts can be associated with cysts in the spleen and also kidneys. Like in your case, ovarian cyst. Sometimes , a big cyst in the liver called the Hydatid cyst can be associated with parasites. Your cyst in the liver is 5.8 cm in size which is not small and requires close followup. Best thing is to have a sonogram every 6 months and see if the cyst is growing in size. Bleeding, rupture and infection are the complications. If there is any suspicion , you need surgery for excision of the cyst by a competent surgical specialist. I wish you well."
},
{
"id": 23919,
"tgt": "Are hard to breathe, dizziness and sleeplessness the symptoms of heart problems?",
"src": "Patient: I am 16 years old, 5'4, and have epilepsy and anxiety. I am normally 135-140 lbs, but have gained about 25 lbs because of the pills I have taking for both diseases. Recently, I have not been sleeping, and I have been very dizzy, and when I walk up the stairs too fast, I find it VERY hard to breathe. Could this be the start of heart problems? Doctor: Hi,None of your complaints are heart related.Breathlessness may be caused by your weight gain, you should try to diet strictly, as with your medications you gain weight.Sleeplessness may be caused by your medications.Dizziness may be caused either by anxiety or by your medications.For last two complaints consult your neurologist to revise the treatment if they disturb you much.Take careCome back if you have any further questions"
},
{
"id": 87855,
"tgt": "Suggest medication for abdominal pain",
"src": "Patient: Hi I'm having uncomfortable stomach pains and I am under the impression I am having stomach bleeding due to the use of various medications lately I have been using for my cold. How much do you actually have to consume in order to get stomach bleeding? ): Doctor: Hi.Thanks for your query and an elucidate history. You can say that you are bleeding indie only if the vomiting is present with blood or coffee colored or there is actual blood or black colored stool. Of course the best way id to have an upper GI Endoscopy and Colonoscopy, tests of the stool for occult blood to prove. There is no such dose that can cause bleeding, it can be a single dose of an tablet like Ibuprofen that can cause this and that is the reason you should always take PPI or Ranitidine with the medicines, and always after food or with milk. So do not worry if you do not any of the symptoms I have marked for you."
},
{
"id": 193804,
"tgt": "What causes lump under the armpit during puberty stage?",
"src": "Patient: My 12 year old son is in puberty. He has quite a bit of pubic hair, under arm hair, acne and one nipple has enlarged and a lump behind it. His penis had enlarged quite a bit and testicles. However, over the past 6 weeks or so he has noticed about 1/2\" in diameter hard lump under his left under arm. In the beginning it hurt, but said it doesn't now. The size has not gone down. I also feel another lump in the other underarm as well. Not as big though. Is this something we should be concerned about? Doctor: Hello, It could be a lymph node. It indicates an infection in the nearby region. If lump persists you can consult a general surgeon and plan for an ultrasound scan. Hope I have answered your query. Let me know if I can assist you further. Take care Regards, Dr Shinas Hussain, General & Family Physician"
},
{
"id": 127788,
"tgt": "What causes tingling sensation in the left shoulder radiating towards the neck?",
"src": "Patient: I have tingling in my left shoulder that has gotten worse, now it crawls up my neck and to my ear. I find myself scratching and rubbing my shoulder, neck and face. I feel that It is the verdibraes in my neck has a pinched nerve. If I hold my head to the side a certain way the tingling stops. the tingling is getting worse and is not being some pain. In order to sleep, I have to lay on my left side with my head proped up in a certain fashion to get any sleep. Can you tell me what kind of doctor or specialist I need to see. My family doctor prescribed Gabapentin but that had side effects so I quit taking it. It did not help me anyway. Doctor: Hello,What I can infer from your complaints is that you are suffering from cervical disc prolapse. This is a condition in which the cervical nerve roots get pinched by disc material.I would suggest you to consult an orthopedic surgeon and get an MRI Scan which will show the condition of cervical disc and roots. Meanwhile, you can start undergo physiotherapy to reduce the pain.Hope I have answered your query. Let me know if I can assist you further.Regards,Dr. Santosh S Jeevannavar"
},
{
"id": 171269,
"tgt": "What causes palpitation in kids?",
"src": "Patient: I have 13 yr old daughter just had an ekg and chest x ray for palpitations, has only had a few palpitations that she complained about. Xray was fine but ekg said left ventrical hypertrophy, her pediatrician gave me a name of cardiologist is this serious, made appt but worried Doctor: Hi...Palpitations without any structural abnormality need not be worried about. In this case the ECG is normal but the Echo cardiogram has shown some abnormality in the form of increased muscle in the heart. Once such findings are there the case calls for further evaluation. I also concur with your pediatrician and suggest that you see a paediatric cardiologist regarding this.Regards - Dr. Sumanth"
},
{
"id": 115335,
"tgt": "Can radiation therapy and diarrhea cause high blood pressure?",
"src": "Patient: Hi, may I answer your health queries right now ? Please type your query here...Can radiation therapy and/diarrhea cause high blood Pressure? My blood pressure has been extremely high for the last two days including landing in an emergency room at a hospital nearby. I'am suppose to be given 38 treatment of which i ace completed 15 today. Doctor: Hi, dearI have gone through your question. I can understand your concern. Radiation therapy may leads to high blood pressure and diarrhoea. It cause injury to viable cells and leads to all this.Consult your doctor and take treatment accordingly. Hope I have answered your question, if you have doubt then I will be happy to answer. Thanks for using health care magic. Wish you a very good health."
},
{
"id": 138310,
"tgt": "Suggest treatment for shoulder pain after spinal fusion surgery",
"src": "Patient: I had a spinal fusion in jan. This year of the 5th and 6th vertebra and was going well but last weekend I was having some neck to shoulder pain familiar of that before surgery. So I went to swing a sledgehammer at work and my right arm went week at my bicep. That Monday I went to my orthopedic surgeon and he X -rayed to confirm everything was in place. He put me on a dose pack do steroids for inflammation but weakness is still there after 5 days. It is getting more difficult to raise my arm but still have full range of motion. I still have more meds to take . Should I wait or get the MRI he wanted to do? Doctor: Hello,I have studied your case.I do not think that this weakness is due to nerve compression. But most probably it is due to shoulder muscle weakness or tear. So I would recommend you to get MRi of the shoulder joint to see if there is any tear in your rotator cuff which can present such picture. If there is tear seen on MRI than you might need a arthroscopy surgery for the shoulder in which tendon will be repaired.I hope this answer will be useful for you. Let me know if there is any other follwup questions.thanks"
},
{
"id": 213470,
"tgt": "Difficulty breathing, increased pulse, anxiety, sweaty palms, sudden waking from sleep due to fear. BP, Oxygen capacity report normal. Causes?",
"src": "Patient: Hello, i am suffering from some health problems since last 10 months. the exact diagnosis is not done by any doctors. i am now too tired of this disease . please do the rigth diagnosis of my health problems and tell me what i have to do? Symptoms: difficulty in breathing , heartbeats increases, anxity, palms get cold &sweat, body not feels free in movement, something like needle stiches to neck and pain of it, fear during sleep & sudden wake up from sleep due to fear or pain or any thing else , the exact reason of waking up from sleep is i dont know. During all this symptoms i had admitted in hospital in emergencily, at that time all reports are normal , like BP, Oxygen capacity, ECG,. my MRI chest,CT-Scan brain, X-ray all report are Normal. Doctor: hello snehalchaudhari1. Thanks for visiting healthcaremagic.com. I am glad that you have had all of the test above because that helped to rule out a lot of diseases. I believe that you are suffering from panic/anxiety attacks. Panic attacks are defined as a period of intense fear or discomfort in which 4 or more of these symptoms develop abruptly and reach a peak within 10 minutes (usually). 1. Palpitations, pounding heart, or accelerated heart rate * 2. Sweating * 3. Trembling or shaking 4. Sensation of shortness of breath or smothering * 5. Feeling of choking 6. Chest pain or discomfort 7. Nausea or abdominal issues 8. Feeling dizzy, unsteady, lightheaded or faint 9. Feeling detached from one's self 10. Fear of losing control or going crazy * 11. Fear of dying * 12. Paresthesias (numbness or tingling sensations) * 13. Chills or Hot flashes I put stars by the symptoms that you mentioned above and put stars by the fear related points because you do not clarify what you are having fear of. You have at least 4 out of these symptoms above. This is a medical condition and is well recognized in the mental/behavioral health community. This condition can be treated with antidepressant medications and/or anti-anxiety medications. I suggest that you return to your primary physician with this knowledge and perhaps get a referral to a mental health specialist if your doctor feels it is necessary. Be well, Dr. Kimberly"
},
{
"id": 43800,
"tgt": "Prescribed Puregon for assisted fertility conception. How is Brevactid different from Utrogest?",
"src": "Patient: hello doctor, i am on 5th cycle of assisted feritility conception . I am asked to take Puregon 50 microlitre from cd2 to cd11 (today). Tommorrow i am supposed to take the brevactid 5000. And after that i am usually prescribed utrogest.but this cycle,my gyn asked me to take another shot of Brevactid 5000 on CD17. I would like to know what this ll do to my cycle and what difference this has to that of using utrogest? Doctor: Hello thanks for your query. The Brevactid 5000 is HCG which is generally given around ovulation to trigger ovulation A second shot might be required if the follicle did not rupture with the first one, and more dose is needed. It is also given in the luteal phase ( after ovulation ) to provide support to the growing embryo in case conception does occur Utrogest is progesterone which is also given as luteal phase support for the embryo. So, please follow the instructions of your Infertility specialist all the best for your cycle"
},
{
"id": 199148,
"tgt": "What causes memory loss, weak bones, sleep disorder when masturbate regularly?",
"src": "Patient: Sir, my age is 30 years and I masturbated since I was 13 years old....sir I did 10 times per day.....i actually felt while doing very tired but i did......sir now my bones are very weak.....my memory is weak.....i could not get sleep at night.....and my family asking me for marriage but sir I am unable because I feel unable to sex......please sir give me correct advised... Doctor: HelloI really appreciate your concernExcessive indulgence into masturbation is harmful to the overall health it causes weakness, fatigue and sexual problems in long runI would like further information to help you betterrelevant medical history like metabolic or hormonal disease do you have problem of nightfall or discharge through penis?Do you smoke or consume liquor?I would advise you for complete abstinence of masturbation for a while with adoption of healthy life style for restoration of optimum health.Please don't hesitate to write back to me if you have any more doubts I will be happy to help you wish you good health"
},
{
"id": 63178,
"tgt": "Suggest remedy for sore black lump on arm",
"src": "Patient: hi i have had a light black lump on my arm that suddenly appeared about 2 years ago, just lately is has started to feel a bit sore so i scratched it and alot of blood pus came out of it, it has started to hurt a bit more and theres a white bit in the middle which when i tried to pick out was attached to a long white stick. it is also weeping now. just wondered if its something i should be worried about Doctor: Hello!Thank you for the query.From your description it seems to be a sebaceus cyst. It is characteristic for this lump to do not give symptoms right after it appears. It usually gives symptoms when it gets infected. If so, pus appears (can be with blood) as well as swelling and pain. Once it is drained, the pain goes away, however the lump usually does not disappear completely.To get rid of this lump, you should consult general surgeon and have it removed. Otherwise the symptoms can appear again.Hope this will help.Regards."
},
{
"id": 38047,
"tgt": "What causes bleeding in the tongue?",
"src": "Patient: Over the last week or so I noticed that around the tip of my tongue was hurting during and after brushing my teeth. It has now gotten to the point that it is bleeding while I m brushing even without brushing my tongue. I tried switching toothpaste after I noticed even though I had been using the same brand for months. Can you tell what may be a cause for this? It is affecting my eating and drinking. Doctor: HiYou are having an ulcer over your tongue tip. Small ulcers 2-8mm in diameter heal by them-self with in 7-10 days time. if there are multiple painful ulcers then it may be herpes. If the size is more or the ulcer is also not painful then kindly you must visit your doctor as there may be serious problem like malignancy. Other wise keep your oral hygiene good, take some multivitamins, avoid smoking, alcohol, irritant type of substances. You will recover soonTake Care"
},
{
"id": 160479,
"tgt": "What causes high WBC in urine of a child?",
"src": "Patient: My seven year old daughter had a urinalysis done due to a possible UTI. There was no growth, but a high white blood cell count. Her father has psoriasis, and thinks that could be what is causing this. She has no symptoms of that, so what could the high white blood cell count be from? Doctor: Hello, Sometimes white blood cells in a child's urine are normal. It depends on the count. The fact that there was no growth in the urine sample means there was no bacterial infection. White blood cells are also released when there is inflammation - serious or not. It would unusual that this would indicate Psoriasis. I would retest her urine a few times and see if it resolves or is of a minimal level. Mostly, without other symptoms, I don't want you to worry. Hope I have answered your question. Let me know if I can assist you further. Regards, Dr. Lisa Baker, Pediatrician"
},
{
"id": 187848,
"tgt": "What can be done for low tolerance to antibiotics as prescribed clindamycin for serious gum infection?",
"src": "Patient: my dentist said I have a serious infection in my gums. he prescribed CLINDAMYCIN. I AM CONCERNED BECAUSE I HAVE A VERY LOW TOLERANCE TO ANTIBIOTICS. I ALSO HAVE SERIOUS IBS AND GERD. READING MORE ON THIS MEDICATION ADDS TO MY CONCERNS BECAUSE OF THE BOWEL PROBLEMS IT CAN CAUSE. SHOULD I SEEK ANOTHER MEDICATION Doctor: Hello, Thank you for consulting with healthcaremagicAs your doctor has prescribed clindamycin for gum infection, it is a good antibiotic for infection.But you should have told your dentist about your GERD problem, as it can cause you other problems. It will be better that you should consult him again and get your medication changed.Hope it will help you.Regards"
},
{
"id": 154898,
"tgt": "What does irregularhypoechonic leison mean in breast ultrasound report?",
"src": "Patient: my breast ultrasound report states that I have an irregularhypoechonic leison, and 'On sonoghraphic grounds, the appearances suggest a significant lesion and would anticipate the core biopsy results to reflect this' What does the term, 'significant' imply in this context? thanks LKS Doctor: Hi, dearI have gone through your question. I can understand your concern. Your ultrasound report suggest significant lesion. It means lesion can not be ignored. It need further investigation to rule out cancer. So you should go for core biopsy or fine needle aspiration cytology to confirm the diagnosis. Then you should take treatment accordingly. Hope I have answered your question, if you have doubt then I will be happy to answer. Thanks for using health care magic. Wish you a very good health."
},
{
"id": 27139,
"tgt": "What causes high heart rate and muscle pain?",
"src": "Patient: I am age 53 male and exercise daily. My normal heart rate is in the mid 50's. in the last few days, my resting heart beat has jumped to near 70-75 per minute. I have also been experiencing some muscle pains in arms , shoulders , and legs. No chest pains. Should I be concerned? Gary Doctor: Hi,It seems like some general disease, maybe you have caught cold or something similar.I would advise you to take temperature and have blood tests done, to check if there is any change, like high WBCs.Wishing you good healthRegards,"
},
{
"id": 54789,
"tgt": "How to reduce weight inspite of having fatty liver and distended bladder?",
"src": "Patient: Hello doctor, I have a fatty liver and distended bladder - my doc asked me to reduce weight immediately. I have the urge to reduce but how do I? Iam urinating every 10 min's and how would i go for a walk? Will I have hair loss problems at this time>? please tell me I have excess of sweat forming etc. Doctor: Hi thanks for asking question.Noted you have fatty liver with distended bladder....Stress is avoided as it is related to over urination...Still for distended bladder consultation to urologist is necessary...It might be sphincter or neurological problem or prostate problem....To loose weight you have to burn calories .Calorie intake restricted.Fruits more.Junk food , fried food taken less...Regular 1 hour active exercise done.Use only good oil in diet like sunflower oil.Alcohol avoidedRefined food avoided....Hope your concern solved....Take care.....Dr.Parth Goswami"
},
{
"id": 178840,
"tgt": "Can crushed allergex tablet be given to a 7 month baby?",
"src": "Patient: hi, a doctor advised that i may give baby of 7 months, half a crushed allergex tab twice a day. she vomits out the syrup and usually the contents of her tummy if she takes the syrup. she has a weak runny rose and a hint of a sore throat. is this acceptable. thanks Doctor: The main thing with Medicines are the dose in which they are given. Crushed tablet can be given to a baby if the dose is properly taken care off. See if 5ml of the syrup and half the tablet are of same dose or not. If both are same no problem in giving."
},
{
"id": 117560,
"tgt": "What is the treatment of high blood pressure and high sugar ?",
"src": "Patient: hai doctor! good after noon Ima a little kidney pataient about 6months . doctor has advaised me to take wysolone 120 mg for 3months. after falloped up reduceing10mg every 14 days till to20mg. due to side effect i have sometimes high blood presser sometimes sugar high. please explain me. sorry for my wick english. Doctor: HI THANKS FOR POSTING YOUR QUERY ON HCM.THIS HIGH BLOOD PRESSURE AND HIGH SUGAR IN YOUR CASE IS SECONDARY TO INTAKE OF STEROIDS.THESE DRUGS ARE KNOWN TO CAUSE THESE FLUCTUATIONS.I SUGGEST YOU GET A SCREENING ECHO AND HBIAC.FURTHER LOWERING THE STEROIDAL DOSE CAN CONTROL THIS BP AND SUGAR VALUES.CONSULT YOUR DOCTOR FOR FURTHER QUERIES.THANK YOU,TAKE CARE"
},
{
"id": 109233,
"tgt": "Can drug interaction cause severe lower back pain?",
"src": "Patient: I am 42 have severe lower back pain the doctor prescribed naproxen and muscle relaxers my son has asthma so I am afraid of flexerall in case he needs me. I went to bed with back spasm and woke up with my lower body shaking and my teeth chattering. I also take celexa for anxiety 40 mg could this be a drug interaction? A panic attack or part of my back problem. The back pain started in earnest Saturday Doctor: HI welcome to HCM.Yes this could be possible drug interaction in your case.Anxiolytics are known to cause frequent drug interactions with analgesics and antiasthmatics.I advise you to use topical analgesic alcohol spray thrice a day along with topical diclofenac ointment thrice daily in between to relieve your back spasms.An MRI scan can be used in case if you do not respond to these standard analgesic medications.I advice you to consult a Neurosurgeon in case if find your pain harder to bear.Post your further queries if any.Thank you."
},
{
"id": 52800,
"tgt": "What does bilirubin levels in the report suggest?",
"src": "Patient: Hi Doctor The following is my current LFT test I had jaundice 5 months back. Is the results having something abnormal I should know about Aspartate Aminotransferase(SGOT) 23.4 IU/L 0 - 35 Method : IFAlanine Aminotransferase(SGPT) 57.0 IU/L 0 - 45 Method : IFCC Gamma-Glutamyl Transferase 45.5 IU/L 0 - 55 Method : IFCC Total Bilirubin 0.62 mg/dL 0.00 - 2.00 Method : Jendrassik - Grof Cord <2.0 0-1 day (premature) 1.0 - 8.0 0-1 day (full term) 2.0 - 6.0 1-2 days (premature) 6.0 - 12.0 1-2 days (full term) 6.0 - 10.0 3-5 days (premature) 10.0 - 14.0 3-5 days (full term) 4.0 - 8.0 Direct Bilirubin 0.18 mg/dL 0.00 - 0.30 Method : Modified Jendrassik-Grof Total Protein 7.86 g/dL 6.00 - 8.00 Method : Biurate Reaction Albumin 5.19 g/dL 3.40 - 5.00 Method : BROMOCRESOL PURPLE Globulin, Serum 2.7 g/dL 2.30 - 3.50 Method : Calculated A/G Ratio 1.9 1.00 - 2.00 Method : Calculated Final Report Doctor: Hi there, All your liver enzymes look perfectly normal to me. I'm not sure why you've been jaundiced. Your total bilirubin is 0.62, which is a normal value. All of the other numbers are normal and don't look to be cause for concern. I hope I have answered your questions. Please feel free to contact me should you have any further questions."
},
{
"id": 113828,
"tgt": "Lower back very sore and aching. How to stop pain?",
"src": "Patient: hi please help me my lower back has been very sore and aching for about maybe about a week now and im not quite sure why and how to stop the pain please can you help??? Doctor: Hello. Thanks for writing to us. The low back pain is usually due to myofascial related problems and improves with mild pain killers like acetaminophen and hot fomentation. The other causes that need to be ruled out after a proper examination are nerve compression, spinal cord infections, disc prolapse, etc. I hope this information has been both informative and helpful for you. Regards, Dr. Praveen Tayal drtayal72@gmail.com"
},
{
"id": 84641,
"tgt": "Can use of Hydrea cause dark grey spots on the tongue and dark skin discoloration?",
"src": "Patient: Hi, my name is XXXX. I have primary myelofibrosis and currently treated with 2 gr of Hydrea everyday due to high platelets. I had splenectomy 4 months ago and have been on penicillin everyday since the surgery. Now I venoticed that I have dark grey spots in my tongue, do you know what has caused these spots? I know Hydrea has caused my hand skin and nail become darker but I haven t read/heard that hydrea causes dark tongue too. thanka so much. Really appreciate your answer. XXXX Doctor: Hi,Hydrea can cause blackening of the fingernails and toenails however it is not known to cause dark grey spots on the tongue. It may be related to poor oral hygiene, low saliva production, liquid diet, and use of antibiotics. It can be prevented by brushing your tongue using a soft toothbrush, Brush after eating and drinking coffee, tea, and alcohol.Hope I have answered your query. Let me know if I can assist you further. Regards, Dr. Mohammed Taher Ali, General & Family Physician"
},
{
"id": 222842,
"tgt": "Suggest treatment for urinary tract infection during pregnancy",
"src": "Patient: hi , I am having frequent pus cells in urine as after i got married i am suffering form the problem .i have taken so much antibiotics but these regenerate again and again.now i m pregnant and i am again suffering from the problem and having too much irritation after passing urine but only in morning time Doctor: First you take plenty of water and get urine culture done. It's better to start antibiotics after the culture report."
},
{
"id": 223060,
"tgt": "Is it safe to fly in 7 months pregnancy?",
"src": "Patient: I am 7 weeks pregnant and we booked a trip to Aruba before we got the great news. Is it advisable to fly? We don t want to lose out on this trip but we are worried because this is our miracle baby and don t want to jeopardize anything. Your thoughts please Doctor: Hello dear,I understand your concern.In my opinion air journey is safer than road and railways.You can go on with the journey if the pregnancy is normal without any complications like anaemia ,any spotting etc.Consider getting an ultrasound to know the well being of pregnancy before the journey.If everything is normal you can fly.Dont compromise the diet during the journey.Take adequate fluids upto 2-3 litres per day to prevent urinary infections.Best regards...."
},
{
"id": 216067,
"tgt": "Is it dangerous to use marijuana and thc pills to manage pain after surgery?",
"src": "Patient: My friend broke his arm and had surgery a week ago..he says the pain doesn t go away with the pills prescribed to him and he s thinking about smoking medical marijuana or taking THC pills or drops for the pain.Is any if these ideas harmful to him and his arm? Doctor: Hello and Welcome to \u2018Ask A Doctor\u2019 service. I have reviewed your query and here is my advice. If pain is persistent then I would better put him on opiod medications like Fentanyl, Tramadol, Morphine which has better pain reduction and not having more side-effects than Marijuana. You can get prescription for Fentanyl patches in your locality and use it or can take patient controlled analgesia Fentanyl IV dosages. Hope I have answered your query. Let me know if I can assist you further."
},
{
"id": 122419,
"tgt": "What is the treatment for muscle pain in leg?",
"src": "Patient: I have symptoms of muscle pain in my legs once I started taking Centrum and Anti Stress tablets from Natures Way. Its been 2 months since I started the tablets and have seen positive results. However the pain in legs is quite bad at times that i can hardly walk. Please advise. Is it anything to do with the tablets? Doctor: Hello, The symptoms seem to be related to myalgia. I suggest using anti-inflammatory medications such as Voltaren. I also suggest using anti-inflammatory cream such as Voltaren gel for local application. Hope I have answered your query. Let me know if I can assist you further. Regards, Dr.Dorina Gurabardhi, General &Family Physician"
},
{
"id": 180328,
"tgt": "What causes bed wetting in a child during rainy season?",
"src": "Patient: hello sir i have a 4.5 year son, he wes his bed during monsoon and winter nearly every night but during other season he stops, But for last one week with onset of monsoon he is wetting the bed daily at night. During day time when he sleeps for 2 hour even though he has taken fluid before sleep he never wets the bed, it happens only in night. no abdomal pain and stool is normal. Is it normal for a 4.5year kid to wet the bed at night in rainy season? we give him milk at night does it enhances urine formation? Doctor: Hello, Bed wetting at night is not a cause for concern till the age of 5 - 6 years. You can limit the fluid intake before bedtime and do not give milk or other liquids for 2 hours prior to sleeping. There is no need to worry. Hope that helps.Regards,Preeti"
},
{
"id": 63611,
"tgt": "Suggest treatment for having painful lump on back of head",
"src": "Patient: Hi, I Have small lump on back of my head it is painful wen a put presure on the lump , i have had it for 3 years now , the lump is like size ov a pee but it has got bigger over the years it did start off really small but naw its bigger . do u know what i should do about it . Doctor: Hi,Dear,Thanks for the query to HCM. I studied your problem in depth and I understood your concerns. Cause and plan of Treatment-In my opinion on the given data,you seem to have Sebaceous Cyst,which is painful from the combing trauma.Treatment is by-Cold compresses/Voltaren Gel locally / antibiotics 5 days time.IF it is reduced good, if not go to ER Surgeon.So I would advise you to consult a ER Surgeon.Hope this would resolve your issues.Act fast.So keep cool and Consult your ER Surgeon, if need be , who would treat it accordingly.So don't build up wrong concepts and create more psychic complications in you which would increase risks and costs to you, but just ask a query to HCM and be comfortable to resolve your health issues.Welcome for any more query in this regard to HCM.Write good reviews and Click thanks if you feel satisfied with my advise.Have a Good Day.Dr.Savaskar M.N."
},
{
"id": 44846,
"tgt": "What should be the normal endometrium thickness be ?",
"src": "Patient: HI IM 27. 6MONTHS BACK GOT MARRIED, I GO THROUGH SONOGRAPHY LAST MONTH i,e ON MAY 11,2011.EVERYTHING IS NORMAL BUT ENDOMETRIAL ECHO MEASURES IS 6MMS.WHAT SHOULD I THINK OF THIS......CAN U PLEASE EXPLAIN ME WHATS THE PROBLEM.NOW THIS MONTH I GOT PERIODS BUT THE BLEEDING IS NOT MUCH THE IS BROWN.LAST 2 MONTHS WE ARE PLANNING FOR A KID...........PLEASE HELP ME OUT.......THANKS A LOT Doctor: hello welcome to health care magic please do not panic. you will get pregnant. just relax. are your periods irregular? please do tell if they are regular or not and for how many days?this time that you got your period you said the bleeding was not much right? did you get the period on time or was it delayed? if it was only slight i suggest you get a blood pregnancy test serum beta hcg titre done. well if that is neagtive then pregnancy is ruled out as of now. the endometrial thickness you mentioned is on which day of your cycle? 6mm thickness is nothing to be worried about. relax and just get the blood test done. also remember most couples take about a year to conceive. just ay cool n calm."
},
{
"id": 84195,
"tgt": "How can symptoms of tandrilax overdose be treated?",
"src": "Patient: Hello doctor, a friend of mine had a overdose of tandrilax, it s a remedy in south america, i ve found some english info about it here: WWW.WWWW.WW Now he can t pee anymore, he drank lots of water, even tried to puke but nothing came out, he s having some stomach pain and feeling a bit dizzy sometimes, he s like this for 2 day but he can t go to a hospital right now, what can he do? Doctor: Hi, Tandrilax has to do nothing with the symptoms your friend is having, may be it cause some stomach pain but it does not affect the peeing. Hope I have answered your question. Let me know if I can assist you further. Regards, Dr. Salah Saad Shoman, Internal Medicine Specialist"
},
{
"id": 151148,
"tgt": "Oligodendroglioma grade II post brain tumor operation, on levipil, frisium, have giddiness and blackouts. Effect of medicines?",
"src": "Patient: My Husband age 28 yrs has been operated for brain tumor in April 2012 and detected with oligodendroglioma grade II. he is on levipil 500mg and frisium10mg twice a day since 20 days. but from yesterday he is suffering from giddiness and blackouts at times and feels the movement of his blood flowing on his left side. is this side effect of the medicine Doctor: Hi, Thank you for posting your query. Your husband has had a brain tumor surgery. Post surgery, there is a risk of seizures (fits) due to either scar of the surgery or due to minimal residual (left over) tumor. Therefore, medicines such as levipil and Frisium have been started. These are anti epileptic drugs that prevent the recurrence of fits. Symptoms of black outs are more likely to be fits, rather than the side effects of these medicines. However, please consult his doctor for sorting this out. Best wishes, Dr Sudhir Kumar Md DM (Neurology)"
},
{
"id": 78234,
"tgt": "How to treat suspicious infiltrate on both upper lobes?",
"src": "Patient: I undergone chest pa xray and its shows suspicious infiltrate on both upper lobes. They recommend a apicolordotic xray and it shows no significant findings. I have no acute or active infiltrates. I just wonder why the first xray shows that I have one. What does it implies? Doctor: HelloYour chest X-ray findings may suggests suspicious infiltrate in both upper lobes.It may be due to infective etiology.You are rightly advised for apicolordotic chest X-ray.It is a special view to visualize apical area because apical area is overlapped by bony ribs and clavicle.Normal chest X-ray(PA view) may miss the findings.Apicolordotic view overcome this problem and its helps in better evaluation of apical areas.Empirically you may be prescribed antibiotics.You may need follow up chest X-ray.If findings and symptoms doesn't improve then tuberculosis etc should be excluded.Get well soon.Take CareDr.Indu Bhushan"
},
{
"id": 113188,
"tgt": "Had a Hip replacement, constantly suffering from soreness in hip, tingles in ankles. Is it an internal bleed?",
"src": "Patient: I have had a hip replacement 4 months ago. Now I am constantly suffering with my hip feeling sore around the wound although it appears ok. My leg also tingles especially around my ankle and feels warm. Its bit like the feeling you get when have had a sprain . Can you tell me if its an internal bleed , circulation or what. The movement in my hip is fine and there is no swelling . Thank you. Doctor: hi Dont be so anxious as far as your walking properly without any instability i dont think there should be any problem . secondly tingling in ankle could be due to handling sciatic nerve during surgery nothing to worry it quite common problem and will get resoled on its own within few months . go for regular follow up with your surgeon and take check x rays if advised"
},
{
"id": 182751,
"tgt": "Suggest remedy for chewing problem due to loose tooths",
"src": "Patient: I have four tooth inmy lower jaw. All four tooth are loose. I have getting tchewing problem.Thease tooth root also loose. I want implant in lower jaw. I want remove all four tooth and replace with new implant inlower jaw. Can you advice to me what should i DO? Doctor: Hello,Read your query as you have mobile tooth in lower in lower jaw see mobility in tooth can be due to poor oral hygiene causing poor periodontal health causing periodontitis , as you are asking for implant yes it is good option to replace missing teeth but for this treatment bone support should be good . For this I will suggest you to consult dentist and go for examination of oral cavity and bone support by investigations by OPG xray. If possible go for implant .Hope this will help you."
},
{
"id": 211909,
"tgt": "Need help as i always get angry very soon",
"src": "Patient: I always get angry very soon. I yell at my family and friends often and there isn\u2019t one person I know whom I haven\u2019t fought with yet. My mom is pretty worried and keeps telling me that it\u2019s going to be difficult after marriage as I\u2019m 22 yrs old now. My mom is quite similar and I\u2019ve always been like this. Lately I feel that some of my friends are avoiding me and now I\u2019m getting worried. Is there anything that I should be doing? Doctor: Hi, thanks for using healthcare magic. You should consult psychiatrist for detailed evaluation of your anger. doctor may advise stress management or anger management techniques. meanwhile you can try self relaxation techniques like Yoga, meditation or self control of anger by reverse counting while angry. thanks"
},
{
"id": 114314,
"tgt": "How can iron deficiency anemia and arthritis be treated?",
"src": "Patient: I have iron deficiency anemia, so far in spite of all the tests there is no known cause. I m taking 2 over the counter Iron tablets w/meals daily, and have had an iron infusion, so far no good results. In the mean time the doctors have me off of anti-inflammatory drugs, so with my arthritis I am in lots of pain. Also I was recently discovered to have A-fib, so I m now on Xarelto, when I think it s all very confusing to me.. Doctor: Hi, I am sorry for your symptoms but anemia along with chronic joint pain, I would prefer to consider this as anemia of chronic disease and I am tilting towards the diagnosis of an autoimmune disorder, where in there could be a Coomb's positive anemia. If the autoimmune disorder is treated the anemia will get better by Itself. I suggest you consult with your physician with this glue and get yourself evaluated please. Hope I have answered your query. Let me know if I can assist you further."
},
{
"id": 159449,
"tgt": "Small cell lung cancer stage, lesions in front brain and lever. Leg tremors, coughing phlegm. Explain?",
"src": "Patient: Small cell lung cancer stage 4, leisons on the front of the brain and liver. Type 2 diabetic. tremors in legs, weakness, unable to walk or stand unassisted, coughing with Phelgm production , slight heart attack, anemia that needs to have a shot administered for the anemia. two rounds of radiation and one of chemo . Patient will not leave bed due to tremors, loss of balance, and inability to walk without assistance. What can the loss of balance tremors and inability to walk be associated with? Doctor: Hello, Welcome to HCM, I am Dr Das. The radiation therapy is a very hard to tolerate. The symptoms you described is just the sign of this intolerance. What you have to do is just give the patient a good nursing care, a good balanced nutritious diet, and multivitamin and antioxidant agent to fight with immune comprised state. Regards."
},
{
"id": 18577,
"tgt": "Should the ER be visited for palpitations and pressure in the chest?",
"src": "Patient: I have had heart palptitations since Thursday of last week, chest feels like something is sitting on it,tired and achy The palpitations are not new as I am on medication for them but just don t feel well.Normally I am a very healthy person I am on metoprolol 1/2 tablet every 6 hrs as needed Not sure if I need to go to dr or not Thank you. Doctor: Hello and Welcome to \u2018Ask A Doctor\u2019 service. I have reviewed your query and here is my advice. I would like to tell you that Palpitations with chest heaviness can then it can be because of underlying heart disease which needs medical attention. Kindly get an ECG and Echocardiography done and consult a Cardiologist Personally for further management. Hope to have been helpful Kind Regards Dr. Bhanu Partap"
},
{
"id": 21340,
"tgt": "What is the treatment for high BP and knee pain?",
"src": "Patient: I am 53 years old, 5 8 and weight 285. I have high blood presure. I fell on my arms and knees. I missed a step. My shoulder and arm hurts very bad at night. I can t sleep. I had a exray and its not broken. Please tell me what I can do to help me rest at night. Doctor: Hello, I understand how you are feeling. Your BMI is >40. You have class 3 obesity, which puts you at a high risk for cardiovascular disease and is th reason for your high blood pressure. You will have to visit a physician for evaluation. You will need to do some test like Lipid profile, ECG, ECHO, Urine test. Take Tab. Ibuprofen 400mg 3 times a day for your body pain.Warm regards"
},
{
"id": 156157,
"tgt": "Can coughing blood be a sign of cancer?",
"src": "Patient: Hi. Four days ago I had surgery on my breast. Reduction, reconstruction and lipo. I had been diagniosed with organising phnemonia few months ago, which eventually I should treat with steriods, but didn't want to before this op. I have been coughing up a little bit of phlem and sometimes there is a bit of blood in it. In November I had a broncoscopy and they preformed lots of biopbsies which all came back clear of cancer and other things (that's why they are putting it down to BOOP) A couple of days after the broncoscopy I was coughing hard and burst a blood vessel and was coughing up blood (I went to the hospital and that's what they told me) It settled down.Should I be concerned about the blood as I am panicking because to me blood means cancer. I do cough hard.I am a non smoker. Thank you Doctor: hi dear. blood on coughing is always abnormal. there are other causes also, other than cancer, for such symptoms eg. tuberculosis, BOOP, ulcers etc. as your previous biopsies were normal, so chances are less that it will be cancer. if your age is less than 50 years, chances are less for cancer. if you are a nonsmoker and female chances are lesser. so i think that the bleeding is related to your previous pathology rather than cancer. but you should consult a chest physician to treat the underlying cause."
},
{
"id": 86262,
"tgt": "Suggest remedy for headache and pain in abdomen",
"src": "Patient: I am a 25 year old male, i left drinking a week ago because i have been experiencing headaches, pain in lower left and upper right region of my stomach, i feel numb in my head and muscle pains. Now the head ache is gone but still feel this unusual pain sometimes in stomach. Should i consult a doctor or eat healthy and wait for it to vanish automatically Doctor: For abdominal pain you can take some antibiotic like metrogyl or zifi-oz after an expert opinion along with pain killer like cyclopam or meftal-spaz for severe abdominal pain. Take rekool-d or syrup gelusil-mps plus empty stomach foe gastric upset. Avoid alcohol completely and take high protein diet."
},
{
"id": 63735,
"tgt": "Suggest treatment for lump in arm pit area",
"src": "Patient: When I breast fed my first born which was 9 years ago I had developed a golf ball size lump under my left arm. It was so painful. I went to see a doctor and he told me it was a milk duct that was pooling milk. He gave me an antibiotic and the lump went down. Ten years later and 2 more breast fed babies until about 1 year ago I still have a small lump in my left armpit area and my left arm will numb. It seems to act up before my period or if I am over tired or stressed. Should I be concerned?? Doctor: Hi, dearI have gone through your question. I can understand your concern. You may have some enlarged axillary lymphnode, axillary tail of breast or galactocels. You should go for fine needle aspiration cytology or biopsy of that lump. It will give you exact diagnosis. Then you should take treatment accordingly. Hope I have answered your question, if you have doubt then I will be happy to answer. Thanks for using health care magic. Wish you a very good health."
},
{
"id": 129297,
"tgt": "Why my knee is popping?",
"src": "Patient: Hi every time i Bend down my knee cracks then when i sit down my knee feels like it is out of place. And then i walk and my knee sounds like its popping. And i am only 15 so i dont think it should me happening. Do you have any.idea what is wrong any information would help. Thank you for your time Doctor: Hello,Can you describe it better? Does it feel that when you bend the knee cap ( patella) goes to the lateral/ outside part of the knee and when you extend the knee it seems to go back to place? I think that you should visit an orthoped in order to do a physical examination and to estimate the health condition of your knee.Hope I have answered your query. Let me know if I can assist you further.Regards,Dr. Emilda Belortaja"
},
{
"id": 103361,
"tgt": "Suffering from asthma from childhood. Using seroflo inhalers. Is it safe to use this drug for a long time?",
"src": "Patient: I am a man of 44 years and I am suffering from Asthma since childhood as my father also is asthma patient. Till 1999 I had taken Deltacortril tablets whenever I had felt the attacks of asthma. Since 1999 I am using Seroflo 250 inhalers 2 puff twice daily. And also kept Asthalin (Salbutamol) inhaler for SOS use. Whether such a long use of this drug is alright or I should stop taking them. Kindly help. Doctor: Hi, Thanks for writing to health care magic. In bronchial asthma during a acute episode we use bronchodilators like salbutamol that you are using. To prevent recurrent episodes we use preventors like budesonide etc to prevent further attacks. After a long use of steroid inhalation your doctor may try and stop those medications to see if there are further attacks. If there are no symptoms you can stop those medications. But if there are symptom recurrences a long term use is required. The side effects out weigh the advantages. So continue to use as recommended by your doctor.Continue to ask further queries as required. Hope this helps. Take care. Dr Y V Siva Sankara Murty M.D.(Pediatrics) Associate Professor of Pediatric"
},
{
"id": 141036,
"tgt": "What causes swelling in the feet after a cervical spine surgery?",
"src": "Patient: hello my husband just had his C,3,4,5,6 done the Dr said it was very bad when he got in there, but before surgery he could hardly walk and numbness were already in feet, with hands fore arms, bothered him to touch huskiness together and siatica bothering him, all symptons pretty much are still here and it seems he is getting weaker , my opinion, and my main concern his feet are swelling it has been 3 weeks since the surgery Thank you Doctor: Hi, The feet swelling is not related to spine surgery. Honeywell have to rule out conditions like DVT and cellulitis as these can cause swollen feet. Consult a physician and get evaluated. Hope I have answered your query. Let me know if I can assist you further. Regards, Dr. Shinas Hussain, General & Family Physician"
},
{
"id": 65743,
"tgt": "Could a lump appearing on hand be related to blood clotting disorder?",
"src": "Patient: I developed a roudish, small lump on top if my hand, after doing nothing to injure it. The lump sort of rolls around but is either atop a vein...or, in it. This happened over a10-min period...suddenly, lump appeared and became purplish in colorish and stingy. I'm worried about developing a blood clotting disorder. Doctor: Hi! Good morning. I am Dr Shareef answering your query. Although it needs a clinical assessment, from your history, it seems to be a hematoma and if there is no history of any injury, and if it increases in size, you have to get it assessed by your family physician or the treating physician to rule out your suspicion of a clotting disorder with related investigations.I hope this information would help you in discussing with your family physician/treating doctor in further management of your problem. Please do not hesitate to ask in case of any further doubts.Thanks for choosing health care magic to clear doubts on your health problems. I wish you an early recovery. Dr Shareef."
},
{
"id": 126848,
"tgt": "What can cause pain and swelling in the foot?",
"src": "Patient: I m having pain and swelling in my foot, located on outside bone of right foot for three weeks. Bone located in middle of foot below ankle bone and swelling is underneath the bone on bottom of foot, limited to underneath outside bone of foot that hurts. Painful when I step on it and when walking. I have not injured my foot; cannot figure out what symptons relate to. Don t know whether to see an orthopedic doctor or podiatrist. Doctor: Hi, It may be due to contusion sprain developed after a minor trauma treatment prolonged walking. As of now you can take analgesics like Acetaminophen or Diclofenac for pain relief. Also apply ice packs for faster recovery. If symptoms persist, better to consult an orthopedics and get evaluated. Hope I have answered your query. Let me know if I can assist you further. Regards,\u00a0\u00a0\u00a0\u00a0\u00a0 Dr. Shinas Hussain"
},
{
"id": 111948,
"tgt": "Have pain in the lower back, stomach pain and dizziness",
"src": "Patient: I have pain in my back lower left side about where my kidney is. when I push on this area it makes me sick to my stomach. I have been having some dizzy spells and vertigo, on and off for about 3months, I have also had stomach pain and bloody stools this has been going on for about 3months too, along with joint pain in both arms at the elbows and wrists. I just found out last night that my grandmother had lupus, and that all my issues could be related to lupus. Doctor: HiI have studied your case. Your symptoms suggest possibility of lupus you can do blood investigation like ANA,ds DNA, if those are positive then medication can be given.I will advise to do MRI whole spine for diagnosis.A holistic approach is required, which includes exercises, meditation , and acupuncture along with healthy diet and herbs. Hope this answers your query. If you have additional questions or follow up queries then please do not hesitate in writing to us. I will be happy to answer your queries. Wishing you good health.Take care"
},
{
"id": 75223,
"tgt": "What causes pain in chest with anxiety disorder?",
"src": "Patient: Hello, I am an 18 year old female, 5'7 inches about 210 pounds, and my problem is chest pain. I was diagnosed with anxiety disorder about 3 weeks ago, and 8 days ago I started to experience sharp fleeting chest pains, reoccurring over a few minutes to a couple hours Doctor: Hi welcome to the health care magic Anxiety induced panic attack can lead chest pain problem.... But before labelling case as anxiety induce pain rule out cardiac causes by.... -Blood pressure measurement-EKG-Tread mill If specific cardiac cause found than treated accordingly For anxiety you might prescribed alprazolam like suitable drug Behavioural counselling beneficial in anxiety treatment Do yoga and moderate exercise daily Take care Consult physician or pulmonologoist for examination with keeping all this in mind"
},
{
"id": 92705,
"tgt": "Gas comes out when urinate, stomach and abdomen pain, test done no infection. what could it be?",
"src": "Patient: Sometimes when I urinate, gas (I'm guessing) comes out. It is not gas from my bowels. I also have pain in my stomach and abdomen. I was checked for a UI and was treated. The symtomes went away for a couple of weeks. Just recently the gas was back. I went today to get another test but there was no infection. What else could it be? Doctor: HIThank for asking to HCMYou must be applying force while passing the urine, or you must be cross checking it whether gas expel or not, because while passing urine one need not to apply any force, urine comes out with it own, even if it comes at the time of voiding why to worry, this is not the indication of any disease, or abnormality you just control your self, every thing will be alright. have nice day."
},
{
"id": 194925,
"tgt": "Are tiny echogenic foci in both lobes of the prostate a serious concern?",
"src": "Patient: prostate: normal in size measuring about (3.4 *3.2*3.0) cm, volume being 16.9 cc and the wight is 16.9 gm. few tiny echogenic foci of about 0.5-0.6 cm are seen in both lobes of prostate. impression; few tiny echogenic foci in prostate (D/D: 1. foci of calcification 2. corpora amylacea Doctor: sometimes prostate can calcify (mostly after recovering from infections). these calcifications appear as echogenic foci on ultrasound. since the size of your prostate gland is normal, and the calcifications are in lobes of prostate i think you dont need to worry about it."
},
{
"id": 174553,
"tgt": "How to treat loose stools in 5 month old child?",
"src": "Patient: My 5 month old baby has been suffering from loose stools since 2 weeks. He is not dehydrated as he is passing urine regularly. I am giving Nutralin B since a week but there has been no improvement in loose stools (around 5-6 times a day). Also, the urine becomes yellow soon after I give him Nutralin B, is this normal? He is exclusively breastfed. What might be the cause and how can I help him? Doctor: Hi having stools 5-6 times/day in a child on breast feeds is not a worry as long as your baby is well,active and no signs of dehydration.Just replenish him with ORS fluids 25ml/each stool if stool quantity is large.YOU CAN ALSO ADD ZINC SYRUP 2.5 ML PER DAY FOR 14 DAYS if stools did not subside in 2-3 days.Hope it answers your query and ready to help you further."
},
{
"id": 53196,
"tgt": "Suggest treatment to reduce ALT and AST levels",
"src": "Patient: my brother is 40 year old. 1 month ago his left temprol was bleed due to high blood pressure. effected area is 4x4 mm at left brain his right side hand and leg effected but little bit moving, memory 90% lost. since his suffering he has given bredin 30mg 4 time daily and capotin 25mg 3 times daily. as per doctors advise yesterday his blood test was taken and found abnormality in liver function test ALT 75 AST 62. has he effected with hepatice or not ? kindly confirm secondly what remedies can be given to him to reduce ALT AST level kindly advise Doctor: Hi.Thanks for posting query at HCM.Usually ALT or AST values higher than \"two times the upper normal limit\", is considered abnormal ( in some countries, ALT or AST values of more than 100 are considered abnormal). \"HepB and HepC should be ruled out\".HepA is a self limited disease in most cases.Value of AST or ALT greater than 85 or above maybe investigated further.Alcohol ingestion and obesity are common causes of fatty liver disease.advice :- abstinence from \"Alcohol\" - LOW fat diet should be followed, AVOID junk food and beverages- decreased oil consumption (oily food)- NO red meat- green vegetables should be ingested daily- use lemon juice (lemonade) once in a day- reduce weight if overweight/obese-\"recheck liver enzymes after 6 to 8 weeks\" and/or ultrasound.any further questions are welcomed.hope to answer your concern.wish you good health.regards,Dr Tayyab Malik"
},
{
"id": 61637,
"tgt": "What causes pulsating lump in neck and abdomen while having ankylosing spondylitis?",
"src": "Patient: I have a lump in my abdomen that is approximately 1 inch long. It is too the right of my belly button. it has been there for about two years now and sometimes it looks like it has edema around it. It is bean shape, movable and painless. I have had a physician tell me that they thought it was some sort of scare tissue. I also have found one in my left upper thigh, not painful, but leg does sometimes ache if I sit for too long. I now noticed about a week ago that I have a similar lump on the side of my neck. I have always had a palpable pulsating lump in my upper neck that comes and goes, But this one is below it. I feel like sometimes it is difficult to swallow, ear fills full and that side of my face feels numb. Also over the last few weeks there are times that my vision gets blurry and I feel as though I am going to faint. I was diagnosed with ankylosing spondylitis about two years ago. Over the last year I have gain almost 30 pounds and cant get it off. I have never had a weight issue. I am a 40 year old female Doctor: Pulsating lump on the neck may be an aneurysm.For this you consult some vascular surgeon. Regarding other lumps, these appear to be neurofibromas for which no treatment is required."
},
{
"id": 136591,
"tgt": "Suggest remedy for dimple on knee cartilage",
"src": "Patient: i am 35, 5 9 and about 180 pounds. Just recently had a scope done on my knee to repair a torn meniscus and the dr found a dimple on my cartilge. Nothing was done at the time as the main concern was the meniscal tear but just wondering what i can do to not require any more surgery. Doctor: Hello, I have studied your case. As per your symptoms you need to do physiotherapy and knee exercises to improve range of motion.Due to old injury to knee there is early degeneration of knee joint.Life style changes will help in reducing pain which include, VISCOSUPPLEMENTATION: Hyaluronic acid injections given into the joint (especially large joints like knee joint) in prescribed dosages will give good short term pain relief in arthritic joints ranging from 1-2 years. You need to consult orthopaedic surgeon for taking it.Autologous chondrocyte implantation may help if no relief.Hope this answers your query. If you have additional questions or follow up queries then please do not hesitate in writing to us. I will be happy to answer your queries. Wishing you good health.Take care"
},
{
"id": 160576,
"tgt": "How to treat worms found in the stool of my baby?",
"src": "Patient: I have found worms in my 5 year old daughters stools this evening. Ive just read that hook worms can be caught through walking bare foot and she always does this, even outside. when she gets treatment will it be effective to treat all the possibilitys of type of worm that it could be? Doctor: Hi,Hookworm infestation is not very common in children, and not easily visible with naked eyes. Usually what we see in stool is pin worms (small, thin white ones with twisting motion). You can give Albendazole 400mg tab single dose. This will clear both pin worm and hookworm.Hope I have answered your question. Let me know if I can assist you further. Regards, Dr. Muhammed Aslam T. K., Pediatrician"
},
{
"id": 80638,
"tgt": "What to do for breathlessness and dizziness?",
"src": "Patient: I feel like I have shortness of breathing and as if my heart is pumping awkwardly I also a tingling or dizziness sensation throughout my body for a brief second or two then goes away it does make me feel as if I m gonna pass out. At other times I feel like vomiting please help what do you think it could be... I ve also just started feeling this way for maybe about a week now Doctor: Hello dear, thanks for your question on HCM. I can understand your situation and problem. In my opinion we should rule out cardiac cause first for your symptoms. Because hypotension can cause breathlessness, giddiness , dizziness and vomiting.So better to get done1. Ecg2. 2d echo. If both are normal then no need to worry much for cardiac cause. Get done chest x ray and PFT ( pulmonary function test ) to rule out pulmonary cause like bronchitis. So better to consult doctor and discuss all these. First diagnose yourself and then start appropriate treatment."
},
{
"id": 48629,
"tgt": "Should I be concerned about bloating stomach and frequent urination before dialysis?",
"src": "Patient: ESRD patient only had dialysis once this week (Tuesday) due to inclement weather (facility closed). I am at the third day of of no dialysis and my stomach is bloated and urinating frequently. Since I am scheduled for treatment tomorrow should I be concerned? Doctor: If you r urinating frequently no need of emergency dialysis for you, bloating of stomach may be because of gastritis, if you have frequent hiccups u need emergency dialysis"
},
{
"id": 13259,
"tgt": "Please suggest treatment for rash on the penis",
"src": "Patient: So recently ive developed a rash on my penis recently and hasnt gone away(Im virgin), the rash doesnt hurt, burn, bother me in anyway which is why I thought it was my fungal infection tinea visicolor which dissapears for almost a year after cream treatment but always starts to come back. Its only on my penis getting larger in size. Its also slightly dry where the red patches are Doctor: Hi. Yes, from your explanation it looks like a fungal infection. You should consult an Internal Medicine Specialist and get evaluated and a thorough clinical examination can be done to rule out the exact cause. In case of fungal infection you can be advised a course of antifungal medicine like Flucanazole or Itraconazole. Also apply a combination of steroid and antifungal ointment. Maintain a good genital hygiene. Daily change your clothes and innerwear and also use cotton innerwear. Hope I have answered your query. Please let me know if I can assist you further. Regards. Dr. Honey Arora, Dentist."
},
{
"id": 21839,
"tgt": "Should a diabetic and heart patient follow diet control?",
"src": "Patient: someone that diebetic, has a pace maker, a difibulator, on blood pressure meds, and many other meds, constantly indulges in chocolate covered raisins, or any other sweets and snacks. His INR is constantly changing is gets to high. Does diet play a part in his ups and downs???? Doctor: Hi, diet plays very important role in all diabetes and heart patients. eg. if pt Is on diabetic medicines.. but is taking mango juice or sweets or chocolates, his sugars will never be under control.similar is for hypertension and heart disease.this will have implications that the medications are not working but in reality, the diet is not being followed and pt is non- compliant.the worse thing is the number of medicines will increase which on turn is worse for pt himself."
},
{
"id": 165702,
"tgt": "How to cure knot on the head after falling down?",
"src": "Patient: my 3 year old son hit the back of his head on the playground. he is walking fine and did not fall asleep. it is a knot on the back of his head. he is talking fine and seems to be his usual self ( a little grouchy). i tried applying ice and he was not too happy about that. what should be my concerns at the moment? it is almost bedtime. thank you! Doctor: Hello dear!I can understand your concern.Its absolutely fine if he is not drowsy,is talking normally,didnt have any sort of fits,bleed from any site like mouth or nose and didnt develope headache or vomiting.So he is perfectly fine by the grace of God.You should not be having any concerns.This small bump will resolve on its own.It will take some time.Hope it will help.Wishing your child good health :-)"
},
{
"id": 141204,
"tgt": "What causes the sudden onset of numbness in the arms?",
"src": "Patient: A few days ago, I had a sudden numbness come over my arms while I was unloading laundry from the dryer. About 15-20 minutes ago the same thing happened. I had just gotten off the couch and walked into the kitchen to get my iPad. When I went to sit down, the same sensation happened. This time i also felt it across my left side back area. It s never happened befor these two incidents Doctor: Hello and Welcome to \u2018Ask A Doctor\u2019 service. I have reviewed your query and here is my advice. I passed carefully through your question and would explain that your symptoms could be related to a cervical spine disorder (bulging disc). For this reason, I recommend consulting with your attending physician for a physical exam, a cervical spine X ray study and nerve conduction studies. A cervical spine MRI study may be needed. Hope I have answered your query. Let me know if I can assist you further."
},
{
"id": 221479,
"tgt": "What causes delay in periods?",
"src": "Patient: Hello Dr., My LMP was on 25 aug 2010 . my periods are absolutely normal & timely appears after 28 days. on 10 sep i had an IUI & from 12 sep night onwards till next 15 days i have been prescribed one duphaston daily at night. My expected mp date was 22 sep , 3 days are exceeded still did not get my periods . What should i conclude ? am i pregnent or its bcos of duphaston periods are delayed. Doctor: Hello dear,I understand your concern.In my opinion the duphaston is a progesterone hormone which supports the endometrium for the implantation.It might not cause delay in periods.As the period is delayed there might be chance of pregnancy.I suggest you to do a urine pregnancy test after a week of missed period for accurate results.So relax.Avoid stress.Best regards..."
},
{
"id": 186076,
"tgt": "What causes puckish feeling in my mouth?",
"src": "Patient: HI, I have kind of Pukish feeling in my mouth from 2 days, i dont feel vomiting though. I have slight head aches, but they come and go. I am eating food 3 times without missing, but feel hungry often may because i am not eating properly. Can u suggest what could help me. Doctor: Thanks for using Health Care Magic.Read your query and based on what you have written ,looks like it is something related to your general health.I would advise you to visit a GP and get a check up done and rule out any health issues.Take multivitamins .Thanks and regards."
},
{
"id": 99433,
"tgt": "What causes breathing problem during running in case of asthma?",
"src": "Patient: I am a highschool distance runner and have allergy induced asthma. My symptoms have been almost entirely gone for three years, so about a year ago I went off my Flovent. Since then I have had no symptoms except for minor breathing issues during really humid hot races. In track I perform really well. 4:33 mile. But in cross country my times dont match up with my track times. I just ran a 5k and am in better shape than last track season but my time didnt match the expectations at all. I ran a 17:01 when I should have been well below 16:20. Is my asthma secretly still holding me back with every breath but I just dont notice it because the symptoms are miniscule? Doctor: Hello,Thank you for asking at HCM.I went through your history and would like to make suggestions for you as follows:1. Asthma is a dynamic disease, which tends to go under remission and may become symptomatic again in unpredictable way. The main cause of this unpredictability is because it is multifactorial, varies with environment, immunity, diet factors, stress levels, etc.2. If you think you are experiencing breathing problem while running, I would suggest you spirometry with post-exercise testing. This tests measures changes in lung capacity with exercise, especially running. If it is not available/possible, you can perform peak flowmetry before and after running, which will measure changes in peak flow rate after running.3. Also, known as exercise induced bronchospasm can be due to \"mild\" asthma, often unnoticed, so above spirometry will also help to identify the status of your asthma and your baseline lung capacity.4. If above testing does not suggest exercise induced bronchospasm, other possible causes of breathing problem during running can be anemia (low hemoglobin levels), other micronutrient deficiencies, nose obstruction, etc.5. In general, I would suggest you to maintain your hydration by adequate amounts of fluids.6. I would also suggest you to keep your inhaler handy while performing running/exercise, so you can take it for wheezing or shortness of breath due to asthma.Hope above suggestions will be helpful to you.Should you have any further query, please feel free to ask at HCM.Wish you the best of the health ahead.Thank you & Regards."
},
{
"id": 11575,
"tgt": "What causes red ring under armpits?",
"src": "Patient: My husband has a large, circular red ring under one of his armpits. The ring is approximately the size of an orange. We are using Cortisone cream at present. What could be the cause of this condition? Would you suggest I take him to a doctor immediately? Doctor: Hello and welcome to healthcaremagicI would keep a possibility of fungal infection. This are because of closely apposing body folds e.g thighs and groin, axilla, submammary folds etc can trap sweat, heat and moisture, therefore making the area favorable for fungal proliferation.If I was the treating doctor I would have asked you to use a topical antifungal like clotrimazole cream, twice daily, regularly for 4 weeks. An Oral antifungal e.g fluconazole once weekly for4 -6 weeks OR oral terbinafine once daily for 2-4 weeks, is also advisable for a more complete cureAn OTC antihistamine e.g cetrizine 10mg once daily would help you in symptomatic itch relief.Keeping the folds dry by using an antifungal dusting powder would help in preventing the recurrence.regards"
},
{
"id": 8607,
"tgt": "Used a cosmetic product. Had mercury toxicity. Stopped using the cream. Will I suffer from the adverse effects from mercury?",
"src": "Patient: hi, my name is yana and im from malaysia. i just want to ask about mercury toxicity. recently, i bought a cosmetic product and used it for 1 month,. then suddently there's report in news paper wrote that those cosmetics (cream) contains mercury which then cause them to be removed from the market. after read that news paper, i decided to stop from using the cream. my question is do my body save from the adverse effects from mercury? until now i dont feel any symptoms of mercury toxicity..theres no pain, dizzy, or else. Doctor: yes if you are not taking thatif you have eliminated the toxic substance your skin is saed from further toxicity as you dont have toxic effects til now"
},
{
"id": 208889,
"tgt": "Suggest psychological reasons for wet dreams at night",
"src": "Patient: Hi !My prob is I get too many wet dreams, which is very frequent, which I think is the reason behind my body weakness ( maybe I am wrong) I don't masturbate. I am too skinny and I feel tired alot. I can't sleep due to the fear of wet dreams. Please Help .. Doctor: Hi,I do understand your concerns regarding wet dreams. Once a person becomes sexually mature, semen production starts which is a continuous process. A person then either has sex, or masturbates and has night falls. Your wet dreams are occurring because you are not masturbating or having sex.Masturbation and wet dreams (nocturnal emission) are a normal physiological process. They don't lead to any physical or mental health problems. It also doesn't make anyone weak.Your weakness may be due to nutritional deficiency. You can consult a general physician to rule out any and get treated accordingly. Wet dreams has nothing to do with it. So, you just need to relax about having wet dreams or if you really want to stop it, masturbate. Both are completely normal. Hope this information was helpful. Best wishes."
},
{
"id": 28889,
"tgt": "How can cough be treated while suffering from sinusitis?",
"src": "Patient: In Oct of 2017 I got a sinus infection. Gradually I developed a cough . The cough still persist to this day. As does nasal drainage, The cough is sometimes extreme and forces me to vomit mucas. At other times it is a unproductive cough. In the mornings often the dry mucas seems to make me cough and there is a GREY snot, mucas. PLease advise. Doctor: Hi,As per your query, you have symptoms of unproductive cough while suffering from sinus infection which is due to poor immunity and chronic respiratory tract infection. Need not to worry. I would suggest you to take steam inhalation 2 to 3 times a day to relieve congestion. You should take over-the-counter antihistamine drug like Benadryl along with cough expectorants.You should drink plenty of fluid and do warm saline gargles. Consult ENT specialist once for proper examination and go for blood tests and x-ray chest. He/she may prescribe antibiotics course nasal corticosteroids, antihistamines and decongestants as well.Hope I have answered your query. Let me know if I can assist you further.Regards,Dr. Harry Maheshwari"
},
{
"id": 31175,
"tgt": "What causes boils between my toes?",
"src": "Patient: Hi sir I have infection between figures of leg and small boils came in each fingure and in some boil blood is also there and leg also swollen pl reply I dont have sugar i have used t-bact mupirocin ointment usp some boils where puchered and water came I have used terbicip tablets also what to do next Doctor: Hi,From history it seems that you might be having scabies infection and now became infected producing this problem.Firstly go for antibiotic medicine course to cure bacterial infection.After that go for scabies treatment with scabies infection with anti-scabies lotion.Apply lotion after proper opening of bumps where scabies parasite lodges.Apply at night after shower, keep this lotion through out night and take shower in the morning.Go for same treatment of your family member if they have this infection.Ok and take care."
},
{
"id": 173493,
"tgt": "Suggest treatment for overactive bladder and mood swings in a child",
"src": "Patient: My nine year old who does have an overactive bladder seems to be wetting even more than usual , is thirsty and hungry all the time and sleeps a lot more . Has had urine checked so know its not diabetes . Also has become a lot more moody and upset about wetting . Doctor: Hi,Thanks and welcome to healthcare magic.Normally babies are dry at night by 3-4 years.But sometimes children will not become dry upto 12-14 years and even more.This is known as nocturnal enuresis .Restricting fluids after 6-7 PM, insisting them to empty bladder before going to bed will lessen the problem.parents should wake them up once or twice to pass urine.Do not criticise your child before others about his problem, rather appreciate him if he is dry sometimes.Imipramine 25 mg daily at bed time will help in getting over this problem.Hope this answer serves your purpose.Please feel free to ask further queries if any .Dr.M.V.Subrahmanyam."
},
{
"id": 168213,
"tgt": "Suggest medication for MRSA",
"src": "Patient: My son was diagnosed with MRSA a month ago (finger blister/lesion) blister was removed and he took medicine for two weeks with the area completely healed. Monday he was seen at the doctors office for swimmers ear . He has been on Floxin for two days and the pain is getting worse. My concern may be crazy but could this be MRSA as opposed to swimmers ear ? He is 14 and has not been able to sleep for more than an hour for the past 48 hours because of pain. Doctor: Hello. It's very unlikely that the ear problem is MRSA--this is more likely just an external otitis that would respond to Cipro antibiotic ear drops."
},
{
"id": 199244,
"tgt": "How to enlarge the size of penis and testicles?",
"src": "Patient: I'm 26 and my height is 5ft and i weight 130 pounds. I'm very concern about my penis sizes cause it looks the same when I went through puberty and it looks like a 12 year old boy's penis. When I erect I only get 3 inches, and my testicles are very small. I just wondering if I have a condition? Doctor: Hello and . your condition and concern are very common. it's not the length that matters', but, whether you're able to have a normal erection and ejaculate semen.exercise regularly and the fat deposit around the external genitalia will reduce. if you remain in doubt, see a urologist. he'll examine your genitalia and if it's abnormally small, he will advice hormonal tests. usually, LF,FSH,S.TESTOSTERONE,PRL,blood sugar,thyroid tests like free T3,free T4,TSH and a scrotal scan may be advised.if you do have any doubts, contact me Dr.Matthew J.Mangat.Chief Consultant Urologist.+919446345671."
},
{
"id": 134872,
"tgt": "Suggest treatment for severe soreness and pain in the knee",
"src": "Patient: Hi I am a runner. I have little or no pain in my knee once warmed up and running. My knee is sorest when I move from sitting to standing especially getting out of the car or up from my desk. It hurts most when I use the other foot to remove my shoe by standing on the back of it and pulling my leg up. I have a consistent but not terrible pain in the back of my knee and around to the top of my shin. But I can still run a pretty casual 30km without it hurting while in action. I m wondering if there is any harm in continuing to run on this knee. Please say there is no harm. Doctor: Hi Dear,Welcome to HCM.Understanding your concern. As per your query you have severe soreness and pain in the knee. Well there can be many reasons for symptoms you mention in query like damaged Menisci , tendinitis , osteoarthritis , gout , bursitis or chondromalacia . I would suggest you to consult orthopedic surgeon for proper examination . Doctor may order certain tests like CT scan , MRI and physical examination . Doctor may prescribe muscle relaxant along with anti inflammatory and recommend physical therapy . Doctor may also recommend surgery in severe damage or to repair the meniscus . For now apply warm compresses , take proper rest , do not massage and take ibuprofen or acetaminophen for pain . You can also apply anti inflammatory and analgesic cream like voletra . Hope your concern has been resolved.Get Well Soon.Best Wishes,Dr. Harry Maheshwari"
},
{
"id": 211231,
"tgt": "Is weakness caused due to psychological disorder?",
"src": "Patient: Dear Sir I am 28 yrs old and going to get married soon .I am very happy with my husband whom I met 7 to 8 months ago.Since last 3 yrs I have been harrassed very much by few people.I have not done any mistake in my part.Today i was a bit nervous and suddenly i felt as sumthing very weak.I never felt this before as I am very bold.I find sumwhat changed in myself.Can I ever return to my original form.I also had a rhinoplasty recently.Was this weakness was because of that or sum pschycological disorder Doctor: Hellothanks for using healthcare magicIt is your apprehension about your future. It happens before marriage if u have bad past. it is definitely a psychological issue. consult a psychologist she would help u.Thanks"
},
{
"id": 222625,
"tgt": "Suggest remedies to get pregnant after going off birth control pills",
"src": "Patient: Hi, I have been taking loette birth control pills for a year. My husband and I plan to have a baby soon. Is it safe regarding the baby s health to get off the pill and continue having unprotected sex or have protected sex (using condoms) for some months before trying to get pregnant? If so, for how long can we do this? Doctor: Hi dear, I have gone through your question and understand your concerns.One can get pregnant immediately after stopping the oral contraceptive pills.There is no adverse effect on the fetus once the oral contraceptives are stopped.Hope you found the answer helpful.Wishing you good health.RegardsDr Deepti Verma"
},
{
"id": 201176,
"tgt": "What causes pain in left groin?",
"src": "Patient: Hi. My husband has had a pain In his left groin since yesterday. It has gotten worse during the night time and pain killers have not taken the pain away. He has also complained of pain in his groin with an erection. It does not hurt when he passes water ?? Doctor: HIWell come to HCMThis could be muscular pain, muscular spasm may be very likely and this could be due poor posture this can be best manage with the following drug, ask him to maintain good posture, else this is nothing to worry 1) Tab Chlorzoxazone with Acetaminophen as per needed hope this information helps, have a nice day."
},
{
"id": 225386,
"tgt": "Took Familon, 321 pills after periods. Had unwanted 72 twice. When will periods start?",
"src": "Patient: I AM RONITA.I AM 30 YEARS OLD.LAST MONTH PERIDS DATE WAS 03-01-2013 .IT HAS FINISHED ON 7-01-13.I STARTED TO TAKE FEMILON PILLS FROM 8/1/13 .AND I TAKE 321 PILLS REGULARLY ALMOST SAME TIME.IT HAS FINISHED ON 28/1/13.IN BETWEEN I HAVE TAKEN UNWANTED 72 PILL ON 7/02/13 AND 10/2/13.I HAVENT START MY PERIODS THIS MONTH.BUT THERE IS BLOOD INSIDE OF MY VAGINA .I WANT TO KNOW EHEN MY PERIODS WILL START AGAIN THIS MONTH? Doctor: Hi,Thanks for writing in.Taking unwanted pills has caused your delayed periods. If you were on femilon, you should have continued taking it. Unwanted 72 is an emergency high hormone dose pill and causes delayed or early periods in most women and it can persist for the next two cycles. Unwanted 72 should be taken only in emergency and within 72 hours of unprotected sex. Taking it regularly can cause cysts in ovary and irregular periods.Hope this helps"
},
{
"id": 163770,
"tgt": "Why is my baby crying for the last 3 days?",
"src": "Patient: My baby has been crying for 3 day in the row now, always tired and wants to sleep all the time but she sleeps really bad as well, (normally she a really happy well baby sleeps well too) I know something is bothering her cause she makes a face like shes in pain, shes not teething her gums are not swollen or red, and she also is refusing solid food, in the evenings she vomits and during the day after she eats her belly gets big and she wont burp.. any idea whats going on with her? i ve taken her to the clinic and they didnt find anything wrong with her.. :( Doctor: hi, this is most probably stomach infection. Child must be crying due to pain abdomen. I repeat examination by doctor should be done. Take care."
},
{
"id": 39814,
"tgt": "Can tract infection harm IUD ?",
"src": "Patient: Hi,My name is Lonesha. I've or should I say I assume I have a tract infection because of the frequent bathroom runs,foul smell,and discharge but my question is Ive had it for over a week with no treatment and I just got my IUD a little over a moth ago,could this infection mess up anything with IUD? Doctor: HI, thanks for using healthcare magicIUD function would not be affected by the infections but IUD are associated with an increased risk of developing infection.Urinary tract infections can present with some or all of the following symptoms: frequency of urination, change in the odour or colour of urine, urgency, discomfort on urination, abdominal pain, fever, vomiting, loss of appetite.You need to consider visiting your doctor for an assessment and treatment which includes antibiotics.I hope this helps"
},
{
"id": 174231,
"tgt": "Suggest treatment for head injury in a 3 week old baby",
"src": "Patient: I was feeding my 3 week old baby and I dropped the bottle on her head. She hasn t been crying and I even gave her a bath. I don t see that the soft spot has sunken in but she is my first and I am worried that I hurt her. The bottle is a six ounce bottle and was completely empty at the time. Should I take her to the hospital? Is this a serious issue. Please help! Doctor: Hi,Thank you for asking question on health care magic.Empty plastic bottle usually does not cause serious injury.You need not worry unnecessarily.She will come round without any problem.Hope this answer will serve your purposePlease feel free to ask any more queries if requiredTake careDr.M.V.Subrahmanyam MD;DCHAssociate professor of pediatrics"
},
{
"id": 10268,
"tgt": "Suggest treatment for hair fall problem",
"src": "Patient: Hello Doctor, i am 28 years old male. I have been suffering from hair fall problem for last few years. I had very dense and thick hairs, but now it has reduced drastically. I consulted a doctor and she advised me to use Mintop (5%) foam as I am suffering from male pattern baldness. Kindly suggest whether this line of treatment is ok or not..is it a long term solution to the problem or not? Doctor: Hello and Welcome to \u2018Ask A Doctor\u2019 service. I have reviewed your query and here is my advice. Mintop 5% will help you but the results will take sometime to be seen. at least a years treatment with Mintop (minoxidil) is advisable. Combining that with PRP (platelet rejuvenation plasma) treatment is doubly beneficial. Here, the doctor takes the patients own blood, spins it in a small machine which separates plasma from RBCs and that plasma if then injected into the scalp area with very thin needles. Also, work on your nutrition level and start multivitamin supplements especially those which contain biotin. Hope I have answered your query. Let me know if I can assist you further."
},
{
"id": 68318,
"tgt": "Suggest treatment for a lump in the chest",
"src": "Patient: A lump on the centre of my chest, its about an inch or so long and its fucking solid it seems movable the depending on how I sit or lay down, its under the skin like attached to the bone it seems,. It appeared a few days ago, it\u2019s not itchy or discoloured but its painful as I said and it seems to be bigger than when I first noticed it I ain\u2019t sure what it is cysts aren\u2019t sore and their squishy this is sore and its not sqwishy Doctor: Welcome to health care magic. 1.The possible cause could be infective aetiology, probably hair follicle infection with collection with in.2.In this case after examination one has to rule out the involvement of chest wall so after examination an ultrasound will help.3.Ulrasound will help to describe the nature of the lesion, its extension. And a lateral chest chest x-ray to rule out bony involvement. 4.Even before a course of antibiotic can tried, on not responsive state you can go for the above investigations.5.AS A REMEDY - do no squeeze the lump, as you will only complicate and delay its healing process, leave it alone that way it will heal better with medication. Good luck.Hope i have answered your query.Any thing to ask do not hesitate. Thank you."
},
{
"id": 164322,
"tgt": "Suggest remedy for seizure",
"src": "Patient: I think my granddaughter had a seizure and when we took her to the er we were released and told to observe her because she didn t have any sign currently. She is 1-1/2 and was in daycare. It scared the heck out of her teacher and they described my granddaughter has walking sortta in a drunken state, her left arm drawn up to her chest and shaking, she said she was non responsive but had a look of horror and then she fell and hit her head on the table. We rushed her to the er and after waiting 3-1/2 hours, went into the room and they briefly examined her (no blood work, x-rays or nothing) just listened to her lungs, looked in her ears and checked to see if she was running a fever. They said she appeared to be fine and to take her home but observe her ever 2 hours. Doctor: I understand your concerns.The history depicted by you is consistent with a seizure. However for epilepsy we need at least 2 seizures 24 hours apart.It would be of great help if you record such a scenario with a smart phone. It would help to understand the attending physician about the condition of child. Otherwise Electroencephlogram (EEG) of the child can be done.Anti convulsants given for seizures are only advisable after the above test and opinion from your attending Pediatrician.Regards"
},
{
"id": 207400,
"tgt": "Suggest treatment for intermittent explosive disorder",
"src": "Patient: From my childhood I used to feel peculiar fears,sensations as if from outside world.Three times during my 21 years of life I did outburst seriously.I became little mild with time.Recently I fell in love with somebody,but somehow that outburst of agression came out.He now Wants to be just a friend.I have also been physical with him.I know it was my fault.But now I just want to forget him.I want be a normal person.Because what happens during those outbursts I have always been unable to explain.I want to be normal.What should I do? Doctor: Hello dear. Dont feel upset. Your problem is treatable. Juat go some sessions of counselling. It will help you a lot. Visit a psychiatrist. To forget your past relationship you should make yourself busy by playing instrument dancing sports etc. For low mood tab escitalopram. Visit a psychiatrist. So dont worry. Hope my suggestions helpful and thankful to you. Take care."
},
{
"id": 192974,
"tgt": "What causes white stuff with bad odor under the foreskin?",
"src": "Patient: hi im a 32 year old male and have recently noticed white stuff under my foreskin and is a little smelly. I do tend to itch alot and im worried because im wanting to start a family this year and with this condition im finding it hard to tell my wife and put off having sexual intercourse. Please can you help Doctor: Hello, The Yellowish/whitish substance is called smegma. Smegma is a combination of shed skin cells, skin oils, urine, sperm cells and moisture. It occurs in both male and female genital organs. In females, it collects around the clitoris and in males, smegma can collect under the foreskin. There is no evidence that smegma causes penile cancer, but its presence over a long period of time may irritate and inflame the penis, which may increase the risk of cancer. It may also make it harder to see very early cancers. Clean the area beneath your foreskin daily twice after pulling your foreskin back. Hope I have answered your query. Let me know if I can assist you further. Take care Regards, Dr K. V. Anand, Psychologist"
},
{
"id": 172609,
"tgt": "Is adenoid removal safe and effective in treatment of breathing problems?",
"src": "Patient: My son aged 7 is having chronic breathing problems due to nasal blockage cause by a septal deviation compounded by a 40% enlarged adenoid.. Our ENT has suggested removal of the adenoid as an immediate measure.I would like to know the pros and cons in terms of general health and immunity as a result of adenoid removal Doctor: Removing adenoids is indicated as your child is having breathing difficulty. Enlarged adenoids may cause nasal speech or breathing difficulty, snoring,obstructive sleep apnoea. Nothing contraindicated in removing adenoids."
},
{
"id": 68675,
"tgt": "What is the hard and painful lump in my armpit?",
"src": "Patient: Two days ago I woke up with a marble size lump under my armpit. The lump is sore, hard, and some white pus has come out of it. Today I also have what feels like the beginnings of a chest cold. I don't know if the two are related, but at the moment I am more concerned about the lump. Doctor: welcome to Health care magic.1. The most possible cause in this case is carbuncle / furuncle - infective cause of hair follicle. 2.Second possible cause are infective cause of abscess formation. And other causes are lymph nodal enlargement, lipomas - fat (soreness and pain are general not seen)3.If you are my patient, after examination i would have sent for an ultrasound lumps to evaluate the nature of lump and it is extensions (from - to).4.An incision and drainage maybe needed if its large and infective, after that a course of antibiotic will help.5.See your GP and get the needful tests / treatment done. Good luck.Hope it helps you. Wish you a good health.Anything to ask ? do not hesitate. Thank you."
},
{
"id": 153631,
"tgt": "Are swollen lymph nodes around the neck sign of cancer?",
"src": "Patient: Please type your query here. Since this query is directed to our panel of specialists and super-specialists, please mention your problem in as much detail as possible. Do not forget to highlight your Age, Gender, Medical History and Treatment History of the problem you Hello.. I have a question I was diagnosed with low grade hpv about a month ago and I wanted to know what are my chances of it going away and how long does it take...I'm 29 years of age...I also have about 7 to 8 lymph nodes around my neck for more than a year.. What are the chances of me having cancer?are facing. Doctor: Hi, dearI have gone through your question. I can understand your concern. You have multiple enlarged lymphnode. It can be due to some reactive hyperplasia, tuberculosis or lymphoma. You should go for fine needle aspiration cytology or biopsy of that lymphnode. It will give you exact diagnosis. Then you should take treatment accordingly. Hope I have answered your question, if you have doubt then I will be happy to answer. Thanks for using health care magic. Wish you a very good health."
},
{
"id": 106713,
"tgt": "What causes flank pain and lumps around the ribcage?",
"src": "Patient: I have left flank pain, left rib pain, muscle knots in lef flank pulling one rib out of place, onset of headache/nausea today, increasing pain - had x-ray last week if upper back and left flank, some mild arthritis that I\u2019m told doesn\u2019t account for rib pain. Urine screen about three months ago, everything looked fine. Abdomen tender when examined by Dr. last week. Was sent for x-ray and to continue PT, f/u w/ PCP tomorrow night, worried about increasing pain, unsure if am should go to ED or wait til PCP appt tomorrow night. Doctor: Hello and Welcome to \u2018Ask A Doctor\u2019 service. I have reviewed your query and here is my advice. You can take Tramadol and paracetamol combination 2 times a day for five days take Omeprazole twice a day for five days take Cefixime twice a day for five days. Take rest. Don't bend your back. Take proper sleep, apply Diclofenac spray at pain site 2-3 times in a day apply warm water bag application at pain site 2-3 times in a day. Hope I have answered your query. Let me know if I can assist you further."
},
{
"id": 7012,
"tgt": "Can abdominal and lower back pain be because of implantation ?",
"src": "Patient: I ve abdominal pain , lower back pain and very little white watery discharge sometimes for past two days. Is it an implantaon pain? I am trying for pregnancy . today is 24th day. We had sex from 8th day till 22nd day. Doctor: Hi Welcome to HealthcareMagic These symptoms are non specific. There may not be any pain during implantation though there may be little bleeding per vagina known as implantation bleeding. As you have had intercourse during your fertile period, there are high chances of conception. Do a urine pregnancy test to confirm pregnancy once you miss your periods . Take care."
},
{
"id": 212984,
"tgt": "Suffering with stammering problem, difficulty in speaking fluently during stressful conditions. Precautions?",
"src": "Patient: myself kiran .from hyderabad.I have a stammering problem occured in the middle(i.e 10 class) till now i do not rectify the problem and when ever i go for any interview i will get fear and i cant speak fluently i that time i will very fastly and suddenly my words gets stopped in the middle?Is there any precautions to be followed,any changes in the food. Doctor: hello welcome to healthcare magic you can do the following things- 1) Relaxation exercises like deep breathing, yoga or meditation 2) You can take Propranolol 10mg/20 mg half hour before facing any interview (Consult a doctor for that) 3) Speech Therapy Hope it helps Good luck Thanks"
},
{
"id": 125057,
"tgt": "Suggest treatment for infective spondylodisctis",
"src": "Patient: Hi Dr., I have upper and lower back pain. My MRI & X-RAY reports show evidence of infective spondylodisctis at D1/2 levels. I am taking below medicines : R-Cinex 600 Pyzina 750 Combutol 800 Benadon 40 Pantosec 40 Please advise me , I am taking correct medicine or not? Thanks for your help Doctor: Hello, Nothing much to worry and your treatment is in the right track. You can go for a bone scan to assess the progress if required. Hope I have answered your queries. Let me know if I can assist you further. Take care Regards, Dr Shinas Hussain, General & Family Physician"
},
{
"id": 11893,
"tgt": "Having dark patches on the skin after using glycolic peel and hydroquinone. Melasma or hyper pigmentation diagnosed. CO2 laser advised",
"src": "Patient: I have dark patches on my skin due to intense glycolic peel and use of hydroquinone . The dermatologist thinks it is melasma or hyper pigmentation! I think they are actual scar burns, because the texture in that area is rough and deep. Not to mention my face has actual blisters in the area where it is dark and rough. I had this peel a year and a half ago, and my skin has not recovered. I tried hydroquinone again, but my skin has not gotten better. I have an olive complextion now it is clearly gone. They told me to try the co2 laser , but I would have to continue the use of hydroquinone for 3 months. Is there another alternative. Doctor: Hi, Thanks for writing in. Hyperpigmentation can be due to postinflammatory changes. Sun exposure can aggravate pigmentation.Keeping the sunprotection is important to treat hyperpigmentation. Use sunscreen with SPF 50. Effective oral and topical medications are now available for Hyperpigmentation . My advise to you would be to use topical triluma cream with good moisturizer. Few treatments like microdermabrasion and kojic acid peel can help to spots effectively. Consult a good dermatologist who can provide you with these options along with effective oral prescription medicines. Hope this helps. Regards DrSudarshan Dermatologist"
},
{
"id": 97550,
"tgt": "Suggest Ayurveda treatment for incontinence",
"src": "Patient: I am diabetic and with medicines, my sugar levels are under control. I am suffering from incontinence and have to get up 4 to 5 times at night due to which I lack sleep. Are there any medicines to cure incontinence - either in Ayurveda or allopathic? Doctor: if i were to treat you have your sugar levels checked again before starting the treatment.finding them normal prameha nashak and mutra marg balkarak treatment will be given.i would suggest medicines like chandraprabha,asanadi vati,ushirasava,for you.strict diet control,exercise,mental relaxation.as a exercise contract and relaxation of urethra opening might help to achieve better control."
},
{
"id": 55080,
"tgt": "Suggest medication for lower quadrant pain on removal of gallbladder",
"src": "Patient: A little over a year ago i had my gallbladder taken out it was sudden and very painful but since then i have had pain in my right lower quadrant pain, I am an alcoholic and I quit drinking for four months to see if that took care of the pain but it didnt the pain is sensitive to the touch and a dull ache and when i bend a certain way it feels like it is pinching or tearing something. I use Ibuprofen for the pain and tylenol but nothing seems to help. Can you tell me anything about what might be going on? Doctor: hi.noted history of cholecystectomy and occurence of right lower quadrant pain. it may be a radiating post-operative pain, but other causes such as muscular strain, infection (diverticulitis), obstruction (possibly new lesions), appendicitis, ileitis must also be ruled-out. it is best if you consult with a doctor, preferably a gastroenterologist or a general surgeon, for physical examination and clinical evaluation. diagnostics (such as ultrasound, ct-scan, blood counts, etc.) and management (medical and/or surgical if indications are found) will be directed accordingly.hope this helps.good day!!~dr.kaye"
},
{
"id": 168354,
"tgt": "What causes red bumps across chest,face,ears and back?",
"src": "Patient: My 18 month old daughter has tiny red bumps across her chest going up to her face and behind her ears, as well as the top of her back. She currently has a bad ear infection and is getting over pink eye, so I wasn't sure if it's an allergic reaction to the medicine or if it's eczema. Doctor: Hi...Thank you for consulting in Health Care magic. Skin conditions are best diagnosed only after seeing directly. I suggest you to upload photographs of the same on this website, so that I can guide you scientifically. Hope my answer was helpful for you. I am happy to help any time. Further clarifications and consultations on Health care magic are welcome. If you do not have any clarifications, you can close the discussion and rate the answer. Wish your kid good health.Dr. Sumanth MBBS., DCH., DNB (Paed).,"
},
{
"id": 82402,
"tgt": "What causes sudden faster heartbeat and pounding head?",
"src": "Patient: For the past couple of days, I have been having problems where my heart will randomly start pounding super fast, it s pounding so hard it hurts my chest, then everyone in a while my head will start pounding also. I just wondered if there was any suggestions on why this could be happening. Doctor: Thanks for your question on HCM.In my opinion you should get done few investigations to rule out cardiac cause.As poundinhead with fast heartbeat can be seen in1. Arrhythmia2. Valvular heart disease.So better to get fone ECG and 2D Echo first.If both are normal than go for Holter monitoring. In this 24 hour recording of your ECG is done. Any arrhythmia can be picked up by this.If this is also normal than we need to rule out anaemia and thyroid problem. So get done CBC to rule out anaemia and S.TSH to rule out thyroid disease."
},
{
"id": 211361,
"tgt": "How is depression, high sex drive and poor sleep related to nystagmus?",
"src": "Patient: I ve noticed that many mornings when I wake up I have nystagmus (my eyes suddenly jump up and down quite noticeably, the rate of those movements is every second or such). That usually lasts only couple minutes. It even happened couple of times during the day but very briefly and in a very mild form. How could this be related to my health problems? My health problems are: 1.Abnormally high sex drive (I can definitely say sex addiction) and I HATE IT. 2.Depression, Anxiety and Depersonalization (probably all caused by the 1. problem). 3.Poor sleep, that may be 8+ hours but it s pretty fragmented. 5.Horrible short term memory, horrible concentration, horrible brain performance (even though I am not stupid at all, just very slow at thinking and my mental clarity is soooo much decreased these days). 6.I ve been diagnosed with OCD/Anxiety as of October 2013. 7.I also seem to have a lot of signs of a high-histamine person, but I still haven t tested it. 8.I have IgE thrice as high as the upper range limit. 9.I have skin rashes, a lot of nasal secret all the time, yet no allergy has ever been found. Other than that I seem to be perfectly physically healthy. What is going on!? :( Doctor: Hiif sucha patient would have been with me it would have been diagnosed as a case of hypomania and treated for this A psychiatrist will give younprescription for it it is not nystagmus as per your description take careDr Lal Psychiatrist"
},
{
"id": 63118,
"tgt": "What causes enlarged bump inside anus?",
"src": "Patient: I am 22 and I have noticed that I have a sore enlarged bump about an inch or so into my anus.its been sore and has been itchy all week.Ive been able to examine it and it dosent look perfectly round I was thinking it might be an enlarged papillae do you think this is what It could be ? Doctor: hi.it is best if you consult with a general surgeon with regards to your anal lesion. yes, it could be an anal papilla, but there are also other possibilities like hemorrhoids, abscess, fistula, etc. a complete medical and physical examination from your doctor will be of great help. management (medical or surgical if indications are found) will be directed accordingly.hope this helps.good day!!~dr.kaye"
},
{
"id": 57137,
"tgt": "What is the treatment for fatty liver grade?",
"src": "Patient: i am suffering from Fatty liver Grade II & i Have also 14 mm gallstone but according to gastroenterologist , do not need to go for operation at this moment because i have no symptoms related with Gallstone. i am taking GOLBI 300 twice a day. is it okay? Doctor: Hi, How are you?Fatty liver can be caused by excessive alcohol intake (Alcoholic steato hepatitis, ASH) or due to excessive fatty food intake and sedentary life style (Non-alcoholic steato hepatitis, NASH). ASH is usually the beginning stages of alcohol induced liver damage and is completely reversible if the patient refrains from alcohol. NASH is being much more commonly diagnosed currently owing to poor dietary habits and sedentary life styles of people. High caloric intake, high fatty food intake and poor exercise regimens lead to deposition of fat goblets in the liver tissue leading to fatty liver (which shows up as increased echo texture on USG). This is also completely reversible in the initial stages if caloric intake is limited and exercise regimens are adhered to. If left un-checked, patients usually become obese, diabetic and in final stages may lead to cirrhosis of the liver which will necessitate a liver transplant.I would suggest a healthy diet and exercise regimen. Although there is no time limit on when to get a repeat scan, I would suggest one, once you have succeeded in losing some weight.Stones in the gall bladder are fairly common finding during ultrasound scans of the abdomen. If it is not causing any symptoms, I would not worry about it and definitely not get it operated on. Hope this helped and please do not hesitate to contact me for further details - rxsuresh@gmail.com"
},
{
"id": 104856,
"tgt": "Sneezing, cold and loss of appetite. Fatigue with headache. Flu symptoms?",
"src": "Patient: Hi, five weeks ago I started having cold symptoms sneezing , runny nose and started feeling really cold and then get hot then cold again it had also affected my asthma as well. I started getting achy all over and lost my appetite . I went to my GP and he said it was flu go home and rest and drink lots of fluid. Which I have done but I don t feel any better. I have a constant headache which feels like drums against my head, I feel so weak and tired, I have pale and yellow tinged skin and I ve lost weight. I get lightheaded sometimes and feel absolutely exhausted if I have to go anywhere. What could it be Doctor: your symptoms does not look like flu since your having from 5 weeks.. the information you have given is not enough to get to the diagnosis.. but your symptoms looks like you really need proper medical attention.. i would suggest you to visit a hospital rather than a clinic.. they will have to do a few blood test and examine you properly to get to the diagnosis.. please take it seriously and visit nearest good hospital.."
},
{
"id": 139581,
"tgt": "What does this MRI report of spine indicate?",
"src": "Patient: Cervical the the cal sac, exactly what happens when there is ib c-5 - c-7 an indentation on the the the cal sac & indentation, disc osteophyte complexes, with mild canal stenosis and moderate bilateral recess narrowing of these levels. MRI recently done. Doctor: Hello, These MRI findings show that there are degerative changes of your cervical spine. From the description it seems that these degenerative changes are moderate and in such cases, conservative treatment usually results in improvement. Discuss with a Neurologist for these issues. Hope I have answered your query. Let me know if I can assist you further. Take care Regards, Dr. Erion Spaho"
},
{
"id": 169080,
"tgt": "Should the tonsils be removed in a 3 year old for chronic tonsillitis?",
"src": "Patient: good evenind drs i have question,my child 3.5 years old, has chronic toncillitis,my pediatric Dr advice to give him multi bottles of ampicillin and after 3-4 months will evaluation if need to removed,he advice to avoide removed it,what is ur opinion thanks Doctor: Hi,Removal of tonsils are not advisable as it takes important part in immune system unless there is recurrent tonsillar infection or chronic pus pockets are there on tonsils.Prevent tonsillar infection by doing warm water antiseptic gargles regularly.Avoid cold drinks and junk food.Ok and take care."
},
{
"id": 10430,
"tgt": "Suggest remedy to stop gradual hair loss",
"src": "Patient: Hi, I'm 25 years old female and have been experiencing hair loss for over 8 years now. I also got my CBC test and thyroid test done but they are all fine - I don't have iron deficiency or thyroid problem, I don't understand the cause of this gradual hair loss. It started after I'd done extreme dieting at the age of 16, but it's been years since I've stopped dieting and eating healthy but still no improvement. I also take multivitamins as prescribed by a dermatologist. Neither does such condition run in my family.. Please help! p.s I am 52 kg in weight and 5'5\" in height. Doctor: Hello and Welcome to \u2018Ask A Doctor\u2019 service. I have reviewed your query and here is my advice. You can use minoxidil 5 % lotion twice daily and Biotin tablet daily one for 6 months. Hope I have answered your query. Let me know if I can assist you further."
},
{
"id": 26244,
"tgt": "Experiencing pain during heart beat",
"src": "Patient: My question is -- sometimes I can feel my heartbeat hurt a little beat. With each beat, it is a very faint, but noticeable pain. What could this be? Its not always, just once in a while. I can feel it in the center of me (where my heart is) and in my back some, too. Doctor: HelloChest pain may be due to many causes like cardio-pulmonary causes,refulx etc.It important to know your age.It is safe to exclude cardiac causes first.You may need proper clinical examination and few investigations.Investigations include routine hemogram,random blood sugar,liver and renal function test,lipid profile,urine RE/ME,chest X-ray (PA view),ultrasound of abdomen,ECG,TMT,ECHO.You may need further test like angiography after these initial tests.Proper treatment depend upon findings.Get well soon.Take CareDr.Indu Bhushan"
},
{
"id": 10528,
"tgt": "What is causing thinning of hair?",
"src": "Patient: I ama 60 year old female who has been under alot of stress for 5 years due to multiple deaths in my family all at once. Getting better. My hair has been thinning for a year or so. I noticed a year ago wads came out in the shoer one or two times. I had been low on Vit D, take 2000 in suppliments a day. I also now take a multi vitamin daily as well. THought perhaps it was due to vitamin deficiency. What are your thoughts. I always had VERY think hair until now..... Doctor: Hello and Welcome to \u2018Ask A Doctor\u2019 service. I have reviewed your query and here is my advice. As per your case history you are having female pattern baldness. My treatment advice is - 1. Use a good herbal hair oil and shampoo for routine use. 2. Take good nutritious diet full of green leafy vegetables and milk. 3. Take an iron supplement and vitamin B12 supplement. If problem persists then consult a dermatologist. Hope I have answered your query. Let me know if I can assist you further."
},
{
"id": 52090,
"tgt": "Both kidneys of my mother are not working",
"src": "Patient: My mother is 68 yrs old and she is diabetic for last 12 yrs and she is taking insulin for last 7 yrs and now recently both her kidneys not working Doctor: HI WELCOME TO HEALTHCAREMAGIC She needs to be on dialysis daily. If donor is available she will get better from Kidney transplantation. She needs tighter control of blood sugars. Regards, Dr. Jagdish"
},
{
"id": 68784,
"tgt": "What is the hard lump at the top of his butt crack?",
"src": "Patient: My husband has stage 4 rectal cancer. He had bladder cancer 20 years ago. He has a hard lump at the top of his butt crack and has extreme pain in his rectum. He does have an infection and is on antibotics. Could this hard lump be another cancer? Doctor: Hello!Thank you for the query.No, this lump is rather not a cancer. However it might be caused by a rectal cancer. Especially if there is a fistula caused by such cancer. The other possibility is that he has pilonidal cyst which has gotten infected. Due to ingrown hair in the butt crack area tissues get irritated and infected. This leads to an abscess formation.For sure he should visit a general surgeon with this problem.Hope this will help. Regards."
},
{
"id": 190608,
"tgt": "Severe tooth pain, numbness in chin, lips. On antibiotics for infection. Root canal done, numbness persists. Cause of concern?",
"src": "Patient: Hello, I was suffering from a severe toothache and my right side of chin and lower lip starting to get numb. I went to the dentist and he gave me antibiotics and told me to come back in a week for treatment and upon my return they proceeded with a root canal . and he said that the numbness will still be there for a while after the root canal due to the fact that there was an infection. It has been almost a week since my root canal and I am going to see my dentist next week. The numbness still remains.How long does it take to go away? Should I be worried? Doctor: hi according to the information provided by you i would say that the one full week was wasted before starting the RCT in the infected tooth. the numbness will go but due to the reason that RCT was stated a bit late the after effects of inflammation will take some extra time to heal. there is nothing to worry. ask your dentist to priscribe you some multivitamin course to hasten the healing process. take care."
},
{
"id": 190744,
"tgt": "Lump on gum, no pain, feels when pressure is applied",
"src": "Patient: Hi, I m really hoping u can put my mind at rest cos I am driving myself mad. I found a lump right up high on my gum above a front tooth which is a crown. I did go to the dentist a couple if days before finding it because the area felt strange when I applied pressure to it, like the crown was going to fall out. The dentist examined the area and said it was fine, it was 2 days later I felt the lump. When you look at the area u can t see anything, the gum looks totally normal and it doesn t cause me any pain at all but I can sometimes feel it when I smile and my lips put pressure on it. It is a hard lump, like bone and I ve no idea how long it s been there. Doctor: The data you have given is not sufficient to reach a definitive diagnosis. Was the tooth infected and/or operated for a root canal treatment before the crown placement? How long has it been since the RCT/crown placement done? For now, with the data given, i would guess it might be due to a periodontal abscess in the region. And if there had been an infection and RCT was done, chances are infection may not have resolved completely before the obturation which may have caused a periapical cyst or an abscess. A radiograph of the tooth is needed to confirm/rule out the diagnosis. If present, it may require an antibiotic and anti-inflammatory treatment and if that doesn't work you may need to get your RCT redone. Either case you will need to visit your dentist to get a clear picture."
},
{
"id": 58270,
"tgt": "Diagnosed with hepatomegaly with diffuse parenchymal disease. Treatment advise?",
"src": "Patient: hi,doc goodevnng im Pfc Daniel b. Diaz may have a hepametogaly with diffuse parenchymal disease & splenomegaly .how to threat that & what is prohited food.actually im here confine Army General Hospital almost 1month now.please help me....doc,i need u help because poor no money....plaese help me....i wish u to help me...pls contact my no.0000.doc,the processing of my doctor in here hospital very slow & not cmpleted the medical materials,specially medicines... Doctor: Hi and welcome to HCM,thank you for your query.You have to find the cause of hepatomegaly first and then it can be treated. it can be vascular,cirrhosis,hepatitis or some other cause. after you establish the right diagnosis the the treatment can begin. Usually dietary habits and certain medicines are enough but sometime more aggressive therapy is necessary. If there is metabolic cause of your liver damage then you need to find ou tis it wilsons disease or hemochromatosis. Wish you good health. Regards."
},
{
"id": 179365,
"tgt": "Suggest medication for nausea caused by a herbal inhaler",
"src": "Patient: Hi, my 10 months old baby played with my (chinese)herbal inhaler (little plactic can)and bit its bottom until it cracked. I thought he had swallowed a bit of the herbal which is coated in the cotton ball inside the package. Soon after he coughed hard and vomit some drool with air bubble. He seems normal right after, should I be worried. Thank you so much. Doctor: If he vomitted within a few minutes and is well now, then why should you worry? In most cases, genuine herbal products are anyway free from side effects."
},
{
"id": 119904,
"tgt": "What should be done for sore and swollen knee after fall?",
"src": "Patient: My mom has fallen on her knee and it swells with activity and is very sore to walk on. The swelling is just below the knee cap (We believe). The bump is roughly the size of a half dollar. Should we see a doctor? Has she fractured anything possibly? Doctor: Hello, Injuries to knee are either of the bone or of the surrounding ligaments or a combination of them. Anything intraarticular would present as swelling of whole of the knee and not only a part of it. You need to get an x ray done,which will be diagonostic,if not proven so you have to wait with conservative management till the time swelling subsides,and at that time. If the symptoms persists you have to go for an mri. Till that time you need to give rest and support to the knee with the help of a brace and do cold fomentation until the pain and swelling subsides. Hope I have answered your query. Let me know if I can assist you further. Take care Regards, Dr. Rohan Shanker Tiwari"
},
{
"id": 80898,
"tgt": "Is fibronodular findings in the lung below clavicle related to tuberculosis?",
"src": "Patient: i am diagnoised with fibronodular findings in the right lung below clavicle. Should this type of findings i.e fibronodular be of tuberculosis only? or there may be some other reason also for this findings i meant some other non infectious disease or due to hereditary lesions? Doctor: Fibronodular opacity is usually due to primary pulmonary tuberculosis but usually healed by itself. So no need to take treatment in that case.fibronodular itself represents healed tuberculosis"
},
{
"id": 6494,
"tgt": "How long does it take to get pregnant after having intercourse ?",
"src": "Patient: hi we had intercourse in last 2 days i would like to know when can i get pregnant and during this period can we have intercourse again Doctor: Hi welcome to healthcareMagic Forum you must wait until the time of periods, if the menstruation is missed at the regular time , then you need to do UPT (urine pregnancy test), if the test comes positive then you are pregnant... you need to further confirm the pregnancy by USG (ultrasound) scanning... yes you can have coitus with your husband..."
},
{
"id": 118950,
"tgt": "Anemic. Lower back pain, mucus in stool, fatigue, no stomach pain. Worry?",
"src": "Patient: Hello I'm a 37 year old female I have always been healthy except for always being anemic I have three children . My problem started about 4 months ago and is continuing . I have bloody stools sometimes the toilet can get filled lower back pain, and now like a mucas in my stool, I am very fatigue and always feel sick . No stomach pain I am starting to worry Doctor: Hi,Have you ever had your kidneys checked? Because there could be something wrong with them, they produce this substance called erythropoetin which promoted blood production by the bone marrow.Plus since you have lower back pain, which is sometimes associated with the kidney it could be that you're kidneys have some problem.The bloody stools could be because of haemrrhoids, but without any tests its hard to say.I recommend you see a doctor right away because for an anemic person to lose blood is a very dangerous situation. After running some tests they can narrow down the cause and accordingly treat you for it.I hope this answered your question."
},
{
"id": 209415,
"tgt": "What is the effect of trileptal for depression?",
"src": "Patient: Can you please tell me what would be the effect of taking trileptal if one is depressed, but possibly not bipolar? Is it possible for the drug to help depression, or does the conceived help of the drug confirm a bipolar disorder? I feel like i might have depression, but i do not exhibit behaviors of spontaneous highs to lows, or manic highs. Yet this is what my doctor prescribed. Doctor: Hi,Thanks for writingTrileptal is Oxcarbazepine. It is a mood stabilizer, however its role in depression alone (non-bipolar) is not well established. Kindly ensure a comprehensive discussion with your doctor in this regard. Hope that helps,Dr A Rao"
},
{
"id": 202677,
"tgt": "Will masturbation cause any damage after urethral stricture surgery? Refraining from sexual activity required?",
"src": "Patient: Will I cause any damage by masturbation after urethral stricture surgery? I recently had surgery to treat a stricture with an cytoscope (I believe) about a week ago and I m wondering if I can cause any damage or worsen the condition by masturbation. My doctor didn t tell me anything about refraining from sexual activity and the stricture itself is fairly far down in the urethra, close to the base of the penis as I recall. From what I ve read, 4-6 weeks is roughly the recovery period but I have very little willpower for this situation. Will I be ultimately ruining the procedure or causing further damage? Doctor: HelloYou have undergone endoscopic surgery of Visual Internal Urethrotomy for stricture Urethra a week ago.Normally it takes 2 weeks to for complete healing of the incised urethra after VIU.Sexual encounter of any form either intercourse or masturbation may damage the incised segment of urethra and will delay the process of healing .So it is better to avoid sexual encounter either sex or masturbation for 2 weeks,Dr.Patil."
},
{
"id": 189854,
"tgt": "Bump inside the gums of a loose tooth. Oozes blood, not painful. What could it be?",
"src": "Patient: I have a strange bump beside one of my loose teeth it is soft and while i was brushing my teeth i poked a little hole in it ad blood came out! I am nervous now because to googled it an things like root canal and surgical came up. But what could it be it isn t really painful but it worries me that blood came out. Please help Doctor: dear friend thanx for sharing your concern, loose tooth means you have localized periodontitis that means infection in your attachment apparatus of the jaw . this infection causes pain, bleeding, pocket. and mobility of teeth. tooth becomes mobile due to surrounding bone loss to which it is anchored. if timely care is not taken then tooth becomes mobile and finally falls off . it is infectious too. as it involves many bacterias therefore. i would suggest you to visit your dentist in the early stages and get OPG done. this would help you to check the amount of bone loss and infection in other teeths too. it can be cured by deep scaling,root plaining, curettage and flap surgery. remember periodontal pathology requires treatment as well as maintenance therapy. you are adviced to maintain regular brushing , flossing , mouth rinsing with antiseptic solution. hope this helps take care thanx"
},
{
"id": 21226,
"tgt": "Is there a heart wrapping surgery that controls the electrical circuits from going off?",
"src": "Patient: I was diagnosed with heart disease in 2005. The doctors found a scar in the right chambler of my heart which causes my electrical circuits to fire off and which causes me to have VT. I have had two surgeries and neither worked. I am now implanted with a ICD and I am on several medications. I have gained over 50 to 60 pounds because of depression and other circumstances. I heard of a new procedure on television where there is something that you can wrap the heart with and it controls the electrical circuits from going off. Is there such a thing and do you think I cold find out more about it. Any help or advice will definitely be appreciated. Thanks from YYYY@YYYY Doctor: Hi!, I am very sorry to hear your Cardiac status.It looks like you have undergone the best possible modality of treatment till date, I could suggest you to review with cardiologist regarding optimising your medications - One thing is that Depression or Anxity itself will trigour the arrhythmias - so it would be of great benifit if you can practise some Yoga / Meditations.Regarding the procedure you are talking about - its an arteficial polymer which id covered arround the heart, its just to reduce size of heart in case of Dilated Cardiomyopathy.Other Procedures would be - Electrophysiologic study (EP Study) & Radiofrequency ablation (RF Ablation)."
},
{
"id": 78858,
"tgt": "What causes left sided pain in chest radiating to right side?",
"src": "Patient: ok. A few days ago my right palm was very itchy, then my left, then the tops of my feet, etc. I noticed yesterday maybe a small bite, which went away, on my left arm. I have been going to the bathroom very often (#2). It feels like gas but it is not. This has occurred for a couple of days now. Last night after dinner my chest started to hurt. It is hard to explain - it was a pain from my heart - something like a panic attach. I breathed, tried not to think about it then it went away - it came back several more times during the nite. Trying to sleep the pain shifted from my left side to my right. Doctor: Thanks for your question on Health Care Magic. I can understand your situation and problem. By your history and description, possibility of stress, anxiety and panic attack related GERD is more. GERD is due to laxity of gastroesophageal sphincter. Because of this the acid of the stomach tends to come up in the esophagus and cause the symptoms of left /right chest pain, chest tightness, burping etc. And all these, stress, anxiety and panic disorder are known to cause GERD. So better to consult psychiatrist and get done counseling sessions. Try to identify stressor in your life and start working on its solution. You may need additional anxiolytic drugs too. For chest pain, take proton pump inhibitors and avoid hot and spicy food. Don't worry, you will be alright. Hope I have solved your query. Wish you good health. Thanks."
},
{
"id": 14356,
"tgt": "Suggest treatment for skin rashes",
"src": "Patient: I have a rash on my back.It starts in the middle of my back,actually there is two patchy areas over my lung area and the rash has spread up my back to my shoulder area.It has also been accompanied by a cold,runny nose and stuffed up,cough, head ache,sore throat and that's about it as far as any symptoms.The rash is a little itchy but not bad.I have had it for 3 days now and no one else in the house has it.The rash looks like little red bumps,I did take some allergy medicine and I thought it was helping but it has not. Doctor: Hi,It seems that probably you may be having viral rash or allergic dermatitis. Kindly consult dermatologist for firm diagnosis. For systemic symptoms, consult physician. He might give symptomatic treatment to relieve cough, cold and fever. Blood tests may be done to rule out other pyre tic conditions. For skin rash, your dermatologist might give antihistaminics to relieve itch. If he feels for allergy, might give steroid in tappering dose. Local application of mild steroid cream may be helpful. I hope you got my answer.Thanks.Dr. Ilyas Patel MD"
},
{
"id": 212824,
"tgt": "Mother having mental problems. Talks loud, behaves weirdly, does not recognize anybody. Does not want medicines. Suggestions?",
"src": "Patient: I have problems communicating with my mom. She is getting worse. She talks out loud and says shes talking to other people. My sister sent her a Christmas card with her picture in it. She threw out and said it wasnt my sister and she was dead. She says weird things, talks out loud in the middle of the night. Help! She wont see a doctor and wont take meds. She thinks that i m not me or thinks theres four of me.... any suggestions Doctor: Dear Shelliszek! Hello. I understand what you are going through. Its difficult sometimes to handle people with mental illness. By the description you have given, its quite obvious that your mother is suffering from serious mental illness probably psychosis. However you failed to mention her age and since when she is having these symptoms. Most of the countries have laws pertaining to in-voluntary treatment of people with mental illness. May be you can contact your police or a local psychiatrist who can guide you regarding the local legal matters and procedure for forced hospitalization. Such patients can harm others or some times themselves. Until you take some help, you be careful with her and take care of her properly. Hope I am able to solve your concerns."
},
{
"id": 114304,
"tgt": "Suggest treatment for hemochromatosis",
"src": "Patient: my husband was diagnosed with hemochromotosis in June 2013, after 8 hospital stays and numerous test he now has liver failure, diabetes and stage D heart failure. His EF rate was betweeen 5-10% now at 20% according to latest Echo, but he can barely walk or talk without being winded. He is on the liver and transplant list as a 1B waiting at home on dobutomine, how long can one stay on the drug without damage to organs? The dr wants to do a right heart cath to measure pressures now sure why...thanks Doctor: Hi, EF is very low. Hemochromatosis surely affects heart, liver, pancreas so I have liver failure, heart failure and pancreatic failure leading to DM. Dobutamine support for heart is essential as it is chronic heart failure. Wait for transplant. Hope I have answered your query. Let me know if I can assist you further. Regards, Dr. Peeyush Kumar Saini, Pathologist and Microbiologist"
},
{
"id": 193172,
"tgt": "What causes a reddish stain on penis after masturbation?",
"src": "Patient: Hi! Yesterday after taking care of myself after two days (mastubating, 3 times) I suddenley noticed I had a dark redish/violet stain like on my penis skin (shaft?). It's not painful and I'm not sure if it got better from yesterday or remained the same. Am just a little worried, can you help? Doctor: HI, It can be due to urinary tract infection, if spot is present on the nerve, it can be due to ischaemic condition. You can take penis doppler to detect if any disease condition is produced. Hope I have answered your query. Let me know if I can assist you further. Take care Regards, Dr S.R.Raveendran, Sexologist"
},
{
"id": 42091,
"tgt": "Why did my all 3 IVF cycle fail?",
"src": "Patient: hello doc, im 30 n my husband 32 yrs. im 5.2 \" n weigh 70 kgs i have undergone 3 ivf cycles with dr. anoop gupta- delhi. everything was very good(lining of uterus, egg quality, e2), i got good no. of eggs, embryo quality was excellent. i got 5 emb, 3 emb(8 cell) transfrd 0n day 3 in cycle 1 n 2. in cycle 3 i got 2 blastocyst on day 5. all 3 ivfs failed. now what shud i do? shud i undergo some test or 4th ivf? im afraid my eggs will exaust. Doctor: Hello,Welcome to healthcare Magic.I have gone through your query and Would like to reply in detail as follows:1. Difficult to assess reason.2. You might benefit with PGD in next IVF.I hope your doubt is clarified. Let me know if you wish any other information.Regards,Dr. Mahesh"
},
{
"id": 192534,
"tgt": "What causes pain and swelling in groin with blood in urine?",
"src": "Patient: My dad went in for an operation today to have kidney stones removed, he has previously had a scan. He was told the camera showed no stones. he has had groin pain for about 2 months and his groin is swollen, he has also been passing blood in his urine. his esr is 23. he has been referred to a general surgeon and an orthopaedic surgeon but no one has given any indication as to what the problem is. Doctor: Hello,It may be due to several causes. For further assessment you may require complete hemogram, urine analysis, ultrasound abdomen after urologist consultation. Further treatment mainly depends upon the underlying condition. For pain you can take tablet acetaminophen. You may require detailed evaluation to rule out urinary tract infection or bladder or renal stone. Keep your self hydrated. You may require antibiotics after consultation. Hope I have answered your query...Let me know if I can assist you further.Regards,Dr Shyam kaleFamily and general further"
},
{
"id": 10301,
"tgt": "How to get rid of gradual hair fall at the center of my head?",
"src": "Patient: Hi Dr. I am having a gradual hair fall at the center of my head and a small patch(1 cm x 1 cm) where sculp can be seen in the left side of my head. Pls provide me the solution for this problem as well as ur contact details as I stay in Bathinda itself. Doctor: Hello and Welcome to \u2018Ask A Doctor\u2019 service. I have reviewed your query and here is my advice. You seem to have androgenetic alopecia. It is characterized by hair loss and gradual thinning of hair from the front, mid and vertex of the scalp. Minoxidil solution and finasteride tablet are approved medications for androgentic alopecia in males. I suggest you to use 1ml of minoxidil solution, twice daily on scalp skin. In addition I suggest you to take an oral antiandrogen i.e finasteride 1mg tablet, once a day for long. Treatment is long term and it is better to start treatment in early grade of hair loss, as it is easier to preserve existing hair than to regrow new hairs. Hope I have answered your query. Let me know if I can assist you further."
},
{
"id": 30006,
"tgt": "Is having cough for 23 days after 2 weeks of cold worrisome?",
"src": "Patient: I ve had a cough for about 23 days now after dealing with a very bad cold that lasted for about 2 weeks, where I had a fever, vomited, and had a sore throat. Could it just be that I am just weak after that so I couldn t deal with this cough more quickly? I don t think it s anything too bad like bronchitis since the phlegm has been clear consistently, but I would still like to know if I should be worried. Doctor: hello, it is possible to last cough for 23 days after a severe upper respiratory tract infection due to laryngeal involvement in the episode. This is more in people with atopy, viral wheezing, or bronchial asthma. you haven't mentioned in the history regarding any if these. However this cough should get lesser by time. But if your cough getting worsened with time you have to see your doctor. Other thing is if you are having fever together with the cough it is a sign, infection still persisting. Other than fever if you are having general malaise, body aches and pains or chills, it also mean infection still persists. Then you have to meet your doctor. If you don't have those alarming symptoms together with the cough you don't have to worry. Steam at least 2 times a day . Take a mucolytic like bisolvin. Your problem would be OK. Thank you."
},
{
"id": 188862,
"tgt": "Tic in he right jaw, chews in the right part of the mouth. Wisdom tooth removed. Suggestion?",
"src": "Patient: 5 or 6 years ago my son was scheduled for 4 wisdom teeth removal. He went in the Dr. office and in 20 min he got back! I was shocked. Dr.G-r pulled out 4 teeth and put stitches in 20 min. That could be funny but my son had some serious troubles aftermath. He got tic in his right jaw. Since then he chews time to time using right part of his mouse and it is untreatable. It is bothering him a lot. Dr G. nor our dentist have no clue how to solve this problem . Doctor: Hi,Thanks for asking the query,Toothache can be due to some broken tooth piece some broken tooth particle that can be radiating to the surrounding tissue.A complete radiological and clinical evaluation is required to determine the treatment plan.I would suggest you to visit to Oral Surgeon and get the checkup done.Hope this helps out.Regards..."
},
{
"id": 42484,
"tgt": "Will these tablets help one conceive?",
"src": "Patient: Dear sir, I am 27 yrs old . I am trying to conceive for last 2 years . I consulted 3 doctors they have given enough of tablets but still I cant. Now at present i have consulted one doctor she told me take metatime 500 mg tablet , fertibex tablet and folic acid. Will i able to concieive with these tablets Doctor: HelloYou are trying for pregnancy from last 2 years & you didn't mention the cause of not getting pregnancy. Any how metatime i.e. metformine is a potent drug used in the treatment of poly cystic ovarian disease ( pcod ) beside in diabetes. Fertibex is clomifene and used in anovulatory infertility , this is used for stimulation of follicles to release ovum 9 from 12 -15 days after 1st day of period date . While folic acid is an iron supplement . Your doctor is in right direction . Yes with the help ( these tablets ) of these tablets you can conceive.Good luck."
},
{
"id": 197183,
"tgt": "What could brown particles in urine after ejaculation indicate?",
"src": "Patient: 36 year old male. Hours after ejaculation I have brown particles in my urine. I have seen a urologist and had a CT scan and an x-ray and they found nothing, went back for a cystoctomy to scope the inside of my bladder and found nothing unusual. I did find out after the x-ray and CT scan that I only have one kidney; I never knew that. I have no abdominal or back pain, nor do I have pain during urination, I only know that something came out when I look into the toilet. I called my urologist after this happened again, I acquired a sample to bring in, they told me its probably a blood clot and not to worry about it and not to bring in the sample. I also have a hydrocele/vericocele on my right spermatic chord. Doctor: HelloThanks for query .Based on your statement that you have posted the brown particle that you have passed in urine hours after ejaculating is likely to be a blood clot mixed with urine which gets turned up into brown colour .In majority of cases this blood clots originates from a small superficial surface of prostate and gets sealed off itself immediately that causes a tiny blood clot to appear either in semen or urine passes after ejaculation .Since this is single episode and all the tests like C.T scan,X-ray and Cystoscopy did not reveal any pathological lesion you should not worry about it .If you have pain or discomfort due to Varicocele that you have on Rt side use scrotal support .This will prevent sagging of testicles and give relief from pain or discomfort .Dr.Patil."
},
{
"id": 44299,
"tgt": "Taking Primolut for ovarian cyst. Have pain, morning sickness and depression. Chances of infertility?",
"src": "Patient: Hello Doctor I am 28 years old and married for 4 years now, just last month discovered that i have a 5.5 cms cyst in my ovaries and my doctor has put on primolut twice a day for 15 days, but i am in severe pain and also morning sickness and depression . I need some feedback of what r the chances that i will go through a surgery and what are the chance i would get infertile. also for your information i had a miscarriage last year. Thanks ( awaiting your answer) Doctor: Hi, Thanks for your query. There is no chance of you becoming infertile after cyst removal. In fact it will help your follicles to grow. the surgery might help you to reduce your pain also. I hope I answered your query, Regards, Dr. Mahesh Koregol. IVF & Infertility specialist."
},
{
"id": 207439,
"tgt": "What causes sudden episodes of confusion and nervousness?",
"src": "Patient: hey,,,i'm 19 years old,,,sometimes i get my mind goes weired,,, suddenly my mind starts saying everything so quickly and if i heard or if i read anything it's the same everything come in so so quickly and that's make me feel nerves,, and it tooks a few minutes and after that everything get normal,,,why is that happening ??? thanx in advance Doctor: This can be happen due to excessive nervousness or some electrolyte imbalance. It can also happen due to reduced glucose levels in the blood. Your history is incomplete ao I can not comment much. You can take salt and sugar rich solution and see the effect. So dont worry. Hope my suggestions helpful and thankful to you. Take care."
},
{
"id": 187163,
"tgt": "What is the treatment for white patches in mouth?",
"src": "Patient: I am 15 years old and have never smoked before. I also never drank any form of alcohol. After having braces for about a year, I notices white patches growing along my mouth (where my teeth touch my gum). I often scrape my braces there and was wondering if this is anything I should be worried about. I have had one patch for around 3 months and another for a week. I did some research and suspect frictional keratosis. However, I am worried it might be something more serious, such as leukoplakia. Can you please help me out? The patches are around an inch long and have a \"scale-like\" texture. Doctor: Hi! Welcome to Healthcare magic.I read your query. Since you are currently having Orthodontic treatment (braces), there are regions where your toothbrush might not reach for effective cleaning. Also, areas where your braces material impinges continuously onto your gums, leaves a white mark called as Frictional keratosis (as you read too). I would suggest you too visit your Orthodontist and ask him for braces adjustment in that particular area. Also you can use Orthodontic brushes specially meant for braces cleaning. Use mouthwash regularly as it removes particles from difficult to reach areas of mouth too. Go for oral prophylaxis every 3-6 months. Maintain good oral hygiene and diet.Hope this answer helps. Thank you!"
},
{
"id": 178576,
"tgt": "Suggest Antihistamine for the child having food allergies",
"src": "Patient: I am looking for an antihistamine for my 9 year old son who has multiple food allergies and environmental allergies. I met with an allergist yesterday and he recommended xyzal syrup which is flavored. i am not sure what the flavor is since he is allergic to a number of flavors including artificial flavors.would you know of what flavors are available? the tablet form contains lactose and he is intolerant to dairy so not sure here as well. please advise. Doctor: Xyzal contains levocetirizine which is an effective antiallergic. Although a child could be allergic to many external principles, he would not be allergic to 'all'. You may still give the medicine a try and see if he has any reactions to it. Chances are that he would not be allergic to them. As of now do not bother about the flavour. The amount of lactose contained in a tablet is not much and so it would probably not be a cause of concern.You may also try giving Montair LC Kid (monteleukast + levocetirizine) in tablet or syrup form.I hope that helps. Feel free to revert back with further queries if any."
},
{
"id": 9905,
"tgt": "Can Mirena cause alopecia areata with acute acne?",
"src": "Patient: i have had my mirena for around two years, a little over a year ago i started lloosing hair i went to a dermatologist who diganosed it at alopecia arearta it sarted of with one circle sport at a time, once the hair grew back another one woulld appear, the circles are now getting bigger and there are always 2 or three at a time. could this be the mirena i also have bad acne that nothing seems to clear up Doctor: Hello, I have gone through your query and your hair loss is not associated with mirena. Alopecia areata is a chronic autoimmune disease of the hair follicles. Hope I have answered your query. Let me know if I can assist you further. Take care Regards, Dr Asmeet Kaur Sawhney, Dermatologist"
},
{
"id": 209652,
"tgt": "How to treat depression, delusions and suicidal tendencies?",
"src": "Patient: thought broadcasting is real.i know because i m suffering from it.i was in depression then my physician gave me a medicine after that i started to feel that every one can hear my thoughts and i am hurting them in some way.they spit after listening my thoughts it seems to be like they are spitting on me it makes me very uncomfortable outside.Because of this rarely i go outside of my house.i m suffering from this problem from last 18 months..and every day i think about committing suicide. thought broadcasting for me is not my delusion it is reality.i m in touch of psychiatrist but he does not tell me about my problem.i have attempted to suicide 4 times.again i m thinking about suicide...please give me some practical advice.i want to know that is there any medicine so that my thought broad casting could be stopped or i should commit suicide..... Doctor: HIThanks for using healthcare magicThought broadcasting is psychotic symptoms and in that case, you need antipsychotic. That would help you to control these symptoms. In your case, depressive symptoms are due to thought broadcasting. Once, this symptom would be controlled by drug, suicidal ideas would also subside. Better to consult a psychiatrist for further management. In case, you need any help, you can ask.Thanks"
},
{
"id": 156502,
"tgt": "Is colorectal cancer transmittable?",
"src": "Patient: I have a friend who had/has colo/re tal cancer. His markers were 3 for a while and has since had a 5 reading and now a 4. We are not intimate at this time but I was curious, and I know this sound ignorant, but can you pass it in some way to another? Saliva, sex, etc. Doctor: No, colo-rectal cancer is not transmitted for one person to other through sexual inter-course, saliva or body secretions.I hope your query has been answered."
},
{
"id": 96686,
"tgt": "Could eating raw chicken accidentally affect health adversely?",
"src": "Patient: I potentially ate some raw chicken for lunch. it was not a big piece of chicken, and i only about about half of it by the time i realized the middle was not cooked. i spoke to the cafeteria workers and they said the chicken was frozen before-hand and went straight from the freezer to the deep fryer. the part that was uncooked, was very raw, but it was also the thickest part of the chicken, so, like I said, I'm not 100% sure i actually injested any raw chicken. Should I be worried? Doctor: welcome to hcm, don't worry nothing will happen by eating a raw chicken. from your conversation i think you have eaten partially cooked chicken pieces.i would like you to remember that during the period of evolution that is when fire has not been discovered people ate uncooked meat and chicken .nothing happened to them. and they were stronger also. if you are feeling ok and if you do not have any feeling of nausea or vomiting sensation nothing will happen. and it will not cause any adverse health effects.hope this explains with regardsDr.Amarnath"
},
{
"id": 24553,
"tgt": "Is snow shoveling possible while having aneurysm?",
"src": "Patient: My sister has a fairly large aneurysm licated behind her heart and down to her stomach and is out shoveling snow. Is there a serious risk - the surgeon told her to continue on as before and she is being monitored. Size I understand is 5.4 ascending and 3.? decending. Doctor: hello there I understand your concern..the aneurysm is fairly large and needs to be repaired. yes a close monitoring is needed a. but be prepared for a surgery in near futurehope that helps"
},
{
"id": 116526,
"tgt": "Suggest treatment for fatigue when suffering from thalassemia minor",
"src": "Patient: Hello, i have heterozygous beta thalassemia minor or intermedia. My HGB is 145/148 which is normal, but my MCV and MCH are low, MCV 61 and MCH 21. RBC 5,8-7,2. Feritine, Fe and everything else is ok. But my red blood cells are to small and low oxygenated to properly oxygenate organism instantly every cell. I have headaches, chronic fatique, long sleep every day. I like judo and swimming very much but i cant practice them because i get very, very thired after training. Is there anything i can do to improve my life energy? Does homeopathy helpes? I'm 21, law student, but i cant keep up anymore because of my low life energy. Does some vitamine supllement alternative therapy can improve my health? Thank you very much! Doctor: Hi,Thanks for asking.Based on your query, my opinion is as follows.1. As the cells are carrying low oxygen, so the increase in no.2. Homeopathy cant help in increasing cell size. Regular schedule along with improvement in stamina necessary.3. Its a genetic problem. Vitamin supplements are necessary to avoid other types of anemia. Blood transfusion will help. Preferably, if you want, go for red cell exchange. Stamina improvement should help. Its a minor variant.Hope it helps.Any further queries, happy to help again."
},
{
"id": 28468,
"tgt": "Suggest treatment for mild concentric left ventricular hypertrophy",
"src": "Patient: My 2D-EchoTest showed that I have a mild concentric LV hypertrophy. What precautions I should take. At present, I am taking the following medicine. 1. Co-Diavon. 2.Stalo-10 3. Ravelol-XL-50. 4. Ecosprin. 5.Riomet-Tio-2mg. 6.Tonnect-Tg. 7. Biostar Gold. Kindly advise. Shankar Pillay Patient e-mail: YYYY@YYYY Doctor: Hi welcome to HCM.I understand your query and concern.Concentric left ventricular hypertrophy in your case is due to Hypertension.The drugs you are taking are perfectly correct.Monitor your blood pressure and heart regularly.I advise you to have a baseline 2 dimensional echocardiography,ECG and lipid profile to assess the basic cardiac reserve of your heart.Restrict the intake of salt to less than 6g/day.Regular physical exercise in the form of brisk walk for 20 min a day for 5 days a week is pretty useful.These drugs should be taken with strict supervision of your doctor .Reduce the intake of fatty and fried food.One pomegranate a day will help to keep your heart at good pace without clot formation.Meditation,yoga,breathing exercises will help. Consult a Cardiologist for further expert management.Post your further queries if any.Thank you."
},
{
"id": 95587,
"tgt": "I am 25 year male & suffering from burning sensation in chest & vomiting. What could be the possible reason for these symptoms ?",
"src": "Patient: I m 25 year male ,weight is 57, suffering from pain like burning in chest at center side not continuous 1 2 time in day for few minute and also suffering from vomiting, Make gaseous in stomach after i taken food cause to vomiting. whats problem Gerd or indigestion ? Doctor: Hi welcome to Healthcaremagic Hi..., GERD is gastro esophagial reflux disease... you must avoid having tea, coffee, oily foods, hot fried items , you need to have food at regular time.. good sleep is also essential... if you follow this after 3 to 4 weeks you will feel the symptoms will reduce and and heal by 6 weeks... Hope I have answered your question.. Takecare.."
},
{
"id": 168530,
"tgt": "What are the complications of having craniosynostosis in an infant?",
"src": "Patient: Hi my daughter is just 8 month old, her progress is not Satisfactory, we shown to doctor, as per him her bones in head been joint early (Craniosynostosis), it should be in 12 to 18 month. So her brain is not being developed, but as per other doctor opinion it\u2019s joint early but there is still some space in her head to grow, we should find the cause, why the brain is not being developed. so we are stil confuse, shell we go for opetation or not. Please help Doctor: Hi...to answer you question precisely - I need to have a look at her MRI brain before suggesting you what to do and how to go ahead. you have said that she is progressing normally - with this I understand that her development is normal.But I have a few questions to ask and I have to suggest you get some investigations done. I suggest you gt back to us at the following link.You can approach me at the following link. Once the page opens there will be an option below my image as \u2013 ASK ME A QUESTION \u2013 click on it. Please find the link below - www.healthcaremagic.com/doctors/dr-sumanth-amperayani/67696Regards - Dr. Sumanth"
},
{
"id": 75949,
"tgt": "What causes chest pain and arm pain?",
"src": "Patient: I'm having chest pain and pain in both arms (worse in left). The arm pain comes and goes every couple of seconds and periodically throughout the day my left arm feels really warm. I've been to the ER three times in the last six months with anxiety attacks. I have had several EKGs done in the last six months which were perfectly normal. I don't feel anxious about anything other than the symptons I have that feel like a heart attack. Doctor: Thanks for your question on Healthcare Magic. I can understand your concern. First of all remove fear of heart attack from your mind because your ecg was normal all the time checked. Such chest and arm pain are seen with calcium and vitamin D3 deficiency. So get done calcium and vitamin D3 levels and if deficiency then you will need supplements. Avoid heavyweight lifting and strenuous exercise. Avoid movements causing pain. Avoid bad postures in sleep. Application of ice packs on affected areas is also beneficial. Don't worry, you will be alright with all these. Avoid stress and tension regarding heart attack and be relax and calm. Hope I have solved your query. I will be happy to help you further. Wish you good health. Thanks."
},
{
"id": 223271,
"tgt": "Should i be worried about an unprotected sex after taking missed pills?",
"src": "Patient: Hi. I've been using diane 35 for almost 3 years now. This is the first time i missed 2 pills for week 3. And took the 2 missed pill and continued taking the other pills. I also took the next pack without skipping. I had unprotected sex after taking up the 2 missed pill Doctor: Hello,Thanks for sharing your health concern with us. I have gone through your query and would like to offer you some information. If you have had the period after completion of the monthly pill course, you need not worry about failure. However, since you missed two pills in a row, you should have used additional protection though missing the third week pills is of lesser risk. If you still suspect the possibility of pregnancy, please go for the estimation of the serum beta-hCG titres as it is a very sensitive and specific test. Further management can be planned accordingly. Hope this helps."
},
{
"id": 16025,
"tgt": "Rashes on face, skin, chronic condition. Applied topical corticosteroid, dry skin. Will venusia cream help?",
"src": "Patient: I have rashes on my face and skin..I ve been suffering from this condition for the last 3 months..My doctor has prescribed antihistamines for 2 weeks and application of a topical corticosteroid ...After the application of the prescribed ointment, my skin becomes very dry ..Can I apply Venusia cream in between? Doctor: Hi, Usually application of steroid cream does not lead to dryness of face. However if you are facing dry skin problem, use a mild soap like dove or pears over face and apply a moisturising cream like venusia or cetaphil. You may also let your consulting doctor know about skin concern, so that skin can be analysed properly and changes in prescribed medicine if required can be done. Take care."
},
{
"id": 160259,
"tgt": "Swollen rib cage",
"src": "Patient: my left rib cage and in between my ribs feel swollen, what could it be and should i go to the doctors Doctor: Hello. . Swollen rib cage indicates that you have had an injury in that area or it is due to overwork of muscles- excessive exercise.Do a hot fomentation and apply a local muscle relaxant. Also take oral muscle relaxant with OTC pain killer. Dr. Rakhi Tayal"
},
{
"id": 95347,
"tgt": "What is causing severe abdominal pain in spite of antibiotics treatment ?",
"src": "Patient: Hi.I am 36 years old femail ,have 2 kids.. I do have pain in my right side it hurt all side. When pressing pain goes to belly button. Doctor did a blood test and found H. pylori IgG ,Abs. I was on antibiotics for a 14 days. But now things got worse. I started to have more pain and about 2-3 days I am very thirsty.An the right side rely hurt. Sometimes is under my ribs,sometimes it is lover. Doctor sending me to Gastroenterolog, but appointment so far away and I do feel bad, and starting to panic that something rely bad is going on. Thank you for help Doctor: hi, welcome to heart care magic family, from your history it seems you might have attach of acute appendicitis.long course of antibiotic might suppress the symptoms but now it is possible that pus is formed and localized to fix part,mostly underneath the appendix.there are all chances that there is rupture of appendix. of course,these are the possibilities i conclude from history. without waiting for appointment of gastroenterologist,you immediately consult surgeon."
},
{
"id": 73456,
"tgt": "What causes intermittent pain in chest?",
"src": "Patient: hi there. im a 20 year old male 190 lbs and experiencing what i think is chest pain, but it comes for a few seconds and goes away. it doesnt spread to my arms or neck, its just like i feel pain in my heart. im not sure what this might be, and i can breathe normally. Doctor: Thanks for your question on Healthcare Magic.I can understand your concern.By your history and description, possibility of pulled muscle is more likely.But better to first rule out heart diseases.So get done ecg and 2d echo.If they are normal then no need to worry for heart diseases.Apply warm water pad on affected areas of chest.Avoid movements causing pain. Avoid heavyweight lifting and strenuous exercise. Avoid sudden jerky movements of chest.Take simple painkiller like paracetamol or ibuprofen.Don't worry, you will be alright with all these.Hope I have solved your query. I will be happy to help you further. Wish you good health. Thanks."
},
{
"id": 157119,
"tgt": "Could the dry patch on the arm, pain in arm and shoulder be a form of skin cancer?",
"src": "Patient: I just had a biopsy taken right forearm, results later in week. Forgot to tell dermatologist about small dry patch on left arm with pimple looking thing in the middle. Have had pain issues with that arm and shoulder, and wonder if it could also be a form of skin cancer. Doctor: Hi and welcome to HCM, thanks for the query.skin cancer is rarely manifested on this way and it should be just followed up frequently. or you can have excisional biopsy or complete surgical removal if you have any doubts about it. Pain in shoulder can be casued by various conditions and most commonly this is arthritis or neuromuscular pain due to injury or muscle pulling.Wish you good health.Regards"
},
{
"id": 181964,
"tgt": "Suggest treatment for bump on side of tongue",
"src": "Patient: I have a very tiny bump on side of my tongue. It stays small and flesh colored but at times gets a bit larger and white colored. Then goes back to smaller and flesh colored. It's small sized about 1/4 of a pencil eraser. Dentist didn't mention it. Please let me know your thoughts.Thank you Doctor: Hello and welcome.Thanks for sharing your concern.Since how long do you have this lesion ?Does it increase in size by a seasonal pattern?you will have to keep a close observation in that.since there is no redness,swelling etc then you can be tension free but please monitor it for any changes.If you find any inconvenience in speech or tongue functions then you can go for simple excision as a treatment.It appears to be lymphatic tissue growth only.Maintain good oral hygiene.Hope it helps.Thanks.Take care."
},
{
"id": 151851,
"tgt": "Can one get cured of torticollis if he takes his medicines regularly ?",
"src": "Patient: my father is suffering from torticollis since 2-3 years age-55 can he be cured i would like to share that he has lost his continuity of tablets from the best neuro sergeon in town and wants to kno if cont. in what time will he be cured completely??? can you help.........? Doctor: Hello, Thanks for your query. It is also called as wryneck. Stiff neck associated with muscle spasm. Common treatments might involve a multi-phase process: - Low impact exercise to increase neck stability. - Manipulation of the neck by a physiotherapist. - Extended heat application. Hope I have answered your query. I will be available for your follow up queries if any. Wishing you good health. Regards,"
},
{
"id": 29643,
"tgt": "Suggest treatment for kidney infection",
"src": "Patient: I had what the Dr. thought was a Kidney Infection , one week ago and was on an antiobodic and pain pills for 6 days, yesterday noon and this am. the pain is back with a vengance , could this be a Kidney Stone or Gal Bladder?? Waiting for a call back from my nurse. Doctor: Hello,Pain in the abdomen can be due to stones in the kidney or gall stones, or could be due to some other lesion in the organ. It can be confirmed by doing an ultrasound scan. If the pain is due to gall stones, it needs to be operated and removed surgically by a procedure called 'cholecystectomy.' If the pain is due to kidney stones, it can be treated by undergoing a procedure called lithotripsy. If it is due to some other organ, then it needs to be treated accordingly. Kindly consult a surgeon with the report for clinical evaluation and appropriate management.Hope I have answered your query. Let me know if I can assist you further.Regards,Dr. Ashakiran S"
},
{
"id": 175324,
"tgt": "How to reduce temperature of 39.7in children?",
"src": "Patient: My daughter has a temperature of 39.7. I am assuming that this too high? I have given her calpol tablets. She seems to have the flu. Is there anything else I can do to bring down the temperature quickly, and should I be particularly worried about the high temperature? Doctor: See as you have said your daughter has temp of 39.7 it seems to be clear that it is due to some kind of infection. Further you have not mentioned the age of your daughter so it is very difficult for me to calculate the dosage of drugs. Anyways, you can give her syrup cefixime 100 mg or syrup cefadroxyl 250 mg 5 ml twice daily with syrup ibugesic-plus or syrup crocin 5 ml three times daily for high grade fever with syrup cetrizine or syrup pheniramine 5 ml twice daily for flu like symptoms. Apart from these medicines you have to do cold sponging over her forehead as it will help you to bring down her temperature. Don't worry, she will be fine soon."
},
{
"id": 131346,
"tgt": "What relieves the pain in the back of the leg?",
"src": "Patient: I was playing football in the backyard and my dog ran up behind me and ran into the back of my leg, there was a sharp pain straight away but eased down after about 2 minutes, I got up and limped to my bedroom and lied down on my bed and the pain came back again and now the pain is almost unbearable, I can't put pressure on my foot at all now. What can I do to stop the pain? Doctor: First, you should go to hospital to take rabies vaccine if dog bite you. You should do x- ray to exclude presence of dislocation or disc sublaxation. It mostly muscle spasm ,but you should exclude other diseases."
},
{
"id": 151053,
"tgt": "Dealing with Parkinson's, on medicines, has slow movement, constipation, imbalance. What could be the causes?",
"src": "Patient: my father in law(82) is suffering from parkinson since 1 1/2 year.For last three months he is taking SYNDOPA 110 AND ROPARK 0.25 Present dose is Syndopa 110 -----1tab----1tab----11/2 tab/day AND Ropark 0.25 --1tab----------1tab per day. At present his movement is very slow,unable to walk alone.heavy constipation ,disbalance.These problems has increased after taking the medicines.Please advice ugtly @ .( YYYY@YYYY ) CELL- 0000. Your suggestions are welcome. Doctor: Dear Mr Kishore Mullick, It seems that your father in law is exhibiting signs of disease progression. Unfortunately, Parkinson's disease is incurable and despite medical treatment the disease progresses. Therefore he needs to be under regular follow-up with the specialist who will assess whether there needs to be a dose increase in his current medicines or any change to the medicines he is taking. Constipation is a common and troubling side effect with Parkinson's disease. The disease itself causes the constipation and most of anti-parkinson's medicines aggravate it. Therefore, he needs to ensure adequate fibre intake in the diet, adequate fluid intake. Stool softners and laxatives will also help. Hope this helps."
},
{
"id": 157024,
"tgt": "Any suggestions for gout crystals on hands and feet in a 73 yr old female with open heart surgery 5 yrs ago and breast cancer?",
"src": "Patient: I have gout crystals on hands and feet. So they want me to take hydroxychloroQuine and colcrys not together and allopuron. But I have decided not to take either. I have had open heart surgery 5 years ago GABG had breast cancer in both breast one 6 years ago the other 4 years. My heart is inlarged. I take diretics and high blood pressure meds. semivastatin, I also have arthritis for the last 33 years, they gave me gold shots I was allergic. I realize I must do something, because of the liver and Kidneys. I am a female 73 years old. Any suggestions, besides changing my diet? Thank you very much in advance. Shirley====== My spelling is terrible forgive Doctor: If your gout crystals are not giving you any problem than you can avoid taking these drugs but if you had frequently painful episodes than uric acid sd be normal. Since you have not mentioned about any symptom I think diet modification and avoidance of sudden heavy alcohol and protein like red meat is ok."
},
{
"id": 178601,
"tgt": "What causes snoring sound from the chest of an infant?",
"src": "Patient: My daughter 6 months old, 7.5 kg weight.. Having sever cold and chest congestion with flem . We are hearing grrr sound from her chest. Also he breath is little fast in the abdomen. Our pediatrician has prescribed nebulization with Asthalin 1 ml + saline water 3 ml for every 4 hours for 3 days. Is Asthalin given for Asthma patients only?? Can i give to 6 months old infant? How long will it take to cure her? Is this signs of Asthma? Since my present place is little cold, I am planning to take her to my native which is 300 kms from my present place. Can i travel now with her? I am worried. Please help. Doctor: Hellowelcome to HCMI understand your concern. And you should not worry at all. Your pediatrician has given him correct medicine.1 asthalin is not only given to asthma patients only. Its given in other medical conditions also like in chest congestion in your daughter 's case.2 yes you can giver to 6 months old infant3. It will take between 4 to 5 days.4 no its not signs of asthma5 yes you can travel with her.If you can answer my queries it will be of great help.did she get any fever?please check her respiratory rate?If you have further questions kindly ask us again.wishing your child good health.with regardsDr Aamir"
},
{
"id": 28020,
"tgt": "What are the side effects of taking metoprolol succinate & diovan?",
"src": "Patient: I am taking metoprolol succinate 50mg once daily and 80mg diovan and 12.5 mg hct and have taken this combination for the last month. I also take metformin 500mg once in the morning along with 5mg glyburide in the morning. I feel really dizzy in the afternoons any ideas what rx changes should be looked at ? Doctor: Hi,You should check your pulse and blood pressure during the day and when you feel dizzy. If you notice that your blood pressure is decreased or you have low heart rate during dizziness, it can be caused by medications, and you will need to see your doctor to correct the doses.If there will be no difference of your heart rate and blood pressure, then you should look for another reason for your dizziness.Hope I could help youWishing you good healthIn case of further questions don't hesitate to askRegards,"
},
{
"id": 39841,
"tgt": "Does atypical mycobacterial infection with positive PCR test warrants complete anti tubercular treatment?",
"src": "Patient: Hello sir, can you please elaborate whether atypical mycobacterial infection of the endometrium documented by a positive PCR test, warrants complete anti tubercular treatment. I am an IVF specialist and have got positive results after doing so. Thanks. Doctor: HIThank for asking to HCMI really appreciate your concern, to rule out possibility of tubercular infection there are number of method or test can be done but important is clinical sing and symptoms if there is no concrete sing of tubercular infection and test are doubtful then the advantage of this goes to clinical sign and in this situation better not to initiate the anti TB drug clinical sign and symptom still remain the ideal way of diagnosis to the extent that some time on the basis of clinical evidence treatment can be started and if this improves the clinical condition then treatment of AKT continued, hope this information helps you, take care and have a nice day."
},
{
"id": 128904,
"tgt": "What causes severe fatigue post a hip replacement surgery?",
"src": "Patient: I am 68, female, had a hip replacement three weeks ago. Feeling great total mobility. Hip worn from uber athletic life. Struggled with anemia most of my life for unspecified reasons. Only once or twice has it been problematic. Did undergo infusion of some sort several years ago and it was a miracle! I saw my cardiologist for a routine check up today. I am suffering from extreme fatigue - not present last week - resting heart rate of 88 - normally in 70s. Very cold. Out of breath on stairs, carrying or lifting does not seem to impact me. Unusual amount of thirst and low blood pressure 103/60. Triglycerides high which cardiologist attributes to months of minimal exercise- waiting for hip otherwise cholesterol 170. Before I left the hospital, they said I was close to requiring a transfusion but they decided against it. Anemia? Diabetes? Any thoughts? This is the longest period two months of little or no physical activity. Super out of shape? Doctor: your symptoms could point towards hypothyroidism. the surgery could have put stress on your body which unmasked the effects of low thyroid hormone. this is made worse by your anaemia.i would order thyroxine and TSH levels to confirm the diagnosis and possibly an ultrasound of the thyroid to ensure no structural issues. i would also order calcium levels as the parathyroid hormones can alsk be affected.once the diagnosis is confirmed, you may have to be started on some medications to normalise the thyroid levels. you will feel good as new once the levels are adequate."
},
{
"id": 141924,
"tgt": "Suggest remedies for dizziness and nausea",
"src": "Patient: today i experienced dizziness x2. lasted about 30sec. with nausea. i am taking bp meds. it was high 190/102 i took my bp meds right away and an 81mg. ASA i was at work. i went home took bp meds hour later it was 172/68. laid down started to feel better and it happened again. lasted about 30sec, never happened before. scary. ive been taking the same bp med dose for about 3yrs. i did not experience any vision change or extremity numbness. i just feel a little weak. and a slight head ache. Doctor: Hello! My name is Dr. Aida and I am glad to attend you on Healthcaremagic! Your symptoms could be related to an inner ear disorder or a TIA (transient ischemic attack) in the vertebro-basilary arteries. For this reason, I would recommend consulting with an ENT specialist to investigate for an inner ear disorder. A Doppler ultrasound of the carotid and vertebral arteries is necessary to examine the blood flow to the brain. Meanwhile, I would recommend you to closely monitor your blood pressure values. Blood pressure fluctuations could lead to such clinical situation or could be a normal physiological reaction to the vertigo and nausea. Hope you will find this answer helpful! Kind regards! Dr. Aida Quka, Neurologist"
},
{
"id": 35272,
"tgt": "What causes typhoid to last more than a month?",
"src": "Patient: My grandmother is suffering from typhoid and its been a month or more(40 days). She has still suffering from fever 98-101 and it inceases in the evening. What can be the reason...her condition is better than before but how can she recover completely.?Thank for the suggestion in advance. Doctor: Thanks for posting you query to health care magic. Typhoid is a bacterial illness ,we acquire it by contaminated food or water . These bacteria spread through blood to bone marrow, liver, gallblader ,lymph node and other organ .Hence complete cure require a course of antibiotic with other supportive measures. First you shoud undergo 1. widal test for blod. 2. Blood culture and sensitivity testing .3. Stol culture for salmonella . After these report you can take antibiotics as per sensitivity . Review me with these report so that I will suggest you complete reatment . hope you are satisfied with my answer . feel free to communicate if any query . regards, Dr.Manish Purohit Infectious disease specialist"
},
{
"id": 103662,
"tgt": "Asthmatic, suffering with thrush, weak kidneys, use pads, eat probiotic yoghurt. Suggestions?",
"src": "Patient: I am an asthmatic 66 years old am on Symbicort & Ventolin . In the past 20 years I have been suffering with Thrush. I have only just learned in past 8-10 years that I must rinse my mouth out after every puff. I have to use a pad all the time because of weak kidneys. It can get pretty annoying at times. I eat a lot of probiotic yoghurt which has acidophilus & it is also organic. Doctor: YOUR ASTHMA SEEM TO BE FROM FOODS AS SUGESTED BY HISTORYAND MOSTLY MILK IS CULPRIT AS IN YOUR TYPE OF DISESE ADS MILK HAS ANIMAL PROTEINS AND THEY ARE NON COMPETIBLE WITH HUMAN POTEINS THEY CAUSE MANY DISEASES INCLUDING ASTHMAWITHDRAW MILK AND DIARY PROTEINS COMPLETELY AND IT WILL REVERSE IN 3 WKTO 3 MONTHSGET ALSO BLOOD SERUM TEST FOR SPECIFIC ANTIBODIES FOR OTHER COMMON FOOD LIKE WHEAT POTATO CHANA RICE NUTS EGG OR ANY OTHER YOU THINK CAN BEOU WILL HAVE MAGIC EFFECT AFTR WITHDRAWL OF FOODS I AMSURE"
},
{
"id": 183857,
"tgt": "What causes tooth ache post a dental filling?",
"src": "Patient: Hi,I've been having a tootache for (approximatley) a week following a filling I hid. At first the pain was in my tooth and spread to around 3 other teeth to the left. However, the pain soon reached the gums like a throbbing pain or heart beat when I'm lying down and is getting really painful and unbearable at times. I can now feel a small bump on my upper lip near the teeth. What could it be? (I'm seeing my dentist today after I though I could deal with the pain)Thanks, Doctor: Thanks for your query, i have gone through your query. The possible cause of pian is becuse of the tooth infection. The type of pain what you are getting is charecteristic feature of pulpitis.The lump could be because of the abscess formation. The normal filling will not solve the issue. The tooth has to be treated with RCT or extraction.Take a radiograph to ruleout the infection. You can take a course of antibiotics like penicillin and metronidazole combination after consulting the physician.I hope my answer will help you. Take care"
},
{
"id": 137852,
"tgt": "Suggest treatment for severe knee pain",
"src": "Patient: I think I dislocated my knee 5 days ago it was very painful it felt like my bone popped out and then back in within seconds..since then I have been resting and keeping it up wrapped in a bandage......my foot was swollen it went down just yesterday its only a bit swollen now.... I am able to put my knee straight with minimum pain but I can not bend it to a certain extetent without feeling pain this pain hasn t yet ton away.....but I ve ruled out broken because I can walk and put preassure without very much pain.... But bending hurts on a scale of 10 a 7 Doctor: dislocation is emergency\u3002I hope there might have some patellar fracture Or some other knee joint injury like tearing of ligaments Or tendons.need to habve investigations like xray .Or may need CT..and so on.so need to seek hospital"
},
{
"id": 176156,
"tgt": "What causes painful urination?",
"src": "Patient: Hi. My granddaughter is complaining who is 3 when she urinates it hurts. There is redness in her private area. I haven t seen it yet but my daughter said there is like an open sore. what can be used? She is potty trained but sometimes has accidents. Unsure if this is the cause not being wiped properly. Doctor: Hello. I just read through your question.Any open sore can be treated with an over-the-counter antibiotic ointment. However, anytime there is a complaint of painful urination, the urine should be testes to rule out a urinary tract infection. I recommend consulting with your doctor so that this may be done."
},
{
"id": 219962,
"tgt": "What are the chances of pregnancy while treating PCOS?",
"src": "Patient: Hi there Ive been on contraceptive pills for three years because of my PCOS now I just stopped it on oct 2015 and so far im having a regular menstruation so I assume my pcos has gone now. Now me and my bf had contact on 30th dec then he cum inside me. Im just so afraid because I dont want to get pregnant yet. So what I did is I took contraceptive pills right away.. I just want to work will there be a chances that I wont get pregnant? if I continue taking pills? right now I dont have any plans yet and im so deperate thats why I took pills right away after 2 hours of contact. pls advise thank you.. Doctor: HiDr. Purushottam welcomes you to HCM virtual clinic!Thanks for consulting at my virtual clinic. I have carefully gone through your query, and I think I have understood your concern. I will try to address your medical concerns and would suggest you the best of the available treatment options.With your history it appears that you were not on birth control pills and you had unprotected sex. It always involves risk of pregnancy.Usually I suggest to take PLAN B or i pill within 24 to 72 hours of unprotected sex act. This usually helps to prevent pregnancy.Just starting contraceptive pill may not have pregnancy prevention effect as that of PLAN B pills.You need to take 5 low dose pills at a time - 2 times with interval of 12 hours,to equal the action of PLAN B. This leads to lot of stomach problems. and starting single pill daily will not be of use. In view of your irregular cycles, I will suggest you to get urine pregnancy test after 3 weeks of that sex act. Even if it is positive ,you can opt for termination with pill safely till 9 weeks of pregnancy.I hope my answer helps you.Thanks.Wish you great health.Dr Purushottam S Neurgaonkar"
},
{
"id": 55224,
"tgt": "Is diagnosis of abnormal liver for symptoms of numb fingers and arms, pain in hips, arms and back correct?",
"src": "Patient: Hi..I m 39 years of age and over the last month have experienced numbness in fingers, progressing weeks after to dead arm feelings. Since six days ago, symptoms have got worse, constant pain in hips,back arms and legs. I had a blood check and it came back abnormal, after seeing doctor today he said it was an abnormal liver 160 four times worse than average whatever that means and also to stop taking my duloxetine for depression...what is the abnormality in. Y liver. Mum passed away with Fibrolamella two years ago. Thank you Doctor: Hello! Thanks for putting your query in HCM. I am a Gastroenterologist. Elevation in liver enzymes has many cause like Hepatitis B, C, Metabolic disorders, NAFLD, alcohol intake etc. So I will suggest you to visit a Hepatologist for evaluation who will advise tests and then manage accordingly. Meanwhile there are few medicines which I found very useful in my patients are ursodeoxycholic acid and vitamin E. Do regular exercise and reduce your weight if you are overweight, avoid alcohol if you drink.I hope I have answered your query and this will help you . Wish you a good health"
},
{
"id": 10036,
"tgt": "Should i continue with these medicines for hair problem?",
"src": "Patient: Hello Sir, I am 24 years old and I am having too much Hair Problems. So i consulted a doctor and he gave me Xpecia and weltin tablet. Also told me to apply MinScalp gel twice a day. Can you please let me know if there is any side effects with these medicines or is it good to go with these medicines. Doctor: Hi, You seem to suffer from androgenetic alopecia... also called male pattern baldness. Your doctor has given proper medicines. They are finasteride, biotin and minoxidil. All these are very effective and harmless. And they may be used for long time. The result may be there after few years continuous treatment. Hope I have answered your query. Let me know if I can assist you further. Take care. Regards, Dr Ilyas Patel MD, Dermatologist"
},
{
"id": 153848,
"tgt": "What causes elevation of LFT parameters and total Bilirubin while under chemo?",
"src": "Patient: sir, my patient is periampullary adenocarcinoma with locally advanced stage, given gencitabin 1000 mg and oxaplatin 100 mg 4 days back, now total bilirubin is 2.2 in comparison to 1.5 before application of chemo, alkaline phosphate comes down to 612 from 685, other LFT parameters elevated moderately, kindly comment on state of health of my patient, Doctor: Hi, dearI have gone through your question. I can understand your concern.Your patients has periampullary adenocarcinoma. He is taking gemcitabin and oxaplatin. Oxaplatin can cause transient elevations of liver enzymes. You should keep close watch on liver enzymes. If level increase further then it can be due to oxaplatin toxicity and you need to stop oxaplatin.Hope I have answered your question, if you have any doubts then contact me at bit.ly/Drsanghvihardik, I will be happy to answer you.Thanks for using health care magic.Wish you a very good health."
},
{
"id": 23553,
"tgt": "Can i take 'covance-D' for pain in limbs?",
"src": "Patient: I am 50 years old.height.5.2 weight.90.I am suffering from severe swelling of hands, legs and ankles. I feel dizziness some times and loss of breath also. I do a lot of physical work around the house and feel tired afterwards. I have been taking Covance D tablet when I feel pain in my limbs. I have seen no side effects so far and am feeling good after I take it. Please advice if it is correct. Doctor: Hey,Tab Covance D is not for pain in the limbs, it is a blood pressure medication. So if you a hypertensive than continue taking it otherwise please check your bp from time to time.Tab Covance D is a very good medication for BP contains combination of 2 medication (Losartan & hydrochlorothiazide). The hydrochlorothiazide part can decrease your Sodium especially at old age. So regularly check your sodium & potassium. Wish you a good healthDr Sameer MaheshwariCardiologist, Fortis hospital"
},
{
"id": 152003,
"tgt": "My BP recently dropped whether active or not. what I should do ?",
"src": "Patient: I have been diagnosed with a 4mm cerebellar ectopia, hypertonicity in my left strap muscle, a basilar vein pushing on my 12th cranial nerve and recently got a blood test back for pyruvic acid which was slightly elevated. my BP is usually 116/79 PR 70-79 it has recently dropped whether active or not to 90/55 and PR 60-65. do u have any suggestions as to what i should do ? i m on no medication as of the moment awaiting a call back from a neurosurgeon . Doctor: Thanks for the query You dont have to worry about the BP until the systolic is 90. But what worries me is the cerebellar ectopia, as u have rightly suggested, meet the nerurosurgeon. Pulse rate is within normal limit Slight diurnal variation and positional variation of BP will be there, dont worry about it Have a healthy life"
},
{
"id": 91297,
"tgt": "Could bloated stomach, frequent headaches and vomiting be related?",
"src": "Patient: Hello,I have frequent headaches on forhead and back and when severe, it is on the face towards the eyes. When teh headache is severe, I need to drink water to vomit. This gives me relief. This is also accompanied at times with bloating of stomach.Krishna Doctor: Hi Krishna,Thannks for writing to HCM.You seem to be having drug induces gastritis with either sinusitis or migraine.The painkillers you might had taken forvthe headache has caused gastritis.If you are having cold also, then the headache is mostly due to sinusitis, else its mostly migraine.If its sinusitis, then you need to take steam inhalation 3-4 times daily to clear them.If its migrane, then you need to see a physician for its confirmation and further management.For stomach bloating, you need to take a ppi like omeprazole or pantaprazole for a few weeks.RegardsDr. Ashish Verma"
},
{
"id": 220512,
"tgt": "Had protected sex. Got heavy periods. Is there any chances of getting pregnant?",
"src": "Patient: I had protected sex with my boyfriend about a month or so ago on my infertile day, the same month i got my period on time it was very normal, heavy and it lasted the usual amount of days(5). The condom was perfect and he didnt ejaculated. But i heard that even after a heavy and normal period a girl can still be pregnant? can i be pregnant? Doctor: hello,.You have protected sex on infertile days. So there is no chance of pregnancy.Don't be much worried.Also you got normal period. Practice always safe sex in future.Heavy period in one cycle is not significant until it happens in every cycle. It may be due to hormonal imbalance or uterine pathology.So if you got heavy period in every month,then do a USG scan for pelvis and consult with gynecologist.Be well."
},
{
"id": 120589,
"tgt": "What causes sore and swelling joints?",
"src": "Patient: sore joints/swelling 2 yrs ago my left ankle was sore and I couldnt walk on it then it swelled up and I went to the dr. and they sent me for all kinds of tests and couldnt find anything wrong. after about 7 mos the swelling went down and all is normal again amd now my left elbow is sore to the touch and has been for 5 weeks and it is starting to swell... I am concerned that the dr cant find what is causing this can you help me out Doctor: Hello,I read carefully your query and understand your concern. Swelling\u00a0of the joints occurs when fluid accumulates in these tissues.\u00a0Pain, stiffness, or both may accompany\u00a0joint swelling. Joint swelling\u00a0can be a symptom of a chronic condition, such as\u00a0arthritis.I suggest to do a rheumatoid factor level for further evaluation.Meanwhile,I suggest using anti inflammatory medications such as Acetaminophen to relieve the inflammation. I also suggest using cold compresses for local application and get evaluated by a rheumatologist. Hope my answer was helpful.If you have further queries feel free to contact me again.Kind regards! Dr.Dorina Gurabardhi General &Family Physician"
},
{
"id": 175851,
"tgt": "What do bloating and rotten burps in a child after being treated for illness mean?",
"src": "Patient: my daughter is 4 years old and shes been ill up and down for the past 12 weeks finally getting her self back to normal. I noticed the day of finished her medication that her stomach was quite big and in my words looked swollen. when she was ill she would complain of head aches and stomach aches but thought nothing of it. shes pooing at least 2-3 times daily but shes got rotten burps that make me gag. Doctor: Hi...Thank you for consulting in Health Care magic.It seems your kid is having viral diarrhoea. Once it starts it will take 5-7 days to completely get better. Unless the kid's having low urine output or very dull or excessively sleepy or blood in motion or green bilious vomiting...you need not worry. There is no need to use antibiotics unless there is blood in the motion. Antibiotics might worsen if unnecessarily used causing antibiotic associated diarrhoea.I suggest you use zinc supplements (Z&D drops 1ml once daily for 14 days) & ORS (Each small packet mixed in 200ml of potable water and keep giving sip by sip) as hydration is very important and crucial part of treatment. If there is vomiting you can use Syrup Ondansetron as prescribed by your paediatrician.Hope my answer was helpful for you. I am happy to help any time. Further clarifications and consultations on Health care magic are welcome. If you do not have any clarifications, you can close the discussion and rate the answer. Wish your kid good health.Dr. Sumanth MBBS., DCH., DNB (Paed).,"
},
{
"id": 45519,
"tgt": "What are the causes of renal cortical thinning?",
"src": "Patient: Hi. I was diagnosed with renal cortical thinning. How do you get this? How do you treat this? I am waiting on a Drs appt with nephrologist so I have not seen one yet. I had what felt like a kidney attack but results came back no kidney, no infection but thinning. Doctor: Hello,In hydronephrosis, interstitial nephritis or pyelonephritis, there is a possibility of renal cortical thinning. Please consult with your nephrologist he will examine and treat you accordingly.Take care. Hope I have answered your question. Let me know if I can assist you further. Regards, Dr. Penchila Prasad Kandikattu, Internal Medicine Specialist"
},
{
"id": 43090,
"tgt": "Trying to have baby but sperm analysis shows dead sperm. How can it be rectified?",
"src": "Patient: Hi, I'm 33 years old and married to a 30 year old woman. We have been trying to conceive for the past 4 years without any success. She has 2 kids from a past relationship. I once impregnated a girl in the past but sadly lost the child at birth. I recently went for sperm analysis and was told that my sperm was dead and only 3 percent was alive. We are desperate to conceive and are not getting any younger. How can we improve this? What remedies can you recommend? Doctor: Hello and welcome to HCM,Presence of only 3 % alive sperms is insufficient for conception.However, since you have mentioned that you were able to impregnate a woman in past, a repeat semen analysis is required.Get the repeat semen analysis after atleast 3 days of abstinence.Sometimes presence of factors like infection of the genito-urinary tract will lead to false results.Post the results of your semen analysis for further opinion.Thanks and take careDr Shailja P Wahal"
},
{
"id": 193118,
"tgt": "How can a bulge between the rectum and testicles be treated?",
"src": "Patient: Hello, I have bulge in between my rectum and testicles.I also notice when i do a kegel squeeze it bulges out more. When i used to do a kegel I would feel it in my penis. Also I never get morning erections or erections unless i force myself and even then it is very weak. I notice that the base of the penis is never hard. Doctor: Hello,Morning erections are never a necessity. Erection comes when you are sexual excited. if you are sexually excited in the mornings, you will get erections. so do not worry about that issues. Regarding the bulge between the rectum and testicles, you should consult a surgeon for physical investigation.Hope I have answered your question. Let me know if I can assist you further. Regards, Dr. K. V. Anand, Psychologist"
},
{
"id": 53042,
"tgt": "What does SGOT level of 57 in LFT suggest?",
"src": "Patient: good morning doctor rao,i am raju 54 years old having diabetes and high bp for last 13years.i am under weight with 45kgs.from my child hood.i am using the following medication for the above as per the doctors advice.1. tozar 50 (losartan potasium)one TAB DAILY2.glycyphase(metaformin)1 TAB DAILY3.VIZYLAC(MULTIVITAMIN)I AM A SMOKER 5 TO 10CIG ADAY2 I TAKE ALCHOHOL 180 ML DAILY.DURING MY RECENT HEALTH CHECKUP,THE RESULTS OF LFT AS FOLLOWS:S.TOTAL PROTEIN 6.9 G/DLS.ALBUMIN 4.1S.GLOBULIN 2.8S A/G RATIO 1.5SGOT 57SGPT 67 ALKALINE PHOSPATASE 235SGGTP 340TOTAL BILIRUBIN 0.9DIRECT BILIRUBIN 0.49MCV 103.5 FLOTHER LIPID PROFILE TEST RESULTS ARE NORMAL.KINDLY ADVICE AND EXPLAIN STATUS OF MY LEVER PROBLEM AND TREATMENT FOR WHICH I SHALL BE GREATFUL TO YOU.WITH REGARDS, RAJU Doctor: Hello, I hope you are well, even though you are not overweight or obese, you have metabolic risk factors such as diabetes mellitus and long-term hypertension, factors that can condition a fatty liver, in addition to the consumption of alcohol that May be contributing to your problem. You present a very slight enzyme elevation, less than double for your gender, however, you need to perform an image study as a liver ultrasound to study the characteristics of the organ. There are several types of noninvasive studies to determine the degree of steatosis / fibrosis that you present for the purpose of performing an early detection and implementing actions for the prevention of disease progression."
},
{
"id": 59792,
"tgt": "Ultrasound showed fat infiltration. Alright to drink occasionally after liver enzymes become normal?",
"src": "Patient: I am a 53 year old female, weight 146-height 5 4 . I recently had an ultra sound because of elevated liver enzymes . There was some fat infaltration. One month after the ultra sound my liver enzymes were AST-29, ALT-57. I was told to continue to abstain from drinking alcohol for a total of three months and have my enzymes checked again. I had been drinking 3 to 4 glasses of wine a night for many years because I have high cholesterol and thought this would help with the cholesterol. If my liver enzymes return to a normal range is it all right to have an occasional glass of wine? Doctor: Hello madam, I have gone through your problem. You said that you are drinking wine for many years and had fatty infiltration of the liver along with raised liver enzymes. Fatty infiltration of the liver is seen in obese patients and patients who take alcohol. You also had raised liver enzymes which shows that you had alcoholic hepatitis. Now your liver enzymes had returned to normal, but still there is a chance for you to have hepatitis (raised liver enzymes) in future if you take alcohol. My advice is to abstain from drinking alcohol and consult your physician for your high cholesterol. There are cholesterol lowering drugs with which you can maintain your body cholesterol at a low level. good luck to you."
},
{
"id": 23341,
"tgt": "Is playing rugby safe after a heart attack?",
"src": "Patient: I'm 30yrs old and 6 weeks ago i had a m.i. heart attack. I have had one stent fitted. Before the heart attack i was considering joining a rugby team... I was wondering if, after a year and i felt alright, would i be able to start rugby or will i have to give the idea up permanently? Doctor: hi dear post myocardial infarction the heart takes almost 12 weeks to settle downrepeat a 2d echo after 12weeks and a treadmill test if every thing is fine in these tests then you can do whatever a normal person can do you can play rugby but remember one thi g do ot over strain yourself ,donot over strain your heart ,just increase your capacity graduallybefore starting to play rugby do brisk waking fir 45minutes a day fir atleast 15days !!thanks"
},
{
"id": 16673,
"tgt": "Suggest treatment for high blood pressure",
"src": "Patient: Hi, I m a 25 years old male, Day before yesterday I was full of sweat and felt vertigo and even was unable to move. next day when get checked my Blood pressure was 140/90. I drink twice in a week. so can you suggest what precautions should i take. Please suggest some medicines as well. Regards A.K Doctor: Hello, I don't know about your body weight, height, and duration you have been having high pressure. In order to rule out any disease, I need more information from you. However, as u want to know only the preventive measures then I would suggest you to cut down drinking (i.e. u can consume 20 g alcohol once a week if your pressure is not elevated), eat more vegetables and fruits, avoid canned, fat-rich foods, and salty foods. walk every day for at least 30 mins. drink at least 3 liters of water per day. Sleep at least eight hours per night. I prescribe medicine if first-line preventive measures are failed i.e , to say if you follow the preventive measures for six months, which i have mentioned above, then I would suggest you take medicine. Moreover, In your case (young blood) I would not prescribe medicine without doing clinical investigations. Hope I have answered your query. Let me know if I can assist you further. Take care Regards, Dr Sandeep Kumar Karn, Internist / Hospitalist"
},
{
"id": 147241,
"tgt": "Can vertigo be caused due to falling on head?",
"src": "Patient: Hi I fell realy hard yesterday when I slipped on some ice .I have a big bump on the back of my head and a sore neck Last night when I was moving from one side to another in my bed it felt like the room started spinning very fast for few seconds. Is it possible that the vertigo is because og the fall ? Doctor: dear sir/ma,am hi injury to head/neck due to fall can very well cause these symptoms . it can cause headache vertigo uneasyness / vomitings etc. get yourself investigated with CT SCAN brain / xrays cervical spine to rule out any injurythanx dr praveen k nathdirector , deptt of neurosurgery"
},
{
"id": 42070,
"tgt": "Can less liquor improve amenorrhea?",
"src": "Patient: hi good morning i am praveen from hassan,my wife is primi with 23 wks of ammenorrhea and she is having 9.56 liguor doctor say its not adequate for the gestation.they have given l-riaz sacchet for one month and said to come after one month.my q is what wil happen if liquor is less,how can v improve liquor,wil it cause any problem in future. Doctor: less liquor can cause premature birth of the baby and some foot deformities. it can be treated if deficiency is severe. by infusion in the amniotic sac. but your wife is having mild deficiency. so sachet are enough. go for routine usg to better monitoring. take care. do not forget to rate the answer. i wish you will have a baby soon."
},
{
"id": 198836,
"tgt": "What causes painful ejaculations?",
"src": "Patient: I have been experiencing painful ejaculations with a very small amount of ejaculate. It creates a uncomfortable pressjre in testicles and feels like something is obstructing the flow. I have had a vasectomy years ago and never had problem until recently. I take effexor xr daily. Doctor: Hello and .Your concern is understandable.At time of intercourse, sperms with some fluid, travel via tubes called vas, from both the testis to the penis, and out with ejaculation.In your case, both the vas have been divided and tied up years back.But, the kind of pain you're experiencing, is known in few cases.You should get a doctor to check the genitalia, and proper treatment will be advised. You should also ask the doctor, if the Effexor XR, is needed.Wish you well."
},
{
"id": 212637,
"tgt": "Having mental disbalance after an accident. What should be done?",
"src": "Patient: iam currently writing from Dhaka, Bangladesh. One of driver s brother met with an accident 5 years back. He was a normal person till then and he was also working as driver till he met with the accident. After the accident he started behaving abnormally and he used to fight and abuse everyone till last year. Last year he started behaving normally after some treatment and now the problem is he is forgetting what happened after every 30 min to 1hr. He some time remembers some things like driving, manual gear first gear etc., What should be done to make him normal. Since they are from poor family, i want to help them to be normal. Please let me know, if i have to send him to India to any good hospital for treatment. Doctor: Hello........ Thanks for your query. It is possible that post accident the person has developed organic personality changes. A CT/MRI would be advisable at this stage. Also assessment of cognition would be helpful for management. Low dose anti-psychotics would be the preferred medication in this case generally. Hope you found the information useful. Regards Dr Sundar Psychiatrist"
},
{
"id": 80730,
"tgt": "What causes productive cough with wheezing?",
"src": "Patient: dear sir i have been suffering with these S/S productive cough from 1.5month ,wheezing,lethargy,fever under 103F,backache,headachemy X Ray Chest was cle monthar but extra expended rib 1 to rib 9 and diaphragm border is normalCBC is that ,Hb.14.4.plts.255.TLC.6.6 and ESR.01sputum AFB smear Neg Doctor: Hello dear, thanks for your question on HCM. I can understand your situation and problem. In my opinion you are having mostly bronchitis. Since your chest x ray is normal then no need to worry much for lung infection. Bronchitis can cause similar symptoms like cough with expectorant and wheezing. So consult pulmonologist and get done PFT ( pulmonary function test ). PFT will also tell you about severity of the disease. And treatment of bronchitis is based on severity only. You may need inhaled bronchodilators and antihistamine drug. Don't worry, you will be alright."
},
{
"id": 20790,
"tgt": "What causes increased BP and pulse and cold sweats?",
"src": "Patient: my blood pressure is 160 over 101 and pulse rate is 90. Head feels kinda tight and have had several cold sweats in the past couple of days. Chest hurt a couple of days ago and had a cold sweat. Is this dangerous? I am 56 5 10 and weight is 160. I take Alprazolam 4 times a day. Doctor: Hello!Welcome and thank you for asking on HCM!Regarding your concern, I would explain that your actual blood pressure values are above the normal ranges. The cold sweats could be related to a possible viral infection. Have you measured your body temperature? Do you have fevers?Coming to this point, I would recommend consulting with your attending physician for a careful physical exam and some tests: - a chest X Ray study- a resting ECG- complete blood count, PCR, ESR for inflammation- thyroid hormone levels for thyroid dysfunction- kidney and liver function tests- fasting glucose- urine analysis for infection. You should discuss with your doctor on the above issues. Meanwhile, I recommend you to closely monitor your blood pressure values and refer them to your doctor. Hope to have been helpful!Kind regards, Dr. Iliri"
},
{
"id": 59121,
"tgt": "Removed gallbladder. Taking Pankreoflat. Stomach discomfort. How is my health?",
"src": "Patient: Thank you doctor. Good Day. I am located in India - Kolkata.My gallbladder was removed 39 days ago by laparoscopy. I am applying medicine to superficial wounds and they appear fine. Presently I am taking a composition of Sodium Alginate, Magnesium Hydroxide, Aluminium Hydroxide, Simethicone thrice daily. I have also started with Pankreoflat tablets(pancreatin and activated dimethicone tabs)twice daily after meals because I had been generating lots of hydrogen sulphide like gas in my stomach after lunch. After taking the medicine farting is improved; but uneasiness in stomach still persists for 2 to 3 hrs. after lunch.Prior operation all tests and whole abdomen USG was done and excepting gallbladder stone everything was okay. I am also taking 2 tsp Lactulose solution at night for last 2 days. Would you kindly tell me how well am I. ( My age: 66.5 years).Thanks again and Regards to you. Anup Kumar Roy. YYYY@YYYY Doctor: Hi and welocme to HCM. You are well.There is no need to be concerned. these problems are common after cholecystectomy. Even more serious symptoms can occur and then it is called postcholecystectomy syndrome. Your body needs some time to adjust on surgery, sometimes 2-3 months. You just need to eat easier food,avoid alcohol and fats,and regulate your stool, you can continue with these medicines, but try to make it all easier by changing lifestyle habbits and dietary. If you notice fever and pain under the rght ribs you should visit doctor to do further tests."
},
{
"id": 51802,
"tgt": "Stone detected in both the kidney",
"src": "Patient: stone detected in my mothers kidney(both) Doctor: Welcome to HCM. Treatment always depends upon the size and location of the stone. Hydrolithotripsy,pcnl, and endoscopic removal of stones. Better you consult your urologist for further guidance."
},
{
"id": 168378,
"tgt": "What causes red spots on baby s body?",
"src": "Patient: Hi I have a newborn whose skin was clear and smooth however he began developing tiny red spots on his cheeks, which then spread to his neck, under his ears and now his torso. I m wondering if it is a heat rash or sensitivity to the laundry detergent? Doctor: Hello. I just read through your question. What you describe is consistent with baby acne. This is harmless. Just keep it clean with plain water and it will eventually resolve."
},
{
"id": 36291,
"tgt": "What does tongue & vaginal ulcers & blisters with cold suggest?",
"src": "Patient: Hello. I have had tongue ulcers for over 1 year and 8 months now. They are continuous and located on the edges of my tongue. They range from pin prick size to dime size. The big ones leave deep holes in my tongue once they heal. I also get vaginal ulcers to the point they scarred the area and have eaten away my urethra. My eyes swell and get bloody sometimes. And about 1 year after those symptoms appeared I started to get large painful bumps under my skin that created bruises. These bruise bumps leave a divot after they heal. At the same time as this I started to get blister sores almost pimple like on my skin to the point that if they gather in patches the skin is raw to and purple red. These leave permanent purple red marks when healed. My elbows also started the last 4 months to get hot and hurt a lot. All of these symptoms get worse in the cold months. Odd but I have not actually gotten a cold or flue since this began. I have never been sexually active. Please help me figure out what I have. Doctor: Good day and thanks for your question. From what you said, it usually get worse during cold seasons. You could be having a moderate - severe form of Cold Urticaria. Kindly see your dermatologist and know your HIV status (its not only through sex you can contact HIV).Hope it helps. Thank you."
},
{
"id": 59931,
"tgt": "Elevated serum ALT levels. Bloated stomach, enlarged liver. Is this curable? Is this due to drinking and smoking?",
"src": "Patient: Hi - my serium ALT has been increased to 59 - also my stomach size has been increased - I dont feel any pain but gas create lots of discomfort please let me know if it is cureable? My doctor has physically examined me and told that size of my liver is increased and asked me to get few other blood and stool tests - is it something serious? I am a heavy smoker and social drinker, a month ago I took 1/3 of viagra twice in 5 days. I am 41 years old and actually don,t need viagra but for 2nd intercourse in a row I had used it. DO YOU THINK THIS HAS CREATED THIS PROBLEM? Thanks for your advise in advance. YYYY@YYYY Doctor: Hi. Your raised Serum ALT cannot be caused by 1 or two doses of viagra. You need to quit drinking and loose weight if you are overweight. You liver is definatley inflammed as suggestive by raised ALT but if you avoid alcohol and take the medications prescribed you should be fine. As far as the gas is concerned you can take proton pump inhbitiors for 2-3 months which would help you. I would be happy to review the reports for you can can send them over. Hope this helps Regards, Dr. Om Lakhani"
},
{
"id": 27635,
"tgt": "Suggest treatment to control blood pressure",
"src": "Patient: i normally have a blood pressure reading of systole numbers in the range of 132- 138 And a dystole number in the 90 Right now it s 108/69 I also am type II diabetic ... I don t feel very well very nauseated and light headed Is this blood pressure reading a good one? Doctor: Hello. Thanks for writing to HCM.I read and understand your question very well.I try my best to solve it.Your recent Blood Pressure is 108/69 which is absolutely normal. You have diabetes. I think this is temporary and this turns to normal. I suggest to do daily meditation for 30 min , also walk 30 min daily.I tell you not to take any stress or tension .Do not think about too much on BP, divert your thought. Do activity you like. Do things you enjoy.I also suggest to check your eyes to ophthalmologist, headache is due to eye problem also.If problem continue than consult a good physician for this.I hope my advice will help you a lot.Thanks and Regards.Dr Vijay"
},
{
"id": 20519,
"tgt": "What causes fluctuations in BP?",
"src": "Patient: bp sys 144 dia 76 pul 62 is this a bad reading? should I be on medication. I gave up smoking 4 months ago and was prescribed Ramipril 5mg for high bp. I had an appointment this morning and was told my bp was low, but had to go to the doctors tonight and given the above reading. Dr has changed medication to Bendroflumethiazide 2.5mg. I am very confused ! Doctor: Hello,Thanks for writing to Health Care magic, I am Dr , Muhammad Ahmad , I have read your question closely, I understand your concern and will be helping you with your health related problem.You do not have to be anxious or worried about what is happening , your blood pressure reading of 144 by 72 was not that bad, you doctor have started you in diuretic which is a recommended drug for blood pressure control so no worries in that.But there are a few things which you are required to do on your part, this on one hand will control your blood pressure better and save you from any worst outcome and on the other hand it will also help your doctor to treat you well.You have to make some life style modifications specially by controlling your salt intake then you have to control fat intake as well.Reduce your weight if you are over weight, it's great that you have already quit smoking , just stick to this decision. Stay physically active and eat health friendly food and your blood pressure will stay in normal limits.The thing which will help your doctor will be your making a blood pressure chart, I would advise you to make a blood pressure chart by measuring your blood pressure twice a day and writing it down with date, when next time you will visit your doctor he will get to know about your blood pressure control.That is very important to take further decisions about your treatment.I hope this answered your question, If you have more queries I am happy to answer you.Regards.Dr.Muhammad Ahmad"
},
{
"id": 138550,
"tgt": "Suggest treatment for swollen ankle and calf because of a fall",
"src": "Patient: I fell on uneven sidewalk on July 3rd and hurt my knee: I had no problem walking and I didn t have much pain just bruising on my knee with tenderness. Now as the bruising is entering the purple/green phase I am starting to get a lot of swelling in my calf and ankle- what should I do ?? Doctor: HiWelcome to healthcaremagicI have gone through your query and understand your concern. Your swelling of foot and calf may be related to edema after injury. It is useful to keep limb elevated. Active toe movement will be useful to reduce swelling. Analgesic such as ibuprofen is useful to get relief from pain. You can discuss with your doctor about it. Hope your query get answered. If you have any clarification then don't hesitate to write to us. I will be happy to help you.Wishing you a good health.Take care."
},
{
"id": 132187,
"tgt": "Can new mattress cause stomach pain and back pain?",
"src": "Patient: I have been having stomach pain for about a week and half. Feels alot like muscle pain like working out but I don t work out. I got a new mattress a little over a month ago and have been having aching back ever since wondered if mattress could be causeing stomach pain also Doctor: Hi Stomach pain is different from back pain. Change of mattress may give rise to back pain but is unlikely to give rise to stomach pain, Try taking pain killers for a few days and if there is no improvement , then go for ultrasound of the whole abdomen."
},
{
"id": 128243,
"tgt": "Is Gabapentin effective for neck pain?",
"src": "Patient: I have severe nerve pain in my neck on the right side going down to my shoulder. I am working with a physical therapist. I also have hydrocordone from a previous surgery. Am I able to take gabapentin along with 5 mg of hydrocodone? Or will the Gabapentin work on the neck nerve pain alone? Doctor: Dear patient Gabapentin is prescribed for radicular pain due to nerve compression. It relieves neurological pain with tingling and numbness. For neck pain you should take tab ultracet twice a day with gabapentin. Hydrocodone is steroidal drug helps to relieve chemical neuritis and can be taken for such pains."
},
{
"id": 20576,
"tgt": "What causes rapid heart rate for few minutes?",
"src": "Patient: I was laying in bed and my heart whent from normal to Extremely fast like i was about to have a heart attack and then it stopped for almost a full minute and then started normally again what happened my friends told me it could be a heart murmer but it really scared me could you tell me what happened? Doctor: HiIncreased heart rate & palpitations can have a lot of causes.It can be normal sinus tachycardia due to anxiety or stress or it could be abnormal supra ventricular tachycardia. For the diagnosis, you need to do ECG at the time of palpitation & tachycardia. If the duration of palpitations are short enough to reach a nearby hospital, we put a 24hr ECG monitoring (Holter Monitor). You also need to do a 2D Echo & thyroid function test to see for any associated abnormality if any.Thanks"
},
{
"id": 2050,
"tgt": "How to conceive?",
"src": "Patient: askingi have been married for 1 year and 4 months i have trying to get pregnant only for 6 monthes as before i was using condoms im asking shall i go to doctor shall my husband go and should i stay after sex in bed and altjougj staying sperms fall Doctor: Hallow Dear, How long you should wait giving the fair trial for natural conception depends upon the age of the couple, which you have not mentioned. Though you are married for 1 year and 4 months, you are sincerely trying for pregnancy for last 6 months only. If you are above 25 years of age, it is the time now that you start investigating both of you. You may not carry all the investigations straight a way. Initially try to find evidence of your egg release. Though in the market home kits are available, I would advise you initially to get the evidence by ultrasonography. This will ensure whether you are ovulating or not; and if you are ovulating, it will inform you the day of your ovulation (egg release). Egg has a life of 24 hours while the sperms are active for 24 hours. Hence, a period of about one week around the day of ovulation is fertile window. Then get husband's semen examination done for the quality and quantity of the sperms. If both these investigation reports are within normal range, try to plan your intercourse at and around the day of ovulation. Lying on your back for about an hour after the intercourse is advisable. After intercourse, some amount of semen does flow out passively, it is hardly 10% to 20% of the total semen; so not an issue. To minimise this flow of semen, the penis should not be withdrawn while it is hard. Let it slip out of vagina passively when it is placid. If you fail to conceive even after six months fair trial like this, get Hysterosalpingography and Laparoscopy done to find out the condition and patency of the tubes and uterus. However, if you are above 30 years of your age, get both of you investigated completely without delay. Report then to your Infertility consultant for further advice. You may ask me a Direct question on this forum with all these reports uploaded. Dr. Nishikant Shrotri"
},
{
"id": 140273,
"tgt": "Suggest treatment for partial focal seizures",
"src": "Patient: I have a history of partial focal seizures since I was 8. I am now 67, and just had my first grand mal about 1 month ago. I was taking Sudafed 5 he energy and mountain dew before the seizure. I have since stopped all Sudafed, all 5 he, and mountain dew, I have not had any partial focal nor grand meals since the last one over a month now. All reg, MRI, cat scans and blood work are normal. Is there anything else I need to do? I went to go, they said come back in 3 months, but no driving for 6 months and no need for referral to neurologist, unless I want lot. What would a Neuro do for me? Not excited about meds. Am I being too lacadazical about this? Doctor: Hello, It is very possible that the trigger for your seizure was an ingredient in one of the items you mentioned (ephedrine from the Sudafed, excessive caffeine from the 5 hr. energy drink, and of course, nobody really knows what's in Mountain Dew or why it glows in the dark! Therefore, I would avoid those substances in the future. It doesn't sound as if your seizure is under that good of a control and therefore, I highly recommend you see a neurologist and get on to medication to control these episodes. Otherwise, you are highly likely to continue to have them and they will become more frequent and more lengthy in their duration, and if nothing is done about them more difficult to control or get rid of. I absolutely recommend seeing a neurologist or even epileptologist if one is available in your area. You may benefit from a good detailed workup to see if a cause can be found and addressed. Again, THAT is what a neurologist could do for you. Look for things that could be triggering these seizures and making sure you are fully protected. I agree with the no driving for yours and everyone else's safety on and off the road. You SHOULD KEEP A SEIZURE DIARY which the doctor should check on your next visit. A good neurologist will insist on a diary and check it over with you to discover things of importance or events that have happened. Hope I have answered your query. Let me know if I can assist you further. Take care Regards, Dr Dariush Saghafi, Neurologist"
},
{
"id": 5080,
"tgt": "Trying to conceive, suffering from mild PCOS. Prescribed fertab. How should this medicine be taken?",
"src": "Patient: Hi, I will be 30 next month and have been trying to get pregnant since January 2012. I was diagnosed with mild PCOS. I get my periods regularly every 40-43 days (duration between first day of my periods). Sometimes I get it in 33-35 days, but that may be once in 7-8 months.My doctor advised me Fertab. Could you shed more light on this prescription and how it can be taken?Thank you. Doctor: Hello, Thanks for the query to H.C.M. Forum. Fertab is clomiphene citrate 25-100 mg . Dose schedule is as following,Starting on the 5th day of menstrual cycle for 5 days regular at same time. If ovulation does not occur: 2nd course100 mg for 5 days after a 30 days interval.Maximum three courses recommended . Hope you have understand . If further any question I will help. Good luck. Dr. HET"
},
{
"id": 82907,
"tgt": "What causing numb toes and burning sole in a non diabetic with RA, Lupus and fibromyaglia?",
"src": "Patient: my toes go numb sometimes and the bottom of my feet burn. I have been checked for sugar and do not have that, but I do have RA, Lupus, and Fibromyalga ?, restless legs. I don't know if this related to any of these conditions. I have been sick for about 20 years and have started treatment about 6 yrs. ago Doctor: Dear Madam,Rheumatoid arthritis can also lead to peripheral neuropathy , it may be the cause behind you burning sole and numb toes .Dr. Shruti"
},
{
"id": 16329,
"tgt": "Body rashes, condiferm-F, rashes went away, spread to another part, skin darkening, flucos 150 okay to take ?",
"src": "Patient: hi doctor i stay in gangtok . my wife is having a problem.she was having rashes on her body back portion so doctor here advised coniderm-F, after a week the rashes removed from that part but it again came in other parts.and some parts become black so we went to the doctor again and he now advised flucos 150and CLOP-GM WHAT TO DO? SHALL WE TAKE FLUCOS 150? Doctor: Yes mam u should continue the medicine. And dont expect the result in one or two days cause skin element takes long time and do stick to medicine time to time. Bye and take care."
},
{
"id": 55015,
"tgt": "What cause severe gal bladder cramps?",
"src": "Patient: I have been having severe gal bladder cramps. Doctor: Hi thanks for asking question.You have not mentioned history in detail and investigation done.The common condition that can lead cramp is gall stone.It could be single or multiple gall stone.I would like to advise you to perform USG and LFT for it.USG will detect stone if present.For it ursodeoxycholic acid can be taken.Second it could be by chlolecystitis like condition also.USG and CRP done for it.If it seems to be bacterial cause antibiotic taken.Take low fat diet.Avoid alcohol and smoking.Regular exercise done.Loose weight if overweight.I hope your concern is solved.Dr.Parth goswami"
},
{
"id": 113719,
"tgt": "Have Osteoporosis. Constant back pain. X-ray, MRI, CT scan done. How to get relief of backache?",
"src": "Patient: Hello Sir, I am Abdul Gafur. Since long time I facing problem with Back Pain . I have Osteoporsis. I am taking medicine to improve the Osteoporosis . It is improving now. But my back pain is not reducing. Once I sit on a Chair or if there is any media or support in my backside, then there is little pain. But once I walk or there is no support in backside, then the pain increase. I have consulted with Orthopedic . They have done X-ray , MRI , CT-Scan but no abnormality found. Please advice me what should I do to reduce my back pain. Everyday I used to walk 00:30 minutes. No improve. Doctor: Please upload MRI, X-Ray and CT for free review. Without scans we can't give reliable advice. Regards!"
},
{
"id": 166947,
"tgt": "How to make infant sleep?",
"src": "Patient: My 5 month old's sleeping patterns keep getting worse. This week he has not slept more than 45 minutes and wakes up crying. I've also had to put gloves on his hands because he scratches his face up. He has also been throwing up after most feedings. He is only on formula. Doctor: hi! months 5-6 can be a challenge to parents as the child gets more interactive with the environment. so he'll sleep lesser with short naps. this is called sleep regression and is normal in babies. although if he's throwing up after every feed you might want to see a doctor. try having absorbing play time with your baby during the day so she can sleep well at night."
},
{
"id": 4231,
"tgt": "Trying to conceive and had miscarriage last time",
"src": "Patient: hello doctor, i paramjeet my wife is pregnent before six month back but our babe is misscarge we have also done torch test but some infection is seen we use medicene in this connection. doctor suggest us you trying to conciv we tried but no result is postive anyway my question is what's we do please suggest us. Doctor: you.have,t mentioned the weeks of pregnancy but if it is early pregnancy loss bot h of you need f urther investigations if you are staying together for last six months."
},
{
"id": 147356,
"tgt": "Slight cerebral palsy, have uncontrollable head movements, minor facial tics, squint in eyes causing unsteadiness in feet. How can I cure this?",
"src": "Patient: I have uncontrollable head movements I ve had minor facial tics since I was a kid now my head moves constantly and my mouth goes to the right side of my face like some sort of force is pulling it there I also squint my eyes so they are closed I have no control over these movements I can t even shave myself or have a shower on my own or go outside on my own as these movements make e very unsteady on my feet what have I got I was diagnosed with very slight cerebral palsy at age 7 but only cuz I had no oxygen to the right side of my brain when I was born Doctor: HIThank for asking to HCMI really appreciate your concern if this is cerebral palsy then there is no permanent cure for this but of course certain rehabilitation centers are there and they are doing good job, that is the only option you have otherwise as long as the medicine is concern there is no medicine that could do something in cerebral palsy, hope this information helps you, take care and have a nice day."
},
{
"id": 220852,
"tgt": "What are the side effects of drop in HCG level during pregnancy?",
"src": "Patient: I went to doctor after finding out I was pregnant and according to my LMP, I should be about 8 weeks pregnant. doctor did an ultrasound and saw a gestational sac which was shaped perfectly, yolk sac and bleeding around yolk sac but no fetal pole or fetus. I have already had 2 miscarriages so she thought it would be another repeat or shall I say another \"blighted ovum\". I was then refered to an imaging diagnostic place the following week after having my hcg levels checked twice and seeing that it has dropped in two days.....to our suprise the following week at the diagnostic place, there was a fetal pole and a heartbeat but she said it was very faint...My question is, can I still have a healthy pregnancy after hcg level dropping but detecting a fetal pole and heartbeat Doctor: Yes you can still have a healthy pregnancy.Get your tests and USG repeated after 1 week.If you suffer from miscarriage again get your self investigated 2 find the cause and also get the fetus investigated for any chromosomal anomalies"
},
{
"id": 102727,
"tgt": "Experiencing difficulty in breathing during cold season , what is the problem ?",
"src": "Patient: i experience difficulty in breathing, when the weather is cold, or i'm stressed or if the area is stuffy or most times just randomly, Ventolin inhaler helps , but a scan showed I wasn't asmathic, so what could be the problem, sometimes, i just get too weak struggling for air and faint. Doctor: Hi, I think you need evaluation in form of complete blood counts, ECG ,Chest X Ray and pulmonary function tests. Cold air can lead on to bronchospasm and present with breathing difficulty. You can take ventorlin inhaler, In case you experience wheezing and difficulty in breathing . Hope it helps."
},
{
"id": 201942,
"tgt": "How to cure a cyst in mid portion of prostate?",
"src": "Patient: Hi I am Deepak 51 yrs male ,My PSA on general check up found to be 7 from last 1 yr. was recomended for TRUS and in that a 5.1 X 2.4 mm cyst is seen in mid portion of prostate just inside in peripheral zone simple in nature. free PSA was 15.6 . kindly advise deepak YYYY@YYYY Doctor: Dear Mr. Deepak,You need not worry about the simple cyst in prostate unless it is causing symptoms like painful voiding or difficulty in passing urine. Single small cyst in prostate does not usually cause any harm. However, PSA of 7 ng/ml is high. Please get a repeat PSA done, since the last value was one year old. The free PSA of 15.6 seems to be wrong. It could have been 1.56 ng/ml (free PSA is a part of total PSA and it cannot be higher than total PSA). Please make sure that you do not ejaculate within 48 hours prior to PSA testing and get a urine microscopy test done to rule out urinary tract infection (since urinary tract infection can falsely increase PSA).If the repeat PSA is more than 2.5 to 4 ng/ml (and freePSA/total PSA ratio is I hope I have answered your query. Please feel free to get back to me after the PSA result.Wish you a healthy life!Warm regards,Dr. Raguram G MS, MRCS, MCh (Uro), F. Endouro.Consultant UrologistAUKI"
},
{
"id": 99135,
"tgt": "Suggest treatment for asthma and allergy",
"src": "Patient: Hi My son has been diagnose with Asthma since February this year.He has a very high allergy level especially for grass, cats and dogs.But he did test positive for almost everything.Now he has some kind of Flu.He is coughing a lot and his left eye has also some kind of yellow discharge and the eye is red.Is it also part of the allergy?Can I give him some adco tusi linctus infants or some Mucospect gor infants?Can I also give him his asthavent pump every 4 hours like the pediatrician suggested when he if he gets flu?he has a fever , so I have gave him ibugestic fever syrup.The fever has come down now. Please help . please help Doctor: Hello dear,The symptoms as mentioned in your post can be attributed to acute exacerbation of asthma, secondary to bacterial infection.Asthma is caused due to broncho-constriction (obstruction of smaller airway passages) which is indicative of Hyper-responsiveness of air passages.Management includes:1. Asthalin & Seroflo inhaler- provide symptomatic relief by causing broncho-dilation (dilating the smaller airway passages, relieving the obstruction & increasing airflow to lungs). So, you can use it whenever he is having an acute attack.2. Analgesic antipyretic preparation (like crocin or ibugesic syrup) to control fever.3. Antihistamine preparations like Allegra & Cough expectorant syrup for symptomatic relief.4. A course of antibiotics needs to be taken to manage secondary bacterial attack.5. Maintain adequate hydration & a healthy balance diet.6. Also make sure that your son is well protected from cold, dust & other allergens.If still there is no improvement, kindly consult his treating Pediatrician/ Pulmonologist for further evaluation and management.Wishing him a good health.Take care."
},
{
"id": 197448,
"tgt": "What causes blood during ejaculation?",
"src": "Patient: Hello I am a 23 year old white male 165 lbs and I had something happen that scares me. I had drank that night some vodka and thats about it; but later that night I ejaculated blood! Part blood but enough to get the picture. So...im nervous and need professional advice. This has happened once before but it was more pink than red. I figured I busted a blood but I need your advice. Thanks Doctor: Hi thanks for contacting health care magic....First infection of accessary genital organs like seminal vesicles , prostate , epididymis might be infected that can lead blood in ejaculation....For that ejaculated Semen send to lab for analysis ....If shows pus cells then antibiotic given....If fever , weight loss like feature present semen tested for tuberculosis .....If needed culture can be done....USG abdomen will also be helpful....Trans rectal palpation and ultrasound will be helpful for rule out prostate cause.Meanwhile healthy and balanced diet taken...Fruits more .Keep alcohol and smoking very restricted.Consult surgeon with keeping my answer in mind.Take care"
},
{
"id": 130776,
"tgt": "Weakness and pain in left leg after robotic hysterectomy and lymph node removal",
"src": "Patient: I had a robotic hysterectomy, including lymph node removal, about 6 wks ago. The night after my surgery it was impossible for me to move my left leg toward my right leg while lying in bed. The first time I walked after the surgery my left leg felt weak and there was tingling in my foot. I also experienced aching in the leg and pain in the groin area together with low back pain. The pain has diminished but I am still unable to move the leg laterally while lying on my back and I am still experiencing weakness in the left leg when walking. It seems to take more effort to lift the leg. I have my final post op visit with my surgeon next week. Doctor: Hello and welcome to \u2018Ask A Doctor\u2019 service. I have reviewed your query and here is my advice. Its a post operative complication. Please contact a physiotherapist as certain exercises and movements can make things better for you. Avoid doing too much stress activities and take rest. Hope I have answered your query. Let me know if I can assist you further.Regards,Dr. Harsh Swarup"
},
{
"id": 128888,
"tgt": "What is the cause of electric shock on the arm with muscle tremors?",
"src": "Patient: what causes electric shock on my lower left arm with muscle tremors? i had skull fracture a year ago but ct scan show normal. i also have chronic neck pain that moves to back of my head. i had the neck pain before the injury. could u please help? email is YYYY@YYYY Doctor: Hi, I had gone through your question and understand your concernsHi, If I were your treating Doctor for this case , I would ask you to do MRI cervical spine to asses any bony abnormality or disc herniation which may cause radicular pain in your left arm . Also I advise you to receive the following medications:-Muscle relaxant as Tizanidine -Pregabalin 150 mg once before sleep to improve the electric shock sensation."
},
{
"id": 165609,
"tgt": "What are the symptoms of hyperthyroidism in the child?",
"src": "Patient: Doctor, i am going to adopt a child whose total T3 is 198 reference [70-200], T4 13 reference [4.8 - 13.5] and TSH 5 [0.4 - 5]. he is 11 months old. All values are in upper range of normal. Is he going to be hypothyroid in the future, currently is he hypothyroid ? Doctor: Dear parent,These are normal values for a 11month old .Ranges being near to upper normal is not suggestive of hypothyroid at present.please repeat the test after 2months ."
},
{
"id": 209406,
"tgt": "How to handle children who are sensitive and nervous?",
"src": "Patient: I was a very sensitive child...maybe even what one might call nervous. As I grew up as with most people, life changes all of that. It introduces you to a thicker skin and harder constitution, usually a good thing, a thing neccessary for life in the worl we live in. Lately, as in the last few months, I've noticed a difference in myself emotionally. I've become enitirely more sensitive then I've ever been in life. things that usually would have rolled off my back, hours later I fing myself still stewing over. Why am I sooooo sensitive lately? Doctor: Hi,Thanks for writingThe way we perceive the world changes with time and with other events in life. So its natural that you would also experience changes in yourself. A psychiatric session would be useful, and is recommended. This will help you to analyse yourself, as well as the life events in a comprehensive fashion.Hope that helps,Dr A Rao"
},
{
"id": 120819,
"tgt": "Suggest remedy for dull ache in the left biceps",
"src": "Patient: Hello. I m 41, and two years ago while doing pullups, I felt something give in my left biceps. As a former athlete I am familiar with broken bones, sprains, tendonitits, et cetera, but have been surprised that my biceps has ached off an on for two years. There is no bulge indicating a torn tendon, and really there has been no loss of strength either. Like I said, I m surprised the dull ache has continued and am asking for a course of action. Thanks. Doctor: Hi, According to me There may be biceps Strain or partial tear of fibers in both above condition you cannot see pop up of belly. I think you can avoid 4 week biceps exercise and allow those muscle fiber to heal and keep your arm in sling and then after 4 to 6 weeks, gradually start exercise if then also pain persists, get an MRI done. Hope I have answered your question. Let me know if I can assist you further. Regards, Dr. Gaurang B. Shah, Orthopedic Surgeon"
},
{
"id": 30727,
"tgt": "Can cervical degenerative disc disease cause tinnitus?",
"src": "Patient: Can cervical degenerative disc disease cause or exacerbate tinnitus? I am 52, weigh 125 lbs and am 5'6\". I am in excellent health but about 5-6 months ago had onset of central tinnitus. Find I have cervical DDD and am wondering about it as cause for tinnitus. Doctor: Thanks for posting your query to HCM . Cervical disc degeneration causes compressive effect over nerve passes from there .It is rare to cause tinnitus by cervical disc degeneration but You need to consult to neurophysician for knowing the level of compression so that measure to treat it can be taken .take care."
},
{
"id": 81414,
"tgt": "What causes sharp pain in my chest?",
"src": "Patient: Hello, i would like to ask a question, i was woke up in the morning 4.30am by a sharp pain in my chest (right in the centre) i thought the pain would go away but it is still painful. Almost every movement i make i would feel the sharp pain esp when i am crouching or when i try to hug my own body.. please help! Doctor: Hello dear, thanks for your question on HCM.In my opinion you are having muscular pain only.But early morning chest pain can be due to cardiac cause. So better to get done ECG first to rule out cardiac cause.If ECG is normal than no need to worry much for cardiac cause. Your pain is worsening with movements, this also favours muscular cause.So try to follow these steps for better symptomatic relief.1. Avoid heavyweight lifting and strenuous exercise.2. Avoid movements causing pain.3. Avoid bad postures in sleep.4. Take good pain killer and muscle relaxant.5. Apply warm water pad on affected site."
},
{
"id": 63194,
"tgt": "What causes lumps on left testicles?",
"src": "Patient: hi recently i noticed a lump on my left testicle and i have know idea what it is some pictures are linked above so can you please take a look at them and tell is there any chance of cancer and what is the cause?thank you WWW.WWWW.WW WWW.WWWW.WW WWW.WWWW.WW Doctor: hi...testicular malignency may present with enlagesment of scrotum due to enlarging testes...initially a ultra sound scan will provide good information about irr"
},
{
"id": 141300,
"tgt": "Suggest treatment for severe neck pain and dizziness",
"src": "Patient: I have scoliosis and a slipped disk, I did something to both of my shoulders after ramming them into the door frame on accident, I had something happen to my neck after my car accident due to bad conditions, and now, I feel a lot of pain to the point it ll make me weak to move, and when I crack them or stretch to release the pain, I get this metal/ blood taste in my mouth and I can also literally feel my bones grinding against each other. It makes me get dizzy as well, to the point I ve blacked out a few times. Woke up to finding my head hitting the toilet. Is this something deeply concerning or am I just freaking out? Doctor: Hello and Welcome to \u2018Ask A Doctor\u2019 service. I have reviewed your query and here is my advice. There can be a number of causes. Treatment depends upon underlying cause. Spondylosis can be an important cause. Please upload medical notes and investigation reports if available. Thanks. Best regards."
},
{
"id": 191490,
"tgt": "What causes elevated sugar levels after getting a Cortisone shot?",
"src": "Patient: My mom has type 2 diabetes and got 2 cortizone shots in her knees about 3:00 pm today. Her blood sugar is 345 now. She took another of her blood sugar pills and is drinking lots of water. She said she feels fine right now. Should we just continue to monitor her level or what? Doctor: welcome to hcm...cortisol increases sugar level ..it is the physiological effect ...nothing abnormal here .."
},
{
"id": 51673,
"tgt": "Why is white fluid comming during urination?",
"src": "Patient: I m 28-Male.White like fluid coming during urination. Kindly advice whats the problem Doctor: Hi,Welcome to HCM.White discharge while passing urine may be normal semen discharge. In that case there is nothing to worry. If you have history of sexual contact, then it would be a good idea to get blood test done for sexually transmitted diseases.If the whole urine is white in colour (like rice water) then it is a condition called chyluria, for which you will have to consult an urologist."
},
{
"id": 82332,
"tgt": "Suggest treatment for pneumonia with inflammation of left lung",
"src": "Patient: I have had pneumonia for 3 weeks. Fist antibiotic was doxycycline. took for one week the developed a sinus infection so took augmentin and stopped doxy. Took that for one week. Very little improvement in how I felt. Chest xray repeated . R lung cleared Left lung ?. Had ct scan of lungs showing inflammation of left lung. Now on levoquin for 4th day and still feel fatigued and nopt any better. All this after 4 months of sinus infections and various antibiotics. I am 64 and very concerned. Doctor: Hi,Thank you for your query. I can understand your concerns.It seems that you have delayed resolution of pneumonia.Inappropriate antibiotics or poor patient compliance, suboptimal dose, incomplete treatment, drug resistance are the common causes of delayed resolution of pneumonia.Immunocompromized patient -decreased host resistance like diabetes mellitus, chronic alcoholism, Agranulocytosis, hypogamaglobulianemia, elderly and debilitated patient are other contributing factors in different cases.Since your response to treatment is slow,co-morbidities need to be ruled out.However a prolonged course of suitable antibiotic /s wil likley to resolve the problem.Regards Dr. T.K. Biswas M.D.Mumbai"
},
{
"id": 63423,
"tgt": "Suggest remedy for a lump appearing under the thigh",
"src": "Patient: I just felt an very large soft not painful lump on my inner thigh. I have had a lower right tummy pain (like gas would feel). I had alower back pain for a couple of weeks sleeping on all kinds of couches and chairs, so I thought the gas like pain was related, now I'm worried. What is this soft pliable lump? I also noticed the same thing under my left arm no pain very movable. i'm scared now that idiscovered the inner thigh one now Doctor: Hi, dearI have gone through your question. I can understand your concern. You may have some soft tissue tumor like lipoma, neurofibroma or dermatofibroma. Or it can be some cyst or other mass. You should go for fine needle aspiration cytology or biopsy of that lump. It will give you exact diagnosis. Then you should take treatment accordingly. Hope I have answered your question, if you have doubt then I will be happy to answer. Thanks for using health care magic. Wish you a very good health."
},
{
"id": 161288,
"tgt": "Does dizziness, fever and headache in a child indicate sinus infection?",
"src": "Patient: My 9 year old son came home from school yesterday complaining of dizziness and headache. This morning he woke up with the same complaints and a fever of 102 in addition. He has asthma, food allergies as well as outdoor allergies to give you a little background. My question is does this sound like a sinus infection? He has had them in the past but this seems a little different than before. Doctor: Hello, Is your child disoriented? Sometimes meningitis also have symptoms of headache dizziness fever. please go to a nearby pediatrician. Hope I have answered your query. Let me know if I can assist you further. Take care Regards, Dr Prasanna Lakshmi, Pediatrician"
},
{
"id": 212738,
"tgt": "On sertaline for mental disturbance, depression and anxiety. How can I fight my lifelessness?",
"src": "Patient: i dont want to live problems r so overwhelming i cry loads i shake loads i sleep more than anyone i know i dont want to eat i dont get out of bed fr wks at a time i dont go outside oftern my doc gave me sertaline 100ml but i dont feel there working is there anything stronger i can take doc said i have depression and anxiety bn on tabs now since nov shouldnt they be working by now Doctor: Hello..... Thanks for your query. I understand your concern with depressive symptoms. Sertraline is a standard antidepressant that is prescribed to alleviate depressive symptoms. However, antidepressants take atleast 6 weeks to start acting. Hence, you need to be patient. Many other effective classes of antidepressants like SNRI's and TCA's are available in addition to psychotherapy (eg cognitive behavioural therapy). So please do not lose hope. Regards Dr Sundar Psychiatrist"
},
{
"id": 150321,
"tgt": "Allergic opiate to pain medicines. Only take NSAID. Suggest non opiate pain medicines after surgery?",
"src": "Patient: I'm having a L5 -L6 spinal fusion in June. I am allergic opiate to pain meds and only take NSAID's now. Do you know of any non opiate pain meds I can take for the pain of surgery? I tried Ultrim which is not necessarily an opiate. My sinuses start pouring syliva into my throat and then I have trouble swallowing and feel like my throat is closing off. Doctor: Hi,Thank you for posting your query.Most likely you are going to have L5-S1 fusion.Good and safe medications for nerve pain in your case include gabapentin and pregabalin. You can use either of them after the surgery.Please get back if you require any additional information.Best wishes,Dr Sudhir Kumar MD (Internal Medicine), DM (Neurology)Senior Consultant NeurologistApollo Hospitals, Hyderabad,My personal URL on this website: http://bit.ly/Dr-Sudhir-kumar My email: drsudhirkumar@yahoo.com"
},
{
"id": 11650,
"tgt": "Temporary brown spots on upper thigh, like pigmentation spots spot on throat. Cure for symptoms?",
"src": "Patient: Hi,I have light brown spots on my upper inner thighs. They are coming and going, but recently I feel like I have more spots. I have them in both sides and they are not itching and they are painless.I'm a healthy, normal woman sizes woman.The spots are different sized, and like I said they have been coming and going. At first I just thought that it was some pigmentation spots, but now I'm getting worried and I don't want them to spread more.I also saw on spot in my throat, a really small one but the same type.Thank you. Doctor: the symptoms which you have mention is cleared that it is a fungal infection but it not proper to justify and treat your symptoms without watching it.so for this kindly upload the picture of your effected area.... then it will be treated or suggested ."
},
{
"id": 175690,
"tgt": "Suggest medication for fever and white/brown splotches on tongue",
"src": "Patient: My 1 yr old daughter has had a fever, not sure of the range just felt heat on her forehead and extremities. It was pretty intense for the first 2 days and the past 2 days it hasn t gotten as intense. She has also been pretty lethargic until the Motrin locks in then she gets a little energetic for a little bit but not to normal levels. I ve also noticed the past few days now that her tongue has white/brown splotches. What is it and how can I help her? She is very whiny and hard to deal with and I really would just like to get her comfortable. Doctor: Hi...this looks like viral illness. In viral illness it is quite common for the tongue to have some white patches. Fever of few days without any localizing signs could as well a viral illness. Usually rather than fever, what is more important is the activity of the child, in between 2 fever episodes on the same day. If the kid is active and playing around when there is no fever, it is probably viral illness and it doesn't require antibiotics at all. Once viral fever comes it will there for 4-7 days. So do not worry about duration if the kid is active.Regards - Dr. Sumanth"
},
{
"id": 40241,
"tgt": "Treatment for cat bite apart from tetanus shot and peroxide?",
"src": "Patient: I was bitten by my cat. He gave me 4 puncture wounds in my forearm. I rinsed it with cold water and ran peroxide on it. I put ozonol and a gauze over it. I haven't had a tetanus shot for quite a while. Aside from getting a shot, which I will tomorrow, is there anything else I should do? Doctor: Hello,Welcome to HCM,As you were bit by a cat over the forearm, which has led to punctured wounds.As the cat also has all the potential to transmit rabies, which is 100% fatal but it is 100% preventable. You should not ignore this and you should take appropriate and complete treatment.I would suggest you to follow1.Washing the wound under running water with soap2.Inj Tetanus toxide3.Antirabies vaccine, intramuscular on to deltoid on days 0,3,7,14 and 28.4.Antirabies serum or rabies immunoglobulins (RIGs) in and around the wound5.Course of antibiotics.This treatment is most and forget to take all the dose of antirabies vaccines on to your deltoid (shoulder).Thank you."
},
{
"id": 94736,
"tgt": "Abdominal pain radiating to lower back, legs. Urine test shows WBC, ultrasound with minimal impression on pelvicalyceal system. Advice?",
"src": "Patient: Salam My age is 25 years and weight is about 63-64 kg I am feeling pain on my right side abdomen ,( first it was on right side now its right near belly button and upper side) from more than 6 months, It radiates in to the lower back, I feel back pain when i wake up in the morning and also i feel tightness on right side. also pain in legs. All the time i feels there is difference at right side. I had all the blood test which are clear including HCV and HBS, The urine test shows. 6-8 WBC I had ultrasound 4 times which was clear only the last ultrasound shows. minimal impression on right side pelvicalyceal system. Em really worried about the condition please advice me. Regards Doctor: Hi Thanks for yyour query. See the pain described by you has quite a few differential diagnoses like pancreatitis, renal stone, chronic appendicitis and pain originating from pelvic organs. I think you need to have urine culture and sensitivity testing because it may be just a chronic cystitis. Again I will suggest you to go for a contrast enhanced computed tomography (CECT) scan of the abdomen because it will delineate the pathology clearly if present. You should consult an urologist. Because cystoscopic evaluation is needed if renal or bladder pathology is suspeted or you can go for intravenous pyelogram after consultation with your urologist. But first of all you should go for CECT abdomen and rest of the investigations should be decided by CECT finding. Thank you."
},
{
"id": 11070,
"tgt": "Suggest remedy for hair loss problem",
"src": "Patient: hello sir , m a 24 yr old girl and m suffering from hair loss since last 2 years, i went to doc and he suggested me minoxidil topical solution usp 5% along with kevon stay on lotion . i use th solution twice daily 1ml every time , since one week i have been using but no results found , even m lossing more hairs ..... and i have irregular menstruation problem too. so plz help me out frm tis problem. Doctor: Hello dear,Since you are giving history of hair fall with irregular menses, you will need a hormonal workup to rule out any hormonal imbalance. If it is found then you will need treatment for the same. Minoxidil increases hair fall initially as it causes the non growing hair to fall out and new hair to grow from the same hair root. You will have to continue with it and should see new hair growth in 3-4 months time."
},
{
"id": 224645,
"tgt": "Worried about effectiveness of birth control while taking amoxicillin? Any need of taking plan B?",
"src": "Patient: Hi, I ve been on Loestrin24 for 6 months now (just finished my 6th pack of active pills yesterday), and have been taking Amoxicillin for 5 days. I had unprotected sex twice last night (he also withdrew before ejaculating). I asked the pharmacist today, and she said that Amoxicillin affects effectiveness of the birth control , but not so much that I need Plan B (because it s only for people who have no type of protection at all). I m still concerned. Should I use a back-up pill (Plan B) today? Also, should I continue my birth control as normal? I should be taking my brown pills starting today, but I m worried about the Amoxicillin and birth control interaction. Plus the sex without condoms , especially because there was another penetration after the first ejaculation . Doctor: HelloThanks for writing to us with your health concern.Amoxicillin does not affect your birth control.There is no need to take Plan B.Take care."
},
{
"id": 25648,
"tgt": "Suggest treatment for high blood pressure after a massive heart attack",
"src": "Patient: hi i had a massive heart attack at 40.and had three stents put in since then.im on heart meds that was nine years ago.my blood as never been up in that nine years the top number i just check it and its 154 over 84.like i said thats the first time yet .its been like this of and on for three weeks Doctor: Thanks for your question on Health Care Magic. I can understand your concern. Yes, you should definitely control your systolic blood pressure below 140mmHg. For this strict salt restriction in diet is needed. You are taking your cardiac drugs since 8 years. So you may need high dose of some antihypertensive drug or you may need additional antihypertensive drug too. Make sure that you are taking newer beta blocker (metoprolol) because it is having excellent effect in post angioplasty patient. It reduces heart damage and also control blood pressure. So discuss all these with your heart doctor and start treatment accordingly. Avoid stress and tension, if you are having stressful life. Hope I have solved your query. I will be happy to help you further. Wish you good health. Thanks."
},
{
"id": 183620,
"tgt": "What is the remedy for painful and swollen cheeks?",
"src": "Patient: I had 11 teeth extracted on wed of last week since then I have had servere pain and swelling in my cheeks?The xtraction was upper.I went to ER to get Antibotics since Dentist did not and I have nothing for pain,it hurts soooo bad,started antibotics at 11 last night and then again at 4am and then again at 11 am and now again at 6 due to driving home from work,yes working whiel this is going on.....what to do???? Doctor: hellooo...read thru ur query....pain in the cheek is obvious after extraction of teeth...but dont worry...it will be healing..but continue with antibiotics as prescribed by the dentist...also do take painkiller(diclofenac) whenever pain is there..also try to have soft diet...try not to bite heavily on that extracted areas(i kno its difficult but u try maximum not to bite much)for a week..and do salt water gargle as many times as possible (minimum 2 times).while spitting dont spit with too much force..just be soft spitting...also while brushing dont touch the areas with toothbrush or fingers..and tongue...follow this for a week..and still if pain persists just go and meet ur dentist...(i feel if u follow this strictly pain will be relieved).be cool and calm...stress will increase pain..hope ur benefitted something from this reply..have a healthy day!!!"
},
{
"id": 71925,
"tgt": "What causes bluish discoloration of lips and nails?",
"src": "Patient: Hi, may I answer your health queries right now ? Please type your query here...I am 66 and have had migraines since I was 19, on thursday I had a bad migraine, took zomig and tramacet, looked very pale all day and at one point I noticed my lips looked blueish, I checked my fingers and my nails were also blue, this is a new symptom to me, should I tell my GP. Doctor: Hello dearWarm welcome to Healthcaremagic.comI have evaluated your query in details .* This seems possible side effect of zolmitriptan , must seek medical attention with your doctor .Wishing you fine recovery .Feel free to ask any further queries .Regards ."
},
{
"id": 22292,
"tgt": "Will bypass help increasing heart Ejection factor?",
"src": "Patient: Hi,my father is 56 year old and had IHD on Aug'10.At that time his heart EF was 30% but recently in April'11 it is 20 to 25%.His BP,Sugar and lipid profile is normal.consultant Dr. is not suggesting any operation. Is bypass will help to increase the heart EF? Doctor: Hello,Not always. It depends upon whether the part of the heart for which we are opening the blood vessels by bypass is living or dead. If that part of heart is living then the chance is there, that heart function will improve to certain extent otherwise not. The test will tell us this information is either pet scan or thallium scan. You discuss these issues with your doctor. Also the ICD or CRT device will be lifesaving here, so also discuss this also with your local doctor. You should have a healthy lifestyle like avoiding fatty, oily and high calorie diet. Have low salt diet and monitor blood pressure regularly. Regular exercises like brisk walking, jogging according your capacity at least 30 min a day and 5 days a week. Eat lots of green leafy vegetables, fruits, fish once or twice a week, avoid meat. Avoid smoking and alcohol if any. There shouldn't abdominal fat deposition or obesity. Get your lipid profile and sugars tested once.Hope I have answered your query. Let me know if I can assist you further. Regards,Dr. Sagar Makode"
},
{
"id": 92429,
"tgt": "What could be the reason behind recurrence of severe abdominal pain, nausea, vomiting and dehydration?",
"src": "Patient: My son is 27 and been having re-occurring bouts of severe abdominal pain, nausea and vomiting to the point of dehydration. He has been to the er repeatedly, receiving fluids and having tests periodically. He has no insurance so he has been unable to follow up. And all they do at the er is re-hydrate him and send him home basically. The last ct said that his liver was notable for the presence of periportal lymphedema. Which stated that it could be indicitative of infectious or inflammatory process in the liver. They didn't mention anything about this to us, nor did they say anything about the labs, which several things are flagged as HIGH or LOW. That was 2 days ago, he is back at the er again today. I am extremely concerned. I guess my question is could there be something really wrong here, this has been going on for about 2 years Doctor: Hi, Welcome to Health care magic forum. So far it is clear that the liver is effected, with periportal lymphadema. Now liver function tests are to be done , to know what is the wrong. S.G.O.T, S.G.P.T, HbSag,SERUM BILIRUBIN, etc are to be done. I advise you to consult a gastro enterologist for diagnosis and treatment. His habbits are also to be taken into account, like alcohol, smoking, marijuana, etc. and should be avoided. Wishing for a quick and complete recovery. Thank you."
},
{
"id": 218189,
"tgt": "Can Tylenol and Amoxicillin be taken together during pregnancy?",
"src": "Patient: Im pregnant, and I have an abscess on my gum. Last night I went to the ER cause the pain was excrutiating. The doctor just gave me Tylenol for the pain, and said it was safe. I visited my dentist today, and he prescribed me Amoxicillin 500mg fo the abscess. I was wondering if it is safe to take them both around the same time? Doctor: Hi, Paracetamol is used to relieve the pain while the antibiotic is to treat the infection. I would ask you to consume the medications at least with a gap of 30 minutes to have better action. There is no proven teratogenicity in human studies with these medications, hence they can be used when necessary. Take them along with food to ensure lesser acidity which is a possible side effect. Hope I have answered your query. Let me know if I can assist you further. Regards, Dr. Shanti Vennam, OBGYN"
},
{
"id": 33790,
"tgt": "What causes swollen lymph nodes in the groin area?",
"src": "Patient: I have swollen nodes in my groin but I dont beleive I have an infection or anything. I just checked out for herpes and all other STDs including aids and all was neg. i just want to know what else could be causing this and if I go to a doc.. will they be able to tell where the infection is in my body with some test? Doctor: Hi, i did review your question. Swollen nodes in groin could be due to multiple reasons. Apart from what causes you have mentioned it could be because of an infection in your feet or in belly or some other cause. I would advice you to go see a doctor and get it properly evaluated. The best diagnostic test if the swelling is for more than 2 weeks and not red and hot, should be to do a fine needle aspiration study (FNAC), this will help determine the doctor what cells it is made up of and will then guide further management and treatment from there on.I hope this helps.Wish you a superhealthy life ahead"
},
{
"id": 208476,
"tgt": "What causes emotional and depression while having nuvaring?",
"src": "Patient: I have been using the nuva ring as a form of birth control for 2 years. Since, I have become so super emotional and depressed. ! minute happy and the next on the verge of tears for such rediculous reasons. Am i imagining that it could be the nuva ring? Doctor: Hello, Please talk to your gynaecologist to shift you to some non-hormonal contraceptive choice, there are many in market nowadays days. The hormonal contraceptives are notorious of causing psychological disorders ( there are several studies available on them), or these may even increase the already existing psychiatric illness, or can lead to relapse in non otherwise stable patient.Your suspicions can be right about this device, though it is not possible to be sure ( as there may be other clauses like stress or family history, etc.) But this is an avoidable thing and can be replaced with other safer choices. If after removing this agent, your mood improves, this will be an indirect evidence that this was the causative factor behind mood changes.So, Best will be to shift to other contraceptive methods.Wish you good luck.Dr. Manisha GopalMD Neuropsychiatry"
},
{
"id": 197344,
"tgt": "What does sonography as Mild Hydrocele suggest?",
"src": "Patient: On finding enlarged Left Testicle, a Urologist after sonography has opined that there is Mild Hydrocele (Left) and Moderate Vericocele Right). But from occasional slight prick left side of immediate adjacent area of my lower abdominal, I feel there can be a sort of abcess. What should be my next step to know the factual position? Please guide me. Doctor: helloThanks for query ,You have consulted urologist who has carried away your clinical examination and have been detected to have minimal hydrocele on left side with moderate hydrocele on Rt side .The Urologist will never miss any abscess during clinical examination .The pricking sensation that you ave could be infection of either epididymis or testicle itself .Take antibiotics and anti inflammatory medications for 10 days .Dr.Patil."
},
{
"id": 105809,
"tgt": "What could be the cause of thick mucus in one with chronic sinusitis and asthma ?",
"src": "Patient: Hi I had a CT scan confirm I have chronic sinusitis and also I have asthma . For over six months now, I noticed when I spit in to a glass water, this constant thick clear phlegm comes out all day long -NON -STOP , thick looking. When I spit in to a glass of water, it appears cloudy and streaks down and settles at the bottom looking cloudy white. Also, the phlegm is thicker in the morning when I wake up. I have been taking predisone , Amoxicillin , doing Nasal Netti pot, and Accupuncture and nothing seems to be curing it. I am wondering if I have candidasis ? At one time my mucus was thick yellow but after taking amoxiciling the sinus infection cleared, but still the thick mucus is coming all day long. Any idea what tests I should have done or what it might be? Doctor: Hi Genxbtq, Chronic Sinusitis and Asthma both can cause this type of mucoid sputum. When there is yellowish or green sputum from nose or mouth along with other symptoms like facial pain, cough, loss of smell and taste etc, there is an indication for Antibiotics. Once you start antibiotics, remember to continue it for longer duration ( not usual 5 days course) like 10-14 days for Chronic Sinusitis. I hope you are taking Inhaler for prevention of Asthma. The dose, and duration of Prednisolone tablet should be guided by your doctor. I would also advise you to increase the dose of preventive inhaler, as soon as you see any first symptoms of flu, sinusitis, allergic Rhinitis. Every year you should take Vaccine against Flu, and you must also take Vaccination against Pneumonia. You must continue doing steam inhalation two times in a day as a routine. Take good care of your food, and heigene. Avoid any allergen, if you have any. Wish you a stable health."
},
{
"id": 201040,
"tgt": "What causes redness around penis after unprotected sex?",
"src": "Patient: About 10 days ago I had unprotected sex with a prostitute for the first time ever, after exactly 30 mins after I had sex , I saw this redness on between the head of my penis and shaft, its been there ever since, it usually turns white and starts peeling off in a way but when I take a shower its red again, I dont feel any pain, or itchiness in any way, I also did use a condom but the then didnt use one for about 3 mins. Doctor: Thanks for asking in healthcaremagic forumIn short: Mucosal erosion may have caused this.Explanation: Getting redness within 30 min of unprotected intercourse with any one is not infection. As per my opinion it is the mucosal erosion for having sex without lubrication or having sex when both are dry. So, abstain from sexual activity for now and visit a doctor to get yourself examined for your lesion and for HIV/STD as you had unprotected sex with a prostitute."
},
{
"id": 98393,
"tgt": "How can cold and chest congestion be treated?",
"src": "Patient: I have Asthma and I started out having a cold one week ago. 5 days ago I went to urgent care and discovered I had an oxygen level of 88. They used oxygen on me 3 times and a nebulizer on me 2 times. They were finally able to get it up to 98, so they released me. Now I still have congestion and I sound like between a squeaky door and percolator. Does it sound serious enough to go to ER or wait and see dr on Monday? Doctor: Hello and Welcome to \u2018Ask A Doctor\u2019 service. I have reviewed your query and here is my advice. If you are on asthma medications, there is a requirement to check life threatening exacerbation. Please consult a doctor. Hope I have answered your query. Let me know if I can assist you further."
},
{
"id": 171827,
"tgt": "Suggest treatment for rash around nose and mouth after catching cold",
"src": "Patient: My 5 year old has had a nasty viral cold flu for 3 days. Copious amount phlegm runny nose. Now has 5 or 6 pimply bumps around nose and mouth. I am trying to keep area clean and least amount of wiping . She has down syndrome and hard to get her to wipe or blow her nose. Is there anything I can put in this area. I ve been putting aquaphor Doctor: Hi,Thanks and welcome to healthcare magic.Downs' syndrome children have depressed bridge of the noseHence they are prone to recurrent cold.Because of continuous flow of nasal discharge and saliva they have rash and infection around the nose and mouth.Keep the area dry as far as possible .Application of oil or Vaseline may give relief .He may need antibiotic if infection supervenes .Hope this answer serves your purpose .Please feel free to ask further queries if any..Dr.M.V.Subrahmanyam."
},
{
"id": 89062,
"tgt": "What causes left side abdominal pain with a history of PCOS?",
"src": "Patient: 26 year old, white female, 200lbs, 5'6\", +hx for PCOS. Currently having mild-moderate left side abd pain x2days. Laxative produces bowel movement w/o strain or discomfort. Pain increased with pressure to area and sudden movement. I'm not sure if it is muscular, ovarian (although it doesn't feel like usual ovarian pain) bowel obstruction, divirticulits? Doctor: most likely it sounds like diverticultiis and i suggest antibiotic treatment with cipro and metronidazolum. if you are not sure you need to do ct scan. wish you good health. Regards"
},
{
"id": 197716,
"tgt": "What does my semen analysis report mean?",
"src": "Patient: DEAR DOC, volume of 05 ml,count is 49 million, motility is 76 % and in the end of report comments are (subnormal sperm progression &morphology, 82% survival in 24 hrs sperm preparation and finally written is suitable for Natural conception &/ or IUI yes and in sperm morphology normal 01/100 strict kruger s criteria abnormal 99/100 defects cephalic 97/100 midpiece 65/100 tail 28/100 42 % spermatozoa with small & with out acrosomal caps seen.30 % spermatozoa with amorphous seen is it normal or ????? Doctor: Hello and welcome to health care magic.Looking vat your description your semen analysis report seems to be normal, however if you can write back to me with more details like since how long you are married, why did you go for semen test? so that I will be able to guide you more accurately.If possible upload all the relevant reports.meanwhile avoid smoking and alcohol consumption, have lots of fresh fruits and vegetablesExercise regularly,multivitamin and minerals like zinc, selenium and calcium will definitely help you thanks for your question, please feel free to ask for follow up question, I will gladly answer you.Best wishes"
},
{
"id": 62844,
"tgt": "How to get rid of lump in left armpit?",
"src": "Patient: DearDr. Krueger, On Oct 31st, my husband found a lump under his left armpit about the size of half of a tennis ball. Our primary then referred us to a general surgeon who removed the lump on 11/8. The surgeon moved up his post-op visit because he felt that based on what he saw that he would be referring us to an oncologist. However, the pathology came back benign. The pathology reoprt did mention granulomas so we were referred to an infectious disease specialist who has run numerous tests all of which have been unremarkable except the c-reactive which was a little high. I believe it was 9. They did notice some nodules on his lungs for which we were referred to a pulmonary specialist who assured us that they were normal. However, he did suggest that my husband obtain a ct of the abdomen and pelvis as the did notice some enlargement of the spleen. About a week ago my husbands left elbow became swollen similar to the look of the axillary. Although he has no history of more than a cold in his 44 years, his father had non-hodgkins and his mother had a brain tumor. They had about a 5 acre farm in which pesticides were used and I feel concerned that this could be related. At the present time he has little or no complaints. He awoke last night with what seemed like night sweats as he had to change his clothes. Finally my question to you. Could the pathology have been wrong and should we seek a consult with an oncologist? Thank you for your time. Terri Doctor: Hi, dearI have gone through your question. I can understand your concern. His lump can be due to tuberculosis, other granulomatous disease like sarcoidosis or some lymphoma. You should go for second opinion of that histopathological examination. Review of histopathological slide gives you exact idea. If still there is any doubts in diagnosis then IHC marker study will help you. Then you should take treatment accordingly. Hope I have answered your question, if you have doubt then I will be happy to answer. Thanks for using health care magic. Wish you a very good health."
},
{
"id": 159571,
"tgt": "Bulging abdomen, discomfort. Can chemotherapy cause it?",
"src": "Patient: My uncle has stage 3 of MM, he is 68 years old. He just got diagnosed . After the chemotherapy he had in the hospital 4 sessions, 30%of the dosage, he is getting cured at home. He is using these drugs Bondronat 50, Sodio Bicarbonato Nova Argentia, Jena Purinol, Desametasone Soldesam. It has been about 15 days that he has problems of constipation , and the nurse has done Emptiness of the intestine . The problem is that his abdomen has bulged a lot and he doesnt feel good. What can be done? Can the chemotherapy cause the abdomen bulge? Is it risky? How long might he live under these conditions? Thanks a lot Doctor: Dear Erinda, Your uncle was diagnosed with stage 3 multiple myeloma? Or malignant melanoma? These are different diseases and so you should give more details if you can. If it is a Multiple Myeloma, he has an incurable disease but nowadays there are several new treatments available which are very effective in obtaining clinical remission. Some of these therapies may cause constipation, and so the symptoms that your uncle has, may have been caused by the treatment. For that, he should take laxatives and eventually ocasional intestinal emptyness. He should drink plenty of water and try to have a diet with high fiber content. With time it is likelly that constipation will get better. Best regards, Rute Alvarez Hematologist"
},
{
"id": 12663,
"tgt": "Suggest remedy for rashes",
"src": "Patient: I have what appears to be a rash on both my shins. There are blotchy areas consisting of tiny red dots, they range in size from a pencil eraser to about 2 x 1 inches. They do not itch, they are not raised, have no borders, they are not dry or scaly and only appear on the front and sides of my lower legs. I first noticed one spot the size of a pencil eraser near my ankle about a month ago and a few days later they are all over my shins. I thought maybe it could have been from shaving with a disposable razor but it has not improved with using a brand new razor each time. My diet, detergent, lotions, soaps have not changed. What could this unusual rash be? Doctor: Hi, It seems to be vasculitis or pigmented purpuric dermatoses. I would recommend you to consult a doctor and get it examined. Take care. Hope I have answered your question. Let me know if I can assist you further. Regards, Dr. Asmeet Kaur Sawhney, Dermatologist"
},
{
"id": 92157,
"tgt": "How long should Cyclopam and Norflox- TzHi be taken for pain in the lower side of the stomach?",
"src": "Patient: Hi I have pain lower side of the stomach now this pain increase and started lower back pain and it's very painful I taking Cyclopam and Norflox- Tz is this medicine will work and can you tell me for how many days I have take and I feel like body swollen please suggest me what to do Doctor: HI. You may have this pain from variety of problems. Take the medicines for 5 days. If no response I would advise you to go for the blood tests including that for kidney ( blood urea and creatinine ) Urine and stool, ultrasonography of abdomen. A proper diagnosis will guide the treatment in a right direction."
},
{
"id": 34284,
"tgt": "Suggest treatment for infectious mononucleosis",
"src": "Patient: I ve previously had strep in January then was diagnosed with mono. Almost two months later I have a growth in the back of my mouth on my jaw that looks like a raspberry but is flesh colored. What could it be? It was bigger but it s slowly going down. Theres a couple of white patches that come and go also. Doctor: Hello,Welcome to HCM,Infection with Epstein-Barr virus (EBV) is very common and usually occurs in childhood or early adulthood. EBV is a contagious infection that spreads from person to person.EBV is the cause of infectious mononucleosis and the symptoms are fever, sore throat, swollen lymph nodes in the neck, and enlarged spleen. The symptoms like nausea, vomiting and diarrhea are very much rare with EBV infection.Individuals with EBV infections requires1.Adequate rest 2.Plenty of Fluids3.Corticosteroids to treat significant swelling in the throat4.There is no specific medicine to treat EBV infections.Thank you."
},
{
"id": 39763,
"tgt": "Can chicken pox spread from a child to an adult?",
"src": "Patient: hi dr grief my name is michelle and my 4 yr old boy has just come out in a rash which looks like chicken pox, i just need to know if adults can catch chicken pox from him as he is seeing his 60yr old nan tomorrow and im not sure if its safe for him to go thank you michelle Doctor: Hello,Welcome to HCM,Chickenpox is a highly communicable 1-2 days before the appearance of rash and 4-5 days thereafter. After which virus tends to die out and looses the capacity to transmit disease to normal individuals.As your 4 year old son has just come out in a rash which looks like rashes of chicken pox, he has all the chance of transmitting the disease to other health individuals including the adults.The scabbing usually begins 4-7 days after the rash and completed by 30 days. During the scabbing period your son will be sterile and will not transmit disease to others.Thank you."
},
{
"id": 47028,
"tgt": "Suggest treatment for enlarged kidney",
"src": "Patient: hi, im 22 female my weight is 45, height 5'4 my kidney is enlarge(big) on two side of the kidney the symptoms i have is feeling likevomitting, breast pain, itching in kidney and have some disturbance in urinating and my body is restless i did not met any doctor this problem was happen last week and i dont know how it happens i was feared and tensed of this problem im very scared to see the doctor please help me doctorsuggest me the solution Doctor: hii have understood your concern.as the history given by you you have1.hydronephrois2.uremic symptomsprobabilities of your problems are1.urinary tact infection with spread to both kidneys2.urethral stricture with back pressure on kidneys3.bilateral stonesyou needs to be evaluated defenitely.i can suggest you to get1.urine routine and culture2.USG of abdomen and KUB3.serum creatinine and serum electolytestreatment suggestion1.tab. lanol ER 650 mg twice dailytab.palozac 0.5 mg once a daydepending upon reports you need further treatment.get beck to me with reports.hope this helps youthanks."
},
{
"id": 23267,
"tgt": "What causes feeling of heart beating in throat?",
"src": "Patient: why do it feel like my heart is beating in my throat,i am 50 years of age,5\"1 and weight 245lbs and i have high blood pressure ,last i check it was 144 over 112.I do take medication verapmil er 120mg every 12 hours,I also take fluoxetine for anxiety 20 mg once daily and sulfasalazine 500 mg twice a day.OH and by the way i am african american. Doctor: Hello!Welcome on HCM!I carefully passed through your question and would explain that your symptoms could be related to a cardiac arrhythmia. From the other hand anxiety could trigger this symptomatology. Your diastolic blood pressure values are significantly high and this means that your actual anti-hypertensive therapy is not very effective in controlling high blood pressure. Coming to this point, I would recommend consulting with your attending physician for a careful physical exam and performing some tests to investigate for the possible underlying causes: - a chest X ray study- a cardiac ultrasound to examine your heart function and structure- an ambulatory 24-48 hours ECG monitoring to examine your heart rhythm trends during a prolonged time- complete blood count for anemia- thyroid hormone levels for possible dysfunction- kidney and liver function tests for possible dysfunction- blood electrolytes for possible imbalance. I would also recommend adding an ARB (valsartan, olmesartan, etc.) to your actual therapy in order to have a better control of high blood pressure. You should discuss with your doctor on the above issues. Hope you will find this answer helpful!Best wishes, Dr. Iliri"
},
{
"id": 15845,
"tgt": "Scaly rash on the ear. Getting cured upon using neosporin. Not cured completely",
"src": "Patient: Hi I have a scaly sort of rash along the scapha of my left ear. It is not really itchy but when I scratched it without knowledge, it resulted in it getting sore. But after one application of neosporin, it has dried out. But the scaliness has not gone...do I need further check? I travelled long distance in an aircraft just before I discovered this. Doctor: Hello.Seem to suffer from ear eczema.For treatment is suitable aplication local corticosteroid(low power) cream and take antihistimine oral(to treat itching and avoid scratching).Quiet.Is not important pathology but must go to your doctor if eczema dont improve in a week.I wish you good health.(In case the answer would have been useful please indicate this)"
},
{
"id": 166729,
"tgt": "Suggest remedy for low grade fever with green mucus in infants",
"src": "Patient: My daughter has had a low grade fever (99.5) for 3 days and has thick green mucous which has been going on for weeks. She seems miserable. Is there anything more I can do for her at home or should I take her in again? She is 3 and has already been through one round of antibiotics, which cleared temporarily, but returned with a vengeance. Doctor: Hello,You should give her acetaminophen or ibuprofen syrup every eight hours to relieve the fever. The green mucus may be caused by a viral infection that won't require an antibiotic. But if the symptoms persist more than five days, you might need to add an antibiotic.Hope I have answered your query. Let me know if I can assist you further.Regards,Dr. Salah Saad Shoman"
},
{
"id": 104289,
"tgt": "Wheezing problem since childhood. Had betamethasone. Difficulty in breathing. Remedy?",
"src": "Patient: Hello doctor, I am having wheezing problem from child. My mother also having same problem.. that time we mostly take betamethasone tablets. After, I came to know steroids are not good. So I go to salbutamoland theophylline combination. Now I am feeling diificult in breathe. I feel it is due to gas. Its coming daily..morning, evening, after eating in night. What is my problem . Tell me remedies ..how to change my daily activities, food.. Doctor: Hi Asthma attack can be triggered by pollutants, upper respiratory infections, weather change, stress, allergens, so avoid these triggering agents. and life style modification is must so follow the suggestions strictly Drink plenty of water Boost your immunity by proper diet like having lots of green leafy vegetables and fresh vegetables. Avoid junk spicy foods and carbonated drinks Learn yoga asanas , pranayama and meditation techniques to improve your Lund capacity and for the good. Blood circulation to the chest. Go for chest physio. Avoid exercises Control your stress level by practising regular yoga. Add pepper , honey, ginger and garlic in your diet. Avoid milk and milk products. Take hot foot bath before you sleep for 15 mins. Have good sleep. To avoid excess gas formation do pavanamuktasana to release the excess gas and change your diet as mentioned above take less amount of dhals avoid cabbage, cauliflower, Peas. Drink minty leaf juice added with jeers which will be very effective for your gastric complaints. Hope this answers your query . Take care wish you speedy recovery. Bye"
},
{
"id": 170179,
"tgt": "Does nal1 baby feed causes redness and swelling on face?",
"src": "Patient: HI I am only breatfeeding my baby.He has entered his 5th month yesterday and I gave him 3 ounces milk in a bottle made of Nan1 baby feed.In one hour his whole face and body became red and swollen up.I have given him drops my docter suggested which has made the redness subside.My query is that did nan 1 cause this reaction and should I try and give him some other brand? Doctor: Hi...I went through the history points you have provided - I was expecting that there should have been a history of cow's milk and formula feed (this too contains cow's milk protein) given to the baby. I feel that your baby is having cow's milk protein allergy. Usually babies grow out of this sort of allergy by 1 year of age. My suggestions for you - 1. Mother should go off cow's milk protein completely. This means that you should avoid consumption of anything and everything related to cow's milk - like - milk/ curds/ ghee/ butter milk/ chocolates/ biscuits/ ice creams etc. Even while buying commercial food products, you need to see the ingredients and if they contain milk - do not consume them.2. Feed you baby only exclusive breast feeds till 6 months of age and then start rice based feeds. 3. If this is followed scrupulously - the baby will stop having diarrhoea in another 3-4 days and then start gaining weight too.4. If you feel your feeds alone are inadequate for the baby, then Zerolac is the only option till 6 months age. I request you to keep me posted about the recovery of the baby and follow of the case.Regards \u2013 Dr. Sumanth"
},
{
"id": 180649,
"tgt": "What causes dry mouth and green discoloration on the tongue?",
"src": "Patient: I noticed dry mouth and my tounge is green in center and near back. My stool is green also. Last night I was experiencing problems breathing, waking and gasping a bit, like too much liquid. I'm scared because I was looking online and was reading about parasites? I've been living in a not very sanitary environment. Any ideas or suggestions? Oh I also drank almost a fill bottle of mouth was about 2 days ago(i know, stupid) thank you for ur help. Doctor: Hello,Dry mouth along with a greenish appearance of the tongue can be due to dry mouth or xerostomia as it can be due to causes like:-Severe dehydration. -Side effects of medications if you take any.-Salivary gland dysfunction.-Nerve damage supplying salivary glands.-Oral infections like thrush. -Sjogren's Syndrome etcetera.Drinking mouthwash can cause burning sensation of mouth and food pipe. Greenish appearance can be due to excessive bacterial overgrowth that can be probably chromogenic bacteria.-For dry mouth chew sugar-free chewing gums.-Drink plenty of water.-Avoid regular mouthwash and use a specialized mouthwash that is formulated for dry mouth patients like Biotene.-You can also instill artificial saliva drops.-Suck tart candies as it also stimulates saliva flow.-Clean your tongue using a tongue cleaner.Hope I have answered your query. Let me know if I can assist you further.Regards,Dr. Honey Arora"
},
{
"id": 132955,
"tgt": "What are the chances of recovery from femur fracture?",
"src": "Patient: My sister, 67, recently broke her hip. A total hip replacement was done last week. She has been in a rehab in Florida. Very agitated, very anxious to get out, go home, etc. Not using good common sense, nor is her husband. Well, last night she got up to use the bathroom, without calling the nurse, and of course, fell and fractured her femur. Her husband said this morning she is back in the ER, and possible surgery. She is recovering from pneumonia since she is in late stage COPD. She also has osterporosis, with 100% chance of fracture. She had to be on the ventilator during hip replacement surgery. I imagine that would happen today. What do you think her chances of any type of recovery are? She could possibly not live through this. I live in Omaha, NE and they are in Florida. I am very upset over all of this, now I feel numb to this whole thing. Thank you. Betty Thomas Doctor: hi Bettyhope this msg finds u in good health I am quiet touched to see u care so much for ur sister being living so apart..1st of all..dont lose hope.. science can do wonders..though it seems difficult. its definitely not impossible. .done properly with timely management. .she can still recover and walkShud u need any help.b free to ask me..m always available for uthanks Take care God bless ur sister n i pray for her speedy recovery"
},
{
"id": 112482,
"tgt": "Back pain. On morgan 10, forceful as needed for fibro. Suggest something stronger for back problem",
"src": "Patient: I take morgan 10 3 Times daily and forceful as needed for fibro. My back has been bothering me for two weeks and it has gotten worse. I won't go to emergency room because I signed a pain contact and I don't want to get in trouble. Can I go to my regular Dr and she is she can prescribe something stronger just enough to get over this back problem? Doctor: Hello,Thanks for writing to us,As per your symptoms probability is lumbar disc bulge compressing your spinal nerves leading to pain in your hips and down.MRI spine will confirm diagnosis so I will advise to do that.Muscle relaxant with neurotropic medication will help to relieve symptoms.Till that time Physiotherapy like USG and SWD will give some reliefProper spine stabilising exercises are very important for maintaining pain free state.I hope that I have answered your query. Let me know if I can help you further. Take care."
},
{
"id": 111973,
"tgt": "What is the reason for frequent urination during the nights with lower back pain when tried to control?",
"src": "Patient: during the middle of the night, every night, i have to wake up an urinate. If I try to hold it, I'm awaken with lower back pain. There are some night I do get up and I still have the lower back pains when I wake up. Once I urinate and release myself most of the pain go away right away. My father died of kidney failure, should I be alarmed? Doctor: HiAs you have increase in your urine output in night you need to do urine sugar and complete diabetes checked.Second think you should do kidney function test for kidney status and x ray kidney with urinary bladder for stone in urinary system.Yes of course it may be heridetary condition.If you have dribbing in urine then uroflowmetry test should also be done.Get well soon.Thank you."
},
{
"id": 82371,
"tgt": "What causes severe pain in the chest?",
"src": "Patient: Hello My name is ebony and I m 21 years old . For the past months or so I have been having really bad chest pains ; as if someone is stepping on my chest with a boot on . My heart has irregular beats , one minute it s beating fast next minute it s so slow I can feel it like it s about to stop any minute ; times it feels like my back has sharp needles in it and I can t move ; I ve been to the hospital plenty of times my EKG is different everytime but yet they don t know what s wrong . I m very scared for my life . help please . Doctor: Thanks for your question on HCM.In my opinion you should get done holter monitoring. Holter monitoring is continuous recording of ECG through out the day (24hours recording). So that exact events ee can see. You may have arrhythmia. And holter monitoring is the best tool for evaluation of it.If this is normal than you should go for coronary angiography to rule out coronary artery disease. So better to consult cardiologist and discuss all these."
},
{
"id": 45734,
"tgt": "Suggest remedy for pain in the kidneys",
"src": "Patient: hi my wife has pain in near kidney, her head has pain everytime, she has a pain in her brest i am newly married. some doctor say she has less blood so her head is paining so we started taking injection but no use and for her brest we done many test the doctor we don't have to worry it be clear by medicine but almost year she is taking medicine but no use when she has a pain she cannot breath. & pain in near kidney doctors she may face a problem for pergency she may near become i don't what to do, & my wife is in india & i am in saudi so i need a proper suggest about her food and all. Doctor: Hi, She is having non-specific pain all over the body and it is suggestive of a clinical condition called fibromyalgia. As a first line management, you can try analgesics like Tramadol for pain relief. If symptoms persist, it is better to consult a neurologist and get evaluated. Hope I have answered your query. Let me know if I can assist you further."
},
{
"id": 166791,
"tgt": "What causes white spots and swollen lips in child?",
"src": "Patient: hi. my 18 month son has single white spots on the inner lips and cheeks also has 3 small sore s on his tongue.has a high tem of 39.2 refusing to eat and cant drink no fluids due to his mouth being sore.....did take him to doctors he said it was hand foot and mouth but the white spots in his mouth was not there when doctor checked him over. could you pleas tell me what theses white spots or and the swollen lips many thanks x Doctor: Hi,White spots on the inner side of the lips could be mouth ulcers. This is also quite common in hand foot mouth disease, so it is the same disease process. The other possibility is that they could be aphthous ulcers.Hope I have answered your query. Let me know if I can assist you further. Regards,Dr. Sumanth"
},
{
"id": 21322,
"tgt": "What causes intermittent heart pain?",
"src": "Patient: Hello doctor, In mid-February 2011, had pacemaker/defibrillator placement for low EF @ 35% and irregular beats. For about two weeks I have had intermittent heart pain and pain on my left side at the level of the heart/breast. These pains do not occur at the same time. Could it be the device is lightly shocking? The pains almost feel like spasming. Heart rate and pulse are a.o.k. I am 57 year old female, 5 7 and weigh 175 Doctor: Madam you had undergone ICD and you must be under medications.You have chest pain or feel like shock , then you must consult your physician.To rule out CAD or ice shocks , please get your ECG and device interrogation done as soon as possible.I don't think there is much to worry about.Take Care"
},
{
"id": 146847,
"tgt": "Leg pain, cramps in toe, taking Advil. Help",
"src": "Patient: I was sitting in recliner both legs felt achy took advil back in chair right top of foot started to cramp and draw big toe downward index toe upward painful then left foot did the same and i had no controll to try and pull them forward i could move feet forward from ankles both feet eased up i walked around and now i seem to be okay cramping and drawing lasted 3 - 5 minutes past 2 months or a lil longer i have been having pain in my legs form the knees down sometimes only one leg tonight it was both Doctor: Dear user, thank you for your questions. its very important to note that if the pain is unilateral meaning one leg if there is any calf size increase or skin redness indications of deep vein thrombosis, an ultra sound for lower limb will help your doctor to test for that. if on the other hand the pain is bilateral and shooting from the spine back to your knees you might want to check for disc prolapse. Now we addressed serious situations, there is a high possibility that those muscle cramps is due to increased physical activity on with low water intake, so drink more water might help you. hope this helped.best regards, Dr. Nazzal"
},
{
"id": 162839,
"tgt": "How can diarrhea in an infant be treated?",
"src": "Patient: Hi..my baby is 5 months old only..i really worried about my daughter.. she is suffering from fever and looseotions..after loosemotions she got fever..I dont know what is the cause of fever in her..she suffered fever and loosemotion from 3 days ago..someone consult me about bifilac sachet..should i guve bifilac sachet to my daughter fir loose motions Please let me know ..is bifilac sachet helful to cure loosemotions of my baby girl?? Please answer soon..i really worried about her..her weight is decreasing day by day because of continous loose motions from 3 days?? Plesse reply..i will more happy ..please Doctor: Hello and Welcome to \u2018Ask A Doctor\u2019 service. I have reviewed your query and here is my advice. In such young age common cause of diarrhea is viral gastroenteritis esp rota virus. For that Symptomatic management done. Maintain hydration with lots of water.The prescribed sachet is pro-biotic and can be prescribed I will suggest you to investigate with stool routine micro examination and occult blood test as well. Take care Hope this will help you Consult pediatrician for examination"
},
{
"id": 208358,
"tgt": "How to get rid of headache and anger?",
"src": "Patient: Hi dr. my name is surbhi. I am 25years old, am married. now a days m feeling very tens and feeling headace.when i have headace am feeling very angry, i can't control my self....for long time....after that i feel weakness also.please suggest me....thanks surbhi.. Doctor: Hello Surbhi; thanks for the query. I would also like to know how is your mood most of the time, your energy levels (rate both out of 10); do you enjoy routine pleasurable activities, how is your sleep & appetite?? Too many times, headache, fatigue, difficult to control anger which are of recent onset (over a few days-months) are infact manifestations of depression. Hence you should consult a Psychiatrist or revert back to me for detailed evaluation. Secondly there are many safe, rapidly effective medications available for treatment of depression whcih can take care of your symptoms (including headache, anger). Escitalopram (SSRI class); Dotheipin, amitryptyline (TCA class) are very effective for headache & anger control. Plus certain lifestyle changes like 30 minutes daily exercise, doing daily yoga-pranayam will also help. Good Luck"
},
{
"id": 93717,
"tgt": "Had embryo transfer done, have cramps in abdomen and lower back, cough and cold. What is wrong?",
"src": "Patient: Hi, I have had my embryo transfer done on 21st of dec 12. its almost 10 days and im getting cramps in my stomach and there is pain in my lower back and legs. aslo with this i am suffering from cold and cough . i am not taking any medicines since the doctor has instructed me to avoid medicines apart from the ivf treatment course. now that all these things are happening im getting nervous and stressed out and getting worried about the results :( pls guide me and let me know whether these symptoms are normal or should i be worried??? Doctor: Hi welcome to Health care magic forum. Thanks for calling H.C.M.Forum. Youhave got embryo transfer on 21-12-2012. now you are having cold and cramps in stomach, pain in lower back, and legs. Now you are supposed to be a pregnant of 5 months, in 2 nd trimester you can use many drugs, but as it is an assisted pregnancy, you can use the pregnancy catogiry drugs for other diseases. Now you have got a flu fever and pains due to that infection, I advise your I.V.F. doctor for treatment, You can have treatment with pregnancy catogiry drugs under the supervision of your doctor, with safty to the foetus. Wishing for a quick and comfertable recovery. Best regards."
},
{
"id": 74159,
"tgt": "What causes difficulty in swallowing food and water after having chest pain?",
"src": "Patient: i got mild chest pain in left side 2 days before. now i feel little difficulty while sollowing food, drinking water, even while sneezing. I have acidity problem only while morining brush [vomit feeling] from last 6 months and have takesn no treatment for the same. Would like to get your opition for the same. Doctor: Hello, you mentioned you have difficulty swallowing solids and liquids . Is it really only for past 2 days or u had difficulty even before that ? Do u really feel that your food gets stuck when it finds it's way down your throat or food pipe and isn't going down properly as it used to ? Is it like initially you had difficulty swallowing solids and then it progressed to difficulty in swallowing liquids as well or it happened for both solids and liquids at once ?Did you have any episodes of vomiting ? Is there pain during swallowing or only difficulty ? What's your age? Any addiction to smoking or alcohol ? Do you have any other respiratory complaint apart from the chest pain you mentioned ? Your symptoms need detailed evaluation . Difficulty in swallowing can be due to issues with your throat or food pipe or something compressing on your food pipe from outside (example - enlargement of lymph nodes around your food pipe or any other growth ) or it can be due to motility disorders of your food pipe (Achalasia cardia) . There are many possibilities so I would suggest you consult a physician in person ."
},
{
"id": 61444,
"tgt": "What causes a lump in the upper left quadrant of the abdomen?",
"src": "Patient: ihave swelling or a lump in myupper left abdomen right under the ribs - it goes from about middle of abdomen across to my side. no pain on palpation, and have also had pain in my left ribs on my back. not constipated, but feel really full after eating a small meal. no temp. or other pains Doctor: Respected user, hi I evaluated your query thoroughly .* As per the narration it seems to be pathology related to stomach , pancreas, spleen as tumors of stomach , pseudopancreatic cyst, splenic cyst, splenic tumors & others .* You require Ultrasound of abdomen & pelvis as basic necessity to find the root cause of worry & further management plans accordingly .Hope this helps you .Welcome for any further assistance .Thanks for using Healthcaremagic.com & giving us an opportunity to assistWishing you fine recovery from the same.Regards dear take care."
},
{
"id": 113882,
"tgt": "How can I get relief from back pain ?",
"src": "Patient: null Doctor: Hello.Welcome.I think you will have to undergo surgery only.Consult a orthopedic surgeon and seek his opinion.No other pathy is going to give you relief.Good luck."
},
{
"id": 86972,
"tgt": "What causes abdominal pain after intercourse?",
"src": "Patient: Hi, I have a pain in lower left abdomen below my naval along with intestinal gas noises around the same location now for three weeks. Two weeks ago I went to a urologist and put me on 1 month of Cipro assuming it is prostatitis but I'm still having having the pain. It seemed to start after protected sex, but was tested for stds (all negative). Is there anything else it could be? Doctor: Hi.Thanks for your query.The pain in the abdomen below the naval with intestinal gas noises can obviously are due to an intestinal infection. The Cipro should have helped you in spite of it was given for Prostatitis.This indicates the need for further evaluation by:- Clinical examination of the abdomen by physical examination, per-rectal examination for prostate, -X-ray of the abdomen in standing position.-Ultrasonography of the abdomen, pelvis, per-rectal also for genito-urinary system.-Routine tests of blood, urine and stool. Once the proper diagnosis has been made, the treatment will be guided. Take care to have plenty of oral fluids, avoid outside foods and beverages."
},
{
"id": 54811,
"tgt": "Suggest treatment for enlarged liver",
"src": "Patient: docter,i am 24 years old.i had got heppatities B when i was 14 years old.at that time my hbsag result was +ve and it was made -ve after 6 month.a few days ago i had taken the ugs of abdomen,then in that it was shown that i had heppatomegaly with grade 2 fatty changes.is there any treatment for this? Doctor: Hi thanks for asking question....Welcome to HCM...Noted you had infection with hepatitis b ..Now you have negative HBSag ...So you recovered fully....Now for fatty liver life style changes needed to resolve it...Take low fat diet...Trans fat food like non veg, baked pestry , butter , cheese taken very less...Avoid alcohol.Regular exercise needed.Refined food avoided.Use very much less oil in diet....Take udiliv 300 mg tablet daily...If developed jaundice contact doctor...Happy to help you further.....Dr.Parth Goswami"
},
{
"id": 111854,
"tgt": "What do you suggest for the pain in my back when i eat with the history of hiatal hernia?",
"src": "Patient: Hello. I was diagnosed with a mild hiatal hernia years ago with an upper GI. Over the past several months I ve had more pain in my chest and shoulder and back when I eat. Worse by night. I am on pantalaprozole and zantac. Still have same symptoms. I just had tests for my gall bladder which were normal. So now my Dr is finally sending me to a specialist, but not for 2 months. I m suspecting a paraesophageal hiatal hernia bc of my symptoms. Everything I eat causes pain in my back, & my food feels stuck, sometimes with nausea. I m worried about waiting so long to see the specialist. How do I know if I should go to the ER? And what can I eat or what should I avoid doing in the meantime? Doctor: Hello. Thanks for writing to us. The pain in the back that you have is due to hiatus hernia. It is best to have small frequent meals and avoid lying down after the meals. You can take antacid and oneprazole for symptomatic relief.I hope this information has been both informative and helpful for you. Regards, Dr. Praveen Tayal ,drtayal72@gmail.com"
},
{
"id": 124558,
"tgt": "What could purple, burning knot behind knee indicate?",
"src": "Patient: I have a purple knot/bump behind my knee sorta hurts/burns felt like a bug bite when I first noticed it. Is more purple and noticable when I extend my leg. Just in case fyi...I was recently around someone who just found out they have mrsa but the bump I have isn t infected looking Doctor: Hello, It could be bursitis or hematoma. Consult an orthopaedician and get evaluated. If there is diagnostic confusion, an MRI scan is required. Hope I have answered your query. Let me know if I can assist you further. Take care Regards, Dr Shinas Hussain, General & Family Physician"
},
{
"id": 127352,
"tgt": "How to strengthen the leg muscles?",
"src": "Patient: For 18 my right leg from knee to foot hurts. It does not hurt to press on the leg. It especially hurts when i drive and thefoot presses on the gaspedal. Or on thebrake. In crampedback seat or airplane too. Also hurts wben I walk up hill. Extra exercisedoes not help a either does extra stretches. What really decreased the pain to two ( before at 7 for pain) Was when I stopped driving. I do not wish to give up driving.. But i can do that..but it does nothing for strengthening.....or increase My mobility. My doctor had suggested walking 3 miles a day and Stretching. That is not helping and I believe that this , leg is inflamed. Maybe,tendinitis or bursitis.?..? I do not knowenough to tell tbe Difference... maybe arthritis is also i nvolved. I am interested in how To strengthen and specific foods that might help. (What foods to avoid) Sincerely. Ria de Groot. 73 yrs old and not on any medicine Doctor: Hello and Welcome to \u2018Ask A Doctor\u2019 service. I have reviewed your query and here is my advice. In order to strengthen your foot muscles u need to do muscle specific exercises like calf raises,assisted squats.also concentrate on diet by taking foods rich in protein, minerals like milk,eggs,fish,meat , Leafy vegetables.This will strengthen your legs and provide you relief. Hope I have answered your query. Let me know if I can assist you further."
},
{
"id": 105634,
"tgt": "Arms and legs waxed, allergy. Can use tenovate-m?",
"src": "Patient: Hi, I got my arms and legs waxed and the next day I had an allergy on my arms. I am not to sure if this allergy is due to the waxing or cuz of the woolen jacket which I wore on d same day....? Any recommendations I am allergic to betnovate ointment..... Is it alrite to use tenovate-m Doctor: Hello Khangura, It is most likely secondary to procedure and you must ensure you get a patch test to exclude chemical sensitivities. I presume the 'allergy' was also on the legs. If you had a similar reaction to waxing before, then it is even more important to exclude the chemical sensitivities before you go through another waxing procedure. Allergies to steroid creams are possible and this can also be checked when you have the patch test. For the time being, antihistamine tablets such as cetirizine 10mg twice daily or allegra 180mg once daily with Calamine lotion on arms and legs should be enough to control the reaction. I hope this was useful."
},
{
"id": 18324,
"tgt": "Is Cardizem safe to be taken while on Phenobarbital and Dilantin?",
"src": "Patient: I am presently taking phenobarbital and dilantin to control seizures. Newly prescribed blood pressure medications (Diovan, then Cardizem) both seem to react negatively and leave me weak and wobbly. Seizure control is old (prescribed back in the 1960s). What would be a more up-to-date regimen? Doctor: Hello Welcome to Ask a Doctor service I have reviewed your query and here is my advice.It is not the drug causing you anything but it is just your doubt and if this is so then you can discuss this with your physician but it is not the drug causing the anything nonspecific you are thinking, it seems to be the functional factors playing the role behind the symptoms you are having.Hope I have answered your query, let me know for further assistance."
},
{
"id": 174054,
"tgt": "Is headache, bruised and swollen eyelid in a child after banging head on dolls house concerning?",
"src": "Patient: My little girl aged 7 has just tripped and banged her head on a dolls house. She has a headache and her eyelid is bruised and swollen. She s had an ice pack on for 20 mins and it is still swollen but she has go e to bed now and is asleep. Is there anything I should be worried about? Thank you Doctor: HI AS YOUR CHILD HAD AN HEAD BANGING, I REQUEST YOU TO BE VIGILANT FOR 72 HOURS TO LOOK FOR ANY VOMITING, ALTERED SENSORIUM, CHANGE IN BEHAVIOUR, SEIZURES AND INCREASE IN THE SIZE OF BRUISE/ SWELLING. IF any of these are present, consult a child specialist immediately."
},
{
"id": 35463,
"tgt": "What is the treatment for fungal infection in the groin?",
"src": "Patient: I am 66 years old male. 75 kgs, 5'10\" in height. I have been having a chronic fungal infection in the groin are for many years with no permanent relief. I have severe itching specially in the nights. In spite of innumerable courses of treatments. LatestI have prescribed Eumolene -M ointment, Zimig 250mg once a day, Triben dusting powder during the day and Teczine in the night. Please advise if this treatment is alright. Also is this contagious ?Please advise. Doctor: Hello,Welcome to HCM,Closely opposing body folds like upper inner thighs, groin can trap sweat, heat and moisture, therefore making the area favorable for fungal proliferation.If I was the treating doctor I would have asked you to use a topical antifungal like clotrimazole twice daily, regularly for 4 weeks. One such cream that you can use is Candid-B cream, twice daily for 4 weeks. For the thigh and groin area.I would also suggest an oral antifungal e.g fluconazole 150mg once weekly for 4-6 weeks for a more complete cureAn OTC antihistamine e.g cetrizine 10mg once daily would help you in symptomatic itch relief.An Antifungal dusting powder e.g clotrimazole dusting powder can also be used throughout the summers, it would not only keep the folds dry but also inhibits the growth of fungus thus preventing recurrence.Thank you."
},
{
"id": 208520,
"tgt": "How to treat the condition of severe aggression?",
"src": "Patient: i have been on nebido now for 9 months and i feel it is not agreeing with me. the main problem i am having is severe aggression. i have gone through counsiling and hypnotheropy and nothing has worked. i have never had an issue with aggression until i started on hrt treatment. dont know what to do or where to turn Doctor: DearWe understand your concernsI went through your details. I suggest you not to worry much. Counseling and hypnotherapy or any other treatment for that matter, may not work until and unless you try wholeheartedly. You must understand that aggression alone cannot be a mental disease condition. It is just a state of mind. Aggression is actually a symptom of mental conditions. If you only have aggression, then that can be cleared with just mental resolve. If there is any other underlying mental diseases, you need psychiatrist's diagnosis and involvement. If you require more of my help in this aspect, Please post a direct question to me in this website. Make sure that you include every minute details possible. I shall prescribe the needed psychotherapy techniques which should help you cure your condition further.Hope this answers your query. Available for further clarifications.Good luck."
},
{
"id": 207449,
"tgt": "Suggest treatment for frequent mood swing",
"src": "Patient: Hi, may I answer your health queries right now ? i am constantly having mood swings . i get angry on the smallest issue and tend to overreact. this is affecting me physically and mentally. i even go to the extent of hurting myself or someone else. please help. Doctor: hi, thanks for writing.. i have understood your concerns..frequent mood swings can be a part of bipolar disorder [mania and depression], reaction to stress, personality problem [borderline personality disorder].usually bipolar mania patient do not have insight for their illness.so you must be suffering from bipolar depression but you shud go for your personality evaluation also because frequent and rapid mood changes with tendency to harm oneself is more going towards personality issues..mood swings reactive to life events is also a different kind of depression..you will get all your answers when you consult a psychiatrist..treatment is different according to diagnosis for which you have to visit a doctor..i think this info will be helpful to you..best of luck.."
},
{
"id": 6820,
"tgt": "Is it safe to take injections to induce ovulation ?",
"src": "Patient: Hi, I am Mona, I just want to check that whether Injection is harmful for have ovaluation or not Pl. advise. Whether Injections are safe for getting Ovulation or not pl. advise? Doctor: Hi, Please mention which injection?"
},
{
"id": 154755,
"tgt": "Could dehydration and anxiety be due to chemo treatments?",
"src": "Patient: My husband has been under chemo treatments for 6 months now after colon cancer was found after obstruction and removal of colon and in the last 2 weeks he has not ate well been having trouble with dehydration and anxiety. The last 3 days are of him non-stop talking, from things he wants to do to god knows what. Not a person can get a word in and if you try he will walk away and get sick.... I told him it was like dealing with a spoiled rude baby... he just kept on talking about the sewer system. Is this a sign the cancer has spread to his brain - He said he will not go back to the Dr. cause the chemo is the problem. Doctor: Hi,Thanks for writing in.Cancer is a difficult condition to treat and depends on the individual case. Your husband had a surgery followed by chemotherapy that has been on for last 6 months. It is possible that due to side effects of chemotherapy he is unable to have a good meal because of lack of appetite. Along with inability to eat there is also decreased liquid and water intake in many people on treatment with chemotherapy. In this situation you must see that he has an adequate fluid intake of 2.5 liters every day to keep his hydration normal. Since a colon cancer can cause liver spread in half of the patients therefore complete follow up is required and ultrasound scans and CT scan might help.Regarding his talking irrelevantly, it is important to know if his serum electrolytes are normal as you have said about his possibility of dehydration. Please get his sodium, chloride, calcium and potassium levels checked. He also has little amount of anxiety that is present due to continuous treatment and not being able to lead a normal life. A spread to brain is less likely if you think about the way he is behaving. If he continues to behave this way and his electrolytes are normal then consult a neurologist. Please do not worry."
},
{
"id": 204776,
"tgt": "How can insomnia along with anxiety be treated?",
"src": "Patient: I have been insomnia since November. It started due to a lot of anxiety I was having at the time. Now it is months later and I am still experiencing it. I don t have trouble falling asleep, I have trouble staying asleep. For the most part, the anxiety is no longer there. Any advice you could offer would be most appreciated. Thank you. Doctor: Hello! Welcome to HealthcareMagic! I have gone through your question and understand your concerns. Presently, your suffering with generalized anxiety disorder(GAD) with associated sleep disturbances. I recommend you to start an Anxiolytic preferably an SSRI. Along with a Benzodiazepine for overcoming sleep disturbances. I also recommend you to start practicing stress management techniques such as: 1) Regular physical exercise in the morning. 2) Healthy lifestyle modifications. 3) Relaxation techniques like mindfulness and meditation. Hope this clarifies your concerns. Thank you for the query. Take care Regards, Dr. Bhagawan Rajana, Psychiatrist"
},
{
"id": 163153,
"tgt": "Suggest remedy for soreness in stomach",
"src": "Patient: my 3 year old has been compaining for about 3 months that his stomach is squeezing - he shows me at his belly button. He has the flu now and has been throwing up, he doesnt want to eat much and he only wants to drink water. Juice makes his stomach squeeze. His doctor has palpated his stomach and sais its in his head - should I get an ultasound - or a second doctor? Doctor: Hello and Welcome to \u2018Ask A Doctor\u2019 service.I have reviewed your query and here is my advice.The baby will explain it in his own terms but it doesn't mean he is being functional or lame. Seems like he is having acidity and that is causing vomiting. Start syrup Polypep thrice a day with syrup Gaviscon. Avoid sour and spicy food and do not keep him empty stomach. I hope I have answered your query. Let me know if you have any further questions. Regards, Dr. Hina Javed"
},
{
"id": 28052,
"tgt": "Suggest treatment for high BP",
"src": "Patient: I am fifty years of age and marred for last twenty years I am patient of high blood pressure for the past twenty years I am taking medicine Amlovas 5mg in morning and Losacar 50mg in the evening. I am having a sex problem from last two years I am not having proper erection if once in while if it happens I will discharge with in one minutes All my blood reports are normal .Kindly suggest me what medicine can I take with my BP medicine to have a normal sex life. Doctor: 1. There is a wide variety of anti hypertensive drugs available today and the choice will depens upon:. The tolerance of the patient to the drug,i.e. how the patient responds and the side effects. . The individual choice of the doctor. . The cost of the therapy - what the patient can afford.2. In middle aged hypertensive patient, beta blockers are the first line drugs but side effect include: dry cough, supress libido ( erectile dysfunction), broncho spasm. 3. If your blood pressure is not controlled with the highest dose of drug, combine either with a calcium channel blocker or an ACE inhibitor. 4. Since you have a long history of high blood pressure, thus:. Check with Lipid profile. Stress test and if necessary coronary angiography. . Check urine for micro albuminuria."
},
{
"id": 175098,
"tgt": "Suggest remedy for cold cough & loss of appetite",
"src": "Patient: hello. my 2 n half year old son started taking dexpoten 4 ml morning n evening for 2 days with the complain of cough n cold and runny nose. last nite he had a disturbed sleep and stooped having anythg from the morning, not even milk.wht shud i do now? Doctor: Thanks for asking on HealthcareMagic.Such symptoms in children of this age is usually caused due to certain viral infections which can last for 5-10 days. Dexpoten contains dexmethorphan which is a cough suppressant. Rather, an expectorant should be given instead.Sudden stoppage of feeding in a child is indicative of an infection and you should take the child to a pediatrician without delay and get him assessed. The doctor would be able to examine him and prescribe appropriate medicines including antibiotics if necessary."
},
{
"id": 92973,
"tgt": "Lump near belly button, extremely bloated stomach, vomiting sensation. Is it Hernia?",
"src": "Patient: I have a lump about 2 inches above my belly button. Dr's can't seem to decide if it is a hernia or a fatty deposit. It has progressively been getting larger. Started as a pea size a few years ago and is now visible, bulging and about 11/2 inches in diameter. I have had ultra sounds and am not getting any answers. 6 months ago I ended up in emergency with the pain worse than it had ever been. My stomach and the area where the lump is was extremely bloated out, felt like I needed to push my stomach out to get relief. That is happening more often, at least once a week but now I start to sweat, salivate like I'm going to vomit and I faint. I hope you can give me some direction. Doctor: Hi,This can be very well decided by physical examination only.If bulging increasing while standing and subsides while lying down, it can be umbilical hernia.While coughing or increasing abdominal pressure if bulging incrasing then also it can be umbilical hernia.Otherwise there is possibility of having Lipoma.Consult surgeon and get examined.ok and take care."
},
{
"id": 42161,
"tgt": "Should i go for DNC test regarding infertility problem?",
"src": "Patient: Hi I m supreetha here from past 1 1/2 year i m trying to become pregnant but every time its not happeining we went to doctor also i hve completed till tubical block test. so doctor has advised me to go for DNC test so please tell me weather should i hve to go for tht test r can i wait for few more days Doctor: Hi Supreetha,Welcome to HCM,There are certain indications for D & C in infertility cases. Your doctor may want to rule out certain uterine lining abnormalities. Your doctor may take the sample of uterine lining and to send to laboratory for tests. Also sometimes if there are small polyps, it can be removed during D & C. We decide to do D& C depending on your age, menstrual history, and many other factors. It is better you discuss openly with your doctor for his reasons of doing D& C.I have tried to give my answer on little information you have given. if you need further details, you may provide all details.Dr H Hamdani"
},
{
"id": 59131,
"tgt": "Diagnosed with mild fatty liver, blood urea. Have pain in soles and around lower extremities. Due to fatty liver or gastric problem?",
"src": "Patient: I have been diagnosed with mild fatty liver and my blood urea and critnine is marginally low. I am not alchoholic but I am on cholestrol controlling medicine. I get swelling near toes with pitting and also have pain in soles and around lower extremities. Is there any connection . Is heaviness on right side under the ribs is on account of fatty liver or gastric problem. I don t normally pass gas that often. Doctor: Hello, chopramukesh14, If your blood urea and creatinine levels are a little low , that is good. You don't want hem to be high which would indicate kidney damage. I am glad that you are being treated for elevated cholesterol. Most of the time this may be associated with overweight problem and also diabetes. Make sure you are checked for this through two hour post meal blood sugar level and also HbA1C levels which indicate how your sugar has been in the last three months or longer. Swelling of the feet and toes is indicative of some fluid retention. Make sure that you cut back on your salt intake and you can use salt substitutes that are available or low sodium variety. I want you to get a repeat sonogram of the liver and get all the liver enzyme tests to see if there are any signs of cirrhosis beginning. It is very important to rule this out. If there is any question , the doctor might decide to do a liver biopsy to correctly assess the level of inflammation or scarring. I also hope that you are on Vitamin-E 800 i.u. daily along with a B-Complex Vitamin. You should be on a low fat diet. Hope this gives you some information that you can discuss with your doctor. I wish you the very best."
},
{
"id": 61026,
"tgt": "Are welts normal after being bitten by chiggers?",
"src": "Patient: I was bit by chiggers and believed there may be some that didn\u2019t bite and are alive in my car that I live in so I went to the mountains over night and it got down to 20 degrees are they dead I left the windows down too. I have had a sore throat since then is there anything I should be worried about. And I have developed new welts several days after since seeing the first welts is that normal . And if I do continue to get a few bites is really any harm that can come to me assuming I don\u2019t get an infection from scratching Doctor: Hello dearWarm welcome to Healthcaremagic.comI have evaluated your query in details .* Usually minor bites are taken care by body`s immune mechanics .* Primary guidelines for better outcome- keep the part clean with antiseptic liquid .- avoid tight fitting shoes or further exposure to leather for sometime .- avoid scratching till possible .- consult your doctor if any area of redness , tenderness appears on the body .Hope this clears your doubt .Wishing you fine recovery .Welcome for any further assistance .Regards take care .Dr. Bhagyesh ( MS , FMAS - consultant surgeon )"
},
{
"id": 23039,
"tgt": "What can cause chest pain and numbness in arms?",
"src": "Patient: I am 25 years old and have high blood pressure and recently had an angiogram done and everything was good but. Now 2 months later i am having chest pain on my left side and middle of the chest and some tingling and numbness in my arms and fingers, both sides. I am taking 100mg of toporal and 2.5mg of amlodipine twice a day along with 1mg of xanax when needed for anxiety. what should i do? Thanks Doctor: hi, likely you cervical problems which is pressing your nerves leading to pain , tingling and numbness. so you should get x-ray cervical spine ap and lateral view.do u have any history of trauma to neck?After confirming a diagnosis we can Start on medicine as well as neck exercises.Till then nerve vitamin tab like neurobion forte can be taken once a day."
},
{
"id": 9209,
"tgt": "Suggest remedy for dry skin",
"src": "Patient: 19, 5'6, 128lb not too much medical history. I got a yeast infection and have been treating it with 7 day Monistat. It seems to have gotten better for the most part, but my skin is really dry now, to a point where if I am not careful I'll end up splitting the skin and making myself bleed. What can I put on it to make it less dry? Doctor: My DearWelcome to the site!I have noted your concerns. The dryness in the area that has been treated for a yeast infection signifies that the superficial dead layers of skin, that were infected at one time, are now shedding to permit the new skin to come up in its place. If the itching and soreness are controlled, then in such a situation, one may switch over from cream preparation to an ointment containing the active ingredient. This change would normally take care of the dryness effect.I hope you will get well soon. Our best wishes!"
},
{
"id": 222858,
"tgt": "Suggest treatment for uterine contractions during pregnancy",
"src": "Patient: i m 37 weeks pregnant. i have low fluids. i have two doctors appointments a week to monitor the fluid. this past week i was in the hospital due to problems at both appointments. tuesday i think was cause her heart rate was staying around the 130s.. thursday i think was cause her heart rate was in the 170s, wouldn t come down, and jumped to 195 for a second and came back to the 170s. they were going to do a c section thursday but then sent me home. my doctor still hasn t told me what has been going on, but it has me concerned. i ve been having contractions since wednesday night.. so for about three days now.. last night they were getting bad and i was having 3-4 in ten minutes, every ten minutes, for an hour. i decided to call the hospital and ask the RN if i should come in. she told me to take tylenol and wait an hour and see if they go away... well they didn t, but it was 3 in the morning, and i fell asleep. i m still having contractions today, but they re still kind of irregular. i m extremely extremely swollen.. legs ankles feet hands wrists and my face a little. i went to the bathroom about ten minutes ago, and after i was done before i could do anything it was like i went again, only i didn t... and it was clear fluid or whatever it was.. then it dripped for a minute or so and i got up and i ve been sitting on my bed and it hasn t really done a whole lot except make my underwear a somewhat damp.. what do you suggest i do? Doctor: Hello dear,I understand your concern.In my opinion the swelling of feet,ankles and body in pregnancy needs to rule out high BP,preeclampsia,anaemia etc.The normal fetal heart rate ranges between 110-150 beats per minute.As there is low liquor,along with high heart rate of fetus the signs of fetal distress should be checked and delivery can be planned basing on that.As you are term there is no problem of preterm delivery.Dont worry.Kindly discuss with your doctor regarding high BP and anaemia.Also get examination done to see for any leaking liquor.Best regards...."
},
{
"id": 195243,
"tgt": "What causes brown spot on the penis?",
"src": "Patient: Last night I noticed an irregular-shaped brown spot on the head of my penis. It does not appear to be raised and is painless. I am sixteen and not sexually active although I frequently simulate sex while clothed with my partner. I also regularly masturbate and it was while doing so that I noticed the spot. I am worried as to what this is and if it will go away. Doctor: Hello and Welcome to \u2018Ask A Doctor\u2019 service. I have reviewed your query and here is my advice.You might be suffering from Zoon's balanitis or Plasma cell balanitis which occurs due to irritation of foreskin over glans penis. Apply Fusidic acid cream for 2 weeks. If you do not get any improvement or if you are having other symptoms such as urethral discharge or genital ulcer, immediately consult your Dermatologist/Venereologist.Hope I have answered your query. Let me know if I can assist you further."
},
{
"id": 178787,
"tgt": "Suggest cure for cough & nausea",
"src": "Patient: Hi. My baby is 40 days old and has recently acquired coughing and ocassional vomiting. This has been going on for the past three days. He coughs quite loudly and almost every hour. The doctor said the infection could be viral and did not advise any medicines as he is too young for any cough syrup. He has not developed any fever. Needless to say, we are feeling very helpless to watch him in this state of trouble. Is it true that medicines do not exist for babies at this age? Will this be gone on its own like common cold or flu? Thank you Doctor: Hi thank you for choosing healthcare magic. It is true that there is no effective cough medicine for this age group, but as it is persisting we have to rule out bacterial infection cause like pertussis as this also causes cough in this age group. A simple blood test will differentiate viral and bacterial cause. After this even if we give antibiotic the cough will persist for another few weeks. You can give her steam inhalation or normal saline nebulisation as they decrease the irritation in throat and decrease the intensity of cough.Hope this has been helpful to you, kindly feel free to contact again for any queries. Kindly rate the answer if so. Take care. Dr J Sravanthi"
},
{
"id": 45463,
"tgt": "cycle lenth 38 days?",
"src": "Patient: My cycle length is 38 days..Is it normal to have cycle of 38 day??When I will ovulate if I have 38 days cycle Doctor: Hi.. Cycle length can vary between 21 to 40 days. As long as the cycles are happening regularly and the menstrual bleeding is not abnormal your periods are normal.. In a 38days, regular cycles ovulation would occur any where on your 22nd to 26th days of your cycle.."
},
{
"id": 189022,
"tgt": "Whitish tongue, fever, puffy gums. Rinsed with hydrogen peroxide. what else to do?",
"src": "Patient: I had a fever thursday, May 23 and slept from 6 pm until friday at 9am. then basicallydid not eat on friday. bufferin brought the fever down along with a cold compress. Now my tongue is white and a little painful, but the gums above the upper teeth are puffky and whitish. I have reinsed with hy peroxide 3%. also tookcaugh medicind and the caugh is much better -- not entirely gone. Doctor: this may be the acute exacerbation of chronic generalized periodontitis. this may be associated with other systemic condition. And hydrogen peroxide rinse will not help you in this condition. so better to take advice of a periodontist."
},
{
"id": 197088,
"tgt": "Does pain around penis indicate an STD?",
"src": "Patient: I had unprotected sex with my girlfriend a couple months ago and i had a cut on my penis, immediately i felt a stinging pain and pulled out right away, since then i got a variocele as well as frequent pain in and around my penis as well as in my testicles, could this be some sort of STI, STD or any infection? Doctor: HelloThanks for query .Pain in penis and testicles without any urinary symptoms or urethral discharge is unlikely to be due to STD .In view of unprotected sex with your girl friend .If you are not sure of her STD or HIV status I would suggest you to get basic screening test for STD and HIV done to be rest assured about acquiring STD.In the mean while take antibiotics like Doxicycline and anti inflammator medication Diclofenac twice daily for a week .Dr.Patil."
},
{
"id": 146522,
"tgt": "What is the treatment for degenerative disc disease?",
"src": "Patient: MRI Lumbar spine w\\wo contrast on 6/11/14: Lumbar Spine; L5-S1 moderate to severe facet arthropathy worst leftward. Degenerative disc disease with mild disc bulge and eburnation, abuts but does not efface ventral thecal sac; contributed to moderate foraminal narrowing bilaterally. Nerve roots have a somewhat peripheral appearance within the thecal sac; findings suggestive of arachnoiditis. Have had two spinal surgeries already. Laminectomy at L2-L3; L3-L4; L4-L5; fibrosis at multiple levels. Kyphotic curvature of lumbar spine T12 thru L3. T10-11 disc bulge; spondyloistheses also present. More surgery??? Doctor: Hi, I had gone through your question and understand your concerns. In my opinion, if there's evidence of arachnoiditis, the treatment should be targeted on pain and symptoms relief, with NSAID drugs and corticosteroids, rehabilitational therapy. There is no evidence that surgery is of benefit in case of arachnoiditis. Hope this answers your question. If you have additional questions or follow up questions then please do not hesitate in writing to us. I will be happy to answer your questions."
},
{
"id": 5622,
"tgt": "Trying to conceive. Have bilateral ovarian endometriomas. Taking duphaston. Help?",
"src": "Patient: Hi im 30 years old female just got married and planning to conceive, when to a gyne to check my fertility status, had an transvi ultrasound and found out that i have prominent and echogenic endometrial stripe of 1.6 cm and have bilateral ovarian endometriomas of less than 3 cm in size ( 3 on the right ovary and 1 on the left ovary). My gyne advised me for fractional curretage and i take duphaston for 3 months, I followed her advise, then went back for follow up last week, I did an ultrasound again and found out that there is an interval increase in the sizes of the ovarian endometriomas but still less than 3cm, but the endometrial stripe thickening is resolved, now measures 0.8cm. My gynecologist still advised me for a fractional curettage , but i am doubtful for the procedure, is ist advisable for me to do the said procedure? she still advised me to take duphaston for 21 days from day 5 of my menstruation Doctor: Hi,Welcome to Healthcare Magic.In young women ( who are having monthly bleeds) endometrial thickness vary throughout the month. It is thin after the periods & thick just before the periods. So, best time for you to have a trans-vaginal scan is just after your periods. If thickness is normal at this time you certainly don't need any fractional curettage. As, this test is not usually recommended in women less than 40 years of age.If endometrium is very thick & irregular even after periods, you have irregular bleeding or inter-menstrual bleeding you may need a hysteroscopy +/- endometrial biopsy which can be done in an out patient department. However, this is a general advise. I don't know your case better than your treating doctor. So, I recommend you to take a second opinion from a specialist Gynecologist. Regards"
},
{
"id": 148738,
"tgt": "Tiredness, dizziness, balance problems and sensitivity to noise and light. MRI shows hypersensitivity in right frontal lobe. What is the cure?",
"src": "Patient: Hi I am 38yr old female 59kg 164cm. I was diagnosed with menieres 12 yrs ago but more recently for the past 6 months have had chronic tiredness, 24/7 dizziness and off balance, sensitivity to noise and light and some nausea. All of these symptoms have been a constant everyday. I also just had an MRI and it noted an intermediate rounded focus of hyperintensity in right frontal lobe. Doctor: your symptoms of dizziness and balancing difficulties and sensitivity to noise associated with nausea go in favour of menieres. are you still using any medications or stopped? if not kindly restart the medications. how is your hearing ? have you been to any ENT surgeon / Neurologist recently? do you have any headache associated with it? if so if the headache is associated with sensitivity to noise, light and some nausea it may probably be due to migrane also. do you get relief from headache from these complaints after sleeping for some time? you mentioned that there is a rounded focus of hyperintensity in right frontal lobe on MRI - is it on T1W or T2W image pls clarify.is there any mass effect or midline shift on MRI ? what is the size of lesion?if the size is small and no mass effect and midline shift nothing to worry emergently but i suggest you consult a neurosurgeon as early as possible because this may be due to a solitary lesion in the brain may be abscess / granuloma/ tuberculoma/any glioma/ infarct. but the MRI findings should be correlated clinically. hence i sugesst you to consult a neurosurgeon meanwhile take tablet vertin 16mg BD, cinnarzine 25mg OD, tablet paracetomal (dolo 650mg) thrice a day."
},
{
"id": 33863,
"tgt": "How to cure mosquito bites at the back of the knees?",
"src": "Patient: i have this \"mosquito\" bites on the back of my knees, which was swollen, itchy and red. then it spread to a semi circle shape with liquid oozing out every time after i bathed. the next day it spread to the back of my another knee. i keep discovering new \"bites\" after i wake up and they are now all turning purple. it has even spread to my thighs and feet. i went to my GP and he just gave me antibiotics and a cream to apply. i even had slight fever when i visited him. the medicine does not seem to be helping and its spreading even further. Doctor: HI, thanks for using healthcare magicIf the oral antibiotics are not effective , you may need to see your doctor for reassessment. It is possible that the infection is resistant to it and you may need an alternative.The use of anti inflammatory medication may help with the fever and reduce the inflammation associated with the bites.This includes medication such as ibuprofen, naproxen, cataflam.You also need to wear protective clothing and insect repellent to prevent new bites.Check the surroundings for any collections of water (even a bottle cap can breed mosquitoes).I hope this helps"
},
{
"id": 37250,
"tgt": "What does pus filled bump near the site of pierced ear mean?",
"src": "Patient: I ve had my second ear hole piercings for about three or four months now. Once they were fully healed I decided to change the earrings I had in. Mind you I have pretty sensitive ears. Anyways, I found some earrings that said they were sterling silver and cleaned them and put them in the earring holes. I did this about a month ago and just realized now next to my earring hole there is a small bump. It looks kind of blackish (I m thinking because the earrings I had in were pretty cheap and managed to stain my ears). This morning the bump had pus in it that I got out. I have cleaned the ear hole and bump and applied Neosporin. What is this bump? I really hope it s not a keloid. I hear that keloids don t puss though. Anyways what is this bump? Doctor: Hi, thanks for contacting HCM,Bump is nothing but any swelling on the skin and it may be pus filled or not, it may be red, pink or white, it may be painfull or not, it may be localized or general in to the skin. There are many different causes of bumps. Bump is the general laymen term for skin eruption. So in your case u developed pus filled bump so it is infected and so u have to take antibiotic oral for 3 to 5 days and if it is painful then take any painkiller. Continue neosporin ointment. Hopefully my advise satisfied u."
},
{
"id": 182724,
"tgt": "What causes blood blister inside the tongue?",
"src": "Patient: Hello my boyfriend has just discovered what appears to be a blood blister on the underside of his tongue, it appeared this evening and he could feel it swelling for a couple hours. It has popped now, releasing a little blood, although it s still sore he hasnt had anymore problems. I was wondering if you could tell me what causes blood blisters? Doctor: Thanks for your query, I have gone through your query.The blood blister what you are describing can be a hematoma secondary to any trauma to the floor of the mouth or tongue. It can be a ranula also. It is a cystic lesion occuring secondary to trauma to salivary glands. Nothing to be panic, consult a oral physician and get it evaluated. If it is a hematoma, nothing to worry, do hot and cold water rinse, do not traumatise that area. If it is a ranula consult a maxillofacial surgeon and get it excised.Do saline gargling. i hope my answer will help you, take care."
},
{
"id": 66825,
"tgt": "Suggest treatment for bump on back of neck",
"src": "Patient: Hi I have a bump on the back of my neck.it doesn t bother me at all it s almost like a pimple but doesn t come to a head.so I decided to squeeze it and this cheesy white stuff came out of it and it had an odor to it.the bump is still there just not as big.wanted to know what it is and what I can do.thanks Amber Baldwin Doctor: Hi,Looking to the history it seems that there might be having infected sebaceous cyst producing cheesy stuff.Consult surgeon and get it excised with complete sac.Meanwhile go for one antibiotic medicine course to control infection.Ok and take care."
},
{
"id": 201713,
"tgt": "Does smaller penis lower chances of conception?",
"src": "Patient: Hi Doctor, thanks. Me and my wife are ttc, when we had sex last thursday we really started great, then for a while my penis got flaccid and she remained very excited, then my penis managed to get up again but it was not as big as before, I inserted it and I ejaculated inside, she had an orgasm after. However, my penis felt small inside her.. Does that lower the chance of conception?? Doctor: Chances of conception depend on many other things like her ovulation dates, your semen analysis etc,If you had ejaculated in her vagina completely, there is high possibility of conception.You should not worry about it due size of penis.Thanks."
},
{
"id": 181626,
"tgt": "Suggest treatment for soreness in flap of skin over back tooth",
"src": "Patient: i am female 110 pounds 5'4\", i am 22 and never got wisdom teeth. still no signs of them. mom never got them either. but i have a flap of skin over my back tooth that has never bothered me before this week but hurts now. is it just an infection? how do i make it go away? Doctor: Hi..I have gone through your query and can very well understand your concern.As per your complain it is possible that the wisdom tooth is about to break through the gums and due to this there is soreness and pain in the gums in that area..Other reason can be that a small portion of gums might have ruptured and there is food deposition between the gums and tooth leading to infection and inflammation and the condition is known as Pericoronitis..But if there are no signs of any erupting tooth the pain can be due injury to the gums in that area by some hard foods..You need to consult a dentist and get a clinical evaluation done as a correct diagnosis and treatment can be done only after knowing the exact cause..By the time you can take painkillers like Advil and do warm saline gargles..Wishing you a good health..If you find the answer helpful please give a Five Star Review and click on found the answer helpful as a token of appreciation..Thanks and regards..Dr.Honey Nandwani Arora.."
},
{
"id": 44898,
"tgt": "Medicines for increasing semen volume",
"src": "Patient: hi, i am looking for a semen size increment with a pills or treatment. so that i can get sperm test.my current semen valume is less than 0.5 ml. Doctor: HELLO WELCOME TO HCM... THERE ARE SO GOOD MEDICINE IN HOMOEOPATHY FOR INCREASE VOLUME AS WELL COUNT . LIKE DAIMANIA , NAT.PHOS, ... MANY MORE . HOMOEOPATHY CAN HELP YOUR PROBLEM WITHOUT SIDE EFFECT. IF YOU WANT ONLINE HOMOEOPATHY TREATMENT THAN MAIL drvipul2001@yahoo.com . FOR SELECTION OF PROPER HOMOEOPATHY MEDICINE WE REQUIRED FULL DETAIL HISTORY . SEND REQUEST AS \" WANT HOMOEOPATHY TREATMENT '' THAN WE SEND YOU CASE PERFORMA . BYE AND MEET AGAIN FOR BETTER HEALTH...."
},
{
"id": 196816,
"tgt": "What causes shrinking of the Penis before ejaculation?",
"src": "Patient: HI,I have got married 2 years back , Me and my wife have had sex lot of times. But of late my penis starts to shrink even before I could ejaculate in her vagina. We are trying desperately for a kid and am worried. Did my semen analysis about 6 months back and it was normal as per the report. I do masturbate every day, could that be the reason. Can you please help me with a way out since am feeling that am not able to satisfy her and sacred a lot. Doctor: Dear service user,Thanks for writing to HCM.I can understand your concern about loss of erection before vaginal ejaculation and also for having child.Loss of established erection is commonly caused by performance anxiety.Occasional failure can occur to almost every man in his life. When this happens, one gets concerned about performance during next sexual encounter. He thinks whether he would be able to satisfy the partner or not. So, his mind thinks mostly about quality of erection and performance.For constant erection continuous exciting/romantic/erotic feelings should be there in mind. If mind is diverted to concern about erection (being scared in your case), you will lose erection.This in turn reduces your confidence and it again affects the erection.Having erection during masturbation indicates that your organs are functioning absolutely normally. Masturbation is a healthy part of sexual activity and it does not cause impotency or infertility.There are effective behavior therapies having strong research evidence for psychogenic erectile dysfunction. You can read about ''sensate focus'' on internet. I suggest having consultation with psychiatrist.There are medications like verdalafil or tadalafil which can be helpful with behavior therapy.It is good to listen that your semen analysis report is normal. As far as pregnancy is concerned, you may consult infertility specialist for same. There are treatment options like artificial insemination which can be useful for you.Wish you good health."
},
{
"id": 137967,
"tgt": "Suggest treatment for shoulder pain",
"src": "Patient: Hi I m a 21 year old female and am experiencing left shoulder pain. It almost feels like a sunburn. Its very painful and some occasions are worse than others. I cant touch it or even have clothing brush against it. I dont have any history with any shoulder problems except this. I have been experiencing it for a little over a year and it has began to progressand spread a little further down my back and part of my neck. Can someone please help me? Doctor: Hello,Thank you for contacting me at \"Ask a Doctor\". I will try to answer your question to the best of my ability.There could be several causes for this problem. I would suggest that you see a primary care physician or an orthopaedic specialist, for a complete physical examination. They will need to examine your shoulder in detail, and also examine your cervical spine.Issues that come to mind include a problem with the shoulder itself, for example, tendinitis, or capsulitis. Another likely issue is that of referred pain from the cervical spine. Sometimes, a disc problem in the neck can cause referred pain to the shoulder. This results in sensations such as burning, tingling, or pain in the arm.In my particular practice, we would certainly consider taking some plain x-rays of the shoulder and cervical spine. I do not think an MRI is needed at this stage.For pain relief I would take an anti-inflammatory such as ibuprofen and apply ice to the shoulder. It's stiffness is an issue, then heat can certainly help.I hope this information is useful for you. Please do not hesitate to message us back if you have any further questions.Best wishes,Adrian Rawlinson MD"
},
{
"id": 50121,
"tgt": "Abdominal pain, have kidney stone. Elevated SGPT, SGOT, Alkaline and phosphate. What could be wrong besides hep c?",
"src": "Patient: i just returned from the er with abdominal pain...i do have kidney stones , however i also was told i have elevated bilirubin , elevated albumin , sgpt, sgot , alkaline phosephatase....all they did was refer me to a specialist, but im worried & i can t get into the specialist for a month....what could be wrong besides hep c?? Doctor: Kidney stones do not cause abnormal liver enzymes. The kidney stone can be a incidental finding. Do you have gall bladder stones? Your enzyme pattern suggest that you have infection and or obstruction in your bile tract. If you have fever don't hesitate to go to ER. Are you on any medication or herbal remedies. If so that is another common cause."
},
{
"id": 201324,
"tgt": "Should a large bruise in the groin after an accident be of concern?",
"src": "Patient: I was riding my bike on a gravel trail. I went to stand up to go faster but the chain switched gears and my foot slipped and the bike jerked to the right and I fell. When I landed I was still on the bike but on my right side so my entire right leg and foot is bruised. When my foot slipped I fell off the seat and hit my crotch on the bar. I have the most horrible looking bruise down there it s right above my crotch. It s red in the middle and then blue and purple around it. And there s like a bunch of red dots on the bruise like I got hit with a softball. And it s painful to switch sitting positions, go up or down stairs, and to stand or sit down, but not that painful. On a scale of 1 to 10 I would say 3. Could I have severely injured myself down there? Should I go to the ER? Doctor: Hi,Showing nature and extent of injury and extensive bruises on the part you might require medical check up.There might be possibility of having blunt injury in scrotum might create problem in future.Go to ER and get examined.Ok and take care."
},
{
"id": 103445,
"tgt": "Persistent cold, no running nose, normal vitamin B12, D, folic acid levels, diagnosed as allergy. On multivitamins, no improvement with levocetrizine, Romilast L. Cause?",
"src": "Patient: Sir, I am having cold ( i often have to spit out sputum which is white in color) for the last 3 months. I was given levocetrizine tablets which i took for totally 16 days in a staggered manner. After which another doctor has prescribed Romilast L as my eosinophils count is high (9600).The doctor who prescribed me Romilast L says that i only have allergy problem & nothing else. even after taking Romilast L for one week there is no relief. doctor has asked me to continue the medicine for 1 month. I was treated for Typhoid in June 2011. My vitamin b12 level is 310 pg/ml, folic acid level is 16.19 ng/ml and vitamin D level is 16.40 ng/ml. I am taking multivitamin tablets since 1 month. I dont have running nose problem. Pl advice. Doctor: hi, you are treated for eosinophilia with apprapriate drug. but along with the drugs use some protective measures to prevent it. you can meet pulmonologist for testing for allergens, later on desensitization after finding the specific allergen. home dust is very common allergen, change your bedsheets very often."
},
{
"id": 197580,
"tgt": "Suggest treatment for Oligospermia",
"src": "Patient: I m 39 year mail Suffering fron Oligospermia, Dr. prescribed me to take Levocarnitine Carnisure-500 with Clonil-25 twice a day, I mam taking the dose as per doctore. i m feeling tired and like vomating also like to sleep, frequent thrist and weekness like feaver. all things are usual? Doctor: Higreetings. If I am treating you then will examine to find the reasons for oligospermia.Any treatable causes like infections of testis and epididymis or varicocele had to be looked for and if so proper treatment will improve your semen.Hence I strongly suggest you to consult an andrologist. hope my answer helps you. Regards"
},
{
"id": 51975,
"tgt": "What is the side effects of wysolone ?",
"src": "Patient: my age is 38yrs,height 167cms,weight 63kgs,i m suffering from nephrotic syndrome ....started intake of wysolone 130mg every alternate day....but i couldnt sleep da night whn i intake wysolone that same day...why...? Doctor: Hi Kudasdesai, Welcome to HealthcareMagic Forum, Insomnia is a known side effect of prednisolone. Please consult your doctor regarding this problem and any dose modification if required. Take care Regards Dr. Naveen Kumar"
},
{
"id": 27996,
"tgt": "Suggest treatment for chest pain,tingling in arms and sore throat",
"src": "Patient: I have been to the ER three times and the dr twice. I am having a lot of pressure, pain, tingling and what feels like heat in the back of my head, neck, pain and pressure in my chest and ribs, pain in my right side below my ribs and on the right side of my back, tingling and pain in both arms and hands and now my tongue is numb and my back feels like it s burning and my throat is sore. They did panels, urinalysis, chest xray, all came back normal. They re saying it s anxiety. They refused to do a ct. I m at a loss. It s just getting worse. Doctor: lookign to your history your symptoms are related to anxiety,you must start anxiolytic medicines ,you should try mouth dissolving clonezepam when your symptoms start and you will immediately feel better,which would almost confirm your diagnosis of anxiety.also you should get your vit B12 level to rule out deficiency.also take round the clock aplrazolam 0.25 mg thrice a day for 4 weeks."
},
{
"id": 160633,
"tgt": "When should be meat introduced to the child?",
"src": "Patient: Hi Gud evening Dr, My son is now 7.5 month old.His birth wt was 4.2 kilo and during 6th month he was 7.8 kilo.Is it fine ?am feeding him with formula and cerelac.He has more attraction towards chicken as when we eat he used to struggle and have it again and again.Is it ok to give 2-3 spoon of minced chicken?Regarding egg yolk how i should give? Doctor: Hi,Non vegetarian items can be well introduced into diet of a 7.5 months old baby. Minced chicken, meat, egg all can be given alone, or used while preparing other items. But take care to introduce one by one at a gap of 1-2 weeks in between, so that baby's tolerance to a particular item is ensured. For cow milk, it is better to wait till 1 year of age.Hope I have answered your question. Let me know if I can assist you further. Regards, Dr. Muhammed Aslam T. K., Pediatrician"
},
{
"id": 175131,
"tgt": "Should I be concerned for the gas bubbles that smells like bad odor in a 5 month old?",
"src": "Patient: My 5 month old was dry heaving and vomiting what appeared to be water. He is on formula with small amounts of rice cereal mixed in. His tummy sounds bubbly and he is passing gas that smells like rotten egg. Need for alarm or to see his normal pediatrician? Doctor: Thanks for consulting in hcm. I think you should stop to introduse ceraeal till she will well, better start later ,from 5,5 month. The digestive system is not ready eat.It is heavy for her, he doesn't have enough enzymes. Nowadays, we usually introduce solids from 6 month, we start from boiled potato or cereal.I advice to your bay for improving digestion take econorm(saccharomyces) 1/2 sashe 2 times 3 days, then darolac 1 caps 1 time 1 month(open capsule and give powder with small amount of water), it is our normal flora, which take part in digestion.From nausea and vomiting give to your baby antiemetics- domstal or ondansetron , keep hydrated with ORS 5-10 ml every 5 min Take care Dr. Svetlana"
},
{
"id": 88773,
"tgt": "What could lymph nodes adenitis with mucous filled bowel movements suggest?",
"src": "Patient: my daughter has been sick for 1 month she is 10 years old she had a ct scan which showered mesenteric lymph node adenitis. She has lost 21 pounds she is in constant pain she was seen by GI this past friday and they have set her up for colonoscopy and EGD. The pain increases after she eats and after she has a bowel movement and her bowel movements have had mucous in them. What could this be please help???? Doctor: Hi.Thanks for your elucidate history. Young child of 10 years loosing so much weight and have Mesenteric Lymphadenitis ans symptoms of bowel movements with mucus and has to go after food is suggestive of Tuberculosis of the abdomen / or a Cancer like lymphoma.IT is good that she will be subjected to Colonoscopy that biopsy can be taken if a lesion is seen in the colon. EGD for secondary changes. The best diagnostic tools in such a case are :Blood- complete blood picture, kidney functions tests, blood sugar fasting and post-lunch, Urine- routine,microscopy, culture and sensitivity ( before start of antibiotic)Diagnostic Laparascopy with FNAC or preferably biopsy of the lymph nodes will give a definite diagnosis.So that a proper treatment can be started ASAP"
},
{
"id": 21201,
"tgt": "Suggest treatment for rapid heartbeat and headache",
"src": "Patient: Hello Doc,I am a male, 32 years, very fit, and not overweight. Yesterday I got a cough and it got progressively worse.Now I can not stop this dry cough, I have a fever of 102.6F yet I am cold. My head is very hot though, I have a headache and rapid heart beat. I am extremely weak and thirsty Any suggestions? Doctor: hellothank you for posting in our forum. you seem to be suffering from some respiratory tract infection on basis of your symptoms of cough, fever headache, dehydration, fast heart rate. please do a chest x ray, a complete blood count and ESR. you can have a short course of antibiotics tab augmentin 625 thrice a day for five days (if you are not allergic to penicillin), cough syrup and tab paracetamol upto three times a day for your fever. It is very necesaary to do the investigations mentioned above.thank you. wishing u good health"
},
{
"id": 140968,
"tgt": "How can multilevel moderate degenerative changes of spine be treated?",
"src": "Patient: MRI SCAN REPORT IMPRESSION AS FOLLOWS.... Multilevel moderate degenerative changes of spine as described L3-L4 mild posterior disc bulge indenting the thecal sac without significant nerve root compression. L4-L5 space shows diffuse circumferential IV disc bulge with central and lateral protrusion intending anterior thecal sac and causing narrowing of bilateral lateral recesses and significant compression of bilateral traversing LS and existing L4 nerve roots. L5- S1 spaces diffuse IV disc bulges, narrowing bilateral lateral recesses causing impingement of bilateral traversing S1 and existing L5 nerve roots. What can I do above symptoms? Kindly reply..... Doctor: Hello, Do you have any problem? Any pain, numbness, weakness, saddle anaesthesia. I will treat the patient, not labs. Hope I have answered your query. Let me know if I can assist you further. Take care Regards, Dr HIKMATULLAH, Neurologist"
},
{
"id": 2734,
"tgt": "When can pregnancy be planned after c section?",
"src": "Patient: I had an emergency c section 10 weeks ago. I had checked in at l&d ward with serious labour like cramps at 39 weeks. The midwife could not hear the hearbeat. The scan confirmed my worst fear, there was no heartbeat and I had placental abruption. How soon can I conceive? I miss my baby so much Doctor: Hi, You can start trying for pregnancy after 12 months, but be alert. Get your regular checkup done. Keep a strict watch on your Blood pressure, because it is one of the major cause of abruption. Hope I have answered your query. Let me know if I can assist you further. Regards,Dr. Khushboo Priya"
},
{
"id": 87159,
"tgt": "What causes shortness of breath and mild abdominal pain?",
"src": "Patient: Hi, since last two weeks i'm feeling some shortness in breathing because i need to breathe from mouth after every 5-6 mintute...i'm also feeling light to mild pain in lower right abdomen and back....two weeks before i was diagnosed with stone of 4 mm size in my right kidney..........please suggest me the cure Doctor: HIi appreciate your concernlooking at your symptoms its clear that shortness of breath is not related to kidney stoneas such the size of the stone is very small that can be expelled out with hydrotherapy ( plenty of oral fluids)in my opinion you must visit to a cardiologist your detailed check up like ECG, ECHOCARDIOGRAPHY and an X ray chest for a confirmatory diagnosisHope i have answered your questionthanks for using HCMtake care"
},
{
"id": 203004,
"tgt": "What are the chances of pregnancy after having sex with torn condom?",
"src": "Patient: hello sir ; actually i had sex on 8 with my gf with condon bt ghe condon get tornd ... n a 1 or 2 drop of spern laid on the mouth of vagina nt inside evn i havnt put my penis inside her pussy ... i was just tryng n it get tornd . so nw is there is any chanc of pregnancy??? Doctor: hi,the changes are high for pregnancy with a torn condom, but again depends on the cycle of your girlfriend if she was ovulating at the time she had the sexual intercourse.hope to have answered your querythanking you."
},
{
"id": 103960,
"tgt": "77 years old, has asthma and heart problem. Had run short of breath. What to do?",
"src": "Patient: Hi, My mom is 77years, and is suffering from Asthma, n had an attack of asthma a week ago, n was gasping for breath feeling week,swollen feet/leg, n then suddenly as she was talking she ran short of breath suddenly and could not breath at all then they rushed her to the hospital and she was given oxygen gas and then she felt better but she is still in hospital....what could it be and can she have this type of attack again and what are her chances of survial.She also has heart problem and has a hole in the heart...but recently she was checked and her heart was fine.Please tell me what to do i am very frightened coz if she gets an attack again then .... Doctor: Hi, Welcome to HCM, Recurrent attack in bronchial asthma is a common problem. Consult pulmonologist and have regular medicine like broncho dilators and anti-asthmatic medicine. Keep Nebulizer at home and use as and when required. Use of Nebulizer will help for breathing problem and visit to hospital off an on will be avoided. Avoid dusty and humid atmosphere. Ask her to go for deep breathing exercise. As you told she was having some cardiac problem, consult cardiologist and get it diagnosed and confirm that your mother is not having this problem due to heart ailment. Ok and bye."
},
{
"id": 190403,
"tgt": "Had wisdom tooth pulled. Should I worry about the pus or blood?",
"src": "Patient: I got my wisdom teeth pulled about a week or 2 ago and just now I tasted something weird in my mouth almost like blood but not exactly like it so I look in my mouth and were my wisdom tooth got pulled there is a tiny hole that everytime I push near it alot of puss/blood comes out I manage to drain it out I guess you can say but u just want to know should I be worried and what s gonna happen to me? And if I go to the oral surgeon what are they gonna do to me Doctor: Hello, Thanks for writing in. I read your problem. First of all i would like to tell you that pain up to some extant after wisdom tooth removal is normal. But as you gave the detail, bleeding and pus drainage is not a good sign. It shows that there is infection in that area. So i will suggest you please visit to your oral surgeon as soon as possible and get the complete examination done. Nothing to get worried,your oral surgeon will provide you the proper treatment which will include irrigation of that area followed by placement of antiseptic dressing. And your oral surgeon must give you antibiotic and pain killers and antiseptic mouth wash like betadine........ Take Care. Dr.Gunjan Gupta"
},
{
"id": 128694,
"tgt": "What does this left knee ultrasound test result indicate?",
"src": "Patient: Please explain in layman s terms ultrasound of left popliteal fossa & posterior calf?Theres a 4.6x2.3x3.8 cm complex hypoechoic area associated with the distal hamstring, along with the midline posteriorly. The area appears multi-loculated and may represent a hematoma from a hamstring injury. The appearance is not typical of a Barker cyst, due to the position.In addition. at the musculotendinous junction of the distal medial gastrocnemius muscle, there are smaller, hypoechoic area, measuring 8x43x5mm. The appearances are suggestive of a small intramuscular tear, with associated haematoma. The Achilles tendon has heterogeneous hypoechoic areas, with the loss of the normal fibrillary architecture, suggestive of tendinopathic changes of the Achilles. Doctor: Dear patient In short you have got blood collection in the back of your knee in the area of muscle and size as described. That might happen due to injury. Part of muscle is also torn at its insertion to posterior knee. At ankle level heel tendon is torn partially. treatment Is rest and wear knee cap. Tab zerodol sp twice a day for ruching pain and swelling."
},
{
"id": 200571,
"tgt": "What causes swelling between testicles and anus?",
"src": "Patient: Hi I ve been feeling a pressure kind of like swollen feeling between the testicles and anus no problems passing urine but the pressure does make me feel like I want to urine more wen I sit but not as bad when standing it s been going on for a month and a half.... what could this be? Doctor: Hi, dearI have gone through your question. I can understand your concern. You may have some piles or cyst. you should go for examination. Then you should take treatment accordingly. Surgical excision is the treatment of choice.Consult your doctor and take treatment accordingly.Hope I have answered your question, if you have doubt then I will be happy to answer. Thanks for using health care magic. Wish you a very good health."
},
{
"id": 40705,
"tgt": "What does the following follicular study report suggest?",
"src": "Patient: Hello, My periods are regular ( 27-30 days cycle usually) and i have PCOS/2nd cycle with clomiphene- In last month my first cycle follicle got ruptured after 2.0X1.1cm (right ovary: day 17th). But didn t concieved after that this month s cycle again with clomiphene 50mg and follicle size is 1.3\u00d71.1 & 1.6\u00d71.4cm (RO: day 12th) and on 2nd scan 1.4X1.5 & 1.4X1.6cm (right ovary: day 15th) but my gynecologist told me it s not growing significantly this time and suggest me to see a infirtility specialist. Please suggest what might be the reason for advicing this? And why my Dr. Suggested me directly to visit infirtility specialist instead of further treatment ?? Also is it so ovaries function in alternations? Because in my case it is Right ovary since last 3 months which is showing maturing follicle!! Doctor: Hello,thanks for choosing healthcare magic for your query...first of all I want to tell you that generally ovulation happens on alternate side but it's not a rule so for now relax about that.This is your second cycle with clomephene 50 mg. We can wait until 20-22 days of cycle for ovum to get fully mature. Your egg is not growing enough but we can wait.As your Gynaecologist has advised you to visit infertologist, so there are many factors which needs to be considered in Patients trying to conceive. If your doctor is advising you for more expert opinion then that doesn't always mean that there is something which is worrisome.You may need to take some injections ideally which should be given in particular facilities.So relax and take infertologist's opinion.I hope this helps. Feel free to contact if any further queries.Regards,DR TEJASHREE BHORGYNAECOLOGIST, DHANORI, PUNE"
},
{
"id": 209612,
"tgt": "Suggest treatment for ADHD",
"src": "Patient: Hi! My name is Will . I have been living and working in El Paso for several years. The last few years I ve had extreme concentration problems ...very hard to finish simple tasks...without loosing interest . I ve tried to make appointments with several psychiatrists ...but they don t accept Humana ..or have long waiting times. My twin brother was lucky enough to find a doctor ...he was diagnoses with ADHD and has been doing really well with the medications he is taking (adderall and something for mild anxiety ) Thanks .Will Doctor: Hello,Thanks for choosing health care magic for posting your query.I have gone through your question in detail and I can understand what you are going through.The drugs useful for ADHD are methylphenidate or bupropion. these drugs help in improving the concentration. Hope I am able to answer your concerns.If you have any further query, I would be glad to help you.In future if you wish to contact me directly, you can use the below mentioned link:bit.ly/dr-srikanth-reddy\u00a0\u00a0\u00a0\u00a0\u00a0\u00a0\u00a0\u00a0\u00a0\u00a0\u00a0\u00a0\u00a0\u00a0\u00a0\u00a0\u00a0\u00a0\u00a0\u00a0\u00a0\u00a0\u00a0\u00a0\u00a0\u00a0\u00a0\u00a0\u00a0\u00a0\u00a0\u00a0\u00a0\u00a0\u00a0\u00a0\u00a0\u00a0\u00a0\u00a0"
},
{
"id": 29504,
"tgt": "What causes chills and sweating post Ventolin intake?",
"src": "Patient: i have just started using ventilin several times a day as I was getting a bit breathless I am now getting very cold and starting to shake I have taken Advil and after about an hour or so break out in huge sweats for about an hour It has been happening couple of times thru the day and night also feel a bit lethargic for the last four days. I have seen a doctor and was sent off to emergency. My heart, blood pressure was good It was the hospital suggested I use ventilin. why would I be shivering and sweating many thanks Doctor: Hi i do care for your concern. The drug ventolin stimulates something called beta adrenergic receptors, which in some case will cause increased sweating, increased heart rate and so on. I would advise to consult your physician and change to other drugs such as budesonide that will help you overcome this situation. Advil has no role to play here. Hope i have answered your question, if you have more feel free to ask. Thank you."
},
{
"id": 161896,
"tgt": "What causes a 4-month-old baby to cry constantly and have bowel movements only once in a week?",
"src": "Patient: Hello doctor my baby is 4 month older n he is very crying in nature sometimes all the day he was crying...I was concerned to many doctor n done his regular checkup every month bt doctor said baby is fine...n he is passing his stool once in week...what should I do for him..plz suggest some fruitful remedy to me Doctor: Hi, This is called evening colic and is quite common in this age group. This happens when the baby sucks at the breast very fast and in eagerness to drink milk will gulp in air too. Unless the air comes out like burping or flatus this discomfort will be there and next time check if the baby is sucking too fast and gulping in air too. You will be more convinced. Usually, I don't advice any medicines for this as they give only temporary relief. The two best ways to relieve this distress is - 1. Do not put the baby in lying position after feeding till the baby burps out the swallowed air. 2. If still crying - put the baby in prone position and keep patting the back gently so that the baby passes off the flatus and gets relieved. Babies of this age may pass motion 10 days once or 10 times a day also. If the baby is active and feeding well, don't worry. Hope I have answered your query. Let me know if I can assist you further. Regards, Dr. Sumanth Amperayani, Pediatrician, Pulmonology"
},
{
"id": 125014,
"tgt": "Can i take this medicine for knee pain?",
"src": "Patient: I am having both the knee pain and the severe one is left knee pain. I have been taking ayurvedic treatment for the past three months and the pain has reduced..Now an Allopathic Doctor has suggested this cartigen forte twice a day for three months. can i take this tablets now? Doctor: Hello, You can continue Carrigen tablets. If symptoms persist you can consult an orthopaedician and plan for an MRI scan. Hope I have answered your query. Let me know if I can assist you further. Take care Regards, Dr Shinas Hussain, General & Family Physician"
},
{
"id": 203481,
"tgt": "What to do for burning sensation during sex after hernia was repaired?",
"src": "Patient: are you here? I need some help I had an inquinal open hernia 7 months ago. This hernia was repaired total three times over the period of 10 years. When I masterbate or have sex it burns in the stiched area. There is no bulge in there. I have been seen by a doctor she said there is no hernia. My sergeon said it s a scar pain. I am worried it s been 7 months and the burning did not go away. Thruout the day I feel discomfort in there but I am worried about the burning sensation Doctor: Hello,Thanks for choosing health care magic for posting your query.I have gone through your question in detail and I can understand what you are going through.Yes, it is probably a scar pain. Its unfortunate that you have landed up with such a problem. It occurs cause during sex there is an increased abdominal pressure and this increased pressure can lead to stretch on the earlier sutured area. You may take paracetamol for the mild pains. You may also take carbamazepine or amitriptyline for modifying the pain. Hope I am able to answer your concerns.If you have any further query, I would be glad to help you.In future if you wish to contact me directly, you can use the below mentioned link:bit.ly/dr-srikanth-reddy"
},
{
"id": 157090,
"tgt": "How long the side effects of robotic surgery continues ?",
"src": "Patient: How long the side effects continues ? I had 2 doses of 22.5 mg in oct 2012 & jan 2013 , I have a lack of desire to have a sex, I. Am 64 year old male prostate patient, I had a robotic surgery in 2010 but unfortunately cancer came back, I finished my radiation therapy in jan,2013 Doctor: Dear,Hope you are doing well. Before answering your question let me know some facts first. What medication (of 22.5mg) you are telling about? Is it leuprolide injection? What kind of side effects you are suffering? Is it only loss of desire of sex? What was the modality used in Radiation therapy? What was the dose of radiation in Gray? expecting your response.. Hope I'll be able to answer your question."
},
{
"id": 93769,
"tgt": "Feeling of gas bubble in abdomen, painful during intercourse. Cause?",
"src": "Patient: Hello. Here is my concern. This has only been occuring for a few days, but I am starting to worry. I have had what feelis like a gas bubble in my upper abdomanial region on the right side of my stomach . It is especially painfull during intercourse. I am just curious as to what could cause this kind of pain and is it serious? Any advice would be greatly appreciated. Doctor: Hi welcome to Health care magic forum. Thanks for choosing H.C.M.Forum. You are feeling like a gas bubble in your right upper abdominal region. Painful during intercourse. Pain in the right upper abdomen is suggestive of infection in the kidney, stones in the kidney, or upper ureter, infection of upper part of ascending colon. liver infection, or gall bladder stones. I advise you to consult a surgeon for diagnosis and treatment. You may need to have M.R.I. besides other routine tests for confirmation. Wishing for a quick and complete recovery. Best regards."
},
{
"id": 110886,
"tgt": "What causes severe back pain?",
"src": "Patient: hi doctor, i have a back pain. dont know how exactly the pain was.. i have it and consulted some doctor. they said as muscular pain.. i did physio therapy. now i again got this pain. its not horrible. but i dont feel comfort normal life. am a software engineer. can u suggest me Doctor: welcome to Health care magic.1.The above symptoms with your occupation seems like, positional problem.2.Initially i would recommend basic back pain exercise, and correction of positional changes.3.If symptoms does not ease, i would ask for X-ray depending on changes an MRI (best investigation to find the disc and chord diseases)4.If money is not matter get MRI done.Anything to ask ? do not hesitate. Thank you."
},
{
"id": 42096,
"tgt": "Does duphaston and clomid help to treat infertility?",
"src": "Patient: Hi doctor,I m 27 years old,59 Kg & got married 4 years before.I m trying for a baby since last 3 years.I have pcos from the age of 20 with irregular cycle.I had done 2 abortions due to pcos.From last 1 year I m taking metformin (3 times in a day) & folic acid (1 time in a day).But 3 monthe before my doc suggested me to take Krimson 35 to regulate my cycle.But now I dont want to take Krimson35 in future.I want to get Pregnant.I Want to know that Dhuphaston+clomid are good and safe medicines to get pregnant in pcos?Is there chances to get pregnant with these medicines? Thanks Doctor: Hello,Welcome to healthcare Magic.I have gone through your query and Would like to reply in detail as follows:1. Yes.2. These medisines help in conceivingI hope your doubt is clarified. Let me know if you wish any other information.Regards,Dr. Mahesh"
},
{
"id": 135510,
"tgt": "Is polio associated with painful legs & mild twisting?",
"src": "Patient: sir my child is 3 years and 7 months of old. the doctor diagnose a typhoid after treatment of 8 days and start treatment according to best. but yesterday morning he has said that my legs are painful and i am unable to walk, after then he starts to walk but then his right legs mildtwisted. some time he walks properly and sometimes he walks with right foot slightly outside. sir he fully vaccieted with polio medicine. I consult with the doctor they said it is due to weakness and will remove in a week but he said it could be a polio sign. Sir i want to know weather is a any possibility of polio if my child is fully vaccinated and what should i do. Doctor: dear madamplease do not get apprehensive, you have not shared your country of residence, and if polio has been declared eradicated in your region.it may just be a simple transient muscular weakness, it would have been better if you would have shared what medication was given to the child.just keep a watch on the child and if it does not improve please re visit the pediatrician."
},
{
"id": 27442,
"tgt": "What causes heart palpitations and chest pain?",
"src": "Patient: Hi, I'm 33 years old female been suffering heart palpitations, not only is my heart racing I can see my left side of my chest fluttering/ moving. I have theses episodes most time of the day and have been suffering this symptom for about 2 months I also have pain and discomfort on the left side of my chest and upper arm like a stinging/burning pain. What could be the cause of my symptoms. Doctor: Dear Ms,Heart palpitation can be related to anxiety, over-tiredness, etc, in your age usually there is nothing serious.Most probably your pain comes from your spine. I would advise you to take Ibuprofen 400 mg, three times a day, after meal, if you don't feel better in a 5 days, I would advise you to have your vertebra checked.Hope I could help youWishing you good healthIn case of further questions don't hesitate to askRegards,"
},
{
"id": 42516,
"tgt": "How long after ovulation is there fluid in the pod?",
"src": "Patient: i hav got irregular periods and took treatment for that..i was given clomiphene for induction..on day 16 follicle on rt was 20mm..my partner is in different state..we mated on my day 17..on day 20,follicle ruptured and there was free fluid in pod..my doubt is that when could have ovulation taken place?for how many days after ovulation will there be fluid in pod?plsss answer me Doctor: Hi,Welcome to HealthcareMagic . .To tell exactly when ovulation has happened you need to get daily ultrasound . Fluid in POD remains for 1 to 2 days and gets absorbed later. Since you have mated on day 17 and you have ovulated somewhere between day 17 to 20 that should not be a problem. Because sperm stays for 3 to 4 days in uterus. Anytime inbetween egg formed if other factors are normal fertilisation occurs.Hope I have been helpful .RegardsDr.Deepika Patil"
},
{
"id": 37434,
"tgt": "How to get rid of intestinal worms?",
"src": "Patient: i have intestinal worms since few weeks. I am extremely irritated about it. I have itches around my rectum every now and then, heavy sugar cravings and have diarrhea most of the time. I have taken albendazole tablet 400mg two weeks ago but of no use. I read in the internet that they kill the parasite and the female parasite leaves all her eggs on the surface of intestine. I read about invermectin but I am afraid to take it without the doctors advice as it can mess with the CNS.I am irritated because of this condition most of the day. I am pretty unsure what to do and need help despirately. Please help!! Doctor: Hi,Many a time second course of Albendazole to eradicate worms from GI tract.Keep your nails well cut as grown nails carry worms ova and gives infection again.Give anti worm treatment to your close members who are also might get this infection.Keep clothes clean and avoid using your towel by others.Ok and take care."
},
{
"id": 71144,
"tgt": "Suggest treatment for pain in the diaphragm",
"src": "Patient: Woke with pain in the diaphragm area. Cannot take proper deep breaths from the abdomen, but rather need to chest breathe . I do have progressive MS and take several muscle relaxants. Thought maybe it was just a spasm but it has continued for 9 hours. Doctor: Hello and Welcome to \u2018Ask A Doctor\u2019 service. I have reviewed your query and here is my advice. As you explain the if this kind of pain continues for more than 24 than it would be better for you to consult your doctor. Probably an abdominal ultrasound is needed. Hope I have answered your query. Let me know if I can assist you further."
},
{
"id": 202259,
"tgt": "Suggest treatment to harden penis",
"src": "Patient: My age is 24, height is 5ft 3inch,weight is 88.Sir my name is salman, i am very weak my penis is not getting hard,my penis is always been small i always been involved in masterbuiting, and my feets paining every time, my full body is week, no power in my muscles. sir kindlly help me please Doctor: HelloThanks for writing to HCMTrue erectile dysfunction is unlikely in your age.Your problem may be due to stress or psychological causes.You should avoid stressful situation.Don't take alcohol or cigarettes if you take these things and try to lose your weight.Masturbation is normally practiced by many persons but its obsession is bad.You should concentrate in your activities.You should avoid provocative literature and videos and try to keep busy yourself in your job.You can also try yoga and meditation,this will help a lot.Hope i have answered your query.Take CareDr.Indu Bhushan"
},
{
"id": 88936,
"tgt": "What causes abdominal pains and painful bowel movements?",
"src": "Patient: Yes, my boyfriend is complaining of lower abdominal pain while he sits, lays, even just moves. It s hurts for him to pass gas and make a bowel movement, but still can. When he lays down, he can feel a pulse in the lower abdominal region. Please help. I m very nervous and I need to know if I need to take him to the ER or not! Doctor: Hi.You should take him to ER urgently. Stop giving him anything orally as he may need to be nil orally. Thanks for your history. It is suggestive of an intra-abdominal collection of infection or pus , as he has the classical symptoms of pain even on movement. This occurs as the peritoneal covering of the abdomen get inflamed enough to get pain even on slight pressure or movement.This is most probably a case of peritonitis ,. and needs to be admitted, evaluated clinically , investigated for blood, urine, stool, ultrasonography, X-ray of the abdomen in standing position. After a diagnosis , Laparoscopy or Laparotomy needs to be performed to remove the diseased organ, have peritoneal lavage. Take him to the ER urgently."
},
{
"id": 106274,
"tgt": "I have been having asthma attacks but today was the worst. Should I be concerned about these or not ?",
"src": "Patient: Hi, I am a 16 yr old female. I have asthma, and have since I was a baby. But for the last 3 or so years it has been really well and I have had NO problems with it, my mother ( and great grandmother) both passed away at early ages from heart diseases, they were said to be asthma symptoms nothing else. My mother also had hyperthyroidism. The last few months I have been constantly tired, can t focus and do not want to, all I want to do is eat and sleep. No matter how many hours I sleep, I m tired. I have depression but have for a few years, the last two weeks i ve been having asthma attacks but today was the worst. I became dizzy my lungs felt like they were closing and I became dizzy with side and arm pains. I used an inhaler and it finally started passing after about 20 or so minutes. Should I be concerned about these or not ? Doctor: Yes , Have a complete check up with a Spirometry. You Asthma has recurred."
},
{
"id": 178341,
"tgt": "Suggest suitable medication for convulsion",
"src": "Patient: Greetings Doctor. My child - 6yrs recently had an episode of convulsion and we visited our padeatric and he started him with Valparin 200mg half tablet twice in a day and recommended to vist a neurologist. We visited a neurologist and he asked to change to Valparin 200mg Chrono one tablet each twice in a day. Please tell me Valparin Chrono can make a child more drowsy/sleepy. As since yesterday he is feeling tired and sleepy eventough he slept for more than 9 hours in the night he went to sleep in the afternoon. Please help us on this, hope this is not a pre sign for one more convulsion. Doctor: Hi, I had gone through your question and understand your concerns.Valproic acid is usually drug of choice for childhood seizures. 400 mg twice day is usual dose for a six year old child , it may cause drowsiness initially , with time child get adapted to this , dose needs to be titrated depending upon control of seizures . Sleepiness appears to be related to medicine which will gradually improve. Hope this answers your question. If you have additional questions then please do not hesitate in writing to us.*Always check for history of allergy before taking any medicine.Take care .\u2022\u00a0\u00a0\u00a0\u00a0\u00a0DISCLAIMER: - All the information provided here is for information purpose only , it is not a substitute for the advice of a physician after physical examination , it is not intended to replace your relationship with your doctor. This information in no way establishes a doctor-patient relationship. Before acting anything based on this information do consult your doctor. I recommend that online users seek the advice of a physician who can perform an in-person physical examination"
},
{
"id": 104895,
"tgt": "Prescribed Tiomist and Formomide for asthma. Safe to use?",
"src": "Patient: dear dr., I am asthmatic since brith and now my dr. has sujested to take respicaps of tiomist ( tiotropium powder )and formonide200( formoterol fumarate and budesonide powder for inhaiation ip ) . please inform what are the side effect of the same ,is it safe to use ?, I am suffering from dibetic , asthamtic , heart problem . my email - YYYY@YYYY thanks , Ajay Dhariwal Doctor: Dear Concerned., Thanks for writing to us. Your doctor has rightly prescribed Dear Concerned., Thaks for writing to us. You have mentioned that you are a Diabetic and a Cardiac Patient in addition to being an asthmatic.You have to keep your Diabetic/BP under control in addition to preventing asmatic attacks. The treatment given by your doctor are absolutely safe as the Respicaps/inhalation products will be acting on the bronchi and respiratory system for minimum required duration thereby preventing systemic side effects(Metabolic Side Effects-Diabetes and Effects on CVS). In addition they will be preventing the dilatation and destruction of Bronchi and Alveoli by preventing Emphysema . Thiomist contains a long acting beta agonist(LABA) with a glucocorticoid).Salmeterol is a brochodilator and budesonide a steroid.They will effectively restore the brochodilatation/reduce secretion due to inflammation in bronchioles and control ROAD(Reversible Obstructive Airways Disease) The IV administration of Medications and Oxygen supplimentation are reserved for emergency situations only. Pls follow up with your doctor for Pulmonary Function Tests at regular intervals like how you go for Sugar and BP check ups to doctor.In this way you can keep control on Asthma as well. Regular excercises such as long walks also helps. Keep your FBS within 126 mg%.,PPBS within 199 mg% and HbA1C within 07g%. Wishing you a safe and speedy recovery., Best Regards., Dr Lt Cdr ASN Bhushan., Ex-Navy."
},
{
"id": 93597,
"tgt": "Burning sensation around waistline, centralized to right side of abdomen. Taking topamax. Suggestions?",
"src": "Patient: Good evening. I have been having stomach problems since March and I've been on Topamax since February.My symptoms are a burning sensation that goes around my waistline or is centralized to my lower right back or right side of my abdomen. I had a colonoscopy yesterday and I am waiting for the results of that.I'm wondering if Topamax is causing my issue. I never had this problem before starting it. Doctor: Hi welcome to Health care magic forum. Thanks for choosing H.C.M.Forum. You have got some stomach problems, since march, burning sensation around the waist line, lower right back, or right side of abdomen. report of clonoscopy is awaiting. You arethe using topamax since february. Topamax is toperamide used for epilepsy, it has a side effect related to the right abdominal pain and backache, But we should not get it confirmed. The causes of right lower abdomen and back pain are Urinary tract infrction, stone in the urinary tract, appendicitis, etc. I advise you to consult a surgeon for diagnosis and treatment. An how colonoscpy is done you may need to have U.S.scanning , besides other routine tests for confirmation. If all tests are negative we can confirm the side effect of the drug. If it is confirmed to be the side effect it should be stopped slowly by tapering the dose and another anti epileptic should be started. Wishing for a quick and complete recovery. Best wishes."
},
{
"id": 145295,
"tgt": "What causes confusion, slurred speech and difficulty walking?",
"src": "Patient: My husband is confused, is tired, normal BP, elevated pulse rate (112) and pulse is erratic. Has been diagnosed in the past with COPD - does not have difficulty breathing. Over the past several months , he has experienced several of these episodes where his speech is slurred and has difficulty walking. He is 64 years old today. We got ready to go for a bicycle ride and he is apparently having another episode. I can always anticipate that he will be okay the next morning after 10-12 hours of sleeping. Doctor: Hello !I read your question and understand your concern. In my opinion the problems of your husband may be related to a low blood flow to the brain, causing Transient Ischemic Attacks. These events may be related to the heart , as you refer that he has elevated pulse rate , erratic, which may be atrial fibrillation. Repeated TIA lead to high risk of stroke if not treated. I would recommend to consult a cardiologist, and have an ECG and a ECG 24hours monitoring and a cardiac ultrasound to see the heart output. He may need anticoagulation if FA , due to high risk stroke. A brain MRI and a neurologic examination would be helpful. I would also recommend and EEG , as these signs may be also caused by epilepsy. Hope to have been helpful!Best wishes, Dr. Abaz"
},
{
"id": 33861,
"tgt": "Suggest remedy for persistent cough and voice problem",
"src": "Patient: Hi. I was diagnosed with asthma last fall (I assume quite mild). I am on a symbicourt inhaler. A few weeks ago I had a head cold. IT went away in about a week, and for a week I was fine. Then one week later, I woke up with a very sore throat, went to the walk-in clinic and got an antibiotic for strep (although no one has called me back to confirm that the swab actually showed strep). I am coughing non-stop. It feels like it is productive, but then my chest feels just as full after I cough. I have been having some sleep apnea issues since this all started. My throat is not sore, but my voice is rough and raw-sounding. It's hardly worth the effort to talk. I am overweight, but otherwise healthy. I eat well and (up until the last few weeks) exercise regularly (both aerobic and weight bearing). I am exhausted from coughing. I just want my voice back, and to stop coughing. Doctor: HI, thanks for using healthcare magicBecause you are asthmatic, your asthmatic control needs to be assessed first to determine if this is the cause of your persistent coughing.You may need a short course of oral steroids and may also need to be nebulised but this can be assessed by your doctor.Post nasal drip is also a very common cause of coughing and change in voice. Some persons may not be aware of the drip.It is common is asthmatics.The use of topical steroid nasal sprays and antihistamines is recommended.I hope this helps"
},
{
"id": 209477,
"tgt": "Can Estelle cause yeast infections in genitals?",
"src": "Patient: Hi there, I was just wondering if masturbating eliminate yeast infections? I am 22, weight 70kg and 172cm tall. I eat healthy and excersize regularly. Recently tried Diflucan, it still didnt get rid of it. Im also on Estelle, not sure if its coz of that. Doctor: Hello,Thanks for choosing health care magic for posting your query.I have gone through your question in detail and I can understand what you are going through.No masturbation doesnt eliminate yeast infection. It requires proper antifungal medications. Hope I am able to answer your concerns.If you have any further query, I would be glad to help you.In future if you wish to contact me directly, you can use the below mentioned link:bit.ly/dr-srikanth-reddy\u00a0\u00a0\u00a0\u00a0\u00a0\u00a0\u00a0\u00a0\u00a0\u00a0\u00a0\u00a0\u00a0\u00a0\u00a0\u00a0\u00a0\u00a0\u00a0\u00a0\u00a0\u00a0\u00a0\u00a0\u00a0\u00a0\u00a0\u00a0\u00a0\u00a0\u00a0\u00a0\u00a0\u00a0\u00a0\u00a0\u00a0\u00a0\u00a0\u00a0"
},
{
"id": 113013,
"tgt": "Back pain, irregular periods, gas. Help me cure this",
"src": "Patient: I noticed when I lay down n feel around my right side feels like a bulge . I get back pain on my right side here n there nothing extreme . No pain besides that. No burning when I pee just irregular period I dont know if it has to do with it but I did stop my depo 6 months ago. N I do get tons of gas pain. Maybe thats the bulge idk please help me ease my stress till I can go see a doctor Doctor: Hello, Thanks for using health care magic for posting your query. I have gone through your given history in detail. Your depot medications may not have much to do with your bulge and on and off back ache and gas pains especially when you have stopped your depot 6 months back. You can stay calm until you visit your doctor who can ultra sound you to see the nature of the bulge. Hope I am able to answer your concerns. If you have any further query, I would be glad to help you. In future if you wish to contact me directly, you can use the below mentioned link: bit.ly/dr-srikanth-reddy Wish you good health, Dr. Srikanth Reddy M.D."
},
{
"id": 188884,
"tgt": "Toothache followed by pain in half jaw on lower left side. Should I be concerned ?",
"src": "Patient: My husband started having a toothache this afternoon, which turned into the half jaw on lower left side and now its like someone yanked out a tooth without any novacaine. He says lighting bolts of pain are spearing his lower jaw. Have you ever heard of something like this? Even though its his mouth, should I take him to the ER? Doctor: Hello,Thanks for writing to us.Initially,cause of toothache has to be evaluated clinically by a dentist.Take ciplox and paractemol tablets.Grossly decayed teeth has to be extracted.Infected pulp ,if present,has to be extirpated.Maintain oral hygiene well.Gargle with warm saline.Abscess if present,has to be drained and curetted.Take care."
},
{
"id": 44849,
"tgt": "Which is the best hospital in Bangalore for treating infertility problems ?",
"src": "Patient: hello, I am a 32 years old married man.we are trying for a baby for the last 1 year. In the last october my wife had suspected oophratis with a beta hcg level of around 250. The b hcg level came down to 3 after a month. what should we do . Which is the best place in bangalore for infertility issues Doctor: Hello; welcome to HealthcareMagic When a couple is staying together for more than a year and inspite of having unprotected sex if there is no conception/pregnancy then the condition can be labelled as Infertility.It can be a male as well as female problem of infertility.Male problem can be ruled out if the semen analysis of a person is normal.As for females there are many causes like :- -irregular menses with anovulatory cycles -blocked fallopian tubes -hormonal imbalance of FSH/LH -PCOS etc So a detailed investigation is required ;once the cause is located or diagnosed it can be treated and you can have baby.For infertility issues please consult a Gynaecologist in Bangalore and she will guide you regarding further management after examination of the patient. Thanks"
},
{
"id": 135396,
"tgt": "Suggest treatment for severe collar bone pain and jaw pain",
"src": "Patient: For the past 8 months or so, I ve had a pain under my collarbone any time I drink alcohol, even just one drink. And for the last few days, I ve had it on and off throughout the day, along with only one swollen tonsil and a slight pain under my jaw (on the same side). Any suggestions? Doctor: Hi Dear,Welcome to HCM.Understanding your concern. As per your query you have severe collarbone pain and jaw pain. Well there can be many reasons for symptoms you mention in query like TMJ problem , muscle spasm , bruxism or cervical spondylosis . I would suggest you to consult orthopedic surgeon for proper examination . Doctor may order CT scan , MRI or physical examination by asking you to do particular movement fo shoulder . Doctor may also refer you to dentist for TMJ problem . Doctor may five night guard along with muscle relaxant and anti inflammatory . For now apply warm compresses and take ibuprofen or acetaminophen for pain .Hope your concern has been resolved.Get Well Soon.Best Wishes,Dr. Harry Maheshwari"
},
{
"id": 206949,
"tgt": "Suggest treatment for vivid dreams and panic attacks",
"src": "Patient: I think I am having waking dreams, this is the best way I can describe it. Each time this happens I get a panicky feeling in the pit of my stomach the \"impending doom\" feeling. Im not hallucinating but I see it as if I am thinking about it really hard and making a mental picture, only it is involuntary. It has been happening for about three days now and I'm really starting to get freaked out, can anyone help me? Doctor: diagnosis: general anxiety disorder.treatment:it is easily treatable. so do not worry.tab clonazepamtab olanzapine.it will help you a lot. these drugs you will get on prescription only so better to visit psychiatrist hope my answer helped you.take care.please rate the answer."
},
{
"id": 25348,
"tgt": "What causes palpitations and chest pain?",
"src": "Patient: Hi sir, This is balasankar 27 male, 5.7\" 67Kg i am suffering with hypothyroid since 18 months i am using 125mg thyrofit tablet. level are normal still i am getting heart pulpitaions chest pain what is the reason, is there any dosage problems please let me know Doctor: Hello DearYou should check again s. Thyroid function test in difference lab and if any difference than go for changes of dose., you can change other brand of thyroid tablet . its many time help."
},
{
"id": 18211,
"tgt": "What does coughing up of thick white phlegm while on Pulmicort indicate?",
"src": "Patient: Hi I had a massive heart attack 9 years ago, cardiologist were amazed that I showed no signs of a heart attack one month after getting 2 stents. Was really sick Jan and February this last year. Ekg was very good. Over a month ago my 9 year old sick with entero rhino virus... He is still coughing and is on zen hale..... Well I was coughing around the same time....swabs were not available at our clinics because of cutbacks. 2 weeks ago I coughed so hard I puked dr put me on Flovent and doxycycline. 10 days ago I seen my family dr and he put me on pulmicort and ventolin nebs. Throat not hurting as bad but I have a sensation of fluid around neck and lungs not bad but noticeable....still coughing up thick white phlegm..... Yes I feel very tired night. What do you think? Doctor: Hello and Welcome to Ask a Doctor service. I have reviewed your query and here is my advice. It seems to be allergic condition if fever condition is not there, allergic condition may not need any specific treatment but it is the self limiting condition, in severe cases symptomatic treatment can be tried like, Levocetirizine with Pseudoephedrine three times in day, have hot drinks, drink more water, steam inhalation can be tried too, else it is nothing to worry, only the worry part would be secondary infection and it has to be monitored well, in case of infection proper antibiotic can be tried but it happens in seldom cases. Hope I have answered your query, Let me know for further help."
},
{
"id": 105185,
"tgt": "Child with asthma, on medicines, recurring runny nose, cough, taken singular course, live in Ireland. Could it be due to cold water?",
"src": "Patient: My 6 yr old son has been asthmatic frm age 2. He s been regularly on becotide daily twice n ventolin whn wheezy.for past 7 mths he s been hvg constant runny nose , cough .doc prescribed singular for a mth,he was ok.again the sniffles began.we live in Ireland..is tis becos of the cold weather here?last mth I tuk him to India, doc thr prescribed montair.he tuk it regularly n was absolutely fine.also per doc advice stopped becotide n started seroflo 50.now but after returng bac to Ireland, symptoms hv started again.so he s bac on becotide.(plus montair).whn will he grow out of this? Is it ok to gv becotide in long run? Doctor: i know the cases like you when come to me they are all allergic to milk very common in my clinic and after stop of milk and diary products thea are without medicines i have treated thousand of children like this you get your child blood test serum for specific antibodies specific for milk add for wgheat potato chana and ginger after the results we prepare custamised medicines for sublingual immunotherapy we start with low dose slit tyo inccreasae immunity so that disease is cured for time being give singular bd tab allegra 30 mgm bd syp ventrolin 1/2 tsf bd no soaps ointments shampoo and ointment containing milk and nuts no mustard coconut al monds,groundnut and soya oils only olive for cooking and for applying stop all milk and diary products inhaler canbeused sos after 3 week you will really see the magic"
},
{
"id": 200754,
"tgt": "What does a knot below penis indicate?",
"src": "Patient: My husband has a knot like the size of a quarter below his penis...he also has two pebble size on his sack as well as up to quarter size knots on inside of thighs what could this be? The ones on his thighs rupture with light blood maybe a little puss or go away...the other ones don t do anything and the one on bottom of shaft is a new one never had before Doctor: Thanks for asking in healthcaremagic forumIn short: May be a boil/cystExplanation: It may be a boil or cyst. So, visit a dermatologist for its removal. Hope this hepls you. Please let me know."
},
{
"id": 181585,
"tgt": "What causes bleeding from teeth and swelling on gums?",
"src": "Patient: Morning Doctor. What can be the reasons for my teeth to bleed always? my gums are also swollen , i even visit the denties to clean them and she gave me the follow-up appointment. Still my teet & gums are still experiencing the problem. What can be the reason? Doctor: Hi Dear,Welcome to HCM.Understanding your concern. As per your query bleeding from teeth and swelling on gums which is mainly because of infection of gums which might be because of improper cleaning of teeth or food deposition between teeth. Gingivitis is the condition which can progress to periodontitis as well if you keeps on ignoring your situation. I would suggest you to visit periodontist once and get a proper prophylaxis for this. Tartar, calculus,stains and plaque on teeth are common causes of gum bleeding. Once plaque hardens to calculus, it cannot be removed using toothbrush and other home remedies. Get a professional scaling and root planning done and after that maintain proper oral hygiene. Do rinse with listerine mouth wash. Hope your concern has been resolved.Get Well Soon.Best Wishes,Dr. Harry Maheshwari"
},
{
"id": 96481,
"tgt": "should have detailed sonographic evaluation of abdomen and pelvis for abdominal pain?",
"src": "Patient: should have detailed sonographic evaluation of abdomen and pelvis for abdominal pain I am a 17-year-old girl and have been told bymy doctor that I have an ovary cyst. I was told these go away on their own. I've had very sharp pains from it causing naseua once every 4-6 months for the past year and a half. I was prescribed birth control, but I don't want to take it because I don't want to rely on it each time a new cyst could form. I cannot stand when the pain happens though and it feels like I'm being stabbed in the left ovary and the pain moves all the way to the right. Is this the same cyst that keeps causing the pain, or is it a new one? I read they are supposed to go away in 1-3 months on their own. Are there any natural ways of making the cyst go away? If not, how do I stop them from reforming other than birth control? Also can I get more information about cysts because my doctor did not tell me much about it or what birth control will do other than stop ovulating. Doctor: You should have detailed sonographic evaluation of abdomen and pelvis for abdominal pain to find out any other reason than ovarian cyst. After that only treatment can be started."
},
{
"id": 134377,
"tgt": "Suggest remedy for persistent chest and shoulder pain",
"src": "Patient: Hi I m having chest pains & shoulder pains nearly a week now feels like my shoulders are heavy n pain in the left side of the chest didn t last long it was on & off pain. Now I m getting pain in the right back side of my shoulder and in the chest... I m kind of worried. My parents both have heart problems my brother too so scared. Doctor: hi,first of all if you have persistent pain you should obviously go and meet a physician. because taking the number of days will only make you worry more unnecessarily. second it may not be s cardiac issue as you will be young enough. and if at all you are feeling you are having cardiac issues then I will recommend and advise you that once meet a cardiac physician and get yourself checked.what say?"
},
{
"id": 24208,
"tgt": "Suggest treatment for coronary heart disease",
"src": "Patient: Dear Sir, My MVR & CABG was done on 1998 and I with Acitrom (anti coagulant). My Doctor advice for MECABALAMIN, 1 mg/day for one week then 1mg/week for one month & 1mg/month for one year (injection). Can any intrection or contradiction with Acitrom any? Thanks. N.K.Jha, India. Doctor: hello Mr. Jha,I am Dr. Deepak. You have been acitrom for a long time. You will be kowning about the Acitrom, the precaution you are supposed to take. now that your doctor has prescribed Mecabalamin, which is a multi-vitamin. You can take with acitrom. There is no contraindication or interaction. It is mainly Vitamin B12, which helps with increasing blood count and for nerve repair or pain. if you take more also nothing will happen as it is water soluble, whatever that is excess will be removed by the kidney.Thank youHope your concern is resolved."
},
{
"id": 94325,
"tgt": "Have pain in abdomen, back pain, fainting and tingling while peeing. Remedy?",
"src": "Patient: Hi, These are all the symptoms I have at the moment: Tingling when I pee,especially after sex, Needing to pee to the point I feel like im going to wee myself, feeling faint and having fainting episodes feeling lethargic and generally unwell dull achy pain in lower abdomen dull achy pain when having sex lower back pain and occasional lower back pain when i breathe in What could this be?I was thinking it might be a UTI but im not sure,because I dont get a sensation when i pee and it doesnt hurt to pass urine Please can you help! Evelyn Doctor: Hi welcome to Health care magic forum. It appears that your partners veginal PH might be acidic, due to some infection hence the irritation while peeing after inter course, for this both the partners should be treated completely . I advise you to show her to a gynaecologist for diagnosis and treatment. the things you have described are symptoms of chronic U.T.I. caused by continuous exposure. All your symptoms suggest some infection or malnutrition. I advise you to consult a physician for diagnosis and treatment,You may need to have C.T.scan of the lower back along with other routine tests for confermation. Wishing for a quick and complete recovery and thanks for calling."
},
{
"id": 56116,
"tgt": "What is the suggestion for high SGOT and SGAT levels?",
"src": "Patient: Hi, my husband just got blood test result doen and his SGOT and SGAT level are high. doctor says his liver function is worse. Doctor also wants him to go through ultrasound first and than go to specialist for detail tests. What should be next step for us? Doctor: Hi and Welcome to HCM.I appreciate your concern for your husband.I have gone through his problems highlighted in form of raised liver enzymes, etc and further investigations like ultrasound.He must avoid taking alcohol (if he consumes), fatty vegetarian and non vegetarian foods, fast and junk foods, he must take fruits and green vegetables with a healthy balanced diet.Apparently he is having liver disease as reflected in the form of high SGOT and SGPT. He needs other liver function tests and ultrasound to ascertain the exact diagnosis. And the treatment depends upon the cause detected. Meanwhile he must avoid the things suggested above.Wishing your husband a smooth and speedy recovery."
},
{
"id": 131717,
"tgt": "What could cause upper buttock pain around sacroiliac joint?",
"src": "Patient: Hello, I have had upper buttock pain around the sacroiliac joint for 2 years - right side. I also have pain in my right calf and the outer side of my right foot. I have seen various health professionals and had 2 MRIs plus other scans and x-Rays..... Everything normal. I am now seeing a chiro who thinks that it is a sacroiliac problem with trigger points in the surrounding muscles. Question... Could a problem with the sacroiliac joint and trigger points affect the sural nerve thereby causing localized pain AS WELL AS pain in my foot? Doctor: highly unlikely you probably have both sacroiliac problem along with L4/5 problem to make sure try walking in a fast pace for 5 min if u have sacroiliac problem your pain should decrease gradually till it fades away also your leg pain should increase (simple test to confirm my opinion)"
},
{
"id": 643,
"tgt": "Can a tubal pregnancy cause negative pregnancy results?",
"src": "Patient: I was just wondering if you could get a negitive pregnancy result if you had a tubal pregnancy? My last menstral cycle was March 20, today being April 3rd. I'm 14 days late. Took two tests both came back negitive. Minor stomach cramps here and there nothing major. Can you please help? Doctor: Hi, Thanks for the query. I understand your concern. I feel there is some error in writting last menses date. Any way.. I take it as 14 days delay in periods .. One thing I want to make clear.. urine pregnancy test is reliable after 8 days delay in period with regular menstrual cycle. Pregnancy test done to diagnose pregnabcy shows a weak +ve with faint test line). In case of doubt blood urine test or usg scan at 5 weeks of pregnancy can be done. Thanks."
},
{
"id": 190874,
"tgt": "Swelling gums with lots of pain",
"src": "Patient: i am 24 years old male have swelling in gums in left down 7th teethfrom last 3 days there is about to grow another teeth in position no 8 wthere is lots of pain what should i do now i am 24 years old male have swelling in gums in left down 7th teeth from last 3 days there is about to grow another teeth in position no 8. there is lots of pain what should i do now? Doctor: Hi, You have an impacted third molar tooth. The condition is known as pericoronitis. It is an inflammation of the gums around the tooth. You have to get an OPG xray taken and your dentist can identify the actual position of the tooth. If the tooth is angulated it will have to be removed surgically and the problem will get resolved. Use salt water gargle 4-5 times a day and consult a Oral and Maxillo-facial surgeon at the earliest."
},
{
"id": 42527,
"tgt": "What should be the normal value for semen analysis?",
"src": "Patient: Iam 30 year old,my Semen Analysis report is colour - grey white, volume - 2.0ml,reaction - alkaline, total sperm count is 45 million/ml, and progressive Sperm is 50%, non-prograssive 20%, non-motile 30%, sperm morphology unremarkable, pus cells 3-5 /hpf, are these report normal..plz reply Doctor: hai,your semen report is normal as per your quoted text.volume,ph,total count were within normal limit.active forward progression should be more than 50 %- your's s 50%- considerably normal.pus cells- occasional pus cells will be there-it indicates infection, but normal semen didn't contain pus cellsmorphology.is normal (typical cells) I conclude your semen report is normal.Thank youhope i answered your query."
},
{
"id": 16634,
"tgt": "What causes hand, fingers and shoulder tingling pain after CABG?",
"src": "Patient: Hello Doctor My father aged 67 years recently had CABG on Jan 21 2014. Discharged on Jan 29 2014. From last Sunday he is having pain in the left hand. It starts from the last fingers and raises till the shoulder. Its like tingling pain. Its paining only in the night. In day its minute. Please suggest me. I just called to the hospital they told me this pain will be there and if pain is severe they suggested to come to hospital. He is diabetes and BP person too. Doctor: Hi, I understand that your father had a CABG done. The surgeon takes a graft from an artery near the shoulder during which sometimes a nerve near it may also get involved. This may cause the pain you are telling about. Further Diabetes also causes neuropathy.I will suggest you take some Vitamin B12 supplement which helps in such type of pain. You can also see a neurologist if the Pain is severe. Hope I have answered your query. Let me know if I can assist you further. Regards, Dr. Sameer Maheshwari, Cardiologist"
},
{
"id": 118956,
"tgt": "84 years. Nausea, bit upset stomach, high BP. Taken gravol. Advice",
"src": "Patient: first and foremost I do not have money if there is a charge I am 84 and relatively in good health, I do have high bloodpressure and take a blood pressure pill once a day, other than that I only take vitamins. At this point I am very nauseous, and have just taken a gravol. which so far did nothing for me, still feel nauseous. My stomach seems a bitupset, at my age I get a little anxious. What do you think I should take or do. Doctor: Hi,It seems that you might have hyper-acidity or gastritis.Take Pentaprazole or Omiprazloe as and when required.For severe nausea you may take tablet Domperidone.Avoid fried, chillies and junk food.Take light diet like curd, rice, buttermilk.Ok and take care."
},
{
"id": 123955,
"tgt": "Suggest treatment for nerve boil under arms",
"src": "Patient: I have pain under my arms which is more like a boil. It was small for more than a year beneath the skin and now it has bulged to an inch. I am told it is a nerve boil which tends to someday rupture. Is there any medicine I can take or is getting operated is the best solution? Doctor: Hello, It could be an enlarged lymph node. Any swelling in the body must be explored & sent for histological examination. Hope I have answered your query. Let me know if I can assist you further. Take care Regards, Dr Nirmal Chander Gupta, Orthopaedic Surgeon"
},
{
"id": 88728,
"tgt": "Is cinex sufficient to heal from abdominal ascites?",
"src": "Patient: hello sir. i started tb treatment on 25th dec10. i was diagnosed with abdominal ascites. i am on r cinex. i have continuous pain in right chest and rt middle stomach. i experience chills too. i am 29 years old and my weight is 86 kgs, i have a dots who is 2 yrs now. Doctor: Hi.As per your history of abdominal tuberculosis, the treatment is a combination of minimum 4 drugs for the initial 2 months. After a proper response on clinical , laboratory and ultrasonography reports, the 2 medicines in combination of R-Cinex is started and to be continued for minimum of 4 to 7 more months. If you have completed the first 2 months of treatment, R-cinex will be sufficient to for abdominal tuberculosis and ascites. The pain in the right chest and right middle stomach can be due to sequel or complication of tuberculosis. It would be better if you request for CT scan of the chest and the abdomen to know the exact problem. There will be no problem with the daughter or other family members, you can certainly get investigated your family members for TB."
},
{
"id": 115900,
"tgt": "How to treat low platelet count, also have aortic regurgitation and stenosis?",
"src": "Patient: i am 48 years male and suffering from low platelet count i am also a known case of bicuspid aortic valve server stenosis with aortic regurgitation on 25 mg metaprolol my physician advised me to take wysolone 20 mg morning 20 mg after lunch and 10 mg at night with half tablet of ditide 10 mg in morning at present my platelet count is 90,000 please advise me Doctor: Hi,Thanks for asking.Based on your query, my opinion is as follows.1. Platelet count of 90000 is not be worried about. It is a bit below the normal limit.2. Due to aortic regurgitation and stenosis, increased risk of thrombus formation and hence low platelet count is possible. Also ITP and Vit B12 deficiency needs evaluation.3. Get a bleeding time, and bone marrow study for evaluation of low platelet count for evaluation of immune thrombocytopenic purpura.4. Platelet count below 20000 is serious, otherwise, nothing to worry. Cause needs to evaluated. If severely low, platelet transfusion can be given. Continue all other medications including steroids, ITP could be a major cause.Hope it helps.Any further queries, happy to help again."
},
{
"id": 167079,
"tgt": "Is diet green tea bad for children?",
"src": "Patient: Is diet green tea bad for children?. I have recently went on a diet to loose weight and wanted to know if diet green tea is good for loosing weight?. My 6 year old daughter drunk some diet green tea and a few hours later she was complaining about her head hurting. Took her to the doctors office and her doctor said it was bad for her. Just wanted to get a second openion. Doctor: dear parent please dont forget that green tea contains caffeine, of course in lower concentration than red tea and coffee.. thats why we dont recommend drinking green tea for children"
},
{
"id": 94752,
"tgt": "Flank pain radiating to pelvis, had fever and chills. History of kidney stones and anemia",
"src": "Patient: I am a 32 yof with 2 days of left flank pain that sometimes radiates to the left pelvic area and also oddly to the right flank. The flank pain is mostly dull and only occasionally sharp/ stabbing in nature. I had a low grade fever last night with chills, but it broke during the night and no fever since. I am extremely lethargic as well. I have a history of small kidney stones in the left kidney that I have always been able to pass before. I also have a history of anemia. I have had a change in bowel habits over the last 2 months and have been more prone to diarrhea, but I attributed this to two rounds of antibiotics given since 7/11/12 ( Doxycycline and Cephalexin ). What could be wrong with me? Doctor: Hello madam, I have gone through your problem. A past history of small kidney stones on the left side and now having left flank pain radiating to the left pelvis indicates that the small stones have slipped down and traveling through the left ureter and got blocked at some point in their course. Now they might be causing obstruction to the urine flow from kidney to the bladder. This can cause Hydroureteronephrosis. A history of fever in this situation indicates an infection in the obstructed urinary pathway and demands an urgent medical attention. Due to this you may be extremely lethargic. History of anemia may be due to excess loss of blood during menstruation or it may be due to iron deficiency anemia. You need to consult your gynecologist if you have excess bleeding during menstrual periods. If you don't have any issues with your menstruation, then you need to consult your physician and get an peripheral blood smear, iron profile, and serum folic acid and vitamin B12 levels. Change in bowel habits and diarrhea may be because of the antibiotics you have taken. My advice to you is that you need to consult a urologist immediately and get evaluated. You may need hospitalization and intravenous antibiotics. Depending on the size of the stone, the level of obstruction and the severity of obstruction further management differs. Wish you a fast recovery."
},
{
"id": 18806,
"tgt": "What causes a discomfort feeling in heart suddenly?",
"src": "Patient: As I was sitting at work I got a fluttering/vibration feeling in the back of my head and felt like I was going to pass out. I also felt like I saw stars. I am alittle shaky, however, I have eaten lunch today and don't have any other medical conditions other then a mixed connective tissue disease. Is this something to be concerned about? Doctor: Hello and Welcome to \u2018Ask A Doctor\u2019 service.I have reviewed your query and here is my advice.A vibration feeling in the back of the head is usually a result of high blood pressure, but seeing stars is due to low blood flow to your retina and brain which makes you feel as if you are going to pass out.It happens to anyone on a long tired day, but with a history of mixed connective tissue disease, you may need to do 2 investigations which are Carotid Ultrasound Imaging to check out the main source of blood supply to your brain and retina, and Echo cardiogram to check out your cardiac output.In case of any positive results, don't panic .. there are many ways to treat them.Hope I have answered your query. Let me know if I can assist you further.Regards,Dr. Mario Aiad"
},
{
"id": 6061,
"tgt": "Taken siphene, progynova, hugog, duphaston. PCOS problem. Why did conception not occur?",
"src": "Patient: My lmp was on June 26th. I took Siphene 150mg for 5 days from D2. From D7 to D12 I took Progynova 1mg for 3 times and on July 9th my ovary size was 34x31mm and 24x21mm. I took Hugog 5000 IU IM and met on next day..)n 16th day I started Duphaston 10mg for 10days and got my periods on July 24th. Why could not i concieve. I have PCO problem since 9 years and a very small fibroid of size 3mm. I am working as a teacher and travel by auto. What could be the reason for missing pregancy. Doctor: Hello. Thanks for writing to us. The findings of ultrasound are suggestive of ovulation but with PCOD, repeated trials are needed for a successful conception. You have to be patient and try again. After rupture of follicle, have intercourse at least twice in next 24 hours. I hope this information has been both informative and helpful for you. Regards, Dr. Rakhi Tayal drrakhitayal@gmail.com"
},
{
"id": 184895,
"tgt": "What can be the reason for swelling in the gums?",
"src": "Patient: Hi, my 21 months old daughter had high fever a few days ago, I notice her gums was swollen? Why is it swollen? Is it due to teething? What's the cure as she cry when she eats, I must be very painful when she bites. Could you please let me know what i can do to help my child? Is swollen gums a command problem for toddlers?Many Thanks. Doctor: Hello!Thank you for posting here.Yes,children are prone to gum problems.Nearly 70-80%% of all children suffer gingivitis.This can also be due to teething.A clinical examination is necessary to help you.You did not mention the general oral hygiene status.Maintain good oral hygiene status.Brushing twice daily is a must.Watch on her brushing technique and do not over use mouth wash.Use a warm saline gargle thrice daily.Clean with a wet sterile gauze over the mucosa daily.Gargle after every meal and snack.Sweets can be taken as part of meal if she cannot avoid them.If this is due to teething,give a Otc pain killer.Give a teething ring which must be clean.Do not allow to chew on anything else.Regards."
},
{
"id": 90509,
"tgt": "What causes lower abdominal pain on the left side during sex?",
"src": "Patient: i have lower abdominal pain on my left side. i experienced a sudden pain in about the same place a few weeks ago while me and my partner were having sex. we stopped immediately and it went away. theres no real tender spots when i push on the area either. ive just finished my menstrual cycle so i'm not too worried at the moment because i figure it has something to do with that but i've never experienced it before and i just want to know what sort of things i should be looking out for, like symptoms of something? thanks Doctor: Hi.Thanks for your query.The pan during sex can be the first alarm sign of pelvic inflammatory disease or tubo-ovarian problem like inflammation.I would suggest you to take an opinion of a Gynecologist and undergo minimum of ultrasonography to find and get treatment. Avoid sex till you are diagnosed and treated well."
},
{
"id": 19109,
"tgt": "Suggest treatment to control high BP",
"src": "Patient: My age 32. I have high blood pressure. Last 3 year, my doctor give me amdolopin 5gm per day for High BP & it is in control. But some day ago My BP is again high So, my doctor change my medicine. After taking the new medicine, now my BP in control but my penis is not hard for any time. Sometime it hard but not long time. I have share it with my wife. Pls give me a solution. Doctor: Hello,There are some drugs which have erectile dysfunction as a side effect as beta-blockers are one such class. So avoid drugs as metoprolol atenolol or carvedilol. You can take tablet telmisartan once a day. Decrease fast fried foods go for morning or evening walk at a moderate pace keep your sugar and lipid profile you should take multivitamin antioxidant combination once a day. Hope I have answered your query. Let me know if I can assist you further.Regards,\u00a0\u00a0 Dr. Varinder Joshi"
},
{
"id": 77829,
"tgt": "How to treat costochondritis causing pain in my left upper breast?",
"src": "Patient: Hi, Im 18 and Ive been experiencing pain in my left upper breast. Right near the armpit as well. Its sort of between the two areas. My doctor said I might have clostochondritis. How can I avoid the pain and make it better? Im a 2nd year Medical Technologist student and stress is unavoidable. And also I had UTI last year, and I think it may be coming back. My hyper acidity attacks from time to time. I'm tired of taking antibiotics since I feel really sleepy and weak whenever I do take them. Please help me :( Doctor: Can be treated with a course of analgesics. would also suggest physiotherapy and local applications along with hot fomentation. . further treatment requires evaluation of the cause. ."
},
{
"id": 47185,
"tgt": "Suggest treatment for kidney stones",
"src": "Patient: hi, I'm 25 yrs old, 5 ft 10\", couple of years ago i was diagnosed with kidney stones, last year i was told it has gotten worse & now my urine color & smell has changed, i have severe pains on my right side & lower pelvic area, dizziness, fainting spells & loss of appetite. Is it that my kidney stones have grown into something more life threatening Doctor: HelloTreatment of renal calculus depend upon many things like composition,position,size of calculus etc.Small calculus may pass spontaneously with adequate hydration.You should drink lot of water and initially you may need syrup like sodium and potassium citrate.You may also need routine hemogram,random blood sugar,urine RE/ME.You may need need further treatment like lithotripsy etc if findings and symptoms persists.You need proper medical evaluation for dizziness,fianting spells.Get well soon.Take CareDr.Indu Bhushan"
},
{
"id": 81165,
"tgt": "What causes bleeding in the nose and the Coumadin level of 7.1?",
"src": "Patient: My dad had his coumadin level checked by his home health nurse today it was at 7.1 so she told him not to take it for the next 3 days but his nose has been bleeding all day. He is on constant oxygen and wears a cpap at night...how dangerous is this? Doctor: Thanks for your question on HCM.In my opinion you should immediately consult emergency department and get done1. Nose packing to stop the bleeding2. Vitamin K injection3. Injection of other hemostatic agents like (ethamsylate and tranexaemic acid).Your father's coumadin level is very high. So the reason for nose bleeding is high coumadin level. It is anticoagulant in nature. It is vitamin K antagonist. So to reverse its action vitamin K should be given parenterally (injection) to stop the coumadin effect and also the bleeding.He may need nose packing and other hemostatic agents too.So better to take him to emergency room and discuss all these and start appropriate treatment."
},
{
"id": 115840,
"tgt": "Suggest treatment for high creatinine level",
"src": "Patient: Had my appendix removed last weekend and have been recovering quite well. I had blood work done two days after and my creatinine was high so the function of my kidneys were in questions so i had ultrasounds and more blood done which came back fine. Now im having some discomfort in my kidney area and what seems to be underneath my oblique muscles but the discomfort/pain comes and goes. Is this serious enough to go to the doctor? even though my ultra sound was fine and im having no problems urinating. Doctor: Hello,you haven't provided enough details so that I would have a complete picture of your situation, particularly the values of your lab tests. Creatinine elevations can be attributed to very many potential causes. A common - and reversible - one is dehydration. This is accompanied by high urea levels (more than x20 the value of creatinine). Other common potential cause is the use of drugs. Drugs may cause renal injury. Non-steroidal anti-inflammatory drugs may cause renal injury and are commonly used after surgical operations to relieve pain.Any of these conditions and most of the other potential causes of elevated creatinine do not cause kidney pain.I hope I've helped!If you'd like more information, please let me know. I'll be glad to answer.Kind Regards!"
},
{
"id": 217232,
"tgt": "Suggest remedy for neck pain from an injury",
"src": "Patient: I just hit the top of my head pretty hard, my friend was lifting me up and we didn t realize how low the ceiling was and my head hit the ceiling pretty hard right on the top. I don t have any pain on my head but my neck hurts really bad, especially on the left side right at the top of my neck and towards the bottom of my head, and it hurts worse when I look to the right... This literally just happened so I don t know if I should be concerned.. I ve had concussions and I don t think I am concussed I m just worried about the neck pain... What should I do??? Doctor: Here you should use cold pack and hot pack frequently and check if you get any relief. Here I must suggest if your pain is intense and high in intensity then do visit doctor and do mri of cervical spine and brain to check there is no internal damage as well disc problem. Later you must take visit with orthopedic as well as with Physiotherapist for further treatment management. Specially if it's only soft tissue pain then physiotherapy will help you lot for soft tissue relaxation and pain relief. As well you can also take some stretching and strengthening exercise program from physio therapist as per your problem which will help you again to have some more pain relief. Take care."
},
{
"id": 201980,
"tgt": "Suggest treatment for ejaculation during urination",
"src": "Patient: hi i feel ejaculating while or after peeing can you pls tell me why is it happening some times i urinate immediately after pee and the urine colour is transparent. Doctor: Thanks for the query I dont know why u feel like ejaculating after u pee, do u masterbate while peeing?? And also i would like to point out that urination and peeing means the same thing and it is good that ur urine is transparent. It means that u r well hydrated. Meet a urologist Have a healthy living"
},
{
"id": 72435,
"tgt": "What causes dryness in the mouth followed by chest burns?",
"src": "Patient: I have GERD and I was exercising I then started to notice that my mouth was getting very dry and I felt I was getting dehydrated, when I tried to spit nothing came out. I felt sick and my mouth was burning I think it was the acid coming up to my mouth. My saliva was very white. I started feeling light headed. I drank water and I calmed down. My throat and mouth and chest still burns though. What should I do? Doctor: Thanks for your question on Healthcare Magic.I can understand your concern. Yes, possibility of acid reflux is more likely for your symptoms. So follow these steps for better symptomatic relief.1. Drink plenty of fluids orally and keep yourself hydrated. 2. Avoid hot and spicy food. Avoid junk food.3. Avoid stress and tension, be relax and calm.4. Take pantoprazole tablet on empty stomach twice daily.5. Keep 2-3 pillows under head in bed to prevent reflux.6. Quit smoking and alcohol if you have these habits.Don't worry, you will be alright with all these. Hope I have solved your query. I will be happy to help you further. Wish you good health. Thanks."
},
{
"id": 58797,
"tgt": "Hepatitis b virus found in blood. What does the blood test report indicate?",
"src": "Patient: Dear Doctor,I donated my blood previous year & they informed me that it was found Hepatitis B virus in my blood Inactive.I tested my blood and the results are:-HBs Ag ( ELISA) - Non Reactive Sample Absorbance Ration: 0.019Anti HBS (ELISA) >500mUI/mLAnti HCV (ELISA) \u2013 Non Reactive Sample Absorbance Ration: 0.013 Anti HIV 1%2 ( ELISA) \u2013 Non Reactive Sample Absorbance Ration: 0.007Please advice me if I am presently infected with Hepatitis.Is there any problem, if I get married, shall this virus passes to my Wife & Children. Doctor: Hello! Thanks for putting your query in HCM. I am a Gastroenterologist (DM).If you have uploaded reports correctly then cheer up. You are not infected at all.HBsAg is negative means Hepatitis B virus was not found in your bloodAnti HBS is 500mUI/ml means that you have immunity against hepatitis B virus infectionAnti HCV is non reactive means heptitis C virus is also not present in your bloodSo you are not at all suffering from hepatitis and dont delay marriage furtherI hope I have answered your query and this will help you. Remain in touch"
},
{
"id": 57721,
"tgt": "Does breathing and visions problems occur after gall bladder removal?",
"src": "Patient: my friend had gall bladder removed yesterday. Was getting morphine intervaniously at hospital and switched ot oral fast acting 30mg just before trip home. Now having some breathing and visions issues along with worry and insecure thoughts. Trying to get her to relax, breath and change thoughts. Any more I can do for her? She has oxycodone at home and I m thinkind it might be better to take that than the morphine when pain returns. Seems to getting a little more settled down. any suggestions? Doctor: HelloThese symptoms are due to anxiety and fear of pain. Ask her to relax ,stop breathing through mouth,emphasize slow breathing and talk positively.DR SAATIISH JHUNTRRAA"
},
{
"id": 73554,
"tgt": "What causes flu with bark cough, tiredness and irritation?",
"src": "Patient: Hi, may I answer your health queries right now ? Please type your query here...hello my name is lisa, i am 30 years old with asthma. 3 weeks ago i was given a 5 day course of antibiotics and steroids for a chest infection. a week later i went to my doctors and was diagnosed with the flu and given a weeks course of antibiotics, its been 2 weeks now and i feel im over the worst but i have an extreme cough that wont go away. its very tiring and lasts all day and night. i have no phlegm and comes from very deep in my chest. very irritating. could you please help me as to what it is. i would be very greatful. Doctor: Respected user , HiWarm welcome to Healthcaremagic.comI have evaluated your query thoroughly .* This is lower respiratory tract infection with systemic manifestations .* Needs thorough clinical evaluation with necessary blood tests , x-ray chest and intensive management according to the results .Hope this will clear your query .Regards ."
},
{
"id": 35453,
"tgt": "Can i consume non-veg diet while on anti-rabies vaccination for dog bite?",
"src": "Patient: I was bitten by astray pup about 8 months old on my leg & had a small wound .I went to a hospital where they cleaned the wound & administered an ATS & anti-rabid injection & asked me to give for 2 more anti-rabid injections after 3 days gaps.Is this Okay & is it true that no non-veg is to be taken until this period of completing the injections.Would appreciate your advice.Fredrick Doctor: Thanks for post your query to HCM.There is no dite restriction during vaccination for rabies . You are not suffering from rabies you are only taking vaccine as a prophylaxis for rabies . No need to worry you can take your regular dite. However you should watch the boitten dog for 10 days if he died during that period you should take complete dose of vaccination i.e. 5 doses on 0 ,3 ,7, 14, 30th day and booster dose on 90 day.take care of local wound also.good luck , get well soon.regards,Dr.Manish purohitInfectious disease specialist ."
},
{
"id": 186257,
"tgt": "What causes bumps above front teeth in a child?",
"src": "Patient: my daughter is 5 yrs old and is having 2 bumps above her two front teeth. painless and now we find one more beside them pl tell us what to do...we already took her to the doctor and she has been using antibiotics for the past 10 days and no improvement Doctor: Hello, Welcome to healthcare magic, I can understand your concern as this small swelling on above upper front tooth can be due to periodontal problem or if tooth is carious due to periodontal infection or due to eruption of permanent front tooth also . For this you should consult your dentist and go for visual examination and IOPA x ray to evaluate periapical infection or due to eruption of permanent tooth.In meantime adive her to do warm saline gargle two - three times a day. Hope this will help you."
},
{
"id": 121181,
"tgt": "What could cause sore muscles?",
"src": "Patient: 55,147,gall bladder removed. have gerd and take nexium.blood pressure is 121/79.have had scans of pancreas,liver,stomach,colon and female organs. everything fine. why do i wake up and my back muscles and ribs are sore when i breathe and move. as time goes on i get better but still there. Doctor: Hello, Based on your description of complaints you seem to be suffering from morning stiffness. This commonly is seen in older individuals, people suffering from spondylosis and rheumatic conditions. In your case it seems to be routine stiffness seen in people above the age of 45. This is a part of ageing and typically gets better within one or two hours after you wake up and start moving. As you have mentioned that almost all your investigations are normal so nothing to worry. I would recommend you to get your VIT D and VIT B12 levels done , and if low to take tablets to get them optimized. Also do some general stretching exercises and walking this will help, and last thing plz check the quality of your sleeping mattress also. Hope I have answered your query. Let me know if I can assist you further. Take care Regards, Dr. Santosh S Jeevannavar"
},
{
"id": 86614,
"tgt": "Should I take humira for ulcerative colitis?",
"src": "Patient: Hi, i m a 40 year old male, about 7 years ago i was diagnosed with ulcerative colitis and about 5 years ago i was put on humira. in the mean time, I keep having weird symptoms that i associate with the ulcerative colitis flare ups but my doctor says he doesn t think so. what happens is i start having pain at the back of my neck where my head meets my neck, when i yawn my diaphragm spasms(twitches), and i feel a little lightheaded. I get hot flashes and what feels like an adrenalin rush. when i fall asleep my dreams are usually crazy weird and when i wake up my heart starts racing and feel like i might pass out. this last part usually last about 5 to 10 minutes then sort of subsides. these symptoms will get worse and worse then i will get my doctor to give me prednisone to get things under control. the reason why i try to link this with my flare ups is i usually have diarrhea and cramping at this point. my doctors can t seem to give me any answers as to what these symptoms are caused by. the only thing that makes these symptoms go away is taking prednisone. Please point me in the right direction or maybe what this could possibly be. Doctor: Hi.Thanks for your query.Prednisolone is a steroid and if it is helping you the diagnosis of ulcerative colitis is done , confirmed . You may please take Humira and other medicines as per the advise of your Gastroenterologist. Take the following care:Check your blood for Sugar and calcium, prednisone and other hormones particularly Thyroid.Test of the stool repeatedly to see that there is no secondary infection or so, which has to be treated in time to avoid complications. Avoid all the foods and beverages which are known to give or enhance the problems. As you know these problems can pop anytime , yo have to be really careful.Control of the stress and anxiety help a lot to control these things."
},
{
"id": 210893,
"tgt": "What do you suggest for sad state of mind when on medication for chronic depression?",
"src": "Patient: I have been diagnosed with chronic depression/anxiety, on many medications over a twenty year period. But I still feel like my mind stays in a state of sadness. I have been in this state so long that I do not remember if I was ever happy! There are very few psychologist who take medicare, my primary health care provider! Sincerely, Eloise Doctor: Hello,Thanks for choosing health care magic for posting your query.I have gone through your question in detail and I can understand what you are going through.From what you have stated it is quite clear that you are still having a depressive episode. You need proper treatment for the same. You have not mentioned which medication you have tried until now. There are various effective therapies for depression like fluoxetine, venlafaxine, bupropion, mirtazapine or escitalopram and they are all very effective and mostly safe. If you have not tyried them it wold be wise to combine your treatment or switch to these antidepressants. At the end it s a trial and error way of treating the things. You may also require some benzodiazepines like clonazepam or lorazepam for a short period like 2-3 weeks to reduce the anxiety and help with your sleep. If you are averse to the drug therapies then there is also an option of cognitive behavioural therapy. This therapy is taken by either a psychologist or a psychiatrist in which the therapist identifies impaired cognitions (Thoughts) and try to correct them. They are 15-20 min weekly session and there should be around 8-10 sessions to help you with your illness.Hope I am able to answer your concerns.If you have any further query, I would be glad to help you.In future if you wish to contact me directly, you can use the below mentioned link:bit.ly/dr-srikanth-reddy\u00a0\u00a0\u00a0\u00a0\u00a0\u00a0\u00a0\u00a0\u00a0\u00a0\u00a0\u00a0\u00a0\u00a0\u00a0\u00a0\u00a0\u00a0\u00a0\u00a0\u00a0\u00a0\u00a0\u00a0\u00a0\u00a0\u00a0\u00a0\u00a0\u00a0\u00a0\u00a0\u00a0\u00a0\u00a0\u00a0\u00a0\u00a0\u00a0\u00a0"
},
{
"id": 102216,
"tgt": "Any suggestion for having irritation in throat that causes continuous cough?",
"src": "Patient: i have a irritation in my throat that causes me to cough i have had this continually for at least 5 years. iv e had an x ray witch was clear .iv e had nose drops .and a test for asthma doc said i might be border line asthmatic and i tried a inhaler for a month. nothing helped .i am 72 yrs. and in very good health. Doctor: Hello,Welcome to HCM,\\The symptoms suggests me that your are chronic allergic manifestations, as you are having throat infection which is followed by cough for last 5 years suggest me that you may be having some allergic reactions.You need to undergo absolute eosinophil count to know the presence of any allergic reactions. If there is elevated levels of eosinophil count, you need to test yourself for most prevalent allergens like dust, mite, pollens, food and chemicals.This help to find out the allergen which is a triggering factor for your condition and producing all these symptoms. After identifying the allergen you can avoid the allergen or you can go for immunotherapy.Meanwhile for your symptoms you can take Tab LevocastOral AntihistaminesSyrup for cough like codeineThank you."
},
{
"id": 183138,
"tgt": "Suggest medication for an infection in the gums",
"src": "Patient: my sons 9 months old, hes got purple bruised gums where his k9 teeth should be, today ive noticed white spots on the brusing, hes only got his bottom front two teeth, i thought it was the front two teeth that came through before teh k9s, could you tell me why his gums have gone like this? Doctor: Hello, Read your query as you have 9 month child has purple bruised gum dont worry this purple gum can be a erupltion buldge of primary teeth , it can be eruption cyst also , for this I will suggest you to consult Pedodontist and go for visual examination and if needed go for investigations iopa xray to determine the eruption to primary teeth.Hope this will help you."
},
{
"id": 85492,
"tgt": "What are the side effects of Duromine on pregnancy?",
"src": "Patient: I have been taking duromine for a week, my partner and I want to try for a baby. How long do I need to stop taking the medication before we can safey try? What are the potential side effect on the child? We are both healthy. I am 34, never had children. He is 40 and has 3. Doctor: Hi, If you took phentermine before pregnancy, it should have no effect on your ability to carry a healthy baby to term. All traces of phentermine should pass through your body. Even if you took your last dose a week before conception, it shouldn\u2019t have any effect on your pregnancy. Very few human or animal studies have been done on phentermine during pregnancy. But the very few that do exist don\u2019t seem to connect the drug to birth defects. Hope I have answered your query. Let me know if I can assist you further. Regards, Dr. Sameen Bin Naeem, General & Family Physician"
},
{
"id": 28040,
"tgt": "Suggest treatment for heavy heart murmur",
"src": "Patient: I have a heart murmur is kind of heavy sound. And I couldnt get a heavy labor job which i will need to lift 50 pounds unless I go to a doctor and clears me and gives me a note that is ok for me to work lifting 50 pounds what are my changes of the doctor clearing me so I can work. Doctor: I have gone through your question and appreciate Your concern. Heavy heart murmur may be due to valve problem of your heart.That is valve may be stenotic or leaking. Consult cardiologist and get echo done.It will tell exactly what the problem is.Lifting heavy Weights and clearance for job should not be your priority.But first u should sort out what the problem is and seeking treatment Accordingly. Thanks.U can write me back for any query."
},
{
"id": 173401,
"tgt": "Suggest treatment for diarrhea and pus cell level of 2-3/HPK",
"src": "Patient: hellow doctors, My baby girl is about 9 months old and she is having a frequent diearria and we had a stool test and result came like every thing is ok but pus cell is like 2-3/HPK i am in need of suggestion please help and how can it be stopped. Regards, Abhishek Mathema Doctor: Hi...the above mentioned stool test is normal. Do not worry. It seems your kid is having viral diarrhoea. Once it starts it will take 5-7 days to completely get better. Unless the kid's having low urine output or very dull or excessively sleepy or blood in motion or green bilious vomiting...you need not worry. There is no need to use antibiotics unless there is blood in the motion. Antibiotics might worsen if unnecessarily used causing antibiotic associated diarrhoea.I suggest you use zinc supplements (Z&D drops 1ml once daily for 14 days) & ORS (Each small packet mixed in 200ml of potable water and keep giving sip by sip) as hydration is very important and crucial part of treatment. If there is vomiting you can use Syrup Ondansetron (as prescribed by your paediatrician).Regarding diet - You can use cerelac...any flavour will do. Avoid fruit juices as they might aggravate diarrhea. You can give zinc supplements & ORS apart from normal vegetarian porridges & soups.Regards - Dr. Sumanth"
},
{
"id": 220678,
"tgt": "Suggest tests to know the paternity of the fetus",
"src": "Patient: I HAVE A QUESTION IF I HAD UNPROTECTED SEX ON MAY 9 & MAY 10 HAD A PERIOD THAT WAS LITE AND ENDED ON MAY 27TH. THEN HAD UNPROTECTED SEX WITH SOMEONE ELSE ON MAY 30TH AND IT JUNE 24 AND I HAVE NOT GOT MY PERIOD. WOULD I BE PREGANT BY THE FIRST GUY OR LAST GUY. SORRY I KNOW IT SOUNDS CRAZY Doctor: Hello dear,I understand your concern.Once the period come the pregnancy is 100% ruled out.But the doubt arises as you are saying the period was light.But if there was pregnancy prior you would experience the pregnancy symptoms like nausea,vomitings,increased urination,sore breasts.Because they usually are present 2 weeks after the missed period.Now a urine pregnancy test can be done to rule out the pregnancy as the period is missed.If the test is positive then ultrasound needs to be done.The ultrasound helps accurately in diagnosing the age of pregnancy.And helps in knowing the probable date of conception.But there is a rare possibility of pregnancy with first guy as you got the period and experiencing no pregnancy symptoms.Hope this helps.Best regards..."
},
{
"id": 18773,
"tgt": "What causes low BP?",
"src": "Patient: hi....m shilpa n my age is 22......i had an intake of 28 disprins on friday n nw m nt able 2 hear properly......i feel fainted as i stand....my b.p. is also vry low........i wnt 2 knw dat wat wud happen 2 me nw?????do i need 2 c a doc or gv sm medical advice pls.......my e-mail id is YYYY@YYYY Doctor: Hello and Welcome to \u2018Ask A Doctor\u2019 service. I have reviewed your query and here is my advice. Low blood pressure can not be it self (Hypotension) it happens when there is loss of fluid from body, have some specific pathology, but most of the time it is the orthostatic hypotension, it happens when a person stands abruptly from sitting or sleeping position it causes the blood pulled down against the gravity and that the reason for fainting, giddiness but it is transient condition may not need any treatment, you can see the doctor on routine basis because other causes of fainting can not be neglected and it has to be ruled out. Hope I have answered your query. Let me know if I can assist you further."
},
{
"id": 137652,
"tgt": "What is the treatment for the damaged olecranon bursa?",
"src": "Patient: Hi, I have recently hit my elbow and apparently damaged the olecranon bursa. I have the classic swelling but no tenderness. I have seen a doctor and even have had the elbow drained once, but the swelling has returned again.......which the Dr. said was very likely to happen. I realize from the online research I have done that this condition takes weeks or even months to subside on its own. My question is simply this: Will indomethacin, an NSAID that I have taken in the past for gout like symptoms, assist with the swelling going down at greater rate, or does any medication help at this point? My elbow is not infected or tender, just swelled with the bursa as big as an oversized golf ball. What about indomethacin? Doctor: Hello,I have studied your case and you need aspiration of the fluid once again. if there is recurrence then this bursa needs to be excised. Otherwise it will not going to improve.I would also recommend you to take physiotherapy in form of warm water fomentation. Avoid hitting your elbow to any hard surface and compression bandage is also helpful."
},
{
"id": 211045,
"tgt": "What causes depression, mood swings and weight gain?",
"src": "Patient: Hi I want some help with hot flush's and all that goes with it, depression , sleeping , gaining weight, mood swings, Iam 67 and have been going through this for about twenty five years. I had cancer of the lining of the womb and have been told that I can not have hormone replacement , is there anything I can do.Valda Doctor: Hithanks for using healthcare magicIn your there may be multiple reason like hormone imbalance after menopause or metastatic change in thyroid gland or may be due to certain drugs. You should get your thyroid profile done to rule out hypothyroidism and consult a physician for further query. If it is due to hypothyroidism then you would improve with thyroxine hormone otherwise you can go for antidepressants.Thanks"
},
{
"id": 69377,
"tgt": "What is the red bump on the inner thigh with blood and plasma?",
"src": "Patient: Bump Ive had a bump on my inner thigh thought it was from a ingrown hair looks like a big zit size of a m.n.m red nothing comes out but blood and I guess plasma its clear just having discomfort not pay and its always closed never pops or anything Doctor: Hi.Thanks for your query.The infection has spread to the surrounding tissues and hence the inability to pop or so. Take a course of an antibiotic and anti-inflammatory medicines as this infection has tendency to spread more. Also check for blood WBC and sugar to be on a safer side"
},
{
"id": 22561,
"tgt": "Suggest remedy for high blood pressure",
"src": "Patient: my grandmothers blood pressure is very low right now and a few hours ago it was very high. i don't know what to do and im really stressed. shes lying on the couch saying she feels cold and dizzy with a bucket next to her because she feels nauseous too. PLEASE HELP is there something she can eat or do to help get her blood pressure back up? we counted 150/70 on the little machine. please help Doctor: Hello,Her blood pressure reading you just mentioned is 150/70, right? It's not a low reading. In fact, it's a high reading. No need to worry about BP. She will need bp lowering medicines. Don't worry about lowering BP readings. Monitor blood pressure and if it remains above 140 then needs treatment. Avoid fatty, oily and high calorie diet, and low salt diet. Regular exercises like walking, according to patient capacity at 30 min a day and 5 days a week. Lots of green leafy vegetables, fruits, fish once or twice a week, avoid meat. Avoid smoking and alcohol if any. There shouldn't be abdominal fat deposition or obesity. Regarding medicines, you can ask local for tab Amlodipine. This nausea feeling could be gastritis. Avoid spicy foods. Also maintain sleep habits regular. Have some walk after meals instead of taking rest immediately. She can take acid suppressant like tab pantoprazole before breakfast.Hope I have answered your query. Let me know if I can assist you further.Regards, Dr. Sagar Makode"
},
{
"id": 194548,
"tgt": "Can the yeast infection become systemic?",
"src": "Patient: I am male and have an active yeast infection (jock itch). I decided it would help if I shaved my pubic hairs. As I was shaving, I nicked myself in 2 places. The nicks bleed, but not bad or for very long. I applied nystatin cream on the area, including nicks, when I got out of the shower. I am also taking oral medication (terbinafine). Should I be concerned with the infection become systemic? Doctor: Hello, If you have a systemic fungal infection then fever might be present. You will mostly have a topical fungal infection. For that, you can apply topical clotrimazole cream over that. Hope I have answered your query. Let me know if I can assist you further. Take care Regards, Dr Parth Goswami, General & Family Physician"
},
{
"id": 206552,
"tgt": "Suggest remedy for stress and abnormal negative thoughts",
"src": "Patient: hello doctor im not a troll or any thing like that i need help i think im mentaly ill i some time want to kill lots of people but i do not like the idea of killing i have had lots of stressfull problims my hole life and i have kept them all locked in side my head i see things and hear things i dont like please give me some advce to help me fix my head Doctor: HelloYour description suggest that you have depressive disorder.For its treatment, you can take tablet zosert 50 one tablet every night. Alongwith that, you can take tablet sizodone 1mg every night. You will better in 10days of taking medicines.Thanks"
},
{
"id": 154144,
"tgt": "How to cure prostate cancer completely?",
"src": "Patient: My father's prostrate cancer returned last yr in August. He has completed 7 rounds of chemo. Two weeks ago they mentioned to my dad that the cancer has spread to other bones and now there are a couple of spots on his liver. His PSA level the last time they took it was 31 and two weeks ago it was 88. I know everyone is different however the dr would not give us a time frame for my dad. A week ago the pain was not bad and he could take a few advil and now he is on 5mg of morphine. Doctor: Hi,Thanks for writing in.Cancer is a challenging condition and prostate cancer that has spread to bones and liver is difficult to treat completely. It is important to know that the liver lesions can be targeted and treated with new treatments like chemoembolizaton and radio frequency ablation. There are certain criteria to be fulfilled before treatment is given. Chemotherapy also is a good treatment option but radiofrequency ablation and chemoembolization are more target oriented treatments from a closer distance and therefore the success rates are higher.Unfortunately your father might be having stage 4 cancer of the prostate. This requires aggressive treatment and if his bone lesions have responded to treatment in the past then it ids good and there are chances that the disease process can be controlled and stabilized. Pain is due to pinching and irritation of nerves in the lower back due to the spread of disease to the bones. Instead of giving him a regular morphine, it is better to give him morphine when the pain is severe. This way he will not develop tolerance to the medicine and can get pain free. Please do not worry."
},
{
"id": 43277,
"tgt": "Taking fertility treatment, cycle of 25-35 days. When will ovulation happen? What are its symptoms?",
"src": "Patient: HI DOCTOR I'M 20 YRS OLD AND MARRIED. NOW I'M TAKING TREATMENT TO GET PREGNANT FROM THE LAST MONTH.MY MENSURAL CYCLE IS 25-35 DAYS CYCLE.I WANT TO KNOW WHEN THE OVULATION WILL HAPPENING IN MY BODY AND WHAT ARE THE SYMPTOMS FOR OVULATION ALSO I WANT TO KNOW WHAT ARE THE TIPS TO BE FOLLOWED DURING OVULATION LIKE(AVIODING ANY FOOD ITEMS,REDUCE TAKING MORE STAIN BY CARRYING WEIGHT ETC...).SOME DOCTORS SAYS AVOID INTERCOURSE AFTER 25 TH DAY FROM MENSURAL CYCLE BCOZ IT MAY AFFECT OUR EGG IF OVULATION IS COMPLETE WITH SUCCESSFULL LIKE THAT...IS THIS TRUE OR NOT... Doctor: Hello, Thanks for the query to H.C.M. Forum. Let me explain when ovulation occurs as , Menses date 1 September ( bleeding starts ) now ovulation occurs any time 14 th September , 15 , ( in 95 % cases). When ovulation occurs there develops mild elevation of temperature ( 99-99.5 F ). Mild degree tenderness either on left or right side depending upon the site of ovulation . Release of ovum is not effected by intercourse at all , all these are myth. Release of ovum depends upon maturation of ovarian follicles and their ( its) burst . So deal according to your need , everything is scientifically proved. Good luck. Dr. HET"
},
{
"id": 209379,
"tgt": "Suggest treatment for behavioural issues",
"src": "Patient: My one of friend has severe problem related to her behavior. She repeatedly say one thing several times and exaggerates this to the extent that she loses her temperament and starts abusive languages even to her parents. she has two months baby but she does not care for her also. Doctor: DearWe understand your concernsI went through your details. I suggest you not to worry much. The condition your friend is facing in most probability is Obsessive Compulsive Disorder limited to obsessive incessant talking. You need to understand about her total behavior pattern to know whether she has some other obsessive compulsive disorder symptoms. Psychological or psychotherapeutical intervention may be needed initially. As time passes, with the therapy and medicine, she should be alright. Do consult a psychologist soon. If you require more of my help in this aspect, Please post a direct question to me in this website. Make sure that you include every minute details possible. I shall prescribe some psychotherapy techniques which should help you cure your condition further.Hope this answers your query. Available for further clarifications.Good luck."
},
{
"id": 96557,
"tgt": "Is severe headache after a concussion normal?",
"src": "Patient: does a severe headache after being diagnosed with a concussion require a visit to the ER. Football injury last night, was told to go to ER if severe nausea and vomiting but nothing said re: headache!! By the way a little free advice plz, I cannot afford.... Doctor: Well come to HCM Thank for asking Your query being appreciated, any kind of head injury that need to be kept under observatio for some time, if it is the headache after the injury then it has to be seen by the CT brain and CNS opinion is must it can not be taken as granted, I would advise to see the ER hope this information helps."
},
{
"id": 32741,
"tgt": "Is e coli infection causing blood clots in urine?",
"src": "Patient: I recently got very sick. At first I felt like I had a bladder infection, because it was uncomfortable to urinate. An hour later, I noticed blood clots in my urine and the discomfort continue to worsen. Within 2 hours my urine went from being normal color to solid red and I was in excruciating pain during urination. The doctor put me on antibiotics and sent the sample for a culture/sensitivity test. It tested positive for ecoli. I am wondering if even though I am feeling much better, do I need to seek follow up care to ensure there isn't damage to my kidney, etc. Doctor: Hello,It is always good to follow up, and I don't think you will much to worry after the visit. It is just a reassuring visit as your infection has already been resolved, and well treated in time.Yes, E. coli is one of the main culprits of urinary tract infections. It causes bleeding in urine too, along with other symptoms of infection like pain and urgency to urinate.Hope I have answered your query. Let me know if I can assist you further. Regards, Dr. Muhammad Hanif"
},
{
"id": 7587,
"tgt": "Pimples, applied toothpaste, appears all burnt. Prescribed Flutibact to lighten dark spots. Is it helpful ?",
"src": "Patient: Hi , I had pimples a few days back, and I tried curing them early. i applied toothpaste one night on them and slept. I read on forums it was an effective remedy. To my surprise I woke up and washed my face to see it all burnt. Those spots I applied toothpaste have become big and brown almost. I visited a dermatologist , but he prescribed a cream Flutibact . i don t know if this will lighten those dark spots though. i am not using any skin lighening cream as well. Could you please tell if flutibact is good for this usage and how effective it is in cleaning marks. how long will it take to get back to my normal skin now. Please help. . :( Doctor: Welcome to HealthcareMagicForum Hi..megha.. pimple are cause due to increased hormonal variation in the adolescent age.. you must not manipulate with finger or try to treat by local remedies, this will flare up new ones.. continue with above antibiotic cream, it will reduce.. Take care.."
},
{
"id": 220267,
"tgt": "What causes dizzy spells, diarrhea & stomach pain during pregnancy?",
"src": "Patient: hi my name is nikita i recently found out that i am pregnant,i experience dizzy spells occassionally,black out ,constantly tired,diahorrea,severe pains in my stomach and back and diabetes also runs in my family what can i do to stop this and can it be severe to the baby that i am carrying Doctor: Dear Nikita,Since you have diabetes in your family, you have to get yourself investigated for diabetes. It is lekely that your diabetes which was in latent phase so far has become overt now. Pregnancy is a diabetogenic condition. Therefore, please report to your Obstetrician and get your blood sugar investigated. If diabetes is detected, it needs to be controlled by either Insulin or by oral medicines. Diabetes leads to many complications in pregnancy which includes high blood pressure, anaemia, urinary infections, foetal anomalies, diabetes to baby, oversized baby, etc. You will have to keep a watch on this. Diabetes itself with or without high blood pressure. and anaemia can cause all the symptoms you have described. Get your blood pressure and blood for haemoglobon checked. Accordingly, you may require treatment. If you are not taking iron-folic acid tablets, please start them. Consume high protein diet. Diarrhoea with stomach pain can be due to amoebic dysentery or worms. Stools examination will clinch the diagnosis. Amoebiasis can be treated with Metronidazole or Secnidazole. If worms are found, you will have to take deworming medicines with Obstetrician's advice as some of them can affect the baby.Most urgent step should be to get investigated for Diabetes.I hope this helps you. Dr. NISHIKANT"
},
{
"id": 158201,
"tgt": "Abdominal surgery for uterine cancer due. What points will be discussed during post surgery appointment?",
"src": "Patient: My surgery is scheduled for Aug. 22. I have asked my doctors office in various ways what to expect and seem to get put off or told we will discuss that at the post op appointment that is 2 days before the surgery. I want to know now so that I have time to process all that is going to happen. I am having abdominal surgery for uterine cancer. Any suggestions? Doctor: Dear,when it is early stage of uterine cancer you need to have removal of uterous. In advanced stages method of management may be different. If you are post menopausal you do not have to worry as the uterous will be removed and you will be free of tumor. Further management will depend upon the histopathological finding. Pre-operatively the surgeon will assess wheather you are fit for surgery. truly,Dr. J. Ticku"
},
{
"id": 31992,
"tgt": "Suggest treatment for allergic insect bite",
"src": "Patient: two days ago, when i went to lay down in bed, i felt a bad sting, and i got up and ask my husband to look at it and when he did, i had a huge knot on my right shoulder blade at the top with a hole in it, with pus and blood coming out of it. So he squeezed all that out, and put alcohol and peroxide on it for me. It burned and stinged so bad, to where i couldnt lay on that side for two days and it still hurts to touch it and lay on it. I have been experiencing pains in my heart and drowiness, coupled with diarrhea. Did i get bitten by a spider? And it also now has a black head and is still swollen. Doctor: Hi, Thanks for posting in HCM. I understand your concern. Experiencing pain, drowsiness and diarrhoea followed by insect bite could be due to biting by black spider. It is mainly due to allergic reaction what he is experiencing such things.Kindly take the following measures to overcome the problem he is facing: 1. Tablet. Loratidine for allergic symptoms. 2. Tablet. Ibuprofen + Paracetamol for pain, swelling and inflammation. 3. Clean the wound site twice everyday with mild antispetic solution like Savlon. The symptoms should subside in about a week time. Hope the information provided would be helpful. All the best."
},
{
"id": 89594,
"tgt": "What is the treatment for stomach pain in the right side?",
"src": "Patient: I have been having an uncomfortable pain right on my stomach and it comes and goes. It is so painful that I double over almost not able to breathe. I thought it would be gases or something, but nothing I've taken helps. It makes it worse. What could these pain be and what can I take to relieve these symptons? Doctor: hithanks for askingi would like to ask about your age and gender any recent heavy work,duration of pain and any medications...........if you are a female and fatty then most probably the symptoms you have given is that of gall stones.........i would suggest to do an ultrasound abdomen and pelvis,full blood count,urea breath test........meanwhile you should take 1) plenty of fluids2) painkiller tablets3) cap omeprazole 40 mg once daily4) tab no-spa forte 1 tab three times dailyyou should consult a surgeon"
},
{
"id": 119884,
"tgt": "What causes knee pain, swelling and cracking sound while walking?",
"src": "Patient: Hello, my right knee is very painful, swollen and cracking after a loud crack the other day while walking. The pain is getting much worse. I don t have insurance until Nov 1st and was wondering if I should go to the emergency room or give it a few more days, It s getting to the point where it s near impossible to walk. Doctor: Hello,Your symptoms could be related to a ligament rupture or menisque damage (part of the knee joint). For this reason, immobilization may be needed. I recommend avoiding walking and putting some ice on the knee, have some rest and taking ibuprofen for the pain. If your situation does not improve, consulting with an orthopedist and performing a knee MRI may be needed.Hope I have answered your question. Let me know if I can assist you further. Regards, Dr. Ilir Sharka, Cardiologist"
},
{
"id": 115978,
"tgt": "Suggest methods to increase the platelet count",
"src": "Patient: I have a low platelet count problem. I have been facing this problem since long time. I feel weakness and don't get proper sleep (sleep only for about 3 hours in a day). I have to work during the day, but since I don't get proper sleep, this issue is of major concern to me. I eat a well balanced diet, fruits, Kellologs etc. I also take vitamin tablets and Raricap medicine. Please advise how can i solve my problem. Doctor: Hello and welcome to HCM,Low platelet counts occurs due to reduced platelet production or increased destruction of platelets in blood or destruction of platelet precursors in bone marrow.Platelet count is not affected by diet.Thus, you need to get peripheral blood examination, platelet parameters and bone marrow examination if required.The above mentioned investigations will determine the cause of low platelet counts.Thanks and take careDr Shailja P Wahal"
},
{
"id": 190646,
"tgt": "Having swollen, painful and sore jaw. Feels like tiny pit in jaw. Not cured by campho-phinique. What could it be?",
"src": "Patient: I m having pain in my jaw and it has been accompanied by a sore (maybe more than one) In the crease where my jaw, teeth , and cheek meet. when i feel it with my tongue it is swollen and feels like a tiny pit in my cheek/jaw. i have been using campho-phinique and oragel to attempt to treat it for 3 days to no avail. What could it be? Doctor: hi sir/mam i have read ur query based up onmit i suspect it may be due to cheekbite i suspect u may have wisdomtooth (thirdmolar) incorporating ur masticatory activity so ur cheek muscles may got hurt when u bite something solution is go to dentist take opg find out thirdmolar position and get extracted appl topical application like dologel \\dentogel\\zeehex gel for ur sore thanks dr.karthikeyan virukshaa dental care coimbatore"
},
{
"id": 220331,
"tgt": "Is sex safe during pregnancy?",
"src": "Patient: 1 during pregant period shall we do sex?2.from which month onwards shall i do sex?3.from which month i want to stop sex during pregant?4we want to put condom during sex when my wife pregantplease i have doubt please clear my doubtplease doctor reply soon... Doctor: Hello dear,I understand your concern.Here are the answers for your queries:-1)Yes you can participate in intercourse during the pregnancy.But it is avoided during the first 3 months and last 2-3 months of pregnancy.2)Condom is not necessary during intercourse as there is no risk of pregnancy.3)In case of complications like spotting or pain abdomen or any vaginal infection better to avoid intercourse during the pregnancy.Have a happy pregnancy and delivery.Hope this helps.Best regards..."
},
{
"id": 146833,
"tgt": "What causes sudden loss of vision and dizziness?",
"src": "Patient: I'm 20 yrs old and a few times now had episodes where suddenly my vision goes and I can't see anything I feel all sweaty and dizzy and sometimes collapse . This has happened about 4/5 times now . It can happen even if I'm just sat down . What can be causing this Doctor: Dear user, thank you for your query, please get your blood sugar level tested, if you are taking any medications refer to your doctor, plus once you feel that again go to nearest clinic get blood pressure heart rate and blood sample for basic hormonal check up. hope that helped. stay safe.Best regards"
},
{
"id": 87663,
"tgt": "What is causing my back and abdominal pain with blood in urine?",
"src": "Patient: I feel like i have been in labor since Sunday. I am 45 and not pregnant. Pain in the back and abdomen, pretty painful at times, vomiting and blood in urine. was menstruating for about 4 days and stopped. Have had catscan and blood test come back clear. Doctors gave me pain medication and told me to see gyn. Any ideas? I have had kidney stones before but a little different pain. Doctor: Hi.Thanks for your query and an elucidate history.The history of pain in back and abdomen like seen in labor without being pregnancy vomiting, blood in urine can be due to the following reasons.::-Severe Urinary tract infection along with pelvic inflammatory disease.-Clot in the uterus-Prolapse of the uterus with associated problems of stone in the urinary tract.Since your CT scan is normal and also the blood tests, we have to think about the causes like we discussed about. An internal examination by the Gynecologist can give a further diagnosis and can have a plan of further treatment."
},
{
"id": 96222,
"tgt": "I have been diagnosed with hernia, weird symptoms, please advise",
"src": "Patient: Hello I have been diagonsed with an umbilical hernia. It has been going on for 2 months now. Would this make my groin area cramp and be in pain? It makes it hard to walk most of the time. Doctor: welcome to HealthcareMagic grojn[lower abdominal]cramps ,pain may be due to hernia if bowel loops are there in the umbilical hernia when there is obstruction in the free flow of intestinal contents due to hernia intestinal colics will give rise to the symptoms as described consult surgeon and follow advice .it will require surgery hope you be alright soon"
},
{
"id": 44690,
"tgt": "Is it possible to get pregnant even after having pcod problem ?",
"src": "Patient: hi am married 6 months back now am planning for pregnancy ,am affected by mild pcod problems.can i get pregnant easily. Doctor: Hi Nandhitha, Welcome to HealthcareMagic Forum. Are your Periods irregular? Have you been putting on weight recently? Kindly don\u2019t worry as it is a treatable condition. Kindly get yourself examined by a Gynecologist so that she can examine you and Prescribe medications which release the egg and help you conceive. To increase the chances of conception have unprotected between the 10th and 20th day of your cycle which is the fertile period. Exercise and go for brisk walks for an hour daily regularly and eat healthy nutritious balanced diet which should include fresh fruits and green leafy vegetables. Avoid stress and kindly have patience. Take your medications religiously and go for regular follow up. These measure will definitely help you conceive. All the Very Best, Wish you Good Health, Take Care."
},
{
"id": 64999,
"tgt": "What causes a large red lump on the stomach?",
"src": "Patient: i am 44 5'7 and 288 pounds and have be quite healthy i had a deep black head on my stomach about 3 inches above my pelvis and i tried to squeeze it about 3 months ago but was to deep to get so i have left it alone now this morning i see that it is now a large red lump but is not painful . it is located right next to my csection scar from 28 years ago what could this be ? Doctor: Hi,Thanks for the query to HCM.I am of the opinion that the squeeze 3 mths back had aggravated it to the red-large lump.In my opinion it appears to be a keloid -a fibrous lump-which has grown in the old scar -of the C-Section.Alert-You should act fast and go to surgeon and get the fnac -done and rule out any malignant change in the scar-keloid.As you had no pain and complaints of any fever and discharge on squeez-this is the most probable possible disease.There are many other possibility which the surgeon-would rule out before giving any treats.Hope you got the answer to your query satisfactorily.Wellcom again."
},
{
"id": 38278,
"tgt": "Is herpes related with, bumps on mouth corner?",
"src": "Patient: I have a bump on my upper lip by the corner of my mouth. I researched a bit and it says it could be a cold sore. Something to do with herpes? But I m in my teen years and I haven t even kissed anyone yet or had sexual contact with anyone. It was red then it got kinda white and I messed with it and clear stuff came out. Is it like a pimple or something? Doctor: Hi,Welcome to HCM.Herpes is one of the causes of mouth sores. This is caused by herpes simplex type1 and it is transmitted by contact and not necessarily by sexual contact.Moreover, herpes simplex ulcers occur as blisters which are grouped and filled with fluid. These blisters heal without scarring.I do not suspect herpes in your case based on the description given in your query.It could be acne.Thanks"
},
{
"id": 28832,
"tgt": "How can an infected biopsy wound on the thigh be treated?",
"src": "Patient: I had a biopsy on my inner thigh 4 weeks ago ... think it may be infected ... could not get into the dermatologist who did the biopsy (confirmed hives) for him to check today. I have some Cipro 500 mg that I use for chronic UTI s ... should I start taking it just in case until I can see the Dr. on Monday? Doctor: Hi,It is inadvisable to start a course of antibiotics before confirming the presence of an infection. You could temporarily use an antibiotic ointment and a cold compress on it.Once you meet your dermatologist, he will see the wound site and may even take a swab if there is some pus before putting you on antibiotics so that the antibiotics can be tailored to the organism that is there.Hope I have answered your query. Let me know if I can assist you further.Regards,Dr. Pranav Balakrishnan"
},
{
"id": 115373,
"tgt": "How to reduce bilirubin level in blood while having jaundice?",
"src": "Patient: My sister recently afftected jaundice. Her LFT results as on 30.8.10 & 15.9.10 are Bilrubin 14 & 32.2, Direct 10.5 & 20, SGOT 1461 & 973.3, SGPT 1350 & 49, Albumin 4.1 & 3.6 and Alkaline phosphate 120 & 109.4 respectively. Though SGOT & SGPT are decresing, Bilrubin is incresing. She is taking LIV 52 . Kindly suggest to go further. Doctor: Dear Friend, greetings from HCM. The bilirubin and the liver enzymes will slowly fall as inflammation in the liver gradually subsides. Liv52 is good and continue taking that.You must avoid taking any drugs with out prescription even too much of paracetamol.Take good food with more of green leafy vegetable s, fruits and take rest It may take a week or two for the levels to come down. She will be completely alright Thank you"
},
{
"id": 213812,
"tgt": "My brother is having a psycological problem, please help him out",
"src": "Patient: Hi all, My brother is having a psycological problem. He was under medication for last 3 yrs. He was ok for some time. but few months before he was strange in his behaviour. Eiother he was too calm or too aggressive. Now he is in hospital. Doctor said he require 10 days. today is his 7th day but no improvement. He is not at all sleeping and talking useless things always. He is getting irritated to family members. I don't think he ll be alright there. So I am thinking to consult a good doctor. Can u help me out if there is any good doctors in Orissa/Bihar/Chhatisgarh area. Doctor: Hi , thanks for your query. He seems to be suffering from bipolar disorder. This will require prolonged treatment. I suggest you visit a good psychiatrist.You can go to any good medical college hospital and consult a senior psychiatrist. HAve a healthy life !"
},
{
"id": 78060,
"tgt": "Experiencing heavy chest pain in arm & back of shoulder",
"src": "Patient: Hi there. I'm 19 weeks along and experiencing heavy chest pains down my arm and back of shoulder blade. It's worse when I lay down. I have had a history of these pains for 5 or so years. Doctors at er say all is ok. But the I e this morning feels a lot heavier and painfull enough to make me sweat. Doctor: Thanks for your question on Health Care Magic. I can understand your concern. In my opinion, we should first rule out heart diseases in your case. Left sided heavy chest pain with arm heaviness and sweating are commonly seen in heart diseases. So get done ecg and 2d echo. If both of these are normal then no need to worry for heart diseases. Sometimes stress and anxiety can also cause similar symptoms. So avoid stress and tension, be relax and calm. Drink plenty of fluids orally and keep yourself hydrated. Don't worry, you will be alright. But better to first rule out heart diseases. Hope I have solved your query. I will be happy to help you further. Wish you good health. Thanks."
},
{
"id": 121053,
"tgt": "What causes sharp pain below my left shoulder blade?",
"src": "Patient: i am 35 years old 53kgs and 155cm tall.i am 34 weeks along with my first pregnancy. for the past four days i am having a sharp pain below my left shoulder blade. it becomes worse when i cough or take a deep breath or when i lie down. it started when i tried to sleep on my left side. Doctor: Hello,Your symptoms are suggestive of a musculo-skeletal pain (or local inflammation). For this reason, there is no reason to panic. I would just recommend using some local warm packs. I am sure that your situation is going to improve spontaneously.Hope I have answered your question. Let me know if I can assist you further. Regards, Dr. Ilir Sharka, Cardiologist"
},
{
"id": 37168,
"tgt": "What causes persistent fever when on typhoid treatment?",
"src": "Patient: My brother is diagonised with typhoid and he is given antibiotics but its not showing much improvement ,he is still getting frequent fever,and the fever is gone only for the period when medicine is in action after that again temparature is rising.I am very tensed.what should i do? Doctor: Hello,I understand your concern.I am Dr. Arun Tank, infectious diseases specialist, answering your query.In my opinion you should do blood and stool culture and sensitivity testing for Salmonella typhi.There is a chance that multi drug resistant salmonella has infected you that's why you are not responding to the treatment.Once you take the treatment as per the report, your fever will all go with treatment.I will be happy to answer your further concern, you can ask me on bit.ly/DrArun. Thank you.Dr Arun TankInfectious diseases specialist."
},
{
"id": 167179,
"tgt": "Suggest treatment for blackened front teeth",
"src": "Patient: I have a son, aged 1 yr and 3 months. He has got 6-8 teeth in total. The problem is his front 4 teeth have got blackened and gums bleed a bit while brushing. I brush his teeth with a soft brush - Colgate. Please suggest if there can be any cure and also if any prevention required for this decay to spread further. Doctor: Hi..Welcome to HEALTHCARE MAGIC..I have gone through your query and can understand your concerns..As per your complain it seems that your son is suffering from Nursing Bottle Caries and it generally spreads rapidly to the teeth leading to multiple decayed teeth..The damage that has already occurred cannot be reversed and you need to get the decay removed followed by filling of the teeth.. To prevent further spread of decay you should clean his teeth after every feeding and maintain a good oral hygiene practice by brushing his teeth twice daily..Hope this information helps..Thanks and regards..Dr.Honey Nandwani Arora.."
},
{
"id": 122423,
"tgt": "Suggest treatment for pain in ankle radiating to leg and foot",
"src": "Patient: My left ankle is the source of where the pain come from I believe. Sometimes, it feels like my ankle is in a ski boot and I cannot move my ankle at all. Sometimes when I move my ankle, the pain shoots up in my shin and it feels like someone has taken a pipe wrench and twist my shin. Sometimes the pain goes down my foot and sends out pulsating brief periods of pain. This has been going on for sometime. Sometimes, I admit that there is not pain, but lately, the pain is happening more and more Doctor: Hello, Your symptoms seem to be related to a pinched nerve. I suggest using anti-inflammatory medications such as Acetaminophen to relieve the symptoms. I also suggest using cold compresses for relieve. I recommend mild stretching exercises to relieve. Hope I have answered your query. Let me know if I can assist you further. Regards, Dr.Dorina Gurabardhi, General &Family Physician"
},
{
"id": 193507,
"tgt": "Suggest treatments for AR and PH",
"src": "Patient: Hi This is Jeeva from Bangalore. I am rhumatic heart patient since 1987 onwards. Recently I came to know I have sivere AR and PH. But day to day life and sexual life is normal. I am doing excersie and yoga. Shall I go for operation or medication is enough for this. Please advise me. Severe AR is there since 2007 onwards. due to my life sytle and yoga I am feeling good. Doctor: Hi, You need a surgical intervention when you have cardiac symptoms. tiredness, breathlessness, chest pain, difficulty doing routine work, frequent lung infection and there is increased pulmonary congestion. Also, check for Ejection Fraction in the echocardiogram report. less 45 % means you are going for cardiac failure - needs surgical attention. Hope I have answered your query. Let me know if I can assist you further. Take care Regards, Dr S.R.Raveendran, Sexologist"
},
{
"id": 35869,
"tgt": "What causes itchy rashes between buttocks?",
"src": "Patient: MYNAME IS NICOLE LESTER I HAVE A RASH OR SOMETHING INSIDE OF BUTTOCKS IT ITCH BUT WHEN SIT OR STAND ITS SORE INSIDE OF BUTTOCKS JUST BETWEEN THE COCCYX BONE SO COULD IT BE A PRESSURE SORE FOR SITTING OR WHAT AND WHAT CAN MY MEDICAL PHYSICIAN PRESCRIBE FOR ME. Doctor: Hi,It seems that due to lack of proper hygiene as this part is not easily reachable for cleaning and due to more perspiration and buttocks remaining togather may lead to good atmosphere to grow bacteria or fungus producing infection.Clean the part properly and apply triple acting cream for few days.Go for one antibiotic medicine like Amoxicilin for 3-5 days.Keep local hygiene clean, dry and airy.Ok and take care."
},
{
"id": 81378,
"tgt": "How to cure chest pain and breathing difficulty?",
"src": "Patient: My husband age 57, has been complaining that his left side of chest is tight and almost fights back when he inhales, his lower left rib hurts to the touch, he was in a car accident in March and a rib was out. He has been to a chiropractor. He is yawning a lot to get air, and very fatigued. Doctor: Thanks for your question on HCM .In my opinion your husband is having musculoskeletal cause mostly due to past accident.But it is better to rule out Cardiac cause first. So get done ECG to rule out cardiac cause.If ECG is normal, than no need to worry much for cardiac Cause.Chest trauma can cause1. Rib fracture2. Pneumothorax3. Pulmonary contusion4. Muscular injury.And all can cause chest pain, soreness on touch and breathlessness. So get done chest x ray to rule out all these.If chest x ray is normal, than it is musculoskeletal Pain mostly. Avoid strenuous exercise, heavyweight lifting and movements causing pain. Take good pain killer and muscle relaxant. Apply warm water pad on affected site."
},
{
"id": 139837,
"tgt": "What causes seizures?",
"src": "Patient: Our son who is now 57 years, started with seizures (pati Pal) when he was 10years old He is on Dilentin & Phem. he will almost always get seizures ,when before a B M or even after his BM s Sometimes, he will even fall, sometimes these nods will last up to 30 Minutes (on & off)He had a VNS implant done in April, but we are not seeing a change that we would like. What causes the seizsures when he need to do a BM??Dolores Hop YYYY@YYYY Doctor: Hello, That his symptoms could be related to reflex epilepsy related to BM. There is not too much to be done in this clinical situation, as he has tried VNS too. I would also recommend discussing with his doctor on the possibility of starting levetiracetam. Hope I have answered your query. Let me know if I can assist you further. Take care Regards, Dr Ilir Sharka, Cardiologist"
},
{
"id": 161599,
"tgt": "How to treat pus cells in stools and stomach pain?",
"src": "Patient: my three year old has pus cells in her stool. we live in kenya africa. she has already had worms, and other ameboas. was wondering what the pus cells in her stool may be indicating. we tested on tues. now on sat her stool is direaha. her tummy hurts before she goes. Doctor: Hi, Pus cells in stools are a normal occurrence. No need to worry since stools contain a lot of bacteria and pus cells. You need to give medications for treating diarrhea now. After the diarrhea is over we need to give drugs for worms and amoeba. You can repeat the antihelminthic drug every 6th month. Hope I have answered your query. Let me know if I can assist you further. Regards, Dr. Rajmohan, Pediatrician"
},
{
"id": 102756,
"tgt": "Had allergic reaction due to bee sting and took steroids, benadryl and now pepcid. Can any of this cause swollen testicles with mild pain?",
"src": "Patient: my 4 year old son had a severe allergic reaction to a bee sting and now he's home. he had to take steroids, Benadryl and is now on Pepcid. his testicles appear to be swollen or maybe swollen around his tesicles, they aren't black and blue but he says it hurts but he's not in tremendous pain. Is there a link between the bee sting and the swollen testicles? Doctor: HIThank for asking for asking to HCMIf the testicle it self being sting then the swelling is likely, but if it is intact then the reason of swelling could b something ales, because in any allergic condition the swelling of testis is not common, keep giving the steroid and I would advise you to start the long acting antihistamine \"Livocetrizine\" will be the best drug, have nice day."
},
{
"id": 156603,
"tgt": "What are the symptoms of prostrate cancer?",
"src": "Patient: If I am having the aggressive symtoms for prostate cancer. I have been dealing with real bad kick in the groin pain and Lower abdominal pain since I was 23 and I am 2 weeks from being 33. I have had blood in my urine and semen I am currently dealing testicular pain and bloody urine and semen the last 2 days. I went and got checked out and I have high PSA levels as the nurse told me they are normal if I was 50 but I m 32.Should I get to the doctor ASAP or not worry? Should I get as much quality time with my kids as possible. Give it to me straight doc?! Doctor: The symptoms of prostate cancer areA frequent need to urinate, especially at nightDifficulty starting or stopping a stream of urineA weak or interrupted urinary streamLeaking of urine when laughing or coughingInability to urinate standing upA painful or burning sensation during urination or ejaculationBlood in urine or semenThese are not symptoms of the cancer itself; instead, they are caused by the blockage from the cancer growth in the prostate. They can also be caused by an enlarged, noncancerous prostate or by a urinary tract infection.Symptoms of advanced prostate cancer include:Dull, deep pain or stiffness in the pelvis, lower back, ribs, or upper thighs; pain in the bones of those areasLoss of weight and appetite, fatigue, nausea, or vomitingSwelling of the lower extremitiesWeakness or paralysis in the lower limbs, often with constipationRegardsDR De"
},
{
"id": 32958,
"tgt": "For how long should a child with chicken pox be quarantined?",
"src": "Patient: My 5 year old has had chicken pox for 5/6 days now. He has had it pretty mild (he had it as a baby too); he has had a very mild fever and he has been his 'usual happy self' throughout. Most of the spots have now scabbed up, but he has 2 spots on his scalp with yellow heads (they do not look like they are about to burst; more like they are drying up). Is he okay to return to school yet? Thank you in advance for your time. Doctor: HiThanks for your query at HCM.I would like to tell you that the child of chicken pox remains infectious till there are scabbing present. Till scabbing is present do not send the child to school.T he AAP recommends excluding affected children from school until the sixth day of rash.Take careHope I answered your queryDr Sheetal Verma"
},
{
"id": 172328,
"tgt": "Suggest treatment for cysts in kidney of fetal baby",
"src": "Patient: Sir ,I am 8th month Preganant in Sonography we came to know that our baby is having amulticystic in left kidney and right is normal .the left kidney has 2 cyst 9mm & 4mm . Sir please advise as i am much worried ,is there any medicine to cure .My due date is on 5th july.please advise Doctor: Hi dear,I had gone through your question and understand your concern .In this situation we should try to find cause:1.Hereditary.2.Mutation factor in first three weeks of pregnancy (sickness, tablets, radiation, etc).3. Infection (perform blood for Ig M,G to Chlamidia, Mycoplasma trachomatis, Ureaplasma urealitycum, Ig G to Cytomegalovirus,Herpes).If something will positive you should treat that infection.4.When baby will born you should examine him for following up infection and observe him by ultrasound in O-3days, 1 month, 3 months it can normalize,because it may be immature kidney"
},
{
"id": 137650,
"tgt": "Does tendonitis cause pain when lifting leg?",
"src": "Patient: hello i have tedonitis and when i lift my leg up a step then the right side of my vagina start to hurt and contracts and the hospital told me to check orthepedic is this common please help i have no insurance and have been feeling this pain for six weeks my name is ve ve Doctor: Hi,Thanks for your query.Tendonitis can cause pain when lifting leg.I suggest you to get it checked out by your orthopaedician.Meanwhile give rest to your leg.Use heat and cold to manage pain. Both heat and cold can relieve pain in your joint. Heat also relieves stiffness and cold can relieve muscle spasms.Apply over-the-counter pain creams. Creams and gels available at the drugstore may provide temporary relief from pain.Take diet rich in calcium, vitamin d3 and minerals.I do hope that you have found something helpful and I will be glad to answer any further query.Take care"
},
{
"id": 33815,
"tgt": "Suggest remedy for bad cough",
"src": "Patient: I am a diabetic on insulin... I have a sluggish colon from nerve damage & I currently take: Metformin: Lorsatan Pottassium: xenical: I have a really bad cough & my G.P had a chest xray done on me which came back clear & told me that my cough is related to acid? Can u please advise me Doctor: Hello,Welcome to Health Care Magic.Thanks for writing.I am Dr. Saddiq ul Abidin. I have read you question completely, i understand your concern and will try to help you in best way possible.The two most important things about diabetes is always, whether its is controlled or uncontrolled? and complicated or uncomplicated?In your case presence of gastroperesis, indicate Autonomic Neuropathies, as you mention in your words as having a bad colon. Another common problem with diabetes is increased tendency to catch infections. Catching a chest infection can be one of those. Here, it would have been more appreciable if we had known any smoking history, current or past, or any history of shortness of breath, presence or absence of sputum, because each of these would decide further line of management.Currently, if your GP suggest it to be because of acid regurge, you can try taking stomach medicines including antacids and motility agents, like Omeprazole and domperidone after discussion with your GP, and if the symptoms persist, then you can proceed with further evaluation regarding a chest pathology.Along with chest X-ray, you may need to have complete blood count, ESR, CRP, as well as sputum or blood cultures. A thorough, throat and chest examination, to rule out any signs of respiratory tract infection would let your doctor decide for the use of antibiotics along with antihistamines (n anti allergics) as well need for a good anti-tussive (cough syrups). Meanwhile avoid oily, bitter or spicy foods and try taking steam inhalations to avoid frequent bouts of coughing.I hope this answered your question.If you have more queries I am happy to answer. Otherwise rate and close the discussionRegards,Dr.Saddiq ul AbidinM.B.B.S(Licensed Family Physician)Resident Medicine."
},
{
"id": 204835,
"tgt": "How can severe depression and memory loss be treated?",
"src": "Patient: I feel so unhappy, moody, very short tempered, tired and i am experiencing memory loss(this seems to be getting worse) I ve had my b12,thyroid, FBC, this is all ok, my iron is showing borderline, doctor doesn t really give any answers as to what this is or why I m feeling like this. Can you please offer me some advise. BD Doctor: in my opinion depression would be the most likely cause is drugs like escitalopram would be beneficialcognitive behavioral therapy with trained therapist is beneficial moderate exercise, good diet adequate water intake is a must"
},
{
"id": 96055,
"tgt": "What are the side effects of masturbation ?",
"src": "Patient: What are the effects of releasing sperm daily ? Doctor: Dear thanks for query.Can you please what are the effects of eating daily ? effects of walking daily ? all the answers will be none,so it this.No effects."
},
{
"id": 205882,
"tgt": "How to deal with mental illness of teenager?",
"src": "Patient: Hi My daughter is 19yo she was dx vasal vagal syncope, 2 yrs ago after a slew of tx meds...all was discontinue she also has a eating disorder getting a good control of it.she just spent a week in hospital with pneumonia she had a thoracentesis where they tap 1500cc. they are not sure what and where it came from maybe viral/bacteria with mono.......my daughter had much improved prior to this from the syncope passing less and milder episodes. Since the past illness the syncope is back in full force,last about 2 min pulse weakwakes up startled lights bother her and she at times stairs for a couple seconds before coming around.We lower her head elevate legs and arouse her with cold water.Please help where do I go from here.We have had tilt test 2 wk cardio monitor 3 day eeg beta blockers antidepressant and salt tabs Doctor: DearWe understand your concernsI went through your details. I suggest you not to worry much. You should know that these problems are not mental diseases, but are mental disorders. Many researches and researchers confirm that medicines alone cannot cure mental disorders. Exercise, Life style changes, change in thinking pattern, relaxation etc are as essential as medicines. Psychotherapy can help you changing your lifestyle and thinking patterns. Yoga and meditation help you to streamline your metabolism and neurological balance. I can help you in this aspect. Psychotherapy techniques should suit your requirement. If you require more of my help in this aspect, Please post a direct question to me in this URL. http://goo.gl/aYW2pR. Make sure that you include every minute details possible. I shall prescribe the needed psychotherapy techniques.Hope this answers your query. Available for further clarifications.Good luck."
},
{
"id": 160381,
"tgt": "Suggest treatment for loose motions",
"src": "Patient: My baby is 1yr. old (12months). For last 2 days she is having loose motions. I don t know whether to call it as loose motion but she is having toilet 3-4 times a day. Toilet smells badly. Also colour of toilet is Blackish Green, Yellow and also it is sticky. Please let me know what kind of food shall i give to her. I normally used to feed her milk, Dal/Rice, Fruit(banana/Apple), Khir etc. Doctor: Hi,Does she have fever or vomiting? Does the stool contain blood or mucus? Is there any abdominal pain or discomfort while pooping?If no above said symptoms, this need not be taken as significant. Some change in stool frequency,colour or consistency is common once in a while, and need not be worried about. You can continue the same feeds as before. If stool is too loose, reducing amount or withholding fruits for few days might help.Take care. Hope I have answered your question. Let me know if I can assist you further. Regards, Dr. Muhammed Aslam T. K., Pediatrician"
},
{
"id": 20399,
"tgt": "Suggest treatment for infected orb",
"src": "Patient: dad had a stroke six months ago, 100 percent blocked cartid artery, he is havin what seems to be mini strokes, there is no surgery options, and now they are saying that he has a orb that is either infected or bleeding, what can be done, what are we facing? Doctor: therapy with anti coagulant and statins should be given to him.please attach copy of MRI for further comments sir"
},
{
"id": 85957,
"tgt": "What does pain in the lower abdomen along with perineal pain indicate?",
"src": "Patient: I have lower belly pain and very tender to touch pain is between rectum and vagina bladder doesnt feel bad.when i pass gas its very sore as it passes by the painful area.its in the area of the douglas pouch i think (i have no uterus)I had a catscan with contrast a week or so ago and then my kidneys were very flared for about 5 days (i have lupus sle) is this cause for concern? What would treatment be.Ive never had such discomfort and tender to touch like this.this is the 3rd day of this and only worsening Doctor: maybe per anal abscess fissure and fistula. sis-bath is to be done.apply appointment's per rectum.get locally examined by surgeon maintain hygiene and sanitation. per rectal examination is to be done if required cryosurgery can be recomanded ultrasound kub can be done.nephrologist can be consulted for examination and advise on treatment. may be related to lupus. second opinion can be taken for treatment."
},
{
"id": 97355,
"tgt": "Suggest alternative treatment for stage 3 cervical cancer",
"src": "Patient: HI ,, I was diagnosed with stage 3b carcinoma of cervical cancer. I have completed all treatments of radio( 28) & internal radio (5)also I could only do2 treatments of chemo due to kidney failure , My tumor size at the time of diagnoses was 8cm , my last PET scan has the tumor at 4cm, 2 surrounding limp nodes are gone or back to normal size. There has been no talk of surgery to remove the existing tumor , I am waiting for my Pap smear to come back to see if there is new growth of cells ,,I guess my question is: will the tumor keep shrinking or stay the same or start to grow again and is there any alternative treatment that I can do is surgery is not an option at this point , after reading info on my cancer the prognoises is I only have about 30% survival rate with stage 3b cervical cancer. thanks Doctor: thank you for sharing your problemas you described you have cervical cancer and radiotherapy treatment given to you also. and you can not go for surgery.at this time i suggest you for homeopathic line of treatment. homeopathy may be much helpful to you. so consult a homeopathic doctor nearer to you. homeopathic medicines are given after proper casing that is case will be evaluated 'person as a whole', means many questions will be asked about life.i hope this answer will be much benificial to you.for more queries plz don't hassitate to ask.get well soon dear friend...dr.manu khimani"
},
{
"id": 70543,
"tgt": "Should I be concerned for baby suffering from recurring/swollen/itchy lump?",
"src": "Patient: Hi my baby has been having reoccuring lumps like she's been bitten by an insect. It starts small and becomes swollen and hard and itches her with no fever. She's been on augmentin and orelox but they still keep appearing. I'm scared and don't no wat to do please help Doctor: Hi.Such lumps need an anti-allergic medicines. The real treatment is to find where is the baby getting these insect bites . Bed/ ground ; Change the linen and go for vacuum cleaning of the sofas , bed etc wherever the baby sleeps or plays.Try to find the reason as it should be taken care of."
},
{
"id": 80321,
"tgt": "What causes blood in mucus discharge?",
"src": "Patient: My husband has a loose congested cough and has had it for a while it seems to worsen after taking his meds. And now has a min amunt of blood in mucus. Wondering if xarelto 20mg ,Crestor or any other meds for heart patients could be causing the congestion Doctor: Thanks for your question on HCM. I can understand your husband's situation and problem. Blood in mucus after coughing is known as hemoptysis. And yes, xeralto can cause this,because it is anti coagulant in nature. So you need to stop xeralto till blood stops. And we need to treat the cause simultaneously. Following are the possible causes for hemoptysis in his case. 1. Lung infection like tuberculosis or pneumonia. 2. Bronchitis. 3. Congestive heart failure because he is on cardiac drugs, so possibility of this is high. So get done 1. Chest x ray to rule out lung infection. 2. PFT (Pulmonary Function Test) to rule out bronchitis. 3. Ecg and 2d echo to rule out cardiac failure. So, first diagnose the cause of hemoptysis and then start appropriate treatment. Till that time stop xeralto."
},
{
"id": 130353,
"tgt": "Suggest remedy for dislocated collar bone causing sore, swollen neck muscle with pain in arm",
"src": "Patient: I fell a week ago Chiropractor say left shoulder lower then right did a corkscrew when I fell.Neck muscle sore swollen spot on collar bone by neck,hurts yet to lift arm up,in upper part of arm.was told that i just about dislocated collar bone.How long before stop hurting. Doctor: Hi i am Dr Ahmed Aly thanks for using healthcaremagic site ,I had gone through your question and understand your concerns .. In my opinion it varies according to the degree or injury and the severity of the swelling and your body built whether muscular or fit . it usually takes not more than 3 weeks with proper management and treatment . For my patients i may recommend NSAIDS like advil tab once daily , hot massaging with topical gels , avoid heavy lifting and vigorous movements , using an arm sling or a shoulder splint to decrease the swelling will be effective .Please click THANK YOU and consider a 5 star rating with some positive feedback if the information was helpful. Hope the above information helps you,Any further clarifications feel free to ask."
},
{
"id": 56565,
"tgt": "What causes elevated SGPT/SGOT/bilirubin while on tuberculosis treatment?",
"src": "Patient: Hi, I am on undergoing tb treatment for past 2 weeks. My doctor advised sgpt,sgot and billirubin test after 15 days. The results of the tests are sgpt-58,sgot-70 and billirubin-0.9. Is it normal to have high levels of these while on tb treatment? What is the meaning of this please help? Doctor: Hi there,Thanks for using HCM.During the initial phase of treatment for tuberculosis medications like INH, Rifampicin, Pyrazinamide and ethionamide are used. Of these four drugs, the first three can affect the liver and cause the liver enzymes to raise.When it raises more than 2-3 times the upper limit of normal it is significant. When it is raised multiple folds, the medications are usually stopped.In your case it is not significantly raised.Moreover you have not mentioned about the baseline levels which is what the liver enzyme levels were before starting the treatment.I would advise you to get a ultrasound scan of the abdomen done.If you have the habit of drinking alcohol, you need to stop it.Is this answer helpful?"
},
{
"id": 64142,
"tgt": "What causes lumps under skin over the jaw line?",
"src": "Patient: I've got a lumps under my jaw line under the skin that aren't visible but can be felt with the hands. They do not hurt although they can catch (it feels as though the lumps have been caught by the jaw bone). I don't feel I'll but they can be uncomfortable when they catch. What are these lumps? Doctor: Hi Dear,Welcome to HCM.Understanding your concern.Thanks for your concern . The symptoms you mention in query are can occur due to two things . Firstly there are many lymph nodes in that area so the lumps can be lymph node swelling , which usually occur due to infection. Secondly the lump can be due to blockage of submandibular gland duct , which occur due to stone or any foreign body such lump is called mucocele.I would suggest you to consult ENT specialist (ear,nose,throat) for proper examination . Doctor may order X-ray or CT scan of that region to confirn the diagnosis .In case of lymph node swelling doctor will prescribe antibiotics at least for week along with anti inflammatory drugs .If it is mucocele then doctor may remove it surgically or perfoem marsupilization.Sometimes the lump that is immoveable ,hard and slow growing could be a cancer so it is important to visit doctor and get it checked.For now give warm compresses and take advil (if not allergic to it ) or tylenol for uncomfortableness and swelling. Hope your concern has been resolved.Best Wishes,Dr. Harry Maheshwari"
},
{
"id": 87217,
"tgt": "What causes stomach pain even after the operation?",
"src": "Patient: hello i have a sister who lives in africa she had her son 3 years back byc-section and was put on alot of drips without havin a catheter 2 drain herurine out, she slipped into a coma and ws later drained alot of urine but nowshe complains of severe stomach ache. she has done 2 operations done to find outwhats causing the stomach ache but they cannot deal with this, please help me Doctor: Hi! Good evening. I am Dr Shareef, a general surgeon answering your query. This is a situation where one might not be able to opine on the problem unless all the papers having information on the intraoperative findings during each surgery done are available. Therefore, I would suggest you to send all the findings with reports to arrive at a tentative cause of your sister's abdominal pain, although a personal physical examination would also be needed to arrive at a conclusion.Thanks for choosing health care magic to clear doubts on your health problems. I wish your sister an early recovery. Dr Shareef."
},
{
"id": 84135,
"tgt": "Could tiredness,sore throat and swollen glands be side effects of venlafaxine?",
"src": "Patient: I have been taking 150mg venlafaxine for about 10 years. Over the last two years I have had symptoms of tiredness, sore throat, swallon glands,nausea and headaches which have been getting progressively worse. Blood tests come back normal. Could these symptoms be side effects? Doctor: Hello, It can be a pharyngitis or sore throat and may not be related to vanlafaxine. As a first line management you can consult a local practitioner and start on antibiotics preferably amoxicillin. Do warm saline gargles for symptomatic relief. If symptoms persists better to consult an ENT specialist and plan for a throat swab. Hope I have answered your query. Let me know if I can assist you further. Take care Regards, Dr. Shinas Hussain"
},
{
"id": 3472,
"tgt": "Should i take 'ovumax' injection if follicular size is >0.67 cms?",
"src": "Patient: Hi, My doctor have asked me to get done the follicular study. I have started that and the endometrium size is 0.67 cms on 26.12.2011. Now the doctor has asked me to get the next scanning done on 29.12.2011 and said that if the follicular size is >= 0.67 cms then take Injection Ovumax 5000 units SC.Please advise what does this indicate. Doctor: Hallow Dear,The egg is inside the follicle. Immediately after menstruation, the follicle and the egg inside are very small in size. Day by day, they go on increasing in size. About 14 days prior to the next menses the egg is released from the follicle. Just before the egg release (ovulation), the follicle attains the size between 18 mm and 21 mm. Endometrium is lining of the uterus. During menstruation its outer layers get shed off when it becomes very thin. Day by day, it also starts becoming thicker. A healthy endometrium is between 9 mm and 14 mm thickness at the time of ovulation.You have mentioned the size of follicle and thickness of endometrium, which are too low; however it is not clear these sizes are on which day of the cycle. If these measurements are at or around the ovulation, they are too low. You will need some medicine to increase the size of the follicle. As the follicle grows in size, endometrium also will become thicker. However, Ovumax is an Injection which contains Human Chorionic Gonadotropin (hCG) hormone. This injection is given to induce ovulation from a well grown follicle. It would not help the growth of the follicle. For growth of the follicle, Inj. HMG has to be given. Clomiphene citrate also may be of use; however, it may hider the growth of the endometrium.I would not advise you to go for these or such injections before you get some of your investigations done which will try to point out the deficiency. Get your blood examined for FSH, LH and AMH. Based on there levels, further management should be planned. Please report to an Infertility Consultant for expert and scientific management.I hope you got the required guidance. Dr. Nishikant Shrotri"
},
{
"id": 146456,
"tgt": "How safe is prolonged use of Acitrom for brain venous thrombosis?",
"src": "Patient: Hi Doctor.Is intake of Acitrom 2mg for brain Venous Thrombosis for more than a year dangerous? My dad had cerebral venous thrombosis twice in 2005 and 2013. He had a second time on last year march 2013 due to the stoppage of medicine(previously he was having Epsolin) . From that time he has been using Acitrom 3mg and now its dosage been reduced to 2mg. Last month the neurosurgeon(one who stopped the medicines previously) has asked to reduce the intake to twice a week, monday and thursday. And this month he told that this medicine has to be stopped. But in the same time, since this is his second time, the neurologist has strictly told that this medicine should be eaten throughout his lifetime. Neurosurgeon is telling this medicine might become poisonous to him if eaten more than a year. Am between the neurosurgeon and neurologist. Please help! Doctor: There are many people who take that drug for many years if not for their whole life. So unless the neurosurgeon has seen a side effect which you don't mention I don't agree with the statement that \"it becomes poisonous\". If on the other hand your father is experiencing a side effect or has other medical issue which render him prone to bleeding (cause main side effect is bleeding) then of course it's another matter and the risks should be weighed. It definitely doesn't make any sense to take the drug only twice a week.I think it was correct to have interrupted the drug the first time, usually treatment with blood thinner after cerebral venous thrombosis is interrupted after 6 months. However since the venous thrombosis reoccurred that means your father has a prothrombotic condition so continuing it now makes more sense to me.I see you mention Epsolin but that is a drug of a different class, it is used to prevent seizures while Acitrom is a blood thinner so I don't understand that part of your story, hope you gave us all the facts right. However if the rest is correct my opinion remains the one above."
},
{
"id": 37348,
"tgt": "Suggest treatment for herpes",
"src": "Patient: Herpes or not? YIKES! This has happened twice to me while wearing a pad-my skin breaks open.The first time-My partner was being rough on the outside of my clothes trying to turn me on by rubbing the outside of my privates over my clothes. The next day it hurt and I had a tear in my skin.Recently when wearing a pad something hurt in the same area. When I examined the area I had 3 tiny clear bubbles and one cut looking spot. The bubbles popped clear fluid and now the spot has turned into an open wound with a red border. It never scabs or crusts, it heals and goes away..when I try to take a picture to examine its too shiny. Doctor: HelloAfter reading your history in this query , these bubbles filled with clear fluid and cuts may be due to these reasons , these include>1 Abrasion or bruise , as you mentioned that your partner was rough on your private part , so this might have develop abrasion or bruise .You mentioned that there is an open wound , so this may be the result of bruise .2 Herpes zoster , as there is small multiple bubbles with with clear fluid and shining appearance indicate Herpes , so physical examination is important .As you mentioned that open wound healed and goes away so in my opinion this is a case of mild abrasion due to rubbing .If it don't spread then no need of any consultation.If spread then consult a dermatologist and get his opinion.Good luck."
},
{
"id": 51849,
"tgt": "66 years old female suffering from 9mm kidney stone",
"src": "Patient: my mother 66 years has 9mm size kidney stone will lothotripsy help in removing the stone fully! Doctor: hi,thanks for query.It the stone is in kidney the size is big.You need to get in IVP done,that is after injection dye x rays of urinary system needs to be done,to see presence of any other stone.Otherwise lithotripsy can definitely help.Please get in touch with a urologist. wishing her good health."
},
{
"id": 221032,
"tgt": "Are vaginal discharge and drop in blood sugar level due to pregnancy?",
"src": "Patient: I have had sex several times this month. We used a condum everytime and I am also on birth control. I try to take my birth control on time but I dont always take it at the exsact same time. I usually take it in atleast a half an hour difference than the usual time tho. I am supposed to be on my period currently it was supposed to come yesterday. But today all I have gotten was clear/white spit like consistancy fluid. It comes out quite often and I feel like I have my period because of cramps and achie pain. I have also noticed my blood sugar dropping very low through out the day. I have never noticed problems with my blood sugar before. Could I be pregnant? Doctor: HI, I have read your query & understand your concern. Blood sugar level depends on the food intake, Pregnancy does not have any effect on blood sugar.. except in case of diabetes/ gestational diabetes when blood sugar is raised in pregnancy.So low blood sugar can not be a sign of conception. Your blood sugar might have lowered due to psychological stress leading to less eating. Yes, in pregnancy vaginal discharge is increased than normal, but it also increases before & after menses. So just rise in vaginal discharge can not be a sign of pregnancy. half hour difference in consumption of pill.. does not matter much.. if you have not totally skipped a pill. In addition you were using condom too.. so there is rare chance of conception. Any doubt can be ruled out by HPT after 8 days delay in period. thanks."
},
{
"id": 118842,
"tgt": "Protruding bone on left sternum, lowering platelet count. Is it Xyphoid or a tumour?",
"src": "Patient: I have had loewring platelets since 2006 and a protruding bone/ tumor a little to the left of my sternum. The doctor thinks it is my xyphoid, but I will be honest, I think it is much more. It has become bigger and I do not have to search for it anymor eit is right there. The insurance company denied a CT scan and having 3 little ones I am starting to get very scared about my healt!h! Any suggestions on who to go to or what you might think it would be? I can t even lay on my back or side because it bother me so much!! Doctor: Hi,Thank you for posting your question here, I will try to answer it to the best of my abilities.It could very well be your xyphoid process, it may have gotten calcified and that is why you feel the lump. But that is only one explanation, it could very well be a hernia or a tumor as well.Since your insurance company isnt letting you take a CT scan, at least get an x ray done, it may show us what we need to see, it's a bit of a long shot, CT would have been ideal, but since it cant be used ask for an xray at least. It is way cheaper, they shouldnt mind.I hope this answered your question."
},
{
"id": 196771,
"tgt": "Suggest homeopathic treatment for gynecomastia",
"src": "Patient: Hello sir,I am rahul & i am 18 years old I have selected in Indian Navy,but in Medical test i am temporary.Because i am Affected by gynecomastia since 1 and half year.But i don't want to do any surgery or Plastic surgery ?I want to cure my disease by Homeopathy.Kindly giving your suggestion.I am waiting for your answer. Doctor: Hi and welcome to Healthcaremagic. Thank you for your query. I am Dr. Rommstein, I understand your concerns and I will try to help you as much as I can.There is no homeopathic remedy for. Surgery is the only possible solution and all other methods are fake. This is caused by accumulation of fats and glands and it can be treated with medications. So you should consult plastic surgeon about further treatment.I hope I have answered you query. If you have any further questions you can contact us in every time.Kindly regards. Wish you a good health."
},
{
"id": 35046,
"tgt": "How long should be waited after a dog's bite for rabies vaccination?",
"src": "Patient: yesterday night i take 1 beer and finished it at 9:30 PM. Then around 11 PM one street dog bites me in my leg (near around foot).At what time should i take rabies injection. Can i take around 1 PM today. I had listen that after drinking alcohol we can not take injection.kindly clear my doubts. Doctor: Hello,Welcome to HCM,Rabies is a disease transmitted by dog bite which is 100% fatal but it is 100% preventable by proper and adequate treatment.As you were bit by street a dog on your leg which has led to bleeding from that site, according to WHO it is categorized into Cat III which requires following treatment.1. Wound should be properly washed with water 2. Inj TT3.Active immunization with antirabies vaccine on days 0,3,7,14 and 28. You can take the anti rabies vaccine as early as possible and you need to avoid consumption of alcohol during the course of treatment as it may interfere with the antibody production.2.Passive immunization with RIGs around the wound.3.Oral antibiotic like Tab Augmentin, twice daily.4.Topical T-Bact ointment.Thank you."
},
{
"id": 89758,
"tgt": "What causes a feeling of fetal movement in my stomach?",
"src": "Patient: I have movement in my stomach, feels like a baby kicking even from the outside I am a 25 year old female, I have a rolling sensation in my lower abdomin. I do not have any pain. I ve had 2 kids and it resembles a baby moving. I took 3 pregnancy test, all negative. There is no pain, no extra gas. No medications. Had Tubal What could it be? Doctor: HI.This is a classical finding of Pseudocyesis meaning feeling of pregnancy. You may please go for ultrasonography to confirm. This happens due to the too much wish to have a mother hood again."
},
{
"id": 20296,
"tgt": "What causes fluttery feeling in the heart after a surgery for gall bladder?",
"src": "Patient: hii im 9 and have just recently had a baby 2 months prem as i had eclampsia , i then found out i had gall stones and had to have an emergancy operation to get my gall bladder out as i had pancretitis , im now getting fluttery feelings in my heart and am getting scared as to what it could be Doctor: Your question cannot be answered easily in this forum. After ecclampsia and an operation, you've had quite a brush with medical care. Your recovery may be punctuated by periods of palpitation that offer no great concern but only someone with more detailed knowledge can answer. You let your doctor know of concern and follow his or her suggestion."
},
{
"id": 52459,
"tgt": "Is chronic calculus cholecystitis under a lover section a cause to worry?",
"src": "Patient: My Father s ultrasound report when we suspected stone... Liver - is notmal in size measuring 10.3 and uniform echotexutre.Intrahepatic biliary radicles and CBD appear normal. Portal and hepatic veins appear normal. A CBD measures~7.5 mm is mildly prominent. Impression - Chronic calculus cholecystitis I just wanted to know if he need to go under surgery or can be cured by medicines. In the Ultrasound report the impression is A CBD measures~7.5 mm is mildly prominent under Liver section. Doctor: Hi. I have gone through are your query.There ar no medicines to expel stones from the gall bladder.Your father has to undergo surgery to remove the gall bladder."
},
{
"id": 184645,
"tgt": "What causes toothache along with dizziness?",
"src": "Patient: hi doctor, i am f21 years old h155cm w40kg. I am feeling pain from my teeth and it makes my head dizzy. it has happened more than 3 months ago till now. i had used analgetic to reduce it pain, but it still happen, at least 3times a week, especially when i'm tired. how can i cure it doctor. pleased help me.. Doctor: Hi,Thanks for posting the query, First off all i would like to tell you to undergo the treatment of the tooth, toothache occurs due to infection, such type of infected tooth needs to be treated with root canal treatment, analgesics will give a temporary relief. I would suggest you to consult a Dentist get a checkup done take an x-ray of the tooth and plan for the treatment.Take care!"
},
{
"id": 128451,
"tgt": "Can Bell's palsey or a concussion cause tongue numbness and difficulty blinking?",
"src": "Patient: I fell on Sunday and hit my face kind of hard and of course tensed up my neck - this morning woke up and have a hard time blinking my right eye one side of my tongue feels like novacane and some strange feeling in right side of face- my question is it from a concussion or bells palsey and can I fly with these conditions? Doctor: Dear patient difficulty in blinking one eye and one sided face decreased sensations are signs of Bell's palsy and since this has happened after trauma should be taken seriously. Please consult neurophysician nearby you for detailed evaluation and further treatment. Please avoid flying till you are examined by neurophyysician."
},
{
"id": 196462,
"tgt": "What is the sticky substance on tip of my penis after i urinate?",
"src": "Patient: after i urinate the tip of my penis always seems to be wet. Then I started noticing that there was a clear sticky substance coming out from my urine which caused the urine to be stuck in my penis. It's hard to get the clear sticky substance out, but when i masterbate it usually does come out. Also, my digestive system has become weird and I have stomache cramps. Please help me relieve my stress, and tell me what's wrong. Doctor: HiGREETINGS From the history it looks like the liquid is precum which is causing problems. You can always wipe it out.It is absolutely normal (you have not mentioned your age).If the skin can be retracted backwards,keep the area clean.Regarding gastrointestinal symptoms ,get a physical examination with the specialist. You have not given the details.Hope you are convinced. Regards"
},
{
"id": 145315,
"tgt": "Suggest treatment for numb sensation after a head injury",
"src": "Patient: I fell two days ago and hit my head on the rear left side. I have never hit my head so hard. The brunt of the impact was about an inch behind my left ear. When I touch the bump or any area around the bump (incl above the injury up to the top of my scalp and around the back of my head) it feels as though I have had a novacaine shot and it is a very odd numb-like sensation. Should I see at doctor? Doctor: Hello. I have been through your question and understand your concern.The numbness sensation derives from the compression of the occipital nerve which passes nearby. You should not be worried because it passes in a few days. But you can some NSAIDs meanwhile with ibuprofen 600 mg daily being the best option.Wish you the best health."
},
{
"id": 153222,
"tgt": "Suggest treatment for cancer",
"src": "Patient: My mother49-50 years old. She had cancer a year ago when she did mastectomy (one). she took the chemical cure. she and all my family suffered alot. Doctors asked she to do another surgery (hysterectomy), she and I refused because the suffering is too much. I want to know if only the medicine can help here without surgry. Doctor: Hysterectomy and bipolar salphingo oophorectomy is advised in some breast cancer management as few varieties are oestrogen dependent. It is very advisable to get the procedure done."
},
{
"id": 99136,
"tgt": "What causes intermittent red rash around eyes?",
"src": "Patient: I have a red pimple rash around my eyes that seems to go away but then seems to return again. It is itchy. This has been going on for some and I have seen my dermatologist who seemed to think it was allergy related but I can't seem to figure out what I am allergic to. Doctor: A red pimple around eyes is rather vague to know if its around eye lashes or not, or probably stye or like that because location is not clear. Though stye may not recur again n again.You may simply seek to take Haridra Khand - a pure herbal powder - though not palatable much - thrice day. It consists of turmeric, sugarcandy etc - no side effects. 4 weeks will be enough. No need to consult physically for this powder.Any kind of allergy or even auto immune allergy can be cured by it. Then what is use of taking toxic anti allergic when your treating doctor has not arrived any solid reason. Still if you do not like the response, you have option to stop anytime.Hope it helps you."
},
{
"id": 72337,
"tgt": "Suggest treatment for radiating chest pain along with recurrent fever",
"src": "Patient: Hi, i was sick for a week and i get recurring fevers and it seems normal but today when i cough i feel a sharp pain in my chest, and then i stinging pain that radiates in my whole body. if i remember correctly from my meagre knowledge of health, the stinging is caused by a bacterial poison. Should i be concerned? Doctor: Hello dearWarm welcome to Healthcaremagic.comI have evaluated your query in details .* This seems with possible lung infection ; needs thorough clinical examination and x-ray chest for further management .Wishing you fine recovery .Feel free to ask any further doubts .Regards ."
},
{
"id": 167156,
"tgt": "What causes a rash near anus in a child after MMR vaccination?",
"src": "Patient: Hi, My baby is 16 months old and recently rec d the MMR vaccine...I initially noticed a rash around the rectom and the vaginal cavity areas and then she was fine...a few days later the rash seems to have developed once again and its like small pimples in shape...what could this be? She is about 21 lbs, and has had an average medical history thus far. Doctor: some children develop a very mild form of measles following MMr vaccine by one week to 11 days.. this includes rash, fever, loss of apetite and general feeling of being unwel.. this continues only for 2-3 days"
},
{
"id": 226347,
"tgt": "Missed a pill Tri sprintec birth control, had sex the same day, following day had nausea and strange stomach cramps. Cause?",
"src": "Patient: Hello, I have been on Tri Sprintec birth control for about 4 months now. I dropped the Friday pill of the first week and wasn t able to take it. So I picked up a new pack the following day and took the missed friday pill I dropped as soon as I received it and took the Saturday pill at the correct time that day. My husband and I had sex on Saturday night. Sunday morning I woke up a tad nautious and had strange cramps in my lower stomach . Should I be worried? Or does it have something to do with missing the pill? Doctor: Hello I appreciate your query. Ideally speaking, when you miss a pill, you should take it the next day, alongwith the pill due on the next day as well ie. two pills the next day, plus the remainder of the pack as prescribed. This you have already done, so no issues. However, the important thing is, to use a backup method, such as condoms, for the rest of the month, before beginning a new pack the following month. You mentioned that this was not done. However, the mild symptoms you mention, are mostly non specific, and have nothing to do with missing a pill. Maybe due to taking a double dose of pills the day before. Please use barrier method for the rest of the cycle to ensure complete protection. Also, just in case you miss your period, perform an early HCG urine pregnancy test. Chances of conception are more if the unprotected intercourse occurred around the time of your ovulation. All the best."
},
{
"id": 9961,
"tgt": "Is Krimson 35 the correct medication for treating hairloss and dandruff in a person with high testosterone levels and PCOD?",
"src": "Patient: Hi. I have pcod for . more than 10 years. I recently have hairfall and the scalp is becoming visible. I have hairloss only in the front. I checked my hormone levels and testosterone is high. Also i checked my thyroid levels and they are normal. My doctor prescribed me krimson 35. Is it the correct medication for hair fall and balding at front? Also my hair is prone to dandruff. Please help Doctor: Hello, I have gone through your query and the medicine prescribed to you is fine. This will help in treating PCOD as well as hair loss. Along with this, I would recommend you to apply minoxidil 2 per cent lotion on the affected areas of the scalp twice daily and take hair supplements like tablet Keraglo Eva once daily for 4-5 months. Hope I have answered your query. Let me know if I can assist you further. Take care Regards, Dr Asmeet Kaur Sawhney, Dermatologist"
},
{
"id": 181293,
"tgt": "Is root canal the method of treatment for tooth infection?",
"src": "Patient: I had a boil, non painful on my upper gum at the upper right incisor for 4 days. When at the dentist yesterday, it opened and has yellow pus, he started me on antibiotics and suggested a root canal for that #8 tooth. The XRay of the tooth didn't show anything. He says the drainage & swelling surrounding the tooth are indicative of infection. Is a root canal my only option? Won't I also loose the tooth? I have just completed invisalign treatment for that tooth and don't want to loose it. Please advise and thanks. Doctor: Hi..Welcome to HEALTHCARE MAGIC..I have gone through your query and can understand your concerns..As per your complain in case if you have a non painful boil over the gum that drained pus seems to be probably due to gum boil or gum abscess and as the x ray did not reveal any abnormality as well as there is n pain in tooth, then the chances of tooth related infection causing dental abscess are rare..Root canal treatment is advised if there is tooth related infection and not fr gum infection..I would therefore suggest you to consult an Endodontist and get evaluated and a thorough clinical evaluation and x rays from different angulations has to be done to rule out the exact cause and treated accordingly..In case if it is not due to tooth infection and is due to gum infection then Scaling and curettage will be advised..Hope this information helps..Thanks and regards.Dr.Honey Nandwani Arora."
},
{
"id": 138453,
"tgt": "Suggest treatment for arm pain after falling down",
"src": "Patient: I had a fall a week ago resulting in a numerous fracture (spiral) as I was falling I reached up a grabbed the top of the refrigerator as my feet slipped out from under me..I feel like I may have also some tendon and muscle damage..I cannot lift my arm from straight down position from the side. Do you think this may be a sign of damage other than just a numerous f2f? Doctor: Hello, I have studied your case.Inability to lift arm can be due to rotator cuff injury which may need MRI for confirmation.Medication like methylcobalamine with muscle relaxant and analgesic will reduce pain, you can take them consulting your treating doctor.You may consult physiotherapist for further guidance. He may start TENS ,or ultrasound which is helpful in your case.You may need to take rest for early healing.Also check your vit B12 and vit D3 in your blood.Hope this answers your query. If you have additional questions or follow up queries then please do not hesitate in writing to us. I will be happy to answer your queries. Wishing you good health.Take careRegards Dr Vaibhav"
},
{
"id": 108594,
"tgt": "What causes back pain when suffering from kidney stone?",
"src": "Patient: i was just diagnosed on Friday with a kidney stone.i had excruciating pain in my back. I have no more pain and did not pass the stone but I have a constant urge to run to the bathroom to pee every five minutes.my lower stomach has pressure . I cant sleep because im running to the bathroom. I feel like I cant empty completely. Is this normal or maybe some other problem? Doctor: due to kidney stones pain sometimes bother around back area may be sometimes stones stuck in tube called ureter sincethere is no proper urination this lead to pain check with your doctor and get the medication for the same In addition to feeling severe pain in your back and/or side below your ribs, a number of other signs and symptoms may also occur with kidney stones. One of the most common is problems with urination. This includes pain while urinating, urine that appears cloudy or smells differently than it normally does, or feeling an urge to urinate more often than usual. Blood present in the urine due to kidney stones can also cause urine to appear brown, pink, or red. may cause back pain"
},
{
"id": 205031,
"tgt": "Does Lexapro or Risperdal cause hallucinations and depression in the elderly?",
"src": "Patient: Hello. My mother has been taking Xanax for a few years now and had an episode of depression due to being taken out of her home environment. She will be 89 this month and has terrible vision and is almost blind, which adds anxiety and depression when she is removed from her home. Just recently, she was prescribed with Lexipro and Risperdal along with her Xanax. She is having problems with hallucinations and bad depression mood swings. Her doctor said she could stop taking the Lexipro but is still taking Risperdal. I think her problem is with the Risperdal and I want her to stop taking it. She has only been on this medication for less than a month. Her doctor wants her to stay on it so she can be examined. Is it dangerous to take her off this medication when it is apparently the reason for her latest episodes? Doctor: HelloAt this age, overdose of any drug can cause hallucinations. If she is still having issues, you can stop her risperdal. It does not causes withdrawal symptoms. Keep your doctor informed about this.Hope this helps."
},
{
"id": 177370,
"tgt": "Is indentation on back of head of a child after a fall serious?",
"src": "Patient: my boys was wrestling and my 9 year old pushed my 6 year old and he hit his head on the window seal. Its on the back of his head there is a indentation, he cried for a few mins. i applied ice for 30 mins. asked him a bunch of questions gave him a snack, hes acting normal. But the indentation is still there and its been 4 hours. should i take him to the ER or call the doctor tomorrow? Doctor: Hi, the indentations you are feeling could be due to the swelling or inflammation of the surrounding tissue. Unless there is vomiting continuously or Seizures or Watery of bloody discharge from ears and nose or Unconsciousness or Altered sensorium or behaviour, you need not worry. If these appear you need to take him to the emergency room immediately.Regards - Dr. Sumanth"
},
{
"id": 160203,
"tgt": "My hemoglobin is dropping even after 13 IV iron shots & i have fainted 3 times in 4 months. why i am having this ?",
"src": "Patient: my hemoglobin is dropping even after 13 iv iron shots why. i have fainted 3 times in 4 months. MY STOOL test is normal previously had candida but now is OK. hemoglobin has dropped from 11.5 to 9.4 in 4 months. at each fainting episode, I felt the urge to go for a bowel movement but had none and within seconds have severe dizziness and faint. all 3 times I was fine and not doing any strenuous activity or am under no stress or emotional problems. I really need to know why i am having this. my doc thinks there could be some internal bleeding or cancer. I am not sure plz throw some light on it. thnx Doctor: CHECK FOR ANY INTERNAL BLEEDING IN ALIMENTARY CANAL,(ULCER,CANCER,WORMS)CHECK RBC CYTOLOGY. IF ALL ARE NORMAL TAKE OZONE THERAPY"
},
{
"id": 40352,
"tgt": "Can vaccine taken a month after a dog bite prevent rabies?",
"src": "Patient: i was bitten of my dog ( it has a small red dot in my finger not bleed but i wash it and i used povidone iodine) i do not vaccinated month after the bite. after 1 months and several day i went to hospital and receive PEP because i was scared to what i heard about rabies. and now i'm still scared even i receive full course of vaccine. my question is can i get rabies? If the vaccine prevent me in rabies even month after the bite? Doctor: Hi,Welcome to HCM.Rabies is caused by a virus called as rhabdo virus which usually enters into the body through bites of an infected animal most commonly dogs, cats and wild animals. This virus may take up to 10 days to 2 years to produce rabies. this time depends on site of the bite, severity of the bite, animal concerned amount of viral deposit.Based on the site of bite, time taken for infection is: Bite on the legs - 60 daysBites over the hands - 40 daysBites over the neck, face, head \u2013 10-30 daysI suggest you continue an finish the vaccination course.Thanks."
},
{
"id": 103873,
"tgt": "Swollen lips and feet, tingling in mouth, red dots all over body. Taking antibiotics. Recommendations?",
"src": "Patient: Allergic reaction with unknown cause. Swollen lips. Swollen feet. Tingling/numbness in mouth. Have been on bactrim antibiotics for other issue. Have not been exposed to anything out of the ordinary that I am aware of. Red dots all over body. Dots do not itch. Took Benadryl and saw improvements until medicine wore off. What do I need to do? Is there another non-prescription med available? Doctor: Hello and welcome to HCM.Thank you for your query.The first question I would like to ask you is whether or not you were on Bactrim previously? Or is this the first time you have taken Bactrim? Also, if this is the first time, did the assumed allergic reaction occur immediately after the first dose or after a few doses, and which dose caused this? Which infection have you been diagnosed to have?The reason why I ask these questions is because, the rash could very well be related to the infection that you are currently being treated for. This will have to be ascertained by your treating doctor, so please do consult him/her for the same.The next thing is, if this is indeed an allergic reaction to Bactrim, you will have to immediately discontinue this medication. For this as well, you will have to report to your doctor, who will confirm the cause, discontinue the current medication and initiate a substitute.In the meantime, avoid any irritation to the area of the rash, wear comfortable clothing, avoid scratching the region, and avoid application of any unprescribed medication on the region of the rash.Also please drink lukewarm water with a little salt in it, at least three times a day. I am unsure about your infection or other medications, so I cannot comment on other effects you have been suffering. The best thing right now would be to visit your treating doctor.I hope I have succeeded in providing the information you were looking for. Please feel free to write back to me for any further clarifications. I would gladly help you.Best wishes."
},
{
"id": 52910,
"tgt": "What causes symptoms of sickle cell disease after gall bladder removal?",
"src": "Patient: I had my gallbladder removed a while ago. I was rushed to ER after passing out and the findings was problems with my gallbladder. Infact i was asked if i had it removed. Now, my question is..since i have sicklecell trait with symptoms, could this be the cause of having my gallbladder removed? Doctor: Hello,Yes, sometimes sickle credit trait causes problem with gall bladder, was that reason in your case only your surgeon can tell.Hope I have answered your query. Let me know if I can assist you further.Regards,Dr. B.dinesh"
},
{
"id": 54491,
"tgt": "Suggest treatment for fatty inflammation of liver",
"src": "Patient: Hello...i am suffering from indigestion , acidity for last couple of days. During my consultation with a family doctor he recommended me for ultrasound.I have fatty inflammation of liver (grade-1).He has given me hepexa M for a month.Kindly tell me is it sufficient or i have to go for other diagonis as well Doctor: Thank you for posting your query.Fatty Liver disease diagnosed.strict follow the advice (take hepaxa drug) and recheck liver enzymes and lipid profile after 8 weeks:you should also:- abstinence from \"Alcohol\" completely and strictly - fat free diet should be followed- NO red meat- vegetables should be ingested daily- Moreover, less activity should be carried out (no sports) BUT walking 30 to 40minutes everyday is adviced and encouraged - use lemon juice (lemonade) once in a day- decrease weight if obese or overweight-recheck liver enzymes after 8 weeks.wish you good health.further queries are welcomed.Health professionals aim to diagnose properly and manage patients according to their limited knowledge. Cure is blessed by the ONE who Created us, whose power and knowledge is unlimited .wish you good health.regards,Dr Tayyab Malik"
},
{
"id": 41282,
"tgt": "Should implantation bleeding be tried this month?",
"src": "Patient: hello..i am married.since 4 yrs..my age is 27..i had two abortions one was ectopic and other spontaneous abortion in 5th month..now i am again trying for preganancy..i hav PCOS..doctor has advised me to take glycomet 850 daily twice...my height is 5ft 3''..and my weight is 64kgs..should i wait for more weight loss??or shall i try this month?? i am very scared this time..i dnt want to take any chances..can u please suggest ?? awaiting reply..thanks in advance.. Doctor: Hello,Welcome to Health Care Magic.Thanks for writing.I am Dr.Nitasha Khan. I have read you question completely, i understand your concern and will try to help you in best way possible.you wil cncieve people with pcos concieve amd have children thats not a thing to worry about in your current state with your height at least wait for lil bit more weight los like at least below 60 because that wil have better results and out comes also take healthy diet excersise amd walk regularly you wil be fine I hope this answered your question.If you have more queries I am happy to answer.Otherwise rate and close the discussionRegards.Dr.Nitasha Khan"
},
{
"id": 193588,
"tgt": "Prescribe medication for mild enlargement of prostate",
"src": "Patient: I have been recently diagnosed (ultra sonogram)to have a mild enlargement of Prostate. I am 56 years old. I do not need to get up at night for frequent urination. However, I am slightly uncomfortable in my right testicles. Could you please describe a medicine Doctor: Hello, You are having a condition called BPH( benign prostatic hyperplasia). It is a disease of old age men and more than half of men above the age of fifty will suffer from this disease. You can take drugs like tamsulosin for symptomatic relief. In severe cases which fails to respond with medical management, surgical removal of the prostate may be needed. Consult a urologist and he will direct you accordingly. Hope I have answered your query. Let me know if I can assist you further. Take care Regards, Dr Shinas Hussain, General & Family Physician"
},
{
"id": 155924,
"tgt": "Could the cyst at the base of the neck be lymphoma?",
"src": "Patient: Last month while visiting my son in NJ I noticed what I thought was a bite on the back of his neck right adjacent to his left bottom hairline. He told me it was a lyphoma, he said lymphoma without the m); said about three doctors had seen it and they said yes that's a lyphoma and that he could have it removed or not, he said they seemed to be unconcerned. Well I am alarmed -- please tell me exactly what a lyphoma is, whether it is cancer or not (son has since told me he has had this rather smooth-looking cyst, about the size of a golf ball but not protruding at all but it is a little pink which is why I noticed it next to his dark hair -- for about ten years which shocked me as he has never mentioned it and I never noticed it before (he has lived away since gradutating from college, is now married w/three children). The bothersome thing is every time I look it up on-screen, \"Lymphoma\" comes up w/it even more prominently and I can't seem to find a clearcut explanation of it. Please tell me all you can and what we should be doing about it at this stage. Thanks so much. Doctor: Hi Welcome to HCMI have gone thru your query about your son I can understand your concern about it .What I could gather from your discription is that \"cyst ' he is having is ten years old and one thing you have not dascribed whether he has any problem or any illfeeling for which he should feel concerned .I will like to suggest you to consult a relevant specialist and ask what is the remedy for this ?Adapting lifestyle to fight cancer is one of the most effective treatment optionsin my view Making dietary changes such as eliminating sugar from the diet as sugar feeds cancer and eating whole foods slows and can even reverse the progression of cancer. Likewise, taking probiotics or eating fermented food such as Juice of Bitter gourd ,& Aloevera eliminates toxins in the body and boosts the immune system, which also cleanses the lymphatic system I must tell you that our health depends mostly upon 'what & how& when we eat and our life style .Natural supplements also serve as an important defense mechanism against cancer. Iodine, artemisinin and green tea extract are three of the most beneficial natural supplements for debilitating cancer of the lymph system. These supplements support immune health while fighting cancer cells to offer recovery from the disease.Magnitude of the survival advantage from improved nutrition can even be greater than the magnitude of the treatment effects being targeted in current clinical drug trials .A diet full of processed foods will produce biochemical and metabolic conditions in your body that will decrease your immunity, so avoiding processed foods is definitely the first step in the right direction.Take proper diet containing essential nutrients proper exercise & proper rest at right time .One of the important strategies to achieve healthy body is a diet rich in fresh, raw whole foods .More of Fiber, fruit, egg fish ,fish oil for Omega 3, green leafy veges . quality, organic, biodynamic, locally-grown food will naturally increase immunityTo keep metabolism on right path to increase the strength of immune system , take all supplements full of antioxidant serve as antibiotics .coconut water , Lemon juice with water twice before meals , Turmeric powder , level spoon with with a cup of hot milk at bed time ,all these antioxidants maximizes natural minerals so that your body has the raw materials it needs , to do what it was designed to do. To kill bacteria on mass basisPhysical activity in order to renew healthy cell and rebuild itself, so you should make exercise a lifelong commitment.Do 30 mins walk/ exercise - from head to toe ,yoga ,pranayam - deep breatihing , Kapalbhatti / Laughing aloud . to detoxify your system to accelerate the process of recovery . .Contipation is the mother of all illnesses so is mental stress /worry /anger The condition is REGULARITYI further suggest you not to worry and take action fast ,Above regimen will surely gradualy , help you to lead happy worryf ree healthy life ahead HOPE THIS HELPS SOLVE YOUR QUERY Take care All the best Don't hesitate to get back if have any further query"
},
{
"id": 216877,
"tgt": "What causes pain around lower anterior abdominal section while sneezing?",
"src": "Patient: Sneezing causes pain to the skin around lower anterior abdominal section. When I sneeze pain is felt in pricks, rather like pins and needles, over a wide area of my lower back. The pain seems to be topical, affecting (or felt in) the dermis with no well defined cause or center. After sneezing the pain lasts for about five seconds and is slightly relieved by massaging the lower back area with the tips of the fingers. when pain subsides, no residual pain is felt and no discomfort is felt at times other than when sneezing.other activities such as backbends, sudden turns, coughing, running or jumping are not producing this effect to any degree. Doctor: Cannot say in your particular case, but you seem to have......Locating the pain by pressing and poking is one of the main diagnostic methods. Surface, and muscle pain can often be differentiated from spine and other internal sources of pain.The character of the pain such as muscular aches or pins and needles (nerve) is also helpful.Irritation of a local surface nerve is uncommon but generally does not mean much in terms of serious disorders."
},
{
"id": 212493,
"tgt": "Feeling nervous while talking to people, difficulty in socializing. Any advice?",
"src": "Patient: Hello. I would like to have help with a nervous problem of mine. I am kind of good in social situations, but when meeting new people, and i think highly of them as in if they are high status people, i either avoid the situation or become really nervous when speaking. Sometimes, I choke right in the middle of the conversation because of my nervousness . I love meeting new people and such, and at night time when i go out to party, i dont get this problem. but in school, (im 16 years old), i cant socialize and talk to new people because my nerves get the best of me. Any tips? I would like to have other advice than going to therapy . Thank you in advance, Alex. Doctor: Hello and welcome to Healthcare Magic. Thanks for your query. The problem which you have described is indicative of social anxiety. Though you have mentioned that you are able to meet people and socialze, there seem to be certain specific social situations where you become anxious and nervous. The best way to deal with this problem (other than going for therapy) is not to avoid such unpleasant social situations and to slowly keep exposing yourself to them in a graded manner. You can also learn and master relaxation techniques, so that when exposing yourself to such situations, you can readily deploy them whenever you become nervous. As you keep practicing this, with every step forward you will be able to gain more confidence and overcome your nervousness soon. Wish you all the best. Regards, Dr. Jonas Sundarakumar Consultant Psychiatrist"
},
{
"id": 29450,
"tgt": "Is Valtrex effective in the treatment of yeast infection?",
"src": "Patient: Thank you for being here for me. My wife is out of town on business and I was cleaning the house to surprise her, when I stumbled upon a Bottle of Rx Valtrex it was prescribed February 3rd and with a Quantity of 10 tablets. There are 9 and 1/2 in the bottle. She refrains from sex often for the past 3 years says she has a yeast infection. Is this suppose to help with that? and if so it said to take 2 tablets on the first day then take 1 a day for the next 5 days. Would it have been a medicine for a yeast infection? Should I confront her for not taking her meds? Signed worried ... Thank you for your time Doctor: Hello, valtrex is an antiviral. It is used for genital herpes, zoster infection and h.simplex infection( cold sores). It is not used for yeast infection s. For candida infection there are antifungal pessaries available . citrimoxazole, micanozole ketoconazole pessaries(vaginal suppository) and oral treatments are available. your wife can meet your family doctor and speak to him about what agrees with her. You are also needed to be examined , investigated and treated if needed. Hope this helps. Thank you."
},
{
"id": 47937,
"tgt": "Suggest diet and remedies for chronic kidney disease",
"src": "Patient: hello doctor, my mother has chronic kidney disease and she had a dialysis once.before dialysis her creatinine level is 7.2 and after dialysis its level is 3.2.. but now we dont want any dialysis more. Is is possible to cure it with the help of medicines. what is the diet for CKD i we had dialysis once. Doctor: Hello, CKD is a very delicate and serious disease. If by doing dialysis once her creatinine has come from 7.2 to 3, she might need again in future if her creatinine levels goes up. As for medical treatment, causes of CKD in most patients is uncontrolled diabetes mellitus or uncontrolled hypertension. So i would suggest you dont be against dialysis because if her creat levels goes up there are other problems which start to surface for which dialysis is the only solution. So talk to your doctor, see whether her diabetes or hypertension if at all present is under control and get her renal function tests regularly checked and under control. Kindly revert back if you have any further questions. Thank you"
},
{
"id": 6756,
"tgt": "Which is the fertile days after periods to conceive ?",
"src": "Patient: hello me n my husband are tryin to have another baby we had sex once on my fertile days can i still get pregnant if we only had sex once Doctor: Dear patient welcome to HealthcareMagic forum yes you got pregnency.if done sex on fertility period. because in that day.ovum are well doveloped.and highly risk period for pregnency. Thank you......"
},
{
"id": 68613,
"tgt": "What could sensitive nipple with lumps underneath suggest?",
"src": "Patient: Hi, I'm a 15 (almost 16) year old male, recently I've noticeHi, I'm a 15 (almost 16) year old male, recently I've noticed a slight pain and sensitivity to my right nipple, when I rubbed it I discovered that there were a few lumps underneath it, and the whole area underneath the right nipple feels hard and tense compared to my left, I'm a little worried, should I be scared? Too embarrased to approach a doctor and say \"RUB MY NIPPLE\", please help. Doctor: welcome to Health care magic.1.It could be due to excessive stress / rubbing the nipple. If so avoid it for some time, as it could be due to infection / inflammation. 2.Other possible causes could be ductal cyclical cause - will be treated by hormonal therapy.3.If its a recent 1day-2day history with for symptoms to resolve - long standing, then do as follow.4.Get an appointment with your doctor and get examined and request for an ultrasound examination - which will help to assess the nature of lump, its source and extensions.5.Depending on the findings treatment should be initiated.Hope it helps you. Wish you a good health.Anything to ask ? do not hesitate. Thank you."
},
{
"id": 212396,
"tgt": "Borderline personality disorder, stress, anxiety. Taking zoloft, lamictal. Causes and cure?",
"src": "Patient: i have been told i have borderline personality disorder .Ive been really dressed and anxious ltely.I have a 21 month old and just had another who is 2 months.i am a stay at home mom and things are a bit crazy and i seem out f control someimes.could it be post-partum or the borderlline.im so sick of bein and feeling like this.i m on 100 mg zoloft ,100mg lamictal ,and1 mg ofklonopin already.if these arent working wha your opinion? Doctor: Hi there, thanks for asking. The medications you mentioned work for post partum and for other kind of mood problems. I am sure that you use the medications under supervision of a specialist. It may take up to several months to see the desirable effects and most of the time their dose should be adjusted during the several weeks. Borderline personality disorder is a diagnosis that should be made with attention and after several sessions. In cases that specialist is sure about his diagnosis, a proper therapy which is a series of talks sessions (group or individual) is arranged for the person and the response is good. I wish you the best."
},
{
"id": 43374,
"tgt": "What does my sperm test reveal? What is the meaning of spermatozoa?",
"src": "Patient: Sperm test done, what do the results mean good, bad so/so? Component Your result SPERM COUNT POST VASECTOMY , HPF 20 SPERM SPERMATOZOA, MOTILE % POST VASECTOMY, HIGH POWER FIELD YES ANALYSIS TIME, SEMEN 16:03 SEMEN VOLUME 1.0 SPERMATOZOA, NORMAL % 30 WBC COUNT, SEMEN 1 SPERM COUNT, SEMEN 21.8 SPERMATOZOA, IMMOTILE % 38 % SPERMATOZOA, NONPROGRESSIVELY MOTILE %, SEM FLD 24 % SPERM SLOW PROGRESSIVE %, SEMEN 33 % SPERM RAPID PROGRESSIVE %, SEMEN 6 % SPERM, PROGRESSIVELY MOTILE, TOTAL, SEMEN 8 m Doctor: HelloThanks for your query.Spermatozoa is the male egg ( sperm ) which when meets with the female egg ( ovum ), then it results in a pregnancy.Please attach or upload your report clearly for me to comment further.It appears as if you have had a vasectomy, but few sperm are still seen in your sample.Take care."
},
{
"id": 10304,
"tgt": "What are the side-effects of the hair-loss medicine, Keragole?",
"src": "Patient: i need information about the usage of the drug keragole forte and i also need that taking this drug may also lead to ageing of the skin or not my age is 20 actually my hair loss promblem started by intaking of the drug called saw palmetto and then i am now using keragole forte need an answer immediately pls. Doctor: Hello and Welcome to \u2018Ask A Doctor\u2019 service. I have reviewed your query and here is my advice. Keraglo forte is a hair nutrient tablet which is to be taken once a day for a minimum of 2-3 months for good results. Unfortunately hair fall or hair loss is not cured by one factor only. You will have to do adjuvant therapies like hair PRP - platelet rejuvenation plasma treatment, where plasma from your own blood is injected into your scalp with the help of tiny needles. This plasma is rich in growth factors and platelets which further strengthen your follicles and promote new growth. Also I suggest you start with topical Minoxidil 5% once daily for at least 6-7 months for good results. Hope I have answered your query. Let me know if I can assist you further."
},
{
"id": 52907,
"tgt": "What causes burning and bloating of stomach post gall bladder retrieval?",
"src": "Patient: I wake up, and my stomach feels like its cramping, and somewhat bloated. If i get up and move around more, it does not seem to bother. I suffer most of the day with burping and feels like gas buildup and somewhat bloating of the stomach. I got my gallbladder out a few years ago, and ever since then i have had issues with bloating feeling. Every morning my stomach feels like its burning etc. PLease let me know. I tried most medicines, and does not work. Some like acid reducers seem to cause weight gain? Not really sure. Just looking for better opinionsS Doctor: Hello,Avoid fatty meals. Try some antigas tablets or powder. Take plenty of water and dietary modifications required.Hope I have answered your query. Let me know if I can assist you further.Regards,Dr. B.dinesh"
},
{
"id": 140146,
"tgt": "What causes numbness in hands,jaw clenching,body tremors and rapid breathing?",
"src": "Patient: had my eeg fri. i had clenching jaw, whole body shook 3 different times ,felt like i was going to throw- up, i could smell metal this was during rapid breathing oh, also sharp pains thru out my body and numbness down to my hands. is this normal ? RC Doctor: Hi, Your symptoms are not normal. There is still a possibility of seizures, even though your EEG is normal. EEG may be normal in about 50% of patients with epilepsy. You should undergo further evaluation, such as long term EEG, MRI of brain and PET CT brain, if needed. Hope I have answered your query. Let me know if I can assist you further. Take care. Regards, Dr. Sudhir Kumar, Neurologist"
},
{
"id": 120753,
"tgt": "What causes hole in hip bone?",
"src": "Patient: About 8 years ago through an xray my doctor found that I had holes in my left hip bone. He called it osteopelekious(misspelled). Last year my youngest son who is 19 was expeiencing the same pain and had his hips xrayed and the same diagnious but his is stationed in his right hip. Can you provide any information. Thank you Doctor: Hi,Based on the history you and your son seem to be suffering from a disease called osteopetrosis. It is a bone disease that makes bones abnormally dense and prone to breakage (fracture) and it is an inherited condition (runs in family).It can cause fractures, low blood cell production, and loss of cranial nerve function causing blindness, deafness, and/or facial nerve paralysis. Affected individuals may experience frequent infections of teeth and the bone in the jaw.Hope I have answered your question. Let me know if I can assist you further. Regards, Dr. Mohammed Taher Ali, General & Family Physician"
},
{
"id": 75192,
"tgt": "How to get rid of pain and itchy feeling in my lungs?",
"src": "Patient: HI I HAVE HAD PAIN AND AN ITCHING FEELING IN MY LUNGS FOR FIVE MONTHS NOW. I AM ALSO BRINGING UP YELLOW PHLEMN AND CANNOT BREATH PROPERLY. MY DOCTOR HAS GIVEN ME QUITE A FEW ANTIBIOTICS THAT HAVE NOT CLEARED IT UP AND I AM NOW ON STEROIDS, BUT I AM STILL HAVING TROUBLE. CAN YOU HELP ME PLEASE. CHRISTINE. Doctor: Hi,Cough expectorantion doesn't always means bronchitis like infection. Cough apart from bronchitis can be from allergy as well as from acid reflux or pneumonic consolidation. Your work up done is as follows:- Auscultation - CBC with AEC value also - Chest x ray -sputum examination - Spirometry only if needed after above report As you are having itching complaint you can have allergic cough and if CBC shows eosinophila than Allegra M like drug course prescribed for that. According to cause specific treatment plan decided. Consult pulmonologoist with keeping my answer in mind.Hope I have answered your query. Let me know if I can assist you further. Regards,Dr. Parth Goswami"
},
{
"id": 149881,
"tgt": "Teenager. History of cerebral palsy and type 1 diabetes, seizures. Diagnosed epilepsy. Depression, mood swings after taking epilim. Any other medication?",
"src": "Patient: Hi my son aged 19 with a history of cerebral palsy and type1 diabetes(diagnosed aged 16) has had 2 seizures within the last 8 weeks. He was diagnosed with epilepsy and started on epilim 500mg twice daily. His mood was not particularly good before the diagnosis of epilepsy as he was struggling to adjust to diabetes and the frustrations of cp. Since being on the epilim his mood has been very low and he has had an episode of acute anxiety. I feel that the epilim is making his mood worse and wondered are there any better drugs out there with better effect on mood eg lamictal? (i have read a little about this drug) Doctor: hi,epilim is very good for seizure, but before this first tell me about his seizure episode.which type of symptoms he had during seizure??is there history of tongue bite, incontinence of urine, unconsiousness, automatism, alter behavior during seizure??depend upon type of seizure if seizure are partial in nature then carbamazepine (mazetol) is also a very good option.and what about seizure control with epilim??? if seizure control is not good then patient may develop mood and anxiety symptoms, with change in medication it will subside within few days.if symptoms lasted for more than 15 days and more disturbing then antidepressant( sertraline ) and antianxiety (clonazepam ) will helpful.so please discribe type of symptoms during his seizure episode, then i will suggess other better medication for epilepsy."
},
{
"id": 17130,
"tgt": "Is an elevated heart rate for a few minutes after cardio workout a concerning matter?",
"src": "Patient: Hello: I am a 69 year old female who attends weekly zumba classes. Last week after class my heart rate remained at 140 for 45 minutes after class ended. Is this something that should concern me? I have recently seen my cardiologist for an annual. No heart problems other than a murmur. Doctor: Hello, It is normal due to sympathetic stimulation while doing cardio. But an exertion and increased heart rate need increased blood and oxygen supply to the heart. In the elderly, the cardiorespiratory reserve is less so it might be dangerous. Do it supervised and as per instructions of your cardiologist. Hope I have answered your query. Let me know if I can assist you further. Take care Regards, Dr AJEET SINGH, General & Family Physician"
},
{
"id": 8495,
"tgt": "Suggest a cream for improving skin complexion",
"src": "Patient: Hi I am a dark skinned asian male, i was using cosmelite which gave me a good complexion however is started getting pimples becoz of that..hence i stopped cosmelite..now i have becum more darker..wat shud i do? shus i continue to use cosmelie is it a good cream for any skin type? Doctor: Hello, Thank you for posting on HCM.Complexion of any person is has two two components; constitutive, one which is predetermined by genetics and racial factors and usually cannot be changed with any treatment. The other component is due to external agents like environmental exposure, sunlight, use of any cosmetics etc and is amenable to treatment.Avoid cosmelite cream as it ccontains steroid and hydroquinone.You can get a series of chemical peels, which actually remove the upper dead layers of skin and stimulate regeneration of collagen and thus newer layers of skin. Many options are available in chemical peels depending on the requirement and skin type. (Glycolic peel, retinol peel, cocktail peel etc)Also, chemical peeling can be preceded by a simple procedure called microderamabrasion or commonly known as skin polishing to augment the results.Also, at home you can try using cream containing kojic acid, tetrahydrocurcumin, arbutin etc for several months (Melaglow cream) for additional benefit and maintenance.Always use broad spectrum sunscreen before going out in sun.Use a plain moisturiser like cetaphil DAM on daily basis for healthy and hydrated skin.Take plenty of fresh fruits and vegetables in your daily diet.Hope this will take care of your issue.Thank youDr Hardik Pitroda"
},
{
"id": 37110,
"tgt": "Suggest remedy for cyst on buttocks with foul smelling",
"src": "Patient: Hello. My 16 year old daughter has a huge draining cyst or boil between her buttocks that is foul smelling. I have been having her soak in Epsom salt, spraying hydrogen peroxide on it, and wiping it with alcohol. What else should I do? Does she need immediate medical care or can it wait until we can see a doctor in the next few days? She is very uncomfortable sitting on a golf ball Doctor: Hi,From history it seems that she might be having infected sebaceous cyst or forming abscess over the part.Meanwhile you consult your doctor, give antibiotic medicine.Clean the part with antiseptic lotion and do dressing with antibiotic cream.Ok and take care."
},
{
"id": 176225,
"tgt": "How to treat sore throat in a 9 years old child?",
"src": "Patient: My 9 yr old daughter has a very sore throat and finding it hard to swallow . She is Feeling generally unwell and lethargic. Says her neck is sore and achy and her ears keep popping and hurt when swallow. Have given paracetamol, gargles, cold drinks etc is there anything else I can give her? Thanks Doctor: Hi,From history it seems that she might be having acute upper respiratory tract infection.She might require one course of antibiotic medicine like Azithromicin or amoxicilin for 5 days.Give her ibuprofen medicine for pain and fever instead of paracetamol as an anti-inflammatory medicine as well.Give her light diet.Avoid cold drinks and junk food.Ok and take care."
},
{
"id": 188738,
"tgt": "Painful mouth, tooth filling hurts, headaches, hot flushes. Is it something serious?",
"src": "Patient: my partner is suffering with severe pain in his mouth, this has been an ongoing issue for over a year which comes and goes but this time it is a lot worse. the pain seems to be centered around one particular tooth which has a filling. it sometimes bleeds and is sometimes smelly. He is having headaches because of it and has been getting hot flushes. I am currently waiting for a call back from an emergency dentist but can you suggest anything other than pain killers as this isnt working. He has also been using a dental oral gel but this isnt numbing the pain. Doctor: Hi thereThanks for asking your queryThe history provided shows that your filled tooth has developed infectionThis tooth is causing you pain around tooth which is radiating towards headGet an xray done to evalute the filling and toothIf infection is deep in pulp with periapical involvement then filling has to be removed and root canal treatment should be doneMeanwhile pain can be treated by analgesicMaintain good oral hygieneConsult your dentist too Thanks"
},
{
"id": 218821,
"tgt": "Can one get pregnant despite taking contraceptives?",
"src": "Patient: I missed two days of birth control on the combination pills not the sugar pills, but I m up to date on them and have been taking them normally for 2 days. My boyfriend and I had sex and he didn t ejaculate in me or before he inserted his penis. What are the chances I could be pregnant? (I ve been on birth control for almost 2 years) Doctor: if you have missed 2 pills than you are supposed to use alternative mode of contraception because missing two pills predisposes to alteration in hormone and can lead release of ovum.Now take i pill as you had unprotected intercourse."
},
{
"id": 202584,
"tgt": "Ejaculated in condom but rubbed the penis on vagina and it is sore. Will I get pregnant?",
"src": "Patient: hi my fella had a condom on he ejaculated it was all in condom and he slid his penis rubbed on my vagina then slid out then up rubbed again contents was all in condom but vagina is sore im dew my period and scared i wont come on how do i know i had sex Doctor: Hello and welcome to health care magic. Your description is a bit unclear. What I understand is that your partner took his penis from r condom after ejaculation and rubbed it on your vagina. If so there is high chance of you getting pregnant. That's because even 1 ml of semen contains millions of sperms and they can swim upwards through the vagina to your uterus. My advice is to go and visit your OBGYN doctor at the earliest. She can prescribe a post coital contraceptive pill if the sex happened within 48 hours. Hope this discussion will be helpful to you. Thank you."
},
{
"id": 207593,
"tgt": "Suggest treatment for hallucinations and disorientation",
"src": "Patient: patient had seizure first time ever in september, i came home and patient was extremely confused in discussions, he started hallucinating, took to hospital and he was sent home with ativan, took one and he got 100x worse, he is now in critical condition Doctor: Hi.Thank you for asking question. I had gone through it.I understand your concern.It is post ictal confusion.It is occur with seizure.After convulsion has stopped there is period in which patients become disoriented and hallucinated.This period can range from 30 min to 2 days.But proper treatment of seizure can prevent further convulsion and post ictal disorientation.Ativan contain lorazepam and only ativan can not help.This kind of convulsion can be treated with medicines like phenytoin, valporate, levitracitam,clobazam etc.Consult best neurologist and get help.I hope i have answered your query.Happy to help you.Thank you."
},
{
"id": 143985,
"tgt": "Suggest treatment for severe nose bleed and dizziness",
"src": "Patient: Hello I normally do not have nosebleeds. And the last 4 days I ve had for bloody noses. These are concerning to me because I have a hard time to get them to stop bleeding. After many attempts with a cold pack on my nose with my head tilted forward it takes about the over 30 minutes to get the nose bleeds to stop. But after many blood clots and lots of blood later they finally stop. Now my face is feeling numb I m kind of dizzy and very very tired Doctor: You must go to an ENT specialist to rule out any local trauma & for management which may include cautery to stop bleeding & nasal packing."
},
{
"id": 116185,
"tgt": "What is the treatment for pulmonary eosinophilia?",
"src": "Patient: Hi, may I answer your health queries right now ? Please type your query here... Hi i have suffered with pulminary easinophilia for 15 years now, and i have just started back work (1 year) i have had so much time off, with this lung condition, which leads to chest infections etc, would i be able to claim DLA, if i were to pack up work and stay at home with my children ?? Doctor: HIWell come to CHMChronic allergic condition may be likely, absolute eosinophilic count need to be done with X-ray chest some other etiology would be possible and this need to be differentiated, steroid some time helps, but you must be qualified for this, better to have a word with physician, hope this information helps, take care."
},
{
"id": 163093,
"tgt": "How can fever with a barking cough be treated?",
"src": "Patient: Hi, may I answer your health queries right now ? Please type your query here... My 2 year old son just started getting cold symptoms yesterday and has had a fever of 100 -103. Tylenol and Ibuprofen has not taken care of it. He is still very warm and has a barking cough. What do I do? Doctor: Hello and Welcome to \u2018Ask A Doctor\u2019 service.I have reviewed your query and here is my advice. First, you should give your child a healthy diet, plenty of water to help increase her immunity. A warm shower will help the temperature to decrease and wet clothes. About the cough try aerosol with NaCl. Combination of Tylenol and Ibuprofen are the best solution. If all of these doesn't help then you should go to see a doctor. Hope I have answered your query. Let me know if I can assist you further.Regards, \u00a0\u00a0\u00a0\u00a0\u00a0Dr. Elona Dashi"
},
{
"id": 84026,
"tgt": "Is there any side effects for neo pride total capsule?",
"src": "Patient: hi sir i m taking neo pride total capsule for nausea problem for last one month. i m feeling better . doctor has adviced me to continue medicine for one more month. iwant to know is there any side effects of the medicine or not . thanx my age is 36. m Doctor: Hello, It is a safe drug and side effects are minimal. However prolonged use for more than six months is not recommended because of cardiac complications. Hope I have answered your query. Let me know if I can assist you further. Take care Regards, Dr. Shinas Hussain"
},
{
"id": 123453,
"tgt": "How to get rid of the pain in my left shoulder?",
"src": "Patient: I HAVE ACH IN MY LEFT SHOULDER , I HAVE DONE MANY TESTS & CT SCAN, BUT I DONT FOUND ANY DISCRIPENCY IN ALL TEST REPORTS, CT SCAN , ONE FRIEND SUGGEST TO ME FOR PET SCAN TEST, IS IT FAVOUR TO ME FOR SHOULDER PAIN & PL YOU GUIDE TO ME FOR RIGHT PLACE IN DELHI NCR FOR TEST & COST Doctor: Hello, I will suggest doing MRI shoulder. If the shoulder pain is persisting from long and no solution provided. Need to see the soft tissue of the shoulder joint. Exercise will help to stabilize the shoulder and improve the overall shoulder functions. Having a word with a physiotherapist who can help with therapeutic ultrasound therapy and TENS therapy for reducing the inflammation and pain. Hope I have answered your query. Let me know if I can assist you further. Take care Regards, Jay Indravadan Patel, Physical Therapist or Physiotherapist"
},
{
"id": 78651,
"tgt": "What causes rumbling sensation in left of chest?",
"src": "Patient: Hi, I seem to be getting a rumbling sensation in the left of my chest. It feels similar to stomach rumbling but it feels like its my heart. I have had slight chest pains for a few weeks now and my pulse feels slightly week after the rumling/growling sensation. what could this be? Doctor: Thanks for your question on Health Care Magic. I can understand your concern. Possibility of gastritis is more in your case. But since it is left sided chest discomfort, better to first rule out heart related diseases. So get done ecg and 2d echo. If both are normal then no need to worry for cardiac diseases. Gastritis appears more likely cause for your symptoms. So avoid stress and tension. Drink plenty of fluids orally. Start proton pump inhibitors. Avoid hot and spicy food. Avoid junk food. Quit smoking and alcohol if you have these habits. Don't worry, you will be alright. But first rule out heart diseases. Hope I have solved your query. Wish you good health. Thanks."
},
{
"id": 45883,
"tgt": "What causes discharge from the penis with gastrointestinal and lower back pain?",
"src": "Patient: I had a penal yeast infection and it seems to have cleared up. I also have some discharge from the penus and gastrointestinal and low back pain. I was wondering if the yeast infection has gone into my kidneys and stomach. Are there tests to confirm this? Doctor: Hello and Welcome to \u2018Ask A Doctor\u2019 service. I have reviewed your query and here is my advice. You can go for swab and check whether the infection persist or not. As of now take antifungal drugs like Fluconazole once a week for four to six weeks. Hope I have answered your query. Let me know if I can assist you further. Regards,\u00a0\u00a0\u00a0\u00a0\u00a0 Dr. Shinas Hussain"
},
{
"id": 59534,
"tgt": "Diabetic. Ultrasound abdomen shows mild fatty liver, cholelithiasis. Suggestions?",
"src": "Patient: My USG abdomen report says, the liver is normal in size(span-14cm),shape,position and shows bright echotexture. No evidence of focal lesion. No IHBR dilatation. Portal vein and common bile duct appear normal in course and caliber. No proximal CBD calculus . Gall bladder : well distended and shows mutiple calculi ranging in size from 5-6 mm. No pericholecystic fluid or wall thickening noted. Kidneys : both kidneys are normal in size, shape and echotexture. No evidence of focal lesion. No calculus / hydronephrosis on both sides. Corticomedullary differentiation is maintained bulaterally. right kidney measures 104x45 mm and left kidney measures 106x47 mm. No suprarenal mass lesion on both sides. Approximately 9mm sixed cortical calcification noted in the upper pole of left kidney. Impression : Mild fatty lever. Cholelithiasis. I was recently diagonised with Diabetes Mellitus Doctor: Hi. You didn't mention the reason for taking Ultrasound Abdomen.If you have pain in the upper part of abdomen on right side below the ribs,then it may be due to Gall bladder stones. Mild fatty liver is not a concern.it is mainly due to alchohol and diseases like Diabetes,Hypertension,Obesity etc.It is reversible. Please consult your doctor and take anti diabetic drugs regularly. You should also meet your surgeon if you have pain abdomen.Otherwise,it is an incidental finding. Wish you good health"
},
{
"id": 44275,
"tgt": "Have amenorrhea. Pelvic scan shows anteverted uterus. Suggested hormonal profile for infertility",
"src": "Patient: I have secondary amenorrea. Result of pelvic scan says: non bulky and non gravid anteverted uterus with smooth outline and uniform myometrial echopattern AP diameter 43mm. Thin endometrical plate noted. The adnexae are free. Pouch of Douglas is empty. Normal bladder in wall thickness and echopattern. Impression: Normal pelvic scan. Suggestions: transvaginal scan, folliculomentary, hsg and hormonal profile for infertility. Doctor: Hello. Thanks for writing to us. Your ultrasound report is within normal limits. The secondary amenorrhea that you are having in the absence of pregnancy is likely to be due to hormonal imbalance. Premature menopause needs to be ruled out after day 3 LH, FSH level assessment. I hope this information has been both informative and helpful for you. Regards, Dr. Rakhi Tayal drrakhitayal@gmail.com"
},
{
"id": 80386,
"tgt": "What causes cough and chest pain with normal chest x ray?",
"src": "Patient: I have had a cough and chest pain for 3 months. The first time I went yo the dr, she said I was wheezing slghtly. She wrote a prescription for prednisone for 10 days. I got better but not good. I went back to dr and she prescribed stronger prednisone for 12 days. I finished that 4 days ago. Today I started coughing again, my chest hurts and I can t get warm. I had a chest x ray on y second visit to dr and everything was normal. now what? Doctor: Thanks for your question on HCM. I can understand your situation and problem. In my opinion you should consult pulmonologist and get done 1. Clinical examination of respiratory system. 2. PFT (Pulmonary Function Test). There is high possibility of bronchitis on your case. Because bronchitis can cause cough, wheezing, chest pain with normal chest x ray. And PFT is must for the diagnosis of bronchitis. It will also tell you about severity of the disease. And treatment of bronchitis is based on severity only. Better to avoid oral steroids as they are harmful to the body. You may need inhaled bronchodilators and antihistamines drugs. So better to consult pulmonologist and discuss all these, first diagnose yourself and then start appropriate treatment."
},
{
"id": 188006,
"tgt": "What could be done to keep an infected tooth intact without removal?",
"src": "Patient: I have 5 teeth on bottom one is crowned and have a bridge. Abcess formed in one next to crowned. I thought it was crowned. Have no pain. Dr. wants to remove middle tooth due to infection but that leaves me with an open hole in middle. I had to stop taking antibiotica afer4 days and seems no more pus. Had no pain but bad taste. What to do? I don't know how to put another tooth in. Dentist says he can't, Doctor: HiThanks for asking Since it has abscess, and if the tooth is firm then you can go for an root canal treatment and teeth can be saved."
},
{
"id": 56938,
"tgt": "What causes pain during urination?",
"src": "Patient: i have a genital condition which the first doc diagonised as an infection caused by bacteria and prescribed an injection with antibiotic for seven days which only caused my pennis to swell severely,the next doctor thought this were genital warts and only prescribed antiflamatory medicine which never improved the situation.I saw a third doc who diagonised this as genital herpes and prescribed acyclovir which indeed reduced/eliminated the the swelling. i am now having very strong pain when passing urine and sometimes see traces of blood. What could this be? Doctor: Hi,Thanks for posting your query.I am Dr.R.K and I am pleased to assist you.Pain in the shaft of the penis while urinating with traces of blood could be due to urethritis ( Infection/inflammation of the urethra).If the pain is felt in the lower part of the abdomen, then it could be due to cystitis ( infection/inflammation of the urinary bladder).Urine culture sensitivity test may be done.I hope that answers your question.Regards."
},
{
"id": 86086,
"tgt": "Suggest treatment for gastric pain",
"src": "Patient: Hi, can I answer your health question? Please type your question here...I am unable to eat anything spicy I am suffering from gastric for the last five years only a temporary cure for few days for the medication I have taken was pantodac 40 mg and domperidone Doctor: Hello and Welcome to \u2018Ask A Doctor\u2019 service. I have reviewed your query and here is my advice. This may be caused by gastritis, GERD or hiatal hernia so you should do gastroscopy to evaluate it more accurately. This is caused by irritation of stomach mucosa by certain food or medications, stress and smoking. SO firstly it should be treated with home remedies and changing dietary habits. You need to avoid fried food, carbonated drinks, coffee, alcohol and spicy food. You should eat more milk products, vegetables and boiled food. Also, certain antiacid medications may be required such as Pantoprasole or Ranitidine. In cases of severe pain, diagnosis should be established with endoscopic tests which include gastroscopy or barium swallow test. Hope I have answered your query. Let me know if I can assist you further."
},
{
"id": 213271,
"tgt": "Been trying to get pregnant. Why did I start bleeding light?",
"src": "Patient: I have been trying to get pregnant since last march 2012 but everytime i get upset by getting my periods however my periods always comes on time 30 to 32 days. i was pregnant in june 2009 but i did t wanted to have baby so its cured with medicine nad this time it taking too long also my work is quite stressfull as i am working in rotating three shifts however i think that also could reflect for conceiving so i decided to leave job and be relax at home. This month started light bleeding during my mid cycle and very light cramping as well also getting little blood during intercourse what is it i talk to DR here but they does not give proper info n consultation can you please explain it would help me i m very upset month by month by trying as i know no problem in me n my husband as well Doctor: Need to check your hormone levels like: FSH, LH, PROGESTERONE AND ESTROGEN Consult a good gynaecologist Stress can cause hormonal imbalance The fact that you had already conceived once in past, unlikely that some major problem would be there Still you should consult a gynaecologist No need to feel guilt about last event in 2009"
},
{
"id": 11955,
"tgt": "How can I get rid of the dark staining over the butt crack ?",
"src": "Patient: Can anyone help me with information on how to get rid of dark staining on my butt crack and if it gets worse over time? Is it normal at my age? Female, 24 yrs old Doctor: hello and welcome to HCM forum, i would like to inform you that darkening may be normal until you suffer from any systemic disease. kindly consult a dermatologist, he may prescribe you a whitening cream for the concerned area. regular use will resolve your problem. i wish you good health."
},
{
"id": 61821,
"tgt": "What causes lump behind the ear?",
"src": "Patient: Hi, I have a hard lump behind my ear (2cm diameter), where the head joins the neck. It feels like it is bone, sticking our from my skull. When I lie down I feel dizzy and need 2 or 3 seconds to get my bearings. I had an xray and the radiologist has asked me to come back for a CT scan. Any indication of what this may be? Thankyou Doctor: Hi Dear,Welcome to HCM.Understanding your concern. As per your query you have lump behind the ear are pointing towards a condition known as cyst or a Sebaceous cysts . These sack-like lumps are made up of dead skin cells and oils produced by oil glands in the skin. It could be due to pressure and impingement of nerve endings. Need not to worry. You should avoid touching or pricking it with any sharp object. I would suggest you to consult ENT specialist for proper diagnosis . Doctor may order certain tests like audiometry, CT scan, tympanometry and do physical examination to rule out condition like lymphadenopathy . If a cyst becomes painful, it may be infected. Treatment may include antibiotics or removal of the cyst. For now give warm compresses to affected area .Hope your concern has been resolved.Get Well Soon.Best Wishes,Dr. Harry Maheshwari"
},
{
"id": 165877,
"tgt": "What causes vomiting in a 4 year old child?",
"src": "Patient: My 4yr old has been vomiting all.night. she vomited her dinner the first few times, then it was yellow and now green. She s now started complaining about her tummy hurting before she vomits. she had a little diarrhea ,but only once. What could.this be? Doctor: HIWell come to HCMI really appreciate your concern, the given symptoms could be due to dyspepsia or it could be due to viral infection in either of this case may not need any treatment because it is the self limiting one but of course care needs to be taken for dehydration and vomiting if vomiting persist then it can be managed with anti-emetic medicine either orally or injectable root, hope this information helps."
},
{
"id": 188546,
"tgt": "Thallium levels in urine were high. Had all teeth extracted which had thalllium in filling, still high. How long it stays ?",
"src": "Patient: I discovered that i had thallium in my amalgum fillings and the thallium levels in my unprovoked urine test was off the charts. I had all my teeth extracted and the Thallium levels went within the considered normal ranges. Now 5 months later it it back up to abnormally high levels, less than before I had my teeth extracted but significantly more than just 3 months after the extraction. The electrical system in my heart is already been effected from the thallium. How long does it take the body to get rid of thallium? Doctor: Hello,Thanks for writing to us.I have read your concern.Since the thallium level has raised even after extraction of all teeth,the cause for raised thallium level has to be evaluated.Usually body will get rid of such toxicity within 6 months-1 year.If you had ingested thallium,gastric lavage has to be done.Please do visit a dentist as well as GP for further reference.Take care."
},
{
"id": 30218,
"tgt": "Suggest treatment for infection in the small intestine",
"src": "Patient: Hello sir, my name is amit, i am from india... my problem is that in november, 2009 i was suffering from typhoid high fever..i was go through medical treatment upto 4 months, but all is vain. every doctor have his unique opinion. they didn't make diagnose .then i meet dr. rajinder , my further treatment was go through under the observation of dr. rajinder. during his treatment their is a fine improvement in my heath. After one and half month he sugesst me start DOT treatement with Cat 3, as a factor of tuberclosis. i took the tretment . for the next 4 months everything was going to be very fine , their is no problem... but now from last 6 days ihave the same problem again..i have high pain in my abdomen every time. doctors said their is an infection in my small intenstine ... i consult 4 doctors in these 6 days including dr. rajinder , but i have no relief..... today i meet dr. yogi and he start my treatment with an injection namely as NETSPAN(NETILMICIN SULFATE INJECTION USP). HE SAY THE TREATMENT DURATION WILL BE 10 DAYS DURING WHICH I HAVE TO TAKE ONE INJECTION DAILY.......I WANT TO ASK IS THIS INJECTION IS GOOD FOR ME . As i think the doctors are unable to diagnose my problem. i wanna help. please help me Doctor: HiThanks for posting on HCMI will say you need to trust your Doctor to get over your problem.Changing Doctors is not the best attitude.You may have to see a Doctor and again for reevaluation to be well diagnosed and also to evaluate evolution on treatment.Abdominal pain could be caused by bacteria infection and in this case, antibiotics will be indicated with netilmicin being one of the drugs well indicated in abdominal infections.However, this drug may cause kidney and nerve problems.Your Doctor will always evaluate the risk and benefits of the drug before prescribing.If i were your treating Doctor, i will face your problem first with a stool culture and antibiotic sensitivity test.I will advise you continue with the drugs as prescribed by the Doctor since you are in the hospital since duration of treatment up to 12days(averagely 7days) may be accepted.However, you may have to have your renal functions evaluated.Hope this will helpBest regards"
},
{
"id": 80834,
"tgt": "What could cause on and off sharp pain in chest in teenage?",
"src": "Patient: hello , im am 14 years old and is having sharp chest pains on my heart about 3 times a week and they last about 15-20 seconds long but sometimes they last 1 minute , im and not over weight i weigh 103.5 pounds and is 5 2. Do you have any idea what this could be? its been going on since i was about 12 or 13 Doctor: Thanks for your question on HCM. I can understand your situation and problem. In my opinion you are mostly having anxiety and related symptoms. At your age(14) cardiacdisease is extremely uncommon.But better to rule out cardiac cause first for your chest pain.So get done ecg. If ecg is normal then no need to worry much for cardiac cause. You are having anxiety mostly.Better to consult psychiatrist and get done counselling sessions. Try to identify stressor in your life and start working on its solution. Avoid stress and tension. Be relax and calm. Don't worry, you will be alright."
},
{
"id": 172552,
"tgt": "What causes red small raised bumps in clusters on the legs of a child?",
"src": "Patient: hi, my toddler has red small raised bumps in clusters on her leg starting to spread to her other leg with a little spot and a tiny one on her belly now. No other symptoms that I know of or that she is displaying. What could this be? She also had bad eczema when she was a baby but has not had it since and it is raised like that but looks alot different. Doctor: Hi...I feel by what you quote he should be having a - Hand Foot Mouth disease. This is one viral illness among all other exanthemas which can cause fever followed by rash over palms and soles. It is a self-limiting disorder and itching can be really worrisome. I suggest you use any over the counter antihistamine if you have one with you now. You can use Hydroxyzine at 1-2mg/kg/dose (Maximum 10mg) every 6th to 8th hourly for 7 days. This can even cause some peeling of skin in the next 4-6 weeks and do not worry about it.Regards - Dr. Sumanth"
},
{
"id": 195931,
"tgt": "Why is my penis hard even after ejaculation?",
"src": "Patient: I'm 31 years old. A couple of months ago I started having erection without stimulation of any kind and my penis is growing and producing too much sperm. Even after the release it's still hard. i consulted a doctor and got tested. The doctor said I'm lucky and I need to release it at least four times a day. is it normal to have erection that lasts more than four hours without physical or psychological stimulation and to produce sperm 15-18ml? Doctor: hellothank you for trusting HCMDear have you feeling any pain during erection??have you feeling any hard band like structures during erection??have you suffering from any systemic diseases??erection lost more than four hours is not a normal phenomenon please consult your doctor and get scan for penile blood supply and other systemic diseases like sickle cell disease,local like peroneys disease etc.. I think I answered to your question if you have more questions feel free to ask.take care"
},
{
"id": 203833,
"tgt": "What could be done for irritation and discomfort in penis and testicles after masturbating,with history of testicle torsion surgery",
"src": "Patient: Okay, i am 18 and i am in 12th grade, i ha testicle torsion surgery in 9th grade, it was all normal afterwards except a little discomfort but i could deal with it, but in the spring of this year i got many tippers and i got hit pretty hard 3 times, after the third hit it started to be more uncomfortable and a little pain for under an hour every now and ten, but they became monthly 3 months ago, now fast forward a bit to 2-3 weeks ago. I was masturbating harder than normal and it felt like i pulled something, twice in the same spot, after that happened i got very very nausoius and dizzy, i had pain for an hour, i went to my doc who did my surgery and he did an exam, he said everything was alright as in no cyst and it didn't twist and no cancer, i forgot to tell him i was masturbating but he said i probs pinched it or something. It has been more uncomfortable ever since that happened, like almost everyday its uncomfortable, i masturbate 3-4 times a day usually and still do, they seem to swell up a little then go back down sort of, or they just get bigger for a little bit and then go back down, but something that is troubling me is that my skin around my testicles are cold, like usually i dont feel them being cold but they are now all of a sudden, and the same thing with the tip of my penis, ik that its normal if they are a little colder but idk, and ik that te left testicle is usually bigger but my right one is bigger, the same side where i think i pulled something, my questions are these:1. I masturbated 3 days after my torsion surgery, nothin to hard but idk, could that affect it?2. Is it inflammed?3. If so what should i do4. What are the signs of inflammation5. Should i get an ultrasound?Please help and ease my mind haha, i have high anxiety so you can tell why i am worrying so much. Doctor: Hi i do not think masterbating causes this kind of feeling after surgery.signs of inflammation can be swelling .pain erythema and increase in local temperature.but inflammation cannot be for so long after surgery even then use antibiotic twice daily and anti inflammatory agent twice daily can decrease the inflammation if present.Yes ultrasound can be done to see for any changes in the passage of sperms.i think simple neurokind plus daily once can cure the condition"
},
{
"id": 104747,
"tgt": "Have an upper and lower respiratory tract infection, given decadron injection, crackling in ears. Sinus infection affecting vision?",
"src": "Patient: Hello, I have an upper and lower respiratory tract infection , i was given a decadron injection and put on azithromycin and prednisone for 5 days. I take proair inhalator every 4hrs to help breathing.I am on my 3rd day. Since i saw my doc i have gotten more sinus congestion. Pain in my ear stopped but now have crackling of both ears when inswallow. Yesterday i started to have vision changes. Near visio is ok but far everything seems blurry. Is this related to medication or a sign that my sinus infection is affecting my vision? Thank you for reading. Doctor: Hi, These both donot have any issue with the vision problem. This may be due to some other problem or is simply a vision problem So visit your eye specialist to have check for the vision. These can manage your respiratory tract infection. Take a decingestant such as loratadine in combination with pseudoephedrine to decrease throat congestion. I hope I am sucessful in solving your query if you have more you can ask me Take care Regards Dr. Azhar Sattar"
},
{
"id": 212780,
"tgt": "Extreme Dizziness, taking medication for stress and depression. Cause for dizziness?",
"src": "Patient: I experience extreme dizziness at the same time every day, it feels like my head is spinning, similar to drunken dizziness but I do not drink at all. It gets so bad that I have to lie down, sometimes for hours and it makes me feel very sick. I take zopiclone and olanzapine for post traumatic stress disorder as well as fluoxetine for depression which I have been taking for years but the dizziness is new. Doctor: Hello, Thanks for choosing health care magic for posting your query. Dizziness can be a psychological symptom, but then you have to rule out the physical reasons for this dizziness. You need to see an ENT specialist who can examine you for vertigo. You may be advised a few tests that will confirm the reasons about the vertigo. Hope I have answered your query. I will be more than happy to help you if you need any further clarifications. In case if you want to discuss with me directly you can use the link: http://www.healthcaremagic.com/Funnel?page=askDoctorDirectly&docId=64634 Wish you a good health!"
},
{
"id": 83139,
"tgt": "I have been having many ailing symptoms that impact the daily living, kindly help me",
"src": "Patient: I have been having many ailing symptoms that impact the daily living of an average 21 year old woman. In November, I decided that I needed to update my prescription on my glasses as that was one of my delimas. The optometrist was very shocked and we took photos and he showed me that my optic disk was swollen beyond an average persons. The light he shined into my eye, when he pulled it away I could not see out of for around 4 minutes. I started journaling my daily symptoms and realized that there were a few things that happened often like numbness fatigue and runny nose along with short breaths. He sent me to a neurologist and since I have been going through painful tests and so many more side affects. He believes its MS and is treating me for Papilledema (which I am not obese) now also...he really is working so fast I dont have a clue as to what is going on. I have understood that lupus and some other things were eliminated but he found three somethings in my spinal fluid that are attacking but are dormant...or something. We are considering steroids as our next step but I have a bladder infection he wants to clear up first so it is not affected. In so much pain and anxiety with it all. What are your thoughts. Thank you. Doctor: Now this looks a complicated case and i suppose you should consult a Neurologist or a Neuropthalmologist for this case. Yes steroids would be helpful. What do you mean by MS? How is papilledema and obesity related."
},
{
"id": 8012,
"tgt": "I have big pimples appearing on my thighs, how to get rid of it ?",
"src": "Patient: hey i have big pimples appearing on my thighs ..i pop em and one will go away but than not too long after another one comes up?am scared what shud i do?i havnt gone to a clinic or something because i donot have insurance. Doctor: hi you have follicutis or boil not pimple. apply topical antibiotic cream three times a day. you can take antibiotic tab azithromycin 500 mg once a day"
},
{
"id": 156536,
"tgt": "What causes bump outside areola with family history of breast cancer?",
"src": "Patient: I have a question. I have never got a mammogram before but I have found a pea sized bump just right outside of my areola but it is not painful to the touch but has gotten just slightly bigger than this morning? and I am just concerned because breast cancer runs in my family. Any suggestions what this might be? Doctor: Any breast lump should be promptly investigated. A family history of breast cancer puts you at high risk as well.I would suggest that you see your doctor for a clinical breast examination, a mammogram and a fine needle aspiration for cytology (FNAC) of the breast lump.Please discuss it with your doctor."
},
{
"id": 45725,
"tgt": "Suggest treatment for Good Pasture's Syndrome",
"src": "Patient: My husband has been diagnosed with Good Pasture's Syndrome and has lost all of his kidney function. I was wondering if it is alright for him to drink Gatorade? He gets tired of drinking water and sometimes drinks too much pop such as 7UP, Sprite and Mountain Dew....he is on a 1500 mL limit of fluids. Doctor: Hi, Better to avoid carbonated drinks and artificial sweeteners as it can worsen his kidneys. Consult a nephrologist and adjust his fluid intake to prevent further renal damage. Hope I have answered your query. Let me know if I can assist you further."
},
{
"id": 21533,
"tgt": "Why the heart is palpitating fast?",
"src": "Patient: I can feel my heart pounding against my chest really loudly like its palpitating and I havnt been drinking any coffee, I've just recently stopped actually. Ive been to the doctor and she said its normal and will go away because you'll stop thinking about it, but I cant stop thinking about it because its so loud and I KNOW it isnt my normal heartbeat, it feels different even tho the doc says its fine.. its really annoying! I just want it to go back to before.. can you please help?! Doctor: Hello,What is your pulse rate? It is better if you get an ECG done. You can take a benzodiazepine(alprazolam or clonazepam) in the meantime to reduce the anxiety which itself may cause palpitations.Hope I have answered your query. Let me know if I can assist you further.Regards,Dr. Imran Shaikh"
},
{
"id": 188380,
"tgt": "Wisdom teeth removed, little soreness, neck swollen, it hurts to touch. treatment?",
"src": "Patient: I got all four wisdom teeth removed a week ago, and there weren't any complications with the teeth. My face feels fine with just a little soreness where the teeth were pulled;however, the side of my neck feels swollen and im not sure but it really hurts to touch and to swallow. Should i go to my dentist and get checked out or what could it be?? Doctor: Hi,The concern would be that there may be an infection developing. Other signs would be a fever, warmth over the affected area, a stiff neck. Your difficulty in swallowing may indicate swelling in the back of your throat and around the area of the removed teeth which may be secondary to infection. I suggest you take paracetamol and ibuprofen. Keep well hydrated with fluids. If you do not improve in 24 hours, I suggest you see your dentist for further advice.Regards,Dr K A Pottinger,MBChB FRCA"
},
{
"id": 14904,
"tgt": "What's the cause and cure for itchy rashes from waist to the knees?",
"src": "Patient: Hi, My partner has all of these rashes covering from the waist down to his knees. He's quite embarrassed about this and were not sure what is causing the itchiness, when he scratches it becomes very flakey and red, mind you he is a very large boy, 140kgs and 6ft 4inches He sometimes believes its the heat because its really bad in the summer but they still remain there all the time. I suggested it could be eczema but hes very doubtful Do you have an idea of what this could be and how to treat it? Doctor: Hi,Welcome to Health care magic forum. It appears that he might be having allergy to some thing,like food, clothes, woolen,cosmetics, or soap, etc. the other causes may be the fungal infection. I advise you to consult a dermatologist for diagnosis and treatment. I usually prescribe to my patient with such symptoms levocitrizine, montelekast,grisofulvin, and mixed steroid ointment. Wishing for a quick and complete recovery. Thank you."
},
{
"id": 211480,
"tgt": "Numbness in hands, feet and lightheadedness. Is this anxiety attack or something else?",
"src": "Patient: Sudden numbness in hands, then feet, then heat surge through body that then went away. I was driving, so I pulled over and got out the car. I was light headed and felt like I was going to faint. Had light pressure, like a hand pushing on the upper part of my chest. Felt really weird. My husband picked me up and took me to the doc. He said it was an anxiety attack and to come back tomorrow if it persisted. It's been several hours and I still feel the same. I looked up anxiety attacks and don't have the impending doom symptom they refer to. Do you have any idea why this is happening? Was afraid I was having a heart attack, but didn't have any chest pain. I'm 48 years old and have a family history of heart disease. My mother died at age 57 from heart failure. I also had blockage in my right iliac artery and had a kissing stent put in one year ago as of last week.Very disappointed that the fee was not mentioned up front. I went through all this for nothing. Thanks for wasting my time. Doctor: HiYou should check you blood pressure for some days.And you need to do a lipid profile also.Have to rule out lipid disorder and hypertension.If you have lipid disorder you should also rule out hypothyroidism.Better have a physician consultation and rule out all.Hope this may helpyou.thanks"
},
{
"id": 58506,
"tgt": "HIV positive, severe stomach cramps, bright yellow urine, itchy body. Liver failure?",
"src": "Patient: Hello i am worried cannot phone anyone. I have HIV and have suffered from excruitating stomach cramps on and off for a year. yesterday the pain went on fo 4 hours until i was vuolently sick. today my urine is bright yellow and my body is itching. there is no bad smell no rash and i feel ok but i am really scared that my liver is failing or something Doctor: Hi, it appears to be the gastritis, may be due to the hepatitis, or due to the peptic ulcer. Don't worry if you are getting treatment with anti viral drugs, you need not worry about the H.I.V. you can have other treatments as usual. So i advise you to consult a gastro enterologist for diagnosis and treatment. Thank you."
},
{
"id": 2584,
"tgt": "Will vitiligo causing small spot on back and being on Pigminto affect future pregnancy?",
"src": "Patient: Hello Doctor,I am 24 years old married female. My husband and me are planning for baby. we are running in 5th year of our marriage and just completed 4 years this last december. But I had vitilogo and small spot left in my back. Doctor has advised to take Pigminto for 1 month. So I want to know will it effect the foetus later or not. kindly waiting for your reply Doctor: Hi,Any drug during formation of foetus can affect foetus except some drugs. So better to avoid any drug when you are conceiving and vitiligo is untreatable so accept the fact and avoid drugs.Hope I have answered your query. Let me know if I can assist you further. Regards,Dr. Sujata Kute"
},
{
"id": 87752,
"tgt": "Suggest remedy for pain in left thigh and abdomen",
"src": "Patient: I donated my left kidney and am now experiencing supersensitivity in the nerves in my left thigh as wel as part of my abdomen...it hurts to touch it, like needles and it sometimes has a burning sensation. It even hurts when it jiggles when I walk. Will this go away? It has been two weeks since the surgery. Doctor: Hi.Thanks for your query and an elucidate history.This is obviously a nerve entrapment or a slipped disc due to typical positioning of the patient (you) at the time of surgery or there would have been a pre-existing prolapsed intervertebral disc which got enhanced now. Or can be due to re-activation of some viral infection of the nerve root. I would advise you the following::: MRI of the whole spine to find the disc prolapse or neurological affection of the spinal cord .Review by a Neurologist to have a detailed clinical examination so that the diagnosis and treatment can be done properly."
},
{
"id": 10973,
"tgt": "Suggest permanent solution for hair fall",
"src": "Patient: my age is 30. weight is 60. height is 5.4.i have a hair fall. its to much badness n falling n also damage n dry. also short. i did alot treatment but always improve for the temporary after same problem come. also expend lot of money but never come quality n effective result. please do help me. i want beautiful, same like before 6 n 7 before. thank you Doctor: Hi... You seem to have telogen effluvium ..... There may be some cause....like anaemia,vitamin and nutritional deficiencies,thyroid dysfunction, stress, anxiety,worries,travelling,any other internal disease,certain drugs.... ..etc. Poor hygiene and poor nutrition may also be responsible for telogen effluvium. You should consult dermatologist for the firm diagnosis. He might advise neccessary investigations to find out the causes.I would like to advise you to take biotin 10 tab daily for few months and apply mild steroid lotion on scalp at night daily.. If you have dandruff use anti dandruff shampoo containing ketoconazole. For hair oiling and shampoo,herbal preparation may be used.Have patience and take long term treatment..for the good result.I hope this would help you.Thanks..Dr. Ilyas Patel MD"
},
{
"id": 47864,
"tgt": "What to do for the pain happening at the time of urinating?",
"src": "Patient: I went to the md on Monday for pain and blood in my urine..she said my rbc to wbc was 3 to 1 and there was an excessive amount of blood in my urine..which was obvious when i urinated ..I am still in pain and they have not recieved my cultur back..i am on antibiotics..do i need to go somewhere else or wait for culture? Doctor: Hi,It will be advisable to wait for culture reports.Meanwhile,you should drink more water daily and can take Tab Oxyspas 2.5mg once daily for pain."
},
{
"id": 115825,
"tgt": "Is low dose of Synthroid, suggested for autoimmune hemolytic anemia?",
"src": "Patient: I have been diagnosed with autoimmune hemolytic anemia and when I stopped taking Ginko Biloba it seemed to be getting better. But then my heart was pounding and it looked like my dose of synthroid was too high, so my doctor told me to quit. Since then the hemolytic anemia has been getting worse again. Could it be that I should start the synthroid again but at a lower dosage? Doctor: Hi, dearyou have autoimmune hemolytic anemia. So your immune system is working against your own rbcs and causes destruction of rbc. Steroid help to suppress immune system and prevent the destruction of rbc.You should start steroid according to prescribed dosage. Consult your doctor and start treatment accordingly. Hope I have answered your question, if you have doubt then I will be happy to answer. Thanks for using health care magic. Wish you a very good health."
},
{
"id": 140901,
"tgt": "Does pituitary gland with white matter disease in an MRI report require surgical intervention?",
"src": "Patient: DO YOU NEED SURGERY WHEN YOU HAVE PITUITARY GLAND WITH WHITE MATTER DISEASE T1 CORONAL AND ANXIAL T2 I AM SHOWING ALL THE SYMPTOM FROM THESE TWO MASS WAITING TO FIND A DOCTOR TO LET ME KNOW WHAT IS GOING ON HAD A MRI FRIDAY AND WHAT KIND OF DOCTOR DO NEED TO SEE? thanks ill just wait tell the july 13 to find out go to more than one doctor I on ssi I don t have to pay to get answer be help full you make good money working every day on the computer should be that you want to help people as a second person for some good a vice God Bless You with the gift gave you to make more money you really need because of your spending to much to make more on the computer HELP HELP LINE GOOD NIGHT Doctor: hi mam. first you need to look at symptoms. if symptoms of vision problems and any others that needs attention from a neurosurgeon. thank u"
},
{
"id": 155858,
"tgt": "Can a 3.4 cm tumor only lead to a cancer?",
"src": "Patient: My husband is being investigated for pancreatic cancer. He has a 3.4 cm tumor an the tail and the PET scan ruled out a PNET. He is going to have a EUS next week as the Dr has given us some hope that this maybe now benign. The interesting thing was an enormous amount of blood tests show all to be in normal range or NAD. I m not a fool and I know that they are doing a biopsy to determine if this is a malignant tumor but I just want to know would we expect his bloods to be more abnormal if it was cancer. He s 55 and was extremely healthy until 8 was ago when he developed pancreatitis with gallstones ruled out and he only drinks socially. I should add there s a 1.8cm lesion on his liver but our pancreatic spec is saying that on the PET scan they are possibly unrelated. I m an RN and my husband is now telling everyone he doesn t have cancer. I wish I could share his optimism but he s now lost 12 kg in 8 weeks. Would appreciate your comments re normal bloods!!!! Thanks a wife of 35 yrs whom still love my husband dearly!!! Jilleen Doctor: Hi and welcome to HCM thanks for the query. it is not easy to determine is this cancer or some benign lesion but size of it isn't promising. it is always good to have hope but cancer in tail of pancreas commonly dont show any lab abnormality including tumor markers and pancreatic enzymes so this should not confuse you. ct feature may show signs of cancer but of course it must be confirmed with more specific test. weight loss isn't good sign too so I wouldn't be very optimistic but there is still reason to hope for cure even if this is cancer.. Wish you good health. Regards"
},
{
"id": 179058,
"tgt": "Suggest remedies for cold in a baby",
"src": "Patient: Hello, My 10 Months old son is suffering from cold from past 2 days. I am giving him steam in the closed room for 10 mins ( twice a day) and the medicines Clamp( 3 ml), combiflame( 2.5 ml)( for 2days); Nozuka(3.5ml) twice a day. He has vomtings and had given Ondam(3.5 ml). I am giving him nossil drops 4 times a day. Please let me know if I need to take care of anything more to make him comfort. Thanks, Sudha Doctor: HelloThank You for contacting HCM.It takes around 3-4 days to get improvement after start of medication. I would suggest you following things:> Give medicine regularly at prescribed time.> make sure that room temperature is appropriate for him. If its cold outside then make sure you keep him warm by wrapping in warm clothes.> Give steam regularly as it gives soothing effect and helps in curing early.> Keep him away from dust.Report to hospital if:> Condition remain same after 5-6 days> Any unusual symptom appears> Condition deteriorates.> There is difficulty in breathing.Hope this answers your question. If you have additional questions or follow up questions then please do not hesitate in writing to us. Wishing you good health."
},
{
"id": 151772,
"tgt": "How does the brain work ?",
"src": "Patient: How does the brain work? I know all thought is based on association, which is formed in a grid of neurons and synapses - but when I (my ego) actively query myself for information, how does the brain transmit the query and how does it collect the memory? Do we know this yet? Please, use all the terminology you have. Doctor: Hi Lux. Thanks for your query and sing Health Care Magic. You question is very simple, however, the answer is very complex. Most of the information about how memory work and how information is retrieved is described in theories as its difficulty to give a definite proof. We definitely have considerable understanding about the physiology of brain, but, its far from complete. It will be impossible for me go into the details here. You can follow the links below for further information. http://www.unmc.edu/physiology/Mann/mann19.html http://www.ncbi.nlm.nih.gov/pubmed/9496622 http://www.medscape.com/viewarticle/469464 Thanks"
},
{
"id": 21902,
"tgt": "What is the average heart valve refraction percentage?",
"src": "Patient: I am 59 and was hospitalized for Congestive Heart Failure. After several tests were completed, I was told that I needed a new heart. My heart refraction was 16%. With the medications that I am taking, my heart refraction is between 50 and 55%. Is this good and what is the average heart valve refraction percentage? Doctor: Hi ThereFirst of all i want to congratulate you for such a wonderful recovery. From the advise of needing a new heart and an Ejection fraction of 16% coming back to a ejection fraction of 50-55% which is near normal range is just wonderful. Yes is is very good and normal range of EF varies between 55 to 80%.You just need to follow all the medical advise strictly as these type of recoveries are very rare.Good Luck"
},
{
"id": 208919,
"tgt": "What causes drastic change in a ones personality?",
"src": "Patient: hi i have a 25 year old daughter who is very sick. in the last 6 months she has become the bich from hell. her personally has completly changed. she is abusive both verbally and physically is doing things out of her character. we had to place her son with family for his safety. why would someone change dramatically all of a sudden. it s like she just snapped Doctor: DearWe understand your concernsI went through your details. I suggest you not to worry much. From the description, I learn that she has personality problems and mental disability. The sudden change may be attributable to the present circumstances or past happenings. Any how the scenario is deteriorating. Better to consult a psychiatrist for expert opinion.If you require more of my help in this aspect, Please post a direct question to me in this website. Make sure that you include every minute details possible. I shall prescribe some psychotherapy techniques which should help you cure your condition further.Hope this answers your query. Available for further clarifications.Good luck."
},
{
"id": 6908,
"tgt": "Does diabetes in my husband cause a reduction in sperm count and will it affect my conception ?",
"src": "Patient: My husband is diabetic now v r planning for a babyBut i heard that as he is diabetic his sperm count will be less n this will affect my preganancy.Is that true?plz reply Doctor: hello welcome to healthcare magic yes its true men with diabetes have low sperm count. Uncontrolled blood sugar level and fluctuating blood sugar levels lead to low sperm count."
},
{
"id": 54948,
"tgt": "What causes appetite loss and vomiting after gallbladder removal?",
"src": "Patient: Recently had my galbladder removed (about 2 months ago). yesterday had no appetite, ate an apple and peanut butter when i got home. Hours later I vomited it out, Immediately after I had watery yellow diarrhea. Haven't slept all night because i needed to use the restroom. Going every .5- 3 hours. I am starting to get concerned. Doctor: hi.noted history of gastrointestinal upset after cholecystectomy. it is best if you do a follow-up consult with your surgeon, for physical examination and clinical evaluation. your condition may or may. not be related to your gallbladder surgery. diagnostics (such as ultrsound, xray, electrolytes, etc.) and management will be directed accordingly. increase oral fluid intake is also recommended. low fat diet is advised.hope this helps.good day!!~dr.kaye"
},
{
"id": 121799,
"tgt": "What could be the chronic pain in the knee?",
"src": "Patient: I worked as a driver helper for UPS and basically I was jumping in and out of the truck for about a month and doing a lot of fast walking . I do remember my ankles getting a little sore, but my left ankle has been chronically sore for about 3 weeks. I can put some weight on it, but it is difficult to maintain a normal gait because of the pain. What could be the likely cause? Doctor: Hello,Your symptoms could be related to arthritis or local inflammation , due to repeated trauma. For this reason, I recommend taking calcium supplements and ibuprofen or Naproxen for the pain. Having some rest is necessary too.Hope I have answered your query. Let me know if I can assist you further. Regards, Dr. Ilir Sharka, Cardiologist"
},
{
"id": 221512,
"tgt": "What are the chances of pregnancy after taking ipills?",
"src": "Patient: I have been on birth control for about 2 years. I think I have missed a total of 3 pills over the course of those 2 years. Last Wednesday and Saturday, I had sex and he came inside me both times (for the first times). I went to take my pill tonight and I noticed I forgot to take my pill on Sunday, so I took it along with my pill tonight. I have almost 2 weeks until my period is expected. Would you be able to tell me my chances of being pregnant? Doctor: HiDr. Purushottam welcomes you to HCM virtual clinic!Thanks for consulting at my virtual clinic. I have carefully gone through your case, and I think I have understood your concern. I will try to address your medical concerns and would suggest you the best of the available treatment options.1] Please do not panic.2] It appears from your history that you have missed the pills during days 10 to 20 of the cycle- which are most fertile days3] Tab PLAN B or i pill can be taken within 24 hours of unprotected sex activity, or at least within 72 hours to be effective.4] Aslo please continue to take rest of the birth control pills as such.5] Wait for onset of periods, if you miss the one, then get urine test done.Even if it is positive, you can opt for pills to terminate unwanted pregnancy.I hope my answer helps you.Thanks.Wish you great health."
},
{
"id": 120854,
"tgt": "Suggest remedy for inflamed rib cage",
"src": "Patient: Have pain on my righ rib cage the docto told me it was inflammation that was five years ago and it is still there and the pain is getting worse. I told that it was not form my back pain and was told it was not shingle. It feel hot to the touch at times and sometime it move. How do you get rid of inflammation. Doctor: Hello,The symptoms can be related to costochondritis which is an inflammation of the cartilage of the rib cage. I suggest using anti inflammatory medications such as Acetaminophen to relieve the inflammation. I also suggest using cold compresses for local application.Hope I have answered your question. Let me know if I can assist you further. Regards, Dr. Dorina Gurabardhi, General & Family Physician"
},
{
"id": 28327,
"tgt": "Suggest treatment for high blood pressure and chest pain",
"src": "Patient: I m a 19 year old male, 5 feet six inches in height. Over the course of a month I have lost approximately 30 pounds, now weighing 142 pounds. I am physically active. I have been having chest pains as of late and have been keeping track jlong9569of my blood pressure, which had been high. I just checked it and it was 174/94 with a heart rate of 104bpm, taken after resting for an hour. I have a raging head ache and my chest is uncomfortable to say the least. Doctor: Dear Sir,I would advise you to repeat the measurements for 3 times with 1 minutes intervals, if the pressure is still high I would advise you to refer to ER. As your blood pressure is quite high and it is dangerousHope I could help you Wishing you good healthRegards,"
},
{
"id": 166482,
"tgt": "How to treat ear infection in kids?",
"src": "Patient: My 5 yr old had an ear infection about 15 days ago. He was given keflex at first. The beginning of this week we took him back to the dr s and his ear infection was worse and now put on amox 400mg susp. He has been stating that his stomach is upset, head hurts in the forehead area, and has dark circles under his eyes. Is this normal with sinus issues? My pediatricion stated there is nothing to worry about and to cont to take the medication. We are just concerned. Doctor: Hello,Amox is strong enough antibiotics for ear infection. Stomach upset like loose motion is a usual side effect of it. It may be due to the same he is not getting enough rest and having dark circles or head hurting. You can use some probiotics like Darolac or Sporolac for stomach upset or use more curd in food. Finish the course of antibiotics as prescribed.Hope I have answered your query. Let me know if I can assist you further.Regards,Dr. Kalindi R Shah"
},
{
"id": 142379,
"tgt": "Suggest medication for frontotemporal dementia",
"src": "Patient: MY Father was prescibed the medication Amantadine HCL 100mg Cap.He has Fronto-Temporal Dementia. Everything I have read on this med. is for parkinsons and flu. Do I give this to my Father ? I know Medication have many different uses. But I need clarification. Thank you Doctor: Hi, your father having frontotemporal dementia can be given tab amantadine .Actually Parkinson disease is one cause of dementia so you are requested to consult neurologist and he may require other drugs for PD .Thanks"
},
{
"id": 905,
"tgt": "Should i go for a laparoscopic surgery in case of infertility?",
"src": "Patient: Hello Doctor,I am a 28 year old woman (married for two years, husband with healthy sperm count), with PCOD grade 2. I am healthy otherwise, at 5 ft 5 inches and 61 kg and have no other health issues. I have done about 8 cycles on Letroz and Fertomid with successful ovulation. My gynec is advising me to go for laparoscopic surgery to diagnose other problems. As this is a surgery, though minimally invasive, I have my doubts and wish to have second opinion regarding this. I would like to know if there is any other way of countering my problem. Doctor: Hi, I think you can go for a laparoscopy to rule out any other cause of infertility. Also, you can know about your tubes status. Ovarian drilling can also be done If required. After that you can ovulate naturally without medicines and can try naturally also. Hope I have answered your question. Regards Dr khushboo"
},
{
"id": 150328,
"tgt": "Have head injury. Titanium plates planted. For flap rotation plates were removed. Gap in skull. Fill up by itself?",
"src": "Patient: hi doctor , my son is 10 yrs old . one year back he fell in a 70 feet well .. he sustained head injury ...he went thru a neuro surgery where titanium plates were planted in his head to hold the broken bone pieces . un fortunately the skin on the top gave way . he had to be put thru another surgery in which a flap rotation was done, also the plates implantd earlier were removed,... nw he has a gap in his skull measuring 6cm by 4 cm. my question to u is .....do such gaps fill up on his own....?do we need to get it covered up...?thnk you Chitra Doctor: Hi,Thank you for posting your query.The gaps in the skull may reduce over time, however, they may not get filled up. Generally, there is no problem. However, it can be covered by a surgery called as cranioplasty. You can discuss more about this with his neurosurgeon.Please get back if you require any additional information.Best wishes,Dr Sudhir Kumar MD (Internal Medicine), DM (Neurology)Senior Consultant NeurologistApollo Hospitals, Hyderabad,My personal URL on this website: http://bit.ly/Dr-Sudhir-kumar My email: drsudhirkumar@yahoo.com"
},
{
"id": 138055,
"tgt": "What causes chills, slight nausea, redness and swelling after spraining ankle?",
"src": "Patient: Sprained my ankle at work and now I have chills on and off for the past 24 hours slight nausea swelling and redness at the front of the foot/ankle I m not 100 percent sure I sprained It I had done a HIIT workout with ankle weights in the morning. Did not have any problems for 10 hours until I went to work and lost my balance and rolled my foot Doctor: Hello,Welcome, and thanks for sharing your concern I went through your query, and I feel, that the fever you have should not be there has you just sprained it, fever is a sign of infection, and you need to go and see your doctor so that he can get some tests done and find out the cause of your symptoms,there are chances that you may have sprained, but then collected blood at the site of sprain can get infected due to any blood born infection, so be careful and consult promptlyI hope my advice would have been useful, in decision making regarding your treatment, still if you have any clarifications or doubts feel free to contact back.Thanks."
},
{
"id": 60276,
"tgt": "Why is my mother facing the same problems even after gall bladder stones operation a year back ?",
"src": "Patient: Gall Bladder stone patient facing similar issues after a year of operation. My Mom is around 50 years old. She suffered with dysentery , vomiting and weakness. She fainted too a few days back. She got her gall bladder stone removed previous year. What do you suggest? Doctor: Welcome to Healthcare MagicShe could be having infective gastroenteritis which can be causing this problem. Is she diabetic. Get her blood sugar checked. There is less fluid in her body which is why these problems. She needs to drink WHO ORS called Electral, it is written on sachet how to use it. This will reduce the loose motion and vomiting. Give her ORS in sips. I would recommend she be taken to Hospital as soon as possible for IV fluids."
},
{
"id": 183211,
"tgt": "What causes tenderness in gums with sores on tongue?",
"src": "Patient: Hi I have a question about mouth sores. I went to my family doctor on friday but he didn't look at my sores at all because I had a fever and my tonsils were all swollen and with white spots on them. It was determined that it wasn't strep but he had me go across the hall for lab work for mono and something else I don't remember. Well he ended up sending me home with tylenol 3 and some time off work because of the fever. Well the sores in my mouth have gotten worse I think. I now have about 4-5 sores on my tongue they are mainly red and one is smaller than my pinky fingernail but whitish in color. Then I also have the roof of my mouth and my gums being very sore and tender to even eat or brush my teeth. I have just started yesterday rinising my mouth with peroxyl and that usually foams up my mouth within seconds. My roomate says my breath smells like it is nothing but infection. Any suggestions on what I should do? Doctor: Thanks for using Health Care Magic.Read your query.Mouth ulcers or sores described here (referred as canker sores) can be due to some factors it can be due to Stress or anxiety related , Nutritional deficiency , vitamin deficiency , immune disregulation .I would advice you to apply a mucopain/dentogel gel two or three times a day.Do salt water gargling for a while.Also betadine mouth wash can be of help to you.Avoid spicy food for a while.It will subside on it own gradually within a week or two.Visit an ENT surgeon and have your tonsil infection checked up.Avoid stress and have a nutritious diet .Multi-vitamin supplement can be taken.If still persistent,consult an oral surgeon.Hope this was useful.Thanks and regards."
},
{
"id": 168250,
"tgt": "What causes knee weight bearing problems with numbness and pain?",
"src": "Patient: Our 15 year old daughter has been having difficulty bearing weight on her knee as it collapses. She started having difficulty and was walking on the leg then started having intermit problems to every time she stands with weight her knee collapses. Recently she experienced numbness with severe pain following. Could this be caused by a cyst that is 9cm x 6 cm x 3 cm? Doctor: Hi.... all these symptoms or explainable if there is a cyst of the measurements that you have quoted. If this is the case, I suggest that you see an orthopaedic surgeon as soon as possible.Regards - Dr. Sumanth"
},
{
"id": 88090,
"tgt": "What causes abdominal pain and foul smell around belly button",
"src": "Patient: for the past two weeks, i have had abdominal pain around my belly button and up to my right kidney. along with the pain, i have experienced bleeding and a fowl smell coming from my belly button. other things i have noticed over the last two weeks, my stool started as yellow and has become dark green. and i am urinating more frequently. i am curious if this is a connection. i have had 3 x-rays, a CAT scan, and full blood and urine work-up but everything has come up normal. what could be causing the pain? Doctor: Hi,From history it seems that you might be having some infection in belly button or having Grannuloma giving bleeding and foul smell from belly button.Go for one antibiotic medicine course for 5 days.Clean the belly button with antiseptic lotion and dress it with antibiotic cream.If problem persisted then consult surgeon and get examined for grannuloma.Ok and take care."
},
{
"id": 73863,
"tgt": "What causes shortness of breath and heavy limbs?",
"src": "Patient: I am 45 year old woman and have shortness of breath and my limbs feel extremely heavy. I some times have pains in my arms sort of like the ones you get when you hit your \"funny bone\". I also have sevier pain on my left side of my neck all the time. Any idea as to my problem? Doctor: Hello dearWelcome to Healthcaremagic.comI have evaluated your query thoroughly .* There are different possibilities of the same as - Systemic circulatory disturbances as diabetes , thyroid hormone level alterations - cholesterol issues - low hemoglobin levels - others .* Need physical examination with evaluation via series of laboratory tests for the same .Hope this will help you .Regards dear ."
},
{
"id": 200310,
"tgt": "Suggest treatment for varicocele",
"src": "Patient: Hii. I am age 35 male. I am suffering from erectile Dysfunction and low sperm count which is leading to infertility. I got myself examined and found out i have Varicocele grade I. Please suggest me good medicine which gives good benefit and no side effects. Sunny Doctor: Hello,,If you have low sperm count then you could benefit from Varicocele surgery. Varicocele is known to cause low sperm count. So, you should go ahead with the surgery. Its not a very major surgery and has the potential to increase your sperm count. But, there are no guarantees. If you opt against surgery, then you can try medications.I usually prescribe a product from Himalya ayurvedics called Speman. I give it for three months and usually find a rise in Sperm count after three months. Choose what you want but I would advice in favor of surgery.Thank you."
},
{
"id": 124101,
"tgt": "Suggest treatment for fibromyalgia",
"src": "Patient: I m 49 had a hysterectomy last year went back in after 5 days for a bowel obstruction no operation after 5 more days in the hospital i would not heal it took 30 more days for the wound to heal now ive been told i had fibromlagia every time i sit i hurt and every time i stand my groin is numb my buttock Doctor: Hello, As mentioned in history I think it is not fibromyalgia. It will be a weakness of the para spinal muscles in the lumbar spine leading to compression on the nerve root via the disc. As you got operated for hysterectomy and later a procedure for the bowel obstruction I can feel that will be a weakness in the core and paraspinal muscles. Please describe the symptoms this to the physician so a physical examination of the core and abdominal muscle strength can be assessed and treatment will be initiated with hot water fermentation and exercise for the core and lumbar spine extensor group of muscles. In my clinical experience of 12 years, I have always observed that patients undergoing abdominal surgery complain about the symptoms of pain in the hip region and lower back. With the help of post operative management under guided physiotherapist should give good results. Hope I have answered your query. Let me know if I can assist you further. Regards, Jay Indravadan Patel, Physical Therapist or Physiotherapist"
},
{
"id": 122081,
"tgt": "Can deca help to treat severe chronic synovitis?",
"src": "Patient: would deca be any good in dealing with severe chronic synovitis am currently on alot of pain killers which mask the problem which does not fix the problem plus having to take other drugs to deal with side effects from the pain killers i am on.Have been told deca causes fluid retention plus synovial fluid increase dealing with joint pain if it draws water into the joint would that be like having extra joint lining within the joints which also means i would not have to have a joint replacement? Doctor: Hello, By deca one would assume dexamethasone, a strong steroid. The effects can be quite localized in space and time for the side effects and still work. So, local injection is the usual way it is administered and overall side effects are generally small. Cannot say in your particular case, but generally. BUT if given as high dose oral, even so, the effects generally last a bit after being given but the side effects go away. Hope I have answered your query. Let me know if I can assist you further. Take care Regards, Dr Matt Wachsman, Addiction Medicine Specialist"
},
{
"id": 205236,
"tgt": "What causes persistent OCD?",
"src": "Patient: i have ocd for last 7 years and having habit of rechecking and confirming certain things again and again by speaking loudly too myself. and i have taken many ellopathy medicine but of no avail. please let me confirm is there any medicine in ayurvedic by which i can get fully cured. i am also now taking patanjali medha wati, ashwagandhirist and sarshvasrishta and medha kawaft but it does not resulted Doctor: hi, i will suggest u that OCD will treat with allopathy medication, ayurvedic i didn't know. with medication u can controlled symptoms up to 70% other will improve with psychotherapy."
},
{
"id": 51066,
"tgt": "Urinary tract infection, kidney stones. Due to excessive protein intake?",
"src": "Patient: Can too much protein cause a bladder or kidney stones? I am a personal trainer and manager of a gym. I eat alot of protein because it;s easy to grab a protein shake and I always have grilled chicken cut up to snack on. I am also a runner and have developed a uti and have symptoms of a kidney stone. How much is too much protein for a 45 year old woman? I also lift . I weigh 115 and am 5 1. Doctor: HI WELCOME TO hcm. Your questions are very relevant. The protein requirement a day is 1 to 2 gram per kg per day. So you can calculate. A dietician can advise on the protein balance for you. Diet high in PURINES like red meat can cause uric acid stones, but not proteins intake as such. But high protein intake associated with alcohol intake can cause stones in the PANCREAS. Hope this helps, Regards DR GS"
},
{
"id": 204730,
"tgt": "What causes repetitive talking?",
"src": "Patient: I keep repeating myself when I talk. Over and over and over and over. I went to an emergency room earler today repeating myself over and over and talking fast. I was given atavan and that calmed me down and now I am back at it again.. over and over, like a parrot, talking fast. it makes no sense. took an atavan earlier and it seems to be calming me down some. What is wrong with me? Doctor: in my opinion just check thyroid functionLooks like mood disorder how is your sleep how is your appetite it would be useful to start on small dose drugs like olanzapine along with ativan"
},
{
"id": 222095,
"tgt": "What are the symptoms of potential pregnancy?",
"src": "Patient: Hello, I m wondering if I could be pregnant. I m on the birth control pill, and have never missed it or had unprotected sex. However I am having a lot of pregnancy symptoms, and my body feels different recently. I had my last period, but it was very different to normal, it would normally last 5-6 days and I would bleed heavily every day, but this time it only lasted 3 days with very little blood. My symtoms are increased urination, tiredness, nausea, breast tenderness, more prominent veins on breasts and light cramps in lower stomach and back. Please help! Doctor: HI, I understand your concern. It's common with some oral contraceptive pills .. to cause side effects having symptoms like yours ( specially progestin only pills- Mini pills )/ If you are very regular in taking your contraceptive pills & did not have unprotected sex .. I do not see any chance of pregnancy in your case. Please do not worry. Continue with your pills regularly, it will adjust automatically. TO avoid any stress/ anxiety.. you can repeat pregnancy test after 15 days.. when you find that too -ve, you would be convinced. Thanks."
},
{
"id": 3083,
"tgt": "Am i pregnant or not as there is no period?",
"src": "Patient: hai doctor, I have PCOD problem. MY LAST PERIOD JUNE 10. consulting doctor said my ovulation start on june 23 rd.after 5 days i start taking duphaston 10mg tab for 10 days. still menstruation not come. iam pregnant or not doctor? pls answer to my questions..thank you doctor. Doctor: Hi there,Welcome to HCM,You will not get your period as long as are taking Duphaston. Stop the Duphaston on 10 July, after doing a urine pregnancy test.If you are pregnant the urine pregnancy test done on 10 July will be positive. In that case see a gynecologist and continue Duphaston.Hope this clarifies your question.Regards."
},
{
"id": 139710,
"tgt": "What causes severe dizziness with sugar and BP normal?",
"src": "Patient: Dear sir, I have started getting severe dizzy spells since yesterday morning, I am 37 age with no history of Bp or sugar, in fact currently my Bp 121/79 sugar is 127 random, I went to doctor and he has given me histamine hydrochloride 16 mg 3 times a day, In fact i was on a very strict diet since over 2 months where i was just eating oats , cucumber, carrots, orange fruit, apples, raw almonds, grilled chiken, boiled chicken, and pearl millet bread , i have lost 2 waistline since 2 months and over 4 kg weight.Please advice what should i do with my diet now continue or change why this dizzy thing causing to me? Doctor: Hello,I will suggest you to do a weight loss of not more than 0.5kg per week (so your weight loss is okay). Betahistidine hydro chloride can cause hypo-tension, so reduce dose to 16 mg twice a day. As you are already on a diet, your problem will be resolved for orthostatic hypo tension. Keep on taking beta histidine for 5 to 7 days and symptoms should improve. If not, you again need to visit your doctor to start working for these dizzy spells. These dizzy spells are usually some inflammation of inner ear which is relived by taking this batehistidine.Hope I have answered your question. Let me know if I can assist you further. Regards, Dr. Muhammad Faisal Bacha, Internal Medicine Specialist"
},
{
"id": 31310,
"tgt": "What causes red itchy bumps on body with chills and sore throat?",
"src": "Patient: My throat started itching real bad. Two days later i had chills and my throat got real sore and swollen. The chills continued on and off for 9 days same with the sore throat, developed nausea and light dry cough. But half way thru the 9 days i started breaking out bad on arms legs and a few on back and stomach, in theses lil red super itchy bumps. Some looking like little blisters. There eveywhere. Of course i itched them and then they would scab. What is this rash. I went to the er and strep negitive but was told it may be an allergic reaction . Its not that i know its realted to my throat. Which also toward the end devloped white patches that can peel off but leaves the throat red and burning underneath. What is all of this, mainly the rash? Doctor: Thank you for your query.It seems that may have contracted Chicken pox (varicella). It is a highly contagious viral infection that is self limiting in most individuals.Symptomatic relief for the itching, fever and flu like symptoms may be required.Itching will cause scarring, so avoid itching.Avoid hot foods/drink/bath. this will aggravate the itching. have foods & drinks that cool your body. Wear loose fitting clothes made of cotton or other natural breathable material.Rest and fluids / I would recommend seeing your doctor if your symptoms worsen or if you have itched the blisters and those have got infected.Also note that you are most contagious in the first week ( before the rashes appear ) and the last week ( when the blisters scab over). The duration is about 3 weeks. Do not pick at the scabs. fluids in the blisters have the virus in them and can be transmitted.Hope this helps and that you feel better soon.Wish you good health. Please get back to us if you need further clarification"
},
{
"id": 144514,
"tgt": "Suggest treatment for asperger s syndrome",
"src": "Patient: Are any of you psychologists? I know a man with Asbergers, who has had moderate success in life yet is seeking ways to better understand his disorder and become better. What typical treatments are offered for this condition? My friend was never fully diagnosed with it. His parents were told that his case was mild, yet he still suffered pain because of it. He s smart yet struggles with social cues and understanding simple directions. He has good wit and insight. Yet, he is seen as stupid by not understanding some things. Any advice to help my friend? Thanks! Doctor: DearWelcome to HCMWe understand your concernsI went through your details. Aspergers syndrome involve delays in the development of many basic skills, most notably the ability to socialize with others, to communicate, and to use imagination. Usual symptoms are: Problems with social skillsEccentric or repetitive behaviorsUnusual preoccupations or ritualsCommunication difficultiesLimited range of interestsCoordination problemsMany people with Asperger's syndrome are exceptionally talented or skilled in a particular area, such as music or math.Right now, there is no cure for Asperger's syndrome, but therapy may improve functioning and reduce undesirable behaviors. The usual therapies given are:Special educationBehavior modificationSpeech, physical, or occupational therapySocial skill therapies drugs may be used to treat specific symptoms such as anxiety, depression, hyperactivity, and obsessive-compulsive behavior.I request you to consult a psychologist near you to get your friend properly assessed for Asperger and then for therapies.If you require more of my help in this aspect, please use this URL. http://goo.gl/aYW2pR. Make sure that you include every minute detail possible. Hope this answers your query. Further clarifications are welcome.Good luck. Take care."
},
{
"id": 80704,
"tgt": "What causes persistant cough and tingling sensation in throat?",
"src": "Patient: I had to be intubated when I went into heart failure three years ago and since then I have been bothered with a persistant cough from my throat, a tingling sensation that makes me cough constantly as it feels as if there is something there, in addition there are times when coughing it feels like a piece of skin or something flapping in my throat. Then there is when I eat and the smallest crumb can cause a fit of coughing, then there is the changing of my voice at times making me sound like I have a hoarse throat. Could this be related to my being intubated? In addition I take .05mg of levithroninxe for my thryoids. Doctor: Hello dear, thanks for your question on HCM. I can understand your situation and problem. Post intubation following are the possible complications that can cause similar symptoms. 1. Bronchitis2. Tracheal injury and stenosis. So get done chest x ray and PFT ( pulmonary function test ) to rule out bronchitis. Get done Bronchoscopic evaluation of trachea to rule out tracheal injury. So better to consult pulmonologist and discuss all these. Possibility of tracheal stenosis is more in your case. So get done bronchoscopy."
},
{
"id": 183947,
"tgt": "Suggest treatment for orange film on tongue and bad breath",
"src": "Patient: What can I use to treat an orange film on my tongue? I've had this for the past 4 months and nothing helps. I have used diflucan, brushing my tongue, antibiotics (Levaquin and Augmentin). I have diabetes type 2 for the past 12 years so this is not the problem. My breath smells horrible no matter how often I brush. Doctor: helloo..read through your query...i have to say that most of the diabetic patientcomplain of this bad breath..its mostly due to the ketone bodies produced in ur stomach as u have less food...so it can be reduced by few simple steps which has to be followed regularly...brush after each meal..do tongue scraping it is the hub of bad breath causing bacteria...do chlorhexidine mouth rinse..and dont keep ur mouth dry...diabetes causes dry mouth...so drink lots n lots of water...kp it wet...go to dentist and get scaling done...it can reduce bad breath to some extent....hope my answer gives u some help...have a healthy day!!!"
},
{
"id": 192132,
"tgt": "Suggest treatment for swollen foreskin after paraphimosis",
"src": "Patient: Hi there, I had surgery today for paraphimosis. The surgeon left while I was still recovering from the nitrous oxide, so I was unable to talk to her properly about what I need to do next. I would like to know if there are any dangers I should be looking out for now that I have had the operation. My foreskin is a real mess as she had to remove a section of skin to return it to its original position. I have noticed that there seems to be a lot of blood clotting right at the end of the foreskin and the foreskin still seems a little swollen. Also when I urinate, it flows out between the stitches, instead of flowing out where it normally would. I am just wondering whether these things are normal or whether I need to go back to the hospital to get them to check up on it. Also I am concerned that if I have an erection in my sleep, the paraphimosis will occur again. Is this a legitimate concern for someone who has just had this surgery? Doctor: Hi, For swollen foreskin you can use MgSo4 solution with bandage around foreskin. Apply antiseptic cream like soframycine over stitches. Take care. Hope I have answered your question. Let me know if I can assist you further. Regards, Dr. Pramod Kokare, General & Family Physician"
},
{
"id": 186128,
"tgt": "What could be the reason for having pain in my cheek lining that makes hard to open my mouth?",
"src": "Patient: I have this really painful cheek lining in the left side but sometimes it feels like it's the lower jaw, and sometimes it feels like it's my molar tooth that really hurts. And what's worst is I can't open my mouth bigger than 2 cm so it's hard to eat. What's wrong with this cheek?? Doctor: Hello, Thank you for consulting with HCM.This kind of symptoms occur in a condition when there is infection in the last molar , which causes swelling in the joint area of mouth and abscess, that is why you are unable to open mouth.Better you visit your dentist and get and an x-ray of that area which will show the area of infection.Hope it will help you."
},
{
"id": 212169,
"tgt": "Mother speaking to herself, negative thoughts and sleeplessness. Started after dad died. Advice?",
"src": "Patient: Hello doctor, This is a concern regarding my mom, whose mental health has become poor due to sort of negative thoughts after a year ago death of my dad. She is 51 years of age. I can seen her speaking to herself, hitting her head with her hands as something disturbing her brain, and lack of sleep. When asked, she is not expressing it in detail as she feels they are very bad evil thoughts and does not want to disturb us. She is restless even when lying down or sometimes at sleep. Please suggest me on what to do to relieve her from this state and make her happy. Doctor: Dear The symptoms corresponds to Post Grief Psychological Disorder. But it hadn't subsided for more than a year. Then it is a concern. Self talk, hitting head and lack of sleep - Note the frequency of these symptoms. Get to a psychiatrist with these details. Because this illness should not develop into psychoses. Wish her speedy recovery."
},
{
"id": 124464,
"tgt": "What is the red bump on toe that is sore and swollen?",
"src": "Patient: My toe (first one next to big toe) has had a small red bump for 2 wks. It has continued to swell and is very sore and swollen now. There is a small white bump there now. If I bend my toe , there seems to be several white bumps appear. I have been soaking my foot in epsom salt water. What can it be? Thanks Doctor: Hi, As what is understand with the history that this could be related to the secondary issues as you haven't mentioned any direct Injury to the great toe. Few questions to be checked before coming to any conclusion. Are your diabetic? What is your age? Did you check your uric acid levels?, etc. Until we have this basic things answered it becomes difficult to come to a conclusion as a diagnosis has to be made on a much more factors. To make it simple I would advice you to reduce the salt in take. take less of acidic drinks or fruits. Try to relax the great toe and dip in normal Luke warm water without any kind of addictive. Post which you can raise the leg up keeping it over a pillow. Please feel free to contact me with more detailed history as a diagnosis is important on factors to be considered. Hope I have answered your query. Let me know if I can assist you further. Regards, Jay Indravadan Patel, Physical Therapist or Physiotherapist"
},
{
"id": 213355,
"tgt": "Has epilepsy. On petril, mezetol, gabantin. Can they cause split personality?",
"src": "Patient: Dear Doctor, My sister is now 50 years old and she has been suffering from epilepsy since she was 8 years old. Various medications were prescribed to her. She used to get many seizures a day. She has seen many doctors both in Kenya and in India. Her current medication is PETRIL MDO-5, MEZETOL SR 400 and GABANTIn 300. There are times when she is very agressive and at times she turns into a SPLIT PERSONALITY. Does the medication prescribed cause a person to have SPLIT PERSONALITY? Doctor: First instnace, it is mendatory ot have seizure contorl, as uncontrolled seizure can cause behavioural problems which you states as Personality disorder. No medication can cause personality disorder. So in my opinion, there are lots of new anti epileptics are available in the market and than can be helpful to patient."
},
{
"id": 31083,
"tgt": "Suggest remedy for redness and boils on scalp",
"src": "Patient: I have staph on my scalp. Have had for about 15 years now. Tried most things including hibiscrub, coal tar, no perfume shampoo's and nothing has moved it. Its got better since I went to the dermotologist about 18 months ago as I was prescribed 1% dalacin T-Scalp location and Dermovate (for the itchyness). This controls it better but I still have redness and boils. Any suggestions? Doctor: Hi, Thanks for posting in HCM. I understand your concern. What problems your son is having could be due to 'seborrhoeic dermatitis' of scalp. Kindly do the following to overcome it. 1. Use alternate days shampoo containing coal tar or salicylic acid. 2. Use shampoo containing Ketoconazole twice a week. 3. Use shampoo containing Selenium and Zinc once a week. 4. Protect from direct sunlight by wearing a clean cap or using umbrella. 5. To control itching, he can take Tablet. Levocetrizine once a day. 6. Apply Clindamycin cream only on the areas you have boils everyday during bedtime.The lesions should subside gradually in a month time. Hope the information provided would be helpful. All the best."
},
{
"id": 18929,
"tgt": "Is high BP cause for worry?",
"src": "Patient: my blood pressure has been running around 130/90....and this morning right when i woke up i took it and it was 140/100....i am 43, do not drink, smoke and im not overweight. overall in good health...i have never never had high blood pressure...but the last couple weeks i have been monitoring it because i had a headache in the morning and it seems elevated in the morning and then tapers out throughout the day.. i made an appt with the cardiologist but they cannot see me for another week. will i be ok. Doctor: Hello,After going through your medical detail, I understand your concern for your health, and I would like to tell you that blood pressure is an ever-changing phenomenon and can\u2019t remain constant every hour as it depends on our actions. Emotional state, stress levels, and other physiological phenomena. During a headache, your blood pressure may rise, but for that, you need to take pain relieving meds instead of anti-hypertensive drugs. Continue with your healthy lifestyle and keep your blood pressure under 140/90 mmHg.Hope I have answered your query. Let me know if I can assist you further.Regards, Dr. Bhanu Partap"
},
{
"id": 86442,
"tgt": "What causes stomach aches after usage of implanon?",
"src": "Patient: hi ive been on the implanon jus going onto 6months now..but ova the past week ive been havin these constant tummy aches..feelin a bit sick and falling asleep at weird times during the day.i took a home pregnancy test on thursday but came up negative.i m not sure as to weather i am or not as ive been having irregular periods.if i was would it pick it up and what are the chances of being pregnant. Doctor: Hi.Thanks for your query. Read and understood your history of using implanon for the last 6 months and you have started pain in the abdomen, feeling sick, sleepy and weird. The pan in the abdomen may not be related to the implanon at all. Although you have not given any other history related to the nausea, vomiting, bloating or any other symptoms, it is possible hat you may be suffering from IBS. Get other investigations like CT scan of the abdomen during the pains, tests of blood, urine and stool, colonoscopy to rule out any other problems."
},
{
"id": 111631,
"tgt": "Suggest treatment for severe back pain that not even allows me to sleep",
"src": "Patient: My name is Arnold, I m 66 years old.... I can t go to bed because I can t lay down, my lower back hurts as soon as it its the mattress, I get excruciating pain before the spine curve flattens against the mattress,then I have problem turning and lifting up.... the same thing happens if I try to lay on my chest. This has been going on for at least 3 months. I laid a hardwood floor on 2d floor of a house, I fell to my face during the renovation but it did not affect me untill I completed the job. Then the low back pain began. Thanks for this opportunity Arnold Doctor: Hi, Thanks for using HCm.In my opinion you should stop worrying. For immediate relief have some pain killer along with a muscle relaxant and have proper bed rest for 2 weeks. Do not lie down on soft mattress use hard one instead, do not sit for longer durations and do not travel long distance for 2 weeks .When getting up from bed always take a lateral position first then hang your legs down and then get up. When working , maintain a good posture of your back . Do not strain your self or over exert. . U can also apply some good anti inflamatory pain killer gel on your back.If the pain increases do some hot fermentation locally and make a routine of doing some back strengthening exercise daily . Start wil 10 mins twice a day to 20 mins twice a day and walk for 30-40 mins a day alteast. I am sure you will be fine in 2 weeks"
},
{
"id": 18597,
"tgt": "What causes high pulse rate and mild chest pain?",
"src": "Patient: I have high pulse during the day 95-105 but normal in the morning. and mild chest pain in ribs and sternum. in the last two days I stayed the night in hospital, Had numerous blood tests, Stress Tests, EKGs,.. My heart seems to be fine and undamaged. I take Candesartan and metropolol. What should I do now to find out the cause of the Heart rate issue and this annoying chest pain. Cardiologist says, don t worry lose 20lbs and exercise. Im 42yr, male, non-smoker, 5 -9 204lbs. Doctor: Hello and Welcome to \u2018Ask A Doctor\u2019 service. I have reviewed your query and here is my advice. After going through your medical details I understand your concern for your health and I would like to tell you that heart rate is ever changing phenomena depending on your activity level, emotional stress and physiological changes. People with obesity can have high heart rate, high Blood pressure issues but as all other cardiac test came out normal then you need not worry about Cardiac Health. Cause of chest pain can be indigestion or sever gastric reflux. It's recommended for you to avoid spicy and junk food and continue with the medications prescribed and loose weight. Hope I have answered your query. Let me know if I can assist you further."
},
{
"id": 207414,
"tgt": "How to treat severe depression?",
"src": "Patient: Well I wanted to know if I had insomnia. I have a really hard time sleeping at night, even if I know that I have to wake up early the next morning. On weekends I can sleep for 12 hours or more. On school days i go to sleep and 3 am and wake up at 6 am. I sleep in school and crash when I go home, I'm not usually irritable but I get that way sometimes. I'm really scared for my health to be honest. I'm 16 years old. I believe I've suffer from severe depression. I am very stressed and weirdly scared of the dark lol (I'm too old for that) but I sleep with the light off sometimes, but I am a little paranoid with it. I see weird things sometimes in the dark. Tell me if this is normal or should I talk to a doctor or therapy. Doctor: HiI understand your cocern.Severe depression can be treated with antidepressant and psychotherapy.ECT means electro convulsive therapy is the choice in severe depression with suicidal thought.Antidepressant like SSRI is useful. SSRI mild dose of escitalopram and paroxetine like drug is very usefulBut before taking this medicines you have to consult psychiatrist and evaluate your self.Insomnia can be treated with zolpidam 10 mg or bnzodiazapin like clonazepam or diazepam.But do take advise.Thank you.Get well soon.Both ECT and antidepressant will helpful to recover from depression.Choice of drug can depend upon symptoms and severity."
},
{
"id": 80087,
"tgt": "Suggest treatment for dizziness and shortness of breath",
"src": "Patient: Today I had about four hours of dizziness and have had dizziness on and off for quite a time but before it would just be for a few moments. I have been having facial swelling and have been seeing an allergist and he has not been able to figure it out. I am tired most of the time and have shortness of breath. My feet hurt a lot and have sharp pains in my legs periodically. Doctor: Thanks for your question on Health Care Magic. I can understand your concern. Facial swelling, shortness of breath, leg pain can be due to allergy. So you need PFT (Pulmonary Function Test) for the diagnosis of allergic bronchitis. Inhaled bronchodilators and inhaled corticosteroid (ICS) along with anti allergic and antihistamine drugs are needed in your case. So better to first diagnose yourself and then start appropriate treatment. Hope I have solved your query. I will be happy to help you further. Wish you good health. Thanks."
},
{
"id": 145465,
"tgt": "Is it safe to have Primibone for Parkinson s disease?",
"src": "Patient: Should Primibone be taken for parkinsons disease? The side effects seem worse than the slight tremor I have right now, which is why my MD wanted me to try it...Or i m thinking I should wait till my regular app. with my Parkinsons Specialist...Any advice would be greatly appreciated..Thank You.. Doctor: Hi,Thanks for writing in.Primidone is prescribed by some doctors to treat the tremor encountered in Parkinsons. However you are right is saying that the side effects of primidone are more difficult to manage than the tremor itself. Studies have been done concerning giving primidone in Parkinsons and it is seen that mean tremor amplitudes were not changed by primidone. Also Tremor frequency was unaltered by the medication. Many doctors say that long-acting propranolol is an useful adjuvant therapy for the tremors associated with Parkinson's disease. However medications are to be changed after asking your doctor. If you feel comfortable and tremors are slight then you can stop primidone and wait for your parkinsons specialist for a more suitable medication to treat your tremors. Please do not worry."
},
{
"id": 61494,
"tgt": "What causes vaginal bleeding during intercourse with lumps in breasts?",
"src": "Patient: I am bleeding vaginal but only when having intercourse. I was suppose to of started my period two weeks ago but have not. About four days ago I bleeding the same only during intercourse. I have a history of hemmoargic cysts on ovaries which I went in for surgery a year ago but during surgery doctor said everything was normal since than I am still in still pains in ovaries n have new symptoms of lumps in left breast which I have. A mammogram tomarow Doctor: Hi.Thanks for your query.Noted the history and understood your concerns.Lump in breast needs to have a mammogram but more important is the FNAC or excision biopsy if anything serious like cancer is suspected. The bleeding during intercourse has many reasons and not related to any ovarian problems.If a woman is bleeding during sex, the probable reasons are as follows:If this is the first intercourse , this is due to rupture of the hymen, if the bleeding stops and does not occur during further acts , nothing to worry.Another reason is that she is either menstruating or has inter-menstrual bleeding.Another reason is she has some vascular anomaly . You need to consult a Gynecologist to do an internal examination to find the reason and get operated and treated accordingly."
},
{
"id": 139536,
"tgt": "What causes numbness and tingling on center of head?",
"src": "Patient: Starting yesterday I have had a very strange sensation on the very top, center of my head. There is a spot that feels as if it s pulsing, right beneath the scalp. If I scratch it, or rub it, it tingles and feels numb. (No pain) Is this some sort of scalp/skin issue, or soemthing more serious? (I m 37 yo female, 5 6 , no recent head trauma; first migraine about a month ago; Hashimotos; recent bout of inflamation in the retina of my left eye) Doctor: Hello,It may be due to pressure point or due to migrainous head ache or neuropathy. Local site infection or dermatitis etc. Use tablet pregabalin m 75 mg once a day. If no improvement please consult your neurophysician he will examine and treat you accordingly.Take care. Hope I have answered your question. Let me know if I can assist you further. Regards, Dr. Penchila Prasad Kandikattu, Internal Medicine Specialist"
},
{
"id": 101776,
"tgt": "Suggest treatment for allergy on face",
"src": "Patient: hi, i got alergy on ,y face for past 3 months... already consulted doctors.... they gave pimple care soap,pearsol ac 2.5 gel,calomil lotion.... but nothing s working on my face..... its spreading over full of my face.... plz say some tips to cure my face alergy Doctor: hello,thanks for your query, allergy over face(utricaria) can be stopped by-find out what allergen(cream,deodrant, cosmetic products) causing allergy. take tab.montek-lc (montelukast+ citrizine) one tab at bed time.for steroid cram oral steroids can be taken with dermatalogist opinion.all the best.take care."
},
{
"id": 88142,
"tgt": "What causes abdominal pain after eating?",
"src": "Patient: my daugther is 7 years old, active, eats anything we put in front of her but she has started complaining of Stomach pains sometimes she throws up, leg pain and headaches. My husband and I are not normally sick, we live pretty heathly life style by phyiscal activity, eating heathly and we have a good marriage. She has been complaining for over 6 months. Sometime she has a fever with her pains? What should I do, I am worry! Doctor: HiPAIN ABDOMEN FOR 6 MONTHS WE SHOULD INVESTIGATE1- BLOOD FOR CBC2-ULTRASOUND ABDOMEN3-URINALYSIS4- STOOL EXAM.FOR OVA AND CYSTAFTER REPORTS YOUR DOCTOR WOULD TREAT BETTERTHANKS"
},
{
"id": 18327,
"tgt": "How to treat fluctuations in the stress test level in a heart patient?",
"src": "Patient: yes I take a stress test yearly for past 13 yrs. for my c.d.l. license put this year the treadmill test was ok but the E.K.G . part of the test was off I have 2 stents I have to go to docs. for stress ego test I need my license to survive. should I be worried Doctor: Hello and Welcome to \u2018Ask A Doctor\u2019 service. I have reviewed your query and here is my advice. After going through your medical query I understand your concern & I would like to tell you that if you have already put up stents in coronary arteries then most likely you are on safer side but be regular with the medication to prevent instant thrombosis and be careful with your diet. Yearly check up is must so that disease can be managed medically or if required any further treatment can be given on time. So kindly continue with follow ups and your medication. Hope I have answered your query. Let me know if I can assist you further."
},
{
"id": 26087,
"tgt": "What is the cause of heart palpitations and shortness of breath?",
"src": "Patient: Hello, Last week I had heart palpitations and shortness of breath and was advised that it was due to my caffeine intake which was lowering the potassium in my body and since then I have had a flushed feeling, hot flashes and tinglining in my right hand.. Why is this happening? Doctor: Hello!Welcome and thank you for asking on HCM!Regarding your concern, I would explain that your symptoms seem to be related to an electrolyte imbalance. From the other hand anxiety, triggered also by caffeine intake could explain all your symptoms. I recommend consulting with your attending physician for a careful physical examination and some tests: -resting ECG -blood lab tests (complete blood count, blood electrolytes, thyroid hormone levels, fasting glucose, kidney and liver function tests, etc.). If the tingling in your hand persists and all the above tests result normal, I would recommend performing a nerve conduction tests (electroneurography) to exclude possible pinched nerve in this hand, not related to the other symptoms. Hope to have been helpful!Greetings!Dr. Iliri"
},
{
"id": 114900,
"tgt": "Can H.pylori infection or labyrinthitis cause low platelet count?",
"src": "Patient: Hello doctor, my platelet has been progressively decreasing for the last couple of years.Jun 2012 - 157 x10^9 Aug 2013 - 160 Aug 2014 - 143 Feb 2015 - 125 Apr 2015 - 111 All other counts on the CBC has been normal except for Basophils (0.1) in the last test (in previous tests always (0.0).My GP had no idea why my platelet count is decreasing. Two months ago, I was found positive for helicobacter pylori and was treated with antibiotic therapy Nexium7 for a week. Haven t been tested again to confirm if it has been cured. At the same time I was viral labyrinthytis when I started to feel vertigo. I am going to see a haematologist soon. I just wanted to know the possible reason for my progressively low platelet count. Should I be concerned? Does the H.Pylori infection or the Labyrinthytis has any role in low count of platelets?Thank you. Doctor: I have reviewed your concern my friend. you are doing right thing by going to hematologist. however, decreasing platelet count can be due to different causes. H pylori and labyrinthitis should not ideally cause this problem, but it can be more due to a condition known as idiopathic thrombocytopenic purpura (ITP). However for this the hematologist needs to review your condition and probably do a bone marrow examination if required and rule out other cause. treatment is steroids which is generally good enough. I hope this helps. let me know if i can help you in anyother waythank you for choosing HCM. May you have a superhealthy life ahead."
},
{
"id": 3177,
"tgt": "Can pregnancy happen after having non penetrative sex?",
"src": "Patient: Hi i had a sexual experience today with my boyfriend and im not sure if i was fertile or ovulating at all. it has been at least a week after my last period. we didnt have sex but i did get sperm in my fingers and inserted it into my vagina. can i get pregnant? what are my chances Doctor: Hello dearI understand your concernYou have did sex after 7 days after the period and it is safe period of the cycle. At that day there is no ovulation so no chance of the fertilization.Again fingering does not cause pregnancy.To move the sperm it require liquid medium so just fingering does not cause pregnancy.To become pregnant, full penetrative sexual intercourse during the fertile days (10th to 20th days) of the period is required.Avoid stress, take healthy diet, drink plenty of water and do regular exercise.Use condom in future to prevent unwanted pregnancy.Hope this may help youContact HCM for further health queryBest regardsDr. Sagar"
},
{
"id": 190084,
"tgt": "Tender, bleeding gums, white spots, pustules on roof of mouth, rotting front teeth. Cure?",
"src": "Patient: Im a 24 year old male... In the last week my gums have become very tender, bleeding , have spots that are white or grey... The roof of my mouth has had what feels like sacs that bleed out when i touch them. A couple of my front teeth have even began to get what looks like rotting toward the base. Once again, all these symptoms just showed up and have rapidly increased in the last week Doctor: Hello there..there are various reasons for that...please mention if you have elevated spots over your palate( roof of your mouth), is it fluid filled? or is it the just red patches?...do you have ulcerations all over your oral cavity or is it just the redness you are experiencing on your gums?all these questions will lead to diagnosis of your problem,,,,if they are fluid blisters the management differs...if it is just gum redness with bleed on stimulation the management differs...consult your dental surgeon for clinical examination and management...Clinical examination becomes very important for management of these kind of symptoms...."
},
{
"id": 166878,
"tgt": "What causes decreased platelet count?",
"src": "Patient: my 9 month old baby was admitted in the hospital and diagnosed had a UTI and Raseola infantum after we got out of the hospital her platelets decreased at 150 then after 12 hours it decreased to 142. Is this dangerous or what might have caused it to decreased? Doctor: Hello and welcome to \u2018Ask A Doctor\u2019 service. I have reviewed your query and here is my advice. Any viral illness can cause decreased platelet count and Roseola is a viral illness. So do not worry about platelet count. It is not dangerously low and I suggest you to follow up with a physician for clinical bleeding tendency and do not repeat platelet count unnecessarily.Hope I have answered your query. Let me know if I can assist you further.Regards,Dr. Sumanth"
},
{
"id": 55027,
"tgt": "Is there a possibility of sgpt level to increase again?",
"src": "Patient: my sgpt level was 1900 and bilirubin level was 19.6 before 5 months and admitted to hospital firstly and again after one month of discharge again i checked and sgpt 85 and bilirubin was 4.4..and i havent checked it again.is there any possible to get it again... Doctor: Hello I share your concern.Yes, there are chances of liver enzymes to increase if underlying cause is not treated.Bilirubin 4.4 and SGPT 85 is still on higher side.In my opinion you should go for liver function test including SGPT and SGOT.Meanwhile continue taking healthy diet consisting of lots of fresh fruits and vegetables and dietary fiber supplement.you should take glucose water and multivitamin supplements.Hope this answers your question.Best wishes"
},
{
"id": 46596,
"tgt": "What causes elevated phosphorus level after kidney transplant?",
"src": "Patient: I have a client who is in for a kidney transplant workup, and does coke on an occasion. It s not chronic use, but enough that it causes concern. I am a social worker, and know he is using. He has been on incenter HD for 10 years. His labs are fairly good, but does have an elevated phosphorus, as his diet is not exactly the greatest. I am trying to find a good study to show him what damage it is doing now, and how it would effect his transplant, if/when he would get it Doctor: any CKD patient may have elevated phosphorus level. after transplantation it should go away.because new. kidney will excrete much more phosphorus in urine."
},
{
"id": 120652,
"tgt": "What causes persistent rib pain with bruising?",
"src": "Patient: My husband is experiencing very painful ribs on his left side. The pain started about 1 month ago. There was a bruise on his skin a few inches down from his rib cage. The pain lasted a couple weeks and then went away for about 1 week. Once the pain started again I looked and there were 2 more bruises on his skin. He went into the doctor and they did xrays, which all turned out fine. They also did blood work - which all came back normal including platelet count. They gave him anti-imflamatory medication and it really helped - took the pain away and cleared the bruising. He stopped taking it because it really messed up his stomach. The pain came back last night with 2 more bruises on his skin. Any thoughts Doctor: Hello,I read carefully your query and understand your concern. Abruised rib\u00a0usually results from an\u00a0injury\u00a0such as a fall or a blow to the chest, which may happen while playing sports.Bruised ribs\u00a0are often\u00a0painful, but over-the-counter\u00a0pain\u00a0relievers and ice packs can help. The\u00a0bruising\u00a0may take several weeks to heal.I suggest using a gastro protector such as Omeprazole in the morning to protect the stomach.Hope my answer was helpful.If you have further queries feel free to contact me again.Kind regards! Dr.Dorina Gurabardhi General &Family Physician"
},
{
"id": 166577,
"tgt": "What could refusal to intake solids in a child mean?",
"src": "Patient: Hi I have a 22 month old Daughter. The last few weeks she has stopped eating her food. All she wants to do now is drink bottles of milk. We have tried all types of food to see if she will eat but nothing works. She has also gained a considerable amount of wait in this time. Can you help? Doctor: dear parent, I understand your concerns but there might be a thyroid gland enlargement causing dysphagia. dysphagia is difficulty to swallow solid foods. thyroid hormonal imbalance may also cause weight gain. you need a thyroid function blood test to confirm the diagnosis"
},
{
"id": 157598,
"tgt": "Have nasopharyngeal cancer, squamous cells poorly differentiated with basaloid features. Will radiation be sufficient?",
"src": "Patient: I have stage 1 nasophryngeal cancer, diagnosed 9/11. Squamous cell poorly differiantiated with basaloid features. It is my unserstanding that Radiation alone is often considered enough. The Rads Doc plans 33 weeks @ 200 for a total of 6600. The Chemo doc talked 10mg/m2 but I can read the RX, he ordered 30.1mg/m2. I m feeling hustled and possibly overmedicated. He said the chemo was to soften up any remaining cells so the Rads Doc could use less, but 200 is a medium dose, not small.. Doctor: Hello , Stage 1 nasopharyngeal cancer , is early to diagnose and ur fortunate and already started treatment is good for u . As doctor adviced radiotheraphy and chemotheraphy is the ideal treatment and its responds good and is equivalent or even better then surgery . As i tell u no drug is overdose in cancer it seems lie that but it never causes toxicity . Hope u complete the treatment course and get treated or completely cured . Get well soon"
},
{
"id": 12654,
"tgt": "back of my scalp I have itchy flaky skin",
"src": "Patient: On the back of my scalp I have itchy flaky skin. It spread to the back of my neck about 3/4 of an inch down and maybe 1 and 1/2 inches across. This spot on my neck is itchy and flaky, as well as red around the edges. I m using head and shoulders shampoo and it has improved some but it s not clearing up well. Do you think this is dandruff, psoriasis, ringworm or something else. Doctor: HI WELCOME TO HEALTHCAREMAGIC Ring worm infection cause loss of hair in the site of infection. it usually round and the edges are red . Dandruff can spread down. It has scales. Psoriasis is usually located on elbows and knees. But it can happen on scalp. It is tough to diagnose without physically looking at the rash. Regards, Dr. Jagdish"
},
{
"id": 199113,
"tgt": "What causes high levels of enterococcus faecalis in semen analysis?",
"src": "Patient: i had my sperm tested and they found high levels of enterococcus faecalis. prior to this, i had groin pain and minor testicle pain. don t know if that might be related. how did i get infected? i ve never had sex, let alone anal sex (im 17). should i be worried? could this be infertility? thano you very much. Doctor: HelloThanks for query.Your semen analysis has revealed infection due to presence of Enterococcus Faecalis .These organisms are present in perineal area and around opening and migrate through urethra to prostate and seminal vesicles .You need to take appropriate antibiotics as per culture report.This is not due to infertility but may cause infertility due to death of the sperms.Dr.Patil."
},
{
"id": 97326,
"tgt": "Can menstrual flow be affected upon meeting with an accident?",
"src": "Patient: my first day of my last cycle was January 7,2014 it only lasted for at least 3-4 days then I was in a car accident on January 17, 2014 rear end from behind I have not had my period or at least what I think until February 28, 2014 and when I first notice it was brownish red on march 1,2014 started off the same way and then turn light red on march 2,2014 I wore a tampon and when I went to change it a long darkish dry rubber string of what to appear to be blood was on the tampon but barely no blood on tampon Doctor: Hi, thanks for using healthcare magicThere are different possible causes for delayed or irregular periods. If you are sexually active, you need to rule out pregnancy as a cause.The other causes are: (1) emotional or physical stress or illness(2)inadequate diet(3)abnormal level of thyroid hormone(4)high levels of prolactin hormone which is released in the brain(5)polycystic ovarian syndrome(6)extreme physical activityIf the irregularity continues then you may need to consider visiting your doctor for an assessment which may include blood tests and imaging.I hope this helps"
},
{
"id": 100670,
"tgt": "Suggest medication for itching in the big toe",
"src": "Patient: I woke up with my right big toe very itchy and when I went to look at it it had a big hardened area at the tip extending to the side of my toe. I squeezed my toe and that area remained white and hard when the pinched area turned red. Its a burning itching sensation. Doctor: HI, thanks for using healthcare magicIt is possible that you were bitten at some point while you were sleeping.This would cause the localized irritation to your toe and would be associated with itching and abnormal sensations such as burning.You should consider the use of an oral anti histamine and also an anti inflammatory pain killer.Examples of anti histamines are benadryl, zyrtec, claritine, allegra, aeriusExamples of anti inflammatory pain killers are: ibuprofen, cataflam, voltaren, naproxenOne of each of these should be available over the counter.I hope this helps"
},
{
"id": 48585,
"tgt": "Would a diabetic with kidney disease (stage 3) need dialysis?",
"src": "Patient: I have just been diagnosed with stage 3 kidney disease as a result of diabetes. I had been taking Janumet for 10 years in addition to actose & glimperide. My dr. took me off Janumet and said I must now take insulin. Will I have to do dialysis in the near future? Doctor: Diabetic and niq stage 3 renal disease...first you were on oral medicine now switch to insulin..it shows you are not controlling your sugar level.As a result your kidneys are effected...We have got two situation ..diabetes and renal disease..Well the need for dialysis increases in such patient BUT if your sugar level is controlled within normal range and you do not have other conditions like hypertension, then you can do it with medicines (angiotensin reninin blockers..candesarton...or ACE inhibitors like lisinopril)for some time.Do your regular blood test and urine test to keep an eye on your kidneys."
},
{
"id": 38387,
"tgt": "Suggest treatment for shingles",
"src": "Patient: my mother was diagnosed with shingles 2 days ago....she is I onstant pain in her head down her face and to her neck. She says the pain is every minute. She went to the Dr and they gave her hydrocodosone, tramidol and nothing is helping. She said she can not bea this much longer Doctor: HI, thanks for using healthcare magicThe pain associated with shingles can be severe and sometimes difficult to treat.There are other pain medications that she may be able to try that may be helpful.Examples are lyrica, gabapentin, tegretol, amitriptyline. These medications have many different uses including treating nerve related pain.She may also want to consider the shingles vaccine, this may prevent future episodes.I hope this helps"
},
{
"id": 80420,
"tgt": "Suggest medication for cough and cold",
"src": "Patient: 20 days back my 8 month old son wt 7.7 kg got cough and cold , doctor prescribed kufril , nasoclear and duolin respules for nebulisation and azithral liquid . He got cured within week .yesterday night he got fever and i gave p-250 2.5 ml and today morning he was very active and i continued p-250 but now he got cold he is taking feeds well. Again cold what's the reason Doctor: It is nothing else than viral infection which is quite common among children. For this you can give the same medicines as your doctor had prescribed before or in my opinion you can give him syrup uthral 100 mg 5 ml once daily or syrup symbiotic-p 125 mg 5 ml three times daily with syrup wikoryl or syrup no-cold 1 tsf three times daily for common cold with syrup kofcef or syrup ambrodil 1 tsf twice daily. You can also use naso-clear nasal drops and salbutamol respules for nebulization as your doctor had already given. Don't worry much as any kind of viral infection will take 5-7 days to recover fully."
},
{
"id": 168205,
"tgt": "Suggest which is the second best milk other than mother s milk?",
"src": "Patient: Hi Doc,My baby is 5 1/2 months old. i need to go to work from next month. till now i have given her breast milk, little juice every day as well as nestum rice (with water).instead of mothers milk, can i give her cow or buffalo milk, or what do u suggest. i understand that there is no replacement for mothers milk, but still kindly suggest the second best.Sue Doctor: Exclusive breast feeding is recommended upto 6 months of age in our country and that means nothing other than breast milk. However sometimes weaning can be done from 5 months of age. Mother's milk is best. If not possible you can give a formula. I will not endorse any particular brand on this forum. Only try to find a economical one. They are more or less same. May be some formula with pro biotics,nucleotides and DHA is better if money is not an issue. Cow milk, buffalo milk or any other animal milk should not be given before 1 yr of age."
},
{
"id": 3464,
"tgt": "When my follicle will get rupture and what is the chance of pregnancy?",
"src": "Patient: Hi Dr . I got a injection hcg 10000 on 3 rd June but till morning 9 am my follicle are not rupture, my follicle size is 20.5 in left ovary now. But around 11 am i have some lower abdominal pain n crams. We had a sex in today morning. What is a chances of my pregnany.when my follicule get rupture n when to intimate plz suggest Doctor: Hallow Dear,Usually, after hCG injection, the follicle ruptures within 24 hours. You have noticed that the follicle was unruptured at 9:00 am; however at 11:00 am, you had some abdominal cramps. It is very likely that this pain was due to egg release (ovulation). It may sometimes be associated with little bleeding also. This is known as Mittelschmerz. The size of the follicle just before the ovulation was 20.5 mm. This is most ideal size of the follicle. Follicles of size between 18 mm and 21 mm produce most healthy eggs which have best pregnancy chances. You had intercourse at the time of ovulation. Sexual relations on the day of ovulation and then at least every alternate day are very productive in terms of pregnancy. You may confirm the rupture of follicle by ultrasonography. Wait for your menses. If you miss them, perform pregnancy test on overnight first morning urine sample after a week; done earlier, the results may be false negative. Alternatively you may opt for Beta hCG test 10 days after your intercourse. Then report to your Gynaecologist. I hope this helps you. Dr. Nishikant ShrotriSo considering all these facts, you have very good chances of pregnancy."
},
{
"id": 123218,
"tgt": "What causes cellulitis in the broken area of the knee?",
"src": "Patient: my husband has a broken knee and has been in plastercast for 6 weeks he has just had it removed and is meant to try and excersise it but now has cellulitus in the foot of the knee he has broken what should we do he is on anti biotics for cellulitus and pain killers what do i do to help him get mobile Doctor: Hello, As the cellulitis may have occurred due to any kind of infection. Doing a simple test for the check of infection and antibiotics based on that will be good. Also for regaining knee functions and mobility one needs to wait for the pain to come down. You can do quick icing in the lumbar spine which will desensitize the lower limb and regain movements of the knee. But for cellulitis to get clear a blood test and antibiotics is a must. Hope I have answered your query. Let me know if I can assist you further. Take care Regards, Jay Indravadan Patel, Physical Therapist or Physiotherapist"
},
{
"id": 31348,
"tgt": "What causes persistent low fever while on Motrin when diagnosed with pneumonia?",
"src": "Patient: My son was diagnosed with strep last tues with positive strep test. He received penicillin shot that day. Fever continued and cough and pain breathing so we went to Dr. Saturday. They diagnosed with pneumonia and gave z-pak. We continued with low fever since then. Followed up with pediatrician Monday night. He still has low fever as soon as his motrin gets out of his system. It runs only 100.5. But we have been taking motrin since last Tues. It is now the following Wednesday. He says he aches all over. He feels alittle better with motrin. But just lays around not doing much, which is not like him at all. Called Dr. Office we are following up tomorrow. Any suggestions? Doctor: Hi thanks for asking question.Here your son is suffering from strep.pneumonia.Mostly penicillin group of drug are sensitive to streptococci.For strep.pneumonia Amoxiclav is drug of choice.Meanwhile maintain your immunity with water and fruit juice more.If drug is effective then your total count by cbc will decrease along with decreasing clinical symptoms.If response is not good then antibiotic might not effective as resistance may develop against it.Culture and sensitivity report might be helpful.Complete the course of antibiotic and then after if infection has been cleared chest x ray will also show improvement.So your fever might comes because just now you have not completed antibiotic course.You will recover well with time.Don't worry.Wish you good health"
},
{
"id": 27979,
"tgt": "What causes tightness in the chest with a racing heart?",
"src": "Patient: Looking for causes of several symptoms- been to ER 2x and done many tests with conclusions: my symptoms are racing heart mostly at night during sleep- wakes me up. Then tightness and pressure on chest like bricks on my chest- worse when laying flat. Light headed ness and dizzy spells.....also have more muscle and joint pain than normal and now a sudden weight gain. I am scheduled to do a stress test and sleep test....have worn heart monitor for 30 days and found racing heart occasionally but no diagnosis or treatment yet and has been going on for a year now....interfering with my daily life- fatigued and sometimes have to leave work because chest pressure and feeling of lack of oxygen Doctor: I feel you are having obstructive sleep apnoea which explains most of your symptoms except chest pain.please get sleep test done and if OSA is established then going for CPAP will solve most of your problem. Please get stress test done to rule out cardiac cause of your chest pain."
},
{
"id": 151283,
"tgt": "Bleeding cyst on ovary. Done hysterectomy. Why syncope?",
"src": "Patient: mine is very complicated but started with a hysterectomy in late june. about 5 weeks post-op I had severe bleeding and pain and ended up back in the OR. I had developed a bleeding cyst on an ovary and the blood pooled around my vaginal cuff-incision. The vaginal cuff broke and my bowel started protruding. The Dr removed the damaged ovary and part of the bowel that adhered to the ovary and then stitched up the cuff. At my last post-op visit, my sutures all had dissolved and my surgeon said I was 100% healed. A couple of weeks later, I developed a milky, sometimes yellow vaginal discharge and a foul vaginal odor. It seemed to temporarily go away with the use of a douche . Now, the interesting part...a week ago i experienced a syncope , passed out and went right into a 2 minute grand mal seizure. the only indication of something being wrong before it happened was a feeling of being flushed--a hottness from the inside out. After numerous tests, I was released from the hospital with no true diagnosis. Today, 7 days after the seizure, that hottness from the inside out returned and it felt like hot flashes or a fever. Simultaneously, the vaginal discharge and odor returned. Now I am wondering if this is all tied together. Is it possible that i developed an infection after the 2nd surgery that then caused the syncope? Something is going on and I m wondering if it is all related or just coincidental. I cant tell if this hotflash feeling is fever or the aura that precedes a seizure. And could fever mean the vaginal infection spread to my bloodstream which ultimately caused the seizure. I need help, I don t know what to think Doctor: What is your Blood Report? Nothing can be commented without that. Please mention them in next question."
},
{
"id": 77089,
"tgt": "What could cause pain in chest while breathing deep with bending?",
"src": "Patient: I was sweeping and cleaning the other day after my son was sanding a new room we are making out of the garage. After that when I took a deep breath it would hurt. Now that is not bad but at the very top of my windpipe I think it hurts. especially when I bend or lie down. Doctor: Thanks for your question on Healthcare Magic. I can understand your concern. In my opinion, you are mostly having musculoskeletal pain. Your pain is associated with movements and specific position, so possibility of musculoskeletal pain is more. So follow these steps for better symptomatic relief. 1. Avoid heavyweight lifting and strenuous exercise. 2. Avoid movements causing pain. 3. Avoid bad postures in sleep. 4. Apply warm water pad on affected areas. 5. Take painkiller like ibuprofen. Don't worry, you will be alright with all these. Hope I have solved your query. I will be happy to help you further. Wish you good health. Thanks."
},
{
"id": 127711,
"tgt": "Suggest treatment for bruised ankle after an injury",
"src": "Patient: Hello. I fell about a week ago on my ankle. i fell off a chair and landed on my right ankle inward. I didnt roll it. I had severe swelling, clear from my mid calf all the way to my toes. The swelling has gone down but my ankle keeps bruising. My pain has not gotten any better either. My pain is starting to become fairly constant. I would describe it as sharp, almost stinging pain. I made a doctor appointment for friday. I have been on crutches and the doctor told me to put by old boot back on untill i visit her. I have also been using the RICE method. Doctor: Hello,There are two possible diagnoses: 1. Ankle Sprain2. Fracture around ankle mostly hairline.I would advise x-ray of the right ankle with tibia-fibula anteroposterior and lateral views to confirm the diagnosis. Please get it done from radiology center nearby you. Meanwhile, continue the RISE method. Ankle binder application would be more helpful. Avoid weight bearing on right ankle. Please pay a visit to the nearby orthopedic surgeon as soon as possible. You can take NSAIDS for pain relief.Hope I have answered your query. Let me know if I can assist you further.Regards,Dr. Jayesh Vaza"
},
{
"id": 173261,
"tgt": "What causes explosive, gassy and smelly poop in an infant?",
"src": "Patient: Hi, I have a 5 month old. He is totally breastfed. He always has greenish poos with white lumpy bits and sometimes green lumps. Also, his poos are very explosive, gassy and smelly. He hasn't often had a 'normal' poo. He is also in a lot of discomfort, and he tends to go about 5-7 times a day and very quickly after a feed. He has also started to be sick after most feeds too, so his gp has prescribed him gaviscon. This helped for a few days, then blocked him up, so we have stopped it and his paedetrician has prescribed ranitidine, which again doesn't really seem to be helping that much as he wakes up screaming in the night and arching his back. He is not hungry but just in pain. Does anyone know what this could be? Thank you Doctor: Hi...Thank you for consulting in Health Care magic.This is called evening colic and is quite common in this age group. This happens when the baby sucks at the breast very fast and in eagerness to drink milk will gulp in air too. Unless the air comes out like burping or flatus this discomfort will be there and next time check if the baby is sucking too fast and gulping in air too. You will be more convinced. Usually I don't advice any medicines for this as they give only temporary relief.The two best ways to relieve this distress is - 1. Do not put the baby in lying position after feeding till the baby burps out the swallowed air.2. If still crying - put the baby in prone position and keep patting the back gently so that the baby passes off the flatus and gets relieved.Hope my answer was helpful for you. I am happy to help any time. Further clarifications and consultations on Health care magic are welcome. If you do not have any clarifications, you can close the discussion and rate the answer. Wish your kid good health.Dr. Sumanth MBBS., DCH., DNB (Paed).,"
},
{
"id": 191292,
"tgt": "What causes elevated blood sugar levels?",
"src": "Patient: I am a diabetic patient having 60 years of age, from 4th of June my sugar level remains stagnant at 300-350, dr. Recommended me to have diabetic diet, on following this diet their is no control on the level of sugar. At last another dr. Recommended me to have insulin, still there is no markable result are seen, further he added to increase the dose of insulin. At present on having all medication and diabetic diet the level of sugar remains between 250- 320. As per reports HbA1c 8.5 Glycosylated haemoglobin 12.8 Blood urea 52.4 Serum creatinine 2.3 Lipid profile Cholesterol- 126.4 Hdl- 28.7 Triglyceride 309.4 Vldl- 61.88 LDL- 35.82 Cho:hdl- 4.40 Urine Albumin - ++++ At present I am having Tenepride M 500 Atorlip-F Mecorney plus Telvas -AM Unistar- 75 Doctor: Hello and welcome to \u2018Ask A Doctor\u2019 service. I have reviewed your query and here is my advice.Your A1C level and your blood sugars are high and this means that your Diabetes is not under control.In these circumstance it is very important to adjust the insulin therapy(regimen and dosis) and in my opinion you may need an intensive basal-bolus regimen.)So,my advice is to consult physically your treating endocrinologist for the right treatement and the follow up.At the other side,i suggest to consult a nefrologist because i see that you have a renal disease ( Diabetic Nephropathy and probably an acute renal disease) and it is very important to have the right therapy,accordingly to your test results(Serum creatinine,blood urea,urine albumine)The atorlip therapy and a low fat diet will help you to normalise your lipidic profile but i suggest to discuss even this problem with your treating endocrinologist.Hope this is helpful.Let me know if i can assist you further."
},
{
"id": 67897,
"tgt": "How to cure painful bruise on the buttocks?",
"src": "Patient: i fell a week ago today landing on my left buttock very hard. i had a bump for the week noticing it was getting smaller. last night getting out of my car i hit the bump. this morning i now have a blood looking bruise. it is still painful when i walk and have a sharp pain in one spot. was this a hematoma of some sort? is there something that can be done or does it need to run its course? Doctor: Hi,Thanks for your query. After going through your query I came to know that you injured your left buttock a week ago and you got hematoma. If you has no fever, then no need to worry much. Hematoma gradually resolved by itself .Thrombophob gel local application three times a day over it(no rubbing) can help in resolving it. Sometimes haematoma does not resolve by themselves. In these cases aspiration with wide bore needle is done to suck the collected blood under aseptic condition for which you need to visit doctor. You can discuss with your treating Doctor about it.I do hope that you have found something helpful.If you have additional questions or follow up queries then please do not hesitate in writing to us. I will be happy to answer your queries. . You can discuss with your treating Doctor about it. I do hope that you have found something helpful. If you have additional questions or follow up queries then please do not hesitate in writing to us. I will be happy to answer your queries. Wishing you good health. Take care."
},
{
"id": 73614,
"tgt": "What is the treatment for shortness of breath and nausea?",
"src": "Patient: Hi. I am 33 weeks and 4 days pregnant. for 4 days I have had a headache and have felt a little short of breath and nauseous. I worried I may have high blood pressure but had my father check with his blood pressure thingy and it was very low...78 over 49. The thing is he is a large man and the cuff is large, not sure if this would be a problem with the results. I see my OB on Thursday, should I make a sooner appt. or are headaches like this common in later pregnancy. Is low blood pressure a serious concern? Doctor: Thanks for your question on Healthcare Magic.I can understand your concern.Yes, you should definitely make sooner appointment with your gynecologist.Hypotension in pregnancy is very serious if not treated.And your symptoms like headache, breathing difficulty, nausea are also symptoms of hypotension.So consult your gynecologist as soon as possible.You will need blood reports and Intravenous fluid for hypotension.Hope I have solved your query. I will be happy to help you further. Wish you good health. Thanks."
},
{
"id": 127947,
"tgt": "What causes pain between shoulder blades?",
"src": "Patient: Hello, I have lumbar spondylosis and for the last 6 months have been having pain higher up between my shoulder blades and spasms over the top of my back. I was sent for an X-ray which showed degenerate changes in my thoracic spine and the starlings of osteoarthritis. For the past month I have been getting pains in my legs, mainly behind the knee and tops of my calves. To rule out pain being caused from my spine I was sent for an MRI on my thoracic and lumbar spine areas. The results have shown a shadow on my thoracic spine area and subsequently I have been referred for another MRI to look at this shadow in more detail. Of course the word shadow has prompted cancer immediately in my head, although my gp explained that I shouldn't worry about cancer initially as she thinks I would have been sent back to the hospital much quicker if this was thought to be the problem. My questions are; would a tumour have shown in the X-ray I had in June and what other causes could be the cause of a shadow on my spine? Thank you Gina Doctor: Hello, I have studied your case.Spine tumour has very low incidence.Shadow can be due to benign hemangioma or osteoporosis.If there is any doubt you can do PET scan.Medication like methylcobalamine with neurotropic like pregabalin will reduce pain; you can take them consulting your treating doctor.Continue your physiotherapy.I will advise to check your vit B12 and vit D3 level. Hope this answers your query. If you have additional questions or follow up queries then please do not hesitate in writing to us. I will be happy to answer your queries. Take care."
},
{
"id": 144085,
"tgt": "Is numbness in hand and face signs of stroke?",
"src": "Patient: hi i had an episode that i dont know what it was, my face went numb and i couldnt get my words out it was like i slowed down and couldnt speak, my arms also went numb and i collapsed on the couch it took about 4 hours before i came right i went to hospital and had a ct scan and blood tests were all good and they said it may be anxiety but i am concerned it may have been a mini stroke or something? can you help me? Doctor: Hi, I am Dr.Bruno. I have read your question with care and understand your concerns. Let me try to help you The episode which you have narrated > can be a 1 ) TIA : Transient Ischemic Attacks or 2) RIND : Reverse Ischemic Neurological Deficit As the names imply, these are temporary events due to low blood flow to brain and if the blood flow is restored soon the functions will be restoredIf the blood flow is not restored, it will progress to strokeSo, I would suggest you to consult a Neurosurgeon / Neurologist and Do tests to check blood Flow in the vessels that supply brain (4 Vessel Angiogram or CT Angiogram) Hope you found the answer helpful.If you need any clarification / have doubts / have additional questions / have follow up questions, then please do not hesitate in asking again. I will be happy to answer your questions.Let me know if I can assist you further.Take care."
},
{
"id": 170917,
"tgt": "What causes pale pink spots on face and torso in children?",
"src": "Patient: my 1 year old has been tired and not feeling well the last 2 days. I thought it was teething, but today I see she has tiny pale pink spots on her face and torso - not a lot, but just enough to notice. The spots do not seem to be raised. All immunizations up to date - with the most recent being a couple of weeks ago. She had croup end of October, and has a slight cough now. Doctor: HiHe is most likely suffering from viral infection which are self limiting and common in this age group. Give him acetaminophen for fever and pain relief along with antihistamine such as Chlorpheniramine or fexofenadine for cough. Also, ensure a good healthy diet and plenty of fluids.Hopefully this will improve with in the next few days.Take care"
},
{
"id": 110707,
"tgt": "How to manage severe back pain and difficulty in walking?",
"src": "Patient: old back disk injury. onset of extreme low back pain in back and hips no leg pain however extreme pain in low back and hips hindering me from walking also difficulty standing up from sitting position and sitting down from standing position. the pain is extreme Doctor: Hello, Thanks for your query.You might be suffering from Acute mechanical back pain.1. Hard bed rest for 2-3 days2. Take some muscle relaxants like myoril 8 mg, etc., along with pain killers like diclofenac 50mg. You can continue using volini spray.3. Hot water fomentation to back 2-3 times a day4. Physiotherapy - Interferential therapy (IFT) to your back will help you relieve the pain5. Avoid driving, leaning forward, sitting for prolonged time or lifting heavy weights. These activities can increase your back pain.Even after a week if your pain doesn't gets relieved, better to get yourself examined by an orthopaedician. He might ask you to take an Xray of your backI do hope that you have found something helpful and I will be glad to answer any further query.Take care"
},
{
"id": 88923,
"tgt": "What causes lower abdominal pain when walking?",
"src": "Patient: Lower, Lower abdominal pain when I am walking only. Went to doctor yesterday, she thought I had urinary tract infection. Prescribed Cipro antibiotic. I m drinking plenty of water. I still cannot walk without severe pain. It does not hurt when I m standing or sitting. Doctor: Hello Lower abdominal pain may be due to many reasons.It may be due to renal or ureteric calculus,pelvic pathology,musculoskeletal cause etc.You need proper clinical examination and few investigations.Investigations include routine hemogram,RFT,urine RE/ME,ultrasound of abdomen.Further investigations can be done if needed.Proper treatment depend upon findings.Take CareDr.Indu Bhushan"
},
{
"id": 12359,
"tgt": "Suggest treatment for psoriasis",
"src": "Patient: Hi...sir my mom is suffering from psoriasis and take many treatment.When she take medicines then for a while r say for a period of time it gets cured and again come back.So plz suggest a treatment r good docs name and nos in chandigarh. With best regard Vaneet Sial. Doctor: Hi,As you said your mother suffers from psoriasis...I would like to say few things about psoriasis... . Psoriasis is a chronic relapsing and autoimmune disease. You may be having scaly plaques on various parts of the body,scaly lesions on the scalp. Exact cause is not known. Genetic tendency, autoimmunitity,stress,dry wheather...etc may be responsible for precipitation and exacerbation of psoriasis. You consult dermatologist for firm diagnosis and treatment.I usually recommend cap acitretin 10 mg twice a day along with methotraxate 15 mg weekly in three devided doses 12 hourly...for psoriasis.This may be taken for long time till good response. Blood tests may be done to monitor the treatment. For itching, you may take antihistamnics like levocetirizin...Avoid steroid ,as there may be severe flare up after stopping it.She may apply mild steroid cream or calcipotriol oint on the lesions on the psoriatic lesions. She may apply moisturiser to improve the skin texture. She should avoid soap bath for few weeks..And she should be free of stress and worries as that might worsen psoriasis. She might get good result.I hope you got my answer.Thanks.Dr. Ilyas Patel MD"
},
{
"id": 72456,
"tgt": "What causes sudden cough with lung odor?",
"src": "Patient: I recently smelled a strange odor coming from my lungs/bronchial tube after a sudden cough. It was not an extended cough more of a sudden clearing hack. The smell lasted about 10 minutes and was sweet and kinda chemically. Should I be worried? I don't and have never smoked though have been around smokers alot. Doctor: Thanks for your question on Healthcare Magic.I can understand your concern. No need to worry for single episode of this kind of sudden cough with sweet smelling expectoration. Do fresh air inhalation, warm water gargles etc. But if such episodes are recurrent then better to consult pulmonologist and get done clinical examination of respiratory system and chest x ray to rule out lung infection.Hope I have solved your query. I will be happy to help you further. Wish you good health. Thanks."
},
{
"id": 88095,
"tgt": "Suggest cure for a sickness in the stomach",
"src": "Patient: after I ate my dinner last night, straight away I had a pain in the upper abdomen, i tried andrews liver salt 1 teaspoon after a while I was burping like a trooper then later I tried rennies, no good this is still with me this morning, how can I get rid of this gas? thanks Doctor: HI.Understood your problem:Pain in upper abdomen after dinner and you tried something at home, yet no relief in the morning. I would advise the following in such a patient:Continue Zantac 12 hourly.Pepcid every 3 to 4 hours when on empty stomach. Add Domperidone or such motility regulators.Soft bland diet. No aerated colas / soda/ drinks.Early dinner. No late nights.Treatment for anxiety (as prescribed / needs a prescription by your Doctor)Lying in a reclining positionWalk around after dinner.Think of the factors which increased the present problem, GERD, anxiety and try to avoid. If no relief, get Upper GI endoscopy done and get prescription for added Medicines. EKG for the heart n a safer side to rule out cardiac problems. This helps all my patients and should help you hopefully."
},
{
"id": 85714,
"tgt": "What does Prednisone do to your body?",
"src": "Patient: I have been prescribed Prednisone to hel with severe ankle pain. I have been following the doctor s instructions except, since the pain went away very quickly, after 5 days I tested the prednisone. I stopped the one dose each morning on Friday morning. 8 oclock that night I got dizzy, weak and dis coordinated. Not so much that I could not continue my activities. I have heard that Prednisone is a steroid and should be avoided. Any response to all this? I can t afford you. Sorry Doctor: Hello, Yes, it's true that Prednisone is a steroid. However due to its potent anti-inflammatory action, we do prescribe in cases of severe arthritis as in your case when the benefits outcome the side effects. So please do follow your doctor's recommendations and do not change dose on your own or do not stop medicine abruptly on your own without tempering as directed by your doctor. Hope I have answered your query. Let me know if I can assist you further. Take care Regards, Dr Albana Sejdini, General & Family Physician"
},
{
"id": 58008,
"tgt": "Untreated Hashimoto's disease, had developed elevated liver enzymes while on medicine. Could Hashimoto's be causing this?",
"src": "Patient: I have untreated Hashimotos disease. Have been on medication on and off. While off the medication, I developed elevated liver enzymes . I have had a liver biopsy and do not have liver disease yet my liver enzymes are still higher than normal. The liver specialist thinks it is from supplements or prescription medication. I know that is possible and he said I would have to be off the offender for several months to determine what is the cause. He does not want me to take any new medications or supplements. I am wondering if it could be the untreated Hashimotos that is causing the problem. My liver enzymes have been elevated from Sept 2009. Doctor: Hi there,Thanks for your query.Hashimoto's is an autoimmune disease, whereby which the immune system recognizes thyroid tissue as foreign and destroys it gradually.Hashimoto's is commonly associated with other autoimmune disorders, such as, diabetes and celiac disease (allergy to wheat, rye, barley and their products). It is therefore possible that you may autoimmune hepatitis. This may account for persistently raised liver enzymes.Thyroid hormone replacement therapy for hashimoto's does not ordinarily raise liver enzymes. You should therefore continue to take Thyroid hormone replacement therapy.Consult your doctor and apprise him of my opinion. I am certain that he will agree with me, order required tests, and- based on the reports- will prescribe suitable/modified treatment as advised.In case of any further queries, revert back to me THROUGH DIRECT PREMIUM QUERY (Paid), addressed to me personally. You will receive- almost immediately- my response in great details.If you find my response helpful and informative, do not forget an \u201cexcellent\u201d (5-star rating) to my answer, to ENCOURAGE ALL doctors- engaged in social service- to render sound advice to the FREE queries. Fond regards and have a nice dayDr. Rakesh Karanwal"
},
{
"id": 124752,
"tgt": "Suggest treatment for loss of normal cervical iordosis",
"src": "Patient: I have recently found out after an xray that I have loss of normal cervical iordosis following I fall I had approximately 3 months ago. What is the correct treatment I should be receiving for this. I am having physio, which I was doing prior to knowing this. But my pain seems to be increasing more Doctor: Hello, Consult a spine surgeon and get evaluated. If you are asymptomatic no further evaluation is required. In case of symptoms like pain or discomfort, surgical correction might be required but it is unlikely in your case. Hope I have answered your query. Let me know if I can assist you further. Take care Regards, Dr Shinas Hussain, General & Family Physician"
},
{
"id": 213063,
"tgt": "Teeth chattering, cold hands and feet, excessive perspiration. Can it be depression?",
"src": "Patient: Hi.. It s the 4th day and my teeth wouldn t stop chattering. The teeth chattering has disrupt my sleep and it gets worse everyday. It s starting to affect the way I talk. I m slurring already and it s beginning to affect my job. Besides that, I also have cold hands and feet . I perspire more than usual. I went to see the doctor and he said it might be thyroid . My sister has thyroid and the doctor said it s hereditary. Besides thyroid, can it be depression? 6 months ago, I have suffered depression. Back then, the doctor prescribed lorazepam and aposertraline for me. I was suppose to take the medication for at least 5 continuous months, but I stopped after 2 mths. Recently, I feel like it s coming back. I m very stress at work and with my life too. Im starting to break things when I get angry... I ll throw and slam everything I could lay my hands on.. I get so angry all the time and emotional too.. My boyfriend is my victim. With this teeth chattering and all, I m getting more stressed up and anxious. I m waiting for my blood test result tomorrow to see if I have thyroid. Meanwhile the doctor prescribes propanalol for me to control my chattering teeth and jaw. It s not working! My whole mouth is still chattering. It s getting worse! I m so angry! Cos I can t believe I was give a lousy medicine. If it s not thyroid, what could it be? My depression? I think I m suffering from bipolar. What s wrong with me? Please help me. I m only 22 yrs old.. Doctor: Hello and welcome to Healthcare Magic. Thanks for your query. It is likely that your symptoms are due to a psychological cause - probably a depression or anxiety-related problem. Moreover, it's clear that you are quite stressed out and are in significant distress. Also, since you've had a recent history of depression for which you discontinued medication permaturely, you have a risk for relapse of symptoms. So, I would strongly advise you consult a psychiatrist for a detailed psychological assessment and further treatment. There are effective treatment options - in the form of medication or counselling / psychotherapy which will help you overcome your problems. Wish you all the best. - Dr. Jonas Sundarakumar Consultant Psychiatrist"
},
{
"id": 24216,
"tgt": "Why am I feeling tight around arm?",
"src": "Patient: About two weeks ago, I woke up and found my left arm feeling tight, like I pulled my muscle. If I straighten my arm, it feels tight. Feeling along my arm, I found that the discomfort was really originating from my vein; and there were a few bumps (4 or more) along the vein. What is this? Doctor: The pain by your description looks non cardiac. Still, to rule cardiac cause tell me what is your age. Any associated disease like diabetes, hypertension. Smoker or not. Get a ECG done. You can get a TMT or stress echo done for cad risk assessment. Get back with the answers.Dr Sameer Maheshwari"
},
{
"id": 91948,
"tgt": "Water is present in abdomen causing pain along with high BP, cholesterol and COPD",
"src": "Patient: I just had a pelvic ultra sound- showed I have ab excessive amt of water between my abdomen & my abdominal organs.....I am quite frightened. I have been having severe pain in the area of my left kidney.& I am extremely tired ALL the time, as well as very weak .I am 65 years old with high blood pressure, high cholesterol & COPD. Also have a stent from a 99% blockage- i take clopidogrel & I quit smoking a year & a half ago. I do not drink. Doctor: Hi and thank you so much for this query.I am so sorry to hear about this excessive fluid that you have in your stomach. this would need to be followed up with a more detailed exam to establish the origin of this fluid. It could be caused by infections or elevated procedures in the system that cause fluid to be forced into the abdomen. I will suggest that you follow up with your primary doctor to get this looeed into. You wold most likely require an ultrasound to measure the pressure in the blood vessels and investigations into possible liver and kidney infections. I hope this addresses your query fully. Thank you so much for patronizing our services and please do feel free to ask for follow up clarifications and information if need be. I wish you the best of health.Dr. Ditah, MD."
},
{
"id": 108239,
"tgt": "What causes lower back pain and tiredness?",
"src": "Patient: I am male 56, weigh 210lbs and 1.84m. for the last 20yrs have had lower back issues through work, and damage to my knees. usually managed with pain killers and visits to Osteopath. Last six months I have had severe cramp in lower legs, mainly night time. nothing helps to get rid, feels like my bones breaking its so painful. now followed by tingling, throbbing of calf and more worrying swelling above ankles and when pressed the area of skin doesnt return it leaves indentation until swelling goes down. I have had blood test, for heart, liver, kidney , lungs, results ok. but low on Vitamin D apparently. more recently i have no pressure when urinating, throbbing legs more often and pulsating with lower back pain and tiredness. Doctor: U TAKE VITAMIN b12 WITH GABAPENTIN TAB EVERYDAY AFTER DINNER FOR MONTHS.. AND shelcal os daily one for 6 months with RANITIDINE"
},
{
"id": 160488,
"tgt": "Suggest treatment for fever",
"src": "Patient: hi,my daughter is 2 yrs old ,she is having fever (102.3)we have given her,crocin liquid but after 6 hours the fever is still 101 degree,should we give her meftal-p,till now no other symptom shown ,only fever just for informaton this is also prescribed by the her doctor when she had same condition earlier. Doctor: Hi, You may give her both medications. Her fever will rise and fall as the medications do their job, but then naturally wear off. Crocin will usually last about 4-6 hours and Meftal-p will usually last about 6-8 hours. They are metabolized by the body differently so you can give one, and if her fever returns before the next dose is allowed, then you can give the other. Please write down what times you give each medication to be safe and to be sure that you don't give the same one too frequently and before it is due. Hope I have answered your question. Let me know if I can assist you further. Regards, Dr. Lisa Baker, Pediatrician"
},
{
"id": 140039,
"tgt": "How effective is taking vertin 16mg for vertigo and labrynth?",
"src": "Patient: Hi i am 29yrs old married with a son aged 5 yrs. i am having severe giddiness problem for the past 5 years. i consulted a ent surgeon took few tests and diagonised with hyper active labrynth and positional vertigo problem and was under medication stemetil 5 mg for 3 months. but still the problem persisted and i consulted a neuro surgeon took an mri brain scan and result was normal. he has asked to take vertin 16 mg which i am taking twice daily. but still i am feeling very much giddi and loosing my balance. even while sleeping i can feel the dizziness. pls advice as i cant do my normal work Doctor: Hi, First of all, you should measure your blood pressure and if it was high,you would take anti-hypertensive drug and do an ECHO on your heart to assess the ejection fraction of the heart. Second, if you take oral contraceptive pills,you should know that some pills can cause vertigo such as progesterone and salt and water retention such as estrogen. So, you can ask your doctor for another method for birth control. When you stop oral contraceptive pills you can take Vertin 16 mg twice per day. Semetil 5 mg is not suitable for your case. Hope I have answered your question. Let me know if I can assist you further. Regards, Dr. Mustafa, Neurologist"
},
{
"id": 79078,
"tgt": "What causes pain in left side of chest under breast?",
"src": "Patient: pain in left side of chest under breast I've had pain in left side of chest since last night ..Sometimes feels like something is weighing my chest down. And next minute feels like someone is ripping my chest open.. I do have asthma too but it doesn't feel like that.. Very uncomfortable to deal with can't sleep.. Doctor: Thanks for your question on Health Care Magic. I can understand your situation and problem. Left sided chest pain with pressure and heaviness in chest is commonly seen heart diseases like angina or coronary artery disease (CAD). So we should first get done ecg, 2d echo and stress test. If all these are normal than no need to worry for heart diseases. Sometimes stress and tension can cause similar kind of symptoms. So avoid stress and tension. Be relax and calm. Don't worry, you will be alright. But first rule out heart diseases. Hope I have solved your query. Wish you good health. Thanks."
},
{
"id": 120116,
"tgt": "What causes dizziness and nausea with a broken ankle?",
"src": "Patient: My husband broke his ankle 8 days ago. He has a cast and although no op is required he has been experiencing extreme dizziness to the point of vomiting when he attempts to get up out of bed. he is due to have a scan today to check for Dvt as a result of the break but wondered if you had any thoughts. I have just had a baby so could really do with him being a little more mobile to help with our toddler and everyday life! Doctor: Hello,Extreme dizziness with vomiting after a episode of injury. Be careful, this can be due to:- Due to medicines which he is taking- Injury to his head (Loss of consciousness, dizziness, ENT bleed and headache are symptoms of head injury). I shall advise you to consult to your doctor without taking much time. Hope I have answered your question. Let me know if I can assist you further. Regards, Dr. Mukesh Tiwari, Orthopedic Surgeon"
},
{
"id": 163739,
"tgt": "What can cause redness in cheeks,low temperature and tiredness?",
"src": "Patient: my son has had a bad cold for about a week and a half, cough, runny nose, he went to the doc and they told me he had a ear infection in his left ear. he has been on antibiotics for4 days. last night he was throwing up and said he has a soar throat. today red checks and no temp actually a low temp96.3. has been laying on the couch all day. when he gets up he wants to lay right back down. He is 5 yrs old and usually a typical boy running all around. So confused what to do. any ideas? Doctor: dear parents your concern is right. he had simple cough and cold but now it seems to have secondary bacterial infection which is extending to ears. better take him to an ENT specialist at the earliest. in the mean time give him warm water to drink constantly. steam of plain water if he is ready to take and warm saline gargles if he can. this is symptomatic only. your ENT specialist will prefer to follow it. please do not delay"
},
{
"id": 51045,
"tgt": "Continuous pain in upper abdomen and back, CT shows absence of calculi. Had passed kidney stone in the earlier week. Cause of pain?",
"src": "Patient: Hi Sir/Ma m, I had a kidney stone last week , which is no more there now. But I am getting continuous pain in my left upper abdomen and back side. On consulting the Urologist again, he said that it is due to the swelling in kidney and urinal infection. I am taking the medicine Niftran 100 mg, but even after 1 week, I am getting the same pain. My CT scan shows no calculi at all. Can there be any other reason of this pain. Please suggest. Thanks, Sarika Rathore Doctor: Hi Welcome back. This is a common problem in clinical practice. If the pain persists, muscle spasm is possible. Medicines to take care of that, pain reliever gel application and warm application with hot water bag should help. best regards DR GS"
},
{
"id": 92072,
"tgt": "What is the permanent solution for hiatus hernia?",
"src": "Patient: My 34 years old brother is suffering from hiatus hernia . Whos stool is blackish and digestion is improper from long time . Dr.are said that it is not confirm after surgery it will be permanently cure . What can I do for permanent treatment.Tell me please. Doctor: Hello.Thank you for the query.Hiatal hernia does not have to be the reason of your symptoms. Black stools usually indicate bleeding from upper part of digestive tract. Indigestion can have plenty of reasons but is not characteristic for hiatal hernia.Such hernia can give no symptoms or cause chest burning, upper middle abdominal pain, nausea. If such symptoms are present abd the diet is good (no fatty foods, no spicy foods, no coffee, no alcohol, no chocolate) medicines (proton pump inhibitors) should be prescribed. If it wont help, surgery can be considered. Surgery can be permanent cure.Hope this will help.Regards"
},
{
"id": 91896,
"tgt": "Could persistent, dull pain in lower abdomen by due to diagnosed MCTD?",
"src": "Patient: I've been diagnosed with MCTD. I've had a persistent, dull pain on the lower right abdomen for several months. All tests for infections and/or cysts are negative. Currently being tested for gallstones as the culprit. Could this be caused by the MCTD medicine I take? Doctor: Hi. MTDC and its medicines can cause this type of pain, yet it is important to locate the cause.I hope you have undergone a CT / MRI scan of the area. A good clinical examination is also of utmost importance as the information one gets may not be seen in any of the tests.Please be examined by a Gastro-Surgeon, telling his your history of disease and treatments so far."
},
{
"id": 27786,
"tgt": "Suggest treatment for heart palpitations in a 52 year old female",
"src": "Patient: I am a 52 year-old female and have started to experience heart palpitations that last all day long. They are more noticeable at certain times than other, but they never really go away. I have read that palpitations are normal, but nothing I ve read addresses someone who is plagued with them all day, every day. What can possibly cause such a chronic problem? Doctor: Mam palipation may be due to increased awareness of normal heart beats. It's important to establish the type of palpitations, because u have it everyday lasting for practically a long time, get an ecg done during the episode, if is more rarer a 24 hour holter may help. This will help know what is the frequency and what are the features of heart beats. If anyone of then shows heart beat which is normal but more than 100, we should rule out causes like increased thyroid, anemia in your age group, however if the type is not normal a review with your doctor for the treatment is warranted. However if the above test prove that the heart rate is normal during your symptoms, do try ignoring the symptoms. Maybe peri menopausal period is the time whereby many patients have similar symptoms. Hoping this will help Regards"
},
{
"id": 185846,
"tgt": "Can i take 'hifenac-p' for tooth ache?",
"src": "Patient: hi i m inindia,its 0100 hrs(night) and i have a severe tooth ach, i found a tablet HIFENAC-p,which contains ACECLOFENAC BP 100 and PARACETAMOL IP 650 plz tell if i can take this as a pin killer for this time so i can pass night with a bit of sleep plz answer fast as i m waiting Doctor: Hello:)Welcome to HCM.You can take hifenac-p for pain,make sure you have something in your belly, at least a glass of milk and you can take it.Take it b.i.d.Take a antibiotic course like Moxikind CV to counteract the infection.Regards."
},
{
"id": 87454,
"tgt": "Suggest treatment for abdominal pain post gallbladder removal",
"src": "Patient: my dad recently got his gall bladder removved...hes having pretty bad pains in his stomach and also we are most concerned about he has little read bumps that just popped up on his face and they look like theve been bleeding but they havent and they dont itch or hurt. What could this be? Doctor: Hi and welcome to HCM, Nothing to worry. There is usually minimal pain associated with this operation. The abdomen will be sore as well as the small incision sites. It will disappear on its own. When you are discharged home you will be given a pain medication to take if necessary. The first few days you may want to take the pain medication so that you will be more comfortable. As you become more active the amount of pain medication needed should decrease quickly. Bumps could be due to some infection, can be treated easily with meds. Thanks for writing into us. Wish you speedy recovery"
},
{
"id": 70261,
"tgt": "What causes a lump on the lip of the clitoris with no pain and fluid filled in?",
"src": "Patient: I have a bump almost the size of a small pea on the lip on my cliterous and I m not sure what it is. it doesn t have a head so I can t pinch it, and it does hurt if I poke at it. it s been there for almost 2 weeks now. I think I remember getting a bump there before and it went away, but now it s back. I ve never had sex before so I m not sure what it could be from. I m wondering if I should get it checked out by he doctor but I want to save myself the emberesment if I don t have to.. Doctor: HI. If the lump has reappeared , it may lead to a problem in future wither by infection or enlargement or abscess formation. I can understand what you feel to be seen by a Doctor- you may please consult a Female Doctor - Gynecologist or a Surgeon and get this checked and removed too ."
},
{
"id": 146123,
"tgt": "What causes headache while suffering from frontal lobe abnormalities?",
"src": "Patient: My MRI sys, a few punctate FLAIR signal abnormalities are seen in the frontal lobe white matter bilaterally which are nonspecific and may be related to mild chronic microvascular changes. These findings have also been described in the setting of migraine headaches. I have sudden.y had multiple headaches for two months. I also had Rocky Mountain spotted tick fever in the past year. Is ther any real explanation for these headaches in this MRI? or could the tick fever be playing a factor in it. Doctor: Hello dear friend,I understand your concerns.Punctate white matter abnormalities on flair MRI , can result due to microvascular changes but also otherwise,are seen in 27% of the asymptomatic or normal individuals.If at all, your episodic migraine has caused these MRI changes,even then your migraine and not your MRI is to be treated.Your tick fever is a thing of past,and whether it has caused these changes or not,hardly make any difference till the time you are asymptomatic.I hope it helped."
},
{
"id": 94916,
"tgt": "Pain in the lower abdomen on sneezing or coughing. Appendicitis?",
"src": "Patient: I have been having pain in my lower right abdomen (slightly under where my belly button is and to the right) it is VERY painful ( stabbing pain) when I sneeze or cough or laugh too hard-- or try to sit up. I still have my appendix, but am not exhibiting any other symptoms of appendicitis . Could it be something serious? It s been two days now. Doctor: Hello! Yes this can be an appendicitis. When you caught or sneeze your peritoneum is moving and it hurts when it is irritated by some inflammatory process in abdominal cavity. If you do not have any others symptoms like nausea, mild temperature rise, and higher heart rate it does not rule out appendicitis, and even observation in hospital may be required. You need to have a least abdomen ultrasound and blood tests performed. If it might be a chance that you are pregnant, a pregnancy test should be done. Pain in this location is characteristic also for an ovaries inflammation and right kidney infection. Hope this will help"
},
{
"id": 134516,
"tgt": "Suggest remedy for bruises in shin",
"src": "Patient: I fell and hurt my shin. A lot of bruising but no pain the first two days. I saw the doctor and he gave me medicine to break up blood cloths. After two days when the swelling and blue went into the foot area, I have started having escalating sharp pain in the shin area. Is my leg getting worse? Doctor: hi,looks like your pain is increasing. kindly see if you apply some ice close to the area where you get pain in the circular pattern. post that slowly follow small ankle and toe movements while sitting on a couch. try performing simple leg raise up and down for 10-15 times for holding each time for upto 5-10 counts based on your holding capacity.hope you will recover and get rid of pain.thanks"
},
{
"id": 17277,
"tgt": "Can a right bundle branch block cause chest pain?",
"src": "Patient: Hi I was diagnosed with RBBB back in October 14. It was found by my pcp on a ekg. I then had an echo done for further testing. The echo only showed the rbbb, everything else was fine. I still have chest pains at times. I work outside and sometimes my chest will hurt. If tests show only a rbbb, is this pain normal? My pcp says it s nothing to worry about but I just want another opinion on the matter. My biggest concern is that I ll have a heart attack which they say I won t, etc. Doctor: Hello There After going through your medical query I understand your concern and I would like to tell you that RBBB is a conduction pattern which is normal in most of the people you need not to worry about any heart attack. Chest pain can be there with gastritis also or anxiety .Hopefully this information will guide you properly Kind Regards Dr Bhanu Partap"
},
{
"id": 25650,
"tgt": "Suggest cause & treatment for irregular heart rate",
"src": "Patient: my friend was told that he has an irregular heart rate 39-40bpm.and that he is lucky that he wake up in the morning.and that he needs a pacemaker to be put in.and for him to go to emergency room.but my friend didn't.what is going to happen to this person if he doesn't soon get the pacemaker in? Doctor: Thanks for your question on Health Care Magic. I can understand your concern. By your history and description, possibility of heart block is more in your friend's case. It is conduction defect in heart when impulses are regularly conducted. So heart is not beating properly and hence blood circulation in the body is also not proper. Pace maker insertion is must for him. If he is not willing for pace maker, he may develop brain damage (paralysis) due to decreased blood supply to brain. Sudden stoppage of heart is also seen due to heart block. So sudden cardiac arrest may occur and it is life threatening. So convince him for pace maker insertion. Hope I have solved your query. I will be happy to help you further. Wishing good health to your friend. Thanks."
},
{
"id": 33235,
"tgt": "What causes fever,congestion and cough?",
"src": "Patient: I had a flu with fever 7 days ago , the fever broke then congestion started which lead to coughing. The last four nights is when it hard to breath at night and the right side where my jugular vein feels swollen. IS this just a simple bacterial or viral infection in the lungs?? Doctor: HelloYou got flu 7 days back , for last 4 days there is severe cough with difficulty in breathing.Such type of symptoms may be due to bronchoconstriction ( as this produces severe cough and difficulty in breathing ) .There develop wheezing sound and rales .When such bronchoconstriction combine with bacterial or viral pathogens there also develop fever .Without proper examination it is very difficult to diagnose , which type of infection , you are having.So consult physician and get his opinion.Good luck."
},
{
"id": 147804,
"tgt": "What causes tingling feeling near my eye and weakness in my arms and legs?",
"src": "Patient: Hi, I have had this face tingling on my left side near my eye and goes round my cheek. It come and goes. I have had it for a couple of years now and am really annoyed I as it is not going away. I get really down when I have it. Also I have been feeling really weak in my arms and legs. This has just stared. I feel like there is something wrong as I don't feel generally well all the time. Just like a blah yuck feeling. I am female and 38. Doctor: Hi, thanks for using healthcare magicThe tingling sensation that you are experiencing is likely neuropathy. This refers to abnormal structure or function of the nerves.It can be central (involving the brain and spinal cord) or peripheral.Your abnormal sensations may be related to peripheral neuropathy.The possible causes are: (1)vitamin deficiency- particularly b12 or folate(2) trauma(3)diabetes(4)exposure to toxins(5) autoimmune disease(6)infections(7)liver or kidney diseaseYou may want to consider visiting your doctor for an assessment.I hope this helps"
},
{
"id": 112793,
"tgt": "Lower back pain, abdominal pain, possible kidney infection, dizziness. On nitrofantoin. What is going on?",
"src": "Patient: I have low back pain and can't seem to go all day at work and I get tired early evening. This is new lately I thought I was getting a kidney infection but doctor said no did an exam he found nothing. Tonight was fine back pain started around 7pm after eating and got tired pretty fast too. Also stomach pains and feels like some bloating.. I thought it could be cystitis cause I get the urgency to urinate. I did get a little dizzy kinda not feeling right tonight. Doctor had me on nitrofantoin when that flares up but he saw nothing even when he checked my urine. I feel like I am hungry but I ate already. Not to mention being bloated feeling in upper stomach I would not eat if I wanted too. Help! Doctor: Hello, Welcome at HCM.I have read complete details of your question.It looks that you are having back pain & urgency of urination with bloating sensation in stomach.This problem can be there due to following reasons--Diabetes-Overactive bladder-Infection of urinary tract-Gastritis-Cholecystitis or inflammation of gall bladder.At present you should go to your general medicine doctor & should have a complete investigation including blood sugar,ultrasonography abdomen & a x ray of lower back.You may have relief by following--Take a good analgesic like paracetamol,three times a day.This is available OTC.-Take an antacid preparation like pantaprazole 40 mg on prescription of your doctor.-Do warm fomentation over your back.-Do rest.-Avoid spicy & oily food items.-Take crane berry juice.Your final treatment will be based on your reports.Stress should be given on blood sugar report.Hope this will help you.With best wishes."
},
{
"id": 163729,
"tgt": "What is the treatment for a pain in the buttocks of a child?",
"src": "Patient: My 5 year old daughter says her bottom is hurting. This has been going on for 2 nights and only happens at night. She has no problem having a bowel movement so i think and it doesn t hurt when she pees so a uti is out. The only thing that calms her is when i rub her butt.. What is this.... Doctor: may be you should look for worms in her stool.worms comes out at night and can cause perianal itching and discomfort...local examination of perianal region may rule out other causes.."
},
{
"id": 75445,
"tgt": "Any suggestion for shaky arms/legs, weakness when having pneumonia in lungs?",
"src": "Patient: My wife fainted about a week ago after standing up from a seated position. She came back to a few seconds later. After about 15 minutes she felt able to sit up and i took her home. That whole night she said her legs felt shaky all night. Went to gp next morning and he thought it might be something virul and prescribed antibiotic. Started feeling some better then got worse after another week. Went back to gp and she passed out again and said she felt like it was hard to breathe. Sent her to er. Doctors did ct and mri on head and chest. Ct showed pneumonia in lungs everything else ok. After 4 days in hospital she was released. Now at home she still feels really week and legs and arms feel trembly. No fever and nonproductive cough. Is the shaky arms and legs normal for a 37 year old. Doctor: Hello dear, Thank you for your question. Infectious diseases like Pneumonia go through 3 stages of development. The first stage may include fever, malaise, weakness, headache, or even fainting as you described. After that, the disease develops with fever and productive cough. The healing phase often presents with weakness and hypotension, which may also be experienced with arms and legs shaking. In such cases, I suggest to my patients to drink plenty of water, in order to prevent dehydration that leads to hypotension. I would advice her not to get up from the bed immediately, but to sit and rest for a bit. I hope this helped.Best wishes,Dr. Alma"
},
{
"id": 199152,
"tgt": "What is the cause of a swollen testicle?",
"src": "Patient: Hi I got a problem for the past 2 weeks I it something under my testicles close to my but hole I can t see what it is but a couple times I had blood coming out It feels like the end of my testicle is a little swollen I don t under stand where the blood is coming from what should I do Doctor: Welcome to healthcare magicI am dr Fahim and I will help you with your problemI have gone through your question, need answers to the following questions for better understanding of your problem- was there any pain or swelling 2wks back in peri-anal area?- is it bloody discharge or sometimes serous/watery or pus?- what treatment have you tried?Anyhow with available information it appears to be perianal fistula, which is the most common pathology of this area. It is diagnosed by Digital rectal examination and prostoscopy. Fistulous Tract is evaluated by MRI Pelvis. Its treatment is surgical removal of the tract under spinal anaesthesia. Its relatively simple operation with good result, however outcome is dependent on extent of the tract and its relation to external sphincters. You need to see a general surgeon, who after examination will guide you regarding further evaluation and subsequent treatment.I hope my answer will help you. Do rate it if you like it.Regards"
},
{
"id": 107984,
"tgt": "Suggest treatment for upper back pain",
"src": "Patient: had a pain in upper back since last monday, feeling like trapped nerve, eases off for while, putting hot water bottle on when it more painfull, then last night app, 3am, the pain was in my chest, had to get up take paracetamol and re heat water bottle, propped up in bed, then eased off, wasn`t to bad when got up, as day gone on pain increased again in chest, was going to contact my gp, but it was too late, no appoitments. just to put my mind at rest,, as last night was scary, it felt like a heart attack, but i dont know what heart attack feels like, i will contact my gp in morning if pain continues. thankyou Doctor: Hello, I have studied your case. Due to compression of this nerve root there is pain in your back and chest.Due to occlusion of vessels there can be pain sensation.I will advise you to do MRI thoracic spine, HRCT [CHEST] and EMG- NCV [nerve conduction study]For these symptoms analgesic and neurotropic medication can be started.Till time, avoid lifting weights, Sit with support to back. You can consult physiotherapist for help.Physiotherapy like ultrasound and interferential therapy will give quick relief.I will advise to check your vit B12 and vit D3 level.Hope this answers your query. If you have additional questions or follow up queries then please do not hesitate in writing to us. I will be happy to answer your queries. Wishing you good health.Take care."
},
{
"id": 149983,
"tgt": "Dizziness, tiredness, stressed, itching eyes. Underlying causes?",
"src": "Patient: Got up quickly this morning at 5 AM to use bathroom. On the way back to bed I was extremely dizzy, room spinning. Even laying down with eyes closed everything was spinning for about 15-20 min. Spinning stopped but I felt dizzy and tired for the next 3 hours. Did not feel like myself until noon. Been under a bit of stress last 2 days and it was a rainy, foggy morning. Eyes were allergy itchy most of day. Feel fine now. Doctor: Welcome to HCM.This occurs due to some changes in vestibuler system in the ear it is called vertigo.In your case triggering factor is stress.You can take tab.betahisidine 8 mg for it after your doctor's advice.Try to avoid stress,take proper sleep,eat good nutritious diet.If it is recurrent than go for complete blood tests.Correct anaemia and other deficiency.Your blood pressure should be checked.you can use anti allergic eye drops.Consult your physician for perfect diagnosis and medical management."
},
{
"id": 65936,
"tgt": "What is the remedy for the cyst on the butt cheek in a 3 year old?",
"src": "Patient: Hello. My three year old son has a cyst on his right butt cheek. It started out as a bump but I noticed this morning that it is now a little bigger and it has gotten hard. I have tried a warm bath for him but what else can I do? I don t want to run to the hospital for every little thing. Doctor: Hi, it is nothing alarming but you should get it checked by your Doctor or a Surgeon. Only by examination would we come to know what it is. It is very difficult to comment upon what it is without examining it. You don't need to worry it does not seem to be anything serious as it would most likely be a Sebaceous cyst or a Lipoma. But it would be better to get it examined by a professional. Take care,Dr Rishi, New Delhi, India."
},
{
"id": 67717,
"tgt": "How to reduce lump above labia?",
"src": "Patient: Hi, I managed to hit the front of my pubic bone with a canoe two weeks ago. It bruised, was swollen and sore, and now the bruising is settling but there is still a lump above the right labia. The lump is just less than the size of a ping pong ball and is fairly tough (not very squidgy) as with other swelling. Is this part of the injury which needs time to heal? What can I do to help the healing process? Thanks! Doctor: Hi,From history it seems having hematoma formation following injury.Now hematoma is getting absorbing.Apply ice pack over the part.Gradually it will be fully absorbed.Ok and take care."
},
{
"id": 32315,
"tgt": "Suggest medication for joint pains due to chikungunya",
"src": "Patient: Dear Doc , have got chikungunya since last two months , the fever went away in the first 7 days but persistent pain in evry joint still remains and weakness upto an extent, is still very much prevelent , till when will i have to bear the pain and what to do about it, no medecine which i have taken is working pls help Doctor: Hi & Welcome.I can understand your concern for the joint pains following Chikungunya fever.Usually the residual joint pain following the attack of Chikungunya persists for some time and it responds well to anti inflammatory drugs like Ibuprofen or Diclofenac.Since it seems these drugs are not helpful in relieving the pain, then the possibility of inflammatory joint conditions like rheumatoid arthritis must be ruled out. You are therefore advised to consult a physician for certain lab tests such as CBC, ESR, C reactive protein, Rh factor, Anti-nuclear antibodies, Anti CCP antibodies to confirm the diagnosis.Meanwhile you may take tab Indomethacin for the symptomatic relief of pain along with Pantoprazole to prevent gastritis.Hope the suggestions given above could be useful to you."
},
{
"id": 24878,
"tgt": "What is the dosage of lisinopril 20 mg for BP of 139 over 111?",
"src": "Patient: I have been on 20 mg of lisinopril once a day for about 60 days. Blood pressure in AM's is about 139 over 85to 90. In the evenings it goes to 139 over 96 to 100. I have a doctors appointment in a week. I excersised tonight and 1 hr later it is 139 over 111 and 15 minutes later it was 139 over 95. Can I take one more pill per day? Doctor: Hello and thank you for using HCM.I carefully read your question and I understand your concern.I will try to explain you something and give you my opinion.You should know that when we treat hypertension our goal is to keep mean blood pressure values below 140 / 90 mmHg.A person might have high value during emotional and physical strees so its mandatory to judge on mean values.To judge if it is necesary to rise a medical dosage or add another class we should judge on mean values and other factors like age, cardiovascular associated rics factors and associated disease.The dosage of lisinopril might be added to 40 mg a day, so it is posible to take another tablet.But if I was the treating doctor I would recommend some examination like a cardiac echo, a full blood analyze and a holter pressure monitoring to evaluate your blood pressure values during day and night and than decide.Hope I was helpfull.Best regards,Dr.Ervina."
},
{
"id": 61699,
"tgt": "Suggest treatment for a painful lump on the hand",
"src": "Patient: I have a lump about the length of the palm of my hand and half inch above my knee cap. It didn t bother me before but since I started going to the gym 4 times a week for 2 hours a time I noticed it was restricting me. It starts to throb as I am working out. I have fh and I am taking rosuvastatin and ezetimibe along with colestogel I also suffer with pain in the hip on the same leg. I just want 2 check if it s something worth bothering my GP about or is it nothing. Would appreciate ur feed bak. Thank u x Doctor: hi.noted history of gym activity associated with lump in palm and over knee cap. these may be inflammatory reactions progressing to callous formation. try wearing protective gears, gloves and knee pads in your case. pain medications may also be of help. if you are bothered by the symptoms and is persistent, a consult with a doctor is best.hope this helps.good day!!~dr.kaye"
},
{
"id": 161182,
"tgt": "What causes lump under the skin of mons pubis?",
"src": "Patient: My six year old complained for the first time today of what looks to me like a lump or small growth under the skin of the mons pubis. It is painful and has a soft feeling to it as though it is filled with fluid. What would this be? Doctor: Hello, By what you say I feel that this could be - 1. Dermoid cyst. 2. Lymph node enlargement. I suggest you discuss this possibilities with his pediatrician and get him appropriately evaluated, please. Hope I have answered your query. Let me know if I can assist you further. Take care Regards, Dr Sumanth Amperayani, Pediatrician, Pulmonology"
},
{
"id": 176012,
"tgt": "Is swollen liver with moderate to severe pain in stomach of a child while having mono concerning?",
"src": "Patient: My 12 year old son has mono, his pediatrician advised me that if he s belly pain worsens that I should have him seen. His Liver is a little swollen and had elevated counts (not sure if I worded that right). This afternoon he is having how he outs it moderate to serve belly pain. Is this something that I should be concerned about? Doctor: Hi,Yes dear, as liver enlargment could be caused by increased intravascular red cells death or obstruction of liver vessels or even infection might be causing enlargement.More it can be evaluated if seen in person by a treating physician.I would highly recommend you to alert your son's treating doctor about his symptoms and signs promptly.So it can be treated on time.Wish you best of luck.Regards, Dr.Maheshwari"
},
{
"id": 100460,
"tgt": "When is serious treatment for asthma needed?",
"src": "Patient: I am 18, just got told I have asthma this year. I am on Asthmanex and Foradil and Ventolin. I was wondering when I should consider my asthma bad enough to go to er. Most times I put it off and say i'm fine. I dont have a peek flow meter or anything so I just wondered.Thanks Kristen Doctor: HelloSince you are an asthmatic patient and taking asthmanex, flotral ( foradil ) and ventolin . All these are bronchodilator and anti allergic .This indicate that you are having asthmatic allergic bronchitis and for your kind information this is a very common disorder in modern life due to so many allergen surrounding around us . These allergens are : Pollen , mite , fine dusting powder, sudden fall and rise of temperature , mold , pests ( cockroaches ) pets ( cats and dogs in home ) fumes , smoking ( active and passive ) .As all these are predisposing factors so avoidance help great.Do these measures :Wash all bed covers in hot cycle every week.Treat carpets w/acricides or tannic acid.Keep dust-accumulating objects in dust cup board.replace curtainswith blinds or easily washable curtains.Use vacuum cleaner ,Encase mattress , pillows & quilts in impermeable covers.Install air- conditioner or dehumidifier .When such patient visit I only prescribe them to take montelukast+ bambuterol one or two tablet in day . All patients respond this treatment very nicely . No medicine except this.Hope this will help you.Take whenever necessary , stop once relief , recurs , use again.In my opinion there is no need of ER .Good luck."
},
{
"id": 40331,
"tgt": "Family diagnosed with coxackie herpangina. Is it contagious and serious?",
"src": "Patient: Hi. I have my 3 instance of coxackie herpangina (diagnosed by visual inspection of doctor) in 5 weeks. Is there a next step to suggest. I am homebound and my husband and son caught in 4 weeks ago and are ok at this point with no recurrance. How serious is this. Should i stay away from family this christmas? Doctor: HI,Welcome to HCM.Herpangina is a self limiting severe febrile pharyngitis, sore throat with discrete vesicles on posterior half of palate, pharynx, tonsils and tongue. Infection does spread by contact and must be avoided as much as possible.Avoid sharing of towels and linen with family members.It will reduce by 10-15 days.Thanks."
},
{
"id": 206943,
"tgt": "Could the severe pain in head be due to stress?",
"src": "Patient: i'm not the one that has this problem... but a guy that i know, getas stressed alot, and he would hit his head on something, or even punch it. he went to the doctor because his head felt weird and they said that he needs to stop getting stressed and all that stuff. after that something happened with him and his girl friend, and he did it again. now he says the pain in his head comes very painful, and he says his head feels \"stupid\" and its a very sharm pain. it comes and goes. what can it be? Doctor: diagnosis:tension headachesomatic complaint.if associated vomiting too can be migraine.treatment:tab amitriptylline 25 mg dailyhe will be fine in week. kindly visit a local psychiatrist for the drug as you will get it only on prescription.hope my answer helped you.take care.please rate the answer."
},
{
"id": 98131,
"tgt": "Leg pain and on calcium supplements. Had side effects. Will ayurveda be better?",
"src": "Patient: I am Madan kumar From Bangalore,I just wanted to brief you the situation about My Mom Leg pain she is 50 yrs old and she has the leg pain for several years,We had consulted many doctors and she underwent many types of treatment but she has not cured at all ,The problem is her legs below the knees are painful she told me that the bone is not painful but the nerves do She could not stand for atleat 5 min and also she has pain in her elbow (Hands)now and sometimes she has the full body pain She was eating the Calcium Tablets from AMWAY,It was okay but she got the side effectes she got ayurveda ,Homoeopathy,etc but still no improvement,she hate eating tablets nowadays and also she is afraid and exhausted to visit the hospitals , I dont understand what to do? I need a suggestion from you Doctor: hello sir , I understand from what situation you and your mother is going through , sir actually at this age our supporting or constructing cells become lesser and destructive cells increased ,and that is the problem due to which she do not have improvement , but nothing to worry , sir i don't know from where you took last homeopathic treatment but in homeopathy we treat such problem with ease , only if we do classical homeopathy , in which we took detail case of your suffering , past sufferings and as well as your nature , because in homeopathy there is no medicine based on this for that disease concept ,instead on the whole history we find out new medicine for every new case , and another advantage of this is no bitter pills and non confusing dispensing , so that patient get relieve on simple manners, so please visit to good classical homeopath once and see the difference,with best wishes for your mother's health , thank you."
},
{
"id": 87040,
"tgt": "Suggest treatment for pain in lower right abdomen",
"src": "Patient: I have pain in my lower right abdomen that is radiating toward my belly button it is a sharp stabbing pain that has been there about 8 hours. I also have blood when I use the bathroom on the toilet tissue. Is this something I should go to E R over or can this wait until next week when I go to the doctor. Pain is moderate nothing I cannot handle and is made worse by standing straight up Doctor: Hi.Thanks for your query.Also thanks for the very specific history.The most probable reasons for such a pain that sharp and stabbing in the right lower abdomen and radiating towards belly button with blood on toilet paper is suggestive of :-Stone in the right ureter or -Appendicitis. This can be easily diagnosed on clinical examination by a Surgeon and a simple ultrasonography.Get this done along with routine blood, urine ans stool tests and the diagnosis will lead to a proper treatment ."
},
{
"id": 113572,
"tgt": "Lower back pain, difficult to sit, stand or walk. Advised Zerodal. Any side effects ?",
"src": "Patient: I m 40 yrs old male, having lower back pain from last week, on fist day it was so painful and difficult to sit , stand or walk, so I took 5 days complete bed rest (sleeping on back), after that visited doctor and he gave Zerodal TH 8 20 tablets for 10 days. So I would like to know that is there any side effect of this tablets, is so please explain. Thanking you Doctor: Zerodol contains Aceclofenac which is a NSAID(non steroidal anti inflammatory drug). Most common side effects are on stomach ie gastritis, gastric erosion and bleed. Not to be taken empty stomach and antacids may be added."
},
{
"id": 24096,
"tgt": "Suggest treatment for high blood pressure and lower abdominal pain",
"src": "Patient: Thank you for being there. I am 70 and fairly active with low=level high blood pressure and not much else major wrong with me. Two days ago I began having pain in my lower left abdomen, which is like an ache, not like a stabbing or prickly pain. It s worse when I try to have a bowel movement, which I have, however, been having. So I m not stopped up in that way. It doesn t get worse when I exercise with a rowing machine or recombinant bike, but sometimes I notice it when I stand up or sit down. I m concerned that I might have something wrong in my bowels. Sometimes, I feel it near my belly button. I m also not as energetic as normal. Doctor: Hey,With the symptoms you are telling, you should get a ultrasound of abdomen done. But it looks more like muscular pain. Try taking antispasmodic drugs & get ultrasound done.ThanksDr Sameer"
},
{
"id": 224779,
"tgt": "Is it needed to take the morning after pill after pre-ejaculation?",
"src": "Patient: I have taken my birth control twice this week late, I usually take the pill at 9pm every night but wednesday and thursday I took it at 1:45am. I then had unprotected sex, last night where precum could have gotten in me but we never went all the way through. Should I get the morning after pill? Doctor: Hi, As you have taken oral contraceptive pill everyday so there is no need to take other contraceptive measures . You should take OC pills regularly but if you miss a pill then take double dose next day. If two consecutive days are missed then another measure of contraceptives should be sought out. Avoid drugs like rifampicine , macrolids , tetrcycline which decreases its efficiency . Even it has complication including failure rate( but less than others ). Hope this ll helpThanks n regards"
},
{
"id": 114219,
"tgt": "Could a back pain and short of breath be symptoms of asthma ?",
"src": "Patient: I woke up two days ago with back neck pain and pain between my shoulders. No fever at all. Last night I began coughing and coughing up phlegm . Today I am short of breath . I have never had this problem before. Could it be asthma? Doctor: No its not Asthma, Its simple Viral Pharyngitis with Bacterial superinfection. Take some antibiotics, warm saline water gargling and some cough expectorants. u will be alright by 3 days."
},
{
"id": 28788,
"tgt": "How can high fever and throat infection in a toddler be treated?",
"src": "Patient: My 2 yr son is having fever since 5 days. Dr said its a throat infection. Started with clamp kid forte suspension. Also given Tylenol or Advil in between. It s been 4 days since the medication. His fever isn t coming down. Can the antibiotic itself cause fever.? Fever ranges from 38.5-39.6 Doctor: Hello,Antibiotic itself doesn't give fever, so I don't think antibiotic is the reason. Infection sometimes continues giving temp although you have begun you antibiotic. I will suggest to wait for 2-3 other days and then to see your doctor. In case the temperature lasts more than 6-7 days when we are using antibiotic-therapy then we may have an antibiotic resistance and need to change treatment. During that time use plenty of water, healthy diet, fruits, vitamins to help him get better. Hope I have answered your query. Let me know if I can assist you further.Regards,Dr. Elona Dashi"
},
{
"id": 141292,
"tgt": "What causes pain at the base of the neck and skull region?",
"src": "Patient: I am having a sharp pain on the left side of neck then I get this very hot feeling that goes into the left side at the base of my brain feels like someone injected a hot liquid into my brain. It only last s for about 5 to 10 seconds. Could it be a warning for a blood clot? thank you Doctor: Hello and Welcome to \u2018Ask A Doctor\u2019 service. I have reviewed your query and here is my advice. Possible causes for nape of the neck pain is muscle strain, occipital neuralgia, herniated disc, whiplash, spondylosis and spinal stenosis etc. Until examination is done it is difficult to say what it is. Use cervical soft collar, use cold and warm compression, use pain killer medication. If symptoms not improved please consult your doctor he will examine and take x-ray of neck and treat you accordingly. Hope I have answered your query. Let me know if I can assist you further."
},
{
"id": 67158,
"tgt": "What causes painful lump and bruise at the upper part of back?",
"src": "Patient: Hi, a few days ago I started having this pain and a tender spot on the very upper part of my back on the top part of the spine. Pain got worse and is now a lump with a bruise on it. Went to GP and she gave me pain meds and prednisone. X ray came back normal. What could this be? I don t recall being hit at this spot. I am a swimmer. Is a possible I overdid it and tore a muscle that is now bulging out???Thanks!! Doctor: HiWelcome to hcmYou must be having abscess (pus). Cause can be infected sebacious cyst or tubercular abscess in either case treatment is drainage of pus for which you have to see a surgeon. Antibiotics tablets only will not help. Prednoson tablet should not be taken preferably. If the abscess tern out to be tubercular/TB then further evaluation of spine/back bone is required along with Tab. for TB.Regards"
},
{
"id": 153444,
"tgt": "Can taking oral holistic medicines,chemo and Avastin together cure colon cancer?",
"src": "Patient: I want to consult an expert oncologist in the context of my brother's colon cancer with metastasis on liver.He is now treated with Avastin in TVM. I want to know whether holistic medicines orally taken together with the chemo and Avastin could help to improve his condition.? Menon Doctor: Hi, dearI have gone through your question. I can understand your concern.Your brother has colon cancer with metastasis in liver. So it is already stage 4 cancer. Chance of cure is negligible. Chemotherapy help as palliative treatment. Despite of treatment prognosis remains poor. Life expectancy is not good. Sorry to say but it is the fact. Consult your doctor and plan accordingly.Hope I have answered your question, if you have any doubts then contact me at bit.ly/Drsanghvihardik, I will be happy to answer you.Thanks for using health care magic.Wish you a very good health."
},
{
"id": 222554,
"tgt": "Is delayed menstruation a sign of pregnancy?",
"src": "Patient: hI! I AM A 25 married girl. I missed 6 days of my period this month. Actually I have irregular menstrual cycle, but the average ccycle should be 31 days. The first day of my last period was on Sept 2, so the next period should be on Oct 3. However, I took a strip pregnancy on OCt 3 but the result was negative Further, I felt cramps but no blood come out How do you think? am I pregnant? when is the best day to take next pregnancy test? Doctor: HelloThank you for ur queryDear.. amennorroea is one of the major signs of pregnancy. You can do the next pregnancy test on the 40 the day. As you have irregular menstruation some times ur periods may be delayed. Dont worry just wait for another month and mean while check for pregnancy .Regards"
},
{
"id": 89241,
"tgt": "Could the abdominal pain be due to elevated lipase levels?",
"src": "Patient: I was suffering from Acute pancreatitis last month. I had upper abdominal pain which lasted for 2 days and after medication it disappered. I was admitted for a week. I had my CT & Sonography done. All my reports were normal except my lipase levels. It keeps on fluctuating. Doctor: Hi. The upper abdominal pain with raised lipase levels is suggestive of Pancreatitis. Get a Repeat CT scan or MRCP and you may need ERCP and stenting of the pancreatitic duct to relived the sphincter of Oddi dysfunction too. This may very well help. You also have to take the pancreatic supplements to keep the pancreas acting least possible,'Also get blood sugar tests done as you are a candidate for development of diabetes. As also go for liver function tests, kidney function tests as these may be deranged"
},
{
"id": 153346,
"tgt": "Suggest treatment for severe BPH symptoms",
"src": "Patient: I am a 63 year old man with severe BPH. For 20 + years I have had TURP numerous procedures. I have been taking Avodart and Uroxitral for many years with no noticeable results. In 2009 I was diagnosed with Squamous (sp) cell Colorectal Cancer. I underwent chemo and radiation in the late spring of 2009. After going in for follow ups for a little over a year it was discovered that the treatment protocol in 2009 had not worked. I underwent Colorectal Surgery in the fall of 2010. At the time of surgery I was told that they had good margins and felt very confident that the cancer had been eliminated. HOWEVER; I was then told that the cancer had eroded through the colon wall and attached itself to my ureter. They felt a little less confident in that area and I have been doing 90 day follow ups to check me out since the surgery in 2010. During this time I have had 3-4 additional TURPS that work for about a month and then I am back to severe BPH symptoms. TUPRS and medications are not working. What are my other options? Doctor: Hi, dearI have gone through your question. I can understand your concern. You have history of colon cancer and it may spread to ureter. You should check for cancer spread first. You should take drugs like prazocin or terazocin for your benign prostatic hyperplasia. If drugs are not working then surgery is the treatment of choice. Go for surgery to remove the obstruction. It will releive your symptoms. Consult your doctor and take treatment accordingly. Hope I have answered your question, if you have doubt then I will be happy to answer. Thanks for using health care magic. Wish you a very good health."
},
{
"id": 43113,
"tgt": "Trying to get pregnant, hsg test shows tubes open, HPT negative, no periods yet. What to do?",
"src": "Patient: hi doctor, I am 26 trying to get pregnant for a yr.i had a hsg test on last month 22nd my tubes were open now.this mnth my menses delayed so i had a pregnancy test bt it came negative . still my period doesnt start yet. please help me what i have to do. Doctor: Hi,Welcome to HCM.Home pregnancy test (urine test) can come negative in early pregnancy. Kindly do one blood test (beta HCG), which will confirm OR rule out pregnancy with 100% confidence. In case it comes negative, still you need not worry as occasional period can delayed in normally ovulating woman.Wish you good health.Regards."
},
{
"id": 94476,
"tgt": "Having lower abdominal pain. Unable to eat or drink without pain. Looking for remedy",
"src": "Patient: I ve been having lower abdominal pain for past couple weeks now. I cannot eat or drink anything without severe pain, not even water. The first occurrence happened back in September of 2012, every since then I ve been getting sick ever week with that lower abdominal pain or the flu and even gotten strep/mono for 2 weeks. I ve lost 15 in probably one to two months. My doctors couldn t figure out what was wrong and suggested I d go to the hospital. Is there anything to do to get rid of this? Doctor: Hi ! Thanks for asking ! The symptoms you had describe has many reasons. If you want correct diagnose need full abdomen ultrasound and contrast abdomen X rays. It will clear your doubt. But for now take antispasm tablet for pain. Take care."
},
{
"id": 25006,
"tgt": "What causes heartburn?",
"src": "Patient: for the past week and a half i have started to get heartburn everyday for no reason i will wake up with it or be sleeping and it wakes me up or anytime during the day and i never get heaartburn ever. i am healthy active 27 year old femail i go to the gym and i dont eat fried food or food that is spicey. what can be the cause of my heartburn? Doctor: Hello and thank you for using HCM.I carefully read your question and I understand your concern. You should not worry about. I will try to explain you something and give you my opinion. There are some reasons that might be responsible for hearburn.First, you should know that typical heart pains have some characteristics.They usaly are retro-sternal pains like squizzing , tightness or heaviness. They may irradiate to both arms , neck and jaw. They last from 5 to 20 minutes. Long lastig pains are not typocal heart pains, and in your age ischemic heart disease is rare.To exclude ischemic heart disease I would recommend a cardiac echo, a full blood analyze and a strees test. Another reason of heartburn migh be a gastrointestinal problem. So you should consult a gastrohepatologyst and maybe a fibrogastroscopi.Also you have to exclude a skelectic problem doing a toracal xray.After all this we can better judge what we are dealling whith and how to treat it. Hope I was helpfull .Best regards, Dr.Ervina."
},
{
"id": 110109,
"tgt": "Suggest treatment for spots on cheek with sinus & back pain",
"src": "Patient: I have a spot on my cheek that feels cold, I also have an uncomfortable feeling in my neck/throat, maybe like a tightness or that someone has hit me there, having said this there is no pain just slight uncomfort. I have sinuses and a history of neck and back problems. Doctor: HelloThank You for contacting HCM.Is there any history of allergy or insect bite? There could be two possibilities First, an allergic response to dust, pollen, particular food or insect bite. Second possibility is Acute Sinusitis. I would suggest you following things:> Firstly, if this is because of allergy then you need to take care in future to avoid such food.> Take acetaminophen for pain> Take hot water in a cup, add 1 table spoon salt in it. Perform gargles (mouth wash) with it three times a day for at least 3 days. It will give soothing effect.AVOID:> Chilled water> Drinks> Spicy foodIf condition does not start improving in 2-3 days then antbiotic therapy can be started.Hope this answers your question. If you have additional questions or follow up questions then please do not hesitate in writing to us. Wishing you good health."
},
{
"id": 104684,
"tgt": "Sneezing frequently while at work. Taking advil daily provides relief. Permanent cure for allergy?",
"src": "Patient: Hi I am a 45 year old woman. For the past three years or so I can t seem to stop sneezing every so often, mostly at work, I do sneeze at home as well, but not as much. I find that the normal allergy medication dosen t work, the only thing that seems to work is Advil , so I m taking Advil on a daily basis and don t really like that idea. I did get checked for allergies but nothing major showed up. Could you suggest what I could do or take to help me stop sneezing continously please. Doctor: Hi, Sneezing could possibly be due to an allergen for sure. Its possible you could be getting relief from Advil. However, Advil is Ibuprofen, a non steroidal anti inflammatory drug/ Treatment of allergies require medical and environmental/lifestyle modifications. It could be worth knowing whether you changed your room, office or are constantly exposed to dust, pollen or cold. If any of these symptoms aggravate the disease, then could try as much as possible to avoid it. Secindly, drugs do exist, ranging from topical antihistaminines, through systemic antihistamincs and corticisteroid. Dosage and mode of administration depends on severity of disease and cormobid conditions. Consulting your internist or allergologists for appropriate testing and management could be worthwhile. Allergologists also have specific immune depressing therapies in case commonly used drugs are not effective. It could help consulting an allergologist. Thanks for understanding and best regards, Luchuo, MD."
},
{
"id": 167603,
"tgt": "Suggest remedy for floppy limbs in 2 week old baby",
"src": "Patient: My 2 week grandson has good head control, a good grip, but his limbs seem floppy as though there is no bone there. Birthweight 7lb 14 ounze. Normal delivery. Low blood sugar levels for a couple of days but these were corrected after his first bottle feed ( mother unable to breast feed. Is this normal? Doctor: h I madam, if you feel these floppy limbs are very hypotonic ( no tone) in which the knees almost touch the table by their own when lying on back ; then he might have congrntial hypotonia and this must be carefully evaluated by the pediatrician, I recommend taking him for fully examined .I hope this is helpful"
},
{
"id": 37788,
"tgt": "Should my girlfriend worry about eating a crunchy banana?",
"src": "Patient: Hey, So my girlfriend took a bite out of a banana and found it was crunchy. Upon further investigation we found that the middle was a crimson red color, was hard, and crunchy! She s worried about it, does she have anything to be worried about? She didn t eat the rest of the banana. Doctor: Hello,Thank you for your contact to healthcare magic.I understand your health concern, if I am your doctor I suggest you that there is nothing to worry. You should take care of any abnormal symptoms. If you find such symptoms kindly visit to ER. Such a small piece of wrong banana won't cause any health hazard usually. I will be happy to answer all your future concern. Thank you,Dr Arun TankInfectious disease specialist.Wish you a best health at health care magic."
},
{
"id": 24695,
"tgt": "What causes swollen eyelid and jaw and gums numb?",
"src": "Patient: i have a black 47 year old man with prostate cancer hes had a stroke almost two years ago he has a bad heart,kidneys and liver but now he woke up yesterday with his left eyelid swollen and jaw and gums hurt and feels numb and the right side of his head hurts Doctor: hello there I understand your concernI feel there has been an embolic phenomenon with your man and he needs to be shown in an ER.A CT scan will verify the same hope that helps"
},
{
"id": 52551,
"tgt": "What causes high blood sugar level despite taking Metformin while having acute pancreatitis?",
"src": "Patient: Hello, I am a male 58 years old. I had acute pancretitis in 2014 so since then I have been on metformin and glimepride but the last moth and a half it has been hihg from 200 to 400. What may be causing the change it was under control and diet has not changed. I see my Dr tomorrow but just want to hear from you. Doctor: Hello and Welcome to \u2018Ask A Doctor\u2019 service. I have reviewed your query and here is my advice. Pancreatitis can damage the insulin producing cells of your pancreas and leads to a state of low insulin and may leads to diabetes. In that case you may require insulin injections to optimize the blood sugar level. Do a fasting blood sugar level and start insulin accordingly. Wishing you good health."
},
{
"id": 84881,
"tgt": "What dose of asthalin inhaler should be given to child instead of ventorlin?",
"src": "Patient: hi my 2 yrs daughter is diagonised to have lower respirstory infection viral. i am adviseed to give ventorlin 3.5 ml 3 times . but she is very fussy with medicines , so can i give her asthalin inhaler thro huff puff kit , for that what should be the dose Doctor: Hi,Actually asthalin and ventolin inhalers contain the same active ingredient of salbutamol which is an effective bronchodilator (opens the airways) for the symptomatic relief of bronchospasm. Inhalers are usually best advised to children of 4 years and above and for younger children nebulizer is the best mode of drug delivery because it gives continuous medication (salbutamol). Since your daughter is not comfortable with ventolin respiratory solution (nebulization) hence asthalin inhaler with huf puf kit can be used to deliver the drug. Each puff delivers about 100 microgram of salbutamol so the usual dosage for children under the age of 12 years: up to two puffs 4 times daily.Hope I have answered your query. Let me know if I can assist you further. Regards, Dr. Mohammed Taher Ali, General & Family Physician"
},
{
"id": 36172,
"tgt": "What is the diet and how long to continue medication in case of stray dog bite?",
"src": "Patient: I had a stray dog bite today morning at 11 am but eventhogh there was no wound there was a scar only on surface went so see doctor and he told to take Rabipur starting from Today till Jan 26 so with that course can i Stop Medicines or have to take any precautions,should i have to control my diet as doctor didn't told about it.if so how many months and any problems or complications Doctor: Hi,Nothing to worry, go for regular ARV shots as advised by your doctor.Take tetanus toxoid shot.There is no need of stopping any medicine if you are taking.No diet restriction,you can take your regular diet.Ok and take care."
},
{
"id": 80084,
"tgt": "What causes weird chest pain?",
"src": "Patient: My wife has had her picc line in for three months now and every thing has been pretty normal with it until last week. She was vacuuming and felt an odd pain in her chest that has happened about once a day ever since. Is this something we should be concerned with? Doctor: Thanks for your question on Health Care Magic. I can understand your concern. In my opinion, we should first rule out cardiac cause for her chest pain. So better to get done 1. Ecg 2. 2d echo. If both are normal then no need to worry for cardiac chest pain. Musculoskeletal pain can be the cause for her pain due to exertion with vaccum. So avoid heavyweight lifting and strenuous exercise. Avoid movements causing pain. Apply warm water pad on affected site. Hope I have solved your query. Wishing good health to your wife. Thanks."
},
{
"id": 154347,
"tgt": "Suggest treatment for tumor in right temporal lobe",
"src": "Patient: my 24 yr old son has a tumour in his right temporal lobe, about the size of a ping pong ball, based on an MRI; he is special needs, non-verbal, and has a chromosome deletion ( 13q- ); he has been having increasing headaches for 1 1/2 yrs; he is disinterested in his favourite activities; his sleep is interrupted; he is eating less; his mother died 18 yrs ago from anaplastic astrocytoma, so maybe a hereditary factor?? what is his prognosis?? - I ve heard 1 - 2 yrs but read cases where the patient was still living 10 yrs later Doctor: Hi, dearI have gone through your question. I can understand your concern.Treatment depends on type and stage of tumour. His mother has anaplastic astrocytoma so he also has chance of hereditary factors. Prognosis also depends on type and stage as wall as grade of tumour and general condition of patient. Treatment options are surgery and radiotherapy. Consult your doctor and take treatment accordingly.Hope I have answered your question, if you have any doubts then contact me at bit.ly/Drsanghvihardik, I will be happy to answer you.Thanks for using health care magic.Wish you a very good health"
},
{
"id": 152493,
"tgt": "How can elevated calcium levels in blood be treated?",
"src": "Patient: Hi am 34 year old and dignosed with breast cancer in year 2010, last year my Ca 125 increaed at 40..i wss oncfident that it was because of stress and unhealthy life..i took all precautuon for one entire month and it came down to 9..currently again my ca 125 level hone up to 35.44..but when i did this test durong that time was undee teemendous stress...pls suggest Doctor: Hello and Welcome to \u2018Ask A Doctor\u2019 service. I have reviewed your query and here is my advice. All levels are still withing normal range. However, I will suggest you to get an ultrasound of abdomen done to rule it out. Hope I have answered your query. Let me know if I can assist you further."
},
{
"id": 33218,
"tgt": "What causes headache , chill and fever in children?",
"src": "Patient: Hello. My 5yo daughter has complained of a mild headache all day. After being out in the heat (off and on, no more than 2hrs at a time), putting her to bed, she has complained about a stronger headache (more in the back of the head) and being extremely cold. We took her temp after her shower and it was 102.5. We will retake it in a few minutes. While she does have allergies she is not stuffy at all and has had no diarrhea or vomiting. Any suggestions as to what it might be or how serious? Thank you in advance! Doctor: Hello..i would suggest to give some paracetamol suspension..and as it is a high grade fever and there are no relevant symptoms i think you should take her to see a G.P and run some blood tests to know what is causing this high grade fever."
},
{
"id": 177863,
"tgt": "What treatment is suggested for colic?",
"src": "Patient: Hi my 1 year old son had colic as a baby. we gave him lactose free formula milk that helped a lot. Later we gave him Similac easy digest which worked very well. Now I want to switch to whole milk, and gave him parmalat first growth milk. He had bad stomach cramps and gas. Can I give him Nido or what must I give him? the Similac formula is very expensive. Doctor: As your baby is 1 yr old and he might have intolerance to lactose or any other milk contains. So you should consult a pediatrics first and do proper test to find out the reason.Till then avoid any milk or milk products..Get well soon..."
},
{
"id": 218302,
"tgt": "Is slight brownish vaginal bleeding during the 7th week of pregnancy a cause for concern?",
"src": "Patient: Hi, I got pregnant with the IUD in situ, I had it removed as soon as the blood test showed positive. I had a tetr blood test and the numbers doubled from 640 to 1440. I had my first ultrasound last week, and it showed a baby sac, and good heartbeat, but also evidence of retro-decidual collection in front of the sac about 1.5 cc . The cervical canal is closed, and i experienced no bleeding except for a light brownish discharge only once after intercourse. I don t know what does that mean, and should I take any medications? because I actually feel it s better to leave things as they are without the interference of medication. Am I right or is my case really risky and need medication? Also should I quit my job and stay home at bedrest? I am 7 weeks pregnant now Doctor: Hi, Looking at what you have described, your beta hcg levels doubling and early scan showing cardiac activity, there does not appear any reason to be worried about. You should consider this as a normal pregnancy so far. So, no bed rest and no medication except folic acid is required at this point. Have a routine 12 weeks scan and let's see how things are. Hope I have answered your query. Let me know if I can assist you further."
},
{
"id": 166884,
"tgt": "Suggest treatment for vomiting and loose stools in kid",
"src": "Patient: I have a two year old daughter that began vomiting on Saturday night and although she is now no longer vomiting and has only had a couple of loose stools, over the last couple of days when changing her nappy she has had a light brown discharge in the front of her nappy. She also has a bad cough. Any ideas? Doctor: Hello and welcome to \u2018Ask A Doctor\u2019 service. I have reviewed your query and here is my advice. For vomiting give her an over the counter antiemetic (other than primperane as its contraindicated in children). For loose stool give an anti diarrheal drug and oral rehydration solution 5 tea spoons with each diarrheal motions to avoid dehydration. For cough give a cough suppressant and expectorant.Hope I have answered your query. Let me know if I can assist you further.Regards,Dr. Salah Saad Shoman"
},
{
"id": 141284,
"tgt": "Is epilepsy curable?",
"src": "Patient: Hello doctor, i want to ask that, is epilepsy completely curable from alopathic medication? as i m getting seizures from past 4 years, in this period i ve been medicated through alopathic medicines, after loosing hope i turned to ayurvedic medicines since 2 months, it is controlling my seizures but idk whether it can completely cure me, i ve faced many problem in studies till, i m 21 years old now and wanna ask whether cmc vellore have complete cure of epilepsy? what may be the cost? and if not then from where i can get the complete cure of this diseases..? please answer me in detail. i shall be oblighed if you will help me... Thank you! Doctor: Hello and Welcome to \u2018Ask A Doctor\u2019 service. I have reviewed your query and here is my advice. It is depends on type of epilepsy, associated with any organic problems it is curable. Example temporal lobe epilepsy present with medial temporal sclerosis which treated with temporal lobectomy. Most epilepsy is treated with medication. Drugs do not cure epilepsy, but they can often control seizures very well. About 80% of people with epilepsy today have their seizures controlled by medication at least some of the time. Of course, that means that 20% of people with epilepsy are not helped by medication. Please give proper history of which type of epilepsy are you suffering from associated problem and EEG report etc. Please consult your doctor he will examine and treat you accordingly. Hope I have answered your query. Let me know if I can assist you further."
},
{
"id": 158214,
"tgt": "Severe pain in hip and back, regular headache. Biopsy positive for prostate cancer, had radical prostectomy. On lupron. Suggest",
"src": "Patient: Dec. 2010 I had a turp and the biopsiy was positive for prostate cancer. My urologist started me on lupron. In march 2013 I had a radical prostecemy. My gleanson score was 5+4. I have six days left of radiation and have had two more lupron shots. My hip and back have really started hurting. I have daily headaches. Should I stop taking the lupron. Doctor: Hello user,I don't think you should stop lipron as it reduces testosterone production in body and helps in reducing size of prostate cancer, and reduces further recurrence of left out prostate tissue into full blown cancer.You are rather advised to have a MRI/CT scan of your back and hip region to rule out metastasis of prostate cancer to these regions !!Hope I have answered your query. If you have any further questions I will be happy to help.Regards,Dr Hari Om ChandrakarMBBS"
},
{
"id": 58203,
"tgt": "Have liver cirrhosis and dark urine. What are the findings from the liver function test?",
"src": "Patient: i have liver cirosis 4 time chemoembolisation done many time done lft test every thing under control but last week i had some problem dark urine no proper sleep body ache ect on 2nd Nov done lft everything shoot up. S.bilirubin (total) 9.5. S.bilirubin ( Direct) 6.9. S.bilirubin ( indirect) 2.60. S.G.O.T 140.SGPT 140. S.Alkaline Phosphatase-275.0 Total Protein 7.0 Albumin-2.3.Globulin- 4.70. A/G Ratio-0.49 Doctor: Hello and welcome to HCM,The liver function test report suggests deranged liver functions.There is elevation of total, direct and indirect bilirubin.The liver enzymes- SGOT and SGPT are also raised.The rise in above mentioned parameters suggest damage to liver cells (hepatocytes).The damage to hepatocytes can ossur due to a number of causes eg. infection, drugs, toxins, etc.An increase in alkaline phosphatase suggests some pathological process in the extra-hepatic biliary tract.A previous history of liver cirrhosis indicates a previous insult to the liver.Consult your treating doctor for liver biopsy, diagnosis of the current pathology in the liver and further management.Thanks and take careDr Shailja P Wahal"
},
{
"id": 132472,
"tgt": "Whats the treatment of leg pain ?",
"src": "Patient: I have constantn leg pain. Especially in my right leg. My leg at times feels like it is numb, at the same time it feels tingly, burning, cold. All while my legs hurt. I have RLS, have had for many years. I have only felt this pain in my legs for about 7 months. The pain is constant. It does not occur after walking. I do have cramps in my right calf quite often while sleeping. I mentioned this to my doctor and she gave me a sample of Lyrica, which has made no difference. My leg appears to be normal in color. I can t stand the pain much longer. Doctor: HiHope this message finds you in good health. I have gone through your message and understand your concern. u seem to have lumbar disc disease causing the problem. get an mri and xray to find out whats wrong.analgesics. anti inflammatory tablets. hot fomentation may help. I hope your query has been answered. Feel free to consult me anytime.Thanks. Take care. God bless"
},
{
"id": 188049,
"tgt": "What is the remedy for removing stains caused by chewing gutka?",
"src": "Patient: I have gutka stains on my teeth. Now that there is a ban on gutka i have almost quit chewing it. I am also a smoker and ready to quit smoking too. How can i remove the stains and whiten my teeth. Please refer a dentist in Bangalore who charges affordable cost for the treatment. Doctor: Hello,Stains due to gutka chewing can be removed by scaling of teeth.Please discontinue tobacco or gutka chewing thereafter.Maintain oral hygiene well.Regarding the dentist and treatment charges in banglore,please do consult with your dentist nearby your hometown.You can discuss with your dentist regarding this.Use flouridated toothpaste as well as chlorhexidine mouthrinses to gargle.Take care."
},
{
"id": 146657,
"tgt": "What does a disc bulge with indentation on the ventral cord mean?",
"src": "Patient: What does it mean to have a small broad-based central disc bulge at T2 - L1 Level with mild indentation of the ventral cord and 2. small bbroad based central disc bulge at L5-S1 level mildly effacing the ventral thecal sac. There is a mild bilateral neural foraminal steonsis at this level from facet arthropathy. Doctor: You are having prolapsed intervertebral disc though site you have mentioned is unusual. Continue physiotherapy. Avoid bending forward and lifting heavy weight. If pain is troublesome then start tab pregaline od and analgesics sos.Hope my advice will help you. Take care. Don't forget to rate me."
},
{
"id": 84503,
"tgt": "Does dolo 650 has any side effects?",
"src": "Patient: Hello doctor, Goodmorning to you!. My question with related to the Dolo 650 mg medicine.Has it got any side effects?.Since Iam using this medicine whenever I get fever.(2 to 3 tablets daily.)Is it safe ?,becuase I come to know that prolonged use of this medicine can cuase liver damage. Please adivse... Doctor: Hi, It isn't so much long term use as it is OVERDOSE. One dose or even 3 a day isn't likely to be bad if spaced out of the entire day. Six at a time is a serious risk of fatal damage to the liver. Six over a month is certainly safe. Hope I have answered your query. Let me know if I can assist you further. Regards, Dr. Matt Wachsman, Addiction Medicine Specialist"
},
{
"id": 203609,
"tgt": "Can a fissure cause bleeding after ejaculation?",
"src": "Patient: my boy friend had some bleeding after ejaculation, he went to the dr and they said he had a fissure. and they are talking about surgery. he has had past surgery for hemmaroids. i was just curious to know how a fissure can cause him to bleed when he ejaculates? Doctor: Hello,It is my privilege to be able to help you,I have reviewed your boy friends history as he had bleeding after the ejaculation he might me having some problem related to the foreskin of the penis.It may be a scratch or fissure.I would suggest you to take him to a Urologist where he can identify the exact cause and find out the reason for the bleeding.Then he can decide whether he requires surgery or not.Thank you"
},
{
"id": 42820,
"tgt": "What causes infertility while having normal results?",
"src": "Patient: We are trying to get pregnant for last 2 years.. Tried 7 cycles with clomid and 2 cycles with Ova shield and now last two cycles with siphene 150mg. taking Glycomet and folic acid as well.Not getting preganant.. Periods is regular and follicular study also done. Took HSG test and tubes are clear and uterus is normal.. what can be the reason of not geting pregnant Doctor: Hi,Thanks for writing to HCM .The cause in your case could be problem in ascend of sperms into uterus or fertilisation or implantation or wrong timing of intercourse. These can't be diagnosed. I would suggest you to go for intrauterine ingestion of sperms (IUI). Here sperms are washed and injected into the uterus so that problem in ascend of sperms is solved. I suggest to try for 3 cycles of IUI . This will increase the chance of conception. You may conceive for the first cycle only.After rupture of follicle egg will be alive for 12 to 24 hrs. Intercourse should be planned within that time or nearer to it as sperm survive for 2 to 3 days in uterus.Hope I have been helpful .Regards Dr.Deepika Patil"
},
{
"id": 68557,
"tgt": "What causes lump on lower left of head?",
"src": "Patient: I have a lump on the lower left side of my head. It hurts a little and the tenderness is now moving up behind my ear. It is also a little numb where it is tender. I was sick for about a week and a half and my cold is now almost gone. Should I be worried? Doctor: Hello!Thank you for the query.If this lump has appeared during the sickness, most probably it is a lymph node. Please note that lymph nodes get enlarged due to infection as a part of autoimmune response. Such node is usually movable and rounded. It should not be a reason to concern.Such lump can be also an abscess caused by sebaceus cyst infection. This condition appears in the area where there was a lump before. Redness, pain and swelling of this area should be present.In case of abscess you need to consult a surgeon and have it incised. If its a lymph node you may wait and see how it will be acting.Hope this will help.Regards."
},
{
"id": 183581,
"tgt": "What is the treatment for swollen and painful gums?",
"src": "Patient: Hi, I removed my lower wisdom tooth (horizontal impaction) before 10 days.Got my sutures removed on the 5 th day from the day of surgery. Now , When i open my mouth to yawn or have a large spoon of food I have a tough strong pain in the lower jaw same side where the tooth was extracted.The gums in that side is swollen,hard and painful. I am worried if my jaws will be painful here after or it is supposed to be like this for few days after a surgical extraction of wisdom tooth Doctor: Hi,Thanks for posting the query, I would like to tell you that pain, swelling, fever, nausea, dizzinesss are common side effects after tooth extraction you dont worry about the symptoms as they will subside after some time by its own, difficulty in mouth opening is due to the swelling, i would suggest you to follow the post operative instructions given to you properly, take lukewarm saline and antiseptic mouthwash rinses at home, take complete course of antibiotics and analgesics, take multivitamin suplements. Take care!"
},
{
"id": 44865,
"tgt": "Which doctor does IVF treatment in Jamshedpur ?",
"src": "Patient: i want to know the doctor s name doing ivf treatment in jamshedpur. Doctor: Hello. Welcome to Healthcare magic. To find a good HIV specialist nearest to your place, we have patient written reviews for doctors in our listing, those can guide you for a better doctor. You can search for a doctor in your area at http://healthcaremagic.com/doctors . OR One of our customer care executives will assist you further in finding a good doctor. (customercare@healthcaremagic.com)"
},
{
"id": 144571,
"tgt": "What causes left arm pain along with numbness and tingling?",
"src": "Patient: Hello, I suffer from left arm pain which I was told might be caused by a pinched nerve or herniated disc C4, C5, C6. Pain started in May with numbness and tingling down my arm. Took an X-ray , don t see much. Planning to do an MRI. I believe this would be the best way to diagnose? Can this be caused by cycling as I was cycling 60 to 80 km a week? Certain positions are soothing contrary to others where I get a shooting pain down the arm, sort of like an electric shock. Moving my head fully forward relieves me of pain. Basically, the worst pain is when I move my head back and sort of tuck in my chin. Any recommendations? Thank you. Lucio Doctor: Pain, Numbness and tingling in arm with electric shock like radiating pain strongly favor diagnosis of involvement of nerve roots. MRI Cervical spine with Nerve conduction studies (NCV) and Electromyography (EMG) will be best way to diagnose and localizing your problem. Such problem doesn't seem to be caused by cycling. I Recommend to use \"soft cervical collar\" and Tab Pregabalin 75 mg BD / Tab Gabapentin 100 mg TDS. Get MRI Cervical spine , NCV & EMG both upper limbs and get reviewed with reports."
},
{
"id": 24345,
"tgt": "What is heart murmur?",
"src": "Patient: The sight in my right eyes was blacking out & I lost the ability to speak for a brief period of time at work, while at theEmergency room the doctor asked how long I had a heart murmur. That was the first time I had heard that mentioned any relationship to the symtoms? Doctor: Heart murmurs are relatively prolonged series of auditory vibrations of variable intensity, quality and frequency.Heart murmurs result from vibrations set up in the bloodstream and the surrounding heart and great vessels (valve or wall) as a result of turbulent blood flow, the formation of eddies, and cavitation (bubble formation as result of sudden decrease in pressure).They are either caused by abnormal valve function or arise from an increased volume, or velocity, of blood flowing through a normal valve. This may occur in a healthy heart."
},
{
"id": 76758,
"tgt": "Suggest an ideal diet to control asthma",
"src": "Patient: Age 19years, hight 6ft, weight 62kg. medical=using salbo inhellar. Question= which quality of food should i eat to get control on asthma? Doctor: Thanks for your question on Healthcare Magic. I can understand your concern. Asthma is allergic in nature.So you might be allergic to some foods also. So better to first get done food allergy test and if you find allergic to any food than you should avoid that in your diet. Overall as a asthma patient you should follow these dietary advice. 1. Avoid Hot and spicy food. Oily and fatty food. This is because these kind of foods can cause GERD (gastroesophageal reflux disease) and this can worsen asthma. 2. Avoid stale food. Eat only freshly prepared food. 3. Take high protein diet like pulses, protein powder, milk etc. 4. Take anti oxidant rich food like fruits, green leafy vegetables etc. Along with good diet, you should also take your oral drugs and inhalers regularly. Don't worry, you will be alright. Hope I have solved your query. I will be happy to help you further. Wish you good health. Thanks"
},
{
"id": 219679,
"tgt": "Is hysterectomy advisable at this time?",
"src": "Patient: I am 39, HIV positive, had a PAP smear that showed dysplasia and I have multiple fibroids ... I have been told a hysterectomy will: 1. take care if the fibroids and 2. prevent the pre-cancerous cells from turning into caner of the cervix. NB. my second biopsy or was it a colcoscopy did not show the pre-cancerous cells ... Is the hysterectomy my only option for both?? Doctor: No , it depends on the degrre of dysplasia. if it is high grade only then the surgery is needed but not the hystrectomy. Hystrectomy will be needed only if the fibroids are giving you problems."
},
{
"id": 82304,
"tgt": "Suggest remedy for hiccups",
"src": "Patient: My husband had a cortisone shot in his right hip three days ago and now for two or three days he has gone into hiccups and I understand this could be a side effect of the shot. Is that true, and since we have not had success totally by giving honey, sugar on the tongue or swallowing ice chips, what can I do to get these stopped. Thanks for your help. Mrs. S. Doctor: Thanks for your question on HCM.Hiccups has no specific cause.But the most widely approved theory is irritation to phrenic nerve. Others are dehydration, pharyngitis etc.Treatment is also not specified anywhere.Drink plenty of fluids, keep yourself hydrated.Take antihistamines. Beclifen is the drug commonly used in hiccups. So you can try this beclofen if hydration does not give you relief."
},
{
"id": 199358,
"tgt": "How to get rid of too much of masturbation?",
"src": "Patient: hey I m 15 years old and I need help to stop masturbating , I masterbate everyday 3 to 4 times a day everyday and I ve started when I was 10 and I m very worried if this will make serious problems when I m older and want to start a family please help and I ve had these painful cramps sometimes when I have a really full bladder and I urinate intensely I get these hard cramps that come from the same muscles that I uses to push out urine and semen . please help :( Doctor: HelloThanks for query.You had been indulged in excessive and frequent masturbation since many years .Frequent habitual masturbation leads to problems like erectile dysfunction ,anejaculation and failure to perform intra vaginal sex .You need to have self determination to overcome this addiction.Please try to reduce the frequency of masturbation by keeping yourself busy in activities like sports,reading ,social work etc so that you will not get free time to masturbate. Routine daily exercise and meditation may help you to quit this addiction for masturbation.Dr.Patil."
},
{
"id": 207138,
"tgt": "Suggest treatment for aggressive behavior in case of OCD",
"src": "Patient: my son is suffering from OCD problem for more than 15 years. He is 29 years old.has done study upto high school only and is not doing anything .For last many yrs he is on following medication: Pacitane -1 ----------OD Resperidone-1------BD Prodep-20------------OD However, we dont find much improvement / control over his repetitive behaviour and anger.He gets extremely furious over petty matters specially whenever his unending demands are declined and creates a lot of fuss in the family.He sometimes , while in extreme fit of anger, pretends to kill himself by holding a kitchen knife in his hand. Pl advise what to do. Doctor: The doses of medications which he is taking needs to be increased. Increase prodep to 40mg and resperidone to 2mg bd."
},
{
"id": 48968,
"tgt": "What do you suggest for bleeding after an abdominal hysterectomy with the kidney that is attached to the uterus?",
"src": "Patient: Hi, I had an abdominal hysterectomy almost 3 month ago; I still have my ovaries,; however, my surgery was a little complicated because I have pelvic kidney that was attached to the uterus and some varicose veins that were fixed; My recovery was very good and I bled for about 7 weeks and then stopped. Doctor: HI MADAM , I UNDERSTAND YOUR CONCERN .IN YOUR CASE THERE IS NOTHING SERIOUSLY TO WORRY ABOUT SINCE YOUR BLEEDING HAS STOPPED SPONTANEOSLY.ITS A VERY GOOD SIGN.I SUGGEST YOU GET COMPLETE BLOOD PICTURE AND A CT SCAN ABDOMEN TO EVALUATE FINE STRUCTURAL AND VASCULAR AS IT S ALL THAT IMPORTANT. DETAILS OF YOUR PELVIC KIDNEY AND GET A FOLLOW UP WITH YOUR DOCTOR ON YOUR NEXT VISIT"
},
{
"id": 121362,
"tgt": "What to do for tenderness near the thumb?",
"src": "Patient: On the top side of my forearm about 2 inches above my wrist on the thumb side it is very tender I. Can t pick up a milk carton and it hurts when I make a fist also when my wrist flexes I can feel a rubbing sensation in that area it feels like something is tearing or rubbing together Doctor: Hi,Have you sustained any injury? If there is no injury, it seems to be tenosynovitis. Apply hot pack, take over the counter painkillers, apply diclofenac gel. Hope I have answered your query. Let me know if I can assist you further. Regards, Dr. Jaideep Gaver, Orthopedic Surgeon"
},
{
"id": 163154,
"tgt": "What causes nausea, emesis odor, vomiting and abdominal tenderness?",
"src": "Patient: Hello. I have 9 yr old with no chronic/significant medical problems. He was nauseous earlier this evening. He began vomiting about 2 hours ago. He has vomited 3 times, approx. 1L. Each time, the emesis smells & looks like diarrhea; very thin, watery & brown. I m an RN & in my 11 yrs of experience I have NEVER seen or smelled such foul vomit. He is not complaining of abd pain (some superpubic tenderness), intermittent nausea. Afebrile. No diarrhea. Regular BM this morning. I ve given him 5mg zofran liquid, however, he s still vomiting. Any suggestions? (Im using my phone to type this, please give me some time to reply. Thanks) Doctor: Hello and Welcome to \u2018Ask A Doctor\u2019 service.I have reviewed your query and here is my advice.It seems like gastritis and the brownish colour can be due to ulcer bleed. Stop spicy food. Start syrup Polypep thrice syrup Domel thrice a day and Risek sachet every night. Give him ORS after every vomit. Give banana for deficiency. If condition persists Transamine can be given after medical advice to stop bleeding.I hope I have answered your query. Let me know if you have any further questions. Regards, Dr. Hina Javed"
},
{
"id": 169979,
"tgt": "Should a child be taken to a doctor after ingestion of a muscle relaxer?",
"src": "Patient: Hi, my daughter is 20 months old & I just caught her sucking on a skelextin ( I think is how you spell it), anyway it is a muscle relaxer. She only sucked on it for about a min & I took it away, but she was already spitting it out anyway. Should I take her to the emergency room? What can happen to her? Doctor: IT IS BETTER YOU CONSULT PEDIATRICIAN, HE WILL EXAMINE YOUR CHILD FOR TONE ASSESSMENT, RESPIRATORY AND HR EVALUATIONAND IT S BETTER TO ADMIT THE CHILD FOR THE COUPLES OF HOURSAS CHILD ARE EASILY DETERIORATE...THANK YOU"
},
{
"id": 222297,
"tgt": "How to confirm EDD during a pregnancy?",
"src": "Patient: Sir i have a question... my wife LMP was 4th March 2010 .... we have got married on 24th March 2010 ... we did sex on 27th march for the 1st time... and she got pregnent.. now EDD calculated was 9th December by dates... and after from ultra sound EDD came was 27th December.... 9th month is started now and doctor is suggesting SGA ... we are confused what should be done now... Doctor: Hallow Dear,Since it is cumbersome and not cost effective to locate the exact date of conception, usually the expected due date of delivery (EDD) is calculated by date of last menstrual period as reference point if the periods are regular. The formula for calculating EDD is :EDD = LMP + 9 months + 7 days. So in your wife's case it come to 31st December. Ultrasound also has calculated 27th December which is very close to the calculation by dates. Calculation by ultrasound has allowance of a week or so. I hope this satisfies you. Dr. Nishikant Shrotri"
},
{
"id": 124736,
"tgt": "How to reduce arthritis and joint pain?",
"src": "Patient: Good morfning Dr Grief, I have just read a long article about Instaflex , a supplement to reduce arthritis and joint pain,which purports to be readlly effective. What is your opi nion? I am 74 years old and have been suffering from knee pain for a year and ankle pain for a few months. Thanks Doctor: Hello, As a first line management you can take analgesics like paracetamol or aceclofenac for pain relief. If symptoms persists better to consult a rheumatologist and get evaluated. Hope I have answered your query. Let me know if I can assist you further. Regards, Dr. Shinas Hussain, General & Family Physician"
},
{
"id": 104007,
"tgt": "Can respiratory arrest lead to VT?",
"src": "Patient: hi i am doing an assignment. i have to do a differential diagnosis. my topic is a 61 year old female with a PHx of hypertension & asthma. no allergies, and is found unconscious & not breathing, the monitor shows up VT. I think i have 2 options or diagnosis, the first being hypertension leading to cardiac arrest, second being severe asthma attach leading to respiratory arrest, leading to cardiac arrest (VT). to go with my second option, i need to clarify that respiratory arrest can lead to VT? Please help me out Doctor: Hi friend, Welcome to Health Care Magic You need more information - for proper suggestion...Either can be right! / there could even be a third unrelated event! Hypertension may have resulted in Myocardial Infarction or Aortic dissection Asthma might have resulted in Tension Pneumothorax... In VT, the priority is to treat - even without a diagnosis, which of course is being evaluated simultaneously... Take care Wishing speedy recovery God bless Good luck"
},
{
"id": 41612,
"tgt": "Suggest treatment to overcome infertility",
"src": "Patient: sir i am marrade and i have no child befor 5 year one time my wife pragnet after 2nd month the pragenecy is start is tube and he is damage within 2 month and doctor do operation and after 2 year we meet with other and thay start bleading and then doctor do aganin operation now we need chalid and doctor speek we 2 madicaly fit plz tell me what i do Doctor: Hi welcome to healthcaremagic.I have gone through your question.As you had two miscarriages before i would advise to do histosalpingography HSG to rule out tubal blockage due to previous miscarriage and tubal pregnancy, as tubal blockage happened due to tubal pregnancy and miscarriages.Then consult your gynecologist for further treatment plan.Hope i answered your question. would be happy to help you further.Take care."
},
{
"id": 25628,
"tgt": "Can swollen lymph nodes be symptom of high blood pressure?",
"src": "Patient: Hi. I'm a 26 year old woman. I've had swollen lymph nodes since I was a kid, under both of my armpits and my inner thigh (groin?). Recently, I went to a doctor for my annual exam, and they did a blood pressure check. My blood pressure, surprising to me, was very high. Inspite my parent's health history of high pressure, I've always had a normal blood pressure. Can my swollen lymph nodes be the cause of my sudden rise of blood pressure? Doctor: No,your swollen lymph nodes will not increased your blood pressure directly...however you will have to rule out the cause of swollen lymph nodes,as some connective tissue disease which affects vascular system ,can increase your blood pressure"
},
{
"id": 188848,
"tgt": "Ear and throat pain, slight bleeding. Growing wisdom tooth. Normal?",
"src": "Patient: My wisdom teeth recently grew all four of them, but the one on the bottom left hasn t completely grown and i started feeling pain with an ear ache, and then my throat now hurts, and when i wake up in the morning i spit a tiny bit of blood , thats barely noticeable! should i be worried? the tooth ache is much less today, but my throat itches and ear hurts! Doctor: Hi,Thanks for asking,As your one wisdom tooth is partially grown so debris gets accumulated under the flap covering the tooth.It causes inflammation, swelling and ultimately pain.Consult a dentist for irrigation under the flap(operculum), follow antibiotic coverage.Maintain oral hygiene and use antiseptic mouthwash.Do warm saline rinses daily for healing.Take care"
},
{
"id": 2396,
"tgt": "Is there any chance of pregnancy even after taking depo shots?",
"src": "Patient: I m currently on the depo shot, I had a period over 28 days ago but haven t had another one yet. I ve never not had my period even while on the shot. I start to feel really sick in the evening but not throwing up. I feel fatigued a lot lately and have had mood swings. Any chance I could be pregnant? Doctor: U must get done pregnancy test.. There can be little changes of getting pregnant.. So confirm it with upt"
},
{
"id": 198514,
"tgt": "What could be the raw spot on my penis after having sex?",
"src": "Patient: I'm a 48 year old healthy male, haven't had sex in a while, had sex with my girlfriend about 10 times in two nights, which I know is a lot. We got a little carried away, and left a raw spot on the bottom of my penis. Now when I urinate it burns about half-way down the shaft of my penis. Doctor: HelloThanks for query .The symptoms of pain in penis ,burning urination and presence of raw spot on penis are due to inflammation of urethra due to excessive and vigorous sex over a short period of time.You need to take broad spectrum antibiotic like Cefixime along with anti inflammatory drug like Diclofenac twice daily .along with topical antibiotic ointment like Neosporin twice daily to be applied over raw area ..Ensure to wash your genitals with warm water twice daily.Ensure to avoid sexual encounters till it heals up completely.Dr.Patil."
},
{
"id": 87187,
"tgt": "What causes right lower abdominal pain with liver enzyme levels high?",
"src": "Patient: Hi, may I answer your health queries right now ? Please type your query here... i have pain on my right side lower abdorm vry sharp some time itgoes round back by ribs ihave have loads of test just on came back with liver enzymes up a bit wat does it mean Doctor: Hi.Thanks for your query.Pain in the right lower abdomen can be due to colitis, appendicitis, infection of the intestine, typhlitis. This infection is all carried to the liver in all cases. and this may be the reason for a bit rise in the liver enzymes. I would advise you the following :::CT scan of the abdomen to find out the cause of all these symptoms and reports. If appendicitis, get operated. If infection get complete course of an antibiotic and metronidazole and probiotic and you will be fine."
},
{
"id": 115313,
"tgt": "What is the treatment for anemia?",
"src": "Patient: Hi there, I am severely anemic and have been for a while now, I have stomach pains after eating and they last all day. I have had a colonoscopy, ultra sound, capsule endoscopy, endoscopy, and a ct scan. And my doctor cant seem to find out what is causing my anemia, I was taking iron pills but have now stopped because it is really hard on my stomach, i am now getting iron injections. Any ideas what could be causing this? I have low energy, iregular heart beats, low blood pressure, light headiness, i would appreciate your help very much, thankyou. Doctor: Hi,Sorry to hear about your condition. Have you checked your thyroid harmone levels?That might explain many of your symptoms. Could you let us know your age as that has a bearing as well. All the best"
},
{
"id": 136158,
"tgt": "Suggest treatment for burning pain and numbness on the leg and foot",
"src": "Patient: I have a burning pain in my left thigh with numbness to my leg and foot, along with shooting pains down my right leg and left arm. I also get some shooting pains in my left leg, but it s more the burning pain that is bothering me there. I am 27 years old, weigh 105lbs and am generally healthy. Im not sure what this is or what I should do? Doctor: HiWelcome to healthcaremagicI have gone through your query and understand your concern.Looking to your age it is most likely a acute disc prolapse . It can be confirmed by MRI of lumbar spine. Till then it requires rest and analgesic such as ibuprofen for pain relief. Rest is advisable in lateral position with both hips and knees flexed. You can discuss with your doctor about it. Hope your query get answered. If you have any clarification then don't hesitate to write to us. I will be happy to help you.Wishing you a good health.Take care."
},
{
"id": 21499,
"tgt": "What causes shortness of breath, rapid heartbeat and sweating?",
"src": "Patient: Hi, I'm an 18 year old girl. Recently I have been finding it harder to sing without getting out of breath. My heart pumps fast and i have a shortness of breathe even when I don't do anything. I think this may be worse when the weather is mild. I also sweat sometimes when im doing something as simple as putting on my make up. And sometimes when i straighten my hair i need to sit down for a second. My hands i shaking but ever ever so slightly. Doctor: Hello,Your symptoms are pointing to generalized anxiety disorder. It is something I come across in my everyday practice. These are some of the things you should do to help with your symptoms:1. Always try to get adequate sleep.2. Avoid smoking or drinking if you do as it will worsen your symptoms.3. Exercise frequently and eat a healthy diet.Take tablet Valerian extracts one to two hours for one to two weeks. It will help you sleep well and also help with your anxiety. If your symptoms persist after two to three weeks, seek consultation with a psychologist.Hope I have answered your query. Let me know if I can assist you further.Regards,Dr. Isaac Gana"
},
{
"id": 39752,
"tgt": "What is the remedy for the recurrent scab with pus formation after an injury?",
"src": "Patient: I have a leg injury due to grass burn on AstroTurf, it forms a scab but has pus, I remove the scab, clean the wound and dry it out natural air or mecurochrome but it just reacurres that when them wound forms a scab again that theres pus under it. Help please, I went to the Doctor 3 weeks back and he just said to let it dry out Doctor: HIThank for asking to HCMI really appreciate your concern the given history is suggestive of some infection part is there and this need to be eradicate the infection then and this would healed, and best antibiotic is Amoxicillin 500 mg three times in day, for the local application you can use Povidone ointment hope this information helps you, have nice day."
},
{
"id": 64909,
"tgt": "What causes lumps below nipple in males?",
"src": "Patient: Hi, i m 30 years old indian male.when i was 14 years old i experinced one small gland beneth my left nipple n it was paining tendorly then after sometime i felt the same in my right nipple as well i dint discuss with my parents that time due to shame n poor financial condition of my family. later on it went on to become flat n covering more surface area around my nipples probabbly 3 cms in radius with 1 cms depth.currently left chest is bigger than right with both having diffrent shape.kindly advise me what will be best option n how many days hospitalisation surgery needs Doctor: Thanks for your question on Health Care Magic. I can understand your concern. In my opinion, you are mostly having gynecomastia (enlargement of breast in males). The most common cause for this is hormonal imbalance. So better to first diagnose the cause for your condition. Consult endocrinologist and get done hormone levels, mammography. If it is due to hormonal imbalance then hormonal therapy will be helpful, otherwise surgical reduction mammoplasty is an option. So better to consult endocrinologist, first diagnose yourself and then start appropriate treatment. Hope I have solved your query. Wish you good health. Thanks."
},
{
"id": 9042,
"tgt": "How can the dark circles under my eyes be removed ?",
"src": "Patient: Hello, I am having dark circles under my eyes. I am taking healthy food. Always i am having a good sleep during night. Little bit tension in life. No results using cucumber slices..Already i hav tried a lot.. so please suggest me to do something,so that dark circle will disappear..I am really feeling sad for this..Pls help me.. Doctor: Hi Pragyan, Welcome to HCM. Avoid stress to your eyes like working long time in computer. Put cucumber slices on your eyes daily in the night before going to bed.You can also try some under eye creams for dark circles."
},
{
"id": 194011,
"tgt": "What causes white head pimple on the shaft of penis",
"src": "Patient: I have what appears to be a white head pimple on the shaft of my penis. I popped it once but it has not disappeared. the skin is not reddened surrounding the pimple. it is sore if touched, it does not ooze. I am sexually active but I dont believe it is a genital wart. Doctor: Hello, The pimple-like growth could be pearly penile papules. Nothing to worry. Just do not break it. Clean the penis regularly. Hope I have answered your query. Let me know if I can assist you further. Regards, Dr. K. V. Anand, Psychologist"
},
{
"id": 75503,
"tgt": "What causes bubbling sensation in chest and breathing difficulty?",
"src": "Patient: Im 24 years old,5 5 and 215 lbs. I am having chest pains from a car accident and I am getting this bubbling sensation in my chest and am having shortness of breath. I was told it sounded like I had a small hole or rip in the sack that holds my lung and it would heal within a couple days and it has been a month since the accident and i still am having this problem. I am also getting pains in my back in the area of where my lungs are when I yawn. Any answer would be helpful ...thank you! Doctor: If the pain is due to covering of the lung it may take some time to heal but there should be no major damage to the structures. You may take some pain killers and apply some local gel. Go slow on physical activity for some time."
},
{
"id": 162259,
"tgt": "How can dry chronic cough in a 12-year-old be managed if no relief is obtained after taking Prednisone and Erythromycin?",
"src": "Patient: my daughter, age 12 has had a dry chronic cough since early june, she has since become exhausted and is now having body aches especially in her back. she can recover her breath ok but is too weak and tired for too much activity. After 3 trips to the doctor she was finally given prednisone for 12 days . She finished that 6 days ago and also tried erithromycin which she just finished today. She has no improvement. She did struggle with asthma as a child but hasn t had any major episodes since age 7. The only episodes recently might require her to use a rescue inhaler after heavy exercise in the heat and then only rarely. She has been a healthy active soccer player up until this point. Doctor: Hi, By what you quote I feel what your kid could be having asthma. I have a few questions for you - 1. How many days per month does your kid cough or feel breathless? 2. How many nights per month does your kid's sleep get disturbed due to the above symptoms? 3. Does your kid feel breathless when running around or playing with other kids? 4. Are the symptoms when there are seasonal changes? 5. Is there any family history of asthma or any other sort of allergies like skin allergy etc? 6. Is a cough always associated with fever? If your answer is yes for any of the above questions, your kid might be having viral associated wheezing or multi triggered wheezing. I suggest you meet a paediatric pulmonologist who is near your place. No point in using antibiotics every time. It will do more harm than good by increasing bacterial resistance. Hope I have answered your query. Let me know if I can assist you further. Take care Regards, Dr Sumanth Amperayani, Pediatrician, Pulmonology"
},
{
"id": 69027,
"tgt": "Suggest treatment for feeling a lump under ribcage",
"src": "Patient: Hello , after childbirth I notice strange feeling under right rib cage , that there is something. After that I notice in the same place vertical lump , It feels uncomfortable when I'm touching it . Now I have swollen glands in right side of my neck and patches in my throat , what it can be ? Doctor: Hi ! Good morning. I am Dr Shareef answering your query. The symptoms and signs you have narrated warrants an early physical examination and related investigations by a general surgeon to arrive at a diagnosis. I am afraid it might not be possible to give an opinion on your problem over the net without a clinical examination. Therefore I would advise you to show your family physician or a general surgeon at an early date.Thanks for choosing health care magic to clear doubts on your health problems. Wishing you an early recovery. Dr Shareef."
},
{
"id": 82134,
"tgt": "What could painful cheat,ear and throat with breathing difficulty suggest?",
"src": "Patient: I can t get to a hospital because I live in a very small village and they don t have a reliable clinic here and I need to know what I have cause my chest hurts my ears my throat hurts and its hard to breathe and I keep sneezing and puked a few times n feeling tired and I am trying to figure out what is going on its been 2days now and it just got very worse today and I need to know what it is Doctor: Thanks for your question on HCM.In my opinion you are having respiratory tract infection.starting from nose till the lungs.So better to start these treatment. 1. Avoid oily and spicy food.2. Drink plenty of water.3. Start macrolide group of antibiotic. 4. Also take antihistamines drug5. Also take anti inflammatory drugs.If not improve in 5 days better to get done Chest x ray to rule out lower respiratory tract infection."
},
{
"id": 40668,
"tgt": "Is Regestrone effective in the treatment of infertility?",
"src": "Patient: I ovulated on 20 th october and i was on progynova tablets from 10-20th day of my cycle . I was given clomiphene for ovulation. From 21 st doctor has asked to take regesterone medcine twice a day for last 10 days of cycle. I am ttc . I read somewhere that it would not help my fertllized egg to attach to uterus.. should i take this . I am confused doctor please help Doctor: Regestrone will help to continue ur conception if ur egg has been fertilised and implanted. its kind of safeguarding u can take tat drug"
},
{
"id": 146077,
"tgt": "What causes dizziness, shortness of breath and shaking hands?",
"src": "Patient: I am a 15 year old African American teenage girl with a 32-35 bmi. Sometimes when I take a shower I get a dizzy feeling after maybe 6-8 minutes of showering. I also have shortness of breath and my hands shake. I feel absolute miserable while this is happening and today I had to hurry up and finish for fear that I might faint. What is happening to me? Doctor: Hello. I have been through your question and understand your concern.Sometimes, while taking a hot bath, especially after a meal, you can have blood sequestration in the skin due to the high temperature of the bath and in the stomach if you are having the bath after the meal. This causes less blood going in your brain causing the symptoms described.There is nothing to worry about, because it is quite common. Just be sure you take the bath three to four hours after the meal and take the bath at a lower temperature. You will see it will not happen again.Hope this helps, please feel free using MHC for further questions."
},
{
"id": 195330,
"tgt": "Is Z- Plasty better than Fernuloplasty?",
"src": "Patient: Hi, I am a 33 year old Male and looking to get Fernuloplasty in Dallas, TX area. I did some research online and found Z-Plasty is the best option. Is that true? Could you advise whom should i reach and the approximate cost to be financially prepared? Doctor: Hello and Welcome to \u2018Ask A Doctor\u2019 service. I have reviewed your query and here is my advice. Yes, the technique you mentioned is to lengthen the frenulum to relieve stretch. But V-Y plasty is also useful. Surgical method may vary depending upon how short is frenulum and associated phimosis is there or not etc. Please consult your surgeon, he will examine and treat you with appropriate method of surgery. Take care."
},
{
"id": 152470,
"tgt": "Does chemotherapy cause pain in the forearm?",
"src": "Patient: Hello, I have a 13 year old daughter who is a cancer (Leukemia) survivor at the age of 5. She has been complaining of pain in her left forearm for about a week. There is nothing visible, no swelling and to the lay person, the tissue feels the same as her right arm. Could this be side affects from Chemotherapy? Or simply a strain? Thank you for your time. Doctor: Hello and Welcome to \u2018Ask A Doctor\u2019 service. I have reviewed your query and here is my advice. It doesn\u2019t look like a side effect of chemotherapy after such a prolong period. Looks more like a simply strain. Hope I have answered your query. Let me know if I can assist you further. Regards, Dr. Deepak Sundriyal"
},
{
"id": 219060,
"tgt": "What are the chances of MSUD in a child?",
"src": "Patient: hi doc,Im Ruby 27 years old from Phillippines,my husband and i are planning to have a baby again.I give birth to my 1st child last July 3,2008.But unpotunitely our son died because of MSUD.he only lived for 23 days only.The genecologist said that there is a possibility to have MSUD childe again.Im polycestis women.My doctor give Dydrogesterone duphaston and Clomifene Citrate.Is this the right medicine for me to have a baby?how can I avoid my child from MSUD disease?please help me.Our life is empty because we have no children. Doctor: Hi Ruby ,This is Dr.Abrar Ahmed Khan From IndiaI Welcome You To HCM & Thanks For Asking Your Query Here You might be aware that Maple Syrup Urine Disease (MSUD) is a genetic disorder and it depends upon probability of passing the defective gene to the babyeach child has a 25% chance of receiving two mutated genes and having MSUD, 75 % chances of a normal baby with no MSUD but out of these 75 % there are 50% chances for receiving only one defective gene and being a carrier(means these baby will not have MSUD but will probably carry this Disease to their next generation), and a 25% chance for a completely normal baby who will neither have MSUD nor will carry it to the next generation Since MSUD is an inherited disease, there is no technique for prevention. But I would suggest you not to loose hope , trust in almighty and contact a A genetic counselor , he can help you determine your risk for having a baby with MSUD. a Genetic testing can tell you if you or your partner is a carrier of the diseaseand DNA testing can identify the disease in a fetus before birth so there is no need to worry , keep hope and best of luck I hope it help May God bless You"
},
{
"id": 122147,
"tgt": "Suggest pain in ligaments near the heels?",
"src": "Patient: I had hurt my ligaments near the heels of both the feet 7 months back. I dont know how because there was no immediate pain which can be traced to an event. I suspect it could have happened because once i had to walk quite long wearing a shoe that was hurting my feet. Or it could be beacuase i had to help my servant to carry a weighty table to fourth floor climbing the steps (i was not usual to such things). my doctor asked me to take bed rest and gave me some tablets containing Diclofenac and some ointments. Later he asked me to take physiotherapy. Nothing helps. Now after 7 months of treatment i am not not able to walk properly. Kindly suggest any therapy. Doctor: Hello, The symptoms seem to be related to Achilles tendinitis. I suggest using anti-inflammatory medications such as Ibuprofen to relieve the pain. I see that you have already treated the problem with physiotherapy. If the symptoms continue, I suggest consulting a surgeon for further evaluation. Hope I have answered your query. Let me know if I can assist you further. Take care Regards, Dr Dorina Gurabardhi, General & Family Physician"
},
{
"id": 167942,
"tgt": "Suggest treatment for cough and running nose in a child",
"src": "Patient: My 8 year old daughter started last night with a cough and incessantly runny nose. She woke up this morning with the runny nose and then began vomiting. She has vomited 3 times since 6 am. this morning. She has a jacket and blanket on. She throws up the ibuprofen. Doctor: your child probably caught a flu , she will tolerate oral fluids soon , start with warm soupsgive her the ibuprofen when her stomach rests a little put cold compressors give her zyrtic hope she becomes better soon"
},
{
"id": 168292,
"tgt": "Suggest multivitamin for a kid",
"src": "Patient: MY BABY AGED 2 YEARS 11KG WEIGHT GOT HIGH FEVER 2 DAYS BEFORE NOW HE IS NORMAL AND FEELING TIRED, SO CAN I GIVE HIM SOME MULTIVITAMIN SYRUP, BEFORE HE IS GETTING FITS WHEN HIS TEMPARATURE GOING HIGH,NOW NOT LIKE THAT SITUATION, KINDLY ANSWER MY QUESTION Doctor: Hi, Welcome To HCM & Thanks For Asking Your Query Here There is no harm in giving multivitamin supplement to you kid but give right doses ( for a baby 2 yr old and around 9-10 kg wt ,the dose should be 5 ml daily divide in 2 doses ) and give it after meal , but stop it if your kid throw it up and if there is any stomach discomfort It is normal for a child to feel tired or lethargic after recovering from illness but if this persist even till one week after giving multivitamins than i would strongly suggest you to see Your dr to further evaluate the conditionI hope it help Thanks & Get well Soon"
},
{
"id": 131075,
"tgt": "What could cause painful joints after recovering from Chikunguniya?",
"src": "Patient: hello doctor, 6 months back i got affected with chickunguniya and got it treated. one month back i got pain in my right hand wrist and i went for the treatment and doctor gave medicine for inflammation. after a week, while walking, i got pain in the right foot and got aggravated and walking is painful now. i took medicine for two long weeks and not cured. i had taken blood test for uric acid and ESR also, which is normal , the doctor said. kindly help me Doctor: the joint pain is due to chikunguinia.physiotherapy treatment will give you relief from joint pain and help you maintaining joint ranges yoga is also helpful for this diseaseas a prognosis the joint pain will become worse for a year and than it will resolve slowly on ownhoping you a faster recoveryhope you find this helpful"
},
{
"id": 523,
"tgt": "What are the suitable condition for conception?",
"src": "Patient: Hi, we got married 1 yr ago and the first 3 months my wife used i-pill few times. After that we completely stopped and trying for a baby now. We are consulting with CMH doctors but we are not convinced with their treatment and it's too crowded also. Overall my wife is not getting conceived till date. Please suggest me a very good gynacologist who can take 100% care about the issue? Doctor: HiGREETINGS First of all get your semen analysis. If it is normal consult any gynaecologist and get the hormonal analysis and a basic ultrasound scan to rule out any abnormalities of uterus and ovaries. If all are normal then the tubal patency has to be checked.Am not aware of any body in your area.Hope you are convinced. Regards"
},
{
"id": 78313,
"tgt": "What could be done for the decreasing oxygen levels having tiredness and fatigue?",
"src": "Patient: My oxygen is running at 82-86 daily, I am always tired and get very fatigued instantly. I have been checked by heart doctors and pulmonary doctors. They tell me there is nothing wrong and live with it. I don t call this living. I am 57 years old , 5ft 6in. tall and weigh 210. I have gained 15 pounds this winter, I have no energy to do anything. 2 years ago they put me on a C-PAP thinking that would help,still nothing, was rechecked this month with no new results. am getting very annoyed. Did a stress test. saying I have low level o2 when I walk slow and medium but when I work harder the o2 improves. What could be causing this. I need answers. Doctor: Thanks for your question on Health Care Magic. I can understand your concern. By your history and description, possibility of bronchitis, interstial lung disease (ILD) and Pulmonary embolism is more. So better to get done fresh reports like 1. PFT (Pulmonary Function Test) for the diagnosis of bronchitis. 2. HRCT (high resolution CT =thorax for the diagnosis of ILD. 3. CT Pulmonary Angiography to rule out Pulmonary embolism. Oxygen saturation of 82-83% is considered low and should be evaluated to rule out above mentioned causes. Better to first diagnose yourself and then start appropriate treatment. Hope I have solved your query. I will be happy to help you further. Wish you good health. Thanks."
},
{
"id": 145351,
"tgt": "Suggest treatment for mental confusion,vomiting and blurred vision",
"src": "Patient: my son got sick in school last week took him to the emergency room he was confusied throwing up and wanting to go to sleep he said his vision was blured the lights hurt his eyes and he was at school htey took samples of his blood and his urine came back and said his alcohol level was positive but their was no acohol smell at all this what it said from his blood/acohol sammples plasma/serum blood alcohlo 13 mg/dl Doctor: Hello ! I understand your concern. I would like to know if these symptoms are persisting or resolved now. The symptoms that your son has had may be related to alcohol intoxication. But its plasma level has to be above 80mg/dl to show these symptoms. He has presence of alcohol in his blood and so alcohol has to be taken into consideration in this situation. May be his brain is more vulnerable to the age ( school age, you have not told his age ). But also other diagnosis have to be excluded like a possible subarachonoid hemorrhage, possible meningitis or just migraine . Has he ever had throbbing headaches ? Did he have fever? It would be a sign of meningitis. Also neck stiffness and headache. If the symptoms persisted a head CT scan and a lumbar puncture would be necessary to rule out subarachnoid hemorrhage. To make a proper treatment we should rule out these diagnosis. At the meantime I would recommend a lot of fluids to lower to zero alcohol plasma level and antiemetics for the vomiting. Hope to have been helpful!Thank you for using HCM!Best Wishes Dr. Abaz Quka"
},
{
"id": 105723,
"tgt": "Cold, running nose, sneezing, unable to eat, weight loss, allergic wheezing",
"src": "Patient: Hi , My Daughter is 9 months old... she is being suffering from cold ( symptoms include running nose, hard breathing , sneezing , coughing) for the past 10 days and she is currently on medication( azithral, Hicet, Prozeet LS) .I was informed that she is suffering from Allergic Wheezing . she is not eating solids since she is not able to sallow.... she drinking only milk.. Am worried about her weight . Doctor: Hello welcome to HCM Since she is suffering from a respiratory tract infection she will find it difficult to swallow solids as it would be painful. Apart from milk give her hot soup, vegetable stalk. You can also give mashed rice in a liquid form. There will be slight decrease in weight in this period which is not to be worried about. This will be because of the medicines she has been taking so long. Hope this answers your query."
},
{
"id": 17908,
"tgt": "Could exercising increase the risk of heart failure while having LBBB and tricuspid regurgitation?",
"src": "Patient: Had an abnormal EKG with a LBBB (QRS - 142) diagnosis. It was confirmed on an echo with the following echo interpretation summary of: Borderline left ventricular systolic function, ejection fraction 50% Normal right ventricular systolic function Tricuspid regurgitation - mild Elevated pulmonary artery systolic pressure - borderline I am scheduled for a stress echo next week but not sure if I should continue exercising. Am I at any higher risk of heart failure if I continue exercising? Doctor: welcome to hcm ..you can continue with mild to moderate exercise plan especially if you are already exercise doing but please avoid heavy stressful exertion and maintain bp ..sugar and Hb...avoid infection if you have any further query then you can come back to hcm i will definitely help you and guide you appropriately with regards dr varinder joshi"
},
{
"id": 64702,
"tgt": "Suggest treatment for a painful lump on the armpit",
"src": "Patient: I have a slightly tender lump in my armpit. It has been there for a few months and only feels painful when I push on it. It is about the size of a quarter. I have also been wheezing and coughing for about the same period. Not sure if they are related. Doctor: Hi,Dear ,thanks for the query to HCM. I studied your query in-depth and I feel concerned about it .In my opinion-its due to the Viral bronchitis with throat infection.Possibility of Infectious mono-nucleosis is to be ruled out. FNAC would fix the diagnosis.Tb -NSAIDs and broncho-dialators for bronchitis with wheeze and Steroids if infectious mono is the treatment.Hope This would solve your query.Wellcome again for any more query."
},
{
"id": 222394,
"tgt": "How accurate are dating scans?",
"src": "Patient: Hi I need to know how acurate 3 dating scans are as a measure of paternity. I know I need to do a dna test but all three scans perfromed at 12, 16 and 20 weeks have provided the same due date and therefore the same conception date and I was with the suppossed father that day. Can you help me? Doctor: Hello, and I hope I can help you today.Estimating your date of conception is only really possible for women who have a 28 day menstrual cycle. By convention, the computer programs that are used to calculate your due date are working from an assumption that a woman has a 28 day cycle, and that conception took place exactly 2 weeks after your last period. If your cycle is longer or shorter, your ovulation could have taken place on a different day than what is estimated.So really just depends on whether you were with your second partner during your fertile window, which really can start 7 to 10 days after the first day of your last menstrual period and continue until a week before your next menstrual period is due.So depending upon how close and timing your sexual encounter with your second partner was, will determine how likely it is for him to be the father. If the margin either way is only a matter of a few days, you really will need a DNA test to accurately predict the paternity of your child.I hope that I was able to adequately answer your question today, and that my advice was helpful.Best wishes for the rest of the pregnancy,Dr. Brown"
},
{
"id": 206383,
"tgt": "What is the treatment for concentration problems?",
"src": "Patient: I have a very poor attention span, i cant concentrate on a tv show, or even when someone is talking to me my concentration lasts a stupidly small time. I was considering asking my GP if he could prescribe me ritalin. I am not sure whether i would need any further testing or anything before hand. I do not want the drug for any form of recreational useage and in fact i would actully prefer one that increases concentraion without those side effects. I am not a fan of drugs that can have uses other than what i need them for eg when i have asked for sleeping pills or painkillers before i have specified i do not want opiates, benzo's etc.I was wondering if you think it would be a good idea to ask my dr for them, or if there is an alternitive to them that you think would be betterAdditional info, i am 21, male. I was been diagnosed with bpd a few years ago. I am not on any medication. I used to drink alcohol too much but i cut down a few months ago to a fraction of the amount i used to drink. Smoker - around 10/15 per day that i roll myself from tobacco. Doctor: DearWe understand your concernsI went through your details. I suggest you not to worry much. You are suppose to refer to concentration and attention as two. Attention span for any adult is maximum 30 minutes. Sometimes, a person can be attentive to an interesting thing for more than 30 minutes, such as movies or games. Concentration is attention plus understanding. For this too, span of below 30 minutes. If you are able to make your studies interesting and based on variable principle, your concentration level can be increased. Concentration comes only when you are interested in what you do. So please be passionate towards what you are doing or do only those things which you are interested in. It is also possible to improve your concentration level with the help of yoga, meditation and breathing exercises.Psychotherapy techniques should suit your requirement. If you require more of my help in this aspect, Please post a direct question to me in this URL. http://goo.gl/aYW2pR. Make sure that you include every minute details possible. I shall prescribe the needed psychotherapy techniques.Hope this answers your query. Available for further clarifications.Good luck."
},
{
"id": 79170,
"tgt": "Suggest treatment for body aches and dizziness with respiratory infection",
"src": "Patient: I have a upper respiratory infection, and sinus infection, extreme coughing, phelm is bright green and hot, this has been going on 3xdys now. suddenly today I felt like I couldn t breath, body started aching severely, dizzy. The only relief I could get was stretching my body out and soaking in hot water, I got extremely cold and chilling. It lasted about 30 mins. still feel achy however not as severe. My calf muscles are jumping constantly. I take Lisinopril, Cymbalta, the only new medication is a antibiotic, decongestant, and water pill. Any ideas, should I go to emergency room? Doctor: thanks for asking your question you should consult a Pulmonologist in the emergency as , inspire of taking the medications your symptoms are not resolvingprobably the antibiotics you have been prescribed empirically are not working green phlegm means a bacterial infection Pulmonologist may request for a chest X-ray and a sputum culture and sensitivity for pyogenic organisms and more appropriate treatment can be started according to the sensitivity reportssteam inhalation ll help a lotthanksfeel free to ask more questions"
},
{
"id": 151326,
"tgt": "Pain in the spine and shoulders. Taking pain killers. Need advice on treatment",
"src": "Patient: Dear Doctor I am having SPASM (L3 borne deviation) problem for the last 15 years and Once in a year I used to get acute pain in my spinal card and will be completely bed ridden for three days till the paid gets subsidized. Off late say in the last two or three year I am getting heavy pain in my both the shoulders with a small bulging and I am regularly taking pain killers. Due to the above said problems I am unable to carry out my regular activities even combing for a while (say less than a minute) and also unable to type (though I type fast) to respond emails. Please advice for suitable treatment. Regards Doctor: Hello, You need to be evaluated for any compression of nerves in the neck region which can cause shoulder pain. Though bulging is not associated with that. Difficulty in combing etc also suggests that there may be compression in the neck region. However, there may be bony problems without neurological involvement which can present like this. You need to first be seen by an orthopedician and subsequently by neurologist if required. Good luck."
},
{
"id": 139582,
"tgt": "Suggest treatment for grand mal seizures",
"src": "Patient: I am being forced to self medicate medicine via telephone prompts, I cannot recognise visually my medicine, I suffered a sub achnoroid brain haemorrage & now have grand mal seizures, my carers telephone as it is easier & cheaper but it is not working correctly, what can I do Doctor: Hello, In your case, evaluation by a Neurologist is necessary. EEG and brain imaging also are necessary for a complete evaluation and diagnosis. After such an evaluation you can be treated correctly. Hope I have answered your query. Let me know if I can assist you further. Take care Regards, Dr. Erion Spaho"
},
{
"id": 124859,
"tgt": "What is the bruise and swelling on my right arm?",
"src": "Patient: I think I have a torn muscle. My left arm feels bruised by the elbow and it feels like a long swollen thing (for lack of a better word..it s not a bump) It feels bruised right to the middle of my forearm and just up to the right side of my elbow. it is a muscle tear? I can still move with out pain...it s only when I touch it. Doctor: Hello, It is quite common after blunt trauma. It is nothing but an extravasated blood underneath the skin. Generally it will settle by itself. If persist you can consult a physician and get evaluated. In that case We have to rule out possible causes like bleeding disorders. Hope I have answered your queries. Let me know if I can assist you further. Take care Regards, Dr Shinas Hussain, General & Family Physician"
},
{
"id": 8046,
"tgt": "Bump on back with pus which looks like a black head",
"src": "Patient: My Dad had a large lump on his back shoulder that looked similar to a black head . But, it was larger than a usual one and more under the skin. My mom decided to pop it, and the discharge was similar to one like a pimple but was much less oily and smelled bad. The other thing is that a lot of puss came out. Do you have any idea what it could be? My Dad is in his late twenties and has no severe medical history . He has delt with mild acne before but nothing like this. Doctor: Hi, It seems to be sebacious cyst on back. You may consult a homoeopath or contact a surgeon. In homoeopathy it can take few month to cure."
},
{
"id": 42313,
"tgt": "Suggest treatment for infertility problem",
"src": "Patient: Hi, I am 28 and me and my husband started planning baby from 2 months. Last month just when my periods ended we had sex regularly till just few days before my next period. my husband is 31 of age. But now i got periods again. Can you suggest something. I didnt consulted any doctor before getting pregnant and also had never taken any pills for birth control. Doctor: Don't worryWait and seePractice sex regularly or alternative day after mensIf not pregnant within six monthsThen do Semen Analysis and consult with Gynecologist."
},
{
"id": 94750,
"tgt": "Ultrasound shows swollen intestines, abdominal and back pain, nausea, foul smelling diarrhea with blood and mucus",
"src": "Patient: Hello, I went to my obgyn a couple days ago because I was having symptoms of pregnancy. They did a pregnancy test and ultrasound that both came back negative. However in my ultrasound it showed that my intestines were swollen. The symptoms i am suffering from is abdominal and back pains, severe nausea , foul smelling diarrhea that has been both bloody and with mucous. Do you know what I may be suffering from? Doctor: Hello first get your stool examination to find out the cause of dysentry it can be due to amoebic infection or enteric bacteria treatment includes fluids, oral or intravenous depending upon hydration status plus antibiotics depending upon cause"
},
{
"id": 149762,
"tgt": "In ICU for Guillain Barr'e syndrome, breathing trait in mouth. Due for surgery. Chances of success?",
"src": "Patient: I just came from the hospital to see my sister and she is in ICU and her sickness is Guillain -Barr'e SyndromeI have talk to her Doctor about how long is this syndrome going to last he could not give me and good answermy sister is 52 years old and she have a breathing trait in her mouth. and Monday she have to go to surgerywill she make through her surgery. Doctor: Hi and thank you so much for this query.I am so sorry to hear that your sister has Guillain Barre syndrome(GBS). i wish her the very best of outcome with this.GBS treatment is mainly medical(plasmapheresis and intravenous immunoglobulins) and supportive. This is followed by intensive physiotherapy sessions for rehabilitation. I do not know of any surgical treatment as of date for this condition. Please, check with the physicians on why they want to take her for surgery. May be for another reason. And if it is for another reason and not an emergency, then i will differ with the decision at this time. Please, get the fcts right.I hope you find this helpful. Please, feel free to ask us more question if they do come up.Thank you so much for using our services and hope you do when the need next arises.I wish her a rapid recovery and good health.Dr. Ditah, MD."
},
{
"id": 194095,
"tgt": "What causes lack of feeling while ejaculating and less semen?",
"src": "Patient: hi my name is mark, I have been taking pasvax xl 400 for the last month or so. I have noticed over the last two weeks that when i ejaculate that i dont have the normal intense feeling. Also i dont seen to produce as much semen. Today during penetration i had a discomfort, almost as if i had ejaculated and was trying to have sex straight away. I waited five minutes for this to pass and then had intercourse and ejaculated same as above. Could this be related to this medication. Am very worried. Doctor: Hi, Most of the medication taken for prostate hyperplasia can cause erection issue but Pamsvax XL 400 (tamsulosin hydrochloride) has very little effect on erection. Still, if your issue persists need to talk to your urologist for change of medication. Hope I have answered your query. Let me know if I can assist you further. Regards, Dr. S.R.Raveendran, Sexologist"
},
{
"id": 30559,
"tgt": "What causes repeated attacks of mild typhoid?",
"src": "Patient: i have had 2 bouts of very mild typhoid in the last 10 months, and symptoms have returned for the third time now. I have never had a positive blood culture, only widal has been positive. I was taking azhithral and taxim O .. why am i getting this again and again and what shld I do now? Doctor: hi sir, welcome to HCG, once widal is positive, its positive for 6 months, if u take treatment it will decrease not completely gone, and its not a big deal, ur antibiotic treatment is also good.. u have to follow for 7 days atleast the treatment, hope u understand my answer, thank u.. no need to worry"
},
{
"id": 13506,
"tgt": "How do I get rid of itchy allergic rashes on my face?",
"src": "Patient: Hi there, I made the mistake of using an expired collagen facemask on my face. I had a terrible allergic reaction which started with a rash and now my skin has a bumby extremely dry barrier, I have tried everything :( can you give me an idea of how to calm this dry skin and clear this up? Doctor: Hi, If you suspect that the allergic rash is due to collagen mask, you should avoid using the mask strictly. If you have itching, you can take an antihistamine tablet. Dry skin can be treated with application of emollient creams such as Cetaphil DAM moisturizing cream. For the management of rash, I request you to consult your Dermatologist. Hope I have answered your query. Let me know if I can assist you further. Take care Regards, Dr. Siva Subramanian, Dermatologist"
},
{
"id": 137324,
"tgt": "Suggest treatment for swollen and painful ankle",
"src": "Patient: Hi I snorted coke 2 days ago roughly 48 hours exact. About a hour or 2 after I done the drug my ankles swelled up. There is no feeling lose or pain my breaths a a little shy. My mind seems the same. Only other symptoms is my muscles feel tense when standing or walking. Should I seek medical attention or eat a banana and do the invisible bicycle on my back? Doctor: Hello, I have studied your case. Yes you should consult your doctor.Drinking plenty of fluids to avoid dehydrationStretching your leg muscles or riding a stationary cycle.Taking diet rich in calcium and potassium or oral supplements of the sameCheck your vit B12 and Vit D 3 levels.Till time, avoid lifting weights, You can consult physiotherapist for help.Physiotherapy like ultrasound and interferential therapy will give quick relief.Hope this answers your query. If you have additional questions or follow up queries then please do not hesitate in writing to us. I will be happy to answer your queries. Wishing you good health.Take care"
},
{
"id": 126896,
"tgt": "What can cause cold sensation and soreness in the ankle?",
"src": "Patient: For about 2 weeks my Achilles tendon was hurting when I walked. After walking a while it would go away. Well the other day I woke up and no more pain, I thought that it had fixed itself. Today I am getting a cold sensation in my ankle and slight soreness. Not sure what is going on. Doctor: Hi, It can be due to minor contusions or ligament sprain. As a first line management, you can take analgesics like Ibuprofen or Diclofenac for symptomatic relief. Apply ice packs and if symptoms persist, better to consult an orthopedician and get evaluated. Hope I have answered your query. Let me know if I can assist you further."
},
{
"id": 95859,
"tgt": "Suffering from severe constipation with loss of appetite. Also have right abdominal pain from last 2 years",
"src": "Patient: i m male,27yrs. im problem having right side abdominal pain since last 2 years..before 5-6 month i have been getting another problem that is hard and dark stool but there is no pain in anus. my ultrasound , occult blod _ve, upper indoscopy and sigmoindoscopy is normal.. doctor other symtom is only loss of appitite. urine pass normally. no swelling in the neck..every day i drink water atleast 2 ltrs...doctor every morning stool not easily goes i must pus. its verry hard..doctor wht kind of problem is this... Doctor: Welcome to Healthcare Magic Good Day This could be due to sedentary lifestyle. Fast walking everyday for atleast 40 minutes to 1 hour. Eat more palak, spinach, broccoli in the diet everyday. Eat fresh fruits like banana, mango everyday. Drink 6-8 glasses of water everyday. Don't take tension. Do Yoga or meditation, deep breathing exercises to help relax. Be cool. You will be fine."
},
{
"id": 211196,
"tgt": "How to tackle a person who does not let anyone drift away from him?",
"src": "Patient: We have a family friend who lives in Japan whom we have known for many years we live in the us). When we met, he seemed like a normal person. He has an exceptional ability with numbers (being able to mentally calculate 6~7 digit numbers in seconds) which we found amazing. But aside from that, we have been noticing that he severely lacks social skills; as he does not seem to have any friends. To give an example, he recently contacted my brother over the phone several times and by email; when he did not receive a response, he said that he is flying over to see us. As in other instances, he is very persistent and refuses/cannot have anyone drift away from him. Doctor: Exceptional math skills !!That's wonderful.A few people are gifted.And a few people have the misery of lacking social skills.This need not be a mental issue per se.Try to keep him involved with people he understands and share the same interests.And just when the friendship is deep enough between them,try to disarm the social bomb making him aware of what boundaries are.Such people might get hurt pretty easily, so be sure you are polite and convey the message straight enough.Good Luck !"
},
{
"id": 46347,
"tgt": "Could hematuria and lower to mid back pain be related to kidney infection?",
"src": "Patient: Post menopausal with blood in urine and lower to mid left back pain.feel swollen all over. Hx of kidney infections as a child and 2 episodes of really bad kidney infection, one 30 years ago and another about 15 years ago. I am 64 and in good health except for this. Is this another kidney infection? No problems with burning urination or pain in that area. Doctor: Hi and welcome to HCM. As an Urologist, i can understand your anxiety.Blood from any orifice is abnormal.This could very well be an UTI.(infection).You'll need to get examined by an Urologist and do the basic tests like :1. urine routine and culture.2. blood routine,creatinine,RBS,LFT,and platelets.3. Ultrasound -KUB,with residual urine.4. A cystoscopy with/without a CT-IVU,may be done,after seeing reports.If you've any doubts,send a direct question in my name,with all the reports.Dr.Matthew J. Mangat."
},
{
"id": 113421,
"tgt": "Distended stomach after previous childbirth. Looks like pregnant. Having back pain. Suggestions?",
"src": "Patient: I had my last child 7 years ago since that time my stomach has never gone back to how it should have. I get alot of lower back and upper back pain and if i am out i have to sit down and rest often before i can walk some more. People ask me if i am pregnant. Mind you I do look like i am 6-7 months pregnant. I dont know what to do, its been getting me down for so many years. I am a single mother of 40 years old. with two children 12 & seven years respectively. Doctor: Hi thank you for sharing your problem. It could be that your lower back has arched forward more than it should due to slippage of back bone called spondylolisthesis. So please get xray of your lower back and revert to me thank you regards"
},
{
"id": 42623,
"tgt": "Which among IUI and laparoscopy is best while having PCOD?",
"src": "Patient: hi I am 23 yr old.. Married for 1 yr &10 months . we diagnised that i have PCO's and in my right ovary there is a 1.8 *2.2 cms cyst. i took medicine for 2 months, past 3 months of scan says, the same measure of cyst ... no increase or decrease in size.... since getting pregnancy is delayed with irregular periods, doctor has now suggested to go for an IUI this time & if not successful will do laproscopy.................Which is good to me........ Doctor: Haiwelcome to hcm you can try ovaral contraceptive pills for 4 months. This is very small cyst which will disappear with pills. If it fails you need laparoscopy .you can have iui after disappearance of cyst. Regards Dr. Vanitha devi"
},
{
"id": 219592,
"tgt": "Suggest ways of conceiving a baby",
"src": "Patient: Hi I am 20 years old and I have 1 son who is 4 and me and my boyfriend has been trying to work on a baby for 3 years and we have been together 4. And I was wondering about taking the prenatal pills and my questions for you are. How much does the pills cost? Where can you get the pills at? Do your baby still comes out looking like you and your baby s daddy? And how long does it takes to work? Doctor: Hi.The pills will be quite affordable, and you should be able to get about 60 tablets (2 months supply) for anything around 5-10 $. You should be able to get the pills at any medical store, just ask for pre-natal supplements or take folic acid supplements. Haha, yes of course your baby will come out looking like you, or your babys daddy, or a mix of the two of you, the pill only helps with preventing growth and developmental abnormalities. The pills are just nutritional supplements, and provide the nutrients your baby needs to grow and develop within your womb, so it basically works immediately after being taken.Are you sure you are taking about pre-natal supplements? Or pills that make you more fertile?Best wishes."
},
{
"id": 160095,
"tgt": "How long it will take to recover after thymectomy?",
"src": "Patient: I am a 50 year old female, had a thymoma removed 2 months ago via sternotomy. They resected the anterior portion of my pericardium and surrounding lymph nodes. The thymoma was mixed AB but encapsulated. How long is it normal to have fluid around the heart and lungs post op? Are there any long term effects from having a portion of one s pericardium resected? Also, how long before I can resume my baseline activities? I used to lift weights and do chin ups...with emphasis on heavy weights, fewer reps. Doctor: Thanks for the query I would like to know how much of the pericardium was removed to tell u how much of long term problems u will face. Let the surgical wound, and sutures come down, u must feel energetic enough to resume your activities. On your next follow up please ask ur doctor as well Have a healthy living"
},
{
"id": 17778,
"tgt": "Suggest treatment for blockage in main artery",
"src": "Patient: by brother in law had just got an angioplasty done last week and two days before he was discharged from the hospital. Today suddenly he could breathe and his body went cold he is shifted in the iCU on is on ventilator. He had 100% blockage in the main vein in left ventricle with ejection fraction only 25%. The doctors have told that the stent has not fitted well. Is his conditions very serious can be recover Doctor: Hello, I would explain that his clinical situation seems to be critical. We can not make any predictions as he is experiencing acute heart failure caused by a massive heart attack. For this reason, the situation is life-threatening (serious cardiac arrhythmia and death may occur). Hope I have answered your query. Let me know if I can assist you further. Take care Regards, Dr Ilir Sharka, Cardiologist"
},
{
"id": 144606,
"tgt": "How can multi-infarct dementia with memory loss be managed?",
"src": "Patient: Mr. Black age 89 has multi-infarct dementia with severe memory loss ,impaired judgement and mood swings. He is also profoundly deaf and has some visual problems. He is increasingly weak and unsteady on his feet and needs to be supervise and assisted living centre in a long term care facility. He won t join any activities and often refuses to go to the dinning room for his meals.sometimes he expresses angry outbursts and is often found crying in his room. He says that he should be able to go out and have a job again.he feels that everything has been taken away from him and often expresses that his life is worthless and he wishes he die. Mrs. Black visits her husband everyday and attempts to rationalize with him and talk him out of his bad moods she appears to care for him very much and often tidies his room and changes his clothes. He usually unloads all of his feelings on her and she goes away feeling frustrated and guilty..how failing physical health brain damage ,institutionalizations and loss have affected mr black mental health and promoted a cycle of deterioration. Doctor: These symptoms seem behavioral and psychotic symptoms associated with multi infarct dementia. Its need multimodal management including medicines + good nursing & home care.1. For prevention of further infarcts- Ecosprin & lipid lowering drugs2. management of risk factors of multi infarct - Hypertension , diabetes mellitus etc.3. antidepressents as you say that he expresses his life worthless4. antipsychotic medicines if he has some self harming behaviour or aggresive behaviour"
},
{
"id": 182262,
"tgt": "Suggest treatment for dark red spot on gums",
"src": "Patient: After brushing my teeth and flossing this morning, two dark red spots appeared on my bottom gums. Its not very big but it is a lot darker than the rest of my gums. It looks like skin came off or something. It isn't very painful and its not raised. What should I do? Doctor: Treatment for Dark red Gums if the cause is gingivitis, than the dentist would opt for removal of tartar and plaques from the gum\u2019s and teeth.Salt is excellent for oral hygiene, gargling regularly with salt water keeps the gum\u2019s disease at bay. It also helps to relieve pain caused by gum disease.Ice applied on the inflamed gum\u2019s gives quick relief Clove oil is another natural oral antiseptic for inflamed gum\u2019s."
},
{
"id": 3084,
"tgt": "Is pregnancy possible naturally without IVF or medication?",
"src": "Patient: I had a miscarriage at 5 weeks this june then had an ectopic pregnancy this september at 5 weeks too, I had a left laparoscopic salpingectony with a d&c. Is it possible that i can get pregnant naturally without ivf or medication to help me ovulate? Im only 29 yrs old. Doctor: Hi there,Welcome to HCM,I done think you have a problem in conceiving, you have conceived twice in a year, but unfortunately both pregnancies had complications. But the fact that you conceived naturally remains.Missed abortions are common in first pregnancies, and since you already had the ectopic pregnancy with tube removed, you have one tube which is remaining.This means you can definitely conceive yourself naturally without IVF or medications. Ovulation does not seem to be a concern for you, as you have got pregnant before. Women who have ovulaution issues will not get pregnant at all.So please try to get pregnant and once urine test positive see gynecologist immediately.Hope this helps.Regards."
},
{
"id": 41715,
"tgt": "Is it ok to have sex when having thick vaginal discharge?",
"src": "Patient: im trying to get my wife pregnant. 20th of the month is her period day starting.. so when can i try a home pregnancy test? if i make a test today is it k? her vaginal discharge is very thick nd milky like waste coming from nose.. what is dat? is to k to continue sex during these days.. Doctor: HI, I understand your concern. _ you can perform pregnancy test- Home test(urine) after 8 days delay in her period. Blood HCG testing can be performed day around her next expected date of menses. -Actually sex while she has thick white discharge is less likely to be fruitful. -as it may hamper movements of sperms. * You should ideally consult a gynecologist * You should check her ovulation day by (home) LH surge test & have sex around ovulation day. * if there is no positive result in 3 months both of you need to be investigated further for getting help in conception. Thanks."
},
{
"id": 108577,
"tgt": "Is Percocet effective for low back pain?",
"src": "Patient: I am having terrible chronic low back pain from an injury (house work) approx 3-4 months ago.I have tolerated Percocet 10/325 previously fro this injury. I was hoping to do an online interview for a prescription of Percocet. I need help please. State TN. Doctor: Dear patient Percocet is combination of oxycodone and acetaminophen or paracetamol. Oxycodone is opioid drug which is centrally acting pain reliever and habit forming if taken for long time. It is very effective for back pain. It has side effects of respiratory depression,addiction and constipation. Paracetamol is nsaids group drug and increases effects of oxycodone. It has side effects of liver damage if taken in excess. Rare skin reaction is also noted. So combination has synergistic effect in pain relief. Dosage is one tablet twice a day initially and then as and when required. Avoid taking drug if you have asthma , cirrhosis or allergy. All the best."
},
{
"id": 124347,
"tgt": "Experiencing pain in left arm & shoulders after going through a bypass",
"src": "Patient: It s been 6 months since my significant other received a double bypass. He has complained about pain in his left arm and shoulder which seems to be getting worse. He is only able to lift his arm chest high without pain. The pain wakes him up at night. His cardiologist says it s most likely nerve damage from the way they positioned him during the long surgery or a pinched nerve.He never went to any physical rehab after surgery, they kept telling him he wasn t ready yet but then we moved from NY to Texas and the new cardiologist made it sound that after 6 mnths it wouldn t help him anyway. Should he see a neurologist for this ? Thank you for your reply. Joyce Fagan Doctor: Hello, My opinion is It's better late than never. Let's come to the discussion where we can find some possible solution for the patient. It is true that after any cardiac surgery one needs to undergo a proper cardiac rehabilitation program under an expert physiotherapist. Now, you are saying that the patient has Complaints in his left arm and is not able to lift the arm. Please make it assure that the cardiac routine diagnostic procedures are done before we come to a neurological point of view. Is the pain he felt is due to cardiac of not that is the first thing we have to keep in mind. If there is no cardiac issue and is ruled out that one needs to not worry and we can jump over the surgery part. Now, let us make it clear, that due to bypass surgery there will be shortening of the pectoral muscles and it needs a good stretching so the shoulder stays in a normal position wherein the biomechanics of the shoulder do not get disturbed and the functionality of the shoulder remains intact. Coming to the neurological point of view, I don't think he will have any nerve compression. Because if it is nerve compression then the symptoms will be like tingling, burning, numbness etc. Now, let's assume that he doesn't have any cardiac or neurological issue then it is straight the job of the physiotherapist to help him out. Undergoing a proper rehabilitation program should definitely help him lift his arm up. Also a good strength in the muscles of the arm is needed to acquire the optimum functional ability. Also breathing exercises should be made a part of the overall exercise program as it has got to do a lot with the overall well being of the patient. Please take the call based on the guidelines. First the heart needs to be assessed. then the surgical part if any muscle is shortened and leading to change in the posture which is changing the biomechanics of the joint. And lastly, the neurological point of view if we rule out step by step all things then definitely we can come to a proper diagnosis and help the patient to have a good lifestyle change. Hope I have answered your query. Let me know if I can assist you further. Take care Regards, Jay Indravadan Patel, Physical Therapist or Physiotherapist"
},
{
"id": 79128,
"tgt": "What causes coughing fit and sudden unconsciousness?",
"src": "Patient: I CAME BACK FROM A CRUISE ON THE WEEKEND AND DEVELOPED A REALLY BAD HEAD COLD, YESTERDAY I WAS DRIVING TO WORK AND WENT INTO A COUGHING FIT, I WOKE UP A FEW SECONDS LATER ON THE OTHER SIDE OF THE HIGHWAY AND REALLY DISOREINTATED, IT ALSO HAPPENED AGAIN IN THE AFTERNOON I HAD A COUGHING FIT AND WOKE UP ON THE FLOOR A FEW SECONDS LATER Doctor: thanks for asking your questionhaving an episode followed by unconsciousness is dangerous so you should stop driving and roaming aloneyou should visit a neurophysician who can conduct some investigations to rule out epilepsy and accordingly you can be treated he may request for a ct head and some neurophysiological studiesthanksfeel free to ask more questions"
},
{
"id": 205938,
"tgt": "Suggest treatment for dizziness and panic feeling while driving",
"src": "Patient: Hi, two days ago whilst driving I suddenly felt dizzy, struggled for breath and felt panicked, I just had to grip the wheel and focused on the car infront, this past after a minute or two. Again today it happened not for as long and my breathing was better but had a tired feeling on the left side of my neck. Please help. Thanks, Wendy Doctor: Hello,Thanks for using healthcaremagic.I read your query and understand your concerns.In my view you need to visit a neurologist as soon as possible if you are above 50 years of age as it may be TIA if you have the risk factors for same.In case you are middle age women the chance of panic attack is high according to your description and seeking psychiatric help will be appropriate.For the treatment of panic attacks both benzodiazepines and SSRI are effective and routinely recommended for same.Your psychiatrist may also choose psychotherapy for treatment of panic attacks.Other than than when you get it stop driving and wait till the attack is over. It will be in interest of your safety and health.Thanks and regards"
},
{
"id": 85090,
"tgt": "Safe to take Magrim Diet Pills along with thyroxin for hypothyroidism?",
"src": "Patient: hi, i have Hypo thyroidism , and my tests of T3 and T4 and TSH are within the normal range, my thyroid gland is working but i take thyroxin 50 mg daily just to help my thyroid.....is it ok to take Magrim Diet Pills. please i need a quick reply as i already started using magrim 4 days ago. Doctor: Hi, It is not safe to take megrim diet pills while on thyroxine. Magrim Diet Pills promoted by street-vendors and websites as an efficient natural product for weight losing in the treatment of obesity. It contains a compound 'sibutramine' which can increase the risk of heart attack and stroke and hence withdrawn from the market because of those dangerous side effects. It should never to taken along with thyroxine as the risk of heart attack and stroke may increase by several times. Hope I have answered your query. Let me know if I can assist you further. Take care Regards, Dr. Mohammed Taher Ali, General & Family Physician"
},
{
"id": 96248,
"tgt": "Terrible pain in stomach",
"src": "Patient: Hello I m having this terrible pain in my stomach but it only hurts to the touch. Doctor: hello i'm having this terible pain in my stomach but it only hurts to the touch."
},
{
"id": 32923,
"tgt": "Suggest treatment for severe spider bite on the foot",
"src": "Patient: I was recently diagnosed with an insect/spider bite on my foot by a doctor in Italy. I am now in Greece on holiday. The only symptom is swelling and redness on the foot. Today, however, I discovered a lot of blood in my stool. I have no pain, no fever. Only slight discomfort and swelling in my foot Doctor: HI, thanks for using healthcare magicThe spider bite and the lesion on your foot should not cause blood in the stool, the two are not likely related.Blood in the stool can be due to different causes including : (1)as a result of straining and constipation(2) anal fissure ,this however would normally be painful(3)hemorrhoids(4)diverticular disease(5) benign polyp in the colon(6)cancer of the colon(7)infection(8)Crohn's or ulcerative colitisExamination of the abdomen and anus, analysis of a stool specimen and imaging may be needed to identify a cause.I hope this helps"
},
{
"id": 5071,
"tgt": "Trying to conceive. Stressed out. History of having handicapped twins and myomectomy. Guide for healthy pregnancy and healthy child",
"src": "Patient: I am Archana Priya, 37 year old lady. Have two prematurely born totally handicapped twins (born Jan. 2005). Undergone Myomectomy in 2011. Present tests suggest everything normal with me and my husband. Trying since last two years for a normal , third baby. Both of us highly stressed due the growing abnormal kids ; therefore not able to give enough time to each other. I dont want to go thru IVF etc. Please suggest and help having a baby. Doctor: hello,thank you for using health care magic,i understood your problemfirst of all stay relax and dont get stressed out easily... you should change your thinking.. be positive..you and your husband should be supportive to each other.. take out time for your self .. if you need you can take counselling sessions from psychiatrist .. it will reduce your stress and stress definately decreases fertility ..if you want consult gynecologist for fertility check up since you having preterm handicapped twins..gynecologist and genetic work up will help you in finding chances of abnormal baby in future..hope this helped you.. take care"
},
{
"id": 208879,
"tgt": "Can Vyvanse cause tiredness with a lack of motivation?",
"src": "Patient: I have been taking vyvanse for about 5 years now. The first couple years I was talkative, I felt normal. Now the years go on and I feel like now when I take it I lose my personality. I wake up early at 7 and work from 8 to 5 every day, by the time its 5, I feel very drowsy, tired, no motivation, not talkative. Could this be an issue due to vyvanse? Or could this be an issue to drinking a lot of coca cola every day? One more question. I take 60 mg of vyvanse and I drink about 3 20 oz coca cola s a day, is this bad when on vyvanse? Doctor: Hello,Thanks for choosing health care magic for posting your query.I have gone through your question in detail and I can understand what you are going through.It could be a persisting depression. Just have some patience and consider changing the antidepressant. Hope I am able to answer your concerns.If you have any further query, I would be glad to help you.In future if you wish to contact me directly, you can use the below mentioned link:bit.ly/dr-srikanth-reddy\u00a0\u00a0\u00a0\u00a0\u00a0\u00a0\u00a0\u00a0\u00a0\u00a0\u00a0\u00a0\u00a0\u00a0\u00a0\u00a0\u00a0\u00a0\u00a0\u00a0\u00a0\u00a0\u00a0\u00a0\u00a0\u00a0\u00a0\u00a0\u00a0\u00a0\u00a0\u00a0\u00a0\u00a0\u00a0\u00a0\u00a0\u00a0\u00a0\u00a0"
},
{
"id": 193973,
"tgt": "Suggest treatment for aligospermia",
"src": "Patient: Sir I am suffuring from alingospermia. i have udergone undeseded testcals operation at the age of 4years. Now i am facing the problem. I am married now. My left testcal is atopy and the right one is functional. I have treated by Homeapathy doctors from aleast 4 years from now, but no results. Can u suggest me any treatment please. Doctor: Hello, The usual treatment is using surgery and medicines for inflammation. Some life style changes can also help. Do talk to your urologist. Hope I have answered your query. Let me know if I can assist you further. Regards, Dr. K. V. Anand, Psychologist"
},
{
"id": 106708,
"tgt": "How can severe backache be treated?",
"src": "Patient: I WENT TO THE ER YESTERDAY WITH SEVERE BACK PAIN. I HAVE HAD 4-5-6 FUSION SURGERY ON MY BACK IN 2008. I AGGRAVATED THE BACK OVER THE WEEKEND AND I COULD BARELY WALK AND NEEDED HELP GETTING UP. I AM 66 AND AM A MILD HEMOPHILIAC(A). I WAS IN SO MUCH PAIN I THOUGHT THE MEDICINE WAS A PAIN SHOT. I CHECED YOUR WEBSITE AND REALIZED IN WAS AN NSAID, WHICH IS A NO-NO FOR ME. AM I IN DANGER? Doctor: Hello and Welcome to \u2018Ask A Doctor\u2019 service. I have reviewed your query and here is my advice. NO your not in danger, but NSAIDs do increase chance of kidney damage and stomach bleeds etc, if you took one time NSAID you shouldn't necessarily be in danger. Hope I have answered your query. Let me know if I can assist you further."
},
{
"id": 125480,
"tgt": "What could cause stiffness and numbness in the lower legs and ankles?",
"src": "Patient: Have stiffness & numbness in lower legs, ankle areas. I use a walker because I have some balance issues & don t want to fall... taking B12 supplements & diagnosis of spinal stenosis several years ago, but I could drive & walk ok. Seems worse, could it be anything else ??? Doctor: Hello, It could be an early sign of arthritis. As a first line management, you can take analgesics like Paracetamol or Aceclofenac for pain relief. If symptoms persist better to consult an orthopaedician and get evaluated. Hope I have answered your query. Let me know if I can assist you further. Take care Regards, Dr Shinas Hussain, General & Family Physician"
},
{
"id": 167451,
"tgt": "What causes bloody stools and IBM after treatment for fever in kids?",
"src": "Patient: my 3yr old daughter has been fighting a fever & constipation. we took her to the doctor n they gave her gentlelax. she did her time taking the meds & now she s done but now she s starting to have bloody stools & painful bowel movements. what should i do? Doctor: Blood in stools can be due to cut over anal skin because of hard stools or it could be because of infection in intestine. I suggest you to do stool routine microscopy test. This will help in confirming the diagnosis."
},
{
"id": 27394,
"tgt": "What does my angiography report suggest?",
"src": "Patient: I am 45 i had a PTCA + Stent to LAD done on 17.11 2011 angiography revealed a Double vessel initial decision was to stent the OM but after predilation in LAD Thrombosunction was done after 30 minutes the thrombus was still found to be increasing and mobile decision was taken to stent LAD ostium Doctor: Decision for stenting LAD wash perfect. If not done on time, would have caused massive heart attack and could prove life threatening."
},
{
"id": 149351,
"tgt": "Has looped blood vessel putting pressure on cranial nerve with facial twitching and throbbing inside skull. Suggest?",
"src": "Patient: Dear Doctor, My almost 80 year old father has a looped blood vessel putting pressure on a cranial nerve and causing him to experience facial twitching and throbbing inside his skull that he has been taking anti seizure meds and botox shots for, but now he is scheduled for surgery where the doctor wants to remove part of his skull to put a cushion between the nerve and blood vessel......Do you know of this type of condition and treatment options? Doctor: Hi,Thank you for posting your query.Based on your description, I think your father has hemifacial spasm, and there is a blood vessel loop around his facial nerve, causing him facial twitchings, spasm and pain.Some people respond to carbamazepine and botox. However, those who do not respond to these, would respond to the surgery.However, please discuss the risks of surgery in a 80-year old person with the surgeon.I hope it helps.Please get back if you require any additional information.Best wishes,Dr Sudhir Kumar MD (Internal Medicine), DM (Neurology)Senior Consultant NeurologistApollo Hospitals, Hyderabad,My personal URL on this website: http://bit.ly/Dr-Sudhir-kumar My blog: http://bestneurodoctor.blogspot.com/"
},
{
"id": 158837,
"tgt": "Mammogram done. Fibroid s or cancer?",
"src": "Patient: Hello - I was just called yesterday from my hospital and was told I needed to be re-checked after a mammogram was done last Friday, April 19th. I was told that changes have occurred since my last check over a year ago and that I had two focal areas of breast nodules that area size 12x5x5 mm; and another 10x3x3mm. Of course, I am concerned and no one will no if they are cancerous until I am re-checked...just wondering if that is a common size for fibroids or just fatty tissue. I am going back for pictures and an ultrasound. Doctor: Hello Welcome to healthcare magic. It is not possible to tell about type of tumor from size because sometimes even small tumor are cancerous while big tumor can be non-cancerous. One way of knowing what type of nodule you have is looking at BIRADS catagory of lesion.It is nothing but standard way of catagorizing finding of mammogram in scale of 0-6 Category\u00a0\u00a0\u00a0\u00a0\u00a0Diagnosis\u00a0\u00a0\u00a0\u00a0\u00a0Number of Criteria 0\u00a0\u00a0\u00a0\u00a0\u00a0Incomplete\u00a0\u00a0\u00a0\u00a0\u00a0Your mammogram or ultrasound didn't give the radiologist enough information to make a clear diagnosis; follow-up imaging is necessary 1\u00a0\u00a0\u00a0\u00a0\u00a0Negative\u00a0\u00a0\u00a0\u00a0\u00a0There is nothing to comment on; routine screening recommended 2\u00a0\u00a0\u00a0\u00a0\u00a0Benign\u00a0\u00a0\u00a0\u00a0\u00a0A definite benign finding; routine screening recommended 3\u00a0\u00a0\u00a0\u00a0\u00a0Probably Benign\u00a0\u00a0\u00a0\u00a0\u00a0Findings that have a high probability of being benign (>98%); six-month short interval follow-up 4\u00a0\u00a0\u00a0\u00a0\u00a0Suspicious Abnormality\u00a0\u00a0\u00a0\u00a0\u00a0Not characteristic of breast cancer, but reasonable probability of being malignant (3 to 94%); biopsy should be considered 5\u00a0\u00a0\u00a0\u00a0\u00a0Highly Suspicious of Malignancy\u00a0\u00a0\u00a0\u00a0\u00a0Lesion that has a high probability of being malignant (>= 95%); take appropriate action 6\u00a0\u00a0\u00a0\u00a0\u00a0Known Biopsy Proven Malignancy\u00a0\u00a0\u00a0\u00a0\u00a0Lesions known to be malignant that are being imaged prior to definitive treatment; assure that treatment is completed If the lesion is suspicious,doctor will do Fine needle aspiration cytology to determine type of the nodule. So no need to worry. Regards"
},
{
"id": 127961,
"tgt": "What causes pain at back of armpit?",
"src": "Patient: I am having an occasional shooting pain at the back of my left armpit three inches below the beginning of the upper arm (about even with nipple). It hurts when I rub the skin of my armpit with my arm and seems to be in the skin, not the underlying tissue. There is also a slight numbness in the same area. Doctor: Armpit or underarm pain can be caused by a number of things that include pinched nerve, pregnancy, cancer, swollen armpits and lymph nodes, muscle twitch, breastfeeding, waxing, cystic pimple, cancer, among other causes.but in your case there may be-Brachial plexus nerve injuries - Shooting pain Pinched nerves especially brachial plexus from stretching, pressure or trauma can lead to this problem. This is often accompanied by numbness and tingling feeling and pain that resemble electric pulses. But exact diagnosis is made on basis of investigation or examination."
},
{
"id": 102692,
"tgt": "Gained weight after taking Zyzal and singulaire for environmental allergies. Any alternative medicine available?",
"src": "Patient: I have environmental allergies (severely allergic to plastic, rubber, formaldehyde, polyester and many other chemicals) and my doctor put me on Zyzal and Singulaire. I have gained 6 pounds in one year on my small body frame and I can't seem to get it off. Are there other pills I can take without the weight gain side effects?I took Allegra for food allergies and that works but Allegra does not work for environment allergies. Doctor: Hi and thanks for the query,Use of steroids like the drug you used could be linked to weight gain. Depending on the clinical scenario and intensity of symptoms, a balance always has to be stricken concerning weight gain and symptoms. Associated measures to control weight like diet control and exercise are generally important adjuvant measures, especially when drug has to be taken for some time. A review to check on a withdrawal scheme when symptoms are well controlled. Kind regards"
},
{
"id": 154066,
"tgt": "Suggest treatment for lung cancer",
"src": "Patient: Someone i know has been diagnosed with lung cancer 5 months ago. He had hip surgery and gradually the cancer spread. The family has been called in to see him because they said nothing else can be done for him. His breathing is spuratic and he only weighs about 60 pounds. Can you give me an estimate on How much longer he has? Doctor: Thanks for your question on Health Care Magic. Lung cancer is very common in modern era. Since doctors are saying that nothing can be done in his case, this means he is having end stage (stage 4) lung cancer. Treatment options for him is palliative care. Palliative care includes relief of pain, maintain nutrition and provide psychological support to deal with end of life issues. Life expectancy in end stage lung cancer is less than 10-12 months. Hope I have solved your query. I will be happy to help you further. Wishing good health to your relative. Thanks."
},
{
"id": 30683,
"tgt": "What causes green thick mucus in mouth?",
"src": "Patient: i have a problem with green thick mucus which make breath diffuc sometime i tasted for asthma its not but my breath level is not showing great mark breathing,what els can i expect .because my throt gets so dry at times even making it hard to swallow my own saliver it feels like choking , pls help? Doctor: hi, well i would seek fot obstructive diseases in nose and nasopharynx, that are the most common cause of chronic infections, and seems like mucus came from lungs, must discard adenoiditis, nasal polyp, sinusitis, and then go for lungs causes, pneumonia, lung abscess, that for begin"
},
{
"id": 137797,
"tgt": "Suggest treatment for a painful lump and bruising on shin",
"src": "Patient: I have had a lump on my shin, with bruising running down both sides of my shin to my feet. It is now 2 weeks and the bruising appeared 5 days ago the lump hurts at times. Also, I have been getting random sharp pain from my lump area it s very painful too. What should I do Doctor: Hi,Thanks for your query.Painful lump over your shin seems to be inflammatory in nature. I advice you to give you rest to the part affected, take anti-inflammatory drug like motrin 1tablet with food as and when required (upto 4 tablets daily) to reduce pain and inflammation and consult your doctor for thorough examination and rule out any infective cause.I do hope that you have found something helpful and I will be glad to answer any further query.Take care"
},
{
"id": 69292,
"tgt": "What causes a lump in groin?",
"src": "Patient: my 16 mth old has a lump in groin cbc was normal and diff too doctors dont know what it is a lymph node or a cyst scared it cancer or something a mother always worries she is not sick and it doesnt seem to hurt her it is the size of a brown bean and it is not painful when touched Doctor: Hi.I can understand the feeling a Mother has . The baby is just 16 months old. I would like you to examine the peri-anal area , if there is any fissure or excoriation / abrasion of the skin. This area drains into the inguinal / groin lymph nodes. If the finding is positive request your Doctor to have a look at this and a course of an antibiotic and anti-inflammatory medicines will cure this problem.If there is no such findings, you can request for an ultrasonography and guided Fine Needle Aspiration Cytology. This will give the perfect diagnosis and a way for a further treatment."
},
{
"id": 13912,
"tgt": "Is Hydrocortisone advisable for mold allergic rashes on the calves?",
"src": "Patient: I believe that I have been around black mold. Both at work and home, plus I have a lot of stress. Currently my calves are covered in red swollen rash/bites. Plus the bumps have a lot of heat to them. And I keep running a high fever, due to the infection from the mold. With the heat in the rash can I use hydrocortisone cream? Doctor: Hello, The red swollen rash on your calves is a sign of allergic reaction to mold. You are already getting treated for mold infection. Please follow your doctor's advice rigorously. Yes, you can apply hydrocortisone cream on the heat rash. Keep well cleaned the affected area. Raise your legs. Apply cold compresses locally. If no improvement, should consult with dermatologist to rule out possible infection. Hope I have answered your query. Let me know if I can assist you further."
},
{
"id": 225295,
"tgt": "Regularly taking pills. Waiting for periods. What does this imply?",
"src": "Patient: Hi..I am regularly taking gynera contraceptive pill for about 6months already...so far so good but lately while im still on d 18th pill I started to have my period..isn t it I will have my period on my free days wherein I don t have to take any pill? What does this implies? Any significance? P Doctor: Are you taking the pills back to back. ...or are you having the pill free period? ?.because tsking the pills back to back may sometimes causr breakthrough bleeding in between the cycle which is sometimes heavy also...now what you can do is that you stop the pills....get a review usg done....and restart the pills after next periods."
},
{
"id": 113220,
"tgt": "History of stillbirth. Have lower back pain, no periods. What is the problem?",
"src": "Patient: i was pregnant with IUI last year. In jan 2013 (7th month), i had to deliver stillborn baby as the cord was tightened around the baby s neck (3 folds). It is almost 9 weeks now and i have joined back work. But i am experiencing lower back pain . I have also not got my periods yet. Please suggest what could be the problem, and how i can take care of it. Doctor: Hello. Thanks for writing to us. The lower back pain after delivery is commonly due to the postural changes and a decreased level of calcium and vitamin D 3. You need to get these levels checked and start taking adequate supplements. I hope this information has been both informative and helpful for you. Regards, Dr. Praveen Tayal drtayal72@gmail.com"
},
{
"id": 44115,
"tgt": "Difficulty in conceiving, no improvement after taking adova, duphaston and all9. What should be done next?",
"src": "Patient: i m trying for baby still 8 months,i had went to doctor he prescribed hsg test and hsg report is normal and after that doctor prescribed me adova, duphaston and all9 for three cycle already i had completed my two cycle still i m not been pregnant.this is my last cycle i m taking my medicine and i m afraid what will happen if this month also if i wont been pregnant,plz suggest me wht to do next. Doctor: Chance of conception after 12 months of unprotected intercourse is 85%. If your age is <35 years, if you are having regular menstrual cycles, Husband's semen analysis is normal & HSG is fine, then you can safely wait for another 4 months even without taking any medicines (Except Folic acid for NTD prevention)."
},
{
"id": 210900,
"tgt": "Are Paxidep for OCD and Frext, Ativan safe for 18 year old ?",
"src": "Patient: my 18 years old daughter is taking paxidep cr 37.5/day for OCD as prescribed by a psychiatrist after prescribing paxidep cr 25. her board's exam. will start on 12 March. In addition frext cr 100/day and ativan 1mg 2/day are also being taken. As a father worried. Please advise me the right path. Doctor: HiThanks for healthcare magicThere is nothing harm in treatment combination. That would control her obsessive and compulsive symptoms. She is on tab ativan 1mg twice a day. In case, she feels sleepiness due to ativan, she could decrease the morning dose, otherwise carry on with the combination. Thanks"
},
{
"id": 215640,
"tgt": "What are the side effects of Meloxicam?",
"src": "Patient: A little over a month ago I had gastric sleeve bypass done and they told me I would not be able to take meloxicam at all for life. I was taking it for bad knees, stopped three weeks prior to surgery but now they re hurting again like before. So today I started taking my 7.5mg pill and plan to from now on. Question is, what harm am I doing to my body, I don t need possible side effects are there any actual side effects. Doctor: Hello, Meloxicam produces severe gastritis and it may harm your stomach (you have only a portion of stomach left!!) You can consider opioid analgesics like tramadol for pain relief. Hope I have answered your query. Let me know if I can assist you further. Regards, Dr. Shinas Hussain, General & Family Physician"
},
{
"id": 173653,
"tgt": "Suggest treatment to cure cold and lung infection",
"src": "Patient: My kids, elder daughter and younger son of 2 yrs age difference always suffering from cold and lung infection. Dr here says it seems to be sort of veezing. Now medicating with Asthakind, Bricarex and Macpod 100, but still when they cough getting the mucus sound in the lungs. Kindly advise, Doctor: Hi...Thank you for consulting in Health Care magic. Greetings from Chennai.By what you quote I feel what your kid could be having viral associated wheeze or multi triggered wheeze. I have a few questions for you -Questions:1. How many days per month does she cough or feel breathless?2. How many nights per month does her sleep get disturbed due to above symptoms?3. Does she feel breathless when she runs around or plays with other kids?4. Are the symptoms when there are seasonal changes?5. Is there any family history of asthma or any other sort of allergies like skin allergy etc.?6. Is the cough always associated with fever?If your answer is yes for any of the above questions, your kid might be having viral associated wheezing or multi triggered wheezing and I suggest you meet a paediatric pulmonologist who is near your place. No point in using antibiotics every time. It will do more harm than good by increasing bacterial resistance.Hope my answer was helpful for you. I am happy to help any time. Further clarifications and consultations on Health care magic are welcome. If you do not have any clarifications, you can close the discussion and rate the answer. Wish your kid good health.Dr. Sumanth MBBS., DCH., DNB (Paed).,"
},
{
"id": 48489,
"tgt": "What causes pain in the kidneys?",
"src": "Patient: Hi I have had a dull ach in left side kidney for 2 months now went to see doctor 3 weeks ago he took blood and urine samples, blood came back with a high wbc count and urine came back with staph saprophyticus and white blood cells. I have been on antibiotics for 7 days and pain to my left kidney is still there i am also getting sharp pains thus has only happened a couple of times in the last week i have also had swelling to my right ancke for nearly 4 weeks please could you tell me what this might be thanks lols Doctor: Hellothere might be stones or infection in your kidneys,you need to get a ultrasound scan for diagnosing the cause"
},
{
"id": 203521,
"tgt": "Can any online doctor advise on semen analysis report?",
"src": "Patient: Dear sir, How are you now? This is Mahbubur Rahman from Bangladesh. I have collected my semen analysis report and it is Sperm count:53.2 Million/ml Motility: Incubation Active motile: 50% Sluggishly motile: 20% Non motile: 30% Pus cells: 1-2/hpf RBC:1-3/hpf Morphology Normal: 80% Abnormal: 20% Chemical Examination: Fructose: Present May I know some thing about it? Doctor: hiiii,.Your semen analysis report interpretation as follows>>1) Sperm count 53.2 million/ml>> NORMAL (Normal value 20-150 million/ml)2) Sperm motility active 50%,sluggish 20%,Non motile 30% >> BORDERLINE to NORMAL (50-60% forward motile sperm is ideal)3) Morphology 80% normal>> NORMAL (At least 70% normal morphological sperm is ideal)4) Pus cell 1-2/hpf and RBC 1-3/hpf>> within NORMAL limit.Your overall semen analysis report is normal. Take healthy diet and avoid stress.All the best."
},
{
"id": 85703,
"tgt": "Can height growth medicines cause high blood pressure?",
"src": "Patient: hi..i am a boy of 17 years old and i am having bloodpressure high for the first time...i am very worried about it. my weight is 78kg and height 5,8 inch. i am taking height growth medicines from 2 months and i want to know that is the bloodpressure high beacause of these medicine effects?? Doctor: no. Indeed the most common hair growth medicine--minoxidil---is a blood pressure medicine. cannot say in your particular case, but generally,"
},
{
"id": 41577,
"tgt": "What causes problem in conceiving with all normal reports?",
"src": "Patient: helo doctor.......I Married 2.6months back now iam 28 yrs of age my huband 31yrs ....v hav lot of stress in our routine work....10 mnths back v undergone for checkup for fertility found no abnormality.....but one haemoragic cyst was there in my right ovary....as per the gynaecologist advice i underwent lap cystectomy since the doctor said me it may be endometriosis....after lap it was a simple cyst but adhesions were there .....the cause for the adhesions was unknown......even endometrium biopsy was done it was normal.....doctor suggested to do yoga....can you suggest some medicines which helps to conceive.... Doctor: HelloYou mentioned that there was a hemorrhagic cyst 10 month back , beside adhesion and also endometriosis .You did not mention detail about the menstrual cycle , you are having . This is most important step no 1 in pregnancy.2nd important point is study of ovulation and its date of release . Semen analysis report of your husband .You are having cyst so this is very common in young child bearing female .In my opinion need not to worry . Consult another gynecologist and get his opinion as in my opinion this would be the best thing for you.Good luck."
},
{
"id": 44102,
"tgt": "How should I overcome hormone imbalance so that eggs grow? Use of Meprate?",
"src": "Patient: hi iam 27yrs me and husband r planing for baby , last month iam been gone with treatemnt ( follicle tube ) last month my report was good and normal but this month my eggs have nt grown much on 10days it was 12m only and it shd be 16m i ask to doctor he said this is becoz of harmone imbalance sometime it happen, can plz tell me how shd i over come from this so that i can conceive as early as possible ,and wht is the use of meprate tablets Doctor: On 10th day, 12mm follicle is OK. Sometimes follicle can develop late. Occasional anovulatory cycles is normal phenomenon. Fastest way of conception Superovulation + IUI OR IVF. But whether its necessary for you will be decided by your doctor."
},
{
"id": 168137,
"tgt": "How to treat a kid with ADD(Attention deficit disorder)?",
"src": "Patient: i have a stepson that is 18 yrs old and i keep finding urine in bottles in his room. He was diagnosed with add when he was a child im wandering is this gonna correct itself . I have confronted him on many occasions and doesnt even realize hes doing it anymore. Is this a psychological disorder or is it a part of his add? He keeps his urine as a trophy and hides it and never disposes of it until i find it and confront him. Doctor: Hi...this is not a clinical feature suggestive of ADD. This is an abnormal reaction and he needs psychological evaluation for it. I suggest you take him to a clinical psychologist regarding this.Regards - Dr. Sumanth"
},
{
"id": 84466,
"tgt": "What are the side effects of DUZELA ?",
"src": "Patient: Hello, I am 28 years old & have a daughter aged 2 yrs 10 months old. I had c-section. I had back problem & insomnia problem since then. Now I m on medication of duzela 40 and mirnite 7.5 since 8 months and trying to conceive again. Is it safe for the child? Doctor: Hi,Duzela is generally well tolerated by most of the patients however it may cause nausea, drowsiness, dry mouth, and heart burn. Regarding the safety of Duzela and Mirnite during pregnancy is concerned there are there are no clinical studies done in human beings to ascertain their safety during pregnancy however, potential benefits may warrant use of the drug in pregnant women despite potential risks.Hope I have answered your query. Let me know if I can assist you further. Regards, Dr. Mohammed Taher Ali, General & Family Physician"
},
{
"id": 56900,
"tgt": "Should i be concerned about the anechoic cysts in my liver and enlarged bile duct?",
"src": "Patient: hello..i was just found to have one or 2 anechoic cysts in/on my liver. there is a septal wall. my bile duct is enlarged, also.as an aside, my mother passed away 1 year ago from mets liver cancer that started in her bile duct. what type of cyst is this and should i be concerned? Doctor: Hello,How are you? my name is Dr Suresh Raghavaiah. I am a Liver cancer specialist and I hope to help you today.I am sorry to hear about your mother and I can understand your apprehension.Anechoic cysts in the liver could well be a simple cyst of the liver which requires no further treatment. But the presence of a septal wall makes me think that the cyst could be infectious in nature. I hope you are not suffering from any symptoms like fever or pain. The dilated bile duct is abnormal, but I will need more history to advise you better. Please let me know what imaging was done and why it was done?Please do get back to me with this and we can discuss it further. But just to re-assure you, I don't think there is anything pathological in the findings.Hope this helped youDr Suresh Raghavaiah"
},
{
"id": 178407,
"tgt": "What causes tingling in arms of a child on Prednisone?",
"src": "Patient: My 11 year old son just started prednisone for an asthma flare today. He is having tingling in his arms and legs. This happened the last couple of times he s been on a short course of prednisone. I thought it was a normal side effect, but just read it is one that warrants an immediate call to the doctor. Is it normal? Doctor: hi..prednisalone causes peripheral neuritis on chronic use not with one day use. in your child the prednisalone started today only, so this may not be the cause for tingling sensation. use one week of B complex syrup, if does not subside consult your pediatrician for further work up and management.if you are happy with my answer kindly rate me.regards-Dr.Surendra.H.s"
},
{
"id": 126543,
"tgt": "What causes swelling in the calf and feet?",
"src": "Patient: My niece (30) was hospitalize yesterday for a pain an inflammation on the right ingle area. She got fever and HA as well. Dr. Did a Eco and said 3 limph nodes are inflamate and took a sample. Today she has both calfskin inflamate and so her feet. What is going on? Doctor: Hi, There could be many reasons for this. To define the exact reason we need to evaluate you clinically. The amount of information you have mentioned in query point towards some infective pathology. I need to see your blood reports and other reports to reach some conclusion. Hope I have answered your query. Let me know if I can assist you further. Regards, Dr. Anuj Gupta, Spine Surgeon"
},
{
"id": 223619,
"tgt": "Do birth control pills protect one from a pregnancy?",
"src": "Patient: I took ampicilin for five days 500mgs every eight hours. My last pill was on Friday. Had unprotected sex on Monday took plan b within ten hours. I ve been on bc for almost five months have not missed one pill at all I take it everyday at the same time. How protected am I from pregnancy? I m stressing big time! Please help Doctor: Hello dearI understand your concernAmpicillin generally decrease effectiveness of the birth control pill and cause pregnancy.But you have used postinor 2 with in 10 hour and also on regular use of the birth control pill, so there is no chance of the pregnancy.No chance of the pregnancy as you used double protection.Avoid stress, take healthy diet, drink plenty of water and do regular exercise.Hope this may help youContact HCM for further heath queryBest regardsDr. Sagar"
},
{
"id": 21671,
"tgt": "Could pain from a shoulder surgery be triggering the Atrial Fibrillation ?",
"src": "Patient: Have been taking 150mg propafenone hydrocloride twice a day for 18 months to control Atrial Fibrillation with good results however last three days AF has returned and the medication has not helped...I have had shoulder surgery 17days ago still in considerable pain could this be triggering it off despite my taking my pills...3 days ago took extra strength panadine (codeine and paracetamol) 8 tabs/24hrs..could this have interfered with my AF medication...my doctor is away this weekadvise will be greatly appreciated. Thank You, Buddy Lorenand Doctor: Yes , The pain activates the body sympathetic system 2hich pushes more adrenaline , cortisol like hormones in the circulation . This would act as a trigger for precipitating atrial fibrillation . As long codeine can cause change in the absorption of propafenone and make it less effective. Codeine is available in different strengths and so I am not able to deduce the total quantity you too in a day , 3p mg every 4 hours is maximum you may take and never more than 360 mg of codeine in a day ( However this is maximum dose and you may have side effects far early withen the prescribed dose ) . Do consult your orthopedic doctor as such severe pain so many days after the operation is a worry and any reversible cause should be sorted out rather than just popping pills. Regards"
},
{
"id": 14436,
"tgt": "What causes skin rashes?",
"src": "Patient: Hi. I have a rash that started on the tops of my legs but has spread to the tops of my arms, belly and neck. It looks a bit like a heat rash, very raised and itchy, spreading fast. The only thing I can think of changing is that I went out for an italian on Friday night, could I be allergic to something from there? Doctor: Hi,You have itchy rash on whole body. The lesions may be elevated and discrete. I think probably you have lichen urticatus.... also known as papular urticaria. It is an hypersensitive skin reaction to insect bite....You consult dermatologist for right diagnosis and perfect treatment. Antihistaminics like levocetirizine and steroids in tappering dose might improve the disease. Apply mild steroid cream on the lesions. You may be alright after few weeks.I hope you got my answer.Thanks.Dr.Ilyas Patel MD"
},
{
"id": 73065,
"tgt": "Suggest treatment for respiratory issues",
"src": "Patient: hello , i have a respiratory problem,during the day i can not breath deeply , i must go infront of an air conditionor to breath , may it com.i think when i breath,i doesnt fill my lungs .i have had it about 2 month .after that i become nervous with faster heart biting Doctor: Thanks for your question on Healthcare Magic.I can understand your concern. Breathing difficulty, nervousness, palpitations (feeling faster heart beats) etc are more suggestive of stress and anxiety related diseases.So better to consult psychiatrist and get done counselling sessions. Try to identify stressor in your life and start working on its solution. You will mostly need anxiolytic drugs too. Don't worry, you will be alright with all these. Avoid stress and tension, be relax and calm. Hope I have solved your query. I will be happy to help you further. Wish you good health. Thanks."
},
{
"id": 181006,
"tgt": "How can a broken tooth be fixed?",
"src": "Patient: I broken my UL6 tooth which was already filled plus had RCT done I went to the doctor and he left me without any temp fill While waiting for app the tooth broken completely and now left roots plus half of my lingual distal tooth wall How can I restore this tooth ? Was that correct to leave the tooth for more trauma and exposure ? Doctor: Hi..Thanks for the query..Well, if after root canal treatment the tooth is not filled nor the capping was done then it does not last long.The reason is that the root canal treated tooth is fragile and will break under forces of chewing..So for the present condition of your tooth you can discuss with an Endodontist if they can do Post and core treatment and save the remaining tooth..But if the tooth cannot be saved then get it removed and replace it with bridge or Implant..Hope this helps .Regards ."
},
{
"id": 46066,
"tgt": "What causes dull pain in kidney area?",
"src": "Patient: I've been having some dull pain in my left \"kidney area\" on my back and side for about a week, but today the pain is really sharp and seems to be covering a slightly larger area. The pain gets worse when I urinate, but I'm not going any more frequently than I usually do. No fever, nausea, anything like that. Should I be concerned? Doctor: Pain over kidney area with radiation to groins is a symptom related to renal stones disease...it may be rarely infection (because u are saying no fever and no burning urination).There is no need to panic but u need to have an Ultrsound abdomen done and complete urine examination to look for any stones and infection. Serum creatinine levels also should be done to find out if the problem is causing any functional derangement. Don't let days pass on as u have to react to the condition which is treatable that prevents further damage ."
},
{
"id": 221654,
"tgt": "Is pregnancy risked by a lead painted home?",
"src": "Patient: I am 6 weeks pregnant our home is 200 years old and does have lead paint i keep up with it when i notice it chipping i paint it we did remove alot of the boards but the outside of the home is a chipping disaster we are planning on siding it but did nt get to it yet we bought this home to fix up now im afraid for my unborn baby i dont sand it or anything like that what are the odds my baby will be harmed? Doctor: Hi, I understand your concern. Lead exposure of a would be mother has various bad effects on the developing baby.like- Weakened bones, effect on development of brain ( causing cognative problems/ learningf disabilities ), babied tend to be underweight, prematurely born, developmental defects in later life. YOu house has an old lead paint, has started chipping.. & needs patching off on. I would insist that you should not be involved in painting activities. Even scraping for removal of old paint expose you to lead.. so better stay away from such work.. I feel as a newborn's arrival expected in the house soon.. the complete painting work should be undertaken once for all with water color/ acrylic or tempera paints . If you have been exposed for few times till now.. you should continue with balanced nutritive diet containing sufficient calcium, proteins, omega three fatty acid, vitamin E along with other vitamins , minerals / have regular health check ups .. Thanks."
},
{
"id": 44655,
"tgt": "Will I ovulate on the 22nd day as my cycle is 35+ days or is it anovulatory cycle ?",
"src": "Patient: Hi .. I have delayed periods ... for the last three months it has been in 35+ days category .. for the last month , my period was in a cycle of 42 days .. I did follcile study this month .. as per the report there was no dominant follcile upto 18 day .. today was my 20 th day ... follicle size in right ovary is 15*13 mm.... Is there any chance that i will still ovalute considering my delayed periods .. the doctor in the scan centre said we can still on 22 day .... i really dont know wethere it will be of any help .. is it an anovalutory cycle ??? I am rite now not on any drugs ... Doctor: hi welcome to HCM read your prob, there are many cause of delayed cycle n small follicals. most common cause are hypothyroidism, diabetes, hormonal imbalance, PCOD, so you should get some blood test for ruleing out the cause like thyroid profile, blood sugur, CBC,ESR. discuss all this thing to your treating doc n start proper treatment. Dr aseem saraswat. 9982583020 aseemadhuri@gmail.com"
},
{
"id": 204400,
"tgt": "What causes a sudden blackout followed by difficulty in standing or walking?",
"src": "Patient: I had an event that was stroke like. I passed out for maybe 1 to 5 minutes at most ( Dont remember passing out or what I was doing exactly ) When i came to i was off and didnt have any cordination.I feel as though I was thinking pretty good but i was not able to walk or stand for longer than a second or 2. This lasted for approx. 2 to 2.5 hours and then just went away like nothing happened. I seen a Neurologist and sent me for (MRI on brain ) ( sleep deprivation test ) think it was called and echo ,sleep deprived ? ( I had a dopler test done for arteries in my neck , I had a heart moniter test done where I had to wear it for 24 hours. Had echo , ekg ,blood tests and all my tests come back negative for anything wrong or that would cause that? I have COPD in maybe the moderate to a little worse than moderate stage. Acid reflux, High blood pressure. I take adderall for ADHD and lamotrigine. I was not taking Adderall for a few days when this happened. I also have numbness in my left leg ( Thigh area and along the side of my thigh ) and weakness that seems to be getting worse as time goes on. If I am on my feet for extended time , I get a painful burning sensation in the thigh as well. I was given Gabapentin for that 400MG.I was not taking that when event happened either. I was taking- ( lisinopril 20-12.5 ) ( Lamotrigine 100 mg. 2 times a day ) ( Omeprazole DR20mg once daily ) (Symbicort 80/4.5 , 2 puffs daily ) ( Spiriva handihaler 18MCG CP-handihaler. once daily ) Just a note: In the past 4 years I have had small episodes of getting light headed a bit and on maybe around 6 to 8 of those times , I have passed out and fell down, but came right back around like nothing happened. Usually happened when I was Coughing or a day or 2 after drinking alot. I was a heavy , Problem drinker for many years and dont drink anymore. This passing out thing use to happen maybe once a month ,but in past 2 years it only happened once and then of course , this major one I just had. It might be worth noting, I have not drank in 2 years ,so it seems that the quick passing out episodes for the most part,almost dont happen since I gave up the booze. My Neurologist says not problem with Brain , No prob. With Seizures that he can tell, Neck or the arteries they check in doppler are all ok ??? I am at a loss and am always worried about another falling out and worried it will be the last one. Oh, also, I am 53 years. Well, I really hope you can help me out with this mystery and I can take action to get the problem fixed. Thanks and hope to hear back soon! Randy. Doctor: Hello and Welcome to \u2018Ask A Doctor\u2019 service. I have reviewed your query and here is my advice. From what I can gather from your investigations and neurological examination is that normally you are most probably suffering from a psychiatric illness from what is called a dissociate syndrome. For confirming, a detailed history is required. If I am your doctor I would take a detailed history and upon confirming it would start you on S.S.R.I's and sedatives which are prescription only medicines. Hope I have answered your query. Let me know if I can assist you further."
},
{
"id": 111931,
"tgt": "Is clonidine the reason for the back pain?",
"src": "Patient: I started taking clonidine 2x/ day a week ago. Have hyperadrenergic POTS. Was having periods of polyuria and nocturia prior to med. that has improved since beginning med. but am having pain in lower back( kidney area). Could it be issue with clonidine. I also take keppra xr, trazodone, effexor xr. Doctor: Hi,I have read your query.Clonidine does not cause back ache. Back pain can be due to many reasons. You should get x-ray lumbosacral spine done in AP and Lateral done send us report or discuss it with your doctor. Only then further management can be advised. - Avoid heavy weight lifting pushing and pulling heavy objects, because it will further increase back pain. - You can take medicine like painkillers + muscle relaxants by discussing it with your doctor. You may need stronger pain-killers which are available on prescription only. You can try ibuprofen or naproxen till you see your doctor. - Apply hot packs for 30 min in morning and evening . I hope it helps. You are welcome to ask further health related queries. Thanks. Dr.Singh."
},
{
"id": 201462,
"tgt": "What causes red rash between the thighs and testicle?",
"src": "Patient: In the area on and between my inner thighs and testicles there is a red area which is only sore when I touch it but is also rather sensitive. I am unsure if it some type of rash or not. I wash my body thoroughly and have not come into contact with any chemicals. I had protected sex with my girlfriend a few nights ago, but we are the only partners either of us have ever had. Could it just be and irritated area or could it be some kind of infection? Doctor: Hello,This seems to be just an irritated area. It can be due to excessive sweat or moisture in these areas. I would advice you to apply clotrimazole powder twice daily in the affected area. Try to keep the area dry. Dont wear tight underpants. Dont rub or itch it harshly.Thanks."
},
{
"id": 105655,
"tgt": "Asthma, red spot on lung, breathing problems, cough, chest pain and lung pain, want to take scan. Malignancy?",
"src": "Patient: My boyfriend is a veteran and he has had asthma his entire life. Two years ago they found a red spot on his lung and resently that spot went away. However they recently found another spot in his lung and though he had no new symptoms two years ago, he now is having severe breathing problems, trouble at night especially, and is always coughing. He also gets this Sharp pain in his chest , near his lungs. All the doctor would tell us is that he could not rule out malignancy at this time and that they want to take another scan in a few months. However in the mean time my boyfriend can t breath and I m affraid he is going to stop breathing at night. WHAT COULD THIS RED SPOT BE AND SHOULD WE BE MORE WORRIED ABOUT FINDING AN ANSWER NOW? Doctor: Hello To control asthma ur boyfriend must use combined inhaled corticosteroid and long acting beta 2 agonist 2 puffs twice daily. also use a short acting beta 2 agonist on sos basis. Also take a 7 day course of oral prednisolone if there is an exacerbation of asthma. Try to avoid cold, dust etc. I don't understand what ur doctor try to mean by red spot in lung. Consult a Pulmonologist immediately."
},
{
"id": 154033,
"tgt": "What sort of cancer can a blister on the vagina cause?",
"src": "Patient: Hi, may I answer your health queries right now ? Please type your query here...my sister was told she has cancer.She found a very small blister on the outside of her vagina.She had a biopsy done, but was not told what kind of cancer it could be. Do you know what kind of reserch I can do,or what she may have. Thank You Doctor: Hi, dearI have gone through your question. I can understand your concern.She may have squamous cell carcinoma. It is most common cancer of vagina. But it can be other type of cancer also. Please send me biopsy report of your sister so that I can help you further. Hope I have answered your question, if you have any doubts then contact me at bit.ly/Drsanghvihardik, I will be happy to answer you.Thanks for using health care magic.Wish you a very good health."
},
{
"id": 142790,
"tgt": "What causes dizziness and vomiting?",
"src": "Patient: I am experiencing bad dizzy spells for a long time. The other day I walked on the tread mill for 30 minutes and when I was done I was very dizzy and vomiting. I had to close my eyes and lay down. I have them when I tilt my head back side to side, if I roll over in bed or just change my position. My doctor told me about 1 year ago that I had bulging ear drums and to take Zyrtec every day but it doesnt seem to helk much.What could be my problem. Doctor: u could be having benign paroxysmal positional vertigo..it's a type of balance disorder.. curable disease.."
},
{
"id": 210295,
"tgt": "My mother came down for breakfast this morning disoriented",
"src": "Patient: My mother came down for breakfast this morning disoriented. She asked me where the children are when she had just spoken to one of them; she just poured coffee in her breakfast plate and even when looking at it didn't know what she had done. She can't find the words to phrase a question or answer a question. She said that two days ago she had a severe headache and felt nauseous. She said she never gets headaches, but got over it. She can't ask questions. Doctor: HiThanks for using healthcare magicYou did not mention the age of your mother. There is multiple reason behind disorientation and treatment in that case depends upon the cause. I think, you should consult a physician or neurologist and get your mother properly investigated. That would help to plan further management.Thanks"
},
{
"id": 28027,
"tgt": "Is EECP treatment helpful for heart enlargement, breathlessness and weakness?",
"src": "Patient: MY BROTHER'S AGE IS 34 YEARS. HE HAS VERY MUCH LOW BLOOD PUMPING. HIS LVEF IS 15% JUST NOW. HIS HEART IS ENLARGING. HE HAS NO HABIT OF SMOKING OR DRINKING. HE HAS PROBLEM OF SHORT BREATHING & FEELING VERY MUCH WEAKNESS. CAN YOU PLZ GIVE ME ADVISE WHAT TO DO & TO GO IN AYURVEDIK OR ALOPATHIC MEDICINES???? IS EECP TREATMENT IS HELPFUL OR NOT??? PLZ HELP ME.... Doctor: he must undergo coronary angiography ,EECP is helpful only if there are significant coronary blocks and no scope for bypass or angioplasty.dont try for ayurvedic medicinescontinue with allopathic medicines ..."
},
{
"id": 47037,
"tgt": "Can kidney be donated whom the donor is a smoker?",
"src": "Patient: i want to donate my kidney to my friend who last night was sent to the hospital for kidney failure they said he needs a transplant as soon as possible, i have the same blood type but it hasnt been a month since i have smoked. i smoked three nights ago. im 19 years old 5 foot 7 and im healthy. so is it ok to be a smoking kidney donor Doctor: Hi & Welcome to HCM.I appreciate your generosity and gesture towards your friend.You will not be considered as an eligible donor unless you stop smoking for at least 8 weeks prior to donating the kidney.This is because there are many known health risks from smoking1. It can cause potentially life-threatening respiratory complications during or immediately after anesthesia. 2. It may also increase the risk of developing blood clots in the leg veins which can break loose, travel to the lungs and potentially cause death. 3. It causes increased mucus production and a decreased ability to clear the lungs which can lead to pneumonia. 4. It causes decreased wound healing. To be eligible you must quit smoking first and after 8 weeks you can donate the kidney to your friend after getting the pre-operative medical fitness. Since smoking is associate with a number of diseases of the heart and the lungs and various cancers so, it's better to quit smoking ASAP.Hope I could answer your query."
},
{
"id": 17131,
"tgt": "Should I be worried about shortness of breath, chest pain, high blood pressure and arthritis?",
"src": "Patient: I am 41 yrs old female. Diagnosed with mild lupus 2 yr ago. Also have high blood pressure, bursitis and arthritis bursitis. I had an echocardiogram and wore a heart monitor two weeks ago due to these episodes of shortness of breath and occasional chest pain left arm pain and palpitations. Also my pcp said he heard something abnormal with stethoscope. I called the office to see if the results are in they said they had to be sent to a cardiologist to be read. Should I be worried? Doctor: Hello, No, there is nothing to be worried about, although, what had been heard by stethoscope could be a mild abnormality of the valves function of the heart, which is treated medically. Hope I have answered your query. Let me know if I can assist you further. Take care Regards, Dr Salah Saad Shoman, Internal Medicine Specialist"
},
{
"id": 109163,
"tgt": "What causes the present lower back problems with pain?",
"src": "Patient: i got shot in the back when i was 12 the scar reopened when i was in weight lifting during football practice when i was 16 and the doctors patched it up again im 18 now and im starting to get lower back problems underneath the scar could it be from my accident or could it be from weight-bearings cause i still weight lift for rugby in college and i go swimming often Doctor: Dear patient it seems your back pain is not due to accident but might be due to adherent scar and weight lifting. Since weight lifting itself leads to back strain and bursitis undernith scar also lead to back pain you need to investigated for both. I would like to advice ultrasound of the scar area and digital radiograph of spine to come to conclusion."
},
{
"id": 206296,
"tgt": "Suggest remedy for abnormal behavior and vision problem",
"src": "Patient: I really dont know whats wrong with me.I cant understant the things i read,i cant concentrate,i've become stupid. I feel like i'm not living and busy watching other peolple live their lives.when i look at things is like i'm seeimng through a thing glass.this started in 2003 and continued to get worse with the years.I became myopic in 2003 and thought that maybe my eye sight was affecting my brain some how and that if i did refractive surgery then my brain would come back.I did the surgery in 2008 ,ICL(phakic iol).But my brain is still the same and worst of all i feel like when i look at sentence its like i have tunnel vision and the vision is converging making it difficult to rea properly and inderstand what i'm reading.I came across i forum about something called brain fog and i have the exact symptoms as the people o that forum.Please help me because my world fallen apart too deep. Doctor: DearWe understand your concernsI went through your details. I suggest you not to worry much. In psychology, we broadly call it obsession. You had that disturbing sensation once. You expected it to happen again and tried. Then you started experiencing it as you were expecting it. Then your body brought the familiar symptom because you are expecting it. Then you started worrying that the symptom is coming again and again and expect it. Body brings it again as you are expecting it. You become worried and so on. That is obsession in simple terms. Ignore it and the problem vanishes. Psychotherapy techniques should suit your requirement. If you require more of my help in this aspect, Please post a direct question to me in this URL. http://goo.gl/aYW2pR. Make sure that you include every minute details possible. I shall prescribe the needed psychotherapy techniques.Hope this answers your query. Available for further clarifications.Good luck."
},
{
"id": 4174,
"tgt": "Will I be able to conceive naturally after the TB treatment gets over?",
"src": "Patient: Hi, After two miscarriages, i have been diagnosed with tuberculosis, but since it is latent, without any symptoms, can't say which part of the body it exists. Now, my gyanae has put me on AKT for 6 months (2+4 months). my question is will i be able to conceive naturally after the treatment gets over? Doctor: Hi, if the infection is other than the reproductive organs infection u can have chance of conceiving. If it is for pelvic infection then their may have adhesion to uterine tubes it delays the conceiving process. if u clear this adhesion u have a high chance of conceiving."
},
{
"id": 194636,
"tgt": "What causes vomiting after testicle surgery on moving an object?",
"src": "Patient: my husband had 2 groin sugeries in his testicles. today at work he strained himself trying to move something and he instantly threw up. He said he took 2 tylenol and felt a little better but now he said its tight. I had him call the doctor that did his surgery but he said he is away till the 30th. Should he go and get this checked now or do you think he can wait. Doctor: Hello, It could be due to conditions like epididymitis. Consult a general surgeon and evaluated. Hope I have answered your query. Let me know if I can assist you further. Regards, Dr. Shinas Hussain, General & Family Physician"
},
{
"id": 92367,
"tgt": "Why would one experience sudden lower stomach pain?",
"src": "Patient: I have a pain in my lower stomach left side area it is not always there but it hurts when I lift something or have to go to the bathroom or get up from the laying down position. What might it be and would you suggest me to go to the VA and see my regular doctor? Doctor: Hiin my clinic I would have told the patient it is torn abdominal muscle Give rest to the abdomen for few days and do not lift heavy objects If you want to visit your regular doctor it will not be bad ideaDr Lal"
},
{
"id": 157877,
"tgt": "Suffering from cervical cancer. Swollen cervical lymph node and abdominal pain. Diagnosed as cholecystitis. What should I do?",
"src": "Patient: I am 33 years old female with cervical cancer which has spread to the left cervical lymph node. I have been on chemotherapy but now my right cervical lymph node also swollen and now I have abdominal pain. I have gone to the internist, he performed USG and he said I have cholecystitis . What should I do ? please help me.... Doctor: Hello Ola3394,Since you have undergone chemotherapy, take it for time being that you are cured of cancer. Now onwards, you should not have worries over your disease of cancer. If you go one worrying over your illness, if you are going to ponder over it again and again and if you are too much afraid of it, you are not going to get free from illnesses. I hope you understood why you got cholecystitis now. You should stop being afraid about your illnesses.Cholecystitis is not a major illness. It is only an inflammatory disease and it can go off by itself. If not disappearing, that means your immunity is not improved and you need a treatment that will improve your immunity. I can confidently recommend Homoeopathic treatment for you for improving your immunity. It will take into consideration all your mental and physical make ups and will cure you with genetic constitutional remedy individualistically chosen for you. Either you may approach an expert Homoeopath or get back to me with more details at the earliest. I can take your case in detail and prescribe your remedy that will keep you healthy for many more years to come. I can even assist you to prepare your case by sending a case receiving format on request in which you can enter your case and return.With best wishes, Dr C. J. Varghese, Homoeopath, drcjvarghese@gmail.com,"
},
{
"id": 55996,
"tgt": "How to treat fatty liver?",
"src": "Patient: My niece is 32 and has been told she has fatty liver steatohepatitis. But she doesn t drink and use to weight only 105. Since her last kid she has develop a big stomach. She has gain 50lbs fast and is retaining fluid. Doctor said he didn t know why. I think its because of the hepatitis. Also she hasn t had a period for the last six months. Her OB said her eggs aren t dopping and her body isn t preducing enough insulin. Could the be related. Also is NASH another name for steatohepatitis? Doctor: HelloFatty liver may be due to many causes like obesity,hepatitis,alcoholism,metabolic disorder etc.She may need few more investigations like routine hemogram,random blood sugar,lipid profile,liver function test,urine RE/ME.Fatty liver often get reverse with treatment of underlying cause.She may need few medicines after complete investigations.She should avoid saturated fat and go for brisk walk regularly.Take lot of fruits and vegetables in diet.She should also try to reduce her weight.Take CareDr.Indu Bhushan"
},
{
"id": 41123,
"tgt": "Suggest treatment for infertility",
"src": "Patient: hello doctor, i am married and it is 1 1/2 years, we didnt have a baby, i was conceived on 6th month of our marriage and it got aborted(missed abortion) in 3rd months, i undergone D&C. And we both went to all test all shows normal...plz give suggestion Doctor: Hello, just be relaxed and dont panic.Get your hormone profile tested and also semen analysis of your husbandIn case you have any questions in future you can contact me directly on http://bit.ly/drmanishajain"
},
{
"id": 133373,
"tgt": "What does hematoma signify?",
"src": "Patient: what is a hematoma and how long do they last, I fell 3 weeks ago getting out of the bath tub on the tile and hit my head above my right eye brow and have a huge hard bump, was very black and blue but that is all gone except for the bump, they did take xrays and nothing was broken they said i had a hematoma how long do they last?. Doctor: hi,thank you for providing the brief history of you.As you had a fall due to which there will be internal bleeding leading to the formation of haematoma. By the grace of God everything is well and I m happy with that.This haematoma will take couple of days to go and kindly keep patience as body will excrete the unnecessary particles out soon.As the body healing takes time and excreting the particle waste takes its own time kindly keep patience and do gentle massage near the haematoma. Everything will be fine soon.RegardsJay Indravadan Patel"
},
{
"id": 128737,
"tgt": "What causes neck pain when diagnosed with cervical dystonia?",
"src": "Patient: I had a small camera put down my nose and went down to my vocal cords..... I have cervical dystonia but the neck pain is different the Dr noticed extra skin in my throut my voice dies change often an MRI was booked within 2 weeks of this test should I be worried Doctor: Dear patient cervical dystopia usually leads to throat pain and change in voice rather than neck pain. Neck pain may be due to muscular spasm or cervical spondylitis. mri is advised for which region? I mean for cervical spine or for larynx? I would advise to get it done for both regions to confirm diagnosis. Treatment would be advisable after confirm diagnosis. thanks."
},
{
"id": 78861,
"tgt": "What to do for sharp pain in middle of chest and soreness in back?",
"src": "Patient: i have been having pain in the middle of my chest off and on for a few months now. over the past week or so i am having worse chest pains that last most of the day off and on. sometimes it is sharp pain today it is dull and achy and my back is sore too Doctor: Hello. Thank you for your question and welcome to HCM. I understand your concern. Well, I am going to try and explain the differences between various origins of the pain which can arise from the ribcage. There is angina. It is described more like mid-chest heaviness or squeezing sensation, that is triggered by exercise and relieved by rest, which radiates to the both sides of the chest, to the jaw and left arm and shoulder. It is indicative of a blockage within one or more coronary arteries and reflects the lowering of blood supply to the heart muscle. An EKG obtained in the moment of chest pain, if it fills this description, would be helpful. Then there is pleuritic chest pain. This kind of pain is sharp, stabbing pain that can happen in any part of the chest area. No radiation like in anginal pain. It arises from acute or chronic inflammation of the linings that are within the ribcage: pleura (lining of the lungs) or pericardium (lining of the heart) or both. In this regard, I would recommend a chest X-ray for pleural inflammation or effusion. In case of pericardial pain and anginal pain, echo cardiogram is helpful, by evaluating the size, motility and function of heart muscle and valves, and also by assessing the pericardium and whether there is fluid build-up within it, or not. Other causes of chest pain are related to the locomotion system of the rib cage: muscle soreness, bone and cartilage-related pain. In the end, stress can also cause chest pain. I hope I was helpful with my answer. Best regards, Dr. Meriton"
},
{
"id": 221541,
"tgt": "How can stomach acidity and indigestion be treated in a pregnancy?",
"src": "Patient: hello i am suffering from accidic stomach and i am in the 33.week pregnancy , i have gaviscon, but the heartburn has gone , but i feel accid i am affraid in the night i am not going to sleep because of indigestion , but in the moment my stomach is not sore . Cold take now a Rennie tablet? Doctor: HI, I understand your concern. acidity & heart burn with indigestion is common in pregnancy due to loosening of esophageal sphincter/ wrong concepts about eating & effect of pregnancy hormones on digestive system. To avoid this some life style changes are required. They are- - Eat frequent & small servings of food ( every 2-3 hours ). have salad/ fruit with each serving. - Avoid spicy, oily food ( to avoid acidity ). Take non spicy ( boiled ) food. - Avoid tea/coffee/ fast. Take freeze cold milk . - Avoid alcohol/ smoking. - have good sleep at least 8 hours in a day./ Avoid psychological stress. - Yoga exercises & light walk for half hour help in digestion/appetite. -Take digestive enzymes with advise of your doctor. -Rennie tablet helps indigestion by displacing the gas . It can be used once in a while. For daily problem its better to change few habits & adopt new ones for permanent cure. thanks."
},
{
"id": 149907,
"tgt": "Have vertigo, head pain. Taken antiinflammatory injection. What to do next?",
"src": "Patient: I started with Virtigo last week Sunday which was the worst i haveever had...I couldn't get out of bed to take the meds for it til 1:00p.m. ..it took til 10:30 that nite for it to ease up so that was a day missed of work.. Since then i have had severe head pain... i went Sat. nite to Urgent Care on 107th/Bluemound which is St Joes.. they gave me two shots in my head and anantiflamitory in my hip.. Followed up with my Dr.on Monday which he gave me Butalbital/acetaminopjec/caff tablets...all they do is make me go to the bathroom. I need someone to tell me what to do next.This is causingme great pain Doctor: Hello, Thanks for the query to H.C.M. Forum. Mere , writing vertigo , not possible to make any judgement. Age, blood pressure , sugar level, tension, Many more things are required to diagnose a patient's disease. Consult a physician and get blood for sugar, lipid profile, C B C , and deal according to the advice of doctor. Hope you will upload complete reports. Good luck. Dr. HET"
},
{
"id": 155592,
"tgt": "What could cause elevated PSA level?",
"src": "Patient: My PSA levels have gone up and down in the past year. Whenever they go up, th Urologist always wants to do a Biopsy. There are risks associated with this procedure, so I think getting bloodwork done every couple of months would be better. There is no history of prostate cancer in my family, so even though I could be a first, I don t know why the DRS wouldn t want to just keep monitoring my psa levels. I am just worried that they all just want to do certain tests that may be unnessary Doctor: Thanks for your question on HCM. In my opinion you should get done prostate biopsy. PSA is only screening test. It is not confirmatory for prostate cancer. So high PSA level does not indicate prostate cancer, and low PSA level does not exclude possibility of it. But if you have high PSA level than chances of prostate cancer are high. So we need to rule out this. And for this biopsy is must.And no need to worry much for biopsy related complications. As it is very safe procedure. And by this we can detect prostate cancer at earliest. So that timely treatment and surgery can be done. And life expectancy can be prolonged. So better to get done prostate biopsy."
},
{
"id": 213545,
"tgt": "Can stress be the reason to get the symptoms of periods?",
"src": "Patient: Hi Doctor, i havent had my period in 4days now, i m getting symptons for periods. could it be cause i am thinking too much or stressed out?? i aint pregnant cause i havent been sick or feeling really ill. Doctor: If you always have regular periods then it is very unlikely to be overdue for 4days. Stress might be a cause but let me tell you, feeling sick or ill is not necessary for pregnancy and you should keep this possibility in mind if you are not on any reliable birth control. Take care"
},
{
"id": 28718,
"tgt": "How can trichomoniasis be diagnosed?",
"src": "Patient: I got treated for trich and my partner tested negative...we are in a monogamous relationship abd do not use protection ...he tested negative and didn't finish his meds...I am starting to experience discharge again and am wondering if his test could have been a false negative and now I have it back again. He has no symptoms and didn't have symptoms when I did and was diagnosed Doctor: Hello and Welcome to \u2018Ask A Doctor\u2019 service. I have reviewed your query and here is my advice. Some men don`t show symptoms for Trichomoniasis so I suggest you to take test for both of you and to start the treatment for both even if it shows negative for him ask him to complete the treatment. I suggest you to take Metronidazole for a week. Hope I have answered your query. Let me know if I can assist you further."
},
{
"id": 165463,
"tgt": "What causes gray skin color, sweating and headache in child with VSD?",
"src": "Patient: my 7 year old daughter has a large VSD and see s here doctor each year only over the last few months she seems more tired, ill looking a funny gray colour ok in a morning but by early afternoon goes completely sweaty says she has headache and feels funny. the local GP said it was a virus but how long can it last? i m now worried its her heart and no one will listen what should i do Doctor: Hello,As your daughter is having large VSD and she is getting more tired and having changes in skin color, she should be started on some heart medications like Lasix, Lasilactone, Digitalis, etc. Consult your doctor for these medications.Hope I have answered your query. Let me know if I can assist you further.Regards,Dr. Khan Shoeb Mohammad Sher Mohammad"
},
{
"id": 216640,
"tgt": "What to do for the pain in the pregnancy stenches?",
"src": "Patient: hello sir, during my pregnancy i had seven stenches but it not given properly at that time after two year of my pregnancy i have sudden pain in that area. what should i do some dr. told me that i have to operate again for this....as of they giveme tab-panat-d,emanzen-d,sporidex Doctor: hi,thank-you for providing the brief history of you.As this is a common complain we find in our clinic posr-partum so physical therapy is advised to strengthen the muscles and also the scar management can be done. Therapeutic ultrasound therapy and TENS therapy helps to reduce pain and exercises helps to strengthen the core again.In my clinical practice most cases respond well to physical therapy post partumRegards Jay Indravadan Patel"
},
{
"id": 157695,
"tgt": "Large masses on cervix, lymph nodes were big. Freeze test is negative. Family history of uterine cancer",
"src": "Patient: Hello, i just had a total hysterectomy done. My Dr was surprised to find that my uterus was 4xs the normal size. There wer large masses on my cervix, and in his words \"they looked funky\" he had the pathalogist flash freeze test them, those were negative. He also found that all the lymph nodes were big on both sides, so he also removed them and sent those in for testing. I am 41 yrs old and have a family history of uterine cancer, including sarcoma, which my 33yr old sister past away from just last year in January. I just don't know if i should be concerned Doctor: Hi, Thanks for using HCM.some of Uterine malignancies run in families. As your Hysterectomy is done, Wait for histopathology report, Depending on that further action needs to be taken.Family history of uterine cancer is a risk factor And also get your breast examined for any lumps or change in consistency. Hope this helps you Regards"
},
{
"id": 55692,
"tgt": "What is the twinge like feeling am I having below my rib cage on the right side?",
"src": "Patient: I am experiencing twinge like feelings on my right side below my rib cage. It comes and goes. More noticeable when sitting than standing. I will admit to heavy drinking for years. I'm getting scared that it may be my liver. This started 3 days ago. Doctor: There can be multiple reasons for this type of painIt might be due to gall bladder stones or acidity or renal pain so I would suggest you to go to your physician for check up"
},
{
"id": 115336,
"tgt": "Suggest treatment for low white blood cell count",
"src": "Patient: My aunt(84)has multiple (almost 2 years) and recently was put in the hospital. She has tried all the treatments with success until now. She is having difficulty swallowing when eating. White blood count dropped. Feeding tube is not an option. She is not in pain. We know now is the time for hospice. My question is how do you think she has to live. Doctor: Hello and welcome to HCM,Low white blood cell counts can occur due to infections especially bacterial infection, bone marrow suppression etc.It is important to know the cause of low white blood cell counts to plan proper treatment.If there is no evidence of infection, bone marrow suppression may be the most probable cause of low white blood cell counts.A bone marrow study may be required.Growth factors are used to stimulate the production of white blood cells depending on the cell component which is affected.Consult your treating doctor for further management.Thanks and take careDr Shailja Puri"
},
{
"id": 14722,
"tgt": "Reason for tiny blisters between the fingers and itching and dryness around the wrist",
"src": "Patient: I'm 32 yrs old I'm 5'3 I don't have any health issues. The palm of my hand, in between my fingers, and a place on my wrist started itching some and I have dried skin on my wrist aftEr It had three tiny blisters and I have tiny blisters in between my fingers. I do have diarrhea on occasions. Doctor: Hello,Thank you for posting on HCM.It\u2019s very difficult to approach conclusive diagnosis unless for proper physical examination, anyways from your description it seems you are suffering from scabies.Scabies is a contagious disease caused due to mite and it spreads among people who are in close physical contact and by use of common household articles.Treatment is pretty straight foreword. Firstly, i would suggest treatment of your secondary infection (if any) with oral and topical antibiotics.The key to successful treatment of scabies lies in treating everyone simultaneously in the family and others who are in close physical contact. Your doctor would prescribe oral drugs like ivermectin which will kill the parasite. You might also be advised application of permethrin 5% cream and a corticosteroid with fusidic acid cream. Apply the medicated creams exactly as advised by your doctor. Take antihistaminics as advised to relieve itching and irritation. Avoid sharing of clothing and toiletries and launder all you bedding and clothes in warm water.Hope this will take care of your issue.Wish you best of health.Thank youDr Hardik Pitroda"
},
{
"id": 206811,
"tgt": "Suggest treatment for compulsive disorder",
"src": "Patient: My question has to do with an associate, and may pertain more towards mental illness. Regardless he is a good friend and I would like to help. He demonstrates compulsive disorder, cannot stay still, walks around all the time, when standing sways from side to side. His correspondence sometimes is missing whole thoughts, does not make sense. He has become irratabile and looking for confrontation, in face will disagree with just about anybody. He does not want to see others grow in the jobs which is a big part of his job. Always negative and assignes projects which go nowhere. Can you help me....John V Doctor: DearWe understand your concernsI went through your details. I suggest you not to worry much. The description given here may not be enough to diagnose your friends proper problem. But it seems to be hyperactivity rather than obsessive disorder. ADHD do have obsessive behavior patterns. Correct diagnosis and proper treatment should be done for a cure. Advising and motivating your friend to visit a psychiatrist is the correct thing you can do.If you require more of my help in this aspect, Please post a direct question to me in this URL. http://goo.gl/aYW2pR. Make sure that you include every minute details possible. I shall prescribe the needed psychotherapy techniques.Hope this answers your query. Available for further clarifications.Good luck."
},
{
"id": 23008,
"tgt": "What is the cause and treatment for atrial fibrillation?",
"src": "Patient: I have intermittent bouts of AF. I can tell when its going to start because I get a chest pain very similar to angina. I had a triple by pass 1 year ago and as a result have AF. Is this pain normal - it goes after a short while but the AF continus for sometimes 8 hours or even more. Thank you. Doctor: in atrial fibrillation your heart increase that leads to pain.for atrial fibrillation you need to consume rate controlling drugs which you must be receiving so you can ask your physician to increase the dose so that will be controlled.Also doctor will prescribe you some blood thinners like dabigatran , make sure you are receiving those to decrease risk of stroke."
},
{
"id": 143041,
"tgt": "Suggest treatment for ankylosing spondylitis",
"src": "Patient: hi I have ankylosing spondylitis since I was 11 I was diagnosedim 23 now since then I been trying to differentiate my symptoms and I m having.chest pains I don t know what is causing it if its anything.to.worry about.I try not.blame everything I feel on my arthritis but therea so many ways my disease affects me other than joint pain Doctor: Brief Answer:You should be visited by a rheumatologist. Detailed Answer:Hello,Thank you for using HCM. I understand your concern. As you are experiencing new symptoms, you should see a rheumatologist. He has to do a physical exam to you, in way to detect the cause of your pains. Your doctor might ask you to bend in different directions to test the range of motion in your spine. He or she might try to reproduce your pain by pressing on specific portions of your pelvis or by moving your legs into a particular position. Also, your doctor might ask you to take a deep breath to see if you have difficulty expanding your chest.You should have a X-ray to check for changes in your joints and bones, also.An MRI may be necessary to provide more-detailed images of bones and soft tissues.Meanwhile you should take in consideration that the main treatment of Ankilosis Spondylitis remains the fisical activity. It would be better to begin the exercises after a shower with warm water which will help you with the stiffness. Swimming, volleyball, etc are the main activities which help decreasing the pain level and conserving the normal posture of your body. Is recommended to stay overthrown for 15-20 minutes for several times a day. If the symptoms does not improve with the fisical activity and the antiinflammatory drugs (CO-DICLO-MISO 75MG), then you have to see a reumatologist for more specific treatment.Hope this is helpful.Let me know if you need more clarifications. Otherwise please close and rate the answer.Kind Regards,"
},
{
"id": 207038,
"tgt": "Suggest remedy for depression and lack of sexual desire",
"src": "Patient: Dear sir, Due to fearness of some undigestible rememberance i became depressed and i am under medications advise by a psychiatrist dr. in chennai for the past 3 years. in the time of 3 years i got cured after 1 year but after consumbing alcohal again i becam patient and i am advise by dr. use meidicine. At present no fearness and i feel sad if see some thing outside. I dont' know this is not disapprearing and sexually there is no ejaculation. Doctor: Hello,Alochol is a nervous system depressant and it can cause sexual problems also.I would strongly advice you to stop consuming alcohol. If the depressive feelings persist even after one month of stopping alcohol, then you will have to start medicines again.Thanks."
},
{
"id": 65773,
"tgt": "What causes lump with appendicitis?",
"src": "Patient: hi my 17yr old son has a lump that is very hot on his left side and is pale and feeling weak with constant pain and has also had trouble breathing and has had to use his ventolin inhaler a few times in the past 2 days (hes an asthmatic) and approx 6 wks ago he was taken to emergency by ambulance and diagnosed with appendicitis but discharged himself when the pain reduced do u think it is the same condtion/? Doctor: Hi.Thank you for your query.Read and understood you son's problems.Appendicitis is usually in the right side of the abdomen not on the left. The hot thing on the left side can be a subcutaneous abscess developed secondary to the appendicular lump or abscess. Pale can be due to septicemia.As he is known asthmatic, you have to take more care and may be give him additional medicines and inhalers to get a proper control over the asthma.The appendicular lump is usually treated conservatively for 6 weeks so that it has to be operated when this becomes resolved. Remember that getting operated on the given date is very important."
},
{
"id": 66386,
"tgt": "What is a pea sized moveable lump below skin on leg?",
"src": "Patient: I was putting lotion on my legs and happened to run across a small pea sized moveable lump below the skin on my lower leg. You cannot see it from the surface only can feel it if you rub carefully to find it. Does that sound characteristic of anything in particular? Doctor: Hi, dearI have gone through your question. I can understand your concern. You may have some soft tissue tumor like lipoma or neurofibroma. You should go for biopsy of that lump. It will give you exact diagnosis. Excision is the treatment of choice. Consult your doctor and take treatment accordingly. Hope I have answered your question, if you have doubt then I will be happy to answer. Thanks for using health care magic. Wish you a very good health."
},
{
"id": 223540,
"tgt": "Should I take a contraceptive after unprotected sex post bleeding?",
"src": "Patient: Hi, I have my monthly periods to come on 28 dec 2013 which posponed to 1 jan from 1 jan 2014 i had bleeding till 20-22 days after which my gynaecologist prescribed me Tab Femilon 21 days on 20 jan 2013. I m taking it regularly. My concern is can i have unprotected sex as my bleeding is stop and i m taking tab regularly or i need to take another contraception method. Pls help i m married fr 8 mnths. Doctor: Oral contraceptive pills has a very good protection rate. Only 6% chance of failure. So you may have unprotected intercourse. But there's a chance of sexually transmitted diseases. So for that condom may be used"
},
{
"id": 63189,
"tgt": "What is the pea sized lump on jaw line of a child?",
"src": "Patient: Hi, may I answer your health queries right now ? Please type your query here... I have just noticed a pea sized lump on my daughters (20 month old) jaw line under her skin but it does not feel like its on her bone. Can you tell me what it could be and if I need to take her to the doctors. She recently had a flu virus but has been well for 3 days now, could this be anything to worry about? Doctor: Hi Dear,Welcome to HCM.Understanding your concern. As per your query pea sized lump on jaw line of a child which does not look like bony is mainly because of presence of infection in a particular tooth in that concerned area which leads top continuous bacterial accumulation and leads to formation of swelling called as abscess. I would suggest you to visit dentist once and get your daughter examined completely. Go for IOPA in that area and get it examined. Go for root canal treatment of that tooth. Take proper course of antibiotics and leads to suppression of infection. Maintain proper oral hygiene. Go for scaling and root cleaning in that area.Hope your concern has been resolved.Get Well Soon.Best Wishes,Dr. Harry Maheshwari"
},
{
"id": 194462,
"tgt": "How to treat erectile dysfunction and premature ejaculation?",
"src": "Patient: i have problem in erection and early release , some one asked me to go for following test , i am writing result of test , so please advise as per result, wat should i do, my age is 32 , maleFREE TESTOSTERONE 11.30 pg/ml TESTOSTERONE 592.10 ng/dl TOTAL TRIIODOTHYRONINE (T3) 108 ng/dlTOTAL THYROXINE(T4) 8.00 TSH 2.41so now please advise me as per report wat u say Doctor: Hello, The test values are normal except for free testosterone. It is high. But the testosterone level is the normal range. The result should clinically be interpreted. Talk to your physician. You can contact me for treatment options. Hope I have answered your query. Let me know if I can assist you further. Take care Regards, Dr K. V. Anand, Psychologist"
},
{
"id": 106953,
"tgt": "How can lower backache be treated?",
"src": "Patient: I am having lower back pain for two weeks. The pain is now causing nausea. I do have lupus. I am on anastrozole from Breast cancer in 2014, diagnosed with lupu in 2008 and take plaquenil I only have mild symptoms if any from this. I take synthroid 75mg. I have a low white blood cell count but think it is from anastrozole but not sure.. not sure which direction to go so need advice Doctor: hi sir/madam,Thanks for your question on Healthcare Magic.Let me advice you for your problem.Low back ache may be a pre-monitory (prodromal) symptom in fistula in ano, sciatica, gouty arthritis, hernia, emaciation disorder etc. Pain is confined to lower part of the spine (back bone) especially lumbar region or lumbo-sacral area (rarely sacro-iliac region also). If it is secondary, earlier history of fall or injury may be associated. Rarely, radiating pain may be complained by the patient towards lower limbs. But it is quite common in low back aches if the defect is in the discs between vertebrae (back bones). Often the movements of lumbo-sacral region like flexion and rotation are hampered either partially or completely.Low back pain may be found in mild form in case of anemia, sciatica, rheumatoid arthritis, hemorrhoids, urinary calculi, uterine disorders etc also.Line of treatment as per Ayurveda:-The general principles of treatment of vata dosha are adopted in case of katishoola (low back pain). It includes various measures to suit its varied clinical entities, stages and associated complaints.1. Snehana (oleation) \u2013 by sneha dhara (pouring oil), abhyanga (oil massage), avagaha (tub bath with oil or oleaginous medicaments), kati basti (retaining medicaments on the back) etc.2. Swedana (sudation) \u2013avagaha sweda (sweating treatment with tub bath), pizichil (kayaseka), nadi sweda (sudation through a tubular device \u2013 local sudation), panda sweda (sudation through medicated paste or powder) etc.3. Mridu Samshodhana -mild purgation.4. Basti (medicated enema) like eranda basti, vaitarana basti, pippalyadianuvasana basti etc.Formulations indicated in Low back ache as per Ayurveda:-1. Dashamoola kwatha.2.Maharasnadi kashaya.3. Rasnaerandadi kashaya.4. Sahacharadi kashaya.5. Gandharvahastadi kashaya.6. Trayodashanga guggulu.And for local application these oils will help:-1. Ksheerabala tail.2. Mahanarayana tail.3. Dhanwantaram tail.4. Maha narayan tail.Avoid these for better results:-1.Bitter, astringent and pungent food2.Cold water3.Fear4.Exhaustion5.Standing6.Driving7.Cold food and beverages etc.Yoga is also very helpful in treating your back ache.Now for your backache do these yoga aasana:-1. Ardha Matsyendrasana(Sitting Spinal Twist).2. Dhanurasana (Bow Pose).3. Marjariasana (Cat Stretch).4. Balasana(Child Pose).5. Halasana(Plow Pose).6. SUPTA MATSYENDRASANA(THE TWO KNEE SPINAL TWIST POSE).7.Adhomukha Shwanasana (Downward Facing Dog).8. Urdhva Mukha Svanasana(Upward Facing Dog Pose).9. Paschimottanasana(Seated Forward Fold).Follow these yogas you will get best results for your problems.Hope i was helpful.Have a healthy day."
},
{
"id": 115364,
"tgt": "Does plateletpheresis treat chronic thrombocytosis?",
"src": "Patient: Hallo Dr. My Sister (42) was diagonised with Chronic thrombocytosis last year.She has since then been on medication and at the moment she is totaly unwell which we do not know if what she feels is from the plateletes or from the medication. I have have been checking out the treatments available and have come recently learnt about plateletpheresis. I feel it`s the only chance she has. Please advice. Doctor: Hi, thanks for sharing your health concerns with HCM! I think it is essential thrombocytosis that needs some genetic/molecular mutation test for confirmation and there is no reasonable condition for high platelet count in this condition!There are medicines/injections and the last resort could be some platelet removal! However, I would like to know her treatment history, any previous disease, bone marrow/other test reports and present/past platelet counts....Hope this answers your question. If you have additional questions or follow up questions then please do not hesitate in writing to us. I will be happy to answer your questions. Wishing her good health."
},
{
"id": 72266,
"tgt": "Suggest treatment for chest pain and soreness after coughing",
"src": "Patient: hi im a heavy cigerette smoker and now everytime i cough i have this pain in my chest right below my right brest and also on the side of my right brest a little below my right arm pit. I can barely cough because it hurts so much and i cant even lay on my right side. It hurts mostly when i cough but now its getting worse to where i have it everytime i move. It feels very very sore, it feels like its swollen but i dont know/think it is, its just a horrible pain almost like a tore,broke, or pulled something? I know i neede to quit and ive already started today is my first day though. What can i do about this pain?? Doctor: Thanks for your question on Healthcare Magic.I can understand your concern. Since you are active smoker, we should definitely rule out pleurisy (inflammation of pleura) for your chest pain.Common causes for pleurisy are lung infection, bronchitis and pleural effusion. So better to consult pulmonologist and get done clinical examination of respiratory system, chest x ray and PFT (Pulmonary Function Test).Chest x ray is must for the diagnosis of lung infection and effusion. If chest x ray is normal then no need to worry much. Musculoskeletal pain is more likely. So avoid movements causing pain. Apply warm water pad on affected areas of chest. Take simple painkiller like paracetamol or ibuprofen. Don't worry, you will be alright with all these. Quit smoking as soon as possible because it is not a good habit. Hope I have solved your query. I will be happy to help you further. Wish you good health. Thanks."
},
{
"id": 69779,
"tgt": "What causes discomforts on he back with lump?",
"src": "Patient: Hi, I have had a little pain and discomfort in my lower back on the left side - and have noticed that I have a lump. It is on my lower left side of my back just above my hip - approx. 2 inches left of my spine. It moves around but seems to be attached to a muscle or something..... should I be concerned? Doctor: Hello!Thank you for the query.No need to be concerned. Most likely it is a benign lesion. It can be a cyst, lipoma or other benign skin lesion. The pain can indicate an infection (sebaceous cyst can be at this location and lead to an infection).I suggest you to consult a surgeon, have an ultrasound of this lump and consider removal of it.Hope this will help.Regards."
},
{
"id": 123775,
"tgt": "What course of treatment should be given for damaged knee cartilage?",
"src": "Patient: My son was diagnosed at 17 yrs old with a rare never seen before disease that has destroyed the cartilage in both knees. A cartilage re - implantation was tried with no success as his cartilage is diseased. His surgeon has stated that nothing more can be done for him. Knee replacements are not considered an option because of his age. We live in Australia, I was hoping you may be able to help . Thanks Doctor: Hello, The surgeon is right, at his age we cannot opt for the knee replacement if the cartilages are diseased and knee joint space is reducing. I will advice to look out for a competitive physiotherapist, who can help regain the knee functions by providing excellent strength in the knee joint muscles on both sides. As the function of the cartilage is to act as a shock absorber their absence role can be taken care by the muscles and other ligaments. It's a time consuming process to deal with exercise but it will help to avoid early ageing of the knee joint and using assisting devices like knee brace will also be benefited. But exercise in proper way to maintain the gap between the knee joint is of utmost importance to avoid pain and ageing of knee joint. We can try this out as no one would wish to have an artificial joint. Hope I have answered your query. Let me know if I can assist you further. Take care Regards, Jay Indravadan Patel, Physical Therapist or Physiotherapist"
},
{
"id": 43490,
"tgt": "Hydrsalphinx, large cystic ovary, stomach pain, severe endometriosis. Done ultrasound, CT scan. What do I fear?",
"src": "Patient: So sorry..this is a long story. I am 43 years old and had my second child in 2011 by IVF. My doctor was wonderful and immediately saw what looked like hydrsalphinx (sp?) and a large cyst on my right ovary covering my fallopian tube completely. I had my surgery and he he discovered that my uterus was also attached to my stomache so he fixed that.( The Hydro. turned out to be just a large cyst) (I also have had two c-sections) Then we proceeded with the IVF and was successful first time around. I also have sever Endometriosis and asked if he felt I should have a Hysterectomy once my child was done nursing. He basically said sure if you feel it necessary..but he indicated yes of sorts Meanwhile I moved back to Phoenix for family reasons and my pain is terrible so I went to my old OB/GYN before moving previously and they gave me a different doctor in the practice for whatever reason. I scheduled an appointment and had previous docs forward stuff blah blah blah......So she wanted a current Ultrasound....it came back as a mass on the right ovary...no surprise. ...then she wanted a CT scan??? I complied. The results came back contra-indicating ........nothing was found on the CT. I called my OLD doc in Tucson and had ALL records sent. BUT NOW......she wants a biopsy of that ovary??? before doing my Hysterectomy ?? What the heck for? Number one....the CT scan indicated nothing and a CA -125 was never run (I'm an old Lab Tech.).............She also never told me the procedure for the biopsy..as if a fine needle will say much Anyway..this has been going on for 4 months and I FINALLY have a surgery date.I really hate to get a \"second opinion\" because then I'm set back a bunch more months before having surgery.I am just sick of the pain and serious blood flow. What is your take on all this testing and the whole Ultrasound/ct scan differences????? I almost considered going back to my infertility doc who did the original surgery before my IVF..........ughI'm a wreck. Sorry such a long gripe...........Carrie Doctor: I can understand what you might be suffering of but its duty of a doctor to check things correctly from each possible way to put you in the least trouble...And yes sometimes the things it self look like a big trouble.. however its not..A simple remedy I will suggest you.. Just try .. although you are going to your old doc.. Prior in the morning just pick up the final sprouting end of your lawn grass and wash it nicely... extract a juice out of it and pour 5 to 6 drops in each nostril daily morning..This is the remedy to reduce your blood flow which may save you from few more test of hemoglobins and other required blah blah blah.. whatever you say to that..:) Secondly if you get convinced by this just kindly let me know.. so that you could be set on some personalised medicines by further evaluation of whatever papers you got printed till yet through various consultations and reports.. Okay..?where you have spent somuch time till yet I think 21 more days would not mind you much to get saved from your disease without getting operated for it..!Still in doubt feel free to communicate...Regards,"
},
{
"id": 79484,
"tgt": "What causes heart palpitations along with cough?",
"src": "Patient: Hi, I was diagnosed with cardiomyopathy 3 years ago. At my last appointment with my cardiologist he said he was surprised how much I had improved. This improvement showed on an echocardiogram. However recently I have had palpitations with a cough more frequently,but it is usually if I drink alcohol or strong coffee. If I abstain from drinking am I likely to improve again. I exercise regularly and am a healthy weight? Doctor: Thanks for your question on Health Care Magic. I can understand your situation and problem. Yes, your symptoms are mostly due to alcohol ingestion. Since you are improving with heart condition, no need to worry for cardiac cause for these symptoms. Alcohol ingestion causes GERD (gastroesophageal reflux disease). Alcohol causes laxity of gastroesophageal sphincter. And because of this patient can have palpitations and coughing. So better to avoid alcohol. Start proton pump inhibitors and prokinetic drugs. Avoid stress and tension. Avoid hot and spicy food. Don't worry, you will be alright. Hope I have solved your query. Wish you good health. Thanks."
},
{
"id": 205026,
"tgt": "Suggest treatment for schizophrenia",
"src": "Patient: My son 39 yrs is chronic patient of Schizophrenia for the last 25 years and presently on Clozapine 400 mg doze. Due to weight loss and reduced appetite he was admitted in Hospital in Dec/Jan 17 for Gastro treatment. After thorough tests scan and EUS he was diagonised with TB of lymph nodes for which Rcinex 450 and Benadon 20 mg besides Combutol and PZide for a period is six months i.e. Upto 15 sept. However with start of TB medicine psychiatric condition aggravated and we find difficult to handle at times How do we know that TB has been completely cured and no further medicine for TB required? Under the circumstances invite suggestions/ advise for further course of treatment. Doctor: Hai * TB is a infectious disease .With adequate treatment patient will be completely cured.Give priority to treat it first . This case will fall under Extra pulmonary TB . There are speciic RNTCP guidelines for the duratuion of treatment. A pulmunologist will help you more. * Schizophrenia --- Can be treated side by side. if require your Psychiatrist can go for Injectabe Antipsychotics or even Modified Electoconvulsive therapy for making symptom under control."
},
{
"id": 63222,
"tgt": "What is the treatment for a lump in the elbow?",
"src": "Patient: I have had a lump on the tip of my elbow for some time, no pain unless I put pressure on it, last night I was rolling over in bed putting alot of weight on the tip of the elbow and it felt like this lump exploded, very very bad pain, now it is really sore to touch. What could this be? Doctor: hi.it could be a cyst (a sebaceous or keratinous type) or an inflammatory reaction. if the pain is persistent and progressive and the wound is not improving/resolving, i suggest you consult with your physician for physical examination. management would be directed accordingly.hope this helps.good day!!~dr.kaye"
},
{
"id": 91766,
"tgt": "Can swollen tongue and lower abdominal pain be the side effects of using tramcinolone cream on body?",
"src": "Patient: Hi, I am a school nurse in an elementary school. i had a teacher approach me this morning stating that she has been mistakingly using tramcinolone cream (0.1%) on her face and body for two months. She is experiencing symptoms such as mild lip and tongue swelling, acne and pain/discomfort in the front, lower portion of her abdomen on both sides (ovarian?) Any recommendations? Doctor: Hi.It is unlikely that this cream can cause such problems unless patient is having uncontrolled diabetes.Please ask her to undergo tests of blood- routine, urea, creatinine, T3-T4 and TSH.. It would wise to see a Doctor , who can clinically examine her and order further investigations. Of course ask her to stop the ointment."
},
{
"id": 13510,
"tgt": "What is the best treatment for skin rash post knee replacement surgery?",
"src": "Patient: i had a knee replacement a week ago and am instructed to only get out of bed 5 times a day for 5 minutes. It s all about pain and edema control. I started with a rash about 2 days post op and it is getting worse. It is itchy and it is only on my back, butt and upper leg (non surgical) . What can i put on it? I started hydrocortisone day 6 post op. I am now day 7 post op. I have a minimum of 6 more days with the above restrictions. thanks. Doctor: Hi, The rash could be an urticaria or a drug induced hypersensitivity reaction or Tinea cruris. Treatment is entirely different for all these three conditions. So, the diagnosis could be made only after proper examination. I request you to consult your dermatologist for confirming the diagnosis and for initiating the treatment. Hope I have answered your query. Let me know if I can assist you further. Regards, Dr. Siva Subramanian, Dermatologist"
},
{
"id": 33931,
"tgt": "How to deal with fever and body ache?",
"src": "Patient: male, 23,240, 6'4\" i'v been tired and had sore legs for the past couple months figured it was just from workin but no matter how much sleep i get i'm always tired. then i got a fever two weeks ago for two days. fever was first noticed by just feeling achey all over istead of just my legs then woke up shaking un controllably, pulled anoother blanket on and woke up hour later sweating so hard the bed sheets where soaked. laid in bed all day feeling like i'd been beaten by a 800 pund gorilla. same but lesser symoptoms the next night then next day seemed gone. still tired and sore legs. light symtoms of fever came back week later and gone next day. just came back again last night a little worrse and i feel sore every where today with some pain in my chest when i fully exhale. i'v had alot of head aches to lately. ideas? Doctor: Hi. Thanks for your query and an elucidate history. There are 2 main reasons of fever with chills. 1. Malaria 2. Close cavity infection. The way it is diagnosed are :Clinical evaluation by a Doctor Tests of blood for malaria WBC Tests of blood as per clinical findings Tests of urine and stool. Ultrasonography X-ray chest. Any other tests as per clinical findings Proper antibiotics and supportive, symptomatic treatment will cure you. I hope this answers your question."
},
{
"id": 176754,
"tgt": "Suggest treatment for eczema in a child",
"src": "Patient: My 5 year old niece has a case of eczema, she was taken to PCP was prescribed flucinolone cream. The doctor instructed them to give her a bath, apply the cream, have her put on wet pajamas, then dry pajamas. Is this correct and how will she have a good night sleep. Doctor: Hello,Thank you for asking at HCM.I went through your niece's history and would like to make suggestions for you as follows:1. For eczema, treatment is usually topical corticosteroids when she has eczema symptoms. The procedure of applying them is also perfect, to be applied immediately after bathing.2. I would like to add that please continue them at least until after a week of cure of eczema symptoms. 3. I would also suggest her to apply moisturizing lotion immediately after bathing once treatment is complete. This will improve dryness of her skin and improve itching.4. If is eczema is recurrent, I would suggest her allergy testing for common food allergens like milk, peanut, egg, wheat, soy, sea food and air borne allergens like house dust mites, mold, etc.This will help you identify the allergens causing eczema to her.5. For symptom relief of itching, I would suggest her to take an antihistamine like hydroxyzine before going to bed,Hope above suggestions will be helpful to her.Should you have any further query, please feel free to ask at HCM.Wish her the best of the health ahead.Thank you & Regards."
},
{
"id": 126548,
"tgt": "Suggest treatment for pain in the operated knee",
"src": "Patient: I had total knee replacement 5 years ago. I have experienced some pain in the knee for the past 3 weeks. I thought it would go away. I have never had pain since the surgery. I have a hard time bending the knee. The pain is no where as severe as before the surgery. Doctor: Hello, The pain 5 years after a knee replacement can be related to scarring or implant displacement. You need to get an X-ray done and consult your orthopedician for an examination. Hope I have answered your query. Let me know if I can assist you further. Regards, Dr. Praveen Tayal, Orthopaedic Surgeon"
},
{
"id": 116984,
"tgt": "What is aplastic anemia?",
"src": "Patient: What is aplastic anemia? My mom has an issue of hairfall right now. She has a low platelets count. She s been taking prednisone but the side effects is her face makes bigger ... It went up a little bit but I let stop taking it coz I can t stand seeing her face so swollen ... After she stop taking prednisone she lost a lot of hair. I m so worried right now . I set her for docs appointment but still not available til next yr coz she s a new patient ... Doctor: Hi,Thanks for asking.Based on your query, my opinion is as follows.1. Aplastic anemia is decreased in marrow cellularity.2. It leads to pancytopenia, reduced cell lines of megakaryocytic, erythroid and myeloid lineage.3. Possibly prednisolone could be causing bone marrow depression. Avoid prednisolone to improve aplastic anemia and it will reduce swelling.Hope it helps.Any further queries, happy to help again."
},
{
"id": 121729,
"tgt": "What could cause consistent fluttering in right foot?",
"src": "Patient: I have consistent fluttering in my right foot. It s on the right side between ankle and baby toe. and it feels like blood rushing but it s not where a major vein is. But it has me concerned because it s been doing it every 5-10 seconds for the last week or more. Doctor: Hello,Your symptoms could be related to a pinched nerve or a twisted muscle. For this reason, I recommend consulting with your attending physician for a physical exam. Nerve conduction studies would help investigate for a possible pinched nerve. I would also recommend checking blood electrolytes and uric acid plasma levels.Hope I have answered your query. Let me know if I can assist you further. Regards, Dr. Ilir Sharka, Cardiologist"
},
{
"id": 17588,
"tgt": "Suggest treatment for tachycardia and high BP",
"src": "Patient: Have MVP, tacacardia, HBP. Sunday while I was exerting myself a bit, I began to have nausea, became light headed and left shoulder pain. Tonight, every time that I get up from laying down, my head instantly begins to throb and I get an instant headache in my forehead and temples. BP-153/101 ANy ideas? Doctor: Hello, Most likely you are facing orthostatic hypotension which is why you are feeling dizzy with a change in position. It's recommended for you to starch your legs for 1 min before changing positions and take tab Vertin twice a day. It doesn't help then kindly consult a physician personally for proper physical evaluation and treatment. Hope I have answered your query. Let me know if I can assist you further. Take care Regards, Dr Bhanu Partap, Cardiologist"
},
{
"id": 148557,
"tgt": "Can ESI injections given for spine pain lead to severe headaches and strange noises experienced inside head?",
"src": "Patient: I experienced Pneumocephlus following an spinla epidural steroid injection. The ESI injections have helped my spine related pain very much. Was scheduled for MRI the day following my 2nd (in 2 weeks) ESI (docs are keeping tabs on a brain lesion of mine every 6 months, thus MRI). My brain looked like Swiss Cheese. Not like the other - compared to images of my brain - post EPS cases I ve seen on line, report read: There has been interval development of multiple foci of extra-axial susceptibility effect involving the posterior fossa, left sylvian fissure and interhemispheric fissure. These areas correspond to T1 hypointense, T2 hypointense foci, suggesting air bubbles or calcification\u2026 Once I saw this, I then recalled, on the evening after my 1st ESI: 1st epidural, I was laying down in bed, turned over and suddenly heard a crazy sound from within the right side of my head, like manic crickets, had pain shooting upward behind right ear, from jaw to top of head. My jaw wrenched itself open (from the right), and it and the right side of my face stiffened, and I could not move it, like suddenly half my face was made of concrete. Felt like I would pass out. I briefly - but calmly - thought I could be dying. Episode passed, though, within one minute. I wondered at that time if it was related to the injections. Forgot about it\u2026 I also had had wierd headaches I won t go into after my second ESI. MY neuro docs were relieved to hear I d had ESI (not their worry, anymore). Question: Should I fear future ESI? Pain Speciaslist or neurologist question? Thank you, Lena Arice Lucas Doctor: HIThank for asking to HCMIf I would be your doctor then I would never advise you for such invasive therapy when other options are available, this kind of intrathecal injection some time create dangerous effects not because of the drug but due to trauma, better have word with your doctor for better way than this take care and have nice day."
},
{
"id": 61615,
"tgt": "Suggest treatment for lump in throat after treated strep throat",
"src": "Patient: hi, in the past week or so ive noticed a lump in my throat, it only is active at night, but as soon as i lay down it seems to appear, and when i roll over or lie on my back it seems to move and I can feel it as it slides around the back of my throat. I just recently got over strep throat and its the same feeling except i cannot see any bateria growing and i have not felt sick at all, other than neasous when i feel the lump. i cannot cough it up there is no mucous or phlegm, and once it is there it wont go away unless i drink a large amount of liquid and at that point it will slowly go down my throat until i lie down again. please help me! Doctor: may be a zenkers diverticulum which is due to the herniatiation pharyngeal mucosa through the killian's dehiscense"
},
{
"id": 80878,
"tgt": "What can cause short breath in kids?",
"src": "Patient: My 5 year old son has been taking deep hard breaths for the last couple weeks. It s almost like he is gasping for more oxygen. He has no pain in his chest. He isn t coughing. I gave him a few of his breathing treatments of Albuterol, but it doesn t seem to help him. He had pneumonia last year. I was wondering if that could ve damaged his lungs some way. Doctor: Thanks for your question on HCM. I can understand your situation and problem. In my opinion you should consult pediatrician and get done1. Clinical examination of respiratory system. 2. Chest x ray. 3. PFT ( pulmonary function test ) if he can perform. Your son might be having post infectious bronchitis. So we need to rule out this in your son.PFT is gold standard for the diagnosis of bronchitis, but pediatric patients might not be able to perform , so clinical examination by pediatrician is needed.So better to first diagnose him and then start appropriate treatment."
},
{
"id": 109458,
"tgt": "Suggest remedy for back pain",
"src": "Patient: Hello Doctor, I am female of 40 years old. I have back pain and my MRI report says as below: ** Exagerrated lumbar larodosis. ** Right paracentral disc prolapse at L4-L5 indenting the thecal sac and displacing the ventral nerve root posteriorly. ** Diffuse disc bulge at L3-L4 indenting the thecal sac and narrowing bilateral neural foramina. Please advise what is the above problem and what's the treatment for this. Thanks Doctor: HelloThanks for your query.After going through your query I came to know that you are probably suffering from lumbar disc prolapse. It is confirmed by MRI scan. There are two treatment options.First option is rest in position of relief(generally lateral position with both knee bend),neurotopics such as mecobalamin and analgesics (Diclofenac three times a day after meals is effective). Sometimes strong analgesic(such as tramadol three times a day after meals) is required. Omeperazole before meals prevent acidity caused by analgesics .Second option is go for surgical operation(Discectomy). You can discuss with your treating Doctor about it. Hope your query get answered. If you have any further questions then don't hesitate to writing to us . I will be happy to help you.You can also write to me directly on below link:https://www.bit.ly/askdrsudhirorthoWishing you good health.Take care."
},
{
"id": 89916,
"tgt": "What causes constant upper abdominal pain radiating to back?",
"src": "Patient: Hi i have been having constant pain in my upper abdomen radiating to my back as well as heartburn this has been for 36 hours straight i can barely sleep is how uncomfortable it is. I have tried tums and ranitidine they only made it worse pepto did help a little but back pain still present Doctor: HI.The reasons for the symptoms you have mentioned can be :Heart attach- myocardiac infarctionPancreatitis Gastric ulcerI would advise you to first get the ECG done, if normal upper GI Endoscopy and ultrasonoraphy to start with. Rest of the investigations as advised by your Doctor on clinical examination."
},
{
"id": 71377,
"tgt": "What are the side effects of Albuterol Sulfate?",
"src": "Patient: My 35 yr old son has always used thee inhalation aerosols: Salbutamol /Assal......a new prescription of Ventolin HFA (albuterol sulfate) has been prescribed however he failed to ask thee Dr. regarding ANY side effect to the Albuterol Sulfate. Do U think it s OK to use IT or R these ingredients a danger? If U should answer....I thank U much, this is my 1st time using Doctor: Hello and Welcome to \u2018Ask A Doctor\u2019 service.I have reviewed your query and here is my advice.Albuterol is a bronchodilator (helps in opening the air ways) commonly prescribed for the quick relief of bronchospasm and wheezing associated with bronchial asthma and other air way diseases.Its most common side effects include increased heart beats, palpitations, and tremors.HFA (Hydrofluoroalkane) is a propellant (spray) used in the inhaler to move the medicine Albuterol out of the inhaler to get better deposition into the airways to produce bronchodilation.This propellant is safe and has no harmful effects as such.Hope I have answered your query. Let me know if I can assist you further.Regards,Dr. Mohammed Taher Ali"
},
{
"id": 84620,
"tgt": "Suggest the dosage of hexilak gel",
"src": "Patient: Doctor, i was prescribed to use hexilak gel ( to apply in night) for 6 months for the scars (due to incession on my skin out of a surgery) and stretch marks in lower abs. I missed to confirm with the doctor if it has to be applied weekly or daily. can you please adivse me? -Aisha Doctor: Hi,Hexilak gel needs to be applied daily as advised by your doctor on the stretch marks for better results.Hope I have answered your query. Let me know if I can assist you further. Regards, Dr. Mohammed Taher Ali, General & Family Physician"
},
{
"id": 196050,
"tgt": "What causes itchiness in scrotum?",
"src": "Patient: Itchy scrotum I always have a itchy scrotum it changes from side to side and some times the whole thing.never had my penis itch badly. My area under my scrotum utches to.its red and there are dark veins.it has been happining for almost 3 years. I mastermate sometimes w my dogs. I have stopped doing this and it still happens whats going on Drew Doctor: hi.welcome to healthcare magic.i understand your problem.my opinion is that,1.dont scratch more.2.maintain good hygiene in your scrotal area.3.shave your pubic hair regularly.4.apply calamine lotion in your scrotal area.thank you.hope my answer will help you."
},
{
"id": 178847,
"tgt": "What causes tickling sensation in throat with breathing difficulty?",
"src": "Patient: My daughter is 13 ,she has a situation that is starting to concern me. It started a couple of weeks ago; but has become more and more frequent recently. Usually, it happens during the night or first thing in the morning (when She first wakes up). she feels a sort of tickle in her throat and the impulse is to cough. However, she has learned that if she coughs, she completely loses her breath - then she spends the next several seconds (seems like minutes) gasping and trying to catch her breath. Once she catchs her breath, everything is fine. The first time this happened (a couple of weeks ago), she just passed it off as that she had probably gotten choked on saliva or something during her sleep. what do u think is causing this symptom? Than you Doctor: All the symptoms match with bronchial Asthma. If there is a family history of allergy or asthma, the chances of having asthma to the child increase. You need to get a chest X ray done and start treatment immediately or these symptoms will go on increasing hampering the growyh of the child. Untill you get to apediatrician you can give Montair LC 10mg once daily, syr Levilin 3 times daily."
},
{
"id": 153369,
"tgt": "How long a person can survive with lung, brain and bowel cancer?",
"src": "Patient: Hi, may I answer your health queries right now ? Please type your query here...when you have lung cancer in both lungs brain cancer and had bowel cancer ,noe he is coughing up a small about of blood twice in two days how long do you think he will have Doctor: Hi, dearI have gone through your question. I can understand your concern.If you have multiple cancer then there is high chance that is it metastatic cancer. You may have lung cancer with brain metastasis. Treatment is very difficult. Surgery is not possible. So chemotherapy and radiotherapy is the treatment of choice. It is palliative treatment and can not cure the disease. So chance of survival is very low. It has poor prognosis. Life expectancy is not good. Sorry to say but it is the fact.Hope I have answered your question, if you have any doubts then contact me at bit.ly/Drsanghvihardik, I will be happy to answer you.Thanks for using health care magic.Wish you a very good health."
},
{
"id": 213937,
"tgt": "When i dont masturbate over a week i feel pain in testis",
"src": "Patient: when i dont masturbate after a week i feel pain in testis sometimes semen leakage while passing stole or at times before passing urine Do i need a proper treatment i had a herbal treatment 2 years ago for this problem . At that time it was swear but this time it just started and i got aware d. im 28 years old. Im phsically fine but this has started again. Doctor: Dear Khan, Welcome to HealthcareMagic Forum. You are perfectly normal. Semen production is a continuous process. If you do not use it ie by means of masturbation or intercourse, it will be emmitted if you are sexually stimulated either by dream or watching porno or attractive persons. Hence do not worry about this. Reduce the interval of the sexual acts to prevent night emmissions"
},
{
"id": 92196,
"tgt": "Why do I have strong dull achy pain across the whole abdomen?",
"src": "Patient: I just hit week 4 post op from my LAVH. I have been having a lot of pain in my right side, sharp and stabbing. Today my 3 year old kicked me in my belly, it hurt so bad that it made me throw up and light headed, now I have sharper pain in my right side and a strong dull achy pain across the whole abdomen. What do I do? Doctor: Hi. A kick by a 3 year old in a recovery phase of LAVH needs medical attention immediately. Stop oral intake. I wold advise you to be seen by a Doctor on an urgent basis , who on general and local abdominal examination would advise you to go for an Ultrasonography examination."
},
{
"id": 25710,
"tgt": "How to control the BP level of 170/105-110?",
"src": "Patient: I am 58. Currently working in a Gulf country with my family of wife9working0 and son 14 years, a victim of AD. For the last 2+ years, my BP wa 160/100. These days I found getting angry fast and often getting irritated and casually checked my BP. It is now 170/105-110. I am not taking any medicine so far. Please advise me if it is essential and how can I better bring my BP down without any medication. My height 5'3\" and have no other ailments other than a knee joint pain due to an injury. Doctor: Thanks for your question on Health Care Magic. I can understand your concern. In my opinion you are mostly having essential hypertension due to stress and anxiety. Uncontrolled stress and anxiety are the biggest risk factors for hypertension. So we need to treat hypertension, stress and anxiety simultaneously to achieve proper control in blood pressure. So consult psychiatrist and get done counselling sessions. Try to identify stressor in your life and start working on its solution. You may need anxiolytic drugs too. Avoid stress and tension, be relax and calm. You should also start anti hypertensive drug after consulting internal medicine doctor. Strict salt restriction in diet and exercise are also needed to achieve control. Don't worry, you will be alright. Hope I have solved your query. I will be happy to help you further. Wish you good health. Thanks."
},
{
"id": 157271,
"tgt": "Is it a lymph node that is swollen like lump around the palm between left boob and collar bone and has been diagnosed for lymph cancer?",
"src": "Patient: I went to the hospital and they told me I have lymph cancer but sent me home. Days later, now my left arm is numb and I have huge like lump round size of my palm between my left boob and collarbone. It kinda goes into my arm pit. Any idea what that could be? im not to sure if i have a lymph node right there that could be that wide ? Doctor: Hi ! Good morning and wish you a New Year 2014. I am Dr Shareef answering your query.In your history, you have not mentioned about the reasons and investigations basing on which your doctor diagnosed you as having a lymph cancer. In case it is a confirmed diagnosis, then the swelling what you have got might be an extension of such problem. Lymph nodes are everywhere in the body, and they can be enlarged due to very many reasons, and cancer is one of those reasons. If I were your doctor, I would advise you for a Fine needle aspiration cytology (FNAC) of the swelling, and depending on the result might advise you for a MRI scan, or chemotherapy or radiotherapy as the case may be. It is not possible to pin point the exact diagnosis without a clinical examination and related investigations.I hope this information would help you in discussing with your family physician/treating doctor in further management of your problem. Please do not hesitate to ask in case of any further doubts.Thanks for choosing health care magic to clear doubts on your health problems. Wishing you an early recovery. Dr Shareef."
},
{
"id": 116827,
"tgt": "How to treat AEC count of 790 ?",
"src": "Patient: I have high aec its 790 when checked and i was taking medications for 1 month till now i didnt find any relief my fingers gets swollen and itiching at times some rashes red of colour and if i eat non veg immidiately it creates a problem what is the best remedy to overcome this problem Doctor: Hi, dear. I have gone through your question. I can understand your concern. You are suffering from eosinophilia. There are many causes like allergy, hypersensitivity, asthma, parasitic infection etc. You should find the cause and take treatment accordingly. Ovetall DEC and steroids are helpful in eosinophilia. Consult your doctor and take treatment accordingly. Hope I have answered your question, if you have doubt then I will be happy to answer. Thanks for using health care magic. Wish you a very good health."
},
{
"id": 183709,
"tgt": "What causes swelling and pain after a tooth filling?",
"src": "Patient: Hi, my husband had a tooth filling on day before yesterday. He had severe pain during the filling as well as after filling for some time. Then the pain started to reduce. but yesterday he started to have fever slightly. Today no fever, but swelling is there. We are going to our dentist this afternoon - in another 5 to 6 hrs. Please let me know if that is ok. Doctor: Thanks for your query, I have gone through your query.The pain and swelling could be because of the tooth infection. Consult a oral physician and get a radiograph done to rule out the tooth infection. If there is infection in the root tip the tooth has to be treated with RCT or extraction. Before that you can take a course of antibiotics like amoxicillin 500mg and metronidazole 400mg tid for 5 days (if you are not allergic). I hope my answer will help you, take care."
},
{
"id": 19623,
"tgt": "What causes a fluctuation in blood pressure?",
"src": "Patient: Last week I fasted 18 hrs for a routine blood draw. Routine exam of heart, lungs were normal. Next day I ate half a bag of nuts prior to popcorn. The next morning there the bowel movement was like sludge and there was red flood in the water. None since and stools are normal. However, my blood pressure has been very low - like 94 over 58 probably being the lowest - and sometimes it is normal - like 115 over 76. I feel fine, no aches or pains....just tired. I would appreciate your opinion. Thank you Doctor: see the blood pressure is not a fixed thing that will not be same everyday and it will not be same the whole day ...blood pressure depends upon the food type taken as some as coffee may increase BP and it may fall after loose motions ..some herbal supplements may cause BP to fluctuate ..."
},
{
"id": 144967,
"tgt": "What causes stiffness in neck and headache post colonoscopy?",
"src": "Patient: My husband had a colonoscopy on Thursday. Everything went well. However, his neck starting getting stiff on Saturday, and now he can hardly move it. It is very tender and he has a bad headache on the back of his head. Is there a connection? When do we know to seek medical attention? Doctor: Hello. I have been through your question and understand your concern.Neck stiffness and headache should not be related to the procedure itself, but can be related to the sedation used during the procedure.If fever should add to these symptoms, and the headache with neck stiffness persist, you should definitely seek medical care.Otherwise, you should not worry about.Hope this helps. Please feel free using MHC again"
},
{
"id": 52366,
"tgt": "What causes elevated liver enzymes?",
"src": "Patient: I was diagnosed with a fatty liver back in 2001 when I had my gallbladder removed. Since then , there really have t been problems except diarrhea on occasion. Recently my liver enzymes were elevated..with bilirubin normal. Ggt was over 2000. I went gor an abdominal ultrasound yesterday, and I am very nervous. I have been a drinker, but never thought to an excess. I had no symptoms prior to the bloodwork. Can you tell me what might be going on? I also have ake many medications, and have a history of drug abuse...not shooting up at all. I was taking 4 different opiates a month as prescribed. I am currently in Subutex treatment. My health has been worse since this treatment began 2 years ago. I am totally disabled...I was just awarded this month for a diagnosis of Avascular Necrosis. Doctor: Hello, Elevated liver enzymes occur due to severe liver inflammation due to excessive alcohol intake, autoimmune disorder, disorder due to gluten sensitivity or Celiac disease, being alcoholic, Obesity, certain medications like Tylenol, statins, non-fatty alcoholic liver disease, CMV or Liver cancer. Avoid eating fried food, red meat, cut on alcohol, sugar, avoid white flour. Take walnuts, Avocado, olive oil, sunflower seed, tofu & greens, avoid junk food. Stay away from addictions if any. Get physically examined, get all liver function tests & other blood workouts done. You may have to get Imaging tests & biopsy done too. Your treatment will depend upon the pathology detected. Hope I have answered your query. Let me know if I can assist you further. Regards, Dr. Nupur K, General & Family Physician"
},
{
"id": 156424,
"tgt": "Does decreasing hemoglobin during typhoid indicates blood cancer?",
"src": "Patient: hello sir,i was having typhhoid for 10 days and at that time my hemoglobin was 7.6 and anisopoiklocytosis macrocytosis and hypochromia.so my doctor gave me some iron b12 folic acid medicine.but my hemoglobin is still decreasing and reached 7.2. is this a symptom of blood cancer..???? Doctor: Hi. I dont think this is symptom of blood cancer. Low Hemoglobin in your case is due to deficiency of iron and vitamin B12/folic acid deficiency. It take months to recover. Repeat your hemoglobin level after one month of treatment. Hope I have answered your query. If you have any further questions I will be happy to help"
},
{
"id": 206914,
"tgt": "What causes abnormal behavior of child while not on Vyanase?",
"src": "Patient: My son is on Vyvanse. He is almost 17 and is on 90 mg. of Vyvanse. He is 6 2 and weighs 240. He is fine most of the day and concentrated and focused when he is on it, but when he comes down, he is super hyper and wants to wrestle with his brother and father and even me. He is also socially awkward when he comes down also, loud and wild. He was on concerta prior to this and it did not seem to do anything. What to do next? Doctor: drug he is taking is good.add rispreridon will correct other symptoms like anger outburst.go side by side group therapy and behavioral therapy. he will be fine.take care hope my answer helped you.plz rate the answer."
},
{
"id": 172511,
"tgt": "Is it normal for the child to have loose motions and loss of apetite while taking antibiotics?",
"src": "Patient: Hi my son aged 4 has nephrotic syndrome but has been remission for about ten months now.Last week heCaene down with a throat infection and his adenoids seemed to be the other problem as he had started to breath through the mouth and snoring a lot at night. He was prescribed orelox. And avamys nose spray.He is on day six of antibiotics but he he still looks weak and has been having semi loose motions starting off with painful cramps. I think he has also lost his normal appetite. Is this normal? Should I stop the antibiotics? Doctor: antibiotics are not indicated for every respiratory infection ,because most of them are viral. Antibiotics do cause loose stools and loss of appetite. However painful cramps may suggest infective diarrhoea and child may also have a relapse of nephrotic syndrome.Antibiotics should be stopped, complete blood counts,urine and stool routine should be done to look for signs if any of infection and treatment instituted accordingly ."
},
{
"id": 26656,
"tgt": "What could cause the tingling and metalic taste when the BP elevated",
"src": "Patient: My husband experienced tingling in his arms and around his mouth. He had a metalic taste in his mouth and felt faint. He immediately went to his doctors office and his blood pressure was slightly elevated but nothing else. What would cause the tingling and metalic taste. He has good dental habits, takes bp meds. Doctor: Hello! Thank you for asking on HCM!Regarding your concern, I would explain that this symptomatology may be related to a seizure. But anxiety and low blood glucose (hypoglycemia) can't be ruled out as other possible causes responsible for this clinical scenario. The slightly elevated blood pressure may have been just a physiological reaction to the event. I recommend consulting with the GP for a careful physical examination, a resting ECG, a chest x ray and some blood lab tests (fasting glucose, complete blood count, thyroid hormone levels, kidney and liver function tests, blood electrolytes,etc.). An EEG is necessary to examine the brain activity to rule in/out the possibility of a seizure. A brain CT scan may be needed in case of abnormal EEG findings. If all the above tests result normal, anxiety would be the most probable diagnosis. Hope to have been helpful!Best wishes, Dr. Iliri"
},
{
"id": 215969,
"tgt": "Is it to be concerned about the pain in between the chest and stomach?",
"src": "Patient: Hi I went to minor medical clinic today . I have pain between my chest and stomach exactly in the center when I inhale fully i feel pain that part but the doctor said it may be gallbladder and not confirmed. I want t o know how to confirm this . Do i need to go to any speciality doctor for this. Is this a serious problem. Will this pain go by just medication or do i need to undergo any surgery. The pain is there for whole day but increasing badly some times if I take ibrufin it reduces Doctor: Hello and Welcome to \u2018Ask A Doctor\u2019 service. I have reviewed your query and here is my advice. If you have constant pain in the abdomen, I would suggest you to take medical attention and go through ultrasound abdomen so that any structural problems, like gallbladder or liver can be seen and get treated accordingly. I Hope I have answered your question. If you have further queries, I'm happy to help you."
},
{
"id": 3780,
"tgt": "What are the chances of pregnancy with ruptured follicles?",
"src": "Patient: Hello i am 39 yrs. My doc said my follicle ruptured. i got my last period on 5th oct. we had relationship on 12,14,16,18,20 oct. can u confirm if i have chances of getting pregnant. age 39, wt - 77kg, ht - 167cm , i had my first son in 1997 cesarian. Doctor: Your concern is appreciated. Every cycle, a woman in reproductive health forms an egg. This egg is initially in a watery balloon which is called as Graafine follicle. When your doctor says the follicle is ruptured, probably it means your egg is released. This happens usually 14 days prior to the next expected period. You have not mentioned whether at the age of 39 your periods are still normal or not; and if so what is the duration of your monthly cycle. I will answer your query presuming your cycle is 28 days, regular. If you had your last period on 5th October, your next expected period should be on 2nd November. So your egg formation dates to about 19 or 20 October. You had contact on 18 & 20 October. So there is a high chance of pregnancy. Anyway, now this is March so you must have got the answer through your next periods. If you have missed them, you must be pregnant. If you do not want this pregnancy, you had an option of undergoing a Medical Termination of Pregnancy legally before 20 weeks of pregnancy; i.e. before 15th February. Of course this depends in which country you are."
},
{
"id": 188597,
"tgt": "Tooth infected even after root canal and advised for extraction. Pain and taking PenVK and Flagyl. Consult another Dentist?",
"src": "Patient: I had a root canal 10 years ago, now the same tooth has become infected. My regular dentist send me to an endontist who said I should get the tooth pulled (don't have one above it anyway ) however, he would not pull it. So I called my regular dentis back..but I have to wait 20 days for the next available appt. Meanwhile I am in such sever pain that I am almost crippled every 3 hours no matter that I am taking Pen V K and Flagyl. I have been on both for 6 days, along with Motrin 800 every 4 hours and STILL its killing me. Should I seek another dentist, and will extraction help the pain go away? I am very worried about the infection going to my face, throat and neck not to mention my heart!! Doctor: Hi,It seems that you got infected root canal tooth and now pulp abscess is formed giving rise to this problem.Unless you got your tooth removed and pus is not drained your problem is not going to solve.Consult new dentist and get examined.Ok and take care."
},
{
"id": 207957,
"tgt": "What is the treatment for passive-aggressive behavior?",
"src": "Patient: I think my boyfriend ins passive aggressive. He came after me the first day of school. after our first date he never called back, then he stopped comig to school, then he d show up. sometimes he d say hi, other times he wouldn t look me in the eyer when he spoke. Then I invited him back to my apartment. He didn t make love to me he didn t kiss me, I saw his face, he made as effort not to get into it. I m really attractive so I knew something wasn t right. He admitted to me that he wanted to tear me to pieces when he first saw me because he wanted me so badly. It s been up and down. He wants me but he plays these strange sulky games. I m confused. Also he has had 2 dui s and a felony for breaking in to beat up his sister s boryfriend. He is highly intelligent, a student as the Uof I. But unable to connect emotionaly. He has grandious ideas about becoming a politician and says that is why our relationship must be a secret. I m 14 years older than him. I know something is wrong with him but I ve never come across this type of behavior. Doctor: DearWe understand your concernsI went through your details. I suggest you not to worry much. To know more about him is the only way to find what is troubling him. I rather say that as he intelligent (in your opinion) he is playing safe. You have the capacity to understand him. As he is an aspiring politician, as he says, he wants to keep himself away from affairs, which might trouble his future. That is a good opinion. At least he is focused towards his future. Things like flirting and having sex is not his concerns. he is much more matured than that. Understand him and adore him.If you require more of my help in this aspect, Please post a direct question to me in this website. Make sure that you include every minute details possible. I shall prescribe the needed psychotherapy techniques which should help you cure your condition further.Hope this answers your query. Available for further clarifications.Good luck."
},
{
"id": 52674,
"tgt": "What could cramps in back with painful gallbladder suggest?",
"src": "Patient: Hi, For the last five months I'm suffering different symptoms like cramps in back back, a pinch in my bag when I burp, kind of pain in my gallbladder area and constant pain in my right side of the rips. Sometimes I also feel very bloated. Normally I can sleep well and everything starts as soon as I start doing daily activities. I have had a colonoscopy, a scan to see if I had gallstones, also an X ray of my diagram and thousand of blood test. I still dont feel well and I really dont know what else to try. Could you help me please? I'm getting a depression. Doctor: if still you are having pain then consult gastroenterolgist get USG or CT scan done. Take pain killers as advised."
},
{
"id": 4763,
"tgt": "Irregular periods, sonography right of ovary enlarged, suggested novolin 21, blood test negative. Suggestions?",
"src": "Patient: hello Dr samuel i am Pravina i got irregular period for a consulted a doctor.... she told me to pregnancy test result was negative she told me to do empty stomach blood test and sonography i did blood test was normal... in sonography right of ovary was enlarged... she has told me to take novolon 21 for 2 month... i am worried i wil be pregnant after words or no ???? Doctor: Hello,Novolin is a combined birth control pill and it helps to regularize your period and facilitate chance of conception.You need regular cycle and proper ovulation for successful conception. You have to track down your ovulation through Ovulation Prediction Kit and have to involve in unprotected sex without any contraceptive method at that time (mid cycle or 14 days prior to due date) to maximize the chance.In the mean time, you must take folic acid and vitamin-E supplements for at least 3 months. Take balanced diet with low carbohydrate, avoid stress and control body wt. Good luck."
},
{
"id": 47172,
"tgt": "Suggest remedies for weakness with kidney stones",
"src": "Patient: Dear Sir My father is suffering from Diabetic foot gangerence and he gone through surgery where he lost his one thumb and 2 fingers of his toe he got small stones in both of this kidney and he is not eating results he is quit week when ever he eat he vomits ,but now yesterday only he is being discharged after dripping 3 units of blood he is eating abit but he is very weak Doctor: Hi,Thanks for writing in.An adult male requires 2000 kcal diet. This should contain balanced amount of proteins, carbohydrates and fats.Your father can follow a sample daily diet chart is as followsEarly MorningA cup of tea/ milk with 2 biscuits of your choiceBreakfast2 parathas with curd or 2 plain rotis with paneer or 2 idlis with chutney/sambhar or 1 bowl of upma or oatsMid morningSoup or a whole fruit (apple or guava)LunchSalad with rotis (2 to 3) or rice (1 bowl) with dal/ sambhar and a cup of vegetable. Occasionally you can add some light fries in lunch. You can add two vegetables every day.AfternoonTea/ juice with 2 biscuitsDinnerSame as in lunch with slight variation. Dinner should be lighter than lunchMilk after dinner He should take iron and vitamin supplementIf kidney stones are normal and there is no obstruction in the kidneys then there is no problem. Please do not worry."
},
{
"id": 122779,
"tgt": "What causes knee pain and high uric acid level after recovering from dengue fever?",
"src": "Patient: I suffered from viral fever one year ago which dr said likely dengue with lot of joint pains with increased uric acid level. Even now there is pain in my right knee & bothers me wheras left knee is absolutely normal with no problem. What could be underlying cause n possible treatment Doctor: Hi, As first-line management, you can take analgesics like paracetamol or aceclofenac for pain relief. If symptoms persist better to consult an orthopaedic and plan for an X-ray. Hope I have answered your query. Let me know if I can assist you further. Regards, Dr. Shinas Hussain, General & Family Physician"
},
{
"id": 30765,
"tgt": "Suggest remedy for persistent chest infections",
"src": "Patient: I have had continuing chest infection for nearly two months. x-ray showed pneumonia in one lung and a few weeks later same in other lung. I have been on antibiotics all this time and have just recovered from head cold.Have had flu vaccinations during all this. I am in my 70s and had whooping cough vacs as a child. Dr. said I haven t got whooping cough but have had some coughing fits similar to whooping cough. Am on large doses of puffers to I guess dry out lungs of pneumonia. I have had asthma and bronchitis (mild) all my life. Some days feel as if I m getting better then symptoms come back.Please doctor tell me what is the name of this bug and when will I get better?Thanks Diana Doctor: thanks for posting your query to HCM .As I understand it is a chronic respiratory problem may develop in to Chronic obstructive pulmonary diseases (COPD) . Improper aeration in all segment of lung because of solidification and improper drainage of secretion because of constriction of bronchiole favour growth of microorganism . Immunity is also decreses in old age . All this facor are responsible for your problem .first you should investigate for culture and sensitivity exmination of your SPUTUM to know the causetive organism of pneumonia and antibiotic to treat the organism . As untill and unless you will not take antibiotic properly and completely you will not get cured .once you will be free from infection it would be easier for your clinician to give you symptomatic treatment .take care ."
},
{
"id": 191698,
"tgt": "Is Glycomet advisable for high blood sugar levels?",
"src": "Patient: Doctor I am 60 years male,my blood suger level is fasting 140 and post is 166 ,as the doctor advice taking glycomet 500mg SR ,morning and evening 1/2 an hour before meals.I have no other deceases.is it can continue ?,any side effect on this medicineor any alternate medicine Doctor: Glycomet is the right medicine for high blood sugars. Glycomet SR has much less side effect that most of the time you won't notice any. You can take this medicine as this is a time tested medicine for diabetes."
},
{
"id": 35572,
"tgt": "Suggest remedy for flu",
"src": "Patient: My sister has night sweats, muscle tenderness/pain, cough, fatigue, chills and as she describes it symptoms very similar to the flu. But since she was on lovistatin for 2 years she thought at first it was from that. But she has been off the drug for threes weeks or longer with no improvement in symptoms. I was thinking she should see an infectious disease doctor. Help? Ashton Doctor: Thanks for posting your query to Health care magic . As i understand your sister is sugffering from upper resoiratory tract infection . it could be of invective or allergic in nature . I suggest you to maintain a good hygienic and nutritional condition as lower immunity is a predisposing condition for infection . As I understand it may be viral in nature .So i suggest you to take 1. Antihistaminic with mast cell stabilizer : Cetrizine 10mg with Monteleucast 5mg tablet once in a day.(Adult dose )Tablet Paracetamol 2 times a day as I in standard dose as per age of your sister for fever and bodyache.Multivitamin with Zinc tablet to improve immune response .Antibiotic like Azithromycine 500 mg once in a day can be taken after consult local clinician as it has no role in viral illness .Hope I have solve your problem . feel free to contact me.Regards,Dr.Manish Purohit"
},
{
"id": 181628,
"tgt": "Suggest medication for toothache with swollen gums",
"src": "Patient: hello sir,its about my mother .she is 42 years old and from the last 3 days she had severe toothache and also had swollen gums because of that she had following problems:pain in her 4 teeth of her right side pain around her nose toothache the dentist told that she had infection in her one teeth which gets spread over her gums of 3 more teeth.she suggested to do root canal but we don't want it so plz suggest some medicine for this we will be vry thankful to u thank u Doctor: Hi Dear ,The symptoms you mention in query are pointing towards a condition known as periodontal abscess .The condition occur due to infection that travel from tooth to the surrounding area of root of the tooth or directly from gums .I would suggest you to :_Consult dentist for proper oral prophylaxis like scaling and curetage along with antibiotics .Doctor may perform root canal if the pulp of the tooth get infected .Doctor may also perform apicoectomy in case of abscess at the tip of the root (in number 4) .Till now you should :-- Do warm saline rinses several times a day . .- Do betadine rinses twice a day .- Avoid hot and spicy food .- Take ibuprofen and acetaminophen for pain and swelling .Best Wishes,Dr. Harry"
},
{
"id": 171449,
"tgt": "What could lump on lower spine in a child without bruising suggest?",
"src": "Patient: My 6 yr old daughter has been complaining about her lower back hurting for about a wee. I checked and saw nothing. I checked again and she has a Lump just off to the side of her lower spine and there is no bruising.. What could this be? I calling her ped in the am Doctor: Hi.... by what you quote I feel that it could be a small lipoma or accumulation of fat. The Other possibility is that it could be dermoid cyst which has been overlooked. I suggest you see your pediatrician regarding this as the diagnosis can be made with certainity only after seeing and touching the swelling.Regards - Dr. Sumanth"
},
{
"id": 100951,
"tgt": "What causes vomiting after consuming seafood?",
"src": "Patient: I ve gotten sick after eating seafood; the first time it was a small amount of clam dip and the second time a couple months later it was flounder. Both times nobody else who ate it got sick. Both times the EXACT same symptoms occurred. Ate it for dinner; woke up at 4:45am vomiting... I vomited for a couple hours and then stopped. This was 5 years ago and I have stayed away from seafood ever since and have not had any issues whatsoever... I went to my doctor who tested me for all allergies and I am NOT allergic to ANYTHING; so I m told. What could this be? Doctor: Hello,Welcome to HCM,Your history and symptoms are suggestive of allergic manifestations, you are allergic to sea foods. I would suggest you to undergo lab investigations like CBC and absolute eosinohil count (AEC). AEC is an indicator of allergy in the body.To confirm that you are allergic to sea foods I would suggest you to undergo skin prick test with different most consumable sea food, this will helps to confirm the allergy to different sea food.You can avoid the sea food to which you are allergic or you can prefer immunotherapy. You can apt for either subcutaneous immunotherapy (SCIT) or sub lingual immunotherapy (SLIT).For the present symptoms I would suggest you to take Tab Montek LC, once in night for minimum of 15 days.Thank you."
},
{
"id": 184002,
"tgt": "Suggest remedies for pain in upper gums and a nodule after denture removal",
"src": "Patient: After six months complete extractions for dentures there is a hard nodule on left upper gum that causes pain on the left side into the temple area. When I put pressure on the nodule area with my tongue the pain will subside. Had nodule removed by oral surgery and for a few days it was better, but the pain and nodule have returned. Doctor: Hello,Thanks for consulting HCMRead your query, as you have nodule after complete extraction it can be bony spicule present in jaw , this can be reduced by surgical procedure Alveoloplasty or that nodule can be fibroma growth in jaw treated by excision dont worry if there can be regeneration of growth it can be induced by denture may be irritation fibroma for this I will suggest you to consult dentist and go for investigations of growth if needed go for biopsy of nodule and if there is any piont in denture causing pressure in jaw or ulceration in oral mucosa go for adjustment of denture as early as possible , Do warm saline rinses.Hope this will help you. Wishing you good health."
},
{
"id": 54726,
"tgt": "What could be the cause of black stools after undergoing endoscopic stenting for gall stones?",
"src": "Patient: My Dad is 79 was having severe abdominal pain and vomiting for about 1 month went to ER monday did CT Scan found very large gall stone blocking duct to liver had Endoscopy on Tues couldn't remove stone was to large had a second endo on Thurs and they quote filleted the duct and removed the stone, put in a stent, cleaned area of slugh then yesterday he started passing dark/black stool each time he went to the bathroom total of 5 times, Dr. immediately did another endo to look for bleeding and to put a sealant over the duct area from previous day he stated he didn't think the blood was coming from that area. Any idea what could be going on with this situation? Doctor: Hello! Thanks for putting your query in HCM. I am a Gastroenterologist.Presence of black stool suggest bleeding from upper GI tract. And with a prior history of ERCP suggest bleeding from the papilla site as papilla has to be cut for stone removal. And if your has checked that site and is sure that it is not bleeding from that site, means it may be blood passed in Gi tract during the procedure and now the bleeding has stopped.So keep watch on stool colour, which may be black for another 3-4 days after which it should be cleared. In case it is not cleared ther repeat endoscopy is warranted. I hope I have answered your query and this will help you . Wish you a good health"
},
{
"id": 38675,
"tgt": "How to treat open wounds?",
"src": "Patient: Hi I ve got surgery 21 may 2014, four days later I got a open wound, my doctor prescribed me silver sulfa, it s getting worst, aand I stopped ... And now the other breast is opening week later have green/ yellow secretions infection , I m taking antibiotics every 4 hour, I covered with gauze also I ve change every day,please I m so scare, my doctor always said that s ok we ll need two months to heal What can I do or eat or what to treat it? Doctor: Hello,Thank you for writing to HCM,Because most open wounds are minor, they can be treated at home by doing the following:1. Wash and disinfect the wound to removing all dirt and debris.2. Use direct pressure and elevation to control bleeding and swelling.3. Bandage the wound with a sterile dressing or bandage (very minor wounds may heal fine without a bandage).4. Keep the wound clean and dry for five days.5. Rest.6. Take acetaminophen for pain if needed.7. Apply ice if there is bruising or swelling.8. Do not pick at scabs.9. Use SPF 30 sunscreen over the area until completely healed.See your doctor if:1. the open wound is deeper than a half inch2. the bleeding does not stop with direct pressure and/or lasts for longer than 20 minutes3. the bleeding is the result of a serious accidenthope you will find it out helpful.Thanks and Regards,Dr. Gaurav Prajapati"
},
{
"id": 5214,
"tgt": "Trying to conceive. HSG showing right tube blockage and left tube dilation. Severe abdominal pain, pain in the leg. Are these related?",
"src": "Patient: Hi I am 31 yr olf ttc for 8 mth. Periods are ranging from 26 days to 30 days ; I got HSG and Day 3 baseline test done ; Ovaries and eggs are good ; HSG indicated right tube bloakage and left tube dilation (20% chance); But left may be clean is what the RE said; I did the test on CD 10 May 17 2013 ; I am on CD 17 today with mile spotting and acute pain in lower right side of the abdomen ; This pain normally comes during menstrual cycle. But this side has been paining now for like 3-4 days ; It comes for about 20 min and goes away ; Occasionally my legs ache too. What could be the reason for the same . Please let me know. Doctor: Hi,Thanks for the query.As you are having right side tubal block and now having pain on the same side, better to consult gynecologist and get examined.As you are trying to conceive and as sometimes you are having shorter cycles, though rare there is possibility of tubal ectopic pregnancy. The other possibility for pain is pelvic inflammatory disease.Consult gynecologist and go for ultrasound to find out the possible cause.For more details you can ask me through: http://www.healthcaremagic.com/doctors/dr-sree-gouri-sr/63429Take care."
},
{
"id": 122253,
"tgt": "Can cocaine use strengthen broken bones and speed up healing?",
"src": "Patient: I recently fractured my fibula while ice skating with my girlfriend, directly above my ankle. My question is a bit of a strange one, but I recently read several articles explaining how cocaine use has been known and documented to streangthen broken bones and speed healing time. Where smoking deprives the blood of oxygen, and copious amounts of alcohol consumed can and will thin the blood causing delayed healing time, will cocaine use speed healing time or streangthen bones? The use of cocaine is associated with negative thoughts, the medical benefits of cocaine go unnoticed by many. Can levemnisole-free cocaine in fact provide positive benefits towards a fractured fibula? Whether its reducing swelling, providing better circulation towards a would? Doctor: Hello. Thanks for writing to us. Cocaine use does not cause a bone strengthening or hastens up the process of healing. Rather a healthy diet and avoiding the cocaine use can help. I hope this information has been both informative and helpful for you. Regards, Dr. Praveen Tayal . For future query, you can directly approach me through my profile URL http://bit.ly/Dr-Praveen-Tayal"
},
{
"id": 65241,
"tgt": "What causes a pea size sensitive lump next to my clitoris?",
"src": "Patient: I have a pea sized lump next to my clitorious. It is white and under the skin. It has been there for about 4 months and has not changed in size. However last night when I was in the shower it became veey sensitive and is giving me a very sharp pain when anything touches it. Doctor: Thanks for your question on HCM. By your history and description, in my opinion you are having infected boil or infected sebaceous cyst. Painful, whitish swelling suggest infection. And clitoris area is having a lot of moisture and hair. So possibility of skin infection like boil and sebaceous cyst is more. So better to consult gynecologist and get done clinical examination. You may need antibiotics. Avoid too much moisture in private parts. Wear cotton undergarments only. Take proper hygiene of vaginal area. Don't worry, you will be alright."
},
{
"id": 188891,
"tgt": "Lower right gum pain, radiating. Wisdom tooth?",
"src": "Patient: I woke up with horrible acute pain in my lower right side of my gums associated with my back tooth and now it's radiating into the bone a little as well. Is this my wisdom tooth or could this much pain start this quickly from just inflamed gums in a localized area (either from stuck food from last night or some other acute gum disease)? Doctor: Dear friend.Thanks for sharing your concern.The cause for acute pain could be an impacted wisdom teeth.The tissues covering the teeth acts like a pouch and accumulates food debris there.upon bacterial multiplication, on decayed food debris and infection the pain arises.it may contain pus too.Therefore i would like to suggest you to visit your dentist and get yourself examined.An x ray can be very helpful in establishing better diagnosis.Meanwhile please do antiseptic mouth rinsing and warm saline rinses.Hope this helps.Thanks.Take care."
},
{
"id": 4747,
"tgt": "Trying to conceive. Have slight bleeding, spotting, past ovulation. What is wrong?",
"src": "Patient: Hi my period is due in 3 days. Me and my husband are trying for a kid, for the past 7 months but with no luck. This month also we had regular sex every other day. Yesterday soon after intercourse I had a slight bleeding or rather spotting (it could be some 13 or 14 days past ovulation now, not sure). Is there anything to worry about. Doctor: Hi,There should be nothing to worry about, just wait for your period and if you miss it go for a pregnancy test with a fresh early morning sample of urine, 3-7 days after the missed period. If pregnancy is confirmed, it could be an implantation bleed; if not, it is the premenstrual spotting. If you wish for an earlier confirmation, you can get serum beta-hCG assay as it is more sensitive and specific. Hope this helps."
},
{
"id": 55902,
"tgt": "How to treat gallbladder wall thickening?",
"src": "Patient: My six years daughter was done ultrasound and found that the report shows that gall bladder wall is mild thickened. The complain is - she is bed wetting till now and ha pain in stomach sometimes. What could be the consequence s, effects, treatment etc.? Doctor: HelloGall bladder generally becomes thickened due to cholecystitis,ascitis etc.Your daughter need clinical correlation.I presume that there is no calculus in gall bladder then pain may be due to acalculus cholecystitis.She may need broad spectrum antibiotics and analgesics.Gall bladder thickening in ascitis is called as pseudo thickening and it is not due to inflammation.This condition doesn't require any specific treatment.Get well soon.Take CareDr.Indu Bhushan"
},
{
"id": 50194,
"tgt": "Suffer from kidney stones and fibromyalgia. Need a suitable diet",
"src": "Patient: Hi. I produce really large kidney stones and have been put on a foods to avoid diet - which was, basically, my whole diet. I am going to start taking Fosomax and was given a list of food to eliminate in this month before starting and then carry through for the next 2 years. Foods to avoid with my fibromyalgia is, well, another long list. Where can I find a list of food which I can eat? Doctor: You may google search for 'National Kidney Foundation' & various diet options for different kidney conditions are suggested.Meanwhile,maintain a high fluid intake(3-4lit) daily,avoid junk food,bakery food,fried food.Do not take over the counter drugs.Have good amount of fresh fruit ,salad in your diet.Stop consuming tobacco,alcohol in any form if you are consuming it.These are some of the tips which are likely to help you.Best of luck."
},
{
"id": 171822,
"tgt": "What causes sticky stool in infant?",
"src": "Patient: i have a baby 20 days old baby.she was giving stools 3-4 times aday. about 5-6 days ago she stopped passing stools. i treated her 2 time with glicerin candel (i m sorry - i don t know the english word for it),and she passed stooles very smelly and look like paste..but she just gave the one stool at the time and then stopped. the doctor gave her magnezium milk,she is treated for 3 days with no help. she is having lot of stumack pain,pushing her legs ,not sleeping and crying a lot - mainly in night. she is breastfeeded.i m smoking about 5-6 cigarretts a day,does this have any conaction??? shuold i be worried for her? what can i do? Doctor: Hi,Thanks and welcome to healthcare magic.Sticky stools with smell may occur if fat content is more.It may be due to glycerine suppository also..crying of your baby may be due to infantile colick.Colimex-DF 5 drops 3-4 times daily may give relief.Hope this answer serves your purpose .Please feel free to ask further queries if any.Dr.M.V.Subrahmanyam."
},
{
"id": 46463,
"tgt": "Why is my sister's kidneys not functioning after diagnosed with blood disorder?",
"src": "Patient: Hi Doctor my sister got a blood disorder nd now her kidneys not functioning nd doing dylises too, she told me to buy some these vitamins nd I'm in Australia nd I want to take this, do you know about these vitamins, She is suffering from blood disorder nd her kidneys not functioning nd doing dyleses too, they are as follows zegavit, evion 400, energy spectrum. sevenseas great help, Randy Doctor: Hello Randy and welcome to HCM.As an Urologist,i can understand your anxiety.Dialysis is done,when both kidneys have failed.In that stage,it would be ill advised to take anything,not advised by the nephrologist. As you've not written,what the blood disorder is, it's difficult to advise,the effects of these medicines.Vitamins are harmless,though.Wish her well.Dr.Matthew J. Mangat."
},
{
"id": 112046,
"tgt": "What is the cause and solution of back pain for no apparent reason?",
"src": "Patient: I'm a girl and Ive just turned 14. My back continually hurts straight down the spine mainly in the middle, my posture is really good and I never wear backpacks. It doesn't hurt while I sit but mostly when I lie down it keeps me up trying to get into a position were it doesn't hurt also if i walk home 2 miles everyday and it hurts when I've been walking for a while. How can I get it to stop and why does it hurt? Doctor: Hello, I have studied your case.Most probable reason for your symptoms could be postural problem as you may be sitting for long time and lack of exercises.Some exercises which can be done after pain has reduced include- Spine extension exercises, Lying on your stomach flat lift leg 6 inches from ground, do it for other leg.Now lift both hand and leg simultaneously, 6 inch off the ground and stayPosition for around 10 breathes. Core stabilizing spine exercises will help.You may consult physiotherapist for further guidance. He may start TENS, or ultrasound which is helpful in your case.Some life style modifications to prevent pain\u2013 Take break every 40 min from computer and do some lumbar rotation, stretches, walk and get back to work. Sit with taking support to your back, and do not watch television constantly.I will advise to check your vit B12 and vit D3 level.Hope this answers your query. If you have additional questions or follow up queries then please do not hesitate in writing to us. I will be happy to answer your queries. Take care."
},
{
"id": 98966,
"tgt": "What causes itching and pricking sensation in fingers and chest?",
"src": "Patient: I am feeling itching and pricking sensation in fingers and chest which lasts for few seconds and go away. After sometime it again repeats. Also I can see very small bumps on my index and middle fingers, not red in color. It has made my skin also very rough. I have been taking Cepodem XP 32 for past 3 days and it started after that. Doctor prescribed me Cepodem for 5 days. Is my itching related to it? I am wondering if it is safe to continue? Doctor: HelloThank You for contacting HCM.In my opinion the symptoms are suggesting allergic reaction. Did symptoms appear after taking some particular food, insect bite? If yes then these are most probably because of allergy. Following measures will help to reduce the severity:> cetirizine one at night for 5 days.> Avoid scratching else scar may form.Report to hospital if:> Condition remain same after 2-3 days> Any unusual symptom appears> Condition deteriorates.> There is difficulty in breathing.Hope this answers your question. If you have additional questions or follow up questions then please do not hesitate in writing to us. Wishing you good health."
},
{
"id": 90744,
"tgt": "Can pain in the stomach be due to back?",
"src": "Patient: Hi there, I have pulled my back last Friday (lift-up a box and the damage was done ...) Since approx. Saturday I also can feel pain on both sides of my lower tummy (feels like period pain) Can it be connected ? (back is still hurting) Many thanks for your answer. Alzbeta Doctor: Yes, it is most likely musculoskeletal pain.While strain in back muscles, it may cause pain in abdomen as well.To confirm, I would suggest ultrasound of abdomen & pelvis, till then muscle relaxant & pain killers are usual prescriptions.Hope this information is useful to you.Thanks. Regards."
},
{
"id": 185353,
"tgt": "What are the side effects for using corsodyl?",
"src": "Patient: Recently I went to a dentist and was told I have gingivitis. The dentist said to use corsodyl and tht I will be referred yo a hygienist. I don't see the dentist again till the 27th and I've been brushing and using corsodyl twice daily. The thing is, my gums are now feeling really sore and everyday I'm worrying about my gums. When I look at my gums, they look okay, but the soreness is now becoming a huge worry for me. I'm constantly worrying that My gums will recede despite the fact I'm cleaning my mouth regularly. I won't be referred to a hygienist until my wisdom teeth are out and I've had a filling done, by which time I fear it will be to late. Doctor: Thanks for using health care Magic.Read your query.I would advice you to stop using the Corsodyl,if the soreness is increased as it a more common side effect. Or you can use a more diluted form of the corsodyl.*Get your teeth cleaned first and then you can reuse it for a few days as prolonged continuous use of mouth wash is not recommended.* Use salt water gargles.Relax and get the treatment done.Thanks and regards."
},
{
"id": 102363,
"tgt": "Will antibodies help reduce nasal congestion?",
"src": "Patient: My husband has such bad nasal congestion for the last two weeks - getting increasingly worse -has seen his PCP-been on antibiotics - recently had a CAT scan of head - waiting results. Face so swollen this morning he looked like a caricature. How serious can this be and if does not resolve - what can happen? Doctor: Hello, Welcome to HCM, Oral antibiotics definitely help to treduce the infection and there by reduces the nasal congestion. As your husband is having severe nasal congestion I would have suggest him 1.Steam inhalation with Vapo caps 2.Oral Nasal Decongestant 3.Oral LEvoctrizine with montelocast The medicines can be taken after getting the prescription from your traeting doctor. Thank you."
},
{
"id": 206318,
"tgt": "Suggest ways to improve self confidence",
"src": "Patient: Sir,I have pigmentation problem from last 15 years..I used lot of mediation,but I couldn t found permanent solution.sir Iamworkinig in bank.because of my prob I can t be confident in my job.is there any permanent solution for my problem.waiting for your answer sir Doctor: Hi dear,Thank you for your query.Your problem is secondary to your pigmentation problem.once your pigmentation is gone away your self confidence is also gets better.First of all consult skin specialist for your pigmentation and if treatment is possible than complete the course.Than consult psychiatrist for your secondary depression and lack of self confidence.You required both medication and psychotherapy for boost up your self confidence.Not to worry all will be good.Thank you"
},
{
"id": 83525,
"tgt": "Can discontinuing omnacotril cause loss of weight and respiratory problems?",
"src": "Patient: My father suffering from Remetoid Artharities for long 6 years and has been taking medicine omnacotril 10mg for long time right now my father not taking it and my father loss his weight to 40kg and and feeling sqeez feeling in unable to walk and getting problem for respieratory Doctor: HiIntense fatigue,weakness,muscle or joint pain ,diarrhoea or constipation can occur as withdrawal symptoms to steroids.Steroid withdrawal effects can take 4 to 6 weeks to reduce since the body needs to adjust to the absence of steroid level in the body.Gradual tapering of the drug by reducing the dose and duration slowly can prevent the withdrawal symptoms.Any other causes of weight loss such as chronic infections should be ruled out.Hope I have answered your query. Let me know if I can assist you further. RegardsDr.Saranya Ramadoss, General and Family Physician"
},
{
"id": 40137,
"tgt": "What is the treatment for tightness in legs and knee, rib cage pain and constipation caused by systemic scleroderma?",
"src": "Patient: I think I have systemic scleroderma, symptoms are legs feel tight and hard up to below the knee, under the left rib cage feels hard and blocking, digestion is not so good.., queasy stomach, right side feels hard...constipation more than normal, I have systemic raynaulds turn purple head to toe since a toddler. Just found out the pcp has been not documenting my concerns for sometime. Pcp has been writing continue as usual. I need to find a rheumtaogist that will take me with poor documentation from pcp. Doctor: Dear Friend.Hi , I am Dr Anshul Varshney , I have read your query in detail , I understand your concern.Tightness cannot be treated as such, but symptomatic relief you can get by applying a lot of moisturizer locally.Also, a good systemic treatment you should take, that would slow the progression of the disease and skin tighteningThis is my personal opinion based on details available here. If you want to discuss your issues further, you may please ask usStay Healthy. Dr Anshul Varshney , MD"
},
{
"id": 200200,
"tgt": "What could the rapid and slow progressive motility rate on pre-seminal fluid test signify?",
"src": "Patient: Hi doctor, I did a pre seminal fluid test and my rapid progressive motility and slow progressive motility is 0 Non progressive motility is 3.8 Can I know if it s good or bad and the causes of it Also my sperm concentration is 5.8 And my sperm count is 11.5 million Doctor: Thanks for asking in healthcaremagic forum In semen analysis sprem structure,count,motility and other things are observed. Progressive motility is seen in healthy sperms. COunt though low, may not hinder you to have children. Please contact urologist/andrologist for help. All the best."
},
{
"id": 35972,
"tgt": "What causes itchy bumps and pop ups all over the body?",
"src": "Patient: Help the pain and itching I went on vacation stay at the hotel 3days came to my grandmom house right afterwards and stayed the nite there, woke up the next morning, with big bumps that hurt abd itch badly, its been two days and abd they still hurt and itch and pops up in different places on my body...plz help me Doctor: Good day and thanks for the question. You probably having an Allergy and could be that you slept on a bed bug or something. Kindly take Diphenhydramine 25mg 3 times daily and Tab Acetaminophen 1g 3 times daily. You can apply Calamine lotion to the sites. Hope it helps. Thank you."
},
{
"id": 175982,
"tgt": "Suggest treatment for accidental consumption of liquid cleaner",
"src": "Patient: My son had a mr clean bottle in his mouth when I came around the corner. He got into the cleaner cupboard somehow the ought he child proof lock and the sprayer was in his mouth. I took it away and rinsed his mouth very quickly with water. Is it absolutely necessary to bring him to a doctor? Or what should I do? Doctor: Hi....It is absolutely necessary for him to be seen by a doctor. Usually these cleaning substances contain ingredients which are harmful for the mucus membranes. I suggest you take him to a physician or pediatrician as soon as possible.Regards - Dr. Sumanth"
},
{
"id": 41161,
"tgt": "What causes sudden increase in the sperm count?",
"src": "Patient: I have 15 million sperm count on March'11, but 2days earlier it becomes only 2 million. I am taking some drugs which help to increase sperm count as well as motility. But this abrupt change shocked me and I want to know that Is it possible or not? If yes, then what should I do for the next time??? Doctor: Hello, in my opinion you need to recheck specimen after absteinence of 3 days and then see the counts.In case you have any questions in future you can contact me directly on http://bit.ly/drmanishajain"
},
{
"id": 175326,
"tgt": "Suggest treatment for vomiting and loose motions",
"src": "Patient: Hellow doctorMy baby is 3 months old n past few days she has been vomoting and loose motioning after every feedI took her to my pedtrcn he has given me medcation for loose motion and stop vomit that has actually worked but fot the loose motion it still doesnt stop Doctor: As you have said your baby has diarrhea and vomiting, first of all the number of episodes of loose motions is very very important factor in case of children that you have not mentioned. You can give him syrup nor-metrogyl or syrup O2 2.5 ml twice daily for diarrhea along with syrup vomikind or syrup ondansetron 2.5 ml twice daily for vomiting with some probiotic like sporolac sachet one at least twice daily. Last but not the least, don't stop to breast fed her continue it as many times as your baby wants."
},
{
"id": 40419,
"tgt": "Could yeast infection be related to pain in tailbone?",
"src": "Patient: i believe that i have a yeast infection (for about 3 days or so now) and last night my tailbone really started to bother me and it has worsened today. i haven't fallen on my tailbone ever and am not sure when it is bothering me. could these two be related? Doctor: Hello,Welcome to HCM,Fungal infection can affect genital area and the area around the anus. The fungus are the normal comensals of these area, it will be kept under control by the bacteria present in this area.Whenever there is a favorable environment in this area it will produce the symptoms of the fungal infection like pain, itching and discrete rashes. As the fungal infection can occur in the anal region that is below the tailbone, the fungal infection can occur in this area.The pain in this area may be due to boils which can occur in this area because of presence of lot of hair follicle in this area.Thank you."
},
{
"id": 15641,
"tgt": "Red and course rash on right thumb across hand. Took antibiotics. Reason for rashes?",
"src": "Patient: Doctor, I awaken this morning and notice a rash from the bottom of my right thumb inch across the back of my hand? The surface of rash is red & coarse. I was on my last day of antibiotics , which I last took at 11am , but did have a glass of wine at 6:30pm with italian food. During my taking antibiotices I did not drink at all. I only comsume alcohol 4 times a year. ??? Doctor: Hithanks for your query.You seed to have developed an eczema of the thumb.You need treatment with topical steroid creams.Oral antihistamines can be taken for the associated discomfort and itching.Avoid using any oils or moisturizers over the affected area.Hope it helpsDr Geetika Paul"
},
{
"id": 223547,
"tgt": "How can future conceptions be avoided safely?",
"src": "Patient: My penis is 17 inches long and I m afraid it s to big every time I ejaculate about 5 to 6 oz. Or more Of seaman comes out and my wife is worried because we just had triplets boys and she doesn t want to try again to have children even though she wants a girl what do I do ? Doctor: Thank your for the question and I am happy to help.I can understand that it is probably hard to find condoms that fit, but if you can, then you should consider using condoms of some kind. Besides that, and especially in light of the fact that you have triplets and do not sound ready to have more children, she should consider some form of reversible birth control that does not involve a delay in pursuing future children. An excellent choice would be the Mirena IUD.I hope that this helps and good luck!Dr. TIm"
},
{
"id": 48310,
"tgt": "What could cause light fluttering in right kidney with dull ache in pelvis?",
"src": "Patient: ive been a nurse for 25 yr so with that goes self dx..ive been having light kidney fluttering on right side..i prob drink half to 1 gal of h20/d for years..no soft drinks in yrs..recently started protein shakes, stopped those x 2wks..had my urine checked no blood,protein or ketones..no urinary s/s...fluttering last up to 5-10 sec few times/day..then a dull achy pelvic feeling..non painful, not req drugs???? Doctor: I suggest you do get at least an ultrasound scan of kidneys and bladder done, and confirm if serum creatinine is within normal limits.The description of your pain us not like the conventional renal pain or colic, and we will be in a better position to advice you if your scan has been done."
},
{
"id": 61449,
"tgt": "What causes a lump on the forehead above the right eye?",
"src": "Patient: my husband just came in with a large \"goose egg\" on his forehead above his right eye. He was catching kid's pitches and said the ball \"just grazed me- it didn't really even hurt\". He feels fine, no headache, no nausea and his pupils are equal. He has an icepack on it now. Do I need to make him go to the ER/medical clinic tonight? Doctor: Respected user, hi I evaluated your query thoroughly.* Hematoma formation over forehead.* I recommend following suggestions - continue ice applications 4-5 hourly- maintain his hydration status- in our clinic we recommend certain antibiotics & anti-inflammatory agents for faster recovery with regular follow up- evaluation may require in form of Ultrasound / CT if later increase further, appearance of pain, vomiting , vertigo / else .* In a nutshell , can wait for 24 hours regarding its progress or else can visit doctor .Hope this helps you.Thanks for using Healthcaremagic.com & giving me an opportunity to assistWelcome for any further queries.Wishing him fine recovery from the same.Regards dear take care."
},
{
"id": 148070,
"tgt": "Is recurred squishy bump on head oozing out clear fluid when popped with history of throbbing headache serious?",
"src": "Patient: I ve always had a small bump on my head (probably about the width of a pea) which I think is a scar from when I split my head open as a baby. A few weeks ago, the bump got really painful, was swollen and it felt like a throbbing pressure on my head. I went to a walk in centre and a nurse told me it should go away in a few days. It did. It s now came back, and I felt it and it felt squishy. I popped it and a clear fluid came out. I just want to know if this could be something serious and if I should seek more medical advice on it. Doctor: have it checked by a neurosurgeon. it can be your brain fluid or what we call cerebrospinal fluid leaking whuch would explain your headaches. did you have a CT scan made of your head or even just a skull xray? a doctor has to get a sample of the fluid you described and have it tested to rule out cerebrospinal fluid"
},
{
"id": 70880,
"tgt": "How can chest congestion and headache be treated while suffering from an upper respiratory tract infection?",
"src": "Patient: I had a cough associated with allergies, and upper respiratory infection diagnosed two weeks ago. I have had a shot in my hip a week ago (not sure of the name) but am still coughing and headachy - no fever. Still feel some congestion in my chest. Now I have a headache in the front, top and back of my head. Feeling in my head is almost like my blood pressure is up, although I do not have high blood pressure. I take Jentaduet for Type 2 Diabetes, levothyroxine for my thyroid, Fenofibrate and Ros...(statin for cholesterol and high triglycerides (300). Also OTC D3 for diagnosed deficiency. Also RX fish oil - but only one today. Doctor: HiAs you explain the history your concerns might be from the viral situation. Don't worry we usually recommend paracetamol in such cases. Hope I have answered your query. Let me know if I can assist you further."
},
{
"id": 65162,
"tgt": "Suggest remedy for lumps with blood thining",
"src": "Patient: My dad Has been on warfarin for a while and now his physician told him that his blood was extremely thin. His primary care physician has suggested he stop warfarin for a bit and see a hematologist. What worries me is that he has black and blues and 2 lumps. Doctor: Hi,From history it seems that there might be having some petechial hemorrhages due to blood thinning medicine producing these bumps.Blood thinning medicine warferin can not be stopped without taking advice from your treating doctor.He might require PTB ( INR ) testing and accordingly your doctor will advice.Ok and take care."
},
{
"id": 101248,
"tgt": "What are the symptoms of asthma ?",
"src": "Patient: Periodically I get a feeling like I could pass out, shortness of breath sometimes after briskly walking or climbing stairs. Along with that feeling I get a feeling of a slight ache in the upper back and shoulders. I do have asthma. Could this be part of that dx. A year ago I had an upper respiratory infection with pneumonia bronchitis which lasted almost 6 wks before I actually felt normal. Doctor: WELCOME.HERE IS YOUR PLAN. ASTHMA IS A COPD, GET YOUR PFTS DONE VIA A PULMONOLOGY CLINIC. STATE ALL THE MEDICINE THAT U R CURRENTLY USING. STATE WHETHER THE SYMPTOMS COME AT NIGHT N WAKE U UP. THANX"
},
{
"id": 219490,
"tgt": "Does anxiety while travelling have an affect on the 14 weeks pregnancy?",
"src": "Patient: I am 14wks 5days pregnant. We are thinking of traveling, flying specifically, at Christmas. I experience severe anxiety/panic when I fly. Normally I would take xanax or ativan. I spoke with my obgyn about my concerns, and he said he would prescribe ativan. After doing a little research and learning that ativan is a cat D drug, I am not willing to take it while I m pregnant. My concern is any ill effects my anxienty may have on my baby if I just tough it out . I don t HAVE to travel. Are there any reasons I should worry about how my anxiety would affect the baby? Also, my anxiety symptoms are shaking, shallow breathing, racing heart. Thank you! Doctor: Hi.Don't worry, anxiety will not cause any issues to your child, but just try your best to stay as calm as possible. But you made the right decision, both ativan and xanax can have an impact on the health of your fetus.All your symptoms are something that will require some amount of control, and distraction, ask your partner to help.Best wishes."
},
{
"id": 130943,
"tgt": "What causes pain and burning sensation in the groin?",
"src": "Patient: I woke up this morning with a severe pain and burning sensation in my left groin. It has eased slightly but every so often produces a fierce dull pain. The skin feels almost as if it is sunburnt. I have been unwell with nausea and stomach pains for the past 5 days and the doctor is sending me for a test for helicobacter. I doubt this is connected. I have not exercised for over a week due to illness so the pain cannot be a strain. Please advise me, I am 46 year old female with no health issues. I dont smoke but drink fairly often. Doctor: In my opinion you are experiencing ilieofemoral artery thrombosis, i recommend a doplar to check it Good Luck"
},
{
"id": 149011,
"tgt": "Diagnosed with Cordial Basal degeneration, having pneumonia also. Suggestions?",
"src": "Patient: My friend has Cordial Basal Degeneration. I understand there is no way to diagnose accurately this disease until a brain biopsy is done post mortem. I don t know how long she has had CBD, but now she has Pneumonia and I know complications from this can cause death. I need more information, in general. Anything you can offer will be greatly appreciated. Donna Doctor: This is a neurodegenerative disease, and as you know that there is no established treatment for this condition however Pneumonia can be treated by antibiotics and ventilator support but this recurs since brain disease is progressive. Stem cell therapy has shown encouraging results in many neuro degenerative conditions but results are not proven. In this particular condition i have not seen any published report of use of stem cell therapy however i feel that once lung infection is under control you may try and see if stem cell therapy works for your friend if you feel like(but this is purly experimental). You should not keep high hopes in this condition."
},
{
"id": 172951,
"tgt": "Could cold sweat on head, teething and bowel problems be related?",
"src": "Patient: My 4month old is having a cold sweat only on her head....she is also teething and having an issue moving her bowels I have gotten her to have an easier time with the bowel move,ends since I started her on prunes...could this all be related or should I be worried Doctor: HiWelcome to the HCMI completely understand your concerns but don't worry. In infants the body temperature regulation which occurs by hypothalamus in brain is not mature. So, many infants have a tendency to sweat excessively especially over head and neck. This improves gradually with age and do not need any specific therapy.The bowel movements can also vary greatly in frequency, consistency and colour every few days in the same infant. If the child is comfortable, feeding well and growth is good then nothing needs to be done. Only thing is regular feeds. If you are giving formula feeds instead of breastfeeding, then check for proper hygiene and dilution. Many a times, dehydration can be the reason for constipation in infants.It's not advisable to introduce complementary feeds before 6 months of age. So wait for now on any introduction of feeds.Hopefully this will help you.I will be glad to answer any further questions.Take care"
},
{
"id": 102488,
"tgt": "What causes headache when playing football, hotness and night sweats with controlled asthma?",
"src": "Patient: my son is an active football playing 9yr old he recently complains of headaches at practices and his body becomes very hot he also has night sweats he does have asthma but is controlled and he has played three years with asthma just the last two weeks has he ever complained or had these issues Doctor: Hello,Welcome to HCM,Main predisposing factor for bronchial asthma is exposure to dust.As your son is having asthma he should very careful.The symptoms will aggravate on exposure to irritant like dust.As he is having fever, headache and night sweat it might be due to invective agents in the body.Playing football requires lot of energy and lot of fluid will be lost during playing, loss of fluids and electrolytes in sweating may also leads to your sons symptoms.He need to hydrate himself properly during playing and be careful regarding dust induced asthmatic attacks.Thank you."
},
{
"id": 222180,
"tgt": "Can non-penetrative sex cause pregnancy?",
"src": "Patient: i had half sex with my boyfriend on 27th nov but he didnt cum.. but he touched my organ by his pink one... is there is any chances of precum on his pink head..but as a precaution i have taken unwanted 72 within 48 hours.. can i become pregnant. my last period was on 22nd nd ended on 25th nov. i hav nt got my periods yet. is any thing wrong. i am very worried please tell me Doctor: No you can't be pregnant. Don't worry unwanted pill can cause irregular cycle. so chill chances of pregnancy is very rare."
},
{
"id": 115382,
"tgt": "Can low platelets lead to dizziness and sleeplessness?",
"src": "Patient: Hi, I have a history of low platelets and just had bloodwork and they are low again (113 thous/mcl). I also struggle with feeling lightheaded/dizzy and don't sleep much at night. Can there be a correlation? From searches on the internet, I am getting Adenal dysfuction as a possibility. I'd appreciate any insight. Thank you. Doctor: Hi, dearI have gone through your question. I can understand your concern.There is no relation between dizziness and low platelets count. SO don,t worry about that. Moreover your count is slightly low. Nothing to worry.Your dizziness can be due to low hemoglobin or Your blood pressure. You should check your blood pressure and hemoglobin.Consult your doctor and take treament accordingly.Hope I have answered your question, if you have any doubts then contact me at bit.ly/Drsanghvihardik, I will be happy to answer you.Thanks for using health care magic.Wish you a very good health"
},
{
"id": 15639,
"tgt": "Having red sore, itchy spot like bumps on chin jawline. Applied sudo cream which helps temporarily. Cause and cure?",
"src": "Patient: Hi I have red sore itchy spot like bumps on my chin and jawline, the dont seem to be spots as i can not burst them, i have 9 in total from one side to the other and well spread out. I dont normally suffer from spots either. I am 35 Male I have tried sudo crem and it helps for a while but the are very itchy and sore when left alone?? Do you have any ideas to what it may be that is causing this? Doctor: HiThanks for your query.You seem to be developing superficial folliculitis.Avoid using oils or moisturizers over the face.Use a topical antibiotic cream over the affected area twice a day.You can take oral antibiotics for the same also.Do not apply any irritants or dettol over the face.Do not use and steroid creams over the affected areas.Hope it helps.DR Geetika Paul"
},
{
"id": 133299,
"tgt": "What causes arm pain?",
"src": "Patient: I lifted something heavy with my arm partially outstretched, elbow bent, with a curling motion, a month ago. Now my arm hurts slightly below the shoulder all the time and is weak and hurts when I try to reach out and pick something up or as I stretch it above my head. Is this serious enough to seek treatment or will it mend if I favor it? Doctor: hi,thank you for providing the brief history of you.As per your history this looks more of a bicipital tendinitis. Undergoing physical therapy like therapeutic ultrasound therapy and TENS therapy should help reduce pain. Also exercises to strengthen muscle will help avoid further injuries.In my clinical practice most patients respond well to physical therapy.RegardsJay Indravadan Patel"
},
{
"id": 212985,
"tgt": "Under stress, do not like smokers, habit of smokers, overreacted in a certain situation, feeling bad. Advice?",
"src": "Patient: Hi, I m 47 years old single mom of 2 kids; 19 and 17. My eldest in is college and my youngest has a mild autism . I spent all my life providing and guiding my kids. I m under a lot of stress . I try and manage for most parts but sometimes trivial things really bother me and I get upset. I don t like being upset for more than 10 seconds. I wonder if I can eat some mood enhancing food to control my mood even more. Last night I got upset because my gentleman friend left me at a pub for the 3rd time in the night to go out for a smoke . I am not a smoker and dislike the whole package from health perspective to smell and the habits of smokers. He has supposedly quit regular smoking but does social smoking. I tried to brushoff the first and the second time I was left by myself but the 3rd time got to me. I don t know why it got to me so much. I don t know if I was too tired and so I reacted to such trivial thing. I felt like leaving and almost took off. I stopped talking and smiling and sat there like I hated the world. Today I feel bad about my reaction though I was not disrespectful or rude. Maybe I should drop it all together. thanks, Pira Doctor: Hello and welcome to Healthcare Magic. Thanks for your query. East irritability could sometimes be a manifestation of underlying stress or depression. If these symptoms are frequent and distressing or if you are having difficulty handling your stress, I would advise you to consult a psychiatrist for a detailed evaluation and further management. There are effective treatment options - in the form of medication or counselling / psychotherapy which will help you overcome your problems. Wish you all the best. - Dr. Jonas Sundarakumar consultant Pychiatrist"
},
{
"id": 14090,
"tgt": "Why haven't rashes and itching reduced despite taking zostrix shot for shingles?",
"src": "Patient: I have had the zostrex shot for shingles, however, I have an inner itching all around my waist but no rash at all. Could I have gotten the Shingles despite the fact that I was inoculated. The itching has been going on for quite a while but I cannot see anything. Anti itch cream helps a bit. Doctor: Hello and Welcome to \u2018Ask A Doctor\u2019 service. I have reviewed your query and here is my advice.You are suffering from Post herpetic neuralgia. This is a complication of Shingles in which pain/itching lasts even after the healing of the rash. You can take Methylcobalamine and Pregabalin tablets along with Levocetrizine tablets.Hope I have answered your query. Let me know if I can assist you further."
},
{
"id": 12251,
"tgt": "What could psoriasis with swelling and redness in legs suggest?",
"src": "Patient: my husband has bad psoriasis on legs and some on body, dr. gave him prednisone for an infection for 2 weeks, took him off and all of a sudden his legs swelled up and all red, now in am his legs are thinner and during day swelling again and maybe there is fluid in there, the drs. dont know. help me. Doctor: Hello, welcome1. Psoriasis is skin disease in which skin becomes scaly and patchy. Itching is also symptom of this. It is bit complicated disease which takes long treatment course2. I am surprised and bit shocked that doctor is giving you steroid ( prednisolone injections) for psoriasis condition which is totally wrong treatment.3. Due to high doses of injection there is interference of kidney functions and retention of water in lower extremities. kINDLY get the kidney function test done to assess the effect of steroids 4. It is very very common side of steroids. swelling will go off slowly slowly5. I see 4-5 case of psoriasis every day but i never ever give steroids to them. As it has lot of side effects one of them your husband got other than this steroids can cause pustular psoriasis or further worse the psoriasis.6. Stop the injections immediately7. Start methotrexate 5-10mg once in week and folic acid rest of the day. This medicine needs proper guidance like liver function test, kidney funtion test, CBC 8. I very frequently give methotrexate to psoriasis patients and get very very good results."
},
{
"id": 61438,
"tgt": "What causes a lump in the center of the upper abdomen?",
"src": "Patient: 63 y.o. white male. Just noticed a soft lump in the center of my upper abdomen (from my sternum to navel) apx 1.5\" wide x 5\" long. Interestingly, I had laparoscopic gall bladder surgery 7 years ago and there were \"holes\" for the surgery at each end of the current lump. Mild aching pain. Noticed lump when flexing stomach muscles (sitting up in bed). Vegetarian for last 21 years (do eat fish). Constipated often over the last 5 or so years. Better recently by avoiding gluten. Wondering if the lump could be a hernia of some sort or possibly and aneurysm. Doctor: Respected user, hi I evaluated your query thoroughly .* Till your narration it seems epigastric hernia most likely , other cause may be lipoma .* Clinical examination with signs of evaluation & ultrasound of abdomen are confirmatory of the lesion .Hope this clears your query .Welcome for any further assistance .Thanks for using Healthcaremagic.com & giving me an opportunity to assistWishing you fine recovery from the same .Regards dear take care ."
},
{
"id": 100577,
"tgt": "What causes sneezing and wheezing during night time?",
"src": "Patient: Hi,Me and my wife are sufferring from sneezing and slight wheezing during night. We are perfectly alright in the morning. And our child who is 2 year old is having no issue. We start sneezing when we move to bed. Is there any chance that bed may be poluted with dust? And in washroom is having a leakage issue and the wall is slightly turned to black color. Also we use mosquito repellent liquids through out the night.Please help me to get rid of sleepless nights.Thanks & RegardsArun Doctor: Hello.Thank you for asking at HCM.I went through your history and would like to make following suggestions to both of you:1. In my opinion, you are spot on! All the three factors that you have mentioned can cause sneezing and wheezing.2. Most common are - house dust mites, the most important source is bedsheets.3. Indoor molds are another important causes - they origin from leakage and moist areas.4. Mosquito repellent liquids may cause sneezing but not very commonly wheezing.5. I would suggest both of you allergy testing for house dust mites and common indoor molds. Depending upon results, an Allergist-Immunologist may prescribe you allergen specific immunotherapy which specifically works on your allergies.6. I would also suggest you regular montelukast & levocetirizine before going to bed and to use salbutamol inhaler for wheezing on as-and0when-needed basis.7. Please correct the leakage area and reduce humidity in bedroom by ensuring good ventilation & sunlight. I would also suggest you to wash in hot water and dry bedsheets every week.Hope above suggestions will be helpful to you.Should you have any further query, please feel free to ask at HCM.Wish both of you the best of the health.Thank you & Regards."
},
{
"id": 154062,
"tgt": "Can keeping iTouch beneath the pillow cause cancer?",
"src": "Patient: I would like to know if my nightly use of an iTouch beneath my pillow can cause cancer. It helps put me to sleep while I listen to an audio book. I am 68 years old and have had breast cancer surgery a couple of years ago. I am about 155 pounds and am about 5'3\". Doctor: Thanks for your question on Health Care Magic. I can understand your concern. Since you are breast cancer survivor, better not to keep i touch under your pillow. Practically it can not cause any harm in general population. But theoretically, sound and signal waves from the device can cause cancer. Actually these waves can act as stimulator and may cause malignant transformation. Probability of this is high in cancer survivors. So better not to keep such devices under pillow for whole night. Hope I have solved your query. I will be happy to help you further. Wish you good health. Thanks."
},
{
"id": 139882,
"tgt": "Suggest remedy for sharp shooting pains in head",
"src": "Patient: Hi, I ve been having sharp shooting pains in my head for about a month now. It started in my left temple then moved to my right. Then it became simultaneous. The pain is now in my forehead and over my eyes.. My temples and points over my eyes are tender to touch. I ve also recently developed a runny nose and frequent urination, about every 3 minutes I need to go. Doctor: Hello,In my opinion, in your case, an infection of your sinuses (sinusitis) should be considered. For this it is necessary to s'e an ENT Doctor and to have imaging studies of your sinuses done. Treatment depends on the diagnosis.Hope I have answered your query. Let me know if I can assist you further. Regards, Dr. Erion Spaho, Neurologist, Surgical"
},
{
"id": 3068,
"tgt": "What are the chances of pregnancy if having delayed periods after protected sex?",
"src": "Patient: Hi docter , my gf and i had a protected sex , but this month her periods are still delay now its more than 6 days her date was 15 of every month but still she is not getting periods. Last month when we had protected sex i still gave her i pill , wen we had sex she was in her period cycle few days her period was stopped and after she had an i pill the next day then he got bleeding...but this month i had not given her any pill, but her periods are still pending .. please let me know what can i advice her to take that she soon get ths periods we dont want pregnancy to b happn and i also dont want to go to any docter or test please u give me the prescription. Doctor: Hello dearI understand your concernIf you are sure about the protected sex then there is no chance of the pregnancy.Delayed period could be due to hormonal imbalance, ovarian dysfunction, excessive stress, change in life style and dietary habit.Wait for the period to come.If period will delay by more than 14 days then consult the gynecologist for detailed examination.Period can be induced by progesterone pill.Meanwhile avoid stress, take healthy diet, drink plenty of water and do regular exerciseHope this may help youContact further if follow up neededBest regardsDr. Sagar"
},
{
"id": 40822,
"tgt": "How to determine ovulation dates while trying to conceive?",
"src": "Patient: Hi Dr , if my last menses was on the 9th Apr when is my ovulation ? Is actually this way January was on the 20th , Feb on the 18th , March on the 15th and Apr on the 9th . and i just had like cramping , pelvic pain on the lower abdomen and kind of feveris pains which lasted from night of 24th to early of 26th Apr. In this case , what do i call this situation and when was my ovulation suppose to have come in this April? I have been trying to conceive for the past few months and have been having sex with my partner which i did all through my fertility period in this month of Apr before i experienced this 2 days pains. Kindly advice Thanks Judith Doctor: Hi, I think you can track your ovulation by repeated ultrasound or by ovulation detection kits. You can be in contact with your husband every 2 to 3 days after your periods stop. It will cover the ovulation time also and increase chance of pregnancy. Pelvic pain can be a symptom of ovulation, but it is not a definitive sign. Hope I have answered your question."
},
{
"id": 226194,
"tgt": "Tubal ligation done. Suffer from insomnia, headache, tingling in the hands and stress. Have depression. Advice?",
"src": "Patient: I'm 32 women who had a baby Sept 2012 followed by tubal ligation. I have experienced insomnia, upset stomach, low blood pressure, puffy veins arms, headache, tingling in hands. I work out 4 days a week for an hour. I drink one latte early in morning but never experienced this. I feel stressed and lack of oxygen and don't know why. I only work 3 hours a day. I have had mild depression on and off with little hair loss. ??? I have been tested for my thyroid already. Doctor: Hi, Thanks for the query. You once consult physician, go for physical examination and go for complete blood picture. If everything is normal, the symptoms can be possibly due to depression. You once consult your regular psychiatrist and discuss related to this. Try to spend more time with your family members and baby that can elevate the mood and can decrease the symptoms. For more details you can ask me through: http://www.healthcaremagic.com/doctors/dr-sree-gouri-sr/63429 Take care."
},
{
"id": 46266,
"tgt": "Is vomiting a normal symptom having experienced renal failure?",
"src": "Patient: My husband recently was hospitalized due to renal failure. He was there for 2 weeks running all types of tests. He returned home with the doctors still not knowing what exactly was wrong. At this point they ruled out the failure part although his kidneys are not putting out the way they should they are still functioning. He returned to ER 3 days after being released due to vomiting. They stablized it and sent him home again. Now he has no appitite and most times cannot even keep down water. I try not to let him take his medication unless he eats, which he does not, so he is not taking his meds. We went to his doctor yesterday where they determined he was dehydrated so they put him on a IV for 3 hours. We left the hospital stopped for bottled water where he then vomited again. Is there something he can take to maybe coat his stomach to help keep things down? Doctor: Hello, As per your description your husband is suffering from some kidney disease. But it's unclear wether it is Acute Kidney Injury(AKI) or Chronic Kidney disease(CKD). Because AKI is temporary and more serious than CKD. But vomiting is a very common condition in any disease. In kidney diseases, recurrent vomiting are common due to Uremia(increase serum urea). This will also lead to loss of appetite. In your case, you have to find out the extent of kidney damage and cause of vomiting by some tests like USG abdomen pelvis, KFT and ABG with Serum Phosphoric level. Meanwhile better get admitted the patient and start IVF with washout urea if increased. As much as put on more injectables rather than oral. So don't be panic and evaluate the patient properly and treat with injections for few days and get down serum urea level if increased by diuretics or dialysis if needed.Thank you"
},
{
"id": 112777,
"tgt": "Having low back pain, over weight, on pain killers. Now numbness and pain in thigh and back. On medication. Advise",
"src": "Patient: Hi samuel. I have history of low back pain. 40 yr old overweight female. Usually managed well with pain killers and exercise from physio. Now 5 wk history of numbness tingling stabbing pain in left thigh and back pain. Also uncomfortable under left ribs. On garbepentin increased today 300mg tds and codeine phosphate 30mg qds. Can't bare the pain much longer. Referred to back clinic today who will assess IF I need an mri ? Doctor: Hello. Thanks for writing to us. The numbness that you are having is likely to be related to a nerve root compression at the level of the lumbar spine due to degenerative changes in the disc. Methylcobalamin supplements will help you.I hope this information has been both informative and helpful for you. Regards, Dr. Praveen Tayal ,drtayal72@gmail.com"
},
{
"id": 151920,
"tgt": "How to cure short term memory loss ?",
"src": "Patient: i am 22 years old my hight is 5.4 and weight is 67 i had problem of short term memory loss i am 22 years old my hight is 5.4 and weight is 67 i had problem of short term memory loss . whatever i read that i forget after one sleep time is not fixed may be 8 to 9 hours i sleep can u help mi for good doctor in mumbai i am not rich, i am from very small family i need to take a tretment for that convenient cost which is convenience for mi my father is retired plz help mi Doctor: Hello Ram,At your age, you do not have to worry about memory problems. In fact, what you feel as memory problem is actually decreased concentration due to which you are not able to focus properly when you read. This can be because of many reasons like stress, lack of interest or distractions.Here are some things you can do:1. Set short term and long term goals, so that you know why exactly it is important for your to study. This will increase your motivation and also your interest in studies.2. Study with full concentration (without distractions), even if it is only for a short period (1 hour). Reading for hours without focus does not help you in long run.3. Take breaks in between so that your mind does not drift away while studying. You can utilize the break to relax and refocus.4. \"All work and no play makes Jack a dull boy\". Integrate some play or exercise in your routine. Deep breathing exercises will also help calm your nerves.5. Do not compromise on sleep time as sleeping less will affect your concentration.If you can follow these (all self help methods and costs only your time), then it may not be necessary to even consult a doctor. All the best."
},
{
"id": 137730,
"tgt": "Can foot pain and red bruise knee be related?",
"src": "Patient: hi. My legs are bright red from the knee down and both ankles look bruised in a circular formation around them. What can cause this? I also have terrible foot pain. I don t know if that s related. The red area and the bruised areas do not hurt but look terrible. Doctor: Dear Patient,your symptoms looks like infection or allergic lesions, you should get yourself screened by your doctor, and take precautions as advised, I feel once an clear diagnosis is made, than proper treatment can be initiated.thanks"
},
{
"id": 4482,
"tgt": "What all treatments should be done to conceive after having 2 abortions?",
"src": "Patient: Hi Doc,my name is Ria. On December 2012 I had a miscarriage of 1 month and then again I got pregnant in April 2013. But, again after 3 months I had to terminate my baby(as per doctor) because heart beat was not found. I am in dilemma now. I want to get pregnant within this year end. Doctor: HiI am very sorry for your losses. Unlike popular opinion abortions are very common and most women have successful pregnancies after 2 abortions. I would encourage you to follow healthy lifestyle and cut down smoking and alcohol. Also you can get a blood test to see whether you are immune to rubella or not. If not, it is advisable to get vaccinated before pregnancy as catching it during pregnancy can harm the baby. Please also take .4mg of folic acid daily while trying to conceive.Thanks"
},
{
"id": 39318,
"tgt": "What causes itching and burning in anus?",
"src": "Patient: The last 3 days my butthole has been itching and.burning. or almostFeels like, when I touch it, there's little cuts because it Burns but it itchesso bad. Reminds me of a yeast infection. I've also noticed I'm a lot gassier then normal but I haven't had a bm since the day before yesterday. Does this sound like hemorrhoids? Doctor: Hello,Welcome to HCM,The history suggests me that your son may be having fungal infection which should be confirmed by physical examination.The fungus are the normal commensals of the genitals and it can sustain for longer period of time in wet and poor hygiene areas.For the present symptoms I would suggest you to follow1. Keep the area dry and clean2.Oral Terbinafine, 250mg,once a day for 2-4 weeks3.Topical agents like clotrimazole need to be applied daily.Thank you."
},
{
"id": 36985,
"tgt": "What could cause chronic cold and cough, antibiotics offered little relief?",
"src": "Patient: He took antibiotic courses for the same. The cold got cured but cough started. When the cough prolonged for a long time, he was prescribed...He took antibiotic courses for the same. The cold got cured but cough started. When the cough prolonged for a long time, he was prescribed...He took antibiotic courses for the same. The cold got cured but cough started. When the cough prolonged for a long time, he was prescribed...He took antibiotic courses for the same. The cold got cured but cough started. When the cough prolonged for a long time, he was prescribed...He took antibiotic courses for the same. The cold got cured but cough started. When the cough prolonged for a long time, he was prescribed... Doctor: HIThanks for posting your query to Healthcaremagic. See persistent cough inspite of using antibiotics is due to 3 reasons1. Improper Diagnosis : It might not be infection but allergic cough.2. Improper Selection of antibiotics :3. Improper dosage or Improper duration of treatment . SO first we need to know whether infection or allergy is the cause of Chronic cough. For that you need to get these test done and revert back with reports. Complete Hemogram , Absolute Eosinophil Count , Serum IgE levels , Sputum AFB stain, Sputum for Culture Sensitivity test. Also let me know what Antibiotics was used , in what dosage and for what duration. Will be looking forward for your reply. Regards."
},
{
"id": 86023,
"tgt": "Should the ER be visited for extreme abdominal pain along with hot flashes?",
"src": "Patient: I just suffered extreme pain in abdominal area, near waist band & belly button, along with such a hot feeling and sweating for about 10+ minutes. Had hard time walking to phone and sofa. Lying down didn't help. I had just eaten half a medium size bowl of spicy and leftover chili, mushrooms and veggies and suddenly felt I couldn't eat another bite, then the pain and massive sweating occurred and I almost never sweat much. I felt extremely hot and felt I had to remove clothing but couldn't do it as moving hurt. Pain overtook much movement. Might this have been appendicitis or pancreatic necrosis? My father did of pancreatic cancer st 88 y.o. I was able to move with difficulty to sofa and then to stairs to call my husband for help. He tested for stroke by asking me to smile and move facial muscles. I passed and am now online seeking answer. Shall I cal PCP or go to ER if episode occurs again? I still feel heaviness in abdominal area and discomfort in stomach but it's not the same as earlier pain. I had had a fine BM this morning. Doctor: Hello and Welcome to \u2018Ask A Doctor\u2019 service. I have reviewed your query and here is my advice. As per my clinical experience, the issue is related to acute episode of severe gastritis following spices, the exacerbation of the symptoms are related to vagal response from the parasympathetic pathway. I do not recommend an ER visit at all, as the condition is quite settled at present, obviously have to be careful next time, before consuming the same spices. Wish you fine recovery. I would like to assist you further in this regard."
},
{
"id": 69720,
"tgt": "What causes thickness on the rib and lump in the diaphragm with a history of breast cancer?",
"src": "Patient: I had breast cancer in the left breast and had radiation since then (2010) I have this nagging pulse and slight thickening on the right side under my right rib. I am over weight, I feel a hard lump in diagphram did ultra sounds, and ct it is not my liver or gall bladder. Sometimes it stiffens and tightens Doctor: Hi.If the CT and ultrasound is normal this lumpish feeling may be just an inflammation of the underlying ribs or muscles.That is why it stiffens and tightens .Take an anti-inflammatory and you will be fine."
},
{
"id": 159036,
"tgt": "Small smooth white spots ringed in red on the roof of my mouth, suggested for biopsy., family history of oral cancer. Suggestion?",
"src": "Patient: I have 5 small smooth white spots ringed in red on the roof of my mouth. They started out as bumps that you could not see but you could feel. On the 13th of march they will do a biopsy and I wanted to see what this might be. I am scared cause my mom had stage 4 oral cancer at 47 and my sister had stage 1 oral cancer at 37, I am a female age 28 Doctor: Hi, Oral cavity cancer can be related to chewing tobacco , so this addiction history is very important. Multiple oral cavity lesion not always signify cancer, there are other benign causes and infections also. As your doctor has suggested a biopsy there may be some suspicion. Anyway, nothing to be worried right now, every disease has treatment. Follow your doctors advise."
},
{
"id": 33713,
"tgt": "Suggest treatment for kidney infection",
"src": "Patient: My seven year old grand daughter has been running a high temperature for five days. She has a knot below her knee and the doctor say it is cellulitis. An ER person wrapped it with an ace bandage and put a half cast that is to be removed at night. She was prescribed bactrium antibiotics for a kidney infection. Does this seem like an accurate diagnosis? Doctor: Hello, I had gone through your question carefully and i can understand your concern very well for your grand daughter. I would like to probe you few questions like is there any signs of inflammation like redness, swelling, etc., is it associated with pain or not, whether any kind of discharge from that knot or not. If you can upload an image of that knot, then it will be greater helpful for me to give you perfect suggestions to you. You can upload an image and come back to me directly anytime, i will be happy to guide you further. I would suggest you to do some blood examinations like CBC with ESR, creatinine and blood sugar. As there is high grade fever for last five days, antibiotics are must for empirical treatment and along with that paracetamol should also be taken to take care of fever. Hope this will help you and come back to me with the image and reports, so that i can guide you fully in right direction."
},
{
"id": 157296,
"tgt": "PSA tested, high reading. Is it due to steroids and clomid had before test? No prostate problem. Father had prostate cancer. Is it an indication I may have cancer or an enlarged prostate?",
"src": "Patient: hi Im 38 years old. I just had my psa tested it was 4.34. I used anabolic steroids and finished 3 weeks before the blood test. I was also using clomid up to 2 days before the blood test. I ve had no prostate related problems what so ever. urine flow is perfect. Is it possible that the high reading is due to the steroids or clomid? Is it possible the reading may be different in a month or so? I m very concerned as my dad had prostate cancer . Is this an indication I may have cancer or an enlarged prostate as I know steroids can do that. Are my levels really high? My dr said it was double what it should be! Doctor: clomifene does not affect PSA but yes anabolic steroids does .it is better to have a physical examination DRE to know whether your prostrate is enlarged as u have a family history. With a PSA level of 4-10 ng/mL, the likelihood of finding prostate cancer is about 25%.but only PSA is not reliable and as you said you had steroids prior to test.get Sare done and then we can think of retesting or biopsy if needed"
},
{
"id": 29809,
"tgt": "What causes recurrent fungal infection despite taking Dazit and Terbinaforce?",
"src": "Patient: Hello Doctors.I have a fungal infection near my groin.I have skin specialist Tab and lotion used for 21 days said by doctor Dazit tab Terbenaforce tab Lulican lotion Skin changed to normal After completion of course again started Itching and swelling,Please give me a favourable advice.I have been used many Onitements tablets gels creams lotion Doctor: groin infections are more likely if your overweight,or if your wearing undergarments which not washed properly.i would suggest you wash your clothes with hot water and dry them in sunlight..aand when you go to bed wash groin with warm water par them dry and apply mild moisturizer and do the same in the morning also..."
},
{
"id": 153878,
"tgt": "What causes acute lymphoblastic leukemia?",
"src": "Patient: Hello, I had been diagnosed with A.L.L. in Feb. 2003 my age was 27 years then. I completed treatment for 3 years and my CBC and Peripheral Blood Smear reports are normal except for low Platelet counts. What could be the reason and what are the chances of relapse. I completed treatment in Apr. 2006 and thereafter I am having no medicines. Doctor: Hi, dearI have gone through your question. I can understand your concern.You have ALL. You took chemotherapy treatment for that. Now you have normal CBC And peripheral smear report. OS no need to worry. Your low platelet count may be due to some other cause like viral infection or non specific cause. No need to worry. Just be relaxed. If you have any signs of bleeding then consult your doctor urgently. Otherwise no treatment is required. Chances of relapse are always there. you should monitor your complete blood count regularly.Hope I have answered your question, if you have any doubts then contact me at bit.ly/Drsanghvihardik, I will be happy to answer you.Thanks for using health care magic.Wish you a very good health."
},
{
"id": 159479,
"tgt": "Hard lymph node after having a cold. Could this be lymphoma?",
"src": "Patient: Hy my name is Edna and I have a small lymp node not moveble and kind of hard...I have it since February, in February I had a bad cold with losing my voice for a couple of days and then everything was fine until know last month I had again a bad cold with losing my voice again so after I recovered my voice I had a bad cough and felt again may lymp node I don t think it change it size at all now I am worried that it can be something serious like lymphoma Doctor: Hi Edina, it looks like infection induced to me. But hard node for one month worries me. I think you must undergo FNAC of the node, in which doctor puts a needle and get the fluid. This after microscopic examination can help to decide or rule out cancer. Please don't wait as its a simple test. Take care."
},
{
"id": 129436,
"tgt": "How to treat the weakness caused by vitamin d deficiency?",
"src": "Patient: Hello Doctor, My Mother-In-Law who is aged around 45 years is suffering from severe weakness from many years[almost 15 years] Now it has been severe and she is unable to walk properly and perform her daily duties. Around 2 years back we have consulted one Rheumatologist and he performed some clinical tests and concluded that she has vitamin d deficiency.He gave some medicines and she used it for only some months and again the situation is normal. Now again i want to show her to some specialist.can you please give me advice to which specialist i have to show her. Thanks, Sreenath Reddy C. Doctor: Hi. I would advise a re-visit to a Rheumatologist as this deficiency may be linked to some auto-immune disease and the Rheumatologist are qualified to deal with those."
},
{
"id": 131864,
"tgt": "Suggest treatment for pain and swelling in neck after possible muscle strain",
"src": "Patient: yesterday was carrying a child and turned my head to the side to shield them from a sneeze and it felt like i ripped something by my adams apple. I assumed it was a strain of some sort and didnt think anything of it but it hurt to swallow for the rest of the night. today when i woke up i felt swollen in my neck area right at the adams apple and i cant swallow without a sharp pain as the saliva passed by my adams apple. no brusing, no red in throat, to problem breathing, talking seems a little deeper voice. its not sore throat type of pain but like a muscle strain. went to urgent care place and they looked at me like i was crazy. said i should get a CAT scan to rule out coronary artery damage! this happened a while back after i sneezed but never this much pain swallowing. Doctor: hi going by your details , In my opinion I think that you have probably strained your neck muscles on one side leading to all the symptoms.I suggest pain killers , gargles , and steam to heal the internal soft tissues. It might take 7-10 days for healing ."
},
{
"id": 91401,
"tgt": "Why do I have severe abdomen pain around my navel along with retaining fluid?",
"src": "Patient: paracentesis/liverSince my paracentesis from abdomen which three & one half liters of fluid was removed along with the radiologist still leaving two other pockets of fluid in abdomen & now I am experiencing severe abdomen pain around my naval along with retaining fluid again... I don't currently have medical insurance so my care and/or treatment isn't quite up to par either... What shouls I do now? Doctor: HI. Once the fluid is removed , the sensitive peritoneal layer is exposed and so becomes painful around the naval area as this is the first to be emptied. The most important part is why are you having so much of fluid into the abdomen. IS it liver cirrhosis or something else. The basic cause has to be diagnosed and treated."
},
{
"id": 169305,
"tgt": "Suggest remedy for severe vomiting, diarrhea and fever in an infant",
"src": "Patient: Hi doctor, My baby is 10 months old. For past 2days severe vomiting, loose stools with foul smell and slight temperature. Doc has advised me to give the following medicine.kindly advice I am giving right dosage. Still the baby is suffering from above said problems. Taxim o 1 tablet 3 times daily, loparet 1/2 tablet 3 times daily, econorm 1 sachet 3 times daily.for loose stools. Emeset 3.5ml 3 times daily for vomiting. P 125 5ml for fever. She weighs 10kg. And she eats only solid food. Arrow root water is given in between. Milk simyl mct. Pleawe advise Doctor: Hi Dear,Welcome to HCM.Understanding your concern. As per your query your baby have symptoms of severe vomiting, diarrhea and fever which is due to dehydration which seems to be due to food poisoning and acute gastroenteritis. It could be due to disturbed metabolism of body leading to electrolyte imbalance. Need not to worry. I would suggest you to take acetaminophen and anti emetic medication. Take diet rich in multivitamins and avoid any oily sharp and fast food. Avoid intake of any outside food. Visit pediatrician once and get it examined. You should get blood tests done to find out underlying cause of condition and start treatment accordingly. Your symptoms will resolve in 3-5 days. You should take proper rest. You should take plenty of fluids to re-hydrate.Hope your concern has been resolved.Get Well Soon.Best Wishes,Dr. Harry Maheshwari"
},
{
"id": 187833,
"tgt": "What could cause swelling and sweet metallic taste post oro-antral fistula?",
"src": "Patient: After wisdom teeth extraction, developed oro-antral fistula, opening between mouth and sinus. My oral surgeon closed the opening with stitches and put me on antibiotics. Now I have a sweet metallic taste in mouth, blood comes occasionally when I spit and a little swelling in the area. Doctor: Dear user,Thanks for using healthcare magic.It is normal things,do not worry about all, because metallic taste can be due to using antibiotics and some blood also comes due to forcefully split and using hot water,food etc.swelling also occur due to injury throw the instrument.so eat soft food and keep continue your antibiotics.you will feel better.thanks."
},
{
"id": 52535,
"tgt": "Does almond and English walnut normalise uric acid?",
"src": "Patient: hi my age is 38female i have heelpain in my right leg my sugarpoint is140infasting i take everyday 2almond and 1 english wallnut in everymorning is good for uricacid or notmy weight 60 kg my hight is 143 inches also suffering from irregular menstrualproblem thanx doctor Doctor: Hello and Welcome to \u2018Ask A Doctor\u2019 service. I have reviewed your query and here is my advice. Almond and walnut doesn't have any proven effect on uric acid or kidney function. The antioxidants and essential oils contained in them may have some good effects on overall effects but clinical evidence are lacking. Do frequent followup tests in case of high values, consult a physician and get evaluated. Hope I have answered your query. Let me know if I can assist you further."
},
{
"id": 96788,
"tgt": "Is Freon inhalation a matter of concern?",
"src": "Patient: I was driving my car when it was leaking Freon from the a/c. By the time I got home I had been smelling it about 2 hours total. That was on Tuesday (2 days ago) and the sick feeling is still with me, but not as bad as Tues. and Wed. and I have swollen glans today. Do I need to be concerned? Doctor: Hi. certain degree of such difficulties can be caused by freon inhalation but it wont cause any serious or permanent consequances. Regards"
},
{
"id": 57600,
"tgt": "Is on and off pain in right abdomen and near the ribs related to distended gallbladder ?",
"src": "Patient: Hi, i had an ultrasound yesterday and results mentioned well distended gallbladder with no gallstones seen. I'm also having a pain in my right abdomen and feel the pain on and off in by back on the right side near my ribs. Is this serious ?Report also mentioned i had lots of bowel gas and scan was difficult.Thanks. Doctor: Hi and welcome to HCM. Thanks for the query.it can be related to gallbaldder even if there is no galbaldder stone in it. there can be polyp or motility disfunction and you should do HIDA scan. Also it can be realted to your liver of bowel. Most common bowel disrders are IBD but inflammatory bowel disease should be ruled out too. Wish you good health. Regards"
},
{
"id": 92892,
"tgt": "Have severe abdominal pain and lower back pain. Pap smear and urinalysis normal. Suggest?",
"src": "Patient: I am 49 years old and am waking up in the morning with severe abdominal cramps and lower back pain. Approximately 30 minutes after I get out of bed, all the pain goes away for the rest of the day. I have been to the gynecologist and my pap smear and urinalysis were normal. Also, I had a sonogram to check my ovaries for any signs of cancer and that was normal. The pain has been getting increasingly worse and wakes me up about 5:00 in the morning. Doctor: Hi, it appears to be the signs of hydronephrosis, due to subclinical urieral stone , or a stone in the kidney. The other cause being not passing urine while going to bed and it causes pressure over the kidney. Hypertension also can cause the pressure over the kidney. I advise you to consult a urologist for diagnosis and treatment. Thank you."
},
{
"id": 191937,
"tgt": "What causes vision problem and tiredness despite having normal blood sugar levels?",
"src": "Patient: I have all the signs of diabetis. I went for blood work for a thyroid issue and Dr said it was finally normal but y blood glucose was high so wanted me to get tested again after fasting and it was 100 today but I have all the symptoms. Been dropping things, tired, vision whorse, irritable, brain fog. Doctor: Classic Symptoms of diabetes are increased appetite, increased thirst , increased urination frequency & weight loss. Describe ur symptoms in detail and post ur test results."
},
{
"id": 53643,
"tgt": "What causes elevated liver enzymes and water retention?",
"src": "Patient: My annual lab tests showed an elevate BUN, and elevated Liver enzymes. Should I be concerned? Last year the BUN was elevated but not as much and now the liver enzymes are elevated. I take daily glucophage, aldactone and was taking a fat burner for awhile. I am 32 years old, female, 5'3\" and 123 pounds. I have been retaining water and its getting out of control. I recently gained 15 pounds over the last 3 months. I exercise 5-6 times per week 60-90 mins of cardio each time. My T3 was LOW however my T4 WNL and my TSH on the low end of normal. I was diagnosed with insulin resistance after the birth of my second child in 2003. Any advice or insight to this issue? Doctor: Hi, You should take both your raised BUN and liver enzymes seriously and try to bring them down with effective Diabetes control, fat regulation, and medication for both. Please consult your Diabetologist/ Endocrinologist for this. Regards"
},
{
"id": 224979,
"tgt": "Is it normal to have abdominal pain and back pain after taking microgynon pill?",
"src": "Patient: Hi I've been taking microgynon pill for 10years approx, this is the first time I've ran my pill 3pkts in a row, half way through 2nd packet I got a little bleeding but I know that Is normalHowever I'm little more than half way through my 3rd pkt and for past few days I have had abdominal, lower back pain, like period pain but a little more (I don't suffer from period pain etc) feels like cramping then eases off then comes back again , this has been for past 2days but I have no bleeding. Is this a side effect and would I be better just stopping the pill and having a 7dsy break? Doctor: Hello ma'am and welcome.Yes these symptoms are common due to a hormonal change. But if you are finding it difficult to cope with these symptoms, then please report to your doctor for a change in prescription.Hope this helps.Best wishes."
},
{
"id": 153233,
"tgt": "Does loose stools/squelching noise around bowel indicate sign of colon cancer?",
"src": "Patient: Hi, I have had lose stools for many years. I am 31. I dont know if it has always been like that but I have noticed my stools are not very round. More oval or flatter. There is lots of squelshing noise around my bowel the last week and lose stools (no blood) sometimes when I eat a big or greasy meal I must rush to the toilet. Are these sypmptoms of colon cancer? As I am not old and no family history. Thanks for your help. Doctor: Hello,Loose stools and undesirable noise during stools are unlikely to be from cancer. It is possible that you have irritable bowel syndrome or a condition like inflammatory bowel disease. If there is no blood in the stool, then the risk of cancer is low.It is suggested that you consult a gastroenterologist and give a detailed history of your problems and get examined clinically. It might require doing a colonoscopy on you. It is a simple office procedure in which a slender tube-like device with a camera at one end is inserted after bowel preparation and anesthesia and the large bowel is visualized. Any suspicious disease areas will be evaluated and it will provide adequate information on the cause for your loose stools. Please do not worry.Hope I have answered your query. Let me know if I can assist you further.Regards,Dr. Vivek Chail"
},
{
"id": 54655,
"tgt": "What is the treatment for fatty liver and high ALT and AST levels?",
"src": "Patient: I am a patient 41 old years with non alchoholic fatty liver , high ALT& AST, high in level of insulin serum .I used below regimen drug : cap ursobile (urso deoxy cholic acid ) , tab methformin & vit E. please guide me. with warmest regards mohamad Doctor: Hi welcome to HCM...You have NAFL...So here you are taking correct drugs.Continue it esp.ursodeoxycholic acid which dissolve cholesterol in bile....Cutt off oil in diet.Regular one hour exercise needed.In morning take grined carrot and spinach juices.Green leafy veg salad and fruits taken more.Refined food avoided.Junk food and non veg very minimal...Use less oil in cooking....Sunflower , canola , pea nut like good oil can be useful....Take care.Hope your concern solved.Dr.Parth"
},
{
"id": 126512,
"tgt": "How can a pulled muscle in the chest area be treated?",
"src": "Patient: Hello, thank you. Well, I was previously diagnosed with Costochondritas (sp?) very painful ribs on my left side, a deep radiating pain. It would go away after a couple months but come back every once in a while. Next Dr. said it wasn t costochondritas but just pulled muscles. Well, it keeps coming back now every few months and now it is the middle of my chest bone. It feels like it was cracked wide open when ever I take a deep breath. It is also still radiating on the left rib cage. I got to the point where I was taking so much over the counter pain med s I developed a small ulcer. Help! Doctor: Hi, You can consult a physician and get an MRI scan to look for any muscle tear or nerve involvement. As of now, you can take analgesics like Gabapentin for pain relief. Hope I have answered your query. Let me know if I can assist you further. Regards, Dr. Shinas Hussain, General & Family Physician"
},
{
"id": 32987,
"tgt": "Suggest remedy for bumps with yellow discharge due to insect bites",
"src": "Patient: hi there are a few unknown insect bites on both of my legs and developed into bumps and extremely itchy in the middle of the bites, there are some yellow discharge. after a few days, it will become circle at the insect bite area. the discharge is quite bothering me what can i do with it? and do you have any idea what kind of insect bite it is? Doctor: Hi..Welcome to HEALTHCARE MAGIC..I have gone through your query and can understand your concerns..As per your complain it seems that you have infection in the insect bite areas leading to abscess formation and yellow discharge is pus..You should take antibiotic treatment oral as well as topical for faster relief..You can take Augmentin 625 mg three times a day,Levoetrizine 10mg once a day [preferrably at night] and Pantoprazole 40mg once a day early morning, you have to take the medicine for 5 days..Apply Triple Antibiotic Ointment over the abscesses by first draining the pus, cleaning with betadine and then apply the ointment twice daily..It will start resolving in twoo to three days and after 5 days you can keep applying the ointment even after oral medicines are finished till the complete healing occurs..If there is no recovery consult a physician..Hope this information helps..If you find the answer helpful please give me a Five Star Review and click on found the answer helpful as a token of appreciation..Thanks and regards..Dr.Honey Nandwani Arora.."
},
{
"id": 133672,
"tgt": "What causes sharp pain in the calf and ankle and tremors?",
"src": "Patient: I am waking up with a sharp pain in my calf and/or ankle (it s like a cramp) and then I shake violently. I am totally conscious and alert so I know it s not a seizure..No doctor can dx it because I cannot reproduce it. My health dxs are kidney failure stage 4, diabetes, hypothyroidism, anemia....Any ideas? Doctor: hithank you for providing the brief history of you.A thorough musculoskeletal assessment and neuromuscular assessment is advised.As you have the medical history of kidney failure stage 4, diabetes and hypothyroidism the reaction of tremor you are facing is because of the combination of the above. Due to the above contions there is a disruption in the neuromuscular channelling and leading to tremor, twitching, fatigue, cramps etc. The pathology is deep to understand and the nerve gets disruption in the signalling leading to the symptoms.Undergoing physical therapy for the same will help you correct the neuromuscular imbalance.In my clinical practice I see cases with such complains and they all respond well to physical therapy.RegardsJay Indravadan Patel"
},
{
"id": 31841,
"tgt": "Suggest remedy for infection in belly button",
"src": "Patient: i had a skin infection took antibiodics had a follow up where i was told i was clear of all infection but now my belly button stinks very bad and I clean regularly because of the stench but i wont go away and its potent and its embarassing how do i get rid of the stink? Doctor: Hi, Thanks for posting in HCM. I understand your concern. Stinking after getting treated for skin infection of belly button can be overcome by maintaining good hygiene. Kindly clean it everyday with mild antiseptic solution like Savlon and using warm water. Apply Clotrimazole with Neomycin cream at bedtime to prevent fungal and bacterial infection. Do not apply deodorant sprays or steroid containing creams. Gradually, the stinking will subside completely. Hope the information provided would be helpful. All the best."
},
{
"id": 100728,
"tgt": "Suggest treatment for nasal polyps in an asthma patient",
"src": "Patient: d/doctor, I ma a asthma patient from last 20 year, I also got operated for nosal polyps one year year before, again nosal polyps reoccured, at present I am taking under mentioned medicine please guid me what medicine i can reduce or add. 1.foramide 400 inhaller 00----00 twice dailly 2.odimont LC at night 3.eropylline-100 bd 4.medrol (steroids) 8 mg. BD 5.CAPSULE RALLICIP D, OD EMTY STOMACH 6.SYRUP BRICAREX 2 TSF BD 7.MUCINAC 600 Mg. at breakfast 1 OD Doctor: HIWell come to HCMAfter the nasal polyp surgery the polyp occurrence is not likely, in my opinion you are on poly drug therapy and this is some time causes more problem than the cure if I would be treating doctor the I would treat this case with Saline water nasal drops, with Tab Loratidine with Pseudoephedrine three times in day, with advise of steam inhalation, hope this helps."
},
{
"id": 10299,
"tgt": "Suggest treatment for severe hair loss",
"src": "Patient: Hi. I m 32 years female. Getting severe hair fall, specially while applying oil. I hav lost most hair from d front. I m not yet married so getting worried more..i already did all d tests n everything is normal. Whn I go with medicines it goes well but whn I stop it, it started agn. So I stop taking medicines a long back. I work in night shifts. Plz help Doctor: Hello and Welcome to \u2018Ask A Doctor\u2019 service. I have reviewed your query and here is my advice. It is very important to find out what is the cause of your hair fall: diet/lifestyle/hormonal changes/dandruff/hereditary/weather changes/poor products usage to name a few. Hair oiling is mostly just a conditioner and may not be enough for your hair fall. I suggest you start applying topical Minoxidil 5% once at night over your scalp, which will stimulate new growth. Also start taking multivitamins with biotin in them. Increase your nutritional quotient in your diet. Hope I have answered your query. Let me know if I can assist you further. Regards, Dr. Smruti Pevekar"
},
{
"id": 101996,
"tgt": "How long does fatigue and nausea last after an asthma attack?",
"src": "Patient: i have had a major asthma attack that almost sent me to the hospital for, but after a 3rd breathing treatment at my pediatrician i started to get better, but now i feel extremely weak, slightly nauseous, and fatigued. How long does this typically last? Should I return to school tomorrow or take a day to rest. Doctor: Hi,sorry to hear about your health. After an asthma attack it is common to get fatigue. This may be because while you were breathless your body had overworked to fight it. Also again asthma and congestion can cause lack of sleep. Yes all this can contribute to fatigue. Do take rest until you feel better. Stay home,eat good food and rest. Allow your body to recover. Take care"
},
{
"id": 120707,
"tgt": "What causes pressure like pain while laughing?",
"src": "Patient: I get a significant pressure/tightening type pain when I laugh. Just now started. Not typically a smoker, will have a couple cigarettes if I have had a few extra drinks on the weekends or something. Have had a bit of a dry cough for a few days, but nothing persistent. Have never had this before. What could it be? Doctor: Hello,I read carefully your query and understand your concern. Muscle pains throughout the\u00a0face\u00a0and jaw are often caused by Temporomandibular Joint Dysfunction.It should be evaluated by an orthopedist. Meanwhile,I suggest using anti inflammatory medications such as Acetaminophen to relieve the pain. Hope my answer was helpful.If you have further queries feel free to contact me again.Kind regards! Dr.Dorina Gurabardhi General &Family Physician"
},
{
"id": 113720,
"tgt": "Lower back pain, bloated stomach, shortness of breath. Pneumonia ruled out after chest x-ray. Respiratory infection?",
"src": "Patient: Hi, I have had lower back pain/inflammation for about 8 weeks, whilst this has eased somewhat and the muscles are less tight there is still a pull if I arch my back. More worrying is my stomach seems to fill up with air when I exert my body -like walking up a hill or stairs and I find difficullty in getting a deep breath/or shortness of breath. Been to a GP, sending me for xrays for chest and back, she thinks I have a respiratory infection of some sort, but has ruled out pluercy or pneumonia . Would this and or the back problen be causing my shortness of breath? Doctor: Hi Ray, Welcome to healthcare magic! Backache is not known to cause shortness of breath or any breathing problem. I think you are not a habitual walker, if you are new to start on exercises then you will have this kind of exertion related breathlessness. Do not walk uphill in the start. Start walking at plains first and walk about 4-5 kilometers a day, later when you build stamina, start having jogging. Avoid stairs and uphill running. They are not good exercises rather put more load on body. I hope this advise would be helpful for you. Still if you have any queries or doubts, please write me back, I would be happy to solve them. Wish you a great health!"
},
{
"id": 25934,
"tgt": "What causes nape pain?",
"src": "Patient: Hi, I just want to ask why is my left side of the nape hurting? the cartilage part of my nape is the one in pain. I checked my BP last night, and it was 140/90. my normal bp is just 120/80. I'm 32 yrs old. is this just ok? Hope you can help. Thanks. Doctor: Hi, dearI have gone through your question. I can understand your concern. Your blood pressure is high. You should take salt restricted diet and measure your blood pressure again. If it comes high for 2 to 3 times then antihypertensive medicine like losartan or atenolol should be taken. Consult your doctor and take treatment accordingly. Hope I have answered your question, if you have doubt then I will be happy to answer. Thanks for using health care magic. Wish you a very good health."
},
{
"id": 167555,
"tgt": "Suggest remedy for eating problem in 7 month old",
"src": "Patient: Our 7 month old babu refuses to eat anything but breastmilk. He weighed 6.2kg at 3-4months but now weighs 5.5kg. We have tried all sorts of things, milk, cereals, but he just doesn t seem to like them. We are worried about his weight. Could it be worms, teething, as some have suggested to us? Thank you. Doctor: Hi...At his age interest in play increases and interest in food decreases. This is quite natural. If he is active and playing around you need not worry. This age rather than the quantity of the food I suggest you concentrate more on the number of times you feed her and also make whatever you feed him calorie dense by adding good amount of ghee to it.PLEASE DO NOT GIVE ANY SO CALLED APPETITE STIMULANTS AS THEY ARE NOT VERY SCIENTIFICALLY PROVEN AND THEY HAVE THE SIDE EFFECT OF EXCESSIVE DROWSINESS.Hope my answer was helpful for you. I am happy to help any time. Further clarifications and consultations on Health care magic are welcome. If you do not have any clarifications, you can close the discussion and rate the answer. Wish your kid good health.Dr. Sumanth MBBS., DCH., DNB (Paed).,"
},
{
"id": 166642,
"tgt": "What causes fever after a vaccination?",
"src": "Patient: my son is 15months old, and got his 15month vaccines on the 19th of March. He s had a fever Monday, Tuesday, Wednesday. Fine on Thursday, Friday, Saturday. His fever came back on Sunday 102.3 and On Monday his fever shot up to 105.1. He also has cold symptoms. His bloodwork shows some abnormal results. Elevated WBC, RBC, Low MCH, Monocytes are also elevated. Neutrophils is also elevated. Chest xray negative. Ears, throat .. negative. Please help. Doctor: Dear parent , I understand your concerns but it seems that your child has developed a mild allergic reaction towards vaccination . allergic reaction requires the administration of an antihistaminic as citrizine once before going to bed until the fever subsides. to relieve the fever you can use acetaminophen or ibuprofen syrup every 8 hours"
},
{
"id": 160287,
"tgt": "MY MING NOT WORK, I AM CHEWING GUTKHA, PLEASE HELP",
"src": "Patient: I AM QUIT GUTKHA BUT MY MING NOT WORK I AM CHAWING GUTKHA Doctor: You can try leaving it gradually. That is reducing the quantitly slowly, so it will cause less discomfort . For ex, having it 5 times a day reduce it o 3 times, for a week. and so on."
},
{
"id": 143551,
"tgt": "How serious is a corpus callosum lipoma?",
"src": "Patient: Hi, am 23 years old and i am diagnosed with corpus callosum lipoma. i suffer often from headaches, (thou i do wear glasses and maybe this is the reason) but besides this there arent any other symptomes .. as much as i know. So can you please tell me how severe this is, and if any other side effects appear in future. Thank you very much Doctor: Hi, corpus callosum lipoma is benign tumor in the centre of brain.Mostly it is asymptomatic or presents with seizures.Generally surgery is not required .It is not severe and side effects dont appear.Thanks"
},
{
"id": 33621,
"tgt": "What is the treatment for a dog bite?",
"src": "Patient: i was bitten by a house dog on my leg yesturday.i went to de doc.and he gave me the initial treatment and send back to home.today i have developed allergic reaction and noticed some rashes(kind of little pimples) on my back and forhead.is this something i should worry about? Doctor: Hello . Thanks for writing to HCM.I read and understand your question very well. I try my best to explain you.As you have dog bite on leg and you should get a initial treatment. I suggest you to take anti rabies vaccine for this. Aslo do regular dressing on leg. I suggest to take Tab. Cetrizine for your allergic reaction.I advice you to consult a good surgeon for this and take full treatment for this.I hope my advice will help you a lot.Thanks and Regards.Dr Vijay"
},
{
"id": 206352,
"tgt": "Suggest remedy for low self esteem",
"src": "Patient: i am having a problem with friend so i have been very stressed and there hating me they been swearing at me and the teacher dont do anything about it and everyone laughs and push me around at the lockers and now i have a black eye and now i eat by the dumsters all alone what should i do!!! :( :( :( : : ( Doctor: You are having mild depression. Take tab escitalopram and tab fluoxetine. You will more confident. Go for counselling sessions. Hope my answer helped you. Take care"
},
{
"id": 201638,
"tgt": "What causes penis pain during an intercourse?",
"src": "Patient: Hi my name is Logan and my question is why does my penis have a slight pain when inserting my penis into my girlfriends vagina? The pain goes away as soon as my penis is all the way in her, so it's not a constant pain. The pain usually occurs if we have sex a second time within a short period of time. Doctor: Hi,Thanks for writing in.If your pain is from only inserting your penis, you may want to try using a lubricant, which will make your intimacy more comfortable and pleasurable for the both of you. A lack of lubrication can pull the skin of the penis in uncomfortable ways during sexual activity during inserting and cause pain. Also indulge in good amount of foreplay to allow adequate arousal and emotional connectivity with your partner. The fact that the pain occurs during a second attempt at having sex might be due to decreased lubrication and more friction between penis and vagina."
},
{
"id": 155493,
"tgt": "What causes excessive belching after a biliary stent insertion?",
"src": "Patient: mom just had a biliary stent for pancreatic cancer stage 2. No mets but the distal common bile duct was obstructed. She is healthy except for thyroid disease (Hashimotos). She has had excessive belching causing a lot of pain in her belly prior to being able to release the air. What is causing the belching? Doctor: A lot of belching means that the pancreatic cancer, besides obstructing the distal common bile duct has also started to obstruct the duodenum (the part of the small intestine that drains the stomach). This can be ascertained by an upper GI endoscopy. Also, if it continues or progresses to vomiting, she may require a duodenal stent. At the same time, i Hope she is undergoing some form of anti-cancer treatment for her disease since pancreatic cancer stage 2 is potentially curable."
},
{
"id": 22231,
"tgt": "Suggest treatment for ventricular tachycardia",
"src": "Patient: My father had a heart attack and did not know it until weeks later when his beats were so off, he went to emergency room. He was in vtack and in trauma unit within minutes. They put a stent in on right bottom side of heart. He was in ICU after that for about a week with a few episodes of the vtach agian. Hes home now but his heart is constantly acting up and the doctor keeps telling him its normal. Hes not so convinced. Should he get a second opinion? And is this so normal that they wouldnt ask him to come in for testing? He has made 10 phone calls and he gets same answer over and over agian. Please help! Doctor: hello,The increase in heart could be due to multiple reasons, it may be normal or abnormal. Ventricular tachycardia is usually a life threatening condition and patients will be very symptomatic and frequently collapses. Now, best thing to clear doubts is get a holter monitoring done, it will record the ecg for 24-48 hrs and we will come to know the state of heart during his symptoms so that we can modify the treatment."
},
{
"id": 97805,
"tgt": "Feel tingling over the whole body, dizziness. Difficult breathing. Home remedies for muscle cramps?",
"src": "Patient: tingling in head full body tingling dizziness full body muscle tightness diffuculty breathing i cant take full breathe in hardly with out having to stress and i jus tried meth(ice)for the first time its been about an 1 hr the fullybody tightness and the trouble breathing are what im worried about if u can give me sum tips or home remedies for muscle cramps like can i eat or drink sumthin or buy sumthin frm the store without going to see doc or do sumthin to make it stop and if u got any for my breathing problem that wld be awesome! Doctor: hi dear and welcome to HCM...Dear here is a home made remedy which really work....mix a tablespoon of vineagar with a teaspoon of honey works in less than half hour....take it once or twice aday...."
},
{
"id": 128059,
"tgt": "What causes recurrent pain in the calf and thighs despite taking Ibuprofen?",
"src": "Patient: I developed a chest cold starting last Thursday or Friday. I may have a mild fever, but nothing serious. What s really bothering me, is that the muscles in my thighs and calves are super painful since Sunday night. Have taken ibuprofen and acetominophen for some relief, and taken hot baths and showers. Also massagingmy legs, doing some gentle yoga, raising legs with help of some wedges. These offer some relief, but the pain comes back again. Doctor: Hello I have studied your case.Fever with thigh pain suggests possibility of viral illness.You may need to investigate in form of complete blood count, ESR, CRP.You may need clinical examination also to confirm diagnosis.MRI may help to diagnose any structural abnormality.Till time you can start medication and antibiotic consulting your treating doctor.Hope this answers your query. If you have additional questions or follow up queries then please do not hesitate in writing to us. I will be happy to answer your queries. Wishing you good health.Take care."
},
{
"id": 29617,
"tgt": "What causes frequent UTI and pain during intercourse?",
"src": "Patient: I have very frequent uti and my Husband and I are trying to conceive so we are having intercourse more frequently. I know this could be the cause of the uti but lately sex is pain full and there is a lot of discomfort with Oder just in the urin. And when I pee I feel like I'm never done what is wrong Doctor: Hello dearWelcome to Healthcaremagic.comI have gone through your concern in depth .@ Pain may be associated with or without urinary infection as inadequate relaxation of vagina , anxiety induced spasms @ Causes of frequent UTI during intercourse are - transmission of bacteria as E.coli and others - use of certain irritating feminine products as powders or sprays - retaining urine long time after intercourse - organisms of herpes , gonorrhea , chlamydia , mycoplasma etc - improper hygiene of private parts - inadequate water intake in much hurry of conceiving - others Hope this will help you definitely .Feel free to ask any further queries in future anytime .Thanks for so much peaceful evaluation of my answer .Regards dear take care ."
},
{
"id": 30911,
"tgt": "How to cure prolonged fever?",
"src": "Patient: I am having a fever for 15 days, the temperature is not to high in the range of 100.F. I had switch 200(2x 2 days) as well as Milivo 500 (1x 7 days), but still the fever is coming. I am using crocin to deal with it. I have done Maleria Parasite test but nothing came up. Can you help me? Doctor: That is a long time to have a fever. It is difficult to help you as I need more history. I suggest you see a physician as soon as possible."
},
{
"id": 100864,
"tgt": "Suggest treatment for breathing problem and dizziness due to severe asthma",
"src": "Patient: I\"m suffering from sinus that causes me a sever asuthma. Know i\"m under medications, I get monthly injuctions \"Xzulair\" but still i get time to time difficulty in breathing and dizziness. I heard that there is a cure in india for this kind of sinkness. I would be appreciated if you could help me in this matter. yous sincerely, Masooma Faridoon Doctor: Hello Masooma Faridoon,Thank you for asking at HCM.I went through your history and would like to make following suggestions to you:1. Sinus and asthma are many times linked and sinus can aggravate asthma. So It is important for you to control both sinus and asthma.2. Were I treating you, I would suggest you daily montelukast and an antihistamine like cetirizine or fexofenadine for upper airway disease (rhinitis and sinus).I would also suggest you regular steam inhalation (please be cautious to avoid burns) and daily nasal irrigation with saline sprays which will also help.3. For asthma, I would suggest you inhalers like budesonide-formoterol for regular use and salbutamol/levosalbutamol inhalers for as-and-when-needed use for asthma symptoms.4. I would suggest you allergy testing which will help you identifying the substances causing troubles to you. Avoidance of the substances causing allergies is the best strategy for managing allergies.Based on allergy testing, an Allergist-Immunolgist may prescribe you immunotherapy which will work on your immune system to gradually improve your allergy symptoms.5. Regular breathing exercises and a diet rich in vitamins & minerals (adequate amounts of green leafy vegetables, fruits, etc) will also improve your lung capacity and immunity respectively gradually.6. Please avoid exposure to dusts, smokes, fumes and air pollution as much as possible.Regarding treatment of sinus/asthma in India, there are many allergists in India.Also, Ayurvedic treatments are available in many parts of India which can be very helpful asthma, along with conventional medications.Hope above suggestions will be helpful to you.Should you have any further query, please feel free to ask at HCM.Wish you the best of the health.Thank you & Regards."
},
{
"id": 28857,
"tgt": "How can pneumonia be treated?",
"src": "Patient: The hospital for pneumonia 4 days ago w antibiotic steroids and nebulizer treatments today s the 5th day and I feel sicker I have a fever losing my voice and my chest hurts I was wondering if I should go back to the hospital it seems like I m getting worse Doctor: Hello,Specific treatments depend on the type and severity of your pneumonia, your age, and your overall health.The options includes:-Antibiotics-Cough medicines-Nebulizer-Antipyretics and analgesics-RestDuration of treatment may vary depending upon your condition and immunity. As your fever is still there, so infectivity is still strong, and you need a longer duration of antibiotic treatment along with other support measures.In case the symptoms persist, you can see your nearby primary care physician.Hope I have answered your query. Let me know if I can assist you further.Regards,Dr. Prabhash Verma"
},
{
"id": 127766,
"tgt": "What causes sore throat/fluid filled pouch on collar bone when having chronic sinusitis/RA?",
"src": "Patient: Hi had a bilateral fezz done last April for chronic sinusitis, have had nine antibiotics since as they can t clear infection. Now told I need another operation to try and sort out this infection. Over the last month have noticed one side of my throat is sore on swallowing and I have a fluid filled pouch on my collar bone. I suffer from rheumatoid arthritis which is active at the moment and wonder if this fluid is connected to that or to the sinus infection. Many thanks Doctor: Hi,Have you had an x-ray of that collar bone? If it is RA flaring, you may have to take Prednisone and the swelling will subside. Also, if there is an abscess (suspected) then an ultrasound can help. Hope I have answered your query. Let me know if I can assist you further.Regards,Dr. Priya Sadanandan"
},
{
"id": 156575,
"tgt": "What are the chances of developing cancer due to smoking?",
"src": "Patient: Hi my name is Chris I am a 44 year old male have smoked 1.5 pks a day for approx 30 years . I had a chest xray followed by a ct and a 7mm non calsified ground glass nodule was found in my right upper lung . I have never had any form of cancer. What would my percentage of presumed malignancy be Doctor: The percentage of developing lung cancer due to smoking is 24.4% for male \u201cheavy smokers\u201d defined as smoking more than 5 cigarettes per day (18.5% for women) people and 15.9% for current male smokers and 9.5% for women smokers.RegardsDR DE"
},
{
"id": 28439,
"tgt": "Suggest remedy for dizziness and low BP",
"src": "Patient: hi there im 59yrs old female and im on norvasc 5mg per day. since past 3 days i been having heavy headed n light diziness , i did monitor my b/p on my home b/p monitor it shows 118/65 to 110/ 60 it been within this range . shall i consult my doctor or stop my norvasc ? thank you. Doctor: Hello madam..It is one of the common side effects due to novac(amlodipine)Your BP is in the normal range.What was your previous BP?Observe for one or two more days..if the problem persists or if the problem is severe approach your doctor.He may change drug or prescribe you a lower dose...Hope my answer helps you..Good day.."
},
{
"id": 168267,
"tgt": "What causes gagging and retching in a child?",
"src": "Patient: My son is 6 months, gagging on solids, even smoothest pure, though yoghurt desserts and milk are gong down ok, last night he vomited too, but no vomit so far today, just gaging and wretching. he has a cold, is drooling, and has just got his font two teeth through. he is very tored and kranky. do you think he has a sore throat? Doctor: Hello,I can understand your concern. When the teeth are about to erupt in oral cavity, the gums become little sore and kids tend to put things in mouth frequently to soothe the sore gums. This tendency to put every odd thing in mouth, makes kids at this age prone to stomach infections. As he is gagging, retching and had vomited, it might be the possibility of him associated with teething. In such case, I would recommend you to visit a pediatrician to see if he requires a course of antibiotics.In addition, the drooling of saliva because of excessive salivation due to eruption of new teeth in mouth. It will stop in some days as the kid will get used to new teeth in oral cavity. In addition, the sore gums and irritation due to erupting teeth can make a child cranky and he may scream a lot. This will also subside soon. If the gums seem too sore, you can apply Orajel to his gums to soothe them.I hope this information helps you. Thank you for choosing HCM. Take care.Best,Dr. Viraj Shah"
},
{
"id": 107573,
"tgt": "Could back pain and breathlessness be related?",
"src": "Patient: I was diagnosed with psuedocholinesterase allergy to muscle relaxant when having section during child birth. I took Tramadol for back pain a year ago ,and found myself passing out and struggling to breath. Could this be related ? I am now told I have heart condition and my heart must have some trauma, these are the only incidences I can think of ? Doctor: Hi there.You could be allergic to opioids like Tramadol. Avoid taking any opioids like Codeine, Tramadol, Morphin etc. Consult an Immunologist and Allergy specialist to confirm this."
},
{
"id": 122254,
"tgt": "What causes periodical severe foot pain?",
"src": "Patient: I have periodical severe foot pain (occurring around every 10-12 weeks.) It is on top of the foot toward the top of the instep. There is a slight reddening and very slight puffiness. I drink mainly only at weekends but am working away from home and just for the last two evenings have had 1.5 pints of real ale. (Not my normal chice.) Historically the pain goes as quick as it comes within 4-5 days. I am 57 years old, 1.90 metres tall and weigh around 101 kg. Doctor: Hello. Thanks for writing to us. The periodic swelling and pain in the foot can be related to posture like flat foot or due to mild inflammation like arthritis in the area. This can be related to gout also. You need to get your serum uric acid levels evaluated. I hope this information has been both informative and helpful for you. Regards, Dr. Praveen Tayal . For future query, you can directly approach me through my profile URL http://bit.ly/Dr-Praveen-Tayal"
},
{
"id": 32972,
"tgt": "Should I do anything else to prevent infection from burn on forearm?",
"src": "Patient: So I made a really stupid decision 2 days ago and I let someone brand me... The entire burn is a little less than 1 inch by 1 inch and it is on my forearm like 3 inches from my wrist. The burn is a light yellow color and the skin surrounding it is a little red. The area is kind of swollen but it doesn t really hurt that bad when I put pressure on it. Twice a day I clean it with soap and water and then clean it out with hydrogen peroxide, then I put this hospital antiseptic solution (povidone iodine) on it, which is like a film, and then finally cover it with a band aid. Is there any way to tell if it is infected? Should I be doing anything else to prevent infection? I can t go to a doctor so anything I can do at home would be great. Any estimate on how long it will take to heal? Thanks. Doctor: I understand your concern.If your lesion gets painful, yellowish discharge coming from it, surrounding area becomes red then this are the signs of infection. Currently you are keeping good hygiene of your wound. My advice is don't wash your wound with Hydrogen peroxide as it is usually used to remove dirt and dust from the injured wound and not in burn cases. Secondly apply some antibiotic+antifungal ointment like Betnovate C on the wound. Do not apply bandage on the burn area, keep it open after applying antiseptic cream.Take tab Aceclofenac twice daily for 5 days for pain, tab famotidine twice daily for 5 days."
},
{
"id": 11511,
"tgt": "What causes dark spots and hyper pigmentaion on the cheeks?",
"src": "Patient: i am using Depiwhite cream and calamine lotion since 10 month but spots didnt go. What i do sir ? i have dark spots and hyper pigmentaion on the cheeks . Iam using Depiwhite in the night and Kojiwit lotion before bath and Markal lotion after bath. What I do use? Doctor: Hello,Welcome to healthcare magic.The hyperpigmentation on your face as described by you could be due to melasma.I would recommend you to use a face wash twice a day ideally a one containing glycolic acid as it helps in exfoliation and to even out the skin tone.The most important product will be a sunscreen. Half an hour before going out apply a sunscreen and reapply it every three to four hours.Continue the creams that you have been using.You could consider chemical peels like glycolic acid peels and retinol peels. You can get these done by a dermatologist.Have healthy diet consisting of anti oxidants ( fruits and vegetables).Hope this helps you.Take care."
},
{
"id": 87304,
"tgt": "Suggest treatment for lower abdomen pain",
"src": "Patient: i have been getting a sharp pain in my lower right abdo and i dont know if it is appendictis, as i have read on symptoms online yesterday i had a moment when i had to walk dodgey as i had that pain when i walked and then it went and when i went to eat in the resturant i had a feeling of neausea and then it went after a couple of hours when i go to the toilet for a stool, i can feel that pain when my bowels satrt to move Doctor: HelloPain in lower abdomen may be due to many reasons like colitis,appendicitis calculus,etc.You need clinical correlation and investigations like routine hemogram,urine RE/ME and Ultrasound of abdomen.Ultrasound of abdomen is most important in your case.It can exclude many possibilities.Proper treatment depend upon findings.You may need antispasmoidic and antibiotics.Get well soon.Take CareDr.Indu Bhushan"
},
{
"id": 4017,
"tgt": "Are the conception chances good?",
"src": "Patient: hi doc,my egg size on 12th day ws 32*27mm,30*23mm, 25*20 mm, et in mm ws 9.6mmand free fluid -nt...on 14th day first egg ruptured ,other ws 36*27mm and again the other one ruptured,et in mm was 11.7mm and free fuid was +nt....plz tell me how can i know wether i stand any chance of concievingthis time?? and should me and my husband shud have sex as its my18th day today,so that i may concieve....thanx! Doctor: Hi dear and thanks for your query.All the parameters of the egg ,endometer and free fluid are appropriate that you conceive in this month.Take folic acid and have intercourse .All the best"
},
{
"id": 121006,
"tgt": "Suggest treatment for tremor in fingers",
"src": "Patient: My father had the problem of tremor in fingers and limbs and doctors say it is Parkinson s disease. He was given Syndopa and Syndopa plus apart from other medicines. He then consulted another Neurologist after 6 months and the Doctor advised him to take Syndopa plus, Parkin and Rasalect. Within a week, he had problems of Hallucinations and then he again consulted the doctor. Doctor reduced the dose of SYndopa and increased Parkin 2 mg for 3 times (1/2 tab each time). Later, he got very much difficulty in his mouth and it appears it is psychological. He is having very tough time from his mouth - most of the time, he keeps on washing his mouth and every week his case of mouth kept on changing. For a week, there are two sets of teeth, he has got cages in his teeth, he keeps on cleaning his mouth, he also says there is something like glycerine, sometimes he feels if there is no muscle and skin on his face etc. possibly this problem has emerged as Side effect of medicine. He has left all the medicines and not taking any at the moment since last one month. THough eventually doctor advised him to take Parkin. What should we do? Please suggest. Doctor: Hello,The tremor that you have can be related to anxiety, hormonal disturbance, electrolyte disturbance, low levels of vitamin B12 or a side effect of medicines you are taking. Detailed investigations are needed to find the cause. Hope I have answered your question. Let me know if I can assist you further. Regards, Dr. Praveen Tayal, Orthopedic Surgeon"
},
{
"id": 120058,
"tgt": "Suggest remedy for tietze",
"src": "Patient: I was diagnosed with tietze last year, which was severe. It has since come back again this month. The main thing this time is I find it very hard to breathe - like an elephant sitting on my chest, is this normal or something to see the doctor about? thank you Doctor: Hello, Get your CT scan of thorax done to confirm your diagnosis. give hot fomentation, take over the counter painkillers ,take proper vitamin D supplements and apply any diclofenac gel. Hope I have answered your query. Let me know if I can assist you further. Take care Regards, Dr. Jaideep Gaver"
},
{
"id": 194425,
"tgt": "What causes blood nodules on entire scrotum causing itchiness?",
"src": "Patient: I am 59 years old and am quite healthy. Since the last 20 years I have had small blood nodules on my entire scrotem and though it does not cause any pain or other issues, from time to time some of them burst, mainly after a hot shower. I would then have to put some gauze there till the blood flow stops. This would get embarissing if it happened outside. Sometimes the area is very itchy too.My GP says there is no cure for this and asks me to apply Celestone cream. Can something be done about it? Doctor: Hi,Multiple lesions on scrotum are angiokeratoma. Your GP does not know. The condition is totally curable. Consult the dermatologist. Each lesion may be removed by electrocautery under local anaesthesia. The skin will become normal.Hope this helps.Regards.Dr.Ilyas Patel,Dermatologist"
},
{
"id": 138185,
"tgt": "Suggest treatment for sore temple and jaw",
"src": "Patient: Hello, my name is Lyn, I am 54 yrs old, 230lbs. Last night I noticed my temple and jaw very sore, I thought from eating taco s. I woke up this morning with upper jaw and lower jaw sore and my neck tender. Is this a sign of something I should b worried about. Doctor: Hello Lyn,Thank you for your inquiry. I think there be a couple of possibilities here.Sometimes your jaw joint ( also called the temporo-mandibular joint) can become irritated - usually after excessive chewing. This will usually settle down with simple remedies such as applying a cold pack every 2 hours and taking an over-the-counter medication like tylenol.If you develop any other symptoms such as fever, swelling, sore throat, headache or vision problems I suggest calling your primary care doctor.I hope this helps. Best wishes and I hope you get better soon.Adrian Rawlinson MD"
},
{
"id": 220966,
"tgt": "Can car accident cause any problem during pregnancy?",
"src": "Patient: Hi, may I answer your health queries right now ? Please type your query here...I had a car accident about 3 hours ago. I'm 15 weeks pregnant and rear ended the driver in front of me. No air bags deployed. I was wearing my seatbelt and didn't feel much of a jar since we were moving about 10 mph. Should I see a dr? Doctor: Hi, Thanks for the query through healthcare magic. I understand your concern. Though you had n accident, without without air bags protection, The best part of the story is you were traveling at a slow speed of 10 & you had safety seat belts on.You also did not feel any jerk. You also do not mention about any uterine contractions/ pain/ leaking or bleeding Pregnancy seems to be safe.Still it's better to have a pregnancy check up/ consult your treating doctor for internal examination/ USG scan .. to make sure about safety of baby. Thanks.."
},
{
"id": 225834,
"tgt": "Had a Depo after treated on Etopic pregnancy. Have Fibroid causing a heavy bleeding. Is it normal?",
"src": "Patient: I am 35 yrs old and I had a depo after being treated on etopic pregnancy. The shot was on 12 dec and i thought by now 24 March the shot will not be effective. I have fribrod and bled for almost 3 wks in Jan and my period have been irregular in March I have been bleedin since 9th it stopped on 22nd and today I started spotting. Is it normal? Doctor: Hi thanks for the query.This bleeding can be due to fibroid or due to progesterone withdrawal can take either haemostatics or next dose of depo to control this bleeding. Also, you should get an ultrasonography done to find the exact situation of fibroid or some other pathology.Hope this information is helpful and informative to you.If you have further queries you can directly reach out to me on the following URL:http://doctor.healthcaremagic.com/doctors/dr-deepti-goyal/65111"
},
{
"id": 201819,
"tgt": "What causes low testosterone?",
"src": "Patient: My husband has low testosterone, and has been given medicine to help with his levels. He's been using androgel for 2 yrs, and his levels are not improving like they should. His average levels are 130. What could be causing his low levels? He is 34 yrs old, and weighs 220 Doctor: HiAt his body weight there is also a possibility that his body might be converting his own testosterone to estradiol. And this could be the reason why his t levels ate not rising despite treatment. Please get his estradiol E2 levels checked and if they are high he should be started on drugs called aromatade inhibitors which should help improve testosterone levels.Another option would be to have a depot dose of testosterone once every 6-12 weeks for better and sustained results rather than the gel."
},
{
"id": 47275,
"tgt": "Any suggestion for having bilateral renal parenchymal disease, grade II?",
"src": "Patient: hi my mother is 57 yrs old, she has a diagnose bilateral renal parenchymal disease, grade II, minimal ascites and unremarkable sonogram of the urinary bladder, what can she do about this diagnose can you give me advice regarding this matter thank you. Doctor: HelloThanks for query .Your mother has been detected to have grade ll kidney disease .You have not mentioned about her Serum Creatinine levels and whether she is diabetic or not in your query..As regards planning her management I would suggest her 1) Have diet with less proteins and rich in vegetables.2) Less salt intake as for as possible avoid table salt 3) Water intake to be 3 liters /day 4) Avoid red meat and eggs 5) She can have fish 6) Monitor Serum Creatinine levels frequently7)Proper control of diabetic and blood pressure with medicines advised by your physician .If followed religiously she should have not to be worried .Dr.Patil."
},
{
"id": 155914,
"tgt": "Can bump at back of neck be symptom of cancer?",
"src": "Patient: my husband has this \"bump\" at the back of his neck, he told it was a fat globule, so he squeezed and we observed this white (no pus) secretion coming out. However it grows every week. and he squeezes it. He went to see the doctor and he was told this was a fat globule. However, im worried as he had Hodgkin Lymphoma 10 years ago....... He has been 'cancer free\" from these 10 years but again I am worried about this bump Doctor: Hi, dearI have gone through your question. I can understand your concern. Your symptoms suggest mostly it may be lipoma. However you should go for fine needle aspiration cytology once from that bump to know exactly. Then take treatment accordingly. Hope I have answered your question, if you have doubt then I will be happy to answer. Thanks for using health care magic. Wish you a very good health."
},
{
"id": 226311,
"tgt": "Was on birth control for long. Got bleeding during intercourse. Due to birth control?",
"src": "Patient: My husband and I went off birth control 2 months ago after being on for 7 years. We have not been using protecting but I did get my period last month a week early and this month after 19 days (counting from when the last cycle started) I started to bleed during intercourse but it did not last the entire day. Then a few days later we had sex again and I had a little bleeding after we were done is this something to worry about or is it just my body getting used to being off the pill? (I m 25) Doctor: Hello It is very common after using the birth control for years, to have altered menstrual bleeding on stopping its use, This is because the hormonal effects take time to wear off. However, bleeding after intercourse is definitely not normal. It could point to some local lesion on the cervix I would advise you to go for a check up for post coital bleeding. Take care."
},
{
"id": 11033,
"tgt": "Suggest treatment for baldness on the head",
"src": "Patient: Sir, I am just 34 yrs and and my face is also too young but due to the frontal baldness my personality is effected and i am also taking Hairbless tablet and taking nizoral shampoo for hairs. i have some hairs on my frontal upper portion but they are too thick kindly suggest me how to strong those hairs and come out of this baldness. Doctor: Hello,Thank you for posting on HCM.I appreciate your concern regarding hair fall. Hair fall is usually ascribed to multiple factors like diet and nutrition, hormones, stress, cosmetic products etc.From your description, it seems you might be having Male pattern hair loss or Androgenteic alopecia, which is due to male hormone, testosterone. If i were your dermatologist/trichologist, i would like to take through history including family history, history of recent major trauma/illness/stress/medications etc, and recommend some basic investigations like CBC,blood sugar, Thyroid function test.I would suggest you a course of oral tablets containing biotin and other essential vitamins and minerals for minimum 3-6 months. Also, would put you on solution containing 10% minoxidil once a day and a hair serum containing peptides for hair growth at night.(Q sera etc). I would advise you use of gentle shampoo and conditioner on regular basis and use of coconut oil twice a week.Avoid combing in wet hair and let them dry by wrapping in towel. Avoid blow dryers and hair-color/dyes. Take plenty of fresh fruits and vegetables in your diet and try to de-stress your routine life.Hope your queries are resolved and wish you best of health.Thank youDr Hardik Pitroda"
},
{
"id": 46340,
"tgt": "What causes kidney infection and blockage in heart?",
"src": "Patient: Gud evening doctor my father age is 51 years . He is having glucose 120.1mg, Urea nitrogen 42.2mg, Creatinine 3.5mg, Sodium 130.6 mg, Chloride 89.8mg, Calcium 8.3mg, and Uric acid 9.0mg. According to the doctors he cant go through angiography due to kidney infection as they r saying, because of which they are in doubts that he might be having blokeage also in heart . He is right now taking medicine prescribed by the doctors of Jaslok hospital, Mumbai. I m not satisfied with them so, could you please let me know the dieses with which he is suffering and wht further step should i take further... Doctor: Hi and welcome to HCM. As an Urologist, i can understand your anxiety.Your father is suffering from CKD or chronic kidney disease or failure.He also has high uric acid levels. All these need urgent medical treatment.If they're wanting to do angiography,the kidney function has to be normal.But as the kidney function is lower than normal,it can't be done.A kidney biopsy will be planned to know,what's the reason for the failure.He'll need to undergo maintainence dialysis or a kidney transplantation.Any other doubts can be cleared,by sending a direct question,in my name.Dr.Matthew J. Mangat."
},
{
"id": 67829,
"tgt": "What causes pea sized lump on the gum with half face swollen?",
"src": "Patient: HI. I got toothache 2 days ago and yesterday, half of my face is swollen. today is a lil bit better but i found out a pea sized lump on my gum in the incisors, where my toothcahe occurs. its still painful but not as irritating as before cos its tolerable. What should I do and what could it be? Doctor: Hi,From the description you give, you may have a dental abscess. This will need antibiotics and pain relief. I suggest you go to the ED for assessment. In the meantime, paracetamol and ibuprofen will help any pain and swelling. Regards,Dr K A Pottinger"
},
{
"id": 181938,
"tgt": "What does a knot below jaw line indicate?",
"src": "Patient: About 3 months ago I had a bad toothache (my left side back tooth) I went to the dentist because I.wanted them to pull it but they wanted to do a root canal and then cap it. I did not have the money for all of that so they gave me antibiotics and pain medicine. I finished the antibiotics and my tooth quit hurting so I never went back to the dentist. About 3 weeks later I got a knot right below my jaw line it's about the size of a quarter and is very sensitive to touch. Is this an infection or something else? Doctor: Thanks for your query, I have gone through your query.The knot you are describing can be because of the tooth infection resulting in the formation of a cyst or osteomyelitis. So consult a oral physician and get a radiograph done to confirm the diagnosis. But get the tooth treated as early as possible. Once the infection is treated the lump or knot will automatically reduce.I hope my answer will help you, take care."
},
{
"id": 56633,
"tgt": "How to lower my SGOT level?",
"src": "Patient: hi there.i was diagnosed with sgot 103 and sgpt 168.it was down from a high of 203 last month but sgot was up from 83.i need to lower down this figures within 1 week.what could be the best thing to do?please help,i feel a lot of pressure right now.tnx in advnce. Doctor: Dear Friend, greetings from HCM. I understand your concern. You need to fund out the reason for high SGOT and SGPT levels. It could be some inflammation and as the inflammation of liver subsides even the liver enzymes also come down. You do not take alcohol, avoid more of carbohydrates. Take more of green leafy vegetables, fruits . do walk daily for an hour.You will be alright . Thank you"
},
{
"id": 196247,
"tgt": "Are cold flashes possible in males?",
"src": "Patient: Usually once a month for the last ten years, i will wake up in the morning and start shaking very badly. I shake bad enough that I can hardly walk, and cannot keep sugar for my coffee on a spoon. I am 57 years old, 5'11\", 195 lbs. Are there such a thing as cold flashes like hot flashes in males? Doctor: Hello and thanks for your question.It's my first priority to help you.Flashes can be associated with andropause due to low testosterone levels.And you should check your serum testosterone levels.Even your blood pressure and blood sugar levels should be checked.Hope this helps."
},
{
"id": 90403,
"tgt": "What causes light pain on the lower abdomen after eating food?",
"src": "Patient: Hello, Dr. Siegel, I am a female of 60. For about 4-5 months I have been experiencing the coming and going dull pain, or in better words fullness and discomfort, at the left side above the waist line. Exactly at the side, and covering a small area in front and a little at the back. mostly after a meal. Would you please response at YYYY@YYYY ?Thank you very much. Doctor: HelloAs you mentioned that this pain is from last 4-5 months , typically after meal ( mostly ) , this type of pain may be due to these reasons , these includes.1 Hyper acidity or gastritis or Hiatus hernia or Peptic ulcer disease , as all these are nearly same , so need not to worry about name . Diagnosis can be confirmed by upper G I endoscopy. Try raising the head of your bed about 4 inches with blocks. It might also help to avoid eating or drinking for 2 hours before you lie down. To help control stomach acid one should not drink alcohol or drinks with caffeine in them or eat chocolates or spicy or greasy foods. Also take some antacid but if pain is severe you may need drugs like proton pump inhibitors ;like pantaloc-D once or twice in day.2 As you mentioned that pain is near waist line and also radiating in the back , so this may be due to renal calculus . Diagnosis can be confirmed by ultrasound of both kidneys .3 Amoebic colitis is the another most common cause of this type of pain . Diagnosis can be confirmed by physical examination by a physician . Stool examination also find entamoeba histolica .In ,my opinion consult a physician and get his opinion as above mentioned above.Good luck."
},
{
"id": 61228,
"tgt": "What causes lumps on the upper arm and shoulder?",
"src": "Patient: My son has a lump on his outside rt upper arm near shoulder. It s about small egg size. It causes no pain or discomfort even when palpitated. Thinking it was muscle related we went to a PT, they suggested an orthopedic MD look at it first. Other than a simple lipoma what could this be? Doctor: Hello dear , hiThanks for using Healthcaremagic.comI have evaluated your query thoroughly .* Other than lipoma it can be neurofibroma or other soft tissue lesion .Hope this will help you for sure .Wishing you fine recovery .Regards ."
},
{
"id": 220001,
"tgt": "Does spotting and abdominal cramps indicate pregnancy?",
"src": "Patient: i got off of my pills about four months ago and now my periods are about 30 days away now, well today i got a slight period which is very little, they look like they seem to be stopping, and brown almost and i have slight cramps. could i be pregnant? or is that just a stupid question? Doctor: Just to know , how long did you take the pills. How were your periods while on the pills. periods can be skipped some time after pills are discontinued. discuss with your gynecologist. for now get blood test for pegnancy done which will give answer to your question."
},
{
"id": 114045,
"tgt": "What are the best exercises for core strength ?",
"src": "Patient: besides sit ups, what are the mussels and exercises you need to do to really build core strength? Doctor: Thanks for the query Looks like you are having a nerve irritation but it wont explain why exactly u are having the pain only while masterbating and not during intercourse. I suggest u use another method like a pillow from next time. If u have no problem getting the erection then u have little to worry Have a nice living"
},
{
"id": 164724,
"tgt": "What causes delayed milestones in a child?",
"src": "Patient: Dear doctor.. My son is 2 and half years old.. since from birth he couldnt suck enough the mother milk.. means in all activities very slow.. later till now he is taking spoon feed only.. no solid food.. we tried a lot.. but negative response... he is fearing to take any solid food in his mouth.. if we feed even a grape, immediately he tends to vomiting.. And also he is not able to walk fluently.. he didnt crawl at all.. straight way he started walking at the age of 1 and half. He got up himself from the bed after his 2 nd year.. before we used to lift him.. if we check him, we feel his head is weighing more.. that is why he is not able to balance.. even when he runs, he cant stop immediately.. cant balance.. falling down.. We bought a tri cycle.. till now he is not able to pedal it.. just sitting on it.. What will be the basic problem with him? pls help me.. Doctor: There are many causes of delayed development ranging from not crying at birth to many congenital ( birth defects) and genetic issues. You need to get him evaluated as soon as possible as child brain grow maximum till 3 years age so early therapy/ treatment is necessary for optimal response. He will require clinical evaluation and may be neuroimaging."
},
{
"id": 175676,
"tgt": "What is the treatment for constant falling on ground head first?",
"src": "Patient: my 4 year old son falls on their head and hit hard to ground or floor and says it's paining?During his 4 year he would have fallen for 20 times and a bump his head. He us to watch tv and play ps3 games.Two days before again he fell down on the ground but bump but it pain till yesterday. Heed help. Doctor: Hi...Thank you for consulting in Health Care magic. I understand your concern. First be reassured that your kid is normal. I will suggest you danger signs of head injury -1. Vomiting continuously2. Seizures3. Watery of bloody discharge from ears and nose4. Unconsciousness5. Altered sensorium or behaviour. If none of them are present, I don't think you should worry about his trivial head injury. For pain you can give oral Paracetamol in the dose of 15mg/kg/dose(max ceiling dose 500mg) every 4-6th hourly. Hope my answer was helpful for you. I am happy to help any time. Further clarifications and consultations on Health care magic are welcome. If you do not have any clarifications, you can close the discussion and rate the answer. Wish your kid good health.Dr. Sumanth MBBS., DCH., DNB (Paed).,"
},
{
"id": 88778,
"tgt": "What is the treatment for Umbilical hernia?",
"src": "Patient: I have a belly button hernia and some dropsy form pulling a little heater up out of a suckion box. As an elderly lady - I get no rest. Is there something i can help hold my stomach up which would help keep the dropsy from hurting my creases in both legs? Doctor: Hi..Umbilical hernia which gone into pain should be operated as this is indicative to have further complications of strangulation and need of an extensive surgery in the future. The truss and all such devices can cause more complication than that caused by using these. So do not try trusses or so. The dropsy too can be corrected by a surgery."
},
{
"id": 164364,
"tgt": "What causes popping in the shoulders of a 11 month old?",
"src": "Patient: Hi. My daughter is 11 months old and we have been noticing for several weeks now that when we pick her up her back and shoulders pop and it has, of course, brought us to worry. Is there an explanation for this or should we seek medical attention? Thanks Doctor: Hello and welcome to healthcare magicThe popping (clunking/ticking/crepitus) sound is the sound produced when ligaments around the joints are squeezed which causes the fluid within the joint to bubble and that popping sound is produced by popping of that bubble.As long as it doesn't hurt swelling or causes redness around the joint it shouldn't be of concern.Thank you for using healthcare magic.Good luck"
},
{
"id": 6910,
"tgt": "I am having high ana level and pre cancer cells in cervix. Will it be possible for me and conceive and have a healthy baby ?",
"src": "Patient: Hi, my name is Amy. i have a very high ana level of 1:2560 and am looking at starting a family soon. i have heard that women with my condition have a higher chance of a miscarriage . I have seen numerous doctors and specialists (rheumatologists) who have said that they cant help me and that i will most likely have lupus down the track, another said that i have mixed connective tissue disease . i have been sick for 6yrs on and off now. my immune problems have made me catch a majority of illnesses from the common cold many times a year, two kidney infections very recently and pre cancer cells in my cervix. I would like to know should i see an immunologist or an obstetrician or both? My doctors have said that they dont really know what to do. Doctor: Hi Amy Welcome to HealthcareMagic ANA levels are quite high suggestive of lupus. You have heard right that there is increase risk of miscarriage. Apart from miscarriage I would like to tell you its not that only lupus affects pregnancy adversely, pregnancy also affects lupus adversely. There is flaring up of lupus. In pregnancy kidney disease is more likely to worsen. Also if you are on lupus treatment then there are certain drugs which affects fetus adversely. If you are planning pregnancy then it ll be certainly a high risk pregnancy and require frequent monitoring and may be admission to hospital. Ideally it should be a team of doctors looking after you-Obstetrician,Immunologist,physician,paediatrician. Take care and all the very best."
},
{
"id": 21015,
"tgt": "Suggest treatment for high blood pressure",
"src": "Patient: My husband was rushing to his office, went into the chemist to get his medication and they took his blood pressure there and then. It was 151 over 78, and normally isn't this high. He is 64 and takes Pizotifen for migraine which has put two stone on him already. Doctor: Yes you are right...this high blood pressure for his age...he may take something like ezapril or atacand to lower his blood pressure..."
},
{
"id": 142131,
"tgt": "Could the marks and pink discoloration on mid spine be pressure point bruises?",
"src": "Patient: I have a large pinkish discoloration of skin around my mid spine and brown bruise marks along my back left ribs. My mid spine is very painful. I have ms and my neurologist doesn't seem to think that it is related - they have mentioned that I may have pressure points on my back. I only weigh 100 lbs. and was sleeping on a very bad mattress which I have replaced with a serta firm firecreek mattress on Thursday of last week. Could these marks or bruises and pinkish skin discoloration, in fact, be pressure point bruises, or should I seek medical attention to determine if something else may be going on? Doctor: Hello!Welcome on Healthcaremagic!Your symptoms do not seem to be related to MS. Repeated trauma or a bad mattress could be the reason of such skin lesions. Anyway, if these skin lesions do not disappear after changing your mattress, I would recommend consulting with a dermatologist for a physical exam and some blood lab tests: - complete blood count for anemia- iron levels- blood electrolytes- coagulation tests. Hope you will find this answer helpful!Best wishes, Dr. Aida"
},
{
"id": 190926,
"tgt": "Why is there itching inside my mouth ?",
"src": "Patient: a few days back i consulted two Dentist regarding thickness on my inner cheek after biting over and over. i am damm worried about this. A DR. (MDS) has said nothing to worry about. but i am still feeling certain itching there not always but when i am worried. Although there is no change in in shape and color of it. Is itching normal please answer. Doctor: hi rakesh, Due to repeated biting on the cheek there occurs this \"linea alba\" which is a very common finding. Now as in your case due to repeated biting it seems that some fibrosis has occured which you can feel as thickening. If its very small in size and almost constant in texture then there is no need to worry and this itching will subseqently subside if you are no longer in habit of biting. Till then you can start with frequent gargles with betadine after diluting it with water."
},
{
"id": 174611,
"tgt": "Suggest treatment for fever and cough",
"src": "Patient: my son aged 2 n half having fever n cough since last one week, doc had suggested to do chest xray pa, report shows evidence of inhomogeneous opacity noted in right lower zone, cbc shows hameglobin 6.8, rbc-3.05, malarial parasite- not seen, sgpt-19. pls help with ur kindl adivce. kalim shaikh Doctor: Hi, welcome to healthcare magic. It seems that your child is having pneumonia. Also, the haemoglobin is less which can lead to severe hypoxia. Please show the child to a paediatrician for the management of the problems. He might also require a blood transfusion depending upon the vital status. He will also require chest physiotherapy, bronchodilators and broad spectrum intravenous antibiotics which can be done in hospital setting. Please take a consultation regarding admission to paediatric care unit. Hope he will get well soon."
},
{
"id": 118378,
"tgt": "How can ESR level be reduced?",
"src": "Patient: My self Vinoti Devi, Female , Age-53year 5 Month, i had HAEMATOLOGY-SEROLOGY Report is below:- HAEMOGLOBIN-12.8, TLC 6,200, Polymorphous 72, Lymphocytes 24, eosinophils 03, Monocytes 01, E.S.R. 43 mm 1st hour (WG) Plz help me what i do to reduce the E.S.R level. on urgently basis. Thanks mail ID YYYY@YYYY Doctor: HiThanks for your query.ESR is non-specific and does not always signify a disease. If you have any infection that needs to be treated, otherwise it can be followed up.Hope this helps.Regards"
},
{
"id": 33270,
"tgt": "Is pneumonia contagious?",
"src": "Patient: hello doctor 3 weeks before i was diognised as left basal pneumonia,i was admitted in hospital n given medicine through IV,after 3 days i was discharged n given oral medicine for 10 days.Doctor told to come after two weeks to take x-ray n will tell the condition .IS there any thing serious abt this n want to know is this contagious.Thanku Doctor: Hi, dearI have gone through your question. I can understand your concern. Pneumonia occurs due to respiratory infection and it is contagious but no need to worry. You have already taken a course of antibiotics. You are called for repeat x ray to check healing process. No need to worry about spread of infection. Hope I have answered your question, if you have doubt then I will be happy to answer. Thanks for using health care magic.Wish you a very good health."
},
{
"id": 217157,
"tgt": "Suggest treatment for pain at injection site",
"src": "Patient: Hello, I am using Penidure 12 Lac Units Deep IM from 8 years and now I am used to it like I do not take test doses. After applying the injection the pain remains for 2-4 days but last week I did the same but the pain has not gone yet and the area of application is swollen up. What is it and What to do next...? Doctor: As you are taking injection penidura I can guess most probably you have rheumatic heart disease. First of all even though you are taking penidura since last 8 years never ignore its test dose even when you are very sure that no any reaction appears. Now as far as your problem is concerned panidura is definitely a quite painful injection and if this pain is unbearable along with swelling of the local area than you can take tab combiflam or tab ibugesic-plus three times daily after meals. In addition to that nothing much you have to do, apply ice if swelling and redness is more."
},
{
"id": 171419,
"tgt": "What causes black colored stool in infant?",
"src": "Patient: 8 month old infant (eating prunes, taking pro-biotics and formula enriched w/DHA, eating other veggies and pears) started having dark black/green/tar-like poops. Is this normal? Also, once baby started eating solids, he will only take two bottles all day (5 oz) in a 10 hr period - we are giving him water, but he s only taking maybe total 3 oz. Is this ok, or should we reduce solids from 3 meals to 2? Doctor: Hi, this could be infection in intestine, do a stool routine microscopy test, stool for occult blood. Based on above reports, child will probably need Antibiotics. Revert back with exact weight of child. I hope this has helped you."
},
{
"id": 68872,
"tgt": "Suggest treatment for lump in my breast",
"src": "Patient: I am 46 years old, 5'5\" 160 lbs. female. I regularly exercise (running) and am in good health. I noticed a \"lump\" over my left breast (a little closer to my shoulder than my breast) several months ago. Recently I have also noticed \"tightness\" in my chest and nausea. Doctor: welcome to Health care magic.1.The way you have explained the location it doesn't look like a breast mass, its more towards lymph nodal mass lesions.2.Tightness in the chest and nausea - could be some URTI ( upper respiratory tract infection)3.It can be confirmed by getting an ultrasound breast and axilla scan done to know where it is arising from and what it consists of and how it involving adjacent anatomical structures.4. hope it helps you.Anything to ask ? do not hesitate. Thank you."
},
{
"id": 207024,
"tgt": "What causes lack of interest in everything and laziness?",
"src": "Patient: im really sad. im lost. i have no direction. i feel i might be crazy. people tell me things. maybe im in denial? ive lost interest in lots of friends and being super pretty all the time. im lazy. ive lost interest in hobbies and i feel like everything is impossible for me. when im offended or hurt everything rushes back to me and im depressed about everything all over again. im really general about everything. sometimes i lose hope. i think about dying. going to heaven or something. how better it would be. but i never think about killing myself. i would never. ive been told im overdramatic by my parents when ive cried over something that really upset me. i dont feel loved sometimes. sometimes i feel everyone secretly hates me. ive been told that i could be bipolar or something. i get lost in thought. i forget a lot of things lately. i sleep all the time. im always tired. i cant get up in the morning no matter how hard i try . i dont want to accept that this is the life. all this money... Doctor: you are very intellect, given a nice history. you are having depression. it is easily treatable. so do not upset.take tab escitalopram.tab rechamp gold.inj methcobalamine.you will be fine in 2 weeks.take care.do not forget to rate the answer.life is very precious.hope my answer helped you."
},
{
"id": 126395,
"tgt": "What causes pain in the upper left leg long after having a knee replacement?",
"src": "Patient: My upper left leg is painful and causing me to limp. It feels like the pain is coming from the femor. I ve had both of my knees replaced and the pain started at the knee then spread up to my hip. At times I need to use a walker to get around. The pain gets worse the more I walk and keeps me awake at night. I am 69 years old and my knee replacements were performed 10 years ago. I have been limping for over a week. Doctor: Hi, You need to repeat a X-ray of your knee. If everything is fine then start doing quadriceps and hamstring strengthening exercises which would have been advised to you earlier and probably you have left them because you feel fine. So start doing them again. Hope I have answered your query. Let me know if I can assist you further. Regards, Dr. Anuj Gupta, Spine Surgeon"
},
{
"id": 126676,
"tgt": "What can cause swelling and pain in the ankles?",
"src": "Patient: I am a 35 year old woman . I just had a baby 6 months ago. When I m pregnant my feet and ankles get swollen really really bad at the end. Well after I had my baby it went away but now it has come back to the worst they ever been snd it hurts to squeeze them and walk on them. What could it be? Doctor: Hello, Edema or swelling during the third trimester occurs due to the body retaining excessive fluid in the ankles and feet due to changes in the blood chemistry and increased pressure in the pelvis veins due to the growing fetus, excessive amniotic fluid can also be a reason. After delivery, the swelling generally goes away on its own. Keep your foot on a pillow while in bed or a stool when seated. Restrict salt intake, take iron and calcium supplements. If the edema remains get Thyroid function tests done, get treated. Hope I have answered your query. Let me know if I can assist you further. Regards, Dr. Nupur K, General & Family Physician"
},
{
"id": 26182,
"tgt": "Suggest treatment for fluctuating BP readings through out the day",
"src": "Patient: Hi good afternoon..yesterday i went to the clinic my pressure was 160/100 then they gave me a mipidipin to lower my bp after 20 mins they took again it is lower than the first 140/90... then the 3rd time it was 100/70....now i go back there to monitor my bp the results is 120/100...is this serious? Doctor: Hello there and welcome, First of all let me tell you not to be concerned with your Blood Pressure as its a variable thing and the more you'll worry and get into this monitoring and reading kind of stuff, the more your Blood pressure will shoot up as its called as body's stress response. My advice for you would be to relax , calm down and let the medical practitioner handle this. Any way even if the blood pressure ranges to be in the higher side(once again I'm telling you not to go into readings as the readings'll only mislead you since High/Low BP is a variable factor which differs from each and every individual), it's always better to get checked by a physician and act accordingly.Once again thank you for using HCM, happy to help"
},
{
"id": 206327,
"tgt": "Is there short term alternating phobia disorder?",
"src": "Patient: Can there be a such thing as STAPD (Short Term Alternating Phobia Disorder), where a person has extremely severe phobia but it is only temporarily and often changes every few months? Where the phobia changes? For example if you have a phobia to thunder and the next month you loose that phobia and is afraid of fire. In this you can have more phobias at more than one time and have repeated phobias...also the time perion you have the phobias could even be years....basically is it possible to have a disorder like this or is there another name for it?? THX Doctor: DearWe understand your concernsI went through your details. I suggest you not to worry much. I can assure you that there is no STAPD. But you should understand that while under generalized anxiety disorder, people are anxious about so many things. May be you are experiencing such a disorder. From the given symptoms proper diagnosis is not possible. Please consult a psychologist for a detailed investigation and psychometric tests to have a proper diagnose.Psychotherapy techniques should suit your requirement. If you require more of my help in this aspect, Please post a direct question to me in this URL. http://goo.gl/aYW2pR. Make sure that you include every minute details possible. I shall prescribe the needed psychotherapy techniques.Hope this answers your query. Available for further clarifications.Good luck."
},
{
"id": 213108,
"tgt": "Feeling sad suddenly, wanting to cry, loss of appetite and weight, nausea. Reason?",
"src": "Patient: I ve been very sensitive lately and I wonder if I might be pregnant, or if this is a sign of depression or bipolarity.... Out of nowhere I start feeling sad and I want to cry when I shouldn t be feeling like this....I have a boyfriend, family, friends, a job and I also go to school...i know I have been stressed out and worried, but i would like to know what s really going on.....and because of feeling like that i have lost some appetite and lost weight, and I feel nauseous sometimes too Doctor: Hello and welcome to Healthcare Magic. Thanks for your query. You symptoms may be indicative of a stress-related disorder or a depression. If you are finding it difficult to cope or function normally due to these symptoms, then I would suggest that you seek professional help. These psychological changes may be very unreliable indicators of whether you are pregnant or not. So, if you have had unprotected sexual intercourse and have missed your period, then it's better to have a pregnancy test done to check. Wish you all the best. - Dr. Jonas Sundarakumar Consultant Psychiatrist"
},
{
"id": 212310,
"tgt": "Have problems falling asleep, depression, took cap, got problem again. What should i do?",
"src": "Patient: hi .. i am a 22 years old.. i have problems falling asleep.. i could not sleep for more than 50 hrs until i finally fell asleep today morning for 5 hr.. i had the same problem a few months back along with depression, i consulted a psychiatrist and he prescribed me some medication.. i took it for a month or so.. and also tried to look for other solutions.. after reading a few articles i tried to control m anxiety which worked.. but its back now and nothing i do helps.. this is the first time i cold not sleep for so long.. what should i do? Doctor: Hi Sneha, Forum, After going through your problem - It seems that you had Depression disorder previously and taken a treatment for month and stopped medicine later on - tried to control thing by other means. Its important to know that when you diagnosed as having depression that means certain neurochemical (serotonin) changes in the brain causing this trouble.When its in initial phase i.e. milder one can be controlled by Yoga,Meditation,Motivating books. But when there is significant change in that neurochemical indicated by disturbed sleep and affecting your routine then - correction of that pathology first and continuing other method along with this later is only solution. Remember medicine takes 3-4 week time to starts its action and minimum of 6-8 month for permanent improvement when there are less chances of recurrence. Though you start feeling better within month of treatment (Giving false impression that I am all-right and can take care of myself without medicine),that was just a symptomatic one and not due to correction of underlying pathology.So after stopping of it within few month you started having problem again. So please consult to your psychiatrist again and restart appropriate treatment,continue it regularly till advised by him. Hope it helps. Thanks."
},
{
"id": 134527,
"tgt": "What causes delay in fracture healing?",
"src": "Patient: Good Day, my son is 16yo and fractured his r medial malleolus 6 weeks ago. it was the bottom portion that snapped off and it was believed the tendon was pulling the fractured bone away from position. unfortunately it is not healing. he was advised to continue wearing the air cast. I am concerned as he is young and question why he is not healing as a typical young healthy boy. his medical history - Nov 2012 my son had his first seizure, 20 days later a repeat. he was diagnosed with epilepsy and has been controlled on meds (valproic acid 750mg per day) since second seizure. I questioned whether this medication had something to do with his not healing. I am going to see our family doctor and request a referral to an orthopedic doctor, what other advise are you able to advise????? Doctor: hi,usually the healing of lower limb takes a longer time of upto 6-8 weeks. as you mentioned it is a malleolus fracture it will be a small chip fracture where surgery will not be of choice. as per the my knowledge the healing of such fractures may take a little longer time as it's a small chip which has to get attached to the main long bone called tibia. later I feel you can use some clacium supplement prescribed by the ortho and usually take vitamin D by directly exposing the body to the sun. try doing some ROM exercises non weight bearing for the knee and hip along with little toe movements to aid circulation.I wish your son a speedy recovery and a good health.thank you"
},
{
"id": 52626,
"tgt": "Can abnormal gall bladder functioning cause hot flashes in stomach?",
"src": "Patient: I have been diagnosed with Bilary dyskensia with a ejection factor of 10%. I am also having a hot feeling in upper stomach just under rib cage. it is hot when I put my hand on it and I get hot flashes but I am not menopausal. Could these be caused by my abnormal gallbladder function? Doctor: Hello and Welcome to \u2018Ask A Doctor\u2019 service.I have reviewed your query and here is my advice.Biliary dyskinesia is not causing hot flashes in your stomach. I think it might be a case of the ovulation period. So, get an abdominal ultrasound done to determine the right diagnosis.Take Mefenamic acid medicine and see if it helps. Antacids can also help.Hope I have answered your query. Let me know if I can assist you further.Regards,Dr. Albana Sejdini"
},
{
"id": 162546,
"tgt": "How can high fever and headache in a child be treated?",
"src": "Patient: I have a question my grandson who is 12 years old. He have symptoms of high fever 102 to 103 for 3 days, low white blood count, dehydration, nausesa, headaches, no energy and no appettite and diziness. One doctor thought he had appendicitis but a cat scan did not show anything. One doctor said he has a infection, but did not give him any antibiotics. What should we do? Doctor: Hello and Welcome to 'Ask A Doctor'. I have read your query and here is my advice. Sometimes a low white blood cell count can be due to a viral infection and last for a couple of weeks. There are other kinds of germs that cause low white blood cell count. These include bacteria, fungus, and germs carried by ticks. When these ticks bite a human, he/she becomes infected. Your list of your grandson's symptoms is worrisome. Because a high fever with headache could possibly be due to meningitis which is a MOST serious infection of the coverings of the brain. I strongly urge you to take him for medical care, perhaps this time to an emergency room right away! Hope I have answered your query. If you have any further questions, I will be happy to help. Arnold Zedd, MD, FAAP"
},
{
"id": 170803,
"tgt": "What causes sticky discharge from ear with awful smell?",
"src": "Patient: I noticed when I kissed my son on the cheek that something smelled awful. Later I noticed his ear was full of a sticky discharge that was a very dark brown. I cleaned out the part that I could see around the outside of the ear canal and it was definately the source of the smell. It smells bad and was very goopy. Could it be an infection? Doctor: Hello. Yes, it certainly seems like he has an infection in his ear. i would advise you to see a pediatrician/ENT specialist and do a culture of the sticky discharge to see what organism grows. They could also look into his ear with an otoscope and see if there is a hole in his ear drum. Usually bacterial infections are accompanied by pain in the ear, fever and yellowish pus.If he had just a mild amount of brown discharge that was not so smelly, we could have attributed that to ear wax, but lets not take chances in this case.Hope this answers your query"
},
{
"id": 135572,
"tgt": "Suggest remedy for radiating pain from wrists to elbows",
"src": "Patient: Hi there. I have recently been get to get to bubbling/fizzing sensation in my left arm. I also have random pain that shoots from my wrist to elbow and sometimes in my finger joints. About a month ago I was given steroids to treat this but pain was in both arms and I couldn t raise them above my head. Many thanks for your opinion. Yvonne Doctor: It appears you are suffering of cervical spondylosisMy advise isInj methylcobalamine 1500 mcg i/m ad *5Tab zerodol P bidPantop 40 odDefcort 6 mg tidMRI CERVICAL SPINECBCSer uric acidFbs ppbs"
},
{
"id": 131784,
"tgt": "What causes ankle pain after having ankle surgery?",
"src": "Patient: I had ankle surgery ten weeks ago.my ankle feels fine.after my foot rests it s sore to walk on sometimes I have two sharp pains that shoot up the side of my ankle.but what bothers me most is the pain I have on the top outside of my foot that runs under my foot.it feels like I have a cramp in the middle of my foot Doctor: Hi please write details of what surgery been done, is it fusion or repair of soft tissues,fracture fixation or implants used if anyA physical check by treating surgeon can clarify present issueTake Tylenol tabs for pain and apply voltaren gelThanks"
},
{
"id": 62988,
"tgt": "What could red painful lumps on wrist suggest?",
"src": "Patient: H round lump, which he says is painful to the touch. He is 7 years of age.i my son has a large red lump on his wrist and the skin is split down the middle. He fell at school earlier in the week and it started as a scratch, but now has turned in to a great big Doctor: hi.it is best if you bring your son to a doctor for clinical examination. it might just be an inflammatory reaction, but just to make sure that it is not infected and there's no need for surgical treatment, he is best evaluated physically. management will be directed accordingly. hope this helps.good day!!~dr.kaye"
},
{
"id": 47917,
"tgt": "What causes swallowing in feet?",
"src": "Patient: Sir I have been detected a stone in my kidney about of 1.5 cms, and as I feel that as much I am consuming the water in that ration it is not being converted in urine passing. Since last few days my feet is swallowed. Please suggest me my course of action Doctor: Hello, with the new addition of the symptoms you are giving i would suggest you to get a repeat USG of abdomen to lkod for any improvement or is there any descent of the stone from its previous position. I would also suggest you to undergo renal function test as your kidneys are not producing enough urine as much you are consuming fluids. It could be possible that the stone has not moved from position and is causing obstruction to urine outflow and is the reason for swelling. So ki dly get 1st repeat scan of abdomen and a renal function test. Thank you"
},
{
"id": 169538,
"tgt": "Suggest ways for weight gain in an infant",
"src": "Patient: This is about my 2 year old son now since he has been born he has had a problem gaining weight we have tried all kinds of formula ND all he would do is throw it up they have had him on a special kind of formula to help him and it did for a little while he couldn t eat if he had drank the milk or he throws up alot now he s off of the formula and he throws up at least 3 to 5 times a day the doctors in Amarillo Texas say nothing is wrong ... Now my son has gained weight he s 24lbs but the throwing up has not stopped I know something is wrong what should I do Doctor: Yours sons weight is fine as per the age and child in dis age group are usually fussy reg.food.u can give protein rich foods like pulses,egg white and meat along with other foods."
},
{
"id": 150102,
"tgt": "Suffer from bulging and torn discs. Extruded disc causing the sciatic pain. reddish blotches on hips and thighs",
"src": "Patient: I have 3 bulging discs, a torn disc & an extruded disc in my back. The extruded disc is at L5-S1 and is the root of my sciatic pain - Last night, my husband noticed that my upper leg around my hips & thighs was marbled like red & white circle blotches. Is this something typical with my back condition or something different? Doctor: Hi,Thank you for posting your query.Skin changes are not usual with disc bulging at L5-S1 and compression of the nerve.Pain is a common feature, which can radiate upto the foot and toes, along with tingling and numbness.So, if you can upload a picture of the affected area, it would help in making a diagnosis.Please get back if you require any additional information.Best wishes,Dr Sudhir Kumar MD (Internal Medicine), DM (Neurology)Senior Consultant NeurologistApollo Hospitals, Hyderabad,My personal URL on this website: http://bit.ly/Dr-Sudhir-kumar My email: drsudhirkumar@yahoo.com"
},
{
"id": 111591,
"tgt": "Can I take carnisure tablets for severe back pain caused by severe physical activity?",
"src": "Patient: Hello Doctor, my brother had server back pain due to his heavy body- weighing about 100kilos. He was unable to sit for hours because of back pain. Due to the nature of his job i.e., IT Professional, it required worling over a PC for long hours. A local doctor had prescribed carnisure tablets and my brother did manage to lose around 14 kilos under medication and also morning walks. he was unable to do so by engaging hisself in morning work only. so adding carnisure with his physical activity has made him lose weight. Is it okay to use this medicine for weight loss? however he did not feel any side effects due to the drug use. Doctor: His physical activity should be combined with caloryreduction in food by expert advise. reduction of weight should not be abrupt. If medication can be avoided it is well and good."
},
{
"id": 123719,
"tgt": "What is the medication for Carpal Tunnel?",
"src": "Patient: I would like to know if Meganeuron OD Plus has vitamin B12, and what are the uses of this drug. I also suffer from Carpal Tunnel. I have been taking Meganeuron since the last 1 year, but dont seem to be benefiting for Carpal Tunnel and I still feel very tired Doctor: Hello, As you are on medication which isn't helped you why don't you try for physiotherapy. Noninvasive and completely safe. The flexor retinaculum is tightened in carpal tunnel syndrome and needs a release of it. With therapeutic ultrasound therapy and TENS therapy, the inflammation and pain will be reduced. MFR and IASTM will help reduce the spasm too. Some exercises to improve muscle strength and avoid spasm as well. Hope I have answered your query. Let me know if I can assist you further. Take care Regards, Jay Indravadan Patel, Physical Therapist or Physiotherapist"
},
{
"id": 9274,
"tgt": "Suggest treatment for itching and dryness in feet",
"src": "Patient: The bottom of me feet are very dry and some places are yellow, I ve previously tried to cut the hard yellow skin off but it s grown back the same as before, it is not raised, just dry and yellow. My feet previously looked like they had psoriasis, but that faded. My feet are sometimes unbearably itchey and hot. In addition, my toe nails are raised and look like a mild fungal infection (yellow, dandruff-ey) Doctor: HIWell come to HCMThis may not be fungal infection but could be dry skin may be due to vitamin A or vitamin E, deficiency initially this can be best manged with moisturizing agent that contained urea, liquid paraffine, lactic acid, and steroid, eat more vegetable drink more water, hope this information helps, take care."
},
{
"id": 123666,
"tgt": "What causes pins and needles in left arm/leg?",
"src": "Patient: hi there for the past few months i have been gettting pins and needles in my left arm every now and again but now for the past 2 days ihave had it all day long also i have some pins and needles in my left leg, i had malignant melanoma 3 years ago thank you regards anne Doctor: Hello, It could be neuropathic pain. As first-line management, you can take analgesics like paracetamol or aceclofenac for pain relief. If symptoms persist better to consult a neurologist and get evaluated. Hope I have answered your query. Let me know if I can assist you further. Take care Regards, Dr Shinas Hussain, General & Family Physician"
},
{
"id": 160159,
"tgt": "How long will Bile Duct Cancer take to spread to the brain ?",
"src": "Patient: How long does it take for Stage 4 bile duct cancer to spread to the brain ? Doctor: Hi Sal. Thanks for Posting your query. Stage 4 Bile Duct cancer means that its already spread outside the primary site. Most of these spread occurs through blood stream. Its difficult to tell in exact time frame, how long it would take to spread to brain. However, the closest answer would be that it can take a few weeks if not being treated. Hope this answers your query. Thanks and Regards Dr Kiran"
},
{
"id": 126348,
"tgt": "What causes throbbing pain in the foot?",
"src": "Patient: my 54 year hold husband has a throbbing , dull pain on top of his right foot. there is some swelling and a red bump. he has soaked his foot in Johnson s Foot Soap , and that has not really helped. His foot feels better after resting, and wearing his shoes with looser laces. we have not tried icing the foot yet. Doctor: Hi, In my opinion, redness, swelling with throbbing pain on top of the foot should be treated as infection, a boil or cellulitis. Please get your blood counts and consult your GP for starting antibiotics if required. Hope I have answered your query. Let me know if I can assist you further. Regards, Dr. Gopal Goel, Orthopedic Surgeon"
},
{
"id": 200316,
"tgt": "What causes shrinkage and pain in penis after ejaculation?",
"src": "Patient: im young (19 yrs old) but i have been having problems with my penis since i was 17 and had peyronie s causing my dick to curve slightly to the right but its been soooo long and symptomps that are not associated with peyronies that make me think that its not what the problem is. i went to the doctor two weeks ago who said its most likely a prostate infection and gave me antibiotics but its not doing anything since i see no change here are my symptoms: 1-shrinkage and pain in penis after ejaculation 2-random stinging sensation in testicle 3-loss of libido 4-lack of blood sometimes when in flaccid (relative to how it should be when in flaccid normally) 5- i have been noticing green popping out small pin head sized vein popping out in 2 areas on my penis 6- on my testicle idk if this is normal or not i have been noticing darkish veins(purple) Doctor: DearWe understand your concernsI went through your details. The symptoms / problems you are giving here are normal for person aged 19, except for random stinging sensation. This sensation could be due to either internal inflammation or obsession. Drink plenty of water and stop masturbation for some days. Clean your penis and keep it hygiene. After a week, if the condition is the same, you need to consult a physician.If you require more of my help in this aspect, please use this URL. http://goo.gl/aYW2pR. Make sure that you include every minute details possible. Hope this answers your query. Available for further clarifications.Good luck."
},
{
"id": 8707,
"tgt": "Bleach cream used, scars on face, skin irritation, itching. Using vaseline. Treatment?",
"src": "Patient: I used bleach cream on my face last night. Today, I woke up with some scars on face and my skin is irritated. I keep on wanting to itch but then stop because I don t want any permanent scarring on my face. I have been using Vaseline to calm my skin. Will my face be permanently scarred? Am I doing it right? Any other suggestions? How long will it be till my face goes back to normal? Doctor: hello thanks for posting your query. whaat you might be having following the bleach cream are not scars but maybe pigmentation. your skin has reacted to the bleach cream. make sure you wash it off thoroughly. also application of cortisone cream twice a day for a week will calm the skin down. it may take upto 2 weeks to recover, in case the pigmentation is excessive you may need to be started on oral medications for which you will have to visit your doctor. if you take proper treatment in time, permenant effects can be avoided. regards"
},
{
"id": 137608,
"tgt": "What causes sharp pelvic pain after myomectomy surgery?",
"src": "Patient: hi had myomectomy surgery nov/24/15 didn t have hardly any pain after sugery just very little..but now I still continue to have a lot of gas constantly and feeling sharp pain near binkini area where cut was made its like the internal stitches pull and does the cold air make it worst here at work?? Doctor: Hello,It is common to have such pain after uterus surgery. This is due to healing through fibrosis. I would recommend you to get ultrasound of the abdomen to see if there is any problem in scar healing. Some time there can be scar breaking also there. I hope this answer will be useful for you. Let me know if there is any other followup questions.thanks"
},
{
"id": 100943,
"tgt": "Suggest treatment for allergy",
"src": "Patient: Hello?I'm a 24 year old female with no medical history or allergies whatsoever. Over the last year, my menstrual cycle has decreased in occurrence. In fact, it comes every 4 months. The first time this happened, I was terrified I was pregnant because I also began producing a sticky, yellowish clear fluid from my breasts-10 pregnancy tests (including one administered by a physician) indicated that I was not pregnant. When I do get my period, it comes with tremendous bruising on my hips, thighs and sometimes forearms. My appetite is very poor. I have had had my iron levels checked but all signs point to normality. I do not carry health insurance and simply do not have the means of diagnostic testing. Any idea what might be going on with my poor body? Doctor: Hi thanks for posting your query on health care magic.Menstrual disturbances in your age group point to some underlying endocrine abnormality, most likely thyroid hormone disturbances .I suggest you to get screened by complete thyroid profile.correction of thyroid abnormality can correct menstrual irregularities if any.Bruising on hips and thighs and forearms has to carefully evaluated through blood coagulation testing like BT ,PT,INR. AN ULTRASOUND of the abdomen helps to visualize and detect any co existent abnormality.consult your gynecologist for further queries.thank you.take care"
},
{
"id": 213698,
"tgt": "Will eraz ex fluid cause any mental abnormality ?",
"src": "Patient: hi doc, my brother whose age 26 year. three years before he started smell Eraz-ex correction FLUID and this time he is totaly addicted. i feel so many changeg in his behaviour. he is always talking about money .he is not intrested to do any kind of job.when he is not take fluid he looking norml.his behaviour is aggresive. he drink alcohol bt in limit.doc he says always abnorml thing like Amerika president OBAMA won ellection with my support. he always coorelate him to any kind of TV news . he says that he handle all over world. plz tel me what we do? hi has mentel problem, depression or addiction i dont Doctor: hello welcome to healthcare magic forum, this kind of behaviour is due to depression and is some sort of dellusion ,give some anti-depressant and take advice of psychiatrist.Take care"
},
{
"id": 77151,
"tgt": "How to treat lung tuberculosis?",
"src": "Patient: hi this is janakiram from bangalore, i have taken esr test on 3 days back result is 20mm/hr.last year i was suffred with lungs tuberculosis and used tablets upto 9 months, then after an year againg i have taken ESR test it given 30mm/hr can u please suggest me whether the disease is there on my body? should i consult doctor? please replay me Doctor: Hi and thank you for choosing HCM to post your question.I read carefully your question and understand your concern.The empiric treatment of lung tuberculosis consist of the following 4 drug regimens:Isoniasid,Rifampin,Pyrazinamide,Ethambutol or Streptomycin.The first 2 months the patient takes 4 drugs and if the patient remains Positive(means that in the sputum of the patient you can see bacilus of TB in direct microscopy) he continues even for another month and if still positive one more month(like i think is your case)and the other 5 months the patient takes only Isoniasid and Rifampin.During treatment patient need to do Direct microscopy of the sputum at the end of second, third, four months and in the end of treatment .Patient is considered healed from lung TB when he has at least two negative direct microscopy of the sputum and there is radiologic and clinical improvement.Esr(erithrosedimentation rate) is a test that detect infammation with conditions such as infection,cancer,immune diseases.It is a nonspecific test that does not tell where in the body the inflammation is(lung,urinary tract ,other organs)or what is causing it(TB,other bacterial infections,rheumatic disease ect)Also ESR can be affected by other conditions besides inflammation so is not an indicator of telling about Lung Tuberculosis.I hope my opinion can be helpful.Wish you good health.Kind regardsDr.Dushi ."
},
{
"id": 129825,
"tgt": "Suggest treatment for muscle pain",
"src": "Patient: Hi there! I have had a really bad cold for the past seven days. It s included extreme lethargy, a bad cough that has developed after the first 2 days, stuffy nose, muscle aches and pain in my lower back and my groin muscles that will not go away. Also some ear pressure. General achy feeling, and the usual, hot and sweaty and then cold sweats. No fever, however. I went to my primary and have taken a dosage of Zythromax, which I just finished the third pill yesterday (which made me nauseous andI vomited once). I ve been drinking as many fluids as I can, but haven t really eaten much for a few days (my appetite is feeling like it s coming back today). Any suggestions for the muscle pain, and especially the groin muscle pain? I feel like I ve worked out, but obviously haven t. Doctor: Hello!Your case is better to treat in hospital, if there is no decrease of symptoms and their manifestation."
},
{
"id": 51561,
"tgt": "Will medicines given after hernia surgery affect the kidney ?",
"src": "Patient: Dear Sir, My father has only 1 kidney in function since childhood. and functioning kidney is almost covered with stone, as told by Neurologist in Bhopal. my father is also suffering with hernia . doctors here says it may be dangerous for kidney to operate Hernia, as medicines given after operation may harm kidney. His age is 68 yrs. his latest sonography Doctor: Hello. Thanks for writing to us. You need to get the calculus of the kidney treated first by laser lithotripsy. After stone removal, once the renal functions come back to normal then he can be taken up for the hernia surgery. His age is also a big risk factor for anaesthesia. I hope this information has been both informative and helpful for you. Regards, Dr. Rakhi Tayal drtayalrakhi@gmail.com"
},
{
"id": 183936,
"tgt": "Does swallowing semen cause bad smell for breath?",
"src": "Patient: Can swallowing semen on a regular basis cause bad breath? I'm in my early 40's, eat a well balanced diet, follow a good dental regimine (floss daily and get teeth professionally cleaned every 4 months). At times my breath smells like spoiled meat / eggs. It's really embarassing. I enjoy oral sex and swallow semen at least three times a week. Doctor: Bad breath occurs due to reasons apart from poor oral hygiene. Gastric problems, sinusitis, respiratory problems, liver disorders, consumption of onions, garlic can result in bad breath. Identify if you suffer from any of these problems and consult the specialist."
},
{
"id": 191062,
"tgt": "Having bilateral white patches in tongue",
"src": "Patient: dear doctor, its been more than 6 months i m having bilateral white patches in my tongue.. i looks like leukoplakia..it does not goes off while brushing.. it is like fissures ..i am so worried weather it may be premalignant or OLH.. about OLH i used to have unprotected sexual intercourse with my girlfriend 10 months ago .. since we broke up i tested negative for HIV 3 times by TRIDOT method of testing..(2 weeks,4 months and 7 months)... what should i do please give me some advice.. Doctor: hello,since you have tested negative for HIV so its almost ruled out.. get examined yourself by your doctor... it can be due to some vitamin deficiency or fungal infection or improper oral hygiene...take care"
},
{
"id": 74202,
"tgt": "Suggest treatment for pressure on chest and labored breathing",
"src": "Patient: Hi, may I answer your health queries right now ? Please type your query here...hello I've been having problems with my chest feels like someone is pressing down on it and I have to keep taking deep breaths then my neck and shoulders have been hurting I've been. Scared to go to the doctors because my son had wilms tumor cancer and my mom had lymthphma cancer and every since they have been diagnosed I'm scared of even going to the doctors Doctor: Respected user , HiThanks for using Healthcaremagic.comI have evaluated your query thoroughly .* This seems more of psychosomatic manifestation of underlying stress or anxiety episode .* Suggestions for better recovery- Drink plenty of liquids always .- Consume balanced nutritious diet .- Avoid stress , anxiety by regular meditations , exercises , YOGA , deep breathing exercises , walking in fresh air .- Take proper sound sleep of 8 hrs .- No smoking or alcohol consumption of any form .- Be positive attitude in life .- Involve more in your likelihood activities .* Follow this strictly for 10 days and then revert back for the progressHope this will help you in your crucial times .Regards ."
},
{
"id": 165538,
"tgt": "Is Similac formula recommended for an infant?",
"src": "Patient: I have a 1 month old, he was on Simalac formula but last week I brought him to emerg because he was spitting up after every meal, so they told me to switch him to Soy formula. Since Monday now he has not gone for a bowel movement, and starting today he has thrown up after every meal. When he is up he is constantly straining himself to try and go to the washroom. I can tell that his belly is hard and that he is very uncomfortable. Doctor: Hello and Welcome to \u2018Ask A Doctor\u2019 service.I have reviewed your query and here is my advice.I feel that we need to rule out idiopathic hypertrophic pyloric stenosis. So, I suggest that you consult your pediatrician or a pediatric surgeon regarding this. Your kid might require an ultrasound of the abdomen for diagnosis.The other possibility is that he might be having a gastroesophageal reflux which your pediatrician will be able to take care.Hope I have answered your query. Let me know if I can assist you further.Regards,Dr. Sumanth Amperayani"
},
{
"id": 152109,
"tgt": "I have bumps on my scalp that seem to be filled with fluid. what it might be ?",
"src": "Patient: I have noticed that i have bumps on my scalp on the left side only that seem to be filled with fluid. its worrying me please tell me what it might be ? i also suffer from synus problem on the left side of my face and have head and eye aches too. i also suffered from infarcts in my brain last year and also had a infection in my left upper molars which went into my root canal and it was draining into my mouth. Plus I was told that my left side on my face the bone near the ear on my skull is pushing into the side of my ear. so as you can see there are lots of factors to consider Doctor: Are you a diabetic--if you do not know--please do a blood glucose test and revert--what about your blood counts?"
},
{
"id": 191251,
"tgt": "How can high FBS levels be brought down while on Metformin and Invanka and a suitable low carbohydrate and low sugar diet?",
"src": "Patient: I have Diabetes 2. I am a 65 year old female....I take Metformin and Invanka.....Lately, even not eating sugar or bread or flour...very little carbs...my sugar is running over 200 in the mornings before eating. This morning it was 283....I go to my physician for my 6 month checkup in April....I want to get this down...I don t want to go on insulin....what can I do? Doctor: Hi, If the fasting blood sugar is high then there's will increased glycogen lysis by the liver. So it may lead to high fasting sugar. It may be due to the dawn phenomenon also. It may also due to the insufficient dosage of before night insulin or drugs. So you may have to take sufficient dosage of oral antihyperglycemic drugs regularly. So you have to change your oral antidiabetic drugs. Please consult your diabetologist, he will examine and treat you accordingly to your fasting, postprandial blood sugar reports. Hope I have answered your query. Let me know if I can assist you further. Regards, Dr. Penchila Prasad Kandikattu, Internal Medicine Specialist"
},
{
"id": 97609,
"tgt": "Is it possible to increase height at the age of 27?",
"src": "Patient: I am 27 years old, my height is 5.2 and i want to increase my height, after seeing advertisement of Dr. ayurveda and stepup herbal body growth formula, i would like to buy but before that i want make sure that it is true at this age i can increase my height, pls. advice Doctor: **1. Height of an individual also depends upon the height of the parents [genetic], if they have short stature, their siblings can/may have the same height.2. No Pharma company will ever describe his/her product not to be authentic, [same is with the product you have mentioned].PS. don't fall for marketing gimmicks.3. There is no such magic Ingredient in any herbal formula which can increase the length of bones [after puberty is achieved]4. eat a healthy, balanced and nutritious diet and practice pranayam,yogasanas, back exercises that stretches the spine which helps in increasing height by 1\u201d-2\u201d inches.PS. Contact with a yoga expert for various Yogasans which can be adopted and practiced in daily routine."
},
{
"id": 28652,
"tgt": "Is coughing a symptom of sinus infection?",
"src": "Patient: I have constant cough. X-rays, clean lungs and esophagus.doctors think it could be sinus problems,they put me on prescription nose spray am pm, also sInus pills which gives a dry mouth and that s it. I always have a productive cough ( much mucus ).any ideas? Thank you! Jim Doctor: Hello and Welcome to \u2018Ask A Doctor\u2019 service. I have reviewed your query and here is my advice. You haven't specified the duration of this symptom, which is very important in order to form a proper differential diagnosis. I can make some suggestions though. First of all, sinusitis may indeed cause cough and phlegm. Sinusitis may even cause chronic cough. In most cases the diagnosis is made clinically (with medical history and clinical examination). An X-ray of the sinuses will help. \"Full\" sinuses do not always mean sinusitis but clear sinuses safely excludes this diagnosis. If your cough and the rest of the symptoms are recent in onset then a bronchitis is a potential diagnosis. Bronchitis cause cough with phlegm and usually lasts for 2-3 weeks (or even more). The X-rays are negative in bronchitis. If your symptoms started more than a month ago then other disorders might have to be considered. In that cases please post a new question with more information about your symptoms and what you've tried so far. Hope I have answered your query. Let me know if I can assist you further. Regards, Dr. Panagiotis Zografakis"
},
{
"id": 63795,
"tgt": "Suggest treatment for a pea sized lump on the groin area",
"src": "Patient: I have an oblong shaped hard lump, maybe the size of a pea, under the skin in my groin area. No color to it, but it did have a flat red spot in the center like a pimple had popped. When squeezed, I got a lot of blood (some stringy blood as well) and a small amount of other matter that looked more like fatty tissue than pus. I put some alcohol and triple antibiotic cream on it, but 24 hours later, it is the same. Doctor: Hi, dearI have gone through your question. I can understand your concern. You may have some enlarged lymphnode due to reactive hyperplasia, tuberculosis or lymphoma or some abscess or cyst. You should go for examination and if needed go for biopsy of that lump. It will give you exact diagnosis. Then you should take treatment accordingly. Hope I have answered your question, if you have doubt then I will be happy to answer. Thanks for using health care magic. Wish you a very good health."
},
{
"id": 131109,
"tgt": "What causes swollen lymph node over the left collar bone and arm?",
"src": "Patient: I have a swollen lymph node over my left collar bone and under my left arm that I have had for about two years. The one in my neck is causing my neck, shoulder and down my left arm to hurt. I showed it to the Dr. that I am seeing now and he wanted to do a ultra sound and he said it was my lymph node and that he was going to watch it. I stay sick on my stomach and tired all the time. I have a temperature everyday of 99.5 to 102 degrees. I have gained about 25lbs in the last year and just don t feel real good. I am only 46 years old and don t think that I should feel as tired and bad as I do. Please let me know if you think if it is a good idea to watch this or do I need to see another doctor. I have already had some lymph nodes removed due to breast implants that I had to have remove. Please advise!!!! Doctor: I understand your symptoms. You should apply topical diclofen sodium or potassium cream twice daily for 2 weeks and oral non steroidal antiinflammatory drugs 3 times daily for one week."
},
{
"id": 192291,
"tgt": "What causes severe itching in penis with yellowish discharge?",
"src": "Patient: Hi!I got sex with a girl i met last week in club.the condom broke,and after sex my sex was itching,two days later my urethra was kind of ticklish,and now there is a yelow dischage but it don`t hurt.i went take an std test but my result will come in 2 weeks what can i do? Doctor: Hi, It may be infection or allergy. Maintain hygiene. Keep the area clean. Wait for the results of STD testing to come. Hope I have answered your question. Let me know if I can assist you further. Regards, Dr. Sujoy Dasgupta, OB & GYN Specialist"
},
{
"id": 166093,
"tgt": "Is black stool common in babies with pus bumps?",
"src": "Patient: is super super dark , but not tarry black, stool normal for a 15m old? her movement first had light brown then the dark ones (in diaper). I am worried because yesterday was biten by mosquitoes and has puss bumps and discoloration also. (bumps that remind me of a spider bite). And those three symptoms sound like dengue shock syndrome to me, but no fever. Can this be the beginning stage of blood in stool? and can you have dengue shock without fever? Thank you. Doctor: hellothe symptoms you have described are perfectly normal.black TARRY stools are alarming,only black could be due to constipation or something in the diet he has taken.Dengue shock syndrome starts with history of fever and it then subsides for like more than 24 hrs but the condition deteriorates.So there is nothing like Dengue shock in your child,dnt worry"
},
{
"id": 224924,
"tgt": "Need medication to conceive after removal of copper T 6months back",
"src": "Patient: hi am swathi i am married 3years completed v have a baby girl she is 2years . after her birth v had copper t and after 1 year 6months v removed . nw almost 6months finished after removing copper t . nw v r planing to have a baby . am nt able to get pregency, am gettng periods regulary after meetng also, startng 3 months my periods where for 5 days and nw it became for 3 days it self and my stomack is becoming big ma nt able to understand wht happing so plz advice me Doctor: Hi,\u00a0\u00a0\u00a0\u00a0\u00a0Thank you for choosing Healthcaremagic. If you have missed your periods kindly get a urine pregnancy test done. There is no specific medication to get pregnant after Copper T removal, what I suggest you is consult an OBGY specialist for Ovulation Induction as you are very anxious to conceive....Please ask if you have more questions. If you are satisfied, please make sure that your ACCEPT my answer so that I receive credit. Good luck!!"
},
{
"id": 113560,
"tgt": "Frequent urination, lower back pain, had tubal ligation. What could be wrong?",
"src": "Patient: i am tired for weeks now since saturday i have frequently been going to the bathroom and this week my lower back has been hurting what could be wrong with me i had the new tubel ligation surgery the one with the insercts i got my period on the 7th it was different then normal but a lot has change since the surgery 2 years ago Doctor: get an MRI of the pelvis and get back to us with the report. a lot of things need to be ruled out before commenting on ur question."
},
{
"id": 206086,
"tgt": "Suggest treatment for symptoms of hallucination and headache",
"src": "Patient: I m really sick with something and I dont know what. I ve been having bad allergies lately but I get them every year. Last night I went to town and couldn t finish driving home bc I felt sick. My friend drove me home. I went directly to bed. I don t know how to explain it but every time I closed me eyes I could see things in my head. I was fully aware and awake but it was kind of like I was hallucinating. I had a fever but it wasn t really on my head but on my back and stomach and legs. I kept catching myself twitching. I don t know what it wrong. I took some Tylenol and finally got to sleep. This morning I have a headache, feeling a little dizzy, and I have a sore throat.. (I ve had it for about 2 weeks.. I figure it s because of my sneezing so much.) Does anyone know what could be going on?! Doctor: DearWe understand your concernsI went through your details. I suggest you not to worry much. Simple hallucinations are possible due to continuous extreme stressful situations. You must introspect regarding your stressful situations and find a remedial action. For the time being, you should consult a physician for treatment and then think about going for a fortnight's vacation with your family.Psychotherapy techniques should suit your requirement. If you require more of my help in this aspect, Please post a direct question to me in this URL. http://goo.gl/aYW2pR. Make sure that you include every minute details possible. I shall prescribe the needed psychotherapy techniques.Hope this answers your query. Available for further clarifications.Good luck."
},
{
"id": 79381,
"tgt": "What causes recurring discomfort on left side of chest?",
"src": "Patient: Hi there , I am a 42 year old woman , whom get discomfort when sednatary in the middle slightly to the left of my chest , this comes and goes , I have always been very fit and have had this for many years now .Any ideas ? Regards Kathy from New Zealand Doctor: Thanks for your question on Health Care Magic. I can understand your concern. Females are at increased risk of cardiac diseases after the age of 40 years. This is because of hormonal changes. After 40 years, when women approach menopause, estrogen level start decreasing. Estrogen is cardioprotective and hence females are at increased risk of cardiovascular disease after 40 years. So we should first rule out cardiac diseases in your case. So get done ecg and 2d echo. If both are normal than no need to worry for heart diseases. Possibility of musculoskeletal pain is more. So take painkiller and muscle relaxant drugs. Avoid strenuous exercise and heavyweight lifting. Apply warm water pad on affected areas. Don't worry, you will be alright. First rule out cardiac cause. Hope I have solved your query. Wish you good health. Thanks."
},
{
"id": 191755,
"tgt": "How to treat rise in the blood sugar level after taking Methamphetamine?",
"src": "Patient: My pal who has been diagnosed w/ diabetes & the 40 something yr old fellow also is a regular (sometimes daily) user (a shooter) of methamphedamine . Well he is new to this an hasn t as of yet been regular w/ doing his insulin .He saw the dr yesterday an was given his perscription (they increased his from 10 to 14 units & increased his oral med dose to 1000 also) & was informed that if he sees readings above 500 to go to the ER. Many times he runs above 500 I hear. Well he has done meth tonight an has just done a reading an it is 565 an was gonna do his insulin (though his girlfriend suggests to hold off an do it at 7am instead of now @ 1am so to start doing it at the same time each day) but now w this high blood sugar reading what should we do ? He didn t wanna go to the E.R. an said it (his high reading) is probably cause he has recently just shot meth . So is he in need of the E.R.? Doctor: Hello, Thanks for the query.It is clear that your friend has highly uncontrolled diabetes. Plus taking methamphetamine can further aggravate and cause severe hyperglycemia as you have now mentioned. This can also lead to a severe complication of diabetes called as diabetic ketoacidosis. Which can be a serious emergency condition needing hospitalization and blood glucose control to be achieved through IV Insulin and other supplements (electrolytes). At 565 mg it is essential that he goes to ER. Taking Insulin at home may still not correct the situation. If he wants to control his blood glucose effectively, revent possible long term complications of diabetes then should refrain from methamphetamine. Otherwise he will face more serious consequences. Ideally blood glucose should be maintained as : FBG <100 mg, PP 150 to 160 mg & HbA1c%<6.8. He is no where close to these recommended levels.It essential to convey these things to him. Thanks."
},
{
"id": 1710,
"tgt": "How the delayed spill in bilateral tubes can delay pregnancy?",
"src": "Patient: We are trying for a child since last 7 years. Based on the doctor suggestion today my wife has gone through the laparoscopy with chromotubation. Reports shows that she is having normal bilateral tubes but showed extremely delayed spill. Doctor has given the appointment for next week but this point created doubt in our mind.Please let us know what is this and how it is impacting. Doctor: hello...thanks for trusting the healthcare magic doctors for ur health related queries.I guess ur concern is about..how delayed spill in bilateral tubes can delay in pregnancy..right.my answer is..there is no direct relation between delayed spill from the tubes n delay in pregnancy. once there is spillage from the tubes means the tubes are patent that means there is no blockage in the tubes.so DON'T WORRY..KEEP PATIENCE. ur wife can be became pregnant in future.delayed spillage may be due to some flimsy adhesions or any infections etc. but after chromotubation( that means dye injected inside the uterus which comes out through the tubes into the peritonium) this types of conditions may clear and it will help to getting pregnancy.so RELAX...nothing to worry.with the above discussion u can try out my suggestions-1. RELAX..everything will be alright. just keep patients.2. feel free to ask ur treating doctor regarding this matter. he /she can help u.3. if u r in hurry then visit nearby infertility specialist. they are expert in this field. he/she can help u.I hope the above information is useful, informative and helpful for u.regards-Dr sudha rani panagar(i can understand ur tense-full situation, DON'T WORRY..be happy.I pray the God that u will be blessed with a cute n healthy baby..hopefully TWINS)"
},
{
"id": 137194,
"tgt": "What causes swelling on calf with bruise and tenderness?",
"src": "Patient: Hi I am 46yo i was involved in an automobile accident today. My left calf began to swell immediately after impact. The area is severely swollen, bruised and is very tender to touch. I am unable to walk comfortably and my foot is cramping. I have been icing the area on and off for hours. Doctor: Hi there.I advise you to keep icing the area. Also most important is to keep the limb elevated over a couple of pillows or a chair. This will help drain away the swelling. Take Tab. Ibuprofen 200mg twice a day for 7 days."
},
{
"id": 160963,
"tgt": "Is Lactogen-1 safe for babies?",
"src": "Patient: i had no sufficient breast feed so doctors suggested Lactogen-1 for my baby now he is 5 month every time he is suffering with bloody motions & throught ulcer, lactogen -1 is safer to my baby if not can we give her cow milk or any other formula milk Plz suggest me sir Doctor: Hello, Cow milk is not recommended at this age, you can try formula feeds instead. Nanpro or similac are recommended by most of the pediatricians. Hope I have answered your query. Let me know if I can assist you further. Regards, Dr. Shinas Hussain, General & Family Physician"
},
{
"id": 189727,
"tgt": "Have cuts inside lower lip. Teeth on lower jaw are jagged. Treatment?",
"src": "Patient: Hi, I have a problem with small little cuts on the inside of my lower lip . On the same side of flesh that lays against my lower teeth . They aren t particularly painful unless my teeth put pressure on them by biting my lower lip, and I believe they might be from my actual teeth cutting into my lip. Admittedly, the teeth on my lower jaw are pretty jagged and not very straight. The cuts just sort of look like precise deep-ish indents that are colored red or dark pink, much darker than the rest of my inner lip. They are only visible if I pull my lower lip down to see the innermost side of the lip. Doctor: DEAR FRIEND. Thanks for sharing your concern. your history suggests that the cuts inside your lower lip is due to the traumatic bite caused from crowded lower(anterior) front teeth. For temporary relief,you can apply gel on the affected area( dologel). For permanent relief,please visit your orthodontist and get appliance to correct your Mal aligned teeth. If the cuts persists,it can lead to ulceration and canker sore too. meanwhile you can take capsules becosules for 15 days. also maintain very good oral hygiene, as Crowding of teeth also results in plaque accumulation and thereby gingivitis . hope it helps thanks."
},
{
"id": 156814,
"tgt": "Does high level of tlc cause cancer?",
"src": "Patient: Hello doctor I mean a post renal transplant patient with a good function of kidney.but my tlc is very high somewhere between 0000_0000. Doctor says it is okay. But I might worried whether is has something to do with cancer?my all para meters really good. Plz help. Doctor: Welcome to Health Care Magic.My dear friend only high tlc is not associated with cancer until it contains abnormal cells in blood.As you told that your doctor says its okay then its mean he has already checked your blood film.I suggest you not to worry about it.Just follow your doctor.Thanx for your query."
},
{
"id": 182856,
"tgt": "What causes sensitivity in upper front teeth?",
"src": "Patient: one of my upper front teeth hurt for like 3 secs when i drink something cold or hot and when i bite down . My teeth have no dark spots , nor yellowness . The gums don't have black spots nor yellow ness ... This happened three days ago . also when i breath in cold air it hurts for like 3 seconds . could it be that , that one teeth is sensative for a couple a days or so ? or is it that the anemel needs that paste to help it ??? Doctor: HiThere could be to reasons for sensitivity.It could be carious inside or enamel of that tooth might have been lost due to various reasons.You need to get x-ray done of that tooth.If its due to caries then get it restored.If its enamel loss then use desensitising tooth paste containing pottasium nitrate.Visit your dentist for further treatment plan.RegardsDr. Neha Sumra"
},
{
"id": 163369,
"tgt": "Suggest treatment for loose stool despite taking Ondansetron and Vomikind syrup",
"src": "Patient: sir i have 1 years baby boy.he suffer vomiting & loose motion in last 2 days.We had taken him to doctor , he prescribed the ondansetron(Orlon) injection,Emigan plus syp, vomikind syp & continue drinking water.but sir after take medicine. vomiting is stop But loose motion still happen.sir please suggest. Doctor: Hello,Your baby is suffering from acute gastroenteritis. Ondansetron and Vomikind syrup are the treatment for vomiting and that responded accordingly. For loose stools, start some pro-biotic such as Eco All or Enterogermina, or Econorm sachet whatever is available. Also, Zinc syrup maintains hydration with ORS and feeding. No need of giving antibiotic.Hope I have answered your query. Let me know if I can assist you further. Regards, Dr. Sachin Kumar Agarwal"
},
{
"id": 89472,
"tgt": "Suggest remedy for abdomen & stomach pain",
"src": "Patient: ive been having sharp pain in my abdomen its in my stomach area, it is sometimes hard and sometimes its soft, i feel like i use the bathroom too much, the pain come and goes in the upper part of my abdomen, i have been wanting to eat raw onions and pickles and peanut butter , its weird cause ground hamburger meat smells nasty and so does chicken but it never smelled nasty before, i get headaches occasionally as well. i dont know what it could possibly be. Doctor: HI.Read the history. The probable cause of sharp pain in the abdomen can be due to gastroenteritis or colitis. The feeling if going to bathroom very often may be due to the same. Get ultrasonography, upper and lower GI Endoscopies, a course of an antibiotics and further investigations of CT scan of the abdomen may help more."
},
{
"id": 126461,
"tgt": "What causes radiating pain around the knee?",
"src": "Patient: I went for an mri today because I was almost positive that I have a blood clot, it was extremely painfull ,still is. There was no blood clot , blood flowed ok. So what is it? It started a week ago. Horrible pain around the knee. Then the pain area moved to 3 or 4 inches above the knee to the inner thigh and the top of the thigh. Some times I feel it around my kidney area,like it is a muscle. With my knee getting most of the intense pain right now. I have no fever or any other symtoms except for being very hard to walk or move around. I am using a walker to get around with lots of pain. I am going to go to the emergency room if things don t gewt better. The thought of all that moving around is not welcome. What do you think? Doctor: Hi, It can be due to early-stage arthritis or ligament related problems. Consult an orthopedician and get evaluated. As of now, you can apply warm compresses and take analgesics like Tramadol for pain relief. Hope I have answered your query. Let me know if I can assist you further. Regards, Dr. Shinas Hussain, General & Family Physician"
},
{
"id": 163280,
"tgt": "What causes formula milk craving in babies?",
"src": "Patient: my baby is 15 weeks old and eats cow and gate formula milk. for the last month he has been eating 8oz bottles which he finishs all of every fed. in the last to weeks i have started giving him stage 2 milk for hungrier babies but he is still eating the same amount. he is now at the point where he eats every 2 to 3 hours and is having a least seven bottles a day or more. he still seems to want more food. what do i do? Doctor: Hi,Do not upgrade formula milk just because baby is demanding more. If she is tolerating well then no need to get worried. Until you feel a rapid increase in weight gain, which is not in healthy limits.Secondly, correct your delution. Sometimes the mother make too much diluted formulas that causes poor satisfaction of baby's hunger. Hope I have answered your query. Let me know if I can assist you further.Regards,Dr. Hina Javed"
},
{
"id": 146868,
"tgt": "What causes tremor in hand and elbow pain?",
"src": "Patient: I have had a tremor in my left hand for several years now following what I remember as a ripping feeling in my shoulder at a gym. Recently I have started to have what some people call a \" rain stick noise\" in the back of my neck, which seems somehow connected to tinnitis. A neurologist who examined me a year ago said my tremorcwascnot consistent with Parkinson's . I also have been having elbow pain on the same side. . I really need to figure out what is causing these things - seems like they might be connected... But I don't know where to start. Help! Doctor: Hello dear,Considering the fact that the Neurologist has ruled out presence of Parkinson disease, the symptoms as mentioned in your post can be attributed to:1. Paraspinal spasm & compression of nerve fibers at the Cervico-dorsal region.2. Inadequate hydration status & electrolyte imbalance of the body3. Dietary deficiencies of certain nutrients like Iron, Calcium, Vitamin D, Vitamin B12, Folic acid, etc. which help in blood formation & nerve conduction.4. Peripheral neuropathy caused due to any fluctuation in blood pressure or blood sugar level.Symptomatic relief can be obtained by intake of nervigenic agents (like Vitamin B6, Vitamin B12, Folic acid) as well as multivitamin & multimineral supplements including iron, calcium & Vitamin D needs to be added in your diet.If symptoms still persist, kindly consult a Neurologist & get a complete Neurological examination done.Certain investigations like complete blood count, serum electrolyte estimation, fresh estimation of blood pressure & blood sugar levels & nerve conduction studies will be helpful to rule out any pathological cause for the symptoms.Wishing you a Good Health.Take care."
},
{
"id": 104313,
"tgt": "Been choking on saliva, have blockage. What is wrong?",
"src": "Patient: For a few weeks I ve been having problems with choking on my own saliva . It will be randomly throughout the day and recently while I m asleep. I literally wake up choking. I m lucky to even force myself to inhale just to cough because it s one of those silent chokes where I literally can t get any air in. Like a blockage. And it really hurts my chest . What could this be? Doctor: it can be sleep apnea dryness of moth blockage of salivary duct or heart problem get all investigations ecg tmt sleep studis and ultrsiund salvary gland"
},
{
"id": 125568,
"tgt": "How can chronic osteomyelitis be treated?",
"src": "Patient: I have since quit all drugs I m 50 yrs old and I look at my self and it s ridiculous . I ve been to the E.R.twice my girlfriend has been in the hospital for. 2, months we both experienced. Severe low back pain she went to the hospital in so much pain they gave her. Ketamine because she was out of control the hospital took her and she has osteomalitis my pain subsided some but now instead of my lowback it re emerged in my hip just to the left of my back bone I have told the E.R. my story and they did catscans. no MRIs and right now I have to stay with a family member for help I have ins.And I m scared I will never ever do it again I have things in my life that cause me to numb the pain but am through and am seeing a physiotherapist but it s like these E.R.doctors are worried about cost to the hospital s I m praying I don t have bone cancer if I knock my self out I can get 2or3 hrs sleep.if me and my girlfriend shared and the pain comes and goes after 5 months osteomalitis would have killed me?Please can anyone advice me I m a good guy I have kids and grandkids I wanna see graduate please any thing to direct. Me I went to neurologist but he needs MRI but that s 2,weeks away and I can t walk thinking about calling ambulance? Doctor: Hello, Chronic osteomyelitis needs proper broad spectrum I.V. antibiotics and bone marrow lavage to remove the infection properly from the bone. Prolonged treatment is needed for weeks to completely remove the infection. I hope this information has been helpful for you. Let me know if I can assist you further. Regards, Dr. Praveen Tayal, Orthopaedic Surgeon"
},
{
"id": 140022,
"tgt": "What causes headache, dizziness and nausea with sound sensitivity?",
"src": "Patient: I often have issues with my sinuses, allergies, and hayfever, accompanied with dizziness, nausea (sp?), and sometimes headaches or veritgo altogether. I find that loud sounds and sometimes bring on, aggravate or cause symptoms to get worse. Is sensitivity to sound a symptom in itself? Or maybe a symptom to an even bigger issue? Doctor: Hello, The symptoms that you have described could be due to an ear problem. Please get yourself evaluated by an ENT surgeon at the earliest. Hope I have answered your query. Let me know if I can assist you further. Regards, Dr. Haleema Yezdani, General & Family Physician"
},
{
"id": 41750,
"tgt": "Suggest treatment for infertility",
"src": "Patient: Need a good PCOS doctor in Hyderabad.Am 35yrs old,5.4 feet,78kgs.Trying to conceive since seven year but not positive results. Have undergone laproscopy but no use... Had been to many doctors but all the more got cheated. Need a good doctor in Hyderabad. Doctor: Hi welcome to healthcaremagic.I have home through your question.As in PCOD polycystic ovarian disease multiple small follicles develop but not a single follicle Develop dominately.I would advise to go for IUIintrauterine insemination, or IVF invitro fertilization. As you had done so many treatment and reports.Hope i answered your question.Would be happy to help you further.Take care."
},
{
"id": 27685,
"tgt": "What causes low grade fever and backache in a heart patient?",
"src": "Patient: My husband had surgery for Afib 2 yrs ago his pulse is 128 he has a headache and is running a low grade fever and bad backache. He also had angioplasty 21/2 yrs ago I checked his pulse he is not does not have an irregular heartbeat YYYY@YYYY Doctor: the pulse you said is high though as you sy its regular , it means that it is waht we call sinus tachycardia , which i believe is secondary yto the fever and backache , which should be evaluated with relevant investigation for the cause of fever and backache . something i may suggest is sonography of KUB for any urinary tract infection , kidney stone or varied causes . if backacke persistant and severe a xray and preferably mri spine lumber for rarer causes which may resulth in low grade fever with backache , if routine instigation as your GP may suggest are normal. however get an ecg done as many a times when the heart rate is fast as human someone can misdiagnose it to be regular ."
},
{
"id": 174332,
"tgt": "How to treat constipation in a 6 months old child?",
"src": "Patient: hello doctor, good evening. my daughter is 6 months old & has been prescribed silybon syrup, carmicide syrup to improve digestion , also for the complaint of constipation. she was Nan-1for last 4months. Now she has been advised for Enfamil A+. how it will work??? plz reply Doctor: Hi . I think you need to start her on weaning foods to improve her constipation. Ragi porridge is a good one to start. Also you can give Rice porridge or kanji. Can add vegetable soup, fruit juice without sugar or little sugar, boiled apple, banana etc. Introduce one new item a week. If on breastfeeding, continue it and avoid powder milk. Changing milk brand may or may not work. It is just a trial. Generally I don't prescribe silybon or carmicide syrup. Also not every child passes stool daily. It varies from child to child."
},
{
"id": 76986,
"tgt": "What causes superficial chest discomfort after quitting smoking?",
"src": "Patient: hi, i spontaneously quit smoking cigarettes exactly 10 days ago because of black spots in my spit. I researched about it and it has to do with my heavy marijuana smoking. I've been smoking marijuana everyday for the past year and a half, about 2 grams of weed a day pretty much. I have been smoking cigarettes for 3 years about half a pack a day (sometimes i smoked less and sometimes i smoked more than so). I do not crave any cigs at all anymore but I am freaking out thinking that there is something wrong with my respiratory system, looking out for any suspicious feels or nudges in my chest and I have felt a few that feel more superficial than lung related. I am 22 years old about 5'5\" and 130 in weight. Doctor: hi,thank-you for providing the brief history of you.A thorough examination of the lungs and musculoskeletal assessment is advised.Since you were smoking in the past and and heavy marijuana smoker as well, due to this, the lung tissue has become weak. Since the body was habituated for 3 years for the supply of nicotine and now sudden stopping will also give some sort of symptoms which you need not have to worry much. If possible take a CT/MRI of the lungs once to assess the lung tissues. Also a pulmonary function test is done to check the lung capacities.I will recommend you to perform regular breathing exercises and respiratory muscle Strengthening exercises which a physical therapist will help you understand and learn.Performing this after an initial Pulmonary function test and later after a month of practice redoing the Pulmonary Function test will help you understand your progress.I have clinically seen cases with such complaints and they respond well to physical therapy. A good amount of people regain their fitness of lungs back after Pulmonary rehabilitation.for any further query/doubts/suggestions you can feel free to contact me. I will happy to help you further from here.RegardsJay Indravadan Patel"
},
{
"id": 61818,
"tgt": "What could lumps and swelling on head suggest?",
"src": "Patient: Hi my husband has lumps and swelling on his head which look a bit like a brain very strange we have looked up a lot of things nothing seems to fit he has had this for a while it doesn t hurt but after shving his head to nearly bald it s really bad what do you think? Doctor: Hi, dearI have gone through your question. I can understand your concern. He has lump on head and looks like brain. First of all he should check for any bony defect. If there is no bony defect then its not brain. It can be due to some soft tissue tumor or skin adenexal tumor or dermoid cyst. He should go for fine needle aspiration cytology or biopsy of that lump. It will give you exact diagnosis. Then you should take treatment accordingly. Hope I have answered your question, if you have doubt then I will be happy to answer. Thanks for using health care magic. Wish you a very good health."
},
{
"id": 224821,
"tgt": "On copper iud, missed periods, spotting, sore and painful stomach. Solution?",
"src": "Patient: Hi I have the copper iud for over 3 years now and had no problems untill recently I missed my period 2 weeks ago in turn I has very very light brown spotting/ discharge . Pain in left side goes from dull ache almost like cramp to very sore. Doctor sent medor smear which all came back clear. I have started spotting again and have slot if pain again. Please help. K Doctor: Hi,In the current scenario, I would ask for a trans-vaginal sonogram along with a blood test for pregnancy to know the status. If pregnancy or any other pelvic pathology has been ruled out, you should test for blood profile as well as hormonal imbalances. Any weight changes, stress and medications are also possible causes for the irregular menstrual pattern. If the IUD is found to be causing it, it needs to be removed and an alternate contraceptive method planned. Hope this helps."
},
{
"id": 8621,
"tgt": "Exposed to sun. Using skinlite cream. Became dark. What should I do?",
"src": "Patient: hello dr i have used skinlite cream for the pass 3 months ,, unfortunately last week i was happend to be exposed in sun during my nss camp.. so that i became dark due to that exposure , so i think i have used skinlite in excess quantity which let me in trouble i have becme so dark after that .. what should i do now ... Doctor: hi Your problem seems to be that of tanning due to excess exposure to sunlight..Firstly, i would recommend that you should not use skinlite cream any longer now...The tanning that has occured doesnt seem to have any relation with the use of skinlite ,,But skinlite has such components that it should be used only for a limited period of time and that too under medical supervision. It is not recommended to be used for longer periods of time... Now as far as the pigmentation is concerned, i would advise you to regularly use a sunscreen (SPF>26) now before any kind of sun exposure.Also you can use a milder lightening agent containing Kojic acid cream and vitamin C on your face twice a day to reduce the pigmentation.. Also take diet rich in antioxidants and Vitamin C (like citrus fruits, salads) Hope it helps Dr Geetika Paul"
},
{
"id": 196996,
"tgt": "Does duralast help to treat erectile dysfunction?",
"src": "Patient: my age 33 yrs. ht 5.5 feet, wt 68kg. now nearly nine months passed after marriage. while during intercourse my penis has become relaxed suddenly and unable to intercourse even a single time. is duralast 30 will be effective for me? is this a phsycic problem? Doctor: helloThanks for query .Based on the facts that you have not been able to have sexual intercourse even a single time since your marriage ( nine months) .In majority of young individuals the cause of Erectile Dysfunction is Psychogenic in origin and mostly due to what is called as performance anxiety Consult qualified Psychiatrist for counselling and treatment .Following measure will help you to boost up your confidence and getting good erection and delay ejaculation. 1) Practice regular exercise for 45 minutes followed by meditation for 1/2 an hour in the morning. 2) Take high protein diet rich in vegetables and fruits and Vitamin A,C,D,E.and Zinc 3)Take anti oxidants like Almonds 5-6 everyday4) Avoid alcohol and smoking..Taking Sildenafil (Viagra) or Cialis as on demand will help to get good hard sustainable erection and to have enjoyable sex.Duralast is Selective Serotinine Reuptake Inhibitor and is generally prescribed for premature ejaculation and does not help in ED .Dr.Patil.."
},
{
"id": 67042,
"tgt": "Can fatty tumour be deep inside the skin?",
"src": "Patient: MY 21 yr old daughter has a lump on her back she went to the doc and getting a cat scan the doc thinks its a fatty tumor I m not seeing where this is anything like the fatty tumors I ve been researching since I m only seeing them on the skin. Can a fatty tumor be deep in your skin? She says its painful. Its been there for 3 yrs and I m worried....ty Rhonda Doctor: Hi, Greetings from HCM. since you have consulted a doctor and acat scan doneyou dont have to be apprehensive .Fatty tumors are usually be lipomas which will not be painful and will not become cancerous These type of tumors are below the skin and sometimes they are not visible externally .If there is associated neural element it can become painful If you are worried you can consult a surgeon and get it removed as it is a simple operation"
},
{
"id": 107264,
"tgt": "Suggest treatment for back pain",
"src": "Patient: 10 days back I started with a back pain went to GP he advised biolfnac..initially thought its a muscle pain applied moov at home..I m an active male in my 50s..Completed the week tab but still the pain remains...Then went to an orthopedic who gav me Vit D injection prescribed tab too to take for 8 wks..Prescription for pain was Hifenac P.for a week twice daily along with a antacid once daily before food..its been over 3 days..pain hsnt reduced and I hv been lying the whole day as sitting on sofa increases the pain..unable to go my regular walks also due to pain n discomfort on the back.Pleae advise how long it ll take the pain to go away so that I can resume my routine activities..Is it anything to worry about...Please advise Doctor: Hi Went through your post and understand your concern as back pain could be crippling sometimes, your orthopedician has done one thing which isn't good and can be dangerously harmful and that is vitamin D injection wkly as much I know each vit D injection contains 600000 units of vit D so you will be receiving a total of 4800000 lakh units daily requirement is 400 it can cause extremely high blood calcium levels and renal failure. I would suggest you don't take any further injections of vit D And you can start sustained release pregablin 75mg once bed time that will help for backache and discuss with your orthopedician regarding vit D so that he doesn't prescribe it to others."
},
{
"id": 70523,
"tgt": "Fairly large pimple or something on the inside of thigh",
"src": "Patient: I ve got this fairly large pimple or something on the inside of my thigh. It is extremely uncomfortable. I currently have it padded with gauze and neosporin. I popped it and there was some blood, and a small amount of pus. What should I be on the lookout for? Doctor: Hi.The proper antibiotics and anti-inflammatory medicines will help to get an early and better recovery. Get a Prescription from your Doctor as antibiotics are a prescription drugs."
},
{
"id": 55934,
"tgt": "What could fatty liver and Calific foci granlues in liver suggest?",
"src": "Patient: sonograhpy report says liver size is normal but is fatty liver. Old Calific foci granlues in liver. I previously was diagnosed with right lower uretric calculi of 7.5 mm 3 months back and started drinking loads of water and yesterdays report showed that the calculi was dissolved(no calculi). But says fatty liver. is this of major concern. my age is 30 M. Doctor: HelloFatty liver may be due to many reasons like obesity,alcoholism,sedentary life,metabolic disorder etc.Calcified foci in liver may suggests old healed calcified granulomatous lesions.These are healed stage and doesn't require any treatment.You may need few more investigations like routine hemogram,random blood sugar,liver function test,lipid profile.Mild fatty liver is a reversible condition.You should increase your physical activity and go for brisk walk regularly.It is good that ureteric calculi has passed from the body.Get well soon.Take CareDr.Indu Bhushan"
},
{
"id": 90242,
"tgt": "Is withdrawal symptoms of niftran and niftas causing abdominal pain?",
"src": "Patient: Hi I am Mala from Mysore I had UTI since 4 years I taken Niftran 100 mg 3 months after the 3 months I am taking Niftas 50mg since 2 months unfortunately I stopped this treatment since one month now i am suffering from mild lower abdomen pain and frequency of urine please please suggest me for this problem Doctor: Hi.Thanks for your query. If the Niftas is not helping and you are getting recurrent problem , you need a thorough investigations to find out the causative problem.I would advise you the following.:::Intravenous pyelography and CT scan of the abdomen particularly for the urinary and genital system. Cystourethroscopy by an Urologist.These is a possibility of a problem like tuberculosis or some obstructive pathology.The correction of the original problem only can give you a permanent relief. This is supported by the :Routine blood , urine test along with Culture and Sensitivity of the urine."
},
{
"id": 157216,
"tgt": "What can a retal cancer patient do to improve the problem with stool passing?",
"src": "Patient: I'M 50 YEARS OLD AND HAVE RETAL CANCER. I HAVE A TEMP COLOSTOMY AND HAVING HAED TIME PASSING STOOLS THERE BIG AND HARD TAKES A LONG TIME TO PASS AND PAINFUL WHAT CAN I DO WHEN THIS HAPPENS TO HELP IT PASS SOME TIMES STUCK AND TAKES LONG TIME TO PASS WITH A LOT OF PAIN BECAUSE OF THE SIZE Doctor: HiThanks for your query.You can try laxatives like paraffin based ones or lactulose, which will make the stools soft so that passing will be a lot easier.Hope this helps.regards"
},
{
"id": 67336,
"tgt": "Suggest remedy for lumps in thigh",
"src": "Patient: Im a male and im 17 iv got spot on my inner thigh its aches at times and when i popped it white sunstance came out but it still felt like lump so i carried on and it started let ingblood out if continued trying to pop it i dont know of its serious or not ? Doctor: Hi,From history it seems that you might be having infected sebaceous cyst giving this problem.consult surgeon and get examined.If cyst is there, it should be excised and get complete sac removed.Ok and take care."
},
{
"id": 25545,
"tgt": "Why do I feel light headed with decrease in BP?",
"src": "Patient: I have had 2 heart ablations within the past 2 months...get lightheaded 2-3 times a day, usually with activity..blood pressure will go high and drop 20-30 points when feel lightheaded..usually have the tunnel vision almost syncope but can sit and work through it.. What can be causing this. I already see a cardiologist and a neurologist who has not ordered any other tests yet. Why is this happening more now? Doctor: Hello!Welcome and thank you for asking on HCM!Regarding your concern, I would explain that your symptoms could be related to orthostatic hypotension. A cardiac arrhythmia can not be excluded either. I recommend performing these tests: - a Head up Tilt test to exclude possible orthostatic intolerance- an ambulatory 24-48 hours ECG monitoring to examine your heart rhythm trends during the day- a cardiac ultrasound to examine your cardiac structure and function- blood lab tests (complete blood count, blood electrolytes, kidney and liver function tests, blood fasting glucose). You should discuss with your doctor on the above mentioned tests. Hope you will find this answer helpful!Kind regards, Dr. Iliri"
},
{
"id": 128479,
"tgt": "Having pain & swelling along the midway of left side & shin area",
"src": "Patient: a plate and 11 screws were put in my left leg 29 yrs ago , i just recently started a new job and after two months on the job i have painful swelling midway along the left side of my shin on my left leg. what could this be and whats the treatment....i had a x-ray done and the plate is secure Doctor: Dear patient ther is possibility of infection. Are u suffering from diabetes? if yes infection is the likely cause. Xray will not show infection. local ultrasound of swelling will show collection if there is infection. Meanwhile start tab cefixime 200 mg twice a day for 5 days. you need to consult orthopaedic surgeon with report."
},
{
"id": 182403,
"tgt": "Suggest remedy for low bite strength of dental bridge",
"src": "Patient: I have a fixed denture with healthy teeth cut after getting an orthodontic treatment for 2 years. The old healthy teeth have been filed to be very short to put the bridges both in the upper and lower jaw. Bite strength is minimum ..what is the remedy Doctor: hello, i went through your query. the bridge you have got done might not be contacting the opposing teeth properly. in scientific terms, it might be in infra-occlusion. Sometimes it will get corrected by itself slowly with the movement of the opposite arch teeth. Otherwise it is better you contact your dentist to get it corrected."
},
{
"id": 160544,
"tgt": "Suggest remedy for acute behavior changes and turning head side to side in 4 year old",
"src": "Patient: My 4 year old turns his head side to side to look out of the corner of his eye. Seems involuntary. He s unable to explain why he does this. It is not constant throughout the day and seems to occur when looking near and far. He did this last year and eye exam was WNL. Also having acute behavior changes, not sure if they are connected. Doctor: Hello, I wish I had video! It sounds like it could be a \"simple tic.\" Children often develop involuntary small muscle contractions that they aren't even aware of. A tic just comes for no reason and goes away for no reason, anytime between a couple of days, to more typically a few months or so. There's not reason to pay attention to it (as hard as that is) or ask him to try to stop it (he may or may not be able to). Other examples you may have seen in other kids is squishing their nose, shrugging a shoulder or super blinking. However, you also mentioned behavior changes - although that is usually not related, any change that's sudden or new or unexplained should be further discussed. Hope I have answered your query. Let me know if I can assist you further. Take care Regards, Dr. Lisa Baker"
},
{
"id": 107621,
"tgt": "Suggest treatment for severe back pain",
"src": "Patient: I have really bad back pain and took Endone with food then Ibuprofen a few hours later but now I feel really sick and think it could be the Swisse mood vitamans with St John's Wort. I ate some more to feel better but I still feel sick. How can I get rid of this sick feeling please? Doctor: Oxycodone Hcl and Ibuprofen, both for pain together should be avoided in my opinion. Excess of Moid tablets or Viramines are not going to help.To my own patients I would say them to gradually reduced Swisse moods and Oxycodone etc and slowly increase Deep breathing, meditation and taking pure ayurvedic Mentat tablets only. It's by Himalaya vo.For back pain, slow incremental daily doses of turmeric powder from 1/2 tsp to 1.5 tsp, over a month, mixed in warm milk or warm water be preferred. Light massage of Rumalya liniment is very helpful too.Hope it helps you a lot."
},
{
"id": 92279,
"tgt": "What could be the reason for having abdominal pain, bouts of fatigue, dizziness, headache and vomiting?",
"src": "Patient: I have been feeling sick for about three weeks. At first I thought it was a stomach flu but it hasn't really gone away. just gotten less severe for a period and then worsens again. I have stomach pain in my upper abdomen a bit more localized to the right. It is accompanied by bouts of fatigue,dizziness, headache and at times severe vomiting. I have also noticed stomach gurgling after I eat Doctor: Hi there,Thanks for your query.You are apparently suffering from Acute Gastritis (inflammation of the stomach with excessive acid formation) or perhaps, stomach ulcers with associated bacterial infection (due to H.pylori) of the stomach.Breath test for H.pylori will confirm/exclude infection. Ideally, you should undergo Endoscopy to see directly, the condition of the stomach inner lining, as well as, ulcers, if any.A course of a combination of anti-acid drug, such as Lansoprazole or Rabeprazole, along with a Prokinetic drug, such as, Metoclopramide or Ondansetron, will give you complete relief. If Breath test for H.pylori turns out to be positive, a 2 weeks' course of anti-H.pylori therapy will also be needed.Consult your doctor and apprise him of my opinion. I am certain that he will agree with me, order required tests, and- based on the reports- will prescribe suitable/modified treatment as advised.If you find my response helpful and informative, do not forget an \u201cexcellent\u201d (5-star rating) to my answer, to ENCOURAGE ALL doctors- engaged in social service- to render sound advice to the FREE queries. Take careDr. Rakesh Karanwal"
},
{
"id": 173678,
"tgt": "What are the side effects of Pentax in shot for 1.5 yr old?",
"src": "Patient: My 1.5 yr old boy got a Pentaxim shot on his thigh yesterday and today his thigh and knee is swollen very badly. I have given him crocin DS to bring down the fever which is not more than 39 degree celsius. How can I bring down the swelling ? Is it normal for the limb to swell up 24 hours post the shot Doctor: give ice fomentation to injection site. yes it is normal for limb to swell up 24 hour post the shot , i generally add syp ibugsic to crocin if swelling in more"
},
{
"id": 37799,
"tgt": "Can flu cause a sore nose with coughing and phlegm?",
"src": "Patient: I have been coughing for more than ten days while taking a generic brand of coricidin cold tablets. Now i am coughing up more phlegm and it drains from my sinuses so much my nose is sore. There is sneezing and I was achy for a few days.. Do i have the flu? Doctor: HelloThank You for contacting HCM.Yes your symptoms are suggesting flu and all symptoms are interrelated with each other. Symptomatic care will help you recover in few days.AVOID:> Chilled water> Drinks> Spicy foodHope this answers your question. If you have additional questions or follow up questions then please do not hesitate in writing to us. Wishing you good health."
},
{
"id": 70679,
"tgt": "What can cause tight sensation in the chest along with shortness of breath?",
"src": "Patient: My husband has been having chest tightness and about 6 weeks ago chest pain. He has had an echo cardiogram and nothing came out. He had one where they gave him liquid through an IV to see about a blockage and they said his heart was strong. He had lung xrays but his lungs are clear. He gets a chest tightening and feels out of breath for about 4 to 5 min and then it releases. Any thoughts on this? Doctor: Hello, As you explain the situation if he was my patient I would recommend him to control the stomach sometimes reflux may be the cause. Hope I have answered your query. Let me know if I can assist you further. Regards, Dr. Jnikolla, Pulmonologist"
},
{
"id": 207489,
"tgt": "Why does mind go blank suddenly and hear loud voices?",
"src": "Patient: This happened once or twice when i was 10, and just happened again recently.What happens it, out of nowhere my mind will suddenly go blank.then for about 4-5 minutes, everything around me seems to be going faster, and all sounds seem to be much louder. When people talk it sounds much more urgent, and even if i lay down in a silent bedroom i can still feel it, that everything is moving faster, and i can hear every little sound louder, then it would stop and go back to normal.What is this? Doctor: HiI had gone through your query.I guess you need to go for investigation.Because og its episodic nature it could be seizure phenomena.EEG means electroencephalogram is require to evaluate this.It could be look like seizure phenomena mostly it can be absence seizure of partial seizure.MRI with epilepsy protocol will be useful.Medicines like levetracitam.valproate etc can be useful once it diagnosed properly.Still have a query then feel free to ask.Happy to help you.Thank you."
},
{
"id": 85329,
"tgt": "What is the benefit and side effect of S26 and lactogen formula?",
"src": "Patient: Hi, my grandson s poo was green with S26 formula but today its yellow like tumeric. could this be the milk ? he is 4 months old and now 5 weeks on S26 after breastfeeding. Mommy also want to change the formula now to Lactogen formula, is this the right formula to change to.? Doctor: Hi, The benefits include any one can give the milk of this formula. It prevents hyperglycemia in big babies and the adverse effects include weight gain and avoiding of mom's milk. I Hope I have answered your query. If you have further doubts, I would be happy to help you. Happy day. Regards,Dr. S. R. Raveendran, Sexologist"
},
{
"id": 110812,
"tgt": "Can UTI and kidney problems cause back pain?",
"src": "Patient: I have been getting constant UTI s since I moved countries. I have now been experiencing back pains for about 4 days now and the pain was kinda bad at first but tonight it has reached it s height and I am in serious pain. I have a UTI ( I have symptoms) right now. If I was not on birth control I would be having my period now. During my period I usually get back pains but never this bad before. I have taken two aspirin and am using a heating pad and the pain has gone down. Is it just back pain, period caused back pain, or kidney infection. ( I was also feeling nausous earlier but not anymore) Should I see a doctor? Doctor: UTI can also result in back pain. if your are near your periods time then it must be from periods. UTI back pain is more middle back and lower side too, if it resemble such pain plz visit your doctor ASAP to avoid further complication. heating pad and meds might give you temporarily relief but not a solution."
},
{
"id": 203315,
"tgt": "Suggest medication for itchiness and patch on scrotum",
"src": "Patient: Hello, I have a dry, itchy patch on scrotum. I also have intense itching, which mainly occurs at night, in my genital area. I also have moist patch of skin on top of penile shaft and red puffy foreskin. The itching is very intense at times. I was wondering what the issue could be and what I can do to treat this. Doctor: Hi,Welcome to Health care magic forum. It appears that you are suffering with the fungal infection,and needs to be treated to the end. I advise you to consult a dermatologist for diagnosis and treatment. I usually prescribe to my patient with such symptoms grisofulvin, multivitamin tablets,and mixed steroid ointment. Wishing for a quick and complete recovery. Thank you."
},
{
"id": 47507,
"tgt": "Suggest treatment to increase HB count in a CRF patient",
"src": "Patient: MY MOTHER IS A CRF patient & her dialysis continue for 2 month 2 times per week,but her HB% day by day decrease.She got 3 bag blood on july 2015 & she take erythropoietin injection 2 times per week & her other investigation report normal now what will i do. Doctor: Hemoglobin not rising in spite of giving EPO injections can be due to the following-1. Iron Deficinecy2. Occult Blood Loss through the GIT3. Blood loss during Dialysis4. Chronic Infections 5. EPO resistance and Bad quality of EPO.Please look into these factors and your Nephrologist or Dialysis Tech will be able to take care of the issue. Recurrent Blood transfusions should be avoided if there is a possibility of going for a Kidney transplant at a later date."
},
{
"id": 206205,
"tgt": "What causes poor concentration and constant crying?",
"src": "Patient: I dont really like talking about my problems, however I feel like I am going to have a breakdown. I am constantly crying for no reason and once I start crying I cant stop. Im having a really hard time focusing, paying attention, and I just want to stay in bed. I have to get up at 7am to take my child to school come home feed animals, household chores, then get ready for work which my shift is 2-11 and then back home to do it all over again. I work usually 6 days a week and I am ready to just walk away from everything. I feel like if someone is to upset me, I am going to loose it ( not physically but definitely in a verbal outburst of some sort). If you knew me you would understand I am the polar opposite. I am always in a great mood always smiling and go with the flow type I am very mellow and laid back. I dont understand why I am feeling like this but this is about the 6 month I have felt this way. Doctor: DearWe understand your concernsI went through your details. I suggest you not to worry much. From the given symptoms, it feels that you are depressed. But diagnosis without proper face to face contact with a psychologist is not advisable.This situation may also arise due to disappointment. Disappointment which arises out of unforeseen life events do bring such symptoms like sadness, irritation, anger, lethargy, lack of interest, lack of appetite etc. Please understand this fact and make sure to introspect. Talking to your friends, out door activities and being creative should give you relief. Find the reason for your disappointment and consult a psychologist if need be. Psychotherapy techniques should suit your requirement. If you require more of my help in this aspect, Please post a direct question to me in this URL. http://goo.gl/aYW2pR. Make sure that you include every minute details possible. I shall prescribe the needed psychotherapy techniques.Hope this answers your query. Available for further clarifications.Good luck."
},
{
"id": 223909,
"tgt": "What causes spotting, cramps, nausea and fatigue when on Depo shots?",
"src": "Patient: Hi I m been on depo for 6 months. I had unprotected sex about a week after my second shot and a few weeks later began having pregnacy symptoms. During the first shot I had only spotting but this time I had spotting, cramps, nausea and fatigue, I recently stopped depo and still having these symptoms. Could I be pregnant? Doctor: Hi Hope this message finds u in good health.I have gone through ur msg and understand your concern.it may be due to some sort of hormonal imbalances or variations in the body,though there may be other reasons as wellNothing to worry about, You should eventually get back to normal.Take multi vitamin and calcium supplements Do consult a gynecologist if symptoms worsenGet back to me for any FOLLOW UP QUERIES anytime.Regards,Dr Mahaveer Patil...(MBBS,MS,Mch)"
},
{
"id": 83712,
"tgt": "What causes weakness after taking Oflomycin?",
"src": "Patient: doctor, i am dsreddy having stomock disorder and fewer for the lasw five days. i was prescribd by mu friend doctor to use oflomycin oz for three days. i used the tablets for three days. but feel weakness and getting cooled some times .so pleasehelp me to use the medicine.sir. Doctor: Hi, Weakness in your scenario is not always due to the tab. Ofloxacin, but can also a effect after diarrhea, which is a common presentation due to dehydration. Below are the tips to bounce back & to overcome weakness : 1. Have plenty of fluids like fruit juices, coconut water, electrolyte powder (& sports drinks) etc to replace the lost electrolytes. 2. Avoid milk & milk-products, alcohol, caffeine for atleast 3 to 5 days after diarrhea . 3. Take probiotics supplements 4. Have good rest. Hope I have answered your query. Let me know if I can assist you further. Take care Regards, Dr. Yogapriya Vasudevan"
},
{
"id": 34281,
"tgt": "How to cure swollen ear lobes after insect bite?",
"src": "Patient: Hello, My name is Samantha. Sometime between sunday afternoon while being outside to Monday morning something bit my ear lobe area. My gland behind the ear is swollen and my earlobe is swollen and red. I took a few antibiotics i had on hand and tylenol. Today is the third day of the symptoms. what should i do? Doctor: Hello,Welcome to HCM,I can understand your concern for insect bite. After reading your query it appears that you have developed insect bite hypersensitivity. This basically caused by hypersensitivity to the insect bite which can cause acute aggravation of the problem and flare up of the lesions.Thank you.As you are not getting relief from the steroids an antihistamine so I suggest you to meet a dermatologist and get your skin examined to get proper best treatment. You can apply steroid creams of higher potency for the early resolution of the lesions. You can take oral anti histamine like loratadine or levocetrizine under medial advice.If you have pain and irritation then you can take a tablet paracetamol or diclofenac.Do not scratch or rub the skin as it can lead to further aggravation.Thank you."
},
{
"id": 152482,
"tgt": "Suggest treatment for recurrent cancerous cells in the face",
"src": "Patient: My mother has cancer that started in her tear glad causing her to loose her eye. She had cancer in most of her lymphnodes and other ares in her face when she had two surgeries to remove the cancer.the healing process took a long time to heal enough in order to have radiation,after two weeks of radiation, then later for the scan it apeared to be gone. A few months later a larger area around the flap that got very puffy with fuid, a biopsy prooved that the cancer was still there. More radiation but little knots that look like blisters started popping up in several parts of her face and spreading more radiation and more blister bumps.it has come to the point that Doctors aren t giving her much time, my question is the cancer coming through, we dont know exactly what to expect when it surfaces, are ther pictures that we can see or find to help us know when it is happening, My sibblings and I are very worried about it! Doctor: Hello and Welcome to \u2018Ask A Doctor\u2019 service. I have reviewed your query and here is my advice. Recurrent cancer cells in the face can be treated by 6 cycles of chemotherapy at 21 days interval. Hope I have answered your query. Let me know if I can assist you further."
},
{
"id": 68704,
"tgt": "What is the treatment for lump in the breast?",
"src": "Patient: Hi, my friend fell against a door corner and developed a lump in her breast. This was 3 years ago and has been getting bigger recently - it is now the size of a golf ball. She experiences pain, this is relieved by ab's. She will not go to the dr's again, at her initial appointment she was referred to specialist who advised it could be cancer. If ab's help the pain, does this eliminate cancer as a possibility? Doctor: welcome to Health care magic.1.No it will not eliminate the cancer. ( it would have ben better if you have mentioned your friends age ).2.It could be the same lesion after a trauma or a new lesion. your symptoms seems like a inflammatory cause or a ductal ectasia.3.If you are my patient i would have examine the lump and ask for a ultrasound breast to evaluate the nature of the lesion (Cystic/duct/abscess), and its extension and origin.4.Some time your friend may need incision and drainage. As antibiotic itself may cause the reoccurrence.5.Suggest to get an appointment and do needful.Hope it helps you. Wish you a good health.Anything to ask ? do not hesitate. Thank you."
},
{
"id": 106793,
"tgt": "How can pain and spasms in the back be treated?",
"src": "Patient: 67 year old male. Slipped, fell down, slammed back against large machinery. 6 days ago. No broken bones, or kidney damage. Injury is on the right side, and when spasm hits, pain shoots almost all of the right side. Received muscle relaxer and pain pills, and ledocaine patches. No pain when I sit, but severe pain (with queasiness) when standing up, picking up stuff, bending. Pain pills help, but patch is useless and so is muscle relaxer. Pain and spasms reduce when I press againt the injury point (a bruise with slight decoloration) when I feel the spasm starting. ER doc gave no info on what to do if pain and spasms occur after meds run out (5 to 20 days) depending on if severity says the same. Advil and Aleve seem best for long term. Thank you. Doctor: Typically the pain from a muscle injury will improve over a few weeks.If it doesn't, MRI of the lumbar spine can be done to look for a deeper injury, to the bones of the spine, or the nerves to the spine.If your pain hasn't considerably improved over a few weeks, discussing with your primary care doctor is helpful to get further testing done."
},
{
"id": 29126,
"tgt": "What causes severe stiffness and soreness in the body while having chronic ITP?",
"src": "Patient: I m 53 years old, female. I have chronic ITP, but it is very stable with no medications. I m trying to lose 30 pounds (5 6, 170 pounds right now). It doesn t matter what workout I do, I am incredibly stiff (muscles and joints) and sore in the hours after. I m trying to exercise daily, so basically, I m just stiff and sore all the time. I take off on weekends, but by Monday, I still feel like my muscles and joints tighten up - they actually feel better when I get moving, but then I sit (or sleep) and I m so tight and sore. I m pushing through, but I m wondering, if I decide to get it checked, what specialty would be my first consult? Doctor: ITP may be secondary and associated with an underlying autoimmune disorders particularly systemic lupus erythematosus (SLE).SLE can cause musculoskeletal manifestations.You may require blood test like Antinuclear antibodies - Best screening test; and Anti-dsDNA -High titers are SLE-specific, to exclude SLE. Consult a Rheumatologist."
},
{
"id": 6297,
"tgt": "Trying to conceive, low sperm count, medicine to increase sperm count",
"src": "Patient: I am 27 years(M) and my wife is 23 years. We ar married for 0ne and half years and are trying to conceive. My sperm count is 43Million. Doctor has recommended fertly for my wife from 3rd to 7th day and have intercourse from 9day. Kindly advise whether there are any problems in this treatment . Whether i need to take nay medicine for increasing sperm count Doctor: I would suggest the following natural methods for a healthy sperm count. 1. Wash the genitalia with cold water twice a day. 2. Avoid woolen underwears. Use cotton underwears. 3. Drink plenty of water. 4. Vitamin E and B complex supplements are beneficial. 5. Regular walk or jog for 40 minutes will improve the blood supply to the lower parts of the body. 6. Avoid alcohol. 7. Avoid cycling or 2 wheeler rides on bumpy roads. 8. Consume lot of fish."
},
{
"id": 92844,
"tgt": "Pain in lower abdomen for 7 years. Recommendations?",
"src": "Patient: i am 28 years old I have always had a pain in my lower left abdomen ever since I was about 21 years old but now its seems worse and I always seem shaky; before or even after I eat. I have seen a doctor when I was 25 and they said it was all the fruit I eat but I have stopped eating as much fruit since then probably none at all now a days and the pain has not gone away? Doctor: TH FOOD WE TAKE SOMETIMES START REACTING WITH BODY PROTEINS TO CAUSE THE PROBLEMS OF NEGETIVE HEALTH AND GIT PROBLEMSI SUGGEST YOU TO GET BLOOD SERUM TESTS FOR SPECIFIC ANTIBODIES FOR MILK WHEAT POTATO CHANA AND OTHER FOODS WITHDRAW FOODS ACCORDINGLY TO GET RESULTS"
},
{
"id": 161531,
"tgt": "Suggest treatment for swollen lymph node in armpit of a child",
"src": "Patient: hi. i have a 8 month old baby boy and a few days ago we noticed a large lymphnode under his right arm pit. we went to the docters but they have no idea what it is. we did blood test a scan and a sonar. but they still dont know. It measures 18mm x 12mm in diameter. it does not hurt him when we touch it but today he started getting grumpy. He s eating and dringking. the colour is a blue purlpe colour. Please answer me back on my email address as it will alow me to sent you a foto of the lymphnode. thank you ( YYYY@YYYY ) Soritha klopper Doctor: Hi, A Lymph node of that size in the arm pitfalls under the category of insignificant lymphadenopathy and need not be worried about, please. Unless he has symptoms like extreme pallor, anemia, bone pains, chronic fever for more than a month, loss of appetite and weight loss - you need not worry, please. Hope I have answered your query. Let me know if I can assist you further. Take care Regards, Dr Sumanth Amperayani, Pediatrician, Pulmonology"
},
{
"id": 173523,
"tgt": "What causes yellow mucous stools with stomach pain?",
"src": "Patient: my son has had a few days with light yellow mucousy stools. he has been having frequent bowel movements within each day...around 8. he has been complaining about pain when he is on the toilet and today i noticed some streaks of blood when i was wiping his bum. he has plenty of energy but has episodes of lethargy. Doctor: It's onset of 'Sangrahani' which is dysentry caused by infection in intestines and accumulation of AAM ( mucous) in intestines.Warm water should be started. The best choice for this is Kutki or Kutaj herb. But it is bitter. So many preparations are available by modifying the palatability.Kutaj syrup or Kutajarishta may be given 8 hrly to 6 hrly as per age. Kutajghan Vati tablets 6 hrly. Lavanbhaskar chooran and Bilv chooran mied equally to be given in curd four times a day. Dose as per age. Any Ayurveda doctor can guide you well. After a week generally mucous cleanses out of intestines. DONT forcibly stop stools if you start this med. It will normalize itself within a week.By acting on intestines, these herbs create immunity too and bacteria do not grow further and day to day possible food infection doesn't take place. Totally harmless unlike antibiotics.If your son cooperates, give him chhachh ( buttermilk) to drink. Better if you can add little roasted cumin seeds powder in it.I not only hope, but am sure that your son will be cured at an earliest. His appetite will increase and he will start gaining strength. All the best."
},
{
"id": 137231,
"tgt": "What is the treatment for the soft tissue injury on the foot?",
"src": "Patient: yes I need to know if I had a soft tissue injury to my right foot particulary my big toe & was outta work little over a month! was given boot & crutches & pain meds as treatment told to elevate soak & stay iff for the duration!! I dropped 20lbs of a case of frozen butter on my foot didn t get swelling & severe pain for three days!! is it a possibility I have a onset of tendernitis!! was told I may have rheumotoid arthritis in that toe today!! I need an opinion!! the injury was in 1999!! ive lived with pain in that toe after injury for years should I address it its getting worse!! cant wear high heals or bend that toe without swelling & pain!! Doctor: Hi there.You can keep the limb elevated. Get an XRay done and report to your Doctor to rule out fracture. You can take pain killer like Ibuprofen 200mg twice a day after meal. Do cold soaks for relief."
},
{
"id": 217786,
"tgt": "Suggest treatment for pain in buttocks and leg",
"src": "Patient: I have had an MRI of my back and have an L5 disc bulge that is causing alot of nerve pain in my buttocks and leg. I coach basketball, and it even hurts when I shoot or blow the whistle too hard. I've had a steroid shot and it didn't help much. What other options do I have? Doctor: Hi, you can go ahead with transforaminal block, decompression discectomy, nucleolysis, piriformis block, per cutaneius laser discectomy. All these options are available with the pain specialists. So consult one in your region."
},
{
"id": 208413,
"tgt": "What causes lower decision power?",
"src": "Patient: one of my relative is delaying every important thing in life . he is 34 ,does wants to take any responsible work, even though marriage. he washes his hands frequently & each time 2 minutes.he afraids what world may think about him ? his decision power is also becomig less. he does not wants to go out also. but he is doing his job satisfactorily . very sensitive in nature Doctor: Hi,I do appreciate your concerns for your relative. From what you have mentioned, your relative might be suffering from Obsessive compulsive disorder (OCD) in addition to other anxiety disorders. OCD is a condition in which a person repeatedly have thoughts regarding something and does some act to undo it. In your relative's case, he might be having obsessions of contamination and washing his hands repeatedly could be the associated compulsion. He might also have other obsessions, which will require detailed evaluation. His not wanting to take decisions and remaining at home might also be related to associated anxiety.I would suggest a psychiatric consultation for him for complete evaluation and initiating treatment. Medications like fluvoxamine would be helpful in controlling his OCD and anxiety.I do hope that I was able to answer your query. Best wishes."
},
{
"id": 31552,
"tgt": "What causes greenish discharge from a burn site on arm?",
"src": "Patient: My daughter has a burn inside her arm from a spray can (yes - self harm). She is currently in a psychiatric clinic and is telling me that the burn is discharging green stuff when dressing changed. Her fingers and arm are swollen and sore. There is no doctor on duty, she has been placed on a list to see a doctor on Monday, should we be doing something before then. They have been putting 'silvervine' on it. Doctor: HI, thanks for using healthcare magicSilver sulfadiazine is the cream used to treat burns. It is an antiseptic that was supposed to help reduce the risk of infection.The green may indicate the presence of a bacteria called pseudomonas. This would require treatment with antibiotics and certain types of dressing.I hope this helps"
},
{
"id": 106142,
"tgt": "Having dust allergy and dry cough",
"src": "Patient: I have suffered from dust allergy and dry cough from last 6 month.Its not continuosly but whenever i am in contact with dust and smokes as well as In AC area.....i have suffered panic dry cough and difficult in breathing....As i have shown to the known Dr. He had given me the Phensydil syp,Doxyfylin tabs as well as Azithromycin....but it,s effective for few days...and agin i have experinced the same problem.....As I have not any asthama or bronchities as per my Family Dr,s checkup but they hints me @ allergic things.Can i get permanant relief from above problem....Please guide me. Doctor: Dear Milind, Welcome to HCM. If You have not already got a blood TC and DC done, you should get it done, just to exclude Tropical eosinophilia. If dust allergy is the cause you may consult a specialist in asthma, who may be having a kit to test for allergies due to various causes. They inject small quantity of various allergens subcutaneously and watch for reaction. Once they identify the allergy causing allergens, they can treat you accordingly."
},
{
"id": 26569,
"tgt": "What causes palpitations in the heart with numbness in the arms?",
"src": "Patient: For the past year I have been having heart palpitations along with other symptoms. I will list them now. -Heart Palpitations all day -Numbness in Arms every morning -Occasional, BAD Vertigo episodes (2-4 times a week) -Flash of light in just my right eye -Painful zit like sores on both legs inner thigh -Weakness of Grip in both hands -Pain in my left wrist and I occasionally cant push my thumb and index fingers together without it hurting a lot (this happens sometimes) I went to the doctor and wore a holter monitor and was told it was benign pvcs and it might be anxiety...but I felt something more serious was wrong. I got a 2nd opinion and got blood work done, and was told I tested postive and negatove for lyme disease. The test that determines if I have it now was negative, and the test that determines if ive ever had it was positive. I had never been diagnosed with lyme...so I may have had it for a while...not sure. Anyway, I took antiobiotics for a month and after 3 weeks on them my palpitations disappeared along with the weird zits on my thighs. I felt a little better and had joint pain still ..but not having the palpitations was a relief. I finished my dosage and about 3 weeks after I stopped...my palpitations came right back full force and the painful small boil/zits came back. I feel like all of these symptoms are related to one another...I dont know what I could have. Please, if theres anybody who can ask me a series of questions to help solve my lifes mystery id be forever grateful. Has anyone had identical symptoms? Or have any doctors on here dealt with similar cases? Im truly desperate. I appreciate you taking the time out of your life to read this. Thank you! Doctor: Hello!Thank you for asking on HCM!I understand your concern and would like to explain that Lyme disease is a chronic disease, which can affect many systems of organs, including the heart, muscles, skin, central and peripheral nervous system etc. . Based on your symptomatology, the fact that you symptoms disappeared while on antibiotic therapy and reappeared, means that this is a chronic infection. Have you been tested for both antigens and antibodies of Lyme disease or only just antibodies?Sometimes the antigens can persist in the blood for years, indicating a chronic infection, even though the antibodies don't show this. I would recommend consulting with an infectious disease specialist for further tests for Lyme disease. An ambulatory 24-48 hour monitoring is necessary to examine your heart rhythm, for possible cardiac arrhythmia. If you were my patient, I would continue the treatment with antibiotics, as it seems that this resistant bacteria is still causing disturbances on your body organ function. Hope to have been helpful!Best wishes, Dr. Iliri"
},
{
"id": 14941,
"tgt": "What could be the rash on back of neck and fingers?",
"src": "Patient: Hi I have a rash on the back of my neck in two spots one near my hair line right in the middle which is small like in size but a bigger bump and then two whole fingers (side ways) below it is a lil bigger patch but low in height ways...it doesnt itch it burns....what is this or what may have caused it... Doctor: Hi,I can understand your concern for the itchy rash on back of neck and fingers. I can make a possibility of Allergic dermatitis. In Allergic dermatitis there are eruptions along with moderate itching and in more aggravated cases there is appearance of more redness which persist for more than multiple of days.The only precaution you must follow is not to touch, pinch pop or squeeze any of them other wise it can get infected and can cause complications.You can apply medium potency steroid cream application of fluticasone cream. You can additionally take antihistamine benadryl or loratadine by mouth. Go for your routine blood test and check for any abnormality in hemoglobin level and thyroid statusTake care."
},
{
"id": 11924,
"tgt": "Is melacare cream safe to use ?",
"src": "Patient: hi bought melacare cream for clearing hyperpigmentation yesterday and was advised by the pharmacist to only use at bedtime , my question is, should i use this cream, is it safe for african skin??? Doctor: MELACARE Cream:Snce it has probably not been advised by a Dr it would be best to start at bedtime on alteernate nights,and notice the response,if there is no marked redness and itcing you could use it every night for 2-3 weeks and then stop for some time.Use sunprotction during day time for next 2-3 months"
},
{
"id": 87002,
"tgt": "Suggest treatment for abdominal pain",
"src": "Patient: hi i am 28 years old and suffer with server endemetriosis i ahve had 4 lapos and lazer surgery and had a sist removed 4 months ago, i was taken back to hospital 2 weeks ago with server lower belly pains bleeding and sickness and was sent home with an out patients app 4 feb, two weeks on and now i am still being sick have very bad pains is my belly and back and am having nose bleeds and my heart keeps on going really fast what shall i do, i am in so much pain i want to go back to the hospital but i feel there is point if they wont do nothing Doctor: Hi.Thanks for your query.Read and understood your history of surgeries for Endometriosis and the complications of pain you have. You have to report back to the Hospital and insist on admission as you have a nose bleed , rushing heart and you may need intravenous treatment of fluids and may be a blood transfusion and re-exploration if need be. If they are not listening you can certainly talk to the in-charge or ask for a referral to the higher medical centers, particularly to the ones where work is done in endometriosis."
},
{
"id": 30999,
"tgt": "What are the precautions and diet recommended for typhoid?",
"src": "Patient: my son 19 years old diagnosed with typhoid with s typhi \"O) +ve 1:160 dil , s typhi \"H\" +ve 1:160 , S.P.Typ: \"AH\" negative 1:20 dil and S.P.typ: 1:20 dil \"BH\" neg today: has been advised to have zenflox plus(OFLOXACIN and CEFIXINE) , PAININ( ACECLOSENAC), PAN D and a to z ( vitamin) and Unienzme . kindly advise re diet and precautions . vegetarian diet . thanks and regards Doctor: Hi, Thanks for posting in HCM. I understand your concern. Diet needs a close attention while treating typhoid fever as it may relapse, if not taken care of. Generally during fever, a bland diet, which is smooth and soothing, is preferred. Care should be taken while cooking food and also choosing foods that are easy to digest. Fluids in the form of tender coconut water, barley water, electrolyte fortified water, fresh fruit juice, vegetable soup, buttermilk and water can be given. Begin with semi-solid diet for a few days and then gradually include fruits like bananas, watermelons, grapes and peaches. As the appetite gets better, semi-solid to solid foods can be given. Boiled rice, baked potato, baked apple, yoghurt, vegetable soup can be given.Gradually, by 3 to 4 weeks, normal diet can be restored.The following foods to be avoided:1. High fibre foods like whole grain cereals and their products like oatmeal, whole wheat bread and raw vegetables in the form of salads2. Vegetables like cabbage, capsicum and turnip as these can cause bloating and gas.3. Oily foods, spices and seasoning like pepper, cayenne and chilli powder to prevent aggravation of inflammation of digestive tract. Hope the information provided would be helpful. All the best."
},
{
"id": 107339,
"tgt": "Suggest treatment for chronic back pain",
"src": "Patient: I live in TORONTO Ontario Canada. I am have chronic back pain that has become unbearable. I am now up to taking 8-10 80 mg. tablets of oxyneo daily...and I am still suffering with excruciating back pain. My family doctor has sent out some referral letters to spine surgeons but they are not taking on anymore referrals. I feel hopeless! Can you help me? Thank you! Doctor: Hi there. Back pain can be really severe and incapacitating at times. Now the first question I should ask you is this: is it a pain which radiates down your thighs and legs, and do you have any associated difficulty in passing urine or stools? if so you may be having features of spinal cord compression and need urgent evaluation by spine surgeon. If you're having back pain alone, then a majority of it can be treated with changing your lifestyle, avoiding lifting heavy weights, or bending down and working. For acute onset pain analgesics may be helpful. Bed rest only for 3-5 days maximum. Prolonged bed rest is actually harmful as it causes wasting of your back muscles which are important stabilisers of your spinal column. So a gradual starting if physio is more useful. There are exercises for core abdominal muscles and paraspinal muscles which can help. Surgery for back pain alone might not give you expected results all the time. it is useful mainly if there is associated cord compression going on."
},
{
"id": 150401,
"tgt": "Pain on hands & buttocks, headache, dizziness. Had spine decompression. MRI done. Advised cervical exercises, massage. Concerned, paralysis?",
"src": "Patient: I have been having a lot of pain that continues to worsen over a 2 year period. The doctors was running test only on my lumbar area because a couple of years ago I had decompression surgery in L4-L5. However I found a doctor who listened to me & ran the test I requested. However he only said come back & see me in a couple of months without any therapy & advised me to go online & look up cervical exercises & make sure I keep my neck lose. In addition he said massages would be good. Please see MRI results below:At the C4-C5 level, a broad based disc osteophyte protrusion appears to be present far lateral to both the right and left and posteriorly extending 3 mm posteriorly and centrally with asymmetric compontents towards each nerve foramen greater on the right than the left of approximately 5 mm to the right. Severe right neural foraminal stenosis is noted with moderate left nerve foraminal stenosis the noted. Contact of the exiting nerve root on the right and left may be present. Anterior cord contact at this level may be present with mild to moderate central canal narrowing to approximately 9 mm.At the C5-C6 level, broad based disc osteophyte spurring is noted that may relate to a broad based disc osteophyte protrusion of approximately 3 mm centrally. Moderate to severe left neural foraminal stenosis is noted with moderate right neuroforaminal stenosis. The anterior thecal sac is thinned but the anterior cord is not obviously contacted. The exiting nerve roots may be affected bilaterally particularly on the left.At the C6-C7 level, a centralprotrusion is noted of 4 mm. Moderate left neural foraminal stenosis is noted with mild right neural foraminal stenosis noted. Mild central canal stenosis is noted with the central canal narrowed to 9 mm. Anterior cord contact may be present. This protrusion descends slightly centrally.My concern is no one has treated me & the pain is so severe that just going to pick up a few things at the grocery store puts me in a lot of pain. While trying to shop I have pain in my feet, hands, and backaches within 15 minutes on my feet.In addition I still work but am in constant pain. The pain in both hands, pins & needles in my buttocks, headaches and the most recent added symptom is dizzeness! When I get home I soak in hot water then go to bed to get relief!Should I be concerned that my doctors has not put me into therapy? Can I end up with permanent paralysis? What should I do now?Thank you.BD Doctor: Hi, Thank you for posting your query. After going through your MRI cervical spine report, it seems to suggest a moderate to severe degree disc prolapse at multiple levels with compression of the nerves at these levels. This could result in neck pain as well as arm pain. In addition to physiotherapy &neck exercises, I would advise you to use cervical collar (soft) and use medications such as pregabalin or gabapentin to reduce the nerve pain. Please get back if you require any additional information. Best wishes, Dr Sudhir Kumar MD (Internal Medicine), DM (Neurology) Senior Consultant Neurologist Apollo Hospitals, Hyderabad, My personal URL on this website: http://bit.ly/Dr-Sudhir-kumar My email: drsudhirkumar@yahoo.com"
},
{
"id": 90380,
"tgt": "Need medicine for stomach pain",
"src": "Patient: I have stomach pain in the centre of my stomach around the naval area above and below this area, have had it for 3 days, thought it was my IBS but have taken ibuscopan which usually clears the pain. But this is having no effect. I cannot sleep now with the pain it is worse after I eat Doctor: Hi ! Good evening. I am Dr Shareef answering your query.With the severity of the pain you have narrated which is disrupting your sleep, I would not advise you to seek help over the net, but to get yourself assessed in the ER of the nearest hospital. Till that you could go for a proton pump inhibitor, and an anti spasmodic drug for a possible symptomatic relief.I hope this information would help you in discussing with your family physician/treating doctor in further management of your problem. Please do not hesitate to ask in case of any further doubts.Thanks for choosing health care magic to clear doubts on your health problems. Wishing you an early recovery. Dr Shareef"
},
{
"id": 70057,
"tgt": "What are the causes and treatment for lymph node under armpit?",
"src": "Patient: i have one swollen lymph node under my right armpit. i have had it for two days now. its not painful till i touch it. i am worried about the cause(i have read all sorts of reasons behind it on the internet such as hiv, cancer and many). please help!! Doctor: Hi.Internet spread more problems than the real knowledge. As per your history , what i can get is simply this is a infected boil or a lymph node. Take a course of antibiotic and anti-inflammatory medicines under your Doctor's supervision . Everything will be alright."
},
{
"id": 137909,
"tgt": "What are the side effects of Pregalin?",
"src": "Patient: Hello DR My mother is 82yrs .Last fifteen days she is feeling acute pain in her left leg from thie to toe .From friday 4th July Dr has prescribed PREGALIN X 75 twice a day with Anaflan.Taking PREGALIN she is feeling drowsiness and uneasyness.Is this medicine effect? I also want to know this side effects.I also want to know for what purpose thi medicine used? Doctor: Hello,Welcome, and thanks for sharing your concern I went through your query, and I feel, you should know that this medication is used for neural category of pain, and it has a side effect of being a sedative. Other side effects areIrritability, feeling numbness,increase appetite, constipation etc but sedation is most common. if you are having trouble taking medication, its not worth the trouble, stop itI hope my advice would have been useful, in decision making regarding your treatment, still if you have any clarifications or doubts feel free to contact back.Thanks."
},
{
"id": 175387,
"tgt": "What is the treatment for loose motions?",
"src": "Patient: my one month 12 day baby was suffering from cold & cough.we went 2 a child splist who gave Azithral Jr,he became well but began to discharge watery stool 5-6 times a day.Again we met the Ped & he gave Folcovit distab.but it didn't work, then he gave Oflomac but still it is not working,what should we do now? Doctor: Hi...Thank you for consulting in Health Care magic.What your kid is having is not a diarrhea and is only a gastro-colic reflux. It is quite common for babies of this age group t pass small amount of diarrhea or loose stools soon after feeds. This need not be treated as diarrhea and especially antibiotics are not indicated.Unless the kid's having low urine output or very dull or excessively sleepy or blood in motion or green bilious vomiting...you need not worry.Hope my answer was helpful for you. I am happy to help any time. Further clarifications and consultations on Health care magic are welcome. If you do not have any clarifications, you can close the discussion and rate the answer. Wish your kid good health.Dr. Sumanth MBBS., DCH., DNB (Paed).,"
},
{
"id": 107888,
"tgt": "Suggest alternative medication for back pain",
"src": "Patient: my husband has been taking aurobindo percocet 7.5-325 for back pain with good results..we went to have a new rx filled and the pharmacy only had alvogen brand, this is not working for him...can we safely double the dose until we see the dr? Can we take 15mg-325 3 time daily safely? Doctor: Dear. you can tell him to take an extra pill but no more than 4 a day for few days until you get the regular pill. it is important to avoid that due to higher doses is acetaminophen can damage his liver.Dr Sara"
},
{
"id": 27197,
"tgt": "Is the practice to give sotalol for cardiac life support?",
"src": "Patient: MY doctor percribed Sotalol 80 mg at home with out being monitored for 3 days in a facility with staff trained in advanced cardiac life support.. I had a defibrillator inplanted on 8-28-2013 and put on Carvedilol 12.5 mg, Lisinopril 2.5 mg and colpidgrel 75 mgThen on 12-8-2013 the defibrillator went off(shocked me 3 time in 10 minutes), went to emergency room and was totally checked out.Then on 12-12-2013 my doctor called and put me on the Sotalol with out being monitored.Is this practice ok ? Doctor: Hi though it's started at indoor facility, because you had defibrillator already placed, it is safe to start you on sotalol at home, because in case of any rhythm disturbance, the defibrillator will help you out, at the same time the rhythm can be know through interrogation. So don't worry about the same. However I would like to point out carvedilol and sotalol are same group, though started for you for different indication, do discuss with your doctor on follow up, as I will suggest titration of carvedilol so you don't have drug interaction. Regards Dr Priyank Mody"
},
{
"id": 113104,
"tgt": "Have back pain and panic attacks causing sleep problems. Took Hydrozine with no effect. Remedy?",
"src": "Patient: HI, IM HAVING ALOT OF PANIC ATTACTS, I CANT SEEM TO RELAX NO MATTER WHAT I DO SUCH AS, RELAXING, YOGA , BREATHING , IT INTERFERS WITH MY SLEEPING, IM HURTING ALL OVER SUCH AS MY LOWER BACK, I HAVE TAKEN HYDROZINE 100 MILOGRAMS BUT IT DOES NOT SEEM TO HELP AT ALL,IS THERE A STRONGER DOES OF IT, I ALSO TAKE THE CAPSULS IF THAT MAKES A DIFFERENCE. THANK YOU Doctor: Hello. Thanks for writing to us. The pain in your lower back can be related to anxiety and stress. This is not necessarily associated with a back pathology. Most of the times taking an oral muscle relaxant helps in taking care of the symptoms which will need a prescription. I hope this information has been both informative and helpful for you. Regards, Dr. Praveen Tayal drtayal72@gmail.com"
},
{
"id": 75321,
"tgt": "Suggest remedy for chest problem",
"src": "Patient: I have had a PICC line in for 10 days due to malabsorbtion in small intestine. Yesterday I started feeling tight chested and then proceeded to vomit all night. I feel like I got run over by a truck and just want to sleep and have someone breathe for me. What could be going on? Doctor: Hello and thank you for asking.As you explain your complains and based in what you have undergone (PICC line) my opinion is that this felling of breath and vomiting are not related with lung problems.It is better that the PICC line to be controlled again probably with an abdominal ultrasound to see what is going on.Thank you Dr.JolandaPulmonologist"
},
{
"id": 119316,
"tgt": "Suffering from anemia, hair fall, bruising easily. Taking irospan. Low iron levels. Treatment?",
"src": "Patient: i had anemia my doctor says it s fine now ,i also had rny surgery in 2005 i want to know how i can bring my ferritin level up it was 0.9 and after weeks of irospan it got to 17 but i feel that i still way to low my doctor don t agree how can the numbers be such a wide range 0.8 - 250.0 whose to say that i feel fine when mine is below 20 my hair is falling out and i bruse easy,and i m so tired no matter what i tried to tell my dr. from what i read most dr.s like to see it at 100 what do you think.sorry but i don t have any money at all i had to stop working due to being so sick ,thank you anyway Doctor: If you had a RnY surgery you are not able to absorb iron taken by mouth (most iron is absorbed in the duodenum, which is bypassed in a Roux en Y surgery). Ferritin is still low, but the best test is to get your iron saturation. You also need your hemoglobin checked. You will probably need iron infusions regularly until you get to normal levels, and then just every once in a while. Also make sure you get your vitamin B12 shots, and other vitamin supplements."
},
{
"id": 63310,
"tgt": "What causes small lump in the pelvic region?",
"src": "Patient: Hi, I m a 36 yo male, 5 10 , 175 lbs. I have no serious allergies and am generally healthy. Today I noticed a small ache in my right pelvic (inguinal) region, and found a small lump upon inspection. Aside from masturbation, I haven t been sexually active in months, so I doubt it s an std. Should I worry? Doctor: Hi, dearI have gone through your question. I can understand your concern. You may have enlarged inguinal lymphnode. It can be due to reactive hyperplasia, tuberculosis or lymphoma. Your masturbation has no relation with it. But any kind of infection can cause reactive hyperplasia. You should go for examination. If needed go for fine needle aspiration cytology or biopsy of that lump. It will give you exact diagnosis. Then you should take treatment accordingly. Hope I have answered your question, if you have doubt then I will be happy to answer. Thanks for using health care magic. Wish you a very good health."
},
{
"id": 100521,
"tgt": "Suggest treatment for bronchial asthma",
"src": "Patient: Hi I m Rosita from the Philippines i ve heard a lot about MX3 but wanted to hear directly from you the effect of MX3 to a patient with diabetes,fungal infection in and out of the body system, bronchial asthma, enlarging prostate( Male70 years of age) and for me and my sister ages 59 and 62 years of age is suffering heaviness of the head and dyspepsia Doctor: HIWell come to HCMIn my opinion the over all symptoms may be related with increasing of acid level in gastric region, some this increase level of acid could trigger asthma symptoms and others too, first thing is to just low down the stress level and try Tab Omeprazole 40 mg once I day, hope this information helps."
},
{
"id": 22465,
"tgt": "What causes severe palpitations of the heart?",
"src": "Patient: Hello, I am a 22 year old female and have been having palpitations/arrythmias for the past 4 years, not frequently. however, in the past 4 months i had a terrible palpitation that made me feel extremely faint. then a week ago i had left arm tingles/pain near my hand and a minute or so following that i had a heart palpitation... then a fast heart beat for about 12 hours and felt fine. then 4 days later i had another episode and have had a fast irregular heart beat, high bp for me 140/80 just relaxing and tingling, numbness sensations in my hands and feet that come and go for the past 5 days. I have been stressed with biochem research and a new hostessing job, fathers aortic occlusion surgery, and my boyfriends depression (thats bounced back to normal)..... however i did take plan B about 2 weeks ago and had a light period after 2 days of taking it which was blackish in color. i have a cardiologist appt in 2 weeks and a gp appt in a couple days. i went to the ER because my symptoms were lasting too long. the ekg and bloodwork were all normal. Doctor: Hi,This can be arrhythmia like psvt or atrial fibrillation, where your heart rate suddenly goes up and BP falls. As you mentioned heart rate was irregular, then there is more likely a possibility of atrial fibrillation. You will need evaluation like ecg, thyroid test and holter monitoring. You should also get one hemoglobin levels done. So visit a nearby cardiologist for these investigation. Also, to abort such episodes you should try to cough hard. Till the time your evaluation is complete, you can ask your local doctor for tablet Ciplar.Hope I have answered your query. Let me know if I can assist you further.Regards,Sagar Makode"
},
{
"id": 1653,
"tgt": "What are the chances of getting pregnant after novarel injection?",
"src": "Patient: Hi i am 27 years old and recently was given clomid due to the fact that I have Pcos days 2-6 and went to see my doctor on day 12 and she said that I had 4 Follicles on my right side which was 23.87x14.96mm, 25.24 x 18.39mm, 18.84 x 17.56mm and 12.53x17.86mm. I was given Novarel injection at 8pm on Day 12 and told to have sex days 12-15 and to start Crinone on day 16. What are my chances of getting pregnant? and could all of my follicles be released with the HCG shot? and all be Fertilized? Doctor: Hi, there is a good chance of getting pregnant. But as you had three matured follicles, there are chances of release of eggs from all three, which can lead to a twin or triplet pregnancy. Hope I have answered your question. Regards Dr khushboo"
},
{
"id": 186459,
"tgt": "What causes sensitivity in lower left side of mouth?",
"src": "Patient: July 12th I had all 4 of my wisdom teeth taken out...Then This past Monday I had to go back to the dentist cause of cold sensitivty on lower left side...He said it was due to a cavity which he filled...But the sensitivity is still there and the pain is getting worse...Could this be my gums where my wisdom tooth was. Doctor: Hi! Welcome to Healthcaremagic.I read your query. You didnt mention if you have sensitivity in one tooth only or entire arch. Also if it is sensitivity to hot or cold?If your dentist told its due to filled cavity and its getting worse, I suggest you to go to your dentist again and have a check up and x-ray done.If required, get RCT done.If the filling is fine, look for any abrasion of teeth near gums. This also leads to sensitivity, initially to cold. Use Sensodyne toothpaste in case of minimal or no abrasion for 2 weeks. In case of large abrasion, you may need filling.All this requires examination by a dentist.I hope the answer helps you. Thank you."
},
{
"id": 87436,
"tgt": "What causes right abdominal pain after gall bladder removal?",
"src": "Patient: I had my Gallbladder removed 2 weeks ago, I have been suffering from right abdominal pains for 2 days. Pain seems to increase in the afternoon. I went to accident and emergency where they told me I had a bad infection (UTI) do you think that is the case as I feels teh pain is going right through me to my back? Doctor: yes ,UTI can cause pain if its causing infection around kidney sometimes stones can also cause such pain radiating to back ,pain because of gall bladder is very unlikely since its almost 2 weeks .get usg kub to rule out infection around kidney /stones drink plenty of fluids cranberry juice /syrup alkasol can help in reveling UTIto review with reports if required"
},
{
"id": 106437,
"tgt": "Should we go to ER if 13 year old boy accidentally takes 2 allergy pills and a heart blood pressure pill?",
"src": "Patient: should we go to ER if 13 year old boy accadently takes 2 allergy pills and a heart blood pressure pill? should we go to ER if 13 year old boy accadently takes 2 allergy pills and a heart blood pressure pill? Doctor: Probably not, but you can call poison control to be safe."
},
{
"id": 109758,
"tgt": "How to treat lower back pain?",
"src": "Patient: Hi , I just received a report from an xray I had on 6/27, because I was having back pain, however ,when I read it it said there were atherosclerotic calcifications within the abdominal wall, how common is this? and should i be worried about it, i have polycycstic kidney and liver, and 4 years ago had half my liver removed, could that have caused this issue? Doctor: Atherosclerotic calcification is common in elder age group. As your half of the liver removed before 4 years. You should do ultra sound and bone scan.So after doing this test you should consult your family physician.Get well soon.."
},
{
"id": 82687,
"tgt": "Suggest treatment for sjogrens & borderline lupus",
"src": "Patient: I have been diagnosed with sjogrens & borderline lupus. I have pain everyday... pain in my spleen, kidney & liver. I get alot of chest pain. If drink 3 litres of water the pain will lessen & sometimes go away. I get alot of chest pain & burping..I don t even know if I can call it burping...it is painful & loud. I believe I have secondary hypothermia...I get so cold my body turns blue...I take a very very hot bath & I can feel my body tempurature coming back to normal. Are these common symptoms of sjogrens? I don t know what to do to help myself...pls pls help. Doctor: for sjogren, you need to go for lip biopsy and ENA profile to confirm itif it is confirm sjogren- biologics is the only answer"
},
{
"id": 183446,
"tgt": "Suggest treatment for headache and pain in jaw after getting root canal treatment",
"src": "Patient: Post root-canal by 10 days, still in a ton of pain with deep jaw ache, metal - chemical taste, more pain after eating. Have crown scheduled for 2 days from now but don't know if I should proceed with remaining problems from root canal. This root canal has taken place over the course of 8 weeks in 4 sessions due to calcification build up they injected a medicine and made me wait to finish it just a week ago Wed. Pain management is non-existent. Ortho won't prescribe any more pain meds. I am on my 2nd course of steriods and of antibiotics. This is causing me severe headaches, pain radiates into the ear and forehead, some throat issues, inability to sleep, etc.. What is the best course of action - go through with a crown, get it pulled or go back to ortho to get another assessment of the root canal job? I am sick of this nonsense. Doctor: hellooo...read thru ur query..i must say that first of all dont get panic...pain after few weeks of rootcanal treatment is normal....its because of the healing procedure.....and no dentist will do crown till ur pain completely gets over..so dont worry about that...but be careful while u bite with that side(avoid biting with that side)..mettalic taste can be due to temporary filling getting disintegrating in saliva..so nothing to worry...in my view i suggest you to go to dentist take for an xray if there is any abscess beneath the root is present can be detected...and if nothing works only as a very last resort extraction have to be done...do salt water gargle regularly.....go to dentist...finish the treatment...be cooll...be happy..happiness can do wonders for our minds...thats best medicine...hope ur benefitted something from this reply..have a healthy day!!!"
},
{
"id": 114141,
"tgt": "Sharp pain in lower back. All my blood reports are normal",
"src": "Patient: Hi, a little over two weeks ago my lower back suddenly started hurting, no sore muscles, just a sharp pain when doing just about everything in lower back. it feels like a knife stab in middle, lower back, a hand or so above tailbone. it just happened..any thoughts? i m 6 2 , 215lb, VERY muscular , 11% bodyfat, cholesterol, triglycerides, all that stuff is normal. i m not diabetic. just a 19 year old guy wondering why the heck my back is hurting. in the past if something hurts, even severe, a few days and its gone, or dull. any help on this please? Doctor: welcome to healthcaremagic you are having some problem in lower spine consult orthopaedic surgeon without delay and get checked and investigations done to get proper treatment take rest and avoidexercises till you get consultation hope you be all right"
},
{
"id": 67215,
"tgt": "What causes small purple lump under breast?",
"src": "Patient: I have a small purple lump under my breast. It appeared about 7 years ago, when I was in high school. My doctor said not to worry that it was common and it ll go away, but she barely looked at it. It s gotten much bigger. My mom passed away from ovarian cancer 2 years ago. So i might just be paranoid, but I figured it would hurt to ask! Would you know what it could be? Doctor: Hi,Any lump in the breasts should not be neglected.Through manual examination of both breasts is required.Mammography will give clear clue about your problem.Consult Gynaec and get examined.Ok and take care."
},
{
"id": 19394,
"tgt": "Suggest treatment for upper left side chest pain when under stress",
"src": "Patient: i get sharp upper left side chest pains when I get stressed, smoke, and sometimes randomly. smoked for 23 yrs a pack a day. they last about 2 to 3 seconds but sometimes sharp enough to grab my chest. im also experiences tingling in my left shoulder blade that last about a minute. My resting heart rate also sits at 110. Doctor: welcome to hcm...pain lasting for 2 or 3 sec is not cardiac disease ..it is not cad ..coronary artery disease..but as you get pain also when on stress and you are smoker ..please get ECGand chestxray and BP measurements ...please cut down on smoking and decrease weight if you are overweight...if all tests come normal please undergo stressechocardiography ..."
},
{
"id": 171215,
"tgt": "Suggest remedy for nose injury in a 5 year old",
"src": "Patient: My daughter got injury of her nose .she if 5years old ,when I visit to ENT specialist he said that, her nasal septum deviated,but she is to small for operation so we can t operate her now because of facial growth will be interrupted, and he advised some nasal drops and anti allergy syrup.sir can u tell e what can I do,during sleeping time she is very irritated,and try to open her nose with finger.when we examine her left nostril their is sweeling ,or growth . Doctor: Hi, You should continue the nasal drops and anti allergy syrup. You also need some more precautions like avoid cold drinks, avoid ice creams. Surgery should be done when she will be around 8 years. I hope this has helped you. Take care."
},
{
"id": 188265,
"tgt": "Persistent throbbing pain after teeth filling. On antibiotics for infection. Pain radiating from teeth to gums",
"src": "Patient: Hi, about three weeks ago i had fillings done on my teeth. Within a week of this procedure, I began experiencing persistent throbbing pain. Returned to my dentist and where it was confirmed that I had an infection. Was put on antibiotics for approximately 8 days. Everything was back to normal, until a two days ago leading up to last night, I experience the same kind of pain, was unable to sleep. I'm not sure if the pain is directly related to gum or my fillings as the pain radiates to other teeth and the gum line. Doctor: Hello, Thank you for contacting us for your problem. If filling has been done in the tooth and after that the pain started it means that there is infection in the tooth, which your dentist also confirmed but this infection will not only be relieved by antibiotic, you need to go for a root canal treatment in the tooth. this pain will only be relieved by ROOT canal treatment."
},
{
"id": 205403,
"tgt": "Suggest treatment for panic attacks,anxiety and pressure in chest",
"src": "Patient: I have having what I would call panic attacks, especially while sleeping. Wake up with shortness of breath and sweating. Freaquently, I have sharp shooting pain under my shoulder blade on the right. It s is constant, but shooting pain. A month or two ago I was urinating blood. That has passed. Althought it seems to come and go. Today, as well as the anxity and the pressure in my chest, the pain under my shoulder blade is shooting pain Doctor: hi,good eveningI would suggest you will start tab. paroxitine(12.5 mg) at night dose and benzodizepines whenever u get stressed and in night also to get better sleep."
},
{
"id": 52694,
"tgt": "Are right-sided abdominal pain and weight loss symptoms of gall bladder stones?",
"src": "Patient: When I was in treatment for breast cancer 10 years ago. Scans showed I was ( loaded)with gall stones. Their words not mine. I occasionally had pain ,right side,. But since I lost 40 lbs. Not so often. Keeping away from fatty foods . Trigger. Question do they go away or disappear on their own. Doctor: Hi and welcome to Healthcaremagic. Thank you for your query. I am Dr. Rommstein, I understand your concerns and I will try to help you as much as I can.Your symptoms may be suggestive of gallbladder stones so you should do at least ultrasound or CT scan to evaluate it more accurately. This is caused by stone in bile duct and is very common condition. Stones are see more commonly in obese people and those with diabetes. You need to change your dietary habits first. Need to avoid fried food, carbonated drinks, coffee, alcohol and spicy food. You should eat more milk products, vegetables and boiled food. If there is no improvement on lifestyle or dietary changes, then surgery is recommended. Other causes include pancreatitis or pleuordinia and should be considered if gallbaldder stones are ruled out.I hope I have answered you query. If you have any further questions you can contact us in every time.Kindly regards. Wish you a good health."
},
{
"id": 61547,
"tgt": "Is a lump behind the right nostril a cyst requiring a CT scan?",
"src": "Patient: I have a hard lump behind my right nostril for a few months now. At first I thought it was a pimple, but the more i touched it, the bigger it grew. Now that I've left it alone, it just simply exists and slightly distorts my nose symmetry. 2 dentists have said that it is unrelated to any dental problems. Is this just a cyst or should I get a CT scan? Doctor: Hello,I have gone through your query and I can understand your concerns.Probable diagnosis: Sebaceous cyst.Differential dagnosis: cutaneous lipoma/dermoid cyst/boilstreatment:surgical excisionAll the symptoms and signs you have mentioned suggest the possibility of a sebaceous cyst.Some other condition like dermoid,boils and cutaneous lipoma have to be ruled out before making a diagnosis.There is absolutely no need for a CT scan for this kind of cyst.Definitive treatment is by surgical excision of the cyst.Excised specimen have to be sent for biopsy to confirm the diagnosis.Hope this information help you.Regards.Dr.Shinas Hussain"
},
{
"id": 77861,
"tgt": "Suggest treatment for low hemoglobin and pressure in chest",
"src": "Patient: My mother has developed low hemoglobin, pressure in upper chest and abd,Loss of appetite, 9 lb weight loss (3 months), acidosis smelling breath or \"rotten tooth\" smelling breath and sudden orange/yellow color outlining all of her teeth. Cardiac concerns have been ruled out. ??? Peptic/bleeding ulcers??? Any thoughts??? Doctor: Thanks for your question on Health Care Magic. I can understand your concern. Yes, possibility of slowly bleeding peptic ulcer is more in your mother's case. Bleeding from peptic ulcer is common cause for anaemia (low hemoglobin). She is also having pressure in chest and abdomen, so possibility of hyperacidity and ulcer is more. So better to consult gastroenterologist and get done upper GI (gastrointestinal) scopy for the diagnosis of this. If ulcer is there than endoscopic treatment can be done. Ask her today avoid stress and tension. Avoid hot and spicy food. Avoid large meals, instead take frequent small meals. Take proton pump inhibitors and prokinetic drugs. Don't worry, with all these, she well definitely improve. Hope I have solved your query. I will be happy to help you further. Wishing good health to your mother. Thanks."
},
{
"id": 214847,
"tgt": "Abdominal pain going towards hips, knees and back during periods. History of 6cm fibroid and novasure ablation done. Natural remedy?",
"src": "Patient: will microgestin can help with lower abdominal /pressure pain which occours mostly 3-4 days during my monthly cycle either due to 6cm fibroid or nova sure ablation sucrssfully done about 2 years ago,no bleeding,shooting pain go to my hips to knees and all around back for those 4days.what is NATURAL cure,otherwise iam healthy 46 years old . Doctor: Hello , 1. Switching to organic foods By doing so you will avoid ingesting pesticides which are known to cause fibroids 2. Fresh fruits and vegetables Eating a wide range of fruits and vegetables (at least 5-7 servings a day) can help to fight fibroids, as can 3. Water As a general health rule drink at least 2 liters of filtered water daily.This will help flush toxins out of the body. 4. Fibroid friendly food Vegetables which have high fibroid-fighting properties are kale and cabbage. Include these as much as possible in your daily intake 5. Red and processed meat Avoid animal fats and processed meat as far as possible. Red meat is particularly bad for fibroids and must be avoided at all cost. Regards."
},
{
"id": 42226,
"tgt": "Suggest treatment for infertility",
"src": "Patient: Dear doctor i having pcod and was detected by oct. 2006 and was on treatment with krimson 35 after taking the medicine my periods were regular later after 6 months went irregular. Later got married in may 2009 at that time also i had irregular periods and was again on krimson 35 since we were planning not to have kids for next 2 years. Now i have completed 2 years of marraige and from may 2010 we had unprotected sex but everytime i dint concieve. I met a doctor near to my hopuse who prescribed melmet and benforce. Continued that medicione for a year and now again nothing has happend. Again met my family doc she has prescribed me with Letzol and glycoment. I have completed the 5 day course of letzol but now iam experiencing a small sharp pain in left side of stomach. Yesterday i stopped the medicine and we havent made any sexual contact. Can i concieve successfully?? Iam too worried Doctor: Hi, Thanks for writing..I understand your concern..After taking letroz from 2nd day of the period for 5 days and follow up with follicular study on day 11 and 13 to know the size of the follicle. When the follicle size reaches 20mm, then planned intercourse around 3 days of ovulation. In the post ovulatory period, progesterone should be taken for 10 days. On stoppage of drug either you will get periods or will be pregnant. This can be repeated for 6 cycles.. IUI can be done to accelerate the success rate. Hope i have answered your question.. Good day."
},
{
"id": 91731,
"tgt": "Why do I have pain on the lower right hand side of my stomach?",
"src": "Patient: I have pain on the lower right hand side of my stomach. It hurts when I breath and when I sit and when I get up. The pain on the scale of 1-10 is about an 7 or an 8. I am sitting in chair now and it hurts. My stomach is also bloated. I have had this pain for about 2 to 3 days now. The pain is more intense today then it was the past 3 days. I have been taking the supplement CLA for weight loss. I have taken one capsule a day but I have not been taking it regularly for the past two months. I take it with 1400 mg of Fish Oil. What is wrong with me. Could I have pancreatic stones or gall stones. I do not have a fever and I am not vomiting. Doctor: HI. Gone through the history you have provided . Odd to know there is no fever , no vomiting in spite of pain in right lower abdomen.7 to 8 ON scale is severe enough. It hurts when you breath- there are few things like apendicular lump, kidney problems and inflamed gall bladder that can cause pain like this on the right side of abdomen.The best way to get the clinical examination by a Surgeon and ultrasonography to find the cause. Pancreatic stones do not present like this."
},
{
"id": 148332,
"tgt": "What is the reason for lightheadedness , pressure in head, sensitivity to light, sound happening only in the evening?",
"src": "Patient: i have been feeling light headed for two weeks almost. it happens every evening while im sitting and watching TV alone, i get this undeniable apathy/ lightheadedness. i eat a normal dinner. the creepiest part is the fact that when im sitting, my right leg feels like the circulation isnt working, i feel pressure in my head, sensitivity to bright light and loud sounds. i dont really notice this during the daytime. Doctor: Hello dear,The symptoms as mentioned in your post can be due to:1. Deficiency of certain nutrients like Iron, Vitamin B 12, Calcium, Magnesium, etc.2. Any fluctuation in blood pressure levels3. Altered blood sugar levels4. Inadequate hydration & nutrition status.5. Peripheral neuropathy.Symptomatic relief can be obtained with nervigenic agents (like Vit B6, B12, Folic acid) & multivitamin & mineral supplementation.Investigations like complete blood count, estimation of blood pressure and blood sugar levels & doppler study of the right leg will be helpful in clinching the diagnosis.There is no need to worry, you will be fine.Meanwhile, maintain adequate hydration, take a healthy balanced diet & avoid stress.Wishing a good health.Take care."
},
{
"id": 152160,
"tgt": "Suffering from Disc Caudal Migration in Lumbar region",
"src": "Patient: hi, my wife is suffering from a case of caudal migration of a disc in the lumbar region (between L4 and L5) due to which the nerve roots in the spinal cavity/canal are getting supressed and she is experiencing a shooting pain in her left leg. She is unable to walk much right now, has got limited mobilty (has to hop most of the time or lie down in bed). Is this an orthopaedic case or a neurological one? Is surgery the only solution or there are alternate ways to remedy the ailment? My wife is 28 yrs old, weight around 90 Kgs and has a history of Hypothyroidism and PCOD Doctor: hi welcome to health care magic When the spongy discs between the vertebrae of the spine in the lower back degenerate, they become weak. This leads to a narrowing of the spinal column, which is what spondylosis refers to. The lumbar region bears the brunt of most of the body's weight, so with time everyone's vertebrae degenerate. Even if you consult an orthopedic consultant for your problem, after investigations, he will tell you that there is no definitive treatment for spondylosis and treating it symptomatically by prescribing analgesics is what they will do. But a good doctor will also tell you that ONLY exercise during pain free period can enhance your chances of reducing the existing lumbar spondylosis and revert it back as far as possible. Well, t is absolutely true that effective stretching exercises like Yogasanas help one get rid of lumbar spondylosis. you can rely upon homeopathy for help! Homeopathic medicines work wonderfully o alleviate the lumbar pain and numbness if associated. The homeopathic medicines act from inside out and heal the dislocated discs and give permanent relief when coupled with yoga. Consult an expert homeopath and get rid of your lumbar spondylosis soon! homeopathy will not only treat disease it will treat person as a whole so your wife will get rid of thyroid ,pcod and lumbar pain simulaneously. good luck"
},
{
"id": 115322,
"tgt": "What does calcium level of 8.9 mg/dl and phosphorus level of 7.5 mg/dl indicate?",
"src": "Patient: I am 23 yr old female 65kg/5 feet 2 inches with a history of TB and gall bladder stones.My calcium level is 8.9 mg/dl and phosphorus level is 7.5 mg/dl.My LFTs are normal.I am thinking of a diagnosis of hyperparathyroidism?? I have taken a test for PTH now and am awaiting the result. Doctor: Hi,Thankyou for your quiry.As parathyroid is a gland that regulate bone miniralization.And when ever there is decreased plasma calcium , parathyroid hormone from gland is released to increase vit D active form formation by kidney , which in turn increase calciumabsoption in intestine and vice versa.Normal range of calcium at this age is 10 to 12 mg and phosporus level normaly is 2.4 to 4.1 mg per dl.That means you have low calcium level and high phosphorus level both factors stimulate parathyroid hormone so hyperparathyroudism.Since all these minerals regulation can be also effected by kidney problem.I would suggest that you should also get kidney profile and urine calcium and phisphorus tests done first and discuss with nephrologist .hopefully my answer will be helpfull for you.regards,Dr.Maheshwari"
},
{
"id": 55616,
"tgt": "Could mild abdominal pain with nausea after gallbladder surgery suggest more blockage?",
"src": "Patient: I have mild abdominal pain with some nausea but it seems to be getting worse. I had emergency gall bladder surgery in January with many stones and one in the bile duct. I am concerned that I may have some more blockage. Would that be a possibility? Doctor: HelloIt is too early to conclude blockade again.Generally blockade in CBD does't recur after removal of gall bladder(cholecystectomy).Pain may be due to many other reasons like hepatitis,acidity,renal colic etc.You may need proper clinical evaluation and investigations.I suggest ultrasound of abdomen immediately.Investigation like MRCP may be advised after evaluation.You may also need routine hemogram,random blood sugar,liver function etc.Proper treatment depend upon findings.Get well soon.Take CareDr.Indu Bhushan"
},
{
"id": 9214,
"tgt": "Suggest cure for dandruff and eczema",
"src": "Patient: I m 18. Since I was younger I ve had dandruff, but after being diagnosed with exima a few months ago, I have had a few sores around my ears and top forehead hairline, that are just pussing it seems like. What are they? theyre constantly leaking clear liquid but will go away and come back Doctor: Hi,You seem to have seborrheic dermatitis. The lesions might occur on scalp,around ears,forehead,face,axillae and groin areas.You consult dermatologist for firm diagnosis. I would like to suggest as follow..- antibiotics- antifungal tab- antihistaminics- ketoconazole shampoo- antibiotic cream on oozing lesions- avoidance of stressI hope this would help you...Thanks.Dr. Ilyas Patel MD"
},
{
"id": 35122,
"tgt": "What is the treatment for dog bite?",
"src": "Patient: hi, my son was bitten by a house dog but they didnot given any injection the dog. my son age is 8 years. and i have given all the doses suggested by the doctors. still i am confused about food. shall i give nonveg dood to my son.pls clear my doupts.thankyou, maivizhi.v. Doctor: Greetings! The post-exposure prophylaxis is a three pronged approach. All three carry equal importance and should be done simultaneously as per the category of the bite 1.Management of animal bite wound2.Passive immunisation: Rabies Immunoglobulins (RIG)3.Active immunisation: Anti-Rabies Vaccines (ARV) As far the food is concern , yes all type regular food that you give him can be taken! there is no restriction to food as far as dog bite is concerned! Regards!"
},
{
"id": 86303,
"tgt": "What causes pain in my stomach and pelvic region?",
"src": "Patient: Hello, I am a 62 year old women, About 4 days ago, I started getting an achey pain in my lower right stomach / pelvic region. It has gotten worse in the last 4 days. Especially if I sit too long, when I get up it hurts with every every step I take. I think it is a UTI and have been drinking cranberry juice for a couple of days now, but it doesn t seem to be getting better. I am about to go to the local clinic, but can you give me any advice or ideas what it might be. Doctor: Pain in lower abdome and pelis in females may be due to some adnexa or ovarian pathology or due to appendacitis. UTI generally cause burning micturation or fever rather than severe pain.These type of features require through clinical evaluation including USG abdomen and routine blood and urine test.Thanks"
},
{
"id": 180991,
"tgt": "What does an ulcer on the roof of the mouth indicate?",
"src": "Patient: Hi The roof of my mouth (about 1\" above the teeth) has been sore for about 2 weeks. It feels like when you eat something too hot and sparsely have burned it. Any idea what that could be also my throat gets parched a few times during the day. Never used to get that. Thanks Doctor: Hello, I had gone through your question I can understand your concern. Oral ulcers mostly on hard palate are traumatic in nature and happen because of the hard food. But in your case it is associated with dry mouth, oral ulcer with dry mouth are fungal infections. Fungal infections are also associated with burning sensational pain when hot, clod or spicy foods are taken. You must apply CANDID gel which is an local anti fungal (ketokenezole) 4-5 times daily. In case of pain relief LIGNOCANE gel must be applied 10 minutes before meals and wash your mouth then and eat. In case the ulcer is not healing for more then three weeks consult a oral Medicine spacialist and get it evaluated. Hope my answer will help you, you can ask further if you need. Regards Dr Muzaffar"
},
{
"id": 126537,
"tgt": "What causes cramps in the arms?",
"src": "Patient: HI , I VE BEEN SUFFERING FROM THESE CRAMPS IN MY ARMS. IT BOTHERS ME MOST WHEN I M SLEEPING AND WHEN I M DRIVING . THE PAIN-NUMBNESS RADIATES FROM MY FINGER TIPS TO MY ELBOW, MOSTLY. I HAVEN T BEEN ABLE TO SEE A DR. YET BUT I WANT TO KNOW IF THERE IS ANYTHING I CAN DO TO HELP ALLEVIATE THIS IN THE MEANTIME . THANKS. Doctor: Hi, As of now, you can take analgesics like Acetaminophen or Diclofenac for pain relief. If symptoms persist better to consult an orthopedician and get evaluated. Hope I have answered your query. Let me know if I can assist you further. Regards, Dr. Shinas Hussain, General & Family Physician"
},
{
"id": 48253,
"tgt": "What does small kidney size indicate?",
"src": "Patient: both kidneys are small in size and shows lobulated contours parenchymal texture notrmal cortico medullary differentiation well maintained. sir may know is there any serious problem because the size of the kidney reduced by 1.5 cms in 10 days my mothers age is 82 years Doctor: HelloGenerally kidneys become small due to chronic renal disease resulting form chronic hypertension,diabetes mellitus etc.Since cortico-medullary differentiation is well maintained,I don't think kidney size is a matter of concern.Difference of 1.5 cm in 10 days may be due to technical error.It is not expected to decrease without any systemic illness.She may need complete renal renal function test(RFT).If RFT is fine then it is not a matter of concern. Take CareDr.Indu Bhushan"
},
{
"id": 165777,
"tgt": "Suggest food diet for baby during summer season",
"src": "Patient: Hi doctor, i am residing in dubai. My female baby is 17 months olds. She is so active, had food properly. But for the past 1 week she stopped having water, milk, juice.. all liquid foods. Having only solid foods like rice. I am worried, being in dubai, summer started, baby must have more liquid items. do advice me how to handle her. Doctor: Hello dear!i can understand your concern.You can make a diet chart of the required calories at this age for your child.That should contain a balanced diet with frequent intervalslike at 8 am in the morning give her egg,sandwich or bread with tea/orange juice whatever she likes.at 10 you can give her fruits rich in water like watermelon or melon/orange can also give cucumber etc and likewise can add different items after intervals containing foods which are rich in calories combined with fruits/vegetables having more water.Similarly you can give her different flavoured milks,you can add different flavors yourself like horrlicks,chocolate,cardamom etc.you can give her plain water too when she is playing around in attractive water bottles or you can make simple plain water attractive by making fruits like strawberry,kiwi,apple slices in water,in a clear bottles.These things attract the child,they drink this super amazing water and also healthy as it has antioxidants.Make one bottle of one litre water with half apple cut in slices,one strawberry and one kiwi in clear bottle.Make her drink this whole day in intervals.you have to keep this water a little cool (not ice cold but you cant make it warm as it will change the taste).Hope it will help.Wishing your child good health :-)"
},
{
"id": 33696,
"tgt": "Suggest remedy for itchy breast inflammation",
"src": "Patient: Lately my breasts are itchy, especially around the nipple area, and on one of them there seems to be a slightly inflammed montgomery gland with redness around it...is there a natural remedy? I don't want to take antibiotics. I also have an antibiotic cream that has some cortizone in it as anti-itch for exema that was on my arm, can I put this on it? Doctor: Hi,It seems that there might be having some local skin infection like dermatitis giving this problem.due to itchinmg there might be having some bacterial infection as well.Apply the cream you are having for few days.If pain or swelling and redness persisted then you might require one course of antibiotic medicine.Ok and take care."
},
{
"id": 5880,
"tgt": "Unable to conceive. Tests all normal. Possible reason?",
"src": "Patient: Hi sir, I am an 24 yrs married woman,from last 2 yrs I am trying 4 a baby but won t conceive even for once. We went to a gynecologist and gone through varies tests,except HSG and all the reports are normal.During my child-hood days I used to eat slate pencil , so I have a doubt that my childhood mistakes is cursing my life.....so please help me to find the problem and guide me. Doctor: Hi Thanks for your query, Eating slate pencils causes many problems like anaemia, dizziness, decreased appetite and abdominal cramps. It has been associated with women's health. But as your all results are normal ( yours 'sand your husband's) , i dont think there is any problem. However HSG is necessary to see the status of your tubes. Some time subfertility remains unexplaiined in many cases. So keep hope and get proper treatment. Best of luck Hope it helps.."
},
{
"id": 74983,
"tgt": "What causes cough and difficulty while breathing with a normal chest X-ray?",
"src": "Patient: 51, female 5'10\" 200 lbs, Fell on my right side, right arm under ribs when I fell. Wed, Thurs, am blood in my mouth. Only lasted two days. Had chest x-rays. No Fracture. Very, painful, to breath, cough, lay down. Was told it was a bruise inside, or bone bruise. No outside bruising. Am taking Humira 40mg injections too Doctor: Respected user, hi I evaluated your query thoroughly.*cause of difficult breathing is associated with costochondritis , irritation of underlying nerves & reflux coughing irritating bronchi & alveoli in presence of normal X-ray.*Along with Humira do following for faster recovery : - deep breathing exercises ( will seem to be difficult in beginning but very important & result giving ) & YOGA at home . - remain ambulatory . - when resting keep your body in semi-reclining position with 3-4 pillows support over back. - walk in fresh air for rejuvenating your lungs with fresh dose of oxygen daily - avoid smoking / alcohol / abuse substances if any. - have adequate sound sleep of 8 hours. - relieve stress / anxiety - have diet rich in protein , iron , anti-oxidants , vitamins , minerals to boost up your immunity and getting better faster . - maintain your hydration , weight , BP , Blood sugar levels . - Local analgesic sprays also help better. - refrain from sleeping where you apply weight over the affected ribs. - avoid weight lifting / strenuous work. - additional analgesic anti-inflammatory agents can be taken from your Dr. - occasional cases require further detailed elaboration as MRI also Hope this clears your doubts . Thanks for using Health Care Magic & reviewing my answer. Wishing you fast and speedy recovery from the same. Regards dear take care ."
},
{
"id": 225988,
"tgt": "Can Hydrochlorothiazide be the cause of a false POSITIVE home pregnancy test?",
"src": "Patient: I'm sure these are silly questions, but I'm grasping at straws here. Can hydrochlorothiazide 12.5 mg be the cause of a false POSITIVE home pregnancy test? I took a Clearblue pregnancy test this morning and it seemed to be missing that all important word...\"not\". Uh oh. I am 46 years old. I am not menopausal. Could my age also influence a false positive? Thank you Doctor: Hi,Welcome to Healthcaremagic,After going through your history,This medication would not affect a pregnancy test.If you have tested at least two weeks after a missed period, and you had unprotected sex during this period, then visit Gynecologist and confirm it with blood test Beta HCG level which is 99.9% accurate and a ultrasonography may be done to confirm.If you do not want to continue pregnancy medical and surgical options to terminate are available.your Gynecologist would guide you for what is best for you.First we need to exclude pregnancy than other causes of amenorrhoea can be analysed and investigated if required.Hope this helps you.Take care.Good Luck.Dr.Akhilesh Dubey M.D."
},
{
"id": 45868,
"tgt": "When can i go swimming after PD catheter out due to kidney transplant?",
"src": "Patient: Hi, i had a very succesful live doner kidney transplan on 1/12/10. I was on dialysis for a year b4 that. I am getting my PD catheter out on 20/1/11. How soon after that can i go swimming, ive missed it so much as havent been able to do it cause of catheter. Doctor: Hello and Welcome to \u2018Ask A Doctor\u2019 service. I have reviewed your query and here is my advice. You can go for swimming now, it\u2019s already been years and you can do all your routine activities. Make sure that the water is clean and free of contamination as it can cause infection. Hope I have answered your query. Let me know if I can assist you further. Regards,\u00a0\u00a0\u00a0\u00a0\u00a0 Dr. Shinas Hussain"
},
{
"id": 54724,
"tgt": "How long can someone live with a portosystemic shunt?",
"src": "Patient: My 58 year old husband has Hep C 1a, cirrhosis, portal shunt and diabetes. His shunt was put in last month. He has had o bout of HE last week. He gets his blood drawn weekly for the HE and anemia. His doctor does not really say too much other than we will do this next or try that. My question is how long can someone live with this portal shunt? It seems like such an unnatural treatment to me. Is this something where someone can live 20-30 years or should we be keeping an upbeat yet cautious approach? I do not want anything to happen to him but I need some realistic answers. Thanks. Doctor: HelloThanks for asking query on HCM. As I understand your husband has HCV related chronic liver disease and diabetes, and has undergone TIPSS- portal shunt. But what you have not mentioned is that -1. What was the indication for TIPSS? 2. What is his present MELD score?. I can answer your question if I know answers to these questions. Best would be to get his MELD score, which would written in his discharge card or can ask your doctor. I would say you need to adopt cautious approach as shunt is usually done for advanced liver disease patient except in a patient with some rare causes. Take care."
},
{
"id": 218096,
"tgt": "What could be cause of intense throbbing pain in head?",
"src": "Patient: I get intense throbbing pain in my head. It started about two weeks ago. Now I have them at least twice a day. They come all of a sudden and last about a minute. It feels like the throbbing and pain surrounds my head. What could be causing this? Doctor: HiThanks for writing in.There are various causes of headache:-- stress- hypertension- migraineIf your pain is unilateral & aggravates on exposure to light,then it can be migraine.You need to rule out the proper cause by consulting physician.Go for proper investigations.Take proper rest.Take paracetamol & diclofenac combination in case of headache.Do not take tension.RegardsDr. Neha Sumra"
},
{
"id": 90740,
"tgt": "Why do I get stomach ache while playing computer games?",
"src": "Patient: i always get stomach ache when im playing an FPS game on the computer, i go to the toilet to take a crap, but sometimes there is nothing coming out and i feel better, and i get back to my game but after 10~15 minutes i get it again. I dont get it when im in school, watching tv, exercising, or doing other stuff, im 13, 168cm and 60kg Doctor: Hi.Dear Child of 13.This is due to a condition called Irritable Bowel Syndrome.Talk to your parents. Ask them to take you to the Specialist called Gastroenterologist for a check-up and treatment. I would advise you to stop playing FPS games and any other activity which increase this problem.You are too young to have this."
},
{
"id": 209980,
"tgt": "Suggest remedy for mental problem causing memory issues and confusion",
"src": "Patient: hi myself ravikant i have problem of mental ihave taking teatment of psychitry in baramati and aiso problem of digestion tab ; solaze50 due to allopathic teatment from 1year iwas became careless and long time not remember email id YYYY@YYYY Doctor: Hello Mr.Ravikant, You have mentioned that previously you have taken treatment for psychiatric problem.It will be very usefull to have much more details regarding the same. Considering your problems related to memory disturbances and confusion, it can occur in many psychiatric disorders. It can happen in depression wherein u have pervasive sadness of mood, loss of interest in routine activities, disturbed sleep and appetite, decreased activities etc. and other major cause for memory disturbances is dementia wherein you tend to have loss of memory, geographical disorientation, difficulty in remembering d names of your relatives, etc. And as you haven't mentioned about your age it's difficult to tell what exactly you have.. I hope to hear from you with much more details.RegardsDr. SushmaConsultant psychiatrist"
},
{
"id": 219623,
"tgt": "Why are progesterone and HCG injections needed for fourth pregnancy?",
"src": "Patient: hello, this is the begining of my 6th week of 4th pregnancy after least menstrual period . married for 9 years i had a missed abortion of 5 weeks after 1.5 years of marriage and then after one year again concieved and a healthy baby girl arrived. then again after a gap of three years I gave birth to a healthy baby girl aged 3.2 now. so this is my 4th. doc prescribed two injections per week ,daily ascard and follic acid tabs. my questions why is it necesacry to take these progesterone n hcg injections this time. regards Doctor: Hallow Dear,Progesterone is a hormone required for stabilizing the pregnancy. This hormone is prepared in large amounts by placental tissue or the precursors of placental tissue. Before placenta is formed, it is the chorionic tissue which produces hormone hCG in large quantities to support the other progesterone producing tissues to produce progesterone. Hence, if there any signs suggesting that the pregnancy is at risk, particularly early pregnancy, either hCG or Progesterone or both are given to stabilize the pregnancy. The signs demanding for these are mainly bleeding in the first three months without pain in the abdomen. Sometimes, if there is bad obstetrics history with previous abortions in first three months, these medicines are given as a precaution. If you do not have any such history of symptoms, you may not require Injection of hCG and/or Progesterone. However, please continue to take Folic acid which is very vital for the growth of the baby. I hope now you can take an informed decision. Dr. Nishikant Shrotri"
},
{
"id": 11796,
"tgt": "Having pigmentation on face due to exposure to sunlight. Remedy?",
"src": "Patient: Hello Dr, Greetings and happy New year. My wife is 28 yrs with one baby of 7 months. She had pigmented facial skin diffused stage only in forehead , mid nose and either side. Slight below the nose and not below the mouth. The pigment type is I fig of this site. WWW.WWWW.WW She is normal in health but at the age of 12 had TB. After DOTS, she was OK and lung is very well now after X ray as I know. She has not taken any oral drugs but some creams at the past. No allergy , no pain. But due to expose of sunlight, skin becomes dark. I am clinical psychologist and parasitologist. I am doing PhD in health science. No other physical and anatomical problem she had. Sometimes constipation only. Would you please give me proper suggestion. Thanking you, Doctor: hi..thanks for your query.. From the clinical picture described, I feel your wife has something called Melasma.also known as Chloasma faciei,is a dark skin discoloration. Although it can affect anyone, melasma is particularly common in women, especially pregnant women and those who are taking oral or patch contraceptives or hormone replacement therapy (HRT) medications.The symptoms of melasma are dark, irregular well demarcated hyper pigmented patches commonly found on the upper cheek, nose, upper lip and forehead. Melasma does not cause any other symptoms beyond the cosmetic discoloration.Treatments to hasten the fading of the discolored patches include: Topical depigmenting agents, such as hydroquinone Chemical peels. Laser treatment. It is extremely essential to use a broad spectrum sunscreen throughout the day , with repetitions every 3-4 hourly,not only when outside, but also even indoor or while cooking. Hope this helps..Take care!"
},
{
"id": 91266,
"tgt": "What is causing severe stomach pain and little bleeding?",
"src": "Patient: My stomach was hurt so bad that i couldnt stand straight up for a day throwing up that morning before all the pain. It hurted to sit up lay on my side and it felt tight to even use the restroom. the next day it still hurt just not as much and now im bleeding a little whats going on.!? Doctor: Hi and welcome to HCM. You need to visit emergency room and do further tests including gastrosopcy. it can be sign of severe gastritis or ulcer and it should be ruled out. most commonly it can pass on conservative therapy but sometimes more agressie approach is neccesary. also,you need to take antiacid medicines and change dietary habits.Thanks for the query. Wish you good health. Regards"
},
{
"id": 103720,
"tgt": "Numbness and tingling in arms, chest pain. On candida diet. Why breathing issues?",
"src": "Patient: I have been having breathing problems for a year now. I have seen an ENT, cardiologist , family doctor, allergy asthma doctor. I haven t had anything help or feel like I get any real answers. I need help in where to go now. I have a word document on my history of my treatments and feelings. No one seems to care. I truly feel that I don t have stress or anxiety. I have had these problems ever since I left South Korea for the military last year. This is my history: 1998 \u2013 Strep 2008 \u2013 Oral Thrush Treatment Aug. 2011 \u2013 Pneumonia Oct 2011 \u2013 March 2012 Fatigue that progressed to extreme fatigue. Which meant that I felt tired by mid-day which progressed to me being so tired by time I came home that I just needed to sleep. Irritable/Grouchy. Chest pressure, numbness and tingling in the hands, arms feel heavy and hard to lift above head, difficulty breathing, sinuses, eye pain, headaches, trouble concentrating/memory (fog), mucous, started to lose voice throughout the day. March 2012 \u2013 Treated for sinusitis . April \u2013 Diagnosed Asthma and given Advair 250 and Ventolin . Today I don\u2019t feel like the Advair and Ventolin are doing much. Suggested to see a cardiologist. Negative results found. Cardiologist hinted at the fact it might be stress. June 2012 - Seen a holistic Dr and was told it could be from a stomach bacteria and treated me with oils a few times. Felt ok for a few days, but the symptoms returned. Eliminated dairy and it lessened the amount of mucous I had. Aug 2012 Started to see Dr. Fost who tested me for allergies and asthma. No allergic reactions . Were any of the tests for food allergies? Had a sinus CT scan. Went to see an ENT with the CT Scan and was told that there is nothing wrong with my sinuses except a little deviated septum and a little tightness in my right nostril. ENT suggested it might be anxiety. Chiropractor said that I\u2019ve developed carpal tunnel in my hands/wrist (right wrist primarily). Sept 2012. Throughout months of my own research and trial and errors at home, I\u2019ve found that my recent ability foul odors (without an odor being present) is due to a stomach bacteria. I went back to remembering that I had oral thrush that never fully went away. This was found to be candida. I spoke with a Blue Cross RN about candida who confirmed that it is a real thing and cause issues like mine. RN also asked about how it could be from GRD. For almost two weeks now I\u2019ve been cleansing, using probiotics, and doing the candida diet. A lot of symptoms feel like they are going away. Sept 27 2012 - Feeling of a little fatigue returning and heavy shoulders. Sept 28 2012 \u2013 Visiting asthma and allergy specialist. Diagnosed with vocal dysfunction cord and not asthma. Taught breathing exercises and taken off the advair and ventolin. For two weeks feeling ok. Twice I was able to get myself to cough up phlegm at night when I has trouble breathing. Oct 8 2012 \u2013 Slipped on diet and had a regular burger and fries with no cheese. Within a few minutes after, I felt extremely tired and took a nap in the car. Don\u2019t believe it\u2019s vocal dysfunction cord. Oct 9/10 2012 - Numbness in tingling in right arm all night. A couple of times for a few minutes I felt it in my left arm. Chest pain in upper right area near arm pit. In the morning on the 10th the pain moved to the center of my chest. I make sure that don\u2019t sleep on my side anymore to make sure that it is not from sleeping on my arms. Confirmed that it isn\u2019t from sleeping on my side. Worried it\u2019s a heart disease still or lung infection/bacteria. Currently, I get numbness and tingling in right arm. Chest pain that moves from upper right area to center of chest. A pain in chest when at work and think about doing a cell search (I\u2019m a correctional officer). Shortness of breath and lightheaded after walking up and down stairs more than a few times. Still on candida diet afraid to eat anything else. I don\u2019t feel like I worry or have stress. I have gotten rid of all possible stressors. Doctor: THESE ARE FOOD ALLERGIESALL TESTS ARE NORMAL BUT I THINK ANY FOOD ARE CAUSING THISGET BLOOD SERUM TESTS FOR SPECIFIC ANTIBODIES FOR MILK WHEAT RICE EGG POTATO CHAna nuts eggstart by withdrawing milk and diary from food completely if you start getting response in 3 wk you also get done for wheat and othr food you take daily and since longi have studied history an different systema are being involved slowlyyour tst will show one to 3 foodseliminate completely and get results in 3 wk to 3 monthsi have treated thousands of ptscontinue supportive therapy till you get it"
},
{
"id": 69221,
"tgt": "What causes lump on collarbone near to neck?",
"src": "Patient: I have a painful lump above my collarbone near my neck, it hurts when i touch it or move my neck and inside there is a little hard knot. The swelling has been reducing day by day, but I am concerned about what this is? There are no signs of redness or bruising I did get a flu shot and the TDAP shot Doctor: Hi,The lump you are talking appears to be enlarged lymph nodes. They are usually when there is infection in the surrounding area. It will be painful and slowly as the infection subsides with or with out treatment, it would disappear.If you want you can get a test called as fine needle aspiration cytology done. It will confirm or rule out anything else if you are worried about.If you have any further queries, happy to help you again."
},
{
"id": 64856,
"tgt": "What does a lump at the base of skull indicate?",
"src": "Patient: I discovered yesterday a lump at the base of my skull on the right side. The lump is like one would get if I walked into a door or got hit in the head. It is painful in that spot and to the touch with sore muscle pain in shoulder and neck. I am a diabetic, but I think this i a lymph node swelling. What can I do to get rid of this lump and stop the pain Doctor: HI,Dear,Thanks for the inquisitive query to HCM.1-I studied your query in depth.2-In my opinion , this painful lump at your skull base is most probably -a lymphadenitis with infective pathology like boil in the scalp-as you are a known diabetic,which appears to be out of controll.3-I would advise you to consult physician who would check the CBC, fbs,pps,and would treat it with antibiotics and NSAIDs,3-a- if no relief, and the pain and swelling in the skull base increases,then sugeon would be needed to drain the infective focus , to get the disease and diabetes-under controll.4-Hope this would solve your query .5-Wellcome for more queries to me On HCM.6-Thnks once again for your inquisitive query.Have a Good Day..!!"
},
{
"id": 80470,
"tgt": "What is the treatment for shortness of breath?",
"src": "Patient: Lately, I am becoming allergic to almost everything. I would have rashes to every medications being prescribed to me by doctors. I am reacting to soaps, toothpaste, eye drops, clothes, gloves, a lot of foods. I am having shortness of breath, even if I am just watching TV, sitting , surfingthe internet, almost very thirty minutes, then will break after 10 mins, then short of breath again. Am I having an inflamed immune system ? Is there a cure for this? What should I do to get back to my previous health? This started 6 mos ago. Doctor: Hello dear, thanks for your question on HCM. I can understand your situation and problem. Since you are having strong allergic history, possibility of bronchial asthma is high.Bronchial asthma is allergic disease , which causes bronchospasm and thus breathlessness.So we need to rule out asthma first. So consult pulmonologist and get done1. Clinical examination of respiratory system. 2. PFT ( pulmonary function test ). PFT is needed for the diagnosis of asthma. It will also tell you about severity of the disease. And treatment of asthma is based on severity only. You may need inhaled bronchodilators and oral antihistamine and anti allergic drug. So consult pulmonologist and discuss all these."
},
{
"id": 70097,
"tgt": "What causes hard lump on my wrist?",
"src": "Patient: Hi, I discovered a hard lump on my wrist. Should I be concerned due to the fact that I had breast cancer and had 24 lymph nodes removed , with 4 being positive for cancer as well . all from the my right arm were i found the lump. Thank u for your help. Doctor: Hi ! Good morning. I am Dr Shareef answering your query. Even though it could simply be a ganglion ( a benign lesion ), with your kind of history, I would suggest you to get it assessed by your treating oncologist to have a definitive diagnosis and management. Good luck."
},
{
"id": 155715,
"tgt": "What are the symptoms of kidney cancer?",
"src": "Patient: I had a CT scan of abdomen to r/o ventral hernia vs. diastasis and results showed broad based diastases of vental wall and indeterminate right midpole renal hypodensity measuring 15mm. with ultrasound recommended. My Dad had kidney cancer. should I be worried? Doctor: Thanks for your question on HCM.In my opinion you are having cyst or benign lesion mostly. But better to get done ultrasound of kidney for more information. If this normal or saying cystic benign lesion than no need to worry.Repeat ultrasound after 3 months.But if suspicious lesion is found than go for biopsy and histopathological examination to rule out malignancy. So get done ultrasound of kidney first."
},
{
"id": 106644,
"tgt": "What causes pain in the lower back?",
"src": "Patient: I have pain in my lower back (flank?) but above my butt. Like right where the kidneys are. The pain is moderate and constant, aching and radiates around to my lower pelvic area. I m 10 years out of menopause, so it s not period pain. I am nauseous and seem to tire easily. The pain started about 1 wk ago. Recently, I was treated for a severe cat bite to my right big toe with surgery followed by IV antibiotics (Invanz for 6 wks} via a picc line. My infectious disease Doc then put me on Augmenten for another 2 months. If it was say.... a kidney infection, wouldn t the oral antibiotics clear that up? I have no dysfunction with urination. And I might add that I am a diabetic. Doctor: Hello and Welcome to \u2018Ask A Doctor\u2019 service. I have reviewed your query and here is my advice. Can be postural muscle spasm or a disc prolapse. I hope this information has been helpful for you. Regards, Dr. Praveen Tayal"
},
{
"id": 172377,
"tgt": "Are there any side effects of taking Roto syrup ?",
"src": "Patient: hello, my baby is 2 months old and she got rota sirop as prevention of not getting rota virus, now from that sirop her poop was really hard in shapes of ping pong balls, from that there was a little blood on side of her poop how dangerous is that and what should i do? Doctor: Hi,Thanks and welcome to healthcare magic.Rota virus vaccine is given in 2 or 3 doses at an interval of 4-8 weeks to prevent diarrhea due to Rota virus.If the child has constipation try Lactulose syrup 2.5 ml twice daily to regularize the bowel movement.I do not think there is any relation to Rota virus vaccine and constipation.Hope this reply is OK for you.PLease feel free to ask further queries if any.Dr.M.V.Subrahmanyam."
},
{
"id": 217796,
"tgt": "Suggest treatment for pain due to dysfunction of sphincter of oddi",
"src": "Patient: I have a chronic pain condition known as disfuction of the sphincter of oddi. I was being treated with IV dilaudid that I did on my own at home. I felt being on the dilaudis all the time was not good for me. I came off the dilaudid. Started on gabapentin 600mg 4x/day and cymbalta 60mg bid Welbutrin XL 300mg bid. I went from 133 lbs to 180 lbs in about a month. I was so blown up and full of fluid I went to the ER. The ER doctor said I was toxic from the gabapentin and needed to come off slowly so I don't have seizures. As I have decreased the gabapentin and the welbutrin XL. I have developed a rash. My FP doctor believes the rash is from stess. Also, she no longer feels comfortable treating my chronic pain issue. I am having trouble getting a pain Dr. to see me. Doctor: Hai.ur really into a chronic and troublesome problem.already u have taken the maximum dose of gabapentin als, , which brings you down.Please go for a tsh (thyroid stimulating hormone) to check out the levels dueto the chronic problemPlease reduce ur stress.The ultimate relief for pain u can go for is with the neurolytic blocks ., fentayl patches under supervision takig care of respiratory depression. Take care good luck"
},
{
"id": 214454,
"tgt": "Suggest remedy for fever",
"src": "Patient: my son had fever on tuesday mild inside 100 . doc gave ibugesic fever and zincocet which he is not taking then she changed to alocet xl n he is reusing to take that too. what shld i do. he likes taste of tuspel syrup shld i give that or pls suggest aby home remedy. he is not eating properly too Doctor: HelloThank you for contacting HCM.Yes. That syrup can be given. You didn't mention age of your child. I would recommend you to Syp Calpol x 1 Table spoon after every 6 hrs till fever settle.. It comes in sweet flavor which child usually likes and takes easily. Apart from that seek medical attention if:> Child fails to eat or drink> Fever continues to rise and not settling> Fever persists for more than 2 days.> Child becomes lethargicHope you son Get Well Soon.Thank You."
},
{
"id": 26725,
"tgt": "Suggest treatment for blood in urine , heart palpitations , headache and fatigue",
"src": "Patient: 94 My labs show elevated platelets, AST, ALT, and bilirubin. There is also blood in my urine and I experience severe headaches and extreme fatigue, along with heart palpitations when I take a hot bath for my sore muscles. Last month my CBC also showed elevated WBC but not this last round of tests. My docs are stumped and are calling it Chronic Fatigue. BP is 156/94 What other tests should I request? Thanks Doctor: Hello!Welcome and thank you for asking on HCM!I read your question carefully and understand your concern. These symptoms may be related to a chronic liver infection or biliary tract infection. High platelets are often a nonspecific sign of inflammation. Blood in urine could be caused by an urinary tract infection or a lesion in the urinary tract , or a coagulation disorder (related to liver dysfunction).I recommend consulting with your GP for a careful physical examination, an abdominal ultrasound and some tests (besides the performed tests): - coagulation tests- Gama GT- alkaline phosphatase plasma levels- inflammation tests ( PCR, fibrinogen, sedimentation rate)- thyroid hormone levels- blood electrolytes - HBV- HCV- blood lipid profileto establish the possible diagnosis. Regarding your high blood pressure, I would advise to discuss with your attending physician the possibility to make some changes to your actual antihypertensive therapy, or to start therapy if your don't have any. Hope to have been helpful!Best regards, Dr. Iliri"
},
{
"id": 118711,
"tgt": "Suffering from iron deficiency, anemia, low Hb count. Taking dexorange capsules. Safe medication?",
"src": "Patient: hii i m 20 Years old female.. m suffering from iron deficiency anemia . My Hb count is 6.5. i consulted 3 doctors.. all recommended me different iron supplements like dexorange capsules, victofol, and Globac capsules. right now m taking dexorange capsules twice a day in morning and at night after meal. and also m taking mebex which doc recommendad me twice a day for 3 days. will this work?? are der any side-effects of this Doctor: Hi. Iron deficiency anemia is a very common problem in young females throughout India, and is usually due to menstrual loss and poor iron content in food. So do not worry. Make sure that your cycles are not too prolonged and that you do not pass excessive clots. Mebendazole (Mebex) has been given to take care of any parasite/ worm infection in your intestine. For iron deficiency, dexorange is not a very good option because the amount of iron available for absorption in this preparation is very less. Please take Fersolate 200mg thrice daily, together with Folic acid 5mg once daily (and add Methycobolamine 1500ug once daily if you are a vegetarian). This combination should supply the necessary iron, folic acid and Vit.B12 which are all important in red blood cell production. Take this for a minimum of 3 months and recheck your Hb at the end of every month, to check the response. Drugs are safe, except for some patients who are intolerant to iron tablets and have diarrhoea or constipation. You may get black tarry stools for which you need not worry. Hope I answered your query. Thank you."
},
{
"id": 85058,
"tgt": "Can ovigyn D, obimet SR and R nine granules make the period regular?",
"src": "Patient: i have PCOD and am trying to get pregnant...doc has prescribed me ovigyn D, obimet SR and R nine granules...am having these for the last 1 month...but i have still not got my period this month...will these medicines help in getting my periods regular? Doctor: Hello, As you have PCOD, you will be needing medicines to regulate your blood sugar. Let me tell you that Obimet SR is metformin which helps lower sugar and R nine granules also stimulate release of insulin and indirectly helps control your blood sugar. Ovigyn D 3 Tablet SR is a combination of medicines used to treat infertility. It helps in maintaining the levels of sex hormones in the body and also works as an antioxidant, which further helps in proper cell growth and function. Hope I have answered your query. Let me know if I can assist you further. Take care Regards, Dr Prabhash Verma, General & Family Physician"
},
{
"id": 214654,
"tgt": "Suggest remedy for pricking sensation in left throat",
"src": "Patient: Hi, I'm having a pricking sensation in my left throat region ( the sensation is similar to a fish bone or sharp object pricking) ... It is creating a constant irritation ... I'm not about its cause ... could you please recommend a remedy for it ... will be highly obliged ? Doctor: **1. Throat infection is/are commonly viral in origin, as such you should consult with medicine specialist or ENT surgeon for examination [to look for particular area] and evaluation. [regarding any foreign substance in the throat region] 2. initially take complete antibiotic course [to prevent secondary bacterial infection along with decongestants and anti-inflammatory medicines. 3.. gargles three times in a day [gargle with warm spinach juice], voice rest, stop smoking. avoid sour, curd/ yogurt, cold drinks and oily food. . take multivitamin supplements. boil 2 spoons of 'yashtimadhu churan' in 4 cup water till 1 cup remains. filter it and add4 spoon honey to it and drink"
},
{
"id": 207939,
"tgt": "What causes mood swings while on Citalopram?",
"src": "Patient: I'm on citalopram 20mg and have been for about 2 months (2nd time of being on it) recently i've been forgetting to take it only every couple of days ill possibly forget 1. i've been feeling worse than ever these past few days despite things going well with friends and work i was wondering if the missed doses being the issue or if i needed to up the dose? i know i probably dont need to up the dose but i just want/need something to sort me out and get me back to normal as soon as possibly as im just exhausted with feeling so down. Doctor: HiI admire your concern.Citalopram is antidepressant and use to treat mood disorder.It has short half like of 21 hours.so it can possible that if you do not take it for one or two days then it produces withdrawal kind of phenomena.Though it is not common but yes occur in some patients.For this you have to take it daily or add low dose benzodiazapine to prevent such feeling if you forget to take it.The dose is appropriate and no need to up the dose .So continue to take same medicines and get well soon.If needed then consult psychiatrist for further assistance.Thank you."
},
{
"id": 77890,
"tgt": "Suggest cause for chest tightening",
"src": "Patient: Saturday evening, I had a sudden tightening in my chest as if someone was squeezing my heart. I became very warm and lightheaded and felt \"out of it\". I had a nitro pill with me and took it and it seemed to relief the discomfort but left me feeling like I was kind of floating. Doctor: HelloChest tightening need prompt medical attention.Besides clinical evaluation,you may need complete investigations.Investigations include routine hemogram,random blood sugar,liver and renal function test,lipid profile,ECG,ECHO,TMT.Angiography may be needed after evaluation.You need proper evaluation by a cardiologist.Get well soon.Take CareDr.Indu Bhushan"
},
{
"id": 92603,
"tgt": "Abdominal pain. Pain occurs during lying down, standing, after sitting for few minutes. Any suggestion?",
"src": "Patient: im having abdominal pain on both sides more on the left. pain comes when laying down and standing up after sitting for 10 mins or more. i had a ultra sound because i thought it was my ovaries and the tech stated my bowels are all over the place she could bearly see my ovaries. im waiting to see the gastrolgist. sometimes i feel like im going to pass out. im dizzy at times. Doctor: Hello, Thanks for the query to H.C.M. Forum. If I would your treating doctor for this pain on both side of abdomen, I would come up with these possibilities , these includes.1 Renal stone , as you stated in your query that pain is on both side and aggravated during standing . Usually stone in any one or both kidneys develops such type of symptoms. Diagnosis can be confirmed by M R I of both kidneys , if ultrasound didn't find any thing.2 Ovarian cyst , usually this causes pain on both flanks . Diagnosis can be confirmed by ultrasound of both ovaries by another tech.3 Amoebic colitis is one of the most foremost reason of pain in both side . Diagnosis can be confirmed by physical examination by a physician . As in amoebic colitis usually intestine palpated .In my opinion deal according to above mentioned causes. First of all consult a gynecologist and get his opinion.Good luck.Dr. HET"
},
{
"id": 43368,
"tgt": "1 year b4 in full abdomen scan i got report",
"src": "Patient: 1 year b4 in full abdomen scan i got report polysystic overies..after some treatment again i took that scan now..report is belated pcod changes..there is any different or both report are same..right ovary - measures 3.1*1.5*2.0cm.left overy measures 2.8*2.3*3.0cms both overies are normal in size and mutliple small anechoic cysts ..final impression bilateral pcod changes Doctor: Hi,Thank you for choosing Healthcaremagic. To make you clear PCOS cannot be just diagnosed by Scan, as its mandatory to have irregular periods(ANOVULATORY periods, altered LH:FSH ratio and scan changes, either of any two will be called PCOS. Hence even if the scan is showing polycystic ovaries without changes in periods need not be a concern.I hope this information has been both informative and helpful for you. In case of any doubt, I will be available for follow-ups. If you like my answers kindly rate it, and write a review as well. Please do not forget to accept it.Thank you,Wish you good health.Regards,Dr ArifYou can consult me again directly through my profilehttp://www.healthcaremagic.com/doctors/dr-arif-n-khan/65133"
},
{
"id": 124669,
"tgt": "Suggest treatment for shoulder pain after an injury",
"src": "Patient: Dear sir, 2 weeks back i felt down when i was ridding a bike I got injured in my left shoulder doctor advise me ,nothing wrong everything fine no fracture he verify through a scab report but synovium , joint capsule problem I cant able to up my hand Doctor: Hello, It could be a ligament or tendon related pain. As a first line management, you can take analgesics like paracetamol or aceclofenac for pain relief. If symptoms persists better to consult an orthopedician and plan for an MRI scan. Hope I have answered your query. Let me know if I can assist you further. Regards, Dr. Shinas Hussain, General & Family Physician"
},
{
"id": 21401,
"tgt": "What causes spikes in EKG reading?",
"src": "Patient: my dad went to his cardo doctor and he did a ekg and told him that he had some abnormal spikes so he wants him to have a echocardiogram. All most a year ago he had a triple bypass and they said his heart muscle had no damage so is this something to be alarmed about?? Doctor: I don't think so there is some alarming however patient undergone CABG must continue the medications prescribed and requires yearly heart examination as adivsed by cardiologist and surgeon.Irregular spikes could be ectopics beats . Please consult your Doctor .Get your electrolytes done as it may be due to deranged electrolytes.Regards"
},
{
"id": 55073,
"tgt": "What causes elevated liver enzymes?",
"src": "Patient: Hi, I am a 33 year old female, married with two kids, 6 and 9. I have only had sexual intercourse with my husband and one other boy in high school. I have never done illegal drugs. I do drink about once a week. My liver enzymes are very elevated and my BUN is just slightly higher than normal. Doctor: hi.noted history of elevated liver enzymes and BUN levels. it is best if you consult with a doctor for physical examination and clinical evaluation. you may have a fatty liver or chronic alcohol intake, hence, elevation of liver enzymes. infective and obstructive causes must be ruled-out. for your BUN levels, your kidneys must be evaluated as well. kidney or ureteral stones must be ruled-out. diagnostics (such as abdominal ultrasound, blood counts, urinalysis, etc.) and management (medical and/or surgical if indications are found) will be directed accordingly. low fat, low salt diet is also recommended.hope this helps.good day!!~dr.kaye"
},
{
"id": 145768,
"tgt": "What causes headache and impaired speech?",
"src": "Patient: I have had a migraine/headache for over a week straight. I suffer from them & have tried numerous preventatives which I thought were working. I cannot lay on my stomach & lift my head up without pressure. My speech hasn t been 100% lately. I mix up my words, say one thing but mean another & often times I have to stop & start over. I ve always been an awful speller but lately I cannot even sound words out properly to type them and its frustrating. About a year ago I had an MRI done which showed a lesion in the white matter of my frontal lobe. My nuerologist said it was normal. I m concerned that there is more to it. I keep giving my old phone number & zip code when asked & its been a good 5 years or more since those have changed. Is this normal? Doctor: Hi,Thanks for writing in.White matter lesions in frontal lobe are seen in many people in the elderly age group. Since your age is not mentioned, I guess you are in your middle age or elderly. The frontal lobes are most severely affected by sub cortical vascular ischemia. In this there is reduced blood supply in the specific areas of the brain and these are more abundant in the frontal region. It is not a serious finding and your doctor might have considered it as a part of normal aging changes in your brain and therefore mentioned it as normal.Such findings have been associated with frontal hypometabolism and executive dysfunction. There is reduced nutrition to these parts with decreased cognitive functions and probably speech at times. The memory problem relating to saying your old zip code shows that you have an intact long term memory but your recent memory is affected at times. Please discuss this with your doctors and you might like to remember things using other methods like writing down important things on a piece of paper."
},
{
"id": 113878,
"tgt": "How can I get relief from back pain that is present through out the day ?",
"src": "Patient: Hi to all, I am 28 years old; I am suffering from lower back pain (right side) for the last 4 months. For first 3 months I feel the pain only at wake up time in the morning and it exists for 2 or 3 hours but now it exists for almost the whole day. In this connection, I have consulted the Doctor and conducted Ultra Sound as suggested. However, the report was found normal and there are no any symptoms of Stone or any other medical problem. Doctor told me it should be Gas problem and gave me Medicines i.e. Diclo-SR & Rentec. I take these pills regularly for about a week and after that I feel no pain but now even these medicines doesn t work an pain is growing day by day. It is therefore; kindly send something helpful suggestions, as I am still suffering from the back pain. Doctor: Hi,Ashwaniji, Thanks for query, You have not mentioned your age. If you are elderly then it is possible you might have degenerative changes in your spinal vartebrae giving risr to low back pain. Go for X ray lumbo sacral region and find out that is there any compression of vertebrae or any osteophytes producing this problem. Consult orthopedic and get examined. Go for back extension exercise and physiotherapy. Take calcium,Vit A,D, and other B.complex medicines. ok and bye."
},
{
"id": 66472,
"tgt": "What causes lump in neck while singing?",
"src": "Patient: I was just standing in front of my bathroom mirror singing while fixing my hair and noticed a lump slowly appear on the left side of my neck. I felt around for it and I can not feel anything there--no lymph nodes or pea-sized knots. Nothing. When I stop singing it goes away. Could this just be a vein popping up? Health history: I am an almost 32-year-old female. I am overweight, but otherwise I am in good health. I had some bloodwork done in February for insurance purposes, and it all came back normal. I have never had any health issues other than seasonal allergies. (I live in South Georgia--allergy capital of the world. ha) Doctor: it could be a thyroid colloid cyst or a laryngocele or mucocele - all are benign conditions and not to worry!USG can diagnose it accuratelyplz consult an ENT specialistall the best!"
},
{
"id": 70158,
"tgt": "What does a lump under the jaw indicate?",
"src": "Patient: Two months ago I had a ct scan due to neck issues. They saw I had large lymph nodes approximately 2 centimeters. I then went to an ENT and he couldn t feel anything so he wasn t able todo a biopsy so he sent me to a nearby facilities to have a sonogram on my neck to help for the biopsy. A biopsy was taken and the results came back good with no cancer Involvement. I then went on November 18th to the ENT cause he wanted to do a check up to make sure there is no growth. He felt no such growth and told me I am fine. Recently however within the past two days. When I bend my neck down and but my chin on my chest if I move my head left or right I feel like a little bump right under my jaw. Should I be concerned or am I just paranoid? Like I said its been less then a month since I went to the ENT. Doctor: Hi ! Good afternoon. I am Dr Shareef answering your query.Although the feeling could be due to the scar tissue following the biopsy (if it was an open biopsy), I would definitely send you to the ENT specialist for a review before you label your symptoms as paranoid.I hope this information would help you in discussing with your family physician/treating doctor in further management of your problem. Please do not hesitate to ask in case of any further doubts.Thanks for choosing health care magic to clear doubts on your health problems. Wishing you an early recovery. Dr Shareef."
},
{
"id": 104971,
"tgt": "Allergy, congested, runny node, sneezing, increased heart rate. Taken allegra tablet and nasal spray. Cause?",
"src": "Patient: Hi, I was at my allergist today and was very congested, runny stuffy nose, sneezing as a result of being off my Allegra for four days prior to the subdermal allergy testing she did today. I showed no reaction to any of the environmental allergens tested for, but to help me out she gave me a small cup full of liquid prednisone , an Allegra tablet and some nasal spray (she said earlier would not aggravate my existing benign heart irregular beat I get from time to time.) That was at noon today. Then tonight at 5pm I started suddenly getting an increased heart rate along with bad irregular beats (PVCs I believe from past experience). Three hours later I am still getting them. Can these be a result of the prednisone the allergist gave me? I told her I had a history of benign irregular heart beats and couldn t take any nasal sprays that had a decongestant effect because of my heart. That is why I m surprised she gave me prednisone which I just read online can cause palpitations and rapid beat. Do you think the one dose of prednisone caused this fast beat and irregular heart beats and, if so, will it subside soon or by morning? Thanks. Doctor: Hello, There is in fact very little evidence to suggest prednisolone can set off premature ventricular contractions; but certainly can set off palpitations if the dose was higher than required. Some patients with adrenal insufficiency who require steroids for life also feel the same once they start and novice patients will feel more of the effect. You have not mentioned the dose you had, or whether you took it on an empty stomach or without PPI cover. Gastritis is a problem when taken on an empty stomach, and that set off palpitations. It should subside within 24 hours, but if you still feel unwell, I would suggest you get seen by a physician as soon as possible for an ECG and a thorough evaluation. Thanks."
},
{
"id": 90020,
"tgt": "What causes a lumpy area in the mid of the upper abdomen?",
"src": "Patient: I underwent surgery for a perforated duodenal ulcer just before Christmas 2010. I was in excruciating pain for 4 months following the surgery in the lower right hand side of my stomach (totally unrelated to where the ulcer was situation. I have also been left with an unusual balloon like lumpy areas in the middle of my upper abdomen. I was extremely fit prior to the operation and very slim. Since the operation I have put on some weight and filled out but now have a fat layer over my stomach that I have never had before in my life. I always had an extremely flat stomach. My question is why do I have this balloon type lumpy area in the middle of my upper stomach now - Why do I now have excess fat covering my stomach I never had before and when can I commence doing serious core workouts (together with total body workouts)? Thank you for your assistance and help in this matter. To be honest I wish this had never happened to me. It was a traumatic experience as the surgeon (prior to the op) decided I needed a bag as my bowels had burst (which they had not). Thanks Doctor: welcome to Health care magic.1.What you presently experiencing similar to the bowel gas formation.2.The excess weight you have gained with the fat layer over your tummy could be because of operation you might have not been physically active, and not moving the operative area and things like that.3.Yes workout is good - but don't do as you have planned, do it gradually with a brisk walk 30 - 45 min and then go on, as you gradually increase.4.Get an examination of the abdomen done by your doctor when ever you get appointment and then get an ultrasound abdomen done - which will rule out the lump which you are thinking of.5.Try to avoid junk /spicy food and alcohol for at least 1 month and see if you find any difference. Hope it helps.Anything to ask ? do not hesitate. Thank you."
},
{
"id": 76642,
"tgt": "How long will it take to recover from breathlessness?",
"src": "Patient: I had a lung biopsy 6 days ago they let me go home and said I had pockets in the lung and to take it easy. Im still short breathed but they said it would take awhile to get over it, how long does it take?Im a 68 year old female and just found out I had breast cancer and now lung cancer. Thank you Doctor: Thanks for your question on Healthcare Magic. I can understand your concern. You are diagnosed case of breast and lung cancer. Biopsy in lung cancer can cause pneumothorax. And this might be the cause for you breathlessness. Other possible causes of breathlessness in lung cancer patient are 1. Bronchitis 2. Malignant Pleural effusion 3. Pneumonia 4. Enlargement of underlying mass lesion. So better to consult pulmonologist and get done clinical examination of respiratory system, chest x ray and PFT (pulmonary function test) first the diagnosis of these causes. You may need antibiotic, inhaled bronchodilator and inhaled corticosteroids. Usual time for recovery after starting appropriate treatment is 1-2 months. Hope I have solved your query. I will be happy to help you further. Wish you good health. Thanks."
},
{
"id": 218368,
"tgt": "Is pregnancy possible despite negative pregnancy test results?",
"src": "Patient: Hi! So on the 25th of may me and my boyfriend were messing around and we were spooning. He rubbed his penis around my anus (he didn\u2019t insert it or cum) and it lasted for about a few seconds. A few days later i took a pregnancy test and it came out negative. I kept taking pregnancy test every week (i took about 3 in total and they all came out negative) to see if i was pregnant. I was supposed to get my period 3 days ago (on the 15th of june) and i took a pregnancy test again today (the 18th of june) and it still came out negative. Am I pregnant?? I dont seem to be having any nausea, vomiting, spotting, or tender breasts like normal pregnancy symptoms, but I have been experiencing cramping like regular pms. Im at a young age right now and im really worried because I don\u2019t want to be pregnant Doctor: Hello dear,welcome to Ask a doctor service. I reviewed your query and here is advise.From what you describe I guess you are not pregnant. The delayed period is due to any hormonal disbalance. Don't worry your period will come soon. Hope I have answered the question. Let me know if I can assist you further."
},
{
"id": 141863,
"tgt": "Suggest treatment for severe spinal pain",
"src": "Patient: I had a spinal injury 40+ years ago and have been in pain all those years. have had fusion to half my back. What s bothering now is that, I sleep far too much. Funny I seem to beable to sleep when I have very severe pain. Am on morphine based drugs, but I just hate all this sleeping and no energy. Am trying to keep this as short as possible Doctor: Hello!My name is Dr. Aida and I am glad to attend you on Healthcaremagic!Your symptoms could be related to the morphine based drugs adverse effects. They are known to cause addiction and tolerance. For this reason, I would recommend discussing with your doctor on the possibility of switching to pregabaline or duloxetine, in order to help for the chronic management of your back pain and gradually stop the morphine based drugs. Hope you will find this answer helpful!I remain at your disposal for any further questions whenever you need!Best wishes!"
},
{
"id": 180327,
"tgt": "Suggest healthy diet for baby",
"src": "Patient: My daughter is 5 months and 20 days...now am giving home made raggi twice a day.....am giving 4 to 6 table spoons at morning and evening am confused please advice me....what can i give her in the noon....can i give cerealac...some people says it cause phlegm please help me Doctor: Hello, You should introduce food items in your daughter's diet gradually. You can give her mashed banana, chikoo; pur\u00e9ed vegetables like pumpkin, carrot; daal soup; sooji; raagi; steamed apple, pear; khichdi and other home made foods. Introduce one food item once every 5 - 7 days. This will help you in identifying if your child is allergic to any particular food. You can give cerelac occasionally but as it is a processed product do not use it on a regular basis. It is always preferable to use fresh home made stuff for babies. Hope you find this helpful.Regards,Preeti"
},
{
"id": 218467,
"tgt": "Does taking Deviry during pregnancy cause any complications?",
"src": "Patient: I use to have my periods on 21 every month. ON march 22 i got periods and upto now i didnt get again. I tested using prega news but result in negative. i consulted doctor and she suggested to take deviry 10 mg. My friends and parents are telling for some people pregnancy result might get late. As i am using this Deviry 10 mg is that going to effect if i am pregnant? Doctor: Hello and Welcome to \u2018Ask A Doctor\u2019 service. I have reviewed your query and here is my advice. If I were your treating doctor, then I would rule out other causes of irregular menstruation. In my opinion, first pregnancy should be ruled out with investigation e.g. USG. There after your line of treatment should be defined. For that you must consult your gynecologist. And yes, Deviry can adversely affect your growing child, if you are pregnant. Hope I have answered your query. Let me know if I can assist you further."
},
{
"id": 36034,
"tgt": "Could high amounts of chlorine have led to yeast infection?",
"src": "Patient: Hi ,hi have had a yeast infection for a little over 24 hrs. I noticed this a few days after a day with my family at the pool. Could the high amounts of chlorine be a factor as to why I have a yeast infection? Can I treat the yeast with a detol soak in the tub? Doctor: Hey thanks for your faith in the health care magic.i have gone through your question and understood your problem.yes it can if u had yeast infection previously.When swimming with a yeast infection, keep your vaginal area dry when you're through with your swim. Hanging around in a wet bathing suit can stimulate the growth of yeast, exacerbating your infection. Avoid putting another suit on, if possible, because yeast thrives in synthetic fabrics. A loose-fitting dress and cotton panties helps keep your vaginal area dry and inhibit the yeast infection.Hope I have answered your query. If you like my answers and wish me to answer in future, bring it to my attention"
},
{
"id": 91405,
"tgt": "Is abdominal pain related to unprotected sex?",
"src": "Patient: please help :(I Have Left Side Pain On My Stomach And In My Left Side Of My Back,Right across were my pain in my stomach is.When i move it hurts alot even when i cough or laugh,It Feels like a stabbing pain..My Period Left 2 days ago,but before my period had came i had right side ovule pain raking my whole stomach,Till both of my ovules started to hurt.I webt to the hospital and got tested and came out negative,probably cus it was to early.But anyways they didnt find anything the was causing my pain..Till now,got my period,it left..nd now i got the pain,in my left side of my stomach takibg my back,i cant even cough or laugh,it hurts badly.By the way,i have unprotected sex with my partner all time. Doctor: HI. This may or May not be related to the unprotected sex. All your symptoms are suggestive of left sided kidney infection and can be due to sex., as the infection of the urinary tract are common in female as the length of urethra is small and the infection can travel very fast up to kidney. Get an urgent ultrasonography, urine - routine, microscopy and culture & sensitivity test. Take plenty of water and oral fluids. Consult an Urologist if required."
},
{
"id": 30595,
"tgt": "What causes lack of appetite while on treatment for TB?",
"src": "Patient: my gland was increases from 2 months doctor dignosis fnac then biopsy also but in biopsy also just the report is only infection then doctor suggest for ADA test and tbigm then the tbigm report is positive now and doctor advise for tb medicine but now i feel like a fear in my heart and i ve no interest for food what can i do for further Doctor: hi,the ADA drugs it self causes loss of appetite, don't worry you will feel better in few days, have a balanced diet and lot of fluids,report to your TB physician if 1) you have severe nausea and vomiting2)unexplained fever or tiredness3)have any tingling sensation in feet and palms4) have jaundice - yelow coloration of eyes, nails and skin5)visual changes6) any unexplained skin rash.at any time don't discontinue the medications with out consulting your physician take care."
},
{
"id": 158961,
"tgt": "Lump on the buttocks. Family history of cancer. Fat deposit?",
"src": "Patient: I am a 36 year old female with a lot of Cancer on my mom s side of the family. Recently, while showering, I felt a lump (about the size of a dime) on the inside cheek of my left buttocks . I showed it to my primary doctor who said he believed it to be a fatty deposit. I think I want to get a second opinion. What do you think this might be? Doctor: Hi and welcome to HCM. It is hard to say without seeing and palpating it. In most cases it is benign. Usually lipoma, atheroma, fiibroma or traumatic haematoma, swollen lymoh node. If it is soft and painless it is probably lipoma. If it is painful and there is redness of skin above it it should be evaluated by biopsy or cytologic puncture. Malignancies have more intensive symtpoms and you shouldnt be worried. Wish you good health."
},
{
"id": 83453,
"tgt": "What are the side effects of Azithral 500?",
"src": "Patient: I am 46 yrs Male and doctor as prescribed Azithral 500 by Alembic for my throat infection resulting fever, sneezing and little cough 1 tablet a day for six days. Should I continue for six days and what is the side effect? I feel liitle pain and uneasyness after sometime of having this medicine. Pls suggest Doctor: Hi, The common side effects of azithral-500 include abdominal pain, nausea, vomiting, and diarrhea. Although it is rare it may also cause abnormal heart rhythm. Uneasiness and pain usually subsides gradually. So, continue taking it as prescribed. Hope I have answered your query. Let me know if I can assist you further. Take care Regards, Dr. Mohammed Taher Ali"
},
{
"id": 48020,
"tgt": "What are the symptoms of kidney problem?",
"src": "Patient: what is some symptoms od kidney problems? periodically I HAVE PAINS DOWN IN MY LOWER BACK THAT FEEL LIKE A LITTLE PIN STICK AND I NOTICE THAT MY URINE HAS A NEW SMELL NOT THAT IT STINKS JUST A LITTLE SMELL. AND THE LITTLE PIN STAB PAIN COME ON THE LEFT AND RIGHT LOWER BOTTOM NEAR MY UPPER POSTERIOR. Doctor: Thanks for your consultation at HCM! I have gone through your Case history and as your physcian i totally understands your concerns. My diagnosis is that you have got Urinary tract infection.... This Uti is giving you these symptoms... Second thing is that you should be looked by Urologist to exclude any renal stones as an underlying cause... Get your urinalysis and XRay KUB and ultrasound done and consult urologist.. Hope your query is adequately answered if you still have any feel free to ask directly on my profile any time.... Regards Dr Saad Sultan"
},
{
"id": 129924,
"tgt": "What cause sharp pain on the right side extending down to the hip?",
"src": "Patient: Since Thursday i have noticed a little pain in my belly button. Friday night when i got out of the shower i went to dry my belly button, then came a sharp pain on my right side all the way down to my hip. I looked in my belly button & something was sticking out. I had my mother-n-law look at it, she pulled whatever it was out. Then all of a sudden pus started oozing out. I still have the pain, & can feel a knot near my belly button. What is going on?? Doctor: you probably have a cellulitis or boil the is causing u trouble, consult a general surgeon and seek help on draining the pus out, this will aid healing. second, u need a course on antibiotics to cover this up."
},
{
"id": 26954,
"tgt": "What indicates intermittent tingling in left hand, thumb and fingers in person with ISH?",
"src": "Patient: i was diagnosied with as about 3 years ago ish, i had a flare up yesterday which was a bad one, when i woke this morning i had a tingling in my left hand, i didnt think anything f it but it still has gone away, its now 5 hours after i first noticed it, the tingling is in my thumb, little finger and middle finger, it comes and goes further up my arm. thanks Doctor: Hello!Thank you for your question on HCM!Regarding your concern, your symptoms don't seem to be related to the heart. The most probable cause would be a cervical disc displacement, which is causing a radicular pain, or a pinched nerve. Another disorder which could cause this symptomatology would be a possible stroke or TIA. I recommend consulting with a neurologist for a careful physical examination and perform a routine check up ( resting ECG, chest x ray, routine blood test, blood electrolytes, kidney and liver function tests, thyroid hormone levels). A cervical spine x ray and a brain CT scan may be necessary to exclude the above mentioned causes. Hope to have been of help!Best wishes, Dr. Iliri"
},
{
"id": 28813,
"tgt": "How can an infected callous on the foot be treated?",
"src": "Patient: I am at work: I wont get home until tomorrow. I was in Mexico I walked on the beach barefoot and then stopped because of a blister on the back of my right foot. I started feeling sick yesterday but not too bad now I have chills, a sore throat and a temp, I noticed there is a red stripe across my left foot... I have a callous on that foot... it must have gotten infected... I have been soaking the other foot in Epsom salts Doctor: Hello and Welcome to \u2018Ask A Doctor\u2019 service.I have reviewed your query and here is my advice.This could be very serious. You need to go to an Emergency Department and be seen immediately.You need antibiotics now. Do not delay.Hope I have answered your query. Let me know if I can assist you further.Regards,Dr. Kathy Robinson"
},
{
"id": 128509,
"tgt": "What causes discomfort under the armpit when suffering from RA?",
"src": "Patient: My wife recently had discomfort on her left side, beneath her left arm pit. She saw her doctor and he had a d dimer blood test run. Normal range is under 250 we understand. Her result was 5036. Our doctor ordered a ct scan, which was performed a few days later. The results were negative. Our doctor then said to let him know if the pain didn t go away,or got worse. There was no additional information provided as to the cause of the elevated d dimer number. My wife has RA, which we understand might effect the d dimer number, but no one has told us anything additional to calm our fears which are born out of our ignorance of the situation. Can you offer any possible explanations in this matter? Doctor: Dear patient elevated d dimers level is sign of acute pulmonary embolism. This.may happen in patients with increased susceptibility for clot formation in blood ie increased clotting factors in blood. Steroid intake for long time may have side effect of increased coagulability in blood. Since some of patients suffering from rheumatoid arthritis are prescribed steroids this may be reason. Are you taking steroids? Ct scan of chest will help to confirm diagnosis of pulmonary embolism. So that may be the reason for ordering ct scan. Pain in arm pit may be the due to minor embolism episode. I think have solved your queries."
},
{
"id": 3072,
"tgt": "Can i get pregnant after removal of one ovary?",
"src": "Patient: hie doct i have gone under surgery in sept 2013 and my right overy is benn removed due to cyst on it. after operation my doct has prescribed my to take dronis 20 i am taking this since last one year. i am worrier weather i can get pregnant or not. will it affect my pregnancy life. Doctor: Hello dearI understand your concernThere is chance of the pregnancy but it is decreased compared to those who have both ovary.Ovulation occur for the alternated ovary.So you can become pregnant.I would suggest to consult the gyneccologist and undergo reproductive hormone analysis, USG scan and ovarian follicle study for better management.Drugs like progesterone pill in second half of period, metformin and clomiphene citrate, folic acid will help to become pregnant.Avoid stress, take healthy diet, drink plenty of water and do regular exercise and reduce 10-12% weight if you are obese.Hope this may help youContact further if follow up neededBest regardsDr. Sagar"
},
{
"id": 64641,
"tgt": "Suggest remedy for lumps in inner thigh",
"src": "Patient: Hi, I am a 26 years old woman. And I have a pretty large lump on my upper inner thigh. I found it a couple of days ago and it is swollen and quite painful now when I walk or touch it. Is there anything I could try for now? Do I need to see a doctor? In that case, what kind of doctor shld I go see? I am very worried. Doctor: Hi,From history it seems that there might be having either ingrown hair follicle infection or enlarged tender inguinal lymph nodes giving this problem.You might require one course of antibiotic and anti-inflammatory medicine course for 3-5 days.Consult your family doctor and get examined.Ok and take care."
},
{
"id": 121710,
"tgt": "What causes sore ball of foot under toes after started driving car?",
"src": "Patient: Since I bought a new car manual one year ago I have a very sore ball of foot under toes extended to knee new strain veins and week to stand or weight bear on must rest leg a lot up mid leg gets hot and jelly feel I feel like bandage may ease pain leg no swell what is wrong with me my mum had varicous veins but she since passed away last June after suffering motor neuron bulbar palsy disease her age 76 my age 49 i am afraid of m.n.d with me as it starts with limbs and it s familiar disease asy cousin who s 54 has it aswell I am daily fit 163 tall and 72kg female please help solve my pain the past year is it the car I have eased off driving much as leg suffers but pain in leg no change lyn Doctor: Hello, Your symptoms seem to be related to varicose veins. I suggest using compression stockings and maintain the legs elevated. I also suggest using an anti-inflammatory medication such as Ibuprofen to relieve the pain. If the symptoms continue I suggest seeing your general doctor for a physical examination. Hope I have answered your query. Let me know if I can assist you further. Take care Regards, Dr Dorina Gurabardhi, General & Family Physician"
},
{
"id": 19071,
"tgt": "What causes low BP in an elderly person?",
"src": "Patient: my husband s bp is 84/52. I have taken it three times. I know the machine is working because I took my bp. he will be 80 oct 29. he hasn t had any health problems except for high bp. he has had diarrhea(sp?). he weighed 204 a week ago. he now weighs 192. Doctor: Hello,Brief Answer: Dehydration most likely caused low blood pressure.Explanation: My opinion is that hypertensive patient who is experiencing any condition that causes him to lose fluids such as diarrhea in your husband case, blood pressure should be checked first. If low all over the day, good hydration is mandatory and some times it may be needed to stop the anti-hypertension drugs or lower their dose to get the blood pressure normal again.I suggest my patients the following: Good oral fluid intake. Follow up blood pressure and treat diarrhea according to its cause. The information provided by you is not sufficient to provide a good opinion. If someone comes to me with this condition I would ask them about types and doses of antihypertension medications.Conclusion: I suggest you check with your physician.Hope I have answered your query. Let me know if I can assist you further.Regards, Dr. Mahmmad Gamal"
},
{
"id": 91565,
"tgt": "What is the treatment for cough and abdominal pain?",
"src": "Patient: Over the years I have had a sharp stabbing pain in my lower right abdomen (slightly below & to the left of my hip bone) occassionally when I sneeze. I have had a bad cough now for several days. Today when I cough, the stabbing pain occurs each time. If I push on that area the pain is not as bad. I am not pregnant. Doctor: HI. This can be due to a nerve entrapped or a hernia. Id the pain coincides with sneezing there has to be a problem.. Consult a Surgeon , who would do a clinical examination and have a diagnosis. May need few investigations if He/ She feels so.Of course you need more treatment for the cough than the pain as coughing is a causative factor , which should be treated first."
},
{
"id": 134316,
"tgt": "Suggest treatment for tinnitus,headaches and stiffness in legs",
"src": "Patient: Hi i am a 29 yr old female with I.H. I m hearing a whooshing sound in my ears headache on left side of face and moving towards the upper right neck pain and stiffness my muscles are weak especially my legs and arms my left side towards my back is killing me I have been put on 500mg or diamox 4x daily but having a problem urinating can u please help I don t what to do Doctor: hi,thank you for giving a brief history of you. kindly see that for your titinus you can do gargling with water continuous for about 60 seconds. this will help the facial and throat muscles to get some strength. along with it you can do hot water fermentation for your neck region. post that do some small neck strengthening exercises like static neck strengthening exercises. also try doing some core strengthening exercise like Pilates. Suppose in case the pain doesn't come down then you can meet up an ortho for which he might guide you for an MRI of spine which will help come closer to diagnosis. for the exercises you can meet up a physical therapist who can help you to learn proper exercises and also assess you. taking a help from an expert is always on a benefit side.with the grace of God I wish you a speedy recovery"
},
{
"id": 131738,
"tgt": "What causes pain in the left hip when standing up after i have been seated for a while?",
"src": "Patient: I get severe pain in my left hip when standing up after I ve been seated for a while, also when getting into/out of car, etc. The pain makes me hobble for the first few paces after I stand up, then starts to ease off but doesn t completely disappear. It doesn t really affect me when lying down/ sleeping. Doctor: this type of pain is related to spine, u need proper x Ray and MRI to rule out spinal pathology, kindly go through it and need clinical examination , Physiotherapy, rest and medicine"
},
{
"id": 66306,
"tgt": "What is the cause of lumps from toe to ankles?",
"src": "Patient: I have noticed on and off little lumps in the vein on top of my right foot from my toe to my ankle. There can be 3-12 at a time. Some of them are almost back to back touching. They do not cause any pain and will come and go. Are they a natural occurrence? Doctor: HiWelcome to hcmSometime dilated veins looks like a lump but soft on touch. Veins thrombosis can occur but it is generally painful. You should show it to a surgeon once.Regards"
},
{
"id": 13140,
"tgt": "What causes large patch on the buttocks?",
"src": "Patient: i have had a large patch on my buttock since 7 years now.it is of a darker skin tone.the skin omn my buttocks is also crusty and dry and feels like a lot od tiny whiteheads deep inside.i've had the bumpy whiteheads feeling since my childhood.can you tell me what it is? Doctor: Hello, Seems it is most likely a fungal infection. Fungal infection is dark or red, itchy and scaly. I suggest you try a topical antifungal e.g. either clotrimazole cream or ketoconazole cream. In addition, you may take an oral antihistamine e.g. cetirizine tablet for symptomatic relief from itching. Hope I have answered your query. Let me know if I can assist you further. Regards, Dr. Kakkar S., Dermatologist"
},
{
"id": 170735,
"tgt": "Suggest remedy for loose motions",
"src": "Patient: Motions for 3 month baby Q : HI............ sir my baby is 3 months old suffering from motions from last 8 days and we are using the medicine and wALAMYCIN dropsand ors .Can you please suggest the alternate for recover. A : please tell me whether you are giving breast feed or external milk Doctor: Breast milk is harmless in lose motions too.Best herbal combo is Balchaturbhadra CHOORNA - a classic Ayurveda medicine. Controls lose motions, vomiting, cold etc of small babies'. It's prepared by mixing four child friendly herbs. BAEL pulp powder also may be added for immediate control harmlessly.If you are in India, you can get easily. For other places, get online or we can send high quality herbs mixed as per proper procedures. Our professional group offers package at concessional rates for HCM community.For any further information or query, pl consult through direct question.Hope it helps."
},
{
"id": 128382,
"tgt": "What causes burning sensation in the elbows?",
"src": "Patient: so for the past 5 weeks, I have noticed that in my elbow area on both arms I am getting a strong feeling that its on fire. this happens when I pick anything up from heavy items to a cell phone. I get these pains and fire feelings. the feeling shoots from elbow to mid forearm. what could this be? Doctor: Dear patient you seem to have tennis elbow. In this condition there is inflammation on the lateral epicondyle of humerus at the extensor muscle origin. this happens in middle aged patients. Pain increases on lifting weights and twisting movements of wrist so you should avoid that. Start tab zerodol sp twice a day and do hot water fomentation. If not improved local depomedrol injection gives relief in almost all cases. Please consult orthopaedic surgeon nearby your area for that."
},
{
"id": 82367,
"tgt": "What causes pleural thickening with patchy opacities in the lung?",
"src": "Patient: My chest xray says: left plueral thickening with patchy opacities of the left upper lung zone.The left hilum is elavated with comparative volume loss of the left lung. The pulmonary vessel on the left shore a hanging down appearance. Foci of calcilification on The right upper zone. Heart size is normal. Doctor: Thanks for your question on HCM. In my opinion you should nit worry much about this report.It is old lesion and must be due to old lesion.You may have old TB and due to this fibrosis occured and long standing fibrosis causes pleural thickening, fibrosis and calcification and volume loss.So you are having left lung malfunctioning. So better to take pulmonary vaccines like pneumococcal and h.influenzae for prevention of future infection.Better to start pulmonary rehabilitation and chest physiotherapy."
},
{
"id": 151465,
"tgt": "Had optic neuritis but no brain lesions. Abnormal spina tap. MRI showed lesions. Multiple sclerosis?",
"src": "Patient: Hi, Im in what they call a holding pattern for ms. In 2001 i had an optic neuritis with no lesions on the brain. The doctor then said he couldnt diagnose me with ms because their were no lesions on the brain but to watch for symptoms because I could develope ms. In 2005 my vision kept blurring in and out.. eye doctor said it wasnt from because my script changed. In 2010 i started feeling light headed and the blurry vision returned. I had abdominal pain starting in 2011 , muscles in legs started to become stiff along with the light headed and vision problems. So I went to the gi who said it could be MS but because celiacs ran in my family he tested for that. That came back positive so I went on a strict gluten free diet hoping I would feel better. The doc also did a transient bowel test which showed my nerves in the bowel werent working. Sign of ms? In may of 2012 i came down with vertigo numbness in my hand and leg on the left side. I had an mri of the brain which showed 6 lesions 3 on each side of the brain. They were consistent with the mcdonald theory but not ms specific in the way they were shaped. So I had a spinal tap I only had 1 oband... My ssep came back abnormal which showed slowing in the thoracic spine . Mri was negative in that area... Blood work pretty much ruled out everything else. Dont I have enough to show that it is ms? I had alot of cis episodes... Thanks for reading.... Lisa Doctor: Hi Lisa, Thank you for writing to us. Based on the description of your symptoms and clinical course, the most likely diagnosis is multiple sclerosis. I agree in 2001, it was not possible to classify you as MS. Also, if one only has recurrent optic neuritis without involvement of brain and spinal cord, it can not be called as MS (there is an entity called as recurrent optic neuritis). Also, if one has spinal cord involvement and optic neuitis (with normal brain), it could be NMO (neuromyelitis optica). Best wishes, Dr Sudhir Kumar MD DM (Neurology) Senior Consultant Neurologist Apollo Health City, Hyderabad"
},
{
"id": 104365,
"tgt": "Itchy hives on the body after morning walk in summer. Cause?",
"src": "Patient: early morning walking/running, creates allergic like response(itching hives) on my skin across my body. I am facing this issue only when I am doing exercise and is more in summer season. This was not the case previously, i mean this kind of reaction is happening to me recently, earlier I used to do long runs and my body never showed any such reactions. What could be the reason, what might have changed in my body to produce such reactions? Doctor: aNY MEDICINES YOU ARE TAKING FOR OTHER PROBLEMS OFF AND ON LIKE ANTIBIOTICS,PARACETML DISPRIN ASPIRIN OF DIETRY SUPPLIMENTS OR CHANGE IN OIL SOAP SHAMPOO OR OTHER APPLICATION MATERIALS ON SKIN SEE THE CAUSE"
},
{
"id": 161070,
"tgt": "Will the intelligence of a child reduce as his seizures continue?",
"src": "Patient: My grandson has been diagnosed with gray matter heterotopia and he switches between being a sweet boy to a very violent boy. He is very intellegent in most academic areas but seems delayed in social skills and fine motor skills. Will his intellegence reduce as his seizures continue and he gets older? what can we do for him to give him a quality of life. Doctor: Hello,Consult a pediatric neurologist and get evaluated. Recurrent seizure might affect the growth and development of the kid. An MRI brain may be useful for further assessment.Hope I have answered your query. Let me know if I can assist you further. Regards, Dr. Shinas Hussain, General & Family Physician"
},
{
"id": 63327,
"tgt": "Suggest remedy for painful lumps on forearm",
"src": "Patient: My 15 year old son has a lump about the size of a quarter on the top of his forearm. it has two small black dots in the center of it. This just came up a couple of days ago. It is not terribly painful or red. He also has an unusually strong headache tonite and is very tired. Doctor: Hi, dearI have gone through your question. I can understand your concern. You may have some benign cyst like sebaceous cyst or dermoid cyst or some other soft tissue tumor. You should go for fine needle aspiration cytology or biopsy of that lump. It will give you exact diagnosis. Then you should take treatment accordingly. Hope I have answered your question, if you have doubt then I will be happy to answer. Thanks for using health care magic. Wish you a very good health."
},
{
"id": 211592,
"tgt": "Taking zyprexa and paxil for bipolar disorder for concentration. Advised to take ADD test. Correct?",
"src": "Patient: I am taking low doses of zyprexa and Paxil for bipolar disorder. I am 63 and have trouble with concentration. The dr. Thinks I may have undiagnosed ADD. I am nervous about taking meds For ADD,if I am in fact tested to be positive,for fear it will have an adverse effect on what I am already taking. What do you think? Doctor: HIThank for asking to HCMNothing to loose in going through the test OF ADD( Attension deficit test) if it turned strongly positive then can be treated accordingly and if turned negative than you have to celebrate it, just go ahead with full confident that it will be alright, have nice day."
},
{
"id": 36841,
"tgt": "What are the chances of catching Hepatitis C?",
"src": "Patient: I have suspicion the last person I had sex with has hepititus c , i had sex with him twice , once 3 months ago and once 1 month ago , I have now had a liver function and I have to go back for another as my atl was high , I had been drinking the night before the test , what is the possibility of hepititus c , should I be worried , thankyou Doctor: HelloIn your case , you doubted that Hepatitis \"C\" transmitted through sex or not .After going through many studies I would like to say that there is not a chance that you got HVC .Now read this brief study for your information:Patients at risk for hepatitis \"C\" include persons who have ever injected illegal drugs, have received hemodialysis , or were born to an infected mother ( perinatal 0 , health care workers who have sustained needlestick accidents are at risk.I mentioned ( however this is not for you ) so that you can understand that sexual act don't transmit HVC.This is clearly mentioned in the book ,that there is no other way of transmission of HVC , so surely you didn't get any HVC infection by this sexual act .Alt high level may be due to alcohol use , so need not to worry .Hope this will help you."
},
{
"id": 192807,
"tgt": "What causes blood in urine even after 4 days of prostate biopsy?",
"src": "Patient: i went for a biopsy test for my prostate after the test the doctor told me to expect some blood in my urine,stool and semen for one are two days.what i notice, when i go to poo & pee there is blood coming down in the urine.my concern is the doctor says one to two days but now its going day 4 now.what must i do? Doctor: Hello,If the blood in urine persist for more 1-2 days than it's advisable to investigate with urine analysis to check for the presence of pus cells and bacteria to rule out infection. You can revert back with report once done Hope I have answered your question. Let me know if I can assist you further Regards,Dr. Parth Goswami, General & Family Physician"
},
{
"id": 202948,
"tgt": "How to get rid of sperm leakage that causes laziness, dull, headache?",
"src": "Patient: Sir am facing problem... am not still married... weekly once or twice.... sperm will leak... am facing problem in inactive... laziness... face will become dull....little head ache will come... and also excess of heat....Plz suggest me the remedy fr dis... Doctor: HelloThis is known as night fall in common term ( semen ejaculated in sleep ).This is a common feature in young adults after 16 -17 years . This is absolutely normal and nothing to do with laziness or inactive activities.So don't try to read or see sex related material . Always be positive and try to do something good .Still feel something bad , consult a psychiatrist and get his opinion and treatment .Here one thing I want to tell you that you are normal ,so don't trust over quacks.Good luck."
},
{
"id": 215917,
"tgt": "Can oxymorphine be used as an alternative for morphine?",
"src": "Patient: I have been on Oxycontin w/5 mg. oxycodone breakthrough for a couple yrs now. My new insurance company doesn t want to pay the cost of the med. When I had my jaw replaced several years back they put me on morphine which made me very itchy and irritable. The insurance co. wants me to try oxymorphine as an alternative. Doesn t the contain the same morphine that I have a reaction too. I need to be on long term pain mgt. meds. because they help me to make it through the night and get sleep. And the long term doesn t tend to wipe me out as did the short term. Your feeling on that please! Doctor: Hello and Welcome to \u2018Ask A Doctor\u2019 service. I have reviewed your query and here is my advice. Oxymorphine and morphine are almost similar and produce similar pain relief. Oxymorphine is more safe because side effects are minimal with oxymorphine. You can safely take Oxymorphine instead of morphine. Hope I have answered your query. Let me know if I can assist you further."
},
{
"id": 183092,
"tgt": "Suggest remedy for swollen lips with white patches on gums",
"src": "Patient: My 8 month old son fell face down off the bed and was bleeding from his mouth and nose and it looks like there's a scratch on his upper lip. His lip is swollen and has white patches on his upper gums. Do you think it's from his two bottom teeth biting into it? What should I do? Doctor: Hello, thank you for consulting with healthcaremagic. It can be possible that when he has fallen from bed, he got hurt on the gums either from lower teeth or from floor, now this has caused a traumatic ulcer on the gums.Better that once you should visit a pedodontist and get complete examination of gums done. And even start with a topical anesthetic gel on ulcer for relief.Hope it will help you."
},
{
"id": 121755,
"tgt": "Suggest remedy for pain in the hands and fingers",
"src": "Patient: I have just gone for ablod test because mt body is painfull allover no matter what I do walk, pickup my armers to get some thing higher ,my hands and finers are painfull In fact one big pain I don t whant to live like which I have binging for the last Two weeks and thepain tablels dopn t help onlly for a wiill and It does half a job Doctor: Hello, Your symptoms could be related to a connective tissue disorder or arthritis. For this reason, I recommend consulting with a rheumatologist and performing some tests: - muscle enzymes (CPK) - complete blood count, PCR, ESR for inflammation - uric acid plasma levels - ASOT titer - ANA and ANCA antibodies titer. Further tests may be needed. You should discuss with your doctor on the above tests. Hope I have answered your query. Let me know if I can assist you further. Take care Regards, Dr Ilir Sharka, Cardiologist"
},
{
"id": 182664,
"tgt": "Suggest treatment for dental abscess in the mouth",
"src": "Patient: my dentist asked me to have xrays on my teeth because of a tooth seeping a little pus,thinking it was a absess now she has refered me to dental surgen because of a shadow on the jaw or under the wisdom tooth could it be something common and not somthing sinister like cancer Doctor: Hello and welcome.Thanks for sharing your concern.As you know the referral causes benefit to the patients,in a way that you get to know the exact cause and appropriate treatment.Therefore please do not worry at all.your present dentist would have asked you to see a dental surgeon,so that the lesion in relation to third molar could be identified and well diagnosed.How do you know it is cancer?The features of cancer are different.It could be a cyst or tumor(that is a mass of tissue).So please do as suggested and leave your worries to your concerned doctor after explaining your troubles.Wish you good health.Thanks.Take care."
},
{
"id": 28534,
"tgt": "What diseases can I get from a used needle?",
"src": "Patient: Hi, I accidentally pricked my thumb last week with my other half\u2019s used insulin needle. He doesn\u2019t have any disease but I am now worried that I could become unwell with something nasty like HIV OR HEP B. I didn\u2019t think much of it at the time but now I\u2019m starting to worry. Doctor: Hi, 1. First of all, it was your husband's used needle. I would like to take it as obvious that you know your husband's infection status e.g HIV, Hepatitis B and Hepatitis C. 2. It says if the source (your husband in this case) is negative of any kind of infection then you need not to worry. 3. It also says that if the source is suffering from any of the infection described above then it's better to take your laboratory test in the window period (entry of organism to detection of antibodies inside the person) itself and start the medicine for that. Hope I have answered your query. Let me know if I can assist you further. Regards, Dr. Harshita, Infectious Diseases Specialist"
},
{
"id": 117349,
"tgt": "Is exforge taken in case of elevated microalbuminuria on blood test for fever and flu?",
"src": "Patient: Hi, I was sick (Flu and Fever) and was intomedication. I had a blood test upon the request of my cardiologist and found out that the microalbuminuria is almost 500. I am taking also exforge 10/160. Could my microalbuminurea high level resulted by medication or the blood pressure medicine? Doctor: microalbuminurea indicates vascular insufficiency in either diabetes,hypertension or kidney disease itself and your cardiologist may proceed to work on you on these lines tofind the cause and treat accordingly.some more tests like blood sugar, urea/creatine,GFR etc may be advised.he shall determine after tests if your medication needs modification after knowing the cause"
},
{
"id": 54199,
"tgt": "Suggest treatment for high urea,creatinine and potassium",
"src": "Patient: age 73, male, already treated for high urea, creatine and potassium in blood and was reduced to normal level. but, after week of peiod now he is having the problem of irritation during urination. Urine analysis reveled albumin trace and 15-20 pus cells. what could be the diagnosis and treatment. Doctor: HiThis is urinary tract infection showing a lot of pus cells. Take cranberry juice & tablet ciprofloxacin 500mg twice daily for 5 days. If having pain & fever, then a simple painkiller can be taken like tablet Paracetamol 500mg\u00d7SOS.Thank you"
},
{
"id": 137640,
"tgt": "What causes sharp stabbing pain in the arms?",
"src": "Patient: Hi my mother 64 yr old caucasion woman. Smoker 40 yrs, weight 8 stone, 5.1 in height. Irish. She has no health issues that we are aware of. She has shoulder pain across her shoulders and shooting pain down both arms, pain comes at different times, it nearly catches her breathe. It only last a short period of time 30 seconds or so. Can you advise. Doctor: Hi,Thanks for your query.Pain in shoulders and shooting pain down both arms is most probably due to cervical spondylitis. In this condition there is nerve compression in neck due to disc herniation in spinal canal.I would recommend her following.1.Get x ray of the neck to confirm the diagnosis.2.She need to avoid alcohol and smoking.3.Avoid keeping neck in same position for long time. Avoid using very high pillow.4.Intermittent cervical traction exercises will be helpful for you.5.Take Pregabalin M one daily a night time after discussing with her doctor.I do hope that you have found something helpful and I will be glad to answer any further query.Take care"
},
{
"id": 65895,
"tgt": "Suggest treatment for bump apeared on forarm",
"src": "Patient: A few weeks ago a pimple like apeared on my forarm that was itchy of and on. I ignored it but must have itched it unwillingly as well as iritated it during showeres. It has gotten bigger, rasied up with tough/ruff skin and bumpy. itches on and off, when a shirt is over it but I can always 'feel' it there. Doctor: Hi, dearI have gone through your question. I can understand your concern. You may have lipoma, neurofibroma, dermatofibroma or other soft tissue tumor. You should go for biopsy of that lump. It will give you exact diagnosis. Then you should take treatment accordingly. Hope I have answered your question, if you have doubt then I will be happy to answer. Thanks for using health care magic. Wish you a very good health."
},
{
"id": 162303,
"tgt": "What causes memory loss and hearing problem in kids?",
"src": "Patient: when i was young i m good at memorizing thing but now i can t memorize a thing and i always forgot where i put things. it starts on my highschool day my brain felt numb like there is some king of electricity pass it. i can t grasp new things and i can t hear clearly. my doctor said its fine but i m worried. please help-! Doctor: Hi, Are you on social media mobile online games. How is your sleep pattern. How is your appetite and diet. Are you stressed. Tension headache anxiety can cause such symptoms. Hope I have answered your query. Let me know if I can assist you further. Regards, Dr. ASHWIN BALIGA, Pediatrician"
},
{
"id": 155086,
"tgt": "Could be a black skin like thing appearing in stool be related to stomach cancer?",
"src": "Patient: HiMy doctor has made for me an colonoscopy test this morning and so I still have a bright blood in stool. But I have noticed Black skin in my stool looks like apple skin (I did not eat apple) is that a blood and should I ask my doctor for anther test like endoscopy to see if I have a stomach cancer? Doctor: Hi, dearI have gone through your question. I can understand your concern. Bright bleeding in stool comes from lower gastrointestinal tract. If you have dark bleeding then it comes from stomach. Still you should go for endoscopy to search the cause of bleeding. Then you should take treatment accordingly. Hope I have answered your question, if you have doubt then I will be happy to answer. Thanks for using health care magic. Wish you a very good health."
},
{
"id": 201725,
"tgt": "Suggest treatment for red itchy spots on the penile head",
"src": "Patient: my husband has small red spots on the head of his penis I am concerned and have not had sex with him because of this what are the causes of this I am clean and never had an illness of any other sex partner however he is showing me these red spots says they itch are slightly painful and he tried squeezing them and it hurt and there was some white liquid HELP ME Doctor: Thanks for contacting HCMYou are concerned about your husband having small red bumps on his penis. From you description he may have all allergy to something. Most likely it could be from detergent or fabric softeners. I recommend changing how you wash his clothes and to try hydrocortisone cream once daily on the red bumps.Hope this answers your question. Please contact us again with your health care questions and concerns"
},
{
"id": 214420,
"tgt": "Suggest home remedy for upper right sided pain after eating",
"src": "Patient: i have been having a sever pain on the upper right side,it came back after i ate 1/2 of a hamburger , i was able to drink a bottle of water and lay down with pressure on pain, in about 1/2 hour it went away but i could literally feel food traveling through my system. i have history of gallstones by ultra sound but no pain . i wonder if it is necessary to go to hospital(which I hate- Doctors do what they want don t listen to you) or if there is a home remedy to get rid of this problem. Doctor: Hello,Since you have gall stones confirmed by ultrasound and the pain occurred after a fatty meal, it is very likely to be a colic arising from the biliary system. You can try an antispasmodic like buscopan and the pain may go off. If it persists or returns, I think you should go to the hospital because a gall stone may have entered and is causing a block in the biliary tree or there may be an infection.I hope you will find a compassionate doctor this time. Remember to ask them one question at a time and most doctors are willing to help you.Hope you get well soon.If you have any further query, you can contact me and I shall be happy to give my opinion.Dr. Noble Zachariah, Kuwait"
},
{
"id": 187957,
"tgt": "What could be a swollen painful bony lump below a tooth?",
"src": "Patient: Hello, I have had a bony lump on my jaw below a tooth broken off at the surface for some time now. It didn't hurt and would get larger and smaller. But overnight that size of my face has swollen. It does not hurt like abcesses I have had in the past. I am wondering what is going on. Doctor: Hello and welcome.Thanks for sharing your concern.Please understand this fact that it appears to be an abscess only.Hard board like swelling could be cellulitis too.it is a kind of long standing infection of spaces in the jaw.please visit your dentist soon get it thoroughly examined and start with your antibiotics and analgesics soon.Do not ignore as prolonged delay might cause drastic damage by spread.Hope it helps.Thanks.Take care."
},
{
"id": 171637,
"tgt": "What causes hanging hair in the rectum of baby?",
"src": "Patient: hi, my baby is 9 months old today while changing her diaper i found out hair hanging out from the rectum. i tried to pull it but its very hard n i was unable to pull it. i am quite worried as to what is it and why i am unable to pull it.Has anyone had such an experience ? worried mum !! : ( Doctor: Hi,Welcome to Hcm,I understand your concern. Hair seen like that is not common in children of this age. If its coming from inside that could mean he has swallowed some hair which is tying to get expelled undigested and getting entangled inside. If I were your pediatrician, I would refer you to a pediatric surgeon who can examine his rectum and try to take it out in a safe manner. Dont worry, the problem can be completely cleared. Take care."
},
{
"id": 5986,
"tgt": "Undergone hysterolaparoscopy diagnosis, uteraine cavity normal, ovaries adherent to the back of uterus. Can i conceive?",
"src": "Patient: im 29 yrs old i have no child for 5 yrs now i undergone hysterolaproscopy diagnosis on that i got a result is uteraine cavity normal and it had omentum adherant to posterior region .the both ovearieswere adherent to the back of uterus . tubes were tortuous . no evidence of endometriosis .my question is can i have a child? Doctor: Hello. Thanks for writing to us. Tortuous tubes with adherence of omentum to the back of uterus is likely to be due to chronic pelvic infection in the absence of endometriosis. You need to take appropriate antibiotics and your chances of conception are still present. I hope this information has been both informative and helpful for you. Regards, Dr. Rakhi Tayal drrakhitayal@gmail.com"
},
{
"id": 57476,
"tgt": "Will taking indocap increase my liver problems ?",
"src": "Patient: i ate 10 indocap s r(75mg each capsule) today around 12:30am.i have not taken anything till now,what to do??????? already i m undergoing treatment for my enlarged liver .medicine name ---golbi 300. nw i feel like vomiting(sometimes),drowsinesss,rest u knw what problems i can have,plz reply soon Doctor: Hi and welcome to HCM. Thanks for the query.You need to visit doctor as soon as possible. this will prbably not cause serious consequances but you need to visit hospital and do further tests as this is too high dosage taken at one time especially considering your liver problems. Wish you good health. Regards"
},
{
"id": 73477,
"tgt": "Which bacteria causes shortness of breath,hyperventilation?",
"src": "Patient: what is the likely bacteria causing shortness of breath, hyperventilation, blue tinge around mouth, that fails to respond to amoxycillin, augmentin and clarithromycin but responds to ciprofloxacin? this infection lasted for around 12 weeks in a normally healthy adult. Doctor: Thanks for your question on Healthcare Magic.I can understand your concern.Klebsiella or legionella can cause severe type of lung infection (pneumonia).These bacteria cause rapid deterioration.So patient is hypoxic (low oxygen) from the beginning.Hypoxia causes hyperventilation, shortness of breath and cynosis (bluish discoloration of lips, tongue).Ciprofloxacin is usually effective in these infections.Hope I have solved your query. I will be happy to help you further. Wish you good health. Thanks."
},
{
"id": 221006,
"tgt": "What causes numbness in the arms?",
"src": "Patient: Why has my right arm been falling asleep for the past 2 months. I ha e a 22 month old that is 30 pounds and I lift him and hold him with that arm. I was pregnant and miscarried 4 weeks ago. I am also over weight and was told a year ago my cholesterol was up but not enough for meds. It happens from mostly my elbow down but I do feel it in my upper arm a little. Mostly at night when I curl my arm under me. Tonight it started when I sat down for the first time leaning to the right and curling my arm. It is hard to get it to completely go away and it is partially numb now curling my arm to type this. I am nervous. There is a possibility I am pregnant again as my doctorsaid it was ok. I have Read about pregnancy induced carpal tunnel syndrome or could it be something worse? Thank you. Doctor: Hi, have read your query & understand your concern. Tingling , weakness & pain in fingers are the symptoms of carpel tunnel syndrome. Aggravation of symptoms on movements like .. lifting child / curling the hand back while sleeping ) suggest that,the numbness in your right arm, seems to be due to nerve compression at neck ( cervical rib )/ spinal level ( cervical spondylosis.) You need to consult a orthopedic doctor to specify the site of pressure by physical examination & investigations like MRI/ CT scan( as per need.) followed by specific treatment.(medicines/ physiotherapy/ surgery in few cases ) Thanks."
},
{
"id": 169453,
"tgt": "Suggest treatment for fever in a child",
"src": "Patient: Our child, 2.5 yrs old boy Sumeru, has fever 103 - 104 degrees from last 4hrs. Fever started suddenly at around 7pm today after he is back from near by children play area/park. He looks tired and sleeping there after. Please suggest what should we do? rush to nearest hospital or wait till morning? we gave him Acetaminophen oral suspension usp syrup 5ml at 7:30pm. Temperature has not come down yet. We are checking temperature every 30mins. Shall we give one more dose (5ml) of Acetaminophen oral suspension usp syrup at 12:00am, after 4hrs gap? please suggest. -- Vinayaka & Hema from Bangalore Doctor: Hi,It seems that he might be having viral fever producing high temperature.you can repeat the Acetaminophen syrup dose after 4 hours if fever is there.But important is to give plenty of water and liquids and keep him well hydrated which will bring down the temperatureY.Ok and take care."
},
{
"id": 22552,
"tgt": "Suggest treatment for a heart ailment in a 50 year old lady",
"src": "Patient: hello doctor..my mom-in law is an heart patient from her childhood...her age is arnd 50..she has been consulting many doctors from childhood..sometimes she becomes fit n sometimes feels unhealthy..now she is not eating well n looks tired ..she is having BP..n now she is not intrested in talking..she always sits quiet..n seems abnormal...please help..the medicines which she is having now is glucobay M50,trinicalm forte,trinicalm plus,neurobion forte and betaglim M1 Doctor: Hello,The kind of description you have given it seem she is having psychological problems like depression or mood disorders. I would advise to talk to her ask for any specific complaints like any chest pain, shortness of breath or any complaints. If not then it seems she is having psychological issues and get one psychiatry consultation. Also you should regarding any worries or stress is disturbing her and try to council her. Otherwise healthy lifestyle like avoiding void fatty, oily and high calorie diet. Have low salt diet and monitor blood pressure regularly thrice a day for one week then once or twice a week. Regular exercises like brisk walking, jogging according your capacity at 30 min a day and 5 days a week. Lots of green leafy vegetables, fruits, fish once or twice a week, avoid meat. Avoid smoking and alcohol if any. There shouldn't abdominal fat deposition or obesity.Hope I have answered your query. Let me know if I can assist you further.Regards, Dr. Sagar Makode"
},
{
"id": 49896,
"tgt": "Continuous nausea, vomiting, severe pain. Cutting back on gluten and alcohol. Possible kidney stones?",
"src": "Patient: My 27 year old daughter is an Rn and has suffered with nausea, vomiting that just will not stop and severe pain. Thought maybe pancreatic or gallbladder. Now she thinks maybe kidney stone. Has refused to go get testing. Any thoughts. Seems to be gluten intolerant so is cutting that out and also has been known to drink too much alcohol so is cutting back on that. Doctor: welcome to HCM!in order to make the diagnosis,exact location of pain must be known.its loalised or shifting pain.there are many causes with above complains.i advice u to get a physicians advice.meanwhiletake simple paracetamol and an antiemetic."
},
{
"id": 131379,
"tgt": "Reason for pain below rib cage with history of liver cyst?",
"src": "Patient: I have been having a pain in my right side below my rib cage on and off for a couple months now. Last week it was real bad. I was told last year that I had a small cyst on my liver. I went to my gastrologist a couple months ago and he did a ultrasound and said he couldn t see anything. Doctor: What is the type of your hepatic cyst? and what is the type of your pain ? Is it colic or stabby?. you should do kidney and liver function tests to exclude any disorders in liver or kiney. If there is abdominal colics and swelling, you should do ultrasound to exclude colon disorders.I am pleased to follow you."
},
{
"id": 85087,
"tgt": "Can an epilepsy patient take Ginette 35 tablet?",
"src": "Patient: Respected doctor, I am an epilepsy patient and am taking encorate chrono 200 twice a day. Now i am advised to take Ginette 35 tablet as a measure for hormonal treatment. I would like to know whether i could take ginette 35 since i am an epilepsy patient. Doctor: Hi, It is not advisable to take these two medications together. Taking Ginette-35 while on Encorate chrono-200 can cause loss of seizure control or symptoms such as tremors, poor muscle coordination, increased seizures, and changes in behavior. It can also result in increased side effects of Cyproterone acetate one of the active drug present in Ginette-35. Consult your doctor for a dose adjustment or special tests to safely take both medications. Hope I have answered your query. Let me know if I can assist you further. Take care Regards, Dr. Mohammed Taher Ali, General & Family Physician"
},
{
"id": 124913,
"tgt": "What causes wrist pain while sneezing and taking deep breath?",
"src": "Patient: I have rheumatoid arthritis, have had an anterior cervical discectomy with fusion (4-23-12) and bilateral total knee replacements (07-03-12). I am a 52 year old female. When I take an unexpected deep breath like a yawn, sneeze, or a deep sigh, I get pain shooting through both of my wrists and hands. Doctor: Hi, Pain in wrist &/or hands upon sneezing may be because of cervical spondylosis. You may take vitamins and minerals and antioxidants along with physiotherapy. Hope I have answered your query. Let me know if I can assist you further. Regards, Dr. Nirmal Chander Gupta, Orthopaedic Surgeon"
},
{
"id": 44041,
"tgt": "Miscarriage, blighted ovum. Should I start Ascard before conceiving?",
"src": "Patient: hello...m dr maira..i had 2 missed abortion this year..first in jan had blighted ovum and second last month due to baby stopped growing in 6-61/2 weeks..in ma second preg dr prescribed cyclogest and aspirin but again missed abortion..now dr is saying i should start ascard right now before concieving..i want to ask whether i should start ascard right now or i should wait when i concieve next time??and whether it has positive effect of taking Ascard before concieving??pls answer me drs!!thanx!! Doctor: Hi, There is no harm in taking aspirin before conceiving. But there is no proven benefit also. Starting aspirin may not solve your problem. You need some investigations to find out cause for repeated miscarriages. Though these investigations are costly, if abnormal some factors can be corrected to avoid miscarriage. You can ask your doctor about those investigations. Some basic cheap investigations are blood sugar level, VDRL, USG (Transvaginal) to find out uterine problems. Costly investigations like karyotype can be done or deferred according to your doctor's advice. Wish you good health."
},
{
"id": 78247,
"tgt": "Suggest treatment for trapped chest wind",
"src": "Patient: hi I m a 22 year old male who is suffering with trapped chest wind and slow burping helps,I swallow air to try and relieve it but often it makes it worse,especially while training,I ve left coffee as well as smoking,nothing seems to be working,I had an endoscopy performed and everything was clear,every night I have these heart pressing pains,feeling like a heart attack a night,my gp performed a few tests to see if it was my heart,but saw my heart was strong,hours after eating it can strike and has no set time,although worse at bed time,doctor already gave me 3 sets of adco controment (10 in each pack 3 times a day before meals) with no success,I do body building so I am fit and active,I don t drink alcohol.I also get a pain under my breast bone as well but as the endoscopy proved I have no ulcer problems,I feel it could be trapped wind there too,earlier on while gyming I had a pain in the upper sternum(where the collar bone meets)with swelling,could it be an injury that caused the trapped wind saga,its been this way for over a month,first episode landed me in the er,for heart attack,the pains aren t that bad anymore,I used to have heart burn but after my omeprazole 20 use that went away,everything is fine except the winds!are there any suggestions from you!thanking you Doctor: Thanks for your question on Health Care Magic. I can understand your concern. By your history and description, possibility of GERD (gastroesophageal reflux disease) is more. GERD is due to laxity of gastroesophageal sphincter. Because of this gas from the stomach tends to come up in the esophagus and cause the symptoms. And you need to follow these lifestyle modifications for better symptomatic relief. 1. Avoid stress and tension. Be relax and calm. 2. Avoid hot and spicy food. Avoid junk food. 3. Avoid large meals, instead take frequent small meals. 4. Start omeprazole. 5. Go for walk after meals. 6. Keep 2-3 pillows under head in the bed to prevent reflux. Don't worry, you will be alright with all these changes in lifestyle. Hope I have solved your query. I will be happy to help you further. Wish you good health. Thanks."
},
{
"id": 114110,
"tgt": "I have pain in my lower/middle back. Can falling from height on the back cause internal wounds or be just a muscle sprain ?",
"src": "Patient: Hi, a couple days ago I fell about 6ft in the air and landed on my back, and since then I ve had pain in my lower/middle left side of my back when I move or try to get up from sitting, could this be internally bruised, or a muscle sprain or something more serious? Doctor: Hi , It is better to show it to an ortho surgeon so that he can get your Xray or CT done. If you are moving it doesn't seem to be serious. In serious injuries one can't move at all and there is numbness in lower limbs but please do not neglect it and consult as soon as possible. Take care."
},
{
"id": 124769,
"tgt": "What could causes dorsal ganglion?",
"src": "Patient: what causes dorsal ganglion..ystdy doctar said that what i am having hard thing in my right wrist is ganglion..and he said if u put pressure on my right wrist it my devolope more..i am worried abt ths...is thr any way to control it..any of ood habits causes dorsal ganglion? Doctor: Hi, A dorsal ganglion is a benign swelling in the wrist (in your case) and it has a tendency to go down on its' own also. So if it is not troubling you, then ignore it else consult an orthopedic surgeon for local steroid infiltration. Hope I have answered your query. Let me know if I can assist you further. Regards, Dr. Gopal Goel, Orthopaedic Surgeon"
},
{
"id": 39554,
"tgt": "What could pain and semi-clear discharge secretion indicate, Ciprofloxacin offered little relief?",
"src": "Patient: Hi, after a warm shower, there is a slight burning pain about an inch from the tip or head inside. At this time a small amount of semi clear discharge can be squeezed out. The pain in only a brief couple of secs. Just finished a 7day bout of cipfroflaxon. Not much help. Your opinion is welcomed. Doctor: Dear FriendWelcome to Health care magic. I am Dr Anshul Varshney , I will shortly try to help you with my opinion.In this case you should see if you had any unprotected sexual intercourse that is high risk.I would suggest you:1. Pus culture2. VDRL3. HIVIf this is infective then antibiotic like Augmentin would help.Based on the available details, this is my best advice for you. You should discuss with your doctor and share my opinion, I am sure he/she will agree to my opinion. If you wish to discuss the problem further , you may please ask.Stay Healthy."
},
{
"id": 209544,
"tgt": "Suggest treatments which can stabilize the mental condition",
"src": "Patient: Good Evening Doctor, My wife age is 30...now a days she is getting very anger with me and my children too...the reason is she was thinking that i have affair with my brother wife(14 hours journey from my city).Very politely i explained her so many times but no use.Now currently she is not sleeping properly/fighting for small issues/beating children very harshly/warning me for(sucide)...what should i do Doctor: DearWe understand your concernsI went through your details. I suggest you not to worry much. The best course of action could taking your wife to a psychological counselor. Any amount of explanation from your side shall not suffice the in this juncture. Over and above, she will get a lot of \"circumstantial evidences\" in regards to this. Therefore never try to explain (I know you did a lot of explanation and nothing came your way).Now she is may not be under a mental disorder like Obsessive Compulsive Disorder. The thought process might just be taking the course of obsession. If all of the family members ignore her comments and obsessions for some time, this problem could be cured. Do consult a psychological counselor. If you require more of my help in this aspect, Please post a direct question to me in this website. Make sure that you include every minute details possible. I shall prescribe some psychotherapy techniques which should help you cure your condition further.Hope this answers your query. Available for further clarifications.Good luck."
},
{
"id": 19325,
"tgt": "Does high BP have a relation to fibroid?",
"src": "Patient: Hi Doctor, I m 45 and has had fibroids since 23. Have 2 boys 16 & 13. While pregnant had low blood pressure but after delivery on routine checks in 1999 had slight high blood pressure. Was not medicated but jus monitored. Had myomectomy in 2001 & 5yr Mirena. Fibroids are back with vengeance and in Oct last yr put on Nifedipine Sustained release tabs 20mg coz BP was 144/104. Get terrible headaches from the med. Any relation between fibroids & high BP & is that the best medication? I m slightly over-weight of late. Thank you Doctor: welcome to hcm ..no there is no relation between fibroids and high bp .as nifedipine gives you headache you can take other more tolerable drug as amlodipine tab 5 mg once a day ...please take care of your weight ..as reduction of weight also decreases bp"
},
{
"id": 23733,
"tgt": "Can these drugs interact adversely?",
"src": "Patient: I am 43 year old, 5'6\", 82 kg wt. Indian. I was diagnosed with hypertension 7 years back. I am taking Tozaar 25 and Tonact Tg for the last 3 years (as my TG level was also high). Though my TG level is normal now, I am continuing this medicine. Recently I have also got some knee pain and the doctor has given Hilin-GM, Ketorol and Belosix.My questions are: 1) Can all these drugs interact and create any adverse reaction? 2) Can I stop Tonact TG and Tozaar 25 as my BP and TG level are normal? 3) Can Hilin_GM to be continues for long or can be stopped after 10 /15 days? Doctor: Hi hope you are doing good. Let me answers your questions. 1) Ketorol(ketorolac) can interact with your anti hypertensive medications, so do not take this beyond prescribed duration. Constant monitoring of blood pressure is a must. Apart from this no major drug interactions present with other drugs. 2) No.You should NEVER stop anti hypertensive and antihyperlipidemic drugs just because you have normal readings now. You have normal values because of the medications. If you stop it, then blood pressure and lipid values will increase and can lead to dangerous consequences. 3) Hilin-GM (Diacerin + glucosamine) can be taken beyond 10-15 days. But after 3 months, the improvement in symptoms must be measured and then decision should be taken about continuing. Belosix (febuxostat) must be taken for long period time. It is used to lower Uric acid levels in blood. Periodic measurements of uric acid levels is required for dose adjustments. Hope the above information answers all your queries."
},
{
"id": 114048,
"tgt": "Lower back nerve pain,and small round bruises which spread quickly",
"src": "Patient: Lower back nerve pain , and small round bruises which spread quickly I am 34, female, about 175lbs with minor herniation of discs in lower back (no major health issues).first syptom was sever lower back nerve pain, similar to sciatic pain, but more severe and usual stretches and positions did not relieve it. Pain is constant, sitting, standing, walking.. a few hours later, I saw, small bruises forming all over my knee , then more bruises, all small and round (like I was poked with a finger ). Larger bruising, and visible veins(not notmally visible) on my feet. Bruses seem to quickly spread...knees, legs, ankles, feet, arms, hands. no other pain than the back... I went to an urgent care center for the back pain , noticed the bruising in the doctor s office, mentioned it to the doctor and she said it was nothing and gave me muscle relaxers. Now the bruising has spread.I did not take the muscle relaxers. Doctor: Welcome to Healthcare MagicAre you allergic to anything you would have taken recently. Any itching over the bruising. It can be an urticarial reaction but if there is frank bleeding spots it could be bleeding disorder. Any fever, trouble in urination, abdominal pain. It warrants proper checkup and investigation. Avoid any new drug you may have started under medical supervision as well, after confirmation that a drug as such that may be causing a reaction. Get a second opinion if it gets worse or any abdominal pain, pain in the sides of the back and blood in urine. Take plenty of fresh fruits and vegetables. Drink plenty of water. Consult your Doctor as soon as possible."
},
{
"id": 15415,
"tgt": "Rashes on upper chest, neck, jaw line, itching, burning. Using eurex cream. Cure?",
"src": "Patient: I have a hives type rash on my front upper chest and neck and jaw line. I have had it for a few days and been treating it with antihistimines and Eurex cream. Sometimes it seems to be reducing in severity and then it will flair up again. Should I take different antihistimines. I have loiritidine and piriton and have taken these regularly or more than recommended on the packet but sometimes itch and burn so bad. Doctor: HI I THINK IT IS ACUTE URTICARIA.CHANGE THE ANTIHISTAMINES TO ATARAX AND LEVOCETRIZINE CAN CURE IT,.ADD A SHORT COURSE OF STEROIDS AND CALAMINE LOTION TO THE TREATMENT"
},
{
"id": 149389,
"tgt": "Severe pain in neck, arms and hands. History of having cervical fusion surgery. MRI shows spinal stenosis. Have degenerative disc disease. On Medrol. Suggest",
"src": "Patient: Hi, I had to have cervical fusiion surgery in 2004 from a compression fracture. I am now having neck pain, pain in both arms and hands. This pain is worse than it was in 2004. The last MRI that I had done shows that I have spinal stenosis but in the last two days, my symptoms are so bad that I am scared. Should I see a doctor immediately? I am currently on a Medrol Dose pack to help alleviate any inflammation. I am having a hard time moving my head up or back. I also have degenerative disc disease. Doctor: Hi, it appears that you had recurance of the degenerative disc disease. I adviseyou to consult an orthopedic surgeon for diagnosis and treatment. I to my patient with such symptoms prescribe neurotropic injections for 10 consecutive days, and one injection every week. Avoid taking potato, other tubers, tomato, egg, chicken , and sea foods till the pain is relieved. Thank you."
},
{
"id": 78592,
"tgt": "What causes left sided chest pain?",
"src": "Patient: I have pain in the left side of my chest below the breast but near the ribs. It hurts if I try to take a deep breath or cough or If i strain to go to the bathroom. it also hurts when I move into a lying down position or when getting up. I feel it when walking. It feels like the burning sensation you get when you've had surgery but I haven\"t. It started 8 days ago and has not gotten any better. My back is starting to hurt now from compensating for the pain. I was in the ER on day 4 because my doctor wanted me to go there for obvious reasons - to rule out any heart-related issues (which was ruled out). They also checked for a blood clot in the lung which was ruled out. The ER doc said it was viral pleurisy but I don't have a cough, etc. It feels more like some kind of trauma-related pain / condition. I do not believe it's pleurisy but what else can cause this kind of pain? It feels more like some kind of tear or something? Doctor: HI,Welcome to HCM.Based on the facts of your query,you seems to suffer from-Pleurisy which is a neuralgic mimicking pain.Other cause could be nerve entrapment due to positional adjustment of the rib cage,with aging.This would explain the trauma-related pain you experience.Chest X-ray would show confulence of ribs with the aging/and disease changes/or adjustments.Hope this reply would help you to plan further treatments with your doctor.Welcome for any further query in this regard.Will appreciate writing your feedback review comments,to help the needy patients like you at HCM.Good Day!!\u00a0\u00a0\u00a0\u00a0\u00a0Dr.Savaskar,Senior Surgical SpecialistM.S.Genl-CVTS"
},
{
"id": 64430,
"tgt": "Suggest treatment for a hard itchy lump on the back",
"src": "Patient: A couple of weeks ago I woke up with a hard itchy lump on my back left side. A short time later another one appeared in the the same place on the right side. They went away in a couple days they were very itchy. Last night I had some irritation in my left knee. This morning there was a hard itchy lump on my left knee and a hard itchy lump appeared on my right knee shortly after. What is this? Doctor: Hi,Good Morning,Thanks for your query to HCM online Clinic.I have read your query in details.-Dear In My opinion,this simultaneous lump on knee is mostly -Erythema Nodosum.-This indicates a Auto-immune reaction activity in your body over sometime.And indicates-?Sarcodosis,crohns , ulcerative colitis?or drug induced reaction?infection induced -specially streptococcal-which is very common throat infection etc.-Remedy -would be to Consult a physician-who would-do CBC/ESR/CRP and would fix the cause of it in your case.-Tb NSAID / Tb Aspirin to reduce anti-inflammatory reaction -Steroids to reduce the auto-immune reaction causing these lumps-Colchicine in aspirin induced cases would reduce this lumps.-Usually these lumps go in 3-6 wks time on their own.They could be treated if it does not go or causes more worry and pain.Hope this would resolve your query.Wellcome for many more queries to HCM from your friends and you in time to come.Have a Good Day."
},
{
"id": 133449,
"tgt": "How to confirm a jaw fracture?",
"src": "Patient: 5 weeks ago I under went double jaw surgery and 2 weeks I had to have surgery again on my lower jaw cause I fractured my left side. My lower jaw on the left side clicks every time I open or close my mouth my oral surgeon just put elastics in 2 days ago to close my open bit and ever since then the clicking has gotten worse and the pain is unbearable it kills to open my mouth or even talk. On my left side where I had my fracture just below it I feel a space like I didn t when I first fractured the same side. What should I do? Wait and see if it will go away or call my surgeon? Did I fracture my jaw again or is a screw lose or could I just be a muscle issue Doctor: hi,thank your for providing the brief history of you.As the TMJ surgery is a complicated one. Taking a proper care is of due importance. Also, the pain has to be high since it is close to the brain and the muscular system is too less to provide a big support. If the muscles were bulky enough like thigh or hip then things were different but any increase in the pain you need to contact your Surgeon again as smallest injury when it becomes big no can predict.Also, you need to be very careful as the TMJ surgeon tries their best and the effort from them is really incredible. what you eat, how you chew, how you posiuon yourself in sleeping etc makes a very high understanding here. Also, I appreciate your patience as well in this duration of the fracture as only you will be knowing how much pain it will be.I don't think it must have got fracture but still having a word with the surgeon will surely help us asssit further.RegardsJay Indravadan Patel"
},
{
"id": 29238,
"tgt": "What causes chest pain when diagnosed with malaria?",
"src": "Patient: I have a question about a young man who is dxed with malaria and typhoid but he is already in the hospital. NO doctors there due to its being midnight. He is complaining about chest pain, short of breath, left arm and left leg hurting. Any ideas? Thanks. Doctor: Hi. In my opinion he should inform the duty staff, they will evaluate the symptoms and will call the doctor if required.These symptoms might be benign and part of his illness. But chest pain and shortness of breath can be due to pulmonary embolism. Pulmonary embolism means blockage of blood vessels of lungs due to clots. It causes chest pain and shortness of breath along with other symptoms. Patients who are ill and bed bound are at increased risk of it. It is a life threatening disease, so it should be ruled out.Hope, my answer was helpful. Regards."
},
{
"id": 122867,
"tgt": "Why does my feet sting and bruise after standing?",
"src": "Patient: yes, my feet starts to sting then a blister or bruise appear more like a burise, what causes that, its being going now for about 3 to 4 months now, i stand alot and my right foot hurts all the time, i have had surgery in the past, but im concern about the stinging and brusing, Doctor: Hello, It could be a ligament sprain or contusion. As of now, you can use analgesics/anti-inflammatory combination like aceclofenac/serratiopeptidase for symptomatic relief. If symptoms persist better to consult an orthopaedic and get evaluated. You can opt for an MRI scan for further assessment. Hope I have answered your query. Let me know if I can assist you further. Regards, Dr. Shinas Hussain, General & Family Physician"
},
{
"id": 224751,
"tgt": "Is pregnancy possible while taking contraceptive pill?",
"src": "Patient: hi doc, i have an unprotected sex on 7th of january this year, i took nordette as ecp (4+4) on the same day that we had sexnow on the 13th of january i had bleeding it lasted until yesterday (jan 15 2014), today (jan 16 2014) i had no more bleedingmy last period was december 26-30( lasted for 5 days, my normal duration is 5-7days)what im asking is, is it safe to assume that im not pregnant since i have my withdrawal bleeding already? or should i wait for my expected period which is expected to be on jan 26 2014what are my chances of being pregnant? please help me im so nervous i dont want to get pregnant yet Doctor: Now you can get pregnancy test and repeat it after one week. If both are negative then you are not pregnant."
},
{
"id": 129096,
"tgt": "How to treat the bruise on my 16 month's old cheek?",
"src": "Patient: Hello i have a quick question. My 16 month old hit herself in the cheek on Monday night. An hr later the bruise began to show. Now its black and red and has a huge bump. It's also a bit black on the inside. It seems like it's getting bigger and darker. Should we take her to the ER? Doctor: Hello,Thank you for using Healthcaremagic.I read your question and understood your concern.I think you should take her to ER to ensure that there is nothing serious with her.Dr. Selmani"
},
{
"id": 127692,
"tgt": "Is mild pain in the thigh post injury a concern?",
"src": "Patient: I fell a week ago on a brick sidewalk when it was raining. When I fell I bruised my hip and that pain went away after a couple of days but I m the middle of my thigh I have this dull ache when I walk or stand. Should I see a doctor about this or is it likely something that will go away? Doctor: Hello and Welcome to \u2018Ask A Doctor\u2019 service.I have reviewed your query and here is my advice.Chances of getting fractured are somewhat higher in girls. So please get X-ray done according to orthopedician\u2019s advice if you are girl. Secondly minor pain beneath bruises after week is suggestive of muscle soreness and deep inflammation. Take tablet Diclofenac + Chlorzoxazone 12 hourly for 2 days.If the problem persists, consult Orthopaedician for further examination.I hope I have answered your query. Let me know if you have any further questions. Regards, Dr. Hiren Hirpara"
},
{
"id": 202453,
"tgt": "Is clomfranil safe for night fall treatment?",
"src": "Patient: Hello My Name is Yasir My age is 28 I am suffering from night fall from last 8 years i used all kinds of medication but it was not cured now i am using clomfranil tablet at night before sleep and my night fall has stopped please guide me should i continue to use it Doctor: Night fall is a common issue among young adults, especially if you are not used to self stimulation and masturbation. By itself it is harmless, and I would personally suggest that you need not use any medicine for it. But if you really are insistent on using a drug, Clofranil is good but has its side-effects. You should not abruptly discontinue it. Day time sedation can be a problem in some patients. Before you withdraw from the medicine, please consult the doctor who has prescribed you this and see if he can taper the drug slowly. Night fall is likely to come down once you have regular intercourse."
},
{
"id": 182499,
"tgt": "What causes a hard lump near the ear?",
"src": "Patient: I have a hard lump in front of my ear on the right side, it feels like bone and as if it s attached to my jaw bone. I also have toothache and have had some welling in a lymph node in my neck on the same side. I had my lower right wisdom tooth removed 2 months ago. Doctor: Hello, thank you for consulting with healthcaremagic. It can be possible that there might be some infection still left in the jaw bone after wisdom tooth extraction, which is causing all these problems. To confirm the diagnosis you have to get a full mouth x - ray done which will show the complete infection. Hope it will help you."
},
{
"id": 27678,
"tgt": "What causes chest pressure with sweating and weakness?",
"src": "Patient: I am a heart patient had a stint year ago, lately I have had chest pressure , all over weakness, and one day sickening jaw ache. I have the pressure or just discomfort in my chest everyday and this week excessive sweating at work. Please help me my blood work is good and negative EKG once. Advice would be appreciated. This has been going on for at least a month. Doctor: The symptoms you describe are considered angina equivalent more importantly idle they occur with exertion . I will strong suggest you to get a stress test done (treadmill test) , and depending on the test results proceed further ad you cardiologist will suggest. Time being you can start on antianginal medication like nitroglycerin extend release or Nikoran dil"
},
{
"id": 143455,
"tgt": "What could cause frequent headaches and partial numbness of face?",
"src": "Patient: hi i have been getting headaches for a few days now, yesterday it felt as if someone has hit me on my eyebrow bone and my face on the right side felt numb. today the whole right side of my face has dropped a bit and is scew, i still have the pain above my brow and a numb sensation on the right side of my face it feels as if i had and injection from the dentist, THAT kind of numb feeling, What could that be/ please help? Doctor: I suppose you have an ear infection,that is leading to your headache and one sided face weakness.you need to go to an ENT."
},
{
"id": 171519,
"tgt": "Suggest treatment for the frequent stool discharge in a baby",
"src": "Patient: I have a two year old son, usually he passes stool 2/3 times in a day. but from last one month suddenly the number is increased to 5/6 times, the stool amount is much and soft, recently from last 4/5 days there are white something is visible in the stool, can anyone advice me on this pls... Doctor: Hi...by what you quote I feel that this could be - 1. Irritable bowel syndrome2. Worm infestation.3. Persistent diarrheaUnless the kid's having low urine output or very dull or excessively sleepy or blood in motion or green bilious vomiting...you need not worry. There is no need to use antibiotics unless there is blood in the motion. Antibiotics might worsen if unnecessarily used causing antibiotic associated diarrhoea.Regards - Dr. Sumanth"
},
{
"id": 7617,
"tgt": "Having pimples on the face. Can you suggest a good face wash?",
"src": "Patient: i am 21 years old and i play football regularly and that causes pimples . if i stop playing , my skin starts becomming clear gradually. my skin type is combination , i dont know which face wash to use. i am using garnier oil clear facewash which dries up my jaw line. i want you to recommend me a good face wash. can you please help me ? Doctor: i hope you have not been picking up on any of the lesions as that will lead to deep scars and pigmentation. first of all creams that you can apply. you need to follow a routine of cleansing, sunscreen , creams daily. cleanse your face with cetaphil cleansing lotion for acne prone skin thrice daily or neutrogena deep pore cleanser is also good. using a face wash with salicylic acid also helps in reducing acne. sunscreen needs to be used daily , neutrogena would be a good choice. creams would be:- clindamycin gel on the active acne lesions in the morning and Retin A 0.025% cream at night on the entire face. Retin A needs to be applied only at night, start with coming it for 2 hours and then gradually increase the time interval to keep it overnight. regards."
},
{
"id": 56495,
"tgt": "Is chewing on tongue the symptom of liver cirrhosis and hepatic encephalopathy?",
"src": "Patient: My husband has been diagnosed with liver cirrhosis from fatty infiltration. He has had hepatic encephalopathy with elevated ammonia levels. He now is chewing on his tongue. (He has no teeth, so no injuries, but his tongue is very sore.) He does this unconsciously, even at night and cannot seem to stop. Is this another symptom from this disease process? Doctor: Thanks for posting your question on HCM!I really appreciate your concern for your husband.In addition to other signs and symptoms, although less common but abnormal tongue chewing is seen in patients with hepatic encephalopathy.Loss of coordinated movements of the jaw and tongue occurs due to injury to the part of brain that is concerned with the movements of tongue and the jaw.cIn comatose patients, the tongue can become juxtapositioned between the teeth resulting in this chewing reflex.You are advised to take an opinion of a dentist for possible use of acrylic mouth gags to prevent the injuries in oral cavity.Wishing your husband a speedy recovery, Insha allah."
},
{
"id": 129542,
"tgt": "What causes severe joint pain in arms, neck and legs with painful rashes?",
"src": "Patient: severe pain I am a 23yo female with severe joint pain. the pain is in my arms, upper back, neck, and right leg. this is not a muscle ache it is an actual joint pain. the pain started last Saturday when I had a random rash appear on random spots on my body. the rash comes and goes but never appears in the same place, the rash is a red, raised, very painful circular like spot. since the rash has appeared the joint pain has been very severe the only time I get comfort is when I am sleeping. I am a medical assistant and the Dr I work for said he has no idea what the rash is or what caused it. I have not changed anything in my daily routine. no one else in my house has it. my Dr thinks it may be viral and said to give it 7-10 days but it is getting worse to the point that I m in tears. I need help Doctor: Hi. You can get a Reactive arthritis from various viral infections. Skin manifestations are common with viruses and you need a check to see which infection this is."
},
{
"id": 168576,
"tgt": "Suggest remedy for chest pain in a child",
"src": "Patient: Hi. My 6 year old son has a very bad chest pain. It s on his left ( assuming his heart). He s breathing ok. This was after having lunch but we were snow sledding 30-45 min prior. This is not the first time. He had an extreme pain a couple of months ago after his soccer practice. What can it help? Please advise. I just called his pediatric and waiting to hear back. We are on vacation but if we have to, we will take him to a 24 hour clinic. Doctor: Hi...chest pain in kids of this age is very unusual. If it is associated with breathlessness or loss of consciousness then you need to seek the help of nearest emergency room. But as both these symptoms are not there - you need not worry.Regards - Dr. Sumanth"
},
{
"id": 27141,
"tgt": "What causes low BP and high heart rate?",
"src": "Patient: What should be the ratio of calcium, potassium, magnesium for a heart patient taking Amiodarone, furosemide,irbesartan, warfarin. Also type 1 diabetic. Experiencing low BP in am that weakens me and increases my heart rate. Since water with a little salt in it seems to revive me, I thought my electrolites might be out of balance. Doctor: Welcome at HCM I have gone through your query and being your doctor I completely understand your health concerns.. The calcium potassium and magnesium levels need to be in normal range for the proper functioning of the human body.. Now coming towards the medicine you are taking.....furesimide and irbesartan are antihypertensive medications. They lower the blood pressure by increasing the urine output. In some instances the dose become relatively high causing low blood pressure along with electrolyte imbalance. You need to consult your doctor so that your medicines can be optimized to prevent these side effects... As far as warfarin is concerned the only related thing is PT INR ,your doctor will decide how much anticoagulantion is required for you. Normally INR is required at value of 2 to 3.. Amiodarone is concerned , it is antiarrhythmic drug and your doctor will adjust your dose.. Last but not the least is insulin you should be cautious regarding hypoglycemia.. So use frequent small meals and keep sweet candies in your pocket to prevent hypoglycemia... Your doctor will educate you about hypoglycemia and ways to prevent it. In short, you need to decrease your antihypertensive medications dose and also look carefully whether you are developing hypoglycemia or not.? Do consult your doctor immediately.. Hope your query is adequately addressed Regards Dr saad Sultan"
},
{
"id": 94406,
"tgt": "Pelvic pain. Removed gall bladder. Negative test results. Reason for pain?",
"src": "Patient: Hi, I have pain in my lower right pelvic area. It comes and goes. I had my gall bladder out since that was the only thing they could find wrong. I had stones. Hoping this would cure my ailment. The pain was gone for 3 weeks. I started back to work yesterday and the pain has returned. I have had multiple test and all my results have been negative. Do you know what the pain could be? Doctor: Hello! Thank you for the query. It would be much easier if you could mention what tests have you had performed, otherwise, it will be very hard to help from here. Such lower right pain can be caused several diseases including urinary tract infection, urinary tract stones (frequent urinating, burning while urinating should be present), ovarian cyst (if you are female), large intestine inflammation (Crohns disease, especially if you are about 20-30 years old, have had diarrheas, blood in stool, weight lost), colon cancer (especially if you are 50 or older, have had constipations, blood in stool), inguinal hernia (the pain aggravates while coughing). Hope this will help. Regards."
},
{
"id": 137998,
"tgt": "What causes pain in heels and ankles?",
"src": "Patient: For about two weeks I have been suffering with pain in my heels and my ankles. especially first thing of a morning and I d does carry on for most of my day. It s becoming annoying and limiting my ability to walk. Any ideas? I have an OC health dept at work, which I my visit tomorrow. I m constantly on my feet at work. Doctor: Hello, Thanks for writing to us, I have studied your case with diligence.As per your symptoms there is possibility of calcaneal spur along with plantar fasciitis leading to heel pain.It is advisable to wear low heel shoe to maintain centre of gravity of whole body.Shoe/footwear should be with soft sole. You can use silicon pad insoles.You may need x ray heel to see bony overgrowth.Start plantar exercises consulting physiotherapist also Physiotherapy like TENS and ultrasound will help.Take calcium supplement with vit D.If no relief with this line of treatment then local injection shot can give pain relief for longer duration.Hope this answers your query. If you have additional questions or follow up queries then please do not hesitate in writing to us. I will be happy to answer your queries. Wishing you good health.Take care."
},
{
"id": 65882,
"tgt": "What treatment to be taken for lump in neck?",
"src": "Patient: I am a 40 year old healthy male and discovered a lump on my neck in clavical area a few days ago, It seems to have gotten bigger in last few days and giving me a slight sore throat and very aware of it when I swallow. Could cancer of some kind appear so suddenly or is it some infection, needless to say I am concerned and worried. Should I be going to the emergency room or wait to see if it gets better> Doctor: Hi ! Good evening. I am Dr Shareef answering your query.If I were your doctor, I would do a general examination of your body and of each organ system, and then go for a FNAC (fine needle aspiration cytology) of the lump to have a cytological diagnosis of the same. It is not necessarily have to be cancer, but could also be due to some kind of chronic granulomatous infection. However, the management would depend on the report of the FNAC, and other related investigations.I hope this information would help you in discussing with your family physician/treating doctor in further management of your problem. Please do not hesitate to ask in case of any further doubts.Thanks for choosing health care magic to clear doubts on your health problems. I wish you an early recovery. Dr Shareef."
},
{
"id": 69053,
"tgt": "What causes breast pain and lumps along with shoulder and neck pain?",
"src": "Patient: Hello there i got a breast pain and i have a 2 lump in my breast both are movable one lump is small and that lump is causing pain to me . And i got shoulder pain neck pain and hand ..is this breast cancer symtons? If i press the brest i got little bit discharge just like milk n i am 22 yrs old Doctor: Hi ! Good evening. I am Dr Shareef answering your query.The symptoms you described are not necessarily be due to a cancer. Many benign breast diseases like fibroadenoma and fibroadenosis could also give rise to such clinical features. The shoulder and neck pain could be an independent symptom of yours not related to the breast problem. If I were your doctor, I would perform a clinical examination of your breasts along with the axillary regions, followed by a mammography and FNAC/excision biopsy of the lesions you have described. Further management would depend on the investigation reports.I hope this information would help you in discussing with your family physician/treating doctor in further management of your problem. Please do not hesitate to ask in case of any further doubts.Thanks for choosing health care magic to clear doubts on your health problems. Wishing you an early recovery. Dr Shareef."
},
{
"id": 181846,
"tgt": "What causes soreness on roof of mouth after Clindamycin dose?",
"src": "Patient: I had an allergic reaction to clindamycin almost 3 weeks ago. I had a full body rash for about 10 days. I went to my PCP twice, received a steroid shot, but I still have soreness in the roof of my mouth, some swelling in my lips (comes and goes), burning and stinging in my lips and tongue. I've gone to my dentist and my ENT doctor, and no one can figure out my mouth issues. The ENT doctor said it might be a lingering effect of the allergic reaction. Any suggestions? Doctor: Hello,Thanks for consulting HCMRead your query as you are allergic to medicine clindamycin yes these effect can be sue to allergic reaction . I will suggest you to do warm saline rinses and you can take medicine like Allegra or Avil by consulting with local doctor .If symptoms persists you should consult physician or oral surgeon for examination .Hope this will help you."
},
{
"id": 151470,
"tgt": "Loss of balance and concentration, poor vision, incontinence and lack of focus. MRI brain ruled out MS. What could be the reason?",
"src": "Patient: My brother is 47 years old and is experiencing severe loss of balance, loss of coordination, lack of focus, poor vision and now incontinence . His condition has deteriorated in the last month and now he can t be left by himself. He has had an mri on his brain and tomorrow will undergo a spinal tap. His neurologist has ruled out MS. What could it be? What should we be asking his neurologist? Doctor: Hello mrs.espo13. Welcome to healthcaremagic.com. A month is a very short time for your brother to develop these symptoms. An MRI should have shown a tumor or another cause like MS. Your brother's symptoms are all most likely related. There is a disease called hydrocephalus. Hydrocephalus is defined as \"water on the brain\" and is more common in children than adults. However hydrocephalus is also found in adults and may occur insidiously (unknown cause) or after a brain injury. Hydrocephalus is characterized, in adults, with rapid onset (usually months), and has the symptoms of poor vision, loss of balance, loss of coordination, mental disturbance (loss of focus), and incontinence. Your brother has a lot of the symptoms of hydrocephalus. A spinal tap is the next step in diagnosing and treating hydrocephalus. If your brother's symptoms improve or resolve after the spinal tap, it would be advisable for you and your brother's neurologist to discuss if your brother is a candidate for a V-P shunt. A V-P shunt is placed in the brain and into the abdominal cavity to drain the excess cerebral spinal fluid from his brain into his abdomen, where the fluid is reabsorbed. I hope that this information helps and good luck to you and your brother. Be well, Dr. Kimberly"
},
{
"id": 79551,
"tgt": "What causes elevated temperature, sensitivity in trachea/upper chest after treated with flu/bronchitis?",
"src": "Patient: After a the flu (began November 20th) and subsequent case of bronchitus (began around November 24th), I have had a persistent elevated temperature in the evenings of 99.8 to 100.4 degrees. On the morning my temperature is quite normal (98.6-99.3). My coughing is almost gone, but still tight and empty when it occurs. I still have sensitivity in my trachea and in my upper chest. I am puzzled why this elevated temperature in the evenings will not go away after weeks. I have had chest Xrays and they have been read as normal. My Sed rate is 38. WBC is in the normal range and RBC is a small amount on the low side. Doctor: thanks for your questioni completely understand your questionit seems like a post viral bronchitis .you should visit a pulmonologist who can request for a sputum pyogenic culture and sputum for afb smear to rule out tuberculosis as evening rise of temp can occur in tuberculosis and xray can be sometimes normal or infection can be in the hidden areasas per post ciral bronchitis it takes time sometimes months to godisscuss with your pulmonologist regarding spirometry if there is an obstructive airway disease , post viralthanks feel free to ask more questions may god bless you with good health"
},
{
"id": 170255,
"tgt": "What causes unusual angle of penis while urinating?",
"src": "Patient: The top section of my 3 year old sons penis always points at an unusual angle when he urinates. The angle can be very extreme at times and has caused his urine to spray his bum when he urinates. (still sits down when using the toilet). Average height and weight. Doctor: DearWelcome to HCMWe understand your concernsI went through your details. I think you have not given the whole description. You must also quote whether the child urinates with the foreskin pulled back. If this is the case, the foreskin has a connection with the urine opening and when the foreskin is pulled back, the connected area also gets pulled and an unusual angle forms. Spraying of urine always happens with the foreskin pulled back. In this given case, the scenario is quite normal and the child's habit is healthy. Otherwise, you may consult a child specialist for physical investigation.If you require more of my help in this aspect, please use this URL. http://goo.gl/aYW2pR. Make sure that you include every minute detail possible. Hope this answers your query. Further clarifications are welcome.Good luck. Take care."
},
{
"id": 192679,
"tgt": "What is the treatment for peyronie's disease?",
"src": "Patient: Recently two family members have been diagnosed with Pierones disease. Both brothers are in early 60's and both served in Vietnam. Their respective doctors at two VA hospitals said they are seeing this very often of late with regard to servicemen in Afghanistan and in their 20's. Is there an established correlation between service overseas and the occurance of this disease? In addition, one brother has been dx in early stage, can it be \"cured\" or abated? Thank you for your time. Doctor: Hello, Environmental factors or working in a particular country does not cause Peyronie disease. The surgical correction will be the treatment of choice and you need an experienced urologist for beer results. Hope I have answered your query. Let me know if I can assist you further. Take care Regards, Dr Shinas Hussain, General & Family Physician"
},
{
"id": 216564,
"tgt": "Suggest treatment for neuralgia",
"src": "Patient: My mother-in-law has been diagnosed with neuralgia. I can t quite find the right type though on the website. She hasn t had shingles, and the stabbing, shooting pain runs down her arms (both sides, but thankfully not simultaneously. It seems to take turns!) Sometimes the pain is so intense she cries and doubles over. She was put on Lyrica a few weeks ago, but it has only helped marginally. Does the drug take time to take affect? And is there something else that could be causing the problem other than neuralgia? Doctor: hi,thank you for providing the brief history.As the pain is radiating in nature I suspect it to be more of the cervical spine nerve root entrapment. Due to the ageing the spine becomes weak and lead to pressure on the intervertebral disc which are situated in between two vertebral bodies. Due to this the disc will lead pressure on the nerve roots. this will lead to pressure on the nerve and pain which will be sharp, shooting in nature.I will advice to undergo an cervical spine scan and them undergo physical therapy which will help ease alot of problems which she is facing.Regards Jay Indravadan Patel"
},
{
"id": 23045,
"tgt": "Is high blood pressure serious?",
"src": "Patient: hi iam an mbbs student..2nd semester..for the past 2 weeks i had continous exams without break..for the last two three days im feeling really exhausted nd tired..nd as my exams didnt go so well im feeling really low nd miserable...i checked my blood pressure today and found it to be 159/108...i know its high..i wanted to kno weather its a serious case nd shud i start medication...or i shud wait till next week and see weather it gets to normal Doctor: Hi dear,This bp for your age is pretty high. But a single reading of raised bp does not define hypertension. You need to have 3 different reading of high bp at 3 different time that too is calm state.Anxiety and stress as you must be knowing increases the sympathetic flow & raises our bp. But that is temporary & comes back to normal as the stressor is gone.Exams are important but your health too is important. Make a schedule for everything & you will see the stress will be gone. Till than decrease salt in diet & quit smoking if you are doing it.ThanksDr Sameer Maheshwari"
},
{
"id": 127552,
"tgt": "How can trigger finger be managed?",
"src": "Patient: Hi, I am a 72 year old woman and have noticed that my right ring finger is becoming trigger like while I am sleeping and in the morning it snaps back. I have a little arthritis is sever joints in my fingers on both hands. I am a vet. Should I seek help with my VA doc to treat this condition. Will it get worse? Is there anything I can take to stop or slow the progression? Doctor: Hello and Welcome to \u2018Ask A Doctor\u2019 service. I have reviewed your query and here is my advice. Trigger finger if mild can be managed with the help of a splint. In case it is persistent then surgery will help. Hope I have answered your query. Let me know if I can assist you further. Regards, Dr. Praveen Tayal"
},
{
"id": 136694,
"tgt": "Suggest treatment for severe pain from elbow to the palms",
"src": "Patient: My left arm from the elbow down to the palm of my hand has sharp severe pains. It started a couple of hours ago and wasn t down to the palm of my hand but now it is. When I try to open my hand all the way it feels like its pulling and the pain is incredible. I have tried a heating pad but no help Doctor: Hello, I have studied your case.Due to compression of nerve root there can be pain and burning sensation along elbow and palm.I will advise you to MRI cervical spine for better diagnosis.For these symptoms analgesic and neurotropic medication can be started.Till time, avoid lifting weights, Sit with support to back. You can consult physiotherapist for help.Physiotherapy like ultrasound and interferential therapy will give quick relief.Hope this answers your query. If you have additional questions or follow up queries then please do not hesitate in writing to us. I will be happy to answer your queries. If you are satisfied with answer do not hesitate to rate this answer at end of discussion. Wishing you good health.Take care."
},
{
"id": 225060,
"tgt": "Will gynaset tablet help in preventing pregnancy?",
"src": "Patient: Hello doctor.. I m taking gynaset tablet on my doctor s prescription because of my irregular periods.. yesterday I had unprotected sex with my boyfrnd.....my question is whether I need to take contraceptive pill for stoping pregnancy or taking gynaset tablet is enough for that....plz help me out... Doctor: gynaset will not prevent pregnancy. You need to talk with your doctor about another contraceptive in order to prevent pregnancy. You should ask your doctor if he can start you on a birth control pill that will both control your periods, AND provide protection from pregnancy. I hope this helps answer your question, please let me know if you have any more questions."
},
{
"id": 173783,
"tgt": "What could cause crying while passing stool in a 54 days old child?",
"src": "Patient: himy daughter is 54 days old. she has some problem , she cries a lot whenever she passes stool . i took advise from my doc he has advised COLIMEX DROPS(antispasmodic and antiflatulent) . i want to know whether it is safe or not?i am giving her 3 drops /3times/day Doctor: Hi,Thanks and welcome to HCM .Probably your baby is suffering from infantile colick.Colimex drops contain Simethicone and dicyclomine for relief of intestinal colick.Normally they are not prescribed bor babies below 6 months old.For younger babies Colimex DF drops are prescribed .Hope this reply serves your purpose.Please feel free to ask further queries in any.Dr.M?V.subrahmanyam."
},
{
"id": 51732,
"tgt": "Can masturbation effect on my health ?",
"src": "Patient: hi I am a renal transplanted patient, I do masturbate 2 times or once in a day , can this effect on my health ? Doctor: hello thank you for your question Masturbation does not affect your health even if you're renal transplant patient. Please feel free to ask if you need further assistance thank you"
},
{
"id": 72383,
"tgt": "What causes left chest pain, arm numbness, right abdomen distention and bruising?",
"src": "Patient: I am burping what tastes like eggs, although I haven't eatenI am burping what tastes like eggs, although I haven't eaten any eggs at all in the past week or so. I have chest pain on the left side as well as a numb left arm, and the upper right quadrant of my abdomen is distended and feels bruised. Doctor: Thanks for your question on Healthcare Magic.I can understand your concern. In my opinion, we should first rule out cardiac diseases for your left sided chest pain. So get done ecg and 2d echo. If both these are normal then no need to worry for heart diseases. GERD (gastroesophageal reflux disease) can also cause burping, left sided chest pain, bloating and abdominal pain.So take pantoprazole and levosulperide combination on empty stomach twice daily. Avoid hot and spicy food. Avoid junk food. Go for walking after meals. Loose weight if you are obese. Quit smoking and alcohol if you have these habits. Don't worry, you will be alright with all these. Hope I have solved your query. I will be happy to help you further. Wish you good health. Thanks."
},
{
"id": 39057,
"tgt": "Suggest remedy for cold and cough",
"src": "Patient: Hi, I haven't been able to lie down for 4 days now with a cough that becomes terribly hacking when I lie down (only 12 hours sleep in the last 4 days). Even stretching my neck or yawning can bring on an attack. I also have a green/black tongue, all over the tongue that lightens when I brush the tongue but doesn't go away. The cough seems to be caused by a circular process of nasal drip down onto inflamed , sore and sensitive lower throat tubes (the bronchial tubes?), which causes a cough, which makes the throat more sore and sensitive for the drip to trigger again. If I do manage to sleep and am woken with a cough I bring up hard to shift green phlemghI also have a blocked nose that doesn't produce anything when I blow my nose. Doctor: HI, thanks for using healthcare magicThe fact that the cough worsens on lying down may imply that there is pooling of secretions.You may need to consider a decongestant, antihistamine and mucolytic.The decongestant would help relieve your nasal symptoms and by reducing the dripping of mucus, help reduce the coughing.Mucolytic would help to break up any mucus so that it can be easily expelled and the anti histamine would also help with the mucus and congestion.These should be available over the counter and may be combination.Examples of oral decongestants are pseudoephedrine and phenylephrine.Examples of mucolytics are guifenesin, bisolvan, mucomystI hope this helps"
},
{
"id": 85193,
"tgt": "What is the dosage of Benforce M?",
"src": "Patient: Hi My doctor has given me Benforce M as i PCOD.Here in this forum I read one query where the lady has mentioned the same medicene she is taking post lunch n dinner.But my doctor has asked me to take before food.Kindly suggest which one is correct???? Doctor: Hi,Benforce M (Active ingredient metformin) is commonly prescribed for the treatment of PCOD. Primarily used in the treatment of type 2 diabetes mellitus, it is found to be useful in treatment of infertility associated with PCOD by decreasing the elevated insulin levels. It should be best taken with meals to help reduce stomach or bowel side effects that may occur during the first few weeks of treatment. To avoid stomach upset, start with a very low dose (500 mg), taken with dinner. After a few weeks, the dose may be increased to 500 mg with breakfast and 500 mg with dinner. A few weeks later, the dose may be increased again to 500 mg with breakfast and 1 g with dinner.Hope I have answered your query. Let me know if I can assist you further. Regards, Dr. Mohammed Taher Ali, General & Family Physician"
},
{
"id": 201196,
"tgt": "What causes itchy yellow blisters on penis and swollen lymph nodes?",
"src": "Patient: Hi, I'm having an issue in my genital region, I had a clear blister on my penile shaft which popped and turned into 6-8 little yellow bumps. They itch but are not painful. I also have severely swollen lymph nodes, all signs point towards herpes, is there anything else it can be? Doctor: Thanks for contacting HCMI am sorry to hear that you have been having blisters on your penis with associated lymph nodes. I agree with you that the most likely cause is herpes infection. But since the blisters ruptured and now you have additional bumps these lumps could be secondary bacterial infection. In this case you would need treatment for both the herpes and for secondary bacterial infection.Another cause could be syphilis as it can cause a non painful open sore on the penis. There are several other STD that can cause open sores and lymph node enlargement.I recommend that you seek medical attention to help diagnose and give you the proper treatment.In summary1. open sores penis2. Probable STD3. Possible causes Herpes, bacterial secondary infection, syphilis, Lymphogranuloma venereum4. Seek medical attention for proper diagnosis and treatmentI hope I answered your question. Please contact HCM again with your medical questions and concerns"
},
{
"id": 163911,
"tgt": "What could constant sticking of tongue in children suggest?",
"src": "Patient: when i picked my daughter(2years old) up from day care yesterday i noticed she kept sticking her tongue in out of her mouth...she did this the rest of the night and did not fall asleep until 7 this morning after she woke up this morning she has continues to stick her tongue in and out of her mouth...its seems to be involuntary Doctor: Hi.... I understand your concern. As she has started doing it suddenly, I feel that she has picked it up on seeing some other kid doing it at the daycare. Usually this is the most common cause of kids picking up habit like this.She will forget and be out of it. Don't worry.Regards - Dr. Sumanth"
},
{
"id": 39961,
"tgt": "How long will jaundice take to be cured?",
"src": "Patient: Hi... Igot job in saudi and as per they procedures they advised me to do medical test. in that it was proved i had jaundice but i never faced jaundice. It was found in my blood. Please help me in how many days it will be cured. I have to leave to saudi and it is veryi mportant to my family Doctor: Dear Friend.Welcome to HCM.I am Dr Anshul Varshney. I have read your query in detail. I understand your concern.This would depend on the cause of Jaundice.If you consume alcohol, it can be because of that.If you don't then you should get Viral Markers and Ultrasound abdomen.Please share your reports with us, so that we can give you a diagnosis.This is my best opinion for you based on available details. If you any have further query, please ask me.Stay HealthyDr Anshul Varshney, MD"
},
{
"id": 12570,
"tgt": "Diet for psoriasis",
"src": "Patient: my mother is suffering from psoriasis from last 6 years. please tell me what diet should she have? what should she eat and what should she avoid? kindly tell me how to maintain her digestion? and how to stop this disease because it is very painful. Doctor: hello im dr satya, your question was interesting, it may help a lot of other people suffering from the same disorder. regarding diet and psoriasis there seems to be no direct evidence of any food that aggravates psoriasis, but any associated condition like for example urticaria, nettle fish, rashes, drug allergy, or drug reaction, and some medications do have a negative impact on psoriasis. fresh fruits, vegetables, balanced food does help in maintaining a good health. last bu not the least, psoriasis is not an allergic disorder but a genetic disorder, which gets its trigers from throat infections, and recurrent cold and cough . thank you and i hope that it may help you, any way i will try and keep you updated if you are interested. feel free to contact. dr satya"
},
{
"id": 129672,
"tgt": "Will taking Ibuprofen help treat stiffness of neck?",
"src": "Patient: my son has a severe stiff neck so much so that he cannot turn his face. I have given him strong ibuprofen and a a heatrub and hopefully he can move a bit without any pain but should I go and see a doctor about it and I am wondering what exactly is it? Doctor: Thank you for your question.The basic reason for neck stiffness is the spasm of muscles which surrounds the neck to protect the spine after a trauma or a Spinal injury.Ibuprofen is an antiinflammatory agent and hence will not helpFlexon MR 500mg thrice a day will relieve your son's pain as it contains paracetamol, a muscle relaxant and an antiinflammatory.To answer your second question Yes you should definitely go see a Dr as the reason for the spasm has to be investigated. As you have not provided me with enough history as to how and when the stiffness happened I am afraid I will not be able to help you although I will say that the commonly it is due to trauma or cervical spondylosisI hope your son feels better"
},
{
"id": 130370,
"tgt": "What causes sharp, throbbing pain just below the navel on the right side?",
"src": "Patient: Hi, i awoke 2 nights ago with a sharp , throbbing pain just below my stomach button, on the right hand side. The pain was nt to severe however, so i thought nothing of it. Since this morning, i am now feeling a vibration in my scrotum, it occurs every few mins or secs and usually only lasts about 2 secs. Doctor: Is this the first time you felt this pain? Any urinary complaints like irritation while voiding or difficulty in voiding? What is you age group? Any association with diabetes?"
},
{
"id": 207225,
"tgt": "What does smiling alone with own self imply?",
"src": "Patient: Hi Doc A colleague of mine starts smiling all of a sudden while looking on the computer screen , she lose her concentration while doing some work and then start smiling all of a sudden within herself. This phenomenon happens all time when she is working alone and nobody attends her. can you tell me what is this? Doctor: DearWe understand your concernsI went through your details. I suggest you not to worry much. Smiling to herself while working and while alone, talking to oneself while alone etc are part of behavior while engrossed in something engrossing. There is nothing to be alarmed of. You should talk to her and appraise her of the situation and advise her to control such behavior. Awareness and some corrective measures can clear these behavior. But there is nothing wrong with such behavior.If you require more of my help in this aspect, Please post a direct question to me in this URL. http://goo.gl/aYW2pR. Make sure that you include every minute details possible. I shall prescribe the needed psychotherapy techniques.Hope this answers your query. Available for further clarifications.Good luck."
},
{
"id": 115249,
"tgt": "Is low hemoglobin in blood due to typhoid?",
"src": "Patient: i m suffering from typhoid and my hemoglobin 4.0. is low hemoglobin due to typhoid?? Also want to know that i suffered simmilar high fever with loose motions on 14-15th of August. Temperature came down after 2days and i didnt took it seriously. now i had high fever since 3-4 days and came to kno today that its typhoid. Want to know at what cycle stage i am in..?? heard that typhoid bacteria multiplies and take around 3weeks.. Want to estimate how long will i take to recover.. Doctor: Hi there,Thanks for using HCM.No, typhoid does not cause a low hemoglobin.Your hemoglobin 4.0 is too low. You are anemic.A peripheral smear test should be done to see what type anemia. Further testing depends on what type of anemia you. The smear may indicate microcytic, macrocytic or dimorphic anemia. Microcytic anemia usually indicates iron deficiency and a macrocytic anemia may indicate Vitamin B12 deficiency.Since the hemoglobin is too low, you require blood transfusion.As far as your fever is concerned, it may take 2-4 weeks to completely recover.Is this answer helpful?"
},
{
"id": 111834,
"tgt": "What causes persistent pain in lower back and testicle?",
"src": "Patient: hello, i'm writing you on behalf of my husband who is 34 years old. He has been having a dull achy pain in his lower right back (feels like an internal organ) for years along with his right testicle. He's seen our family doctor quite a few times. They've tested his urine, regular blood work, and tested for some stds. He's also seen a chiropractor thinking is was a pinched nerve. Our family doctor has started blowing him off like it's all in my husbands head. My husband is getting so concerned about this that it's causing him to have anxiety and panic attacks. Do you have any suggestions or recommendations? Doctor: I have studied your case. It could be a hernia which is not diagnosed. Or it may be a spine problem. Other possibilities are kidney issue. So my suggestion is to do a ultrasound of abdomen and a detail check up of spine and kidney. A fnac of testis can be done. If everything turns out to be normal than your husband should be relieved of the stress."
},
{
"id": 4499,
"tgt": "When will embryo implantation occur after transfer and will there be any physical indication?",
"src": "Patient: for me, two embryos are transferred ,one is with 5cells and the other with 7 cells on day three and my doctor suggests progestrone injection for 15 days and ask me to come for blood test on day 14 - when will embryo implantation occur ,will there be any signs indicated tome physically after 5 or6days Doctor: implantation should occur within 2-3 days of the transfer. usually there are no signs or symptoms until 2-3 weeks after implantation. unfortunately you won't know anything by 5 days. you will need to wait until your doctor does an ultrasound to confirm if the transfer was successful. I hope it works out for you!"
},
{
"id": 204643,
"tgt": "How can bipolar disorder be treated?",
"src": "Patient: I am seeking to find the best bipolar medication for my husband. He is currently taking depakote but is having some of the severe side effects such as low RBC, low hematocrit, and bad loss of hearing. The blood tests are recent and show these changes since he s been taking depakote since december 2017. He is 82 years old and will have a defibrillator installed on Friday because of frequent PVC s. I am concerned to find a better medication going forward. Doctor: Hello,The treatment of bipolar disorder is with mood stabilizer and antipsychotics. Please consult your psychiatrist regarding this. Hope I have answered your query. Let me know if I can assist you further.Regards, Dr. Soujanya"
},
{
"id": 30584,
"tgt": "Suggest treatment for chlamydia infection",
"src": "Patient: I was diagnosed with clymidia 2 years ago, an the gyno toldme I probrobly had it for a while considering the severe cramps I was getting Me and my bf have been trying to conceive ever since I was treated, and I can`t get pregnant. Can I maybe have a surgery done to fix the damage to my tubes due to having clymidia. Please help Doctor: HiI had gone through your question and i understand your concern, See , Treating persons infected with Chlamydia prevents adverse reproductive health complications and continued sexual transmission, and treating their sex partners can prevent reinfection and infection of other partners.The standard recommended regimen for treatment of Chlamydia is: Azithromycin 1 g orally in a single dose OR Doxycycline 100 mg orally twice a day for 7 daysI hope this information was useful.Wish you good health.Thankyou."
},
{
"id": 20794,
"tgt": "What causes difficulty in breathing?",
"src": "Patient: I am having difficulty breathing. I feel shortness of breath, tightness of my rib cage, and I have chest pain, back pain, but this is sometimes. I have seen a heart doctor, gotten tests done. He said that I have PVCs and some galloping but I don't have asthma and he said I may have some anxiety. I feel like I am not getting enough air to my lungs and it really hurts under my shoulder blades on my back. I don't know what to do and have been experiencing this ever since I was about 12, I am 18 now. What could it be or what should I do? Doctor: Get an appointment with a psychiatrist in your area and get yourself evaluated for anxiety disorders too"
},
{
"id": 47982,
"tgt": "Does chronic Kidney disease cause diarrhea?",
"src": "Patient: Yes, recently I have had pains on my side and back. I am 54 years old, have high blood presure under control and diagnosed with chronic Kidney disease. Recently from time to time I have had almost all liquid diahreah, especially when stressed. What should I do?I will see my Doctor on this...thanks anyway. Doctor: Hello and welcome to HCM.Your concern is understandable. As an Urologist, let me advise you, that with CKD(Chronic Kidney Disease), you're prone to electrolyte (salt and potassium), changes with diarrhoea. The kidney function tests, like blood creatinine, and urea with electrolytes need to be checked, and corrected according to changes seen. An Ultrasound scan of the abdomen may be advised, if needed.You may need admission after an examination.You should be careful to avoid potassium rich diets,like fresh fruits,coconut water, and adding table salt.Take fluid intake slightly more than urine output.You may contact me by name, with reports ,for expert opinion."
},
{
"id": 20109,
"tgt": "What could constant heart palpitations suggest?",
"src": "Patient: Hi, I am 30 year old female and i have been having what I think are palpitations everyday for about a month now. It just feels like a hard thud in my chest or like a skipped beat and thats it. I do not have any other symptons. I was on the birth control camila aka (mini pill) I stopped taking that 2 weeeks ago because I thought maybe thats what was causing these but they have not to seemed to have gotten any better. I usually get between maybe 10-25 a day. I also dont drink caffiene and I'm also breastfeeding. I can't see my doctor for another 2 weeks. Should I be worried? I'm scared to death! Doctor: Welcome and thank you for asking on HCM!I have gone through you question carefully. Palpitations in young females is usually due to low hemoglobin ( anemia) and thyroid problems (excess). If these two things are ruled out .. you can be assured there is nothing to worry.Hope you will find this answer helpful!Wishing all the best, Dr. D Sunil Reddy"
},
{
"id": 56671,
"tgt": "How can gallbladder stones be treated?",
"src": "Patient: Hi I am 37 year old male having a stone in gallbladder 7 month back ultrasound shows 9 mm size now its 13 to 14 mm some times having discomfurt when my stomach is empty & its feels like having some thing that side it is formed by colestrol it is not giving hard pain or any big problem any suggestion & diet recommendation thnx Doctor: Hello Gall bladder calculus need surgical removal.It may be asymptomatic or causes mild symptoms in some patients.Since you have diagnosed gall bladder calculus,you should get rid of it.Gall bladder calculus may cause obstruction and infection which may cause emergency,so it is advised to get rid of it.It may also be cancerous if it is retained for prolong period.Now a days,gall bladder calculus is removed laproscopically and recovery is very fast by this method.You should also avoid saturated fat.Take CareDr.Indu Bhushan"
},
{
"id": 5287,
"tgt": "Had unprotected sex. Was taking Novelon tablet. Pregnancy test seems negative. History of failed IUI. Pregnant?",
"src": "Patient: took Novelon tablet for 9 days starting from 2 day of my period and then stopped , we had unprotected sex on regular basis . now i have missed my period by 8 days and home preganancy test is negative , can i be preagant as we r trying since 5 yrs . as per my medical history in 2006 i sufferd from abdominal kochs for wihich i took 11 months AKT ... after 1 yr we started trying for baby, till date i had 5 IUI cycles all unsucessful. i hv my periods regularly never missed my periods , but do suffer from PCOSPlease help Doctor: Hi there ~I understand your concerns, however I believe that if your pregnancy test is negative you need to repeat it after one week and if still negative you need to see a gynecologist. A pelvic examination as well as a full test for any abdominal pathology like a pelvic examination would be necessary. I believe that your PCOS may be acting up. Anyway, you need to be seen by a gynecologist to determine what needs to be done. Get an appointment right away.I hope this helps. Take care and have a lovely day!"
},
{
"id": 93144,
"tgt": "Have pyelonephritis and chronic IC. Abdominal pain. Taking antibiotics. How long can I expect the pain?",
"src": "Patient: I have pylonephritis and was just discharged from hospital. I have chronic I C and this is what brought the pylonephritis on. I am still having terrible abdominal pain just like IC Should I still be having this pain and how long can I expect to have it. I am on antibiotics for 2 weeks. I noticed in this article that it said it is in the interstitial area of the pelvis so I assume it may continue until the pylonephritis is over. Please explain. Doctor: Hi and thanks for the query,Normally, the treatment of pyelonephritis, depending on the drug after two weeks as you describe should be sufficient. However, with some resistant and atypical strains, this might take some longer.An important aspect in the management of pyelonephritis, especially in a patient with other risk factors, or in case its a recurrent phenomenon, is an active search to identify the causative organism. Urine cultures and antibiotic sensitivity tests to prescribe the appropriate anti biotic to tackle the particular germ in question could be very useful. This not only aids in diagnosis, but in measuring the efficacy of the treatment when completed. Ultrasound images of the kidney could also help, because in some patients, renal abscesses, especially when symptoms persist should be actively sought for.Suggest you consult a nephrologist. thanks and best regards,Bain LE, MD."
},
{
"id": 76135,
"tgt": "What causes cough and wheezing?",
"src": "Patient: i am 22 and have had a cough and flue like symptoms for almost 2 months now. i went to the dr three weeks ago and had a blood test and chest x ray taken which both came back normal. yester day when i had a follow up my dr said that the cough is most likly apshma related which i find questionable. i have aphsma but have not had serious problems for years and have not noticed any wheasing. the dr told me that they did hear wheasing and i believ them but apshma does not explain the head ache and muscle pain and fatigue. i'm willing to trust the dr and take the new alirgy med she perscribed in the hopes that this will all go away but was simply wondering about other possibilities Doctor: Thanks for your question on Healthcare Magic. I can understand your concern. In my opinion, you are mostly having post infectious bronchitis. Bronchitis is inflammation of airways and this is common after viral upper respiratory tract infection (URTI) like flu. Bronchitis can cause cough, wheezing, headache, fatigue, muscle pain etc. So consult pulmonologist and get done 1. Clinical examination of respiratory system 2. PFT (pulmonary function test). PFT will not only diagnose bronchitis but it will also tell you about severity of the disease and treatment is based on severity only. You will mostly improve with inhaled bronchodilator (formoterol or salmeterol) and inhaled corticosteroids (ICS) (budesonide or fluticasone).Don't worry, you will be alright with all these. Hope I have solved your query. I will be happy to help you further. Wish you good health. Thanks."
},
{
"id": 10061,
"tgt": "Why am I losing my hair?",
"src": "Patient: hi i m 27 yrs old i started loosing my hairs from last 2 years and they are going from my frontal side of head............................and nw they are almost gone from that part.......................is it coz of stress or dietary problem coz i suffered from both the things from 2 yrs.........................plz suggeat some remedy....... Doctor: Hi, It may be telogen effluvium. Consult the dermatologist for the perfect diagnosis and proper treatment. There may be the cause as you mentioned like stress and nutritional deficiencies. There may be some other cause also. So consult the dermatologist for the perfect diagnosis and proper treatment. A series of investigations may be done to find out internal abnormalities. You may take tab. biotin. Take dietary supplements like vitamin B12, vitamin E, calcium, zinc, iron, protein. Apply mild steroid lotion. Apply almond drops on the scalp. Relax and avoid stress. Hope I have answered your query. Let me know if I can assist you further. Regards, Dr. Ilyas Patel, Dermatologist"
},
{
"id": 111285,
"tgt": "Does Duolt 20mg for back pain cause side effects?",
"src": "Patient: Hi,iam suffering from seviere upper back pain,there is no problem in mri scan,doctor sujested to take dulot 20 mg for 3 months,but this capsule is using fot anti depression?why he prescribed for me?can start to take this capule?is there any side effects? Doctor: Hello, back pain mild to moderate may occur due to postural causes. When severe there may be underlying problem. But as ur MRI is negative, it may occur due to somatoform disorder i.e without actual pathology but due to chemical imbalances in brain. So ur doctor might have suggested u the anti depressant medication. The side effects are present but are mild. So it's better if u use them and have regular consultations with ur doctor to review ur symptoms n side effects. Don't worry n take care. All the best."
},
{
"id": 158111,
"tgt": "Mom had utern cancer 3rd stage renal failure copd chf she had blood clots in lungs. Hand swollen. Finger black. Treatment?",
"src": "Patient: my mom has utern cancer 3rd stage renal failure copd chf she had blood clots in both lungs she has been on lovenox for a few months now she woke up the other day and her left hand was swollen and across her finger were bblack and bthe top of her hand was sore to touch we put ice and heat on it It seems to be cahngeing everyday now the swelling has moved to the side of her wrist and some of fingers look like their going to pop what should we do Doctor: Hi, your mother has uterus cancer, 3rd stage, renal failure, copd, clots in the lungs, so any thing is possible, but as the swelling of the fingers is changing from place to place, it could be the allergy to the tablets she is using or to the changes in the tissues. As she must be under the treatment i advise you to consult your doctor. Thank you."
},
{
"id": 190673,
"tgt": "Can Oral sub-mucous fibrosis be recovered by surgery ?",
"src": "Patient: I have suffering from Oral sub-mucous fibrosis(LOCK JAW). can surgery is option to recover it ? Doctor: hi.oral submucous fibrosis can be treated by surgical excision of fibrotic bands, but the habit should be discontinued first.after excision of fibrotic bands reconstruction should be done by buccal fat pad or by nasolabial flaps.go to oral surgeon for ur problem.wish u good health"
},
{
"id": 71485,
"tgt": "Suggest treatment for discomfort in the back and chest when I breathe",
"src": "Patient: I am having some discomfort in my upper/middle back, and chest when I breathe in. A little bit of tightness in the throat as well. I was moving some furniture over the weekend and thought maybe I pulled something but now I don t know. I am 48, female, smoker, diabetic, high blood pressure as well. All those things have been doing well and under control. This pain happened this weekend and continues. Thoughts? Doctor: Hello,By your history and description, the possibility of musculoskeletal pain is more likely. -So apply warm water pad on affected areas of chest and back.-Avoid heavyweight lifting and strenuous exercise. -Avoid sudden jerky movements of chest and back. -Take simple painkiller and muscle relaxant drugs like ibuprofen and thiocolchicoside. -Don't worry you will be alright with all these. Hope I have answered your query. Let me know if I can assist you further.Regards, Dr. Kaushal Bhavsar"
},
{
"id": 160064,
"tgt": "My sons belly button has disappeared or closed, what caused that ?",
"src": "Patient: My son has colon and liver cancer and his belly button has disappeared or closed, what caused that? Doctor: Hello Welcome to healthcare magic. The belly button or umbilicus flattens or disappears when the abdomen is overstretched or distended due to any mass or fluid collection inside.In cancer it could be the tumour itself or the complications causing fluid collection in abdomen.It's reversible but depends on the patients condition . Hope I have answered your query. Thanks"
},
{
"id": 192780,
"tgt": "What is the treatment for numbness in the testicle?",
"src": "Patient: Hi, may I answer your health queries right now ? Please type your query here... I went to see dr about numbness in testicle. The phy. asst. checked my prostate then the urologist came in and checked me again, when he checked me it was very painful he put his finger right and left rubbing inside. Later my anus and rectum became very planful. Now when I get just a little erection I feel pain and sqizzing in my butt with pain, after coming I have pain for days. I've been to dr after dr and they just keep saying its prostate infection and been on at least 3 different antibot. I need answers, I can't get reg. erection and coming. Doctor: Hello, Numbness happens When a person loses some or all of the feeling in their penis, it has become numb. The penis contains many nerve endings, and it is usually very sensitive. Numbness can occur when a person sits in a certain position for a long time. It can also result from low testosterone levels, another health issue, or an injury. Peyronie's disease, nerve problems, prostrate problems, etc can also cause these numbness issues. But proper evaluation is necessary for successful treatment. Therefore I suggest consulting a urologist for physical examination, diagnosis and treatment. Hope I have answered your query. Let me know if I can assist you further. Take care Regards, Dr. K. V. Anand, Psychologist"
},
{
"id": 71422,
"tgt": "Suggest treatment for cough, chest irritation and the inability to walk",
"src": "Patient: I had ablation April 17 and was very well until Sept when I had a viral infection which lasted 6 weeks and triggered fibrillation for which I was in hospital overnight and discharged on Bisoprol. This irritated my chest resulting in a cough inability to walk any distance. Is there an alternative to this medicine, I am due to go on holiday, if my symptoms persist, I will have to cancel my plans. Would be grateful for any advice you can give. Doctor: Hello, As the symptoms severity is much, evaluation with x-ray chest and basic lab tests to define the pulmonary insult is a must. I do not recommend traveling in this condition. Current relief with gargles, deep breathing exercise, cough preparations till further guidelines with details of reports.Hope I have answered your query. Let me know if I can assist you further.Regards,Dr. Bhagyesh V. Patel"
},
{
"id": 25940,
"tgt": "Is angiography required to be done immediately after angioplasty?",
"src": "Patient: my mother in law had a Angioplasty surgery done yesterday night.. now tommorow morning again we have been told a angiography will be done. A angiography was done before the operation yesterday. Then is it neccesary to do it again immediately after the angioplasty surgery Doctor: Hello I have gone through your query.Thanks for using HCM.Routinely angiography is not needed after angioplasty .But under special circumstances when primary angioplasty is done in critical situation then only one needs either check angio or to tackle remaining blocks.My best wishes Dr.Rajesh Teli,MD."
},
{
"id": 34417,
"tgt": "How to cure pus and redness around the cut on leg?",
"src": "Patient: I have had a small cut on my leg for a few weeks and now have poison ivy. The cut seems to have gotten very puffy and dark red with dark purple/blackish in the center. It is also showing signs of white pus. It is not painful and there is no smell associated with it but it seems to be getting worse. I have cleaned it with hydrogen proxide and anitbotic ointment for the past two days. Some one suggested it may be the bite of a brown recluse spider, but I have had the small hole type cut for several weeks already. Any insight would be great. thanks Doctor: Hello dear,Thank you for your contact to health care magic.I read and understand your concern. I am Dr Arun Tank answering your concern.It seems that the wound is infected.For such a infected wounds it is necessary to do culture and sensitivity. Once you have culture and sensitivity report you can get good treatment of antibiotics. This will give you good response. Please take the drug under your doctors guidance.Continue the application of antibiotics ointment and hydrogen peroxide. Please maintian good cleanliness and hygiene with frequent cleaning and dressing.Avoid injury to the affected part.I will be happy to answer your further concern on bit.ly/DrArun.Thank you,Dr Arun TankInfectious diseases specialist,HCM."
},
{
"id": 112107,
"tgt": "Dull pain in back, sudden stuffy nose, frequent urge to pass stools. Could it be anxiety?",
"src": "Patient: Hi, i went to the doctors last week due to having a dull pain in my back below my right shoulder blade, this happened more frequently when turning to the right and breathing in. I went to my local doctor and was told that it was probably pleuracy, and was told not to worry. However this pain has been on and off over a 3 week period, and at this moment in time i get the often stingy feeling in my back. I also keep feeling the need to go for a poo and have a stuffy nose all of a sudden. I don't know if most of this is anxiety or a serious problem, and would like your feedback on the matter until k make another appointment with my gp. Dean palfreman, 21 years old, 5ft9, 11.4 stone Doctor: Hello,Thanks for choosing health care magic for posting your query.I have gone through your question in detail and I can understand what you are going through.Yes it is possible that it could be pleurisy as the symptoms mentioned by you fit well into the diagnosis. But there is also a possibility that it could be a vertebral pain as the vertebral pain also causes similar pains. If the symptoms persist then you should get a MRI of the spine done. Hope I am able to answer your concerns.If you have any further query, I would be glad to help you.In future if you need any further details about your query, you can use the below mentioned link:bit.ly/dr-srikanth-reddyWish you good health,Kind regardsDr. Srikanth Reddy M.D."
},
{
"id": 222278,
"tgt": "What causes stoppage in menstruation?",
"src": "Patient: Mr Girlfriend had period on 4th December we had sex on 14 December then taken i-pill in 2 hrs... She didnt hd periods till 4th January and took MT-PILL on 7th January without testing... still no periods.. done pregnanccy test on 11th Januar which was negative...? Doctor: Hi dear, I have gone through your question and understand your concerns.I pill has the side effect of delaying the next expected periods by2-3 weeks.If the urine pregnancy test is negative, then the pregnancy is ruled out.I will suggest her to not to worry and wait for the periods to come.Hope you found the answer helpful.Wishing you good health.Dr Deepti Verma"
},
{
"id": 133922,
"tgt": "Why there is pain and lump type feeling under my right ribcage?",
"src": "Patient: I ve been going to the gym (cardio and weights) I have a slight pain on the right side with a lump type feel. It is located right below my ribcage. when I was on the elliptical yesterday my heart rate went up to 155 during the tough workout. should I be concerned. Doctor: hi,thank-you for providing the brief history of you.As to let you know one of the primary act is thorough musculoskeletal assessment is advised and also the cardiac clearance before undergoing gym. It is on a safer side to get clearance and then proceed.As mentioned by you that you have a pain under the ribcage or below it, it could be most of the time a muscular pain. Due to overtraining there will be a sudden mucle spasm due to stretch contract mechanism on the elliptical. Taking a rest with hot water fermentation should reduce the pain and after 3 days you can go back to gym. Also, if the pain doesn't come down, than taking up a thorough musculoskeletal and cardiac assessment is advised on a safer side. Since you have discomfort on the right side, the worry is less, may be just a muscle spam on the right hypochondria. It should subside in short time.This is on of the common injury in gym, as the body is not prepared to undergo the stress & strain on the muscles. Undergoing slow and steady training under guidance will help avoid such injuries in future.RegardsJay Indravadan Patel"
},
{
"id": 245,
"tgt": "Is pregnancy possible after unprotected intercourse while using Novelon tablets?",
"src": "Patient: Hello! I am having my periods from Wednesday (16th March 10). I was taking Novelon from Monday (14th March 10) as I wanted to delay my periods because of some professional engagements and family commitments. In the Meanwhile on Thursday & Friday (17th & 18th) I had Intercourse without any protection. I am still on Novelon. But Will I get Pregnant in this condition. Urgent Help Required. Doctor: Hello,No, you won't get pregnant. In a normal 28 days cycle, ovulation occurs around 14th day and pregnancy occur during that time if u keep a relationship.Hope I have answered your query. Let me know if I can assist you further.Regards,Dr. Sheetal Agarwal"
},
{
"id": 95284,
"tgt": "What is the treatment for pre-gastric pylaris ?",
"src": "Patient: age 30 years female weight 52kg. suffering last two months with gastric , abdominal pain , chest pain . Endoscopy result was pre-gastric pylaris. pls advise Doctor: welcome to healthcaremagic it's not clear from your statement the ailment you are having please consult physician/surgeon and get a check up gall bladder function to be seen also"
},
{
"id": 89376,
"tgt": "What could the smell from abdomen while drinking water be?",
"src": "Patient: When ever I take a large gulp of water I can feel it going down my throat and into my stomach. Suddenly as i am breathing normally a bad smell araises from what i think is my stomach. I sense the smell when I exhale. It stays for a little while and then goes away until i take another gulp of water. Until now i dont notice it when i drink anything else. Only with H2O. This is something thats been happening to me recently. How come??? Doctor: HI.The smell which you are getting from the abdomen while drinking water can be due to Cardia achalasia, wherein the esophagus hold the food. Another reason :This is called Foetor in medical language. You have a history of blockage of one nostril. SO this can be Due to Chronic Sinusitis, the sinus can have pus which gives you the feeling of Weird smell. I would advise you to have a consultation with an ENT Surgeon, get Clinical examination done; CT scan of the Sinuses and an X-ray of the chest is mandatory.Get the treatment according to the reports. IF the CT scan is normal we can consider the possibility of a problem in the Sensory System and you may need an opinion of a Neurologist."
},
{
"id": 210167,
"tgt": "How safe is taking Lexapro ?",
"src": "Patient: I took 1 10 mg lexapro fr the first time this morning and have been experiencing extreme nausea, jitteryness, and nervousness and weird burning sensations in my chest and arms. When should it go away? Im not going to take anymore after this... But when should this feeling go away and what can I do to make it better? Doctor: HIThanks for using healthcare magicElimination half life of lexapro (escitalopram) is 12 to 14 hours and a drug needs five elimination half life to move out of body completely, that means, drug would take one to two day to come out of body. Such side effects usually happens with initial doses of antidepressant, so nothing to worry about them. Just talk to your physician or psychiatrist and discuss about these side effects.Thanks"
},
{
"id": 23412,
"tgt": "What causes hallucination and increased bp after taking seroquel xr?",
"src": "Patient: I have frequent anxiety attacks or at least that s what they think is happening. I started taking SeroquelXR 50mg and the past two nights have been restless, BP went up to 151/110, had hallucinations, and had for about an hour deathly thoughts. I think these were all side effects from the medication and not the anxiety attacks that I m use to due to I ve never hallucinated or had any bad thoughts with the anxiety attacks. With the anxiety attacks my BP usually does go up, heart races, dry mouth, shortness of breath, and at times I start shaking. It last for a couple of hours and then I m just drained but otherwise fine. I don t like having any of these symptoms regardless if it is the medication or anxiety. What can I do? Doctor: Hello!Welcome and thank you for asking on HCM!I understand your concern and would explain that your symptoms could be related to seroquel XR adverse effects. Seroquel is usually useful to treat hallucinations and agitations, but it may also cause the opposite (aggravate them). You should discuss with your psychiatrist on the above issues. You may need to switch to another antidepressant like (sertraline, venlafaxine or mirtazapine), which have not the same adverse effects as seroquel XR. Hope you will find this answer helpful!Kind regards, Dr. Iliri"
},
{
"id": 223205,
"tgt": "Suggest birth control measures while on antibiotics for bacterial vaginosis",
"src": "Patient: I had a vagina infection called bacterial vaginosis, I took strong anti biotics for 5 days I took two a day, when I was off the tablets I had unprotected sex twice but he didnt cum in me, what are the chances of me getting the infection back and how do I prevent this? Doctor: Hello,First of all, the antibiotics used to treat bacterial vaginosis (BV) will not affect the effectiveness of the birth control pill. You did not state whether you are on the pill or not. Also, bacterial vaginosis not considered an\u00a0STD\u00a0so it will not be caused by having intercourse. It involves an imbalance of the bacteria already present in the vaginal area. Ways to promote normal vaginal bacteria include yogurt and probiotics as well as avoiding douches or other vaginal hygiene products.\u00a0Hope I have answered your query. Let me know if I can assist you further.Regards,\u00a0Dr. Timothy Raichle"
},
{
"id": 118184,
"tgt": "What does the elevated WBC level in my blood test report indicate?",
"src": "Patient: my age is 41 right now... i have just done blood test and result is my WBC shows 10400.. Doctor suggest me to take antibiotics like Zifi and Mobin.. is it work out? please suggest me that i am very much fond of bananas... everyday i am eating 4 to 5 bananas at the time... please suggest me Doctor: White blood cells, also called leukocytes, are an integral part of the immune system. They are responsible for protecting the body against infections and foreign bodies. The concentration of white blood cells in the blood stream is a key indicator of health. The normal concentration of white blood cells in the body for men and non-pregnant women is usually considered to be in the range of 4,500 to 10,000 leukocytes per microliter. A high white blood cell count usually indicates and the presence of an infection that the body is trying to fight. It may also suggest:\u2022 Anemia\u2022 Adrenal or thyroid gland issues\u2022 Bone marrow disease\u2022 Immune system disorders\u2022 Inflammatory disease\u2022 Leukemia\u2022 Medication-induced increase of leukocytes\u2022 Severe stress\u2022 Tissue damage. RegardsDR De"
},
{
"id": 28409,
"tgt": "Suggest treatment for rapid heart rate",
"src": "Patient: I have been taking 25m maxzide for years but the last year I am experiencing a pounding heart a day later and for a few days after. I only take it one day a week but I am concerned about my heart pounding. My doctor seems to think it is all in my head though I do have an irregular heartbeat which he says is not uncommon. Doctor: Hi,After going through your history, I want to know1) For what are you taking maxzide only on one day a week?2) Your age.3) Any other medical illness.4) Habits like smoking, drinking alcohol, any drug use.5) Any other symptoms like sweating, weight loss, chest discomfort.Occasional heart pounding in a healthy adult in the absence of giddiness, sudden black outs is most of the time normal.If you are really concerned about it you may under go following tests-1) ECG done during period of heart pounding (palpitation) it can give more information.2) Holter's monitoring or electrophysiological test for heart.3) 2DEcho (echocardiography of heart)4) Serum electrolytes, renal function test.5) Thyroid function test.You can take anxiolytic medication after consultation with your GP."
},
{
"id": 84502,
"tgt": "What to do for stomach burning sensations after taking oleanz (10mg?",
"src": "Patient: Hi I am BCA student I have SCHIZOPHNIA I was taking oleanz (10mg) I have sever stomach burning sensations so i had stopped taking up the medicines . please suggest me should i continue taking the medicine I have a sleep disorder please advice and thanks Doctor: Hello, It is better to consult with your doctor because of side effects that this medication has. You can't stop abruptly without consulting with psychiatrists. Hope I have answered your query. Let me know if I can assist you further. Regards, Dr. Blerina Pasho, General & Family Physician"
},
{
"id": 50284,
"tgt": "Child with lowering kidney grade, has undergone kidney biopsies, taken medicines. How can she regain health?",
"src": "Patient: Since 2006 my daughter had been continued unded the adviced of a reuhomatologist doctor, during this treatment period did kidney biopsy four times due leaking protien in urine . First biopsy done at Gang Ram, then twice at Fortis Hospital in Delhi. In the mean time given Mepthera (Retuximap) weatern drug in interval of times but still my child kidney grade is becoming VI to V in biopsy. More over taking other medicine like Steroid , HCQS, Renodap, Takfa, and other B.P.medicines etc.So pl. advice, t o get back her healyh Doctor: She will be required to be seen in detail before a decision for revision of treatment can be undertaken.It appears from your description that your child is having Active Lupus Nephritis which needs to be treated effectively"
},
{
"id": 225397,
"tgt": "On next choice plan b pill, delayed periods, acne, diarrhea and cramps. Could be pregnant ?",
"src": "Patient: I took the next choice plan b pill on January 1st 2013 and got my period roughly 5 days afterwards and then again on the 18th. Then I took that same pill on the 31st of January. And now I m supposed to have my period on the 6th of February what I m a day late, but I have had acne , diareah, and cramps . Could I be pregnant? Doctor: Hello ma'am.It all depends on when you took the pill, as if taken within 24 hours the pill has a 98% efficacy, if taken within 24-48 hours it has a 85% efficacy and if taken within 48-72 hours it has a 58% efficacy.Your other symptoms could purely be hormonal, so please wait and conduct a home pregnancy test using an early morning sample if you do not see your periods within the next week. Report to a doctor after that.Hope this helps.Best wishes."
},
{
"id": 49092,
"tgt": "Should i be worried about a shadow after a kidney ultrasound?",
"src": "Patient: Hi my son had a kidney ultrasound done a week ago and they seen a shadow , we had another one done today and showed the same thing. I have an appointment on feb 5th to see the specialist and I m panicking..I hope it is just the pic messed up. SHould I be worried? Doctor: Shadown on kidney ultrasound usually means a calcification like a kidney stone. A CAT scan will really highlight what is causing it and that would be the next step."
},
{
"id": 6398,
"tgt": "Are there any chances of pregnancy since I had unprotected sex and have pregnancy symptoms but my pregnancy test shows negative results ?",
"src": "Patient: Hi, so im 18 years old im 5 5 an aboust 130pounds i just had my daughter 5months ago.. an soon after my 6week check up i gotten the depo shot the whole 3mon i was on the shot it was not stop bleeding my appt due for my next shot was july 1 2011 i never got it i have had inter cours since unprotected an have not had my period yet i have been having alot of discharge brown and clear and only get a little bit of blood when i poop? I have once in a while get sick and throw up have sharp shooting pains down there like when i was pregnant and my feet get swollen if im on them to long.. i took a test and it says negative.. im wondering if this is normal? Doctor: hi welcome to hcm.see it may possible that you breast fed your baby so you should not have your period.as you said you go through pregnancy test which was negative then no need to worry about that. or you told about brown discharge it may because of infection so you need once visit of gynec if it continue itchy having bed smell . thanks dr.dhara dhara.shah84@yahoo.in"
},
{
"id": 137316,
"tgt": "How can recurrent knee pain caused by injury be treated?",
"src": "Patient: I have injured my knee before , the doctors said it was a sprain. I still have troubles with my knee. I am a dancer , and today I think i worked too much while dancing. My knee started to hurt and everytime I move It feels like it s about to pop, what is going on ? And should I be worried about it getting re injured Doctor: Hello, I have studied your case.As per your age and history there is possibility of soft tissue injury in knee.Most commonly meniscus injury occurs may be associated collateral ligament injury. You may need to do MRI knee to see for any functional defect.As per your symptoms you need to do physiotherapy and knee exercises to improve range of motion.Rest and ice fomentation along with supportive brace will help to reduce locking episode and pain.Small meniscus tear can be managed conservatively with physiotherapy with USG and TENS. Large meniscus injury can be managed by arthroscopic repair.Hope this answers your query. If you have additional questions or follow up queries then please do not hesitate in writing to us. I will be happy to answer your queries. Wishing you good health.Take care."
},
{
"id": 142601,
"tgt": "Can I take Relent along with vertin for tremors and disorientation feeling?",
"src": "Patient: Suffering from cervical prob gap C 4 n 5 n C 5 n 6. Heavy head especially behind the ear above neck n great discomfort. Audiometric test shows 20 to 25% hearing loss in left ear. ENT advice to see Neuro n do MRI Brain . B 12 is 420 n D3 is 25.Feeling tremors in the body n feeling disoriented. Can I take tab Relent alongwith vertin 16mg. What should be my next course of action Doctor: HIWell come to HCMI really appreciate your concern, if the vertigo is due to vestibular dysfunction then you can take the vertin, and you can have both the medicine together, but in my opinion better to find out the cause of vertigo, once it is found then it has to be manage accordingly, you have not given the basic detail like age and gender hope this information helps."
},
{
"id": 26217,
"tgt": "What are the side effects of Triolmezest tablet?",
"src": "Patient: Dear Doctor,I have been prescribed Tablet Triolmezest for hyper tension for 5 days. after 4 days of consumption, my blood pressure became 80-130 from 100-150. but my main doubt is would this consumption of tablet cause any erectile disfunction? please advise Doctor: Hi,Thanks for asking us!Triomezest is a pill that contains three medicines namely Amlodipine, olmesartan and hydrochlorthiazide. While a combination of the three drugs is excellent in bringing down blood pressure as seen even in your BP readings. Amlodipine and olmesartan are not known to cause erectile dysfunction.Hydrochlorthiazide can cause erectile dysfunction only at high doses such as 50 mg. You are most likely taking the 12.5 mg dosage that will not usually cause erectile dysfunction. So you may stop having any worries about your medicines. Hope I answered your queries.Cheers. Dr. Melvin"
},
{
"id": 211846,
"tgt": "75 year old, seeing people in house who is not there. Taking aricept medicine for a year. Is this part of dementia?",
"src": "Patient: Hi, I have a 75 year old father-n-law who is experiencing seeing people in his house that is not there. This morning he said that my mother-n-laws Indian statue collection was looking at him and laughing at him and he saw a rabbit or squirrel under a shirt on a foot stool. His only medication he takes is aricept and has been on this for year or so. Is this part of dementia? Doctor: Hi,Thanks for using healthcare magic.I have gone through your available history.Visual hallucination and delusions are part of psychotic symptoms associated with dementia.You should consult psychiatrist for detailed evaluation and management.After confirming diagnosis and ruling out other causes, doctor may advise him to take risperidone or aripiprazole like anti psychotic agents in addition to cognitive enhancers.Doctor may advise Vitamin B12 blood test to rule out its deficiency as a cause.I hope It may help.Thanks."
},
{
"id": 30682,
"tgt": "Suggest remedies for itching and painful skin on my testes",
"src": "Patient: hello there.... m suffering from intense and extreme skin dryness on my testis skin... due to dryness the skin is breaking apart and creating pores... pus and blood is oozing out in small amounts...but that doesnt matter.... wat matters is the pain... its really painful if i wear my underwear Doctor: Hi,From history it seems that there might be having some chronic infection on local part producing dryness and cracks.This can be due to fungal infection or eczema.Due to irritation and itching now there might be having secondary bacterial infection giving rise severe pain, pus and induration.You initially require one course of antibiotic medicine foe 5-7 days to control infection with proper dressing with antibiotic cream.After control of infection proper diagnosis of this lesion can be made.Consult dermatologist and get examined.Ok and take care."
},
{
"id": 211507,
"tgt": "Struggle with mixed depression and anxiety symptoms. On Pristiq. Could I ask a doctor?",
"src": "Patient: Depression and anxiety query:Hello, I have an extreme child trauma history of abuse, neglect and various others. Have come out with a mixture of diagnosis (mainly and in no particular order: Depression, Anxiety, Social Anxiety, Obsessive Compulsive, Post Traumatic Stress and Borderline Traits). I have managed majority of these, but still struggle with Mixed- Depression and Anxiety symptoms. I have seen Psychiatrists this past year and have been on Pristiq for approximately 1 1/2 years now. I have tapered down from 150mg, to 100mg, to 50mg, to 25mg and have now come off of anti-depressants completely.I have very specific queries in regards to managing my symptoms. I wish to go on natural therapies mixed with Clinical Psychology treatment which I am currently in my 2nd year of therapy. I am starting on St. John Wort's in a few days once the Pristiq is out of my system. I have been researching DMAE and Niacin quite extensively, but just not sure where to take it from here and would like advice as to an experts opinion as to my best options. I do not use illicit drugs, use nicotine nor on any other prescription drugs. I do however, drink more than I'd like (3-4 times weekly, mainly weekends where intake is approx-5 s.d.). I have family history of depression, schizophrenia, bi-polar, anxiety, drug and alcohol addiction and abuse etc.If would really appreciate to have some advice from a professional in the field's recommendation. Doctor: Hi, I can understand your concern about medication.Considering strong family history,Drinking almost 3-4 times per week give indication that high possibility that you might be having underlying untreated psychiatric disorder.Such kind of illness is like Diabetes or Hypertension which can be only controlled on medication and not going to be completely cured.Most of time illness is episodic so you feel better in between episode and feels that you able to control your illness-but remember next time when it reappears it going to be more and more difficult to treat and resistant one.Ongoing continuous medication saves you from these fluctuation and helps to maintain your quality of life as normal as possible.Medicines are like specific extract from your natural product with avoiding amount of other chemicals which not going to needed for improving current illness.So its always advisable to continue medicine and along with this continue psychotherapy from experts.Instead of mentioning name of psychiatric illness mentioning current disturbances will be much helpful to advise any further management.Hope this will helps you.You can Post any further query if you have."
},
{
"id": 221485,
"tgt": "What causes delayed periods and negative pregnancy test?",
"src": "Patient: Hi, may I answer your health queries right now ? Please ty pe your query Hi, im 26 years old and I m sexually active, my last period was october 8, and its now december 5. I have taken 3 pregnancy test and all were negative, on november 22 I was having light cramping and then light pink spot when went to bathroom, then nothing else, now I have notice that the center of my nipples are white. Could I be pregnant but not show up on a test? I do feel sick at times. Doctor: HI, I understand your concern. You have 2 months delay in periods,with slight spotting after 1.5 months.. pregnancy test done twice was -ve.. this strongly suggests absence of pregnancy.Still if you are doubtful USG scan of pelvis would give perfect deciding result. Absence of pregnancy is mostly due to hormonal imbalance due to- Being on Depo contraceptive/ PCOD/Ovarian or thyroid problems. Long standing debilitating illness/ psychological stress. * your symptoms of being sick, nipple signs are due to hormonal imbalance only. * You need a gynecologist's consultation & needful investigations for pin pointing the cause.followed by specific treatment. Thanks."
},
{
"id": 213138,
"tgt": "Intermittent episodes of tremors, increased heart rate, breathing trouble, stabbing heart pain. Panic attacks?",
"src": "Patient: Every once in a while I will get uncontrollably shaky and my heart rate is already naturally over 100 but it beats faster at certain moments. Sometimes there will be moments where it hurts to breath and there is a stabbing pain on the side where my heart is. I have gone to a doctor but they say it is just a panic attack. My problem with them summing it up to being that is that these symptoms will only occur randomly. I could be feeling great with no worries, I could be completely stressed out. It never has a connection. I am currently at work and I can t seem to calm down, I keep shaking and my heart is beating a little bit faster. What is wrong? Doctors keep summing it up to anxiety or panic attacks. Is that true? Doctor: Hi Anxiety is likely but not the only cause. There could be several reasons - it sometimes happens normally also! Anxiety may not be the case with you - random and no relation, as you say. It may be Mitral Valve Prolapse , anomalous pathways and so on. Proper treatment needs proper diagnosis. First thing to do will be to see a cardiologist and get examined - especially during one such episode. In this situation, we start with routine ECG, ECHOcardiogram, blood tests (cell count, sugar, thyroid). If there is no clue, we go further - 24 to 48 hour ambulatory ECG monitoring (Holter). Your doctor will decide on the need for medications - beta blockers for example can help in both anxiety and palpitation Wishing you a speedy recovery Good luck"
},
{
"id": 123906,
"tgt": "Have I broken my shoulder?",
"src": "Patient: i fell and my right arm broke the fall with all my weight on it and I heard some kind of noise and felt something in my shoulder. It is not bruised yet, but somewhat tender when I move it to high. I have iced it and am wondering what this injury could mean. Doctor: Hello, As the mechanism of injury indicated it will be an outstretched hand during the fall. This must have damaged the muscles of the shoulder joint and the shoulder joint ligaments too. If there is no Bruise or swelling then it will be a ligament and muscle injury which is common. I will advise you to stabilize the shoulder in an arm sling and do hot water fermentation. If possible an x-ray will be good on the safer side. Doing stability exercise like static biceps, static triceps, static deltoid should help to stabilize the shoulder joint. Post which after 3-4 weeks start strengthening exercises which are free exercises mostly to regain the shoulder range of motion. Post achieving this, strength conditioning should help get good strength. In my clinical practice of over 12 years, most cases with shoulder injuries are advised for shoulder rehabilitation program and respond well. Hope I have answered your query. Let me know if I can assist you further. Regards, Jay Indravadan Patel, Physical Therapist or Physiotherapist"
},
{
"id": 63998,
"tgt": "What is the painful lump just below my breast?",
"src": "Patient: I am a 29 year old female. I'm 5'8, 220 lbs. Right under my breast, on my stomach, I have a firm spot. That's the only way I know to describe it. It only hurts when I touch it but last night the whole upper portion of my stomach was like this and painful Doctor: Hi,DEAR,Good Evening,Thanks for the query.I studied it in details and understood your health concerns regarding it.Lump caused under right breast-on the stomach-is mostly due to the Epigastric hernia ? or due to the boil under the breast.Hope this would help you to plan the treatment with your doctor.Wellcome for any further query in this regard.Wishing you good health.Good Night.Dr.Savaskar M.N.From INDIA."
},
{
"id": 107300,
"tgt": "What causes sharp pain and stiffness in the lower back?",
"src": "Patient: My wife 50 years old wakes up with stiffens entire back pain lower back right and left throbbing and occasionally lower left back sharp pain. The only thing that relieves pain is heat spray.stretching , yoga , heating pad chiropractor just seems to make it worse. Doctor: Hi Hope this message finds you in good health.I have gone through your query in detail and understand your concern.She may be suffering from lumbar spondylosis.Considering her age,she must be having osteoporosis,leading to degenerative disc disease of the lumbar spine.Get her MRI of the lumbar spine done.Physiotherapy,yoga and using a lumbar brace would be helpful.Consult a orthopedist.I hope the answer was useful.Get back to me for any FOLLOW UP QUERIES anytime.Best wishes"
},
{
"id": 174027,
"tgt": "Suggest treatment for severe diarrhoea and high fever",
"src": "Patient: Hi, my 4 yr son got severe diarrhea last week , he was having high fever also, so peadiatrician kept him on drip for one day with continous injection for next five day. So he was fine untill he was having injections but on 6th day he got again diarrhea, after that his medicine was replaced as he is taking ofloxacin and ornidazole suspension with prebiotic sachet.Now he is fine and having no complaint of loose motion but some time he complaints for mild stomach pain , so there is somtehing which is not cure yet or any kind of infection.please guide me. Doctor: Since the fever and loose motions are controlled, we can expect that the infection is controlled. Is the pain in the tummy severe or prolonged? If not, I think it is not of a great concern. Make sure he eats at regular intervaks and takes plenty of water."
},
{
"id": 79911,
"tgt": "Are there any restrictions like travelling, doing pranayama while taking medication for ILD?",
"src": "Patient: I am 57 year old recently diagnosed with I L D. I have been put on dosage of Wysolone 10 mg one per day. ,Mucinac 1 tab three times a day. Serflo caps( 500mg) two times a day. I suffer from controlled hypertension and do not have diabetes. Treatment has started 7 days back and i feel better. I want to know what kind of precautions do i need to take. is there any travel restriction?. Whether i can do pranayam etc(Yoga). which i used to do earlier. Doctor: Thanks for your question on Health Care Magic. I can understand your concern. You are taking wysolone. This is oral steroid. Oral steroids are known to cause immunosuppression. So possibility of infection is high in your case. So traveling in crowded area, visiting hospitals etc should be avoided to prevent infection. Eat Healthy food like fruits, vegetables, start multivitamin and antioxidants tablet to improve your immunity. Yes, you should definitely do yoga and pranayam. Both are good for health. Better to enroll yourself in Pulmonary Rehabilitation center where chest physiotherapy and deep breathing exercises are done. These will improve your lung functions. Hope I have solved your query. I will be happy to help you further. Wish you good health. Thanks."
},
{
"id": 175245,
"tgt": "Suggest treatment for high fever",
"src": "Patient: My 3yr old has a very runny nose, a high temperature, cold feet, aching arms, and stomach pains she is also complaining her eyes hurt. She is very lethargic .She hasn't been eating properly for over a week now either only eating very little. Please help Doctor: Thanks for consulting in Healthcare. It seems your daughter has acute viral infection, acute rhynopharingitis, she feels pain in gurgle, her appetie decreased,thats why:1.Try to keep her with plenty of water or ginger, lemon and turmeric powder tea2.Cut onion, leave with a water near her bed, fitoncids will kill viruses, you can drop onion water in nose 1 ml 4-6 times, very effective3.Drop nose with saline and nasal drops , especially before sleeping4.Use calpol syrup from temperature or adol suppositories 250 mg if she can not drink5.Tulsi juice 5ml 2 times will help to decrease temperature6.Sualin 1 tab 3 times for resolution with anti inflammatory effect7.Physical methods:cover her wet sheet or take a bath with usual hot water and her temperature will decrease after 2 min8. For increasing immunity and decreasing temperature Vitamin C 500mg is also helpful, give during 5 days9.Multivitamins-zincovit 5 ml 1 time 1monthRecovery soon"
},
{
"id": 144362,
"tgt": "Would I need operation to treat the findings in MRI report?",
"src": "Patient: My MRI report said - central and right paracentral disc bulge causing right anterolateral thecal sac indentation.right lateral recess stenosis resulting compression over the L5 traversing nerve on right side at L 4-5 level. will i become alright without any operation ? Doctor: hi,thanks for providing the brief history of you.as you mentioned about the MRI reports you missed to mention about your symptoms. because most of the scans may reveal many things but has to correlate with clinical signs and symptoms of the patient which is always mentioned in the reports.in case you have not tried physical therapy you can please try the same as it is a non invasive way of helping patients with disc injury and other most cases.your physician will help you by medicine which will help you to get ease of pain symptomatically.also to mention that use of a therapeutic ultrasound therapy and TENS for this issue by a physical therapist helps the pain level to come down. post that an exercise regimen is implemented which helps strengthen the muscles and release the compression from the nerve which also reduces the symptoms to a great extent.usually most patient of mine have responded well under the medicine and physical therapy. I never had to recommend the patient to undergo surgery in 99% of cases.with the grace of God I wish you a speedy recovery"
},
{
"id": 226218,
"tgt": "Had unprotected sex. Taken plan B. Worried about pregnancy",
"src": "Patient: I take birth control and i last took it saturday had unprotected sex on sunday then realized i missed it so i took plan b on tuesday along with the pills i missed should i be worried that i am pregnant. my period is suppose to be next week but i thought i was suppose to get it early if i took plan b. im scared that im pregnant but have no symptoms to show it. Doctor: Hello, Pregnancy is possible even by missing a single dose of birth control pill. Since you have taken PLAN B ,within 72 hours,the chances of pregnancy become very less. The efficacy of PLAN B is 89% and so you do have some chance of getting the pregnancy. Wait for your periods and if you miss them,then get the urine tested for pregnancy. You may then act accordingly according to the result. If pregnancy is absent,then repeat the test. If repeat test after a week is again negative,then ou may consult your doctor for the necessary action. If pregnancy is positive,then you may consult your doctor for the choice between medical or surgical abortion. Thanks"
},
{
"id": 78767,
"tgt": "What causes chest pain with stinging and burning feeling?",
"src": "Patient: Hello There Here is my Dillemma and what I have been dealing with for years. I am 31 now. This has been going on since early 20s Constant left Chest Pain. Feels like a Dull Ache that sometimes is tolerable but sometimes it flares up to a stinging, Burning muscle spasm. For the last 3 to 4 years years i have been to the ER on multiple occassions. Always get the Nine Yards EKG, Chest XRay, Blood work. Everything checks out great, Im discharged. Last visit was this week and the time before was in August. In then many times over years. I saw my primary care doctor who is a lung specialist as well. Gave me a full physical and at the end said im Healthy. 8 months ago i was diagnosed with a Hiatal Hernia and Acid Reflux. I probably had that for years. It causes Weird Symptoms. My GI Doctor thinks im experiencing the symptoms she diagnosed me with. It is very frightening. My Primary care Doctor gave me Carefate this week and my GI Doctor gave me Dexilant. I have had this awful sensation in my chest and at times i rather have my leg hurt all day, Nobody thinks I have a Cardiac Issues and give me clean bill of health. Could this be Stress and Axiety, It makes me panic for years :-/ I need some assistance please! Hope all is well Mike Doctor: Thanks for your question on Health Care Magic. I can understand your concern. Hiatus hernia can cause all the symptoms described by you, so do not worry and continue your treatment as prescribed by your GI specialist. Also eat small frequent meals instead of large meals. Exercise regularly , do not lift heavy weights, keep your head end elevated by 2-3 blocks or pillows, give a gap of 2-3 hours before you sleep after your dinner. Don't worry, you will be alright. Hope I have solved your query. Wish you good health. Thanks."
},
{
"id": 99245,
"tgt": "can allergies to pollen cause ear pain?",
"src": "Patient: As an athletic 57 year old, I play competitive tennis 4 days a week and I run 3 days a week about 3 miles. Within the last 3 days I ve had ear pain ( throbbing ) and lightheadness on and off during the day.. My doc cleared out the ear wax but it didn t help. It just pulsates with pressure. Today I played tennis and had shortness of breath by the end of the hour. Could this all be due to allergies to pollen? The pollen is high here in Dallas. I played tennis yesterday and was fine. I also took 2 Mucinex tablets before I went to play today. Could that cause a shortness of breath? No stuffy nose but headache pressure and ear pressure. Help, Becki Doctor: HI, thanks for using healthcare magicThe headache pressure and ear pressure may indicate that you have a sinus congestion (which can be caused by pollen) and possibly a ear infection.The use of anti histamines , decongestants and possibly medication for an ear infection, if present , may be needed.Mucinex is not known to cause shortness of breath.If this associated with chest pain, palpitations,then you may need to see your doctor.I hope this helps"
},
{
"id": 121150,
"tgt": "Does Spondylitis cause pain & tender calf muscles?",
"src": "Patient: I am an patient with Anklosing Spondylitis. With the weather in Kansas becoming cloder my symptoms are flairing up! I have had pain in my calf and up the inside of my left leg, with tenderness around the inside of my knee joint. Is this something that can be caused by my AS? Doctor: Hello,I read carefully your query and understand your concern. The symptoms seem to be related to anklosing spondylitis.I suggest using anti inflammatory medications such as Acetaminophen to relieve the symptoms. Cold compresses can be applied locally for pain relief. Hope my answer was helpful.If you have further queries feel free to contact me again.Kind regards! Dr.Dorina Gurabardhi General &Family Physician"
},
{
"id": 208379,
"tgt": "What causes temporary memory loss?",
"src": "Patient: I KNOW A PERSON WHO HAS JUST STARTED WITH FORGETTING THINGS, SUCH AS SOMETHING WHICH WAS SAID OR DONE THAT DAY AND SHE DOES NOT REMEMBER UNTIL IT IS POINTED OUT TO HER. IS THERE AN UNDERLYING MEDICAL PROBLEM THAT SHOULD BE CHECKED ON FOR THESE EVENTS. Doctor: Hi, thanks for the query. Memory problems in a person need certain details like age of person, nature of onset of problem(rapid Vs. Gradual), areas of life affected (like difficulty driving, wearing cloths, difficulty in household work etc.); whether person himself/herself is aware of the problems or whether his/her relatives are complaining & the concerned person is unaware or trying to hide his/her problems etc. Hence kindly provide more information. Lastly memory problems are seen in host of conditions ranging from old age related memory difficulties, dementia to as a symptom of depression. Hope this helps. Good Luck"
},
{
"id": 212363,
"tgt": "Having behavior problem of talking to myself, anger. Work, life unaffected. Is it normal?",
"src": "Patient: Hi i am by profession a fashion designer and running an interior firm too, running since past three four years.. since few months i have develop this unrest in me.. unrest means i swear out loud (mostly when i am alone, its like i am talking to myself or angry to myself) and i start speaking out when i start thinking deeply... i sleep well, do my work without any problem.. living a normal life.. i would like to know why i do this , is it normal..or do i need to take precaution Doctor: You should ask yourself the following questions How do you feel about yourself Do you feel depressed Do you feel somebody is trying to harm you Do you see abnormal things Do you hear any voices Are you able to sleep well Do you have any plans to harm or kill yourself Do you smoke or drink excessively Do you use any drugs or medications Have you been diagnosed or treated for any psychiatric illness If you have problems in any of these areas, then it is important for you to visit a psychiatrist. Also if you habit of talking to yourself is not affecting you from otherwise carrying on your life and activities, it is not a cause of concern."
},
{
"id": 140515,
"tgt": "Are intrusive thoughts a psychiatric or neurological disorder?",
"src": "Patient: I have had issues with depression and have been diagnosed with bipolar NOS, but lately my brain has been going crazy with intrusive and constant odd thoughts like... What would Grover Cleveland s head look like on a horses body, cobras wearing neckties, rabbits running the senate of Portugal, zebras flying, rhinos discussing their favorite apple varieties and reindeer holding the golden girls hostage. I could go on and on, I asked my psychiatrist if it was a neurological disorder and she didn t have a clear answer. I am getting worried and actually missed work last week because the thoughts were so pervasive. Is this psychiatric or neurological? Doctor: Hello, Intrusive thoughts are a feature of psychiatric disorders. More exactly, these symptoms are related to obsessive-compulsive disorder. Discuss with your Psychiatrist for these issues. Hope I have answered your query. Let me know if I can assist you further. Regards, Dr. Erion Spaho, Neurologist, Surgical"
},
{
"id": 37982,
"tgt": "How can Lyme disease be treated?",
"src": "Patient: Hi, I have Lymes disease. I was bitten by a monkey 18 months ago in a zoo in Honduras while on a cruise. Everyone else was taking photos with the monkeys so they handed it to me and it bit me. I have been sick ever since then. I know Cats claw is good for helping eradicate the spirochete bacterial infection. Is there anything else that you know of that will get rid of Lymes for good. Peggy Doctor: Hello, Thank you for your contact to health care magic. I understand your concern. If I am your doctor I suggest you should first diagnose yourself for the Lyme Disease before you got treatment. You can get tested yourself with ELISA, IFS or Western Blotting to confirm the diagnosis. If you are confirm as having Lyme disease you should take doxycycline 100 mg two times a day for 14 days to cure the disease. The causative agent Borellia burgdorferi a treponema bacteria is very sensitive to the drug and you will be treated for with in 14 days.I will be happy to answer your further concernYou can contact me. Dr Arun Tank. Infectious disease specialist. Thank you."
},
{
"id": 185806,
"tgt": "What causes sewage taste in mouth and suggest treatment?",
"src": "Patient: hello basicly i have a sewage taste in the back off my mouth yet iv changed my diet i am eating like alot of fruit and vegetables but i aint eating alot either like one meal a day today iv decided to eat breakfast and lunch but its the sewage taste im worried about Doctor: Hello,The condition of your mouth including tongue, teeth and gums can be a factor. Have you had a recent dental exam and cleaning? Be sure to practice good oral hygiene including brushing your tongue. Infected tissues can change your taste. Infections under the gums can drain and these can give you a bad taste. Drainage fom the sinus to the back of the throat such as a post nasal drip can affect your taste. A dry mouth, again, affects taste. Other habits such as smoking or recently stopping a habit such as smoking affects taste. Medications? Any recent antibiotic treatment can affect the balance of your oral bacteria. The back of your throat may collect debris prone to attack by bacteria. Tonsils have crater like areas for debris to get caught. Tonsil stones or tonsilloliths are unpleasant. They can be foul smelling and require removal. Have a doctor check this area. You may be able to see some debris yourself and can try vigorous rinsing. A foaming, peroxide rinse may offer some assistance. Professional treatment may include removal, antibiotic treatment and removal or reshaping of the tonsils.I hope you are able to remedy this situation soon. Additional questions are welcome."
},
{
"id": 39112,
"tgt": "How long does anti rabies vaccination work in body?",
"src": "Patient: how long is the effect of anti rabbies in the body when vaccinated? i have been bitten by our cat i think 2010 and i have some scratches too..i had taken rabipur, they vaccinated it to me 3x.. and now, our cat bit me again..the one that bit me is a domesticated cat. do i have to be vaccinated again?thank you Doctor: Hello,Welcome to HCM,Cat is a known reservoir of rabies virus and it can transmit rabies to the humans. Rabies is 100% fatal but it is 100% preventable by proper and adequate treatment.As you were bit by a cat previously for which you have received the entire course of anti rabies vaccine. Once again you were bit by cat recently.According WHO categorization of animal bite, you will be categorized into Re-Exposures. You should receive two doses of anti rabies vaccine on days 0 and 3. The vaccine should be taken to shoulder.Thank you."
},
{
"id": 69246,
"tgt": "Suggest remedy for lumps in armpit",
"src": "Patient: I have a lump in my armpit that started the size of a pea and has grown in a couple years to the size of a half of a peanut shell. It is oval and not visable at all. It does move around if touched. Slightly sore to touch. Otherwise not really bothersome. What could this be? Doctor: The lump you have described is likely an axillary node... just get it examined by your surgeon... if clinically indicated may need imaging and Biopsy/FNAC"
},
{
"id": 7748,
"tgt": "How can I cure my open pores on face due to acne ?",
"src": "Patient: Hello! excuse me, I have a hard time to get rid of some small holes on my face especially on my both cheeks. It is from acne that I had before, so can you please tell me how to cure it or tell me the kind of effective medicine to cure it? It just the small acne on my face and when I try to get them out, my face start having small hole. so it is hard to erase those hole from my face. Doctor: Hi, Acne scars as mentioned on both cheeks in above case, needs treatment other than single drug. Chemical peeling- which is done at hospital setting ,is one way to clear holes. Another option is non-ablative laser treatment which trigger the changes in dermis without injuring epidermis."
},
{
"id": 24900,
"tgt": "Suggest remedy for breathing problems and blurry vision",
"src": "Patient: I AM 44, HAVE SEVERAL HEALTH ISSUES LIKE RA, SLEEP APNEA, INTERSTICIAL LUNG DISEASE, LUPUS, PULMONARY HYPERTENTION AND FIBROSIS, HYPERTENTION AND SJOGRENS SYNDROME RIGHT NOW I AM STRUGGLING TO BREATH WITH EVERY MOVEMENT I MAKE AND NOW MY VISOIN IS BLURRY IN MY RIGHT EYE AND I AM BLIND IN THE LEFT. SO WITH THAT BEING SAID, Y IS MY VISION BLURRY? ALSO I AM ON OXYGEN Doctor: The constellation of difficulties potentially causing symptoms and complicating your daily life are such that reliable advise can only come from someone with a very intricate understanding of you. In most instances, your rheumatologist with be the best quarterback for you as each problem may have its own specialist in control. Start there and work with an opthalmologist, preferrably one who works with your Lupus specialist, to seek a cause for blurred vision."
},
{
"id": 137299,
"tgt": "What causes soreness and blister after hitting my back?",
"src": "Patient: I fell a few days ago at the clothing store and hit my back on the bottom of a metal rack. I had X-rays and everything looks ok, but I am extremely sore in some of the area and there appears to be some very small blister-like areas. is this something to be checked further by a doctor? Doctor: don't worry, rib contusion hurts for many days also without a fracture. are there swelling and vesicles on the skin above?"
},
{
"id": 1777,
"tgt": "What are the best conditions for conception?",
"src": "Patient: Hi I am Suganya.... I am 29 yrs old. i am doing my follicular study. on 11 th day my RT ovary is 10.0 * 10.0 mm and LT ovary is No dominant follicle. on 13 th day my RT ovary is 14.0 * 12.0 mm and LT ovary is No dominant follicle. on 15th day my RT ovary is 19.0 * 16.0 mm and another one is 14.0 * 11.0 mm and LT ovary is No dominant follice. Can i know when it will get ruptured. And when i need to have intercourse to get preganant..... Doctor: Hi, a follicle ruptures when it reach a size of more than 17 mm. So, your follicles can rupture any day now. If you want to be sure of the rupture, take an injection for that after consulting your doctor. Be in contact with your husband for next 2 to 3 days. Take progesterone for next 2 weeks. Do a urine pregnancy test after that. Hope I have answered your question. if you have any other query I will be happy to help. Regards Dr khushboo"
},
{
"id": 150955,
"tgt": "Seizures, seeing things which are not present. Not on medications. Causes?",
"src": "Patient: hello, for a very long time now i have been seeing things that where not there and i have been feeling like i am almost not in reality i almost feel like im dead or in a dream. I am not on any medications but about 5-6 years ago i did have a seizure. i did have these things happen to me before but after i had one it has gotten worse over time. Do you know what it could be? or possible things i could do to help myself Doctor: Hi, You got hallucinations then, and depending whether they are sensory or motor, a neurologist decides on the appropriate treatment. A CT scan is desirable in this case with an exam to monitor and measure your electrical activity in the brain called an Electro Encephalogramme done in nuerology services. With results of these exams, a clear cut diagnosis is then made and a proper treatment plan proposed. I suggest you visit your neurologist. Thanks and fast recovery, Luchuo, MD."
},
{
"id": 26722,
"tgt": "How to monitor heart condition at home?",
"src": "Patient: My father had a heart attack just the other day, it happens frequently, I would just like to know if there is any way he can check at home if his heart is ok? He doesn't have money for a doctor and I'm living too far from him to help directly... Please help Doctor: Hello. Thank you for your question and welcome to HCM. I understand your concern. I would like to provide you only with the medical side of your concern, as I cannot speak about other administrative things. There is a window period after a heart attack that is accompanied with the highest risk of complications. A after heart attack patient, even at his best looking condition, can produce these complications, because the damaged part of the heart muscle goes through some gradual changes which start to happen 30 minutes after the attack up to six weeks, when this process is completed, and, finally, the previously normal tissue of the heart is replaced with non-functional scar tissue. This window is known to be 7-10 days counted from day one of the heart attack. These complications are sometimes serious and potentially life-threatening. Therefore, as a cardiologist, I would strongly recommend that he should be hospitalized in a specialized intensive cardiac care unit for further monitoring and treatment. Everyone can always take the risk and go home the day after the heart attack. But, it is a risk I would not recommend to anyone, both as a person and, especially, as a cardiologist. I hope this answers your query. Wish him a good health. Kind regards, Dr. Meriton"
},
{
"id": 152539,
"tgt": "Is fibroglandular density a symptom of cancer?",
"src": "Patient: Just diagnosed with fibroglandular density going for further 3d and ultrasound in two weeks have family history of breast and bone cancer mom and sister both have died , last year younger sister xxx of breast cancer what s percentage this may be early cancer Doctor: Hello and Welcome to \u2018Ask A Doctor\u2019 service. I have reviewed your query and here is my advice. Breast density is a comparison of the relative amounts of fat versus fibroglandular tissues in the breast. Women with \u2018dense\u2018 breasts have a higher percentage of fibrous and glandular tissue and less fat tissue. (Fibroglandular parenchyma, basically means breast tissue). For a woman, having dense breasts is associated with a higher risk of breast cancer. Only age and BRCA gene status carry a higher risk than dense breast tissue according to research. A woman who has a first-degree female relative with breast cancer has almost twice the risk of a woman without this family history. If she has more than 1 first-degree female relative with a history of breast cancer, her risk is about 3-4 times higher. In your case, you're at high risk because of a strong family history of breast cancer, you and your doctor will develop a screening plan tailored to your unique situation. Recommended screening guidelines include: a monthly breast self-exam, yearly breast exam by your doctor or nurse practitioner, mammogram every year starting at age 40. Your personal screening plan also may include the following tests to detect any cancer as early as possible: MRI (magnetic resonance imaging) of the breast, ultrasound Hope I have answered your query. Let me know if I can assist you further."
},
{
"id": 151033,
"tgt": "Shooting pain from lower back to leg, difficulty walking. Spinal problems?",
"src": "Patient: Hi Doc, I am xxxxxx, 49 yrs old and up until yesterday, I was fine until my back went out with a huge pain from my leg to my lower back and thought great I ve slipped a disc in my spine and having trouble walking, I ve been exercising my spine hoping to push it bac into place. i m an active person and had no warning, what can I do? Doctor: Hi, Thank you for posting your query. As you suspect, the most likely cause of your symptoms could be a slipped lumbar disc. For confirmation of diagnosis, an MRI of lumbar spine can be done. For pain relief, you may take pregabalin or gabapentin capsules. Physiotherapy is also useful. Wishing you good health, Dr Sudhir Kumar MD DM (Neurology) Senior Consultant Neurologist"
},
{
"id": 23706,
"tgt": "Suggest remedy to relieve chest pain due to atrial hypertrophy?",
"src": "Patient: hi I ave had chest pain on and off for the last 6mths, i am female 40yrs, smoker, crohns disease.Last mont stared to get worse with stress, the pain goes up to my shoulder through my back and down my left arm.Last week doctor done ecg and it said possible right atrial hypertrophy with poor R progression in right precordial leads with normal synus rythmn. At present i am experiencin bad chest pain after hot bath causing me to slouch over.What should i do? Doctor: Hi,Most probably your pain is caused by spine problems, if you have pain at the moment, take Ibuprofen 400 mg.About atrial hypertrophy; ECG is not sensitive for detecting atrial hypertrophy, it means that in majority of cases there is no atrial hypertrophy when on ECG we suspect it. To detect atrial hypertrophy one should undergo echocardiography.To rule out cardiac origin of your pain you should undergo stress test.Take careCome back if you have any further questions"
},
{
"id": 169665,
"tgt": "How can acne on the body which oozes be treated?",
"src": "Patient: My 4wk old baby has a lot of acne it was just her face but now it covers her scalp, entire neck and her chest it has started to move down onto her upper back. Its orange. And the ones on the back of her neck started to ooze I noticed it when I changed her sleeper which it had stained the neckline. Should I be worried or is this normal? Doctor: Hi Dear,Welcome to HCM.Understanding your concern. As per your query your baby have symptoms of acne on the body which oozes which occur due to blocked ducts of sebaceous glands . Dead skin and bacteria entrapped inside cause infection and the dark spots after acne are post inflammatory hyper pigmentation. Need not to worry. I would suggest you to apply vitamin C serum at night. You should start with use of topical antibacterial clindamycin gel as a spot treatment for acne during day time. Protect your skin from sun exposure. If condition persist even after a month the consult dermatologist . Doctor may recommend techniques like laser therapy and dermabrasion and prescribe antibiotics , retinoids and tretinoin for acne .Hope your concern has been resolved.Get Well Soon.Best Wishes,Dr. Harry Maheshwari"
},
{
"id": 164974,
"tgt": "Suggest treatment for small cluster on cheek in kids",
"src": "Patient: My 2 year old daughter has a few small red mps on her bottom lip on the right side and another small cluster on her left cheek? Could this be from chapped lips and skin her cheeks always get dry and cracked even though I put baby lotion on her cheeks several times a day Doctor: Hi, this can occur in multivitamin deficiency. It's called cheilitis. Start a multivitamin syrup for 2 month. Take care."
},
{
"id": 149483,
"tgt": "Nerve conduction study done, abnormal sounds heard. Glucose metabolism compromised, problem in thought process. Suggestion?",
"src": "Patient: I had a nerve conduction study. My Neurologist said the sounds he heard when he put the needles in my legs were \"abnormal\". Sounded like \"Baseball cards in bicycle spokes\" and other noises. I was told my glucose metabolism is compromised. I have a second EMG scheduled for my upper extremities. I have problems with thought processes at times. I am told the polyneuropathy is symmetrical. Does any of this sound familiar. Doctor: Hi,Thank you for posting your query.Your nerve conduction tests are suggestive of symmetric peripheral neuropathy. Symmetric means similar involvement of both sides (legs in your case). The most likely cause of neuropathy in your case is high blood sugars. Neuropathy refers to disease of nerves in the legs and hands. Common symptoms include tingling, numbness, weakness, pain, etc of legs and hands. Abnormal thought process is not connected to neuropathy.Treatment would include control of blood sugars, and use of medications such as pregabalin, if you have pain in the feet.I hope it helps. Please get back if you have any more queries.Best wishes,Dr Sudhir Kumar MD DM (Neurology)Senior Consultant Neurologist"
},
{
"id": 219380,
"tgt": "What precautions can be taken while travelling during pregnancy?",
"src": "Patient: Hi Doctor , My wife is in 6th week pregnancy, and we need to travel for family marriage function on next week. I had already book Flight ticket but now confused with lots of review, Need your advice, how safe she is in flight traveling and what all precaution she or we need to take care to have safe travelling /celebration.Currently she is having thyroid and taking thyronorm 75mg tablet. For Pregnancy she is taking All 9 \u2013D and Gestone 200 mg .Please help me to clear my confusion Thanks in advance Pankaj Kumar Doctor: HI, Thanks for the query. I would try to help you to my best. - In facxt as first 3 months fetus is a bit delicate & pregnancy is not yet setteled.. so whenever possible travel should be postponed till second three months ( 4th to 6th month ) of pregnancy.But if it's essential to travel, - Before planning for travel, your wife should be checked up thoroughly by her treating doctor & permit her for travel. She may prescribe few medicines to be taken during travel. Take them regularly BE careful about your routine medicines ( specially Thyronorm & Gestone ) . - try to have reclining seats/ use seat belts to prevent slightest of jerk.inquire about carrier before you start for the journey. Thanks."
},
{
"id": 225501,
"tgt": "Took levonelle after unprotected sex. Experienced nausea, irregular bleeding, abdominal pains. Side effects? Pregnancy chances?",
"src": "Patient: Hello,I lost my virginity the early hours of saturday morning and had sex without protection. I went to my local pharmacy and took the morning after pill 'Levonelle'I've experienced the side effects stated to me by the pill leaflet and the nurse; nausea(i have not actually been sick), irregular bleeding(only light), lower abdominal pains and tiredness.I took the pill approx 14 hours after having intercourse and i'm worried about the percentage rate of success. I have also noticed today that the veins on my breasts are very blue and this has raised concerns for me after researching it on Google. Can this be a normal side effect from the pill? And what is the likely hood of being pregnant after taking the pill 14 hours after.Please help me, I'm really worriedThankyou Doctor: hellothanks for your query.When the emergency contraceptive is taken within 24 hours, it is more than 95 percent effective in preventing a pregnancy, so chances of your conception are quite low.the bluish veins would not indicate pregnancy , as the symptoms would not appear so soon after intercourse.It is mostly due to the pill side effects.ensure that you have protected sex always.Take care."
},
{
"id": 180019,
"tgt": "Suggest treatment for fever and rash in a child",
"src": "Patient: Thank-you so much! We had our Grand daughter today. Both my Husband are in health care. Stumped as our children are grown but....our grand daughter Molly is turning one year in about a week. She had just gotten over a fever for a day or two & began developing a rash today. Looked like measles to us. Tried to keep her happy & calm and out of the light. Mom said She would have to look at the immunization records to see if Molly has had an MMR Seriously both parents are on top of everything to the pt of being Neurotic but both are in a Wedding\\ tomorrow and so much going on. Do you think Molly should see a Dr. right away? Wet diapers and nursing fine. Seems to be in a happier frame of mind. Help and many thanks, Tonya R Doctor: Thanks for expressing your concern on HealthCareMagic. Apart from measles there are a number of viral infections that cause fever with rashes (viral exanthem). They can also be caused by certain bacteria. I am not giving you a list of the probable conditions as they would only make you more confused. There is a definite characteristic of each of these infections and definitely your paediatrician would know what to look for. It is no use panicking. It could also be a not-so-worrysome simple condition wherein not much has to be feared. On the other hand, many of the conditions are contagious so upto the point of time that your paediatrician suggests, I would insist on keeping the child isolated from the others."
},
{
"id": 163372,
"tgt": "What causes severe right-sided tibia pain and easy bruising?",
"src": "Patient: My four year old has complained of severe pain located in his right tibia. He was first brought to his Pediatrician when he was two. Xrays showed nothing and again at age 3. Conclusion was growing pains. The pain will wake him from his sleep mostly, but will also occur randomly in the day just not as often as it does at night. Occurs 2-3 times a week for the past 2 1/2 years. Sometimes he will limp but is not an occurring or daily symptom. He will bruise easily. He plays rough, too. He has an history of headaches, 1 migraine, EEG was clear. Other symptoms related/unrelated = irritability, small appetite, frequent nose bleeds. Doctor: Hello,It seems like some clotting defect early platelets count is mandatory. Level of vitamin K vitamin factor nine and eight should be done. Protect children from falls, tell us reports, we will see it further. Also, tell us about the family history.Hope I have answered your query. Let me know if I can assist you further. Regards, Dr. Hina Javed"
},
{
"id": 168344,
"tgt": "What causes lumps on the face, hands and feet of a child?",
"src": "Patient: My 1 yr old developed random welts on her face, hands, arms and feet. The will randomly appear and are inconsistent as to how long they last. Some stay on her for 1 hour and some disappear with in 15 minutes. We do not have animals and she has not been exposed to anything different. What could these be? Thank You! Doctor: Hi...Thank you for consulting in Health Care magic.By what you quote it should be an urticarial or a simple skin allergy. You can use Hydroxyzine at 1-2mg/kg/dose every 6th to 8th hourly for 7-10 days. Most important thing to be remembered is that it has a propensity to recur (called as second crop) within 10-14 days. If this happens, you can start using the same medicine but I suggest you get the kid evaluated with your paediatrician.Hope my answer was helpful for you. I am happy to help any time. Further clarifications and consultations on Health care magic are welcome. If you do not have any clarifications, you can close the discussion and rate the answer. Wish your kid good health.Dr. Sumanth MBBS., DCH., DNB (Paed).,"
},
{
"id": 44867,
"tgt": "In what way can primary fertility be treated ?",
"src": "Patient: am 29 years old female with primary infertility mI no issues. My FSH level is 15 and prolactin is 40. arried for 8 years, but no children. My husband has Menses are regular, but unable to conceive through allopathic treatments like IUI, IVF etc. Is there a cure for my problem through ayurveda . Please suggest Doctor: hello welcome to health care magic its good option to go for alternative mode of treatments like ayurveda and homoeopathy as it is free from side effects. you can consult homeopath near by. goood luck"
},
{
"id": 150265,
"tgt": "Multiple sclerosis patient, taking Glatira with Zeptol. Feel dizzy after stopping Zeptol. What to do ?",
"src": "Patient: Hi i am a Multiple sclerosis patient and i have been taking Glatira along with zeptol Cr 300 for close to an year ...and recently about 2 months back i have left zeptol, but now i feel very dizzi, so my doctor asked me to start of with Zeptol cr 200 but i have taking 300 mg ....is it advisable like that ...as i feel very very dizzi, and i feel i may fall down any time....so can you please suggest me Doctor: Hi,Thank you for posting your query.Zeptol CR (carbamazepine) is a drug used for treating epilepsy (fits) and nerve pain. So, if you have any of these symptoms, zeptol cr may be continued. If not, there is no need to continue this medicine. Dizziness is a known side effect of zeptol cr.Please get back if you require any additional information.Best wishes,Dr Sudhir Kumar MD (Internal Medicine), DM (Neurology)Senior Consultant NeurologistApollo Hospitals, Hyderabad,My personal URL on this website: http://bit.ly/Dr-Sudhir-kumar My email: drsudhirkumar@yahoo.com"
},
{
"id": 98134,
"tgt": "Have dark spots. Why do I get them? Will homeopathy help?",
"src": "Patient: I m roop have problem of dark spots since 4 years due to daiting .. but from 3 years m conscious for my health .. have 11 Hb , thirod is ok , I did lots of treatment with help of dermotologist but results are for short timer , used many creams like kojik acid , melaline ,elocon , depiwhite , dermiline , pendrem, and so on .. but most effective was elocon and pendrem which are so strong .. but when I dropped these tubes , pigments again come . I m in so tension .. now I leaved all these treatment and taking homeopathy which is for skin deases ,, but no result ... please give me guidance what I ve to do .. I m 30 year and belong to Punjabi culture .. Doctor: hi roop,u had taken many diffrent pathy med but u could not tell me about ur discolouration of spot like colour shape ,on cheek/forehead/nose /where? have u any menses/liver problem, sure i can help u.pls u give me these ans/if u can send me ur photo of pigmentation by e mail. than very good 90% sure i can help u."
},
{
"id": 137633,
"tgt": "What causes painful nodule in clavicle?",
"src": "Patient: Painful nodule above my clavicle on the left side near my throat. It s also very tender under the clavicle bone in the chest area, sore on both sides under my armpits and both shoulders very fatigued. I am scheduled for a massage tomorrow, but after reading all of this I may need to see my primary care Physician as well. My shoulders have always been where I carry stress had fatigue, but this is the first time it s been accompanied by the clavicle and pain and chest muscle pain. Any insight would be greatly appreciated. Doctor: Hi,Thanks for your query.Well, without a clinical evaluation it would be difficult to determine the cause of the lump. Possibilities that may need to be considered include enlarged lymph node, lipomas, cysts, growths from underlying connective tissue etc.I would suggest getting this evaluated by an orthopedician for an accurate diagnosis and appropriate management. You may get the X-ray of the area affected and FNAC (fine needle aspiration cytology) of lump under his/her guidance.I do hope that you have found something helpful and I will be glad to answer any further query.Take care"
},
{
"id": 102105,
"tgt": "Should i use epipen for allergy on arm?",
"src": "Patient: My right arm from fingers to upper arm is swollen and I don't know what bit or stung me. I have a epipen that I carry because I have an allergy to bee stings. The swelling is only on the right arm, should I use my epipen in this instance? Doctor: if you are very unconfortable you can use it as it will be helpful in treating the present swelling and will prevent to increase swelling in other parts"
},
{
"id": 213771,
"tgt": "How can I cure my stress which leads to bad stomach pains and diarrhoea ?",
"src": "Patient: Hi my name is Jessie I am 21 years old I am adopted and i recently found my birth mother and every time i talk to her i get so stressed out from all the drama she tells me that i get so stressed out that i have really bad stomach pains and direaha Doctor: Hi, Welcome to Healthcaremagic It shows the symptoms of stress you are undergoing. It is better to share your thoughts with a close friend to ventilate your stress and you may require to see a psychiatrist who can offer a professional help in this matter. Take care."
},
{
"id": 18675,
"tgt": "Is going on upside down rides safe while having SVT?",
"src": "Patient: I am 19 years old. I was diagnosed with S.V.T when i was around 8 years old. I have not had an episode in many years. I was on beta blocker till around 11/12 years old. My episodes used to be triggers by bending over/upside down [tying my shoe,pick something up] or too much strain in gym class. I am going to an amusement park and want to know if i would be ok to go on rides that go upside down. i have been on big fast rides before but never upside down..could going upside down trigger an attack after all these years Doctor: Hello and Welcome to \u2018Ask A Doctor\u2019 service. I have reviewed your query and here is my advice. You need to keep away from rigorous sports. too much of anxiety can trigger similar episodes . All these events can increase the adrenaline surge. Hope I have answered your query. Let me know if I can assist you further."
},
{
"id": 113876,
"tgt": "What causes back pain and nausea after long walk ?",
"src": "Patient: Im 21 female. my right side in my back hurts if I walk for a long time I get lightheaded my breath gets heavier my hands get clamy I also get nausea .What could it be. I dont take any medication for anything. when I was 16 doctor said I was anemic . Doctor: hi u should check ur HB and VITAMIN B12 level both may be lower in ur case it look like no serious problem only thing u need to eat healthy and be relax dont stress more drink plenty of water better to consult doctor"
},
{
"id": 101448,
"tgt": "What are the chances of developing a food allergy as an adult?",
"src": "Patient: I was on augmentin and became very itchy, no hives just very itchy. I went to the ER and they gave me IV Benadryl. Ever since then I have been weary about having an allergic reaction because that same day that I became itchy I also had some shrimp sushi. I have never had any food allergies and 25, but read that a food allergy can just appear at any age. What are the chances of developing a food allergy as an adult? Doctor: HI, thanks for using healthcare magicIt is understandable to be concerned. It is possible to discover allergies as an adult.If you want to determine what you may be allergic to, you should consider allergy testing. This can be in the form of a blood test or skin test. It is possible to test for a lot of different allergens at the same time.Your doctor will be able to tell you where you can have the test done.I hope this helps"
},
{
"id": 80447,
"tgt": "What is the treatment for recurring viral lung infection and low grade fever?",
"src": "Patient: hi have been twice to my local Dr. I have been ill since February with viral illness also he called it a viral lung infection. Now I am experiencing sporadic low grade fever that is accompanied by upper to middle aches and soreness in my back. Whenever I experience the back symptoms a fever follows. I am experiencing the sweats, and also am extremely tired and sleep at least 10-11 hours a day. Is there any recommendation you can give me to get rid of this once and for all. Dr. did prescribe z-pac not much of a help, but did somewhat elevate symptoms. Thank you for any advice. Debra Doctor: Hello dear, thanks for your question on HCM. I can understand your situation and problem. In my opinion, we should first rule of bronchitis In your case because recurrent viral infection can cause post infectious bronchitis. So better to consult pulmonologist and get done1. Clinical examination of respiratory system. 2. PFT ( pulmonary function test ).PFT will also tell you about severity of the disease. And treatment of bronchitis is based on severity only. You may need inhaled bronchodilators and antihistamine drug. Also get done adult respiratory vaccination with influenza vaccine to prevent recurrent viral infection."
},
{
"id": 210513,
"tgt": "Suffering from anxiety and depression why do I feel difficult to concentrate and remember?",
"src": "Patient: I have long suffered with anxiety and depression. I have had the same job for 21 years and have recently been promoted. My new job is very challenging and I love the work I do. I am finding that I feel like a kid again, though. I have a very hard time staying focused, or staying on task. My new job comes with a lot of training. I usually fade away after about 10 minutes of face to face training. I read through the online training that I am required to do and end up having to read it several times because I remember nothing I have read. Could I have ADD Doctor: HiThanks for using healthcare magicDue to underline depression and anxiety, you are not able to concentrate, that why, you are feeling problem in attention. This is not attention deficit disorder. In that case, you should get proper treatment of depression, so that would to refocus your attention. Consult a psychiatrist for better treatment.Thanks"
},
{
"id": 166615,
"tgt": "What causes bedwetting in a child?",
"src": "Patient: I have a 7 year old boy who last night pee the bed and now since 9 pm has not stop peeing he is not coplaining about pain or discomfort.. diabetes runs in my husband side of the family and when my son goes pee it is not a whole alot it is more like dizzle Doctor: Dear parent , I understand your concerns but this might be due to diabetes since it runs in the family . diabetes symptoms includes also increased thirst and hunger. you should have a blood test for the sugar level . this symptoms could also be due to a urinary tract infection which needs a urine analysis to be confirmed"
},
{
"id": 126779,
"tgt": "What causes numbness in the neck?",
"src": "Patient: My son is sick....25 years old........vomiting all night........complaining of painful visible swelling back of neck on left side/complaining of numbness in portion of right hand and small finger/ no fever/slight headache What could cause swelling and numbness? Doctor: Hi, It might be due to conditions affecting cervical spine. Consult an orthopaedician and get evaluated. Hope I have answered your query. Let me know if I can assist you further. Regards, Dr Shinas Hussain, General & Family Physician"
},
{
"id": 67777,
"tgt": "What causes a lump in the middle of the neck?",
"src": "Patient: A few days ago I shouted or screamed the loudest I ve ever done before & when eating later that day I had a dull pain under my jawline on the left side of my neck/throat. For the next 2 days bringing me to today I had the dull pain but only when ever I ate other than that it was non existent. Then after getting home from shopping earlier today I generally brushed my hand down my neck and felt this massive lump about the size of a gold ball. When I looked in the mirror it was clearly visible. So I shouted to my dad outside who was leaving for work, he booked an appointment at the doctors for 5pm by that time the lump had shrunk to the point where it wasn t so visible but you could definitely feel it. Anyway the doctor said I most likely cracked/tore the gland when screaming this then caused bacteria to enter when eating causing an infection. So he prescribed me a regular dosage of the antibiotics amoxicillin of 1 dose of 250mg 3 times a day, and tomorrow I have to go to the hospital to have a blood sample taken. Personally it feels like the lump ness has moved ever so slightly to the middle of my chin. Is this a serious problem to be worried about even after getting the medical advice. I am currently aged 15 and male & never had anything like this happen, and I am not too worried per say of what s happening but a little concerned on what could happen, could someone shed me some light on what is going g o? Doctor: yes this condition will need follow up with your doctor and this soft tissue injury should subside with medical treatment. It is worthwhile waiting to give it a chance to heal this way. On your follow up it will be more clear if anything in addition will have to be done or not( like some investigation or surgical intervention). On the whole you should be fine and follow examination should be able to rule out the need for any additional treatment"
},
{
"id": 164472,
"tgt": "Suggest remedy for infrequent, painful, dry stools with extreme thirst in a child",
"src": "Patient: My son is three. His stools are infrequent, painful and always large and dry like clay. He is thirsty all the time. He complains that his tummy hurts and then request a drink. I am more and more concerned, he is growing fine and in good spirits, but he has a weak appetite. Doctor: Hi.... I understand your concern. But if he is also underweight or failing to thrive, then I suggest that you get your kid evaluated for cystic fibrosis because the other symptoms which you are quoting are suggestive of cystic fibrosis.Regards - Dr. Sumanth"
},
{
"id": 36163,
"tgt": "What does 's.typhi antigen O-1 in 160 and antigen H-1 in 160' suggest?",
"src": "Patient: I Have Been Diagosed with Typhoid.I wanna know whether it is the starting stage of it. Here is my WIDAL TEST REPORT: S.Typhi antigen O : 1 in 160 dilutions S. Typhi Antigen H : 1 in 160 dilutions S. Paratyphi A : S. paratyphi B : Doctor: Hello,This does raise the suspicion of typhoid but is not diagnostic for typhoid fever.You must get blood culture and typhidot test done to confirm the typhoid.Thanks"
},
{
"id": 136670,
"tgt": "What causes stiff neck and swollen lips?",
"src": "Patient: Hi my name is Carolyn I am 51 years old I noticed a love in my right area by the elbow be Mrs. I was 20 I was causing problems are often well just recently today s ago I woke up with severe pain no is swollen it was red and prior to their two days before that my notice my right upper lip was swollen went for no apparent reason this and left side neck have been stiffen up as well. Thank you Doctor: Hello I have studied your case.This swelling can be elbow olecranon bursitis.If bursa is painful surgical excision is treatment.If you want temporary relief intrabursal injection can be given which will give long term pain relief.There may be recurrence of pain after injection.Till time you can do physiotherapy like Ultrasonography and TENS.MRI cervical spine will help you further.Hope this answers your query. If you have additional questions or follow up queries then please do not hesitate in writing to us. I will be happy to answer your queries. Wishing you good health.Take care."
},
{
"id": 43711,
"tgt": "Trying to conceive, partner suffering with astenozoospermia, no improvement with medicines. Best treatment?",
"src": "Patient: Hello doctors, I am from bangalore I am 29 years old and my husband is 32 years old we have been married for 7+ years now and trying for baby from past 3-4 yrs. My husband is diagnosed with astenozoospermia problem, His Analysis shows that sperm count=65 mill/ml, a+b=0.7%, c=1.5% and d=97 % and in his resent analysis shows that there are 1-3 motale sperms. Back in 2006 he had kidney stones while investigating further we came to know that he is having single kidney by birth and his seminal vesicle is blocked. We have met 2 doctors till now one suggeted to go for ICSI this doctor had put him under medication like Natoz,HIQ + and profert-M there were no much changes in the analysis and another has suggested for TESE with ICSI. Kindly suggest which procedure is best for us. Doctor: Hello, Thanks for the query, Asthenospermia refers to motility defects in sperm. This could be due to problem in sperm production., malnutrition, testicular injury, overheating,genetic, smoking. In your husband's case best treatment option would be: A transrectal ultrasound (TRUS) in order to confirm the presence or absence of ejaculatory duct obstruction, Presence of a cyst or calcification at the ejaculatory duct, The size of the seminal vesicles will be measured to determine whether there is any significant obstruction. Following which last resort would be ICSI, TESA, PESA. Wish you good luck. regards, Dr Nilofer"
},
{
"id": 78623,
"tgt": "Suggest treatment for chronic cough , asthma , dizziness and headache",
"src": "Patient: hello doctor for the last 6 days ive been dealing with a coughing problem at first i thought it was just an asthma attack .. which i usually deal with a inhaler or if it get worse a nebulizer ... ive been coughing rusty green sputum, feeling tired and weak, dizzy, headache, and pain in my chest area .... i wanted to check if it was lung cancer since my family has a history with cancer ... so can lung cancer be detected by blood test? if so what tyoe of blood test is needed Doctor: Thanks for your question on Health Care Magic. I can understand your situation and problem. By your history and description, possibility of lung Infection (pneumonia) is more. Lung infection is common in asthma patients due to poor lung defense. And cough, greenish phlegm, weakness, dizziness etc are common symptoms of pneumonia. So better to get done chest x ray first. Chest x ray will also be helpful in screening of lung cancer. No blood tests are available for diagnosis of lung cancer. Non specific tumour markers like CEA, LDH, CA125 etc are raised in lung cancer. But these are non specific and hence diagnosis can not be made on their values. Screening of lung cancer is done by chest x ray. And fir confirmation, CT thorax and biopsy are required. But at present, you need chest x ray to rule out pneumonia. Hope I have solved your query.Wish you good health. Thanks."
},
{
"id": 42428,
"tgt": "What should be the normal size of follicle on 15th day of follicular study?",
"src": "Patient: Hello doctor, I am going through the follicular study,this is the 3rd time that am doing it ,,in 1st FS study,i did not get the positive result,,,and at d 2nd study Dr.gave me 50mg tablet to increase the follicules but unfortunately the result was not good.this is d 3rd time of study my Dr.gave me 100mg tablet(godova) ,today is my 15th day of menstrual cycle and my FS shows left ovary 16,8,7 mms and right Ovary 9,8,7 ,,,I have thyroid taking 100mg thyronorm,reports are nor Que.what should be the normal size of follicle on 15th day,my treatment is in right track ,and is there any chanches of getting pregnant in this cycle? Doctor: Hi,Welcome to HealthcareMagic .Your treatment is going on right track. You need to continue . On 15th day it should be more then 15mm one dominant follicle. You need to follow the growth . When it reaches more then 19mm you may take HCG injection to rupture it. Ya chances are there you may become pregnant this time.Hope I have been helpful .RegardsDr.Deepika Patil"
},
{
"id": 7819,
"tgt": "What are the home-remedies for treatment of acne ?",
"src": "Patient: How do i get rid of acne? are there any simple ways that i can atleast reduce acne in my house without going to see a doc? Doctor: dear p't welcome to\"HealthcareMagic\"forum why not.you reduse your acne in house. wash your face gently worm water 2-3 times without shop,face wash, after wash use papaya pulp with rose water in your face. for complete result.call 9827079438 thanks..............."
},
{
"id": 212914,
"tgt": "Suffer from anxiety and depression, taking Valium. Is this PTSD or bipolar ?",
"src": "Patient: PTSD vs bipolar. I feel like I have PTSD. After years of sexual abuse by my father. Doctors seem too clump everything as bipolar.I m not bipolar I take valium twice a day 10mg. I usually take them at night to sleep I suffer from anxiety and depression ,although I do disaster relief volunteering.I go to the gym,I quit smoking I take care of my grandson. Depression comes and goes Anxiety stays Doctor: Dear Rachelwesson, Hi! I totally understand your concern for your health and its obvious that you are suffering a lot. To comment, whether it is a bipolar disorder or not, we need to have a detailed history of you and the same is lacking in the information given by you. We agree that there was a significant stress in your childhood, but the same can predispose you to depression, PTSD as well as anxiety disorder. Although you are taking \"Valium\" its more of a cosmetic relief and you may land up with benzodiazepine dependence and hence it will be always wise to start you on some form of SSRi medications. The other best option will be to consider psychotherapy and cognitive behavioural therapy. basically you need to see a psychiatrist for a detailed history and evaluation , who would like to run a few psychological tests on you, before he starts you on either psychotherapy or pharmacotherapy or both. I hope this answers your concern."
},
{
"id": 176291,
"tgt": "What causes pain and numbness in finger after an injury?",
"src": "Patient: My daughter came down on her hand from a fall off her bed. She said she rolled over and off the bed. She has been complaining with pain ever since. She has been taken ibuprofen and had it in an ace bandage. She has been unable to use it. Today I took her to Dr. And for a xray. The xray was negative for a break. However tonite she is still hurting and now says her hand and all fingers but the pinky is numb and she can t feel them. Her pinky expecially hurts and her hand is swollen.. Doctor: Hello,Thankyou for your quiry.From the history it seems that she might have injured her radial nerve.Which supplies medial side of hand.But if she is having numbness in lateral side then it could be ulnar nerve.As nerves tend to be deep below the muscles and pass near to bones,so when ever there is strech or tendinous pull or fracture ,the nerves can be injured.My suggestion would be ,she may need physiotherapist and needs checked by a orthopaedic doctor for further discussion.Hopefully my answer will be helpfull.Regards,Dr.Maheshwari"
},
{
"id": 72245,
"tgt": "What causes a painful feeling in my chest when I try to breathe in deep?",
"src": "Patient: It's happening right now, but I have this really painful feeling in my chest when i try to breathe in deep well, it started when i was just breathing regularly and now I've tried to breathe in deep to see if it has gone away and it hasn't, this is not the first time, it's happened before right now its the right side, but sometimes its both sides. and I don't know what it is, but I haven't worried about it, because it usually goes away. Doctor: Thanks for your question on Healthcare Magic.I can understand your concern. Chest pain on deep breathing is commonly seen with pleurisy (inflammation of pleura).And common causes for pleurisy are lung infection, pleural effusion, bronchitis etc.So better to consult pulmonologist and get done clinical examination of respiratory system, chest x ray and PFT (Pulmonary Function Test).If all these are normal then no need to worry for lung diseases. Sometimes simple musculoskeletal pain can also cause similar symptoms. So avoid movements causing pain. Avoid heavyweight lifting and strenuous exercise. Apply warm water pad on affected areas of chest. Take simple painkiller like paracetamol or ibuprofen. Don't worry, you will be alright with all these. Hope I have solved your query. I will be happy to help you further. Wish you good health. Thanks."
},
{
"id": 139177,
"tgt": "Suggest treatment for severe knee pain",
"src": "Patient: yes i have had knee pain for about 2 months,but it does not hurt when im playing golf,but afterwards its pretty tuff,also i can not bend down into a catcher stance and can not put any weight on it...i did the injections they were no help..should i get a mri,or just tuff it out..thanks david hall Doctor: Dear David hall, I understood your concerns... you may have swelling in knee also. Go for ultra sound and IFT therapy for 15days. Do simple muscles twitch gently. You gradually improve."
},
{
"id": 140938,
"tgt": "How can multiple age indeterminate compression deformities within the lower lumbar spine be treated?",
"src": "Patient: I have a question about a diagnosis I recieved. Its was a examination of my Lumbar spine, I m having pain in my lower back. I Dont understand the doctor s terms in the summary. This is what the letter read. There are multiple age indeterminate compression deformitites within the lower lumbar spine and within L1 and L2. There is generalized osteopenia. There is mild degenerative disc disease with disc space narrowing, most pronounced at L1-2 and L3-4. There is grade 1 anterolisthesis at L4-5 and L5-S1. At L4-5, spondylolisthesis measures 6mm in neutral, 11mm in flexion and 4mm in extension. At L5-S1, there is no dynamic instability. There is trace retrolisthesis at L1-2, L2-3, and L3-4. Doctor i have no idea what this means, can u please explain this diagnosis to me in simple terms? Doctor: Hi, Findings suggest age-related degenerative spine disease. You may need physiotherapy for strengthening back muscles. Hope I have answered your query. Let me know if I can assist you further. Regards, Dr. Indu Kumar, Radiologist"
},
{
"id": 207219,
"tgt": "Suggest psychiatric treatment for constant doubt about people",
"src": "Patient: SIR MY MOTHER IS HAVING A PROBLEM OF DOUBTING PEOPLE SHE FEELS THAT EVERYONE IS TELLING SOMETHING VERY BAD ABOUT HER AND THAT PEOPLE ARE ALWAY BEHIIND HER TRYING TO TEASE HER THIS HAS STARTED LONG TIME BACK BUT NOW IT HAS BECOME VERY SEVERE HERE REPORTS OF BRAIN SCAN ARE ALL SHOWING NORMAL COULD YOU SUGGEST ANY GOOD PYCHIATRIST IN THRISSUR TOWN Doctor: DearWe understand your concernsI went through your details. I suggest you not to worry much. From the information given by you, I suspect the case as paranoid schizophrenia. You should visit a psychiatrist for proper diagnosis and proper treatment. Early diagnosis and treatment should cure her. Jubilee Mission hospital and Ansar Hospital, Perumpilavu are good for psychiatric treatments. Please consult them.If you require more of my help in this aspect, Please post a direct question to me in this URL. http://goo.gl/aYW2pR. Make sure that you include every minute details possible. I shall prescribe the needed psychotherapy techniques.Hope this answers your query. Available for further clarifications.Good luck."
},
{
"id": 27413,
"tgt": "What causes cardiac arrest?",
"src": "Patient: 4 weeks ago during a minor heart operation to try to repair a slow heart beat I went into cardiac arrest. I was clinically dead for 25 minutes and 9 attempts were made with the electric paddles to try to resuscitate me. I was put into an induced coma for a few days in ICU and in hospital for 2 weeks but now 4 weeks later I am still in a lot of pain and find it difficult to sleep. Any movement give me bad chest and back pains. Is this normal ???? Doctor: Hi, u were given cardio pulmonary resuscitation. This may cause many hairline ( minor fracture) of the ribs which may cause the pain with change in position , this is normal and will resolve by 6 weeks also electric shock may cause soreness which way be persieved as pain. However I don't about your inherent heart condition, the reason for which the surgery was being done. So a more detailed history with investigation s done for the same will help me help you better in understanding the cause and the reason for present symptoms . Regards"
},
{
"id": 174457,
"tgt": "Is Allopathic medicine intake safe for a kid?",
"src": "Patient: my child is 3 years old and is having cough from last 4 months. Doctor advised me to get some tests done. They found E.S.R level raised upto 40 in reports. So he told me to give my baby montecope syrup for 6 months daily. (5ml daily) Is it safe to give an allopathic medicine to a kid for so long? After this, will other medicines work on him? Doctor: Hi your doctor prescribed montecope based on examiination and diagnosis he made- Allergic cough and given for 12 weeks.Yeah, you can continue it for 3 months and no harm in it.Hope it answers your query."
},
{
"id": 167788,
"tgt": "What cause blood in stools in babies?",
"src": "Patient: I have a 6 week old daughter and her pediatrician tested her positive for microscopic blood in her stools. Her growth and weight are fine and the pediatrician said her stool looks fine as well. She wouldn t have even tested it had I not mentioned a green tint in her stool which at the time of mentioned was corrected after I did a little research and changed my breastfeeding methods so that my baby gets more hindmilk. Her pediatrician thinks it might be a food allergy but after a week of restricting my diet there is still microscopic blood in her stools. Could this be something else or nothing to worry about at all since she otherwise seems very healthy? I should mention she also has what the pediatrician thinks is thrush and a diaper rash but the pediatrician doesn t think the blood is coming from the diaper rash. Doctor: The most common cause of bloody stool in the newborn is cow's milk protein allergy , it's very uncommon to have it in exclusive breast fed infant , restricting the diet of you by removing diary products can help with this but not all the time , unless there is frank blood , poor weight gain or anemia you should not worry about it .The less common causes are : -Anal fissure from constipation , which is common after the 3rd week of life .-Vitamin k diffecency which is common in exclusive breast fed babies unless given a shot of vitamin k shortly after birth which is standard .-less common causes like Michele's diverticulum and necrotizing enerocolitis are very unlikely in your babys case Remember all ways to check your nipple for cracks , because if the baby socks blood , it will appear in here stool .I hope this helps"
},
{
"id": 196109,
"tgt": "What does my sperm count in the test result indicate?",
"src": "Patient: Hello Sir, I am 31 years old married guy and below are the particulars of my sperm test report. Please have a look into this and interpret the results to me.. should i consult a doctor for a baby. Volume: 2 ml colour: Pearly white LIQUEFACTION-NORMAL: 25Min Reaction: Alkaline Sperm Count: 70 million/ml Active Motile: 40% Sluggish: 30% Non Motile: 30% Epithelial Cells: nill Pus Cells: 6-8/HPF Morphology Normal: 60% Abnormal: 40% I am waiting for a quick reply. Thanks in advance for your support Doctor: hi.welcome to healthcare magic.i analysed your reports.my opinion is that,1)your semen analysis report is normal except that of increased pus cells.normal pus cells in semen analysis should be upto 2.but you have 8 pus cells.2) increased pus cells in semen analysis test suggests some infection in your genital tract.3)kindly consult your treating doctor.he will advise you a antibiotic course.thank you.hope my answer will help you."
},
{
"id": 76206,
"tgt": "Suggest medication for cough with mucus",
"src": "Patient: Good mornging Doctor!! My mother is suffering from cough with lot of mucus, which is not coming out. She is suffering from Diabetes and under medication. She is about 59 years old. and her weight is about 65 kg. Kindly suggest a good mucolytic agent. Doctor: Hi thanks for contacting HCM ..According to history your mother might have mild bronchitis infection of respiratory tract ....It might be allergic or viral infection...Sputum examination and CBC will be helpful for it differentiation...Take antihistamininc drug like benadryl ...If cough troublesome mucolytic agent can be given like ambroxol cough syrup or tablet .....Drink hot water that help in cough breakage ...Excess fried food restricted for few days as they can aggravate phlegm...Steam inhalation for two to three times a day ....If still no benefit consult physician for auscultation....Take care"
},
{
"id": 163249,
"tgt": "What could bloody discharge from nipples in boys suggest?",
"src": "Patient: my son is thirteen and has been having a discharge from his left nipple which now looks bloody. He is due for surgery for a pilonidal sinus soon. Are these conditions related in any way - maybe by puberty? More importantly, does he need to see our doctore about nipple discharge. Thank you Doctor: Hello,Bloody discharge from the nipple even in an adolescent boy is not to be ignored. There could be an abscess lingering beneath the nipple or it could be hormonal changes.Whatever it is, I suggest that you get a consultation from your pediatrician regarding this.Hope I have answered your query. Let me know if I can assist you further.Regards,Dr. Sumanth Amperayani"
},
{
"id": 188318,
"tgt": "Done tooth extraction. Prescribed megamox, metronidazole. Taking losartan and metformin. Dark green stools",
"src": "Patient: Dear Doctor,I had a tooth extracted some 4 days ago and was given megamox 1g (taken every 12 hours) together with metronidazole 200mg (taken 3 times daily) both after meals. It should be taken for one week. I mentioned to him I'm taking daily losartan 50mg and metformin 500mg. I noticed dark green stools. My stools are soft but not loose. I went to the dentist and told him about it but he said it's because of the antibiotics. I didn't feel any weakness, pain - just normal. Can you give me a piece of your opinion sir? Thank you very much. Gabby Doctor: Hello and welcome.yes your dentist has suggested you correct reason for having loose stools and also change in colour.once your course of antibiotic is completed you will be fine.Any further persistence of symptoms should definitely worry you else do not worry.Thanks.Take care."
},
{
"id": 68588,
"tgt": "Could the lump on butt cheek be due to cellulitis?",
"src": "Patient: I have a very large hard and painful lump on my right butt cheek, I've seen a doctor at an urgent care facility and she diagnosed it as cellulitis, but it hasn't formed a head yet, I'm taking my antibiotics but it doesn't seem to be doing anything. Are there any other diagnosis that this could be related too? Doctor: Hi,The infection may not develop into an abscess with a head. I suggest you continue to take the full course of antibiotics. If the infection has not cleared up then this may indicate that the infection is causing a deep abscess. This would possibly need surgical drainage. Regards,Dr K A Pottinger"
},
{
"id": 91390,
"tgt": "What causes pain around stomach if not kidney stone or hernia?",
"src": "Patient: I have had a pain in my right side(to u) of my stomach! It started just hurting when i went to te bathroom but then it would happen randomly! It hurts when u push i a specific spot! It's not a kidney stone or hernia! Had it checked out, any ideas what it could be Doctor: HI. This can be due to swelling of the large bowel - colon. How are the stools and urine ?Any other associated symptoms you have ?"
},
{
"id": 181019,
"tgt": "Why does my mouth fall open when I sleep?",
"src": "Patient: When I sleep my jaw falls open. I never used to happen, only about 3 weeks ago it started. It causes me to wake up in the night to close my mouth. It doesn't appear that I am breathing out of my mouth when it opens. What can I do about it? Is it because of weak muscles? Can this be fixed? Doctor: Hello. Welcome to Healthcaremagic.com, There are many reasons that during sleep mouth opens. 1) There may be the blockage in nasal passage because of recent cold or sinusitis. 2) Second thing is obesity, if you are little over weighted. 3) Improper sleep, upside or downside facing during sleep. 4) Maloculusion, upper and lower jaws does not fit properly. You must check which one is there in your case and you will get rid of that. Thanks for the query...... Regards........"
},
{
"id": 169379,
"tgt": "Which medical speciality deals best with pain in calves/knees in a child?",
"src": "Patient: My 5 year old son has been complaining about pain in his calves behind his knees for a couple weeks. He says it gets worse at night and he limps during the day. I m concerned that there s something wrong. He s had x-rays but they show nothing is wrong. Who should I take him to? Doctor: Kids will feel \"growing pains\" and it is very common. The legs are the biggest complaint from children and it does occur during the night in most cases. The limping during the day is what concerns me. To answer your question, I would recommend that your son see an orthopedic doctor who specializes in bones and joints.Good luck!!"
},
{
"id": 200902,
"tgt": "How to stop small cracks and cuts from coming back on foreskin?",
"src": "Patient: How do I stop the small cracks/cuts coming back on my inner foreskin? There doesn t seem to be a pattern as to when it occours it terms of having sex or masturbating or neither. I first discovered red and soreness when I was relatively young. The cuts actually leave a white scab kind of thing behind when they heal which isn t very pleasant. Do you have any advice on how I can overcome this problem? Doctor: Thanks for asking healthcaremagic forumIn short: CLEANLINESS CAN PREVENT IT WITH LUBRICATIONExplanation: Scab/white flakes over glans are due to dried semen/smegma, so in order to prevent allergic reaction and inflammation over there, keep your glans clean by washing it with water every day during bath. Dryness can also cause this, so keep the area moist and well lubricated(Note: please use safe lubricants which are tested)"
},
{
"id": 211791,
"tgt": "Unable to find a pain doctor, taking opiates for long. Why am I refused to be prescribed pain medication ?",
"src": "Patient: I have been seeing a pain doctor for almost 2 years now,and i had to move to another area.Now i am trying to find another pain doctor and I am having problems finding someone to give me medication similar to what i have been on.I am out of medication now and every doctor i see does not want to give me anything.What am i suppose to do now? Cant i become ill without these opiates that i have been taking for so long?And why would a pain doctor refuse to give you pain medication?I have heard from some people that i may be red flagged and would like to know what that means and how i could find out.I have now seen three doctors so far and still cant find one for me.HELP Doctor: Hello,You have been taking opioid medicine for so long that it is very likely that now you are dependent on opioid, this is the reason the doctors are hesitant to prescribe you the opioid. It is advisable you get treated for opioid dependence by a qualified psychiatrist. Methadone is an option and you need to take it supervised. Without opioid you shall get withdrawal (cramps, vomiting, diarrhea, lacrimation, running nose, etc.) and require indoor management.For you pain the reason has to be found out and treated subsequently but not with opioid. Other options are pregabalin, amitryptiline, optopril, and others. Hope it is helpful."
},
{
"id": 173176,
"tgt": "Suggest treatment for fever and widal test showing 1;40 and 1;80",
"src": "Patient: dear sir my son age 12 yrs,suffering fever from 18th.april till the date.his temp.becomes 102 degee fehrenhite at every 10 to 11 hrs.temp.cured with paracetamol tab.his widal test results 1;40 and 1;80.pl suggest diatery plan as well as treatment.thanks Doctor: many thanks.widals test is quite old test and not done in modern hospital.we are using blood culture test or IGg or IG m. if positive for typhoid or salmonella then needs intravenous treatment of antibiotic for 7 to 10 days according to culture sensitivity.then needs eradication of it with cap.amoxicillin 500mg for 3 weeks.Food advice is soft and non spicy better more liquids should given during illness.take more liquids and fruits."
},
{
"id": 109182,
"tgt": "Suggest remedy for back pain",
"src": "Patient: hi went to my doc yesterday with back pain ,he gave me 2 injections,at was at 3,oclock ,but as the day went i started to feel very sick in my stomack ,also as if i was very drunk ,i couldnt sleep started to feel as if my body couldnt move and nervouse,papations chest pain ,im still feeling sick today ,i will never get a injection ,with out asking what it is ,im a diabetec type 2 ,also have a hitus hernia .im 52 thank you Doctor: HIWell come to HCMWith the history of diabetic some time analgesic effects comes very late or some time this may not, reason for this is unclear, better to bear the pain, this is the best way, as go on bearing this after some time you would feel much better, more over analgesic is not good for health, second you can also try physiotherapy, that would be even much better than the pain killing medicines, take care, have a nice day."
},
{
"id": 85474,
"tgt": "Does Evion 400 help to cure psoriasis?",
"src": "Patient: i am 36 yrs old unmarried female pts of hypothyroid and having wt 85 kg. imy hairs was to thin and head base is clearly visible from some place. I am suffering from psoriasis and preseanty it is sever in legd base . I started taking Evion 400 once daily , is it help me to come out these problems Doctor: Hello,Evion is a supplement that contains vitamin E and vitamin supplements may bring some improvement in patients with psoriasis but are not found effective in treating it. It is important that you talk to your doctor and have medications that are approved for the treatment of psoriasis. Also, if you are not having any treatment for your thyroid function, you will need to take treatment for this as well. Evion will not treat or cure any of these conditions.I hope this answers your query.I remain at your disposal for further medical assistance.Regards,Dr. Antoneta ZotajGeneral and Family Physician"
},
{
"id": 192896,
"tgt": "What can be done to stop habit of masturbation?",
"src": "Patient: Hi, may I answer your health queries right now ? Please type your query here..doc, how do i stop my mind to shake my penis .is this dangerous for health or in making girl pragnent protein milk in my penis start leaking out when i touch my penis what should i do .plz help me Doctor: Hi, Masturbation for thrice a week is normal and good to health. To avoid over masturbation, regular exercise, physical games and yoga can help you. Hope I have answered your query. Let me know if I can assist you further. Regards, Dr. S. R. Raveendran, Sexologist"
},
{
"id": 11404,
"tgt": "Suggest remedy to stop hair fall",
"src": "Patient: hello everyone i am having some rapid hair loss just after coming to new home i suddenly feel hairs less volume and day by day they r going lesses i am worried im 25 n half male how to control this and what kind of baldness it is? should i shampo daily or oiling? Doctor: HiThank you for asking HCMI have gone through your query.You should get your hemoglobin level and TSH which will rule out anemia and thyroid disorder respectively which can be a cause of it.If you are getting hair fall on frontal side it is most likely male pattern hair loss.In such case minoxidil lotion helps.But it should be used for at least 3 months to get some benefit.It should be continued also.You can take vitamins for hair like biotin.Avoid stress and take nutritious foods.But if you have scales like silvery appearance then it can be scalp psoriasis.In such case if mild coaltar shampoo application helps.Local corticosteroids like clobetasol will be needed if no improvement.It is better to get a dermatologist consultation to rule out the type of hair loss you have.Hope this may help you.Let me know if you have any further query."
},
{
"id": 88377,
"tgt": "Why am I feeling discomfort on left side of stomach?",
"src": "Patient: I am having localized minimal discomfort on the left side of my stomach approximately 2/3 fingers from navel, and 2/3 fingers below navel. Hx of several abdominal surgeries and IBS....no temp... no vomiting, and bowels are formed and consistent for past few days. Doctor: Hi ! Good morning. I am Dr Shareef answering your query.With the history of IBS, and several abdominal surgeries (?reason), I would consider a routine stool test for ova and cyst and treat it if positive, and in case of no relief, might consider sending you to a gastro enterologist for an endoscopic evaluation (colonsocopy) of your intestinal tract, and possibly an ultrasound abdomen to rule out any intra abdominal organ pathology. As this was not an intolerable discomfort or pain, I would not consider for any medications till the reports of investigations were at hand which would decide on further management.I hope this information would help you in discussing with your family physician/treating doctor in further management of your problem. Please do not hesitate to ask in case of any further doubts.Thanks for choosing health care magic to clear doubts on your health problems. I wish you an early recovery. Dr Shareef."
},
{
"id": 123537,
"tgt": "What causes pain in foot after knee surgery?",
"src": "Patient: Since I had knee surgery and have been off my feet alot, the skin on my legs from the knees down has turned a mottled brown, only in the front of the leg all the way to the end of my foot. Do you know what could cause that? it doesn t hurt at all. Thank you Doctor: Hello, As the knee is fine and there is no pain post surgery I think will be the discoloration of the skin due to vascular insufficiency. try doing hot water fermentation and do ankle toe movements. Usually, the skin discoloration doesn't happen in many of the cases. Taking a blood routine will also be good to see if any infection is causing the issue. Hope I have answered your query. Let me know if I can assist you further. Regards, Jay Indravadan Patel, Physical Therapist or Physiotherapist"
},
{
"id": 178448,
"tgt": "Suggest treatment for cold and high fever",
"src": "Patient: Hi my son is 18 months old. All of a sudden yesterday morning at 3 he was running temperature of 103. We r giving sumol ds qnd clavam forte and meftal inbetween as the is not coming down at all. He has cold as well. We live in bangalore. Plz advise... Doctor: Most of the cough and cold in children is caused due to certain viruses. Viral infections do not respond to antibiotics. Sometimes, doctors prefer adding antibiotics like clavam forte to be on the safer side keeping bacterial infection in account. The medicines are proper and you would have to be a bit patient.I do not know if the dose of Sumol DS containing paracetamol is proper. It is to be given at 15 mg/kg/dose. Meftal P reduces fever even more effectively. Sponging with water at normal temperature would be helpful. You may also give some anti allergic like levocetirizine and some steam inhalation."
},
{
"id": 106430,
"tgt": "What is the treatment or Eczema in a child?",
"src": "Patient: Hi Doctor. I am asking this question for my son of 14 yrs old. He has an allergic like skin irritation of blackish color of about 2.5 cm area on the leg where the socks comes in contact(just above the angle) and he had itching on it. the doctor said it can be an eczema like change and gave some medicines and ointments after that it was subsided even-though the discolored skin is still there. Now we use only cotton socks. my mother has eczema for years on the legs. So i want to know whether this disease is completely curable or not? what is the latest in eczema management -not for the symptomatic management,but what is the latest treatment for a permanent cure if available? and which place? Thank you Doctor: Self-care measures, such as avoiding soaps or other irritants and applying creams or ointments, can help. Avoid skin irritations, avoiding extreme temperatures, and using bath oils, lotions, creams or ointments to lubricate your child's skin. Light therapy (phototherapy) is the recent technology, but it has its own sideeffects. Consult a Dermatologist in your area."
},
{
"id": 201768,
"tgt": "What causes dripping of semen during sex?",
"src": "Patient: my semen used to shoot out earlier but now it just drips. Is it due to the reason that during intercourse i tried the semen not to spill out here and there and tried to stop it with outer skin of the penis. does it affect the potency or any is it a symptom to any harmful thing on a later stage Doctor: Thanks for contacting HCMI am sorry to hear that you have problems with ejaculation. You are mostly concerned about the force of the ejaculate material at time of ejaculation. As men get older the force of the ejaculate will decrease and will not affect fertility. The semen that is ejaculated just needs to be inserted into the vaginal vault to begin conception. Hope this answers your medical question. Please contact us again with your health care concerns and questions"
},
{
"id": 127856,
"tgt": "What causes chest pain along with tingling sensation in the jaw?",
"src": "Patient: My son is 21 years old and had been having chest pain lasting 3hours last evening. He said he also felt some pressure In his chest and experienced tingling of sorts In his jaw area. He said he felt it some info his throat area also. In the past few months he has experienced a pounding rapid heart lasting a couple hours also. He does not live at home so I was not there to take him into the emergency room. Doctor: first You do your investigations consultant cardiologistuntil I will suggest you some Ayurvedic medicinetab arjunadi vati 2bd tab sarpgandha 2 bed time."
},
{
"id": 171066,
"tgt": "What causes frequent gasping in infants?",
"src": "Patient: My son is 5, and over the past 5 days he s begun gasping for breath only while sitting. He s able to run and lay normally and I don t notice this breathing while he s sleeping. But I am concerned that when he sits still while playing games, coloring, watching movies, etc. he repetitively takes deep breaths, saying he has to . I took him to a pediatrician last week to rule out asthma, which he did. 4 days later it seems to be getting more pronounced. What could it be? He has environmental allergies, but generally very healthy: and is about 39 lbs. Doctor: HiWelcome to the HCMI have gone through your question and understand your concerns. The symptoms of breathlessness only while sitting and not during physical activity, sleep and other times is most likely due to functional disorders. If he is normal on clinical examination by a pediatrician, then I would recommend you to ignore the symptoms. Most cases settle down with in a few days. If it persist, then visit a psychotherapist for proper evaluation and management.Take care"
},
{
"id": 141166,
"tgt": "What causes head pain during a physical activity?",
"src": "Patient: On Nov 12 2015 I was hit in the back by a car, the hospital said I was okay and I was at work the next day, I m now trying to prove nerve damage, spinal damage, dislocated,my ribs, shoulder blades, vertebrae aren t where they re supposed to, if I open a pickle jar, my spine twist and pain shoots into my head, I ve been referred to a neurologist but don t have health insurance, im afraid of the injury and what it s done to my skeleton, I m looking for a neurologist to prove nerve damage so I can move forward to seek diagnosis and treatment, please help Doctor: Hi, It may be well possible that there has been a spinal damage. However, if you need proof, I am afraid that simply contacting a neurologist, whether on this site or elsewhere is not enough, the diagnosis is confirmed through tests which can't be done online. So I understand your difficult situation regarding health insurance, but it is hard to prove anything without imaging tests like MRI. As for management it consists mainly of physical therapy and medication for neuropathic pain. After so much time it is doubtful that there would be any benefit from a surgical procedure (which again needs a MRI to determine whether there was any indication in the first place). Hope I have answered your query. Let me know if I can assist you further."
},
{
"id": 149987,
"tgt": "Heating up body, bright red face and eyes, dripping wet, loss of consciousness, low BP usually from early spring to late fall. Suggest",
"src": "Patient: HI! I BEEN DEALING WITH THIS PROBLEM FOR 25 YEARS AND NO ONE KNOW WHAT CAUSES IT. I've had every test there is . my problem is it happens any time from early spring to late fall. I get this weird feeling throw out my body I heat up and my face turns bright red and the whites of my eye are also red. i start losing control of my body. i can hear but i can\"t respond.I GET DRIPPING WET AND I GO UNCONSCIOUS. MY BLOOD PRESSER GOSE SO LOW THERE CAN'T GET IT. SOME TIME I'M OUT FOR 45 MINUTES. ALL THEY DO A THE HOSIPTAL IS OXGYEN AND A IV. DO YOU HAVE ANY IDEALS IT SUCK LIVING LIKE THIS. THE OLDER I GET THE WORSE IT SEAMS TO GET. Doctor: you are dealing with allergies of pollens ad as you grow immunity decreases and attack increasesget blood serum tests fr specific antibodies for pollens of your aea at that seasonand go for immunotherapy to get rid of it"
},
{
"id": 226357,
"tgt": "Upper abdominal pain, strange sensation in throat, mood swings since intake of marvellon. Is this due to pill?",
"src": "Patient: I had the copper coil removed and started taking marvellon and since I ve been taking it I Have developed pain in my upper abdomen and strange sensation in my throat and a feeling like my food is stuck. Maybe a hearyburn feelibg too (i never suffered with this previously) coukd the pill be causing this. I have felt sick at times any I don t seem to be enjoying my food when I have this discomfort. I have been taking it for 6 months now. i gave also been very tearful. please help Doctor: Hi, Thanks for the query. Marvellon which is a combined oral contraceptive pill can cause side effects related to gastrointestinal system and can also cause symptoms like depression etc. But these symptoms will usually subside soon without need of medication. As you are having the symptoms since 6 months, better to consult your doctor and discuss regarding changing the pills. Take care."
},
{
"id": 61501,
"tgt": "What causes a sore lump between the vagina and rectum?",
"src": "Patient: I have had a sore lump between the vagina and rectum for over a week. It got harder, then I squeezed it and a lot of puss came out. I thought it might of been a boil or ingrown hair and would be better now that I popped it. It still is very sore, ,, what should I do at home because I don t have health insurance? Doctor: Hi.Thanks for your query.Noted the history and understood your concerns.Such a lump between anus and vagina having pus and soreness can indeed be an infected boil with pus formation but can be beginning of a fistula in ano.This can be diagnosed by only a proper per- rectal digital examination, proctoscopy and ultrasound or MRI .In spite of no insurance you have to this done and a visit to a doctor is needed to get a clinical evaluation, examination and to get a proper prescription of an antibiotic and anti-inflammatory medications, proper cleaning of area with betadine and sterile dressings."
},
{
"id": 81628,
"tgt": "Will taking Advent syrup safe for cough?",
"src": "Patient: Hi, My son had cold initially which got treated but then he had severe cough problem from last 3 weeks and we are giving him advent syrup from last 7 days now. doctor suggetsed 3.5ml B.D for him but by mistake we have given him 5 ml in the morning and 5 ml in the evening? will it have any side effects on him. he is 2.3 years old. Doctor: Thanks for your question on HCM. Advent is combination of amoxicillin with clavulanic acid, an antibiotic. And in pediatric patients, dose of antibiotic must be accurate. So 5 ml is high for him. And the common side effects of it is gastritis and loose motions (diarrhoea). And I think you will need antihistamine syrup too.So better to consult pediatrician and discuss all these. You may need chest x ray to rule out LRTI (lower respiratory tract infection) as cough is not improving since 3 weeks."
},
{
"id": 215597,
"tgt": "What causes swelling in the right ankle during the daytime?",
"src": "Patient: Out of the blue my right ankle swells up with no pain and no injury. Will go down over night and swell up during the day. 60 year old female with good health. No illnesses or prescription meds. Am about 30 pounds overweight and sit at computer 12 - 15 hours a day. Not much exercise in past 6 months due to work and school. Doctor: Hello, Most probably it is due to inflammatory conditions like small varicose vein. As of now, you can use analgesics/anti-inflammatory combination like aceclofenac/serratiopeptidase for symptomatic relief. If symptoms persist better to get an ultrasound Doppler to make a diagnosis. Hope I have answered your query. Let me know if I can assist you further. Regards, Dr. Shinas Hussain, General & Family Physician"
},
{
"id": 143399,
"tgt": "What is the treatment for neck pain?",
"src": "Patient: i recently just moved my neck not long after waking up and i heard a kind of 'crunch' noise, then when i went to sit up i can't move my neck or shoulder to the left without being in agony. turning to the right is less painful. and also to move my left arm it hurts at the shoulder. ? Doctor: Hello!Welcome on HCM!Regarding your concern, I would explain that your symptoms seem to be related to a musculo-skeletal disorder. I would recommend having some rest for some days. Holding a cervical collar may be helpful. For the pain I would recommend taking painkillers like ibuprofen, coupled with a muscle relaxant like flexeril. Your symptoms will improve gradually. If the problem persist, I would recommend consulting with your doctor for a physical exam and a cervical spine X ray study. A cervical spine CT scan may be needed. Hope you will find this answer helpful!Best wishes, Dr. Aida"
},
{
"id": 161617,
"tgt": "What causes post nasal drip in a toddler?",
"src": "Patient: Hello Doctor, My name is Suma, have a daughter who is 3 yrs old. We are in Sydney Australia. My daughter is suffering from post nasal drip from almost 4 months.. inspite of showing to my doctors and even ENT Surgeon we are not able to find out the exact reason for this. she is on antihistamine (Claritine 2.5 ml daily once). Can u pls hlp me out someway to control her cough and running nose. I will be in bangalore during may during which i will be visiting u. waiting for ur reply Doctor: Hello, This may be -1. Moderate to severe persistent allergic rhinitis.2. Upper airway resistance syndrome. She may need intranasal steroids. I suggest you discuss this with her pediatrician. Hope I have answered your query. Let me know if I can assist you further. Regards, Dr. Sumanth Amperayani, Pediatrician, Pulmonology"
},
{
"id": 75813,
"tgt": "Suggest remedy to relieve pain in chest near heart",
"src": "Patient: My husband has a pain in his chest by his heart had it for about 10 days but has got worse he was diagnosed with acute myloid leaukemia last september and had the chemo daunarubicin which they have warned can cause heart problems. He says the pain feels like it is brusied what would you recommend He is 6ft 45years and about 103kgs Mary Doctor: Thanks for your question on Healthcare Magic. I can understand your concern. In my opinion, he should immediately consult doctor and rule out heart diseases for his chest pain. So get done ecg, 2d echo and stress test to rule out heart diseases. If all these are normal then no need to worry for heart diseases. He is known case of CML (chronic myeloid leukemia). And this can spread to bone (ribs) and lungs and they inturn cause chest pain. So get done CT scan and PET scan to rule out rib and lung metastases. So don't wait at home. Take him to the Hospital and discuss all these with his doctor. Hope I have solved your query. I will be happy to help you further. Wishing good health to your husband. Thanks."
},
{
"id": 64102,
"tgt": "What could soft bumps on the labia minora suggest?",
"src": "Patient: Hello - I have noticed a series of small purple raised but soft bumps on both sides of my labia minors. The are not painful or bleeding and they seem to be where there is clearly a dark vein under the skin. Is this anything I should worry about. I don t look often so not sure how long but there certainly was on one or two last time , a couple of months ago. Doctor: HI,Dear,Thanks for the query to HCM.I understood your concerns about the labia lump.Cause-In my opinion its -varicose veinous lumps mostlyTreatment-a-Take opinion from your ER Gyanaec if it remains and grows in size in next 2 mthsb-I would advise Daflon cap/ and Vit E- till then.Hope this would resolve your worry.Hope this would help you to plan the treatment with Your family doctor.Wishing you early recovery.You are always Wellocome for further query to HCM in this regard.Have a Good Day...!!"
},
{
"id": 169522,
"tgt": "Suggest treatment for severe cough in a child",
"src": "Patient: My 3 year old son has developed a cough in the past couple of days and is gasping for air between coughs. Today is worse than yesterday. Happens more at night and during the mornings. Not so much during the day. His nose seems clear and not much Flem when coughing. Any ideas? Doctor: look like Hyper reactive airway disease as cough is more in the nights and early mornings.syp.ascoril LS to be given along with levolin nebulization ."
},
{
"id": 80829,
"tgt": "How to cure typhoid?",
"src": "Patient: My mother has tested positive for S typhi (o) and S typhi (H) upto 1:80 . i don`t understand the technicality but we are told she is suffering from typhoid. She knows from her medical history that she is a carrier of the bacteria. Is she really suffering from typhoid? Ran fever 2 days ago with nausea and dairrohoea lasting only few hours. Doctor: HIWell come to HCMTest result need to be correlate with clinical condition for clinical diagnosis because such titration could be positive if patient had a history of typhoid fever, and this may not be active phase of infection, the best way to rule out the possibility of this urine test and stool test for detection of \"Salmonella typhii\" else this could be anything else if not correlated with clinical condition, hope this information helps, take care."
},
{
"id": 114693,
"tgt": "How to control ASO level of 317 IU/ml ?",
"src": "Patient: Dear Doctor, I am Sudip Mukherjee (37/Male) from Burdwan District (West Bengal \u2013 India). I have a pathological lab. A few months (Probably_March\u201914) ago I feel joint pain on wrist and knee. 21/04/2014 = Then done blood test for Uric Acid, where result 7.1 mg./dl. & RF=14.2 IU/ml. Pain increasing day by day and compel to consult doctor then he prescribed (07/05/14) the medicine \u201cFeburic-40\u201d for ODHS_1 month. 19/05/2014 = CRP-3.4 mg/I, Rheumatoid Factor-5.2 IU/ml. 21/07/2014 = ASO \u2013 251, URIC ACID \u2013 9.0 mg/dl., CRP-3.5 mg/I, Rheumatoid Factor-6.5 IU/ml. Doctor change the medicine after shown the pathological report = \u201cCIPLORIC-300\u201d (ODHSx10) then BDPC. After taking medicine on 25/08/14 = Blood report of Uric Acid 3.7 mg./dl. And ASO-317 IU/ml. Now advise me, how I can control the ASO level. Doctor: Hi, ASO titres indicate previous exposure to streptococcal bacterial infection. It is difficult to tell whether your present problem is related to that., what ever it may be, elevated ASO in an Adult is not a concern. Your condition seems to me like gout which needs further evaluation. Just ignore the ASO. Consultation with a Rheumatologist will certainly help you. All the best."
},
{
"id": 91125,
"tgt": "Why is my stomach hurting so badly?",
"src": "Patient: I stuck a q-tip in my belly button because it has been hurting above the navel and it also had nasty smell of crusty stuff around it. It was bad bleeding but just even a bit of blood on it scared me. Is it anything serious? Or does it just need cleaned with peroxide? & also, my stomach has been hurting all over. Doctor: Hi.You have to get this checked by a Surgeon as this is looking to be a -Granuloma of the umbilicus. Sometimes does heal with antibiotics but many times needs a removal by a small surgery.What bothers me is the pain , you say all over. The only umbilical granuloma should not cause pain all over. There may be an another reason. It is possible that this has given inflammation to the inner part of the abdominal wall."
},
{
"id": 160892,
"tgt": "Suggest medication for high fever",
"src": "Patient: My 2.5 year old daughter got a fever of 102 out of nowhere yesturday, a few hours later she threw up (she hadnt eaten all day but had a little bit of juice, which is what came up). A couple hours after that she started screaming and crying that she hurt, when asked what hurt she said her back and she d hold her lower back. She is not constipated. This went on for about 8 hours last night before she finally fell asleep. Today she seemed alright till the afternoon when she got another fever. She then slept for 3 hours and woke up seeming well again. Later in the evenig she started screaming about her back again. She throws herself onto the floor or couch screaming in pain. Tylonal doesnt seem to be helping for anything but the fever. Doctor: Hi, This is probably a kidney problem. She needs to do complete urine analysis and take catafly and cirprofloxacine as well for any underlying infection. Hope I have answered your query. Let me know if I can assist you further. Regards, Dr. Salah Saad Shoman, Internal Medicine Specialist"
},
{
"id": 214809,
"tgt": "What are the home remedies for hearing loss and er glue?",
"src": "Patient: I am suffering from er glue...left ear hering loss, right ear getting the same..i saw doctor and he prescribed oxifloxacin (one ablet for five days) and a nose spray... I have this condition now for almost 2 weeks...what can I do at home to treat this because I have to work and I can not hear wellll Doctor: Hi, Welcome to Health care magic forum. As you describe it may be the glue, or some thing else which is causing the hearing loss. I advise you to consult an E.N.T. surgeon for diagnosis and treatment. I usually prescribe for the wax in the ears,wax removing ear drops,daily 2 times, for 10 days, and to be cleaned with the ear buds once every 2 days so that the wax is removed layer by layer. Wishing you a quick and complete recovery. thank you."
},
{
"id": 89979,
"tgt": "Can high cholesterol lead to abdominal pain?",
"src": "Patient: Hello. Sorry for disturbing. I have an abdominal pain.. down in the left and up in the right...feeling like a liver pain. My gall bladder was removed 3 years ago..and since then my digestion is very quick...and it s like some kind of irritable colon.I was to see a doctor last week. My blood tests are good..meaning the liver,kidney,infections,blood sugar...just my cholesterol is big 7.1. I have to say that I have ulcer too(lucky me). I took some medicine with mint oil ..with no result. Now I am taking colospasmin..and it s better..no pain in the left...just the 1 in the liver area. What can it be? I need some help...because I have this pain for more then 2 month. Thank you in advance. Laura Doctor: Hi.Thanks for your query and an elucidate history. High Cholesterol can not cause an in abdomen. You have pain in abdomen for 2 months. I would suggest you the following . First of all you have undergo the tests as you are suffering from Abdominal pain for the last 2 month. ::UltrasonographyCT scan of abdomen to rule out Gastrointestinal Cancer.blood, urine and stool tests Enteroclysis to see if there is an Intestinal Obstruction.The pain in the left can be due to Colitis or Irritable Bowel Syndrome IBS and the one in the right upper can be due to liver problems although the liver functions tests may be normal ."
},
{
"id": 162425,
"tgt": "How can speech delay in a toddler be treated?",
"src": "Patient: My baby is about 23 months he always use his hand push me to understand his demand. His ear no problem. He can say papa, duck like this not Over10 words. His adenoids gland have problem. He want to say but can\u2019t speak out. Make some sound which I not understand. What should I do Doctor: Hi, If he is 23 months old with speaking difficulty and using sign language then it is an indications that he needs to be worked up for - 1. Isolated speech delay 2. Autistic Spectrum disorder. 3. Hearing defects. I need to take more history to assess him. Hope I have answered your query. Let me know if I can assist you further. Regards, Dr. Sumanth Amperayani, Pediatrician, Pulmonology"
},
{
"id": 196374,
"tgt": "How to get rid of nightfall?",
"src": "Patient: Hi, may I answer your health queries right now ? Please type your query here...kkk siriam hari..age 25..i use to masterbute and now i almost stopped it..from der nightfall started..nd iam loosing ma good health due 2 dis,,,pls help me.ma facial hair growth is also weak. Doctor: HiGREETINGS masterbation is a healthy way of satisfying your sexual disires and doesn't have any side effects.You can do daily or alternate days.You will be enjoying better if you do on alternate days rather than daily.Semen is produced daily in our body and masterbation will not affect your health or fertility.One precaution is never hurry to discharge, delay ejaculation always by atleast 5 to 10 min.Night fall indicates your genital system is intact.so nothing to worry.Hope my answer helps you. Regards"
},
{
"id": 70661,
"tgt": "Why do I feel a knot and get sharp pain under right rib cage while breathing?",
"src": "Patient: At anytime while breathing in,there s a sharp pain right under the right side of the rib and painful to breath in after that,I could touch and feel a knot on the area,slowly I massages it.then it would disappear and I can breath normally again.but I sometime have pain and a warmlike sensation.what could it be?I have problem for years now. Doctor: Hello, As you explain the history you should do a chest X-ray. Please discuss with your doctor. Hope I have answered your query. Let me know if I can assist you further. Take care Regards, Dr Jnikolla, Pulmonologist"
},
{
"id": 109881,
"tgt": "Suggest remedy for pain in the lower back",
"src": "Patient: I drove for an hour then sat through a nine hour class, then drove home an hour. When I woke up the next day my lower back ached, but I had to do the same thing the next day. This was Tuesday and Wednesday. And now it s Friday and it still aches, but as I walking I sneezed, pretty hard, and my legs almost have out. I had to sit for a whole and couldn t walk.. I laid down for a little bit and when I got up it still hurt but not as bad, however I stand crooked now, and it hurts my lower back to put pressure on my right leg. Walking hurts as well and I walk crooked.. If I m standing in the mirror it s noticeable that something is wrong and my back is crooked. Will a chiropractor be able to fix this? Doctor: Dear Sir/ Madam,Thank you for posting your query at healthcaremagic.comI advise you to-- put icepack multiple times in a day. Do not allow the part to become numb. It is to be kept not more than 5 minutes.- Muscle relaxant and anti inflammatory- RestIf pain is still not coming down consult the orthopedician and get x-ray done.Please feel free to contact me if you need any further information.With best wishes,Dr. Tehzib Saiyed"
},
{
"id": 66112,
"tgt": "What causes lump on the upper jaw causing sensitivity and pain in the mouth?",
"src": "Patient: Hi. I have on my upper left side a small gum colored zit or bump on my gum. I just noticed the bump yesterday but it has been hurting for days. When I touch or push it the pain comes. If I leave it alone that side of my mouth is just sensitive. Any ideas what it could be? Doctor: Hi! You got a common pattern lump!In day to day practice I often get such lump on the upper jaw causing sensitivity and pain in the mouth and after proper examination and some tittle investigations like microscopic / pathological test, I come to following diagnoses:1. an abscess in periodontal location 30% cases2. infected periapical cyst 20 % cases3. minor injury to gum mucosa and related infection / edema 10% cases4. pyogenic granuloma like bloody benign tumor-like lesions 30% cases5. other benign tumors 8% cases6. cancer 2% casesTherefore maintain oral hygiene and see a dentist for relevant antibiotics / necessary steps in needed.regards,"
},
{
"id": 165336,
"tgt": "What causes club foot and delays in development?",
"src": "Patient: My son was born with clubfeet and later on showed developmental delays in everything, walking at 3, still struggling very much with talking (at 4y, 3 m), sensory problems, high pain tollerance. He is getting occupational therapy to help him achieve his goals and if one look at him seems really normal and is quite affectionate. Neurologists have tested for genetic diseases, MRI scan, considered Autism - but still to today no outcome. The only thing on the MRI noted in the report was a lower diffuse peak of N-acetyl aspartame acid in his brain. We did not get answers but I suppose believed that we can only wait and hope, until yesterday when we learned that my sister law needs to make a terrible decision. Her first baby (she is 30 weeks pregnant) was noted to have agenisis of his Corpus Colossum and they now have to decide if they want to abort or raise a special needs child. Can there be a connection between the two. On Wikipedia symptoms of corpus colossum diseases show delayed development, motor and sensory problems, late potty training (only age 4!) ets. Please advise. Doctor: Agenesis of Corpus Callosum can be detected on MRI. As MRI has already been done, Agenesis of Corpus Callosum is ruled out.Developmental delay can have many different causes, such as genetic causes (like Down Syndrome ), or complications of pregnancy and birth (like prematurity or infections). Often, however, the specific cause can not be identified.If your child is responding well to occupational and behavioral therapy, future outcome will be satisfactory."
},
{
"id": 28556,
"tgt": "Does shingles cause recurrent pustules on the body and constant headaches?",
"src": "Patient: I have constant pain seemingly related to my shingles. Now originally the site of my rash, though unusual, is a spot at the end of my tailbone with which it caused shooting, tingling, burning pain before and continue for months and months after. I really cant say that I actually ever got relief, I m just being optimistic!! Is this possible #1? #2) I have been having recurring pustules that do not ooze or pop, that seem to be directly related to a nerve ending in my scalp and hairline along with constant headache and nausua. Could these flares be also related to shingles because it really feels to me like a shingles rash. Oh and there may be a slight clear substance I could get out if I tried but very little and very tender to the touch. If not shingles, what is your best guest. This is very painful, irritating and distracting condition, please help... Doctor: Hi, I have read your query & yes the recurrent pustules, itching, pain, prickling skin, rash are symptoms of Shingles. Visit ER, get physically examines, get STD tests done. You should start on the antivirals & antibiotics without delay. The rashes on the scalp often cause a headache, nausea & then fever. If neglected the immune system will further get weakened may complicate to Herpes Zoster spreading & affecting the eye. Such infections cause corneal damage & vision problems later in life. Get connected to your PCP, he may need to give you stronger nerve blocks, medications & pain killers. Take nourishing diet & Vitamin supplements. Hope I have answered your query. Let me know if I can assist you further."
},
{
"id": 183342,
"tgt": "Can a broken tooth lead to swelling in mouth?",
"src": "Patient: i have a tooth that broke in half a year ago and then i slacked and didnt get it fixed it hurt un controllably for 2 weeks i have no medical insurance and today i woke up and my face a jaw and mouth around the tooth is swollen. im a laid off emt so i have needles and such in my bag and was wondering if i should drain the puss out. and there are no lumps so its not abcesed Doctor: Thanks for your query, I have gone through your query.The pain and swelling is because of the tooth infection secondary to the trauma. Consult a oral physician and get a radiograph to rule out the infection. Mean while you can take a course of antibiotics like amoxicillin 500mg and metronidazole 400mg tid for 5 days (if you are not allergic). Once you get money, you can get the tooth treated with RCT or extraction.I hope my answer will help you, take care."
},
{
"id": 77056,
"tgt": "Suggest dosage of Sumo tablet prescribed for chest congestion",
"src": "Patient: Sir my daughter is 12 years old and she has chest congestion and fever 101 degree celcius. Doctor has advised me sumo tablet for fever. In the prescription cared, it is not clear whether we should give half tablet or one. Kindly advice how much dose of sumo should I give to my daughter. Doctor: Hi I can understand your concern...Mostly here you have congestive symptoms by viral flu....If I am not wrong the drug you are saying is sumo which is nimesulide drug.....For relief from fever nimesulide should not used....this drug also banned in some country...According to me take paracetamol like safe drug for it....Along with antihistaminic drug can be given..As far as dose of nimesulide concern this tablet can be taken once or twice daily....Dr.Parth"
},
{
"id": 92000,
"tgt": "Should I be concerned for trace amounts of free fluid in pelvis?",
"src": "Patient: After some abdominal discomfort, my gastro ordered an abdominal/Pelvic CT Scan with contrast. Findings showed no acute abnormality of the abdomen or pelvis, but there was \"trace amounts of free fluid in the pelvis\". I am a 58 year old female, complete hysterectomy in 2008. Some research shows the med community needs more info to pinpoint the problem. Just curious on your take...Thanks Doctor: Hi. Trace amount of fluid in the pelvis without any other symptoms should not be worried about. You already have a history of hysterectomy and Ct showing no other abnormality. So relax and get simple sonography done after 6 weeks to see the status. IF persists then only to go for the further tests.. IT is very difficult and not advisable to aspirate such a small fluid."
},
{
"id": 105624,
"tgt": "Allergic asthma, persistent chest congestion, breathing difficulty post taking Prednisolone, bronchodilator,thephylline. Reason?
",
"src": "Patient: Hi, Last month, on 17 April, I was diagnosed with Allergic Asthma by a doctor, and prescribed Bronchodilator 2mg, 20tab and prednisolone 5mg 2 tab and mucolytic syrup . after complete these medicine i was not yet good and again visited to doctor and he has given medicine as thephylline 100mg and mycolytic. again in between I felt breathing difficult and visited to doctor and got medicine pronchodilator 2 mg half tablet 3 times and prednisolone 5mg two times, and fixit for gas. today i finished my prednisolone 5mg, still i am feeling chest congestion like mucus inside and i don t have full breath. so today againd doctor prescribed me prednisolone 5mg 40tab two times a day and asked to complete remaining medicine too. i just want to know what may be the reason that i am still not got back to normal in my breathing. is it okay to continue these medicine? Doctor: Hello Thanks for your query. I will try my best to provide you with best possible professional recommendations to your questions. Allergic asthma is the most common type of asthma. The most important thing in allergic asthma is to find out the cause and avoid or to sensitize to it. Allergens, which are small enough to be inhaled deep into the lungs, include windblown pollen from trees, grasses, and weeds mold spores and mold fragment animal dander and saliva dust mite feces, cockroach feces. An important part of controlling your allergic asthma is to limit your exposure to allergens. Determining the allergens to which you've become sensitized, and then taking steps to minimize your exposure to them, is likely to improve your allergic asthma control. Regarding medications you are already taking them. I advice try to find out the cause and avoid them, like try using face mask while going out, avoid the pollen and if you have pets avoid them for few days. Yet again, I do hope that you have found something helpful and I will be glad to answer any further query. Regards"
},
{
"id": 75485,
"tgt": "What causes acute chest pain?",
"src": "Patient: I have sharp, knife-like pain in the center of my chest just above my breast line. the doc has ruled out gall bladder and says the pain is in my esophagus. No indigestion or nausea - no difficulty swallowing - no pain when lying down or sitting. But sharp pain when walking. Doctor: Hi welcome to health care magic. Here you are having sharp chest pain in center of chest,.So mostly it could be from acidity or esophagus pain.... Another possibility of muscular pain also can be there..... For few days avoid excess spicy foods. ..Drink more water.. Avoid smoking... Fried food less.... Ibuprofen tablet or paracetamol can be taken to relieve pain... If pain increasing, cough, dyspnea present than ekg, chest x ray like investigation needed Advise-physician consultation for examination ....Pulmonologist also can be consulted Take care"
},
{
"id": 79875,
"tgt": "Suggest treatment for a Laryngospasm patient",
"src": "Patient: My father is a patient of dementia for last few years. A recent persistent failure of response made him hospitalized. He has been released with Ryles tube and catheter. We had to hospitalized him again for. Laryngospasm Presently, he is at home under nursing care but his general health is deteriorating very fast. He has lots of secretions/cough, stomach cramps and a simultaneous fall of oxygen saturation. He is completely bed ridden and very hard to move anywhere. I was wondering what could be done for him! Doctor: Thanks for your question on Health Care Magic. I can understand your concern. Best way is to take your father to hospital because possibility of pneumonia is more in his case. Dementia causes aspiration of food content in the lung and this causes aspiration pneumonia. If you want to treat him at home than tracheostomy is an option. In this trachea is opened in front part of neck and tracheostomy tube is inserted in it. Through this tube sectioning should be done to remove secretions in the chest. Hope I have solved your query. I will be happy to help you further. Wishing good health to your father. Thanks."
},
{
"id": 54034,
"tgt": "Is a drain necessary after a laparoscopic cholecystectomy?",
"src": "Patient: I had a laparascopic cholesystectomy on friday so i am 4 days post op now. I had a drain inserted after my surgery which i had removed yesterday before i was discharged from hospital. Why would i have needed this drain, and is it normal to hurt me now? I am having a bit of pain when i move and breath in deeply, and also when coughing. Doctor: Hallo,First of all , laparoscopic cholecystectomy is one of the most performed surgeries by abdominal surgeons. In most cases there is no need to place a drain.Sometimes when a operation was more \"bleeding\" then expected it can be useful to put a drain to have remaining fluid removed postop out of your belly. That's better to have it out than it would stay in off course.So it's difficult to see if a drain was necessary for your operation, that's most of the time a peroperative decision by the surgeon. The pain after removing the drain must go away pretty quick. Sometimes patients might suffer for 1-2 days after removing and a simple pain killer can relief the pain. It depends also on which type of drain was used (with or without suction for example). If the pain continues or even gets worse, then I perform a control ultrasound and blood exam to check whether everything is fine. There might be a small collection of fluid for example that might irritate the diaphragm which causes pain, coughing etc...But I would wait another day, and if the pain gets worse, it's time to contact your surgeon again.Hope you get well and take care."
},
{
"id": 80581,
"tgt": "What should I do with my Thymic Tumor?",
"src": "Patient: Hi. I had a thymic tumor (kinda unusual for an adult, I guess) removed 5 years ago after watching it grow for 3 years. Have had pneumonia twice since. 3 benign lesions found on lung. Sick now with that heavy compression in my chest that led to finding the tumor but also on edge of pneumonia. Think I will stave it off, but for some odd reason, got to thinking about that tumor. Should I just forget it? Doctor: Hello dear, thanks for your question on HCM. I can understand your situation and problem. In my opinion you are having recurrence of thymic tumour. Symptoms of chest compression is suggestive of thymic tumour compression.So better to get done CT thorax with contrast to know the exact size of of thymic tumour. Pneumonia is also bettr seen in CT thorax. So further plan of treatment is based on CT thorax report.If thymic tumour is large enough to cause compression then surgical removal should be advisable. If only pneumonia is there then you need antibiotics. So consult your doctor and get done CT thorax and plan treatment accordingly."
},
{
"id": 195642,
"tgt": "Suggest treatment for erection problem",
"src": "Patient: Hi Doctor, I am a 22 year old male who lives an average lifestyle. Unfortunately i have been watching porn and jerking off few days a week sometimes daily. I dont really jerk off like normally a guy would. I tend to friction masturbate which mean i rub it against my matress or pillow, my dick would still be in my pants. I noticed that my penis seems shorter and that my erection is not as hard. My weight has more or less maintained. What should i do? Thanks Doctor: Hello and Welcome to \u2018Ask A Doctor\u2019 service. I have reviewed your query and here is my advice. According to history provided you are now having erection problem and let me know that masturbation could not lead erectile dysfunction so don't worry about that.now for your erection dysfunction kindly check your blood pressure fast and also check your random blood sugar and cholesterol estimation.you have mentioned that you are obese and so let me know you that obesity can lead erectile dysfunction problem.sorry reduce Your Weight by Lifestyle changes.let me know that excessive anxiety also can lead erectile dysfunction like problem.Yoga meditation also will be beneficial and if you have diabetes, hypertension then it should be treated. Hope I have answered your query. Let me know if I can assist you further."
},
{
"id": 79125,
"tgt": "Does any scar in lungs on x-ray suggest TB?",
"src": "Patient: I suffered from TB and is cured now. I am planning to visit Muscat-Oman and require a family visa. Whetther during tests conducted by GCC approved labs, if any scar is present in the lungs will they reject my visa. Thaks and regards A.K. Balasubramanian Doctor: Thanks for your question on Health Care Magic. I can understand your situation and problem. Yes, presence of scar tissue on chest x ray is strong reason to reject your visa. Visa authority can reject your visa due to having possible active chest infection like tuberculosis because of scar on chest x ray. So we need to prove that this x ray finding is due to old, healed tb and not due to active tuberculosis. For this you need to consult pulmonologist and get done CT thorax with Contrast and bronchoscopy with BAL (bronchoalveolar lavage) analysis. If both are negative for tube than you can argue with visa authority that x ray lesions are due to old, healed, inactive scar and not due to active, infectious tuberculosis. So consult pulmonologist and discuss all these. Hope I have solved your query. Wish you good health. Thanks."
},
{
"id": 30314,
"tgt": "Do the widal test results show that I suffer from typhoid?",
"src": "Patient: SIR ITS MY QUERY REGARDING THE FOLLOWING VALUE OF WIDAL TEST.... O1:320 H 1:320 AH 1:160 BH1:80 DONE ON 27 JANUARY 2014 AND ANOTHER TEST ON 12 FEBRUARY O 1:160 H 1:80 AH1:160 BH1:80 SIR I JUST WANT TO KNOW WHAT THIS TEST SIGNIFY? M I SUFFERING FROM TYPHOID??? Doctor: hi.. your report is suggestive of current or past typhoid fever.Widal test is a non specific test and can be positive after months of treatment from typhoid , as these are antibodies which found in blood.so you need to answer1. still you have fever ?? 2. test done which day of fever??3. any other symptoms??so as of now its difficult to interpretate current report due to lack of history. write me about above answers for better help.take care"
},
{
"id": 44910,
"tgt": "Can Ayurveda help to increase sperm ?",
"src": "Patient: hi im farah.. m married from last 8years but didnt conceive yet... ma husband have the problem of nill sperms he did medicine many where but failed every where dr advised us to adopt so its gone 2yr i adpoted baby gal.. nw shz of 2.5 ... i very happy with her.. but stil i want to conceive plz help me out from dis blame dat m baanj n all... can ayurved help to increase sperm? m25 yr female n ma husband is of 37 hz healthy 83kg in weight n his height is 5.10. Doctor: Welcome to Healthcare Magic Good Day Absence of sperm usually indicated block to the tubes that transport the sperm. IVF can help in such cases. Kindly visit Hospitals who do IVF and take a consult and this can be treated and you can get pregnant with your husband's sperm. It requires few tests to see if his sperm will fertilize your egg, for this you need to consult the Gynaecologist in such a Hospital."
},
{
"id": 45682,
"tgt": "Will Kidney problem lead to weight gain?",
"src": "Patient: Hello, my name is Dianne. I have suffered from hypertension for over 6 years. Several medications did not work. My right kidney was removed in November of last year. My bp came down. I am now on only 200 mg of labatalol 2 times daily. I has been increasing. About two weeks ago I experienced and was treated for bronchitis with Z-pack then started Cipro three days ago. However, I am extremely out of breathe now and have muscle pain from my upper neck to my lower back. It feels as though I am bruised to the point I cant even wear a bra. I have tightness in my chest just to the left side. I also feel very tired and have been unable to work because of this and have gained 6 pounds in just a weeks time. Could this possibly be from my kidney. The last test showed 74% function of the remaining kidney. That was in January, It took two er visits with the chest pain for them to say it was bronchitis. Doctor: Hello, Sorry about your health condition, yes, kidney dysfunction may cause fluid retention and weight gain.Also uric acid accumulation lead to gouty arthritis or joint pains. Electrolyte imbalance also present like this. Pulmonary edema also may present like kidney failure.Avoid spicy food and alcohol avoid smoking and caffeine drinks.Stop Ciprofloxacin intake. Get it done complete blood picture and renal function tests. Hope I have answered your query. Let me know if I can assist you further. Regards, Dr. Penchila Prasad Kandikattu , Internal Medicine Specialist"
},
{
"id": 208860,
"tgt": "Suggest treatment to overcome inferiority complex",
"src": "Patient: I think , i have too much of inferiority complex. i would like to defeat this problem. when saying in practical this problem is 1) I can't able to write legibly when anybody watching me. My writing is too much of slow & my writing looks not good 2) i can't able to speak with new person2) i have too much of stage fear. Doctor: DearWe understand your concerns.I went through your details. Inferiority complex means you feel inferior in front of others. This also shows your lack of self confidence. If you are worried about your hand writing and speed of writing, then improve it. There are several tools. If you are bold enough to speak in front of people, get yourself trained for that and remove stage fear. There is no use blaming your disadvantges and feel inferior. Consult a psychologist for further help.If you consider my help, Please post a direct query with every details. I shall prescribe some psychotherapy techniques to practice.I assure good relief. Hope this answers your query. Available for further clarifications. Good luck."
},
{
"id": 122220,
"tgt": "Suggest treatment for uric acid problem which causes knee pain",
"src": "Patient: i am having uric acid problem from last 3 years but not getting proper solution from doctors I need a proper solutions is there any hospital for this to solve my problem, I am having pain in my legs because of this problem i m only 26 and take proper diet no oil less protein foods etc. Doctor: Hello,I read carefully your query and understand your concern. Your symptoms seem to be related to gout arthritis. O suggest using Allopurinol to lower the uric acid level for two months.After the treatment you should check the uric acid level again.I also suggest using anti inflammatory medications such as Ibuprofen to relieve the pain. I suggest to continue using a specific diet with less protein. Hope my answer was helpful.If you have further queries feel free to contact me again.Kind regards! Dr.Dorina Gurabardhi General &Family Physician"
},
{
"id": 97245,
"tgt": "Suggest treatment for dog bite",
"src": "Patient: hi, i m sandesh, on 18-nov-12 at early morning,the street dog has biten me on left leg.after that i have taken anti rabies vaccine of 1 doessage still i have to take 2 dossage with certain time limits.does it is sufficient.what are the food precaution to be taken.please answer me. Doctor: Hi and welcome to HCM. Thanks for the query.there is no food precaution,would should have been well cleaned and antirabies should be given as recommended. it is enough to prevent possible infection.Wish you good health. Regards."
},
{
"id": 211481,
"tgt": "Tearing feeling in chest and pain in arms. History of hypochondria and anxiety. Reason?",
"src": "Patient: Hi my name is Evan. For the last month I have had a problem with reflux. My diet is not good lots of fatty foods, acidic drinks. I started a new job last week and have till today have had a tearing feeling in middle of my chest as well as everynow pains in arms or legs. I do have a history of hypochrondria and anxiety. Could my worrying about my reflux condition and stress of new job cause this feeling in my chest. I have been tested this week and my blood pressure is normal. Please help I am worried as it is near my left breast and worried about my heart. I also feel pressure on my chest when I lay down or in the night. Doctor: Hello, Thanks for the query to H.C.M. Forum. I would come up with these possibilities and measures for these symptoms , these includes.1 The first possibility is of hyper acidity ( gastritis ) , diagnosis can be confirmed by upper G I endoscopy.Try raising the head of your bed about 4 inches with blocks.It might also help to avoid eating or drinking for 2 hours before you lie down.To help control the stomach acid one should not drink alcohol or drinks with caffeine in them or eat chocolates or spicy or greasy food.Also take some antacid but if symptoms are severe then you may need drugs like proton pump inhibitors.2 The next possibility may be coronary artery disease due to tension and anxiety . Diagnosis can be confirmed by E K G & ECHO of heart.3 Tension and anxiety develops acute pain behind sternum due to esophagitis .Diagnosis can be confirmed by clinical as well history of illness by a physician.In my opinion consult a physician and get his opinion.Good luck. Dr. HET"
},
{
"id": 174979,
"tgt": "What causes hard stool in children?",
"src": "Patient: My 6 months old son will not stop cring he is having truble pooping and his tummy is hard we have tried everything I have even gone to our dr and told us to us suppositories and he poops and is fine for an hour and starts to cry the only time he isn t really cring is when he is eatting and he isn t hungry bc when he is full he stops but still cries he has a bit of a cold and is only really happy in his swing Doctor: HIWell come to HCMIf the child is crying in this age of 6 months then this may not be always due to abdominal problem but this could be something else, some time this could be non specific if the child is well comfortable with swing then there might not be any problem, some time this could be due to crying tendency, putting suppository was not good idea, take care."
},
{
"id": 79753,
"tgt": "What causes pain moving across chest?",
"src": "Patient: Having pain across chest below chestplate,somewhat of a burning feeling. no pain in jaw or arms or any anywhere else. BP is usually 120/80. now 136/83 . pulse rate usually around 65bpm. now around 55. pain about 7 of 10. Pain was coming and going but now somswhat constant Doctor: thanks for asking your questioni completely understand your problemno need to panic, the most common cause in young person is gastritis or peptic ulcer which can cause central chest pain.for this you can use a proton pump inhibitor like pantoprazole and you can consult a pulmonologist or a general physician .if gp wants to get an ecg done to rule out other causes , that would be betterthanks/ regards feel free to ask more questionsmay god bless you with good health"
},
{
"id": 166542,
"tgt": "How can upper respiratory tract infections be treated?",
"src": "Patient: My 4 year old. Doughter has had several upper respiritory. Infections to the point it's more often than not. We have noticed the small piece at the top of her throte is attached more on one side than the other so it's off to one side. Quite a bit. Most of the time as soon as she goes to sleep she starts coughing a lot and is bringing up a remarkable amount of clear mucas. Any ideas Doctor: the clear mucus indicates mostly allergy and post nasal discharge .that coukd be due to improper nanagement of past infection or if she is diagnosed before as having bronchial asthma that will need to be reassesed from her physcian to change her treatment plan and your role is to monitor her general condition , and increase warm fluids and decrease milk and banans at night because they increase the thickness of mucus"
},
{
"id": 183098,
"tgt": "What causes the disappearance of dimple after wearing braces?",
"src": "Patient: I went to the dentist to get fillings and when the swelling went away I saw that my right dimple has disappeard . And I thought it would come back and then I got braces I ve have them for about a week now and my dimple still hasn t returned will it ever come back ? Or it s gone forever ? Doctor: HelloDue to teeth movement there will be slight change in your facial profile but it ll not occur suddenly or just in a week.I would suggest you to disscuss it with your treating dentist."
},
{
"id": 152415,
"tgt": "Is a mass in the groin area post mesh implantation a cause for concern?",
"src": "Patient: my fiance has a lump by his inner thigh close to the side of his penis. it almost looked oblong. He also has a round lump in upper stomache. We thought it might be a hernia because he just had a mesh done earlier this year . He went to the dr and had a mri. Well today he had a fo;;ow up and the cd from the mri. The dr checked him and said it is not a hernia and that it is a mass. He is scheduling a CT so they can see it better he said. im worried and i know he is to. A couple of years ago my fiance had cerbral stroke. Im hoping that doesnt manifest again. Should we be concerned? im trying to type this before he gets back from the store. i dont want him to know im worried Doctor: Hi, Scrotal masses are common findings in primary care and can develop from all anatomical structures of the scrotum. In this report, we present the case of an unusual testicular mass caused by the migration in the scrotum of a mesh used for inguinal hernia repair. Hope I have answered your query. Let me know if I can assist you further. Regards, Dr. Monish De, Oncologist"
},
{
"id": 41366,
"tgt": "What is the duration of treatment in case of IVF?",
"src": "Patient: i am Gopikrishnan[35 yrs] and my wife is28 yrs old.and married 4 years.my wifewas taken 4 times iui.but no result. so we are planing go to ivf treatment. i have lesscount and my wife is having irregular period.if we want to do IVF with donor egg, how long will it take us to be there for the entire treatment,thanking you,d.gopikrishnan. Doctor: hello,thank you for your query,first of all i have to tell you that if your wife have normal ovulation then you can do ivf by using her egg..irregular menstruation can be cured by medications properly.and the time taken for this procedure is total 4-6 weeks for a cycle..you have to spend a day for sperm and ovum extraction.on fifth day generally the embryo transfer is done..in case of couple age less than 35 chance of success is 30-35%. you have to consult an infertility specialist for this procedure.hope this will helpful.best of luck..regards."
},
{
"id": 167428,
"tgt": "Suggest remedy for redness in newborns",
"src": "Patient: my 5 week old baby seems to have some red small pimple like things on her cheeks and chest in her 3rd week. her soft skin is very rough to touch now. i wash her face twice with plain water. is there any remedy for the redness to subside and soft skin to come back? Doctor: avoid clothings purchasrd newly without washclothings should be properly rinsed with plenty of water.you can start multi vitamin drops"
},
{
"id": 137526,
"tgt": "Can weight loss cause popping out of ribs?",
"src": "Patient: I recently lost about 75 lbs over a 10 month period (maintaining for 6 months), and I ve been having ribs pop out of place a lot lately. It s very painful, and I was wondering if it could have anything to do with the weight loss or if it s something I should have looked into further? My chiropractor has adjusted at least 5 ribs (all different ones I think) back into place, in the last 2 months. The first one pinched a nerve and messed with my right shoulder blade. He doesn t really seem concerned, but this NEVER happened to me before, and it seems like it s becoming more frequent. Should I be worried? Thanks in advance for your advise.Vanessa Doctor: Dear Vanessa,I can understand your concern, I feel you are worrying more, than what is required, if you had no such symptoms before loosing weight, than these can be attributed to your weight loss, but let me assure you that, this will get ok if you start regular exercises, to strengthen your muscles of the chest, so consult a physio and do chest exercises, also, aerobic exercises are good for chest muscles, and they can be done as tolerated. also keep good diet.I hope you heal yourself,Take care,Thanks"
},
{
"id": 111595,
"tgt": "Could back pain be a result of muscle strain or ligament tear?",
"src": "Patient: Hi, i have serious back pain.The center if my spine to be exact.It happened while lifting( which i do every day) a 30 lb concrete cylinder.I uasuallu pick in up from waist length.As i was to put it back down, thats when i felt the pain.Now hurts more than when it happened.Not sure its a muscle strain, tear or ligament isssue or disc problem.I cant sit or stand straight.Please help. Doctor: In my opinion all you need to do is give you back a little rest. For immediate relief have some pain killer along with a muscle relaxant and have proper bed rest for 2 weeks. Do not lie down on soft mattress use hard one instead, do not sit for longer durations and do not travel long distance for 2 weeks . You can also apply some good anti inflammatory pain killer gel on your back. If the pain increases do some hot fermentation locally. After 2 weeks start with some back strengthening exercised. It will help you build you back muscle but do not over look at the symptoms right now. As it is still early days for you and you can be fine and back in action with these small things. Good luck"
},
{
"id": 110003,
"tgt": "Suggest medication for severe back pain in a diabetic patient",
"src": "Patient: Iam Sivaram, My Mother (Name Ammu Iyer) is having severe back pain. She is 60 yrs old and undergone brust cancer treatment in 2008. She is having BP. Diabetics were there when she was undergoing tretatment for Cancer which was brought in to normal.... what medicine should be given to her for an immediate relief? Doctor: Hello, Thanks for your query. After going through your query I came to know that your mother are suffering from chronic backache. Looking to her age I consider it to be lumbar spondylosis. Rest in position of relief, Cartisafe D once daily, NUROKIND GOLD ONCE DAILY and analgesics (DICLOFENAC 100 MG SLOW RELEASE TABLETS) give relief. Sometimes strong analgesic(such as ultracet three times a day after meals) is required. I think she will need this. OMEPRAZOLE before meals prevent acidity caused by analgesics. Sometimes vitamin D deficiency can aggravate this so serum vitamin D test is advised if it is low than vitamin D supplementation will be required. Avoid long continuous standing. Sit in a straight posture. Eat milk, fruits and green leafy vegetables daily. You need to done MRI TO RULE OUT other causes. You can discuss with your treating Doctor about it. I do hope that you have found something helpful and I will be glad to answer any further query. Take care."
},
{
"id": 65495,
"tgt": "Suggest treatment for large painful lump on collar bone",
"src": "Patient: Hi There, About 2 months ago I collided with another player in basketball smashing shoulders. Though I felt a crack up my neck xrays showed no fractures. I have a large lump on my right collar bone near the middle of my body and get alot of pain up and down my neck and also shoulder. Should i see a physio or chiropracture? Thanks Doctor: Hi dear thanks for the query on HCMYou need to meet the physiotherapist for you sports related activities...If there is no improvement or there isnpain better consult the orthopedic doctor for the needfulThank you"
},
{
"id": 181951,
"tgt": "What is the growth on the gums indicate?",
"src": "Patient: I had growth removed from my gums today and the dentist who removed it said he had never seen anything like it in 30 years of being a dentist he sent it off for biopisiy no pain had it for several months had x ray of the area about a month ago and nothing was found any thoughts? Doctor: Thanks for your query, I have gone through your query.The growth over the gums can be a pyogenic granuloma or peripheral ossifying fibroma. Nothing to be panic, consult a good oral physician and get a second opinion. Your dentist might have not seen such cases, so take opinion from another physician. Once the biopsy report comes we can treat further, if the lesion reoccurs.I hope my answer will help you, take care."
},
{
"id": 206241,
"tgt": "Suggest medication for depression, chronic anxiety and panic attacks",
"src": "Patient: I moved from Arkansas to IL. but the only phycritrist is at Blessing Hospital. I was being treated for depression and chronic anxiety with panic attacks. So far off my reg. meds I was on for 14 yrs. because the Doc here doesn t like what I was on. He doesn t know me. He is treating me for Bipolar and not really treating me for anxiety and panic attacks. My electroshock, shaking and can t calm my brain down to one idea is getting the best of me. I ve been here 5 mos. and nothing is working to my advanage of having a full life. I was on Trazadone 300mg. at night/ klonapam 2mg at night, if needed 1mg during the day. Now Trazadone 200mg, mirtazapine 15mg, Lorazepam .5mg. I need another doc that cares to read my other docs reports. He says now that I have an addictive behavior because I told him I need a higher mg of Lorazepam or please put me back on Klonapam that I haven t been able to take since May. What is a patient to do when the only Phycritrist available in town treats you like a lier or addict. I m 56 yrs. old and would like to beable to do the things I used to do before I moved to IL. Doctor: Hii can understand your concern.Anxiety and panic can part of depression and treatment of depression will improve anxiety.Lorazepam and klonapam both are sedative and anxiolytic which will help to reduce anxiety and improve sleep.Dose adjustment require according to need. 5 mg lorazepam is ideal dose. so no need to raise it further.But Higher dose may indicate tolerance or dependance. So It this case need to change drug if dependance occur.Trazadone is very good molecule for anxiety and depression.so do follow his advise because excessive requirement may indicate addiction and also may cause harm to your body.You can take opinion of another psychiatrist in near by town.Feel free to ask me.Thank you.Get well soon."
},
{
"id": 11370,
"tgt": "Suggest treatment for hair fall and less hair growth",
"src": "Patient: hi! i am 20 yrs old,female..i am having great problem regarding my hair..i am having hair problem,though they dont fall more than 20-25 per day,but since last 3-4- years,hair regrowth is more or less zero..so my scalp is becomming sparse day by day...what to do??? Doctor: HIWell come to HCMHere in your case, because you are female so hormone imbalance could be the cause of this, and this need to be check out, some endocrine disease also need to be rule out, reproductive hormone profile is must, thyroid function test is also necessary, rule out the fungal infection of hair, till then take great care in diet, have a fresh fruits, no stress, hope this information helps."
},
{
"id": 192112,
"tgt": "What causes low libido in men?",
"src": "Patient: hi i'm 25 yrs old and my whole life i have never felt any pleasure sensation in my penis during sex, all i feel is the warmth and wetness but no pleasure. I love women so i dont understand, but i will admit that since 12 or 13 yrs old i have masturbated several times daily with no lube just bare hands what can i do or try to change my problem? Doctor: Hi, Since you have masturbated several times from 12 years of age. The nerves that supply the penis would have undergone too much wear and tear which leads to reduced pleasure or desensitisation. Sometimes when we are regularly masturbating penis might feel numb. I suggest you to avoid coitus for a month, and then try to stimulate your penis through oral sex and intercourse. It might make a difference and get your pleasure back. If it does,you can regularly maintain adequate gap (days) between every intercourse session. Take care. Hope I have answered your question. Let me know if I can assist you further."
},
{
"id": 116350,
"tgt": "Is it safe to have Aspirin to reduce inflammation and thin blood?",
"src": "Patient: Hi. I have the MTHFR gene, and am having a hard time treating it. I got on an Optimal Prenatal with methylated vitamins ( I am not pregnant ) Should I also be taking baby aspirin to thin blood and reduce inflammation? I feel like I bruise easy already. I ve changed my diet, and am taking some good supplements but still struggle with inflammation. Doctor: Hi, dearI have gone through your question. I can understand your concern. Aspirin is very useful in reducing inflammation. It is also useful as blood thinner. However thete is slight risk of bleeding. You should keep that thing in mind. Consult your doctor and take treatment accordingly. Hope I have answered your question, if you have doubt then I will be happy to answer. Thanks for using health care magic. Wish you a very good health."
},
{
"id": 205243,
"tgt": "Suggest treatment for ADHD",
"src": "Patient: Hello, we have a 14 year old son who doesn't seem to have a filter when it comes to his mouth (no cursing)just say things that normally you would mumble or keep to yourself when speaking to your parents (like y'all stupid..hate this place etc). He was diagnose with ADHD, but after reading information about this it seems that may not be the case. He was prescribed medication, but we are not wanting to take that route (afraid of side affects). My question is should we get a second opinion to see if something more is going on? YYYY@YYYY Doctor: hithanks for using HCM.yes, you should get a second opinion regarding your child's behavior as why is doing in that way. He might be or not be suffering from ADHD that can only confirmed by evaluation of child by a psychiatrist and preferably a child psychiatrist. thanks"
},
{
"id": 132398,
"tgt": "How long will it take to have a complete cure after a ankle fusion surgery?",
"src": "Patient: hi i just had ankle fusion surg mon. tibia and ankle fused also bone graft i also had 3 torn ligaments 2 chipped bones and a centimeter and a half min hole in my ankle. its a non weight bearing so i keep my foot over heart all day and im wondering how long i should stay off foot so any guideance or advice. recovery to. i use 2 be athletic and im told i can never run again. never saw dr after surg but suppose 2 see him next week thanks kevin Doctor: There is a strong recommendation to stay off your ankle until you follow up with the physician who preformed the surgery."
},
{
"id": 215642,
"tgt": "Is Kratom safe to take for chronic pain?",
"src": "Patient: Hello, i am a 40 year old male I suffer from numerous medical conditions. Here s a quick list, frontemporal dementia, epilepsy, chronic fatigue, and I also have 5 herniated disks. For years I was taking prescription pain meds but for the past 3 years i ve been taking kratom instead. Some family members have been telling to stop taking this herb. Please advise, Thank you, Randall! Doctor: Hello, Kratom is not that much effective for pain conditions. You can consider analgesics like tramadol or gabapentin for pain relief. Hope I have answered your query. Let me know if I can assist you further. Regards, Dr. Shinas Hussain, General & Family Physician"
},
{
"id": 79388,
"tgt": "What causes chest pain while using crutches?",
"src": "Patient: Hi I broke my ankle and sprained my wrist two weeks ago.they put a cast on me now week ago but I can t really use crutches because of my wrist. I still get on crutches and had the usual aches from using them. I noticed the second day of using them that I would get a chest pain. I get the pain mostly when I am getting ready to stand up. It s not pain from Gerd it s a pain in the middle of my chest and it is very painful. What could be causing this Doctor: Thanks for your question on Health Care Magic. I can understand your situation and problem. By your history and description, possibility of musculoskeletal pain is more in your case. But better to rule out cardiac cause first. So get done ecg and 2d echo. If both are normal than no need to worry for heart diseases. You are mostly having musculoskeletal pain. So take painkiller and muscle relaxant drugs. Apply warm water pad on affected areas. Avoid movements causing pain. Don't worry, you will be alright. Hope I have solved your query. I will be happy to help you further. Wish you good health. Thanks"
},
{
"id": 113116,
"tgt": "Fluctuating haemoglobin levels. Having excruciating lower backpains radiating to upper back. Suggestions?",
"src": "Patient: I recently had my hemoglobin levels checked and the first reading was a 6.6 and so they did another. The second one was 11.8. I am a 23 year old female by the way. So I know that my iron level has to fall between those two readings. After that doctor visit, I have been experiencing excruciating back pains. They began in my lower back but are now mid way up my back. I thought it was my menstrual cycle but that has now gone off and the pain is still here. Any advice?? Doctor: hi, anaemia itself can cause any kind of body pain and back pain , but if pain is persistent for long duration kindly get eaxamined yourself by orhopedic surgeon and get an xray of back done thanks"
},
{
"id": 217001,
"tgt": "What causes pain and bruise under nose?",
"src": "Patient: Last 2 days I have felt pain on my face under my nose. I rub over it with my finger and it feels bruised. My front teeth (2) have been capped over 3 years ago. I floss around them as well as my daily routine dental care. But this pain has me worried. I do have allergies since spring has started, but I have never had this type pain. What could it be? and what should do? Doctor: Hi..Can understand your concern..As per your complain pain and bruised feeling under the nose with the history of capped tooth can be due to any infection in the tooth as there can be dissolution of cementing material between the cap and tooth or I'll fitting cap can lead to seepage of food debris into the cap and can cause infection in tooth which can progress below the root tips of tooth and cause pus formation, pain and bruised feeling..Inflammation of gums can also cause cause such symptoms..In case of activation of Sinusitis or allergy can also cause pain in the region below nose so it will be better to consult a dentist and get an x Ray done to rule out for any dental infection..In case it is not ruled out you should consult a physician to rule out allergic reaction or sinus inflammation..Hope this information helps..Thanks and regards..Dr.Honey Nandwani Arora.."
},
{
"id": 103650,
"tgt": "Nasal congestion, difficulty breathing, sneezing attacks. Due to allergies?",
"src": "Patient: Hi Dr, My 8 year old son has been having stuffy nose for the last couple of weeks. Every day morning he gets up with breathing issue and after sometime he starts sneezing . He atleast sneezes 8 - 10 times within 10 - 15 mins. Not sure if he is allergic to something. Could you tell what s causing it and what s the easy remedy? Doctor: Hi and thanks for the query,Its important It s important to know whether the child has presented with such symptoms before or not, any family history allergies or asthma. What you describe is compatible with allergic rhinitis or asthma. A chest X ray might be required. Clinical evaluation to exclude any other conditions like nasal polyps through a careful clinical evaluation is important. Measuring blood eosinophil levels if raised shall be indicative of an allergic component. Immunoglobulin E levels f raised are almost diagnostic of allergic rhinitis or an allergic state. My suggestion is you book an appointment with an ENT specialist for a proper planning and evaluation.Thanks and best regards,Luchuo, MD."
},
{
"id": 204115,
"tgt": "What course of treatment should be done for fatigue and concentration difficulty in an ADHD patient?",
"src": "Patient: My husband Robert is 65 years old and was diagnosed with ADHD four and a half years ago. He has been working on his high school diploma and has noticed his attention span getting worse over the last seven months. He has been under much stress trying to learn Algebra. His body shakes,he cannot concentrate, has mental and physical fatigue. Sometimes he has blurred vision. What do you recommend? Thank you, Donna Doctor: Hello, Adult ADD is a cause of concern. Your husband is 65 and therefore the treatment is always psychiatric as well as could use psychotherapy. The combined treatment can do good for him. Hope I have answered your query. Let me know if I can assist you further. Regards, Dr. K. V. Anand, Psychologist"
},
{
"id": 176477,
"tgt": "What is the treatment for small welts on buttok in a 16 month old?",
"src": "Patient: 16 month old has a large welt approx. 3 around on her bum. I first noticed small welts about ten days ago but they would come and go. She has not eaten anything different that I am aware of. Her face is a bit red around her cheeks and is irritable. Oh and the welts turn white when pressure is applied. Doctor: I think this is benign cystic condition and not much to worry about this!an FNAC test can confirm this and if antibiotics fail then a surgical excision is necessary for complete cure!regards,"
},
{
"id": 191873,
"tgt": "What cause hot flash on face in a diabetic?",
"src": "Patient: My 91 year old father complains that hid face gets really hot when he lays down (especially on his right side). He can t sleep so sits up on his recliner most of the night. Has gone on for months. History or diabetes (on insulin), atrial fib, pacemaker, bladder/prostate cancer. Doctor: Most commonly, they arise as a result of a dramatic drop in testosterone levels in men who have their testes surgically removed (as part of the treatment for prostate cancer) or who are taking medications that counteract the effects of testosterone."
},
{
"id": 210108,
"tgt": "Suggest medication for anxiety and panic attack",
"src": "Patient: hi my wife used to complain about the noise of hoovering from next door,or like she used to think that next door guys is keeping eyes on her throughout the day and later on she starts forgetting things like cooking shopping like most of the social life she is under treatment now she dont complain about noises and the guys following but keep forgetting things to do in just a seconds..is she will be cured? Doctor: Hello,The symptoms of your wife are not related to anxiety or panic or depressive disorder. They are just part and parcel of her psyhosis. She is recovering from her illness and the voices and suspiciousness have reduced, but the distraction or inability to concentrate is still there. Many patients when get free from their positive symptoms ( like hearing voices etc), develop negative symptoms- memory problems, lethargy, inability to work properly and lack of motivation for most things. This also gets better with time and treatment. Patient should be motivated for doing all kinds of household works, and encouraged by others. She should bot be nudged and insulted for her declined performance ( negative comments only deteriorate them). Then gradually, with continous efforts, she will definitely become better.Donot worry, most of the time there is no permanent memory loss.Wish you good luckDr. Manisha GopalMD psychiatry"
},
{
"id": 132922,
"tgt": "Why am I having pain in my left ankle?",
"src": "Patient: Today had pain in my left ankle and just within the past hour I noticed an indentation in my lower left leg in front of my calf area. I applied slight pressure to area and felt as if there was no fatty tissue like skin against bone. Was not very painful with pressure applied just a slight painful sensation. Doctor: Your age please?what you have described as \"Indentation\" is possibly 'Pitting Oedema'.This treatment is being suggested on bases of the information provided.I would like to examine & investigate you in detail.Rule out Diabetes & hypertension. Any way it may be tried,--. Dolokind Plus (Mankind) [Aceclofenac 100mg +Paracetamol 350mg] 1 tab. OD & SOS. X 5days.--. Caldikind plus (Mankind) 1 tab OD x 10days.(You may need help of your local doctor to get these medicines.)--. Fomentation with warm water. Avoid direct flow of AC or Cooler.--. Sleep on a hard bed with soft bedding.--. Keep a pillow under the legs at night.-- .Do mild exercises for Knee & Legs. --.(Take help of a physiotherapist or visit www.drncgupta.com). --. Do not ignore, let it not become beginning of a major problem.Do ask for a detailed treatment plan.If no relief in 2-3 days, contact me again. (phone.91 000000)Kindly make sure, there is no allergy to any of these medicines. (Contact family doctor, if needed).For emergency treatment visit nearest hospital.-Hope I have answered your question, If you have any further question, I will be happy to help.-Kindly rate the question.-Wish you a quick recovery & good Health."
},
{
"id": 10369,
"tgt": "Suggest treatment for hair loss in children",
"src": "Patient: My daughter is 13 years old and is a normal, healthy teenager. Her hair strand has always been thin and light, and she keeps it shoulder length. I noticed six months ago that her hair even after washing quickly become oily and and a bit smelly around the crown. Thinking back now , about two months ago she complained that a lot of hair was falling out when she washed it in the shower. This last week she has developed a bald spot about the size of a dime and is very upset and embarassed. She likes to play with her hair when reading / studying but I don't believe this is the cause. Should I go to our family doctor or buy something obver the counter first ?? Doctor: Hello and Welcome to \u2018Ask A Doctor\u2019 service. I have reviewed your query and here is my advice. She has to be examined to confirm if the bald spot is alopecia areata. An autoimmune condition.there are many treatments available for this condition and the hair grows back. Hope I have answered your query. Let me know if I can assist you further."
},
{
"id": 19696,
"tgt": "Could gargling with salt water causes high BP?",
"src": "Patient: I have high blood pressure that I keep controlled by meds, diet and exercise. I've had a tooth extracted and was told to gargle with a salt solution 5x a day, especially after eating. My blood pressure has now jumped. Could there be a correlation? Thanks. Doctor: Dear patient thank you for your question. If you are not swallowing the salt water no need to worry but there is a possibility to swallow little amount of salt water while gargling each time that may raise your blood pressure... Always benefits out weigh risks... If symptoms not improved consult your doctor.."
},
{
"id": 143113,
"tgt": "What causes feeling of restlessness and lightheadedness along with numb toe?",
"src": "Patient: For the past few days I have been feeling restless and mildly depressed. Before that I have had numb toes (right foot) and still have a cough from a cold I had a few weeks ago. In the past two days I ve been feeling light headed and as if I weigh an immense amount less than usual when I weigh the same. I am eating healthy but I am a smoker. Female, 19. Doctor: Hello!Welcome and thank you for asking on HCM!Regarding your concern, I would explain that your symptoms could be related to inflammation or chronic anemia. That is why, I would recommend consulting with your attending physician for a careful physical exam and some tests: - complete blood count for anemia- kidney and liver function tests- blood electrolytes for possible imbalance- fasting glucose- PCR, ESR for inflammation. I would also advice checking your blood pressure. Low blood pressure could be the cause of your troubles. Meanwhile, it is important to have some rest for a couple of days and take plenty of water to stay hydrated. Hope to have been helpful!Kind regards, Dr. Aida"
},
{
"id": 98724,
"tgt": "Suggest medication for dust allergies, leading to either blocked or runny nose",
"src": "Patient: My wife has had constant problems with allergies or issues with dusty christmas decorations. She goes between having a stopped up nose or it is draining like a river. What over the counter meds should be considered? We have a humidifier in the room and run a air filter in the room where she is sitting/sleeping. Doctor: Hi,Looking at your wife's symptoms, she is probably suffering from Allergic rhinitis (Inflammation of nose due to Allergy). Common causes of allergic rhinitis include the allergens like fecal matter of the house dust-mite, animal proteins from domestic pets, and industrial dusts and fumes. It would be advisable to do Allergic test through which it would be possible to find out that which substances (Allergens) your wife is sensitive and causing symptoms.The management of allergic rhinitis consists mainly of (1) environmental control measures and allergen avoidance, (2) pharmacological management Environmental control measures and allergen avoidance involve both the avoidance of known allergens (substances to which your wife is found sensitive-which aggravates her symptoms) and avoidance of nonspecific, or irritant, triggers. For dust mites, covering the mattress and pillows with impermeable covers helps reduce exposure. Bed linens should be washed every 2 weeks in hot water to kill any mites present. Thorough and efficient vacuum cleaning of carpets and rugs can help. Dust mites thrive when indoor humidity is above 50%, so dehumidification, air conditioning, or both is helpful. Control measures for mold allergy focus on reduction of excessive humidity and removal of standing water. For animal allergy, complete avoidance is the best option.Exposure to smoke, strong perfumes and scents, fumes, rapid changes in temperature, and outdoor pollution can be nonspecific triggers in patients with allergic rhinitis. Consider avoidance of these situations or triggers if they seem to aggravate symptoms.She might find relief by taking following medicines:Steroid Nasal spray like Fluticasone propionate or Mometasone-helps against allergy and reduces inflammation, Oral Decongestant like Phenylephrine and Nasal decongestant drops like Xylometazoline - they help to decongest the nose and sinus cavity and helps in nasal blockage (if you are not suffering from Hypertension) Antihistaminic like Loratidine(Claritin) - helps against Allergy,Mucolytic - like Ambroxol - it will help in breaking to mucus secretions. Saline nasal spray-helps to moist the nasal cavity and removal of allergen particles from nose. Also do steam inhalation.Hope this answers your question. I would be happy to answer if you have any follow-up questions.Regards,Dr. Riddhi Shah"
},
{
"id": 41219,
"tgt": "Does Astymin help in conceiving successfully?",
"src": "Patient: Good morning Doc i am 33yrs been married for 1 yr now with no sign of concieving. i had a fibroid section 2yrs ago. i have a gynea looking after me though , last month i was made to blow my tubees and as of now no sign of preg. i am worried i need help, A friend suggested i start taking Astymin is it right. Doctor: Hello, in my opinion get simple blood tests done like TSH and prolactin and Try intercourse on alternate days from day 10 to day 18 of your menstrual cycle to have best chances of conception. Alternatively take ovulation induction drugs to stimulate multiple eggs in single cycle and thus increasing chances of conception even more.In case you have any questions in future you can contact me directly on http://bit.ly/drmanishajain"
},
{
"id": 123926,
"tgt": "What is the blue bruise line at the middle of my upper leg upto knee?",
"src": "Patient: i have a blue bruise line starting at the middle of my upper leg (quad) and ends just before my knee. The line is straight and about a quarter of an inch thick. i did not bang into anything etc but i did just go for a run. It hurts a little bit at the top when i press on it. Doctor: Hello, It may be a linear rash due to the impact due to the stitching point of your dress. It may be cutaneous larva migraine or urticaria. But until the examination is done it is difficult to say what it is. Apply moisturizing cream. Please consult your physician he will examine and treat you accordingly. Hope I have answered your query. Let me know if I can assist you further. Take care Regards, Dr. Penchila Prasad Kandikattu, Internal Medicine Specialist"
},
{
"id": 19655,
"tgt": "Suggest remedy for pain in the right side of my chest",
"src": "Patient: i have been having pain in my right side of my chest under my breast bone by my ribs it hurts when i lie down at night on my side or on my back it also hurts when i try to get up from lying down and when i breath i have tried co-codamol it aint doing nothing i have never had this pain before what could this be? Doctor: it is most probably skeletomuscular disorder...take tab lyser d twice for few days ...to rule out CAD...please get ECG and chest x-ray done ..and get it BP done ..if any problem u can contact me at drvarinderjoshi@gmail.com"
},
{
"id": 158264,
"tgt": "Suffering from cancer. Recommended further treatment. On morphine. Further?",
"src": "Patient: My mother 44 year old suffering frOm cancer for the last 8years now 4stage is started, deadly cancer cells spreaded almost every were in her body.the hospital from where the treatment was given now recommended futher treatment through Paliative care. 6 days left we started morphine but still headache and other related pains are there . Yesterday onwards vomiting started so we are not able to give morphine . I want two know lifespan of such patients who have started morphine Doctor: Hi and welcome to Healthcare magic.Thank you for the query.Lifespan doesnt depend on morphim intake ,it depends on severity of cancer spreading and cardiorespiratory function. So this is hard to predict without precise findings. BUt 4.stage and palliative treatment indicate that this is very serious condition.Wish you good health. Regards"
},
{
"id": 222729,
"tgt": "What are the early signs and symptoms of pregnancy?",
"src": "Patient: I started my period on Dec 20th last month.....In November I started on Black Friday which I think was the 25/26.....Tomorrow is the 20th and I havent got my period yet; could I be pregnant? I had a large gush of white orderless mucos; I thought I started my period at first but when I went to go check in the bathroom, my panties were soaked in that white discharge. Also, I havent been taking care of myself at all. Im bulimic; i throw up, OD on laxatives and take diet pills, smoke cigarettes too.....did I harm my baby? If I am pregnant, can I stop my bad habits and still have a healthy baby? Help! Im scared! Doctor: Hello,In the current scenario, the ideal thing to do is to go for a pregnancy test at once to confirm/rule out pregnancy. You can go either for a HPT or the blood test. Excessive white discharge can be a feature of pregnancy as well as premenstrual. You are right in asking whether your habits can harm the baby if you are pregnant. Yes, they can and you should avoid them. If you follow healthy habits from now on, if you are pregnant, there is not much to be worried about possible harm to the fetus since you are in very early pregnancy. Regular follow up, proper diet, rest, medication and periodic sonograms are needed. Take care."
},
{
"id": 156246,
"tgt": "Does non itchy blotches in the legs suggest skin caner in patients with past hodgkins lymphoma?",
"src": "Patient: my grandmother has red non itching and non hive formed blotches all over her legs, she has a history of non hodgkins lymphoma x20yrs but has been in remission ever since, she went to the doctor yesterday, he ran lab test that were normal but said he had never seen anything like it and it looked like someone threw scolding hot water on her legs, she has to wait a month to see a Derm but could this be cancer coming to the surface of her skin from the lymphoma? Doctor: HI THANKS FOR YOUR QUERY. THIS CONDITION IS VERY UNLIKELY TO BE SKIN CANCER FROM NON HODGKINS LYMPHOMA S SHE HAS BEEN IN EXCELLENT REMISSION SINCE THE PAST 20YRS....THE BLOTCHES ON HER LEGS COULD BE DUE TO INFECTION,UNCONTROLLED DIABETES,NEUROPATHY,HOT WATER RELATED ETC....SO YOU TO CONSULT YOUR DERMATOLOGIST FOR EXPERT MANAGEMENT,;ALSO SUSPICION HAS TO BE HIGH FOR TUMOUR RECURRENCE.ALSO KEEP IN TOUCH WITH AN ONCOLOGIST.THANK YOU."
},
{
"id": 14334,
"tgt": "What cause rash like red spot appear on chest?",
"src": "Patient: hi, so a few days ago tiny red dots have appeared on the chest cleavage area. They are not itchy or have a burning or tingling sensation. Im wondering if it's a rash. Although, i recently started using a new body wash that may be the cause of the tiny red dots? im wondering why these dots have appeared. Doctor: HelloThank you for posting on HCM.From your description it seems like Miliaria which are tiny papules caused due to excessive sweating.Nothing to worry as its self resolving. For soothing effect apply calamine lotion twice a day.Cold compresses additionally will also help you.Sprinkle ample amount of good talcum powder over such areas to absorb excess sweat.Hope the information was useful.Take careDr Hardik Pitroda"
},
{
"id": 130895,
"tgt": "Is surgery required for broken knee cap?",
"src": "Patient: I fell on ice yesterday and BROKE the patella..knee cap...I can actually see the separation on my knee...xray shows same results. I am 76 years old and already have a partial replacement in that knee. Is surgery required? Right now the leg is immobilized until I see the doctor on Thursday. Doctor: In my opinion yes surgery is a must , i sorry for the bad news but there is no other choice Good Luck"
},
{
"id": 86920,
"tgt": "What causes pain in the stomach after running?",
"src": "Patient: Hi, I have had a gastric bypass last May and have lost 30kg and am now 87kg, 171cm tall. I have been running regularly and am finding that my stomach really hurts when I have finished.mmimstill have quite a bit of flab. Is there a way I can support it while running. Doctor: Hi.Thanks for your query.Read the history and understood your problem.It is good that you have undergone gastric bypass and have results of reducing 30 kgs. Flab will go slowly as the body will recoil it back slowly.You can use abdominal strappins meant for this purpose while supporting. Stomach can hurt a bit as there will internal scarring and fibrosis. Nothing to worry if there is no bloating, nausea and vomiting"
},
{
"id": 214244,
"tgt": "Suggest natural remedies for clogged sweat glands in upper thigh area",
"src": "Patient: I live in a very hot climate and keep getting clogged sweat glands, particularly in the upper inner thigh area. Is there any diet change, natural remedy or what can I do? Antibiotics take forever to work and by the time they do I have another one starting up. Doctor: Thankyou for the Question.t climateIn hot climate make sure of hydration of your body,take plenty of fruits and vegetables,drink minimum 8 glass of waterHome remedi..\u00a0\u00a0\u00a0\u00a0\u00a0Take fresh methi leaves. Grind it and make a paste. Apply on skin and leave it to act 10 minutes. Wash away with warm water;hope my answer will be helpfull for you.regards,Dr.Maheshwari"
},
{
"id": 146282,
"tgt": "Can a steroidal injection treat back pain at the lower thoracic levels?",
"src": "Patient: i m having severe back pain at the lower thoracic levels. I have a congenital deformity of L-5 S1 fusion, aka 5 1/2 . However now the SEVERE pain is in my back at waistline. Nothing relieves this. I m a patient care tech on a busy surgical floor full time, I m also a massage therapist part time. May there be a need just for a steroidal injection? Dawn Leigh White, LMT, NCTMB da Doctor: you need a work up if the pain is debilitatIng. they are usually X Ray, ct scan and / or mri of the lowerothoracic spine. only after that the management can be decided. steroid injections are usually not given in this part of the spine."
},
{
"id": 105858,
"tgt": "Are tenseness and tussle in the chest symptoms of asthma ?",
"src": "Patient: Hi.iam 20 year old man and i have allergy problem last 8 year.and last one week i have feeling like tenseness ,tussle in my chest . last friday i have done asthma test . the result was normal. now am taking homeopathy medicin.now i have feeling tenseness and tussle more.so what i have to do? is this going to asthma? Doctor: Hi,thanks for query.Tightness in chest and discomfort could be due to breathing problem or acidity.You need to visit your physician to see whether it is breathing problem or something else.Please take regular treatment for this.Avoid strong allergen like smoke,strong smell etc.Take light diet,avoid junk food and tobacco in any form.bye."
},
{
"id": 66692,
"tgt": "Need advice regarding a lump on left side of neck",
"src": "Patient: Hi, I have a lump under my skin on my neck, I just noticed it while I was massaging my neck. It s on the left side of my neck. It isn t tender to the touch and doesn t hurt when I press on it. It s a little smaller than the ball of my finger pea size. It s smooth to the touch and is moveable. I am worried that it could be serious, as anyone would with a lump. I have not other symptoms of any kind. Doesn t hurt to swallow. Could it be from sleeping wrong? It hurts on the left side of my neck when I turn my head to the right. That s the only thing I can think of. Doctor: Hi, If I were your treating Doctor for this case of solitary neck lump, I would come up with three possibilities, these include: 1.\u00a0\u00a0\u00a0\u00a0\u00a0lymph nodes due to some chronic disorder like viral infection, dandruff, throat/neck/ear infection, tuberculosis, lymphoma or even malignant diseases in elderly people!2.\u00a0\u00a0\u00a0\u00a0\u00a0The second possibility is of benign and simple conditions like lipoma or neurofibroma; there is nothing to worry about these!3.\u00a0\u00a0\u00a0\u00a0\u00a0The last possibility is of sebaceous cyst or some other cysts like branchial cyst or lymphatic cysts! Nothing to worry about these also!I suggest you to go for an FNAC and ultrasound of the lump for confirmation and to relieve your concerns!Hope this answers your question. If you have additional questions or follow up questions then please do not hesitate in writing to us. I will be happy to answer your questions. Wishing you good health."
},
{
"id": 14311,
"tgt": "Reason for rash through out the body?",
"src": "Patient: my boyfriend has a rash that starts on 1 arm almost heals and the moves to the other arm.this has been going on for 6 months plus.He may have pancreatic cancer.they have done biopsies to verify this just waiting for results.Nobody has given him an answer about the rash though.Can you help explain it? Doctor: Thank you for your question!From the way that you describe a migrating rash it could be erythema migrans, dermatitis or urticaria.Erythema migrans (Lyme disease) is caused by an infection after a tick bite. The rash maybe slightly itchy, it extends over two weeks, to produce another lesion in another site. Does he have the rash only in the arms? Has he experienced any of these symptoms? If yes he should take blood for serology.If it is urticaria the lesions are more like wheals... and also it could be dermatitis.Sometimes these can be paraneoplastic signs that come prior to an internal cancer,Anyway if a patient presents in my clinic with these symptoms I would check him with some patch test to exclude dermatitis and urticaria, skin biopsy and serology if the diagnose is unclear.I hope this helps!If you are unclear with the answer please try contact me again. Wishing you and your boyfriend good health!Dr. Migena GEGA"
},
{
"id": 202162,
"tgt": "Why is the head of my penis glossy?",
"src": "Patient: I have a gloss surface on the head of my penis that showed up over night. It looks like a blister but has no redness or pain. Moreover it is not raised like a wart. The skin surface is glossy. I have had it for five days. Do you have any idea of what it might be? Doctor: Hello .It could be seen in case of phimosis ( covered skin) inflammation \\ infection to the gland, and physical stress. Any how i suggest you to let a doctor examine and come to conclusion. Things like this can not be ignored. Do not hesitate if you have to ask. Hope it helps you. Thank you."
},
{
"id": 124784,
"tgt": "What could cause heaviness and achy feeling on both calves?",
"src": "Patient: I have heaviness and achy feeling in both of my calves. It does not hurt when I walk. Have had it for months, it s daily. Would like to know what is causing it. I do a Silver Sneaker s class 4x a wk., exercise. I no longer walk as much as I used to due to moving, climate. Doctor: Hello, As a first line management, you can take analgesics like paracetamol or aceclofenac for pain relief. If symptoms persist better to consult a general surgeon and plan for a Doppler scan. Hope I have answered your query. Let me know if I can assist you further. Regards, Dr. Shinas Hussain, General & Family Physician"
},
{
"id": 205566,
"tgt": "Suggest remedy for tension and headache with ESR 25mm/hr and 35mm/hr",
"src": "Patient: I have esr 25mm first hr. 35 mm second hr. actually i have problem go getting in tense quicky. with that i am getting pain immediately on the top of the head and chest also, chest because of gastric. So please suggest If if esr what to do. Doctor: Hello thanks for asking from HCM for your health related queriesI can understand your concern. You have persisting headache and problem in getting tensed quickly. The headache is present chiefly on top of head and is associated with chest and epigastric discomfort. The location of headache and association of headache with tension suggest more towards tension headache. But your ESR is in higher range and shouldn't be ignored. Raised ESR and severe throbbing pulsatile headache chiefly in temples can be the presenting feature of temporal arthritis, though your symptoms are different. Still I'd advise you to visit a doctor for evaluation and to rule out this. In tension headache low dose Amitriptyline, Nortriptyline etc drugs are used. These drugs help to control the headache and result in improvement. Apart from this for symptomatic relief drugs like Paracetamol or Diclofenac etc can prove useful. Please consult a doctor for detailed prescription. Thanks, I hope this helps you. Take careDr. Seikhoo Bishnoi,Psychiatrist"
},
{
"id": 172041,
"tgt": "Are symptoms of continuous fever and body ache normal in case of pseudohypoaldosteronism?",
"src": "Patient: my daughter is 5 years old. She has pseudohypoaldosteronism. For the past couple months she has continuous fevers and complains daily that her body aches. Her doctor runs test but nothing is showing up. I'm concerned there is still something going on. Any ideas? Doctor: In pseudohypoaldosteronism, there is a lowering of the plasma sodium and relative state of dehydration ... which can produce fever. Please give her extra salt and water to counter this problem. Electral water or something similar would be okay if you cannot give higher quantities of sodium chloride."
},
{
"id": 18884,
"tgt": "Suggest treatment for elevated cholesterol levels, fatty liver and fatigue",
"src": "Patient: i have been loosing weight exercise and diet combined for a long time and cholesterol is still high. i had a very small cancer removed from my liver- almost missed, i have a fatty liver i have gotten in shape, but still levels wont go down i am not eating foods high in cholesterol. i have fatigue and force myself to exercise i take a multiple vitamin almost everyday. hard to walk up hill legs are soar, my liver is fatty. i cant digest dairy or cholesterol. Doctor: Hello,Regarding your concern, I would like to explain that your daily physical activity does not seem to be really effective in lowering cholesterol levels. On the other hand, persistent cholesterol levels could be related to a liver disorder. Coming to this point, I would recommend taking a statin (atorvastatin, simvastatin) in order to help lower cholesterol levels. Increasing food rich in omega 3 fatty acids (fish, fish oil, olive oil, walnuts, etc.) can help improve your cholesterol levels too. Hope I have answered your query. Let me know if I can assist you further.Regards,Dr. Ilir Sharka"
},
{
"id": 37446,
"tgt": "Why my son get infected with cold so easily?",
"src": "Patient: I alongwith my son aged about 10 years reside at Ranikhet in district Almora(Uttarakhand). My son has problem with his nose. He easily gets cold and always contiues to drag sometimes phlegm and sometimes dry air upwards through nose for the last three /four years. I shall be highly obliged if you could please advise me about the line of curative treatment so as to get rid of this problem. Thanks. Doctor: Hello,I understand your concern.I am Dr Arun Tank, Infectious diseases specialists, responding to you.This are the common viral infections.It do not require any treatment except symptomatic treatment.This includes chlorpheniramine maleate for blocked nose and running nose, paracetamol for fever, you can also do hexidine gargle as and when required whenever you feel its viral infections.I will be glad to answer your further query. Thank you for your contact to health care magic.Wish you a best health.Thank you,Dr Arun Tank."
},
{
"id": 42669,
"tgt": "Why i am not getting conceived even though my periods are normal?",
"src": "Patient: Hi, m trying to conceive since the past few months n did every efforts but failed. My cycles are regular n last were over on the 6th of Jan so whats the best time to conceive? also me n my husband are under the Lycored tablets and me also taking 'Folinal' by cunsulting a doctor. Question is: Whats is the best time to conceive and what else i can do to be pregnant. Doctor: Hi,Thanks for writing to HCM.The best time to conceive is you should do intercourse during fertile period. It is day 12th to 16th for a cycle of 30 days. If your cycle is longer then you need to consult your doctor and get details of your fertile period.To increase chances you need to sleep in same position after intercourse for 30mins with pillow under buttocks. This prevents spillage of semen outside.If you are trying for more then 1 yr or your age more then 26 then discuss use of ovulation inducing drugs like clomefine with your doctor. This helps multiple follicles to mature and rupture and increase chances of pregnancy.Continue Lycored and folinal as it helps in producing good eggs in you and healthy sperms in your husband.Hope I have been helpful.RegardsDr. Ashish Verma"
},
{
"id": 167701,
"tgt": "What causes blood in stool in kids?",
"src": "Patient: Doctor Neelam: My 2 and a half year old son, Sukhvir is suffering from Cow Milk Allergy and is passing fresh blood in stool since he was 4 months old. We got him treated from everywhere but no solution and now doctors in Dubai Hospital who is Paediatric Gastroenterologist has diagnosed him with either Ulcerative Colitis or COw Milk Allergic, and given him Sodium Glycogate and some multivitamins and calcium medicines but still he is passing blood in stool. We have done all the blood test for CBC and the kid is still pale and suffering. PLease if possible, you can assist us and we can fly down for an appointment with you.. Thanks. Doctor: Hi...Greetings from Kanchi Kamakoti CHILDS Trust Hospital, Chennai.I went through the history points you have provided - I understand your concern.I was expecting that there should have been a history of cow's milk and formula feed (this too contains cow's milk protein) given to the baby. I feel that your baby is having cow's milk protein allergy. Usually babies grow out of this sort of allergy by 1 year of age. My suggestions for you - 1. If still on breast feeds - Mother should go off cow's milk protein completely. This means that you should avoid consumption of anything and everything related to cow's milk - like - milk/ curds/ ghee/ butter milk/ chocolates/ biscuits/ ice creams etc. Even while buying commercial food products, you need to see the ingredients and if they contain milk - do not consume them.2. Feed your baby only exclusive rice based feeds. But at 2.5 years it is highly unlikely that the baby is still having cow's milk protein allergy.I wish your kid a speedy recovery. If you need any future medical consultation and suggestions, I will be glad to help. You can approach me at the following link.Once the page opens there will be an option below my image as \u2013 ASK ME A QUESTION \u2013 click on it.In this way even in health care magic you can follow up always with a single doctor who knows the history of your kid completely. Please find the link below - www.healthcaremagic.com/doctors/dr-sumanth-amperayani/67696I request you to keep me posted about the recovery of the baby and follow of the case.Regards \u2013 Dr. Sumanth"
},
{
"id": 2760,
"tgt": "Will oosure help me in getting pregnant?",
"src": "Patient: i was suffering from pcod $ was trying to concieve for the past 4 years i m taking metformin but now doctor was said that it doesnt work for me so now i m taking oosure can i concieve this time is i m on the right way Doctor: Hi,I think the medicines which you are taking now is fine and sure can also help, but the thing is that you need some medicines for growth of your eggs. Also when egg becomes more than 17 mm one injection might be required for its rupture. After that you need to be in contact with your husband for next 2 to 3 days. Progesterone support may be needed for next 2 weeks.Hope I have answered your query. Let me know if I can assist you further. Regards,Dr. Khushboo Priya"
},
{
"id": 104195,
"tgt": "Does the use of seroflo inhaler affect vision? Can this damage be reversed?",
"src": "Patient: Does inhalation of Seroflo affect the vision . Since a doctor advised the same to me five years back I used the drug for a few months. There occured a problem with one of my eyes where the optholomologist says some internal lens of the eye is broken and can t be repaired. now again i feel asthmatic. Can I use the inhaler again? i am of age 36. Doctor: these inhalrs are sterids the ey problem can be due to this if you consult your doctor he can put you on nonsteridal inhalers and non steroidal antiallergics for prevention rather thn using same and getting further complicatiobs ad side effects"
},
{
"id": 155552,
"tgt": "Does incidental calcification with enlargement of pineal gland suggest tumor?",
"src": "Patient: Had a ct scan of my head last week because of metal taste in mouth and left side tongue numb for over a month and dizziness. The radiologist impression says incidental calcification and enlargement of the pineal gland, which may suggest the presence of a pineal region tumor. Recommend MRI with and without contrast for further evaluation. Does this mean I have a tumor? Doctor: Thanks for your question on HCM. In my opinion you should not worry much about cancer. Causes of pineal gland calcification are1. Hypercalcemia2. Hyperthyroidism3. Benign old granuloma4. Rarely malignancySo better to get done MRI brsin for better evaluation of pineal gland lesion.Also get done s.calcium level and s.TSH to rule out above mentioned causes."
},
{
"id": 109310,
"tgt": "Suggest remedy to relieve back pain on waking up",
"src": "Patient: i am missing a vertebrae in my back i wake up every morning with back pain i cant drive in a car for more than an hour without my back killing me, it is starting to get to me at work now so that i cant lift much or that when i do my back will throb i take aleve almost every morning and most jobs that i apply for i cant get bc i cant pass the physical test is there anything i can do i have a child to support or should i look into disability Doctor: U have to need stretching exercises ...so proper walking need daily and take dairy product also in ur diet...brisk walking also be added in morning walk ..."
},
{
"id": 18823,
"tgt": "Suggest investigations for uncontrolled hypertension",
"src": "Patient: my mother is a hypertensive patient currently taking the hypertension medication. recently her bp has been fluctuating from 140/85 to 220/12 in matter of hours. what tests should she do? the family physician is asking her to do CT scan, blood work, and etc. unfortunately, with no insurance, it is hard to get these done.. which one is most vital test that should be done first? Doctor: Hello and Welcome to \u2018Ask A Doctor\u2019 service.I have reviewed your query and here is my advice.You should start with Renal Artery Angiography as Renal artery stenosis. This may happen as a complication of longstanding hypertension and elevates the blood pressure to these numbers especially on standing up. It is very common.Hope I have answered your query. Let me know if I can assist you further.Regards, \u00a0\u00a0\u00a0\u00a0\u00a0Dr. Mario Aiad"
},
{
"id": 2502,
"tgt": "Will fibroids cause problems in getting pregnant?",
"src": "Patient: i just found out that i have several fibroids one of which is in a submucosal position measuring 2.4x 2.2 cm will this affect me from gettting pregnant i am 42 years old 198lbs height 5ft 8inches tall clean bill of health so far until today one child 19years of age Doctor: Hi, Fibroids is the most common benign masses in uterus. Just the presence of fibroids do not warrant removal. Submucosal fibroid will need removal, as it can interfere with baby attaching to uterus.Can be removed hysteroscopically.You already have risk factors - Age >40 yrs, Overweight & Multiple fibroids .Long gap since last pregnancyBest would be removal of fibroids , if not pregnant within 3-4 mths , plan for IVFAll the best"
},
{
"id": 175434,
"tgt": "What causes stomach pain in the morning?",
"src": "Patient: My 4 year old son wakes early every morning for the last five days complaining of a stomach ache and then throwing up what appears to be a clear mucus. We run a cool mist vaporizer during the night and limit his food and drink intake during the evenings to a more bland diet, and we prop him up on a pillow to sleep to keep his upper body a little more elevated - but all to no avail. He s fine throughout the remainder of the day. He went through the same ordeal last year and was seen by both the emergency room who simply prescribed a medicine to merely cease the vomiting, following by a visit to his pediatrician who guessed that it could be just post nasal drip. However, this year he doesn t seem to have any congestion which could be the cause. When he was two years old, he suffered from CO poisoning with complications arising after his recovery - stomach aches and vomiting frequently throughout the day and night. He was finally put on Prevacid, a proton-pump inhibitor, for the duration of six months which stopped the episodes. The physician had concluded that the CO poisoning had likely caused damage to his organs, leading to acid reflux. Yet with this previous experience, his current pediatrician is uncertain and doesn t give us any recommendations. I would appreciate some insight. Doctor: Please visit a gastroenterologist.May need ogd scopy.xray abdomen vertical. And usg needed .to rule out infection please do stool routine & urine test .put him on antispasmodics."
},
{
"id": 54855,
"tgt": "How can auto immune hepatitis with feet swelling be treated?",
"src": "Patient: Thanks for enquiring about my health. I am sorry for reverting back to you. As I told you I am suffering from auto immune hepatitis for more than a year. I enclose two repoprts one being the latest one for you to peruse and suggest me the course of treatment. Lately, I have developed swelling on my feet and doctors say that it is due to protein deficiency. I am taking protein supplements. The liver parameters are very high and as I am told that as billi rubin level decreases other parameters would also decrease. I await your reply. Doctor: Hi thanks for asking question.Noted you have autoimmune hepatitis.Do you have taken any immunosuppresive drug??You are not improving.So after consulting doctor corticosteroid or/and azathioprine like drug should started....With these drugs you will benefit ...So consult doc for starting it...For swelling diuretic also can be taken if not decreasing....Take low salt diet.Low fat diet taken with more fruits....Avoid alcohol and smoking if habit....Take care....Dr.Parth"
},
{
"id": 215517,
"tgt": "Is intake of high doses of Dilaudid advisable?",
"src": "Patient: Hi, my husband is on dilaudid via a pca pump after being in the hospital for ten days for pain management because of a chemo extravasation on 2/16/17. The chemo was DOSEtaxel. He is concerned that being on high doses of this opioid is not healthy for him. He s been on it for almost three weeks. What say you? Doctor: Hi, Yes, he's going to feel it when they take him off it. There is likely to be some withdrawal and it will have to be gradually lowered. The pain should be down by 3 weeks. Hope I have answered your query. Let me know if I can assist you further. Regards, Dr. Matt Wachsman, Addiction Medicine Specialist"
},
{
"id": 88361,
"tgt": "What causes swelling in the stomach?",
"src": "Patient: Hi, about 6 months ago I realized that my stomach had enlarged about 6 inches. My weight had increased about 20 lbs and I started having pain in the rib cage area, mostly at the back. I ask the doctor and she thought that maybe I just needed to lose some weight. Then about 3 months ago she arranged for an ultra sound and it showed nothing remarkable except a fatty liver. Then she arranged for me to see a specialist who sent me for a CT Scan. The test involved taking a creaming mixture and then the scan was done. I was told not to take any Metformin until I had a blood test. The scan was done on July 29th, 2014 and as a result I have not been able to take Metformin anymore. Also they say my Creatine is high and my Egfr is low. Creating currently is 108 and Egfr is 44. What is happening to me? Thank you Doctor: Hi.Thanks for your query, read the history and understood your problems.High creatinine and low egfr are suggestive that you are suffering from a kidney failure.This is causing you to gain weight by deposition of more fluid into the body or can be obesity. I would advise you to first see the Nephrologist and get further tests and a proper treatment ."
},
{
"id": 68540,
"tgt": "How can a tender breast lump with pus be treated?",
"src": "Patient: I have recently discovered a lump on the inner part of my left breast. it's pretty big for a lump and the first time I squeezed it, a considerable amount of pimple-like pus came out. Now the lump is still there, no more pus coming out, and it's tender to the touch. It doesn't feel the same as breast tissue and i'm nervous about it. Doctor: Welcome to Health care magic.1. Your history suggest that you are having an infective breast abscess.( painful lump with discharge) 2.Please do not squeeze / compress the lesion / lump.3.In this case visit your GP let it get examined - request for an ultrasound breast.4.That will give exact nature of the lesion / lump ( solid, cystic,mass) , its source ( muscle, dust, gland ) and its extensions. 5.Treatment in this case would be a small incision and drainage and good post procedural care with a course of antibiotic will help. Good luck.Hope it helps you. Wish you a good health.Anything to ask ? do not hesitate. Thank you."
},
{
"id": 51969,
"tgt": "Both the kidney size contracted and increased creatinine",
"src": "Patient: my name is bishnu kheriwal live in west bengal india 24year male , i have kidney problem ,both kidney size contracted and serum urem urea and creatinine increase so i want your help to improve the size and blood patho test report s.urea -approx.100 creatinie-4.00 left kidney size-3.00c.m right kidney size- 9.01 c.m according to 30 may blood test report and u.s.d. report i want your group[ kind attention and diet consultation and homeo medicine prescription Doctor: Hello friend. Raised blood urea level and serum creatinine, alonge with u.s.g. report suggestive of contracted kidney shows that you are suffering from chronic renal failure. And for this condition you should consult nephrologist as early as you can."
},
{
"id": 77320,
"tgt": "Suggest treatment for chest pain and rapid heart beat",
"src": "Patient: Hello, Doctor. I found this Web site on a search engine link after typing in the keywords, Saline IV causes stomach bloating? . My name is Nicole. I was 27 in January 2014 when I experienced very tight chest pain & the sensation of feeling like I couldn t breathe when I finished my plasma donation at a new Plasma Donor center. The last part of the donation calls for saline to run through the IV so it can calm the heart rate & sort the blood back to normal preventing clots. I alerted the employees at the facility that my chest was very tight. That employee had a nurse come in to check me. Later, paramedics made two trips to see me. I had told them to leave since I was feeling better. After several minutes of monitoring, sitting up in the waiting room eating gold fish & sipping on water, my heart rate never got below 120. When paramedics arrived the second time & placed me on the gurney, heart rate was at 150. I told one of the paramedics my heart was never this high when at rest or sitting down. After being hooked up to an IV at the ER, my heart rate never got below 99. That didn t shock me too much. I went in the plasma place feeling anxious due to the frigid temperatures outside & having a lot of anxiety-induced thoughts in my mind. However, I felt fine until the saline went thorugh my system at the end of the donation process. Keep in mind this was only my second donation. My first donation was a success with no issues a couple of weeks prior. Do you think the saline solution might have caused my chest to become tight due to bloating my stomach? Doctor: I dont think so. as saline only helps maintain the necessity of the body functions. you can meet a cardiologist for the same and get yourself assessed with possible advised tests. post that if the cardiologist permits for some small exercises under guided physical therapist kindly do it. it will help you retrain your cardiac system and get your heart rate fall under the normal parameters. but do meet a cardiologist before any exercise."
},
{
"id": 180366,
"tgt": "Can a deep cleaning procedure along with Acclean after a dental appointment wash traces of Marijuana out of the mouth?",
"src": "Patient: Hi, I took a swab drug test after I left the dentist . I had a deep cleaning procedure and they also gave me a medicated mouthwash called Acclean. However about 26hrs prior before the drug test I smoked some weed, I was wondering if there s a chance that the medicated mouthwash along with the deep cleaning could have washed traces of the thc out of my saliva. I swished with the mouthwash less than 10min before the test. Thanks Veeda Doctor: Hi, Well, there is no sure shot document that deep cleaning can help in clearing swab test, if you have recently smoked weed. However, cleaning the mouth thoroughly can help in clearing old saliva and this can help in reducing the positivity of the test as the new saliva has lower concentration of the drugs. But the best way is to avoid using recreational drugs at least 72 hours before the swab test. Hope I have answered your query. Let me know if I can assist you further. Regards, Dr. Honey Arora, Dentist"
},
{
"id": 87083,
"tgt": "What causes low grade fever with lower abdominal pain?",
"src": "Patient: My daughter had had a low grade fever off and on since tues., 5/11/10. 19 yrs. No other symptoms, besides some swollen glands in her neck. We were taking advilToday, 5/16, she has had fever most of the day, some puffyness around the eyes, some pain in lower right abdomen. Our other daughter is recovered recently from mono. We have an appointment tues 5/18. Weare now giving her tylenol. She is taking fluids, water oj, gatorade. Doctor: Hi.Thanks for your query.The reason of the low grade fever with swollen glands, puffiness around the eyes and pain in the right lower abdomen can be due to Viral fever- flu.There is a possibility of appendicitis or colitis giving similar symptoms. I would advise you to take her to the ER as there is no point in waiting to see a Doctor after 2 days as by this time she will be examined by a Doctor, investigated and properly treated. Please take her to the Emergency room to get investigated. Blood, urine and stool and an ultrasonography will help to have a differential diagnosis and get an appropriate treatment at the earliest."
},
{
"id": 568,
"tgt": "Can pregnancy happen after having unprotected sex?",
"src": "Patient: Me and my girlfriend had unprotected sex the day after her period I'm really scared shes pregnant..I used the pull out method but know this i was far from an orgasm...after I pulled out i didnt end up having it until like 10 minutes later...I'm aware of precum and I peed atleast 3 times before sex I know for a fact I peed before sex..please help me whats the probability shes pregnant...I dont know if this helps but we were in the pool before sex does that also help wash sperm out of the penis? Doctor: Hi, I think if the precum or ejaculate didn't come in contact with vagina, there is no chance of pregnancy. So, don't worry about it."
},
{
"id": 71026,
"tgt": "What causes chest pain and shortness of breath?",
"src": "Patient: Hello I have been getting chest pain in january for about 3 weeks in jaanuary while i had a stressfull period. I was fine for about 3 months until another stress period came and got chest pain, lack of sleep and started feeeling short of breath everyday. I seems to feel better now, i dont know what it could be Doctor: Hello and Welcome to \u2018Ask A Doctor\u2019 service. I have reviewed your query and here is my advice. * As per my clinical experience, the narrated symptoms indicate physical expression of underlying anxiety issue. Still before final comments, I request to undertake an EKG and X-ray chest as primary line work up. Hope I have answered your query. Let me know if I can assist you further."
},
{
"id": 215255,
"tgt": "Suggest treatment for severe upper thigh pain",
"src": "Patient: My name is Linda and my question is, how can avoid back surgery if the orthopedic states that is the only thing at this point that is going to stop my pain. I was in an accident one year ago and my back is injuried. The pain causes my upper thigh a great deal of pain. I have taken medication and given shots. the problem is that my last bone in my back is out of place.The orthopedic said the only way to fix it so that I will not be in so much pain in to have a spinal fusion. Doctor: Hi, The location and movement of a pain gives clues as to what is going on. I was sceptical until you said \"last bone\" which would give this pattern of pain. It isn't the most common back area affected, but when it does it is inner upper thigh. There isn't good data on whether these procedures work for pain. Hope I have answered your question. Let me know if I can assist you further. Regards, Dr. Matt Wachsman, Addiction Medicine Specialist"
},
{
"id": 8377,
"tgt": "Wiil Diprovate-RD really reduce white marks on face?",
"src": "Patient: Age: 22, Height: 5.6, Weight : 56 Recently i got some white marks on my face which was termed as pitiryasis versicolor or teanea versicolor in medical sciences. the doctor prescribed me with Diprovate-Rd . Will it really reduce the mark? because i dint find any result after using that for 1 week. Doctor: Hello,I can understand your concern for the white marks on face.As per the doctor's diagnosis the condition of white marks is tinea versicolor also known as pityriasis versicolor.Basically tinea versicolor or pityriasis versicolor are fungal infection caused by superficial fungus. It is common to find white colored branny spots over face or chest due to infection with this fungus.But it is also surprising that doctor has prescribed you a steroid based cream of Diprovate RD. It is a steroid and never use in case of tinea versicolor or pityriasis versicolor.In my cases of tinea versicolor I use an anti-fungal cream or if required an anti-fungal tablets. SO I suggest you to visit your doctor again and get either a proper diagnosis or a proper treatment.All the best."
},
{
"id": 121637,
"tgt": "What causes pain in the sternum?",
"src": "Patient: I m 52 and weigh 250. I m having pain in my sternum mostly and a little in my right shoulder. I suspect that it is related to indigestion which I have had lately due to the fact that I ve eaten foods which tend to give me indigestion. The problem started while I was sleeping at night. Doctor: It can be do to food and indigestion but you should visit your doctor for excluding cause such as heart related."
},
{
"id": 27446,
"tgt": "Is the resounding of heart beat be due to the intake of Metoprolol drug?",
"src": "Patient: Hi I am a 53 year old female. I have no heart disease that I am aware of. I do take metoprolol 25 mgs daily for a fast heart rate. It usually runs about 90 bpm, so doctor suggested this medicine. But last few nights, as soon as I lay down at night, I can feel and hear my heart beating. Is this normal? I do not have any caffine at night, however I do have wine, and I tend to have sweets. I immediately feel my pulse, and it feels normal and regular, and it doesn't feel fast. But it keeps me up as I can distinctly feel and hear the thumping. Should I be worried? Should I up my metoprolol at night? Thanks for your answer. Doctor: Hi,Usually it is not dangerous if you feel your heart beating when you are lying. I would advise you to check your blood pressure for a week in the evenings, as sometime people can feel their heartbeats because of high blood pressure, but if it is normal before going to bed, there is nothing to worry.Also I would advise you to avoid sweets and alcohol in the evening, and try to be in the fresh air much.Hope I could help youWishing you good healthIn case of further questions don't hesitate to askRegards,"
},
{
"id": 95503,
"tgt": "I just went to the bathroom and noticed my stools were a dark green",
"src": "Patient: I just went to the bathroom and noticed my stools were a dark green..is this serious ? Doctor: Hi, Dont worry, it appears to be a solitary episode. Eat well and have lots of water, it will disapear. If it is a recurring problem, stools need to be examined by a Pathologist to rule out any pathogenic infection including fungal etc. Stool exam will also bring out any metabolic disease. Dont worry, it may not recur. Thanks"
},
{
"id": 215905,
"tgt": "What causes pain in right breast near nipple area?",
"src": "Patient: Hello Dr. Manjunath, I want to ask you why I am experiencing the pain on my right breast, nipple area over a month or two, which is like being poked by needle when I press or touch it. By the way, I am a 20-year-old man. Is that normal in a man?Thank you.Joe Heguo. Doctor: Hello,Thanks for the query.It is probably be a non specific pain and nothing much of any clinically significant.As of now you can take analgesics like ibuprofen or diclofenac.Wishing you good health.Thanks"
},
{
"id": 208906,
"tgt": "Need help in diagnosing and managing a medical problem",
"src": "Patient: Hi.My patient is a 18 y/o girl , a known case of bipolar disorder.she have developed psychosis Since 5 days ago(acutly and without any precipitants) Now she is admitted , cpk is rising from 500(yesterday) to 19000(today morning) and 56000 now(afternoon).LDH is 2000.HB is 11.4 wbc:8800 U/A: 1+ blood with 8-10 RBC, BUN,cr,Na,K :normal She is oriented,with a low grade fever without rigidity, without pain, normal neurological exam,All drugs are holded(previous drugs:lithium and risperidone from 1yr ago till today morning) WOULD U PLZ HELP ME TO DIAGNOSE AND MANAGE THE PROBLEM Doctor: Hello,She is developing NMS most probably but appears to have better prognosis as she is hospitalised and drugs are stopped. It is good she has no rigidity and some tests are normal, she is oriented and neurolgically normal.The only managment presently is supportive and involves controlling fever, maintaing fluid and electrolyte balance and prevention of kidney functioning.Let her be hospitalised, under 24hrs survillance till her CPK and Blood counts improve. She cannot be managed at home presently.The treating team is doing good work, I think you should trust them.Wish you good luck.Dr. Manisha GopalMD Neuropsychiatry"
},
{
"id": 84031,
"tgt": "Will Tetralysal 300 mg work after consuming alcohol?",
"src": "Patient: Hi, Im jo.I m taking tetralysal 300 mg for my acne for almost 1 and half mos. now.I have drunk 2 bottles of beer last night.will it still have the same efficacy?Should I continue with the medication or it would no longer be that effective since I drunk alcohol. Doctor: Hi,Alcohol may reduce the efficacy of tetralysal. If consumed while on tetralysal alcohol or alcohol containing beverages may decrease the efficacy of tetralysal. Avoid consuming alcohol during the course of treatment with tetralysal.Hope I have answered your question. Let me know if I can assist you further. Regards, Dr. Mohammed Taher Ali, General & Family Physician"
},
{
"id": 137152,
"tgt": "What causes inflammation and swelling lower leg after a shin injury?",
"src": "Patient: I had a blow on my shin on June 4th from the edge of a chair on which I had been standing. Almost immediately, there was a large goose egg about 1 1/2 inches in diameter, which stayed for several days but gradually subsided and took a few weeks to completely disappear. There was a slight indentation with no breakage of skin, probably where my shin hit the edge of the chair. About 1 week later, I had an urgent care doctor(we were traveling at the time) look at it and X-rays were taken, indicating no bone break or chips present. I have some concern now, however, about reddening of the skin below the injury down to my ankle and foot as well as some swelling of the ankle/foot. This is now almost 1 month later. Is this normal? Can I treat it in any way? BTW, I THOUGHT YOUR WEBSITE WAS FREE...!! Doctor: Hello ! Your concern is well received. One month is too long a time to wait to see '' if it heals by itself ''. However , if its not causing too much concern in your day to day activities , exercising, walking , pain , and there isn't much swelling , the movement of the foot and ankle is ok , too chances are that it isn't anything serious. However its a good idea to get it checked though by an orthopaedic to know if there is some residual inflammation , and if you need medication. more important is the course of recovery - if it has worsened over one month or improved over one month. If it has worsened , you should visit the doc right away ! If there is swelling and pain , you will also benefit from physiotherapy to manage the same. Hope this helps ."
},
{
"id": 120743,
"tgt": "What causes pain in toe after falling a metatarsal head?",
"src": "Patient: I have a fallen metatarsal head in my right foot 2nd toe. I have seen a foot doctor about it as it hurts enough to have stopped me from running. I am in pain most of the time and am currently wearing a toe separator since my second toe is now leaning to my first and have begun wearing DANSCO shoe. I am also a hairstylist so I stand all day. Is there a fix for this problem or do i need to just adjust my lifestyle to prevent pain? Doctor: Hello, It looks that most probably you may having fracture or ligament injury .Both of them needs some time (At least 3 weeks ) for recovery. With present treatment You may also do following to have relief- -Take a good analgesic like aspirin,ibuprofen or paracetamol. -Do warm water fomentation over affected area, at least 3 times a day. -Do not bear weight over affected limb ,at least for 3 weeks. Hope I have answered your query. Let me know if I can assist you further. Take care Regards, Dr. Mukesh Tiwari"
},
{
"id": 199349,
"tgt": "Why do I get burning sensation after ejaculation and have pain in groin area?",
"src": "Patient: Started out with burning sensation after ejaculation and then progressed to burning sensation on and off with slight pains in the penis and groin area that come and go that seemed to be relieved by ibuprofen . also have more frequent urges to pee. The burning after ejaculation is gone but still have the feeling of having to pee constantly. Doctor: HelloI appreciate your concernLooking at your description this could be signs of UTII would like further information to help you better likeYour age, sex and relevant medical history like diabetes or other metabolic diseaseSince how long you have this problem?I would advise you to consult your doctor for battery of investigations like urine microscopy as well as urine culture and sensitivity testYou may require a course of antibiotics depending upon the reportsHave plenty of fluids which will flush off the urogenital tractMaintain good hygieneThanks for your questionwish you a speedy recoveryBest wishes"
},
{
"id": 66938,
"tgt": "What causes an armpit lump with soreness, redness and tenderness?",
"src": "Patient: I have had a lump in my arm pit which was quite sore with redness around, this has now reseeded to a small lump. This has taken about 3 weeks and now the lump is still there but I have to look for it. My both arm pits are tender. I have in the past (10 years ago) had rectal cancer so try not to get paranoid. Doctor: if you are female then it is related to benign changes in the axillary mammary tissue and if you are male then these are due to infection of the skin adnexal tissue or sebaceous cyst!i think not to worry muchall the best!"
},
{
"id": 193262,
"tgt": "What is causing bad smell after prostatectomy ?",
"src": "Patient: Hello Doctor,Since I've been operated on (prostatectomy) over three years ago (have also received radiotherapy in the prostate bed) I've been having this peculiar sensation of smelling something burnt. The smell isn't with me all the time but can last for many minutes at a time. Could this be an after effect arising from the operation perhaps due to anasthetics? Doctor: Hello, The mentioned abnormal smell is not related to anesthesia or operating procedure. It seems to be due to some inflammation of nasal mucosa. You can consult nearby physician or otorhinolaryngologists for a detailed evaluation of the nose. You need not to worry about that abnormal smell. Hope I have answered your query. Let me know if I can assist you further. Take care Regards, Dr Parth Goswami, General & Family Physician"
},
{
"id": 9748,
"tgt": "Is hair loss a side effect of over intake of Biotin?",
"src": "Patient: how much BIOTIN is to much for hair growth , I have been taking it for years ,, but now it s falling out ,, ince to10,000 mcg is that too much ? also found I WAS TAKING TOO MUCH V E I M 85 YR. OLD. ALSO IS MORINGA OLEIFERA extract 1000mg worth taking for memory an energy ??? Doctor: Hello, The standard oral dose of biotin is 5-10mg per day for hair fall. Hope I have answered your query. Let me know if I can assist you further. Take care Regards, Dr Kakkar S., Dermatologist"
},
{
"id": 114036,
"tgt": "My hamstring has become tight after a back surgery, what can i do for this ?",
"src": "Patient: i had back surgery 2 years ago and i ve had problems ever since with my right hamstring always being tight i had back surgery 2 years ago and i ve had problems ever since with my right hamstring always being tight the doctor that did the surgery cut out a bone spur from the right side of my hillyact crest. not only is my hamstring tight but my hip bothers me all the time.i am also losing the mobility in my right leg. it gives out on me all the time. i ve had mri since the surgery and also had doctors say that i have scar tissue wrapping base root nerves s-1 l-4 or l-5 cant remember please help the doctor i see has done nothing they all think that im a pill seeker i just want to get better Doctor: Thanks for the query Your MRI findings go hand in hand with the symptoms u r facing right now. I am sorry but not much can be done to releive the pain. U will have to see a good physiotherapist and also try ultra sound for pain. U can continue with the pain killers prescribed to u Have a healthy living"
},
{
"id": 146284,
"tgt": "Is dizziness for a very short duration while driving a car dangerous?",
"src": "Patient: is feeling dizziness for 3 secs ,while driving the car a dangerous thing for a person of 60 years old and a little bit obese with a bmi above the average ....the person being normally healthy with no antecedent health problems.....no diabetes.. cholesterol within llimits. Doctor: Hello dear,First of all,my concerns for your problem.Complaint of episodic dizziness for few seconds as mentioned by you,needs to be clarified for certain key points to guide you better-1)Since how many days/months or years, are you facing this problem?2)What is the frequency of these episodes,like,1or 2 episodes per month or so?3)Is there any specific posture,associated with these episodes,apart from the one you told,ie, while driving?Its importance is to rule out syncope or postural hypotension,which is mostly associated with standing posture.4)Do you have any type of premonitory symptoms just before the episode,like palpitations,anxiety,paleness on face ,excessive sweating-commonly associated with syncpe.(It is mandatory to mention here ,that syncope along with these is associated with transient loss of consciousness of few seconds).5)So as above mentioned,along with syncope,transient loss of consciousness may be associated with other causes like seizures(which usually also have involuntary activity of the body)-So,did you lose consciousness any time?6)Is it associated with a sudden postural change-like neck flexion or extension,rising from the bed,lateral flexion(associated with benign positional vertigo).7)Do you have history of ringing sensation in ears,or hearing loss(Condition like meniere's disease may have these spells)8)Lastly and least likely,do you have any double vision,or facial deviation or loss of eye movements in any direction(to rule out transient ischemic attacks)?Please reply with these,so that i can help you, to formulate a diagnostic and therapeutic plan for you"
},
{
"id": 75276,
"tgt": "Suggest treatment for blood in cough",
"src": "Patient: HiI am having blood in my cough from last 3 days...although not that much severe ..but m too worriedwhat could have been the reasons behind it ,,... and how serious is it???I have taken some painkillers last week as i felt severe headache..doctors said it is migrain pain and i was given some antibiotics...pls suggest Doctor: HelloI find your concern very important.If you are sure that the blood comes from coughing despite from your clinical situations you should have some examinations as follows:Blood countChest CTFibrobronchoscopyPPD testSputum smear for tbAccording to the above tests results it will be and the diagnosis and treatment too from a lung diseases.Wish you good healthDr.JolandaPulmonologist"
},
{
"id": 131604,
"tgt": "What action is necessary for twisted foot with no swelling and hurting achilles tendon?",
"src": "Patient: My 7 year old son got his foot caught in an ellipitcal machine last night (fell off and it got twisted in the bars). It hurt a bit but he was fine last night, running and playing. This morning he could not walk on it and he says it hurts by his Achilles tendon. He can t put weight on it and he says it hurts too when it hangs down (like when he is hopping on the other foot). There is no swelling at all and no bruising. I gave him so Motrin and talked to his teacher at school and he will be sitting with it elevated all day. Any idea what it could be? Do we need to head to the doctor? Doctor: It is likely an internal bruise nothing more(if no swelling ) just use the rest ice compression elevation ...If a swelling appears then go to ER for xray.."
},
{
"id": 130122,
"tgt": "What causes slight pain in groin and pelvic area?",
"src": "Patient: I have a slight pain in my groin/pelvic area. It is not annoying, almost resembles a gas pain. I believe it is a muscle ache due to the ache acting up when I am working abdominal muscles at the gym. How do I know if this is the onset on a hernia? Doctor: Hi...All lower abdominal pains and groin pain may not be sourced only by hernia.. please don't be suspicious on that always..I would like to ascertain certain features before heading towards investigation and confirmation of early symptoms of hernia..there should be pain while coughing...pain need to be radiating to groin need to be same and that should not alter while squeezing my thigh..there should not be any pain originating from the mid back region...there should not be any alteration in pain when I press on the front pelvic bone junction areaHave a good Anti gastritis tabBe careful while working out with heavy weights in gym .. use proper abdominal supporter and protect yourselfHope this is helpful for you..."
},
{
"id": 183168,
"tgt": "Suggest treatment for enlarged papillae on tongue",
"src": "Patient: I have an Enlarged tongue papillae on the middle left side of my tongue. It is about the size of a dime and when irritated, becomes painful. It is slightly different in color than my tongue.What are my treatment options, can it be removed and is it cancerous?This has been on my tongue for over five years. Starting out as a very sensitive slit or indentation that I noticed after eating. Doctor: HelloThanks for consulting HCMRead your query as you have enlarged papillae on tongue dont worry this is normaly seen in some patients it is not cancerous but I will suggest you to consult dentist and go for oral examination and if needed you can go for investigations like cytology . Inmeantime do warm saline rinses and take one Capsule like vitamin B complex .Hope this will help you."
},
{
"id": 222315,
"tgt": "Suggest remedy for severe fever and vomiting during pregnancy",
"src": "Patient: Actually My wife 2 months 7days pregnant. She is feeling like vomitting after taking food and not taken food. Also suffer fever in day & night within 1-2 hours. Also no taken medicne any bcoz medicene sideeffects to baby... Please Guide Us. Thanks in advance! Doctor: Hallow Dear,Nausea and vomiting of pregnancy or morning sickness is a notorious part of the pregnancy. Please try to observe following:1. Let her not cook the food. In fact she should keep away when the food is being cooked. 2. Let her eat dry food. Liquid food regurgitates easily. have short but frequent feeds. 3. Eat only what she likes and feels like eating.4. Avoid oily and spicy food. 5. Avoid tea/coffee. Buttermilk is a good drink. Particularly in the morning when nausea is more. 6. She may have more liking for sour items. Let her have them. 7. By this vomiting can be avoided, however, nausea cannot be conquered by any means. However, my experience with the patients for over 40 years has shown me that banana or melon with plenty of sugar in cold water takes away the nausea due to any reason. 8. If persistent vomiting makes her dehydrated, take her to the hospital. She may need IV fluids lest it becomes a vicious circle. Dehydration can be adjudged by pinching skin on the back of the hand. If the fold remains, she is dehydrated. There are many medicines which stop vomiting; however, some of them have ill effects on baby in the womb. So as far as possible, try to control it by modifying eating habits. If still vomiting cannot be controlled, please visit an Obstetrician and take medicines with his/her advice only. As it is nausea and vomiting of pregnancy lasts only till 12 weeks of pregnancy. So within 3 weeks, she will be relieved of it. If she can pull on with it without medicines, great. Dr. Nishikant Shrotri"
},
{
"id": 110131,
"tgt": "What could cause pain in vertebra?",
"src": "Patient: hi doctor , i have been suffering form mild back pain at level t8 t9 in about 4 months .iam not sure if i have a bulging or not case the Pain is vary which effect the both side masclus but when some one press on it i feel some relief also i feel pain exactly inside the vertebra Doctor: Hi,Welcome to healthcare magic.After going through your query in my opinion it is muscular pain as pain is mild. Rest and analgesics is its treatment. Sometimes vitamin D deficiency can cause this so serum vitamin D test is advised if it is low than vitamin D supplementation will be required.Other causes of pain can be seen by x-ray and MRI of thoracic spine. I think your query answered.Welcome to any follow up query."
},
{
"id": 72452,
"tgt": "What is the effect of loss of lung on future life?",
"src": "Patient: sir before 3 year due to road accident i loss my lung and after 4 surgery done by dr. thoracoplasty, pneumonactomy, pneumothoractomy, and many time icd done. i m doinf m .pharm still i have surgical pain and i cant cocentrate in my study. my behavior is so anger after accident and i m so affraid for my marriage Doctor: Hello dearWarm welcome to Healthcaremagic.comI have evaluated your query thoroughly .* This is post traumatic stress disorder , needs to be taken care with help of expert psychiatrist doctor .Hope this clears your doubt .Wishing you fine recovery .Welcome for any further assistance .Regards dear take care ."
},
{
"id": 152009,
"tgt": "I have a problem of wrist drop after a fracture",
"src": "Patient: hello!! i underwent a surgery 2 weeks before for ma fracture for the right hand bov d elbow below d shoulder area.... but now i m having wrist drop problems.... can u suggest me few exercises...and how much time it may take 2 recover!! can i start writing within 3weeks from now !...as i gt my college... (i m wearing a SPLINT now!!) plz help me doctor!! Doctor: Hi,Rakesh, Thanks for query, Recovery depends upon to how much extent the nerve is damaged. If minor damage is thee within 3-4 weeks you may be alright but it takes time if more damage is there. Go for physiotherapy daily and regularly.this will help most. Ok and bye."
},
{
"id": 220833,
"tgt": "Why does the ultrasound scan not show a baby in a pregnancy?",
"src": "Patient: yes i went too the doctors office and they took a pregnancy test on july 28 that was positive and had me get an ultra sound to see how far i was but the tech said she could only see a sac and couldnt see the baby and i had blood work done the same day and on july 29 my doctor called and said my blood work was positive and that i was 3-4 weeks pregnant why didnt the tech see the baby then is it because it was to early or could it be a blated ovum this is my first pregnancy so i dont know how it really feels to be pregnant though i also go back on august 3 for blood work and august 4 to see the obgyn is that way to early what would they do for the first visit and would you only bleed if you have a miss carriage though just woundering Doctor: 3 - 4 weeks pregnancy is too early to detect heart beat in USG. It may further take a week or so to detect it.so relax and every bleeding is not necessarily a miscarriage. Sometimes it may just be threatened and pregnancy can be saved"
},
{
"id": 92463,
"tgt": "What could cause discomfort below ribs after surgery for Diverticulosis? Surgery had adhesion related complications",
"src": "Patient: I had surgery due to diverticuli disease two years ago. During the surgery there where adhesions that connected my descending colon to my spleen. The original surgery got more complicated due to this. I have had discomfort just under my rib cage ever since. During the last several months the discomfort is increasing from a few hours daily to all day. The surgeon indicated that my stomach might be pushing against another organ causing the discomfort. Doctor: YES YOUR SURGEON IS RIGHT YOU NEED TO GET INVESTIGATED FOR THE ORAN CAUSING THE DISEASE AFTER CONSULTING GASTROENTEROLOGISTOT IT CAN BE FOOD PROTEIN ALLERGY AS ONE OF FOOD PROTEIN START SUDDENLY REACTING WITH BODY PROTEINS IF LEFT UNDIAGNOSED INCREASES GET BLOOS SERUM TESTS FOR SPECIFIC ANTIBODIES FOR MILK WHEAT OR OTHER MAJOR FOODS THIS CAN ALSO HELP IN MY OPINION IF NOTHING COME IN OTHER REPORTS"
},
{
"id": 220615,
"tgt": "Suggest remedy for severe pains in stomach",
"src": "Patient: Hello, I m 23 weeks pregnant, I am suffering from really bad pains in my stomach which i ve had since i was 19 weeks! I went to he hospital and they have said it is ligament stretch but the pain seems a lot worse i also have flu like symptoms and are exhausted to the point of going back to bed a hour after I ve woke! I can t get in to see my doctor and really don t want to waste anyones time at the hospital especially if it something so trivia Doctor: Hello dear,I understand your concern.Iam sorry for your condition.In my opinion there are various causes for pain abdomen in pregnancy like muscle cramps,preterm pains,any abdominal conditions like bowel disturbances,any urinary tract infection(UTI) etc.The preterm pains are associated with tightening of abdomen along with cervical changes.The bowel disturbances are associated with loose motions or constipation.UTI might or might not be associated with urinary symptoms.The muscle cramps, ligament stretch have only pain with no associated symptoms.As you said all the reports are normal I hope ultrasound and urine examination are normal.So as your doctors said it might be muscular pain or ligament stretch as there is just pain.Some non specific pains tend to start in pregnancy which subside spontaneously before or mostly after delivery.So relax.Avoid stress.Take adequate fluids upto 3 litres per day.Take panadol for pain.Hope this helps .Best regards..."
},
{
"id": 171677,
"tgt": "What causes raise in ESR levels in a baby?",
"src": "Patient: Hi doc, my name is bobin, and i have a 2 months old baby girl who is having high ESR level, 140 as per lab results...She is having inflammation on her cheek and swelling was also there which has decreased now...When consulted with our pediatrician she said some infection was there and recommended for taking advent injection 150 mg for seven days to reduce ESR levels...My baby is active, she is feeding well and all other sensations are normal.. Kindly advise why ESR levels gets raised in a 2 month old baby? Doctor: Hi Bobin,Welcome to Hcm,ESR is an inflammatory marker like crp which you must have heard.So any infection or inflammation meaning swelling can result in high ESR.As your pediatrician rightly did in your child by advicing antibiotic to reduce infection and to check for lowering of ESR.Other common reasons are if there is any tuberculosis, ESR can be high but that needs a strong suspicion to evaluate if there is a strong exposure history to someone with TB. However I think the best thing is what you have been asked to do and I hope there is no other reason in your child.So continue what you are doing now. Hope for the best. Take care."
},
{
"id": 225289,
"tgt": "Is it safe to have unprotected sex when switching over to patches from depo shot which caused missed periods?",
"src": "Patient: Hey I ve been on the depo shot for about 3 years and decided to switch to the patch it seemed much more manable. How everI have not had my period for 2 years due to the depo and now I m on my third week which is the week I should not wear the patch but if my period does not come down can I have unprotected sex ? Doctor: If you are continously taking depot snd then patch without having your periods then most likely you are not ovulating .....and if you have any sex in the period mentioned then there is negligible chance of pregnsncy because that is your immediate premenstrual phase where there is no chsnce of conception. ...so you can have intercourse during that period but remember to take the patch on once period starts"
},
{
"id": 4984,
"tgt": "Unprotected sex. Finished mensuration. Taken birth control pills. Will I get pregnant?",
"src": "Patient: Hi.I've finished my menstruation on 19 July 2013 and I start to take pills on 22 July then 23 July I have unprotected sex with my boyfriend but we widrew it. Then I continuing taking my pills but the last 3 tablets of my pill for 8,9,10 August I didn't take it because I lost it . Today August 11 2013 I having my meansturation but its really low. I was expecting that my menstruation is on 19 August 2013. Is there a chance I get pregnant ? Doctor: Hi,As you had sex during non fertile period, chance of having pregnancy is less.And as you got your period even less, there iless chance of having pregnancy.Having less period might be due to oral contraceptive pills.You got your period early might be due to not taking last 3 pills and its withdrawal effect came earlier.Ok and take care."
},
{
"id": 176418,
"tgt": "Suggest treatment for sinus infection",
"src": "Patient: 16 year healthy male, diagnosed a week ago with a sinus infection, was running low grade temperature, put on antibotics, name uncertain. His behavior has changed dramatically. His speech is slurred. When asking him a question, his response is cat dog . He is sleepy, pale. He states, he feels as if he as amnesia, complains of extreme tiredness. He was taken to emergency room, all tests, drug screens were negative. He was not presenting symptoms at time he was being triaged. He appears fine at times, then has an episode described above. But he is again presently symptoms again tonight. He has been caught running naked inside/outside home. Tonight he stated, he murdered 2 people in a warehouse, that he is involved in a gang. He is a straight A student, polite, helpful, involved. Family very concerned. Doctor: Hello. I just read through your question.It is very unlikely that these episodes are being caused by any treatment for a sinus infection. Though his drug screen was negative, you mention that the symptoms began after the er visit.It seems that he is either taking a drug that cause psychosis, or having r psychotic episodes due to a real psychiatric illness. In either case he must be seen by a psychiatrist for treatment.."
},
{
"id": 121173,
"tgt": "Suggest treatment for hematoma knot on inner thigh caused by an injury",
"src": "Patient: Hello, 15 weeks ago I flipped my 4 wheeler over on me, broke my collar bone, tore my MCL and got a hemotoma about the size of a cantalope on my inner thigh (Dark Blue and purple) with an egg shape knot in the center. The bruise is only about 1 inch wide and 3 inches long now The knot is about quarter size. Will this ever go away? Doctor: Hello, Based on your description , you are suffering from heamtoma in your thigh secondary to soft tissue injury . Usually the hematoma removes over a period ranging from 2 to 6 weeks . During this period you have to observe the hematoma and maintain good hygiene so that it does not get infected. Usually with this it resolves on its own. Visit your doctor if there is increased pain or size. Hope I have answered your query. Let me know if I can assist you further. Take care Regards, Dr. Santosh S Jeevannavar"
},
{
"id": 204559,
"tgt": "What causes sudden slurred speech and memory loss?",
"src": "Patient: My daughter, 52 years old was making dinner when her conversation was jumbled, she handed a torn tortilla to her husband with raw turkey. He recognized something was very wrong, yet she did not remember any of this. She does not drink, smoke or do ant kind of drugs. She is having irregular BP READINGS like 190/175. Doctor: Hello and Welcome to \u2018Ask A Doctor\u2019 service.I have reviewed your query and here is my advice.She might have had a transient ischemic attack. Consult a cardiologist or emergency department. Go for an ECG to rule out any cardiac pathology. Her uncontrolled irregular BP is one of the reasons for such attack. Take proper treatment before she suffers another attack. Hope I have answered your query. Let me know if I can assist you further.Regards,Dr. Rohit Kothari"
},
{
"id": 31888,
"tgt": "What does increasing ESR means?",
"src": "Patient: Dear sir,my aunt 61 years of age weighs 50 kgs. She has been losing weight gradually since the past 6-7 years.She has been having fever since the past 3 months.The fever has been coming everyday and subsides on taking SUMO only(paracetamol doesnt work). All the reports like blood , urine test,urine culture have come normal. Chest,skull,abdominal x-rays, also reveal everything to be normal. sputum test showed negative result. TBFERON detection test from Religare,mumbai revealed gamma interferon in the blood sample as INDETERMINATE NOT DETECTED.She has been taking tb treatment since the past one month.Fever still come everday, only respite is she feels better during the times she doesnt have fever and 1/2 sumo now gives respite from fever for 24 hours, while the earlier requirement was one tablet. Do you think we are on the right track, since no particular test for confirming tb was conducted.Her ESR 3 months back was 88mm and two weeks back it was 44mm.Please tell us when can we hope to get complete respite from fever? Doctor: dear friend can you please tell does she feel feverish or the fever is actually recorded on a thermometersecondly are there any other associated symptoms like aches and pains or full body ache.does she feel the small joints of hands and feet ate stiff in the morning if so how long does the stiffness last it is very difficult to comment on your problem without many more detailed information please discuss the possibility of rheumatiod activity with your doctor"
},
{
"id": 153805,
"tgt": "Suggest prognosis of merkel cell cancer with a small inoperable arm tumor",
"src": "Patient: Hi, may I answer your health queries right now ? Please type your query here...Hi My Name is Imelda Jane..... My Father has Merkle Cell Cancer.. He had a large tumor removed under his arm in his Lymph nodes... Its has returned small, painless and inoperable... What path does this henious afflication generally take? Is their a pattern to this rare disease... Doctor: Hi,Thanks for writing in.Merkel cell cancer is a rare and aggressive form of skin cancer. It is influenced by sun exposure and appears as a painless nodule in the skin. If there are lymph nodes involved then it is stage 3 cancer. Since the reports have come as inoperable, surgery might not be the best treatment of choice for him at present.Chemotherapy and radiation therapy are two important strategies that might have to be planned by his doctor and given in a regulated manner. Radiation will limit the aggressiveness of the cancer and reduce tumor size. The chemotherapy will destroy the cancer cells. Please do not worry."
},
{
"id": 221745,
"tgt": "What causes abdominal pain/discomfort during pregnancy?",
"src": "Patient: hi doctor..m 12weeks pregnant and due for an u/s in the coming week .i had previous u/s in 9thweek and all was ok and baby had a heart beat of 174bps.the problem is now m feeling i'll discomfort in my abdomen from the past two days ,sometimes l'll pain too.m very much worried.what can be the possible reason.plz help minu Doctor: HelloPain in early pregnancy may be due to many causes,this need immediate medical attention.After clinical evaluation,you may need ultrasound of fetus for fetal well being.If fetus is safe then other causes like appendicitis,colitis etc should be excluded.You may be prescribed uterine relaxants and progesterone etc.It is important to watch for bleeding/spotting.You should take proper rest.Get well soon.Take CareDr.Indu Bhushan"
},
{
"id": 12957,
"tgt": "What causes itchy, red rashes on chest and stomach?",
"src": "Patient: I have a red rash on my chest around my breast area and on certain parts of my stomach - it is slightly itchy ( i noticed 2 days ago a rash on my legs which look a little like a shaving rash but that has settled down but today it has started on my chest and stomach) I feel great so no other symtoms Doctor: I have gone through your query and it could be an eczematous rash.I would recommend you to apply topical steroid like cortisone cream on the affected areas twice daily for 8-10 days along with frequent and regular application of a moisturising lotion on entire chest and abdomen 2-3 times in a day."
},
{
"id": 36720,
"tgt": "What could peppery taste in mouth suggest?",
"src": "Patient: My adult son has had a peppery taste in the back of his mouth...as if he had put a spoon of black pepper in his mouth. He no longer enjoys eating because of this. Years ago he was diagnosed with HS disease, could this have anything to do with it or is this something we should worry about. Doctor: HIWell come to HCMAt the most chances of stomatitis need to be ruled out if this is not like that then this could be functional condition and not due to any disease, stomatitis can be treated with local mouth gargle and Povidone iodine gargle would be best, else this is nothing to worry, take care and have a nice day."
},
{
"id": 1015,
"tgt": "Can bulky uterus cause troubles in getting pregnant?",
"src": "Patient: Hi Doctor, I'm in the early forty. I just got married and there is no child yet. I had fibroid surgery about 3 months ago. I went for check-up last week and a pelvic scan was done. The result shown slightly uterus bulky. Can i still get pregnant. Thank u. Doctor: Hi, I think you can get pregnant. But as your age is on a higher side, do some blood tests like thyroid profile and prolactin levels and a day 2 FSH, LH AND AMH level. It will tell you regarding your chances of conceiving naturally. Hope I have answered your question. Regards Dr khushboo"
},
{
"id": 62964,
"tgt": "What causes lumps on the mons pubis?",
"src": "Patient: Hi, I ve noticed over the past 6 months that I have tiny bumps that have formed on my mons. It started out as white spots, now they are like mini light colored moles. They do not itch, hurt or bother me at all. Just the sight of it scares me a bit. Any idea what this can be? Doctor: hi.based from your description, yes, it could be folliculitis. this usually occurs after shaving or when a hair is accidentally pulled out and it gets infected. a consult with your gynecologist is best for clinical evaluation. management will be directed accordingly.hope this helps.good day!!~dr.kaye"
},
{
"id": 207804,
"tgt": "How to get rid of entomophobia?",
"src": "Patient: Hi Dr. XXXXXXX,I am a entomophobia, fear of bugs and insects! I was able to control it for many years, up until now!Last year, my job ended up having bedbugs, and I got bit on my ankles! Every time, I see one now, I have panic attacks! I almost hyperventilate! I feel like vomiting at times! Now I am losing sleep at home, and don't know what to do!Please! I need advice! Should I see a Psychiatrist? Doctor: Hi, thanks for using healthcare magicIt would be best to see a psychiatrist for help with this phobia. Phobias are types of anxiety disorders and there are different types that can occur.Most can be successfully treated and cured.Treatment would involve counselling and psychotherapy. Gradual exposure to the source (in your case insects) may also be suggested to allow for desensitisation. I hope this helps"
},
{
"id": 217831,
"tgt": "Need treatment for pain in thumb due to a injury",
"src": "Patient: Hello, recently I fell on my thumb and I heard it click very loudly. I can still move it normally but it hurts when I do move it and I did some research and I think I have Skiers Thumb but I am not sure. If there is any way to help treat this and make it better, please help? Doctor: Such injuries need immobilization in plaster cast for 3 weeks.i strongly recommend immobilization in my patients as it is very prone to convert in chronic pain which troubles for many years otherwise. It also requires radiological evaluation to see if surgical intervention is necessary"
},
{
"id": 66891,
"tgt": "What causes small raised lumps in the foot?",
"src": "Patient: I ve had these small raised bumps on the inner side of my left foot since childhood, however these small raised bumps have become somewhat bigger and have been migrating along the inner side of my left leg as well. I have seen several dermantologist and have received different diagnoses. All I want to know is what it is and how can I get it go away. I have tried a few different steroid creams and topical meds that have seemed to make it worse. It has even been diagnosed as a linear nevea, which could be removed with surgery and leave a long scar. Please help... Doctor: topical medicines won't be helpful in your case; however not to worry about this also!these are small lipoma or neurofibroma-like tumors and you should consult a general surgeon for opinion they should really be removed or not!an FNAC test can confirm the diagnosisall the best!"
},
{
"id": 165910,
"tgt": "What causes frequent fever in kid?",
"src": "Patient: Doctor; my son 4yrs is having fever coming regularly after 5hrs (101 to 103)...He is on Combiflam and Taxim/O for past 3days..the fever keeps on resurfacing and does not subside...It touched 104 yesterday and we did sponging and went to doctor and they prescribed jonac (suppositry) for immediate relief..Looks like he has throat infection...and he keeps having this for the past 3-4 mths...Could it be a case of Streptococci..he has mouth blisters.....Can you please recommend what would be the next course of medicines.. Doctor: just get his bllods checked for any change in cell counts (CBP), CRP for infectionstart on amoxicillin antibiotic till the results"
},
{
"id": 101618,
"tgt": "What causes lip numbness and swelling?",
"src": "Patient: I'm 50 years old. 6\"2\" tall. 210 pounds. I'm having an odd allergic reaction of some kind. My lips are numb and very swollen. I'm not sure but I think I may have started to feel symptoms last night...puffiness. All I can think of that I've done differently is I used \"Just for Men\" on my goatee and mustache yesterday afternoon and may have gotten some on my lips. I've used the product before and I do sometimes itch pretty badly. If it's not that, I hate Snapper last night for the first time around 1am....I don't know if I have some weird reaction to Snapper, I'm just trying to think of all the possiblities. I've taken a couple of Benedryl to see if they help at all. I'm in no discomfort, other than my lips are numb. If you have any ideas, please let me know. Thanks. Doctor: HI, thanks for using healthcare magicIt is possible that it is an allergic reaction. The benadryl would help, it may take a little time for it to start working.It may be the just for men since you have experienced a previous reaction to it. In allergies, the reaction intensifies each time the person is re exposed to the allergen.It may be best to avoid this in the future.I hope this helps"
},
{
"id": 165329,
"tgt": "What could cause stomach pain and the unconsciousness?",
"src": "Patient: My 8 year old son was exercising with his dad (which his does nightly) he complained that his stomach hurt...grabbed it...and my husband said he looked like he could cry and then passed out. What should we do? We have given him water and he is eating (he has not ate like he usually does today and slept a lot today) but he says the pain is gone. Doctor: Causes of syncope in a child may include dehydration, hypotension, neurogenic diseases, cardiac diseases etc.As your child was exercising, complained of abdominal pain and felt unconscious, as per guidelines episode of syncope after exercise needs to be investigated to rule out cardiac causes. Hence consult your doctor and ask for appropriate investigations."
},
{
"id": 17864,
"tgt": "What causes a rapid heart beat with panic attacks?",
"src": "Patient: in two of last three nights I awakened from a dream to notice a fast pulse & I could feel my heart beating ; then I went into a state of shivering for a while. I had a large normal looking stool after I settled down. In very early morning I noticed another bout of fast pulse and then I ll settle down. I don t typically have a stool at night. I ve routinely urinated at night without to date having been medically tagged (e.g. all lab results normal) ; and so I again urinated twice before this bout of shivering hit me. But this shivering is a novel event for me?! Doctor: Hello, Regarding your concern, I would explain that your symptoms could be related to a panic attack or irritable bowel syndrome. Anyway, before coming to this conclusion, I would recommend performing some tests (complete blood count, PCR, ESR and a stool analysis). Hope I have answered your query. Let me know if I can assist you further. Regards, Dr. Ilir Sharka, Cardiologist"
},
{
"id": 53258,
"tgt": "Suggest treatment for cirrohsis of the liver",
"src": "Patient: Hi, my father is cirrohsis of liver patient for the past six years. Currently his report showed decompensated liver disease with active urinary trac infection and renal failure. He also was found to have anemia with no obvious GI bleed. UGI endoscopy showed grade 1 esophageal varices. What should we do? Doctor: Hi, I had gone through your question and understand your concerns.The most important is quitting alcohol and healthy diet. Also, medications which are hematotoxic should be avoided. This is necessary to prevent disease progression which is lethal in most cases. At early stages it can be treated with these measures but but in case of progression, only liver transplantation may help. Diet should be balanced and healthy and get regular exercise. Limit high-carb foods such as bread, grits, rice, potatoes, and corn. And cut down on drinks with lots of sugar like sports drinks and juice. If there is viral hepatitis as udnelying cause then antiviral medications are required.Wish you good health. Kindly regards."
},
{
"id": 215815,
"tgt": "What cause pressure in my neck and a strange taste in mouth?",
"src": "Patient: I have a lot of pressor on the back of my neck and head. the area that it is coming from when you put any pressor on it i get a strange taste in my mouth. my ears wont stop ringing. also im between my period but im having bloody mucuas discharge . the lymph on the sise of my neck are all swollen. Doctor: Hi, The symptoms are not complex and need to be evaluated to make a proper diagnosis. You can consult an ENT specialist to look for any disturbances in your inner ear which can cause similar symptoms. As of now drink plenty of water and you can empirically start on a short course of antibiotic. Hope I have answered your query. Let me know if I can assist you further."
},
{
"id": 184193,
"tgt": "What could gum paleness with pain after bruising of teeth indicate?",
"src": "Patient: My 5 year old hit and bruised all of her front teeth top and bottom about 4 months ago. Just recently, she will cry when I help her brush because the gum at the base of her two bottom teeth \"hurt really bad\". I noticed right at the base of the teeth the gum is white and squishy, only on these teeth. The tissue almost looks dead. What could this be? Doctor: Thanks for using Health Care Magic.Read your query.You have not mentioned in your query , whether she had undergone any treatment after the hit.Was any of her teeth treated?The symptoms described suggests an abscess formed and when pressure is applied in that area is causing pain.I would advice you to visit her pedodontist and get a radiograph done to see the position of the teeth and also the abscess .Also the underlying permanent teeth position is to be evaluated for planning the treatment as soon her permanent teeth will be erupting .For the pain you can give paracetamol as and when required and if required antibiotics after consulting the dentist.Hope this was useful,Thanks and regards."
},
{
"id": 84602,
"tgt": "Will taking Niacin will help to pass the urine drug test?",
"src": "Patient: hi i was wondering if Niacin will work to pass a urine drug test on Monday or Tuesday, they are random? I took two hits of meth on Friday and that was my first time in 4 month. Are my levels going to be high enough to register and can i drink water all weekend to get it out of my system? Doctor: Hi,Niacin might not help in your case. There is no proper scientific evidence for niacin use. The problem with this method is that niacin pills have varying levels of effectiveness from person to person, depending on their height and weight, as well as their metabolism rate. People with higher than average body fat content have reported this method to fail. Niacin intake for flushing toxins can take a toll on liver and cause many other side effects.Hope that was helpful. Let me know if I can assist you further.Regards,Dr. Saranya RamadossGeneral and Family Health Physician"
},
{
"id": 67690,
"tgt": "What causes bump in genital area?",
"src": "Patient: Hello, I noticed I ve have a relatively small sized flesh colored bump towards the very bottom of my general shaft. It s been there for about 4 months now and has not gone away. All i can remember when It appeared was that I had previously shaved there, I always shave my general area and keep it clean but have had ingrown hairs in that area before, however they just go away. I m scared this might be a wart but I do remember around the time I got it, I was not sexually active. I m wondering if It s a general wart or an ingrown hair infection and how I go about handling this. Doctor: HelloWelcome to Health care magic.I went through the history you have provided here. It seems from the history that you are having bumps in the genital area.This is because of inflammation of the hair follicles resulting in furuncle or because of enlargement of the lymph nodes secondary to infection in the area.This can be treated by the use of oral antibiotics and anti-inflammatory drugs.Do consult your doctor for examination for confirmation of diagnosis and prescription of drugs.Do write back to me for further questions.Wishing you a good health.Thank you"
},
{
"id": 9996,
"tgt": "Can Mintop 5% Lotion be used by women for severe hair loss?",
"src": "Patient: Hi..i hav sever hair loss problem so consulted my doctor & he advised me to use mintop 5% solution.He also mentioned that coz u hav severe hair loss I recommend u to use 5% solution..but wen i checkd this product online i came to know dat mintop 5% is for males..I haven t started it.so can u plz guide me wether I should use it or consult my doctor again OR switch to mintop 2% ???? Doctor: HI I have evaluated your query thoroughly you can use 5percent of mintop solution. usually hair loss is More in male than female. so in Google they. mention it like this. we are treated so many females with mintop five percent solution don't worry. apply it thanks for trusting health care magic regards bye take care"
},
{
"id": 165038,
"tgt": "What causes a skin rash after chest infection in a child?",
"src": "Patient: Hi my five year old has been very Ill all week with really high temp I took her to our Doc and she said it was a chest infection & prescribed antibiotic, the temp continued for a full week! Yesterday she broke out in spots like hives (almost like a bee sting) the rash appeared all up both arms and hands on her ankles and lower leg also her ears and face! I brought her straight back to our Doc who said it was some sort of allergy! She gave her a course of steroids but today the rash is still visible although it just looks like dull red spots under the skin is this normal after a reaction like that? Thank you Grace Doctor: Thanks for consulting at Healthcare Magic.Allergic hives may take about a week to resolve completely even after treatment.As your doctor has already prescribed steroids, you ask him to add antihistaminics like Citrizine for early relief."
},
{
"id": 60822,
"tgt": "What causes a painful lump in the right thigh?",
"src": "Patient: I have a mass at the top of my right thigh, in the front, just below the crease area. It s been there for months, but lately has increased in size and seems to be extending in a horizontal direction. It s painful to sleep on that side and causes me to wake up; it is a dull ache and spreads. The best way to describe the throb/ache is like throwing a rock into the water....if that makes sense. It seems to ache right at the spot, then spreads and I can feel it go down my leg. This is recent, the last month or so (the pain and waking me up). Doctor: Hello and Welcome to \u2018Ask A Doctor\u2019 service. I have reviewed your query and here is my advice. You seem to have inflammation/infection of a lymph gland in femoral triangle in front of thigh. You need a course of antibiotic, anti-inflammatory drugs under expert guidance of a surgeon. Hope I have answered your query. Let me know if I can assist you further."
},
{
"id": 100699,
"tgt": "What causes swollen face and lips with numbness on the tongue?",
"src": "Patient: My husband is 64 years of age and is having ? swollen face on one side, lip on one side very swollen. It starts on the lip then travels up the right side of his face. On the first occasion his tongue went numb and he was admitted to hospital and was treated for an allergic reaction to something. Then Instructed to buy an epi pen. He has now had two more attacks and has been given tablets to take at the onset. These attacks last up to 12 hours and have only been the swelling of the lip on the right side. He has had blood test and allergies. Tests today. Nothing is making sense as he wakes up in the morning with it. Can you help me. Doctor: Hi,Thank you for your query. I can understand your concerns.Your husband has angioedema of lips which spreads subsequently to face.Angioedema can affect tongue with numbness due to compression of nerves. He should take tablet Monteleukast (10 mg) and Levocetrizine (5mg) at bed-time for a longer period.Since it has now become chronic ,a short course of oral glucocorticoid (steroid) will help. Epinephrine (epi-pen) will be life saving if the swelling spreads to upper respiratory passage (larynx or voice box) causing acute breathing difficulty.Regards Dr. T.K. Biswas M.D.Mumbai"
},
{
"id": 178637,
"tgt": "Suggest treatment for cracked skin on the labia majora of a child",
"src": "Patient: HELLO for about 2 weeks now when i change my daughters diaper she complains and also grabs like puts her hand over her clitorus she is 18 months btw. so last night i said let me try and see because she doesnt let me really touch and when i see on one side of the over laping skin of clitorus i saw red cracked skin (irritated). I was putting ointment on it a&d for two days to see should i be concerned also she grinds herself to sleep like humps the bed. Doctor: Thank you for posting your question.The redness that your have noticed on the labia majora of your child is likely to be a diaper rash. It maybe avoided by frequent changing of diapers, cleaning of the area and keeping the area dry. Also while cleaning make sure you clean from front to back i.e. vagina to anus. This would prevent any infection from the anal canal to affect the vagina. Apart from that, you may try applying any zinc oxide containing rash cream around the vaginal opening(not inside). Having said so I suggest you also keep a watch for any discharge from the a opening, difficulty in passing urine or increase in rash in which case a medical consultation would be advisable. Try to prevent her from itching the area as it may exacerbate the rash.If you have any more doubt do write back."
},
{
"id": 6918,
"tgt": "Which days are fertile to conceive after periods ?",
"src": "Patient: what are the best time to do baby making .considering if i get my period on 14dec i would be expecting it only on 18th jan.. thats how it goes on with me. whats would be the best time? Doctor: Recollect the last three months and note the LMPs in Dec, Nov and Oct. Now average out the cycle length (Day 1 to next Day 1). Lets say your average cycle length is 34 days. Now, from the day your periods start in Jan, add 34 days. If it starts on 18th Jan, then we come to 18 + 34 = 21st Feb. Then, from 21st subtract 14 days. We come to 7th Feb. Thats the most probable day you are likely to ovulate on. Add 2 days before and after 7th Feb. The best time for you to conceive is 5th to 9th Feb."
},
{
"id": 125170,
"tgt": "Is enlargement of the supraclavicular node harmful?",
"src": "Patient: Hello, I had a fluoroscopic guided CT right shoulder arthrogram earlier this week. Part of the results mention an abnormal enlarged 1.9 x 1.4 cm right supraclavicular node. Radiologist suggests a followup contrast enhanced CT neck for further assessment. Should I be concerned? Doctor: Hello, It could be an inflammatory swelling. As a first line, you can start on a short course of antibiotics. If swelling persists then you can consult a general surgeon and do a fine needle biopsy. Hope I have answered your query. Let me know if I can assist you further. Take care Regards, Dr Shinas Hussain, General & Family Physician"
},
{
"id": 127789,
"tgt": "How to treat intense sciatica pain in the right leg?",
"src": "Patient: hi I have sciatica pain radiating down my right leg. the pain is 5/10 on good days and 8/10 on bad ones. the most difficult time is when i get up from a lying position. it takes me on average of 30 minutes until I am able to stand on my right leg because the pain is just unbereable. thank you. antonio Doctor: Hello,What I can infer from your complaints is that you may be suffering from lumbar disc prolapse with pinched lumbar nerves.I would suggest you to undergo physiotherapy sessions which include muscle stretching and strengthening exercises. If no relief, then you can get an MRI scan after consultation with your physician.Hope I have answered your query. Let me know if I can assist you further.Regards,Dr. Santosh S Jeevannavar"
},
{
"id": 224242,
"tgt": "Severe abdominal cramping after mirena insertion. Can I wait for some time and go to doctor?",
"src": "Patient: I had mirena inserted a littles over 3 weeks ago. I ve had some minor on and off cramping which I know is normal. However, since Monday I have been experiencing severe sharp pains in my lower right abdomen . Usually in the morning shortly after getting up. I have an apoinment with my obgyn Monday to have the mirena checked. Is it okay to wait that long of should I seek immediate attention? Doctor: Hallow, I fully empathise your sufferings.Mirena is an intrauterine device. When with Mirena in you start getting severe pain in abdomen, the cause could be:1. The uterus cannot get adjusted with it and is trying to throw it out. This is more commonly observed in women without any previous pregnancy.2. There is infection in the uterus and the device.3. The device is trying to perforate through the uterus. You will have to report to the Gynaecologist at the earliest. Ultrasonography/X-ray can reveal about the perforation aspect. Rest two will have to be evaluated by the Gynaecologist by clinical examination. You may have to get the device out if it is not tolerated by you. If there is infection, you may require a course of antibiotics. I hope I have guided you with sufficient information."
},
{
"id": 182018,
"tgt": "Suggest treatment for bump on back of tongue",
"src": "Patient: I am just getting over a head cold and still have a dry cough which wakes me up at night. I am taking Mucinex and coughing up phlegm. I am 64, 5 3 , and weigh 150 lbs. I just noticed that there are bumps on the back of my tongue. Do you know the cause of these and if I can get rid of them? Doctor: Thanks for your query, I have gone through your query.The bumps at the back of the tongue can be a enlarged circumvallate papilla. This can occur secondary to any bacterial or fungal infection or trauma. Consult a oral physician and get it evaluated. Mean while you can take a course of antibiotics like amoxicillin(if you are not allergic), do saline gargling.I hope my answer will help you, take care."
},
{
"id": 69654,
"tgt": "What are the small painless lumps in the groin?",
"src": "Patient: I am 25 years old Male. I have small nodules/lumps on all over the skin of my scrotum. I have noticed it since I was 14/15 years old and I never had a check up done for this. There is no pain nor swelling. Can you tell me what this condition is and is it fatal?No one in the family has it. I am 6 ft tall and weighing 60 kg. Doctor: Hi.Thanks for your query and an elucidate history.Dear , this is called multiple sebaceous cysts and is NOT fatal. This is like acne which have not resolved. There is nothing serious into it unless your partner is going to be worried about it. It is always a better idea to get this physically checked by a Surgeon / Doctor who know about it , may be an Urologist."
},
{
"id": 36900,
"tgt": "How long does medication for bacteria infection take to be effective?",
"src": "Patient: How come my prescription for my bacteria infection isn t working? It s been six days since I ve taken it and I feel it hasn t improved. However I had went to a walk in clinic instead of my actual doctor. It still burns and itches down in my female parts. Doctor: Hi Thank you for asking HCMI have gone through your query.The antibiotics took might not be effective.In such case a culture should be done And antibiotic should be changed according to culture and sensitivity.Can you elaborate your symptoms? Burning and itching can also be due to genital herpes infection.You had any history of unprotected Sex?In case of genital herpes antivirals like acyclovir will help.Hope this may help you.Let me know if any thing not clear.Thanks."
},
{
"id": 141841,
"tgt": "What causes severe pain in the lower spine?",
"src": "Patient: Hi I am My name is RE. I am looking at some X rays of my spine and the clinical exam notes. The Impression is, 1) Sacralized lower lumbar transitional vertebral body showing spina bifida occulta 2) Mild convex left scoliosis ICD 9 - 756.17, 737.30, 756.15 After injury My right leg involuntarily jolted from the Glutis Maximums, Quadriceps and Ham string muscle, each time my aorta expanded and contracted near my sciatic nerve. After 18 months the chronic condition stopped by oral medication Voltaren. After stabilization chronic pain remained and instability of the lower spine. Questions, 1) Is there a physical therapy that will be helpful? 2) What diagnostic procedures will be helpful regarding further understanding of my injury and condition. 3) Can a MRI give further insight into my condition for the purpose of therapy. I look forward to your Response, RE Robert Eugene Van Wormer Doctor: Hello Robert!My name is Dr. Aida and I am glad to attend you on Healthcaremagic!I would recommend performing a lumbar spine MRI study to examine the spine and exclude possible bulging nerves, which are causing irradiating nerve pain in the legs. It is also necessary performing a nerve conduction study and some blood lab tests: - complete blood count- PCR, sedimentation rate for inflammation- blood electrolytes- vitamin D levels for possible vitamin D deficiency. Hope to have been helpful!Kind regards!"
},
{
"id": 191769,
"tgt": "What causes eye floaters when diagnosed with diabetic retinopathy?",
"src": "Patient: I have diabetic retinopathy and had a hemmorrage last thursday. Will the floaters in my eye cause me to see shadows that i might confuse with flashes? I did have a retinal tear in the other eye but it has been lasered and has been looked at by my specialist, but could the laser cause me to see lights as well? They are really flashing lights, just more flickers of lights Doctor: Hello, Thanks for the query.Following retinopathy (proliferative) if pre-retinal bleeding occurs, then the blood is not rapidly cleared from the posterior cavity of the eye. This can affect vision. After a certain period floater remain in the eye which can create some hindrance to the vision. Plus after the laser burns some flash lights are seen. These do not affect vision, but will remain. One need not worry about it. But it is essential to get a periodic retinal examination doen to see if there is any worsening of the existing condition. Plus strict blood glucose control is also a must. Thanks"
},
{
"id": 5835,
"tgt": "Trying to concieve. All tests normal. Took medications. No results. Semen comes out of the vagina after having sex. Is that the problem?",
"src": "Patient: Good afternoon, I have been together with my wife for 5 yeras now trying to have babies but no sign of that till this moment. we seen Doctors done sperm count tests, my wife cleaning, taken several medications but nothing. Every Doctors says we are ok. thats fine, Let me just say that I have for all this time just noticed one thing which i may share with you. everytime we had sex, my wife would stay on bed for about half an hour but as soon as she got up I will see the semen coming out of her. could this be the problem? Doctor: Hello Thanks for the query. You should have consulted a proper infertility specialist by now, I hope you have. the phenomenon that you mention is entirely normal. It is very natural for some seminal fluid to get discharged, however, for fertilization, please remember that only a handful of sperms are required. If she doesnt immediately get up after intercourse, the necessary amount has already entered the vaginal and cervical canal. ALso, please make sure the basic laboratory tests, necessary hormonal tests, sperm counts, fallopian tube testing etc has been done. Sometimes, there is a condition called unexplained infertility, for which you should not give up, but keep trying the avenues of ART ( assisted reproductive technology ) under guidance of a Specialist Infertility practitioner. Take care."
},
{
"id": 36391,
"tgt": "Suggest remedy for loss of appetite when on treatment for TB",
"src": "Patient: My grandfather of age 80 years weight 43.6 kg , recently dignosed as tuberculosis and under medice macox plus 450 01 tablet (empty stomach) nulev-500 01 at night randec-150 bd with richnim 1/2 tab bd since last four days . He is suffering frm gastic issue and not hungry . He is afraid that started avoiding the meal. On consult to physician he has given Digestozym syp for three time which also not appears to resolve his issue. Kindly recommend possible solution considering his age and problem. With thanks om ji Doctor: Hello ,I understand your concern. I am Dr. Arun Tank, infectious disease specialist, answering your concern.TB treatment can cause gastritis, it is very common phenomenon.In my advice your father should continue the medication, this treatment requires some time to get adapted. once your father get adapted the treatment becomes very smooth.Prescribed medication was right and appropriate you can continue with it.Your father can take the medication with the cold milk, this can decrease the gastritis and helps in discomfort of stomach. Please do not discontinue treatment because once discontinued it will be very difficult to treat it once again.I will be happy to answer your further concern you can contact me here or you can contact me on bit.ly/DrArunWe wish you a best health at healthcare magic. Thank you,Dr. Arun Tank"
},
{
"id": 47536,
"tgt": "For what purpose is the tablet fabujet prescribed for?",
"src": "Patient: tab Fabuget 40 mg is prescribed for what purpose? and how long one should take these tabs?I was prescribed Zyloric 100 for Uric Acid problem and now this tab is replaced by Fabuget 40 mg. Is this tab is for uric Acid? Kindly reply.My E mail ID is: YYYY@YYYY Doctor: Hi, welcome to HCM.Zyloric and febuxostat, both are prescibed for high uric acid level in blood.Febuxostat is better alternative to zyloric with less allergic side effect.So, continue with febuxostat for atleast 3-4 months and then check uric acid level.It is necessary to continue this tablet if someone has recurrent attacks of gout.Maintain adequate hydration.I think this would be helpful to you.Best wishes. TC.Dr Jay Patel"
},
{
"id": 155984,
"tgt": "What is the treatment for tomors in waist?",
"src": "Patient: I am Tanvir, from Bhubaneswar Odisha (India)- i am having tumor in my waist by which i feel strong pain to stand for a longer time. The doc attending me @ Apollo Bhubaneswar advices me to get it removed at the earliest. is it safe and cost effective to have that operation @ Apollo Bhubaneswar????? Doctor: HIWell come to HCMIn my opinion the lesion first need to be confirm as long as the nature of tumor is concern and for that certain test that need to be done and these are CT imaging and biopsy without this removal of tumor is not advisable, hope this helps."
},
{
"id": 223682,
"tgt": "Shall i give another pill within two days to prevent pregnancy?",
"src": "Patient: i had unprotected sex last nite with my gf,i gave her unwanted 72 within an hour of post sex,but again today morning i had sex with her and now also m not sure, will that pill which i gave her last nite will work or i need to give her another pill..?? her period date is round about 12-14 of the month,i had sex sex on 1.6.14. Please help me.. Doctor: Hello dearI understand your concernUnwanted 72 pill protect only against the one unprotected sexual intercourse.Unwanted 72 pill does not protect you against the second intercourse.I would suggest to take another unwanted 72 as early as possible with in 24 hour after sex to prevent the pregnancy.It cause side effects like withdrawal bleeding, delayed period, pain and fatigue.Avoid stress, take healthy diet and do regular exercise.Hope this may help youContact HCM for further health queryBest regardsDr. Sagar"
},
{
"id": 176310,
"tgt": "Suggest treatment for runny nose and cough",
"src": "Patient: My 4 month old daughter has her first cold (i think) she has had a runny/stuffy nose and a slight cough. Has a hard time drinking a bottle as her nose is so plugged and today the snot has turned green however it is no longer coming out of her nose it s coming out of her eye. About every hour her eye is matted shut with green discharge can the mucus from a cold come out of her eye? I have a Dr. appt for her 4 month check up tomorrow afternoon but didn t know if I should take her in sooner. She is not running a fever and she is eating a little and sleeps a little wakes up often due to the fact she is so plugged up. Doctor: Hello,Welcome to the HCMBrief ..It is chlamydia or neissaria gonorrehea infection.Detailed answer...Since I have gone through your daughter's history,it seems that she has got either one of above mentioned bacterial infection.For that she needs to be swab test done first and antibiotic course for 2 weeks after discussing with your doctor.As it can cause visual problem,she needs to be seen by doctor as soon as possible.Thankyou for your quiry.Regards,Dr.Maheshwari"
},
{
"id": 207572,
"tgt": "How to cure anxiety and depression?",
"src": "Patient: i recently changed prescription medicine from Paxil 30 mg to Effexor (not sure of spelling) 37.5 mg. My dr. advised that it was ok to just make the switch. Stop paxil one day and start efexor the next. It has been almost a month now and I have become angry, no tolerance, and depressed all of the time. Is this normal and will it take more than one month to begin to take effect? < Doctor: DearWe understand your concernsI went through your details. I suggest you not to worry much. Many researches and researchers confirm that medicines alone cannot cure mental disorders. Life style changes, change in thinking pattern, relaxation etc are as essential as medicines. Psychotherapy can help you changing your lifestyle and thinking patterns. Yoga and meditation help you to streamline your metabolism and neurological balance. Please consult a psychologist for further information.If you require more of my help in this aspect, Please post a direct question to me in this website. Make sure that you include every minute details possible. I shall prescribe the needed psychotherapy techniques.Hope this answers your query. Available for further clarifications.Good luck."
},
{
"id": 142621,
"tgt": "What causes indentations in the back of head?",
"src": "Patient: What causes indentions in the back right side of my head that are very painful to the touch and also feels like there is no skin and I am just touching bone. Has been getting worse over the past year. I cannot scratch my head, brush my hair or dry my hair with a towel because it is so sensitive. Doctor: Hi, Welcome to HealthCareMagic.com I am Dr.J.Mariano Anto Bruno Mascarenhas. I have gone through your query with diligence and would like you to know that I am here to help you.This condition requires clinical examinationWe cannot offer any opinion without clinical examinationplease consult a Neurosurgeon at the earliestHope you found the answer helpful.If you need any clarification / have doubts / have additional questions / have follow up questions, then please do not hesitate in asking again. I will be happy to answer your questions. In the future, for continuity of care, I encourage you to contact me directly in HealthCareMagic at http://bit.ly/askdrbruno Best Wishes for Speedy Recovery Let me know if I can assist you further.Take care."
},
{
"id": 3390,
"tgt": "I am looking for a baby and can you explain me what are my problems?",
"src": "Patient: Hai, MY name is padma priya i got married in dec 9 2012.after marriage my periods are irregular and i consult a gynaecologist and she found pcod.As per doctors instruction i follow krimson-35 for 5 months from october 2013 to feb 2014, again i consult the doctor and she made a pelvic scan .The scan report shows normal, but this march month my period date is 12th but i got 22nd ,10days late and she gave me fertyltablet ,primolut tablet,glyciphge.And she prescribed me to take follicle scan on 8th day,10th day,12th day from my period starts.am looking for a baby .can u explain me what are the problems i have ?my weih is 70 g height 5.2 ft.please suggest me Doctor: What you need is a full evaluation with complete infertility work up, hormone profile and all. You may actually need hormones to improve you fertility. But you need a complete evaluation"
},
{
"id": 75488,
"tgt": "What causes recurring pain in chest in a 20 year old?",
"src": "Patient: Hi,im a 20 yr old female,weighing around 200 lbs, and for the past 24 hours i keep having reoccuring chest pains on the right side of my chest. Its a sharp stabbing feeling,it comes and goes. I have no serious medical conditions,besides having seizures back in 2007. Any ideas on what could be causing this? Doctor: If you have no fever, cough and in good general health the pain could be of musculoskeletal region or it could be because of a peripheral nerve getting irritated. Take a pain killer for some time and if pain persists consult your local doctor."
},
{
"id": 80604,
"tgt": "What causes left-sided chest pain when trying to bend wrist?",
"src": "Patient: about 3 months ago i woke up one morning and my chest kinda hurt. the pain was on my left side of my chest and i had trouble getting up and ready because of the pain i couldnt get up fast. the pain comes 3 or 4 times a month now randomly and it occurs when i try to bend or twist. Doctor: Hello dear, thanks for your question on HCM. I can understand your situation and problem. As a rule in left sided chest pain, we need to rule out cardiac cause first. So better to get done1. Ecg2. 2d echo3. Stress test. If all of the above tests are normal then no need to worry much for cardiac cause. You are having simple muscular pain. So take simple painkiller and muscle relaxant. Don't worry, you will be alright."
},
{
"id": 55642,
"tgt": "Suggest possible treatments for hepatitis B",
"src": "Patient: hi i'm a nurse ang I completed the 3 doses of hepa b vaccine..last dec. i am pricked and the patient is a carrier of hepatitis b. The next day, I came to the laboratory and have some test on my blood, it resulted non-reactive so i got my booster on hepa b..am i immune with hepa b?thanks:) Doctor: Hello! Thanks for putting your query in HCM. I am a Gastroenterologist. After 3 doses of Hep B vaccine there is very high chance of immune to hep B infection. If you are still worried then get your Anti HBs antibody titre check, if it is more than 10 then relax. I hope I have answered your query and this will help you . Wish you a good health"
},
{
"id": 223549,
"tgt": "Why is it burning while peeing?",
"src": "Patient: About two days or so, I took plan b bc the condom broke and the next day I had to pee a lot. When I went to go pee i wouldn't pee as much as i used to and at the very end it would burn and hurt. Also when I wiped myself there was clots and little specks of blood coming out. Doctor: Hi,I have gone through your query and understood the concern. According to your symptoms, it appears as though you are experiencing urethritis as evidenced by buning sensation after voiding urine. This can happen due to the trauma incurred during a rough intercourse as the female urethra is superficially placed and vulnerable to injury. The bleeding can be from the urethra or a withdrawal bleeding due to the pill. Please get urinalysis and inform your health care provider for proper management. Hope your query has been answered. Wish you good health."
},
{
"id": 6329,
"tgt": "How long will it take to conceive after hormonal medication ?",
"src": "Patient: Diagonised with PCOS ,thyroid( TSH 7.9)...Taking fertyl50mg and duphaston to conceive...how long will it take to conceive with fertyl 50mg..iam 24 years old Doctor: Hi, getting pregnant with Fertyl will increase your chances of concertion but it is difficult to tell in how much time. Thanks"
},
{
"id": 163159,
"tgt": "Is it safe to give monistat to 6-year-olds?",
"src": "Patient: My child is six yeras old. I am an RN but I do not practice in peds and am uncertain in the safety in giving my six year old monistat. She was recently on antibiotics for a boil. So, she was on it for about a week as well as wound dressings, I don not want to put her into a poor situation again...her boil was quite severe Doctor: Hello,Now, you want your 6 years old child to use Monistat drug. This is an antifungal medication, mostly applied to external (skin) and vaginal yeast infection.If she has used antibiotics orally for boils, this might have led to intestinal candida overgrowth. Probiotics are suggested. Anti-fungal can be used if stool culture is confirming candida overgrowth in feces.Next, if she is having vaginal yeast thrush, then, can use miconazole cream instead of Monistat.Hope I have answered your query. Let me know if I can assist you further.Regards,Dr. Albana Sejdini"
},
{
"id": 47695,
"tgt": "What causes a shadow in kidney on CT scan?",
"src": "Patient: I had a CT scan which showed two stones in my left kidney but also showed a shadow covering one third of my right kidney. I am currently awaiting a second CT scan with contrast. Please could you advise your experience with patients showing a shadow covering one third of their right kidney?ThanksGraham Doctor: Hi, welcome to HCM.Shadow in your right kidney may be a cyst.That requires CT scan with contrast for further evaluation, its staging and further plan of action.Most of the time benign cyst requires no treatment except routine follow up.So, don't be panic. Let the CT scan report come.You can communicate with me further after report. Best wishes. TC.Dr Jay Patel."
},
{
"id": 127383,
"tgt": "How can shoulder pain be treated?",
"src": "Patient: I believe there are a couple problems with my left shoulder. The left shoulder is elevated higher than my right, I am unable to flex my left trap, my rotator cuff may be injured/torn, and I have a minor pain when in my left shoulder blade when I pinch them together. This may be due to my injured rotator cuff. I can flex my right trap, but not on the left. I have tried multiple stretches, but I need more insight as to how I can fix this. Thanks. Doctor: Hello and Welcome to 'Ask A Doctor' service. I have reviewed your query and here is my advice.The pain in your shoulder and decreased rate of motion to your arm might be related to rotator cuff injury.The severity of the injury might be from a strain of the muscle or tendon of the rotator cuff, to a partial or complete tear of one of the rotator muscles.The symptoms may be due to fluid accumulation or calcium deposit within the shoulder joint.To diagnose a rotator cuff problem you need to perform a X-ray to find any joint spurs, MRI is the test of choice for shoulder problems also ultrasound might be used to diagnose problems pf the tendons and muscles.You can use Naproxen daily and Tylenol for pain relief, because only physical therapy isn't always enough for pain managment.Hope I have answered your query. Let me know if I can assist you further."
},
{
"id": 215117,
"tgt": "Burning sansation in palm and sole of feet",
"src": "Patient: Dr. Sir, I have burning sansation in sole of foot and palm sometimes and also burning sansations over legs below the knee. It mostly happens in afternoon and evening in some high. Sir give me the treatement guidelines. Doctor: Hello and welcome to healthcare magic!! Dear Laxman if this burning sensation has started recently it would be better to get a blood sugar levels tested, since diabetes is one of the main causes of peripheral neuropathy. You can also take one Nervijen capsule everyday for two weeks and see if it helps. Since it is affecting only the hands and legs it is indicative of glove and stocking type of peripheral neuropathy. take care!!"
},
{
"id": 85886,
"tgt": "Suggest treatment for severe abdominal pain and bile vomiting without a bowel movement",
"src": "Patient: I have of sever abdominal pain. I work swing shift so he ate lunch at 8pm which consisted of a peanut butter and honey taco wrap. I had a bowel movement at noon but not since then. I puke bile when I go to try and use the bathroom for a bowel movement. I m assuming due to trying to force it. Symptoms occurred about a half hour after lunch. what could this be? Doctor: Hi, Severe pain in abdomen associated with vomiting bile after lunch should not be ignored.I would advise you the following in such a situation to confirm or rule out any cause.Rush to nearest Hospital or ER.X-ray of abdomen in standing position.Ultrasonography of abdomen and pelvis.Tests of blood, stool and urine.Clinical evaluation by a Doctor, preferably a General Surgeon can help to get the probable diagnosis and to be correlated with investigations.If there is a surgical cause like intestinal obstruction or perforation you may need surgery.IF the cause is medical like gastroenteritis or so then medical management will help you.Maintain hydration and electrolyte balance.Hope I have answered your query. Let me know if I can assist you further. Regards, Dr. T Chandrakant, General Surgeon"
},
{
"id": 191350,
"tgt": "What causes burning sensation in the head while suffering from diabetes?",
"src": "Patient: I have type 2 diabetis, which I have had for 8 yrs. I am a 58 year when I wake upold male. The problem I am experiecing is when I awake after sleeping 4 hrs or more, My head starts to get a burning feeling which becomes severe at times. It is almost the same symptoms as when my sugar level drops to 70 s 0r low 80 s. Only intensify it by 100. My hands at the same time begin to burn severely. My mouth and throat become extremely dry which makes me start to cough. My lips begin to swell, as well as my face. I have what would appear to be an allergic reaction to something. However, I havnt eaten or drank anything before the sleep or upon awakening. This has blocked my airway almost completely. Some of these (attacks) are sometimes far more severe than others. The least of these (attacks) will last for 5 minutes or so. The worst is being transported to hospital. Medics Have treated me as that of a reaction. IV hookup with Benadryl and done. I had MRI 2 weeks ago and was given a clean bill on the brain. One last thing, these attacks occur in different places. I feel there is something else seriously wrong, not an allergic reaction. Doctor: This symptoms more often look like thyroid involvement.I would suggest you to get your thyroid profile done and follow up"
},
{
"id": 219029,
"tgt": "What causes weight and persistent abdominal pain during pregnancy?",
"src": "Patient: Hi my name is Dia and I am currently 10 weeks 6days pregnant and I am very fearful for my health. I haben t gained a pound and since finding out I was pregnant I have lost at least 10 pounds. My normal weight is 132.00 but I am 121 at the moment and have been very sick and unable to keep down or digest any real food. I am in constant pain with aching stomach and crippling cramps; both abdomen and back. I do not know what to do. I have no money for an abortion but I also am scared if I continue in this pregnancy I may become seriously ill or die from deteriorating health. Please tell me what I should do. I am very scared Doctor: Hi, Thanks for asking. I understand your concern. Please relax. Loss of appetite,nausea,vomiting in early pregnancy is due to pregnancy hormones... to which pregnant lady is not used to.It goes away as she gets adjusted to it...so relax,it's a normal process. Only thing is you seem to be more sensitive & reactive to hormones... so the excessive symptoms. In such conditiin you aught to get IV fluides along with vitamins &medicines to control nausea / vomiting /low apetite ....followed by normal diet. All other symptims like absiminal & bacj ache need to be examined & trejatejd accordingly. Please viait ER / some charitable or givernment hospital. Thabka."
},
{
"id": 187570,
"tgt": "Will hole in gum line and bone heal itself?",
"src": "Patient: I am 44 years old and had my wisdom teeth taken out 9 days ago. I am in a lot of pain. there is a large hole and cut in the gum line . It appears that a large chunk of bone was broken out and all of my gum is gone next to the last two teeth. will the hole in the bone and the gum line repair itself? Doctor: Hello, Welcome Thanks for consulting HCM, I have gone through your query, as you have undergone wisdom tooth extraction, now you have pain in extraction socket, as you said you have pain and feeling of bone it can be formation of Dry Socket , so for this you go to dentist and get proper Irrigation of socket and Zinc oxide eugenol pack on socket so that healing of socket can occur , If Bony spicule is there in socket go for Alveoloplasty , Do warm saline gargle two - three times a day and take medications advised by your dentist, Hope this will help you. Wish you good health."
},
{
"id": 109327,
"tgt": "What does this MRI of spine indicate?",
"src": "Patient: I am a patient with severe neck pain, shoulder pain and tingling in both my arms. On July 11, 2014 I had an MRI done and it reads, as follows: C6-7 small paracentral disc herniation impressing mildly upon my spinal cord which is new when compared with prior study from 2009 . Because of the pain I am in should I be worried that its impressing on my spinal cord? Doctor: HIWell come to HCMThe report of MRI could be suggestive of disk degenerative disease and this need to be treated accordingly some time cervical traction gives good result in my opinion better to consult orthopedic surgeon else analgesic would be the option, have a nice day."
},
{
"id": 219546,
"tgt": "What are the symptoms of potential pregnancy?",
"src": "Patient: hello! my name is jailene im 18 years old my fiance and i have been trying for a baby we had our first unprotected sex and 2 days later i got the lightest period ive ever had i probably wouldnt have had to change my pad the whole time (if that was sanitary, but thats gross) but it did last aboout 6 days i have no sympotms of pregnancy but ive gained 15 pounds since! am i pregnant? Doctor: Hi.If your bleeding lasted for 6 days and along with the fact that you bled 2 days after having intercourse, it some how rules out the chances for pregnancy. But the whole process of conception itself is a miracle, and so I would still recommend a home pregnancy test using your early morning urine sample about 2 weeks from the time of the unprotected sexual contact.Best wishes."
},
{
"id": 9813,
"tgt": "Suggest treatment for controlling hair loss and dandruff",
"src": "Patient: I am having hairloss and dandruff issue....my doctor prescribed finabald 1mg albolical and minscalp 10% . when i am using minscalp lots of tiny hair follicles are accumulated near my desk and able to see lots of dandruff than before. Any Suggestions? When i applying minscalp i am able to bald area . if it drys up it get covered. But in my office many are able to see my bald are and started asking questuons Any suggestions ? I am using these medicines for a month. Any suggestions Doctor: Hi Dear,Understanding your concern. As per your query you have symptoms of hair loss and dandruff which is mainly due to fungal infection, improper cleanliness and stress leading to telogen effluvium.Need not to worry. I would suggest you to consult dermatologist for proper examination. Doctor may order skin patch test, blood test and physical examination and may prescribe finpecia at least for 3-4 months along with antifungal medications. Take Indian gooseberry powder (half spoon) with water once a day and use almond oil for hair massage. Avoid use of hard cosmetic products, gels, sprays and shampoo's.Hope your concern has been resolved.Best Wishes,Dr. Harry Maheshwari"
},
{
"id": 26957,
"tgt": "Suggest treatment for the keloid formation after heart surgery",
"src": "Patient: I have keloid formation after the heart surgery on the chest. Is it right that I should have injection of Tricot? Is there any side effect, if 5-fluorouracil combined with it ? Should I have cryotherapy as well this this injection? Kindly suggest the right way. Doctor: Hi,Thank you for your query. I can understand your concerns.Intralesional corticosteroid injections (Tricot/Triamcinolone) )can be taken if local therapy has failed. Intralesional injections are more effective on younger scars.When used alone, however, there is a variable rate of response and recurrence.Success is enhanced when used in combination with surgical excision.5-fluorouracil or cryotherapy is not standard treatment.Regards Dr. T.K. Biswas M.D.Mumbai"
},
{
"id": 134545,
"tgt": "Suggest treatment for neck pain after a spinal fusion surgery",
"src": "Patient: I have a bad neck; fusion from C4-C7, with C2 &3 hyp. I have continue neck pain, a knot at the base of my skill. my head feels cold alot. I always have trouble sleeping. I get facet injection and nerve block in the greater region bilaterally. I am still very uncomfortable. Doctor: hello, i am understanding your concern. in this kind of conditions, you need to go for therapies either physical therapy, massage therapy or acupuncture therapy. these therapies will help to reduce your muscle spasm and will subside your pain which is due to some trigger points. i hope these therapies will really help you."
},
{
"id": 153378,
"tgt": "What are the symptoms of potential cancer?",
"src": "Patient: I have been post-menopausal for 12 years. Suddenly, last January, I started some spotting. Sometimes it was light, and sometimes it was dark. I saw my GYN. She did a vaginal ultrasound, a hysteroscopy, and eventually a D&C to be safe. The results were normal with everything. However, whenever she would do the vaginal ultrasound, there was one area that always looked like it had thickened. The bleeding continued off and on. We changed HRT. We changed progesterone. Finally, five months later, I got a second opinion. Same routine. Same tests, except this time I was sent to Dr. Platt, ultrasound guru, and he saw that same area that looked thick and had extra blood flow and said, \"You need to get a D&C now.\" I had another one. Same result. Nothing. Bleeding continues to start and stop. This last week, I started bleeding more than ever before. Sort of like a small period, with red blood some and light blood at other times. I use the Vivelle patch, but only use the smallest dosage, cut it in half, and replace only one time per week. I've done that for 10 years. I use bioidentical progesterone that my GYN had mixed for me. So my estrogen is very little. Last week, I went back to my original GYN, and she did another ultrasound, saw that same area that looks thick, and this time we decided to send me to an oncologist. Last point: I am leaving on Sunday for an international vacation, and I cannot see the oncologist until I return. How to control my fears about cancer? Oh, and my mother had breast and cervical cancer (still alive), my grandmother had breast cancer, and my daughter died eight years ago from sinus cancer. I've always been super healthy. Whew! I know that's a lot. I'm thinking that perhaps the recent change of progesterone has stimulated this bleeding? I'm also wondering what the heck that thickened area is that they keep seeing even after a D&C? Thanks. Doctor: Thanks for your question on Health Care Magic. I can understand your concern. In my opinion, you should not worry much about cervical cancer. You had D&C (dilatation and curetteg) twice and both were negative for malignancy. So possibility of malignancy is very low in your case. So no need to worry and enjoy your vacation. Yes, your bleeding is mostly due to hormonal imbalance and for this you need to consult gynecologist after your tour. Also consult oncologist after tour to remove your fear. Thickened area is mostly due to fibrosis. So at present, no need to worry for cancer. Hope I have solved your query. Wish you good health. Thanks."
},
{
"id": 189392,
"tgt": "Have impression due to tearing of several interdental septae. What could the cause and the remedy for this?",
"src": "Patient: Hello. I'm a dental student and I'm currently trying to take a good two-step C type silicone impression, however once I remove the tray from the typodont I notice tearing in several interdental septae. I can't find any information about the causes of this defect in textbooks, and none of the teachers mentioned this problem. Maybe you could guess what may cause such a defect of the impression? I would be really grateful. Doctor: Hello, Thanks for sharing your concern. While taking impression,you must be careful regarding the force applied applied ont he impression material used. Excess force applied can lead to distortion of impression taken. Please take your own time as you are working on the typhodont. Exact tray selection ,material choosed,adequate consistency of mixing of impression material all these matters. Also apply uniform pressure on both sides of tray. Take care."
},
{
"id": 58764,
"tgt": "Abdominal scan showing fatty liver. Elevated SGOT and SGPT levels. Indigestion problems. Worried",
"src": "Patient: hello doctormyself Deepak.i am a 24 year old boy having problem with my digestion.whn i was 22 hen first time i have got stomach and liver problem .after usg abdomen and liver function test my doctor diagnose me with high level of SGOT,SGPT BILUBARIN(kind of mild jaundice).My doctor prescribed me with some medicine including udiliv 300.after 2 months treatment my liver becomes normal in functioning and stomach was also functions well.but after 2 months again i got stomach infection high level of Enzymes then again gone through treatment and this process is continuous till now repeatedly with some time interval. recently i again having usg Abdomen and i got fatty liver but enzymes level are normal and having problem in digestion(indigestion is my basic problem started when i was 22 and till now continuous).at present my doctor prescribed me with Udimarin Forte,librax and Mebaspa medicines.i feel very weak at present and less of appetite ad less body growth.i am near about 65 kg with height of 5'10.i think this is not a good physic value for a young boy.please help me to get rid of this problem permanently because at present i am just 24 and it is not a age for this kind of diseases.please suggest me how to make make digestion system strong and to make my health better with growth and immunity.waiting for your valuable suggestion. Doctor: Hello! Thanks for putting your query in HCM. I am a Gastroenterologist (DM).Elevation of transaminase levels requires investigation1. Alcohol if you are taking then please avoid it2.Other common and treatable causes like Hepatitis B and C virus, Serum ceruloplasmin for wilson disease especially if you are young, ANA and SMA for autoimmune disease3. Lipid profile4.Ultrasound abdomen to see for liver echotexture, portal vein diameter, and size of spleenWhat is stomach infection? Please mention your symptoms and not conclusion or interpretation. However if you mean by upper abdominal discomfort (dyspepsia) then take a proton pump inhibitor like rabeprazoleI hope I have answered your query and this will help you. Remain in touch and get-well soon."
},
{
"id": 55591,
"tgt": "Does a high sgpt level affect the heart?",
"src": "Patient: sir,my husbands sgpt level is 88,cholestrol is 200,tryglycerides is113,hdl 33,ldl 150 and fasting sugar level is 109.Pleace give valuable advice?Need to take a treatment?how can he control this?There is chance of risk in his heart health? YYYY@YYYY Doctor: Hi, dearI have gone through your question. I can understand your concern. His SGPT level is slightly high. It has no risk of heart disease.But he may have ssome liver ddisease. He should go for complete liver function test and take treatment of that. His cholesterol levels are some what problematic. His HDL cholesterol is low which increases the risk of heart disease. He should take low fat diet with high amount of polyunsaturated fatty acids. He can also take drugs like atorvastatin to control cholesterol levels. It is prescription based medicine so consult your doctor and take treatment accordingly. Hope I have answered your question, if you have doubt then I will be happy to answer. Thanks for using health care magic. Wish you a very good health."
},
{
"id": 212561,
"tgt": "Having lot of stress, confusion, lonely feeling. What do I do?",
"src": "Patient: hi! I m Natasha Baira, i m looking for information that will find Attachment for my pain of feeling and thoughts of who i am, how i feel, what to do, and when to start or how to find that connection in whatever your belief and feeling of life society, and what can i do for myself esteem of inspiring in thought of Affirm in our Ability to our self instinct of knowing who we really are in the mind of our own thoughts of thinking and doing whatever is avoiding stress and lonely at heart of feeling of not knowing ourselves for the right restriction in life of determination. Doctor: Hi there ~ I am glad you are on the forum and it seems like you have a lot on your mind ~ a lot of metaphysical and philosophical nature. You also seem to be suffering from depression and anxiety, as is evident from the language you use. However this can only be determined by speaking with you and taking an entire history for which I suggest that you use a qualified counselor and / or a psychiatrist. Take care and I hope this helped !"
},
{
"id": 191005,
"tgt": "How to clean tounge and teeths of 18 months old baby?",
"src": "Patient: How to clean tounge and teeths of 18 months old baby? Doctor: hello welcome to health care magic well u can use some soft brushes for baby and u can use some cloth and clen it cautiously"
},
{
"id": 82098,
"tgt": "What causes pain in chest during breathing after a fall?",
"src": "Patient: I fell down a few days ago and now my upper left side of my chest hurts when I breathe in. I did not fall on my chest, just on my behind and my right armpit landed on the top corner of a chair. Did I just bruise my inside or could this be more serious? Doctor: Thanks for your question on HCM.In my opinion it is muscular pain only. As you had fall and muscular pain occurs commonly after fall.But we need to rule out internal damage first. Internal damage can be in the form of1. Rib fracture2. Pulmonary contusion3. Pneumothorax etc.So get done chest x ray and rule out all these.If x ray is normal than your symptoms are mostly due to muscular pain.Try to follow these for symptomatic relief. 1. Avoid strenuous exercise2. Avoid heavy weight lifting3. Avid bad postures in sleep.4. Take painkiller and muscle relaxant. 5. Apply warm water pad to the affected area."
},
{
"id": 111508,
"tgt": "What causes lower back pain after UTI treatment?",
"src": "Patient: Hiya,Recently I have been really suffering, I'm a 19 year old usually healthy female. Iv just been treated for a uti but am now suffering lower back pain & sickness. I was ok and recovered till I went out yesterday evening in the car, then it seemed to return ... Any help ? Doctor: Hello,I had gone through the case and found that it might be due to weakness or persisting infection.So go for repeat urine test and take multivitamin Tablet A to Z twice a day and Tab.Calcium once a day for a month.Healthy dietary intake and proper water intake is also needed.Hope my answer will be effective for you.Thanks"
},
{
"id": 12842,
"tgt": "Suggest treatment for skin rashes",
"src": "Patient: My daughter had some kind of allergic reaction which caused a red rash all over her body. They kept her in the hospital in CCU to make sure her breathing wasn't affected. Her breathing was fine so she was discharged. Now the rash is turning from red to brown. Is this normal? It is ALL OVER HER BODY. Doctor: Hi, It is normal for rash to turn brown. It means that the rash has started resolving. Hope I have answered your question. Let me know if I can assist you further. Regards, Dr. Asmeet Kaur Sawhney, Dermatologist"
},
{
"id": 2332,
"tgt": "Why am i not able to conceive even after undergoing ovarian drilling?",
"src": "Patient: 5.4' ,70kg I done - Hysteroscopy, laparoscopy ,ovarian drilling ,D/C under Ga on )05/05/10 under Dr pranab Dasgupta atAMRI. Uters -nomal size, moblie ,surface smooth . Ovaried -both ovaries polycystic ,ovarian drilling done ,serous fluid drained. chromotubation done - good view ,surface smooth ,both ostia visualised Cx -Endocervix health, but still not consive Doctor: Hi , Ovarian drilling alone is not a treatment for PCOD. Try to reduce 4-5 kg weight but diet restriction/ Low calories diet & regular exercise for middle segment of body.Hope GOOD VIEW in report means the tubes were patent. Hope you have confirmed Husband's semen parameters. If cycles are irregular & scan shows no good follicle growth- kindly go ahead with Ovulation induction tablets with hormone injections for eggs to come at right time & number , thus increasing pregnancy chances. PCOS has a very good pregnancy chances, don't worryAll the bestDr.Balakrishnan"
},
{
"id": 26858,
"tgt": "Suggest remedy for chest pain and numbness after bypass surgery",
"src": "Patient: Hello dr my bypass sergeryhas done 3 months berore butwher being cut for opration iv vertical two projectionfrom insidefeeling hard and paining also chest sidefeeling num and pain what todo in this case. pain will be normal or after some monts it wiil be automatically normal pl give me suguest Doctor: Hi, the symptoms you describe is commonly seen post bypass surgery. It generally resolve near completely with time. How much time would vary on each individual. As the sternum is opened up for bypass surgery and during the surgery as many small twigs of nerves are cut while opening up the chest wall and harvesting grafts, you will have the chest pain till the sternal bone would heal and the numbness should decrease over time. Till than you may take symptomatic treatment as per your local doctor and continue with cardiac rehabilitation. Regards Dr Priyank Mody"
},
{
"id": 42035,
"tgt": "What can cause infertility with back pain?",
"src": "Patient: Hello Doctor I have been trying to conceive for 2.5 years now. Have got all tests recommended by the doctor done such as hormone test, thryoid, lipid, iron profile, ovulation test, test to check blockages in fallopian tubes and husband's sperm analysis. Everything has come out normal. What can be possible reason for my repeated failure to conceive? I do not have a regular cycle, 30-50 days. This cycle, however is currently on day 89. I have a few out of the ordinary symptoms, backache since the last 3 weeks, gas and occassionally frequent urination. I had a lab urine test done on day 85 and it came out negative. Will appreciate your advice. Thanks. Doctor: WELCOMEGET YOUR PROLACTIN LEVEL DONE...RULE OUT PCOS IN YOUR CASE...MAKE YOUR CYCLES REGULAR FIRST BY USING HORMONES AND THEN DO REGULAR TIMED SEXTO BECOME PREGNANT...CONSULT INFERTILITY SPECIALIST...OVULATION INDUCTION MAY BE NEXT OPTION FOR YOU..THANKS"
},
{
"id": 1786,
"tgt": "What causes the problem in getting pregnant?",
"src": "Patient: i have irregular periods ranging from 30-40 days my last cycle was nov 9th on dec 17th i had brownish spotting for 2 days only when i wipe been nauseous minor food cravings, lower back and abdomen pain constipation, husband been nauseous and eating more junk food.body feel warm, its dec 28th and pregnancy test is negative still have had period. is there something wrong with me. Doctor: Hi, I think you should go for evaluation first. Do a thyroid profile and prolactin levels and a ultrasound for your uterus and ovaries. Also a semen analysis of your partner should be done. If everything is fine, you can take some medicines for growth of your follicles and track your follicles growth by repeated ultrasound. When follicles reach a size more than 17 mm, take injection for rupturing the follicles. Be in contact with your husband for next 2 to 3 days. Take progesterone for next 2 weeks. Do a urine pregnancy test at home after that. You can try like that for 3 to 6 months. If you have not got your periods still, if pregnancy test is negative, you have to take medicines for inducing periods. Hope I have answered your question. If you have any other query I will be happy to help. Regards Dr khushboo"
},
{
"id": 104949,
"tgt": "Developed large circular flat discolourations on back, hip and shoulder blade. Could it be due to allergy?",
"src": "Patient: Hello: I have developed very large circular, flat discolorations on my back that at first, looked like bruises ie: sort of dark red. I have two that are perfectly round and two that are very large and oblong. I went to my primary care and he stated he thought it was exzema and gave me a topical cream, however, they do not itch , not raised, not dry or flaky. Only knew they were there because someone pointed it out to me. Within the last two days, I have developed two more, one on my hip and one on my shoulder blade. Could this simply be an allergy? I do take Simvistatin (40mg) and have been for approximately 1 year. I am 53 years old and have never experienced anything like this. I live in Florida (humid climate) and have been under stress so not sure if these things contribute. Any guidance would be appreciated. Doctor: hello and welcome to HCM could you please clarify how large the lesions are and where precisely they are located i.e upper back or lower. you mentioned living in a humid climate, do you sweat a lot? do you have oily skin? does any one else in your immediate surroundings have these lesions. i would advise you to have a basic blood test done and visit a dermatologist this condition you are having might be a simple fungal infection(tinea versicolor). allergies to drugs or food usually cause a fixed lesion that appears and goes over time. good luck"
},
{
"id": 62480,
"tgt": "Suggest treatment for a painful lump on the stomach",
"src": "Patient: Hi, Yesterday I woke up with a small bump on my stomach just an inch below my breast. Today it seems bigger and more painful. It is very red. I squeezed it really hard and it seemed to change its shape, a small amount of puss came out but that has not happened since. It is about the size of a grape under the skin. What could this be? Doctor: Hi,From history it seems that there might be having ingrown hair follicle infection producing boil with induration and pus.You might require one course of antibiotic medicine for 3-5 days.clean local part and apply antibiotic cream.Ok and take care."
},
{
"id": 41028,
"tgt": "Does hydrotubation procedure improve fertility?",
"src": "Patient: Hello doc. wish u a great day ahead, doc i have an ovarian cyst in my right ovary but my right tube s clear, after doing my hsg test i came to know that my left tube s blocked but left ovary s fully normal, that s way my doc. advised me to make clear the left tube have to done the hydrotubetion cycle for 3 times after my menstruations, so today i have done my first hydrotubation test n 2 tests r remaining. Now doc. my question s if i done this 3 hydrotubation cycles may i get pregnant? 4 yrs running of my marriage n i never concieved among these period so doc. please suggest me to do the same....my age s 28 and weight s 65 kgs n my height s 5'3''.regardsdevanshiindia Doctor: Dear Devanshi,I have gone through your history. It is not that the egg released from one ovary is always picked up by the to be of that side only. The other tube also can pick up that egg. However, the chances of pick up by the same side tube are more.If the tube is blocked by some flimsy tissue, hydrotubation may break the flimsy tissue and the tube may get opened. However, it cannot be guaranteed. If hydrotubation cannot open the tube, then tubal reconstructive microsurgery need be done. This is a plastic surgery on the tube which will remove the block in the tube and make it patent.I hope this explains you the situation to your satisfaction."
},
{
"id": 100559,
"tgt": "What does exercise induced asthma, cold and blurry vision on waking up mean?",
"src": "Patient: a 21 year old daughter at college. seems to have exercise induced asthma with cold trigger (just recently.) more concerning is two episodes of blurry vision in her right eye, after waking from a nap. Duration less than 5 minutes. One episode had a headache. the Md up at school placed her on prednisone for her lungs, albuterol inhaler, Advair and zyrtec. Could these systems be related to the meds? Doctor: Hello.Thank you for asking at HCM.I went through your daughter's history and would like to make suggestions for her as follows:1. I would mostly agree with her current treatment, but were I treating her, I would add montelukast in her treatment.2. Blurring of vision is not a common side effects of these medicines, so personally I would suggest to report the symptom to her treating doctor who may decide further actions.3. I would suggest her to take plenty of fluids during day as well as to have adequate rest, which will be helpful for recovery.Hope above suggestions will be helpful to her.Should you have any further query, please feel free to ask at HCM.Wish your daughter a very good recovery and health.Thank you & Regards."
},
{
"id": 75720,
"tgt": "What causes chest pain and discomfort?",
"src": "Patient: I am being treated for hypothyroid and went to the cardiologist with chest pain/discomfort. I was given an EKG that had an abnormal result which prompted a nuclear stress test. I took the first part of the test today and the doctor said everything looked good. Is it possible that the new onset of pain/discomfort is just a new normal for me without any underlining cardiovascular issue? Thank you so much for your help. Doctor: Thanks for your question on Healthcare Magic. I can understand your concern. Since your nuclear stress test is normal, no need to worry for heart diseases. Yes, it is possible to have non cardiac cause for your chest pain. Most likely cause in such situation is musculoskeletal pain. So avoid stress and tension, be relax and calm. Avoid movements causing pain. Avoid heavyweight lifting and strenuous exercise. Avoid bad postures in sleep. Apply warm water pad on affected areas. Take simple painkiller like ibuprofen. Don't worry, you will be alright. Don't worry about heart diseases. Hope I have solved your query. I will be happy to help you further. Wish you good health. Thanks."
},
{
"id": 35761,
"tgt": "Does low platelet count, fever and rashes on face indicate dengue?",
"src": "Patient: I have a platelet count of 75000. Age = 48 yrs Height = 162cm , weight = 72 kg I have the following symptoms : Fever rashes on face , foot , leg constant headache since 6 days uneasiness unable to sleep Is it possible that I have dengue? What is the possible course of action? Doctor: Thanks for your query at HCM!I am Infectious Disease Specialist! I went through your query!Your symptoms suggest you are having acute viral infection could be dengue also.I suggest you to investigate yourself for NS1 antigen and Dengue Ig M antibody, Malaria smear and rapid test (ICT) and Widal test along with blood culture.Usually treatment of dengue is symptomatic. Take NSAID to relieve fever.Drink plenty of water and good diet. Take adequate rest. Get your platelet counts monitored regularly daily once/twice. If counts drop below 20,000 you might require platelet infusion.If any unusual bleeding from any site ( Gums, skin, etc) is seen visit your doctor immediately.Happy to take more queries! You can also write a review for me. If you would like some more information, I will be happy to provide. You can take a follow-up query. You can thank if information was found useful.Take care!Dr. Sheetal VermaInfectious Disease Specialist"
},
{
"id": 178112,
"tgt": "What causes red spots on the ears and behind the ears?",
"src": "Patient: My toddler has small red dots on Hus arms, near his ears and on the back of one ear. It looks like he got poked with needles. They are not raised. He is a very fair skin child. I don t know if I should be concerned. This bout of them has lasted about 2 weeks so far Doctor: Hemolytic-uremic syndrome or HUS is characterised by low platelet count (thrombocytopenia) which manifests as pin point subcutaneous haemorrhages as you are describing. They need not be raised from surface. You need not be concerned about these but you need management of the underlying disease process."
},
{
"id": 85022,
"tgt": "What are the side effects of taking Azoran?",
"src": "Patient: hi my name is dhruv and i use suffer from asthama but as gradually asthma is gone my skin has became itchy from past 3 yrs and desease is called atopic dermatis.my skin has improved but doc has given a medicine called azoran i want 2 know for wat purpose its used... Doctor: Hello, Your doctor has prescribed you Azoran which contains Azothiaprine and is an immune suppressant drug. Let me add that you should take a second opinion as this does not seem to be the right drug. In such conditions, I usually prescribe Montair LC once daily to my patients. Hope I have answered your query. Let me know if I can assist you further. Take care Regards, Dr. Prabhash Verma, General & Family Physician"
},
{
"id": 92437,
"tgt": "What could be the reason of having abdominal pain accompanied by diarrhea and dark urine?",
"src": "Patient: Hi my name is Matt I am 24 years old I have really bad abdominal pains on the right side and on the left accompanying with diarrhea thick feeling dark urine and every time I go to the hospital they run my blood test because last time I went based on my white blood count was normal my pancreas enzymes were normal and that's all and they said I had gallstones that there are no need for surgery right now and that's all I got I think are not getting the answer Doctor: HIThank for asking to HCMI can understand your problem this could be intestinal infection which may required antibiotic and the Doxycycline 100 mg just one tab once in day nothing to worry about it even if you have a gallstone it is nothing to take with your diarrhoea this are two different conditions but the main concern right now is your diarrhoea and this will go away soon have nice day."
},
{
"id": 98898,
"tgt": "What causes raised rashes after taking soya nuggets?",
"src": "Patient: Dear docter, I am a 38 yr old lady, weighing 68 kgs.I had taken soya nuggets in food few days back and got red raised rashes all over body after one day , had Avil twice daily for five days , the symptoms subsisded for one day and then again they reappeared.The rashes always come at around 2 a.m in the night.The rashes have itching sensation too.Between these two incidents before the rashes appeared i got breathlessness and felt giddy too.My blood pressure was 130/90.earlier i never had high BP. Then again after two days i got severe allergic reaction on face this time rashes came on face neck, arms and legs.Consuted a docter who gave me Wymlsone 1cc inection and Avil 1/12 cc injection.and tablets Atarax, enzocort and Allegra 180 mg for 10 days .injection was for a single day with which i did get relief.I have a query why is the allergic reaction coming back again.Is it that the allergen is not what I am thinking of can it be something else due to which I am getting exposed and having allergic reaction again. In the past I did get allergic reaction to dark maroon grape juice and once to blanket used in the train. will having enzocort (6mg)once a day(6days)create health problems can i stop having that after 6 days as recommended will stopping it abruptly have any problems? can you help me with my query? thanks Doctor: This is a very good question when you asked ' can it be something else'Yes, it can be anything that may cause allergy to someone. No allergy test can be comprehensive enough to determine full range of allergens. Only a sufferer is the best assessor to find the cause/ causes by closely watching routinely intermittently/ sporadically used things orally or externally to see if something causes reaction or allergy.Every drug has side effects. In some its more, in others its lesser.To our own patients we treat on the basis of Ayurvedic principles to manage the vitiation of 'Kapha pitta' humors. It's a basic treatment where body becomes strong/ immune enough to stand against allergens.Turmeric, Neem and Sugarcandy powders are given in a combo to come out of chaos in a few weeks. No side effects. No interaction, very safe. Haridra Khand - a coarse powder of Ayurveda classics is also medicine of choice in such situations. A bit bitter with unpleasant taste but patient who uses it regularly for some time, is a winner to win over allergy.Hope it helps you too."
},
{
"id": 19000,
"tgt": "Can sinus bradycardia cause constant low heart rate?",
"src": "Patient: I have a low heart rate--48-52 pretty much all the time--I have had a stress test that showed evrything ok--however--I am always exhausted and one of the ekg's showed sinus bradycardia consistent with anterseptial infarct--Since I had the stress test --and it as okay--is there anything to be concerned with Doctor: Hello,I don't know if your electrocardiogram (ECG) seen by a doctor or you just have seen the computer reading. I am considering the first one, so you have past myocardial infarction, and now your stress test was okay, and you have sinus bradycardia and feels exhausted. Well, its good that stress test was okay. If you are taking beta blockers that are common in a patient who had infarction you should see the doses you take and probably low these doses or change the beta blockers. I recommend echocardiography, 24 hours ECG recording and a visit to the cardiologist.Hope I have answered your query. Let me know if I can assist you further.Regards, Dr. Anila Skenderi"
},
{
"id": 175934,
"tgt": "Suggest remedy for persistent diarrhea",
"src": "Patient: My son is 2 and a half y/o. For the last month he has had diarrhea nearly every day, with an occasional solid b.m. every now and then. He isn t having a b.m. any more frequent than usual. I ve stayed away from fruits and sugary foods/drinks but still the diarrhea. He doesn t have any other signs of illness. What could be the problem and how can I fix it? Doctor: Hi...It seems your kid is having viral diarrhoea. Once it starts it will take 5-7 days to completely get better. Unless the kid's having low urine output or very dull or excessively sleepy or blood in motion or green bilious vomiting...you need not worry. But I have few questions for you- Questions:1. What was the birth weight and what is the current weight?2. How is the attaining of developmental milestones?3. Is the stool oily and floats in the pan?4. Is there any colour change in the hair or skin?5. Is there any history of recurrent serious infections?Kindly get back to me with answers and I will be glad to help you.I wish your kid a speedy recovery. You can approach me at the following link. Please find the link below - www.healthcaremagic.com/doctors/dr-sumanth-amperayani/67696"
},
{
"id": 145415,
"tgt": "What causes pressure in head and dizziness?",
"src": "Patient: Hey my head feels weird, like it has a little pressure and slight dizziness at times. My heart feels like if it beating a lil fast and not too long ago while I was in bed I felt like I couldn t move, or open my mouth to talk as if I had a very mini stroke it seizure. Please help. Doctor: Hi,Thanks for writing in.In your case the possibility of a seizure or evolving stroke needs to be evaluated.Dizziness can be due to many reasons and can happen due to decreased blood flow from heart to brain. When it happens, the blood flowing through the vessels from heart to brain might have a reduced flow. These are by the carotid and vertebral arteries on either sides. This can also happen due to momentary reduction in blood flow through the carotid arteries due to the pressure of a cervical rib or degenerative changes in the cervical spine, when you look up, down or turn your head.A clinical examination by the neurologist is essential to know the investigations to be further done for you. While bony cervical ribs or degenerative changes in cervical spine can be seen on X ray, a brain scan and other investigations to study the flow of blood to the brain might also be required as a part of evaluation.Dizziness can also occur due to problems in the inner ear and balance mechanism of the body."
},
{
"id": 3789,
"tgt": "Can unprotected intercourse while on contraceptive pills cause pregnancy?",
"src": "Patient: my name is Heber and I had sex without a condom, my girlfriend is in the second week of contraceptive pills on the 14th pill, and I'm worried she could get pregnant and I'm thinking of giving her the morning after pill, thanks for listening! Heber\"my name is Heber and I had sex without a condom, my girlfriend is in the second week of contraceptive pills on the 14th pill, and I'm worried she could get pregnant and I'm thinking of giving her the morning after pill, thanks for listening! Heber\"my name is Heber and I had sex without a condom, my girlfriend is in the second week of contraceptive pills on the 14th pill, and I'm worried she could get pregnant and I'm thinking of giving her the morning after pill, thanks for listening! Heber\"my name is Heber and I had sex without a condom, my girlfriend is in the second week of contraceptive pills on the 14th pill, and I'm worried she could get pregnant and I'm thinking of giving her the morning after pill, thanks for listening! Heber\" Doctor: HI Heber,Is your girl friend taking contraceptive pills regularly as per instructions. If the answer is yes, then there is no need to use emergency contraception. Contraceptive pill taken correctly is more than 99% effective. It means that of 100 women using contraceptive pills correctly for one year less than 1 woman gets pregnant. So there is no need to give her morning after pill. Don't worry about it.Since there is a small possibility of method failure, if her period gets delayed, do a pregnancy test. I advise you to use barrier method of contraception like condom to protect yourself from STDs.I hope I have answered to your satisfaction. If you have any further queries do contact me through healthcare magic"
},
{
"id": 189511,
"tgt": "Pain on hard palate. Had injury in mouth. Is it the effect of the injury?",
"src": "Patient: My hard palate is very painful to touch on the curve up behind teeth(if that makes any sense). It is very painful to touch. I fell and hit my mouth when I was 12, and was told I wouldn't know of any nerve damage until I was about 15 or 16. I am 15 now. There is no discolouration of the front teeth attached to the once injured gums.. Am I now possibly feeling the affects of the injury? Doctor: Hello, Thanks for writing in. pAIn on hard palate may be due to- Associated decayed/periodontally infected/traumatised tooth. Use of dentures causing allergy also can result in such situations. A thorough clinical and x-ray examination is required to find out the cause of this. Meanwhile,pus if present has to be drained/curetted. Take complete course of antibiotics. Discontinue use of dentures. Hope this helps."
},
{
"id": 101933,
"tgt": "How to treat the difficulties due chemical smell?",
"src": "Patient: Hi, I Inhaled Bleach & Power Germ Chemicals At My Cleaning Job Today About 45Minutes Ago. 15 Mins After Inhaling The Chemicals, I Started To Sweat, Struggled To Breathe And My Throat Is (Wheezy) Also Hurting When I Breathe. What Do You SuggestThanks Doctor: Hello.Welcome to HCM,As you have inhaled the bleach while cleaning it will reach the lungs and causes irritation of the respiratory tract and the eyes.As soon as you are exposed to the bleach you should leave that place and you should come to the open environment so the symptoms will come down. The condition will improve in 30 -45 mins nothing to worry.The inhalation of fresh air will replace this bleach and make you comfortable, so nothing is required for your condition all these symptoms will improve.Thank you."
},
{
"id": 16738,
"tgt": "What is the cause and treatment for high BP?",
"src": "Patient: I am 16 years girl, I am 5 -4 in height and weight 73 kg (160 lbs), It is first time when I had a breathing problem/congestion at night and I was upset, .....then I check my BP, it was 158/80 at that time, I wanted to ask you about this . what is causes and treatment. thanks Kulsoom Doctor: Hello, A single spike in blood pressure could be related to anxiety or the discomfort that you have been experiencing. For this reason, I would recommend you to closely monitor your blood pressure values in the next days. If your blood pressure values are persistently high (above 125/85 mm Hg), I would recommend consulting with your attending physician for a physical exam and some blood lab tests (thyroid hormone levels, complete blood count, PCR, ESR, kidney and liver function tests, cortisol and aldosterone plasma levels, etc.) in order to investigate for metabolic disorders which could lead to secondary high blood pressure at this age. Hope I have answered your query. Let me know if I can assist you further. Take care Regards, Dr Ilir Sharka, Cardiologist"
},
{
"id": 206944,
"tgt": "Is the bipolar disorder hereditary?",
"src": "Patient: year after year I started to discover am not like the other guys around me I feel different, but this year every thing was reviled my uncle had a bipolar syndrome and i may have it so i felt totally depressed and I felt my future is rewind I wanted to kill my self , and when I told my best friend rather than supporting me he make fun of me in front every one and our friendship is destroyed he started avoiding me, and he told his family about me , so then I felt more in depression and I couldn't sleep well and I cryed at night and then started hearing hard rock (metelica) and songs about death , I dont no what to do my life is a hell. Doctor: diagnosis: severe depressiondue to presence of stressor.treatment:do not worry bipolar disorder is not heritable.you are not having any bipolar disorder.just visit a local psychiatrist to overcome your depression. . as your disease is easily treatable by drugs like escitalopram. so nothing to worry.hope my answer helped you.take care.please rate the answer."
},
{
"id": 38558,
"tgt": "How to stop constant coughing at night?",
"src": "Patient: I am taking lansorprazole 30 ml twice a day for ascid reflux. I cough all night and it does not seem to help. Been on since April. Maybe I am taking it wrong? I take it just before breakfast and right before dinner. It is not allergies took meds for that and nothing changed. Doctor: HI, thanks for using healthcare magicThere are different causes for chronic cough: (1) post nasal drip- this is the most common cause of persistent cough. Some persons may have a silent drip where they only have the cough but do not recognize the drip is presentIt is treated by avoiding the allergen if possible and using topical steroid nasal sprays like nasonex, nasocort, rhinocort, flonase, avamys.(2)asthma(3) GERD- gastroesophageal reflux diseaseSome persons may have more than one of these combined to cause symptoms. You may want to combine a topical nasal spray with your GERD medication.I hope this helps"
},
{
"id": 149122,
"tgt": "Giant cell vasculitis of brain, high BP, postural hypertension, stroke, headaches. Intolerant to steroids. What can one do?",
"src": "Patient: I have a dx of Giant cell vasculitis of the brain. My BP is quite high when sitting or standing (roughly 200/100) and normal (roughly 125/60) when lying down. I have real problems with headaches, even worse when my BP is normal rather than high. Several years ago I had a stroke in the right pons, before getting the vasculitis dx. I am intolerant of steroids and NSAIDS. I would like some explanation if you have one. All I find is information on postural hypotension. I have postural hypertension. Doctor: Hi,Thank you for posting your query.Postural hypotension is most likely related to side effects of the blood pressure medications. Please send us a complete list of medications that you are taking at present.If medication as a cause is ruled out, then, we need to exclude autonomic dysfunction as a cause of postural drop in blood pressure. There are several bedside tests that a doctor can perform to exclude a diagnosis of autonomic dysfunction.I hope it helps. Please get back if you require any additional information.Wishing you good health,Dr Sudhir Kumar MD (Internal Medicine), DM (Neurology)Senior Consultant NeurologistApollo Hospitals, Hyderabad, IndiaClick on this link to ask me a DIRECT QUERY: http://bit.ly/Dr-Sudhir-kumarMy BLOG: http://bestneurodoctor.blogspot.in"
},
{
"id": 67243,
"tgt": "What causes a lump that is movable inside the knee cap?",
"src": "Patient: A couple years back and in 13 a plug went in to my knee but it wasn t to bad.it didn t go deep so I didn t go to the doctor my mom just pulled it out and over time it heeled but its a little lump that moves around in my knee it doesn t hurt anymore ....what is that lump moving in my knee cap??? Doctor: Hello there...May be it's a old collected blood due to trauma...Or may be a fat collection...Can be confirmed by simple investigation like ultrasonography...If it become painful or increasing in size may need treatment in form of simple excision..."
},
{
"id": 148759,
"tgt": "Electric shock feeling in back, neck and head. Triggered by coughing or sneezing. What is it?",
"src": "Patient: Hi, occasionaly over the last couple of weeks i have got a weird sensation from doing a few different things. It feels like a fast / sudden vibration or some kind of electric shock that runs from the upper part of my back, up through my neck and then to the top of my head. Acts such as coughing, sneezing or exhalling air quickly seem to be the main things that trigger this feeling. I first noticed it playing basketball a couple of weeks back and thought that maybe it had something to do with me back because it was such a hard vibration feel. I decided to do some quick sprints and some jumping and quickly realised that this did not trigger this sensation. A bit of a weird one for you but it kind of freaks me out a little. Doctor: hi u are having electric shocl like sensation along the spine n this increases on coughing n sneezing. this all happened after playing basket ball. Probably I suspect this to be a kind of radicular or neurogenic pain caused secondary to injury to the spine which would have resulted in cervical disc prolapse causing spinal cord compression.For this u need to take absolute bed rest for 3-5 days,wear a hard cervical collar n take medications like tab Myospas 1 tab twice dailuy for 5 days n tab pantocid one tab before breakfast for 5 days.if it doesnt suside consult a neurosurgeon n he will advise u an X ray cervical spine n if require an MRI if u have any weakness or continuing pain."
},
{
"id": 104947,
"tgt": "Taking Montair LC due to lung congestion from dust allergy. Any side effects?",
"src": "Patient: Hi Doctor, My husband has dust allergy and because of this he has lung congestion (once or twice a week) and he takes Montair LC tablet and bricarex syrup at night.. later, for 1 week , he wont have any problem.. does this tablet Montair LC has any side effect ? what is the remedy for this, as i am really worried.. please help !! Doctor: Hi thanks for your question. Montair LC is Monteluast sodum and levocetrizin hydrochloride is antihistaminics used to treat allergy. Montair L c doesn\u2019t have any serious side effects and most common side effects are: Dizziness,Nausea,Appetite changes,Drness of mouth, unusuall weakness ,Trouble in sleeping ,Mouth pain Hope this answers your question"
},
{
"id": 47851,
"tgt": "Terrible back and abdominal pain after starting Lisinopril for LPHS with smelly urine",
"src": "Patient: I'm 17 years young. For about 3 years now I've been having some kidney problems. Recently they told me that I have LPHS. They put me on Lisinopril. But last night I started having some abdominal and back pains. Im familiar with the back pains. It was the worst! The next morning I went to the rest room and I peed(as usual), but it have a terrible odor. It smelt very much like feces. Thats been going on for about a month now. But this time it looked like I pooped! Just one nugget. It was about the size of a chicken nugget. No blood either. What is wrong with me? what is wrong with me? Doctor: Hi,The diagnosis you have mentioned is rare and that of exclusion.I strongly advise you to undergo thorough evaluation at a good institute/Medical School.There seems to be an abnormal communication between urinary and faecal passage or RPN.Simple MRI & MRU test may be able to resolve the issue."
},
{
"id": 106925,
"tgt": "How can severe backache be treated?",
"src": "Patient: Thank you , I have never felt so much pain, its like every cell is suffocating. when I walk my insides hurt, my head hurts so much, I'm in tears. I had some back pain and they tried to inject something into my spine, the bones were so close together, they tried several times on each side. It's like I went in one person and came out another, for 12 days I've been in extreme crazy pain...please help me Doctor: Hello, I have studied your case.Most probable reason for your symptoms could be postural problem as you may be sitting for long time and working on computer/table for long time.When such patient comes to my hospital we usually take x ray spine or if required MRI to see for any nerve compression.Medication like methylcobalamine with muscle relaxant and analgesic will reduce pain; you can take them consulting your treating doctor.Some exercises which can be done after pain has reduced include- Spine extension exercises, Lying on your stomach flat lift leg 6 inches from ground, do it for other leg.Now lift both hand and leg simultaneously, 6 inch off the ground and stayPosition for around 10 breathes. Core stabilizing spine exercises will help.You may consult physiotherapist for further guidance. He may start TENS, or ultrasound which is helpful in your case.I will advise to check your vit B12 and vit D3 level.Other less likely possibilities can be spinal disc bulge,TB spine Hope this answers your query. If you have additional questions or follow up queries then please do not hesitate in writing to us. I will be happy to answer your queries. If you find this answer helpful do not hesitate to rate this answer at end of discussion.Take care."
},
{
"id": 75920,
"tgt": "Suggest remedy for breathing problem and pain in chest",
"src": "Patient: Hi Dear, Dr. Samuel. N. Grief I am 36 years old, 1.68 cm, 64 kg. I had problem in my heart valve in the past but by medicating it is removed. Now I think I have any thing else in my heart. when I am nervous about study, my children and how I can manage them I breathe quickly and sometimes I have a pain in my chest. I am an active woman, play swimming two or three times in a week and walking a lot. During going up stairs or play swimming fast I have short breath and it bothered me because I am not weak. I would be grateful if you could consult me.with best regard. Doctor: Thanks for your question on Healthcare Magic. I can understand your concern. Since you had valvular heart disease in the past, we must rule out heart related diseases like heart failure or coronary artery disease (CAD) for your current symptoms. Because heart failure and CAD are common after valvular heart disease and cause similar symptoms like breathlessness on exertion and chest pain. So get done ecg, 2d echo and stress test (trade mill test) first. You may need diuretics and other cardiac supportive drugs on the basis of these reports. Don't worry, you will be alright with appropriate treatment. Consult doctor and discuss all these. Hope I have solved your query. I will be happy to help you further. Wish you good health. Thanks."
},
{
"id": 92161,
"tgt": "What could cause bulging in lower right abdomen that has recurred after years, I occasionally indulge in cocaine?",
"src": "Patient: i have a buldge in my lower right abdomen. I have no pain or discomfort whatsoever. I noticed this about 3 years ago but it just went away. Now it is back. I get regular physical examinations. I had a colonoscopy last June. I don't know what it could be. The only thing I do wrong is occasionally a little cocaine. I am a non smoker, very health by practicing yoga 5 time a week and walking 5 times a week. My diet is very healthy. Our rule at home is that we don't buy anything that won't go bad in 3-4 days except canned white beans. I eat a ton of fruit as well.Thank you in advance. Eric Doctor: HiI think first thing you should get done is xray abdomen and ultrasonography along with routine blood investigation.This will ruke out appendicitis or any infection in colon.You need to see if you have dark coloured stool or fresh blood in both urine or stool. Which will rule out colon pathology.If there is bulging or pain while straining or coughing this will rule out hernia.Get well soon.Thank you."
},
{
"id": 87660,
"tgt": "What causes severe stomach pain?",
"src": "Patient: This evening as I was walking downstairs I suddenly got bad stomach pains and had to vomit. I produced a lot of it over a couple of hours. All of my vomit was bright yellow (so it's bile, maybe). I am concerned because all I have eaten today were a few glases each of apple and orange juice so I don't know why I am feeling so sick. Also typically if I have a stomach ache I feel better after I vomit, but this time the pain remains. What should I do? Doctor: Hi.Thanks for your query and an elucidate history. The pain in abdomen all of a sudden which is relieved by vomiting can be due to the following reasons:-Gastritis with outlet obstruction at the opening into the small intestine due to ulcer at the pylorus -Small intestinal obstruction when the vomit is all yellow and bile. I would advise you the following:Get an examination done by a Surgeon as physical examination reveals may diseases better than any tests known.Upper Gi Endoscopybarium studies if the obstruction is expected to be in the small intestine as the endoscope reaches only the first part of the duodenum. Get started on Domperidone and Ranitidine. Take antacid get.Soft bland dietIF no relief , further investigations as already advised."
},
{
"id": 61901,
"tgt": "What causes white lumps around vaginal opening?",
"src": "Patient: Hi, I went to my family planning clinic last week and had a STI test and it came back negative. I am extreamly worried as I noticed about 2 weeks ago white lumps forming around the opening of my vagina and the is also a few lumps forming into a line I have no idea what this could be and it s causing alot of stress, I am too embarrassed to go and be examined by my doctor. Could you suggest what I should do? Doctor: Hi,From history it seems that there might be having some friction bumps or razor bumps causing this problem.Apply triple action cream.Keep local hygiene clean and dry.Avoid wearing synthetic inner wears.Avoid more perspiration.Ok and take care."
},
{
"id": 139155,
"tgt": "What causes right-sided stiff neck, difficulty in concentration and headaches?",
"src": "Patient: what would cause the following symptoms: stiff neck right side Difficulty focusing headache lower cranium right side above hairline nsaids pain delivers do not help at all cubital node swelling right side Clavicle itchy rash right side occurs on and off Extreme fatigue Doctor: Hi, I value your concern regarding the symptoms. I have gone through your symptoms, and in my opinion you would be having tension headache, I would advise you to consult a neurologist for this and he may if required order an MRIHope this answers your question. If you have additional questions or follow up questions then please do not hesitate in writing to us. I will be happy to answer your questions. Wishing you good health.Special note- Any medication prescribed needs to be taken after consultation with your personal doctor only."
},
{
"id": 166107,
"tgt": "Suggest remedy for vomiting with diarrhea with pain in chest",
"src": "Patient: My son is 5. He has been complaining about random pains in his body. We took him to urgent care yesterday morning, when a fever showed up, and his Dr. had no openings. They found an ear infection. He was started on Amox. Then last night he began vomiting and diarrhea. About 5 minutes ago he started complaining that the pain in his arm was back. (it has been here off and on for over a week, but half the time I don t know if he is pretending). However, following his cry about his arm, he grabbed his chest, started screaming that his heart hurt and for me to do something. I called the Dr. and left a message. So my question (finally) is, could this be a heart attack? Should I take him to ER or am I ok waiting for the Dr. to call back? Oh, the pain is in his left arm. the other random pains have included his right leg and his head. Doctor: hi.as per your complaints,it cant be a heart attack.it can be general body aches associated with the fever due to ear infection. kindly wait for your doctor to respond"
},
{
"id": 102288,
"tgt": "What is the treatment for constant cough/cold that has caused wheezing?",
"src": "Patient: Hi My son is 5 + and for the past 3-4 years he has constant cough / cold which results in slight wheezing and he has to be on doulin respules till he is better... is there any other treatment or can be this be overcome with age... our doctor is saying if this continues he would be full blown asthmatic.. .. we had got his allergy test done.. he is allergic to dust and dog dander.. we have removed all the carpets from home an dare trying to house our pet dog in a kennel or open space outside.. please advise Doctor: HIThank for asking to HCMI really appreciate your concern, this is a allergic constitution if I would be his doctor then I would surely treat him with the following treatment, 1) Tab Loratidine one tab three times in day2) Tab Prednisolone 20 mg three times in day for three day ( Reduce the dose by 10 mg every after third day till you reach the last dose of 10 mg three times in day for three days ) after this regime it is better to have Tab Terfenadine once in day for three months, hope this information helps you, have nice day."
},
{
"id": 81268,
"tgt": "What could it be if having menthol feeling when breathing?",
"src": "Patient: My boyfriend has this cold and thinks he may have pneumonia. I looked it up on web MD and am still not sure. The odd thing is that he is complaining now of a menthol feeling when he takes a breath? Is this a sign of a serious condition and should he seek a medical professional? Doctor: Thanks for your question on HCM.In my opinion, you should not worry much about his problem. But better to be sure.Menthol feeling in mouth can rarely be seen in lung infection (pneumonia).So better to get done chest x ray first.If chest x ray is normal than no need worry much for pneumonia.So this is not a sign of serious condition but better to rule out pneumonia by chest x ray.Don't worry much. Be relax and calm."
},
{
"id": 113792,
"tgt": "Neck and upper back pain radiating to ribs, throbbing lower back pain, numbness and tingling in arms and legs. Cause?",
"src": "Patient: I have upper back and neck pain that radiates all the way around to my ribs, under my heart . Just had a stress test and EKG , and all is well. The upper back though seems to never end, and hurts worse when standing or walking. Also my lower back seems to be bothering me lots as well, as it often gets a throbbing, pulse-like feeling in it, and it really hurts there when i tilt my head downward. It dont feel like a muscle, as it feels like its actually in my spinal cord or something. I also sometimes have numbness on the outside of my right thigh, and my arms go numb and tingle often for no apparent reason, especially when lying on my back??? Doctor: hi thanks for ur query u have pain upper back and neck ur stress test and EKG is normal u told about feeling pain while tilting ur head that indicates cervical spondylosis u again told that feeling numbness on outside ur right thigh , numbing of ur arms also.....so these are indicative of nerve compression ao i think u may have a generalised disease of spinal cord may it be generalised demyelinating disease like Multiple Sclerosis thrombing pulse like feeling also suggest it so go for an MRI WHOLE SPINE BOTH T1 AND T2 weighted images and also nerve conduction velocity testing of upper and lower limbs i think u ll be diagnosed by this consult a neurologist soon thank u"
},
{
"id": 62736,
"tgt": "Suggest treatment for painful lump in ribs",
"src": "Patient: I am 28 years old and approximately 112 lbs. 4 feet 11 inches tall. The only medical history I really have is depression, 2 c-sections, and tonsil and adnoid removal. I found a hard lump on my ribs and I wouldn t say tender to the touch, but more painful to touch. Doctor: Hi, dearI have gone through your question. I can understand your concern. You may have some soft tissue tumor like dermatofibroma or neurofibroma or some bone tumour. Yousshould go for x ray local part first. Then if needed go for biopsy of lump. It will give you exact diagnosis. Then you should take treatment accordingly. Hope I have answered your question, if you have doubt then I will be happy to answer. Thanks for using health care magic. Wish you a very good health."
},
{
"id": 47346,
"tgt": "Suggest treatment for a 'mass at parapelvic region' of the kidney",
"src": "Patient: my mother is 50 yrs old. Her CT scan report finding as follows kidney- right kidney shows a soft tissue mass at parapelvic region (24x22 mm).impression - a soft tissue mass at parapelvic region of right kidney. Please suggest possible treatment. Is removal of right kidney necessary? Doctor: Good day and thank you for being with healthcare magic! It depends on whether the parapelvic mass is renal in origin or not. Chances are it might be soft tissue mass (lipoma, sarcoma, liposarcoma) and doesn't need to remove the kidney. Regards, Manuel See IV"
},
{
"id": 138471,
"tgt": "What causes constant body twitching?",
"src": "Patient: Last night I started twitching really bad. And today it s just gotten worse. I ve never twitched like this before. It s like every minute or so I twitch. Whether its a small one or like my body is being jerked. It JUST started last night. What s wrong with me?.. Doctor: Hi,Thanks for your query.It seems that you have muscle twitches or fasciculations. These are small, local, involuntary muscle contraction and relaxation visible under the skin arising from the spontaneous discharge of a bundle of skeletal muscle fibers. You might consult your doctor who can order 1. a few blood tests including thyroid function tests, 2. Serum electrolytes including Serum calcium and potassium levels. 3. Serum B12 levels In case the above tests are normal further testing can be done including - nerveconduction studies, Electromyography, etc. These tests might help to diagnose any underlying neurological issues. I do hope that you have found something helpful and I will be glad to answer any further query.Take care"
},
{
"id": 213633,
"tgt": "What is the best medicine for ocd, anxiety and depression ?",
"src": "Patient: what s the best antidepressant for ocd/ bad anxiety and depression i am currently on Remeron 45mg 1 at bedtime and Klonopin 1mg 1 in the morning and 1 at bedtime. but my doctor also wants me to start taking Celexa 10 1 at bedtime, but is that okay to be on both antidepressant s at the same time? and how can i get my psychiatrist to up my Klonopin dose upped to 2 mg or should i get on a different antianxiety medicine? Doctor: Hi, It is usual for psychiatrists to combine two anti-depressants when one medication is not effective. However, it is advisable to put in place an individualized plan of management (especially for OCD and anxiety) which includes not only medicines, but also non-pharmacological methods like behaviour therapy and relaxation techniques. Continuing Klonopin at high doses for long duartion should be avoided as it can cause dependence. Discuss with your psychiatrist about your preferences and the most bothersome symptoms which you want him to target first. This will help in setting up priorities after which problems can be dealt with accordingly."
},
{
"id": 129377,
"tgt": "What does this MRI result of the left knee post injury indicate?",
"src": "Patient: I had a fall on black ice in Dec that injured my left knee and was wondering if a grade 3 focal chondral defect to the medial femoral condyle and an enlarged plica would have been a result of that fall. I never had any previous issues with my knee prior to the fall. I also sprained my MCL and hamstring. Have you seen this before? Doctor: Hello.Thank you for your question.I think that the osteochondral defect can be a result of your fall but not the enlarged plica. The plica is something that you were born with but did not caused you any trouble.Now if the knee is swollen and painful for long time and does not get better, I suggest you to see an orthopedic surgeon as you may need arthroscopic ( with camera) surgery for osteochondral defect.I hope i answered your question."
},
{
"id": 217924,
"tgt": "Suggest treatment for severe pain in left chest and shoulder",
"src": "Patient: suffering from severe pain in left shoulder and left chest ...pain is so much that i'm unable to breath properly ...its paining while i take a deep breath ...when i lean or change my position the pain increases for 10-15 secs then it became normal ...please help ... Doctor: This pain cud be due to sum pleuritic pain, or mucle catch, or related to chest infection, or heart related, or gastritis. Pls meet ur physician,he will help u out"
},
{
"id": 188241,
"tgt": "Have black spot inside mouth. Normal X ray. What are the upcoming risk?",
"src": "Patient: I'm from Middle East. My question is that I had a very little black dot or spot inside my mouth, specifically in the soles of the cheek. It has been in my soles cheek for long time prob more than a year. The dot or spot that I had is normal which means I have never seen any strange changes on it. There is no changing in this dot. I had X- rea for my cheek at my Dental clinic, and they said nothing dangerous in my mouth. Is it normal or something will be dangerous in the future?? Please answer me I'm very tense and worried. ): Doctor: Hello, thank you for communicating with us for your problem. There can be many reasons for this black spot. But if there is no problem since one year and there are no changes associated with that then there is nothing to worry it can just be a melanin pigmentation. but still you have any adverse habit of taking areca nut, pan masala or gutkha then it will be much better to give a regular visit to your dentist."
},
{
"id": 97505,
"tgt": "Suggest better medicine for high blood pressure",
"src": "Patient: my husband has been on lisinipril (20mg) for high blood pressure but it hasn't been successful in controlling it. today the nurse practitioner changed his prescription to etodolac 400 mg I looked this med up and its for inflamation fo arthritis should he call the doctor tomorrow? Doctor: Hi there, thanks for posting your query on this forum. Hypertension if not controlled can damage arteries, heart, kidneys, brain, eyes, etc. If lisinipril is not able to get blood pressure to normal you must consult the doctor instead of the nurse, he will change the anti-hypertensive depending on the history that your husband has. Good that you were alert and prompt in rectifying the error of the nurse.Depending on the response of the other anti-hypertensive he may decide whether to do any further investigations or not.Thanks & Regards,Dr. Kunal Lokare"
},
{
"id": 37460,
"tgt": "Suggest cure for pain and swelling in finger due to dog bite",
"src": "Patient: I was bitten accidentally by my dog nearly 2 weeks ago on my right hand, It became very infected and I had two courses of antibiotics, the wound is now clean. My GP has said that I can leave the wounds uncovered now and they are drying up well. However my finger is still swollen and painful. I have got co codamol and naproxen. Is there anything else I can do to help? thank you Doctor: Hello,I understand your concern.I am Dr Arun Tank, Infectious diseases specialists, responding to you.I suggest you to take the vaccination that is rabies and tetanus toxoid.Rest of the things are right. You can add serratopeptidase to add up in healing process. Nothing more is required. I will be glad to answer your further query. Thank you for your contact to health care magic.Wish you a best health.Thank you,Dr Arun Tank."
},
{
"id": 78135,
"tgt": "What could cause a person to get choked?",
"src": "Patient: What could cause a person to get a choking, gag typeFeeling for no apparent reason, or when it is cold outside,when excited, nervouse,like a job change. After singing a few songs. Been having this happen since I was about 18,I'm 32 now. Been getting a little worried about it. Doctor: Hi. I can understand your concern. You could be having asthma or simple anxiety.Possibility of asthma is more in your case. So better to consult pulmonologist and get done clinical examination of respiratory system and PFT (Pulmonary Function Test). PFT is needed for the diagnosis of asthma. It will also tell you about severity of the disease and treatment is based on severity only. You may need inhaled bronchodilators and inhaled corticosteroid (ICS)Don't worry, you will be alright. Hope I have solved your query. Wish you good health. Thanks."
},
{
"id": 177985,
"tgt": "What are the side effects of MMR vaccine in a child?",
"src": "Patient: Hi, My Daughter is 17 months old and she was immunised with the MMR vaccine 4 days back and she developed fever since last 2 days (today is 3rd day). The temperature is shooting up to 102F, we are giving her the paracetamol+ibuprofen (combination) suspension. however it is working temporarily and she is getting back the fever again in 6 hours. She is not eating anything and not even opening her mouth for any thing, apart from drinking some spoons of water. She is still on breast and always preferring only that. Please advise Doctor: Hi, I'am Dr Suresh K Yadav MD (paediatrics), I had gone through your question and understand your concerns,It is not due to MMR vaccine as fever after MMR vaccine usually occurs after 7-10 days , and such a high grade fever is not usual after MMR vaccine . Moreover fever is running on 3rd day, her appetite is poor , she needs to evaluated for cause of fever and need of hospitalisation and intravenous hydration . Meanwhile continue antipyretic plus add tepid sponging to bring down fever.Hope this answers your question. If you have additional questions or follow up questions then please do not hesitate in writing to us. I will be happy to answer your questionsTake care."
},
{
"id": 192988,
"tgt": "What causes erectile dysfunction and premature ejaculation?",
"src": "Patient: Hello Sir,I have been masturbating since 4-5 years. Now I am 22 and now a days while masturbating my sperm fall very soon and the erection which comes is for a very shorter period of time and i think i'm suffering from genetal dysfunction. pls advice.Lucky (22) from Jamshedpur. Doctor: Hi, It can be due to hormonal imbalance, stress-induced and varicocele. Hope I have answered your query. Let me know if I can assist you further. Take care Regards, Dr S.R.Raveendran, Sexologist"
},
{
"id": 3250,
"tgt": "How to conceive while suffering from PCOD?",
"src": "Patient: Hi I m,ruchi 7 years married 32 yr old and I hav a 6 yr daughter.I m trying to concieve for more than a year.I hv pcod all other reports are normal my doctor has prescribed me mefomin and cervifert..now I m very worried weather I ll be able to conceve or not . Doctor: Hello dearI understand your concernPCOS is very complex disease and difficult to manage sometime.But do not lose hope, pregnancy is possible once hormonal imbalance and insulin resistance has been corrected.Main aim is to regularize the period and correct hormonal imbalance.I would suggest to take treatment under the advise of the gynecologist.Clomiphene citrate first five day of period and progesterone pill in second half of the period will usefulContinue metforminFSH analogue can be usefulTake proper treatment for 6-8 months.Avoid stress, take healthy diet, drink plenty of water and do regualr exercise (reduce 10-12 % body weight)Hope this may help youBest regardsDr. Sagar"
},
{
"id": 201696,
"tgt": "What is the treatment for erectile dysfunction?",
"src": "Patient: his pennies become weak while doing intercourse .what is the problem i do not understand and feeling afraid. so how can i solve this problem. i want solution urgently now. can u plz tell me now. not in the email..because i don t continue my email account Doctor: you have not describe proper and complete details of your problem but i will try which will help you. this may be due to some anxiety, stress or tension with you or your family if its is sudden. it may occurs in diabetic patients and alcoholics. first try to overcome your fear or stress or anxiety or stress. discuss your tension with your close friend or family member. if personal than freely talk with your wife. it will help you more. still problem persist than consult sexologist and take therapy. without consultation dont take any medicine from outside....."
},
{
"id": 27379,
"tgt": "What causes stomach drained of brown fluid after pacemaker surgery?",
"src": "Patient: white male age 65 morbidly obese pacemaker uses walker. broke left hip in a fall had to wait to reduce coumaden when turned on side for surgery experienced air way obstruction lungs drained of flem unconcious and not breathing on his own for days now is having stomach drained of brown fluid. Doctor: Patient on pacemaker, may be on aspirin. It can cause g I bleed. Moreover, as patient is obese may have obstructive sleep apnea with metabolic syndrome. Requires immediate ICU care and treatment."
},
{
"id": 147803,
"tgt": "Any suggestion for sharp pain caused after hitting left side of spine?",
"src": "Patient: Hi! I slipped on the ice and fell down the front steps two days ago. I hit my back on a step on the left side of my spine. There is no mark on the skin, but I have a great deal of pain. The pain was not as intense on the day it happened. Yesterday it became greater and today it is even worse. Movement is not limited, but when I eat, the pain grows more intense. I have been using ibuprofen, 800 mg. Should I do something more? Doctor: Hi,Thank you for posting your query.It is unfortunate that you sustained injury after slipping and falling on ice. The pain seems to be due to local injury and is not suggestive of any nerve or spinal cord injury.You could continue ibuprofen tablets. For additional pain relief, you may also take tramadol tablets. In addition, local heat/hot water bag application would also be helpful.I hope my answer helps. Please get back if you have any follow up queries or if you require any additional information.Wishing you good health,Dr Sudhir Kumar MD (Internal Medicine), DM (Neurology)Senior Consultant NeurologistApollo Hospitals, Hyderabad, IndiaClick on this link to ask me a DIRECT QUERY: http://bit.ly/Dr-Sudhir-kumarMy BLOG: http://bestneurodoctor.blogspot.in"
},
{
"id": 50125,
"tgt": "Underwent renal transplant. On medicines for back pain, uric acid, erection problem, bp, blood sugar, stomach ache. Advice?",
"src": "Patient: i am 35 year old male undergone renal transplant on 22/11/2011 after 1 year of hemodialysis . donor is my mother of 64 year After transplant various treatment has been followed 1.5 time plasmapheresis with IVIG 2.treatment for CMV PCR 3.Treatment for border lineACR but the serum cretinine level is still range bound to 1.7-1.9 other problem like back pain,uric acid,erection problem,blood presure & blood sugar controlled with oral medicine,irregular stomach medicine i am taking is pangraf 3mg BD Myfortic 720mg BD Aten 50mg dilzem cd 180mg semcal alfa 500mg xyrolic 100mg septron apriglim-2 gezo-sr 500mg kindly advise Doctor: I would look at your tacrolimus trough drug levels as you seem to be on high dose Dlizem CD+Tacrolimus combination. If your tacrolimus drug levels are more than 5 units talk with your doctor if your tacrolimus dose can be reduced.*** Problems associated with renal transplant are unique to every patients and do not do anything without consulting your personal doctor. This includes what i had suggested you***"
},
{
"id": 128044,
"tgt": "How can swollen purple discolouration on the foot be treated after a bone fracture?",
"src": "Patient: Broke the three middle bones in my foot 12 weeks ago. Still dealing with swelling and my foot turning purple especially around my big toe. Not sure if this is normal or something to be concerned with. A compression sock helps with both symptoms however if not wearing the pain comes back. Thank you. Doctor: Hello These symptoms could be due to vascular occlusion around your ankle.Colour Doppler may help in diagnosis.This can lead to purple toe.Hope this answers your query. If you have additional questions or follow up queries then please do not hesitate in writing to us. I will be happy to answer your queries. Wishing you good health.Take care."
},
{
"id": 86929,
"tgt": "Suggest cure for upper abdominal pain and cough",
"src": "Patient: Hi, I began with upper abdominal pain, more above the ribs also. A dry cough started 24hrs after this which caused pain in the upper abdomen when I coughed. I am now coughing up a white frothy sputum along with some green thick with it. Last night I had night sweats and today I have some muscle aches. I have also noticed some swelling in my knuckles since this occured. I get boughts of rapid breathing. I have felt slightly better after some rest. Doctor: Hello.Welcome to HCMThere are different causes of such symptoms .But needs proper evaluation to diagnose .i shall advise you to takeTablet paracitamol 650 for feverTake any mucolytic cough syrup with salbutamol combinationTake quinolone antibiotic Ppi tabletsIf still does not subside do a sputum for afb and consult your physician.Hope this is helpful..Regards."
},
{
"id": 139793,
"tgt": "Suggest treatment for GB Syndrome and squint eyes",
"src": "Patient: Hello Dr, My son affected by GB Syndrome 2 weeks back. At that time his eyes are also affected and looks like squint. Doctors told it will be automatically. But we need 2nd opinion whether we need to take separate treatment for the eye problem. Please confirm do we need to go for separate treatment? Thanks and Regards, Siva. Doctor: Hello, If the Guillain-Barre was triggered by the Zika virus, the red eyes should improve in a couple of weeks. Topical solutions (eye solutions) with cortisone may help somewhat. Discuss with your Doctor for these issues. Hope I have answered your query. Let me know if I can assist you further. Take care Regards, Dr. Erion Spaho, Neurologist, Surgical"
},
{
"id": 220334,
"tgt": "Is it possible to have an ectopic pregnancy?",
"src": "Patient: I went for a checkup with my OB last friday and there is a chance that I am pregnant in my left tube. My hcg levels were tested 3x s and keep doubling. My ob seen a mass in my tube and a very small mass in my uterus. Is it possible to have an ectopic pregnancy and have doubing levels? Thank you Erin Doctor: HI, Thanks for the query. I have read & understood your concern. Beta HCG level that shows doubling in consecutive readings .. may not be a distinctive sign to diagnose an ectopic pregnancy. There is usually history of pain in one side of abdomen/ slight bleeding in case the pregnancy is ectopic... These are alarming signs & once experienced, the trans vaginal USG is indicated for diagnosis. Your USG with a mass in tube & another small mass in uterus is a bit confusing, you should proceed for follow up USG for confirmed result. Thanks."
},
{
"id": 50010,
"tgt": "Test shows kidney appearing echogenic, compensatory hypertrophy, renal parenchymal disease with chronic cystitis. Explanation?",
"src": "Patient: please tell me the meaning of this one please both kidney appears echogenic. RT kidney smaller in size meas 6.5c.m. Lt kidney shows mod compensatory hyperprophy with enhance echo geniciy meas 9 c.m conclusion ; bil. Renal parenchymal disease with chronic cystitis (adv.All relevant renal investigations) please sir\\mam tell me about all that in a simple English i didn t get that scientific words or tell me about in which kidney have more problem. Doctor: HiThanks for the query.Your reports indicate two problems:1) There is evidence of disease /damage to both the kidneys with right substantially more than the left.2) There is evidence of long standing infection in the urinary bladder.Please see a nephrologist for further appropriate treatment.Hope this helpsGood luck."
},
{
"id": 102405,
"tgt": "Treatment for Asthma,reflux cough,esophageal spasms with anxiety",
"src": "Patient: Asthma/reflux cough and esophageal spasms and anxiety. How do I get rid of this??? Zantac makes the spasms worse, Nexium causes jittery shakiness and afraid to try anothe rmed. Take prilosec 20 mg in the am and gavascon as needed. Nebulizer for asthma. Cough is worse in the cold but also seems gerd related. I'm a mess. Have Barretts and Gerd. Doctor: Hello dear,The symptoms as mentioned in your post are all related to each other.Gastro esophageal reflux disease is caused by the reflux of acid from stomach to esophagus due to relaxation of the sphincter at the lower part of esophagus.The cough can be explained on the basis of aspiration of refluxed acid.The role of anxiety needs to be ruled out as an etiological factor.Management consists of:1. Lifestyle modification- like weight reduction, sleeping on the left side, or with upper body raised.2. Dietary modification- avoid consuming irritating substances such as acidic & rich foods, caffeine, nicotine, and alcohol.3. Medical management- Proton pump inhibitors like Omeprazole, Pantoprazole,etc. (which prevent acid formation in stomach)4. Nebulization + Bronchodilator for relief from asthma.If symptoms still persist, it will be better to consult a Gastroenterologist.There is no need to worry...you will be fine.Wish you a good health & speedy recovery.Take care."
},
{
"id": 194872,
"tgt": "Can Drotin A be taken for pain during erections?",
"src": "Patient: Recently operated for BNI & BPH. Now there is a painful errection in the night while in sleep & pain is at the place where penis is attached to the body & not the other end. It pains also while passing last few urine drops. Urologist has prescribed Drotin A to take when the pain starts. Please adviise why drotin a has been prescribed for this Doctor: Hello, Drotin A is a brand name of Drotaverin and it is a muscle relaxant and it relieves the spasms and can be helpful in painful erection. Nothing to worry and your treatment is on the right track. Hope I have answered your query. Let me know if I can assist you further. Take care Regards, Dr Shinas Hussain, General & Family Physician"
},
{
"id": 54246,
"tgt": "Could pain in/around stomach while sneezing/coughing/breathing be due to gallstones?",
"src": "Patient: hi i think i may have goldstones i went to see the doctor yesturday & she said it could be bt im too young.my mum said i not tho ive got a reallly bad pain in my stomach and up my right side it hurts when i sneeze, cough and even breathe in. the pain keeps moving around my stomach and up both sides and i can hardly walk its that painful. do u have any ideas ? Doctor: Thank you for posting your query.history of epigastric discomfort/ abdominal discomfort and also assoicated with eating.further evaluation needed.what is your age? height? weight? bowel habits and color? nausea?vomiting>differential is: GERD, gastritis, ulcer disease, herniaif your youngvisit gastroenterologist and test for H.pylori and get an ultrasound of abdomenOR take famotidine tablets before meal for 2 to 4 weeks.if the H.pylori test comes positive, then you might need eradication therapy.meanwhile:- include yogurt in your diet - use two pillows under your head, when you lie down/sleep.- eat your last mean 3 hours before going to bed-Avoid spicy food, oil rich food, - No smoking and NO alcohol. - do NOT eat mouth full and swallow air while chweing food by keeping lips zipped.-no cold drinks and NO junk food- try to avoid drinking water at the end of a meal.if any further questions, feel free to ask.Health professionals aim to diagnose properly and manage patients according to their limited knowledge. Cure is blessed by the ONE who Created us, whose power and knowledge is unlimited .wish you good health.regards,Dr Tayyab Malik"
},
{
"id": 57851,
"tgt": "Which is the best treatment to cure fatty liver with mildly coarsened echo texture?",
"src": "Patient: Hi Sir, I came to know through blood test reports that i got HBsAg Positive and my doctor suggested me to do ultrasound scan and in that SGPT result is 99 and in IMPRESSION: Fatty liver with mildly coarsened echotexture, Mild splenomegaly. Is this so dangerous to health? Is it curable? How may months required to cure this?What will be the best treatment to cure this? Doctor: a low fat diet with a low calorie count in toto along with regular physical exercise ,preferably intensive workouts is the only way.Non-Alcoholic Steato-Hepatitis or Nash/commonly called fatty liver is a life style disease which needs to be treated by diet and exercise."
},
{
"id": 82860,
"tgt": "Can I take plenquil to treat discoid lupus and sjogren s syndrome?",
"src": "Patient: Almost 2 years ago, I was diagnosed with SLE; specifically Discoid lupus and sjorgen s syndrome. I am currently prescribed and taking 400 mg of plenquil daily. Lately and especially right now, it s feels like every joint in my body aches. Why is that? Doctor: Hiplenquil is the anchor drug for this disease. It should be continue. but if you have pain and aches, you need to see rheumatologist for evaluation.take care"
},
{
"id": 10075,
"tgt": "Recommend treatment for hair loss during trichodynia",
"src": "Patient: I strongly believe that I have trichodynia, my scalp in the middle of my head is always sore and sometimes very painful and it causes headaches not only that I keep suffering from hair loss in that area. What can I do to stop the pain and hair loss for good? Doctor: Hi, It may be folliculitis. Consult the dermatologist for the perfect diagnosis and proper treatment. A course of antibiotic like Doxycycline along with anti-inflammatory drugs would improve the condition. Cleanse the scalp with Savlon liquid during a bath. For hair loss, take tab. Biotin. Hope I have answered your query. Let me know if I can assist you further. Regards, Dr. Ilyas Patel, Dermatologist"
},
{
"id": 149815,
"tgt": "Withdrawing from Cipralex, been on lexapro, started cipralex. Will destroy my brain?",
"src": "Patient: I am withdrawing from Cipralex , 10mg. Have been on Lexapro for nearly 15 years and moved to Canada- the doctor here started me on Cipralex. It was 20mg, but, not helping me so I decided to get off of it and cut it in half for a year. I am having what appears to be (by other posts on here) classic symptoms of withdrawl. I am taking St. Johns Wort as directed for a week now and have been 3 weeks off the Ciprolex. How much time does it take for me to feel like myself again! Have these doctors destroyed my brain with all this medicine?? Doctor: Hi,Thank you for posting your query.These abnormal symptoms would disappear in 4-6 weeks.There is no need to worry, as antidepressants do not cause any permanent damage to brain. You would be well soon.Please get back if you require any additional information.Best wishes,Dr Sudhir Kumar MD (Internal Medicine), DM (Neurology)Senior Consultant NeurologistApollo Hospitals, Hyderabad,"
},
{
"id": 72984,
"tgt": "Suggest treatment for persisting chest infection",
"src": "Patient: I have had a chest infection for about 3 weeks, had 2 lots of different anti biotics, an x ray which was clear and now I am losing my voice but I do not have much pain in my throat, only when I am coughing. I am also coughing up white phlegm which alternates in colour from green to yellow and white again. Please help Doctor: you lung infection cause and your antibiotics detail needed, but you can go through sputum examination ; sputum for gram staining, bacterial culture sensitivity, and also fungal staining. you will get your answer"
},
{
"id": 100284,
"tgt": "Can Montelukast be used as preventive measure for asthma in a child?",
"src": "Patient: my child of 8 yrs have allergic asthma and during winter they suffer the 1-2 episodes. This time she is well after therapy of nebulization with clenil and clenilc compositum,also taking 4th or 5th time course of montelukast 5 mg.My Q is her doctor has advised clenil nebulization with 0.5ml daily for more than few weeks even she remained on nebulizatioln for 3 weeks now is it safe to continue as preventive measure and about montelukast course again and again every year. thanks Doctor: HelloThis type of allergy is a precursor of asthmatic allergic bronchitis i.e. ASTHMA .But unfortunately peoples in India don't accept this , as I have noted.When such patient visit my clinic I prescribe them to take montelukast+ bambuterol ( combination 0.2 mg /kg body weight ) one tablet in the night and patient respond very well . I don't prescribe nebulizer or steroid inhaler .In my opinion this is safe drug for asthmatic allergic patient.When patient become symptom free, stop the drug and symptoms recurs , start the drug .Since asthma is an allergic disease and patient have to take treatment , whenever feel discomfort.Avoidance for dust , mite , mold , fungus , sudden increase or decrease in temperature , pollen , fumes , smoking help in asthmatic cases .Hope this will help you.Hot coffee and steam inhalation also act as bronchodilators."
},
{
"id": 163274,
"tgt": "Could headache and vomiting in a child be due to virus?",
"src": "Patient: Hi. My son is 6 years old and he has complaining of severe headache and then he vomits soon after. This started Saturday and I thought it was a stomach virus but he only vomited once. This evening, Monday, he began to become lethargic started complaining of a severe headache and then threw up. Could this still be a virus or should I see a physician Doctor: Hi,Your main concern is headache not vomiting. If I would be your doctor, I would suggest you vision test, a simple eye check up will clear many things.After that, go for BP check up and consult again with us.Hope I have answered your query. Let me know if I can assist you further.Regards,Dr. Hina Javed"
},
{
"id": 125141,
"tgt": "Suggest treatment for black bruised swollen shin",
"src": "Patient: Hello, my friend was wind surfing and fell off. The board slammed into his right front shin. It swelled up immediately to about four inches in diameter. He received several stitches, but his foot is black & blue and after sitting for a while, his leg feels numb when he gets up to walk. Is this normal? Doctor: Hi.. Well bruising is a common finding after a injury due to spillage of blood from blood vessels into the soft tissues. Lump can be due to Hematoma formation.Numb feeling occurs due to trauma to superficial nerve endings and inflammation.. It can take a few days for the condition to improve. For now he can elevate the leg while lying down. Do cool compresses over the injured area. Take anti inflammatory painkillers like Advil if there is pain. Do not exert mmich and can apply Thrombophob gel to reduce the bruising.. Hope I have answered your query. Please let me know if I can assist you further. Regards. Dr. Honey Arora, Dentist."
},
{
"id": 59161,
"tgt": "High LFT levels. Have Lichen planus. On steroid injections. Are these problems related?",
"src": "Patient: hello Dr., i am 35, facing Lichen Planus for past two years, LFT is bad, Leucocytes 11,640, SGOT 39.8, SGPT 63.7, Alkaline Phosphatase 120.6, AG ratio 0.6, S Bilirubin 1.1 mg%.Got ultrasound done, the report is good.i have been taking steroids injections to treat Lichen Planus, took six last march'12 and 4 till april'13.Does Lichen Planus occur due to Liver Infection?please guide. Doctor: Welcome to healthcare magic. Thank you for your query. Your liver function report is not so bad. SGPT is mildly elevated. SGOT, S.bilirubin & alkaline phosphatase are normal The major abnormality is altered AG ratio. But it should be correlated with clinical feature & other blood test. There is no correlation between lichen planus & liver infection. Take care."
},
{
"id": 151808,
"tgt": "how to know whether the mole on the back(spine) is cancerous or not ?",
"src": "Patient: hey, my husband has a mole on his back located exactly in the middle of the back(spine). its itchy sometimes and he has had it for 2yrs now. am worried beacuse its on the spine. doctors say he shouldn t tamper with it but what if its cancerous. everyone is fearing to touch it n examine its fluid since it is on the spine. please tell me what to do? Doctor: Welcome to Healthcare Magic Good Day It may have some connection to the inner layers of the spinal cord, This is why Doctors say not to tamper with it. You may get an MRI Spine to confirm. Show to the Dermatologist who will help find a solution for him."
},
{
"id": 152854,
"tgt": "What do you suggest if having pain in mouth after oral suregry for cancer?",
"src": "Patient: I have oral lichen planus and dry mouth pre surgery. In July of 2013 I had oral surgery for cancer on my tongue. By the 5th day of radiation I was hospitalized as I could not even drink water because of the pain. I had a feeding tube inserted. A year and a half later I am in constant pain...some days better than others. My doctor recommended GelClair. This is in appeal with my insurance company. Do you know of any way to get a sample to see if this even works? My doctor called the company...no luck. I have to wear an uncomfortable chin strap to sleep or I wake up in agony. Forget the whole eating problem. So far nothing has helped for more than half an hour. Do you have any other recommendations? Doctor: Hello,I can understand your concern. Pain is the most common and agnosing complication after cancer detection and treatment as it is also a side effect of all the treatment modalities including surgery, radiotherapy and chemotherapy. GelClair is a special formulation designed to mechanically adhere to the oral mucosa and thereby soothing the surgical and other lesions of the oral cavity. It has shown success in inflammation of the oral mucosa following cancer treatment and irritation caused by surgical lesion. I am sorry to inform you that there is no way we can provide you with samples of the molecule for trial. But I can sure advise you to go for it as it is worth giving a try in failure of other methods to control pain.If the pain gets intense, there are some advanced pain control options such as neurolytic therapy and opioid deliveries etc. that can be advised to you and referred to the specialist performing them by your oncosurgeon. I hope this information helps you. Thank you for choosing HCM. Let me know if I can of any further assistance to you.Best,Dr. Viraj Shah"
},
{
"id": 119297,
"tgt": "Is low blood a cause of dysmenorrhea and anteverted uterus contribute to pain?",
"src": "Patient: Does low blood a cause of dysmenorrhea? I am experiencing mild to severe dysmenorrhea every month. I feel weak, I vomit and I almost passed out. I had check up before with ob-gyne and requested to had ultra sound . My ulta sound impression is: Anteverted, unenlarge unterus with adenomyosis endometrium is compatible with menstrual phase. Bilateral polycystic ovaries. Unremarkable cervix . The doctor said, I had nothing to worry at all. Take Pain killer if I had cramps. Is the Anteverted uterus and adenomyosis contributary factor to the pain? How can I prevent this severe cramps? What vitamins do i have to take to improve my health preventing menstrual cramps? Doctor: Hi Dear your having dysmenorrhea which is mild to sever and your ultra sound report is normal size anteverted uterus,adenomyosis and endometrium comptible with your menstrual cycle,and bilateral polycystic overain disease.Dear anteverted uterus is normal position of uterus and is not causative factor for pain.the cause of your dysmenorrhea is poly cystic overies.consult a gynecologist to get treatment of your polycystic overies,and your dysmenorrhea will disappear"
},
{
"id": 211458,
"tgt": "Going to Psychologist for stopping Oaxil cold. Shaky and sweating in hands. Used all types of anti depressants. Is this Parkinson s disease?",
"src": "Patient: Hi, i, am looking for a neurologist for my cousin who lives in Pt. Richie, Fla. I, live in New Jersey. For the past 2-1/2 years she has been going to a psychologist because she stopped her Paxil cold turkey. She then started shaking and sweating in her hands. The psychologist before her put her on all tpes of anti-depressants. And, no help to this day. As, a matter of fact she is now shaking inside her body and she can t explain it. I, am trying to find help for her as she lives in New Jersey also, but, moved to stay with her sister in Florida. And, no help has been given to her. I, hope you can help her. She said she does not have Parkinson s. Please, can you help her. Doctor: Hello,Thanks for choosing health care magic for posting your query.I have gone through your question in detail and I can understand what you are going through.Paxil has a problem with discontinuation. On discontinuation it results in severe withdrawal symptoms. It may not be parkinsons disease. She may need some other antidepressants and there are many other benign options like escitalopram etc. Moreover never get a prescription from a psychologist as they are never trained in writing medicines. Always insist on prescription from a psychiatrist. Hope I am able to answer your concerns.If you have any further query, I would be glad to help you.In future if you wish to contact me directly, you can use the below mentioned link:bit.ly/dr-srikanth-reddy"
},
{
"id": 15061,
"tgt": "Red, itchy, inflamed skin rash on thighs and lower legs, tried prednisolone and steroid cream, prevention ?",
"src": "Patient: I have a red, very itchy, inflamed rash on my neck, thighs and lower legs. It is intensely itchy and has been this way for months. I had a similar outbreak this time last year. I was given prednisolone and steroid cream. I also tried a scabies treatment and some anti biotics. Nothing I tried relieved the issue. Eventually it went away however now it is back I am concerned it will not clear up again. I have introduced no new products or foods to my usual routine. Please help!! Doctor: Hi,Thanks for your querySeems like you have developed an eczematous skin condition.These types of skin allergic reactions tend up come up or exacerbate every year at about the same time.Air borne allergens play an important role in such cases.Treatment of these includes the use of both topical and systemic steroids.You seem to be similarly treated previously.I would advise you to see a dermatologist and get a skin biopsy and blood allergy tests done.So you should visit a nearby dermatologist for proper assessment and treatment.Hope it helps Best wishes for a speedy recoveryDr Geetika PaulMD (Dermatology)"
},
{
"id": 70242,
"tgt": "What is the treatment for lump on both the wrists?",
"src": "Patient: Hi, I have a lump on both my wrists and on my right hand every couple of months it gets guide painful. It feels like a bone and I can't use my hand properly when I get the pain. Do you know what it could be and what I can do to get ride of it instead of wearing a bandage every month? Doctor: Hi. These look to be ganglion. They are usually self limiting. Occasionally they cause more pain and restrict the movements of the wrist indicating too much pressure inside. Consult an Orthopedic Surgeon for confirmation of diagnosis and treatment. He may inject steroid or advise you an excision. There are high chances of recurrence."
},
{
"id": 115495,
"tgt": "What does low level magnesium indicates in my blood test?",
"src": "Patient: Hi, I recently had blood tests showing low levels of Vit B s, Magnesium, underactive thyroid, low DHEA etc. I am a type 1 diabetic but eat pretty good. I had a vit B12 shot at the Doctors office yesterday and felt pretty good. Around 4:00 today I started feeling a crash. Now my muscles are tight and kind of numb. I already took 350 mg of Magnesium. I m getting cramping in my feet. Could it be due to the B12 leaving my system? He just gave me a test dose and said it would be out of my body in a day or two. Am I taking too much Magnesium and not balancing it with other electrolytes? Thanks, Doctor: Dear friend, Greetings from HCM... i understand your concern ... YOU GOT IT RIGHT you need to take care of all the electrolytes ... Check your serum sodium/chloride, potassim / ionized calcium ....because, low potassium/ calcium could be the reason ... you dont take magnesium supplement alone .... you have to consume a product which gives you all the vitamins and minerals together in a balanced form (nutrilite dialy / revital etc.,) you have to take good amount of green leafy vegetables, fruits, nuts, (banana & coconut water rich in potassium)...daily ... to meet the needs of minerals for your system ... thank you .. do contact us for more queries ..."
},
{
"id": 205058,
"tgt": "What do mood swings between euphoria with high activity and depression indicate?",
"src": "Patient: I want to find out if my 26 year son could have some kind of mental illness or if its due to drinking but I am not sure where to start. I think he has a drinking problem. He has periods where he is very interactive with our family, very involved, happy, positive, focused, honest, upfront, shares, he has goals and dreams and he seems happy. But there are times when he is very withdrawn, quite, angry, lies, secretively, negative, no goals or dreams and seems very depressed. Doctor: he may be suffering from alcohol dependencf syndrome.contact a psychiatrist or me ...dr sarathsnair@gmail.com"
},
{
"id": 198538,
"tgt": "Suggest treatment for nocturnal emission and erectile dysfunction",
"src": "Patient: Sir I have one problem that my penis now doesn t have than much power and also the erection is small.Sir the matter is that I used to masturbate a lot for the last 5 years now my age is 19.sometimes I masturbate more than two times in a day .So sir now I am facing problem. Also sometimes in the night now in my sleep I sometimes wet my bed due to this. so sir please help me.Also my ejaculations are earlier. Doctor: HelloThanks for query .Problem of Erectile Dysfunction(ED) that you are facing is mostly due to frequent habitual masturbation since long time .The only long lasting solution to resolve your problem of ED is to avoid masturbation.This requires self determination .Please try to avoid masturbation by keeping yourself busy in activities like sports ,reading ,social work etc so that you will not get free time to masturbate.As regards your problem of night emission ,it is normal and natural to get night emission at the age of adolescence .It gets resolved without any treaatment over a period of time .Dr.Patil."
},
{
"id": 177901,
"tgt": "Is Motrin recommended in case of increased temperature in an infant?",
"src": "Patient: My 23 month old was fine all day. We were outside and it was warm so I thought she just got over heated but she feels really warm But rectal temp is 101.. She wanted to only go to sleep and lay down.. Gave her Motrin. Could it be teeth? She wS drooling more today? Doctor: Hello. I just read through your question. Using motrin to treat a fever of 101 is fine. This could be teething, as you suggest. However, if the fever persists beyond 2-3 days, I recommend consulting with your doctor to make sure nothing else is going on."
},
{
"id": 161868,
"tgt": "What causes toddler to refuse taking solid food?",
"src": "Patient: Hi, may I answer your health queries right now ? Please type your query here...i have a girl baby .she is one and half year old she doesnt eat properly ..and frm 6 days she is not taking solid food .she is only on milk .plz advice me what should i do Doctor: Hi, At her age interest in play increases and interest in food decreases. This is quite natural. If she is active and playing around you need not to worry. This age rather than the quantity of the food I suggest you concentrate more on the number of times you feed her and also make whatever you feed her calorie dense by adding a good amount of ghee to it. Hope I have answered your query. Let me know if I can assist you further. Regards, Dr. Sumanth Amperayani, Pediatrician, Pulmonology"
},
{
"id": 128866,
"tgt": "What causes lower abdominal pain on the surgical site?",
"src": "Patient: I m a 59 yr old female that had hip replacement surgery 12 weeks ago. Just recently started working out at a gym. Treadmill & some weights. Was doing some leg lifts and felt a sharp pain in my lower abdomen on the surgical side. Could I have done anything to the implant area or just pulled a muscle? Doctor: Hi, I had gone through your question and understand your concerns. With such case in my clinic,I will reassure my patient,This is muscular stretch pain after hip surgeries,I also prefer doing X RAY for your hip and lessening your activity for one month."
},
{
"id": 154545,
"tgt": "How long can a patient with myeloproliferative disorder/chronic myelogenous leukemia survive?",
"src": "Patient: My relative (29 years old) is diagnosed cml positive.Bone marrow biopsy examination revealed Myeloproliferative disorder, consistent with CML chronic phase.Chromosal analysis observation is : 46,XY,t(9;22)(q34;q11).Right now he is taking one dose of Imatinib 400 mg daily since last 40 odays & well stable.pls let me know the further conseqenced of this disease. Can he get married & have healthy offspring. How long will he survive. Doctor: Hi, dearI have gone through your question. I can understand your concern. Survival rate depends on phase of chronic myeloid leukemia. He has philadelphia chromosome positive so it is good prognostic indicator. His age is also favourable. So if he take proper treatment then 5 year survival rate is high. It has good prognosis then the acute leukemia. Hope I have answered your question, if you have doubt then I will be happy to answer. Thanks for using health care magic. Wish you a very good health."
},
{
"id": 53805,
"tgt": "Suggest treatment for hepatitis-B infection",
"src": "Patient: have a nice day doc,\\ i have a serious problem, i found out that during my delivery i am one of the hep b positive. it so sad and made me more hassle. im 26 years old having a 1 kid. almost 5 years i know but no medication at all. is this dangerous? what shall i do? what are the medication? Doctor: hi.thanks for posting query.1. HepB can not be eradicated completely in patients infected with it.however, complications can be prevented with proper care and treatment.Treatment is advised if AST and ALT are > ULN and HBV DNA count on PCR is very high. Liver enzymes and HBsAg status and HBV DNA PCR maybe done regularly every 6 to 12 months,Visit hepatologist for proper interpretation of results.2. hope your son got the vaccination as per schedule and was also given IVIg at birth against HBV.hope to answer concernregardsDr Tayyab Malik"
},
{
"id": 161982,
"tgt": "Suggest treatment for frequent passing of stools",
"src": "Patient: Hello Doctor My son is 2 years 7 months, He is having fever from yesterday. Today he passed motion almost 10 times. Now I am noticing blood in it. he is complaining stomach pain also. I am bit worried. Could you please help me to stop his stools. Every 5 min he is going to toilet now. Doctor: Hi, It seems your kid is having viral diarrhea. Once it starts it will take 5-7 days to completely get better. Unless the kid's having low urine output or very dull or excessively sleepy or blood in motion or green bilious vomiting, you need not to worry. There is no need to use antibiotics unless there is blood in the motion. Antibiotics might worsen if unnecessarily used causing antibiotic-associated diarrhea. I suggest you use zinc supplements (Z&D drops) & ORS as hydration is a very important and crucial part of treatment. If there is vomiting you can use Syrup Ondansetron (as prescribed by your pediatrician). Regarding diet - You can use cerelac, any flavour will do. Avoid fruit juices as they might aggravate diarrhea. You can give zinc supplements & ORS apart from normal vegetarian porridges & soups. Hope I have answered your query. Let me know if I can assist you further. Regards, Dr. Sumanth Amperayani, Pediatrician, Pulmonology"
},
{
"id": 122633,
"tgt": "Need treatment for swollen knee cap",
"src": "Patient: Hello, I have a swallow muscle above my knee cap ( I don\u2019t fell the pain) but it is swallow you can see the difference from the other knee. I have applied some regular cream on it. I believe is the Quadriceps Tendon or Vastus medialis muscles, any suggestion how to treated this? And I do run after works sometime not regularly but during the week I ran maybe two to three times. Doctor: Hello, Should apply ice or cold compresses on the swollen knee cap. Apply antiinflammatory cream locally. Massage with essential oil. Sometimes, wrapping with chopped onion with salt can reduce the swelling. If not, should see orthopedic specialist as might need to run MRI and other investigations to determine the right cause, and appropriate treatment. Wish fast recovery. Dr.Albana Sejdini"
},
{
"id": 79401,
"tgt": "What causes blue veins and pain on chest after falling down?",
"src": "Patient: I fell today in the bath tub and landed on my left side, middle of the ribs. I am in pain, hurts when I take deep breath but just think I bruised my ribs. But now 9 hours later I have bright blue veins running across my chest and breasts. never had that before, can even see them under slight tan. Lots of heart problems on both sides of family. should I be worried? Doctor: Thanks for your question on Health Care Magic. I can understand your situation and problem. You are having blunt chest trauma due to fall in Bath tub. By your history and description, possibility of musculoskeletal injury is more. And the things you are seeing, slight tanning, redness, veins etc are due to inflammatory edema because of musculoskeletal injury. So no need to worry much for heart damage. Better to start painkiller, anti inflammatory and muscle relaxant drugs. Apply warm water pad on affected areas. Avoid heavyweight lifting and strenuous exercise. Avoid movements causing pain. Don't worry, you will be alright in 1-2 weeks. If not improving after 1-2 weeks or worsening than get done chest x ray to rule out internal damage. Hope I have solved your query. Wish you good health. Thanks."
},
{
"id": 151507,
"tgt": "Fell and broke thumb, pain in scapula of back and shoulder. Done MRI. Buldging disc in the vertebrae with pinched nerve, edema, indigestion. Taken cortisone injection. Whats the relation between swelling and nerve?",
"src": "Patient: Two years ago I fell and broke my left thumb . After a short period of time I began to have pain in the scapula area in my back, then my left shoulder began having tremendous pain and limited motion in my left arm. I have had two MRI s and a torn rotator cuff was eliminated. It was determined that I have a bulging disc in the 5-6 vertebrae with a pinched nerve causing the limitation of motion and pain in my back and down my left arm to the left thumb. This may have been caused by the fall. Now I am having pitting edema in my left ankle. Also I have severe indigestion (which I feel is not connected) and I am not taking any medications. I did have cortisone injections in my neck which helped the pain for a short while, but is returning. Is there any connection with the swelling and the pinched nerve? Doctor: Hello elmogreeneyes. Welcome to healthcaremagic.com. A broken thumb does not cause a pinched nerve at the c5-6 level. That is coincidental. The fall could have caused the pinched nerve at the same time that your thumb was broken. The nerve intervations of c5-6 may explain the pain in the left arm that radiates to the thumb, however, those nerve paths do not have a connection to the difficulties of your left leg. Unilateral swelling (swelling on only one side) indicates a disease process that is not related to your heart. Some causes of unilateral swelling include fractures, ankle sprain, thrombophlebitis (blood clot), cellulitis (skin infection), venous stasis dermatitis (skin infection due to pooling of fluid in the lower extremity). If your leg is hot and red, go in for emergency evaluation of your swelling, if not, contact your family physician for evaluation of your condition. Be well, Dr. Kimberly"
},
{
"id": 117591,
"tgt": "What is the reason of two different blood groups in a person?",
"src": "Patient: Hi Any one htere,I have question on open heart surgery. patient Age is 46 Mitral valve scerosis weight 69 we have confusion on her blood group we checked her blood group in different places we are getting 2 results for that O-ve and O +ve.can you sugggest what might be the reason we would need to collect the donors based on this only for surgery Doctor: You might have du positive. It is a weak d antigen. So any person with negative blood group should be checked for this weak d. It can be confirmed by du test. And you should search for o negative donor. if youbhave o negative or weak d in both cases you need o negative blood. So search for o negative.thanks for using health care magic."
},
{
"id": 115531,
"tgt": "What causes breathing trouble and fatigue?",
"src": "Patient: I am a 39 year old female and was just released from the hospital with a bad kidney infection and chronic anemia i was given a blood transfusion as my hemoglobin was 7.4 it rose to 9.0 i have been taking the iron and calcium as directed by my doctor but my breathing and fatigue and multiple other symptoms are not better.When will they get better? Doctor: Hello and welcome to HCM,Anemia is known to cause difficulty in breathing thus anemia is the most probable cause of breathing complaints and tiredness.Your hemoglobin is improving, however, hemoglobin value of 9 is also low and it has to be increased upto 12 gm%.Iron supplements will be helpful if the anemia is due to iron deficiency.However, complete hemogram, red blood cell indices and peripheral blood examination is required if it is not done previously.Kidney infection will be treated by antibiotics.I hope you have got course of antibiotic for your kidney infection.Thanks and take careDr Shailja Puri"
},
{
"id": 116624,
"tgt": "Suggest treatments for high creatinine level",
"src": "Patient: My creatinine came back 1.1 and my eGFR at 55 - this has never happened before. B/p is maintained under good control on b/p med; no diabetes; 56 yrs old and weight 198#, ht 5'4\". Only think I can figure is that I was taking aleve in April/May for a knee injury. I have taken it occasionally since then. I am afraid of getting kidney disease, not being able to turn this around. Am big table salt user, will discontinue that. Drink lots of water. Just became vegetarian Nov. 1. What else can I do? Doctor: Hi,Thanks for asking.Based on your query, my opinion is as follows.1. Creatinine level is only mildly on the higher side. It keeps on varying. It varies with muscle bulk, and exercise. Keep following up monthly once.2. eGFR for this age with hypertension is good. 3. Avoid excessive salt. Continue with medication regularly. Mild exercises and protein diet is helpful. You can discuss with your doctor regarding use of ACE inhibitors to reduce kidney damage, if you are not using it.Hope it helps.Any further queries, happy to help again."
},
{
"id": 92203,
"tgt": "What could be the reason for heaviness in chest, throat, upper abdomen and pain in the back?",
"src": "Patient: Hey Dr, I have this feeling of heaviness in my chest, heaviness in my throat, pain in my upper abdomen, pain at the middle of my back and a hollow/ strange feeling of deepness or like a hole in btw my chest and my upper abdomen. I m so scared Dr, pls what is wrong with me Thank you Pam Doctor: Hi. There can be many reasons for this symptom-complex, the commonest can be due to heart problems like myocardiac infarct. Another possible reason may be related to oesophagus. I would first advice you to undergo EKG/ ECG to rule out heart problem. If this is normal would like to go for x-ray of the chest and upper GI endoscopy. This can be a serious problem unless proved otherwise, needs urgent medical attention."
},
{
"id": 78254,
"tgt": "What causes pain on the chest below the pectoral muscle?",
"src": "Patient: My husband started feeling pain in the right side of his chest right below is pectoral muscle. He doesn't recall doing anything that could have caused a cracked or broken rib. It hurts him to take deep breaths, put pressure on the area, and when he lays down it hurts to sit up or turn to the side. And it got worse after he sneezed. There is a faint bruising, but doesn't look swollen. What could it be? Doctor: Thanks for your question on Health Care Magic. I can understand your concern. By your history and description possibility of musculoskeletal pain is more in your husband's case. So ask him to avoid movements causing pain. Avoid bad postures in sleep. Avoid heavyweight lifting and strenuous exercise. Start painkiller and muscle relaxant drugs. Application of warm water pad on affected areas is beneficial in such cases. Don't worry, he will mostly improve in 5-6 days. If not improving then get done chest x ray to rule out internal lung pathology. Hope I have solved your query. I will be happy to help you further. Wishing good health to your husband. Thanks."
},
{
"id": 185675,
"tgt": "How to cure tightness and swelling in glossopalatine arch in mouth?",
"src": "Patient: I have a tightness and swelling in my Glossopalatine arch in my mouth with developing black bruise looking marks on my cheeks inside my mouth. Over the weekend I had a stress test with radioisotopes, and a CT Scan with dye. I had severe pain, difficulty breathing, and my D-DImer was elevated, but they never found a clot. Are they related? Doctor: helloi have gone through your query.A nuclear stress test measures blood flow to your heart muscle both at rest and during stress on the heart.this test is done to look for a coronary artery disease or another heart problem, test can be done to pinpoint the cause of symptoms such as chest pain or shortness of breath.D-dimer test can be performed when conditions such as acute deep vein thrombosis is suspectedD-dimer values i dont thnk your palatoglossal swelling is related to your heart condition or any of these recent tests.swelling in palatoglossal arch can be due to an infection mostly a viral or strep infection.for bacterial infection you need to take antibiotics.maintain proper oral hygieneas you might seem to have a heart condition i wud suggest you to visit a doctor before taking any medicationHope this ans helpsDr. Shesh"
},
{
"id": 80519,
"tgt": "Suggest treatment for acute bronchitis",
"src": "Patient: Hi there I was diagonised by acute bronchitis do I have been coughing for ovrt three weeks, when I cough it feel like my breath is taken away I gasp for air . Doctor prescribed celery isle dosage is 2 tabs for times a day and brazenpam 2 three times a day Doctor: Hello dear, thanks for your question on HCM. I can understand your situation and problem. In my opinion you should consult pulmonologist and get done1. Clinical examination of respiratory system. 2. Chest x ray. 3. PFT ( pulmonary function test ). Chest x ray is needed to rule out lung infection. PFT is needed to for the diagnosis of bronchitis.It will also tell you about severity of the disease. And treatment of bronchitis is based on severity only. You may need inhaled bronchodilators and antihistamine drug. So better to first diagnose yourself and then start appropriate treatment."
},
{
"id": 76240,
"tgt": "What could cause frequent dizziness and loss of breath?",
"src": "Patient: My daughter has complained over the last few weeks of being dizzy and she actually fainted today while sitting down she was skating and she had to have a friend help her off the ice she don t remember coming off the ice but she does remember sitting down she said she got dizzy then she don t remember anything she said everything went black and then she heard her friend calling her name and she said she felt very weird when she came to and it felt like her throat was pushing in that it hurt she also complains at night that she always has to take deep breaths when she first lays down its like she looses her breath I m starting to worry about this is this normal in children she is 11 years old and I don t know if I should take her to the hospital or wait to see her family doctor Doctor: The different causes that can present with fainting include1)Anemia(blood deficiency)2)seizures3)low blood pressureI would recommend to get it checked thoroughly"
},
{
"id": 120677,
"tgt": "What causes severe pain in the lower ankle?",
"src": "Patient: Hello! I have just come by this site out from an advertisement; I do not know if this is worth asking but there seems to be a pain in my lower ankle or my upper foot, somewhere near or on the talus bone. It hurts spontaneously from time to time, or when I run and am playing a sport, and I sprint to another side, (this is the right foot by the way) and use it to sprint, it hurts excruciatingly harsh. Or when I jump sometimes, that area hurts a LOT! What could this be? I am eager to hear your diagnosis. Doctor: Hello,I read carefully your query and understand your concern. Ankle pain\u00a0is most commonly caused by a sprain. A sprain is an injury to the ligaments connecting the\u00a0ankle\u00a0bones. I suggest using anti inflammatory medications such as Acetaminophen to relieve the pain .I also suggest using cold compresses for local application. Hope my answer was helpful.If you have further queries feel free to contact me again.Kind regards! Dr.Dorina Gurabardhi General &Family Physician"
},
{
"id": 64609,
"tgt": "How to get rid of hard lump in between her rib cage and thigh?",
"src": "Patient: my dog has a hard lump on her side between her rib cage and thigh, its growing bigger and its hard as rock, he one leg on the same side is all swollen and she cant walk on it. she has also lost lots of weight but still has her usual appetite. i cant figure out whats wrong with her Doctor: Hi,Dear,Thanks for the query to my HCM clinic.I studied your query indepth.In my opinion -its a question on the wrong platform.I would advise you to better consult a Verternary doctor locally and he would resolve your issue.Hope you have cleared your worry.Would wellcome you you to HCM for any more queries on it or on other health-problem."
},
{
"id": 4848,
"tgt": "On ciphene. Ovulation study shows ovulation is late. But no periods this time. Urine test negative. Suggestions?",
"src": "Patient: I tuk ciphene 50 tablet .then my doctor suggested the ovulation study. In that my ovulation period is late.means take 19 days.But ovulation hapend and period came on july 3.The same day i met my doctor and she suggested to take ovacare forte a+b for 4 month.no other tablet given.But In this month no period came upto this time .what is the reason.I done urine prgnancy test on august 20.It shows negative.can i wait some more days for my prgnancy or please suggest it require immediate apponitment to my doctor Doctor: Hi,It looks like your doctor has tested your system with ovulation induction to see if you respond or not and your system has responded. Ovacare forte is a nutritional supplement. It is likely that you may not have ovulated this cycle and hence have not yet seen a period. Yet, if you wish to exclude/confirm pregnancy you should get serum beta-hCG assay along with a trans-vaginal sonogram for definite confirmation. Please contact your doctor for further assistance. Hope you find this information helpful. Wish you good health."
},
{
"id": 64387,
"tgt": "Suggest treatment for lump in the heel",
"src": "Patient: I have a sore lump near my heel.. the best way to describe where it is...its on my right foot, if you touch the back of your hell and move your fingers to the left about 1 inch, it is right there. It's not on the bottom but where the side of my foot and the heel come together. It's hard and sore...what is it? Doctor: HI,Good Evening.Thanks for your query.I studied the query in-depth.I understand your concerns.In my opinion-you have-Achilles Tendinitis due to overstress and overuse.Consult orhto-surgeon--Treatment would be-a-Tb NSAID-/ ice compresses to reduce inflammation.b-REducing the high heel,c-Rest /elevation d-and Weight reduction.Wishing you early recovery and good health.Wellcome again."
},
{
"id": 121872,
"tgt": "How to treat dents on the rib cage?",
"src": "Patient: I have a dent on both sides of my rib cage. I asked my doctor about it, and he said I would grow out of it. I don t know if it affects my breathing, but I have really bad stamina. Over the summer I trained a lot and still had bad stamina. How do I get rid of the dents in my ribs? Doctor: Hello, The dents on your ribs will change due to continuous bone remodulation. Anyway, I recommend checking vitamin D and calcium plasma levels, for possible deficiency. Hope I have answered your query. Let me know if I can assist you further. Take care Regards, Dr Ilir Sharka, Cardiologist"
},
{
"id": 55033,
"tgt": "Does cholecystectomy lead to green stool?",
"src": "Patient: I had a cholecsytectomy about 7 months ago and for the past few days, i have been passing green stool. I havent eaten anything that woukd have lead to this. I also have some abdominal pain at the location of my now absent galbladder. Should i be concerned? Doctor: hi.noted history of cholecystectomy, occurrence of pain over the post-operative site and green stools. are you taking in any medication or supplement right now? it is best if you consult with a doctor, preferably a gastroenterologist or a general surgeon, for physical examination and clinical evaluation. recurrence of stones somewhere along the hepaticopancreaticobiliary tree must be ruled-out. diagnostics (such as ultrasound, bilirubins, liver function tests, etc.) and management (medical and/or surgical if indications are found) will be directed accordingly.hope this helps.good day!!~dr.kaye"
},
{
"id": 202421,
"tgt": "Red rash on scrotum, itching, burning, white bump with pus, frequent urination. STD symptoms?",
"src": "Patient: hi, i have a reddish rash showing up on my scrotum that is itchy at times, and is starting to burn a bit. the rash showed up over night. there is also a white bump on my scrotum, doesn t hurt but not sure what it is. i have had similar whiteish bumps that when i pop them a sort of pus comes out with a solid piece. only one white bump. I should say that i had unprotected vaginal sex with a new partner about two weeks ago... bad decision... but i can t find any std that would describe this. also, i have a slight burning in the tip of the penis , and somewhat frequent urination . (been going on about 5 days now) oddly, there is no burning when i urinate, only after. it is almost gone today though, doesn t burn much at all right now. it gets worse when i sit down, as i get up and walk around it almost goes away. this problem is my main concern. I am mostly worried about std s. i know the most common symptom of an std is no symptoms at all. that being said, i have had every symptom of every std i know of in the last week and a half. which brings me to my conclusion that i am a hypochondriac. but still would like a clean bill of health that says so on paper. is it too soon to get tested for std s? i know three weeks is the standard, but I m getting anxious here! i should also include that im 25 years old, heterosexual, and only ever had one unprotected partner. I hope i explained everything well enough.. Thanks Doctor: You should have your urine and penis tested for chlamydia which may not show up on tests. Or take a course of treatment to see if things get better.Please rate 5 stars. I strive to provide you with the best answer possible to your question!"
},
{
"id": 139846,
"tgt": "What causes tremors in hands,dizziness and hot flashes?",
"src": "Patient: I feel like I have a fever, especially in my face, but my temperature is only 97.8. My skin is a little more red than usual on my cheeks though. I also feel a little dizzy and my hands and arms are a bit shaky. What could be the reason and how would I handle it? Doctor: Hi, You may be suffering from thyroid gland dysfunction, so I suggest to do complete blood picture and thyroid gland tests. Hope I have answered your query. Let me know if I can assist you further. Regards, Dr. Salah Saad Shoman, Internal Medicine Specialist"
},
{
"id": 131865,
"tgt": "What is the remedy for the ankle pain caused due to the working condition?",
"src": "Patient: Hi I am stood most of their day in my job I ve suddenly started to get ankle pain in my left ankle , when I m home I rest it , I try to get up after the rest and I can t put pressure on my ankle I have not injured it at all I have scholl gel pads in my shoes to see if they will help but they haven t through the night my ankle aches , then I get up for work and once again can t put pressure on it , it eventually wears off so I can hobble about , but then it all starts again once I ve finished work can u help ? Doctor: Hi, welcome to HCM, you need to strengthen your calf muscles. Wear weight bands on calves and exercise. Every night--10mts ice fomentation, followed by 10mts heat, followed by pain balm. Strap your ankles during working hours. Try to move about intermittently. Regards"
},
{
"id": 89913,
"tgt": "What is the treatment for abdominal tuberculosis?",
"src": "Patient: Hi. i am diagnosed for abd. Tuberculosis and on ATT( in partial doses as it caused Hepatotoxicity earlier. Now my complaints are Loose motions 4 to 5 times a day and Pain in lower legs while sitting and getting up (weight lost is 6 kg). What is the reason for this?What is the treatment? Doctor: HI.You are a diagnosed case of abdominal tuberculosis and already under treatment. Hepatitis i caused by a few medicines and not all so you have to get the proper dosage as per your weight. This is very important for a cure , other wise you will not be cured and will continue with the problems. You have to take Liver protective supplements as per guidance of a Gastroenterologist, multivitamins and minerals and high proteins and you have to have normal hemoglobin."
},
{
"id": 211785,
"tgt": "Difficulty in blocking noises around while concentrating, exaggerated fears, fear of punishment and rejection. On medication. Help",
"src": "Patient: I cannot seem to block out the music in restaurants. Drives me crazy even if it is barely audible. I can't stand music even classical in Clinic waiting rooms. I cannot stand conversations happening around me while driving. I cannot seem to block out any unwanted noise while trying to concentrate. Am i autistic ? As a caretaker It drives me crazy if someone leaves fabric or any potentially flammable object on a light fixture even if it is safe life a fluorescent bulb. Exaggerrated fears? As I child I was severely strapped for things I don't even know why. I think I made noise and my dad couldn't concentrate on what he wanted to do. Hey I was a kid. Kids make noise. I am 58 and fear some kind of punishment or something bad is going to happen. I am a people pleaser not because I care but because I fear rejection or punishment. I feel I need some rational thoughts pumped into me all day. Left to myself I have no choice but try meds. Doctor: Hi, it appears that you have some problem with your mood. I advise you to consult a psychiatrist for diagnosis and treatment. I also advise you to check the hypertension and blood sugar which also may show such changes. Anemia, and hypoprotenimia also may cause these changes, so i advise you to take more of green leafy vegetables, pulses, sprouts, and proteine rich foods. Thank you."
},
{
"id": 49741,
"tgt": "Kidney donor. What precautions can cleanse and make kidney healthy for donation? Calcium build normal?",
"src": "Patient: I am getting ready to be a living kidney donor for a friend of mine. I am a vegetarian who eats very little salt and drinks about 1 to 2 gallons of water a day. Is there anything else I can do to cleanse my kidneys. I would like them to be as healthy as possible prior to the surgery. We have already been genetically matched, and I am getting the paperwork for the urinalysis . They did find two tiny spots on my kidney so they would like to do a 3 day urinalysis, sending one to Chicago. We do not think the sports are stones or cancer , just calcium build up. Anything you can tell me to get my kidneys in the best health would be greatly appreciated. Thank You, TEG Doctor: Hi, Appreciate your resolve to donate kidney to your friend. Normally body will take care of itself. There is nothing you can do to cleanse your kidneys. Maintain adequate hydration. Drink at least 3 ltr of water daily. Avoid smoking and alcohol before surgery. Avoid drugs which are toxic to kidneys most commonly pain killers. Don't take any medication without prescription. Spots in the kidneys are most Likely calcium Plaques which may a precussor of stone formation. Check if you have a family history of stone formation. Wishing you well"
},
{
"id": 59155,
"tgt": "Pain on side, back, abdomen, sick. Collapsed gallbladder, gallstones on scan. On analgesia. Consult doctor?",
"src": "Patient: Hi i have had what i think is biliary colic since 2am sunday morning, yesterday the pain was so bad i had to go back to bed, taking analgesia regularly but its not helping. Feeling sick as well on and off.I had a scan a few weeks ago which showed that my gallbladder had collapsed and i have multiple gallstones.Today when i got up i thought that i had pulled muscles in my side, back and abdomen due to the pain yesterday. But am wondering if its colic still as the pain is in the same places. Should I see my gp tomorrow?Thank you Doctor: Hi and welcome to HCM. This sounds like biliary colic, but also there can be asociated panceratitis or hepatic distension pain. i dont consider this is an emergency, but you should consider cholecystectomy as soon as possible.These stones can cause intensive pain, cholecystitis or pancreatits so be aware of it. If you notice dark urine,jaundice, pain in the back fever and vomiting you should go to ER. WIsh you good health."
},
{
"id": 143361,
"tgt": "What does my MRI report suggest?",
"src": "Patient: I am 31 yrs old and was sufeering from giddiness and voimitting feeling.The doc suggested to get a Brain MRI done and teh result is as follows - Loss of cervical lordosis with minimal early posterior disc osteophyte complex not causing any cord compression Can you please advice.. Doctor: Hello!Thank you for asking on HCM!Regarding your concern, I would explain that your cervical spine MRI report indicates just chronic degeneration of the cervical spine, without any clinical significance. There is no description of brain MRI. Coming to this point I would recommend consulting with an ENT specialist for a physical check up and labyrinthine tests to exclude an inner ear disorder. Taking Meclizine or Dimenhydrinate would help improve your situation. Hope you will find this answer helpful!Best wishes, Dr. Aida"
},
{
"id": 31282,
"tgt": "What are the symptoms of UTI ?",
"src": "Patient: I believe I have a UTI, and I have had them a few times. I took the OTC test and the nitrites were negative but the leukocytes were positive. My concern is I have light red (pink) blood on the toliet paper but there is none in the urine. I know its not my period because I got off about a week ago. Doctor: Hi, Thanks for posting in HCM. I understand your concern. Urinary tract infection (UTI) is an infection involving the kidneys, ureters, bladder, or urethra. These are the structures that urine passes through before being eliminated from the body.Symptoms of urinary tract infection include urgency or increased frequency of urination, pain or burning sensation while urinating and presence of blood/discharge per urethra. It may be accompanied by fever, abdominal pain, nausea and vomiting. Kindly get the following tests done. 1. Urine microscopy examination. 2. Urine culture and sensitivity. 3. Ultrasound abdomen for visualisation of urinary bladder.Kindly consult Urologist with the report for appropriate administration of medications and its management. Hope the information provided would be useful. All the best."
},
{
"id": 116710,
"tgt": "Suggest treatment for high SGPT",
"src": "Patient: hi, my question is that I have elevated SGPT (124), I am 37 years old i do weight strength and take body building supplements(whey, amino acids, CLA, creatine) i am not alcoholic or suffer from any medical condition, what is the cause of the liver enzymes elevation, and if it is from the supplemts which one is responsible for that? Doctor: Hi, dear. I have gone through your question. I can understand your concern. Your sgpt level is high. Iy is a liver enzyme. So it suggest you have some liver damage. It may be due to some viral infection or drugs or toxin. You should go for complete liver function test. Then you should take treatment accordingly. Hope I have answered your question, if you have doubt then I will be happy to answer. Thanks for using health care magic. Wish you a very good health."
},
{
"id": 219397,
"tgt": "What causes stoppage in periods?",
"src": "Patient: i took duphaston to start my periods which i missed this month i took 10 mg tablet twice daily for 6 days then stopped now its 3 days i stopped the medicine still my periods has not started what to do i have done my blood pregnancy test which was negative and also sonography which was normal. Doctor: After taking progesterone , you need to wait longer , maximum at least 2 weeks. if no periods after that, you need further evaluation by the gynecologist including the pregnancy test."
},
{
"id": 225806,
"tgt": "Spotting of pink discharge, diarrhea and severe cramps. History of merena 10 year IUD insertion five years ago. Help",
"src": "Patient: I had a merena 10 year IUD inserted after I had my son five years ago. Recently (two days ago) noticed spotting of pink discharge, diarreha and out of this world cramps! ! I walked in to the emergency room and waited for approximately 8hrs and was unable to wait any longer.......very scared and need answers now, looking on line scares you more, so i hope you will be of help! Thank you for your time Doctor: hi,thank you for your question,well pink discharge can be because of genital track (birth canal) ,it is commonly associated with IUD.consult gynecologist for conformation and indentification of organism causing infection.abdominal pain,pale stool, persistent loose motion are also one of the side effect of merena..nothing sever not worrytake care.."
},
{
"id": 38783,
"tgt": "Will it be safe to take Motrin(800mg) for bee sting on right bicep?",
"src": "Patient: I was stung by a bee (I don't know what kind it was) yesterday on my right bicep. Today I have aches in my right arm from the sting to my hand. I just took a motrin (800mg) which I hope will help. I would hate to waste my doctors time if this is going to go away soon. Any advice and Happy Holidays! Tom Doctor: Hello,Yes there is no harm in taking Motrin.This definitely would go away in next couple of days.Thanks"
},
{
"id": 47950,
"tgt": "What is causing painful movements in the kidney?",
"src": "Patient: My right kidney is malrotated and I was going to a kidney specialist, had a stent put in. It didn t help so a week later my stent was removed. I ve also had plenty of c scans and mri s done, been hospitalized due to blood in my urine and pain. Lately I ve been feeling my kidney move and its been very painful. Is there any other options for me? Thank you. Doctor: Hello and welcome to HCM.As an Urologist, i must assure you, that a malrotated kidney is only a variation, by birth, and not an abnormality or disease. It's well seen in a contrast CT-scan.But, unless, it gets obstructed, or some definite cause for treatment like a stone blocking it, arises, no intervention is needed.I fail to understand, why you had several CT scans, MRI's, DJ stenting, etc.You're welcome to send a copy of treatment with scan reports here, in my name, for an expert opinion."
},
{
"id": 26802,
"tgt": "What could cause shaky and dizzy?",
"src": "Patient: I am feeling shaky almost nervous. I get dizzy very easy. I am tired all of the time. I am thirsty all the time. My head hurts a lot. I had heart surgery in 2008 for aortic regurgitation. Two months ago I was hospitalized with chest pain and they kept giving me potassium. I need help Doctor: Hello!Thank you for asking on HCM!Regarding your concern, I would like to explain that many disorder could cause this clinical scenario.I would recommend consulting with your GP for a careful physical examination, a resting ECG, a chest x ray and some blood lab tests : - complete blood count for chronic anemia- kidney and liver function tests - blood electrolytes ( including calcium and magnesium)- thyroid hormone levels and cortisol plasma levels ( to exclude possible thyroid and adrenal gland dysfunction)-fasting glucose and glucose tolerance test ( diabetes could cause chronic fatigue and thirst)- a cardiac ultrasound to examine your cardiac function. -inflammation tests (fibrinogen, PCR, sedimentation), because an autoimmune disorder could mimic such a clinical scenario. -blood and urine osmolarity ( this test helps define the cause of your thirst)I also advise monitoring your blood pressure in both sitting and standing up position. A head up tilt test is necessary to exclude possible orthostatic hypotension. Hope to have been of help!Best wishes, Dr. Iliri"
},
{
"id": 99258,
"tgt": "What causes popping sensation in head and itchy hands?",
"src": "Patient: My hands and a few other spootty areas itch badly, I have had a ear popping sensation in my head for about 2 mos or so. Have been treated with antibiotics, but doesn t go away. I can keep it at bay with Benadryl tablets and cortizone creme for hands. Generally on my hands. I have recently found black mold in my bedroom, could this be why? Doctor: HI, thanks for using healthcare magicIt is possible that the black mold is contributing to the allergies that you are having.The ear popping can be due to abnormal function of the eustachian tube which is a tube that connects the middle ear to the back of the throat.This can occur in allergiesYou can consider allergy testing so that you can accurately determine what is provoking the reaction.I hope this helps"
},
{
"id": 183562,
"tgt": "What is the treatment to fix the gap between the teeth?",
"src": "Patient: I was flossing my teeth and suddenly a little hard thing came out from between my 2 front bottom teeth. Now I feel like there is a gap between my 2 front teeth. Is there a way to fix the gap? Is the hard thing that came out from flossing plague that had accumulated and hardened or a part of my tooth hence causing the small gap when it came off? Doctor: Hi,Thanks for posting the query, According to your clinical symptoms i suppose that you might have removed a junk of calculus in between the teeth while flossing, dont worry this is not a part of tooth and needs to be removed, there is no need to fill the gap just maintain a good oral hygiene, brush and floss your teeth regularly....Hope you find this as helpful,Take care!"
},
{
"id": 192320,
"tgt": "Suggest treatment for cut on penis every time after having intercourse",
"src": "Patient: my problem is i always get a cut on my front part of penis whenever i have intercourse with my wife,it gets bleeding and pains a lot.i have consultant a skin specialist and he told me that i need to do circumcision.but i dont want to do circumcision.so do u have any other solution for this problem.already i have tried somany ointments. Doctor: Hello, The cuts happening to your penis head has to be because of tight or dry skin. You can use some lubricating gel or liquids to maintain the moisture on your penis when having sex. Therefore I suggest consulting a skin specialist for physical examination, diagnosis and treatment. Hope I have answered your query. Let me know if I can assist you further. Take care Regards, Dr. K. V. Anand"
},
{
"id": 56687,
"tgt": "Why is my infant s bilirubin level increasing?",
"src": "Patient: My daughter had a baby 5 days ago and he had jaundice. He was placed under blue bili light. Within 24 hours his bilirubin level went from 19 to 12. They were sent home but had to go back for bilirubin level today, and it was 14. Why did the level go back up? He is eating well and passing meconium stools. Doctor: Thank you for sending us your query. The baby is has physiological jaundice, which is a self remitting condition. it usually persists for first 3 weeks of life. However if the symptoms persists then congenital hyperbilirubinaemic conditions, such as Crigler-Najjar syndrome or Dubin Johnson syndrome or Gilbirt's syndrome or Rotor's syndrome may be considered. In such scenario, you should consult a paediatrician to pin point the exact cause. In most of the cases no active treatment is required and patient leads a normal life. Take care."
},
{
"id": 176256,
"tgt": "What causes severe food allergies?",
"src": "Patient: Hello my child has several food allergies (soy, peanuts/tree nuts, strawberries, wheat, eggs. When we last took her to her allergist who ordered labs all of her allergies were outrageously high though we make a conscious effort to keep her away from her allergens etc. What could possibly cause her blood test results to all be abnormally high? Doctor: Hello. I just read through your question.The tests only indicate that the allergy exists. It is not an indication of exposure. Patients with allegy who have no recent history of exposure will still test high."
},
{
"id": 205165,
"tgt": "How can schizoaffective disorder be treated?",
"src": "Patient: Can schizoaffective be managed in a calm home setting. Do most due to severe symptoms continuously keep them in the hospitals, may be state. I have awomen I care about very much who has this condition. turned to alcohol severy over the past 10, instead of taking invega or other anti phycotics. she shuts down and cant communicate unless she is drinking. she also claims she feel water, spit on her face, and washes her hads continuously while she has bad anxiety. Doctor: take fluoxetine 20mgs with lunch and olanxapine 5mgs at night....after 6bdays increase dose of fluoxetine to 40mgs"
},
{
"id": 127071,
"tgt": "What is the treatment for pain in the ankles?",
"src": "Patient: Hello Doc: I am a 70 year old man in good health (no smoking and drinking). But I have consumed too many sweets/soda over the years. Over a year ago I began to experience on and off pain in my ankle. What is the treatment for this pain? I just began taking Glucosamine hcl and it appears to be working. Anything better out there? Doctor: Hi, In my opinion taking sodas and sweets over a period has no direct effect on ankle. It is not likely to cause direct pain unless you have developed diabetes or gouty arthritis. If Glucosamine is helpful then continue. I suggest you undergo a check up for diabetes and gout also, along with X-rays of the ankles. Hope I have answered your query. Let me know if I can assist you further. Regards,\u00a0\u00a0\u00a0\u00a0\u00a0 Dr. Gopal Goel"
},
{
"id": 166416,
"tgt": "What causes passage of white strings in urine of a child?",
"src": "Patient: my 3 yr old has recently battled high tempreture and feeling unwell. His urine sample was checked and came back ok but i noticed today, (2weeks later) he has white stringy bits in his urine. His tempreture is normal now though he still doesn t seem his normal self, is a bit grumpy and cries more than usual. anyone any thoughts? Doctor: Hi... by what you say it could a worm infestation or a urinary tract infection. I suggest you get a urine routine done and get back to use with the results or consult your pediatrician.Regards - Dr. Sumanth"
},
{
"id": 26157,
"tgt": "What are the symptoms of heart attack, conjustive cardiac failure and myocardial infarction?",
"src": "Patient: i never understand difference between heart attack, conjustive cardiac failure and myocardial infarction.i always become confused while understanding all these terminologies. i am a medical student but never understnd it .please answer my question.i shall be very greatful Doctor: hello. I have gone through your query.Thanks for using HCM. basically heart attack and Myocardial infarction are synonymus .It is a condition where there is total blockage of one of the three coroniaries supplying blood to heart.There is total cut off of blood supply to particular segment of myocardium which results into necrosis that is death of tissue. Congestive cardiac failure is term to denote failing function of ventricles. mainly when right ventricle fails and it produces odema,ascites,neck vein engorgement and hepatomagaly.This is known as CCF. My best wishes Dr.Rajesh teli,MD."
},
{
"id": 213004,
"tgt": "What is the remedy for tingling sensation in the head and chest during nervousness ?",
"src": "Patient: i have a weird tingling sensation in my head and chest but it only happens when i get nervous im 25 4 11 and about 200 pounds other than being over weight im pretty healthy. thank you Doctor: Hello and welcome to Healthcare Magic. Thanks for your query. Your symptoms could be anxiety-related. Anxiety can manifest itself with not only psychological symptoms, but with physical symptoms also. If you are having recurrent such symptoms or having difficulty managing your stress, then I would advise you consult a psychiatrist for a detailed psychological assessment and further treatment. There are effective treatment options - in the form of medication or counselling / psychotherapy which will help you overcome your problems. Wish you all the best. - Dr. Jonas Sundarakumar Consultant Psychiatrist"
},
{
"id": 205507,
"tgt": "Suggest treatment for stress , seizure and anxiety disorder",
"src": "Patient: Hi. My husband has been under a lot of stress lately. Due to my health issues. I have recently in the last month been having seizures, these are new onset seizures. I have never had one before. He has never had blood pressure problems until now, and has never had a major anxiety disorder until now. Recent;y, his blood pressure hasn't been below 152/98 and the highest its been is 178/118. He has been started on a BP medication Atenelol, but has been experiencing numbness in his right hand on his ring finger and pinky finger. It is constant, and its starting to get to where its worrying me! Should we ask his Dr to change his med? Doctor: DearWe understand your concernsI went through your details. I can understand. High blood pressure could be due to the anxiety disorder associated with your health issues, as well as white collar syndrome. Thinking about having his blood pressure checked might trigger high blood pressure. His anxiety also needed to be managed. You should talk to your doctor regarding this. After thorough check up, on finding nothing is wrong organically, you should request your husband to consult a psychologist for anxiety related behavior and cognitive therapy. He should be alright within months.If you require more of my help in this aspect, please use this URL. http://goo.gl/aYW2pR. Make sure that you include every minute details possible. Hope this answers your query. Available for further clarifications.Good luck. Take care."
},
{
"id": 51308,
"tgt": "Continuous discomfort in back, history of MRI with renal prominence. What should be done?",
"src": "Patient: Hi there, I m currently suffering discomfort and have been for the last 3 weeks in the left side of my back. I suspect it might be my kidney . I am passing urine very regularly although not painful to pass. I had a MRI scan back in 2010 which reported mild left renal prominence. I have tried not to dwell on this but this problem is reoccurring and again causing me discomfort. What can I do? Doctor: Possibly this relates to having some disc problem ., but having a unilateral pain suggests U atleast get a sonogram of Ur kidneys to feel sure U have no trouble."
},
{
"id": 152859,
"tgt": "What causes profuse anal bleeding post oral cancer surgery?",
"src": "Patient: my father had an oral cancer surgery on 29th jan 2014.he is ok with that.he was prescribed with cremaffin syrup.suddenly for the first time he bleeded profusely instead of stool.he came home last wednesday and we gave one dosage of cremaffin.the very nxt day he started bleeding again.colonoscopy is done and they could nt find the source of bleeding yet but bleeding has stopped probably they gave some medicines.can u suggest anything in this situation? Doctor: Hi, dearI have gone through your question. I can understand your concern.He has cancer surgery. Cancer patient has high tendency of bleeding. Cancer itself can leads to low platelet count. More over drugs used in cancer treatment also causing tendency to bleed. So it can be due to that or some other cause. He should check her complete blood count and coagulation profile. Then take treatment accordingly.Hope I have answered your question. If you have any doubts then feel free to ask me. I will be happy to answer.Thanks for using health care magic. Wish you a very good health."
},
{
"id": 180816,
"tgt": "Can Doxycycline be taken for gum infection?",
"src": "Patient: I have a gum infection, was given doxycline 100 mg, his was given to me today in Toronto by my sisters dentist as I was flying to New York to a meeting. Do I take 1 or 2 today and then 1 for 5 days. I come home on Tuesday when I plan to see my dentist who has been out of town. Doctor: Hi. Thanks for the query If you have gum infection then it is probably due to infectious deposits over the gums in the form of plaque and tartar. It creates a continuous source of infection. Other causes of gum infection if it is localized can be inflammation of gum around wisdom tooth or Pericoronitis. Doxycycline 100 mg is appropriate in case of gum infection and yes, you can take twice on the first day and then once daily. But for complete resolution you should get the cause of the infection treated and if it is due to plaque and tartar then cleaning of teeth and area in the gum pockets should be done. Hope this helps Regards."
},
{
"id": 223414,
"tgt": "How to ensure that no fetus develops after sex?",
"src": "Patient: i last had sex about 2 weeks ago and i am due for my next period in 10 days, so have not yet taken a pregnancy test, but i feel like strange and baby-like maybe going on in my stomach, so i was wondering if there are any sort of natural ways which i could ensure that no foetus continues to develop? Doctor: Hello,I have gone through the query and understood your concern. If you have regular 28-30 day cycles, you had the intercourse during your safe or non-ovulation period. Hence, your chances of conception are nil. There is no need to do anything to avoid conception. You can safely wait for your next natural period. Please do not stress since stress and anxiety can alter the menstrual cycles by disrupting the hormone milieu. Hope you find this information useful. Take care."
},
{
"id": 157118,
"tgt": "Should I be concerned for the increased density in the right breast diagnosed through Mammogram?",
"src": "Patient: I had a mammogram 2 days ago, and they called and want me to come back for additional views, because I have an area of increased density in my right breast not noted on previous mammograms, I am 53 , and have had mammograms every year since I was 40. What would this mean? And should I be concerned? Doctor: Hi and welcome to HCM, thanks for the query.Well, you don't have to be concerned but you need to do ultrasound and biopsy to rule out malignancy. It can be any benign breats lesion but also it can be tumor in some cases. so do further tests so that you can sleep well.Wish you good health.Regards"
},
{
"id": 142377,
"tgt": "What causes severe fatigue, brain fog and memory loss?",
"src": "Patient: I have Extreme fatigue and worse fatigue upon exertion. I have inflammation and all my joints with pain, nausea, diarrhea regularly Brain fog memory problems muscle spasms sometimes I lose use of my hands, I feel dizzy most times other times seems like my eyes spin sometimes I will seems I walk like I m drunk. Anytime I have to do anything I have to sleep to make up for it, when walking I can get pains and my feet we re ankles that can stop in my tracks Doctor: Hello!Welcome on Healthcaremagic!Your symptoms could be related to a metabolic disorder or a connective tissue disorder (considering the joint and muscle pain). For this reason, I would recommend consulting with your GP for a careful physical check up and some tests to investigate for the possible causes underlying your symptoms: - a complete blood count, PCR, sedimentation rate for inflammation- blood electrolytes- vitamin D and vitamin B12 plasma levels for possible deficiency- thyroid hormone levels for possible thyroid gland dysfunction- Rheumatoid factor levels- Anti-nuclear antibody levels- a nerve conduction study to exclude possible neuropathy. You should discuss with your doctor on the above tests. Hope you will find this answer helpful!Best wishes, Dr. Aida"
},
{
"id": 134301,
"tgt": "How much time does the fractured tibia take to recover?",
"src": "Patient: Hi, I had flu about a week ago and since then have had a cough. It is starting to go and is not on my chest, mostly seems like I still have a lot of catarrh in my system. I am pretty much back to normal apart from cough. Would it be ok for me to go swimming as I feel like some exercise might help get things moving. Doctor: hi,the question and the history are different. still I will go along with your history. You mentioned you had flu and having cough. You should do deep breathing exercises and perform costal breathing exercises to make lungs stronger. I know it's boring thing to do but it does miracles as well. If you want to go for swimming you can go if you have a facility of warm pool. Avoid cold water for time being in the pool as it may give a chances to go closer to cold again. after a week or so you may go for swimming but for time being if you avoid it will be good.with the grace of God I wish you a speedy recovery"
},
{
"id": 102501,
"tgt": "Can advair ( administered for adult on set asthma) cause weakness in the body in sunlight?",
"src": "Patient: I have just been diagnosed with Adult On-set Asthma and have been on Advair for 2 months now. The minute I step out into the sunlight I feel weak as water. I will be seeing a Pulmonary Specialist in 10 days...But I am wondering if this is a reaction from the Advair? Doctor: Hello,Welcome to HCM,Asthma is curable with proper homeopathic treatment. These inhaler can give you temporary relief but can not cure it. Long term use of inhaler also can cause many side effects.Homeopathic treatment is based on total understanding of patient on physical and mental level. And that's why detail history of patient is needed for proper homeopathic treatment. Your problem can be cured permanently with homeopathy.So consult to good classical homeopath nearby you or contact us.Have a better health."
},
{
"id": 108127,
"tgt": "Suggest remedy for numbness on my back and neck",
"src": "Patient: hello iam 32 years old i have been in an acident where i have had back and neck injurys where from my waste down it losses feeling and sometimes cant get hard unless i get oral sex then it only stays up for a short while i need help what should i do? Doctor: DEAR FRIEND, I UNDERSTAND YOUR CONCERNS, SO YOU NEED TO TAKE CARE OF YOUR BACK... SEE IN THIS TYPE OF CASE PAIN IS THE MAIN FACTOR WHICH TROUBLES HIM DUE TO IRRITATION OF NERVE ROOTS. NO NEED TO WORRY, YOU NEED TO TAKE IFT AND TRACTION THERAPY FOR YOUR BACK IN PHYSIOTHERAPY CLINIC... IF ANY LOCALIZE PAIN IS PRESENT THEN TAKE ULTRASONIC THERAPY.... YOU NEED 15 days TO THIS THERAPY!! THEN ONLY START WITH VARIOUS EXERCISE... TILL THAT TIME WAIT FOR PAIN TO REDUCE...."
},
{
"id": 189526,
"tgt": "Bump on wisdom tooth, itchy, clear fluid filled, ruptured. What is it?",
"src": "Patient: I have a small bump where my wisdom teeth used to be. It can become itchy and annoying, at which time I will typically rupture it (think small blister) and clear fluid and sometimes blood are released. This usually solves the itchy/pain issue and the bump with resolve for a week or two, then recur, starting the cycle over again. Is this something to be concerned about Doctor: Wisdom tooth is the last tooth to erupt in the oral cavity and often its eruption is associated with symptoms like pain and discomfort in eating , difficulty in opening mouth , swelling in the jaws and cheek bite. wisdom teeth are often tilted in position and impacted in the bone , gums overlying the tooth usually undergoes inflammation known as pericoronitis . The tilted position of the tooth leads to entrapment of the cheek mucosa in between the teeth during biting. Due to cheek bite , the mucosa gets inflammed and appear red or white due to trauma. i would suggest you to go for symptomatic treatment at home like gargling with lukewarm saline water or antiseptic mouthwash like chlorhexidine or betadine. you may need an OPG radiograph done for you and accordingly the tooth can be extracted surgically. i would suggest you to start with antibiotic Augmentin 625mg along with analgesics like aceclofenac two times daily for 3 to 5 days . i hope this helps . take care."
},
{
"id": 195123,
"tgt": "What causes bumps under the scrotum?",
"src": "Patient: i was wondering what would cause mild bumps cyst like under the skin of a mans scrotum? looks like a pimple but not draining. sometimes after a long period of time some may be squeezed like a pimple and release like one with a white thick like fluid but not always. non painful. and can shaving in these areas cause them? Doctor: Hi, You can attach a picture in the followup query so that we can make a better clinical assessment. Most probably it can be a folliculitis. Hope I have answered your query. Let me know if I can assist you further."
},
{
"id": 132279,
"tgt": "Why my joints are swelling with pain?",
"src": "Patient: I am 5 months post hysterectomy, this last two months I have noticed that my joints are hurting and swelling some.. test done for RA and thyroid -- all within normal limits.. Vitamin D low and has been for two years -- highest now is 22 GYN doesnt think i need hormone replacement yet??? Doctor: HIWell come to HCMI really appreciate your concern, if this is the joint pain and it is not due to arthritis condition and not even due to hormone changes condition then it may be due to musculoskeletal condition and it can be managed with tab Diclofenac 50 mg once in day, pain is nothing to with hysterectomy hope this information helps."
},
{
"id": 225697,
"tgt": "History of fingering with possible sperms in fingers. Suggest safe contraceptive pill",
"src": "Patient: can fingering soon after ejaculation lead to pregnancy? This is a situation where I am still in dilemma as to whether my hands had sperms at the time of fingering which followed soon after ejaculation. It's been a moment back and so kindly suggest in the given situation which contraceptive pill would be safe to use and would have least side effects with gaining of weight or others and the duration and timeframe within which it should be taken? please suggest Doctor: Hi, of course the possibilities of pregnancy are very less, but not completely abscent. In this case consider it as a routine ejaculation. and consult a doctor for advise. There are products like unwanted 72 etc to prevent pregnancy. Thank you."
},
{
"id": 150924,
"tgt": "MRI brain shows multiple tuberculomas in bilateral syhian fissures & conernous sinus region, advised biopsy. Is it necessary ?",
"src": "Patient: my wife, 52 yrs., 12 months back, diagnosed with Hyponatremia & Cryptococcal Meningities and treated accordingly, following with under treatment for Meningities Tuberculasis and on AKT for 11 months. Recently on MRI brain shows multiple tuberculomas in bilateral syhian fissures & conernous sinus region....Dr. has advised for Biopsy of the lesion &C/S. Is it necessary for Biopsy and has any risk or any alternate test can be done? AAAAA, AAAA(AAAA) Doctor: Hi, Thank you for posting your query. The initial treatment for tuberculous meningitis was started on empirical grounds (without confirming the diagnosis). If the current MRI had not shown any new findings, then, the doctor would have continued the TB treatment for at least another 7 months. Since, there are tuberculomas, it is possible that we are dealing with multi-drug resistant TB or some other infection/disease, so, I also agree that biopsy of the brain lesion is the best option at this point. Best wishes, Dr Sudhir Kumar MD DM (Neurology) Senior Consultant Neurologist Apollo Hospitals, Hyderabad"
},
{
"id": 9028,
"tgt": "Can i use skinlite cream to become fair ?",
"src": "Patient: Hi sir, i am divya.I am dark in color.Can i use skinlite cream,liva to become fair Doctor: Hello, Thanks for your query. You consult a cosmetologist and you can go for bleaching, skin peels and few other techniques which would be done for few sittings. They would be helpful for you. Do not simply rely on the new products. Hope I have answered your query. I will be available for your follow up queries if any. Wishing you good health. Regards,"
},
{
"id": 41130,
"tgt": "What causes clots with mucus during IVF cycle?",
"src": "Patient: HI, Ive had my 3rd ivf cycle and have been using crinone gel, I have just found out Im pregnent but today I found alot of clumps of crumbly gel on my underwear. i have also been constipated and when I went to the toilet lots of the dried clumps came out.. PLs help, is this normal?? Doctor: Hello, it is common for the gel to come out with the mucus but that is nothing to worryIn case you have any questions in future you can contact me directly on http://bit.ly/drmanishajain"
},
{
"id": 159549,
"tgt": "Osteosarcoma relapsed to lung, damaged right lung. History of limb salvage surgery and chemo for osteosarcoma in tibia. Prognosis?",
"src": "Patient: Sir i was diagnosed with osteosarcoma in my right tibia in august 2009. i took 6 chemo and underwent limb salvage surgery. Now osteosarcoma has relapsed to my right lung and my doctors say that treatment wont help much. i have 3 nodules with involvement of lymph nodes and destruction of right lung. how long can i survive without treatment? Doctor: Hallo Thanks for your query.Sorry to know about your illness.You have a Osteosarcoma of Tibia and you have undergone salvage surgery and now you have a secondaries in the lung Unfortunately it is a terminal stage of cancer and there is little hope of getting cured by any therapy.It is also very difficult to say about survival period once it has spread to lungs or other organs."
},
{
"id": 15921,
"tgt": "Red blotchy rash on neck. Rash on tummy. Soar throat. Applied calomine lotion. Cause?",
"src": "Patient: Hi. My child has developed a bit of a red blotchy rash on part of his neck. He had a rash yesterday before going to bed which was on his tummy and back, but this morning it had cleared up again, so I didn't bother to make him an appointment with the Drs. Now its come back, but not as severely as it was yesterday. Does it sound like it could be something he's eaten that's causing the blotchy rash.Oh and he's not poorly with it - no temperature and says he feels fine, although he did have a sore throat a couple of days ago. He said it was a little itchy last night so I put calomine lotion on him. Doctor: Hello, Thanks for the query. From your history it looks like an exanthematous rash, most probably due to the (sore throat ) infection which he had 2 days back. Other things which we should consider is drug rash and urticaria or hives. Drug rash is very itchy and you do not give any history of taking medicines prior. Urticaria or hives again is very itchy and it is raised skin coloured to red lesions(wheals) which is not classically blotchy. Exanthematous rash will subside on its own. You may continue calamine lotion. hope this helped, take care."
},
{
"id": 511,
"tgt": "Does Duphaston have any side effects?",
"src": "Patient: I want a baby.As per the prescription of doctor i am regularty taking folvite for last 19 days. AS WELL AS DOCTOR ALSO PRESCRIBED TO TAKE DUPHASTAN AFTER 16 DAYS OF PERIOD. my last period date was 27th march. But, againg i have period on 10th april. IS THERE ANY SIDEFFECT ? NEED YOUR SUGGESTION. Doctor: Hi, Thanks for the query. I understand your concern. Folic acid when taken before conception & continued in first 3 months it helps healthy fetal development &also prevents certain congenital anomolies in fetus. Dupjhason helps development of good inner lining of uterus ( needed for implantation of embryo in uterus).. it is also essential . as it supports maintenence kof pregnancy. *So both the drugs given to you are helpful for cinception. * The bleeding on 10th of April is not likely to be side effect of any of the pills you are taking - But it is possible following a break in taking tab. Duphaston.... OR.. -It can be ovulation bleeding too. Thanks. Thanks.."
},
{
"id": 35563,
"tgt": "Suggest treatment for Hepatitis E",
"src": "Patient: Hello Doctor, My cousin is diagnosed wih Hepatitis E. He has had typhoid twice and jondis twice in last 1 year. This time docs here are saying that he has hepatitis E. I want to know what speciality doctor is right to tackle this situation. We are tired of running from pillar to post in last 1 year. Doctor: Thanks for asking in healthcaremagic forumIn short: Visit Gastro-enterologistExplanation: you can visit gastoenterologist for the problem. Usually Hep E is just like Hep A which subsided by itself with symptomatic treatment without long term complications. Just have high carbohydrate diet without oil/spices. All the best."
},
{
"id": 92494,
"tgt": "Have type 1 neurofidbramatosis. Severe abdominal pain and vomitings after eating food",
"src": "Patient: I am a 24yr old female who has neurofidbramatosis type 1 (nf1). I have been having horrible pain in upper left abdomen that goes into my back between my shoulder blades. Also anytime I eat or drink anything I am either sick to my stomach in horrible pain or both. I even throw up a lot after eating. I can not live like this anymore the pain nausea and vomiting is ruining my life. Its been going on for two weeks now the pain has been getting worse. Doctor: Hello!Thank you for the query.Such upper abdominal pain which radiates to the back is usually caused by gallstones. Sometimes pancreas inflammation can give such symptoms however the pain is usually so strong that forces patients to visit a doctor.If it is gallstone issue, your symptoms should get worse after fatty foods spicy foods or alcohol. Sometimes similar symptoms are present with stomach issues like peptic ulcer.I suggest you to do not wait any longer and visit your doctor. Abdominal ultrasound, blood work, liver tests should be done at first. If gallstones will be confirmed, laparoscopic surgery should be done. If nothing will be found, gastroscopy is necessary.In a meanwhile please avoid fatty foods, spicy foods, coffee and alcohol.Hope this will help.Regards."
},
{
"id": 80977,
"tgt": "Is it concerning if the chest x-ray says perihilar/basilar segmental atelectasis and parenchymal scarring ?",
"src": "Patient: My husband has been having high blood pressure issues in the last few months. He has been seeing an internal medicine doctor for this, but we are not sure about him. My husband has had an EKG which was normal, a stress test that was borderline (showed some changes, but recovered quickly per his doctor), and lab work. He is taking Hyzaar/HCT, Cardizem, Baby ASA, and Amlodipine. Most recent BP was 156/90. He recently had blood work (last week) and his cholesterol was 300, his Triglycerides 1500. The doctor said he does not need meds for these levels. Here are my biggest concerns: My husband is becoming increasingly fatigued. He feels as though he cannot breath when laying down and sleeps sitting upright. He loses his breath easily and cannot do simple tasks such as climb the stairs without taking a break most days. We have gone to his doctor three times to express our concerns, however the only thing he has done is a chest xray. The doc told me it was normal, but upon reviewing is medical records, it actually showed Mild perihilar/bibasilar subsegmental atelectasis and parenchymal scarring change with associated volume loss. I looked online and it defined this as small areas of the lung collapsing. Should I be concerned and finding a different doctor? My husbands symptoms are not improving, just slowly getting worse. His doctor said it is only related to the high BP and will get better when they can get that regulated. Doctor: Thanks for your question on HCM.I can understand your situation and problem.Following are the possibilities for your husband's CT thorax abnormalities.1. Chronic congestion due to high blood pressure and cardiac diseases.2. Lung infection3. Pulmonary fibrosis.So better to consult pulmonologist and get done1. Clinical examination of respiratory system.2. PFT (pulmonary function test)3. Recent 2d echo4. Bronchoscopy as and when required.If chronic congestion is the cause then he may need more diuretics to remove congestion from the lung.Antibiotics are needed for lung infection.So better to first diagnose him and then start appropriate treatment."
},
{
"id": 71514,
"tgt": "Will Ibuprofen and xopenex subside fever, burning sensation in eyes, chest congestion and body aches?",
"src": "Patient: Low grade fever, terrible burning eyes, terrible chest congestion, body aches, some chills. Ears hurt some. Shaky. Worst part is burning eyes. Taking ibuprofen and drinking lots of fluids. Xopenex breathing treatments. Had flu shot in October. Lots of flu in our area. Help Doctor: Hello,Ibuprofen drug will take care of fever and body ache. Xopenex with levosalbutamol drug contents will relieve chest congestion.Burning eyes will be best supported by cold water splashing 5 minutes each every 4 to 5 hourly.Hope I have answered your query. Let me know if I can assist you further.Regards,Dr. Bhagyesh V. Patel"
},
{
"id": 216085,
"tgt": "What is the treatment for back pain and leg pain?",
"src": "Patient: Hi, I was having back pain along with acute pain at legs and somewhat at hands. I have undergone MRI and below is finding from MRI reports IMPRESSION: Early subtotal disc dessication and degeneration of L5-S1 vertebral level with mild disc bulge seen associated with thecal indentation Doctor has given me medicine for 15 days and asked for rest (didn t ask for bed rest just said don t bend forward and don t carry weight, that s it) and and said I will get relief in 15 days, but it has been 4 days while taking medicine but I didn t get any pain relief. Could you please advice me that how much days generally it takes for pain relief. Also, one of my friend who had slip disk in the past said that doctors generally gives medicine for 15 days to stop the pain and after that start traction etc. Please advice. I am worried because age is just 24 years Doctor: Hello and Welcome to \u2018Ask A Doctor\u2019 service. I have reviewed your query and here is my advice. I\u2019m sorry this has happened to you and at such a young age. back pain can be very debilitating. like your doctor said, you start off with less invasive treatment and give time. In addition to your medication, you can also try physical therapy and physiotherapy. After this, you can try some epidural steroid injections which calm the nerves down in your back and can give you pain relief and the ability to move around. Hope I have answered your query. Let me know if I can assist you further."
},
{
"id": 168288,
"tgt": "What causes constant tiredness and lethargy?",
"src": "Patient: My 15 year old boy who is very active and plays hockey about 5 x s a week is always very tired. Wants to sleep constantly but forces himself to get to hockey or training. Eats very well but has great difficulty concentrating. Drinks plenty of water. Has recently gone through a major growth spurt. He is 15, 5 10 and weighs 155 pounds. Has black circle under his eyes and when not doing something physical he sleeps, hard staying awake at school. Doctor: Hello! Thank you for writing us here. He might be having stress from school or maybe from his sports. He's growing and energetic at this age of life. But you have to make him relax and tell the importance of proper sleep. I won't suggest any medicine for him but just talking to him would help. He wants to do a lot and that's really good. Also its common at this age group that boys are highly energetic. As time passes he would be alright but still I would suggest you to council him about his health.Best regards,Dr Gunjan"
},
{
"id": 140556,
"tgt": "What are the side effects of Thalido 100 mg,Tab Lowplat 75 mg and Risex?",
"src": "Patient: Hi, my mom has multiple myleoma, she is taking Cap Thalido 100 mg- Once daily at night Tab Lowplat 75 mg - Once daily Cap Risex (morning and night) She is 70 years old. our concern is side effect of these drugs like memory loss, speech sluggishness and oral comprehension Doctor: Hello, All medications that are prescribed by doctors have side effects and the ones you mention your mother is taking are no different. If the symptoms are INDEED those of the drugs and not due to other condition such as Vitamin B12 or D deficiencies, or INTERACTIONS of any of these medications with other drugs she may be taking for other conditions collateral effects of the drugs that could cause DEHYDRATION (lack of fluid volume) or HYPOGLYCEMIA (low blood sugars) then, you could meet with the physician and bring these items up so they can explain if any options to those drugs exist or whether these medications are distinctly necessary for her to have any chance at slowing down the progress of the disease. The family will have to weigh the EXPECTED BENEFITS of the drugs and SIDE EFFECTS you are witnessing to decide if it's an acceptable tradeoff. These are not easy decisions and should be made together with your mother's doctor and your mother if possible. Hope I have answered your query. Let me know if I can assist you further. Take care Regards, Dr Dariush Saghafi, Neurologist"
},
{
"id": 186752,
"tgt": "What is the cause of pain in the right lower cheek?",
"src": "Patient: My 3 year old little girl has been up since the middle of the night with complaints of a hurting right lower cheek. She has no temperature or any other symptoms, but I'm concerned she eats too many sweets and consequently has tooth ache? She seems in alot of pain Doctor: thanks for your query, the pain could be because of the tooth infection or any injury to cheek caused by a sharp tooth. consult your oral physician and take radiograph of the affected tooth.. i hope my answer will help you. take care."
},
{
"id": 76202,
"tgt": "What causes pain on left side of chest and gastric problems?",
"src": "Patient: I am 36years old male.I am having pain in my chest-just to the left of my chest inside 2nd rib bone from top.sometime the pain is around 3-4 rib bone.It from the front and sometime from the back.I have severe gastric.some time i feel ball of air just above stomach to the left. might there be connection between the pain and gastric.toady i am having continious pain at the said point.Thank you. Doctor: Hi thanks for contacting HCM...You have not mentioned which gastric problem you have ....You could have reffered chest pain ....Treat you gastritis or ulcer like condition by strong antacid drug like rabeprazole or pentoprazole ....for two week.....Restrict gastric irritants like .../excess spicy /excess tea and coffee / smoking and alcohol / non veg etc....If your ribs tender to touch it could be costochondirtis for which brufen like antiinflammatory drug given. ..If your pain constricting then investigate with echo and blood pressure checking...Consult physician for this.Take care"
},
{
"id": 94938,
"tgt": "Had panic attacks. Why is my stomach burning? Emergency?",
"src": "Patient: Hello my name is julita. I never had this feeling before . I m a smoker and I ate Chinese food but after that I felt like I had a little panic attack which I always got. My question is I have a burning feeling in my stomach ever since I ate the Chinese food . Am I okay or should I go to the er because I m a little worried about this feeling. Doctor: Hi, May be the chinese food was too spicy or some thing that has caused gastritis . burning sensation in stomach may be the features of gastritis.SMOKING ITSELF CAN GIVE RISE TO GASTRITIS. You may take medicine like mucaine gel 2 tsf 3/4 times to get relief from the burning . If the symptom persists you may take peptard d 1 daily for few days . getting a check up from physician will be helpful Hope you be all right ."
},
{
"id": 177115,
"tgt": "What causes diarrhea after having Clamp syrup?",
"src": "Patient: hello doctor, my 4 months old daughter had fewer so my paed diagnosed ear infection and prescribed clamp syrup, now my daughter is passing loose motion ever since taking this medicine, now my doc has suggested enterogermina , is it right to giv e this medicine Doctor: enterogermina is a probiotic which improves digestion and reduces the quantity of stool volume. Clamp syrup(Amoxy-clav) has a well known side effect of causing loose stools. but this is transient and will resolve once the drug is discontinued.Try giving some other drug for ear infection like cephalexin or cefuroxime."
},
{
"id": 193310,
"tgt": "What causes discharge of semen during urination and bowels?",
"src": "Patient: Dear Dr. From past few days i feel mild pain on my stomach (i believe its due to because of my tight pants) and 4days before when i was pooing solid semen came out from my penis.. when i tried to discharge the stool liquid semen also coming... same thing happened yesterday also? can u please advise me wht i have to do? Doctor: Hello,There could be simple logical explanation for this problem is that you might have retrograde ejaculation means semen enters your bladder instead of coming out of urethra or there is some semen left in urethra after the last ejaculation. However its not possible to have semen in bowels, that whitish discharge could be due to mucus in stool which could be due number of reasons like infections, fissures, bowel obstruction or Crohn's disease. I suggest you to visit your doctor for detailed checkup regarding your problems and treat you accordingly.Hope I have answered your query. Let me know if I can assist you further. Regards, Dr. Sameen Bin Naeem, General & Family Physician"
},
{
"id": 65759,
"tgt": "What can be the reason for lump in the crevice of the large labia?",
"src": "Patient: I am a 53 yr old female, haven't been sexually active for over 5 years and I have a swollen \"bulb\" like area in the crevice of the large labia. The whole left side of my groin is tender and I have an area on my left thigh that is sensitive to the touch. what could be the cause of this...Oh, I am the mother of 5 children Doctor: Hi,From history it seems that you might be having infected Bartholin gland cyst giving this problem.due to this infection tthere might be having enlarged, tender inguinal lymph nodes giving troublr in the groin.Consult Gynaec and get examined.Keep local hygiene proper.Ok and take care."
},
{
"id": 39822,
"tgt": "What causes fever after dog bite?",
"src": "Patient: My three year old was bit by a friend dog. It wasn't nothing serious just enough to break the skin a little. Cleaned it right after it happened and this morning, now she is running a 101.0 fever and there is an infection in her hand which I've been cleaning it also. The dog has had all of his shots, so why is she running a fever? Doctor: Hello,Welcome to HCM,Rabies is 100 % fatal but it is 100% preventable by proper and adequate treatment. As your child was bit by vaccinated pet dog but it is necessary to give atleast 3 doses of antirabies vaccine to your child.As you cleaned the wound thoroughly with soap and water it will wash away the virus if it is present in the biting site.The symptoms of fever is may be due to presence of some other infection in the body. I would suggest you to consult your paediatrician at the earliest.Thank you."
},
{
"id": 136270,
"tgt": "What is the treatment for sinus infection?",
"src": "Patient: My 23 year old daughter has been through 2 sinus surgeries and before we consider a clinical trial to suppress IL-5 and IL-13, I would like to know more about this elevated bilirubin test amend what questions to ask her immunologist . Thank you in advance! Doctor: Elevated billirubin because of hepatopathy.Reason may be drug or infective or toxicity. What kind of trial you are undervgoing kindly inform so that detailed information can be given."
},
{
"id": 209941,
"tgt": "Suggest treatment for aggressive behaviour",
"src": "Patient: helllo doctor my nae is khoy from cambodia I have a sister who has mental illness ( lost controll when she gets angry) in this cawhese how can I solve or treat her cuz now reject medicin and not respect to other when advise her to take medicin my email: YYYY@YYYY Doctor: HiThanks for using healthcare magicI think, she needs antipsychotic medication. In that case, she should consult a psychiatrist for proper medication. With antipsychotic, she would become normal. If she does not welling to take oral medication, then you can give her long acting injection after psychiatrist consultation. Thanks"
},
{
"id": 100190,
"tgt": "Are canker sores or cold sores contagious?",
"src": "Patient: Hello, I have two questions I m sure you can answer, 1. Are canker sores or cold sores the contagious ones, and which can contain the herpes virus? 2. Why does the inside of my lips turn a pale color or sometimes almost white when I eat like salt n vinegar chips or something like that? I hope you can answer these, I would love to know, thank you! Doctor: HI, thanks for using healthcare magicCold sores are due to the herpes virus. There are different types of herpes viruses but cold sores are usually associated with the herpes 1 virus though some persons may have sores due to herpes 2.Herpes 1 infections (cold sores) are very common.The change in color of your lips with certain foods may be due to reaction to the active ingredient present.I hope this helps"
},
{
"id": 148338,
"tgt": "Does one require a neurological approach while suffering from shingles and fibromyalgia leading to dizziness and weakness?",
"src": "Patient: I am inquiring about the concern of a dear friend who has been mostly confined to her bed for the past 6 weeks due to the following symptoms: extreme weakness, dizziness, hands shaking, aching all over, headache, limited intake of food, became dehydrated and had to go to the hospital for iv fluids. She's had a cat scan and blood work which did not show anything, and the doctor she's seen in Scottsdale, AZ does not know what's wrong. Early in Sept. she had Shingles which affected her back, arms, neck and torso. Do you think she should be examined by a neurologist. She also has had Fibromyalgia for years. Doctor: Hi,Thank you for posting your query.I appreciate your concern for your friend.Yes, it would be very useful for her to be examined by a neurologist. Her present symptoms may or may not be related to the past episode of shingles.After examination, she may require MRI of brain and spine, and blood work such as thyroid profile, serum vitamin B12 level, etc to identify the cause.I hope my answer helps. Please get back if you have any follow up queries or if you require any additional information.Wishing you good health,Dr Sudhir Kumar MD (Internal Medicine), DM (Neurology)Senior Consultant NeurologistApollo Hospitals, Hyderabad, IndiaClick on this link to ask me a DIRECT QUERY: http://bit.ly/Dr-Sudhir-kumarMy BLOG: http://bestneurodoctor.blogspot.in"
},
{
"id": 156492,
"tgt": "Are crazy mood swings and screaming a side effect of medicines?",
"src": "Patient: my father was diagnosed with stage 4 colon cancer in April 2012. He was completely healthy and had no idea it even was there. He had his first surgery in May to remove the large tumor, 14 lymph nodes and his appendix. In July he went back for the spots to be taken off his liver so they took the entire right side of his liver and large pieces of the left side along with his gallbladder. after the 2nd surgery, his blood pressure medicine was not working right so they switched him to lopressor. since coming home he has no desire to do anything and has crazy mood swings where he just screams. we are not sure if this is all coming from the medication or something else. My mom has even questioned if the cancer spread to his brain because he is not the same person he was before going in for the 2nd surgery. After the first one he was back riding his bicycle 10 miles on some days. He was not given a life expectancy but his doctors told us that he will be with us 10 years from now although they would have to go back in and clean up his liver again within that time. We are trying to figuring out what is causing all these changes in him so any help would be great! thanks Doctor: Mood swings and personality changes need to be investigated in a cancer patient who already has stage 4 disease with liver metastasis. His brain should be imaged with an MRI scan. If no evidence of metastasis in brain is found then he should be seen by a psychiatrist for evaluation as the stress of chronic illness might be causing it. Please discuss it with your treating oncologist."
},
{
"id": 198009,
"tgt": "Is weakness common in case of excessive masturbation habits?",
"src": "Patient: I m 18 years old... and i m doing hand practicing since 4 years 3times in a week....... my weight is 46kg... i feel weakness after hand practicing and my hand are also very thin.... here many doctors ans that hand practicing does not harm health but i feel very weakness after hand practicing.......! how can i left this habit?? what should i eat after hand practicing to overcome weakness..??????????? Doctor: DearWe understand your concernsI went through your details. Whatever you read here is correct. Handpractice cannot cause any health problems if practiced with moderation. In your case you are masturbating thrice daily which is in excess and you should reduce the frequency. Masturbation is not unhealthy. But when you masturbate energy is spend. If you masturbate thrice daily thrice more energy is spend. Naturally you will feel tired and your health will deteriorate. But it is more trouble if you be anxious about masturbation. I suggest you to reduce your masturbation frequency to thrice or four times a week. You will see the result within a month. You should also engage in physicial activities and games.If you still need my assistance in this regard, please use this link. http://goo.gl/aYW2pR. Please remember to describe the whole problem with full detail.Hope this answers your query. Please feel free to post follow up queries. Available for further clarifications.Good luck. Take care."
},
{
"id": 182122,
"tgt": "Suggest treatment for growing bottom wisdom tooth",
"src": "Patient: I am looking for a dentists opinion. I am 36 and my bottom wisdom tooth is growing in. I have no pain in my tooth but I keep biting down on my gum where it is growing. Do I have to get my wisdom extracted or can the gum be cut to help my wisdom grow in better. Doctor: Best possible way to decide on removing your wisdom tooth, Would be to do an x-ray and look at position. If there is any bone obstruction which we normally term it as an impaction, then the dentist decides on extraction or removal. If there is soft tissue covering and persistent irritation and inflammation, then the dentist would approach with simple procedure of removal of the soft tissue covering only called operculectomy."
},
{
"id": 168007,
"tgt": "What causes periventricular leukomalacia in a child?",
"src": "Patient: 19 months baby, with low birth and hasn t cried after the birth, in the last four months of time he had 3 episodes of febrile convulsions and today he had an convulsion with no fever, lst time when diagonised in CT sacn it shown as PERIVENTRICULAR LEUKOMALACIA? would like to know what might be the problem for my child Doctor: Hi... by what you say I feel that your child is having periventricular leukomalacia which means the lining around the fluid content of the brain is having a pathology, which is leading to this particular problem of seizures or fits without fever. He requires further evaluation and an MRI brain along with anti seizure medication.Based on his motor development and neurological evaluation he might even require physiotherapy.Regards - Dr. Sumanth"
},
{
"id": 15582,
"tgt": "Itching along the tapes holding dressing of complete hysterectomy, prescribed Clindamycin, Hydromorphone, worsening. Solution ?",
"src": "Patient: Hi, I underwent a complete hysterectomy last Wednesday and upon my release, I was given a prescription for Clindamycin and Hydromorphone to be taken 4 times a day. I am done the Clindamycin but still have Hydromorphone. After a few days of taking these medications, I started itching along the tapes holding my dressing. I had the staples removed yesterday and last night and today the itching is much worse and have a rash down my back, back of my legs and neck and on my abdomen. What should I do? Doctor: Hello,Thanks for the query.You might have developed an allergic reaction to plaster.Dont worry it is a common thing,Take an antihistamine and apply desonide lotion.This will take care of your condition.Please meet a dermatologist if it persists.Let me know if you have any other doubt.you can ask a direct question to me on this forum, following the below link.https://urldefense.com/v3/__http://www.healthcaremagic.com/doctors/dr-rahul-kumar/64818Wishing__;!!Mih3wA!SBzm6_kI6hCZ58EPH6N_05MFfiPbxWXT0a2TJCdFQObRWm5mV5ur7hUOMa8clQ$ you a good health.Thank you"
},
{
"id": 190192,
"tgt": "Dental work done under anaesthesia, suffering from nosebleeds. Treatment options?",
"src": "Patient: Hi there. My 5yr old son had dental work under general anaesthetic & was discharged approx 6 hrs ago. His nose has been bleeding minimally on & off ever since. My ex-wife has him so I can only go one what she tells me. Ive asked her to contact the hospital to check but I m looking for answers while I wait to hear from her. Thanks. Doctor: hello...you have not mentioned what procedure had been carried out...nasal bleed is normal owing to a traumatized naso tracheal intubation ( tube placed through nostril that goes inside the trachea for airway maintenance during anesthesia) ...that persists for some time. if profuse bleeding persists you can consult your ENT surgeon for further management."
},
{
"id": 224224,
"tgt": "Changed birth control tablets, had sex, having sore nipples. Pregnancy or side effects?",
"src": "Patient: Hi I had a month on Lo Loestrin Fe birthcontrol and then switch after a month to Tri-Sprintec. I did not skip any pills just went straight to takin Tri-Sprintec after finishing my pack of Lo Loestrin. My boyfriend ejaculated inside me during my first week of Tri-Sprintec, Im scare since i switch and was on tri-sprintec for only a week that I could get pregnant.However my boyfriend insisted that ill be fine since I was already on Lo Loestrin for a month. Now my breast are sore after a week of being on sprintec and my nipples have grown however the soreness is not as bad as a previous time when i was pregnant. Im scare that i might be pregnant since my breast are sore, or if its just a common side effect of tri-sprintec but i dont understand why my nipples have grown a bit pls help? Doctor: Hallow Dear,Both the pills you have mentioned are birth control pills. Lo Loesterin Fe is a low oestrogen pill hence it may cause low bleeding. Tri-Sprintec is a average Oestrogen-Progesterone pill. If you were on one brand pill regularly and just change the brand without having any gap, the chances of protection are very high. Moreover, the intercourse happened in the first week of menses. This is little away from the fertile period.Egg release in women happens 14 days prior to next menses. Considering 24 hours life of egg and 72 hours activity of sperms, a week around this day is considered as fertile period. Already on birth control pills and intercourse away from the fertile period, the chances of pregnancy are very low. You have not mentioned whether you have missed period or not. Missing a period is the first and cardinal symptom of pregnancy. All other symptoms appear later.Your breast and nipples symptoms could be due to rough handling also. Still if you are in doubt, please wait for the periods to miss. If at all it misses, then 8-10 days after that perform urine pregnancy test and/or ultrasonography for detection of pregnancy; done earlier, they may prove to be inconclusive. Beta hCG tests are conclusive even 2-3 days after missed period.I hope you are satisfied with the explanation."
},
{
"id": 37111,
"tgt": "Suggest treatment for naval infection",
"src": "Patient: Had an infection in my naval and the doctor gave me anti-biotics 500mg 4 times a day, that night after taking 4 i was up all night throwing up. I stopped taking anti-biotics and the nausea is gone but i feel very fatigued all day and have no appetite. Doctor: Hello,Welcome to HCM,If it is a simple infection like a furuncle, with no surrounding swelling or fever, a local antibiotc cream like Bactroban (mupirocin) is sufficient after cleaning the area well with an antiseptic lotion like betadine.The symptoms of poor appetite and fatigue would be due to the side effects of the antibiotics. Since you have stopped the antibiotics, it will disappear in a day or two.If you have further query, I am available.Wish you fast recovery and good health."
},
{
"id": 193815,
"tgt": "How to treat Benign Prostatic hyperplasia?",
"src": "Patient: I have been diagnosed with BPH. My prostate is 2X the normal size. I am taking Avodart and Flowmax. I had a hip replacement 3 years ago and am otherwise in good health. Is Cooled Thermotherapy an appropriate treatment? Will it affect sexual activity? Doctor: Hi, For Benign prostate enlargement, you have been prescribed correct medication that is dutasteride and tamsulosin to relive your urinary symptoms and to decrease prostate size. Your concern is regarding the usage of cooled thermotherapy in which microwave used to destroy prostate tissue. So it can be used as a treatment modality and it is one of the safe procedure. You can discuss with your urologist regarding that. Hope I have answered your query. Let me know if I can assist you further. Take care Regards, Dr Parth Goswami, General & Family Physician"
},
{
"id": 42485,
"tgt": "Suggest remedy to treat infertility an regularize periods",
"src": "Patient: hi this is shrutika my periods are so irregular i m 26 year old married descperately waiting for baby i had pcos done in mjan 2011 then dodc said egg formation is not happening properly also last month she gave me fertyl 50mg but no result yet. i want baby pls reply Doctor: Hi,Welcome to HealthcareMagic .Along with fertyl I suggest you to go for IUI. It is intrauterine ingestion of sperms . In this the sperms are washed and processed and healthy sperms are placed in uterus . This will increase chances of pregnancy .once you start ovulating normally your cycles will regularise. Be stress free. No treatment for infertility give 100% results. They increase chances of conception. Hope I have been helpful .RegardsDr.Deepika Patil"
},
{
"id": 72331,
"tgt": "Suggest remedy for recurring breathing problem",
"src": "Patient: i am sufferung from breating problem in the night times, when i consult nearest doctor he told that it is like dust elergy. He gave me some medecine. when i use those medicines i feel better but when i stop it again the same thing. can you please suggest what can do and whom to consult. Doctor: Thanks for your question on Healthcare Magic.I can understand your concern. You are having dust allergy and breathing difficulty. So possibility of asthma is more likely. So better to consult pulmonologist and get done clinical examination of respiratory system and PFT (Pulmonary Function Test).PFT will not only diagnose asthma but it will also tell you about severity of the disease and treatment is based on severity only. You will mostly improve with inhaled bronchodilators (formoterol or salmeterol) and inhaled corticosteroid (ICS) (budesonide or fluticasone).Oral combination of antihistamine (levocetrizine or fexofenadine) and anti allergic (montelukast) once a day at night is also beneficial.Don't worry, you will be alright with all these. Hope I have solved your query. I will be happy to help you further. Wish you good health. Thanks."
},
{
"id": 221149,
"tgt": "Suggest treatment for panic attacks",
"src": "Patient: Im 37 weeks pregnant, and I have had really bad panick attacks at night since about 20 weeks. They used to only be at night, and now they are during the day as well. How can I cope with this? I feel as if im going to have a break down and cry, I dont sleep at night, and have a two year old to get up with first thing in the morning. Im exhausted, on the verge of going crazy and Want this pregnancy to be over?! Can I be induced or can I do anything to get this baby out now!!!? I had this with my first pregnancy and it was alot easier to deal with then because I didnt have a two year old to deal with the next day. HELP! Doctor: Hello, and I hope I can help you today.Panic disorder is a well documented psychiatric condition the can become worse in pregnancy because of the hormonal changes.If you are having problems coping now, pregnant and caring for your other child, I do not think that having a newborn at home will help your sleeping situation or the stress of your life.I strongly suggest that you discuss your situation with your OB prenatal care provider. There are medications you can take when you have a panic attack that can be used safely during pregnancy.Certainly, induction before 39 weeks of pregnancy is not recommended unless there are serious medical complications for either the baby or the mother. Furthermore, being induced early increases your risk of cesarean section, and recovering at home after cesarean section and caring for two children is certainly more difficult than after a vaginal delivery.So in summary, I strongly suggest you discuss your problems with your prenatal care provider. He or she may be able to give you medication that you can take when you have a panic attack, are also something to help you sleep, that are safe to use during the pregnancy and will help you get some more sleep and therefore feel better during the day.I hope that I was able to adequately answer your question today, and that my advice was helpful.Best wishes for the rest of the pregnancy,Dr. Brown"
},
{
"id": 99999,
"tgt": "Can chest pain be caused by asthma?",
"src": "Patient: I grew up in Arizona and in 2001 I started coughing and was short of breath. Found out I have asthma, am taking advair 100/50, do pretty well except in cold weather, I live in Tennessee now and the winters really hurt me. Ok, started having chest pains upper back pain, somtimes really bad, somtimes its liveable. Could this be from my Asthma or somthing else in my lungs? thanks beverly Doctor: HI, thanks for using healthcare magicAsthma would not normally be associated with pain in the back. The pain may be related to another lung issue or something else entirely.Pain can be related to any structure in the area. This includes the skin (eg shingles), bones, muscles and other soft tissue and then underlying organs.It is possible that the pain is related to one of these structures.It would be best to visit your doctor for an assessment if it continues.I hope this helps"
},
{
"id": 13042,
"tgt": "Suggest treatment for rash and blisters on skin",
"src": "Patient: Yes. A week ago I began to have ache in my right kidney area, sometimes with pain moving about. Then it resided but was replaced by a rash that started on the same side and in two days has spread around my middle area. Today there are small blisters in the new area and larger one in the area that started yesterday. I have made an appt. with my dr. but he cannot see me until 8/20. No fever, urine or constipation.. I can email a pic. Thank you but I am on medicare. Doctor: Hello, Your symptoms seem to be related to shingles. A photo would be very helpful through the diagnosis. I suggest using Acyclovir cream for local application. I also suggest using Tylenol for the pain. Hope my answer was helpful. If you have further queries feel free to contact me again. Regards, Dr. Dorina Gurabardhi General & Family Physician"
},
{
"id": 52476,
"tgt": "Is focal heterogeneous enlargement on the left lobe of the liver a symptom of a serious ailment?",
"src": "Patient: I have just done the ultrasound scan and the results say I have a focal heterogeneous enlargement on the left lobe of the liver. Gallbladder and spleen appear normal. Is this bad? I seldom drink. I don t smoke. I am not even overweight. Please advise. Thank you! Doctor: Hello and Welcome to \u2018Ask A Doctor\u2019 service. I have reviewed your query and here is my advice. It may be due to many causes like hepatitis etc. CT scan may be advised. Treatment depend upon findings. Hope I have answered your query."
},
{
"id": 155673,
"tgt": "How safe is taking Parit-20,roseday-10 and beplex forte tablets?",
"src": "Patient: hello doctor, i am of 30 years of age and for last 2 years, i get a choking of my throat, i have gone to almost 13 doctors they prescribe me the same medicine such as parit-20, roseday-10, beplex forte tablets, i do get relief, but for sometime only, now i am beginning to think that i may have some sort of cancer and i am fed up of my life due to this diease, so if you can help me, i will be very greatfull to you. Doctor: Thanks for your question on HCM.You are taking parit which is rabeprazole(antacid), roseday which is rosuvastatin (lipid lowering drug) and beplex forte which is multivitamin. So if you are not having high lipids than no need to take roseday at your age (30).In my opinion you should consult gastroenterologist and get done1. Upper GI scopy2. Esophageal manometry and motility study3. Barium swallow4. CT neck.This all are needed to rule out esophageal pathology. And if everything is normal than no need to worry as you are not having any disease."
},
{
"id": 131422,
"tgt": "What are the symptoms of fibromyalgia?",
"src": "Patient: Hi there, I have been experiencing recurrent neck spasms, and shoulder pain, for about a year, and lower back pain on rising which has become increasingly painful, I also have the MTHFR A1298C polymorphism, generalised anxiety disorder, heart palpitations and regular head aches, alternating diahhroea and constipation, ongoing muscle twitching and fatigue.. recently I have been having muscle jerks too, I have had lots of blood tests, which show low iron, subclinical hyperthyroidism and am always told that I should go on antidepressants which I am not comfortable with. I had glandular fever shortly before my symptoms began approx 2 years ago and I m wondering if its possible that I have fibromyalgia? I also need to find a doctor that is familiar with the MTHFR as none I have seen thus far know about it yet.. I must find out if the doctors listed on this site bulk bill and see if I can get an appointment.. your thoughts are much appreciated. Kind regards Doctor: HiDo not worry.Just take folic acid tabs and methycobalamin 1500iu daily, lyrica 100 mg BD, Amitriptyline 10 mg bed time.you will notice most of your symptoms,diarrehea,constipatiin, body ache,fatigue shall disappear in 2 weeks time or so. Also take tramadol 100mg BD for pain.it will have miraculous response.Consult a rheumatologist also for any more evaluation.Folic acid and methycobalamin should continue for atleast 3 months as mainstay of MTFHR problem...it will improve many processes of metabolism, restoring themBest wishes"
},
{
"id": 55670,
"tgt": "What causes elevated values of liver enzymes?",
"src": "Patient: Yes hi. I have recently had blood work done and my doctor said my liver enzymes are elevated. I ve also been on a low dose of thyroid medicine for about 6 weeks now. For the past week I have noticed slight swelling on the top of my right foot and ankle. Could that be liver related? Doctor: Hi, dearI have gone through your question. I can understand your concern. Your liver enzymes are elevated. It can be seen in liver damage. It can be due to viral hepatitis, alcohol, drugs or toxins. You should go for complete liver function test and ultrasound abdomen. It will give you exact idea about cause. Then you should take treatment accordingly. Your thyroid problems and swelling in ankle has no relation with your elevated liver enzymes. Consult your doctor and take treatment accordingly. Hope I have answered your question, if you have doubt then I will be happy to answer. Thanks for using health care magic. Wish you a very good health."
},
{
"id": 61410,
"tgt": "What causes a tender lump on the neck?",
"src": "Patient: three year old complaining of lump under skin on side of neck is tender to touch. When feeling it moves under the skin. Size is about nickle in size. No neck injuries ever. Just got off antibiotic for strep and a UTI about 5 days ago. Occasionally complaining of stomach hurting but not when using the bathroom. No sore throat any longer. Doctor: Respected user , hiThanks for using Healthcaremagic.comI have evaluated your query thoroughly .* Tender neck lump is mostly lymph node enlargement following strep throat .* Usually recovers once acute phase of infection is subsided .Hope this clears your query .Welcome for further assistance .Regards ."
},
{
"id": 205231,
"tgt": "Suggest treatment for anxiety",
"src": "Patient: While I understand I m not a sociopath I still have not found the reason why I have trouble feeling certain emotions (love, humility) my only possible explanation would be my self diagnosed aniexty, just really looking to see if anyone had a clean answer.. Doctor: Hi, i will suggest u to consult psychiatrist. along with i will suggest to take tab.inderal (10) for anxiety symptoms whenever u get. but u give detail information regarding your anxiety producing situation which give us better treatment suggestion."
},
{
"id": 59477,
"tgt": "Upper abdominal pain below the ribs. Loss of apetite. Tiny polyps on gallbladder. What could it be?",
"src": "Patient: Hi I am having upper abdomenal pain which extends down my right side mainly at the base of my right rib area. My appendix area isnt sore at all. I cant eat or drink and when i do i get sharp pains and want to vomit - i have no appetit at all. It started 3 weeks ago with contraction like feelings in the first week then pain and discomfort constantly since. I have had a ultrasound done and all that showed was some tiny pollups on my gall blader. I have also had bloods and a urine test which all came clear. My poo test was fine as well. Though I did notice some stone like items in it and was told they werent stones. What could it be? Doctor: Hello, Symptoms of gallbladder polyps are usually very mild or nonexistent. The main symptom is usually pain and tenderness in the upper right abdomen. The pain can be steady or intermittent. If the pain is severe, it is usually caused by gallstones, not polyps. If there is no other problem with the gallbladder other than the polyps, no treatment will probably be performed unless the polyps are extremely numerous or over 1 cm in size, which could indicate cancer. In that instance, the gallbladder will most likely be surgically removed. Theoretically, if the polyps were large and/or located near the opening of the gallbladder, they could interfere with the normal drainage of bile out of the gallbladder and cause symptoms similar to those potentially caused by gall stones.\u00a0 If all other possible causes of your symptoms of pain are ruled out, then it is worth reconsidering the gallbladder polyps as a cause of your pain.\u00a0 Thanks."
},
{
"id": 119569,
"tgt": "What causes sore tailbone and urinary incontinence?",
"src": "Patient: Hello Doctor. My 45 year old husband has recently developed a sore tailbone and urinary incontinence at night. Could you give us any idea what the problem could be? Both the sore tailbone and incontinence started at the same time. As far as he can remember he has not injured himself in that area. He enjoys good health generally. Doctor: Hello, Sore tail bone can be due to injury, prolonged sitting or after a long trip, infection or slipped disc. If it is associated with urinary incontinence then I shall advise you to consult to your doctor at earliest. He may be prescribed a MRI to rule out the problem. Take care. Hope I have answered your question. Let me know if I can assist you further. Regards, Dr. Mukesh Tiwari, Orthopedic Surgeon"
},
{
"id": 221744,
"tgt": "What causes itchiness in the legs during pregnancy?",
"src": "Patient: Hi, my wife is pregnant and 8th month is running. Recently due to itching in legs doctor suggested for LFT. she has a high level of sgot and sgpt. SGOT- 279 U/L ,SGPT- 248 U/L and Serum Alkaline Phosphatase- 310 U/L. doctor has given Ursodeoxyholic Acid Tab and Levo Cetrizine. Is this safe during pregnancy. Doctor: HelloItching is commonly seen in pregnancy.Raised SGOT and SGPT may be due to cholestatic jaundice and it is common during pregnancy.Your wife has been prescribed right medicines.Ursodeoxyholic acid tablet helps in regeneration of liver cells.It will correct raised SGPT and SGOT.Levo cetrizine is an anti histaminic and it will relieve from itching.These medicines are absolutely safe during pregnancy and it is recommend by FDA.Get well soon.Take CareDr.Indu Bhushan"
},
{
"id": 51837,
"tgt": "Explain the ultrasound scan report?",
"src": "Patient: My fathers ultra sound scan reprt shows that right renal lower polar anechoic cortical cysts of 56x48mm,23x22mm noted and they had written impression as right renal lower cortical simple cysts ans moderate prostatomegaly.can u please explain me about this.. Doctor: Hi,Welcome to HCM. Cyst is a closed sac, having a distinct membrane and division on the nearby tissue. It may contain air, fluids.renal cyst is a benign non-neoplastic mass of unknown etiology arising in renal parenchyma.Prostate is gland of male reproductive system, enlargement of this gland causes urinary incontinence. I hope i answered your query."
},
{
"id": 170475,
"tgt": "Concerned about my son tilting his head when held upright",
"src": "Patient: my baby was born on 23d of jan this year.his weight was 2.7kgs.my edd was on feb 8th.now he is 5 months old.my problem is when we put him on shoulder he slightly puts his head to left side, while we make him to sit on our lap,i can see him doing this.he moves his neck normally.he communicates wit me,puts his leg to mouth.turns around from stomach to back lifts his head.i took his 2 hands and lifted he gets up brings his head to me.iam just concernd about the bending of head to left side.please help me Doctor: Hi, when you put your child on shoulder the child's head is tilted on neck side, this could be due to increase tension in neck muscles particularly sternocleidomastoid muscle of neck. This often gets corrected as child grows but if this problem persists after 2 year than surgery is required. I hope this will help you. Take care."
},
{
"id": 101923,
"tgt": "Will allergies cause blood stains in mucus?",
"src": "Patient: My forteen year old who suffers from allergies started having bloody mucus. It's not present when blowing his nose but only when he snorts in and then hocks it up. He also sounds congested but his nose is stopped up. Any idea what could be causing this? Doctor: HI, thanks for using healthcare magicThe blood is likely the result of the inflammation of the lining of his nose due to the allergies.If he is able to adequately treat his allergies then the blood tinged discharge would also likely resolve.You may want to consider a topical steroid nasal spray and antihistamine. Some persons need additional allergy medications such as singulair.In addition, if the source of his allergies can be determined by allergy testing, then they can hopefully be eliminated.I hope this helps"
},
{
"id": 151936,
"tgt": "I want to know the latest advanced treatment against Ctomegalovirus",
"src": "Patient: hi doctor, i konw if it is for free many of u will ignoe it. But, i want you to know that i m trying helping some one who is in great need of you. My friend's daughter is 1 and 1/2 years but she can't sit, stand has no teeth, and doctors say that she is behind her age. Please guide me, i can't see my frnd in such deep trouble. I want to witness a miracle. Doctor: Hi, if her milestones are delayed, you can give her homoeopathic medicine, Baryta carb 1M, and consult a good homoeopathic physician. Homoeopathy gives very good results in developmental disorders."
},
{
"id": 138601,
"tgt": "Suggest remedy for swollen ankles with red spots and ulcers on tongue",
"src": "Patient: hi having a problem the last 2 weeks with lower feet,legs and ankles swollen ,there are red spots and there are hot to the touch...also much fatigue and ulcers on the tongue...Dr checked for blood clots .there were none and gave me keflex...but no help Doctor: Hi, swelling in dependent part of body is sign of edema, and that along with rash or spots, could well be an indicator of viral infection, watch for fever and myalgia, as well as any other associated symptoms.Ulcers on tongue could be due to deficiency of some water soluble vitamins, like vit C And B. taking suppliments with multi vitamins can help to some extent."
},
{
"id": 53216,
"tgt": "What is the life expectancy of a person with 25% functional liver?",
"src": "Patient: My son in law was in the hospital a month ago with advanced liverdisease, he had 1 1/2 gal of fluid drained from his abdomen and two dayslater another 3/4 gal. The biopsy confirmed he has only 25% of his liverleft, he is very yellow in the skin and eyes. Can you tell us what toexpect? I know it is hard to say but how long might he have? Doctor: Hi and welcome to Healthcaremagic. I understand your concerns.It depends on the exact cause of liver failure and generally 25% is something man can live with for more years, but if there is cirrhosis or tumor present then the prognosis is very poor and may take from few weeks till few months. Jaundice and ascites are generally bad prognostic factors and I am afraid that you should not be very optimistic but it is har t predict sch things and in everx case you should be with him and give him all necessary support.I hope I have answered you queryKindly regards. Wish you a good health."
},
{
"id": 7112,
"tgt": "With conradi syndrome can I conceive at the age of 39 ?",
"src": "Patient: Can i have a baby? I am 3 9 and i have conradi syndrome Doctor: Hi Welcome to HCM Read your prob, this syndrom happen due to genetic abnormalitis. it has many no. of symptoms , this does not affact fertility so you can get pregnant. nothing to worry. as this is rare syndrom n occure due to mutation of gene so there is less chance that your baby will have this syndrom."
},
{
"id": 27700,
"tgt": "What are the treatments for angina attack?",
"src": "Patient: I have coronary heart disease .. had angina attack last night .. during sleep .. followed by tremendous pain in right temple .. pins and needles in my face .. lasted about 10 mins .. ambulance called blood pressure was 131 over 152 .. decreasing along with the pain on crew leaving .. suggest it needs checking out .. refused ambulance but going to see GP tonight .. thanks Doctor: Hello ,I have gone through your question.first of all if you are sure that it is anginal pain then you should take sublingual isordil 5mg.and you must go to ER.as now you are pain free please go for ECG and further work up .my best wishes .Dr.Teli,MD"
},
{
"id": 99267,
"tgt": "Suggest effects of dampness & mouldy environment on health",
"src": "Patient: We have a damp house, the wardrobes are very smelly & the clothes are slightly mouldy,no seen mould on walls though, my daughter suffers from severe eczema & I can not shake a cough I have had for a few weeks, with no history of asthma, what effects does a slightly damp house have on my family? any link to our health current problems ? Doctor: HI, thanks for using healthcare magicThe damp conditions would increase the formation of mold and also other fungi.In addition, dampness also increases the risk of acute asthmatic attacks, sinusitis. This can cause cough, sneezing, runny nose, rash, itching eyes.I hope this helps"
},
{
"id": 38165,
"tgt": "Am I having chronic mono?",
"src": "Patient: Hi! I was diagnosed with mono about 4 weeks ago, but I have had it since January. I have had good days to where I can get out of the house and other days to where I won't have an appetite or get out of bed! I was reading some info about chronic mono and was wondering if I could have it? Doctor: Hello,I understand your concern.Most cases of mononucleosis, mono, are caused by infection with the Epstein-Barr virus (EBV) and get it only once. Sometimes mononucleosis symptoms may recur months or even years later.Once you're infected with EBV, you carry the virus for the rest of your life. The virus may reactivate off and on. The virus can then be detected in the saliva but one does not usually become ill.In mono one often has a general feeling of malaise, starting with non specific symptoms such as fatigue, headache and sore throat. Your main symptoms are lack of appetite and lack of energy and these are not likely to be the cause of Mono.I think you should see an Internist and have your condition investigated and diagnosed.Dr. Noble Zachariah"
},
{
"id": 72579,
"tgt": "What causes constant shortness of breath?",
"src": "Patient: hi....i am having shortness of breath all the time..and there are times it jus goes away. The most difficult part is that i m unable to talk freely due to this. i am unable to speak long sentences and this is frustating me. This is taking a toll and coming in my routine life . i have been to doctors and they have all kind of possible tests and they cant see why. I m really frustated and need to start breathing n talking . Its like when i talk i stop breathing. Thiw has been hapenin since 3 months and i don knw wht to do Doctor: Thanks for your question on Healthcare Magic.I can understand your concern. In my opinion, we should definitely rule out asthma or bronchitis in your case. So get done PFT (Pulmonary Function Test) first.If this is normal then no need to worry for asthma or bronchitis. Sometimes undiagnosed stress and anxiety can also cause breathing difficulty. So consult psychiatrist and get done counselling sessions. Try to identify stressor in your life and start working on its solution. You may need anxiolytic drugs (propranolol and flunarizine combination) also.Don't worry, you will be alright. Avoid stress and tension, be relax and calm. Hope I have solved your query. I will be happy to help you further. Wish you good health. Thanks."
},
{
"id": 161405,
"tgt": "How much fluid should be given to an infant while teething?",
"src": "Patient: HI I have a 6 month old who is teething. He has started refusing his milk in the last couple of days but will have food and water. Just wondered how much fluid he requires a day so i can monitor it. He is probably having just under 6 wet nappies a day at the moment but seems ok in himself other than the obvious teething Doctor: Hello, There is no specific requirement at this age group and you can give on-demand fluids. But age-appropriately you may give the baby 600 to 700 ml of total fluid per day at this age group if you are not giving any milk. Hope I have answered your query. Let me know if I can assist you further. Take care Regards, Dr Sumanth Amperayani, Pediatrician, Pulmonology"
},
{
"id": 135555,
"tgt": "Suggest remedy for swelling & tingling in ankle & feet",
"src": "Patient: I m 34yrs old 207lbs & 5ft 7in I m swelling in my ankles/feet/& legs again however feeling numbness/tingling/pins & needles in my left leg more then my right. Firstly is that bad - ie the pin & needle feeling in such a case or not - already plan to go to the ER was just looking up info til my ride gets here Doctor: HiU seem to b having some sort of vitamin or calcium deficiency It causes swelling of nerves causing tingling numbness U should get an detailed blood profile done n give me a follow-up Take care"
},
{
"id": 127786,
"tgt": "What causes muscle soreness despite a normal blood work?",
"src": "Patient: Hi I am 64 years old and about 8 months ago I had a panel of blood work done with my bun test slightly elevated. About 2 months ago I saw my doctor again and complained of muscle soreness and possibly joint pain, lack of energy etc. Noticing the elevated bun test he did a complete work up on that with the bun test coming back as normal and all additional test in that regard also showing up as normal. My symptoms continue to persist. Any suggestions? Doctor: Hello,What I can infer from your complaints is that you may be having age related osteoporosis. It is great that all your blood workup is normal.I would suggest you to get your VIT D3 and VIT B 12 level. if already not done. If the reports show low levels, then you need vitamin and calcium supplementation.Hope I have answered your query. Let me know if I can assist you further.Regards,Dr. Santosh S Jeevannavar"
},
{
"id": 71117,
"tgt": "What causes thick greenish sputum and intermittent cough?",
"src": "Patient: Starting about a week ago, I coughed and noticed yellowish phlegm even though the cough was not frequent and I didn t cough up much at all. The past few days it is thicker and green but the cough is not frequent. Yesterday after showering and dressing for the day I was so tired. No temperature but I get real cold and later too hot. Should I be very concerned. Surgery to repair a broken tibia was done Feb 19, 2018. I have no spleen Doctor: Hello and Welcome to \u2018Ask A Doctor\u2019 service. I have reviewed your query and here is my advice. * The green sputum with intermittent cough, weakness, fever are signs of lower respiratory infection. * You must get an x-ray chest and complete blood counts done to manage it precisely before it gets worsen. * Till that time, maintain hydration with plenty of liquids, do gargles with salted lukewarm water. Hope I have answered your query. Let me know if I can assist you further."
},
{
"id": 172358,
"tgt": "What causes bleeding from gums?",
"src": "Patient: My almost 2 yr old has what it looks like to be bread in between her 2 front teeth. I tried to take her toothbrush and remove it and her gums started to bleed . You can see she is in pain. I thought she wasn t eating cause of her 2 yr mollers but now I m thinking infection Doctor: Hi,Thanks and welcome to HCM .Gums are very delicate in a 2 years old baby.The bristles of the tooth brush may injure gums and lead to bleeding.Gum infection and vitamin C deficiency may lead to bleeding .within 1-2days if the bleeding does not subside consult your doctor/ dentist.Hope this answer serves your purpose .Further queries invited .Dr.MV.Subrahmanyam."
},
{
"id": 149864,
"tgt": "Growing lipoma on back near the spine. Diagnosed not cancerous. Chances of nerve damage?",
"src": "Patient: I was diagnosed with a lipoma on my back. It is about an inch in diameter proximal to my vertebral column. I was told these are not cancerous, but I would like to have it removed because it appears to be growing instead of decreasing. Since it rests so near to my spine, what are the chances of nerve damage. Also, is it likely to return? And oh, I almost forget no insurance, so I'll be paying out of pocket. How much, please? Doctor: Hi and welcome to Healthcare magicLipoma are exxtremely rarely malignant orbecome malignant. These are benign soft tumors which cant press nerves or important surrounding tissue. f there is cosmetic concern it can be relatively easily surgically removed in local anasthesia. Hi and welcome to Healthcare magic. Regards Wish you good health."
},
{
"id": 105688,
"tgt": "Skin rashes, citrizen, dexoma, avil injection, omnacortil 10, altarax 25mg, skin allergy",
"src": "Patient: Our daughter has been suffering skin rashes from the last twenty days. We have administered some allergy tablets like citrizen and dexona . But the problem is being repested after 24 hours .Yesterday one of doctors administered avil injection and given omnacortil 10 and altarax 25mg for skin allergy to my daughter. Please advise in this regard Doctor: give antiworms"
},
{
"id": 81662,
"tgt": "What causes persistent pain in chest with difficulty in breathing?",
"src": "Patient: i have this problem when 1 morning i woke up with this pain in my left chest and i also have difficulty in breathing, coughing, burping, i can only have 1 sleeping position since this pain bothered me, in fact there are times that i can no longer sleep due to too much pain that no position could be done to endure the pain. I think the pain affects my upper left abdomen, left shoulder and chest. This pain is already 6 days today. I already consulted a cardiologist and he advised me to have ECG and xray and i did, he also prescribe me pain reliever and a medicine for blood circulation... i followed him and already took the medicine, after hours of subsiding the pain, it will return again some times more intense. I am already suffering a lot and i no longer can wait Monday to have the ECG and xray result be released. I am really suffering, pls help me explain this, i am worrying a lot. what should i do re this... Doctor: Thanks for your question on HCM. Since your cardiac work up is normal, I think you are having GERD ( Gastro Esophageal Reflux Disease ) mostly. It is due to laxity of gastroesophageal sphincter. Because of this the acid of the stomach tends to come up in the esophagus and cause the symptoms. Try to follow these steps for better symptomatic relief. 1. Avoid hot and spicy food. 2. Avoid large meals, instead take frequent small meals. 3. Avoid stress and anxiety. 4. Start proton pump inhibitor. 5. Go for walk after meals. 6. Keep 2 - 3 pillows under head in the bed. 7. Avoid smoking and alcohol if you have this habit. 8. Loose weight if you are obese. If not improving after these then get done ECG again."
},
{
"id": 106335,
"tgt": "I wake up always in the middle of the night, and cant breath at all",
"src": "Patient: wake up always in the middle of the night, cant breath , flushed i wake up always in the middle of the night, cant breath, flushed... on the verge of passing out and sometimes i have, have to use my nebulizer machine (i have asthma ) sweating profusely. sometimes this last about 20 mins. Doctor: Hi.. ... How many times do you wake up from deep sleep with such symptoms? The symptoms that you describe are referred to as hypopnea... Multiple such episodes occuring throughout the night is a feature of sleep apnea syndrome... You need sleep studies(polysomnogram), weight reduction programme and correction of an ENT problem if any... If such episodes are severe and multiple, you may need the help of breathing support such as CPAP when going to bed... Consult a pulmonologist, who will examine you, evaluate, suggest specific treatment plan and guide you further... Regards"
},
{
"id": 27840,
"tgt": "Suggest remedy for pain in upper chest and back with crystals around the heart",
"src": "Patient: I have upper chest and back pain. I went to emergency and they checked my lungs and heart and everything is fine, some crystals around my heart and I will follow up with doctor but they said this awful pain is probably just a strain. What can I do to get rid of this paid? It gets worse and worse the more I move. What else can this be. I made an appointment with my regular doctor but can t get in until next Friday. Any suggestions? Doctor: Dear Sir/ MadamThanks for your query.1. It is not clear that what your doctor meant by crystals around heart.2. A pain which worsens by moving is not likely to be due to angina or heart attack.3. Pain which worsens on movement can be due to muscle, bone or joint related problems. Pain arising from inflammation of coverings of heart and lungs causes such pains which changes with posture and respiration.4. Re-consult your ER doctor and at least get a prescription of pain killers if it seems a local cause to them. It is not possible to make an exact diagnosis without an examination.SincerelySukhvinder"
},
{
"id": 206082,
"tgt": "Suggest treatment for anxiety and depression",
"src": "Patient: I am on 4 different medication for anxiety/depression and what to come off all of them. Do you think that s to much? It dosen t help I have anxiety all the time and the psychiatrist just tried adding abilify to the other four medications. our opinion????? Doctor: DearWe understand your concernsI went through your details. I suggest you not to worry much. The psychiatrist prescribe medicines after consulting your present and past condition and according to their experience in practice. You have not given the names of all those 4 medicines and therefore, I cannot comment on that.You should know that these problems are not mental diseases, but are mental disorders. Many researches and researchers confirm that medicines alone cannot cure mental disorders. Exercise, Life style changes, change in thinking pattern, relaxation etc are as essential as medicines. Psychotherapy can help you changing your lifestyle and thinking patterns. Yoga and meditation help you to streamline your metabolism and neurological balance. Please consult a psychologist for further information.Psychotherapy techniques should suit your requirement. If you require more of my help in this aspect, Please post a direct question to me in this URL. http://goo.gl/aYW2pR. Make sure that you include every minute details possible. I shall prescribe the needed psychotherapy techniques.Hope this answers your query. Available for further clarifications.Good luck."
},
{
"id": 81849,
"tgt": "Suggest treatment for tightness in chest",
"src": "Patient: Hi I m a 31 year old lady, I have just been diagnosed with low blood pressure, below my normal level s which is low any way, I have started getting punch like feeling in my chest I m quite concerned!! Any tps or answers would be grateful, I ve booked in t see my GP also?? Doctor: Thanks for your question on HCM. Low blood pressure with chest tightness is seen in many causes.But common ones are1. Cardiac cause2. Anemia3. Hypothyroidism etc.So get done ECG and 2D Echo to rule out cardiac cause first. If both are normal than no need to worry for cardiac cause. Get done CBC ( Complete Blood Count ) and S.TSH to rule out anemia and thyroid diseases. If both are normal than it must be anxiety and stress related. So avoid stress and tension."
},
{
"id": 208992,
"tgt": "Suggest remedy for mental health problem",
"src": "Patient: hi! my name is Aswin, studying in 12th class. i think i have some psychological problem. but i am not sure about it. Actually, my problem is that i use to imagine a lot, something about my life. Actually , its my hobby.Everyday i use to imagine about something. This has become a daily routine in my life. when i saw a movie i use to imagine such situations in my own life and create a new story. because of this i am facing a lot of problem. Like when i am imaging something very sad, i use to be emmotional, and happy in the case of a happy occasion in my imagination. Also i was having a girl friend but i was not so close with her due to some situations. But i was loving her sincerely.Even though she was not so close to me, when she left me i was felt so bad that i don t know what is just happening to me, idon t what to do and what i am feeling at that time. i think this is because i use to imagine a lot about her that a strong affection was created in me for her. when i discuss these with my friends, they told me that you are feeling this because may be i am having these type of experiences first time in my life. but its not the true i experiences these type of situations more than 5 times, as i was having many girl friend before. now, i want to conform whether i am having a psychic problem or not. so, please me. Doctor: HiThanks for using healthcare magicI think, you have some adjustment problem. Sometime, due to some reason,people become obsess about any people. In that case, they feel the same way, you are thinking. Better to consult a psychologist and try to control such thoughts. You can also try some relaxation exercise that would keep to calm or relax in tense situation. In case, you need further help, you can ask.Thanks"
},
{
"id": 38825,
"tgt": "What are the symptoms and treatment for influenza?",
"src": "Patient: I just have a situation. I am 18 and went to the pediatrics for flu like symptoms but I have had a lot of complications with mono before and my doctor told me she was testing me for the flu, mono and testing my cbc's. Well about 3 days later she calls me to tell me that all my blood work was fine but I had weed in mu system can she test me for drugs like that without my consent?? Doctor: HelloWell you were tested for flu , C B C , mono .But doctor found 'weed \" in your blood . Now your question is \"can she test weed , without your consent \"Actually this is a legal question and depend on the country , you belong , so take an advise from a legal expert."
},
{
"id": 168349,
"tgt": "Suggest better treatment for constipation and sticky stool",
"src": "Patient: My son is four years old and suffered with chronic constipation since I stopped breastfeeding at approx six months old. He is on Movicol which usually works ok. He is actually able to open hiis bowels every day without Movicol for about two weeks. Then he reverts back to not going for a few days and when he does finally go his stools are very very sticky! Any help? Doctor: Hi...Thank you for consulting in Health Care magic. I think your kid is having habitual constipation. I have certain questions and suggestions for you.Questions:1. Did your kid pass motion or meconium on day one of life?2. Since how long is the kid constipated?3. Does the kid have any bleeding along with hard stools?4. How much milk does the kid consume per day?5. Does the kid eat fruits and vegetables (fibre diet) appropriately?You can get back with answers at the following link - www.healthcaremagic.com/doctors/dr-sumanth-amperayani/67696Suggestions:1. Natural methods are the best to relieve constipation.2. Constipation is a risk factor for UTI3. Maximum milk consumption per day should not exceed 300-400ml4. Minimum 3-4 cups of fruits and vegetables to be consumed per day5. Toilet training - that is - sitting in Indian type of lavatory daily at the same time will help a lot.Hope my answer was helpful for you. I am happy to help any time. Further clarifications and consultations on Health care magic are welcome. If you do not have any clarifications, you can close the discussion and rate the answer. Wish your kid good health.Dr. Sumanth MBBS., DCH., DNB (Paed).,"
},
{
"id": 200221,
"tgt": "What does blood in semen indicate?",
"src": "Patient: Hi doctor, My husband just noticed blood in his semen, he says twice and is freaking out. He is 60 yrs old and had a heart attack at the end of August 2011, takes daily meds morning and nighttime. Can this be potentially serious, he immediately thinks the worst, why I m seeking some enlightenment. Should we seek a urologist? MJ in Mississauga On. Doctor: Thanks for asking in healthcaremagic forum It is unusual to get blood in semen without injury/infection/other causes. So, please visit an Urologist immediately for investigating the cause of bleeding from urinary tract. All the best."
},
{
"id": 225954,
"tgt": "Loss of sex drive and mood swings after using implanon. Had a miscarriage",
"src": "Patient: Hi ,I was previously on the implanon then i was taken off it again within 2 weeks i was pregnant. unfortunatley this lead to a miscarraige. so i decided i wanted to go back on the implanon but since i been on it i feel my moods are alot worse and i have lost my sexual drive. Im not sure if this is because of the miscarraige or if the implanon has just made me less interested. Doctor: Hi,Hormonal imbalance created by contraceptive devices can lead to mood disturbances and as you said, your miscarriage is bound to have a subconscious inhibitory impact on the sexual drive fearing the consequences. Please discuss with your health care provider if you have the need to change to another mode of contraception and try to get some counselling to allay your fears. Hope this helps. Take care."
},
{
"id": 177171,
"tgt": "How to handle tantrums?",
"src": "Patient: How do I get my 7 year old daughter from throwing so many tantrums? I have had enough of it and just want her to stop. She is unreasonable and I am not able to talk to her when she is in the midst of this fits. I often walk away from her or lock myself in a room just to keep myself calm, but she follows me and kicks the door. This outburst occur when she doesn t get her way. Doctor: Temper tantrums can not be controlled pharmacologically. the parent has to keep his calm and give multiple time outs to the child once the tantrum starts.The best way is to intercede before the tantrum is about to start.Give choices to the child and explain him/her about the condition.dont show your anger upon him, it will only worsen the condition.Show him how would you react if you were angry at some point and make him learn to control his anger as you do."
},
{
"id": 112462,
"tgt": "Have back pain in the rib cage. Dehydrates, drinks more coffee. Pain induces breathing problem. Treatment?",
"src": "Patient: My husband experienced sudden onset of back pain very specifically right side just at the bottom of his rib cage. He tends to dehydrate and drinks more coffee than water at times. He is an athlete, very healthy. Pain is not radiating anywhere else. The pain is currently so bad he is having a hard time breathing. Not SOB, but just his breathing is hard I think due to pain. The pain is worse when lying down. Sitting upright and forward helps, but does not subside pain. Any clues. ER visit? I have him drinking water and he just downed 32 oz. Suggestions? Doctor: Hi, the pain at the bottom of the right rib cage suggests that there may be a stone in the kidney, or at the pelvis of the ureter. It also may be the infection of the kidney. There could also be gall bladder infection, or gall stones, with associated gastric irritation. I advise you to consult a surgeon for diagnosis and treatment. He may need to have M.R.I.for confirmation. Thank you."
},
{
"id": 15664,
"tgt": "Rash on breast region cascaded down abdomen. Bumps on back, arms, inner thigh, big red patches. What is it?",
"src": "Patient: I have a rash primarily located on my breast region that has cascaded down my abdomen, with light small bumps beginning on my back, arms, and inner thigh. The small bumps look similar to previous contact dermatitis, however in the area mostly concentrated under my breast I have bigger red patches around 2-5cm in length. I am quite itchy and neither fungal nor bacteria creams have worked. What may I have? What can I do about it. Doctor: Hi, thanks for your query.As per your description of the rash and its areas of involvement it seems to be a fungal rash. It occurs in the areas which are prone to be moist like body folds. moisture and sweat facilitates the growth of fungi.To treat the same you may require a course of oral anti fungal drugs along with local anti fungal creams, and oral anti histamines for itching.You may consult a dermatologist for exact diagnosis and appropriate treatment.Hope this solves your query. You may write back if you have further queries in this regard.Wish you a good health."
},
{
"id": 166785,
"tgt": "Suggest remedy for green mucus from one nostril and problem in breathing and sleeping",
"src": "Patient: my baby is eight weeks old and from day one has had green mucus from one nostril whilst the other nostril is dry mucus now one of the nostrils he is unable to breathe through. he has no other signs of infection and eats and gains weight well just has truoble breathing and sleeping now. any advice will be great Doctor: Hi,Cough and cold are viral 95% of the times in children. For cold you can use anti-allergics like Cetirizine and for nose block, saline nasal decongestants will do. For fever - Paracetamol can be given in the dose of 15mg/kg/dose (max ceiling dose 500 mg) every 4-6th hourly, that too only if fever is more than 100F. I suggest not using combination medicines for fever, especially with Paracetamol. For cold you can use Cetrizine at 0.25mg/kg/dose every 12 hourly for 3 days. But at this age you cannot use any medication for cold because no medicine has been licensed at this age group. For nasal block, plain saline nasal drops will do, every 4-6th hourly to relive nasal congestion.Hope my answer was helpful for you. I am happy to help any time. If you do not have any clarifications, you can close the discussion and rate the answer. Wish your kid good health.Regards,Dr. Sumanth MBBS., DCH., DNB (Paed)."
},
{
"id": 29063,
"tgt": "What causes aspiration pneumonia?",
"src": "Patient: I have been diagnosed with aspiration pneumonia the ER doctor said it was due to being strangled unconcious a month ago but I read on line it could be due to a bad tooth, which I have. I asked my PCP to allow the dentist to pull the tooth in question after the xray showed it was badly infected. My doctor said that because my blood pressure was so high (we have been trying to get it under control for 2 years now still averages 180-190/124-102... But if I was able to have the teeth pulled the infection would clear up and my blood pressure would go down...Am I not seeing things right or is my doctor being over cautious?? Doctor: Hello,You need to get Blood pressure under control. Such a tooth extraction really needs good experience, equipment and back up facility also.Hope I have answered your query. Let me know if I can assist you further.Regards,Dr. Kaushik Saha"
},
{
"id": 109568,
"tgt": "What causes severe burning sensation and ache in lower back?",
"src": "Patient: Hi, may I answer your health queries right now ? Please type your query here... I have for couple days had a burning sensation in lower back..above my waistline. Back also aches. Also upper right abdomen. Perhaps strained bag lifting suitcase in airplane compartment...... Doctor: Hello,As you complain of pain in the back and lower abdomen or has two most possible reasons.1 Renal or ureteric calculus, urinary tract infection (UTI)2 Muscular.For these two conditions, I would advise doing an ultrasound of the abdomen to rule out stone. So consult your family physician and do further plan accordingly.Hope I have answered your query. Let me know if I can assist you further.Regards, Dr. Simit Jadia"
},
{
"id": 148061,
"tgt": "Is it safe to take zanax while taking keppra for Meningioma tumor?",
"src": "Patient: My father is taking Keppra 500 mg 3 times a day. he just had a Meningioma tumor removed six weeks ago. I just discovered that he is also taking zanax 1.mg. he cuts the pill in half. I'm pretty sure he only takes it once a day before bed and maybe not even every day. I am sure his doctor does not know about it....stupid question, but is this harmful to him? If so, how? Doctor: HI, thanks for using healthcare magicXanax is the trade name for a medication called alprazolam. It is in the family of medications called benzodiazepines.If he is using a small dose and not daily, he may not experience any significant effects but this class of drugs is associated with developing dependence to the medication and tolerance (needing more and more medication for the same effect).There are also other potential effects such as confusion, dizziness, drowsiness.I hope this helps"
},
{
"id": 87961,
"tgt": "What causes stomach pain and vomiting?",
"src": "Patient: went to the doc today thinking i had the crud. over the weekend i woke up vomitting and thought it was food poisoning and i have had diarrhea ever since with some stomach pain. the doc said it may be my appendix. she put pressure on right lower abdomin and ever since i have had some pain and also vomiting. what do you think Doctor: Hi.Thanks for your query and an elucidate history..Curd causing a gastroenteritis not heard of in fact can help in restoring the bacterial flora as it is a probiotic. Since you have now got a pain in the area of appendicitis after putting a pressure and your Doctor has suspected appendicitis, I would request you to get your Blood, urine and stool checked. Also to get a confirmatory diagnosis by Ultrasonography and / or CT scan and t go for a Surgery if present on investigations ."
},
{
"id": 126539,
"tgt": "What causes hip and back pain after prolonged sitting post injury?",
"src": "Patient: Hy.. i had meet an accident in nove 2015..am badly injured in it..exra shows some space in my back joint...now am having some problems..when i sit for a long time i had pain i my hips joint .. back also in bakbone...i cant move for a while when i continuously work or walk...what i do..how i resolved this matter please suggest me.... Thank u... Doctor: Hi, According to your query, your symptoms are suggestive of sacroiliac joint dysfunction. For that take some painkiller twice a day regularly for a month. Start walking and increase gradually up to 4kms at a stretch. Basically, you need to strengthen your core muscles. Hope I have answered your query. Let me know if I can assist you further. Take care Regards, Dr Anuj Gupta, Spine Surgeon"
},
{
"id": 221520,
"tgt": "Can different blood groups of the partners lead to illness during pregnancy?",
"src": "Patient: My blood type is O+ and I was married previous and my pervious husband also was O+. I was pregnant for several times and I never get sick though out my previous pregnancies. After I got divorce I got remarried for a new husband and his blood type A+ and we decide to have a new baby then after I get pregnant I became very sick, I was having high fever, vomiting a lot, and tireless which I never had this symptoms before. My question is since my new husband and I have different blood type is that the cose of my sickness? Doctor: Hi,No a different blood group should not cause you to have such symptoms. A different blood of a new husband could probably give you a slightly increased chance of high blood pressure, and pre-eclampsia in the later part of pregnancy.Fever, vomiting and tiredness, could be a infection unrelated to pregnancy. You should see a physician and get get blood test to check for infections and take medicines for fever and the other symptoms. Do discuss the safety of medicine during pregnancy before taking them.Hope this helps.Regards."
},
{
"id": 124019,
"tgt": "How to treat swollen foot?",
"src": "Patient: My friend got stung on the foot approx. 3 days ago . It s been swollen ever since . It s to the point where she puts her foot on the floor & her toes dont touch. It s beginning to turn purple and red. I advised her to go to the do tor and she doesn t believe she needs to. Does she need to go to the doctor? Doctor: Hello, As seen in history, your friend has an ankle injury and I will straight away request for the x-ray and the orthopedist recommendation. As turning the color to purple or red indicates a pathology and needs to be examined. Most of the times, the injury is ignored and it heals in an unnatural alignment and which may lead to abnormal changes in the biomechanics. As the foot is one of the important parts of the body since it has to bear the whole body weight I would not recommend taking this lightly. Any abnormal bio-mechanical changes will not be good and may lead to other abnormal bio-mechanical functions of the joints like - ankle, knee, hip and lumbar spine. To avoid such issues it is recommended to see an orthopedist without having a second thought. Usually, a team of orthopedist which includes even a physiotherapist will help to work behind the foot injury and give optimum results. Hope I have answered your query. Let me know if I can assist you further. Regards, Jay Indravadan Patel, Physical Therapist or Physiotherapist"
},
{
"id": 217150,
"tgt": "How can shoulder, hip and back pain be relieved?",
"src": "Patient: G day ... I have / had been taking Di-Gesics for the past 5 or 6 years - and now I am being told that I cannot obtain it any more ?!?!?!? I have Endone - left over - but I DON T want to take IT ( it knocks the hell outa me :o( Can you tell me what I CAN take - that will relieve my pain ( injured shoulder, hip & lower back from a ladder fall into a ship s engine room in 2007 ) .... ??? Thanks Rick Doctor: Take hot fomentation for 15-20 min for each joint. at hip, shoulder and back after that apply diclofenac gel ."
},
{
"id": 200964,
"tgt": "What causes low sperm count?",
"src": "Patient: Hi,i m 24yrs old, i almost masturbate daily from the AGE of 18 and never had sex......now i m finding that my sperm is not as thick as it used to be and the volumes are also very less.........i am just confused, i just wanted to know why is that so and is it possible that sperms can get over or there could be a problem in having a baby from my wife...... pls no joking on this i want a good answers from u Doctor: Thanks for asking in healthcare magic forumIn short: Sperms are produced daily(takes total three months for a sperm to mature) But do not get confused.Explanation: Please do not go by your volume, thickness it can vary from time to time. Sperms are produced continuously so if you are a normal male your sperm will not get over till you reproduce. So, do not worry if you have so much of doubt then go for semen analysis and you will find the answer for your question."
},
{
"id": 108711,
"tgt": "Suggest cure for severe lower back pain",
"src": "Patient: Hello, I went to the doctors yesterday and I have a water infection, he has prescribed antibiotics, overnight I got pain in my lower back and have been in considerable pain, this morning the pain isnt as bad but I feel unwell, do you think I should go back to my doctor or give these antibiotics time to work. I had a cystocopy last year and I have chronic cystitis cystica and i also had my uretha stretched, i went through chemo last year so I havnt healed. Doctor: back pain may be due to urine infection ascending from cystitis to ureter or kidneys. Urine culture and senstivity to antibiotics should be done and one or more anti-biotic be added. Aslo colony count in urine routine examination, no. of pus cells, hematuria, epithelial cells, leukocyte count.An ultrasound of KUB (kidney, ureter, bladder) Take urispas BD and roliten tabs BD for symptomatic relief. Avoid spices in diet and take plain water plenty"
},
{
"id": 145654,
"tgt": "What causes episodes of fainting?",
"src": "Patient: Grandson is 17 and needed to have bowel movement, was traveling and couldn t get to rest room at the right time, had to wait. passed out upon arrival at convenience store. Had also fainted one other time when a shot had to be administered. Is this normal for a teen? Doctor: Hello,I understand your concern on your grandson .I assure you it us nothing you have to worry about.In adolescents a lot of changes occur in a short period. Also their behavior is changing rapidly.In my opinion the troubles of your grandson are caused be a so called vagal dysfunction. It is more common in this age because the nervous system is still not mature. Anxious situations too are the cause. He will grow up and everything will be all right.Hope to have been helpful.Thank you for using HCM.Best wishes Dr. Abaz"
},
{
"id": 112698,
"tgt": "Taking Proair for lower back pain. Reason?",
"src": "Patient: My doctor just put me on Proair HFA a couple days ago and I started having severe pain from my lower back down to the bottoms of my feet. The past 1-10 scale completely off the charts. So I wanted to get to the bottom of it all .I found out I was taking a least 3 or more drugs that is listed on your website that to be known interactions with this new drug I'm currently on.I couldn't sit or lay down and stand more than a minute. I would have to say out of all the surgeries this the worse pain that I every felt. What now? Doctor: Hi Any potent analgesic will aggravate your respiratory symptoms as you are on bronchodialator ( salbutamole inhaler) for the management of your back pain you can try rubificiant cream that will help lot and find out the cause of pain and treat it accordingly first of all consult good neurophysician. Have a good day"
},
{
"id": 35594,
"tgt": "What are the diseases diagnosed by blood and urine?",
"src": "Patient: hello doctor , i am a girl of class 10, we have been given a project on diseases. i need your help, i want to know the names of some diseases that can be diagnosed through blood ,and some diseases that can be diagnosed through urine, but they should be non-pathogenic ones. Doctor: Hello there,You are looking for non-communicable diseases I think. The ones which cannot be transmitted.Some that can be diagnosed by blood:1) Anemia2) Thalessemia3) Thyroid disease4) Heart Attack/Myocardial Infarction5) Rheumatoid ArthritisSome that can be diagnosed by urine:1) Nephrotic Syndrome2) Diabetes3) Kidney failure/renal failureHope this helps,Please ask me if you have any further queries,Dr Arun A"
},
{
"id": 22439,
"tgt": "Suggest treatment for chest discomfort after heart surgery",
"src": "Patient: Hello. My dad had open heart surgery almost one yr ago. I feel that he may have overdone activity early on.He had to help move things out of my grandmothers house because she was put in a nursing facility 2 monthes after surgery. he has since gone to the er and they said it was not his heart. Is it still possble to get chest discomfort from muscles one year after surgry? And when can you be fully recovered? I'm worried about my dad and when i talk to him about it, i can tell he's also scared. My dad is a 62 yr old that is an ex-smoker and is currently retired.Just letting you know alittle on his background! Doctor: Did they put graft during surgical procedure? did he had this kind of pain right after surgery? If so what medications was given? And yes pain can occur after 1 year if he has done strenuous activity. He has to be careful for this with this he has to cut down salt intake and limit his physical activities."
},
{
"id": 224874,
"tgt": "Had unprotected sex, on birth control, vomiting around midnight. Is anything abnormal going on?",
"src": "Patient: Hi, I am a 14 year old girl and me and boyfriend last had unprotected sex on March 7th, 2012. I had my last period end on March 1st. I took the morning after pill and am now on birth control . But I seem to be having some scary signs: I throw up around midnight quite frequently and I have spotting only in the mornings and only when I don t take the birth control pill. Is anything going on that I should know about do you think? Doctor: Hi, You had unprotected sex during safe period and took contraceptive measures so chances of pregnancy is least . After stoping of OCpills withdrawal bleeding occurs so no need to worry . vomiting is somtimes have side effect of vomiting . So in my opinion confirm your pregnancy status through UPT if you don't get your menace after two weeks .Take anti emetic ( domperidone , metoclopramide.....)but if symptom persists or aggravates then visit gyne clinic for further management .Thanks n regards ."
},
{
"id": 214670,
"tgt": "Suggest home remedies for abortion",
"src": "Patient: hi.. im 27 year old married women.. i have not got my menstural cycle this month but since 8 days i am getting brown discharge daily.. today when i performed pregnancy test it came positive but a very light line appeared.. i dont want to have a child now n wish to abort.. Is there any home remedy i can do to get it aborted.. i am 4 weeks pregnent now.. please help.. Doctor: **1. since UPT [urine pregnancy test] came positive [still light line], get tested at Doctor's office to confirm your doubt.PS. since home pregnancy tests, can give false positives, make an appointment with Gynecologist to have the tests done above, since faint line suggest that maybe the pregnancy hormone HCG was just detectable enough to give the faint line, test again after 4 days2. brown discharge with missed period indicates implantation bleeding i.e. [when embryo implants in womb after conception]3. Without an opportunity to physically examine the patient and monitor the process of termination, it is very unsafe and unethical on my part to suggest you the measures. So please visit your healthcare provider/ gynecologist for safe termination.PS. . Medical termination of pregnancy laws remains the same even for ayurveda. Under the act, abortion can be provided only by a registered medical practitioner who has the necessary qualification, training and experience in performing MTP and only at a place which has the facilities, meeting standards specified in the Rules and Regulations of the MTP Act. . Use birth control so that you don't find yourself having to make traumatic decisions about an unwanted pregnancy."
},
{
"id": 109771,
"tgt": "Suggest medication for severe back pain after a surgery",
"src": "Patient: My father just had back surgery last week. His physician told him to double up on his pain medicine and he just ran out of it Wednesday. He is still in major pain due to a fall that he just recently had also. Can they prescribe him more pain medicine? Doctor: Hi, thank you for posting.Your father can take non steroidal anti inflammatory medications such as Advil(Ibuprofen). But before taking Advil, your father must be sure that the surgery wound is not infected.If the wound is infected then your father should take antibiotics.Wish you good health.Dr. Behar."
},
{
"id": 118714,
"tgt": "Difference between Trajenta & Regelide medicines prescribed for controlling blood sugar levels?",
"src": "Patient: i just would like to know the difference between Trajenta & Regelide medicines prescribed for controlling blood sugar levels. My Mom who is 71 years og age has been taking Reglide for quite some time now however she got admitted to hospital due to high sugar levels and was recommended Trajenta there. So what exactly is the difference between these two medicines and which is one better, in case you help me understand. Doctor: Hi. The difference between these two drugs is based on the chemical nature and mechanism of action in lowering the blood glucose levels. There is no comparison between these drugs possible as they belong to different classes. These drugs can be used alone or in combination to control blood glucose levels. I dont think you may be able to understand more than this unless you are in the medical profession. Thank you."
},
{
"id": 71482,
"tgt": "How should COPD disease with face and eyelid swelling be treated further?",
"src": "Patient: My father is having COPD disease & he is on Inhalers use daily (Duolin) Doctors of lala Ram Swarup TV hospital Mehrauli after complete diagnostic told me that surgery cant bring any improvement so he is continuing the drugs. From the last few Days he has lost appetite & sleep. He has lost self confidence & saying that anxeity problems he is facing. Since he was not sleep from the last one week he gave him 0.25 mg alprazolam to induce a temporary sleep after taking which My father has a sleep but his face,eyelds are swallon which seems a allergic reaction. Since he is not in a position to move to hospital could you advise what i should do. Doctor: Hello,If you are saying its allergic condition. You should take him to the doctor as soon as possible. On the other hand, COPD patients should not take sedatives. May result in respiratory depression. Other causes must be ruled out. Right heart failure is a common cause. 2D echo is must as per his blood pressure and other vitals. Diuretics may be needed. Kidney functions should also be tested to rule out other cause of facial puffiness.Hope I have answered your query. Let me know if I can assist you further.Regards, Dr. Vishal More"
},
{
"id": 40394,
"tgt": "What causes extreme pain when suffering from shingles?",
"src": "Patient: Hi I am in extreme pain from a herpes/shingles outbreak. I am in between doctors as my insurance has changed and need a refill of my Vicodin 5/325 Can I get that online? I guess I will go to urgent care and see if they can help me. Doctor: Hi,Welcome to HCM.I understand your concern as shingles pain can be very distressing and can last for a few days to months.Shingles is caused by a virus called varicella zoster which usually remains latent in the neurons and gets activated in persons with low immunity to form rashes on the body supplied by that nerve. There is also associated nerve inflammation which causes this severe neurological pain or neuralgia. You can get prescription for antivirals like acyclovir and pain medication from a GP nearby you. This condition will last for at least 1 month.Wishing you a speedy recovery.Thanks."
},
{
"id": 217785,
"tgt": "What causes pain in the chest when sneezed after injury?",
"src": "Patient: Hi, I was elbowed on my right upper chest about a week ago in a basketball game and my right chest hurts when I sneezed or coughed. The pain is concentrated on that upper region of the chest, just at the point where one can start feeling the rib bones with his fingers. Slight pain can also be felt when rubbing in between the rib bones. The pain has been there until today (12/29/11) and I am wondering whether I should go and get an X-ray to check for cracked rib(s) or don't go get an X-ray as it may be internal bruising? Doctor: Hai, u hav got a blunt trauma to ur chest with out external injury injuriesInjuries to the chest especially the ribs are very painful dueto rich innervation of the nerves.U cam go ahead for a chestxry ap (anteroposterior view) to rule out any bony injuriesTake good painkillers like acelofenac or tramadol added with antacids at least for aweek.These pains take at least 2 to3 weeks to subside.Follow deep breathing exercises with chest physiotherapy to prevent atelactasis.Local application of pain gel forms will be of good help.Take care .good luck"
},
{
"id": 159002,
"tgt": "Have back ache. Left breast is bigger than right and have pain. Breast cancer?",
"src": "Patient: It started about a year back...my work involved travelling with a heavy laptop on my back. This led to severe backache which persists till date. But all of a sudden, I started experiencing pain in my left breast. The pain lasted for a week or so and then I noticed that my left breast had become bigger than my right one..it protrudes more than the right one..i dont know whether it is a sign of breast cancer .. Wanted your opinion Doctor: Dear Ma'am, Sorry to hear about your problem but I am glad that you are seeking medical attention for the same. Breast cancer usually presents as a painful lump in the breast and painful enlargement of the breast is usually not a sign of breast cancer. There might be still be a small chance that this could be something suspicious. To rule that out you should have this problem evaluated by a breast/ general surgeon. Also you need to have an ultrasound of the breast for better evaluation of the lump. Please feel free to ask any other query or share your USG report. Rohan Khandelwal Breast Surgeon rohankhandelwal@gmail.com"
},
{
"id": 102948,
"tgt": "Child. Exercise induced asthma. Help from inhalers while exercising. Safe to take inhalers in between practice?",
"src": "Patient: My 12 year old son has exercise induced asthma. He just finished cross country running season and would take one puff of an inhaler before running a 2 mile race (about a 6 minute mile +/-). It worked wonders. Now he is starting basketball tryouts and the practices are 2 hours long (for tryouts) with a ton of sprinting. Is it safe for him to use the inhaler once before practice and once half way through? Doctor: Namaste Welcome to Healthcare-Magic Greetings of the day Its totally safe to take. Apart from it steam inhalation before sleeping and after Getting up will also be helpful. Take care Regards"
},
{
"id": 95559,
"tgt": "Having body pain and feverish tendency, is it serious ?",
"src": "Patient: I have query regarding colon problem she is having pain at her abdomen , her motion is irregular for long time,, now she is having body pain, and feverish tendency.doctor said she is having colon related problem. is it serious Doctor: hello,looks like she is having an infection of the colon.it can be from bacterial to protozoal.i would advise you to get certain tsts done like stool examination, to find out the organism causing it. body pain can be due to the infection and fever.i would advise you to continue the treatment prescribed to you.in case the symptoms worsen meet the physician immediatelely.i hope i have answered your query.take care"
},
{
"id": 37144,
"tgt": "How can hematoma be cured?",
"src": "Patient: My 75 year old mother had a storm door slide across the top of her foot causing a hematoma. It is about the size of a quarter and very swollen. She is diabetic. So far, she has taken two ibeupfren and has her foot wrapped in ice and is elevated. Should she seek professional medical attention?Thank you. Doctor: Hi,As she is diabetic, risk of having infection at the side is there.Go for one antibiotic medicine course for 3-5 days.Continue applying ice pack for few days.Ok and take care."
},
{
"id": 171153,
"tgt": "What causes blood in stool when suffering from amoebiasis?",
"src": "Patient: my baby has amoebiasis.the doctors prescribed her metronidazole,zinc and hydrites.She does not have fever now but what I am concern about is she has difficulty in defecating though she s suffering from loose bowel movement. When I checked her diaper there is blood in her stool. Is this normal?she s been taking the prescribed drugs for 2 days?Please help. Doctor: Hi, in case of dysentry due to amoebiasis, it may sometimes take 2 weeks to recover fully. I advice you to complete the course of metronidazole and wait for few days. Take care. Regards - Dr Deepak Patel, MD Pediatrics"
},
{
"id": 128452,
"tgt": "What causes right-sided arm and shoulder pain while on Tramadol and Prednisone?",
"src": "Patient: I have been experiencing severe pain since Fri afternoon in my upper R. arm & shoulder. It has now spread across my shoulders. & a bit to my lower back. I was given Tramadol at an emergency room on Sat after seeing a Clinic Doctor who sent me to ER. I saw my Primary care today. She gave me prednisone injection (not in the pain area, but hip) & 10 days more of prednisone FYI I just finished 11 days of prednisone & 10 days of Levaquinn. I have been told that these drugs have been known to cause tears in muscles & ligaments. Recommendation??? I have Rx for CAT SCAN, but cannot get in til next week!!! Doctor: Dear patient you have got radicular pain in right arm cause of which mostly is nerve root compression at the level of cervical spine. Prednisone is given to reduce chemical neuritis and should relieve pain by now. since you are having a pain even after 10 days of prednisone diagnosis needs to be confirmed by MRI of the cervical spine. Please get ot done from radiology centre nearby you. If mri is showing disc prolapse further treatment option is epidural steroid injection. Second option is surgical decompression. You need to consult neurosurgeon with the report."
},
{
"id": 200751,
"tgt": "Suggest treatment for inflammation on testicle",
"src": "Patient: Hi, i have a ball~like inflamation on my left testicle around the epididymis and its very painful, the pain is also felt at the lower left abdomen. I did culture & sensitivity lab tests were they found clouded yellow pus cells.this has been developing for over seven years now & i cant perform well sex~wise. Am worried, could this be cancer? Doctor: Thanks for asking in healthcaremagic forumIN short: It is an infectionExplanation: Epididymo orchitis can cause this. So, please visit a doctor who will prescribe you antibiotics for this. Please complete the course and revisit if not subsided. You will regain your sexual ability once this infection gets cured. Do not worry. Good luck."
},
{
"id": 186086,
"tgt": "Why the tip of my tongue is sore and tender?",
"src": "Patient: The tip of my tongue is sore and tender to the touch, my tongue is also has a burning sensation. Last week my tongue palates had turned white, so I've been swishing hot salt water but not getting any relief. I have had a terrible eye allergy this last week but don't k ow if this could be part of that. Doctor: i read your query.This does not seem to be caused due to the eye allergy. You did not mention your age and other habits. Cut down on smoking if you are a smoker. Dont stress yourself. White patch and burning sensation suggests lichenplanus.Visit your local dentist if it persists.hope this helps."
},
{
"id": 196342,
"tgt": "Suggest remedy for low testosterone levels",
"src": "Patient: Hello I had blood work done a month ago and my AST and ALT were in normal range, however my Testosterone level was around 100. I am a 32 year old male. My Dr. put me on 42.5 mg of Andro Gel and now my AST and ALT is 700 +/-. is there anything I can do to treat low testosterone that will not affect my liver enzimes? Doctor: Hello,I would like to suggest you something natural home remedies.First if you have excessive weight than reduce your weight.Second thing take every day handful of nuts .Third take 2 onions every day.Reduce intake of sweet Have faith better health."
},
{
"id": 207694,
"tgt": "Suggest ways to get better score in acadamics",
"src": "Patient: i'm 16 yrs old .my name is sravya. i'm doing my eleventh class. i want to join armed forces by doing MBBS. I get around 80 -87% in studies. But my friends are getting more than 90%(abt 4-5). i got 92% in 10th class. im not understanding what to do?? i read upto hours and my mom keeps me disturbing in between. i can't avoid her as she is going to live for only few months. PLEASE HELP ME!!! Doctor: HiWelcome to HMI understand your concern.1st thing to remember is never do competition with other.2nd thing make appropriate goal yourself.Only reading can not help.Revision and discussion is also important.Used to discuss with your friends.Make schedule of daily routine and follow it very strictly.Keep enjoy in between reading.Reading for 1 hour straight then take 10 min. break in between you have to revise all.Do take advise from your teachers.Still have a query then fell free to ask.Happy to help you.Best wishes for your future.Thank you."
},
{
"id": 121748,
"tgt": "What causes sensation of water droplets splashing on back of the leg?",
"src": "Patient: What causes the sensation of water droplets splashing on the back of my leg as I am walking? As if I walked in a puddle and water flicked up and touched the lower calf? My leg is absolutely not wet; just a peculiar focal sensation. It lasts a couple of minutes and then stops. Duration: 2 weeks. Doctor: Hello, Your symptoms could be related to a pinched nerve (probably caused by a bulging disk in the lumbar vertebral column). For this reason, I would recommend consulting with a neurologist for a physical exam, a lumbar spine CT scan or MRI study. Nerve conduction studies may be needed too. You should discuss with your doctor on the above tests. Hope I have answered your query. Let me know if I can assist you further. Take care Regards, Dr Ilir Sharka, Cardiologist"
},
{
"id": 30153,
"tgt": "What causes fever with rapid weight loss?",
"src": "Patient: My wife(51 YEARS OLD) has been having temperature for now almost six weeks. Since last two weeks, the temperature has been ranging between 99 to 101 deg F. During hospitalization,following tests were carried out and they were all negative. OX 19,OX2,OX K,IGG,IGM, S.TYPHI(O,H),S.PARATYPHI(AH),S.PARATYPHIA(BH),ULTRSOUND OF ABDOMEN AND PELVIS,URINE CULTURE(NO GROWTH),CHICKUN GUNIA,RAPID MALERIA,CARDIAC COLOUR DOPLER,CT SCAN OF BRAIN,CHEST AND ABDOMEN,PAUL BUNNEL,PROCALCITONIN,CMV Igm Ab, PARANASAL SINUSES, IGG,IGM, ESR, P.C.R ETC. Following tests have indicated possibility of tuberculosis, one out of three urine samples has shown ACID FAST BACILLI, TUBERCULIN TEST CHECKED AFTER ONE WEEK SHOWS SIZE OF 15 MM, WHILE THE TEST REPORT ON THE DATE OF TESTING HAS REPORTED 7MM. IVP TEST WHICH STATES,\"THERE ARE CHANGES OF EARLY GRANULOMATOUS DISEASE IN RIGHT KIDNEY LIKE TUBERCULOSIS. Based upon the above three symptoms, she was prescribed AKT 4 drugs. This drug resulted into constipation, loss of appetite, which was prooved by increase SGPT to 285. AKT 4 was subsequently stopped after two weeks of use and SGPT came back to normal. After stopping AKT 4, she was being given following drugs, combutol (1200mg) Levoflox (750 mg) Since last one week, one more drug (IV INJECTION KONAMYCIN 0.75gm ) was given and last two days, one more drug R-CIN 450mg was given. Inspite of all these drugs, she has been having temperature, although the appetite hs increased. After starting the IV injection and R-CIN 450 mg capsule, she has started vomiting and so we have stopped all the medicines since yesterday and approached a Hemeopathic doctor, who has advised to stopp all the elopatic drugs for three days. I seek your expert opinion on the subject as my wife has lost @6 kg during this period and she is still having temperature to the extent of 101 max. Doctor: hi, I m glade to answer you.I think that the tuberculosis treatment have been successfull, and need time to eradicate tuberculosis, but a gastritis is widely expected and need Gastroscopy with biopsy.I would add to last tuberculosis treatment a Proton pomp inhibitor like pantoprazole 40 in the same time a Gastroscopy with biopsy is indicated.may my answer will helpfull"
},
{
"id": 217422,
"tgt": "Having noticed pain in heel due to high heel shoes",
"src": "Patient: today while walking to my car I rolled my foot/ankle outward, I was wearing Dansko shoes which have about a 2 inch heel, immediatly my foot hurt not my ankle but my foot, lateral side about half way down the foot. It is still painful even when not walking on it. Doctor: thank you for posting your query on hcm,your pain is due to ligamental sprain from slip of foot.sometime mild imflammation may occure,but don't worry. you can take simple analgesic like ibuprofen or if pain is severe take like combination of aceclofenac+paracetamol+serratiopeptidase. it will relieve your pain and other symptoms. if symptoms appears more severe once got examined by your family doctor.hope this answer will be helpful to you.for more queries plz don't hassitate to ask.get well soon"
},
{
"id": 100553,
"tgt": "Could itching on body be due to an allergic reaction?",
"src": "Patient: Hi, I'm allergic to alot of abx,PCN's included. My daughter was recently given Amoxicillin 400/5ml. I accidently got a little bit on my finger 2 days ago and absently licked it off without thinking about it. This morning I woke up and realized i was having an allergic reaction very similar to how I react to PCN. I took a Benadryl and it seemed to help, but my itching has returned. Is it possible for me to be reacting to my daughter's medicine or could I be reacting to the new Antacid or Prozac that I recently started? Doctor: Hello,Thank you for asking at HCM.I went through your history and would like to make suggestions for you as follows:1. As you are allergic to antibiotics, I would first think of amoxicillin. I would suggest you to take an antihistamine like Benadryl (that you have already taken) or cetirizine or hydroxyzine for 3 days.2. Antacids like Prozac can rarely cause allergies but it is not very common.Hope above suggestions will be helpful to you.Should you have any further query, please feel free to ask at HCM.Wish you the best of the health and a very good recovery.Thank you & Regards."
},
{
"id": 96508,
"tgt": "Gall bladder stone in ultrasound examination",
"src": "Patient: I am 42 years old female i have 2 kids and they are healthy. my cholesterol levels are normal and my height and weight is also normal. in my corporate health check up i used to get sludge in gall bladder in ultrasound examination of stomach from last 3 years. this year it came as stone in gall bladder ? why stone there what shall i do ? Doctor: The sludge in the gall bladder lead to stone formation. if the stones are small ask your doc for medicines to dissolve it if possible, if not and they are more in number and big, then get it operated."
},
{
"id": 147877,
"tgt": "How to get rid of pain in the cervical area of my neck?",
"src": "Patient: I have pain in the cervical area of my neck. I have had exrays.utra sound and ctscan. I have been diagnosed with mild arthritis and degenerative discs c2.c3.c4 c5. I have been through physical therapy, I have had those shots )cortizone and now am seeing an ortho doctor who has given me shots of marcaine and lidocaine. Am still hurting. What can I do. . Doctor: Hello & Welcome to HCM,I had gone through the case and found that these are for temporary relief.You must go for proper cervical exercise with Vitamin D-3 once a week with milk and avoid bad life style.Do not use pillow or use cervical pillow for sleep.Having more pain then can take painkiller.Hope my advise will be effective for you. I will be pleased for further query.Thanks,Dr.Soni VermaHomoeopathic Physician and Fitness Trainer"
},
{
"id": 70342,
"tgt": "Noticed a hard lump at the back of the neck below the skull",
"src": "Patient: I have a hard lump on the back of my neck at the base of my skull. I have had this lump for a couple of weeks now. It is hard and immovable and it shrinks and grows throughout the day. A couple of different times, it has hurt. This pain usually occurs in the morning and then goes away. It is bigger when it is painful and then it shrinks back down again. I had a stiff neck for about 2 weeks before the lump appeared. What could this be? Doctor: Hi.By mere history this looks to be hemangioma by your history of reducing and increasing nature of the swelling... Get this checked by color doppler and Consult a Surgeon to confirm or rule out any sinister problem ."
},
{
"id": 127331,
"tgt": "Can Aleve be taken daily for arthritis?",
"src": "Patient: Yes, I don t know if I have regular osteoarthritis or rheumatic disease. I do have Hashimoto s and my brother has psoriatic arthritis. My question: is it OK to take Aleve everyday? for how long? Doctor: Hello and Welcome to \u2018Ask A Doctor\u2019 service. I have reviewed your query and here is my advice. Aleve is a brand name of Naproxen and it is belongs to the family of analgesics known as NSAIDs. It is relatively safe when compared to common analgesics like Diclofenac and Ibuprofen. Prolonged use of more than six months is not recommended due to gastritis and related side effects. Prolonged use can also cause cardiovascular side effects like angina and stroke. You can consult a rheumatologist for expert opinion. Wishing you good health. Thanks."
},
{
"id": 102057,
"tgt": "What causes severe pain, redness and swelling after flu vaccine and pneumonia shot?",
"src": "Patient: Hi-I am a 66 yr old female who received both a flu vaccine and pneumonia shot on Friday, the 10t of Jan. I have been experiencing sever pain, redness, swelling not only along the area of the shot, but up my arm, toward my neck and back. I have been putting cold compresses and taking Tylenol but today is Sunday ad there is no markable change in my symptoms-IT HURTS TO TOUCH ANY OF THESE AREAS AND HURTS TO MOVE MY ARM. iS THIS A NORMAL REACTION TO EITHER OF THESE SHOTS? Joan Dessoye Doctor: yes as they are live attanuated vaccine the introduction of viruese in body causes local reactions at the site and the body develops antibodies which fight against them to destroy the viruses and give immunity to body to prevent the attackssometimes the reaction is less sometime more you can use anti allergics tabs along with tylenoland can apply local antibiotic antiallergic combination ointment to relieve this"
},
{
"id": 16523,
"tgt": "Are a heart valve issue and atrial fibrillation causes for exhaustion?",
"src": "Patient: For the past several months whenever I am exhaustedly tired I lose my voice. It gets so weak and I don\u2019t hardly have the energy to struggle to get my words out. No pain, however? I do have a heart valve issue and AFib. Could this be related? Thank you. Doctor: Hello There After going through your medical query I understand your concern and I would like to tell you that Afib can cause stroke or ischemia in the body wherever the clot will stuck if you are not taking and blood thinners. So it\u2019s recommended for you to consult a cardiologist for the same and get an evaluation done.Hopefully this information will guide you properly Kind Regards Dr Bhanu Partap"
},
{
"id": 203582,
"tgt": "What causes tingling sensation on penis tip post urinating and dribbling afterward after unprotected sex?",
"src": "Patient: I recently had unprotected sex just over a week ago. about 4 days ago, i started having a tingling sensation at the tip of my penis after urinating and dribbling afterward. I have a wife and family and don't want to infect them. What could this be I need some advice ? Doctor: DearWe understand your concernsI went through your details. I suggest you not to worry much. Please keep away from unprotected sex with even known people except your wife. That is safest. Tingling sensation is not a proper symptom to address. You need to worry if there pain, itching or burning. Dribbling could be caused in normal circumstances such as mild infections or skin irritation. Even though I don't see any STDs, In any case, it is always better to consult a skin VD specialist for further diagnosis and advice.Hope this answers your query. Available for further clarifications.Good luck."
},
{
"id": 201066,
"tgt": "What causes burning penis post mastrubation?",
"src": "Patient: I am 22 male and have a problem from like 4 years now. well sometimes after mastrubation I feel burning sensation in my penis and then this sudden urge to pee, but the pee won t cone out, I will have to stay in bath for 10 to 20 mins. the process is long and quite painfully and annoying. I have sex regularly but it only happens after mastrubation. don t know who to ask for help, so asking here.I need answers, its kinda scary now. Doctor: Thanks for asking in healthcaremagic forumIn short: There is nothing abnormal with you.Explanation: When the ejaculation is prolonged (the duration), burning sensation can occur after ejaculation and feeling to urinate is also a normal feeling after this. But you cannot urinate suddenly after this because your passage to urinary bladder is cut off whenever you are excited to make way for semen.(Semen and urine have same passage after some distance, so one closes to make way for another and vice versa). Wait for some time as you did and urinate to prevent any Urinary tract infection. Enjoy, you do not have problem."
},
{
"id": 138316,
"tgt": "Suggest treatment for swollen and painful collar bone",
"src": "Patient: Hi my name is Mark I am a 34 year old male I have a swelling just above my collar bone on the right side, it is soft and squidgy, it comes up for a couple of weeks and then goes down, I also have a pain in my shoulder that I can only describe as feeling like it is going to come out of its socket it is very painful, my arm has also gone a grey/blue colour for a few minutes a couple of times, I have been to the hospital and they could not find anything, I have been suffering with this for 2 years and it s getting worse. Any advice would be great thanks Doctor: Dear Sir/MadamI have gone through your query and read your symptoms.In my opinion, you could be having vascular swelling and you need urgent diagnosis of the condition, as this could be life threatening. so please seek help from a vascular surgeon.I hope that answers your query. If you want any more clarification, contact me back."
},
{
"id": 91282,
"tgt": "Why does my friend who has HBP suffer from abdominal pain?",
"src": "Patient: I have a friend who has high blood pressure and is on tablets for this but he keeps complaining of stomach ache and thinks it is to do with the tablets so has not taken them for 2 days but he looks quite ill and stands bent over with the pain in his stomach. Do you think it is his blood pressure getting worse or a side effects of the tablets? Doctor: Hi.Thanks for your query.This is neither due to high blood pressure nor due to the tablets.The pain in abdomen which causes the person to be pale and ill; and makes one stand bent over is a serious pain. Unless proved otherwise this indicates that there is an inflammatory process going on in the abdomen like appendicitis or so.He needs to see a Surgeon immediately , to be examined by the Specialist to see there is tenderness and rebound tenderness. undergo immediate investigations as 2 days have already gone with the pain.Needs Standing X-ray abdomen to rule out gas under diaphragm, ultrasonography and may be a CT Scan to see what is wrong.He may need an urgent exploratory abdominal surgery."
},
{
"id": 219861,
"tgt": "Is bleeding with clots normal in 3rd trimester?",
"src": "Patient: I had bleeding and passed a small clot and am in my 3rd Trimester. I went to labor and delivery they monitored me for 4 hours and the bleeding stopped and the baby had a very active heart beat. I was not dialated and was not having contractions. Is any of this just normal? Also, all that was done was that I was put on pelvic rest, does this seem right? Doctor: Hi there,,Bleeding in the 3rd trimester is due to one of two causes Abruption, which is associated with high blood pressure and pain abdomen, trauma to abdomen, or it is due to placenta previa where the placenta is attached to the lower part of the womb and it causes painless vaginal bleeding. An ultrasound to identify the exact location of the placenta is essential.A physical examination to monitor your blood pressure, urine albumin and fetal heart rate pattern is also necessary.Rest is advisable, as repeat bleeding may make delivery a necessity. Blood cross matched to your blood group should be made available and corticosteroid injection to help fetal lung maturity needs to be given.Hope this helps.Regards."
},
{
"id": 758,
"tgt": "Why am I not getting pregnant?",
"src": "Patient: hi...doctor...im...married for 2 years.....trying to conceive..but not able..last may 2009 i got pregnant but at the same time i was diagnoused with enteric fever.due to heavy medication it got aborted..now so months passed but still not conceiving...doc has started on with gynovit & folic tab..do pls help me.. Doctor: Hi,I understand your concerns.Following is my reply:1)\u00a0\u00a0\u00a0\u00a0\u00a0PLease get your husband's semen analysis doneYou can contact me anytime directly to ask question by pasting following link in your browser:http://bit.ly/askdrsoumya"
},
{
"id": 78543,
"tgt": "What causes a buzzing feeling in the groin while breathing?",
"src": "Patient: for about a week now I have had a buzzing feeling in my groin when i breath in and out. I thought it might be my cell phone, then maybe the sound from the TV so i turned everything off or put away in another room, and it still does it when i breath in and out. Doctor: Thanks for your question on Health Care Magic. I can understand your situation and problem. By your history and description, possibility of inguinal hernia is more. Inguinal hernia is mostly containing bowel loops. These loops are hollow and balloon like structure. During breathing, intra abdominal pressure increases and this can cause laxity in abdominal muscles. This laxity can precipitate hernia. And bowel loops when protruding through defect can cause buzzing feeling. So get done ultrasound examination of abdomen to rule out hernia. Quit smoking if you are smoker because this is the most common cause for laxity of abdominal muscles. Hope I have solved your query. I will be happy to help you further. Wish you good health. Thanks."
},
{
"id": 43733,
"tgt": "Sperm analysis done. What does the finding indicate? Any solution?",
"src": "Patient: My report details: sperm count 180ml / sperm concent 90 / morphology NIL / Morphology per 100 sperms: head defects - 100%,midpiece defects-40%, tail defects-80% / vitality 15% / motility within 1hour: classses a-NIL and classes a & b - 10%. Pus cells: 8-10/hpf Rbc: 4-6 /hpf Epithelial cells: 1-2 /hpf Comments: Gross teratoasthenozoospermia. Note: Small agglutinates and aggregates(+). Will appreciate if anyone could help me with my above report and solutions to my problem. Many thanks Doctor: Hello, Thanks for the query, Teratospermia is a condition characterized by the presence of sperm with abnormal morphology that affects fertility in males. Asthenozoospermia is the medical term for reduced sperm motility. Teratoasthenospermia is the combination of both. Tratement includes use of antiestrogens, Intracytoplasmic sperm injection (ICSI), injecting sperm directly into the egg. Even with severe teratozoospermia, microscopy can still detect the few sperm cells that have a \"normal\" morphology, allowing for optimal success rate. So good luck to you, regards, Dr Nilofer"
},
{
"id": 1632,
"tgt": "Suggest remedy for getting pregnant",
"src": "Patient: hello,i am 25 yrs old and a mother of 3 yr old daughter now me and my husband are planning for 2nd baby but i am diagonised by pcod both overies doctor have putted me on chromiNAC-A and letroz 2.5 today is my 5 dat of period i m taking medicines what should i do n when can i conceive i want to get pregnant is it possible i a overweight i had cesarean first but i dont have any problem like sugar high, b.p thyroid etc Doctor: Hi, I think you can take the letroz which helps in growth of follicles and track your follicles growth by repeated ultrasound. When follicles reach a size more than 17 to 18 mm, take injection for rupturing the follicles. be in contact with your husband for 2 to 3 days after injection. Take progesterone for next 2 weeks. Do a urine pregnancy test at home after that. You can try like that for 3 to 6 months. Hope I have answered your question. Regards Dr khushboo"
},
{
"id": 209030,
"tgt": "What is the treatment to get rid of stammering?",
"src": "Patient: Hi sir,my age : 28, height : 164 cm, weight : 68 kgs, singleam working as engineer in one cement company in andhra pradesh, india, am suffering from stammering from my childhood, am an engineer, so i supposed to talk to our superiors directly or through mobile regarding work, at that i can't able to converse properly. not only these aspects am suffering at all situations while working or in my daily life. so please give me some suggestions regarding this Doctor: HiThanks for using healthcare magicIn your case, best treatment is speech therapy. You should consult a therapist for proper therapy. At age of 28, it is difficult to get complete recovery, but with therapy you would able to control it. You should also try to follow some relaxation technique that would help you to stay clam in tense situation. In case, you need further help, you can ask.Thanks"
},
{
"id": 132004,
"tgt": "What is the cause and solution for a distinctive loud squishing sound on knee?",
"src": "Patient: I have a very distinctive loud squishing sound coming from one of my knees. I have just started exercising again and now I have some discomfort around the front lower part of the same knee joint. Do I have tendon issues? - I have been taking anti inflammatory tablets and motoring on. Is this wise and could I be doing serious damage Doctor: Hi You have loud squishing sound from one of your knees and have discomfort in the lower part of the same knee . In my opinion this sound is not normal and you might be suffering from a condition called Discoid Meniscus. I suggest that you get an ultrasound / MRI of the knee for complete diagnosis."
},
{
"id": 21076,
"tgt": "Is Loprin the only medication to be taken at present?",
"src": "Patient: Age:75. HBP diagnozed 190-90 yestrday. No blood pressure taken since the age of 34 when it was as high as now. Cosntant vertigo sitting on computer 12 hours a day, doing research on Magement Sciences; spondiltis, black spots on left eye and weak eye sight (left eye) diagnosed; vertigo persists despite hbp treatment (one doze of medicine under tongue - lasting 24 hrs.); Loprin 75mg. prescirbed, generally beneficial but vertigo pesists. Going to change the power glasses today. Perhaps this is the correct treatment. Generally heathy, regular half hour exerise since 20, still cotinues, food mainly vegetables and fruit, no meat, plenty of fresh water. Would Loprin be enough? Other medication stopped until results are determined about spondiltis. Your opinion? Am going to change my spectacles today. Would bringing bph be beneficial? Mahmud. Magagement Consultant to Cantt,Hospital, Rawalpindi. Doctor: Hi,Welcome.This is Dr Sameer, cardiologist.Loprin is a NSAID which is a anti inflammatory medication for joint pain & joint inflammation. It has nothing to do with bp & vertigo.You have a pretty high bp & that needs treatment. I would recommend you to start with 5mg of Amlodipine once daily & get your Kidney Function tests done so that medication could be changed to a better one. For vertigo, i will recommend starting a drug called Vertin 16mg thrice daily & get your lenses checked.Come back if you need more help.Thanks"
},
{
"id": 210878,
"tgt": "How to treat male/female hormonal imbalance with agitation and tremor in a 59 years old male while on onezol, fluox, simvastatin, clonazepan, propranolol, bendrofluazide, aripiprazole and seroque?",
"src": "Patient: Hello, My family is becoming more concerned with my brother who is having a number of drugs to treat his present problem. He lives alone, has always been a bachelor and is now 59. Earlier 2013 he became increasingly upset about his neighbours who he overheard, were plotting to get him. He has had periods like this and given the vicinity where he lives, some goings-on are very plausible. However, this time was more extreme and he became very perturbed and called the police (not the first time). He has been on hormonal drugs for a long time to balance his male/female imbalance and the doctors usually monitor this if my brother has concerns. He was admitted to a psychiatric ward for tests then put on this concoction of drugs to help him - please forgive my spelling: onezol, fluox, simvastatin, clonazepan, propranolol, bendrofluazide, aripiprazole and seroquel. He has always had a tremor but now can barely carry things, has extreme difficulty using his eftpos card, cannot lift a cup to his lips etc.... He is suffering from night sweats, waking in a pool of wetness, he is unable to remember what day or time it is and this in itself is leading to confusion, missing appointments and the inability to take his medication at the appropriate times. What would you recommend? Doctor: HiThanks for using healthcare magicI think, he has on overdose of antipsychotic that why he has excessive tremor and sedation.Due to sedation, he has drooling of saliva at night. Seroquel leads to excessive sedation, either dose of seroquel should be less or clonazepam can be stopped. For tremor, dose of propranolol should be increased or get him investigated for drug induced parkinson.Thanks"
},
{
"id": 186661,
"tgt": "What causes pain and swelling o jaws?",
"src": "Patient: hi, I fill my teeth in MAy of this year, subsequently the jaw become swolen and painful. after inspection i realised that there was a boil at the root of the teeth. the dentist said it was an absess and the filled tooth should be removed. the basess is not painful. should i be worried Doctor: thanks for your query, if the tooth is root canal treated check whether it is properly done and check the amount of bone surrounding the tooth if it is nt sufficient then go for extraction of the tooth...nothing to worry..its a simple procedure..i hope my answer will help you..take care.."
},
{
"id": 74045,
"tgt": "What causes stabbing pain in chest with nausea and fatigue?",
"src": "Patient: Hi, I'm a 45 old male, quite healthy and athletic, playing competitive tennis three times a week and running, doing sit-ups and push-ups almost every night for 30 times each, non-smoking. About 7 month ago, I felt sudden stabbing pain in my left chest area, followed by a feeling of warmness rediating and releasing from where the stabbing pain originates, followed by nausea and fatigue. This happened three times in a period of 40 minutes. I had ECG and Holter monitor on the next day and everything came back fine. Since then, there has no more stabbing pain, but it develops into a pattern of sudden onset of severe chest, back and sholder pain with slight nausea, very deep and dull, worse off when I stretch or tilt back or trun my head left. Had my blood test, stress test, ECGs a few month ago and Dr don't think I have heart problem. One thing I want to mention is that I did have tremendous pressure in life and some anxiety issues when this first happened, could this pressure and anxiety cause my problem? I have been trying meditation, relaxing techniques since and i do think I am really relaxed now, sleep well and enjoy life more. But this sudden onset of deep chest and back pain (at the bottom of my left shoulder) happens almost once a day, lasting about 30 minutes each time with slight nuasea. I now know this deep chest pain will not kill me but still get worried and annoyed when it comes on. Can you please advise as to what it is and how I can get rid of this completely. Thanks heaps, Dave Doctor: Thanks for your question on Healthcare Magic. I can understand your concern. No need to worry for heart diseases for your symptoms. Since you are having stress and anxiety, your chest pain is mostly due to these only. Counselling done by psychotherapist is best to treat stress and anxiety. So consult good psychotherapist. Try to identify stressor in your life and start working on its solution. Anxiolytic drugs also help to combat anxiety. Don't worry, you will be alright. Avoid stress and tension, be relax and calm. Hope I have solved your query. I will be happy to help you further. Wish you good health. Thanks."
},
{
"id": 25726,
"tgt": "What medication is suggested for dizziness and BP of 116/62?",
"src": "Patient: My Dad has high blood pressure 4 stents in his arteries around the heart including one in the y, he has cardiovascular disease and PTSD and diabetes so he takes a variety of meds. Anyway my Mom just witnessed him get dizzy and almost fall down. He says this happens sometimes but she has never seen it happen. His Bp was 116/62 when he sat back down and my mom made him check it. Isn't that low for him? Doctor: Hello!Welcome and thank you for asking on HCM!I passed carefully through your history and would explain that his blood pressure values are not too low to cause fainting or dizziness. Orthostatic hypotension could be the cause of his symptomatology. I recommend measuring his blood pressure during sitting and standing up position. If there is a decrease in his blood pressure more than 20mmHg during standing up position, compared to sitting position, this could indicate orthostatic hypotension. A Head up Tilt test is needed to rule in/out this disorder. I would also recommend performing some other tests to exclude other possible causes related to his heart: - a resting ECG and cardiac ultrasound- some blood lab tests (complete blood count, kidney and liver function tests, blood electrolytes, fasting glucose and HbA1C). If orthostatic hypotension is confirmed some modifications to his actual therapy may be needed. Hope you will find this answer helpful!Greetings!Dr. Iliri"
},
{
"id": 17717,
"tgt": "What causes rapid heart rate and dry mouth?",
"src": "Patient: Hi I accidentally took 3 Alavert in the span of 18hrs instead of 1 every 24hrs. I have the fast heart rate and dry mouth, but I ve been drinking lots of fluids to flush it out and I puked. Should I still go to the hospital? My fast heart rate has slightly gone down. Doctor: Hi, Alavert (Loratadine) overdosage is causing tachycardia, dry mouth due to its anti-cholinergic (antimuscarinic) effects. Its biological half-life is on average 8 hours (range 3 to 20 hours); hence its effect will pass off. Since about 40% of the drug is excreted into the urine, adequate urination with plenty of water by mouth is all that is needed. Visit to a doctor is always is advised in case of overdosage. Hope I have answered your query. Let me know if I can assist you further. Regards, Dr. Tushar Kanti Biswas, Internal Medicine Specialist"
},
{
"id": 188986,
"tgt": "Prescribed Ibudone for pain due to wisdom teeth. Need stronger drug?",
"src": "Patient: Hi im going to have one of my wisdom teeth cut out. Its on my bottom right and my dentist tells me that she cant do my left side of my wisdom teeth because she cant get to that side good but she can do my right. she is sending me to a surgon to have those on my left done but next week she is suspose to do my bottom right because its causing me alot of pain. i was wondering if it was okay that i let her do it with her not being able to get to my left side i scared she may just want my money. i cant afford to have my teeth more messed up than they are. also she said i will be awake but i would be on gas and that worries me. she had giving me Ibudone 5-200mg generic for reprexain and it barley helps me now with the pain and i was wondering with one tooth being done will she give me something stronger than that or will i even need anything stonger? Doctor: Hello,First of all let me suggest you that ,to get a proper treatment you should trust on your dentist .Or else you can go for second opinion too.Usually antibiotics has to be taken in complete course ,prior to extraction of wisdom teeth which is inflamed.Avoid anxiety and stress as it can worsen the condition.Gargle using warm saline and betadine.Maintain oral hygiene well.Please have an open discussion with your dentist regarding this.Hope this helps."
},
{
"id": 208568,
"tgt": "Is sizodon plus safe for psychiatric patients?",
"src": "Patient: Hello, patient age is 36yrs/female,height 5 ft 3inches, weight is 65kgs, her behaviour is sometimes very harsh but nothing is in her mindShe talks sometimes irrrevelantlyShe feels tired whole dayShe sleeps a lot-she takes pills sizodone pluswant to know more about the drug sizodone plus. Is it specifically given to schizophrenia/depression patient or can it be given to other patient with different psychatric problems? Can it be discontinued after a year or so....What if this medicine is causing increase in sleep...how can the patient lead a normal lifeDoc. am waiting for your reply.... Doctor: I appreciate your concern and asking on health care magic. In response to queries I would say:1. Sizodone plus is resperidone and it is an antipsychotic. Its use varies according to the disease. It blocks that dopamine pathways in the brain that are involved in various mental processes including psychosis, mood and anger. In high doses it acts as antipsychotic and in low doses it is used as mood enhancing and reducing anger and irritability. 2. It can be stopped depending upon the resolution of symptoms and the course of treatment. Its dose can be reduced if causing increased sleep. I hope you find it helpful"
},
{
"id": 37869,
"tgt": "Is HSV, related with swollen bumps on labia ?",
"src": "Patient: I have a tiny bump on the edge of my labia minora that is not really painful but slightly uncomfortable. I had this before, squeezed it and a clear fluid came out. It then proceeded to spread along the edge of my labia and the bumps swelled up and were more painful. I do have HSV2 but none of my other outbreaks were anything like this so i dont know if its associated. If you think its HSV, it is safe to take acyclovir while pregnant? Doctor: Hello,Thank you for your contact to healthcare magic.I understand your health concern, if I am your doctor I suggest you that your complaint is related to HSV type 2, there is no study which shows teratogenicity of acyclovir, but it is category B drug you can take it. I will be happy to answer all your future concern. Thank you,Dr Arun TankInfectious disease specialist.Wish you a best health at health care magic."
},
{
"id": 206033,
"tgt": "Is Lexipro effective in treating depression?",
"src": "Patient: I have been taking Lexipro 10 for approximately two months and still didn t feel like a happier person so I increased it lately to 20 per my doctor in the last week and a half and am experiences very sad emotions, like thinking of things and just crying and sobbing, almost is a hypersensitive condition. Yes, I am going thru some things which are hard to handle at this time but to cry at a drop of a hat....I am not liking it. Also, should I back off and go back to 10 and should I take this in the AM or PM or doesn t it matter? Doctor: Hello thanks for asking from HCMLexapro is escitalopram and this is a SSRI drug. You have been on this drug since last 8 weeks and then the doses has been increased. Even then you have symptoms like crying spells, depressed mood. If a drug fails to show response in about 2 months then it is called as non-response. Usually I choose to change the drug in such situation. Other drugs like Mirtazepine can prove useful in such situation. The drug help in recovery in resistant depression. Augmentation with drugs like Lithium, Lamotrigine or some other mood stabiliser can help in early recovery.Cognitive therapy can also help in early recovery. The drug is usually prescribed in morning hours because it can cause insomnia. Consult your psychiatrist for treatment and prescriptions.Thanks, hope this helps you."
},
{
"id": 92732,
"tgt": "Abdominal pain. Having colitis, endometriosis. Taking prednisone, rash, hives. Solution?",
"src": "Patient: Hello, I was diagnosed with Colitis 3 years ago, but recently I am having abodomenal pain. The doctors can't find what is wrong. My symptoms don't match with my diagnosis. 3 weeks ago I could not even drink water I got severe pain. Right now I am on Prednisone for 2 months since nothing else worked. After 4th day of taking this medication, I developed rash and hives all over my body. I also have Endometriosis but no one knows what is the cause my problem Doctor: NamastheWelcome to Healthcare-MagicGreetings of the dayI would advise you to get back to was with all your relevant investigations and work-up done so that we can assist you further. One possibility of Inflammatory bowel disease also has to be considered which reuires colonoscopy and biopsy.Has Colonoscopy been done?Kindly get back to meRegards"
},
{
"id": 23209,
"tgt": "Suggest remedy for hypertension",
"src": "Patient: Dear Doctor, I'm from Nepal, last 4 month; I'm suffering from hypertension and taking Amlobet 5 mg and Lotan H 50 mg as per suggestion by doctor. Continue I'm checking my blood pressure day by day as well as same doctor now (from 3 month ago) my blood pressure is 120/80 regular. Recently 1 week ago (30 May, 2011) I had checked my whole body like: Diabetes, Liver Function Test (SGPT), Blood Sugar, Echo, Kidney Function Test (Blood Urea, Serum Creatinine, Serum Uric Acid, Urine R/M/E) tests these all are normal. My age is 38 years, and my weight is 68 Kg. I'm regularly walking 4 km at morning from 2 years ago. Also I'm very careful to take my regular meals. Now I need to stop medicine one by one (first Lotan H 50 mg and after some weeks Amlobet 5 mg) is this my right decision? pls suggestion. Deepak Nepal Doctor: Hi, Bp medication has to be stopped gradually by reducing the dose. They can not be stopped all of sudden. Don't stop it yourself. Keep a record of bp & let the doctor decide about the medications.Dr Sameer Maheshwari"
},
{
"id": 40774,
"tgt": "What does the following semen analysis report suggest?",
"src": "Patient: Hi Sir. I married since three years and my wife did not get pregnant cuz of me. Please sir give me suggestions. I will be very thankful to you. My semen result is ss follows. Volume = 4ml. Ph=7.5 Pus cells = 10-12/hpf RBCs = nill Actively motile= 5% Sluggish motile = 5% Non motile = 90% Normal Forms = 70% Abnormal forms = 30% Total sperm count = 45 million/ml Doctor: Hi I think you have asthenospermia and semen infection also. Get a semen culture and sensitivity test done. You may need some antibiotic according to the culture report. You can take some antioxidant and vitamin supplements for 3 months. Repeat a semen analysis after one month with 3 to 5 days of abstinence. If report is same, you may need IUI for conceiving. Talk to your doctor regarding this. Hope it helps."
},
{
"id": 83628,
"tgt": "Is cramps a side effect of Deviry intake?",
"src": "Patient: Hi as advised by my doctor I took Deviry 10mg twice a day from 15h day of my cycle to the 25th day. Today is my 27th day and since yesterday evening I ve been experiencing sever cramps, is it ok?? If so what can I take to get relief from pain. I took cyclospams but it did not work. Please help Doctor: Hi,Yes, this tablet may cause cramps in few patients. If it is severe you may take tramadol to control it. If there is no improvement, please consult with your treating doctor he will examine and treat you with alternative medication.Take care. Hope I have answered your question. Let me know if I can assist you further. Regards, Dr. Penchila Prasad Kandikattu, Internal Medicine Specialist"
},
{
"id": 144942,
"tgt": "What does ventral csf space being effaced 14 15 2mm effacement mean?",
"src": "Patient: I had a CT scan done, and several years ago i also had a MRI done, i know that my l-5 s-1 are pushing on my theacial sac and now it is showing a ventral csf space is effaced l4 l5 2mm effacement, i m not sure exactly what that means for me and my back. The disk hernieation went from 2mm to 6mm Doctor: Hello dear, Your Mri report means that you have lumbar disc prolapse which is pushing over thecal covering of csf space. But still no nerve root compression or canal stenosis are reassuring findings. you may visit your neurologist or neurosurgeon for proper examination and evaluation. hope you found the answer helpful Regards DrNeeraj Kumar Neurologist"
},
{
"id": 181703,
"tgt": "What causes soreness post molar teeth replacement?",
"src": "Patient: Hi, my right molar broke a few weeks ago and the dentist made me a new one and put it in for me. Now i cant eat on that side as it feel extremely sore when putting pressure on it. Also on my gum line directly under my molar there is a small painful lump which make my tooth and gum throb. is this an absceds? If so please what can i take Im also 22 weeks pregnant...please help. Very painful Doctor: Thanks for your query. I have gone through your query.The soreness following replacement of the missing tooth can be because of the any sharp margins below the prosthesis or it can be because of the left out root piece infection. Nothing to be panic, consult a oral physician and get it evaluated. You are not suppose to get exposed to radiation. You need to wait till you deliver. Mean while you can take a course of antibiotics like amoxycillin(if you are not allergic) and painkillers like paracetamol. Do saline gargling. I hope i have answered your query. Take care"
},
{
"id": 44649,
"tgt": "Does laser hair removal in the bikini region cause infertility in women ?",
"src": "Patient: im a 27 years old female , I started lazer therapy for hair removal in pikini area and i finished 2 sessions, then the same dermatologist told me that there are new researches talking about risk of infertility and it is not recommended in female in reproductive age. is this true? i want to continue the sessions but i am afraid now.plz i want an answer. thanks Doctor: Hello, Laser hair reduction is safe. I have not come across any case with infertility caused by laser hair reduction. even if there are a few reports, it is too early to to be worried about this problem. So go ahead with your sessions. Thanks"
},
{
"id": 111425,
"tgt": "How to treat the pain in the back caused by an old injury?",
"src": "Patient: Sir one of my friend got a back injury several years ago but it eventually gets healed but after those few years now he experiencing some pain at that area when he stand for a long time or do some rigorous work one at a time..so can you tell me what is the reason behind it? Doctor: Hi, welcome to our site. I am Dr Saumya Mittal, MD.Read your query. That is a very significant question and i appreciate your problem. I will try my best to answer your queryWhenever there is a back injury, the symptoms may heal fast mostly, if superficial. And a person feels relieved that he/she has healed because the symptoms are gone. But many times a small defect may be left behind, like say compression of the disc that lies between the bones. Now the person feels everything is fine and resumes the normal activities. However the pain increases slowly as the compression that the person is carrying slowly gets worse.I guess something similar may be happening in your case as well.I hope this helps you. Inform the reports mentioned above/if any other so I can be of help further. I have given you the answer to the maximum considering the information provided. The results of the tests could further enhance my answer to you.Please do understand that some details could be extracted from a detailed history and examination.Please feel free to ask another query. I would be glad to help you. Looking forward to your return query with the details asked so that I can help you further. (If the answer has helped you, please indicate this)Best of luck."
},
{
"id": 180089,
"tgt": "Suggest diet plan to be followed to cure difficulty passing stools",
"src": "Patient: Hi, My name is Deepthi. My son who is of 3 yrs age feels difficult to pass the stool. We have tried all the medicines to clear the problem. Right now we are using Naturolax(one Tbsp in the evening mixed in 250 ml of water) and Y lax(bed time) to make him pass stool. We give him fruits,leafy vegetables regularly. But still the problem is there. If we use Y lax then only he will pass stool next day other wise he will not pass the stool. What can we do to make him pass stool regularly?Please suggest. Age : 3.8 yrs Height : 120 CMs Weight : 25 Kg How many Liters of water can he take per day? Please advice. As of now he drinks one Liter water per day. Also He doesnt show interest in chewing the food. Doctor: hi....Suggestion on constipation...1. Natural methods are the best to relieve constipation.2. Constipation is a risk factor for UTI3. Maximum milk consumption per day should not exceed 300-400ml4. Minimum 3-4 cups of fruits and vegetables to be consumed per day5. Toilet training - that is - sitting in Indian type of lavatory daily at the same time will help a lot.Hope my answer was helpful for you. I am happy to help any time. Further clarifications and consultations on Health care magic are welcome. If you do not have any clarifications, you can close the discussion and rate the answer. Wish your kid good health.Dr. Sumanth MBBS., DCH., DNB (Paed).,"
},
{
"id": 179386,
"tgt": "Suggest dosage of tylenol for mild fever",
"src": "Patient: My grandson, who is 5months old, had to get all four of his immunization shots a day and a half ago. He is running a slight fever and seems to be in discomfort. I was told by his doctor at the time of his shots to give infant Tylenol if this were to happen. He didn t say what dose and the bottle says ask doctor. How much do I give? Doctor: Hi...Paracetamol can be given in the dose of 15mg/kg/dose every 4-6th hourly that too only if fever is more than 100F. I suggest not using combination medicines for fever, especially with Paracetamol.Regards - Dr. Sumanth"
},
{
"id": 136257,
"tgt": "What causes swollen bilateral retroauricular lymph node?",
"src": "Patient: what would cause bi-lateral retro-auricular lymph node swelling in an otherwise healthy 11-yr old male. unknown duration... just noticed today. no other lymph chain swelling noticeable. no malaise, only aware of pain on palpation of the swollen lymph nodes. Doctor: Hi, Bilateral retroauricular swelling with pain on palpation could be due to infection or mumps. In my opinion, you should give brufen 200 mg tablet three times a day for 2 days and start antibiotic amoxyclav 375 mg three times a day for 3 days and give adequate liquid diet. However, since this is a prescription medicine, i advice you to meet the local doctor to confirm the diagnosis. I hope this has helped you. Take care. Regards Dr Deepak Patel, MD Pediatrics"
},
{
"id": 71131,
"tgt": "What causes difficult inhaling oxygen while having decompression sickness?",
"src": "Patient: I am a ex US Navy Diver that was treated for decompression sickness in the 60 s and 70 s. We regularly did bounce dives to depths for hours on time with only 12 hours time between starting a new set of bounce dives. Completely different tables than today. I have exhibited some of the symptoms of DCS over the years but the last 4 1/2 years I have had to go to 4.5 liters of O2 to maintain above 90 SPO2. My Pulmonary function test are normal with no obstructions in the lungs or airways. Heart has no blockage and my blood pressure is 122/72. The doctors that have worked my case say I simply no longer absorb O2. I have been told this could be residual effects of DCS. Do you have any experience or recommendation? Doctor: Hello and Welcome to \u2018Ask A Doctor\u2019 service. I have reviewed your query and here is my advice. As you explain the history, it would be better for you to do a DLCO test to evaluate the diffusion capacity of the lungs of CO. Please discuss with your doctor."
},
{
"id": 182534,
"tgt": "Has there ever been a link between wisdom teeth extraction and something akin to seizures?",
"src": "Patient: Has there ever been a link between wisdom teeth extraction and something akin to absence seizures? We are searching for a cause to some episodes my son is having and the timing with the extraction is suspect. Twilight sedation was used and only Ibuprofen for pain afterwards. Doctor: Wisdom tooth extraction can only be a cause precipitating the episodes your son would have been susceptible to it already. Extraction of teeth creates a certain stress that the body has to deal with which may also be evoked by things like heavy exercise, swimming etc. Its just that the cause would have already been and dental extraction just made it to manifest. You need not worry though, just be under proper follow ups for the seizures with the medical practitioner and do report it to the dentist in any subsequent visits so that the necessary protocol can be followed."
},
{
"id": 12970,
"tgt": "What is the cause for rashes on the skin as steroid and antihistamine didn't work?",
"src": "Patient: I have a rash and am unable to pinpoint the cause. I have been to a dermatologist who prescribed claritin in am and antihistimine in pm and steroid creams. Nothing seems to help. I am logging food intake and changed laundry detergent. Can you help me determine likely causes Doctor: Hello, Rashes on skin commonly occurs due to side effects of medications, allergies, eczema, autoimmune conditions, psoriasis, contact dermatitis or shingles. Since antihistamines and steroids are not working. You are suggested to have a clinical examination by a dermatologist done. You may have to get blood test, test of an area of the scaly or crusted skin, skin biopsy & STD tests done. You will be treated with antivirals or antibiotics depending upon the severity of the infection detected. Stop taking any food that causes allergy, medicines causing side effects & reduce stress. Avoid alcohol, smoking & addictions if any. Avoid too much exposure to the skin, wear loose fitting cotton clothes. In the meantime you can apply Crisaborole (Eucrisa) ointment locally. Phototherapy in also an effective treatment. Hope I have answered your query. Let me know if I can assist you further. Regards, Dr. Nupur K, General & Family Physician"
},
{
"id": 87950,
"tgt": "What causes stomach pain and blotches in area of pain?",
"src": "Patient: hi I am a 46 yo female. I've been having stomach pain since last night. It feels like my stomach is cramping. The pain comes in waves and is right in the middle of my abdomin. I just changed and noticed my stomach is all red and blotchy where I am experiencing the pain. What should I do? I am very rarely sick and thought this may be a bug but I am worried about the discoloration. Doctor: Hi,As this lesion developed abruptly there might be having some poisonous bite giving pain and allergic reaction in surrounding area.Take antihistamine like Cetrizine or Benadryl for 2-3 days.Take paracetamol or Ibuprofen for pain.Apply Zinc calamine lotion on the part.Ok and take care."
},
{
"id": 24813,
"tgt": "What elevates the resting heart rate with blurriness of vision?",
"src": "Patient: I have had an elevated resting heart rate in the 120s even into the 130s and 140s for the past 3 weeks the doctor I saw said it could be inappropriate tachyacardia. I am 29 years old and I am a little over weight at about 130. I have been suffering migraines for over 2 years and the doc all told me I was depressed and the meds made me pack on weight. Recently I have had 4 different episodes of losing vision in my right eye. Any help would be greatly appreciated. Doctor: hello there I understand your concernI feel your symptoms mimic migraine and are fallout of that onlyYour obesity is pushing you in a corner. Your loss of vision in the right eye doesn't seem to be embolic You require a full body check with investigations directed towards full blood profile inclusive of thryroid and xrays ekg and an echocardiogram with a tmt to assess your exercise capacityhope that helps"
},
{
"id": 75296,
"tgt": "What causes chest pain and difficulty in breathing?",
"src": "Patient: The doctor told my nan in hospital she has a weak heart. She was admitted for chest pains and really bad trouble breathing. How long does she have? She has had diabetes for 12 years now and she just turned 89. He said it was about the tablets she was on too. Doctor: HelloI can understand you concern.There are some heart diseases that when they are not well controlled they can damage the lungs too.The patients in this situation begin to complain of difficulty in breathing and sometimes cough.Diabetic patient have heart problems too and it is very important to take in control the sugar values.So my opinion is that in your nan case cardiac problems related to diabetes have cause these problems.I wish a fast recovery for your nan.Thank you Dr.JolandaPulmonlogist"
},
{
"id": 214910,
"tgt": "Hyperpigmentation, sensitive skin. can cosmo peel help me?",
"src": "Patient: Hi doctor, I have hyper pigmentation and skin is really sensitive. I have mix of hereditory pigmentation and pigmentation due to acne . I also have lentigens. A beauty clinic suggested me to go Cosmo peels. But one of the dermotologist told me that there is no cure to this. There are treatments available but those treatments may have side effects i may loss the colour of my skin. I would like to know if cosmo peel can help me out, but am also scared of going for it. I would not like to take any risk Doctor: hello this is dr satya, welcome to health care magic facial pigmentary lines, lentigenesare inherited and acne leads to pigmentation which is acquired. satisfactory treatment is not availble for such conditions. cosmo peel is expensive and may not adress the condition. lightening of the skin of the face color may be done by a few sittings of q swithched laser. the results are going to be temporary. camoflage is a good alternative which hides away the abnormal color temporarily without affecting your sin. it is posssible to continue treatment later on thank you dr satya"
},
{
"id": 152309,
"tgt": "My 8 years old cousin has not perfect brain by birth. Is there any treatment for this?",
"src": "Patient: gudevening sir, this is akhila from andhrapradesh [hyderbad] and i saw your profile in your hospital websites and finding you are the neurologist so i need some help from you please give your valuable guidence. coming to the point my cousin have some medical assistence. he was 8yrs older guy by birth he was not perfect he was not able to work with his own in common works also like walking,talking etc.. and his brain was not given signals for pissing, eating,and for drinking water also. clearly his brain was not working he doesn't no any thing. this is our problem is there any option to rectified this problem in your hospitals please kindly help me and my email id was akhi.022@gmail.com please send mail on that email address Doctor: Hey I am not the neurologist. This is a open community I guess. You can consult the neurologist by clicking the above button saying Talk to the Neurologist"
},
{
"id": 185853,
"tgt": "Is runny nose normal after wisdom teeth removal?",
"src": "Patient: I just had my top wisdom teeth removed 3 days ago. Is it normal for my left nostril to be runny? The pain and swelling have finally gotten better but I have an itchy runny left nostril. I was also told not to blow my nose or sneeze for a week so the runny nose thing is driving me nuts. I am taking Vicodin and Penicillin VK. Doctor: HiWith your query, as such there is no relationship between runny nose which is of viral origin and wisdom tooth extraction. Reason for not blowing the nose is it creates negative pressure and leads the extracted socket to bleed. For runny nose you can use anti histamines and steam inhalation for better relief. Hope I cleared your doubt"
},
{
"id": 155513,
"tgt": "Suggest foods to be taken after surgery for saliva gland cancer",
"src": "Patient: I had surgery 3 weeks ago for Saliva gland cancer the Dr. took the entire roof of my mouth out I could not swollow pills I have been doing Liquid IBprofen and hydrocodone for about 6 weeks now. I have such terrible heart burn and found out today the milk is not good at all for coating and helping the situation. I don t know what to eat or what to do. Doctor: Thanks for your question on HCM. In my opinion you should eat following things. 1. Drink plenty of fluids. Except milk. Drink fruit juice, water, vegetable soups etc.2. Avoid hot, spicy and hard food.3. Take mashed, boiled food as it is easy to digest and easy to chew.You can also consult dental surgeon for implants. You can put implant in roof of cavity and can have normal chewing.For GERD avoid large meals, instead take frequent small meals. Start proton pump inhibitor."
},
{
"id": 40832,
"tgt": "What does this ultrasound report indicate?",
"src": "Patient: Hey I did an ultrasound and the report stated the anteverted measures 86x53x69 mm in diameters, a secretory phase endometrium was seen, a 9x7 mm fibroid mass is seen in the anterior myometrium, right ovarian volume= 14.1 cc, left ovarian volume= 11.0 cc, a corpus luteal cysts was seen on the right ovary, more than 15 , 2-3 mm follicles was seen in each ovary consistent with the presence of polycystic ovaries and a moderate amount of fluid was seen in the posterior cul de sac, most likely as a result of a recent rupture of an ovarian cysts and no adnexal mass was seen . My question to you now is what does this mean ? Would I ever be able to get pregnant I had a miscarriage back in 2014 where my water broke suddenly at 6 months and the baby was breech and didn t even weigh a pound could that be because of my diagnosis because I been trying to conceive ever since and have been unsuccessful ? Doctor: Hi I think you will need some medicines for ovulation. The fibroid is very small. Don't worry about that. You can get pregnant. Consult a doctor. Also do a thyroid profile and prolactin levels. A semen analysis of your husband should also be done. Hope I have answered your question."
},
{
"id": 101913,
"tgt": "How to cure bronchitis?",
"src": "Patient: hello sir . i am 25/m and i ve been told tht i am suffering frm bronchitis and been using budamate 400 and reduced to budamate 200 . My problem still persists of having breathelessness ,and pain in the chest section ...... may i know what can i do further to reduce or cure it ......??????? Doctor: hello, thanks for ur query, u can take tab. lukotas 3d ( montelukast+ levocitrizine+acebrophylline) one tab at bed time with ur budamate inhaler for one month. try to avoid allergens like dust, chilled climate , cool drinks, ice creams and strong deodorants. all d best. take care."
},
{
"id": 41889,
"tgt": "How successful is Ovitrop injection in a pregnancy?",
"src": "Patient: Hello doctor,I am under pcos and pregnancy treatment. I used siphene 100mg and metformin for 5 days.Today 7th day she prescribed (fsh)ovitrop 75iu.What is the success rate of ovitrop injection and how many days i need to take this injection.I heard there is chance of multiple pregnancy how to avoid it. Doctor: Hi welcome to healthcaremagic.I have gone through your question.Ovitrop inj contains follicular stimulating hormone. Which helps in stimulating follicles in pcod ( polycystic ovarian disease).There are chances of multiple pregnancy aa it stimulates more than one follicle, and may fertilised more than one follicles cause twins, triplets, quadruplets.This injection may be needed for 3/5 days according to need. Success varies from patient to patient.Hope i have answered your question.Would be happy to help you further.Take care."
},
{
"id": 94378,
"tgt": "Having stomach pain. Done with ultrasound of abdomen. Result showing thickened endometrium. Reason?",
"src": "Patient: hi doctor my sister remarks as a result ultrasound normal sized uterus and ovaries thickened endometrium cant rule out early pregnancy fluid in the culdesac ,other findings culdesac;with minimal fluid no gestational sac nor complex mass seen at the time of scan,doctor she is suffering stomach ache in right lower abdomen,pls help me to know hw this is syptoms came from,have a nice day doctor godbless Doctor: Well endometrium goes through phases, its thickness is different in every stage of cycles. In youn woman such pain usually comes from pelvic organ disorders but also bowel,appendix and bile ducts disorders should be considered. If she has prolonged bleeding or cramps during periods she should do hormonal tests to see is there any abnormality. If not then CT scan should be done to vizualize more precisely. Wish you good health."
},
{
"id": 176589,
"tgt": "Suggest treatment for hand-foot-and-mouth disease and excessive bleeding from mouth",
"src": "Patient: Hi doctor...my 2 year old daughter was diagnosed having hand foot mouth disease for past one week. I noticed that her gums bleed terribly while she asleep. What should I do since the bleeding is getting worst? She finished one course of augmantine which was given for 5 days and now on pcm totally...I m very worried that it could lead to something else. Doctor: unusual bleeding from gums will need hematological evaluation. please do complete blood count , peripheral smear, PT/APTT and meet pediatrician on priority."
},
{
"id": 134662,
"tgt": "What causes lumps and pain in the knee?",
"src": "Patient: Thank you! I have pain in my right back leg where I bend it. It has several small lumps there like maybe at a vain, I cannot see this but can touch it. It has some soreness to it at times my leg swells, only when I keep it elevated does the pain ease. Hope it s not blood clots or worse. Can you help me with this issue? Doctor: hellosmall lumps or swellings behind the knee may be bursitis- a condition in which small protection pouches of membrane called bursa get inflammed and there is collection of fluid there.The usual treatment is by anti-inflammatory medicines and compression bandage, rest till the inflamation subsides.If it doesnt then diagnostic aspiration is done to rule out infection and cause of swelling followed by suitable antibiotics and drainage if necessary.So consult an orthopedic surgeon who will decide and treatthanks"
},
{
"id": 149488,
"tgt": "Operated for cerivcal myelopathy. MRI shows Diffuse bulges. Indenting anterior subarachnoid. Ligament is thickened. Treatment?",
"src": "Patient: Hi,I am Meena Sheth. I was operated for cerivcal myelopathy in August, 2012. My latest MRI REPORT state that Diffuse bulges are seen from C3-4 to C6-7 levels, indenting the anterior subarachnoid space. The posterior longitudinal ligament is diffusely thickened.Pl do let me know in simple language what do you mean by above. Doctor: Hi, cervical myelopathy means - spinal cord in the region of neck is having issues. The diffuse bulge from c3-c4 to c6-c7 Levels means the disk between the two vertebras is being pushed inside compartment (its slipped from normal position) which creates an indentation / grove like structure in spinal cord and compressing from the front side at separate locations between c3-c4 and c6-c7. Posterior ligament is thickened - means the cover on the back portion of the vertebral column is thick. Thanks. hope it helps.Dr.JAY PATEL."
},
{
"id": 185086,
"tgt": "Suggest medicine for swollen gums",
"src": "Patient: my mother has a swollen gums i think a damage tooth is the cause. we only have cefixime 200 mg capsule available. my mom wants to take it now, will it work? my mom is 50 years old, about 5ft and 4 inches and around 60 kg in weight.... she has no heart condition and non diabetic... Doctor: Thanks for your query, i have gone through your query. the swollen gums could be because of the gum infection or tooth infection. consult a oral physician and get radiograph done to rule out these two conditions. if it is tooth infection then get the tooth treated with root canal treatment or extraction. if it is gum infection then get your teeth cleaned. you can take cefixime 200 mg if no other drugs are available immediately. ideal drug could be penicillin and metronidazole combination(if you are not allergic). i hope my answer will help you. take care."
},
{
"id": 111755,
"tgt": "What is the remedy for the pain under my rib cage after a back injury?",
"src": "Patient: hello, two days ago i fell backwards and landed on my upper back. i was fine. but yesterday i was really sore and i noticed that i had a stomach ache the whole day. it wasn't in my stomach area but more upper like directly under my rib cage. today the same discomfort and i have been passing a lot of gas and diarrhea. what happened? Doctor: Hello, I have studied your case. Due to trauma to back there can be injury to vertebra and rib ,you may need to do X ray spine and MRI spine.Due to compression of this nerve root there is back pain.For these symptoms analgesic and neurotropic medication can be started.Till time, avoid lifting weights, Sit with support to back. You can consult physiotherapistfor help.Physiotherapy like ultrasound and interferential therapy will give quick relief.Core stabilizing spine exercises will help.I will advise to check your vit B12 and vit D3 level.Hope this answers your query. If you have additional questions or follow up queries then please do not hesitate in writing to us. I will be happy to answer your queries. Wishing you good health.Take care"
},
{
"id": 137149,
"tgt": "What causes ankle, knee and arm pain?",
"src": "Patient: Please write your query hereI m 36yrs old and some of my symptoms are: can t take the heat even 70degrees seems too hot I ve been getting more headaches and migraines i have a slightly crooked spine my stomach is tender and on and off it cramps up close to labor pains I ve had an ultrasound they found a follicle on my ovary but that s not the cause of my pain i wake up my ankles hurt i get random knee hand arm pain i don t know what s going on i have a three yr old i have to take care of and i can t even lift him please help i started taking omaprazole i was getting sick eating a small amount of food its only helped a little. thank you Doctor: This is a kyphoscoliotic deformity of your back spine which is showing its bad signs... It's causing damage to Ur nerves. I advice you a magnetic resonance image of the deformed spine to know the extent of damage of your spine as early as possible before it worsens. U need some lifestyle modification n spine braces. If that's not helping u will definitely need surgery. Don't take pantoprazole only. A pregabalin 50 mg every night will work effectively. Thank you"
},
{
"id": 189534,
"tgt": "Severe pain in the tongue due to mouth ulcers. Not able to eat. Need immediate cure",
"src": "Patient: Hi i had severe pain in my tounge i was nt able to eat also i consulted a doctor nd they tld me that i had mouth ulsers nd they prescribed me with a gel nd few pills nd it healed withing 3 days. Nw recently 3 days back while havin lunch i bit my tongue nd it startex bleeding but at that time it was nt painin nd the followibg day i was nt able to speak or eat it hurts badly.even now itd the same. My mum is preparing delicious food for dinner and i so badly wanna eat but the pain is insane pls temme wat to do to take away the pain immediately? Doctor: Hi, Thanks for asking the query, Recurrent mouth ulcers occurs due to many reasons like immunlogic defects, vitamin & iron deficiencies, traumatic self inflicted bites , toothbrushing and dental trauma, hormonal imbalance, allergic factors. You have traumatised the tongue accidently while eating, dont worry apply Dologel oral ointment topically over the affected area, Take multivitamin tablets daily . Apply cortisone acetatae topically over the affected areas. Maintain a good oral hygiene, use saline water gargles, antiseptic mouthwash gargles. Avoid eating hot and spicy foods. Hope this helps out. Regards..."
},
{
"id": 98125,
"tgt": "Severe leg pain, has Neurofibroma in leg nerves. Had surgery, lumps still felt, pain persists. Alternative medication ?",
"src": "Patient: Hello Sir, My mother has been suffering from severe leg pain and the doctor said she has Neurofibroma in her leg nerves. She underwent a surgery but the pain is severe and she can still feel the lumps . The recent ultra sound scanning report(post operation) says Schwannomas shows multiple inter muscular solid hypo echoic lesions noted at medial aspect of lower leg and thigh. And the Largest lesion is about 17x10mm size. Nature of Specimen: Excised? Neurofibroma Gross: Received a 2x1.5x0.5 cms soft tissue mass. Cut section capsulated, pearly white. Micro: Sections studied show the features of a Schwannoma. Her doctor says there is no use to do the surgery again as the lumps are so many and are tiny in size. He is suggesting to suffer with the pain and these lesions are not harmful. My mother started to use the Homeopathy medicine since 2 months from now. Initially there was bit relief with the pain and she can t feel the lumps also. But the pain doesn t go away completely. I am worried if these are really not tumors and should we continue using the Homeopathy patiently. Pls throw some light on this. Thank you so much Doctor: Hello Satya, Thank you for your query. Schwannomas are the benign tumors arising from the nerve sheath. These are common among people with history of neurofibromatosis. These Schwannomas very very rarely become malignant, there is only 1% probability of it. The pain caused in the leg is due the pressure these tumors cause on the nerves. As the other surgeon suggested, surgery is not the cure for this condition as schwannomas recur again even after surgery. The best method is to relieve pain and the pressure symptoms rather than excising them. Homeopathy is one of the best alternative medicine for the neurofibromatosis. The medicine helps in stopping the recurrence, and reducing pain. There are some cases which i treated had tumors reduce in size and then disappear. It might take little time but definitely will help in treating this condition. I suggest you continue using it as your mother found some improvement. You can feel free to contact me anytime..I would be happy to help you Thank you Dr.Shalu"
},
{
"id": 126337,
"tgt": "What causes tightness and pain in the knee?",
"src": "Patient: Worked out last Friday, including a 35 min run...everything good. My left knee tightened up that evening...never have had any problems with either knee. Since then, when I get up in the morning, knee feels OK but as the day goes on, it tightens up and is painful. When I sit for a long period, it is painful to stand and strertch out my leg. Eventually it stretches out and feels better but still somewhat painful. I am 63 years old and in excellent health. Never had any problems with my legs. Doctor: Hello, If I was your treating doctor for this condition of the knee, it could be of the following possibilities. First - Mild ACL - Anterior Cruciate Ligament injury. Often we can diagnose with a physical examination. An ultrasound and an MRI will confirm the diagnosis. If it is so I suggest providing rest to the joint, use kneecap while moving around and also avoid jerky and quick movements of the knee. Secondly - mild osteoarthritis - This can be diagnosed with an x-ray. It can be diagnosed with physical examination too. I suggest to consult a physician and get an X-Ray of the joint done to rule out the other possibility. Hope I have answered your query. Let me know if I can assist you further. Take care Regards, Dr Mahesh T S, Ayurveda Specialist"
},
{
"id": 182446,
"tgt": "Suggest treatment for a dental problem involving broken tooth",
"src": "Patient: I had a root canal done a few years ago, but never got it capped. The filling held and I honestly forgot about it until one morning 90% of the tooth broke apart in my mouth. Occasionally food gets it the opening of the tooth, and I remove it, but I noticed a very small piece of \"pink\" came out. What damage if any can happen if the gutta percha comes out? Doctor: hello, i went through your query. keeping a broken tooth inside the mouth is always carries the risk of getting it infected. the food accumulating in that site not only affects that tooth but also the gums of the adjacent teeth also. So it is better to remove the remaining part of the teeth and later replace it with artificial teeth. Hope my answer helps."
},
{
"id": 60794,
"tgt": "What causes a hard, sore lump near the ear?",
"src": "Patient: I have a lump about the size of a marble in front of my ear. Right before where the hollow of my cheek is. I just recently discovered it and it\u2019s a little sore to the touch. It is hard and doesn\u2019t move around. I am just wondering if I should be worried? Doctor: Hello and Welcome to \u2018Ask A Doctor\u2019 service. I have reviewed your query and here is my advice. In most cases cause is benign and should not worry you. If this I soft and not painful then this in most cases indicates swollen lymph node, lipoma or atheroma. And if this is hard and immobile swelling behind the ear would be a bony swelling, most commonly known as osteoma. These swellings are generally asymptomatic and requires no treatment. However if it increases in size or you develop pain then I would like you to meet a ENT doctor for further evaluation and treatment. The other swelling in the neck is likely to be a lymph node and since it has not increased in size for past few years then you have nothing to worry. No treatment is required. Hope I have answered your query. Let me know if I can assist you further. Regards, Dr. Ivan R. Rommstein"
},
{
"id": 193307,
"tgt": "Suggest remedy for hair loss,weakness and shakiness",
"src": "Patient: Hi, am Teddy and 22yrs of age.i have been masturbating for 6yrs now n i just recognised the quick process of my hair loss leading to baldness.i may look healthy in appearance but you will only see a shaky fingers if u ask me to point my fingers to your eyes.i have tried so many ways to quit this evil thing but i still can't stop when i know i'm growing weak and shaky after carrying heavy objects.please help me. Doctor: Hello,Masturbation does not have any side effects. This is socially, spiritually and ethically unacceptable thing. You can avoid masturbation by keeping yourself busy, spend more time with family and friends, don't be alone for a long time, join some support group and do regular exercise to keep yourself healthy.Hope I have answered your query. Let me know if I can assist you further. Regards, Dr. Sameen Bin Naeem, General & Family Physician"
},
{
"id": 132451,
"tgt": "How to cure sciatica nerve pain in leg?",
"src": "Patient: Hi.. My mom (58 years old) is suffering from Sciatica Nerve Pain in Right Leg. Very difficult to walk and sometimes in the night, pain starts. Lots of medicines had already taken but no relief. Her weight is 80 and height is 5 8 . No High BP, no Sugar, Uric Acid under control, No chronic disease. Doctor: Hello,Welcome and thanks for your query.She has a pinched nerve from the disc pressing on the sciatic nerve. It would be good for her to lose weight and to have an anti inflammatory pain reliever.She can use Neurontin at bed time to give her pain relief and better sleep.Do get back to me if you have further queries."
},
{
"id": 8725,
"tgt": "Dark patches on face. No improvement with peeling, using kojivit. Effective remedy?",
"src": "Patient: I have dark patch on face (cheek) and did two sessions of peeling. Also doctor prescribed Skinlite to apply on affected areas. When I use the cream my dark patches minimise. But doctor cautioned me against using the cream for long period. He has now prescribed Kojivit, but after applying it for more than 10 days no improvement. Pls suggest Doctor: hello and welcome to HCM forum, dark patches on the cheeks is a very common problem nowadays, as you stated that you have applied creams and have also undergone chemical peeling. i would like to inform you that patches take time to disappear completely , so you will have to be patient. besides peels, we now have lasers(Q-switched, IPL) to treat pigmentation areas, they require atleast 3 sessions, they are painless and very effective. i would recommend you should visit a dermatologist who perform all these cosmetic procedures with lasers, continue using Kojivit, it has whitening properties and will definitely help to some extent. Also, start taking oral antioxidants, they have depigmentation properties. i wish you good health."
},
{
"id": 85080,
"tgt": "What are the side effects of taking the thyrofit tablets?",
"src": "Patient: Sir i had my thyroid chech . TSH was 10.6 Dr. prescribed thyrofit 50 for 1 month . After i month i was feeling ok he said take for 2 months more. Can prolonged use be harmful & what r the side effects of it. Please help.My e-mail is YYYY@YYYY Doctor: Hi, Based on the history given you seem to suffering from hypothyroidism ('underactive thrydoid') for which your doctor has prescribed thyrofit. Since your thyroid can not produce enough thyroid hormones you have to take it as a 'replacement therapy' for life long. It is not known to cause any harmful effects on prolonged use. Generally it is well tolerated by most of our patients. However, its common side effects increased heartbeats, palpitation, sweating, lack of sleep, weight loss, and tremors. These side effects can be prevented by adjusting the dose of thyrofit. So, continue taking thyrofit as prescribed by your doctor and get the thyroid function tests as requested by your doc. Hope I have answered your query. Let me know if I can assist you further. Take care Regards, Dr. Mohammed Taher Ali, General & Family Physician"
},
{
"id": 11169,
"tgt": "Need treatment for hair fall",
"src": "Patient: Hi. I m 46 yrs old and I noticed for the last few months Im having extraordinary hair fall. I tried changing shampoo, tried some herbal alternatives to stop hair thinning but it still continue. I m afraid, this will come to worst. I am not taking any medicine nor vitamins. I appreciate so much to hear from you. Doctor: Hi, you are suffering from telogen effluvium.It is a condition where more hair comes in telogen phase and looses contact with their root.take tab biotin 10 mg daily.wash your hair with volumizing shampoo.continue this treatment for 3 months.this will stop your hair fall.Thanks"
},
{
"id": 63213,
"tgt": "What causes painful lump in neck at the site of injection?",
"src": "Patient: I had a procedure today where they were supposed to give me an injection in my neck to basically burn the nerve endings. I ended up with a golf ball size lump in my neck and the pain is worse. They did an ultrasound and it is fine. What could this be? Doctor: Hi,Dear,Thanks for your query to HCM virtual Clinic.I studied your query in all the details put forth by you.I understood your health concerns.More information is needed like-Was the compression bandage given for that lump or not?or the injection is left without any compression bandage?Cause of painful lump in neck post-Chemical denervation-Still from the data given from you,following causes need to be ruled out-Moslty your lumps seems to be- either a venous hematoma/ or arterial hematoma , post injection,due to the accidental blind puncture of the perineural vessels in the neck.-ORa effused injection material in perineural tissue, which is causing pain due to tissue irritation from the ablative chemical use for neck nerve chemical denrvation.(Phenol or Alchohol).Still I would suggest you to take Second opinion from ER Surgeon as I don't have any first hand clinical information or a pohotograph of this lump.To confirm other causes I would hence suggest you to consult ER Surgeon or PCP doctor.But you need to be worried as it would complicate into infective and other problem associated with it.And act fast on the above suggestions,which helps you to plan treatment with your ER Surgeon. Hope this would resolve your query with full satisfaction.Welcome for any further query in this regard to ME by direct question at HCM.Write excellent review and hit thanks, to narrate your experience ,which would improve my rating for many other visitor's to HCM.Wishing you fast recovery.Have a good day.Dr.Savaskar M.N.Senior Surgical Consultant.M.S.Genl-CVTS"
},
{
"id": 21070,
"tgt": "What causes chest pain when you climb the stairs?",
"src": "Patient: i am having a stable RBBB which is asymptomatic. do i have to treat it? my TMT & echocardiogram tests are normal but, i feel chest pain in climbing stairs after a meal. The pain subsides soon after i finish climbing stairs. my cariologist suggested an angiography. what do i do? Doctor: hellothanks for posting here. RBBB does not need any treatment. only newly occurring RBBB neds evaluation. I guess you have got RBBB from long back. your symptoms are very typical for effort angina. therefore you must go ahead with an angiography. i agree that your TMT is negative, but TMT head a sensitivity of 70% i.e. it will diagnose only 70 out of 100 positive cases. so please go ahead with s cautionary angiography.thank you"
},
{
"id": 135862,
"tgt": "How to treat a leg lump caused by injury?",
"src": "Patient: Hello, I fell mountain biking today. I landed on a 3 inch tree stub about the width of a nickel or quarter. When I finished the ride & got home I notice a large goose egg. It s on the side of my leg about 8 inches below hip. I m trying to ice it all night so I can go on my trip Monday & Tuesday & ride again. Any Thoughts? Thanks KB Doctor: Thanks for your query. I have gone through your query. The lump can be because of the inflammatory swelling secondary to trauma. The other possible cause can be because of the fractured bone. Nothing to be panic, consult a general physician and get it evaluated. You need get a radiograph done to confirm the diagnosis. Mean while you can take analgesics like diclofenac and antiinflammatory drugs like serratiopeptidase (if you are not allergic).I hope i have answered your query. Take care."
},
{
"id": 33881,
"tgt": "What causes redness and tenderness in hand after stab injury?",
"src": "Patient: hello my name is simon.. i,m 31, 6 foot 1 inch and 13 stone two days ago at work while welding the copper coated welding wire pierced the palm of my hand. My hand has obviously been sore since. last night and today hand was very sore and a red patch around puncture seems to be getting larger? was wondering if maybe i should go to doctors incase of infection or poisoning due to the copper? its not really swollen much but very tender and red! any advice would be appreciated Doctor: Hi,From history it seems that there might be having secondary bacterial infection producing swelling, induration and pain.You might require one course of antibiotic and anti-inflammatory medicine course for 3-5 days.Clean the wound and do dressing with antibiotic cream.Ok and take care."
},
{
"id": 115198,
"tgt": "What was the episode with a severe head spin?",
"src": "Patient: Hi, during a relaxation exercise I experience a massive head spin that suddenly came on and by putting my had a a flat surface made it stop, I was sitting up with my eyes closed at the time but I have also had them when sitting with my eyes open. I do suffer from acute migraine. Doctor: Hello dear,Thank you for your contact to health care magic.I read and understand your concern. I am Dr Arun Tank answering your concern.This problem arises from the vestibular system it is located in the back of the ear. It helps in setting our position in the space.If its disturbed than people may suffer Vertigo.I advice you to take betahistine, it will help in resetting of the vestibular apparatus and so can be help full in recovery. Please take it under your docyors guidance.Please do not start exercise very rapidly. First do some worm up to stable your system for exercise. Once the system is stabilised you are free to do exercise.Migraine treatment should also be taken with it.I will be happy to answer your further concern on bit.ly/DrArun.Thank you,Dr Arun TankInfectious diseases specialist,HCM."
},
{
"id": 10564,
"tgt": "Suggest remedy for hair loss and thinning",
"src": "Patient: I have been on methotrexate for several years. Now I noticed my hair seems to be coming out. I have always had very thick hair. Now its thin and very unmanageable. I am very discouraged about this. I don t want to be crippled, so I don t know what to do. I can t afford to pay for an answer! Doctor: hi there.1. we can protect the remaining hair and we can try to make them thick, don't worry.2. your concern is chronic telogen effluvium leading to baldness.3. you can start using mintop 2% , apply 20 drops once daily bed time.4. start using trichogro hair supplements daily one after breakfast and use scalpe shampoo weekly twice head bath, use only drinking water for scalp wash.5. prefer diet rich in proteins like egg white, sprouts etc.this will help you."
},
{
"id": 923,
"tgt": "What medicine should i take to stop the pregnancy?",
"src": "Patient: this may we sex many times. i dont remember , may 19 we started to have sex and its withrawal method/ it was 5 days before my period we had sex again, and his sperm comes inside me. should i get pregnanT? and help me. what medicine should i take to stop to get pregnant Doctor: hello,As per your question you have not mentioned the last date of menstruation and also from your explaination you said you had intercourse 5 days before you got your periods,if after the intercourse you got your periods and after the periods you have not had any sexual contact with your partner then there is no chance that you can be pregnant.If you had intercourse after you got your periods if you mention the date It will be easy to give you details about it.If there is any doubt you have as I dont know exactly you are due on your periods or you already have them.it better to go for a urine pregnancy test,you can do it by yourself.just get the pregnancy test kit and follow the instructions.Even after the doubt persists I would suggest you kindly visit a doctor and get your doubts cleared.Hope this helped."
},
{
"id": 89598,
"tgt": "What causes abdomen pain with bloats in patients with endometriosis?",
"src": "Patient: Hi. I have endometriosis and have had diverticulitis before. Since this morning I have pain in the lower left part of my abdomen and the pain is increasing as is my nausea. The pain sometimes radiates to the right and rectum. When I turn on my side it generates a different pain and I am bloated - my lower belly is tender but hard. Any ideas? Doctor: Endometriosis itself Is Very painful at times.If this pain is new then there may be associated urinary tract infection or irritable bowel syndrome constipation type. As you may be already taking some medicine s for your existing disease that may cause constipation as well. Give urine sample for testing. Mean while take pantoprazol with domperidon empty stomach. Add urine alkaliser for time being. If constipated add mild laxative.mild analgesic like acetaminophen may be used as nausea might be aggravated. See your doctor if required."
},
{
"id": 165986,
"tgt": "What causes a lump in the armpit in a child?",
"src": "Patient: my grandchild is 6 months old and when you go to pick her up under the right armpit towards her back about level with breast she has a lump. yesterday while getting her ready for a bathand she was standing in crib she turned and you could see a lump about 1 plus inches wide and 3 plus inches long. what could this be. Doctor: This may b an enlarged lymph node. You should go to your pediatrician because this size is huge and abnormal."
},
{
"id": 7220,
"tgt": "Endometrial thickness-14.7mms, right dominant follicle-21.5mms. Is it normal?",
"src": "Patient: my name arunima,age -26, follicular study day-13 ,endometrial thickness-14.7mms, right dominant follicle-21.5mms , is it normal? Doctor: Yes arunima, this study is normal. Size of the follicle is quite good , it should rupture within 24 hrs which can be seen on scan and you will find some free fluid in pouch of douglas."
},
{
"id": 44388,
"tgt": "Size of egg got smaller after follicle rupture. What should I do?",
"src": "Patient: hello mam in my last cycle egg was not ruptured till 22nd day .its size was increased till 37 mm.in this cycle the size of eggs was not increased on 18th day it was 15.2but on 21 day they got smaller.pls tell me what should i do. Doctor: Hi , welcome to H.C.M.Forum. did you have the scaning on 22nd day of last cycle, 18th day this month, and on 21st day of the period. if that is the case also your doubt might be correct. but egg can't be visualised in scaning also,as it is a single cell and you need an electonic microscope tosee it. any how don't worry. if it is for conceiving consult a gynaecologist, and have a baby. thank you."
},
{
"id": 224919,
"tgt": "How long will unwanted 72 disturb the periods ?",
"src": "Patient: 1. Last period date 10th feb to 15 feb. Had unprotected sex on 21st feb afternoon. and. took unwanted 72 pill after 2 hours. 3 Is this pill work for my wife. autual her next periods date 10 march but periods came on 6/03/14 and surprised is that periods finished after 2 days its never happening before.So she is afraid of pregnancy. How long the periods will get disturbed because of this Pill. Doctor: Hi , Having menstrual discharge in between 20-35 days is completely normal .So last menses came after 25 days is absolutely normal . When menstruation occurs there is sloughing off of uterine wall hence no implantation and pregnancy cannot sustain . As she had menstruation 17days following unprotected sex (again had progestin only pill 2Hr later) so negligible chance of pregnancy .So don't worry everything is fine .Thanks n regards"
},
{
"id": 81594,
"tgt": "What causes fluttering feeling in chest while taking deep breaths?",
"src": "Patient: What would cause a fluttering feeling in my chest. Which tends to be worse when I take a deep breath. Sometimes followed by my heart beating very hard for a few beats. I am 22 and healthy. However I do take methadone and i am a recovering heroin addict and i have been clean for almost a year. Doctor: HIWell come to HCMI really appreciate your concern this is functional condition and may not be due to withdrawal of narcotic, never ever think in that way, because every one those have left this feels some odd symptoms and thoughts that this could be the withdrawal and they again started taking the drugs, don't do that, nothing is wrong and everything would be fine soon, take care."
},
{
"id": 134682,
"tgt": "Suggest remedy for pain in knee",
"src": "Patient: My name is Junior 24 years old,6 2 and 220lbs , i got injured in a soccer game in October 2013. After jumping for a header, my left leg came step down first, and i heard something like a rubber band stretch then pop. I look down at my knee, i saw it bend outwards. So i just dropped to the ground i was in seriously in a burning pain. i couldn t put any weight on my left knee, I went to the E.R i got X-rays and no bones were broken. Is my soccer career over? Please need help. Doctor: helloThat waslong time ago in 2013 that you injured, what of the period after that till now-what were the symptoms, could you walk? was knee locked at anypoint of movement, have you had an MRI done?If not then I may suggest a visit to knee surgeon for MRI for any meniscal, cruciate ligaments ijury to knee, if possible an arthroscopic examination.Torn meniscus and ligaments if found can be repaired or replaced or reconstruct of antr cruciates with carbon filament synthetic ligament or reinforcement, this is for the surgeon to decide and possibly, with adequate investigations, treatment may be successful to put you back on soccer field. But you must approach a skilled knee surgeon who handles sports related injurys and is well experienced in repair surgery with latest knowhow,so try to engage at a specialized centreBest wishes"
},
{
"id": 157634,
"tgt": "Have follicular lymphoma, sciatic pain. Treated with rituxan. Could this be the lymphoma?",
"src": "Patient: I have Follicular Lymphoma ( initially large cell follicular ) treated with R CHOP and put in remission for 3 years. Relapsed one year ago with follicular only and treated with Rituxan. Had a 3 month remission and than relapsed. Being treated again with Rituxan. Cancer returned to left side of neck and clavicle both relapses. However--I am now experiencing sciatic pain ( with no cause) could this be the lymphoma? 38 years old female Doctor: Hi,Follicular lymphoma is a indolent disease, so the patient leaves with a disease for a long duration. Although the involvement of bone is rare, it can involve any part of the body. You need to undergo MRI of the spine to see for the cause of sciatic pain.It may also give a clue if there is relapse of lymphoma to spine and bone scan also may show the uptake. The involvement of spine can be conformed only with biopsy of the lesion. With the presence of disease elsewhere in the body, biopsy of spine may not be required and requires treatment for lymphoma.Hope i have answered your question."
},
{
"id": 161843,
"tgt": "How to improve skin complexion of a child?",
"src": "Patient: My 4 month old baby girl was dark when she was born. Now her face complexion is little better. But the rest of her body is very dark. Can anyone give some suggestions on to improve baby s complexion? My mom says i can use moong dhal and pure turmeric powder mix for the fair complexion. Anyone tried this? Doctor: Hi Dear,Understanding your concern. As per your query your baby have dark complexion and you want to get it improved. Well if it is genetic chances of getting fair complexion are less. I would suggest you to :- - Apply baby cream or gel containing azelaic acid twice a day ,which is a natural product obtained from cereals ans it safe alternative.- Apply sunscreen while going outside to protect your skin from sun exposure which can cause further damage .- Exfoliate your skin once a week.If condition persist after one month then consult dermatologist . Doctor may recommend antibiotic cream or antioxidants friendly to baby's skin. Hope your concern has been resolved.Best Wishes,Dr. Harry Maheshwari"
},
{
"id": 195029,
"tgt": "Suggest alternative to Doxycycline Hyclate for prostate enlargement",
"src": "Patient: I'm a 63 year old male, non-smoker, non-drinker, exercising and healthy, with symptoms of benign prostate enlargement. Yesterday, visited large practice in major city that specializes in urology for exam. Exam did not consist of PSA, digital exam or blood work and physician prescribed doxycycline hyclate 100mg twice daily for 21 days and wishes to re-examine me in six weeks. I am severely allergic to all penicillins and cephalosporins, thus the prescription above. After examining the very serious and potentially fatal side effects of doxycycline hyclate, I am inclined to not take this medication. Question 1. What are the probabilities of serious side effects with this medication.? Extremely low, moderate or high? Question 2. Is there another antibiotic that could be prescribed with less serious potential side effects? I understand that the physician is just trying to eliminate the possibility of a prostate infection prior to doing a PSA, etc., but, I'm really lacking enough information to decide if this prescription is worth the risk. Thanks, Doctor: Hi, Doxycycline is not used for BPH. It might have been prescribed for prostatitis (inflammation or infection of the prostate). You can consult a urologist and start on drugs like Finasteride or Tamsulosin. If medical management fails you might have to undergo surgical removal of the prostate. Hope I have answered your query. Let me know if I can assist you further. Regards, Dr. Shinas Hussain, General & Family Physician"
},
{
"id": 218038,
"tgt": "What is the cause of pain in leg when on cortisone injection?",
"src": "Patient: My right foot has mortons nuromas, and my foot doctor fitted me with crthodic. Pain was still bad and he gave me injection cortisone , told him was having leg pain, 4days later foot feels fine but dull ache in leg. Should i give it more time or see my M.D. Doctor: Hi, since the pain is in the leg which is different from the pain in the foot that you had before, I would be concerned particularly about a blood clot in the leg. Maybe its just muscular pain but I would like to be sure its not a clot- this can be easily excluded by an ultrasound of the veins in the leg. Since this is a specialized test that needs a physician to order and interpret it, I would recommend that you give a call to your MD. Please let me know if that was helpful. Thanks!"
},
{
"id": 106389,
"tgt": "diagnosed with eosinophilia count 8",
"src": "Patient: my daughter is diagnosed with eosinophilia count 8. she is pregnant. can she be administered hetrazen. Doctor: Hi.. Hetrezen has to be avoided during pregnancy.. This medication can be taken following the delivery...."
},
{
"id": 86287,
"tgt": "What causes persistent pain in stomach?",
"src": "Patient: I have pain that never leaves me. It is all on my left side of my stomach and my side and my lower back by my kidneys. The left side of my body is so tite and tence,it's like a hard baseball. A baseball is held together with lots of rubber band wraped very tight. I eat a lot of fiber and my stool looks like it is not possessing it because I can see the oatmeal flakes. Or do I have parasite in my body because I have all this pain. Doctor: HelloLeft sided abdominal pain may be due to many causes like problem in left kidney,intestinal,musculoskeletal causes etc.You may need clinical correlation and further investigations.Investigation include routine hemogram,liver and renal function test,urine RE/ME,ultrasound of abdomen.Ultrasound of abdomen can exclude many conditions.CT scan of abdomen may be advised after evaluation.Proper treatment depend upon findings.Get well soon.Take CareDr.Indu Bhushan"
},
{
"id": 65482,
"tgt": "What causes knots on thighs with fibromyalgia?",
"src": "Patient: I can run my hands down my thighs when laying on my side and it is very very tender and I feel hard like knots. I have been told I have fibromyalgia back in 1992.. Could these be nodules? At times it s so hard to get a good night sleep. YYYY@YYYY Doctor: Hi, dearI have gone through your question. I can understand your concern. You have fibromyalgia in past. Your lump may be due to some soft tissue tumor like lipoma, neurofibroma or dermatofibroma. You should go for biopsy of one of the lump. It will give you exact diagnosis. Then you should take treatment accordingly. Hope I have answered your question, if you have doubt then I will be happy to answer. Thanks for using health care magic. Wish you a very good health."
},
{
"id": 99140,
"tgt": "What causes itching rash on hand, face, arm and foot?",
"src": "Patient: I have some kind of rash that has started on my hand that is now spread to my face, arm, foot and not trying to sound gross but butt hole and in my mouth also. It itches like crazy and looks like little bubbles or pores filled with water. what could it be? Doctor: It is clear case of some kind of allergy and infection following some material attack, food or drug intake that has crashed the body. But filled with water is more suggestive of infective vescicles lime. You haven't mentioned changes in temperature, days taken to spread and any blood work out.Better to meet a doctor near you for physical examination and blood work.Hope it helps you."
},
{
"id": 40083,
"tgt": "What causes HPV when having single partner?",
"src": "Patient: I am a 56 year old woman and just had my first abnormal pap smear showing I have HPV. I have had ONE partner my entire life, my husband of 30 years. My husband has had only ONE partner (me) in the 30 years we have been married. How could I have contracted this virus? Doctor: Hi,Welcome to HCM.HPV is human papilloma virus which is responsible for warts in different areas of the body. Plane warts causing HPV can be spread by using same towels, close contact and fomites. Some HPV which usually are high risk and cause genital warts are spread through sexual route. These infections sometimes get resolved spontaneously without any symptoms. I suggest you get yourself screened for high risk HPV which is 16, 18 types. Otherwise no need to worry.Thanks."
},
{
"id": 126075,
"tgt": "Suggest an effective alternative for Flexeril",
"src": "Patient: I have been on flexiril for at least 10 yes. I asked my primary if there is anoer type of muscle relaxer that might help,since it doesn t seem to work effectively. He said no,its the best for you. Even with my lyrics and tramadol, I get no relief from the tight neck and shoulder muscles. Any suggestions? I have no insurance or income,so getting help is past impossible Doctor: Hi, Flexeril is a brand name of Cyclobenzaprine. You can try alternatives like Baclofen which is quite cheap and effective. Hope I have answered your query. Let me know if I can assist you further. Regards, Dr. Shinas Hussain, General & Family Physician"
},
{
"id": 21044,
"tgt": "What causes fluctuations in BP with trembling in legs and sleeplessness?",
"src": "Patient: Respected Doctor, I am Mr Sakti Pada Chakraborty ( 43 ) ,a teacher by profession,hypertensive under Concor.Cor 2.5 .Presently since last Tuesday feeling unwell .I feel very bad at the office hour when I get ready for the Office. I have consulted with a local physician ( an MD in Physiology) who adviced for lipid profile and other test which are as follows HB= 13.30, FBS= 99.90 ,Choles= 187 , LDL= 119.4 HDL=43.Creatine=1, TSH= 1.73 ,Serum Uric Acid= 4.70 On those findings, I am adviced for the following medicine: Rx 1. Cap.DiavitPlus 30X1 at night ( for my merve complication) 2. Tab. Rozustat 5mg 30x1 ( for LDL ) Furthermore he has asked me to take a stock of the progression. In necessary he will change the Concor Cor 2.5 which I had discontinued and may not functioning at this hour and thereby I am not tolerating. But for the time being I feel very bad at the peak hour of my movement at the working place and seems to me that BP gets high. But I feel better at the afternoon. At the office hour my legs, hand tremble just like a neuro/ nerve patient.At night sleep impaired/ disturbed by bad dreams and urineflow around 3.00am or like. In such a situation, I shall feel much obliged if you kindly advice me what to do. I hail from AGARTALA :TRIPURA:INDIA Sincerely Sakti Pada Chakraborty YYYY@YYYY 0000 Doctor: Dear Sir,you have really described your symptoms quite well which makes it more easier on my part to judge. As far as your reports are concerned they seem to be perfectly alright.whatever you are experiencing is probably because of your high stress levels possibly at your work place. you can try using Nexito 5 mg at night which would keep your nerves cool.so the apprehension would be less at the same time you start taking propranolol 10mg thrice daily which will take care of the tremors"
},
{
"id": 43850,
"tgt": "Trying to concieve. Semeen analysis shows non-motile 95%, abnormal sperm 100%. Solution?",
"src": "Patient: Good day Sir, I have two issues. My last issue was around 1998. Since then I have been trying but no any issue up till now. I tried so many natural solution. Last week week I did another semenaayis the result go does, sperm count 16.0 million: sluggish 5%: non motality 95%: and abnormal sperm 100% is there any solution to my problem. Doctor: Hi, Simplest solution for conception for you is to do IVF/ICSI with good sperm selection either with pICSI OR IMSI. Wish you good health."
},
{
"id": 69758,
"tgt": "What could be huge,red lump in the middle of the chest?",
"src": "Patient: I have a blood colored lump about the size and shape of an egg in the middle of my chest. It looks like a large blood filled bump. The lump isn't sore but I can feel my clothes pressed against it so I am always aware that it is there and it doesn't feel normal. Any idea what it could be? Doctor: Hi.The lump as you have described and in the center of the chest can be a scar.. so classical only on the chest.Get a consultation of a Doctor , if this is a scar you can get it injected as surgery has not so much of good results.. There is always a chance of recurrence."
},
{
"id": 223535,
"tgt": "Why am I bleeding since I have taken birth control?",
"src": "Patient: Hi, I have a question about my birth control pill. I just started a low hormone pill exacrly three weeks ago after coming of the patch. I have not stopped bleeding since. Is there any way to stoo this? Whats going on? How long will it last? Please help. /: Doctor: Hi, I think you can take some medicines like Tranexamic acid to stop the bleeding. Consult a gynecologist. This is a direct effect of low dose hormonal pill. So, don't worry about it. It will stop by taking medicines. Hope I have answered your question. Regards Dr khushboo"
},
{
"id": 170189,
"tgt": "What causes sore nose?",
"src": "Patient: my son is 5 months old and hes got a sore nose and when he lays down he sounds bunged up but his nose doesnt seem runny. he also sneezes alot but has done since birth and no more now than usual. his nose the last few days has been crusty and sore to touch infact he wont let me go near it. its started leeding a little too why? Doctor: Hi, Welcome to HCM. I have read your question in detail and I know that you are very concerned about your child but don't worry. Sore nose with crusts in a 5 month baby occurs due to dryness usually. In my opinion, you should use normal saline nasal drops 1 drop in each nostril three times a day for 3 days. It will help in correcting sore nose in baby. I hope this will help you. Wishing your child good health. Take care."
},
{
"id": 191673,
"tgt": "What causes frequent urination in a diabetic?",
"src": "Patient: I have been urinating extrenely frequently and urgently sometimes soiling my pants a bit. I also have been very tired, and unable to quench my thirst. i have looked up symptons and it seemed that it could be diabetes. I havent seen a doctor but i bought a hone blood sugar kit and it reads 508 mg/dl Doctor: Thank you for your question.The symptoms you mentioned (increased frequency of urination, tiredness, thirs etc.) can be due to diabetes as you say and also may be due to some other causes. You checked your blood sugar and found out that it was 508 mg/dl, which is rather higher than normal.But to confirm Diabetes, you should have specific blood test, HbA1c check. Fasting blood sugar (after no calories for 8 hours) which should be 80 _ 108 mg/dl. You should have a good and balanced diet plan with low carbohydrate, and more vegetables and low-fat meat. Regular exercises such as walking, jogging is advisable. The further specific medications and treatment will be better consulted after your HbA1c level result has attained. Thank you.Best of luck and have a nice day."
},
{
"id": 68913,
"tgt": "What is the lump on my son's neck?",
"src": "Patient: my son has a lump in his neck, thyroid tests clear. He had a biopsy yesterday and we now wait for results. The lump sometimes reduces in size - today we thought it had disappeared but now it is back. It only hurts because it is pressing on his throat. We are worried it is thyroid cancer. His voice is ok (though he shouts as his dog on his farm and this causes hoarseness!!!). He has had a very large amount of stress recently and has lost weight - could the lump be linked to stress? Doctor: welcome to Health care magic.1.The different things can arise are thyroglosal cyst, brachealcleft cyst,lymphocele, and a lipoma.2.I this case i would get an ultrasound neck done and see the content and extension of the lesion and come to a conclusion.3.As you have already got a sample taken - wait for the result.4.And after that needful action can be taken.Anything to ask ? do not hesitate. Thank you."
},
{
"id": 214330,
"tgt": "Suggest natural remedy for indigestion",
"src": "Patient: I am using Omezol 20mg which contains Tartrazine for indigestion. The relief I have had is wonderful ..... but I want to know if there is a Natural alternative that I could try that would or could be just as effective, or another drug that does not have tartrazine in it. Doctor: hello madam,thanks a lot for your query on HCM,nice to see you switching to natural therapy for your life style,..there is no substitute for it...do follow a healthy lifestyle...do go for regular exercises with atleast 30 minutes of brisk walking daily...stick to a regular timetable for food...eat adequately at adequate time..dont over burden or undereat at any time...eat good food and dont heed to fast foodtry to stick to these tips..do be positive...things will improve gradually..."
},
{
"id": 124040,
"tgt": "Suggest dosage of voveran 50 for tail bone pain",
"src": "Patient: i am suffering from tail bone pain since last evening. and i am having trouble to sit for a long time. i had this problem before and the pain was stopped by voveran 50. is not it harmful to take much painkiller? if not, then in what quantity may i take? Doctor: Hello, As you have a pain in tail bone I will advice you to either take an x-ray/CT/MRI. As the pain coming back to back is indicative of having inflammation at the bone leading to recurrence of pain. Taking painkillers for short duration is not harmful but taking it for long run may damage the kidneys leading to other complications. With the tail bone pain mentioned in the history, I suspect you have weakness or the hip muscles which is leading to excessive pressure over the bone and causing inflammation. Also I suspect there will be a week spinal muscles due to week the load is becoming excessive which shouldn't happen. Doing exercises for hip muscles and core stability exercise should help you in long run. If your job is designed to sit and work for longer hours I will advice you to buy one piles pillow and sit over it when working. This will discard the excessive weight transferring to bone and make it more into momentum. In my clinical practice of over 12 years most cases who have tail bone pain have responded well to exercise and precautionary measures. Hope I have answered your query. Let me know if I can assist you further. Regards, Jay Indravadan Patel, Physical Therapist or Physiotherapist"
},
{
"id": 50910,
"tgt": "Diabetic, kidneys failed, low hemoglobin, increase in urea in blood. Dialysis recommended. Suggestions?",
"src": "Patient: Hello sir my aunt is 67yr old... she had diabetes which she did not care to treat and now both her kidneys have failed...her hemoglobin count is 5..and there is a rapid increase in the urea in her blood.. she was recommended dialysis ..but she is refusing as she thinks that she cannot? bear the pain during dialysis... can u please give a solution to this problem Doctor: Hello, Healthy kidneys clean the blood by filtering out extra water and wastes. They also make hormones that keep your bones strong and blood healthy. When both of your kidneys fail, your body holds fluid. Your blood pressure rises. \u00a0This condition is called uremia \u00a0Your body doesn't make enough red blood cells. You develop fatigue, nausea, and loss of appetite. Untreated uremia may lead to seizures or coma and will ultimately result in death. If your kidneys stop working completely, you will need to undergo dialysis or kidney transplantation. Your aunt has to go for dialysis at the moment and ultimately she has to opt for kidney transplantation,as dialysis can't go permanently. Thanks"
},
{
"id": 194523,
"tgt": "What causes penile bleeding after masturbation?",
"src": "Patient: xThis may sound stupid, however, I am extremely worried. I am a teenager and as one does, I decided to 'experiment' with various methods of masturbation. However, today once I had finished I noticed there was bleeding on the head of the penis, not out of the urethra. The frenulum was bleeding as were a part of the head of the penis, (the glands from what I've gathered from diagrams). Straight after noticing it I washed the cut with water, before applying both soap and water. In total I've washed it around 4 times today, yet when I pulled down my foreskin earlier there was a goo on both of the bled areas (puss?). What should I do, I am getting seriously worried, should I see a doctor, is it infected or is it a mere cut? Thanks Doctor: Hi, Your foreskin cut is not clear as per mentioned history. You are suspecting some pus or collected dirty material there. So you need to rule out infection of that wound. For that, you can post a photo of the lesion or consult Urologist nearby. You can apply topical antibiotic cream over that. Hope I have answered your query. Let me know if I can assist you further. Take care Regards, Dr Parth Goswami, General & Family Physician"
},
{
"id": 58321,
"tgt": "Fatty liver. What kind of diet and treatment should be followed?",
"src": "Patient: My doctor says I have a fatty liver. She only told me to not drink alcohol, which I don't. I have never been a big alcohol drinker. It used to be occasional intervals, like weddings or parties. But I haven't had anything to drink in a long while. I just don't enjoy it. So, my question is what do I do about a fatty liver. Is there something I should be taking? Doctor: hello dearthanks for your query at HCMit might be hard for you quit alcohol, but what ever little amount your taking might damage liver that is already in course of shut down.the most important thing taht you can do now is quit alcohol, attend alcohol withdrawl classes to quit it.1 take plenty of water and 2 fresh fruit juices3 avoid junk foods4 avoid high protein diets.wishing for early recoveryregards"
},
{
"id": 8770,
"tgt": "How long is it safe to use depiwhite cream containing hydroquinone ?",
"src": "Patient: Hi, I was directed by my doctor a year ago to use depiwhite cream, which contains hydroquinone . I used it for 3 weeks and stopped as the pigmentation faded. I started using it again only 2 days ago, application all over my face , as I developed an uneven complexion due to sunburns. I use it overnight. I want to know if this cream has a high percentage of hydroquinone and how long can I use it for and if it has any side effects on the long run Doctor: Hi....dear weaam., Thanks for choosing HCM., DEPIWHITE CREAM...it contains hydroquine which can irritate skin.., in the long run making it more sensitive and there may be side effects like redness.., and drying with using depiwhite..., Otherwise theres no problem...,if you have been using depiwhite for a month.., or more and have noticed no side effects. .., In this case you may continue using it but give your skin a break.....! so use depiwhite for 3 weeks , then 1 week don't use it..., Like this...don't get any problem and harm with using of DEPIWHITE.., thanQ"
},
{
"id": 1223,
"tgt": "Is there a higher pregnancy chance if certain blood type donor is used?",
"src": "Patient: Is there a higher pregnancy chance if I use a certain blood type donor? My main concern is with the baby. Mine is A+ so not a complicated one. Probably makes no difference but knowing this journey maybe tricky, any tips will be of great help. many thanks in advance! Doctor: Hi.There is no such association ma'am. No particular blood type or group has a higher or lower chance of conception or fertility.Best wishes."
},
{
"id": 112007,
"tgt": "Bruising and swelling in lower back after falling. Stomach sore with hot sensation. Taking nurofen and paracetamol",
"src": "Patient: Fell off a ladder , Friday evening , bruising and swelling to lower back ! Stomach feels sore and funny hot sensation in the tummy, feel like I have been kicked in he tummy ? Very stiff back , ? Been taking nurofen and paracetamol , should I get checked out by my gp ? Doctor: Hi there. According to me you should get an Xray of the back done immediately .Nurofen is mostly Ibuprofen and should be avoided if you are having stomach problems .In fact the stomach soreness might be secondary to Nurofen ingestion. Treatment :1. Complete bed rest2. Analgesic like Tramodol as it has negligible gastric side effects3. Muscle relaxant like Thiocolchiside4. Local analgesic gel massage5. Lumbosacral corset to be worn while you are not lying down. Further treatment will depend on X ray report.I hope my advice has helpedGood luck"
},
{
"id": 94049,
"tgt": "Frequent abdominal cramps, bloating, diarrhoea. Pain radiating to right side, acid reflux. IBS?",
"src": "Patient: Hi, Starting in September 2012, I have experienced frequent abdominal cramps and bloating with bouts of diarrhea . The pain will shift from the left to the right side of the abdomen and will be accompanied by acid reflux . My GP has said that the intestines felt inflamed, however I have taken a number of blood tests that came up negative for Celiac s and Anemia. I m still waiting to see a Gastroenterologist . Is this just irritable bowel syndrome, or something else? Any input is greatly appreciated Doctor: Hi welcome to Health care magic forum. Thanks for calling H.C.M.F. As you describe you are having abdominal cramping, bloating, and diarrhoea since september 2012. As it is a chronic disease, it appears to be irritable bowel syndrome. Other possibilities are amoebic dysentry, or bacterial infection of the gut, with out proper treatment. Of course as you have appointment to consult gastroenterologist, you will come to know the diagnosis of the condition. You may have to go for colonoscopy besides other routine tests for confirmation . Wishing for a quick and comlete recovery . Best regards."
},
{
"id": 197656,
"tgt": "How to heal fixed drug eruption on the glans of penis?",
"src": "Patient: I had a fixed drug eruption on the glans of my penis and had this formally diagnosed, this was over a week ago and it hasn t healed as sadly I m on holiday, moving around and I seem to be rubbing off any healing, what should I do, in detail, to help it heal? Doctor: if that eruption is not causing any pain or itching pks do nit disturb it as it may turn worse. rather keep it as it is"
},
{
"id": 86362,
"tgt": "What causes abdominal pain and diarrhea on daily basis?",
"src": "Patient: Yes. I am experiencing abdominal pain, diarrhea on a daily basis. Originally, it was just in the morning, once or twice bowel movements. Then, four times a morning; now all day and evening. I cannot digest anything (liquid or solid) without incident. I am 76 years of age and a female. Doctor: Hi. Thanks for your query.Read and understood your concerns, regarding increased frequency of passing motion and inability to digest anything.I would advise you in such a case the following.First of all, get the stool tests done.Blood tests including that for sugar thyroid function testsWhen required a course of an antibiotic, metronidazole, probiotics ,symptomatic and supportive treatment can be done.If no complete relief, further tests of Colonoscopy and CT scan may be needed to find the cause and get treated accordingly."
},
{
"id": 162983,
"tgt": "Why does my 6 years old wake up after sleeping crying and anxious?",
"src": "Patient: I have a six year old daughter who lately has been waking up in approx 1 hour to 1 1/2 hr after going to sleep. when she wakes up, she gets out of bed and sees me but does not make sense, she starts to cry a little and appears to be anxious..I believe it s appears to be sleepwalking but this happens at least 3-4 times a week. and also she is sweating. All I have done is assure her everything is okay and bring her to her bed....should I really be concerned about this and get professional help Doctor: Hello and Welcome to \u2018Ask A Doctor\u2019 service. I have reviewed your query and here is my advice. Well maybe are just bed dreams but when this happen frequently than the best solution is to ask for professional help. Hope I have answered your query. Let me know if I can assist you further."
},
{
"id": 199180,
"tgt": "Suggest treatment for premature ejaculation and erectile dysfunction",
"src": "Patient: Sir I'm 32 yr old .I started masterbating wen I was only 15, twice thrice a day which made very weak.in my thoughts I could get erection but wen near my partner no errection ,only fluids come out of my penis.then I stopped masterbating .but wen I materbate nowadays at night I can't fell sleep n also their mild headache involving my eyes also.now in 32 n I'm engaged wen I'm near my fiance I could not get hard erection . Doctor: DearWe understand your concernsI went through your details.Your current problem is due to anxiety related to ill-information or wrong knowledge. I assure you masturbation is safe and not unhealthy. But anxiety related to masturbation can provide you with mental stress and could give you performance anxiety. You can marry and you will not have any erection problems. You should consult a psychologist for sexology related counseling. If you require more of my help in this aspect, please use this URL. http://goo.gl/aYW2pR. Make sure that you include every minute details possible. Hope this answers your query. Available for further clarifications.Good luck."
},
{
"id": 74667,
"tgt": "What causes joint pain while using AKT 3 for TB infection?",
"src": "Patient: Hi, I am suffering from TB infection.Doctor siad to take AKT4 for 2 months then AKT2 for 4months.After 6 months when i get tested again TB infection is positive.Now doctor said to take AKT3. I am taking AKT3 since 10 days back and felt joint paint in my knee.Is this dangerous?What are symptoms to know that TB infection is over? Doctor: Hello,Joint pains are mainly due to Rifampicin tablet in AKT3. Symptoms to know that you are free from TB are improved weight, improved appetite, no fever, cough, and difficulty in breathing.Hope I have answered your query. Let me know if I can assist you further.Regards,Dr. Siva Kumar Reddy"
},
{
"id": 24867,
"tgt": "Suggest remedy for dizziness and collapsing after taking bp medicine",
"src": "Patient: hi i am kundan age 32 weight 90 approx height 5.\"10\" suffering from anxiety attack were have the feeling that i will collapse since last 4 years but doing lots of ECG and other test but its normal only trychardia detected since last 15 days i develop the blood pressure of 130/100 doctors precribed me tazloc or telpres h 40mg and etizolam at bed time but while i take the bp medicine i feel like driziness,collapsing it will happen till 1 hrs then i get fine what s the prblem kindly suggest Doctor: Thanks for your question on Healthcare Magic. I can understand your concern. No need to start antihypertensive drug in your case. Your slightly elevated blood pressure, tachycardia etc are mostly due to uncontrolled stress and anxiety. So we should first treat this stress and anxiety. So better to consult psychiatrist and get done counselling sessions. Try to identify stressor in your life and start working on it's solution. Counselling plays very important role along with anxiolytic drugs for control of stress and anxiety. Don't worry, you will be alright. Consult psychiatrist and discuss all these. Hope I have solved your query. I will be happy to help you further. Wish you good health. Thanks."
},
{
"id": 104006,
"tgt": "Had inflammation of lungs. Noticed flu virus during washing. Taken fostair steroid and ventolin inhalers and doxycline. Guide",
"src": "Patient: Hi there, I have gone downhill since before 2008. The consultant advised me it was the bird feathers on my elderly parakeet that caused scarring of the lungs. (re-occurrent chest infections) New consultant, 2012, does not agree with allergy and thinks it is something else. I have just recovered from pneumonia (hospitalised) and tamiflu, they found this in one of my lungs, after bronchoscopy to find cause of inflammation in the lungs. I was too ill for them to do biopsy, but they performed washings...that is how they came across the flu virus. Have taken the emergency meds for that now and hopefully on the road to recovery. I have fostair steroid inhaler and ventolin inhalers and doxycline for a month. I have a consultant appointment in early May. I do trust my consultant and he wants the answer to what is attacking my lungs as much as me. I hate inhalers though, especially the steroid one, as have osteroporosis...on a natural diet and exercise programme for this. Any advice you can give me? Doctor: Hello,Welcome to Healthcare Magic.I am not certain what the diagnosis is at this stage: posibilities (1) central bronchectasis, with asthma and aspergillus sensitivity (not certain if this test was done) is ABPA (Allergic Bronchopulmonary Aspergillosis); (2) or depending on the nature of your work, or the number of avian species at home it is hypersensitivity pneumontis.You need the following tests:1. HRCT Chest2. Precipitins3. or Aspergillus IgG4. Total IgE levels5. Skin prick test to aspergillus species6. Pulmonary function testsYou must get these tests done when you are well from this recent infection.A combination of steroids and antifungals could be tried if it is hypersensitivity pneumonitis.Best Wishes."
},
{
"id": 135208,
"tgt": "Why my shoulder pops when I take deep breath?",
"src": "Patient: For about a month now my right shoulder tends to pop when I breathe in deeply when I lie down on my back, and recently, when I am just sitting down upright. It isn t painful at all, but it is somewhat uncomfortable and annoying as it seems to be happening more frequently lately. I m thinking that it may be due to my posture but I am uncertain. Please help me figure out whats wrong with my shoulder! Doctor: Hello,i have studied your case. if popping means that there is cracking sound then it is not a serious issue and you can ignore it. If there is sub lunation of the shoulder then it can be due to weak muscles and it needs proper checkup for this you need to take ultrasound of the shoulder to see if there is any abnormal tendon moving.I hope this answer will be useful for you.Let me know if there is any other followup questions.thanks"
},
{
"id": 189671,
"tgt": "
Have dentures with frequent sore mouth and ulcers, used Colgate peroxyl mouth sore rinse. Solution?
",
"src": "Patient: Hi I m Elaine, 63 years old I ve been wearing dentures about 10 yrs without success. I have always had problems with the inside of my mouth. As a child I could not even have bubble gum for the ulcers it would cause inside my mouth. So I have grown use to having Sore Mouth I have a huge or what feels like a huge place in the roof of my mouth. As you well know, any foreign object in your mouth feels lots larger than actual. I bought some mouth wash after salt water didn t help, Colgate Peroxyl Mouth sore Rinse, set my mouth a blaze! I continued to use it even up to yesterday, no pain no gain, I thought. This morning I wake up with a full blown sore mouth and a perfect out line of my dentures inside my mouth. My mouth looks like I have the Thrush. I haven t worn them in about 48 hrs. At this point I don t know what to do? Call my MD and ask for something for the thrush? Doctor: Hi, Thanks for asking the query , Ulcers and sore mouth can be because of irritation by the denture , candidal infections , use of certain medications , nutritional deficiencies . Oral thrush is due to candidal infection in which there is pain and difficulty in swallowing, a feeling that the food get stuck in the throat and the mid chest area and fever. Visit the dentist and get the checkup done . Microscopic examination will confirm the diagnosis. Maintain a good orl hygiene . Take vitamin suplements. Stop medications causing allergy. Quit smoking habit , if present Restrict fermented foods which encourage candida growth. Replace the denture if causing problem . Hope this helps out . Regards..."
},
{
"id": 209629,
"tgt": "Can stress/grief cause symptoms like loss of hair with vomitting?",
"src": "Patient: I just recently lost my man (an Emergency Medicine doctor) in November....since then I have been losing my hair copiously, vomiting in the middle of the night. Pain in the mid upper quadrant, and horrible itching all over. Is this stress and grief related or a medical issue? Doctor: Hello,Thanks for choosing health care magic for posting your query.I have gone through your question in detail and I can understand what you are going through.These could be psychological. As they are starting immediately after the stressful event,. Have a course of anti anxiety to help in improving anxiety symptoms.Hope I am able to answer your concerns.If you have any further query, I would be glad to help you.In future if you wish to contact me directly, you can use the below mentioned link:bit.ly/dr-srikanth-reddy\u00a0\u00a0\u00a0\u00a0\u00a0\u00a0\u00a0\u00a0\u00a0\u00a0\u00a0\u00a0\u00a0\u00a0\u00a0\u00a0\u00a0\u00a0\u00a0\u00a0\u00a0\u00a0\u00a0\u00a0\u00a0\u00a0\u00a0\u00a0\u00a0\u00a0\u00a0\u00a0\u00a0\u00a0\u00a0\u00a0\u00a0\u00a0\u00a0\u00a0"
},
{
"id": 143515,
"tgt": "What causes painful knot on lower part of right side of head?",
"src": "Patient: I recently came to the ER because I had what I thought was a crook in my neck. They gave me muscle relaxers and pain meds because the swelling in my shoulders were noticeable and he said since it s soft tissue no X-ray was needed. Well I went home and the muscle relaxers helped my shoulders but I now have a knot on my lower part on my head on the right side and it aches from time to time but hurts when I turn to the right. What could it be? Should I go back to the ER? I have been lightheaded and weak, just want to lay down and that s far from the schedule and lifestyle habit I keep. I m always on the go and full of energy. Doctor: Hello, it is a common dysfunction in the muscle most the people have it due to the erratic lifestyle. we use a lot of electronic gadgets like phone, iPads , laptops or TV. we generally tend to forget that we are not in the right posture, which causes imbalance in your cervical and lumbar muscles, which in turn cause pain, numbness or any other symptom. if you can correct your posture in all your activities it can get rid of such problems. would like to advise you meet a Physical therapist who can solve your problems and get back to your healthy and active lifestyle.thank you"
},
{
"id": 27873,
"tgt": "What causes numbness in back of head, spinning, nausea when on high BP medication?",
"src": "Patient: The back of my head feels numb, the last few days I felt dizzy, sometimes the room fills like its moving and when driving I feel nauseated. I m taking prozac, high blood pressure medicine, and I have been very emotional the last few days. I took tylenol but no improvement. What should I do? Doctor: When u feel dizzy, have u checked ur bp? If it is normal, then need to examine to find out other causes of vertigo."
},
{
"id": 59009,
"tgt": "Suffer from abdominal pain, distention and constipation. Have fatty liver. Lack of appetite. Any suggestions?",
"src": "Patient: Hi I suffer from abdominal pain, distention and constipation. I have been told that I have a fatty liver. I have also had a history of vomiting, lack of appetite. Today I have eaten two mince pockets, three cornchips and a muffin. I am so bloated I could not even think of eating again. I am so uncomfortable. I have had to force myself to eat. It appears that there could be so many diagnoses for my symptoms. I just feel like not eating again. I am taking vitamin b tablets for my liver as per my doctors instructions. Doctor: Hi,Welcome to HCM,With available information you have pain in abdomen, and distension and also known to have Fatty liver.Fatty liver could be Alcoholic Liver Disease (ALD) or Nonalcoholic fatty liver disease (NAFLD), depending on you take alcohol or not.Other factors which contribute are diabetes and Obesity, with metabolic syndrome.High cholesterol and Triglyceride level can also cause fatty liver.Many drugs are involved in many patients common one are corticosteroids, antiviral, amiodarone .You should go for complete check up like complete blood count, Liver function tests, kidney function tests, Lipid profile, Serum Amylase and Lipase and Endoscopy to find if anything is detected.Treatment depends on cause would involve Proton pump inhibitors, Prokinetics, Statin, a weight loss programme and complete stoppage of alcohol and smoking if any.Diet modification taking food with low glycemic index and protein and lot of green vegetables and avoiding simple sugars animal fats and regular physical exercise as to treat such an issue multi pronged approach is essential for success.Hope this information is useful to you.Take care Good Luck.Dr.Akhilesh Dubey."
},
{
"id": 87932,
"tgt": "What causes numbness of limbs with abdominal pain?",
"src": "Patient: for the last couple of years i've been getting extremely light-headed sometimes with black spots flickering over my eyes. also my limbs sometimes go numb and give out. especially my legs. I also get extreme abdominal pain regularly. also have chest pains sometimes. help? my doctors won't listen. Doctor: Hi.Thanks for your query and an elucidate history. Your problem is for a long time- couple of years and you are getting light-headed with blackening, limbs going numb and extreme pain in abdomen. You problem may be due to severe form of colitis, wherein you get deficiencies of many vitamins, minerals and trace elements. I would advise you to go for colonoscopy and CT scan of the abdomen in addition to the routine blood, urine and stool test.The proper diagnosis, its treatment and symptomatic treatment for your symptoms with therapeutic dosages of multivitamins, minerals and trace elements would give you the best possible results."
},
{
"id": 151138,
"tgt": "Problem with supply of blood from heart to brain. How to treat it?",
"src": "Patient: The patient First Had a) After 13 days of the occurrence of This problems Status epilepticus occurred and it was not able to be controlled for First 4 days . After that it was controlled but the patient need Seizure Detection, but he is still often having this problem which is severe in condition. After that he had problem in supply of blood from heart to Brain , this happens often, even in a minute it can happen 20 times. Can You help us in this problem Doctor: Hi, Thank you for posting your query. Epilepsy is caused due to abnormal electrical discharges in the brain. Anti-epileptic drugs are necessary to control these abnormal discharges, so as to stop the epileptc fits. There are several good medications to control the seizures, such as valproate, carbamazepine, levetiracetam, etc. In addition, if there is problem in blood supply to brain, drugs such as aspirin may be required. Please discuss these issues with your neurologist. Best wishes, Dr Sudhir Kumar MD DM (Neurology)"
},
{
"id": 68514,
"tgt": "Suggest treatment for persistent bump on right side of neck",
"src": "Patient: For almost 7 years, I have had a bump on the right side of my neck that is probably a lymph node. It almost never changes, and even gets bigger when I am sick and smaller again when the infections resolve. Is this something I should worry about? A new doctor I have never seen before basically seemed worried about it and sent me for a CT which I am waiting still to schedule. Doctor: Welcome to Health care magic.1.Lymph nodal enlargement is seen in case of any infection / inflammation, any benign and malignancy causes.2.As you giving history of enlargement after any infection - so its of infective aetiology ( most possible cause).3.The doctor might be suspecting / want to rule out malignancy.4.A contrast enhanced computer tomography will help in identifying the mass lesions / malignancies.5.An FNAC - fine needle aspiration cytology might be needed for further evaluation. Good luck.Hope it helps you. Wish you a good health.Anything to ask ? do not hesitate. Thank you."
},
{
"id": 7656,
"tgt": "Acne breakouts on jawline, temples, cystic in nature. On vitex, burdock root. Hysterectomy done, ovaries retained. Treatment?",
"src": "Patient: Hi, I had a full hysterectomy at the end of February, I kept my ovaries...i m now finding that my jawline and temples are breaking out--I ve never experienced acne before. I ve always had good skin - now i have cystic acne ....are there any non-prescription remedies to help. I m trying Vitex and Burdock root...is there anything I should be doing to the cysts directly? Doctor: Hello. Thanks for writing to us. The cystic acne are commonly seen due to hormonal disturbances in the body. The main treatment involves systemic antibiotics and retinoids. Both these are prescription only drugs. You need to consult a dermatologist for the same. I hope this information has been both informative and helpful for you. Regards, Dr. Praveen Tayal drtayal72@gmail.com"
},
{
"id": 97311,
"tgt": "How to treat the lumps caused by a fall and the related after effect of it?",
"src": "Patient: Wife just fell down while walking. Landed on knees first, continued falling flat on her face. Had lumps quickly form above & below her right eye. We are applying an ice pack every half hour for 5 minutes as treatment so far. Possible carpet burn to ear side of right eye. She has no pain...yet. Doctor: Hi and welcome to HCM. Thanks for the query.There is no some treatment which could facilitate this healing process but time. ice packs can help to relive pain and decrease heamtoma but she needs to rest and wait for several days till bruisings subside. Wish you good health. Regards"
},
{
"id": 105260,
"tgt": "Sneezing, dry cough. Why troubled by blisters? Why trouble swallowing?",
"src": "Patient: Hi , I had a little sneezing and cough mostly dry cough . Cold is gone automatically but it feels like throat has some blisters i can feel it while swallowing and i feel pain and throat feels very dry. do not have fever but little throat and pain and burning in food canal. i am planning to gargle with saline water is it ok or should i take any medicine i did take benadryl twice yesterday and day before yesterday night. Doctor: all allergies sneeze lead to wheeze postnasal discharge of allergic sinuses present thisd take metrogyl 200 mgm bd tab singular 10 mgm bd syp tossex 1/2 tsf bd syp mucaine gel 2 tsf tds do 10 days apply neosporin h eye ointment in nose get xray pns waters viewand treat"
},
{
"id": 37266,
"tgt": "What is the cause and treatment for low grade fever?",
"src": "Patient: I think I may have drank some lemonade from a thermos that was washed but sitting in a cabinet and not thouroghly rinsed yesterday. I now have a low grade fever 99.3 (my normal emp usually runs below 98.6 sometimes), tired and have a bit of a sore throat. Would that be the cause and how to remedy? Doctor: Hi,It seems that you might be having upper respiratory tract infection giving this problem.Take some pain reliever or analgesic for a day or two.After 2-3 days if you are having same problem, you might require one course of antibiotic medicine.Avoid cold drinks and junk food.Ok and take care."
},
{
"id": 119302,
"tgt": "Taking warfarin for Factor V Leiden blood clotting disorder. Is it possible for me to donate blood, platelets or bone marrow?",
"src": "Patient: I have Factor V Leiden a inherited clotting disorder. I am currently taking warfarin (10mg/day) and will be for life. Based on my readings, I understand I am restricted from donating blood . Am I also restricted from donating platlets and bone marrow? I could not find a definitive answer on American Red Cross website.Thank you! Doctor: Hello, Inherited disorder of the type you are having are caused by the changes in the DNA of cells. Every cell in our body contains the same set of genes and any cell that you donate will carry that defect in gene. So bone marrow donation can not be done. As you are on regular warfarin, you can not donate platelets also. However your concern and interest in going all out to donate these precious products is admirable."
},
{
"id": 51452,
"tgt": "Diagnosed with renal colic, have a sedentary working style. Best remedy and preventive measures?",
"src": "Patient: I am a 31-year-old male, sedentary worker diagnosed with renal colic a month ago. What is the best remedy for renal colic and how to take preventive measures? Doctor: Hi there. Thank you for posting your question with Healthcaremagic. I will do my best to help you. The prevention of kidney stone is a complex subject and there is loads of good information on this on the Internet. I would recommend the Mayo Clinic's web site as an excellent place to start. Also, you need to discuss this with you own doctor or urologist. First of all it depends on what type of stone is developing in your kidney or bladder. This has to be determined. If you can, develop the habit of urinating through a sieve just in case you can catch a small stone for your doctor to send for analysis. Most stones are calcium based. Dietary restrictions in calcium is however no longer recommended. This is because it does not seem to work and, in any case, calcium is an important mineral for proper bodily functioning. Some stones are composed of uric acid in which case allopurinol, to lower uric acid levels, may be prescribed. All recurrence rate of all stones can be reduced by increasing daily fluid intake. The cheapest way to achieve this is by drinking lots of water. The aim should be to increase fluid intake to the point where you are passing at least four pints of urine every day. I hope that I have answered your questions at least to some extent and been of some help. If you have anymore questions please contact me again anytime. Dr Andrew Rynne. www.doctorrynne.com"
},
{
"id": 153551,
"tgt": "How should hands swelling and pain with breast rashes be treated?",
"src": "Patient: Hello, I am a breast cancer survivor, 3yrs out, with lymphadema in my left arm. I have had flair ups of the lymphadema before, but this is weird and extremely painful. Thursday night while driving home from work I realized I couldn't grip the steering wheel with my left hand. Got home, took my sleeve and glove off and my hand was painful and swollen. My pinky finger is a sausage, the joint where my pinky meets my hand is extremely painful, the palm of my hand is swollen, and my wrist is painful. Then today I am developing a rash from underneath my breast to my waist??? I was going to go to the emergency room, but doubted they would be able to do anything...any ideas? Thanks in advance... Doctor: Thanks for your question on Health Care Magic. I can understand your situation and problem. Since you are breast cancer survival patient, following are possible causes for your symptoms. 1. Lymphedema. Breast cancer is known to involve axillary lymphnodes. And it can cause obstruction to the lymph flow. And hence painful swelling of hand can be seen. 2. Cutaneous spread of breast cancer. Rashes on breast and painful edema of hand can be due to metastatic involvement of skin. 3. Cellulitis (skin infection). Cancer patients are having lower immunity. So skin infections are very common. So better to consult emergency room, discuss all these causes. First diagnose yourself and then start appropriate treatment. Hope I have solved your query. Wish you good health. Thanks."
},
{
"id": 110044,
"tgt": "Suggest remedy for persistent back pain",
"src": "Patient: Hello Doctor, I am suffering from spinal cord problem since 2 years. It started due to dry cough and also went to gym which has aggravated the problem. I had taken MRI scan and found that slip disc got bulged and it is touching the nerves which is causing pain in the right leg. I underwent ayurvedic treatment which gave relief for one and half year. Now for the past 6 months i am getting pain in my spinal cord area i.e., on my backbone. Can you suggest me the best remedy for this as I am not able to bear the pain. Thanks & Regards,Vinayak Doctor: HiThank you for asking HCM. Your problem is most likely due to intervertebral disc herniation. Usually most people get that in lumbo sacral region. For such condition in my patients I treat with NSAIDs like aceclofenac muscle relaxants like cyclobenzaprine and neurotropic drugs. I also send them to physiotherapy along with this. Lumbar supporting belts also is helpful. For some it will take 1 month to recover. For some it may take 3 to 6 months. If no improvement then a surgical correction is advised. Hope this may help you. Let me know if you have any further queries."
},
{
"id": 169313,
"tgt": "Suggest treatment for fever with loose motion and vomiting",
"src": "Patient: hi , my son has been unwell .started with fever on 14 night which lasted for 2 days along with loose motion n vomiting, vomiting lasted for 2 days . gave him emeset for vomiting and baby gesic for fever and started him on gnorm 3 times a day. got a microscopic stool test done and result showed 2-3 /hpf. it has been almost 7 days but his motion has still not got well. how long can the problem last? Doctor: Hi Dear,Welcome to HCM.Understanding your concern. As per your query your son is having symptoms of fever with loose motion and vomiting which could be due to gastroenteritis which is infection due to poor immunity of body. It is due to increased susceptibility of body of getting infected. Need not to worry. I would suggest you to do tepid sponging for immediate relief as you having high degree fever. Avoid oily and sharp food. Drink plenty of fluids. You should take small and frequent meals and avoid large meals intake. You should take proper antibiotic course in combination with Ibuprofen/ Acetaminophen. Visit pediatrician once and get it examined. You should get routine blood test done and start treatment after proper diagnosis. You should avoid intake of outside oily and fast food. Normally viral fever take one week duration to get recovered.Hope your concern has been resolved.Get Well Soon.Best Wishes,Dr. Harry Maheshwari"
},
{
"id": 204955,
"tgt": "Does a low dose of Effexor cause high blood sugar level?",
"src": "Patient: Hi, I was wondering if a low dose of Effexor (37.5 mg ) can raise your blood sugar? I have been on it for 17 years since I lost my son. I ve been taking the lowest dose for at least 13 years and I usually skip a day or 2 during the week. It is Effexor XR. Tried 6 or 7 times to get off, withdrawal side effects are horrible. Thanks, Charlie Doctor: HelloEffexor can cause rise in blood pressure sometime. But it does not cause increase in blood sugar levels.Consult your doctor to see if it is diabetes.Thanks."
},
{
"id": 205291,
"tgt": "What causes frequent mood swings, sleeplessness and headaches?",
"src": "Patient: I don t know where to begin, it feels so overwhelming to be talking to someone about what I m going through. I know I need to make an appointment with a doctor, but fear and lack of finances hold me back. I experience extreme lows in regards to my mood and often find myself crying over the least little thing. On the flip side of that coin, so to speak, it seems to take nothing to set off my temper. At the end of the day I cannot seem to be able to rest and sleep, and yet when I am in bed I want nothing more than to stay there. Sleep is a most elusive thing and I find myself tired quite often. The thought of being around people, especially groups of any size, sends my mind buzzing and my heart racing. I am experiencing more often pain across my lower back, sometimes it is nothing more than a twinge and other times is sharp and breath stealing. I suffer from acute prolonged pain in my right arm where I broke when I was a child. My appetite leaves much to be desired, usually eating once a day, and not very well when I do. I have headaches that last for days that make me have a sick feeling in my stomach and even more sensitive to light than normal. I have to force myself to take Tylenol for any reason. I do not do drugs or drink and am feeling at the end of my rope. Any help would be greatly appreciated. Doctor: My friend, you just have described every typical symptoms of a person suffering from depression. Depression is a syndrome that occurs mainly due to decrease in serotonin level in brain, which is a neurotransmitter. It results in decreased energy, increased fatigability, lack of interest, decrease urge for social interaction. The person finds no pleasure in any activities that he used to enjoy much previously. Sleep may increase/decrease, there is serious lack in self confidence, guilty feeling. Most of the time the person feels hopeless, helpless and worthless. Depression however is curable. There are numerous drugs available that balances the serotonin level. Alongside, therapy with an experienced psychologist always adds to the effect. I'll suggest you to immediately get in contact with a psychiatrist near you. Get well soon."
},
{
"id": 190792,
"tgt": "Wobbly upper front teeth due to injury, four teeth are loose, cant eat solid food",
"src": "Patient: My daughter is 4 years 4 months old. 2 months back she bumped in the centre table but there was no bleeding in her mouth . Later that day she told that her upper front tooth was wobbly. I went to the dentist . But they said keep an eye if color changes. It has been 2 months already the color has not changed but now all four teeth in the front of the upper gum are loose. Appointment is not available at the moment. The teeth are hurting because she is not able to eat even a biscuit. Just soft food. I am really worried. Is this some sort of deficiency? Please help me. Regards Doctor: hi. welcome to HCM of course everyone matter for their childrens. actually the dentist advised you for colour changes were meant that your daughter's tooth might have got infection aur may have lost its blood supply. but if for 2 months if there are no colour changes then it means there is nothing to worry about. now your concern about the loosening of the upper four gums is a natural process leading to the appearane of her permenant teeth. it is normal for this age and all her teeths will be replaced by the permenant ones. however she is in a growing age so she may need calcium supplement to enhance her growth process. meanwhile use soft diet which can readily swallow. thanks with regards dr.ishfaq"
},
{
"id": 191809,
"tgt": "What causes intense foot pain despite taking Lyrica?",
"src": "Patient: I am a 64 year old African American female. I have Type II diabetes, AIC 5.4. Why do I have so much pain and neuropathy in my feet. I take 450 mg. of Lyrica each day, which helps minimal. What s your advice? I thought that your answers were free. Sorry, I don t have the money to pay for your answers. Really? Doctor: Avoidance of neurotoxins (alcohol) and smoking, supplementation with vitamins for possible deficiencies (B12, folate), and symptomatic treatment are the mainstays of therapy of diabetic neuropathy. Chronic, painful diabetic neuropathy is difficult to treat but may respond to Lyrica ( gabapentin).If there is no response, it is reasonable to switch tp other agents like duloxetine approved by the U.S. Food and Drug Administration (FDA) for pain associated with diabetic neuropathy. However remain assured the pain of acute diabetic neuropathy may resolve over time.Regards Dr. T.K. Biswas M.D.Mumbai"
},
{
"id": 145275,
"tgt": "Suggest treatment for brain tumor",
"src": "Patient: Yes Thank You... My sister was just diagnosed with a Brain tumor...right frontal lobe...it was removed very successfully..then we found out it was cancerous , with the origin in the lungs, Adenocarcinoma... 3 in the lungs and so far 1 lymph node involved... the brain also has 2 symmetrical tumors in the cerebellum... She had some rehab after the surgery and came home yesterday... what do you suggest for the treatment. and what is her prognosis? Mary Ann Doctor: Hi Mary Ann, do you have any follow up scheduled with a medical oncologist? Adenocarcinoma is a type of non small cell lung cancer, but there are many different kinds and it is hard to know how to prognosticate without knowing exactly what kind your sister has. A medical oncologist will be able to answer your questions about the extent of disease in her body better than your neurosurgeon. Do you know what the plan for treatment is of her cerebellar tumors? Sometimes, depending on the size of the tumor, your sister may be a candidate for additional surgical resection. Other times, Gamma Knife/or radiosurgery is recommended. I wish you and your sister the best of luck and hope this was helpful."
},
{
"id": 62099,
"tgt": "What causes a lump on the back after prolonged sitting?",
"src": "Patient: it s not about giving blood. . . . It is about sitting in a chair at Macy s (a massage chair) and having it hurt my back. A large lump immediately (ten to twenty minutes after) appeared on my back. A lump almost as big as a golf ball. Three hours later it is the size of a quarter, as if it was a ball the size of a quarter. The nurse on call at the college (BYUH) told me it would go down, it was probably a hema...... and put ice on it. I did for two hours and no change, In fact it got a little bigger. If a blood vessel did break, how long does it last in lump form? If it is still there in the morning, should I go to the doctor? I do take extra strength excedrine (2) and ibuprophin (2) each day for some type of pain. Rebecca Doctor: Hi,Dear,Thanks for your query to HCM.Studied your query in full depth of its details.Reviewed it in context of your health concerns.Based On the facts,You seem to suffer from-Aspirin induced bleeding tendency which gave rise to hematoma Lump on the back.The friction while massage for a long sitting might have caused the bruise,which bled due to the aspirin induced clotting defect,causing contiued bleeding giving rise to quarter size lump on the back.USG / CT study if need be to assess the extent of damage to the bones and soft tissues of the back,would fix the cause of such a lump after prolonged sitting on massage chair.Long term use of the Extra strength excedrine which contains aspirin ,caused this hugh lump from meagre/trifle trauma to back.You should hence go to ER-To check the Bleeding and clotting time defect if any as it would need special treatment from Sugeon / Physician Team.Hope this reply would satisfy you and would help you to plan further treatment with your doctor there.Welcome any further query in this regard,which would be replied in next session..Good Day!!Dr.Savaskar M.N.Senior Surgical SpecialistM.S.Genl-CVTS"
},
{
"id": 101678,
"tgt": "Is frequent nebulisation safe in asthma patients?",
"src": "Patient: hello doctor, my mother is 80yrs old and an asthma patient for 40yrs. she used seretide evohaler till 3yrs back. she did not get asthma since 2yrs. now she is getting asthma since 15days. iam giving asthalin nebulisation and it was under control but yesterday night asthma came under control only after 3 nebulisations. Is it safe to give frequent nebulisation? Doctor: Hi,Asthalin caitains salbutamol,in many patients its reported to cause,palpitation,tachycardia,muscle cramps,so it can precipitate various cardiac condition,So I recommend you to use it under physicians guiadance, many rotacapinhaler are available in market it maintaince asthama well,please consult a pulmonary medicine physician."
},
{
"id": 85596,
"tgt": "How to get rid off diarrhea and stomach cramps after taking the betahistine dihydrochloride?",
"src": "Patient: Ive been taking betahistine dihydrochloride 16mg x 3 for just two days, and started to experience gastric problems, starting with nausea, indigestion and then violent vomitting and diorrhea. I have stopped taking them. How long will it take to get out of my system, as I am still suffering with diorrhea and stomach cramps. Doctor: Hello, Your symptoms seem to be related to side effects of Betahistine hydrochloride. I suggest to stop the medication. I also recommend to treat the symptoms by taking an over the counter treatment for the diarrhea such as Imodium. I also suggest to take Pedialyte to prevent dehydration and drink liquids. Hope I have answered your query. Let me know if I can assist you further. Take care Regards, Dr Dorina Gurabardhi, General & Family Physician"
},
{
"id": 105161,
"tgt": "Swollen feet, ankles, legs, rashes on legs post flying. Taking Ciprofloxavin. Allergic reaction?",
"src": "Patient: Since flying from XXXXX to XXXX I ve had swollen feet ankles and legs which I usually get for a few days. However I have also got a rash up both legs and pain in feet and ankles. No calf pain so no concerns of DVT just concerned about rash as thought initially it wa a sensitivity to Ciprofloxavin that I took for chest infection but since ceding the meds rash hasn t really improved. PMH- CA breast lymphoedema L arm and breast asthma and hypothyroidism Doctor: Hello, It is important that a DVT is excluded (at least by clinical examination, if not by doppler) as your past history puts you in a slightly disadvantageous position. However, if you are now not taking tamoxifen etc then the risk is low, but if the flight exceeded 6 hrs without much feet exercise, then it is better to exclude a DVT. It is unlikely a reaction to ciprofloxacin, unless you have a past history of quinolone allergy. The rash also means a DVT must be exlcluded. If the rash is itchy, then venous pooling with dryness of skin can cause a rash, when antihistamines will help. But I would still recommend getting this checked out. Thanks."
},
{
"id": 192607,
"tgt": "Can taking Ubicar tablet reduce sperm count?",
"src": "Patient: I would like to know a little about Ubicar. My husband has been using it and he complains it has reduced the sperm flow. Could you please tell me the review of this medicine. Doctor: Hello, Ubicar does not have any impact on sperm count. If the sperm count is persistently low, consult a urologist and get evaluated to look for other causes like varicocele. Hope I have answered your query. Let me know if I can assist you further. Take care Regards, Dr. Shinas Hussain, General & Family physician"
},
{
"id": 99447,
"tgt": "What causes breathing difficulty and sore chest while coughing?",
"src": "Patient: I was coughing very hard because of a peanut I swallowed \"went down the wrong pipe\". My chest was VERY sore - sharp scratchy pains when I coughed. I have had pleurisy before. I also tend to have diffiulty breathing after jogging for two minutes. Is there something you can identify now as a problem? Doctor: HI, thanks for using healthcare magicThe coughing can cause strain of the muscles in the chest wall which can cause pain.The shortness of breath on activity may be related to the heart, lungs or due to systemic problems such as anemia.It may be best to see your doctor for an evaluation which would include blood tests, imaging and ECG.I hope this helps"
},
{
"id": 195358,
"tgt": "Is thin skin around the penis normal?",
"src": "Patient: Hello Doctor, My pennis skin is became very thin ,now even if l masterbeating that skin is damaging , could you please suggest me what medicine or any cream or any oils or lubricants needs to be use to become thick skin on my pennis,am still vergin sir,Thanks in advance !! Doctor: Hello and Welcome to \u2018Ask A Doctor\u2019 service. I have reviewed your query and here is my advice. It is quite normal and nothing to worry. The skin texture and thickness varies between person to person. No treatment is required as of now. If you are so much worried, you can consult a urologist for just reassurance. Hope I have answered your query. Let me know if I can assist you further."
},
{
"id": 167347,
"tgt": "Could conjuntivitis cause constant vomiting and fever in an infant?",
"src": "Patient: hi, my 8 month old son as been vomiting all day not managing to keep anything down especially his milk, he had this one week ago which lasted for 7 days with loose stools aswell. he also seems to have conjuntivtus could this be causing his temp? should i phone the dr? thanks Doctor: In my opinion it would be unlikely that conjunctivitis would cause a temperature. If there is a severe infection in the eye called orbital cellulits they could get a fever but there would be severe swelling in the skin around the eyeThe fever is likely from the viral illness causing the vomiting and loose stoolsI would suggest trying Pedilyte or GatoradeI would take the child to the emergency room if they don't make urine in 12 hours, are not acting correctly, or if you have any concernYou could call the on call doctor to see if they would call you out Zofran, a medicine that can help with vomit ingredients I would suggest Motrin or Tylenol for the fever"
},
{
"id": 173399,
"tgt": "Suggest treatment for eczema in a 6 years old child",
"src": "Patient: Hi. My daughter is 6 years old and has had what has been identified as eczema. I am worried if it is something more serious since she now has an outbreak that seems like a raised hot red skin allover her chest, abdomen and back with some indications also on the neck and face. She has had this in the past and it seems worse when red and hot (very itchy). After some time, the entire areas peel off. That is a sign of relief generally. Would you know what is going on? (Also, we do not give her wheat and corn since we have observed some reactions in the past.) Finally, I am worried about her liver functioning properly. Perhaps this is not relevant, let me know please. Thank you. Doctor: Hi... I sympathize with the kid and understand your concern. As a matter of fact even my kid had this when young. The trick of the management is keeping the skin moist and never allowing it to be dry. For this oil massage and lotions like Oilatum will help. Regarding cure options - eczema is a sort of allergy...allergies can only be controlled and can never be cured. But usually kids grow out of it...that is as the age progresses.Hope my answer was helpful for you. I am happy to help any time. Further clarifications and consultations on Health care magic are welcome. If you do not have any clarifications, you can close the discussion and rate the answer. Wish your kid good health.Dr. Sumanth MBBS., DCH., DNB (Paed).,"
},
{
"id": 164264,
"tgt": "How to treat cough in kids?",
"src": "Patient: Hi, my 11 month old has had a cold on and off for 4 weeks have taken him to him to his pediatrician a couple times but had not got anything for relief he is normally on pulmicort and albuterol for wheezing but now he has a cough and is having a hard time sleeping the cough is so severe that it almost makes him gage while coughing what can I give for relief??? Doctor: Hi, welcome to HCM. You need to do an x ray chest to know the cause of long cough. An examination by doctor should be done."
},
{
"id": 68970,
"tgt": "Suggest treatment for bump under arm with green stuff",
"src": "Patient: Every now and again I get a round bump under my arm. I can feel the whole thing...it is completely round and is tender to the touch. It will come and stay for a week then leave for a couple weeks or a couple months but always comes back. This morning I noticed it was much closer to the surface of my skin so I gently squeezed it and green stuff came out. Is this something I should see my primary doctor for? Is it urgent? Doctor: It is most likely to be a sebaceous cyst. You should have it removed by a small operation if it is bothering you. Take care."
},
{
"id": 10683,
"tgt": "Does androgen blocker like Spironolactone arrest hair fall?",
"src": "Patient: hi i am 25 yr female having hair loss in frontal region. i am taking follihair and minoxidil 5% twice daily for 1 mnth n iron n calcim suppl. have normal testosteron n dheas normal and dont have pcot. low vit d and familial dyslipidemia. after using minoxidil i think my hair fall increases. some times i develop nodular cystic acne. my doctor dignosed me seborrhea and telogen effluvium. i am very depresses. should i take androgen blocker like spironolactone or something else for how mch time please help me Doctor: You most probably having female pattern hair loss which starts with thinning of hairs which over the time leads to progressive hair loss...As you say you have started with minoxidil 5% and follihair...minoxidil is the most trusted and FDA approved drug for the condition and you are on the right regime...yes its true that in the beginning people noticed even a more hairfall but dont panic it will go in a couple of weeks and you will notice new hairs soon...remember the full effect of minoxidil takes 6-12 months...dont switch medicines and have patience...and yes get yourself checked to rule out PCOD which could be the root cause of all your problems...thanks"
},
{
"id": 72360,
"tgt": "Does chronic dizziness and shortness of breath need medical attention?",
"src": "Patient: On 2nd May 2010 i started with vertigo and a cconstant feeling i was going to pass out. Since then my symptoms are : dizziness, lightheaded, short of breath, chest feels tight, heart palpatations, pain in my left arm, feeling weak, tired all the time, and generally unwell. Usually my blood pressure is 120/60 today it is 107/79. I've been unwell with this for 4 months now - should i be concerned? I'm a 25 year old female. Doctor: Thanks for your question on Healthcare Magic.I can understand your concern. Since you are young (25 years old) with normal blood pressure, possibility of stress and anxiety related symptoms is more likely.But better to rule out heart diseases first. So get done ecg, 2d echo and Holter monitoring (24 hours continuous recording of Ecg).If all these are normal then no need to worry for heart diseases. Consult psychiatrist and get done counselling sessions. Try to identify stressor in your life and start working on its solution. You may need anxiolytic drugs (propranolol and flunarizine combination) too.Don't worry, you will be alright with all these. Avoid stress and tension, be relax and calm. Hope I have solved your query. I will be happy to help you further. Wish you good health. Thanks."
},
{
"id": 14282,
"tgt": "Suggest treatment for itchy rash on hips",
"src": "Patient: I have a very itchy rash that started out on one hip and is now on both. It is not an area where my clothes or underwear rub. It is swollen like hives now and is very uncomfortable. I have not changed any laundry detergents or soaps. I have tried cetaphil, but it isn t helping. I had a reaction like this with pregnancy, but I am not pregnant. Any ideas? Doctor: Hi.As per your case-history your probable diagnosis is fungal infection.My treatment advice is-1. Maintain proper hygiene- bath twice daily, wash clothes in hot water, keep clothes and your belongings isolated from other family members.2. Dont use cetaphil , it wont work.3. Use clotimazole cream twice daily and take cetrizine tablet for itching.4. if it doesn't clear in 15 days then consult a dermatologist for further guidance.Thanks.Dr.Harshit BhachechM.B.B.S, D.D.V.L"
},
{
"id": 137971,
"tgt": "What causes pain under jaw while turning head to left?",
"src": "Patient: When I turn my head to the left and put pressure on my jugular vein I get a pain under my jaw. Also when I bend over and turn my head to the left I get the same pain. No swelling and it doesn t hurt any other time. I went to the dentist and they took some x rays of my teeth and found nothing. I have been referred to an oral surgeon. Any thoughts would be appreciated. Thanks. Doctor: Hello,Thank you for contacting me at \"Ask a Doctor\". I will try to answer your question to the best of my ability.Your symptoms are very nonspecific. I would suggest that we need to take a closer look at your temporomandibular joint. This should be something that the oral surgeon would take a look at. Other things that cause problems in the neck area would be inflamed lymph nodes and possible infection. Again, he should be checked by the oral surgeon. Is very difficult to Give few more specific information as I do not have a lot to go on at this time.I hope this information is useful for you. Please do not hesitate to message us back if you have any further questions.Best wishes,Adrian Rawlinson MD"
},
{
"id": 59465,
"tgt": "SGOT/SGPT high. Repeated test. Values came less. Suggested liver ultrasound, biopsy. Second opinion required?",
"src": "Patient: Hello doctor, Recently that is on 2nd of this month i went for a routine health checkup.Surprisingly i found that my SGOT/ SGPT values are elivated to 82/139 and platelet count is 1.28 lakh. I repeated the test after 3 days the values came douwn to 78/83 and platelet count 1.30 laks my doc also suggested me to do PT and even this PT came to be 18.5 and INR as 1.35. Note that when i had my liver function test 6 months back everything was normal. Not sure what wrong now. I tested for HepB and C both are non reactive My doctor suggested me to take hepa merz,ursocol,pantoid and a-z and he is also suggesting for liver ultrasound though i have done it as a part of my health checkup and which shows normal liver studies. He s also asking for endoscopy and liver biopsy . Please help me in this regard not sure i am on right path with this? Thanks in advance! Doctor: Hello. Thanks for writing to us. The minor elevation in the liver enzymes that you are having is not worrisome if the hepatitis B and C are negative. This happens most commonly due to mild fatty infiltration of the liver that can be detected on ultrasound scan. I hope this information has been both informative and helpful for you. You can consult me again directly through my profile URL http://bit.ly/Dr-Rakhi-Tayal Regards, Dr. RakhiTayal drrakhitayal@gmail.com"
},
{
"id": 167054,
"tgt": "How to treat throat infections in children s?",
"src": "Patient: my 6 year old a month ago had upper respiratory tract infection, was treated with antibiotics, within 10 days or less she suffered from urinary tract infection with high fever, treated with taxim for 10 days, urine culture showed staph aureus. She had pus cells 8-25 and epithelial 1-2. Now after two weeks she again had very high fever every 4-5 hrs. Urine test showed pus cells 1-2 but squamaous and transitional epithelial cells 8-10. Also to subside her fever we used crocin and meftal as advised. She has throat infection and treated wiith Augmentin. She also threw up and once also some small amount (5mL) of blood. Could that blood be effects of crocin and meftal? Doctor: Hi....Crocin will not cause this, but Meftal can cause this as it is a NSAID drug and gastritis is a common side effect of NSAID drugs.Meftal contains Mefenamic acid which is a NSAID. It carries the risk of gastritis and renal problems in the long run. Please do not use Meftal. Regards - Dr. Sumanth"
},
{
"id": 73785,
"tgt": "What causes pain in left side of chest?",
"src": "Patient: the left side of my chest has been hurting today.when i breathe in and move around its a constant pain. even when im still it hurts below my chest area and in my back.i also just had a bloody nose.does this mean anything? p.s. i have a heart murmur. thank you Doctor: Thanks for your question on Healthcare Magic.I can understand your concern.We should definitely rule out heart and lung related diseases in your case because you have murmur and nose bleeding.So consult doctor and get done blood pressure monitoring, ecg, 2d echo and Holter monitoring (24 hours continuous recording of Ecg).Also get done chest x ray to rule out lung diseases (infection, effusion).If all these are normal then no need to worry for heart and lung diseases.Musculoskeletal pain can also cause similar symptoms. So take painkiller like paracetamol or ibuprofen. Apply warm water pad on affected areas.Don't worry, you will be alright with all these.Hope I have solved your query. I will be happy to help you further. Wish you good health. Thanks."
},
{
"id": 5106,
"tgt": "Trying to conceive for baby, had miscarriage 3 years ago, normal periods. Taking tertomid. Suggestions?",
"src": "Patient: hi doctors im 25 years my husband and i have been trying for a baby tor two years now with no luck my doctor put me on Fertomid 50mg this is my second cycle on it im taking it on day 5-9 and today on day 6 im seeing spoting is this normal no any other Side Effects three years ago we had a miscarriage that left us devistaterd should i be taking any vitamins with this medication i Don't know what to do i have a normal 28 day cycle my doctor siad if i Don't fall pregnant within three months then he will refer me to a Specialist any advise Doctor: Hi,Thanks for discussing your problem with us.Any women who is trying to conceive should take Folic acid.As you have had a miscarriage, you are suffering from secondary subfertility. I think you should have seen fertility specialist before trying any medicine from GPs. The best course of action for subfertile couples is to have full investigations first. Once a cause for low fertility is found treatment can be specific & targeted which is likely to be more successful. If this spotting is part of your periods, there is nothing to worry about. If you have finished your periods & again this started you should report this to your doctor.Wish you best of luck."
},
{
"id": 99960,
"tgt": "What causes shivering, rash on chest and hotness?",
"src": "Patient: My son has just woken up shivering but saying he's hot. His temprature is normal but he does feel hot on his body. He's been ill for the last few days. It started with a cough and then a rash appeared on his chest (this disappeared under a glass) the rash has gone now. Should I worry about the shilvering when hot? Doctor: HI, thanks for using healthcare magicThis would be chills. Chills can often accompany a fever so this is not unusual.He needs to keep as cool as possible. With chills, persons can sometimes have the urge to wrap up but this would further increase his temperature.The use of paracetamol should help the fever. I hope this helps"
},
{
"id": 210198,
"tgt": "Suggest treatment for a shopaholic patient",
"src": "Patient: I am the husband of a hoarder. My wife can not get rid of anything and she is buying things we don't need and it has piled up to the point that:I can't even walk into the 3rd bedroomThe dining room is stacked half way up with junkThe kitchen counters have so much stuff on them it is hard to work in the kitchenThe storage shed is so full that I can't put the lawn mower in there.I believe this will lead to divorce. I can't live like this and I don't think she is able to change either.Where can I get help. One more thing. My wife is seeing a psychiatrist about this disorder. It isn't doing any good. it is only getting worse. Doctor: HiThanks for using healthcare magicI think, she has obsessive compulsive disorder and for that she needs anti-obsessional drugs like fluoxetine or fluvoxamine. With medicine, her symptoms would decrease with time. You can also take help of psychologist for thoughts stoppage technique. Rest, you can discuss with her psychiatrist.Thanks"
},
{
"id": 16825,
"tgt": "How to treat raised heart rate,shivering of hands and legs?",
"src": "Patient: Hi my name is lisa, I am 29 years old and I have been having problems with my heart racing for four years now, when it starts to race my face gets hot and my mouth gets dry and I start to shake, my hand and feet get sweaty and cold, I can t stand to be around heat to long what could this mean and all of this started happening when I stop smoking Doctor: Hello, You may need to take bisoprolol tab to slow down your heart and relieve this symptom which I guess from the cessation of your smoking. Hope I have answered your query. Let me know if I can assist you further. Take care Regards, Dr Salah Saad Shoman, Internal Medicine Specialist"
},
{
"id": 31012,
"tgt": "What are the symptoms of H. Pylori infection?",
"src": "Patient: Several years ago I had h pylori and went thru antibiotics and pepto bismal treatment. I was told once I had it I couldn t have it again. I am having lots of problems that seem to be same symptoms. My doctor has ran many tests. My sedimentation rate keeps going up. The last one was 75. Do you think I should ask him to test for h pylori again. I have had breast cancer, am diabetic and have fibrmyalgia Doctor: Hi thanks for contacting HCM...You had taken antibiotic for h.pylori ..So it should be resolved with drug if correct drug given.For it amoxycillin plus clarithromycine needed with omeprazole...ESR is non specific ....It raised in malignancy also.You have breast cancer so also ESR can be raised.......So treatment for breast cancer done....Take care.Hope your concern solved.Dr.Parth"
},
{
"id": 1260,
"tgt": "Can pregnancy happen with non penetrative sex?",
"src": "Patient: hi, i and my boyfriend were fooling around, and he touched the tip of his penis to my vaginal opening, but no penetration and no ejaculation.this was march 6th.still, i took i pill,it was 48 hrs later.i got my periods 2 days later than expected date(march 23rd).the first day flow was ok, but very less on 2nd and 3rd day.sholud i still take a pregnancy test? Doctor: Hi.No ma'am, a pregnancy test will not be required at all as there was neither any penetration, nor any ejaculation. The delay in your period was because of the ipill itself. And above all, you already had your period, so there are no chances of pregnancy at all.Best wishes."
},
{
"id": 19756,
"tgt": "What causes shortness of breath and palpitations while trying to sleep?",
"src": "Patient: i am having really bad shortness of breath while trying to sleep, my right side will go numb and my heart races feel sick, palpritations, like a strangling feeling it the throat, sinking feeling into the bed, and i tight chest, this has been happening for 9 months and steadly getting worse, i have been diagnosed with anxiety disorder, and i take citalipram 10mg. i have been taking this for 6 mmonths, i have had all kinds of blood tests done for my dizzy spells during the day a mri of the head which olny showed a pituitary cyst. i have had a ultrasound of my neck ct scan of sinusis and EEG and a which is all normal, what do u think this could be ? Doctor: Hello sir and thanks for posting your query here.Although what you are having could be anxiety but I could not see any mention of heart test reports.And as you said you are having progressively increasing breathlessness and have difficulty in sleeping, this could very well be due to heart problems. So I would suggest you to see a cardiologist and get your ECG and Echo done to see the heart function. If there is any problem with the heart function, obviously you will need medicines for that and your breathlessness should settle down.I hope this information would help you.Thanks and all the best."
},
{
"id": 6569,
"tgt": "Are there chances of pregnancy in a woman on birth control pills and irregular periods ?",
"src": "Patient: Hi Doc, My age is 27 female, I have irregular periods.My last menustrial date is 21 April.. i did not have my periods from 45 days unfortunately i had an unprotected sex on 46th day and i had an i pill with in 24 hours. Now i m not sure what will happen will i have my periods or will i get pregnent? Please advise what needs to be done. Thank you. Doctor: Hello As you had unprotected sex on 46th day of cycle & had I pill with in 24 hours ,so chances of pregnancy are almost nil.wait & watch.,nothing else. Hope I have answered your question well to your satisfaction. Wish you speedy complete recovery at the earliest."
},
{
"id": 42024,
"tgt": "Is 20% sperm morphology normal in fertility test?",
"src": "Patient: Hi Doctor, I m 26 yrs, my semen analysis report is as follows;color :Light yellow,Vol:2,Consistency:liquid,pH:8,Liquification time:30 min, total Count:98,(Mortility) Rapid progress:30%,(Mortility) Slow progress:30%,(Mortility) Non progress:30%,(Mortility) Non mortal:20%, (Sperm Morphology) Normal:65%, (Sperm Morphology)Abnormal Forms:35%,RC:Nil/HPF,WBC:2-3/HPF Pleas tell me whether this report is good or bad with Fertality point of view. Doctor: Hi, Welcome to HealthcareMagic . In your report 65% sperms are normal morphology. That is considered normal. Progressive motility is less in your case. But not too less that it affect fertility to greater extant. I suggest you to take drug containing lycopene, zinc and co enzyme q which improves sperm motility. Also avoid smoking and alcohol and reduce weight if you are obese. Hope I have answered your query. RegardsDr.Deepika Patil"
},
{
"id": 215627,
"tgt": "What causes body stiffness and pain in the legs after waking?",
"src": "Patient: i am achy and stiff every time i get up to move after being stationary for a little while: after sitting at my work desk for maybe 20 minutes or more, when i wake up and get out of bed... My feet and legs just hurt and it takes a minute or two to walk it out and then it doesn t hurt any more. I am 53 but i feel like i am an old lady the way i get up after sitting in the chair for a short while. This has been going on for a few months now. What could be possible causes? Doctor: Hello, It could be an early sign of arthritis. You can take analgesics like tramadol for pain relief. If symptoms persist you can consult an orthopedician and get evaluated. Hope I have answered your query. Let me know if I can assist you further. Regards, Dr. Shinas Hussain, General & Family Physician"
},
{
"id": 148066,
"tgt": "What causes shortness of breath and numbness in lower legs while taking flagyl for IBS?",
"src": "Patient: I am 67 years old and take generic for Flagyl. Two 500mg per day for IBS. I have shortness of breath that eases after sitting or laying on my right side. When up and moving it is very difficult to breathe and with exertion almost impossible. There is also swelling and numbness in the lower legs and feet. Urination at times is more like dribbling and takes a long time to stop. Other times it is very slow and weak stream, then once or twice a month as normal as it was at 50. I also have severe allergies that have not been diagnosed yet and the meds keep me dehydrated. Doctor: HIThank for asking to HMCI can understand your concern, you have to keep your moral high, because of the highly depressive state of mind such symptoms are very likely, you have to come out of this condition, apprehensiveness, obsession, these are not good for health, you have to be mentally fit, otherwise there would be no limits IBS is the near example of this, I hope this information helps you, take care and have good time."
},
{
"id": 109744,
"tgt": "What causes lower back pain and urine incontinence?",
"src": "Patient: Constipation is my usual situation. Every once in a while I have stools that are formed but slide out with little effort. I have been having lower back pain for about a weekI I suspect bladder infection as I have had it often. Backpain is worse this time. some urine incontinance. Any suggestions? I'm 78, 5'3\" and 168. Doctor: As per your complaint of back pain anf urine incontinence, you should do some investigations for them.For urine inconti. Do ultrasound to rule out prostate hypertrophy, urine routine micro and sugar for diabetes and x ray of lunbo sacral spine for lower back ache.These test help you to find any abnotmality if it is there. And then consult your family physician.."
},
{
"id": 58373,
"tgt": "Removed gallbladder, diarrhea, bright red blood n stool, nausea. Need to go to ER?",
"src": "Patient: Hi, I had my gallbladder removed 2 days ago and have been recovering as I think I should be. However, today I started having very bad diarrea, it has gone from dark brown to a very pale yellow and looks oily. The most recent time (about 1/2 hour ago) I had some bright red blood in my stool. I am also very naseaus but that has been on and off since I came home and I've been using zofran. what would cause this to happen? should I go to the emergency room or is this something that can wait til morning when the office is open? Thank you Dee Doctor: Hello! Thanks for putting your query in HCM. I am a Gastroenterologist (DM).It appears that you are suffering from acute gastroenteritis. For this take a course of antibiotic like ofloxacin and ornidazole and continue with zofran for nausea.I hope I have answered your query and this will help you. If you have any further query I will be happy to answer that too. Remain in touch and get-well soon."
},
{
"id": 12012,
"tgt": "What is TT and what could be the reason for skin problem will it be curable ?",
"src": "Patient: Sir, my wife has developed a large bright patch on her face . a dermatologist has carried out blood sample test, which has come normal and a test Skin for AFB , by taking a piece of skin from the affected area, which has come negative. the Doc has prescribed Rifampicin and Dapsone for it. he has said to be under medication for 6 months and written the disease as T T . what is this T T in medical terminology? She had a minute patch on her left cheek for quite some years, which has all of a sudden flared up recently. her age is 34 years, she has thyroid and no other complications. going by the prescribed medicines, could any skin specialists enlighten me on what could be the probable reason for my wife s skin problem? Is it curable? Thanks Doctor: hi TT is type of leprosy and it take long time to cure but it is curable now spot may last up to years but i dont know why he has given in ur test as ur biopsy is normal . there is alter sensation of touch and temperature in leprosy case if they are alter then she need to start these medicine this is major disease and it is legal duty of doctor to inform about it please consult him again and ask for explanation why he has started it better to consult other dermatologist before starting medicine ."
},
{
"id": 58657,
"tgt": "High level of bilirubin. What should be done to reduce this?",
"src": "Patient: Good day doc last july 2, 2013 I undergo medical test as required in going abroad and my sgpt result 1.89 and my total bilirubin 2.97 after 2 weks I had blood extraction for another test that yielded same result. Right now I'm undergoing medication of inlex(same as godex) for six days now does this reduce my sgpt w/c is 67(another unit used . Also my bilirubin is 29.4 umol/l. What should I do to reduce this? Doctor: Hi and thanks for this query.I am so sorry to hear about these disturbing laboratory findings. Do you have any more symptoms???The best treatment of elevated bilirubin is to know the cause. It is often caused by a problem in excretion or the liver being unable to handle it for one reason or another. The best thing to do is to get a doctor's consultation so that he can examine you and maybe ask for more tests to find out wat the cause of that could be. from there, you will now receive drugs depending on the possible causes. there are no drugs tat just reduce bilirubin levels because all by themselves, they do not represent any disease but signals the presence of a disease.Hope you find this helpful and informative.If you have more questions send them to me and i will provide more answers.Wish you well.Chobufo, MD"
},
{
"id": 57357,
"tgt": "What are allergic symptoms of hernia mesh implant?",
"src": "Patient: My husband had a hernia repair surgery about ten days ago he had the mesh implant now he has a rash on his whole body and is very itchy and cold and he had no reason to be cold with the heater going I was wondering if it is an allergic reaction to the mesh he also does not have a spleen so I am very worried he took some benedryl and it did nothing and he has never had any kind of rash ever till now Doctor: HIThank for asking to HCMI really appreciate your concern for your husband, if your husband does not have history of any allergic condition then the mesh would be the reason for that you have to talk to the surgeon, mean while the allergic symptoms need to be treated with the \"Tab Levocetrizine 10 mg three times in day\" hope this information helps you, have nice day."
},
{
"id": 32717,
"tgt": "Suggest treatment for itching due to chicken pox",
"src": "Patient: Dear Madam, This is to inform you that, I am a 30 year old guy from Bangladesh. Since my childhood I have never ever been infected by chicken pox. For the last 4 days I am infected by this disease. The infection is huge on my chest, back, face, hands, fingers, legs, hips, under toe. I am also carrying the infection of this chicken pox inside my mouth over my tonsil gland. As a result it is quite hard for me to eat anything since the pain is huge through my neck and I am finding it very difficult to deal with the itching related issue of chicken pox. Do you have any suggestion for me? Doctor: Thanks to HCM I can understand your severity of disease .in chicken pox most crucial thing is pain &itching don't try to burst the vesicles over the body because it may spread to unaffected parts also plz apply calamine lotion for soothing effect .. take tab, levocetrizine 5 mg once daily for 5 days along with paracetamol and antivirals plz change you dressings daily ,better use only cotton clothes. ok don't worry it will be relieved shortly once severity of disease reduced , ok"
},
{
"id": 136603,
"tgt": "Suggest treatment for swollen and painful thumb",
"src": "Patient: I jammed my thumb playing volleyball 4 days ago. Initial pain, but not after. Swelling went down alot after 2 days. I can now move my thumb back, but can t bend the knuckle all the way forward, probably because of swelling. There is a little bruising near the knuckle, and just a bit near the base. Should I see a doctor? Doctor: Thank you for using HCM. There's been a blunt trauma to your thumb and based on the severity of injury there could be ligamental tear and probably fracture which may require splinting and immobilization of thumb. Please visit the ER get a X ray done and follow up with orthopedician."
},
{
"id": 116784,
"tgt": "Suggest treatment for low hemoglobin",
"src": "Patient: my mom is 68 years and lately we discovered her hemoglobin level is 6.2 ,always it was betwwen 10 and 12, however it was now checked after almost 2 years, so i dont know how long shes been at this level can u tell me how dangerous is this and how to treat it thanku doc Doctor: Hello, Thnx to contact us. I understand your concern. If I am your doctor I advice you low hemoglobin is not a serious condition untill the hemoglobin goes down very much. I advice you to take tablet Livozen Z to your mother for two times a day. once the hemoglobin level is achieved near 8 gm tablet will be taken once a day. Take tablet two to three month more when you reach the near hemoglobin level.I will be happy to answer more of your concerns, kindly know me,Wish you a very good health at health care magic. Dr. Arun Tank. Infectious Disease."
},
{
"id": 74749,
"tgt": "What causes chest pain?",
"src": "Patient: I have 2 take 4 different kinds of pills with food. When I do, a few hours later I throw up. After throwing up so many times, my chest hurts. It feels like there is something stuck in my chest every time I swallow. Even if I don't, I get these sharp pains and they are painful. Why is that? Doctor: Respected user, hi I evaluated your query thoroughly.* Related to drug induced gastritis with acid reflux giving chest pain.- we need all drugs details for further guidelines .Hope this helps you.Thanks for using Healthcaremagic.com & giving me an opportunity to assist you.Welcome for any further assistance.Regards dear take care."
},
{
"id": 50071,
"tgt": "On dialysis for kidney failure. Infection on the toe. Using knee walker and walking with a cast. Any advice?",
"src": "Patient: I am in kidney failure, in dialysis . I have nephrology in both feet 2 months ago I had an infection in my left big toe . They wanted To amputate but I manage to get rid of the infection. I have been walking with a cast to avoid pressure on my toe and Also been using a knee walker. I can t still stand on my foot. The big toe is very tender, I have new skin on that toe. Is this the nephrology that causing this pain, the toe is not swollen or red so I don t think it s infection...// Doctor: Hi and thanks for the question. I suspect you mean 'Neuropathy' when u say 'Nephrology in both feet'. Are you diabetic? If so, neuropathy is not unusual to have.Are you sure you have cleared the infection entirely, since you still have pain in ur toe. Please get it checked out by your treating physician/ surgeon ASAP. Regards. RB"
},
{
"id": 185969,
"tgt": "For what exact reason is Propeptide Type 1 Collagen test is carried out?",
"src": "Patient: Hi. I had a blood test for some dental work I need to have done. The test is Propeptide Type I Collagen. Can you tell me exactly what this is for? I needed to take it because I took Fosamax for several years. I stopped taking the Fosamax in January. How long before it gets out of my body, if that s the purpose of the drug holiday. Thanks. Doctor: hellothnx for your query.you didnt mention ur age. the test u r mentioning indicates the bone turnover.Type I collagen is the predominant collagen in bone.it is used to evaluate and monitor the rate of bone resorption and formation to monitor some metabolic bone diseases such as osteoporosis.A \u201cdrug holiday\u201d is a discontinuance of bisphosphonate like Fosamax for a length of time to achieve a reduction of risk of osteonecrosis of the jaw (ONJ). It is suggested that you consider interrupting bisphosphonate treatment for 3-4 weeks prior to dental surgery and restarting after bone healing.for patients who have taken an oral bisphosphonate for more than 3 years, discontinuation of the oral bisphosphonate for 3 months prior to oral surgery may reduce the risk. The bisphosphonate can be started again once osseous healing has occurred. hope this ans helpsRegardsDr. shesh"
},
{
"id": 90039,
"tgt": "What causes cramping in the stomach?",
"src": "Patient: Hi I Had My Tubes Tyed 22 Months Ago.. Weeks Ago I Had What Appered To Be My Normal Period.. It Lasted About 9 Days... And Then This Am I Woke Up In A Large Amount Of Bright Red Blood. After Two Tampon's In Three Hours The Blood Is Now Dark Brown Almost The Color Of Prune Juice.. I Have Some Abdominal Cramping And feel Very Sick To My Stomach.. Is There Something Wrong???? Doctor: welcome to Health care magic.1.Tube lagation generally does not give the above symptoms.2.Reason for bled need to be seen.3.If you are my patient i would ask for an ultrasound pelvis none to see the status of your uterus and ovaries to rule out many causes. ( Mass/inflamations/Bulky ovaries)4.Yes there is some thing wrong and get medical help as soon as possible.Anything to ask ? do not hesitate. Thank you."
},
{
"id": 76743,
"tgt": "What causes vision loss , dizziness , numbness and breathing problem?",
"src": "Patient: Last night I had an odd episode and trying to figure if it warrants seeing a doctor. I am a 31 year old female about 140 pounds and had a healthy physical last year. But while swimming at my friends house late last night I encountered heavy breathing, loud ringing in my ears, loss of vision, shaking, dizziness, numbness in my hands, and almost completely passed out. My friend noticed my pale face and white lips and had me lie down. After an hour I felt good enough to drive home where I vomited the rest of the night with complete exhaustion. Today I still feel weak and moving fast still takes my vision and I did have ringing in my ears for about a minute with spots floating in vision while sitting. Can someone please give me some advice? Thanks! Doctor: Thanks for your question on Healthcare Magic. I can understand your concern. Sudden onset breathlessness,dizziness, numbness, shaking, collapsing, loss of vision and ringing in ears are seen with 1. Pulmonary embolism 2. Heart diseases like heart attack 3. Epilepsy (seizures). So better to consult doctor and get done 1. Clinical examination of whole body. 2. CT or MRI of brain and EEG (electro encephalogram) 3. CT pulmonary angiography for the diagnosis of pulmonary embolism. 4. Ecg and 2d echo for heart diseases. If all these are normal then sometimes stress and anxiety can also cause similar symptoms. So consult doctor and discuss all these and first rule out major diseases. Hope I have solved your query. I will be happy to help you further. Wish you good health. Thanks."
},
{
"id": 114478,
"tgt": "Suggest treatment for anemia, loss of appetite and severe hair loss",
"src": "Patient: my daugther has the following symptoms Anemia Loss of appetite Gluten intolerant Soy intolerant dairy intolerant Gastro paresis Acid Reflux Migraines Nausea Vomiting Hair shedding excessively when combing or even just washing Abdominal pain Joint pain Shortness of breath Pelvic pain Back pain Upper abdominal pain Chest Pain Fatigue Weakness Can t sleep, If I do sleep I don t sleep well Lightheaded Dizziness Usually cold especially in fingertips Sudden sleepiness Neck Stiffness Back stiffness Brown urine or dark urine What should we considering Doctor: HiAfter hearing your history i would advise you to treat anaemia with iron supplements and making changes to your diet.If the underlying cause of iron deficiency is loss of blood \u2014 other than from menstruation \u2014 the source of the bleeding must be located and stopped. This may involve surgery. Treatment for folic acid and B-12 deficiency involves dietary supplements and increasing these nutrients in your diet.If your digestive system has trouble absorbing vitamin B-12 from the food you eat, you may need vitamin B-12 shots. At first, you may receive the shots every other day. Eventually, you'll need shots just once a month, which may continue for life, depending on your situation.For loss of appetite:you have to dothe following testan ultrasound of your abdomena complete blood counttests of your liver, thyroid, and kidney function (these usually require only a blood sample)an upper GI series, which includes X-rays that examine your esophagus, stomach, and small intestinea CT scan of your head, chest, abdomen, or pelvisIn some cases, you will be tested for pregnancy and HIV. Your urine may be tested for traces of drugs.If your decreased appetite has resulted in malnutrition, you may be given nutrients through an intravenous line.Your doctor may also prescribe oral medication to stimulate your appetite.Your doctor may refer to you to a mental health specialist or counselor if your loss of appetite is a result of depression, an eating disorder, or drug misuse.Loss of appetite caused by medications may be treated by changing your dosage or switching your prescription. Never change your medications without first consulting your doctor.If your decreased appetite is caused by a short-term condition, you\u2019re likely to recover naturally without any long-term effects.However, if your decreased appetite is caused by a medical condition, the condition could worsen without treatment.If left untreated, your decreased appetite can also be accompanied with more severe symptoms, such as:extreme fatigueweight lossa rapid heart ratefeverirritabilitya general ill feeling, or malaiseIf your decreased appetite persists and you develop malnutrition or vitamin and electrolyte deficiencies, you can have life-threatening complications. Therefore, it\u2019s important to seek medical attention if you have anorexia that does not resolve after an acute illness or that lasts longer than a few weeks.For your severe hair loss you ca treat it with Minoxidil (Rogaine) 5% is the only topical medication approved by the FDA for female-pattern hair loss. The once daily use foam treatment regrows hair in 81% of the women who try it. Liquid options of 2% and 5% solutions are available over the counter.RegardsDR DE"
},
{
"id": 160147,
"tgt": "Is it possible to spread cancer from one peson to anothere person ?",
"src": "Patient: Hi, this is neelakumar from chennai is there possible to spread cancer from one peson to anothere person ? Doctor: No this can never happen"
},
{
"id": 218396,
"tgt": "Is pregnancy possible while on Yamini pills?",
"src": "Patient: My wife(21years) had a pcod on right ovary measuring 25\u00d725 mm. Her gynocologist inject her hcg n told to intercourse during 3 days. But after 10 days c got periods. And after consulting doctor c prescribed to take yamini from tablets from third day of periods for 2 months And a question is.. is there chance of being pregnant or we can intercourse during these time..or will wait for 2 months.. we just need baby without any hassle. please suggest me the right way of treatment and safe pregnancy... Doctor: Hello and Welcome to \u2018Ask A Doctor\u2019 service. I have reviewed your query and here is my advice. Yamini is oral oestroprogestative combined pill. It is used for contraception. In case of PCOD it is used to put ovaries at rest and regulate cycle. The fertility will come immediately after stopping pill. You can try for baby after two months. Hope I have answered your query. Let me know if I can assist you further. Regards, Dr. Olgeta Xhufka"
},
{
"id": 177203,
"tgt": "What causes enlarged thymus in a kid",
"src": "Patient: My eight month old baby is having soft stool with blood in it for the last one month. Doctor found his liver and spleen enlarged. Otherwise he is happy and eats well. He was given a course of antibiotic when he first showed diarrhea like symptoms. Doctor now wants to wait and watch. The baby was diagnosed with an enlarged thymus at birth. Do I need to worry? Doctor: Hi...enlarged thymus is common in new born babies. That is not to be worried about. But blood in the stools going on for 1 month without any other illness or fever cannot be bacterial infection or infection related. It should be a cow's milk protein allergy or a non-infective inflammation. I suggest you have a second opinion regarding this.Regards - Dr. Sumanth"
},
{
"id": 146330,
"tgt": "What causes persistent numbness in nose and upper lip?",
"src": "Patient: I had an allergic reaction to a sulfar based antibiotic that caused kidney failure. I am getting better but my nose and mouth feel numb as if I had a shot from the dentist. This happened on Sept. 22, hospitalized for 3 days. The hospital did all types of test from CT scan, MRI, EKG, etc. nothing indicated a problem such as stroke. I am getting stronger every day. However this numbness that feels like nose and upper left lip, especially nose feels strange, nasal secretion. No one seems to know what is causing the problem. I am having a gastroscopy procedure done on Wednesday. All of this started because I was diagnosed with bronchitis 3 times this year. I have problems swallowing and coughing. Should I be concerned? Doctor: Hi, I understand your concern. It seems you have been investigated extensively for the numbness. * I think your problem is something apart from your allergy,bronchitis. I would suggest Plain x ray cervical spine AP & Lateral view. It may reveal cervical spondelitis causing pressure on facial nerve supplying nose & upper lip. Once diagnosis is made specific treatment will be advised by a neurologist. * Investigations for allergy & treatment is another thing you need. Visit Allergy specialist/ ENT specialist for specific advise. Thanks."
},
{
"id": 197021,
"tgt": "Suggest medication for premature ejaculation,pain,itching and spot at the base of shaft",
"src": "Patient: I had unprotected sex about 5 days ago. I am unsure if this person was \"clean\" but if I had to guess I would say she was clean. I have masturbated probably 4-5 times since the experience with her. Today I awoke to see a small purple spot on my penis shaft near the base. The spot is pretty small, about the size of a pencil eraser. It is not raised and there are no bumps. The spot fades around the border, and it is also worth noting that the spot is located on a visible blood vessel. It is on the right side of the shaft near the base, and I am right handed.There is no pain or itching associated with the spot. I have no abnormal discharge from my penis and urination is normal. Squeezing the location produces no pus/sebum and it does not feel as if there is any sort of tumor under the skin. I don't want to go to a doctor just yet, but I plan to monitor the area for a few days and see if any changes occur.I have been looking for several hours online, and looked at hundreds of pictures of early STD symtpoms and my spot looks nothing like any of them. I am assuming and hoping it is a burst capillary or something. Could a professional please advise me on this situation and any possible treatment, or perhaps a surefire way to know it is not an STD and is a burst capillary. Doctor: HelloThanks for your query .Based on what you have described in detail I would state that as you have rightly though for the purple spot at the base of the penis is a sub cuticular hematoma resulting from a rupture of small vessel due to dry and vigorous shaking of penis while masturbating . In most of the cases it gets absorbed within 2-3 weeks and does not need any active intervention nor to be worried about .As regards your doubt acquiring STD .In view of it being a unprotected sex with unknown woman and you are not being sure about her STD status I would suggest you to get basic screening blood test for STD done to be rest assured about acquiring STD .Dr.Patil. ."
},
{
"id": 7036,
"tgt": "Could i be pregnant with low hemoglobin level ?",
"src": "Patient: I am a female of 30 years of age. Whenever I check my HB, I am told it is very low around. What could be the possible causes for this. I usually experience heavy bleeding and lots of clots whenever I am menstruating. What causes this and what is the effect. Can I still get pregnant with low HB level without any problem? Please Help. Thank you. Doctor: Hi u r suffering from menorrhagia i.e heavy menses n so u r anemic i.e. Hb low. Before pregnancy u shud hav gud HB for a healthy child n safer delivery. U need to c a gynaecologist for finding cause of menorrhagia n treatment.also u need to take iron tablets for increasing ur Hb. Thnx"
},
{
"id": 113042,
"tgt": "Pain in lower to mid back, fine during day, stomach feels tender. What to do ?",
"src": "Patient: For the second night I have woken up with pain in my lower to mid back which gets intense after 5 minutes. I need to apply hot water bottle to the area to ease the pain and take ibruprofen. Back is fine duri g the day but I am women up with it. Stomach also feels tender. Had a Gladstone attack recently upper right abdomen a month ago but now pain has moved to lower mid back Doctor: Hi Welcome to HCM The lower back pain is origination from the your vertebral column at the level of lumber or sacral vertebrae which is due to disc problem mostly due to degenerative changes which causes compression on the spinal cord and results in the pain along the distribution of the nerves which often affects one or both of the limbs. A visit to neurologist is recommended. You can use diclofenic sodium for pain relief which is on the counter available drug. Thanks Regards"
},
{
"id": 110631,
"tgt": "What causes back pain?",
"src": "Patient: Hello, I was having lower back pain and MRI report gives that L3-4 disc shows dehydration, bulge with small osteophytes causing mild impingement on theca. What causes this problem? What is the cure for this? Can I do spinal exercises? My pain sometimes recedes but comes back.I am having this problem since the past 4 months.Started taking doctor s help since a month.Do advise. Doctor: Hello, Thanks for your query.It seems you are suffering from degenerative disc disease & it occurs due to repetitive overload or stress to the disc & it increases the risk of disc herniation & spinal canal stenosis.The main treatment of such pain is bed rest along with pain killers and muscle relaxants. Muscle relaxants can help with your symptoms if used in appropriate dosage in combination with a potent analgesic.You can get the appropriate drugs prescribed from your Orthopedician after examination.Meanwhile you can follow these measures:- If the pain is severe, you need bed rest till the pain resolves. - Get some analgesics prescribed and apply analgesic spray or ointments.- While resting, keep a pillow under your knees if it doesn't bother you.- Avoid lifting heavy objects.- Back strengthening exercises and stretching exercises daily as advised by a Physiotherapist.Drink lots of fluids since dehydration affects the disc. I do hope that you have found something helpful and I will be glad to answer any further query.Take care"
},
{
"id": 138945,
"tgt": "How can shoulder blade pain on movement be treated?",
"src": "Patient: I have pain above my left shoulder blade - I woke up with this. I was fine when I went to bed. I have take 6 Davila today. My daughter noticed, after I could not move from bending over, that there is also a brown spot there that looks like dirt. This pain came after carrying my 15 pound grandson around . I am otherwise in good health...138 pounds, 61 years old, anxious at times and some sleep issues but otherwise nothing. I walk about 3 - 5 miles a day - in an urban setting. Oh also hurts when I take deep breaths. And it is Advil not Davila Doctor: You seem to have torn the superior capsule of your acromio clavicular joint, get an X Ray done & also clinical examination by an orthopaedic surgeon to assess you clinically is equally impoetant. consult an orthopaedician"
},
{
"id": 32706,
"tgt": "Does pain in glands indicate upper respiratory tract infection?",
"src": "Patient: I am a 45 yr old woman. I started having pain in my glands in my neck. I went to the doctor and held said I have a upper respiratory. Infection in my head area. My ears, throat & sinuses. Are clear. I do not have a tooth issue. I do have numbness in my lower lip and swelling in my left eye. I have been running low grade fevers for 3 weeks. I am starting my 3rd set of antibiotics & I am starting to get nervous. Does this sound like a respiratory infection or maybe something else? Would blood work show the cause? Doctor: Thanks to HCMI CAN UNDERSTAND YOUR WORRY .any lymph gland swelling in neck region generally related to infection for that draining area .upper respiratory tract means from nostrils to entry of trachea here you have history of swelling in left eye which leads from URTI..so don't worry this common for ear nose throat and eye ..plz take precausion not to expose dusty area , overcrowd places avoid cool items alon with cool environment take good antibiotics and anagesics symptoms will be relieved ok don't worry"
},
{
"id": 63529,
"tgt": "How can a lower lip lump which is raised be treated?",
"src": "Patient: Hi, may I answer your health queries right now ? Please type your query here... I have a little black bump inside my lower lip, its raised, starting to change a little over the last two months in size. None of the photos I see online are close to what this is. Is it a type of cancer? Doctor: Hi Dear, The lump or swelling could be a mucocoele that is a cystic lesion arising from minor salivary glands...nothing to panic.As per your complain the lump over the lower lip can be due to inflammation as well as HEMATOMA formation because of some minor injury .The hematoma is formed due to injury to the small blood vessels leading to spillage of blood into soft tissues as well as there is release of inflammatory exudate and overgrowth of fibrous tissue and appearrence of inflammatory cells at the site of injury.You should do alternate hot and cold packs over the lump..Apply Thrombophob gel from outside as it helps in faster dissolution of clot.You can once consult an oral pthologist for evaluated to rule out the nature of lesion as in a few conditions surgical removal can be done.Avoid any cosmetic product for now , because lump can also form due to allergy to cosmetic products .Hope your concern has been resolved.Dr. Harry Maheshwari"
},
{
"id": 92113,
"tgt": "What causes abdominal pain and chest pain with painful mole on breast?",
"src": "Patient: I am 18 years old and I have abdominal pain for 4 months. I also feel chest pain, and a dark-like mole grew on my breast, i believe it is not a mole. It hurts so much when touch or when im wearing tight clothes especially when im wearing bra.. and also, Its been 5 months since I had my menstruation. I am scared and dont know what to do. Doctor: Hi. Abdominal pain with no menstruation can have many reasons related to uterus and tube, for which you need to for for clinical evaluation by Gynecologist and ultrasonography of abdomen. A black mole like thing on the chest needs to be biopsied ."
},
{
"id": 209759,
"tgt": "How to overcome depression?",
"src": "Patient: had a bang on my head march 26th wilst on holiday came home and got a bad chest, had 2 lots of penicillane had to hols my head when i coughed been doctors 6 times just gave me tablets for depression, have a tightness on my fore head and tenderness on right side of head thye say i don't head scan Doctor: What going in in your mind needs to be understand for better treatment. Are you feeling low and sad. Kindly tell us."
},
{
"id": 167500,
"tgt": "What causes running nose and distended belly having diagnosed with protein intolerance?",
"src": "Patient: Hi, I have a 10.5mth old infant who has been diagnosed with Multiple Food Protein Intolerance. He is on special formula Elecare and seems to only tolerate Rice Cereal and Pears but he still has a constant runny nose and distended belly. He has acid reflux as well which has since lessened on special formula but I was wondering what could be causing the runny nose and distended belly still. Thanks Doctor: Hi....by what you say I feel that it could be a common cold (if the duration is short) and allergic rhinitis (if the duration is long). The runny nose is not related to the multiple food protein intolerance, but distended belly is related to the protein intolerance. Usually kids with malabsorption can have this. Kindly get him to a pediatrician so that it can be checked if he has any ascites due to low protein (especially albumin) in the body.Regards - Dr. Sumanth"
},
{
"id": 223103,
"tgt": "Can I get pregnant in spite of being on birth control?",
"src": "Patient: hi, me and my girlfriend are really worried right now because when we were having sex last night, the condom broke and i didnt notice so i ejaculated inside her, the condom was a trojan spermacide kind and she is also taking birth control for about a year and she never misses a pill, she also cleaned it all out after it happened, should we be worried about pregnancy? Doctor: If she is taking pills regularly than there is very very less chance that she would get pregnant. Chances are really negligible. Don't worry."
},
{
"id": 1693,
"tgt": "Is siphene 50 safe and effective for conceiving?",
"src": "Patient: Hi...I m 27years and my husband is 33years..nd my blood group is B neg and my husband s blood group is B positive...last jan 2014...I lost my baby in my 9th mnth...as if I knw that anti d injection was not given in my 7th mnth...which causes my baby s death...in my earlier pergnancy I had taken siphene 50 to conciev...nd nw from 2mth I m trying to concieve..so had taken siphene 50 again from my 3rd day..my cycle is now of 33days...my last period is on 21st may...is it safe to have the siphene again.. Doctor: Hi, you can take siphene for 12 cycles overall. After that it increases the chances of ovarian cancer. So, better to take it under the guidance of a doctor, because if you plan IUI, you will again have to take siphene. So, don't try more than 6 cycles with siphene naturally. Track your follicles growth by repeated ultrasound and when your follicles is more than 17 to 18 mm, take injection for rupturing the follicles. Be in contact with your husband for next 2 to 3 days. Take progesterone for next 2 weeks. Do a urine pregnancy test at home after that. This will increase chances of your pregnancy. Don't take siphene blindly. Hope I have answered your question. Regards Dr khushboo"
},
{
"id": 127722,
"tgt": "How to treat incontinence in a disabled wheelchair bound patient?",
"src": "Patient: I have a friend who has been in a wheel chair for 15 years and incontinence really keeping her home bound. She has taken some medications that are no longer working. I have seen ads on t.v. for some kind of procedure where patients said Sorry I waited so long . I feel sorry for this lady and would like to give her information on her options. She is about 83. Thanks, YYYY@YYYY Doctor: Hello,For incontinence, you can take tablet Urikind, do Kegel\u2019s maneuver. Also, go for ultrasound abdomen and pelvis.Treatment mainly depends upon underlying conditions. Kindly follow with detailed history or consult with physician.Hope I have answered your query. Let me know if I can assist you further.Regards,Dr. Shyam B. Kale"
},
{
"id": 188084,
"tgt": "What is the cure for pain and soreness in teeth, headache, ear pain and swelling in throat?",
"src": "Patient: My teeth on the right side are sore and throbbing and I have a headache like never before along with my temple and pulsating with pain and my throat is swelling below my chin as is my cheek. Also my ear is aching and everything hurts way more when I go to lay down Doctor: Hi,Thanks for asking the query,Toothache indicates infection of tooth.A detailed checkup and x-ray of the tooth is required.Root canal treatment has to be done.Start with a course of antibiotics and analgesics under the prescription of your Dentist.Take care!"
},
{
"id": 220877,
"tgt": "Can I get pregnant with non penetrative sex?",
"src": "Patient: My boyfriend and I dry humped. He made a mistake and ejaculated on my panites. Is there any chance that I could be pregnant? That was also the last day of my period. The 21 of May. My periods usually are about 35 to 38 days apart. I have been feeling gasey and a tad bit nauseous, but i think that its all in my mind because i only feel that way when I think about the possibility of being pregnant. Is there any chance that I could be? Please help me ASAP! Thanks Doctor: Chances of your getting pregnant are very remote.If u are so troubled u can get a simple urine pregnancy test done ."
},
{
"id": 58384,
"tgt": "Stomach pain, took pepto bismol, alka seltzer, applied heating pad, removed gallbladder. Recommendations?",
"src": "Patient: I am having a weird pain in my stomach. I took some pepto bismal and drank an alka seltzer and applied a heating pad, bu nothing seems to help. I also feel kind of shaky. The pain I'm feeling is similar to how I felt when I used to have gallbladder attacks, before I had my gallbladder removed. The pain seems to lessen slightl when I stand. Doctor: Hi ! Good morning.I would like to know if you consume alcoholic beverages or not. If you do, then I Would suggest you to get an ultrasound done to know the state of pancreas, and also go for a serum amylase and serum lipase test to support the diagnosis. If doubt exists, you may have to go for a CT scan as well.With a history of gall stones in the past, I would also like to know if there is no fresh formation of stones/calculi in the common bile duct, which may give rise to such pain not getting relieved with medicines. This can also be diagnosed with the help of an abdominal ultrasound. If diagnoses, I would refer you to the emergency room of a hospital if the pain is severe enough to disrupt your daily activity. If the pain is mild, then I would refer you to a general surgeon/gastro enterological surgeon for further management.In the mean time, you can start with a proton pump inhibitor, a prokinetic drug, and a probiotic. In case the pain was due to hyper acidity, these drugs will give you relief. Also, I will advise you to stop consumption of alcohol and stop smoking if you do. Avoid spicy and fried food specially from outside.I hope this information will help you out. Thanks for using healthcare magic for your health query. Wishing you an early recovery. Dr Shareef."
},
{
"id": 20622,
"tgt": "What causes fast heart beat and headache?",
"src": "Patient: My heart beats so hard that you can feel it in my stomache. My husband can lay his hand on my belly on the right side and when he pushes a little you can feel it beating. When I say beating I mean that you can feel it like a baby kicking. I have had 2 kids and that is exactly what it reminds me of. But when you feel my chest you can't really feel my heart beat. I have been tired and seeing spots a lot. I have also had a lot of headaches and it is really starting to worry me. Doctor: Hello!Welcome on HCM!I carefully passed through your question and would explain that your symptoms could be related to a metabolic disorder (chronic anemia, thyroid dysfunction, etc.) or low blood pressure. Have you checked your blood pressure values lately?Regarding the pulsations in the belly region they are related to a abdominal aorta. They can be felt normally in elegant persons. But they can be more prominent during chronic anemia or thyroid dysfunction. From the other hand an aneurysm of the abdominal aorta can not be excluded either. For this reason, I would recommend consulting with your attending physician for a careful physical exam and some tests: - a chest X ray study- a resting ECG- an abdominal ultrasound- complete blood count for anemia- thyroid hormone levels for thyroid dysfunction- blood electrolytes for electrolyte imbalance. You should discuss with your doctor on the above issues. Hope you will find this answer helpful!Wishing good health, Dr. Iliri"
},
{
"id": 14954,
"tgt": "What is the cure for honeycomb like looking rash on thigh?",
"src": "Patient: My sister age 33, has suddenly noticed a honeycomb or spiderweb looking rash on her right thigh, also her left leg is slightly swollen mostly in the calf to ankle area. No swelling in the leg with the rash. The rash is not raised like hives, does not itch or burn. No pain other than periodic muscle cramp in left leg. Doctor: Hello,Welcome to healthcare magic.From your description, the rash may be a pattern called as livedo reticularis.It is a pattern which could be associated with autoimmune disorders and antiphospholipid syndrome. This could also explain the painless swelling of the other leg.She requires a detailed evaluation for the same.However, a honeycomb rash on the thigh can also occur in a condition called erythema an igne. This occurs after keeping a hot surface in contact with the skin for a long time ( laptop, hot water bag, etc) . But this is not associated with limb swelling.Hope this helps you.Take care."
},
{
"id": 113763,
"tgt": "Back pain, bruising on right side of spine, no pain, knots in the back. Treatment?",
"src": "Patient: Hi I have notice there is a bruise look a like right on my spine . There is no pain on it either however I do have back pain for about 2 months know I am 20 years old female....this morning my back really hurted on my right middle side right abouve my ribs....most of the time its my upper back that is unbearable. I was told by couple of people that I have a lot of nots on my back ...help worried Doctor: Hi welcome to HCMForum. As you have described it appears to be herpes infection a viral infection with severe nerve pain. watery shiny boils seen along the course of the nerve. It should immediately treated with anti viral drugs to prevent perm- -anent pain due to nerve damage. if my diagnosis is correct. ofcourse there are other possibilities also so i advise you to consult a dermatologist for diagnosis and treatment. dont worry what ever it is curable completely if it is treated prom- -ptly and complely. thank you."
},
{
"id": 139253,
"tgt": "Suggest treatment for severe joint pain",
"src": "Patient: Sir i am suffering from joint pain in whole body Since 2011 there is no pain only in special place. I have taken penteds 800 1 year but no relief... And also take penidur injection after 3 weekly.. after taking I have tested ASO titer that s normal so my doctor it s stop at 30/04/2014 ... but now m feeling again this type of pain and again test of ASO. Whose value of 480 mg/dl and just start again penidur 12 LA.. And my echo test is normal.. So what I do??? Doctor: Dear Sir/Madam,I thank you for having faith in Health Care magic and for seeking help on your issue; I have gone through your symptoms, and in my opinion, you will need to consult a Rheumatologist to get your self diagnosed and then cured.I hope I have been able to provide you with a satisfactory solution, if you want to clear any doubts about my opinion, feel free to contact.Special note- Any medication prescribed needs to be taken after consultation with your personal doctor only.ThanksDr. Narender Saini"
},
{
"id": 194732,
"tgt": "What causes irritation in the rectum and pain in the testicles?",
"src": "Patient: respected sir i m from austria and patient of prostatitis since last 2 to 3 years.i have always pain there in my rectum like irritation and also uncomfortable in my testicles.i took already too much antibiotic but the symtoms are still.me also took ayurvedic medicin for 3 month course but that was also not sufficient.i m very tired for my sickness. tell me about urs method of medicin can i recover my sickness.is urs threapy is permanent solution of prostatitis.i hope u answer me soon. thanks a lot. Doctor: Hi, It could be due to conditions like prostitis. Consult a urologist and get evaluated. Nothing much to worry and you will probably improve with antibiotics. Hope I have answered your query. Let me know if I can assist you further. Regards, Dr. Shinas Hussain, General & Family Physician"
},
{
"id": 130135,
"tgt": "How to treat peripheral neuropathy?",
"src": "Patient: I am having alot of nerve pain and am scheduled to see a neroligist soon but wanted some answers now. I think I have Periperal Neuropathy from what i have looked up and I was wondering if there is some kind of decompression surgery they can do on your back to get the pain to stop? I have tried muscle relaxers and I am on a high dose of gabapentin and it is not working. Doctor: Hi i am Dr Ahmed Aly thanks for using healthcaremagic site ,I had gone through your question and understand your concerns .. In my opinion i think you may ask your physician for steroid injections which may be helpful prior to any further intervention , try chiropractor sessions , B12 vitamin supplementation , hot massages with topical gels and some spinal straightening exercises and even yoga are very helpful . Further researches mentioned TENs ( transcutaneous electrical nerve stimulation ) , acupuncture and plasma exchange . In my opinion i recommend MRI and a neurosurgeon point of view for minor endoscopic interventions are of very high value with nerve compression or even surgical fixation with rods and plates if needed .Please click and consider a 5 star rating with some positive feedback if the information was helpful. Wish you good health,Any further clarifications feel free to ask."
},
{
"id": 17550,
"tgt": "What causes chest discomforts, joint pain and headache?",
"src": "Patient: hi. 3 years ago i went on a night out with mates and took cocaine. the day after i got chest discomfort, nausea lightheaded i went to the hospital who done a ecg, blood test water sample and chest x-ray of which all come back normal. i have been in and out the hospital the past 3years they have never kept me in over night. my chest discomfort is not as bad as what it was i had it constanly when it first started now it comes and goes. i also get pain in left arm and jaw pain in both legs headaches dizziness do you think its heart related or anxiety Doctor: Hello, I understand your concern and would explain that your symptoms seem to be related to anxiety. Nevertheless, I would recommend consulting with your attending physician and performing some tests in order to exclude other possible causes: - an exercise cardiac stress test - complete blood count, PCR, ESR for inflammation - blood electrolytes for possible imbalance - thyroid hormone levels for thyroid gland dysfunction. Hope I have answered your query. Let me know if I can assist you further. Regards, Dr. Ilir Sharka, Cardiologist"
},
{
"id": 48563,
"tgt": "Suggest treatment for peripheral neuropathy",
"src": "Patient: My wife recently had an episode that resuted in a diagnosis of peripheral neuropathy. With that she had total nubness (if not paraysis) on the left side of her face and neck. As her symptoms (especially pain in hands and feet) have gotten better, she has had a ticklish cough throughout and it seems to be getting worse. She has coughing spells with clear mucus taht make her feel like she is suffocating. She also has tightness in her throst that gives her a feeling like she cannot swallow. Any ideas? Doctor: HIWell come to HCMIf this is pure peripheral neuropathy then underlying cause of this need to be found out, more over this may not be neuropathy but could be due to some functional condition still this can be treated with the following medicines.1) Tab carbamazepine 200 mg once in day 2) Tab Vitamin B12 once in day3) Tab Alprazolam 0.5 mg at bed time hope this information helps, have a nice day."
},
{
"id": 79434,
"tgt": "What is the treatment for pain in the right side of the chest?",
"src": "Patient: Hello I ve gone to two doctors for probaly the past month. This all started when I had pain on the right side of my chest they said it was a lung infection and gave me penicilin it helped and it went away but 3ish weeks later all of a sudden I felt very tired and dizzy and got a bad headache all of a sudden and the chest pain was back. They put me on penicilin again which is not helping anymore and then I go to another doctor and he says it s aniexty disorder which I strongly disagree with and I also have an itching burning feeling under my rib cage/ breasts on and headaches very tired and chest pain any idea what it could be? Doctor: Thanks for your question on Health Care Magic. I can understand your situation and problem. No, this is not anxiety related. By your history and description, possibility of recurrent lung infection or bronchitis is more in your case. So better to consult pulmonologist and get done clinical examination of respiratory system, chest x ray and PFT (Pulmonary Function Test). Chest x ray is needed to rule out lung infection. And if it is present than you need higher antibiotics because you are not improving with penicillin. PFT is needed to rule out bronchitis. It will also tell you about severity of the disease and treatment of bronchitis is based on severity only. You may need inhaled bronchodilators and inhaled corticosteroid. So better to first diagnose yourself and then start appropriate treatment. Hope I have solved your query. I will be happy to help you further. Wish you good health. Thanks."
},
{
"id": 119979,
"tgt": "What causes chest pain, shaking and severe spasms?",
"src": "Patient: teenager. Had 2 nights of chest pain, uncontrollable shaking, severe spasms to back and rib cage, passes after about 2 hours. Taste of blood in mouth but no blood. But is left with aching back and heavy dull chest. 2nd episode worse than first. Blood test, urine tests and ECG all showing as normal Doctor: Hi, This seems to be case of gastro-esophageal reflux disease. Esophageal varices may be present additionally. Endoscopy will be a diagnostic tool. Hope I have answered your question. Let me know if I can assist you further. Regards, Dr. Rohan Shanker Tiwari, Orthopedic Surgeon"
},
{
"id": 136144,
"tgt": "What causes shooting pain in the leg with a large bruise?",
"src": "Patient: Having shooting pains in my leg and have a huge rash like bruise. it started as what I thought was a bite of some sort after I itched it I noticed it was all red and purple around the red. I am having shooting pains and a bit of numbness if I am still Doctor: tankx for asking me the qn..first you see any bite mark there...or see the bruise is black or red in colour...dont worry...if uou cudnt see any bite marks drn you can start treatment...apply beclomethasone ointment thrice a daytab.ibrufen 1-0-1 for 4 dayatab.ranitidine 150 mg 1-0-1 for 4 daysjus wait for 1 week and let me knw the results..i hope i answered your query!"
},
{
"id": 208507,
"tgt": "What causes lack of emotional feelings for someone?",
"src": "Patient: Hi i think ive experienced some kind of shock with my boyfriend..with him and only with him i feel no intuition, no emotions..neither sadness happiness pleasant feelings..i feel llike a robot...i cant react the way i want to say what i want..like a part of my consciousness is gone...whats wrong with me and how do i fix this? Doctor: hi dear,what kind of shock you have with your boy friend?since when you have such complain?with other friends and family member you have any problem related to emotion or not?is there any other associated symptoms?consult psychiatrist for detail history and counselling.Thank you"
},
{
"id": 123184,
"tgt": "What cause mild indentation of thecal sac compression?",
"src": "Patient: hi yes i was told today that i have mild indentiation of thecal sae compression and I m still confused if it is reall serious. I will be getting a back specialist hopfully soon. but i m not finding a lot of info about it just questions others leave online. Doctor: Hello, As the thecal sac compression is not a major thing to worry if noticed on an MRI. Because we always have to correlate the MRI results with the clinical symptoms. Usually, this thecal sac compression happens due to the weakness of the muscles of the spinal column leading to changes in the normal alignment of the spinal column. This inturn leads to thecal sac compression. Having cervical collar if it's the neck region and lumbar brace if it's the lumbar region, it will provide stability to the spinal column involved. Exercise to improve the stability and strength will be advised to current the alignment. Hope I have answered your query. Let me know if I can assist you further. Regards, Jay Indravadan Patel, Physical Therapist or Physiotherapist"
},
{
"id": 25240,
"tgt": "What precautions are to be taken in patients with stent by angioplasty?",
"src": "Patient: I am on medicines: Olmin-H, Tonact-20, Tolol XR 500, Deplatt A -75, Ranozex-500 and Nikoran-5My main arteries have stents by Angioplasty....Side small branches are having blockage...so during walk after 10 mins I used to have some disturbance...so after angiography Dr. gave me additional mediceine Nikoran.....I am fine now.....What precautions I should have now Doctor: Hello and thank you for using HCM.I carefully read your question and I understand your concern.I will try to explain and give you my opinion. First of all , it is a very good thing that you have resolved the problem whith main arteryes.You should know that during first 6 month it is a risk for stent stenosis.So it is mandatory to make regular control after 1,3 and 6 months. For the future your only duty is to fight and controll what we call carfiovascular risk faktors.This factors are Hypertension, diabetes, high cholesterol levels, smoking. So if you are hypertensive you should try to keep your blood pressure below 140 / 90.If you see excessive values you should meet your doctor to change therapy.You should keep normal glicemic values if you are diabetic.You should not smoke and the most importat one you should keep your cholesterol as low as possible. Your LDL cholesterol levels should be below 70 mg/dl. You are under treatment know and you should always take the medicine and keep a healthy diet.It is essential to eat as much fibre, vegetables and avoid high cholesterol food, fast food, pizzas, sweeties. It is recommended to have regullar physical activity lime walking around 30-40 minutes a day.Hope I was helpfull.Best regards"
},
{
"id": 208260,
"tgt": "Suggest remedy to overcome fear of all things",
"src": "Patient: My name is aishathI'm 24, 5\"247.2kgsmy problem is i get afraid of small small thingseven at day time I'm afraid to be alonei Don't see anything which makes me afraid yet i feel scared.. if i got up late i'm not able to sleep again...tell me a solution please Doctor: DearWe understand your concernsI went through your details. I suggest you not to worry much. From the description you gave, I am to understand that you could be troubled with generalized anxiety disorder with some specific phobias or fears. Generalized anxiety disorder is normal with 60% of population and does not need any medications. But needs life style changes and relaxation. But you should get yourself assessed for the depth of the problem. Consult a clinical psychologist for assessment.If you require more of my help in this aspect, Please post a direct question to me in this website. Make sure that you include every minute details possible. I shall prescribe the needed psychotherapy techniques which should help you cure your condition further.Hope this answers your query. Available for further clarifications.Good luck."
},
{
"id": 75912,
"tgt": "Suggest treatment for severe cough",
"src": "Patient: My mom a type 2 diabetic giving insulin twice a day since last 15 yrs has been badly coughing for good 9 months after coming to apollo kolkata she was asked to undergo a bronchoscopy which states her both right and left airways and trachea are filled with creamy pus and she is diagonaised with bronchiolitis a very bad lung infection.. She is taking injection broadactam 4.5 thrice a day for 7 days. Do you think this is the remedy for her? And being a type 2 diabetic what are her chances of recovery and what precautions should she take or does she need to go for more advance tratment? Please help also suggest me best pulmonologist in kolkata im showing dr ashok sengupta and i want to change my doctor!! Doctor: Thanks for your question on Healthcare Magic. I can understand your concern. Diabetic patients are at increased risk of bacterial, fungal and parasitic infection due to poor immunity. Lung infection is especially more common because lungs are exposed to environmental air directly. Broadbectum injection is for bacterial infection. Best treatment for lung infection should be guided according to culture and sensitivity report. Culture report will isolate the causative organism and sensitivity report will guide about effective antibiotic treatment. So get done repeat bronchoscopy and BAL (bronchoalveolar lavage) analysis, bronchoscopic Biopsy for further evaluation. Also get done BAL for fungal culture and anaerobic culture to diagnose fungal and uncommon bacterial infection. On the basis of these reports, she might need antifungal or other higher antibiotic with anaerobic coverage. In Kolkata, visit wokhardt hospital. Hope I have solved your query. I will be happy to help you further. Wishing good health to your mother. Thanks."
},
{
"id": 11674,
"tgt": "White spots around lips, pigmentation on face. Using creams. Permanent cure?",
"src": "Patient: I have some white spots around my lips , pigmentation on face...right now i m using venusia craem and taking Keraglo Fore tablets...I have also thyroid problems....do shifting duty...How can i remove white spots and pigmentation from my skin? It happens 1 and 1/2 yaers ago..at that time doctor give me elovera and momisoft cream to use first 10 months, then told me to apply only elovera cream...after that three moths later told me to use Clindamycin gel and Alovera cream together but it makes my skin more pigmented...now i m using Venusia cream...my kin & hair is very much dry...what i should do? Doctor: Hi,Thanks for writing to us.There could be many causes for white spots like P alba, vitiligo, naevi, post inflammatory hypopigmentation etc. Similarly the pigmentation could be melasma, sun damage, post inflammatory hyperpigmentation etc. It is dfficult to suggest a treatment without examining the lesions. The treatment that you are using right now is inappropriate and wont help in treating these specific concerns. Please meet a qualified dermatologist nearby for diagnosis based appropriate treatment.TAke care"
},
{
"id": 9681,
"tgt": "Dark patch in area near right ankle, but there is no pain",
"src": "Patient: Hi, i have a black patch on the area near my right leg ankle since a month. That portion also does appear slightly swollen though there is no pain. what is the cause for this dark patch and swelling? many thanks. I am 28 years old and am 53 kgs with no medical history . Doctor: hi u have dark elevated patch suggest u may have nevus or dermatitis. itch is there? ? apply good moisturizer cream three times a day. better if u send me ur pics. dont scratch or prick it"
},
{
"id": 15995,
"tgt": "Itchy rash on the foot. Dryness with scales. Cure?",
"src": "Patient: Hello I have an itchy rash on the instep of my left foot . it is not red. it is a 2 patch, darker colour to my skin . it appears dry and a little scaly. it began when after removing my sock at the end of a days work i scratched the spot because it itched. i scratched until i removed the upper layer of skin. the spot became irritated and turned into this. Doctor: hi..thanks for the query From what you describe, you might be suffering from foot eczema.You can follow the following steps: 1) Use ample emollients or medicated moisturizer multiple times in a day to keep the area moist and hydrated. 2) Use only pure cotton socks 3)Have oral anti-histaminics like Tab.Allerga to relieve the itch.It is important not to scratch. 4)Topical corticosteroids may be required in your case.These relieve the itch and reduce the eczema pathology. It is important to remember that the lesions are recurrent and it is important to do follow ups for reviewing treatments from time to time with your dermatologist. Hope this helps..Take care"
},
{
"id": 48714,
"tgt": "What causes blood and pus in urine after undergoing lithotripsy?",
"src": "Patient: I am 32 year old female that had a lithrotripsy done three weeks ago on a 5mm stone and now I have blood and pus in urine and another stone doctor is going to explore it in the morning to see if it is lodged in uretha what can be causing all of this now Doctor: Hi,Thanks for writing in.Most persons pass blood in their urine after the lithotripsy procedure. This is normal and should clear after several days to a week. Blood and pus can be due to injury of ureter by dislodged smaller fragment and subsequent infection. Lots of fluids should be taken to encourage the flushing of any gravel remaining in the urinary system. Treated persons should follow up with a urologist in about two weeks to make sure that everything is progressing as planned. You may need to take a course of antibiotics to control infection."
},
{
"id": 222727,
"tgt": "Is taking Utrogestan 200 orally instead of inserting it vaginally harmful?",
"src": "Patient: hi, I have a similar question than before: Im 8 wweks pregnant and had been ask to take utrogestan 200 for a month( i had abdominal pan , but uterus was fine and no bleeding) however i just realize it was suppose to be taken vaginally and i took it orally!! Im worried it might harm the baby?? I have had horrible headches and migranes since last saturday and they dont stop, I now realize that this might be the cause?? can u tell me if it will harm the baby ..please! Im worried . thank you, alejandra. Doctor: Hello dear,I.understand your concern.In my opinion the utrogestan can be taken orally or vaginally.With vaginal insertion the absorption is better and systemic sideeffects like nausea,headache are less than oral intake.But there is no harm to you or your baby with oral intake.So relax.Take panadol for headache.Avoid painkillers in pregnancy.Best regards...."
},
{
"id": 202793,
"tgt": "What diet could be taken to increase the strength of the penis?",
"src": "Patient: Dear sir, Recently I am suffering from night fall..sir after 3 month I will be married..my pennis is not so long and I am fearing about my marriage..sir,please sujjest me for food and medicene which help me to improve the strongness and for my narriage Doctor: HelloThere are few misconception in the mind of man about SEX and size of PENIS in all over world.Let me explain that the normal size of PENIS after full erection is 10-15 cms ( 4\"-6\") .As far as concerned about the thickness of the penis it depends upon the muscles girth in penis .The pleasure of sex ( coitus or intercourse ) depends upon the time taken by a male or female to reach orgasm ( i.e. ejaculation ).So in my opinion night fall is a natural process ,so don't bother about that.If premature ejaculation is there then , you must consult a psychiatrist and get its treatment ( but mind one thing don't entrap in the net of QUACKS).Hope this information will be helpful for you.Since all medicines are for prescription so consult a psychiatrist.In my opinion food don't increases length of penis.Good luck."
},
{
"id": 157477,
"tgt": "Have three growths in neck, advised FNB after ultrasound. Is it goiter or cancer? Have family history of cancer",
"src": "Patient: Good evening, I had an ultrasound last Wednesday and was told I need a FNB tomorrow. I have 3 growths on my neck, largest approx 5 cm, noticed only in the past 6 weeks. Do you think it's a goiter or possible cancer? I have 3 sisters with cancer. One of my sisters had sarcoma on sternum and uterine, all 3 had breast cancer. Doctor: Dear,It neither seems to be prostate cancer nor a goitre.The nodes need to be evaluated. FNB or open biopsy of one of these nodes will confirm the problem. Medically there are many reasons to have the lymphadenopathy in the neck. Let the biopsy confirm it.Truly,Dr. J. Ticku"
},
{
"id": 153791,
"tgt": "What are the damages caused due to over dosage of radiation?",
"src": "Patient: My father was treated for prostate cancer and was given radioactive seeds. The process was quite a few years ago when the seeds were new. He was grossly overdosed and is suffering with the damages of the radiation. We would like to find a doctor anywhere in the US that might help him. Where do we look and what category do we search? Doctor: Hi,Thanks for writing in.You exactly need to discuss the side effects which your father is experiencing probably due to overdose of radiation. Radiation can virtually affect any organ system of the body and it will be essential to know what exactly is troubling him right now, years after taking radioactive seeds treatment for cancer of prostate.Treatment of radiation side effects is by getting attention for the particular problem that he is facing. It might be bowel related, urinary system related or spine related especially for radiation given to prostate. The best person will be to consult a surgeon and neurologist and then proceed with the confirmation of the condition and treatment. Please do not worry."
},
{
"id": 143939,
"tgt": "What causes soft spots on head?",
"src": "Patient: Mom has about 3 soft spots on her head. She . Has fallen but had a ct scan at hospital. They didn t see anything at the time. Mom recently was taken off of Coumadin. Also she s been dehydrated lately from the fluid pills. Could any of these cause the soft squishy spot that I feel in my mom s head? Doctor: Hello and thanks for using HCM.I have read your question and understand your concerns.Soft squishy spots on head after head trauma are mainly related to accumulated blood under the skin.This possibility is increased by Coumadin use that increases bleeding possibility.Fluid or water pills are not related to these soft spots.If these spots tend to grow in size, evacuation through a syringe and elastic bandage may be necessary.Hope you found the answer helpful.Take care."
},
{
"id": 8944,
"tgt": "Why is there color difference between my right leg and hand ?",
"src": "Patient: my question is to skin doctor .sir/mam, i m in a great problem. my right hand & leg is lees bright than my left though i take care of both,but how it is possible? pls give me a solution,my skin type is normal.i noticed it about 3weeks ago,what should i do? pls pls pls pls pls pls pls pls give me the solution.my age is 22, Doctor: Hi...dear user., Thanks for choosing HCM.., Don't fear about this.....! If you have numbness and pain present either of limb ... then think of...ok.., According to your complaint ...seemsw very simple.., It could be.......Due to Sun tanning on one side...., It is due to constant exposure of that part in the SUN....ok.., So don't worry ...be positivwe ..take care of your health..good luck"
},
{
"id": 158373,
"tgt": "Have breast axillary tissue under arm pit. Is it growing? Is it cancer?",
"src": "Patient: Hi, I don t know if you re the right Doctor to put this question to but here it goes... I have what is known as breast axillary tissue under my left arm pit. It has been there ever since I hit puberty and I am now 30 years old. I have 4 children and gave birth to my last child last November. I have noticed that it is now bigger. Its become a bit of an annoyance to me and its noticeable in my clothing. I think its so obvious to everyone. I have an appointment with a surgeon but I don t want to go empty-handed. I don t know if I should do a mammogram , ultrasound or x-ray? please advise me! also do u think this growth spurt may be a symptom of early cancer or is it just due to my recent pregnancy? Doctor: Hi there, The breast axillary tissue is called axillary tail of spence and is a normal finding. Let the surgeon examine you thoroughly and let him advise for any investigations if needed. I'm sure you can inform him of your concerns etc but a good physical examination is most important. Take care, Regards"
},
{
"id": 98200,
"tgt": "ls there any alternative treatment for hernias ?",
"src": "Patient: ls there any alternative (NON SURGICAL) treatment for Inguinal hernias ?I have hernia at 2 places opening is 25 mm and 31 mm.There is no pain no other complication.My height is 175 CMs.Weight 86 K.G.Will reducing weight help? Doctor: You are around 10 to 15 kg over weight Kindly reduce it For hernia there is no effective remedy in alternate medicine and you need to get operated Till you get operated you can try the Hernia Belt of VISCO company"
},
{
"id": 160620,
"tgt": "What could cause low grade fever and headache?",
"src": "Patient: My niece is 4 yrs old and has a low grade fever and a headache. She has been to the doctor and is on amoxicillin. She started out with a headache, then a low grade fever that escalated to 105 and vomiting. My concern is she has been on antibiotics for 2 days, she was tested for strep throat and that was negative. What could be causing the headache? Doctor: Hi, Does she have abdominal pain or loose stools? Fever, headache and vomiting is most likely from a viral fever. Let her take some rest, take paracetamol 3 times daily even if afebrile(for headache) and drink plenty of fluids in small frequent sips. This will subside in 3-4 days. Nothing to worry if she is otherwise playful and taking feeds. Get back to doctor if fever does not subside by then, headache is sever or vomiting persists. Hope I have answered your query. Let me know if I can assist you further. Take care Regards, Dr. Muhammed Aslam TK"
},
{
"id": 190630,
"tgt": "Wisdom teeth removed, loss of sensation in tongue, mouth ulcers, lose papillae. Taking zevit, lyrica, folvite. Treatment?",
"src": "Patient: hi, i had 4 wisdom teeth removed 6weeks back and since then left side of my tomgue has completely lost sensation. now the problem is i keep biting y tongue from left innerside at the end. lot of mouth ulcers also hurt me a lot. yesterday i bit my front right part of tongue by mistake and one papillae became loose. it kept hurting me and today night it has come off.( a small pink round ball). Is this normal. What should i do for the frequent ulcers. I am taking tablets - zevit, lyrica , folvite (all one each) from 5weeks. what should i do to make my tongue normal. pls help..its affecting me physcologically as well Doctor: Ok, first of all you should have consulted your dentist atleast 5 weeks back, as probably your left lingual nerve may have gotten injured by the needle used for Local Anesthesia (LA). Now, since you are already using Zevit, Lyrica, & Folvite, for 5 weeks, and still you have no improvement, then the injury may be severe, and it is high time that you cunsulted a neurologist. Do ask him/her wether adding Neurobion would help, or would need to see a neurosurgeon. In the meanwhile, ask you Dentist though to fabricate a suitable Oral Screen with an incorporated Tongue Gaurd to protect your tongue for most of the time."
},
{
"id": 87767,
"tgt": "What causes pain in upper left stomach area just below ribs?",
"src": "Patient: I have pain on my upper left stomach area, right below my ribs. If I am standing it really doesn t hurt but setting or leaning forward does. It also has a sensation like something there is knotting up and almost like it is moving. The pain isn t severe but it is pretty constant. I am 44 yr old female, over weight with high blood pressure. Thanks Doctor: Hi! Good afternoon. I am Dr Shareef answering your query.If I were your doctor, I would like to have a clinical examination of yours followed by an ultrasound of whole abdomen to see the status of all the intra abdominal organs or any other related lumps. Apart from this, I would also go for some routine tests like a CBC, a blood sugar, serum amylase and lipase, and a LFT. A routine ECG would also be worth doing. I would also advise you to avoid any alcoholic beverages or smoking if you do. Further management would depend on the clinical findings, and investigation reports. Till then, you could go for a anti spasmodic and a proton pump inhibitor drug for a symptomatic relief. I hope this information would help you in discussing with your family physician/treating doctor in further management of your problem. Please do not hesitate to ask in case of any further doubts.Thanks for choosing health care magic to clear doubts on your health problems. I wish you an early recovery. Dr Shareef."
},
{
"id": 86064,
"tgt": "What causes abdominal pain and diarrhea?",
"src": "Patient: Since April 28 intermittent episodes of abd pain nausea vomiting watery stools at times severe have to go to er. Other times last couple hours. When the episodes are severe will start with belching rotten eggs then nausea vomiting then watery brown stools. Er said go to primary md primary md gave prilosec and abd CT done normal. Waiting gi consult. Since April 28 I'll with symptoms at least once a week. In between times feel fine. Never know when it will hit. History of these symptoms for years no diagnosis given. Doctor: Hello and Welcome to \u2018Ask A Doctor\u2019 service. I have reviewed your query and here is my advice. * As far as my clinical experience is concerned, this is intestine infection. * Primary guidelines to manage the same - Plenty of liquids in sessions. - Bland vegan diet with organic fresh fruits. - Anti emetic, PPI with GI specific antibiotic prescription with your doc. - Add probiotics in natural form as fresh curd in diet. Do give me further chance to assist in future. Hope I have answered your query. Let me know if I can assist you further."
},
{
"id": 1581,
"tgt": "Any chances of getting pregnant after missing pill?",
"src": "Patient: I am taking Femilon contraceptive pills, I usually take it late night, at around 10.30pm-11pm. Yesterday which was 17th day of my cycle I forgot to take the pill, I took it today morning at 9.15am. I had unprotected sex on Thrusday, which was 16th day of my cycle. What are my chances of getting pregnant? Doctor: Hi , How are you doing ?I don't see any reason for concern. Femilone is hormones which stop the normal ovulation which should have occurred around Day 13-14. So now you have forgotten pill in the comparatively safe period & luckily have taken the forgotten pill within 12 hrs. There won't be any problem, just keep taking the remaining pills, as before at night. Pregnancy chances are nearly Zero. Don't worry, But don't forget it againHope I have cleared your queryAll the best Dr.Balakrishnan"
},
{
"id": 173363,
"tgt": "What is the white dot on baby's gum?",
"src": "Patient: Hi..im myra from Malaysia...i noticed something came out like pus on my 6 months baby gums. its only 1 small white dot on his gum..i assumed was a ulser but its not..can u advice me what should i do and any medcine that i can use because this is my first son and im so worrieddd...thank you Doctor: Hi, I had gone through your question and understand your concerns. You should be worried but not too much. You may easily wash his mouth with water and baking soda powder. Drop 5 drops of 1 % Candid trice a day and apply Pantosal to affected area. Hope this answers your question. If you have additional questions or follow up questions then please do not hesitate in writing to us. I will be happy to answer your questions. Wishing your baby a good health."
},
{
"id": 191941,
"tgt": "What causes high blood pressure while suffering from diabetes?",
"src": "Patient: My wife is diabetic, has high blood pressure and high cholesterol as well as generalized anxiety disorder. She had stopped taking all her medications and her anxiety level increased. She is back on her medications. This morning she was very dizzy and fell down a few times. I did not give her the prescribed medication for blood pressure this morning...Co-candestran HTC 16/12.5mg. I was able to get hold of a instrument to measure blood pressure. I just checked her pressure and it was 104 over 69. The question is...should I give her the pressure medication if her pressure is under 120 over 80? Is it possible that her dizziness in the morning was caused by low blood pressure? Thank you for your kind response. Sorry, I tried to proceed and found out that advise required payment. I am in Toronto Canada and we have free medical consultation paid by OHIP...Thank you all the same Doctor: HI, thanks for using healthcare magicIt is possible that her dizziness is related to her blood pressure readings especially if her reading is normally higher than this.You may want to withhold the medication for now and consult her doctor. She may need a trial off or with reduced medication to see what her readings are like.I hope this helps"
},
{
"id": 218309,
"tgt": "How can diarrhea and abdominal cramps during pregnancy be treated?",
"src": "Patient: Hi I'm 21 weeks pregnant with my third child . I am having problems with diarrhea and cramps. When I had my scan a week and a half ago everything with the baby was absolutely fine and she is moving to and kicking. I am not bleeding. Is there anything I can do Doctor: Hi, Diarrhoea and cramping in pregnancy can be treated with probiotics such as Sporolac tablets thrice daily for 3 days. Drink lots of water and avoid spicy and fatty food. It's safe to use Sporolac tablets for diarrhea in pregnancy. Try to avoid food from outside. Hope I have answered your query. Let me know if I can assist you further."
},
{
"id": 85367,
"tgt": "Is there any risk of stroke after stopping the medication?",
"src": "Patient: My doc asked me to stop taking Losartan Potassium HCTZ (50 & 12.5 mg) because of side effects like sleep problems, eye pain and increase in heart palpitations. I stopped 4 days so far after taking it almost 18 months and my BP now is 125 / 87. I m 40+. Any risk of stroke? Doctor: Hello, A daytime blood pressure above 130/80 (either one of those numbers) is considered to be high blood pressure, which is a known risk factor for stroke. So if your BP continues to have a lower number higher than 80, I would consider talking with your doctor about how to manage that. Diet and exercise can often lower it without medications as well. That being said, if this is your only risk factor for stroke, and your cholesterol is good, and no diabetes, I would consider your risk for stroke to be very low.Hope I have answered your query. Let me know if I can assist you further. Regards, Dr. Aaron Branch, General & Family Physician"
},
{
"id": 194730,
"tgt": "Why is there no ejaculation of sperm in intercourse?",
"src": "Patient: i am venkatesh from Bangalore , my age 41 sperm is not coming while doing intercourse if 1 hour play also since 6 months and if I do intercourse full of my head and body getting irritaion feel like rub and i will get side lower stomach and back pain kindly suggest me and also i am having uric acid problem 6.5% Doctor: Hi, Consult a urologist and get evaluated. We have to rule out possible causes like retrograde ejaculation. Hope I have answered your query. Let me know if I can assist you further. Regards, Dr. Shinas Hussain, General & Family Physician"
},
{
"id": 7414,
"tgt": "Pus filled painful pimples around pelvic area and legs. What is the treatment?",
"src": "Patient: my son has several pimples on him, very painful, liquid to hard puss with black dots after popping them, punget order, yellow to white puss, bleeds alot after puss is gone there is a little black dot deep inside, mainly around his butt cheeks, pelvic area and now on his legse, about 8 of them spread throughout thiese areas Doctor: Hi..user.., It could be Bacterial infection .., called...IMPETIGO CONTAGIOSUM.., So use Topical antiboitics....Mupirocin.., Oral Cephalexin tablets.., Cleanzing the lesion parts..,"
},
{
"id": 217200,
"tgt": "Suggest remedy for jaw pain extending upto neck and chest",
"src": "Patient: i have a severe pain starting in my jaw that spreads down my neck into my chest. it then travells into the muscles of my arms down into my fingers. the pain is so bad i can t sit stand lay down or walk and it hurts bad. it makes my arms want to curl and it makes me want to bend over . it started about 3 weeks ago a couple of times a day. now it won t go away and i m in agony. Doctor: This looks like it can be related with your heart, cervical spine or jaw joint weak ness. For heart just check do you have pain and breathing difficulty on walking up hills or walking for long distance ? If yes then do your electrocardiogram with tredmile test and visit cardiologist. If no then follow the next.Check do you have regular pain from your neck to radiating to your hand as well to head? As well do you have any numbness or tingling or pulling feeling in hand? If yes then it can be related to your cervical spine. You need to do a MRI of cervical spine and check with ortho doctor.If no then check next,Do you feel pain starting only from jaw joint and radiating down, as well chewing difficulty if yes then again need to visit ortho here based on assessment he will suggest you either surgical or medical conservative treatment.In any case you need to visit doctor and get it evaluated so visit doctor and take proper treatment.You can also use hot pack for temporary relief at home frequently with soothing and relaxing effect on soft tissues.Take care."
},
{
"id": 81042,
"tgt": "What causes tightness in the chest and irregular breathing?",
"src": "Patient: I have Dystonia, muscles in my neck, back, shoulder, and hip and involved. I have recently been having pain and tightness in lower ribs. My breathing is irregular and i have a hard time expanding my lungs. i often feel light headed and will see stars. Could this be another simtom of my Dystonia? should i be worried? Doctor: Thanks for your question on HCM.I can understand your situation and problem.Yes, your dystonia can cause breathing difficulty.Breathing consists inspiration and expiration.Both these are done by muscles.Inspiratory muscles expand the rib cage and allows lungs to take air in.In expiration lung recoil and expiratory muscles allow air to breath out from the lungs.So in both these activities, muscles play very important role.Since you are suffering from dystonia, possibility of involvement of muscles of respiration is high.So dystonia is seems to be the cause for your breathing difficulty.Consult your doctor and discuss all these and start appropriate treatment."
},
{
"id": 89481,
"tgt": "What causes fullness and pain in lower abdomen with no appetite?",
"src": "Patient: For the past 3 days I have had a fullness and pain in my lower abdomen. I have no appetite before mid-afternoon. Simply seem to disappear by 7-8PM. My guess was a bladder infection, but no pain with urination. I am a 34 year old female and am positive I am not pregnant. Doctor: HI.The causes of Fullness and Pain in Lower Abdomen with loss of appetite can be due to Enteritis , Typhoid, and such problems of the intestine. This is a natural process when there is intestinal infection or affection of whatever reason , the first thing the body responds by loss of appetite , This is a natural protective mechanism. I would advise the blood tests _ CBC , widal for typhod , culture and sensitivity of the blood for typhoid, liver functions as hepatitis is one of the the important reasons for loss of appetite. Ultrasonography if required by the Doctor who would examine you and prescribe you the proper treatment . This can not really occur due to Urinary Bladder infections"
},
{
"id": 88514,
"tgt": "What is the treatment for pain in the waist and stomach?",
"src": "Patient: I have pain on my right side about the waistline. It also hurts on my right side in my stomach area. I thought I had a bladder infection and my doctor prescribed Sulfamethoxazole - tmp 160mg for 10 days. It seemed to relieve it, but the pain perists. Thanks, Nancy Doctor: Hi Nancy.Thanks for your query.The cause of the pain above the waistline on the right side has to be diagnosed on physical examination by a Surgeon and diagnostic tests undertaken to prove or to rule out the probable causes. The probable causes are:Right kidney infection / impacted stone in pelvi-ureteric junction.Gall bladder stones and / or infection.Liver pathology.Colitis on the right side.Appendicitis particularly in retro-cecal position. and so on.I would advise you the following Get the blood , urine and stool tests done. Ultrasonography as a preliminary survey to find the cause. CT scan of the abdomen and other investigations to confirm.Take a proper course of an antibiotic, metronidazole and symptomatic treatment . You may need an additional antibiotic with the Sulfamethoxazole."
},
{
"id": 46598,
"tgt": "What is the procedure to get an indwelling catheter?",
"src": "Patient: Dear Sir, I need a help from you about my father's kidney dialysis. he got bypass for heart attack in 2005.He is suffering diabetic from 1990. Now his age is 59 named Md. Abul Hashem. He has also Kidney problem. his creatinin is 435. He got already three fistula which none of these is working. He is getting dialysis by catheter. But if it is continuing it might be infected. I heard that there is a permanent catheter available in India. So please suggest me what should I do for my father. If there is any treatment available , please let me know that I can arrange necessary steps.Regards & ThanksJasim UddinE-mail: YYYY@YYYY contact no- 00-88-0000Country: Dhaka, Bangladesh. Doctor: it is not correct that all peritoneal catheters will be infected. if done under strict aseptic method you can continue dialysis with that also. permanent catheters are not long term solutions , it actors for 6 month to 1 yr only."
},
{
"id": 78491,
"tgt": "What are the symptoms of lung cancer?",
"src": "Patient: Hi I have a dull ache in my back feels quite sharp and damp ? Also pain on same side in my shoulder Iam a smoker the doctor has gave me an antibiotic Iam worried its lung cancer Iam 33 also been out of breath and coughing up flem no blood just green pls help thanks Doctor: Thanks for your question on Health Care Magic. I can understand your concern. Since you are smoker, possibility of bronchitis, lung infection and lung cancer is high. All these can cause similar symptoms. But lung cancer is less likely because of your age. But better to consult pulmonologist and get done 1. Clinical examination of respiratory system 2. Chest x ray 3. PFT (Pulmonary Function Test). Chest x ray is needed to rule out lung infection and lung cancer. PFT is must for the diagnosis of bronchitis. You may need inhaled bronchodilators, inhaled corticosteroid (ICS) and antibiotics according to the cause. You need to quit smoking as soon as possible. So consult pulmonologist and discuss all these. Hope I have solved your query. Wish you good health. Thanks."
},
{
"id": 6974,
"tgt": "My follicles are ruptured but not able to find foetus. Why is it ?",
"src": "Patient: Hello Doctor , I have one doubt. i went to clinic on 30th. that time my was ruptured. doctor to me to have contact that night. But Doctor we can find out at what exactly it was ruptured. Bcoz 29th afternoon we had contact. we can not find fetus is there or not. Doctor: Hi Welcome to HealthcareMagic Pregnancy occurs when sexual intercourse is done around the time of ovulation. you had ovulation on 30 th ( eggs would have got ruptured) so your doctor asked you to make contact on 30th to increase chances of pregnancy. If you miss your next periods confirm pregnancy by doing urine pregnancy test and consult your doctor. Take care."
},
{
"id": 10519,
"tgt": "Can hair loss be genetically related?",
"src": "Patient: hi i am 27 years old suffering from hair loss and consulted a docter and been adv to apply x-pecia gel twice a day and shampoo ketoconazole and zine pyrithione shampoo , bio-hgf forte capsules and xpecia 1mt capsules , reason for my hair fall as advised by doc was genetic so is it a right advise Doctor: Hello and Welcome to \u2018Ask A Doctor\u2019 service. I have reviewed your query and here is my advice. As per your case history of hair fall, my treatment advice is - 1. Androgenic alopecia is a genetic hair fall. 2. Take good nutritious diet full of green leafy vegetables and milk. 3. Take an iron supplement and vitamin b12 supplement. If problem persists then consult a dermatologist. Hope I have answered your query. Let me know if I can assist you further"
},
{
"id": 143648,
"tgt": "Can blurred vision, headaches and sleeplessness be due to concussions?",
"src": "Patient: My son has had 3 concussion over a year period. These were over a year ago. He has has reoccurring headaches, sleep issues since then. Lately he has experienced blurred vision in one eye after being hit in the head by a ball in gym or in games with friends. Can this all be a result of the concussions that were over a year ago? Doctor: Hi, Welcome to HealthCareMagic.com I am Dr.J.Mariano Anto Bruno Mascarenhas. I have gone through your query with diligence and would like you to know that I am here to help you.Blurred vision, headache, sleeplessness can occur due to concussionHope you found the answer helpful.If you need any clarification / have doubts / have additional questions / have follow up questions, then please do not hesitate in asking again. I will be happy to answer your questions.Best Wishes for Speedy Recovery Let me know if I can assist you further.Take care.-oOo-PS 1 : After all your doubts have been cleared, kindly (a) Close this Question (b) Rate my Replies and (c) Give your Feedback. PS 2 : In the future, for continuity of care, I encourage you to contact me directly in HealthCareMagic at http://bit.ly/askdrbruno"
},
{
"id": 10730,
"tgt": "Suggest treatment to reduce hair fall",
"src": "Patient: My hair falls out all day long, every day. It does NOT fall out in patches or clumps. It falls out sparingly but all day long. I don't do pony-tails. I don't iron my hair. I don't even blow dry my hair. And it falls out even more if I brush it, comb it or wash it. I wash it with an organic coconut shampoo and use the same type of conditioner. I'm 44 years old and relatively healthy. I also eat pretty healthy. By the way, I use to have a great deal of thick, shiny, healthy hair. I probably have less than a quarter of that now. The only reason I think I haven't gone bald is because it does grow.... but obviously, not at the rate that it falls out. It's as if the follicles won't hold on to the hairs strands. Please let me know if there's anything that may be causing this and / or if there's anything I can do. Thank you. Doctor: Hello and Welcome to \u2018Ask A Doctor\u2019 service.I have reviewed your query and here is my advice.Given your history and medical details, I assume you are a 44 years old female experiencing this hair loss. How long ago did it start? Any probable causes you can think of prior to it starting? Maybe an illness, fever, typhoid, prolonged stress or trauma, change in location of residence or lifestyle, any new medications started, or use of any hair products or dyes previously? Is the skin of your scalp is normal? Have you checked any history of thyroid illness? Is there a family history? Female pattern baldness could be a reason, mainly genetic. Going by your age, hormonal changes close to menopause could also affect hair fall and growth cycle, slowing it down considerably. Is there hair loss elsewhere on the body? Narrowing down on a probable cause can help make changes.As for treatment, continue your healthy balanced diet with sufficient protein, and veggies for vitamins and minerals. Adequate water, rest. Avoid stress and pollution. Continue avoiding chemical hair products and styling. Use a hair growth shampoo and conditioner like hair4U or Renocia, Anaphase. Apply scalp massage oils like warm almond or coconut oil before a bath or overnight. Take a hair supplement capsule daily such as Keraglo-Eva.Keep in mind that the cause of your hair fall must be corrected and also will take 2 to 3 months to see the results. Visit your dermatologist to check your blood for hemoglobin, iron, thyroid levels and a general check of your health. They can also rule out other possible causes after an examination.Hope I have answered your query. Let me know if I can assist you further.Regards,Dr. Dinah Levillard"
},
{
"id": 5753,
"tgt": "Have bilateral poly ovaries, irregular periods. No problems in the tubes, on hcg injection. Chances of conception?",
"src": "Patient: Hi.. my age is 31, My last period was 5th January 2013... I have Bilateral Poly ovaries. I have irregular periods. I am going for the dr. last 6 months, 1st follicular study says MSF, 2nd says 19mm & reptured on 19th day- ET - 6.5mm, 3rd scan says MSF, 4th month i went for sline sonogragphy, that time dr. said there is no problem in the tubes , everything good, 5th again i went for follicular study 17th day 18mm & ET 8.3, So doctor put hcg injection 0000iu and ask me to intercourse on next three days... & asked me to take maintain tablet, apcot powder & fol 123. is there is a chances for concieve. Doctor: Hi, You did not say if you are on any drug for induction of ovulation. According to your follicular studies, chances of conception are good as the thickness of the endometrium is also optimal and tubes are patent. You may have to watch at least 6 cycles for a positive result while on treatment. Continue medication as suggested by your doctor. Good luck."
},
{
"id": 26260,
"tgt": "What causes feeling uneasy and frequent hiccup?",
"src": "Patient: i feel uneasy and frequent hiccup (I don't know the what to say to that, it is not normal hiccup we call it Dacar , like fart it comes up from mouth) which gives me pain in heart . and also feel uneasy in my back neck . can you suggest what this symptom mean ? Doctor: Hi, It would have been great if you would have mentioned your age. Considering your age between 30-50, with such symptoms it is required to rule out acidity and angina (heartache). I would advise you to get an ECG, Treadmill test and Echocardiography done. If angina is ruled out with these tests, your doctor may prescribe you antacids for acidity.Also, Avoid high fat foods, take frequent walks and avoid lying down just after your meals. Let me know in case of your further queries and/or reports.Dr Vishesh RohatgiMBBS, MD, PGDCC"
},
{
"id": 68060,
"tgt": "What does a small lump in buttocks indicate?",
"src": "Patient: What does a small lump in buttocks indicate? I have noticed a hard small egg-sized lump in my buttock toward the area where the two cheeks come ... a sharp pain when i walk and sit. What is this?Additional information: Lump is not visible on outside ... Doctor: Hi. Since when is the pain and swelling? Any fever? It could be a pilonidal abscess or a sinus if there is any diacharge from it. I suggest you see a Doctor soonest who would probably advice you a course of antibiotics and anti inflammatory medicines. Let us know if it persists or you have any more queries. Take care. Dr Rishi, New Delhi, India."
},
{
"id": 25520,
"tgt": "How harmful is to intake alcohol and cocaine under propranolol medication?",
"src": "Patient: My husband is a binge drinker, and now addicted to cocaine. He has high blood pressure and is taking propranolol. He has been in 3 detox centers in the last 6 months. I know mixing these together are harmful but how harmful? I am just waiting for him to get to his low so he will accept treatment. Doctor: Thanks for your question on Health Care Magic. I can understand your concern. Propranolol is beta blocker. It will cause reduction in blood pressure and heart rate. Alcohol and cocaine, both are CNS depressants (they suppress brain and it's functions). So alcohol and cocaine will also cause hypotension and low pulse rate. So if patient is on propranolol and alcohol and cocaine, chances of hypotension, bradycardia (low pulse rate) and cardiac arrest is more. So better not to take alcohol or cocaine if you are taking propranolol. So counsel your husband to quit alcohol and cocaine because he is taking propranolol. Hope I have solved your query. I will be happy to help you further. Wishing good health to your husband. Thanks."
},
{
"id": 225146,
"tgt": "Am i at high risk of pregnancy after missing lutera tablets?",
"src": "Patient: Hi I ve been on lutera and I missed taking my pill once or twice but I made it up and I haven t been very consistent with the time but mostly around the same time but not the same time. And I had unprotected sex and was wondering if I have a high risk of getting pregnant Doctor: Hi,Thanks for the query. Lutera is a combined oral contraceptive pill. If you took the tablets as per the guidelines, the possibility of pregnancy is less. For more details : http://srsree.blogspot.com/2013/11/general-guidelines-of-using-combined.html If you took the missed pill within 24 hours, possibly complete protection can be expected. If you missed one pill completely and didn't use any additional method of contraception, then there is possibility of pregnancy. So, you wait for your withdrawal bleeding. If you do not get withdrawal bleeding even one week after completion of active pills, once go for urine pregnancy test. If the test comes negative, you can continue with your pills. Take care."
},
{
"id": 143365,
"tgt": "What precautions should i take in es1 lumbar?",
"src": "Patient: Hi, I am having an Es1 lumbar and am trying to find out if this is something I should be careful with. I have a tear and I work around a lot of horses. Should I be taking precautions? how long should I stay away from the horses? What exactly is the procedure? I keep getting the It is a simple procedure Then I see videos of needles going into a spine. I cannot find the procedure listed above. Doctor: Hello!Welcome on HCM!Regarding your concern, I would explain that this procedure consists on the injection of a steroid shot in the epidural space. This procedure may cause a temporary relief of back pain, resulting from bulging discs. The procedure itself is quite safe and there is no need to hospitalize. But, the limitation of physical activity is more related to the main problem which is the discal herniation or bulging, than to the injection procedure. It is important avoiding prolonged standing up and sitting position in the next days. Heavy weight lifting should be avoided too. Physiotherapy may be helpful to reduce the back muscles spasm. All there precautions are necessary to avoid exacerbation of the back pain. Hope you will find this answer helpful!Kind regards, Dr. Aida"
},
{
"id": 104404,
"tgt": "Red dots all over body. Itchy. what this is?",
"src": "Patient: Hi, a few days ago I started to observe small red dots on my torso and breast . They only itch once in awhile and they are spread out all over. I haven t switched any of my lotions, body washes, or other, and do not have any allergies that I am aware of so cannot pin point where this is coming from. Most of the spots are a little larger than a pin point but a couple have gotten larger and look dry in the middle. Any ideas as to what this is? Oh, I also have a, what I think is a bite, that is on my upper, inner, thigh that has been there for two weeks now and it itches quite a bit off and on. Doctor: Seem to be fungal infection which occurs in the areas of sweat keep areas dry and airy no oils take forcan 150 mg 0nce a week apply antifungal cream amnd powder for 3 months and if itching can use loratidine tablet once a day"
},
{
"id": 198865,
"tgt": "What could cause strong and elastic cum during ejaculation?",
"src": "Patient: My cum when i ejaculate in congealed and sometimes very strong and elasticy, it also has runny normal cum mixed with it, it appears if i cum at first chance rather than masturbation for a longer period of time then it is more likely to be congealed. I am 14 and worried, thank you. Doctor: DearWe understand your concernsI went through your details. There is nothing to be worried about. Being just 14 years young, you should concentrate more on your education. Thickness of your semen depends on many factors like stress, de-hydration, physical fitness, absence of other medical condition, tiredness etc. It has been seen generally that the delay in ejaculation process makes it more diluted because of the liquids added from prostrate gland. Semen with more liquid base will be able to travel easily through women vagina. Hope your worries are answered. If you require more of my help in this aspect, please use this URL. http://goo.gl/aYW2pR. Make sure that you include every minute details possible. Hope this answers your query. Available for further clarifications.Good luck."
},
{
"id": 185336,
"tgt": "What could be a recurring bump on gum?",
"src": "Patient: This is more of a dental issue, but I figured you could still help. I have had a bump on my gum for about 2 months now. It is on the uper left side of my gum above a tooth i had a root cannal on. When it becomes irritation I would push on it and it would \"pop\" but then just come back again. What could this be? It isnt white or anything and its a little smaller than a pinky nail. I need help! Doctor: Hello, thank you for consulting with healthcaremagic. This bump on the gums shows that there is some infection still left below the tooth, this bump is because of an abscess which is formed due to the infection. Better you should visit your dentist again and get the examination done of the root canal treated tooth.Hope it will help you."
},
{
"id": 125401,
"tgt": "Suggest treatment for painful arm after an injury",
"src": "Patient: I fell nearly 3 weeks ago and my arm hit the top of a chest of drawers, the arm was very badly bruised and took 10 days for the bruising to go away. My arm is still painful when i do certain jobs with it, I m that I may have chipped a bone or something, what do you think. Amanda Doctor: Hi, Bone chipping is very unlikely in your case since you have been working with this arm for nearly 3 weeks. It is likely to be a muscle contusion and it may take longer to heal. Hope I have answered your query. Let me know if I can assist you further. Regards, Dr. Gopal Goel Orthopaedic Surgeon"
},
{
"id": 101232,
"tgt": "Suggest remedy for chronic nasal blockage and feeling of imbalance",
"src": "Patient: Hello, I am suffering with chronic nasal blockage from many years & also severe imbalance feeling all the time. Tried many medicines but they work only temporarily. Could you please suggest what can bring my life back to normal. Please respondkalpanaemail- YYYY@YYYY Doctor: HIThank for asking to HCMI really appreciate your concern and if this is the nasal congestion then could be relieved by drinking more water and have a steam inhalation if the symptom does not improved then saline nasal drops can be used, hope this information helps you, take care and have a nice day."
},
{
"id": 195406,
"tgt": "What causes black discoloration around the urethra?",
"src": "Patient: Around the pee hole is dark or black and seem to be spreading I have no burning or irritation but a whole lot of pain in the upper left side under the breast and it travels around to the back area under the shoulder bone and straight down the side then to the lower part of the stomach Doctor: Hello and Welcome to \u2018Ask A Doctor\u2019 service. I have reviewed your query and here is my advice. Discoloration around urethra is not a thing to worry about if it is not causing any symptom. It is normal to have slight discoloration around the genitalia and urethra in some males.It is not a symptom of any sexually transmitted disease.However if it becomes associated with any symptom like itching,burning,discharge from penis or pain in penis,then it needs to be evaluated. I think I answered to your question if you have more questions feel free to ask. Take care."
},
{
"id": 85929,
"tgt": "What can cause severe stomach ache and nausea?",
"src": "Patient: Each month, for about a year or more now, I experience severe stomach pain that induces nausea, and it is starting to concern me. It feels like a blunt kind of pain, and at times I think it\u2019s gas or a need to use the bathroom for number two, not to be gross. I struggle with constipation and it often resolves itself in the form of painful diarrhea. These past two months, I noticed that this pain came the same day I tried new foods that did not particularly sit well with me. I think that may have something to do about it, but what about the other months? I just noticed today that I\u2019m mid-cycle and ovaluating, which made me wonder if PMS has something to do with this? I never get these pains when I\u2019m having my period or on the days surrounding it. I would appreciate input, as this is starting to overwhelm me. Doctor: Hi, The most likely issue of altered gut flora mechanics with hormone imbalance. Upper GI endoscopy and basic lab tests including thyroid hormone levels are mandatory here to get checked completely. Hope I have answered your query. Let me know if I can assist you further. Regards, Dr. Bhagyesh V. Patel, General Surgeon"
},
{
"id": 23309,
"tgt": "When will the Coversyl benefits be visible?",
"src": "Patient: My 77 year old mother Height 5 ft, weight 150 lbs) has been on Coversyl 4mg/day for four weeks. Her doctor increased the medication to 8 mg/day, four days ago. Togther with Coversyl, she also takes half a tablet of Metoprolol. Latest blood tests show that cholesterol is slightly high, she has protein in urine and blood in urine. Her blood pressure is presently 185/87 and sometimes 187/90 with heartbeat of 67 or 57. When should we see the beneficial effects of the Coversyl 8 mg ? Doctor: Hello,Effects of coversyl may be observed from the first day the patient starts to take, but we see the maximum effect in two weeks. So if there is still high blood pressure in two weeks we add another medication or change it.Take careCome back if you have any further questions"
},
{
"id": 216942,
"tgt": "Suggest treatment for ankle pain",
"src": "Patient: Last night i was at cheer practice and while I was tumbling, i landed on my ankle/leg weirdly. It hurt a little whenever i landed, but the pain was not too bad. I kept practicing and my whole leg would hurt when i would do certain things, but it was not too bad. By the end of practice, i could not walk on it at all without tons of pain. I iced it this morning, and last night and also took advil this morning and last night. Today my ankle, knee, and thigh area are swollen. I can put some weight on my leg without too much pain, but I can not walk normally without TONS of pain. I have had problems in the past with patella tracking, but it has not bothered me recently. Doctor: Hi there thanks for your question. If you have any pain when you stand or walk on your injured ankle, it suggests severity of injury and warrants further work up whcih wil include a X ray at the least. Ice and rest usually give relief in pain and swelling over the first two days. But you are not expected to have pain on standing or walking in mild injuries.Such pain should suggest you to take your injury seriously to avoid long term ligament related pains. Hope this helps All the best. Regards.Dr.SBK"
},
{
"id": 72637,
"tgt": "What causes shoulder pain radiating towards the arms, cough and congestion?",
"src": "Patient: i went to see doctor because i got a bad cough with phlem, runny noise and sinus pain. i had gone grocery shoppoing and got bad cough attach and felt like i pulled something. told my doctor and she sent for xrays which came bck normal, but when i cough due to phlem and i need to expell i cough quite hard and get the same feeling i got at grocery store. pain inb my shoulder that runs down my arms and in both my breasts. it usually takes about an hour forn pain to subside. what could be going on here? Doctor: Thanks for your question on Healthcare Magic.I can understand your concern. First of all, no need to worry for lung diseases as your chest x ray is normal. Coughing can cause strain on ribs, intercostal muscles and shoulder joint. So possibility of musculoskeletal pain is more likely for your symptoms. So apply warm water pad on affected areas of shoulder. Take simple painkiller like paracetamol or ibuprofen. Avoid movements causing pain. Don't worry, you will be alright with all these in 2-3 days.Hope I have solved your query. I will be happy to help you further. Wish you good health. Thanks."
},
{
"id": 120938,
"tgt": "Can foot surgery cause pus and pain?",
"src": "Patient: Just recently, I had surgery on my foot back in june. the wound on my foot has healed up nicely. For a couple of weeks now my big toe has been swollen and paining. this past week it has been pussing. I cant seem to figure out the reason why this is happening and cant find a cure. Do you have any suggestions? Doctor: Hello,I read carefully your query and understand your concern. The symptoms seem to be related to a toe infection.I suggest to use Hydrogen Peroxide to clean the toe. I also suggest using an antibiotic cream for local application such as Neosporin cream.I suggest to cover it with a sterile bandage.If the symptoms does not improve you should see a surgeon.Hope my answer was helpful.If you have further queries feel free to contact me again.Kind regards! Dr.Dorina Gurabardhi General &Family Physician"
},
{
"id": 217897,
"tgt": "What cause perinium and anus discomfort specially when sitting down?",
"src": "Patient: Hi. Male, 33yrs old. Having perinium and anus discomfort specially when sitting down. Ultrasound of prostate,Kidney and bladder negative. Culture tests on urine and blood even prostatic fluid negative. My urologist termed it non bacterial prostitis. Is he accurate on this? Doctor: He may be. Because there are a lot of cases of bacterial prostatitis that generates negative results on investigation. You may also be having CPPS, where there is no specific pathology."
},
{
"id": 96253,
"tgt": "Have blood and pain while passing motion ?",
"src": "Patient: sir i have bleeding while passing motion and there is pain while passing motion Doctor: Hi, You should be examined for piles.Use a laxative or stool softener at night.Also increase fibre in the diet.You can use lignocaine jelly before passing motions as local application. But ultimately if piles is confirmed on examination then you may need surgery."
},
{
"id": 175766,
"tgt": "What is the cause and treatment for 6 year old with rash and crusty ears?",
"src": "Patient: My 6 year old daughter has a rash behind her ear. Her ear gets crusty at the end and she has a cut at the bottom of her ear where her ear connects to her head. That seems to leak out a clear fluid. Now the rash has spread to the back of her neck. What could it be?She also says it itches. Doctor: Hi Dear Welcome to the HCM,Fungal infection. tenia capitis and otomysois may be the cause.Wash the hairs with ketoconazole shampoo.Antifungal drops to be poured in the ears.Apply some neosporin ointment in the ulcerated area.Hope the query is answered.thanks"
},
{
"id": 168161,
"tgt": "Suggest treatment for vomiting along with cough and cold",
"src": "Patient: Hi Doctor, my five and a half year daughter has been vomiting from today morning. She is suffering from mild cough & cold from last 2 days. In this situation should I give him VOMIKIND Syrup on S.O.S. basis? If yes, what will be the doses considering that she weighs 18.5 kg? Doctor: Hello and Welcome to \u2018Ask A Doctor\u2019 service.I have reviewed your query and here is my advice.Yes, you may give vomikind. Vomikind contains ondansetron which decreases the vomiting. Each 5 mL of vomikind contains 2 mg of ondensetron. For 18.5 kg you may give 7.5 mL of vomikind (0.15 mg/kg/dose).Hope I have answered your query. Let me know if I can assist you further.Regards,Dr. Pradeep Kumar"
},
{
"id": 135577,
"tgt": "What causes itching on wrist after taking steroid injection?",
"src": "Patient: I was diagnosed with Tenosynovitis in my wrist after suffering with pain for 3 months .My Gp gave me a steroid injection which I have had in my other joints before.I have since developed intence itching around the wrist and it s quite red .What is causing this? Doctor: You might be having an infection . How many days since you had the injection. Best thing to do do is not to scratch it too much, and show it to gp as soon as possible."
},
{
"id": 151402,
"tgt": "Lumbar spondylosis test done. What does the report indicate?",
"src": "Patient: Hi i need to ask for my father Limbar spondylosis with diffuse disc desiccation small posterocentral bulgw with focal annual tear at L1-L2 and l2-l3 level withoutformal narrowing diffuse disc bulge with right lateral propensity at L4-L5 which is association with flaval hypertrophy causing mild central canal and moderte bilateral foraminial narrowing resulting in compression of cauda euina right traversing nerve root Difuse disc bulge with left lateral propensity an left paracentral annaular tear at L5-sl level Doctor: Hello Rama Prasad and welcome to HCM Your father is suffering from PIVD (Prolapsed Inter vertebral Disc).The nature of treatment is medication with physiotherapy and may need surgery but decision for surgery to be confirmed by clinical examination. Any nerve root compression needs surgery depending upon the symptoms present. Sometime they get relieved by some precautions and medication. Precautions to be followed are: 1. Avoid jerky movements of spine. 2. Wear LS belt while traveling or driving. 3. Take complete bed rest in worse conditions. 4. Do regular spine exercise as advised by your physiotherapy. 5. Use hard bed to sleep. 6. Take some analgesic and apply diclo gel locally in pain. 7. Don't lift weight. Hope this will help you. Best wishes"
},
{
"id": 3909,
"tgt": "Are there pregnancy chances after oral sex and fingering?",
"src": "Patient: hi, two days ago my partner put two fingers into my vagina .. we were doing oral sex but i didn't touvh his penis and neither did he so no semen was present but I'm afraid that i might get pregnant .. Must i be worried ? Please help and thanks for your service Doctor: Hi, thanks for writing..Fingering and oral sex doesnot cause pregnancy. So you need not worry about pregnancy. Only vaginal sex with sperm entering the uterine cavity during fertile period can cause pregnancy.. hope I have answered your query.. Good day.."
},
{
"id": 90596,
"tgt": "What causes lower quadrant stomach ache?",
"src": "Patient: Hi, i am anjali Mahindru. my age is 27. I got married 10 months before. after my marriage i am frequently suffering from stomachache in lower part. I got all my test i.e, ultrasound , urine & culture test but every time pus cells comes 20-25 and sterile after 48 hrs incubation at 37c. Doctor: Hi.Thanks for your query..This must be due to recurrent UTI- urinary tract infections.This is common after marriage and usually starts as honeymoon- pyelitis, meaning occurring after marriage.Since the length of urethra in females is very short, they can get infections frequently and easily.If the pus cells are present but the culture is negative, one can think about tuberculosis as the commonest cause. .Both of you- meaning your husband also has to take medicines. Go for IVP and further investigations to rule out any other problem like obstruction in the urine out-flow. Visit an Urologist for a second opinion."
},
{
"id": 169491,
"tgt": "Suggest remedy for high fever with runny nose",
"src": "Patient: my 14 month old started running a fever today it hit 102 but now is 101.2. he has been having a runny nose for the past couple of days and i figure it is his sinus because mine is messed up to. what should i do check his temp and if it persists with his runny nose call his doctor or what Doctor: Thanks for your query, I have gone through your query.The fever can be secondary to the sinus infection. Nothing to be panic, consult an ENT surgeon and get it evaluated. Mean while you can take a course of antibiotics like amoxicillin(if you are not allergic), antihistamine like cetrizine and antipyretic like paracetamol for the fever. Take steam inhalation.I hope I have answered your query, take care."
},
{
"id": 141062,
"tgt": "What causes dizziness with a headache, fatigue and itchiness in the right eye?",
"src": "Patient: When I get up I feel light headed and dizzy with a headache, no energy at all hot and cold flashes. Get sick after I eat about 1/4 of the time. More forgetful than normal. Right eye itches and hurt, won\u2019t stop watering. Blood shot, feels like there is some thing in it, but there and this has been going on for over a month. What can I do to return to normal. Doctor: Hi, You have a collection of symptoms which may be related but could be from different underlying causes as well such as GI disturbances vs. allergies vs. mild cognitive impairment. None of these by themselves may be serious but taken together are certainly bothersome. You should consult a physician for a complete physical examination which could include blood work, urine, and history in order to get things on track. Hope I have answered your query. Let me know if I can assist you further. Regards, Dr. Dariush Saghafi, Neurologist"
},
{
"id": 32445,
"tgt": "What causes pain and swelling in genital area?",
"src": "Patient: i'm a male of 22yrs, i have pain in the genitals to the right side along with swelling after ejaculation and after getting a erection, the feeling is kinda like \"blue balls\" but last for about an hour. what can be the cause of this? And what can be doing to ease the pain Doctor: Welcome at HCM I have gone through your query and being your physician i completely understand your health concerns. For how long u are suffering from it? Any other associated problem? like diabetes or hypertension? Any history of TB in past or exposure to TB? Are u running fever? Any other lymph node swelling in ur body? do u have any dragging sensation in testis? any dilated veins u feel in testis? any history of trauma to genital tract? Based upon your history it is difficult to interpret. you need a proper consultation which requires examination If i am ur attending physician i will get your SCROTAL ULTRASOUND done to look for any pathology like hydrocele or varicocele. i will rule out other causes too.meet your doctor so that you can be examined and investigated fully. Meanwhile stay calm and use scrotal support Get well soon Hope your query is adequately addressed if you still have any feel free to ask RegardsDr Saad Sultan"
},
{
"id": 79207,
"tgt": "What causes tired eyes, headache and clogged sinuses with chest pain?",
"src": "Patient: my eyes are feeling tired they hurt, my head hurts my sinuses clog in the morning i have a cough that hurts, I m spaced out a lot i have to relie on pills to get me through the day i don t know whats wrong i also get sharp pains in my head and if i move my head to fast it hurts and my items appear somewhere but they end up its liked fractured from one place to another i play volleyball and my aim was very off im the number one player but it was very off it was fractured too it aim towards the wall which is not usually i always get my serves in every time i walk upstairs i get tired quick and breathe very hard i don t know if i have a fever everyone feels my forehead but it s always hot i feel like i want to sleep all day which i do because i have got very lazy chest pains come sometimes but they aren t bad as my headache thats been lasting a couple days now help me! Doctor: thanks for asking your question symptoms seems like you are suffering from - upper resp tract infection- sinusitisyou need to consult a Ent specialist or a Pulmonologist who can request for a X-ray of he sinuses or if needed a mri to see the deformity or infection o the sinuses , this will lead to a certain diagnosis and accordingly treatment can be planned in my patients I usually start them on decongestants and leukotreine inhibitors and some steroid nasal sprays steam inhalation with vapocaps help a lotthanksfeel free to ask more questions"
},
{
"id": 13313,
"tgt": "What causes red rash and burning sensation on the insteps of feet?",
"src": "Patient: I have a slight red rash on the instep of both feet that cause some tingling and numbness. I am not a diabetic and don t believe to have circulation problems. no itching associated with the rash but I do have a slight burning sensation on my in-steps. Do you have an idea what 5this could be ? Doctor: Hi, It may be vasculitis. There may be inflammation in the vessel wall of feet. Consult the dermatologist for the perfect diagnosis and proper treatment. Antimetabolite like azathioprine might improve the condition. Keep the feet elevated. Warm water soak may benefit. Hope I have answered your query. Let me know if I can assist you further. Regards, Dr. Ilyas Patel, Dermatologist"
},
{
"id": 44521,
"tgt": "Rapid linear progressive motility-0%,normal motility value,diet to increase count",
"src": "Patient: Hi My husband semen analysis is as follows. Total Sperm count = 22 million per 2ml, Rapid lenear progressive motility - 0%, sluggish linear progressive motility - 30%, non progressive - 20% and non-motile - 50%. Is this analysis ok or is there any problem ? Is the pregnancy possible with this result ? what food should be taken to increase Rapid lenear progressive motility ? what should be the normal % of Rapid lenear progressive motility ? Doctor: Dear Neelam,As ur Husbands semen analysis reports shows he is having OLIGOSPERMIAhe needs to take medicine for 60days.if is any bad habits like smoking, Alchol consumption he should stop them.1.CAP.ADDYZOA 2cap BID, 2.Tab,NEO 2cap BID3.countplus granules powder 5gm with milk twice a day,after taking this medicine for atleast 2 months surely he will improves his total sperm count , goodmotility, u can go for semen analysis after 2months u will see postive reults all the best .Dr.VIJAYKUMAR.G"
},
{
"id": 60846,
"tgt": "What causes a painful lump on the leg?",
"src": "Patient: hi I get this pain and I feel a lump it hurts I start puching it in when I do extercise and begins to hurt down my legs till I get up and rubing the aira what could it be.?in the lower addomenal I also have this lump on top of my abdominal every time I bend I rub it in so the pain will go away. Doctor: Hello and Welcome to \u2018Ask A Doctor\u2019 service. I have reviewed your query and here is my advice. * The painful lump on the leg indicates possible soft tissue lump as sebaceous cyst, lipoma or others. * I recommend to consult with a photo picture of the lump in the follow up interaction for best possible guidelines. Wish you fine recovery. Regards, Dr. Bhagyesh V. Patel, General Surgeon"
},
{
"id": 140477,
"tgt": "Suggest treatment for ankylosing spondylitis and numbness in leg",
"src": "Patient: Hi, I ve been victimized by ankylosing spondylitis for thirty-five years. Sixteen years ago this upcoming August a resection osteotomy and fusion in the t-11, t-12 (andersen lesion) was performed on me by a prominent spinal specialist at Buffalo General hospital. Recently I was diagnosed with drop foot by my rheumatologist, after complaining of progressive numbness in my right leg (I ve also had hip replacements). My rheumatologist believes the numbness generates from my spine (I ve been dealing with a dull, constant ache on my lower right side for some time). My problem: the surgeon in Buffalo no longer accepts medicare patients, and I can t afford to pay out of pocket. Therefore, I need to find a new surgeon in the upstate New York region (not far from the Elmira/Corning area where I live). Equally frustrating is my inability to get a good radiology reading of what s going on since the hardware in my spine (rods and pedicle screws from s-1 to t-6) generates a lot of scattering. Not sure why but every time I go to my local hospital for a ct-scan I m told it is too hard to determine a reading of the area in question due to the scattering. I attribute to lousy, dated equipment, since I usually get a good read on x-rays in Buffalo. I wrote my rheumatologist last week, but, as in the past, he s overwhelmed and doesn t get back to me. If you have any suggestions I would appreciate your advice. Thank you, Doctor: Hello, You can easily get an X-ray or an MRI scan and both modalities can help out easily as to what is going on. Hope I have answered your query. Let me know if I can assist you further. Take care Regards, Dr Uday Singh Raswan, Neurosurgeon"
},
{
"id": 98562,
"tgt": "What causes swelling and tingling sensation in the lip after eating food?",
"src": "Patient: Hi I am a female 40 years old experiencing what I thought was an allergic reaction then thought maybe it was herpes now not sure.....On Tuesday just after eating lunch my top lip began to tingle and swell. No apparent bumps or cold sores were visible. Since then I started taking Valcyclovair as I have had cold sores in the past. My top lip is swollen double the size of my bottom and the skin on my top lip all the way to my nose is oosing a clear fluid. Skin is irritated and raw not sure what it could be any suggestions? Doctor: It looks like an allergic reaction to whatever you are that day. visit the ER. You might want to have a shot of Hydrocortisone. The symptoms should come down. Pay attention to the ingredients of that dish and avoid it in future."
},
{
"id": 176202,
"tgt": "Suggest treatment for runny nose and yellow discharge from nose",
"src": "Patient: Good DayI have an 20 months old boy wts. 13.5 kg and hes having runny nose and dry cough for 3 days.Today, I noticed yellow secretions coming out of his nose.What medicine would ypu recommend?I have flumed syrup but im afraid to give.Hop eyou can help i feel worry about my babies now.Thanks Doctor: Children often suffer from common cold which is caused by certain viruses. Often there could be secondary infections leading to yellow nasal secretions. Flumed is a medicine, the composition of which varies with the country in which it is being marketed. I assume that in your country it contains an antiallergic (Dexchlorpheniramine) which should make it right to be given. Please check if that is the composition. Alternatively, you can give some other antiallergic. Other than that you need to give steam inhalation. Just sit with the child in your lap and place the source of steam before you. The water vapour would be in the air and the child will automatically breathe it in. It will help in diluting the cough. You may also give saline drop in case nasal blockage occurs. I do not think that antibiotics would be required unless it lasts more than 7 days."
},
{
"id": 191166,
"tgt": "I have swelling between upper canine and first pre molar on palatal side",
"src": "Patient: treatment of swelling present between upper canine and first pre molar on palatal side Doctor: In Ayurveda we recommend a paste made from turmeric salt and mustard oil is applied after bush at night. Internally use punarnava guggul 2bd"
},
{
"id": 5714,
"tgt": "Chances of getting pregnant without penetration but ejaculation around vagina? Irregular cycle, periods delayed",
"src": "Patient: Hi doctor... I have a question... What is the chance of getting pregnant without penetration but there was ejaculation around the vagina ... I am a virgin and this happened on the 27th day of my cycle... Normally i have irregular cycle between 23 and 28 days.. Now i am on my day 30 and still didn t have my period. I have lost a small weight earlier. Thank you Doctor: Hello As you are a virgin, without penetration, there is a next to nil possibility of pregnancy. As ejaculation occurred around the vagina, some seminal fluid might have technically entered the vaginal canal, but this is a very remote possibility. As your maximum cycle length is 28 days, and you are already on day 30, please take a urine pregnancy test early tomorrow morning ( first voided sample of urine ). That way you would know for sure. Take care."
},
{
"id": 48136,
"tgt": "How long will the surgery for kidney removal due to large mass take?",
"src": "Patient: Approximately how long will surgery take to remove a kidney which has a mass inside it almost as large as the kidney...they are to remove the entire kidney...all other scans, of other organs,bone scan all are clean...we are just curious if they are able to do it lapriscopically (sp),or if they have to cut him to remove it...time frame is what we are wondering....not sure if the Dr. told him or not...seems I was told 3 to 5 or more hours? does this sound correct? Doctor: Good Day and thank you for being with Healthcare Magic! For both laparoscopic and open procedures a radical nephrectomy would take around 3-6 hours depending on the complexity of the kidney and the mass.I hope I have succeeded in providing the information you were looking for. Please feel free to write back to me for any further clarifications at: http://www.HealthcareMagic.com/doctors/dr-manuel-c-see-iv/66014 I would gladly help you. Best wishes"
},
{
"id": 141713,
"tgt": "How to get rid of hard bump on head?",
"src": "Patient: i hit my head 2 months ago slightly in my forehead, I had a bump, and it was swollen, it has since gotten smaller, but its now 2 months later, and I still have a hard bump on my head, but just smaller, is this normal, or could something more serious be happening? Doctor: It probably is normal, and calcification of the blood under the scalp.This may get better over the next few months, but it could harden up and become calcified as well.This does not sound like a serious issue."
},
{
"id": 83678,
"tgt": "Can timolol cause shortness of breath and tiredness?",
"src": "Patient: I have just started to take timolol for eye pressure. It has been about 4 weeks. On my runs, I have noticed some shortness of breath and feeling there is something with my leg muscles (unusual tiredness). Can the use of timolol be causing these systems? Doctor: Hi, Suggestive of timolol-induced side effects. Timolol is a beta-adrenergic blocker commonly prescribed in the form of eye drops to reduce intraocular pressure in patients suffering from glaucoma. Although it is used locally in the eyes, some amount of the drug gets absorbed into systemic circulation (blood) producing unwanted side effects as you seem to have developed following its use. Ask your doctor for an alternate effective but a safer medication for the treatment of glaucoma. Hope I have answered your query. Let me know if I can assist you further. Take care Regards, Dr. Mohammad Taher Ali"
},
{
"id": 27036,
"tgt": "Suggest treatment for Dementia",
"src": "Patient: My mother whom is 80 yrs old has been diagnosed with dementia, in the ER just yesterday she could not talk they did a cat scan of her brain showed me all the white spots and stated she has small vessel disease and he used another work that I believe started with a M (too technical for me to remember) it meant the same thing so he said. she was admitted he stated she may of had a tia, the attending physican today cancelled the mri and started with the heart as she heard something they did a echo cardiagram and the results arnt back yet. Could the heart have something to do with the dementia, she is very confused very fast almost within the last two weeks she has been really confused.What is the long term prognosis is it possible for her to be so confused so fast? Doctor: Hello!Thank you for your question on HCM!I understand your concern, and would like to explain that acute episodes of confusion are normal in persons affected from dementia. The cardiac tests, have resulted normal, and we can exclude this as a possible cause of this confusional state. Regarding the small white spots in her brain, they indicate for a small vessel disease causing a vascular dementia. They are not related to any cardiac cause, but to high blood pressure or diabetes. Dementia is a progressive disorder, which is treated with different neurotropic drugs (donepezil, rivastigmine, memantine) and to help maintain as much as possible an adequate cognitive function and a lot of emotional support is needed. Unfortunately the situation deteriorates progressively. Antipsychotic drugs may be necessary in case of frequent confusion episodes. Hope to have been helpful!Greetings! Dr. Iliri"
},
{
"id": 187641,
"tgt": "Why do I grind my teeth?",
"src": "Patient: hello. i have small holes which also have cavities in them located on most of my bottom teeth right on the gum line. I noticed a few months ago that I grind my teeth. Is this the cause? I've gotten fillings before but they just fall out in a matter of weeks. I do no have insurance to cover my bills for the dentist until recently I found out I am pregnant so I will eventually qualify for medicate after they approve me. But this can take a long time. What can I do in the mean time? Should I try to work something out with my dentist? What do you suggest? Doctor: Hi,Thanks for posting the query, Grinding of teeth is known as bruxism in which a persons intentionally or unintentionally grind his teeth especially at night. It can be a result of bad bite or can occur due to psychological factors like stress. I would suggest you to get a checkup done.If it is due to bad bite it can be corrected.It can be treated with a appliance called as night guards.Take care!"
},
{
"id": 42996,
"tgt": "Is there any chances for me to conceive?",
"src": "Patient: i have had abortion 7 years ago and i have been trying to conceive for almost a year now and nothing good happened and i went to a doctor and he said i have low iron and we also did a pap smear and he said i have abnormal cells, is there any chances that i will conceive? Doctor: Hi,In the current scenario, you also need to test for tubal patency since you have had an abortion in the past; correct your iron deficiency through oral or parenteral supplements based on the amount of defect and get properly treated for the abnormal smear. The chances of conception are good if the infection can be cured and nutritional deficiencies corrected and even with doubtful patency of the tubes, IVF can be tried. Hope you find this information useful. Wish you good health."
},
{
"id": 206951,
"tgt": "Suggest treatment for stress and depression",
"src": "Patient: Hi my name is Carlos I'm actually trying to find a doctor for my sister. She's about to turn 54 years old and for the longest she goes through extreme mood swings. She can be having a great time at a party and right after get angry for no reason. She lives in Brooklyn in 11229 zip code area has medicare and i think Infinity insurance. If you can recommend a doctor for her ill even take her. Thanks Doctor: Be hopeful !Stress and depression are quite normal for a living being. However, if it intereferes the individual and society, we can modify our lives to a healthy living. Unfotunately, you did not mention if she is widow or with family. The clues are suggesting that she needs someone to talk it out, as she is comfortable in a pary settings. The proper medication from a Psychiatrist and some therapeutic intervention from the qualified mental health professional will be of help at this juncture. Talk to her whenever possible. Do not leave her to fend for herself. The family support will work wonders along with the mental health intervention. God bless all,Dr Rajesh Thottingal Kalam,Rehabilitation Psychologist from india"
},
{
"id": 58393,
"tgt": "Ultrasound showed anechoic cyst in liver, cortical cyst in kidney, enlarged prostates. Further treatment?",
"src": "Patient: Ask Dr. Nitin Rao a questionDr. Nitin Rao, Gastroenterologist , SurgicalI shall answer queries once a day Certified expert What do you get:Three FREE follow-up questions A full refund in case you are not satisfied with my answers Please write your health query in detail below my father is 85 years old and in the recent ultasonograph , the report mentions:- 1) a small well defined anechoic cyst of 2x1.3 cm size in the left lob of liver 2)kydney--cortical cyst are seen bilaterally, the largest on the right measures 3.4x2.9 cm and on the left 4.5x3.7 cm 3) prostrate --size enlarged(5.4x5x5.3cm-77gm) echotexture- heterogeneous kindly let me know whether it is cancerous and what should be the line of treatment Age: Years Gender: -- Select Gender -- Male Female no past ailment Medical History: none Current Medications: none Known Allergies: none Doctor: Hi and welcome to HCM. Thanks for the query. These are common cysts and in 97% cases benign and you dont need any treatment unless there is rapid growth or maliganncy potential.At this point you need only frequent follow up to check size growth both of liver and kidney cysts. Enlraged prostate should be treated by urologist and certain medicines may be necessary. Wish you good health. Regards"
},
{
"id": 71529,
"tgt": "Can vitamin-C be given to boost the immunity against respiratory infections?",
"src": "Patient: I ve seen the effects of high doses of Vitamin C can have on the bodys immune system particulary for respiratory problems, so if we were to do this for my elderly father who has just been diagnosed with Bullous Pemphigoid, would that help his body fight infections in general ( ie against colds, flu or even pneumonia,) even though hes on prednisone? We realise it won t help his diagnosis but just to keep him as healthy as possible while his immune system is suppressed. What else could you recommend we could do to help give him some quality of life? Natual remedies or not. At present his condition is controlled with only antibiotic ointment and 20mg of prednisone. Is this condition likely get worse? He is a diabetic.Thank you for your time. Doctor: Hello,As per the narration, he has diabetes and so long terms use of steroids will not be advisable to him to save his kidneys from further damage.Vitamin C will be supportive to give along with balanced nutritious diet, deep breathing exercises.Hope I have answered your query. Let me know if I can assist you further.Regards,Dr. Bhagyesh V. Patel"
},
{
"id": 78194,
"tgt": "What causes fluttering feeling in chest during driving?",
"src": "Patient: I am a 40 year old healthy female. This morning while driving I experienced a fluttering feeling in my chest accompanied by dizziness. The episode lasted about 4 seconds at most. Once over I felt a little weird but then returned to my normal self. I have on occassion experience this fluttering feeling in my chest before but never like this. I don't feel that this could be Aflutter, my heart rate did not seem fast and I have no previous medical history. What are you thoughts? Doctor: Hi. I can understand your concern. Ideally ecg and 2d echo should be done first in your case. If both are normal than no need to worry for heart diseases. GERD (gastroesophageal reflux disease) can cause similar kind of chest pain. GERD is due to laxity of gastroesophageal sphincter. Because of this the acid of the stomach tends to come up in the esophagus and cause central chest pain and nausea. You are taking proton pump inhibitors (pepcid). But along with drugs you need to follow certain below mentioned lifestyle modifications for better symptomatic relief. Avoid stress and tension. Avoid hot and spicy food. Avoid junk food. Avoid large meals, instead take frequent small meals. Quit smoking and alcohol if you have these habits. Go for walk after meals. Keep 2-3 pillows under head in the bed to prevent reflux. Loose weight if you are obeseDon't worry, you will be alright. Hope I have solved your query. Wish you good health. Thanks."
},
{
"id": 111219,
"tgt": "What causes stomach ache and severe back ache?",
"src": "Patient: Hi I'm 19 I have a 4month old baby and I've been suffering with random leg bruising and cramps also I have been suffering with stomach ache and really bad back ache in my spine I had bloods taken but they came back clear any idea what's wrong with me? Doctor: Hello,I had gone through the case and found that it might be renal stone or any other problem.So go for ultrasound of whole abdomen and diagnose the case.Meanwhile take mild painkiller and light food.Hope my answer will be effective for you.Thanks"
},
{
"id": 168811,
"tgt": "Suggest methods to increase appetite in infants",
"src": "Patient: my baby is 8 months and weight is 8.6 kg. he has started refusing food from last 1 week. i used to give him vareity daily - mashed fruits like banana, apple, rice khichdi, dal, mashed vegetables, curd and cheese. Please suggest how can i increase his interest and how much he should eat. Doctor: Do not force feed the baby and he has no weight loss and try only demand feeds than compelling him to feed; babies are always moody when it comes to their feeds and saying no is a pleasure to them at this age. As long as he is healthy with adequate weight gain, don't force yourself to force feed him."
},
{
"id": 94716,
"tgt": "Abdominal pain, tender to the touch, back pain, unable to take deep breath",
"src": "Patient: I am experiencing upper abdominal pain tender to the touch feels like someone hit me in the gut. Hurts thru to my back. I am unable to take deep breaths it hurts so much. I am uninterested in food and cannot eat much feel full after eating small amounts. I am a 53 year old woman and I have already had my hall bladder removed Doctor: Hi Shelton, Thanks for the query. A chronic gastric ulcer or a gastritis is a possibility as your symptoms increase on eating food. You should probably get an upper Gastrointestinal endoscopy done. A chronic H.Pylori may also be a cause. A medical gastroenterologist would be able to help you with this. Consider taking proton pump inhibitors like Pantoprazole. Eat bland diet, drink plenty of water. Get well soon."
},
{
"id": 59847,
"tgt": "Diagnosed with Autoimmune liver failure, on steroids. Took Piriteze tablets, liver function tests high. Relapse ?",
"src": "Patient: Hello there, Back in 2006 i was diagnosed with Autoimmune Liver Failure and spent six weeks in Hospital. They then started me on a cause of Steroids Prednisalone then changed me over to Azzaziaprin. I responded really well to these treatments and then was told that all of my tests were back down to normal so they stopped treatment and i have been fine for three years, just before i went to spain back in July i took some Piriteze tablets which i have read can cause problems with the liver and abnormal liver function tests and that is when i had a blood test a few months ago and was told that the Liver function tests were high (262) is this down to the tablets or is it a relapse? Many thanks, AAA Doctor: Hi Tom, usually autoimmune liver disease comes and goes. You can check your liver enzymes one time and they may be normal, and then one month later they may be up and so forth. Usually a low dose of azathioprine (generic) is enough to keep the disease under control. Keep in mind that if you don't keep it under control (with normal liver enzymes), then the inflammation can progress to \"cirrhosis\" which when the liver is almost non-functioning and requires a liver transplantation. Make sure you go to a liver specialist for treatment recommendations. You may even need a liver biopsy."
},
{
"id": 214123,
"tgt": "I hear voices in the wind like some evil some good, how to get rid of them ?",
"src": "Patient: Voices In The WindI hate it but i gotta live with it.... I hear voices in the wind... some evil some good....\u00a0 why do they keep pestering me? I recognise some of these voices as people from my past... people who damaged me in some way... i have forgiven these fools but i still hear them... Why won't they leave me alone? Any ideas for getting rid of them?\u00a0 I've been diagnosed as Schizophrenic 10 years ago but i believe also i have the gift (if you can call it that) of Telepathy!! I'm already on meds for my illness but they havn't completely rid me of the voices as you can see... I've seen a Psychiatrist/Councellor/Doctor about this but it looks like i'm just with it for life!! I do wonder though if maybe i'm hearing the dead!! BTW I AM A SPIRITUAL PERSON Doctor: If u have heard about it then a Psychic healer might help you."
},
{
"id": 219847,
"tgt": "What are my chances of getting pregnant at this time?",
"src": "Patient: Okay well me and my boyfriend fooled around about 3 weeks ago. We were skin on skin but he didnt put in because i didnt want to have sex. He did NOT cum and he only pre cumed when i gave him a hj. Im not on birth control and I didnt know you could get pregnant without having sex. Ive never kept track of my period but I think I usuall get it around this week. I have took 2 preganancy tests and they both came out negative. I have been feeling sick and my boobs just started hurting today but alot of the symptoms are things that i have normally like (intigestion, tired, mood swings, etc.) People say that im just freaking out but I dont know. Im still freaking out! Could i be pregnant? Could the pre cum have gotten inside of me some how? PLEASE HELP ME!! :( Doctor: Hallow Dear,You have right information about precum. Precum does contain few sperms which can fertilize the egg. However, the precum can cause the pregnancy only if enters the vagina. You had masturbated your partner and his precum was ejected outside your vagina. Hence, there is no possibility whatsoever of pregnancy due to this hand job. Moreover, both the pregnancy tests have reported negative. So forget about pregnancy. The discomfort in your breasts can be due to rough handling of the breasts. I would advise you to use a reliable birth control method like condom, pills or intrauterine device in future during peno-vaginal sex to avoid such apprehension. One fact is sure that mutual masturbation will never lead to pregnancy. I hope this relaxes your mind. Dr. Nishikant Shrotri"
},
{
"id": 145727,
"tgt": "Suggest treatment for cyst in brain,vomiting and headache",
"src": "Patient: I have been diagnosed with a cyst on my brain, the dr. said he didn t think it was cancerous. For six years now i have had headaches, I wear bifocals now, my hearing is not that great, i stay tired, i have memory and thinking problems, i sometimes have speech problems along with left side drawing in my face and arm. At times my sexual desires are very high for me, my emotions are all over sometimes with alot of pressure in my head. I have been getting sharp pains with instant vomiting. i fall to sleep easy. I didn t ask any questions because all i was interested in was none cancerous. should a cyst be giving me all these symptoms? Doctor: Hi,Thanks for writing in.A cyst in the brain, present for more than 6 years is a stable lesion. However, it is important to know the location of the cystic lesion to know the exact implications and to confirm if your symptoms are due the cyst or there is any other condition that is responsible for vomiting. As you report emotional changes and sexual desires being part of your complaints, it is important to know if there is any anxiety related stress or a similar condition largely causing the symptoms. Please detail us about the location and size of the cyst in your bain and any recent enlargement is size. Cysts in the posterior fossa and causing pressure effects on the brain stem including pons and medulla can lead to vomiting."
},
{
"id": 129269,
"tgt": "Could pain in knee be caused by strain of the vastus medialis or sprain of medial collateral ligament?",
"src": "Patient: I have a history of trauma to the knee. A physician diagnosed me as having sprained my medial colateral ligament and a 1st degree medial meniscus tear. The physical therapist disagreed and said it was a only a strain of the vastus medialis. So i was treated for that. I was fine until returning to biking left my knee sore. Now I'm experiencing slight pain. My knee becomes extremely stiff after having a shower. What is going on? Doctor: Hi. My opinion is that a vastus medialis strain would have healed with physical therapy, but a meniscal injury can cause persistent pain or discomfort. An MRI will give a definite diagnosis. Meniscal tears do not heal easily and you may need an arthroscopy to clean it. I advise a visit to an Orthopaedic surgeon. Hope this answers your question."
},
{
"id": 174823,
"tgt": "How to treat diarrhea for 1 year old?",
"src": "Patient: my 1 yr old has had greeny slimy fishy smelling diarrhea, and some vomitting for a couple days now. her bottom is red, have been putting desitin on it but, but she screams when i change her diaper. i have been giving her pedialyte as well. please help. Doctor: Most likely your daughter is suffering from acute gastroenteritis. Unless associated with bloody stools, the condition is self limiting. Give Pedialyte or ORS (Oral Rehydration Solution) and plenty of other fluids. Replenishment of the lost water and salts as the mainstay of treatment. Treatment is not supposed to bring down the frequency of passage of loose stools. The adequacy of the hydration would be evident from the frequency of passage of urine. If that seems to be fine, you need not worry. But unless the child grows lethargic, I would not get concerned. It may still take 3-7 days (from the onset) to stop. The excess moisture in the diapers has caused perianal erosions which are painful and hence the child screams while the diapers are being changed. Try to keep it open to air and apply Siloderm Mixi ointment. It should provide relief.You may give zinc supplementation with Zinconia Syr 5 ml twice daily for 5 days. It will help in the intestinal healing as the diarrhoea causes some damage to the inner wall of the intestines. It is preferrable to avoid milk and wheat based items for a week or so. But you may also give plenty of curd or yogurt. It contains plenty of beneficial bacteria, same as that contained by Bilfilac, and is supposed to be helpful in such a situation."
},
{
"id": 196482,
"tgt": "What causes blood protein level high even after having coumadin?",
"src": "Patient: My father in law was tested today and advised that his blood proton levels were still to low and he should remain on Coumadin. He has been on Coumadin for about a week and a half to get his clotting controlled in order to allow him to have a prostate biopsy. Doctor: Hi, I had gone through your question and understand your concerns.Well, i am not sure if you understood your doctor good. Blood proteins are usually not related to coumadin therapy. And coumadin is used for prevention of deep vein thrombosis. Blood protein sare something else and you shoul dspecify what kind of proteins are yut talking about. If you mean tumor marker PSA then it will not reduce without urologic intervention, If you mean prothrombine time then he should titrate coumadin dosage till adequate prothrombin time is established.Hope this answers your question. If you have additional questions or follow up questions then please do not hesitate in writing to us. I will be happy to answer your questions. Wishing you good health."
},
{
"id": 31624,
"tgt": "What causes persistent low fever, body ache and insomnia?",
"src": "Patient: i am having a low grade fever and body aches, more on back and legs every evening.. i am also observing Insomnia and eye burning. and along with this i have strong headache, which is located at temporo-occipetal region on both side of my brain. i am already sufffering from Haemorrhoids and Irritable bowel syndrome. please help me Doctor: Hi Dear,Welcome to HCM.Understanding your concern. As per your query you have symptoms of persistent low fever, body ache and insomnia which seems to be due to viral infection due to compromised immunity of your body and susceptibility of getting infected. Sometimes initially the infection is viral but with times secondary bacterial infection occur , which cause further complication. Need not to worry. I would suggest you to consult ENT specialist without any delay. Doctor may prescribe anti viral acyclovir/ antibacterial penicillin along with anti inflammatory medications. Doctor may order blood test to confirm diagnosis. You should take proper rest. Yous hould take muscle relaxants if fever is of high grade.Hope your concern has been resolved.Get Well Soon.Best Wishes,Dr. Harry Maheshwari"
},
{
"id": 20614,
"tgt": "Suggest remedies for palpitations and sporadic dizzy spells",
"src": "Patient: I am having almost constant palpitations and sporadic dizzy spells, even when I m just sitting doing nothing. My heart feels like it has the odd kick start with a large thump and I can feel it in my throat. I can t get an appt unless its an emergency until next wed, should I ask to see the duty doctor in a hurry ? Doctor: Yes you need to urgently visit your doctor as it may be a heart rythm problem going on, an urgent ecg would help you in this case and may prevent any future complications too"
},
{
"id": 202154,
"tgt": "Suggest remedy for erection problmes",
"src": "Patient: Hi i am 30 yrs male i am finding difficulties in eraction and also my penis is not hard enough to penerate the surface of vagina as tried and failed twice few month back as i have been practing materbating from last 14 yrs and also in pronography please advice me as getting married next month. Doctor: Hello dear,Regarding the problem of Erectile dysfunction, it is due to decreased blood supply to the penis.Though medications like Cialis & Viagra increase blood flow to penis & maintains erection, it can be improved by having a diet rich in fruits, vegetables, fish, nuts & honey.Connections between erectile dysfunction & excess use of porn has been reported in many studies.For those affected, recovery takes place by 6-12 weeks & mainly involves avoiding the causative factor, that is watching porn.Avoid stress...practice meditation.Avoid smoking & alcohol.Despite these measures, if you experience difficulty, it will be better to rule out other causes like high blood pressure, blood sugar and other local causes that impair blood flow to the genital organ.Wishing you a Healthy Life.Take care."
},
{
"id": 210814,
"tgt": "Suggest remedy for reduced learning abilities, stiff hands due to intake of medication for schizophrenia",
"src": "Patient: Hi! My son is taking a lot of anti depressants for treatment of schizophrenia. Although they r helping him in relieving his moods & depression but these r hindering his learning abilities.He is taking them regularly since 2 yrs now but gradually they r reducing his learning abilities.He is not able to learn his lessons. He forgets the lessons he learns & is not able to write them properly in exams.With his high school board exams approaching, I'm really worried.Pls suggest something. One more problem is that his handwriting has really become too bad & illegible because medicines are causing lot of stiffness in his hands.Pls suggest something.Doctors hv tried altering a lot of medicines but they can't put him totally off these medicines. Doctor: Hello,Thanks for choosing health care magic for posting your query.I have gone through your question in detail and I can understand what you are going through.His learning abilities have been hampered probably due to the cognitive deficits of the schizophrenia itself rather than the medicines. But the stiffness in hands I agree is due to the medications and if he is not tolerating then the medicines need to be changed. You have not mentioned which medications he was earlier on and hence I cannot suggest you some new meds but overall the option of olanzapine or quetiapine are quite safe. Hope I am able to answer your concerns.If you have any further query, I would be glad to help you.In future if you wish to contact me directly, you can use the below mentioned link:bit.ly/dr-srikanth-reddy\u00a0\u00a0\u00a0\u00a0\u00a0\u00a0\u00a0\u00a0\u00a0\u00a0\u00a0\u00a0\u00a0\u00a0\u00a0\u00a0\u00a0\u00a0\u00a0\u00a0\u00a0\u00a0\u00a0\u00a0\u00a0\u00a0\u00a0\u00a0\u00a0\u00a0\u00a0\u00a0\u00a0\u00a0\u00a0\u00a0\u00a0\u00a0\u00a0\u00a0"
},
{
"id": 36770,
"tgt": "Suffering from insect bites on one side of the leg",
"src": "Patient: About 4 years ago I was bitten by a hunter dog on the leg. A couple of months later I felt a bite on same leg - like an insect bite. Since then I have lots of insect bites all on same leg but no-were else on the body. They are not beg bug bites (looked at pictures of these, but similar) but confined to upper half of one leg and buttocks - please advise - have been to doctor and was given cream to no avail. Doctor: HIWell come to HCMIf this is insects that bite on your leg then try to cover the legs with socks or stocking, perhaps this is not the insect bite, but could be allergic reaction, this can be treated with Tab Levocetrizine 10 mg three times in day, for local application Beclomethasone cream can be applied, hope this information helps, have a nice day."
},
{
"id": 47801,
"tgt": "What does the following kidney function test report indicate?",
"src": "Patient: Kidneys Test (10/02/2014): FINDINGS: There is mild right hydronephrosis/extra renal pelvis. The kidneys are normal in size, contour, cortical thickness and cortical echogenicity. Right kidney: 13.7 cm. Left kidney: 12.8 cm. There is normal renal perfusion. No renal calculi are identified. There is no evidence of hydronephrosis. No renal mass the as identified. IMPRESSION: Mild right hydrophones/extrarenal pelvis. Otherwise unremarkable kidneys (normal) Should I worry, Dr.? Doctor: Hi,Thanks for writing in.The left kidney is normal in size and appearance.The right kidney size is normal but there is an appearance described as increased fullness of the renal pelvis. The renal pelvis connects the kidney collecting system to the ureter. In your case, there is no infection or stone which is causing obstruction to flow of urine. However this can be seen in normal people due to increased pressure of urine in the kidney or also as a normal variant from birth.It is suggested that you please get the ultrasound scan done after passing the urine and this will confirm any back pressure effects on the right kidney. In most cases it does not require treatment. Please do not worry."
},
{
"id": 116555,
"tgt": "Can low serum creatinine level on blood test be the cause of dizziness and sore throat?",
"src": "Patient: My daughter is 12years old, 165cm and 48.2 kg. doesn t feel very well since 1 year ago. Dizziness and weakness are the major concern. She is pale, sleeps more than my baby with 11months. Some times she has Stomach ache and sore throat but not every month. The last time a two weeks ago, she went on hospital dizzy, vomiting, headache. The blood results low are: Serum creatinine 56umol/L, RBC 3.90, WBC 4.1, Eosinophils 0.08. In February she went on hospital dizzy . The blood results was a little similar. Doctor: Hello,Thank you for your contact to healthcare magic.I understand your health concern, if I am your doctor I suggest you that dizziness may be because of some electrolyte imbalance. I advice you to do serum electrolyte testing. And if deficiency is found you can deal it with accordingly. I will be happy to answer all your future concern. Thank you,Dr Arun TankInfectious disease specialist.Wish you a best health at health care magic."
},
{
"id": 208849,
"tgt": "Suggest treatment for anxiety",
"src": "Patient: Hello. I am 19 and I have fairly bad anxiety. What I mean is, I have very bad anxiety. I have for the past year or two. I am very overweight, and my anxiety attacks are mainly illness phobia related ones. I am under alot of stress the last week, I have applied for an apartment that I am desperate to have. About an hour ago, I had a heart thumb. Skipped a beat. Etc. It frightened me, so I took a deep breath and it felt strange. This caused me to have an anxiety attack, and I had mild chest pain on the left bottom side and i got nervous and a little sick to my stomach. I believe most of these were symptoms from the following anxiety. I had an ECG in March because I went to the ER with arm pain. They said everything was fine and it was probably a muscle/nerve issue. I know, logically, that I'm fine. My doctor has told me that the heart does this to 'catch up' with itself. I don't want to go to the ER, I love 25 minutes away from the nearest hospital. The pain is gone, I am just still a little nervous. The only thing still bothering me is that I get a gassy rumbly feeling in my upper abdomen, sort of below my heart. I passed gas a few seconds later. Could that be gas, so close to my chest and heart? Doctor: Dear User,Thanks for using health care magic.From the available description it appears that you are having very problematic anxiety symptoms which needs to be addressed. I understand your will of not visiting ER but you can visit your primary physician. It os absolutely essential to get it treated as soon as possible before it makes you helpless completely. Before you can visit a doctor you are advised to practice some stress management strategies which are available to you. 'Hope I have answered your query. If you have any further questions I will be happy to help\".Thanks"
},
{
"id": 97138,
"tgt": "Suggest ointment for healing abrasion after accident",
"src": "Patient: I had an road accident and my ankle got a crack (lower end of right tibia) and there was an abrasion. The abrasion did not cured after two week application of saffromycin ointment. May I use T bact or there is any better ointment for healing of my abrasion. Doctor: Hello,Yes, you can use T-bact ointment if you have already started using it.Superficial abrasion will heal faster compared to deep abrasion. injury over bones take more time to heal. So keep using ointment.These measures will help your wound heal faster.1. Kindly keep your wound clean and dry, since an infected wound will take more time to heal. 2. See if there are any contaminants in the wound preventing healing.3. Wash your wounds with antiseptic solution, dry it with cotton and apply ointment as much as required.4. Take help from trained nurse or health worker.Hope I have answered your query. If you have any further questions I will be happy to help.Thank you,Dr. Narasimha G L"
},
{
"id": 174654,
"tgt": "What could presence of E.histolytica (in cystic form(0-1/HPF)) in stools indicate?",
"src": "Patient: hello doctor my 8 month baby girl weight 7.45 kg is suffering from loose motion since 9 days.done routine examination of stool in which microscopical exam states presence of E.histolytica in cystic form(0-1/HPF).i have to labrador pet in my family.general reason for infection & treatment.thank u Doctor: HI based on your query, I can reassure you that E.coli are the common bugs in our gut and noneed to trat with any antimicrobials which will further aggravate the loose stools. Just replenish your child with more ORS fluids and zinc syrup to prevent dehydration"
},
{
"id": 200273,
"tgt": "Suggest treatment for problem with foreskin retreaction",
"src": "Patient: hi there, I have always had a problem which is when retracting my foreskin I find my penis is very sensitive to touch. In warm water it s easy to retract and touch but otherwise it s extrememly sensitive. I am 24 yrs old and have always suffered from this problem. San. Doctor: Hello dear,Thank you for your contact to health care magic.I read and understand your concern. I am Dr Arun Tank answering your concern.It is cold sensitivity to the penis.There are many people suffering from this issue. The only possible treatment for this problems is surgery of the foreskin. This surgery is named as circumscision. You should visit the surgeon nearby and get your self operated.Till you visit the surgeon please avoid force ful retraction as some times it can cause injury.I will be happy to answer your further concern on bit.ly/DrArun.Thank you,Dr Arun TankInfectious diseases specialist,HCM."
},
{
"id": 164477,
"tgt": "Suggest medication to reduce high fever in child",
"src": "Patient: My ten year old son has a temp of 103.5, he s well hydrated with water and Emergen-c, and did give a cough suppressant and decongestant around 4 hours ago. He hardly ate all day, slept on and off, and slept in good blocks of two-3 hours so far this night. We have not given fever reducers but did a warm shallow bath, temp did not reduce after cooling off after bath Doctor: 103.5 is high grade fever, immediately give him antipyretic medicine like paracetamol even a combination of two antipyretic.do sponge Ur baby with tap water.make him undress to minimal clothing"
},
{
"id": 7566,
"tgt": "Severe acne, oozes white pus on popping, leaves dark mark on healing, advised isotroin. Is it effective and safe?",
"src": "Patient: hi, i am chandan 21 year old guy from India. i have severe acne on my face from past 6 years. its in the form of nodules and when it breaks open white puss oozes out, after healing it leaves behind a dark mark or a crater kind of markings on skin. i have undergone treatment for the same but none of them has been effective. i was prescribed doxy-100 and levoflox along with acnesol gel to apply on the pimples , during the treatment period acne reduces but returns back within 10 to 15 days after the treatment process. a friend of mine(who finished his Medical degree and working as an intern now) suggested isotroin for 7 days during night after dinner, he also mentioned about possible side effects. i wanted to know whether will my acne problem reduce and also how safe is it take the isotroin for my acne problem. Doctor: this is no cure i advise you to stop mik and diary products no ils mustard coconut walnut groundnut almonds soya apply olive cook in olive no milk and nut containg soap oils creams and ointments take oil free and greese free diet tab doxitab 100 mgm bd for 10 days apply clindac a gel morning and evening calamine in after noon do 2 months and ask for next step"
},
{
"id": 148845,
"tgt": "Small bump behind neck, dizziness, euphoria on rubbing it, sharp pain in tongue. Cure?",
"src": "Patient: I have a small bump on back of neck, either muscle or spine , not sure, when I rub it it gives me a weird sensation across back of head and neck , makes me kind of dizzy or euphoric, and feeling lasts for extended time. I sometimes wake up with soreness at site and across back of head. I also have been experiencing sharp pains in tongue while I sleep and sometimes bite tongue in sleep. Doctor: Hi,Thank you for posting your query.Your symptoms require detailed evaluation by a neurologist or a neurosurgeon.If possible, please send a picture of the bump over the back of neck. It may help in the absence of being able to examine you.Sometimes, an ultrasound or MRI scan of the bump can give us more clues regarding its nature.Blood tests such as vitamin B12 level and thyroid profile should also be tested.I hope it helps.Please get back if you require any additional information.Best wishes,Dr Sudhir Kumar MD (Internal Medicine), DM (Neurology)Senior Consultant NeurologistApollo Hospitals, Hyderabad,My personal URL on this website: http://bit.ly/Dr-Sudhir-kumar My blog: http://bestneurodoctor.blogspot.com/"
},
{
"id": 163639,
"tgt": "What causes child to pass loose stools with blood?",
"src": "Patient: hi doctor, my 1 year old kid is suffering from loose motion for the past 6 days.at first it was frequent but now its not frequent but he passes loose stools and there is slight blood stain also.Bloodstains where there from the begining itself.Is it dyssentry.What should i do?please help me Doctor: your child has dysentry, you need to go for stool culture for confirmation..and need to start antibiotics..keep the child well hydrated and also give zinc for early recovery.."
},
{
"id": 196577,
"tgt": "Will the sore scrotum and rectum be due to fungal infection?",
"src": "Patient: I had sex on Friday night with a condom, woke up today and had sore scrotum n rectum. Went to doctor who says it is a fungal thing, had blood & ursine tests for herpes etc.... Negative! But how likely is it that it is fungal and not herpes??? I am soooo scared! In Tanzania!! Doctor: Hello Thanks for the query There is a possibility of fungal infection. However with the use of condom chances are less.Please get yourself investigated. Let me know if you have any other doubts. Thank you."
},
{
"id": 155113,
"tgt": "Does surgery make cancer to spread if having nodules?",
"src": "Patient: I am faced with either a needle biopsy of my lung, I have a nodule actually several, or surgery to extract the nodule and find out what this is. Two doctors have said this doesn t look like cancer but they can t say for sure. There are other nodules on my spine and lymph nodes. I can t make up my mind which to do. Does surgery make cancer spread or if air getting to cancer is just an old wives tale? Doctor: Hi, dearI have gone through your question. I can understand your concern. You should go for needle biopsy of nodule. It will give you exact idea whether it is cancer or other lesion. Then you should take treatment accordingly. No need of surgery right now. Hope I have answered your question, if you have doubt then I will be happy to answer. Thanks for using health care magic. Wish you a very good health."
},
{
"id": 9403,
"tgt": "What causes a dry and itchy patch on areola of the breast?",
"src": "Patient: For about a month I have had a dry patch on my right areola. This sometimes is itchy. The skin will flake off and then will dry up again and do th same thing. Please can you let me know what this might be as it doesn't seem to be going even after using a prescribed fungal cream. Doctor: Hello,Thank you for posting on HCM.It seems you are suffering from Tinea corporis, a kind of fungal infection.I would suggest you to consult your dermatologist for proper management of the condition.I usually recommend proper course of oral anti-fungal drugs like Fluconazole or Itraconazole along with anti-fungal cream like luliconazole for local application at bedtime. You can additionally use antifungal dusting powder containing sertaconazole during day time and a soap containing ketoconazole for rinsing of affected areas. Take oral antihistaminics for itching as required.Maintain hygiene over those areas and avoid wearing tight undergarments.Hope your queries are resolved and wish you best of health.Kindly spare some time to rate my answer and drop your valuable review at the following link:http://doctor.healthcaremagic.com/doctors/dr-hardik-pitroda/67169If you require any further assistance in future, you can reach me directly through the above mentioned link.Thank youDr Hardik Pitroda"
},
{
"id": 150692,
"tgt": "Swollen bulging area on the back along the spine. Has a herniated disc from an injury. Are these related?",
"src": "Patient: Hi, My son has a swollen or bulging area in the middle of his back that lays along the spine 3-4 inches long and 1.5 inches wide. It is not red or sensitive to the touch. He also has a herniated disk that is lower in his back from an injury. What is the swollen (raised area) in the middle of his back? He says its uncomfortable there and we noticed it when he laid on his stomach . Doctor: Hi, Thanks for writing to health care magic. Since how long did you notice this swelling. Is it there since birth or of recent origin. Is there any discharge or a ooze seen coming out of the swelling. Are there any other complaints associated. Please reply in detail so that I can help you better. You may contact me with further details on the following URL........... http://www.healthcaremagic.com/Funnel?page=askDoctorDirectly&docId=65124 In such scenario the questions are directly forwarded to me .............. and I can answer your queries regarding your kid online regularly. Hope this helps. Take care Dr Y V Siva Sankara Murty M.D.(Pediatrics) Associate Professor of Pediatrics"
},
{
"id": 190123,
"tgt": "59 year old having nausea. Upper endoscopy report normal. Other reasons?",
"src": "Patient: This is for my mom whose is 59. Since last 5 weeks or more she is getting nausea . She has become very sensitive to the smell of certain products like cooking oil and she nauseates on eating fried foods. But she does not vomit or show any other symptoms. She has her upper endoscopy done and ultrasound too. Everything looks ok. What could be the reason for such symptoms and whom to approach for? Worried!!! Doctor: Hello Welcome to HCM You didn't mention her complete medical history like was she suffering from any chronic infection? Has she got some problem with gall bladder like stone?? Is she on some medication?? I would suggest you to give her green leafy vegetables & fruits.Ask her to take more salad. Avoid spicy & oily food stuff. increase apetite. Take oranges as they Concern general physician for further opinion. Take Care Regards Dr.Neha"
},
{
"id": 22366,
"tgt": "What causes variation in heart beat and dizziness?",
"src": "Patient: I am 61 years old, 5' 10\", 178 lbs., My blood pressure is 120/80, I am hypertensive for 15 years, I take norvasc 5 mg. and concore 2.5mg once a day. I am having this sudden, almost a split second dizziness, as if it will knock me down, but just as instantly, I will be able to recover. Then this will be followed by heart palpitations. My kind of palpitation involves only a feeling that my heart is pounding harder than usual rather than an actual increase in my heart rate which stays at 65 to 70 beats per minute. I'd have a 24-hour bp monitor, what was seen was that at that time when I felt that dizziness, there was a skip in my heat beat which my doctor says is not a problem. But I feel pretty uncomfortable with it and I want the problem solved. Thank you. Doctor: Hello,We first need to figure out the reason, whether there is any extra beat as well called as APC or VPC or there is missing of the beat. How much is your usual heart rate, is it on lower side like in sixties or fifties, or it is more than that. If there is vpc or apc then you need to increase the dose of Concor or if there is missed beat then dose of Concor to be reduced. For better discussion, you need to upload the ECG. So discuss these issues with your local doctor.Hope I have answered your query. Let me know if I can assist you further. Regards,Dr. Sagar Makode"
},
{
"id": 180239,
"tgt": "What causes extreme pain on neck?",
"src": "Patient: Hi my 4 yr old granddaughter is having extreme pain on right side of her neck. Not an insect bitem noredness. Think maybe a pinched nerve as it has been ongoing now for approx. 5 hours since we arrived home from shopping. No fever. I gave her Tylenol tablet 4 hrs ago and she slept for about 1 1/2 hrs. Heat doesn t seem to help. but she likes a cold, wet facecloth which I have folded and placed around back & side of her neck. Thanks for any advise. Raymona Doctor: HIThank for asking to HCMI really appreciate your concern, the neck pain in this age is not common and this could be common cold or some lymph node could be there and that might have stiff the neck muscle and this is not the neurological problem symptomatic medicine would be helpful, hope this information helps you, have a nice day."
},
{
"id": 68923,
"tgt": "Suggest treatment for lump in the injection site",
"src": "Patient: i have recently been put on gonapeptly injection for 3 months for treatment of fibroids in preparation for an open myomectomy i have had a sucessful myomectyomy but i have noticed a lump in the injection site which has no grown slightly bigger i am concerened and i have recenlty spoken to my doctor bout this and i am about to go for a scan. will apprciate any information or help on this. thanks Doctor: Hi Dear! This is Dr. Rajiv Ranjan MS, FIAGES ; from Gaya ,Bihar, India talking to you.A lump at the injection site persisting ,growing and troubling is an injection abscess or an antibioma until proven otherwise.Therefore, you must visit a local general surgeon and get the lump excised or you may visit my center for the same if possible. Thanks!"
},
{
"id": 221648,
"tgt": "What does the absence of fetal pole indicate at 4th week of pregnancy?",
"src": "Patient: hi i dont know if you can help. i am pregnant and my last period was end of november witch would put me at around 6 weeks i went for a scan on 14th jan and they could only see the pregnancy sack and yoke but not the baby the lady said im probably 4 to 5 weeks should i be worried Doctor: Hi, I understand your concern. Absence of fetal pole at 6 week gestation of pregnancy indicates it's a chemical pregnancy/ blighted ovum.. where fetus is not formed, but only hormonal responce like pregnancy occurs. This eventually is terminated automatically or by evacuation surgery. But before going to a conclusion, it's better to cross check by a follow up scan after a week at another USG center. Thanks.."
},
{
"id": 203701,
"tgt": "How to stop using sex pills for getting strong erection and enjoy sex?",
"src": "Patient: Hi, i have problem ,i hope you can help me, i masturbate since 13 years old,3 times a day, every, morning lunch and before bed time, i could not fall as sleep if i dont masturbate, now i m 60 years old now, i cannot get long erection, seen like no connection from my brain to the penis,i took sex pills, my penis keep erection, but no sensation, cannot cum, only by hand, i m taking sex pill once every weekend, i am afraid i might die from heart attack, because sometime i have problem breathe when i sleep.how i can stop using sex pills and still enjoy sex ? please help , Jimmy Doctor: DearWe understand your concernsI went through your details. I suggest you not to worry much. You are sixty. People become tired in your age for sexual matters and average male can only engage in sexual activities once in a week or so and that too with much lethargy. Here you are talking about masturbation thrice daily. I am afraid to say that your mind had become addicted to masturbation which troubles your body and mind. Over and above you are insisting in sexual activities also.Please, in your age, you are supposed to act like a matured, father figure, not like a romeo or sex maniac. Please understand, and visit a psychiatrist and do according to what he suggest.Hope this answers your query. Available for further clarifications.Good luck."
},
{
"id": 73208,
"tgt": "What are the symptoms of pneumonia?",
"src": "Patient: diagnosed with type b flu on sunday, getting worse even with meds, now have rattling breathing and cough, only slight fevers that come and go, chest hurts, throat hurts, very weak, no appetite. And almost 33 weeks pregnant... could this be the sign of pneumonia coming on? Doctor: Thanks for your question on Healthcare Magic.I can understand your concern. Yes, your symptoms are suggestive of pneumonia more. Pneumonia (lower respiratory tract infection) is common after upper respiratory tract infection (URTI).Worsening cough, chest congestion, rattling sound from chest, weakness, loss of appetite etc are characteristic symptoms of pneumonia. So better to consult pulmonologist and get done clinical examination of respiratory system and chest x ray (with abdominal shield).You should definitely start higher antibiotics to recover faster as you are pregnant. Hope I have solved your query. I will be happy to help you further. Wish you good health. Thanks."
},
{
"id": 187312,
"tgt": "What should be done during the pain of wisdom tooth extraction?",
"src": "Patient: I had a wisdom tooth extracted 3 days ago and I am still in a lot of pain. The blood clot fell out on the morning of the third day and my jaw feels very stiff. I can only open it about an inch and have a hard time seeing where the tooth's socket is.I don't have a fever just lots of discomfort. What should I do? Doctor: Hiii,I've gone through your query and as per you describe your condition there is infection at the extraction site. To get relief from pain you are adviced to do-1) Add 1 tbs of salt in warm water and gargle with it thrice a day,this will definitely give some relief. you can use alcohol free mouth wash also.2)if you smokes then stop smoking until the wound heals because smoking causes unbearable pain in such situations.The extraction site has to be cleaned and dressed with medicated pack for that you have to go to your dentist becaue mere above steps are not sufficient to reduce the pain. your dentist would prescribe antibiotics and pain killers which would help in healing of wound.I hope you get well soon. Please give us a feedback."
},
{
"id": 169424,
"tgt": "Suggest treatment for stuffy nose and cough in kids",
"src": "Patient: My 21 month old son has not been eating and drinking very well for the past couple of days. He has a stuffy nose which is preventing him from breathing properly. He is also waking at night with a bad cough. I am a bit worried also because his symtoms are really affecting him. Should i take him to the hospital? Doctor: Hi Dear,Welcome to HCM.Understanding your concern. As per your query your kid have symptoms of stuffy nose and cough. Well the symptoms you mention in query occur due to many reasons like upper respiratory infection ( bacterial or viral ). Need not to worry. I would suggest you to consult chest specialist . Doctor may order certain test like nose or throat culture and test , blood test and chest X-ray to confirm the diagnosis and rule out other condition like viral infection. Doctor may prescribe antibiotics , anti inflammatory and decongestants. Avoid intake of outside food. For now take over-the-counter cough medicines, cough suppressants, or expectorants , which give relief from coughing . Hope your concern has been resolved.Get Well Soon.Best Wishes,Dr. Harry Maheshwari"
},
{
"id": 23063,
"tgt": "Is a recovery possible in a person suffered with heart attack and in coma?",
"src": "Patient: My friend had a Heart Attack. All I know from talking to his wife is that they did not perform any surgery, and that his heart stopped beating for about 10 minutes before any attempt to resuscitate could be made. He is in a \"self-induced coma\" on life support and they are starting a process to turn the life support down to see if the body reacts or starts to show signs of taking over. My questions are: What is the potential for brain damage?; and Is a recovery possible? Doctor: DEAR USER,THANKS FOR CONSULTING WITH HCMI UNDERSTAND YOUR CONCERN.. I M REALLY SORRY AS ANSWERING ABOUT RECOVERY OF A CRITICALLY ILL PATIENT BEFORE GOING THROUGH ALL THR REPORTS AND THE PATIENTS PRESENT CLINICAL STATUS IS A BIT CUMBERSOMEBUT I BELIEVE THE DOCTOR IN THE ICU OF YOUR PATIENT CAN TELL YOU BETTER AND THAT WILL HELP YOU ALOT.. YOU CAN TALK FREELY TO HIM ABOUT THE PATIENT.HOPE I ANSWERED YOUR QUERY. YOU CAN MESSAGE ME FOR ANY FURTHER CONCERNS"
},
{
"id": 134575,
"tgt": "What causes arm pain with dizziness?",
"src": "Patient: I am a 32-year old female and I have been having pain in the left side toward my armpit for a couple of weeks now. It feels like pressure. Today, I have been extremely dizzy, but Imthought this May be due to getting a vitamin shot yesterday. I have also experienced nausea on and off, as well as heartburn. Can you please tell me what you think this May be. Doctor: you can surely visit a physician and get yourself checked. a cardiologist will be of recommendation."
},
{
"id": 118722,
"tgt": "Low HB, iron values, had increase in count post transfusion. Normal test results. Cause?",
"src": "Patient: hi there, I am 34yr old female and have a HB of 10.6 (it was at 65 in January when i was 36 weeks pregnant and received 3 units of blood ) and my iron is at 6. Since the transfusion, it climbed to 80 then 110 (ferrous fumarate 3x day) but seems there may be an absorption issue. What could be causing this? Various tests were done post transfusion, not GI or Endoscopic though. Doctor: Hi. The most common cause of anemia in females is menstrual loss which leads to iron deficiency anemia. Im not sure if your menstrual cycles are normal and that you do not have too much blood loss. If not, there may be parasitic infection in your gut which can cause blood loss. A stool for occult blood may be done to look for occult blood loss through peptic ulcers. The other condition which can cause recurrent anemia, which im not sure if you have is celiac disease. Guess you should not worry right now since your tests are normal. Thank you"
},
{
"id": 185460,
"tgt": "What are the dark spots on the tongue?",
"src": "Patient: I am 26, 5ft 3 and iron deficient. I also have recurrent vaginal thrush. I have had dark spots on my tongue since I was quite young. My tongue is a bit thick as well. Should I be concerned and what might be wrong with me? Also can these go away? If so, how? I also have purplish colour on the side of my tongue which I only got this year. Could this be because I have taken iron tablets? Doctor: Hello, Thank you for consulting with HCM.Iron deficiency does not cause these kind of pigmentation, it can be possible that these are racial pigmentation, means in some people they are present since childhood and there is nothing to worry about it , as they do not cause any problem.But keep observing them, if in future there is problem in them then go for the examination and treatment.Hope it will help you."
},
{
"id": 222351,
"tgt": "What is the best treatment to get pregnant?",
"src": "Patient: I have been in a loving and committed relationship for three years now and have begun discussing having children. My boyfriend was injured in high school and as a result, got fixed 6yrs ago. I really want children and even though he s got two, he wants to make me happy but everything I m read about reversals are that they are expensive and are not guaranteed to work. I guess my question is, what are our options? Doctor: dearread your queriesreversal surgery in a good hand gives good result.Apart from male factor we need to check you.get the opinion of any urologist for your boyfriend . another option is of adoptionthink itDr.Mira Butani"
},
{
"id": 28903,
"tgt": "How can chicken pox in a child be managed?",
"src": "Patient: Hi, my daughter, 4 y/o is having chicken pox, do you recommend cethapil for her? if so, is it okay to use cethapil lotion too? And is it okay to apply calamine lotion (calmoseptine oinment) to open wounds? Please give me your recommendations doc..thank youso much Doctor: Hello,Cetaphil is a good lotion that can be used. So can calamine lotion. In my opinion, you should avoid applying it on open wounds.Keep her well hydrated and avoid contact with other kids. Help prevent your daughter from scratching at the vesicles, to minimize scarring.Hope I have answered your query. Let me know if I can assist you further.Regards,Dr. Pranav Balakrishnan"
},
{
"id": 169914,
"tgt": "Why is saline + duolin nebulisation prescribed for cold in babies?",
"src": "Patient: Hi, My daughter is 14 months old. we are from India, presently staying in south part of the country. She keeps catching cold and cough often. Earlier, she used to be presecribed only saline neblisation with the usual syrups of chlorpheniramine, terbultaline etc. But, this time she has been presecribed saline + duolin nebulisation. What does that indicate ? Is she predisposing towards asthma ?? Doctor: Hello, Thanks for posting your query at Health Care Magic. Duolin, as you have rightly surmised is used for babies with wheezing or fast breathing, which will be greatly helped by the salbutamol and ipratropium in Duolin. They must have used Duolin as she had a wheeze, which could be due to viral infection as such. If there is a stronjg family history of asthma, there is a chance that your baby is predisposed to develop it. If not, the wheezing and fast breathing is probably because of intercurrent viral infections. What can you do to decrease the frequency?1. Vaccinate her with influenza and pneumococcal vaccine2. Decrease triggers for her wheeze or fast breathing. Avoid the following: - agarbatti/incense -perfume -face powder -dusting or sweeping near the baby - pet hairs -cockroaches -soft toys 3. Wash her bedsheets weekly in hot water to get rid of dust mites which will cause frequent wheezing4. Clean AC filters once in 10 days5. Keep an allergy diary: each time your child develops fast breathing, note down all the foods she has eaten that day, any particular activity she has done. you can compare notes over a period of time and try to identify a common allergen Hope she feels better and this has helped you. Take care!"
},
{
"id": 1718,
"tgt": "Is weight gain,back pain,irregular periods signs for pregnancy?",
"src": "Patient: HI Doc, I am 25 and recently I have been gaining alot of weight. my eating plans are the same and sometimes its just 2 small meals a day. im always tired and sleepy, and have severe lower back pain thats on and off. my legs are sore and i have an irregular period. The past 3 months i bled 1 day only for a couple of hours. I thought that i was pregnant but the 3 home tests that I have done was negative. please help. thanks. Doctor: Hello,Thank you for asking your question in healthcare magicYour symptoms mentioned here are most suggestive of hypothyroidism (decrease in thyroid hormone levels)Weight gain despite decreased/same food intake, tiredness, increased sleepiness , muscle and joint pain,menstrual irregularities ,hair and skin thinning are some of the symptoms of hypothyroidism.Further history,physical examination,lab investigation (TSH, free t3. free t4) is also required to determine hypothyroidism and it's cause.You will also need a pregnancy test (urine beta HCG) done .Hypothyroidism is treated with thyroid hormone replacementI recommend you consult with your doctor to get checked and for further work up.Hope I was able to help you,Regards.Dr.Lekshmi"
},
{
"id": 189788,
"tgt": "Dizzy, fainting after taking anesthesia injections for filling and crowing teeth. Is it dangerous ?",
"src": "Patient: hello doctor , i m a 23 years old female . i am asking about dental local anesthesia i have to get all my teeth filled and crowned , i have been using anesthesia for 7 months nw with approximately 2 injections per week. at the beginning it was going fine but during the past three weeks i am feeling very dizzy and fainting all the time and this feeling increases on the day i visit my dentist . i would like to know if repetitive doses of anesthesia for a long period of time is dangerous and could cause me further problems . thank you Doctor: dear friends,thanks for sharing your concern,local anesthetic (LA) will temporarily interrupt conduction when absorbed into the nerve. a blockade of afferent nerve transmission produces anesthesia and lack of all sensations. blockade of those fibers transmitting pain , sensation only results in regional analgesia.the perception of heat cold and pressure are abolished by higher concentration of anesthetic agent then are required for interruption for pain.the local anesthetic produces loss of function in following order, pain , temperature, proprioception and skeletal muscle tone .the return sensation in the reverse order is expected.some times patient under the effect of anesthesia traumatize their tongue cheek or lips for this reason longer acting agents may be contra indicated.remember the local anesthetic solution are eventually absorbed in to the general circulation and can have toxic manifestation in overdosage.yes repipititive usage of local anaesthesia have too many side effects like syncope,muscle trismus,haematoma,and adverse effects on CNS(central nervous system),and CVS(Cardio vascular system.therefore i would suggest, minimal usage of anaesthesia is best,if required post treatment analgesics could be prescribed.please donot be apprehensive about dental treatment.if required you can take anti anxiety agents like alprazolam.hope this helps.thanks."
},
{
"id": 134970,
"tgt": "What causes uncontrollable shaking of the thumb nail?",
"src": "Patient: Hey doctor. Been having a weird thumb shaking happening in the past day. When I fully extend my left thumb it begins to shake almost uncontrollably. I have a cold and have been taking nyquil/dayquil. And also taking ginseng in the morning for energy during activities in the day. Very uncomfortable and is very annoying PLEASE HELP! Doctor: Hi Dear,Welcome to HCM.Understanding your concern. As per your query you have uncontrollable shaking of the thumb nail. Well there can be many reasons for symptoms you mention in query like low blood sugar , too much coffee and tea or essential tremor. The symptoms mentioned in query can also termed focal task-specific tremor or dystonia. In some cases performing some particular task or some particular trigger factory will repeatedly start the tremor/dystonia after a certain amount of time. The symptoms can also be first sign of Parkinson\u2019s disease or psychogenic tremor . I would suggest you to consult neurologist for proper examination . Doctor may prescribe atenolol , gabapentin , clonazepam or botox injections . In some cases doctor may recommend physical therapy or surgery . For now do stretching exercises under expert , keep your hand in hot water tub for 15 minutes and avoid alcohol and caffeinated product.Hope your concern has been resolved.Get Well Soon.Best Wishes,Dr. Harry Maheshwari"
},
{
"id": 134044,
"tgt": "What causes body pain with an injury?",
"src": "Patient: yes I just recently had an mri don due to job injury about a year ago,it came out normal but im still in pain everyday.My physician says he don t know what it could be,he ordred physical therapy again.im wondering if it has to do with some nerve damage? Doctor: hi,as mentioned by you the history of your illness, what I can understand is you have a general body pain. Now to understand your pain the MRI was suggested. Apparanlty, with the grace of God it came normal and it is a good news. Now what needs focus is to understand that due to workstation , there is something called as - Repetitive stress injury. This injury are divided into wide based classification. On a general note when you perform the office work while sitting or standing in a same position for longer hours there is a static contracction in the muscular system and leads to the metabolism of the muscles and early fatigue. which leads to the pain. To help this issue. you need to perform regular gents body exercises, breathing exercises and meditation. The exercises helps improve the cellular metabolism with the help of oxygen assisted by the respiratory exercises.Performing regular exercises will help you further and avoid pain and aches in future. I have personal experience of dealing with patients with similar complaints and with this protocol of exercises they are helped.RegardsJay Indravadan Patel"
},
{
"id": 148105,
"tgt": "What are the possible treatments for headache on forehead?",
"src": "Patient: My nose got hit two days ago. It caused some swelling, pain, headache and I got sinus drainage down the back of my throat right after. Now, two days after, swelling is gone, no bruises but I still have the bad headache in my forehead area and pain\\tenderness on the bridge of my nose. Should I be concerned about the continuing painful headache in my forehead? Doctor: Hi, I have read your query and according to me you should get CT head and start NSAIDS with antibiotics till result of CT scan.thanks for using HCM."
},
{
"id": 56841,
"tgt": "What are the possible treatment for hypopituitarism ?",
"src": "Patient: My mother is hypopituitarism patient . She is on wyzalone 5 mg for 7 years . Recently she had vomiting, pain in groin , lower back pain, fatigue fever 100-101, loss of appetite. Her liver - FNAC - positive for malignancy. ( possible neuroendocrine Tumor) . Her endoscopy shows GAVE of the Antrum, duedinitis. Her DOTA pet scan shoes poorly differentiated lesion in IV segment of liver. Can you guide us on wat this could mean. Will the treatment be chemotherapy or surgery . Will my mother improve with chemo. Doctor: Hi, How are you?My name is Dr Suresh Raghavaiah. i am a Liver and Pancreas specialist and I hope to answer your question today. I am sorry to hear about your mother's diagnosis. It is unusual for GAVE to occur by itself and is usually a symptom of Liver cirrhosis and portal hypertension or chronic Liver failure. But given your mother's history she does not seem to be suffering from either of these conditions. The treatment of GAVE is usually done by endoscopic coagulation with which excellent results have been obtained. it is very rare for it to need surgery to treat it.Given the lesion in the liver, she will need to undergo a colonoscopy as well to look for the primary lesion. If all the reports appear normal, and only the isolated segment 4 lesion of the liver is present, then it is very much necessary to get it surgically removed. Since it is a single lesion, the surgery is pretty straight forward. Please do let me know where you reside, so I can guide you to any available specialists in your area. Hope this helps and hope your mother starts to feel better.Dr Suresh Raghavaiah"
},
{
"id": 151909,
"tgt": "Will taking solian tablet bring me out of hallucination ?",
"src": "Patient: hello, i don t know how to ask.i have a problem with my brain from 4yrs ago,i had some halucinations and my doctor told me that is a psychotic episode,i am in deppresion fase 2,close to schizophrenia and gave me solian..i stoped this treatment from january cos i wanted to know how i feel and the trues is i am ok except that my brain is still vibes and is in pain most of the time.my question is ..can be posible to have a wrong diagnosis? cos i never heard about any depression or schizophrepia with pain and vibes and wich starts on 33yrs old.and i feel on my brain all my emotions also,in different locations.now i am 37 and i know that when i started to going crazy i was nervous,not depressive.excuse for my english and thank u. Doctor: Thanks for the query If your question is whther the initial diagnosis was wrong then i think it is highly unlikely because u did have hallucinations and it definitely improved after taking the medication and it looks like u r in relapse now after stopping them. It is a little risky to stop the medications on your own a= If your question is since you are 37 is there any other cause for your bodily pain then yes it is possible that there might be another cause for the pain. In either case please see your doctor and dont take independant decisions. Have a healthy living"
},
{
"id": 117617,
"tgt": "Please suggest remedy for hbsag antigen in the blood",
"src": "Patient: Hi, may I answer your health queries right now ? Please type your query here...im pregnent by 4 monts when i went for chek up HBsag reactive found by 13.53% plz suggest me wht shd be my precation for me and wht shd be do for my upcoming child plz help.... Doctor: Hi, you have hbsag positive but it does not mean that you are infected at present. You should go for complete liver function test aswell as hhepatitis b profile like hbe hbc etc. Then take treatment accordingly. Because many time hbsag is positive due to past infection or vaccination. and to preventthe spread hephepatitis b yo your child ypu should take all the precautions like go for vaccine and go for caserean etc . Consult your doctor and take treatment accordingly. Thanks for using health care magic."
},
{
"id": 154668,
"tgt": "What is the lifespan when having esophageal cancer?",
"src": "Patient: I ve been told by my oncologist last Thursday that there is no more that chemotherapy can do for my esophogeal cancer that has metastisised to my humourous, scapula and vertibrae. I would like some indication on what to expect from here and some sort of indication of how much time I have left before I am permanently hospitalised. My oncologists answer was do you have your affairs in order at home Doctor: Hi, dear I have gone through your question. I can understand your concern. You have esophageal cancer with multiple metastasis. It is in last stage. No surgery will help you. Treatment of choice remains chemotherapy. And despite of treatment prognosis remains poor. One should not expect much from here. Life expectancy is very low. Hope I have answered your question, if you have doubt then I will be happy to answer. Thanks for using health care magic. Wish you a very good health."
},
{
"id": 223820,
"tgt": "Is it common to have periods once after taking Plan B and again a week later ?",
"src": "Patient: I ended my period June 25. I had protected sex but found out the condom broke after he was finished so I took plan B the next morning. It s my first time taking plan B and experienced the side effects. Now, a week later I got my period again! What does this mean? Doctor: HiDr. Purushottam welcomes you to HCM virtual clinic!Thanks for consulting at my virtual clinic. I have carefully gone through your query, and I think I have understood your concern. I will try to address your medical concerns and would suggest you the best of the available treatment options.1)First of all do not panic.2)You have rightly taken Plan B. These emergency contraceptive pills are to be taken within 24 hours of unprotected sex act, or at least within 72 hours to be effective. 3) Withdrawal bleeding that occurs after PLan B is very erratic, It can be early or delayed , scanty or heavy. You may get some bleeding even before your expected date of periods and later on you may have your routine period also.I hope my answer helps you.Thanks.Wish you great health.Dr Purushottam S Neurgaonkar"
},
{
"id": 172111,
"tgt": "What is the dosage of ondem for vomiting to 4 year old?",
"src": "Patient: Hi my child is going to be 4 yrs in 2 months and for no apparent reason has thrown up thrice and complaining of stomach pain .. Gave him crocin but he threw that up .. Thinking of giving him ondem to stop the vomiting .. How much dosage should I give him ?? Thanks Doctor: Hi...the dose of Ondanisteron is not age dependent and it is weight dependent. It is 0.15mg.kg per dose and it is given 8th hourly once that too half an hour before the feeding.Regards - Dr. Sumanth"
},
{
"id": 140758,
"tgt": "Is lump behind the right ear related to seizure?",
"src": "Patient: my daughter, age 4, had a seizure a few months ago and after an eeg they found there was some abnormal activity on the right back part of her brain. this morning she woke up with a lump behind the right ear. could those 2 things be related? i m very worried about her. Doctor: Hello, The lump is not related to the seizure activity, at least not directly. Seizures originate in the brain which is confined inside the skull, the lumb on the outside of the skull is not in contact and can't irritate brain cortex to cause seizures. Such lumps are common in infections, such as ear or throat infections and usually subside by themselves (although it would be wise to refer it to her pediatrician as he might want to inspect it). Hope I have answered your query. Let me know if I can assist you further. Regards, Dr. Olsi Taka, Neurologist"
},
{
"id": 141943,
"tgt": "What is causing pain, dizziness and blackouts?",
"src": "Patient: hi i have this problem where i get really bad pain in my head i also get dizzy a black out then feel weak or don't feel my body at all then fall and i feel what i grabbed or hit. i dont always fall but i do get the pain dizziness and blackouts they have been getting worse and i dont know what causes it but i would like to know so any answers? Doctor: First of all, get your Blood Pressure measured as that can be caused for your blackouts. What's your Haemoglobin level? if all these are within normal range, get a ncct brain done on advise of your doctor. Get back with the report and i will be able to help you out."
},
{
"id": 82685,
"tgt": "What are the symptoms of lupus?",
"src": "Patient: i have been to several specialists and had dozens of tests run in the last year to no avail. i am looking for a fresh set of ears to hear my symptoms and see if the compilation of the problems i am having maybe spark an idea of what could be ailing me.. I have had possible diagnoses of everything from lupus, to lyme disease, to RA to celiac sprue.... all with no concrete diagnosis. I have even seen an infectious disease pathologist who took 13 viles of blood for testing that brought us back to square 1. needless to say, i am very frustrated and nearly want to go to medical school just to figure this all out... any takers? Doctor: you didnot write any symptoms, so i cannot suggest anything..but symptoms of lupus are - mouth ulcer, rashes over body, rash over face on sun, recurrent infection, hair fall, low blood count etc."
},
{
"id": 168073,
"tgt": "Suggest treatment for canker sore and swollen gum",
"src": "Patient: I m watching a baby 17months old that has canker sore, swollen bleeding gums. What can I do to make her feel better. Mom told me to but salt on a wet towel and rub it in her mouth but it look like that would hurt. Is there something else less painful I could do? Thanks Doctor: Hello,I can understand your concern. It seems that the oral cavity is not cleaned of the baby while bathing her. Before the teeth erupt into oral cavity, the oral cavity and gums should be cleaned gently with a clean cotton cloth. After the eruption of teeth, toothbrushing two times a day should be done and appointment should be made with a dentist every 6 months for examination of the oral cavity of the children.Right now, to make her feel better, you can apply local Benzocaine gel (Mucopain) over the sores and painful areas in the mouth. This gel can be applied 3-4 times in a day. In addition, for the sore bleeding gums, plaque attached to the teeth and making gums inflamed can be the reason. To resolve bleeding gums, an appointment with the dentist or pedodontist should be made for ultrasonic scaling of the child.I hope this information helps you. Thank you for choosing HealthcareMagic. Take care.Best,Dr. Viraj Shah"
},
{
"id": 65441,
"tgt": "What causes lumps under skin in the abdomen after losing weight?",
"src": "Patient: Hi, I am a 20 year old female who has been working out a lot lately and have thus lost some weight. I am starting to notice some lumps under the skin in my abdomen though. They all have an identical match on the other side and do not hurt. They are also all small. Should I be concerned? Doctor: hi dear thanks for query on HCMthe swelling are suggestive of multiple lipomas(fatty tissue aggregate). if they are painless you can observe(wait and watch).if the swelling increases in size or they become painful you can get it removed after consulting a surgeonthank you"
},
{
"id": 93996,
"tgt": "Abdominal pain like cramps. Bleeding in rectum during bowel. What can this be?",
"src": "Patient: Hi I am a 37 yr old woman. I started having abdominal pain yesterday, kind of like cramps but different. When I have a bowel movement, I have been bleeding from my rectum. My stomach and intestines keep hurting. This happens every so often and then it goes away after a while. I'm thinking of going to a gastroenterologist. What can this be? Doctor: Hi welcome to Health care magic forum. Thanks for calling H.C.M.F. You had abdominal pain, associated with blood motions repeating often, and there is relief of pain in between the passing of blood. It appears to be the amoebic dysentry, or irritable bowel syndrome but it must be a chronic happening since long time. Your choice of consulting a gastro enterologist is correct, with the treatment with anaerobic antibiotics along with other drugs you will be all right. Besides i advise you some diet recommendations as take bland foods, avoid taking spices, junk foods, and oily foods. Wishing for a quick recovery. Best regards."
},
{
"id": 177746,
"tgt": "Is Colimex advisable for gastric problem?",
"src": "Patient: My grand daughter is 4 and half months old.Dr. has mentioned Colimex (as general prescription at time of discharge from maternity hospital) for repeated crying,fitful sleep due probable gastric problem. I read on net that Colimex is meant for babies 6 months and older and Colimex Df is meant for younger babies. Should we use Colimex for her? Doctor: You can stop colimax..mostly it may be due to evening colic..u can just give proper burping to the baby...nothing else require.."
},
{
"id": 196329,
"tgt": "Why are my testicles overheating?",
"src": "Patient: I have a problem with my testicles overheating. When they are very hot it feels like I am going to pass out. I can only walk so long till they heat up and I have to get to a cool place or take a shower to cool them down. I just hate this, any advice you can pass along. Doctor: Dear userWe understand your concernsI went through your details. I suggest you to Visit an urologist: Get evaluated to find out if you have a varicocele or other physiological abnormality that contributes to over-heating your testicles. If you require more of my help in this aspect, please use this URL. http://goo.gl/aYW2pR. Make sure that you include every minute details possible. Hope this answers your query. Available for further clarifications.Good luck."
},
{
"id": 219969,
"tgt": "What causes heartburn & indigestion during pregnancy?",
"src": "Patient: Hello, I took a pregnancy test yesterday and it was positive. I have been experienceing regular heartburn and indigestion for 2 weeks now.Also 2 weeks ago for one whole day I was severly nausea and then haven t been since. This is my 5th pregnancy. I have had 2 miscarriages-one in which I miscarried after pregnancy symptoms disappearing. I have two children and with both of them I experienced severe morning sickness starting at week 2-3. I am worried that I may be trying to misscarry because I have no morning sickness. I have prescription of Prometrium left over from last child that I never threw away. My doctor had prescribe it to help me not miscarry and I had a healthy boy after having 2 miscarriages. Could I be misscarrying? Should I take the Prometrium to help prevent it or is it too late? Doctor: Please don't take any treatment on your own.Its not necessary to have morning sickness in every pregnancy.If u have any doubts get yourself examined by a gynaec"
},
{
"id": 1536,
"tgt": "Is it possible to get pregnant with non-penetrative sex?",
"src": "Patient: i and my partner had foreplay yesterday, after which he got an erection and his semen was spread on my body. however, i had my pants on. is there even a remote chance of pregnancy? also please tell me can i get pregnant by any chance if we had our pubic areas in contact bt with clothes on? if yes, please suggest a preventive measure. Please reply at the earliest. Doctor: Hi, if you had your clothes on, there is no chance of pregnancy. Semen should come in contact with your vagina to make you pregnant. So don't worry. RegardsDr khushboo"
},
{
"id": 35795,
"tgt": "Suggest treatment for pyoderma",
"src": "Patient: Am 34 yrs old... 5 years back I suffered from pyoderma.For that I take steroid to take control over it. First time I take it for 15days and again I continue it for 3 months.After that I got married while checking my sperm count it goes down to 18 to 1million. After 3years I got the treatment now its increases to 18million. sometimes it shows 75million. Nxt time it suddenly decreases to 16. what should I do? Doctor: Thanks for your query at HCM!I am Infectious Disease Specialist! I went through your query!The first-line treatment for disseminated or localized pyoderma gangrenosum is systemic treatment by corticosteroids and immunosuppressive agents like cyclosporine. Topical application of Clobetasol, Mupirocin, and Gentamicin alternated with Tacrolimus could be beneficial.Papules that begin as small \"spouts\" can be treated with Dakins Solution to prevent infection and entire wounds cluster also benefit from this disinfectant. Wet to dry applications of Dakins can defeat spread of interior infection. Heavy drainage can be offset with Coban dressings. Grafting is not recommended due to tissue necrosis.If ineffective, alternative therapeutic procedures include systemic treatment with corticosteroids and mycophenolate mofetil; mycophenolate mofetil and cyclosporine; tacrolimus; thalidomide; infliximab; or plasmapheresis.Happy to take more queries! You can also write a review for me. If you would like some more information, I will be happy to provide. You can take a follow-up query.Take care!Dr. Sheetal VermaInfectious Disease Specialist"
},
{
"id": 190145,
"tgt": "Had root canal treatment previously. Now, having dental pain. Solution?",
"src": "Patient: I have recieved a root canal almost 10 months ago, however my dental allowance was fully used up so I did not complete the job waiting to the dental allowance again. I am now experiencing lots of pain and do not know what to do not having any money to get help before my dental allowance on janurary 1st. what can i do now untill then to help the pain go away, I can not even eat with the pain Doctor: Hello Welcome to HCM I would suggest you to get your root canal completed as soon as possible otherwise it will become very difficult to save your teeth.It will lead to abscess formation & will take even more time. Mean while you can take some analgesic & antibiotic to escape from pain.You can also place clove oil dipped cotton pellet in your teeth for temporary relief. Take Care Regards Dr.Neha"
},
{
"id": 106294,
"tgt": "Which cream can i use for scabis ?",
"src": "Patient: i have skin allergy(scabis) from last 1 year and i shown to one doctor and he prescribed following Dermotraid cream but its not working ....help me Doctor: hello welcome to healthcare magic try ayurveda medines mahamanjishtha adi kadha 4tsf with equal water and ointments: gandhak malhar after bath and shat dhaut ghrta at night thanks regards dr. bhagwan"
},
{
"id": 9141,
"tgt": "i suffer from menstrual disorder",
"src": "Patient: i am a 19 year old girl. i suffer from menstrual disorder since menarche. i have been underweight since childhood, my weight is 42 kgs and my height is 164cms. i also have sinus. my concerns are my menstrual problem, weight problem, and sinus. my nose runs whenever i cry, which is very embarrassing. and my nose gets blocked too. i get a cold every month. i even suffer from constipation very often. my complexion turns brown usually although i am fair. please suggest yoga exercises for all these problems. Doctor: Saw your msg..First of all , check whether you are anemic. Also check up whether you have cyst in ovaries. You must avoid taking all junk foods,fried things,any type of fast food.Try to take breakfast without fail ,lunch on time and early dinner .Try to add lot of fibre and greens in your food.Eat fruits in between.Avoid refrigerated,reheated, or overnight stuff, spicy and masala food.Try to take head bath daily in normal water(not hot water).Take two tablespoons of broken dal(which is normally mixed with chutney) once in a week.This will clear your menstural problem.If followed all other instructions above,your nose block and constipation will also be relieved. Now contact any yoga master and try to do Pranayama ,breathing,Sarvangasan(shoulder stand),Sethubandhasan (Bridge),Headstand,a little bit of leg raising asanas under proper supervision will definitely cure your prblem permanently."
},
{
"id": 45701,
"tgt": "Are mild hydronephrosis and hydro-ureter in the left kidney a cause for concern?",
"src": "Patient: my x ray and sonography report says following. X-ray abdomen report --left lower ureteric cavity Sonography abdomen--left kidney shows mild hydronephrosis and hydroureter.Right kidney shows a calculus of 4 mm middle calyx with normal pelvicalyces. what is issue in left and right kidney?stone might be remove itself if consume more water? Doctor: Hi, In kidney shows hydronephrosis means due to stone in the ureter urine flow block lead to urine accumulated in the kidney calyx lead to swelling called hydronephrosis. Your report suggests that bilateral stones with left stone in the ureter and right stone in the right kidney with left hydronephrosis. So you may consult your doctor he will examine and treat you accordingly. Hope I have answered your query. Let me know if I can assist you further. Regards, Dr. Penchila Prasad Kandikattu, Internal Medicine Specialist"
},
{
"id": 61258,
"tgt": "Suggest remedy for lumps behind the ear with headache",
"src": "Patient: I have developed a knot (bump or lump on the back of my head behind my right ear. Have experienced some balance issues and the entire right side of my head feels like pins and needles (such as when one of your limbs has fallen asleep). Have also experienced some headache in my forehead. Doctor: Hello dear , HiWarm welcome to Healthcaremagic.comI have evaluated your query thoroughly .* Usually lump behind ear is post auricular lymph node , but as there is balance issue this does seem to be more serious than this as tumor or else .* Strongly recommended to get MRI done for final diagnosis and further management .Hope this will help you for sure .Wishing you fine recovery .Regards ."
},
{
"id": 60805,
"tgt": "What does a lump on the lateral gluteal region indicate?",
"src": "Patient: I think my question needs a surgeon .. Thank you in advance. Have a lump on my lateral left gluteal region , it started few months ago as a lemon size painless lump which I discovered accidentally then it grew to be as big as a big orange , sometimes its slightly painful specially when I climb stairs also I find difficulty in sleeping on the left side , I have a history of lipoma on my back and right arm , I am 62 hypertensive patient not diabetic Doctor: Hello and Welcome to \u2018Ask A Doctor\u2019 service. I have reviewed your query and here is my advice. * According to my long surgical experience, this lump on gluteal region also indicate gluteal lipoma most likely. * However, I would like to evaluate the issue with a photo picture of the lump for further management guidelines. Hope I have answered your query. Let me know if I can assist you further."
},
{
"id": 145527,
"tgt": "What causes dizziness with something moving feeling in head?",
"src": "Patient: HI there I have been experiencing a strange feeling in my head like dizziness and feels like something is moving in my head. I have pain in my temples especially the right one and my face feels sore as well - this has been for 2 months now-I do not experience this when I am asleep, but when I wake up it starts and continues until I go to sleep. I went for a ct scan, it is not my sinus. I also feel pressure on my ears. Doctor: Hello!Thank you for your question on HCM!Regarding your concern I think your troubles are very suggesting for tension type headache. It is a common headache caused by contraction of the scalp muscles. Your normal CT scan is another argument in favor of this diagnosis. If I were your doctor I would recommend treatment with an antidepressant like amytriptiline. You must take it under medical prescription and a cardiologist consult as it can cause heart problems. Other drugs I would recommend are paroxetine, citalopram. I recommend you to take one of these drugs for some months. It will be very helpful.Hope to have been helpful!Best wishes Dr Abaz Quka"
},
{
"id": 146945,
"tgt": "Had a coughing fit. Had massive lumps and bruises on forehead. Why is it swollen?",
"src": "Patient: My father had a fall yesterday afternoon. He said it wad from a coughing fit. I think it wad a seziure (from a stroke 5 years ago). He has a massive lump and bruise on his forehead and his eye is bruised and swollen. His left eye has trouble staying open. His forehead is massivley swollen. He wont go to the Dr, and says his ok. What should i do? Doctor: I think you should take him to a doctor definitely.There are many possible causes of his fainting. As you said, it may be a seizure. And in that situation, earlier he gets to a hospital, the better. Then we can do an eeg for him. If you delay it, eeg may be negative despite it being an actual seizure.There is also the possibility that this is a TIA, or a brief stroke. In that case his medicines may need to be readjusted.There are other cardiac conditions also that need to be confirmed such as Atrial FibrillationI definitely would think that he should be examined by a neurologist for sure."
},
{
"id": 60866,
"tgt": "What does a lump in the groin area indicate?",
"src": "Patient: In 2010 I found a small lump in the left side of my groin. My doctor ordered an ultra sound and it showed nothing so thought it may just be fat and was told there's nothing to worry about.Now over the past year it has grown and is unsightly and when felt it aches a bit. So its nearly 8 years later. Could this be a hernia (I've had constipation for years) or a nothing lump that became a something lump? Doctor: Hello,The possible lump in the groin can be a hernia or others as soft tissue lesion. Primary evaluation with clinical examination and latest ultrasound to check underlying defect is mandatory.Hope I have answered your query. Let me know if I can assist you further.Regards,Dr. Bhagyesh V. Patel"
},
{
"id": 197918,
"tgt": "Do male enhancements work?",
"src": "Patient: Hello,Just wondering if any of the male enhancement (non-perscription) actually work. (extenze, etc..) There are tons of \"reviews\" but they're all biased for their own company. What's the real deal. Do they work? Can they really increase your size? Can it be permanent? Where can I go to get real answers about what products are real?Thanks! Doctor: DearWe understand your concernsI went through your details. Male enhancements or supplements never work. They all are false claims. Male penis size improvement is not possible even by an inch. That is for sure. Nature has given its verdict and man made can seldom overcome it. Do not believe in these false promises and loose money over the product. These male enhancement product do not work at all.If you still need my assistance in this regard, please use this link. http://goo.gl/aYW2pR. Please remember to describe the whole problem with full detail.Hope this answers your query. Please feel free to post follow up queries. Available for further clarifications.Good luck. Take care."
},
{
"id": 126791,
"tgt": "How can swelling in the neck and a painful lump on the arm be treated?",
"src": "Patient: I fell two months ago on ice and landed on my shoulder. I developed swelling at the base of my neck looks like a lump and causes pain in my left arm and neck when I move in certain directions. I had a mri and cat scan both are negative for tumor and/or muscle ligament damage. Primary doctor says a fat deposit but that makes no sense to me. I can not close my collar on my shirt. Doctor: Hi, If CT scan and MRI studies are normal you need not to worry as both are very sensitive and specific for diagnosis of spine and muscle pathology. Swelling may be due to soft tissue oedema or hematoma at base of neck. Local application of ice packs and tablet Chymoral Forte thrice a day will help in reducing soft tissue swelling. Give rest to shoulder and neck for a week and swelling will reduce. Hope I have answered your query. Let me know if I can assist you further. Regards, Dr. Jayesh Vaza, Orthopaedic Surgeon"
},
{
"id": 136366,
"tgt": "How to treat C4-5 posterior disc complex and unconvertebral hypertrophy?",
"src": "Patient: would like an opinion 1) should I see an orthopedic or neurosurgeon 2) Have to climb stairs 20 x a day, make it worse? Will try to make this easy to readCervical.................C4-5 mild loss of the normal cervical lordosis with a mild focal kyphosis. C3-4 mild posterior disc bulge without evidence of significantspinal canal or neural forminal stenosis C4-5 posterior disc oesteophytic complex which result in mild spinal canal narrowing. Minimal bilateral neurall foraminal narrowing C5-6 small posterior disc complex and uncovertebral hypertrophy resulting in mild bilateral neural foraminal stenosis Minimal spinal canal narrowingLumbar...................Osteoporosis L3-4 diffuse bulge. worse on right questionable slight mass effect on right l4 nerve root L4-5diffuse disc bulge, small broad based central and right posterior paracentral disc herniation with extension to right neural foramen slight mass effectt on right L4-5 nerve roots is posible L5-S1 posterior annular tear and small broad based posterior central disc herniationALL of my pain is in my muscles,total bilateral down to feet including feet, walk flat footed.....Knuckles in hands enlarged mostly pinky and thumb knuckles again bilateral.Muscles in neck stiff 24/7 Can all of this derive from the spinal situation.....or a muscle disorder. dr did blood test for MS, came out positive. He said only way to tell for sure is a brain cat scan looking for lesions. 35 yrs ago diagnosied with hydrocephlaus with lesions and report had the word blockage. At loma linda and night before surgrey was cancel.I am 68 so really dont want surgery. lumbar is 24/7 cervical when pain occurs is on a scale of 9.9 Would like your opinion and I know its hard without seeing me. Thankyou Also numbness, tingling twitching burning charly horse, Numbness entails both legs and fingers on both hands. sorry this is so lengthy C4-5Posterior disc complex and uncovertebral hypertrophyresulting in mild bilateral neural forminal stenosis. mimimal spinal canal narrowingLumbar............... Doctor: HelloFindings suggests multi-level degenerative changes in cervical spine.Degenerative changes are generally age related findings.It is due to chronic stress on annulus fibrosis.At C5-6 level,there is disc complex with hypertrophy of uncus which is causing mild narrowing of neural formamen in both sides.Minimal narrowing noted in central spinal canal.In lumbar vertebra there are more degenerative changes.There are diffuse disc bulge at multiple levels .Majority of degenerative disc disease patients are benefited by conservative treatment in the form of physiotherapy,analgesics and muscle relaxants.Surgery is advised in non responding cases.Physiotherapy helps in strengthening of paraspinal muscles and it prevents further changes.Since you have mild findings,most likely you will need physiotherapy only.Get well soon.Take CareDr.Indu Bhushan"
},
{
"id": 33917,
"tgt": "How to confirm if cured from HCV?",
"src": "Patient: MY WIFE HAVE GOT HCV RNA QUANTITATIVE VIRAL LOAD BY TAQ MAN <15 IU/ML AFTER 24 WEEKS OF TREATMENT BY PEGINTERFERON AND RIVBARIN FOR GENOTYPE 3 HEPATITIS C VIRUS. KINDLY TELL ME THIS <15 IU/ML MEANS NEGATIVE HCV OR STILL SHE IS INFECTED WITH HCV VIRUS. PLZ TELL ME THAT TREATMENT WAS SUCCESSFUL OR NOT AND SHE WILL HAV ANOTHER PROBLEM IN FUTURE BECAUSE OF THIS? Doctor: Hi! I am Dr. Saddiq and i would like to look into your concerns. I would briefly describe the course of actions from this point onwards.1. Yours wife's PCR report is suggestive as if it was a qualitative PCR analysis and not that quantitative one, because for qualitative ones they have a certain cut off value and its testing limitations down to a certain viral load, which in this case was less than 15 IU/ml. That tells us that her viral load was so less (or either negative) to be detected in the test. in any case, briefly speaking she has responded well to treatment which is a good news.2. The next step would be having a quantitative PCR every six months for one year, and then yearly for three years to see for a sustained virological response (SVR). If it stayed negative, no action is suggested.3. If it becomes positive, upto a certain viral load, than that would be called as treatment relapse, and in such case either repetition of whole peg interferon therapy for another six months is recommended or use of new antiviral drugs, which were in pipeline previously and now been recommended by different drug authorities ti b used in certain countries. The one in use for genotype 3 is Tab.Sibasfovir. If i were your wives physician, then God forbid if in the case of a relapse, I would have opted for Sibasfovir instead of a second course of interferon as it has been shown to have more effectiveness then the previous one. It is recommended as 400mg tab. once daily for six months, alongwith the usual dose of Ribavirin."
},
{
"id": 113456,
"tgt": "Having low Vitamin D3 and back pain. How to get relief?",
"src": "Patient: I ham having low Vit. D3 ( 5.5%) and back pain since last 10 days....took a week off to rest and got li l better but did some stretching and it has worsened 20% again....now it is no longer at a point ( hip) but travels down the thigh also ( leg feels slightly heavy and numb and pain is there) Kindly advice what to do ( Took Myoril, Movon ,Nerubion etc for 10 days..) Doctor: Hi jayant ! It feel like sciatica pain. Do search this on net. If you moving and working whole day. Take rest !. About d3 loss .consult with M.D. Take care bye."
},
{
"id": 27613,
"tgt": "How to treat 110 heart rate with cough and fever?",
"src": "Patient: My heart rate has been at 110 for the last 2days. I have a cough and a low fever. Would this usually make your heart rate go up. I had also been tested for svt recently because I have palpitations and a shots run of svt... Is it just the cold making my heart race? Doctor: The reason for your heart rate shooting up to 110 in the last two days is infection resulting into fever. It is natural for the heart to increase its heart rate to meet increased oxygen demands of the body due to fever. So as soon as the fever and infection settles the rate will come to normal. In svt the heart rate is generally more than 150. However if the rate remain high even after the fever subsides do get an ekg"
},
{
"id": 225236,
"tgt": "What could be the brownish skin like pieces coming out while on depo shot?",
"src": "Patient: So a week or two ago I had sex with my boyfriend and he came inside me, but I am on the depo shot. Anyway I have had normal spotting which happens with my birth control, but today it was like...a weird brownish color, and sometimes there is like white...skin looking peices. WHat is going on? Doctor: Hello. Thanks for writing to us. The brownish skin like pieces coming with the brown discharge are likely to be the endometrial layer of the uterus that is shed during every menstruation while bleeding. This is not worrisome.I hope this information has been both informative and helpful for you. Regards, Dr. Rakhi Tayal ,drrakhitayal@gmail.com"
},
{
"id": 114034,
"tgt": "I am suffering from back pain because of gas. How to manage?",
"src": "Patient: I have been lately getting stabbing pain in the upper back because of gas . I know it is gas since it eventually moves forward after nearly 24hrs and hurts my stomach. Is there a way to deal with this? I took several tests including blood , ct scan , etc but there were no red flags. Not sure why it has started to hurt my back. Hurting my stomah is manageable but not back. Please advice. Doctor: Hi, Welcome to HCM. Read your problem. I f all reports are normal then it should be gastritis, you can take a proton pump inhibitor for at-least 5 days. Avoid oily,spicy and fried food. Have lot of water and add salads to your meals. Take care."
},
{
"id": 159389,
"tgt": "Acidity, constipation, bladder pain. History of bladder cancer. Treatment for symptoms?",
"src": "Patient: HELLO DOCTOR THIS IS XXXXXX. I HAVE BEEN DIAGNOSED WITH BLADDER CANCER IN 2010 AND MY TREATMENT HAS BEEN FINISHED NOW BUT THERE ARE CERTAIN PROBLEMS LIKE ACIDITY, CONSTIPATION , WHICH MAKE MY URINE BLADDER FULL OF PAIN SOMETIMES, SO IS THERE ANY MEDICINES WHICH MAKES MY URINE BLADDER RELIEF....PLEASE SUGGEST IT...I WILL WAIT FOR YOUR RESPONSE REGARDS Doctor: Hi thanks for your question. You have have been treated for bladder cancer.The symptoms you are having all could be part of your bladder cancer treatment . Most probably you have recived chemo and radiotherapy after surgery for treatment of bladder cance.Radiotherpy causes what is called as radition cystitis ,which gives bladder pain and other symptoms.Acidity could be due to analgesic (pain killer) which you have taken for pain.I would suggest you to consult your treating surgeon ,who may examine to rule out complications of chem radiotherapy and may prescribe you appropriate treatment Hope this answers your question."
},
{
"id": 23260,
"tgt": "Suggest treatment for inducible ischaemia",
"src": "Patient: my tmt test shows resting ecg normal.Functional Capacity normal.HR response to Excersice appropriate.BP response to excersise normal.Resting bp normal.Chest pain none.Arrrhymias none.ST changes Depression upsloping on peak exe became down sloping in late recovery.TEST POSITIVE FOR INDUCIBLE ISCHAEMIA.can u please sujust the best couse Doctor: DEAR USER,THANKS FOR CONSULTING WITH HCMI UNDERSTAND YOUR CONCERNYOU REPORTS POINT TOWARDS A STABLE ANGINA THAT CAN BE INDUCED BY EXERCISEI SUGGEST YOU TO CONSULT A CARDIOLOGIST SO THAT HE MIGHT DO A PROPER EVALUATIONHE MIGHT ADVISE YOU A PER CUTANEOUS ANGIOGRAPHY IF REQUIRED AND MIGHT START ON DRUGS LIKE ASPIRIN, CLOPED OR MAY ADVISE ANGIOPLASTY AS PER THE DIAGNOSISHOPE I ANSWERED YOUR QUERY. YOU CAN MESSAGE ME FOR ANY FURTHER CONCERNS"
},
{
"id": 136924,
"tgt": "Suggest treatment for severe foot pain",
"src": "Patient: I have had severe foot pain for over a year .it is mostly in the heel it has now started in the other foot. I have tried massage .cortisone shot ,ice,heat, over the counter pain relievers, ultrasound therapy , inserts and xray sometimes I can barely walk and prolonged standing and walking are most days unbearable. I'm a 44 year old female with no major medical problems no recent injurys to feet and awful insurance any suggestions to a diagnosis or relief? Doctor: Hello, I have studied your case. As per your history I will like to give differential.There is possibility of early plantar fasciitis, or can early arthritis.For plantar fasciitis,Shoe/footwear should be with soft sole. You can use silicon pad insoles.Avoid exposure of cold to foot.Start plantar exercises consulting physiotherapist also Physiotherapy like TENS and ultrasound will help. Take calcium supplement with vit D.Do exercises of foot before getting out of bed.As per your history another possibility of inflammatory arthritis.You may need to do blood test to confirm diagnosis.When such patients come to our hospital I advise them blood test like RA factor, ACCP, ESR, complete blood count, HLA B27, thyroid profile. For these symptoms medication like methylcobalamine and analgesic will reduce pain; you can take them consulting your treating doctor.If not relieved then you may need arthroscopic release of fascia.Hope this answers your query. If you have additional questions or follow up queries then please do not hesitate in writing to us. I will be happy to answer your queries. Wishing you good health.Take care."
},
{
"id": 173048,
"tgt": "Suggest treatment for loose motion and buttock rash",
"src": "Patient: Hi Doctor, My baby daughter is four month old. Due to loose motion she is suffering from severe rashes in her buttock part. We are using Himalaya diapper rashes cream & coconut oil, but we didn't get any good result. She doesn't allow us to touch the area . Please advise... Regards, Sonia Doctor: I understand your concern. I would first of all stop bottle feed if he is on bottle and give a probiotic powder and racecadotril suspension of powder available in a sachet. hen I would advice to give only breast milk and to keep the area dry and cover with cotton cloth after applying an antibiotic cream.Diaper rash can be traced to a number of sources, including:Irritation from stool and urine. Prolonged exposure to urine or stool can irritate a baby's sensitive skin. ...Chafing or rubbing. ...Irritation from a new product. ...Bacterial or yeast (fungal) infection. ...Introduction of new foods. ...Sensitive skin. ...Use of antibiotics.Take care and come back if you have any more doubts.Regards"
},
{
"id": 52687,
"tgt": "How to treat excessive fatigue while having enlarged liver and hiatus hernia?",
"src": "Patient: Hi, I have gone to the doctor and have been told that I have an enlarged liver. It is palpable. I have extreme fatigue, bloating, and foamy urine. I am a perimenopausal 54 year old woman. I have a healthy diet, am 5 8 and 147 lbs. I am a moderate drinker but never to excess. Never smoked, don t do drugs. My enlarged liver presses on my stomach and makes it difficult to,eat much at a time and causes discomfort after eating.I am so,tired of feeling bad. I have had an endoscopy, a colonoscopy and a CT scan. They found,a small heital hurnea and the enlarged liver. Doctor: Hello and Welcome to \u2018Ask A Doctor\u2019 service.I have reviewed your query and here is my advice.Usually, liver enlargement may indicate some types of hepatitis or fatty liver diseases, and in most cases, it doesn't require any specific treatment.If this is caused by diet, obesity and diabetes then dietary and lifestyle changes will be beneficial in every case. Alcohol and viral hepatitis are most common causes, and in this case, medical evaluation is required.Also, if there is infectious disease or biliary duct disorders then you need to do some tests to rule out more serious issues which require specific treatment.You need to check your viral markers and do liver biopsy if necessary.Hope I have answered your query. Let me know if I can assist you further.Regards,Dr. Ivan R. Rommstein"
},
{
"id": 97766,
"tgt": "Will alternative medicine work for pleomorphic adenoma?",
"src": "Patient: Dear DoctorPlease advice treatment whether alternative medicine will work for the below mention -CYTOLOGY.FNAC ORAL CAVITY SWELLING. SMEARS SHOW CLUSTERS OF BASALOID CELLS, PLASMACYTOID CELLS AND SPINDLE CELLS SINGLY SCATTERED AND IN DUCTULAR ARRANGEMENT. MYXOID STROMAL FRAGMENTS ARE SEEN.MACROPHAGES ARE SEEN IN BACKGROUND.DIAGNOSIS FEATURES SUGGEST - PLEOMORPHIC ADENOMA.COMMENTS ADVISED- EXCISION BIOPSY. Doctor: Hi...i dont think any alternative medicine will work...its better you get the tumor removed before its grows large to cause more problems..Dr. Ashish Verma"
},
{
"id": 45692,
"tgt": "What causes pain and discomfort in the lower abdominal area?",
"src": "Patient: I have a on going pain and discomfort on the left side low by colon kidney. At first it felt like a muscle pull. Its radiating to my side and towards by back. It feels like it is hot I am going in for an ultrasound next week but I wonder what it could be. My blood tests reveal elevated kidneys and elevated glucose. Doctor: Hello, This can be due to problem in your kidney like stones or cysts. Pain from the kidney stones can refer or radiate to back side or to thigh and groin region.Your tests show abnormality and high glucose level in blood can definitely be due to some disorder there. Let me tell you whether you have any other history like diabetes Diabetes mellitus, gall stones etc. You may need an ultrasound.You are therefore suggested to see a physician and get properly examined so that proper treatment can be initiated. Hope I have answered your query. Let me know if I can assist you further. Regards, Dr. Soheel Hussain Zargar, Dentist"
},
{
"id": 22984,
"tgt": "What is the treatment for high blood pressure and pain in the rib cage?",
"src": "Patient: my husband went to emergancy room about week ago due to his blood pressure being 162/118-they hooked up an ekg to him while he was still hooked up they took some chest xrays-he said after they where done he felt a sharp pain by his rib cage right after the xrays-my question is it possible to get electrical shock from the xray and ekg wires Doctor: No mam, x-ray and ecg cannot cause such kind of pain. Reason has to be something else.For blood pressure he is track his blood pressure regularly and consume low salt diet. Also take a medicine regularly. Healthy lifestyle and regular exercises will help"
},
{
"id": 115299,
"tgt": "Does a slightly elevated d-dimer mean a blood clot?",
"src": "Patient: i went to the hospital 3 weeks ago and my d-dimer test was slightly elevated...they did a ct with contrast and it was normal...i was having a burning back pain as a symptom on my left side that would radiate to my rib cage....i notice this pain sometimes after i eat....does a slightly elevated d-dimer mean a blood clot regardless of what the ct shows.. Doctor: I dear. I have gone through your question. I can understand your concern. Slightly elevated d dimer raise the possibility of disseminated intravascular coagulation. You should go for complete coagulation profile including FDP. This will give you exact diagnosis. Then you should take treatment accordingly. Hope I have answered your question, if you have doubt then I will be happy to answer. Thanks for using health care magic. Wish you a very good health."
},
{
"id": 139400,
"tgt": "Can coccyx injury cause numbness and pain in the body?",
"src": "Patient: I fell on my coccyx 2 weeks ago and after seeing to Doctors have been advised it is badly bruised. I have been taking strong pain medication but i am now experiencing pins and needles/numbness to the left side of my body. Would this be related? What treatments would you suggest to help? Doctor: Hello, Your current symptoms are not related to coccyx injury. You need detailed neurological examination followed by MRI of brain and spine. In the interim, you can take pregabalin capsules for relief of symptoms. Hope I have answered your query. Let me know if I can assist you further. Take care Regards, Dr. Sudhir Kumar"
},
{
"id": 21998,
"tgt": "What is the treatment for blocked artery?",
"src": "Patient: Hi, may I answer your health queries right now ? Please type your query here...hi i just had angioplasty but i was told they couldnt open the blocked arterie because it was very narrow i was told i have got a heart disease there is nothing to be done but keep on taking my medication i am very depresed since comining out of hospital what am i suppose to do now Doctor: Hi thereyes this happens in some cases in which the lesion inside the vessel is tight and the guide wire can't be crossed. Don't be depressed be positive and continue ur meds regularly.Good Luck"
},
{
"id": 15834,
"tgt": "Mole, skin bumps, blemishes. Anything to worry?",
"src": "Patient: I have one dime sized, smooth feeling, dark brown mole or blemish on my chest that the skin rubs off of every few months. Now I have another one like it on my back and a few smaller sized, lighter colored rough skin bumps on my legs and arms that the skin also rubs off of every few weeks. IAre these dangerous type skin blemishs? Doctor: HEllo , Itchy skin, or pruritus, is the result of an irritating stimulus on the skin. It is most commonly caused by dry air, skin irritants, or side effects from some medications. It can also be caused by allergies or bug bites. Lotion usually helps most dry skin, although there are also natural remedies that may be better for people with sensitive skin.There are several options for natural remedies for itchy skin. Natural astringents are a great place to start, as it will both sooth and moisturize the skin as it works at removing the stimulus making the skin itch. Witch hazel and St. John\u2019s Wort both work well. Lemon juice may also be used.Aloe vera, a naturally occurring gel from aloe plants, is one of the best remedies for itchy skin. It contains vitamin E, which helps to heal the skin. Aloe has natural soothing qualities to stop the itch and reduce inflammation caused by scratching. It also contains anti-fungal and antibiotic agents to help destroy certain causes of itchy skin. For the best results, individuals should use aloe fresh from the plant by just trimming off a part of a leaf and applying it to the affected area. Regards, Dr.Sharmila"
},
{
"id": 165141,
"tgt": "What causes squeezing pain in heart?",
"src": "Patient: 11 year old girl complaining of squeezing pain in her heart on and off for the past 5 days. Doctor seen her on day 2 - examined her and she had an ECG but could find nothing. Previously had heart scan few months ago as complaining of thumping sensation in her heart. These episodes had been going on a couple of years but they would only last a minute or so and there would be no pain. Now this squeezing sensation has caused her to cry because of the pain. Should I be happy with the family doctor s diagnosis or should I seek further help? Doctor: you must do echocardiogram as ecg is not diagnostic test for and may do cbc for anemia until that keep eating soft food and rest"
},
{
"id": 111151,
"tgt": "Suggest medication for bulging disc and degenerative disc disease?",
"src": "Patient: back question hello my my,name is Audrianna in 21 and 145lbs and im 5'4. I have serious pain in my back I've been through physical therapy and tried medication like gabatin or mobic which hasn't helped. Sometimes the pain shoots down my legs and hurts in my hips as well it hurts to bend down to pick stuff up and I cannot sleep at night. I have been back to the doctor and they can't seem to do anything. I've had mri and other imaging done they told me I had bulging disks and degenerative disk disease and my sciatica nerve \"acts up\" if you would call it that. I've asked for some other type if pain medication but they don't wantbto give me narcotics because of my age but my point is if it will help me and give me some relief then why not? I need some kind of help I'm in pain everyday and im tired of hurting I need advice on what I should do... Doctor: Hi,At this age of 21 years degenerative and bulging of discs are not common.So there might be having some another reason for this problem.Lack of physical exercise and sedentary life style can give rise to this problem.Go for back extension exercise daily.Take calcium, vitamin A and D supplements.If require you may take NSAID medciine as and when required.Ok and take care."
},
{
"id": 155196,
"tgt": "Suggest treatment for ovarian cancer",
"src": "Patient: Yes, Thank You. I was diagnosis with ovanian cancer. in 2012 and was cancer free for all this time. Now I find out that it has returned as many tumors. It is a rare type called an yolk sac tumor even rarer because I am 62 years old. Can you explain this to me? Doctor: You are right. Yolk sac tumor tumor at your age is quite rare as it mainly affects young women. So first thing you should do is to make sure you have the right diagnosis. If you are showing at a standard cancer center then its alright, else take a second opinion from one. If it indeed is yolk sac tumor and it has come back, then the best option for you will be to have chemotherapy and then if there is any residual disease after that then to undergo surgery."
},
{
"id": 108842,
"tgt": "Suggest treatment for pain in neck, shoulders and back",
"src": "Patient: hello doctor..... I alwayz have pain in my neck, shoulders and back after i do some work or do exersion.... And pain is always starting from evening..... But after i apply some balm or iodex... And do some massage.... In morning i feel fresh.... But the pain is very severe at night.... Please help... Doctor: You have to take some rest for some days... And do relaxing exercise... Also take tab. Myospas twice a day ND tab. Nervemax Sr before sleep at night.."
},
{
"id": 154134,
"tgt": "What are the symptoms of leiomyosarcoma?",
"src": "Patient: I have a history of uterine fibroids. I underwent a myomectomy last 2008 but they did not remove the small fibroids since they were too small at the time. They have since grown and are not really round in shape. I am worried about leiomyosarcoma. What are the symptoms for this type of cancer? Doctor: Hi,Thanks for writing in.Leiomyosarcoma is a malignant form of leiomyoma or fibroid. It is said that 1 percent of lesions appearing to be leiomyoma might be a leiomyosarcoma. It is cancerous and therefore shows aggressive behavior and appearance with irregular margins and infiltration in to the surrounding areas.In most patients a leiomyocarcoma can be told apart from a leiomyoma only by pathological test. Please get a transvaginal ultrasound scan done to know the exact number and nature of the lesions and if they infiltrate in to the surrounding myometrium. If your symptoms like pain and menorrhagia are severe and if you do not wish to have children later then getting a hysterectomy is the definitive solution. Please do not worry."
},
{
"id": 49829,
"tgt": "Child having blood in urine. Ultrasound of kidneys and bladder. Consult ER?",
"src": "Patient: My 12 year old son has occasionally had blood in his urine. Not much, and probably wouldn't have notices if he hadn't \"dribbled\" a bit on the side of the toilet bowl, as it wasn't enough to color the water, just the drops. This happened 3 times during the last week. We took him to the doctor and they sent him for tests today (ultrasound of kidneys and bladder) as well a full blood work-up. We will get results in a few days. Tonight when he went to the bathroom, there were actual drops of blood that came out after he urinated. They were much heavier, obviously, and dropped to the bottom of the bowl. Should we wait until tomorrow to follow-up with his doctor, or head to the ER? Doctor: HelloThanks for writing to HCMIf bleeding is not profuse then you can wait for reports.There are many reasons for hematuria(blood in urine).Causes may be renal or urinary bladder calculus,urinary tract infections,tumour,trauma,familial causes etc.Treatment depends on investigation reports and clinical findings.Get well soon.Take CareDr.Indu Bhushan"
},
{
"id": 220320,
"tgt": "What is the best treatment to get pregnant?",
"src": "Patient: My fiance and I are trying to get pregnant. We have been trying 2 months now and have taken 6 tests. the first 2 were false 3rd was invalid 4th and 5th were faint positive and the last one, which was another brand. came back false. I have had irregular periods my whole life the last one I can remember was about 2 years ago, and even then they were very light. about a month and a half ago I saw light spotting. I do have some symptoms along with a gut feeling. Symptoms are as follows: Largening, firmness and itchiness of the breasts, dizziness, occassional nausea. Could I just have a low hCG level? If so will that harm the baby? how can we get a for sure answer? Doctor: You could get blood test done 2 confirm pregnancy.its is more sensitive or you could just wait for a few more days and do a repeat test.sometimes variations may be there in test results due 2 faulty strips"
},
{
"id": 78902,
"tgt": "What causes breathing difficulty with lower abdominal pain?",
"src": "Patient: Hi Doctor,I am 29 years old and am into my 32 week of pregnancy.I feel difficult to breathe and has light pain in the lower abdomen. I am also having lower back pain, once I sit down on the bed or sofa, and when I move or try to get up,my lower back pains severely. I also feel baby movements continously without any break, and also have contractions.please help me Doctor: Thanks for your question on Health Care Magic. I can understand your situation and problem. Some degree of breathlessness is common in later stage of pregnancy. It is due to pressure effect of gravid (enlarged) uterus on lungs and diaphragm. Another causes are bronchitis and anemia. So better to consult doctor and get done clinical examination of respiratory system and hemoglobin estimation. Lower abdominal and back pain can be due to urine infection. So get done Urinary microscopic examination. Better to first diagnose yourself and then start appropriate treatment. Don't worry, you will be alright. Hope I have solved your query. Wish you good health. Thanks."
},
{
"id": 51526,
"tgt": "How can we treat tetania due to renal failure caused by number of chemotherapies earlier ?",
"src": "Patient: hello, i am from Greece. My daughter had cancer in the liver in the age of 8 years old. She is now 27 and healthy. But due to the great number of chemotherapies, the only problem she has is that the kidneys maybe doesnt work properly and she loses potassium and magnesium . So very often she is in the hospital with tetania. What can we do? Doctor: Hi, Thanks for writing in. Apart from supplementing potassium and magnesium, I am afraid we cant do very much for your child. I would suggest that you give her plenty of fresh juices, fresh fruits and vegetables and Pedialyte to supplement the electrolytes. She should be better in a few days. However, this needs to be done only after consulting your physician. Hope this helps. Regards"
},
{
"id": 103026,
"tgt": "Having cough and phlegm. Feeling short of breath. Have allergies. Use inhaler. Should I see a doctor?",
"src": "Patient: I was sick 2 weeks ago. There was no fever but a severe headache and vomiting. After all the symptoms went away I was left with a cough that feels like I need to clear phlegm from my throat. The phlegm is clear. It's been more than a week and it's not going away. I have noticed that it gets better for 1-2 days and then gets worse again. Sometimes if I am talking I get short of breath. I do also have allergies and I use an inhaler when it gets bad. How do I know if this is just an allergy or mild cold that will clear or it's something more serious, and should I see a doctor even if it seems to be improving? Doctor: Hello Thanks for posting to HCm Do your blood testcomplet blood count ESR X-Ray chest Take medicine REgards"
},
{
"id": 174280,
"tgt": "Any suggestion for kids suffering with dysentery after drinking nan pro 1?",
"src": "Patient: -hello doc gud morning i am having twins borned at 2nd nov 12; due to sm complecation their mother was at hosptal so could nt feed them well, yeterday when i went to pedi. He suggested me nestle nan pro 1; when i started fedding kids they started to hav disentry what should i do in that case? Doctor: Hi,Thank you for asking question on health care magic.Feeding with non pro-1 will not cause loose motions/dysentery.Dysentery is caused by bacterial infection.when a new food is offered, usually some alteration of bowel habit will occur.Get the stool examined and give treatment as per doctor's suggestion.Sporolac sachets with OZ syrup may help.Hope this answer will serve your purposePlease feel free to ask any more queries if requiredTake careDr.M.V.Subrahmanyam MD;DCHAssociate professor of pediatrics"
},
{
"id": 45961,
"tgt": "Where do they inject for stent removal between the kidney and bladder?",
"src": "Patient: HELP! If it is local, where do they stick the needle??? How long does it take and how will I feel afterwards? I intend having nothing to eat and drink from the evening before because pain makes me sick. I had the stent placed last week after removal of a kidney stone -the stent is between my kidney and bladder. I was offered to have it removed under general but there is a waiting list and the stent is painful and uncomfortable. I just hope this procedure is quick Doctor: Hello and Welcome to \u2018Ask A Doctor\u2019 service. I have reviewed your query and here is my advice. Double j stents are placed between kidney and urinary bladder in the ureter. They can be removed easily in local anesthesia the most common way is to just insert lignocain gel in the urethra and wait for 5 to 10 mins then they will pass cystoscope through your urethra and localize the lower end of the stent and pull it out of your urethra it is just a 15 min procedure and you will not be feeling any sort of pain after local anesthesia. Hope I have answered your query. Let me know if I can assist you further."
},
{
"id": 192631,
"tgt": "Is my semen report with total count 120 million /ml normal?",
"src": "Patient: sir i have semen analysis test sir i want to know that my result is normal or not method obtained masturbation volume 2.8 m/l viscosity thick total count 120 million /ml active 90% deAD 05% SLUGGISH 05% RED CELL NIL PUS CELL 03...04 SIR I WANT TO KNOW IS IT OK OR NY PROBLEM Doctor: Hello, I have gone through your samen analysis report. The given report is perfectly normal and nothing abnormal there to worry. The sperm count and motility(ability to move and to fuse with ovum) is normal. As per this report your fertility will not be affected. Hope I have answered your query. Let me know if I can assist you further. Take care Regards, Dr. Shinas Hussain, General & Family physician"
},
{
"id": 5545,
"tgt": "Trying to get pregnant. Having period pain, but its postponded. Had a miscarriage. Why is the pain?",
"src": "Patient: hi.. im yurekha from chennai.. I would like to conceive but this time also im getting my period pain before an week.. Its like i would get periods anytime.. but its postponding.. my actual period date is on march 11th.. usually i use to get my periods 2 to 6 days b4. sometimes 8 to 10 days before.. i had my miss carr last december.. pls let me about the mild period pain im getting now.. Doctor: Hi Ms. Yurekha, If you have had regular cycles prior to this, you should get pregnancy excluded first. You should get a trans-vaginal sonogram and see a specialist for a proper appraisal of your situation. It is important that the cause of your previous abortion is known as it could have an influence on your future conception. The cause of your pain could be anything from a premenstrual cramp to the pain of early pregnancy. Only an examination could identify that. Wish you good health."
},
{
"id": 16211,
"tgt": "Rash on collar bone, oozes clear liquid, itchy. Peroxide & bacitracin don't help. What could it be ?",
"src": "Patient: I woke up yesturday & had something that kinda appears to be a rash on my collar bone. After looking at it closer today, it appears to have a center pin whole & when squeezed, clear liquid comes out. There are about 20 bumps, which most of them seem to be in a row. It is very itchy & when touched, it feels like glass is poking into my skin. I woke up 6 days ago & had the same issue but it was covering my whole collar bone & chest & I used peroxide & bacitracin & went to bed & when I woke up it was gone. 4 days after that now, it has happened again but peroxide & bacitracin are not helping. I ve never had this before but it hurts. What could it be? My whole collarbone is red, swollen & you can see the bumps. Doctor: SEEMS TO BE HERPES THE Y ARE VERY PAINFUL AND TAKE 3- 4 WEEK TO GO MEDICINES DONOT EFFECT MUCH CAN USE DOXITAB 100 MGM TWICE A DAY AN USE TRAMADOL FOR PAIN APPLY GENTION VIOLET 2% OVER THE SPOTS AND APPLY NEOSPORIN POWDER OVER IT 3 TIMES LEAVE OPEN WAIT FOR 3 WEEK PAIN MAY STILL TAKE MORE THAN 3 WEEK TO GO"
},
{
"id": 62284,
"tgt": "What causes painful lump above eyebrow?",
"src": "Patient: I woke to find a lump has appeared just above my right eyebrow not that big but abit red and hurts when touched yesterday I had really bad back ache and some chest pain 2 weeks ago I had an operation to have my gallbladder removed was wondering if it could be a blood clot Doctor: Hi,Dear thanks for the query to HCM virtual clinic.I studied your query in full details updated from you.I understood your health concerns.Based on your query data, In my opinion ,Dear You seem to suffer from periorbital painful infected clot developing in to abscess/ or it could be a boil in hairfollicle with weak defences due to postoperative stress.Treatment-Would be by Surgeon who would fix the cause from above possibilities and would get USG / CT to understand the nature of the painful lump above eyebrow right side.Moslty aBoil seems to be cause in your case.Treatment would be by medicines and if not by surgical drainage.Possibility of this lump being clot during the post-operative period could not be ruled out and needs to be treated with blood thinners like warfarrin following initial use of heparin at the surgery.Antibiotic cover would be needed for removal of infected clot or it could be just a Clot in peri-orbital eyebrow tissue,with pain due to sudden vascular obstruction.Hope this reply would help you to evaluate your case and treat it with your doctors in time to come.Hope this would resolve your query and worry and Anxiety accompanied with it.Welcome for any further query in this regard to ME.I would love to help you out.Awaiting for any further query.Wish you fast recovery from this intriguing health problem.Have a Good Day.Dr.Savaskar M.N.M.S.Genl-CVTS -Senior Surgical Consultant"
},
{
"id": 224676,
"tgt": "On tri cyclen 21 for birth control. Dosage for taking tricyclen 21?",
"src": "Patient: hi. i am on tricyclen 21 regular for birth control . i am on my period and for some silly reason, i started to take my pills again this past sunday when i was suppose to start taking them on thursday (tomorrow). I am wondering what to do. Do i keep taking them or wait til tomorrow to start again or will my birth control be messed up anyways? my emotions and hormones are going crazy and would love to get back on track. please help! thank you!!!!! Doctor: just keep on taking them, and the birth control will still work. the hormonal symptoms will straighten out as well. don't be concerned if your period doesn't come this month or if it is very light, as by starting the pills early you may lighten or prevent your period. there is nothing wrong with doing this.I hope this answers your questions, please let me know if you need any more help."
},
{
"id": 159246,
"tgt": "Getting old ulcer pains. Advised test for colonic cancer. Any painless way of testing?",
"src": "Patient: just had flu and lost a stone in 7 days(from13stone 10 lbs to12stone 10lbs) now getting old ulcer pains but doctor now wants me to have tests for cancers-colonic and down the throat camera procedures.are there no other less painfull ways of checking for cancers like blood tests.why not try to assess if old duodeanal ulcer has flared up.i shouldv say here i was off my food whilst i had the flu virus.i am now eating normally but dull ache of familiar ulcer pains still persist Doctor: Hi, ulcer may be gastric or duodenal. nonspecific abdominal symptoms should always be dealt carefully. what your doctor has suggested is the right decision in my opinion. some blood tests like CEA can give a clue but not diagnostic. do not worry upper G-I endoscopy , colonoscopy are painless. proper local anesthesia is required. consult you doctor."
},
{
"id": 203540,
"tgt": "Can variation in the sperm count ( from 30 mn to 15 mn within 3 months) or vitamin deficiency make pregnancy difficult?",
"src": "Patient: hello sir,I am Ramesh, 32 years. married since 2 years, trying for pregnancy since 8 months, before that I used condoms.3 months back I have done, semen analysis, the volume is 1.5ml and count is 30million.last week again , I have done semen analysis, the volume is 1.5ml and count is 15million.is the variation in the count normal or any lack of vitamins problem.please clarify and suggest me.thanks and regardsramesh Doctor: Hello,,I had gone through your history and understand your concern.Over 15 million sperm per milliliter is considered as normal.For couples trying to conceive we recommends to have sex regularly, even when its not your fertile time of the month. Only because it takes time for sperm to both mature and to accumulate. Sperm numbers are not everything. Sperm must be able to swim in a sustained and a forward manner.Regarding your query about the use of Vitamins. Vitamin C in high concentration protects sperm from oxidative damage and increases sperm volume and motility.Antioxidant nature of vitamin C prevents the damage of the sperm.If you want to increase your sperm count you can doAvoid hot-tubs and even tight underwearRegular exercise Healthy DietKeeping calmGarlicBananasVitamins rich in antioxidants.Hope your doubts are cleared.Thank you."
},
{
"id": 138826,
"tgt": "What causes cramps in the lower pelvic area?",
"src": "Patient: Hi, I started to have cramping in lower pelvic area. I am post menopause. I went to my obgyn, had ultrasound done of pelvic, got normal report. 12 months later getting stronger cramps again. Had gallbladder out over a year ago. Started burping a lot. (normally never burp) could these two be related problems? Also bowel movement I found a thread like string. Any suggestions? Doctor: Hello,I have studied your case. I can assure you that this lower pelvic pain is not related to gall bladder problem. I think that it is mostly due to some Gynae related problem. I would recommend you to get ultrasound again to confirm the diagnosis. If there is any utreine myoma or growth than you can get removal of uterus. I hope this answer will be useful for you. Let me know if there is any other followup questions.thanks"
},
{
"id": 144451,
"tgt": "What causes chest/back pain after having steroid injection between C7 and T1?",
"src": "Patient: Today I had a steroid injection between my C-7 & T-1 to relieve pain for herniated discs at C-5 & C-6. Since having the shot I am having excruciating pain in my chest & back & can t move my right arm without being in pain. The pain feels similar to a flare up of costochondritis. What could it be & should I wait it out? Doctor: This seems to be myofascial pain which may aggravate during activity . The steroid injection will help only in the neuropathic pain. These are other components of your pain .This requires more of local pain agents such as voveran gel with hot fomentation at least twice daily.If its not settling down you have to consult a pain specialist for evalaution of any trigger points of pain . Nothing much to worry as it is only a nagging type of disease rathjer than a serious problem.You will have to take a week rest and practice good sleep habits , stress releiving habits. Get well soon"
},
{
"id": 112923,
"tgt": "Crumbling spine, broken back. What s the cure?",
"src": "Patient: Hello there, my father has a crumbling spine following an un diagnosed broken back when he was much younger, he is currently taking Co-Dydramol and Tramadol but do not know in what quanties, he used to have injections into the spinal to kill the nerve ends but not now as the pain was to severe he can only just about walk now but has to stop every few steps because of the pain, are there any stronger pain killers that he can take.Thank you Doctor: hallo dear friend was there any fracture in your spine you need to take a fresh x ray of your spine if any disc space narrowing is there you may need MRI spine MRI spine give important details, there can be lumbar canal stenosis mean while take rest , take some analgesic -nerve pain relever like tryptomer or pregabalin, hot or cold fomentation avoid lifting weights sit with taking support to your back slowly you can start spine extension exersizes"
},
{
"id": 144551,
"tgt": "Suggest treatment for multiple calcified brain lesions",
"src": "Patient: Hi.I have been taking medicines from neurocysticercosis for the past 27 years approx(my age is 40).The current dosage is zenretard 500 in the morning & zenretard 600 + gardinal 60 at night.There has been no fit recurrance for the past 14 - 15 years.The current brain MRI shows multiple calcified lesions.Do I need to take the medicines for life or decreasing the medicine gradually and eventually stopping it can be considered.What are the chances that the fits wil not occur now that I didnt have any for the past so many years.Am a bit hesitant to consider reducing the medicine but would like to.Thank You. Doctor: I understand your concernsif you have no recurrence of seizures for last 14-15 years, your thoughts to taper them is genuine.Previous neurocysticercosis might be all calcified in so many years. Though calcifoed and gliotic areas in brain can also cause seizureI will advice you to get MRI Brain with contrast (to see for any new active neurocysticerci in brain) and EEG to look for any epileptiform discharges.If there be no new cysticerci & perilesional edema and no epileptiform discharges in EEG, you can try to slowly taper your antiepileptics (of course with consultation of your neurologist). Nobody can predict exact chances of recurrence of seizure. But if your investigations are in favor, you can try taper the medicines.I wish you all the best"
},
{
"id": 41703,
"tgt": "What happens to the other follicle?",
"src": "Patient: Hello doctor, I had PCOD. This month I had letrozol 5mg twice a day, and I had two follicles,one on the right of 20mm and other on left with 14mm. They had given me Ovumax 10000 injection for rupture. One day after the shot only my dominant follicle of 20mm was ruptured.I had IUI that day. My question is what about the other follicle of 14mm? Doctor: Hi welcome to healthcaremagic.I have gone through your question.As Ovumax is prescribed to rupture follicles and if 20 mm follicles is ruptured the 14 mm size follicle could be subsided.Hope I answered your question. Would be happy to help you further.Take care."
},
{
"id": 51703,
"tgt": "What is the procedure to donate my kidney to my wife ?",
"src": "Patient: My wife is suffering from CRF for at least 3 years. Her present creatine leve is 7.5, Blood Urea 169, Hb% 5.9, Sugar (fasting) 89 and BP varying between 130/80 and 160/90. Her other parameters, like Lipid profile, are more or less normal. Now, I wish to have Kidney transplantation for my wife, instead of giving Dialysis at regular intervals. I am ready to donate my kidney. My blood group is O+ and my wife s is A+. How to proceed in this case ? Wish to know the protocol in this case. Doctor: you can donate your kidney to your wife. however, you need to be seen by nephrologist and urologist and a huge load of investigations need to be done before you are deemed fit to donate your kidney"
},
{
"id": 85351,
"tgt": "Is Epilim advisable for pelvic pain?",
"src": "Patient: I suffer from chronic pain in the pekvis I have a neurstimulator implanted and take 10 mg methadone a day to control pain. My Dr has suggested I try Eppilim. I am a bit reluctantant as I have tried Lyrica and pregablin and couldnt cope with the side effects. I ve also read Epilim can cause sedation and I just don t want to get tired Doctor: Hi, I would suggest that you try Epilim (sodium valproate) if your pain is still persistent and not able to cope with current medications. Epilim works in a different way than Lyrica/ pregabalin. I would suggest you to discuss pros and cons of using Epilim with your care provider and initiate the treatment. You might need to have baseline and follow up liver function tests if you are starting on Epilim. Hope I am able to help you. Thanks."
},
{
"id": 116238,
"tgt": "Suggest treatment for low hemoglobin and tingling in feet and fingers",
"src": "Patient: I am diabetic since 1995 and aged 66. My glucose levels are under control with FBS at 110 and PPBS at 165. Recent tests for blood count showed low hemoglobin at 9,4 which increased to 10.8 after treatment for nearly 2 months. Presently the urine test indicated albumin at 31 and I am experiencing tingling in the feet and ends of hand fingers. Kindly suyggest a suitable course of treatment. Doctor: Hi,Thanks for asking.Based on your query, my opinion is as follows.1. Uncontrolled diabetes mellitus has led to diabetic neuropathy. It has also led to early stages of nephropathy.2. Control of blood sugar within normal range is essential to prevent further damage. For neuropathy additionally alpha lipoic acid with vit B12 will be helpful.3. For mild anemia, vit B12 and iron supplements will be helpful. Add more protein to diet.Hope it helps.Any further queries, happy to help again."
},
{
"id": 42351,
"tgt": "How to get pregnant?",
"src": "Patient: I DID SOME BLOOD TEST LAST YEAR JULY AND HAD INCREASED ANDROGENS. THE DOCTOR PRESCRIBED BC BUT ITS DIFFICULT FOR ME TO TAKE THEM AS I AM WORKING AND THEY GIVE ME SIDE EFFECTS. MY WEIGHT IS 98LBS AND MY AGE IS 25 YEARS AND BEEN MARRIED FOR 3 YEARS. WE ARE TRYING FROM THEM BUT NEVER GOT PREGNANT. I GET MY PERIODS EVERY MONTH BUT NOT ON TIME.I WANT TO GET PREGNANT. Doctor: HelloIncreased level of androgen in female.Irregular bleeding or not in time .Getting no pregnancy in spite of 3 years married life .All these problems may be due to these reasons in young child bearing age female , these include.1 Poly Cystic ovarian Disease ( PCOD ) as this is the main reason of such symptoms . Diagnosis can be confirmed by ultrasound of both ovaries . 2 Hormonal imbalance , as this is another cause of such problems . Get in blood examination for F S H/ L H ratio, testosterone levels .3 Tension is another such cause in working women , so think about this also.Your doctor prescribed you Birth Control pill , so in my opinion this was for regulation for bleeding disorder and one such cause if PCOD. So in my opinion consult an INFERTILITY specialist and get his opinion regarding pregnancy.Hope this will help you."
},
{
"id": 83281,
"tgt": "What are the side effects of Thermodol DR?",
"src": "Patient: Dear sir, Good morning I have problem of central nerves system. The doctor suggest me to use Pracetamol THERMODOL DR. I want to know what is the indication of this given tablets and can use continue use one or two month . kindly suggest me any side effect? Doctor: Hello, Thermodol Dr 650 MG Tablet DR is used to temporarily relieve fever and mild to moderate pain such as muscle ache, headache, toothache, arthritis. Major & minor side effects for Thermodol DR includes Nausea or Vomiting,Allergic skin reaction,Bloody and cloudy urine,Fatigue and headache. Hope I have answered your query. Let me know if I can assist you further. Take care Regards, Dr. AJEET SINGH"
},
{
"id": 65497,
"tgt": "What causes feeling of lump in throat while swallowing?",
"src": "Patient: I feel a lump in my throat when I swallow. It doesn't hurt. It doesn't make me have trouble breathing. It is better at night and in the morning before I eat and gets more noticalbe during the day. Oddly, when I drink water, it is the most noticable. If I drink herb tea, it is less noticable. My tonsils look a little swollen, but not bad at all. What is this? Doctor: Hi dear thanks for the query on HCM You need to get an examination by a physician.There are various swelling that can appear in neckThyroid swelling, thyroglossal cyst, laryngeoceleZekers diverticulum,lymph node swelling, lipoma or neurofibroma.You need a ultrasound neck... If swelling is solid a fnac of the swelling for pathological diagnosis.Based on diagnosis treatment can be initiated.Thank you"
},
{
"id": 25749,
"tgt": "How to treat accelerated heart beat?",
"src": "Patient: I am experiencing an accelerated heart beat a odd times. I was just sitting down watching television and it started. My heart feels as if it is about to jump out of my chest. This is not the first time. It has been happening since I got sick over the christmas break. Doctor: Thanks for your question on Health Care Magic. I can understand your concern. In my opinion, we should first rule out arrhythmia (rhythm disturbances in heart) in your case. Arrhythmias like atrial fibrillation (AF), atrial flutter, supra ventricular tachycardia (SVT) etc can cause rapid heart rate with pounding sensations. So better to get done ecg, 2d echo and Holter monitoring (24 hour continuous monitoring of ecg) to rule out these causes. You may need anti arrhythmia drugs on the basis of diagnosis. Avoid stress and tension, if you are having stressful life. Don't worry, you will be alright. Hope I have solved your query. I will be happy to help you further. Wish you good health. Thanks."
},
{
"id": 52926,
"tgt": "What are the ill effects of alcoholism?",
"src": "Patient: Going through rehab for alcoholism I was told that if your tolerence for alcohol decreases that means that your liver is not good. This was around 3 years ago. Since then I have noticed that my tolerence for alcohol has decreased. I went in for detox back in June this year and asked them about my liver and was told that my liver was fine. I have never had the jaundice look that I have heard about. Are there possibly other health problems I may not know about? I am in an outpatient rehab program and a counselor there told me that my tolerence was going down and that is not good but did not give my an answer why. However the nurses in detox told me that if your tolerence goes down it does not mean that you have liver damage. So my question is I guess what does it mean? Doctor: Hello,Only from external tests it is not possible to tell about liver damage. You need to do a biopsy for that and there is no relation between tolerance and liver damage.Hope I have answered your query. Let me know if I can assist you further.Regards,Dr. B.dinesh"
},
{
"id": 164348,
"tgt": "What causes blood during urination while on Cipro?",
"src": "Patient: My son is taking 2- 500 mg a day for bladder infection. I noticed the instructions said not to take with milk. His is on a milk based formula diet. In an effort to give his medication every 12 hours, his gets his first dose an hour before his feeding (6:45 am) and second dose at 6:45 pm with his suppertime feeding. He gets fed every 3 1/2 to 4 hours so I can get 5 feedings in a day. He has been taking Cipro since Thurs. with no problems, but this morning there was blood in his urine. Help roggeps@ suddenlink.net Doctor: Hi, welcome to HCM. Can understand your concerns. You should do urine routine microscopy and culture test. It will help to find out the cause of the red colored urine."
},
{
"id": 152240,
"tgt": "Pain in the top of my head",
"src": "Patient: I have a pain on the top of my head, the pain will last for days and then it will go away. My head don t hurt, that area is tender in the top of my head. My main doctor suggested that I go to a Dermatologist, I am thinking that I need to go to a Neurologist first. Doctor: hello take belladona 200 two doses ..its a homeoapthic medicine.. if the problem still persist .. consult a homeopath god bless"
},
{
"id": 125959,
"tgt": "How can swelling and discomfort in the ankle due to edema be treated?",
"src": "Patient: Recently every night I have some foot and ankle edema and pain and burning. I know I have diabetic neuropathy but never had the sudden onset of edema before. Not sure what to do to relieve swelling and discomfort. I elevate my legs and feet and take ibuprofen for pain. No effectiveness at all. Doctor: Hi, Apply ice packs and apply compression bandages. If symptoms persist better to get an MRI scan for further assessment. Hope I have answered your query. Let me know if I can assist you further. Regards, Dr. Shinas Hussain, General & Family Physician"
},
{
"id": 14004,
"tgt": "What causes recurring itchy rash on chest?",
"src": "Patient: I have this rash that comes and goes for the past year on my chest and there are times it s worse then others. I itch at it most of the time. I have not changed anything in my daily life. Have any suggestions? I do not currently have allergies either. Doctor: Hello and Welcome to \u2018Ask A Doctor\u2019 service. I have reviewed your query and here is my advice. As per your description it seems like something called \u201cseborrhic dermatitis\u201d. It\u2019s a kind of fungal infection and can be cured with proper treatment. As of now apply topical anti-fungals like Clotrimazole over the affected area and keep the area dry and clean. Oral anti-fungals like Fluoconazole also may be needed. In case of dandruff, treat dandruff first. Consult a dermatologist and he will direct you accordingly. Hope I have answered your query. Let me know if I can assist you further."
},
{
"id": 13080,
"tgt": "What caused a hot rash on my son's leg",
"src": "Patient: hello, my four year old son has a rash which is hot to the touch on his legs. it started yesterday on his arms and now its on his legs, his diet remains the same and all his habits . The only thing different that he has done is going to the beach, and its not sun burn. could you shed some light on what is could be? Doctor: Hello,I read carefully your query and understand your concern. The symptoms seem to be related to rash dur ti overheating. I suggest using Calamine lotion for local application. I also suggest giving liquids and Pedialyte solution. Aloe vera gel can also be very soothing. Hope my answer was helpful.If you have further queries feel free to contact me again.Kind regards! Dr.Dorina Gurabardhi General &Family Physician"
},
{
"id": 161933,
"tgt": "What causes eye ache, fever with cold feet and a hot forehead?",
"src": "Patient: Hi, may I answer your health queries right now ? Please type your query here..my daughter , she is 12 years old has eye ache since this afternoon she said and also has a mild fever i think.she said she feels cold , her feet is very cold and forhead hot. Doctor: Hello, Look all these might be prodromal symptoms of some infection! Might be some viral/bacterial URTI (Upper Respiratory Tract Infection)? You just need to ensure: proper diet, hydration status maintenance, complete rest n wear complete dressed up (to avoid exposure). Give Paracetamol for Fever & pain. Hope I have answered your query. Let me know if I can assist you further. Take care Regards, Dr Ajaygupta009, General & Family Physician"
},
{
"id": 85456,
"tgt": "What does metallic taste in mouth mean?",
"src": "Patient: I have been nausea, extreme fatigue with a mild fever for last 3 weeks, I have seen my Dr and he thought it was a stomach bug and gave me metronidazole 500 mg 4 times a day for 7 days and if not better to call, called him 4 days after with no improvement was still not able to eat solid food even bread without feeling sick, so he called in ondansetron 4 mg every 6 hrs. Was able to eat food finally last Friday but was still nausea even with meds. I contacted my Dr on Monday bc I was having symptoms of burning when I urinate even though I was on meds still. Went yesterday and the password told me I have a lot of white blood cells and blood in my urine. She gave me another antibiotic nitrofur-macro 100 mg 2 a day. Last night I woke up at 3am with a pain in my left shoulder blade and a metallic taste in my mouth. No one can tell me what it could be, any recommendations? Thank you Doctor: Hello, From what you are reporting, I think it is due to urinary tract infection. Should treat it adequately. Plus, I think you should get a few investigative tests done to rule out other pathologies related to liver, gallbladder or pancreas. Discuss this further with your caring doctor. Hope I have answered your query. Let me know if I can assist you further. Take care Regards, Dr Albana Sejdini, General & Family Physician"
},
{
"id": 52564,
"tgt": "Suggest treatment for elevated ALT and AST levels",
"src": "Patient: My ALT WAS 1741 and my AST 948 and my liver specialist just kept repeating my labs for 4 weeks until I could hardly walk on my own and I went to an internist who send me to the hospital. This was about 19 months ago and as I look back I just can\u2019t figure out why he didn\u2019t start me on prednisone immediately. What would standard procedure be? Doctor: Hello and Welcome to \u2018Ask A Doctor\u2019 service. I have reviewed your query and here is my advice. If the enzymes are persistently high and the diagnosis is unclear, it is better to go for a liver biopsy to make a diagnosis. You can seek from a hepatologist for expert opinion. Hope I have answered your query. Let me know if I can assist you further."
},
{
"id": 26666,
"tgt": "Can a mini stroke cause sore spot on back of head?",
"src": "Patient: Can a mini stroke cause a person to have a sore spot on the back of the head? I went to local er because I was having slight chest pain and very bad headache but I woke up that morning with a real bad sore spot on the back of my head. At the er they did ekg xray and gave me baby aspirin and treated me for migrane My Blood pressure was going up as well and I am on bp meds. can you help me understand why I still have this sore spot on my head and my heart feels like its fluttering at times. Doctor: Hello!Thank you for asking on HCM!I read your question carefully and understand your concern. Your symptoms seem to be related to high blood pressure and migraine. I would exclude any possible stroke related to your symptomatology. You should know that high blood pressure can cause vascular type headaches, mimicking migraine attacks. The sore spot on your head is related to a temporary dysfunction and vessel constriction in that area from the vascular headache and high blood pressure. I agree with your doctors that the best therapy to control your symptoms would be aspirin (for the prevention of possible vessel occlusion related to the constricted vessels) and arrangement of your antihypertensive therapy. Regarding the palpitations (heart fluttering) that you have been experiencing I would recommend performing a careful physical examination, a chest x ray, an ambulatory 24-48 hour ECG monitoring and some blood lab tests like: - complete blood count- thyroid hormone levels- kidney and liver function tests- fasting glucose- blood electrolytes , etc, to rule in/ out any possible cardiac arrhythmia and investigate for any metabolic cause, related to this clinical scenario. If all the above tests result normal, anxiety should be taken into consideration as a possible factor contributing in high blood pressure and tachycardia. Hope to have been helpful!Best wishes, Dr. Iliri"
},
{
"id": 24633,
"tgt": "Is stopping of heart rythm for two seconds serious?",
"src": "Patient: I started work again, my family has been under extreame stress.When I was at work by heart rhythm stopped for about two seconds a couple of times.I have been through ecg,ekg and all emergency room stuff.They say nothing is wrong.Should I make my doctor send me for other test? Doctor: Yes do your holter and then on that tracing we can decide u need pacemaker or not... if u have 2 second pause ecg then send it to me"
},
{
"id": 103751,
"tgt": "Palpitations in the right corner of left eyebrow, numbness. History of asthma, sinus. Treatment?",
"src": "Patient: Hi, from last one week I m having palpitation in the right corner of my left Eyebrow (right above the nose ). It is not continuous but starts all of a sudden and then stops it self. While palpitation, I feel numbness at the same place and saw can see skin tissues movement. Please note that I do have the history of Asthma and resenting experienced Sinus symptoms too. Please advise Doctor: SINUS IS COMPLICATION OF ASTHMA AND SINSES ALSP EFFECTS EYECAOSING SYMPTOMPS YOU WROTESINUSES CAN BE CAUSED Y PCM BRUFEN VITAMINS CALCIUM MUKIVITAMINS IRON DIET SUPPLIMENTS USED DURING ASTHMABETTER GET ALLERGY TEST AND IMMUNOTHERAPY TO COVER BOTH ALLERGIESPRESENTLYTAKE ALLEGRA 120 MG BDGELUSIL 1 TSF TDSAPPLY NEOMYCIN H EY OINTMENT IN NOSE EYES AS ;OCAL APPLICATION CHROMAL FORTE DROPS IN EYES AT NIGHT AND SEA WATER 2 DROPS AT NIGH IN EACH NOSEDICLO AS PAIN KILLER SOSYOU CAN CONTINUE TILL YOU GET FURTHER ONVESTIGATIONS"
},
{
"id": 85328,
"tgt": "Is it safe to stop Warfarin and start using Disprin cardio care?",
"src": "Patient: Good afternoon doctor , i was put on Warferin for life, i suffer extreme pain in my back and hipjoints i want to stop the warfiren and use Disprin cardio care as it says on the package it unclots the blood same as what warfein does. is it safe for me to do so? Doctor: Hello, I am not sure what Disprin cardio care is, but it sounds like it might be aspirin, in which case it would not be safe to replace your Warfarin with it. Whatever Disprin is, it is unlikely to be as effective as Warfarin is for preventing blood clots. Since blood clots can potentially be very dangerous, I would definitely not stop taking the Warfarin without discussing it with your doctor. Hope I have answered your query. Let me know if I can assist you further. Take care Regards, Dr Aaron Branch, General & Family Physician"
},
{
"id": 25265,
"tgt": "What causes high blood pressure and pulse reading?",
"src": "Patient: I am a 48 year old female and stopped in my local pharmacy to check my BP and pulse........my doctor had advised I should do this a few times per month after giving a high reading at the doctor's office a 2 months ago. I have always had thru my life a reading of 110 over 70 until a few months ago. Yesterday it was 134/50 with a pulse rate of 103. I have not felt \"right\" for a few months now. A Nuclear Stress test and ches x-ray were good but I still feel there is something wrong. What should I do and what could be going on. I'm am afraid of stroke or heart attack at this point. Thank you for your time. Doctor: Hello , the blood pressure reading is still in nomral range and doesn't require any medication at present. Just cutting down salt from your diet and regular aerobic exercise should be enough .Now coming to the pulse rate , yes it's just borderline high , maybe just the anxiety of you going for a check up may be enough to push the pulse and blood pressure. Just keep a tab on both reading s by measuring it once quality while and inform your doctor if it's more than 140 / 90 persistently . I would also suggest you to get your thyroid profile done . Regards Dr Priyank Mody"
},
{
"id": 149592,
"tgt": "Pain due to prolonged sitting and sleeping. Cervical spine MRI showing T1 and T2 weighted sagittal and axial images. No disc herniation. Any suggestions?",
"src": "Patient: Hi Doctor, I just got my MRI done for my neck and spine as I am continuously feeling pain after prolonged sitting and waking up from the bed. I am 31 years old. Here is the report : Cervical Spine MRI T1 & T2 weighted sagittal and axial images were obtained. Normal Dimensions spinal canal with normal spinal cord including cervico-medullary junction The vertebral bodies height with their bone marrow signal intensity are preserved. No significant disc herniation, nerve root compression or exit foramina obliteration. Thankyou doctor. Doctor: Hi,Thank you for posting your query, and providing the MRI spine report.I would like to tell you that your MRI report is absolutely normal.So, the cause of your pain is mostly mechanical. You would benefit from neck and back extension exercises. In addition, please ensure a good posture while sitting.A few sessions of physiotherapy would help in reducing the pain further.Best wishes,Dr Sudhir Kumar MD DM (Neurology)Senior Consultant Neurologist"
},
{
"id": 160677,
"tgt": "Suggest treatment for child suffering from recurring ear infections",
"src": "Patient: my son has recurrent ear infections or c/o water in his ears.Has been on amoxicillin,augmentin and past year on omnicef and suprax which does not seem to resolve these episodes completely. I have also noticed that he is not even getting a fever with the recent c/o of earache which always ends up as an infection. He is 5 years going on 6 in january. what do you recommend? Doctor: Hi,As per your query your child suffering from recurring ear infections which seems to be due to fungal infection, wax build up, and fluid accumulation. Need not to worry. I would suggest you to:- Take steam twice a day- Apply warm dry compresses to external ear- Cover your ear while bathing- Put Otogesic ear dropsI would suggest you to consult Pediatrician for proper diagnosis. Doctor may examine by otoscope and may prescribe antibiotics amoxicillin at least for a week along with decongestants. You should go for audiometer testing as well to check hearing level.Hope I have answered your question. Let me know if I can assist you further. Regards, Dr. Harry Maheshwari, Dentist"
},
{
"id": 107711,
"tgt": "What causes pain in upper left side of back?",
"src": "Patient: I have on three different occasions had pain on the upper left side of my back just a little above my waist that extends to my side and front (or vice versa) that causes vomiting and upset stomach. I have to sit up in the chair to sleep because the pain is so severe that I can t lay down. It s also very painful to get up from a sitting position. Emergency room keeps saying muscle spasms so they giuve me muscle spasm and pain prescriptions and send me home. The occasion have been four months apart starting with Sep 2014 Doctor: Hi welcome to HCmI have gone thru your query regarding severe pain in upper back and can understand your concern .If we find out and remove the real cause of our pains ,90% of our problem is solved . It is often caused by poor posture . Upper back pain has become a familiar complaint from people who work at computers most of the day. Often, upper back pain occurs along with neck pain and/or shoulder pain. Often, muscular irritation and upper back pain is due to either de-conditioning (lack of strength) or overuse injuries . Muscle strains, sports injuries, auto accidents, or other injuries can all result in pain from muscular irritation.Low immuity due to lack of activity , lack of essential nutrients & hyderation and faulty life style are some of the basic causes for your sufferings . A correct diagnosis of back pain requires diagnostic tests (such as an MRI scan) and correlation with physical symptoms are required to determine the extent of the damage and develop a treatment plan.When you must lift, bend your knees and keep your back straight. Avoid twisting. Keep the load close to your body.Getting Inclusion of balanced diet with essential nutrients and antioxidants in natural form , plenty of fiber raw ginger ,garlic , calcium and vitamin D, a spoon of Turmeric powder and 35 ml of Aloe Vera may help prevent osteoporosis, which can lead to compression fractures and back pain .Regular walk ,stretching exercise- from head to toe when youare not in pain ,yoga, panayam , deep breathing , meditation . All help alley pains and give resistance to body from diseases and gradually help alley pains , will gain healthAvoidance of fried ,fast foods ,tea coffee ,alcohol ,smoking over exertion ,sleeplessness dehyderation , all further help .Take 2-4 doses of Homeopathic Colchicum 30 / half hourly to alley pains Hope this helps solves your query .Take care , All the best .Wish Get well soon .Don't hesitate to get back tf have any further query ."
},
{
"id": 71635,
"tgt": "What causes fat loss around only ribs and sternum?",
"src": "Patient: Hi,I'm 22, 6 foot 2 inches and weigh 16 stone 7 lbs, but over the past few months my ribs/ sternum are becoming increasingly visible, even though I'm not losing any weight. I'm not doing any weightlifting/ vigorous activity so no increase in muscle weight.What could be causing this? It's as though I'm losing fat from around my ribs/ sternum but nowhere else, and not losing weight. Doctor: Thanks for your question on Healthcare Magic.I can understand your concern. No need to worry for this. This is hormonal changes of the body.Testosterone in males causes more fat distribution in abdomen and less in ribs area.It also causes muscular build-up by increasing muscle mass. So no need to worry for this. This is physiological changes of body. Hope I have solved your query. I will be happy to help you further. Wish you good health. Thanks."
},
{
"id": 115582,
"tgt": "What causes numbness in the right side of body?",
"src": "Patient: Numbness in side of face, neck, arm and leg My stepdad is 34 and has had numbness in the right side of his face, neck, arm and leg.that comes and goes every time it happens he goes to the ER, the firs time they gave him adivan the second time they gave him bp medication. His cat scans were clear. Is it MS, is he going to die? Doctor: Hi,Thanks for asking.Based on your query, my opinion is as follows.1. As CT scan is normal, it is not multiple sclerosis.2. Possible vitamin deficiency needs to be thought of, particularly B-complex group of vitamins.3. Sensation would be present all over, but variation is known to occur. If there was to be MS, spasms, organ involvement, etc would have been seen.4. Start on vit B-complex supplements. Avoid alcohol. Back exercises to strengthen vertebral column to reduce spondylosis risk is also necessary. As CT scan is normal, transient ischaemic attack also is not seen. Continue on blood pressure medication. More details necessary for further opinion.Hope it helps.Any further queries, happy to help again."
},
{
"id": 6998,
"tgt": "I am taking duphaston. When will I get pregnant ?",
"src": "Patient: hi im trying to get pregnant , i have PCOS ,my doctor adviseD me to take duphaston 2 times a day and to have regular period.I takE it for 3months and my periods are on the exact date. I stopped last month because im delayed for 1 month and 16 days.Before the day to visit my doctor i have a severe back pain then i get my period. The doctor advised me again to start taking duphaston on my 16 and 25 day.When do you think will i get pregnant?im 24 y/O. Doctor: Hi Welcome to HealthcareMagic Keep trying. Follow your doctor's instructions. Reduce weight if you are overweight by physical activity and balancing diet. You ll be pregnant soon. All the very best."
},
{
"id": 106424,
"tgt": "Cough since 2 weeks, What could be the reason ?",
"src": "Patient: My husband is getting cough since 2 weeks. And moreover it is dry cough though he does not smoke. What could be the reason ? Doctor: Dry cough is due to allergy and throat irritation due to nasal infection or allergy as well. This can be improved by taking antiallergic medications for timebeing."
},
{
"id": 131935,
"tgt": "Suggest treatment for swollen and painful knees",
"src": "Patient: yes, thank you. 2 things, one both knees are swollen, not necessarily fluid, tennis balls behind both knees, and more recently marbles and pain in and around both kneecaps, as well as swelling above and inner side of knees. No fever, just discomfort and pain. secondly, I also was told I have anti bodies for lupus ( I am a smoker). Since both ovaries, removed, coincidently or not, most joints in my body hurt plenty more than befor., I am 48 yrs, i feel like my 80 yr. old mother, she walks better than me!, (pls do not tell me myofibralgia, i believe that is a myth) I had the scan for articulars, the one with hot spots, and dye injected, nothing showed up but reg. wear and tear. except in left hand, i find this hard to believe considering the discomfort I am feeling ( seem to have a crisis 2 wks. ago,sort of flare up, severe pain and that s when knees started up) left shoulder sore for years and hand, big thumb inpeticular, for the past yr, 7 times worse. Hard to get any diagnosis lately here in quebec, system here has gone awry. What are yr. thoughts, thank you, Doctor: Hi I have gone through your history and details. In my opinion you are suffering from Osteoarthritis of both knees with B/L Baker's cyst . I would suggest that you take pain killers . Hot fomentation and collagen powder daily and if you feel no relief in about one month then plan for knee replacement."
},
{
"id": 7095,
"tgt": "Is it possible to conceive with ovarian cyst ?",
"src": "Patient: yesterday i take HSG ang USG abdomen and pelvis scan. HSG shows right Fallopian tube is not visualised. No peritoneal spillage of contrast media seen from right fallopian tube, Left tube is normal.USG shows left ovary is enlarged in size measures 5.4 x 4.7cm & shows a well defined cystic 4.1 x 3.7cm with clear internal echoes & distal enchancement. Imp:Left ovarian cystic -? Retention cyst. it is possible to conceive.the blood is normal. Doctor: Hello. Thanks for writing to us. Your right fallopian tube is blocked. Chances of conception are present with the help of normal left fallopian tube. The hypertrophy of your left ovary might be a compensatory hypertrophy. The cyst seen might be a functional cyst which can reduce after proper treatment. You need to consult your gynecologist for ovulation induction therapy which will help in conception. I hope this information has been both informative and helpful for you. Regards, Dr. Praveen Tayal drtayal72@gmail.com"
},
{
"id": 80633,
"tgt": "What does \"fibronodular densities in the right mid lung, ptb minimal\" means?",
"src": "Patient: Hi im arvee my x-ray clinical diagnosis findings say,apicolordotic view shows fibronodular densities in the right mid lung...ptb minimal..what those it mean?and what medicine will i take for this?because im working right now and did not get my medical card and mayor's permit because my medical result says im not fit to work.. Doctor: Hello dear, thanks for your question on HCM. I can understand your situation and problem. Your x ray is suggestive of Infection in right lung.On x ray appearance, it seems more of tuberculosis. So I advice you to consult pulmonologist and get done1. Sputum examination for tuberculosis. 2. Bronchoscopy and BAL ( Bronchoalveolar lavage ) analysis if sputum is not available. For tuberculosis, you need to take atleast 6 months of treatment. You need to take isoniazid, Rifampicin, Pyrazinamide and Ethambutol for 2 months, followed by Rifampicin and isoniazid for 4-6 months. Since this is long course, better to consult pulmonologist and start appropriate treatment and do regular follow up."
},
{
"id": 217711,
"tgt": "What causes pinching pain in nipples with shoulder pain?",
"src": "Patient: Hi, I travelled last month around 600 kms and 2 days after that i felt that i ve slight pinching symptomn nexr my left nipple , sometimes around my left nippe...then later i recognized that on i have ache on my left back side near spinal gaurd. sometimes i feel the slight pain on my left shoulders as wekk i dont feel any discomfort while breathing etc... I took ECG and the Dr said am okey with respect to Heart! Pls advice. Regards Raghu Doctor: this type of pain usually starts because of poor posture while travelling and and carrying baggage over the shoulder. this is generally corroborated under the Scapulocostal syndrome ."
},
{
"id": 55160,
"tgt": "What is the recovery time for lap cholecystectomy?",
"src": "Patient: I have been tod I have gul stones and I have been asked to book an appointment at the hospital. I would like to know how long i will be off work for. I am going for an lapcoly cystectomy operation. i will like to know how long i will be off work for. Doctor: hi. some of the advantages of doing a laparoscopic surgery over open technique are faster recovery, less hospital stay and less postoperative pain. you will have very small port incision/s. some institutions/surgeons perform laparoscopic cholecystectomy as an ambulatory case. some surgeons prefer overnight admission, alotting 1 day immediately after surgey for post-operative observation, then for discharge the following day. if you're work is not involved in lifting heavy objects/strenuous activities, you can be back to work as early as the third-fourth day after the surgery. bear in mind, that your physiologic response may be different with others (or case to case basis). less co-morbidities would also mean less chance of having complications during the post-operative course. a short-interval follow-up with your surgeon is also advised.hope this helps.good day!!~dr.kaye"
},
{
"id": 47156,
"tgt": "Is it possible for the successful transplantation of the kidney from a non relative person?",
"src": "Patient: I m 34 years old and my blood group is O+, I have Kidney diseases due to B.P., But the problem is I have no Kidney donor from my family bcoz my mother and my real brother;s blood group are not match with my blood group. If I could receive kidney from non relative person with his/her permission then my kidney transplant can do in your Hospital, is it possible for me? otherwise pls tell me what should I do? Doctor: HelloThanks for query .Based on the facts that you have chronic renal failure and advsied Renal Transplant but unfortunately there is no matching donor in your close relations ..In a given situation you will have to search for matching unrelated donor and his kidney can be transplanted to you after getting permission from a state level organ transplant committee who will scrutinize your case in detail and confirm the facts that the unrelated donor has willingly agreed to donate his kidney to you without any financial transition between both of you .This is routinely done in most of the well reputed Hospitals and Govt medical colleges in most of the states in India,Dr.Patil"
},
{
"id": 150676,
"tgt": "Depression problem",
"src": "Patient: my father is having depression and he always thinks negative Doctor: hi dear, negative thinking is a part of depression.tab Escitalopram10mg once daily will help.try to find out if there is any stressor."
},
{
"id": 202929,
"tgt": "Have hard lump on penis, hurts to touch, not sexually active. What is it?",
"src": "Patient: I have this hard lump on the body of my penis . I m 14 and not sexually active. It hurts to touch but doesn t ooze or anything. Can you please tell me what it is? Doctor: HIThank for asking to HCMI can understand this looking to your history given here this situation can be treated with the \"Tab Ibuprofen 400 mg twice in day if the lump does not responds to this then clinical examination is must hope this information helps you have nice day."
},
{
"id": 91685,
"tgt": "What is the treatment if right side of abdomen pains after waking up?",
"src": "Patient: Dear Sir Every day early morning whenever I am waking up after my sleep, my right hand side of the abdomen is paining always. My doctor checked the urin and told that a chance of stone is there. What is the best treatment. Pain is not much. As per doctors instruction, now I am taking plenty of water. Any medicine I have to go for or any other treatment. Please let me know. Best regards. Farooque YYYY@YYYY Doctor: Hi.You say the pain is only in the morning ...Do you have normal bowel habits, meaning is there any constipation or difficulty in complete evacuation of stools.Try a mild laxative in the night , this may help you. Of course get a check up done by a Surgeon and at least ultrasonography done."
},
{
"id": 81771,
"tgt": "What causes chest tightness with breathlessness?",
"src": "Patient: I am a 46 year old female, 140 lbs. healthy. I have some anxiety lately but am having tightness in chest and hard to breath, waking me at night, heart racing, sometimes coughing...pulse around 52 at times to 60 bp just a little higher than normal for me....tired. Doctor: Thanks for your question on HCM. In my opinion you are having anxiety related chest tightness only.But we need to rule out cardiac and pulmonary cause first.So get done ECG and 2D Echo to rule out cardiac cause. Get done chest x ray and PFT ( PULMONARY FUNCTION TEST ) to rule out pulmonary causes. If both are normal than mostly you are having anxiety related symptoms. So avoid stress and tension. Be relax and calm. Get done psychiatrist opinion and counselling sessions to find out stressor. Try to cope up with it."
},
{
"id": 185082,
"tgt": "What causes blood from mouth while spitting?",
"src": "Patient: hi! my age is 35,i am 60kg ,my height is150cm,i am a patient of high blood pressure,i too have stone in my gall bladder for which the treatment is going on. today in the morning when i was spiting then i found blood coming out of mouth . may i know the reason for this? well i am a female. Doctor: Thanks for your query, i have gone through your query. the bleeding gums could be because of the gum infection secondary to deposits. or it can be because of the drugs like blood thinner(ecosprin), that might aggravate the incidence of bleeding while brushing. consult a oral physician and get your teeth cleaned and maintain oral hygiene. you can use topical gum astringents like stolin gum paint. I hope my answer will help you. take care."
},
{
"id": 163747,
"tgt": "Suggest medication for a tiny black mole",
"src": "Patient: I just discovered a tiny black mole on my 6 month olds lower back. About one month after he was born I was diagnosed with malignant melanoma, only stage one. I had surgery to remove the cancer but I am now fearful it could have spread to him during my pregnancy. Doctor: very less chance of spreading this melignant melanoma, just observe..there is no need to doing anything for your child, this could be normal mole..just you need to observe for change of size or shape.."
},
{
"id": 25025,
"tgt": "Suggest treatment for high blood pressure",
"src": "Patient: for a few months, as soon as I put my head on the pilot I feel like a heart beating in my right ear. After two hour sleeping I woke by the noise in it and decided to take my blood pressure, for my surprise it is 171/110, or 180/108, etc...wat can this be? Doctor: Hello and thank you for using HCM.I carefully read your question and I understand your concern. I will try to explain you and give my opinion. You should know that we talk about hypertension if we have mean value that exceeds 140 / 90 mmHg.A person might have high value during emotional and physical strees so its mandatory to judge on mean values. Of course your values are pretty high so it is necesary to treat them. Because of this high values you might experience headache, blurred vision, noise in your ear ect.Hypertension is divided into two groups. Essential hypertension, usaly in older age that is a chronic disease whithout an identified cause and secondary hypertension meaning that is a secondary couse of it.This might be a renal disease, an endocrine problem or a cardiovascular anomaly. So, if I was your treating doctor I will recommend some examination like a cardiac echo to evaluate heart function and walls, a full blood analyze to chek renal function, an abdominal echo and a holter pressure monitoring to evaluate your values during day and night. After all this we can better judge what we are dealling whith and how to treat it. Hope I was helpfull. Best regards, Dr.Ervina."
},
{
"id": 64549,
"tgt": "What causes lump on the right side of jaw line?",
"src": "Patient: Hi, i have recently found a ball like object on the right side of my jawline about the size of a large grape. Witch was making the whole right side of my jaw swollen, I ve also become extreamly I ll with the following : vomiting, fever, migraine, nausea and dizziness lasting around the same time i found this thing about five days. if you have any idea what this can be please help Doctor: Hi..Can understand your concern..As per your complain lump on the jawline along with other symptoms of vomiting, fever, migrane, nausea and dizziness are indicative of infection..It can be due to a deeply infected tooth leading to formation of pus below the root tips of the tooth and accumulation of pus in soft tissues can lead to swelling..When the infection by some way enters the blood it can cause septicemia and generalized features of infection as you have..If the lump is below the jaw line it can be an inflamed and swollen lymph node which is also a sign of infection in any part of the body either near or distant..I would suggest you to consult a dentist and get evaluated for the exact cause of the problem..In case of infected tooth antibiotics, painkillers and treatment of the tooth either by root canal treatment or extraction can be relieving..In case of swollen lymph node the curing of infection by antibiotic treatment can provide relief..To reduce fever Tylenol can be taken..Anti emetics like Avomin can help in reducing vomitings..Hope this information helps..If you find the answer helpful please write a positive review and click on find this answer helpful as a token of appreciation..Thanks and regards..Dr.Honey Nandwani Arora."
},
{
"id": 147382,
"tgt": "Having pain on the top of head after an injury",
"src": "Patient: i banged my the top of my head on the bathroom cabinet on wednesday, i had a small lump but no cut since then the bump has gone down but it is still sensative in that area and if pressed hurts alot, if i lean forward my head feels heavy and i feel dizzy when i get up and one of my eyes is tired is this normal? Doctor: This is something called post head trauma pain syndrome. There is many a times minor injury to the nerve rootlets and small vessels on the scalp which might lead to a persistent numbness, increased sensitivity and headaches in the patients Please go to your neurologist if the pain and increased sensitivity in unbearable. He/she will be able to help you, I am sure."
},
{
"id": 58479,
"tgt": "Left lobe of liver removed due to tumor. Something stuck in stomach. Can tumor grow again?",
"src": "Patient: Last summer I had the left lobe of my liver removed because of a football sized tumor as well as my gall bladder. I am at work tonight. I have a feeling like something is stuck in my upper stomach. When I wear a bra I get sicker its got to be pushing on something and Im throwing up too. Could the liver tumor grow to be huge again in one year. Do I need to go to the ER or get into my Dr. on Mon.thanksjen Doctor: Hi, it appears to be the gastric irritation, due to peptic ulcer, or irritant foods. Or it could be due to the boil on the skin of the chest. The possibility of the recurance of the cancer are less. So don't I advise you to consult a surgeon for diagnosis, and treatment. Thank you."
},
{
"id": 5814,
"tgt": "Trying to conceive. All medical test normal along with husband's semen test normal. Looking for suggestion",
"src": "Patient: Sir, I am 40 Year old and my husband is 35.We are married since 2 Years.We are trying to conceive but with no results.I have undergone all checkups,my Periods are very normal,Date to Date. My husband has undergone semen test .All our tests are O.K.I am attaching all my Reports for your reference.Please suggest. We are situated near Nagpur. Please suggest. Regards Sapna Shrivastava Doctor: Hello Thanks for your query Am unable to access your reports At 40, your ovarian reserve , that is, capacity to produce mature follicles should be assessed. Please consult an Infertility specialist for the same. Also your tubal patency should be assessed. At 40, I think you should strongly consider IVF also. Take care."
},
{
"id": 22526,
"tgt": "What causes a dull pain in the chest and rapid heart beat?",
"src": "Patient: hi i have a dull pain in the centre of my chest which is worse when i lie down or move about. it also pulsates with my heart beat and when my heart gives a big thud (beat) it hurts more, feel like there is a thump n my throat and chest, i had this about 5 yrs ago and tests came back with nothing. i have had it over 24 hrs now, could it been indigestion ? Doctor: Hi,This looks like a gastritis and reflux disease. Still one ECG just to rule out heart disease, if you have any risk factors like elder, smoking, high bp, or cholesterol, then avoid fatty, oily and high calorie diet. Have some walk after having food instead of taking rest. Have multiple small meals instead of heavy meals. Have regular sleep habits and avoid stress. Eat lots of green leafy vegetables, fruits, fish once or twice a week, and avoid meat. Also, avoid smoking and alcohol if any. You can get prescribed tab Pantoprazole before breakfast once a day for 2 weeks.Hope I have answered your query. Let me know if I can assist you further.Regards, Dr. Sagar Makode"
},
{
"id": 160508,
"tgt": "Suggest treatment for cold and cough in a child",
"src": "Patient: my son is having cough and running nose since last 2-3 months. He was on course of antiabiotics twice-once it was amoxcilin and second time was on a different antibotic. Now the doctor has given Levocetirizine Dihdrochloride and Acebrophylline syrup since he thought it is because of Allegric Bronchitis and adviced to do this course for 1-2 months. After this course was started the coughing has reduced a bit in last 2-3 days but I am wondering if his coughing can be stopped completely and what could be the cause why he keeps getting this cough back every week. Doctor: Hello, I would wonder about asthma with such a long time cough. But it would be helpful to know whether there is complete clearing between episodes or not and whether you think the cough vs runny nose/congestion is the major symptom. The antibiotics were probably given to treat a sinusitis, and if so, there should have been improvement around day 3-4 or 5-6 and then continued improvement until no runny nose. Has he tried any albuterol? I don't know where you are located, but in the US we use an inhaler or nebulizer machine to blow in medicine to the lungs. We don't really use theophylline like syrups in kids anymore, although that might help if it's asthma. Hope I have answered your question. Let me know if I can assist you further. Regards, Dr. Lisa Baker, Pediatrician"
},
{
"id": 193893,
"tgt": "Will masturbation cause any health issues?",
"src": "Patient: Sir,i m 4rm assam & i am 18 yrs old.my problem is that i've the habit of musterbation from my childhood for which i feel very uneasy.my interest is in both men & women.thus,i wants to know how i can control myself to stop the habit & if the habit is injurious to health or not &why Doctor: Hello, Masturbation is not harmful to health. if you want to stop it, just stop it. If you stop it, you may face nocturnal emissions. Hope I have answered your query. You can contact me for treatment options. Let me know if I can assist you further. Regards, Dr. K. V. Anand, Psychologist"
},
{
"id": 125468,
"tgt": "What could cause sharp pain in arms especially on lifting heavy items?",
"src": "Patient: I have a side sharp pain when i throw objects with my arm, my right side only. The heavier the object the more pain it causes. It a super sharp pain instantly when i try to throw things. Do you know what it might be and how to resolve the issue? Thanks Doctor: Hello, It could be a simple muscular pain. As of now, you can use analgesics/anti-inflammatory combination like aceclofenac/serratiopeptidase for symptomatic relief. If symptoms persist better to consult an orthopedician and get evaluated. Hope I have answered your query. Let me know if I can assist you further. Regards, Dr. Shinas Hussain, General & Family Physician"
},
{
"id": 190848,
"tgt": "What treatment other than excision can one take to infection in the molar ?",
"src": "Patient: Hello doctor, Thanks in advance for your advice. Doctor, I do have swelling(which is very mild) and pulling sensation of my left face for the past 2 months, and my doctor had tested me and said I had parotitis, and he said it will be cleared by itself, but till date, it hasn t get cleared; and he sent me to a dentist; he took an OPG test and said i had infection in my third molar teeth and asked me for excision of this tooth. My complaint now is severe chills and slight pain and slight swelling in my left jaw as well as my right jaw... Can you please suggest me what to do.. I am scared to remove my third molar because of the fear of getting ill again, since i have been ill for the past 2 months.. I have attached my OPG image here.. Please do spent some time and help me.. thank you.. Doctor: Third molar infections are quite chronic in nature. Please get the 3rd molar removed as the source of infection will be gone. There is no problem with the 3rd molar removal it is regular procedure where in it is done under local anesthesia. Please do not have a myth that 3rd molar extraction results in complications. The swelling can be controlled by cold pack, avoid hot food and start antibiotic coverage. Get the tooth removed during the antibiotic cover to avoid retrograde infection. Use betadine gargles to keep mouth free of infection"
},
{
"id": 208116,
"tgt": "Suggest treatment for excessive crying",
"src": "Patient: Im 16 I cry a lot. Adults of people my age when they try to help i cry when they talk about whats wrong with me i cry. When my parents have meetings with my parents and they look at me i cry. I dont know why. When i talk about my problems or try getting help in genral or even if someones saying How i can do better I tend to start shaking a little but then i cry i cant stop it just continues worse the more i try to talk about it to someones face... Doctor: Hi,I can understand your concern regarding your symptoms. Your frequent crying can be related to underlying depression which results in low mood. In addition, you seem to be manifesting anxiety which results in feeling nervous in front of people and shaking.You should talk to your parents regarding your symptoms and consult a psychiatrist for complete evaluation. You may respond well to medications like escitalopram which will help you overcome your symptoms. You should also start exercising daily which will help you relax and feel better.I do hope that i was able to answer your query. Best wishes."
},
{
"id": 186455,
"tgt": "How to get rid off from pea like bone under jaw?",
"src": "Patient: i have a pea like bone under jaw bone..when u feel with thumb it detached and feels has moved ..u can feel on the other side too..not so much lumpy but hard like pea and bone..it seems to move..no pain at all just felt while resting thump on both sides of jaw Doctor: Hi! Welcome to Healthcaremagic.I read your query. You have not specified exact location where you feel bone fragment under jaw bone. This can be Tori or exostosis. These are bone growths in jaw. Usually inner side of lower jaw near premolars or buccal side of upper jaw. They are usually asymptomatic and do not require any treatment.However, if there is irritation or ulceration on its overlying mucosa, it can be reduced by a small procedure.For accurate diagnosis, I suggest you to visit a dentist and get it checked.Hope the answer helps you. Thank you!"
},
{
"id": 8907,
"tgt": "What should be done for pimples on the face ?",
"src": "Patient: hi doctor ,ma sister is 22yrs old and she has lots of pimples on her entire face .sometimes it bleeds.she stop using cosmetics etc but it is still same.what to do Doctor: Hello Thanks for your query.\u00a0\u00a0\u00a0\u00a0\u00a0 Wash the face many times a day,keep it dry.avoid cosmetics and consult skin specialist for proper treatment,Avoid fat fried foods.As pimples are due to blockage of certain glands \u2018Hope I have answered your query, I will be available to answer your follow up queries, \u201cWish you Good Health and trouble free speedy recovery\u201d"
},
{
"id": 135023,
"tgt": "Suggest exercises for broken ribs",
"src": "Patient: I fell down from five steps of staircase at home, I cut my head, right side, I had 2 ribs brokers (fifth and sixth and bruise on my pelvic. I had x-ray on chest and pelvic bones. The hospital said it will heal itself. They gave me pain killer Today is the 14 days after my injury, what kind of exercise should I take. Doctor: Hi Dear,Welcome to HCM.Understanding your concern. As per your query you are wondering about exercises for broken ribs . Well there can be treatment option for broken rib. I would suggest you to consult orthopedic surgeon for proper examination . Doctor may order x-ray , CT scan or bone scan . Doctor may wrap elastic bandage around your chest along with splint and prescribe muscle relaxant along with anti inflammatory . Doctor may also recommend deep breathing exercises for broken rib , which are very effective along with physical therapy .For now take proper rest , apply warm compresses on broken several times a day and take ibuprofen or acetaminophen for pain . Also do not sleep on side of broken rib. Hope your concern has been resolved.Get Well Soon.Best Wishes,Dr. Harry Maheshwari"
},
{
"id": 128488,
"tgt": "Suggest treatment for Adenomyosis",
"src": "Patient: Dear Doctor, My wife is suffering from Adenomyosis for past four years, She has taken alopathy, ayurveda, Homeo and now with sidda, She was completely following doctors advice and went on continuing a tretment for few months before switching over to other. But no use suffering from lot of pain. please advise Doctor: dear patient If adenomyosis is treated by all methods of conservative treatment and not relieved definitive treatment if you have completed your family is hysterectomy. Family completed is married and having children. Severe pain so as there is interference with daily activities final choice after conservative treatment is removal of uterus. please consult expert gynecologist and take their opinion."
},
{
"id": 184596,
"tgt": "What causes excessive saliva with blood while waking up?",
"src": "Patient: I'm 30 years old, weighing 91kg;165cm tall. I have problem. Every morning when I woke up, my mouth is always full of saliva, mixed with blood..i don't know if its my gum bleeding. Also, i guggled out sticky, coffee grained colour stuff which sometimes contained blood traces in it. This happens only in the morning. After i clean my teeth,it will disappear except for excessive saliva produced. Can anyone help!! Idee Doctor: Thanks for your query, I have gone through your query.the excessive saliva with blood could be because of the gum infection. In case of gum infection or any infection in the oral cavity excessive saliva will be produced. The bleeding or blood could be because of the gum infection.Consult a oral physician and get the teeth cleaned and you maintain oral hygiene, you can use mouth washes. once you clear the infection the excessive saliva and bleeding will stop. I hope my answer will help you, take care."
},
{
"id": 12598,
"tgt": "How many minutes should I apply the ointment for psoriasis treatment ?",
"src": "Patient: iam using halobetesol propionate creaam with combinaton of moyzen for psoriasis treatment.for how many minutes i have to apply to my body iam using halobetasol propionate &salicyclic acid ointmment with combination of moyzen for psoriasis .for how many minutes i have to apply dis creams to my body.my age was 24. Doctor: Hi!Chandrakumar, Welcome to HealthacerMagic forum, these are good creams for psoriasis ..you can use it twice daily on the affected area...and leave it..don't wash it off... take care, Dr.chawda"
},
{
"id": 54909,
"tgt": "What causes enlargement of liver and how to treat it?",
"src": "Patient: my daughter had a gen. exam last Thursday in preparation for her operation next week for tonsils and adenoids...and just today they called me and said that according to the test my daughter has a big liver.....so am worried and want to have infos about it...thanks Doctor: Hi thanks for asking question...Noted you have mentioned big liver.Usually you might have done USG of abdomen and diagnosed with big liver.There are lots of cause that should be evaluated like hepatitis, gall stone, non alcoholic hepatic steatosis , bile duct problem , hemolytic anemia etc...LFT and peripheral smear examination needed with retic count and LDH estimation...So consult doctor for rule out causes of hepatomegaly...If bacterial infection present then first treated with antibiotic...Take care...Dr.Parth"
},
{
"id": 212568,
"tgt": "Have anger outbursts, pulls hair from head and is suspicious. On medication. Alternate treatment?",
"src": "Patient: Hi! I have a friend who is a mother of 2 & had been physically abused by her mum since she was a baby. This friend has been pulling her hair (literally) from her head, brows, lashes & has anger outbursts, is suspicious, fears her husband will leave her etc. She was prescribed Citanew 10mg, Tryptinol 25mg for a week & then 50 mg after that & Tab Dosik 1.5mg. She says that she is not functional now & is always sleepy. Is there something that can be done for her without the meds or should there be other help along with the meds? Doctor: Hi there, thanks for asking. Pulling hair from the head and eyebrow is called trichotillomania and it is often accompanied with anger bursts as you mentioned. Citanew and Dosik are good medications for such a condition which are commonly prescribed by doctors. I am sure that her doctor has prescribed Tryptonil for a special reason. In most of the time it can cause sleepiness during the day, since it remains in the body for long hours. She can talk about this issue with her doctor and she may decrease or discontinue Tryptonil to see if her sleepiness is improved. Her condition may become quite satisfactory after several weeks with just Citanew and Dosik. The response time for trichotillomania (pulling hair) and its related symptoms is longer than depression and is almost like the time needed for obsessive compulsive disorder (about eight weeks). I wish the best health for her."
},
{
"id": 108154,
"tgt": "How can contusion on buttocks and lower back pain be treated?",
"src": "Patient: yes we need advise right away please my boyfriend fell at work has a contusion on his buttocks and pain from lower back all the way down his leg has seen doctors who sent him to a nerologist (cant spell\" but say he has a buldged disk l5 n l2 but they say they can not perscribe pain meds for what ever reason he needs help cant keep going to er need answers fast please Doctor: Dear caring gf He has sustained trauma to his lower back and both buttocks with contusion and this needs to be investigated with Xray of lumbosacral spine anteroposterior and lateral views to rule out bony abnormalities. How he was diagnosed with disc bulge? Was mri of lumbosacral spine done? If mri was done and there was disc bulge at L5 level with nerve root compression , it may be considered the cause for pain all along his lower limb. if Xray is normal you need not to worry. Take rest for 3 days and avoid forward bending and weight lifting. Start tab diclofenac plus thiocolchicoside combination twice a day for pain relief alongwith tab pregabalin 75mg at bedtime. Thus will help in pain and radiculopathy. If Xray report is abnormal you need to consult expert orthopaedic surgeon with report. All the best"
},
{
"id": 53482,
"tgt": "Suggest treatment for abdominal pain and liver hemangioma",
"src": "Patient: Hi, may I answer your health queries right now ? Please type your query here...Had a recent ALT and AST done both elevated ALT was 7 greater than the upper limit and the upper limit of the AST was 36 and mine was 53... Right sided abdominal pain...recent ultrasound for a follow up of a liver hemangioma (1.8cm at largest) showed the lesion stable and gallbladder polyps (no greater than 4mm) stable. GERD and Gastroparesis... On Prevacid 40 year old female... lots of nausea but this is longstanding. Doctor: Hi and welcome to Healthcaremagic. Thank you for your query. I am Dr. Rommstein, I understand your concerns and I will try to help you as much as I can.This hemangioma is very small and this cant cause such symptoms so hemangioma should not be treated if smaller than 10cm. Other findings and symptoms are concerning and there is obviously liver damage which shoul dbe evaluated. it may be viral hepatitis or fatty liver disease, you need to do ultrasound and check viral markers for a start. then further therapy can be planned.I hope I have answered you query. If you have any further questions you can contact us in every time.Kindly regards. Wish you a good health."
},
{
"id": 191003,
"tgt": "what is the main cause of sore throat after denture placement ?",
"src": "Patient: what is the main cause of sore throat after denture placement . Is it because of upper or lower denture? Doctor: Hi, Consult an ENT specialist to identify the cause. The dentures should be cleaned daily using a denture cleansing paste before going to bed. Fungal infection could develop if the dentures are not kept clean."
},
{
"id": 107362,
"tgt": "Is lower right-sided back pain along with stiffness a serious concern?",
"src": "Patient: Hello, I am a 33 year old male who is morbidly obese and who recently started having pain in my lower right side of my back. Pain is sharp and roughly a 7 or 8 on a scale of 1 to 10. It is made worse by trying to stand, move, stretch or move my arm in various ways. The area of pain also feels stiff and tight. I ve taken pain killers and its done very little to help. Is this serious enough to warrant a trip to the ER? Doctor: Hi i am Dr Ahmed Aly thanks for using HealthcareMagic site ,I had gone through your question and understand your concerns .. In my opinion i think you have a kind of sciatica due to nerve compression , sometimes such neuropathic pain subsides simultaneously with rest and even without medications . You may try to avoid heavy lifting , bad sleeping or sitting postures , over weighing , obesity and vigorous movements will be helpful . Physiotherapy sessions with a good chiropractor , spinal straightening exercises , hot massages and even yoga are of very good value in most of cases also decreasing your weight prevents further pressure on your spine and thus lowering the compression on your spinal nerve roots . For my patients i recommend painkillers like advil tabs when needed , hot massaging with topical gels of anti-inflammatory , B12 vitamin supplements , muscle relaxants will be helpful in most of cases , in more severe cases steroid tabs or injections are prescribed . If pain persists after physiotherapy and medications i recommend MRI on spine to exclude disc herniation or any spinal injuries and follow them up with your neurologist for proper evaluation and management . Please click and consider a 5 star rating with some positive feedback if the information was helpful. Wish you good health,Any further clarifications feel free to ask."
},
{
"id": 101754,
"tgt": "What causes cold and flu and breathing problem?",
"src": "Patient: hi, I am suffering from cough and flu cold for more than 20 days, i am healthy person weighing 80 kgs height 59 and i can do lots of exercises without getting tired but these days once i step on to treadmill i can't even jog for 5 mins getting, i get problem of breathing and i struggle for normal breathing as if i got a asthma attack. i never had asthma but had wheezing before 8 years, it got cured after having levoflaxin 500 mgs. now am stuck with extra fat and cant even jog kindly advice me on this . it will be really greatful Doctor: Hi,You need to get evaluated for respiratory system once for asthma or pneumonia or only upper respiratory tract infection as these are mainly responsible for acute onset of illness. So you need evaluation once using xray chest and examination.Till then take antiinflammatory drug like aceclofenac with anti histamine like cetrizine, do regular salt water gargle three times a day and steam inhalation. These will help you.Hope this information helps you. Feel free to ask if any further queries and I will be glad to answer them.Regards"
},
{
"id": 173945,
"tgt": "Suggest treatment to cure ear infection",
"src": "Patient: dr. bhatti - my daughter is 9kg and 17 months. got penicillin resistant ear infection that even spread to her cheeks for which she was given azomax/azythromycin for 5 days. and cipro ear drops. the infection went away but has come back after 2 weeks. what to do\\/ thankyou Doctor: Hi,It seems that there might be having resistant type of ear infection not responding well with some antibiotics.Go for culture and sensitivity test of ear discharge.This test will give specific bacteria and antibiotic most sensitive to these bacteria.After report go for long specific antibiotic medicine course.Afterwards put specific antibiotic ear drops as well.Ok and take care."
},
{
"id": 7472,
"tgt": "Suffering from acne due to tuberculosis, appears on chest, back, buttocks. Medication for treatment?",
"src": "Patient: how to cure acne from tuberculosis . After my 3rd month in taking medicine, acne start to occur in the head then chest then at the back. I already finish my medicine with total of 6 months and its already been almost 2 months that I didn t take those medicine but still acne still appears. And now it started to appear in my butt and arms. Please help me Doctor: Hello, If you are certain the skin lesions are acne then you can use Benzoyl peroxide 5% lotion/cream/gel or salicylic acid cream + oral antibiotics such as erythromycin or cloxacillin till lesions disappear. Avoid taking antibiotics for too long. If no improvement, it means it is probably not acne lesions, then you'll need to see a dermatologist or your doctor for further management. Thank you"
},
{
"id": 115630,
"tgt": "What does \"weak reactive\" in vdrl test indicate?",
"src": "Patient: 1)My vdrl test came back saying Weak Reactive. What does this mean? I got this diagnosis in 1994. I am now 42. Do I have Syphilis?2)Two weeks ago I had an HPV test with my pap smear. It showed positive for Chlamydia but did not show if I had Herpes which I believed I had for years. Could I have thought wrong about the Herpes? Doctor: Hello and welcome to HCM,VDRL test is done for syphilis.A weak positive VDRL test does not prove that you are suffering from syphilis.However, a reactive or weakly reactive VDRL is an indication for confirmatory test which is TPHA test (Treponemal Pallidum hem-agglutination).If TPHA is also positive, then it is confirmed that you are suffering from syphilis.A positive test for Chlamydia shows infection with Chlamydia only.For Herpes simplex infection, you will need to get serological test for herpes simplex infection.Herpes is characterized by multiple painful fluid filled vesicles.In case you have such history, you can get test for herpes simplex infection.Thanks and take careDr Shailja P Wahal"
},
{
"id": 181252,
"tgt": "What causes jaw pain while chewing?",
"src": "Patient: Hello, I started Invisalign about 3 or 4 weeks ago and just a week and a half ago I started experiencing jaw pain on my left side very close to my ear and it hurts while chewing. I'm very nervous because I'm still having pain and I think my jaw may be dislocated. My orthodontist had me stop the Invisalign until the 7th of December to see if my jaw heals...any idea what I can do to help it? Doctor: Hi..Welcome to HEALTHCARE MAGIC..I have gone through your query and can understand your concerns..As per your complain pain in jaw while chewing when you are wearing Invisalign can be due to pressure implicated over the teeth to make them move to come in proper alignment and makes chewing uncomfortable..At times pressure changes can effect the jaw leading to pain in the jaw joint area too, however not necessary that it is dislocated..I would suggest you to consult your treating dentist or an Oral Surgeon and get evaluated and a panoromic x ray will be taken to rule out any discrepancy present with jaw joint as well as all your teeth will be evaluated through it..As of now you can take anti inflammatory painkiller like Ibuprofen along with muscle relaxant like Chlorzoxasone and also take a soft diet..Avoid excessive mouth opening like while yawning..Avoid stress and take care..You can also discuss with me further by sending scan of your Panoromic x ray by asking a question by clicking on my profile and clicking on \"Ask me a question\"I will evaluate your x ray and guide you how to proceed further..Hope this information helps.. Thanks and regards.Dr.Honey Nandwani Arora.."
},
{
"id": 170708,
"tgt": "How to improve an infant s body weight?",
"src": "Patient: Dear Doctor, My baby girl is 8 .5 months . Her birthweight was only 2kgs as i had gestational hypertension. Her current weight is 6.5kg. She is exculsively breastfed & i don t give her cerelac etc..Its all pure home cooked food. What shud i do to improve her weight. Shall i start giving her health mix powders available in the market ? Thanks in advance for your reply. Doctor: Hi! thank you for choosing healthcare magic.Its okay to give home cooked meal. How often do you feed her? do you still have enough breast milk? At 8 months, you should have introduced finger food already. You can feed her 2-3 times a day. How is her appetite? If appetite is good, you don't have to give vitamins or health powder supplement. Continue breastfeeding as long as you feel that there's still adequate supply. If you have a pump, you can quantify the amount that you give her. For her age, she can already consume 8oz per feed. Make a food diary and monitor weight every month. Hope I was able to help you. Regards, Hannalae Dulay-See, M.D.Pediatrician"
},
{
"id": 21241,
"tgt": "Does atrial fibrillation cause shivers?",
"src": "Patient: My brother has A fib and has had it for nearly a year now....procedures to corrct it haven't worked. He has been experiencing severe chills lately....needing to go under an electric blanket to get comfort. No fever. The chills last maybe an hour or two and then disappear. Could this be related to the meds he is on or the A fib?? The chills leave him \"wasted\" and no energy. Doctor: Hi!, Firstly I wanted to know his age.In any case Chills (Shivers) could be due to various reasons, few of which could be Fits (Epilepsy) - If your brother is of younger age, of some sort of infection in body.Yea atrial fibrillation itself could bresent as shivers in case he is having an episode of tachycardia (Fast heart rate) - persiving it to be chills / shivers.Other reason could be if really having A fib, there could be decreased blood circulation to brain - causing generalised weakness and giddiness / tiredness.I would suggest you to consult a Cardiologist at the earliest"
},
{
"id": 108335,
"tgt": "Suggest cure for a lymph node on the back",
"src": "Patient: I am 16 and i feel some pain in my breast and armpit and theres a lymph node slightly enlarged down my backs side of scalp or neck and its really painful and plus i had some throat infection during this month maybe the cause of lymph node my parents are doctors and they say that unmarried girls do not get breast cancer i searched on google about the lymph nodes at back of the neck and it says that the lymph nodes at back of neck signifies breast infection or tumor. please tell me what should i do and which tests should i get and is it true that 16 year old don't get breast cancer Thank you Doctor: Dear patient It's true that at 16 years of age you do not get breast cancer. Lymph node at back of neck may be due to scalp infection or local throat infection and is common in young age. Best thing is to get excision biopsy done. In this procedure involved lymph node is removed under local anesthesia and sent for histopathology examination. It gives correct diagnosis and also removes painful lymph node. All the best."
},
{
"id": 195332,
"tgt": "When will patechiae disappear?",
"src": "Patient: I take coumadin for treatment of pe. I have factor five Leiden. I am 47 years old. I finish my coumadin in March. But last night after sex I developed patechiae everywhere I am not concerned from a health stand point but as I know this can happen. But rather tis is vanity. How long till they disappear? Doctor: Hello and Welcome to \u2018Ask A Doctor\u2019 service. I have reviewed your query and here is my advice. Petechiae means platelet dysfunction. Therefore, check your platelet count immediately. Also check for prothrombin time and INR etc. Until examination is done it is difficult to say what it is. Please consult your physician immediately he will examine and treat you accordingly. Take care."
},
{
"id": 2695,
"tgt": "What are the chances of pregnancy after taking morning after pill after unprotected sex?",
"src": "Patient: I had unprotected sex 3 times within 24 hours, 30 hours the most,but took a morning after pill the next day, what are the changes that I could get pregnant. My partner did not cum in me the 3 times. The first one he did but we were in a pool and as we got out the sperm was on my swim suit bottoms . The other 2 he pulled out but I am not too sure if he got any in or just around the vagina area. I also peed shortly after each time. Doctor: Hi,I will suggest you the best possible treatment options. 1) First of all do not panic. 2 ) As per your history, you had unprotected sex act, and there was penetrative sex also. Spillage of semen inside and around vagina or in case of withdrawal method sperms in the pre-ejaculation secretions can lead to pregnancy. 3) As you have mentioned that you have taken emergency contraceptive pills within 24 to 72 hours of unprotected sex act, it is quite effective in preventing pregnancy. Usually 95% women get successful results with proper use of this method. So do not panic. 4) If at all you do not get withdrawal bleeding after emergency contraceptive pill, please rule out pregnancy by getting urine pregnancy test on morning's first sample of urine or blood beta HCG test 15 days after your unprotected sex act. I hope this answer helps you. Let me know if I can assist you further. Thanks,Dr. Purushottam Neurgaonkar"
},
{
"id": 22226,
"tgt": "Suggest treatment for heart murmur after attacks of SVT",
"src": "Patient: My 19 yr old granddaughter, weight 109, innocent heart murmur since birth , had 1st SVT attack 4 wks ago AT WORK as a CNA in local hospital. This past Fri., she had 2nd attack, again AT WORK. Is returning to cardio this Wed. for results of all tests, incl. echocardiagram. Just began work at this hosp, 11 days BEFORE her SVT attack. Now, ins. say it is/was pre-existing condition and her work ins. won t cover for 18 months. On top of work she had lost, being off 3 days, after 1st attack and 5 days after 2nd, she is now in SEVERE financial distress and her nerves are causing problems. Bites nails, can t sleep, can t eat. Ablation was suggested BUT BUT she has NO insurance. Savings gone. Doesn t even have $$$ to pay the cardio with whom she has appt day after tomorrow. I would give her anything I have, if needed, I would donate a kidney (and, I know she doesn t need a kidney - that is merely example of what I would do for her. I am retired, on SSA. DO NOT HAVE MONEY...She is falling apart in front of my eyes. What do we do? Her dad has been on SSA for 9 years; 2 bypasses, 12 yrs apart, that is why my granddaughter had a THOROUGH cardiac eval shortly after birth (11.07.1991) Doctor: Hi,This is Dr Sameer, cardiologist.I fully understand your concerns about the illness of your granddaughter & the financial issues you are facing.First i would like to tell you that if she is having recurrent attacks of SVT than this murmur will not be innocent & may be associated with underlying heart problem. You need to do at least a echo to rule out serious problems. If they are not there than it is fine.Second, for recurrent SVT's we generally recommend for ablation, but there are many patients who are financially not so sound to undergo that & in them we prescribe medicine which they have to take life long. We prescribe Tab Dilzem SR 90mg starting from once daily dose & than increasing it gradually if required.Hope it was helpful.Thanks"
},
{
"id": 44525,
"tgt": "Brown spotting, positive online test. Am I pregnant?",
"src": "Patient: ive been using fertility friend online an i took a test yesterday morning an it says positive according to the online site im 3 weeks... i have lots of symptons but woke up this morning with brown spotting... everything online says dont be worried but i am ive never been pregnant before .... i just need to know whats going on am i really pregnant? Doctor: hello stacci if you really want to know u are pregnant then do serum beta hCG test all the best bye"
},
{
"id": 175610,
"tgt": "Suggest treatment for big bump on head after an injury",
"src": "Patient: my son fell from his highchair almost a week ago. He didnt have a bump on his head till 2 days after the fall. its almost been a week and he still has a big bump. Is this normal?after the fall we immediately took him to the doctors. She checked him out and said everything was fine. But I am worried about the big bump. Doctor: Hi...this big bump will take 4-6 weeks to settle and the fluid in it to get absorbed by itself. Nothing to worry. This is just subcutaneous edema and will settle by itself.Regards - Dr. Sumanth"
},
{
"id": 188323,
"tgt": "Wisdom tooth growth, softness around gum, ear fullness, tightness of lower lip. Why is there pulsating sensation?",
"src": "Patient: Hello, I think I may have a wisdom tooth growing in the back of my left side. Since then I have notice a soft to touch gum, bitter taste only on that side, ear fullness in my left ear. Also on yesterday, I felt as if my left lip felt tight whenever I smiled..What has prompted me to call this morning, is now, I feel like there is a pulsating sensation/pulling in my lower lip pulling to the side. Please advise if this could all be related. Doctor: Hi,Thank you for asking question on health care magicProbably you are suffering from viral infection of the parotid gland (mumps)it passes off normally within 1-2 weeksTake painrelieving drugs and warm saline garglingHope this answer will serve your purposePlease feel free to ask any more queries if requiredTake careDr.M.V.Subrahmanyam MD;DCHAssociate professor of pediatrics"
},
{
"id": 57920,
"tgt": "What is the reason for having slurred speech, hallucinations, confusion in a 85 year old after laparoscopic cholecystectomy?",
"src": "Patient: My mom is 85, had Lap cholecystectomy done Monday the15th, and was admitted to ICU from Post OP due to Tachy, Hypertensive, then became hypotensive and is now presenting slurred speech, hallucinations, and very confused. She is in a rural hospital I would like her transferred to a bigger facility where there are specialist to evaluate her current status. What do you thing is happening. Doctor: HelloThanks for writing to HCMYour mother is probably suffering from postoperative delirium which is very common among old age patients.She need psychiatric consultation.Anti psychotics like haloperidol and chlorpromazine as well as the second-generation antipsychotics olanzapine and risperidone are equally effective.However other causes like hypoxia should also be evaluated.CT Scan of brain should also be done to confirm other possibilities.Hope i have answered your query.Take CareDr.Indu Bhushan"
},
{
"id": 64182,
"tgt": "What causes small lump on back of head?",
"src": "Patient: Hi! I have a small lump on the back of my head, on the left side right before you get to my hair line. It feels like it s possible it could be an ingrown hair or something cause it feels like I may have a little sore on the lump. It doesn t really hurt to touch it, just a little sore cause I keep messing with it. But I have noticed I have been dizzy and had headaches lately and I wonder of it s causing it... what does it sound like to you? Has me a little worried. Doctor: Hi! Good evening. I am Dr Shareef answering your query. Although it requires a good clinical assessment for reaching at a diagnosis, from your history it seems to be like a sebaceous/dermoid cyst whcih could be there for years together without any problem till it gives rise to infection. And I do not think this lump to be a problem or any serious conditoin. In case it starts giving you trouble, you could go to any hospital and show it to srugeon for the management after the required investigations.I hope this information would help you in discussing with your family physician/treating doctor in further management of your problem. Please do not hesitate to ask in case of any further doubts.Thanks for choosing health care magic to clear doubts on your health problems. I wish you an early recovery. Dr Shareef."
},
{
"id": 171238,
"tgt": "Suggest treatment for vomiting , runny nose and cough in a child",
"src": "Patient: My son (14 months) has been coughing, has runny nose and was vomiting for a few days. The symptoms started 2 weeks ago. When he breathes i can hear a little noise (probably in his chest). This noise goes away when he coughs and re-appears after a little while. The cough and runny nose symptoms are subsiding now but I am wondering if there could be any signs of asthma here. I am concerned regarding asthma because he has eczema already. Doctor: Hi...Thank you for consulting in Health Care magic.Cough and cold are viral 95% of the times in children. For cold you can use anti-allergics like cetirizine and for nose block, saline nasal decongestants will do. Paracetamol can be given in the dose of 15mg/kg/dose (max ceiling dose 500mg) every 4-6th hourly, that too only if fever is more than 100F. I suggest not using combination medicines for fever, especially with Paracetamol.For cold you can use Cetrizine at 0.25mg/kg/dose every 12 hourly for 3 days.For nasal block, plain saline nasal drops will do, every 4-6th hourly to relive nasal congestion.Hope my answer was helpful for you. I am happy to help any time. Further clarifications and consultations on Health care magic are welcome. If you do not have any clarifications, you can close the discussion and rate the answer. Wish your kid good health.Dr. Sumanth MBBS., DCH., DNB (Paed).,"
},
{
"id": 50190,
"tgt": "ISCS done, BP high, breathing problem, creatinine level high. Cause of high creatinine level?",
"src": "Patient: my wife had lscs on last 17th she had bp around 150/90 through out the pregnancy and after lscs her bp comes to normal but after 2 days she has some difficulties like breathing not properly and went to hospital found bp high and creatinine lvl is 2.9 on next day 3.9 and today 4.1 wat will be the cause of high creatinine level at the time of pregnency kidny function test was done and is satisfactory Doctor: Your wife seems to have gone into Ecclampsia related kidney injury.Other cause could be injury to yhe ureters during surgery.I advise you consult a Nephrologist immediately to manage her condition."
},
{
"id": 134871,
"tgt": "Suggest remedy for skin darkness and soreness above elbow",
"src": "Patient: Hi. After a long day as a cashier I get darkened skin and soreness in the region of skin above and below the eye brow and to the side of my eye sockets in my temple, Is this fybromyalgia as I get this I get this in my arms and shoulders if excessive use. Doctor: Hi Dear,Welcome to HCM.Understanding your concern. As per your query you have symptoms of skin darkness and soreness above elbow which is mainly due to hematoma , folliculitis , infection or joint injury . It can also be due to pots inflammatory inflammation hyper pigmentation . I would suggest you to start with application of clindamycin gel on these areas.Apply cold compresses and wash it with cold water to prevent the accumulation of oil ,dust and bacteria on your skin. You should apply topical antibacterial ointment such as 2 % betnovate. Apply vitamin E oil at night and kojic acid cream. Consult dermatologist for proper examinarion and treatment . Doctor may also refer you to orthopedic surgeon for bone and joint problem . Doctor may drain the hematoma or prescribe anticoagulant . Hope your concern has been resolved.Get Well soon.Best Wishes,Dr. Harry Maheshwari"
},
{
"id": 149789,
"tgt": "History of hepatitis E, kidney and liver failure. Have disc problem, memory loss. Worried",
"src": "Patient: Hi sir I m atlas khan. Age 28 years ,hight 6 feet, weight 90kgRespected sir in 2005 I got happititus E My kidneys and lever got failed and was in comma on vent for one month. After it I recovered it now 2013 by the grace of God my all reports kidney lever are normal. I have disc problem as wellWell I want to disscous with u about memory loss. Mine memory is not working that much good I met people n after a couple of days I dnt remember them and during discussion when some one interrupt me I forget what ever I was talking or what was the topic. I m worry bec in this age it's not good sighn. So plz guide me. ThanksAtlas khan YYYY@YYYY Doctor: Hi, as you are in coma for one month it may take some time for recovery, It might be due to some changes in the brain during, coma and hepatitis. I advise you to consult a neurologist for diagnosis and treatment. You may need to have a recent M.R.I. to know the position of the brain. Thank you."
},
{
"id": 19690,
"tgt": "Can being overweight be related to palpitation and dizziness?",
"src": "Patient: I had a palpitation yesterday and then felt very very dizzy. It lasted all of 5 sec. I went to urgent care. They took my BP (167/94) and my heart rate was 101. I am on Lisinopril/HCTZ (20 mg/12.5). My BP last month was 128/78 and my heart rate is usually in the 70's. The doctor said the EKG looked ok but my heart was beating a little fast. He said I should follow up with a cardiologist, which I am going to do. He said it was difficult to determine what happened. I don't smoke, I am overweight, I drink very little caffeine. Any ideas? Doctor: Hello!Welcome and thank you for asking on HCM!I passed carefully through your question and would explain that your symptoms could be could be related to a cardiac arrhythmia. Of course, being overweight, drinking coffee and smoking can trigger this clinical situation too. But, I would recommend performing some tests to investigate for cardiac arrhythmia, before concluding that nothing serious is going on: - a cardiac ultrasound- an ambulatory 24-48 hours ECG monitoring- thyroid hormone levels for thyroid dysfunction- complete blood count for anemia- blood electrolytes for possible electrolyte imbalance. If all the above tests result normal, there is nothing to worry about. Reducing caffeine intake and avoid smoking will improve your situation too. Hope you will find this answer helpful!Kind regards, Dr. Iliri"
},
{
"id": 141118,
"tgt": "What do hand tremors lasting for hours indicate?",
"src": "Patient: my hands have been shaky lately it starts when I wake and can last for hours. I have a problems getting my self together and don t feel like myself. My mother died after suffering from strokes, many years in a wheelchair before dying from cancer. I rarely drink, a glass of wine occasionally. I have gestational diabetes 25 years ago. other problem my thyroid has been enlarged for many years 10 haven t seen a doc in several years. Have problems getting any decent sleep, problems with night terrors at times. suspect that my hands also suffer from psoriatic? arthritis diognoist with a heart problem nitrovalve regurgitation not prolapse.. And yes I will be making an appointment my husband will make sure of it. what can the shaky hands be about cant anyone answer this question I only make enough money to pay part of my rent and my partner works on and off pt with copd didn;t realize money was required have none diane kennedy YYYY@YYYY Doctor: Hi, There are thousands of reasons for tremors like seen in Parkinson DS or cerebellar disorder. Vitamin B12 deficiency is not a common reason. Hence, you need to tell more about your symptoms to get it correctly diagnosed. Hope I have answered your query. Let me know if I can assist you further."
},
{
"id": 188030,
"tgt": "Why are mouth ulcers persistent despite having a steroid treatment?",
"src": "Patient: Hi Doctor. My name is Imran , age 45. I am suffering for mouth ulser from last 2 months and it is very painful . I consulted couple of Doctors including an ENT and a specialist of Internal Medicine . They gave me anto acidity tablets to control reflex and steroids that was helpful to cure upto some extent. Now it started again. Can you please guide me how to get rid of it and with which specialist I should consult for the treatment.Thanks Imran Doctor: Hello,Actually there are different types of mouth ulcers, all of them are not treated with steroids.I think you should consult an oral medicine specialist as he will examine the ulcers thoroughly and then give you medication, and even he will go for biopsy if needed for the ulcers.Till then you take some topical ointments such as mucopain or pansoral to apply on the ulcers and some lycopene containing multivitamines.Hope it will help you."
},
{
"id": 74741,
"tgt": "What is the treatment for pulmonary consolidation?",
"src": "Patient: My mom is diagnosed with pulmonary consolidation with sepsis and her urine out put is less. AdmItted in AMC last night with complaints of fever and chest pain and bbreathing difficulty. Her blood pressure is dropping as well according to doctors in hyderabad. The doctors says maojr organs can be affectedd. What can be the management and recovary rate?? Doctor: Hello Welcome to HCM. Lung consolidation means infection of thelungd and sepsis means invasion of blood with bacteria. Since the bacteria comes into the blood, it spreads to other organs and can cause urine tract infections., brain and other main organs. This causes the drop in blood pressure. The treatment is iv fluids to raise the pressure, and to start on BP increasing medications which are given intravenously. Also intravenous broad spectrum antibiotics should be added. Blood culture, urine culture and sputum culture should be done. The culture will help identify the oragnosm causing the infection and also show the antiobiotics that are effective against it. Diuretics should be given to increase the urine output and must be closely monitored. If organism cannot be identified by culture and the patient is not able to bring out cough, then a Bronchoscopy must be done which involves inserting a small tube with camera at its tip through the nose into the airways and take samples from the area affected. This sample can be sent for culture. Antibiotics can be modified according to the culture reports.The infection may also affect the functions of the kidney and cause kidney issues. The outcome is hard to comment on and the recovery may take hospitalisation up to 10 days and rest till one month. Wishing her good health Regards."
},
{
"id": 225428,
"tgt": "Shifted from Triphasil to Qlaira due to heavy bleeding. Have low iron level. Have breakthrough bleeding",
"src": "Patient: Hi, I'm 32 years old and used Triphasil previously for the past 7 years I haven't been on any kind of contraceptive but a month ago the Dr started me on Qlaira because of heavy bleeding I also do have cysts and my iron levels are currently low at 4mmol instead of 9mmol my HB however is 11. I had my periods and bleeding was much less on the pill but now I'm experiencing breakthrough bleeding not heavy but moderate and bright red, what could be the problem? Doctor: Hello and Welcome to \u2018Ask A Doctor\u2019 service. I have reviewed your query and here is my advice.Qlaira is also a contraceptive pill containing hormones in it. Breakthrough bleeding and spotting are common in initial few months. Not to worry about this. Continue your contraceptives regularly. You need to consult a doctor if your symptoms persist. You may need to get PAP smear and endometrial study tests done. Hope I have answered your query. Let me know if I can assist you further.Regards, Dr. Vidya KR"
},
{
"id": 207420,
"tgt": "Suggest remedy for depression and borderline personality disorder",
"src": "Patient: hi. i trough out everything in 1.5 days. i think i have fever and pschylogically i am depreesed. i need to cry and feel afraid to have social contact and need somebody to help me at the same time. last night i was thinking hurting my self.i am few sessions with a doctor who told me i have borderline personality disorder. i was fine. i cant sleep now and feel troughing out in last 3 4 hours ... help me Doctor: A face to face session with the doctor who has been treating you could be helpful. As he already knows about your history details. Sleep disturbances could be because of some disturbing thoughts or others. It needs to be clarified."
},
{
"id": 125201,
"tgt": "What could cause bumps on big toe?",
"src": "Patient: Hi. I have noticed lately that I have these bumps on my big toe. It happened when I started using the pool at the Y, but didn t notice them until I looked while drying my feet. My Dad thought it was just from the water getting under dead skin (I did notice some dead skin coming off my toes after swimming), but they never itched or hurt so I paid them no mind. I looked again a couple of days ago and they re still there and I m going a bump or two on my adjacent toes. Some of the bumps on the big toe have black dots in them. Is this just a bad case of warts or a fungus? Or something else? Doctor: Hello, Most probably it will be a wart. Once the diagnosis is confirmed you can go for cauterisation or cryosurgery. Consult a dermatologist and he will direct you accordingly. Hope I have answered your query. Let me know if I can assist you further. Take care Regards, Dr Shinas Hussain, General & Family Physician"
},
{
"id": 112737,
"tgt": "Have pain back, near liver, below sternum and shoulders. All tests done. Taking antibiotics. What can this be?",
"src": "Patient: I've been in hospital twice during the last 3 weeks... first they thought it was galstones or blood clot through my lung. It started with pain in my lower back... felt like the start of labour... then it moved to the front over my liver and I had some stabbing pain too.... the to just below my sternum (very sore when I breath in ).... then I also had pain in my right shoulder and up the right ride of my neck. They could not find blood clots or galstones.... my bloods seemed normal except for CRP 92 and D-dimer 663 and after the third test they told me my liver function was slightly off...Bili 12, ALP102,GGT43,ALT61, CRP 11. By week two the pain was much less and they send me home with antibiotics. Now another week on.... I feel like something get stuck under my ribs - feels like when a baby pushes with his feet against it (not sore, but pressure).... I also get a feeling of movement in my stomach from time to time... again like when you are pregnant... I am not!!! Had my period a week ago.... and I am 46. They also found a haemangioma in my liver during the scan, but said it was small and could not have caused the pain (1-2cm). I am worried about this... something is not right. Doctor: Hello, Welcome at HCM.I have read details of your question.As per your details ,it looks to be a pain due to gall bladder problem.There can be inflammation of gall bladder which is not detected by other investigations.At present you should have HIDA scan.This is a test which is able to detect the dysfunction of gall bladder,which may be the reason for your pain.Hope this will help you.With best wishes."
},
{
"id": 161368,
"tgt": "What does pus cells (60-80) and some epithelial cells in urine indicate?",
"src": "Patient: hi, my 8yr old daughter tested pus cells (60-80) and some epithelial cells in urine. what does it mean? Also, she has an allergy like reaction allover her body. She is always often coughing, she gets treatment, gets ok, but it never really goes aways. What could it be? Doctor: Hi, This means there are two possibilities. 1. Allergy or inflammation-induced sterile pyuria where there will not be any urinary infection but there will be plenty of pus cells in the urine. 2. Urinary tract infection. If there is no fever or stomach pain with chills and rigors you can wait. But if there is fever she will require urine culture and butter to start on antibiotics, please. Hope I have answered your query. Let me know if I can assist you further. Take care Regards, Dr Sumanth Amperayani, Pediatrician, Pulmonology"
},
{
"id": 166490,
"tgt": "Does having chest congestion cause white patches on eyelids?",
"src": "Patient: hi doc, my son is 4yrs old and he often gets a lower chest conjestion and stuffy nose. due to this i can see that both his eyelids have started discoloring that is a white patch on both the eyelids r seen. i really don t understand why this has happened, what is it. is it only due to the cold, and conjestion that this discoloration has happened or is it some skin disease that is forming. can u plse help and guide me? Doctor: Dear user,Unfortunately, it is impossible to say what this might be without the benefit of actually seeing it."
},
{
"id": 157907,
"tgt": "Hypothyroid under control, have pain around thyroid with popping and fizzing sound, pain below chin, swallowing difficulty. Cancer sign?",
"src": "Patient: Hi. I have hypothyroid . I have been on Armour thyroid for some time. My numbers were o.k.back in February, but I have not test since then. In the past couple of weeks I have had major pain in and around my thyroid, actually extends up the left side of my throat. Really hurts right below my chin . It is painful to swallow and feels like when I do, everything is going to the right side. I am also hearing a popping and fizzing noise that seems to be around my thyroid. I had a blood test on Friday, but have gotten no results yet. Is it possible it is cancer? Would the blood test show anything or should I ask for an ultrasound? Doctor: The symptoms that you have related dont seem to be related to your thyroid disorder. Rather, the problem seems to be within your throat. It could be a simple infection or it could be something sinister. Your symptoms are common to both cancer and acute infections. If your symptoms have been there for sometime now or if you are a smoker/tobacco chewer/alcohol abuser then i suggest you visit an ENT specialist or a cancer surgeon and get an endoscopy done (laryngoscopy). If not you can take a course of antibiotics for a week (Tab Ciprofloxacin 500mg twice daily + Tab fexofenadine 120mg twice daily). Also take lots of water and do warm saline gargles twice daily. If the symptoms subside then alright else get the endoscopy done."
},
{
"id": 210021,
"tgt": "Suggest remedy for depression while being on clonapin and prozac",
"src": "Patient: i have been on prozac for over 30 yrs and for the longest time i feel it is no longer working for me my dr added clonapin and has been changing around the dose of that lately i feel extremely depressed and dont care about things i used to i am not happy at all i have treid making changes i thought would help but they did not i dont believe my meds r working anymore Doctor: HiThanks for using healthcare magicI think, now you need another antidepressant with prozac. You can try venlafaxine or mirtazapine. Both are from different group and help you to get better quality of life ahead. Rest you can discuss with your psychiatrist. In case, you need further help, you can ask.Thanks"
},
{
"id": 152779,
"tgt": "What does 1-86 kappa lambda ratio indicate?Suggest further course of action",
"src": "Patient: Dear Dr. Indranil Ghosh, As per advise of doctors in medanta hospital and as suggested by you, We ve got 4th cycle of chemotherapy (Bort-Dex) completed. After which doctor prescribed for tests Serum free light chain, CFC & KFT. I ve attached the report. Please have a look. Last Serum free light chain was done 1 month back after 3rd cycle which showed Kappa\\Lambda ratio to be normal (0.88). But this time after 4th cycle kappa lambda ratio is increased to 1.86. Doctor here will be putting her on maintainace after reviewing the reports. Please suggest what does the current report indicate and further course of action? Thanks, Bhuwan Grover Doctor: HiSerum free light chain (SFLC) testing is ordered to help detect, diagnose, and monitor plasma cell disorders (dyscrasias),Light chains are proteins produced by plasma cells. They are used to assemble immunoglobulins (Ig), antibodies that target and neutralize specific threats to the body such as bacteria and viruses. Two light chains and two heavy chains combine to form an immunoglobulin,With a group of conditions called plasma cell disorders or monoclonal gammopathies, a plasma cell becomes malignant, divides uncontrollably, and produces large amounts of an abnormal monoclonal immunoglobulin (M-protein). This protein may take the form of an intact immunoglobulin or of one of its component parts \u2013 a kappa or lambda light chain,Free light chains will normally be present in the blood at low levels, with a kappa/lambda ratio of approximately 0.26 to 1.65.right now its 1.86 which is elevated so follow the treatment as suggested by oncologist and attach all details for further comments"
},
{
"id": 126655,
"tgt": "How can severe pain in the shoulder be treated?",
"src": "Patient: i recently painted the inside of our home and hurt my shoulder. it got well and then ive been gardening and swinging a sledge hammer and its out again. i cant move backwards and sometimes cant move in front of me .i find it hard to hold heaver things also. in pain all time. Doctor: Hi, If pain is continuous day and night and range of motion is painful reason may be rotator cuff tear in shoulder. I would like to advise: 1. X-ray of involved shoulder anteroposterior and axillary views to rule out bony abnormalities. 2. MRI of involved shoulder to check condition of rotator cuff muscles. please get it done from radiology center nearby you. Meanwhile keep shoulder splinted in shoulder immobilizer brace. Take tablet Ultracet P twice a day for pain relief. Visit to orthopedic surgeon with reports is recommended. Hope I have answered your query. Let me know if I can assist you further. Regards, Dr. Jayesh Vaza, Orthopedic Surgeon"
},
{
"id": 48820,
"tgt": "Suggest treatment for kidney stone",
"src": "Patient: HI , i am sunil my cousin is having a minimal fulness of right pelvicaycal, and kidney also enlarged due to this, he is suffering from pain, vomitings and giddiness. and doctors find a stone in initial stage and that is at the edge of the kidney. so i kindly request you to suggest the treatment. and tell me about the risk.thank q Doctor: Hi,Thanks for writing in.It is important to know the exact location of the stone. If the stone is causing obstruction to the flow of urine from kidney to bladder and is stuck in the ureter then attempts must be made to clear it from its present location. If it is a small stone less than 6 mm then it can come down by itself following drinking of large amounts of fluids. If it is more than 7 mm then it may have to be broken into smaller parts and then flushed down the ureter. Even if it is a small stone, while passing down the ureter it an cause severe pain and vomiting. He must continue taking fluids and trying to flush the stone."
},
{
"id": 184957,
"tgt": "Suggest treatment for toothache and pain in gums",
"src": "Patient: Help my husband has severe toothache. The tooth has a hole on it. He also complained about pain in the gum of a previously removed tooth. The tooth was removed about 8 years ago. Why is he feeling the pains now. Please what antibiotics and pain reliever do you prescribe that will be effective. Ge doesn't sleep at night because of the pain. He just started taking 400mg of flagyl 3 times daily for two days now,but it seems not to be working Doctor: Hi thereThanks for asking your queryAccording to history provided your husband should consult a dentist for complete recoveryEither he need root canal therapy or he has to go for extractionMedicines will give only temporary relief, Thanks"
},
{
"id": 99445,
"tgt": "Could outbreak of hives be due to Mucinex?",
"src": "Patient: I've had a rash (more so hives) every day for the last 2 weeks. It started with a bad sore throat and earache on September 28th. Then, I had a lot of drainage and congestion. I started taking Mucinex DM and then broke out in hives. I thought the Mucinex might be causing the hives, so I stopped taking that. Since the sore throat and earache was not going away, I went to my doctor last Monday. He said I \"might\" have strep throat, but didn't test me for it. He wrote Pharnygitis on my doctor's note for work and put me on an antibiotic (Amoxicillan 500 MG capsules to be taken 3 times a day). I've been taking that, but the hives keep appearing every day. Not sure if the hives are an allergic reaction to both medicines or if it is just a part of this viral infection. I have to take Benedryl every night to get relief from the itching. Do you know what else could be causing the hives? I am broken out all over from scratching. Didn't know if you knew of a specific virus going around that is causing hives? Doctor: Hello,Thank you for asking at HCM.I went through your history and would like to make suggestions for you as follows:1. As your hives started after taking Mucinex DM while having a sore throat with earache, I would think of either the infection itself or Mucinex DM.2. Although amoxycillin is a very common cause of hives, I would not think of amoxicillin you had hives even before taking amoxycillin.3. At this moment, it is difficult to find the exact cause of hives, it could be Mucinex DM, viral infection or strep infection. As you have already started amoxycillin, throat culture will not be helpful. There are many possible viruses causing such symptoms with hives, so I usually do not advise my such patients to go for testing for viruses, especially because there are no antiviral medicines for most of the viruses.4. Were I treating you, I would suggest you to complete the course of Amoxycillin and take an antihistamine like levocetirizine or cetirizine in morning and diphenhydramine (Bendadryl that you are already taking, may cause sedation) before going to bed at least for 7-10 days. If hives return after that, I would investigate my such patient for other causes of hives.5. I would also suggest you to apply calamine lotion over the itchy areas. Please do not scratch the itchy areas as scratching would aggravate itching.Hope above suggestions will be helpful to you.Should you have any further query, please feel free to ask at HCM.Wish you a very quick recovery and a very good health ahead.Thank you & Regards."
},
{
"id": 79440,
"tgt": "IS the life span if having a interstitial lung disease?",
"src": "Patient: Good evening sir,My father is 45 years old ,as undergone a CT scan, it was advised by the doctor.The report stated that he has interstitial lung disease.The doctor is threatening us that the life span is only 10 years more. Is it true sirwhat action shall we take to save my father Doctor: Thanks for your question on Health Care Magic. I can understand your father's situation and problem. Interstial lung disease (ILD) is very broad term. It is actually group of diseases. Amongst ILD, IPF (idiopathic Pulmonary fibrosis) is the most common amongst ILD. And it is having worst prognosis amongst all ILD. So first confirm with your doctor whether he is having IPF or not. Prognosis of ILD is also depends on severity on PFT (Pulmonary Function Test). So also get done PFT. Newer drugs like perfinodone, N acetyl cysteine (NAC), Picnovit etc are giving good prognosis in ILD. Perfinodone is the only US FDA approved antibiotic drug. NAC and picovit are proved antioxidants which improves lung inflammation. Chest physiotherapy and deep breathing exercises are also useful. So first know the type of ILD and discuss about these drugs with pulmonologist. With recent advances, prognosis of ILD is more than 10 years. Hope I have solved your query. Wishing good health to your father. Thanks."
},
{
"id": 208344,
"tgt": "How to convince patient for good diet and exercise?",
"src": "Patient: hi, my father-in-law(FIL) is a dialysis patient, he is not co-operating for medicines,exercise and diet, he says he will be all right if we leave him as his wish, he is having wheezing problem now and then but he wants to drink all juices, ice soda etc., he do all things just opposite to what we are saying, he is taking everything as we are ruling him, and he says that he is the head of the house and we all should do according to his wish, he is a retired ITI principal, he often mention that and wont do anything we are saying. Doctor: Hi there, thanks for posting your query on this forum.You are a very caring son in law indeed seen from your concern about his health.Your father in law is suffering from Chronic Renal Failure (CRF) and probably Bronchial Asthma as well. Mostly in people who have retired or who have been diagnosed with serious illnesses it is seen that their mental state changes which could lead to such behavior.You can't change his behavior but you can only make him aware of the possible outcome if he is not co-operating for medicines, exercise & diet. Consult a Homeopathic Doctor to improve his mental state.Regards,Dr. Kunal Lokare"
},
{
"id": 81708,
"tgt": "What causes severe cough and breathing problems with ESR value of 110?",
"src": "Patient: Hi,I had severe cough and breathing problem associated normally when strenous work like climbing the stairs etc., is done, so got examined, was prescribed medicines like l-cin c montair lc and ascoril and also got a blood test and chest xray pa view done, both the blood test and chest xray were normal, however, the ESR value returned with 110, is there anything to be worried i am 40 years. Doctor: I think you have asthma as your severe coughing and breathlessness suggest.Xray chest in asthma and blood tests both would be normal, However to confirm your asthma you need to get a spirometry test done.Esr 110 can be a sign of inflammation but per se it doesnot suggest anything.See a good pulmonologist and you should be on proper treatment"
},
{
"id": 212702,
"tgt": "Numbness in the hand and foot while having stress. Reason?",
"src": "Patient: I am occasionally feeling numbness in my left hand & foot , also it happens for a couple of minutes & post this I feel pain in left, It has happened thrice so far & usually occurred when ever I had any stress like working on excel sheets or thinking deeply. Please suggest what it could be. Also my Hb is too low @ 7.7, I am a female, age 26 Doctor: Hi Shalini, Thanks for writing your query. Numbness and pain in your hand and foot can be due to decreased blood supply to that part. In your case this may be due to low hemoglobin. You should take measures to increase your hemoglobion. Eat green leafy vegetables, jaggery, pomergranate, beet root in addition to fresh fruits and vegetables. Also get a dose of de-worming done as parasitic infestation in the body can also result in low hemoglobin. Taking iron preparation under medical supervision wil also be beneficial. You should get yourself examined by a Gynaecologist, if you are having more bleeding during mensturations. Also get your urine examined for presence of any RBC. I hope this is helpful to you. Thanks."
},
{
"id": 47810,
"tgt": "What causes thumping and throbbing pain in flank other than kidney stones?",
"src": "Patient: I ve had problems with kidney stones for about 3 months. I ve passed three stones with great difficulty. I still have 2 stones in my right kidney that are to large to pass. My urologist said that the stones should not be causing me any pain, but I have a thumping, throbbing pain in my right flank. My urologist said that Lithotripsy would have to be done so the stone could pass. He explained that this could be very painful, but that I should be having no pain at this time. If not kidney stones, what should we be looking for? I have been an RN for many years. I am not trying to self diagnose, but I still believe that the pain is kidney stone related. HELP! Doctor: Hi,Thanks for writing in.The kidney is connected to the urinary bladder through the ureters. It is possible that there might be small stones in the ureter which are undetected. At time even after passage of stones from the kidney and through the ureter there is mild injury to the wall of ureter and this causes pain and discomfort. Please discuss with your doctor and take prescription for an alpha blocker and urine alkalizer to help reduce the acidity of urine to prevent more stone formation. The alpha blocker will relax the smooth muscle and treat symptoms of spasm due to any problem in the ureter. Please do not worry."
},
{
"id": 11465,
"tgt": "How to treat dark spots on face, neck, hands and back?",
"src": "Patient: hello dr ,i am having some dark spots or patches all over my face ,neck,hands,&back on consulting skin specailist,he recommended me CELIN 500mg TABS ONCE DAILY 90 days , Sunban lotion every morning ,Cleavg cream at bedtime for 90days. just two days before i had started...iam worring vry much because i had done so many treatments on my skin but not get any positive results . will this much treatment is sufficient for my skin ?can i get my orginal skin colour back ,will it work ?how much time will it take ? Doctor: As you saidm you have got so many treatments then probably you would have developed PIH, post inflammatory hyper pigmentation, and this is curable and temporary .The best solution for this problem is just to wait and let your skin recover, n let d new cells come up. the treatment given to you is fine, continue with that for at-least 2 months to see any results."
},
{
"id": 11965,
"tgt": "Can any online doctor solve the problem of post inflamatory hyperpigmantation and brown spots on forehead ?",
"src": "Patient: I need to consult best doctor in dermatology section mainly who is best for cosmetology ..I have problem of post inflamatory hyperpigmantation and brown spot on forhead.. Doctor: Hi...Biswajitji..., Thanks for choosing HCM., POSTINFLAMMATORY PIGMENTATION..., first try for medical therapy...., 1) BAN A TAN...curatio company 2 times daily..., 2) Tab Lycopene...daily for 30 days..., not responding...go for..., 3) Chemical peeling.....or 4) Dermabrasion...with Microderm abrador..., 5) Last but not least and too best is LASER...... but expensive.....DIODE LASER...long pulsed 810 nm is widely used....,"
},
{
"id": 147440,
"tgt": "Child had epilepsy. Cured after given encorate. Started severing and shaking teeth. Is it epilepsy mild attack?",
"src": "Patient: hello sir , my son is 4 years old now he had two attcks of epilepsy at the age of 2 and 2 year 3 months and since then we gave him encorate twice a day for 1.5 yers we discontinued it 15 days abck as per doctors advise but unfortunately last night my son woke up and was shivering and his teeth we also shaking ..................it was a night mare or is it epilepsy mild attack............pls suggest wht we do ? Doctor: In my opinion the withdrawal from the drug should have been tried after a seizure free interval of at least 3 years.even after this the withdrawal should be in gradually tapering doses each at an interval of 6 months.It may take upto 2 years more after a seizure free interval of 3 years."
},
{
"id": 136774,
"tgt": "What causes pain in groin area and down to foot?",
"src": "Patient: I have pain in my left thigh in the groin area it hurts down the inner thigh to left side if the knee and down to the foot when the pain started it woke me from sleep excruciating pain that was about a month ago and has gotten progressively worse Underneath my foot at times, my toes and the ball of my foot is white like there is not any circulation any suggestions Doctor: Hi,Thanks for your query.Possibility of compression of nerve root in spine, leading to pain in your thighs radiating to feet.I will advise you to do MRI spineI advise my patient\u2019s analgesic and neurotropic medication like pregabalin, you can consult to your treating doctor and do required investigation.Till time, avoid lifting weights, Sit with support to back. You can consult physiotherapist for help.Physiotherapy like ultrasound and interferential therapy will give quick relief.Another possibility of lumbar canal stenosis which will be ruled out on MRI.I do hope that you have found something helpful and I will be glad to answer any further query.Take care"
},
{
"id": 43782,
"tgt": "Unable to impregnate partner, burning sensation on urinating. Treatment for infertility?",
"src": "Patient: I seem to be not making my partner pregnant we intimate almost every day but nothing happens,I had std s few years back I treated them n completed the treatment but after one or two months I still feel the same(burning urine ) and this has been going on for a while now,any help I can get maybe to clean my internal organs and produce Doctor: Hi, thanks for your query. I suggest you get semen analysis done to check if your sperms are fine. You also get HSG test done for your partner. These 2 tests are simple and basic. They will help in knowing the problem of infertility in majority of people. For burning urine, please get urine culture test done. Take antibiotics based on the report. Regards, Dr. Mahesh Koregol"
},
{
"id": 176144,
"tgt": "Why does my toddler have high fever?",
"src": "Patient: My 4 year old daughter has had a severe headache, on the top of her head she says, has been spiking fevers to 105 degrees and won t drink water, juice anything to help me reduce the fever. She does not have strep, negative test yesterday, no sore throat, runny nose or painful ears and we ve had her on alternate courses of generic tylenol and motrin which brings the fever down to 101 or 102 but up it goes again and the headache is almost always there. This had been happening since the wee hours of Wednesday morning and I don t know what else to do. Any suggestions? Doctor: Fever with headache could be caused by falciparum malaria or some bacterial infection affecting the nervous system (meningitis) - both of which could be serious. Although viral fevers could also present with high fever, body ache and headache, the serious conditions need to be ruled out first and I would strongly recommend that you visit a pediatrician immediately.As of now tepid sponging and tylenol will help in reducing the temperature. Meftal P is more effective than tylenol but is a prescription medicine and I would suggest that you get it prescribed by your pediatrician."
},
{
"id": 43449,
"tgt": "History of miscarriage. Irregular periods. Unable to conceive",
"src": "Patient: hi, i have taken yamini ls for 1 and half year to stop pregnancy after birth of my first child. i got a miscarries prior to birth of my first child. the cause was thoyroid and herpis virus. after taking 6 month ocuvir the pregnancy was concieved. my periods were irregular by 5 days up each month. after first child we tried for pregnancy. since then one and half year has elasped. we are facing a grief period. please advise. Doctor: Hi,Thanks for your query. I read your query and I understand your concerns. Following is my reply:1) Get Semen analysis of your husband to know sperm parameters.2) Please get basic tests for your self as follows: On day 2 of cycle get FSH, LH, AMH. Get Follicular scan for antral follicle count.Get HSG ( Hysterosalpingogram ) to test patency of fallopian tubes.I hope I answered your query. I will be available for any follow up query you have.Regards,Dr. Mahesh KoregolIVF & Infertility Specialist"
},
{
"id": 170046,
"tgt": "What causes vomiting in a 6 month old after starting solid food?",
"src": "Patient: hi my baby is 6 n a half month old...i hav started giving him solid food.since last 1 month he was crying at night n not sleeping. though he was eating evrythin n pooping properly so i nvr realised wat d problem is...but den since last 2 days he started vomiting watevr he was eating...den i gav him pedialyite...n den 2day half strength formula...sugesst me wat 2 do now n wen 2 start his solid food n how? Doctor: It is common problem in children of this age group to vomit, as there is transition of food from liquid consistency to solid, Generally once child completes 6 months, weaning is started with complimentary foods. Initially it should be semi solid consistency and gradually increase the consistency depending on tolerance or palatability. If child develops vomiting after having a particular food, just withhold that food temporarily, continue with previous food which child was tolerating. Presently Start your child with an anti emetic medication domstal drops / Ondem Syrup orally according to baby weight. Withhold oral intake till 15 - 20 minutes of giving medication. If still not controlled better to consult your Paediatrician for an Injection. Which will solve the problem"
},
{
"id": 147610,
"tgt": "What are the chances of recovery after seizure leading poor speech?",
"src": "Patient: Hi, My Dad, 89 years old had a seizure and his speech is not understandable and he sleeps all the time in the hospital. He s not diagnosed as having a stroke. What are the chances for his recovery? I realize his age is against him and each person is different. He has a pacemaker but no other issues. Doctor: Hi,Thank you for posting your query.I have noted your father's symptoms. It is not clear why he sleeps most of the day.A single seizure should not lead to excessive sleepiness. We need to look at his anti-seizure medications. Some of them can lead to sleepiness. Also, he needs to be stimulated, made to watch TV, talk to relatives, sit up, etc. Hospital stay, especially in a dark room away from sunlight could contribute to his sleepiness.I hope my answer helps. Please get back if you have any follow up queries or if you require any additional information.Wishing you good health,Dr Sudhir Kumar MD (Internal Medicine), DM (Neurology)Senior Consultant NeurologistApollo Hospitals, Hyderabad, IndiaClick on this link to ask me a DIRECT QUERY: http://bit.ly/Dr-Sudhir-kumarMy BLOG: http://bestneurodoctor.blogspot.in"
},
{
"id": 153502,
"tgt": "Suggest care to be taken for cancer patients",
"src": "Patient: Hello,although I do not have Cancer myself,I was wondering if you could offer some advice on treating Cancer patients in general,using Complementary therapies such as body massage.I work at a Womens Health Group in the UK and find that many therapists are reluctant to treat Cancer patients due to the fact that they are unsure whether or not the Cancer could be spread throughout the body by using therapies.I would be grateful if you could let me know what the current thoughts/research are on this,as obviously,our aim is to help people to feel better and do no harm.Thankyou,Yours faithfully,Mrs G.Watson. Doctor: Hi,Thanks for writing in.Cancer is a condition which arises due to abnormal proliferation of cells in any organ system of the body. This might have origins in certain viruses but cancer as a condition is not spread by body massage. It is true that any person might harbor an infection so massage therapist have to take precautions when giving massage to any person irrespective of whether thy have a disease.Cancer patients are as good candidates for body massage as compared to the general population. General precautions are to be taken for any person getting a massage, whether or not they have cancer. I hope your organization provides services to cancer patients just the way they serve the general population maintaining basic hygienic practice. Please do not worry."
},
{
"id": 79385,
"tgt": "Suggest treatment for mild bibasilar fibrosis",
"src": "Patient: I have been diagnosed with Mild Bibasilar Fibrosis. I do not have trouble breathing as the information indicates is a symptom but I have a deep and continuous cough that sometimes produces phlem. I would like to know what can be done to get rid of the cough. Thank you Doctor: Thanks for your question on Health Care Magic. I can understand your concern. Mild Bibasilar fibrosis can be early sign of interstitial lung disease (ILD). It is also seen in smokers and patients with previous history of tuberculosis or pneumonia. So if you are smoker than you should quit smoking as soon as possible. For coughing, you should take antihistamine drugs. You should also start newer antibiotic drug - perfinodone. It is the only drug which is approved by US FDA for reversal of fibrosis. So consult pulmonologist and discuss about starting these drugs. Hope I have solved your query. Wish you good health. Thanks."
},
{
"id": 54378,
"tgt": "What causes pain in liver?",
"src": "Patient: Hi. I had gall bladder surgery three years ago but it did not correct the pain that I had experienced for almost a year prior. I kept saying that my liver hurt but my doctors said it was SOD (sphinctor of odi dysfunction). Last October I went through an ERCP procedure so any obstruction of the bile ducts could be cleared. I was in ICU for 10 days with pancreatitis as a result but had hopes that this would correct the constant pain that I had. Doctor said it would take six months to heal. It has been nine months now and the hurt is the same as before. I am very careful to avoid fatty foods. I don t eat red meat, dairy products, or fried foods. I never drink alcoholic beverages and rarely drink sodas. Where do I go from here? My two doctors are the best. What is causing my pain? Doctor: Thank you for posting query.further evaluation needed.liver pathology (pain) maybe caused by viral infection, alcohol, certain drugs, autoimmune diseases, gall stones, increased iron, increased copper, increased fat in body. as most of your test negative, kindly get following done also:1. serum iron and ferritin level2. serum copper 3. autoimmune profile (ANA, anti dsDNA)4. abdominal ultrasound 5. lipid profilevisit hepatologist after receiving reults of above investigation.- vegetables should be ingested daily- Moreover, less activity should be carried out (no sports). - use lemon juice (lemonade) once in a day- walk 30 to 40 minutes everyday.- or swimming is alternative to walking- NO sports and NO jogging- liv52 maybe taken-\"recheck liver enzymes after 6 to 8 weeks\".if any further questions, feel free to ask.Health professionals aim to diagnose properly and manage patients according to their limited knowledge. Cure is blessed by the ONE who Created us, whose power and knowledge is unlimited .wish you good health.regards,Dr Tayyab Malik"
},
{
"id": 60907,
"tgt": "What does a lump on the left bicep indicate?",
"src": "Patient: Hello. I am 42 yrs old, 6'2\", 172 lbs. I have had this small pea size lump under the skin on my left bicep for years now. I have noticed, as my bicep muscle has gotten smaller due to not working out as much anymore, that the small lump has begun to hurt when it is touched or rubbed on by my backpack strap or any other tight clothing etc. It almost feels like there is a jagged edge to it and it is connected to my muscle somehow. Thoughts? Doctor: Hello,The possible issue for a lump on biceps is soft tissue lesion as sebaceous cyst or lipoma. You need definite treatment after clinical confirmation of the diagnosis.Hope I have answered your query. Let me know if I can assist you further. Regards, Dr. Bhagyesh V. Patel"
},
{
"id": 22017,
"tgt": "Suggest remedy for pain in chest",
"src": "Patient: Hi, may I answer your health queries right now ? Please type your query here..gud evening sir i got married in novermer 2010 and after marriage after some time a pain started in my chest and now it have incresed a lot. i nwver had this type of pain before Doctor: hello, I need to know more about the symptoms, like precipitating factors, factors that increases pain etc. Since you are young, we are keeping the possibility of heart disease little lower. I guess the pain is not exertional and relieved with rest, which is typical of cardiac pain. It can be a gastritis and reflux disease. Do you upper abdominal pain, nausea, bloating, burping, increase in pain on food, sour water feeling in throat or chest burning, if yes it further supports the diagnosis. You should avoid fatty, oily and high calorie diet. Have some walk after having food instead of taking rest. Have multiple small meals instead of heavy meals. Have regular sleep habits and avoid stress. Lots of green leafy vegetables, fruits. Avoid smoking and alcohol if any. You can get prescribed tab Pan DSR 40 mg beforebreakfast once a day for 2 weeks. Secondly does this pain increases on applying pressure over the area, if yes it could be costochondritis and it's a musculoskeletal type of pain and needs prolonged nsaid therapy. Does this pain increases on deep breathing and coughing etc. if yes it a pain arising from lining of lungs and needs nsaids treatment and evaluation."
},
{
"id": 93269,
"tgt": "Child birth, tenderness in abdomen, cough. Endometriosis infection?",
"src": "Patient: I am 32 years old and delivered a baby 3 weeks back. For few days after birth I had tenderness in my abdomen and sides.It felt tender on pressing and even while walking or bending. Doctor put me on cephalaxin and towards the end of finishing the antibiotic course, the pain went away. But then I got viral and a bad cough , and the tenderness in abdomen came back. Could it be that muscles are weak and its from coughing, or could it be endometriosis infection. On my last meeting with doc, while I was still having pain from last time, doctor was concerned about infection. Doctor: Hello,Anything will increases the intra abdominal pressure causes pain in a postpartum woman like you.Treat the cough you will be fine.Infection is one other cause of pain which delays the wound healing process.If you suspect infection it should be treated completely to prevent recurrence."
},
{
"id": 218016,
"tgt": "What is the solution for difficulty in walking, heaviness in legs and hamstring pain?",
"src": "Patient: I am 52, disabled female, who for the past few years have having difficulty in walking. My entire right leg feels heavy and stiff. My right thigh has been constantly in pain and the nerves in the thigh seem like they are asleep all of the time. Last night I thought was not going to make it home, because of all of the issues I am having with right leg. At night my right foot and hamstring have tendencies to cramp and it affects my sleeping. Doctor: your symptoms of numbness and weakness in the right leg are concerning. I would recommend you see a physician immediately."
},
{
"id": 122437,
"tgt": "Do I need to see the doctor for severe pain in my left arm that is slowly spreading to the neck and back?",
"src": "Patient: I have had shoulder pain the last three days. Today it is significantly worse in my left arm. I massaged it and it made it worse. It is a dull ache with sharp pains as well. It is now radiating down my arm, into my neck, chest and back. My elbow is starting to ache as well. The pain in my right shoulder is almost gone now. Ibuprofen does not help (I\u2019ve taken 2000mg today). Is it safe to wait a couple days to get it looked at with the symptoms I am having? What techniques can I try to lessen my pain? Doctor: Hello, Severe pain in the arm that is spreading to the back and neck can be related to spondylotic changes in the spine or due to frozen shoulder. You need to consult a doctor for a proper examination so that definite treatment can be started. Hope I have answered your query. Let me know if I can assist you further. Regards, Dr. Praveen Tayal, Orthopaedic Surgeon"
},
{
"id": 157236,
"tgt": "ME patient with continuous ear, chest infections, diarrhoea, night sweats, weakness. Should i see a haematologist?",
"src": "Patient: Hi there I had been diagnosed with ME when I was 22(now 38), although I have been used to swollen glands/glandular fevers and tiredness throughout my life, recently this has become different and more intense. I am getting severe drenching night sweats at least twice a week, feeling very weak, and have swollen glands in my right side of the neck. My blood test results came back negative for TB,Malaria and the doctor has assured me that if i had Leukaemia it would have shown up. I have a family history of cancer with more than 15 cases in my direct family. My doctor wants me to see a Haematologist, but we waiting for 3 weeks to let the current antibiotics I am taking work out. Is this normal - to wait? I am getting concerned because I am not getting better. I have been struggling for the last 4 months with continues ear infections, chest infections, diarrhoea, extreme dizziness and swollen glands. Doctor: Hi and thanks for the query,I do buy the idea of your doctor for you to wait till the antibiotics go through. This shall give an opportunity for a keen follow up and detection of some subtle signs that might not have been detected from the very onset. It is however important to get the opinion of the hematologist and infectiologist. In depth evaluation to exclude properly leukemia or other tumoral process is compelling. Kind regards"
},
{
"id": 111889,
"tgt": "Any ideas about chronic low back pain, headache, neck pain, and neck swelling?",
"src": "Patient: Hi I am a 27 year old white female. I have been a chronic low back pain sufferer for 8 years. I have 4 bulging disc, 2 bilateral par defect fractures, disc and joint degeneration. I have 3 children, all of which i carried within a 5 year period. About 3-4 months ago I began getting very tense headaches, some worse than others, accompianed by mild to moderate neck pain, as well as swelling in my neck, and like a knot or very hard feeling beneath the skin. the headaches have been accuring more often latley. I get atleast 1 every 24-36 hours, it starts out very dull and the course of the day becomes so intense at times I have to turn the lights out and massage my temples, and neck. Any ideas? Doctor: Looks like your spine is having lots of Degeneration. Your stress is contributing in neck pain and headache. I would recommend you back muscles physiotherapy. Apart from that avoid stress, take 8 hours good sleep. Take vitamins and calcium in diet. If pain is severe use local ointment than tablets as side effects of former is less."
},
{
"id": 145137,
"tgt": "What causes difficulty breathing post anesthesia injection?",
"src": "Patient: I was just leaving a dentist office where anesthesia was injected into the path of trigeminal (sp?) nerve (corner of mouth drooping and winking affected). I have allergies and immediately after leaving the dental office I blew my nose. It felt like my nostril collapsed and it is very difficult to breathe through the affected nostril. Thoughts? Doctor: Hello! I read your question and understand your concern. This is probably not due to the nitrous oxide or the local anesthetic . There are a couple of possibilities: One is that if local anesthetic may have been given close to the nasal cavity which can cause irritation and these symptoms. This should be gone within 24 hours.There is also the possibility that anesthesia in a special area on the palate (greater palatine foramen) was given which could also cause these symptoms. Again, they should be relatively short lived.If the problem persist I recommend you to go to the ORL specialist and have a good physical examination with a nasal speculum. A full blood work and other tests may be needed. Hope to have been of help!Greetings!Dr. Abaz Quka"
},
{
"id": 31716,
"tgt": "What is the treatment for vomiting and fever?",
"src": "Patient: my sister seems very sick. she has been vomiting and having a fever of 101 scince yesturday. she only seems to get sick during the night though. her tongue is dark red with a white spot to the side. she has red spots on her arms. What is this? Can it be yellow fever? shes always getting bit by bugs. Please help. Doctor: Hi Dear,Welcome to HCM.Understanding your concern. As per your query your sister has symptoms of fever and vomiting along with spots on tongue and arms which is due to compromised immunity of body and increased susceptibility of getting infected. Fever is happening just because of that. Need not to worry. I would suggest you to visit general physician once and get it examined. You should take proper course of antibiotics to fight against infection. Drink plenty of fluids. If not taking any antibiotic start Norfloxacin for 5 to 7 days. Get culture test of urine done again after few days and start treatment accordingly. Avoid intake of outside food. Drink plenty of water as well. You should take plenty of fluids to rehydrate body. Take diet rich in multivitamins.Hope your concern has been resolved.Get Well Soon.Best Wishes,Dr. Harry Maheshwari"
},
{
"id": 162070,
"tgt": "Can children contract genital herpes from parents?",
"src": "Patient: My son is being seen by a doctor for infection in his foreskin and has been referred for a circumcision. My partner and I both have genital herpes and we have noticed small lumps on my son s penis under the foreskin. We are terrified that he has contracted the disease..how is it best to tell? Doctor: Hello, Unfortunately, it is possible that the contracts the disease through vaginal delivery. Hope I have answered your query. Let me know if I can assist you further. Take care Regards, Dr Salah Saad Shoman, Internal Medicine Specialist"
},
{
"id": 15336,
"tgt": "Small rash on lower back, stomach, started spreading. Food allergy?",
"src": "Patient: Hello, I woke up 4 days ago to a small rash only on my lower back and stomach. Now it is starting to spread to my upper torso on the sides of my breast and arms. The only thing I can think of is that I ate a lot of cereal and milk being that I didn't have it in a while. I don't have any allergies, so I am trying to figure out what it can be. Is it a food allergy or did something bite me. Thanks! Doctor: milk is major offender in patientsif you have taken mikit can be cause of allergy and anything can react at any time at any agetak antiallergic tab for 3 wk and apply anti allergic ointment on areaand get blood serum tests for specific antibodies for foods you take to confirm and withdraw to keep well"
},
{
"id": 168434,
"tgt": "What causes pale blotches on the forehead in biracial baby?",
"src": "Patient: Hi, My son is 8 months old and is biracial. I have noticed over the past couple of weeks he has a few minor pale blotches/spots on his forehead. Not very noticeable, but I notice it. Is this normal for biracial children, or african american children? Doctor: HiThe reason can be pityriasis alba. This usually needs good hygiene, Sun protection and mild steroid use for 1-2 weeks."
},
{
"id": 188649,
"tgt": "How can sores on the roof of the mouth be treated ?",
"src": "Patient: The roof of my mouth is sore for last one week. I have been applyiing Daktrin oral gel withiout any relieve. I am male aged 52. Doctor: Hello and Welcome to HCM, Thanks for writing to us. Cold Sores and Canker Sores are the most common causes of sores on roof of mouth. Warm salt water works well and is soothing. The ulcers normally take about 2 weeks to go away whether you use something or not to treat them. The roof of the mouth is involved in speaking, eating and just swallowing, so it can be very tender. There is a medicine called Debacterol that will take the sting out of it. have a healthy and well balanced diet with special emphasis on citrus fruits and green vegetable to boost immunity and facilitate the process of healing. Hope this helps you. Wishing you good health... Regards."
},
{
"id": 203127,
"tgt": "Is masturbation the cause for the shivering in hands and legs?",
"src": "Patient: hi doctor this is ken for last 10yrs am using mastrubation now my age is 25yrs now what my problemis my hands and legs are shewering more and now am feeling very fear about this while typingthis Qn also my hands are shaking more by shewering please give me wright solution for this Doctor: Hi, Welcome to Health Care Magic, thanks for posting your query . I understand your concern. Masturbation is quite natural in your age group and it has nothing to do with the shivering problem that you are facing. It might be related to a anxiety issue or you may be under a stress of some kind lately. I suggest you to meet a physician who will be able to evaluate your symptom and examine you thoroughly . Please get a nerve conduction test done to rule out the nerve related causes of your symptom. Also if you are in a stress/ if you feel nervousness most times its better to consult a psychiatrist to help you relax .Do eat healthy and include a high protein diet . Hope this helps , and answers your query satisfactorily .Do write back to us if you need further assistance .Take care , Wishing you good health .Regards."
},
{
"id": 155975,
"tgt": "What could be fibroadenoma found in mammogram?",
"src": "Patient: Hello i am 35 and i recently carried out a mammogram and was diagnosed with a fibroadenoma. I also carried out a breast ultrasound and nothing was seen. I feel a dull aching pain which comes on and off. I do not have a family history of cancer. Please advise Doctor: Do not worry fobroadenomas are common in this age and the pain usually corelates with your mentstrual cycle as the effects of harmones causes the on and off pain. It is a benign non cancerous conditionIf there are multiple or complex fibroadenomas you are at an increased risk of breast cancer of 2 times compared to a woman with normal breasts.Simple fibroadenomas are nothing to be worried about."
},
{
"id": 136828,
"tgt": "Suggest treatment for strained calf muscle",
"src": "Patient: i strained my calf muscle after a thorough warm up . the strain occured as i was sprinting at the conclusion of my workout. this happened about 14 hrs . ago . since then i have been icing it 20 mins. at a time keeping it compressed wiht an ace bandage when not icing keeping it elevated and resting it . i have been rubbing down with bio- freeze, too. how long should i do this before introducing heat and is there something else i should or should not do. Doctor: Hi,Thanks for your query.The initial management for your condition is as follows -- Give adequate rest to the injured part and avoid any movements that produces pain- Apply ice wrapped in a damp towel to the injured area for 15 to 20 minutes every two to three hours during the day- Compress or bandage the injured area to limit any swelling and movement that could damage it further. You can use a crepe bandage or a simple elastic bandage - Keep the injured area raised and supported on a pillow to help reduce swelling. If your leg is injured, avoid having long periods of time where your leg is not raised.- Gentle massage with a pain killer cream over the injured area might be helpful If your symptoms are still persisting after the above measures, consult an orthopedic surgeon for further evaluation and management.I do hope that you have found something helpful and I will be glad to answer any further query.Take care"
},
{
"id": 190362,
"tgt": "Is it normal for gums to look split after tooth extraction?",
"src": "Patient: I had an extraction a week ago. The tooth was loose, but broke during the extraction. The dentist got all the tooth without too much trouble. I ve had little to no pain in that area since the extraction, but I noticed a few days ago that above the extraction site, the gum looks split. I have to admit that I was a little too scared to look at the area before I finally did and noticed the split. It looks fine, but I m wondering if this is normal or if I should make an appointment to disrupt my day and drive an hour to the dentist. Doctor: hi after the extraction process , the healing starts to take place. this happens by organization of blood clot in side the extraction site. as the new tissue starts forming from inside out the visible split will disappear and the gums will star to look and feel even and regular following the bone contour. so i would say that you don't need to see a dentist for this particular condition right now. you can relax and carry on with your work for now, but as i tell my every patient that they must get the oral and dental examination done every 6 months so same i will tell you. take care"
},
{
"id": 178780,
"tgt": "Suggest treatment for vomiting in kid after eating fish",
"src": "Patient: Hi, My 11 month old baby is vomiting after eating fish (very small amount of fish). It happened 3 times already over 1 month period; every time after trying to give him dinner containing fish (he hates fish). Is he allergic to fish? I didn t observe any other allergic reaction. Thanks Anna Doctor: Thank you for posting your question.Allergy to any food item would not manifest as vomiting alone but with rash/itching/swelling/breathing difficulty or diarrhea. In the absence of any of these symptoms it is unlikely that your child is allergic to fish, in which case it clearly means that the reason your child vomits out fish is that the taste does not suit her. Absence of vomiting to other food items reinforces this theory.In such a case I would suggest that you either change the method of preperation of fish in such a way that the basic taste gets camouflaged or accept the fact that your daughter is not a fan of fish and omit it from her diet. You may later try reintroducing it slowly and look for response as taste preferences are known to change over time."
},
{
"id": 98334,
"tgt": "Does allergic reaction to mosquito bite cause severe swelling on the body?",
"src": "Patient: I am getting eaten alive by mosquitos this year (in CO, where it is very dry, but unusually humid this summer), and am afraid I might be having an allergic reaction. I have giant sores and swelling, and they tend to get me in similar spots every time. The swelling and building of liquid (such as in my ear) haven gotten significantly worse. Is this an allergic reaction? Doctor: Hello and Welcome to \u2018Ask A Doctor\u2019 service. I have reviewed your query and here is my advice. As you explain the history yes it might be a sign of allergic reaction. However some blood tests are needed for evaluating better this.Please discuss with your doctor and stop taking it. Regards, Dr. Jolanda"
},
{
"id": 123637,
"tgt": "Why are my knees and wrists stiff?",
"src": "Patient: I knees and wrists are stiff and sore evey 3-4 days, i look like an old man walking around the place. I feel like i have a flue comming on. I go to bed after taking one limsip and a heavy T-shirt and Tracksuit bottom, sweat like a pig that night and next morning right as rain, like nothing wrong. Doctor: Hello, It could be arthritis. As first-line management, you can take analgesics like paracetamol or aceclofenac for pain relief. If symptoms persist better to consult an orthopedic and get evaluated. Hope I have answered your query. Let me know if I can assist you further. Take care Regards, Dr Shinas Hussain, General & Family Physician"
},
{
"id": 173975,
"tgt": "How to treat green poop in a 6 1/2 years old child?",
"src": "Patient: my baby is 6 1/2 month old now. She was born 40 days premature. She is on mother feed, farex 2, cerelac and solid foods. For last two days she has pooped green which is my worry now! what could be the cause of her pooping green. Recently i have switched from cerelac rice to cerelac wheat apple! Doctor: HiWelcome to the HCMI understand your concerns but don't worry. Occasional changes in poops of infants occurs commonly when we change to complementary feeds. You should be concerned about the colour and consistency if:1. There is blood or mucous present which may occur when there is bacterial or parasitic infection of gut.It can also occur in some food allergies.2. Very loose wastooland frequent stools suggestive of diarrhoea.3. Persistent green coloured poops may be due to increased lactose or other sugars in the diet.4. If child has fever along with the changes in poops with poor appetite.At present, I would recommend a proper complementary diet such as mashed fruits, porridge, boiled and mashed pulses, cereal based formulas for complementary feeds. Make sure you take a good care of hygiene.For any further questions, you may contact me."
},
{
"id": 573,
"tgt": "Can pregnancy happen if condom breaks during intercourse?",
"src": "Patient: I had sex 2 times the same night, we always use a condom and withdrawal, also the first time i used spermicide, but not the second time (about 2hours later). The thing is the condom broke just before he came and get out, and the tip of the condom was inside me (i was able to get it out). I'm wondering about pregnancy chances, i've just finished my period 1 day earlier and it started 1 week ago, my month is about 26-28 days. My dr is away, but she told me to stay away from the morning after pill because i took it like 4 months ago and i'm waiting my body to stabilize so i can get implanon without affecting my body.Thank you so much, i'm very worry because i dont wanna mess with my body an my dr is not around. Doctor: Hi and welcome to HCM.. I thank you for posting your query here and I'm definitely going to help you out.. I am really sorry for your state but here I'm to tell confidently that you will not be pregnant, do not worry.. I being a surgeon, we encounter so many such cases day in and day out.. I read your question completely and have understood it well enough.. I will analyze all your points below.. 1. You had unprotected sex 2 times and you used spermicides too.. Well, that itself tells you. There's no chance of pregnancy with the use of spermicidal agents.. And even though you didn't use for 2nd time, it's effective. 2. Condom breakage is the most common problem encountered. But reading, your menstrual cycles I can say with certainty that you are safe.! 3. One day earlier completing periods again is the most important history. Generally, ovulation occurs 14 days after completion of periods. Especially in your 26 to 28 day cycle. So 100% there's no chance of ovulation and hence pregnancy even if he had ejaculated completely into your vagina! 4. No need for any kind of pill. Just chill and be happy.. 5. And convey my regards to your doctor too.. Hope this helps, in case you need any further assistance, please let me know... Take care and be happy. God bless you."
},
{
"id": 267,
"tgt": "Can pregnancy happen with intercourse near ovulation day?",
"src": "Patient: I am waiting on a period and nothing yet and a few days late. We had sex on the day after ovulation and could possibly be pregnant. The days leading up to ovualtion are the worst if you do not want to become pregnant that is, correct? How high are the chances the day after? I have 3 children already so I keep pushing off the testing as I tell myself I am not. So out of curiousity is the few days after ovualtion not a good idea either for the future? Doctor: Hello and Welcome to \u2018Ask A Doctor\u2019 service.I have reviewed your query and here is my advice.It is rightly said by you that it is not a good idea as the chances of getting pregnant are quite high if you have sex near the ovulation period.So, it is better to practice safe sex during that period if you don\u2019t want to get pregnant.Hope I have answered your query. Let me know if I can assist you further.Regards,Dr. Ramchandra Lamba"
},
{
"id": 92602,
"tgt": "Hip pain increase on sitting, abdominal pain in pelvic region. Pain in left leg, stiffness. Total hysterectomy done",
"src": "Patient: For several weeks I have had left hip pain that increases with sitting. I have abdominal pain in the pelvic region as well. Sometimes I have to use a cane to walk. The pain is in my left leg to my foot sometimes and the entire side becomes stiff sometimes. I can often walk it out but I have to be careful bending, lifting, sitting, or squatting. I have had a total hysterectomy over a year ago. Doctor: Hello, Thanks for the query to H.C.M. Forum. I would come up with these possibilities for this severe pain in the left hip joint, these includes. 1 Pinched nerve ( sciatica ) , diagnosis can be confirmed by C T scan and X ray of lumbar sacrum region . As you mentioned that there is severe pain radiating to leg , foot , as all these symptoms are an indication of sciatica nerve involvement (pinched)2 Degeneration of head of femur . Diagnosis can be confirmed by C T scan & X ray of head of femur.3 Osteoporosis , diagnosis can be confirmed by blood examination and bone density test.4 Osteoarthritis , diagnosis can be confirmed by C B C and E SR .Hysterectomy have not any role in this pain.In my opinion consult an orthopedic surgeon and get his opinion. Good luck. Dr. HETGood luck. Dr. HET"
},
{
"id": 43605,
"tgt": "Had vasectomy reversal, no sperm showing up on the tests. What should i do? Is there a blockage in the tube?",
"src": "Patient: Hello Doctor, My husband and I are tying to have a baby. He is 45 years old and has had a vasectomy reversal 3 years ago however, there still is no sperm showing up on the tests? What should he do now? Do you think there is a blockage in the tube? He had visited the specialist who performed the surgery and he advised that having another surgery will lessen the chance of us conceiving and has referred us to a fertility clinic to see if there is a blockage. Is there anything else we can do? Doctor: In my opinion, if there are no sperms in the semen after 3 years of vasectomy reversal, it means that the surgery has not been successful. Even though anatomical restoration may have been done, functional blockage is still present. Chances of success after repeat surgery are poor, hence you should consider IVF/PESA/ICSI as an option for fertility."
},
{
"id": 218920,
"tgt": "What does this FSH test result indicate?",
"src": "Patient: HELP! i got my FSH results what does it mean? \u00a0\u00a0\u00a0\u00a0\u00a0FSH\u00a0\u00a0\u00a0\u00a0\u00a027.9\u00a0\u00a0\u00a0\u00a0\u00a0mIU/mL\u00a0\u00a0\u00a0\u00a0\u00a0Final Prepubertal children 0.0- 2.8 mIU/mL Follicular 2.5- 10.2 mIU/mL Midcycle 3.4- 33.4 mIU/mL Luteal 1.5- 9.1 mIU/mL Pregnant 0.3 miu/mL Postmenopausal 23.0- 116.3 mIU/mL Doctor: Hello and Welcome to \u2018Ask A Doctor\u2019 service.I have reviewed your query and here is my advice.The FSH results show you are in a follicular phase of cycle that is egg has not ruptured yet. The egg will rupture and then you will enter the luteal phase.Hope I have answered your query. Let me know if I can assist you further."
},
{
"id": 65804,
"tgt": "What does slight bump on pelvis with slight bleeding indicate?",
"src": "Patient: Just recently yesterday morning i noticed a slight bump on my penis close to my pelvis and was concerned that it was something serious. i first noticed it when it bled slightly, but after that it just remained a little swollen and had a small entrance mark resembling a cut. Up until now the swelling has gone down a little, i wasnt sure if it was a bug bite or maybe something more serious, what are your thoughts on this? Doctor: Hi, it sounds strange.Did you suffer any trauma to the area or any sexual encounter in the recent past. These things could explain why it happened.Other than that it would be difficult for me to say, specially without examining you.You could observe it or better would be to show it to a Surgeon who can examine you and suggest accordingly.It is probably nothing serious but it would be better to be sure and get it out of your mind by seeing a Professional.Take care,Dr Rishi, New Delhi, India."
},
{
"id": 12247,
"tgt": "Suggest remedy for psoriasis and opinion on medicines Mega 3 and Allegra",
"src": "Patient: Sir, I'm suffering from psoriases since last eight months.Is this disease in curable? What mega 3 of Dr.reddey effects to cure psoriases and as well as aquasol A Capsules, besides allegra teb? Kindly advice me in this regards. With thanks. D.K.Jain Doctor: Thank you for your question!!Psoriasis is a chronic inflammatory skin disease, which remits spontaneously in 1/3 of patients, but the course is not predictable. It can be controlled rather than cured, and you can keep it in control but with a careful avoidance of trigger factors that can exacerbate it, also with the continuous use of medications.Allegra tab is fexofenadine an antihistamine to suppress pruritus. Mega 3 are omega 3 oral supplements which showed some degree of improvement in psoriasis lesions. Aquasol A capsules are vitamin A supplements which also help with psoriasis.How long are you on these medications? Have you noticed improvement?Are your lesions localized or generalized?Anyway I would like to give you some recommendations and general principles how to control the disease.I am not aiming to get rid of the psoriasis, but to make the condition tolerable.Try to avoid the factors that exacerbate psoriasis such as: Infections (tonsillitis, pharyngitis), trauma to the skin, stress or emotional upset, some drugs such as beta blockers, lithium etc.You can use the medicines you are taking, except Allegra tab unless you have pruritus.If a patient the same with your situation would meet my clinic, I would suggest to move to some topical preparations such as Vitamin D3 analogues (calcipotriol) applied 2 times daily for 3 months, also a topical cortiocosteroid cream applied twice a day also for 2-3 weeks. You can also mix these two together and apply them. You should talk to your doctor about prescribed medications.It is crucial that you use moisturisers 3 times daily, and if the situation is not under control your physician can advice you about systemic treatment.Be careful to do the screening before starting a systemic treatment.I hope this helps. Feel free to ask again if anything is unclear.Wish you good health!Dr. Migena GEGA"
},
{
"id": 4701,
"tgt": "Had unprotected penal, vaginal contact with fingering. No penetration. Due for periods in a week. Suggest to avoid pregnancy",
"src": "Patient: About 15 hours ago I had penal/vaginal contact, but no penal penetration, and I'm wondering how likely pregnancy is. My partner had some liquid on the tip of his penis, but he didn't ejaculate for another 10 minutes or so. The thing that worries me most is that from what I've just read, there is sperm in even that little drop, and if he fingered me after (I don't remember what order everything was) it could've entered my vagina and possibly fertilized an egg, and I'm probably a week or two from my next period (My periods are not very regular, and I'm 18 years old). What is the likelyhood, and what should I do? Thanks. Doctor: Hi,You are right. Even a drop of ejaculate or even the pre-ejaculate is sufficient to cause conception during the fertile period. As of now, you will have to wait until your next period and if you miss it, go for a pregnancy test 3-7 days later. Else, you can get a serum beta-hCG titre studied two weeks from the time of intercourse. This can be conclusive. Please see a specialist for further management. Hope you find this information useful. Take care."
},
{
"id": 126658,
"tgt": "What can cause pain and burning sensation in the shoulder and forearms?",
"src": "Patient: im 18 and I was lifting boxes 3 days straight, my right arm hurts in my shoulder and bicep, sometimes my forearm burns. I can t lift my arm or bend it. What do i do to treat this besides ice and how long will it take to heal. I assume its tendinitis but I have no insurance for a doctor. Doctor: Hi, I have read your details and in my opinion, you are suffering from overuse injury of shoulder muscles and biceps, may have affected the neck also. I suggest that you take painkillers, muscle relaxants and physiotherapy apart from hot fomentation now. it is likely to take 3-4 weeks. Hope I have answered your query. Let me know if I can assist you further. Take care Regards, Dr Gopal Goel, Orthopaedic Surgeon"
},
{
"id": 55429,
"tgt": "Suggest alternative means for treating gallbladder than surgery",
"src": "Patient: my father is 67 years old he had his ultrasound yesterdAY, It said that he has enlarged liver with diffuse parenchymal disease. Consider hydrops of gallbladder. Does he need to undergo operation to take away gall bladder?? Is this contagious? Is there any other option than operation?/suurgery? Doctor: Hi. I went through your concerns regarding your father medical conditions. Let me rephrase that your father has gall stones which are asymptomatic with diffuse liver parynchymal disease (liver cirrhosis). There is high risk of decompensation of liver disease once your father receive general anaesthesia for gall bladder surgery. I would suggest to wait and see. There is another option of starting capsule ursodeoxycholic acid which can solubilize the gall stones. But it needs at least 6 months. It's a safe drug for your father. It has narrow spectrum of side effects like nausea, vomiting, diarrhea, headach, dizziness and dyspepsia. But most of patients tolerate it very well I wish good health for your father."
},
{
"id": 28421,
"tgt": "Are the any side effects of taking Olmezartan for BP?",
"src": "Patient: Dr, I am age 66yrs with BP under control with TRIOLMEZEST 20 mg per day. Now my creatinine level is 1.5mg.and taking Nefrosave tabs 2per day and Nodosis tab 3 per day. Can I take Triolmezest 20 mg per day. Is there any side effects causes by olmezartan? to my kidney function? Doctor: Hithank you for trusting HCMI read your query and understand your concernFirst I am sorry for the symptoms you are suffering but you have to be calm as your symptoms don't lead to any risky disease In my opinion it may be a minimal renal disfunction due too many causesYou should use healthy diet full of fresh foods and liquids(two-three liters a day)Avoid olmesartan use (avoid ace and arb)You should be switched to calcii blockersDiscuss with your doctor about my opinionHope I have been of helpWish you good health and fast recovery Feel free to ask for further questions or doubtsBest regardsDr Eris RanxhaNeurologist"
},
{
"id": 114211,
"tgt": "Why is there back pain in the bottom left side above my hip ?",
"src": "Patient: i have a really bad pain in my back in the bottom left side just above the hip, im also pregnant . The pain gets so bad it causes nausea . Should I be worried Doctor: dear kadi... as u r pregnant its obvious to get backaches.. take rhustox 200 single dose for the pains its a homeoapthic medicine so safe,sure n faster results.. god bless.. drvigsclinic@gmail.com"
},
{
"id": 25787,
"tgt": "What causes heart palpitations along with tingling sensation in left arm?",
"src": "Patient: Hello. I am a 26/b/f. I have been experiencing some heart palpitations for the past couple of years now. But in the past few weeks things have gotten worse with the palpitations and last night I started having a tingling sensation in my left arm with a little pain. What could this be? Could I be experiencing some type of heart failure? Doctor: Thanks for your question on Health Care Magic. I can understand your concern. At your age of 26 years, heart failure is unlikely. But we should first rule out arrhythmia (rhythm disturbances in heart) in your case because it can cause palpitations and left arm tingling sensations in young patients. So consult cardiologist and get done ecg, 2d echo and Holter monitoring (24 hour continuous monitoring of ecg). If all these are normal then no need to worry for arrhythmia. Sometimes stress and anxiety can also cause similar symptoms. So avoid stress and tension. Be relax and calm. Consult psychiatrist and get done counselling sessions. You may need anxiolytic drugs too. Don't worry, you will be alright but first rule out arrhythmia. No need to worry for heart failure. Hope I have solved your query. I will be happy to help you further. Wish you good health. Thanks."
},
{
"id": 58741,
"tgt": "Had gall bladder removed. Gassy pain. Dark blood from vagina, cramps. Cause?",
"src": "Patient: Hi. I am a 60 year old female. I have not had a period in about 13 years. I had my gallbladder taken out six days ago lapro. I have had the pains and not being able to have a bowel movement b/c of the gas they put inside you for the surgery. Today (6th day), the gassy pain seemed to be worse. I noticed when I used the bathroom, there was dark blood coming from the vagina . I felt so bad, I used the heating pad and took a short nap. I awoke with lots of dark blood, and also had lots when I went to the bathroom again. I feel like I have cramps again. What would cause this? Thank you! Doctor: Hi and welcome to Healthcare magic.Thank you for the query.I dont think that gallbladder removal and vaginal bleeding can be related. Cholecystectomy can cause certain digestion difficulties and you should be patient for several more weeks till your digestive system adapts. Your vaginal issue should be checked by gynaecologist.Wish you good health. Regards"
},
{
"id": 59441,
"tgt": "USG shows mobile calculus in gall bladder lumen. Taking thyroxine for hypothyroidism. Your view?",
"src": "Patient: Myself 59 yrs Indian female vegetarian weight 70 kg Height 5 feet 1 inch.My USG report suggests a 18 mm size frely mobile calculus seen in gall bladder lumen . Wall thickness normal .No edema .No pericholecystic fluid collection . Rest is normal. I am asymptomatic . I am hypothyroid taking .75 mcg thyroxine tab for last 5 years. Radiologist suggested surgery saying it is better to do it earlier since it may not be possible to do it at older stage if required. Sir what is your viewpoint ? Thanks Doctor: Hi, I would suggest you to undergo surgery at this stage once you are normal for thyroid hormonal studies. It is best to get the gall bladder removed when you have not developed any local complications. Once these develop surgery may be difficult with a chance of complications rising. A laparoscopic procedure will not cause much pain or discomfort. Regards"
},
{
"id": 143673,
"tgt": "Does swollen/bruised blood vessels indicate pinched nerve/blood vessel?",
"src": "Patient: While riding in the passenger seat, I hit my knee and started having excrutiating pain and could not fully extend it. I went on to the beach a few days later and fell on the same knee in the surf hitting it really hard. When I got out of the water I noticed the blood vessels started swelling up and bruising. Could I have a pinched nerve/blood vessel, and when I fell on it, it released it Doctor: Hi, Thank you for using HCM. As far as I understand, there is nothing to worry about. You seem to have injured your knee accidentally twice at the same spot. If there are no cuts, you could try applying Ice packs 3-4 times a Day. If there is an associated redness, pain and/or swelling, you could take a short course of painkillers plus anti inflammatory m\u00e9dication. Hope this helps. Take care."
},
{
"id": 56008,
"tgt": "What causes increased bilirubin level?",
"src": "Patient: i am suffering from jaundice, bilirubin count 3.5. I took rest for 15 days and i was gaining energy so i thought i was getting cured and so i started smoking. But after 15 days when i tested once more my bilirubin count has increased to 4.9. is there a problem with smoking. Doctor: Welcome at HCM i have gone through your query and being your doctor I completely understand your health concerns.. Are you having itching over body?? Are u having white stools and dark urine?? Are you have any blood disorder ? In my opinion smoking is not causing increase in bilirubin levels. You should better consult hepatologist regarding it to rule out the possible causes of acute hepatitis. Meanwhile take rest and avoid spices. Do drink plenty of water... Stay safe Regards Dr saad sultan"
},
{
"id": 171373,
"tgt": "Can dosage of 6-MP be decreased while undergoing leukemia treatment?",
"src": "Patient: my brother s is a leukemia patient .he is 4 years and 2 months old . currently he is undergoing final mentainanace ttt with MTX and 6-MP . along with septran .his current SGPT is 77 U/L while his SGOT is 48 U/L please could you tell me if we should decrease his dosage of 6-MP ? Doctor: Hi, 6 MP(Mercaptopurine) is stopped when total bilirubin levels are more than 2 mg/d or SGOT, SGPT >200.However, in patients of leukemia this is not the only criteria, we have to also look for absolute neutrophil count, and any associated complications like pancreatitis. If I was treating your brother, then I have kept this things in mind also. If only SGOT and SGPT are in range of 70 to 80,then there is no need to decrease 6 MP. I hope this has helped you. Wishing your brother good health."
},
{
"id": 170531,
"tgt": "Suggest dosage of ofloxacin suspension for 2 year old kid",
"src": "Patient: what is dosage(ofloxacin suspension) to be given for 2 year old child Case history My child was found to have E coli infections when we took the urine culture,60000 colonies/ml.now,is it advisable to use ofloxacin and what dosage should be given for 2 year old child Doctor: Hello, dosage of ofloxacin in a child is given according to weight and not by age of child. Secondly, urine culture is showing e.coli more than 60000 colonies per ml but we have to look at the complete report like organism is sensitive and resistant to which antibiotics. I think that you should revert back with exact weight of child and complete report of urine culture test. Good day."
},
{
"id": 60253,
"tgt": "How long does a person live if he has adeno carcinoma of liver with out surgery or any therapy ?",
"src": "Patient: Name: Ghulam Ahmed / Age: 59 yrs/Ht: 5.9\u2019/ Gender: Male/Non Alcoholic /Non Smoker /No BP/Non Diabetic, looks fit , one month back i.e. from 1st week of May 2011 had some abdominal discomfort . Enzyme test (SGOT, SGPT, alkyline phosphate) show high levels, Bulirubine is normal, apetite is OK and after series of tests was diagnosed with adeno carcinoma of GB, primary GB involving liver , multiple leasions in liver. FNA test came positive ? Is it curable? How long can a person live if no surgery is done or no therapy is taken? Doctor: Dear Ghulam , Ca gall bladder with secondaries in liver carries poor prognosis. Depending on extent of involvement 6 months to one year survival can be expected without any treatment. treatment is surgery +chemo/radiotherapy. Take care."
},
{
"id": 98565,
"tgt": "Suggest medications for nasal allergy with patients with low potassium levels",
"src": "Patient: I am 39 years old. I have respiratory and nasal allergy.From last two years I am taking montulukast and livocettizin tablet once a day and use athalien inhaler Last Six days ago my potassium level is become very low Please suggest me what drug I take which is safe for me Doctor: Asthalin inhalers can worsen hypokalemia,But you need to get investigated for your decreased potassium,whether it is due to high loss or decreased intake.Montelukast levocetirizine can be continued and atrovent can be tried after consulting your doctor"
},
{
"id": 203270,
"tgt": "How to heal catheter removal mediated penile and scrotal swelling?",
"src": "Patient: i just had my lymph nodes removed from my abdomin a few days ago. after the cathider was removed my penis and scrotum swelled up with fluid. i was told this was normal, but it feels like the fluid is seeping out the skin because its like wet but i am not sweating. can this happen? Doctor: Hi. Was this a open or laparoscopic surgery? Cather causing swelling of the penis and scrotum and is not seen. It must be from the abdominal wall. You must already be on antibiotic cover and supportive medicines.. Just wait and watch is the policy as you must already be under the medicines. You may not need something else to be done unless there are signs of an active infection of the areas you mentioned : like redness, local raised temperature, tenderness and oozing of pus. This can happen and not very unusual finding."
},
{
"id": 51352,
"tgt": "Frequent urge to urinate, scan showed renal stones, prescribed potrate MB6, bapter. Will it help?",
"src": "Patient: Last 3 month i feel more urinate. I consult kidney sepacility doctor in chennai. They advise to take urine and blood test . My blood and urine result show normal. After that they take scan. 5.5 mm stone appear in my bladdar. He told now the stone is passing stage. He priscribe to tables Potrate MB6 and bapter capsules. past one weak i taken this tablet. I feel not good. But docotor advice i have to take tablet for 20 days. I feel urinate by every 2 hours. defenetly the stone will pass with in 20 days? or i have to take some treatment for this issue. Kindly help me Doctor: Hi there, your feeling of more frequency of urination can be related to urine infection, high blood suger, partially obstructing ureteric stone, irritable bladder. needs testing by urine routine and culture to r/o infection, fasting blood suger to r/o diabetes. Bladder stone 5.5mm - generally should come out by itself in an adult. but if urinary symptoms are troubling. it can easily be taken out by cystoscopy. as far as medincine is concerned, if not tolarating you can stop it, it will not alter outcome."
},
{
"id": 109978,
"tgt": "Suggest medication for pain in the mid lower back area",
"src": "Patient: Hello, I am a fourteen year old female. For over a month I have been experiencing pain in my mid to lower back. I have had an ultra sound to rule out any problems with organs. The hospital has ruled it as just a muscle spasm and have prescribed physio and muscle relaxants. When I got home I noticed that my hips are uneven (right higher than left) and that my shoulders are uneven (left higher than right) I had a back X-ray and am awaiting the results, I was just wondering what you thought it may be Doctor: Hi,Welcome to healthcare magic. After going through your query I think You are suffering from acute backache due to slip disc.Treatment of it is rest and analgesics . Diclofenac three times a day after meals is effective. Sometimes strong analgesic(such as tramadol three times a day after meals) is required. OMEPRAZOLE 20 MG before meals prevent acidity caused by analgesics. Second possibility is simple muscular pain. Treatment of it is similar .Muscular pain takes 5-7 days to relief while disc prolapse may take2-3 weeks and even more.To further investigate MRI of the region is advised. You may consider this after discussing with your treating doctor. If you are thinking to hospital then see a orthopedician. I think your query answered.Welcome to any follow up query."
},
{
"id": 119366,
"tgt": "Does a person with thalassemia minor suffer frequent cuts on the lips ?",
"src": "Patient: Hello, I am 37 years old female. Apart from minor thallesimia, I have no other serious health problems. For some years now the skin on my lower lip becomes very dry and peels off. It becomes more severe as my menstruation date nears ,sometimes cuts develop on my lower lip esp when I peel off the dry skin .I feel very thirsty and drink lots of water and apply lip balm but still my lower lip is mostly dry and the skin on it peels most of the time. Please advice. Doctor: Hi,Thanks for query, Many a time you have habit of moving your tongue on the lip very often will result wetness and then dryness of the lip leading to cut on the lip. Quit this habit if you have. Apply moisturizing cream or Vaseline on the lips. Do not pill off the dry skin as this will lead to bring infection and infected cut. Take B. complex,Iron and multivitamin supplements. Keep oral hygiene proper. Ok and bye."
},
{
"id": 76656,
"tgt": "Which specialist to consult for the below CT / MRI findings?",
"src": "Patient: Hi, I am 30 yrs old(Male). I am having pain in my in right side back of chest since March'2011. I have been diagnosed with CT scan and MRI which says that I have lobulated margined expansile osteolytic lesions in relation to right sided poeterior elements of D9 vertebra with involvement of adjacent bone , extraosseous components and associated extradurtal compression of dorsal spinal canal around D9. The lesion measures about 44 mm X 39 mm in axial plane and about 36 mm X 43 mm in sagittal plane. Please advise suitable doctor in AIIMS to be consulted for the suitable treatment. Doctor: Thanks for your question on Healthcare Magic. I can understand your concern. I have gone through the MRI report you have attached. It is suggestive of osteolytic mass lesion in the vertebra which is also causing compression on spinal cord. Possibility of malignant looking mass is more likely by this description. You should consult orthopedic doctor for this. You will need Biopsy from the mass to confirm the diagnosis. Enblock resection with placement of implant today stabilize the spine are needed. So consult orthopedic doctor in AIIMS discuss all these. Hope I have solved your query. I will be happy to help you further. Wish you good health. Thanks."
},
{
"id": 201418,
"tgt": "What does the sperm analysis report indicate?",
"src": "Patient: hello doctors, I make a sperm analysis, below are the results of the test:Sperm Analysis: ResultAspect:WhiteVolume:2.0Viscosity:NormalPH:8.0Liquefaction time:20Sperm Count:93 mill/mlAbnormal forms:20%WBC:1-2RBC:RareImmature cells:NumerousMotitlity after liquifaction: 70%Motility after 1h: 60%Motility after 3h: 55% Doctor: Thanks for contacting HCMYour sperm analysis is completely normal. You should be able to impregnate your significant other. You are very fertile.Hope this answers your question. Please contact us again with your health care concerns and questions"
},
{
"id": 100255,
"tgt": "What causes swelling on eyes after using hair dye?",
"src": "Patient: I dye my hair 4 days ago since then my forehead and eyes have swollen up bad.went to dr in a.m got a shot and medication.what can I do to help with the swelling.they gave me a steroid pack and fuild pill to help.but is there anything else I can do at home? Doctor: Hello,Thanks for posting your query on HCMYou developed an allergic reaction to the dye and an angioedema due to the same.You require to take an anti histamine preferably the second generation type without sedation.Avoid any home remedies and creams.Eat a wholesome diet, relax and sleep well. you would get well soon.In future you should avoid the dye that caused you the reaction. If you ever want to use any dye in the future you should apply it over a small area on the body and see if it produces any reaction before using it on your scalp.Take CareDr. Noble Zachariah"
},
{
"id": 56332,
"tgt": "What causes elevated liver enzymes?",
"src": "Patient: I have been unwell with tightness around chest, pain beneath ribs, had 2 fainting episodes. On anti biotics for Pleurisy. Since last night have a stiff jaw and trouble swallowing, and breathing zeems shallow. ECG in December and 7 day heart trace were normal. Liver enzymes raised but lft normal. Have a neurology appt next month. Very afraid. Could it be anxiety? Please help. Alone with my 3 year old and scared of passing out. Inside of mouth totally numb. Thank you. Doctor: HelloRaised liver enzymes like raised SGPT,GGT may indicate liver injury.It may be due to many reasons like hepatitis,alcohol intake,altered lipid profile,medicines,auto immune causes etc.You may need few investigations like routine hemogram,random blood sugar,lipid profile,viral markers.You may need ursodeoxycholic acid tablet.It helps in regeneration of liver cells.You may need few more medicines after full investigations.Get well soon.Take CareDr.Indu Bhushan"
},
{
"id": 33816,
"tgt": "Suggest treatment for positive hbsag and hbeag in blood",
"src": "Patient: hello dr. this is satish iam 24 years old and i am hbs ag +ve and hbe ag +ve. i dont knot when it attacked, two years back i was tested by blood its hbs ag +ve at that time i dont know about this disease . few months back i met an accident i tested my blood and doctor told me that its hbs ag+ve. and he said there is no cure. sir i want to know what will happens with this disease, and when will i effected ? and is there any cure in medical science or in ayurvedic treatment any thing else so please guide me Doctor: Hello,Welcome to Health Care Magic.Thanks for writing.I am Dr.Saddiq ul Abidin. I have read you question completely, i understand your concern and will try to help you in best way possible.First of all, I hope you world have a speedy recovery the accident you had recently. The brief details you have shared refer to as, you have been diagnosed recently, incidentally as having hepatitis B infection while having a blood screening for your other issues.Since it was incidentally found, it means it has not given you any problem in the past, that is why it was left undiagnosed until now, and it is a good news.Hepatitis B virus is among the few causes of chronic liver disease, and currently some antiviral drugs are available in the market which are given from time to time depending upon individual cases and disease behaviors.A good thing about hepatitis B is, a low progression rate to Cirrhosis or liver damage, which occurs in less than 10 percent untreated people. However need for a regular check up for any signs of early cirrhosis or any space occupying lesion in liver, is advisable.In your case HbeAg positive is suggesting an active disease, and if along with that if you run a Liver function test, which turns out to be raised than normal, along with a positive quantitative PCR of Hepatitis B virus, above 200-300 ( this criteria varies in different guidelines), than this may necessitate treatment commencement with tenofovir or other antivirals. But that is a sensitive domain and needs a detailed evaluation and workup by a gastro-entrologist.As far as your concerns regarding prognosis and severity of disease are concerned, Nothing is as darker as you have perceived and their are multiple options available, but you just need to develop a close liaison to a good gastro-entrolgiost.Lastly, a family screening and vaccinations is my most important suggestion to you, which should be commenced as soon as possible.I hope this answered your question.If you have more queries I am happy to answer. Otherwise rate and close the discussionRegards,Dr.Saddiq ul AbidinM.B.B.S(Licensed Family Physician)Resident Medicine."
},
{
"id": 159206,
"tgt": "Had breast cancer. Cancer cells found in lungs lining, acid reflux, stomach wall enlarged. Is it cancer?",
"src": "Patient: My mother had breast cancer 5 years ago. In decemeber they found cancer cells in her lining of her lungs. At the time they also found her stomach wall was thick but they dismissed it do not being anything. She told the doctors that she was having acid reflux and she could not eat and had little to drink. We are know in febuary and ahe is back in the hospital and her stomach wall is still enlarge. And now they thick it s cancer Doctor: Sorry to hear about your mother's problem. She is suffering from metastatic breast cancer, which has already spread to the lungs and the doctors are suspecting spread to the abdomen as well. At this stage she is a candidate only for palliative chemotherapy and hormonal therapy (if the cancer is ER, PR positive). To confirm the spread to the stomach, a biopsy can be done, but it is not going to change the line of management. She can be given acid reflux medicines by the doctor. The lack of appetite can be explained due to wide spread cancer. I would suggest that you consult your oncologist and palliative care specialist. For pain management, you can go to a pain clinic. Hope she feels better after the treatment"
},
{
"id": 75254,
"tgt": "What does this chest X ray report indicate?",
"src": "Patient: I'm 49 years old, female and non smoker. I got the result of my chest x'ray dated April 8 which reveals the following result: lungs are clear.heart, great vessels, diaphragm and bony thorax are intact. diaphragm is low-set.I just want to know the meaning about my x-ray report . your help is highly appreciated. Thank you very much. 3months) then, they become VERY VERY difficult to treat and may possibly never fully resolve. Get him to either a concussion specialist or a headache specialist in my opinion and make sure he is not taking too much OTC medication or prescription medication and make sure he is documenting his headaches so that they can be optimally followed and treated based on his reporting in order to get rid of them ASAP. Hope I have answered your query. Let me know if I can assist you further. Take care Regards, Dr Dariush Saghafi, Neurologist"
},
{
"id": 78642,
"tgt": "Suggest treatment for pectus carinatum",
"src": "Patient: My 17 year old daughter was just diagnosed with pigeon chest. I was looking online and noticed that there is a new procedure out for this condition that is less invasive than repositioning the breast bone and removing the cartilage. Which procedure is better? Doctor: Thanks for your question on Health Care Magic. I can understand your concern. Honestly speaking, no treatment is required for pectus carinatum (pigeon chest). It is developmental defect of rib cage. Surgical correction is very risky and 100% successful correction of the deformity is next to impossible. But if patient is having underlying lung developmental defect, breathlessness, collapsed lung, low oxygen saturation etc than only surgical correction is required. Just for the cosmetic purpose, better not to go for surgery. And if surgery is indicated than repositioning of breast bone with removal of cartilage is best option. Hope I have solved your query. Wishing good health to your daughter. Thanks."
},
{
"id": 53336,
"tgt": "What causes elevated liver enzymes?",
"src": "Patient: I had a blood test for life insurance. Everything on the test came back good except my SGOT and SGPT. SGOT is 160 u/l and SGPT is 138 u/l. Tested negative for hep b and hep c. I'm 29 and in good physical conditon. Not over weight or obese. Don't drink more than once every other week. What could be the problem? Doctor: Hi,SGOT and ALT are reasonably sensitive indicators of liver damage or injury from different types of diseases or conditions, and collectively they are termed liver tests or liver blood tests. So the next step is to look for certain liver damage and most common cause is fatty liver disease. It should be confirmed by ultrasound. Other causes are viral hepatitis,cirrhosis, medications, alcohol intake or autoimmune diseases. To verify exact cause, US, bilirubin levels and tumor markers should be additionally done. Then appropriate treatment can be started.I hope I have answered you query. If you have any further questions you can contact us in every time. Wish you a good health.Kindly regards,Dr. Ivan R. Rommstein"
},
{
"id": 160697,
"tgt": "What causes diarrhea in a child after taking Paracetamol for fever?",
"src": "Patient: My daughter who is now 2 years 1 month, was having fever since yesterday. Took her to clinic and was given paracetamol. She started to having diarhoea after taking the medicine and today her motion is in green colour. Is it normal as she didn t have any solid food yesterday? Doctor: Hi,The diarrhea is not caused by paracetamol, but is part of the febrile illness (probably acute gastroenteritis). If she is otherwise active, playful and taking feeds, nothing to worry. Continue paracetamol as needed, and give ORS solution or salted rice water in frequent small sips after each stools to avoid dehydration. I also used to give zinc suspension 5ml once daily for 14 days to promote bowel recovery. If fever is not getting down in 2-3 days, she becomes lethargic, not taking taking feeds or persistent vomiting, kindly review with doctor, as this may need antibiotics and other measures.Hope I have answered your question. Let me know if I can assist you further. Regards, Dr. Muhammed Aslam T. K., Pediatrician"
},
{
"id": 112977,
"tgt": "Took punch to my ribs. Pain in sternum and back. What should I do?",
"src": "Patient: good day, last week i took a hard punch to my ribs (about 10cm under my armpit) it didn't hurt much at the time, but the pain came the morning after, i had to roll myself out of bed. the rib itself is not sour but whenever i take a deep breath i experience severe pain in my sternum and my back, and it seems to be getting worse. what should i do?thank you Doctor: Hi, The pain is a result of the blunt injury caused to the ribs and the underlying muscles, I would suggest you to go for a check up or X ray, to rule out any injury to the underlying lung pleura(a covering of the lungs), try taking homeopathic remedy Arnica 200, 4 pills every 3 hours till the pain subsides. Take care, get well soon."
},
{
"id": 94818,
"tgt": "Getting sever abdominal pain after eating raw salted almonds and tuna fish sandwich. Looking for cure",
"src": "Patient: Yesterday I ate a small bag of raw salted almonds around 3pm, a few hours later I had a tuna fish sandwich on pita bread. Shortly after that I developed the most intense and sever upper/mid abdominal pain and cramping I ve ever experienced. I did move my bowels, but for the amount of pain I was experiencing it was not very much. I did not seem to have gas. Today I m feeling a little better, but the pain is still there, it is just tolerable and intermittent. Should I see a doctor? Doctor: Hi, FROM YOUR DESCRIPTION IT SEEMS THAT THE FISH YOU TOOK MAY BE THE CAUSE OF YOUR SYMPTOMS. THE SYMPTOMS MAY BE DUE TO SOME INFECTION CAUSED BY THE FOOD THAT YOU TOOK MAY BE DUE TO SOME CONTAMINATION OR INFECTED ITSELF BETTER TO CONSULT DOCTOR AND FOLLOW ADVICE . HOPE YOU BE ALL RIGHT BY THIS TIME ."
},
{
"id": 138260,
"tgt": "Suggest treatment for rashes with swollen knee after meniscus repair",
"src": "Patient: I had meniscus repair left knee 3 weeks ago. developed a rash from the knee down only on surgical left knee. Rash was light red now it is dark red and won t seem to go away. Knee still swollen and having difficulty walking on that leg. I have a very noticeable limp. I have taken benedryl and used Hydrocortisone cream then changed to Nystatin cream not sure what else to do. My ortho said it probably is due to the stitches or ace bandage that i had, however the stitches and ace bandage was removed about 8 days ago. Doctor: Hello,Welcome to the magical world of health care, I went through your query, and i feel these are either iodine sensitivity as is seen with the local application of providone iodine in some patient. it will settle do not worry. your treatment is right.I hope my advice would have been useful, in decision making regarding your treatment, still if you have any clarifications or doubts feel free to contact back.I hope a 5 star rating if you feel guided in your treatment,Thanks"
},
{
"id": 183071,
"tgt": "Suggest remedy for pain due to infected teeth, swollen and abscessed glands",
"src": "Patient: I have two infected teeth at the moment, swollen glands are present as well. They re abscessed, I m sure. So, my question is this: I have full scripts of both 500mg Cefuroxime as well as 500mg Amoxicillin. I can t get to the dentist until this Friday and I need to start a round of something NOW, because, for obvious reasons, the dentist will not pull teeth as infected as mine presently are. I m sure that the Amoxicillin would be the best choice, even though the Cefuroxime is stronger. But, I ve taken and stopped Amoxicillin so many times in the past that I m sure I ve built up a resistance to it. So, which one do I start? I know I need to get to the dentist, but please, which would you recommend? Doctor: Hi,Thanks for posting the query, I would suggest you to take tab almox clavulanm BD and tab aceclofenac BD for 3-5 days, instead of the given medications. Antibiotics and analgesics will give a temporary relief unless the tooth is treated with Root canal treamnet you will face pain and infection, i would suggest you to consult a nearby Dentist get an x-ray done and start with the treatment.Hope you find this as helpful,Take care!"
},
{
"id": 60844,
"tgt": "What do painful lumps on the back indicate?",
"src": "Patient: I have a row of lumps on the left side of my back/and left side, starting about 3 inches down from my arm pit, elongated, and they are under the skin, not visible on the top of skin and very painful when touched. They have been increasing in number and size, I am going to my family doctor this week, do you know what this may be ? Doctor: Hello and Welcome to \u2018Ask A Doctor\u2019 service. I have reviewed your query and here is my advice. * The possible reason for painful lumps here suggest multiple neurofibroma or others. * I recommend to get check up by general surgeon for best possible evaluation and guidelines.Primary relief with analgesic can be possible. Hope I have answered your query. Let me know if I can assist you further."
},
{
"id": 63794,
"tgt": "Suggest treatment for a painful lump on the armpit",
"src": "Patient: Yesterday I just discovered a lump on my left armpit but I m not sure if it is a lump? And this morning my breasts were itchy and it was hurting but the pain was not so much. Since last week my underarms have been itching and I m really worried if it s some type of breast cancer. I told my mom about it but she said I was just overreacting. Doctor: Hi, dearI have gone through your question. I can understand your concern. You may have enlarged axillary lymphnode due to reactive hyperplasia or soft tissue tumor like lipoma or some axillary tail of breast. You should go for examination of axillary region and breast. If needed go for biopsy or fine needle aspiration cytology. It will give you exact diagnosis. Then you should take treatment accordingly. Hope I have answered your question, if you have doubt then I will be happy to answer. Thanks for using health care magic. Wish you a very good health."
},
{
"id": 187295,
"tgt": "What causes sensitivity and pain in the molar tooth that has been filled?",
"src": "Patient: i waa suffering from toothache in one of my molar three wks ago. so i visited a dentist .. he prescribed me cosymoxyl 625 (2 times daily ) for 3 days after taking it for 3days i visited him again then he filled my tooth temporarily bcz there was a bit gap in my molar teeth. since then it was going fine but from yesterday eve i am feeling a bit of sensation and a light pain in the sane teeth .. what should i do.. please suggest.. Doctor: Hello and thank you for writing to us,I would like to inform you that the pain in your tooth is from the infection which has gone deep in to the tooth , therefore a temporary filling will relief pain temporarily.The symptoms presented by you suggest that you need a root canal treatment on your tooth.I will advise you to see a specialist for that.Until you see your dentist you can take pain killer for pain relief and perform warm saline rinses.I hope i answered your query.I wish you good healthThank youTake care"
},
{
"id": 166740,
"tgt": "Suggest virus name which causes low fever and no pain",
"src": "Patient: My daughter had a virus when she was 6 and was not able to walk for over a week. I took her to the hospital when it happened and they ran endless tests. She had a low fever but was in no pain. The doctor treating her at the hospital said he had seen it only a few times in his 35 years as a doctor. I need the name of the virus for a new doctor she is going to see? Doctor: Hi,Is she vaccinated for polio? If not and if she travelled to the endemic area such as Asian countries it can be acute paralytic poliomyelitis. Polio virus is very rare and causes limb paralysis.Hope I have answered your query. Let me know if I can assist you further. Regards,Dr. Hazan"
},
{
"id": 204244,
"tgt": "How can depression and anxiety be treated?",
"src": "Patient: My adult son has delusional disorder and has been on Risperdal he doesn t feel it is working the best for him, he has low motivation, had high weight gain, depression, memory loss and extremely unhappy. There is diabetes in our family history. He is also on celexa for depression and another drug for anxiety (not sure name of it). What would you recommend as an alternative antipsychotic. Doctor: Hello and Welcome to \u2018Ask A Doctor\u2019 service. I have reviewed your query and here is my advice. In delusional disorder the person suffering from it do not have insight to know that he is getting better or not. He believes these ideas to be true. If you feel the delusional belief is not coming even after giving Risperidone to a maximum tolerable dose for adequate period of time; then his drug can be changed to some other antipsychotics. Risperdal can be prescribed. To be noted is all antipsychotics are equally efficacious and they are chosen according to patient profile. Hope I have answered your query. Let me know if I can assist you further. Regards, Dr. Rohit Kothari"
},
{
"id": 223153,
"tgt": "Can displacement of mirena cause sickness?",
"src": "Patient: I have a girlfriend who has had the mirena in now for 3 years. She mentioned this past week that she thinks it has moved. She said everytime she pees it feels as if her mirena is going to fall out. Now she is so sick she cant move. Could the mirena be causing her to feel this way? Doctor: Hello and Welcome to \u2018Ask A Doctor\u2019 service. I have reviewed your query and here is my advice No , not at all . Displaced Mirena will not cause any sickness . Hope I have answered your query. Let me know if I can assist you further."
},
{
"id": 127407,
"tgt": "How can swelling on the collar bone be treated?",
"src": "Patient: my right collarbone has a lump, where it begins, and the whole bone is swollen, so it is larger than the collarbone on my left...my left collarbone is normal.....i have no pain.....i fell down some stairs a year ago and was in severe pain in my left shoulder, arm and hand....hydro and physical therapy helped,and i thought i had recovered.....until i noticed the swelling about 2 months ago.....it looks like i have osteoarthritis....i have arthritis in my hands and knees.....and i think in my neck from the fall ...my neck is always painful when i turn my head.....i am 81 years old.....my question is what , if anything, can i do about the swelling in my collarbone, as it is very noticable.... Doctor: Hello and Welcome to \u2018Ask A Doctor\u2019 service. I have reviewed your query and here is my advice. Probably you had fracture of collar bone when you had a fall and now it's healing hence the bump. kindly take X-ray. Hope I have answered your query. Let me know if I can assist you further."
},
{
"id": 89191,
"tgt": "What is the treatment for severe abdominal pain?",
"src": "Patient: my name is Melissa I am 35 years old, I had a tubal surgery done in 2013 after my son was born, I have a lot of lower tummy pain, can I get pregent after having a tubal done. I feel like I am pregent, my tummy feels full and it is cramping too. I have missed my last two months on my period. I am very frighten and don t know what to do. If you can help me that would be great. Thank you Doctor: HI.The simplest way is to get a pregnancy test and ultrasonography of the whole abdomen to decide the diagnosis. If this is negative then you are suffering from Pseudocyesis.Pseudocyesis : Means there is no actual pregnancy but the patient gets all the signs and symptoms. This is confirmed by a negative ultrasonography. He patient who has the extreme desire of having Motherhood again develops this sort of a problem. No one really knows the reason and the exact mechanism by which this happens. Consult your Senior Gynecologist who has seen such more cases for counselling and further management.There is a very rare chance that the tubes get recananlized and then you may be pregnant.IF you want to be pregnant, you van get the reversal done by surgery of tuboplasty."
},
{
"id": 99834,
"tgt": "Suggest medication for severe cold,breathing problem and runny nose",
"src": "Patient: i have had a cold now for 12 weeks, will not go have been to the doctors was given a course of tablets about three weeks ago, which did not work. have been back today to see GP said there is nothing he can do. i can't breath,i have a blocked and runny nose have cold sores inside my nose and around it, also on my lips. i have taken cold and flu tablets nasal spray, i have also had time off work due to go back this week, its getting me down 12 weeks is a long time? Doctor: Hello,Thank you for asking at HCM.I went through your history and would like to make suggestions for you as follows:1. I usually prescribe my such patients regular Levocetirizine + Montelukast for 4-6 weeks depending upon response.2. I also suggest them to have adequate amounts of liquids during day and a nutritious diet.3. Please avoid exposure to dusts, smokes and air pollution as much as possible.4. I would suggest you allergy testing for common air-borne allergens such as house dust mites, molds, pollens, insect proteins, animal dander, etc. This will help you identify the substances causing allergies to you as well as to know the measures to avoid them.5. If you have nose congestion, regular steam inhalation will also help you in a long run.Hope above suggestions will be helpful to you.Should you have any further query, please feel free to ask at HCM.Wish you the best of the health ahead.Thank you & Regards."
},
{
"id": 44691,
"tgt": "What treatment can be taken to improve the egg quality ?",
"src": "Patient: Im 31 years old, 5 2, 44 kgs. have been trying to conceive for 5 years, undergone IUI for 7 cycles and 1 cycle of IVF, the doctor said that the quality of egg is not very good, wht has to be done? Doctor: welcome to HCM egg quality depends on multiple things like hormone level,nutrition status.even hormones are influenced by thought process,thought of happiness and good mood will improve this.kindly add good quality of multivitamins like combination of vitamin E and folic acid,inocetol etc.omega 3 also hv proven roll in egg quality.u can add this too.nutrilite products are best"
},
{
"id": 177059,
"tgt": "Suggest treatment for hydrocephalus",
"src": "Patient: My son is 3 years old going on 4 in December he was born with hydrocephalus. He s been potty trained for 7 months now. Recently he has been wetting himself alot Could this be a sign of a malfunction. He s not showing any other signs of a malfunction he s overall a happy child. He s just been wetting himself a lot please help! Doctor: hydrocephalus can progress to cause pressure symptoms which can cause anything from false localized signs to severe neurological deficit. once acquired, if bed wetting has recurred , is suggestive of a pathology. hydrocephalus can cause pyramidal and extrapyramidal side effects and bladder and bowel incontinence.Get an imaging study done for the brain and take a neurologist opinion."
},
{
"id": 151942,
"tgt": "Anxiety brain issues",
"src": "Patient: Why do I never want to wake up in the mornings? I can sleep for 12 hours or more per night. My brain is not round, instead the left side feels flat and there are a few lumps on it and also some hollow spots. When I drink, a few times a week, I drink lots of alcohol in short periods of time. My ear tingles when I turn my head to the right sometimes. I feel as if someone is always judging me in a negative way. Doctor: Wash your head with soap twice a day and learn to read minds."
},
{
"id": 99889,
"tgt": "Can bleach fumes trigger asthma attack?",
"src": "Patient: Hello, I was wondering if bleach fumes can trigger an asthma attack. I was diagnosed with asthma about 20 years ago, and told it was \"environmentally triggered\". After about a year of mild breathing problems off and on, I seemed to have \"gotten over it\" (if that's even possible). Recently, I have 2 episodes that brought back that frightening feeling of not being able to breathe. The first was during a severe wind storm, where there was a substantial amount of dust in the air, so I went indoors, and within a few hours, I felt a lot better. This was about 9 months ago. Today, the janitor at the school where I work used a VERY strong bleach solution in her mop bucket when cleaning the cafeteria floor, and within minutes I was coughing and experiencing the old familiar tightening of the chest, and difficulty breathing. Almost 6 hours later, I am still coughing a scratchy irritated throat type cough, accompanied by some thicker phlegm, and I am still experiencing moderate difficulty breathing. I seem to be able to inhale pretty deeply, but it doesn't seem like I'm getting much air. =( Could the fumes from the bleach solution have triggered this? Doctor: Hi thanks for using healthcaremagic...Asthma is hypersensitivity of the Respiratory system...As a result of the increased response there will be an increased production of secretions ,ie.increased phlegm....Bronchospasm ie.constriction of the respiratory tree...initially starts as cough and later it becomes breathing difficulty..Anything which causes Asthma is called trigger factor...There are so many trigger factors---dust,smell,food....Hence the smell of bleach can irritate the respiratory tree and can act as a trigger factor...Anti-histamine and inhalers can help you...Thank you..."
},
{
"id": 6906,
"tgt": "What could the problem be if I am having irregular periods and not conceiving after having unprotected sex ?",
"src": "Patient: my periods are irregular and i am spotting or bleeding heavly my periods are irregular and i am spotting or bleeding heavly for about 2 and ahalf weeks at a time my husband and i are trying to get pregnet and we have had no luck we have a 8 year old all ready and no birth control in over 4 years what could this be Doctor: Welcome to Healthcare Magic You probably have hormonal disturbance. Have you got your thyroid hormones checked recently. Hypothyroidism can cause heavy bleeding and possible infertility as well. Even PCOD can cause similar disturbance. Also do get a thorough checkup by your Gynaecologist to rule out other problems like excess uterine thickness which can cause heavy bleeding which may require D&C procedure to confirm as well. Not to worry. There is a possible solution in every case. Have faith in God and your Doctor and do not hesitate to get a thorough examination by your Doctor in order for proper solution."
},
{
"id": 225643,
"tgt": "Had unprotected sex. Took Marvelon pills. Side effects?",
"src": "Patient: Hi, may i ask aquestion my wife and i had unprotected sex, she was in the 15th day of period unfortunately, 5 hours later she started taking Marvelon pills 3 times aday for 3 successive days , what are the side effects expected upon her health and how far this might prevent pregnancy and what is your kind advice ,waiting for your answer please Doctor: Hi,It is an oral contraceptive pill. It may bot help as much as morning after pill. It does not provide perfect contraception. Since already times has lapsed wait and watch for cycles to occur.Regards"
},
{
"id": 143627,
"tgt": "What does my MRI suggest?",
"src": "Patient: MRI REPORT postero central herniation of l-3/4 l-4/5 l-5/s1 intervertebral disc leading to smooth indentation over thecal sac with bilateral neural foraminal srenosis.# degenerative changes in lumbar spine. k/c/o pott s apine involving t-9 and t-10 vertebral bodies. followup mri for comparison. severe collapse of t-10 vertebral body SEVERE COMPRESSION OF THECAL SAC AND SPINAL CORD DUE TO POSTERIOR BULGE OF COLLAPSED T -10 VERTEBRAL BODY. Doctor: Hi, Welcome to HealthCareMagic.com I am Dr.J.Mariano Anto Bruno Mascarenhas. I have gone through your query with diligence and would like you to know that I am here to help you.Problem : postero central herniation of l-3/4 l-4/5 l-5/s1 intervertebral disc leading to smooth indentation over thecal sac with bilateral neural foraminal srenosis.# degenerative changes in lumbar spine. k/c/o pott s apine involving t-9 and t-10 vertebral bodies. followup mri for comparison. severe collapse of t-10 vertebral body SEVERE COMPRESSION OF THECAL SAC AND SPINAL CORD DUE TO POSTERIOR BULGE OF COLLAPSED T -10 VERTEBRAL BODY.Meaning : Due to Previous Disease (Tuberculosis) and also wear and tear, there are degenerative changes in the Spinal Cord and also nerve roots. Further Treatment can include 1. Drugs 2. Traction 3. Surgery Hope you found the answer helpful.If you need any clarification / have doubts / have additional questions / have follow up questions, then please do not hesitate in asking again. I will be happy to answer your questions. In the future, for continuity of care, I encourage you to contact me directly in HealthCareMagic at http://bit.ly/askdrbruno Best Wishes for Speedy Recovery Let me know if I can assist you further.Take care."
},
{
"id": 197049,
"tgt": "Suggest treatment for erectile dysfunction",
"src": "Patient: hiii doctor,my age is 25,hieght 5.11 . i used to masturbate regularly since 8 years so now im suffering with erectile disfunction,since 4 years.even iam also facing very fast ejaculation during masturbation.i consulted homeo pathy medicine ,but it does nt shown any result. wat to do.with in a year i have to marry.im not understanding wat to do. Doctor: HelloThanks forquery .You have been indulged in frequent habitual masturbation since many years and now facing problem of erectile dysfunction and early ejaculation .These problems are due to habitual masturbation and need self determination to over come this .addiction for masturbation Try to reduce the frequency of masturbation by keeping yourself busy in activities like sports ,reading ,social work etc so that you will not get free time to masturbate.Following measure will help you to boost up your confidence and getting good erection and delay ejaculation. 1) Practice regular exercise for 45 minutes followed by meditation for 1/2 an hour in the morning. 2) Take high protein diet rich in vegetables and fruits and Vitamin A,C,D,E.and Zinc 3)Take anti oxidants like Almonds 5-6 everyday4) Avoid alcohol and smoking..Dr.Patil."
},
{
"id": 43100,
"tgt": "Does the brown discharge after ICSI procedure mean that treatment is not successful?",
"src": "Patient: I am 11 days post embryo transfer through icsi procedure. 1 good quality egg transferred day 2, I am getting brown discharge and now it has turned black, quantity of discharge is minimal and today's home pregnancy test proved negative. Does this mean the treatment is unsuccessfull? Doctor: Hi,It could be implantation bleeding also. But you cannot say for sure.Home pregnancy test wont be positive so soon. Please get serum beta hCG test done after 2 days to confirm pregnancy.Regards"
},
{
"id": 214658,
"tgt": "Suggest natural remedy for reducing darkening of lips",
"src": "Patient: Hi there. Can you suggest any healing practices to get rid of the darkening of my lips? It's because of the wound that later been healed but it left dead skin cells at the sides of my lip. Please don't recommend some medicine that I would take. Just natural way of healing it! Thank youu! :D Doctor: Hello there the natural healing process would take a little while to lighten the lips.meamwhile you may try these natural ways Use a mix of lemon juice with honey and apply over lips and leave it overnight.Mix of almond oil with honey also helps.Coriander and cumin.powder mix may be applied.Rose water with rose oil can b applied..strawberry extract also helps.Honey can b Used Twice daily and Later ice cubes can also b rubbed..Hope these home Remedies help you out"
},
{
"id": 19349,
"tgt": "What causes chest pain?",
"src": "Patient: I've having chest pain since . I think I may have pulled a muscle. The pain is constant, although laying a certain way helps. I am slightly concerned and am mainly looking for reassurance that it's not an MI. I have no other symptoms, but to be proactive, I took an aspirin when I went to bed and another just now (6 hrs. later). I don't recall having this pain ever before. I suspect I may have pulled a muscle or eaten too much for dinner (although I have gerd & it's not like that). Any comments? I am a 64 year old woman, overweight, but no family history of heart disease. I see my doctor regularly. Doctor: HiThank you for your query. I have read your query and understood your concern. Chest pain can be due to various causes all of them different in characteristics. since you are worried about MI the typical characteristic of chest pain due to MI is sharp or dull aching over central or left side of chest usually radiating to your left arm or back of the neck or left shoulders . Along with chest pain you might have profuse sweating , dizziness.If you are having all these symptoms kindly visit emergency room immediately.I hope I have addressed your concern well. if satisfied kindly close this query with a good rating.Thank you wish you good health"
},
{
"id": 126343,
"tgt": "Suggest treatment for pain in the thumb",
"src": "Patient: Recently my thumb has hurt. Today I was unable to write using my right hand. I have issues with my ankles as well. I ve tried a brace, orthotics, a boot and steroids. The steroids helped for a short time, but the pain returned as the steroids wore off. I have recurring issues with other joints as well. Could this be RA? Doctor: Hi, Pain in thumb after injury or pain in ankle joint does not mean that you have RA. I suggest you get the blood tests for arthritis and consult an orthopedic surgeon for the thumb injury. Hope I have answered your query. Let me know if I can assist you further. Regards, Dr. Gopal Goel, Orthopedic Surgeon"
},
{
"id": 184391,
"tgt": "What could cause tooth infection?",
"src": "Patient: HelloI am 41 220lb good health but I have had a tooth infection for about a year now that I have been able to drain on a daily basis but recently I have been having dorado pain at the muscle connection at the elbow. Hand it is even hard to pick up a glass of water with out being in pain. Could the tooth have any effect on my arms? Doctor: Thanks for your query, I have gone through your query.There is no relationship between your arm and tooth infection.Your tooth infection can be secondary to decay, Consult a oral physician and get a radiograph done to rule out the infection and get the tooth treated with RCT or extraction.I hope my answer will help you, take care."
},
{
"id": 160268,
"tgt": "My friend is suffering from blood cancer, please help",
"src": "Patient: Hello sir, My friend is suffering from blood cancer. His age is 19 . Cancer is in beginning stage. what is the chance for death ? Doctor: Hi and welcome to healthcare magic forum. Blood cancer is a very broad terminology. There are many types of blood cancer and the survival rate will depend upon type and severity of the cancers. Now a days many of the cancers are treatable and we have bone marrow transplant. It is very difficult to tell that all blood cancers will lead to death or all can be cured, without examining the patient or seeing the patient blood reports and bone marrow result."
},
{
"id": 79966,
"tgt": "What is the treatment for wheezing?",
"src": "Patient: As i said earlier am taking Montair Lc and Brofy 100 for last 2years till date, the moment i stop talking the pills then the problems starts i.e sound from the lungs, dry mouth, blurred vision, drowsiness, i cannot sleep at night without taking these pills, so therefore i only take them when i fell like to have them. earlier i use to take them daily but now a days m taking them with a two day gap so that i can have less side effect of it. but i know that is not the solution. Two year back when i started my treatment my doctor told that i have to take these medicines regularly for two years and the will be ok in all aspect, but it seems that taking these medicine can only make me ok. Also i would like to share that i have undergone a nasal surgery ( septoplasty fess ) in the month may 2012,after that my nose is ok in every aspect but the throat & lung problem is still there. What should i do to cure this problem completly from the root. Doctor: Thanks for your question on Health Care Magic. I can understand your situation and problem. By your history and description, possibility of bronchitis is more in your case. Inhaled treatment is the gold standard treatment for bronchitis. Inhaled bronchodilators and inhaled corticosteroid (ICS) are first line treatment for bronchitis. It is having excellent effect on improvement of lung functions. They are safe for long term use. So for optimal control of your wheezing, better to consult pulmonologist and start inhaled treatment. Hope I have solved your query. I will be happy to help you further. Wish you good health. Thanks."
},
{
"id": 131789,
"tgt": "Is it safe to take Prednisone for Polymyalgia rheumatica?",
"src": "Patient: A person was born with congential heart problems --- lots of them. Has had aorta mechanical surgery in 1998, carotid artery on one side., pace maker etc. Recently was told he has PMR and put on Prednidone of 20 mg. a day and then not tapered off -- abruptly stopped for nearly a week. Then put on 15 mg. for 4 weeks and 7-1/2 for four weeks...after taking the 7-1/2 mg. he ended up in the hospital .. he has had congestive heart failure 4 times in less than a month. Could this be related to the Prednisone not prescribed correctly? Doctor: Stopping the corticosteroid abruptly can cause heart failure. Corticosteroid has a diuretic effect . Sudden withdrawal may result in loss of diuretic effect and cause congestive cardiac failure"
},
{
"id": 44956,
"tgt": "Minimal free fluid in POD. What does it mean?",
"src": "Patient: hi,i m 27yrs old female,i m planning to conceive ,today is my 12th day of period n my follicular report is RT9*9mm n LT f1 12*9mm hemorrhagic/rupton f2 10*7mm. endo thickness 6.3mm .impression is minimal free fluid in POD.what does it mean ? is there chances of conception ? please help me :) Doctor: hello welcome to health care magic yes every thing seems quite normal ,,go for concieving on 12 ,14 ,16 ,18 day ..it vill help you out surely..please lie down on your right side after love making.this will help to quick concieve. all the best take care payal mail me at payalrawat16@yahoo.com"
},
{
"id": 217974,
"tgt": "Hurt leg black & blue,noticed swelling and pain due to an injury",
"src": "Patient: I dropped a steel gate on my big toe yesterday and it is black & blue (area is a band going around toe from root of toe nail area to second knuckle of big toe) Only slightly swollen and on a scale of 1-10 the pain level is a 2 or 3. I have not taken any pain reliever as not needed yet. It doesn't hurt when I stand on it. What do you recommend. Doctor: Hi,This does not appear to be a serious injury and is likely to recover spontaneously in the next few days. You may find the nail turns black. I suggest elevation when possible and paracetamol and ibuprofen for the pain as needed.Regards,Dr K A Pottinger,MBChB FRCA"
},
{
"id": 140069,
"tgt": "Suggest medicine for paralysis",
"src": "Patient: i am 72 yrs old and suffring from paralisis in right side since last 7 year ago .dr. advice to me on 4-1-2012 after addmite in his hospitl 5 days 1.platloc-as 75mg tab 1/day,epilive 500mg tab-3/day,frisium 5mg tab-twise a day,limcee tab -3/day ,lasix 40mg tab 1/day,pozt-dsr tab twise a day,atorsave 40 - 1/day,cognicid 800mg 3/day,alonerv -1/day . please check and advise better medisin for improve my health Doctor: Hi, It would appear from your description of the paralysis and medications that you suffered a stroke in 2012. You've not really said much about how you are currently doing but my guess is that things are stable. After 7 years from a stroke the paralyzed portions of the body will have reached a stable point of recovery. Medication wise, it would appear that your doctor is covering you for the correct and adequate items that could cause problems in the future such as cholesterol and it also appears he is giving you some type of supplementation for good health of the brain tissue itself. I might recommend your doctor look into your B12 and Vitamin D levels in the bloodstream. Those 2 things are very important to good nerve health following a stroke. I would talk to the doctor about doing blood work and looking for B12 levels that are no less than 450 and D levels that are 60-80. Otherwise, the balance of your medication list appears to be quite reasonable and I'd suggest it continue. You may also benefit from some form of personal REHABILITATION EXERCISE therapy depending on how paralyzed the right side may be and if your speech and language have been affected you may be able to benefit from speech therapy as well if you never had any in 2012. Hope I have answered your query. Let me know if I can assist you further. Regards, Dr. Dariush Saghafi, Neurologist"
},
{
"id": 12305,
"tgt": "Suggest medication for psoriasis",
"src": "Patient: i have psoriasis and i dont know if these two are connected in any way but everytime i travel and come back, usually 8/10 times, i would get a realli bad throat infection, i thought the first timme that i was allergic to anything, but im not as far as i know, could i please get some advice on this. kind regards nasra salah Doctor: Hi,As you said, you suffer from psoriasis. It is a chronic and relapsing skin disease. I can understand your anxiety for your disease. But,you have to keep patience. You might get relief slowly. And due to some factors it might relapse. You are right,throat infection might precipitate the disease. In fact any infection.....Remember it is a controllable disease.You consult nearby dermatologist for firm diagnosis and perfect treatment.I would like to suggest few things as follow....- acitretin 25 mg daily - methotraxate 15 mg weekly in three devided doses of 5 mg 12 hourly. - Blood tests ..routine & LFT may be done at regular intervals- Steroid with salicylic acid oint may be applied twice a day on the psoriatic lesions.- Soap bath should be avoided. - All precautions may be taken to avoid dryness of skin. Winter might worsen due to dry weather. So, take care.After few months,you may be alright. But, maintenance therapy may be continued. And if there is relapse, you may restart the treatment.Thanks.Dr. Ilyas Patel MD"
},
{
"id": 32686,
"tgt": "Suggest treatment for stabbed wound in the chest",
"src": "Patient: I've accidentally stabbed myself in the chest with a craft knife, what do i do? The wound is clean and not bleeding much. The size is approx 1cm length by 2/3 mm deep. my breathing is ok but I've got a kind of indigestion feeling and a slight feeling of pressure. I'm just a little concerned in case i'm bleeding internally. I've patched it up and it's been about an hour ago now. Am i ok to go to sleep Doctor: Hello, welcome to HCM. You are absolutely fine. The wound seems to be deep. it heals on its own but takes lot of time. Also you should take care that it should not be infected. The wound should be cleaned with antiseptic solution like savlon if the knife was soiled.i advice you to take appointment with doctor. with 3-4 sutures and local soframycin like ointment , the chances of getting infection can be avoided. All the organs are fine. Nothing to worry. Take care."
},
{
"id": 117218,
"tgt": "What are the symptoms of elevated ammonia level?",
"src": "Patient: What are the symptoms of an elevated ammonia level? My 30yr old son takes Depakote for a seizure disorder and bi-polar he also has autism, so I can t ask him about how he feels. For the past few weeks he has been very tired, clumsy, his eyes are beginning to take on a yellow hue though no one seems to notice this but me, and he has black circles under his eyes. All the usual suspects have been ruled out, infection, anemia etc. I was just sitting here at dinner watching fall asleep at the table and the thought popped into my head perhaps his ammonia level is elevated. I can t find much info on what the symptoms, so any information would be much appreciated before I drag him through another blood test. Thank you for your time Doctor: Hi and thank you so much for this query.I am so sorry to hear about these symptoms that your son has been experiencing. Unless the liver is not functioning normally, there would be no increased ammonia levels. With you reporting a yellowish discoloration of his eyes, it could be possible that he has an ongoing liver disease. Ammonia in excess levels would induce sleepiness, obnibulation and eventually coma. This would be unlikely without a severe liver disease. Get hi to the doctors should you be concerned but ammonia per se without a suggestive history would be unlikely.I hope this helps. I wish you well. Thank you so much for using our services and do feel free to ask for more information and clarifications if need be."
},
{
"id": 76730,
"tgt": "What causes feeling of something is blocking inside throat or chest?",
"src": "Patient: This is the first time I experience this. I sneezed really loud and hard earlier, which was a relief coz it felt like something was tickling my nose, then right after that, until now, it feels like there's an air-bubble or something blocking inside my throat or chest. But I could definitely feel it right in my chest part, but half feels like as if I tried to swallow a whole meat and it got stuck in the middle of my esophagus near the middle of my chest. I'm worried, that there's reaelly something solid blocking in my heart or it could be something serious to be ignored, or something I could do something about at home. Please help. Im 28 years old, female, with family history of heart problems, been also having throat infections at least 3-4 times a year (which they said is not good and my tonsils should already be removed or else bacteria could reach my hear). I would really appreciate your help, big time. Doctor: Hi thanks for contacting HCM...First try to rule out respiratory causes particularly first starting from nose by rhinoscopy to look for polyp ....Post nasal drip can lead such blocking sensation in throat.....Second rule out chronic pharyngitis By throat examination...As chronic infection might lead dry mucosa and such symptoms....Here esophageal cause like achalasia cardia has to be ruled out if dysphagea present...X ray , manometry like investigation helpful....If needed endoscopic or bronchoscopic examination done.Your physical examination in detail has to be done with detail history for work up....Take care....Dr.Parth"
},
{
"id": 216104,
"tgt": "How to relieve pain caused by torn ligament?",
"src": "Patient: I had surgery approximately 8 years ago for a broken ankle and later torn ligament. I fell two days ago and it feels like I may have torn my ligament again as it feels like the ligament is moving around. Should I put ice on ir ot heat to help with the pain until I can get into a doctor? Doctor: Hello and Welcome to \u2018Ask A Doctor\u2019 service.I have reviewed your query and here is my advice.Yeah, this is hard to treat. The ligaments grow/heal slowly if at all. Generally, the advice is to immobilize, ice, and aspirin in the short run.Hope I have answered your query. Let me know if I can assist you further."
},
{
"id": 156720,
"tgt": "I had ovarian cancer long back and now i am having swollen lymph nodes. What does this indicate?",
"src": "Patient: 11yrs ago I had ovarian cancer. Had complete hysteric. Followed by 6 month chemo. Today I am told I have swollen lymph nodes With what appears to be growth on them located on the corresponding Side where the cancer was. Is this an indication there is cancer Present in the nodes? Thanks. YYYY@YYYY Doctor: yes it is possible to have cancer in the lymph node and hence required to do biopsy to confirm the same."
},
{
"id": 7695,
"tgt": "Pimples with scars, redness on face after using melacare hc. Solution?",
"src": "Patient: hiii..........i m neetu my age is 22 years i have suffering from pimples and their scars from last 4 years the pimples are more in winters than summar season i tried everything on my face but no any result of them now pimples are almost disappear and scars left i used melacare hc tonigh my face get red then what i should? Doctor: Hello. Thanks for writing to us. The pimple scars on the face are most commonly caused due to reactive pigmentation in the area. Such scars do get lightened on their own with th epassage of time and avoiding exposure to sunlight. For immediate effect, techniques like dremabrasion are useful. I hope this information has been both informative and helpful for you. Regards, Dr. Rakhi Tayal drrakhitayal@gmail.com"
},
{
"id": 14216,
"tgt": "Suggest treatment for rash,itching and bad odor in armpits",
"src": "Patient: I am an active tennis player runner who has had very little body odor under armpits though I do sweat quite a lot. These past 2 months, I have experienced a smelly odor that doesn t seem to be cured with deodorant or even completely gone after shower. I ve stopped using deodorant (have never used much in past) and I ve started washing with anti bacteria soap. It doesn t seem to be the cure. The rash is under each armpit in different areas and itches horribly. Please advise. thx. Doctor: Hello and Welcome to \u2018Ask A Doctor\u2019 service.I have reviewed your query and here is my advice.1. Your concern may be intertigo secondary to candidial yeast infection.2. You can try ketogold anti fungal soap for body wash instead of antibacterial soap.3. Try lulifin cream once daily bed time for 30 consecutive days, even if there is improvement.4. Dust abzorb dusting powder twice daily as a maintenance.5. Bath twice daily and try to maintain dry environment in the armpits.6. Avoid wet clothes.This will help you, don't worry.Hope I have answered your query. Let me know if I can assist you further.Regards,Dr. Sandeep Golla"
},
{
"id": 215432,
"tgt": "Suggest treatment for painful hip bursitis",
"src": "Patient: I am 62 yrs young, 5ft 4 inches tall & weigh approx. 240 I have very painful Bursitis in my hip. What causes bursitis? It feels like where the hip bone meets the leg bone down deep inside there is a sharp constant pain. I cannot sit on anything hard, I usually sit on a pillow, it hurst when I walk & go up stairs. The pain goes half way down the back of my leg to my knee. In August it will be 2 years. I have been to therapy and had & had something called needeling, & laser therapy, It helped the very 1st time but noit after that. I have had massage therapy & that made it hurt worse. I have seen a rhuemotologist & she said I do not have RA. I went to the chiropractor and he is the one who told me it was bursitis. Then I went to the Dr. & had it xrayed, he said he did not see anything on the xrays. It was so bad I could barely walk. The Dr. gave me a cortisone shot and I was already on anti-inflamatories. The shots only last about 3 to 4 months and I have had 2 of them and now it is hurting pretty bad so it is time for another shot. The shots do not make the pain go completely away but at least it is bareable. This just came on al of a sudden. I did not injure my hip. The cortisone shot is a steroid so that causes weight gain, sleeplessness and messess with my blood sugars although I am not diabetic that I know of. I hope you can help me. Doctor: Hello, Wear and tear on the area causes irritation. Mostly, the lining wears out and there is bone rubbing on bone in the hip joint. If the lining is gone we don't have a good fix for it yet. We likely will in a few years. And the injection of lubricant (SYNVISC) anti-inflammatories, building up the surroundings and learning to ease off on pressure on the area (physical therapy) are the usual treatments. Hope I have answered your query. Let me know if I can assist you further. Take care Regards, Dr Matt Wachsman, Addiction Medicine Specialist"
},
{
"id": 139016,
"tgt": "Suggest treatment for disc protrusion bulge",
"src": "Patient: T12-L1 LEVEL DISC DESICACION AND MODERATE LEFT SIDED DISC SPACE NARROWING THERE IS 5MM MIDLINE DISC POTRUSION DISC WITH MILD SPINAL STENOSIS L4-L5MODERATE DISC SPACE NARROWING THERE IS DISC BULGE 3MM DISC POTRUSION LEFT FACET HYPERTROPY L5-S1 DISC POTRUSION ON LEFT WHICH ABUTS THE LEFT S1 NERVE ROOT FACET HYPERTROPHY MILD FORAMINAL STENOSIS IS REALY Doctor: Hi, Time and then we have been telling patients to write their complains along with MRI findings, to help you better. all such reports are weighted on the basis of your symptoms and examination an then proper treatment is initiated. your MRI is suggestive of some compression of the nerves and some degenerative changes. If you can provide us with your symptoms, then only can I help"
},
{
"id": 98277,
"tgt": "How can one cure sore throat, cough and runny nose while having asthma?",
"src": "Patient: two days ago started with a sore throat then sneezing and slight coughing then running nose then block nose and phlegm in chest which is green and when I cough I getting pain around both sides of ribs. I went to the doctor because I have asthma and got antibotics. Doctor: Hello, So you explain that you have started the treatment. Just continue the treatment and take a lot of liquids. Hope I have answered your query. Let me know if I can assist you further. Regards, Dr. Jnikolla, Pulmonologist"
},
{
"id": 45157,
"tgt": "Does agglutination reduce male fertility ?",
"src": "Patient: Doctor Please have a look at my semen analysis report and kindly suggest Volume- 2.5 Ml Appearence- Normal pH-8 Liquefaction- 30 min Sperm Count/ml- 127.5 Sperm Count/ejaculation- 255 mil Motility (1 Hour)--50% Motility Grade Progressive (PR)- 20% Not Progressive (NP)-30% Immotile - 50% Normal Forms -60 % Abnormal Heads 20% Abnormal midpieces - 10 % Abnormal Principal pieces -08 % Excess residual cytoplasm -02% WBC/ml - 400,000 Agglutination - Present Doctor: Hi Welcome to health care magic \u00a0\u00a0\u00a0\u00a0\u00a0you can become father ,but it will be better if you improve your semen quality with good diet ,stopping smoking & alcohol ( if you are taking)and antioxidants & vitamin E \u2018Hope I have answered your query, I will be available to answer your follow up queries, \u201cWish you Good Health and speedy recovery\u201d Disclaimer"
},
{
"id": 56481,
"tgt": "What medicine shall take to control my Billirubin level?",
"src": "Patient: Hi, i have gilberts syndrome. It doesn t really bother me that much it s just the mild jaundice i get from it. I eat well, exercise all things i need to do, but it still bothering me. I have been to my doctor but he said i don t need treatment because it s only mild. Is there any drug i could take to rescue the billirubin? thanks ash Doctor: Thanks for posting your question on HCM!I appreciate your concern towards the bilirubin level.Gilbert's syndrome is a benign, familial disorder characterized by intermittent jaundice. It is due to low activity of the conjugation in the liver which normally detoxifies the bilirubin in our body.Gilbert's syndrome doesn't require treatment. The bilirubin levels in your blood may fluctuate over time and you may occasionally experience jaundice. But this usually goes away on its own, and it doesn't cause any ill effects.Each doctor you visit should be told that you have Gilbert's syndrome, so any medications can be selected with this in mind.Lifestyle modifications:Eat a healthy diet full of fruits and vegetables. Avoid extremely low-calorie diets. Stick to a routine eating schedule and avoid fasting or skipping meals.Avoid alcohol, manage stress, consider exercise or quiet time alone to cope with stress.Take care of your health."
},
{
"id": 69924,
"tgt": "Noticed a lump on the foot near the ankle",
"src": "Patient: I have noticed that my 20 month old daughter has developed a lump on the top of her foot near her ankle. When I first notice it I thought it may me an insect bite but on feeling it, it feels like bone. She is not in discomfort, she is walking fine and it doesn't appear to hurt when touched firmly. Doctor: Hi and welcome to HCM. Thanks for the query.It could be bone exostosis and this is benign condition which doesnt require treatment in most cases unless there is significant pain. also it could be some other subcutaneous lesions such as fibroma or lipola and in this case it can be surgically managed. Wish you good health. Regards"
},
{
"id": 153385,
"tgt": "Can fever and headache be connected to removed tumor in stomach?",
"src": "Patient: Hi, may I answer your health queries right now ? Please type your query here...I was diagnosed with a beneign carcinoid stomach tumor last December. I have recently been running a low grade fever and headache in the lower part of my head in the back. Could there be a connection to the tumor that was removed? Doctor: Hi,Thanks for writing in.A benign carcinoid tumor is a tumor which might cause release of neurochemicals which increase blood pressure and cause headache.It is possible that the benign tumor might have secreted certain chemicals which are directly related to causing headaches. Such tumor cells might be present elsewhere and they might continue to release the chemicals. I am not sure if you have been detected as multiple endocrine neoplasia which is also called MEN syndrome. In this condition there might be benign tumors in several places which might cause release of chemicals even after the removal of the stomach tumor. This requires detailed assessment regarding possibility of other similar tumors. Please discuss with your doctor and get complete evaluation done for it. Please do not worry."
},
{
"id": 76307,
"tgt": "What causes fluttering and twitching feel under the right rib cage?",
"src": "Patient: For the past couple of weeks I've been feeling a fluttering / twitching feeling under or by my right rib cage. I have a fatty liver and didn't know if this is a sign that something may be worsening? I've read it could be detoxing too. It doesn't hurt, but is more annoying (like an eye twitch). I'm overweight and can't reach under my rib cage like some suggest. Should I go to the doctor? Doctor: Thanks for your question on Healthcare Magic. I can understand your concern. In my opinion, we should first rule out arrhythmia (rhythm disturbances in heart) because this can cause cause Fluttering and twitching sensations in rib cage. So consult doctor and get done 1. Ecg 2. 2d echo 3. Holter monitoring (24 hours continuous recording of ecg). If all these are normal then no need to worry for heart diseases. Sometimes, stress and anxiety can also cause similar symptoms. So avoid stress and tension, be relax and calm. And loose some weight. This will cause subjective wellbeing and treat your fatty liver too. Don't worry, you will be alright. But first rule out arrhythmia. Hope I have solved your query. I will be happy to help you further. Wish you good health. Thanks."
},
{
"id": 55056,
"tgt": "Is abdominal pains and heavy bleeding normal after surgery?",
"src": "Patient: I am 46, peri menopause, 4 hernia surgeries in one year along with gall bladder and merkels divertcuilm surgery. Opened from sternum to belly button now have mesh holding me closed. Problem:pain around my belly button sight ( last surgery 12/12) and bleed heavy after heavy lifting...... Normal? Doctor: hi.noted history of several surgeries for hernia and recent abdominal pain associated with heavy bleeding. is the pain associated with redness, swelling or warmth over the operative site or not? post-operative pain can occur even for those who have been operated on several years back. pain/discomfort must be tolerable in character, resolving spontaneously. if the pain is persistent/progressive, it is best if you do a follow-up consult with your surgeon for physical examination and clinical evaluation. re your heavy bleeding, it is best if you consult with a gynecologist with regards to this matter. your abnormal menses may be due to your menopausal status, but other gynecologic pathologies for abnormal uterine bleeding (such as endometriosis, myoma, etc.) must also be ruled-out.. diagnostics (such as ultrasound, blood counts, etc.) and management (medicla and/or surgical if indications are found) will be directed accordingly.hope this helps.good day!!~dr.kaye"
},
{
"id": 16040,
"tgt": "Rashes across the chest. Could it be lyme disease or bulbar palsy?",
"src": "Patient: Yes - my brother in law has been trying to get information on both lyme disease and bulbar palsy als. He has many unexplained issues which make him want to question whether his diagnosis of bulbar palsy could really be lyme disease and I have read that lyme could be mistaken for als. He only had one blood test for lyme and it was negative. He had a rash across his chest which has been unexplained but they haven t suggested getting another blood test? Doctor: Hi , Welcome to HCM. Thanks for writing in. Lymes disease is caused by borrelia.It starts with skin rash which wax and wane for long periods after many years it can cause neurological features.A serological test for lymes disease can be helpful for diagnosis. A visit to dermatologist will allow confirmation of diagnosis and subsequent treatment. Hope this helps. Wishing you good health. Regards. DrSudarshan MD Dermatology"
},
{
"id": 182029,
"tgt": "Suggest treatment for soren boil in gums",
"src": "Patient: I notice about week ago I had a sorenes on my upper left side of my gum. when looking future I notice that I had a boil on and that it has some puss inside of It . It doesn't hurt but it' there, and it look like it getting worst. beside going to my healthcare, is there anything that I can do on my own? Doctor: Accumulation of pus indicates chronicity of infection. Definitive treatment would be prophylactic antibiotics with relieving the area accumulated with pus."
},
{
"id": 101869,
"tgt": "Can I take anti-histamines for feeling tightness in throat and wheezing?",
"src": "Patient: Hi I suffer from allergies so take regular over the counter anti-histamines which usually do the trick. But sometimes my throat can feel tight and I get quite wheezy. Nothing too scary but not pleasant either, just wondering if I should see my GP or persevere with the anti-histamines? Thanks Doctor: HI, thanks for using healthcare magicIf you have episodes of wheezing, you should consider visiting your doctor for an assessment to see if additional medications, other than the antihistamines are required for control.If there is a history of wheezing then the use of inhalors may be suggested.Topical steroid inhalors would be used if there are any symptoms of post nasal drip. These may also require a prescription.I hope this helps"
},
{
"id": 226368,
"tgt": "Stomach pain, headache, diarrhea, fatigue. Varying pregnancy test results. On birth control",
"src": "Patient: Well I ve taken two pregnancy tests one was positive and the other was negative and I ve had stomach pains and head aches and diarrhea and fatigue im also six days late for my period I m on birth control but don t always take it at the same time every day so I m guessing there s a chance I could be pregnant but I m not sure, help please Doctor: Hi, Thanks for the query. Urine pregnancy test kits sometimes may not detect early pregnancy. As you have not taken the pills regularly we cannot completely rule out the possibility of pregnancy. So you better go for blood test for pregnancy which will give more accurate result or urine test with early morning midstream urine sample. Take care."
},
{
"id": 16917,
"tgt": "Suggest treatment for pain and pus formation in scar of heart surgery",
"src": "Patient: I underwent open heart surgery in 2000 and developed keloid scar in chest. Now a days it is paining and sometimes fuss is coming out of it. I am 75 years old and this problem is for the last five years and the severe pain and burning sensation is there always. My consultations with cardiologists was apply coconut oil. What is your advise to relieve pain? Doctor: Hello, Your symptoms are suggestive of a keloid or possibly a superficial infection. I advise you to meet a dermatologist. They would be able to treat the same with intralesional steroid injections or cryotherapy. You could also ask your doctor for analgesics for the pain. Hope I have answered your query. Let me know if I can assist you further. Regards, Dr. Pranav Balakrishnan, General & Family Physician"
},
{
"id": 115820,
"tgt": "Suggest treatment for B12 deficiency",
"src": "Patient: I have been diagnosed with a B12 deficeincy. I received a shot and felt better. 10 days later I received another. I was beginning to get low on energy after the second shot on day 3. Days 4-7 my health dramatically declined. My walk and balance was off, I was emotional and every task seemed like so much work. I received a shot on day 7 and 8. Today is day 8. I feel about 50% better but I'm tired and my walk is off. I also have a headache. I saw a hemotologist yesterday to do more blood work. He's looking for an intrinsic factor antibody (or something like that). Should I be seeing a neurologist as well? Doctor: Hi, dearI have gone through your question. I can understand your concern. You jave vit b12 deficiency. So treatment is vit b12 injection. You should complete the course of vit b12 injections along with folic acid. Neuropathy is due to vit b12 deficiency. No need to see a neurologist. When your vit b12 level come back to normal, you will also feel improvement in neuropathy. No need to worry. Just be relaxed. Consult your haematologist and take treatment accordingly. Hope I have answered your question, if you have doubt then I will be happy to answer. Thanks for using health care magic. Wish you a very good health."
},
{
"id": 87535,
"tgt": "What causes mild abdominal pain?",
"src": "Patient: Hi, I am 49, male and been suffering with mild abdominal pain. I have Non-Hodgkins lymphoma and my CT of the abdomen shows mild thickening seen in the terminal ileum. I had a Colonoscopy with biopsy. Nodular mucosa was found in the rectum. The rectal mucosa appeared somewhat nodular and multiple biopsies were taken. The rest of the colon appeared normal including the cecum, appendiceal orifice and ileocecal valve. The terminal ileum was intubated and appeared to have lymphoid hyperplasia with multiple biopsies taken. No ulcerations, stricture or mass lesions seen in the terminal ileum visualized multiple biopsies were obtained and sent to pathology. The mucosa apperared less nodular on fetroflexion. What does this mean? Doctor: HIWell come to HCMWhat ever the findings are there these are suggestive of multiple pathologies of gastro-intestinal tract, some are significant some may not be specific, over all findings need to be correlated with the clinical condition and further investigations may require for final diagnosis, hope this information helps, take care."
},
{
"id": 25268,
"tgt": "Suggest medication for elevating BP levels",
"src": "Patient: Hello Doctor, I m Hemant. My BP is going high. 6 days back it was 170/120 n now it is going around 150/110 to 130/95. I m preparing for civil services so my routine is to keep sitting on in my study n keep studying. physical activity is normal n m doing workout for 1 hour daily. All the reports regarding my lipid profile, blood serum, urea all r normal. i always hav a headache n not at all able to concentrate on my study. Plz help. Doctor: Hello and thank you for using HCM.I carefully read your question and I understand your concern. I will try to explain you something and give you my opinion .We talk about hypertension if we have mean value that exceeds 140 / 90 mmHg.A person might have high value during emotional and physical strees so its mandatory to judge on mean values during day and night. There are two types of hypertension : Essencial hypertension or idiopathic, meaning that it has not a definite cause and secondary hypertension, meaning that it's a secondary couse of it.This is found in younger ages and the cause might be renal , endocrine or any cardiovascular anomaly.Essential hypertension is a chronic disease and it should be treated all live.High blood pressure values can give you headache, blurred vision, dizziness ect.You don't mention your age but as I explained above it is essential to exclude secondary causes if you are young. As you explain you have all ready done blood analyze and they are normal. This is a very good thing. If I was your treating doctor I will recommend even a cardiac echo, an abdominal echo to evaluate renal structure, your urinary chatecolamins and a holter pressure monitoring. All these can give a better view of your mean blood pressure values and exclude secondary factors.If the pressure holter monitoring finds mean values above 140 / 90 than you shoud treat it with medications. Mean wile avoid excessive salt intake I coffee and energy drinks becouse they rise blood pressure values.Hope I was helpfull. Best regards."
},
{
"id": 47641,
"tgt": "Suggest natural remedies to remove kidney stones",
"src": "Patient: Hi ...em 42 years old lady ..suffering from a kidney stone of 12 mm.. left kidney..but not suffering from any severe pain...the stone is lying between L2 and L3.... em taking ayurvedic medicines(potrate-m, demoiish,renalka,berberis vul.q)...the stone is slightly conical in shape...i dont want to go for a surgery can u plz give me some suggestions so dat I can get rid of it through natural remedies...I ll be very much obliged...thank you.. Doctor: Hi, welcome to HCM.Kidney stone which are>6mm in size usually do not pass spontaneously.Do basic urine report with culture, S.creatinine and sonography.By these, we can confirm that stone is obstructing urine passage or not.Obstructed stone require immediate urological intervention.Maintain adequate hydration, salt restriction, and avoid non veg food as much as possible. These simple measures can prevent the kidney stones to a considerable extent.You can then consult urologist or you can get back to me with above reports.I think this would be helpful to you.Best wishes. TC.Dr Jay Patel."
},
{
"id": 129869,
"tgt": "Why am I having pain in my right shoulder?",
"src": "Patient: I have pain in shoulder that radiates into right shoulder blade, armpit, down my arm into the palm of my hand. Previously had 2 cortizone injections over last 1.5 yrs. for rotator cuff inflammation. Never pain in my armpit, forearm or hand though until now. Thanks! Doctor: Hi there. Firstly rotator cuff pain is more localized to the region around the shoulder. What you might be having here would probably be a nerve compression. This can happen at the level of your spinal cord, vertebrae to the many nerves supplying your hand. So if a patient presented to me that's what i would be ruling out.Especially since you mentioned that your pain is radiating, that is why i would be suspecting this all the more. You need a checkup of your right upper limb muscles to see if there is any associated weakness or any sensory loss, and a spine evaluation as well. Its also more likely if you had any history of doing any strenous activity or trauma in the recent past. So i would suggest that you get yourself evaluated for the same."
},
{
"id": 71957,
"tgt": "Suggest therapy for a problem in the lungs",
"src": "Patient: Hi...good evening Dr. I have expirienced of flueral effussion ...then Pulmunary decided to make an open biopsey due on effussion in my right lung that cause of mal breathing...that on my x ray seen a flueral mass...Is that operation can naturally epirienced a expanding of a half of the due on air leak of the lungs? and until now after four months of my operation i was feel emmune of my both arm... is the effect of my operation thats why i feel like this?? what should i do? Please i need your advice...Artor Asedo , thanks Doctor: PLEURAL EFFUSION WITH MASS MAINLY DUE TO MALIGNANT CANCER KINDLY GO TO ONCOLOGIST AND ALSO PLEURAL TAPPING FOR TEMPORARY RELEIF"
},
{
"id": 52695,
"tgt": "Suggest treatment for jaundice",
"src": "Patient: Hi Dr, My Brother suffering from jaundice for past two weeks. He had taken Ayurveda medicine two times from kerala.(weekly once). But even after taking medicine two times still there is no improvement in his health. His eyes looking yellow compared to before. He is not feeling hungry past two weeks. I would like to know how much time will it take to cure? and do we need to take scan? please give some suggestions. Thanks in advance Doctor: Hi and welcome to Healthcaremagic. Thank you for your query. I am Dr. Rommstein, I understand your concerns and I will try to help you as much as I canWell, jaundice is just a sign and not diseases, and treatment depends on exact cause.Jaundice is usually sign of certain liver disease and high bilirubin value. Causes of jaundice vary from non-serious to potentially fatal. Levels of bilirubin in blood are normally below 1.0 mg/dL (17 \u00b5mol/L) and levels over 2-3 mg/dL (34-51 \u00b5mol/L) typically results in jaundice. High bilirubin may be due to liver diseases such as cirrhosis or hepatitis, infections, medications, orblockage of the bile duct In the developed world the cause is more often blockage of the bile duct or medications. Blockage of the bile duct may occur due to gallstones, cancer, or pancreatitis. Medical imaging such as ultrasound is useful for detecting bile duct blockage.Treatment of jaundice is typically determined by the underlying cause. If a bile duct blockage is present surgery is typically required, otherwise management is medical.Medical management may involve treating infectious causes and stopping medication that could be contributing. I hope I have answered you query. If you have any further questions you can contact us in every time.Kindly regards. Wish you a good health."
},
{
"id": 12942,
"tgt": "Could rashes and itching be a reaction to sutures?",
"src": "Patient: I had a lumpectomy 11 days ago. This was my 2nd lumpectomy re-incision. The surgeon used dissolving sutures and glue. I'm experiencing a red bumpy rash surrounding the incision and my body is itching on my neck, chest and back. My doctor said that it didn't appear to be infected and told me to use Balmex on the rash. The rash is tolerable, but the itching is horrible. I continually take Benadryl. Could this be a reaction to the sutures? If so, what is the treatment? Doctor: Hi,It may be irritant contact dermatitis caused by either suture material or dressing materials. Consult the dermatologist for the perfect diagnosis and proper treatment. I would recommend you to apply mild steroid cream on the affected areas. And take antihistaminic like cetirizine to relieve itching. If needed, take a course of oral steroid in tapering dose.Hope I have answered your query. Let me know if I can assist you further. Regards, Dr. Ilyas Patel, Dermatologist"
},
{
"id": 217246,
"tgt": "Suggest remedy for pain in knee and thigh",
"src": "Patient: It started about a month ago my hip felt like it was going to break when I bent over.Now my knee and whole thigh is in pain. Cant straighten my leg have to have a pillow under the knee. the wrong movement gives me seized up tention from my hip to my foot. what can I do to relieve this at home.? Doctor: Here you need to check the symptoms whether it is only at knee and thigh level or starts from back and goes to thigh and knee. As per your explanation if it's from back then it is some thing called as nerve or hamstring muscle pull. I suggest to use hot and cold pack alternatively on your painful area. As well here need to do some physical tests. So visit orthopedic or Physiotherapist to get it evaluated properly. And they will also help to plan proper treatment with medication and physical therapy. At home as off now with out checking with physical test I can only suggest hot and cold pack use with some muscle relaxant and pain killer medications, but if it is not relieved then need to visit ortho or physio. Take care."
},
{
"id": 216079,
"tgt": "Suggest remedy for pain and soreness in chest",
"src": "Patient: I was hit in the chest a couple of weeks ago. The bruising and pain has left, on the left side. The right side I get a burning pain that is above my breast to the right, it shoots into my back shoulder blade. It causes my right arm to ache and tingle alittle bit. It happens mostly when I move but it does happen when I am sitting still. It happens a dozen times a day but doesn t wake me up at night. It hits on the pain scale of 8-9 for any where from 4 to 10 mins. EKG is fine. BP up a bit. Waiting on Chest X-Rays. Any ideas? Doctor: Hello and Welcome to \u2018Ask A Doctor\u2019 service. I have reviewed your query and here is my advice. I had seen your medical presentation and understood your anxiety.Chest pain is more concerned for many people as heart is located. But there are many causes of chest pain. It can be muscle pain, lung pain, rib pain or esophageal gastric pain. All these factors are to be considered before diagnosing the exact cause. Coming to chest pain, you were having bruising on left side and burning pain above right breast. Probable most common cause could be acid reflux or gastritis (inflammation of stomach).As pain was changing with your sitting position, it could be due to muscle pain. So all these factors are to be considered before confirming the cause. It is better to go for the following investigations:1. ECG2. Chest x-ray3. 2D Echo4. Gastric function testsComing to your case, my opinion is there were no classic signs of heart pain. So the most probable cause might be gastritis. You can follow the below measures:1. start using antacids in the morning before taking breakfast2. avoid spicy food 3. better to have frequent small mealsSo don't get worried you will be relieved soon from your symptoms. You can go for investigations and if any abnormality is there you can contact us at anytime. Hope you will be recovered soon from your symptoms. Hope I have answered your query. Let me know if I can assist you further."
},
{
"id": 83441,
"tgt": "Suggest opinion about minoxidil for hair loss",
"src": "Patient: Hi,i am femal 23 years old my wight 52kg ,lenght 160m, i had hairloss since 4year,i use many thing but not effect at all, i read about minoxidil is it effective for my case?, and if i use it then cut it suddenly is it cause baldness, help me doctor please.. Doctor: HiThe cause of hair loss should be identified.Hair loss can occur in hypothyroidism but it is not the only cause. Nutritional deficiency such as anemia,Hormonal imbalance, stress, excessive weight loss can all cause hair loss.Dandruff,Seborrheic dermatitis and many infections can also cause hair loss. Localised loss of hair can occur in skin conditions such as Alopecia areata which should also be ruled out.Minoxidil is a vasodilator that increases the blood flow to scalp thereby stimulating the hair follicles and increasing hair growth.The hair growth take few weeks to start and after stopping the drug, the hair growth may stop in 2 to 3 weeks.The drug can also cause dandruff. Incase you have dandruff,it should be treated first before starting minoxidil.Hope I have answered your query. Let me know if I can assist you further. RegardsDr.Saranya Ramadoss, General and Family Physician"
},
{
"id": 161941,
"tgt": "How can cough with crackling sound in a 7-month-old be treated?",
"src": "Patient: my niece is 7 mos has a cough with crackles sounds for more than 1 week and colds for 3 days now and the doctor was previously administered salbutamol and antibiotic but she is done taking all the meds yesterday, but the manifestation still presence, what kind of meds should I give her?thank you Doctor: Hello, I would like to suggest you start again 5 days of Antibiotic course: Drop. Amox as per the body weight. Shall be better n safe option. Along with cough Syp containing : expectorant +bronchodialator & Antihistamines(Anticold) preparation. B-Complex Syp. Also need to be given simultaneously. I would also suggest you give lukewarm water - soups frequently. Need to monitor temperature 4 hourly. Avoid exposure to cold weather. Hope I have answered your query. Let me know if I can assist you further. Take care Regards, Dr Ajaygupta009, General & Family Physician"
},
{
"id": 95646,
"tgt": "Suffering from Fundal Gastritis, what should i do ?",
"src": "Patient: i am currently diagnosed with fundal gastritis and have been recommended dosage of mucaine gel and pantocid tablet. I suffered with acute pain in abdomen for 2 hours before I rushed to the emergency room. Upon various tests, Doctor prescribed above medication and has recommended a review after 15 days and medication for a month. Is this diagnosis common for a 35yr old(5 6 ,150lbs) with 2-3 times a week alcohol and a lot of spicy food consumption?? Doctor: Hello Thanks for your query.\u00a0\u00a0\u00a0\u00a0\u00a0 It is a common problem in every age group and in persons who consume spicy food & alcohol \u2018Hope I have answered your query, I will be available to answer your follow up queries, \u201cWish you Good Health and trouble free speedy recovery\u201d"
},
{
"id": 551,
"tgt": "Can non penetrative sex cause pregnancy?",
"src": "Patient: My boyfriend had some cum on his hands I don't know if it was on his fingers or not but there was some on his hands. Anyways, he poked my vagina area as if he was fingering me but he only did it once and I had yoga pants on, underwear, a pad (which is plasticy and sticky on the bottom) and a tampon in. Is there anyway i could be pregnant? Doctor: Hi and welcome to HCM... I thank you for posting your query here and I'm definitely going to help you out.. I being a surgeon, we encounter so many such cases day in and day out.. I read your question completely and have understood it well enough.. I will analyze all your points and give the best possible explanation.. 1. Semen on hand. Semen Penetration into vagina leads to pregnancy.. 2. Yoga pants, underwear and pants on. But poking of hand into vagina. To be honest, there's absolutely no way you can be pregnant with this scenario.. 3. With all these things on, there is no way that the semen and hence sperm could have entered in! And hence I suggest you not to worry about it at all.. 4. Just forget this.. We come across so many such cases. And I can say with certainty that, you are safe. 5. There would have been a question mark if you had not wore any of the mentioned things. Now, it's impossible.. Just chill.. Hope this helps you, in case you need any further assistance, please let me know.. Take care and God bless."
},
{
"id": 140088,
"tgt": "What does disc degeneration at C3-C4 and C5-C6 with stenosis of spinal canal mean?",
"src": "Patient: I am a 23 y/o female. I have had multipul sever trama too my neck and head while growing up. I suffer from tension headaches. 9 months ago i started to loose my sense in my hand and feet. Deep throbing pain. certain movments are worse. any thing you could tell me about my mri results? I live in guam and the doctors here cant figure it out. mild disc space narrowing with disc degeneration at C3-C4, c4-c5, and c5-c6. mild posterior disc bulge and bilateral uncovertabral hypertrophy resulting in stenosis of the central spinal canal and bilateral mild non-impinging neural foraminal stenosis. Doctor: Hello, Your results of the neck suggest a mild to moderate degree of osteoarthritic change which seem a bit out of proportion to the complaints you're having. I would suggest that an EMG/NCV (electrical study) be performed on neck and affected to limbs to determine whether there are any conduction blocks. I would also get bloodwork done looking for Vit. B12, folate, TSH, FT4, and Vit. D levels which could show a metabolic cause to your symptoms. It does not appear from the MRI that there are any PINCHED NERVES to worry about but the EMG/NCV studies may show something else. Hope I have answered your query. Let me know if I can assist you further. Take care Regards, Dr Dariush Saghafi, Neurologist"
},
{
"id": 184151,
"tgt": "Is swelling normal after anesthesia for cavity filling?",
"src": "Patient: My son went to the dentist yesterday to get a cavity filled. They gave a local Anesthesia in his mouth. when he got out the dentist , his started swelling up . I want to know how can we cure this or is it normal for his lip to swell up after anesthesia Doctor: Thanks for your query, I have gone through your query.The swelling is not common after local anesthesia. It can be a mucocele secondary to biting over the lips when the lips were numb and causing trauma to the minor salivary glands in the lips.It can also occur secondary to inflammation following lip biting or it can be secondary to residual infection.Consult the oral physician and get it ruled out.I hope my answer will help you, take care."
},
{
"id": 109557,
"tgt": "Is it safe to fly after injection for back pain relief?",
"src": "Patient: Hi, may I answer your health queries right now ? Please can you confirm if ok to fly after receiving pain relief injections in the back for nerve ending problems in the legs. I am due to fly from Ireland to UK on Friday and I had these injections 2 weeks ago today on 15th March. Doctor: Yes...you can travel ..there is no contraindication..have a safe and sound journey...Hope this is the answer.."
},
{
"id": 48257,
"tgt": "Should treatment for blocked kidney be started right away?",
"src": "Patient: I am very concerned for my 73 year old friend. She was very sick a few days ago throwing up bile. She had a CT scan today and was diagnosed with a blocked right kidney and an obstruction in her stomach or intestine. Her doctor did not put her in the hospital today. She told her to go Monday for an ultrasound. I am concerned that time is wasting and that she should go to the hospital now, not when she is in pain. Isn t this a medical emergency? My friend is on blood thinner and other medications for congestive heart failure, which she has had the past 10 years. She had laproscopic surgery in January for the removal of her gallbladder and gall stones. Do you recommend that she go to the ER and get admitted to the hospital instead of waiting around until Monday to get an ultrasound and/or Upper GI? My email address is YYYY@YYYY . Thank you. I didn t realize this was a web site that charges. Sorry for the inconvenience. Doctor: Hi,Thanks for writing in.At age 73 years, having a blocked right kidney and obstruction in stomach or intestine is a medical emergency. It is suggested that you please clarify details on the kidney block and intestinal obstruction. A blocked kidney can be observed and taken care of for a few days if kidney functions are acceptable. However obstruction in intestine is an emergency situation and the patient must not be ignored. She might surely require upper GI or ultrasound but those investigations are to be done under the directions of a surgeon who is overlooking the intestinal obstruction. Please also regulate her fluid intake as per instructions from her doctor and keep her hydrated."
},
{
"id": 142545,
"tgt": "Suggest treatment for confusion and weak memory",
"src": "Patient: I often feel cloudiness in my brain. makes it difficult to recollect, diminishes my IQ and learning becomes difficult. Once in a while I have this cloudiness cleared and I feel okey. Just last week I was fine and sharp. Now its gone back again. Could you offer any help.thank you. Doctor: Hello!Welcome on Healthcaremagic!Your symptoms could be related to lack of concentration and anxiety. Thyroid dysfunction and chronic anemia can also trigger this clinical situation. Coming to this point, I would recommend performing some blood lab tests (complete blood count, thyroid hormone levels, fasting glucose, blood electrolytes), in order to be sure that everything is OK. If the above tests result normal, I would recommend performing Yoga and a lot of physical activity. You should know that physical activity can improve our brain performance, without the need of drugs. Hope you will find this answer helpful!Best wishes, Dr.Aida"
},
{
"id": 225519,
"tgt": "On copper IUD, have had an odd cycles with less bleeding, then got B.V, morning sickness. Is it normal ?",
"src": "Patient: hello i have the copper iud and have had it for about 4 years now about a month and a half ago just a few days before my normal cycle i had sex and a odd cycle not much bleeding then i got b.v. and not im on my cycle it was 2 weeks late. and have thrown up with the strange familiar feeling of morning sickness strck me . Is this very likely not really right ? Doctor: Hi,Welcome to HCM.Copper IUD is usually useful for 3 years. If it is 4 years since, it is high time you get it changed. There have been reports of pregnancy occurring with an IUD but very very rare. I suggest you get a detailed examination done by a gynecologist and if still want to continue family planning, get your IUD changed. Thanks."
},
{
"id": 94061,
"tgt": "Bad abdomen pain, back pain, breathlessness. What is causing this?",
"src": "Patient: Thanks. I have been waking up at night with bad pain and pressure in the upper abdomen about 3 times a week for about 2 months. I also have some lower back pain and trouble getting enough air into my lungs. Acid reducers, such as alka selzer, work for a little while. But the pain comes back. The pain was so bad this morning I was about to have my husband bring me to the ER. Then we did the alka selzer and I had relief for about an hour. Then the pain gradually started to come back. Doctor: Hi welcome to Health care magic forum. Thanks for choosing H.C.M.F. You are waking up with bad pain and pressure in upper abdomen since 2 months.Lower back pain and air hunger. It appears that you have gastric hyperacidity , or sleep apnoea , due to some silent heart problem, and back ache could be due to spondylosis or gastric pain it self. I advise you to consult a physician for diagnosis and treatment.You may have to go for gastroscopy, E.C.G. besides other routine tests for confermation. Wishing for a quick recovery. Best regards."
},
{
"id": 176486,
"tgt": "What causes the heel pain in children?",
"src": "Patient: hello doctor , recently i have turned to amul brand, its been 3 months my kids are complaining about a pain in their heel is it due to amul taaza milk or any other reason. my son is 10 years and my daughtr is 8 years please suggest me should i stop giving this milk thank you Doctor: THANKS FOR YOUR QUERY.Amul is not only the cause of the heel pain of your children.The most common cause of pediatric heel pain is a disorder called calcanealapophysitis which usually affects 8- to 14-year olds.Heel pain is caused by other conditions such as plantar fasciitis, tendonitis, bursitis, bone bruises or fractures.Sometimes, the simple strategy--Rest, Ice, Compression, Elevation--resolves pain, but when healing does not happen soon, it\u2019s time for clinical evaluation. Anti-inflammatory medications, physical therapy, or other treatments tailored for patients may be necessary.Preparation and recognition of warning signs can help prevent or reduce the severity of heel pain.Supportive shoes are a must, but it\u2019s not always obvious when to switch shoes or adapt them for the best fit. Consider these guidelines:\u2022 Use inserts to raise the heel, especially in flat-footed cleats\u2022 Discard shoes that caused pain. Don\u2019t use them for another sport\u2022 Wear well-constructed shoes designed for specific sports\u2022 Switch cleats often because they are not supportive shoes.Don\u2019t skip warm-up or cool-down exercises.HOPE THAT HELPS."
},
{
"id": 193094,
"tgt": "What causes pain in testicles with yellow colored urine?",
"src": "Patient: Sir, I got pain just above right testicle for past 1 weeks onwards. By pressing with hand also I can feel the pain. Also passing yellow colour unine. While I used to bend towards front side also I got pain. Sir what may be the reason and what sort of medical checkup is needed and medicine required ? Doctor: Hi,It can be due to bacterial infection and urinary tract infection. kindly take urine test and urine culture. Hope I have answered your query. Let me know if I can assist you further. Regards, Dr. S. R. Raveendran, Sexologist"
},
{
"id": 15912,
"tgt": "Reddish circular rings in the groin, buttocks and hips. Working with the radioactive chemicals. Concerned",
"src": "Patient: yesi have aquired smaller red circular spottish rings around my groin, buttox, and on my hip bones. this all started after i started a new job that deals with radioactive oil field waste. cancerous chemicals etc. plsase help me figure out what this is. Doctor: Dear Austin, Hello, welcome to Healthcare Magic. You are having fungal infection of the above mentioned areas known as Tinea cruris. You should take oral and topical treatment of antifungals for around 2-3weeks period under advice of your doctor. This problem aroused because of your occupation which I suppose is having exposure to heat and humidity. Both of these things increases the propensity of fungus to grow. So you should maintain proper hygiene with twice daily bath, wearing loose cotton clothes, changing your undergarments and general garments two times a day and maintaining a dry environment to prevent growth of fungus which will prevent recurrent/ further infection. With best health wishes, Dr Sanjay K Kanodia"
},
{
"id": 21077,
"tgt": "Suggest remedy for chest pains",
"src": "Patient: Have had minor minor chest pain for the last three weeks. Just today it s gotten really really painful. Have an appt. with the doctor tomorrow. I am 48 and have a-fibs. The pain is in the back and chest and hurts when I breathe deep. Felt like I was going to pass out twice while driving. It started when I was lifting weights three weeks ago and last week when I lifted it hurt really bad so I stopped. Doctor: Hi,Cardiac chest pain does not increase on breathing or change in position. Cardiac chest pain increase on exertion & decrease on rest. So your pain doesn not look like a cardiac pain & seems more like muscle pain.Passing out during driving could be because of decrease in heart rate or bp to a very low level. I will recommend you to get them both monitored.Thanks"
},
{
"id": 195557,
"tgt": "Suggest remedy for erectile dysfunction and premature ejaculation",
"src": "Patient: Dr. I have promblem ED & PE also since seven years. So far no cure has given me satisfaction..I have taken lot of herbal, unani, hamdard and many more.. and since three years I consulted English Dr. he examined prolact & Tes. and told that it is low..for that he perscribed the medicines of Testorin gel, adnrivel, seroxat tab, two injection of NIBIDO and four tab of LEVITRA...Still I have no improved at all..my erection sustainied less than one minute only and immediately ejaculation...pls help and to come out this promblems permanently..thnaks waiting your suggestions and remdey Doctor: Hello and Welcome to \u2018Ask A Doctor\u2019 service. I have reviewed your query and here is my advice. Well first of all you need to understand that premature ejaculation and erectile dysfunction often occurs together and they have more common causes rather than hormonal disturbances for example high uric acid levels, high lipid levels and diabetes. When a patient come to me with erectile dysfunction i always order these tests serum uric acid , fasting lipid profile and HbA1c levels and if something comes wrong in them then i would treat it first then i will go for treatment of PE and ED. If everything comes out to be fine then i mostly state my patients on tablet Tadalafil 20 mg half tablet 1 hour before sex and tablet Dapoxitene 30 mg before 1.5 hours before intercourse. Hope I have answered your query. Let me know if I can assist you further."
},
{
"id": 161435,
"tgt": "Suggest treatment for cough and fever in an toddler",
"src": "Patient: My 2 years & 8 months old son is suffering from cough and fever since yesterday afternoon. I have already given him P250. It works for at least Six hours. But today morning he vomited the medicine as well. What to do. Immediet reply solicited. XXXX Doctor: Hello, It may be due to viral fever. For fever you can give syrup Acetaminophen and cough syrup after pediatrician consultation. You can give nebulization. Watch for symptoms like high grade fever and breathing difficulty; he may require urgent care then. Keep him warm. Hope I have answered your query. Let me know if I can assist you further. Regards, Dr. Shyam Kale Family and general physician"
},
{
"id": 164672,
"tgt": "What causes a red hard bump on the buttocks despite using HCTZ cream?",
"src": "Patient: My 12 yo daughter had what looked to be a small insect bite between her butt cheeks more on the right side--we used HCTZ cream on it by itself but it did not go away. Not as much itching, but painful for her to sit and is keeping her up at night. Since it had a white head on it,we sterilized a needle and lanced/scraped it and some pus like stuff came out. She had a little bit of short term relief, but by the next evening the area around the bump is red and there is a hardness under the skin and the pain has gotten worse. We ve been giving her Ibuprofen and she continues to use the HCTZ but she is not sleeping and it does not seem to be healing. She is so embarrassed about it. Should we go to doctor/ER? Doctor: Hi, I guess there is an abcess... The HCTZ cream will not work. Needs a course of antibiotic. Please consult your doctor for the same. Do Not Start By Yourself....Thanks... Tc"
},
{
"id": 128934,
"tgt": "What causes right-sided chest pain and mild nausea after an injury?",
"src": "Patient: 58 year old female in good health. Fell from ten foot ladder. Landed on right side. Didn t go to dr because no insurance. So sore couldn t get comfortable to sleep. Pretty sure cracked wrist and a rib. Wore a wrist brace five weeks and just resting. Arm bruised everywhere. After five weeks I still have a pain behind my right breadt that goes front to back. I always feel just slightly nauseated. Then suddenly hunger pain. My elastic waisted shorts are now very uncomfortable at waist. Doctor: Hello,Thank you for using healthcaremagic.I read your question and understood your concern.I think you had fractures of the ribs with internal bleeding in the lungs or pnumothorax ( air in the lungs). That may cause you the nausea feeling.Normaly this would be complains of the first days after the injury. After 5 weeks now, the fracture of the ribs is healed alreadybut you may have blood collection remained inside your lungs that is causing this problem. It may require more time from your body to absorb the blood.I wish you quick recovery.Dr. Selmani"
},
{
"id": 113177,
"tgt": "Back pain after childbirth. Discomfort in ribs",
"src": "Patient: Hello my names is Diana I m 19 and I had my baby about 6 months ago . I never had a problem with my back during my pregnancy but when I had my baby , a month later .., I have severe back pain like I rather die . It s unbearable and I always end up puking during the pain I get ! I feel like my ribs are squishing my lungs and my stomach ! It happens evey once a MONTH like no lie I m to young to be even having this ! I also had the epidural but they assured me that it wasn t it .. Doctor: hi diana back pain after pregnancy and lactation is very common . this is mainly because of increase lumbar lordosis and laxity of abdominal muscles, nutrional deficiency of vitamin d and calcium which is very common after pregnacy and lactation . Take proper calcium vitamin d suppliments, abdomonal muscle strengthening excercises as advised by your gynaecologist and the physio therapist . and if you still have pain after doing all this things consult orthopedic surgeon i diana"
},
{
"id": 52383,
"tgt": "Does taro root cause gallstone formation?",
"src": "Patient: Hello. I m a female, 53 years old. Six months ago I had a pancreatic attack. I was hospitalized. My hospital stay was for three weeks and then I came home. About the same time I was diagnosed with Type 2 Diabetes, and I had a gallbladder removal in January 2018. Now I m trying to eat healthy and doing my best to side-step any sort of gastrointestinal discomfort. I have a guestion about the menu: I found a tropical plant called taro root , I know that it can help regulate insulin and glucose levels in my body, so it is good for diabetic patients. But what about people who was diagnosed with chronic pancreatitis? Am I suppose to have some side affects? And also is there a possibility that taro roots will cause gallstones to form? I m so sorry for my english! I need an advice. Doctors in my country have no idea what taro root is. Thank you. Doctor: Hi, There is no reason to suspect that taro roots cause gallstone formation. If the root is good for your diabetes, it is probably also good for your gallstones. Hope I have answered your query. Let me know if I can assist you further. Regards, Dr. Anders Mark Christensen, General Surgeon"
},
{
"id": 153789,
"tgt": "What causes chronic headache and giddiness after a chemotherapy?",
"src": "Patient: hi my father has been undergoing chemotherapy treatment .he had undergone a total gastrectomy surgery 3 months back.he was on his third chemo session when his total blood count dropped down to 1000.he is being given injection to raise the bloodcount..but sincethe past 3 days he is suffering from chronic headache and giddiness.the headache goes on for 5 minutes and this countinues as a cycle every half another or more frequent.is this because of low blood count? the doctors say the headache is not attributed to chemo treatment.has taken ct scan, mri scan chest xray..everything seems normal.chemotablet used is Xeloda 500mg.(taking 2000mg per day on a 14 day couse in a 21 day cycle) Doctor: Hi,Thanks for writing in.Chemotherapy is a stressful event and patients might have certain side effects after taking chemotherapy. It is possible that your father is still having low hemoglobin and anemia because of which there is regular headache. The doctors probably intend to say that the headache is not a direct result of chemotherapy but due to hemoglobin. Since CT scan and MRI scan have shown normal findings therefore it is unlikely to be due to any bleeding or infection in the brain. However headache due to anemia might be occurring. Right now focus should be on increasing his nutrition and giving him a daily diet of 2200 kcal is recommended. These might be as foods which he likes and water and fresh juices are to be given in adequate amounts. Please protect him form any infection by taking precautions. Please do not worry."
},
{
"id": 199907,
"tgt": "What causes white heads like under penis head in frenulum area?",
"src": "Patient: So in the past couple of months, i ve Noticed a cluster of whitish Zit like whiteheads which give off a pearly color under my Penis head in the frenulum area, I know they are not Pearly Penile papules as i ve looked online for those and they do not look like they would be since these i have are not around the neck of the penis but in the specific frenulum area by the urethra. When i touch them they feel a bit hard and they do not feel painful unless i try to pinch them or grab all of them and sort of like pinch the entire, The entire cluster would feel like touching a circuit board with solder metal heads in the same aspect of touch and feel. Doctor: HIWell come to HCMTwo possibilities are very likely one is formation of smegma (If not circumcised) and would be warts, although clinical observation is must to confirm the final diagnosis, because all the skin lesion demands this and tis is the ideal way of diagnosis, hope this information helps, take care."
},
{
"id": 35306,
"tgt": "Can you get sinus infection from cataract surgery?",
"src": "Patient: Can you get sinus infection fron cataract surgery?I had surgery in my left eye and still feel a pressure when bending over. At times I feel pressure in my forehead and the bridge of my nose and right eye. I have Asthma and sleep apeanea so have a history of sinus infections Doctor: Hello dear,Thank you for your contact to health care magic.I read and understand your concern. I am Dr Arun Tank answering your concern.No cataract has nothing to do with sinus infection. It may be a coincident that both condition like cataract and sinus infection imerged as such.You should take salbutamol and inhaled steroid as per your situation and under your doctors guidance. This will cure your problem with chest.For sinus infection you should take paracetamol for pain and antibiotics to prevent secondary infection.I will be happy to answer your further concern on bit.ly/DrArun.Thank you,Dr Arun TankInfectious diseases specialist,HCM."
},
{
"id": 152024,
"tgt": "What is the function of autonomic nervous system?",
"src": "Patient: describe the structure and the functions of the autonomic nervous system Doctor: Hello, welcome to HCM. The autonomic nervous system (ANS or visceral nervous system) is the part of the peripheral nervous system that acts as a control system functioning largely below the level of consciousness, and controls visceral functions. The ANS affects heart rate, digestion, respiration rate, salivation, perspiration, diameter of the pupils, micturition (urination), and sexual arousal. Whereas most of its actions are involuntary, some, such as breathing, work in tandem with the conscious mind. It is classically divided into two subsystems: the parasympathetic nervous system (PSNS) and sympathetic nervous system (SNS). Relatively recently, a third subsystem of neurons that have been named 'non-adrenergic and non-cholinergic' neurons (because they use nitric oxide as a neurotransmitter) have been described and found to be integral in autonomic function, particularly in the gut and the lungs."
},
{
"id": 13794,
"tgt": "What is the cause and treatment for small bump on the knee?",
"src": "Patient: Hi, I have a small bump on my knee and I'm not sure if it is harmful... It doesn't hurt and is pretty small, but I don't know what it is, why it's there, or if it needs to be removed. It kind of feels like there is a small, firm bubble underneath my skin or like a bump that is about the size of a bead... What could it be? Doctor: Hi, It can be a small wart-like lesion and does not indicate any significant disease condition. It will be better if you attach a picture along with the query. Hope I have answered your query. Let me know if I can assist you further."
},
{
"id": 61092,
"tgt": "What does a painful lump under the rib cage on the left side indicate?",
"src": "Patient: I have had this pain for about three weeks on my left side immediately under my rib cage. I don't notice it until I bump it or touch it or bend to the left. Nothing brings it on and nothing makes it better. It's like a muscle tear but I'm not sure how I would have done it. Woke up the day after Christmas with it. Any ideas? Again I only feel it if it is touched basically. Doctor: Hi and welcome to HealthcareMagic..I thank you for posting your query here and I am definitely going to help you out.. Please note. 1. Did you have any kind of trauma there?2. If no, then please do not worry. If there is a swelling, get an fnac and ultrasound scan done.3. Or else take analgesics and forget it. It should heal.Hit 5 stars.."
},
{
"id": 114021,
"tgt": "I am naseas and have back pain, please suggest",
"src": "Patient: i am naseas and have back pain, and also knots in my breast Doctor: Hi, Thanks for query, Without menstrual history and time of having sex it is difficult to conclude. There are plenty of causes of nausea and back pain. Ok and bye."
},
{
"id": 164605,
"tgt": "What causes testicle redness with pain?",
"src": "Patient: Hi, may I answer your health queries right now ? Please type your query here... Hello. My one year old boy just got over a very bad viral infection. his bottom had a very bad rash the the pediatrion prescribed some antibiotic cream for. it s pretty much all cleared up but then today his testical (testeicale only) is bright red. he s in a lot off pain. Doctor: Helloit is torsion of testis, it needs urgent treatment.go to emergency department of hospital immediately"
},
{
"id": 177102,
"tgt": "Is stored breast milk good for babies?",
"src": "Patient: Hello doctor, I have a two month old son born with a birth weight of 2.3 kg in august. At present his weight is 4 kg. Due to family circumstances I will have to join back work in the third month. In my absence from home how do I feed my son... I wish to breastfeed him as long as can...heard about pumping but I am not sure how to go about it... Is it safe to store breast milk ? For how long can I store? How do I store it ? Can I use bottle to feed the pumped milk ? Will pumped and stored mill cause infection or will be as effective as breast feed ? Kindly help your suggestions will be highly appreciated. Thanks in advance. Doctor: Hi Dear Welcome to the HCM,Yes. Breast can be stored with these circumstances and can be utilised later as the baby feed. It will be better it should be given the same days with spoon and bowl. Better to avoid bottle. Bottle are the important source of infection.If the milk stored is of more than 4 to 6 hours duration than it is advisable to biol it.Hope the query is answered.thanks"
},
{
"id": 156939,
"tgt": "Should i be worried as i am having frequent urination, rare periods and farting?",
"src": "Patient: I am a 19 year old girl and my period has been messed up since I was about 17. I rarely have periods maybe 2-5 times a year. I had thyriod cancer when I was 8, so thats been removed and now I take a pill in its place. Lately ive been peeing a lot and farting a lot. Is there something wrong? Doctor: HI, thanks for using healthcare magicIrregular periods can be due to: (!) abnormal levels of thyroid hormone- your doctor may want to make sure that your levels of hormone replacement is adequate(2)emotional or physical stress or illness(3)inadequate diet(4)polycystic ovarian syndrome(5) high prolactin levels- this is a hormone released in the brainYou may want to consider visiting your doctor for an assessment which would include blood tests and imaging of the pelvis.In terms of the frequent urination and flatulence , this may not be related to the irregular period or each other.Excess flatulence is often related to intolerance to certain foods or excess use of gas producing foods such as beans, broccoli, cabbage, turnips, brussel sprouts, carbonated drinks.There are different causes of increased urinary frequency, your doctor may suggest a 1 to 3 day urinary diary to identify how much you are drinking , how often you use the bathroom and the amount of urine.I hope this helps"
},
{
"id": 161726,
"tgt": "What is the treatment for high fever in a baby?",
"src": "Patient: Hi, my baby son is 14months old and he is having intermittently high fever. i gave him paracetemol this morning. now, in the evening he has shivers foolowed by fever. i gave hime paracetomol again. the fever is dying. should i be worried. should i take him to A&E? Doctor: Hello, Fever of a few days without any localizing signs could as well a viral illness. Usually, rather than fever, what is more, important is the activity of the child, in between 2 fever episodes on the same day. If the kid is active and playing around when there is no fever, it is probably a viral illness and it doesn't require antibiotics at all. Once the viral fever comes it will there for 4-7 days. So do not worry about the duration if the kid is active. Paracetamol can be given that too only if fever is more than 100F. I suggest not using combination medicines for fever, especially with Paracetamol. Hope I have answered your query. Let me know if I can assist you further. Regards, Dr. Sumanth Amperayani, Pediatrician, Pulmonology"
},
{
"id": 32146,
"tgt": "Suggest medication for severe cold and stuffy nose",
"src": "Patient: I had a severe cold in early november,which apparently didn't go away entirely, and now Ive been coughing for the past three weeks. there's a tickling in my throat, I also have to constantly clear it for mucus that seems to be stuck there,and added to that now is the chest discomfort i feel because it seems like my nose has closed up especially when lying down. I really need some help . Doctor: Hi Dear,Welcome to HCM.Understanding your concern. As per your query you have severe cold and stuffy nose and chest discomfort which is due to upper respiratory tract infection which is leading to sinusitis as well. Need not to worry. I would suggest you to avoid intake of cold liquids. You should take green tea every morning. You should take antibiotic Ciprofloxacin and antihistamine Levocetrizine for 3-4 days. Avoid intake of excessive oily, fried and spicy food. You should do warm saline gargles on daily basis and take steam inhalation as well. Visit ENT specialist once if symptoms keeps on persisting and get blood tests and X-ray chest done and start treatment after proper diagnosis. Hope your concern has been resolved.Get Well Soon.Best Wishes,Dr. Harry Maheshwari"
},
{
"id": 129802,
"tgt": "How to reduce swelling in toe and discomfort while walking after hitting cot?",
"src": "Patient: My left leg fourth finger(second from small finger) had hit with steel cot leg on 12.11.12. I have applied Thrmbophob ointment three times daily till date. I also taken suganril tab 3days 3times daily.Till date , Iam not comfort in walking as earlier and finger till swelling and unable to touch and move suggest. Doctor: Hello!Seems like Your toe needs X-ray to confirm diagnosis!Its better to make it and get proper treatment - splinting if needed.Injured toes is better keeping elevated, cold for 15min every hour."
},
{
"id": 95624,
"tgt": "I have a pain below my left breast from five years. Sometimes this the area of the pain murmurs",
"src": "Patient: I have a pain below my left breast(upper abdomen under the ribs) since five years and it has been troubling me and giving me too much discomfort. It is not very severe. i still go out my normal activity but can not do strong work. Sometimes this the area of the pain murmurs as if it were my stomarch. What is it? I have a pain below my left breast(upper abdomen under the ribs) since five years and it has been troubling me and giving me too much discomfort. It is not very severe. i still go out my normal activity but can not do strong work. Sometimes this the area of the pain murmurs as if it were my stomarch. What is it? Doctor: Hi, Thanks for query, It seems that you might have Costo chondritis of ribs. Take analgesics and apply pain relieving cream locally. Possibility of having hyperacidity or gastritis is also to be ruled out. Avoid fried and junk food. Ok and bye."
},
{
"id": 214006,
"tgt": "Suppliments for extra energy in workouts and curb appetite",
"src": "Patient: can i try oxyELITE pro, from GNC, while taking SSRI Celexa ? oxyELITE is used for extra energy during workouts and contains an appetite depressant. It lists Dimethylamylamine on the label as well as 100 mg caffine. if not, are their other suppliments to allow a little extra energy in workouts and curb appetite ? Doctor: Please don't try this... iwas on both and it gave me massive anxiety attack and insomnia and I havnt had those before or since.... do not try it."
},
{
"id": 110880,
"tgt": "How to treat pain and bruise on spine?",
"src": "Patient: I have a bruise on the base of my spine. It has been there for many years. I suffer from extreme back pain and went or a massage the other day. The masseuse asked if i had a spinal tap or epidural as the mark is consistant with that. I know that i havent. What ould it be? Doctor: HI THANKS FOR POSTING YOUR QUERY ON HEALTHCARE MAGIC.I WOULD LIKE TO TELL YOU THAT FEW SPINAL INJURIES CAN PRESENT LATE ,MAY BE AFTER FEW YEARS AFTER THE INITIAL INSULT..THE THING TROUBLING YOU CURENTLY COULD BE MOST LIKELY A \"MISSED/PROLAPSED INTER VERTEBRAL DISC\".THIS CAN IMPINGE ON YOUR SPINAL CORD AND PRODUCE SYMPTOMS LIKE LOW SEVERE BACKACHE,TINGLING,NUMBNESS,PARESTHESIAS DOWN THE LIMB.YEARS LATER THEY CAN ALSO PRESENT WITH BOWEL AND BLADDER DISTURBANCES.I SUGGEST YOU AT ONCE THAT YOU UNDERGO AN EARLY MRI(MAGNETIC RESONANCE IMAGING)OF LOWER-LUMBAR SPINE TO ASSESS ANY TRAUMATIC CONTUSION TO THE CORD/HEMATOMAS/SLIPPED DISC/VERTEBRAE.IF FOUND THEY HAVE TO BE TREATED SURGICALLY.MEANWHILE I PRESCRIBE PAINKILLERS LIKE TRAMADOL TO REDUCE YOUR PAIN.CONSULT AN EXPERT NEUROSURGEON AT THE EARLIEST.THANK YOUTAKE CARE."
},
{
"id": 190464,
"tgt": "Installed crown on molar. Why is there pain now?",
"src": "Patient: I am a 92-year-old male. A couple of years ago, my dentist installed a crown on the third molar ( wisdom tooth ) in my lower left jaw. Afterward, I started complaining of pain in the area and even some in the left rear side of my tongue . He examined my mouth but saw nothing to cause it. He referred me to a dental gum specialist who also saw nothing. The initial dentist removed the crown in early 2011 but the pain persisted. The tongue pain may have gotten worse. In the last month, he referred me to a dental specialist: an oral surgeon who saw nothing, even after a sort of CT scan . He suggested extraction of the wisdom tooth which was done a couple of weeks ago. The tongue pain (rear, far left) is much worse, especially when I chew, even though I keep the food as far away on the right as possible. My speech is impaired. Suggestions? Doctor: hello and welcome to HCM forum, i am glad you wrote in, according to the dental history provided by you it seems that you have been suffering with this problem for quite a while now, i would like to inform you that in my opinion, the pain is due to a neurological problem, the nerve that supplies your tongue might have been affected years ago. but, the reason is unknown as your dentist could not diagnose it. i advise you to visit a neurologist once as he/she might be able to provide you with a treatment plan . Also , i assume that your tongue does not have any ulcers, kindly be calm and see a neurologist soon, i wish you good health, take care."
},
{
"id": 130834,
"tgt": "Is it ok to use heating pad for adhesions from ulcer?",
"src": "Patient: Hi, I am recovering from a suture removal due to RNY surgery 5 years ago. The suture caused an ulcer and had to be removed. My surgeon told me I most likely have some adhesions on my right side. I was wondering what I can do for the pain at home other than pain meds because they cause me to have constipation. Is it OK to use a heating pad? Should I wear a tight top to hold the muscle in place. I have most of my pain when bending over and walking. Doctor: Hi i am Dr Ahmed Aly i dont think the hot compresses is a good idea in an infected or bare skin areas wearing tight tops on a clean antiseptic dressing with taking your antibiotics and medications would be ok multiple dressings should be frequent and enough , avoid bending over and vigorous activities till healing of the wound is acomplished"
},
{
"id": 130633,
"tgt": "Why do I have uneasiness & pain in left hand nerve?",
"src": "Patient: i am a i h d patient since 2003 and in last week i consulted physician as i was feeling some uneasyness & pain in left hand in a particular nerve. he prescribed Ismo 10, ecosprin av 75, rancad 500, & nebicard 2.5 Can you guide me the benefit of Rancad & Nebicard Doctor: Hi,Rancad tablet decreases the oxygen requirement of the heart by shifting its metabolism from fats to glucose. As a result, heart works more efficiently. Nebicard tablet relaxes the blood vessels and reduce the blood pressure, making it easier for a weak heart to pump blood at a slower rate. Both needs to be taken at a fixed time everyday. Apart from this regular exercises ; yoga poses like Pranayam, Surya Namaskar will help you in maintaining your blood pressure levels and heart fitness.Hope this helps. Let me know if I can assist you further.Regards,Dr. Jenis Bhalavat"
},
{
"id": 130554,
"tgt": "Pain in the knee",
"src": "Patient: I HAVE PAIN IN MY LEFT INSIDE KNEE.IT HURTS WHEN I WALK. PICTURE KICKING A SOCCERBALLWITH THE INSIDE OF YOUR FOOT.BY ACCIDENT IF I HAD TO HIT SOMETHING ,IT WOULD BRING TEARS TO MY EYES.IT'S REALLY PAINFUL WHEN THAT HAPPENS.THE PAIN LAST ABOUT10 TO 15 MIN THEN IT GOES AWAY. Doctor: Hi,Given the history it feels you are suffering from O.A knee (probably grade 1) starting where meniscus which is present inside the knee joint starts eroding leading to pain on activity. I suggest you to apply hot pack over knee twice a day, and do some quadriceps strengthening exercises. Hope you will find the answer useful. Let me know if I can assist you further. Regards,Dr. Harsh Swarup"
},
{
"id": 31505,
"tgt": "Suggest remedy to prevent spreading of Staph infection",
"src": "Patient: Hi, may I answer your health queries right now ? Please type your query here... I have been diagnosed as having Staph in my left ear. I have a drain tube in it and it has been draining for 6 weeks and did not respond to Ciprodex, Blephamide. I was first diagnosed as having Candida and Fluconazale was prescibed for one 100 ml pill for 5 days only. Now the culture shows Staph. I am very concerned about spreading the Staph infection to others such as from tennis balls when I play often and to my wife. I have a number of psoriasis like patches and am afraid it will spread to them as well. What is the best wipe-type product to help prevent the spread? Milton Thrasher - Sarasota, FL 82 years old Doctor: Hi thanks for asking question in HCM.This staphylococci infection is mainly hospital acquired.But it can spread from person to person through direct contact.So it can be acquired through skin contact also.As far as ear infection is concerned then less chance of spread from that source.Your wife has to take care about not coming in direct contact with staph.affected lesion to prevent infection.Thanks.I hope i have solved your query."
},
{
"id": 136759,
"tgt": "What causes severe bruising in back after a fall?",
"src": "Patient: I had a bad fall down a flight of stairs Wednesday night I have a really bad bruise on the lower backside of my back and I hit my head on the door busting my eye and causing it to bleed but the past couple days I ve been feeling sick to my stomach bad headaches and occasional spotting when I urinate. I m not sure if I need to go to the hospital or not Doctor: Hi,Thanks for your query.From the description that you have provided, it seems that you have suffered a soft tissue injury. You need to provide rest to the back. Avoid massaging the area. Doing a hot fomentation will be helpful. Avoid forward bending.I would suggest getting this evaluated by an orthopedician for an accurate diagnosis and appropriate management. You may get the Xray/MRI of the area affected under his/her guidanceI do hope that you have found something helpful and I will be glad to answer any further query.Take care"
},
{
"id": 76969,
"tgt": "What causes pain in right side of the rib cage?",
"src": "Patient: I'm 19 about 9 stone 5 foot 3 inches I keep getting pains on the right side of my rib cage its also a little bit to the front and a little bit to the back it also sometimes hurt down the right side of my belly to I've had it for about 2 weeks on and off Doctor: Hi thanks for contacting HCM...Here according to history it seems simple musculoskeletal pain.....Advise ......You can take simple analgesic for pain.Hot pad applied.Avoid strenous activities....Physiotherapy helpful...Proper sleep posture needed.Avoid heavy weight lift.If still pain remain then we will do further work up...Dr.Parth"
},
{
"id": 196967,
"tgt": "Are there any side effects of taking Ereco-50 ?",
"src": "Patient: I am taking Erecto-50(slidenafil citrate tabs) and feel good. But sometimes I suddenly fail midway and my organ loses rigidity. Why? I am 65 and can I increase the dose to 100mg? Can I take the tabs every alternate day. I could feel side effects like headache and gastro problems. Any cause for worry? Doctor: HelloThanks for query .Based on the facts that you have posted it seems that you are facing problem of ED and have been getting satisfactory erection to penetrate vagina after taking 50 mg of Sildenaafil .However you loose erection during due course aafter going ahead .This is most likely to be to inadequate dose of Sildenafil .Increase the dose of Sildenafil to 100 mg .Please note that Sildenafil has to be taken one hour before only on the day of planned sexual activities and not on every alternate day .Dr.Patil."
},
{
"id": 209887,
"tgt": "Prescribe medicines for Asperger s syndrome",
"src": "Patient: My 13 year old has been diagnosed with adhd, aspergers, and skin picking disorder. He has been on 500 mg of depakote, 20mg of citalopram, 20 mg of focalin and 10 mg Zyrtec. Yesterday a received a report that he was stuttering and had a tremor in his hand. I have not noticed this at home. Which of these meds might be the culprit? Doctor: HiThanks for using healthcare magicDepakote contains valproate and it causes stuttering and tremor. In that case, better to decrease the dose of valproate to 250 mg or shift to another mood stabilizer like carbamazepine or oxecarbamazepine. Rest, you can consult his psychiatrist. In case, you need further help, you can ask.Thanks"
},
{
"id": 81486,
"tgt": "Why isn t antitussive cough syrup helping deal with dry cough?",
"src": "Patient: Hi, I have dry cough month back because I had drunk excess cold water. to cure it I took some anti-tussive cough syrup. After two weeks it turned into chesty cough. I have been taking cough syrup, but its not getting better. As of present I cough all day long with thick yellow coloured mucus. As night when I go to sleep I have to keep a clove bud chewing inorder to stop cough so that I could sleep. Please help.!! Doctor: well long lasting cough need proper evaluation clinical examination chest X ray sputum examination hemogram we can review with reports"
},
{
"id": 89403,
"tgt": "Should I be concerned about abdominal spasms ans pain?",
"src": "Patient: Hello. I'm a 38 year old female. A few months ago I had my fallopian tubes removed. I am now in the process of IVF (second cycle). A week ago I had two 5-day blastocytes transferred. Three times during the past week, always while asleep, I have had what feels like spasms in my lower abdomen area. After the spasm subsides, I have lower abdomen pain. It subsides after a few moments. I have no other symptoms (bleeding, fever, etc.). Should I be concerned? Doctor: You should be concerned, get blood and urine tests and also ultrasonography to see that there is no PID or peritonitis ."
},
{
"id": 223611,
"tgt": "Does using birth control affect chances of future pregnancies?",
"src": "Patient: Hi im zoya from mumbai Ive been married for 3 years now V were not ready for a baby dat is y my husband ws using da control method during intercourse i never became pregnant but nw v r plannin fr a baby frm da last 2-3 months Do yu think dat delay un pregnancy may cause problems nw Doctor: Hello,I have gone through your query and understood the concern. Use of contraception, especially in the manner adopted by you, does not cause delay in achieving a natural pregnancy in ordinary circumstances. Use of hormones can delay return of ovulation and hence cause a delay. As you did not use any medication, there is no such worry. Please try sincerely for at least six months before going for further help. Try to have regular intercourse, at least on alternate days during the fertile period of your cycle. Hope I have answered your query. Take care."
},
{
"id": 127145,
"tgt": "What causes recurrent pain on the left side of the hips and ribcage?",
"src": "Patient: Left side pain that comes and goes every few minutes. Started this morning at work I was noticing it maybe every 30 minutes approx but now that I\u2019m home resting every few minutes. It goes from very little to no pain to a stabbing pain that is pretty sharp. Then it goes away and comes back. It\u2019s more on my left side between where my hip starts and my rib cage starts. Any ideas? Is it worth going to the hospital or should I schedule an appointment with my regular doctor later in the week? Or I can wait and see if it goes away on its own or gets worse. Thanks Doctor: Hello, The pain you are feeling on left side that comes and goes every few minutes, could be due to colitis or Irritable bowel syndrome (IBS) especially you are also having bloating or other changes in bowel movements (bm). I advise to take over the counter Mephenamic acid and antacid mixture. Start probiotics and Simethicone for the bloating. If it won't ease the pain you feel, then go to ER as might need immediate diagnostic tests and treatment. Try my suggestions and come back to me for further assistance. Hope I have answered your query. Let me know if I can assist you further."
},
{
"id": 161820,
"tgt": "How do high phosphorus levels affect a newborn?",
"src": "Patient: My newborn has a his phosphorus level at 8.9 HIs pedritician says that it should be atleast 6.0 w are alternating between breastmilk and formula pm 60/40. He got a blood test yesterday and the pedriatician will see him next monday. How bad is for newborn to have high phosphorus ar this early stage of his life. Doctor: Hi, An elevated serum phosphorus (P) has been anecdotally described in premature infants receiving human milk fortified with donor human milk-derived fortifier (HMDF). It might make bones weak if left out like that. He needs follow up and change of HMDF if he is on it. Hope I have answered your query. Let me know if I can assist you further. Thank you. Regards, Dr. Sumanth Amperayani Pediatrician, Pulmonology"
},
{
"id": 137355,
"tgt": "Suggest treatment for severe thigh pain",
"src": "Patient: Experiencing severe pain in thigh area of front & exterior left leg to hip, especially when going up or downstairs or when getting up from sitting position to standing. Been trying to use to use right leg & now it is also happening there. Not as severe. can u please tell me what is causing these symptoms. CANCEL MY QUESTION & REMOVE MY EMAIL PLEASE Doctor: HelloI have read your query. There is possibility of lumbar canal stenosis leading to thigh pain. I will advise you MRI spine and EMG NCV study. Medication like methylcobalamine and pentoxphylline may help you. I hope I have answered your questions. If you have further questions please feel free to contact us. I will be happy to answer. Take care."
},
{
"id": 147477,
"tgt": "Burning chest pain, syncope, no fever, no energy, cold fingers. Ranitidin?",
"src": "Patient: Hi! I m currently experiencing burning pain around my chest , heart and solar plexus. I fainted last wednesday after a workout at the gym , i m pretty healthy overall and workout regularly. I had just been sick and gone from training for 2 weeks, so it might have been a hard start. I spent the night over at the hospital where they did blood tests, EKG , AKG (bycycle test), and everything seems to be good with my heart. I got the diagnosis Syncope . Few days later i did the same program and almost fainted again after the workout, but i layed down for a while (45ish mins) and got a little better after i drank som saltwater. Now i still have this burning feeling around my chest. I have to add that i have had gastritis last year, and it has come and go. Might this cause the pain? Should I take ranitidin tablets or should i be more concerned? Because my fingers are cold, but I have no fever, and my body feels slow and dull, tapped for energy. Need help ASAP please! Regards, Doctor: Syncope & coldness of fingers may be due to pressure on your cervical nerve-roots.It may be just postural pain or a pathology of cervical vertebrae.You should go for an M.R.I. of cervical vertebrae.If it does not show any abnormality ,then you should take measures like avoiding pillow etc.You may also need exercises of neck."
},
{
"id": 174770,
"tgt": "Infant fussy and cranky even after breast feeding and lactogen feed",
"src": "Patient: hi, my baby is 5 months old now i am following lactogen along with breast feeding. i am giving 2scoops with 60 ml of water for each feeding in a interval of 4 hours(5 times a day). but she is fussy and cranky after consuming also. why is it so. does she feels hungry? should i increase the amount or no. of feeding? can i introduce any other food like cerelac or raggi mix. Doctor: Mother's milk is the best possible feed for a baby and there can be no alternatives for the same. It is produced in response to the baby's demands. Most formula milks are cow milk based and hence difficult to digest. Do not give the formula that the child cannot tolerate. 60 ml is unlikely to be adequate for a child of this age. You may also try increasing the amount of feeds and see if that satisfies the child. It is best to give the child feeding on demand and as much as the child demands. Often the child ingests air along with feed. This air can distend the stomach giving rise to discomfort and the child this could be the cause of your child crying. Proper burping and giving Colimex drops can provide relief.Although weaning to semi solids is suggested from 6 months of age, you can start them at 5 months if the demands of the child is not met properly. So you can introduce other foods like cerelac or raggi mix."
},
{
"id": 142351,
"tgt": "Suggest treatment for brain hemorrage",
"src": "Patient: Dear Sir,My sister suffered a brain hemorrage and now is still in a coma after 72 hours. Doctors said pupil in one eye is responsive and the other is not. Her body is able to maintain blood pressure and her internal organs are functioning normally. She is taking one breath by herself while the remaining 14 is being done through the machine. Do you think there is still hope of survival or is she brain dead? Doctor: hello.I have read your message.The condition does sound serious.However, there are other tests that need to be done before saying the patient is brain dead.Just pupil is not enough.Can you tell us the exact location of the bleed? And the size of the bleed?Better still, if possible, upload the images of the scan.Best of luck.Awaiting your response.Dr S Mittal"
},
{
"id": 213871,
"tgt": "Why there is no moustache and beard growth for me ?",
"src": "Patient: Hello Sir.. I am 25 years old Male with 62 kgs weight. I have very less density of moustache and no beard . No hair on chest too. But there is good growth of hair in pubic areas. Can you suggest me the reason of less/no growth of moustache and beard and remedies to cure it? Please help me in this issue. Doctor: Hello Thanks for your query.\u00a0\u00a0\u00a0\u00a0\u00a0 It is due to some hormonal imbalance ,so consult some endocrinologist for treatment .Complete hormonal assay is essential \u2018Hope I have answered your query, I will be available to answer your follow up queries, \u201cWish you Good Health and trouble free speedy recovery\u201d"
},
{
"id": 153068,
"tgt": "Is Daxotel effective in treating liver cancer?",
"src": "Patient: my mother has 51 years old ,before 2 years my mother had abreast cancer then she felt pain in the right side she made Ultrasound the Dr said that she has a liver metastase and she must use 2 kind of chemotherapy 1- daxotel 2- xeloda after she had taken the first dose the Dr told her that she have to stop the daxotel because it has bad effect on the liver . Is that right ? please answer me because there are no symptoms on her related to the liver cancer except the pain . again answer me as fast as you can .thank you. Doctor: Hello. I appreciate your concern . Daxotel and xeloda is a good combination chemotherapy in a patient of breast cancer. However if the liver function test are not good then daxotel should be avoided. You can go for liver function tests for your mother to decide whether to give daxotel or not. Hope this solves your query. regards"
},
{
"id": 109073,
"tgt": "What causes severe pain in lower back?",
"src": "Patient: hi I would like to know what I should to do I have really bad beck lower pain and I used only tylenol and some hit pads but still hurts I am take caring of paralise person spine cord ijnury C4 and C5 so always I lifting her but main essue is my work I m doing landscapeing and I am female 25y old what I should do pain is really often. Doctor: HiThank you for asking HCM. I have gone through your query. Your problem can be due to repetitive stress injury or due intervertebral disc injury from heavy weight lifting. A physical examination with an orthopedician will be helpful in ruling out this. MRI might be needed in case of positive findings for intervertebral disc injury. Usually I manage such cases with NSAIDs like Nimusulide , muscle relaxants like tizanidine and local anti inflammatory gels. Muscle strengthening exercises and physiotherapy will be needed if it is long standing. Hope this may help you. Let me know if anything not clear. Thanks."
},
{
"id": 9717,
"tgt": "Suggest treatment for rash on scalp with hair loss and sore on upper lip",
"src": "Patient: I'm a fair skinned red head and over last year have been very sick, was in Hospital for GI bleed after having exploratory lap for recurrent cervical cancer, and wt loss from 135 to now wt 108, Dr. said everthing fine and that my body went thru a likely immune response after surgery because wbc's jumped to over 25 that same day and HGB went from 13 to 8.0 that was back in jan, following mt I developed a rash on scalp with hairloss noted, whole body rashes more frequent at nite, then a sore appeared on my upper lip, had bx of a sore on my back that also came up and looked the same as others, dermotologist said Lupus, had injections in lip and scalp with ketazole or something like that, then was also given desonide cream and oint, to put on lip, back and scalp, have had 2 more injections since then in lip and scalp. when went back to dermotologist again i told him that the creams were not helping my lip, so he decided to do a bx of the lip which was small and only required 1 stitch, f/u said that it was some kind of beginning cancer and now they are saying that they recommend i see a plastic surgeon, I mentioned to him the same day that the lymph nodes in my neck have been swollen for atleast 2-3mts and family Dr. put me on antibiotics, which by the way did nothing for them, when I went back I told her they were stilled noticable and she said sometimes they never go back down, but now this is worrisome because it isnt the nodes behind my ear, it is the ones in my neck and when I pointed this out to derm, he said that the plastic surgeon could also bx them, I'm a female and young at that 35, hx of HPV in 2004 with types 16 and up. even though I never had a wart. I had a complete hysterectomy in 2006 due to cin3,. I don't want to have a disfigured face, but I have complained now since Jan that i have been weak, sleep alot, vit d low. I just feel that my concerns have been ignored and would really like your opinion. Doctor: Hello,Since you have a history of HPV, you are suggested to get repeat HPV tests done. Some strains of HPV may cause harmless oral lesions like warts, but get clinically examined & get biopsy done to detect oral cancer at initial phase so that treatment can be started at the earliest.HPV affects hands, face, skin or scalp. To prevent loss of hair take diet rich in minerals, calcium, iron, magnesium and protein, take vitamin B12 rich food or supplements. Take Logihair, one cap daily for 30 days. Your treatment will depend on the clinical condition & the reports of the investigations done. Hope I have answered your question. Let me know if I can assist you further. Regards, Dr. Nupur K, General & Family Physician"
},
{
"id": 83730,
"tgt": "Does Femodene cause depression and weight gain?",
"src": "Patient: I am 39 and have been on and off the pill fpr years I stopped last year and have been very down with bad PMT so have just started back on Femodene , is this a good idae ? I have read that it can cause depression and weight gain , two things I am prone to thanks for you help regards becky Doctor: HiYou have mentioned about taking the pill previously.In case you have taken Femodene before and had severe side effects and thinking of restarting the drug again,you can start with another combined pill.Newer drug may be more acceptable and can cause reduced side effects.But only after starting the drug and watching for the pattern of side effects can help in further assessment.The cause of inability to gain weight should be identified.Nutritional deficiency,Chronic infections,Malabsorption syndromes,Parasitic infestations, Hyperthyroidism should be ruled out.Hope I have answered your query. Let me know if I can assist you further. RegardsDr.Saranya Ramadoss, General and Family Physician"
},
{
"id": 12913,
"tgt": "What causes itchy rashes in the knee",
"src": "Patient: I have an itchy rash inside of my knees...worse on the left than right knee. My right knee has been itching for several days, with no sign of any rash, bites, etc. Last night, however, I noticed a rash. The rash is raised areas, but not very red. It's actually kind of hard to discern. Also, I've feel like I've been running a low fever for about a week and have had swollen glands. I'm also absolutely exhausted all of the time. I'm 48 and just attributed everything to perimenopause, but the rash seems an odd thing, as I've never had a rash ever, anywhere. I've also been experiencing headaches and muscle aches, which isn't that usual for me. I have a dr's appt. today, but thought I'd ask anyway. Thank you. Doctor: Hi,You seem to suffer from menopausal syndrome. Consult the gynaecologist for the perfect diagnosis and proper treatment. The rash in popliteal areas may be due to allergy. For that consult the dermatologist.Hope this helps.Dr.Ilyas Patel, Dermatologist"
},
{
"id": 95395,
"tgt": "Pain in abdomen while on the treatment for liver abscess",
"src": "Patient: age-69yrs, female, gall bladder & appendix removed, treated for liver abcess. now her serum lipace is 694. under treatent of a general physician. should i go for gastrologist? Doctor: Hello Thanks for your query.\u00a0\u00a0\u00a0\u00a0\u00a0 If you are not relived by the treatment of general physician then go to the gastroenterologist without any delay. \u2018Hope I have answered your query, I will be available to answer your follow up queries, \u201cWish you Good Health and trouble free speedy recovery\u201d"
},
{
"id": 155022,
"tgt": "Need advice on treatment for metastases of TCC",
"src": "Patient: Hello Doctor, My father who is 84 years old is suffering from metastases of TCC. Currently he is asymptomatic. One Oncologist consulted wants to put him through chemotherapy. A second opinion from another Oncologist says that at this late point it is best to leave the patient untreated. Need your advise. Also, if any treatment can be provided to avoid pain and suffering, i would like to know about the options Doctor: Hi,Thanks for writing in.Sometimes it can get difficult on deciding the treatment of a patient having cancer with metastasis like your father. Chemotherapy can prolong survival in TCC with metastasis. It is important to know accurate details of the tumor in bladder and metastasis location. If the patient is asymptomatic then it is possible to further extend his survival by giving chemotherapy. Chemotherapy agents that have shown activity in metastatic bladder cancer include paclitaxel, docetaxel, ifosfamide, gallium nitrate, and gemcitabine and these are promising as they extend the survival of the patient even with metastasis. I suggest you discuss in detail with the second oncologist and then take an informed decision.Pain relief is an important part in treating cancers and this assumes a greater significance in those with metastasis. Pain relief is given using a step approach an that much of pain relief is given which can allow the patient to stay in comfort. Pain relievers are usually opioids and controlled medications."
},
{
"id": 150417,
"tgt": "Have breathlessness. Going to have deviated septum fixed and enlarged adenoids reduced. What will be the effect?",
"src": "Patient: I am scheduled to have my deviated septum fixed and my enlarged adenoids reduced, how will this affect me long term? I have trouble breathing through one of my nostrils and coupled with the development of asthma over the last couple years, I have trouble breathing especially when running. Will this procedure help me breathe better? Doctor: THIS PROCEDURE CAN HELP YOU IN BREATHING NORMALLY BUT AS YOU HAVE ASSTHMA AND ASTHMA IS TRIGGERED BY DUST AND AS YOU WILL HAVE MORE BREATHING MORE AIR WILL GO TO LUNGS AND ASTHMA CAN INCREASE ETTER GO FOR TRAETMENT OF ALLERGIC ASTHMA BY CONSULTING ALLERGEOLOGIST AND IF YOUR ASTHMA IS TRETED YOU CAN THINK OF SURGERY AND NOW A DAYS ASTHMA CAN BE TRETED WITH IMMUNOTHERAPY"
},
{
"id": 209208,
"tgt": "What are the signs of having Schizophrenia?",
"src": "Patient: h,i m wondering about my son,he is 19 yrs. of age,an ever since he was young child he has had a bd temper,he used to be good for a few mts. then crazy for next few mths. this was when he was about 4,at 10 mths. old he would bang his head off a wall if he was mad...now he s 19 an he can blow up in seconds,he s ok when he wakes up then within seconds his mood changes he becomes very verbally abusive,he can be physically abusive,he always beat on his 2 sisters when he was younger age 4 or younger,he says it feels good to hurt ppl. it gives him a rush,rite now he only hurts verbally,but u can see rite through his eyes when he gets in a certain state,an he scares me,it takes everythin he has not to be physical,but everyone that knows him,knows it only a matter of time,do u think he could be scitoprania(sorry for the spelling) i have a first cousin of mine that is,an ppl. through my sons life has said i should get him tested,there is something not rite with him,an my family doc. likes to just say he just likes to get wat he wants,e s ok..but he never tried to look at my son...plz. help Doctor: DearWe understand your concernsI went through your details. I suggest you not to worry much. From the given symptoms (the symptoms always are biased, they are according your view point) your son could not be suffering from schizophrenia. This could be Borderline personality disorder (BPD) and is a serious mental illness marked by unstable moods, behavior, and relationships. Most people who have BPD suffer from Problems with regulating emotions and thoughts, Impulsive and reckless behavior and Unstable relationships with other people.Therefore you should make sure to consult a psychiatrist for treatment. In my opinion, psychiatric treatment along with psychotherapy is a must in this case. If you require more of my help in this aspect, Please post a direct question to me in this website. Make sure that you include every minute details possible. I shall prescribe some psychotherapy techniques which should help you cure your condition further.Hope this answers your query. Available for further clarifications.Good luck."
},
{
"id": 65592,
"tgt": "What causes knee pain with lump on the top?",
"src": "Patient: Thanks. I have had knee pain for about 2 months now. No blood clots found and no broken bones. After about a month of the knee pain I noticed a lump on the top left of my knee (left knee). The lump is soft and does not hurt to touch. Please help. What is goin on with me? Doctor: Hi! thanks for sharing your health problem with HCM!After going through your problem of knee pain with lump on the top, I would like to consider following possibilities AS PER MY ROUTINE CLINICAL EXPERIENCE:1. benign cystic conditions like bursa, ganglion cyst etc possibility 60% 2. Inflammatory like synovitis, synovial herniation, patellar dislocation, synovial effusion etc , possibility 20%3. benign solid tumor or tumor-like conditions like giant cell tumor, lipoma, neurofibroma, dermatofibroma, cysts possibility 20%Therefore, it is could be a cause for worries in the future!.And you must confirm the diagnosis by ultrasonography and needle biopsy as advised by your doctor and thereafter discuss with him the possible treatment options.Regards,"
},
{
"id": 37841,
"tgt": "Suggest remedy for loss of appetite,stomach upset and viral infection",
"src": "Patient: Monday I was diagnosed wirh a virus. The doctor said it should last a day. However since monday I have not been able to eat. I take a bite of something and my appetite is gone along with stomach pain. If I even eat anything I quickly have to go to the restroom which results in diarrhea each time. I havnt been able to have a full meal and im suddly feeling weak. What is wrong with me? Doctor: Hello,Welcome to HCMYou have developed a gastritis which would take sometime to heal.If it is viral only, it would heal with a proton pump inhibitor.But it may be related to a bacteria called H.Pylori and if so it would require a triple regime.You have also developed increased gastrocolic reflex, the reason for you to go to the rest room after a meal.If you have further query, I am availableDr. Noble Zachariah"
},
{
"id": 81661,
"tgt": "Is twinge in upper chest while being overweight be due to anxiety?",
"src": "Patient: Hello...I am a 42year old female who is unfortunately about 25 pounds overweight....I do try to watch what I eat but fall off the wagon ever now and then. I used to exercise by running on my elliptical 5-6 days a week. I havent done that in some time since i had a herniated disc in my neck and was operated on last year. I did start back to the exercise about 3 days ago though. II have no history of heart trouble, no high blood pressure and my cholestrol runs borderline at times. At age 40, i was experiencing some anxiety and weird pain in my chest area so the doc sent me for a stress test.The cardiologist said mt heart looked fantastic and all was well. That pain turned out to be acid reflux. flash forward to present day...the last 5 days or so i have been feeling little twinges in my upper left chest area. Not painful but it scares me. I do not feel them when i ecercise and do not have shortness of breath or nausea/sweating. It seems to be mostly when i lie down at night. I am a major worrier and I am not sure if this is anxiety related or not. Of course wondering if your having a heart attack does little to sooth the anxiety. I currently take no medications but my multivitamin and fish oil. I did however just try a very low dose birth control pilll for the 2 months. I am no longer taking because the side effects were too much for me. I am not a smoker. I dont feel like I am having an emergency but thought maybe a call to the pcp is best. Advice? Doctor: Thanks for your question on HCM. I can understand your situation and problem. I think your symptoms are mostly related to stress and anxiety. But since you are over weight , taking oral contraceptive pills , I think weneed to rule out cardiac cause first. So get done1. ECG2. 2D EchoTo rule out cardiac cause. If both of these are normal than no need to worry much for cardiac cause. It is mostly due to anxiety. So avoid stress and tension. Be relax and calm."
},
{
"id": 143202,
"tgt": "Is it required to do surgery for spondylotic changes with thecal sac indentation?",
"src": "Patient: Doctor my sister had back and leg pain and directed to take MRI. The report says cervical and dorsal spine shows spondylotic changes and disc desiccation and there is a mild compression of bilateral exiting nerve roots at the foraminal lever due to l.5 & 4 disc protrusion. In the cervical region at c.5 &c.6 level the.is a disc bulge with thecal sac indentation. all other things are normal. Local doctor suggested for surgery. Is it necessary and will it give permanent curation or is there is any other treatment which can cure this without surgery. Doctor: Hi.I am Dr Mittal.I have read your message.I think I can help you.However I will need further information.You see, the decision for surgery is most often based on the clinical needs of the patient. The symptoms like difficulty in walking, degree of pain, bowel bladder complaints are the contributory factors to decide the need for surgery.MRI is not the only criteria for surgery.So, you need to tell me more about the patient before I can make a comment on the need for surgery.Besides, you can always meet another spine surgeon and get a second opinion.Meanwhile, you can get a prescription for pregabalin, and methylcobalamin. Make sure that the patient takes rest and does not bend or lift heavy weights.Physiotherapy will have to wait till I know more about the patient, unless the doctors who have seen her there do recommend it.I have tried to make it as simple as possible.Please add the information requested so I may be able to help you further.Best of luck, Dr Mittal"
},
{
"id": 99429,
"tgt": "Suggest treatment for asthma and allergy",
"src": "Patient: my son is 6 years old and has been diagnosed with Asthma. he has been coughing for a month. He has been on antibiotics and steroids. The cough has subsides somewhat but he coughs a lot only when hes eating only. not sure why but he is irritated by it and so are we. He s seen an allergist and was put on Qvar. Doctor: Hi,Asthma is itself a allergic disease due to cold or dust or pollens or grass etc irrespective of the triggering factor.You can go for pulmonary function tests if not done.Treatment is mainly symptomatic which might require inhalational steroids,beta agonists,anticholinergics as MDI or rotahaler as a regular treatment.To decrease the inflammation of bronchi and bronchioles,your son might be given oral antihistamines with mast cell stabilizers(montelukast or zafirlukast) and steroids (prednisolone) with nebulization and frequently steam inhalation.The best way to cure any allergy is a complete avoidance of allergen or triggering factor.Consult your pulmonologist or paediatrician for further advise."
},
{
"id": 91189,
"tgt": "Suggest remedy for stomach pain",
"src": "Patient: Hi I have stabbing pains in the middle of my ribs at the top and tightening pain in my lower stomach like I have been doing sit ups but i havnt been doing any excersise in a long time?? And sore ribs also and stabbing pains in my stomach sometimes it keeps moving in different areas in my stomach. Doctor: Hi,Abdominal pain could be very difficult to exploit at times without a proper clinical review, a better characterization of the pain or a proper physical examination by your doctor. Causes could range from simple medical problems like gastritis and infections to surgical emergencies like acute appendicitis that could be missed from the start. A complete review by your doctor, blood tests and at times an abdominal ultrasound from your doctor would be most useful. Kind regards"
},
{
"id": 159790,
"tgt": "Non-hodgkins lymphoma,swelling in supercervical area,red scaly skin,mono",
"src": "Patient: I am a 55 year old female. My two older sisters have both had non- Hodgkins lymphoma . About a month ago I noticed visual swelling in the supracervicular (spelling?) area of my neck/shoulder/clavical. I also have had a red scaly unhealed skin irritation in this same area for about 5 months. I have had numerous cases of mono dating back to the age of 7. It is dfficut to do palpatation in this area on myself. Is this something to be concerned about? Doctor: hi mam, with a strong family history of NHL itzz not necessary u worry about it but a significant supraclavicular lymphadenopathy does require tests jus to rule out infection vs lymphoma. kindly get examined by a specialist ( palpate the node shape, size,consistency) get a FNAC/ SERUM LDH if necessary. as u have not mentioned any A/B SYMPTOMS OF lymphomas ( fever/drenching night sweats/weightloss more than 10% of bodyweight, anorexia ). kindly meet ur physician and get the things done. all the best"
},
{
"id": 209853,
"tgt": "Need medication for having scary thoughts after a death of family member",
"src": "Patient: hi I am 16 years old and I am scared of everything, I can't shower or use the toilet because I thinkSomeone will get me and I can't watch a scary movie or a documentary about anything. My parents think this is because I witnessed my grandma die and her funeral die whenI was four at Pakistan. Doctor: HiThanks for using healthcare magicI think, you have specific phobia. In that case, you need proper treatment in form of antidepressant and behavioral therapy. Antidepressant would help to control phobic thoughts and exposure therapy would help to decrease this false behavior. Better to consult a psychiatrist and psychologist for proper diagnosis. In case, you need further help, you can ask.Thanks"
},
{
"id": 173583,
"tgt": "How to treat constipation of an infant?",
"src": "Patient: infant constipationhave infant baby girl one month five days n for the last ten days she has not devacate.The 3rd day took her to hospital n prescribed amlico and ors medicines,nothing changed.Seventh day went back to hospital n prescribed liquid paraffin.Today is tenth what i do now Doctor: Hi,Thanks and welcome to healthcare magic.Constipation in a one month old infant needs probing.In the newborn period the baby may not pass motion for a week sometimes .It may be also congenital hypothyroidism .First relieve constipation by Dulcolax suppository baby size.for a few days.Latter Lactulose syrup daily may be given to maintain normal bowel movements.Then sreen for congenital hypothyroidism .If found positive consult pediatrician for confirmation and treatment.Hope this answer serves your purpose .Please feel free to ask further queries if any.Dr.M.V.Subrahmanyam."
},
{
"id": 26909,
"tgt": "Suggest treatment for cold , low blood pressure and fever",
"src": "Patient: doctor said iam very loe I went to Maplewood saint joinh Emergency hospital last night ,because I was very cold high fever dr said i am very low blood presure doctor send me to xray blood test every they are good dr give me tylenol take 2 piil every 4hr then send me home . right now i am repead again woul you tel me what to do next please . thank you Doctor: Hello and thanks for writing.I can understand your concern and would try to help you in the best possible way. Sometimes high fever may lead to low blood pressure. I am sure that your doctor has investigated you for the fever and has come to a conclusion that your fever is probably due to flu and has put you on tylenol.Be assured that once your fever is controlled then you BP would normalise to your pre illness levels. In the mean time you may drink plenty of fluids along with salts to help prevent any dehydration and also raise your BP. I also suggest you to take ample bed rest and avoid physical activity like getting up from bed suddenly and prolonged standing."
},
{
"id": 162754,
"tgt": "How can food allergy in a toddler be managed?",
"src": "Patient: hello sir, my 2 year and 6 months old daughter has some food allergies she has reacted with kiwi fruit and peanuts i have stopped after reaction now doctor has done some blood test after peanut allergy reaction,and report says she is allergic with all tree nuts ,wheet and coconut .so doctor has told me to stop these food but she is totly fine with these food which blood says is not ,I cannot understandwhat is best for her . b Doctor: Hello and Welcome to 'Ask A Doctor' service. I have reviewed your query and here is my advice. I assume that the initial testing for food allergies was a skin test, called the prick test. Negative tests are accurate for excluding food allergy.However, positive tests can occur in patients without symptoms related to a particular food. The blood tests were most likely RAST testing. Some allergists believe the RAST has no advantage over skin testing. Not too long ago there was a study in the medical journal, Pediatrics,(journal of the American Academy of Pediatrics) which said that neither skin-prick tests nor RAST testing are diagnostic of food allergy.What this tells us is that these tests are not perfect to diagnose food allergy. The best evidence of whether or not your daughter has food allergies is that she has been given these foods and is \"totally fine.\"You have shown that it is safe for her to eat these foods. Hope I have answered your query. Let me know if I can assist you further. Arnold Zedd, MD, FAAP"
},
{
"id": 212976,
"tgt": "Feeling of fear on seeing new person, driving, fan movements. No relief with treatment. Cause and remedy?",
"src": "Patient: hi i am S.K.NAG from bhubaneswar,i am always feel affraid to see building,new peson ,whn drive bike,when i sleep at room ,to fan movement, when i going on lift ,when i see showroom.i am suffering this since last 1 year,i am consult doctor but i am not ok til now. plase give me information about the problem what can i do . Doctor: Dear Cocerned., Thanks for writing to us. The condition you are describing is a simple mood disorder termed as \"Phobic Neurosis\". The treatment involves finding out the root cause of Phobia by Pschoanalysis and treating the same by constantly speaking to you or invoving you in Psychotherapy by a qualified Psycho therapist/Psychiatrist.It is a long term treatment. In addition Oral Anxiolytics or Tablets are also prescribed for this as adjuvant therapy. Consult a Psychiatrist pls. Thanks, Regards., Dr Lt Cdr ASN Bhushan."
},
{
"id": 90660,
"tgt": "Suggest treatment for fullness/heaviness of abdomen and shortness of breath",
"src": "Patient: Dear doctor, i am suffering from gastroentorology since three months & i am from INDIA.....i have consulted many doctors(7-8) but the medicine they gave till now didn't recover me. I am suffering from fullness & heavyness of abdomen and i can't breathe properly. I have gone for endoscopic trearment,ecg,ultrasound but all the reports are positive or normal so please help me to cure....thank you. Doctor: are you obese with central obesity?if the answer is yes then wt reduction is must.central obesity causes breathing difficulty.now comming to your other problem fullness and heaviness is your tongue white coated?if yes then this heaviness is due to aam.so it shows that you have tivra agni mandata.if i was your treating doctor i would advice you agni vardhak and aam pachak chikitsa,if your bala permits then i would give you a madham sarak aushadhi for aam shodhan.also light diet,frequent drinking of hot water,butter milk impregnated with hing,jeera,saindhava,pepper is advisable."
},
{
"id": 52649,
"tgt": "Suggest tests to confirm hepatitis in the elderly",
"src": "Patient: I had hepatitis when I was in the 3rd grade, I was held out of school for what seemed like a long time, I had to have my blood tested often and they had me on a diet that included a lot of steak. I m now 71 years old and don t know which type I had and if there are any concerns I should be aware of. I remember I was told I should never give blood. Doctor: Hello,You might have HBsAg carrier stage in which patient can live a normal life if no hepatic damage occurs. To rule out the type of hepatitis, you can investigate with viral hepatitis profile by as following-Serum HAV antibody testing by Elisa. -Serum HBsAg testing by Elisa. -Serum HCV antibody Elisa test. Usually, these three viruses lead to hepatitis. So rule out it. Consult a physician for examination.Hope I have answered your query. Let me know if I can assist you further.Regards, Dr. Parth Goswami"
},
{
"id": 167038,
"tgt": "What causes red and yellow blisters all over the body?",
"src": "Patient: my 22 mth old daughter has couple red spots on legs, few in genital area and now approx 10 down side her body, under her arm and down towards her tummy - all look the same, red, yellow small blister in middle and all surrounded by circular bluish bruise like thing-what could they be? Doctor: Hi....skin lesions are best diagnosed and treated after direct visualisation. Please upload images on this forum and get back to us.Regards - Dr. Sumanth"
},
{
"id": 28866,
"tgt": "Can MRSA of the lungs be treated?",
"src": "Patient: We have a friend whose son is hospitalized with a variety of illnesses. What landed him in the hospital in the first place was a gran-mal seizure. He has had further seizures & stroke. The latest diagnosis is very severe since it is MERSA of the lungs. We fear for him and his family. Does this young man s family have any hope for him to recover from such severe health problems? Doctor: Hello,Yes, if given proper treatment MRSA can be eradicated. Based on the history your friend seems to be suffering from hospital-acquired MRSA infection which can be life-threatening if not treated.MRSA (methicillin-resistant Staphylococcus aureus) is a specific bacteria that is often resistant to (is not killed by) several types of antibiotic treatments which can be harder to treat or become worse if the appropriate treatment is delayed.He should be examined by a lung specialist (pulmonologist) to evaluate his condition properly. Since MRSA is resistant to many antibiotics, the treatment will depend on the culture and sensitivity of the blood/sputum. The doctor will choose antibiotics that the laboratory reports as susceptible on the resistance test they perform in each culture. The antibiotics like Vancomycin or Linezolid or fifth generation cephalosporins are found to be effective in susceptible infections. Further, since MRSA is a highly contagious disease every effort should be made to avoid making direct contact with skin, clothing, and any items that come in contact with your friend.Hope I have answered your query. Let me know if I can assist you further.Regards,Dr. Mohammed Taher Ali"
},
{
"id": 200924,
"tgt": "Suggest treatment for masturbation addiction",
"src": "Patient: hi i am 28 years old . i have sex problem that i did so many times handjob because i can t control my self when see some one like sexy and sexual thing that time im going to this bt i can t take more time within in 5 minutes it s gonna be lickout so im worried about my future life when i ll merry .please give me suggesion what should i do ? please sir........ Doctor: Hi, If I were your treating Doctor for this case of so called addiction I would suggest you not to worry about future life and indulge yourself with harder physical activities!Masturbation do not affect any one's sex life but take care not to hurt or infect your organ... you might have some counselling by a psychiatrist if still not satisfied!Hope this answers your question. If you have additional questions or follow up questions then please do not hesitate in writing to us. I will be happy to answer your questions. Wishing you good health."
},
{
"id": 4569,
"tgt": "Is it possible to get pregnant without taking prolifen while having all the reports normal?",
"src": "Patient: hi doctor i m 27 years old and got married 1 and a half year before, 6 months before i got pregnant by taking prolifen tablets but got early miscarriage now doctor again suggested me to take prolifen for pregnany and when succedded then start taking loprin for 3 months whats the reason of it? why i cant get pregnant without prolifen when everything is ok according to doctors? Doctor: Hi,Thanks for the query. Prolifen helps in inducing ovulation. In case of regular periods, ovulation is possible without taking induction drugs also.You have to plan unprotected intercourse around the time of ovulation to increase the possibility of pregnancy. Ovulation can be tracked by different methods like cervical mucus method, ovulation kit etc.For more details : http://srsree.blogspot.in/2013/10/ovulation.htmlIf you are not getting the ovulation regularly, better to take the help of ovulation induction drugs as advised by your doctor. Start taking folic acid supplementation from nowonwards. Take care."
},
{
"id": 170714,
"tgt": "Can appendicitis be cured by medicines?",
"src": "Patient: sir,my son is 10yrs old,he had pain in stomach after having sonography done the result was of apendix .it was noted as subtle thickening of the appenicular wall noted with probe tenderness to suggest sub-acute appendicitis.no lump seen.sir please suggest me that can this apendix is cureble by medicene or be operated. Doctor: HI...usually sub-acute appendicitis is something which will require conservative management and antibiotic therapy initially and then later on will definitely need surgery. I suggest you consult a pediatric surgeon regarding this and the kid might require even hospitalization and Intravenous antibiotics.Regards - Dr. Sumanth"
},
{
"id": 148687,
"tgt": "Panic attack, cold and sweaty feet, shaking hands, stuffy nose. Help",
"src": "Patient: Hi doc good morning and thanks for your time I am 18 years old and my name is kayda I have once got a panic attack and after I was having nerves problem it doesn t affect me every day but when it does I am weak my feet are cold and sweaty and my nose stuffy along with shaky hands. Sometimes I think that something really bad is going to happen but doctors said that nothing wrong I am also afraid of going out and being all alone by my self can u please help me. Doctor: Hello KaydaFrom your history it appears that you are suffering from phobic anxiety and at times go into panic I would have given to my patient one SSRI group of drug plus one anti anxity benzodipine group of drug in case of more severity or non responsiveness I would have added betablockers in small doseI suggest that you meet your psychiatrist for prescription and discussions to ger reassuranceLet me tell you it is treatable conditions so dont worry pleaseTake carevDr Lal Psychiatrist"
},
{
"id": 1331,
"tgt": "What are the chances of getting pregnant with blighted ovum?",
"src": "Patient: im married 2yr 8months ago.. i conceived in nov2010 but got natural abortion.. again i conceived in oct 2011 but unfortunately it is blighted ovum. doc gave me MT pil and misogol tablets i have discharge large mass of tissue.After 10 days she did MTP.. yesterday she did MTP. AND gave me feminol, polka-oz omee-d, etc .. tablets.. why blighted ovum happens? and when r the chances to get pregnant again? feminol is contraceptive pills but y doctor suggested that tablets as im eagrly waiting for next pregnancy? and how long we need to use this contraceptive pills? i want to get pregnant? please reply me Doctor: Hi ,How are you doing ?I understand your concern. Let me assure you that you will surely get pregnant & will deliver a healthy baby.You get a good baby only when a healthy sperm meets a healthy egg at the right time , at right place & sits at right place in uterus. Accidents like these do occur , but not always.In blighted ovum, either the egg or the sperm was not healthy. We always try to avoid pregnancy for 3 mths after any abortions, so that the underlying facts for the abortions disappears. Don't bother to find the cause, as many are not recurrent. So that's why your doc has advised Oral pills. Folic acid will reduce the chances of abortions. Do have the following tests to rule out few treatable causesHusband - Blood sugars - fasting & post-prandial Semen analysis- preferably at any infertility centerWife - Blood sugars - fasting & post-prandial Serum Prolactin & Thyroid profile aPTT, Lupus Anticoagulant ( LA) Anticardiolipin antibobody ( ACA- IgG & IgM)All these can be causes but treatable .Hope I have clarified your queries, do write back if any more queriesAll the bestDr.Balakrishnan"
},
{
"id": 220765,
"tgt": "What is the best treatment to get pregnant?",
"src": "Patient: hi there. i am trying to get pregnant. its been 1 and a half year now. i have a 2 year old son so i no i can concieve. my periods are irregular and the last time i was on was the 3rd of august and have not had one since. i have done a home preg test and its neg. i really want to get preg. some times i have pain were my ovaries are the funny thing is that i feel movements in my tummy. i really dont no what is happening. Doctor: Hello dear,I understand your concern.In my opinion the irregular cycles can be the cause for infertility.The causes for irregular cycles are PCOD,thyroid abnormalities, overweight or obesity,stress etc.So I suggest you to consult your doctor to rule out the cause for irregular periods.Once the cause for irregular periods is treated the chance of conception increases.So nothing to worry.Avoid stress if any.If overweight or obesity weight reduction might be helpful.Nothing to worry.Best regards..."
},
{
"id": 96478,
"tgt": "acalculous cholestytis",
"src": "Patient: My father is diagnosed with acalculous cholestytis and was admitted in hospital for 4 days and was given antibiotics and is fine now. I wanted to know is surgery required or will it be cured by antibiotics. He does not have pain in abdomen now Doctor: Hi.. Acalulous choecystitis secondary to infection can be treated with antibiotics alone. Surgery in such cases is indicated if there are any impending complications developing.\u00a0 However if the cause has been some other factor, then surgery is mandatory folllowing a course of antibiotics... So\u00a0 leave it to the treating doctor to decide it surgery is essential in your dad...."
},
{
"id": 39177,
"tgt": "What causes fordyce spots over labia majora and minora?",
"src": "Patient: Hello, I have what I m sure are Fordyce spots on my lady region. I noticed them about a month ago, but I wasn t sure what they were. Now I realize they are quite clearly Fordyce spots all over my labia majora and minora. I ve done my research, so I m not terribly concerned about them. However, over the past two days my lady region has been slightly itching. I was wondering if the Fordyce spots could possible have anything to do with it, or if there s another reason for it. Thanks Doctor: Hello,Welcome to HCM,Fordyce spots are visible sebaceous glands that are present in most individuals. They appear on the genitals and appear as small, painless, raised, pale, red or white spots on the labia.These are not associated with any disease or illness and they are non infectious. No treatment is required unless the individual has any cosmetic concerns.No treatment is required for Fordyce spot except for cosmetic removal of labial lesions if the individual wishes.I would suggest you to consult a dermatologist to confirm the diagnosis of fordyce spot.This can be treated by laser treatments such as CO2 laser and electro desiccation have been used with some success in diminishing the appearance of this condition.Inflamed glands can be treated by topical application of ointment clindamycin. Thank you."
},
{
"id": 9117,
"tgt": "I am 28 years old. How can i remove stretchmarks in my stomach area ?",
"src": "Patient: hi i m mona jain, age 28 single, i hv lots of streachmarks in my tummy portion n even loosed skin there(due to weight loos) n i want to reduce it as now i m getting married n i hv to wear a saree in my daily routien..? u hv any solution for this Doctor: Apply oint. Clarina(Himalaya) for around 1-2 mths"
},
{
"id": 25158,
"tgt": "Is shoulder pain a sign of heart attack or a result of injury to thumb?",
"src": "Patient: hello, i smashed my left thumb two weeks ago and it it is still swollen abd the nail looks like ts doing to fall off and is still very sore. a week ago my left shouder started aching badly and wont stop. can this be related? i had a heart attack in march of 2007 and i know shoulder pain can be a symptom. im 6 foot and weigh 220 lbs Doctor: shoulder pain can be angina but it shouldn't be at same pin point place. your pain is muscular or skeletal. you should take some analgesic if pain is severe"
},
{
"id": 69746,
"tgt": "What causes a knot in right upper quadrant?",
"src": "Patient: I have a knot inmy right upper quadrant. it is firm and causing no pain. I am a white female, 5'5 and 180 in generally good health with a history or ovarian tumors taken care of by hysterectomy 10 years ago. My CA125 has been wnl since hysterectomy. I am a rare to never drinker Doctor: Hi. This looks to be lipoma as this is not painful and you do not have any other symptoms.You have to undergo a proper clinical examination to be done by a Doctor who can catch many of the problems on your proper history and physical examination and can then guide for further investigations and treatment."
},
{
"id": 21652,
"tgt": "Suggest treatment for headache and high blood pressure",
"src": "Patient: Hi, My mother suddenly started feeling like her head was reeling badly this morning (room swaying) and when she went to the doctor her BP was 160/100. She is 61years old and she's been having indigestion all day. She was fine and in the evening her BP came down to 125/80 and now again she feels like her head is reeling. Please advice.. Doctor: Hi There I understand your concern for your mothers health, I would like to suggest you to keep a regular check on her Blood pressure if it is staying above 140/90 mmhg then I would like to start her on some low dose Antihypertensive medicine like AMLODIPINE once daily. But before that it's better you take her personally to a doctor so that she could be examined properly. I would also like to suggest to maintain a low salt diet for her if her BP is staying high.I wish her good health"
},
{
"id": 132034,
"tgt": "Can calcification causes ribs to be more thinner and pressured?",
"src": "Patient: I ve recently started working out and have lost some weight about 20 lbs. I m a 44 y/o male. I m 5-10 170lbs I ve noticed that the lower portion of my left rib cage seems to stick out further and feels bonier making it uncomfortable to lie down on my left side. There is no pain it s just pressure. This was never a problem before. When younger I was thin and my rib cage was slightly visible. It would have been obvious if it was not symmetrical then. I do not drink a lot of milk and haven t for many years. Is this just a calcification and a normal part of ageing? Doctor: U should go for bone investigations like serum calcium and phosphorus, bone densitometry and orthopaedics opinion."
},
{
"id": 219984,
"tgt": "Can you see if a baby has cervical ribs during prenatal ultrasound?",
"src": "Patient: I m hoping someone can help me out. I m pregnant and have bilateral cervical ribs. I have read about the increased risk in childhood cancers in children who have cervical ribs. So i have two questions. 1 the cancers that are increased, are they something that cord blood/stem cells would help? 2 Can you see if a baby has cervical ribs during a prenatal ultrasound? Doctor: HiDr. Purushottam welcomes you to HCM virtual clinic!Thanks for consulting at my virtual clinic. I have carefully gone through your query, and I think I have understood your concern. I will try to address your medical concerns and would suggest you the best of the available treatment options.first of all do not panic.About cancers and stem cells- use of stem sells in the therapy of various diseases is being vastly resaerched.It is claimed to be useful against the treatment of certain diseases, some types of white blood cell cancers. So my suggestion is to talk with your local stem cell bank about cost effectiveness and then decide to ahead with the same.As far USG of baby is considered, cervical rib is as such very small and will be very difficult to detect on USG.Even in adults it becomes difficult to detect on X ray chest.I hope my answer helps you.Thanks.Wish you great health.Dr Purushottam S Neurgaonkar"
},
{
"id": 185557,
"tgt": "What causes pain in gums after being treated for sinusitis?",
"src": "Patient: hi I am being treated for sinusitus with 4mg methylpredisolone almost finished with dosage but now it seems like my gums hurt? I have a dental appt this week. I am 69 and in good health female I cannot tell if its my gums or my sinuses? I do have a history of sinusitis but no dental history. Doctor: Hello,You can experience oral discomfort from sinusitis. If congested with difficulty breathing, you may tend to breath through your mouth and dry out your tissue. Dry gums can be sore. Post nasal drip can also cause discomfort. Acidic liquid can again bother the gums. It will take time for your inflammation to subside. You are taking a potent drug which tells me you had a great deal Of inflammation. Many patients only require an antibiotic to treat sinusitis. Steroids have many side effects. Sore gums are not a major side effect but may be possible. Make sure you have ruled out an allergic reaction. Keep up with good oral hygiene and make sure to rinse your mouth. Warm saltwater rinses may offer you some relief. Keep well hydrated and be sure to keep a healthy diet without any vitamin deficiencies. If your oral symptoms continue or worsen see your doctor to rule out a yeast infection or other type of oral Imbalance of bacteria that may require additional treatment. Be sure to give your dentist a full medical history concerning your recent treatment and your dentist will be glad to provide needed treatment for your gums. I am glad you have a dental visit planned. I hope you will be feeling better shortly."
},
{
"id": 26556,
"tgt": "What causes shortness of breath and elevated heart rate with weakness in legs?",
"src": "Patient: Hi, I have been having some health issues related to shortness of breath. Originally the doctor treated me for pneumonia but upon results of a chest x-ray they found no pneumonia and no fluid on the lungs. I was sent to a cardiologist because an EKG came back abnormal and my heart rate was elevated to 123. The cardiologist ordered an echo, he says it came back normal. He doesn t beleive there is an issue with my heart. My family doctor has called and requested a follow up appointment due to an elevated alkaline phosphate level. My other symptoms include shortness of breath upon any kind of excretion, something as simple as walking into the next room leaves me out of breath to the point that I feel like I am suffocating. When this occurs my legs get weak and my arms become like jelly and I feel light headed. I am having trouble sleeping since this has started, most nights I m not tired but when I am I cannot lie flat because I can t breathe. I spend most of the day dozing off due to lack of sleep but am not actually resting. Doctor: hi, you have not mentioned your age and sex. possibility of pulmonary embolism has to be ruled out, u can get doppler study of your leg veins and arteries also pl get done ct scan of thorax,along with some blood tests for kidney thyroid lipids etc."
},
{
"id": 177992,
"tgt": "Is it safe to give frozen breast milk to a toddler?",
"src": "Patient: Hi im a working woman and my baby is 3 years old, is it safe to give her breast milk which is saved in refrigerator? My baby is not taking lactogen. She is taking my milk in bottle but giving her breast milk through bottle cause motion. Kindly suggest me formula Doctor: Hi, I'am Dr Suresh K Yadav MD (paediatrics), I had gone through your question and understand your concerns,Yes you can give her breast milk saved in refrigerator . You should train her to take milk by spoon. Any approved formula available in market which suits her taste can be given. one more question ,is she 3 year or 3 months old . In case of 3 years she should take directly by glass, and no need of formula , she can be given same milk as used for adults . She should also take other diet apart from milk .Hope this answers your question. If you have additional questions or follow up questions then please do not hesitate in writing to us. I will be happy to answer your questionsTake care."
},
{
"id": 74492,
"tgt": "What causes pain in upper left chest?",
"src": "Patient: Hello. I have been experiencing pain in my upper left chest for quite some time. I do have a lesion on my 3rd anterior rib but do not yet know if it is mets (need further testing). My question is, while I am 99% certain this extreme pain is due to the tumor, I cannot seem to find a 'description' online of the type of pain these tumors cause (eg: dull, sharp, constant, etc.) I dont think it is my heart at all but I do have an extensive cancer history. I am a 36yr old female. Can you describe how rib tumor pain usually is described by patients? Thank you in advance. Doctor: HiTjere is no specific term for this.It is prescribed from patients with a lot of words.Regards Dr.Jolanda"
},
{
"id": 77818,
"tgt": "What causes lung pain when suffering from cold?",
"src": "Patient: I have a cold and the left side of my lungs hurt slightly so does my back when I breath out. the other night I had alot of gas and so my hurt started to beat really fast. what can this all mean. I have no problems in my body. is this bronchitis or what can I please get an answer I am 14 years old Doctor: This is pleurisy type of chest pain..pain due to cough will arise from throat, bronchus and pleura ..so kindly take a short course of azithromycin 500 mg once a day for five days and reevaluate if it is paining.use salt goggles. .and nd throat lozenges mam..if it is paying at the depth of breathing then better get an chest x ray or ultrasound done as it may show up some fluid or patches"
},
{
"id": 122852,
"tgt": "Can neuroma cause pain in buttocks?",
"src": "Patient: I have been experiencing a feeling of sitting on a stone. The pain is right in the middle of my left buttocks and happend when I sit on a semi firm surface. I have a Morton s neuroma on my foot and this pain is very similar. Can you get a neuroma in your buttocks? Doctor: Hello, It is not related to a neuroma. Most probably it will be a neuropathic pain like sciatica. As a first line management drugs like gabapentin can be tried. If symptoms persist better to consult a neurologist and plan for an MRI scan. Hope I have answered your query. Let me know if I can assist you further. Take care Regards, Dr Shinas Hussain, General & Family Physician"
},
{
"id": 156086,
"tgt": "Suggest treatment for cervical cancer",
"src": "Patient: I have regular periods, but I have had a D&C performed. Lately I have been having problems which have been tagged as vulvuvaginitis by my doc. Biopsy of cervical tissue shows ulcerations, and TVS shows a left adherent ovary. I am completely distraught because m scared of having cervical cancer and can I take any vaccination for cervical cancer at 27yrs? Any suggestions on vaccination?M about 68kgs and 5.1 ft tall. Doctor: Cervical cancer would have been picked up in any one of these procedures.Yes you can be vaccinated. Gardasil a quadrivalent vaccine is available. You are over weight . regular exercise and diet changed are advised.Get pap smear done every 2 years.Practice safe sex and avoid multiple sexual partners."
},
{
"id": 83571,
"tgt": "Could acidity and less erection be due finax tablets?",
"src": "Patient: Hi, My dermatologist had prescribed me Finax tab and a particular hair oil for cure of hair loss. However after taking tabs for 1 month I noticed increased acidity and also lesser errection. I am afraid that it might lead to impotency so I have stopped taking this tablet now. Please suggest. Doctor: Hi,Yes, these are some of the common side effects of finax. The common side effects of finax include headache, weight gain, abdominal pain, and swelling of the fact and lips.Since it acts by preventing the formation of active male sex hormone, it can cause decreased interest in sexual intercourse, inability to have or keep an erection loss in sexual ability, desire, or performance.Ask your doctor for an alternate effective but a safer medicine for hair loss like minoxidil lotion.Hope I have answered your question. Let me know if I can assist you further. Regards, Dr. Mohammed Taher Ali, General & Family Physician"
},
{
"id": 117531,
"tgt": "Does Pernicious anemia causes body odor?",
"src": "Patient: I have been having a strange body odour For almost 4 years! No doctor could find anything ... just recently i was diagnosed with PERNICIOUS ANEMIA. I have read that this disorder causes body odour. Will this body odor go away once I begin treatment? How long does it rakes? My doctor diagnosed me 2 months ago but have not given me b12. I dont think he knows much about this..please Help me Doctor: Hi,Thanks for asking.Based on your clinical history and query, my opinion is as follows:1. Pernicious anemia is due to intrinsic factor deficiency, which leads to vitamin B12 deficiency.2. Vit B12 can be taken in injectables form, and it should show response within 15 days. 3. Due to increased activity due to anemia, you might be sweating a lot. This could be the possible cause of body odor.4. Take Vit b12 in injection form and followup.Hope it helps.Any further queries, happy to help again."
},
{
"id": 182578,
"tgt": "What causes hole formation on the inside of cheek?",
"src": "Patient: age - 25height - 5ft 5weight - 10 stonemedical history - asthma, eczema.I've had this pain in my mouth for about 4 days, each day getting worse. its where my wisdom tooth would be, so i though it would be my wisdom tooth growing. i finally had a good look inside my mouth and notice a small hole forming on the inside of my cheek. i am a smoker, between 15 to 20 a day. i fear it could be something to do with that. any advice will be great. Doctor: Thanks for your query, i have gone through your query.The hole what you are describing on the cheek can be a pus discharging sinus tract secondary to wisdom tooth infection.or it can be a traumatic ulcer. If it is near thesecond molar then it can be a inflamed salivary gland duct opening.Nothing to be panic, consult a oral physician and get it evaluated.If it is a wisdom tooth infection or inflamed salivary gland duct opening, take a course of antibiotics like, amoxicillin 500mg TID and metronidazole 400mg BID for 5 days And get the tooth removed aftrr that. If it is a traumatic ulcer you can use topical anesthetic like 2%lignocaine gel and choline salicylate apply 3-4times daily for 5 days."
},
{
"id": 56327,
"tgt": "Suggest treatment for abnormal ALT and SGPT levels",
"src": "Patient: I had a series of blood tests and the only out of range was ALT/SGPT: 91 U/L the normal value reported by the lab is 0-53 U/L. I repeated the test after 2 weeks and the result was 89 U/L. Is that an alarming value? does it indicate that my liver is damaged? The AST/SGOT in the first test was 35 U/L and in the second was 34/U/L. I am 32 healthy man 6.01 187lb I don t smoke or drink, I do a lot of sports I eat healthy, not so much meat.. I take vitamin and mineral supplements. I would like to understand what can cause this abnormal value. Is it as high as hep virus infected livers? Can stress, fatigue allergies also increase it? Thank you Doctor: Dear welcome to HCM and thanks for choosing us. I understand your concerns. Firstly, I would say you should be cautious but don't worry too much. You have most probably got hepatitis, which is inflammation of liver cells. In the beginning it is reversible. However I would like to know following your general health like weight, blood pressure, ageI would advise following to heal your liver 1 healthy and balanced diet2. less animal proteins for 3 weeks and then continue as per your habitTo know about possible disease in future you need to do following1 liver function test at least once in three months till you get normal reports. Then you can do once in six months2 immediately report to your doctor if you get pain in abdomen, fullness in abdomen, yellowish urine, yellowish skin, extreme fatigue. I hope my reply will help you. Please write to us or me for more questions. I wish you good health. Kindly rate this reply. Thanks"
},
{
"id": 193642,
"tgt": "What could be the bubbles rising around the genital area?",
"src": "Patient: Hello Doctor, There are bubbles rising around my genital organ (M) Earlierly it was only one, and now there are around 6 to 7. The color is same as the one around the organ but bit shiny. They dont pain, neither secret any smelly fluid. But the no. is increasing.What is the reason for these and the cure? I am 24 yrs old, 5.9\" tall, 57kgs. (I tried sex with a social worker 7 months back) Doctor: Hi, If you had unprotected sex - need to rule out sexually transmitted infection. If not it can be bacterial infection / fungal infection. Share a picture for conformation. Hope I have answered your query. Let me know if I can assist you further. Take care Regards, Dr S.R.Raveendran, Sexologist"
},
{
"id": 16325,
"tgt": "Allergic rashes on face, like measles",
"src": "Patient: Hi, I think I m allergic to something I don t know. I ve got a rush all over my face , it more like measils. Would like to know if allegex will be good for my condition. Thanks Mpho Doctor: hi mpho, Allergic rash on your face might be due to- i) Hair dye ii) Sun exposure or iii) Any cosmetic Whatever the cause, You could take an antihistamin tablet e.g. ATARAX 25mg twice/thrice daily if ALLEGEX is like that you can continue but before that u must avoid apply culprit agents. Regards, Dr. M M Mahmud"
},
{
"id": 150668,
"tgt": "Done abdominal hysterectomy. Tingling in tongue, dizziness, palpitations. Help",
"src": "Patient: hi doctor, I had a total abdominal hysterectomy 3 weeks ago. I obviously had a general anesthetic too. I developed some new symptoms last week which seem to be getting worse, I have a numb tingling feeling in my tongue , have a dizzy blury heavy feeling in my head and have dizzy spells and heart palpitations too. I am 31 years old Any advise would be appreciated Thank you Doctor: Hi, Thank you for posting your query. First of all, I wish to reassure you that your symptoms do not suggest any serious neurological problem. Also, they do not seem to be related to general anesthesia. Possible causes could be vitamin B12 deficiency, anxiety, stress or exhaustion. Please get back if you require any additional information. Best wishes, Dr Sudhir Kumar MD (Internal Medicine), DM (Neurology) Senior Consultant Neurologist Apollo Hospitals, Hyderabad, My personal URL on this website: http://bit.ly/Dr-Sudhir-kumar My email: drsudhirkumar@yahoo.com"
},
{
"id": 91476,
"tgt": "How to treat abdominal pain?",
"src": "Patient: Dear sir/madam, Now my sister gets so pain in her stomache and I don t know what to do now. There are Eldoper and acticarbine medicine here but I don t know what are they used for as I know they are related to stomache. Please kindly tell me as soon as possible. Regards! And here is my mail: YYYY@YYYY Doctor: for abdominal pain need to use antispasmodic tab sosbut need to find out cause by ultrasound abdomen or endoscopy or boththe various possibilities are gastritis. duodenitis, gall bladder infectionshernias,uti ,or reproductive organ inflamations or infectionsafter diagnosing cause we may go for further management according to organ involved"
},
{
"id": 118257,
"tgt": "What is the fastest way to increase the hemoglobin level?",
"src": "Patient: I am having liposuction in multiple areas. What is the fastest way to raise my hemoglobin? I was told that when you quit smoking (which I have) that your levels drop. Help!!! I need to be prepared. Whats the most Iron I can take in a day? Is there a such thing as to much? Doctor: that will depend on the cause of low hemoglobin.We should treat the cause.First of all we should have a peripheral blood film study to know if there is deficiency of Iron or folic acid/B12.If iron deficiency is there, a course of anti worm may be needed.If it does not work,you need further investigations for low B.P."
},
{
"id": 82253,
"tgt": "What causes a feeling of constant pressure in the chest?",
"src": "Patient: Hello, I have been experiencing constant pressure in my chest that does not go away. This has been going on for 4 months. I have been for a stress test on my heart and an EKG, complete physical with a variety of tests and nothing is found. My next step is to go for an ultrasound at the end of this month. It is very concerning. The doctor thought maybe stress but I do not think it is this. It is there all the time. I describe it as someone pressing on your chest. Doctor: Thanks for your question on HCM.In my opinion, since your cardiac work up is normal, you should get done PFT (Pulmonary Function Test) to rule out bronchitis. Asmany times bronchitis appears only as tighness of chest and cough develops later.So this can be early bronchitis. Another possibility is GERD (Gastro Esophageal Reflux Disease). In this there is lexity of gastroesophageal sphincter. Due to this the acid of the stomach tends to come up and cause the symptoms. So GERD is another possibility in your case."
},
{
"id": 189679,
"tgt": "Considering having dentures by pulling teeth. Never taken care. Time taken to fix it?",
"src": "Patient: I m considering having all of my teeth pulled out and having dentures at the age of 27 - I ve not taken care of my teeth and i m incredibly embarrassed by them so i guess i am guilty of wanting a quick fix . I don t know anything about this procedure and all i read about continually are people who spend thousands having repair work just to end up having this done anyway. I guess to summarize my query i d like a ball park figure of what it costs and how long it takes. Sorry if this seems abrupt i ve never even asked anybody about this before. Thanks in advance for any reply. Doctor: HELLO, aS you said,you are at the age of 27 now,usually at a younger age of this,dentures are not indicated. You have not mentioned why you need to get dentures?are they mobile? iF your teeth are extremely mobile,we can think of dentures after extracting entire teeth. Or else,deep scaling and root planning is indicated. Mobile teeth can be fixed using splinting with wires. Take complete course of antibiotics. Maintain good oral hygiene."
},
{
"id": 39520,
"tgt": "What causes infestation in different parts of body?",
"src": "Patient: EVERYONE I KNOW THINK I 'M NUTS. I THINK I HAVE SOME SORT OF INFESTATION THAT IS EVERYWHERE. WHEN I SAY EVERYWHERE, I MEAN EYES, NOSE, MOUTH, EARS,SCALP, GUMS, AROUND AND UNDER MY FINGERNAILS AND TOENAILS, GENITALS, AND EVERYWHERE ELSE YOU AND I CAN THINK OF. WHAT COULD THIS BE? Doctor: Hi,Welcome to HCM!Are you having any symptoms? If you have symptoms like fever, weakness, bodyaches etc, that may suggest of some generalised infection.However, in absence of any symptoms and if you just feel that you are infested, you might be needing a psychiatric opinion. There is nothing wrong in it as you might be suffering from some phobia and it can be get rid of with the help of Psychotherapy and medications.Hope this information helps. Feel free to ask if you have any doubt.Wishing you a speedy recovery and good health.With warm regards,Dr. Sridhar"
},
{
"id": 59883,
"tgt": "Suffering with cirrhosis, advised liver transplant. What is the cost and success rate of transplantation?",
"src": "Patient: my brother in-law have suffered from cirrhosis disease and Dr. preferred for liver transplant. i want to know what is the expected cost for transplant and what is the success rate after the transplant liver. the patient is only 32 years old and never drunk alcohols in his life but long time ago have accident with him and blood bottled inserted in his body. Due to that blood gradually damaged his liver. Patient stomach becomes bigger and water accumulated in liver and stomach. please advise what we have to do. we are belongs to from ahmedabad gujarat. Kind regards, Dilip Sathvara Doctor: Hello Dilip:When patients develop advanced or \"end stage\" liver disease, this is called cirrhosis, which is irreversible. The liver is only functioning to a minimal degree. One of the indications that the liver is failing is acumulation of fluid inside the belly, this is called \"ascites\". There are other problems associated with cirrhosis (confusion which is called \"encephalopathy\", esophageal varices which are blood vessels in the esophagus that can burst and bleed out, liver cancer..).Liver transplant is definitely a good option. The survival after a liver transplant is about 85-90% at 5 years after liver transplantation and depends on the experience of the center, the reason why he is getting transplant (hepatitis C patients do slightly worse than the rest), the quality of the care post-transplant. To qualify for a transplant the doctors will calculate a score based on some of his labs, which is called the \"MELD\" score (a high score above >15 is really bad, and a low score is good By the way the cost of a liver transplant varies from country/city. In the USA it is very expensive, probably around 300000$ or more."
},
{
"id": 20469,
"tgt": "What causes the pulse to drop at the start of running?",
"src": "Patient: 48 yo white male, athletic 10 years ago (rowing, running 10k races, cycling) now 100lbs over weight, on 10mg Lisinopril for now 4 years. Starting get back into shape starting stationary cycle in winter, twice week, 9.5 miles in 30 minutes, adjusted to level 4 of 4 most difficult in weight loss setting. Now spring and getting outside again. Have run 5 times so far, each run profile the same, starts with stretch, 1 block walk, then begin run with hill up (vertical about 40 feet), then walk, jog, walk, etc.. This for 1.2 miles ending with down hill. Wearing GPS heart monitor watch with chest band. QUESTION: I begin to run and for first 2 minutes, my pulse drops from 73 to about 40, then sharply climbs over the next two minutes to 143-160 (depending on day) then stays in that range for rest of run, basically reflecting pace of run-walk. Total jog lasts about 19 minutes with total elevation change about 80 feet vertically. ...I have an appointment with my GP in July. Any thoughts? 5/31/2011 Thom Doctor: Hello,There are many possibilities, first the monitor may be giving a false readings and may need to be checked as dropping of heart rate to 40 without any symptoms like dizziness syncope is a far-fetched. Then as an athlete for many years, so far, without any history of medical importance makes you in a good cardiac performance, but for sure you may be asked to do some simple tests like EKG, echocardiogram, or just a precise cardiac examination that's all for now.Hope I have answered your query. Let me know if I can assist you further.Regards,Dr. Mahmmad Gamal"
},
{
"id": 51941,
"tgt": "I cannot urinate. Had appendic problem but operated",
"src": "Patient: had appendic problem already operated but now having cannot urine problem..can i know what is cause Doctor: Hi...It depends on how your surgery went...if it was a difficult one which lies close to the ureter on right side...then this might cause a few problems...but can you please make your question clear?? do u have frequency or burning while passing urine...and your age???"
},
{
"id": 97671,
"tgt": "Any alternate treatment for amyloidosis on forehead that is radiating after laser treatment ?",
"src": "Patient: hi: i was diagnosed with amyloidosis on my forehead....i am undergoing laser treatments for the past one and half year, but it is not responding and it is radiating to all the forehead....please let me know if there is any treatment for amyloidosis in ayurveda...thank you. Doctor: **1. In Ayurveda we consider Amyloidosis due to blockage of channels [sroto avrodh] due to AMA production [Autoimmune reaction] resulting in aggravation of KAPHA Dosha.2. Take Diet/Herbs rich in properties of 'Pachan' 'Deepan' 'Bhedan': like ginger, turmeric, cloves, cinnamon, black pepper, fruits are fine, except citrus fruits, Avoid curd,Banana, Milk, non-veg diet, soft drinks, processed/packaged food3. You can ask for prescription medicines like: arogyavardhini vati, shallaki, turmeric/haridra.PS. Most individuals affected with amyloidosis require treatment for periods ranging from six to nine months, depending upon the severity of the condition. Once patients demonstrate a decrease in abnormal protein deposition, further treatment is given to prevent a recurrence of the condition, and since is an Autoimmune condition thus Rasayan [Immunomodulation[ Chikitsa is required."
},
{
"id": 6513,
"tgt": "Will unprotected sex a day before ovulation lead to pregnancy ?",
"src": "Patient: hello i had unprotect sex with my girlfriend could she get pregant but we had it one day before her ovulation Doctor: Hi ! Yes, there are chances of her being pregnant.I think you already know that having sex in unsafe period around ovulation (9-16)can cause fertilization.You should avoid that period."
},
{
"id": 48130,
"tgt": "Will sex be okay if the ultrasound of the kidney is fine after the removal of stent?",
"src": "Patient: My stent was placed om sept. 6. it was removed the middle of February. I have not had sex during this time because masturbation was extremely painful. I go for an ultrasound of kidney on Wednesday March 26. I assume if all is well sex should not be a problem. Doctor: In my opinion if urine is clear , no symptoms of burning or pain, lab test is clear than there is no harm in sex.One should take plenty of fluids after removal of stents."
},
{
"id": 109578,
"tgt": "What causes severe pain in back between ribs?",
"src": "Patient: Hi Doctor,Of late I've noticed that I'm having severe pain in the back in between my ribs..both on the right side and the left side..it seems like its due to gas...I had similar kind of pain once almost 2yrs ago but recently the frequency has increased...its almost once a month or...Also one more thing whenever I have a heavy meal I have to rush to clear my bowels within half an hour..>So my query is should I need medical attention..is it like the initial stage of something..I would be really glad if you answer me..Many thanks! Doctor: Hello. Thanks for writing to us. The pain could be due to gastritis or related to a muscular issue. Take antacids and belching agents and have a muscle relaxant prescribed by your orthopedician.I hope this information has been both informative and helpful for you. Regards, Dr. Praveen Tayal . For future query, you can directly approach me through my profile URL http://bit.ly/Dr-Praveen-Tayal drtayal72@gmail.com"
},
{
"id": 68864,
"tgt": "What is the treatment for head and neck lymphedema?",
"src": "Patient: Hi, may you pls help me.my aunt nd her husband are ill in different ways so i need your advise nd good medication.My aunt is 65yrs old nd her weigh is 110kg she is swollen from neck up to the head.Her husband is mentally ill nd he likes to run out naked.Would you pls prescribe a medication to stop him run away,he is 81yrs old nd he weigh 112kg.Please help. Doctor: welcome to Health care magic.1.There are multiple reasons which causes the lymph nodal enlargement.2.Before treating the Lymph node, we need to find out the reason for the enlargement.3.Comming to your uncle, he needs psychiatric treatment and that drug will not be given on counter with this online prescription.4.So if you would like to help him, take him to concern department and get treatment.5.Hope it helps you.Anything to ask ? do not hesitate. Thank you."
},
{
"id": 42181,
"tgt": "What is the cause and treatment for infertility?",
"src": "Patient: hi, i am 34 yrs old and married, no children, 4yrs living with my partner...78 kg, 5`7 height, wife 5 ft, 58 kg. my sperm always flactuated and i do varicocelectomy which i think the cause of my low sperm count with pus cells 8-12. this might be the cause why my wife could not be pregnant. she was evaluated from her hormone- normal, hysterogram was patent. thanks Doctor: yes varicocele causes reduce sperm count.you can go for semen analysis and sperm count, and invitrofertilization techniques. it will solve your problem. she will get pregnant easily by these methods. hope my message is helpful and thankful to you. so dont worry go for these evaluations and have a baby soon."
},
{
"id": 51127,
"tgt": "Pain in lower abdomen, burning sensation while urinating. Could it be kidney stones or UTI ?",
"src": "Patient: i treated a UTI about 3weeks ago. but currently, i feel so much pain in my lower abdomen and a burning sensation when passing out urine. could i be having kidney stones or maybe a reoccurance of the UTI? please advise on what to do because the current lab tests show that i have intact red blood cells and pus cell 2+. Thank you Doctor: Hello Dear and thanks fort the query More probability is of Renal stone or infection of the kidney (Nephritis). Recurrent UTI occur in both the cases and you need to be investigated for that. Get your sonography and urine culture done and accordingly treatment can be planned. Mean while take plenty of water, urinary alkalizer for burning maturation antibiotic can be selected after the culture report. But as we have to find out the cause for recurrent infection consult your urologist and get the above said investigations done. Hope this will help you. Best wishes"
},
{
"id": 21506,
"tgt": "What causes loss of appetite and fatigue after Aortic Aneurysm graft repair?",
"src": "Patient: Mother hadAortic Aneurysm graft repair 11-7-11 she has ben unable to eat and very fatigued. Dehydrated and unable to eat and no one seems concerned. 77 years old has lost over 20lbs and cannot hardly get out of bed to go t o restroom alone what should we do ? Doctor: Hi ThereAfter reading the details provided by you i understand your concern for your mothers health. Considering her age and the procedure she has gone through and her present status, to be honest she is not in a good condition. I would like to advise you to take her to hospital as she will be requiring a Holistic treatment approach and well as IV fluids and probably NASOGASTRIC tube feeding as she is not able to eat and also she has lost weight.I wish her good health"
},
{
"id": 216558,
"tgt": "Suggest treatment for pain on shoulders along with bloating",
"src": "Patient: bad pain behind right shoulder blade, gas, bloating, lower right pain around pelvic bone...took 6 gasx, 10mg dicyclomine, 2 tylenol, 40 mg nexium all of which barely work to relieve symptoms...last gallbladder function and ultrasound gallbladder test normal six months ago...suffer from acid reflux.. all of this started within hours of eating some dried fruit Doctor: Thanks for contacting healthcare magic. You have problem of acid reflux so it cause bloting and pain problems. For gastroesophageal reflux disease you have to modify your lifestyl like.....Avoid citrus and spicy foods. Take small amount of meal and chew properly. Avoid alcohol drinking and smoking. Elevate your head during sleeping time. Avoid Immeadiat sleeping after meal. Avoid stress .Endoscope is helpful for further diagnosis. Consent a physician for further treatment. I hope my guidance is helpful to you. Take care. Thanks."
},
{
"id": 201591,
"tgt": "Can masturbation addiction cause any problem in future life?",
"src": "Patient: Well..I am 30 year old man ,who is currently single and have never got married.I used to ejaculate so often though! That started around 8 years ago.At the beginning ,I did that like once a week ,but within the past 3 years ,I used to do it nearly everyday.I tried to give up ,but every time I decided that ,I only gave up for a week or maximum 9 days then back to the same dirty habits.My question is: I heard that people who ejaculate so often will become barren and can not give birth to a baby. If I totally give up although I have done that for nearly 8 years ,is it still possible to give birth to a baby if I marry soon like within 6 months or after a year? I have never checked my semen ,so I do not know whether ejaculation has driven me to be barren or not ,but I recently heard that it is terrible and decided to give it up.And would be there any negative effects if I give it up forever? Somebody said one time of ejaculation equals to making love with a lady for 12 times ,is that true?I am worried about myself.Please reply me asap.You should take into account that I have no girl friend and I do not have sex with ladies,because my tradition prohibits me to do that with a lady who is not my wife. Doctor: Hello dear,First of all, remove all myth and misconception from your mind regarding masturbation.There is no any harmful effects of masturbation as such.However, if it is done regularly then it will result in thinning of semen with low sperm count.Since you are feeling that you are addicted to masturbation and it is interfering with your normal functioning, you can try out the following measures:1. Improve your social life by spending more time with friends and relatives.2. Keep your mind active and busy at all times either through curricular or extra-curricular activities.3. Have a diet rich in fruits, vegetables, fish, nuts & honey.4. Avoid fast foods & lipid containing diet.5. Avoid smoking & alcohol.6. Stay away from pornographic materials.7. Keep away stress, think positive.8. Exercise regularly & practice meditation.Wishing you a healthy lifeTake care."
},
{
"id": 136963,
"tgt": "What causes severe pain in the finger bones?",
"src": "Patient: Hello doctor from so many days I have pain in my finger bones actually I have a bad habit to press my fingers like (patakha nikalna) from my fingers n now by this habit my bones are damaged so please. Give me some medicine for this pain I had so much pain in my bones of fingers Doctor: Hello There is possibility of injury to ligament of metacarpal or phalange joint. there can arthritis of hand joint in future de to such habit.Use ice packs to reduce swelling, inflammation and pain.Physiotherapy with ultrasound and Tens will help you,For early healing you can take Tab celin [vit C] and antioxidant with anti-inflammatory [Tab enzomac plus ].If there is no relief them you may need to do MRI.When such patients come to our hospital I advise them blood test like RA factor, ACCP, ESR, complete blood count, HLA B27 to rule out polyarthritis.Hope this answers your query. If you have additional questions or follow up queries then please do not hesitate in writing to us. I will be happy to answer your queries. Wishing you good health.Take care."
},
{
"id": 28669,
"tgt": "What causes positive Widal test result despite lacking the symptoms?",
"src": "Patient: sir ,i am suffering from fever from last 2 days i have done my blood test and widal test is positive it mean i have typhoid but from yesterday i doesent have fever ,headach or other symptoms of typhoid .....i am confused ...report is clearly seen postive at widal test but i have no symptoms of this... Doctor: Hello and Welcome to \u2018Ask A Doctor\u2019 service. I have reviewed your query and here is my advice. You might have sub clinical typhoid fever. It is nothing but a variant of typhoid fever where the symptoms are minimal. You can take appropriate antibiotics according to protocol to tackle the illness. The complications are less in sub clinical conditions. Hope I have answered your query. Wishing you good health. Thanks."
},
{
"id": 99090,
"tgt": "Suggest treatment for asthma attack",
"src": "Patient: two years ago, I was intubated due to an asthma attack. About a year ago I began to hear a popping sound coming from my chest after I stretch. Especially after waking in the morning. The popping provides relief. I noticed that this happens after sleeping in a fetal position with my chest sort of caved in. What is going on? Doctor: Thank you for writing in HCMI thing first you need to confirm the diagnosis of asthma by clinical symptoms.Supportive investigation like spirometry ; serum IgE may be done.Serious Cardiac problem like ischaemic heart disease and heart failure also to be rulled out if clinically suspected after cardiovascular examination.If history and examination suggest asthma..then you have to start some reliever medicine like levosalbutamol for transient symptomatic relief along with controller medicine like long acting bronchodilator and steroid combination (metered dose inhaler with spacer).Better to meet with local pulmonologist...for dose and demonstration of technique.Thank you."
},
{
"id": 176081,
"tgt": "Suggest treatment for high fever in a child",
"src": "Patient: Dear Sir, My daughter 10 years, 48 kgs is suffering from fever since saturday afternoon the grade of fever is rising with every passing day by almost 0.5-1 deg f paracetamol(650) is not effective, doing sponging, now we have administered combiflam, fever is 104.7def f, please advise. Doctor: Hi,As she is having very high fever requires evaluation of her case.She requires complete blood checking like TC, DC, Platelet count to find out of having high fever.Go for routine and microscopic urine check up.After report your doctor will go for treatment accordingly.Continue with paracetamol or combiflam clock wise.Important is to give plenty of water and liquids and keep her well hydrated so that there will be profuse perspiration to bring down the temperature.Give her complete rest and light diet.Ok and take care."
},
{
"id": 153881,
"tgt": "Suggest treatment for stomach cancer",
"src": "Patient: my mom had undergone stone surgery when her gallbladder was removed last year.from tha past few months,she is experiencing continuos vomiting.afterlong treatment the doctor suggested she is suffering from stomach cancer.she is in advance stage.pllease suggest what to do ? Doctor: Hi, dearI have gone through your question. I can understand your concern.Your mother has stomach cancer. Treatment depends on type and stage of stomach cancer. In early stage surgery should be done to remove tumour. In later stage chemotherapy and radiotherapy is the treatment options. Consult your doctor and take treatment according to stage of her cancer.Hope I have answered your question, if you have any doubts then contact me at bit.ly/Drsanghvihardik, I will be happy to answer you.Thanks for using health care magic.Wish you a very good health."
},
{
"id": 127198,
"tgt": "Is pain below the shoulder blade after an injury a symptom of rib cage fracture?",
"src": "Patient: I fell off of a bicycle sunday and landed on my left side. it is very painful if using it. in any way. Seems that a muscle or something on the back below the shoulder blade is where the most pain is from ?? Is there anything there or could I have cracked a rib ?? VERY paiful yo caugh or breath deep ?? Doctor: Hello and Welcome to \u2018Ask A Doctor\u2019 service. I have reviewed your query and here is my advice. It can be a rib fracture, most probably some minor contusion. Get an X-ray chest to look for any rib fracture. As of now, take analgesics like Paracetamol for pain relief. Wishing you good health. Thanks."
},
{
"id": 177660,
"tgt": "What causes cold sweat and fluctuating body temperature in children?",
"src": "Patient: My 17 month old is having cold sweats, his hands are feet are cold and clammy and he has soaked through his clothes in the last 48 hours with sweat. He doesn t have a consistent fever and his temp keeps changing. What might be the cause of this and should I be taking him to the hospital or is this something I can monitor and see my GP if it continues? Doctor: Hi...being cold and clammy could be a sign of shock and you say there is on and off fever too. I suggest you see a doctor immediately. Meanwhile for monitoring him, you can assess his urine output and intake. if both are poor, then you need to see a physician immediately.Regards - Dr. Sumanth"
},
{
"id": 151196,
"tgt": "Cervical stenosis, herniation @ C1-C7 pinched nerve. Numbness in fingers, increasing pain, taking cymbalta. Alternate medication ?",
"src": "Patient: I am a 49 y/o male with cervical stenosis , hernations @ C1-C7 pinched nerve Right side , numbness in thimb , pointer middle finger , @ night pain and numbness is woresnsed, I was given cymbalta, 20 mg , by Neuorlogist, Just stated to taking it, is it a good medicine, I ahve been on clebrex, mobic, soma, flexoril , naproxen NO Help I Sustained head trauma on Septmeber 11, 2001 at the terrorist attacks. The pain and numbness are getting progressively worse. Doctor: Hello. I'm Dr. Christensen. I'm sorry you're so uncomfortable. As you've discovered, cervical stenosis can be resistant to treatment, especially when multiple levels are affected (as in your case). Cymbalta is one of several medications that are used for conditions like yours. Pregabalin (Lyrica) and gabapentin (Neurontin) are other examples of medications used to treat pain caused by damaged nerves. It usually takes several weeks to determine whether or not these drugs will be helpful for a given patient. Unfortunately, most patients have at least some residual discomfort even when the medications work well. Therefore, other modalities -- physical therapy, acupuncture, core strengthening, chiropractic manipulation, etc. -- should also be explored. Some patients do eventually need surgery when all other options fail to reduce their pain. Cymbalta is generally well tolerated, but it does have some side effects. The most commonly reported ones are nausea, fatigue, dizziness, constipation and insomnia. Young adults using this medication may experience suicidal thoughts. Some patients have suffered from liver damage while taking Cymbalta, and this drug could cause serotonin syndrome (an uncommon side effect characterized by agitation, tremors, rapid heart rate, sweating, flushing, diarrhea and sometimes seizures). Finally, Cymbalta should not be discontinued abruptly, as this could lead to a withdrawal syndrome (insomnia, electric shock-like sensations, agitation, confusion and headache). If you and your doctor decide Cymbalta is no longer indicated, you should taper your dosage gradually. I hope that answers your question, and I hope you feel better soon."
},
{
"id": 194317,
"tgt": "What causes discharge of clear fluid from erect penis?",
"src": "Patient: I am 53 yrs old, 5-10, 193lbs. I am in good shape and eat well. I have just recently stopped smoking. Like 2 days ago. I had blood test and urine sample taken. It showed me positive for herpes Simpex I IgG but Negative on Herpes Simplex I IgM. Same result with Herpes Simplex II IgM - pos / Herpes Simplex II Igm - Neg. My concern is that only when I have been having an errection which is many times in the day. I have a clear fluid or precum dischage. I am working overseas and have not been with a woman for 6 months so when I think of my new never been with girlfriend or speak with her on the phone I get an errection and I have a lot of liquid discharge. I have not been with this woman and I am very stressed and concerned. Is it because of my age. No deceases detect except for the herpes which I was told that I had come in contact with someone and I had it it at some point of time. HIV 1&2 Negative. I have no problem with getting an errection or holding it for a good amount of time. Help please dont want to lose the girlfriend I have yet to meet. Doctor: Hello, Precum, Pre Ejaculatory Fluid PED. It\u2019s a normal secretion which comes before you ejaculate so sperm can travel smoothly and get out of the penis. If no or reduce PEF chances fewer chances of pregnant a female. You can control the episodes of PED by controlling your erotic feeling. Methods to control Erotic feelings are 1.\u00a0\u00a0\u00a0\u00a0\u00a0Sex education 2.\u00a0\u00a0\u00a0\u00a0\u00a0Physical exercises 3.\u00a0\u00a0\u00a0\u00a0\u00a0Sleep adjustment 4.\u00a0\u00a0\u00a0\u00a0\u00a0Quality of sleep 5.\u00a0\u00a0\u00a0\u00a0\u00a0Healthy and balanced diet 6.\u00a0\u00a0\u00a0\u00a0\u00a0Programming for leisure time 7.\u00a0\u00a0\u00a0\u00a0\u00a0Development of spirituality 8.\u00a0\u00a0\u00a0\u00a0\u00a0Taking a cold bath. Hope I have answered your query. Let me know if I can assist you further. Take care Regards, Dr S.R.Raveendran, Sexologist"
},
{
"id": 148905,
"tgt": "Sudden shortness of breath followed by spasms. On steroids post disk replacement surgery. Hypertensive",
"src": "Patient: I have been experiencing sudden shortness of breath that ends briefly with a body spasm that isn't controlled it's happened twice this month. After the spasm breathing is easier. The episodes last about a half an hour. I do have high blood pressure but do take medication for it. I have had several panic attacks but it has been over 15 years and was not like this at all. ? I have had disk replacement surgery in my neck a year ago but have been going through a series of steroid injections but my spasms are generally in the mid back and arms. Sorry not paying didn't understand this. Please disregard my inquiry and my email Doctor: HIThank for asking to HCMI can understand your problems, I think it is not the dangerous one and will come around for that you need to keep your stress level low because such symptoms are related with the mental condition like anxiety depression and other, for safety purpose I would advise you to just go for EKG test, this will rule out the possibility of any cardiac elements every thing will be fine just take it easy have great day."
},
{
"id": 118959,
"tgt": "Increased BP and worsened edema after taking off medication due to increase in lasix dosage. History of enlarged heart due to BP. Guide",
"src": "Patient: My doc just took my off my blood pressure meds because he increased my lasix dosage - In the last 2 days my blood pressure has increased - it is currently 161/101 -AND my edema has gotten much worse. I cannot get my doc to call me back but I am feeling off - should I take a dose of my losartan? I already have an enlargement in my heart from chronic bp issues Doctor: Hello,Your doctor might have taken off losartan for a reason.May be that your Potassium level are high.But the blood pressure is pretty high and you need combination of Antihypertensive drugs with Lasix now to control the Blood pressure.Edema may be due to Cardiac and renal abnormality.Consult your doctor before adding other antihypertensive."
},
{
"id": 128752,
"tgt": "What causes dull ache on left side of the face?",
"src": "Patient: I have a dull ache and strange feeling on the left side of my face starting from the top of my cheek and going to on the left side of my head had a sinus infection for three months had three lots of antibiotics no pressure now just this dull annoying ache Doctor: Dear patientyour symptoms indicate diagnosis of sinusitis most possibly involving maxillary sinus. plese consult otolaryngology specialist nearby you. meanwhile you should start antibiotic course of tablet cefixime 200 mg twice a day. thanks."
},
{
"id": 173274,
"tgt": "Suggest remedy for passing urine in bed by 8 year old",
"src": "Patient: My daughter is almost 8yo and still wets the bed every single night. The smell is really, really bad and she sleeps through the entire night and never wakes. We are a very supportive family and I was happy to allow her grow out of it and keep washing every day and we NEVER make a big deal about it. It doesn t bother her at all. However, do you think I should take her the doctor? Doctor: Hi...Thank you for consulting in Health Care magic. I have few questions for you -1. Has your son stopped bed wetting at certain age and again started having it2. If so at what age he stopped and when did it startIf he had never stopped bed wetting form birth...you can try some behavioural changes which I can elaborate after proper history and interaction with you , but the effect will not be immediate, but it will be permanent. If its a recurrence after attaining night time control, I suggest you see your paediatrician to rule out pathological causes like urinary infections or Voiding dysfunction.Hope my answer was helpful for you. I am happy to help any time. Further clarifications and consultations on Health care magic are welcome. If you do not have any clarifications, you can close the discussion and rate the answer. Wish your kid good health.Dr. Sumanth MBBS., DCH., DNB (Paed).,"
},
{
"id": 41193,
"tgt": "Suggest treatment for infertility",
"src": "Patient: Respected sir, I have completed 5 years of marriage life. We did not try to get pregnant these years. Now we want to and trying for pregnancy for last six months. My wife cycle is accurate as 28 days although not succeeded. We have done all tests like sheman test as well as her tests too. All results are positive. Please help us out. Thank you. Doctor: Hello, you need ovulation induction and then follicular study to see the formation of egg in her body and also will guide for best days for intercouirseIn case you have any questions in future you can contact me directly on http://bit.ly/drmanishajain"
},
{
"id": 175204,
"tgt": "Suggest treatment for cough and cold in a child",
"src": "Patient: my 9 yr old shih Tzu has been coughing and honking sound.. not all the time seems when she goes out in the cold was the onset .. I ve been putting we we pad out for her and won t let her out.. I don t want to take her to the vet unless I really have to.. is there something I could give from home? Doctor: As you have mentioned it is clear your daughter is suffering from viral infection nothing else. For this you have to give her cap amoxycillin 250 mg three times daily or tab cefadroxyl 250 mg twice daily after meals with tab no-cold or tab diominic-dca twice daily after meals for common cold with syrup asthakind or syrup grilinctus-bm 1 tsf three times daily for cough. You also have to take your daughter for salbutamol or duolin nebulisation at your family doctor's clinic as it is very helpful for the baby."
},
{
"id": 182613,
"tgt": "Suggest treatment for tooth ache with loss of appetite",
"src": "Patient: My daughter is suffering from a bad toothache that started yesterday. It is now affecting her ear. She can t eat or hardly drink. Her dentist appointment isn t until Wednesday and I don t want her to go with pain for that long. what should I do? Doctor: Thanks for your query, I have gone through your query.The tooth and the ear pain can be because of the tooth infection and You need to get it treated.Since you have not mentioned the age of your daughter, considering her age less than 14years, If i am her treating doctor, I would have prescribed a course of antibiotics like amoxicillin 250mg TID (if not allergic) and analgesic like paracetamol syrup TID for 3 days.After taking the course of these drugs you can get the tooth treated endodontically.I hope my answer will help you, take care."
},
{
"id": 219240,
"tgt": "What does a = symbol in a scan report mean?",
"src": "Patient: hi i am nisha and now i am pregnant now and i have no vomiting ,dring first 3 month i feel vomiting sensation and i vomitted yellow colourlly ,and in my 5 month scan report shows me siemens id --straigth to siemens id i found that an = symbol so u r answer is? Doctor: HI.I really did not get your question, but for the nausea you can try doxylamine (e.g. doxinate), put 4 tablets per day during pregnancy for the morning sickness you are experiencing.Best wishes."
},
{
"id": 192408,
"tgt": "What is the treatment for premature ejaculation?",
"src": "Patient: I am 33 and married, my wife is away and i started masterbating a lot while watching porn, and now as soon as i get aroused i leak pre=cum, even before an erection, i also always had a problem with pre mature ejaculation, if i stop materbating for a week or so, will this problem of pre-cum go away? Doctor: Hello, Pre cum is a normal phenomenon and you should not try to stop it. By the way, you should try reduce the masturbation frequency to thrice a week. Therefore I suggest consulting a psychologist for counseling. Hope I have answered your query. Let me know if I can assist you further. Take care Regards, Dr. K. V. Anand"
},
{
"id": 113963,
"tgt": "Suffering from sever back pain. Will this effect in pregnancy ?",
"src": "Patient: I am 25 year old married female, 60 Kg, 154 cm, I am suffering from lower back pain past one year. I am a software engineer who has to sit for 9-10 hrs daily. My pain is only while sitting and have no trouble doing anything else, like walking or sleeping or, climbing stairs, sneezing, coughing, sex or anything like dat, even for hrs . I cannot avoid sitting and working as dats my job While I take leave frm office , I have no pain at all. As advised by my doctor I tok X-ray and he said theres nothing wrong and I have to exercise and did not give me any medicines. I do 15 mins of streching exercise but pain hasnt decreased much. I am very much worried now as I am trying to conceive now and how will it be affected by my back pain, Will I have problems when I am pregnant? and during dleivery due to my back pain? Doctor: Hi Muna Mohammed, . You will have problem once you conceive as the back pain may aggravate. It is better get yourself examined by a Orthopedist to rule out Lumbar Spondylosis, if X ray is not showing anything get a CT scan done. Take breaks in between your work and get adequate rest. You may need medications. Apply heat pads. Wish you Good Health."
},
{
"id": 26718,
"tgt": "How diet change controls cholesterol numbers?",
"src": "Patient: My husband had been asked to go on cholesterol meds his results are HDL 0.9. LDL IS 3.0. NON HDL is 3.9. Chol/ HDL ratio is 5.3. The optometrist at his last test advised that he could see cholesterol in his eyes. My question is : is it really necessary to go on the medication. He has also got a blood glucose level of 6.7. That was a fasting test and it was the second time at the same reading. His dr has also prescribed meds for this as well. My husband was diagnosed with CCL in 2001. Not sure if this has anything to do with him prescribing the above medicine. What do you think? Could we get the levels down with a change in diet? Regards Marg Doctor: Hello!Thank you for asking on HCM!I read your question carefully and understand your concern. I would like to explain that Chol/HDL ratio is a more confident indicator compared to total cholesterol or LDL. A ratio above 4.5 in men, indicates a high cardio vascular risk. That is why I agree with your doctor in starting therapy with statines to reduce this independent risk factor for cardio vascular disease. Regarding his glucose levels, I would advise to perform further tests like : - glucose tolerance test- HbA1c to examine better his glucose metabolism and rule in/out possible diabetes.In case of an initial diabetes, diet modifications would be more appropriate than medical therapy, to reduce high blood glucose levels. Some other tests are needed to exclude other metabolic disorders: - a resting ECG-thyroid function tests-cortisol plasma levels-blood electrolytes-kidney and liver function tests. I don't think this medications are related to his past medical history. Hope to have been of help!Best wishes, Dr. Iliri"
},
{
"id": 24855,
"tgt": "Suggest treatment for enlarged heart with history of pericarditis",
"src": "Patient: My father suffered what we thought was a heart attack two weeks ago. We later found out that he had paracarditis and was hospitalized for 5 days. They released him under the advice that he see the cardiologist within a week. He has gotten worse, and went to urgent care. They took xrays of chest and said his heart was also enlarged. He is having hallucinations also. His cardio appt was cancelled today because of high winds and power outages. What should we do? Doctor: Hello there I understand your concernPerhaps your father has accumulated fluid in the protective lining of his heart giving rise to the enlarged shadow on the skiagram. He needs to be put under observation, a bedside echo to be done which will reveal the amount of fluid and accordingly the steps will be taken. His hallucinations maybe on account of his heart being under stress and creating difficulty to pump thereby creating problem with oxygenation.Hope I have been able to answer your query..Speedy recovery to your dad"
},
{
"id": 95552,
"tgt": "Lower abdominal pain and nausea. Is it Peptic ulcer or Appendicitis?",
"src": "Patient: I've been having lower abdominal pain and nausea for about 4 days now. It's gotten to the point where I'm smelling food and feeling nauseated afterwards and any time I eat something or take an antacid to try and feel better it actually feels worse. I am concerned that, since this is not going away, it could be a peptic ulcer or appendicitis. How long should I wait before going to see a doctor with symptoms like this? Doctor: no waiting in the case of appendicitis because the delay you make it may cause perforation and death,so see your doctor immideately"
},
{
"id": 137920,
"tgt": "What causes neck pain , muscle twitching on the face and dizziness?",
"src": "Patient: hello i am a 39 year old male EMT, i am having some concerning symptoms that might lead me think i could have ALS. i am having uncontrollable muscle twitching in my face on the right side under the eye, uncontrollable muscle movement and shaking of the right and left hands that at points require me to use my other hand to help control the affected hand to perform simple tasks like using a mouse. i have difficulty in swallowing and a constant feeling of choking or a blocked airway when i finish eating to the point that it often triggers my gag reflex and i vomit. severe back and neck pain most days visual disturbances, and painful eyesight, 1 syncope episode, dizziness that is chronic. and a generalized feeling of exhaustion and lack of sleep even when i sleep for 8 hours or more. i am not having any issues with the low extrimites except for pain which is present in both due to preexisting injuries. Doctor: Hi,Thanks for your query.*** It seems that you have muscle twitches or fasciculations. These are small, local, involuntary muscle contraction and relaxation visible under the skin arising from the spontaneous discharge of a bundle of skeletal muscle fibers. You might consult your doctor who can order 1. a few blood tests including thyroid function tests, 2. Serum electrolytes including Serum calcium and potassium levels. 3. Serum B12 levels In case the above tests are normal further testing can be done including - nerveconduction studies, Electromyography, etc. These tests might help to diagnose any underlying neurological issues. I do hope that you have found something helpful and I will be glad to answer any further query.Take care"
},
{
"id": 183426,
"tgt": "Suggest remedy for pain and swelling in gums",
"src": "Patient: For the last two days, whenever i sleep on my Left side, i wake up with severe pain in my gums right behind my last left molar nd around it. The appearance is swollen and this morning, the top part of the \"gum\" area behind my molar looked bruised. After i woke up, i tried to move my mouth in efery direction thinking maybe i did something in my sleep to two nights in a row but i couldnt reproduce the pain. That area is still sore. After i would wake up in pain and go to sleep on the other side, the pain on the left would slowly ease down. What do you think is going on? Doctor: Thanks for using Health Care Magic.Read your query.Erupting wisdom tooth or impacted wisdom tooth( if there is no place for it to come out) usually present with the pain and swelling along with difficulty of opening the mouth and many other symptoms.For the pain ibuprofen ( if you are not allergic to any medicine can be taken).I would advice you to visit your oral and maxillofacial surgeon and have a radiograph done.If impacted you can have it extracted and if erupting with the normal position,wait till it is completely erupted.Antibiotics like amoxicillin on prescription may be needed.Do salt water gargling.Hope this was useful.Thanks and regards."
},
{
"id": 11782,
"tgt": "Face and body skin color don't match, had done advance chemical peeling once. What can I do ?",
"src": "Patient: I am 22 year old boy,my body skin colour is very fair but my face skin colour is not matching my body skin colour, for these i had done advance chemical peeling but it is effictive a little but still its not matching my body skin colour, now wat to do can i get back my normal skin colour on my face that matches my body skin colour i am very worried abt it Doctor: Hi..thanks for your query. You can follow the steps below: 1)Use a sunscreen with SPF more than50 2)Take oral vitamin c supplements and antioxidants 3)Apply a mild Glycolic acid based or Kojic acid based skin lightening cream every day. 4) Continue chemical peels every 15-30 days.These are latest and safest methods of skin lightening when done under the supervision of trained dermatologists.They frequently produce optimum results after 4-6 sittings. hope this helps. take care!"
},
{
"id": 147165,
"tgt": "What does this MRI report of spine indicate?",
"src": "Patient: Hi I have a small annular tear at l3-l4 disc, small disc protrusion with annular tear L4-L5 no neaural compression, broad-based disc protrusion at l5-s1 making contact with the traversing right s1nerve root no nearal compression what does this mean in english please other than alot of pain constantly????????? i am female and 36yrs old Doctor: Hi,Thank you for posting your query. I have noted your clinical and MRI details.Based on MRI spine report, your nerves in lower back (lumbar) region have got pinched due to slipped disc. Disc is the soft tissue located between two bones (spine) in the lower back.Symptoms of disc prolapse include back pain, leg pain and tingling of legs.Initial treatment would consist of pregabalin or gabapentin capsules.Physiotherapy and back extension exercises would also help you.Only if the above measure fail, would you require epidural injections or surgery.I hope my answer helps. Please get back if you have any follow up queries or if you require any additional information.Wishing you good health,Dr Sudhir Kumar MD (Internal Medicine), DM (Neurology)Senior Consultant NeurologistApollo Hospitals, Hyderabad, IndiaClick on this link to ask me a DIRECT QUERY: http://bit.ly/Dr-Sudhir-kumarMy BLOG: http://bestneurodoctor.blogspot.in"
},
{
"id": 222665,
"tgt": "Is TSH level 6.93uIU/ml during pregnancy harmful?",
"src": "Patient: hi, I'm 13 weeks pregnant now, TSH at 5 weeks was 8.23 uIU/ml (N.R 0.27-4.2) then 11.62 uIU/ml at 9 weeks and 6.93 uIU/ml at 11 weeks. I'm taking Eltroxin 250 mc right now (before prg. I was taking 150 mc, after prg increased to 200 mc then to 250mc) . Another test will be done tomorrow. Are these levels will affect the foetus although increasing the dosage? Doctor: Hi dear, I have gone through your question and understand your concerns.Increasing TSH levels are unlikely to affect the fetus if the dose of Eltroxin was increased timely.I will suggest you to remain in close follow up with your OBGYN specialist to monitor your TSH and adjustment of the dose.Hope you found the answer helpful .Wishing you good health and.Dr Deepti Verma"
},
{
"id": 56720,
"tgt": "What does my liver ultrasound result indicate?",
"src": "Patient: im 45 years old and my liver ultrasound result is normal size liver with non-specific diffuse parenchymal disease. All other bloodtest results are within normal range, and hep b reactive. What does the ultrasound result means and what can i do to protect my liver from further damage? Doctor: HelloYour ultrasound findings suggest liver parenchymal disease.Most likely it is due to hepatitis (hep b reactive)in your case.In majority (up to 90 %)of the cases,it resolves spontaneously without any complications.However,it may be chronic and carriers in some cases.You need regular checkup.Antiviral medicines can be prescribed after assessment.In most of the cases,patient only need conservative treatment.You may need assessment of clotting factors.Get well soon.Take CareDr.Indu Bhushan"
},
{
"id": 70604,
"tgt": "What are the symptoms of pneumothorax?",
"src": "Patient: I was getting a nerve block in my left rib cage about 2 1/2 months ago at the doctor punctured my long and caused a up I was in a lot of pain when that happened but and spent three days in the hospital then I came home and I felt a lot better but slowly that left rib cage is starting to hurt again and I am worried that it\u2019s I I\u2019ve had x-rays and the x-rays don\u2019t show anything but I\u2019m afraid that pneumothorax is cause some kind of infection or something is that possible Doctor: Hi, Pneumothorax is a condition in which the space between the pleura in the lungs fills up with air. The pleura is a lining of the lungs that is just below the rib cage and just above the lungs. There is a space between 2 pleura. When there is a puncture in the pleura and it is communicating with the outside, there can be air entering into the pleural cavity. This is known as pneumothorax or air in the thorax. Some symptoms of this include pain in the chest, pain while breathing, difficulty in breathing, coughing, and pain while coughing. Most pneumothorax is very easily detected in an X-ray. There are, however, other diseases which occur with a needle prick that enters too deep and these can cause chest pain and breathing difficulty as well. Getting another opinion may be a good idea in case the pain persists. Rarely, an abdominal or cardiac cause may cause chest pain and getting those systems checked may help. Hope I have answered your query. Let me know if I can assist you further. Take care Regards, Dr Vignan Rachabattuni, General & Family Physician"
},
{
"id": 191022,
"tgt": "Treatment of tooth pain",
"src": "Patient: I have tooth pain on and off for two years. It has started painting more and more since last 2 days it is paining too much. Is there any medicine I can take for that? Last few time one friend suggested combiflam and amoxy antibiotic and it was better but now it s not at all working. Doctor: Hi, First of all medications should not be taken on own. It can lead to antibiotic resistance. You have an infected tooth which is causing the pain. The tooth could be decayed which has resulted in the pain. The tooth should be root canal treated and capped or removed to solve your problem permanently.Consult a dentist at the earliest."
},
{
"id": 179094,
"tgt": "What causes rashes on face of a child while getting canines?",
"src": "Patient: Hi My 1.8 years old daughter has been prescribed levolin 3ml thrice and clamp forte 3 ml twice daily for five days. She reported fever and cough with running nose. After two days of medication I m noticing driness in cough n increase in running nose. A lot of rashes on her face . Shall I consult the doc again or wait for course completion. she is getting her canines (teeth) all four simultaneously. I wonder R the rashes related to it or medication. thanks Doctor: Hello, thank you for posting your question.Your child seems to be suffering from a respiratory tract infection. The most common cause for such infections, especially when associated with running nose is viral. In such cases, appearance of a rash as a part of the disease process may be expected.Apart from this, the antibiotic prescribed to you (clamp forte) is also known to cause rashes and itching. Thus here the rashes could either be a part of the viral infection or due to the antibiotic prescribed to your child. In either case the next best step would be to strictly monitor the progression of these rashes. If you feel that they are spreading to other areas of the body, developing blisters/pus or crusts, involving the oral or genital mucosa etc. immediate medical consultation would be needed. If the drug is found to be the offending agent, immediate stoppage of drug along with appropriate treatment would be needed.If you have any more questions regarding the case, do write back."
},
{
"id": 22968,
"tgt": "What causes vertigo in a hypertensive person?",
"src": "Patient: I had major head trauma 20 months ago and suffered vertigo; then was cured of that by a procedure done by a physical therapist. For some reason (possibly that I changed my blood pressure medication) it came back after all that time and for a week now I have been experiencing the vertigo and dizziness every day. Note: I also lost my hearing in one ear from the fall and was told by my doctor that the vertigo may have come from the seeds slipping in the eardrum to one side????? Doctor: most likely reason for vertigo is ear damage. it can be treated with vestibular sedative drugs. I usually treat my patients with Tab vertin 8 mg three times a day. so get it prescribed from your doctor.also keep track your blood pressure. consume low salt diet"
},
{
"id": 187250,
"tgt": "What is the cause and treatment for jaw pain?",
"src": "Patient: I've been experiencing on and off again jaw pain on the left side back base of jaw, it seems to come and go. I do get press in my upper cheekbone sort of like sinus pressure, and right now it could be. I'm 28 years old male and within the last year I have had a cardiovascular exam with stress test/ekg/echo so I figure most likely scenario is that it's TMJ, just attempting to reassure myself. Doctor: Hello, thank you for consulting with healthcaremagic. Jaw pain can be because of many reasons like-first may be one of your lower tooth is infected of left side which is causing this-second may he your salivary gland of left is having stone which is causing this problem-third may be your your tmj only is causing this-fourth can be your jaw bone is only infected.But to confirm all these you have to visit a dentist and get a full mouth x - ray done which will rule out the problem. Hope it will help you."
},
{
"id": 71570,
"tgt": "What is the remedy for tightness across shoulder blades causing chest pain?",
"src": "Patient: I get gout in my left big toe and ankle but only mild attacks which diclophenic keeps under control .i also suffer with tightness across the shoulder blades causing pain to the far left and right of my chest this comes one day and gone the next could this also be gout Doctor: Hello,Yes, as you explain the history, it might be gout. You should do an x-ray of the shoulder to evaluate it.Hope I have answered your query. Let me know if I can assist you further.Regards, Dr. Jnikolla"
},
{
"id": 22649,
"tgt": "What is the life expectancy of person with moderate mitral valve regurgitation?",
"src": "Patient: I have been diagnosed with Moderate Mitral valve regurgitation two days ago. I am a 30 year old mother of one toddler. My symptoms include, palpitaions, fluttering in my chest, some feeling of heaviness or pressure in my chest, excessive sweating, mild shrtnes of breath and fatigue with little exertion. I would like to know what my life expectancy may be now that I am taking the diuretic HCTZ 12.5mg daily and trying to loose the 35 pounds I'm overweight by my BMI. I have been advised to have yearly Echos done and I just wanted to know if something can go terribly wrong just suddenly. Am I a ticking time bomb really/ Doctor: Hello and welcome to \u2018Ask A Doctor\u2019 service. I have reviewed your query and here is my advice. There is no need to worry much. Just get yearly echos done and if heart function goes down and regurgitation increases then valve replacement can be done. You can have a good life expectancy. Surgery is advised at one point, and it should be done to halt the decrease in heart function. However consume a low salt and fluid restricted diet. You should visit a hospital immediately if you develop shortness of breath at any point of time.Hope I have answered your query. Let me know if I can assist you further.Regards, Dr. Sagar Makode"
},
{
"id": 196393,
"tgt": "Suggest medicine to improve sexual drive",
"src": "Patient: sir good morning at the time of sexual relation what can i do to improve increase the time of sexual time.i can not getting that much of time at the time of sexual time.is any medicine in aurvedic and alopathi to solve this problem?pls suggest some medicine for to solve this problem BY prince Dubai Doctor: Dear userWe understand your concernsI went through your details.SEXUAL DESIRE AND SEXUAL STAMINA cannot be built on one day. Moderate exercise can help in increasing sex energy to a great extent. Simple push-ups or 30 minutes of regularly walking or jogging can work wonders in building sex power. Oysters,, almonds, cashew nuts, sesame seeds, walnuts, Fruits like apples, apricots, blueberries, cherries, papayas etc. dates, chocolate, pumpkin seeds, onions etc can also help you to improve your sexual power. A healthy lifestyle that keeps a person active with proper food and adequate exercise help in improving one\u2019s sex power. Diet plays a big role in one\u2019s sex life. Healthier the person better is the sex life. Sex life reflects the lifestyle one leads. If you require more of my help in this aspect, please use this URL. XXXX. Make sure that you include every minute details possible. Hope this answers your query. Available for further clarifications.Good luck."
},
{
"id": 127631,
"tgt": "What causes pulsing sound in the ears while treating fibromyalgia?",
"src": "Patient: I have suffered with Fibromyalgia for the last 21 years. For the last year or so I have noticed a wooshing and slight throbbing sound in my ears when I stand after sitting for a half hour or so.My blood pressure is under control so am curious what else this could be. Doctor: Hello and Welcome to \u2018Ask A Doctor\u2019 service. I have reviewed your query and here is my advice. The sound in the ear can be related to fibromyalgia.So treatment consist in maintaining under control fibromyalgia. There are several treatments that can relieve the fibromyalgia symptoms. I suggest you to consult your doctor for further evaluation. Hope I have answered your query. Let me know if I can assist you further."
},
{
"id": 100371,
"tgt": "Suggest medication for frequent sneezing and difficulty in breathing",
"src": "Patient: hello i have been having this weird experience lately. It happens randomly like every 2 months twice on once. I wake up sometimes and i sneeze alot and after that i cant seem to be able to breath properly and am a bit scared. is it my heart? i have been smoking for 2 years now and not that much. pls help me Doctor: Hello,Thank you for asking at HCM.I went through your history and would like to make suggestions for you as follows:1. Were I treating you, I would prescribe you daily montelukast and levocetirizine for at least 2 weeks.2. I would also suggest you to use inhaler salbutamol or levosalbutamol for difficulty in breathing symptoms on as-and-when-needed basis.3. From your symptoms, I would not think of heart problem first. I would first think of nose lung problem which may be due to allergies. Of course, smoking can worsen any lung problem.4. Personally I would suggest you allergy testing for inhalant allergens like house dust mites, common molds & pollens in your area. If you are allergic to any particular substance, avoiding them is the best option.5. Personally I would suggest you to seek professional advise for quitting smoking as it can worsen any respiratory problem. I would also suggest you to avoid exposure to dusts, smokes and air pollution in general.Hope above suggestions will be helpful to you.Should you have any further query, please feel free to ask at HCM.Wish you the best of the health ahead.Thank you & Regards."
},
{
"id": 26366,
"tgt": "What causes dizziness and feeling shaky suddenly when i am in toilet?",
"src": "Patient: I woke up this morning and went to the bathroom, when sitting on the toilet I broke out into a random swear and began to go very dizzy and shakey. I was sweating to the point where it was dripping down my chest. This episode lasted about fifteen minutes. What could this be? Doctor: Hi,You have blood pressure drop, which caused such symptoms. It can happen sometimes in the morning after bath or toilet. You shouldn't worry, there is nothing serious, just drink much liquids.Wishing you good healthIn case of further questions don't hesitate to askRegards,"
},
{
"id": 161794,
"tgt": "Suggest treatment for child suffering from epilepsy",
"src": "Patient: i have a child of age 1 year and 3 months and recently we have come to know that he is suffering from epilepsy. may i know about the cure and the duration of the treatment that will be needed? i even want to get my doubt clarified that it will cured permanently or not. Doctor: Hello, First of all, we should know the family history because the epileptic syndrome is hereditary. Second, we should know about the prenatal period of your child because some infection during the prenatal period can cause brain damage and epilepsy. The natal period and the postnatal period are crucial because any small problem can affect the child\"brain. The developmental milestone can help us detect the brain disorders, your child should walk without fall and he should build a tower of 2-3 cubes and he should speak 10 words and understand more. If there was any limitation, your child could have an underlying brain disorder. An MRI can help us detect the epileptic focus in addition to an EEG. I there was not a brain lesion; the prognosis would be good with anti-epileptic drugs. The duration of the treatment is between 2-3 years, then we withdraw the drugs after 2 years of normal EEG. Hope I have answered your query. Let me know if I can assist you further. Take care Regards, Dr Dr/Mustafa, Pediatric Neurologist"
},
{
"id": 218943,
"tgt": "How can the pregnancy date be calculated?",
"src": "Patient: hi, i am pregnant at the 9th month and the baby my arrive in a fortnight. but the dating is not exact because of ultrasonic test is not reliable, my doctor says, which could be because of my pregnancy diabetes, what should do. is there a way i can show my test result to see if there is other interpretations about my pregnancy date? Doctor: Hi Thanks for the query. I understand your concern - Mostly expected date of delivery is conventinally calculated by adding 9 months & 8days to the 1st day of last menstruation . Still no body can definately predict about exact date of delivery. .. So it can be plus or minus 8 days from The calculated date. -Some people believe it to be 40 weeks from last menstrual date. Here also it stands variation of 1 week more or less . -babies off pregnancy diabetes women tend to be overweight... that is why the error in calculation. Please don't bother about this calculation. Continue to visit your treating obstratitian as per her instructions... She would guide you for betterment of you &your child. Thanks"
},
{
"id": 191747,
"tgt": "What causes thinning arms and legs in a diabetic patient?",
"src": "Patient: Hello Doctor Good Evening My husband diabetic his last month sugar rest report after food 204. 1. I just want to know it was under control? 2.his weight was 80kg he was tall too.his weight remains the same but i feel like he is becoming thinner his hand arms and legs below knee muscle top becoming thinner. 3.he is active and energetic too. Plz do suggest me did I want to worry about this? Doctor: Hello, Thanks for the query. I have gone through the contents and understand your concern. When diabetes is in control blood glucose Fasting is Therefore, my suggestion is he should achieve better blood glucose control, (as mentioned earlier), plus do at least on two days a week some strength building exercise. Plus to increase a bit of protein intake every day. Thanks."
},
{
"id": 151834,
"tgt": "What treatment should I take to bring mobility in my legs as I am a neuro patient ?",
"src": "Patient: Dear Dr., I am a male in gender and 64years of age and a neuro patient during one year,now I can`t move my left leg and can`t sign with right hand . I visited Dr.Trishit Roy( Neurologist at kolkata)privately and continue his prescribed medicine Tab.-TRIBANTIN-300 and under test- MRI but reports are normal and I am under physiotherapist daily also but no improve any.May you suggest something for improvement? Doctor: Hello \u00a0\u00a0\u00a0\u00a0\u00a0As MRI is normal ,so no organic disease detected,continue with conservative treatment along with regular physiotherapy. \u2018Hope I have answered your query, I will be available to answer your follow up queries, \u201cWish you Good Health and trouble free speedy recovery\u201d"
},
{
"id": 113799,
"tgt": "Cramps in the lower back before micturation. Cause?",
"src": "Patient: Noticed when I have to pee I get like a crampy feeling in my lower back. What is that? Doctor: 1) it can be normal 2) There can be some reflux of urine from ureterovesical junction 3) You may be suffering from some small stone consult some urologist and get your self checked and investigated."
},
{
"id": 4122,
"tgt": "Is there risk of pregnancy after penis touching vagina?",
"src": "Patient: is there any risk...wherein i might have touched the base of the penis....and later touched her vagina....though in the meanwhile i might have wiped off my fingers by touching bedsheet or something... and im pretty sure i havent touched the tip of penis directly Doctor: HIThank for asking to HCMI really appreciate your concern looking to the history given here I could say that the insemination is matter, if the semen gone inside the vagina then this could cause pregnancy else no need to worry about the penetration or touching, this wont make any difference if no semen is there, hope this information helps you, take care and have a nice day"
},
{
"id": 94801,
"tgt": "Upper abdominal discomfort, pain radiating around sides, bruised feeling on deeper breathing. Treatment?",
"src": "Patient: Hi, I ve been experiencing discomfort in my upper abdomen especially when I take a deeper breath. I already suffer from gall bladder problems but this feels slightly lower. The pain/discomfort is also radiating around my sides and slightly in my back, although not as sharp as the usual gallbladder pain. It s almost a bruised feeling upon taking a deeper breath. Doctor: Dear friend, welcome to HCM! With a gall bladder problem, such constant pain on deep breathing could be due to a gall bladder pathology or others like early pancreatitis. Especially if it is increasing with deep breathing, dull aching, constant pain, also going to the back, we should look at the pancreas. You will need a good physical exam and labs (like serum lipase, LFTs) and USG to elucidate the cause. Is there any nausea, fever, fast heart rate? Please be guided by your doctor if there are positive clues. Take care and please keep me informed of your progress. Good Luck! I hope to have answered your query satisfactorily. Thank you!"
},
{
"id": 139477,
"tgt": "Is sharp burning pain after a L3 fracture normal?",
"src": "Patient: l 3 fracture I broke my l3 a week ago. I am home now in a brace and taking meds for pain. I have been experiencing savere pain still including numbness in foot sharp pains in my leg and my back feels like it is on fire like burning. should I go back to er or am I ok? Doctor: Hi,Usually after a L3 fracture, there may be entrapment of the root of nerves or adhesion that may cause Pain and intractable burning sensation.Visiting the Er for the same is beneficial to undergo tests and proper treatment for the same. Avoid mobility at the same time.Hope I have answered your question. Let me know if I can assist you further. Regards, Dr. Vignan Rachabattuni, General & Family Physician"
},
{
"id": 94077,
"tgt": "Urine tests showed strep in urine. Had take PICC line medicines post appendectomy earlier in the week. Related?. R",
"src": "Patient: My 12 year old daughter had an appendectomy a month ago. 10 days later she had a procedure to drain an absess from infection due to the appendectomy. She completed PICC line antibiotics a week ago, but has had urinary issues for a couple weeks. Today urine test came back that she has strep in her urine (UTI). Is this related to the appendix/absess issues? Why didn't the IV antibiotics kill any bacteria present? Doctor: HI appendix with mild ascites.... can cause ureter to get infected.....and secondly strep and staph are skin germs and rarely the ureter can be injured during operation....... they can be due to scar mark abcess if fistula is formed or simply SHE GOT DIFFERENT STRAIN OF STREP FROM THE ANTIOBIOTIC U GAVE..EVERY ANTIOBIOTIC HASS DIFFFERENT SPECTRUM OF ACTIVITY.... JUST BY NORMAL ASCENDING URINARY TRACT INFECTION ............... TAKE CARE"
},
{
"id": 357,
"tgt": "Can person under treatment for epilepsy have healthy pregnancy at age of 41?",
"src": "Patient: I would like to start a family. I am 41. I take medication for epilepsy, 200mg lamotrigine and 450mg phenytoin daily but have been seizure free for a number of years. I have regular menstral cycles and am not aware of any other fertilty issues. I also take 5mg of folic acid daily. Should i be able to conceive naturally and have a healthy pregnancy? Doctor: Hello,I have gone through the query and understood your concern. It is indeed possible to conceive under modern forms of treatment even while undergoing treatment for epilepsy. You need to have a proper clinical evaluation and plan further management. The anti-epileptic medication has to be modulated and changed to safer drugs which can be used in pregnancy. Also, considering your age and health, it is better to opt for ART to save precious time. Please contact your health care provider for further information. Wish you good health."
},
{
"id": 181881,
"tgt": "What cause tingling sensation and swollen gums after wisdom teeth removal?",
"src": "Patient: hello, Dr.Chakraborty,My daughter had 4 wisdom teeth taken out on Monday of this week. However, she is still feeling tingling and a little swollen and really concerned about this. She went back to the dentist today and checked her out and told her to give it time. HOWEVER, we are worried. Please your thoughts Doctor: Hello,Thanks for consulting HCMRead your query as you have tingling sensation and swollen gums after wisdom teeth removal dont be worried so much sewlling in gums is seen for one week after wisdom teeth extraction . Follow post operative instructions and take proper course of antibiotic prescribed by your dentist .Hope this will help you."
},
{
"id": 175894,
"tgt": "What causes loss of appetite,gastric pain,wheezing and sulfurous odor in stool?",
"src": "Patient: Hello, we have 6mo old identical twin boys. They are experiencing a loss of appetite, GERD-like symptoms, some wheezing, discomfort/gas pain, but most notably....very sulfurous smelling stools. The peds MD has tried Zantac, omeprazole & Reglan, only worsening pain w/ babies not wanting to sleep. I have tried an infant homeopathic remedy, Hylands baby colic tablets, which have been somewhat successful in treating the gas pains. They are on Neocate formula, the most hypo-allergenic, pre-digested formula on market. Also...most expensive. Are there any concerns with a lack of ability to break down sulfurous compounds? Any other thoughts greatly appreciated. Lisa/Pharmacist Doctor: MOST OF TOP FEED BABIES DO HAVE THIS PROBLEM BECAUSE TOP FEED HENCE THESE ARE NORMAL THINGS HAPPENS IN TOP FEED BABY"
},
{
"id": 211647,
"tgt": "Seeing animals and people in yard, hearing them at night, no signs of dementia. Reason?",
"src": "Patient: My Mother, age 79, has started seeing animals and people in her yard. The animals she describes as strange looking and the people are wearing brightly colored clothing. That escalated to her hearing them at night on the roof and now she thinks they are trying to look in her windows. She has seen a neurologist and had a chemical analysis of her blood run, an MRI of her brain, a chest xray, a CAT scan and now we are wondering where else to turn. She has no signs of dementia like forgetting people or what she last ate etc. What can this be? Doctor: Hello,What she is experiencing could be Visual hallucinations and Auditory hallucinations. A patient experiencing hallucinations start hearing voices or seeing things / visions which are not there.Hallucinations may occur in neurological illness as well as psychiatric illness like psychosis and respond well to antipsychotic medicine like Olanzapine, Aripiprazole, Quetiapine etc. In my view, she should be seen by a psychiatrist also to rule out any psychiatric illness.Kind regardsVikas"
},
{
"id": 220405,
"tgt": "Suggest treatment for loose motions during pregnancy",
"src": "Patient: Hi I am 27 weeks pregnant with my first pregnancy. I have got Cervical stitch in my 22nd week. I am facing loose motions since last 10 days...I have lost 2kgs since then... My doctor suggested me to take Vibact DS and STIBS but of no use... I am very much worried for my child. Please help. Doctor: Hello dear,I understand your concern.In my opinion persistence of loose motions since 10 days Inspite of taking medicines needs to be evaluated.Stool culture and sensitivity or microscopy might help in accurate diagnosis and treatment.They help in ruling out any infection.And if at all infection is there antibiotics might be needed.Avoid spicy and oily diet. Take bland diet.Take adequate fluids upto 3 litres per day,fruit juices,coconut water daily to combat dehydration due to loose motions.Take electrol powder and probiotics as suggested by your doctor.Nothing to worry.Avoid stress.Hope this helps.Best regards..."
},
{
"id": 144351,
"tgt": "What to do for the three vertebrae that sicks out from the spine?",
"src": "Patient: I recently noticed that I have three vertebrae that stick out on my spine on my middle - lower back. I do not know how long it has been like that. They look more tan than the skin on the rest of my back. They do not hurt and I have not been experiencing back pain except for when I have been standing for a long amount of time. Is this something I should ask my doctor about? Doctor: hi,thanks for mentioning brief history of you.well, since you mentioned the popping of two-three spinous process in the lower thoracic region it is even not giving you any symptoms like pain and aches you need not to worry much.first thing you can do is to get an x-ray done to rule out if any bony issue is there.once ruled out you can meet a physical therapist who will help you do posture correction. since most movements in the body are flexion induced one tends to bend the spine for standing and sitting with comfort.you can even learn postural correction which will help you ease of your worry.with the grace of God I wish you a speedy recovery"
},
{
"id": 91581,
"tgt": "Suggest treatment for abdominal cramps",
"src": "Patient: Hi....I have been experiencing severe abdominal cramps (lower abdomen), chills, headache, fatigue. I ate out on Wednesday night and got severe diarhea within 30 minutes. On Tursday I was fine...thought it was just a little thing...kept on with my regular diet Thursday but felt bloated, crampy, and gassy off and on. On Friday my lower back was aching so I went to bed to get off my feet and took advil. I have a painful lower back which is chronic and I will be starting physiotherapy next week. But on Saturday I developed a bad headache, chills, I felt achy and crampy. I had no diarrhea but gas ,really bad chills in my abdomen, and the severe cramps off and on. No fever that I know of....but this morning I have such severe cramping it is hard to walk... and before I go to the bathroom. Severe cramping and chills again and 99 temp. The chills feel like they are inside me and down my back. No gas today. The severe cramps are on both sides and center of my lower abdomen. I have had my appendix out and this feels different in any case. I get cramps, go to the bathroom, no blockage, but the cramps are horrendous. I am 60 years old, do not smoke, and am a teacher and artist. I am a mother of three grown adults. It feels like severe gas pains but I feel tired and cold and have a headache.Thanks so much,B Doctor: Hi. Thanks for an elucidate history. This is a classical history of gram negative intestinal infection. And you need to take a very proper treatment to avoid further complication which can occur very fast in such infections.- see you have almost no fever but all the severe symptoms of an infection .Rush to ER / Hospital - admission and intravenous fluids and antibiotics."
},
{
"id": 57435,
"tgt": "Is surgery necessary if gallbladder is filled with calculli, have bulky anteverted uterus, fibroid, thick endometrium?",
"src": "Patient: I have been told my gallbladder is completely filled with calculli. The intrahepatic ductsand CBD are not dilated. CBD measures 4.6mm. I also have bulky anteverted uterus measuring 52mm in AP diameter. There is a 30mm intramural fibroid in the posterior. Endometrial thickness of 9.8mm ... what does this mean for me and will surgery be nec essary Doctor: hi thank you asking HCM IF you have a gallbladder filled with calculi you need to consult surgeon . you need open surgery or laparoscopic treatment , ALL the best Dr>klerida"
},
{
"id": 188098,
"tgt": "Should i meet a dentist for tooth pain and swollen gum?",
"src": "Patient: I woke up this morning with pain behind my last tooth on the bottom right and it hurts to chew food but not to sallow. The gum behind my last tooth is cearly more swollen than the gum on the opposite side of my mouth. Should i go see a dentist, by the way im 35 weeks pregnant. Doctor: Hello,The type of pain in the tooth you are mentioning with swollen gums it means that there is an infection in the tooth you should visit your dentist for the treatment of infection because without that the pain will not be relieved.If you are 35 weeks pregnant then right now initial treatments can be done but with precautions.Hope it will help you."
},
{
"id": 95419,
"tgt": "I get frequent abdominal pain. Is it because of abortion ?",
"src": "Patient: hello doctor, i am 21 years old. i am about 5ft 5 , i weigh 60 kg, i am a female, i am of the blood group AS. i usually have abdominal pain just a little above my pubic area and its usually painful and affects my standing position. whenever this happens, i feel very bloated as my stomach becomes bigger, when this happens, i go to the toilet and after easing my self and letting gas out i feel better. but the problem is, it happens frequently. however, some months back, i had a scan which showed a bulky non gravid anteverted uterus with heterogeneous echo pattern. i have always had the pain even before i had the scan and after also. i also have menstrual cramps during my cycle which lasts at times for a day or two. before the scan, i had a medical abortion in which the pregnancy was about 4 weeks and 3 days old. pls doctor, i am confused and dont know what to do. i need your advice. what is the cause of this abdominal pain? does the M A have any thing to do with my scan result? what can i do ? Thank you Doctor: Hi; welcome to HealthcareMagic The cause of your abdominal pain above the pubic region can be due to infection in pelvic region;may be due to bulky uterus;infection of intestine that is lower bowel/rectum but as the pain gets relieved it could be due to tenesmus if the pain comes and goes on its own after some time and is better after passing stools or gas.This sort of pain if you have it could be due to infection of intestine for which you need to take antibiotics.The bulky uterus can be as a result of MA but there can be other reasons that need to be examined so please consult a Gynaecologist for examination and treatment. Thanks"
},
{
"id": 152446,
"tgt": "Is blood in urine a sign of lymphoma of the stomach?",
"src": "Patient: My question is regarding my mother; she is 62 yrs. old with no previous medical concerns, non-smoker, and non-drinker. However she does not regularly see a doctor for annual check ups. Just this month she cancelled a trip over seas due to finding blood in her urine. Ever since then she has had all kinds of testing done including: a physical with breast exam, blood work (Glucose-fasting, cholesterol, TSH, CBC), urinalysis, mammogram and bone densitometry, seen an OBGYN and had a pap, and a pelvic/abdominal ultrasound. Out of all these tests the bone densitometry showed signs of osteoporosis and the abdominal ultrasound showed some enlarged lymph nodes, everything else was normal including WBC. So her doctor recommended an abdominal/pelvic CT scan to \u201crule out malignancy\u201d we did the CT and everything was normal except \u201cat least 3 enlarged lymph nodes within the root of the small bowel mesentery the largest measuring 3.3x 1.8 x 2.9 cm. There is no pelvic or inguinal lymphadenopathy by size criteria. No ascites\u201d \u2013this is quoting the radiologists report. In this report he recommended we follow up with a chest CT to look for any other lymphadenopathy or overt intrathoracic malignancies and also that we biopsy the abdominal lymph nodes by ultrasound guided fine needle aspiration. So we did the chest CT and it came back as a normal chest CT with \u201cno signs of intrathoracic lymphadenopathy\u201d. Now we are booked in for the Fine needle biopsy on Thursday. My question is have we missed looking into anything that may have caused these 3 enlarged lymph nodes or could this be a purely isolated case? Would a PET scan help diagnose anything lurking that we may have missed? I understand that lymphoma could be a possible diagnosis and that is why they are doing the biopsy but does lymphoma generally start in the abdominal lymph nodes?Thank you very much Doctor: as no remarkable lymph nodes cud be seen in thorac and all her other tests are normal ...now only option left is to biopsy of abdonimal glands ..biopsy will be done and according to me biopsy is best method to diagnose an exact cause of any growth...second best are tumour markers ...trust your doctors they are on right path .."
},
{
"id": 186354,
"tgt": "Suggest treatment for bad breath",
"src": "Patient: Hi....my breath stinks like poop and this is causing me problems everyday.I . i have had this problem since my teens and im now 32. The problem seems to worsen when im stressed and has also worsened after I gave birth.. I HAVE HAD TO RELY ON sweets AND SUGAR FREE gum to mask the problem but now these dont help...i have tried most brands of mouthwashes and oxygen toothpaste and these have not helped...now one of my tooth has a cavity and has worsened the problem even further. im going to get this removed soon but I know the problem will persist as i had bad breath before the cavity. My life has been ruined and i cant form any meaningful relationships with other people! Doctor: thanks for your query, i have gone through your query, the possible reasons for bad reath could be the deposits, gum infection, us formation in the oral cavity, decayed tooth, respiratory tract infection, diabetes. consult your oral physician to rule out all these things and get your teeth cleaned and the decayed tooth filled and maintain oral hygiene. get your blood investigation done to rule out diabetes. i hope my answer will help you. take care"
},
{
"id": 224544,
"tgt": "How does one differentiate between plan b and implantation bleeding?",
"src": "Patient: I took plan b emergency contraceptive on Wednesday, 2 days after unprotected sex, which occurred on the third day of my period and my period stopped right after. Now on Sunday, I woke up to a dark red, brownish spotting, is there a way to tell the difference betwen Plan B side effects and implantation bleeding? Doctor: Hello and welcome,I understand the issue however there is no way to tell the difference between bleeding due to Plan B and implantation bleed. However as the sexual episode was on day 3 of periods, the chance of pregnancy is zero. Also implantation bleed usually occurs about 8-10 days after the sexual episode while this has occured earlier. So it is most likely that the spotting is due to plan B.Hope this satisfies your query. Thanks for using HCM.\u00a0\u00a0\u00a0\u00a0\u00a0Feel free to ask any more questions that you may have. Dr Madhuri BagdeConsultant Obstetrician and Gynecologist"
},
{
"id": 182485,
"tgt": "Will damage nerve during tooth extraction heal?",
"src": "Patient: i had my left wisdom tooth extracted, yesterday. Now when i touch the Lips and the lover portion, am not getting much feel of touch. doctor suggested that it can be because of the nerve injury, as he had to cut some bones during the process of extraction, How long will it take for the nerve to regrow Doctor: Yes it will heal but it will take a minimum duration of 4-6 months depending on the amount of damage for complete healing. You can take neurobion tablets which are multivitamin b-complex tablets and would enhance the healing. Though it will take sometime for complete healing you will start getting your sensation back earlier. And it happens in many cases so don't worry about it. Take care."
},
{
"id": 5754,
"tgt": "Prescribed with siphene and susten. How long should I wait to get the pregnancy test done?",
"src": "Patient: Hi doctor, i have been prescribed siphene 100mg for 5 days starting from 3rd day of my last period.i underwent follicular monitoring and on 18th day i was told that the eggs are released and from the same day i have been using susten vt 200.today is my last tablet.till how long should i wait to do a pregnancy test in case i dont get my periods? Doctor: Hi, Urine tests for pregnancy are not likely to be positive before the third day of a missed period whereas blood tests can be performed with good results as early as a week before a missed period. I suggest that you wait for a week after you miss a period as HPTs do not read a positive result with very low levels of hormone as they are not so sensitive. You may consult your doctor for further advice. Good luck."
},
{
"id": 148104,
"tgt": "What are the findings from the brain MRI report?",
"src": "Patient: On my MRI report it says fFindings:Supratentional brain: Very minimal bifrontal subcortical high FLAIR signal may suggest prior vasospasm. Demyelination process or vasculitis is felt unlikely . There is no evidence of hemorrhage, mass effect or midline shift. Ventricular size is normal.I got so upset cuz I that she was telling me what it meant but the clinician was jus a nurse....... I left too quick... Lol please give me an idea to continue to pursue a neurologist r behavior therapy? Thanks so muchAlso I've lost hearing loss n vision within last two years that's wat began this.. Now it's been about 5 days since I've eaten well Doctor: You're mri finding are not contributing to suggest hearing or vision loss. I think you should see a neurologist who will decrease your anxiety. Are you also having hypertension. This may cause such symptoms. Mean while you can take methylcobal tablet. It is a neuropathy healer vitamin."
},
{
"id": 148370,
"tgt": "Had fits, taken Epsolin, feeling restless, dry mouth, sleeplessness. Suggestion?",
"src": "Patient: at present a patient is under aged 76 ur is under convelsion/fit now .epsolin inj. and decadran inj has been given twice with duration of i hr. bp reading was 100/70 .at present his condition is feeling of unrest.talking unergal.dryness innmouth .sleep is not there what u suggest to be given reght now.his history of convelsion /fit is alredy .but from a long time he not suffered from this pl help Doctor: Hi,Thank you for posting your query.Epsolin (phenytoin) is a good medication for treating epilepsy or fits, so, it should be continued. For sleep disturbance, there are a few options. Zolpidem is a good sleeping pill, which does not lead to addiction or hang over. I use a dose of 10 mg at bed time for my patients.Clonazepam is another option, which reduces anxiety and controls fits, in addition to producing sleep.I hope my answer helps. Please get back if you have any follow up queries or if you require any additional information.Wishing you good health,Dr Sudhir Kumar MD (Internal Medicine), DM (Neurology)Senior Consultant NeurologistApollo Hospitals, Hyderabad, IndiaClick on this link to ask me a DIRECT QUERY: http://bit.ly/Dr-Sudhir-kumarMy BLOG: http://bestneurodoctor.blogspot.in"
},
{
"id": 110998,
"tgt": "What causes burning and stabbing pain in the back and hip?",
"src": "Patient: I'm a 53 yr. old female, 5 ft tall, weight 195lbs and still loosing weight. My problem is a burning and stabbing pain in my back and rt. hip. I have been receiving pain injections in my lower right side of my back and right hip for over a year, they seem to only last for a short time. I also have a pinched nerve in my back at L-4and L-5. the pain is so bad that I have a difficult time trying to stand up from a bending over position. One Dr. suggested that I may need back surgery if the injections continue to only last about a month. I still continue to have pain even with the injections. The pain is so intense that it interfere's with my daily activities. I have a prescription for Vicodin and Zanaflex which doesn't do much for the pain. Doctor: Hello, Thanks for your query.The description suggests that you have lumbar disc disease with radiculopathy. The initial management for this problem is conservative by following means : - Bed rest - analgesics and ocassional muscle relaxants - Methylcobalamine preparations which helps to alleviate your \"pins & needles\" sensations - lumbar back support while sitting for longer periods - intermittent pelvic traction If you dont get any relief within 3 weeks, you need an orthosurgeons consultation to decide upon the need for surgical intervention.I do hope that you have found something helpful and I will be glad to answer any further query.Take care"
},
{
"id": 159992,
"tgt": "Suffering from cancer, please suggest tratment",
"src": "Patient: hi..a patient is suffering from :MR findings reveal endometrial growth with suggestion of inflitration into the myometrium(stage 1B)...please suggest Doctor: Hi, Welcome to HCM. You will need to do hysterectomy (removing the uterus) followed by radiotherapy or chemotherapy. For deciding the proper line of management, you will have to consult an oncologist who will advice some tests and depending on the findings in those tests he will decide what line of management has to be followed and how many courses of chemotherapy will be required or how many sittings of radiotherapy and its dosage calculation. Wishing you all the best."
},
{
"id": 225433,
"tgt": "Started using birth control. Should i take plan B? What do i do with my birth control?",
"src": "Patient: Hi I just started birth control the sunday after my period ended and its been only 5 days.I had unprotected sex tonight but it lasted literally 5 seconds because we were in a shower. He didn't ejaculate but I was wondering if I'm at risk for pre-cum. Should I take Plan B tomorrow morning? And if so, what do I do with my birth control? Do I wait a day and then keep taking it from the same pack? Doctor: Hi,Since you had already started birth control and its just 5 days, chances of pregnancy are less. You dont need to take a plan B.Regards"
},
{
"id": 43611,
"tgt": "Secondary infertility. What does the impression bilateral free intraperitoneal spillage of contrast mean ?",
"src": "Patient: Hi, I m 30 years old with secondary infertility . I got my HSG done indicating my uterine cavity normal in size and shape. Mobile filling defects are identified in the uterine cavity representing air bubbles. Both fallopian tubes are outlined with contrast and appear normal. Free intraperitoneal spillage of contrast is seen bilaterally. I am confused what does the impression bilateral free intraperitoneal spillage of contrast mean. Is my HSG report normal? I would appreciate an answer. Thanks in advance. Doctor: Hello, Bilateral free spillage is normal thing and indicate there is no Fallopian tube blockage.Absent spillage of contrast means there is tube blockage.Blockage of tube is one of the common cause of infertility. Hope this helps.Regards."
},
{
"id": 111534,
"tgt": "Suggest medication for severe degenerative disk disease",
"src": "Patient: I have severe degenerative disk disease. I am not a candidate for surgery per my orthopedic physician. I have had numerous nerve blocks and it doesn t help . My pain is at the L5 disk. I take Lortab 750/500 but that doesn t help either. Is there any pain medications that WILL help? I am under a physicians care. Doctor: pain killers and immobilization in lumbo-sacral brace may ease pain , sometimes traction to both lower limbs can be alternative."
},
{
"id": 156574,
"tgt": "What should we do for my mother's swollen legs?",
"src": "Patient: my mom has liver cancer stage IV. her legs have swollen too much. is that a very bad sign? she has abumine infusion but d feet is still getting bigger each day. what should we do? Truly appreciate any feedback. Thanks. please email me @ YYYY@YYYY Doctor: Your mom can take tablet aldactone 50 mg twice daily after food for 5 days.It may help reduce her swelling in legs.RegardsDR Dr De"
},
{
"id": 2526,
"tgt": "Will I be able to conceive after taking ikaclomin?",
"src": "Patient: hi dr im yvette i do have problem my period then i wanted to get pregnant they diganosed me having polycystic ovarian syndrome..then my dr gave me ikaclomin, is there possible i got pregnant if i do take this tablet? thanks YYYY@YYYY my email godbless Doctor: Hello dearI understand your concernIkacolin contain clomfene citrate and it help in maturation of follicle, induce ovulation by increasing FSH and LH hormone.If you have normal period, normal ovarian follicle study, patent ovarian follicle study then there is high chance of pregnancy.You should take treatment under advise of gynecologist and regular monitoring by USG scan is required.You can also ask for progesterone pill after ovulation: It support implantation and early pregnancy.If maturation of follicle and ovulation will not occur then FSH and HCG shot can be usefulTablet ovacare for maintaining reproductive health and decrease insulin resistanceAvoid stress, take healthy diet, drink plenty of water, do regular exercise and maintain body weight according to normal BMI.Hope this may help youContact further if follow up neededBest regardsDr. Sagar"
},
{
"id": 144912,
"tgt": "What causes lack of patience, headaches and pain in upper arm?",
"src": "Patient: I am suffering from lack of patience, prolonged headaches and pain in my upper left arm. The pain the the left arm just started this afternoon. Patience and headaches have been ongoing for the past few weeks. I recently went off my anti-anxiety medication, Sertraline, because it made my feel like I had no energy. I currently am taking an anti inflammatory for my left shoulder that my bone & joint guy recommended for upper arm/shoulder pain. Thoughts?? Doctor: Dear; I reviewed your question and will give you my medical advise. First of all,Sertaline is a good medication to help anxiety and control the lack of patience. I also recommend you to try relaxation techniques as yoga and do regular exercise, that help to relieve the stress and anxiety.Headache is probably secondary to tension and anxiety too, so you should benefit from Sertaline and exercise too. Shoulder pain is probably tendinitis or osteoarthritis and will improve with the anti-inflammatory and exercise. However, if it doesn't, you might need an evaluation with MRI of the shoulder to check if it is not a rotator cuff problem,Thanks for using our service and hope my advise was helpful.Dr.Sara"
},
{
"id": 74810,
"tgt": "Suggest treatment to lung infection which is causing weight loss and sleeplessness",
"src": "Patient: I am a 64 yrs old woman, weighing 40kg and height 5 2 taking medication for hypertension since 1992. I have been an allergic asthma patient since 1994 and have been on inhalers like asthalin and Aerocort. After a pneumonia attack in 2005, I was put on Forocort inhaler (1 puff) 1-1-1 and Duova inhaler (1 puff) 0-0-1. Recently in Dec 2010, I was hospitalized after a severe lung infection and asthma attack and was treated for wheezing, fever and breathlessness with steroids and antibiotics for 3 weeks. I experienced a lot of side effects like mouth sores, loss of appetite, insomnia, tremors, depression, fear and anxiety resulting in weight loss and weakness. Presently, my asthma is quite manageable except for severe weakness, insomnia, bloating and fullness with my blood potassium lowering to 3.1. I was prescribed with Glycomet for induced blood sugar, Losar for BP, Forocort inhaler 1-0-1 inhaler, Slow potassium tablets 1-1-1, Multivitamin, Calcium and Albuterol inhaler ( 2 puffs once in the late afternoon mostly). My general health has not improved remarkably primarily due to sleeplessness, bloating and constant fullness. So, I have now been given Glipiside for Blood sugar, Kimpride (1-1-1) ,Potklor syrup (2tsp-0-2tsp), Montasma plus, Dell 400 and Asthalin (as needed or emergency). I have now tried to take walks for 15 min in the past 2 days inspite of feeling very weak in my legs and feet, fatigue and tired. I am also taking Potassium rich foods. Please provide me with your kind suggestions since I am feeling sick of taking medications and not improved on my health. Doctor: Respected user, hi I evaluated your query thoroughly.* Enough has been given in forms of medications till now .* In my opinion now you require a holistic approach for handling your symptoms rather than additional drugs , these things will help you definitely but at slow rate with lifetime results - maintain your hydration levels by taking plenty of liquids .- balanced nutritious diet rich in all components like carbohydrates, vitamins, minerals, anti-oxidants , trace elements , fats , proteins etc.- keep your BP , Blood sugar, S.Potassium,Cholesterol levels under control .- avoid excess sugars / artificial sweeteners / oily / spicy- avoid smoking / alcohol / any abuse substances if using - avoid stress / anxiety for which meditations & YOGA are corner stone of the therapy - most neglected & most vital part of management in your case.- have regular sound sleep of 6-7 hours- keep weight chart record- avoid steroids till possible - '' BE POSITIVE '' attitude in life , till now you have fought so much like a brave soldier & still God wants you to fight ahead towards every possible ailments with laughing face .- remain ambulatory , manage fresh air walking till possible Hope this helps you.Welcome for any further queries.Thanks for using Health care magic & giving me an opportunity to assist you in the time of your medical needs.Wishing you a happy healthy life ahead.Regards dear take care."
},
{
"id": 198962,
"tgt": "My penis size is very small. Remedy?",
"src": "Patient: Dear sir I am kabeer from ksa my native place in trivandrum....my PBM is now I am 24 year old but my penis size is very small....me very affried sir because my marriage already fixed but not noe AFR 2 year....sir any treatment available for this case.....I need or big help plz thank u very much Doctor: DearWe understand your concernsI went through your details. First of all you should understand that even a two inch erect penis can satisfy most of the girls across the world because women vagina presents pleasure nerve endings upto two inches from the opening. I don;t think your penis size is smaller than two inches in erect condition. I am sure you will be able to satisfy your wife after marriage and therefore, there is nothing to worry.You should consult a psychologist and get a sexual education. You are anxious about penis and sex life because of your lack of knowledge.If you require more of my help in this aspect, please use this URL. http://goo.gl/aYW2pR. Make sure that you include every minute details possible. Hope this answers your query. Available for further clarifications.Good luck."
},
{
"id": 105739,
"tgt": "Allergic bronchial asthma, Asthalin respule,Budecort. Continued wheezing?",
"src": "Patient: my child is suffering from allergic bronchial asthma since birth, i nebulize him with asthalin respule 2.5ml mor. eve. daily and budecort 0.5mg mor. eve.daily and telekast-L kid tab. once daily and nasonex nasal spray 50mcg once daily. but still there is coughing on exertion or running and there is a wheezing sound when he sleeps which is coming from his nose which is audible.can be there any volve involvement there?what is your opinion what should i do? Doctor: Pl start him on seroflo inhaler(100 mcg) 2 puffs twice a day for long term control. You may give a short course of oral omnacortil 2 mgm once a day for 5 days."
},
{
"id": 12224,
"tgt": "How to find out vitiligo Symptoms ?",
"src": "Patient: how to find out vitiligo Symptoms i got small white dots on my fingers Doctor: Hi.. Vitiligo may not present with any other symptoms such as tiny white( (depigmentation) patches.. Apart from these other symptoms may include whitening of the scalp hairs, white patches of the oral cavity.. Depigmentary patches are also known to occur on the retina and can be seen on eye examination.."
},
{
"id": 135488,
"tgt": "What to do for the swelling in the foot?",
"src": "Patient: I tripped and the left outside part of my left foot hurts and it s a big swollen. I can walk but I have a bit of a limp. This happened to me in my childhood quite a few times but it always healed after a couple if days. Should I be worried. It happened today. My graduation is tomorrow so I m kind of nervous about possibly having to limp across stage. Any home remedies that really help with this issue? Doctor: You have ankle sprain with possible tear of the ligaments. You need to wear a non adhesive elastic bandage. Keep the limb elevated and apply ice. Take pain killers twice a day for 5 days after meals. Avoid bearing weight on that foot for 2 weeks. Wear Basketball shoes for better protection henceforth."
},
{
"id": 198670,
"tgt": "Any effect on fertility after getting electric shocks?",
"src": "Patient: hello Dr. Grief. I would like to ask about my problem on fertility. I am worried if I can have a baby with my wife, because I experienced electrical shock 7 years ago at 440 volts and I was unconscious for 15 minutes that time, there was no burn noted all over my body. Thank you Doctor: DearWe understand your concernsI went through your details. There is no chance of that sort. Electric shock do not have any direct impact on your reproduction ability. You may check yourself. Consult a specialist and get your semen analyzed to be yourself sure. Don't worry unnecessarily and don't be obsessive and anxious. If you require more of my help in this aspect, please use this URL. http://goo.gl/aYW2pR. Make sure that you include every minute details possible. Hope this answers your query. Available for further clarifications.Good luck. Take care."
},
{
"id": 192048,
"tgt": "What causes dizziness and imbalance after a fall?",
"src": "Patient: Dear Sir, My father, 75 year old man is a diabetic since last 25 years and is on insulin twice a day. He was also diagnosed with Ischemic heart disease some 15 years back and is on medication since then. A month back, suddenly, he felt dizziness and fell down while performing puja in standing position. His chin got hit & also left ear bled for 2 days. After that, he slowly recovered and everything became normal within a fortnight. Only, he felt dizziness occassionally. However, recently he has developed walking problems like loss of balance, lack of strength in one leg. We had not conducted CT Scan after his fall last month, so we are not aware of any clotting in the brain. We are not able to know whether the walking problem is due to the previous ear injury or some disorder in the brain. He is being taken for a CT Scan and consulatation with Neurologist on 6.4.10. Sir, please let us know the gravity of the problem and advise us on the further course of action. Regards. (B.M.Baral) E-mail: YYYY@YYYY Doctor: Thankyou for your query.Your father`s dizziness might probably due to hypoglycaemia as a result of insulin. Kindly check the blood glucose fasting as well as post prandial and HbA1c. Do mention the units of insulin your father is taking. Also get his TMT done. Are there any other symptoms like shortness of breath, pain in chest etc.Kindly get an opinion for angiography from any cardiologist. Regarding loss of balance or lack of strength in one leg do consult a neurosurgeon as that might be related to bleeding from ear. Kindly tell the CT scan findings also.Feel free to get in touchGod bless"
},
{
"id": 14977,
"tgt": "Will zinc tablets and dermaspray cure cold sores and cuts on left side of lip, lesions on left side of the body?",
"src": "Patient: I suffer from cold sores above my top lip left handside of my face.( for about 8-10 days already) Developed cuts in the corner on left handside of my mouth,I also noticed a red raised lesion on upper chest and on abdomen left handside of body, which is very itchy and has a burning sensation. I'm using a host of natural medicines: L-Lysine, Vit B & C complex, Bettamed, LP299V Probiotic. I'm at my wits end and not sure what else to do. Decided to buy Zinc tablets and a Spray - Dermaspray to give relieve to the \"bumps \".I'll really appreciate some advice. Thank you. Doctor: Hi, Welcome to Health care magic forum. They appear to be the eruptions due to the defeciency of vitamin bcomplex, or viral infection or shancroid, etc. I advise you to consult a dermatologist for diagnosis and treatment. Take more of green leafy vegetables, pulses, sprouts,and protein rich foods to hasten the recovery. Wishing for a quick and complete recovery. Thank you."
},
{
"id": 94131,
"tgt": "Intense pain in upper abdomen, history of renal stone. Also pain in supra-clavicular region. Advice?",
"src": "Patient: Hi I have intermittent pain in my left supra-clavicular region. No palpable lymphadenopathy . No history of malignancy . Negative skin TB test this year. No fever. Recently I had cold with a lot of coughing. Some associated pain in my left side upper abdomen front and back. Pain increases in intensity with breathing. It starts suddenly, lasts ~1-2h and then is gone. Remote history of renal stone . Currently no blood in urine Doctor: Hi Pain in the supraclavicular region is due to musculoskeletal cause in your case. As there are no palpable lymphnodes and no fever,TB can be excluded. In cold,due to secondary infection cough can occur.As there was history of renal stone,pain abdomen can be due to recurrence of stone. Get an Ultrasound abdomen and confirm the diagnosis. Till then,please take Antispasmodic Dicyclomine. If you come back with Ultrasound report,i will clarify further Wish you good health Regards"
},
{
"id": 88326,
"tgt": "What causes lower abdominal pain?",
"src": "Patient: I am 27 yrs old and have been experiencing a lot of pain in lower abdomen and pelvic area since last 4-5 monthly cycles. It starts few days before or during the peroids. The gynecologist told me that its Endometriosis and I have a cyst of 2 cm in my ovary. She has adviced me to take Novelon for 3 months (3 cycles of 21 days each with 1 week gap in between each cycle). Is is safe to take Novelon and will it actually help to cure the cyst? What are its side effects? Can it also affect the prospects of conceiving in future? What should be the treatment for this problem?Thanks.Shivi Doctor: Hello, Welcome to HCMI am Dr Rakesh Sharma answering your query.I can understand your concern.It is safe to take Novelon, for you.It is not known for treatment of cyst.Side effects are nausea, vomiting,.abdominal cramps, weight gain.It will help in conceiving in future.Your doctor is giving you right treatment.Hope this will help you.Good Luck."
},
{
"id": 24579,
"tgt": "Why does hospital visits cause BP to drop?",
"src": "Patient: Hi, my mother-in-law is in the hospital for a broken arm. She was fighting with the attendants while they were trying to cast her, so they sedated her. When she should have been waking up, her blood pressure dropped to 70/30. The last time she was in the hospital the same thing happened with her blood pressure when they sedated her. Why does this keep happening? Is there something else going on that they aren't telling us? Doctor: Sedative drugs causes fall in blood pressure. It depend on which drug and at what dose it is given ...so don't worry"
},
{
"id": 183688,
"tgt": "Is it possible to wear braces without removing the tooth?",
"src": "Patient: Hi, yes i have all my fully grown teeth but i seemed to have a baby tooth inside my gum. And my dentist said that before getting braces that I needed to get my fully grown adult tooth taken out . what happens if I decide not to get my tooth taken out and just get braces? Doctor: Hello.thank you for sharing your problem with healthcare magic. Generally over retained deciduous teeth need to be taken out.In your case its orthodontic problem .and sometimes due to lack of adequate space permanent teeth need to be taken out too .In that case you can't skip extraction and go with braces only. you can get better treatment from orthodontist than general dentist. You can consult for further queries. Wish you good health."
},
{
"id": 61627,
"tgt": "What could lumps on back of head in infants suggest?",
"src": "Patient: I have an 18 month old who has been actively playing all day. He runs into things, trips over things, and falls down all the time (as normal 1 1/2 year olds do!) But I just noticed a lump on the back top of his head! I was a little concerned and my husband said since he s still acting normal to wait a while and see if it goes down... he has a regularly scheduled dr. appointment tomorrow morning, would it be okay to wait and see how it is by then? Doctor: Your child must have suffered trauma over the back of head. It appears to be blood collected ( haematoma ) . Nothing to worry. It will resolve.Consult your doctor as per fixed appointment."
},
{
"id": 139352,
"tgt": "Suggest treatment for hip and leg pain",
"src": "Patient: Please write your question herepain in hip and right leg .. the leg hurts worse than hip.. can t hardly do anything without it hurting ..my inflammation shows up to be 7xs higher than normal in blood.. it came in out of place as a child and i had a wreck on an a.t.v and has hurt since..been on 7.5 lortab and diclofenac for 2years but no one seems to kne.. they told me sciatic nerve I ve been told the cartilage is almost gone .. no one knows what they are doin and I m only 34 very active and this has slowed me down and puts me in the bed at times.. any suggestions?? Doctor: Hi I feel you need to be more specific with your symptoms. Are you on Heparin and if yes for what, moreover , what hurts you more, walking or rest, and where mainly in your hip knee or any other joint also. to me it seems your age is not that much to cause osteoarthritis, and a proper history and examination and will give a better clue."
},
{
"id": 95872,
"tgt": "Bulky Uterus with large fibroid. Can i be pregnant with this ?",
"src": "Patient: My marraige is arranged and from the past one mointh my periods are irregular and my I used to suffer from stomach pain from many years by taking scanning I came to know that I am having bulky uterus with large fiboid. Therefore I request you to please suggest me whether to go for surgery or else is there any medicine to cure without any surgery and also let me know will there be any problem for getting pregnancy or any infertility problems may arise in future. I have already consulted gynaec and they have given me medicines for three months to compress the fibroid then after they are going to do surgery and remove it. But recently a fibroid was removed from the breast and keloid was formed and I am worried again any such problem may arise. So I request you to please give me a best solution for which I shall be grateful to you. Doctor: Hi Deepthi, Welcome to HCM. There is no reliable medical management for surgery. You have not mentioned the size and number of fibroids. If the fibroids are small and of sub-serous type, it should not affect pregnancy, but if they are large and intramural in location then it might interfere with pregnancy. As for keloid formation, there is small chance that it may occur again, but usually it is seen in the chest and adjacent areas."
},
{
"id": 17474,
"tgt": "Suggest treatment for heart stroke",
"src": "Patient: Good to hear that i have got atleast some body to answer my query. my uncle has got HIV. W came to know only when he got the heart stroke and sent the sample for testing.he has left anterior fascicular block and also anterior injury. his HIV values didnt cross the normal limitis value. they r at the border line of normal limits. where can i get the treatment for his heart stroke. its been 10days he got the heart stroke. please reply me as soon as possible. Doctor: Hello, After going through your medical query I understand your concern and I would like to tell that the best option for him will be thrombolysis therapy in which he has the possibility to recover from that heart injury. Kindly consult with your treating Cardiologist for a safest and beneficial way out for his cardiac health. Hope I have answered your query. Let me know if I can assist you further. Regards, Dr. Bhanu Partap, Cardiologist"
},
{
"id": 210719,
"tgt": "Can adderall be taken to improve concentration and focus with family history of ADD?",
"src": "Patient: I have failed out of college twice, I have the hardest time even finding the drive or motivation to get out of bed in the morning. daily tasks are seeming impossible and this is something that i have battled with since a young age. ive never performed well in school. ADD runs in my family and no one is brave enough or are too embaressed to see a doctor for it, but i cant let it control my life anymore. My cousin courtney takes 20 mg xr adderall everyday and she said it changed her life. Who can i see in a quick manner and possibly talk about a prescription for it? i want to start school soon but i want to be able to hold focus and concentration. Can any doctor help me?? Doctor: HiThanks for using healthcare magicFor ADD, better to consult a psychiatrist and get yourself evaluated that whether you have ADD or not. If you have, that at what extent. In all case, there is no need of medications. If you have mild ADD, you can improve with certain techniques and moderate to severe ADD, you can take drug after psychiatrist consultation.Thanks"
},
{
"id": 121902,
"tgt": "What causes throbbing of blood vessels?",
"src": "Patient: My daughter has been experiencing throbbing of blood vessels in her forearms and thighs. It does not hurt but is disconcerting. It also seems to occur in other parts of her body. Has been occurring for about a month. Do you have any thoughts about what might be going on? Doctor: Hello, Welcome to Ask a doctor services. I have read your query.Throbbing of the blood vessels occurs commonly in Phlebitis. In superficial vein phlebitis patients usually complain of burning & throbbing veins in the legs & sometimes in the arms.Neglect may complicate to medical conditions like varicose veins, etc. You are suggested to get clinically examined by a GP. Get Blood & required Imaging tests done. Get treated the factors causing the symptoms.Hope I have answered your question. Take care.Regards,Dr Nupur K"
},
{
"id": 48258,
"tgt": "Suggest treatment for kidney stone and UTI",
"src": "Patient: Hi sir, I am suffering from a 3mm kidney stone with UTI and little pain on left side of stomach. i have already consult with a MD he gave me a powder type Ayurvedic medicine i had that more then 2 weeks but there is no improvement and still facing the same problem. pls help. Doctor: Hi,Thanks for writing in.A 3mm stone in kidney is a small one and unlikely to cause significant urinary obbstruction. It is a non obstructive stone but can cause slight discomfort to you.In general, people with kidney stones are asked to drink lots of water and visit wash rooms regularly. The stone will dissolve in few months. Please reduce foods containing oxalates like non veg, vegetables like spinach, tomato, cauliflower, brinjals, nuts like kaju, pista and almonds. Avoid holding urine for long intervals. This will help prevent the formation of more stones. Please do not worryIt will help if you also get examined for any stomach problems like ulcer. That can also cause pain that with not respond to kidney stone treatment. Please take less spicy foods."
},
{
"id": 164057,
"tgt": "Suggest remedy for hard and bloody stools in infants",
"src": "Patient: Hello my daughter is 9 days old. I have been feeding her breast milk and enfamil gentlease. She seems to have a hard time pooping and when she does a hemroid sticks out. Sometimes it even bleeds. What should I do to help her? I heard giving both breast milk and formula isn t good for baby but I don t produce enough milk to satisfy her. What should I do? Doctor: Give her colic aid drops or any other simethicone drops available in your area. Gripe water can also be added to her formula or given otherwise. Try to be sure she does not pass hard stools. Make sure the ratio of formula to water is correct and not concentrated more."
},
{
"id": 210947,
"tgt": "Suggest natural remedy for ADD",
"src": "Patient: I have a six year old boy who was diagnosed with ADD but we are not comfortable with putting him on medication. We have begun giving him 1/2 cup of coffee each morning, and is working, according to his teachers. The problem is that after lunch and the coffee has worn off, he is right back to not being able to focus and stay on task. I would like to send chocolate covered coffee beans for him to snack on at lunch time, but not sure how many... Doctor: DearWe understand your concernsI went through your details. I suggest you not to worry much. Attention Deficit Disorder is not as you think. Please understand. A 6 year old child is supposed to have vibrant and creative mind and he cannot stick to any task continuously for more than 5 minutes. You cannot diagnose it as ADD. Your child is normal and other children are not vibrant and creative, if you compare. You need to give him training, not stimulants. Otherwise stimulants will be a habit and it ruins health in the future. Teachers will say so because their task is cut off. They are supposed to train your child. If they don't, you do. Give him enough rewards (non monetary, no stimulants), appreciations etc. Sit with him. Help him in his attention. Easy and creative tasks are good for improving attention. There are several methods. I shall help you further in this aspect. You can contact me with a direct question in this portal itself.Hope this answers your query. Available for further clarification.Good luck."
},
{
"id": 87244,
"tgt": "What causes lymphadenopathy and pain in upper abdomen?",
"src": "Patient: My name is Dr. Berger Marius and I'm a specialist in ObGyn and specialist in anaesthiology and intensive care. The question is not from my specialty but more a demand for help for my cousin. He is a 48 y.o. healthy. He visited your country,Bangalore, one year ago during a mision of Israely gv. Since 6 mo. he is suffering of generalised lymphadenopathy and pain in the rt. uper abdominal quadrant. All the tests including pet scan,US, blood, serology were negative except the reactive lymph nods on byopsy. The pain in the UAQ is intermitent. No changes in the bowel movents, no loss of apetite or wheight. May be you can help us to think of a disease that you met in the population you are taking care. Thank you Doctor: Hi.Thanks Dr Berger Marius. Read the history about your cousin.The reactive hyperplasia of the lymph nodes on biopsy after visit to Bangalore, India is suggestive that this may be in fact be reactive hyperplasia. Yes, in India we get this picture very commonly. I would advise you to give him a course of antibiotic and oral steroids like Deflazcort in tapering off dosage for 3 weeks and see if this helps. If the nodes and pain in the upper right abdominal quadrant still remains, you can go for another biopsy to rule out Tuberculosis. Also subject the node for TB PCR. If positive give ATT for 6 months.The pain in the right upper quadrant of abdomen can be due to lymph nodes or biliary problems. You mat go for repeat scan after 3 weeks of treatment if there is no relief within 3 weeks of the treatment as mentioned above."
},
{
"id": 123748,
"tgt": "How to cure stiffness on the knee after the surgery?",
"src": "Patient: Hi I had my acl / meniscus surgery on 13 June 2012. 3 wks before. Due to soft bones surgeon used staples to fix the graft, I was only allowed the knee movement from today otherwise I was keeping it just straight in the brace. Now I started bending it but it s only 30% & pains an lot. It s stff. Will I get my knee motion back..?. Doctor: Hi, To gain the knee movements after any surgery is a painful and prolonged process. You will have to continue physiotherapy, take pain killers, and push yourself. Sometimes even then the movements do not become normal. Again some procedure may be required. My only suggestion will be that you follow your doctor's advice sincerely for a good result. Hope I have answered your query. Let me know if I can assist you further. Regards, Dr. Gopal Goel, Orthopaedic Surgeon"
},
{
"id": 114131,
"tgt": "I am suffering from lower back pain 24/7. MRI results say I have L4-L5 left for aminal disc protrusion",
"src": "Patient: Hi I am 30 years old and suffer from lower back pain 24/7. My MRI results say I have L4-L5 left for aminal disc protrusion and annular fissure causing moderate neuroforamina narrowing. I ve taken medicine for the pain ( Medrol. Norflex, Percocet, Toradol, Keralog and neurontin) but none seem to help. I can t walk , stand or sit for more than a couple of minutes and recently I ve been experiencing a tingling cold sensation on both of my feet and left arm. Is there some kind of theraphy to help ease the pain and help with the other symptoms I m experiencing, or surgery is really necessary??? Thanks T Doctor: The cold sensation in feet and left arm is because of the compression of the sensory nerves in spinal cord. Well, looking into your young age you may have several options Surgery is one option which may give miraculous results. There are two options - one is open surgery and other is through a scope. The protruded disc material can be removed. Also Ozone Therapy can be tried. Physiotherapy is a double-edged sword. It may give good results or it may lead to stoppage of whatever little motion you have left."
},
{
"id": 71993,
"tgt": "What could cause pain over the heart and breathlessness?",
"src": "Patient: Hi my name is Christine and I have been dealing with a pain over my heart for about a week now and it hurts to breathe or lift up my left arm. Now the pain is traveling to my back. I can only take short breaths when it occurs also. Do you know what this could be? Should I go to a hospital? Doctor: hi. by the symptoms as u narrated. it cld be a muscular pain. but age s not mentioned. kindly get one ecg done just in case to rule out any problem related to heart. i feel no heart problem I'll be there in ur case. u can just take simple analgesic. that I'll solve prob. thank u"
},
{
"id": 91719,
"tgt": "Any suggestion for 13.3 wbc when having abdominal pain and MCTD?",
"src": "Patient: When I have lab work done my doctor always says it is normal. Then when I reauest a copy I see several values out of the notmal range for instsnce my wbc was 13.3 that isn t notmsl is it? I had been c\\o abd pain I havr MCTD a nd am on immunosuppressants Doctor: Hi, thanks for using HCM.use of immunosuppressants do changes some of the blood test results, considering that your doctor might have said normal report.WBC report varies as you are in medications. not to worry, Regards"
},
{
"id": 35710,
"tgt": "Could the pimple on genital organ be fungal?",
"src": "Patient: Hi Doctor,I need help,in my private there was a small pimples pass two years,no feel,but this years there was lot of pimples n look like fungus n verY itchy ,some kind of liquid coming out with blood,it's really bothering,I can see the thick skin,how you can advice me Doctor: Brief answerveneral disease to be evaluated in detailDetailed answerprogressivly increased number of lesions with itching and bloody discharge is suggestive of venaeral diseaseplease visit your doctor for detailed check upThanks for posting your question on health care magicBest of health Regards"
},
{
"id": 165618,
"tgt": "Why does a baby cry for long periods?",
"src": "Patient: my landlord has over 11-month female baby. Yesterday night about 1240, she started crying and it took about an hour. After that, she cried again about 0320 am till morning. I held that baby and she has no fever and her tummy, her hands and her feet were also warm. I do not know what happened with her. Her mother is not educated. Please advise me the possibilities. Doctor: Hello! Thank you for writing! There are some simple causes why a baby cries: the baby can feel hunger, is wet and needs to change the diaper, has temperature. This are the mos banal causes of crying. Is the baby good in weight, defeates and urinate well? I suggest to see a pediatrician to do a total excamination of the baby. Wish the baby good health. Thank you for writing."
},
{
"id": 111234,
"tgt": "What should I do for severe back tail bone pain?",
"src": "Patient: I ve had tailbone pain for the past 1 1/2 years. I don t remember injuring it at all. The pain is more severe on days that I sit a lot. I asked my doctor about this and she pretty much dismissed it as nothing. Any thoughts on this would be great! Thanks Doctor: Hi Sorry to hear about your problem.You have a condition called COCCODYNIA. This is basically inflammation of the tip of the tail bone. The tip of the tail bone has a set of three very small bones with joints between them. People who sit for long periods of time involving perhaps, a desk job can transmit considerable pressure to the tip of the tail bone. Treatment involves sitting on a ring cushion with a hole at the centre of the cushion. You should sit with the tip of your tail bone on the hole and buttocks on either side. Also injections given at the spot can reduce inflammation at the point.Hope this helps,Do let me know if you have any more queries on this one,Dr Purushottam Reddy Padala MS(Ortho) FRCSConsultant orthopaedic and joint replacement surgeonHappy Joints Orthopaedic centre. Hyderabad. India"
},
{
"id": 56435,
"tgt": "What causes vomiting with sweating after alcohol ingestion?",
"src": "Patient: Hello. 3 nights ago, I consumed alcohol, whiskey, about 20 ounces over 5 hours. Don t drink very often. (I am 6 5 280 lbs, 44 y.o., no major mediacl issues - no diseases, no surgeries, etc.) I did have dinner first. The next morning I vomited yellow, no food. Within the next hour, I vomited again, this time green, no food. No headache, no diarrhea, no signs of jaundice or even bloodshot eyes. I have no appetite still 3 days later (but I am eating, having bowel movements), and am sweating mildly non-stop. I have had terrible armpit B.O. since then also. No urge to vomit since morning after . No heartburn. Did I permenantly damage my liver? Doctor: Good morning Sorry to hear about this illness I can reassure you that these symptoms don't indicate permanent liver damage. These symptoms indicate that you might have stomach upset (called alcoholic gastritis) due to binge drinking of alcohol. I would suggest you avoid this behaviour of binge drinking (means on a large amount of alcohol and a point of time and off alcohol for long periods of time) as this is more hazardous than consuming small quantities of alcohol over prolonged period of time. I would suggest you to get a liver function test on your blood to clear further doubts. So take of your drinking behavior to stay healthy Hope this would answer your question wish you a speedy recovery"
},
{
"id": 226114,
"tgt": "Need instructions on taking the birth control pills",
"src": "Patient: Hi I have been taking Yaz however I have not changed to lavlin . I'm not sure where to start my last Yaz pill was Friday and I would normally start again on Wednesday (4 day sugar pill). Should I be starting lavlin on Wednesday or take the sugar pills Wednesday Thurs and Friday as per there packet (7 day sugar pill) and start on Saturday Doctor: Hi.Thanks for asking in Healthcare Magic.Yaz contains 28 (24+4) pills with 24 active and 4 sugar pills and Levlin has 28 (21+7) 21 active and 7 non hormonal pills. Since you are already on Yaz and the sugar pills get over on tuesday, you should start the active pills of Levlin from wednesday.The small beige coloured pills in Levlin pack are the hormonal pills and the large white coloured pills are the non hormonal pills. You should start with the small hormonal pill on wednesday. Once the 21 hormonal pills get over, the non hormonal white pills can be taken and during this time you will get your menstrual cycle. Levlin is a low dose hormonal pill. In case you have doubt about the identification of the hormonal pills in the pack, you should seek the help of your treating Dr, otherwise, if you start with the wrong pill, there may be a contraceptive failure.I hope this is useful."
},
{
"id": 226130,
"tgt": "Suffering from mood swings, headache, dizzy and nausea. Taking levest. Side effects of pill?",
"src": "Patient: Hi, I have been on The pill (levest) for about a month now and have noticed a lot of changes and I m not sure if it s because of the pill. I keep having extreme mood swings and snapping at my family, I have a few headaches and the other day it was one of the non pill days I felt really dizzy and nauseous and weak. My energy is really low all the time to the point where I can just lay in bed for hours or even on my floor cause I just can t be bothers with anything. And in the past 2 weeks I have felt extremely depressed to the point where I randomly cry at things and get really upset and unhappy with my life even though i don t have any huge problems? I also get really paranoid and look too much into what people are saying to me. Also have been bleeding the whole time. Is this due to the pill and should I switch /come off it? Thanks! Doctor: Hi.Thanks for asking in Healthcare Magic.Do not worry. The problems you are having are due to the influence of hormones present in Levest. The progesterone, Levonorgestrel in Levest is responsible for headache, mood swings, nausea, dizziness, depression and fatigue. Levest also results in bleeding/ spotting between periods. It may take a minimum period of three months for your body to get adjusted to the effects of such hormonal pills. If these side effects continue and interfere with your daily activities then you can switch for another OCP combination with lesser amount of progesterone. Triphasic pills contain same amount of estrogen in all 21 tablets but varying strength of progesterone for every 7 pills in the pack. Progesterone is started at a minimum dose and increased every 7 days. Triphasic pills are claimed to mimic the hormonal pattern that occurs during a normal menstrual cycle. Since progesterone is causing the side effects you can consult your treating Dr and opt for a triphasic pill and that may solve your problems.I hope this is useful to you."
},
{
"id": 37146,
"tgt": "Suggest remedy for persistent cough",
"src": "Patient: Hello. I think that I might have bronchitis. It all started in 10/21. Usually when the seasons change up here in Northern California, I get Nasal Drainage. Usually after the Nasal Drainage, I get a sore throat and a little bit of a cough. It s usually gone within seven days. But this is different. After about a week passed (and mind you I never got a sore throat), I started coughing. I figured my immunity was up and my throat was spared. But the coughing has persisted up to this day. How do I get rid of this garbage? Doctor: Hi,It seems that you might be having lower respiratory tract infection, mainly bacterial.Go for one antibiotic medicine course for 5-7 days.Take some cough suppressant syrup to get symptomatic relief.Avoid cold drinks and junk food.Ok and take care."
},
{
"id": 111756,
"tgt": "Lower back pain while sitting/ standing and better when standing?",
"src": "Patient: I was diagnosed with a uti yesterday and was given a shot and I am taking antibiotic which today is the last day of the medicine. But I have been having low back pain when I sit or get I get up from a chair I am more comfortable standing up the pain in my lower back feels like something is pulling Doctor: I think along with UTI either you are having a renal stone or back muscle spasm. Both these disease causes such type of pain as you have mentioned. So now i would advice you to get ultrasound of your abdomen and X ray of your spine. These two investigation give us clear picture about your disease. If it turns out to be back sprain than take muscle relaxant like myoril and myospaz. If It is renal stone than meet a urologist."
},
{
"id": 98525,
"tgt": "How can swelling on the tongue be treated?",
"src": "Patient: my husband had a allergic reaction after eating at different resturants one time he ending up in the er,with hives and his tongue swelled up he s has a epic pen now but we are not getting any answer from our dr. we ve done different test but nothing is showing up so what can we do now? Doctor: hi sir/madam,swollen tongue can be easily treated by the help of ayurveda.Let me advice you about tongue swelling.Swollen tongue is actually an abnormal condition in the tongue occurring due to tongue inflammation, where either a part or the entire tongue gets enlarged, and fatter than the normal tongue size. It is true that tongue in its swelling conditions is taken as a kind of defence mechanism which helps in fighting against the infection and aids in healing. However, when there is an excessive swelling for longer time; the swollen tongue can be actually very much harmful.It must be mentioned that tongue inflammation or the swollen tongue condition can result from the excessive fluid accumulation in the tongue tissues and termed as Angioedema. This swollen tongue can also occur due to excess deposit of abnormal substances like the amyloid protein and also can be result from any kind of allergic reactions to certain medicines.Symptoms Of Swollen Tongue:-1. Enlarged, puffy tongue due to excessive swelling2. Difficulty breathing when tongue swelling outgrows to the throat or the air passage.3. Discoloration or redness seen sometimes due to inflammationProblem while swallowing foods.Causes of swollen tongue:-1. Tongue sores usually caused due to virus.2. Strep infections can cause swollen tongue.3. Sometimes burning of tongue while eating food in hot conditions can lead to the trauma and swollen tongue.4. Vitamin D deficiency can also lead to swollen tongue and other tongue inflammation.5.Certain allergic reactions can cause swelling in tongue.6. Patients suffering from genetic disorders like down's syndrome, are likely to have swollen tongue.7. Tongue cancer, leukaemia etc causes swollen tongue.8. Breakdown of skeletal muscle tissue or Rhabdomyolysis can cause swollen tongue.9. Hypothyroidism, amyloidosis etc can also lead to tongue swelling.10.Excessive alcoholism and smoking also causes swollen tongue.Next i am going to tell about the home remedies you should follow:-1. Take enough of butter, soups, yoghurt, gravy and broths in regular diet. In fact try to switch on the semi-solid foods which help in providing relaxation in swollen tongue condition.2. Cold temperature helps in reducing the tongue swelling. So keep some ice cubes in your mouth and relax yourself.3. Include celery sticks in diet and provide enough relief to your swollen tongue.4. Make a dilute solution of turmeric in a warm glass of water and gargle with the solution to get relief from tongue inflammation and swollen tongue.5. Drinking small amount of water at regular intervals help you ease the swelling of tongue.6. Placing a spoon of sugar on the tongue also help you in reducing the swelling of tongue.7. Soak some rosemary, mint and aniseeds in a bowl of water overnight, strain the mixture and use it as a mouthwash in the morning. This will help you greatly.8. It is also advised to take less amount of salt and prevent excessive production of saliva, which might cause difficulty swallowing with a swollen tongue.9.Aloe Vera as Herbal Home Remedy for Sore Tongue,because of the anti-inflammatory properties, Aloe vera is used as a herbal home remedy for sore tongue to reduce pain and inflammation. All you can do is extract the gel from an Aloe vera leaf and apply it directly over the affected area in the tongue. You can also rinse your mouth few times with the aloe vera juice for reducing the inflammation.10.Baking soda, due to its anti inflammatory property can be used as a natural home remedy for sore tongue. You need to make a paste by adding \u00bd tablespoon of baking soda to a glass of water and apply it in the affected portion of the tongue or the mouth.11.Honey is a wonderful home remedy for sore tongue. You can apply honey to the affected area in the mouth for better healing of the sores. Honey can also be added to lukewarm water for rinsing the mouth twice or thrice a day.Use any of this home remedies to get a excellent results.Hope i was helpful.Have a healthy day."
},
{
"id": 224918,
"tgt": "Could taking logynon as emergency pill after having unprotected sex avoid pregnancy?",
"src": "Patient: I had unprotected sex But my boyfriend did not ejaculate inside me but I was so worried I live in Middle East and they do not sell morning after pill here So I took logynon as emergency pill I took 4 of the dark brome pill And after 12 hours I took 4 white pill This like a week ago But I still worried am I pregnant Since I feel tired with Nausea and stomach pain going to vomit Please reply Doctor: Hi, Welcome to Health care magic forum. The symptoms of the nausea,stomach pain,vomiting feeling could be due to the pills, so don't worry about it.if you go on worrying about it,you may get the false pregnancy. I usually prescribe to my patient with such symptoms omeprazole, domperidone. Wishing for a quick and complete recovery. Thank you."
},
{
"id": 160631,
"tgt": "What causes startle response in my son?",
"src": "Patient: My son was diagnosed with having an Exaggerated Startle Response episode last night. He has a cold and possible had a low grade fever and Sunday night was sleeping on my husbands lap. He jerked awake and went tonic with his body, legs. His arms were drawn up and fingers were clawed. What caused this? The neurologist says this just sometimes happens. He is 3. Doctor: Hi, Is this the first episode? How long the jerking and posturing lasted then? Any drowsiness or sleepiness following the episode? If first episode, sudden jerking and tonic posturing in a child who is suffering from fever and cold is likely a febrile seizure, especially if this was associated with up rolling of eyes or frothing from mouth. Usually this needs nothing other than treatment of fever. Only in recurring cases we give anti epileptic prophylaxis. Startle response is likely if such episodes comes during sudden sensory stimuli like loud noise or flash of light- this will be just momentary with no disturbance in sensorium or posturing. Kindly get back with more details or discuss with your pediatrician. Hope I have answered your query. Let me know if I can assist you further. Take care Regards, Dr. Muhammed Aslam TK"
},
{
"id": 55759,
"tgt": "Can Hep C be cured with medicines?",
"src": "Patient: I had hipatitis c with genotype 4 i took pygasses for 3 months and it was found that the virus was un detectable i stopped the medication it is now 3 months sinnce I last took the medicine Does it mean that i am cured for good? genotype 4 not much is written about it what us the chance of success as compared to genotype 2 nad 3? pl help Doctor: Welcome at HCM i have gone through your query and being your physician I completely understand your health concerns.... Have u stopped inj on doctor advice? In my opinion it should be taken atleast for six months.....as far as PCR is concerned, cure in hcv is defined as by the negative PCR after six months of stopping treatment. This is called SVR. You need to get your hcv PCR after six months.... If it recurs meet your hepatologist and discuss about the sovaldi......As far as other management is concerned you need to get aldactone along with inderal and syp. Lactulose..... Meet your physician for adjustment of dose..Get well soonRegardsDr saad sultan"
},
{
"id": 123064,
"tgt": "How can possible fracture shown on CT scan be confirmed?",
"src": "Patient: I had a MVA in 4/2010. The ER report says possible fracture of C5 by CT scan. I finally had an MRI this march and there is a dark line that lloks like a break in the vertabrae on the MR image. Could that be a fracture that cas seen on the CT scan in ER?? Doctor: Hello, As you had a past history of C5 fracture. What are the symptoms you are having is the first thing to be noted. Based on that we can help assist what can be the measures taken. Usually, the fracture would have healed by now and there might be some healed scar present which might have been noted in the CT scan and that is common. If you have any symptoms like pain which is radiating in nature then using cervical collar will be advised. Simple neck exercise too to strengthen the muscles of the neck. The dark line seen in the CT will be the line of the healed bone. Hope I have answered your query. Let me know if I can assist you further. Regards, Jay Indravadan Patel, Physical Therapist or Physiotherapist"
},
{
"id": 224311,
"tgt": "When must i-pill be taken?",
"src": "Patient: how long does an i-pill take action, say i had sex with my wife on thursday the 23th of may, will the pill still be effective if we had unprotected sex after four days from the date of consumption of the first pill, if not then for how many days does the pill takes its action? Doctor: Hi, I have gone through your question. I pill or the emergency contraceptive pill has its action in preventing only that pregnancy which can occur by the last intercourse episode. It does not have contraceptive effect for the next unprotected intercourse. You need to take a repeat course of i pill for this act.I would suggest you to use regular contraception, as repeated i pill intake have side effects and have increased failure rate.Hope you found the answer helpful.Wishing you good health."
},
{
"id": 154035,
"tgt": "What are the symptoms of prostate cancer?",
"src": "Patient: Hi. I am writing concerning a friend. He was diagnosed with an enlarged prostate some time ago. He now has chronic pain in his tail bone area, lower bak, and hips. He has been running a low grade emperture for a few weeks with some pain in his uper left shoulder area. He has been on several rounds of antibiotics including z-packs and nothing has helped. Are these signs of prostate cancer and if so, how advanced could it be? He refuses to have any real tests done. Doctor: Hi, dearI have gone through your question. I can understand your concern.Symptoms of prostate cancer varies according to stage. In very early stage no any specific symptoms are seen. Fever and weight loss can be there. Later on difficulty in urination, bleeding from urethra can be seen. Back pain may occur due to bone metastasis. He should go for prostatic biopsy. It will give you exact idea. MRI of spine is also necessary to search metastasis or other cause. Consult your doctor and take treatment accordingly.Hope I have answered your question, if you have any doubts then contact me at bit.ly/Drsanghvihardik, I will be happy to answer you.Thanks for using health care magic.Wish you a very good health."
},
{
"id": 146539,
"tgt": "Can bulimia cause bulging discs in the cervical spine?",
"src": "Patient: Hello. I have been diagnosed with bulging discs in the cervical spine and slight cervical osteoarthritis. I alpm wondering if this condition could have been perhaps caused by my bulimia (through the violent vomiting). I have purged on average twice a week for the past 15 years until recently. I am 35. Thank you. Regards. Doctor: Bulging of discs indicates degenerative changes which is a normal phenomenon and don't have yo do anything with bulemia. Hope my advice will help you. Take care. Don't forget to rate me."
},
{
"id": 206513,
"tgt": "Suggest remedy for mental health problem",
"src": "Patient: Hi, I feel I have some problem with my speech. I feel it really difficult to phrase what I want to say I think I have lost the sense of words. This has started to happen after a small house hold accident. I slipped in bathroom and have lost recognition to any one or anything at all for whole night but slowly came out of it. Since then this lost of recognition is with me. it was frequent in earlier days but getting better now, all by itself. i some time lose sense of judgement or decision making or I am listening but actually I do not know what other person saying or forgot about something at all. I am absolutely normal but people could be thinking that I am talking weird, its embarrassing and I think I am making even more mistake and annoy people at my job. I am planning to all this for last 4 months because if I say to my doctor without planning they will say may be its nothing and I will miss important detail to tell them. Doctor: DearWe understand your concernsI went through your details. I suggest you not to worry much. I sincerely suggest that you are victim of your own negative thinking. If you are sure that you will not be able to convince others about your current problem, then possibly you are over doing. In psychiatry, we have clear cut symptoms and definitions to disorders related to mind. If you visit a psychiatrist / psychologist, they will be able to diagnose the problem you are in, just interviewing you. You don't even have to explain all these things. Just don't worry about what others will think about your problem, just approach a capable and reliable trained person and he will treat you well.Psychotherapy techniques should suit your requirement. If you require more of my help in this aspect, Please post a direct question to me in this URL. http://goo.gl/aYW2pR. Make sure that you include every minute details possible. I shall prescribe the needed psychotherapy techniques.Hope this answers your query. Available for further clarifications.Good luck."
},
{
"id": 219278,
"tgt": "What causes difficulty in conceiving a baby?",
"src": "Patient: Hi, my wife is unable to conceive. She was having hypothyroidism and PCOS in right ovary and her periods are also irregular (period cycle is after 2 - 3 months and will last for at least 6 days with heavy bleeding. As consulted by a gyne, she has taken 4 cycles of Herface (contraceptive pills) and she is taking Thyroxin (50mg) for hypothyroidism which has now settled down to normal. Now after 8 months of treatment she is still unable to conceive. Now her doc (gyne) has advised to have an ultrasound of lower abdomen (TVS), below is the report: ULTRA-SOUND LOWER ABDOMEN \u2013 TRANSVAGINAL U/S ALSO DONE. Uterus is anteverted normal in size, shape and outline. Uterus measures 53*68*31mm Myometrial and Endometrial abdomen are normal. Cervix is bulky with multiple nabothian cyst of 4mm. Endometrial thickness is 6.1mm Both ovaries are enlarged in size, shape and echopattern. Evdience of irregularly arranged multiple 6 to 10 mm size follicles. Right Ovary volume : 16 ml, left ovary volume :14ml No adnexal mass is seen. There is no fulid in the cul-de-Sac Impression \u2013 *Bilateral Multicystic Ovarian Changes. *Chorinic Cervicites. Doc also told for an for ovulation test (i am not sure if i have written the correct test name, she(gyne) asked to visit my wife within 24 hours of periods for the test and told that the report will come after 25 days) but she was unable to perform the test (or she was unable to take the sample from her vagina, she told that my wife has a bifurcated septum inside vagina/ovary). I just wanted to be sure that she (gyne) is going tin the right direction. Now after the US report she is suggesting for another 3 cycles (months) of Herface/Meprate (contraceptive pills) which means for 3 months she wont be able to conceive and i am not sure if after 3 moths will it fix the problem or not. Also she has asked to visit after 3 days of the periods for another test (that i don t know) and also asking to go for the 1st test i mentioned(withing 24 hours of periods). Any kind of advise would be highly appreciated. Doctor: Hello and wecome to healthcare magic.From the information given it seems your wife is suffering from PCOS. In this syndrome the main problem is that mature ovum production is hampered and in absense of that the periods become irregular and infertility is seen.If i would have been your doctor i would have gone for a hysterosalpingogram or saline infusion sonography to see whether the fallopian tubes are patent and also to see if there is any structural abnormality of the uterus. All these tests are for your wife.I would have also asked for your semen analysis.If the above tests were favourable then i would have gone for ovulation inducing drugs and given you a schedule of timed intercourse.I could have given a more detailed answer if i knew few details like you and your wive's age, any past history of conception, number of months or years you have been trying for a baby etc.Please do write back if you have any other questions."
},
{
"id": 58770,
"tgt": "Have had gallbladder out and have burning pain there with pain at end of ribcage. Cause for pain?",
"src": "Patient: I have had my gallbladder out but have frequent burning pain where it used to be. Nothing triggers it. It can wake me up at night or come randomly during the day, no pattern. It's not severe, just painful and annoying. I also have pain on my right side at the end of my rib cage and it is tender to touch. Pain is achy feeling. Both usually happen at the same time or alternate. What could be going on? Doctor: Hi Dear Welcome to the Health Care MagicDear friend because you have under gone chloistectomy. Removal of the tissue from the liver bed has left some scar inside your body. Previously it was smooth, now a cavity and rough area is created which is now gradually healing with fibrosis causes all these type of symptoms.Nothing much to worry, gradually with due course of time you will forget the symptoms.Even if still it is bothering you much, you can under go ultra sound scan of the upper abdomen to rule out any abnormality.Hope your query is answeredThanks"
},
{
"id": 194951,
"tgt": "How can erectile dysfunction be treated while under a lot of stress and excessive travelling?",
"src": "Patient: I have had Afib for a few years and it is completely under control. lately, I have had some ED symtems, stressful job, travel a lot etc... Is there an over the counter drug or supplement that could help? I currently take small doses of propafenon, diltizen and Benicar.... Doctor: Hi, I think you should try to reduce your stress first. Try some yoga and meditation. Don't take any over the counter drug as it can be harmful to you. Get yourself examined by a urologist to find the actual cause of erectile dysfunction. Get a proper prescription and then only use a medicine for this."
},
{
"id": 78435,
"tgt": "Suggest treatment for idiopathic pulmonary fibrosis",
"src": "Patient: Dear Doctor, my mother is suffering from IDIOPATHIC PULMONARY FIBROSIS it was diagnosed a few months back through CT Scan, also a blood test of Enzyme Immuno Assay where its value has been at 92.0 level.Kindly advice best treatment and medicine. As doctors have told us that its incurable. Its further growth can be manged somehow. Please HELP Doctor: Thanks for your question on Health Care Magic. I can understand your concern. Honestly speaking, there is no cure for IPF (idiopathic Pulmonary fibrosis). It is chronic progressive fibrosis of the lung. Following things should be done to achieve optimal treatment and reduce the rate of worsening of lung functions. 1. Home oxygen therapy. Low dose oxygen should be given continuously for atleast 16-18 hours a day. This is the best treatment. 2. Start perfinodone. It is the only available and US FDA approved anti fibrotic agent. It reduces rate of fibrosis. 3. N acetyl cysteine is having excellent antioxidant, mucolytic and expectorant properties. And hence it is beneficial in IPF. 4. Enroll her in Pulmonary Rehabilitation center where chest physiotherapy and deep breathing exercises are done. These will surely improve her lung functions. 5. Vaccination with pneumococcal and Influenza vaccines is advisable to prevent recurrent respiratory tract infection. Hope I have solved your query. Wish you good health. Thanks."
},
{
"id": 92906,
"tgt": "Pain in lower stomach, loss of appetite, dry mouth, frequent urination, headache. Any ideas?",
"src": "Patient: Hi,For the past week or so I've had an ache in my lower stomach just above my pubic bone which sometimes changes into sharp pains, I have been suffering from lack of energy, loss of appetite and have felt really thirsty and constantly have a dry mouth, I haven't had any more trouble then usual going to the toilet but have been passing urine more often then usual due to the added amount of water im in taking for my dry mouth. I've also had more discharge then usual and slight headaches? Doctor: Hello, Thanks for the query to H.C.M. Forum. I think you are somewhat confused as the pain on the left side just above pubic area means pain in abdomen not in stomach. Anyhow the possibilities may be of ,Any kidney stones which may present such type of pain with desire of defecation and frequent urination. For this purpose you need to get an ultrasound abdomen and kidney areas . Loss of appetite , lack of energy are due to sharp pain. I suggest you to consult a physician and get an Ultrasound of abdomen (left).Get in blood as well urine tests. Good luck. Dr. HET"
},
{
"id": 116583,
"tgt": "Should cyclosporine be restarted after atgam treatment in aplastic anaemia case?",
"src": "Patient: I am 69 years of age, weight approx. 59 kilos and have had aplastic anemia for about 5 years. I have received the Atgam treatment and have had what is called a partial recovery. I have continued on cyclosporine until about four months ago. My recent blood test shows my platelets as around 50. I have just noticed a very large bruise on my leg. Is this a danger sign? Should I recommence the cyclosporine? Doctor: Hi, dear. I have gone through your question. I can understand your concern. You have aplastic anemia. Your platelet count is reduced with signs of bleeding. You should consult your doctor urgently. You need platelet transfusion first. Then plan for cyclosporin according to your doctor's advice. Hope I have answered your question, if you have doubt then I will be happy to answer. Thanks for using health care magic. Wish you a very good health. Don't forget to click thank you."
},
{
"id": 81544,
"tgt": "Any suggestion for Suspicious densities are noted in left upper lobe ?",
"src": "Patient: Dear Doc, My 16 year old daughter Chest PA Findings: Suspicious densities are noted in the left upper lobe. Suggest apicolordotic view for verification Heart and great vessels are within nornal Other chest structures are not remarkable. Can you tell me what to do, to whom Am I gonna consult .. Doctor: HIWell come to HCMI really appreciate your concern, actually clinical condition is very important, you have not mentioned for what x-ray being done because this need to be correlated with the clinical condition, this may be due to many clinical reasons, unless know the symptoms difficult to say anything, hope this information helps, take care, have a nice day."
},
{
"id": 45723,
"tgt": "Suggest treatment for PKD with shortness of breath",
"src": "Patient: Dear Dr. I'm from Sri Lanka. My mother is 68+ & having PKD. It was found in 79' and was under medication for High Blood Pressure since 85'. She started hemo-dialysing April/09 once a month and at present doing dialysis once a week. But she is continously having disnia. What can be the reason behind it ? What can we do about it ? Is this her final stages ? Doctor: Hi, 68 year old polycystic patient will have by now developed renal failure. As she is on dialysis one can assume her kidney are not functioning to the extent to remove waste products from blood. In addition she will also have high blood pressure and bone pains due to low calcium levels. One option would be to do renal transplant, but the age would not permit. Secondly, you can opt for peritoneal dialysis, provided if doctor deems it right. and Lastly, you have to do Hemodialysis which you currently are doing thrice weekly. You will always be given medications for high blood pressure and calcium supplements. Hope I have answered your query. Let me know if I can assist you further."
},
{
"id": 23292,
"tgt": "Is an IVF safe after an open heart surgery?",
"src": "Patient: I have a history of an open heart surgery back in 1990 Im 42 years old now and been trying to get pregant, so we been seing fertility specialist in Stanford. We tried 4 times to do a surogacy but nothing happen now that my time is limited I want to just carry the baby by IVF is it possible for me to do this? please please please someone can give me an advice. Truly daisy rios Doctor: Hi,.Pregnancy including ivf is safe during pregnancy if the heart is strong enough to bear the strain of pregnancy. If you were operated at a early age it means you have a congenital heart disease. So you need to get a 2D Echo done if not done for a long time. In that we need to see mainly the pulmonary pressure. If that is high getting pregnant is high risk. If normal than you can easily get pregnant & be in regular touch with a cardiologist.So get a echo done before ivf.ThanksDr Sameer Maheshwari"
},
{
"id": 32866,
"tgt": "Is 38.5 degrees Celsius normal body temperature?",
"src": "Patient: hello, at this moment i have a sore throat, cough, a little bit of a headache if i move around too much and a fever. My temperature is 38.5 degrees Celsius. Is this temperature normal for a fever and if it is not can you tell me what i should do to get better? Thank you Doctor: Welcome to HCMAccording to your history provided,In my opinion your are suffering from viral fever.Normal body temperature is between 36.5 to 37.5 degree Celsius.At present you should do following things1.Drink plenty of water2.Take Tab. Wikoryl, three times a day after foor for 3 days3.Steam inhalation ,4-5 times a day4.salt with warm water gargles,4-5 times a day5.Full bed restIf fever doesn't subside after 2 days then you should consult your physician for further work upRegardsemail - o_bodke@yahoo.co.in"
},
{
"id": 121601,
"tgt": "Suggest medications for swollen calf",
"src": "Patient: My left calf is swollen 2 days ago it felt like I had pulled a muscle in the back of my leg. That pain has gone away nowon the inside of the calf there is a spot about the size of a silver dollar that is quite tender painfull. I do have buldging veins. This happened a coulple of months ago went to emergency room they did a sonogram on my legs from crotch to ankle and did not see any clots or other problems. About a week later swelling was gone as was all soreness and ternderness. Any ideas Doctor: Hello, Your symptoms could be related to a sprained muscle or ligament. For this reason, I would recommend using cold local packs and trying local massage with anti-inflammatory ointments. If the pain persists, consulting with an orthopedist would be necessary. Hope I have answered your query. Let me know if I can assist you further. Take care Regards, Dr. Ilir Sharka, Cardiologist"
},
{
"id": 58131,
"tgt": "High urine protein test results. Elevated albumin and creatine levels. Had a colostomy. Are these high levels due to surgery?",
"src": "Patient: Hello, I just got back extremely high urine protein test results. Microalbumin , UR 939 Creatine , UR 120.6 Microalbumin, Normalized to Creat 778.6 The blood test was done shortly after shurgery that involved a colostomy and a suprapubic catheter. I have had significant amount of blood in my urine. I am wondering if the surgery could result in such hight protein readings. Doctor: Hi and welcome to HCM,thank you for your query.Colostomy as any other abdominal procedure may cause elevated these findings but just temporarily. If this persist ,you should check your kidney and liver function and see is there some other disorder. These findings may indicate just uroinfection or kidney stone but there also can be some more serious kidney disease or intraabdominal fistula.Wish you good health. Regards."
},
{
"id": 63254,
"tgt": "How can a forearm lump with swelling and pain be treated?",
"src": "Patient: Hi, may I answer your health queries right now ? Please type your query here...lump on inside forearm above elbow crease . its right on tendon in arm deep inside. i now have pain and swelling in arm and hands. going to see ortho doc in a week. but i would like to get some info now on this so i dont have to worry. Doctor: hi.it could be a cyst, a fibroma or a lipoma based from your description. i suggest you consult with a doctor for complete medical and physical examination. diagnostics (blood exams and imaging) and management (medical or surgical) will be directed accordingly.hope this helps.good day!!~dr.kaye"
},
{
"id": 33946,
"tgt": "How to treat swollen glands around the neck?",
"src": "Patient: hi doctor, my 2 yr old nephew has few swollen glands around his neck. doctors told for montoux test which showed some kind of positive result but was not full proof, so doctors told for some more tests which would finally test the disease. those tests are totally normal yet doctors say that he has been slightly affected by tb virus and to medicate him with tb medicine for 6 months. what to do, pls suggest Doctor: Hello.Tuberculosis is bacterial disease and in past it was great burden for mankind since it was untreateable.Nowdays, there are potent antiobiotics and more and more people are using it.I fully understand your concern, specialy concerning long period of using drugs. But risk of complication of developed disease is far more greater then possiable side effects of drugs.I advise you listen to your doctor.All the best."
},
{
"id": 206229,
"tgt": "What causes hallucinations and emotional outbursts?",
"src": "Patient: My mother is 65 yrs old, she had a stroke back in January of this year. Before that for a good year or more she was constantly getting angry and yelling and screaming. To voices she hears and at time visiually see these people who are not there. She would not go to the doctor before January when she had her stroke but she is now seeing a general doctor but the problem with that is that she will not tell him about these voices she hears. When she was in the hospital the norologist did 2 spinal taps on her which her colestrol and white blood cells were off the charts in the 200's!! Well the norologist went on vacation and within a week my mother was sent home unable to walk from the stroke and now since then is once again walking but seems to drag her right leg a little at times. My question to you is what can be causing my mother to suffer from hearing these voices and getting angry, calm, sad, or crying Roller Coaster Ride of Emotions. She says she knows that something isn't right on the back inside of her head. I will tell you more as we speak as I am hoping this site is real. Doctor: Hi.i had gone through your query.In old age these type of emotional problems are common after stroke or vascular events.Depression or anxiety is associated with stroke in 40 to 50 %cases.Hallucination is symptoms of psychosis and it is associated 10% in patient of stroke.Mood variability or catastrophic reaction is common.Require psychological evaluation and treatment.Low dose antidepressant with anti psychotic will help. but require detail evaluation before taking any medicines.Caution is require for drug drug interaction.Consult psychiatrist and get help.Thank you."
},
{
"id": 80532,
"tgt": "What could be causing the pain on right side of my rib and the swollen vein?",
"src": "Patient: I have two veins that appear to be swollen along the side of my right ribs. I recently had an infection from a breast implant, then remove=al, and yesterday they took out the drain tube. What could be causing the pain on the side of my rib and the swollen vein? Doctor: The pain and the distended vein is due to the infection which was in the area of the implant..The pain and the distended vein will take few weeks to settle after the infection gets cleared.."
},
{
"id": 62983,
"tgt": "What is the tingling sensation in my nipple after I got hit in the head?",
"src": "Patient: I have question I got hit with a refrigerator it hit me hard and at the same time it scrape down so I fell like a hard lump where it hit me, so I went with my doctor and he said he felt something hard but that he didn t feel a lump but he still send me to get ultrasound just to make shure it wasn t nothing bad, so he got the results and everything was normal , but I have two months already since I had that accident and I have been feeling like a tingling or needles sensation something like that in my nipple and its been every day and ill day , you think I should get worried cause I m scared but like everything was normal at the ultrasound what do think it could be thanks. Doctor: Hi, Thanks for query to My HCM virtual Clinic.I studied all details of your query.I understood your health concerns about tingling in your nipple after hit in the head.Cause of tingling in your nipple after hit in the head-After reviewing your query facts,In my opinion these 2 complaints are not related to each other.If any relation ,its the anxiety from correlating these 2 complaints.Reasons for this opinion is -that the lump after the head hit was confirmed as normal after USG study of that lump and 2 months have passed normally.Probable reason for tingling in the breast needs to be ruled out for which I would suggest Second Opinion from Surgeon,who would physically check the nipple part and would treat it accordingly.Still if you feel obsessed,I would suggest CT Skull and opinion from NeuroSurgeon.I thinks you should not worry till the CT studies are done.Wait with a cool head and act according to above suggestions.Hope this reply would help you in resolving this complex query and anxiety with it.Don't forget to hit thanks and write excellent review comments to help needy visitors at HCM.Welcome for any further query in this regards.Dr.Savaskar M.N.M.S.Genl- CVTS.Senior Surgical Specialist."
},
{
"id": 89165,
"tgt": "What causes burning pain and knot in abdomen?",
"src": "Patient: I experienced a knot in my abdomen, about an inch above my navel, causing a burning pain. This occurred while on my 3mile daily walk. The knot is palpable when I stand , but softens and can t be felt when I sit or lie down. The burning has subsided and I have no other symptoms at this point. This question is for a 67 year old male. Doctor: hello welcome to HCMit may just be a temporary swelling in your abdomen.as it is not causing you any problems, do not be much bothered about it.if it does not regress in few days, then do visit a surgeon"
},
{
"id": 2604,
"tgt": "How long will it take for me to get pregnant?",
"src": "Patient: Hi, I was 432lbs 20 months ago. I have completely changed my health, eating and exercise habits and have lost 159lbs. I have Poly cystic kidney disease and have been trying to get pregnant for 8 years with no success. I am 33 years old. I have recently started having normal periods again every month. How long will it take for me to get pregnant? Doctor: Hi,I think there is no definite time by which one can be told to get pregnant. In your case I think you have already tried for 8 years. So now you should take some medicinal help to conceive. You can take some medicines for growth of your eggs and also some medicines to support your pregnancy. Also do some tests like thyroid profile and prolactin and an ultrasound for your follicles. Also your husband should go for a semen analysis.Hope I have answered your query. Let me know if I can assist you further. Regards,Dr. Khushboo Priya"
},
{
"id": 196178,
"tgt": "Suggest remedy for burning in prostate area and pain in front of penis",
"src": "Patient: my urologist has prescribed alfoo 10 mg OD and urimax 0.4mg OD.i have very low flow and hesitancy in passing urine. i was operated for prostate for the second time 3 yrs back. i have burning in the prostate area and pain in the front of penis. prostate is normal in size and psa is 0.9. pls advise. Doctor: your urologist have given you the correct medications.kindly drink more water.it reduces the burning sensation and helps you in passing urine freely."
},
{
"id": 115306,
"tgt": "Should i go for repeated blood tests if having an enlarged spleen?",
"src": "Patient: 2 years ago my spleen showed up enlarged 14cm, my blood was ok just my white blood count was a liitle low but still within the norm. I have been under observation since then going for blood test and ultrasound. As off 6 months ago my spleen went down to 12.7cm and my blood is ok according to my hematologist. Should I be getting additional testing or is this method sufficient. I have another appointment scheduled in a couple of weeks to check spleen and blood again. Doctor: Hi, dear. I have gone through your question. I can understand your concern. Your spleen is enlarged. It may be due to your infection. You are going on right tract. Just monitoring of spleen size and blood count is sufficient. If you don't recover then you should go for bone marrow examination. Hope I have answered your question, if you have doubt then I will be happy to answer. Thanks for using health care magic. Wish you a very good health."
},
{
"id": 88097,
"tgt": "What causes constant lower right abdominal pain?",
"src": "Patient: I have had constant pain in lower right abdomen area for the past three months - the pain varies from tolerable to intolerable and depends upon stress and other factors I do not know b.c. i do experience the pain when I am in a low-stress environment. Please also note that I look tired all the time even if I get sleep [pronounced dark circles]. Everyone always tells me I look like crap even if I feel ok at that moment [i.e. the pain is dull or at a tolerable level -mild discomfort is the best it gets]. I have had the following images taken: 1) CT 2) ultrasound. I have had blood work and urine work done more than once. The images show two kidney stones in right side. But, the urologist is CERTAIN that the stones are NOT causing my pain, but he does not know why I have this pain, the source of this pain or how I got this pain. The urologist referred me to the Gastro doctor - we did blood work and urine - ultrasound - took the RX Prilosec ---reviewed the CT -- Gastro says everything [all anatomy depicted in the full abdominal ultrasound] is fine and that he does NOT know why I have this pain - Gastro says next step is to look inside stomach or do a endoscopy on both ends but that he says highly highly unlikely that the endo will shed any light on this right side abdominal pain that is definitely NOT muscular or tied to my obliques. So, here I am, still with this pain and left without answers. I am going for yet another opinion and peak next week at a Cedars Sinai Gastro. Please, any ideas what this pain is due to and why I have it, what it is etc? Also, what should I do at this point. Again, assume urologist is correct in that my stones are NOT at this point lodged in a manner to cause the lower-right ab pain I speak of. WHAT IS THIS?????? Thank you in advance. Doctor: Hi ! Good morning. I am Dr Shareef answering your query.As you have not mentioned anything about the size of the stones in kidney, it might not be possible to opine if they were the culprits for your pain or not. Other investigations being normal, if I were your doctor, I would advise you to stop eating from out side if you do, as many components of food from outside, specially the fast food might contain some ingredients to which your intestines might be more sensitive and give rise to such symptoms. I would advise you to add more of roughage and enough liquids in your diet,and to avoid any alcoholic beverages and smoking if you do. In addition, I would advise you to go for a routine test of your stool for ova and cyst, which when treated surprisingly cure many of the chronic abdominal pain. I am sure your clinician must have examined your abdomen clinically to rule out chances of any recurrent appendicitis. Also I would go for a routine/microscopic and culture sensitivity test for urine to rule out a silent UTI which is not uncommon. In case of female patients, a PID (pelvic inflammatory disease have to be ruled out) by a gynaecologist. Till then, you could go for some anti spasmodic drug and a proton pump inhibitor and a probiotic drug for a symptomatic relief.I hope this information would help you in discussing with your family physician/treating doctor in further management of your problem. Please do not hesitate to ask in case of any further doubts.Thanks for choosing health care magic to clear doubts on your health problems. I wish you an early recovery. Dr Shareef"
},
{
"id": 146633,
"tgt": "What causes numbness in feet?",
"src": "Patient: I feel a numbness in both of my feet. Had nerve conduction test with little to no results. Then on to a neurologist who did 2 MRI only to find that I have a disc pressing in on my spinal cord. (I have a spot just underneath where my bra connects about the size od a lemon that is also numb). I have 2 degenarate discs as well in a higher area of my back. To add to this, I now feel as though my legs get cold internally, and it feels as though I m walking with a tennis ball taped to the ball go my right foot. The neurologist said he would not touch me due to risk of being paralyzed . I have a high tolerance for pain, but my foot is not tolerable. Which type of specialist such I seek to help with this? By the way I m not diabetic. Doctor: Considering history and anesthetic patch over chest we should rule out leprosy. Probably you are having small fibre neuropathy in which ncs nay be normal. Try tab pregalin 75 mg once dailyHope my answer will help you. Take care. Don't forget to rate me."
},
{
"id": 50903,
"tgt": "Child with Vesicoureteral Reflux (VUR) suffers kidney damage, scar. Can kidney be normal without surgery as child grows?",
"src": "Patient: My son has been diagonised with unilateral grade 3-4 VUR & his left kidnsy has scarred , its 36 % working.now he is on antibiotics .should we need surgery or will he outgrew it ? he had UTI at 6 months old.now he is 2 yrs 3 months.Is there any chance of resolving it.Will the affected kidney shrink again if there is no infection hereafter Doctor: Hello and welcome toHCM In vesicoureteral reflux the direction of urine flow is reversed.Vesicoureteral reflux (VUR) is an abnormal movement of urine from the bladder into ureters or kidneys It is classified into Five grades depending on the severity. Grade I \u2013Grade V. Medical treatment is low dose antibiotic prophylaxis until resolution of VUR occurs. The goal of treatment is to minimize infections, as it is the infection that cause renal scarring and not the vesicoureteral reflux. Medical management is recommended in children with Grade I-III VUR as most cases will resolve spontaneously. A trial of medical treatment is indicated in patients with Grade IV VUR especially in younger patients or those with unilateral disease. For Grade V VUR surgery is the only option. When medical management fails to prevent recurrent urinary tract infections, or if the kidneys show progressive renal scaring then surgery may be necessary. Surgery is necessary in cases where infection recurs despite prophylaxis with antibiotics, if the VUR is severe (Grade IV & V)and if there is failure of renal growth, formation of new scars and renal deterioration. Since your child had only 1 episode of UTI and he is on regular prophylaxis we can continue with medical management .Urine cultures should be performed every 3 monthly to exclude any new infection. Get a repeat radiological examination to see if the working of kidney function is deteriorating. Good perineal hygiene, and timed and double voiding should be done to decrease the reflux. But still i will suggest you to consult a good urologist to help take a wise and timely decision for surgery. Hope I have answered all your queries. wish you good health for your son."
},
{
"id": 165999,
"tgt": "What causes 5-year-old to be clumsy and have lack of concentration?",
"src": "Patient: My daughter is 5 and her teacher has noticed that she is very clumsy i too noticed this for about 2 years and thought it was because she was still developing her developmental skills. Today her teacher advised me to see a doctor about her she's noticed also that my daughter cannot sit and consentrate and does not like writing. I can't wait till friday (her appointment day).Can you please tell me what could be wrong with her i'm really worried. Doctor: This may b due to Attention deficit hyperactive syndrome or Tourette syndrome. Thyroid hormone abnormalities may also cause these problems. so you should consult with pediatric development neurologist."
},
{
"id": 28011,
"tgt": "Can Olmezest Beta be stopped after BP comes under control?",
"src": "Patient: Hi I am 27 years old and diagnsoed with hypertension having b.p. at 150/110 usually. I have been prescribed Olmezest Beta 50, I want to know if there is a possibility of stopping the medicine in futute once the blood pressure is under control.ThanksAnupam Doctor: Ideally antihypertensive medicines must never be stopped.but as you are 27 you must undergo investigations to find out any cause suggesting you may secondary hypertension.if u find cause then first address the removeal of cause and then it may be first reduced and then stopped.if no cause found then it should be considered primary HT starting at young age and u must continue medicine life long."
},
{
"id": 14923,
"tgt": "Cause for rash on both sides of face along with itching?",
"src": "Patient: out of the blue I have a rash on both sides of my face has circular shapes and colored inside circle.itchy.ijust had hernia surgery 3 weeks ago..rash is spotty.some of the rash doesn t have circles.not sure if the percoset has anything to do with this?i have looked at pictures on line and cant seem to match it up. Doctor: Hi. Thanks for posting your concern.Judging from the description it seems to be an Urticarial drug rash.If i was your treating doctor i would have put you on oral antihistamines like cetrizine or loratadine once or twice daily for a week.Depending on the severity of rash you might require a shot of hydrocortisone OR Pheniramine which can be repeated every 6 hours.Topical soothing lotion e.g calamine will help sooth the itching and discomfort associated with it.regards"
},
{
"id": 61656,
"tgt": "What causes lump in armpit?",
"src": "Patient: hai Doctor, Am biju 24 years ols male. for the past few months i have some stone like substance on my left arm pit. and this substance grows and reduce alternatively. and there is no pain for some days. and pain for some days. what could be the reason . and any remedies Doctor: HiWelcome.I have gone through your query.Based on the information, I feel it might be folliculitis or subeceous cyst. But it need to examined to confirm the diagnosis.The treatment depends on the diagnosis.I would suggest you to see general surgeon for examination and further help.Take care."
},
{
"id": 193343,
"tgt": "Does masturbation cause health problem?",
"src": "Patient: Hi, may I answer your health queries right now ? Please type your query here...Hello sir I am from chennai in India, I am married man i hanebeev married about 22 years my age is 46. lase 2 year I didnot sex with my wife, but i used my self is it any problem about my health? Doctor: Hello Masturbation does not cause any sort of health problems. However in some instances excessive masturbation can lead to lower sperm count with each shot. I hope i have answered your query."
},
{
"id": 72586,
"tgt": "Suggest treatment for chest discomfort with shortness of breath",
"src": "Patient: Hi, i have been experiencing chest discomfort for the past few weeks. It was as if something is pressing onto my chest area. it would ocasionally make me feel breadthless and pain as well. i have been coughing for the past one year already. May i know what is the cause of it? Could it be some symtoms of lung cancer? Thanks. Doctor: Thanks for your question on Healthcare Magic.I can understand your concern. Chronic cough (cough since 1 year), breathing difficulty, chest pressure, pain etc are more suggestive of lung diseases like bronchitis, lung infection (pneumonia or tuberculosis) and lung cancer.So better to consult pulmonologist and get done chest x ray and PFT (Pulmonary Function Test).Chest x ray is must for the diagnosis of lung infection and lung cancer. PFT is needed for bronchitis. You may need antibiotics, inhaled bronchodilators and inhaled corticosteroid (ICS) based on the reports.If you are smoker then you should quit smoking as soon as possible. Hope I have solved your query. I will be happy to help you further. Wish you good health. Thanks."
},
{
"id": 29661,
"tgt": "Is Valacyclovir an effective medication for shingles?",
"src": "Patient: I have had four outbreaks of shingles this past year. I'm on an antiviral (ValACYclovir-1 gm/day) now to prevent outbreaks. What are the signs I should look for as far as adverse effects of this medicine? And, have you heard of anyone having this many outbreaks? They are on my face or in my ears, usually. Doctor: Hello,Thanks for trusting Healthcaremagic!I have gone through your query and I can understand your concerns.Valacyclovir more effective than acyclovir. Side effects are very minimal.You can go for valacyclovir with out any fear.Thanks"
},
{
"id": 12724,
"tgt": "Suggest remedy for rashes",
"src": "Patient: I,v had a rash since monday and has been getting worse since then, I got some cream, Benadryl. and some tablets, Piriton. I thought I mite be allergeted to something, But last night I had to call my husdand home from work because, I woke up at 4am very swetty the bedroom was spinning and I could not walk, I cralled to the sofa and rang my husdand, whan he got home I was sick, in a bowl very much, He had to go back to work, so I sat on the sofa and must of fallen a sleep, So today I just fill dizzy, lite headed, and a bit drunk. The rash is still making me very itchy. can you help. Thank you, From Debbie Doctor: Hi,The symptoms that you have described can be seen as a side effect of using Benadryl and Piriton. These are antihistamine drugs and they commonly cause:* Dizziness* Drowsiness* and PalpitationsThese effects are increased when one has consumed alcohol along with the medications as well.Some things that you can try to help reduce the itching can be Icing the area locally to numb the receptors for itching. Using a cool lotion like calamine can help as well. In case these symptoms persist, please visit a doctor to get the right dosage of medications to help. Also, avoid taking drugs from the same class of medications.Hope I have answered your question. Let me know if I can assist you further. Regards, Dr. Vignan Rachabattuni, General & Family Physician"
},
{
"id": 161488,
"tgt": "What causes white patch on cheeks of a child?",
"src": "Patient: Hi, may I answer your health queries right now ? Please type your query here...my daughter is seven and ahalf yrs old.She has developed a small white patch on the cheeks.Can u tell us the cause and remedy for this?Does it have any correlation to worms in the stomach,as it is more than a year that i have dewormed her? Doctor: Hello, Most commonly white patches on the face are due to a much earlier dryness or redness on the face, due to atopy or simply dry skin. It can be prevented by using moisturizing cream regularly. It's not due to worms. Hope I have answered your query. Let me know if I can assist you further. Take care Regards, Dr Cajetan, Pediatrician"
},
{
"id": 136960,
"tgt": "What causes pain on fracture side when i moved from walker to tripod?",
"src": "Patient: I have several pelvic fractures and an L4 fracture. None required surgery. Since I have graduated from walker to tripod I seem to be having a lot of hip pain on same side. Is this normal? Also after all most five weeks out still hard to lay on my side. Is that normal Doctor: Hello, I have studied your case. As per your history there is possibility of arthritis post fracture.You may need to do X ray and blood investigation again.Consult orthopaedic surgeon for further investigation and treatment.Tripod will lead to excess weight on fractured bone leading to more pain.This may not be normal for healing of your fracture.Consult your doctor again.Hope this answers your query. If you have additional questions or follow up queries then please do not hesitate in writing to us. I will be happy to answer your queries. Wishing you good health.Take care."
},
{
"id": 9918,
"tgt": "Suggest remedy for hair loss",
"src": "Patient: hello doctor, i am swetha 19 years old. i am having continuous hairfall from last one year when i am combing or shampooing my hair. i also found that my lost hair has white follicle at the tip of hte root. please kindly suggest me something to stop my hair loss. waiting for your reply doctor. thank you. Doctor: Hi, Understanding your concern. As per your query you have symptoms of hair loss which could be due to improper hygiene of scalp, nutritional deficiencies, dandruff, stress and clogging of skin pores. Need not to worry. I would suggest you apply Minoxidil on hair follicles and take Finasteride preparations as well for hair loss. You should do oiling of scalp regularly to avoid dandruff. I would suggest you to try to reduce your stress. You should consult dermatologist once and get it examined. Start treatment after proper prescription. Avoid taking oily, fried and spicy food. You should keep your hair massaged to increase blood circulation. Hope I have answered your query. Let me know if I can assist you further. Take care Regards, Dr. Harry Maheshwari, Dentist"
},
{
"id": 48764,
"tgt": "How to treat kidney stones?",
"src": "Patient: Liver transplant 11 years ago - colon removed 9 years ago - no appendix no gall bladder - labs done every month - last month all ok - labs drawn on Monday - liver numbers elevated - bilirubin 7.5 enzymes elevated - no fever - no chills - a little jaundice in eyes - just finished an ultra-sound - does not seem to be an obstruction - but quite a few kidney stones - what do you think Doctor: Hi,Thanks for asking.Based on your query, my opinion is as follows.1. Possible hepatitis. Require a hepatitis liver serological study to rule out any viral causes.2. Kidney stones management depends on the size. If smaller, large amount of fluids can be useful. If large and in kidney, might require surgery.3. Take plenty of fluids. Get yourself evaluated for possible hepatitis. Hope it helps.Any further queries, happy to help again."
},
{
"id": 183973,
"tgt": "What causes pain in lower gums?",
"src": "Patient: My gums hurt in one spot in the front bottom of my teeth and I dip but that's not were I put my dip what could it be it started hurting yesterday evening and hurt more when I got up this morning and I'm going to see if it hurts in the morning tomorrow Doctor: Hello, thank you for consulting with healthcaremagic. This pain which you are mentioning can be because of two reasons-first it can be possible that one of your teeth is infected in the same region and now the infection has spread to the apex-second it can be possible that your gums might be infected. To get it confirmed you have to visit a good dentist and get an x - ray of the region. As it will show the infection site. Hope it will help you."
},
{
"id": 70028,
"tgt": "What causes hard tender lumps?",
"src": "Patient: My boyfriend found 2 lumps about 6 months ago and was told by one doctor it was just a cyst and it would go away. Well, they haven't, they've gotten harder and are tender at times. He is scheduled to meet with a surgeon in a couple weeks to see what they think. Could it be cancer? Doctor: Hi,Thanks for writing to HCM.At this age, we usually dont think of cancer, so you also take it out of your mind.Show it to the surgeon as clinical examination is the best investigation in this case. He may ask for a needle biopsy of them if he feels the need of it and finall go for their excision.RegardsDr. Ashish Verma"
},
{
"id": 165516,
"tgt": "What can cause severe headache in kids?",
"src": "Patient: My 6 yrs old Son got send home from school today, he said his head hurt. So I gave him tynelol for the pain.. he was fine after that. He just wrote up from sleeping screaming my name and said his hurt REALY hurt. I turned the light on and he grabbed his head and said it hurts I turned it off and he puked a little bit. What can this be??. Doctor: Hello and Welcome to \u2018Ask A Doctor\u2019 service.I have reviewed your query and here is my advice.Headache in kids need to be considered serious only when the below symptoms are there: 1. Vomiting continuously2. Seizures/fits3. Watery or bloody discharge from ears and nose4. Unconsciousness5. Altered sensorium or behavior. Without the above red flag symptoms, just feeling low energy levels points out toward migraine and other sorts of tension headaches which are on the rise in current scenario due to excessive academic and peer stress the kids are experiencing.I have a few questions for you.1. How long has the headache been there?2. Does it occur always on forehead or keeps changing?3. Is it associated with blurring of eyes or double vision or giddiness?4. Is there any family history of migraine or single side headache?5. Is he very angry when he gets headache and does the ache get better after a good sleep?6. Does he vomit when there is headache?Kindly get back to me with the answers to above questions and also mention his weight so that I can suggest you the formulations which are good. But as of now, you can give oral Paracetamol.Hope I have answered your query. Let me know if I can assist you further.Regards,Dr. Sumanth Amperayani"
},
{
"id": 186657,
"tgt": "What does discoloration inside cheek indicate?",
"src": "Patient: Hi, I have a discoloration in the inside of my cheek. It is darker in color. It was noticed by my dentist who has referred to get it checked. I have had this for about 5 years and in fact a different dentist commented on it around two to three years ago but didn't think it was anything to worry about. However I am really concerned because of the referral. Doctor: thanks for your query, the discoloration could be because of physiological pigmentation or fordyce spots or arterovenous malformation consult your oral physician and get it diagnosed properly.. i hope my answer will help you..take care"
},
{
"id": 151240,
"tgt": "Severe nerve pain in backbone, body pain. Surgery prescribed. Alternative treatment?",
"src": "Patient: Dear Dr. My father s age is 55+ and is suffuring from sever nervouse pain at the backbone side which creates other nerves pain in the body all the time.we have consulted with the several doctors but all have suggested for the surgery but its too risky as suggested by others dr. as well. kindly suggest thanking you Doctor: Hello Raju and thanks I understand your problem but if all medical therapy fails than we have no option other than surgery. It is not so risky if done by a qualified spine surgeon. If you are scared from surgery than follow some instructions that helps you in maintaining painless spine: 1. Do alternate hot and cold fermentation. 2. Use hard bed to sleep, avoid pillow. 3. Can apply anti-inflammatory gel gently to reduce pain. Use analgesic drugs in worse condition. 4. Do regular physiotherapy and exercise by consulting you physiotherapist. 5. Avoid all long term sitting, standing and driving and wear Lumbrosacral belt. 6. Avoid jerky movements of back. 7. Complete bed rest is best treatment in worse condition. Hope this will help you and if this fails than go for surgery. Do not feel scared as every surgery has its own risk. This surgery has as much risk as other general surgery has. Best Wishes"
},
{
"id": 193550,
"tgt": "What causes pain in testicles?",
"src": "Patient: hello doctor, i am 42 years old male..in my scrotum,right side of the testicles a small boll is (1 cm lenth x 0.5 cm width)there,,,some times its giving little pain,,,also when i am going long drive also little pain has come.. kindly tell me the solutions..regards Doctor: Hi, It can be due to abscess. Share a picture for conformation. The local antibiotics can help you. Hope I have answered your query. Let me know if I can assist you further. Take care Regards, Dr S.R.Raveendran, Sexologist"
},
{
"id": 30500,
"tgt": "What causes amoebic appendicitis?",
"src": "Patient: My nephew aged 28 is diagnosed with amoebic appendicitis in the liver. He was admitted in the hospital for 1 week and still has pain on his right side below the rib cage. Whom do we contact and what is the cause of this appendicitis? Thanks in advance. Doctor: Hello and thank you for your question.First you need to get clarification of the diagnosis. The liver and the appendix are two different organ so you can not have appendicitis of the liver. You can have amoibic infections in both organs however. If this is not being treated adequately then he should see an infectious disease specialist for further testing and treatment.Regards"
},
{
"id": 217915,
"tgt": "Experiencing severe pain at lower left & back of the body",
"src": "Patient: severe pain lower left front and lower back on left side of both making it difficult to walk in pain every morning trying to get up must sit at edge of bed for 45 minutes before getting up to get in shower. As day goes on more I walk the pain eases but once I sit it is difficult to get up and pain increases at night Doctor: Ucan pls go for mri scan for ur lumbar region, this sort of pain cud be due to sum prolapse or herniated disc, or compression. Pls meet an orthopaedic surgeon or neurosurgeon, wid gud physiotherapy relaxation exercises."
},
{
"id": 103286,
"tgt": "Allergies/sinus problems, ear pain. Taken Augmentin and antibiotic. Ear pain persists. Consult ENT?",
"src": "Patient: Hi. I have a family history of allergies/sinus problems as well as a tendency - the past few years - to clench my teeth while sleeping. This makes me a TMJ candidate. As a consequence, when I developed an on again, off again ear pain in early June, I did not rush to the ENT. I did not seek any medicine. Last week however, I saw the dentist about a tooth on the same side as the ear pain (my lymph node was also briefly swollen under that ear ), he advised me to see an ENT to cover all bases. I was then given Augmentin for any that infection may have not been visible on the dental x-ray near the tooth (it has a crown). My ear does not feel particularly bad since I started the antibiotic. The lymph node swelling went down a few days ago after that visit and as of now my external auditory meatus feels just a bit crusty but is not particularly painful. Is it imperative that I see an ENT to determine for certain whether I have been suffering from an off again on again ear infection since June or should I wait to see how I feel after I finish the antiobiotics? Doctor: Hello dear,First of all, the antibiotics prescribed by your doctor is correct.Please take them as advised.However, if the symptoms still persist, it will be better to consult an ENT specialist for further evaluation & management.Investigations like complete blood count, X ray of para-nasal sinuses will be helpful in clinching the diagnosis.Treatment usually involves:1. Analgesics like paracetamol2. Antibiotics depending upon culture & sensitivity reports to remove the causative organism.3. Antihistamine preparations can also be used to provide symptomatic relief.Meanwhile...avoid taking cold food & drinks, maintain adequate hydration & take soft, non-spicy foods.Wish you good health & a speedy recovery.Thanks & take care."
},
{
"id": 141272,
"tgt": "Is a swelling on the right side of the head a concern?",
"src": "Patient: My daughter in law has had a history of tension since age 14, she says. She is experiencing swelling on the right side of her head. Should she be concerned enough to go to Cleveland Clinic. She saw multiple dr s this time and the neurologist concluded that he has never seen this before and referred her to Cleveland Clinic. I believe she had this happen before (chronic). Doctor: Hello and Welcome to \u2018Ask A Doctor\u2019 service. I have reviewed your query and here is my advice. Any swelling over the head needs to be investigated. This would require clinical examination, followed by a scan of the affected area. Further treatment would depend on the findings of the above. Hope I have answered your query. Let me know if I can assist you further."
},
{
"id": 88886,
"tgt": "What causes right abdominal pain with urination pain?",
"src": "Patient: My husband was experiencing pain in right abdominal area last week for several days. It became so severe that he went to the ER in Denver during his business trip. He was worked up for appendicitis (CAT scan and blood work.) They released him with no diagnosis. Today he is complaining of pain upon urination with no blood present. Any ideas? Doctor: Hi,From history it seems that he might be having appendicular pain or urinary tract infection with renal colic.Go for routine urine check up.You might require one course of antibiotic and some antispasmodic medicines.Take plenty of water.Ok and take care."
},
{
"id": 23228,
"tgt": "How to treat chest pain?",
"src": "Patient: Hello, I have been having chest pain for months now. I have been working with a cardiologist who gave me a clean bill of health. Everything was done short of a catheter. I exercise almost daily and was in the 98th percentile for my age, which is 44. In any case while working with the cardiologist I developed a blood clot that was superficial. I h ad a pain develop in my back about a month ago. It was excruciating and went up to my jaw and down the left arm. I could not event chew gum it hurt so bad. Saw my family doc again. He did a d-dimer test which came back as .9 where normal is .4. I am now occasionally coughing up small amounts of blood. I am sweating profusely from time to time and have a general fog or dizziness where I feel that I am favoring my left side. Any thoughts? Doctor: I highly suggest you revisit the doctor for formal testing with Chest-X ray and Blood Work. This sounds as if it were a potential atherosclerotic like disease of the central arteries, with some blockage leading to the symptoms described. I would also rule out pneumonia and or infection of the chest. Note that D-Dimer is very non-specific and can be raised for a number of reasons. Nonetheless coughing up blood, profuse swatting and the unilateral fogginess and dizziness are symptoms that in and of themselves would would warrant formal workup and ADMISSION TO THE HOSPITAL. I wish you the best of luck."
},
{
"id": 9904,
"tgt": "Does masturbation lead to hair fall?",
"src": "Patient: Till Age20 I had not much hairfall... But after I started mastrubating twice or thrice a week, After 1 year, I got baldness in the front part of the head. Is that because of mastrubation or any other reason for huge hairfall... This huge loss is in the span of just 1 year. I am terribly depressed. Doctor: Hello, I have gone through your query and your hair loss is not associated with masturbation. It could be hormonal, hereditary or stress related. Hope I have answered your query. Let me know if I can assist you further. Take care Regards, Dr Asmeet Kaur Sawhney, Dermatologist"
},
{
"id": 45417,
"tgt": "I am a male. I have no problem at all since unfortunately krimson-35 is taken by me today",
"src": "Patient: I am a male. I have no problem at all since unfortunately krimson-35 is taken by me today.it is my mistake.i know that krimson-35 is not for a man.then what to do ? please help me out. Doctor: If you haveswallowed only one tab, then there is nothing to worry. You may experience some side effects of this tab like headache, nausea, vomiting. These symptoms would av wane away by itself by now I guess. If you continue repeating this little accidents, then my friend, you might have get new clothes- female wears!"
},
{
"id": 141125,
"tgt": "Suggest treatment for mental deterioration in a bedridden patient",
"src": "Patient: Hello, my name is Vincent and I have questions concerning my grandmother. Recently she went to the hospital for weakness and dehydration. Then to a rehab center to help heal the pain in her leg. But her visit at the center effected her physically and mentally. A month ago on the day of the hospital visit she could still walk and her mind was right. Now she s completely bed ridden and unresponsive. As in now we would say something to her and sometimes she ll look and acknowledge you, and sometimes not. My concern is will she mentally improve, or better yet what s causing the change? We were thinking it s her medicine causing her to behave this was, but I m too afraid to cut her off anything. Even if it s as needed. Sorry if this was too long. I m just starting to worry. Thank you Doctor: Hi, One condition that may have caused such issues is stroke. So, a neurological evaluation and brain MRI are necessary to evaluate correctly this possibility. If stroke will be excluded, then rapidly developing dementia may be the condition that is causing such issues. Discuss with your grandmother's doctor about these issues. Hope I have answered your query. Let me know if I can assist you further."
},
{
"id": 210677,
"tgt": "Symptoms of panic attacks and breathing difficulty persist after stopping phentermine for weight loss",
"src": "Patient: My wife started phentermine for weight loss about a week ago and now she is experiencing panic attacks and having a spells with difficulty breathing. She stopped taking the medicine but the symptoms are still lingering mildly. What can she do to ease the occurrences or make them cease altogether. Doctor: HIThanks for using healthcare magicPhentermine is a psycho-stimulant drug and it cause anxiety symptoms. If she is not getting relief after stopping it, then give her low dose benzodiazepine after physician consultation. In benzodiazepine, she could take clonazepam that would decrease the anxiety, but for that you have to consult doctor. This drug is not available over the counter.Thanks"
},
{
"id": 189558,
"tgt": "Having TMJ syndrome. Gums swollen, soreness behind front two teeth. Related to TMJ?",
"src": "Patient: Good morning Dr. Grief, Question, I have TMJ Syndrome and I was wondering was this associated with the the other day I woke up and my top gums were swollen? They went down bout a day later but right behind the front two teeth is VERY painfull and soar. Makes it hard to eat, almost feels like a cut or something. Do you have any idea what that is? Doctor: hello there , TMJ problems can be associated primarily with the pain and soreness in the upper front teeth and swollen gums as you have mentioned . Swelling in the gums indicate inflammation of the gums known as gingivitis , it is caused due to poor oral hygiene , deposition of bacterial plaque and calculus deposits deep in the gum pockets and on the root surfaces of the teeth. you may also experience bleeding gums , pain , sensitivity and redness in the gums . This needs professional scaling and root planing by a dentist to remove all the irritants ,plaque and calculus which will subside the gum swelling rendering good oral health. Pain in the upper front teeth is may be due to some dental infection or decay process , it can be confirmed by taking a X ray examination of the area accordingly treatment can be planned. i hope this helps , take care."
},
{
"id": 190961,
"tgt": "Can i have amoxicillin to treat my infected wisdom tooth ?",
"src": "Patient: I suspect my wisdom tooth is infected. Would you recommend Amoxicillin to treat this? This is all I am able to get right now. I do not have insurance so I can t just go get whatever I want. What are your opinions on Amoxicillin? Thanks. Doctor: Hi,thanks for query.Amoxycilin is a good and effective antibiotic.It has been used widely by dentist in various infections.In case of infection after proper diagnosis 5-7 days treatment is needed.Please get in touch with dentist in case of problem.Take light diet ,try to avoid injury to that part.bye."
},
{
"id": 171110,
"tgt": "Is it to be concerned about dry blood in child's nostril?",
"src": "Patient: My 13 year old son's nostril constantly have dried blood and it looks gross. I have him clean his nostrils and moisturize with vaseline, we use the cool mist humidifier. The condition only goes away when he takes antibiotics, but I do not want him on antibiotics constantly. He has been tested for allergies and gets allergy shots. This all started 2 years ago when a dr prescribed a nose spray Doctor: Hi,Welcome to Hcm,I can't boil down to a specific problem in your child. However, I get a feeling that you are over treating him probably for something and that's what is causing this issue. Some local applicants in the nose can cause irritation over the skin of the nostril which can cause bleeing at the tip. I would suggest you to consult an ent specialist so that a proper examination of the nose could be done to rule out any local problem. Take care."
},
{
"id": 158666,
"tgt": "Mole on cheek with pimple under it. Pricked bleeding continually. Family history of skin cancer present. Cancer?",
"src": "Patient: I have a mole on my cheek, and I had a pimple under it. I picked the pimple, and it took forever to stop it from bleeding. But it just keeps scanning over, and my mom is keeping a close eye on it. She has skin cancer and same with her dad, and 2 of her brother, so I think she might be keeping a close eye on it for that reason. Is it possible that it is skin cancer? Doctor: Hi,In your description you say that the pimple is under the mole; if it is distinctly separate from the mole with a clear skin in between, you should not worry much. However you should know the changes of a mole turning into a melanoma and watch for them; they include sudden change in shape, size and colour of the mole, itching of the mole, bleeding from the mole etc. However, if you have a doubt better to visit a doctor and getting it cleared. Regards,Sreenivasan"
},
{
"id": 103187,
"tgt": "On and off swollen lips moving from side to side. Swollen face, itchy welts on head after taking Benadryl. Allegic reaction?",
"src": "Patient: I recently had a weird experience, like an allergic reaction, to my lip that swelled up overnight for no apparent reason.I am 69 and healthy. I have been traveling to visit friends and then once again I had a swollen lip that moved from side to side??? Then I took a benadryl and then I had itchy welts on my head.....today better, but still swollen in my face. Teeth are fine? Doctor: Hello,Thanks for posting your query.Swollen lips as well as face can occur usually due to any infected tooth underlying.The swelling may not be associated with allergic reaction of the drug administered.Get a thorough evaluation by a dentist.Infected tooth has to be root canal treated or extracted depending on the severity of infection.Usually antibiotics/antihistamines has to be administered.Discontinue use of Benadryl and get it replaced.Hope i have answered your query.Take care."
},
{
"id": 20380,
"tgt": "What does this ECG test result indicate?",
"src": "Patient: Hello I am 28 years old I want to show my Ecg results could you help me to know if these are normal? I would appreciate it greatly ! Vent Rate - 106 BPM ,PR - 157 ms,P/QRS/T 130 71 178 ms,QT/QTc - 322 429 ms,p/QRS/T-54 44 39 deg,RV1/SV5-0.12/0.00mV,RV5/SV1-1.52/0.87mV.Is It Normal ? Thanks Doctor: The resting heart rate of 106 is fast but could be anxiety while having a test. PR is a time interval and is normal. P/QRS/T are direction on a 360 degree scale. The P is pointed slightly rightward but is probably normal if your young and thin. All of the other values are perfectly normal."
},
{
"id": 171244,
"tgt": "What causes bumps on the hip of a child?",
"src": "Patient: My eight year old son has a bump on his lower lip that looks like a blood blister. However, it has been there for quite some time, what should we do? I don t want to over react, but do I need to get him medical attention or is there an over the counter fix? Doctor: hi,sorry to hear about your son,you didnot mention for how long the blister is there,if it is for ,less than a week,there is no need to worry,but if it had been for long duration without pain and if it is hard,then he needs furthur evaluation by a pediatrician.hope you got your answer.good day"
},
{
"id": 124309,
"tgt": "Reason for deep ache in the collar bone?",
"src": "Patient: I have a deep ache in my collar bone specifically starts aching at night time. Im thinking it might be from lifting weights especially weighted dips. I am a bit of a worrier so im thinking this is the worst and i have an HIV infection. Would love to know what you think? Doctor: Hi, Since you have other medical complications you need to take a guidance from your physician if you can really do strenous exercise like weight lifting. This kind of weight lifting exercises aren't advised to people without having any medical clearance. I feel you have sprained the Acromioclavicular ligament and that might be cause for the pain. For time being try to avoid lifting weights and focus more on breathing exercises along with hot water fermentation at the pain site. You should do well in 5-6 days. Also, will request you to consider your primary physicians advice before doing any heavy exercises. Hope I have answered your query. Let me know if I can assist you further. Regards, Jay Indravadan Patel, Physical Therapist or Physiotherapist"
},
{
"id": 112994,
"tgt": "Have back pain, diabetics. Have pacemaker. No medication. CAn massage been done for back pain?",
"src": "Patient: Hi I went to massage school but am not licensed yet, my neighbor had a pacemaker put in after having knee survey and his heart stopping on the surgery table. I know he also has diabetes not sure what else, but he asked if I could massage him because he is having back pain. Can I massage him or should I decline I would not want something bad to happen? Thanks Doctor: Hi, Thanks forposting your query. As you have not licence of massage therapist. I advice you to not to do massage , knowing his other problems. You should decline for this and refer him to at right person who will expert in dealing with back pain like orthopaedic Surgeon or spine surgeon who can suggest him best treatment after thorough evaluation. Hope this will helps you. Regards Dr Saurabh Gupta Orthopaedic Surgeon"
},
{
"id": 223186,
"tgt": "What causes brown vaginal discharge after taking Plan B?",
"src": "Patient: my girl did unprotected sex then within 48hours she took emmegency pills plan B 6 mg then after 8day she saw brown discharge in her vaginal but till now her period do not come but she feel pain in pelvin and those brown discharge contunue to come so what happen? Doctor: Hello,The morning after pill may not only cause irregular spotting and bleeding but it may also have an impact on the next period. Although most women still have their period on time, you may have it several days later or earlier than usual. So the symptoms are just a side effect of the pill.Hope I have answered your query. Let me know if I can assist you further.Regards,Dr. Dorina Gurabardhi"
},
{
"id": 182092,
"tgt": "What causes blister like bump on gums after wisdom tooth extraction?",
"src": "Patient: i recently got my wisdom teeth removed last wednesday, the 22nd. and the healing process has been going very smoothly, except for the right side of my mouth. i have seemed to develope a blister like bump on my gum where one of the teeth were removed, and it is a bit painful. withstandable, but still very concerning. i just want to make sure it is normal or if you have any thoughts?! Doctor: If you are experiencing pain, any localized infection should be ruled out. Kindly get yourself examined clinically to rule out any infection. Continue to pain killers as prescribed to alleviate pain."
},
{
"id": 104023,
"tgt": "Have a history of sinus problems, sinusitis, allergies & infections. Mucus will be blood tinged, sometimes pink in color and sometimes not. Concerned?",
"src": "Patient: Hello, I have a history of sinus problems, sinusitis, allergies & infections. Recently when I blow my nose the mucus will be blood tinged, sometimes pink in color and sometimes not. I also sometimes have little clots when I blow my nose. I have also had some shortness of breath on occasion. Just wanted to get some advice, thank you. Doctor: SINUSES ALLERGIES EVER GO AUTOMATICALLY YOU HAVE TO TREAT IT THGEE ARE ALLERGIES AND ALLERGIES GOES ON INCREASING AND THERE ISPOST NASAL DRIP WHICH GOES BACK AND INVOLVES THE CHEST CAUSES SHORTNESS OF BREATH AND BLEEDING IS ALSO SIGN OF SINUSITIS IT IS ALLERGIC SO GET XRAY PNS DONE AND CONSULT ALLERGEOLOGIST TO TREAT ALLERGIES SO THAT THEY DONOT RECUR AGAIN FOR TIME BEING YOU CAN USE ANTIALLEGICS ALLEGRA ONCE A DAY AND FEXOFENADINE ONCE A DAY SYP PIRITON 1/2 TSF NIGHT TAKE ANTACID ALSO APPLY NEOSPORIN H EYE OINTMENT IN NOSE BD SEA WATER DROPS 2 DROPS IN EACH NOSE NIGHT YOU CAN CONTINUE OR TAKE OFF AND ON FOR 7 DAYS BUT FOR CURE YOU HAVE TO CONSULT THE SPECIALIST"
},
{
"id": 72806,
"tgt": "What causes chest pain,shortness of breath and cough?",
"src": "Patient: hello doctor, i m kanika agrawal from gwalior, M.P. age 21, i have severe pain in my chest from last 1 week and it is very difficult for me to breathe.. little cough, and this infection occurs for 2-3 days (short duration) many times in a year but this time it has been for 1 week.. when i consult a doctor , he used to tell dat it is only an infection.. but i thought that i would something else. plz suggest... Doctor: Thanks for your question on Healthcare Magic.Recurrent cough, breathlessness, chest pain in young patient should always be evaluated for asthma.So better to consult pulmonologist and get done clinical examination of respiratory system and PFT (Pulmonary Function Test).PFT will not only diagnose asthma but it will also tell you about severity of the disease and treatment is based on severity only. You will mostly improve with inhaled bronchodilators (formoterol or salmeterol) and inhaled corticosteroid (ICS) (budesonide or fluticasone).Oral combination of antihistamine (levocetrizine or fexofenadine) and anti allergic (montelukast) once a day at night is also beneficial.Don't worry, you will be alright with all these. Hope I have solved your query. I will be happy to help you further. Wish you good health. Thanks."
},
{
"id": 174462,
"tgt": "Suggest treatment for fever and cough",
"src": "Patient: my baby is 1yr 7days old and last sunday i felt my baby was little bit hot..then monday morning he had cough.. i gave him him adol drops but the temp never went down then tuesday morning he had cold so i decided to bring him to the hospital..his pedia advised that it was just a normal infection/bacteria so he gave me adol syrup prospan and nasal drops...its been 3days taking those medicines but no changes at all..what shld i do?..thanks Doctor: Hi as advised and examined , your baby is having respiratory tract infection .It may be viral /bacterial .the differentiation is by activity of the child along with intake of feeds.If really looks toxic with decreased activity, it indicates bacterial infection and needs further tests and antibiotics.Otherwise, support medications are enough.Give her plenty of fluids ."
},
{
"id": 75113,
"tgt": "What causes chest pain near the sternum?",
"src": "Patient: Hi, Im 22 and have recently had chest pain on the right side of my sternum. The pain at first was quite mild and have experienced it before and it would come and go but today it got much worse. The pain was very sharp and localized to about 1 rib. The area around it was tender and upper body movement would sometimes relieve or increase the pain. The pain hasnt spread to any other areas and Ive found shallow breaths helped lower the pain. I havent had any injuries or incidents which could have caused any broken/cracked ribs. Doctor: The characteristics of the pain you describe are compatible with musculoskeletal pain. It is characterized by a localized pain that can be exacerbated by cough, respiratory movements of the chest wall, movements of the superior members and palpation of the painful point, which will present with tenderness and, eventually, with a muscle node. Musculoskeletal pain varies in intensity and duration. It can be mild or very intense with impairment of daily life activities. It can last a few hours even if untreated or last many days, weaks or months. A localized, long lasting musculoskeletal pain that does not resolve after the use of traditional analgesics and anti-inflammatory drugs is classified as myofascial pain syndrome and frequently requires multidisciplinary intervention."
},
{
"id": 94953,
"tgt": "Constant abdominal pain with urge to urinate with nausea. Cause?",
"src": "Patient: A few days ago, on Sunday I woke up with extreme abdominal pain . I later threw up several times and had diarrhea all day. I thought I had food poisoning from some food at a wedding. Monday I still had diarrhea and food poisoning symptoms. The following days, Tuesday through Thursday I had a sensitive stomach. I feel nauseous and have a headache on and off. I feel very tired. My tongue also feels like it has lost it s taste buds. I have had a loss of appetite. I am concern because I woke up to an extreme pain on my left abdomen this morning and had to go pee. I still have an aching abdomen. This is not a cramp or the same feeling as a food allergy . The pain in my abdomen is more constant on my left when lying down. I feel very nauseous. Doctor: Hi, You need a course a good intestinal antibiotic which can clear off the infection. You may need a combination of ofloxacin and metrogyl like Oflox D. It is a prescription drug; please get it prescribed by your treating doctor. Do not wait for the infection to subside on its own, it can lead to complications. You also need probiotics. Take plenty of fluids orally, consume Yoghurt. See your doctor at the earliest. Get well soon. Regards,"
},
{
"id": 193871,
"tgt": "What causes difficulty in ejaculation?",
"src": "Patient: i am a 44 year old male, 5'8\" tall, 170 pounds, go to the gym 6 times a week, very fit and eat well. My issue is I do not feel the full sensation of ejaculation anymore and it is also harder to reach ejaculation. My libido is still very strong. I visited my doctor last year because I had problems urinating and when I ejaculated, it came out in a watery substance with very little feeling. i thought I had prostate problems but i had that checked by my family doctor, a urologist and proctologist. I have had a colonoscopy also and I am found to be normal and healthy. i have always been a rather nervous individual and tense. To be honest, and this is extremely important, i feel that my sphizter has slowly tightened up over the last 20 years, and it is relevant to this issue because I do not feel through my sphizter anymore, either when i orgasm or even if i push in general without having to go the washroom. All 3 doctors do not notice that my sphizter is tight at all. i not know what is wrong with me. I have lost sensation. Doctor: Hello. I have reviewed your query and here is my advice. Delayed ejaculation (DE) occurs when a man needs more than 30 minutes of sexual stimulation to reach orgasm and ejaculate. DE has multiple causes, including anxiety, depression, neuropathy, and reactions to medications. Please let us know whether you have any of these conditions. No drug has been specifically approved for DE, but medications used for conditions such as Parkinson\u2019s disease have been shown to help. Counseling, psychotherapy, sex therapy etc can also help you. Hope I have answered your query. You can contact me for treatment options. Let me know if I can assist you further. Regards, Dr. K. V. Anand"
},
{
"id": 84078,
"tgt": "Could taking medicines like hydralazine, coreg, fobic and testosterone lead to sickness?",
"src": "Patient: I take several medications ,hydralazine ,coreg, fobic, norvasc, fih oil, vitimin c&d and testostrone 1.62 and recently Istated (1week) I started a new med. chlorthalid and got sick. Would a combination of the other meds. be the results of getting sick ,if so which one could be the ploblem , Thanks Stan Doctor: Hi. Usually, it is the underlying conditions that are sickness. Often the sickness is coincidental. Certainly, if someone were on one or more medicines and got a virus, one would be sick. It would not be due to the medicines. I cannot say in your particular case. Hope I have answered your question. Let me know if I can assist you further. Regards, Dr. Matt Wachsman, Addiction Medicine Specialist"
},
{
"id": 93909,
"tgt": "Swelling, irritation under foot, emptiness in stomach. Uric level increased. What is going on?",
"src": "Patient: my uric acid level rise to 6.1, i feel pain, swelling and irritation under the foot. also pain in left thigh. i take nonveg and drink (small 30ml of three peg) one or two time in month. somebody told me to take two small piece of garlic with water on empty stomach, is this good? kindly suggest me diet, medicine and precautions. Doctor: Hi and welcome to HCM. You should check vascular supply of feet and control urat levels. This is obviously caused by hypeuricemia and there are certain medicines for it. Also you should do doppler ultrasounds to check venous and arterial flow. There is no food which can help except low urat food. Also if inflammation occur you should take antiinflammatory medicines. Wish you good health."
},
{
"id": 212507,
"tgt": "Sensitive to small issues, feel depressed and cry a lot. Is it Depression?",
"src": "Patient: Hi! im ayesha and im 20yrs old i jus wanted to know that since my dads death i have become very sensitive to petty issues i like being alone, depressed and i cry alot without any reason, its not that i show it to people but since the last two three months i have started loosing friends out of my circle and most people think though im caring but still its better if they stood away from me! Doctor: Hello and welcome to Healthcare Magic. Thanks for your query. I'm sorry about your father's death and I understand that you are going through a difficult time. It is common to have some depressive symptoms after the loss of a loved one - this is called a grief reaction. But if the symptoms are excessive or prolonged, then it may be indicative of a pathological grief or a depression. Since you mention that your symptoms are quite prominent and distressing, I would suggest that you see a psychiatrist for a detailed evaluation. There are effective treatment options in the form of counselling or sometimes, medication, which can help you overcome your problems. Wish you all the best. Regards, Dr. Jonas Sundarakumar Consultant Psychiatrist"
},
{
"id": 109025,
"tgt": "What causes back pain and blurred vision after masturbation?",
"src": "Patient: Hi doctor, I am 26 year old from india and living in Australia and had been masturbuting since 2005 that is since 10 years I had eye sight since 2007 Bcuz of this habit and I feel goog and healthy while I don t do it but as soon I start doing it I do this at least 6 to 7 times a day and now a days I am feeling back pain ,blurred vision , no hair growth sply head and feeling lazy all the time and can t remember things as I am a IT student...please help me Doctor: Hi,Nothing to worry, your problem is mainly of psychological origin.It has nothing to do with masturbation.Masturbation has no any bad effect in the body or health.Having masturbation this much time is much more.Concentrate your mind in study.pass some time in entertainment, sports and chatting with your friends.Take high nourishing diet.Ok and take care."
},
{
"id": 164658,
"tgt": "What causes low grade fever?",
"src": "Patient: My 8 year old son came on Friday with a pounding headache and went to sleep at 6:00, very unusual for him and since then has been spiking a fever every night the highest being 103.5. He has also been very lethargic during the day with a low grade fever. Doctor: Hello,Hope your son is feeling better now.There are several reasons for low grade fever. I would suggest you to get his blood work done (compete blood count, Differential blood count and also blood culture)For the fever you can give him paracetamol till the time reports are out. In case of any infection, the doctor would start with suitable antibiotics.Take good care of him."
},
{
"id": 32387,
"tgt": "What causes dry mouth and bumps on the back of tongue?",
"src": "Patient: Hi I had protected sex with a girl I met in a nightclub in Zambia, she was not a prostitute. But, from the very next day I was extremely worried that I contracted HIV. I could not sleep the whole night through for weeks. 6 months after the night I had a HIV test and it came back negative. However, its been 13 months since that night and I am now sure that I am developing symptoms of HIV. I have had a dry mouth for 3 weeks now, and have raised red bumps on the back of my tongue, and I have just noticed a few tiny red blotches on my shin. pleases help I am struggling to cvope with the anxiety Doctor: Hi Dear ,Welcome to HCM.Understanding your concern.As per your query you are having symptoms of dry mouth and bumps on the back of tongue which you are presuming as HIV, but is not due to that. Its not a symptom of early infection and what you are experiencing is most surely due to anxiety. The mouth is an inhospitable environment for HIV and saliva does contains molecules that inhibit the virus. Your risk is even further decreased by the fact that you used protection as well. Don't panic about it, its all your anxiety. Dry mouth, white coating could be due to oral thrush or leucoplakia. You should take multivitamin tablet to booster immunity of oral cavity. Chew on few drops of lemon or put them in oral cavity to stimulate taste secretions. Maintain proper oral hygiene by brushing, flossing. Use antimicrobial mouth rinse on daily basis. Use tongue scrapper lightly to remove debris and coating on tongue. If you are still anxious , visit oral pathologist and go for few tests such as PCR, P24 etc to confirm.Hope your concern has been resolved.Get Well Soon.Best Wishes,Dr. Harry Maheshwari"
},
{
"id": 101786,
"tgt": "Suggest treatment for asthma and allergic rhinitis",
"src": "Patient: Im 21 yearsold..Im having asthma and allergic rhinitis for the past 10 years. Im on budesonide inhaler for asthma but my rhinitis has become worse recently. Now I have rhinitis and associated symptoms everyday. What treatment will be suitable for me? Doctor: Hi and thanks for the query,Allergic rhintis would require long term antihistaminics for months, inhaled steroids and injectable steroids at some point. Injection of Triamcinolone at 3 months intervals have been helful. Astma might require inhaled steroids to be taken even up to a year in some circumstances to obtain pure asthma free periods and improve quality of life. Inhaled steroids (seretide) and Zaditen taken for over 6 to 12 months could be needed at some point. The opinion of your pulmonologist and allergist could be helpful . Kind regards"
},
{
"id": 177977,
"tgt": "What causes smelly and loose stools in a child?",
"src": "Patient: i JUST FINISHED AN ANTIBOCTIC FOR A BACTERIAL INFECTION AND STILL HAVE UPSET STOMACH, i WAS WORKING WITH A 3 YEAR OLD AND HE HAD REALLY SMELLY bMs AND LOOSE STOOLS ? i WANDERED IF IT WAS C DIFF BECAUSE THE MOTHER AND HE WERE SICK SO MUCH ? iM SORRY i DIDN T KNOW IT COST SORRY ! Doctor: smelly loose stools can be due to infection as well as post antibiotic treatment ,it is likely that you must have contacted when you where handling child and their belongings"
},
{
"id": 29269,
"tgt": "How can MRSA be diagnosed?",
"src": "Patient: I have been diagnosed with MRSA in my eyes with my first culture about two weeks ago. I have taken the meds and now the 2nd culture (last week) shows Proteus Mirabilis. This did not show up in the first culture. I am concerned that the MRSA might be eradicated in my eyes but not in the rest of my body. Now I have this new bacteria, Proteus mirabilis. How do I know if I still have MRSA in my body. The eye doctor said he only can tell me about my eyes. Is it common to eradicate one bacteria and then the culture show a different bacteria? Doctor: Dear patient,If the culture shows that MRSA is eradicated in your eye, then it means that you don't have it any more there. This makes it possible that other bacteria can infect your eye, and replace MRSA, or fill the gap left after MRSA's eradication.Regarding the presence of MRSA in your body, I would say that it is hard to speak 100% for sure, but if you don't have any complaint, than you don't have MRSA. It wouldn't go unnoticed, but you would start showing signs of infections and malaise. So, in lack of these, I would say that you don't have MRSA in your body.Hope I have answered your question,Manoku Eni M.D."
},
{
"id": 188684,
"tgt": "Reddish bumps on the back of the tongue, excessive saliva production, whitish tongue. Difficulty in swallowing. Remedy?",
"src": "Patient: I have these reddish bumps on the back of my tongue accompanied by EXCESSIVE saliva and a very slight amount of whitish-yellowish coating on my tongue--again, only in the back. I cannot chew well as the bumps are very irritable. Swallowing , even water, is also a little difficult. This has never happened to me before so I m a little concerned. fyi A few days ago I got a canker sore and have been rinsing with salt water. The sore is still there. I ve also been very stressed lately. Doctor: Hello,Welcome to HCM,Do you have any pain in these reddish bump? If pain is there then it can be apthous ulcers...There are various reasons for it to occur...like-stress,anxiety,certain vitamin deficiency...any hormonal changes..etc...but its recurrent in nature....In your case proper clinical examination is required for proper diagnosis...You can take vit B12 supplements..and chlohexidine mouthwash...Get your proper check-up done...Take care..."
},
{
"id": 10740,
"tgt": "Is diffuse hair fall permanent?",
"src": "Patient: HI. I had a query about my hair fall.I was going through male pattern baldness from the age of 16 and now i m 24.but i had a rapid diffuse hair fall in last 2 months may b reason i understand is going on a crash diet for weight loss.i want to ask that whether i will recover through this diffuse hair fall or is this a permanent hair loss and what diet should i take now.Secondly doctor has given me medicine finasteride 1 mg popularly known as propecia too for stopping hairloss.but i have read and listened it side effects about impotency (erectile dysfunction).tell me please about this too .thankyou Doctor: Hi.As per your case-history you have male-pattern baldness exacerbated by dieting.My treatment advice-1. Don't panic. Stress increases hair-loss. Stop dieting and have good nutritious diet2. It can be controlled upto age of 30-32 as it is hereditary.3. Continue propecia for 3 months. It's side-effect is reversible and only felt during initial phase of starting drug.4. Apply minoxidil 2% solution 1ml twice daily. Continue for 12-18 months.Consult trichologist for further advice.Thanks.Dr.Harshit BhachechM.B.B.S,D.D.V.L"
},
{
"id": 212847,
"tgt": "Irritability, excessive anger. Taking adderall medication. Treatment of side-effects?",
"src": "Patient: um, well adderall is seriously making me super irreitable/angry/mad.etc. i need something for motivation!!! something that will get me out of bed and ready to start a new day! also, i need something that will help with the super, super, feeling of being overwhelmed! shot,! i ll take any advice anyone could help me with!!! please!!! Doctor: Hello, The fact that you are using adderall suggests that you are probably having a condition called ADHD. And the complains of decreased motivation and energy and feeling over whelmed suggest that you might be having a co-morbid illness of depression. We need to know further details form you regarding this Do visit a psychiatrist and discuss with him your depressive symptoms. Hope I am able to solve your concerns. If you need further detailed discussion, I will be more than happy to help you. Dr. Srikanth Reddy"
},
{
"id": 78753,
"tgt": "Is there any suggested treatment for flared ribs?",
"src": "Patient: My son just had the Nuss procedure for his Pectus Excavatum and his chest looks great, no to mention he is breathing better. However, along with the Pectus Excavatum, he also has extremely \"flared\" ribs, I mean the doctor he had never seen a protrusion like his. My question, what treatments are out there for \"flared\" ribs Doctor: Thanks for your question on Health Care Magic. I can understand your concern. If he does not have any issues either cosmetically or for his breathing then it would be fine to leave it at it. Try to maintain the posture straight and exercise to build the muscles. Consult a physiotherapist for the same. Don't worry, you will be alright. Hope I have solved your query. Wish you good health. Thanks."
},
{
"id": 109316,
"tgt": "What causes acute back pain with nausea?",
"src": "Patient: Night before last I felt 'normal' - yesterday morning I awoke w/ a sharp pain in the back left side of my neck going down into my shoulder - increases much worse if I tilt in the other direction to far obviously - feel the same today; not any better. The pain is pretty strong if I move the wrong way; cause small waves of nausea. Could this amount of pain just be from one night's bad sleeping position or should I see someone? Doctor: Hi Welcome to healhcaremagic After going through your query I concluded that you may be having muscle spasms in neck. It may be related to sleeping in wrong position. Its treatment is rest and analgesic such as ibuprofen for pain relief. Vitamin B and C are helpful in recovery. Second possibility is cervical disc prolapse. It can be confirmed by MRI of cervical spine. Its treatment is analgesic such as ibuprofen. Rest to neck and Mecobalamine is helpful. Keep the neck in as comfortable position as it can be kept. You can discuss with your Doctor about it. Hope your query get answered. If you have any clarification then please don't hesitate to write to us. I will be happy to help you. Wishing you a good health. Take care."
},
{
"id": 220102,
"tgt": "Is this 32 week growth scan of the baby normal?",
"src": "Patient: hi this scan was done when i was 32 weeks 5 days are these results normal i have GD Single IUP cephalic heart rate 143 bpm AFI 76 mm placenta anterior cervix 4.3 cm BPD 77 mm 30 weeks 6 days, head circumference 31 weeks 2 days, abdominal circumference 305 mm 34 weeks 3 days, femur length 58 mm 30 weeks 2 days weight 2035 grams EDC 4/17/2014 Doctor: Hello, and I hope I can help you today.The combination of the head circumference, abdominal circumference, and femur length is used to calculate the estimated weight of the baby, which is normal for a 33 week pregnancy. I do not know what your original due date is, but if the \"due date\" estimated from the ultrasound is within 2-3 weeks of your original due date, then they do not change it based on an ultrasound result.The estimated fetal weight of 2035 grams is normal at 32 weeks 5 days.I hope I was able to answer your question today and that this information is reassuring to you.Best wishes, Dr. Brown"
},
{
"id": 138213,
"tgt": "Treatment for swollen cervical lymph node",
"src": "Patient: I have cervical lymph node swelling just on the left side, I have begun snoring incredibly loud at night- so loud that I wake myself as well as my spouse, and I have a constant ache in my left upper arm and left shoulder. I went to my PCP today, and she ordered TSH level and a CT of the neck and chest. Anything else I ought to ask my MD to check? I am 40 yrs old with a negative health hx, no tobacco hx, and min. ETOH use. Since my time for taking care of myself is limited due to other commitments, I just want to make sure that I can be as efficient as possible when I do go see my MD. Doctor: Hello, I have studied your case. if your neck lymphnode is enlarged then you should get needle tests. If there is any pus or any cell then it can be checked under microscope. Most probably it will be infection and you will required Antibiotics. For your snoring you need to see an ent specialist. I hope this answer will be useful for you. let me know if there is any other followup questions. thanks"
},
{
"id": 174220,
"tgt": "Suffering from scarlet fever",
"src": "Patient: Hi my son suffered from scarlet fever a month ago since then he has been falling sick and always given antibiotic.currently he is having cold. I am afraid he should not get fever and be on antibiotic. Please tell me why this is happening to him.Worried mum Doctor: HiAfter complete treatment with antibiotics the chance of complications are reduced. Complications now occur very rarely. However, if they do occur, they can be serious.Complications due to the spread of the infection can occur early in the infection and may include the following:Ear infection (otitis media)Throat infection and collection of pus (abscess)Sinus infectionPneumoniaMeningitis and brain abscessLater complications can (rarely) occur a few weeks after the infection has cleared. These occur as a result of immune reactions in the tissues. These may include:Rheumatic fever (which can damage the heart)Kidney damage (glomerulonephritis)If your child has only cold and no other symptoms, you may give him chlorpheniramine . But in case he develops high fever or any other symptoms, do contact your physician."
},
{
"id": 212662,
"tgt": "Have reasons to be upset. But cannot verbalise them. Usually keep things to myself. How to overcome that?",
"src": "Patient: How can I share my feelings? I dont know how to share/express how i feel. Usually its not a problem i dont have many things that bother me. I have a very very long fuse and i hardly get upset. I tend to keep things to myself if that makes sence. In this case i want to share my feelings with my fiance. We have been togeather for about 6years, and he knows when somethings wrong... he usually stats with whats the matter? Whats wrong? ect My problem is i cant really answer the question. Its like i dont know whats wrong. For example he will say something and it bothers me, but im not sure why. I used to just ignore it, lately ill sit there for a while and say i dont know how i feel about that. I feel like i have reasons to be upset but i dont know how to verbalise them. I know its a problem, i want to fix it. But i dont know how to start. How to share without feeling stupid? Or sound like im complaining. Doctor: Hello......... Thanks for your query. The lack of ability to articulate one's thoughts/feelings is a delicate issue. I would suggest you to seek help from a trained clinical psychologist who can train you in better social skills, assertiveness training and undertake a behavioural analysis to suggest you remedies. This issue can lead to anxiety disorders/depression. Hope you found the information useful. Regards Dr Sundar Psychiatrist"
},
{
"id": 13199,
"tgt": "Suggest treatment for rashes that itch and pain all over the body",
"src": "Patient: i have these white crystals thats come out of my skin. it started out that it only realllty iched and was o9n my tummy, i have been dieagnoed with end stage live diease. cirroisius. causes from hep c. no i d9ont drinik. thins is a very bad problem now,. it its all over myt bodsy and it iches and causes rashes and pain. people think i have lost my mind. most people dont get5 it. it is dribving me nuts. what can i do Doctor: Hello, It is common for end stage liver disease to be associated with itchy skin due to deposits of bile products in the skin. We do usually recommend to take antihistamine meds to ease the itching. Apply aloe vera gel (or fresh aloe) on the itchy skin. Hope this helps. Dr.Albana Sejdini"
},
{
"id": 222620,
"tgt": "What are the early signs and symptoms of pregnancy?",
"src": "Patient: Hi had recently just got off my period but i was late for couple of days, & when i finally came on I had an Really heavy Flow that only Lasted a Day , then I started spotting Brown spots for a couple of days , Also my Breast became very itchy & tender , Now my Stomach Is also Itchy Could I be Pregnant? Doctor: Hi dear, I have gone through your question and understand your concerns.Scanty flow during menstruation and symptoms like breast engorgement and tenderness can be due to pregnancy.I will suggest you to get a urine pregnancy test done to confirm the diagnosis and get appropriate treatment.Hope you found the answer helpful.Wishing you good health.Dr Deepti Verma"
},
{
"id": 177120,
"tgt": "Suggest remedy for growing kidney and shrinking MDK with multicystic dysplastic kidney",
"src": "Patient: Respected Sir, My Son has Multicystic Displastic Kidney of left side since birth. Now he is at the age of 2 years and his BUN is 7 mg/dl and Creatinine (serum) is 0.5 mg/dl with HB. 10.5, HCT 32 and WBC count is 12.3. as per ultrasound his round kidney is growing larger in size and his MDK is shrinking and only water drops are shown. Kindly advise me in this situation what i should have to do? either ruled out or kept on monitoring. Muhammad Riaz Doctor: the renal function of the child is absolutely normal.This suggests that the normal kidney is functioning normally. MCDK,the cysts usually starts regressing by the age of 7 years, but the risk of hypertension and development of cancer(wilm's tumor) is there.get annual ultrasound and Blood pressure monitoring with 6 monthly KFT. if their is any chane of cyst enlargement, stromal size increased or hypertension, then nephrectomy might be required."
},
{
"id": 174527,
"tgt": "Suggest first aid for mosquito bite for an infant",
"src": "Patient: Dear Sir, recently two three mosquitoes have bite my baby child running 2yrs 8mnths in the parking area of a shopping plaza, as the marks of bites are clearly visible on her legs, i am scared ........ as i have heard few of the cases of Dengue in our city. Pls advise what should be the immediate remedy/first aid to avoid dengue? Doctor: HiWelcome to HCMDon't worry.Every mosquito doesn't lead to dengue fever. It's caused only by Aedes aegypti mosquito which is infected.Diagnosis of dengue fever is usually made when a patient exhibits the typical clinical symptoms of headache, fever, eye pain, severe muscle aches, and petechial rash.For the time being just apply calamine lotion on local areas of mosquito bites. In case the child has any symptoms, you can go for blood exination to detect dengue antigen or IfM antibodies to dengue virus."
},
{
"id": 63291,
"tgt": "Suggest treatment for lump where roxycodone injection was administered",
"src": "Patient: Hello I injected myself a week ago with roxycodone and now 9 days later the site is red warm swollen and a hard lump has taken over the bend of my arm I hv been massaging it and keeping a heating pad on it 30 min on and off and it just seems to be getting worse what s wrong what do I do Doctor: I studied your query in all the details put forth by you.I understood your health concerns.Cause of the health problems you queried-The lump on the arm is -Post-Injection Lump ? Injection Abscess mostly.As you have all the history for it.Remedy-Cold Compresses would help you more.So stop the hot compresses.This would help still if full blown abscess is not developed yet.If you have fever with it,consult your ER doctor andTake Antibiotics and anti-inflammatory under his cover.If it still grows worst, I would advise you to consult ER Surgeon, who would treat it by Surgical Drainage,if need be,after needful investigations.As I don't have any direct photo of the lump / or not many details from you, I would advise you to consult surgeon. Hope this would resolve your query with full satisfaction.Welcome for any further query in this regard to ME by direct question at HCM.Write excellent review and hit thanks, to narrate your experience ,which would improve my rating for many other visitor's to HCM.Wishing you fast recovery.Have a good day.Dr.Savaskar M.N.Senior Surgical Consultant.M.S.Genl-CVTS"
},
{
"id": 219771,
"tgt": "Is Meprate safe during early pregnancy?",
"src": "Patient: i had irregular periods.pregnancy test done which was negative, meprate was taken for 5 days.still didnt get periods,pregancy test was positive.now i m 11 weeks pregnant taking susten &folinine tablets also injection hucog 5000.Dr said meprate is safe &wil not affect ur baby.is it so.should i opt for abortion Doctor: Hello,Thanks for writing to us.Meprate is a type of progesterone pill and contains medroxyprogesterone acetate as active ingredient.If you had taken the pill before confirmation of pregnancy, then ill-effect to your fetus is very much rare or minimal.It may cause virilization of female fetus in few percentage cases. However, it is quite unlike if you had taken just for 5 days.Do one ultrasound scan and other anomaly tests before deciding for abortion. In most cases, it DOESN'T cause any type of congenital anomaly of the growing fetus and women deliver healthy baby.Hope, I have answered your query. Do periodic check up to your doctor and proceed other necessary tests. Good luck."
},
{
"id": 168878,
"tgt": "Suggest medication for speech disorder",
"src": "Patient: Dear Sir/Madam, My son is 10 years old and he is speaking very less. he can speak all words but not able to speak complete sentance fluently.he can listen and having very sharp mind. he is studying in 2nd class. he born as prematured child and doctors told us that he will take some time for fluent speaking. last three years we have started speech therapy also but improvment is slow. please suggest us. Doctor: HiIt means that he should continue speech therapy .You should also stimulate him at home to talk, try to sing loud., etc.If he is otherwise mentally normal and intelligent this i\u00dfthe only way.By the way check his hearing by audiometry testsGood luckGod bless Dr uma"
},
{
"id": 136269,
"tgt": "Suggest treatment for rectal stump with colitis",
"src": "Patient: I had a iliostomy about 16 years ago. They left the rectal stump on but since then i have found out that the stump has colitis in it so i wont be able to reconect. The worry i have is the doctor told me 16 years ago that i would need my stump removed as it is slowly dying and there would be a high risk of developing cancer in the stump and anus.please help as i am worried that i still have this stump after 16 years. Kind regards Paul. Doctor: You must visit you physician for the solution. You must not neglect this. Whatevervusaid are actually true. Chances of cancer cn not be excluded. Other did like UC or CD can be suspected, So upload your health records for details."
},
{
"id": 94910,
"tgt": "Abdominal pain, had appendix removal, stomach making noise, urge to urinate, bladder fullness",
"src": "Patient: i have a problem my lower left abdomen hurts but i got my appendix remove in 2005 and also my stomach sometimes hurts an be making noise. and every time i go to the bathroom and have to urinate like the feeling like my bladder is full. but once i get to the bathroom is like 1 or 2 drops and the i be have lower abdomen pain and stomach pain Doctor: Hello Thanks for the query It looks like you are suffering from inflammation of the colon. Please approach a gastro enterologist for quick examination and if required a colonoscopy. Please do not delay the matter any further. You might require treatment with anti biotics and pro biotics for a week along with antacids I hope I have answered your query Regards"
},
{
"id": 23469,
"tgt": "Can one exercise despite a soft plaque in the artery?",
"src": "Patient: Hi, I am a 37 year old male, formerly heavy smoker (10-25 cigarettes per day for 15 years). Alcohol consumption: 35 units per week. Diet: generally healthy. BMI: 22.8. Some weeks ago I was admitted to hospital with chest pain. The presentation of the pain was a generalised tightness across my chest accompanied by episodes of stabbing pain in the centre of my chest, the latter usually of very short duration (<1 second). The chest pain was also accompanied by a pain in one of my top teeth. I was also experiencing sensations of impending doom. ECG and blood tests were conducted which were all normal. Blood pressure and cholesterol readings also within normal healthy limits (blood pressure: 112/74, Cholesterol: total: 3.62, LDL: 1.66, HDL: 1.72, Triglycerides: 0.53). Stress test was conducted and the readings were normal to stage 4 (12 mins) of the Bruce Protocol. However, a CT coronary angiogram found soft plaque on one artery (less than 20% narrowing of artery) and as a result I was prescribed baby aspirin and Lipitor 20mg. I have not had chest pain in recent weeks and can exercise without symptoms (jogging for 20 minutes a few times a week.) My question is, is it safe to increase the intensity of my training regime? I wish to exercise each day and at a greater intensity and duration. Doctor: hinoted all your details your ct angiogram is perfectly normalyour stress test timing was perfect you reached stage 4you cardiac enzymes were normalconsidering these facts its highly unlikely that the pain is cardiac in origin cardiac pain is mimicked by many sorts of ailments like acid peptic disorders or anxiety or gastritis,even muscular sprain can also present with same symptomradiation of cardiac pain is restricted to lower jaw ,it cannot migrate to upper jawso my answer is yes even 45 minutes of strenous exercises 6 days a week atleast is indicated in every medical literaturedont worry"
},
{
"id": 140007,
"tgt": "Suggest treatment for seizures in smokers",
"src": "Patient: i m am 52 yrs old. i have complex partial seizures, extreme anxiety, add, and the dreaded menopause. I take 4 draws from a marijauna cigarette daily and it seems to help tremendously. I know what pot does to a teenagers brain. Is it harming mine? Doctor: Hi, There are several good, effective and safe drugs to treat complex partial seizures. These include Carbamazepine, Oxcarbazepine, Lamotrigine, etc. Regarding marijuana, its role is not proven, and moreover, it is not the first line treatment for complex partial seizures. Hope I have answered your query. Let me know if I can assist you further. Regards, Dr. Sudhir Kumar, Neurologist"
},
{
"id": 39060,
"tgt": "Suggest treatment for pain in foot due to piercing of rusted nail",
"src": "Patient: Hi Doctor,A few days back a rusted nail pierced throgh my foot. I took one anti-tetanus injection immediately. Though there is no wound now, I feel terrible pain on that spot while walking. Does it mean there is infection inside? what medicine should I take now? Doctor: Hello,Welcome to HCMIts good that you took a dose of TT injection soon after the wound site.If your previous TT injection dose was 10 years back,you need to take 2 doses of injection.Also as you are having pain in the wound,you need to take a course of antibiotics and pain killers,as you are having infection.Hope you are happy with the answer,Thank you"
},
{
"id": 165245,
"tgt": "Is it advisable for a child to undergo radiation exposure?",
"src": "Patient: Hi. I recently took my son to the pediatrician who said his soft spot felt closed. He was 5.5 months old. The doc ordered skull xrays which came back perfectly normal. I am now worried sick and have feeling of guilt due to radiation exposure. Is my son going to be okay. Doctor: Till date there are no studies to prove increased risk of cancer or genetically heritable diseases in humans from exposure to the low dose radiation used in current medical imaging like X-ray, mammography etc."
},
{
"id": 187375,
"tgt": "Stitches came out after implant and skin flap is open. Any need to restitch it?",
"src": "Patient: Hi, I had the first part of an implant done today, about 3 hours ago. The stitches just came out and the skin flap over the post is open. I can't get a hold of my dentist. Do I need to get it re-stitched?I'm wondering if I should go to an urgent care as I am not sure it's a good idea to wait until Monday Doctor: HeloWelcome to HCMIf stiches are just 3 hrs before done then it should get restiches done but since you are getting in hold of your dentist and that open stiches or flap is not bleeding or causing problem then you can wait but take care of open flap.Thanknyou"
},
{
"id": 116881,
"tgt": "What is causing the nausea,lack of sleep and loss of appetite?",
"src": "Patient: Hi I am 13 years old. Female. I weight 4.5 Stone (63lbs) My BMI is 12.3. I have been suffering from nausea 24/7 for a little over 1 month. I have not vomited and have not for 6 years. I have severe emtophobia, so it makes it worse. I am Celiac and i think i may also be anemic. I am suffering from lack of sleep as the feeling of nausea keeps me awake. I am not eating very well currently, as it makes me feel more nauseated. The sight and smell of food makes me worse... Please help me...What could this be? Doctor: Hi, dear. I have gone through your question. I can understand your concern. You may have some liver disease or kidney disease. It causes nause and vomiting. You should go for complete liver function test and renal function test. Also go for complete blood count and peripherals smear examination for your anemia. Then take treatment accordingly. Hope I have answered your question, if you have doubt then I will be happy to answer. Thanks for using health care magic. Wish you a very good health."
},
{
"id": 75226,
"tgt": "What causes chest pain on left side?",
"src": "Patient: Hi I'm 24, female, about 150 pounds and have no history of any health problem. I am currently experiencing chest pain on the left side directly underneath the left breast. It gets worse when I take deep breaths and stand. When I am resting there is virtually no pain at all. Doctor: Hi and welcome in HCMI can understand your concern.According to the history better to exclude some problems doing some tests:EKGChest X rayAbdominal UltrasoundBreast UltrasoundAfter having the results and if they are positive than the relevant specialists (cardiologist,pulmonologist or GP) will advice you for the next step.Thank you and best regards.Dr.JolandaPulmonologist"
},
{
"id": 55694,
"tgt": "Should i be worried about blood stained bandage?",
"src": "Patient: my husband had his gallbladder removed 3 days ago..his first shower was this morning which was a very light and gentle shower. He just patted gently on his stomach and shortly after the onced clean bandage was red with blood. Should we be concerned and go to emergery? thankyou Doctor: After surgery slight bleeding can occur with changing bandages so you should not worry.. The patient should take bath regularly and take care not to strain"
},
{
"id": 98221,
"tgt": "Swollen lymphnoids, blisters on lip, tingling cheek, ulcer on tonsil. What is the problem ?",
"src": "Patient: Two days ago I woke up with a blister on my top lip and a bunch on my bottom in one area. Today I woke up and lymphnoids are swollen and left toncil is swollen and has a sore on it and the left side of my face is tingling/slight burning. I have had a couple fever blister in my life but never any of the other symptoms. I am 24 year old female and am health. What is the problem ? Doctor: Hi The infecting organism you are having appears to be virulent and as a bodily resistance the lymph glands swell up. This infection need be checked immediately. have saline gargles and internally have haridra khand with tab. allerin and karpooradi vati."
},
{
"id": 179770,
"tgt": "Suggest treatment for my kids cough and cold",
"src": "Patient: MY 5+ Month child is having cold and cough. My Ped told that the chest and lungs are clear and no medication required. But my child has discomfort sleeping as his nose makes a grrr sound all the time. Also he has cough and my ped said no medicine for cough till 2 years. But we cant see the baby suffering. pls suggest a remedy Doctor: Thanks for posting your query at HealthCareMagic. Your child probably has a condition called bronchiolitis which is the version of common cold in children. It is caused by a virus. Some antiallergics may be given but that is optional. Put some saline drops in both the nostrils. It would be best if you could give him some nebulisation or steam inhalation.Hope that helps. Feel free to contact back in case of further queries."
},
{
"id": 137394,
"tgt": "Suggest remedy for soreness in chin",
"src": "Patient: I fell this morning on my walk and hit my chin on the sidewalk. I don t think there is damage to jaw, but there is a large purple swelling under the chin, and it seeped blood most of the day. Will this resolve itself in time? The bleeding has almost stopped. Doctor: Hello,Welcome and thanks for your query.You have developed a contusion and bleeding into the subcutaneous tissue. It normally gets absorbed with time as the oozing has stopped, but take care that it does not get infected.You should clean the area well with an antiseptic like betadine , apply an antibiotic cream like Bactroban and keep the lesion covered with sterile dressings."
},
{
"id": 141585,
"tgt": "How long do i need to wait before i can take an oxycodone?",
"src": "Patient: I have a spinal injury and was prescribed oxycodone for pain relief 10/325 (2x daily) and physical therapy. I had to stop and switch to tramadol because i needed to study for a major exam. I took 50mg of tramadol at 8am this morning, but my back is starting to get painful. How long do I need to wait before I can take an oxycodone? I was told only not to take them together. My dr is unavailable right now, can you answer this question for me? Thanks Doctor: Hello and Welcome to \u2018Ask A Doctor\u2019 service. I have reviewed your query and here is my advice. You should wait at least 8 hours to take Oxycodone after the Tramadol use. Hope this helps. Take care."
},
{
"id": 84316,
"tgt": "Are burning sensation in armpit and breast side effects of thyronorm?",
"src": "Patient: 2 months before i was test my thyroid the doctor said i was thyroid i was take 100 mg thyronom daily but now my thyroid was normal for 2 months i was take thyronom now i was burning sensation in armpit and breast after i take thyronom it was any side effect of medicine Doctor: Hello, Medications have a very poor direction sense. It is basically impossible for them to go to a specific location and their side effects are all over. The exceptions are when the organ has a particular reason to grab the medicine, not the other way around. So liver damage or kidney damage or growing organ damage are common. Accumulation in fat like in the skin can be a pattern. But not one side. Hope I have answered your query. Let me know if I can assist you further. Take care Regards, Dr. Matt Wachsman"
},
{
"id": 55638,
"tgt": "What causes raised SGPT level?",
"src": "Patient: hi my brother have viral hep A in sep2012, on starting his billirubin was 3.9 &sgpt was 2800,after 2weeks his billirubin raised upto 19 then his sgpt was only 284,on 39th day of hep billirubin was 3.4& sgpt is 112 but a week before sgpt is 110,why the sgpt is not coming down Doctor: Hi, dearI have gone through your question. I can understand your concern. His SGPT level is very high initially with hugh bilirubin level due to hepatitis A. Now his SGPT level is decreasing. It is slightly high but its decreasing so it will come back to normal within few days. Don't worry much about that. Avoid fatty food and alcohol. Take high protein diet. Drugs like liv 52 is helpful. Consult your doctor and take treatment accordingly. Hope I have answered your question, if you have doubt then I will be happy to answer. Thanks for using health care magic. Wish you a very good health."
},
{
"id": 151787,
"tgt": "Is their a cure for Leukodystrophy ? How are hearing and vision losses caused by it treated ?",
"src": "Patient: Hi, my sister has been diagnosed with leuckodystropy 15 years back. That docters said there is no treatment for this. She is has hearing and vision problem. She can only see or hear 5% ... lease let me know if there is any treatment now ?? Or can she go for cochlear implanttaion fas hairing aid ??? Her age is now 28 now. Doctor: welcome to hcm cochlear implantation is a good option if there are no contraindications..but if auditory nerve is involved it wont help u..treatment options are very few..hope for the best..take care"
},
{
"id": 161517,
"tgt": "Is use of nebulizer safe in infants?",
"src": "Patient: My son is 20 months old and he suffers from repetetive cough, cold and chest congestion. When ever i stop medicatication, he face again these problems. He was on nebulization for last three months and now on serflo 125 inhaler, l montus kid. I dont know is this the right medication done? Doctor: Hi, You are on the right path of medication but when the symptoms improve they should be reducing it to lower dose. Hope I have answered your query. Let me know if I can assist you further. Regards, Dr. Sumanth Amperayani, Pediatrician, Pulmonology"
},
{
"id": 182812,
"tgt": "Could bad taste in mouth with memory and dental problems be connected?",
"src": "Patient: Thanks I have had gum problems, all this year my dentist said I had a Teeth that need pulling but when i went for appt I only recieved filling, Now we have new contract for our facility and have a mobile van. They pulled 2 teeth (1 wisdom) SAID TEEH HAD DECAYED AND SHOULD HAVE BEEN PULLED LONG TIME AG0. i. CONTINUED TO HAVE PROBLEMS BAD TASTE MEMORY,PROBLEM DIABETIC. WENT BACK AND WAS PUT ON ANTIBIOTIC FRIDAY, WORRYR=ED ABOUT PYRIA, GENTIVITIS ANY SUGGESTIONS IF ANTIOBIOTI DOES NOT WORK R WORRY, Doctor: Thanks for using Health Care Magic.Read your query.Chronic gingivitis is usually associated with bad taste , bleeding from gums and many other symptoms . Controlling the sugar levels is very important in a diabetic to prevent the gum disease.If I were your treating dentist I would advice you to continue with the antibiotics , maintain your sugar level for proper healing of the socket and to prevent more infection.Salt water gargling will be helpful.Once the socket is heal , have your teeth cleaned from the dentist .Oral hygiene is very important to prevent the recurrence of symptoms .Hope this was helpful.Thanks and regards."
},
{
"id": 149963,
"tgt": "Numbness in legs, severe headaches. MRI shows two hyperintense flair foci in right frontal lobe. Significance?",
"src": "Patient: I had a MRI recently and these were the findings...my Dr. says everything was normal.\"Two small hyperintense FLAIR foci in the right frontal lobe subcortical white matter are nonspecific and do not meet MR criteria for demyelinating disease.\" If it was normal...why are my headaches so severe, whyare my legs and feet numb and why is my memory so terrible? I have 4 family members withMS.... Doctor: Hi,Though MRI brain does not meet criteria for demyelinating disease, but MRI brain is not normal, it has two small hyperintensity in R) frontal lobe which can produce pain and weakness in left half especially L) lower limb(leg), these area also precipitate seizure hence episodic headache can be the only manifestation at times.Hence this lesion to be considered as ischemic and should be treated with antiepileptic especially sodium valproate with antiplatelets.Further blood sugar , thyroid profile and lipids to be checked and treated accordingly.Regards"
},
{
"id": 36524,
"tgt": "Whats the cause and treatment for soreness around the eyes?",
"src": "Patient: the outside corner of my eye sore when I blink or touch it and has a little discharge there was a little red bump that appeared a few days ago and turned white then fell off. I've done some research and have received a bunch of different possible answers but with no final conclusion what do you think it could be? Doctor: Hi,From history it seems that you might be having infected stye giving this problem.Due to swelling and soreness you get you might get pain during blinking.Put antibiotic eye drops 3-4 hourly.Go for one antibiotic medicine course for 3 days.Do hot application to eyes.Ok and take care."
},
{
"id": 128698,
"tgt": "How to cure total left side paralysis?",
"src": "Patient: Hello Sir My Father has total left side Paralysis for last 6 years now he can able to walking with sticks so I just wont to know my father has nothing any any improvement on his Left Hand so plz Guide me about this Matter and sir is there any possibility for surgery ya any thing else Doctor: Dear caring son your father must be suffering from post CVA left sided hemiparesis involving upper limb and lower limb. This happens due to uncontrolled hypertension. General progress is with physiotherapy and time lower limb recovers partially so that patient is able to walk. Upper limb especially hand recovers poorly. Since 6 years has passed and there is no recovery , chances of improvement is poor. You need to consult physiotherapist who can guide you better. Hope this is useful. Surgery is not helpful in anyway."
},
{
"id": 97502,
"tgt": "Is accupuncture an effective treatment for a fixed drug reaction?",
"src": "Patient: Is accupunture an effective treatement for a fixed drug reaction? I have had vaginal irritation from a fixed drug reaction from NSAIDS for over 6 weeks. Feeling desperate. Thinking of trying accupuncture. Thoguhts? I am 45 years old, in otherwise great physical shape, 5\"3, 113 lbs. Doctor: **1. Often people with chronic allergies [6 weeks] show signs of Spleen or Kidney Deficiency as well as Lung signs according to Accupuncturist/Traditional chinese Medicine. T2. The goal of the acupuncturist is to develop a plan which addresses the person's acute symptoms and provides relief, while addressing the underlying immune system imbalance which is thought to be at the root of the person's allergies. Treatments often include dietary modification, the use of specifically chosen herbal formulas, and acupuncture.3. Look for an acupuncturist with experience on www.Acufinder.com"
},
{
"id": 103424,
"tgt": "Is it advisable to take Romilast L everyday?",
"src": "Patient: My son (age-10 yrs) is having allergic problem. Everyday, he has to take a table t-Montec LC kid or Romilast L 5mg as per my doctor s advice. Is it advisable to take that tablet every day or how long we servive with this way. WE tested his blood also and found Ige as about 1600. Kindly advice... Doctor: Hello dear,The presence of increased levels of IgE antibody indicates that your child is sensitized to certain allergens.And the immune system of the body reacts on coming in contact with such allergens.Montelukast preparations- used as a maintenance therapy to relieve symptoms of seasonal allergies & asthma.Romilast L also contain Cetrizine, an anti histamine preparation which provides symptomatic relief.The decision about how long to continue the drug should be taken by the treating physician based on the clinical findings & co-relating them with the investigation reports.Till then, make sure your child maintains adequate hydration, proper nutrition & is well protected from dust, cold & other allergens.You can ask me any further queries using the below link: http://www.healthcaremagic.com/doctors/dr-polash-sannigrahi/64925Wishing him a Good Health.Thanks & take care."
},
{
"id": 99756,
"tgt": "Suggest medicine for cold, cough and body pain",
"src": "Patient: Hello. I've got cough n' cold and severe body pain since last 7days.Terrible neck pain (all around) and I m getting vibrations on the central part of the skull each time i cough. Tried various medicines like Combiflam and Nicip. Nothings seems to be working out.Kindly help by suggesting some good medicine. :-( Doctor: Hii appreciate your concernLooking at your symptoms it seems that you are suffering from upper respiratory infectionin my opinion you need a course of antibiotics like augmentin 1 gm twice daily tab acetaminophen for pain reliefSyrup benadryl for coughtab pantoprazole to reduce drug induced hyperaciditydo warm saline gargles and steam inhalation Hope this helps youwish you good healthtake care"
},
{
"id": 196750,
"tgt": "Reason for semen discharge while urinating",
"src": "Patient: I am 45 years old, 5'9\" & weigh 175 lbs. I do not have any health issues other than seasonal allergies. I just passed a noticeable amount of semen or sperm when I urinated. I did not recently have sex or masturbate. This is the 3rd time it has happened over a period of about a year. What could this possibly mean? Doctor: Dear user,Thank you for writing to HCM.I understand your concern about having sperm in urine.There might be two possibilities 1. Most likely, the fluid is of sex organs and tiny glands in the way of urine tube that come out from penis during voiding urine. This is not semen. The smell of the fluid and consistency may be like semen. Nothing to be worried about. This is normal thing and not disease.2. Retrograde ejaculation (Ejaculation of semen does not occur outside but it occurs back into the urine pouch during sexual orgasm). So when one passes urine after sex, it is mixed with semen and appears cloudy. This is very less likely in your case as you neither had sex nor masturbation.I suggest seeing urologist for the same.Wish you good health."
},
{
"id": 134034,
"tgt": "What causes constant pain above the hip?",
"src": "Patient: 27 female having constant pain just above my left hip. Hurts more with movement and sitting. Less pain while lying down. Had this pain for 3 days. Along with low back pain. Just had a CT and pelvic ultrasound a week ago. Nothing showed up. Would really like some answers please! Doctor: hi,Thank you for providing the brief history of you. By reading your reports, what i understand is there is no systemic pathology as the CT and pelvic ultrasound is normal a few weeks ago.What i suspect it by your mentioned history is, you may have a lumbar spine degenerative disc disease, as the symptoms stand classical for it. On functional position there is pain, on rest the pain is absent. This is the classical sign of lumbar spine degenerative disc disease. Also, an MRI can help confirm the same and also will provide the knowledge of how is the status of the soft tissue in the place.With proper rest and physical therapy the pain will be relieved and some exercises will be taught which you can perform at home as well. With my personal experience i have seen cases with similar issues, and with physical therapy 99% recovery is achieved.I wish the same from you.RegardsJay Indravadan Patel"
},
{
"id": 25358,
"tgt": "How to control high blood pressure of my father?",
"src": "Patient: My dad had an heart attack 2 years back and his BP has been 80/150 and has not been coming down inspite of taking beta blockers and other bp medicines.He has had a bypass and has leaking valve as well.He has been trying different medicines and is very frustrating.He has diabetes type 2. Doctor: Hello and thank you for using HCM.I carefully read your question and I understand your concern.I will try to give you my opinion about this problem and explain you somethings.When we treat hypertension our goal is to take mean blood pressure values below 140/90 mmHg and in a ischemic heart disease and diabetic person even lower.In case of your father If I was your treating doctor i will search for reasons that keeps the blood pressure high even taking different medicines as you say.It is essential to control his renal function. A diabetic person might have alterations of his renal functions that maintains his blood pressure high.My opinion is he should do some examinations like blood analyze, creatinin, a cardiac echo.If heart function is normal, if renal function is normal and if I was the treating doctor I should add to B-blockers some other medicine like ACE-Inhibitors like ramipril,enalapril ect. This are a very good class of medicament, very effective, especially in diabetic patients.Of course the dosage should be initiated with the smallest one and than they can be raised or add another class like diuretics or calcium-blocker.So, my opinion is you should not worry and discuss this alternatives with your treating doctor.Hope I was helpful.Best regards."
},
{
"id": 9262,
"tgt": "Can a nasal dry skin with hardness, scabbing and itching be treated?",
"src": "Patient: Hello I have had for the past 2 weeks a hard dry skin area right next to my nose below my eye it looks like a scab . I m starting to think its a fungus can I put econazole nitrate cream??? It itchs like crazy when I put the cream on ... Does that mean it s working? Doctor: Hello,I can understand your concern for the dry skin with hardness, scabbing and itching at the sides of nose. It evident from your query that you have \" Dyssebacia\u201d condition which is due to blockage and plugging of the sebaceous glands. It is a condition which is found specifically around the nose. The dry flakes which falls off from the skin are dry and yellowish material and is considered to be part of seborrhoeic dermatitis.I generally prescribe \"Fluticasone\" cream initially over the area for two times in a very thin layer. Once the improvement is seen then I shift to plane antifungal creams. I suggest you to be in good follow up of a dermatologist to get the earliest improvement.All the best."
},
{
"id": 127300,
"tgt": "Does phlebitis cause severe pain and swelling in the legs?",
"src": "Patient: I am a 58 year old female. I was diagnosed with tenosynovitis of the left ankle. I have had this for several months. Pain wasn\u2019t too bad. But now my whole leg and foot are very swollen and I have extreme pain even at rest. I\u2019m worried that it might be phlebitis. Hope you can help Doctor: Hello and Welcome to \u2018Ask A Doctor\u2019 service. I have reviewed your query and here is my advice. It could be a cellulitis. Phlebitis is unlikely. You might require intravenous antibiotics to for faster recovery and to avoid unwanted complications. Consult a physician and start antibiotics. Hope I have answered your query. Let me know if I can assist you further."
},
{
"id": 61237,
"tgt": "Is a large lump on the neck a matter of concern?",
"src": "Patient: I have a large lump on the back of my neck. It started out gumball size and grown about 2 1/2 times the original size, I fell on the ice two days after Christmas last year while taking the dogs out and I landed on my hip and arm and my head had gotten whipped back but didn t hit anything. Two days later I noticed my lump had moved further down my neck and much closer to the spine--nearly on top of it. I can still grab it and it bothers me, however the few doctors I ve asked about it don t seem to be concerned. My regular physician thinks it could be a calcified lymph node. My very aged neurogist dosen t seem to be able to see or feel it. I will. grab it and pull it out for him and he slaps my hands away and then touches my shoulder and asked if it s here then he touches the top of my head and asked if it was there. Clearly he isn t taking me seriously. Should I be concerned about this growing soft tissue mass that has the ability it move location? Doctor: Hello dear , hiWelcome to Healthcaremagic.comI have evaluated your query thoroughly .* This seems in relation with hematoma ( post falling on ice ) .* Needs evaluation with ultrasound examination and further mode of action to cure it .Hope this will help you for sure .Wishing you fine recovery .Regards ."
},
{
"id": 81787,
"tgt": "Is traveling safe while suffering from bronchitis?",
"src": "Patient: I just found out I have bronchitis, and suppose to fly out on Friday... Its Tuesday, and I have the antibiotics to help. I also have high blood pressure but the bronchitis is pretty bad, almost pneumonia. Is it safe to travel.. Plane ride is 1hr 45 minutes. And then a 45 minute ride. But I m going from cool weather to pretty warm weather Doctor: Thanks for your question on HCM. In my opinion you can fly. No need to worry much. Bronchitis needs bronchodilator treatment along with antibiotics. So better to consult pulmonologist and start inhaled bronchodilators.Inhaled treatment in the form of meter dose inhaler containing LABA (long acting beta 2 agonist) and inhaled corticosteroids are mainstay of treatment. So better to start this so that your bronchitis can get controlled before air travel. And you can also use inhaled treatment in plane.So better to consult pulmonologist and start inhaled treatment."
},
{
"id": 58813,
"tgt": "Liver cysts. Enlarged abdomen with weight loss. What could be the treatment?",
"src": "Patient: My mom have liver cysts i need more information of it like treatments what can she eat what cant she eat because shes keep telling me she cant eat any thing with antibiotics i am afraid that some doctor in china is giving her the wrong information she they also told her there s no way to treat it. shes been having liver cysts for about 5 years now it could be longer, her abdominal is pretty big and she lost a lot weight thought the year few other thing the doctor told her is not this kind of liver cysts can only be control she have to watch the stuff her eating and things like that. Doctor: need to know what type of cyst. simple cysts do not need treatment. if hydrated cyst the needs surgery."
},
{
"id": 158738,
"tgt": "uick breathing during nights. Sleeping pulse rate 102 and pulse ox 95. History of finishing last chemo. Serious?",
"src": "Patient: My husband has finished his last chemo on April 1 st. This past few nights his breathing has become quick whole sleeping. I have a pulse ox meter at home. it reads his pulse and pulse ox. He had a sleeping pulse rate of 102 and a pulse ox of 95. It is hard to convince him to go to ER since he spend a lot of time there recently. At what pulse ox level should I force him to ER? Doctor: Dear Ma'am, Sorry to hear about your husband. A pulse ox of 95 is alright and is normal for some people. If the pulse ox value drops below 90, then you should consider going to the ER. The other alarming finding is the pulse rate of 102, which is slightly more. This should be evaluated by your oncologist and an infection should be rule out. Hoping for the best."
},
{
"id": 205422,
"tgt": "Suggest remedy for vertigo caused by anxiety and depression",
"src": "Patient: Light headed with a little bit of vertigo. Shaky and some anxiety. Ate a granola bar and I feel a little better. I also just started my period. Haven t felt this way before that I remember. Not feeling thirsty but a little depressed. More like sad...I just want to sit and not do anything but the world is not a bad place. Hope that gives you enough information?! Thank you! Doctor: DearI went through the details. you are already onto the period. I suspect these vertigo and anxiety symptoms are related to the stress and tiredness associated with your period. I suggest you to introspect with what activities you did for the last three days. Was there some activities which involved stress? I hope so. The tiredness must have caused your vertigo and you became anxious as in the natural case. Taking rest for a day should solve the problem. Please consult a doctor if symptoms persist. Take care"
},
{
"id": 9779,
"tgt": "Suggest treatment to stop heavy hair fall post delivery",
"src": "Patient: hello i have delivered a baby three and half months back. during pregnancy my hairs were shiny and healthy. now after 3 and half months of delivery i am suffering from heavy hair fall. i have consulted to my beautician. she told me i have lil bit of dandruff. so i started using shampoo for dandruff. Now, please advise, what should i do to stop this heavy hair fall?thanks Geet Kapoor Doctor: Hi, It is normal to have hair loss after delivery.It usually settles in 5-6 months.I would recommend you to apply anti-hair fall serum like folliserum twice daily and take hair supplements like tablet follihair. Hope I have answered your query. Let me know if I can assist you further. Regards, Dr. Asmeet Kaur Sawhney, Dermatologist"
},
{
"id": 103508,
"tgt": "Getting hives after having cold and cough. Is it possible to be allergic to cold and cough medication?",
"src": "Patient: Is it possible to be allergic to cold and cough medication? I get hives every time I suffer from a cold and cough. Can it be that my resistance is low and the hives come out then. I also get hives when I am feeling some stress but they are easily managed on these occasions. The hives are usually only on my inner arms and wrists during these occurence. During a cold I have hives on my abdomen, inner arms, waist , upper legs, on my hand and cheek . Hand and cheek are new Doctor: YES THE ALLERGUES CAN BE DUE TO COLD AND COUGH MEDICINESTHE PARACETOMAL BRUFEN CALCIUM IRON VITAMINS MULTIVITAMINS AND DIET SUPPLIMEWNTS SOME ANTIBIOTICS LIKE SULPHA OR MICROLYTES CAN INCREASE THISIN COUGH AND COLD USE PURE ANTIALLERGFICS LIKE FEXOFENADINE EBASTINE MONTELUCAST AND BRONCHODILATOR SYP IF YOU WANT CAN GO FOR ALLERGY TESTS AND TREATMENT FOR CURE"
},
{
"id": 78837,
"tgt": "What causes dizziness,pressure in chest and burning in stomach?",
"src": "Patient: my husband has been experiencing dizzy spells,light headedness,chest PRESSURE,blacking out,and burning in the stomache after eating.been to 2 emergency rooms,paramedics checked him out doctor visists and medical clinics and noone can give him an answer.What can be causing this? Doctor: Thanks for your question on Health Care Magic. I can understand your husband's situation and problem. Possibility of GERD (gastroesophageal reflux disease) is more because his symptoms are more after eating. GERD is due to laxity of gastroesophageal sphincter. Because of this the acid of the stomach tends to come up in the esophagus and cause the symptoms of chest pain, burping, burning stomach, chest tightness, pressure etc. So ask him to avoid stress and tension. Avoid hot and spicy food. Avoid junk food. Avoid smoking and alcohol if he has these habits. Ask him to start proton pump inhibitors and prokinetic drugs. Walk after meals is helpful to prevent GERD. Don't worry, with all these he will definitely improve. Hope I have solved your query. Wishing good health to your husband. Thanks."
},
{
"id": 225277,
"tgt": "Will intake of nordette 4 pills followed by another 4 pills after 12 hours as contraceptive be effective?",
"src": "Patient: hi, i had unprotected sex last dec 8 so i decided to take nordette as an ecp at 11 am (4 pills) and another 4 pills after 12 hours. Just had another unprotected sex this dec 10 at 1 am and i take another dose of pills at 8:32 am and will take another 4 after 12 hours. will it be effective? will it harm me in some ways? pls help! thank you Doctor: Hi Thanks for posting your query at HCM Since you have taken your medication in the right dosage with in the stipulated time . this method is perfectly going to prevent you from becoming pregnant . BUT every method of emergency contraception is not 100% and may have a failure rate which is less than 1% in this case. You must wait till your next menstrual cycle comes YES Of course there are certain side effects such as weight gain , edema abnormalities in liver function . But they occur only if you are on long term schedule about which you have not specified So I HOPE all your doubts must have been cleared Take care"
},
{
"id": 42632,
"tgt": "Why are we not able to conceive?",
"src": "Patient: hi,im jnkna im twenty one,i have been married for two years,and i have been trying to get pregnant for three years now,but no luck,i have had so many test with my body with a gynae,but found nothing wrong,i have irregular periods and suffers from anaemia,but they say that its got nothing to do with the fact that cant conceive,my husband is 31 and doesnt have any children,he is not a fan of alchohol and cigarrettes,so what could be the problem? please help me Doctor: HAI WELCOME TO HCM UNEXPLAINED INFERTILITY IS THE ONE REASON WHERE PEOPLE WONT CONCEIVE AFTER ALL NORMAL TEST REPORTS.IT MAY BE DUE TO FAILURE OF FUSION OF SPERM AND EGG.INVITROFERTILISATION IS THE TREATMENT OPTION FOR YOU TO CONCEIVE.CONSULT AN INFERTILITY SPECIALIST."
},
{
"id": 31948,
"tgt": "What is the treatment for cough?",
"src": "Patient: Hi I had fever and cough 4 weeks back. I took medicine and get cured from fever. But caugh is still there. I stopped medicine after 2 weeks. Since last 1 week, I am getting little blood in the morning cough. I used to smoke 4-5 cigaratted per day. since pas 7-10 days I stopped smoking. could you please advise what should I do now Doctor: Hi, dearI have gone through your question. I can understand your concern. You have chronic cough with fever and now you have bleeding in cough. Yoh may have tuberculosis or some other lung pathology. You should go for x ray chest and sputum examination. It will give you exact diagnosis. Then you should take treatment accordingly. Hope I have answered your question, if you have doubt then I will be happy to answer. Thanks for using health care magic. Wish you a very good health."
},
{
"id": 65778,
"tgt": "What causes lump on head?",
"src": "Patient: Hello, well I ve recently discovered this bump on my head. I ve also had symptoms that I think are linked to it. I ve heard cracking in my head like my bones are splitting, I ve felt heat in my head & my eyes, and my stomach & torso. I ve had indigestion & head pains and some gradual chest pains. I ve seen my doctor about this & he said he couldn t feel a lump but it s clearly protruding. He prescribed me naproxen & said come back if it doesn t work. I even had to go to the er before I could get a doctor s appointment & they couldn t find anything wrong. I m just worried it could be something very life threatning. Doctor: Hi!, I would like to summarize your case first like this:Your 'feel' a bump on the head which is recent, protruding and is associated with heat in eyes/head, stomach, torso and cracking sensations, gradual chest pain, indigestion etc...but doctor fails to identify i; it is resistant to analgesics...Therefore, I must suggest few possibilities in this case:1. a neurofibroma2. metastatic cancer3. neuropsychiatric problem Therefore please see a neurologist as most of the symptoms are related to nerves and get x-ray or CT scan of the region for a diagnosis;regards!"
},
{
"id": 42537,
"tgt": "Is the sperm count normal?",
"src": "Patient: Vol-2ml, Already liquified, pH-Alkaline, Count-85 Million/ml, Motility Result - blank Range -upto- 40%, Rapid linear Progessive-50%, Sluggish L.P. - 20%, Non Prog -10%, Immotile-20%, Agglutination-10%, Abnormal Forms-10%, WBCS-0-1, Immature Germ Cells-15%, Epithelial Cells-1-2. This is my report is this normal? Doctor: hai,your semen analysis report is normal.your total count is normal (range should be between 20 -200million/ml)and your motility - rapid linear -50 %( should be more than 50 %) anyway it is considered as normal.i didn't find fructose level.all other parameters are normal.overall your semen report as per your quote is normal.thank youHope i answered your query"
},
{
"id": 136814,
"tgt": "Suggest treatment for a pulled muscle in groin area",
"src": "Patient: My father, age 88 was standing up and pulled his, I guess is the groin area, right above the underbelly and the leg connects. You can feel a knot under the surface. he says it s painful. I got him to put an ice bag on it and he says it helps. ? How long should I give it to heal before taking him to the doctors? Doctor: Hi,Thanks for your query.The symptoms you have described are more likely to be symptoms of muscles strain in groin area.To get relief from the pain and for quick healing:1. Do hot fomatation at the site.2. Apply local anti-inflammatory gel 2-3 times a day.3. Take anti-inflammatory drugs like Ibuprofen to reduce pain and local swelling or inflammation.4. Do not try to do exercise that involve painful muscles. Give proper time to heal.I do hope that you have found something helpful and I will be glad to answer any further query.Take care"
},
{
"id": 1802,
"tgt": "Does Susten 200 increase my chances of conceiving?",
"src": "Patient: Hi I am 26 years of age i was undergoing follicle study i ruptured on day 16th my follicle size last measured was 23 mm. my doc has adviced me to take Susten 200 Tablet from the day my follicle ruptured for next 10 days . I just want to know does that increase my chances of concieving. Doctor: Hi, yes that will increase chances of your pregnancy. Susten tablets help in supporting implantation. So continue taking it for 2 weeks and then do a pregnancy test after that. Hope I have answered your question. If you have any other query I will be happy to help you. Regards Dr khushboo"
},
{
"id": 12736,
"tgt": "Suggest remedy for rashes in penis",
"src": "Patient: yes I have what looks like a rash on my penis that is fire red. It has been there for about 3 weeks. Now fluid is coming from the sides and when i pee the urine splits and that part is blood shot red and the entire penis burns all the time and more when urinating Doctor: Hi Dear,Understanding your concern. As per your query rashes in penis seems to be due to allergic reaction and could be fungal infection of skin. Need not to worry. I would suggest you to apply coconut oil. Take hot sitz bath. Keep area clean and dry. Wear loose and cotton clothes. Take oral antibiotics sych as Cephalosporins as it will give immediate relief. If symptoms keep on persisting visit dermatologist once and get it examined completely and start treatment after thorough examination.Hope your concern has been resolved.Best Wishes,Dr. Harry Maheshwari"
},
{
"id": 61794,
"tgt": "Suggest treatment for lump under the skin",
"src": "Patient: had in 1985 hammer toes worked on with surgery to peg middle joint of toes together but in past year notices hard lump under 3rd toe next to baby toe that hurts when baby toe presses next to it the lump seems under the skin and seems to move slightly dont think its a calluos also im diabetic Doctor: hi.based from your description, it could be a tendon cyst, lipoma (fatty deposit) or a fibroma (fibrous tissue formation). other lesions that must be ruled out are gout and arthritic lesions. it is best if you consult with a doctor for physical examination and clinical evaluation, especially if it causes discomfort for you. if it's indeed a cystic formation, lipoma or fibroma, definitive management is excision and antibiotic treatment after.hope this helps.good day!!~dr.kaye"
},
{
"id": 162199,
"tgt": "Suggest treatment for stomach pain and weight loss in a child",
"src": "Patient: My 6 year old constantly has a sore stomach. doesn t want to eat and is not gaining much weight. In 2 weeks she has gone from 20.7kg to 18.1kg and her weight seems to fluctuate between these. She is becoming increasingly lethergic & gets sick easily. I have taken her to doctors numerous times and nothing, i took her again today...she had some blood tests and waiting for a stool sample. She has blood tests before, every thing seemed fine except her iron levels were a little low. Any suggestions on what else I should do. Doctor: Hi, Whatever you are describing is called Chronic Pain Abdomen in medical terms. The differential diagnosis of abdominal pain in children varies with age, gender, genetic predisposition, nutritional exposure and many environmental factors. The causes are many including - constipation / acid peptic disorders / inflammatory bowel disorders / irritable bowel syndrome/worm infestation etc. I suggest you see your paediatrician or a pediatric gastroenterologist for this. Hope I have answered your query. Let me know if I can assist you further. Take care Regards, Dr Sumanth Amperayani, Pediatrician, Pulmonology"
},
{
"id": 96614,
"tgt": "What causes nausea, speech problem and swollen eye after a head injury?",
"src": "Patient: I got a severe concussion 5 days ago. CT scan was fine. I vomited for a couple days. Still nauseous. Balance is bad. Vision is strange- white lights, tilted, everything is farther away. Speech problems. I m easily agitated & other than that my emotional feelings are flat. Other things but I think you get the point. I have an eeg & MRI scheduled for next week. This afternoon my right eye started to hurt. Tonight, it s swollen and looks like a black eye. It hurts worse. I hit the very back top of my head FYI. Do I have to go back to ER again? Doctor: Hi, Thanks for your question. I do understand your pain and discomfort. As per your history indicates you should do two things now:-1)Do a Detailed point wise neurological clinical examination by a neuro surgeon 2)Do a MRI scan now ,not in next week3)Now admit yourself where neurosurgical ICU unit is present. I hope I was able to address your query. If you have any further questions, please do not hesitate to write to me. Wishing you all the best. Thanks"
},
{
"id": 140621,
"tgt": "What diet, exercise and pain management be followed in case of RHD?",
"src": "Patient: PLEASE HELP!!! Freedom, and Freedom From the PAIN!!! Please Help!!! I have been diagnosed, (at U.C.D.M.C. Sutter Coast Hospital, Mad River Hospital) with AVN/ON, full skeletal system, with \u201cspecial complications, vascular and arterial reroute, RHD{Reflex Sympathetic Dystrophy, etc., etc., just to name a few complications\u201d from steroid, Solu Medrol and Prednisone Poisoning. I have had both hips replaced. Left hip twice, due to rare infection! Found via Radio Active Isotopes, Bone Scan. Hip removed, bedridden without a hip, for over a year, then replaced. Three Teeth Surgically Removed . Due to complications of Avascular Necrosis of the Teeth and Jaw Bone! Have had recurrent, asepsis and sepsis every 2-3 years. Being taken via ambulance, just day\u2019s, sometimes month\u2019s, after having pain meds cut in half, or more, each time, and again! Exactly what he was told not to do, repeatedly, by my Dr. s at U.C.D.M.C., S.C.H. and M.R.H.! I have tried everything my current Dr. has asked, or pushed me into, that I am capable of doing. I ve had endure endless, painful test, retest, pills and more pills. That I have had to bring in information on, from U.C.D.M.C.. To prove to him, that I should not even of tried it, or had it done. That it was hurting me and crippling me even more! I just want not to be in pain, hurt anymore and have something resembling a life back! I have been trying to get a referral for a Medtronic Pain Pump. U.C.D.M.C., Follow Up Care calls me every year, that I m hospitalized, with a pain pump, or a new pain pump comes out. Asking, if I had all up to date information, parts, pieces and if I wanted to try out, or have my current pain pump replaced. Is there anyway I can expedite getting a referral, or skype to see a physician. So I don t have to endure another lonely and painful year, of unnecessary, painful test and pills that hurt me! Specialist that have no experience, patients with or have even heard of AVN/ON. I do not wish to burden, risk my life or those around me. To be taken via ambulance, only to awaken in the hospital for the 9th time. Being asked by Dr.s , the same questions, why, how, and who, has let my pain gotten, so far out of control, yet again?! Why is your current Dr. not listening to you, or followed our instructions and orders?! All Dr. s I ve spoken with agree, with the proper nutrition, exercise and pain management, I should be able, to do anything, (non weight bearing) within reason. Right now, I am not, because of all of the above, all due to pain management. I would like to be healthy enough, that I am able to have surgery, safely, when and if needed, including Stem Cell Therapy. To date, they have had no success, to cure it that way, as of yet. Please, Please Help! I can t take the pain or abuse much longer. Any direction, guidance or help would be more than greatly appreciated. Please Help!!! Thank You, Mr. Robert E. Hunter (707) 464-2019 (707) 9546-220(707) (707) 954-6220 {text only} YYYY@YYYY Doctor: Hello, As what I see the history I am sorry to hear this. Coming straight to the point. You need a competitive physical therapist who can understand the RSD. This is a quite complicated one to be handled by a regular therapist. This condition mainly gives sharp shooting pain which is unbearable and uncontrollable by any individual who is undergoing it. In my clinical practice of 12 years seen bunch of cases with RSD and what has done wonders to them is the self-confidence of regaining the movement and also ability to bear the pain beyond the control. There is a saying which I always mention to RSD patients of mine - No pain No gain. So for having gain, let's take up the challenge now without having any kind of delay. This RSD will lead to lot of muscle wasting and also leads to overall weakness. First thing first. Take TENS therapy for a minimum of 30-45 minutes. Start the mobilizing of the limb in passive mode by a physical therapist so the joint get free and also get muscle stretch which might be painful. Again take the TENS therapy for 20-30 minutes if the pain is unbearable. Continue the same till you get adequate movement of the muscle and joint. Please understand that this condition needs a lot more effort from your own side to cooperate with the competitive physical therapist. Use hot water fermentation whenever you find time and post which please do active stretching if you can so that you maintain the mobility achieved while in session. If 8 weeks of proper competitive therapy is followed I am hopeful to get results to the optimum. Coming to the Hip, you need a good muscle strengthening now. having recurrent THR is not a good thing to hear for the surgeon or even the patient. Just that, you need a competitive physical therapist as I mentioned earlier to work on you case. The therapist should show 1000\u2105 of interest in you and you should show 1000\u2105 of interest in the therapy to get the cent percent results. I know, this sounds to most as a philosophical talk but the truth is I have achieved results in my RSD and THR patients to the maximum ability of their to have a independent ADL, which is more of the help Therapy takes times but its assurance that it will give wonderful effects if assessed, examined and planner properly. Hope I have answered your query. Let me know if I can assist you further. Regards, Jay Indravadan Patel, Physical Therapist or Physiotherapist"
},
{
"id": 19523,
"tgt": "What causes recurrent light-headedness?",
"src": "Patient: PAST YEAR EXPERIENCE LITE HEADEDNESS. HAVE BEEN TO HOSPITAL 3 TIMES. MIR, CAT- SCAN EKG, EEG. CHECKED CORTID ARTERIES, BLOOD FLOW. ALL OK I AM TOLD. REACHING OUT FOR OTHER IDEAS. LOST 20 LBS PER DOC ORDERS. THESE ATTACKS LAST ABOUT 2 MINUTES. LAST 1 ABOUT 3 HOURS AGO. 64 yrs old. Doctor: Well come to HCMThank for asking,I really appreciate your concern, if the concern tests are unremarkable as long as the headache are concern then it could be migraine and nothing could be else, it is a kind of depression and anxiety causes the symptoms and it may aggravates the migraine, such symptoms pretty common in female, I would suggest Tab Carbamazepine 200 mg once in day, hope this information helps."
},
{
"id": 182689,
"tgt": "Suggest treatment for jaw pain and clenching after dental procedure",
"src": "Patient: I've had a lot of dental work done from June to August this year. 3 crowns on my left side & that's where my pressure stems from. My last procedure was august 14 & I'm still have severe throbbing in my jaw & daily headaches. My dentist said that it would take time for my nerves to calm but the pain in consistent & sometimes unbearable. I also grind at night & suffer from stress related TMJ. I wear a night guard & take 2-6 Advil daily but it's not helping at all. I've never experienced this kind of pain from dental work before. Any other advice on how to minimize my pain & clenching at night? Doctor: Hello, Thanks for consulting HCMRead your query as you have pain in teeth after dental treatment and crown placement this pain can be due to some reason that can be infection present in treated tooth and as you have history of clenching also cause pain and tmj disorder but dont worry I will suggest you to dont take so much stress , stress is one of the main reason for clenching and tmj disorder . For this I will suggest you to do warm saline rinses two - three times a day and wear nightguard regularly . Inmeantime you can take analgesic like ketrol dt or diclomol by consulting with your local doctor . Consult dentist for examination of oral cavity , investigations iopa xray of painful region to rule out the cause of pain .Hope this will help you."
},
{
"id": 160855,
"tgt": "Suggest treatment for loose motions in a child",
"src": "Patient: hello doctor my son is 15 months old he had vomiting since last 2 days with fever, today fever has subsided with regular doses of crocin and vomiting is under control, but loose motion is not in control and baby is very cranky...please let me know of the diet for baby and any medicine for loose motion.. Doctor: Hi, You may try buscopan syrup or Gastreg syrup to control this loose motion, and when it stops, try to do stool analysis. Hope I have answered your question. Let me know if I can assist you further. Regards, Dr. Salah Saad Shoman, Internal Medicine Specialist"
},
{
"id": 169150,
"tgt": "What causes histolytica cysts with RBC : 3-5/HPF?",
"src": "Patient: my grandson is 4 months old, and based on his stool exam he has: E. Hystolytica cysts: 1-2/hpf PUS cells: 15-20/hpf RBC : 3-5/HPF what does it mean? Is this curable? Does he need to confine in the hospital? Is this serious desease for a 4months old baby? Doctor: hello. I have gone through your question and understood your query.entamoeba histolytica is also known as amebiasis. it is an infection acquired through food or water. it can be easily treated by oral medicines. infection can be prevented by avoiding feeding bottles or proper sterilisation of feeding bottles. hand washing with soap before preparing feed or prior to feeding baby. also use phenol containg disinfectant for floor.person feeding child should be screened for entamoeba histolytica as well."
},
{
"id": 125404,
"tgt": "Is riding heavy bike after cervical disc replacement surgery not advisable?",
"src": "Patient: my curvical disc replacement surgery done at C6 C7 level on 22 aug 12 successfully my age is 28 yrs and my name is ankit gupta i am having royal enfield bike ( 350 CC ) i saw in the internet that after surgery i cant ride that heavy bike doctor please tell me the right answer Doctor: Hi, If you have been following the post operative rehabilitation programme regularly, then in my opinion , you should not have any problem in driving motorcycle. However I would prefer that you discuss this with your operating surgeon. Hope I have answered your query. Let me know if I can assist you further. Regards, Dr. Gopal Goel, Orthopaedic Surgeon"
},
{
"id": 28627,
"tgt": "Are blue rashes on the arms and legs symptoms of influenza A H3N2?",
"src": "Patient: OK....I became infected with most likely the A strain flu....my incubation symtoms included a right arm soreness that comes & goes for a few days, a slight tightness in my back upper right and just yesterday a slight scrachy throat that subsided somewhat (am awaiting the actual hard raw sore throat which should arrive any time now). AND just yesterday a few (WAVES) of nausea (no vomiting) could not eat...That ended and today can eat but slight queaziness. When this started I had thick yellow mucus...now long gone & (what most will never realize) but on my body B/4 every virus one or two tiny bluish marks on my skin...either on legs or hand & arm...this time BOTH. Virus always breaks out when these spots disappear and that is about where I am at now. Is this the A N3H2 monster?? Doctor: Hello and Welcome to \u2018Ask A Doctor\u2019 service. I have reviewed your query and here is my advice. A blue skin rash is not characteristic for any strain of influenza. It may be your own idiosyncrasy in response to a viral infection as people's bodies all vary but it is not something one would expect. People can get a non-descript \"viral exanthem\" (i.e. rash) from a large number of different viruses. One of the hallmarks of influenza is that when the symptoms come on, they often come on FAST. It is not unusual for patients to remember the exact time when it started. While I can't say specifically without examining you, my you may have or had an upper respiratory or enteric (gut) viral infection. Luckily, if you live in the US, N3H2 has largely subsided, so much lower risk that that is what is going on. Hope I have answered your query. Let me know if I can assist you further."
},
{
"id": 121083,
"tgt": "Suggest treatment for bump like swelling in knee",
"src": "Patient: Hi, I m a 31 female and I fell hard on my right knee a week ago. My knee immediately swelled all over, but especially on the right side--there was a bump a little larger than a golf ball. I could walk on it, though it was sore, and I immediately went home and iced it and elevated it. The next day was more painful, but, again, I could still walk. The swelling went down, and now, a week later, it s terribly bruised and still painful--especially when I touch it, try to kneel, and sometimes even bend it). My main concern is that there s a solid hard painful bump that s the size of maybe a large marble (?) right were the major swelling occurred. Do you think I just need to keep icing it, or should I see a doctor for an x-ray? Doctor: Hi, Ideally X Ray should be done of the affected knee joint just to be sure that the bones are ok, because sometimes even a small undisplaced crack manifests with pain and difficulty in ambulation only. As far as swelling is concerned, it could just be a organized haematoma but my concern is that if there is a significant haematoma over the joint, it usually suggests a significant soft tissue injury inside the knee joint which can be meniscus or a Ligament injury, so ideally 3 weeks of Rest and Ice packs and even after that your symptoms are still there kindly visit your Orthopaedic specialist and get necessary investigations done. Till that time as i have mentioned. - Ice Packs 3-4 times a day - Compression bandage - Anti inflammatory medication along with medicine to reduce swelling. Hope I have answered your query. Let me know if I can assist you further. Take care Regards, Dr. Akshay Kumar Saxena"
},
{
"id": 39600,
"tgt": "Shall i concern about the swollen lymph node under armpit?",
"src": "Patient: hi, i have just come back from thailand. about 3 months ago i had sex with a prostitute and when id finished i realised the condom had broke. i aim to get tested on monday morning. about 1 week ago i had 1 swollen lymph node under my armpit. Is this a cause for concern? i am worried. thanks Doctor: you have to get yourself thoroughly examined to look for enlarged nodes in other parts of your body as well.of course you should get yourself tested for venereal infections and hiv."
},
{
"id": 214882,
"tgt": "Taking Cipralex for stres. Safe to take herbal remedy like Kalms?",
"src": "Patient: Hello,yes I am currently taking 20mg of Cipralex each day and although I was initially prescribed these for anxiety issues,I am finding that my stress is a major difficulty for me right noe. I was wondering if a herbal remedy such as Kalms would be safe to take? I had considered Passionflower,but as I experience periods of depression ,I am concerned about the possible sedative effects?? Doctor: hello welcome to health care magic Cipralex contains escitalopram which is an SSRI i.e. antidepressant. It can also be used in depression with anxiety. I advise you to continue cipralex as told by your doctor as it takes 10-14 days to have a full effect and you should take the drug for few months or even more as advised. You can take herbal remedies like kalms and passion flower but discuss all the stuff with your treating doctor. You can also undertake psychotherapy/counselling sessions and relaxation exercises. Hope it helps Good Luck Regards Dr. Gourav Monga Consultant Psychiatrist"
},
{
"id": 177306,
"tgt": "What causes pain in testicles?",
"src": "Patient: My 11 year old will be 12 next month is complaining today of pain in his right testicle after his bath today. He says it still hurts and the pain just started today. He said if he moves his penis a certain way while using the bathroom it will hurt and if he touch it, it hurts. Is this a sign of puberty or something serious? Doctor: Hello.I just read through your question. though many cases of testicular pain turn out to be nothing serious, the pain can sometimes be due to torsion of the testicle. This is considered an emergency. If the pain is persistent, I recommend consulting with your doctor right away."
},
{
"id": 38650,
"tgt": "Suggest precautions to prevent ringworm/fungal infections",
"src": "Patient: Hello. My 6-year-old daughter was diagnosed with ringworm on the back of her hand and her palm. She was given a prescription and is being treated, but I m concerned about it spreading to other parts of her body since it is on her hand as well as others in the family being exposed. Besides washing her hands frequently, what are other ways we can prevent further spreading? I ve read that the fungi can live on items such as clothing and furniture. Doctor: HIWell come to HCMIf this is the ringworm infection then this need to be treated with Miconazole cream local application and Grisofulvin 100 mg once in day for one week, but it is important to confirm the diagnosis, because this is very important for treatment to start, hope this helps."
},
{
"id": 168338,
"tgt": "Suggest remedy for hydrocephalus",
"src": "Patient: Hello, I work in an NGO in Tanzania. One of my clients has a 5-6 months old baby with hydrocephalus. This, apparently cannot be treated in Tanzania, so the baby needs to be transferred somewhere he can be treated. But this can take forever in Tanzania. I would need scientific evidence that explains the urgency to treat this baby in order to pressure the decision-makers to send this baby away for treatment ASAP. Could you help me with this, or do you know where I can find information? Thanks a lot! Doctor: Hi,Welcome To Healthcare Magic.I can understand your concern.I have seen patients with same condition. So, I strongly suggest you, to go for pediatric neurosurgical opinion earliest because as u know this condition is little bit serious and requires intense treatment at multi-specialty hospital where life threatening diseases are treated under supervision of experts according to need.So, I strongly recommend you to take the child for consultation without waiting to the nearby center where all above facilities are available.Wish that child a good health,Take Care, Thank You.From:- Dr. Prashant Shah"
},
{
"id": 64099,
"tgt": "What causes increase in the size of breast lump after breast massage?",
"src": "Patient: hallo Dr, i have a pain in the left breast and i have already seen the Gynecologist. he has said that there is no problem in my breast. is it ok to use ampiclox. or can i continue to massage it? infect in the beginning, the lump was very small, but when started to massage it, it was extended to be big! so, i go on? or i stop it? Doctor: HI,Dear,Thanks for the query to HCM.I understood your concerns about the painful lump in the breast.when your doctor has told you that its normal,Why you want to start antibiotics on your own?Why you massaging your painful breast ?The increase of breast lump-may be -due to increased blood circulation and supply and does not appear to be because of the infection in it -as your doctor has ruled it out.And also there is nothing to indicate in your query facts.So stop all this and take second opinion if -you still worry.Hope this would help you to plan the treatment with Your family doctor.Wishing you early recovery.You are always Wellocome for further query to HCM in this regard.Have a Good Day...!!"
},
{
"id": 120661,
"tgt": "How bulging disc and constant leg pain can be treated?",
"src": "Patient: I have sciatica and the dr. was unable to remove L4 & 5 last friday because of severe inflamation he also said i had a bulding disc that did =nt appear on the mri and from the sound of it he had difficulty. now its constant pain radiating down my leg, Doctor: Hello,I read carefully your query and understand your concern. Your symptoms seem to be related to sciatica pain.I suggest using anti inflammatory medications such as Ibuprofen to relieve the pain. I also suggest to do gentle stretching exercises. If the symptoms continue, you may need a prescription painkiller,so I suggest to consult a neurologist.Hope my answer was helpful.If you have further queries feel free to contact me again.Kind regards! Dr.Dorina Gurabardhi General &Family Physician"
},
{
"id": 26255,
"tgt": "Is it safe to take supplements like niacin,fish oil and magnesium to lower the blood pressure?",
"src": "Patient: I am 55 yo woman - have not exercised for a few years after a foot injury - BP ranges from 145 over 80 to 139 over 72 - I get a really high rate at doctors office 175 over 90 as I hate the blood pressure cuff and the way it makes my arm feel. I am taking a bunch of supplements like niacin, fish oil, coenzyme Q10, Coleus foskholii, magnesium and potassium to help lower my blood pressure. I always have eaten healthy. Is it safe for me to start exercising? Doctor: Hello!Welcome and thank you for asking on HCM!I understand your concern and would explain that the episodes of high blood pressure during doctor appointments are related to anxiety, also called White Coat Syndrome. These episodes are not harmful and do not need any treatment. Regarding your blood pressure values, during most of the time, you seem to be borderline blood pressure. This means that for the moment some lifestyle modifications, besides the supplements you are taking would be helpful to reduce you blood pressure: -modify your diet (reduce salt and caffeine intake)- perform a lot of physical exercise (4 km of walking per day is recommendable, aerobics, etc)-lose weight, if you are overweight. Hope to have been helpful!Best regards, Dr. Iliri"
},
{
"id": 173332,
"tgt": "What causes irregular heartbeat to a newborn?",
"src": "Patient: Hi,I am a very concern sister. My sister just had a new born baby and they are in the hospital now for four days, your new born baby has an irregular heart beat, I will like to know what cause this to happen and how it can be corrected? ( This is St.Vincent and the Grenadines).Thank you Doctor: The heart\u2019s regular rhythm or beat is maintained by a small electrical circuit that runs through nerves in the walls of the heart. When the circuit is working properly, the heartbeat is quite regular; but when there\u2019s a problem in the circuit, an irregular heartbeat, or arrhythmia, can occur. Some children are born with abnormalities in this heart circuitry, but arrhythmias also can be caused by infections or chemical imbalances in the blood. Even in healthy children, there can be other variations in the rhythm of the heartbeat, including changes that occur just as a result of breathing. Such a fluctuation is called sinus arrhythmia, and requires no special evaluation or treatment because it is normal. If your paediatrician says that your child has a true arrhythmia, it could mean that her heart beats faster than normal (tachycardia), very fast (flutter), fast and with no regularity (fibrillation), slower than normal (bradycardia), or that it has isolated early beats (premature beats). While true arrhythmias are not very common, when they do occur they can be serious. On rare occasions they can cause fainting or even heart failure. Fortunately, they can be treated successfully so it\u2019s important to detect arrhythmias as early as possible."
},
{
"id": 153435,
"tgt": "Suggest medication for infections and vomiting after an operation of both limps being removed?",
"src": "Patient: my mother had a big operation both limps removed in upper legs cancer removed on front tip of vagina, and keeps getting urine infections vomiting in and out of hospital,with dehydration since operation in January 2014 she is 70 . is this normal recovery on amocillian 750 gram 250 twice a day. Doctor: Hi,Thanks for writing in.Since your mother has recently got surgery done to remove cancer and lumps therefore she might be under care of her doctor and taking amoxicillin under prescription. Please allow her to complete course of amoxicillin which is an antibiotic as recommended by the doctor. Changing antibiotic is not required if the recovery is going on well.For vomiting, she should be given medicines like domperidone or ondansetron in combination with proton pump inhibitors if the vomiting is due to gastritis. Please discuss this point with her doctor and then give her the medicine as recommended. It is possible that her symptoms are from lying down in bed for long time. Please make her sit up for at least 30 minutes after meals and give adequate fluids. Please do not worry."
},
{
"id": 187989,
"tgt": "What is the treatment for scalded tongue, lump on neck, jaw soreness and dry mouth?",
"src": "Patient: Tongue feels as though it has been scalded. Small lump on right side of neck. Soreness in right lower jaw w/slight swelling. Dentist took xray. Confirmed it is not my teeth. Mouth staying dry. Have already taken antibiotics for 7 days for sinus infection. Doctor: hi,thanx for your query. reasons for scalded tongue are :nutritional deficiency,oral candidiasis a fungal infection of mouth,diabetes,anxiety,depression. this small lump on right side of neck is a lymph node swelling ,might be due to infection.soreness in right lower jaw might be because of infection to sub mandibular salivary gland.burning mouth can be treated by sipping fluids in regular intervals , use of alcohol free mouthwash,making changes in diet and lifestyle,spicy ,hot & acidic food should be avoided.a very simple and tasty medicine is mixing honey with milk and coating the tongue with the mixture.identifying the cause of the disorder is the key to starting the right course of treatment recommended for this condition. hope this is helpful."
},
{
"id": 161655,
"tgt": "Suggest treatment for cough",
"src": "Patient: I have a daughter that is 3months old she was born at 27weeks and weighs a little over 6lb she had a synagis monday and ever since then she had been coughing and not wanting to wake up to eat its like she is trying to sleep until the cold is gone.. She hasnt ran a fever i checked it about 30mins ago and it was at 96.5 so i rapped her up every nurse that i have talked to said there is nothing they can give her what should i do? Doctor: Hi, A 3 months old neonate may develop cough due to some allergy, mild viral illness etc. Generally, we will not use drugs to suppress cough in this age group. You can use simple saline nose drops only. avoid any allergic substance around the baby, like talcum powder, smoke, dust etc. If there is no fever then no need to worry. We can wait and watch. Cough is a protective reflex so no need to panic. Hope I have answered your query. Let me know if I can assist you further. Regards, Dr. Rajmohan, Pediatrician"
},
{
"id": 91544,
"tgt": "What does abdominal aorta atherosclerosis without aneurysm mean?",
"src": "Patient: I just had a ultra sound of my stomach and they found [Adbominal aorta atherosclerosis] with out aneurysm. But what does this mean no one will give me a nuts & bolts answer please help. I live in a very small county with very poor medical care. I could use some direction. Tnank You. Doctor: HI. If you are elderly , nothing to worry about. This is a process of coating of the internal layer of the aorta by atheromatous plaques and are found in many patients without any symptoms.This may be taken as a aging process provided you do not have any medical issues. If the integrity of the aorta is normal , just forget about this. If you are still bothered , consult a Cardiologist who would ask you to undergo few tests like blood lipid profile and may advise you to take some medicines like statins to reduce this ."
},
{
"id": 166842,
"tgt": "What causes loose watery stools in a baby?",
"src": "Patient: My daughter is having loose stools and she is 8 months now. I have given her gripe water but it s not really working. Now I don t know if it is symptom of teething or she has some stomach problem because it is showing lot of water in her stool and I am afraid of Diarrhea. Doctor: Hi,You should give her oral rehydration solution with every time she passes loose stool. Along with an anti-diarrheal drug, offer only soft and easily swallowed food. This should take 2 or 3 days to recover.Hope I have answered your query. Let me know if I can assist you further.Regards, Dr. Salah Saad Shoman"
},
{
"id": 112502,
"tgt": "Have degenerative lumbar spine spondylolisthesis. Numbness in back, pain in legs and hips. Meaning?",
"src": "Patient: hi, my name is phillip. i have degenerative lumbar spine , spondylolisthesis of L4 on L5, endplate spurring in the proximal lumbar spine, the small of my back goes numb and down the right buttock just to the touch, and pain in both legs and hips. i have a lot of back pain . the same thing with my neck. i have asked my Dr but don t get much of an answer. what does all this mean? i am 35 and healthy. doing activities is starting to be too much, this all started around my junior year in highschool and seemed to get worse over time. your input and advice is appreciated. Doctor: Hello. Thanks for writing to us. Your spine has undergone a degenerative change that is causing a compression of the nerve roots coming out from the spine. This is responsible for the symptoms that you are having.I hope this information has been both informative and helpful for you. Regards, Dr. Praveen Tayal ,drtayal72@gmail.com"
},
{
"id": 43184,
"tgt": "Low egg production. Undergoing ICSI, given buserelin injections and Gonal F. Due to low count given Humog",
"src": "Patient: im 31 yrs old, and going through icsi. Husband azoospermia. im diagnozed with endometriosis. started buserelin injections on 30/Sept. started gonal f on 26-oct.scanned on 31-st oct, Dr. only saw 3 eggs. now, they given me humog. Going for another scan on 04th/Nov.Question: why am i not producing a dozen eggs as predicted by the doctors' considering my age? this Doctor: Hi, since you are 31 years only, you should have more eggs. Please get blood test called AMH and send me report. If possible please take second opinion about scan from a second doctor for egg count in good IVF Centre. Regards,"
},
{
"id": 110260,
"tgt": "Suggest remedy to relieve back pain quickly",
"src": "Patient: Hello; i'm 31 yrs old man; I am a film director and i have to go for outdoor shooting from 2nd november. i am suffering from a back pain since 10 days ago. I am taking Hifenac-Mr twise a day from last two days. Anyhow i have to go for the shoot. please help Doctor: Hi,Welcome to healthcare magic.After going through your query I think You are suffering from acute backache.Treatment of it is rest and analgesics . Diclofenac three times a day after meals is effective. Sometimes strong analgesic(such as ultracet three times a day after meals) is required. OMEPRAZOLE 20 MG before meals prevent acidity caused by analgesics .To further investigate MRI of the region is advised. I think your query query answered.Welcome to any follow up query"
},
{
"id": 86609,
"tgt": "Suggest treatment for mesenteric adenitis",
"src": "Patient: Thank you Doctor, After 3 months and 2 CT scans, I was given a diagnosis of Mesenteric Adenitis. The pain is acute and comes and goes, with each episode getting worse. The Adenitis was viral so I was not given antibiotics, just Percocet and an anti nausea medicine. It has been a week since my diagnosis and I am still in bed from the pain and nausea . I read that this condition can actually lead to an acute appendicitis if not treated properly. My appendix showed no swelling and my white count was 0. Do I have a reason for concern ? Thank you. Bonnie C. Doctor: Hi Bonnie.Thanks for your query.It was wonderful to know that it took 3 months for a diagnosis of Mesenteric Lymphadenitis.This is a condition in which the multiple lymph nodes in the mesentery get enlarged. The commonest cause is always secondary to some infection or cancer. I thin you are less investigated than required as you are still in bed with pain and nausea.I would advise you the following:-Consult a General Surgeon.-Get further investigations to see if there is any other problem related to blood cancer or so. You writing of white count of 0 is not acceptable, this is either your you ignorance or something else you could not note.-You need a Diagnostic Laparoscopy and Biopsy of the lymph nodes for the proper diagnosis. This allows to see all the interior of the stomach and appendix and other parts of the intestine and pelvic contents. Can tackle any problem hence seen. -A course of broad spectrum antibiotics and anti-inflammatory medicines. I hope this answer will help you to get to the root-cause and proper treatment."
},
{
"id": 56926,
"tgt": "Can a coarse liver cause abdominal pain?",
"src": "Patient: hi i had an ultrasound today as ive been experiencing severe abdominal pain at night for about 4mths they told me i had what looks like a coarse liver and to go to doctors in a week for my results, i am really concerned please can you tell me what this means? Doctor: Hi,Thanks for posting your query.I am Dr.R.K and I am pleased to assist you.Coarse liver does not cause pain. Only when liver gets enlarged and its covering gets stretched you feel pain in the right upper abdomen.Coarse echoes of liver is seen if there is any scar tissue in liver as in cirrhosis .I hope that answers your question.Regards."
},
{
"id": 166233,
"tgt": "Is a water blister identified as birthmark for a 5 month baby any concern ?",
"src": "Patient: Our five month old baby seems to have a water blister (fairly large) on what our pediatrician identified as a birth mark just inside her elbow (about a half inch long and a bit darker than the skin surrounding). Wondering what might cause it, or if it is something to be concerned about? Doctor: hi, water blister is not a birthmark. Birthmark are usually hyperpigmented or hypopigmented patches on skin. But it is never associated with looks of water blister. You should get the child examined by skin specialist doctor. Take care."
},
{
"id": 111480,
"tgt": "Suggest treatment for back pain",
"src": "Patient: I have a severe back pain from past 5 years, I tried lot of medicines but it will help temporarily. Recently I went to a doctor and gave me 3 types of tablets which should be used when there is lot of pain. And he suggested for excercise only nothing other than that. Is there any final solution to my ache please. Doctor: Hello, I had gone through the case and found that you must go for the MRI of spine and know the exact cause of pain. Also do vitamin D3 blood test.There might be lumbar spondylosis, slip disc, herniated disc or sciatica.Take calcium once aday and Vitamin D3 once a week. If pain is severe then go for physiotherapy. Otherwise do back exercise After getting the diagnosis.Different type of exercise for different cause. But take medicine what I suggest you for 3 months. Your pain will subside.Hope my answer will be effective for you.Thanks"
},
{
"id": 202767,
"tgt": "Large extent tear in the urinary meatus, torn frenulum, cracked penis, erectile dysfunction, peyronie's disease. Treatment advise?",
"src": "Patient: Hello Dr.Chandan .My name is Rakesh having 24 years old.unmarried,uncircumcised, phimosis problem.The main concern is that my urinary meatus(peehole) was torn to a large extent to 2 cm approximately during humping on pillows,bed sheets,sometimes wrapping towels around penis .My frenulum was also tom.Now my penis is cracked,erectile dysfuntion,deformities,shrinking penis,,peyroine disease ,Please help me inorder to resolve my issues. Doctor: First under go surgery for phymosis. Rest of the thing will be alright slowly. The surgeon operating will take care of cracks and tear."
},
{
"id": 26293,
"tgt": "Suggest treatment to control blood pressure",
"src": "Patient: Last week I visited the Doc to check my BP, due to a slight headache, it read 92/190, I was given HCT 50, (Only Once) then I had my BP monitored, for 5 days, No drugs take, But the last read 80/120, three days ago, today I felt a bit abnormal, visited the Doc again today it was 170/105 (Was given HCT50TEN10) Kindly advice Doctor: So it clearly a case of hypertension, initial control u can start with envas 5 mg and betaloc 50mg both once a day. Make a chart of blood pressure two times a day for 5 days and review after 5 days"
},
{
"id": 48318,
"tgt": "What could cause hematuria while on C-.Flox,Avosod and Potassium Nitrate?",
"src": "Patient: I noticed that my urine was pinkish in color and upon checking the doctor confirm there is blood in my urine.The doctor prescribe C-.Flox (antibiotics),Avosoda(Urinary Alkalinizer) and Potassium Nitrate .Jfyi,I had kidney stone 14 yrs ago and can it reoccur? I went for my urine test today and attached is the result for your kind advice.Your kind advice will be highly appreciated --------------------------------------------------------------------------------URINE FEMEAPPEARANCE -HAZYCOLOR -STRAWSP. GRAVITY 1.010 1.005-1.030pH -8.5 (H ) 4.8-8.4PROTEIN -NILGLUCOSE -NILKETONE- NILBLOOD- 3+MICROSCOPICWBC -NILRBC -20-50EPITH -OCCCRYSTALS - & CASTS- NIL** Doctor: Hi!Welcome to HCM!I understand your problem, nothing to worry it will be resolved in a matter of time.As per the problems discussed in your question, red colored urine with the evidence of blood 3+ and 20-50 RBCs will NIL WBC are highly suggestive of stones in the kidney or urinary bladder.To confirm the stones in your urinary tract, get the ultrasound scan done and consult your GP.I hope I have explained the problems listed in your question.REgards!"
},
{
"id": 46232,
"tgt": "Suggest treatment for kidney stone and infection",
"src": "Patient: Hi, may I over the last hear i have been to the same dr numerous times with flu like symptoms including a painful and cough and severe back pain. He Just kept ignoring me and put me on antibiotics. This past week i felt so ill and was crippled by pain in my back and side. Turned out to be very low potassium m\u00faltiple kidney stones and kidney infection Doctor: Hello Welcome to HCM.. I am really sorry to hear this about your health but I'm definitely going to help you out.. I thank you for posting your query here.. I being a surgeon, we encounter so many such cases day in and day out.. I read your question completely and have understood it well enough. I will analyze all your points and provide necessary explanation and how to go about from now on..But, need few other details for better understanding of your case A. Your gender B. Age. C. Kidney stone size and location if anyAnyway, please note these following things.. 1. Severe back pain that you tell, yes seen in renal calculus or stone. I'm sorry that your doctor couldn't get the diagnosis.. 2. Kidney infection and kidney Stone. Well, I would like to tell you that kidney infections are common complications of kidney stones.. And, the cause that is stones have to tackled with priority to avoid further infections. 3. Low potassium may be the cause for renal stones in few patients.. 4. Please do these following things . A. For kidney infection, Get urine routine and urine culture tests. This will tell us the infection extent, organism responsible for infection and antibiotic to be given. This has to be treated first.. Later, for kidney stone B. Various treatment options are available.. And it depends on size and location of stone. Most stones less than 2 cm can be managed with shock wave lithotripsy, a non surgical means. So, let me know the details.. Hope this helps you, in case you need any further assistance, please let me know. Take care and God bless."
},
{
"id": 106785,
"tgt": "What causes severe backache while suffering from UTI?",
"src": "Patient: I went in to see a doctor last weekend on Sunday, it\u2019s Thursday now. I went in due to a UTI, but I\u2019ve been taking antibiotics since Sunday. I had intense back pains when I went in and I\u2019m still having the same kind of pain. I\u2019m really scared that it\u2019s something more than a UTI. Doctor: Hello and Welcome to \u2018Ask A Doctor\u2019 service.I have reviewed your query and here is my advice.I have gone through your query. Better to do a urine culture and sensitivity test to rule out pathogen causing infection and to know the susceptible antibiotic.Ultrasound examination also should be done to rule out possible kidney damage.Hope I have answered your query. Let me know if I can assist you further.Regards,Dr. Nishad BN"
},
{
"id": 139698,
"tgt": "Suggest treatment for Parkinson s disease",
"src": "Patient: Dear Respected Doctor,My 78 year old father is suffering from Parkinson s Decease for the last about 15/17 years. He is 5 ft. 11 inches tall and weighs about 75 kg. He has no other health problem like diabetes, blood pressure or heart problem etc. He is receiving Syndopa plus, Amantrel, pramipaxole and pacetin for the last 8/10 years without any satisfactory response. kindly inform what best can be done? Mail: vijaysharma7@ WWW.WWWW.WW Doctor: Hello, He is having multiple Parkinson\u2019s drugs. If the medical management fails, you can opt for newer techniques like DBS (deep brain stimulation). Consult a neurologist and he will direct you accordingly. Hope I have answered your query. Let me know if I can assist you further. Regards, Dr. Shinas Hussain, General & Family Physician"
},
{
"id": 71329,
"tgt": "Suggest cause for chest tightness in asthma patients",
"src": "Patient: Hello I ve recently been experiencing some tightness in my chest whenever i stand up. I have asthma so I ve been thinking it was just that. I use my inhaler but the pain usually comes back after a few hours or so. I currently live in a college dorm and i try my best to keep it clean and free of allergens. Any advice? Doctor: Hello,The most common cause for chest tightness in asthma patient is worsening of asthma. So, it is better to consult pulmonologist and get done a clinical examination of the respiratory system and PFT (Pulmonary Function Test).If PFT is normal, then no need to worry about asthma as a cause. Sometimes stress and anxiety can also cause similar symptoms. So, avoid stress and tension, be calm and relax.Hope I have answered your query. Let me know if I can assist you further.Regards,Dr. Kaushal Bhavsar"
},
{
"id": 65351,
"tgt": "What does a hard lump below the rib cage indicate?",
"src": "Patient: i have a hard marble size lump on the left side below my rib cage I also have been getting Charlie horses and the sensation that my stomach is turning inside out. I am also have lower Back pain that can get very severe.putting me on my couch with heating pad for most of day and over course of sleep. This has been going on for approximately 2months. I am a 32 stay at home mother of 3 please help Doctor: Hi, dearI have gone through your question. I can understand your concern. You may have some soft tissue tumor or lymphoma or some other mass. You should go for ultrasound test first. If needed go for fine needle aspiration cytology or biopsy of that lump. It will give you exact diagnosis. Then you should take treatment accordingly. Hope I have answered your question, if you have doubt then I will be happy to answer. Thanks for using health care magic. Wish you a very good health."
},
{
"id": 210685,
"tgt": "Suffering from bipolar and depression, what should be taken to replace seroquel?",
"src": "Patient: Hi I take seroquel with lithium and gabapentin for bipolar, i sleep too much and am somewhat stiff, i also do not feel that it is working for depression and only wreaks havoc in manic states making me psychotic and cycle more , i would like stop seroquel but do not know what to replace it with any thoughts? Doctor: HIThanks for using healthcare magicIn bipolar disorder, you would get depression or manic switch. In that case, you need a mood stabilizer that could control these switch. Better to take valproate along with lithium to control further episode. You can carry on serquel for sleep purpose. Rest you can discuss with your psychiatrist. RegardsDr. Abhishek Kapoor"
},
{
"id": 188889,
"tgt": "Diagnosed Periodontal disease, teeth feel loosening of gums, left lymph node swollen. What should I do ?",
"src": "Patient: I recently had a dental appointment for the first time in over a decade due to my fear of the dentist. I was told I have Periodontal disease and my next appt. is to have one side of my gums scraped. Its not Til the 17th of June but for the last few days my teeth feel like they are coming out of my gums and I can't eat a thing without severe pain. Also my left lymph node is swollen and it hurts. What should I do? Doctor: Dear friend.Thanks for sharing your concern.please do not be hesitant in getting dental treatment done as your mouth is the way to your stomach.Therefore any infection in the mouth is reaching your stomach and making it worse.poor oral hygiene also leads to heart diseases especially periodontal infection.This small bit of information is not to scare you but to make you aware of the fact.please get cleaning called (scaling)done.it can be done in two to three appointments based upon the severity of the disease.once the deposits,like calculus and stains are removed,the teeth appears to be loose,which is not so actually.when the deposits are removed,it makes room for the gums to grow up and hold the tooth firmly,thus preventing their mobility.Therefore all your symptoms related to teeth will be fine after complete mouth cleaning(oral prophylaxis).Even the enlarged glands will be infection free.Hope it helps.Thanks.Take care."
},
{
"id": 18693,
"tgt": "What causes tingling sensation in the legs while on BP medication?",
"src": "Patient: Hello, I m 69 years old 5-11 215 lb I can walk 2.4 miles in 45 min , non smoker for last 15 years I am on BP Med and it is controlled 130 over 78.but i am by no means Chiseled Recently when I stand and walk after setting I get a tingle in back of my legs Mostly in my left one and my feet, Walking for a couple of minutes and it goes away never had this before Doctor: Hello and Welcome to \u2018Ask A Doctor\u2019 service. I have reviewed your query and here is my advice. I passed carefully through your question and would explain that your symptoms do not seem to be related to any vascular disorders. Anyway, I would like to know your current therapy, because several anti-hypertensive drugs may lead to such adverse effects. From the other hand, I would recommend performing further tests: - complete blood count for anemia - PCR and ESR for inflamamtion - blood electrolytes for any possible imbalances - uric acid levels. Hope I have answered your query. Let me know if I can assist you further."
},
{
"id": 28186,
"tgt": "What is the life expectancy in a person having Anterior Stemi and had cardiac arrest?",
"src": "Patient: My daughter aged 38 years has just had a MI called a Anterior STEMI, she had a cardiac arrest in the ambulance and had a PCI put in her lad on immediate arrival at harefield hospital. She has a significantly impaired apex with moderate overall contraction LVEF 43%. She no longer smokes. What her life expectancy? Doctor: Hi,In her age if she takes her medications regularly, follows healthy lifestyle she has all chances for good life expectancy.So you should encourage her not to get disappointed, take her medications, eat healthy food, low in animal fat and salt, exercise regularly. It is very good, that she stopped smoking.Hope I could help youWishing you good healthIn case of further questions don't hesitate to askRegards,"
},
{
"id": 214347,
"tgt": "Suggest home remedy for muscle pull",
"src": "Patient: Hi, I visited the hospital last thursday with pains in my shoulder. After xrays they advised it was a pulled muscle. I have had pain in my shoulder constantly and am finding sleeping difficult. I am just trying to find out the best home treatment, as co-codamol and mandafen are not easing th3 pain. Doctor: Hi there.Thanks for writing in.You are already on pain relieving medications. What you can do is avoid sleeping on the affected shoulder .Serratiopeptidase might help relieve the internal swelling/ edema.Vitamin C tablet might help improve healing process.Application of pain relief gel like diclofenac gel might help. The gel should be lightly applied and not massaged on the affected area. If massaged, can cause burning sensation in the skin.Half an hour after application of gel, do hot fomentation on the affected area. Do hot fomentation 3-4 times a day.If none of the measures seem to help, do consult a physiotherapist.You might also want to get your Vitamin B12 levels checked.Hope my answer was helpful to you.Regards,Dr. Divya Kuttikrishnan"
},
{
"id": 38,
"tgt": "Why is Provera prescribed when trying to get pregnant?",
"src": "Patient: hello my name is Brenda, I have been trying to conceive for 8 months with no success. My doctor did blood work and all these other tests to see if i had a fertility problem and the results would always come out just fine. so after that my husband went in and did a sperm count and semen analysis, and the results were perfect too. My doctor has put me on metformin 1000 mg a day. i have been on the metformin for almost a month and a half. the thing is that before all this i used to have very irregular cycles and my doctor put me on provera 'birth control' for 3 months to help regulate my periods which helped for the whole 3 moths, but when i finished my period was gone for about 41 days and after that them have been much better ive been getting them around every 31 to 32 days. Like i was saying my doctor put me on metformin... but i have been searching the web and they only give that to women who have pcos and i dont have pcos. my doctor told me to take the medication for 3 months and to go back after the 3 months to see what would happen. so I just want to know why would she put me on this medication? is it going to help me conceive? Doctor: provera is medrox progesterone. ..it is to get your period....u do serum tsh and serum prolactin den do hsg on day 8th den do follicular study after taking clomiphene from day two den follicle size increase to 18 mm den rupture den iui fr early results"
},
{
"id": 88486,
"tgt": "What can cause sharp pain in lower abdomen?",
"src": "Patient: Hi we've been trying for a baby for 5 months. My last period was 6th feb although this was unusual and lasted 2 Days only. For over a week I have had almost pulling and sharp pains in my left and lower abdomen. I've tested negative this morning is there anything else this could be? Doctor: Hi.The cause of the sharp pain in the lower abdomen can be :Tubal pregnancy - with your history of trying for pregnancy.AppendicitisOvarian cyst twistingTO massSalpingitis and so on.The best way is to undergo an urgent ultrasonography, get a physical examination done by a Gynecologist and / or a Surgeon to ascertain the cause and get treated properly. To confirm whether there is an emergency."
},
{
"id": 78372,
"tgt": "What causes shortness of breath and raspiness in lungs?",
"src": "Patient: My throat hurts really bad, it feel like i swallowed a knife. I have shortness of breathe and my nose is plugged on both sides, and getting up and moving around makes me really dizzy. When i try to breathe in really deeply, there s a raspyness in my lungs. what is it? Doctor: Thanks for your question on Health Care Magic. In my opinion, you are mostly having bronchitis. Viral infection is the most common cause for bronchitis. So better to consult pulmonologist and get done 1. Clinical examination of respiratory system 2. PFT (Pulmonary Function Test). PFT is must for the diagnosis of bronchitis. It will also tell you about severity of the disease. And treatment of bronchitis is based on severity only. You should drink plenty of fluids orally and keep yourself hydrated. Avoid oily and spicy food. Start antihistamines and anti inflammatory drugs. Inhaled bronchodilators and inhaled corticosteroid are also needed. Don't worry, you will be alright. Hope I have solved your query. Wish you good health. Thanks."
},
{
"id": 78662,
"tgt": "What causes chest pain?",
"src": "Patient: i am 33 years old male.i have pain in chest.my ecg,eko,and mri of chest is clear.my dorsal spine have problem in d7,d8,d11,d12.i have no signs of angina .my doctor gave me beta blockers.but i jogging 35 minutes and walk for 25 minutes.plz tell me whats going on with me .i am in tension. Doctor: Thanks for your question on Health Care Magic. I can understand your situation and problem. Since your extensive cardiac work up is normal, no need to worry for heart related diseases for your chest pain. You have been prescribed beta blocker for anxiety and stress. Since you are in tension, possibility of stress and anxiety related chest pain is more in your case. Along with drugs (beta blocker), counseling is also important. So better to consult psychiatrist and get done counseling sessions. Try to identify stressor in your life and start working on its solution. Don't worry, you will be alright. Avoid stress and tension, be relax and calm.. Hope I have solved your query. Wish you good health. Thanks."
},
{
"id": 81729,
"tgt": "What causes difficulty in breathing after intake of certain foods?",
"src": "Patient: I cannot breathe at night if I ingest casein, peanut products, almonds, or soy products. I m wondering if this problem is related to non-alergic sensitivity to these foods or is it related to inflammatory response? How to overcome? I use a CPAP machine... I also use Triamsinalone corticosteroid possibly linked to glaucoma which I want to avoid using and I also use another nasal Spray... can t rememaber its name off-hand... --Bill-- Doctor: You are an allergic person as you are using nasal spray i suppose.The difficulty in breathing that you describe after ingestion of certain food products can be due to your allergy and lead to an asthma attack sometimes.However you are on triamcinolone steroids that would be the treatment for any allergic disease but for an instant relief it wont help and hence you need to use an inhaler that provides instant relief.You can take ventorlin or asthalin nebulisations or inhalation sos"
},
{
"id": 6666,
"tgt": "Are there any chances of pregnancy if their is rupture in ovulation sac and sperm count is 127 million ?",
"src": "Patient: hi , my sperm count is 127 millieon and active motile percentile is 31% is it normal ?is it fertile? my wife had sonography report saying ruptured ovulation follicile sac on 15 th april and she gets her period on 10th may which is early in cycle date which is 12 of month what is the chances of pragnancy please advice Doctor: Hi Amit, Welcome to HCM. Your sperm count is normal but motility is reduced. Normal is 80 %."
},
{
"id": 15884,
"tgt": "Painful cluster rashes on the upper arm. Cause?",
"src": "Patient: On Saturday, I went to Catalina Island on a ferry...and on the way back home, i noticed a rough cluster of rash like bumps on my inner upper arm. They are quite painful (not excruciatingly) and have now formed black dots in the middle of the bumps. The pain is still there. What might this be? I dont remember being bitten by any insect or anything...and i cannot think of anything that may have caused it Doctor: Hi, Clusters of bumps on your skin can be alarming , as bumps may be a sign of an infectious or allergic process going on your body. Skin infection can also occur due to any reason. The bumps are due to either allergic reaction to something or infection in skin. The reason for allergy could be any substance which comes in contact frequently, may some cream or chemicals etc, Itching leads to damage the layer of the skin and leaves a scar. Apply Dhurva thailam daily."
},
{
"id": 13237,
"tgt": "What causes rash to spread on body?",
"src": "Patient: I apparently came in contact with poison ivy 5 days ago. A rash started to show up on my arms the next day and my lips began to tingle. It s not the first time I ve had poison ivy so I went to the store and got some Benadryl and some Ivarest cream. By the next morning my lips were swollen with little blisters on the edges and my eye was almost swollen shut. I went to the doctor s office and got a shot and prescriptions for a topical cream and a pill pack of methyliprednisolone. I have been religiously taking the pills and applying the cream as well as taking baths in Aveeno oatmeal bath twice daily but while the rash is getting better in some areas it is continuing to spread in others (my face is nearly cleared up but now the rash is on my stomach where it wasn t before). My other concern is there is a softball size rash on the back of my knee that is turning dark purple. I m just wondering if I should be worried or if there is something else going on? Doctor: Hi Dear,Understanding your concern. As per your query you have symptoms of rash which is spreading across body which is very common due to allergic reaction and contact dermatitis.Need not to worry. I would suggest you to keep this area clean and dry , apply cream containing azelaic acid , apply warm compresses with soaked towel and apply vitamin C serum at night. You should avoid touching or pricking rashes. If condition doesn't get better then consult dermatologist for proper examination . Doctor will examine physically along with blood test and sample for lab test . Doctor may prescribe immunosuppressants , antibiotics like clindamycin , anti inflammatory or anti fungal . Doctor may also prescribe benzoyl peroxide ointment .Hope your concern has been resolved.Best Wishes,Dr. Harry Maheshwari"
},
{
"id": 188775,
"tgt": "Taken antibiotics for hole in gum after wisdom teeth removal. Cause?",
"src": "Patient: I had my wisdom teeth out years ago and have never had any problems until recently when it felt like the gum had ripped open leaving a hole on the inside of my gum under where the wisdom tooth would have been. Dentist gave me antibiotics but no better. Now having to be referred to dental surgeon. Any ideas what could be causing this? I am also having toothache when I chew on the tooth next to it Doctor: Hello,Usually the socket heals after 1-2 weeks post extraction.The pain may be due to bone fracture or nerve damage.A thorough clinical as well as x-ray evaluation will help to aid the cause of toothache.Meanwhile,maintain oral hygiene well.Gargle frequently with warm saline or betadine.Avoid impinging the socket with fingers/pins.Please do visit a dentist and get the treatment done.Hope this helps."
},
{
"id": 52539,
"tgt": "What causes high Urea and Creatinine?",
"src": "Patient: my mothers urea was 72 and creatinine 1.6 14 days back she is diabetic hypertensive and had lung effusion and water retention in whole body then doctors started various treatments for 10 days and gave her dytor 200 mg iv a day nw aftr 1 week of duischarge we did a kft urea is 275 and creatinine is 2.7 what and why is this will it get back to normal now Doctor: Hello welcome to 'Ask A Doctor' service.I have reviewed your query and here is my advise.You were having fluid retention in body. For that torsemide is given and it is good treatment. Your serum urea and creatinine increased suggestive of nephropathy. Hence hypertension and diabetes has to be controlled with medication to improve kidney function.Take low salt diet. Telmisartan like antihypertensive drug can be useful in renovascular hypertension. Doppler renal angiogram and USG scan advisable. I suggest you to consult Urologist for examination. Hope I have answered your question. Let me know if I can assist you further."
},
{
"id": 159879,
"tgt": "What is the life expectancy rate for a patient having pancreatic cancer ?",
"src": "Patient: hi, if a 39yr old male was to be diagnosed with acute pancreas cancer , how long should he expect to live? Doctor: hello welcome to healt care magci,this totally depends ombern tnm classification that is number of metastasis ,i.e the spread of tumour is in which region ,size of the tumour age of the patient as young cells can resist more than the older ones and history of the patient and so on ,depicting all this can diagnose how long is he expected to live.take care"
},
{
"id": 192224,
"tgt": "Is my penis normal?",
"src": "Patient: hello sir i am vettrivel from coimbatore..i having one doubt about my sex organ...for every human being the skin cover 75% of sex organ and at edege it was some big ..but for me the skin cover full sex organ..i have only small hole at the end of my pencis Doctor: Hello, If your foreskin covers your penis head fully, it is normal. But you should make sure that you are able to pull down your foreskin completely back. If you experience pain during pulling it backward you should consult a surgeon for a minor surgery named circumcision to correct the problem. Therefore I suggest consulting a surgeon for physical examination, diagnosis and treatment. Hope I have answered your query. Let me know if I can assist you further. Take care Regards, Dr. K. V. Anand"
},
{
"id": 219757,
"tgt": "What are the early signs and symptoms of pregnancy?",
"src": "Patient: iv been on microgynon for about 6 months now and havnet missed any pills but have been late taking them a couple of times (only a couple of hours out) and have been on antibiotics once during that time but more often than not we use condoms as well. for the last 4-5 months iv been suffering from pregnancy like but still getting bleeding on the right days and only had negative tests so far (iv taken about 6 over the last 4 months), im wondering could it be the pill causing my symptoms or do i need to go get hCG blood test done. i would prefer to avoid the blood test since im phobic of needles but i need to know so i can try get prepered for 3 children if i am pregnant Doctor: Hello dear,I understand your concern.In my opinion there might not be any chance for pregnancy in your case.The following points favour the above statement:-1)There was no irregular intake of pills.Taking pill a few hours late doesn't alter the effectiveness of pills.2)You got the periods on time and there was no delay.3)All the urine pregnancy tests have been negative.Though antibiotics effect the birth control pills all the antibiotics might not do so.Only a few like rifampicin,rifabutin effect the hormonal pills.So relax.There is no need of any blood HCG testing.If there is any pregnancy it would show up on urine pregnancy test also though late than blood test.So nothing to worry.Avoid stress.Stress regarding the thought of pregnancy also makes one to feel the pregnancy symptoms.Hope this helps.Best regards...."
},
{
"id": 8088,
"tgt": "To what extent can laser treatment help to remove black spots ?",
"src": "Patient: I am having black spots after pimples on my face , how to remove the spots? please suggest remedy. Laser treatment help up to what extant? Doctor: Hello Jpbiosc, Welcome to Healthcaremagic! Acne scars/ post acne pigmentations respond to a number of treatment methods like peelings, microdermabrasion, dermaroller, radiofrequency, & LASERs. Topical creams are also useful alone or in combination with these treatments. LASERs are of different types. All these methods are operator-dependant & with a good doctor any of these methods will give you about, say, 50-60% improvement. Cost varies from methods to methods & also between different centers. I hope this clarifies your query."
},
{
"id": 52556,
"tgt": "What does this liver function test result of an elderly person indicate?",
"src": "Patient: My wife has a score of 15.8. Her liver doctor said if it goes to 16.? she ll need to go on the transplant list. She s has fibromialgia and is not well most of the time. She s 69 and has memory loss, constant diarrhea, sleeps a lot and bruises at a touch. My question is what is the score called and what does it mean? Doctor: Hello and Welcome to \u2018Ask A Doctor\u2019 service. I have reviewed your query and here is my advice. The score you have mentioned may be some thing called MELD( model for end stage liver disease). It is calculate based on bilirubin and Creatinine If the values are above 15 we may consider liver transplantation. Hope I have answered your query. Let me know if I can assist you further."
},
{
"id": 73765,
"tgt": "Which type of bronchitis displays prominent hilar congestion and vascular markings?",
"src": "Patient: There is prominence of both hilar shadows with prominent vascular markings suggesting bronchitis in both lower zones, otherwise, both lung fields and either C.P. angles are clear.No evidence of any cavitory leson, which type of bronchitis, its treatment and is it curable? Doctor: Thanks for your question on Healthcare Magic.I can understand your concern.If you are smoker and having chronic complaints of cough, breathlessness then you are mostly having chronic bronchitis.So get done PFT (Pulmonary Function Test).PFT will not only diagnose bronchitis but it will also tell you about severity of the disease and treatment is based on severity only.You may need inhaled bronchodilators (formoterol or salmeterol) and inhaled corticosteroid (ICS) (budesonide or fluticasone).Quit smoking if you are a smoker.Hope I have solved your query. I will be happy to help you further. Wish you good health. Thanks."
},
{
"id": 76226,
"tgt": "Can occasional chest pain after assault indicate damage to heart?",
"src": "Patient: I a male and am 43 years old about two meters. I was assaulted by a man he kicked me hard on the chest, I lost brief consciousness. I occasionally feel a bit of pain on the area of the heart. Could there be damage in my heart and will it affect my heart health life span? Doctor: Hi. The blow to the chest wouldn't damage your heart but this may be a rib fracture or a breastbone fracture so you need x-rays to check for those possibilities"
},
{
"id": 14812,
"tgt": "What causes a red rash under the arm?",
"src": "Patient: I have a red rash under my arm. Last time I had it, I was pregnant with my daughter & gestationaly diabetic. I am now neither diabetic nor pregnant. A doctor said then that the rash was related to kidney function - or the lack thereof... Just wanted some insight? Doctor: Hello,Thanks for the query,Red rash under arms can be a fungal infection.The area of skin folds is very moist and is prone to develop these infections.Treatment can be with oral antifungals as well as with topical antifungals.Various antifungal powders like candid are available.These can help reduce the moisture as well as control the infection.Please meet a dermatologist for exact diagnosis.Let me know if you have any other doubtsThank you"
},
{
"id": 73587,
"tgt": "Suggest treatment for hyperaeted lungs with low set diaphragm",
"src": "Patient: Hi, muay I answer your health queries right now ? Please type your query here... both lungs are hyperaeted with low-set diaphragm, Hazed and fibrotic densities are seen in the right upper lobe with intervening lucencies. The right interlobar fissure is tickened.Heart is not enlarged. Costopheric sulci are normal. thank you Doctor: Thanks for your question on Healthcare Magic.I can understand your concern.I have gone through the x ray report you have mentioned.This report is suggestive of old healed fibrotic lesions with emphysematous changes (hyperaereted lung field with low set diaphragm).So get done PFT (Pulmonary Function Test) to identify the functional loss of lungs.If PFT is suggestive of obstructive Airway defect then patient will need inhaled bronchodilators (formoterol or salmeterol) and inhaled corticosteroid (ICS) (budesonide or fluticasone).Deep breathing exercises, incentive spirometer, yoga and pranayam are also good in restoring lung functions.Hope I have solved your query. I will be happy to help you further. Wishing good health to your patient. Thanks."
},
{
"id": 103120,
"tgt": "Sneezing, taking medication, no improvement. Remedy?",
"src": "Patient: hello sir, am 24 year old student. 24*7 i am like sneezing, winters or summer now everything is equivalent to me. when consulted a doctor, he checked my blood reports and said i have blood infection. Now its been 1 year am taking his medicines and still i have the same problem. Please help me what to do and how to cure it. Doctor: sneeze is always allergy and it requires anti allergy tab not antibioticas it is chronic infection and superaded infection shows infection of bloodneed to use anti allergic tabsneomycin h ointment in nose bd sea water 2 drops at night each noseuse continuously till you consult ent doctor to find cause of allergies"
},
{
"id": 198869,
"tgt": "What causes burning/pain in left testicle?",
"src": "Patient: Hello DoctorI have a dull pain in my left testical. Have consulted my family physician. He has prescribed ezoflam and citro soda. My pain has reduced but frequent urination has started from last 2 days. However there is burning or pain. What should i do Doctor: HelloI share your concernLooking at your description there could be a possibility of urine infection I would advise you for urine culture and sensitivity test and accordingly a course of antibiotics is recommended under supervision of doctor, based on culture reportMeanwhile have plenty do fluids 3 to 4 liters a dayYou can take urine alkalizing agents to reduce its acidity and subsequently burning.Hope this answers your questionPlease feel free to ask for follow up questions I will gladly answer you Best wishes"
},
{
"id": 59287,
"tgt": "Pain in upper right abdomen. Due to gall bladder issues or pulled muscle?",
"src": "Patient: This is the third day in a row that I have had a pain in my upper right abdomen . I only feel it when I take a deep breath or do certain workouts. It hurts enough that I have been cutting my workouts short or modifying them. I am optimistic that it might be a pulled muscle, but am fearful that it is my gull bladder . Any ideas what might be causing the pain? Doctor: Hi Pain in the upper part of the abdomen on right side may be due to Gallbladder stones.It may also be due to kidney stones. A simple Ultrasound abdomen will confirm the diagnosis. If Ultrasound abdomen is normal,then it can be musculskeletal pain. Please take anti inflammatory drug till Ultrasound abdomen is taken. If you come back with the report,i will clarify Wish you good health Regards"
},
{
"id": 218484,
"tgt": "Is pregnancy possible without penetrative intercourse?",
"src": "Patient: Hello I have a question me a a guy did oral he came and it got on my hand I wiped it off and went to use the bathroom when I wiped part of my hand touch my vigina after I went home 20 min later and washed with summer eve soap in the shower is there possibility of me getting pregnant? Doctor: Hello and Welcome to \u2018Ask A Doctor\u2019 service. I have reviewed your query and here is my advice. According to your question, the possibility of pregnancy is less as there was no direct insemination but cannot be completely ruled out. Washing will usually eliminates the risk but if you touched before that, there is little chance. The chances are little more if you were around the time of ovulation at that time, that is around 14th to 18th day in case of 28 to 30 days menstrual cycles. Better to wait till your expected period of menstruation, if at all you miss it, please go for urine pregnancy test once to rule out the possibility of pregnancy. Hope I have answered your query. Let me know if I can assist you further."
},
{
"id": 111538,
"tgt": "Is severe lower back ache related to hysterectomy?",
"src": "Patient: Hello, I have an 11cm fibroid and am awaiting a hysterectomy. I have severe lower back ache on the right side and wondered if this was linked and is there anything I can do to ease it. No tablets seem to help. I also get a shooting pain in my pelvis and down my leg. Thanks, Sash. Doctor: Hello,I had gone through the case and found that this pain might be pressing of sciatica nerve. During hystrectomy , doctor gives spinal anesthesia, which some time causes puncture of nerve.So go for CT or MRI of lumbo sacral vertebrae and diagnose the cause and also take Vitamin D3 and Vitamin B12 blood test.treatment is to take nerve strengthening capsule, take physiotherapy massage and do leg exercise .Hope my answer will be effective for you.Thanks"
},
{
"id": 120882,
"tgt": "What causes muscle cramps and pain in quadriceps?",
"src": "Patient: I am a 33 year old active female (5 7 , 125 lbs) that struggles with chronic muscle cramps & pain in my quadriceps. It usually occurs if I sit for longer than 20 min. If I get up and move it helps relieve some of the pain and discomfort so I try not to sit for too long. I have never taken anything for the pain. I do however have several family members with autoimmune disorders. Doctor: Hello,I read carefully your query and understand your concern. Your symptoms seem to be related to muscle cramps.I suggest using a spasmolytic such as Baclofen three times a day.I also suggest using magnesium supplement for muscle relaxation. I recommend warm compresses for local application. Hope my answer was helpful.If you have further queries feel free to contact me again.Kind regards! Dr.Dorina Gurabardhi General &Family Physician"
},
{
"id": 174060,
"tgt": "What type of food to give for my baby?",
"src": "Patient: Hi Doctor,My baby boy is 9 1/2 months old, having greenish liquid since last 1 1/2 months. We dont know what to do??? We consulted so many doctors, every one says its normal. One doctor told me that dont give cerelac, we did it & it works. But now what to give as an alternate to Nestle Cerelac??? Doctor: Hi,Thank you for asking question on health care magic.this is the time when you should wean him from breast feeding and shift to natural foods.Cerelac is not the only solution.Give him fruit juice ,boiled and smashed potatoes,smashed ripe banana, double boiled rice with curd or buttermilk.Only curd with sugar can be given.Double boiled rice with moong dal ,boiled rice with milk and sugar etc.,Not only cerelac you can choose alternative milk cereal mixtures from the market.Hope this answer will serve your purposePlease feel free to ask any more queries if requiredTake careDr.M.V.Subrahmanyam MD;DCHAssociate professor of pediatrics"
},
{
"id": 219255,
"tgt": "What causes labor pain during 7th month of pregnancy?",
"src": "Patient: Hi, my wife is 7 months pregnant, she got preterm labour pain from 26 week, my gynac suggested to take duvidilan retard. She is taking full rest. I just want to know can this medicine lead us to 9 months to deliver baby? Or there is a risk of delivering a baby before it get mature. pls help. Doctor: Hello, Thank you for your query. Duvadilan retard (isoxsuprine) relaxes the uterine smooth muscle, thus preventing Labour. If your wife takes adequate rest and medications there is a very good chance that she will complete the term and deliver a full term baby. Is her cervical length normal?(more than 3 cms). If less than 2.5cms, chances of preterm Labour is higher. This can be treated by cervical encirclage followed by bed rest. Monitor the baby movements. Have your wife practice some relaxing techniques like meditation, breathing exercises to calm her mind. False contractions (Braxton - hicks) will be present now and can happen at any time. This is usually felt as a tightening of the uterus and may be associated with some discomfort but not pain. It is irregular and can be triggered by emotions, hunger, full stomach or baby movements. If the contractions are regular and become more frequent with period like pain, contact her obstetrician right away. In this time, try not to panic and practice calming breathing exercises along with your wife. If it occurs after 37weeks, baby will be mature enough to survive on its own, provided there are no other complications. If this occurs before 37 weeks, steroid injections to speed up the lung maturation process may be necessary. If before 35 weeks, attempts to delay Labour will be made. If that doesn't work out, your baby may need close monitoring in an intensive care till he is stable.This is just so that you may be prepared. In most cases, we are looking at the first outcome. There is a risk of preterm Labour. However, if monitored closely, with rest and medications, this can be minimized. Being prepared for any event is necessary. Keep the doctor's, hospital's Labour room numbers handy along with a few people who will be able to assist you during that time. Pack a weeks worth of clothes for yourself, wife and baby. Plan for someone to travel back and forth between hospital and home or someone to stay with your wife while you do so. Hope this helps. Please let me know if there is anything else I can help you with. Wish you all good health and I pray for a safe confinement."
},
{
"id": 189000,
"tgt": "Painful lump on jaw after molar teeth extraction. Taking painkillers. Normal?",
"src": "Patient: I just had my lower molar extracted on thursday and I have a golf ball size lump in my jaw line and is in horrible pain.. we back to dentist today and he said that I do not have a dry socket but I have had one before and it seems as I do have a dry socket. I am having to take painkillers every 2 hrs. is this normal? he said it was a difficult extraction and that the gum tissue is very swollen..I cant seem to get the pain under control. what can I do? Doctor: Hi,Thanks for asking the query,The swelling can be as a result of trauma caused at the time of extraction. There could be a chence of any root piece left in the socket to rule out this you need to visit the Dentist and get an x-ray done. Take complete course of antibiotics and analgesic prescribed to you by the Dentist.Take lukewarm saline rinses and antiseptic mouthwash gargles.Apply ice packs externally over the affected area.Take care!"
},
{
"id": 130685,
"tgt": "What does chronic back, hip and joint pain indicate?",
"src": "Patient: i have been suffering with chronic back hip and knee joint pain for the last seven years i am attending my gp i am attending a pain specialist and ive had scan and mri s and hip x rays but im still not been told what is wrong ive been getting cotasol injections in the hip and the back but im getting no releife imhardly able to move around and im getting worse # Doctor: Hi,Chronic back pain and joint pain indicates weakness of muscles. So doing active exercises and taking diet rich in calcium, vit b 12, protein with lot of green vegetables and fluids may help you out. You can also opt for home physiotherapy services from near by physiotherapy center. Your physio will help you out with exercises and in pain management also. And keep your surrounding warm and wear woolen clothes.Hope you find the answer useful. Let me know if I can assist you further.Regards, Dr. Harsh Swarup"
},
{
"id": 133865,
"tgt": "What causes pain in the groin/hip area?",
"src": "Patient: hi I have a strong crampy pain in my left groin/hip area. This has gradually increased in intensity over about 3-4 weeks. I am 51 years old. I am otherwise very healthy and fit and tonight I could not complete a pump class at the gym. I thought it may be appendicitis but now know this occurs on the right side. Doctor: hi,thank you for providing the brief history of you,A detailed assessment is advised. Also an MRI of lumbar spine.Since this symptoms are due to ageing factor and repetitive movements, an MRI of the spine is suggested to see the soft tissue pathology and any nerve entrapement.It can be a nerve impingement in the lumbar spine or may be a spasm of the hip flexor, depending upon the assessment and MRI we can figure out.Also, undergoing physical therapy will be helpful, as with therapeutic ultrasound therapy the muscle spasm can be reduced and later stages exercises .Since your age is 51, there is a possibility of pain and aches, as age progresses. So kindly take care of yourself and do not perform anything unusual which troubles you by pain.RegardsJay Indravadan Patel"
},
{
"id": 104423,
"tgt": "11 year old on steroid shots for allergies and respiratory problems. Knot and painful bruise at the injection site. Anything can be done?",
"src": "Patient: My 11 year old daughter received an 8/80 steroid shot because of extreme allergies and respiratory problems. That was 4 days ago. Minutes after the shot, she was crying from pain. The clinic iced it, and the pain seemed to subside. Since then, she will periodically (quite frequently) cry out in pain. There is a knot and a bruise . The knot isn t visible, only to the touch. She says it feels like a stabbing pain that comes and goes. Doctor: think this is a case of Wheat allergies I advise you to get blood serum tested for wheat,milk potato,rice,nuts,eggs to rule out other food allergies In these cases Elimination diets do wonders You can control only with medicines and steroids but no cure After reports go for elimination and desensitisation therapy for food allergies if you want"
},
{
"id": 43795,
"tgt": "Taking Metformin and Letrozole. Bleeding and cramps. Any positive news?",
"src": "Patient: I m on cycle day 14 and I m currently taking metformin , iron,b12 and on cycle days 3-7 5mg of letrozole to induce ovulation . I noticed around 5am I was spotting and didn t think much about it and figured maybe it s ovulation bleeding . But after having it last all day along with cramps that have now started tonight and gotten stronger, I m wondering if it s something else going on. Anyone else taking some of these meds and have any positive news? I was really hoping that this cycle the meds would work and we d finally be pregnant. I m feeling pretty down and worried that maybe yet again there will be no baby or that maybe somethings happened that will make it impossible to conceive. I know I m likely over worrying but I could really use some positive reassurance Doctor: Hello Thanks for your query. The mild spotting and cramps around day 14 definitely point to an ovulation bleed. You would be having follicular monitoring and ultrasound in this cycle, so you easily know what is the cause. ALso, you should see an Infertility specialist for your problem, as there are various causes of infertility. Your basic hormonal tests, semen testing of your partner, your tubal patency tests and ability to ovulate are some cardinally important issues. These should be thoroughly evaluated. THere is no reason in this era of ART ( assisted reproductive technology ) why anybody should be childless. Do not lose hope and keep consulting a specialist. All the best for this cycle."
},
{
"id": 71474,
"tgt": "Suggest cure for severe chest pain followed by breathing problems",
"src": "Patient: hello i am a 31 years old female . I have a breathing problem since i was young i headed to many doctors they said I don't have any major problem i only have fast heart beating. Now a days I am having a difficulty in breathing ,a severe pain in my chest as if it is closed and i am trying to open it through breathing and during this pain i feel like I am being swinging and that my heart is going up and down, without forgetting feeling of tiredness all the time and some tingling in my right arm(specifically in my hand and fingers)and a pain in the whole arm as i feel right now while I am writing this. And I don't know if this had to do with my problem but in three years ago I had a big round blue mark on my thigh and I didn't ask a doctor about it because I was convinced it's due to my psychological state and the stress i was having at that time. Doctor: Hello,Since, you have been to many doctors and they all said that everything is normal, no need to worry for major heart and lung diseases.Possibility of stress and anxiety related symptoms are more likely. So better to consult psychiatrist and get done counselling sessions.Try to identify stressor in your life and start working on its solution. You may need anxiolytic drugs (Propranolol and Flunarizine combination) too.Don't worry; you will be alright with all these. Avoid stress and tension, be relaxed and calm.Hope I have answered your query. Let me know if I can assist you further.Regards,Dr. Kaushal Bhavsar"
},
{
"id": 102977,
"tgt": "Chest infection, pneumonia, asthma. Taking methotrexate for psoriatic arthritis",
"src": "Patient: Hi i am a 43year old woman with psoratic arthritis i take metotrexate weekly for almost 3years it works for arthritis but i have had continuos chest infections, peumonia twice in six months and have also developed asthama, my lung function tests are reading at only 43% and im worried i will have long term lung damage.I feel it is time for my consultant to treat my arthritis with another drug which may not be as difficult for my lungs to deal with. Do you think my lung problems are connected with the metotrexate. Doctor: THE LOW DOSE METHOTREXTATE CANNOT DAMAGE BUT IT HELP TO REDUCE ASTHMA AND CHEST MAY PHYSICIANS HAVE STARTED GIVING INCLUDING ME LOW DOSE METHOTREXTATE IN TREATMENT OF ASTHMA ALSO I HAVE LOT OF DATA PUBLISHED IN JOURNALS LONG TERM LOW WEEKLY DOSE OF METHOTREXTATE IN ASTHMA IT MAY BE LIKE THIS YOUR ARTHRITIS MIGHT BE ALLERGIC AND WITH TREATMENT ARTHRITIS CONTROLLED BUT ASTHMA FLARED YOU NEED TO INVESTIGATE FOR CAUSE OF ALLERGIES BY CONSULTING ALLERGY DOCTOR AND FIND THE CAUSE AND TREAT ACCORDINGLY YOU MAY BE BENEFITED BY THIS WAY"
},
{
"id": 5203,
"tgt": "Possible pregnancy from exposure to spilled sperm on toilet seat?",
"src": "Patient: I know it's maybe a silly question, but I'm really scared. I ejaculated on the toilet seat than cleaned it up with toilet paper. When I left the bathroom my sister immediately went inside and used the toilet. And I'm worried that I may left some sperm on the toilet seat or on the paper too. Are there any chances that my sister got pregnant? Doctor: Hi, Thanks for writing to us. The chances of pregnancy are not there in your case because the sperm will die immediately when they came in contact with air. So you don't worry about it. Good luck. Take care."
},
{
"id": 137207,
"tgt": "Suggest treatment for recurring left plank pain",
"src": "Patient: I have left flank pain that started mid last year that continues. The flank pain comes and goes but is worst when I am lying down. I ve seen a Dr. who referred me to a urologist. The urologist had me do a cat scan, get blood and urininalysis work done and I ve had an ultrasound but there was no diagnosis. Just prior to flank pain occuring, in case it s relevant, I went to the emergency room thinking I was having a heart attack but there was nothing found. Recently, in the last week, I have a fullness in my upper left stomach area. There is no pain, only a sense of fullness. Doctor: Hi there.It could be urinary infection or kidney stone. Drink 2-3 litres of water daily. Drink 3 teaspoonfulls of Syrup Alkaline citrate thrice a day. Eat fresh fruits and green leafy vegetables. Avoid smoking, alcohol, spicy, oily or fatty food."
},
{
"id": 101040,
"tgt": "What can cause swelling in the lips?",
"src": "Patient: Hello doctor i am 22 years old.and from last 6-7 months i am having some swelling in my lower lips(outer and inner).it occurs sometimes only and goes away by itself.i went to a doctor he said its an allergy .i took an treatment for abt 4 months now of Allegra 60mg .while i was in medication it did not happen.as i stop medication it starts again. Doctor: Hi, thanks for using healthcare magicIt means that you would have been re exposed to the allergen (the source of the allergic reaction).You may want to consider allergy testing to determine what you causes your allergic reaction.Allergy testing can be by blood tests or skin prick allergy testing.Once you can determine what it causing your reaction, your exposure can be reduced.I hope this helps"
},
{
"id": 41689,
"tgt": "Suggest treatment to get pregnant",
"src": "Patient: hi, am female 23yrs, i am getting problem with my ovulation, and am taking clomid, today is the 13days of my cycle and i went to the doctor, n after scanning he said that that my ovules are 7mm,and its too tiny, give me a solution, i want to be pregnant Doctor: Hi welcome to healthcaremagic.I have gone through your question.Clomid contains clomiphene which stimulate ovaries, develop follicles. If i am your treating doctor then i will start Clomdi with 50 mg and dose can be increased upto 100 mg from 3rd day for 5 daysContinue this with 3 cycles then switch to other medicines or injection can be started.Consult gynecologist for further advise.Hope i answered your question.Would be happy to help you further.Take care."
},
{
"id": 153935,
"tgt": "Suggest herbal remedy to treat level 2 breast cancer",
"src": "Patient: I have just been diagnosed with level 2 breast cancer not in lymph nodes. Dr putting me on letrozole to shrink tumor. How can this shrink my tumor and help ladies get pregnant? I am 60 years old thank you is there any herbal meds that can do the same that don't have bad side effects? Doctor: Hi i did review your concern. Letrozole is a estrogen production inhibitor. In this way it decreases estrogen in body and decreases tumor growth of those tumors which are tumor dependent like breast cancer. It also helps in pregnancy as it indirectly increases LH and FSH secretion and helps in ovulation in initial stages.I would advice you to do the same. Ayurvedic medications have no scientific basis however they can be tried if you have any person who has a successfull experience with it. I hope this helpswish you good luck."
},
{
"id": 92703,
"tgt": "Suffering from painful abdomen, tiredness, dizziness, anemia, blood loss, vomiting, nausea. What is the reason?",
"src": "Patient: Hello, for the past few weeks I have been suffering from multiple problems. I have been experiencing extreme tiredness, pains the the side of my abdomen and very bad dizzy spells. I had a baby 11months ago, I was anaemic after due to blood loss and my periods have never been the same since, period pains are a hell lot worse. 2 months ago I had a very weird period, lots of spotting and light period, this month I was passing out and vomiting due to the pain but still a light period. Ever since I keep experiencing these symptoms listed above including nausea And feeling faint. I'm due to see my gp on Monday but just wondering if you have any ideas? Doctor: it look to be food protein allergymay be wheat a you have tirenessget allergy specialist advisw who can do blood serum tests for specific antibodies for milk wheat potatowithdraw food and you will be fine in few wk"
},
{
"id": 71578,
"tgt": "Suggest treatment for severe chest pain",
"src": "Patient: I m having inner vein pain on each arm, it s been reoccurring pain on and off for years. Four months ago I had severe chest pain, middle left, and since middle right, stabbing pain that is difficult not to shout out in agony- and I definitely have a high pain tolerance. Went to the ER the day AFTER I was up all night in pain, the freq upped to a break every few seconds and grabbing my chest and crying all night- last week same exact thing but in my right leg, after 7 hours of computer work, both hosp visits did their checks- no dvt and nothing in my lungs. I am overweight. I can t think of anything I could have- embarassing history- grandparents were cousins and married each other..genetic defect? mother has acute anemia and something else i can t pronounce that makes her reject blood transfusions and gr-grandfather had hemophelia. Just found out my med history when i found my biological family last yr. plz help! Doctor: Hello,As you explain the history i would recommend you to control the stomach too for any reflux.Hope I have answered your query. Let me know if I can assist you further.Regards,Dr. Jnikolla"
},
{
"id": 103763,
"tgt": "Skin rashes due to flea infestation. Itching under the skin. Allergy?",
"src": "Patient: I have no animal but there is a serious infestation of fleas where I use to work in home health aid I went to the Doctor I get creams 2different tables it help but I continuebeing bitten .I am not working there anymore.It s like there is things under my skin that itches badly .i m allergy my skin look like I was beaten with a rope Doctor: it is allergy and allergy cn take 3 wk to subside if exposed one to an allergen it can be scabiesflas an medicine took for other problemoil soap shampoo dye hena or toxic application used on bodytake anti allergic ebstine 10 mg bdor 3-4 wknon toxic oils shampoo creams and skin applicationsaply crotorex hc skin oinment in aaffectd skin bdapply emscab lotion in whole body for 5 nights boil clothes and bed sheets etc bfore reusethis ill cover every type of cause written above rake 3 wk"
},
{
"id": 87398,
"tgt": "What causes recurring abdominal pain?",
"src": "Patient: MY adominal pain is ocuring after every 8 to9 months. i tested my thyphoid test. report is below S TYPHI O 1:80+ S TYPHI H 1:80+ S P TYPHI AH 1:80+ S P TYPHI BH 1:80- DOCTER SAID it is past thyphoid. no need to take medicine my question is is it harmful for in future Doctor: Hi.Thanks for your query. It is good that you did Widal test for typhoid and this is equivocal. It looks that you are either a traveler or eat out of the home. You should get investigations for other causes also as the test is equivocal meaning can be positive or negative. I would advise you the following:Blood culture and sensitivity before you are started on antibiotics,Other tests of blood, urine and stool.Ultrasonography and CT scan of the abdomen. Enteroclysis if indicated. A good clinical examination of the abdomen by a General Surgeon and further tests , and treatment according to the findings.."
},
{
"id": 158239,
"tgt": "I remember everything about the colonoscopy done, Doctor discussions with me. Is that possible?",
"src": "Patient: How am I able to remeber my Colonoscopy I just had? Everyone I know who has had one says they wake up and can not remember anything. I remeber everything. Atleast I think I do. Nothing bad about it, just wondering how I can remember. I was talking to the Doctor while he was doing the procedure, etc. I was watching the procedure on a monitor, we talked about a few things that he saw. The nurse pushing on my stomach while I lay on my side. If I don't remember it all, I sure remember most of it, I talked about it and explained it all to my husband on the way home. I found it interesting to watch, is that why I stayed awake? Doctor: Hello, Thanks for the query to H.C.M. Forum. It is very nice to note that you yourself saw the colonoscopy procedure , that a nice thing. But , what is your query?If any thing ask?Good luck. Dr. HET"
},
{
"id": 112047,
"tgt": "What is the treatment for lower back pain leading to difficulty in leg movement after an injury?",
"src": "Patient: My 55 year old husband fell off of a ladder from a height of approximately 12 feet and landed on his back (not directly) but slightly to the left. He landed on the asphalt driveway. He broke two ribs and punctured a lung as well as fracturing his L5 vertebrae. He spent 5 days in a trauma center. His ribs seem to be healing well but he has low back pain that is causing him a lot of pain. He has problems lifting his left leg and has started to \"swing\" his leg forward in order to walk. The limp is extremely evident. The accident happened exactly 4 weeks ago. What is our first step in finding the appropriate help for him? Doctor: Hi, thanks for writing to HCM.As your husband had sustained a fall with fracture of L5 vertebra, I suspect that there could be an associated disc protrusion causing pressure over the nerve roots that is causing some weakness in the muscles leading to the limp and altered gait.I suggest an MRI scan of the Lumbar spine which clearly dipicts the severity of the disc protrusion,spinal cord compression and any other soft tissue pathology in the spine. Nerve conduction studies may also be needed to establish any neurological injury in the leg.Management depends on the condition diagnosed.Hope this information is helpful. Good day"
},
{
"id": 21538,
"tgt": "What causes chest pains, shortness of breathe, fatigue and light headiness?",
"src": "Patient: My resting heart rate has been as low as 28 and usually in the 30s. I havent been very active in almost 20 months and been to the cardiologist before. I have bicuspid aortic but they said it was minor and that I'm fine. I suffer from chest pains, shortness of breathe, fatigue, light headiness, and the occasional fainting. Any ideas what might be wrong and why all the test have came back negative? Doctor: A resting heart rate of 28 is quite low. Your symptoms may be due to sick sinus syndrome in which the physiological pacemaker ( sino atrial node) is diseased. I would suggest you get a 24 hour holter monitoring done. The cause for such a low heart rate needs to be evaluated. You may also need a permanent pacemaker in the future."
},
{
"id": 198075,
"tgt": "What causes redness and itching on the glans?",
"src": "Patient: Hello Sir, I m a 19 yr old male. I have a question. I wish you could help me out please. From the last 1.5 months I had itching and redness on the glans and in the below end of glans. I m uncircumcised . And I masturbate almost twice everyday. My doctor gave me prulifoxacin 600mg for 5 days after which my redness and itching went away. I reckon it was Urinary Tract Infection. Later when the meds finished i masturbated, although the itch and redness were gone, the itch is sligthtly present like 5% of the whole. And Please note it only happens after masturbation rest of the time , its fine and it all disappears while the itch after masturbation is only for like 15 mins with a very very slight sensation of prick inside the penis and on the left side of the glans and some engorged red veins just for sometimes after masturbation and then its gone. I ve never had sexual relations yet but yes i admit i have alot alot alot of sexual thoughts across the day and that i stay erect numerous times in a day. The doc was unwilling to provide me any meds cuz i ve met two docs in the past and they said that i ve got mental stress and need psychiatrist. It s true a bit that i ve never been to school since 7th grade which i left challenging the principal and since then i ve completed and excelled in everything from school students and infact have defeated many teachers as well in knowledge despite of learning on my own at home. I ve immense and very hyper (above normal) enthusiasm in biology and especially in Microbiology. It may have been my half knowledge thats causing stress as i ve been stayin very afraid towards diseases but its certainly not OCD and due to recent outbreak of ebola even though there hasnt been a single case of it In India but im staying too precautious . I excercise everyday with dumbells and pushups . I listen music and don t like eating sweet much especially in the morning or as meal. Hence i cut tea from my daily schedule and only drink milk without sugar . I only like spicy hot, bitter and sour but sweet at its least. Recently i ve had abdominal pain below the left side of the ribcage (not the rear side) for six months which had gone for months but came back since yesterday and indigestion for 1.5 months which both the doctor say that s because of my overthinking. And yes last month i did a CBC in which everything was normal except. haemoglobin which was 0.5% less than normal but the docs said its not even a 1% matter of concern and infact i can donate blood with those levels and also my ultrasound i.e sonography (abdomen) reports were completely normal. I m just scared although i know its nothing serious since i never have had sex. I just wish to know whats the problem with my glans (balans) and why the itch . Does UTI take time to heal after 10 days of finishing medicines? Or is it due to excessive masturbation (although i paused for 2 days but of no avail) and its weird but i ve been masturbating since the age of 5 (although no ejaculations till puberty) but it hasnt every happened before or is it due to overthinking as the docs said? I thank u very very much sir Doctor: HelloThanks for query Based on the facts that you have posted you seem to be intelligent guy but unfortunately wasting your knowledge and time which are of least importance in day to day life .The itching and redness over glans that you get after masturbation is mostly due to infection of glans penis due to contamination (Ballanitis).You need to take broad spectrum antibiotic like Cefixime along with anti inflammatory drug like Diclofenac twice daily .along with topical antibiotic ointment like Neosporin twice daily.Most of your problems are due to anxiety .It is better you consult qualified Psychiatrist for counselling .Reduce frequency of masturbation by keeping yourself busy in activities like sports ,reading ,social work etc so that you will not get free time to masturbate.Dr.Patil."
},
{
"id": 198367,
"tgt": "Suggest remedy to control masturbation",
"src": "Patient: Hii doctor.., am a final year B.Tech student. i started masturbation since at the age of 13 and have been continuing till today. i think that i got addicted to masturbation and am unable to control it. So i kindly request to suggest me some steps for controlling the masturbation. Doctor: DearWe understand your concernsI went through your details. Please understand masturbation is normal, natural and do not affect your health directly. You are safe to masturbate in moderation, that translates as thrice or four times a week. Habitual masturbation can be controlled with physical activity, exercise, games, late night activity etc. Engage yourself in the above activities so that you are almost totally exhausted. Then you will not have any energy left for masturbation. If you require more of my help in this aspect, please use this URL. http://goo.gl/aYW2pR. Make sure that you include every minute details possible. Hope this answers your query. Available for further clarifications.Good luck. Take care."
},
{
"id": 27727,
"tgt": "Suggest treatment for atrial fibrillation",
"src": "Patient: I am a 76-year old female with atrial fibrillation, having logged blood pressure for 3 months. BP is good, heart rate is consistently staying at 100+ and irregular heart symbol, according to digital BP machine. This I have discussed with triage nurse and am to see heart doctor next week. Am in good health, have no stressors in my life except am a caregiver for my husband. No problems there either. After logging BP s for over 3 months, the doctor changed me from Propranolol 240 mg per day to Metoprolol, just last week. It seems my heart rate is still increasing with no apparent reason off and on the last few days, BP still fine. Still having the irregular heart symbol off and on. No chest pain, no shortness of breath or discomfort. Just the quivery feeling when the heart rate is up. I am resting more. No exercising, just the trips to store at this time. Doctor: Hello as you describe you fall in the triage of persistent /permanent AF. With all studies it has been proved that controlled rate gives equivalent as to bring rhythm (irregularly) under control, so you will have irregular heart rate mostly forever and you should not be worried for it. However you need to bring heart rate under control so they will increase the dose of metoprolol or add another drug. Also I recommend you to be on anticoagulants (a kind of blood thinner) to prevent clot formation and avoid future risk of stroke"
},
{
"id": 150999,
"tgt": "Taking epilex chrono 300 for epilepsy. Any medicine to stop it? Effect of medicine on brain?",
"src": "Patient: Hi, myself surekha. i m taking epilex chrono 300. i have left temporal lobe epilepcy. now i m 27 years old. i have first seizure in my 10th class. and in this duration i have only 3 times epileptic seizure . i m taking daily 1 tab daily. so my question is: can we stop it by any medicine? and also i wanted to know, how the epilex chrono 300 medicine work for epilepcy. ? what are the exact changes made by this medicine in brain? Doctor: Hi, First do not stop treatment on your own. To stop treatment it is necessary to see: -Risk benefit of continuing treatment -Seizure free period from last seizure -EEG findings Epilex chrono work by increasing inhibitory neurotransmitters like GABA and decreasing excitatory neurotransmitters like GLUTAMATE role in brain. I hope this information has been both informative and helpful for you. Wish you Good Health. Regards, Dr. Ashish Mittal www.99doctor.com"
},
{
"id": 63509,
"tgt": "How to cure painful lump behind the knee?",
"src": "Patient: Hello. Am experiencing a large bump behind my right knee joint. Over the last couple of months it has grown to be a real problem with lift off on my step up. Doesn t hurt to step down. Only feels a little wobbly on the way back down. Was injured about 20 years ago while riding a Harley in a Highway accident by drunk driver of a truck. My question is could the 2 be related after so long a time and also should I see a Doc about this? Don t have insurance , sure don t want to waste money or someones time if all I have to do is apply ice or heat to relieve symptoms. Whomever you are; I thank for your time and for possibly being able to give some answers as to why this is happening. Thanks, Amy Doctor: Hi,Dear.Thanks for the query to HCM virtual clinic.I studied your query in all its details and understood your health concerns.Cause and Remedy-In my opinion the cause of the painful lump behind knee is related to 20 yrs truck accident,which lead to arthritis of knee joint.The lump seems to be MB (Morant Baker's)Cyst on back of the knee joint.USG would fix its diagnosis.Xray knee would confirm knee arthritis.This lump needs consultation from ER Ortho-Surgeon.who would treat by medicines first or by surgical removal of the MB cyst lump.Hope this would reply your query to your satisfaction.Write Excellent review and hit thanks if this reply is liked by you and is helpful to you.Welcome for any further query in this regard to HCM.Have a Good Day.Dr.Savaskar M.N."
},
{
"id": 53112,
"tgt": "Suggest treatment for HCV induced cirrhosis",
"src": "Patient: I am a 59yr old woman with HCV caused cirrhosis. About 50% of the time my doctor takes my blood pressure it is normal (120/70 or 80) and the other times it's high (160/105) or more. Had heart tested a couple of years ago and it was fine. Pre-cirrhosis it was always low. Ideas? Doctor: Hello , thank you for your question . Liver cirrhosis needs a complex treatment . If you were my patient i would suggest you to get an appointment with gastroenterologist . You need to get regular ultrasonographic and biochemistry exams to check the status of the desease. I have to warn you that liver cirrhosis is a main cause for liver cancer . You should switch to a liver friendly diet - no fried foods , no fatty things , lemons etc. etc. . Its is good for you to take regularly hepatoprotective medicines . You have to be aware of one of the complication of cirrosis - portal hypertension . You may develop varices of the esophag which can bleed badly . Your condition needs dedicated lifelong treatment . Hope i helped you"
},
{
"id": 54252,
"tgt": "What diet should be taken for inflamed liver?",
"src": "Patient: Hi, may I answer your health queries right now ? Please type your query here...my mother is a 50 year old single person who doctors says she has a very in flamed liver. She has not been taking care of herself since my parents divorced. Her personal dr told her it was very inflamed, can it be reversed or is it permanent, can she change her diet, I know she drinks but I don't know how much, what should she do at this point to reverse or make it better Doctor: Hi thanks for contacting HCM...According to history your mother mostly having alcohol induce hepatitis....First of all stop alcohol drinking.....Low fat diet advisable as inflamned liver can't digest fat easily.Refined food avoided.Trans fat like junk food , ghee , butter ,,meat less .....Green leafy veg more.Take some good food for liver like.....-mustard green -spinach -papaiya -carrot -dandelion -grape fruit juice -Indian gooseberry etc.....Rest done....If needed viral marker study done for viral hepatitis....With keeping this in mind you can consult her to gastroenterologist.Take care."
},
{
"id": 35703,
"tgt": "Why am I so weak and have low immune power?",
"src": "Patient: i am a girl of 17yrs age.i am very thin.before i was in a normal weight but after malaria fever i became very thin.when i was studying 9thstd i got malaria.now my weight is 40.and i am suffering lot from cough.even i take little bit of ice i get cough easily.is there any problem?reply me Doctor: Hi, thanks for sharing your health concerns with HCM! If I were your treating Doctor for this case of recurrent infections, I would say you that malaria is related to infection/mosquitoes and there is very less to do with immunity as single infection by malarial parasites doesn't give life-long immunity; chest infections/common colds are also infective but humoral immunity plays a great role here; might be you are deficient in some proteins/vitamins etc therefore you need to go for routine blood tests and immunoglobulin measurement for confirmation and to relieve your concerns!Hope this answers your question. If you have additional questions or follow up questions then please do not hesitate in writing to us. I will be happy to answer your questions. Wishing you good health."
},
{
"id": 112666,
"tgt": "Back pain, doing physiotherapy for lumbar strain, tingling down right leg, CT shows mild degenerative loss. Suggestions ?",
"src": "Patient: I got hurt at work and i'm having back pain ..I went to the work comp doctor and she said it was a lumbar strain so she set me up with pt. ..I have been doing pt. for 4 weeks and seeing the doctor every week.. when I walk I get tingling down my right leg ..I have had a ct scan done at the local hospital because workman comp wouldn't do one .. The ct said Mild degenerative loss of normal disc height at t11-12,t12-l1and l5-s1.mild disc bulges at multiple levels ,with endplate osteophytesat l5-s1 and mild to moderate bony foraminal narrowing on the right and mils foraminal narrowing on the left at l5-s1.mild multilevel lower lumbar spinal canal narrowing secondary to facet arthropathy and disc bulges ..no acute paraspinal soft tissue abnormalitynonobtructing left nephrolithiasis.. what do nu think I should do all I wont to do is stop hurting and find out what's wrong with me Doctor: Hi,Thanks for posting your query.Tingling and numbness in lower limb seems to be due to pinched nerve cause by bulge disc and osteophyte formation.Physiotherapy in form of exercises and lumbar traction is advisable to manage the condition.Avoid forward bending and heavy weight lifting. If pain persists, consult spine specialist for further management.Hope this helps you.Regards,Dr Saurabh gupta"
},
{
"id": 11228,
"tgt": "What is the treatment for alopecia areata?",
"src": "Patient: I'm suffering from the disease alopeciaAnd all my body hair fell from about 5 yearsCorticosteroids have been used for several sessions and suspended himI am 25 years oldAnd 160 linearWeight 52I hope to visit any specialist in any place in India for treatmentI am from Saudi Arabia Doctor: hi, welcome to HCM,you are suffering from alopecia universalis.as a treating dermatologist i used steroid mini pulse. potent steroid (Halobetasol lotion) for topical application. minoxidil 5% once daily for local application. tab biotin 10mg + PABA + ca pantothenate once daily.most importantly i rely on intra lesional injection of triamcenolone acetonide in sessions.this gives very good result.thanks."
},
{
"id": 9340,
"tgt": "What causes dry circular rash on thigh?",
"src": "Patient: I have a dry circular rash on my outer left thigh. It's 1.5 inches in diameter. Resembles a bullseye or ringworm. Does not itch. It had been there for 3-4 months. Three weeks ago patches of dry skin started appearing on the left outer thigh and now on the rt outer thigh. Now have about 10 patches each thigh. Sometimes get sharp stabbing or pin stick pain all over my legs or pain in my lower legs that feel like bad shin splints. Also notice a reddish purple rash covered by a silky smooth silvery white thin layer of skin on my left shoulder blade. Two weeks later another one appeared on my right shoulder. None of these rashes itch. Prior to these rashes I have developed a sensitivity to some bath products which caused yeast infections. I now bath in apple cider vinegar and honey. What is the cause of all the skin infections. Have noticed two spots on my face now. Hiv test taken 3 weeks ago ands 6 months ago are negative. Doctor: Hello. Thanks for writing to us at healthcaremagicThese patches could either be Discoid dermatitis Or Fungal Infection/Tinea.The distribution of the patches however goes more in favor of Discoid dermatitis rather than fungal infection.Outer thighs are a relatively common site for these patches because this area is dry; Tinea is common in the inner thigh area.A KOH examination would prove to be conclusive in that regard. Topical OTC antifungals e.g clotrimazole 1% cream may be tried if KOH is positive for fungal elements.On the other hand if it is negative for fungal elements treatment would eb along lines of discoid dermatitis.Since underlying dryness is the predisposing factor in Discoid dermatitis, therefore I would suggest an OTC intensive moisturizer over the whole affected area.Specifically for the patches, I would suggest a topical potent steroid cream e.g clobetasol propionate cream, twice daily for 2-4 weeks.Topical steroids are prescription products. I would suggest that you visit a dermatologist in your region for a confirmatory diagnosis as well as appropriate prescription.Regards"
},
{
"id": 35866,
"tgt": "Suggest remedy for budding yeast cells in stool examination",
"src": "Patient: My stool examination during routine medical check up indicated ' budding yeast cells present'. The other observations including puss cell count indicated is within the range specified. What is this phenomenon and how is it treated - My doctor suggested Tinidazole and Orfloxacin Doctor: Hi,Report suggests that you are having mixed Gastro-intestinal infection, bacterial and protozoal.Medicine ofloxacillin and tinidazole is very rightly given to cure this infection.Take light diet.Ok and take care."
},
{
"id": 97725,
"tgt": "Is there an alternative to execdrin and caffeine that does not have aspirin?",
"src": "Patient: My husband is having a procedure done and has to stop taking anything with aspirin in it five days before. HE TAKES AT LEAST 6 EXECDRIN AND NEEDSD THE CAFFEINE. IS THERE SOMEHING HE CAN TAKE THAT DOESN.T HAVE ASPIRIN? thank you Sorry I thought this was a free service,,,please disregard Doctor: Hi, He can take codeine/caffeine tablet only combined with acetaminophen (Co-codamol, for example). You should look at the active ingredients of the otc analgesic and choose those aspirin-free. All the best!Dr.Alba"
},
{
"id": 145008,
"tgt": "What causes unexplained rashes after having pituitary surgery?",
"src": "Patient: I have had unexplained rashes after having pituatary surgery for five years. Had shingles in July this year. Just found out I have hyperthyroid. Getting a 24 hour scan and uptake next week. Could the shingles have caused hyperthyroid? Also could the rashes be related to the rashes? Doctor: Hi I did review your concern. Rash can be or cannot be related to the pituitary surgery.One would like to see the type of rash before commenting on it.Shingles usually does not cause hyperthyroidismHyperthyroidism is caused either by toxic nodular goiter or graves disease. you would require antithyroid medication under doctors guidance like propylthiouracil or methimazole to curb the thyroid hormone synthesis.I hope this helpsWish you all the bestThank you for using healthcare magic"
},
{
"id": 149909,
"tgt": "Taken Syndopa for YPD. Taking Pramipexole. Tremors on right side. Help",
"src": "Patient: Dear Doctor,I am in urgent need of help. Please give me direction. I am a 37 yrs old Software Professional from India. Was diagnosed YPD 7 yrs back.Was given Syndopa high dose for trial but it was not effective. Then consulted with AIIMS Neurology HoD India, She started Pramipexole, currently at high dose of Pramipexole SR. But I am 120% sure this is not PD. The current situation is, tremors on the right side, not a big issue. The challenge is that I cannot walk even two steps. It is so rigid. Though I can do Power yoga, cycling or run a bit too.Please help me.A few observation-1.\u00a0\u00a0\u00a0\u00a0\u00a0Got B12 checked , it was 185, D3 was 10. Have started injections from 2 weeks. Though Dr says my weight etc is little more so its highly unlikely.2.\u00a0\u00a0\u00a0\u00a0\u00a0Another Neurologist says, it is due to troubled childhood, anxiety and depression. I think he is right but I do not want his sleeping pills.3.\u00a0\u00a0\u00a0\u00a0\u00a0Have troubled stomach and Migraine since 20 yrs.Best RegardsNaresh Doctor: Hello Naresh,Thank you for your query through Healthcare magic.I understand the seriousness of your sickness at this young age. I hope that your disease is diagnosed as a case of young- onset Parkinson's disease ( YPD). I am a Homoeopath and so I will be answering your query in the Homoeopathic perspective.In Homoeopathy, the name of your disease is less important. What is more important is your condition of health. The diagnosis gives us the idea which system , organ or tissue is affected. In addition to that a Homoeopath needs the diagnosis of the patient. That we will give the idea how you are reacting to the life situations. For that a detailed case taking is required. Only a Homoeopath will do it in the perfect manner.Therefore my advice to you is to approach an expert Homoeopath at the earliest. He will find for you your genetic constitutional remedy and will cure YOU. Once YOU are cured, all your neurological complaints will be disappearing. You will become free from the drugs you are taking at present. Since you were on those drugs for a prolonged period, it may take some time for you to taper the dosage of them and finally you will be free from all the medications.My best wishes for a rapid and gentle cure through Homoeopathy.Dr. C. J. VargheseHomoeopathdrcjvarghese@gmail.com"
},
{
"id": 196465,
"tgt": "What causes bleeding penis?",
"src": "Patient: Doctor i am just got married and my age 26 male, i did sever time sex with my wife only fronte side but when i tried first time on her back positon i got bleeding on my penis skin and after several week its was goog and now i tried masturbition suddenly agen bleeding from pensi skin, i saw top of pensi skin just cut , would please what was the reason the bleeding from my penis skin Doctor: HiGREETINGS Most common cause could be a tear either in the foreskin or in the frenulum.If it is a frenulum tear you may have to consult a surgeon or urologist and get it repaired.If not any antibiotic ointments will help you.Hope my answer helps you. Regards"
},
{
"id": 57672,
"tgt": "Cause for pain in back and around gall bladder?",
"src": "Patient: I have been experiencing pain around gallbladder. Also pain in lower right back. I feel full and can not eat. And the past 4 days my stomach has really swollen up mainly on right side or bloated. I have had all the gall bladder test done and nothing seen. On the 17th I will have a scope to look at the inside. I am always tired and it is getting worse each day. What could this be? Doctor: HelloThanks for writing to HCMYou need proper clinical examination and investigations.Investigations include routine hemogram,RBS,LFT,RFT,Lipid profile,Serum calcium and vitamin D level,ultrasound of abdomen,upper GI endoscopy. Further investigations can be done if needed. Ultrasound of abdomen can exclude many conditions.Your problem may be due to acidity.Ta present you can take tablet Pantoprazole 40 mg once daily half hour before food for six weeks.Hope i have answered your query.Take CareDr.Indu Bhushan"
},
{
"id": 124903,
"tgt": "Is there any option of putting artificial elbows?",
"src": "Patient: Dear doctor, My mother having joint pain since 1995, she is 65 years old now. From last two years her joints start dissolving. Now she didn t have both elbow joints. How we can avoid further damage plus is there option that we can put artificial elbows? Regards & Thanks, M. Amir Doctor: Hi, It seems elbow joints are damaged due to bone resorption. In such situations, instability is the main issues at elbows and hyper movements will affect the functions of upper limbs. There is orthosis with elbows joints locks are available which will be controlling excessive movement at elbows and provide stability. You are advised consulting orthopaedic expert and Orthotist to get a solution. Hope I have answered your query. Let me know if I can assist you further. Regards, Dr. DEORISHI TRIPATHI, Orthopaedic Surgeon"
},
{
"id": 213486,
"tgt": "Gender identity crisis, attracted to men, in love with a girl. How can I get over this?",
"src": "Patient: Good day Doc, Please help me, I really need someone to talk to if there is any solution to the problem that I am having right now. I am suffering from a gender identity crisis and I m really confused with myself. I m a male and in a relationship with my girlfriend for almost 3 years. I love my girlfriend but it seems that I am attracted to men. I don t know what s wrong with me. I really need help. What are the things that I need to do. I don t have any experience with male to male intimate relationship coz I am controlling myself and I don t want to cheat my girlfriend coz I love her very much. I just want to become straight guy doc. Thank you and I hope you ll give me useful answers and recommendations pertaining to my problem. Thank you so much and more power. Doctor: Hi mysteryman123185. Thanks for your questions. You are suffering from gender identity disorder where a male/female thinks his/her mind as opposite sex and they attract with person of same sex.But actually why it happens no body knows and we dont have any preventive measure for that neither we have any drug to control them. The best therapy is psychoeducation or psychological counselling and that help a lot to cope up with the problem so please visit a psychiatrist and tell your problem in detail,he will surely help you. Best wishes."
},
{
"id": 175928,
"tgt": "What causes formula feed and drinking water refusal?",
"src": "Patient: Hello doctor.. My daughter completed 10 months this 6th.. She used to drink (70 to 90 ml per feed) her formula 4 times a day. I give her formula when she naps in day/sleeps at night. Suddenly she stopped drinking all of a sudden totally. She loved water.. Even refuses that now. Not any new teething... Herurine amount is lesser than normal... Doctor: Hi...unless she is dehydrated you need not worry. Activity is more important than anything else. I suggest you observe for another 2 days and if the urine output is gradually coming down or if she is dull and lethargic, I suggest you take her to the pediatrician.regards - Dr. Sumanth"
},
{
"id": 178674,
"tgt": "What causes bump between shoulder and back in a 4 year old?",
"src": "Patient: My four year old has a small bump between her shoulder and back of her neck. It has been there for maybe a year? A doctor had suggested it could be a cyst. You can hardly notice it, there is a faint blue bruise and it is painful to her when I graze it while doing her hair or helping her dress. Wondering what it is and if I should be concerned? Thank you. Doctor: Hi, may be when your doctor saw it could have been like a cyst. The very fact you are saying that it is painful and mild bruising is there, I would recommend you to get back to your doctor and show . May be its an abcess also, you never know. Only when one sees , it can be identified.Tc"
},
{
"id": 176732,
"tgt": "Is a swollen cheek a sign of conjunctivitis?",
"src": "Patient: can a swollen cheek be a sign of conjuntivitis? My son has a small sty on his left eye that appeared yesterday. I have been cleaning it with a warm wet wash cloth and following with a warm compress for about 5 minutes. Today his right cheek is slightly swollen from lower eye to his jaw. Are the two symptoms of conjuntivitis or could it be something else? Doctor: Hello. I just read through your question.The swelling of the shift would not be conjunctivitis. Conjunctivitis would be a redness to the white of the eye with some kind of discharge from the eye. It is possible that the swelling and redness of the cheek could be a skin infection. I recommend consulting with your doctor so the correct diagnosis can be made and appropriate treatment plan implemented."
},
{
"id": 171338,
"tgt": "What causes white discharge with foamy urine in children?",
"src": "Patient: Hi My 2.5 year old son has a white smelly dried up stain in his underpants look like maybe it was dried discharge and when he went to the toilet his urine was foamy when he first started to urinate. he has also been very grumpy lately and not at all his self. Do you have any idea what this may be and do i need to take him to the doctors? Doctor: Hi,Welcome to Hcm,When foamy urine is seen in a child, possibility of urinary tract infection should be thought of or rather ruled out. So, if I were your pediatrician, I would get a urine routine done in your son and then decide regarding further management. However, not having fever is an unlikely feature of urinary infection. Hope the suggestion helped. Happy to help with further queries."
},
{
"id": 40218,
"tgt": "Will the fluid oozing out from the wound of my scalp go away?",
"src": "Patient: I had some stitches in my scalp about 8 days ago. Will be removing them in about 3 more days but there is still a pinkish oozing coming from the wound. I think is is a dime sized area that wasn t stitched but the skin hasn t grown back yet. Will this eventually go away. Chris Doctor: Hello.Welcome to HCM,As your wound was stitched following the injury and there was a discharge from the wound.The discharge is may be due infection of the wound, it should be kept clean and dry for rapid healing of the wound.As you a mentioned regarding some part is not stitched but it will heal, no problem. You require physical examination to see for the granulation tissue which is a good sign of wound healing. Keep the wound dry and clean. Thank you."
},
{
"id": 98360,
"tgt": "How can severe allergic reaction to food be treated?",
"src": "Patient: my son has food allergies. he had a reaction about 3 hours ago to something he ate. he's 14 years old and i gave him 3 tsps of benadryl. He had swollen lips, a bumpy tongue and itchy throat. most of it subsided quickly. he still has what he said feels like a lump in his throat and his body he said feels tingly. he had a slight rash on his arms and belly but it went away. he just took another two teaspoons of benadryl, has not had any trouble breathing. Do i need to take him to the ER though? Doctor: Hello and Welcome to \u2018Ask A Doctor\u2019 service. I have reviewed your query and here is my advice. Considering the symptoms, you have to take him to the emergency room, he needs medical observation.As you say he has severe allergic reactions, so he might need prednisolone injections.Hope I have answered your query. Let me know if I can assist you further."
},
{
"id": 39533,
"tgt": "How many anti rabies shots to be taken after a dog bite?",
"src": "Patient: Doctor i got bit from a street dog while riding a bike ... bit was like a scratch and didnt injure too much pain also was less... though i put TT injection and anti rabies injection on same day... Now its 3rd day on pain also no fever i am healthy should i want to put anti rabies injection 4 times ...? Doctor: Hello,Welcome to HCMYes,although its just a scratch you need to take full course of antirabies vaccine,since its a street dog.5 doses of vaccine t be given at 0,3,7,14 and 28 days.'0' means the day you had bite.It is very much essential that you take the complete course of antibiotics to prevent the development of rabies even though you are healthy.Also you should watch the dog for 10 days.Watch for any change in its behaviour.Hope you are happy with the answer.Thank you"
},
{
"id": 55046,
"tgt": "Suggest diet plan when having hepatic encephalopathy and cirrhosis",
"src": "Patient: hi dr. theres a patient having 55years of age suffering from liver disorderhe is suffering from hepatic encephlopathy, acites, cirrhosis from last 5 years.hes taking proper medication for this. i want you to tell me about the diet chart he should strictly follow for proper prevention. Doctor: Hi thanks for asking question.Noted you have cirrhosis since long time with ascites.So strict diet control needed.In breakfast fat free pasta or green leafy salad or suitable fat free things eaten.In lunch and dinner cook with very very less oil.green leafy veg sabji will be good for cook.use oil less roti ...Meanwhile whenever hungry fruits eaten.Refined sugar not taken.Cheese, butter , pestry , meat not taken.If possible mild exercise can done.Strictly avoid alcohol and smoking.Early in morning take papaiya seed Mix with lemon juice one tablespoon daily...Garlic is good taken empty stomach as one or two after peeling with water.As you have ascites take less salty food.Diuretic drug can be taken for ascites.For portal hypertension beta blocker can given.Take care.Dr.Parth Goswami"
},
{
"id": 159250,
"tgt": "Lung cancer, have persistent fever. What could be the safe dosage of DOLO650 for cancer patient?",
"src": "Patient: Hi Doctor my mother is lung cancer patient and she is currently running a persistent fever opf 101 F for the last 2 days. We have already given her 3 tablets of Dolo 650 ( Paracetamol 650) during the day, the last being given some 4 hours ago. The question is what is the safe dosage of DOLO 650 for Cancer Patients running fever Regards Neeraj Doctor: Hi, paracetamol is very safe drug. it has hepatic (liver) toxicity at very high dose which is subjective. lung cancer if not spreads to liver her liver function seems to be normal. then you may use the same dose for even 4 times a day. but cause of fever to be detected and to be treated accordingly. paradetamol is not definite treatment. you should go for a complete blood count. consult oncologist."
},
{
"id": 43751,
"tgt": "Done follicle study. Have PCOD. Is IUI treatment okay?",
"src": "Patient: Hi doc. Stephen i m maghi i m planning for IUI this month and continues follicle study is going on as today was my 11 th day and the growth is 16 mm and only one follicle is there on the rt ovary and to the left ovary 2 foll with 10mm growth so can u pls tell me is it ok for iui i m under treatment from last 6 yrs n i m having pcod prob ... so i m really tensed whetheer it would work out or not can u pls suggest me .. And pls tell me what precaution should be takebn after IUI as doc recommend for normal routine way but i dont want to take any risk at this time .. so pls can u help me out... Thank You Maghi Doctor: Hi, Thanks for your query. I read your query and understand your concerns. Following is my reply: Follicle growth seems to be good. Since you are trying for 6 years you go ahead with IUI for 3 cycles. If you dont conceive consider option of going for IVF. I hope I answered your query. I will be available for any follow up queries you have. Regards, Dr.Mahesh Koregol IVF & Infertility Specialist."
},
{
"id": 89009,
"tgt": "What causes severe abdominal pain?",
"src": "Patient: I have been to the toilet 5-6 times now. Last night sit was light brown and red like pure blood. With terrible abdominal pain on lower left side with copious amounts of gurgling. It has woken me twice, the pain. It s remains there like a cramp, and now there was gas followed by mucus and a trace of pink( blood) smells a bit fishy too Doctor: HI Thanks for posting your query . It seems like you are suffering from Dysentery . Dysentery could be due to bacterial or parasitic infection . It has to be found out what is causing dysentery . I advise you to get Stool routine and stool for Ova and cyst test done . This test will tell whether your dysentery is due to bacterial or parasitic infection. Also it's better to start antibiotics till you get the reports of the test. Ask your doctor for the prescription for antibiotics . Any clarifications feel free to ask ."
},
{
"id": 613,
"tgt": "Does severe abdominal cramping affect the survival of embryos?",
"src": "Patient: I mistakenly ate celery (i am allergice to celery) the same day of my embryo transfer. During the night, i had severe cramps which was followed by diarhea.....do you think that my embryos could have died because of it? does that affect anti-bodies? does severe cramping affect survival of embryos cause my doctor said the embryos would implant within 24hrs? Doctor: Hi, Thanks for the query. I understand your concern. Embryos are transferred in uterine cavity. If the endometrium is well prepared... of hormonal suppliments are sufficient.. &there is no uterine contraction,...abdominal cramps with a single loose motion without signs of gross dehydration &electrolite imbalance would not cause any embryo loss. The antibodies formed here are for the allergen( celery) & would not have any effect on embryo. You should take rest ,drink lot of water & ort powder to maintain hydration & also continue with the medicines advised. Thanks"
},
{
"id": 184355,
"tgt": "Does the permanent tooth need to be extracted or braced?",
"src": "Patient: I'm 18 and i have a baby tooth in front of the adult one, the problem is that the adult one is not in line with the others such that it is in a vertical position while the others are in a horizontal one. when i was younger a dentist took some x-rays and told me that when the baby tooth will fall out the other one would be extracted, but now another dentist told me ( without taking an x-ray) that the baby tooth should be extracted and the adult one will be put in place with a brace, but i;m not really sure about it. what should i do? 10x Doctor: Thanks for your query, I have gone through your query.The deciduous tooth can be removed after taking the radiographs and confirm the presence of the permanent tooth. once the permanent tooth is there then the deciduous tooth can be extracted and the permanent tooth can be aligned by braces or orthodontic treatment. Consult a orthodontist and take a opinion. You can get radiographs like OPG and lateral cephalogram to rule out the presence or absence of teeth and also plan the treatment.I hope my answer will help you, take care."
},
{
"id": 205318,
"tgt": "Are taking Nexito and Clonotril advisable for depression and stress?",
"src": "Patient: Hi from past 15-20 days am feeling mentally stressed , mentally disturbed in work area.My boss always scold me threatening me to terminate from my job even though I work for 12 -13 hrs per day..from 2 days am getting suicidal ideation so I consulted doctor today..he told me that I have depression and somatic complaints ..stress at work..so he suggested me to take Nexito 5 mg,Nexito 10 mg morning and clonotril tablets night..please tel me is it ok to take to overcome this ... Doctor: in my opinion you are suffering from depression ,hence you are having significant problems at work. i would recommend you to take nexito 10 mg and clonotril.25 mg . the medications would help you. in addition if you want you can also take some therapy. it will also help you."
},
{
"id": 130884,
"tgt": "What causes stiff neck, back pain and one sided numbness?",
"src": "Patient: I have had multiple symptoms for about 2 months now. Started with stiff neck/kink (constant). Then threw my back out 2 times in 1.5 weeks time. Nausea came on with pain from back. Neck still hurting through all of that. Then, there is the numbness all along the right side of my body (mostly limbs) even I'm my right labia. I'm totally perplexed. My right breast and armpit have been hurting as well. Doctor: I understand your symptoms. These symptoms mostly due cervical spine stiffness with spinal nerves compression. cervical spine x ray and CT should be done.Start with vitamin B12 injection once every 3 days and topical and oral non steroidal antiinflammatory drugs 3 times daily for 2 weeks."
},
{
"id": 56179,
"tgt": "Suggest remedy for large hepatic cysts in liver with gastritis and being on omeprazole",
"src": "Patient: Hello, I have two large hepatic cysts on my liver, and also have gastritis. I'm on omeprazole 20mgs twice daily. My diet is low fat and I'm not a big meat eater (I do eat it) My doctor did an ultrasound and found no masses. It's been a month now and I do feel better but, I'm not pain free. I always feel better if I don't eat. What is your opinion? Thank you,( AP ) My E-M is YYYY@YYYY Doctor: Hi. I think then both your cysts and gastritis have a contribution to your symptoms: your cysts by the \"mass effect\" cause an early satiety. So apart from Omeprazole I'd suggest eating with small portions and a prochinetic like Domperidone."
},
{
"id": 32279,
"tgt": "Is low BP while having treatment for viral meningitis a cause for concern?",
"src": "Patient: Hi - I m I m hospital having been diagnosed with viral meningitis and have a couple of questions: On my fourth litre of intravenous fluids. Anti sickness and painkillers. Hope to go home tomorrow BP has just measured 107 / 49 is that an issue? Also waiting to have a rash I have developed looked at by the Dr. How long should I be telling work to expect me to be at home ? Thank you Doctor: Hello, hope you are feeling better now. BP being on the lower side, without a high heart rate is normal in some individuals as is not a cause for concern. More important is how you feel overall, any giddiness or drowsiness, how much you are peeing etc. Rash is a part of the viral illness and will go away on its own. Take rest for 3 to 4 days at home, and then head to work if you are feeling better. Take care ! Hope this answers your questions"
},
{
"id": 9291,
"tgt": "Suggest treatment for dry skin and cracks in feet",
"src": "Patient: Hello Sir, Thanku for this opportunity. My Age is 31. My skin was very dry. And it seems to be shrinked not tight. Especially in winter, I suffer a lot with this dry skin. My foot is also very dry with cracks. Is there any remedy for that. Pls give ur suggession.ThankuRegards Doctor: hi, you are suffering from xerosis and planter keratoderma .it is an abnormal dryness of skin which gets worse in winter.Treatments include:Moisturizers, especially creams or lotions that contain urea and lactic acidTopical steroids -- for areas that get very inflamed and itchy.reventionDo not expose your skin to water more than needed.Use cool bath water.Choose gentle skin cleansers that are free of dyes and perfumes.hope this is useful.thanks"
},
{
"id": 182218,
"tgt": "What causes lump behind the tooth?",
"src": "Patient: hi. just woundering...... im worried about my bottom teeth. for a few months my wisdom teeth have been trying to come through, im 25 , but there is no room. the pressure is horrible it comes and goes for a week or so every few months, but its done something to my front tooth, teeth. i hav overcrowed teeth anyway at bottom , one of my front tooth on bottom sits behind others always has but its moving more i think and now there is a bone like hard sensitive lump behind that tooth ? what is it. it has affected the way i bite and eat. is it my tooth pushing out from underneath. ??? please help is this costly ???? Doctor: Thanks for your query, I have gone through your query.The pain and swelling(lump) could be because of the gum infection over the wisdom tooth (pericoronal infection leading to space infection).Consult a oral physician and get a radiograph done to rule the amount of space present for the tooth to erupt if space is there for the tooth to erupt then the gum covering over the wisdom tooth can be removed.If the space is not available then you have to get the wisdom tooth removed. Mean while you can take a course of antibiotics like amoxicillin and metronidazole (if you are not allergic). The wisdom tooth might damage to the next second molar. so get it treated as early as possible. There might be movement of the front teeth because of the pressure from the wisdom tooth.I hope my answer will help you, take care."
},
{
"id": 140384,
"tgt": "Are heart rate variations a serious concern in a brain aneurysm patient?",
"src": "Patient: my stepson had a brain aneurysm five years ago and he is vent dependent and Is fed through a g-tube. He is also not mobile. requires 24 hour care. last night his heart rate was 39-40 and his oxygen was 99-100. His heart rate has never dropped and stayed that low. it will occasionally drop if he is cold but as soon as we put blankets on him and make him warm his heart rate goes back to 50 s. Doctor: Hi, There are other reasons that heartrates can become low and fail to correct and perhaps in your stepson, one consideration would be an infection in the bloodstream from either a bladder, kidney, skin or pulmonary infection. Low heartrates that do not correct may be signalling a situation where bacteria have gained access to the bloodstream and are beginning to affect his hemodynamics. If he is at home I would contact his doctor and have someone come out to take a look at him and consider getting some labs and/or other tests performed. Hope I have answered your query. Let me know if I can assist you further. Regards, Dr. Dariush Saghafi, Neurologist"
},
{
"id": 20289,
"tgt": "Suggest treatment for Tachycardia and RUQ pain with high BP",
"src": "Patient: I am 33 y/o G3P2 at 32 weeks suddenly experiencing SOB, Tachycardia (120's-140's) at rest for the past 24 hours. I also have RUQ pain, not always associated with meals. I have chronic hypertnesion on Labetolol 100mg TID. A PE has been ruled out as of yesterday, but there were no other concerns for the symptoms. I am concerned, should I be? Doctor: Yes and undoubtedly your doctor is as well since the testing was done to look or pulmonary embolus. At this point of pregnancy, many of us are similar but none are the same. Your doctor knows the information specific to you that will help to determine the likely source of change and what should be done. He or she is the best resource."
},
{
"id": 163921,
"tgt": "What does pus on finger of toddler mean?",
"src": "Patient: Hello there. Just this morning, I discovered a pus on my 5 month s middle finger (just like when you get ingrown). I suspect a slight tear on the skin may have caused it while nail clipping. We went to the doctor and he prescribed Mupicin and Amoxycilin (100mg/ml) 3 times a day for 7 days. Would taking antibiotics hurt my baby? Can we not just wait for it to heal by itself? or would topical cream be sufficient? Thanks in advance. Doctor: hi..as per your concerns , what is not clear is the exact site of discharge , is there a nail swelling, is there any redness, any discomfort in the baby, or any lethargy associated with fever, how frequent is he discharge and how big is the injury/wound?usually mupirocin ointment is the topical antibiotic used in such situations but if shown to a doctor and amoxicillin was given is maybe for prophylactic measure which would help in healing . do not worry no issues as such for you to worry about:) just make sure u feed the baby well . keep the are under good hygiene ."
},
{
"id": 133581,
"tgt": "What causes pain in leg while turning aside after an injury to knee?",
"src": "Patient: I m 37yrs old. I met with an accident 6 months before and hand 5 stitches beside my right knee The wound was deep but no bleeding was there. I still feel pain there.From last week I have pain in my thigh and feeling pain.in whole leg while turning aside during sleep. Pain.is also there while descending stairs n while sitting or standing. Can u.plz tell me the reason 4 that Doctor: hi,thank you for providing the brief history of you.A thorough musculoskeletal assessment is advised.As you had an injury which was treated with stitches there will be a load shifting leading to muscle imbalances. Performing exercises for the knee joint will be helpful.undergoing a proper clinical physical therapy assessment will help understand the muscle weakness involved and with guided exercise the pain will be avoided.on a longer run it is advised to undergo proper physical therapy rehabilitation of the knee to avoid future early degenerative changes in the knee.RegardsJay Indravadan Patel"
},
{
"id": 104788,
"tgt": "Frequent swelling in lips, occasional tingling. Could this be food allergy?",
"src": "Patient: In the past few weeks I have woken up with swollen lips 3 times. I ve has helped and the swelling has gone down. Today they feel a little tingly And irritated. Swelling has gone down. I have not worn make up for 2 weeks and have only used vasoline or Ice for relief. Could this be a new food allergy?email YYYY@YYYY . My name is Rachel and I am a 30 year old female. Doctor: Hello Rachel, Food allergy with lip tingling and swelling (angioedema) is likely provided you get the feeling immediately after eating the 'offending' food. Any foods can cause this, but has to be almost immediately or within the hour. Blood pressure pills such as enalapril, captopril, trandolapril, lisinopril etc so please ensure your current medications are checked with your doctor if you are on any.. If now food allergies are unlikely, then this is idiopathic angioedema and you need to be on cetirizine 10mg once daily for 1 month to see if this recurs. If it still recurs on antihistamine, please get complement proteins C3 C4 leves checked. Best Wishes."
},
{
"id": 66443,
"tgt": "What causes a hard and painful lesion on the butt?",
"src": "Patient: I have a painful and hard lesion on my left bum, deep not in front but rather on the cheek of the bum and is so hard and very painful. Yes I do sit on the hard chair for long hours as I work on the computer, could it be the cause. I m also a diabetic.... Doctor: Hi, thanks for sharing your health concerns with HCM! If I were your treating Doctor for this case of painful butt lump, I would come up with three possibilities especially as you are a diabetic, these include: 1.\u00a0\u00a0\u00a0\u00a0\u00a0an abscess2.\u00a0\u00a0\u00a0\u00a0\u00a0The second possibility is of an infected benign cyst like sebaceous cyst\u00a0\u00a0\u00a0\u00a0\u00a03.\u00a0\u00a0\u00a0\u00a0\u00a0The last possibility is of an inflamed bursa or tendinitis !\u00a0\u00a0\u00a0\u00a0\u00a0Overall, it is benign but infective and not to worry about this but you could go for FNAC test for confirmation and appropriate line of treatment!Hope this answers your question. If you have additional questions or follow up questions then please do not hesitate in writing to us. I will be happy to answer your questions. Wishing you good health."
},
{
"id": 16019,
"tgt": "Severe itching & small red rashes on abdomen & then all over the body. Treatment ?",
"src": "Patient: During my first pregnancy (15 yrs ago) i had experienced severe itching & small red rashes on my abdomen & then all over the body except on my face.After the delivery the itching slowly disappeared. Almost after 13 yrs during winter i had same problem again that only abdomen and upper thighs. Our family doctor prescribed Clobetosol with Propisylic acid cream for local application, this worked out well ,so i continued using for subsequent winter period as i had the same problem . This year the same treatment is not at all working on me, instead the rashes are spreading towards my lower legs with severe itching. I want proper advise and treatment for this. Doctor: Your symptoms are suggestive of Acute Urticaria. This is usually caused by either intake of a new prescription drug like penicillin or an OTC drug like aspirin, systemic infection (either bacterial or viral), food additives, seafood or intestinal worms. The last is usually only of significance in children. Stress is also a major cause of an outbreak. Now about the drugs you are on - they are unlikely to be the culprit since you have been on them for so long. Many times it is not possible to find out the cause, however since most episodes of acute urticaria are self limited within 4-6 weeks of onset of symptoms, it is not imperative to do so. I would recommend taking Tablet Claritin 10 mg once or twice a day for 2 weeks + calamine lotion over the affected areas whenever symptoms occur and seeing whether they recur on stopping the medicine after 2 weeks. If they do then restart treatment for another four weeks and then stop and see. If they still recur then you will require some blood tests and will need to see a doctor in person. Please note that if you have any lip swelling or difficulty in breathing you should see doctor right away. Hope this information is useful, please feel free to ask if you have any queries. Please feel free to ask if you have any queries."
},
{
"id": 219949,
"tgt": "What are the chances of pregnancy after taking Postinor 2?",
"src": "Patient: Hi doc, i have 3 questions i need answers to and will greatly appreciate your help and response. 1) i had unprotected sex 3 days after my period and took postinor 2 about 10hours after. Am i still at risk of getting pregnant? 2) i have been having irregular periods for about 5-6 months now. what could be the cause/ 3) i usually have boils around my private part before or during my periods...is it normal or what could be the cause? Doctor: Hallow Dear,Let me answer your queries one by one. 1. In fact there was no risk of pregnancy from the unprotected sex performed just 3 days after the period as it was far away from fertile period. The egg is formed 14 days prior to next period. It has life of 24 hours while sperms are active for 72 hours. Hence a period of about 8-10 days is considered as fertile period. Intercourse during this period has maximum chances of pregnancy. However, your menses being irregular, it is very difficult to locate the day of egg release. (May be you could be having problem in egg release also.) Still, considering your menstrual irregularity, you have taken a wise step in taking Postinor 2 after the unprotected intercourse. Postinor 2 is quite effective post coital emergency contraceptive pill if taken within 72 hours of unprotected sex. You have taken it after 10 hours. So you need not worry about pregnancy. You are well protected. 2. Your irregular menses need to be investigated thoroughly. There are many conditions that lead to menstrual irregularities. Some of them are Polycystic Ovarian Syndrome, Thyroid dysfunction, any other reproductory dysfunction, ....... it will fill your complete desk top if I start enumerating all of them. I can guide you on this issue thoroughly if you inform me your age, menstrual pattern, age of menarche (first menstrual appearance), marital or sexual history. You can inform me all this by asking a Direct question. Else, you may report to the Gynaecologist. Irregular menses may result in infertility due to non-release of eggs. For the time being, you may try cyclical birth control pills for three months for regularization of menses as a preliminary step. 3. Boils around the private parts are usually due to infections; may be sexually transmitted infections like gonorrhoea, herpes genitalis, etc. You need to report to STI specialist for correct diagnosis and appropriate treatment. You may need antibiotics treatment. Also you will have take care of genital hygiene and be demanding for clean sex only. I am sure this has given you some direction on your reproductory health problems. Dr. Nishikant Shrotri"
},
{
"id": 102745,
"tgt": "Are the hives around the arms, neck and backside and swollen lumps allergic reaction to raspberry extract?",
"src": "Patient: I make fudge every year, normally orange creme. This year, I tested out a raspberry and creme fudge within 24 hours. About 2 hours after making it, I had small but multiple hives appear all over my arms, torso, neck, and backside. Today, not even 12 hours later, they are huge swollen lumps and I am shivering; cortizone cream is not helping much. Is this potentially an allergic reaction to raspberry extract? Doctor: Thanks for query on HCM.Your symptoms and history suggest allergic reaction due to local cream.Try to avoid present local cream or switch over to other cream.Only local treatment is not enough for this.Continue to apply cortizone cream.You have to take levocetrizine and short course of mild steroid.Be watchful for next time.Otherwise consult dermatologist for further guidance."
},
{
"id": 43655,
"tgt": "Difficulty in impregnating, sperm count related issues. How can I improve it?",
"src": "Patient: hello doctor ... here there is a problem that i got a report done for my sperm count n related issues .. as not able to hav child ... the rapid progressive 09%,sluggish progressive 03%,non progressive motile 05% and immotile sperm 80% ... i know m lacking in rapid progressive ... can u plzzz plzzzz advice me how can i improve upon that ... my sperm count is 26.5 mil./ml ... plzz advice my married life is going very tensed ... n my family environent is totally disturbed as m not able to get a child .. plzz help Doctor: Hello, Welcome to HCM, I am Dr. Das The Sperm count is within normal limit ( as per WHO 2010, the normal limit is >15 millions /ml) Sperm motility is lower than required ( as per WHO 2010, the normal limit is >50% motility). So, right now, you need vitamin E and other micronutrients containing Multivitamin agent. Coenzyme Q is also necessary. Consult with an infertility specialist for further management. Regards."
},
{
"id": 209349,
"tgt": "What causes dizziness, upset stomach, hot flushes in a panic disorder patient?",
"src": "Patient: I am having dizziness, upset stomach, pins and needles and numbness feeling through my body, extreme hot flushes and sometimes feeling like I can't breathe good. I do have panic disorder and have had complete blood tests which have all been perfect. Doctor: Panic disorder is a type of anxiety disorder. In this disorder there is hyperactivity of autonomic nervous system. Autonomic system hyperactivity leads to increase in levels of adrenaline which leads to all of the above symptoms."
},
{
"id": 94723,
"tgt": "Light internal pain on left stomach, endoscopy normal. What is the remedy ?",
"src": "Patient: hii sir, I am something feeling some internal light pain on my left stomach like air in my stomach but when i press the area i dont feel anything . And not in a particular place but any place in lefet belly. i did endoscopy and everything found ok. I think l have gain weight bcos my wt was 70kg now its 76 kg. There is no symptoms nothing. I am feeling hungry, tool and urin is ok. No hard pain in my stomach. no food irritation. only thing is my urin colour is yellow but after drinking water its as usual natual color. Genarally when i sleep i feel the light pain. Can u clear me wat exactly it is and wat is the remedees for it. Doctor: Hello! Thank you for the query. Such symptoms can be caused due to diet causing bloating, diverticles or some other large intestine disease. It can be also caused by stress and can be not significant. However as in left abdominal area mostly large intestine is located it is advisable to have some diagnostics before diet change etc. Before upper GI endoscopy you should have abdominal ultrasound and full blood work, liver tests, pancreas tests, pregnancy test performed. After this tests, if no findings, colonoscopy might be necessary especially if you also suffer from constipations, blood or mucus in stool. Only after this tests you can explain your symptoms with bad diet. Beans, broccoli, cauliflower, cabbage, sprouts, onions and garlic can cause such feeling if large amount consumed. Physical activity always makes intestines to work property. Hope this will help. Regards."
},
{
"id": 176672,
"tgt": "Suggest remedy for stomach pain after a fall",
"src": "Patient: My 10 year old daughter was climbing on a wooden fence today when the middle rail that she was standing on fell. She landed on the top rail on her upper abdomen/lower ribs. She also had on her book bag that added an extra 25 lbs. She fell to the ground and laid on there for about 15 min in pain but still breathing normal. All afternoon she has held her stomach in pain, but still eats and breathes normal. I m sure she will be sore but wanted to know are there any symptoms I should be on the look out for? Thank you! Doctor: Hello. I just read through your question.The pain should improve a little bit everyday. If you see that the pain is not getting better day by day, you should consult your doctor. Additionally, vomiting, difficulty breathing, or blood in the stool, would all be reasons to seek medical help."
},
{
"id": 95612,
"tgt": "Constipation in baby who had an anus surgery and colonostomy done during birth",
"src": "Patient: my baby is put on milk of magnesia and sometime she is having constipation problem. anus surgery and colonostomy done during birth Doctor: THankd for the query Your query has no question in it, i kindly ask u to reconsider and frame a better query so that we can be of help Have a healthy living"
},
{
"id": 53640,
"tgt": "Suggest treatment for pain in gall bladder due to 8-9mm stone and cholecystitis",
"src": "Patient: i have been diagnosed to have a cholecystitis, i've been hospitalized last jan 14. on my ultrasound report, it has been seen that i have a 8-9mm gall stone. after my 2 weeks of taking antibiotics and medicines, the pain now is always. it hurts everyday. do i really have to undergo an operation? Doctor: Hello and thank you for asking HCM.I have read your symptoms and i understand your concerns.Cholcecystitis,means infection of the gallbladder. And since you have a stone treating it with antibiotics probably will not work. And since you are having pain always,it means that surgery is the only option left. If i was your caring doctor ,i would do some blood tests first to check the liver enzymes (AST,ALT,GGT,SGOT,SGPT ect), levels of bilirubin and the pancreatic enzymes. Than i would recommend to check up with a surgeon and have the surgery because if left untreated you might have serious consequences. According to your history,yes i am afraid that surgery is the only option for you.Meanwhile have this diet,to help your body also:-avoid concentrated fats like oil, butter, margarine, fat on meat -fresh fruits and vegetables-avoid alcohol until after you have had your operationIf you have any other question,feel free to contact me.Dr.Endri Katro"
},
{
"id": 73179,
"tgt": "Can Pneumococcal vaccine be taken after being diagnosed with pneumonia?",
"src": "Patient: Hello Doctor, I am COPD patient. I was taken a vaccine called Pneumo 23 almost 4years and 8months ago. The doctor advised me to take the vaccine once in every 5 years. But before the completion of 5 year I am diagnosed with Pneumonia. What should I do? Should I take the vaccine before the completion of 5 year period? Please advise. Doctor: Thanks for your question on Healthcare Magic.I can understand your concern. Pneumococcal vaccine is live attenuated vaccine. So it is contraindicated in active infections like pneumonia because it can flare up infection. So better not to take pneumococcal vaccine at present. Get complete cure from current pneumonia and then take pneumococcal vaccine. Don't worry if time period between two vaccine is more than 5 years.Hope I have solved your query. I will be happy to help you further. Wish you good health. Thanks."
},
{
"id": 211138,
"tgt": "What causes bed wetting incidents after taking saphris for bipolar disorder?",
"src": "Patient: Hello, my son is 14 years old, and suffers from bipolar disorder..He is on 900 mg of lithium daily and 10mg of saphyris. He just started the saphyris 4 1/2 weeks ago. He has started having bed wetting incidents at night now. I was just wondering if this was a side effect of that medication. I have looked at some of the clinical pharmacology for it, but don't see it listed. If it is a side effect, will it stop, or lessen? Is it a serious side effect? He was on abilify but had some adverse effects and we had to discontinue it and start the saphyris. Doctor: hithanks for using healthcare magicSaphyris does not cause urine incontinence, but due to its sedative effect sometime patient would not able to control urine while sleeping which lead to urine incontinence. it is not a serious side effect. Same could be the reason in ur case also. In that case either need to stop the drug or decrease the dose is required. Rest u can discuss with his treating psychiatrist.thanks"
},
{
"id": 22889,
"tgt": "How to lower cholesterol levels?",
"src": "Patient: I have been experiencing various strage symptoms this past month. I was put on flecanide 50 last Dec. And propanolol 40 to take with it to prevent arrythmias. I have had lone afib a few times a year for a few hrs and have had all the work ups done with normal results. I also have had svt on a d off all my life with no problems except inconvenience. So they tried to ablate it in Dec and it was not successful. So that is why I am on flecanide and propanolol. This has worked well until last month. I has a routine d&c for what they thought was a polyp and turned out to be nothing. During the procedure my heart rate went down to 30 and they gave me robinal. A week after that I was started on protocol for a cholesterol of 278. My cholesterol has always been ok before that. 7 days later and 10 days after the d&c I began experiencing lightheadedness and a bp of 90/60 and a pulse in the 40s. I went to the hospital and was kept overnight on tele. All was ok hr in 40s-50s. Had normal echo in am and sent home. I hadn't taken propanolol since the night in hospital and began to experience pounding heart rate headache shakiness and over all terrible. Cardiologist said it was beta blocker withdrawal. They put me on half dose, back on flecanide,off protocol, and did stress echo and 24 hr halter and said it may have been vago vasel related. I went back to the gun for post op f/u and c/o lower and cramps. Found to have vaginal infection, on abs now. What bothers me is that I am still having these strange lightheaded moments and I don't know why. Everything seems to check out ok. It is not like fainting or dizziness. My pulse and bp is ok except that my pulse does get somewhat faint during it. Sometimes I get a cold feeling on the side of my head with and lingering after it. I am normally very very healthy and I don't know what is happening with me. Is there something I can do to find an answer? I am 62 and a little overweight. Thankyou Doctor: DEAR USERTHANKS FOR CONSULTING AT HCMI UNDERSTAND YOUR CONCERN.. BUT FROM YOUR HISTORY THAT YOU PROVIDEDA PAST HISTORY OF LONE AD AND PSVT WITH FAILURE OF ABLATION THERAPY POINTS THAT YOU HAVE THESE DIZZINESS SYMPTOMS ARE A PART OF CARDIAC SYNCOPE LIKE FEATURESTHOUGH YOU HAVE NO LOSS OF CONCIOUSNESS IT'S PROBABLY BECAUSE OF UNDERLYING CARDIAC ARRYTHMIA I SUGGEST YOU TO GO FOR A 24HR HOLTER MONITORINGGET A ECHOCARDIOGRAPHY DONECONSULT A CARDIOLOGIST SO THAT YOU CAN BE PUT ON A PROPHYLACTIC INTRACARDIAC DEFIBRILLATOR THAT CAN PREVENT FURTHER RISK OF ARRYTHMIAHOPE I HAVE ANSWERED YOUR QUESTION.. YOU CAN MESSAGE ME FOR ANY FURTHER CONCERNS"
},
{
"id": 1489,
"tgt": "Is there possibility of conceiving after taking fertibex clomesoul and betnesol?",
"src": "Patient: I am 26yrs old lady. I am married before 1 1/2 yrs and got conceived in after one month of marriage.Because of certain situations i aborted that one with tablet in 37 days. Now past 8 months, we are again trying for baby. But i could not make it. Everything is normal except Hb is 9. I am advised to take Fertibex for 30 days and clomesoul-50 tablets for five days(3rd - 7th day of period) and betnesol(3rd - 7th day of period). Is there any chances for me to get pregnant this month. My periods are normal 28 days. Doctor: Hi, there are chances of pregnancy. But I think you should track your follicles growth by repeated ultrasound and when your follicles is more than 17 to 18 mm, take injection for rupturing the follicles. Be in contact with your husband for 2 to 3 days. Take progesterone for next 2 weeks. Do a urine pregnancy test at home after that. You can try like that for 3 to 6 months. Hope I have answered your question. Regards Dr khushboo"
},
{
"id": 51024,
"tgt": "Blood in urine, nephrotic syndrome, protein leakage. Interstitial inflammation in kidneys. Causes?",
"src": "Patient: My father, 72 yr old, having little blood in the first morning urine, but no pain or burning sensation. He is also having nephrotic syndrome , more than 10 gm protein leakage per day. Glomerullies were not found during kidney biopsy . So no proper identification of the category of Nephrotic syndrome was made. Only interstitial inflamation in kidney were found. Immunofluroscence report shows IgA. What can be the cause? Doctor: Hi welcome to HCM. Once you rule out stones; infection or tumour by doing urine analysis and ultrasound scan, we need to think of the glomerulo- nephritis as the cause. Kindly do the above tests and we can take it form there. best regards DR GS"
},
{
"id": 82428,
"tgt": "Suggest treatment for chest cold",
"src": "Patient: for a week now been battling a chest cold which is now a chest and head cold...stuffy runny nose coughing throwing up weakness and tingling in my arms and hands stomach pains hot and col flashes inability to sleep and no appetite..should i continue to wait it out? Doctor: Thanks for your question on HCM. In my opinion you are having severe upper respiratory tract infection (URTI) Due to viral infection. So better to follow these.1. Drink plenty of fluids.2. Warm water gargles 5-6 times a day.3. Anti histamine drugs.4. Anti inflammatory drugs.5. Macrolide group of antibiotic. Try to follow these for 5 days. If not improved than consult doctor."
},
{
"id": 185298,
"tgt": "Suggest medication for Pyorrhea",
"src": "Patient: Hi, may I answer your health queries right now ? Please type your query here... HI DOCTOR AM FROM OMAN CAN YOU KINDLY UPDATE THE MEDICINE FOR PYORRHEA. I HAVE NO PAIN, BUT LITTLE BIT BLEEDING & HEAVY BAD BREETH. KINDLY UPDATE ME WHAT IS THE MEDICINE AVALIABLE IN OMAN Doctor: Hello!Thank you for posting in HCM.Bleeding and bad breath suggest a bad gum health.This cannot be diagnosed as pyorrhoea without a clinical evaluation.Debri between your teeth cannot be removed with a home remedy.You must get a professional scaling done with your dentist.The symptoms can be controlled with a change in oral health habits.Use a warm saline mouth wash.Maintain a good oral hygiene.Gargle after every meal.Floss daily.Consult your physician if you have a acid reflux.See your dentist every 6 months for a general cleaning.Regards."
},
{
"id": 155081,
"tgt": "Is lumpectomy a good solution to recover from tumors?",
"src": "Patient: I would need an oncologists opinion. I have been diagnosed with breast cancer. Had a lympectomy, and a few nodes taken out, and they had no cancer in a small margin of the tumor (which was very small) and nodes were clear. It is called a negative 3. The oncologist said the best chance of this type of cancer from coming back is chemo and radiation. Which I have had my first but wonder if I am making the right decision. He feel it is the right one for me. Do you think it is. Doctor: Hi, dearI have gone through your question. I can understand your concern. You have breast cancer with negative nodes. Modified radical mastectomy with chemotherapy and radiotherapy is the treatment of choice. You may have triple negative cancer it means you have ER, PR and her 2 neu negative. So hormonal treatment is not effective. You need chemotherapy and radiotherapy after surgery. Consult your doctor and take treatment accordingly. Hope I have answered your question, if you have doubt then I will be happy to answer. Thanks for using health care magic. Wish you a very good health."
},
{
"id": 36108,
"tgt": "Are fever, head ache and nausea symptoms of flu?",
"src": "Patient: i feel so sick. i have fever, head ache nausea. this started on tuesday. yesterday and today i woke up feeling good. But today after the gym as soon as i got back home the fever hit me all over again. this time hard. my temp was 102.3 and i feel nausea again. is this the flu? Doctor: Hi, dearI have gone through your question. I can understand your concern. Your fever , headache and nausea can be due to flu, malaria or thyphoid. You should go for complete blood count, test for malaria antigen and widal test.Then you should take treatment accordingly. Till that you can take paracetamol and pentoprazol for symptomatic treatment. Hope I have answered your question, if you have doubt then I will be happy to answer. Thanks for using health care magic. Wish you a very good health."
},
{
"id": 152802,
"tgt": "Can Metaline destroy the remaining adrenal gland?",
"src": "Patient: I had my right adrenal gland removed because of a tumour on the cortex...the histolics show a small spot in the centre of clear cells .not divided ..they are tslking about using Metaline ? as a precaution measure ..will this destroy my one remaining Adrenal Gland ?? Doctor: Hi and welcome to Healthcaremagic. Thank you for your query. I am Dr. Rommstein and I will try to help you as much as I can. It will not destroy your adrenal gland but it may causes certain changes in hormonal levels. You should check your cortisol andaldosteron levels if you plan to take it frequently. I hope I have answered you query. If you have any further questions you can contact us. Kindly regards. Wish you a good health."
},
{
"id": 79522,
"tgt": "What causes chest pain while sneezing and coughing?",
"src": "Patient: Hi i have a question i was involved i. A car accident 2 months ago i been havin some chest pai. ..when i presed to har on my chest it hurts,when i snezz,when i cough,and even sometimes feel just a little short on breath...i made a dr appt for this friday...i what im feling bad Doctor: Thanks for your question on Health Care Magic. I can understand your situation and problem. You had blunt chest trauma due to car accident. And this can cause following complications which cause similar symptoms like chest pain on coughing, sneezing and breathlessness. 1. Rib fracture. 2. Pneumothorax 3. Pulmonary contusions 4. Musculoskeletal injury. So get done chest x ray to rule out all these complications. If chest x ray is normal than no need to worry much. You are mostly having musculoskeletal pain. So avoid movements causing pain. Avoid bad postures in sleep. Avoid strenuous exercise. Start painkiller and muscle relaxant drugs. Apply warm water pad on affected areas. Don't worry, you will be alright. Hope I have solved your query. I will be happy to help you further. Wish you good health. Thanks."
},
{
"id": 188180,
"tgt": "Suggest natural treatment for metallic taste in mouth",
"src": "Patient: Hi, I am experiencing some metallic taste with food. This is annoying and I d like it to stop. Do you know what it might be? Also, do you know how to find a doctor who is interested in more natural treatments? If I look at supplements, it always says to ask your doctor, but where I live, the doctors say just be careful and look up the information. Does anyone know what is going on? How to find a physician who has knowledge of herbal and other alternative therapies. Doctor: Hello, thank you for sharing your problem with us.There are many reasons for metallic taste in the mouth, it can be because of any medication that you are taking, because of cold when your nose is blocked or can be because of smoking also.First thing you should do you should rule it out why it is happening and find out the cause because without knowing the cause no treatment will be effective for you.And also one thing you can do to drink a lot of water to reduce this sensations."
},
{
"id": 188557,
"tgt": "Saliva test shows 6.0 ph. Tried with protein, carbohydrate food. Smells like vinegar. Due to oral hygiene?",
"src": "Patient: I tested my saliva and found out its 6.0 ph most of the time and it is the lowest around 5.5 ph in the morning when I wake up , when eating it raises to 7.0 ph. I experimented with different diets during several months (such as 80% fruit, vegetables; 20% protein, carbohydrate containing food) however, ph of saliva remains the same level about 6.0. Also I noticed it has smell like vinegar. I had several cavities in teeth recently and I think this is the reason because my oral hygiene is good. What is wrong with me? Doctor: Hi,Nothing to worry about pH of your saliva.This might be due to having cavities in your teeth.Go for the treatment of cavities.Keep oral hygiene proper with proper tooth brushing, gargling.Avoid more carbohydrates, chocolates and aerated drinks.Ok and take care."
},
{
"id": 38617,
"tgt": "What is the treatment for high fever and cold?",
"src": "Patient: My father 58 years old has under gone a kidney stone removal procedure by endoscopy method and a stunt was placed. Since then he is having WBC count between 25300-15000 and hemoglobin levels of 9-10.3. along with this ,his urine report says it contains pus cells 10-15 and traces of WBC and proteins. His urine culture is negative. He is suffering from frequent high fever, cold and severe uneasiness. write now using ciprofloxacin drug for this. He is having high BP with cardio myopathy (since 17 yaers),Diabatis and chronic pancreatetisis.How to over come recent issues please suggest. Doctor: HelloThis is Dr Adil Ramzan. Thank you for asking question. A high white blood cell count suggests an acute infection. I would also like to know the percentage of neutrophills and some other cells to have a better idea about the acute or chronic nature of infection. anyhow, high wbc count suggests that there is an ongoing infection inside your father's body. Having pus cells in urine, suggest there is urinary tract infection and usually the first drug that is periscribed in the absence of culture and sensitivity report is ciprofloxacin. Moreover, if your father's wbc count don't decrease within next 2 -3 days then you may consider shifting him to hospital for a course of injectable antibiotics such as ceftriaxone. Moreover, if culture report is negative but still there is fever and high wbc count then i would continue antibiotics and explore for any other source of infection my asking more questions and doing examination and also i may require few more investigations. for example a chest x-ray as you have told that your father is also suffering from cold. Kind regards. Dr Adil"
},
{
"id": 123057,
"tgt": "What causes tremors in the morning times?",
"src": "Patient: thanks for your expert medical tips. I am 45 years &taking 3 tablets of Liv-52 thrice a day for the last 30 days. After this i feel tremors in morning times (eaten or not eaten) my fasting blood sugar : 140 and post meal is 219. Could this due to excessive dosage of Liv-52? My parents doesnt have diabetis. Doctor: Hello, As the medicine Liv-52 will not be causing tremors. If you are taking this medicine there will be a possible diagnosis for the prescription of this medicine to you. We need to know this. Also, the tremors are not connected to the medications and may be a possibility of some other pathology involved. Giving a detailed medical history for what this medicine is advised can help us give a possible reason for the tremors. Also, your blood glucose levels are on a higher side and need attention. With proper Medications, diet and exercise your blood sugar levels can be controlled. Hope I have answered your query. Let me know if I can assist you further. Regards, Jay Indravadan Patel, Physical Therapist or Physiotherapist"
},
{
"id": 40319,
"tgt": "How to treat frequent urination with uti test strip showing leukocytes but negative nitrates?",
"src": "Patient: I used a Wallgreen uti test strip and it showed leukocyites but negative nitrates. I only have frequent urine for symptoms. I did have a nasty UTI and ended up allergic to 5 different antibiotics so I am worried. I have no insurance and also have diverticulitis. Doctor: Hi,Welcome to HCM.If leucocytes are positive and nitrates negative, it indicates infection by bacteria which do not reduce nitrates. Frequent urination is the most common symptoms of an UTI. I suggest you get a urine culture sensitivity and please mention to the testing lab, the antibiotics you are allergic to. Drink adequate water and maintenance of personal hygiene is a must.Thanks."
},
{
"id": 115519,
"tgt": "What causes decline in iron levels?",
"src": "Patient: my husband operation in his legs to clear out plaque and insert stents in JaNUARY. PRIOR TO THE OPERATION HIS IRON LEVELS WERE SLIGHTLY LOW HOVEVER SINCE THE OPERATION T LEVELS ARE WORSE AND HE HAS BEEN REFERRED TO THE HAEMOTOLOGY UNIT IN HOSPITAL. COULD HE HAVE LOST TOO MUCH BLOOD Doctor: Hi, dearI have gone through your question. I can understand your concern. Causes of low iron are either nutritional deficiency or due to chronic blood loss. He should go for complete blood count, peripheral smear examination and anemia profile. Then start treatment accordingly. He should take iron rich food like dates, green leafy vegetables, dry fruits, jeggary etc. Oral or injectable iron can also be taken. Consult your doctor and take treatment accordingly. Hope I have answered your question, if you have doubt then I will be happy to answer. Thanks for using health care magic. Wish you a very good health."
},
{
"id": 98096,
"tgt": "Fallopian tube blocked. How to unblock tube IVF or laproscopy?",
"src": "Patient: My fallopian tube are blocked -: 1. Utrine cavity is well outlined & Arcuate in cotour. 2. No filling defect is seen. 3.Right fallopian tube is faintly upto isthmic end. 4. Left tube is not visuzalized at cornual end. 5. Arcuate utrine cotour. 6. Bilateral fp tubal bolcked What treatment should i go for IVF or laparoscopy to unblock the tube. Doctor: 1. bilateral blocked Fallopian tube: no role of medicine as there is no problem in Physiology but has to be Surgical as problem is with the Anatomy of the body. 2. any h/o Tuberculosis in past? 3. you should go for IVF."
},
{
"id": 69376,
"tgt": "What is the sensitive lump inside my belly button?",
"src": "Patient: hello, i have had what i assumed to be a spot inside mt belly button for about four or more months now. It doesn't have a head on it like a spot it is just a hard lump that is sensitive to touch. I wake up with a crust in my belly button as if it has been leaking. what could this be? Doctor: Hi,It seems that you might be having some infection in your belly button due to non cleaning of the part and deposition of dust and formation of crusts.There is another possibility of having Grannuloma in belly button.Consult surgeon and get examined.Ok and take care."
},
{
"id": 205963,
"tgt": "Suggest treatment for attention deficit disorder",
"src": "Patient: I have ADD and have been taking Adderall for 11 years (30 mg each am). It does not seem to work as well now. I am age 61. Should I have my MD increase my dose or try another medicine in case I have developed a tolerance? What do you recommend trying? Doctor: DearWe understand your concernsI went through your details. I suggest you not to worry much. ADD is a mental disorder which needs medicines and life style changes for cure. You should know that these problems are not mental diseases, but are mental disorders. Many researches and researchers confirm that medicines alone cannot cure mental disorders. Exercise, Life style changes, change in thinking pattern, relaxation etc are as essential as medicines. Psychotherapy can help you changing your lifestyle and thinking patterns. Yoga and meditation help you to streamline your metabolism and neurological balance. Please consult a psychologist for further information.Psychotherapy techniques should suit your requirement. If you require more of my help in this aspect, Please post a direct question to me in this URL. http://goo.gl/aYW2pR. Make sure that you include every minute details possible. I shall prescribe the needed psychotherapy techniques.Hope this answers your query. Available for further clarifications.Good luck."
},
{
"id": 142992,
"tgt": "What to do if having a probable diagnosis of CJD disease and cannot be fully diagnosed till death and having brain and nerve issues?",
"src": "Patient: my husband has a probable diagnosice of cjd diease lumbar protein levels were 14-3-3 losing nerve function in legs and arms also has lesions on brain vision problems cant work tremors and start of dementia the neurolisgts gave us 2 years he says there is no 100% daigosis till death we are ar a loss what should we do I am already paying different drs for test and daignious I don t think I want to pay online again too sorry Doctor: If you are dealing with a neurologist who knows well how to use the 14-3-3 test then, the likelihood of an incorrect diagnosis is small. The 14-3-3 is best applied when the PRETEST probability of having CJD is HIGH..in other words, just on clinical history and physical examination the neurologist believes that CJD is high on the list of possibilities. Assuming the clinical diagnosis is highly likely to be CJD, the neurologist will likely pursue other testing measures such as an EEG and MRI of the brain with FLAIR and DWI sequences (more sensitive and specific in most patients than the 14-3-3 assay). The neurologist will also work very hard to rule out all possibilities of any other type of infectious or demyelinating illnesses (including inborn errors of iron/copper/zinc metabolism) as well as reversible or treatable forms of dementia in categories such thyroid encephalopathy and B12 deficiency. Under these circumstances then, the clinician will have done everything possible to find every other diagnosis. Therefore, if this type of comprehensive investigation does not yield any positive results then, what you are left with is the original presumptive differential which should be then, diagnosis of the most likely clinical diagnosis. This could be verified upon autopsy but in my experience it is rarely if ever the case that families will ask for that kind of confirmation if the workup as I've described has been carried out. In other words, the diagnosis is as certain as it can be under these conditions. If, however, this type of workup has NOT been pursued then, there is still more to do in my opinion to solidify the final thoughts and judgments of what your husband may or may not have.Just as an aside- I understand the implications and use of the 14-3-3 CSF assay in quite a bit of detail since it was originally developed and tested at my training institution during the time I was a neurology resident. The doctor who headed up the initial discovery of the protein and its subsequent approval for CJD screening by the FDA was Head of the Neuropathology Division at that time. Trust me when I tell you that we were intensely trained on when and how to use this test as well as its interpretation as well as when NOT to use it and when NOT to put very much stock into the results. Feel free to send more questions to: www.bit.ly/drdariushsaghafi and I would also appreciate your 5 STAR rating of our interaction."
},
{
"id": 76411,
"tgt": "What causes cracking sensation on the chest and shoulders with pain?",
"src": "Patient: I have cracking in chest and shoulders with minor pain. Been going on for 2 years. If i sit for a while I need to stretch again and cracking comes back. Sometimes it's hard to breathe until I take an aleve. Also sounds wierd but xanax helps. I feel good when lying down but when I get up froma sleep it starts again. Any answers? Thanks Doctor: Thanks for your question on Healthcare Magic. I can understand your concern. Your discomfort is associated with specific movement and position. So possibility of musculoskeletal cause is more likely in your case. So avoid movements causing pain. Avoid bad postures in sleep. Avoid heavyweight lifting and strenuous exercise. Apply warm water pad on affected areas. You are also improved with Xanax. It is anxiolytic drug. So you are also having stress and anxiety. So better to consult psychiatrist and get done counselling sessions. Try to identify stressor in your life and start working on it's solution. Counselling plays very important role along with anxiolytic drugs (Xanax). Don't worry, you will be alright. Avoid stress and tension, be relax and calm. Hope I have solved your query. I will be happy to help you further. Wish you good health. Thanks."
},
{
"id": 21668,
"tgt": "What causes faster heart beat?",
"src": "Patient: hi im 33 now my pulse rate is 90 at present ECG shows normal BP 110/80 and sugar level in blood is 90 after fasting but sometime means i felt nervous my heart beat is going fast so is there any decease to me give me answer now? and as i have to go any medical tests for this? Doctor: Hi ThereYou are totally fine dear. Your Bp is normal. The only problem is that you are worried for your health so don't worry just enjoy your life.Anxiety cause faster heart beats. No you need not go for any tests.Good luck Dr Bhanuu Partap Cardiology"
},
{
"id": 154529,
"tgt": "Is thickening of blood a reaction to chemotherapy?",
"src": "Patient: My 69 year old brother just completed his 1st round (5 days) of chemotherapy for aggressive Large B cell lymphoma. During the chemo he was on large doses of steroids. He is also a double kidney transplant and is on immunosuppressants. He hasn t had any reaction to the chemo at this point but tonight he thought he was going to have a bowel movement but instead started passing blood clots..quite a bit. He had a clean colonoscopy 10 days ago. One of the drugs given was the blood thinner Lovenox (sp?) is this a reaction to the chemo? Doctor: Hi,Thanks for writing in.If your brother has passed clots in stool then please talk to his doctor at the earliest. This might indicate that his thinned down and leaking some place in the intestine. This is an emergency. You have understood the incident slightly wrongly. What has happened is his blood is thin due to medications. This will prevent blood from clotting usually. He has started bleeding at some point in his bowel. This is causing bleeding to occur. As the blood is thin therefore it is not able to clot fast resulting in leaking and blood loss.Due to the above the blood that has leaked in to the colon and rectum is being seen as clots in the stools. Please get his doctor to attend him."
},
{
"id": 123021,
"tgt": "What are the side-effects of the medicine, Myoril? Is this a steroid?",
"src": "Patient: ortho doctor prescribed myoril 4 for the complain of back pain and neck pain and scalf muscle tightness. please let me know whether it is a steriod if not what are its sie effects, since I am allergic to most of the drugs and alredy suffering from serious gastritious. Doctor: Hello, Myoril is not a steroid. This is an oral muscle relaxant. The allergy to any chemical substance cannot be detected without a test. It can aggravate gastritis. Hope I have answered your query. Let me know if I can assist you further. Regards, Dr. Praveen Tayal, Orthopaedic Surgeon"
},
{
"id": 166619,
"tgt": "Suggest remedy for social anxiety in a child",
"src": "Patient: My son age 9 became afraid of gong any where over this past year. he and my 2 other children are homeschooled and we meet weekly with a grop of About 30 other homeschoolers. He doesn t want to go any more to that or his music lesson or even venture outside in the yard. What do you suggest we do. Doctor: dear parent, the most common causes for such pain are 1) splenic flwxure syndrome which is a colonic irritation due to inflammation . in this case the pain is increased when sitting down2) enlarged spleen or splenic disorders . in this case a sharp pain that is tender to touch with hardening of this area of the abdomen"
},
{
"id": 141631,
"tgt": "What causes weakness and radiating pain in the arm?",
"src": "Patient: Hello, I have had 3 episodes where both my arm get pain starting at the top of my arms (on the inside) travels down my entire arm, and then they are very weak. I have to stop what I am doing and rest my arms until they pain goes away and the strength returns to my arms Doctor: Hello and Welcome to \u2018Ask A Doctor\u2019 service. I have reviewed your query and here is my advice. You must be having compression of nerve at your neck(cervical ) level.You need MRI Cervical spine to look for any compression.compression may be either due to prolapsed disc or soft tissue hypertrophy or any osteophyte or bony spur.On the basis of MRI findings you may be treated conservatively or require operation. Hope I have answered your query. Let me know if I can assist you further."
},
{
"id": 189325,
"tgt": "Had dental surgery. Took hydrocodeine. Concerned?",
"src": "Patient: My doctor prescribed me 10 mg. hydrocodeine after having dental surgery. After taking one of them, it didn't help my pain at all and I looked at the number on the pill and it had M357 on it. Is this a 10 mg or not. My pharmacy is a training pharmacy for Texas Tech and I'm wondering if one of the students might have given me the wrong medication, by accident. I am sorry to take your time, but I didn't know I would be charged for this answer. Thank You, Nan Carrasco Doctor: Hi, Thanks for asking the query, Hydrocodeine is a narcotic analgesic used to treat severe pain and has antitussive action to treat cough. If it is not working you can visit the Dentist and get the prescription changed. For relieving pain after surgery it will be better if you take anslgesics such as acecelofenac serratiopeptidase combination which will be more effective. Follow the post surgical instructions properly. Use lukewarm saline and antiseptic mouthwash gargles twice daily. Hope this helps out. Regards..."
},
{
"id": 145573,
"tgt": "What causes twitching of index finger?",
"src": "Patient: Hi, lately my Index finger left hand twitches for a few seconds every 10 mins.. feels weird., & lately had a tooth extraction. the needle into the back of my gum made me gasp for air , & shake for a minute or two. Not sure if that hit a nerve but my finger goes moves from left to right . Can you explain perhaps? Doctor: Hi,Thanks for writing in.The index finger is supplied by a nerve called the radial nerve. It is made of roots arising from the neck. These roots travel along the arm and become a nerve which ends in supplying the small introssseous, flexor, externsor and lumbrical muscles of the hand.The index finger also has introssseous, flexor, externsor and lumbrical muscles and twitching is brought about by the irritation of any one or more of the muscles in the area.It will help to go to a neurologist and get examined completely. This will help in knowing if the radial nerve is getting pinched at an particular location and that causes twitching. Most likely it is a minor problem. Please do not worry."
},
{
"id": 207808,
"tgt": "What causes severe panic attacks with difficulty in breathing?",
"src": "Patient: Hi I had a sever panic attack a couple hours back, and although I have had a lot of stress due to a wide variety of factors -- I cannot negate that I have had problems breathing and over the past two weeks I have been unable to shake off cold virus as I still have problems with my nose being open to let me breathe appropriately. Doctor: DearWe understand your concernsI went through your details. I suggest you not to worry much. Severe panic attacks and breathing difficulty are possibly due to the stress you are mentioning. May be you are trying to rationalize your problem with the current cold virus conditions. Treat both of them different and if the cold is persistent, please consult a physician for further management. Regarding panic attacks, you should take holiday for a couple of weeks and enjoy or take rest.If you require more of my help in this aspect, Please post a direct question to me in this website. Make sure that you include every minute details possible. I shall prescribe the needed psychotherapy techniques which should help you cure your condition further.Hope this answers your query. Available for further clarifications.Good luck."
},
{
"id": 194847,
"tgt": "How can premature ejaculation in less than a minute during foreplay be treated?",
"src": "Patient: Hello sir i am 29 years old currently in bangalore , i am in love with a girl(whom i am about to marry in couple of months)we had sex 3- 5 times and i couldn t last for more than 1 minute, she is not satisfied that i can see that even she says she is ok, i have read many articles of premature ejaculation symptoms are the same. I am about to marry her but i am in a feeling where i may not make her happy. Is there a way where i can get out of this. And i am very shy kind of person to discuss this with others or donno how to approach anyone with this. Please suggest. Doctor: Hello,Anxiety is the main problem for premature ejaculation. Also, check for sugar levels and testosterone levels. Avoid alcohol and smoking. Eat green leafy vegetables helpful. After erection intermittent intercourse with squeezing, the technique may useful. Need counseling by sex therapist and psychiatrist helpful. Drugs like SSRI is useful in premature ejaculation. Please consult your sex therapist for further evaluation and treatment.Hope I have answered your query. Let me know if I can assist you further.Regards, Dr. Penchila Prasad Kandikattu"
},
{
"id": 6961,
"tgt": "My semen report showed the presence of gram positive cocci and aerobic spore bacilli. Is this normal or a reason for non pregnancy ?",
"src": "Patient: hi my semen report mixture of gram organism grown on gram stain showed the presence of gram positive COCCI , MICROCOCCUS AND AEROBIC SPORE BACILLI, SUGGESTIVE OF SKIN CONTAMINANTS is this report ok or a reason for non pregnancy Doctor: hi welcome to HCM.you have not send complete semen report.it shows mostly infectious cause thats it.its not the cause for non pregnancy."
},
{
"id": 211739,
"tgt": "Have mental health issues. Had nervous breakdown, diagnosed with mental illness. Did psychotherapy. Transiting to professional life. Treatment?",
"src": "Patient: Well I have mental health issues and I feel bad and cheated. I worked as a teacher in 1997 thru 2000 and I really did not want to work in this profession. I ended up having a nervous breakdown and was diagnosed with a severe mental illness. Today I would like to back to work but there seems to be an absence of seriousness concerning my case. have seen more mysticism than reality and I just want someone to talk to that is reputable and equipped to deal with serious mental health questions. I have spent several years in psychotherapy and it seems that there is always some question about my personal relationships, that gets in the way of my being able to focus on the issue of going back to work. I worry tremendously because my social life has been halted as a result of going into therapy while transitioning into my professional life. I don't know what to do. Doctor: Hi and thanks for the query,I think you need to sit and reconsider a few things of utmost importance regarding your health. I still think hope is not lost and many things could be done.You rebuild self confidence and believe in yourself. You must learn to enjoy the job you are doing. Working in an environment with supportive people could be useful. Being close to your family could be of much help. I think you might need to consider the basics of living and live simple. Exercise and living an environment with your loved ones and family could help. Be positive towards life. Thanks and hope this helps. Kind regards.Bain LE,MD"
},
{
"id": 132045,
"tgt": "Suggest cause for joint and ovary pain with cramps",
"src": "Patient: I have had horrible SI joint pain that travels up my lumbar spine and down my tailbone at times for 3 weeks. It has progressively gotten worse. Now I am also experiencing ovary pain and menstrual cramping the past couple days. What could be causing this? Doctor: Hi, I had gone through your question and understand your concerns. its a symptom of getting spondylitis, but do not worry about it.i suggest you for ultrasonic therapy to be taken for quick relief from pain.do some back strengthening excercises twice a day for 10 days.more important is to maintain correct posture while sitting and standing, avoid lifting heavy weight.you will be back to normal in few days."
},
{
"id": 22874,
"tgt": "How serious is to have a hole in heart?",
"src": "Patient: Hi. I just found out that i have a hole in my heart. My cardiologist was very vague and did not tell me where it was and how bad it may be. Im 21 and have never had heart problems before but both my mother and older sister have heart problems. Should i be concerned? Doctor: Hiyes you should be concerned about the hole in the heart, it can be a atrial septal defect or ventricular septal defect or padent ductus arteriosus. a 2d echo should be done to rule out the size , location and nature of the hole, different types of defects are treated differently.A 2d echo should be done asap for further plan of action."
},
{
"id": 29161,
"tgt": "Suggest remedy for swelling of the leg during bacterial infections",
"src": "Patient: Hello Dr.I have query regarding bacterial infection.In my family, one member is affected by bacterial infections in leg.(with swallon below infected area)We have shown it to Dermatologist here 3 days back. Dr has prescribed.below medicines.Mupirocin ointment, Linezolid, multivitamin n multiminerals.The bacterial infection is curing but swallon is increased very much. It is near ankle of leg.Please suggest how to cure this swallon.Thanks. Doctor: dear user...to suggest remedy we must confirm diagnosis first....from your story we have to possibilites ..first when the infection occured caused lymphatic swelling as infection enhance blood and chemical factors that lead to that swelling ..in this case you can use just ice packs on it and paracetamol and it will be okay.....second purposal is that there is what is called cellulitis and in this case he will feel pain and tenderness when touching it ..cellulitis is caused by bacterial infection and he should take antibiotis such as amoxicillin we prefered injections to reach to inflammed tissue as oral antibiotics may not be effective ......simply no danger of that......hope that i helped you ..wish you healthy life..thanks"
},
{
"id": 191349,
"tgt": "How can diabetes be managed?",
"src": "Patient: 5 months ago diagnosed with diabetes. Metformin caused bad GI upset. Glimipride working well. Having really bad muscle cramps/spasms-especially in forearms. Very dry mouth. Recently new symptoms of very foul gas/bloating, bulky stools. could be malabsorption syndrome? (Dr. Google) Doctor: bulky foul smelling stools need not be malabsorption syndrome always, it can be simple infective diarrhoea.A course of Metronidazole would help the cause.As far Diabetes is concerned keep check on your HbA1C"
},
{
"id": 116352,
"tgt": "Suggest treatment to increase gamma level and reduce albumin and cholestrol",
"src": "Patient: I have been donating plasma but recently my gamma levels were too low and my albumin is too high. I don't know what to do to correct this. I've also been told that my cholesterol is out of whack and should be taking Lipitor. Could it all be related? I need to get the protein levels back in line. Doctor: Hi, dearI have gone through your question. I can understand your concern.Your albumin level may be high but no need to worry about that.You should control your cholesterol level. You should take low fat diet with high amount of polyunsaturated fatty acids. Also take high fiber diet.You can start drugs like atorvastatin or pravastatins. But it is priscription based medicine so consult your doctor and take treatment accordingly.Hope I have answered your question, if you have any doubts then contact me at bit.ly/Drsanghvihardik, I will be happy to answer you.Thanks for using health care magic.Wish you a very good health."
},
{
"id": 65315,
"tgt": "Suggest remedy for lump on scalp",
"src": "Patient: Hi, i have been constantly wearing hair weaves for the past few years but since my recent stop, my hair have been growing again with the treatments that i use (organic products). I have noticed for the past few months not a red grown flesh (a lump or bump) that is on my scalp and feels hard to the touch. This area does not hurt but worries me. Could this be as a result of the constant pulling of the hair through extensions? Would it grow bigger? Your advice is greatly appreciated. Doctor: Hi, dear I have gone through your question. I can understand your concern. You may have some sebaceous cyst or dermoid cyst or some skin adenexal tumor. It can increased in size. You should go for biopsy of that lump. It will give you exact diagnosis. Then you should take treatment accordingly. Hope I have answered your question, if you have doubt then I will be happy to answer. Thanks for using health care magic. Wish you a very good health."
},
{
"id": 207136,
"tgt": "How to get rid of fear of death?",
"src": "Patient: Respected Doctor, First I may appreciate for the online help doing by you. My father is age more than 65year & now a days he is staying in fear of death & crying like baby. Please help me to suggest how could I bring out from this problem & motivate towards prosperity.. Regards, Ranjit Kumar Sabat +0000 YYYY@YYYY Doctor: Dear Ranjit, old age deprresion is frequently in people above 65years of age and perticularly when the people feel dissatisfied with their past life. You start him with los dose of escitalopram. Please take care of other medical problems, if he has any."
},
{
"id": 67652,
"tgt": "Can lumps in ear suggest a lymph node?",
"src": "Patient: I have a kidney bean size hard immovable lump behind my left ear. Its not painful but I have had it for about 5 months and within the last year lost 46 pounds without changing anything and have been having horrible joint and lower back pain. I also have been extremely tired. What could the lump be? Just a lymph node? Doctor: Hello...Its a Good thing observed by you....It seems to be lymph node from your mentioned history... but we have to concern about weight loss and joint pain...It may be due to some hidden malignency...Kindly consult ur family doctor and get investigated for Lymph node first... in form of local sonography or needle biopsy nd then after further investigation or treatment....."
},
{
"id": 126149,
"tgt": "What can cause swelling in the ankle and hands while on Aspirin and diabetes medication?",
"src": "Patient: I am experiencing swelling in the ankle and hands. 57 year old male, 5\u201910, 315 lbs. Meds taken daily low dose aspirin, diabetes pill, statins, cocq, vitamin daily. Kind of tired, back of heels sharp pain. Never have had any major life threatening events Doctor: Hi, It is not related to aspirin or diabetes. Consult a physician and get evaluated. A Doppler scan will be helpful to rule out conditions like DVT which can present like this. Hope I have answered your query. Let me know if I can assist you further."
},
{
"id": 194741,
"tgt": "Suggest tests to confirm impotence",
"src": "Patient: Hello, i m ankit ,24 i had undergone an operation in my penies when i was 2 months old. since then the size of my penies is very small and not very big at erection.my parents want me to marry but i m very nervous.can i hav sex and hav a child.and can i satisfy my wife and are there any chances of being impotent.please help.....thank you Doctor: Hello, You are worrying too much and that too unnecessarily. Length and size of penis is not a concern in marriage and child birth. A per son with even a two inch erect penis can satisfy women and can have babies. So don't worry about it. You are just 24. Focus on your career and education. Rest can be taken care of when the situation arises. Hope I have answered your query. Let me know if I can assist you further. Take care Regards, Dr K. V. Anand, Psychologist"
},
{
"id": 202166,
"tgt": "How do i get to know about hitting puberty?",
"src": "Patient: Hello I am a 13 year old boy who wants to know if i will hit puberty i have a 1.7 inch penis when soft and about a 2.8 inch penis when hard i do masturbate and when i do clear fluid comes out i have pubic hair, tiny black hair on my armpits, the same for on my lip Doctor: Hello welcome to Healthcare Magic.Do not worry all the above things appears normal for your age.Its little too early to say anything.Regarding size, one can do nothing other than wait.do not worry - take care.Need to ask ? Do not hesitate. Thank you."
},
{
"id": 46594,
"tgt": "Does overdosage of Allopurinol have adverse effect on health?",
"src": "Patient: Hi, My husband has been prescribed allopurinol for gout and I have just discovered that he has been taking double the prescribed dosage, probably for a few months now. He is also taking Coversyl for blood pressure and Lengout for gout. Are there any health risks associated with him taking double the required dosage of allopurinol? Doctor: patient is taking colchicine(lengout). so it must be acute gout. in acute gout allopurinol should not be used.rather it can precipitate acute gout .double dose will increase the risk further."
},
{
"id": 114593,
"tgt": "Is platelet count of 60000 a cause for concern?",
"src": "Patient: Hi Doctor ,I am 41 years working professional ,I had a mentrual period and Soon after that I had gone for blood test my platelet count is 60000. Is any thing serious about it and what precautions and food should I take to make up my platelet count. Neena . Doctor: Hi i do care for your concern. The platelet count that you have mentioned is well below the normal range. Since you have mentioned that you have taken after your periods, its wise to take a fresh blood testing for platelet count. If found to be abnormal then consult your physician as you need more evaluation.Hope i have answered your question, if you have more feel free to ask. Thank you."
},
{
"id": 12326,
"tgt": "What can cause guttate psoriasis?",
"src": "Patient: Hi, may I answer your health queries right now ? Please type your query here... Hello, My son is 15 and has guttate psoriasis. With treatment how long does it normally take for it to go in remission? What is the precentage of kids that it doesn't? What else can I do to try to help him? Thanks! Doctor: Hello. Thanks for writing to us at healthcaremagicGuttate psoriasis is a skin condition in which small, red, and scaly teardrop-shaped spots appear on the arms, legs, and middle of the body. This type of clinical presentation is common in children.Guttate psoriasis is said to be usually preceded by a streptococcal sore throat infection.Guttate psoriasis is usually widespread. Therefore treatment options for guttate psoriasis are whole body UV B Phototherapy, oral antibiotics like Erythromycin.Sunlight is believed to play a beneficial role in guttate psoriasis. Topical treatment includes topical emollients, topical steroids, topical vitamin D anlogues like calcipotriol, calcitriol.The patches respond quite well to treatment and the response is usually evident within 2 weeks.Regards"
},
{
"id": 124791,
"tgt": "What could cause hip pain?",
"src": "Patient: hi doc, I m 42 yrs old and found higher sgpt -120, and sgot-117. My hip is pain and I m very often to urine especially during nightimes. I m suspecting my gallbladder is not strong enough. I m actually never smoke and take any alcohol. Please advice and thank you in advance. Rgds, Marcelo Doctor: Hello, It is not due to gallbladder related problem. You can rule out gallstones or other related problems by an ultrasound. As a first line management you can take analgesics like paracetamol or aceclofenac for pain relief. If symptoms persists better to consult an orthopedician and get evaluated. Hope I have answered your query. Let me know if I can assist you further. Regards, Dr. Shinas Hussain, General & Family Physician"
},
{
"id": 150672,
"tgt": "Have bipolar disorder, loss of appetite, anger, restlessness, confused state. On valparin. Help?",
"src": "Patient: My father is 80 years old and he is a bipolar disorder patient. He has been down for almost 1.5 years. Now all of a sudden he has become hyper active since from last few days. He doesn\u2019t sleep at all, very confusing mind, very restless, always talking (repeating the same thing), very angry. Interferes in everybodys business. We are all finding it very difficult to tolerate him. He had been prescribed following tablets by his health care provider. 1) Valparin \u2013 500 mg (0-0-1) 2) Oleanz \u2013 7.5 mg (0-0-1) 3) Nexito \u2013 5 mg (1-0-0) He took the above tables for some time and overnight stopped taking it (which he does frequently) saying that these tablets are causing him side effects. He frequently says when he is down his lever has stopped functioning; he doesn\u2019t have appetite and all the problems in the world. We have got him tested for everything and all the reports are normal including BP . Can you please suggest, since he is not taking any tablets, can we give the above tables by mixing the tablets in food. Doctor: Hi friend, Welcome to Health Care Magic If the tablets offer relief and if he is refusing to take them, the only way is to give them mixed with food or fluids. A neurological assessment is necessary - to exclude frontal lobe lesions at this age, which can mimic psychiatric disorder. And his psychiatrist should review him... Take care Wishing speedy recovery God bless Good luck"
},
{
"id": 76875,
"tgt": "Suggest treatment for preputium clitoridis lipoma",
"src": "Patient: Hello. I have a tiny (3mm) preputium clitoridis lipoma. My brother was recently diagnosed with a very high grade paratesticular liposarcoma with metastatic disease to the lungs and numerous other local lesions. Should I have mine removed given family history and suspected Li-Fraumeni syndrome per family cancer risk assessment? I am 42 years old, 5'6\", 150lb and in good health. Doctor: Hi thanks for contacting HCM...Here your lipoma size very small and it should be excised....Here chance of malignancy is less.....As you have positive family history you can do genetic screening.....P53 gene is tumour suppressor gene...Its mutation called li fraumanin syndrome .For that PCR or FISH testing can be done...If that gene mutation present then chances of multiple tumour can be there and regular follow up needed.....Take care....Dr.Parth"
},
{
"id": 26204,
"tgt": "Will high blood pressure cause constant yawning?",
"src": "Patient: my daughter is 17 years old. she weighs 124 pounds and is 5 foot 2. she has high blood pressure with no known cause. she has a slight thickening of the muscl between the two chambers of her heart. lately she cannot stop yawning constantly. this has been going on for about a week now.I was wondering if this could be related to her blood pressure or her heart. Doctor: Hello!Thank you for asking on HCM!Regarding your concern, I would explain that persistent yawning may indicate a low perfusion of the brain. This could be related to different disorders : -chronic anemia-a low heart function-a metabolic dysfucntion-sleep apnea and obstruction of the airways. I recommend consulting with the GP for a careful physical examination and some tests : - resting ECG and cardiac ultrasound to examine her heart structure and fucntion- blood lab tests : complete blood count, PCR, sedimentation rate, blood gas analysis, kidney and liver function tests and blood electrolytes). A polysomnography may be needed to rule in/out possible sleep apnea. Hope to have been helpful!Greetings!Dr. Iliri"
},
{
"id": 180518,
"tgt": "Can an abscess on the gum cause fever and joint pain?",
"src": "Patient: I had a surgical abortion Tuesday and now I am feeling ill. I have what feels like the flu, suffering with muscle and joint pains, fever and feeling extremely fragile. I was giving a dose of antibiotics which I had to take after the procedure and I did to stop infection. Also I have what seems like a tiny boil come up on my gum with puss in it, however there is no pain at all. So I'm wondering if I should be worried about being so ill after my abortion or could it potentially be an abscess on my gum cause I know that can make you unwell, but I always thought an abscess is painful. Thanks Emma Doctor: Hello and Welcome to \u2018Ask A Doctor\u2019 service. I have reviewed your query and here is my advice. It looks like you have infection in your body that is the cause of the symptoms that you are facing that can be probably a viral infection as you have flu like symptoms.The boil over the gum does not seems to be the culprit, as your explanation is not looking like a severe gum infection causing abscess or a boil formation..So my suggestion is to consult your doctor or an Emergency room and get yourself checked and a clinical examination and if needed an x ray can be done to confirm the diagnosis and treatment can be done accordingly..For now you can take Acetaminophen to reduce fever and aches.For the gum boil you can do warm saline gargles, antiseptic mouthwash gargles and apply Chlorohexidine gel over it.If it does not improve or continue to worsen consult a Dentist and get checked.If it looks like a tooth abscess an x ray can be done for confirmation..Hope I have answered your query. Let me know if I can assist you further. Regards, Dr. Honey Arora"
},
{
"id": 139071,
"tgt": "What causes soreness and pain in the legs?",
"src": "Patient: I bought a product in Australia that is to relieve tired, sore, heavy and aching legs. It is called Lioton gel 50 g --when I looked it up online, all I came up with was a gel with heparins in it. This product claims to have aescin, vitamin B4, bromelain, menthol, and boswellia (frankincense). Are these related or similar to heparins? (The product is made by Menarini of Italy) Thank you for any info you have! Doctor: Here in heading your question is what causes pain. And soreness in leg and in explanation of your question you are asking about Lipton gel.Let me tell you first question on causes of pain. It can be various resion to have pain in leg like over use of soft tissue ( with extra walking or jogging ) pain in leg due to soreness in soft tissue, there might be pain due to varicose vein, it can also be due to diabetic neuropathy, it can also be due to atherosclerosis of deep blood vessels of leg. Now there is only one way to rule out the specific cause is visit your doctor for proper examination and go ahead with required tests and then plan for treatment as per report.Your next question was on Lipton gel. It is anti inflammatory gel which helps to reduce inflammation in soft tissues and even in blood vessels so the person will have relaxing effect and the pain becomes less. Yes it also contain heparin in it.Take care."
},
{
"id": 17050,
"tgt": "What causes frequent heart palpitations with rapid heart beats?",
"src": "Patient: ive been having heart palpitations quite frequently recently, ive been losing breath and feeling really dizzy and shakey. my heart beat seems to be fast most of the time and im really worried about it, i do smoke and i also smoke weed alot, and drink a lot, im only 18 but is it possible for something to be really bad? theres also heart problems in my family Doctor: Hi, You are having an arrhythmia. You should avoid all stimulants as weeds, smoking, alcohol, tea, coffee, herbal products. These things are not good in 18 years old. As you are also in high-risk family history, please be careful, also undergo ECG at the least. Please do not stress your body, take plenty of sleep and well-nourished diet. Hope I have answered your query. Let me know if I can assist you further. Regards, Dr. Varinder Joshi, General & Family Physician"
},
{
"id": 147364,
"tgt": "What causes feeling pressure in my head and hearing sounds in ear?",
"src": "Patient: I have a strange feeling in my head, almost as though something is hitting my forehead from the inside. When this happens my ears pop and my ears ring. It hurts a little but is more scary than anything else. It also makes me slightly dizzy. What could that be? Doctor: HIThank for asking to HCMI really appreciate your concern your headache must be due to the stressful condition and the emotional stress is the most likely cause and another is anxiety and depression such headache is tend to induce the hallucination, you need to keep the stress level very low for the symptomatic relief you can have, \"Tab Acetaminophen three times in day, and Tab Chlordiazepoxide three times in day, hope this information helps you, have nice day."
},
{
"id": 21843,
"tgt": "Is heart failure related with shortness of breath,heart palpitations and mitral regurgitation?",
"src": "Patient: My doctor diagnosed me with High BP and prescribed Toprol XL (100 mg) one a day, Diovan HCT (16-12.5 mg) one a day, Lasix (40 mg) twice a day, KLOR-CON M (20 tab) one a day. I am also a RA patient. My symptoms are lightheadedness, shortness of breath, heart palipitations, and swelling of the lower extremities (legs and feet). My doctor says I have a small amount mitral regurgitation. Is diagnosis and my symptoms, would be considered heart failure? Doctor: Hi, As per your doctor, you've mild MR which can cause occasional palpitations and mild shortness of breath.however, it doesn't point to heart failure. .it may be an early sign but get an echo done ane if echo is showing normal functions then doesn't seem so. if you're having leg swellings also,then I suggest you consult a cardiologist aa you may need aggressive treatment. and it may early sign if heart failure"
},
{
"id": 179013,
"tgt": "What causes recurrent defecation with blood stains in stool of a child?",
"src": "Patient: My 8 year old son was suffering from recurrent deffaecation with blood stains stool. on 19th july 2014 he had stool examination which reveals of occultblood, mucus present ,pus cells 6 -8,rbc 12 -15 .At the same time sonography was normal .Doctor diagnosed prostitis and suggested tablets cefixim, Nitaxonit,penta prazole with domperidone ,reflora kids .my son has another stool examination on 29 july 2014 reveals normal results . But after a week or two again he is facing the same problem blood stains stool with recurrent deffaecation.what should we do ?n suggest Doctor: You should consider going in for a stool culture examination to see which bugs are causing the problem and to which medicines (antibiotics) are they likely to respond. Only after this you may decide about which drugs to give. On the other hand, if a culture cannot be done or is inconvenient, costly or not available, you can repeat a six day course of Cefixime only and see what the results are.Dr. Taher"
},
{
"id": 158549,
"tgt": "Feel warm/hot on the face and back of the neck, stress. Have thyroid cancer. On Synthroid. Cause and cure?",
"src": "Patient: For the last few months I have had this feeling of being hot in the back of my neck and feeling warm in my face as well. I am a 37 year old female with a history of thyroid cancer. I have also been having severe anxiety. In the last few years I have been under overwhelming stress. I have also been running a low-grade fever (99) for several months as well. When I have the feeling it is nearly constant and will last for days at a time. I have an endocrinologist appointment next week. I still have half of my thyroid and am on 75mcg of Synthroid a day. Doctor: Hi, Thyroid cancer can be of different types. You should have mentioned the exact subtype. After thyroid surgery there are possibility of radioiodine ablation . You should have mentioned the detailed treatment history. Anyway, thyroid hormone replacement has to be done and this measure has been taken to you. But the exact dosing of thyroxin should be maintained based on the TSH level. Your symptoms may be due to this. Consult an endocrinologist for proper dosing."
},
{
"id": 98807,
"tgt": "Why does breath smell metallic?",
"src": "Patient: my daughter who is nearly 17 has breath that smells like metal, we went to the doctors about 2 months ago because she was having bloating feeling and mentioned the breath then, he gave her some tablets for her stomach and said it should help the breath aswell, but the metal smell is still there, She has had it for a long time now?? Doctor: Welcome to HCMI have read and understood your query and appreciate your health concern for your daughterIst of all you need to check few thingsis your daughter having oral ulceration.Dies she Have gum bleeds? or there is recurrent episodes of cold flu or sinusitis?Bad breath can arise more commonly due to tongue,nose and gum problems beside other reasons.Your daughter can do few things to overcome this problemAsk her to brush her teeth after every meal,use garlics and onions less in meals,gargle with mouth wash for atleast 30 seconds.Floss the teeth.drink as much as fluids as possible bcz dehydration can also cause this.Keep mint with her abd chew gums.Meet a otorhyinologist if the problem persists I hope this was useful.Regards Dr Faeza"
},
{
"id": 198927,
"tgt": "What does\u00a0this\u00a0semen analysis test result mean?",
"src": "Patient: Please help me understand the below analysis Color: White Viscosity: Normal Liquification Time : 30 mins PH : 8 Volume : 3.5 ml WBC: 4-8 /hpf RBC: 2-3 /hpf Bacteria: Nil Epithelial Cells: Nil Total Sperm Count: 102,750,000 million/ml Normal Sperm Cell: 85% Abnormal Sperm Cell: 15% Double Head: 0% Tapered Head: 0% Rounded Head: 0% Coiled Midpiece: 5% Pin Head: 5% Without Tail: 0% Giant Head: 5% Double Tail: 0% Motility After 30 mins : 70% active, 10% Sluggish, 20% non mobile After 1 hour: 60% active, 10% Sluggish, 30% non mobile After 2 hours: 40% active, 10% sluggish, 50% non mobile Please advise how to treat the wbc and rbc issue Doctor: HelloYour semen analysis findings may indicate semen infection.Infection is indicated due to presence of excess pus cells and RBC in semen.Normally semen shouldn't contain any pus cells.You may need investigations like routine hemogram,random blood sugar,semen culture and sensitivity,ultrasound of scrotum.You may need to take antibiotics based on culture and sensitivity report.Motility is normal.Normally it should be at least 55 %.Sperm count is also normal.Normally it should be at least 20 millions/ml.Fertility is expected to increase with control of infection.Other findings like pH,liquefaction time etc are normal.You should take healthy and nutritious diet.Avoid alcohol and cigarettes if you take it.Get well soon.Take CareDr.Indu Bhushan"
},
{
"id": 153303,
"tgt": "Suggest remedy for painful hard stool movement during prostate cancer",
"src": "Patient: my dad has been diognosed with prostate cancer and is in the middle of treatment the problem is when he goes to the loo he says it hurts and only a small bit comes out the doctor mentioned cranberry juice but that doesnt seem to be workin is there anything u suggest to relieve the pain? Doctor: we live in a time where prostate cancer patients are living much longer then a decade.He has to take high fiber diet and stoolsoftner like Lactulose 20 ml with warm water night if does not relieve can be taken more frequently like 3-4 times a day.Dr.Deepak"
},
{
"id": 212994,
"tgt": "Addicted to opium. Taking bupin anti-depression medication. Will super accelerated neuron regulation therapy help?",
"src": "Patient: Hi sir was heavy opium addict for 3 years nd I went to psychologist he suggested me bupin nd few anti depression medicines now I m addicted completetelly on bupin and other anti depression medicines for 3 years but I m not able to reduce medicines .infact I m now completely addicted on my medicines , as I was addicted on opium in previous years , now I want Super accelerated neuron regulation therepy cud I take this therepy in XXXXX how much it cost ? Please answer me Doctor: Dear sunnie Thanks for consulting us. Nothing to worry it is not too hard to quit those medicine. This medicines are not so bad like the opium you have taken in the past. You dont have to take it for long time also. thousands of person with these type of problem is getting cured everyday throughout the world. No need for Super accelerated neuron regulation. there are hundreds of alternatives. Just consult a good psychiatrist. Along with medicine a good counselling will also be helpful for you. If required go to some DEADDICTION SPECIALIST. Dont worry its just a matter of time. Best of luck."
},
{
"id": 30018,
"tgt": "Suggest treatment for valley fever",
"src": "Patient: diagnosed as reactive (1-16) to valley fever. office just called to tell me to reduce dosage to 200 mg per day. pharnacy just called to tell me that long time use of simbastatin may be issue and waiting for dr retrun call. just recovered from pnuemonia on left lobe. little confused and dr nurse could not tell me how severe VF is on me.no signs like rash or cough. help!! Doctor: Hello!I have been through your question.Valley Fever (Coccidioidomycosis) usually causes a primary pulmonary infection (as I think you have based on your history) which often resolves without therapy.\u00a0\u00a0In some cases a chronic\u00a0\u00a0pulmonary infection or, rarely, disseminated disease (extrapulmonary infection) may develop. Primary pulmonary cocci usually may not need treatment but it depends from risk factors , illness severity indicators, and findings on follow-up of patient\u2019s symptoms, exam, and serology every two to four weeks. Usually it treats with fluconazole 400 mg daily or itraconazole 200 mg twice daily until titer is stabilized at \u2264 1:4 and asymptomatic. The infection could be clear when the titer is usually 1:2, 1:4 or undetectable. Persistence of titer > 1:4 as you have (1-16) may indicate active infection includingdissemination, but to be sure I want to ask this titer result is after starting the treatment or in the beginning. I think is a good news that your symptoms are improved but you need to do a physical examination by an Infectious disease specialist,every two weeks titer, lab test and chest radiography as well to define better your situation. I hope my answer help you.I wish you a quick recovery."
},
{
"id": 109343,
"tgt": "Suggest treatment for back pain",
"src": "Patient: I am 16 female and weigh like 128 I had a baby 14 months ago. I have un protected sex with my husband. For the last three days I have been in pain my lower and middle back and when I lean back to try and pop it my belly button hurts like crazy and everytime I eat the pain gets worse Doctor: HIWell come to HCMBackache could be due to poor posture and this might have caused muscular spams, or spine disk problem, better to have some rest and following medicine can be tried1) Tab Diclofenac 50 mg sustain released once in dayhope this information helps, have a nice day. (Try to maintain good posture )"
},
{
"id": 147712,
"tgt": "Stiffness in neck with limited range of motion, blurred vision. Normal MRI, blood tests. What is the cause?",
"src": "Patient: hi My son 22 years old and been having issues with his neck for more than a month. He has stiff neck , limited range of motion , blurry vision . MRI and blood test shows nothing wrong. What it can be? Doctor: HIThank for asking to HCMI really appreciate your concern for your son, this could be muscular spasm and this would come around with the muscle relaxant, if I would be his doctor then I would treat him with the following treatment, 1) Tab Acetaminophen one tab three times in day2) Tab Chlorzoxazone one tab three times in day3) Tab Chlorodiazepoxide one tab three times in dayhope this information helps you have a good day."
},
{
"id": 107853,
"tgt": "What causes lower back pain with sensitive skin?",
"src": "Patient: Hiyaa i dunno if you can help me but could you give me any advice please. Iv had Pain in my left side for about two weeks now and since last week had it in my right side aswell. There nothing on the skin serfice as i can see, But the skin has gone abit sensetive when something touches it like clothes or just walking,in school or sleeping. Since last night the pain has spread to my lower back and find it hard to sleep as i cant lye down or sit on chairs without being in terrible pain. Which is very hard as im still a student and need to be conserned with learning for my GCSE exams That are here. But i cant sit still or think straight when im in pain all the time. Iv Tryed Taking Ibuprofen as a pain helper, It Seems To lessen the pain but comes straight back as soon as it wears out. The pain doesnt seem to be getting any better by time. Seems to be getting worse, and now its Effecting my learning and Keeps me up all hours ov the night. Do u have any Advice that could help me With the pain or know what could be wrong :/ By Nicol x Doctor: Hello,Hyperesthesia of skin is due to nerve irritation by spinal compression, you need mri lumbar spine. Meanwhile take pregabalin methylcobalamin."
},
{
"id": 147040,
"tgt": "Suggest treatment for grand mal seizure",
"src": "Patient: I have a terrible headache, my jaw hurts, and last week I had a grand mal seizure. My husband canceled the ambulance when I started to come out of it, the only thing I remember is him telling me as we were nearing the hospital that I had a seizure and he was taking me, so I freaked out and didn't want to go. My head right above my ears hurt so bad + migraine that starts at the base of the back of my head. I had one seizure as an infant, now I'm 32. I used to take warfarin but haven't for 3 years. I also had a full hysterectomy in April 2011 so I take Estradiol. Should I go to the emergency room because I didn't after the seizure and days later my head is killing me? Or should I wait? Also, I do not drink alcohol and haven't for nearly 5 years. Doctor: HIThank for asking to HCMI really appreciate your concern and I could say that the seizure that you had might not be the grand mal seizure because if this this was the grandma seizure then one need to be on medicine for life, but I think it is nothing like that so it must be hysteria and that mimic seizures, you have not mentioned for what you are on Warfarin, take care and have a nice day."
},
{
"id": 65343,
"tgt": "What causes a hard lump with odor on the neck?",
"src": "Patient: Hi, My husband has noticed a hard lump on the back of his neck (center),it doesn't move. It keeps getting bigger and is very painful. It hurts to move his neck. Today he noticed a strong odor on his finger after rubbing it. There's no odor if you sniff the bump, but if you rub it there is a awfull odor. He is 65, weighs 165 and is 5' 11\". I'd appreciate any advice. Thank you Linda Doctor: Hi, dearI have gone through your question. I can understand your concern. Hr may have dermoid cyst or sebaceous cyst. It cause lump with odour. You should go for fine needle aspiration cytology or biopsy. It will give you exact diagnosis. Then you should take treatment accordingly. Surgical excision is the treatment of choice. Consult your doctor and take treatment accordingly. Hope I have answered your question, if you have doubt then I will be happy to answer. Thanks for using health care magic. Wish you a very good health."
},
{
"id": 183113,
"tgt": "Suggest treatment for red blotches on roof of mouth",
"src": "Patient: I originally had strep A and was put on an antibiotic (amoxicillin). I started taking the antibiotic on friday and it is now thursday and i have gotten sick again and have a temperature of almost 100 degrees and I have red blotches on the roof of my mouth. what does this mean? Doctor: Hello, Thank you for consulting with HCM.It can be possible that you might be having a viral infection that is why you are having fever also and ulcers on the roof of the mouth.If it is a viral infection it will heal on its own as it is a self limiting disease, you just have to apply a topical anesthetic ointment on the ulcer for relief.Hope it will help you."
},
{
"id": 208728,
"tgt": "What causes sudden rapid decline in mental function in an aged person?",
"src": "Patient: 98YO FEMALE, 1MO RAPID DECLINE IN MENTAL FUNCTION, SEEMS TO UNABLE TO EXPRESS THOUGHTS. MILD R EYE LATERAL DEVIATION, INTERMITTENT SLOW TREMOR, POOR RECENT AND REMOTE MEMORY. NO LATERALIZING SIGNS, ABLE TO CALL RELATIVES BY NAME AND EXPRESS NEEDS. HX OF LUMPECTOMY FOR BREAST CA 4 YRS AGO, PACER, A FIB, HTN MED MS & BENADRYL OCC FOR SLEEP. WAS MENTALLY SUPERNORMAL 2 MONTHS AGO, 2 COLLEGE DEGREES. Doctor: hiiiiiat the age of 98 almost most of the brain function shows more or less kind of dysfunctionsif cerebral atrophy become more prominent than it seems more sudden worsening in symptomsand she had a lot of medical history tooso at age of 98 it is expecteddue to medical illness with HTN there are possible chance of small lacunar infract in brain also but can be confirm by CT scanage related senile atrophy also produces same changesso if it is fast in progression u need to consult neurologistbut as per my opinion it can be possible to stop progression but we will not reverse back all damages that she had alreadyso supportive and only symptomatic management is possiblethank u hope for the best"
},
{
"id": 37511,
"tgt": "Suggest medication for an infection in the anus",
"src": "Patient: Hi I ve suffered for years with diarrhea, blood and mucus in stools, severe left abdominal pain, vomiting, fatigue, dizziness, and pick up every infection/virus going! I m currently under investigation for Crohn s disease, Ulcerative Colitis and bowel cancer (still undiagnosed after 15years) lately my anus has been very painful, itchy, burning, and now has lots of tiny yellow dots accompanied by red dots like pin pricks. What could it be? I m very worried and in alot of discomfort Doctor: HIThanks for posting your query to Healthcaremagic. It seems like you are having small boils over your Anal region . You require an Antibiotic therapy ( Either Tablets or local Cream) for the treatment of this problem . Also there is a possibility of having Anal fissure ( minor tear near Anus) which requires an Anaesthetic jel application . So I advise you to consult a Surgeon for this and get treated. Also ask for a prescription of Laxatives if your bowels are not soft. Hope you find this information useful.Any clarifications or Queries feel free to ask"
},
{
"id": 133489,
"tgt": "What causes pain in arch of my foot and knee?",
"src": "Patient: Hello for the past couple of years I have problems with the arch in my foot on and off and a about a year ago my right keen stated having pains and later on it when to my left keen .it s rare for them to both hurt at the same time but it Wil happened every now and then. Also when I wake up in the mornings my arch in my foot will hurt so much to the point that I have to set down and wait for the pain to go down. And the pains in my keens and feet aren t on and off any more .they have stated happening very day. For example right now I can t stretch out my right keen at all when I try to I get this awful feeling in my keen, it s like a bubble that s being crushed and explodes a sense of altering pain that that travels through my keen and other parts of my leg. Doctor: hi,thank you for providing the brief history of you.A thorough neuromuscular and musculoskeletal assessment is advised.As you have pain in the knees which keeps altering and also in the feet then a thorough clinical examination of the knee and feet is advised.Now something to understand here is when one side of the knee is having injury the brain itself acts as a protection and diverts 70% of weight to the other side of body and that is how the pain gets shifted from one side to the other.Also, the pain is having change of nature which may be due to neuropathic issues which needs a proper assessment. If the SLR Test is positive clinically than it might be related to the pinched nerve, if this test is negative then the issue is muscular.Also, if you have a bubbling sensation than understand of the vascular system is needed . If any skin color changes are present in the lower limbs a vascular hemodynamics is compromised which is leading to pain factor. If this is not present then have to come back to knee.usually when knee is Injured the clinical assessment is focussed to the knee only and may be an MRI to understand the soft tissue Injury as well. Performing a patella grinding test will help understand the muscle strength status. Any injury in the knee will lead to patella grinding test positive and the muscles will be weak as well.After a thorough clinical assessment, physical therapist will be able to assist you in strengthening the muscle groups and regain the mobility.Iny clinical experience after a thorough clinical assessment patient is advised to undergo physical therapy and majorty of them respond well to physical therapy.RegardsJay Indravadan Patel"
},
{
"id": 84716,
"tgt": "What purpose is mucaine gel used for?",
"src": "Patient: Mucaine Gel is use for which purpose my wife has pain in stomac and moving to back side please adbvise YYYY@YYYY is my ID this is indian medicine and available at home my wife has pain in stomach and moving back side is is advisable to give her this gel??\\ Doctor: Hi,Mucaine Gel is a combination of antacids (aluminium hydroxide and magnesium hydroxide) with a topical local anesthetic (oxethazaine). It is commonly used for the symptomatic relief of pain, heartburn, acidity related issues of the stomach such as gastritis or ulcer. It can be given to her for immediate relief but she should be examined by a doctor for further evaluation and to rule out other issues such as biliary colid (gall stones).Hope I have answered your query. Let me know if I can assist you further. Regards, Dr. Mohammed Taher Ali, General & Family Physician"
},
{
"id": 190992,
"tgt": "Swelling in gum from last 1 month",
"src": "Patient: sir i have swelling in my gums and it s too much swelled plz tell me.it s starts about 1 month before.this is like a tumour .... Doctor: hi welcome to health magic you need to be more specific regarding the swelling is localized or generalized , does it bleed, the region and is it increasing in size also specify the size and if possible u can email me a picture of the swelling at dradarshdesai@yahoo.co.uk and it is best you consult a local dentist ..wish u a early recovery take care dr adarsh desai ( Oral Maxillofacial surgeon & Implantologist)"
},
{
"id": 15378,
"tgt": "Hive like rash all over body, knots on legs. Having RA. Cause of rashes and knots? Anything serious?",
"src": "Patient: My son who is 44 years old has recently been diagnosed with RA. He has been to a dermatologist also for a hive-like rash all over his body along with \"knots\" on his legs, thighs, and back of his head. This doctor did a culture and was determined that is not an allergic reaction, but a part of the RA he has been diagnosed with. I am concerned about the knots and rash. It is currently under control with steroids. What else could this be and is this serious? Doctor: Hello,Thanks for the query.These knot like lesions are rheumatoid nodules.The lesions keep coming and going.In fact These are characteristic feature of RA.You dont need to worry much.These are not dangerous.Treatment is symptomatic.Dose of steroids is increased or decreased according to severity of disease.Please meet a dermatologist for exact diagnosis.Let me know if you have any other doubt.you can ask a direct question to me on this forum, following the below link.https://urldefense.com/v3/__http://www.healthcaremagic.com/doctors/dr-rahul-kumar/64818Wishing__;!!Mih3wA!SBzm6_kI6hCZ58EPH6N_05MFfiPbxWXT0a2TJCdFQObRWm5mV5ur7hUOMa8clQ$ you a good health.Thank you"
},
{
"id": 125515,
"tgt": "What is the cause of headaches and my left arm feeling numb?",
"src": "Patient: Early this year I went to the ER because of chest pain they diagnosed me with Costo and prescribed me with a generic anti-inflammatory well the pain has yet to go away and now I have frequent headaches on my left side along with my left arm feeling numb. Is my condition worse than what they are telling me? Doctor: Hello, If symptoms persist, bettet to consult a neurologist and plan for an MRI brain to rule out possible intra cranial (inside the brain) causes. As a first line management, you can take analgesics like Paracetamol or Aceclofenac for pain relief. Hope I have answered your query. Let me know if I can assist you further. Take care Regards, Dr Shinas Hussain, General & Family Physician"
},
{
"id": 198066,
"tgt": "Suggest treatment for scrotal pain",
"src": "Patient: Hi Doc i sometimes feel pain under my balls specially after urinating ,according to my lab result i dont have prostaliasis diseases and my ultra sound shows i dont have any infection neither any short coming disease and the pain is sharp kindly need your assistanceyours john Doctor: DearWe understand your concernsI went through your details. You don't have to worry as your lab test results and scan results are satisfactory. The pain could be some sort of muscular one. Don;t worry and act according to the advise of your doctor. Some analgesic should do the trick.If you still need my assistance in this regard, please use this link. http://goo.gl/aYW2pR. Please remember to describe the whole problem with full detail.Hope this answers your query. Please feel free to post follow up queries. Available for further clarifications.Good luck. Take care."
},
{
"id": 58251,
"tgt": "Have gallstone, billiary colic attacks, intense pain in right side, shoulder. Is there more risk with mobile versus non-mobile stone?",
"src": "Patient: Have 1.5cm gallstone. I had 4 billiary colic attacks during the last 2 years, the last one happened 1 day ago. They usually last for 15-30min, very intense pain in my right side and the shoulder. One ultrasound report said that the stone was mobile and the other report that it is non-mobile. What does it mean? Is there more risk with mobile versus non-mobile? Doctor: Hi,Thanks for posting your query.Stones are usually mobile. Mobility is one feature which helps to differentiate stone from a polyp.But sometimes, a stone may get impacted at the neck of bladder. This causes obstruction to the bile flow and results in pain.If you get recurrent attacks of pain, then gallbladder has to be removed.I hope that answers your question.Regards."
},
{
"id": 58557,
"tgt": "Itchy rash on scalp, back, face. Stones in gall bladder. Treatment?",
"src": "Patient: Hello. My mom has been having very very itchy rash on her scalp, on the sides of her back and some on her face for a while. She is really suffering. She has stones in her gallbladder and the doctors say that this skin problem might be because of those stones. Is it possible? She has lost many kilos, too even though she has a good appetite. Would you please help us? Doctor: Hi and thanks for the query,It should be very clear from a complete dermatological evaluation that there is not skin infection or inflammation. when this is excluded, it is reasonable to attribute her symptoms to gall bladder disease. an abdominal ultrasound is very important. A blood test to test for bladder inflammation, levels of bilirubine, both free and conjugated is valuable to assess the degree of bile accumulation, that could explain the itching symptom. I suggest you consult a gastroenterologist for a complete clinical review.Thanks and kind regards,Bain LE, MD"
},
{
"id": 219275,
"tgt": "What causes soreness and swelling under armpit during pregnancy?",
"src": "Patient: I am currently 28 weeks pregnant and I have soreness and swelling under my right armpit. My obgyn examined me about 2 months ago and said that she didn t feel anything abnormal and that this was normal during pregnancy. Should I still be worried or get a second opinion? Doctor: Hallow Dear,During pregnancy, there is generalized engorgement of the breasts as a whole. This is due to the hormonal changes in the body. Though the breast is seen on the front side of the chest wall, it has got a tail which extends up to the armpit. This is known as The Axillary Tail of the Breast. As the breasts get engorged, depending up on the degree of engorgement, this Axillary tail also gets engorged. So what you are feeling the soreness and swelling under the armpit is nothing but the part of breast engorgement involving the Axillary tail of the breasts. I do not feel you need any second opinion. In fact I have given you the second opinion with physiological and anatomical explanation. :)I hope this takes away all your worries."
},
{
"id": 147458,
"tgt": "Have MS, getting weaker, losing weight, high blood sugar. Could MS be damaging pancreas?",
"src": "Patient: I have MS with all the usual issues: fatigue , neuro pains, dizziness/ vertigo , muscle weakness , frequent urination , weak stream, etc. Recenty (past 4 months) foot pain has increased, in the last two months getting weaker, in the last month I have been feeling dehydrated, and last week - when I have been eating more than I should - I expected to have gained 3 or so pound but lost 7 unexpectedly (wieghed 200 when I expected 210.) Thought my MS may be masking diabetes so I got a freind to use his blood tester and test my blood sugars. After eating at noon and not eating anything else for five hours my blood tested at 202. I m guessing diabetes. So I have an appointment to see my GP today. Question: could my MS auto immune issue be damaging my pancreas? Or is this just another health issue unrealted to MS? Doctor: HIThank for asking to HCMI really appreciate your concern, if this is Multiple Sclerosis, then this may not damage the Pancreas, this is fluctuating type of pathological disease that mainly hurts the peripheral and motor nervous system and such condition reversed mostly living not permanent damages, your moral, and will power is the important factors, otherwise no need to worry about this, hope this information helps you have nice day."
},
{
"id": 154515,
"tgt": "Suggest treatment for prostate cancer",
"src": "Patient: My husband was diagnosed w prostate cancer from biopsy last week. He is 76 and vigorous\\very active..only take supplements..no Rx....had no symptoms. Psa went from 12 to 17 in 6 mo prior reason for biopsy. It is stage one w Gleason of 7. Now we are choosing between robotic removal or radiation pros and cons. Erectile dysfunction not as important as incontinence..how can we find/evaluate statistics? How important is choosing dr s..his surgeon is 45 and has done 100..as opposed to dr 200 miles away who has done 4500... Doctor: Hi,Thanks for writing in.Prostate cancer is an important diagnosis to be discussed in detail. As the prostate cancer is biopsy proven therefore a MRI scan prostate with whole body PET CT scan is required for proper staging of the condition. While MRI scan will tell the accurate local spread of disease, the PET CT scan will tell about distant spread of disease if any.Since it is Gleason 7 category lesion, complete removal might be possible at the moment. Doing a cancer surgery is more about skill and also the stage of disease. I guess robotic resection is a better option than radiation, radiation might be required subsequently depending on the response to initial treatment.If the surgeon has done the qualified training and has the ability of doing a robotic surgery then even a younger surgeon is reliable with the skills. Statistics on incontinence and erectile dysfunction might vary with technique and stage of disease. In early stages and with a skilled surgery, there is low incidence of side effects. Please do not worry."
},
{
"id": 99017,
"tgt": "What causes thin scratches on back during night?",
"src": "Patient: I ve had thin scratches appear occasionally on my back during the night. I m not scratching (can t reach there), clean sheets, my husband does not seem to have the same issue, they sting and sometimes bleed, often two to three of them in a row. wondering what this could be from. Thank you. Doctor: difficult to say without seeing directly but it might be tinea infection . chek your blood for diabetes mallitus . local antifungul ointment application might be helpful."
},
{
"id": 19256,
"tgt": "What causes fluttering in heart area with a feeling of skipping heartbeats?",
"src": "Patient: hi i would like to know what u think about this please. over the last few years i have had a few frightening problems happen then stopped for a year and i thought it had gone but then reoccurred again about 4 months ago let me explain - first started when i was working i went to pick something up and then my heart started to beat really fast and hard for about a minute also couldn\u2019t breath good during the time as started to panic but no chest pain. then if i seemed to cough it would go away this would happen again a few times along a 6 months period but it was always when bending over to pick something. about a year on from that time i had gave up smoking drinking more water trying to eat more healthy as i went hospital many times complaining of dizziness and couldn\u2019t breath good that was the reason I made chances to my life, they did heart monitor test and all seemed ok so the 1 year went on since all the changes and i thought everything was back to normal however about 4 months ago i was driving and bent slightly to grab my steering wheel to pull my self up to get out the car and bang it happen i was so scared and seriously thought i was going to die. i then coughed and tried to stay carm and it went back to normal after about 30 secs. huge fast beats pounding feeling dizzy slightly and i panic. NO PAIN during the times i am trying to figure my self what the hell is wrong i tried to cut out caffeine and increase water intake as i said not smoking or drinking and eating healthier food i was just so disappointed it came back after all this time. but i have noticed now and then i get like a flutter around my side heart area near my ribs and also my pulse seems to flatter on my neck PULSE BUT no pain thou and then last for a min goes back. also when i do get pain in my chest its not at these times but the pain does seem to hurt on pulse beat like heart beat then ok then back every beat to the right hand side but as i said this is on and off between the problems above not during. CAN YOU HELP I AM NOT TO CONCERNED AS ITS BEEN ANOTHER 3 MONTHS AND NOTHING APART FROM THE FLUTTERING WITH IS OK BUT I NOW AND THEM SEEM TO SKIP A BEAT AND THINGS BUT NOT OFTEN WITH THEN CAUSES ME TO COUGH ;-) THAT SEEMS TO BE MY ONLY TRICK BUT SO SCARED THAT IT MAY HAPPEN AGAIN I HONESTLY THOUGHT THAT WAS IT FOR ME I WAS HAVING FLASH BACKS DURING THE TIME OF MY LIFE. ;-( I AM 27 AND SLIM 10ST AND ATHLETIC I EXCERSISE AND DON\u2019T DRIN OR SMOKE AT ALL Doctor: Hello and welcome,Occasional and sudden fast heart beats, pounding, dizziness, chest pain that are short in duration are highly suggestive of paroxysmal supraventricular tachycardia (PSVT). The episode of PSVT usually goes away with vagal maneuvers (coughing, deep breathing, muscle tensing or putting your face in ice-cold water). PSVT is an abnormal heart rhythm (arrhythmia) that occurs when a short circuit rhythm develops in the upper chamber of the heart which results in a regular but rapid heartbeat (150-200/min) that starts and stops abruptly. Alcohol, caffeine, anxiety, exercise or sudden movements, bending over may trigger the episode of PSVT. PSVT is diagnosed with continuous heart monitoring (Holter). One time ECG may be normal because the abnormality happens during the episodes. You did great job by avoiding alcohol, caffeine, quitting smoking and eating healthy food. These are called life style changes which is initial step for the treatment of PSVT. Your episodes are very rare and short in duration therefor additional treatment is not indicated. Try to avoid situation that triggers the episodes (bending over) and use vagal maneuvers when the episode happens. You should seek medical attention if PSVT episode does not go away with vagal maneuvers or the episodes become longer up to few minutes. Also avoid over-the-counter decongestants or herbal medicines that may also trigger the episode.I hope the above information will be helpful for you. Please, feel free to ask me if there is anything else you need to know. Thank you,Malik Amonov MD"
},
{
"id": 83381,
"tgt": "Are drowsiness and sleepiness common side effects of proluton injection?",
"src": "Patient: Are drowsiness and sleepiness common side effects of proluton injection. Am 32 weeks preg.. Started getting severe contractions last week and was given progesterone inj. A week later ie today, I have been given proluton inj again. is there any harm to the baby? Doctor: Hi,Proluton injection contains hydroxyprogesterone and is used to prevent premature labor in pregnant women in order to reduce the risk of giving birth too early. It prevents miscarriage and lower the risk of premature birth in women who have had one premature delivery in the past. Do not worry. This drug does not cause any harm to the fetus and in fact beneficial in prolonging the pregnancy to term.Hope I have answered your question. Let me know if I can assist you further. Regards, Dr. Saranya Ramadoss, General & Family Physician"
},
{
"id": 187012,
"tgt": "Why my gum is swollen around my tooth?",
"src": "Patient: on the upper left part of my jaw my gum is swollen around my tooth. the tooth itself is very sensitive, even when i drink water and it touches the tooth it hurts, and to make things worse i don't feel to well. i have this headache, sluggish feeling. what could it possibly be? Doctor: Hi,Thanks for posting the query, This could be due to infection in the tooth, i would suggest you to get a checku done take an IOPA x-ray of the region. At home take lukewarm saline and antiseptic mouthwash rinses.Go for complete mouth scaling and polishing.Take care!"
},
{
"id": 117985,
"tgt": "What is the meaning of enlarged blood cells?",
"src": "Patient: I am worried. My parner (55 yr old man) has just been given the results of his blood test (he went because he had a lot of diarrorreah and is losing weight a bit - and stressed). He was told he has \"enlarged red bood cells\" - but that all other blood results results are OK - no problems showing with liver function etc. But he has been asked to return for a further test - the Dr doesn't seem too worried - he mentioned possible anaemia. But my partner now has to wait nearly 3 weeks for the next blood test. I've looked up on NHS pt info and it says a lot about leukemia?? What could \"enlarged red blood cells\" mean? Doctor: Hi,Do not worry.Enlarged blood cells means macrocytes. These are cells which have much larger volume than normal RBC.These are seen in anemia (low hemoglobin) due to vit B12 or folic acid deficiency.A follow up is advised to identify the amount of vitamin deficiency by doing biochemical assay.Recent diarrhea was another symptom and was not the cause of vitamin deficiency.Any further queries, happy to help again.Dr. Prakash HM"
},
{
"id": 65776,
"tgt": "Suggest remedy for lumps in liver",
"src": "Patient: my mum has been told today that she has a fatty lump on her liver , what could this mean, she recieved a kidney transplant 3 years ago and now has a uti that they cant get under control, on scan they said the kidney is fine but theve found a fatty lump on her liver, Doctor: Hi.Thanks for your query.Read and understood the history about your Mother. She had undergone kidney transplant 3 years ago, has UTI that is not getting under contro, the ultrasonography shows kidney is fine but they found a fatty lump on the liver. From this report one thing is certain that there is no cancer or so, hence nothing much to worry about.Fatty lump on the liver needs no active treatment.-To confirm that this is benign, it is better to get Contrast enhanced CT scan or MRI and keep this under observation.-Get Liver function tests done, and if no need to worry if these are normal."
},
{
"id": 101128,
"tgt": "What causes weakness in legs and asthma in a child?",
"src": "Patient: When my daughter dances she has weakness in her legs when she is done dancing (tap) hard. She has exercise induced asthma and has braething problems as well. She uses an inhaler before she starts. What coul be the cause of this? It is very scary when it happens.She is 17 years old and weights about 140 lbs, Doctor: Hello.Thank you for asking at HCM.I went through your daughter's history.I would like to know a few more details like -Does she have nose symptoms (nasal congestion, sneezing, etc) ? Does she have breathing problems without exercise? Does she have night-time cough? Does she have any other physical problems?However, from your given history, I would like to reply your queries as follows:1. If she has breathing problems apart from exercise, I would suggest her taking montelukast alone for at least 2-3 months.2. If she has frequent nose symptoms also, I would suggest to add regular cetirizine or levocetirizine for at least 2-3 months.3. I would personally suggest her to get allergy testing done. This will help her know what she is allergic to and how to avoid it.4. It is perfect that she is using inhaler before she starts. However, if this is not enough, I would make two suggestions: a. To take a montelukast an hour before dancing. b. To perform a brief exercise (like running) for 5-10 minutes, approximately 45-60 minutes before her dancing time. This may help her performing dancing session more smoothly.5. For leg weakness, I would suggest you to check her hemoglobin levels, if low, they should be corrected with oral iron, folic acid and vitamin B12. 6. Were I treating her, I would also prescribe her vitamin and mineral supplements including vitamin D and calcium in form of multivitamins for 3 months.7. Personally I would also suggest her regular stretching and relaxing exercises, especially involving lower limbs.Hope above suggestions will be helpful to her.Wish her the best of the health.Should you have any further query, please feel free to ask at HCM.Thank you & Regards."
},
{
"id": 195101,
"tgt": "Suggest OTC medication for difficulty in reaching orgasm",
"src": "Patient: I had radical prosectomy about 12 years ago. At that time the urologist prescribed cialis as a treatment therapy, a pill a day, but I couldn t afford it. Now I am completely impotent. I can however achieve orgasm, with extreme diificulty. I have good medical insurance. I notice that viagra helps a bit, but insurance still won t pay. Can a urologist help. or is it time to shop around for a monastery, Sleepless in Fairbanks Doctor: Hi, You can try herbal supplements available in the markets. For proper treatment you have to consult an andrologist and get evaluated. Hope I have answered your query. Let me know if I can assist you further."
},
{
"id": 162576,
"tgt": "Does loneliness cause Asperger s syndrome in children with ADHD?",
"src": "Patient: Good afternoon, Doctor. My son was diagnosed with ADHD, combined type, at age 6. He is now 15yo going 16 this coming September. As far as i can remember, he had always wanted to have a friend---someone to talk to, to play with, to share things with. i ve read so much about adhd, autism and asperger syndrome. Will he develop asperger syndrome if he continue to have no friend? He is into computer games now because he has not found someone whom he can talk to and spend time with. Doctor: Hello and Welcome to \u2018Ask A Doctor\u2019 service. I have reviewed your query and here is my advice. Communication is one of the most important human social skills. You can do two things: find out nearby social activities they we organized your meeting near by developmental therapist' or you can join group of people in your area who have similar problem and you question regarding asperger is no. Hope I have answered your query. Let me know if I can assist you further. Regards, Dr. Vikas Bhalla"
},
{
"id": 11445,
"tgt": "Suggest remedy to stop hair fall and increase hair growth",
"src": "Patient: hi doctor this is prasanth from india... recently i m under going severe hair fall.. i was critisised severely by my friends as i m just 20 years of age... since my father is bald, it may be because of heridity problem i may posses hair problem... but its really a worrying factor for me... i lost most of the hair and i m very close to baldness.. my forehead is very big.. suggest me a solution to get back my hair and some to increase the hair density since it is very less in nature... Doctor: Hello dear,The symptoms & case history mentioned in your post suggest that you might be having Androgenetic alopecia.Also known as Male pattern baldness, it usually follows a pattern of receding hairline & is related to genetic makeup and male sex hormones.Management includes:1. Minoxidil preparation to be applied directly to the scalp to stimulate the hair follicles.2. Finasteride preparations that interferes with the production of male hormone linked to baldness.But these medications have to be used regularly under the guidance of a Dermatologist...as hair loss tends to return on stoppage of these medications.So...it will be better to consult a Dermatologist & start treatment.Wishing you a Good Health.Take care."
},
{
"id": 30696,
"tgt": "What causes shivering post a bike crash?",
"src": "Patient: im not pregnant how ever earlya today i was riding my motorbike in a old chalk quarry it is now abandond and has been for over 10-15 yrs and it now looks like a lake were one of the stepdowns has filled with water i crashed my bike into it earlya today and woundered if you could help me understand wat i am feeling, wen i chrashed into it it was so cold that i oculd not get my breath even tho i was above water its like my lungs had frozen(posssibly shock of the cold like jumping into a ice pond like the sas do on training) i then had to go in and out about another 5 times to drag the bike out but im now having trouble breating its like my lungs are giving up but i dnt believe it to be this bad i got a pain in my throat i cant swallow andmy lungs kill every time i breathe in i got water dripping out of my nose and keep going to be sick but the sickness is not from my stomach it is from my lungs and as far as i can remember i did not swallow any water can you shed some light onto why i am so cold and finding it hard to breathe (yes i am aware there was a risk of hyperthermia but once again i do not believe i have this either as i have been able to warm up abit but i am still shivvering) this happened at 5 pm british gmt it is now 11:27 pm british gmt Doctor: Hello,Welcome to HCM.First of all thank you for sharing your concern with us .Breathing problem and shivering after an accident in cold weather can be due to following causes:1. cold 2. lung injury I would recommend to get a chest X ray done to rule out any possible injury in chest and discuss further with your Healthcare providers. If clinical examination and chest X ray is fine then such symptoms are due to cold and musculo-skeletal injury which will resolve on it's own but first rule out any possible chest trauma as mentioned earlier .Best Regards,Dr.S.Tomar"
},
{
"id": 167778,
"tgt": "What causes shivering and cough in children?",
"src": "Patient: Hi there my 3 1/2 yr old son woke up with the horrible cough almost sound like a bark. He spit out some phlegm a couple of time all in the midst of cryin and shaking.. However once i calmed him down he began shivering like crazy. His hands,feet and body were ice cold.. But his forehead was norma. Right inbetween. I cant figure out if he has no fever why the intense shivering and what about the barking couch Doctor: the shivering and cold feet and hands are all indicates fever even if his forehead seems normal , try to take a rectal or ear temperature if possible and it will be high .fever and barking cough are signs of upper respiratory tract infection , caused by viral infection , which is self limited disease and will ends by it own .control the fever , keep him hydrated is all you need to do , usually cough syrups won't take out the cough but it can ease it .If you notice any strider: high pitched sound when your son inspire , or you feel he is having difficulty breathing you need to take him to his doctor to assess him and measure his oxygen level , because some of these viruses can obstruct the airways and cause them to spasm .I hope this helps"
},
{
"id": 150633,
"tgt": "Frequent fainting. Affecting normal life. Blood tests normal. No apetite. Reason?",
"src": "Patient: I have been fainting for the last 2 weeks, first it was just after a workout then it was doing anything from sitting to walking across my kitchen, i have been to the drs 3 times last week and will be going back tomorrow. they seem to dismiss it and it is very frusterating. they have told me not to go to work, not to drive and not to workout. i need to get back into a routine and need answers. I have had no apetite but all of the bloodwork showed up normal and not conclusive of any viruses . please help. Doctor: Hello,Thanks for the query,I understand your problemThe causes of repeated fainting could be due to low blood hemoglobin, heart rhythm problem, postural hypotension and rarely could be due to seizures. You need specialized care and investigations. You can consult a local cardiologist and/or a neurologist who will interview you to obtain a detailed of the situations of the spells that you have and this will guide us regarding the further investigations. You would require ECG to check your heart rhythm, CT scan of the brain, holter monitoring, tilt table test etc. Once the cause is established, we will be able to offer you the best possible treatmentHope this clarifiesBest wishesDr Gopal K Dash MD, DM, Post-doctoral fellowship (Epilepsy)Consultant Neurologist and Epilepsy specialistNarayana Hrudayalaya Hospital, BangaloreMy Blog in the Web site:http://www.healthcaremagic.com/doctors/dr-gopal-krishna-dash/64344"
},
{
"id": 151717,
"tgt": "What could be the cause and remedy for difficulty in gaining natural muscle in one with carpal tunnel ?",
"src": "Patient: i just got back from the chiropractor and he told me i have carpal tunnel . He had asked me about my fitness levels i am 18 and female and i do at least 30 min of jogging (2.5-4mi) 3-5 times a week and sometimes i follow up with a sport ( basketball or racquetball). He felt my arms for muscle and said i really didn t have any and suggested that maybe my hormone levels were low and i can t gain muscle as easily as a normal person, but because he doesn t specialize in that he cannot diagnose anything. What could be a possible fix or explanation as to why i am not gaining small amounts of natural muscle? Doctor: Hi welcome to H.C.M.Forum. regarding muscle growth depends upon the size of the parents and blood relatives. inspite you can try to cope up. use more of fats, tubers especially sweetpotato, fleshy fruits green leefy vegetables. incline more for junk foods .it is not a factor of harmones. thank you."
},
{
"id": 94288,
"tgt": "Pain in the lower abdomen and pelvic area. Could this be PMS?",
"src": "Patient: I have lower abdominal pain . Around my pelvic area more at my right side. I am also having headaches and random back pain and tender breasts, on and off.. My upper legs hurt too...It looks like PMS as I should be having my period anytime now. Last month I had it at the 11th. Could there be any other reason? It s the first time these symptoms Are occuring. Especially the pain pain in the abdomen Doctor: Hi there, Thanks for writing in. Most likely what are experiencing might be symptoms of PMS. It should get better in 2-3 days, once you get your periods. I'm assuming you don't have any other symptoms like vomiting , or high grade fever, which if present might point to some other pathology. Meanwhile you might want to take a meftal spas tablet for symptomatic pain relief. Drink warm water at regular intervals. Hope this helps. Regards, Dr. Divya Kuttikrishnan"
},
{
"id": 110747,
"tgt": "What causes lower back pain when I sit on a toilet?",
"src": "Patient: Lower back pain when i sit on a toilet. I am \u00e0 m\u00e2le 74 years old and get a sharp pain just above th\u00e9 sacrum. I suffer from an oversised prostate for which i take flowmax and avodart daily. Any suggestions, i have discussed with my Dr and he has no advice. My e mail is YYYY@YYYY Doctor: it could facet joint arthropathy, common in your age group. Proper history examination required. visit you pain physician he will guide you medical and intervention measures depending on the cause."
},
{
"id": 94505,
"tgt": "Dull right-sided abdominal pain. Ultrasound detected hernia. Blockage or inflammation of the intestine?",
"src": "Patient: Sure! I have a dull pain in my right side of abdomen . it has been going on for sometime now. the ultrasound i did only showed a small hernia . I am suspicious of inflammation or blockage of the intestine! I don t know what kind of test I should do to diagnose the problem! The pain shoots to my back as well on the right side! Doctor: Hello, Welcome to Healthcare Magic. I appreciate your efforts for medical consultation in so much distress. Well, if your ultrasound is telling that you have hernia on the same side as the pain then it is quiet possible there is formation of adhesion in the content of hernia and in that case you may need surgical intervention and for that you will have to consult a GI/ Gen Surgeon..And to get rid of your suspicion of obstruction/inflammation you can get CECT Abdomen with oral contrast done\u2026it will clear all your doubts\u2026 Wish you good health. Regards"
},
{
"id": 116926,
"tgt": "Can Sangobion be taken for alpha thalassemia trait?",
"src": "Patient: In my blood test results, it shows that I have alpha thalassaemia trait and my MCV and MCH is below the average range. I constantly suffer from shortness of breath, weakness for many years. Can I take Sangobion (a kind of iron supplement)? Or should I take spirulina to alter the condition. A doctor suggest me to take folic acid, another doctor ask me to take antidepressant drug. Another one suggest hormone treatment. I am 47 years old now. I only have one day of menstruation and only traces of it for almost half a year. Ms. Hong. Doctor: Hello, Thank you for your contact to health care magic. I understand your concern. If I am your doctor I suggest you to take sangobion. As your MCV and much both are below normal there is strong need of increasing the iron and so haemoglobin. I will be happy to answer your further concernYou can contact me. Dr Arun Tank. Infectious disease specialist. Thank you."
},
{
"id": 172291,
"tgt": "What treatment is suggested for accidental intake of toilet cleaner?",
"src": "Patient: A 11 year kid I know drank Harpic Toiler Cleaner by mistake. He has been rushed to a hospital A 11 year kid I kA 11 year kid I know drank Harpic Toiler Cleaner by mistake. He has been rushed to a hospital and has been given a stomach wash but continues to vomit blood and bubbles...what further medicare... Doctor: Hi read ur question Harpic toilet cleaner is not so much poisoning . but it is very irrenent solutionWhich causes gastritis and vomiting . stamach wash is clear the harpic from stamach but the solution which was absorb may cause vomintig and gastritis . For gastritis and vomiting anti emetic and antacid or any of injectable anti gastritis drug given Make sure that ur baby is admitted in hospital till complete recovery is present"
},
{
"id": 97268,
"tgt": "What to do for the pain the knee cap followed by an accident?",
"src": "Patient: fell directly on my knee cap while ice skating. it was painful...swelled, bruised.. this was 3 weeks ago. It is still painful to the touch specially at the exact point on impact. when I move it there is a grinding sensation. If I bump the spot the pain is intense. I can t even sleep on my stomach the weight of my leg on the knee cap is bad:( Doctor: Hi and welcome to HCM It's high time for you to consult a orthopaedic doctor. Bad knee injury should not be neglected and you need a thorough examination by a doctor. The grinding sound suggests that you may be having a ligament tear, the treatment depends on the extent and type of the tear. Till the time you consult doctor follow these instructions 1.Dont put weight on that leg2. Take bed rest 3. Avoid any kind of sports 4. Apply ice packs on affected area three times daily for 15 minutes 5. You can over the counter pain killer medications 6. Consult orthopaedician Regards Dr Santosh"
},
{
"id": 198050,
"tgt": "Suggest treatment for semen leakage, memory loss and erectile dysfunction",
"src": "Patient: hi sir am 26 years old .am suffer from semen leakege during night .i doing masterbate from age of 15 .i need some home remedy to recover from this problem please help me.i have lots of problem memory loss,body weak erection dyfunction so please i want imediate ans from u. Doctor: Dear, We understand your concernsI went through your details. You don't have to worry about your masturbation habit. Masturbation is normal and natural. Masturbation cannot cause any physical or mental problems if your habit is within limit. Semen leakage during night (night fall, nocturnal ejaculation) is also a normal and automatic process. This process is natural and automatic and therefore you cannot control it. But again, there is no health issues regarding your semen leakage. It is just the accumulated semen, that is leaking. These two issues, masturbation and semen leakage cannot cause any memory problems, weak body, weak erection etc.But if your knowledge about the above facts are wrong, you might have anxiety related symptoms and that is what you are suffering now. Please consult a psychologist for counseling.If you require more of my help in this aspect, please use this URL. http://goo.gl/aYW2pR. Make sure that you include every minute detail possible. Hope this answers your query. Further clarifications are welcome.Good luck. Happy New Year. Take care."
},
{
"id": 198168,
"tgt": "What is the cause and treatment for blood during ejaculation?",
"src": "Patient: My husband ejaculated and there was a small amount of blood in it. It burned when coming out and he has become nauseas. Everything is fine, he says he has no groin pain, no fever, no. He's and this is the first time it has ever happened. What is going on? Doctor: DearWe understand your concernsI went through your details. Blood in the semen is called hematospermia or hemospermia. Seeing blood in the semen can make a man anxious. Fortunately, it doesn't always signal a major medical problem. For men younger than 40 with no related symptoms and no risk factors for underlying medical conditions, blood in semen often disappears on its own.But for men 40 and over, chances are higher that blood in the semen needs evaluation and treatment. Causes of Blood in the Semen could be infection, prostrate related problems, STDs etc. In any case, it is better to consult a doctor. If you require more of my help in this aspect, please use this URL. http://goo.gl/aYW2pR. Make sure that you include every minute detail possible. Hope this answers your query. Further clarifications are welcome.Good luck. Take care."
},
{
"id": 17599,
"tgt": "What causes heart pains?",
"src": "Patient: I recently have been having heart pains. I don t think that it is heart burn because I took an antacid which didn t do anything for the pain. Last time I was seen at the doctor my blood pressure was 120/86, and they told me to keep an eye on the lower number. Do I need to seek medical advice or can I just wait it out and see if there is any improvment. Doctor: Hi, In a suspected case of cardiac pain, apart from clinical evaluation, you need to undergo ECG, Treadmill test or Stress test, Chest X-ray and 2-D Echocardiogram. The golden test for ischaemic heart disease causing chest pain is Coronary angiography (CAG). Please consult your doctor. Hope I have answered your query. Let me know if I can assist you further. Regards, Dr. Tushar Kanti Biswas, Internal Medicine Specialist"
},
{
"id": 24534,
"tgt": "Suggest medication for heart palpitations,anxiety and dizziness",
"src": "Patient: My daughter has suffered from vertigo for more than 2 years. It is accompanied by palpitations,general weakness,a heaviness on her chest.She has been under the psychiatrist for anxiety but has had no relief after taking diazepam and seroxat.The dizziness is daily to some extent or other and sometimes she can barely stand.Problems with the ear have been ruled out. Doctor: Thanks for your question on Healthcare Magic. I can understand your concern. In my opinion, she should consult another psychiatrist and psychotherapist. All her symptoms are due to uncontrolled stress, anxiety and panic disorder. Counselling plays very important role in control of these symptoms. And psychotherapist are the best persons who give counseling. She will also need newer anxiolytic and anti depressants for better symptomatic relief. So don't worry, she will be alright with proper counselling and newer treatment. Hope I have solved your query. I will be happy to help you further. Wishing good health to your daughter. Thanks."
},
{
"id": 196085,
"tgt": "What causes vein to protrude from left testicle?",
"src": "Patient: hi my boyfriend realized that there is something different about his left testical. on the bottom /tip of the left one there is something protruding it is on both testicals but the left one is more pronouced. it looks like the vains are protruding. The area dose not hurt or feel hard . it feels like fleshy or spongy type of lumpIm 20 years old male Doctor: Hello Thank you for trusting HCM Dear you may be suffering from varicocele, don't neglect it consult your surgeon he will advice correct treatment to you."
},
{
"id": 168051,
"tgt": "Suggest supplements for G6PD deficiency",
"src": "Patient: hi!im maren from the philippines. my 1 1/2 yr old son is g6pd deficient.what food should i feed him?i m still breastfeeding. should i also avoid the the foods to be avoided by g6pd deficients. does it helps if we exposed him to sunlight every morning? Doctor: Foods that include MENTHOL, like candies, toothpastes, biting gums, mouthwash, etc., is required to be avoided by people along with G6PD deficiency. LEGUMES for instance fava beans, kidney legumes, and lima beans must not be consumed. Also, soy pinto beans, and foods containing soy, similar to tofu, miso or soy healthy proteins, are a strict no-no. Peanuts, licorice, alfalfa plants sprouting up, and products that contain these food types should be avoided continuously. Foods that carry ascorbic acid or nutritional C in large quantities shouldn't be consumed. Artificial inorganic dyes, especially the color pink, should be avoided by of those with G6PD deficiency. Refined sugars, corn syrup, and light flour, should be refrained from inside diet, as the person is deficient in the respective enzymes was required to break down these foodstuff. Drinking tonic mineral water or eating bitter melon is forbidden for individuals suffering from G6PD lack. Some additional foods that ought to be refrained from are earth-friendly peas, field peas, black-eyed peas, and a few edible pods like Chinese language pea pods and perfect peas. Lentils, as well as some beans which include black beans or refried beans ought not to be eaten. Avoid Chinese meals, as they contain herbal remedies like Rhizoma coptidis, margarita, Flos lonicerae, along with Calculus bovis, that might possibly trigger a hemolytic tv show. Packaged foods may contain some things that provoke a reaction, while should be avoided. These food types include margarine, jarred greens dressings, canned fish plus meat, hot dogs, ready-made meat, sausages, tinned sauces, potato chips, low-fat cheeses, and various sauces like Worcestershire gravy or sweet and wrong sauce. Read the constituents in processed foods mindfully before buying or consuming the products. If you try to stop certain foods, and understand the triggers, it becomes simpler to curb the symptoms of your disorder. Prevention is always the very best cure. Take care!"
},
{
"id": 126414,
"tgt": "What causes a crack sound while sprinting or jumping high long after an injury?",
"src": "Patient: Hi, I got injured a year and 5 months ago I would go to the doctor about 3 times a month but they would never find out what\u2019s wrong to this day I can\u2019t sprint or jump very high in a jumping house because I know that at one point I will hear a crack. Do you any of you guys know what\u2019s wrong with me. And by the way I did go to therapy for about 2 months and it didn\u2019t help. Doctor: Hi, It could be a locked knee or foreign body trapped inside. You can consult an orthopedician and plan for an MRI scan. Hope I have answered your query. Let me know if I can assist you further. Regards, Dr. Shinas Hussain, General & Family Physician"
},
{
"id": 67793,
"tgt": "What causes buttock cheeks lumps?",
"src": "Patient: I just had a baby 12 days ago and I had a small tear that needed stitches. That has healed and now I have noticed red, pimple like bumps on my butt cheeks. I take showers regularly, and I have been wearing pads due to bleeding from healing. What has caused these red bumps and what are they? Doctor: Hi, dear. I have gone through your question. I can understand your concern. You may have some infection or abscess formation in that area. You should go for examination. Then you should take treatment if needed. Consult your doctor and take treatment accordingly. Thanks for using health care magic. Wish you a very good health."
},
{
"id": 64210,
"tgt": "What is the treatment for a lump in the left arm pit?",
"src": "Patient: I have a lump under my left arm Pit..bigger than a marble..it seems to be attached..is under the skin..not on the outside like a bite..it is very hard mass..and has been there sinse Sunday..i do not have medical Insurance..should I just wait and see if it changes..there is NO PAIN WITH IT Doctor: Hi,dear- I studied your query to HCM and understood yur health concerns.Don't worry as its not a cancer.-Mostly your armpit lump is furuncle in early stage and needs waiting.-If it pains with redness take tb- diclofenacAnd floxacin 200 mg under advise from Er doctor.This will resolve your lump.If it grows in size with pointing You need to drain it with held of ER Surgeon.Wish you fast recovery.Have a Gd day."
},
{
"id": 135213,
"tgt": "What causes pulsations and soreness in the shoulders?",
"src": "Patient: ive been lifting for about two years, i play football as well.. but i was bench pressing and i knew my form was off now my left shoulder pulsates randomly and hurts at random times. I keep working out but my shoulder still hurts, ive had this pain for about 4 weeks Doctor: Hello,I have studied your case and I think you are having sub acromial bursitis. In this condition there is fluid accumulation in the shoulder and this can cause such pain. For confirmation you need to get MRI of the shoulder.it is good that you are not doing any hard work but you need physiotherapy and hot water fomentation. Also some time there is injection of steroid needed. I hope this answer will be useful for you.Let me know if there is any other followup questions.thanks"
},
{
"id": 185942,
"tgt": "What causes increased amount of saliva?",
"src": "Patient: I quit smoking this January and experienced a significant increase in saliva. How long will this increase go on. I did not experience the increase in coughing. This saliva thing is driving me crazy. I didn't notice this at first. I am also taking Prystique a new form of effexor. I have asked the therapist if this is from the drug. They do not think so. Dry mouth is more expected. Anyway, if you have any ideas on this I would greatly appreciate it. Thank you Doctor: Hello! Read your query.These drugs are supposed to reduce the salivary flow.Smoking also reduces salivary flow to a noticeable extent.You did not mention the reason for taking Prystique.Check if you have gastic reflux disease.This can be due to a poor clearance rather than excess production(related to your depression)You must consult your physician regarding this.Use a mouthwash diluted in 1:1 ratio.Hope this helps."
},
{
"id": 65408,
"tgt": "What causes hard lump on top of my spine?",
"src": "Patient: i have a hard lump on the top of my spine between my shoulder blades, causing alot of neck and shoulder pain and numbness in right arm it has been there awhile but grown the last 6 months i am 33 yrs of ageits also causes pain down my back, what could it be Doctor: Hello!Thank you for the query.In this location most probable is benign soft tissues lesion which gives some compression on spine. It can be a sebacues cyst or lipoma. Both are benign, and do not turn into a cancer. I suggest you to consult your doctor, have soft tissues ultrasound to confirm this diagnosis. Such lump can be removed with local anesthesia and small incision.Hope this will help.Regards."
},
{
"id": 216815,
"tgt": "Suggest treatment for severe hip pain and bump on head after fall",
"src": "Patient: Hello i fell off of a makeshift fourwheeler and hit my head on concreate and my hip. My head a goose egg just a little sore only in that spot no headache eyes are dilating normally. My hip on the other hand. ... it hurts so bad to sit stand lay down even laugh for that matter could it be broken? Doctor: I wish to recommend you to visit an ortho specialist for the same. as an x ray might be needed to look into the issue. mean time you can use icing for pain relief and any swelling. thank you"
},
{
"id": 155989,
"tgt": "Should i be worried about cancer with tightness in chest,bumps in veins and pain in shoulder?",
"src": "Patient: I am a 44 yr old female smoker, having shortness of breath, chest feeling tight, eyes dialating different, \"bumps\" in veins, pain between shoulder blades. Ongoing for awhile. progressivly getting worse. Chest xray showed no tumors.Only that my ribs were very spaced out. Could it still be lung cancer & xray just missed Doctor: HiYour symptoms are not in favour of lung cancer. But, get an ECG or it's better to go for Treadmill test to rule out heart problem. If it is normal,then CT Scan chest may be needed to rule out Lung cancer. Hope I answered your query. Wish you good health. Regards"
},
{
"id": 202104,
"tgt": "How to get rid of masturbate addiction?",
"src": "Patient: hello doc. i m 25 male.., i use to masturbate regulary due to this i cnt concentrate in my studies and on other things and i m becoming skiny. so i want to stop it, i heard that there are drugs to eliminate sex drive.. so could you suggest me............... Doctor: Hello dear,First of all, do not get influenced by any drugs claiming to eliminate sex drive.Masturbation is a normal phenomena at your age, and does not have any side effect on body or mind, unless you get addicted to it.Since you are feeling that masturbation is interfering with your normal functioning, you can try out the following measures:1. Improve your social life by spending more time with friends and relatives.2. Keep your mind active and busy at all times either through curricular or extra-curricular activities.3. Have a diet rich in fruits, vegetables, fish, nuts & honey.4. Avoid fast foods & lipid containing diet.5. Avoid smoking & alcohol.6. Keep away stress, think positive.7. Exercise regularly & practice meditationThese measures will be helpful in improving concentration & builds up the confidence level.Take care."
},
{
"id": 89092,
"tgt": "Suggest treatment for severe abdominal cramping",
"src": "Patient: 56, WF, Since yesterday afternoon .... severe abdominal cramping, strikes like labor pains, about every 15 mins. Normal BM this morning, cramping did not stop. Unable to pass gas in spite of abdominal swelling and much rumbling in stomach. Pain is mostly near ab hernia below umbilical area. Doctor: Hi.Thanks for clear-cut history, which is suggestive of an obstructed Hernia with a classical history of Distention, Borborygmi (rumbling), and severe pain in abdomen. Get an X-ray of the abdomen in standing position to get a diagnosis clear and if necessary an ultrasonography of the abdomen and abdominal wall to get a confirmed diagnosis.If present get admitted and get investigated to be cured by a surgery, which may be undertaken urgently. Stop taking anything orally."
},
{
"id": 189662,
"tgt": "Had a root canal, crown fitted, abcess formation, blisters on gum. What can be done?",
"src": "Patient: thankyou. I had a root canal completed and post and crown fitted. the inner wall has cracked and has formed an abcess. the abcess swells and forms a blister on the side of the gum due to the pressure build up from the post being fitted. Upon removing the post, the abcess has subsided (i assume because it has a place of release). I obviously need to get the tooth(and abcess) removed. So a couple of questions, are there any potential health issues whilst the tooth remains (with abcess)? Are there any health issues or complications that may arise from the tooth being pulled, given that it contains infection? Should i take a course of anti-biotics? Anything further that you may care to add? thanks for your time and response. Andrew Doctor: Hello, I am Dr.Neha Gupta (dentist) and am glad to address to your query here. it sound like you have dental abscess, a potentially painful and dangerous condition that could get worse unless you get rid of the cause of the infection which is uaually the nerve of the tooth. popping the pimple could provide temporary relief because the puss that has been traped at the apex of the tooth is finally released from the gum of your mouth. this condition needs to be fixed by either performing root canal treatment or extraction of tooth.antibiotics alone cannot fix the problem. Take care and please keep me informed of your progress at healthcaremagic through my profile directly. Regards"
},
{
"id": 26024,
"tgt": "Is it dangerous to smoke weed with Mitral Valve Prolapse?",
"src": "Patient: Is it dangerous to smoke weed with Mitral Valve Prolapse? I am asking it because last time I smoked I felt like I am going to have a panic attack. My heart beat very fast, I thought It will explode, had a chest pain on my left side and the chest pain remained for 5 days constantly. Doctor: yes.it's bad for ur health if u smoke weed as u have mitral valve prolapse.You should Go for prosthetic valve plasty and then u can smoke within limits."
},
{
"id": 185599,
"tgt": "What causes infection in wisdom tooth?",
"src": "Patient: I am in pain, infected wisdom teeth. I have antibiotics from yesterday but the swelling is the same. Yesterday I was also told that I have high prolactin level and I might have some tumor that's easily treated with some drugs. Is my infection caused by this and could it get worse? (it's not getting any better and it's very painful) . Doctor: HiThanks for your query with health care magic, it would have been better if you had mentioned the antibiotics you are on. its fine may be your wisdom tooth is impacted and it has got pericoronal flap i believe. Nothing to worry you also take pain killers along with the antibiotics then the pain will come down and seek dentists attention immediately otherwise infection might spread to other areas also like sub mandibular spaces.I think you are talking about dentigerous cyst which is commonly associated with impacted wisdom tooth even that can be taken care by your dentist.Kindly visit your dentist for further decision and treatment.Hope this helps you."
},
{
"id": 154139,
"tgt": "Is Myeloproliferative and lymphatic leukemia curable?",
"src": "Patient: Hello, my dad he is 82 years old and has been sick for the past two years. He kept telling us the doctors don't know what he has. He recently told me he has bone marrow cancer. Myeloproliserapye loi &lymphatic leukemia ( not sure if I have the right spelling. He says the doctors can't treat him because the treatment would be worse than his illness and they could only guarantee at most a 10 percent chance that it would work. Anyways I am a bit confused and hope you could provide me with some info. His health seems to be getting worse. Thanks for any info you can provide. Doctor: Hi,Thanks for writing in.Myeloproliferative disease is a condition affecting the bone marrow and usually changes are visible in the bones of the spine. Right now it is suggested to study the MRI scan of his spine if done earlier and if not doe then doing it important. The MRI scan will show the changes which might indicate any bone marrow pathology like myeloproliferative disorders.If he has any problems associated to a particular bone then that needs to be investigated in detail. He might also require a bone marrow biopsy to know the exact nature of cells and confirm condition including myeloproliferative disorder or leukemia. If he has been consulting his doctor for two years then it is most likely slowly progressive. Any treatment and its suitability can be discussed only after making a confirmed diagnosis by doing bone marrow study and immunohistochemistry. Please do not worry."
},
{
"id": 153212,
"tgt": "Can osteogenic type II b osteosarcoma in knee be recurred?",
"src": "Patient: My 19 year old son was diagnose with Osteogenic type II b osteosarcoma on the knee (about 5cm). He had limb sparing surgery and systematic Chemotherapy (Adryomicin, cisplatin and methotrexade). He has had his first 3 month check up and every test is clear, his hair is growing back and he is feeling better and better. The surgery revealed that the chemo had killed >95% of the cancer...When I read the survival rates, they are quite low...Is this accurate, is this one of worst cancers, can we be optimistic, is there any vitamin/food that can help...He is doing so well that I cannot imagine this thing coming back....Thank you Doctor: Hi,Thanks for writing in.Osteogenic sarcoma is a bone forming cancer. Stage IIb is early disease and he is lucky to undergo limb sparing surgery and combination chemotherapy. The goal of cancer treatment is to destroy as many cancer cells and prevent the progress of the cancer.The first step is stabilizing the disease and once this is achieved then the goal is destroying the cancer cells. If the doctors have said that more than 95 percent of the cancer cells are destroyed then your son is in a better position than most patients. He should continue treatment and follow up as recommended. Nutrition is important during treatment and recovery and he should be eating a high protein diet will vitamins and minerals.The risk of recurrence or disease spread persists and close follow up is the only way early disease manifestation might be noticed and aggressive treatment given. If the disease is under control for 5 years then the risk of recurrence is low. Apart from the primary cancer location, it is important to get chest X rays and CT scan chest done as required because osteogenic sarcoma stage IIb has a risk for spread to the lungs. Please do not worry."
},
{
"id": 16899,
"tgt": "Suggest treatment for rapid heart rate",
"src": "Patient: Everytime I stand up for a long period of time, especially if I m holding something (such as a child), my heart beats fast, and I get lightheaded to the point where I have to sit down or I feel like I m gonna pass out. Should I be worried about this? Doctor: Hi, There is nothing to worry about, but I prefer to make some more tests to be sure, like ECHO imaging on the heart to see everything is going right. Hope I have answered your query. Let me know if I can assist you further. Regards, Dr. Salah Saad Shoman, Internal Medicine Specialist"
},
{
"id": 118140,
"tgt": "Does Livogen help for anemia without side effects?",
"src": "Patient: Hello. I'm 18 and anemic. I used to take iron supplements when the doctor first told me I was anemic but stopped after 6 months. I haven't been getting enough Iron off late as I stay in a hostel. Is it alright for me to take Livogen tablets? Will there be any side effects?Thank you. Doctor: Livogen tablet contains ferrous fumarate and folic acid.It can be used in microcytic hypochromic anaemia to increase iron.The side effects are sore throat, trouble swallowing, severe stomach pain, blood in your stools, diarrhea, nausea and vomiting.RegardsDR De"
},
{
"id": 68235,
"tgt": "What causes cyst like bump on upper cheek bone?",
"src": "Patient: I have what appears to be a cyst on my upper cheek bone. It doesn't hurt and there is nothing topical to see. Just a bump. Is there something I can do without going to a doctor?And if you recommend a doctor, what kind? A dermatologist?Thank youkK Doctor: Welcome to health care magic. 1.The description suggest the possible cause could be the cyst arising from the bone - dentigerous cyst. And other possible cause for pain less lump are salivary gland dilatation due to distal obstruction. 2.This if you are my patient i would have examined and ask for an ultrasound to assess the nature of the lump, its source and extensions.3.In case of dentigerous cyst an x-ray panoramic view will be helpful, it will detect the nature and extensions of the cyst.4.Further CT might be needed for better demarcation of the lesion and cyst description.5.Sugget forst to see your GP after that the GP with initial examination or investigation decide where to direct you. Good luck.Hope i have answered your query,any thing to ask do not hesitate to ask.http://doctor.healthcaremagic.com/doctors/dr-ganesh/62888"
},
{
"id": 40388,
"tgt": "How do I know if submandibular gland has an infection?",
"src": "Patient: Hi I belive my right submandubular gland might have a infection in it or maybe a stone. I did have 3 tiny sores on inside of my cheek three days ago . Now it hurts and is very swollen. It does have a slight red line too. What is the worst that can happen to me? Doctor: HI. IN an infection , the sumandibular gland is swollen, painful, may increase in size with food, there may be a fever with chills.The opening duct is under the tongue near the center called frenulum. The three sores in the cheek may be due to another reasons.Nothing worst can really happen to you, if you visit an ENT Specialist to help coming to a correct diagnosis and carry on the proper management as per your Doctor's advise ."
},
{
"id": 225902,
"tgt": "Using Nuvaring. Heavy bleeding. Why am I bleeding with Nuvaring in?",
"src": "Patient: Hi, I am a 21 year old who has been on nuvaring for around two years, and I love it. I use nuvaring continuously to only get my period every three months. A few days ago, I took the old nuvaring out and immedietely put a new one back in, and it has been in since then. However, tonight I have fairly heavy bleeding which I believe is my period. Why do I have my period with the nuvaring in? Is this okay? Should I leave nuvaring in? Doctor: Hi,Thank you for posting your question here, I will try to answer it to the best of my abilities.From what I can tell you arent using it right, you're supposed to keep it in for three weeks and then take it out for one week during your period.Nuvaring doesnt stop periods, it is merely a contraceptive method which is somewhat similar to how birth control pills work.As such, from now on, use it for three weeks, then pull it out for one week during your period and then once your period is over, place a new one.I hope this answered your question."
},
{
"id": 38778,
"tgt": "Suggest treatment for painful boils in groin and armpit",
"src": "Patient: I have had a condition with boils in my groin area for over 30 years now. Antibiotics do nothing to them. They get very painful and eventually drain out pus. I am so tired of living like this. I have scarring all over that area and up by buttocks. I have just noticed tonight that I am starting one in my arm pit. is there any remedy that I can use? I have been to two doctors and they tell me i just have to live with it. One was a GP and the other a GYN. Doctor: Hi,Welcome to HCM.Painful boils are most commonly caused by pyogenic bacteria like Staphylococci and Streptococci and unfortunately these bacteria develop resistance to antibiotics very easily. I would suggest you get in touch with an infectious disease specialist and a dermatologist who can help you deal with this.Recurrent boils are more commonly seen in armpits and groin due to inflammation of the apocrine sweat glands in these areas.This condition is called hidradenitis suppurativa and is due to blockage of the hair follicles on the skin, or the sweat gland openings.Hidradenitis suppurativa is more common in obese people and in cigarette smokers. I would suggest you quit smoking and lose weight if you are obese.I hope I have helped you with this answer.Thanks"
},
{
"id": 180921,
"tgt": "What does pain in the jaw indicate?",
"src": "Patient: Last year I had two of my teeth taken out which required me to have numbing injections and while my face was still numb I opened my mouth and something cracked or popped really loud in my right cheek but I couldn t feel any pain due to the injection. Several times after that the same cheek would crack and sometimes it would ache slightly but never anything too serious as it would go away by the next day. Today however it happened again but there is a lot of pain and it is aching a lot.Moving my jaw hurts a lot when i move it and when I stay still. I don t know if I ve injured it somehow or what? Doctor: Hi Dear,welcome to HCM.Understanding your concern. As per your query you have symptoms of pain in the jaw which seems to be due to TMJ dysfunction and locking of jaw in a particular position. It could be due to insertion of injection in muscle which is leading to injury and difficulty in opening mouth.Need not to worry. I would suggest you to rest properly and don't bend frequently forward. You should take light food and avoid hard food from that side. You should relax jaw from locked position and bring it back to normal. For now take ibuprofen for pain and swelling. You should consult ENT specialist and oral and maxillofacial surgeon for proper examination and start treatment accordingly. Avoid excessive use of mobile from that side.Hope your concern has been resolved.Get Well Soon.Best wishes,Dr. Harry Maheshwari"
},
{
"id": 211778,
"tgt": "Experienced psychosis, being treated with Soloquin. Will its sedative effect wear off ?",
"src": "Patient: My son who is a computer programmer and Masters of IT student travelled to the UK from Australia & his antipsychotic drug Solian stopped working on the flight over. He experienced psychosis and spent some weeks being treated with other drugs. He has now been admitted to a high care facility and being treated with Soloquin. My question: Soloquin is an older drug and seems heavily sedative with sometimes severe side affects. Will the sedative affect wear off so he can resume his life? Is there a more modern drug that would help him?Thanks AAAA Doctor: Hello,First of all you should maintain him on regular medicines and follow ups with psychiatrist. The longer is duration of untreated psychosis, harder it gets to treat.Then coming to drugs Soloquin contains Clozapine, which is though older but it is by far the most potent anti psychotic agent currently available. Sedation would wear off gradually over a period of time and side effects should not be a problem if you keep monitoring for them regularly and have a good diet and exercise plan for him. He should be able to work and encouraged to once his symptoms are under control.There is another drug Aripiprazole which does not cause sedation and side effects are minimal. Then there are newer drugs such as Iloperidon, Asenapine, Amisulpride (Solian) which are options which you can discuss with your psychiatrist.Hope it is helpful. Feel free to ask any more queries. Take care.Regards."
},
{
"id": 15284,
"tgt": "Bad hives, infection, heat rashes. Blood test showed infection. Treatment?",
"src": "Patient: HI , A couple years ago I started to get really bad hives . My doctor did a blood test and he said that I have an infection in my body. I want to all of doctors here and got tested and they could not find any infection in my body.This year I started to get really bad heat rashes and I m going crazy. Is there a permament solution for heat rashes or a cream that could stop the heat rashes? Please help.. Thank you .. Doctor: Hi,It appears that you have the problem of urticaria or hives. These linear red streaks are basically manifestation of acute urticaria. In most of the cases, there is no underlying cause but sometimes can be caused by worm infestation or food.You can take tablet loratadine or levocetirizine twice in a day and apply calamine lotion to soothe the itchy areas."
},
{
"id": 165662,
"tgt": "Can teething and cold lead to spasms in an infant?",
"src": "Patient: Hi there, my son is 6 months old and has recently started doing cluster spasms where he shurgs his shoulder to his right ear. It lasts only a second or so and goes in clusters of 3 to 10. Some days he has multiple clustures and some days he has none. He is developing normally and shows no mental or physical delays. He is currently teething and has a cold and the doctor did find fluid behind both ears. The doctor saw video of it and told me to keep an eye on it and get more video to bring to the check up in three weeks. Should I be worried about this behavior or is this something many infants do as part of their immature nervous system or learned behavior. Any answers would be greatly appreciated as I have been a nervous wreck. Thanks Doctor: Good afternoon..!!Your 6 month old baby has a condition serous otitis media (fluid in the middle ear).It causes discomfort / irritation / pain in the ears. So your baby's behaviour could be possibly due to this ear problem.So if persisting, it's better to visit pediatric ENT surgeon and pediatric neurologist, if needed, for further evaluation and management.Thank you, have a nice day..!!!"
},
{
"id": 76566,
"tgt": "Suggest treatment for shortness of breath due to COPD",
"src": "Patient: shoetness of breath cant cllimb the stairs cant stay on my legs was diagnosed with lymes disease two years ago. was recently diagnosed with copd was on spireva it stopped working told to go to a pulmonary dr.i have no medical coverage. Im going to be 54 years old am 4'10 and only weigh 82 pounds and am loosing weight. I took a leave from work because Im having trouble functioning. It feels like my blood pressure is dropping while walking along with major shortness of breath Doctor: Hi welcome to health care magic...Noted you are having COPD ....There are no permanent treatment for this....For breathlessness bronchodilator through metered dose inhaler can be given....If dyspnea more lead acute attack then levosalbutamol can be given through nebulization....Avoid respiratory irritant like smoking , environmental pollution , dust exposure to less.....Here if long term duration of COPD then pulmonary hypertension induce right side heart failure that is cor pulmonale has to be ruled out.....ECG or echo useful...Take care....Dr.Parth"
},
{
"id": 67945,
"tgt": "How can a painful back lump be treated?",
"src": "Patient: so my husband had a lump on his (mid)back and i massaged it quite aggressively 2 days ago....larger mass area that moves around but having sharp stabbing pain. ibuprofen allows some relief but comes back after med level drops. No fever, area not red. Has appt friday but he is quite uncomfortable. Doctor: Hi. It could be a lipoma or a lump containing fat cells. It usually does not require removal but if it is painful then you consider removal or Excision by a minor Surgical procedure. Feel free to ask any further questions if there are. Take care. Dr Rishi, New Delhi, India."
},
{
"id": 185752,
"tgt": "What to do if am having toothache since getting filling done a year ago?",
"src": "Patient: I had fillings done a year back but the toothache never went away. When I showed it elsewhere I was told the pressure was building up so they advised to take the filling out. They took the filling out & filled it temporily to see whether their initial diagnosis was correct. By this time I got in touch with my Dentist (who had done the filling earlier) and was told that if it didnt trouble me much I could wait till I could come home. (I am living abroad.) I am not using the side where the filling was done. There are days when the pain becomes too intense. My question is (1) Is it ok to prolong for sometime till I see my dentist? (2) Is there any tablet that will numb the toothache? Doctor: Hello!Welcome to HCM.You can prolong the treatment,but there is always a risk of pain,swelling which can shoot up.You can temporarily be on a course of 5 day antibiotics and analgesics (amoxicillin500 and ibuprofen) to subside the infection.It is acceptable if the waiting is for a couple of weeks to months,but not as long as a year.There is no tablet or medication which can numb the tooth.Pain in the tooth after a filling suggests there is irreversible pulpitis. You must get a root canal done.As you mentioned you had a temporary filling done, i suggest you to wait to come home to get the treatment if it is few weeks away.Regards."
},
{
"id": 95994,
"tgt": "I have pain in left side of stomach",
"src": "Patient: 10 days have passed after the completion of typhoid+malaria treatment and now I have pain in left side of stomach. Today I noticed that I had pain after I ate oily food (but I am not sure that it happens only after eating oily food) I don t have pain when I press that part of stomach. The pain subsided after 10 mins. It was a mild pain and it happens irregularly,like once in 1-2 days. I also have mild fever, a wave of chill and weakness irregularly. Can these all symptoms (including fever and mild chills) be due to enlarged spleen? Thank you for reading my query. Doctor: Hi,thanks for your query.Presence of fever even after complete course of medicines are over needs careful evaluation.You may need some investigation like ultrasound etc.to come to a conclusion.Please talk to your doctor.Either it could be gastritis that is drug induced.Take light diet,more of liquids.Please get in touch with your doctor.bye and take care."
},
{
"id": 100122,
"tgt": "Does blisters on face & elbow with hyper nature suggest allergic reaction?",
"src": "Patient: My son had an endoscopy yesterday. They are checking for celiac disease. He developed these blisters on his right elbow and a few on his face. The ones on his face are gone but the ones on his elbow are still there and they look just like fever blisters. He is also acting pretty hyper. This is how he usually gets when he's had something he is allergic to. Should I be concerned? Doctor: HI, thanks for using healthcare magicIf these are the symptoms that you would normally observe when he has an allergic response then you may want to consider giving him an anti histamine.These are available over the counter.Eg benadryl, , zyrtec, claritine, allegraI hope this helps"
},
{
"id": 100112,
"tgt": "How to treat internal fever?",
"src": "Patient: Hi,am 28 man,I sufferd from internal fever from a year that causes me allergy in the eyes, feel high temperature without appearing in thermometer and every time I go to the doctors they give me deffrent medicines and I geting better then afte finish the treatment the fever returns. So please tell me what I have to do?...Thanks Doctor: HIWell come to HCMCore temperature is main reading as long as fever is concern if this reads normal then no need to worry, least concern about the surface temperature of skin, even if this is high, some time because of some stress one can feel malaise, and sick but actually this is not due to some disease but this may be functional condition, avoid the bizarre thoughts, have a nice sleep, try to keep the stress level very low, everything would be fine soon, take care."
},
{
"id": 218106,
"tgt": "Could increase in deep breathing cause chest pain?",
"src": "Patient: Hi, I am noticing chest pains when I take deep breaths only. I have increased the amount of time I deep breathe as I am doing yoga and my doctor encouraged me to deep breathe as well. Could this increase in deep breathing be causing the pain in my chest? Thank You, Maureen Doctor: Hi Maureen, Thanks for your query. Deep breathing does not cause chest pain under normal circumstances. It however, occurs when one or more of the chest muscle(s) go into spasm ('catch' or 'tightening'). A short course of an anti-inflammatory drug (such as ibuprofen) and a muscle relaxant (such as, Chlorzoxazone) will relax the affected muscle, following which, pain during deep inspiration will be relieved. Consult your doctor and apprise him of my opinion. I am certain that he will agree with me and will prescribe the advised drugs in appropriate doses. If you find my response helpful and informative, do not forget an \u201cexcellent\u201d (5-star rating) to my answer, to ENCOURAGE ALL doctors- engaged in social service- to render sound advice to the FREE queries. Take care Dr. Rakesh Karanwal"
},
{
"id": 82450,
"tgt": "What does swooshing sound in upper lung after bone marrow mean?",
"src": "Patient: my daughter had a bone marrow done recently and she also has muscular dystrophy. i think when they put her under,they might have damaged her lung/bronchial area. I was checking her lungs when she got home and the right side of her chest/lungs were clear. when i got to the left side, bottom lobe was clear, but the upper lobe had a swooshing sound at times. What does this mean? please help Doctor: Thanks for your question on HCM.Bone marrow aspiration from sternum may cause complications lije damage to heart and lungs.So possibility of pneumothorax or internal damage can be theAnd the sound you are decribing is seen in pneumpneumothorax. So better to get done chest x ray first to rule out pneumothorax. If this is normal than no need to worry."
},
{
"id": 121752,
"tgt": "What causes tingling around mole on the back of shoulder blade?",
"src": "Patient: Hi, I have a mole on the back of my shoulder blade which is quite raised. its never caused any problems before but the last week I have noticed it has had a tingly feeling all around it coming and going throughout the day. Is this anything to be worried about? Doctor: Hello, Your symptoms could be related to local inflammation or a lipoma. For this reason, I recommend consulting with your attending physician for a physical exam and performing some blood lab tests (complete blood count, PCR, ESR). Hope I have answered your query. Let me know if I can assist you further. Take care Regards, Dr Ilir Sharka, Cardiologist"
},
{
"id": 120836,
"tgt": "Suggest remedy for pain in knee",
"src": "Patient: I fell on my knee hard about 15 years ago, I was sore and in pain for two weeks, then it went away, I could jog and ride a bike, then 10 years later I was playing volley ball, the pain came back for one week, then went away. One year later I was swing dancing when my partner turn me really quick that I felt my knee pop, I was in pain for a whole week. I don t play volleyball but still dance but just careful how I turn. I continue to jog and ride a bike, until recently back in September 2012, I was perparing for a party which caused me to be on my feet for three days in row, after the party the next day, my pain came back for a couple of weeks, then went a way, then two weeks ago I was squatting for along time doing chores, it was fine, until the next day, I was in pain, I m still in pain.but I still did some bike exercise. Now it seem harder to walk, bend, sit at my desk for along period of time. The pain is all over the leg beside the knee, expecially the thigh. Do I need to go to the doctor and get an x-ray, I ve never been to the doctor about my knee. I know now not to squat, play volleyball. I dance carefully too. If the pain goes away, do I just take care of my knee more carefully. Do I just need bedrest for a week. I feel now it take longer to heal. What do I do now? Thank you. Doctor: Hello,I have read complete details. Looking at your details it looks that most probably you may have a meniscus or ligament problem.In this case only having a X-ray will be not sufficient. You should have MRI of your knee. At present you should do following to prevent further injury:- Use a knee cap over the knee- Avoid turning on your knee. Avoid running, squatting, dancing and putting pressure over knee.Clinical examination and MRI result will further decide next course of treatment. Hope I have answered your question. Let me know if I can assist you further. Regards, Dr. Mukesh Tiwari, Orthopedic Surgeon"
},
{
"id": 49553,
"tgt": "Feel sick after quitting smoking, BP 140/100, slight kidney disease?",
"src": "Patient: Im 6 foot 4 330 pounds and recently quit smoking , since i stopped I feel worse, I went to emergency room and blood pressure was 140/100 they said i have slight kidney disease.. How can it be called slight Doctor: HiThanks for your query.We need some more information before we can determine the severity of your kidney problem.You need to do the following tests, if not already done:1) Urine R/M and UPCR2) Blood tests: urea, creatinine3) Ultrasound of the kidneysBased on the results we can offer you more suggestions.Hope this helpsGood luck."
},
{
"id": 198839,
"tgt": "Should i be worried of cut in genitals after massage?",
"src": "Patient: Went to a thai massage parlour. Got massage on genitals. While massaging near groin, got a cut as long as 2cm on skin. She used massage oil. She did repeeatedly massage the wound area and had fingernail contact on it few times again. Is there anything to worry about? Doctor: Hi there and thank you for your question.I appreciate your concern regarding this matter and I'll try to help where I can.If one sustains a superficial scratch on the skin, then you should look out for signs of infection at around the third day after the injury.These signs include redness around the site of the wound, pus discharging from the wound itself or generalized fever experienced by the patient. If you start to notice these signs, then I would suggest that you see your doctor as soon as you can so that they may prescribe the appropriate treatment for this issue.In order to prevent an infection from occurring you can continue with the following care. You can clean the wound with a saline solution (salt water) 3 times a day until the wound has made a scab over it. You can then let the area granulate on its own.I hope this helps and good luck."
},
{
"id": 201087,
"tgt": "Suggest treatment for bleeding on pulling back foreskin",
"src": "Patient: Hi, a few days ago, i was having sex with my girlfriend. During foreplay, when she pulled my foreskin back, she did it a bit too rough and broke my frenulum. It started bleeding, and eventually the bleeding stopped. I tried washing the area but it there was a sharp pain when i tried to wash it. However, the pain got a lot better today. I also noticed that there was something that might ve been part of my frenulum still on the head of my penis, and I think there are a lot of nerves there since it is the same pain feeling when i Touch it. Is everything ok? Do i need to get circumcised? And will my frenulum grow back? Doctor: Hi,it seems that you got your frenulum injured and bled.Due to rough pulling of fore skin gave this problem.If foreskin completely pushed back on glans then you might not require circumcision.At present there might be having some soreness on the part due to infection.Apply antibiotic cream locally for few days.Frenulum will be alright after infection subsides.Ok and take care."
},
{
"id": 97122,
"tgt": "Is experiencing weakness and shortness of breath part of recovery after car accident?",
"src": "Patient: I had a car accident on December 6th 2011 and I am experiencing weakness and short of breath, middle of my back all the way to the front rib cage hurts. CT scan of the head came back ok. No CTscans of my chest or back taken. X rays taken and of my lumbar and cervical came back ok too. So should I continue to seek medical attention or is it part of the recovery process.? Thank you for your time, Becky Doctor: hello, thanks for your query, , chest pain, breathless ness on exertion associated with weakness not associated with cough could be due to some deficiencies like iron, anaemia or calcium or electrolytes. I don't think 2011 injury a cause for it. visit an ent surgeon do blood test to rule out any deficiencies.all d best. take care."
},
{
"id": 152807,
"tgt": "Can hyperplasia patient take hcg diet?",
"src": "Patient: Hi, I had an Hyperplasia complex with focal atypia regressed 2 years ago , I want to do the hcg diet and my doctor told me I can t , before he told me the only way I will not have the hyperplasia coming back is 1- need to loose weight 2- taking progesterone pills 3- getting pregnant and my question is while a women is pregnant she developes thousands of ius of the hcg hormone why I can t have much less ius of the hormone to do the diet???? Doctor: hi,I am not sure how this site allows doctors to see reports, Because i see no options to see any reportsHowever , it appears that you want to loose weight and is inclined to use hcg diet for the same. However the question first arise is \"wether hcg diet is effective\". This type of diet is often prescribed with low calorie which is the real cause of wight loss achieved. As far as your question goes, exogenous hcg is never proved to be free from side effects. You pituitary gonadal axis may get suppressed though this effect is more pronounced in males. Moreover it should be avoided in hormone dependent cancers like breast and prostateI know that you are neither male nor breast cancer case, but i strongly suggest against taking exogenous hcg for weight loss, which can be achieved with planned diet and exercise program"
},
{
"id": 131793,
"tgt": "What does marginal osteophytes at multiple levels indicate?",
"src": "Patient: 3mm of retrolisthesis of L4 back on L5. Marginal osteophytes at multiple levels and the largest is seen anterior to space at l4-l5. Degenerative changes posterior elements L4-l5 and L5-s1. Mild sclerosis about the left mid s1. Marginal osteophytes at every level of the LS spine. Focal sclerosis left mid S1. Hips: superior acetabular osteophyte. Small sclerotic focus, right intertrochanteric region. Mild degenerative changes at the hips. note: calcium depost caused stress fracture in 1990. Will this just get worse or should I start thinking of partial disability. PT. 55yo male works construction. Injuries were between 1980-82 while in military. Unemployed at this time. Pain in hips moderate to extreme x20+ years now. Back off and on since 1982. Doctor: Hi welcome to HCM, what you have is lumbar spondylitis. Osteophytes are small bony growths in the spine that develop as part of spondylitis. Retrolisthesis is the spinal matter prolapsing backwards, sclerosis means hardening of the spinal soft tissues. Generally all this points to degenerative changes probably set off by the fracture you sustained. Yes construction work can worsen the problem. And you can consider partial disbility. Regards"
},
{
"id": 215946,
"tgt": "Is Hydrocodone safe to be taken while on Tramadol?",
"src": "Patient: I recently began taking Hydro, 5-325mg, for pain. My question is I am already taking Tramadol, 50mg, was supplimenting it w/Tylenol, but not getting much relief. I have stopped the Tylenol, of course, but is it ok to take the Hydro & Tramadol together? Alone neither of them help much. Doctor: Hello and Welcome to \u2018Ask A Doctor\u2019 service. I have reviewed your query and here is my advice. You can take Hydrocodone and Tramadol together. They won\u2019t interact with each other as it doesn\u2019t produce any adverse reactions. Keep in mind that you may feel excessive sleepiness while combining these two drugs. Wishing you good health. Thanks."
},
{
"id": 214007,
"tgt": "May any damage occur due to cancellation intake of Cetalopram 20mg ?",
"src": "Patient: I have been taking Cetalopram 20 mg a day for almost two years. At the moment I am living in a country where this kind of medicine is not regularly available. It will also not be possible for me to legally receive it from another country. What shall I do ? May any damage occur due to cancellation of its intake ? Doctor: Hi Vadim, Citalopram belongs to SSRI group of antidepressants. If you have tolerated and benifitted from it for two years and now are unable to get it, then first evaluate the need to continue it. If your psychiatrist feels you still need an antidepressant two years after being well, then ask him if he could change it to any other available SSRI type antidepressants. This may help as all of them act in the same way. If stopping it is thought to be the best option to take, DO NOT STOP SUDDENLY. You will have to keep reducing some fixed dose of medication periodically and then stop. Yes, if you stop suddenly, it will cause something called as withdrawal reaction/problem as it is short acting unlike long acting antidepressant like Prozac/fluoxetene. Hope this helps."
},
{
"id": 171880,
"tgt": "How to make my baby to eat?",
"src": "Patient: Hello doctor, i had a 2 year old boy, his problem is that , when i am giving food to him he is keeping in mouth only,not swallow and also he didnt chewing. He had no interest in taking food, milk or anything. How can i change the condition of my baby Pleas reply doctor Doctor: Hi...Thank you for consulting in Health Care magic.I take this opportunity to tell you certain scientific facts and relieve your distress - 1. An active is a well kid even if she/ he is not eating well. 2. Development of a kid is as important as or I would say even more important than growth alone.3. As the kids grow their interest in food decreases and interest in play activities increases so that they eat small quantity and run away to play. As parents we need to be more patient and feed than less quantity but more aliquots per day.4. This age rather than the quantity of the food I suggest you concentrate more on the number of times you feed her and also make whatever you feed her calorie dense by adding good amount of ghee to it.5. I suggest you not to use appetite stimulants on long run as they may cause drowsiness.PLEASE DO NOT GIVE ANY SO CALLED APPETITE STIMULANTS AS THEY ARE NOT VERY SCIENTIFICALLY PROVEN AND THEY HAVE THE SIDE EFFECT OF EXCESSIVE DROWSINESS.Hope my answer was helpful for you. I am happy to help any time. Further clarifications and consultations on Health care magic are welcome. If you do not have any clarifications, you can close the discussion and rate the answer. Wish your kid good health.Dr. Sumanth MBBS., DCH., DNB (Paed).,"
},
{
"id": 167418,
"tgt": "What causes chest pain and pressure?",
"src": "Patient: hi, mi kid has chest pain and pressure, he was at his bed playing to his cell and the he feel a pressure in the chest and some difficult to breath, he is shaking a little and sweating, his in a good mood, actually he are smiling, no blurry vision, no headache, no pain in the stomach, no dizziness and walk normal, he feel a little scary about the pressure and now can t go to sleep, can you tell me if this is a emergency and need to be carried to an hospital . thanks Doctor: may be only myalgia chest. there is no history of cough. Needs chest support by elastic bandage. To be reviewed after about a week."
},
{
"id": 61215,
"tgt": "What causes a lump in the neck under the jaw?",
"src": "Patient: Hi. I am a 21 year old male and am having a problem in my neck on the left side right under my jaw and kind of near the pulse. I have found a lump and am worried. Things feel swollen kind of. I haven't been eating much and have been sleeping about 14 hours a day. I know this sounds like cancer symptoms but am hoping its not that serious. Any help would be much appreciated Doctor: HiThanks for the query..As per your explanation of symptoms and location of the swelling as you told that it is located right under the jaw shows that the lump is due to swelling in the sub mandibular salivary gland which is located just below the lower jaw..The swelling can be due to obstruction in the ducts of the salivary gland due to formation of crystalline stones in the ducts of the gland and they are formed from the minerals present in the saliva.It leads to back flow of saliva into the gland leading to swelling..It can also be due to formation of cysts or tumours into the glands.It can also be due to infection in the gland but an infection most commonly causes painful and tender swelling..If there is already any infection in any part of the mouth, face or any other body part and if the swelling is round and mobile can be mostly a lymph node swelling..YOU NEED NOT TO GET PANIC AS IT IS MOST COMMONLY A NON CANCEROUS SWELLING..I would suggest you to consult an Oral pathologist and get evaluated for the exact cause of the swelling and get treated..Obstruction in the ducts can be cleared by locating and removing the stones either by palpating and pushing the stone towards canal opening and in case of large stones a small incision placed inside mouth and stone is removed..Infections in gland or lymph nodes can be treated by antibiotics..Hope this helps..Regards.."
},
{
"id": 14792,
"tgt": "Suggest medication for rashes and swelling",
"src": "Patient: Hi,I have rashes and swelling came today and i feeling itching. Rashes will come whenever i take alcohol and otherwise it will not come. Can you suggest a medicine to cure these rashes and swellings. Also please let me know why rashes are coming whenever i drink.Thanks,Surya Doctor: Hello and welcome to healthcaremagicI would keep a possibility of Acute Urticaria. It presents as itching and skin welts/swellings, which can be distributed all over the body.Alcohol is one of the well known causes of urticaria.Urticaria OR hives can occur due to various causes like(to name a few):--Infections(bacterial , viral, fungal) --Drugs(Pain killers/ antibiotics etc) --Alcohol Intake--Certain foods and food additives/preservatives --Physical Urticaria: Cold temperature, Heat, Pressure(Prolonged Standing, Sitting), Dermaographism etc. --Cholinergic Urticaria: in response to strong emotions, exercise, spicy food etc --Autoimmune disorders like SLE, RA etc. --A few cases have no underlying cause and are known as idiopathic urticaria. An oral antihistamine like Fexofenadine (Allegra 180 mg tab) OR Loratadine (Alaspan 10mg tablet) once or twice daily for a week would be suitable. They are non-sedating and wont affect your alertness level. A topical soothing lotion like calamine would help you symptomatically. take care"
},
{
"id": 81040,
"tgt": "What is the treatment for pneumonia?",
"src": "Patient: My dad is 79 and in February was admitted to hospital with Pneumonia. Unfortunately he had prior for many years had problems with swallowing and choking. He now has a gastrostomy tube and is nil by mouth as his Epiglottis is not working correctly. Is there anything that can be done we don t seem to be getting any answers Doctor: Thanks for your question on HCM.I can understand your situation and problem.In my opinion, your father is having aspiration pneumonia mostly.He is having following risk factors for aspiration.1. Old age2. Difficulty in swallowing3. History of choking4. Non functioning epiglottisAnaerobic infection is most common cause for Aspiration pneumonia. So better to get done sputum culture and sensitivity to identify causative organism and start appropriate antibiotic therapy.Just make sure that his antibiotic regimen contain anaerobic coverage.So discuss all these with his treating doctor and start proper treatment."
},
{
"id": 56971,
"tgt": "Suggest treatment to lower bilirubin count",
"src": "Patient: Hi. My brother has been detected with Hep A and is admitted to Manipal Hospital, Bangalore. His bilirubin count is not coming down and is at 30 and he is getting scratch all over his body. MRCP was done and everythg is normal. Docs are a bit confused and have suggested endoscopy. Pls advice. Doctor: Hello,Can you please tell when did he acquire the infection( when did he get it), because it takes time for the virus to come down, initially It will go up and later it will stay in that phase and then comes down, some people may also develop itching , but if MRCP is normal, nothing to worry, please follow your doctors advise, keep repeating Liver function tests every week, if they are in a down trend than nothing to be worried, otherwise he has to be evaluated for other causes of hepatitis also. Hope I could help you.Thank you."
},
{
"id": 83939,
"tgt": "What are the side effects of flucloxacillin?",
"src": "Patient: hi , i have been taking flucloxacillin for a infection and yesterday was the 3rd day of taking them but i started vomiting yesterday all day from 5am till about 6pm i felt really ill and was really red in the face also i was feeling cold but my skin was hot should i stop taking these tablets Doctor: Hi,Nausea, vomiting, abdominal bloating, allergic rashes can occur commonly with flucloxacillin and other penicillin derivative drugs. You can stop the drug and use alternative drugs like erythromycin if it is effective for your infection.Visit your physician for further management.Hope I have answered your question. Let me know if I can assist you further. Regards, Dr. Saranya Ramadoss, General and Family Physician"
},
{
"id": 704,
"tgt": "What causes difficulty in getting pregnant?",
"src": "Patient: hello, my husband and i are trying to get pregnant, but we hhello, my husband and i are trying to get pregnant, but we have been trying for seven months. i have never been on the pill, or anything of the sort, but it seems like the sperm come out of me too soon. could these be the problem. i am only 20 yrs old and he is 21. Doctor: Hi, Thanks for the query. I understand your concern. 1. Semen does come out after some time of sex. If you think it'early... try to lie down for half an hour ,with a pillow under buttocks. This would retain it longer. 2.pregnancy is possible only around ovulation (which is 14 days before the expected date of menses). Try to have sex 17th to 11th day before the next period date. 3. If no +veresult in one year... consult a gynecologist. Thanks."
},
{
"id": 195267,
"tgt": "What are the side effects of masturbation?",
"src": "Patient: This is selvam, Age29, single. I am doing masturbation from last 19 years still doing weekly once or twice. I can\u2019t control. my Penis is string and very thin. It normally below 2 inch. When erected 11cm. If I do the masturbation today, the next one and two days I get pain in right and left side of my lower stomach when I pull or push the stomach. i went to toilet or urine I relief that pain. Some time after pass the urine I got very much pain just 5 to 6 sec.. at between two legs under the Urine part. This pain like surrounding the urine part nerves pull in to one point Please advice what type of this problem? Whom to consult like urologist, sexiyalogist,\u2026 Doctor: Dear Selvam,Hello and Welcome to \u2018Ask A Doctor\u2019 service. I have reviewed your query and here is my advice. First of all I like to inform you that masturbation is completely harmless activity and not associated with any form of ill or side effects. The other part of query indicates that you are probably dealing with urinary tract infections which is best dealt by urologists. Hope I have answered your query. Let me know if I can assist you further."
},
{
"id": 82208,
"tgt": "What causes chest pain and dizziness with all normal reports?",
"src": "Patient: Hi Doctor, I had chest pain and not able to breath properly so thinking something is wring with my heart...I had ECG,EKG,Spirometry and Full blood count tests.... All were normal and there is 0.2 grms bad cholestrol in litre Blood. apart from all were normal... thes tests were done 2 months back ..now again i have breathlessness and some tightness in chest on anf off ...so had chest xray where in the doctor said that was normal... when ever i had breathing problem i used to take prednisolne tablet and it was normal... today i felt heart palpation and heart rate very fast and dizziness is there which is not going from past 3 days after taking stugeron tablet also... i have tingling sensation in my nerves. what is it ...is it related to heart ? Doctor: Hi...this type of problems may be caused by a lots of diseases. R U a smoker? what is ur PFT report? U first do a Treadmill test for excluding cardiac angina. Tell me ur PFT reports. U better consult a cardiologist. Don't take prednisolone on ur own...it may causes lots of side-effects."
},
{
"id": 96680,
"tgt": "Suggest remedy for constant bleeding from little finger due to injury",
"src": "Patient: Hi, my mother shut her little finger in her car door by mistake yesterday and since then it has not stopped bleeding. The nail has split near the bottom of her finger and she said it came right off when the accident happened but she has pushed it back down. Yesterday I made her hold her hand above her heart on a load of pillows and a pack of frozen peas for a couple of hours before bed time which helped the blood start thicken but it has never stopped bleeding yet. Doctor: Take it serious normally blood should clot with in 10 min, u r saying it not clotted for long time, do investigation , ct , pt , aptt, bt, platelet count..dont neglect...there is some abnormal"
},
{
"id": 34184,
"tgt": "Suggest care and diet for klebsiella oxytoca infection",
"src": "Patient: My Wife(age 28) is suffering from KLEBSIELLA OXYTOCA INFECTION she is already admitted to hospital and doc giving her an antibiotics. let me know the necessary care needs to take of her what she can eat and what not. she is right now suffering from loose motion & cough. Doctor: There are no diet restrictions with pneumonia, but her diet should be plenty of fluids (i.e soup) and drinks water 8-10 glasses dailly and she should eat fruits.Hope she get well ASAP!"
},
{
"id": 129077,
"tgt": "How to reduce muscle pain after forceful chiropractic adjustments to get the rib re-located?",
"src": "Patient: I had some forceful chiropractic adjustments to get my rib re-located (somewhere around T-6 through T-10) on the left side, as well as on my neck (the bones at the bottom of my skull). My body did not respond well to these and I have been having muscle spasms/joint problems ever since - it feels like I ve been in a car accident and got whiplash & some other issues. Any ideas for getting my body back to normal (permanently)? For temporary relief, I ve tried acupuncture, massage (deep tissue & Swedish), heat/ice, swimming/walking/stretches, and anti-inflammatories. Thanks for any tips you might have! Doctor: thanks for your question ,First of all you have to do new x rays for your ribs and painful joints to asses any bony problem .You have to reduce physical and mental stress.Then you should recieve muscle relaxant (sirdalud 4 mg daily before sleep)..NSAID(Meloxicam 15 mg once daily ) in addition to warm fomentations for 3 weeks at least."
},
{
"id": 100391,
"tgt": "Suggest medication for sore throat,cough and stuffy nose",
"src": "Patient: I was at my grandmothers this weekend and the air in her house is really dry and I ended up getting a sore throat and a really stuffy nose. I have a bad cough and it s all been going on for 2 1/2 days. I don t have a temp so my parents don t believe I m sick. But I really just don t feel well and there s a nasty taste in the back of my throat I always get when I m sick and it won t go away. Doctor: HelloAll the symptoms ,which mentioned in your query ( sore throat , stuffy nose, cough , nasty taste )are an indication of allergic rhinitis . Allergens , as you mentioned are , in the house of your grandmother . When such type of patient visit I prescribe them to take montelukast+levoceterizine tablet once or twice in day . I also advise my patient to take AZITHROMYCIN 500 mg once in day for 3 days . Hot saline gargles help to clear from throat .But some times patient feels bronchoconstriction so take montelukast+bambuterol tablet once or twice in day .You will get relief within 2-3 days .Good luck."
},
{
"id": 131234,
"tgt": "What could cause bruising all over legs, cramping in foot and knee with weightloss?",
"src": "Patient: I have been getting bruises all over my legs for the past few months for no reason and a lot of cramping in my foot and knee, I also have been loosing a lot of weight for no reason and petechia which at first I thought was hives or something, could this all be due to my anemia or something else, I am just concerned cause it s been progressing over the year Doctor: According to your symptoms, you should do some lab investigation to detect the excactly cause like CBC, coagulation profile tests, fasting blood sugar, serum insulin test, thyroid function tests. Loosing of body weight is serious symptoms of many reasons. You should do investigations quickly and i hope to follow up you."
},
{
"id": 63733,
"tgt": "What is the treatment for a lump in the breast?",
"src": "Patient: Yesterday I went to the Doctor to check a lump I found on my breast. She said it felt soft and movable and would like to check it in 2 weeks. I now have red flat blotch with a center on it on my skin but I am not sure if you would say it was on my breast or my chest. I have very small breasts and not sure if to be concerned about this lesion. I hope I have dedscribed this ok for you. Thank you Doctor: Thank you for asking Healthcare majic. My name is Dr Ehsan Ullah & I have gone through your query.you have mentioned that you had lump at breast and consulted a Dr.Depending upon your age and family history , i can be of variable nature...nowyou have Noticed red flat blotch on breast/chest which you, yourself are not pretty sure ...you can do self examination by standing in front of mirror and check it andIdeally. all such conditions which are Not usual to you and appear in the breast, need timely assessment by your physician or at Breast Clinic.Hope this may help you. Let me know if anything not clear. Thanks."
},
{
"id": 30992,
"tgt": "What are the symptoms of potential swine flu?",
"src": "Patient: Hi, may I answer your health queries right now ? Please type your query here...my brother is suffering from fever ,cough and pain in throat since 3-4 days.he is sneezing too...all his test for malaria and viral are negative.should we go 4 swine flu test?? Doctor: HiThis could be flu and he needs testing for that, but it could also be a bacterial bronchitis or even pneumonia so he also needs a chest x-ray and a sputum culture to see if he needs a course of antibiotics"
},
{
"id": 13707,
"tgt": "How can whole body rashes with blistering, itching and lumps be treated?",
"src": "Patient: Hi I have had a rash which started around four weeks ago originally thought they where bites started on my back travelled to legs and arms stomach an chest doc first of all said chicken pox then said scabies used treatment for this and stil not cured not as bad as it first was jus getting random ones here and there instead of groups I got one on sole of my foot an two in between my toes which have turned into what look like water blisters and the palms of my hands are itchy and have come out in tiny lumps any idea of what this is Doctor: Hi, It could be eczematous dermatitis, specifically pompholyx or dyshidrotic eczema. This condition usually presents as itchy, grouped, vesicles which may coalesce to form blisters. The fingers of hands and toes are favoured sites. I suggest you to use a potent topical steroid and antibiotic combination e.g clobetasol propionate and neomycin combination cream, twice daily for a few days. In addition, I suggest you take an oral antihistamine e.g Tab cetirizine 10mg once a day for symptomatic relief from itching. Hope I have answered your query. Let me know if I can assist you further."
},
{
"id": 179674,
"tgt": "Suggest treatment for severe cough in kid",
"src": "Patient: DEAR DOCTOR, MY SON IS OF 4 YEARS, WEIGHT 15.9KG, HE IS HAVING PROBLEM OF COUGH ( DRY / WET ) SINCE LAST 2 YEARS, SOME DOCTORS SAYS ITS BECAUSE OF HIS LOW HAEMOGLOBIN/RBC COUNT WHICH AS PER THE LAST TEST REPORT IS 9. NOW THE DOCTOR HAD RECOMMENDED THESE MEDICINES , SYP SOVENTUS JR. ( 5 ML 3 TIMES A DAY ) , SYP CLAMP ( 7.5ML 2 TIMES A DAY ), SYP MAXTRA ( 5ML 3 TIMES A DAY ), AND TOLD US TO NEBULISE HIM WITH 1ML OF BUDECORT REP WITH 1 ML OF ns. 3 times a day for 3 days, then to withdraw it with 2 doses and then with once a day. Pls suggest. The Chest X ray was very clear. Thanks and Regards Vaishnaw N..A. 0000 / YYYY@YYYY Doctor: Hellowelcome to health care magic , Based on your child's history and treatment received most likely reason reason for his cough seems to be allergic (reactive airway disease) , chances of which are more if there is family history of allergic also . If your child responds immediately to nebulisation , chances of allergic cough are very much. Inhalation therapy are best recommended treatment for such type of cough, also you need to check your immunization , apart form routine vaccines he needs yearly flu vaccination. As these are prescriptional drugs you need to consult your pediatrician to take a course of these drugs as per the severity of illness.Hope I answered your queryThank U"
},
{
"id": 186554,
"tgt": "Could voice be affected from having abscessed tooth extracted?",
"src": "Patient: I had an absecessed tooth extracted Wed. Not much pain. Thursday I started getting hoarse and by Friday I could barely whisper. Have not talked at all today. Have been on antibiotics for 10 days now. Generally, feel tired all over. Should I be worried? Doctor: thanks for you query, i have gone through your query. there is no relationship between the hoarse voice and the tooth removal. this can be due to throat infection. consult your ENT surgeon for opinion and needful. do saline gargling. i hope my answer will help you. take care."
},
{
"id": 56969,
"tgt": "Is drinking a cause for hepatitis?",
"src": "Patient: Hi thereI have been feeling incredibly nauseous for 3 weeks now. I have no appetite, and I have always had a very healthy appetite. I am tired and particularly so after even mild physical exertion.I saw a B.E.S.T. practitioner who detected Hepatitis virus in my liver and also told me that my gall bladder is also stressed.After that, went to see my doc who put my symptoms down to Giardiasis and prescribed Fasigyn. My nausea has improved a lot over the last 3 days since taking the Fasigyn but it now come and goes and is not constant or so intense.What concerns me about the 'suspected' hepatitis is that thinking back over the last few months, I have always suffered terrible hangovers after even just 1 glass of wine. I was reminded of this last night when after about a week of no wine at all I had a glass... And woke up (and spent the entire day) feeling awfully 'hungover'.Any advice or suggestions would be helpful.Ps. I am awaiting an appointment with a gastroenterologist.Thanks :) Doctor: Hello, Yes alcohol consumption is one of the cause for hepatitis, if it is taken daily,in high quantities , and if you are taking only wine and that too not regularly or daily, the chances are lesser for you to have Hepatitis because of alcohol, please consult your Gastroenterologist and please get evaluated for other causes of hepatitis, I.e if you still have hepatitis. Hope I could help you THANK YOU."
},
{
"id": 162449,
"tgt": "How can pressure in the chest and dizziness be treated?",
"src": "Patient: My daughter is 15 and suffering from chest pressure/tightness and dizziness. We have seen a peds cardiologist and EKG showed an AV block and mitrovalve regurgitate, we are trying to figure out what is causing this. They have started her on a salt/potassium supplement and lots of water. She is currently wearing a heart monitor. BUT, what do we do for her symptoms in the meantime, she\u2019s miserable! How serious is this?! Doctor: Hi, I am sorry your daughter is suffering so. I am sure it is difficult for you to help her. As a cardiologist the physician she has been seen by specializes in conditions that affect the heart. As chest symptoms and dizziness can be due to heart problems, the heart specialist is evaluating your adolescent for these possibilities. An EKG will show AV block and a cardiac arrhythmia. An ultrasound of the heart will find mitral valve prolapse(or regurgitation). Not all young people with AV block and mitral valve regurgitation have chest pain or discomfort. The heart monitor will look for a cardiac arrhythmia over a much longer period of time that the few seconds it takes to perform an EKG. Salt supplementation and extra fluids are treatments for patients who faint and are felt to be dehydrated. Other possibilities of dizziness: -anaemia -drugs(SOME antibiotics), alcohol, and aspirin -heat exhaustion. Still, another condition is hyperventilation syndrome. This usually occurs during adolescence, for both boys and girls. Symptoms of hyperventilation syndrome include dizziness, headache, chest pain, fast breathing, difficulty breathing, fainting, along with panic attacks. Panic attacks are sudden episodes of fear, screaming, weeping, hyperventilation, chest pain, dizziness, and memory difficulties. Attacks are about 20 minutes in duration. Hope I have answered your query. Let me know if I can assist you further. Regards, Dr Arnold Zedd, Pediatrician"
},
{
"id": 42849,
"tgt": "Give suggestions to conceive quickly",
"src": "Patient: Hi i am geetha my native place is in chennai I am staying with my husband in pune . He is working in pune . I am undergoing treatment in chennaiDr advised me that my uterus size is reducing it is not increasing (she said for egg donor is best for you)what i have to do please help me to get conceive . Doctor: Hi,Thanks for writing to HCM. I would like to tell you about some facts regarding infertility. The first thing to conceive you need healthy eggs which ovulates normally. And also you should get your husband semen analysis before starting your treatment. If uterus size is less then your uterus is not adequate to hold the pregnancy. For that you need to go for donor womb called as surrogacy . You dont need donor egg if your are ovulating normally.The order of invesigations includemale semen analysisfemale -ovulation study Tube patency Endometrial analysisTreatment depends on what problem is there. For small uterus surrogacy is used.For ovulation defects donor egg is used.Hope I have been helpfull to you.RegardsDr.Deepika Patil"
},
{
"id": 59159,
"tgt": "In coma due to cirrhosis and hepatitis. Bleeding in the esophagus. High toxin levels in brain. Recovery?",
"src": "Patient: My 51 yr old Brother has been in a coma since Friday Night, It is from Cirrohosis & Hepatitis, His Band broke that had his esophogus from bleeding, So was never hooked up to any type of Life Support Machine, Just a Morphine Drip, His Brain Toxin Level is almost 300 when the dr. said normal is 40..How much longer do I have to see him like this..& Is there anything the dr. can do to help things along..I'm Sorry IDK It cost to use this service..I am having a hard time..esp. having to try & bury my brother due to no insurance.. Doctor: Hi, Thanks for writing in HCM. i cannot imagine the stress and turmoil through which you are passing through. Your brother is in a critical condition. But nevertheless do not lose hope. Aggressive medical treatment can reverse the condition provided the liver reserves are a bit promising.As there is a combination of cirrhosis and hepatitis the liver status would be very poor. And if he has bled the condition would be critical. Surgical intervention is ruled out as he is on life support. The doctor is left with the only option of medical treatment followed by a wait and watch policy. If the patient comes out of life support then other possibilities for treatment could be explored. Regards"
},
{
"id": 97962,
"tgt": "Is there any ayurvedic or unani medicine to gain weight ?",
"src": "Patient: I am 28 year old height 5 3 weight 53 kg approx. I am very upset about our health I don t want take any medicine So i want to ask you for gaining my weight. can you suggest any aurvedic OR Unanai capsules or any thing You action will be highly appriciated in this regards Thanks & Regards // Azam Doctor: 1. keeping in accordance with your height to weight ratio it seems that you are slightly having few kilos less than what is desired. 2. you are UPSET itself is one cause of not gaining weight, because in these cases metabolism of a person is affected and increased leading to faster removal of food and reduced absorption. 3. or it can be due to inheritance,faulty food habits,increased physical activity with decreased energy(food) input, or if suffering from any problem related with digestion? 4. sometimes low hemoglobin or worm in stomach is also the cause of not gaining weight,check your Hb% level and stool test. 5. Mango shake is an ideal drink for you as one is free from protein (mango) and one is free from sugar (milk). 6. Don't let yourself stressed out, take diet rich in calcium (milk,curd), Vitamin D (milk,exposure to sun), magnesium (green leafy vegetables). 7. do compound exercises,increase protein and complex carbohydrates in your diet, sleep a lot (which will conserve energy needed for awakening). 8. make sure you are gaining weight on muscles and not FAT,so apart from eating also follow a good weight lifting programme."
},
{
"id": 31879,
"tgt": "What could bloating, nausea and painful abdomen after being treated for H pylori suggest?",
"src": "Patient: I have been diagnosed with H Pylori 5 weeks ago (gscope - biopsy). The doctor treated me wit antibiotics, Klacid, Flagyl, Nexium and Zantac. The Flagyl made me feel sick and my stomach worse but I finished the course. I was on a 5 day course. I feel worse I am so bloated it is not even funny, I get Nauseous in the morning and when I eat I feel better, but then my stomach start feeling bloated again I feel very Uncomfortable and in pain and some kind of movement just below my left side rib, I am scared to eat because its going to make me feel better for a while and then worse again. My bowel movements seems to be normal but I burp alot. I am scared of stomach cancer, are these symptoms related to stomach cancer. Sometimes I feel some Uncomfortable on the right side as well. I am only 31 and otherwise in good health. Please help? Doctor: HI, thanks for using healthcare magicStomach cancer would have been seen on endoscopy and would have shown up in the biopsy that you had a few weeks ago.It would not develop in the weeks since the biopsy, cancer takes months to years to develop. Since the biopsy 5 weeks ago did not show cancer, then this is highly, highly unlikely.Bloating is a common symptom and can be related to your history of h pylori, reflux. It can also be due to the use of gas producing foods such as certain vegetables or fruits.Constipation is also a possible cause.The use of reflux medication such as nexium, prilosec, prevacid, aciphex, may help.I hope this helps"
},
{
"id": 84880,
"tgt": "Is Montek LC safe for use by hepatitis B patient for allergies?",
"src": "Patient: I am a chronic hepatitis B patient and on anti-retroviral medication, I am also suffering from allergy related cough and cold and asthma with heavy chest conjestion. Can I take Montek LC on a regular basis as that helps to fight against allergies. Does it have any side effect on my liver. Doctor: Hi,It can be taken with careful monitoring of liver function tests. Montek-LC is a combination anti-histamine drugs Montelukast and Levocetirizine commonly prescribed for the conditions listed in your query. Montelukast is known to cause liver injury in patients with underlying liver disease such as hepatitis or those who consume alcohol. However, no dosage adjustment is required in patients with mild-to-moderate liver disease. Monek-LC can be taken with careful monitoring of liver function tests and any early signs of liver injury like fever, abdominal pain, vomiting or jaundice should be reported to your treating doctor after stopping the treatment.Hope I have answered your query. Let me know if I can assist you further. Regards, Dr. Mohammed Taher Ali, General & Family Physician"
},
{
"id": 88149,
"tgt": "Suggest treatment for persistent abdomen pain",
"src": "Patient: Yes, hello. I have been having moderate pain in my upper abdomen for approximately 3 days. It comes and goes some of it has been sharp and others just a dull throb. I am also disinterested in food, and occasionally have felt dizzy and have gotten hot flashes. I have felt this pain before in the exact location described but it has always gone away before. Doctor: HICause of pain in upper abdomen depend upon location of pain-Right side-Central-Left sideDisinterest in food IS in favor of HepatitisAdvised to get Ultrasound upper abdomenBlood test for LFT(LIVER FUNCTION TEST )Take,fresh fruits,semi sold, non spicy diet unless the diagnosis is confirmedThanks"
},
{
"id": 124793,
"tgt": "Suggest treatment for a swollen foot and engorged veins",
"src": "Patient: I hit my foot while walking. I hit a wheel chair pretty hard and heard a pop. I can bend my little toe but i got a bruise and now my foot looks swollen. I also noticed that the veins on the outer part of the left foot are engorged and after elevating the foor the throbbing pain stops and the veins dissapear. Doctor: Hello, It could be a contusion. As of now you can immobilise the foot and apply ice packs. Generally, the symptoms will settle in two to three days. If persists you can consult an orthopedician and plan for an MRI scan. Hope I have answered your query. Let me know if I can assist you further. Regards, Dr. Shinas Hussain, General & Family Physician"
},
{
"id": 90728,
"tgt": "What causes upper abdomen pain which radiates to spine?",
"src": "Patient: Hi I'm 34 y.o female, about 3 days ago I've got bad pain in the upper part of my stomach that radiates to the middle of my back right to my spine. This pain feel like a straight shot. It gets worse when I take a deep breath. Can you tell me what can it be? Thanks. Doctor: Hi! Good evening. I am Dr Shareef answering your query.If I were your doctor, after examining you clinically,I would advise you for some routine blood test like a cbc, blood sugar, serum amylase/lipase, a LFT, and an routine ultrasound abdomen/CT scan abdomen, to rule out any possibility of inflammation of pancreas specially if you consume alcohol. Till that I would advise you for some kind of anti spasmodic drug, proton pump inhibitor, and to avoid eating from outside and consuming alcohol if you do.I hope this information would help you in discussing with your family physician/treating doctor in further management of your problem. Please do not hesitate to ask in case of any further doubts.Thanks for choosing health care magic to clear doubts on your health problems. Wishing you an early recovery. Dr Shareef."
},
{
"id": 224012,
"tgt": "Is unprotected intercourse safe during the free period while on Althea pills?",
"src": "Patient: Hi good day. I am taking Althea pills now. (last pill tonight) correct me if Im wrong but i will take it the next set on October 10? during the 7 day free pill, is it still safe having sex with my husband without using condom? Ive read so many negative issues using pills..please enlighten us regarding taking pills.. I am married and with 3 kids. thank you very much for your help. God bless. Doctor: Hi,Welcome to Healthcare magic.I am Dr Ramadevi Wani. I appreciate your concerns.Althia is a effective contraceptive pill. If taken correctly it is more than 99% effective in preventing pregnancy. If you are taking these pills correctly the pill free seven days are safe. You may have sex without using condom. It does not result in pregnancy. But please remember that you need to start the next pack at the right time (exactly after a week) to prevent pregnancy.I hope this is helpful.If you have further concerns, I will be happy to answer.Best wishes,Dr Rama"
},
{
"id": 107398,
"tgt": "What causes frequent urination with lower back and pelvic pain during peri-menopausal phase?",
"src": "Patient: I have constant urination with some lower back and pelvic pain. My Doctor did bloodwork which was all normal. I am in peri-menopausal. She thought that I may have an overactive bladder but I would like to examine this a little further before taking any medication. Please advice Doctor: Hello,The frequent urination you discribe may be because of disc herniation in L5-S1 level, where the nerves which inervate urinary bladder start.You should have a lumbar MRI and after that try some physical therapy to strenghthen your muscles.Best regards,Anisa"
},
{
"id": 118819,
"tgt": "Weight loss, lower blood glucose level, low BP. History of taking VSL#3 for IBS. Can any other probiotic be tried?",
"src": "Patient: Sir, I had IBS for the last 6/7 years. For the last 5 years I was taking 1 capsule of VSL#3 after lunch. I am 5'9\" and was 62 KG. Since then I have lost about 8 KG weight. Recently my blood glucose level have gone to 67 Fasting & 111 PP. My Blood pressure has also took a dive, it is 70/100. I have stopped taking VSL#3 .What could be the probable cause? Can I try any other probiotic? Doctor: Hello. Is unlikely that an probiotic causes weight loss, decreased blood sugar levels and decrease in blood pressure.If at this period of time you have had frequent episodes of diarrhea associated to IBS, that's a possible cause.Go to your doctor for exploration and study causes of the decrease in weight, blood sugar levels and blood pressure.Although adverse effects referred are not caused by the probiotic that you have taked, if your physician considers it appropriate, other probiotic may be prescribed.I wish you good health.(In case the answer would have been useful please indicate this)"
},
{
"id": 1763,
"tgt": "What are the chances of pregnancy with IVF-ICSI?",
"src": "Patient: Just had first IVF-ICSI 4 days ago, wondering about signs of pregnancy? My breasts hurt last few days but not today and I felt some cramping last few days but not today? I had 2 embryos ( own eggs) transferred and I just turned 44 2 days before transfer. PT is not for 1 more week. I have one child ( natural) when I was 32. I am giving myself Progesterone shots daily.Dr. said I did well with transfer and had 1( 2grade cell plus) and 1 ( 3grade cell plus) Don't understand this grading system so much. My fsh was 8 I think and estradial went up to 738 before retreival. I egg didn't fertilize but other 2 did ( one at 21 1/2 and one at 22 1/2) What are my chances and what symtoms should I be currently having? What should my BBT be reading now if I am pregnant? Also, weight is 245, 5 ' 8\" and just turned age 44. Pretty healthy no health issues. Just taking Pre-natal and Calcium pills. Also, will baby aspirin help with ( hopeful) implantation? Doctor: Hi, you won't get any symptoms now. It is too early. Get your bhcg test done on day 14 after transfer. Around that time, you may have bloating, acidity and weakness. Aspirin will help in implantation. Hope I have answered your question. If you have any other query I will be happy to help. Regards Dr khushboo"
},
{
"id": 158157,
"tgt": "Biopsy post CBD operation shows malignancy, has numbness in back, flatus, abdominal pain. Reason?",
"src": "Patient: My mother undergone whiples surgery due to stricture in CBD. Biopsy report of post operation of CBD shows malignancy in between portion of CBD leaving both the end of the CBD. Though she became very weak after surgery but now she is OK. But she feels numbness in her back & more gastric. She also feel some abdominal pain . please suggest what could be reason for this. with regards. Doctor: Hi welcome to health care magic forum.the reason for abdominal pain is probably this malignant process or surgery effect. this is to be expected in such pathologies. numbness and paion in back and stomach can also be sign of CBD or pancreatic tumors but it should be evaluated by further imaging tests. numbness isnt something that should worry her at this point. she should find enough strength to fight against this serious diseaseHope I have answered your query. Wish you good health. regards"
},
{
"id": 87959,
"tgt": "What causes left abdomen pain with rectal bleeding and constipation?",
"src": "Patient: last night i was totally fine had a bowel movement with blood in the stool the acute lower left abdomen pain started lasted hours felt ok today little bits of pain i feel i need to have a movement but nothing happens and my lower back and abdomen are in pain again Doctor: Hi.Thanks for your query and an elucidate history..The history of sudden onset blood in loose motion is suggestive of severe colitis. You still have a feeling of passing stool and pain in lower abdomen and back. This indicates the need of a 5-day course of an antibiotic, metronidazole, activated charcoal tablets and probiotic.If the problem is relieved, nothing to worry about. If no response then you have to undergo the tests of blood, urine and stool, ultrasonography and colonoscopy to rule out any sinister problem. And the further treatment will depend upon the the findings ."
},
{
"id": 129126,
"tgt": "What is the cause of pain?",
"src": "Patient: hi just had open reduction on wrist for distal radius fracture, (had hardwear put in plate with 8 screws ) surgery yesterday feeling better today coping with pain but feeling like sharp poking pains or scratching where incision was. is it my stiches poking me? extremely unconformable. thanks! Doctor: Hello Thank you for using healthcaremagic I read your question and understood your concern It might be the stitches or it might be the plate and screws poking on you soft tissues but you will get used to it soon.Dr. Selmani"
},
{
"id": 14496,
"tgt": "What is the cause for red rashes on back,neck and belly?",
"src": "Patient: I have started to get a splotchy red rash on my neck, back, belly and ankles that appears between 7 and 8 each night, haven't eaten anything new, no new laundry soap and am fine all day. Last night I took benadryl, but it leaves me sleepy and out of it the next day. YYYY@YYYY Doctor: Hello,Thank you for posting on HCM.The condition you have referred to is called Urticaria or hives. Its an allergic manifestation of skin, where an allergen leads to release of certain substances from your blood, leading to itchy skin rash and swelling over soft tissues. Its proper management requires thorough history, clinical and laboratory work-up.You may have to go for specific tests like patch test, food prick test, IgE antibody levels etc. As for treatment part, best would be the avoidance of allergen as far as possible. Try to eliminate possible triggering foods from diet. I would also advise you various anti-histaminics for long duration( atleast 3 months) with or without oral corticosteroids. For non- responding cases there are many other drugs like dapsone, cyclosporine, montelukast, omalizumab etc which can be used in certain selected cases. Hope this will help you in resolving your query.Thank you Dr Hardik Pitroda"
},
{
"id": 62527,
"tgt": "What causes lump on breast along with tiredness and pain?",
"src": "Patient: hi I am 45 years old ! AND JUST FOUND A LUMP SIZE OF A PEA ON MY RIGHT SIDE BREAST UNDER THE NIPPE ! HAD A MAMAGRAM AND THEY CALLED ME BACK THE SAME DAY ASKING ME TO GO FOR AN ALTRASOUND ! I HAVE BEEN FEELING VERY TIERD THE PAST YEAR AND PAIN IN MY BREAST AND BAD MUSCLE ACHS WITH SHORTNESS OF BREATH 1 SHOULD I BE WORRIED ? Doctor: Hi, dearI have gone through your question. I can understand your concern. You may have some benign breast lesion like fibroadenoma, fibroadenosis or some benign cyst. You should go for ultrasound study and biopsy of that lump. It will give you exact diagnosis. Then you should take treatment accordingly. Hope I have answered your question, if you have doubt then I will be happy to answer. Thanks for using health care magic. Wish you a very good health."
},
{
"id": 110376,
"tgt": "Need medicines for hip pain",
"src": "Patient: HELLO , IM HAVING UNBELEIVABLE PAIN IN MY RT HIP, IM TOLD BY MY DOC THAT I NEED A HIP REPLACEMENT. MY PROBLEM IS THAT IM 100% COMM AND WORK AT MY DESK. I HAVE ONE WEEKS VACATION AND COULD MAYBE TAKE A SECOND WEEK OFF FOR FOR RECOVERY, BUT ANY LONGER COULD BANKRUPT ME. IS IT POSSABLE TO BE BACK TO WORK IN TWO WEEKS Doctor: if there is an indication for hip replacement then there may not be short cuts and 3 weeks is normal time to do sedentry permissible work with guarded movement.yes youmay defer surgery under cover of pain killers butthing that is inevitable cannot be avoided for long.however you may seek second opinion with a hip surgeon to evaluate the necessity or alternative to surgery"
},
{
"id": 162429,
"tgt": "Are multiple discrete reactive mesenteric lymph nodes seen in umbilical region and right iliac fossa of a 7-year-old a concern?",
"src": "Patient: multiple discrete reactive mesenteric lymph nodes seen in umbilical region and right iliac fossa, largest measuring 14.8*8.5 mm in right iliac fossa My son was affected on this issue, he is 7 year old. Can you explain what is happen in general english.Thank you for your help. Doctor: multiple reactivr lymph nodes in abdomen at 7 years of age. I need further information to conclude. is he having fever? any abdominal pain? any loss of weight and appetite. if none of the above symptoms are present, then probably you need not worry about those lymph nodes. if symptoms are present, then I would like to suggest you to consult local physician and do the necessary investigationsthanking you"
},
{
"id": 222523,
"tgt": "What are the symptoms of pregnancy?",
"src": "Patient: Hi, I m worried I may be pregnant but I m not sure...I just got done with my period about a week ago, and the last day of being on my period I had sex with my boyfriend. We started out unprotected for about a minute or two then he put a condom on. The next morning I woke up feeling nasueas and had a huge headache but it lasted most of the day so I don t think it was morning sickness, but the past two days I ve had a very light brown discharge showing up here and there in my underwear and some small bubbly feelings in my stomach. He never ejaculated into me but I m afraid the pre seminal fluid may have gotten me pregnant...or am I just ovulating or something? Doctor: Hallow Baby,You are too anxious. Woman releases egg (ovulation) only once in the cycle and that is around 14 days prior to the nest expected menses. Considering 24 hours' life of the egg and 72 hours' sperm activity, the fertile period is about a week around the day of ovulation. You had sex on last day of your menstrual bleeding. Even if before putting on condom he had ejected precum into your vagina, on this day the chances of presence of egg are almost nil. The brownish discharge was the remains of menstrual blood coming out late. The symptoms of pregnancy like nausea, vomiting appear about 8-15 days after missing a period. So these symptoms have nothing to do with your pregnancy. Associated headache suggests the possibility of stress of pregnancy and/or influenza. If the symptoms have vanished, you need not do anything for them. You should get your menses on expected day. If you continue to be under stress, it may delay your menses. If at all you miss your period, wait for 8-10 days and then get urine pregnancy test done for diagnosing the possibility of pregnancy. I am sure, you will not need it. I hope this clarifies your doubts. Dr. Nishikant Shrotri"
},
{
"id": 187130,
"tgt": "What could be the blister on the upper gum line?",
"src": "Patient: I have had a blister on upper gumline, where I recently had a lot of work done. Dentist is watching it, need to go back in 3 weeks. I have burst the blister before thinking that would heal it; but it is back??? should I leave it alone?? Hoping it's not Ca. Doctor: Hello, Welcome Thanks for consulting HCM, I have gone through your query, as you have blister dont worry there is formation of pus which can be due to periodontal problem Periodontal Abscess or Periapical Abscess . You consult your dentist again and go for proper treatment done. If it is gum problem then go for Oral prophylaxis and if it is periapical Pathology then go for Root Canal treatment done .Hope this will help you. Wishing you good health."
},
{
"id": 115640,
"tgt": "Suggest treatment to reduce eosinophil count in blood",
"src": "Patient: Hi Doc,from last few days am getting pain in my body(specially in joints and bones). Frequently am getting some sort of Headach specially in the morning and night time. I am also suffering from some throught infection.When i concerned with respective the doctor and got my reports on the same problem, i came to know that every thing is normal but the count of the Eosinophils (White blood cell) is high in my body, its around 11%.May i know your suggestions on that please.How can i reduce that eosinophils count from my body?Is there any chances of any Cancer ?Is there any side effects of that ?Please guide me on the same.Thanks. Doctor: Hi, dearI have gone through your question. I can understand your concern. Your eosinophil count is high. There are many causes of high eosinophil count. Asthma, allergy, hypersensitivity, parasitic infection etc are the causes of high eosinophil count. You should search the cause and take treatment accordingly. Steroids or DEC may be helpful. No need to worry about cancer. No chance of cancer. Just be relaxed. Hope I have answered your question, if you have doubt then I will be happy to answer. Thanks for using health care magic. Wish you a very good health."
},
{
"id": 134981,
"tgt": "What causes high blood pressure , numbness in the hands and heaviness in the chest?",
"src": "Patient: Saw cardiologist last wk...high bp tingling and numbness in hands and feet extremely heaviness in chest racing, hard heart beat, extreme fatigue and body aches...stress test and monitor scheduled for next wk....I wk for a Dr, saw my echo report today...please explaine severity... Aortic insufficiency pht 663, lft atrial enlargement, tricuspid regurgitation, normal lv systolic function, lvfs 35.92% lvot 1.91/ lft atrium 43ml/m2 Doctor: Hi Dear,Welcome to HCM.Understanding your concern. As per your query you have high blood pressure , numbness in the hands and heaviness in the chest . Well the symptoms you mention in query can occur due to many reasons like cervical spondylosis , medication reaction or side-effect , tachycardia , high blood pressure , heart attack , chest infection and angina . I would suggest you to consult cardiologist for proper examination without any delay . Doctor will take history , order blood tests like ECG , TMT , complete blood profile along with troponin t test and check vitals specially blood pressure . Doctor may prescribe sedatives , isosorbide dinitrate , beta blockers or calcium channel blockers along with diuretics . Doctor may also recommend angiography in some cases . For now take proper rest , have proper diet , do not exert your body , lower your salt consumption and apply warm compresses to chest .Hope your concern has been resolved.Get Well Soon.Best Wishes,Dr. Harry Maheshwari"
},
{
"id": 190887,
"tgt": "Why am I unable to open my mouth completely and how can I fix this problem ?",
"src": "Patient: dear sir my name is akash i am 21 year my mounth open just 1.5 inch pls. tell me how to open fully???? Doctor: Hi, You have not mentioned whether you have the habit of tobacco chewing. There are several causes for restricted mouth opening like trauma, oral submucous fibrosis, oral cancer, Fracture of mandible etc. Please mention if any history of trauma or habits present."
},
{
"id": 201566,
"tgt": "What causes numbness in left testicles post hernia repair surgery?",
"src": "Patient: Hi I Had a hernia repair ( lower left abdomen ) done back in may but since then I have been numb in the area between my left leg and crotch as well as this I have recently noticed while checking for lumps my left testical does not feel right, where as with the right testical when rolling in my fingers there is a slight discomfort and what feels like a vein/cord attached to the testical yet none of this with the left testical, (it s moving freely in the sack), am I imagining things or is there something behind these symptoms Doctor: Hi,Thank you for your query. I can understand your concerns.You have developed some minor complication following left inguinal hernia repair.1.Testicular descent is a complication believed to be related to complete division of the cremaster muscle. The cord structures lose their tethering effect, allowing the testicle to descend into the most dependent portion of the scrotum. Due to the elasticity of the scrotum, time and gravity cause it to elongate,giving a feeling that the left testicle moving freely within the scrotal sac.2.Numbness in the area between your left leg and crotch is due to section of the left ilioinguinal nerve while repairing hernia.Regards Dr. T.K. Biswas M.D.Mumbai"
},
{
"id": 40600,
"tgt": "What does the following follicular study report suggest?",
"src": "Patient: I did a follicule scan on day 13 and found LO dominant follicule size 10mm and RO dominant follicule size 14mm ET 11mm Fluid in POD positive. Is it right time to conceive? I have PCOD , regular 29 cycle. I am bit overweight due to junk food intake and work stress. Medication pills does not suit me, I get nervous breakdown due to medicine intake. can i get pregnant? Doctor: Hello,Definitely, you\u2019ll get pregnant but then I\u2019m not sure about this cycle. It would have been of more help if you would have got yourself serial follicular study this cycle, free fluid, regular cycles, and your ET suggest more towards ovulation and with PCOS and the size of your follicles, I\u2019m not very sure if you ovulated. Next cycle I\u2019d suggest you go for some hormonal tests on the second day of your period especially LH levels, and go for serial monitoring of follicles.Hope I have answered your query. Let me know if I can assist you further.Regards, Dr. Avanti Sathineedi"
},
{
"id": 166945,
"tgt": "What causes fever,diarrhea and vomiting in a child?",
"src": "Patient: Hi, may I answer your health queries right now ? Please type your query here... My 4 month old has a temperater of 38.5 degrees celcius and she has diareah and is throwing up. she also has no energy should i bring her in to the clinic or hospital??? what s wrong with her Doctor: hi! your baby might be having acute gastroenteritis.you need to monitor her urine passage. keep giving her breast milk and Ors. for fever give paracetamol. if she's throwing up to much and not able to accept orally you should take her to the hospital."
},
{
"id": 193910,
"tgt": "Suggest treatment for increasing semen motility",
"src": "Patient: hi,i'm 37 yrs old my height 1,76m and i weight 85 kg and i\"ve been married 1 yr ago, i just received my semen test which include something that disturb me which is the semen motility,it is written 30%....does this number prevent me from having a baby...and if it does..what should i do?thank u. Doctor: Hello, We have to rule out possible causes like varicocele which can give rise similar symptoms. Consult a urologist and get evaluated. A scrotal Doppler might be required. Hope I have answered your query. Let me know if I can assist you further. Take care Regards, Dr Shinas Hussain, General & Family Physician"
},
{
"id": 136233,
"tgt": "What is the source for foot toe pain?",
"src": "Patient: I m not sure what the source of the problem was, but for the past four weeks or so my right foot s big toe has been hurting at the base. I cannot move my toe far in any direction without pain, especially far up or down. It also aches dully and itches bad at night. There is no (external at least, what I can tell) bruising, swelling, or discoloration. What could be going on with it? Doctor: Hello, This could be a osteoarthritis of the Joint of the toe which can cause pain as you mentioned. this can also be from inflammation of the tendon below the foot.are you diabetic and check if you uric acid levels are normal. You may take dilcofenac potassium or Indomethacin 50 mg twice a day.if these blood levels are raised then appropriate medication will be needed"
},
{
"id": 210646,
"tgt": "Can stress cause numbness in my arm?",
"src": "Patient: yes I do have a question? I am experiencing an arm that has gone numb underneath I started noticing that it had no feeling about a week ago as I was using a washcloth on me in the shower. I have been under a lot of stress lately and didn't know if that may be the cause or what? Can a person get bells palsy somewhere on the body besides the face? Doctor: HIThank for asking to HCMI really appreciate your concern stress can cause any thing and not just the numbness only, no need to worry about the Bell's palsy because this is may not be due to the stress, hope this information helps you, have a good day"
},
{
"id": 145688,
"tgt": "What causes brain freeze?",
"src": "Patient: Two weeks ago I tried paxil for headaches/anxiety. On it for 5 days. Had brain freeze a few times...but was very very tired and light headed. Waited a week and started 12.5 mg of Zoloft last night. Didn t sleep much last night...certainly not tired today. I did have a few moments of brain freeze this morning and that sensation across my upper body. Scary! Headache is much better today. Question: Is the brain freeze and tingling something to worry about? I ve had a mri of my brain recently and it is fine. The headaches started 6 weeks ago. I m 70 yr old female. Doctor: Hello!Thank you for the question. I understand your concern. I think you don't have to worry about it. The brain freezing and tingling are common symptoms of some types of headache. The brain MRI has ruled out any other serious problems. I would like to know more about the duration, precipitation factors and localization of your headache. In my opinion its a Tension type headache. Zooloft is very helpful for this situation. You should continue taking it for at least 3 months. You will see results from the third weak of treatment. You should have a headache diary to register time and duration of headache every day. So you will be able to make the real difference of the therapy. Hope to have been helpful .Best wishes. Dr Abaz Quka"
},
{
"id": 30982,
"tgt": "Is Tetanus shot needed if dog bite draws blood exposing flesh?",
"src": "Patient: Hi iwas playing with my 9!weeks old pup when I caught my finger on he's tooth and split my finger open and was bleeding - now it's open and shows bobbly flesh- I have placed paper stitches but doesn't seem to be working as it's paining and swelling do I need a tetnus jab ? Doctor: Hi & Welcome.Yes, you need to take a booster dose of tetanus toxoid.Before visiting a hospital, the wound should be washed and flushed immediately with soap and water for 10\u201315 minutes. If soap is not available, flush with water alone. Then the wound should be cleaned thoroughly with 70% Alcohol or Povidone solution.Avoid covering the wound with paper stitches or dressings. Suturing facilitates further inoculation of rabies virus.If necessary suturing should be done by the doctor after infiltration of wound with rabies immunoglobulin (RIG). The sutures should be loose & should not interfere with free bleeding and drainage. The secondary suture of bite wounds results in better cosmetic outcomes.If your puppy is vaccinated and the vaccination certificate is valid, then there's no need to take the anti-rabies vaccine. Otherwise you will have to take 5 doses of anti-rabies vaccine on days 0,3,7,14 & 30. Observe your puppy for 10 days following the bite and if the puppy is found to be healthy then you may discontinue the remaining doses of vaccine on day 14 and 30.And make sure that your puppy is vaccinated.Take care."
},
{
"id": 86308,
"tgt": "Suggest treatment for severe rib cage pain and abdominal pain",
"src": "Patient: I have had a pain under my left rib cage for almost 12 years. Around the time it began, I received a CT scan which revealed nothing. I lived with it for the next 11 years. Last year I mentioned it to my GP and didn t get much info. She tested my liver, pancreas, ect. levels which again were normal. It seems lately to be more annoying/painful/bothersome than it has in the past and seems to be changing in its symptoms. It began feeling like part of my lower rib cage was stuck under the rest of the rib cage, and upon manipulation, it actually moves, which the other side does not. This offers some temporary relief. Now it gives me abdominal pain that does not feel digestive related. This pain is located to the left of my navel about 1.5 inches and can extend up above my navel in between the breastbone and navel. It feels more of a pulled muscle, but is constant and is not only occurring when the muscle is pulled. I also sometimes feel the pain radiating to the back side almost exactly where I feel the stuck rib pain and sometimes I feel a twinge on my left side just below the ribcage. All of these feel related to one another. I have a physical on the 25th, and I want to know what questions to ask and/or what tests to ask to have done. As a side note, I recently had the symptoms of a UTI including hematuria and flank pain. I noticed increased flank pain before the symptoms of painful urination and hematuria, but I attributed it to my ribcage thing because it felt similar, only more intense. After visiting the dr, she confirmed a high pH and I began antibiotics to clear it up. I received the results a few days later and my doctor stated no specific bacteria was found. However, my symptoms have cleared with regards to the UTI. Please help, it s gone on long enough! Thank you:) Doctor: dear friend,pain at the the left rib cage since 12 years- could be due to inflammation of the costochondral junction (where cartilage and bony part of the ribs unite) which is called as costochondritis. this condition is usually relieved by simple pain killers. persisting symptoms can be treated with locally injecting low dose steroids. please mention your age and gender when you are next question."
},
{
"id": 123127,
"tgt": "What causes sharp nagging pain on left side under breastbone?",
"src": "Patient: I am 45. I am experiencing a sharp nagging pain on my left side. Been going on for about an hour. Seems to be effected by movement..gets better if I change posistion. But then returns. The pain is located under my breastbone. Can you tell me what this might be? Thanks Doctor: Hello, This should be related to the intercostal muscle spasm. Doing hot water fomentation, using loose clothing and simple deep breathing exercises should provide relief. Hope I have answered your query. Let me know if I can assist you further. Regards, Jay Indravadan Patel, Physical Therapist or Physiotherapist"
},
{
"id": 184661,
"tgt": "Suggest treatment for tooth gaps",
"src": "Patient: i am 15 and have a impacted canine tooth on the right upper side of my mouth. my dentist pulled out the baby tooth and now there is a gap because the adult tooth is backed away from the others, is there any way i can get this gap filled without braces? its my year 10 formal this year and i just want the gap covered! is there a temporary or permanent solution. PLEASE HELP Doctor: Thanks for your query, I have gone through your query.The gap created by the extraction of the deciduous tooth can be corrected by orthodontically moving the tooth into the gap. Since you want it temporarily you can go for a removable partial denture. Denture comes in the form of a plate(made up of acrylic) attached with a tooth. You can use it for short period after that you can correct your impacted canine.I hope my answer will help you, take care."
},
{
"id": 124156,
"tgt": "Suggest treatment for knee pain and heaviness in legs",
"src": "Patient: i am 53 my knees (both equally) are so very achy. each and every time i get up it is painful and i have to wait to take that first step. my legs feel like they weigh a ton and during the night they burn. moving them all u hear is crackling HELP!! Doctor: Hi, What I understand by the history is that you have muscle weakness and due to this joint inflammation and thus the joint pains. Using how water fermentation and allowing it to rest will help reduce the inflammation. Also using knee brace should be of additional support to the knee joint to get less pressure. As it is a weight bearing joint it needs to be treated well. Doing some strengthening exercise for the knee muscles like static quadriceps which can be performed by pressing the knee down and static hamstrings which can be performed by pressing the heel down. Along with this you can try doing straight leg raise as well. Please continue the exercises for 10 days and let me know if you find any changes. For burning sensation you need to get the uric acid test done to confirm any other pathology. Hope I have answered your query. Let me know if I can assist you further. Regards, Jay Indravadan Patel, Physical Therapist or Physiotherapist"
},
{
"id": 8072,
"tgt": "i have some black color spots on my face that is increasing day by day",
"src": "Patient: hi i am 30yr old from nearly 5 years i am developing some black colour sutures on my face .These sutures are increasing in number & in size day by day iam feeling very much depressed plzz help me Ananaya. Doctor: Hi friend, For black suture you can take ayurvedic or homeopathic treatment. Neem (Azadiracta indica), Giloy (Tinospora cordifolia), Manjistha (Rubia cordifolia) , Haridra (Curcuma longa) are very effective for skin problems. Take extra amount of water. Avoid extra amount of proteins and fat. Avoid difficult to digest food.Avoid the types of food which irritate your skin. to get more information, you can contact me any time."
},
{
"id": 171932,
"tgt": "What are the red bumps on his face?",
"src": "Patient: Hi there my son is 4 and had general anethisia on friday to remove 4 teeth and do some crowning, and now has tiny red dots on his face what is this from, they are not bumps just dots. And also he says his tongue feels like its full of spiders is this normal? Doctor: I am not sure about the tongue sensations. However the red spots are probably harmless and should go away soon. If they do not go within a few days, please get his CBC (a full hemogram) done to see what his platelet counts are. If these are normal, and the spots do not go, or new spots appear, take him for a bleeding time/clotting time study to rule out a clotting-related problem.Hope this helps.Dr. Taher"
},
{
"id": 143053,
"tgt": "What causes dizziness?",
"src": "Patient: i ve just experienced a dizzy spell whilst sitting at my desk. The sort you may expect if you bent over to pick something up from the floor and got back up to quickly. My vision became a bit blurred and seemed a bit shaky and I experienced a tingly sensation in my head and shoulders and my face feels a bit hot. I experienced something similar whilst out shopping before christmas. Any idea s what it could be? Doctor: Hi, Welcome to HealthCareMagic.com I am Dr.J.Mariano Anto Bruno Mascarenhas. I have gone through your query with diligence and would like you to know that I am here to help you.Question : Any idea s what it could be?Answer : This can be 1. Anemia 2. Hypotension3. Vestibular Dysfunction4. Vestibular MigraineYou need to consult a Neurosurgeon for further evaluationHope you found the answer helpful.If you need any clarification / have doubts / have additional questions / have follow up questions, then please do not hesitate in asking again. I will be happy to answer your questions. In the future, for continuity of care, I encourage you to contact me directly in HealthCareMagic at http://bit.ly/askdrbruno Best Wishes for Speedy Recovery Let me know if I can assist you further.Take care."
},
{
"id": 87328,
"tgt": "What causes pain in diaphragm area while breathing and sitting?",
"src": "Patient: i am currently experiencing pain in the diaphragm area. When breathing in it hurts quite a lot and i cannot press into the area at all. I am finding it hard to sit down or lie down and being upright is the only comfortable position. I do have a hiatus hernia and I m wondering if it could be connected? I am a 32 yr old female Doctor: Hi! Good afternoon. I am Dr Shareef, a general surgeon answering your query.Considering the history of having a hiatus hernia, if I were your doctor, I would advise you to get yourself assessed by your gastro enterologist at the earliest to see if your hernia got complicated giving rise to such pain. Till then I would advise you with an anti spasmodic and a proton pump inhibitor drug for a symptomatic relief.I hope this information would help you in discussing with your family physician/treating doctor in further management of your problem. Please do not hesitate to ask in case of any further doubts.Thanks for choosing health care magic to clear doubts on your health problems. I wish you an early recovery. Dr Shareef."
},
{
"id": 184119,
"tgt": "What causes black and rotten teeth?",
"src": "Patient: yes I wish I could convey my alarm. I just looked in my husbands mouth and his teeth have turned all black and rotten. I beleive this has happened in the last year....since he has had a chronic cough. The Dr. has never noticed this during his annual physical. He is 71. Doctor: Thanks for your query, I have gone through your query.The black discoloration of the teeth could be because of the stains or decay.Consult a oral physician and get yourself examined to rule out these things. If it is stains then get your teeth cleaned once. If it is decayed teeth then get the teeth restored. When it is in the initial stage you get it restored other wise it will get infected and results in pain swelling and other complications.I hope my answer will help you, take care."
},
{
"id": 42283,
"tgt": "What is the best treatment for infertility in women?",
"src": "Patient: Dear doctor, My wife has had two failed iui s (at 38), two failed ivfs (at 39) and now a failed FET at forty. 2 good quality blasts in each case. She has been tested normal for all bloods, killer cells, receptivity etc. Is ivf a numbers game and we try again d or should we go for donor eggs. thanks Doctor: Hi,I read your query and I understand your concerns.Following is my reply:1) Age of both of you is high.2) Please go ahead with IVF or ICSI as soon as possible without wasting more time on IUI.Let me know if you have anymore questions.Regards,Dr. Mahesh Koregol"
},
{
"id": 222199,
"tgt": "Does endometriosis affect pregnancy?",
"src": "Patient: Hi Doctor,I am 34 yrs old, I did myomectomy two months ago, initially the gynae thought I had fibrod until the myomectomy was done, it was discovered during the surgery that I have endometriosis. I have been placed on goserelin (Zoladex), Can I have healthy baby with endometriosis? Is there any cure for the disease Doctor: Hello dear,I understand your concern.ln my opinion endometriosis is one of the causes of infertility.The severity depends on the stage of the disease.Laparoscopic resection also offers good success rate.Medical management also is good.The problem in endometriosis is adhesions and chocolate cyst of ovary.There is a chance for natural pregnancy but if not occurred there are artificial reproductive techniques(ART).So nothing to worry.Avoid stress and anxiety.Best regards..."
},
{
"id": 30868,
"tgt": "What are the symptoms and treatment for influenza?",
"src": "Patient: Hello Doc!!! Iam 28yrs old and from past 2months Iam suffering from bodyache,slight fever,uneasyness symptoms of flue.I went to our family doc and he used to give me medicine on flue,viral infection it worked on me but temporarily,I got the Blood report and its clear nothing is shown in it.Currently Iam taking treatment from some other doctor who gave me medicines for Malaria Cifran500,Malarid15,Mefloquin250, He told me told take the medicine for 10days and i did that.. as now I have stopped the medicine Iam back to Square 1, Iam suffering from same problem again the medicines work but temporary. Please Help.. Doctor: thanks for posting your query to health care magic.Your symptoms are suggesive of pyrexia of unkown origin. There are so many causes of such kind of fever .I want to know some other point; Is fever associated either with cough, cold , running nose , pain in abdomen , loss of appetite ,burning during urination , increased frequency of urine ,or any other symptomsIs fever continuous or inermitent What is maximum temperature during episode of fever Did you loose weight during past few month .Is there any history of tuberculosis .Family history of tuberculosis .What was your complete blood count report If you have blood culture examination . Reply me with above query so that we can make a diagnosis and treat accordingly. as you have already taken medication so not start any treament without diagnosed the condition .however you can take antipyretic like Paracetamol 750mg as and when required if not sensitive to drug for symptomatic releif .thanks ."
},
{
"id": 30579,
"tgt": "What causes severe itching between thighs?",
"src": "Patient: My mother, aged 62yrs, suffering from severe itch between thighs in d past 3-4 months. Consulted 4-5 doctors, including 2 dermatologists. Still suffering. So far medicines taken were O=Ointments: 1] Sep 2010: O.QuadriDerm, O.Nizral + T.Flucanazole 150 2] Oct 2010: O.Lullifum, Flucos powder, T.Nizral 3] Lactacid lotion, O.Premarin, T.Allegra 4] T.Chlorohil? 0.25mg {for sleeping}, AVIL injection 5] Oct 2010: Cap.Clavam 625, T. Arc-AP, T.Rantac 150, T.Gocan 150 (Flucanazole), T.Zylera 5mg O.Sebifin, O.FinTRIX {This time apart from itching betn thighs, also had Sore/blister on buttocks} O.Sebifin 6] Nov 2010: O.Lobate GM, T.Xaria{Montek LC} 7] Dec 2010: O.CloteRite BN, T.Cetirizine{only @ night , 5mg} Still v donno d name or remedy? Doctor: HIWELCOME TO HEALTH CARE MAGIC I CAN UNDERSTAND YOUR CONCERN TOWARDS YOUR MOTHERI HAD GONE THROUGH YOUR MOTHER SYMPTOMS, SHE IS HAVING ITCHING SINCE 3-4 MONTYHS AND IT HAS NOT BEEN REDUCED IN SPITE OF TAKING ABOVE MENTION DRUGS. ACCORDING TO ME SHE IS HAVING NEURODERMATITIS. IT IS A TYPE OF DERMATITIS, WITHOUT ANY PATHOLOGICAL CONDITION.IT MAY BE DUE TO TENSION,STRESS OR EMOTIONAL DISTURBANCE.ITS MORE OF PSYCHOLOGICALKINDLY VISIT YOUR NEAREST PSYCHIATRIST FOR COUNCIL LING AND SPEAK TO YOUR MOM REGARDING ANY OF HER PROBLEMSIF SHE IS NOT FEELING BETTER WITH ABOVE MEDICATION , ASK YOUR DERMATOLOGIST TO START WITH MILD STEROID OINTMENTS FOR THE RELIEF.I HOPE I HAVE CLEARED YOUR QUERY TO SOME EXTENTTHANK YOUTAKE CARE THANK YOU"
},
{
"id": 108106,
"tgt": "What can be the reason for dizziness, gum bleeding and back pain?",
"src": "Patient: hemo: 170hem: .50wbc: 13.0 neu: .64 lympho: .27 mono : .04 eosi: .04 baso: \u00a0\u00a0\u00a0\u00a0\u00a0.01rbc: 5.3platelet 540 (previous: 840, 750, 539)have been diagnosed 20082011 frequent dizziness in the morning and before bowel movement, gum bleeding, bruise in legs once a month, clubbing of fingers in left hand..once in while backpain..already done bone marrow extraction and ckl positive..please advise what sickness and what to do. Doctor: Hi, i did see your concern. gum bleeding can be because of various reasons. A coagulation disorder is likely if it is occuring often, also a vasculopathy is possible. I would advice you to show an experienced physician and a hematologist as these problems need extensive study and treatment is possible with correct diagnosis. few conditions that can cause this are von willibrand disease,thrombasthenia gravis and so on..i hope this helps. do let me know if you have any more questions for me.wish you a healthy life ahead.thanks"
},
{
"id": 82058,
"tgt": "What causes left sided chest pain which radiates to shoulder?",
"src": "Patient: Hi I have had left-sided chest pain all day. The pain radiates up to my left shoulder tip but not down my arm. The pain eases a bit with rest. I have no significant medical history. I am a healthy 53 year old woman in the normal weight range. Should I go to the ED? Kathy Doctor: Thanks for your question on HCM. In my opinion you should go to emergency department and get done ECG to rule out cardiac cause for your chest pain.Risk factors for cardiac disease in you are1. Age more than 50. Postmenopausal women tend to have higher risk due to loss of protective effect of estrogen. 2. Your symptom of left sided chest pain with radiation to left shoulder seen most commonly in cardiac disease.So we need to rule out cardiac cause first. So consult emergency department and done ECG ."
},
{
"id": 62679,
"tgt": "What causes a lump in the left side of the abdomen?",
"src": "Patient: I have a long one inch wide soft lumb to the left of my belly button. My stomach muscles did seperate and still are apart 9 months later. I m slim so can see the bulge espeically after meals. I also lost 7lbs from my pre-pregnancy weight unwillingly about one week after giving birth don t know if that has something to do with it? Doctor: Hi,It seems that you might be having hernia producing bulging from abdominal muscles.Due to weakness in recti muscles, starting of hernia is there.Consult surgeon and get examined.Take high protien and nourishing diet.Do abdominal recti muscle exercise to strengthen the muscles.Avoid constipation and cough reflex.Ok and take care."
},
{
"id": 198426,
"tgt": "Why is my semen leaking and have back pain with weight loss?",
"src": "Patient: hi, am 27 years old guy, i have some problems from last 3 months, like leaking of semen and pain in my back little and not wait loss but no fat i have in my body, and i feel my legs and biceps are shrinking my cloths are lose now west is loose, and i have consulted to a doctor he gave me some multivitamin capsuls but it does nt work as i want , and because of work i cant eat timely.please help me out Doctor: DearWe understand your concernsI went through your details. Please understand the basics. You are worried and anxious regarding the semen leaking problem. Semen leakage is a natural process and there is no health issue with that. For body, semen is a waste product and if it is in excess, body throws it out through urine or through nocturnal emission.This semen leakage has nothing to do with back pain, losing weight, sleeplessness, slackening muscles, lack of appetite etc. Your doctor is correct. But more than multi vitamin tablets, you need to have stress and anxiety free life style and nutritious diet. I suggest you to consult a psychologist for counseling. If you require more of my help in this aspect, please use this URL. http://goo.gl/aYW2pR. Make sure that you include every minute details possible. Hope this answers your query. Available for further clarifications.Good luck. Take care."
},
{
"id": 70335,
"tgt": "Any suggestion for growing lump on front of leg below kneecap?",
"src": "Patient: I took a fall on a rough gravelly surface 4 months or so ago. I had to dig a couple of small pieces of gravel out of the wound but I believe I got them all. A small soft lump remained after the wound healed. It is not painful to the touch but kneeling on it is painful. Over the last couple of months it has increased in size. Overall diameter is the size of a half dollar and it is raised at the center point approximately 1/3 of an inch. If I am sitting in a chair with my feet on the floor, the lump is on the front of my leg just below the kneecap. It is not particularly uncomfortable but the fact that it is getting larger is giving me pause. Any thoughts? Doctor: Hi,Thanks for writing to HCM.I think that you missed one small piece of gravel inside.That may be causing foreign body granuloma. So better show it to a surgeon for a physical examination and get it removed.Dr. Ashish Verma"
},
{
"id": 190019,
"tgt": "Wisdom teeth extracted, small holes in gums. Possibility of dry sockets?",
"src": "Patient: I had all 4 wisdom teeth extracted this past Monday (currently Friday night). Everything seems to be healing extremely well except I ve recently noticed small holes in my gums where the bottom teeth once were. Is this normal or should I be concerned about the possibility of dry sockets? Also, how long would this typically take to heal? I uploaded a picture of the gum area. Thanks Doctor: Hello and welcome to HCM It has been four days only since you got your wisdom teeth extracted. After extraction of the tooth a space is created where the teeth once were present. This space will initially be filled with blood and slowly the granulation tissue will grow into it. Granulation tissue is fibrous tissue with newly erupted blood vessels. Slowly entire area will be converted into a firm soft tissue as your gums. So, there is nothing to worry. Do not be over concerned with that area. Leave it on its own to heal. In the meanwhile you can take saline gargles and pain killers if it hurts. In case there is excessive pain you can report it to your dentist to rule out infection. Thanks and take care Dr Shailja P wahal"
},
{
"id": 32542,
"tgt": "What causes vomiting and fever post a bite?",
"src": "Patient: Hi, may I answer your health queries right now ? Please type your query here...I got bit on the top of my hand 10 days ago...my skin was punctured in 3 different places. I am very ill now with vomiting,dizzy,chills,low grade fever. What could be wrong with me.? Thank you so much Doctor: Hello,This could be a very serious infection from the bite. Please see a doctor as soon as possible. You probably need to be on antibioticsRegards"
},
{
"id": 85682,
"tgt": "Can consumption of warafin cause numbness in fingertips ?",
"src": "Patient: I had my aortic valve replaced in July of this year then had complications with blood clots to my lungs. Am currently taking warafin and have developed numbness in my fingertips. At first it would come and go now it seems to be all the time. Is this a serious side effect of the medication? Doctor: Hello,warfarin is not very likely to be causing this symptom. You'd better check with your doctor for other potential causes which may include carpal tunnel syndrome, cervical radiculopathy, peripheral neuropathy, etc. In any case doctors first exclude the most common causes before attributing a symptom to a rare side effect.I hope I've answered your question. Please let me know if you need further assistance.Kind Regards,Dr Panagiotis Zografakis,Internal Medicine Specialist"
},
{
"id": 146378,
"tgt": "Is the inflammation at the back of head causing concussion and neck pain?",
"src": "Patient: Hi there:) I hit the back of my head almost 3 months ago because of a wooden beam I did not see. I have been on Prednisone for a week treatment . I recently went to the doctor and she saw that there was still a bit of inflammation in the back of my head(she believes I suffered a concussion). I am getting pain on the left side of my head that radiates down my neck, to my shoulder and down the rest of the arm. Why is this ? :( I do not have much money so I have not done any scan of whatsoever:/. Doctor: Hi welcome to HCM.In case persistent pain at the back of the head following head injury, its extremely important to suspect brain contusion. This can result in seepage of blood into the nearby tissues causing irritation of the meninges and leading to set up of inflammation. This can cause persistent neck pain. Its extremely important to have a non contrast CT scan abdomen to rule out meningitis and signs of raised intracranial pressure. Following which a CSF lumbar puncture can be done to start an empirical antibiotic therapy to treat associated meningitis if any. Therapy can last for 21 days. Consult your neurosurgeon for further help.Thank you."
},
{
"id": 131921,
"tgt": "Is it safe to take Pen-Vi-K tablets for an open lesion on my ankle?",
"src": "Patient: I have a small open lesion on my ankle just above my ankle bone. I am three days from home on a tour bus. I am type ll diabetic. I have a foil packet with 2 Pen-Vi-K tablets with no dosage size on the packet. I was given these some time in the past. I am not allergic to penicillin and have taken it in the past. I m also on warfarin. Do you think it would be safe to take this medicine? Doctor: Hi just having an open lesion, without having pus, is not an indication to take antibiotics . I suggest you do regular dressing of the open lesion above your ankle with good antiseptic cream and if pus comes or redness increases then consult your doctor before starting antibiotics."
},
{
"id": 60638,
"tgt": "Consequences of gall bladder stones during pregnancy?",
"src": "Patient: hello doc my sister is 24 years old and she has 2 gall stones from last five years but she has no pain from last 2 to 3 years. but recently she got married and she is planning for a baby. my concern is that there is any chance in abnormalities in baby due to gall stone. will she face any complications during pregnancy due to this stone. Doctor: dear patient welcome to \"HealthcareMagic\"forum the problum is only pain.because its untoalrable.so if posible you may take first gallstone treatment.and then think pregnancy. thank s....."
},
{
"id": 107265,
"tgt": "Is surgery advisable for spinal stenosis?",
"src": "Patient: I have spinal stenosis, and a ruptured disk of the lumbar spine. I last saw my orthopedic doctor in April of 2016 and have not been able to see him due to relocation of his practice. Consequently I am on a regular dose of Tylenol 4 for pain management. The last time I saw my doctor there was mention of surgery for the stenosis. I m now experiencing a lot of pain so where I take more of my pain medication than I d like to manage it. Do you think surgery is the way to go? Doctor: Hi Went through your post, low back pain can be crippling some times, usually the indications for surgery for prolapse disk are focal deficit which means weakness of some part of leg and canal stenosis so I would suggest you should go for surgery and live pain free."
},
{
"id": 32223,
"tgt": "What are the symptoms of post viral fatigue ?",
"src": "Patient: I've been poorly for a couple of months, 3 weeks ago went to the doctors and last week he diagnosed me with glandular fever, I have been suffering lots with aches and pain epecially in my eyes, back of my neck and upper back and rob area, am getting better now and these symptons are not as bad as they were but have since started with the backs of my knees they have started to really ache, is this part of the post viral fatigue?? Doctor: Yes, it is. General malaise can persist even up to 3 months. Such fatigue is fairly common in infectious mononucleosis (glandular fever). You can consider taking corticosteroids if you develop any complications."
},
{
"id": 224818,
"tgt": "Should i be concerned about taking 15th pill instead of 14th pill of Crisanta LS?",
"src": "Patient: Hi, i am taking Crisanta LS and by mistake i have skipped 14th day pill in the series and taken the 15th one in the series. I have taken the 14th day pill on the 15th day, and i'm still continuing the pills as normal. Please advise if the pills will still work as before. Doctor: Hello and welcome to HCM,\u00a0\u00a0\u00a0\u00a0\u00a0I am Dr Nilajkumar a consultant gynecologist and I will be helping you in your queries. It seems that you have taken the day 15 pill on day 14 and the day 14 pill is unused. I also assume that you have not missed a pill on any day, just taken it one day prior. If you have taken pills every day from the start of the pack, then continue taking the pills and take the day 14 pill on the last day after finishing the pack. All pills have same drug so it does not matter on which day it is taken. Just take 24 pills continuously without a pill free day and that is all. The pills will work and you are protected. Hope this was helpful. Feel free to ask any further queries and I will be happy to help.Thanks for using HCM.Have a good day.Dr Nilajkumar Bagde\u00a0\u00a0\u00a0\u00a0\u00a0\u00a0\u00a0\u00a0\u00a0\u00a0Consultant Obstetrics and Gynecology"
},
{
"id": 13209,
"tgt": "What caused itchy pimple like rash?",
"src": "Patient: I am a 37 year old male who has had a rash since Christmas that has not gone away they look like pimples as some of them if I squezze them pop.... they are itchy and uncomfortable and have gotten worse even after trying medicinal soap the only changes I have made since then are my diet... though it has been allot healthier... i did stay in a old minors cottage house and in the morning i noticed it first i am going to seek medical advice though i am worried and its affecting my confidence i have not had chicken pox yet but my partner hasn't caught it from me, ive also noticed that ive been sweating allot more from this with any excertion that i do... Doctor: Hi, As per your query you have symptoms of red itchy pimple-like rash which occurs due to hair follicle infection and when pores of sebaceous gland filled with sebum, dead skin and bacteria get infected. I would suggest you start with the application of aloe vera gel and apply azelaic acid containing cream twice a day as this is a safe and natural extract of cereals. You should apply topical antibacterial cream. Apply aloe vera gel to keep moist. I would suggest you consult dermatologist once and get it examined. You should go for antibiotics (Clindamycin), a corticosteroid ointment, and hydroquinone. Do not scratch pimples as they will get infected. Avoid exposure to sunlight and keep areas clean and dry. Hope I have answered your query. Let me know if I can assist you further. Regards, Dr. Harry Maheshwari, Dentist"
},
{
"id": 185796,
"tgt": "What causes missing wisdom tooth in upper jaw?",
"src": "Patient: Hi doc, I suddenly realized today that most of my wisdom tooth in the upper jaw is missing. Which is VERY strange and SCARY. I have no idea what might have happend. I started smoking about 4-5 months back and just smoke about 1-2 cigerettes per day. Do you think that it is due smoking? If not what could be the reason and the possible solution? What should I do? Doctor: thanks for your query, i have gone through your query. the cause for the missing wisdom tooth is genetic etiology. the most common gene responsible is mutation in a gene called PAX9. the other reason could be the evolutionary changes. because of the change in food pattern from generation to generation the use of third molars is negligible. so the third molar fails to develop even if it develops fails to erupt because of reduced size of the jaw bone. nothing to worry about missing wisdom tooth because it is functionless tooth. you can consult a oral physician and take radiograph to confirm the missing teeth. i hope my answer will help you. take care"
},
{
"id": 3170,
"tgt": "Suggest treatment for irregular periods and getting pregnant",
"src": "Patient: Yes my question is that I have my irregular periods n last I had my periods on 1 May till 15 may n then till now I have no periods so i am taking this syrup masturd from hamdard so can u tell me what should I do for my periods can come n then I can try for the baby thanku Doctor: Hello dearI understand your concernFirst you have to find the cause of irregular period.Your symptoms could be due to hormonal imbalance, polycystic ovary disease, endometriosis, adenomyosis, thyroid dysfunction.I would suggest to consult the gynecologist and undergo reproductive and thyroid hormone analysis, USG scan, ovarian follicle study to confirm the cause.Specific treatment will depend on the cause.Drugs like birth control pill and progesterone pill can be useful to regularize the period.Once your period become regular, you chance of become pregnant will increase.Drug like clomiphene citrate and FSH analogue could be usefulAvoid stress, take health diet, drink plenty of water and do regular exerciseHope this may help youContact further for follow up if neededBest regardsDr. Sagar"
},
{
"id": 209246,
"tgt": "Suggest right treatment for constant crying and depression",
"src": "Patient: I am currently being treated for depression. It has been really bad the past several weeks and doctor is trying to figure out meds. She is also suggesting that I go to a mental facility where there are doctors that can figure out the right meds. The place is called Prairie St Johns. Anyway, my question is how can psychiatrists in that facility figure out the right meds in just several days while it takes a doctor many times prescribing different meds before they figure it out ? I am at a wits end right now and cant stop crying. I am considering going to the facility but my husband thinks I am looking for an easy way out. What is your opinion? Doctor: DearWe understand your concernsI went through your details. I suggest you not to worry much. In my opinion you should go to the medical facility and convince your husband. The doctors there should have more experience in treating such patients with severe depression. Every doctor come across several kind of depression in their practice period. No two cases can be the same. Attending different cases continuously is their experience. Most of the times hind site also works wonders. Such experienced doctors diagnose as a panel and they can identify your exact problem for sure. Don't worry.Try psychotherapy along with the medication. It works wonders.If you require more of my help in this aspect, Please post a direct question to me in this website. Make sure that you include every minute details possible. I shall prescribe some psychotherapy techniques which should help you cure your condition further.Hope this answers your query. Available for further clarifications.Good luck."
},
{
"id": 49925,
"tgt": "Calculus in kidney. CT scan of abdomen and pelvis done, had ureteroscope and lithotripsy. Meaning?",
"src": "Patient: I had ureteroscope and lithotripsy last week for a 8 mm calculus in the right ureter,however the pre and post iv contrast enhanced CT of my abdomen and pelvis report says that I've 5mm calculus in the upper pole and 2 mm calculus in the lower pole of left kidney. What are the most probable treatments so that I can get rid of all my stones? Doctor: Hello,After the ureteroscope and lithotripy, the right ureter is treated already and as for the left upper pole and lower pole calculus are small in size. if its asymptomatic you can be on observational follow up and 3 mths ultrasonongrapy can be done for any inscreasement of size of calculus. Need to drink plenty of water and activities like jumping or running exercise which might migrate to bladder. if it fall on ureter part you can have tamsulosin which inscreases the motality of the ureter."
},
{
"id": 177453,
"tgt": "Suggest treatment for painful sores on scalp",
"src": "Patient: My 3 year old has multiple (counted 10) oozing scabs in his scalp. They ooze yellow puss, they don t seem to bother him but can be seen through his hair and his hair is matting around them. He usually has one or two but these have appeared within the last day. He won t let us touch them. Doctor: Hi Dear welcome to the HCM Infected seborrhic dermatitis is the probable diagnosis.Can be treated with antibiotic like cephalexin for a week with recommended doses.Ketoconzole shampoo 2% to be done twice weekly regularly for many weeks . so that fungal infection is properly eradicated.Avoid oil application.Hairs to be washed daily and dried.Hope the query is answered.thanks"
},
{
"id": 116613,
"tgt": "What needs to be done for a systolic pressure?",
"src": "Patient: my diastolic blood pressure is perfect running between 60 and 70. My top systolic runs from 140 to 160. my question, I walk 4 miles 3 times a week, and run wind sprints 3 or 4 50 yard ones when I do this. I have tried to lower my top number by diet, I weight 198 and I am 6 feet one half inches tall. I take vitamins and some mineral supplements. I have C -6 Radioparty nerve damage to my left side of my neck and runs down my left arm from an industrial fall accident. I do not smoke or use tobacco in any form, I do not drink alcohol or use drugs. I do not eat greasy, salty, or fried foods . I have a compression fracture of my lower 5th vertebrate in my lower back and have 2 discs dissolved. I have been at this weight + or -- 5 lbs for the last 7 years. What else can I do to lower my systolic pressure? Doctor: Hello welcome to HCM. I have read your question and understand your concerns. Your systolic blood pressure is in the category of grade 1 hypertension which indicates that even after every effort you have tried if blood pressure remains same on multiple times than your doctor should start a mild dosage of blood pressure medicines. My advice to you is - 1. Actually you should mention your age frst because after the age of 60, we usually allow systolic blood pressure up to 150mmhg. 2.Ask your doctor about 24 hrs ambulatory blood pressure monitor, which will be given to you at home and it will record your blood pressure for whole 24 hrs including sleep. It will give definite idea about how much is the fluctuations in blood pressure. 3.Go for lipid profile, thyroid profile, kidney profile and blood sugar levels. As all of them can cause rise in blood pressure. 4.Continue all other measures which will control blood pressure. Thank you. Have a nice day."
},
{
"id": 13788,
"tgt": "Suggest remedy for rashes",
"src": "Patient: hi i have a rash around my neck,chest,shoulders and down my back.I stayed away in a complex with out taking my own bedding,so slept on top of duvet for three nights.I have seen a doctor and we agreed it may well of been bed bugs.He gave me Foban cream-but 16 weeks later i still have the rash,not so itchie but anoying,imbarising and it wont go away.Can you surgest a cure for me please.SORRY I WAS TOLD THIS WAS A FREE SERVICE. Doctor: Hi, You can try low potency over the counter steroids like Betamethasone for symptomatic relief. You can also take antihistamines like Pheneramine in case of itching. Hope I have answered your query. Let me know if I can assist you further."
},
{
"id": 27457,
"tgt": "Suggest treatment to control blood pressure",
"src": "Patient: I have had elevated blood pressure for about 3 months now. It is as high as 188/100. I get red rash on face, chest, and upper back. I itch all over, not just at rash sites. I ve been on a mild blood pressure pill for about 2 weeks, but I can t tell that it s helping. My blood pressure has always been on the low side, until recently. I get a low grade fever when I have the rash. This mostly occurs in the evening. I do have fibromyalgia, I ve been told. I have anemia. I m 63 year old female. I also have an awful lot of joint pain and stiffness, although, the past week it has been better. Can you help me to figure this out? Doctor: Hello!Thank you for asknig on HCM!As you are having persistently high blood pressure values, it is recommended to optimize your antihypertensive treatment.But before doing this, you must check your actual clinical status biginig with a comprehensive physical examination to address all those systemic complains you are experiencing: identify possible skin infection, inflammation or hypersensitivity, etc, articular involvements, as well as other possible body system implications.In addition, some medical tests, like resting ECG, chest X ray study, complete blood count, PCR, liver and renal function tests, thyroid hormone levels, electrolytes and fasting glucose, etc. would be necessarty to identify any possible organ involvements.If suspicions for allergy of infection are sustained, then further more specific tests would be necessary.Facing possible renal or liver function implications, an appropriate antihypertensive therapy optimisation would be feasible.You need to discuss wit hyour attending doctor (internist or cardiologist) to review all medical tests and find the right management strategy for your complains (possibly addition of combinet antihypertensives, treatment of inflammation, etc).Hope to have been helpful to you! Greetings! Dr. Iliri."
},
{
"id": 73829,
"tgt": "What does this chest X-ray report indicate?",
"src": "Patient: I have RA and went off plaquenil due to side effects now I have been diagnosed with asthma and chest xray shows left hilum calcified lymphnode and calcified graniola in my left lung last week. My grandmother had lymphoma and my grandfather had RA, is this something to check further? I have appointment with my RA doc next month. Doctor: Thanks for your question on Healthcare Magic.I can understand your concern.Yes, you should definitely get done CT thorax.Rheumatoid arthritis (RA) can involve lung and cause nodules cavities, fibrosis.Chest x ray is screening tool for lung diseases.Since you have Calcified nodule and lymphnode on chest x ray, CT scan is needed.CT scan is 3 dimensional imaging of lungs and have better quality and interpretation about lung lesions than chest x ray. Hope I have solved your query. I will be happy to help you further. Wish you good health. Thanks."
},
{
"id": 12177,
"tgt": "Should I be concerned for few grey hairs on my toddler ?",
"src": "Patient: my toddler has a few grey hair on his head . should i be concerned ? Doctor: Hi Arpita, Welcome to HealthcareMagic Forum. Does he have any other symptoms like, Changes in skin pigment, Decreased muscle mass, Diarrhea, Failure to gain weight and grow, Fatigue, Large belly??? If yes, he needs a Further evaluation by a Pediatrician to rule out if he has Kwashiorkor, It is a form of malnutrition that occurs when there is not enough protein in the diet. He will require a Urinalysis and Blood work up. Apart from grey hair, if he has no other symptoms, Then the cause may be hereditary, that is, it may run in families. It is always better you get him evaluated by a Dermatologist. Feed him more calories and Protein rich diet which should include, eggs, chicken, milk, Fish, fresh fruits and Green leafy veggies. Calories are given first in the form of carbohydrates, simple sugars, and fats. Proteins are started after other sources of calories have already provided energy. Vitamin and mineral supplements will also be needed. I Wish him Good Health, Take Care."
},
{
"id": 62545,
"tgt": "Suggest treatment for swollen lymph node in the armpit",
"src": "Patient: Hello I am 31 years old and for the last 6 months I have been getting swollen lymph nodes under my armpits no injuries no cuts from shaving and I have been fine no sickness I went to the Drs a month ago and they told me that I had a lump just above my breast tissue and not to worry but I don t understand why my lymph nodes keep swelling up and becoming hard Doctor: Lymphnodes in axillary region (as a matter,,anywhere) mostly get enlarged and become palpable (toucheable) to our hands....mostly due to a primary source of infection nearby....in ur case it cud be infection within breast tissue or upper limb.....but however the problem becomes bigger if they appear...\"hard\" or continuously enlarging and are multiple on both sides(armpits)...and in other regions like neck and groin regions......In these situations....a malignancy(cancer) arising within lymph nodes(lymphoma) or arising elsewhere (in ur case breast mass shud be looked upon as a possible source).must be searched for.......However....a Gen.surgeon needs to examine u carefully and with help of some investigations like FNAC or Excision Biopsy from the node, an accurate diagnosis can be made easily and treatments started as required.... RegardsDr Rakesh Akula MS"
},
{
"id": 35614,
"tgt": "Suggest medication for infection in upper part of body",
"src": "Patient: .last year my father had a heart surgery. in this year my father has a infection in the upper part of his body. today blood came out from that infection & it does n,t stop till now. so, sir please suggest some medicine or anything else, cause we are currently out of town. Doctor: Thanks for asking in healthcaremagic forumIn short: Infection requres antibiotic prescription which is not a over the counter medicineExplanation: It isnot clear about the symptoms of your father. Where is he having infecton ie, chest/throat/skin ? Please send the details for any suggestion. Waiting for your details."
},
{
"id": 206525,
"tgt": "What causes the lack of ability to think freely?",
"src": "Patient: i have a query regarding my father.for the past couple of years he speak very less,he doesnt reply to any question being asked straight away.he has been a kind of misanthrope.he assumes that watever he thinks he speaks is right and rest all are wrong.the way he reacts to things is weird. i think his ability to think freely has been obstructed and there is some disorder.i want him to get cured and back to normal and happy life.please help Doctor: Hello,The symptoms which you have mentioned does not point to any diagnosis. But i advice you to take your father to a nearest psychiatrist for detailed evaluation and treatment before the problem increases further.Thanks."
},
{
"id": 69589,
"tgt": "What is the cause and treatment of bump on forehead ?",
"src": "Patient: I have this weird bump on my forehead, and it's been there for about 2-3 weeks. I'm not sure what this thing is, because it isn't a pimple because when i try to pop it nothing comes out, it just turns from skin color (white) to red like blood is flowing into it. Any ideas on what it could be? Doctor: Hi ! Good afternoon. I am Dr Shareef answering your query.I would advise you to report your self to a general surgeon in your area for a clinical examination of the lesion, as it might not be possible to opine on the same over net without examination and further investigation as per the need.Thanks for choosing health care magic to clear doubts on your health problems. Wishing you an early recovery. Dr Shareef."
},
{
"id": 27087,
"tgt": "Suggest treatment for low blood pressure and low pulse rate",
"src": "Patient: I care for a 79 year old man...going through a lot of testing...everything within normal range....blood pressure and pulse way to low...very weak and tired.....heart is good...had echo cardiogram done....what else can be done for him? Has dementia/alzheimers going to neurologist tomorrow had mri done...getting results. Doctor: With my exp. all his MRI etc will be normal. His low BP is due to dementia resulting in poor conditioning. Low heart rate is not a problem unless he has heart block to dx by EKG. Best treatment is hydration, keeping him active and engaged esp. walking to increase his muscle tone. must be careful to avoid his falling."
},
{
"id": 125733,
"tgt": "What causes pain and stiffness behind the knees?",
"src": "Patient: I am a 56 yo male, that runs marathons and does yoga. I have pain in the back of my knee, it is sore and stiff after I sit a bit and move from sitting to walking. After I walk a bit the soreness deceases. The pain started as a weird feeling behind my knee cap after a yoga class. Then the next am I ran 3 miles and had to stop due to the pain. Doctor: Hi, Pain and stiffness in the knees can be due to developing arthritis (RA or OA). Pain and stiffness behind the knee are usually due to Baker's cyst. Baker's cyst can also cause swelling behind the knee and also stiffness and discomfort while flexing the knee. There are times when Bake's cyst goes away by itself but at times it needs to be treated. Visit ER, get all tests for RA done and get diagnosed. RA or any injury to the knee commonly triggers Baker's cyst formation due to excess secretion of synovial fluid. Physical therapy, steroid injections or drainage are effective in treating Baker's cyst. Get evaluated and treated at the earliest. In the meantime, for relief from pain, you can take NSAID (Naproxen.) Hope I have answered your query. Let me know if I can assist you further. Regards, Dr. Nupur K, General & Family Physician"
},
{
"id": 211189,
"tgt": "What am I suffering from if I feel worthless, no hope in future, no social instincts, staying indoor, like God speaking to me?",
"src": "Patient: hi doctor.tks 4 ur time,im depressed 4 yrs,,feeling worthless,,no hope 4 da futhure,,wishing god wld take me,,i dont want 2 c my friends anymore,,they cant understand why im like this,,i stay indoors all of the time,i do have prayer meeting sometimes and really get on a high when god speaks 2 me.,looking 4ward 2 ur reply.thank u. Doctor: Hi welcome to health care magic forum. You have come exactly in time, you are turning from the depression, to the higher form. You should immediately see a psychologist for diagnosis and treatment. With in 15 days you will be in a position to be a social person. You can meet your friends, can take desicions, and can can lead peacefull life. Take more of green leafy vegetables, pulses, sprouts, and protein rich foods to have a healthy body and mind. Practice, yoga meditation, reading books, and hearing music. Wishing for a quick and complete recovery.Thank you."
},
{
"id": 110825,
"tgt": "What causes squeezing tingling numbness in back?",
"src": "Patient: Hi I have squeezinG. Tingling and numbNess feeling in back of. Neck an area in left shouldder blade and Some tinglinG in left hand with fingertip numbness. It will subside but comes bAck more intense with dizziness and some nausea when not happening. I feel some preessure on back of neck. I hAve had 3 neck surgeries with hardware but have never felt Anything like This. Should I go. To er or just wait to see. My neck surgeon? Please help I m scared I do have high blood pressure and thyroid disease Doctor: Hello, Thanks for your query.According to your symptoms it looks like there is compression of nerve in cervical [neck area] spine. This leads to numbness and tingling in your hand.You may need to take MRI spine to see for severity of nerve compression.Some medication like methylcobalamine with muscle relaxant and analgesic will help, you can take them consulting your treating doctor.Some exercises which can be done after pain has reduced include- gradual neck rotation, touching your ears to shoulder, shoulder shrugging exercises. You may consult physiotherapist for further guidance. He may start INTERFERENCE THERAPY, ULTRASONIC THERAPY, which are helpful in your case.Some life style modifications for you \u2013 Take break every 40 min from computer and do some neck rotation, stretches, walk and get back to work. Sit with taking support to your neck, do not use thick pillow in bed.I do hope that you have found something helpful and I will be glad to answer any further query.Take care"
},
{
"id": 147220,
"tgt": "S1 nerve sheath cysts, on each side of spine, not filled with liquid. CT scan, biopsy, needle aspiration done. Are these tumors?",
"src": "Patient: I have been diagnosed with S1 nerve sheath cysts . One on each side anterior to the spine . I went for a fine needle aspiration with ct and biopsy two weeks ago. They were unable, after many attempts and punctures of the cysts to pull any fluid. Not even enough for a biopsy. I am now told by neurosurgeon that he still believes these are cysts and another biopsy is not necessary. That we are going to leave them be. I have never heard of a cyst with no fluid. Two nurses did tell me they are not fluid filled and may be tumors. I m just confused. Please help. Should I worry? Should I push for another biopsy, and if so what type? Doctor: HIThank for asking to the HCMI really appreciate your concern, and I would like to say that this is such case that without observing the complete case report including the image film it is not possible to say any thing for sure from the history given I could say that you need to have the second opinion and concentrate on the clinical symptoms this is the very crucial factor as long as the prognosis is concern, take care and have a good luck."
},
{
"id": 58585,
"tgt": "Sudden chills, fever and vomiting, CAT shows normal gall bladder, elevated liver enzymes. Causes ?",
"src": "Patient: Hello Dr. Grief. My husband has been suffering from sudden chills, fever & vomiting. Physician thought it was probably gall bladder (stones) but after a cat scan and sonogram showed the gallbladder was fine. However, the liver enzymes are elevated so Monday he is being referred to a gastroenterologist. Are you willing to give it a shot as to what the problem could be? Doctor: Hi,Thanks for using Healthcaremagic,Your husband is having fever with chills and vomiting, this is good his CAT scan and ultrasonography is normal.He has raised liver enzymes there are few common possibilities viral hepatitis, or other viral fever, other differential would be malaria, toxic injury to liver like due to alcohol or even few drugs?He needs clinical examination along with history to know the cause, Complete blood count, smear and antigen test for malaria, Urine analysis, history of alcohol intake and drugs. Treatment depends on cause.He must take rest and if fever is there tepid sponging can be done. He should be well hydrated and take light diet as tolerated.Hope this information helps.Take care.Good Luck.Dr.Akhilesh Dubey M.D."
},
{
"id": 13219,
"tgt": "What causes red rash with tiny bumps on the side of the wrist?",
"src": "Patient: i had a red rash about the size of a fifty cent piece for 5 days. it began to fade away and i have 3 red small bumps in a line in the middle of what was the rash. it did not itch and it is located on side of left wrist right on my vein. should i be concerned? Doctor: Hi, As per your query you have symptoms of red rash with tiny bumps on the side of the wrist which seems to be due to contact dermatitis. Well, there can be many reasons for symptoms you mention in a query like an allergic reaction, psoriasis, skin infection, fungal infection. Need not to worry. I would suggest you keep your skin clean and dry, protect it from the sun. If the condition doesn't get well then consult a dermatologist for proper examination. The doctor may prescribe immunosuppressants along with anti-inflammatory and corticosteroid ointment. Drink plenty of fluids to rehydrate the body. You should apply only petroleum jelly and take an antihistamine like benadryl. Avoid touching or pricking rashes. Hope I have answered your query. Let me know if I can assist you further. Regards, Dr. Harry Maheshwari, Dentist"
},
{
"id": 39445,
"tgt": "Are malaria and kidney infection symptoms similar?",
"src": "Patient: i have a friend who is in the hospital today. she had fever & trembling. the doctor check the blood for malaria & the result is positive. The doctor knows she got diabetes but they did not do the test. 2 hours ago i told them to check for blood sugar & it is 12. She is having a drip with glucose, so the doctor stop the drip. I'm worried if she doesn't have malaria but has infection in the kidney. Her other kidney was removed last year. Please advise. Doctor: I have read your question.she is diabetic with single functioning kidney and is admitted with fever due to malaria. she is at risk for chronic kidney disease due to diabetes mellitus. malaria can cause damage to kidneys due to breakdown of red blood cells if it is due to plasmodium falciparum. i would advise treatment with antimalarial drugs as per type of malaria and good control of her sugar levels..infection of kidney is an independant entity and is not associated with malaria. i would request for urine routine examination and serum urea and creatinine levels. however malaria is a treatable infection with drugs and she will be in good health with treatment. i hope i have answered your question."
},
{
"id": 146003,
"tgt": "Can brain TB with lack of hands and legs movements be treated?",
"src": "Patient: Hi Doctor, My father suffering from TB in brain. Doctors confirmed after MRI and CT scan. Treatment started immediately after scanning result and having antibiotics since last two months. In initial stage he was face some visual issues and now his condition is not good . Now he can t possible to move his hands and legs. Not eating food and his boy is very weak now. Can you please suggest is it curable and which is the the better hospital in India for Brain TB. Doctor: Hi,Thanks for writing in.Brain TB is a condition that requires proper investigations and treatment. By doing CT and MRI scans the features of TB can be confirmed. It is important to examine your father in detail clinically and then treat his condition. He requires taking TB medicines for a long time and also he must be taking medications to control seizures and improve brain function. A lot depends on neurological rehabilitation. His arms and legs can be made to move slowly in the beginning and then slowly he can pick up his neurological functions. Recovery cannot be predicted completely and requires a lot of patience and family participation. The best institutes in India include AIIMS NewDelhi, NIMHANS Bangalore, SGPGI Lucknow, PGI Chandigarh, JIPMER Pondicherry to name a few."
},
{
"id": 8357,
"tgt": "Is Meladerm an effective skin cream?",
"src": "Patient: I am Evan from Bangladesh.i want to know about meladerm cream.how does it work? Is it good for skin? if this product good for skin i will purchase this. i am searching this product all over Bangladesh.but cant find this. please help me to provide this answer where can i found and how much ? Doctor: Hello,I can understand for using the meladerm cream for your skin.This cream basically contains kojic acid and vitamin C as its main ingredient which are though quite safe but should always be used under dermatological supervision.There are various type of skin textures and accordingly we have to decide the product according to the sin type and skin pigmentation. So I always suggest my patients to use any of the cream under the supervision of a dermatologist only.I suggest you to use sunscreen in day time according to your skin type and plenty of fruits and good nourishing diet to make your skin glowing.So I suggest you to meet a dermatologist and get proper treatment for your exact concern for the skin.My all best wishes for your healthy and glowing skin."
},
{
"id": 212545,
"tgt": "Mental problem. Repeat things , forget spellings. Was in special ed classes . Will i be able to lead normal life?",
"src": "Patient: i was just wondering if i had some sort of mental illness . i do the samethings everyday, act like a child, have meomory problems and only 24 soon to be 25 next month. i had balance problems as a kid and always was in special ed classes growing up. i have to learn things over n over before i would get it n then forget it at a drop of a hat. my spelling n grammer sucks n i suck at math. i am always finding myself doin the same things everyday. Doctor: Hello and welcome to Healthcare Magic. Thanks for your query.I understand that you have been facing a lot of difficulties regaring your memory and learning. It is very difficult to pin point a diagnosis without a detailed evaluation. But with the brief history of your symptoms, it is possible that you could be having a learning disability.I would advise you consult a psychiatrist to have a detailed evaluation of your problems, in order to arrive at a diagnosis. Further management needs to be be planned according to the diagnosis, and the nature and severity of your problems.Wish you all the best.Regards,Dr. Jonas SundarakumarConsultant Psychiatrist"
},
{
"id": 70733,
"tgt": "What cases pressure in chest?",
"src": "Patient: Started with pressure in my chest then I started to feel weak. Last night I could not get out of bed my bones hurt and skin is sentitive to touch. Loud sounds make me jump out my skin. Had a cough with green mucus now it has subsided a bit but hurts when I do have to cough. I also have pressure headache and chills. Then every once in a while my face will stary to burn up. Doctor: Hello, As you explain the history I recommend you to see your doctor for a better evaluation of your situation. Hope I have answered your query. Let me know if I can assist you further. Regards, Dr. Jnikolla, Pulmonologist"
},
{
"id": 166998,
"tgt": "What causes liver enlargement in the fetus?",
"src": "Patient: hi Doc, I am 29 years old and weight 95 kgs. I had a baby last month and she died soon after the birth due to the liver enlagrement that is what my doc says. My due date was 12 may and i got operated on 16 may i dint get and pains for delivery so i got operated. when the baby born, baby had passed stool inside my womb ans it has already drank. please i want to know the reason for its death... and the reason for the liver enlargement. please reply me doc i want know will it effect the 2 nd kid..? Doctor: hi!! I can understand your concern. in my opinion you had a post dated pregnancy which might be the reason for passage of stool in utero. but that is not necessarily related to the liver enlargement that your doctor mentioned. liver enlargement could be a congenital problem which needs proper investigation. there is a risk of recurrence of this issue in your next pregnancy so I would advise you consult your doctor in your next issue."
},
{
"id": 152390,
"tgt": "Is a feeling of pressure above the colon upon bowel movement a sign of colon cancer?",
"src": "Patient: I was noticing on recent bowel movements that I feel pressure several inches higher in the colon. I have had mild hemorrhoids and bleeding, but this does not feel natural. Could this be signs of colon cancer? I eat oatmeal, apples, and drink mineral oil. I also take a probiotic, stool softener, and herblelax and occasionally miralax. Thanks! Doctor: Hello, At the moment that you have diagnosed hemorrhoids I don\u2019t think that you should worry. Hope I have answered your query. Let me know if I can assist you further. Take care Regards, Dr Esmeralda Sera, Oncologist"
},
{
"id": 135046,
"tgt": "Suggest medication for swelling & bruise on arm due to an injury",
"src": "Patient: Hi. I had a chair thrown into me last night to my arm and it is now swollen, brushed & grazed. I have cleaned the graze with tap water and coated it in Germolene, it stops hurting when that is applied but without it, it starts to hurt. What is the best way to treat it? Thankyou. Doctor: Hi..Welcome to HEALTHCARE MAGIC..I have gone through your query and can understand your concerns..As per your complain it seems that due to sudden blow here has been injury to the soft tissues of the arm along with laceration of the skin..Swelling is due to inflamed soft tissues along with accumulation of inflammatory fluid while bruising occurs due to rupture of superficial blood vessels due to trauma leading to accumulation of blood in soft tissues causing bruising and hematoma formation..I would suggest you to clean the lacerated area with antiseptic solution and apply a combination of antibiotic and steroid ointment over it..Do cool compresses over the swelling..You can take anti inflammatory painkiller like Aleve or Motrin..You can also apply Heparin gel over the bruise to fade the bruising faster..In case if you got the laceration that was contaminated with dust or dirt you should get a Tetanus Toxoid injection..Hope this information helps..Thanks and regards.Dr.Honey Nandwani Arora."
},
{
"id": 138582,
"tgt": "What could painful gas bubbles on rib cage suggest?",
"src": "Patient: Hello! I m Jamie, and I keep having this gas bubble in my rib cage area. It s on the left side of my rib cage on the bottom of the rib cage. The gas bubble will pop and come back approximately 20+ seconds. It s been causing a lot of pain, and it is hard to breath when it comes. What should I do? Doctor: HelloI have read your query. There is possibility of pneumothorax leading to symptoms. I will advise you x ray and CT scan. CT guided aspirations will help you. You may need clinical examination and further blood investigation. i hope I have answered your questions. If you have further questions please feel free to contact us. I will be happy to answer. Take care."
},
{
"id": 65576,
"tgt": "How to get rid of bump on my butt cheek?",
"src": "Patient: I have a bump on my butt cheek that had been there for years, I m starting to wonder what it is. It is skin colored, and not painful at all. Recently I ve tried to squeeze it away like with a pimple, and this Brown stuff came out and it kind of squished/molded to where I squeezed. But it only stays smaller for a couple days. Any idea of what kind of bump it is? Doctor: Hi! thanks for writing to us sharing your health problem!After going through your description of the long standing, painless, skin-colored bump on your butt-cheek filled with brownish paste-like material I would like to consider only one one possibility that is a inclusion cyst or sebaceous cyst!You should not worry at all for this but take care not to squeeze or injure it anyway as that could invite serious infection! Therefore, please consult a local surgeon for a clinical examination and surgical removal which is the definitive cure for it!Regards,"
},
{
"id": 98615,
"tgt": "Suggest treatment for allergic bronchitis",
"src": "Patient: Dear,sir i am suffering from allergy .every day flow of water from my nose(nak pradah,or nazla) from 2-3years.my treatment is continuously Patna &, muzaffarpur allopathy & hemopathy but problem is not properly sought out.nearby I am suffering from asthma.hence sir I am very hopeful for you regarding solved my problem .I am very tried. Dr.said that allrgy bornckytis,some dr said allery rainitics&some dr said sinius.pl.help me sir for better treatment and suggested best doctor at patna.thank you Doctor: Hello Sir, I just have gone through your problem. Being allergic now a days is very common problem because of decreasing immunity and increasing pollution. First of all we have to work on your immune system. Problems like allergy are raised because of low immunity. SO to increase your immune level I would like to suggest you some tips:1. Add dry fruits in your diet like Walnuts, Almonds, Cashew etc.2. Take a good Multivitamin capsules from market which includes antioxidants too.3. Keep frozen,fried, canned or precooked foods or fruits at bay.4. Eat healthy and freshly prepared food.5. Eat fresh fruits specially citrus fruits. Citrus fruits contain vitamin C which helps in building up the immunity.6. Drink a plenty of water. Keep yourself hydrated.7. Try to drink hot drinks instead of cold ones. Cold things aggrevate allergic reactions.Regarding medicines I would like to prescribe you Ayurvedic medicines..which can be proven to be helpful to you. 1. Laxmivilaas ras 2 bd after meals with one glass full of water.2. Pratishyaghan wati 1 tds after meals one glass full of water.3. Afsantees powder 5gm + Giloy satv 1 gm - 2 times with honey empty stomach.4. Tab soothshekhar ras 2bd after meals with one glass full of water.5. Add one bay leaf one small cinnamon stick one clove and one black pepper globule in hot boiling water...let them simmer for 10 mins and then take the steam. This will open the airways of your lungs and the breathing will be easy.I hope above mentioned treatment and remedies will help you. Still if you feel like asking me anything , please contact meWish you a speedy recovery.Dr. Tripat Mehta"
},
{
"id": 217075,
"tgt": "Suggest remedy for headache,nausea and back pain",
"src": "Patient: I started with a bad headache 6 days ago, then sickness, maybe because of the pain. A couple of days later, suddenly in the middle of the night I woke up with back pain across the sacrum area, spreading down the middle of the back of the thigh, both sides. I am unaware of any exercise/inury sustained recently. Once a very strong painkiller wears off, it all comes back: headache, nausea, back pain. What can it be? Doctor: Consultant your doctor ..what about blood pressure. ...headache with nausea hypertension should be ruled out. ..for sacrum pain start tab Pcm 500 mg sos"
},
{
"id": 156467,
"tgt": "Suggest treatment for prostate cancer iv stage",
"src": "Patient: Dear Sir Good morningMy father have Prostate Cancer iv Stage, he have operated Hormone therapy ,currently we have tested PSA 25 right now 1) He is not able to walk and talk2) not able to take food 3) not able to identity the person alsoPlease suggest , what can i do for my my father Doctor: Standard treatment options for stage IV prostate cancer include the following:1.Hormonal manipulations.2.Bisphosphonates.3.External-beam radiation therapy (EBRT) with or without hormonal therapy.4.Palliative radiation therapy.5.Palliative surgery with transurethral resection of the prostate (TURP).6.Watchful waiting or active surveillance.RegardsDR De"
},
{
"id": 192627,
"tgt": "Do vasectomy or prostasis cause ejaculation to have brown coloring?",
"src": "Patient: I recently had a vasectomy in the last 4 weeks. A few weeks prior to that I was diagnosed with Prostasis and treated with Cipro. In the last few days I have discovered my ejaculate has a brownish tint. Could this be due to the vasectomy or the prostasis and should I be concerned? Doctor: Hello, It is quite common after vasectomy and is due to minor blood staining of the semen. Nothing much to worry and it will settle by itself even without any treatment. Hope I have answered your query. Let me know if I can assist you further. Take care Regards, Dr. Shinas Hussain, General & Family physician"
},
{
"id": 134385,
"tgt": "What causes finger and toe numbness?",
"src": "Patient: I am trying to get some possible answers to some symptoms that I have been experiencing. I have numbness from my forearms to my fingers, especially my left. My toes also get numb at times, and tingling in my face. I am also experiencing bouts of depression. I have done cardio and neurology scans and nothing turns up. Just trying to get another opinion. Doctor: hi,first of all thanks for mentioning little part of the history. sometimes depression also leads to unnatural sensation too. if your cardio and neuro advised you that all fine then thumps up. it's not a major issue. sometimes spasm of the muscles leads to compression on the nerves beneath it. so doing some regular exercises for like arm swinging and deep breathing will help. also try to perfrom simple neck exercises and simple straight leg raising for the Strengthening the core muscles. you will feel ease of your symptoms. if pain still persist then you can take an advise from an ortho or a physical therapist.thank you"
},
{
"id": 67381,
"tgt": "What causes soft swollen elongated lumps across both shoulder blades?",
"src": "Patient: I have soft swollen elongated lumps across both of my shoulder blades. I have pressure at the base of my throat and I feel like I have an object at the back of my throat. I have hypothyroidism and take synthroid for this. I have had 75% of my thyroid removed in a previous surgery. Could the remaining thyroid be causing this swelling, pressure and pain? Doctor: Hi.Thanks for your queries- you have three different ones:The soft swollen elongated lumps across both shoulder blades can be due to a condition called papilloma, Get them checked and removes as they are hanging at the pressure points of the body. you get 2 benefits- 1. the disease is removed and 2. you get the histo-pathological diagnosis. The feeling like an object behind the throat can not be due to thyroid for sure as thyroid is lying low in the neck. There may be lymphoid hyerplasia or some real lump or may be just a globus hystericus related to psychological problem. This may indicate the anxiety you have or the not so good thyroid control. Since you are on synthroid the remaining thyroid can not increase if you are on the right dose. Consult an ENT Surgeon to get the examination done. Get the related endoscopy and other tests done to get an appropriate treatment ."
},
{
"id": 118574,
"tgt": "Diagnosed Spleenomegally, anal and mouth bleeding",
"src": "Patient: I am suffering from spleenomegally. my spleen has enlarged and it has become 15 cm in length. My platelet count become 0000 and frequest suffering from bleeding through mouth and anal. Please advice a spleen specialist or any specialist for treatment immediately. Regards Doctor: Dear sir/madam we need to see the cause for splenomegaly in the first step and later the decision about the treatment shuld be taken....in last option you can have a splenectomy....please consult a medical oncologist he will do.need full"
},
{
"id": 216202,
"tgt": "Suggest remedy for pressure in eyes due to bump on head",
"src": "Patient: Hello. I hit the top of my head on a metal bar at work over a month ago. I got a small cut on top of my head and a bump on my forehead above my left eye. The cut went away and the swelling of the bump went down but there is still a small bump and it hurts a bit when I touch it and if I look up I can feel pressure from that area to my left eye. Should I be worried? Doctor: Dear patient You have got hematoma over your eye post trauma which is resolved partially. There is nothing to worry as it will resolve over time. If in doubt local ultrasound of the swelling will be confirmation of diagnosis. Tab chymoral forte twice a day before meals will help in early reduction of swelling."
},
{
"id": 203602,
"tgt": "Are the findings in the semen analysis report normal?",
"src": "Patient: Hi, Greetings to you doctor. I have done my semen analysis ane below are reports. Microscope Examination Sperm count - 56 millions /ml Total Nornam forms - 60% Abnormalities Tail abnormalities: 10% Head abnormalities: 10% Degenerated form: 20% Pus Cells : 1-2/hpf Red blood cells: not detected Epithelial Cells: Absent Precurson cells: 0-2/hpf Nature of moblility A Rapid linear progressive IV &III: 20% B Sliggish Linear Progressive II : 30% A+B Total forward progressive : 50% C Non Progressive I : 30% D Non Progressive 0 ; 20% Could you please analyse and kindly let me know if there is any abnormalities in this. my mail id is : YYYY@YYYY Many thanks in advance with warm regards Laxman Doctor: Thanks for query Laxman.your semen analysis is absolutely normal.counts,motility,normal forms are all within normal limits."
},
{
"id": 164912,
"tgt": "Which kind of worms appear in stool of a child?",
"src": "Patient: I saw a couple of long red/dark pink flattish things (not sure if they were worms) in my 4-yr old's stool 2 days after i gave her anti-helminthic drug (zentel). Were these worms? if so, which kind of worms were they as they were really long and seemed to come out with every piece of stool. Doctor: Hello and welcome to healthcare magicFrom your discription.i can conclude that they were worms.Long worms in clusters are more likely roundworms (Ascaris lumbricoides).There presence can be confirmed by stool examination for ova and parasites.Hope you find this answer satisfactory.Good luck."
},
{
"id": 60471,
"tgt": "Which syrup can be used to cure liver jaundice ?",
"src": "Patient: which is best syrup for liver jaundice to cure ? Doctor: Hi,Harcharan Singh, Thanks for query, There is no medicine to cure jaundice,the medicines given are all supportive. Take B.complex syrup,vit c and syrup contain liver enzymes. Important is take rest,more glucose to protect liver. Avoid alcohol. ok and bye."
},
{
"id": 7577,
"tgt": "Suffering with acne, medications resulted in scars, severe hair fall. Help?",
"src": "Patient: i m suffering from acne since last 4 yrs..went to homeopathy treatment but tht increased my my condition to worse..then showed a dermatologist n he gave me isotorein 20 mg i had for a month my acne was cleared but red scares were left..n stopped the medicine than coz my hair was falling a lot, all was ok till 8 months again i started getting acne ,... again i showed to a doc he gave me the same medicine,,,i had for 10 days .. by more acne statred coming on my face n hir fall problem.. d acne r very hard n thy donot tinture ..he had given me flucort cream to apply .. rite now i had stop tht medication Doctor: This type of persistent/ recurrent acne may be due to deficiency of certain vitamins and minerals. You must have done some internal skin antibiotic course. If not, you can take Doxy 100 mg twice daily for 10 days (check that you are not allergic to doxycyclin). Along with this start following vitamin therapy: Vitamins: Vitamin niacin 100 mg, three times a day, and Vitamin A - 50,000 IU, three times a day. Vitamin E - 400 mg once a day. This vitamin therapy should be continued for a month. Mineral: Zinc-50 mg three times a day. Continue for one month and then reduce to 25 mg three times. Garlic is called a wonder herb for acne. Rubbed raw garlic on acne several times a day (after sensitivity test that you are not allergic to garlic). Pimples disappear without scars."
},
{
"id": 36858,
"tgt": "What is referred as trench mouth?",
"src": "Patient: I have what can be referredTo as trench mouth and have been on zpack and steroids for 3 days. I have seen not much improvement but it hasn't gotten worse: I'm scared something could be really wrong with me... But don't want to stress as it is a cause for what I have -- Doctor: Hello,Welcome to Healthcare Magic.Trench mouth is a bacterial infection of mouth and gums in which gums become painful and ulcers also develops. Sometimes bleeding too.It is mainly due to bad oral hygiene, poor nutrition, smoking, alcohol and stress in life.It affects in your age group more.Maintain good oral hygiene, avoid smoking and stress to cure it.Brush twice daily.Apply lignocaine gel before eating.Take paracetamol or ibuprofen to relieve pain.Hope this will help you.Take care..."
},
{
"id": 125608,
"tgt": "Suggest treatment for costochondral separation",
"src": "Patient: I have been diagnosed 2 weeks ago with a costochondral separation. I am worried it s not healing back properly, as a rather large bump has formed. I don t know if this is because the bone and cartilage are healing misaligned, or if it s just inflammation. Doctor: Hello, Most of the cases improve with conservative management. If symptoms persist surgical correction may be required. Consult a thoracic surgeon and get evaluated. Hope I have answered your query. Let me know if I can assist you further. Take care Regards, Dr Shinas Hussain, General & Family Physician"
},
{
"id": 192767,
"tgt": "Suggest diet during treatment for hepatomegaly with Grade 2 BHP",
"src": "Patient: My father is 52 years old. In the recent health checkup doctor diagnoses the problem as Mild Hepatomegaly with Grade ii BPH. My father is a vegetarian and Non-Alcoholic. Can you suggest what type of diet is best suited in the current stage. Also can you tell whether this is a serious problem ? Doctor: Hi, I can understand your concern for your symptoms, avoid oily foods and alcohol. take green leafy vegetables. Hope I have answered your query. Let me know if I can assist you further. Take care Regards, Dr S.R.Raveendran, Sexologist"
},
{
"id": 106278,
"tgt": "INCREASE OF WBC & POLYMORPHS",
"src": "Patient: Hello sir, i am 27 years old (M). On my blood check up Total WBC comes 13800/cmm , ( doctor sail its range is 4000-10000) & Polymorphs comes 74% & Lymphocytes 19%, Eosinophiles 3%. Because of alergy or any infection or any Worm problem, every time ,white cough comes up, my energy level comes very down & some time there is quite breathing problem also. I dont want to go for any alopathy. So PLEASE give me its solution in any ayurvedic way. VERY thankx in advance. Doctor: Hi, The blood report is normal it does not show any features of allergy or major infection as such. Take Revital multivitamins, drink plenty of water and consume plenty of fruits and vegetables. Regards Dr. Naveen Kumar"
},
{
"id": 92156,
"tgt": "Stomach paining and have Amoebiasis by drinking tap water. How to treat it?",
"src": "Patient: hi, im jane my boyfriend has an ambiases he said hes stomach is aching a lot especially on the left side,and hes balls also is aching hes thinking that its bec. of hes ambiases,is he right about that and what medicene should he take,he got the ambiases from drinking top water here in the philippines. Doctor: HI. One has to make sure that he is suffering from amoebiasis. Is he having stool with mucus? The treatment is simple - to take tab metronidazole 400 mg thrice daily ( I hope is available over the counter). To drink boiled and cool water. This has nothing to do with the pain in the testes. There will be another reason for this . He may add on antibiotics which will help treat the intestinal and testes problem at the same time say like cefixme or oflaxacin, for which he needs to see a Doctor too as these are prescription medicines. He also gets clinical examination by the Doctor."
},
{
"id": 215814,
"tgt": "Suggest remedy for pain in rib cage",
"src": "Patient: complaining of both lower and upper back pain. occupation involves lifting and stacking cartons of varying weight. The pain was also felt in the rib cage. Some areas of feet were red with broken skin in the area of the 5th metatarsal notch. blood pressure was a little above normal and his ears appeared flaky also. Doctor: Hi, The rib pain and metatarsal pain are unrelated. You can take analgesics like Ibuprofen or Diclofenac for pain relief. If the symptoms are severe you can consult a physician and get evaluated. Hope I have answered your query. Let me know if I can assist you further."
},
{
"id": 31455,
"tgt": "Suggest treatment for blisters on legs",
"src": "Patient: my son had friction burns on his leg from playing football on astroturf and he had been scratching it at night, his leg is now full of blisters and looks extremely sore and a clear liquid is coming out of them - he says it is not painful just itchy - i have covered them to stop him scratching but it looks infected to me - what can i do? Doctor: Hi Dear,Welcome to HCM.Understanding your concern. As per your query your son have symptoms of blisters on legs. Well there can be many reasons for symptoms you mention in query atopic dermatitis, dermatitis , herpetiformis , herpes simplex and pemphigus vulgaris infection. Need not to worry. I would suggest you clean that area with betadine , keep it dry and clean , take antihistamine like benadryl and apply over the counter antibiotics cream . If condition persist then consult dermatologist for proper examination . Doctor may order skin patch test , blood test and take sample for lab test . Doctor may prescribe corticosteroid ointment , antibacterial or antiviral depending upon the diagnosis . Hope your concern has been resolved.Get Well Soon.Best Wishes,Dr. Harry Maheshwari"
},
{
"id": 208428,
"tgt": "Suggest medication for panic attacks",
"src": "Patient: Hi, Ive suffered a good few panic attacks in the past 2 months. Mostly when I have been on a night out drinking alcohol. But also in the last week I started to get one while I was training. As I wasnt under the influence of alcohol I was able to control it, rather than let it get completely out of hand. On the very first night it happened I collapsed and then had a panic attack. Doctor: Dear User,Thanks for using health care magic.From the available description it appears that you are self medicating your panic attacks which is going to do more harm in long term. The benzodiazepines group of medicines are recommended for acute control of panic attacks and in long term the antidepressants are right choice for treatment of panic attacks.You are advised to see a psychiatrist who will take more history to find out the suitable medicine for you. You can also put direct query to me with more details.'Hope I have answered your query. If you have any further questions I will be happy to help\".Thanks"
},
{
"id": 184028,
"tgt": "Why my gums are sore and bruised?",
"src": "Patient: I have a crown on a tooth and about a week ago I started having pain next to it. The tooth does not hurt but my gums are sore and it looks like a small bruise next to my crown. The pain has not worsened but has not gone away. should I see my dentist or my doctor? Doctor: Hi,Thanks for posting the query, I would suggest you to apply dologel oral ointment topically over the bruise at home take lukewarm saline and antiseptic mouthwash rinses, maintain a good oral hygiene take multivitamin suplements, if the syptoms still persists get it examined by the Dentist.Take care!"
},
{
"id": 13982,
"tgt": "Suggest remedies for an itchy patch at the base of my spine",
"src": "Patient: I have an itchy patch right at the base of my spine, the skin looks normal and it did not itch while I was away in Greece on holiday, but started again when I came back, it is not constant but seems to occur at random times. Double base cream helps to soothe it. Doctor: Hello and Welcome to 'Ask A Doctor' service. I have reviewed your query and here is my advice.Perhaps it is either a fungal infection or it could be an eczema patch. A fungal infection is usually a ring shaped peripherally expanding lesion with central clearing and raised, red, scaly margins whereas a chronic eczema patch is dry, red and scaly throughout. For a fungal infection I would usually suggest an OTC topical antifungal e.g clotrimazole 1% cream, whereas for an eczema patch, a moisturizer along with a topical steroid cream would usually work. Apart from specific therapy you may also take an oral antihistamine e.g cetrizine 10mg tablet for symptomatic relief.Hope I have answered your queryLet me know if you need any more assistance"
},
{
"id": 39406,
"tgt": "What causes fluid on wounds?",
"src": "Patient: I took a fall while hiking others day and have two 1inch circle scrapes on my elbow. I cleaned them very well when I got home and have kept them covered since then. I notice that when I change the bandages each day that there is a clear fluid laying on top of the wounds. Is this normal? Doctor: The clear fluid is coming from the granulation tissue in the wound. It is normal and is helping in the healing of the wound. Eventually a scab will be formed, which will dry up and fall off."
},
{
"id": 140368,
"tgt": "What causes body odour,brain fog and memory loss?",
"src": "Patient: i have had this body odour for over a year i am 21 years old, i have got this type of brain fog which is realy not going well with my studies i used to be realy smart but now my meomery has been a bit degraded due to this at that time i used to drink heavly,wondering if what it might be a symptom of please help Doctor: Hello, Your symptoms could be related to a metabolic disorder. For this reason, I recommend performing a brain MRI and checking vitamin B1, vitamin B12 plasma levels for possible deficiency. Some other blood lab tests may be needed (complete blood count, PCR, ESR, thyroid hormone levels, uric acid plasma levels, fasting glucose). You should discuss with your doctor on the above tests. Hope I have answered your query. Let me know if I can assist you further. Take care Regards, Dr Ilir Sharka, Cardiologist"
},
{
"id": 165529,
"tgt": "How can a growing ear lump be treated?",
"src": "Patient: my baby had a small lump behind her left ear from 3 month. shown it to her doc at the time and she said it nothing to worry . Only show if it increses in size. Now my kid is 15 months old. Now i have notice the lump has grown to the size of a small pea. It is moving and not paining. Should i worry ??? Doctor: Hello and Welcome to \u2018Ask A Doctor\u2019 service.I have reviewed your query and here is my advice. I feel that it could be a dermoid cyst. If the lump is not bothering her and is not increasing in size drastically and very fast I don't think you should worry about it. But, if it increases in size fast then you need to see a pediatric surgeon for biopsy and histopathological diagnosis.Hope I have answered your query. Let me know if I can assist you further.Regards,Dr. Sumanth Amperayani"
},
{
"id": 141317,
"tgt": "What causes warm sensation in the neck and head?",
"src": "Patient: Hello I have been experiencing a rush of warmth from my neck into my head, I have never fainted but I feel light headed, and the warmth radiates into the pelvic area. I feel pressure in my forehead. I am not on any meds and am 58 years old. BP has always been low to normal. Doctor: Hello and Welcome to \u2018Ask A Doctor\u2019 service. I have reviewed your query and here is my advice. Pressure in head may be due to tension type headache. You need to decrease stress. Drugs like Amitriptyline , Sertraline or Duloxetine can be helpful. Hope I have answered your query. Let me know if I can assist you further."
},
{
"id": 192562,
"tgt": "What causes burnt skin near penis and how to cure it?",
"src": "Patient: dear sir! i am 22 years old male.. i have a problem near laps between penis.. the skin burned bothsides on the laps.. and when i rub on the testies and laps i will get some dust which smells bad.. am worried.. please let me know the reason for this.. Doctor: Hello, You can apply over the counter silver containing antibiotics like silverex ointment. If there is any signs of infection, oral antibiotics also be required. Hope I have answered your query. Let me know if I can assist you further. Take care Regards, Dr. Shinas Hussain"
},
{
"id": 139298,
"tgt": "Suggest treatment for ankle pain",
"src": "Patient: i had a fall in june and severly hurt my ankle but i did not go to the hospital as i thought it was a bad sprain. I have put up with the pain since then and finally had an xray. I received a letter from my doctor to day saying i had to phone for a consultation about my ankle? Doctor: Dear patient You have got chronic pain in ankle that might be due to arthritis or loose bodies. So I would like to know what is your Xray report ? If Xray Is showing signs of reduced joint space it is arthritis. According to me MRI of ankle is better investigation for chronic ankle pain. MRI gives detail information of not only bone but also soft tissue and ligaments. So visit radiology center nearby you and get it done. Visit expert orthopaedic surgeon with report."
},
{
"id": 138404,
"tgt": "What causes headache, lower body pain, varied temperature and dizziness?",
"src": "Patient: I woke up feeling somewhat heavy chested and had a useless cough with it. Within 2 hours I deteriorated to extreme chills and shaking, was hot and cold in different body areas, felt dizzy and incredibly weak and my hands felt limp. And body aches like I ve never experienced all lover, particularly from waist down. I now have a headache and have been trying to drink more water. What s happening? Doctor: Hello,I have studied your case and I think that you are most probably having a viral infection. It is involving upper respiratory tract. So I would recommend you to do following. 1.Meet a physcian and start taking antibiotics like Ofloxacin tablet 400 mg twice daily.2.Avoid cold temperature.3. Take brufen 4oo mg daily. I hope this treatment will be helpful for you.Let me know if there is any other followup questions.thanks"
},
{
"id": 183461,
"tgt": "Suggest remedy for soreness and abscess in gums",
"src": "Patient: My gum desease seems to have worsened since being on Toprol XL and 2 weeks ago I was switched to amlodipine and already I have abcesses in two new places and very sore gums. Is there any BP drug that would not cause my infected gums to get worse. did Toprol contribute to the issue? Doctor: Hello,Medications can have side effects that stress the oral environment especially with new or changed medications that can creat an imbalance. Your system has to take time to adjust. You may experience dry mouth and poorly hydrated tissue. Please maintain excellent daily oral hygiene. Your gums may be extra sensitive and very reactive to daily plaque buildup. Signs of imflammation include redness, swelling and sensitivity. Rinse with warm salt water. Keep a proper diet with balanced vitamins. Avoid deficiencies and make sure to take vitamin B12. Everyone is Individual and react differently to medications.If you had a compromised periodontal condition before starting high blood pressure medication, you are at risk of additional periodontal stress. Increase your reoutine professional dental cleaning visits. Use a toothpaste for sensitive teeth. Consider scheduling a visit with a periodontist if your condition does not improve. Anti-inflammatory medication may offer some relief during this adjustment time period.Thank you for your inquiry. I hope your feel better soon."
},
{
"id": 204549,
"tgt": "What causes pressure behind the forehead and fatigue while suffering from anxiety?",
"src": "Patient: My daughter, now 8, had a rather sudden onset of change in behavior at age 5 1/2 years old. She has increased sensitivity to clothing, sensation, and noise. Has been diagnosed with anxiety, ADHD, and sensory processing disorder. Now she has been reporting a weird pressure like sensation behind her forehead, complains of fatigue all the time , has had noted memory deficits, mood and anger outbursts, cognitive and learning decline in school to the point that speech therapy is treating her for cognition, and the school has initiated an IEP. She has muscle weakness and fine motor difficulties, with OT treating her at school, and her physician recommending further PT. She complains of intermittent stomach pain always around her bellybutton that resolves pretty quickly. She has difficulty with concentration, staying on task, and following even one step directions. Her grades decline drastically throughout the school year last year, even with multiple supports in place. I m questioning the possibility of a physiological cause as the root of all these random symptoms. None are severe enough on their own to draw attention, but combined, they make her life pretty difficult. Her doctors are quick to assign a psychological diagnosis, but like I said, onset was pretty abrupt at the age of 5.5. It was like we suddenly had a different child. And she has just declined since then. Doctor: Hello and Welcome to \u2018Ask A Doctor\u2019 service. I have reviewed your query and here is my advice. The pressure sensation, fatigue memory deficits, anger, cognitive and learning decline, etc. has to be due to her ADHD and anxiety disorder. You should work with your psychiatrist to streamline her treatment. If you require more of my help in this aspect, please use this URL. http://goo.gl/aYW2pR. Make sure that you include every minute details possible. Hope this answers your query. Available for further clarifications. Good luck."
},
{
"id": 128795,
"tgt": "What causes a painful burning sensation in the upper legs while on Fluconazole?",
"src": "Patient: I Have taken about 6 or 7 fluconazole tablets in the last 4 weeks. My upper legs are very painful and its a feeling of burning as well as within the private area. My doctor says I don t have a yeast infection so I may have been treating the burning feeling as yeast. Can this come from too many fluconazole tablets.? Doctor: Hello,This is not a common side effect of the fluconazole but I suggest that you stop taking it and see if the problem goes away. If you do not have a yeast infection then you should not need the fluconazole.Regards"
},
{
"id": 23184,
"tgt": "What does \"borderline positive stress test for inducible ischaemia\" mean?",
"src": "Patient: had a stress test. walked for 6 min 23 seconds Bruce Rotocol. test terminated when reached THR. achieved 87% of THR. moderate effort tolerance. horizontal downsloping ST depressions anterolaterally at peak & recovery. stress test borderline positive for inducible ischaemia at workloadnachived. i am a 54 year old woman. my weight is 60.9 kg. height 168 cm. have rAised total & ldl cholestoral levels (total 228.0 & LDL 151.3 mg/dl) what does all this mean? thanks Doctor: it means u are probably have coronary artery disease and you should see the cardiologist and further tests done like echocardiography, angiography, starts the medicine for the same"
},
{
"id": 103861,
"tgt": "Child have wheezing. Had a sip of soda earlier. What to do now?",
"src": "Patient: My seven years old son had a problem this evening. A sip of soda went down wrong way. He was coughing for about ten minutes, and than stopped. He has an asthma, but did not have the asthma attack, so I did not give him medicine. We were not at home at that time. He fell asleep after we got home, at his regular bedtime. I went to check him and I can hear wheezing. Should I wake him up and give him medicine or let him to sleep? Do we have to go to emergency? Thank you for your answer. Doctor: your soda intake worsoned the conditioni think if you start with short course of steroids you can skip energencytake betnesol 10 mg tdstab montelucast 5 mg tdssyp asthalin 1 tsf tdssyp tossex 1 taf nightdo for 5 days and that control this episode then you can consult your allergydoctor to treat then otherwuise you wont be able to know which food increses the allergies"
},
{
"id": 150439,
"tgt": "Have classified granuloma in brain. Having seuzyres and epilepsy. CT scan done. What to do?",
"src": "Patient: Hi, I have classified granuloma in right side of brain..its recently found after done CTScan, I am facing with Seuzyres and epilepsy..seuzyres stares from my left had after that i will be in to unconscious for 10 to 30 min. this was first time came when i was 13 years, 2nd time 14years , 3rd time 15years, now i am 29 years after log time (13 years) now last month it again came.. plz suggest me how do i over come this Issue . Doctor: Hi Ranjith, Thank you for posting your query. Calcified granuloma refers to the old healed tapeworm infection of the brain, and is a common cause of epilepsy and seizures in India. Treatment requires long term use of antiepileptic drugs such as oxcarbazepine. There is no need to worry. Best wishes, Dr Sudhir Kumar MD DM (Neurology) Senior Consultant Neurologist"
},
{
"id": 48696,
"tgt": "What causes pain in kidneys of teenager with masturbation habit?",
"src": "Patient: sir i am 16 yrs old and i am having minor pain in my kidneys ....i had a excessive masturbation history like 2 times in a day and i am using drug (co-amoxiclav) Augmentin 625 mg of Glaxosmithkline gsk.plz diagnose me. My height is 5'11 and i weight 62 kg. Doctor: Kidney pain has no relation ship with ur mastrubation..its wont vause any problem, its inspite healthy habit of mastrubation once a day..If pain persist go for ultrasound of kidney to ruleout kidney stone"
},
{
"id": 60416,
"tgt": "Can mx3 cure the hepatitis b ?",
"src": "Patient: hi can mx3 cure the hepatitis b , or may it worsen the hepatitis b. Doctor: hi, welcome to hcm thanks for the query instead of taking mx3 you should consult your physician about it he will suggest you some tests like liver function test and complete blood count after all things become clear from the report your doctor will provide you medications follow that treatment plan strictly take care. wish you a good day"
},
{
"id": 81120,
"tgt": "What causes swelling of lip with cold?",
"src": "Patient: Sir I am a Tuberculosis patient and under Continuation Phase of DOTS therapy and have completed 4 months of medication. When I wake up this morning I noticed my upper lip is swollen, slightly numb and has got some light dots. Also I have got cold and runny nose from last 1 day. Other symptoms are that my chest has very slight tinkling now and then. Please help Doctor: Thanks for your question on HCM.I can understand your problem.In my opinion you are having viral upper respiratory tract infection (URTI) mostly. Swelling of upper lip is seen with URTI due to angioedema.It is aleergic reaction.And prompt treatment with antihistamines and oral steroids is needed.So better to consult doctor and discuss all these and start appropriate treatment.With proper treatment swelling of lip will reduce in 2-3 days. So don't worry much, just start treatment. And continue your DOTS treatment."
},
{
"id": 123216,
"tgt": "Suggest remedy for knee pain",
"src": "Patient: my wife is suffering from severe knew pain post 5 months of her delivery .. now whenever she wants to get up from chair or from floor she faces a lot of difficulty .. pain is mostly at the knee area .. also she is not feeling fresh .. always feels very dull and less energy Doctor: Hello, As post pregnancy females tend to have extra kilos which may lead to abnormal pressure on the internal structures of the knee joint leading to inflammation and the pain. I will advise to use a knee brace, do hot water fermentation, do the diet to shed extra kilos, exercises like static Quadriceps, static hamstring, straight leg raise, pelvic floor muscle strengthening, etc to regain full functions of the knee. A physiotherapist will be helpful to her. Hope I have answered your query. Let me know if I can assist you further. Take care Regards, Jay Indravadan Patel, Physical Therapist or Physiotherapist"
},
{
"id": 211015,
"tgt": "Any suggestions for hallucinations, disorientation to time, place, date, sleepiness in a person with Hashimoto's disease & migraines?",
"src": "Patient: My 16 year old granddaughter has episodes of hallucinations, complete disorientation to time, place, date; followed by extreme sleepiness where she can sleep solidly for 8-12 hours. She has been to several neurologists who tell her it's all in her head & have done nothing to alleviate her symptoms-just seek mental health counseling. She has Hashimoto's disease & severe migraines (since age 3). She is about to graduate from High School & attend college, but cannot drive for fear of one of these episodes will cause a wreck. Any suggestions? I can be reached at YYYY@YYYY . Thank you. Sarah Wagoner-worried grandma Doctor: HIThanks for using healthcare magichashimoto thyroiditis cause weight gain, depression, mania, reactive hypoglycemia, constipation, migraines, muscle weakness, joint stiffness, menorrhagia, myxedematous psychosis, cramps, memory loss, vision problems, infertility and hair loss etc. In your daughter, these migraine and psychotic episodes are due to hashimoto's disease. In that case, consult a endocrinologist to control the illness, this would help to control the symptoms.Thanks"
},
{
"id": 102754,
"tgt": "Is it possible to develop white flour allergy at the age of 50?",
"src": "Patient: hello there. is it possible that at the age of 50 i could have developed a white flour allergy? i suffered an intense reaction today after eating 2 crusty cobs with extra mature white cheese on i made myself this morning. hope these clues help. thank you. francis dacey of liverpool. Doctor: May be the type of wheat used for flour may be causing allergic symptoms.Change the brand and hope your symptoms subside."
},
{
"id": 214532,
"tgt": "Suggest home remedies for fat tissue buildup?",
"src": "Patient: I have had Gyno for about three years now after foolishly using a pro-hormone when I was younger, a mistake to say the least,do you know anything about the use of Turmeric root to combat the fat/tissue build up and/or any other possible remedies before surgery. Thank you Doctor: Dear my suggestion to you is that first of change your attitude about carrying fat tissue. You should be fit in terms of BMR[basal metabolic rate], normal blood sugar and a normal blood pressure. Don't go by looks, few people have tendency to put weight and there are many reasons for same. So idea should be to treat the root cause, it can be hypothyroidism, cushing syndrome, hypothalamic causes or a genetic association. Any cause has to be treated to get rid of fat and then prevention off course is better than cure, so go for lots of fruits and salad along with workout at least three times a week if not daily. Treatment with orlistat has some effect as it lowers fat absorption and finally surgery should be the last resort.........tc care"
},
{
"id": 128501,
"tgt": "What causes shortness of breath along with swelling in the ankles?",
"src": "Patient: My feet and ankles have been swollen for the most part of a yr.. They may go down for a day or two, but they always swell back up.. Ive also been having shortness of breath.. My dr ha put me on water pills, potassium, and several different meds, I have changed my diet, and stopped drinking soda and nothing has worked.. Its gotten so bad that its hard for me to walk due to the pain and the swelling is putting pressure on my ankles and causing them to go out.. Any suggestions?? Doctor: Dear patient pain and swelling in both ankles with shortness in breath my first diagnosis would be heart problem. Are you hypertensive? if yes this may be sign of early heart weakness. If you have no heart issues or hypertension local ankle pathology needs to be taken care of. where exactly is your pain ? if it is in lower part of ankle plantar fascitis is likely , if ot is begins ankle retrocalcaneal bursitis or a chilled tendinopathu is likely, if it is on lateral aspect of ankle peroneal tendon bursitis is likely. What i mean is there are multiple possible diagnosis and mere ankle pain will not help me in identifying your disease. Detailed examination by expert orthopaedic surgeon is required to identify the pathology. Meanwhile start tab zerodol sp twice a day for pain and swelling relief."
},
{
"id": 85294,
"tgt": "Will there be side effects of registron on baby?",
"src": "Patient: hi i have been using registron 5 mg tab from 01 of tis month when my periud occured on 5th of last month , we had unsafe sex during the month but up to date 14 i have not had periods and i think i am preg. shall there be any side effects of tab on baby Doctor: Hi, Yes. It produces masculinization of female fetus and congenital abnormalities. Consult a gynecologist. Hope I have answered your query. Let me know if I can assist you further. Regards, Dr. S.T. Balamurali, General & Family Physician"
},
{
"id": 34558,
"tgt": "How to treat streptococcus agalactiae vaginal infection?",
"src": "Patient: I'm a 35yr female I've been diagnosed with streptococcus agalactiae vaginal infection I'm not pregant I gave birth 9 months ago. What is the cause of this infection? could it be something wrong we are doing during intercourse. We use olive oil as alubricant because I'm very dry, could this be the cause and who is the source of the infection me or my husband? what treatment should my husband take? I've been given erythromycin and polygynax vaginal capsules for 12 days but nothing for my husband i don't want to be re-infected and what preventetive measures should i take to prevent this or any other infection from occuring? can we have sex during the treatment period? Doctor: Thanks for posting you query to health care magic.Streptococcus agalactiae can be present without causing harm to you in a carrier state .this organism cause infection to baby during vaginal delivary . so presently you do not need to worry . the infection can be received from any environmental source so better to avoid use of lubricant as sterility is doubtful of olive oil. you are using . for your dryness you should contact to gynecologist as it could be due to some hormonal imbalance and hormonal treatment will solve your problem. treatment can be taken by both of you at same time and during treatment you can use condom during sex .hope you would be satisfied with my answer . feel free to communicate if any query .regards,Dr.Manish PurohitInfectious disease specialist"
},
{
"id": 103277,
"tgt": "Rashes on arms due to poison ivy, swollen and itchy penis. Emergency situation?",
"src": "Patient: I was exposed to poison ivy two days ago and have broken out in rashes on hands, wrists and arms. In addition, my penis is swollen. There is no redness in the penis as with the other inflammations, yet the itch is exactly the same as other areas. The swelling on the penis is on the under side, in the otherwise loose skin behind tip. I have been circumcised. Can I wait a day to see if swelling goes down, or is this an emergency situation. There is no pain other than the itching and no problem with urination. I am 67 years of age. Thank you. Doctor: Hello,To control the rash, itching and swelling antihistamines and steroids can be used.It controls the inflammation and help reduce swelling.Inspite it the symptoms persist and progresses then you may need to consult the doctor for management."
},
{
"id": 103164,
"tgt": "Have trouble breathing, painful, popping sensation. Was due to excessive work out?",
"src": "Patient: Hi im a 28 year old male and playing soccer last week i got sandwiched between 2 players, one running at full speed. i heard/felt a crunch/crack and it caused a lot of immediate pain. i came off feild and hurt badly to breath. I tried to runn and made it through light jogging before i had to stop due to the pain and trouble breathing. the pain was EXTREMELy sever when i got home and i had to stay hunched over in order to breathe. after about 2 hours i was trying to sit up and felt a very slight pop or click and the sever sharp pain subsided a bit and i didnt have to stay hunched over to breath anymore. does this sound like i definitely broke it or could it be a crack or maybe it was slightly dislocated and when back in place? Doctor: muscle pullmuscle sprain muscle tissue damage or hairline crack of the ribs or may be broken all need to be explored by getting xray chest pa view and consulting the gpmean while you can take anti inflamatory analgesics and antiinflamatory creams to control itnee rest wit pressure bandage"
},
{
"id": 22323,
"tgt": "Can hypertensions medications be dropped?",
"src": "Patient: hi, I am 40 years old. I was diagonised with high BP in 2004 and since then I am taking Losar A - 1 tablet every day. My weight is 78 kgs and I am 5 feet 10 inches tall. I would like to know whether I can gradually reduce the dosage and discontinue this medicine or do I have to take it for life as my BP shows normal on with the intake of this medicine. please advice Doctor: hello,these medicine needs to be continued lifelong. Your bp is normal because of your medicine and if you stopped it, it will again rise. You should have a healthy lifestyle like avoiding fatty, oily and high calorie diet. Have low salt diet and monitor blood pressure regularly thrice a day for one week then once or twice a week. Regular exercises like brisk walking, jogging according your capacity atleast 30 min a day and 5 days a week. Lots of green leafy vegetables, fruits, fish once or twice a week, avoid meat. Avoid smoking and alcohol if any. There shouldn't abdominal fat deposition or obesity. Get your lipid profile and sugars tested once.."
},
{
"id": 48737,
"tgt": "Suggest plans to reducing albumin levels causing stressed kidneys and having diabetes",
"src": "Patient: I know I am headed for failing kidneys. My dr. said my blood tests looked good but my kidneys appear stress. I am diabetic. She said my albumin was 10 pts. above the max. I got a juicer and have been drinking vegetable & fruit juices. I am serious about lowering my test score. 8o Doctor: Hi welcome to HCM. I have read your question and I have understood your concerns. Loss of protein in urine is a sign of kidneys damage in any disease, also in a diabetes. But the most important parameter is S. Creatinine and GFR(glomerular filtration rate). Please just see your reports and discuss with your doc about that parameters.My advice to you - 1.Avoid excessive fluid and salt at it will raise your blood pressure in kidney DISEASE . DON'T TAKE EXCESSIVE FRUIT JUICES - THEY SHOULD BE AVOIDED BOTH IN KIDNEY DISEASES AND DIABETES.. SO restrict fruit intake. Upto early stages of kidney damage normal protein intake in diet is allowed. 2.Tight control of blood sugar is of utmost importance. Target will be FBS 3 . Don't get panic, in early stages kidney diseases are preventable. Take regular follow up with your kidney specialist and do every 2 monthly blood reports for kidney and diabetes. Thank you."
},
{
"id": 197898,
"tgt": "Whats the cause for foreskin not to retract during masturbation?",
"src": "Patient: My foreskin will not retract over the glans during masturbation as it kind of feels too tight, however last night I lost my virginity and the foreskin retracted inside my girlfriends vagina painlessly without me knowing about it until afterwards when we finished. It is slightly sore today. Is this normal? Doctor: HelloThanks for query .You had difficulty in retracting foreskin over glans penis while masturbating but it was freely retracted when you had intercourse with your girl friend .This could be mostly due to tight foreskin that was not being retracted while masturbating due to devoid of lubrication However during intercourse female vagina is well lubricated due to vaginal secretion which helps in painless retraction of foreskin over glans penis .The soreness that you have on glans penis can be cured with antibiotics and topical antibiotic ointment Neosporin for 5 days .Do use lubricant jelly like Xylocaine Jelly while having intercourse .This will prevent injury to glans penis and frenulum .Dr.Patil."
},
{
"id": 82720,
"tgt": "What causes feeling faintish light headed and feverish?",
"src": "Patient: Hi. I am a 42 year old healthy woman. For the past few days I have been feeling faintish, light headed, queezy, shakey and feverish. Today I have pain in my upper back. I never had sugar issues or hbp. HOWEVER, I am in remission from Antiphospholipid Antibody Syndrome/Systemic Lupus Doctor: It could be POTS which is related to SLE , can you send me the reports of ANA, ENA profile and antiphospholipid profile . It will help me to guide you better along with your BP readings."
},
{
"id": 180060,
"tgt": "What could cause continuous vomiting?",
"src": "Patient: MY Son 2.5 years old having vomting since 15 days whatever he eating return he vomting nothing dilute in his body we test his blood C B C Report repot will be normal we do sonography that report also normal we admited him in Ghati they can*t dognoise please suggest me Doctor: thanx for the query . you have mentioned about vomiting but you not told about any other symptoms like fever or loose stools. Any way as you have told that CBC and sonography were normal and the vomiting are still persisting you need to see pediatric gastroenterologist and may be your child may need a endoscopy.i hope this satisfy you."
},
{
"id": 37589,
"tgt": "How to treat herpes in my genitals?",
"src": "Patient: I am very embarrass about going to the doctor regarding herpes that I have down below. The pimples always comes back because of my stress level. I ve been taking l-Leyden and yesterday I went to an Asian drug store and I explained my condition and she recommend FLUCORT F is an ointment and ginger 3x a day. Can you give Me any advice. By the way I have seen some improvement. Doctor: Hello,Welcome to HCM,I have gone through your query.Your problem is most likely genital herpes.It can be diagnosed well when the infection is active. Swab should be taken from the fluid in the blisters.Cell culture can be done from that.You can also run a PCR blood test.But even the negative results doesn't exclude herpes.I usually give antiviral treatment acyclovir or famcyclovir for atleast five days in my patients with such case.Acyclovir ointment for local application also will be helpful.Avoid wearing tight clothes which may irritate the blisters.Keep area clean by using saline water.You can apply ice pack or wet tea bags to soothe pain.It can recur after this also when your immunity becomes low.So episodic treatment should be done in such case.Thank you."
},
{
"id": 43597,
"tgt": "Taken HCG injection, prescribed Gestofit. Correct dosage prescribed?",
"src": "Patient: dr gave me hcg injection on 12th day and told to be together also gave to eat gestofit tablets but i didnt see that .next when i visited her on 20th day she sccaned and told that one seed has been ruptured .then iasked about gestofit which i had not taken .she said to have from the 20th day my question is that is it correct to take gestofit from 20th day Doctor: Hi Welcome to HealthcareMagic If you had intercourse at the time of ovulation then there is good chance of conception. Gestofit tablets are progesterone tablets which are given to support pregnancy. It is usually given after ovulation. As you did not take medicines after ovulation, you can start taking it from day 20th onwards as it will be useful now also. All the best Take care"
},
{
"id": 170649,
"tgt": "Suggest treatment for severe cold and cough in children",
"src": "Patient: my 21 months daughter is having severe cold and cough and is not eating any thing.also now vomitting if i try to give her any thing.i have domestal suspension at home .howmuch sall i give her and after what time of giving medicine should i try to feed her .thank you. Doctor: Brief answer:Give cough syrup. Detailed answer:Hi, welcome to HCM. Your child has severe cough and cold due to which during coughing she has vomiting. In my opinion, there is no need of a antivomiting syrup like domstal. Give a cough syrup like chericof 5 ml PO three times a day for 3 days and your child will be fine. I hope this will help you. Take care. Regards:Dr Deepak Patel, MD Pediatrics"
},
{
"id": 27397,
"tgt": "What causes persistent cough when taking ecosprin for heart problem?",
"src": "Patient: my sister is 9 years old and a heart patient.she has undergone 3 major operations.she takes half ecospirin 75 everday.she has persistent cough.is it a side effect? what are side effects of aspirin on lungs? she had vomitted blood because she had crack on right lung because of more pressure.aspirin wont affect that na? Doctor: Aspirin can cause bronchospasm and at times bleeding, from mucosa of lungs and gastro intestinal symptoms. It can cause gastric ulcers and in this case upper gastro endoscopy is must go rule out ulcers and to treat immediately. Till diagnosed stop ecosprin and consult doctor immediately"
},
{
"id": 214300,
"tgt": "Suggest natural remedy to stop menstrual bleeding",
"src": "Patient: I have irregular menstruation for past 10 years. periods came 3 months once nowadays. I have nonstop bleeding above 20 days. Bleeding stopped after i consult the doctor and take medicine. I have this problem for 3-4 years. I have pcod also. Now periods came after 5 months and i have non stop bleeding(heavy). Now i am running 8th day. Please advice how to stop bleeding through natural remedy. I am 30 years old. Please help me.. Doctor: HelloPcod is the main reason of your irregular bleeding .Cysts are often caused by hormone changes .To be frank , if cysts are not to be cured you will not get relief .Using birth control pills can sometimes regulate hormone levels so that the cysts can be taken care of. So consult your gyne and most probably birth control pill and metformin are the best drugs for PCOD cases .Need not to fear , once you stop birth control pill as long as there are no medical concerns , you should not have any problems in future for irregular bleeding .So consult gyne get his opinion.Hope this will help you.In my opinion there is any natural remedy for pcod case.Good luck."
},
{
"id": 209619,
"tgt": "Suggest treatment for constant unwarranted anxiety, tachycardia, hypotension, dizziness and other symptoms",
"src": "Patient: I suffer from constant unwarrented anxiety, tachycardia, hypotension, dizziness, shakes, any physical activity can trigger things, or even at rest, I had a total hysterectomy 7 years ago Are these related??? How can anyone feel like this everyday. I,m calmer at nite or laying down the minute I get up everything startes??? Doctor: Hello,Thanks for choosing health care magic for posting your query.I have gone through your question in detail and I can understand what you are going through.The condition that you are having is basically panic attacks. This is a disorder of anxiety and occurs cause of imbalance of certain neurotransmitters in the brain. especially serotonin, nor adrenaline, and dopamine to some extent. Medicines are the best options to take care of these symptoms. Paroxetine is the the most popular medicine and my drug of choice. I generally prescribe my patients 12.5 mg per day at night which takes care of anxiety symptoms. Also certain medicines like propranolol and clonazepam can be give as required when there is a panic episode or expected situation where panic attack can appear.Hope I am able to answer your concerns.If you have any further query, I would be glad to help you.In future if you wish to contact me directly, you can use the below mentioned link:bit.ly/dr-srikanth-reddy\u00a0\u00a0\u00a0\u00a0\u00a0\u00a0\u00a0\u00a0\u00a0\u00a0\u00a0\u00a0\u00a0\u00a0\u00a0\u00a0\u00a0\u00a0\u00a0\u00a0\u00a0\u00a0\u00a0\u00a0\u00a0\u00a0\u00a0\u00a0\u00a0\u00a0\u00a0\u00a0\u00a0\u00a0\u00a0\u00a0\u00a0\u00a0\u00a0\u00a0"
},
{
"id": 61759,
"tgt": "Suggest remedy for lump in shoulder",
"src": "Patient: Hi, My daughter (18) has a very small lump that seems to come and go under her left shoulder (front of body, not back). It s less than 1/2 inch long and maybe 1/4 inch wide. She says it stings a little when you touch it, but there s absolutely nothing on the skin s surface. It was there a few days ago, then disappeared and today it s back, and there seems to be another one parallel to it. Any suggestions as to what this may be? I feel that if I take her to the doctor they re going to have no idea... so I might as well do a lot of homework first. Thanks! Doctor: hi.based from your description, it could be a lipoma (fatty deposit), fibroma (fibrous tissue formation) or a cystic (sebaceous or keratinous) formation. if the lesion is persistent and is bothersome for her, a consultation with a doctor, preferably a general surgeon, is best for clinical evaluation. diagnostics such as x-ray and ultrasound may be requested. definitive management is excision biopsy and/or according to her diagnostic results, management will be directed accordingly.hope this helps.good day!!~dr.kaye"
},
{
"id": 118748,
"tgt": "Reduced triglyceride level from 264 to 80 after taking astrovastin calcium. Can medicine be stopped?",
"src": "Patient: my triglyceride level became high(264) and asked doctor and he advised me to take astrovastin calcium 160 four months and then i rechecked my triglyceride level now it is at 78. I asked to my doctor then he advised me to take astrovastin calcium 10 for three months and after i rechecked my triglyceride and now it is 80.Can i stop to take medicine and what should i eat for a healthy life. Doctor: Hi,Now you can stop taking Atorvastatin after consulting your doctor.Take low fat and carbohydrate diet.Avoid sweets, cheese, panier and other oily things.Go for regular physical exercise like walking, running, swimming for 30-40 minutes daily for five days in a week.Ok and take care."
},
{
"id": 108007,
"tgt": "Suggest remedy for chronic back pain",
"src": "Patient: I am a cronic back pain sufferer with depression and bi polar. I take MSER daily and recently broken my hand. I was put on Vicodin and just then I broke out with cysts under my arm and was put on vicodin again. All of these conditions were painful. I am afraid the doctors will think I am an addict and refuse my MSER for my back pain. Can this happen Doctor: DEAR SIR I HAVE UNDERSTOOD YOUR CONCERNS, SEE THIS IS CALLED AS PSYCHOSOMATIC DISORDERS, BUT NO NEED TO WORRY. SEE DEPRESSION AND BIPOLAR DISORDERS CAN BE CURE VERY EASILY BUT IF YOU GO FOR MODERN MEDICINES SOME SIDE EFFECTS WILL BE THERE.... GO FOR YOGA PARANAYAMA BREATHING EXERCISES LIKE ART OF LIVING 6 DAYS WORKSHOP YOU CAN ATTEND YOUR AL;L PROBLEMS CAN BE CURE VERY EASILY....."
},
{
"id": 172033,
"tgt": "Suggest possible diet food for a 8 month old?",
"src": "Patient: Hello, I am a grandma of an 8 month old girl. She weighed 6lbs 6 ozs when born last May. She is now 8 months and weighs 14lbs 8 ozs. She is happy and healthy, but never seems to be interested in food or milk. My daughter breast feed until 6 months, but because she wasn't gaining well she decided to switch to formula. She has been getting cereal, vegetables and fruit for about 3 months now, but every meal is a struggle for my daughter and at the most she will only eat a couple of tablespoons of cereal or an ice cube size portion of vegetable or fruit. The most she will take at a bottle feeding is 5 to 6 ounzes, but most times this is much less, around 3 or 4 ounzes. My daughter sees other friends that have younger babies and how much they eat and is worried. Do you have ant advice? Doctor: The most important advice I can give you is that every baby is different and have different rates of metabolism. Some kids need more food to generate the same weight gain, some, less. In my humble opinion as a pediatrician, I think your granddaughter is doing quite well, weight-wise, so I would not worry. I would suggest you stop bottle-feeding her completely and let her decide how much she wants to eat with a feeding cup. This is completely possible - and you will find that she will accept milk more readily."
},
{
"id": 205933,
"tgt": "What are the symproms of OCD?",
"src": "Patient: my 27 year old son has to walk all the time, things have to be put in certain places because he says they are in his way. If I says the words ok or good-bye he wants me to say alright or later. He also eats certain foods because he does not want to get fat. Would these behaviors be ocd or anxiety? Doctor: Hello thanks for asking from HCMOCD or obsessive compulsive disorder is an anxiety disorder. Obsessions are repetitive thoughts, ideas or impulses that are dominating, distressing and occurs in mind persistently. They produce significant distress to individual. In order to prevent the distress individual tries to do some ritualistic behaviour that are called as compulsions. The symptoms your son is showing like putting thing in a particular manner can be due to Obsession of Orderliness, which is a common type of obsession, though detailed evaluation is needed to make diagnosis. His habit of asking alright or later when you say words like Ok or Good bye can also be due to some underlying anxiety. I would advise you to take him to a psychiatrist for evaluation and treatment if needed.Thanks, hope this helps you."
},
{
"id": 192172,
"tgt": "What causes cuts on the head of penis?",
"src": "Patient: hmmmm its kind of hard to taLK ABOUT BUT I NEED help im a big guy so my penis goes inside and hides will for about 2 months now when i go and pee and he comes out it hurts and has a white crest on it ...and now i have like little cuts and i have tried cream and antibatics cream and nothing working...is there anything i can try that i can afford to buy i dont have the money to see a doc Doctor: Hello, First of all, I need to know if you are circumcised or not. These cuts might be due to friction, scratching or accidents. This could also be due to sexually transmitted infections. First of all I suggest you to keep your local hygiene clean and apply lotion or petroleum jelly to avoid friction cuts and avoid intercourse/masturbation till your full recovery. If not improving then I suggest you get tested for sexually transmitted infections and get treated accordingly. Hope I have answered your query. Let me know if I can assist you further. Take care Regards, Dr. SAMEEN BIN NAEEM"
},
{
"id": 102630,
"tgt": "How to get rid of running nose, sneezing, dry itchy eyes and dry lips while with 7 weeks pregnancy?",
"src": "Patient: hey my nose has been running and ive been sneezing for a week now,dry but runny itchy eyes,dry lips... and I cant sleep these symptoms wakes me up in the middle of the night. I just found out I was prego dec.3 and my first appointment is jan.22 and in the past week ive taken 3 different types of sinus and allergy meds on one each and nasal spray afrien or whatever it is but nothing is working and I cant breath through my nose what can I do HELP!!!!!!!!oh im no more than 7weeks pregnant Doctor: Hi, I have read your query and according to me you should take nasal spray fluticasone+azilastine &Tab dexchlorpheniramine meleate.you should also consult your gynaecologist. Thanks for using HCM."
},
{
"id": 209757,
"tgt": "What causes tiredness and sleepy after taking medicines for split personality?",
"src": "Patient: I was attacked by split personality on feb 6th after that took medicines like olanzapine,clonozapine,oleanz,fluanxol,joykem,parkin pills.From past 1 month i stopped medication i.e.,from june now i feel tired and sleepy all day please help me with a solution one more thing is m kind of inactive all day and just feel like sitting numb without even getting up.i took oleanz on decreasing dosage like 20mg 10mg 5mg is it due to the medicines its causing inactiveness other than this my mental state is perfectly alright Doctor: Medicines in such a large no is probably causing you sedation and not allowing you to remain active to work. Ask your doctor to lower down doses if you are stable now."
},
{
"id": 175017,
"tgt": "Suggest treatment for severe stomach pain in a child",
"src": "Patient: My daughter complains of her belly hurting quite frequently. She can go a couple days or even a week at times with no issues. Then she will complain for a few days in a row again. It can come and go in waves in the same day as well. Tonight she puked and it smelled like sulfur. Should I be concerned ? Doctor: Your daughter is suffering from some sort of abdominal infection and digestive problems what I can say from your history. You have not mentioned the age of your daughter so it is very difficult to say about dosage of the drugs. Anyways you have to give her syrup oflox-oz 5 ml twice daily with syrup cyclopam 5 ml twice daily for stomach pain with some digestive enzymes like syrup unizyme 5 ml twice daily. Along with give syrup bandy-plus 10 ml stat at night and 10 ml stat again at night after 7 days."
},
{
"id": 209025,
"tgt": "Suggest remedy for mental health problem",
"src": "Patient: hello doc i have undergone a breakup because of family reason frm my boyfriends side...i have accepted everything but since dat day I did'nt got a single tear in my eye's,which is making me restless,my brain nerves are bursting...please suggest me some ways to have tear's Doctor: hi dear,sometimes due to shock it happen not to have tears.and due to that you are not able to express your feeling and feelings hidden inside.due to that you have restlessness and your nerves are bursting.so consult psychiatrist for detail counselling and medication.sometimes it resolves it self and sometimes it turns in to depression.Thank you"
},
{
"id": 150766,
"tgt": "Diagnosed with Trigeminal neuralgia. Spots on brain, constant numbness in fingers, intermittent overactive bladder problems. Reason?",
"src": "Patient: Hi, I was diagnosed with trigeminal neuralgia of the lower left branch in 2008. My first mri showed 2 spots on my brain bilaterally but my neurologist said he couldn t diagnose ms at the time. The spots are very symmetrical in nature he said. My thumb and index finger have had constant numbness for a few years now but I thought it was due to an old injury. I also have intermittent overactive bladder problems. I m on medications and have had several procedures for my trigeminal neuralgia. I ve had a few instances in the past where I had trouble walking when I stood up to go into the bathroom at night, but that was attributed to medication problems. My newest mri says There are a few scattered nonspecific white matter changes noted within the cerebral hemispheres bilaterally. While nonspecific, white matter changes can be seen with demyelinating processes or small vessel ischemic change among others. Do I need to worry that this could be ms? I am 42 years old and 38 when the tn started. Thank you Doctor: Hi, Thank you for posting your query. First of all, I would like to reassure you that your MRI findings are most likely not suggestive of multiple sclerosis (MS). As noted, white matter lesions in the brain can be caused by various conditions, including migraine. There are specific locations of white matter lesions in MS, so, the chances of you having MS is quite low. Best wishes, Dr Sudhir Kumar MD DM (Neurology) Senior Consultant Neurologist"
},
{
"id": 100880,
"tgt": "Is spreading of painful rash a symptom of allergic reaction?",
"src": "Patient: About two weeks ago, I started taking three additional supplements to try to increase my energy: iron, niacin, and Gingko. About a week ago, a rash begin forming under my arms. I thought maybe it was a reaction to lotion, deodorant, laundry soap...so I went back to my old deodorant. Over the past week, the rash has turned into a dark red, quickly spreading, painful rash with raised bumps that expand from my armpit to short-sleeve level, down my side, and now behind my knees. A splotch just appeared inside my wrist yesterday. I stopped taking the supplements yesterday and I m trying to alleviate the symptoms with Polysporin and Caladryl. Do you think these symptoms will go away? Should I see a doctor or wait to see if the symptoms go away...if I should wait, how long would you recommend? I m wondering if I m having an allergic reaction to those new supplements. I have no known allergies to foods or medicines. Thank you for your help! Doctor: HI, thanks for using healthcare magicIt is possible that it is an allergic reaction to your new supplements. In addition to the medications that you are using, you should also try oral anti histamines.If it is an allergic reaction, then these would help. Oral antihistamines are available over the counter, examples are benadryl, claritine, allegra, zyrtecIf there is no improvement in 36 to 48 hours or there is worsening, then you need to consider visiting your doctor for assessment.I hope this helps"
},
{
"id": 212177,
"tgt": "Had mental uneasiness. Took serlift, lonazep. Now recurrence of symptoms. Take same medicines? Low dosage? Suggest",
"src": "Patient: Dear Sir,before 5 years back i have some mental uneasy problem.As per doctors prescription serlift 100mg withlonazep .5 mg daily take since 2 years after that gradually i leave that medicine.Now i am feeling like before 5 years symptoms.suggest me can i repeat that medicine.if repeat can i start with lower mg. i await your suggestion thank you Balakrishna Doctor: Hi, thanks for using health care magic. I have gone through your available history. You should not stop or reduce the dose of drugs on your own without consulting psychiatrist. In many psychiatric illness, long term use of drugs is many times needed. so you should consult psychiatrist before re-starting ant medicines. as you were better with sertraline, Doctor may restart the same drug again. Meanwhile you cans tart doing stress management and relaxation techniques like yoga, meditation and breathing exercise. thanks"
},
{
"id": 19838,
"tgt": "What causes persistent fever after an angioplasty surgery?",
"src": "Patient: I had angioplasty with one stent 5 days ago. Since the second day i have had fever (ab 99.5). I take levofloxacin 500 mg twice a day and Acetaminophen 500 mg 3 x a day. The fever still there, eventhough after taking acetaminophen decreased to 98. I am 58 weigh 138 lbs. Is that normal? Doctor: Hi, Thanks for the query. Persistent fever after angioplasty can come due to the following reasons:1. Because of the heart attack itself - The fever may last for a few days and then go a way spontaneously. 2. Due to medicine/ stent - The body may react to the drugs or stent placed during angioplasty. 3. Fever can also occur due to associated infection.As you will understand, it is difficult to tell the definite reason for fever without doing some tests (like routine blood count, ESR etc) and physical examination. hence, you should visit your treating doctor for the same. If you have any doubts, please feel free to contact us."
},
{
"id": 39504,
"tgt": "Is three injection of anti rabies vaccine enough for dog bite?",
"src": "Patient: Hellow i was bitten with my pet dog, and im already taking a 3 session shot for anti-rabies 0,3,&7 days..my company need me to goback to work which is complict to my 14days vaccination..3shots is ok or i needthe14th dayshot...im a seaferare....pls hepl me Doctor: Hello,Welcome to HCMHas you have bitten by pet dog,3 doses of antirabies vaccine is enough.You can go back for work.But you have to observe the dog for 10 days.If it develops the symptoms of rabies or change in its behaviour within this period,you need complete the full course.Hope you are happy with the answer.Thank you"
},
{
"id": 27057,
"tgt": "What causes dizziness with headaches and irregular heartbeats?",
"src": "Patient: i usually feel dizzy and tired and run out of breath.i am having frequent headaches and heartbeat is irregular i can hear its sound easily.sometimes my hands and legs start vibrating and lower jaw,left hand,neck and shoulder ache.i am getting irritated easily.why all this is happening Doctor: Hello. Thank you for your question and welcome to HCM. I carefully read your query. The symptoms you are describing with tremors of limbs and jaw, dizziness, irritability and irregular heart beats, make me suspect for a thyroid gland dysfunction. Therefore, I recommend that you do a full thyroid control:- blood levels of TSH, fT3, fT4;- ultrasound of the thyroid gland. Furthermore, this heart beat that you are experiencing, can be irregular. In this case, the highest probability is that we have to deal with a supraventricular arrhythmia, probably atrial fibrillation (instead of atria being contracted in harmony with the cardiac cycle, their contraction is uncontrolled and they do not function properly to propel the blood into the ventricles). This is a benign arrhythmia, but if more than 48 hours on this arrhythmia, atria can generate little blood clots (thrombi) due to not contracting properly. Thus, I suggest you to do an EKG, echocardiogram and a 24-hour Holter rhythm monitor registration, to confirm or exclude this diagnosis and, also to start treatment either to achieve rate control or rhythm control. You can further discuss these matters in details with your treating cardiologist. I hope I was helpful. Best regards. Take care."
},
{
"id": 3779,
"tgt": "After how many cycles of regestrone and metformin will I conceive?",
"src": "Patient: Hi , I didnt get my periods for last 55 days, and after consulting a gyneacologist, it was found as PCOD. She prescribed me regestrone and metformin. I am planning for pregnancy and i am not conceived yet.My doubt is , for how many cycles it is advisable to take regestrone? Will I conceive soon?? Doctor: Hi,I wish you had provided some more details to help you better. What is your age and BMI? Is this the first time that you have irregular cycles?How long have you been married?Regestrone contains progesterone hormone. It is used to induce periods. it does not help you in conception. It is important that you and your husband should consult gynecologist and get some basic subfertility investigations done to find out if there is any other cause for delay in getting pregnant ( hormone tests and fallopian tube patency test for you and semen analysis for your husband).If it is found that PCOD with irregular ovulation is the only cause for delay in periods and if you are obese then it is extremely important that you reduce weight. Reducing weight by 5-10% can itself regularize ovulation and increase chances of pregnancy. Your doctor will discuss with you and start metformin + clomid to induce ovulation. These medications are used for six months. Your chances of getting pregnant are about 40% and having live baby is around 27%.If you don't get pregnant with this treatment your doctor will discuss with you use of gonadotrophins (hormone) or ovarian drilling (surgery) to induce ovulation.take one tablet of folic acid dailyTo increase your chances of conception maintain normal BMI, quit smoking ( if you're a smoker) and have intercourse at least three times in a week.If you have any further concerns do contact me through Healthcare Magic."
},
{
"id": 130461,
"tgt": "What is the cause of pain in arm?",
"src": "Patient: I work out and life weights recently I've had a really bad pain in my right arm along the ulna, one of the muscles in the area seems to be a little bigger than the same area on my left. I was wondering if the pain is coming from a pinched nerve there, the pain can go down into my palm but never travels up. thanks you Doctor: Hello,You lift the weight. its more possibilities of there is muscle injury or strain or ligament sprain with nerve stretch. Improper weight lifting causes arm pain. And your right hand is looking slight heavier than left so it may be swelling due to strain or sprain. When ever you push the affected area the pain goes down it means nerve is also affected. But the other causes are rotator muscle injury, it occurs when over head weight lifts as bench presses, shoulder presses etc. in this case the pain radiates up to elbow. Other reason is tear of rotator cuff, patellar tendonitis, back sprains and strains, herniated disc.Do some stretching before weight lifting. For the swelling reduction take pain killers like ibuprofen and naproxen. Apply hot and cold fomentation. Although you don't get the relief get MRI and consult the ortho.Hope I have answered your query. Let me know if I can assist you further.Regards,Dr. Ankur"
},
{
"id": 178606,
"tgt": "Suggest suitable medication for cold, cough & wheezing?",
"src": "Patient: My son is 67days old,has been suffering from cold, cough and wheezing for 2 days. Doctor adviced to give 1. Salbair neb 0.63mg 2. Bunase 0.5mg 3. Ascoril LS 0.75 td 4. Zedocef drops 1ml bd 5. Omnacortil drops 0.75ml td is this much medicines needed in this age? Doctor: Dear Sir/ Madam,thank you for posting your query at healthcaremagic.comAs this age is crucial and system is building up the child, if neglected can cause some residual infection locally. For cold, cough and wheezing all the medication are necessary and to be administered on time. This will enhance the proper breathing pattern of your child and clear the airways.Hope you find this informative. Your thank you note will be very much appreciated!With best wishes,Dr. Vishwanath Patil"
},
{
"id": 48109,
"tgt": "Can swallowing semen affect the kidneys?",
"src": "Patient: I have a question that is a little racy but I am not really comfortable asking my nephrologist in person. I have IgANephropathy and have taken on a pescetarian diet. I have been in remission for 3+ years. I am wondering if would be negative effects on my kidneys (I still have close to 100% functionality) if I were to swallow the semen of my significant other. Doctor: Good Day and thank you for being with Healthcare Magic! As long as your significant other doesn't have any STDs then it will be perfectly safe to swallow the semen. Drink plenty of water after swallowing to dilute it in your stomach. I hope I have succeeded in providing the information you were looking for. Please feel free to write back to me for any further clarifications at: http://www.HealthcareMagic.com/doctors/dr-manuel-c-see-iv/66014 I would gladly help you. Best wishes"
},
{
"id": 176979,
"tgt": "Suggest proper treatment for meridian amblyopia in a 5 year old",
"src": "Patient: My son is 5 years and 3 months old. He has just been diagnosed with meridian amblyopia OD +2.5 -3.75x170 OS +1.75 -2.5x015 My ophthalmologist prescribe glasses and ask to wear it for 6-8 weeks than go back for check up. She said if his vision doesn t improve, she will prescribe patching. I am wondering if this is too little too late? I saw an article on Internet that it is only effective before 5 years old and after this age, it is difficult to treat once their brain develop. Is this true? Also, I saw from this site that there is a similar case 2 years ago but milder condition than my son. Her doctor suggest patching for 6 hrs a day and with light therapy. Did her son recover well? Doctor: dear user, thanks for your query.Amblyopia can be treated only before 6 year of age. your doctor is treating him in right direction. I suggest you to have faith in your doctor and follow her direction."
},
{
"id": 69248,
"tgt": "Can lethargy and a lump in the breast be related?",
"src": "Patient: Hi there, I recently visited our local doctors clinic (about 5 weeks ago) to have a lump in my breast near my underarm checked. The doctor suspects it is a 'breast mouse' but wanted me to have a breast ultrasound to check this. I have been waiting since then for the appointment for this to come in the post, not worrying too much. However in the past week I have felt very lethargic for no apparent reason and wondered if this could be related? I am 27 years old and otherwise in good health. Thankyou. Doctor: The lump in your case appears to be a fibroadenoma...a benign swelling... if small it can disappear on its own over time... if larger may need to be removed by surgery.... There is no relation between your breast lump and lethargy... if persistently lethargic get yourself checked to rule out other causes"
},
{
"id": 53001,
"tgt": "What is the treatment for enlarged liver?",
"src": "Patient: my husbands son is said to have an enlarged liver. he is 12. his mother said that his liver could explode if he is playing in sports. Is this so? Also, they said that the liver was swollen because of excess fatty foods and after dieting for a month that the swelling would go down. Is this true? Thank you so much. Doctor: Hi and welcome to Healthcaremagic. I understand your concerns and I will try to help you as much as I can.This is not true so much. Liver cant explode and he may do sports if he feels fine. There are many causes of liver enlargement but in his age this is usually result of ebstein bar infection or fatty liver. Treatment depends on exact cause. if this is caused by diet, obesity and diabetes then dietary and lifestyle changes will be beneficial in every case. Alcohol and viral hepatitis are less common but it should be ruled out with blood tests such as AST; ALT and HBsAg so you need to consult pediatrican about these tests.I hope I have answered you query. If you have any further questions you can contact us in every time.Kindly regards. Wish you a good health."
},
{
"id": 11983,
"tgt": "What is the cause of brown pigmentation on the cheeks in an aged person ?",
"src": "Patient: hi,am 58 yr old housewife, am having light brown coloured patches in my both cheeks with no itching since few months. what is it and the reason for that? Doctor: Hi Welcome to HealthcareMagic forum. this condition is known as melasma, caused by hormonal changes as in menopause in your conditions, also seen in ladies taking hormone replacement therapy and contraceptives they generally resolve by themselves when the hormones set back to normal or once the medicines are stopped/pregnancy is over. It can be tackled by using a skin lightening cream containing kojic acid, hydroquinone, for few months. consult a dermatologist for the right cream. Wish you good health"
},
{
"id": 121560,
"tgt": "What could the reason be if suffering from swollen knuckles usually in the morning?",
"src": "Patient: Hello, I have notice for about the past 8 weeks or so that my knuckels on my right hand are swollen a bit and are looking little grey when I try and make a fist....they hurt even if I squeez them a bit...I am 46 years old and have never had any problems with my hands and am in great shape.... I recently have seen a docotor about my kidneys though ...as I have just found out that since birth I have had reflux to my kidneys..that explains my fever and sore skin in the past that is reoccuring....always passes in about four days......I found out on the 16th after exrays and bladder dye testing about the reflux......do you think that the reasons my knuckels are sore is due to this. My left hand is not as bad as my right..but, it too is sore......and when I wake up in the morning is when it is the worst.....what could this be.....arthritis?? Thanks for your help, Laura Horner Cedar Park, TX Doctor: Hello,The swelling in the knuckles in the morning can be related to arthritis. It is best to consult a rheumatologist for a few investigations. Hope I have answered your query. Let me know if I can assist you further. Regards, Dr. Praveen Tayal, Orthopaedic Surgeon"
},
{
"id": 151861,
"tgt": "Can C5 vertibra degrasive spondylitis cause head ache ?",
"src": "Patient: I feel some headache problem from 6 month ago one doctor is that time advice you have no any problem just like this is migraine he advice the medicine betacap+10 sun company medicine i use that medicine 3 month i am feet that time but again this problem rising now i am taking homoeo pathy medicine now feeling ok but i checkup one thing i got the spondilitist the report is c5 vertiber mild degrasive i feel that this is the also reason of headache please suggest now Doctor: from history it seems that you are having migraine pain because you got relief with use of betacap medicine. cervical spodylitis doesn't cause such type of headache."
},
{
"id": 10361,
"tgt": "Suggest treatment for hair loss and hair breakage",
"src": "Patient: Hello, i have had keratine treatment in the past and in every ocasion i feel a burning sensation while in the process. I am suffering from hair loss and hair breakage and wanted to know when should i worry about this. I do have a lot of hair and it grows very quickly but the hair loss is starting to worry me. Doctor: Hello and Welcome to \u2018Ask A Doctor\u2019 service. I have reviewed your query and here is my advice. As per your case history of hairfall, my treatment advice is - 1. Use topika hair oil and shampoo for routine use. 2. Apply a mild topical steroid lotion like beclomethasone lotion once at night on scalp. 3. Take an iron supplement and vitamin B12 supplement once daily for 3 months. Hope I have answered your query. Let me know if I can assist you further."
},
{
"id": 86936,
"tgt": "How to treat severe abdominal pain with non functional kidney?",
"src": "Patient: respected sir,my mom is 49 yrs old.she is having her right kidney in pelvic region.she is suffering from stomach pain from several yrs.but now it is going extreme.we have done {CT IVU,IVU}and doctor is saying to do DTPA test.both CTIVU and IVU shows right kidney is not functioning. we have done DJ stain on her body and left for 2 months and it is removed 5 days ago .so please suggest what shall we do for her pain recovery?? Doctor: Hi.Thanks for your query and an elucidate history.First of all remember non-functioning kidney can not cause the pain as there is no excretion so that there is no pressure development. Hence you have to consult a General surgeon to get the diagnosis for the pain in the abdomen. Secondly, go for DTPA scan as non-functioning Kidney should not be kept as it may cause hypertension and other problems. Get this removed if the another kidney is normal on DTPA scan."
},
{
"id": 119814,
"tgt": "Suggest medication for freeman sheldon syndrome",
"src": "Patient: My 22 year old daughter has Freeman Sheldon syndrome. She often complains of one of her knees having pain and popping/cracking, especially after she has been on her feet for a long time. Besides just taking Ibuprofen for the pain and sometimes some swelling are there other joint meds she could take? Doctor: Hello, In freeman sheldon syndrome there occurs muscle contractures,which leads to restricted range of motion and muscle imbalance. So before treating the cause of pain we need to exactly diagonose the cause for the pai. Your daughter needs to get a lower limb examination done along with a xray to look for muscle imbalance or bony changes which will help guide the further line of management. Hope I have answered your query. Let me know if I can assist you further. Take care Regards, Dr. Rohan Shanker Tiwari"
},
{
"id": 57020,
"tgt": "Suggest treatment for fatty liver",
"src": "Patient: i am 50 years old male my ultra sonography says that i have grade 2 fatty liver and my blood report sgpt is 104 , sgot is 40,and bilurubin is 0.8. i am a pure vegetrerian and non drinker. how can i reduse fat in the liver .i am not obese. iam worried please help me. Doctor: Hello,You labs suggest that you have fatty liver with raised liver enzymes. In this case I would suggest you get the following blood tests doneTests for hepatitis b and cANASerum ceruloplasminIn case these are negative the we would make a provisional diagnosis of Non alcoholic steato hepatitis.Treatment would consist of regular exercise , reduction of fat and free sugar in diet and vitamin e. In case the shot and sgpt remain persistently high you might need a liver biopsy Hope this helpsDo get back in case you have further queriesRegardsDr Samir Patil"
},
{
"id": 58208,
"tgt": "High SGPT level. Hepa test and ultrasound normal. Are my antibiotics causing increased liver enzymes?",
"src": "Patient: i am 35 years old and my sgpt is 135,i ve done all hepa test and ultrasound but there s nothing wrong with my organs.i have next labtest next day and i m afraid my level will not drop down coz i did 3x test already.i suspected it was my prescripted medication antibiotic that s affecting my liver enyme rise high.please help me how to lower it fast.i tried all healthy diet and sylimarin but it s not helping. Doctor: Hi,Thanks for posting your query.There are umpteen number of causes for raised SGPT.Yes, certain antibiotics do increase the SGT level, but not all. You have not mentioned which antibiotic you are taking. So it is difficult to say whether it is actually causing your raised SGPT.Though viral hepatitis is one common cause of raised SGPT, there are other causes as well like autoimmune hepatitis and metabolic diseases. You need to get these tests also done.I hope that answers your question.Regards,"
},
{
"id": 217807,
"tgt": "What causes bad chills and severe body aches?",
"src": "Patient: I have been having sudden onset on very bad chills.Teeth chattering. Very severe body aches and weakness. Actually leaned over in bathroom and kept going...HEAD first. Took a couple of minutes to get up because of the body pain and weakness. Also comes with bad headache. Has happened 3 times in 1 week. Doctor: Hai.bad chills with teeth chattering with body aches can be due to , any reasonslikecerebral malaria, fever due to urinary tract infection, excessive cold sensation,any other fevers, pls get ur complete haemogram, smear for malaria (thick and thin), urine routine torule out the possible problems . Please meet a physician so tat u dont land up in chronic problems. Good luck"
},
{
"id": 3242,
"tgt": "Can having unprotected sex on the first day of periods cause pregnancy?",
"src": "Patient: i have had unprotected sex with my partner during her first day of periods , the second day her periods stopped with few spots and her periods usually last for 3 days... today is her 3rd day and shes having cramps in her lower abdomen, what are her chances of her getting pregnant Doctor: Hi.. Thanks for writing.. If you had intercourse on the first day of periods there is almost no chance of pregnancy.. pregnancy occurs only when ovum is available.. So don't worry.."
},
{
"id": 180545,
"tgt": "How can cuts in the lower lip caused by dentures be managed?",
"src": "Patient: I have a flap of skin on the inside of my lower lip, I have top dentures but no lower ones as they cut the inside of my mouth to shreds. What may have caused the flap to form in the last 12 to 18 months and what can be done about it? I am 67 years of age & have had no natural teeth since I was 22. Doctor: Hello and Welcome to \u2018Ask A Doctor\u2019 service.I have reviewed your query and here is my advice.A flap of skin on the inside of the lip can be due to causes like a skin tag, soft tissue overgrowth or a fibroma, it can also be a wart. So, first of all, a clinical examination is a must to rule out the exact cause of the growth. In case if it is a fibroma or a skin tag it can be surgically removed and can be sent for a biopsy and in case if it is due to viral warts then it at times resolves on its own or antiviral medicine can be advised. Hope I have answered your query. Let me know if I can assist you further.Regards, Dr. Honey Arora"
},
{
"id": 191478,
"tgt": "Suggest affordable medication for diabetes",
"src": "Patient: I am a diabetic the doctor order medication that cost $300 for 30 day supply I would like a suggestion of a non expensive drug. Meformin gave me side affects and Glyubride 2x a day is not affective so that is why my doctor orded this lxarendo? I don t have it with me. Miss Edwina Doctor: Hello and Welcome to HealthcareMagic,I can offer a natural suggestion to you that is quite inexpensive. Studies have shown that the herbal Berberine is as effective as Metformin with fewer side effects when used at the same dosage level as your Metformin was prescribed. So if you were taking 500 mg 3x per day of Metformin, the dosage of a high quality Berberine supplement would be 500 mg 3x per day according to the clinical studies.If you decide to try this, I would purchase a good glucometer, control strips and/or solution or a self-calibrating unit and test your blood sugar every morning while taking the Berberine to determine its effectiveness and to also check your glucometer to a lab value within 10 minutes of testing to assure good quality control of your meter. Also, discuss with primary physician before implementing this suggestion.Of course, a good nutritious diet free of added sugars and high in fiber along with a consistent exercise program is the first line of therapy to control blood sugar and medications second. The purchase of nutraceuticals should be from a high quality supplier, generally one that only sells professional brands is best.I hope this information is helpful in some manner and please remember, I do not know all the nuances of your situation or how high your blood sugar is running. I just know the clinical studies on Berberine are impressive and wanted you to be aware of a natural, cheaper, safer alternative that MAY be beneficial in your situation.There may be other opinions to this matter and without knowing intracies of your blood sugar situation, this may be a workable option for you.Regards, Kathryn J. Shattler, BS, MS, RDN"
},
{
"id": 140788,
"tgt": "Is it okay to fly with fractured skull?",
"src": "Patient: My son sustained a 4 inch linear fracture on the back of his skull a couple of days ago. There was no bleeding or brain swelling, and other than a headache, he seems ok so he was sent home with me. I have a trip planned for Sep 3 where we will be flying on a plane for 5 hours. Is it ok for him to take the trip and fly? Doctor: Hello, That sound it's mostly caused by bubbles formed into the neck articulations. Since you feel symptoms related to body and head position, it may indicate also instability of your cervical spine, seen mostly in degenerative changes. Discuss with your Doctor, and get x-rays of your cervical spine in order to understand better your condition. Hope I have answered your query. Let me know if I can assist you further. Regards, Dr. Erion Spaho, Neurologist, Surgical"
},
{
"id": 76227,
"tgt": "What causes chest pain and needle like feeling?",
"src": "Patient: I had needle like feelings in my chest near my heart area. And I went to the hospital, and the Dr. said that it wasnt heart or lung related. My heart beat reads regular but I am going back in for another test. My chest still is soar feeling. I am 22 173 lbs 5'6\" Doctor: Hi Dear !! Thanks for your query to HCM .Read and reviewed your query and health concerns. IN the given situation of Yours,You seem to suffer from-hidden / undetected GERD with ?IBS.Other Causes for IBS causing GERD , and other causes of GERD with acid reflux needs to be ruled out for fixing Chest pain with needle like feeling in chest.I would suggest Consults with Surgeon and get physical examination and investigation for other causes of GERD and IBS leading to Chest soreness and Chest pain with needle like feeling in chest ,before starting any definitive treatment accordingly.Hope this would help you to plan further of this complex illness of yours.If need be, update any health issue 24 x 7 by a direct question tod ME, at following HCM link-Dear, if satisfied,Don't forget to close this query with YOUR pleasing feedback comments to rate this reply and service, to boost the morale of incoming Emergency patients like YOU, at HCM services.If you want to update more details and ask more update queries ,You are most Welcome herewith !!Good Day!!Wishing Good Healthy Life in time to come!!Dr.Savaskar M.N.Senior Surgical SpecialistM.S.Genl-CVTS"
},
{
"id": 16912,
"tgt": "How can the heart muscles damage be assessed?",
"src": "Patient: sir, I am 36 years old male, recently had chest pain and was detected to have acute coronary syndrome with troponin T positive 0.98 ng, 100% stenosis in the left circumflex artery and got a PTCA with stenting to that artery. Now that i have passed 25 days after PTCA, I am still worried about having a second heart attack or some clot or stenosis in other places or even sudden cardiac arrest.. Please advise me what I should do? how to find out what percentage of my heart muscles got damaged Doctor: Hello, If there was no blockage in other arteries during coronary angiography, then the risk is very low to get another heart attack or cardiac arrest if you take your medications and follow a healthy lifestyle. And the percentage of the damaged heart muscle may be assessed either by Echocardiography or cardiac MRI test. Hope I have answered your query. Let me know if I can assist you further. Regards, Dr. Lilit Baghdasaryan, Cardiologist"
},
{
"id": 10039,
"tgt": "How can severe hair loss be managed while experiencing stress, financial strain and weight loss?",
"src": "Patient: I am 45 years old and experiencing a dramatic change in my hair texture within the last 6 weeks. It has begun to fall out in handfuls, well over 300 hairs a day, has dulled in texture, feels chalky and I ve lost weight. I have been under increased stress due to job loss and financial strain, unable to afford healthcare and losing weight. Doctor: Hello and Welcome to \u2018Ask A Doctor\u2019 service. I have reviewed your query and here is my advice. Hairfall can occur due to a number of causes like Androgenic Aloepecia caused due to sensitivity of hair follicles to androgenic hormones and genetics while other causes can be fungal infection,nutritional deficiency, stress, long standing illness, side effects of medicines..Also if there is weight loss then probably stress is a cause however you also need to get your thyroid and blood sugar levels checked..You should consult a Trichologist and get evaluated and he can advise you investigations like hormonal assy, blood tests, trichogram, done to rule out the exact cause of the problem and treat you accordingly..You can be advised to apply 5% Minoxidil lotion over the scalp..You can be advised to take Biotin and multivitamin supplements..You can also be advised to take Finesteride orally if there is androgenic aloepesia as it suppresses androgens..You should massage the scalp with warm coconut oil..You should take a nutritious diet and drink plenty of water..Also add more of proteins, nuts, green vegetables and fresh fruits in diet with milk and clarified butter in diet and regular exercise to gain ideal weight.Hope I have answered your query. Let me know if I can assist you further. Regards, Dr. Honey Arora"
},
{
"id": 104981,
"tgt": "Lump in supraclavicular area, suggested due to allergies. Having lethargy, stomach pain. Due to allergy?",
"src": "Patient: my daugter is ten years old and I will say she has never seemed to be a well child. She has delt with a great deal of allergies in her life and is on sublingual immunotherapy drops. Last year in august we discovered a 1.5cm lump in her left supraclavicular which she has had two ultrasounds to confirm the second of which confirmed and increase in size to 2cm around march. she was sent to see an oncologist which is also her allergist who told us not to worry its just due to her allergies that the node is swollen because its not growing fast enough and she is not loosing weight. She is a small child (thin and short) to begin with. She has also been diagnosed with IBS by sick kids before all this came about after an ultrasound of the digestive track due to pain in the stomach and constipation/ diarrhea . Now she seems to get quite a few days of the month that she is very lathargic and pain in the stomach area just under the breast bone. I am very conserned regarding possible lymphoma but unsure if being a year since we first found the lump and no other b symptoms that we can think of other than being lathargic more frequently if this is still a possibility or the allergy diagnoses is more likely/ Doctor: Hello, I am bit confused as your allergist is also an oncologist and you need to figure out what he is board certified in - allergy or oncology? The important investigation in this regard is a complete blood count, LFTs, thyroid functions, the ultrasound reports and a FNAC report of the lump from the pathologist and reviewed by a Haematologist. Supraclavicular lymph node swelling is a significant swelling that must be investigated in detail. Her allegies and SLIT does not take precendence at the moment."
},
{
"id": 97001,
"tgt": "What causes pain on the right side post a bike accident?",
"src": "Patient: I crashed my bike going really fast downhill in the rain, I landed on my right side. I was a bit dizzy for a couple hours but that went away, my right side HURTS bad. It was ok after work but when I sat down when I stood back up I could hardly stand up and walk it hurt so bad. is this anything to worry about? Doctor: Hi...if your able to walk with minimal pain then unlikely a bony injury it is mostly soft tissue injury.Things to do.Take pain killer ( tab. Diclofenac 50mg ) twice a day.if any abrasion present do a dressing and keep it clean and dry.Take inj tetanus toxoid.Take rest for next couple of days. But if the pain persists or if the wound is big then visit the hospital.Thank u"
},
{
"id": 182099,
"tgt": "Suggest remedy for tooth pain",
"src": "Patient: A few days ago, I had a terrible toothache. It was on a tooth that already had a root canal done on it. The tissue surrounding the tooth was very swollen, especially on the roof of my mouth. It was on the weekend and I could not stand the pain, so I visited the ER. In the ER, the Dr. gave me a shot to numb the area (just like at the dentist), a rx for anti-biotic and pain pills. The tooth pain went away and was replaced with a severe raw feeling on the roof of my mouth on the side where the toothache was. I also have a canker sore on the inside of my lip and 2 cold sores trying to form on my upper lip. I am in so much pain! Help! Doctor: Thanks for your query, I have gone through your query.The pain and swelling over the gums can be secondary to tooth infection or gum infection. This can be because of improper root canal treatment or it can be because of deposits causing gum infection.Nothing to be panic, consult a oral physician and take a radiograph to confirm the diagnosis. Once it is confirmed, it can be treated with root canal treatment or scaling and root planing.The soreness over the lip can be a aphthous ulcer secondary to stress. Nothing to worry, you can take topical anesthetic and analgesic like lignocaine and choline salicylate respectively.I hope my answer will help you, take care."
},
{
"id": 60598,
"tgt": "I have high liver enzyme levels. Is this something i should be conserned about?",
"src": "Patient: i am 53 years old, had blood work done earlier this week, reveived a call that my liver enzyme levels were 134 and 80, the dr will run additional test. I drink alcohol daily, do i need to stop drinking until they run more test. is this something i should be conserned about. Doctor: YOUR LIVER ENZYMES ARE HIGH DUE TO ALCOHOLIC LIVER DISESASE AND YOU SHOULD GO FOR USG ABDOMEN . YOU SHOULD STOP ALCOHOL AND AVOID HEAVY , FATTY FOOD."
},
{
"id": 84500,
"tgt": "What may cause delay in periods after taking Deviry?",
"src": "Patient: Hi, I have taken Deviry 10mg for 5 days and I took it from 7th July to 12 July. Normally I would get within a week of taking the tablets and this time its my 8th day after taking the tablet and I didnt get my periods still. I got light bleeding 2 days back and after that nothing happened. Please help. Thanks, XXXX Doctor: Hi,Yes, it is common after taking medications to have light periods. It can also be little prolonged ,more painful and brownish in color. Sometimes spotting can occur if the drug is not taken at a fixed time every day for two to three times a day.Watch for the bleeding pattern in subsequent cycles.Hope I have answered your query. Let me know if I can assist you further. Regards, Dr. Saranya Ramadoss, General & Family Physician"
},
{
"id": 26564,
"tgt": "Does left shoulder pain signify heart attack?",
"src": "Patient: My left shoulder is hurting, does is it mean I am having a heart attack. Lately, I get a minor chest pain also. I thought because it may be because I sometimes lift weight at work. I hear that when your left shoulder hurts, it is sign of having heart attack. Doctor: Hello!Welcome and thank you for asking on HCM!I understand your concern and would explain that your symptoms are not related to any cardiac disorder (including coronary artery disease and heart attack). There is nothing to worry about! Your symptoms seem to be caused by inflammation of the soft tissues surrounding your shoulder joint or a cervical column disorder (a possible disc displacement, causing radicular nerve pain). I recommend consulting with your GP for a careful physical examination, a shoulder joint x ray, a cervical spine x ray and some lab blood tests for inflammation (complete blood count, PCR, sedimentation rate). A nerve conduction study may be needed to exclude a possible nerve compression or radicular problem. Meanwhile I recommend avoiding heavy weight lifting and perform physiotherapy. Painkillers (acetaminophen, ibuprofen) are helpful to relieve your pain. Hope to have been helpful!Greetings!Dr. Iliri"
},
{
"id": 31048,
"tgt": "What causes recurring bacterial infection?",
"src": "Patient: I have had the same bacterial infection FIVE times in three months. The first four I got it two days after intercourse. The fifth time I did not have intercourse at all, and it still came back. I have had Metronizole (excuse my spelling) pills three times, and the creme this last time. No changes in my hygiene, no changes in diet, stopped using tampons. HELP Doctor: Hi, Thanks for posting in HCM. I understand your concern. Recurrent infection what you are experiencing could be due to resistant organisms to the medications you have used. Kindly get the following investigations done. 1. Urine culture and sensitivity. 2. Vaginal swab culture and sensitivity. Also, it would be better that you and your partner both get tested for HIV status. You need to be treated with appropriate course of antibiotics depending on the nature of organism. Kindly consult Gynecologist with report for proper evaluation and management. Hope the information provided would be helpful. All the best."
},
{
"id": 59038,
"tgt": "Diagnosed pvt. How long a collateral vein help?",
"src": "Patient: Hi doctor,my brother 32years old and diagnosed pvt ..They told he has some collateral vein around liver and it can survive him..now he is using anticoagulant..I want to learn how long does collateral vein go on?also portal vein didn't open even a little..whole my family so wory about this situation..can u please aware me ASAP Doctor: HiPortal vein thrombosis(PVT) is a common clinical condition affecting portal vein.It is caused by Cirrhosis,pancreatitis,diverticulitis etc.Anticoagulant should be used to dissolve the clot.No one can predict the collateral vein duration.Portal vein clot sometimes take longer time to get dissolved.Please consult a senior Gastroenterologist and proceed.Wish him speedy recoveryRegards"
},
{
"id": 137825,
"tgt": "Suggest remedy for pain in left thumb",
"src": "Patient: My left thumb hurts when I bend it. When I touch the palm side of my thumb, it feels really sensitive. This is day 3. It seems too look a bit swollen, but still normal. It just has that annoying pain. I cant tell if it just needs icing,,tylenol, or if I need to go to the clinic. Its just really, really tight when I bend it, and that is where the pain is. Doctor: Hi,Thanks for your query.According to your description , pain over your thumb seems to be inflammatory in nature. It could be an acute attack of gout.I advice you to give you rest to the part affected, take anti-inflammatory drug like motrin 1tablet with food as and when required (upto 4 tablets daily) to reduce pain and inflammation and consult your doctor for thorough examination and rule out any infective cause.I do hope that you have found something helpful and I will be glad to answer any further query.Take care"
},
{
"id": 133914,
"tgt": "What causes sharp pain in arms, shoulders and wrist?",
"src": "Patient: hello I have been taking Celebrex for some time for arthritis pain. I have been fortunate to have avoided pain in my arms and shoulders until just recently. the pain will actually awaken me from a dead sleep. in my arms shoulders, wrist. I am sitting in my recliner awaiting my narcotic pain killer to start working so I can finish my nights sleep. and suggestions come to mind? Doctor: Hi,thank you for providing the brief history of you.As mentioned by you, about the sharp pain in the arms, shoulders and wrists, i will advice to undergo a complete neuromuscular assessment. Also, this sharp shooting pain can be due to the pathology of the nerve. and for which your ortho may advice you for an MRI, which will help understand the physiology of the soft tissues in the cervical spine. Post which with a combination of medicine and physical therapy, the symptoms will come into control, also, on a later stage, exercises will be taught to strengthen the muscles and avoid nerve impingement, which is common for this kind of sharp pain. Usuaully, patients recover with good health in 2-3 weeks, and later stages, exercises are followed at home.RegardsJay Indravadan Patel"
},
{
"id": 33585,
"tgt": "What causes the pain in chest rather than my throat when i cough?",
"src": "Patient: I am 29, 5'10\", 210lbs. I have been sick for about three or four days with hot and cold flashes and soar muscles accompanied with a cough. When I cough I feel a cutting, scratching feeling in my chest with only a small amount of throat discomfort.. Most of the symptoms are going away but the cough and the pain it causes in my chest when I cough is still present. What is wrong? and what causes the pain to be in my chest rather then my throat? Doctor: HI, thanks for using healthcare magicCoughing can cause pain in the chest as well because these muscles are involved in movement of the chest during a coughing episode.With persistent coughing, the muscles may be strained.You may want to consider a cough suppressant to reduce your coughing and a pain killer to reduce the pain that you are experiencing.I hope this helps"
},
{
"id": 15859,
"tgt": "Red blotchy rash on chest, red marks on skin, have psoriasis. Was suggested dermagraphia. How to cure?",
"src": "Patient: Hi,I get a red blotchy rash on my chest after drinking alcohol, and also in emotional situations like when I feel upset, angry or stressed. I also get red marks on my skin easily, like after hugging someone, if a dog scratches me, if I scratch my skin etc. I first noticed the rach on my chest about 10 years ago but it only seems to be getting worse. A doctor recently suggested that I have dermagraphia. I have psoriasis and therefore very sensitive skin. My wedding is coming up and I don't want to be having to worry about red rashes all over my skin! Can you suggest anything that may help?! Doctor: Hello Welcome to health care magic I can understand your situation. I appreciate your effort to seek online support. Dermatograhic urticaria is a condition which involves inflammation of skin when it is scratched or rubbed. It is a type of allergic conditions. It mainly involve young ones. Its treatment involves antihistaminics. Psoriasis is a chronic autoimmune disease and it needs long and continuous treatment. I advise you to consult a qualified dermatologist for proper evaluation and ,management. Meanwhile, try doing relaxation techniques like deep breathing, yoga or meditation to alleviate stress and anxiety. Hope it helps Good luck Regards Dr. Gourav Monga Consultant Psychiatrist"
},
{
"id": 50997,
"tgt": "Puffy dark under eyes, frequent urination with burning. Any kidney problem?",
"src": "Patient: Hi, i like to ask you question about my under eye circles.They are not dark, just a bit puffy and people commenting on how tired i look.Im sleeping very well, 9 hours every night.Last 3 weeks i feel i need to empty my blader almost every 15 minutes.every time its just like a few drops, after that a bit burning feeling, and after 15 minutes is starts again.Do you thing my circles could be related with the kidney problem? Thank you, Ivana Doctor: hi ivana, your description of puffy eyes with urine problems could suggest a problem with your kidneys. you need to get your urine checked as well as get renal function tests done to rule out kidney disease a renal ultrasound will also show if there is any problem with the kidneys the symptoms you are experiencing for urine could just be a simple urine infection which is unrelated to the puffy eyes which could be an allergy of some kind of drug or food. would suggest visit a physician for follow up ant treatment hope to have answered your query takecare"
},
{
"id": 109957,
"tgt": "What causes severe back pain?",
"src": "Patient: I m 14 years old and my back has been hurting really bad.. It s almost as if it needs to be stretched but I have Ben doing stretches for a few weeks and it still hurts and is really uncomfortable... I don t know what it will be and if it is something what is the cure... A back brace? Ice? Heat? Medication? Doctor: Hi, Thanks for your query. After going through your query I came to know that you are suffering from chronic backache. It may be due to vitamin and trace element deficiencies specially vitamin D and vitamin B 12. You need to done MRI to rule out other causes.Back exercises, neurotropics such as mecobalamine and analgesics (DICLOFENAC 100 MG SLOW RELEASE TABLETS) give relief. Sometimes vitamin D deficiency can aggravate this so serum vitamin D test is advised if it is low than vitamin D supplementation will be required. Avoid long continuous standing. Sit in a straight posture. Eat milk, fruits and green leafy vegetables daily.You can discuss with your treating Doctor about it. I do hope that you have found something helpful and I will be glad to answer any further query. Take care."
},
{
"id": 29174,
"tgt": "What causes swollen neck glands, fatigue and sore throat?",
"src": "Patient: I am a 52 year old Caucasian male. I have had mono when I was a teen and again at 40 yrs old. I have had swollen neck glands, swollen glands under my jawline, and mild soreness in those areas and the throat. it is getting very irritating, I am also quite fatigued, even tho getting 8 hours of sleep. I have felt this way for about 5 weeks now. I really have no other cold symptoms...what could this be ? Doctor: do warm saline gargle 3 times a day.take azithromycin 1 tab daily for 3 days.consult a medicine specialist check whether you have anemia or not"
},
{
"id": 109358,
"tgt": "What causes a huge bruise in the middle of my back?",
"src": "Patient: I have went to a chiropractor alot and they are never able to help me. And for the last several years in the middle of my back, right on my spine, I feel like I have a huge bruise. It hurts for my shirt to even touch it, but there is no skin discoloration at all. It's extremely sensitive. What should I do? Doctor: HiThanks for your queryAfter going through your query I came to know that you are suffering from chronic backache. It may be due to vitamin and trace element deficiencies specially vitamin D and vitamin B 12. You need to done MRI to rule out other causes. Back exercises, neurotropics such as mecobalamine and analgesics such as diclofenac give relief. Serum vitamin D test is advised if it is low than vitamin D supplementation will be required. Avoid long continuous standing. You can discuss with your treating Doctor about it. I do hope that you have found something helpful and I will be glad to answer any further query. Take care."
},
{
"id": 47075,
"tgt": "What causes chest pain and nausea after stopping sensipar for hemodialysis?",
"src": "Patient: I am an 11 year hemodialysis patient treated with Sensipar and Hectorol, My doctor took me off of long term, 7 years, 90 mg. Sensipar, b/c my iPTH was 142. This is the second time he has done this for low PTH and I am really feeling horrid with chest and join pain, inability to breath and sore mouth and gums, nausea, constipation, and much brain fog. My lungs also feel like they are on fire, and I m virtually sleeping 18 hours a day. I can barely stay awake, even to the point of I feel like I m passing out. All these symptoms came about after the discontinuance of Sensipar. Are all these symptoms related to Sensipar discontinuance? I get my lab work tomorrow, but highly suspect extremely elevated phosphorous and calcium. I have with few exceptions maintained these at normal levels. Did abrupt cessation of the Sensipar cause this? Should I just lower the dose? I have no doctor or dietician available for a week. Doctor: Hello and welcome to HCM.As an Urologist, i can understand your concern.You should know that,both Sensipar and Hectorol,are being given, to control the hyperparathyroidism,usually seen in people on dialysis.You should be considering a kidney transplant,which is the best option. Check Hb,creatinine,sodium,potassium,calcium phosphorus and uric acid.Your symptoms are likely, due to high creatinine, with elevated calcium.Ensure your salt,protein and fluids are strictly as advised. Don't change any medication on your own. You can contact me, with a direct question.Dr.Matthew J. Mangat."
},
{
"id": 92255,
"tgt": "Child having stomach pain after falling on stomach while ice skating. Any chances of internal injury?",
"src": "Patient: Hello. My 7year old son fell on his stomach pretty hard while ice skating yesterday (several times). He complained of stomach pain last night when going to bed. He said several times during the day today that his stomach is still hurting. When i ask specifically to show me where, he says his whole stimach. When i feel around to see if thhere are any tender spots they seem to be on the left and right sides of his stomach. Not in the middle. He has no fever, and has been able to eat and drink during the day. He does not have a big appetite though today. I am just worried because i cant tell ifits muscular or if he has some sort of internal injury. Any help would be greatly appreciated. Thank you! Doctor: Thanks for query.it would be better if an ultrasound abdomen is done to differentiate between muscular and internal injury.consult a general surgeon for evaluation of pain abdomen.regards"
},
{
"id": 178554,
"tgt": "What causes red dot on hands/legs of baby after starting wheat cerelac?",
"src": "Patient: Hi..I started wheat cerelac for my 4 months old daughter two days back. And yesterday she had developed red dot like rashes on her hands and legs. Is dat an allergy.? I have stopped using her cerelac and on breastfeed now. Wat can I do now for the rashes ? And is she allergic to wheat? Doctor: Hi...according to the WHO (World Health Organization) the best way to feed any baby till 6 months of age is 100% exclusive breast feeding till 6 months of age. The gut will be too tender before that in some babies and will not tolerate cereal proteins like wheat and rice.The rash you have mentioned could as well be an allergic one to wheat.Stop all cereal proteins will 6 months of age and feed her only breast milk till then.Regards - Dr. Sumanth"
},
{
"id": 135169,
"tgt": "Suggest remedy for pain in sternum with excessive belching",
"src": "Patient: my son is two years old and has a developmental delay and not walking yet. when he stands on my sternum it hurts and it feels like it gonna break. he only weighs 25 pounds. I have also been burping a lot and I have acid reflux bad. been going on for 4 day and have not done any excessive lifting what can this be from. Doctor: Hi.The acid reflux could in itself be causing all the symptoms you are experiencing. They can include belching, heartburn, regurgitation of food or drink into the mouth, bitter taste in the mouth with regurgitation, sensation of lump at the back of the throat, dry cough, hoarseness of voice, etc. Treatment includes a list of dos and donts, but for now you can stary using 1 tablet of 30 mg lansoprazole once a day every morning.Best wishes."
},
{
"id": 23231,
"tgt": "What else other than acid reflux can give pain in the chest?",
"src": "Patient: i have chest pains. They sometimes feel like spasms most the time on my left side around my breast. I thought i was having a heart attack six months ago becuz i felt presure in my chest but the doc at the er told me it was acid reflux. I do sometimes have chest pain that feels like pressure but lately just had what feels like spasms. Could it be acid reflux that is causing the feeling of spasms or could it be my heart. Im getting scared to go back to the doc but wondering if its my heart is stressing me out. Doctor: DEAR USER,THANKS FOR CONSULTING WITH HCMTHERE ARE MANY CAUSES OF CHEST PAIN RANGING FROM HEART PROBLEMS TO RESPIRATORY PROBLEMS TO ACID REFLUX ETC AND JUST CHEST PAIN DOES NOT INDICATE ACID REFLUXI SUGGEST YOU TO CONSULT YOUR PHYSICIAN AND GET A ECG DONE AS IT IS ON THE LEFT SIDE OF YOUR CHESTYOU REALLY DONT HAVE TO BE SCARED ABOUT GOING TO THE DOC.. V ARE ALWAYS TO HELP YOU SO IT WOULD BE BETTER YOU CONSULT A PHYSICIAN AS EARLY AS POSSIBLEHOPE I ANSWERED YOUR QUERY. YOU CAN MESSAGE ME FOR ANY FURTHER CONCERNS"
},
{
"id": 26685,
"tgt": "Can I take nicardia for BP?",
"src": "Patient: Before 1 year i check my BP(160/100) doctor give me nicardia 10mg daily 2 tablets i take evryday tablet before 6 months i check my BP(140/70) still i am taking daily twice tablet now i am 42 years age.Hole life iam taking tablet or i stop my medicine(Nicardia 10mg).Pls, advice me.I am smoker. Doctor: Greetings! Welcome to HCM and thank you for your question. I understand your concern. If a diagnosis of high blood pressure (hypertension) is now established, and treatment, as above mentioned, is installed and adhered to, then it may be also for a lifetime. You see, blood pressure figures have to be controlled, if you are diagnosed with hypertension. Every long-standing, constant finding of high blood pressure figures, higher than 140/90 mmHg, is accompanied with slow, gradually progressive, but certain damages to all organs in the organism, starting from the heart which has to pump, for a long time, against a increased resistance in the periphery. There, of course, are a lot of drugs that can control hypertension effectively and safely. One of them is the calcium channel blocker, nifedipine, which you are already on. No matter what medication is used, the final aim is to keep blood pressure figures under 140/90 mmHg. Also, I would recommend you to quit smoking, as it affects blood pressure, have a healthy physically active lifestyle, i.e. 60 minutes of physical activity every day, and limit your salt intake in diet, as much as you can. I hope I was helpful with my answer. Wish you a good health. kind regards, Dr. Meriton"
},
{
"id": 220819,
"tgt": "Is pregnancy possible through intercourse 6 weeks postpartum?",
"src": "Patient: Hi i just had a baby during the postpartum wait i had sex during the 6th week with a condom and my husband ejaculated in the condom but it broke in me when he finished, it was a condom with spermicide which kills the sperm 2months now i feel coldness in my stomach and tightness and i wanted to know what's going on with my body, ? and i had a period two weeks prior to the incident Doctor: Hi, Thanks for the query. I have read & understood your query. Possibility of early ovulation after delivery depends on-- - weather you are breast feeding or not In breast feeding mothers the ovulation is usually suppressed till there is exclusive breast feeding of the baby. - The condom , though broken had spermicides.. that again reduces the chances of conception. * now your symptoms of coldness/ tightness in stomach can be psychological stress/ pregnancy. Pregnancy can be excluded b home pregnancy test. Thanks."
},
{
"id": 139547,
"tgt": "Is stem cell therapy effective for spinal cord injury?",
"src": "Patient: myself Namrata bharti, recently i came to know about stem cell therapy for spinal cord injury. my brother met with an accident 10 yrs ago and got c5, c6, c7 injuryies resulting left leg and left hand partially restricted movments. currently he can walk only with the help of Walker and his balance is also not controled , whether it can be treated with stem cell therapy or not thanking Doctor: Hello, Stem cell therapy is currently not well established but it claims that it is effective for spine injury. Consult a state of the art spine centre and they will direct you accordingly. Hope I have answered your query. Let me know if I can assist you further. Take care Regards, Dr. Shinas Hussain"
},
{
"id": 195756,
"tgt": "How to get rid of libido?",
"src": "Patient: respected sir, i am 29 years unmarried young boy. I have the problem that sometimes i want to sex relation with girls & do sometimes. but when i have no work to do then i feel only sex and desire to sex. so i cannot do work consciously. when i feel sex then it creat sex in my mind plz. guide me Doctor: Hello and Welcome to \u2018Ask A Doctor\u2019 service. I have reviewed your query and here is my advice. It is normal at this age. You should be prioritizing things in your life.Learning yoga and meditation will help you. Doing exercises, gym,learning new hobbies are other alternate methods. Don't take any medicine to decrease your desire. Hope I have answered your query. Let me know if I can assist you further."
},
{
"id": 179675,
"tgt": "Suggest remedy for lump at the back of head in a 1 year old",
"src": "Patient: Hello doctor, yesterday night my son 1.75 years old got hit in cupboard drawer he got hit in the back of his head. We just rubbed and stopped his crying. Today there was a slight swelling in that place but no bleeding. Can I apply iodex or any other option if u suggest it will good. Thanks Doctor: Hellowelcome to health care magic , Lump at back of your baby is due to hematoma ( blood accumulation ) and soft tissue swelling , swelling will persist for 2-3 days . If baby is fine ( no vomiting, active , playful , not irritable or drowsy) you need not worry, it will resolve by 2-3 days . Do not apply iodex (it may cause secondary infection )Thanks"
},
{
"id": 19723,
"tgt": "What causes constant burping in heart patients?",
"src": "Patient: Hi, may I answer your health queries right now ? Please type your query here...One of the symptoms my uncle experienced pre heart surgery was burping, he hoped this would resolve post surgery, but it has not. During surgery he had 5 by-passes and the cardiac surgeon had already told him he had extensive damage to his heart. Can you explain the burping? Doctor: Hello!Welcome and thank you for asking on HCM!I understand your concern, and would explain that the burping is not related to the heart. It is related to a stomach disorder or indigestion. Anxiety can also trigger burping. For this reason, I would recommend performing a fibrogastroscopy. Eating in small portions and avoiding spicy food can help too. Hope you will find this answer helpful!Wishing all the best, Dr. Iliri"
},
{
"id": 178539,
"tgt": "How can cough and TB in a child be treated?",
"src": "Patient: my 1 year old baby has a cough for more than two months until now.is this a sign that he has a tuberculosis?because he was exposed one year ago to a patient having diagnosed with pulmonary tuberculosis.My child undergo a tuberculin test twice and the result is positive with TB..how to treat his illness? Doctor: Hi...he needs further evaluation as to whether he really has got an active disease or not.As there is a history of contact with a case of TB and the kid is only 1 year old, definitely I would start on preventive therapy with isoniazid and rifampicin. I advice you to consult your pediatrician with this clues.Regards - Dr. Sumanth"
},
{
"id": 70327,
"tgt": "Is a painless bump in shin (because of injury) dangerous?",
"src": "Patient: My 20 yr old son hit his shin on January first...there is a fairly hard swollen bump still smack dab in middle of his right shin...should we be worried that it is still swollen? no pain, he works out at gym just fine...swelling has gone down since injury happened... Doctor: Hi. There is not much to worry about as there are no symptoms and not hampering his daily activities including gym.May be a consultation with an Orthopedic surgeon would help to alleviate any fears you have,"
},
{
"id": 162827,
"tgt": "Do silent reflux and underdeveloped lungs cause swallowing difficulty?",
"src": "Patient: I have a 2yr old that has refused to swallow for several days now. I took her to her PCP and they just did a standard run through. They sent me home saying probably the weather change. Today I told her to stick her tongue out so I could see how her throat looked. I was extremely panicked to see her epiglottis. Is this normal? Ive never seen it before , also I have 3 other children and Ive never seen anything like this appear out of no where. Im quite certain it wasnt there before as she has had a swallow study and camera run through her nose by an ENT and they didnt say anything about this. Only diagnosis she had at that time was underdeveloped rings which improved with age and silent reflux which also imporved. Is this normal and could this be the reason she wont eat ? Doctor: Hello and Welcome to \"Ask A Doctor\" service. I have reviewed your query and here is my advice. Yes, her epiglottis has always been just below her throat and above her vocal cords in the same position as the epiglottis in your other children. In my experience as a pediatrician when peering into the open mouth of a child, I have often seen their epiglottis. For the most part these have been young, healthy children and I was looking into their mouth and throat as part the routine physical exam. The children had opened their mouths very wide. In each of these patients the epiglottis was normal. Has she ever refused to swallow before? Does she have fast, labored breathing? Is she able to swallow her saliva? My suspicion as to why your infant won't swallow is that it is probably due to silent reflux. A patient who has silent reflux has a backup of stomach acid into their throat, esophagus, and larynx. Also, area below the vocal cords may be narrowed. Somewhere in this part of her upper respiratory or gastrointestinal tract there may be irritation or blockage. If she is not eating, is she drinking? It is vital that she not become dehydrated. I urge you to take her back to the ENT doctor as soon as possible. Hope I have answered your query. Let me know if I can assist you further. Arnold Zedd, MD, FAAP"
},
{
"id": 11199,
"tgt": "How to treat severe hair loss?",
"src": "Patient: sir i am having very much hair falls since before a month, i am using mintop 2% solution, it showed me some effect before but now m feeling it unusefull, my friend adviced me to use keraglo medicine, also majitrich, what should i do ? please help me !! Doctor: Hi, welcome to HCM,First of all you need to be diagnosed that whether it is due to Telogen effluvium or Androgenetic alopecia.common drugs which are used in both disease is-tab biotin 10 mg+PABA one tab to be taken daily.wash your hair with volumizing shampoo.Capixyl serum to be applied 1 ml once daily.In androgenetic alopecia i used to prescribe minoxidil 5% lotion 1 ml to be applied twice daily.Hope this is helpful.Thanks"
},
{
"id": 132535,
"tgt": "What could cause adrenal pain in people with many fractures?",
"src": "Patient: When younger, had a super-chiropractor who was always adjusting me for thyroid and adrenals. Now 54, never been to an endocrinologist, have an intense, aching pain in adrenal area. have had hematuria, stopped when drank more clear water. Could I have something serious? 4,5,6 or 7 fractures in back as well! Thank you. Doctor: Hello. Thank you for writing us here.hematuria is commonly seen in case of kidney stones. You might had taken calcium supplements for the fractures, isn't it? most likely your kidneys tried to excrete the excess of calcium which ultimately got deposited and causes pain in the lower back/ kidney area and also in the flank region. I would suggest you to have an ultrasound done for the diagnosis and also drink plenty of water so if you have stone in kidney, it could be excreted out. I wont suggest any medication for now as it would require special investigations to be done. you just need to drink 5 to 10 liter of water everyday to get relief from your symptoms.best regards,Dr Gunjan"
},
{
"id": 23690,
"tgt": "Is a surgery required for the bicuspid aortic valve?",
"src": "Patient: I recently discovered the my husband is having bicuspid aortic valve and the size of aorta is 4.0cm and the left atrium is of 3.9 cm and for left ventricla ESD=3.8CM,EF=65%,PW=1.1CM ,IVS=1.0CM, FS=36%.What to be done is this the severe one or do we require surgery please help.....the age is 32 years and weight is around 75. Doctor: Hi,If he has severe aortic stenosis or aortic regurgitation, then surgery is needed. Otherwise he needs only follow up and infection prophylaxis.Take careCome back if you have any further questions"
},
{
"id": 184244,
"tgt": "Suggest treatment for pain and swelling in jaw",
"src": "Patient: I have a broken tooth that is highly infected i went to the emergency room because the pain was so severe, the doctor gave me a shot in my gums under my tooth in two places that was supposed to block the pain , it never quite did after those shots my right jaw and cheek are swollen i feel slight pain in my neck the tooth that is broken is my bottom right i think number 28 this is my second day of antibiotics i am taking amoxicilain 875 mg two times a day, the doctor also gave me a scrit for Ibprofen and percoset i took 2 percosets and an ibprofen 600 mg and im still in pain my jaw feels very hot to the touch i put a ice cube inside my mouth and it instantly melted thats how hot my jaw and bottom cheek feel, also im extremely naucaus i have a dentist apt wed morning should i wait that long i am very scared ? Doctor: Hello, Thanks for consulting HCM, Read your query, as you have pain and swelling in jaw due to decayed tooth this pain and swelling in decayed tooth may be due to formation of pus at apical region of tooth leads to gormation of periapical abscess . I will suggest you to consult dentist for treatment of tooth as early as possible , in meantime you can take analgesic in dispersable tablet like Paracetamol or ketorol dt if you are not allergic to these drug. Hope this will help you. Wishing you good health.Regards, Dr. Priyanka tiwari"
},
{
"id": 13007,
"tgt": "What could the red rashes on body suggest?",
"src": "Patient: i woke up this morning and was violently sick, this continued for about 30 minutes ending up just bringing up stomach bile. Then a rash came up that covered my entire front and back and was bright red as if i was sun burnt.Can you tell me what it is. Doctor: Hello, I have gone through your query and it could be either a viral infection related rash or a drug rash. If it persists consult a doctor and get it evaluated. Hope I have answered your query. Let me know if I can assist you further. Take care Regards, Dr Asmeet Kaur Sawhney, Dermatologist"
},
{
"id": 117507,
"tgt": "Suggest treatment for elevated esr and wbc count in blood",
"src": "Patient: i am 49 yr old male with a good medical histroryl.i developed sore throat when on holiday two weeks ago with persistent fever and swollen lymph nodes the fever is on and off and reaches 39C but has been 37.00 for the past 24 hrs. the doctor suggested blood tests which showed normal levels but high ESR 100 and WBC 12000. he suggested to consult an oncologist if it stays high. should i worry Doctor: Hello and welcome to HCM,High white blood cell (WBC) count is due to infection.Infections cause an increase in WBC counts.A differential count is required to know the type of infection -bacterial or viral.A high ESR is not specific for any disease but it is elevated in chronic inflammations.Fever with enlarged lymph nodes and elevated ESR is an indication for conducting investigations for a malignant lesion e.g. lymphoma.Thus, consult your physician for aspiration cytology from the lymph node.Aspiration will detect the cells in the enlarged lymph node and thus, help to determine the cause of lymph node enlargement.Definite treatment will be started once diagnosis is confirmed.Thanks and take careDr Shailja P Wahal"
},
{
"id": 201129,
"tgt": "What causes redness on tip of penis?",
"src": "Patient: Hello I have been tested for stds and all results negative I have some redness on tip of penis and underneath shaft of penis also sugar is normal to rule out diabetes taken a fungal cream still no change my brother is a neurosurgeon talked to him he said go see dermatologist Doctor: Hi,Thanks for writing in.There is no need to get worried if your tests for all STDs are negative. It can be a fungal infection or mild bacterial infection. A fungal cream usually takes a week or two weeks of application to show any visible change but the itching disappears in 2 to 3 days. The other possibility is a mild infection of the foreskin (if uncircumcised) or the glans. This might require a short course of antibiotics. Many times color changes in the tip of penis without any symptoms can be a normal occurrence. A dermatologist will confirm your condition and give you more specific treatment. Please maintain genital hygiene and wear cotton inner clothes if you sweat a lot."
},
{
"id": 64980,
"tgt": "What causes lumps on fingers?",
"src": "Patient: On the skin between my finger and thumb there are some pin head small lumps, also have some on third finger that has a brown dot underneath (blood i guess) and some on my other hand on the palm.... just wanted to know what it may be and if i need to get it seen to? Doctor: Hi,Dear thanks for the query to HCM.These lumps appear to be -infestation-?AS no details are given-most probably it- could be scabies? .You need skin doctors opinion before he treats for the cause.Hope you got the answer.Wellcome again."
},
{
"id": 51778,
"tgt": "I am suffering from urinary track infection",
"src": "Patient: last week i suffered with pain when urinating and blood bleeded then i consulted with doctor and he said it s a urinary track infection given some citric cyrup . After that i feel better and today again the blood bleeds after motion. please advice Doctor: Hello suresh. Any painless bleeding in urine is dangerous.without wasting time consult a urologist to rule out Ca bladder."
},
{
"id": 224956,
"tgt": "Is there any way to prevent facial hair growth while taking depo shot?",
"src": "Patient: Hi, I just had my first shot of depo provera and never knew the fact that it can grow facial hair, if is supposed to happen when will I notice the efect? Also is there any way to prevent facial hair to grow? How can I get rid of it? I am now really stressing as I never thought of having hair on my face! help please Doctor: Dear is the the facial hair growth was not like this earlier , you must report the side effect to the gynecologist. The mechanism by which it occurs is due the inhibition of the FSH and LH hormones from a part of the brain .Thus the further release of female hormones is inhibited and that is how it is effective in birth control. Probably there is increase in androgens , that might have caused facial growth The ideal treatment is that either we can go for cosmetic removal . Or an estimation of hormones that 1. Prolactin 2. Dihydro - epiandrosteinodone to be done in this case to be done Then we can decide the further course of treatment"
},
{
"id": 185383,
"tgt": "Suggest remedy for red bumps in mouth",
"src": "Patient: Hello, My daughter (21 months) woke up yesturday with 2 tiny red bumps by her mouth...she then got a fever before bed and this morning woke up with tons of tiny red bumps on her bottom and genital area. I just put her to bed and just noticed that she has more red tiny bumps under her knee. What could all these dots be.....the daycare I work at has had a case of scabies and inpetigo but my daughter dosn t attend this daycare. Could it be scabies?? Doctor: Hello, These red bumps looks like they are ulcers of a viral lesion. Better please do not delay it visit a good pediatrician and get a complete examination done with investigations.Hope it will help you."
},
{
"id": 127783,
"tgt": "Does bruising on the buttocks require medical attention?",
"src": "Patient: I fell down our wooden staircase inside the house last week. I hit my hind end on five steps on the way down. I put ice on it that evening, but now after the swelling has gone down the area really hurts to sit upon. I did not go to see a doctor, should I? It is bruised almost all the way across both buttocks. I did see stars about five minutes after it happened. Thank you. Pat Shepard Doctor: Hello,What I can infer from your complaints is that you have. Had a traumatic injury to your buttock and pelvic area? If you have been able to walk around comfortably after the fall then major fractures have been ruled out. However, I would advise you to get an x-ray of the pelvic area after consultation with your physician. Hope I have answered your query. Let me know if I can assist you further. Regards, Dr. Santosh S Jeevannavar"
},
{
"id": 9191,
"tgt": "How can facial dry skin with eye redness and itching be treated?",
"src": "Patient: I have been suffering from severely dry skin on my face for about 6 weeks. The outside corners of my eyes are red and my eyes themselves are itchy. I also have very chapped lips and the area around my nose is red and itchy and raw. My nostrils have cracks, too. I have had this in the past but it has never lasted this long. Is it possible I have an infection? Also interesting is that I don't have any dry skin anywhere else... Doctor: Hi,It seems that probably you may be having atopic dermatitis. You have itchy sensation on face.And you have cheilitis and fissures on nasal areas.Kindly consult the dermatologist.I would suggest..- tab methyle prednesolone 8 mg thrice a day for 5 days then to tapper 4 mg every 5 days.- antibiotic like azithromycin 250 mg thrice a day for one week- antihistaminics like desloratidine once a day at night daily till itch is relieved- apply mild steroid with fusidic acid cream on the cracked areas and lips- moisturiser cream like liq paraffin on face daily in morning after bath- avoid soap on face- avoid exposure to sun- avoid fish,chicken,eggs,nuts,bringel,curd,milk products,banana in diet- avoid contact with dust,fume,cosmetic,pollens,wool..etc- keep mind cool,no worries and tension- have patienceThis would help you..ThanksDr.Ilyas Patel MD"
},
{
"id": 111072,
"tgt": "Can discectomy and laminectomy result in increased back ache?",
"src": "Patient: i had a distectomy and laminectomy 2 years ago discs L 1and 2 , it has now got so bad that i cant walk, the last few day i have had really bad pain in my lower back, i cant walk now and pins and needles in both legs front and back, im in cosntant pain even though i have taken my oxycontin and gabapentin and diclofenic, i also have really bad wind and tummy pains like a dull ache, im really concerned, what do you think this could be? Doctor: Hello, I hope that i can better help you understand your situation. From your symptoms it may be that you may have another disc herniation. While discectomy/laminectomy can help with radicular pain, you are more likely to re-herniate because the disc is disrupted at this level.If rest, anti-inflammatory meds, and pain medication are not working, you need to be evaluated by your physician and possibly be treated with steroids and get some current imaging of your spine.I hope this helps."
},
{
"id": 21391,
"tgt": "What causes pressure in chest and shortness of breath with RBBB?",
"src": "Patient: I had an EKg this afternoon and it showed a possible left atrial abnormality , and incomplete Right Bundle Branch block, and probable right ventricular hypertrophy. I have been extremely short of breath and I feel like I have an elephant sitting on my chest. I take 100 mg. of metaprolol ER daily. I do have moderate valve regurgitation. What is going on here? Doctor: Hi,Right Bundle branch block with Right Ventricular hypertrophy & LA enlargement is definitely pathological. It might be because of increased valve regurgitation or new valve abnormality.You need to repeat the echo soon and if possible get a stress echo done too. You can follow up after the reports or if you have any more questions.Thanks"
},
{
"id": 44021,
"tgt": "Done beta HCG test during IVF. Ultrasound showed possible to sacs. Am I pregnant?",
"src": "Patient: i got my ivf on 3 rd oct this year and after 8 days got my first beta hcg which was 26 then after 48 hrs a second test showed rise to 556 levels after 48 again i did third beta hcg and an ultrasound , though an ultrasound showed possible to sacs but no confirmed report , third beta hcg showed 1117 levels . does it mean i m pregnant Doctor: Hi, It means that you are pregnant. But whether its single or twins will be confirmed by ultrasound afterwords. Wish you good health."
},
{
"id": 123522,
"tgt": "How long should a make shift cast be worn after a finger surgery?",
"src": "Patient: i have a broken finger which i had surgery on the have it pinned. it is now in a make shift cast. how long will this cast be on? or when i go for my 1 week appointment will they put a fiberglass cast on? should i be able to move my finger in there? because i can and it hurts. Doctor: Hello, It has to be kept for two to three weeks. Following cast removal, physiotherapy will help in faster recovery. Hope I have answered your query. Let me know if I can assist you further. Take care Regards, Dr Shinas Hussain, General & Family Physician"
},
{
"id": 70850,
"tgt": "What can cause shortness of breath while suffering from diabetes?",
"src": "Patient: Shortness of breath. Had chest X-ray, was a smoker for 34 years, quit 18 years ago. Had stress test and another heart test, they all came out fine. My spine is fused from T-2 all the way down. My doctor thinks this may be the cause of the shortness. I have type two diabetis. Any thoughts? Doctor: Hi, A case of diabetic ketoacidosis is associated with hyperventilation and short breath due to accumulated ketone bodies in the blood. If the X-ray, stress test are negative; this is the prime concern to rule out herewith. Hope I have answered your query. Let me know if I can assist you further. Regards, Dr. Bhagyesh V. Patel, General Surgeon"
},
{
"id": 112147,
"tgt": "How long will it take to strengthen back after doing exercise for back pain?",
"src": "Patient: had lower back pain for 3 months now and was told this was due to a weak back...esp gets weak as the day goes on, sitting and standing make it worse.how long will it take to strenthen my back as being doing exercises for 2 months and still not much better Doctor: Dear patient.Greetings from health care magic!!! I am Dr Thakker and shall be answering your question.Back strengthening exercises usually increase the back muscle tone within 2 weeks time. It also depends on the type of exercise, duration and the correct technique. I would advice you to visit a physiotherapist to learn back extension exercises. If you are gym going, I am sure you would have access to the back extension machine, rowing exercised and traps pull down.I hope this helps you out. If you have any more questions, please do not hesitate to write back to me.Yours sincerely,Dr Thakker."
},
{
"id": 147975,
"tgt": "What causes numbness in face and arms?",
"src": "Patient: hi i need to aski have a male whos 50 years old he woke this morn was feeling dizzy when he stood up when he came to sit he look very pale i had him drink a gatoraid an soup his color came bac about 30 min i went to check he broke out in a heavy pouring sweat and his left side of his face went numb hes answering qestion right with his name an soforth i checked about ten min ago now his right arm is numb hes telling me hes ok he just wanted to rest hes sleeping ok for the moment Doctor: Hi, welcome to our site. I am Dr Saumya Mittal.Read your query. That is a very significant question and i appreciate your problem. I will try my best to answer your queryI would like to know the previous medical issues.It sounds to me that we are talking about a possible stroke.A stroke usually occurs during bedtime and may present with vertigo. The numbness may in fact be due to the same problemI would suggest an MRI brain, along with MR angiography.And there after a regular aspirin may be advisable.I hope this helps you. Inform the reports mentioned above so i can be of help further. Best of luck.I have given you the answer to the maximum considering the information provided. The results of the tests could further enhance my answer to you.Please do understand that some details could be extracted from a detailed history and examination.Looking forward to your return query with the details asked so that I can help you further.(If the answer has helped you, please indicate this)"
},
{
"id": 226200,
"tgt": "Pain in the belly button, spotting. On depo shots. Could that be the reason?",
"src": "Patient: Heya I'm natasha, iv been on the depo conreception injection 2months now and two weeks ago I got a sharp pain under my belly button then started bleeding but it was more like spotting and it was browish pink blood then went red then stopped, then a week after that I got the same pains on the night and the day after sex I started spotting the same blood pattern again. Iv been on the depo and other contreception befroe and never experienced this, do you know what the problem is or some advice on what to do or what might be causing it? Doctor: Hi, Thanks for the query. Depo can cause side effects like pain abdomen, breakthrough bleeding etc. So possibly the bleeding could the side effect of the depo. If the same thing gets repeated, you once consult gynecologist get examined to rule out the possibilities like pelvic inflammatory disease. Then you can go for symptomatic treatment in future if needed. For more details you can ask me through: http://www.healthcaremagic.com/doctors/dr-sree-gouri-sr/63429 Take care."
},
{
"id": 65121,
"tgt": "What does a lump in armpit indicate?",
"src": "Patient: Hello, I'm 44 years old and I've just discovered a lump in my armpit just a few minutes ago this evening as I was taking a bath to get ready for bed. It wasn't there earlier today at all (I'm rather sure as I take a bath mornings after my exercise, then evenings before bedtime). It is almost the size of a pea. The skin over it is not reddish at all (looks rather normal) but the lump feels rather sore when I touch it gently. The lump doesn't seem to roll under the skin when I touch it and the very skin covering it seems to be somehow stuck or connected to it (like it seems to be just right under my skin). I palpated further and found another one that is almost like the first one I have just found except that it is smaller. At the moment I I'm having my monthly period and for me it is not unusual that my skin gets really sensitive or that my breast gets lumpy and so on...It is just that this is the first time I\"ve found a lump in my armpit. Could this be just something benign like a furunculitis (I shaved yesterday with a rather dull shaver)? I'm rather worried. Doctor: Hi, dearI have gone through your question. I can understand your concern. You may have some enlarged axillary lymphnode due to reactive hyperplasia or some furuncles or infection. You should take a course of antibiotics. If it doesn't respond to treatment then biopsy diagnosis is advisable to rule out lymphoma or malignancy. Consult your doctor and take treatment accordingly. Hope I have answered your question, if you have doubt then I will be happy to answer. Thanks for using health care magic. Wish you a very good health."
},
{
"id": 72135,
"tgt": "What causes fluttering sensation in chest?",
"src": "Patient: I have what feels like a flutter in my chest, like my heart skipping. It has been constant for a couple weeks now. It is bothersome and i dont like the way it feels. I do not experience any lightheadedness, or anything else with it. But it happens all day. I have noticed that if I have a glass of wine or a beer, it seems to stop. I take tylenol pm's every night to help me sleep and it stops when I take those as well. Please help!!! I need to know if I should go to ER, will it go away, I just want to know what to do to make it go away. Doctor: Thanks for your question on Healthcare Magic.I can understand your concern. Fluttering sensations in chest are seen with arrhythmia (rhythm disturbances in heart).So consult cardiologist and get done ecg, 2d echo, stress test and Holter monitoring (24 hours continuous recording of Ecg).If all these are normal then no need to worry for arrhythmia or other heart diseases. Sometimes undiagnosed stress and anxiety can also cause similar symptoms. So consult psychiatrist and get done counselling sessions. Try to identify stressor in your life and start working on its solution. You may need anxiolytic drugs (propranolol and flunarizine combination) too.Don't worry, you will be alright with all these. Avoid stress and tension, be relax and calm. Hope I have solved your query. I will be happy to help you further. Wish you good health. Thanks."
},
{
"id": 118348,
"tgt": "What is the normal T-cell count to have a safe surgery?",
"src": "Patient: GETTING 2ND OPIONION ON COLOSOMY DR SAYS NEEDS TO OPERATER AND REMOVE DUE TO ITS TO CLOSE TO BONE AND HARD TO REACH AND SEE AND TOUCH- SO IM GETTING SCANS AND WITH 3D IMAGING - MY T-CELLS ARE 230 - AIDS FOR 20 YEARS T-CELL WEREW JUST COMING UP AFTER2 YEARS - FROM 0 ZERO - IS THERE A T-CELL# OR ABOVE NUMBER THAT I SHOULD HAVE TO HELP ME THREW THE SURGERY-? Doctor: t cell count above 500 is suggestive of good immunity. This shows you will recover fast and chances of infection is very less. Chances that these cell number will improve are very less specially since you have AIDS since 20 year, I would suggest you to go for this surgery if your doctor allows. Any way with low T cell count you are prone to infection so avoid contacting with infection like wear mask, take clean and home made food. avoid going to public places where you can catch infection through droplets."
},
{
"id": 47082,
"tgt": "Is my kidney function normal according to this report?",
"src": "Patient: hi my creatinine+eGFR in dec2013 creatinine was 111 umol/L in Jan2014 urine creatinine -16.76mmol/L, UrineAlbumin-182mg/L and urineACR-10.9mg/mmol in April2014-creatinine+eGFR- creatinine-106 umol/l and est.glomerular filtration rate-62 and my urine Acr -4.3mg/mmol I am just wondering if my kidney is functioning good and if not what should I do to improve. Or is it improving. Thank you Doctor: Hello and welcome to HCM.As an Urologist, i can understand your concern.From the reports sent by you, there's no recent reports ofkidney function.Test your serum crreatinine. normal is 0.8-1.2.(74-106moles/ml).Normal urine albumin-creatinine ratio(ACR) is upto 30.So, you're advised to check serum creatinine(0.8-1.2), with urine ACR.Check your B.P.,and see if you've puffiness over face and swelling over feet. If you've any doubts,send them as a direct question,to me.Dr.Matthew J. Mangat."
},
{
"id": 60361,
"tgt": "When is it safe to resume work after typhoid ?",
"src": "Patient: last 10 days I have been suffering from typhoid. last five days I had low temperature around 96f to 98f. and little dizziness , weakness and little headache ,but I could not get energy. last day again I went for widal test which proved negative, I have urgency to join my service without any further interruption in the form of other disease. please sir suggest me shall i join my service without any further fear. Doctor: Hi, Though the fever has subsided in 5 days, it's due to the medicines you have taken.compleate healing of internal system takes 2 weeks. So you will have to be care ful till then. i.e.- 1. avoid exersion / physical work. 2.have light (easily digestible)&nutritious food .Drink boiled water. 3. take Vit. B Coplex & vit C to help the healing process. So you have to take rest for2 wks. to get aperfect cure."
},
{
"id": 136997,
"tgt": "How can persistent ligament straining be reduced?",
"src": "Patient: I had a right knee replacement a two and a half years ago and was able to exercise and return to work as a nurse after a few months. Six months again I strained the left hamstring, and the right leg had to pull the left one up and down my spiral staircase and do extra work. Last February, the right medial ligaments cramped up in horrible pain after I did a lift and turn movement, not that heavy an object. It seems to be the right medial collarteral ligament. It was very painful, but no pop. My orthopedic surgeon checked it out and did an xray, and didn t feel there was a tear or a problem with the knee replacement. But every 2 weeks or so it has gone out again, I have horrible pain for 2 to 4 hours, even with hydrocodone and baclofen. I have been doing pool exercises with PT and she was pleased with my progress, but it continues to go out periodically, last time was simply swinging my leg into the car a little fast, I had my knee brace on. After each of these episodes, it is weak for 3-4 days, I use a walker, then it gradually gets stronger again. Is the ligament strain starting over again each time, anything I can do? I need to get back to work and my life! Doctor: Hello, I have studied your case. There can be possibility of infection in TKR.This can be differentiated by investigate like CRP, CBC, ESR.PCR culture is very useful if infection is suspected to find culprit organism.PET SCAN will also give more details.If infection is confirmed debridement/ revision operation is required.Hope this answers your query. If you have additional questions or follow up queries then please do not hesitate in writing to us. I will be happy to answer your queries. Wishing you good health.Take care."
},
{
"id": 64296,
"tgt": "Suggest treatment for a lump on the thigh and knee pain",
"src": "Patient: I had torn meniscus surgery for monthss ago , I also had to have three injections in the knee after the surgery and now my leg from the thigh down to the ankle feels tight and sometime tingling, no numbness. I have the disease inclusive body myositis also and am wondering if that has something to do with the tight muscles . The knee is still swollen and feels like a slight lump or hard knot right above the knee. . When going to my six week check the PA said I could start walking even with it hurting. After four weeks of walking & &severe pain went back to see surgeon, said I should not have been walking. Been in two sessions of therapy and water and with Utla sound & leg massage. Nothing seems to be working, having to take Ivory every four hours. Doctor: Current ratingGeneral Surgeon, Dr. Mukund Savaskar's ResponseHi,Good Evening.This is Dr.Savaskar from India,attending your query to my online HCM Clinic.-I studied your query in depth and Understood your health concerns.-Accordingly on the facts given by you-Causes I would suggest is as follows-a-I think you are a case of-Post-miniscal-repair-Knee Synechial rigidity with post -injection rigidity.b-Knee swelling and lumpy rt knee-suggests Synovitis also.c-Surgeons negative remarks suggest a Mis-Managed-knee- post-Miniscal repair with rigid knee and lumpy thigh-mostly due to synovial thickening and swelling.Treatment would be-a-Arthrolysis of the rigid knee b-Rigid knee braces there-after for the mobilisationc-Supportive Diclofenac-SR for 10-21 days post-arthrolyis.d-Rest-Elevation and static knee Exercises.e-Tb-Doxycycline-200 mg with Tb Metronidazole 400 be -to control infection-with proton pump inhibitors for control -metronidazole- gastritis -for 10/or-21 daysf-Hot-UV therapy and g-UST-Ultra Sono-Therapy for SOS Sessions of 2wk-21 days-period Hope this would help you with the relief you needed. Wishing you healthy living and early recovery of this query.Wellcome for more queries till you get the satisfied.Thanks for your query to HCM Clinic.Have a Good Day...!!Dr.SAVASKAR M.N.M.S.GENL-CVTS,Super specialist and Senior Consultant-and Expert in Non-Curable-Disease therapy for Cancer,Asthma,etc,Rejuvenation therapy and Tissue failure -reversal therapies."
},
{
"id": 223546,
"tgt": "How long will petogen stay in the body as protection from pregnancy?",
"src": "Patient: Hi I have been on petogen for a year now after giving birth in march 2013. My last injection was scheduled for 07-05-2014 and i did not go. I have then had unprotected sex. Could i be pregnant? or is the contraceptive still in my system? How long will it stay in my system before i can fall preggies again? Best regards Lee Doctor: Hi Lee, How are you doing ?Petogen is a depot preparation , which if taken for duration as 1 yr may stay for 4-6 mths after stopping. But a better way to know if the hormone levels are normal is , if you get periods the hormones are normal. But this is not definitive, you can get pregnant even before you have periods .But after 3-4 mths of last injection the chances of pregnancy graduallu increases & by 6 mths the hormone is mostly gone.Hope I have clarified your queries, do write back if any more queriesAll the bestDr.Balakrishnan"
},
{
"id": 64386,
"tgt": "What causes recurring painful lumps on the chin?",
"src": "Patient: I have a around 1 inch round lump near my chin which has just arisen. This is 3rd such lump in last 3 months and quite painful, also bleeds sometimes. First two got cured in around a week. Doctor said it is Phycosys Barbarine, though I am unable to find out any such reference on net. Doctor: HI,Good Evening.Thanks for your query.I studied the query in-depth.I understand your concerns.In my opinion-you have-Recurring folliculitis.I would advise -FBS,PPs to rule out the diabetes-which may cause its recurrent infection.-If you shave with the barber -Sycosis Barbae-a psuedofolliculitis is the other cause.This is caused by the irritation of the barbers shaving cream and the ingrown hair.-It differs from the folliculitis-and is also known as the 'Barbers ITCH'. .Consult your skin doctor/family physician.Tb-NSAID,Antibiotic,and anti-diabetic if your sugar is high.-This would relieve your painful lump on the chin.-Wishing you early recovery and good health.Wellcome again."
},
{
"id": 134616,
"tgt": "Suggest remedy for pain in arm",
"src": "Patient: I was discharged about fifteen minutes into donating plasma and platelets because my arm started to form a hard bubble during the process. I have had a rather large bruise on my arm ever since, but not really painful. Then today, nine days later, my arm began hurting really badly and I am having a cold sensation in the area the needle was inserted into. It hurts to apply pressure to the area and is just very painful all around. Any ideas what could be wrong and how to alleviate it? Thanks! Doctor: it can be anything which can not be predicted without seeing the injured part. you can kindly proceed a close by specialist and get urself assessed. some diagnostic procedure they might recommend kindly go through all of them. after all they are all sitting down to help u. kindly visit the closest specialist"
},
{
"id": 176915,
"tgt": "How to treat headache in a 10 years old child?",
"src": "Patient: My 10 yr old has been having headaches (back of his head), neck pain and some eye pain for a few weeks. At its worst it brings tears and has missed some school. We tried alleve morning and night, good diet and proper hydration for 5 days. It never went away. He has an upper and lower wire in his mouth. I removed both prior to a dentist visit (for 5 days) to see if that would help. It got better but didn t go away. The dentist asked if he had fallen. I had forgotten that he had. He missed the soccer ball and landed on the asphalt, back, head and tailbone. He kept the wire out and sent us to an Osteopath that we have been to before. He worked on his pelvis,neck and head. He immediately felt great but after a day it all came back and a few days later he was in tears again. I intend to call his pediatrician again on Monday. I guess I m wondering if we should wait out the treatments with the osteopath to see if they start to help (he had a lot of tension in his neck), go to the eye doctor or a neurologist. Thoughts, ideas? - Jenny Doctor: thanks for your query.From your history your child has a chronic headache which is not episodic and not associated with nosea and vomiting. It may be referred pain from teethes or neck or may be due to refraction errors in eyes. I suggest you to visit a ophthalmologist for eye testing first and if it is normal consider a neck xray to find out any problem in neck. also remember that these all problem may be due to orthodontic treatment going on his teethes.regards- dr deepak"
},
{
"id": 189562,
"tgt": "How to maintain the teeth clean ?",
"src": "Patient: hi i am 28 years old, i do brush my teeth twice in a day still my teetch is not so clean as it should be please give me some tips for clean teetch and for godo teetch thanks Doctor: Hi Welcome to HCM I would like to mention few things that will help you to maintain your teeth:- 1. Brush your teeth regularly in proper method. 2. Remove any irritants if any which causes gingivitis. 3. Remove plaque mechanically using chlorhexidine mouthwashes, listerine or triclosan. 4. Use soft bristle toothbrush and dental floss. 5. Brush your teeth in back-forth and up-down motion. 6. Visit your dentist and get your teeth cleaned professionally to remove the subgingival plaque in every 6 months. Take Care Regards Dr.Neha"
},
{
"id": 152965,
"tgt": "What kind of pain will be associated with ovarian cancer?",
"src": "Patient: I have had pain in lower right back for about 3 and 1/2 weeks. Thought it was muscle strain. Went to chiropracter. No better. Felt like a muscle spasm over kidney area for first 3 weeks, but in last several days, have noticed radiant pain moving toward front right. I am 46 years old, had a vaginal hysterectomy 8 years ago. Ovaries are still intact. I did not have uterine cancer. I am freaking out that my back ache could be ovarian cancer. I don t think it s kidney - as I have had stones before, and have had none of the symptoms associated with them this time. I feel some pressure on bending forward on lower front now also. Pain can be relieved while lying on back or walking. Can you describe the kind of pain that is associated with ovarian cancer? I can t find anything that described the back pain, though I know it can be a symptom. I have an appt w/ my GP on Friday. Thank you for any help you can give. Doctor: Hi,pain in lower rt back could be due to renal stone or may be spinal in origin.Pain which are neural in origin generaly get relieved by lying down.I don't think the pain which you had described is due to ovarian malignancy.Thanks"
},
{
"id": 139422,
"tgt": "Does neurocardiogenic syncope cause frequent fainting spells?",
"src": "Patient: well, my Mom has been living with neurocardiogenic syncope her entire life, but recently shes Been having fainting spells almost everyday. normally its during her shower before work, but some times its when she gets up in the morning to get in the shower. at work she is normally standing all day yet has very few instances of fainting at work. The good thing is that she can tell when shes going to have one and can lay down before she actually faints. i guess my question is, is it normal to have fainting spells this frequently? because my i want to believe that shes having fainting spells but some times it just feels like shes just skipping out on work. Doctor: Hi,Neurocardiogenic syncope, is also known as vasovagal syncope, is a fainting spell that happens when the body overreacts to certain triggers. Triggers could be emotions, extreme heat, dehydration, pain, long standing. To reduce the change of the spills I suggest good hydration (1-2 liters of water per day), avoid heat (Shower with warm to cold water rather than hot water), eat healthy and regular meals, and finally find ways to cope with stress.If your mother is dealing with stress at work she might have more spells. However, if the spells are increasing in frequency it is important to visit your family doctor. To rule out blood pressure issues, diabetes, or illnesses that affect the autonomic nervous system.Take care. Hope I have answered your question. Let me know if I can assist you further. Regards, Dr. Ahmad Nazzal, General & Family Physician"
},
{
"id": 148595,
"tgt": "Taking Phenytoin and Clonazepam. Have deja vu syndrome. Help",
"src": "Patient: Dear Dr.Keerthi, Lovely to know you are here to help us.I had to start taking A.E.D.s after a 2Hr. operation on my Femur to fill a cyst.when I was 12 Yrs.Now I am 55 !! I was on many A.E.D s like Primidone ,clonezepam,Topiramate, Phenytoin ,Phenobarb,Na-Valproate etc.at different times.The longest used is Eptoin .Now, I have started having the Deja-vu syndrome , especially during the night, when I am all on my own,listening to radio transmissions.I feel as if I want to jump across small rivers,try to catch plants on the other side of a stream,cannot remember where I am or was, do not know what next to do even after I take my toothbrush into my hand,forget to apply soap on my body when taking a shower......it has no end. furthermore, after I go to bed at around 10:00 p.m. I get the Deja-vu syndrome around 2:00 or 2:30 a.m. the next morning & find no sleep afterwards.At present I am on Phenytoin(100mg - t.d.s.) & Clonezepam(1mg) -t.d.s.Please advice on this.I would be very thankful if you can give reasons, especially over this site iself so that many may contribute, Thanks & Kind regards, Love,G.S. Doctor: Hi,Thank you for posting your query.Your symptoms are suggestive of complex partial seizures.MRI brain-epilepsy protocol and long term video EEG would help in confirming the diagnosis.You would benefit from medications such as oxcarbazepine or levetiracetam.Please discuss this with your neurologist.I hope my answer helps. Please get back if you require any additional information.Wishing you good health,Dr Sudhir Kumar MD (Internal Medicine), DM (Neurology)Senior Consultant NeurologistApollo Hospitals, Hyderabad, IndiaClick on this link to ask me a DIRECT QUERY: http://bit.ly/Dr-Sudhir-kumarMy BLOG: http://bestneurodoctor.blogspot.in"
},
{
"id": 194947,
"tgt": "Would excessive masturbation during childhood lead to erectile dysfunction in an adult?",
"src": "Patient: Dear Dr. I am 41 year (male) my problem from last few weeks I have problem of erection while I am with sex. I mate Ayurveda doctor he told me that due to heavy mustervation in child days this problem arose. I am unable sex at all. please help me to protect my conjugal life Doctor: Hi welcome to the health care magic First of all search for following causes of erectile dysfunction... -Smoking, alcohol history, as they should be avoided -Diabetes by RBS -prostate problem by USG and PR examination -Hypertension -Obesity by measuring your BMI-thiazide diuretic like drug history -Depression etc... Sometime cardiac problem might be the cause for which EKG done According to cause further specific treatment can be given Sildenafil like PDE 5 inhibitor will be beneficial that is taken before sex Consult physician or psychologist for examination and further work up Hope this will help you"
},
{
"id": 20022,
"tgt": "What causes increase in blood pressure inspite of changing medications?",
"src": "Patient: Hi, my husbands blood pressure has been on average 178/110 with some as high as 190/130. this has been going on for aprox 2 months. On Nov.2 his Dr. put him in hosp. for a week, changed his meds then let him go, he saw her a week later with no change. She then put him on Diovan 160mg, saw him 2 weeks later with no significant results. Last Fri. she doubled the dose of Diovan and wanted to see him tomorrow, now she is sick and can't see him til next week. His bp has not really gone down much and just now it is left arm 171/109, right arm 174/111. We live in a small town and not too sure about the level of care. Should he go to the hopital? Doctor: Hello!Welcome and thank you for asking on HcM!I passed carefully through your question and would explain that his blood pressure values are still high. This means that Diovan is not effective in controlling his blood pressure values. For this reason, I recommend consulting with another doctor and discussing the possibility of adding new anti-hypertensive drugs to his therapy (a diuretic like hydrochlorothyazide or a calcium channel blocker). It is also necessary performing some tests to investigate for the possible causes underlying his uncontrolled high blood pressure: - a chest X ray study- a resting ECG and cardiac ultrasound- complete blood count for anemia- fasting glucose for diabetes- thyroid hormone levels for thyroid dysfunction- cortisol and aldosterone plasma levels for adrenal gland dysfunction- blood electrolytes for possible imbalance- kidney and liver function tests. You should discuss with his doctor on the above issues. Kind regards, Dr. Iliri"
},
{
"id": 187467,
"tgt": "What should i do for swelling in bottom lip after dental work?",
"src": "Patient: I got dental work today so my bottom lip is still completely numb (it's been 4 hours since). I was eating diner and I fully bit down on my bottom lip, it's turning purple. What's wrong and what should I do given I can't feel anything and it's little bleed?? Doctor: Hello, Thanks for your query.The numbing stuff (novacaine) can make your lip swollen, and then when you bite the swollen numb part, it only gets fatter! So here's what to do. Ice works wonders for swelling, as well as Advil. Apply ice for 15 minutes, take Advil capsule, and then apply some cankersoar medicine on the open cut. You'll be much better in a few days. I do hope that you have found something helpful and I will be glad to answer any further query.Take care"
},
{
"id": 13393,
"tgt": "Suggest remedy for rashes around the ears",
"src": "Patient: Hi I had a running stomach where i had to go like twice ever hour this past weekend but now it stopped and i am ok but i notice now i have rashes around both my ears could this be connected and what can i use for treatment ...my stomach is back to normal now Doctor: Hi, It may be contact allergic dermatitis. It is not related to stomach upset. It may be due to contact with some allergens like dust, oil, shampoo, soap etc. Consult the dermatologist for the perfect diagnosis and proper treatment. Antihistaminics like Cetirizine might improve the condition. Apply mild steroid cream. Hope I have answered your query. Let me know if I can assist you further. Regards, Dr. Ilyas Patel, Dermatologist"
},
{
"id": 33951,
"tgt": "What can cause blackout and light head after breast reduction?",
"src": "Patient: Hi, I had a breast reduction 11 days ago, and Im feeling pretty good. I havent taken any meds in about 3 days and Im not in much pain at all. Ive been showering and washing my incisions and putting antibacterial ointment on them as told by my doctor. So today when I was standing and cleaning off my incisions to go get my stitches removed, I became very lightheaded and started to blackout. I sat down and in a couple minutes..I was fine.. any ideas? Doctor: Hello dear,Thank you for your contact to health care magic.I read and understand your concern. I am Dr Arun Tank answering your concern.It is a hypotension that has caused you make this.Nothing to worry. It is common in some operative patients.As during the operation our body system becomes somewhat weak.So when you remain standing for prolong period or you can stand after prolong sitting such episode can occur.But it is nothing to worry in single such episode. But if it recurr on second or third time than you have to check your blood pressure. Taking a appropriate treatment can cure your such a attack.Please take enough sleep and forget the episode that has occurred with you. This will help cure your problems.I will be happy to answer your further concern on bit.ly/DrArun.Thank you,Dr Arun TankInfectious diseases specialist,HCM."
},
{
"id": 13629,
"tgt": "How are hives treated?",
"src": "Patient: HELLO, I HAVE BEEN BREAKING OUT WITH HIVES FOR 8 YRS. EVERY DAY I WOULD LIKE TO KNOW IF YOU OR DR LOCATE IN DALLAS, TX. THIS PAST WEEK MY HIVES IS OUT OF CONTROL. I GET HIVES ALL OVER MY BODY EVERY DAY. I HAVE BEEN TAKE PREDNISONE EVERY OTHER DAY. I NEED SOME HELP, SOON AS POSSBILE. Doctor: Hi,Hives on the body seems to be most probably due to allergic reaction.I would suggest you to consult an Allergist and get evaluated and a thorough clinical evaluation and investigations like allergic tests like Patch test, RAST test, food challenge test etc can help in diagnosis and treatment can be done accordingly..The tests will help in ruling out the exact cause of the allergen and once you know the cause of the allergic reaction you can avoid exposure to it to prevent allergy attack..The allergist can advise to oral anti allergics as well as injectable steroids for relief..As of now you can take anti-allergics like Levocetrizine, Allegra to control the itching and swelling..Do cool compresses..Application of calamine lotion can help in soothing the skin..You can also be instilled an Immunotherapy that will help in reducing the severity and frequency of allergic attack..Hope I have answered your query. Let me know if I can assist you further."
},
{
"id": 109011,
"tgt": "Suggest treatment for lower back pain",
"src": "Patient: I am 41 years old and overweight. I have been having low back pain for many years, triggered by too much standing or bending, but it has been tolerable with ibuprofen. Since it got cold a few weeks ago it is unrelenting, not brought on by activity, and much worse (but still not severe). My mom, who is deceased, had a lot of back problems but I don t know what all they were besides arthritis. I had xrays done last night and have the images but it is going to be a week before I get a report due to Thanksgiving. I was hoping to have someone look at them and tell me if there appears to be anything unusual. Doctor: Hi,Looking to the history your low back pain might be due to some degenerative changes in your lower vertebrae producing oste- artheritis .There might be having osteophytes formatiom, slipped disc or damage to vertebrae giving pinched nerve pressure or local muscle stiffness.Go for x-ray lumbo-sacral region.Consult orthopedic surgeon and get examined.Physiotherapy, short way diathermy will be of much helpful.Take calcium, vitamin A and D supplements.Avoid weight lifting and more bending as it might aggravate your condition.Ok and take care."
},
{
"id": 194835,
"tgt": "How can addiction to masturbation leading to a lack of social life and depression be treated?",
"src": "Patient: Hi I am 18 years old and I can t stop masturbation . I was 12 when I started it. Now I feel I have destroyed my whole life. I started to feel less happy and energetic. And because of masturbation I spend more time on it rather than having friend, studying, doing sports, having hobbies, ..etc. now I feel alone, depressed, unhappy and ashamed . I need help Doctor: dear userfirst you should not be shamed of that severe masturbation is acommon problem especially in your age and you shouldnot be shamed of that as it will lead you for depression and stress and psychological problems that will harm you more and more.....try to see your doctor to check if you have any infection genital or urinary...as infection may be acause of severe irritaion lead you to have great desire to masturbate.....2nd try to monitor your life style and have healthy food reach of fibers and fo regular excercise will help alot ....regular excersise utilize your energy and masturbation will decrease....you may have psychological problem with sex or may be you have emotional disorders....at last try to live simply away from stress or emotional troubles and do regular excersise and have friends and be happy .......dont get shamed .......wish you healthy life"
},
{
"id": 44560,
"tgt": "Low sperm count, infertility , more intercourse the solution ?",
"src": "Patient: my fiance was told that he has low sperm count and that his chances of having a child are slim so if he gets off more than once can that help our chances of having a child Doctor: There are many known causes for male infertility. The initial diagnostic test is a semen analysis. Studies suggest that normal fertility is associated with sperm counts of > 48 million/mL with a motility of > 63%, with > 12% having normal morphology. Subfertility is seen when sperm counts are There is a genetic component, usually, with azoospermia (absence of sperm) or oligospermia (low sperm count). However, there may be other reasons such as low testosterone levels. Other disorders of the testes are often associated with impaired spermatogenesis (low sperm production). In any case of impaired fertility, consultation with a fertility specialist to determine the exact cause of infertility. Increased frequency of ejaculation does not increase your chances of becoming pregnant. Be well. Dr. Kimberly"
},
{
"id": 201268,
"tgt": "Does circumcision cause inability to ejaculate?",
"src": "Patient: I had a circumcision last of April, I am 64 years old, had semi erections while walking around for a few months, now that I have a new girl friend, since June, we have been having sex since July, but I never climax, or ejakulate, I dont even get to the point of sexual spasms? I enjoy sex with her although I am unable to maintain an erection with out taking half of a viagara. Is inability to ejakulate someting to worry about, or a side effect of circumcison? girl friend wonders If I am just to psychologically blocked? My wife of 41 years died 2 years ago?? Doctor: DearWe understand your concernsI went through your details. I suggest you not to worry much. I can assure you that circumcision is not the reason for your present condition. Premature ejaculation, delayed ejaculation, erectile dysfunction and lack of interest in sexual matters are mainly due to anxiety and lack of knowledge. You are anxious or rather apprehensive. You are worried about your sexual performance. Such worries, rather apprehension drive you anxious.Have faith and confidence. Stop worrying about future. Psychotherapy techniques should suit your requirement. If you require more of my help in this aspect, Please post a direct question to me in this URL. http://goo.gl/aYW2pR. Make sure that you include every minute details possible. I shall prescribe the needed psychotherapy techniques.Hope this answers your query. Available for further clarifications.Good luck."
},
{
"id": 41719,
"tgt": "What are the chances of pregnancy having intercourse after the rupture of follicles?",
"src": "Patient: My period started on the 7th of July and I went to my appointment on the 28th of July and they indicated my follicles are open, then had sex on the 28th until 2 of aug I took a LH surge test and it prompt positive on ovulating and wonder if I had missed my chance on getting pregnant? Doctor: Hi welcome to healthcare magic.I have gone through your question.As lh surge indicates ovulation timing with more accuracy and you had intercourse after rupture of follicles and there after has great chance of conceiving.Hope i answered your question.Would be happy to help you further.Take care."
},
{
"id": 8055,
"tgt": "How to make my skin smooth & fair ?",
"src": "Patient: Hello Doctor ! I m 19 years old. I was having so many pimples on my face . Now i don t have much pimples but skin became so rough, it doesn t look good. What should i do to make my skin fair, smooth & shiny ?? Please help me ... Thank You Sagar Jaiswal Doctor: hi sagar welcome, probably you might have use so many lotions to clean the face during pimples. now what to do , do the face wash with alternate warm and cold water apply some red sandal wood and turmeric powder in a paste form over face and wash after some time (20 minutes) with cold water. thanking you"
},
{
"id": 119577,
"tgt": "What causes pain radiating from sacral area to the hip?",
"src": "Patient: I started having pain which radiated from my sacral area, over my hip and down the front of my thigh. I went to a chiropractor who helped with the back and hip pain but i have a constant intense throbbing pain in the front of my thigh....any ideas? I am 60 yrs old. 6 ft. tall and approximately 150lbs. Have a history of osteopenia. Doctor: Hello,Your symptoms could be suggestive of chronic degeneration in the coxo-femoral joint or sacro-ileal. For this reason, I would recommend performing a pelvic X-ray study, coupled with both hip joints X-ray study.Take care. Hope I have answered your question. Let me know if I can assist you further. Regards, Dr. Ilir Sharka, Cardiologist"
},
{
"id": 226253,
"tgt": "Bruise and pain on the depo shot site. Anything to be done?",
"src": "Patient: i recived the depo shot 3 days ago this is my second shot the nurse that did it this time told me this was her first time giving this shot when she gave it to me it hurt more then the last one she said not to rub it or itd bruise and i did not rub it and i now have a large bruise, pain and and when you touch around where the bruise is it feels like theres a large hard bump in there and this did not happen the first time should i be worried Doctor: Hi, Welcome to the forum There is nothing to worry, it will get fine in 1-2 days. You can apply a thin film of neomycin oitment on the bruise. If it is painful you can take an analgesic. Do not scratch or rub. To reduce the inflammation and hardness you can apply thrombophob oitment, it will aid easy blood circulation and reduce the swelling. Do not let the bruise get infected. However, if you see any pus forming, you should take antibiotics to help wound dry up and get well early. If wound remains the same get clinically assessed and treated. When you take the next shot, do not take it in the same spot, take care."
},
{
"id": 33852,
"tgt": "What causes feverish feeling and cold sweats?",
"src": "Patient: My body feels like it is burning up, but I am cold and clammy to the touch, I cannot tolerate heat in anyway shape or form, I must sleep with a fan on at all times. I sweat terribly in my face, my husband feels me and says your skin is so cold, yet I feel like I am burning up. I used to think they were hot flashes, excessive, but no other woman I know having had a hysterectomy does this. This has gone on for years. My last thyroid check was borderline high. What could this be? I live in a small town, and the Dr s here seem to drop the ball a lot, always suggesting it is anxiety. Doctor: Hello,Welcome to Health Care Magic.Thanks for writing.I am Dr.Saddiq ul Abidin. I have read your question completely, i understand your concern and will try to help you in best way possible.You symptoms of sweating irritability, intolerance to heat and feeling of hot flushes suggest they are caused by hyperthyroidism. If you are loosing weight too and your resting pulse is raised more than 90 BPM and you are having diarrhoea often, you need to get your thyroid checked.Yes there can be a part of anxiety too in causing your symptoms but most part of your problem is suggestive to a thyroid issues if you can control that things will settle down.I would suggest you to visit your doctor immediately and get a thyroid test done. If it is high then you can proceed further to follow this us and get it treated.Till then you can stay in fan and air conditioner and i would suggest my similar patients to use Inderal twice a day but it is highly stressed that you should visit the doctor as soon as possible.I hope this answered your question.If you have more queries I am happy to answer.Otherwise rate and close the discussionRegrads.Dr.Saddiq ul AbidinM.B.B.S(Licensed Family Physician)Resident Medicine."
},
{
"id": 185791,
"tgt": "What causes declassification of the top teeth which looks like spot?",
"src": "Patient: My two yr old son has decalcification on his top teeth. I have been brushing his teeth up to six times a day. I have noticed a tiny spot on two of his teeth. The dot is located on the white spot it is the color of his tooth its just located on the spot. I don't know if its a cavity or if its a sign of remineralization. How can u tell if you teeth are remineralizing? Doctor: Hello:)Welcome to HCM.Excessive tooth brushing is contraindicated.You will erode the enamel with this.Just brush twice,once in the morning and once before bed.You can make him rinse with a saline mouth wash once a day.White spots can be sign of decayed teeth or fluoride discoloration.See your dentist for a clinical examination. Re-mineralisation in teeth can happen, but it is very minimal.Regards."
},
{
"id": 70973,
"tgt": "Can you get swollen lymph nodes from bronchitis?",
"src": "Patient: So about a month ago, I went into my doctor with sinus congestion and cough. I had also noticed that on both sides of my collarbone a lymph node had become swollen. He said that I had a long standing sinus infection and gave me Biaxin, as I am allergic to -cillin. He said the lymph nodes should go away when the infection is gone. Two weeks later, the sinus pressure had relieved but the cough had become worse. I went to my backup doctor who said the sinus infection had turned into bronchitis. Thus, he gave me Doxycycline and a 5 day course of 10 mg prednisone. He also ordered a chest X-ray that I am awaiting the result of. My question is neither of the doctors seemed worried that both of my supraclavicular lymph nodes were swollen. Would the lymph nodes still be swollen with the bronchitis? Neither doctor really felt them. they are soft and move around when pressed but with some discomfort and feel like the size and shape of small bean. At what time frame should I ask for them to look into the lymph nodes more?A little about my family history: my family is generally healthy besides heart disease on my dad's side of the family and my great-grandpa died from lung cancer. Doctor: Hello and Welcome to \u2018Ask A Doctor\u2019 service. I have reviewed your query and here is my advice. Usually bronchitis doesn't cause lymph nodes. As you explain the history yes there is an infection present and you should discuss with your doctor for this. Hope I have answered your query. Let me know if I can assist you further. Regards, Dr. Jolanda"
},
{
"id": 21564,
"tgt": "What causes stabbing pain in the middle of chest?",
"src": "Patient: Hi, may I answer your health queries right now ? Please type your query here...i have a sharp stabbing pain in the middle of my chest under the thyroid gland that only lasts for a second and it leaves me with a dizzy spell and sometimes a head spin .. Sometimes i feel a fast pulse in my lower throat after the sharp pain that happens in the middle of my chest . Doctor: Hi,Localized chest pain is rarely cardiac. But stabbing chest pain can be because of abnormality in integrity of aortic wall. Considering the dizzy spells, the possibilities of it is pretty high.So to rule it out, i will recommend you for a 2D Echo of the heart & aorta.If that is normal you can get a TMT done to rule out coronary disease as other possible cause.Come back if you need any more help.Thanks"
},
{
"id": 58509,
"tgt": "Pain in lower ribs. Elevated SGOT, SGPT, CRP. Need to go to ER?",
"src": "Patient: I have pain under my lower ribs where my liver is located. I do not have a gallbladder or appendiz anymore. My SGOT and Sgpt were elevated including elecvated CRP. My doctor wants me to wait a week and repeat labs. I personally think I need a CT or US sooner than later. I worked just as a unit secretary in an ER for many years. Would you go into the ER or would you wait? Doctor: HelloThanks for writing to HCMYou didn't mentioned your age.There may be various reasons for pain.You need proper clinical evaluation and investigations.Pain may be due to compromised liver function. You didn't mentioned about bilirubin level and alcohol history.Any way, you need routine hemogram,LFT,RBS,Lipid profile,RFT,urine RE/ME,viral markers, ultrasound of abdomen and ECG and CT Scan may be done if needed.Stop taking alcohol if you are alcoholic.Musculoskeletal causes of pain should also be ruled out.Get well soon.Take CareDr.Indu Bhushan"
},
{
"id": 59117,
"tgt": "Gall bladder removed due to stones in the bile duct. Had infection. Difficulty in breathing. Reason?",
"src": "Patient: Father in law had stones in bile duct and hen had his gallbladder removed. He then had an infection so they inserted a drainage bag for two weeks. Now he can not breathe after eating. His stomach gets really hard and it suffocates him. He has to press on his belly using a table to gain his breath back. It now happens more often and not after eating. What can this be from? Doctor: Hi and welcome to HCM. First you should rule out intraabdominal fluid accumulation and pleural efusion. You need to do ultarsounds and chest xray. He oviosuly had comlications after surgical procedure and now he should have frequent check ups. ALso it can be casued by postcholecystectomy syndrome and it usually takes few months after this till digestion stabilize. Wish you good health."
},
{
"id": 151061,
"tgt": "Have frontal cortex calcification. Have loss of balance, slow at thinking. How important is this?",
"src": "Patient: i have frontal cortex calcification the size of which is about 5 inches right beneath where the dura matter was penetrated by trauma. Also a space occupying lesion midline shift and a piece of the cerebellum sheared off in a lipomous anomaly.........what do you think of the importance of this....i find loss of balance inability to organize and multitask and generally am slow at thinking and responding execially under pressure Doctor: i would like to know when was the trauma . the lipomatous abnormality might be congential"
},
{
"id": 54659,
"tgt": "What are the side effects of primary complex(rifampin, isoniazid & pza)?",
"src": "Patient: My granddaughter is 4 yr old & is on medication fo primary complex ( rifampicin, isonizid & pza) for less than a wk now. Are ther effects that we must be worry about? I know t liver damage is one effect, can this giv other problems like recurrent fever? Just this morning we notice a slight blood ( very small amount dry blood on one of her teeth-upper). Must we be alarmed? Thank you. God bless. Doctor: Welcome at HCM I have gone through your query and being your physician I completely understand your health concerns These medicines have got variety of side effects like liver damage, derangement in renal functions , increased uric acid levels. There is need to get the frequent liver function tests analysis so that dose of the drugs can be adjusted. In addition to it, it also causes pink discoloration of body secretions like in tears , urine and slaiva. they do not cause recurrent fever. maintain proper follow up with your physician. Meanwhile stay calm Get well soon Hope your query is adequately addressed if you still have any feel free to ask RegardsDr Saad Sultan"
},
{
"id": 74319,
"tgt": "What causes chest pain with breathing difficulty?",
"src": "Patient: I have had alot of heart test done , also ultrasound and blood work all a came back good but still have different epoisodes i deal with chest pain having the feeling to always take a deep breath all the time ,they put me on xanax for anaxiety and also had an upper gi it also came back good so what could it be Doctor: Respected user , HiThanks for using Healthcaremagic.comI have evaluated your query thoroughly .* There is no mention about evaluation of x-ray chest , pulmonary function test which evaluate lungs .* If not done , strongly recommended and if done and normal then it proves to be a psychosomatic issue .* Recommended to - meditate thoroughly - deep breathing exercises - YOGA - avoid smoking ( active / passive ) - avoid exposure to dusty environments , if needed use face mask . - avoid stress, anxiety . - have regular sound sleep of 8 hrs .Hope this clears your query .Welcome for any further guidance .Regards dear take care ."
},
{
"id": 71137,
"tgt": "Suggest treatment for coughing up of mucus despite taking Albuterol",
"src": "Patient: My 78 year old mother has the flu which caused Copd exacerbation. She is taking Cipro and was given guaifenesin with codeine cough syrup. She has been fever free for 3 days now. She also was put on a prednisone taper starting at 60mg for 2 days 50 mg for 2 days and now we are down to 40 mg tapering down over a week and then going down 10 mg per week. she also take Spiriva once daily and symbicort 2 puff twice daily. Along with albuterol breathing nebulizer treatment about 4 times per day. She is eating well but is still fatigued. She is on 02 2.5 liters per nasal cannula during the day and bipap at night. She still has mucus which is hard for her to cough up. Her 02 sats are between 90-95% with O2. my question is concerning the cough syrup and whether I should continue to give it too her. Would it be better just to give her guaifenesin alone without the codeine? She had a chest xray a week a ago from Saturday no pneumonia was noted. She is also very confused but I know that goes with the disease process and meds. Do you have any suggestions she has been sick for a couple week now. Doctor: Hello and Welcome to \u2018Ask A Doctor\u2019 service. I have reviewed your query and here is my advice. According to history don't worry. Usually in COPD, it is nearly normal to have sputum. However, if you think that this is not so normal for her situation than better to do a chest X-ray. Regards, Dr.Jolanda"
},
{
"id": 26421,
"tgt": "Can a boiled egg per day be helpful to reduce high BP?",
"src": "Patient: I am 38 and having hypertension problem for the last three years approximately. Can I use one boiled egg per day? Is it helpful to reduce the high blood pressure. Please advise how and in which form the egg is most suitable for hypertension problem. Thanks Imran Doctor: Hello Imran, There is no direct Co relation would off egg with hypertension, it doesn't decrease your blood pressure nor taking it will increase the pressure. So you may take eggs, boiled are always better because it's healthy and egg white is one which is Rich in protein, while the yellow is rich in cholesterol. So if you are overweight and your lipids are bad, you should try avoiding egg yellow, and if you are otherwise fit, 2 parts egg white and 1 part egg yellow is a balanced combination. Regards Dr Priyank Mody"
},
{
"id": 81507,
"tgt": "What are the symptoms of Sarcoidosis?",
"src": "Patient: Hi I have a friend who is experiencing lacrimal gland swelling, general body aches, and yellowish vision from one eye. She has in the past had and-ray that revealed 3 dark spots. She also has all over body aches and was wondering if this could be Sarcoidosis? Doctor: Thanks for your question on HCM.Sarcoidosis is multi system disorder. And affect all most all organs of the body.The most commonly involved organs are1. Lungs. Hilar and lower zone nodules.2. Skin. Sarcoid tubercle.3. Brain. Meningitis and sarcoid nodules in brain.4. Eyes. Conjunctivitis and iritis.5. Kidneys. Glomerulonephritis. Etc.So your friend's symptoms may be due to sarcoidosis.But he should first get done serum ACE level. If this is high then possibility of sarcoidosis is there.So consult pulmonologist and discuss all these."
},
{
"id": 99860,
"tgt": "What is the treatment for allergy , headache and post nasal drip?",
"src": "Patient: Hello, My children and I have all been suffering with increasingly bothersome allergies for the past 2 weeks which is normal here. However, for the past 2 days we have all developed the same symptoms: headache and post nasal drip (continuing from the allergies) cough and significant pain in our lungs when we cough and extreme fatigue. Is this likely due to our allergies increasing in severity or is it more plausible that we're all coming down with something? Thank you! Doctor: Hello,Thank you for asking at HCM.I went through your history and would like to make suggestions for you as follows:1. I would like to know your ages, history of fever, any other symptoms, your medications, previous allergy testing, etc. However, at present, I would make suggestions from your given history only.2. I usually suggest my such patients regular montelukast and levocetirizine for a few weeks depending upon response.3. Regular steam inhalation also helps to relieve congestion & headache.4. Please have plenty of warm fluids during day.5. Please avoid exposure to dusts, smokes, air pollution and cold air as much as possible.6. As all family members have worsening simultaneously, I would think of some common trigger factor such as infection, allergen exposure, exposure to irritants, cold air, environmental change, etc.7. If you have not been tested for allergies, I would suggest you allergy testing for common air-borne allergens such as house dust mites, molds, pollens, insect proteins, animal dander, etc. This will help you identify the substances causing allergies to you as well as to know the measures to avoid them.8. Based on allergy testing, an Allergist-Immunologist may prescribe you allergen specific immunotherapy which gradually improves allergy symptoms by working on immune system.Hope above suggestions will be helpful to you.Should you have any further query, please feel free to ask at HCM.Wish you the best of the health ahead.Thank you & Regards."
},
{
"id": 19662,
"tgt": "Is it not normal to have 190/90 BP?",
"src": "Patient: My grandmmother is 81 yrs old. Checked her bp several times over a 3hr period and always got 190/90. Doctor gave her CAPTOPRIL 10 minutes ago. Her hands and feet are cold. She's weak and can't stand up. She can move her arms and feet and didn't have a hard time in the bathroom 3 hrs ago. Pls. advise. Tks. Doctor: In my opinion she should be started only on Amlodipine (Norvasc) 5mg once daily in the morning. Wait for atleast a week while having her B.P recorded during that week 3 times per day. The new drug added may take some time to add its antihypertensive effects. For her the target resting BP should be below 140/90mmHg. The dose of Amlodipine may be titrated to 10mg daily if BP still going above 160/90mmHg. If you need further help regarding BP control stategy, feel free to contact."
},
{
"id": 111381,
"tgt": "Could back pain be treated by advil?",
"src": "Patient: I have back pain, right side, above waist, no symptoms otherwise of kidney infection nor do I get them. It's getting worse since last night when it started suddenly, Advil didn't dent it. Trying to figure out what's causing it. I'm physically active, no problems. Thanks! L Doctor: Quite often the cause of back pain is musculoskeletal. You may have slept awkwardly, or incurred some other gradual physical trauma. Topical ice would help control the stifness/spasms. You also could apply topical pain cream such as Aspercreme. If there are no other symptoms, the it's just a matter of rest and pain control until it heals and resolves. But if over the counter pain meds aren't cutting it, then you may need to see your doctor to get something stronger, and/or a muscle relaxant. Another quick option is chiropractic."
},
{
"id": 13628,
"tgt": "Suggest medication for itchy bumps around the vagina",
"src": "Patient: I had a c-section a month ago tomorrow and I ve had itchy bumps around vaginal opening for about a week now. I had a yeast infection and treated it with monistat but the outside bumps won t go away. When I walk and do activities that rub the Ares they itch really bad and hurt. Doctor: Hi, The itchy bump on your vagina could be due to yeast infection you had previously. I highly recommend you to consult with your gynecologist and run vaginal swab testing to confirm yeast infection. For the moment, I advise you to keep good local hygiene. Avoid sexual intercourse until betterment. Apply Clotrimazole vaginal cream locally. Applying yogurt or vaginal probiotic cream can ease itching. Hope I have answered your query. Let me know if I can assist you further."
},
{
"id": 64252,
"tgt": "What causes lumps with bruising on the bicep and forearm?",
"src": "Patient: My husband had a huge bruise on his bicep area that had a huge lump in it and it has started to go away and now he has one on the same arm on his forearm area that is once again a huge bruised area with a lump. Any reason to be concerned about this? Doctor: Hi,Good Evening.Thanks for the query to HCM.I studied it in depth and I understood your health concerns.-Causes -for lumps with bruising on biceps and arm-1- Mostly it appears to be -URTICARIA ? HIVES due to IU-idiopathic Urticaria.2-You need not be concerned but be cautious to treat them to reduce their discomfort and ill health.3-Thus a Physician would fix the causes of the lumps .Dont get scared and uneasy,but be cautious and act fast.Hope this would help you a lot to relieve you.Wish you fast recovery-and healthy life.Wellcome to HCM with more queries till you are satisfied.Have a Good Day...!!Dr.SAVASKAR M.N.M.S.GENL-CVTS,Super specialist and Senior Consultant-and Expert in Non-Curable-Disease therapy for Cancer,Asthma,etc,Rejuvenation therapy and Tissue failure -reversal therapies."
},
{
"id": 42267,
"tgt": "How can one conceive after repeated failures?",
"src": "Patient: hi iam maya , and my age is 25 now, My husband is suffering from HIV and we are partispating in safe sex still now , four months back I have gone through IUI treatment using donars sperm donation and got failed and next month I had IVF treatment and got failed , now again iam planing to get IUI after 8 months time gap from last treatment , could you advice me what to do Doctor: HIThanks for posting your query to HCM . Before going for assisted conception you need to get checked for Thyroid disorders , Vitamin D3 levels , Diabetes , and the status of your Uterus lining ( Endometrium ) If at all , these having been tested and are in normal limits then there is nothing extra you need to do . Just follow the advise from your doctors , take the pills as advised . One thing I want to know is at what stage did your IVF failed ? Were your doctors able to make an embryo and implant it inside your uterus or they were not able to form the embryo or were they not able to retrieve your Egg ? Because solution depends on these factors . Kindly revert back to me with details for my questions .."
},
{
"id": 66295,
"tgt": "Need treatment for knot on the leg after falling down",
"src": "Patient: I fell from the step in the bedroom while getting into bed. There's a knot on the right side of my lower right leg, but the worst is whenever I put weight on my foot or try to bed my foot upward. There's also a terrible bruise on the ball and surrounding area of my foot. Doctor: Hello and welcome to HCM,Knot on right side of leg along with bruising on ball of foot can be seen after trauma to the foot and leg.Bruising can resolve after few days and the lump can also resolve after few days.Most of the lumps after trauma are due to collection of blood in the subcutaneous tissue.In case there is associated pain, you can take pain-killers and hot fomentation.Consult your healthcare provider if lump and bruising do not resolve within few days.Thanks and take careDr Shailja Puri"
},
{
"id": 177704,
"tgt": "How to treat swollen, red tonsils, dry cough and runny nose?",
"src": "Patient: My 12 year grandson has allergies he currently takes prescription Zyrtec and prescription flonase. He quite frequently get swollen,red tonsils, dry cough,,sometimes runny sometimes stuffy nose. This time around it has been 2 weeks of him being miserable. And his pediatrician keeps advising some other over the counter medicine. He never gets a fever with this. Doctor: Hi...he seems to be having moderate to severe persistent allergic rhinitis and I am glad that he is on Flonase and Zyrtec which is going on in the right way. If I were your treating pediatrician I would add on - Monteluekast, 10mg per day once daily at 6PM for 8 weeks. It doesn't work immediately but on the long run it is very useful in reducing chronic allergic symptoms like this. Kindly approach your pediatrician regarding this.Regards - Dr. Sumanth"
},
{
"id": 78517,
"tgt": "Is forecox tablet the right medication for TB?",
"src": "Patient: i am suffering from tb as diagnosed by the doctors. i have been advised to take forecox tablet (2 in empty stomack). i have complited one month. The problems are high weakness, diziness and loss of wiegt. i want to know whether the tretment is going in right doirection. Doctor: Yes...the treatment is right...The mentioned symptoms are just few side effects of the Anti-tubercular drugs...."
},
{
"id": 194215,
"tgt": "What causes blood in semen?",
"src": "Patient: Hii Sir..i have a problem when i had intercourse with my wife after dat when my wife went for pee she saw some blood drops coming out with my semen i could'nt get is the problem with her or with me is dis a serious health problem wat shuld i do now..please help me am totally disturbed is dis create any infertility problems please guide me..thnx.. Doctor: Hi, No need to worry next time have sex with a condom so will know for sure. If it's your blood will be inside condom then we can think of the diagnosis. Hope I have answered your query. Let me know if I can assist you further. Regards, Dr. S.R.Raveendran, Sexologist"
},
{
"id": 104954,
"tgt": "Running nose, wheezing, breathlessness during winter. Problem persists even after putting nasal drops. Solution?",
"src": "Patient: i have allergy problem..during winter season i get severe running nose ..some times wheezing and breathlessness.. i used seroflo inhaler during night but for the past few days my nose is blocked in the upper portion and i cannot take air through the nose..i used nasal drops i get relief and again aftr 6 hrs the problem repeats again.. what should i do? Doctor: Hello, It is very likely you suffer from house dust mite allergy and will need either blood tests or specific IgE tests to confirm this. Other aeroallergens such as moulds, animal dander (if you have pets) should also be checked. You need a combination of long-acting antihistamines such as Fexofenadine 180mg in the morning, mometasone furoate nasal spray 1 spray per nostril once daily, Montelukast 10mg at night for the winter months (3 months) to see the benefit. Obviously the best decision on duration & time of treatment will be decided as per the results. Best Wishes."
},
{
"id": 99927,
"tgt": "Suggest treatment for asthma",
"src": "Patient: Hello Doctor, I am 50 yrs. old. My problem is asthama since 1980 & I am taken daily a table of Betnesol. If I cannot take this table I feel some problem. 1. I can not Sleep properly 2. headack problem 3. constipation problem Can I releave from this problem . Doctor: Hi thanks for using Healthcaremagic I think here the problem is steroid dependence than asthma Sure, you will have to be on some steroids But inhalational steroids is also effective..You cannot stop oral steroid(betnesol) all of a sudden.Your body is used to steroids from outside,hence when you stop betnesol enough amount of steroids won't be produced in the body.Hence we will have to stop it gradually..Longterm steroid therapy can make you Cushingoid (excess of steroids)Hence it is better to consult an Endocrinologist and he can help you...."
},
{
"id": 92423,
"tgt": "Having abdomen pain, mouth dries, heart hurts, getting acid reflex. What to do?",
"src": "Patient: hello i am having som upper but lower abdomen pain at night. My mouth dries out then about ten mins after my side start hurting. I Noticed that when i was pooping the other day a ver small amount of blood came out my pee hole. Also my heart hurts sometimes when im having acid reflex Doctor: Hi, The lower abdominal pain and drying mouth are sign of a acid reflux (GERD). I'd suggest to: - consult GI specialist for further evaluation - run upper fibrogastroscopy - eat often and small portions - do not lay down immediately after eating - use 2 pillows when sleeping - avoid chillies, fried, spicy, prickled foods - avoid coffee, alcohol, carbonated drinks With regards to the blood coming from your pee hole could be related to possible urinary tract infection, for which I suggest to : - do urine analysis - abdominal ultrasound All the best! Dr.Behar"
},
{
"id": 207278,
"tgt": "Suggest treatment for mental disorder",
"src": "Patient: Hi, I am looking for contact details of Psychiatrist from calicut region. This for a treatement of a person who is going through mental disorder for last few years. We had approached to many doctors in past but doesn't cure problem. Any help in this regard will be much appriciated regards, Rajesh Doctor: DearWe understand your concerns.I went through your details. I suggest you to keep calm. Mental disorders are of two types. Some strictly requires medicines and the other do not need medicines. The first category needs medicines along with psychotherapy and rehabilitation therapies. Second type needs psychotherapy and other psychology techniques and exercises. You mentioned that treatment is not effective. To get proper treatment and cure, proper diagnosis is needed. I do not know any other details about the mental disorder which you re mentioning here. I am from Ottapalam, Palakkad. You can post a direct query for me giving all the details. After knowing all the details, I shall provide you the right treatment guidelines.In case if you need more of my services in this regard, please post a direct query. I am happy to reply you.Hope this answers your query. Available for further clarifications.Good luck."
},
{
"id": 180901,
"tgt": "What do small bumps on the soft palate indicate?",
"src": "Patient: I have tiny flesh colored pin size bumps on my soft palate and around the area where my nosal passage would drain into my throat. I don't feel them, there is no pain and I noticed then a little over a month ago. I've got post nasal drip almost daily, from what I think is allergies. What are they? Doctor: Hi..Thanks for the query..Small flesh coloured bumps on the soft palate and throat with post nasal drip can be due to causes like Sinusitis, Allergic reaction etc..So first of all you should consult an Emergency room and get evaluated and a clinical examination can be done and if needed a sinus x ray or blood test to rule out allergies can be done..For now you should do warm saline gargles and antiseptic mouthwash gargles.Sip on warm drinks and avoid spicy food..Hope this helps..Regards.."
},
{
"id": 71228,
"tgt": "What is the remedy for tingling sensation in the chest?",
"src": "Patient: Experiencing odd chest pain in my right chest. Not really pain, but tingling that comes on my right pectoral muscle. My chest is sort of sore as well. There is some tightness associated when the tingling sensation happens. Over last day the pain is now radiating to my center chest, even slightly to the left side. Doctor: Hello and Welcome to \u2018Ask A Doctor\u2019 service.I have reviewed your query and here is my advice.By your history and description, the possibility of pectoral muscle spasm is more likely. Most common cause for this is calcium, vitamin D3, and B12 deficiency. So better to get done serum levels of calcium, vitamin D3 and B 12. If the deficiency is there then you will need supplements. Also, apply warm water pad on affected areas of the chest. Don't worry, you will be alright with all these. Hope I have answered your query. Let me know if I can assist you further.Regards, Dr. Kaushal Bhavsar"
},
{
"id": 31747,
"tgt": "Suggest treatment for ulcer in the tongue & lips",
"src": "Patient: My son has had a cold, symptoms similar to strep with a low grade fever off an on. yesterdat Dr. said not strep or flu but gave arithromyacin just in case. Just noticed 3 large red splotches on his tongue and a little ulcer on inside of his lip. Any thoughts?. Doctor: Hi..Welcome to HEALTHCARE MAGIC..I have gone through your query and can understand your concerns..As per your complain in case if the splotches and ulcer in tongue have appeared after taking antibiotic Azithromycin it can be due to allergic reaction or side effect with the antibiotic while it can also be due to apthous stomatitis secondary to throat infection and inflammation..I would suggest you to consult your treating physician and get evaluated and he can advise you to change the antibiotic.As of now he can gargle with numbing mouthwash containing Lignocaine..You can also take multivitamins daily...Take anti-inflammatory painkiller like Motrin..Do warm saline gargles and suck ice pops..In case if it is due to side effect of antibiotic anti-allergics like Benadryl can be taken..Hope this information helps..Thanks and regards.Dr.Honey Nandwani Arora.."
},
{
"id": 127320,
"tgt": "Should an orthopaedist be consulted for sciatic nerve pain?",
"src": "Patient: I was to!d I have degenerate disc in my lower back and I have sciatica with it .Its been about 6weeks with this and i t has been so painful and my nurse practioner wants to do physical therapy but they don t want to give me anything for pain. I m not sure about what to do but I have worked for 33yrs. And I woke up to this and I have been trying to walk and excersise to work it out and its not working.Should I see a orthopedic doctor? Doctor: Hello and Welcome to \u2018Ask A Doctor\u2019 service. I have reviewed your query and here is my advice. Yes, the nerve compression should be detected after an MRI scan. Proper medicines will help. I hope this information has been helpful for you. Regards, Dr. Praveen Tayal"
},
{
"id": 170301,
"tgt": "What causes wheezing and severe cold in a 2 year old?",
"src": "Patient: I have a kid 2 years old. He has severe cold for the past two weeks. This severe cold resulted in Veezing trouble. Doctor prescribed Levolin which we are regularly giving and my sons chest is clear now. On our observation, we feel that after giving the Levolin my son gets more angry and he is not comfortable. Does levolin has this side effect ? Doctor: Hi...sometimes very rarely Levolin can cause increased heart rate and anxiety. This can prevent as short lived aggressiveness. You need not worry about this. Sometimes it can even cause tremors of hands and feet. But these are not major or life threatening side effects. They will be resolved as soon as you stop the drug.Regards - Dr. Sumanth"
},
{
"id": 44842,
"tgt": "Can i be pregnant with one tube blocked ?",
"src": "Patient: Hi, I can t conceive because one of my tube is closed. Is there any way I can be pregnant? Doctor: Hi. Welcome to HealthcareMagic. Have you undergone a hysteroscopy already. There are modern techniques which can solve your problem. Please consult a gynecologist so that she can access your reproductive system and give you the best possible method. Take care"
},
{
"id": 169917,
"tgt": "What causes vomiting while on Bactrim?",
"src": "Patient: Hi my 9 yr. old daughter has been on bactrim for 3 days now for a bladder infection and strep throat, now before she was put on bactrim she had bugbites on her face, they started to clear up but now they look worse she also vomited twice today should I be worried? Doctor: Hello, yes Bactrim can cause vomiting. My advice would be to switch to amoxicillin which is a very good drug for both bladder infections and strep throat. Stop the bactrim, and give ondansetron for the vomiting. Nothing to worry, she will get better soon. Hope this helped you."
},
{
"id": 4628,
"tgt": "Have sore nipples, cramps, nausea and brown discharge for long. Negative Home pregnancy tests. What is wrong?",
"src": "Patient: I have been having worsening nausea for the past two weeks, nipples erect constantly, and cramps. I usually have a five day long period. This month I had brown discharge for one day and half. Three home pregnancy tests have been negative. What could this be? Doctor: Hi,Thanks for consulting us.This could be a sign of an ectopic pregnancy, which is a life threatening condition if not treated properly.You should have a blood pregnancy test called beta hCG & one trans-vaginal ultrasound to exclude the same. Wish you good health.Take care."
},
{
"id": 95098,
"tgt": "Mild hydroureteronephrosis, abdominal pain, vomiting, given buscopan, hemophilia patient",
"src": "Patient: My son scan report says that he has mild hydrouretonephrosis . what should I do next he has abdominal pain we have given buscopan for his pain and he has vomiting also. inspite of giving him domstal he vomits sometimes. very important that he is a hemophilia patient hemophilia A factor VIII deficiency 3%. Kindly tell me what to do. Doctor: hello first find out the reason wheather hydronephrosis is due to renal stone or other reason. if it is due to stone than treatment will depend upon location of stone and its size"
},
{
"id": 199880,
"tgt": "Suggest treatment for swelling and tenderness in testicles",
"src": "Patient: After having sex last night, i missed a hard thrust and strained my penis. This morning my penis is not saw but my balls are swallon and very black, it hurts to sit and the taint region between my balls and anus is extremely tender. What should i do? I feel i can manage the pain but having blackish blue balls is fearing me out a little! Doctor: Hi,From history it seems that there might be heavy injury on scrotum due to hard impact while sex.There might be having hematoma causing black appearance.There might be having possibility of injuring your testicle as well.Consult sexologist and get examined.Injury to testicle might give rise to big problem.Ok and take care."
},
{
"id": 140543,
"tgt": "Is episodes of light headiness of increasing intensity and frequency any serious indication ?",
"src": "Patient: periods of light headiness of gradually increasing intensity and frequency over 2 months growing from 1-2 seconds to1-2 min without loss of conciousnes but progressing deminsished conciousness,Strong normal cardio pulmonary health.No unusual activity or intake.Is it a serious indication? Doctor: Hi, There are a few possibilities. One would be hypoglycemia due to improper release of insulin and/or suboptimal eating habits (which could also include dehydration). Another could be headache aura perhaps most likely related to migrainous vertigo where the spells you refer could be part of the vertigo portion of the disorder. A 3rd possibility is that of orthostatic hypotension that may be occurring inappropriately or in response to overactivity/oversensitivity to medications used to control blood pressure, etc. The only way to know for sure would be to have a thorough examination by your primary doctor followed up by a specialist as indicated if the primary is unable to figure out the problem. Hope I have answered your query. Let me know if I can assist you further. Regards, Dr. Dariush Saghafi, Neurologist"
},
{
"id": 134271,
"tgt": "Suggest remedy for chronic pain caused by degenerative arthritis in neck",
"src": "Patient: Wondering why after open hysterectomy surgery I was given a machine to practice taking deep breaths but after laparoscopic gall bladders removal no doctors seem to be worried that I can t take amdeeombreath without severe pain. Like and 8 to 9 on theain scale...as a result I m breathing very shallowly and am worried about this. I m a 45 year old woman with chronic pain from degenerative arthritis in my neck, sacrum and hips, herniated disk in lower back and hashimotos disease controlled with thyroid meds but with a goiter... Doctor: hi,thank you for giving a brief history of you.as you mentioned you have an issue with the deep breathing and also you have Arthitis issue.I request you to try doing simple neck exercises initially. then proceed onto slow costal breathing exercises and pursed lip breathing exercises. later on proceed onto abdominal breathing. lastly try performing deep breathing as well.respiratory system is of mere importance and shouldn't be avoided in any case by anyone.also meeting a pulnonologist will be of utmost importance and help. he might ask for the MRI or CT of lungs which will help assess the status of the lung tissues.after the exercise of breathing, which if you don't find easy than meet up a physical therapist. for your pain in the arthritic regions a therapeutic ultrasound therapy and TENS therapy will be used which will help ease of pain.now since you have Arthitis of neck, lower back, sacrum and hips. you need to focus on doing regular slow free exercises for all of this joints.once the muscle starts adapting to movements and get strength , proceed onto the next level by core strengthening exercises.any form of exercise performed will give an add-on to your strength in muscle , balance the oxygen saturation levels in blood, improve the cardio respiratory endurance, improve the metabolism and importantly boost your immune system.balancing the metabolism in the body is important to control the fluctuations of the thyroid levels.for any further help always a physician and a physical therapist is a close by. a close visit and getting assessed again will help you get the maximum output.I have seen many cases of such, and out of that 90% have been helped with exercises. patients under me who have performed for 12-24 weeks have got a big change and then regular modification and assessment have helped to plan the next level.due to the shallow breathing you might consume more of your energy which will make you feel faigue and tired soon. so rehabilitation of the Pulmonary system stays at utmost importance.with the grace of God I wish you a blessed health"
},
{
"id": 12205,
"tgt": "Blue line in center of Intergluteal Cleft. What could this be ?",
"src": "Patient: I am a college aged female. Recently, I have noticed that I seem to have a blueish color in my midline of my intergluteal cleft or in slang terms, the crack of my butt when I spread them apart. I also sit on my tailbone/butt a lot, sometimes to the point that when I stand up my butt area feels numb just like when your arm or legs fall asleep. I also checked to make sure it was not lint. There is also no visible tear or pimple that I can see. I also took several hotter than normal showers last week. It seems odd to me, but I was not sure if there was any concern. What could this be? Doctor: hi it may be simple nevus that present from long time and you notice recently. it is very common in asian to have bluish brown patch over the gluteal area. it is not harmfull and may be vanish off gradually. ask your parents if it present since birth? have u any pain,itching,injury over that area?"
},
{
"id": 56592,
"tgt": "Suggest remedy to remove gallstone",
"src": "Patient: i was dignosed with gallstone . 20 mm and is easily seen in simple X-ray(KUK) and ultrasound.it is completely silent stone(its free floating). i rarely get a feeling of like some air trapped in that region(very rare).doc suggested to remove gallbladder.BUT MY ONLY WORRY IS THAT THE STONE IS 20 MM IN SIZE AND DOUBT THAT IT MAY GET STUCK IN THE DUCT, WHICH IS AGAIN A PROBLEM( I THINK A BIG PROBLEM). HAS ANY PASSED SUCH A BIG STONE WITH FLUSH AND CONFIRMED THE RESULT WITH ULTRASOUND? Doctor: Hi there,Thanks for using HCM.Bile flows through bile duct.The maximum diameter of bile duct is about 6 mm.So a gallstone of 20 mm can never get flushed and pass through the bile duct.It has to be removed only surgically.The gallbladder can be removed laparascopically.But we generally do not advise surgery for a silent gallstone.Only when it causes obstruction or inflammation causing a 'sore' gallbladder, we advise surgery.Is this answer helpful?"
},
{
"id": 21046,
"tgt": "What causes heart flutters?",
"src": "Patient: for the past while i have had heart flutters , unbelievably scary there not to intense but i get the feeling in the base of my neck , i have had EKGs and blood tests and they come back negative. Today i had them really really bad . The worst yet , what causes this? i literally get them out of no where Doctor: Hello SirI would recommend you to consult a cardiologist and get a 24 hrs holter monitoring and echocardiogram done. if still the reports are normal then should go for a tread mill test and an external loop recorder reading done. if still everything is normal then nothing to bother much..some medications will help"
},
{
"id": 139816,
"tgt": "What causes tingling sensation in head and blurred vision?",
"src": "Patient: Im 47yrs 4 the past 2 wks been feeling a tingling hevey sensation on the top part of my head my vision changes blurry and i have to bl;inck to be able to see i also alot of ansaity my heart beats fast the i cant breath i have to breath in to be abl to breath in Doctor: Hello, In some cases, these symptoms may persist after a viral infection for example. Changes in blood pressure, anemia, also may cause such symptoms. I think that you should see your primary care doctor for these issues. Hope I have answered your query. Let me know if I can assist you further. Take care Regards, Dr. Erion Spaho, Neurologist, Surgical"
},
{
"id": 208836,
"tgt": "What causes mood swings for every few days?",
"src": "Patient: My husband behaves quite abnormally every 15 days especially on full moon days and no moon days. For no reason he becomes furious and gets anger. He shouts for a long time for even silly things. This happens every fortnight. Can you please clarify the cause of this? Doctor: HIThanks got using healthcare magicI think, he has rapid bipolar disorder. In that, patients usually have frequent mood swing and that last for few days to many days. That could be the reason in your case. You need to consult a psychiatrist for proper mental evaluation as well as management. In case, you need further help, you can ask.Thanks"
},
{
"id": 156622,
"tgt": "Suggest treatment for Prostate cancer?",
"src": "Patient: I have early stage Prostate Cancer......I have early stage Prostate Cancer......I have early stage Prostate Cancer......I have early stage Prostate Cancer......I have early stage Prostate Cancer......I have early stage Prostate Cancer.................. Doctor: Treatment for early stage prostate cancer depends upon the age of patient, expected survival, and serum PSA levels(for risk stratification).Options are radical prostatectomy, radiation therapy or active surveillance.Please discuss with your oncologist."
},
{
"id": 16093,
"tgt": "Rash in between the thigh and genitals. Applied hydrocortisone cream, no improvement. Suggestions?",
"src": "Patient: Hi, a few days ago I got what appeared to be a flare up of athletes foot . Then my toes and top of my foot swelled a little. Cleaned and dried it thoroughly, used the normal AF cream. That seems to be getting better. No I have a rash on my upper quad that seems to swell a little too. Its in the area where my underwear line would be on my leg. Itchy. Tried hydrocort cream, doesnt seem to help. any ideas? Doctor: sounds like a fungal infection of the groin region. hydrocort might look as if it is helping initially, because it will suppress the symptoms, but the fungal infection will actually get worse in the meantime. try using the antifungal cream that you have used on the foot. something with clotrimazole or terbinafine would be perfect. use it twice daily for 2 weeks. if the itching is really severe, use the AF cream in combination with hydrocort. Once this has settled down, use a powder with 1% clotrimazole as a preventive measure for the next few weeks. especially if you are in humid weather or perspire a lot."
},
{
"id": 91777,
"tgt": "Can meftal be given instead of drotin to a child suffering from stomach pain?",
"src": "Patient: hi , my 7yr daughter was complaning abdomine pain from last 2 week .doctor suggested her drotin srp for pain but its not worked then i started meftal srp. is it safe for her and how long i can give it to her.her ultrasound reports are normal. pls.tell what are the other possiblites of this pain. . Doctor: Hi.7 years old child and has pain in abdomen for 2 weeks- The probable diagnosis is a lymph node mass / worms/ enteritis / typhoid to name a few. Since ultrasonography is normal , I suppose there are no lymph nodes.Other probable reason are worms- give a dose of dewormin.I hope she has reveived a course of an antibiotic and metronidazole.Go for stool test for three consecutive days - gives a great information sometimes.You can gice he meftal spas- for 5 days. But again to tress the importance of a proper diagnosis.If necessary you can go for another opinion or see a Pediatric Surgeon ."
},
{
"id": 136547,
"tgt": "What is the treatment for fractured shin bone?",
"src": "Patient: I may have cracked my shin bone because I was playing football and I went to kick the football really hard and some one put there foot out and I kicked their studds and there is now a constant pain through out the hour and it Hurts when i touch is but I can run on it Doctor: Hi,Thanks for your query.Signs and symptoms of a fracture are: intense pain and tenderness, swelling, deformity of the bone involved, limitation of movementNormally for soft tissue injuries with no bony involvement , the following is suggested: R.I.C.E where r : rest , needs to rest , (2) I: ice , (3) C:in the presence of swelling, a compression bandage can be helpful to reduce the swelling and pain (4) elevation of the limb is also usefulAnti- inflammatory pain medication can also be used , on a full stomach.I do hope that you have found something helpful and I will be glad to answer any further query.Take care"
},
{
"id": 191273,
"tgt": "What could cause sleeplessness after taking Lantus and Metformin to manage diabetes?",
"src": "Patient: Unchecked diabetes for several years. Dr prescribed 15mg Lantus (needle) & 2 tabs metformin daily. It s been close to 72 hours since I last slept. Can t continue and can t see the Dr. until 15 Jan. What s the best approach to troubleshoot these dosages?? Doctor: hi,I have gone through your question.Diabetes treatment needs lifestyle modifications like low carbohydrate diet , regular exercise,peroidical checkups ,taking medicines(anti diabetic drugs) as prescribed by your doctor ."
},
{
"id": 112120,
"tgt": "Experiencing pain at middle of upper back and dizziness, what is the problem ?",
"src": "Patient: Hi there,for the last couple of weeks ive been having back ache/pain in the middle of my upper back (between my shoulder blades) I went for a run and it was like I couldnt take a deep enough breathe in , i was really scared. Im 15 years old and I am female. I have a headache and woke up very dizzy and feeling quite sick but I haven't thrown up. Please help, Doctor: HIThank for asking to HCMI can understand your problem, you went for run and then you have this problem, it is common in female person having backache after the run because of the mammary glands, during the running time this moves randomly and tends to pull the back muscles unevenly, and this may cause great pain,, before running you should have a advise from trained person for proper garment, now have muscle relaxant \"Chlorzoxazone\" with \"Diclofenac\" will give good result have nice day."
},
{
"id": 42221,
"tgt": "What does this motility test signify:Immediate 50 %,Non motile 40 %,Motility Index 1.9 %,pus cells 6 to 7/hpf?",
"src": "Patient: Hi Dr. XXXXXX, I want to tell you that i got married since 2 years still i have not blessed with baby. just 2 days ago, i have done my check up and i found that My motility (a) Immediate is 50 %, Non motile 40 %, Motility Index 1.9 %, pus cells 6 to 7/hpf and autoaggliutination is present. so please assist me... thanks...Regards, Doctor: Hi,Thanks for writing to Healthcaremagic.After reading your report I conclude your sperm motility is little less and mild infection is there. Rest all parameters are normal.For infection I suggest to take ciprofloxacin for 5 days after consulting your doctor. For low motility I suggest you to start drugs like lyoc-q or paternia which contain antioxidants and zinc etc which have shown good results. It takes long time to show results. So minimum 3 months you have to take and get repeat test done. since you are near border line chances of good response is more.I would suggest that you rule out varicocele in you by a doctor. If it is there, get it operated. And do some life style modifications like -- Reduce weight, do exercise.- Stop smoking and alcohol. - Avoid caffeine.- Ware loose under garments.- Healthy diet with multivitamin and zinc.- Avoid trauma to testis.Hope I have been helpful.Regards Dr. Ashish Verma"
},
{
"id": 62700,
"tgt": "Suggest treatment for lump on the tongue",
"src": "Patient: i have a small bump/cyst on top of (right side not left side) TONGUE and my hiv+ doctor told me to go to the doctor i see when i need a cyst removed, like dr. gabay did when i had cyst on my buttocks. i am scared !! i am trying to quit smoking and i pray to god it is not cancer as i have been living with the hiv+ virus for 20+ years. i have gingivitis yet by the Grace of God my t-cells last time i check was 800+ and virus load undetectable. can you tell me who / what doctor is best doctor to treat/ REMOVE this cyst/bump from MY TONGUE? The doctor i went to in past does tummy tucks, and ear, nose and throat doctor and i want to be double sure i am going to right place? thanks Doctor: HI,Welcome with your query to HCM.Based on the facts of your query,you need to Consult Surgeon or ENT Surgeon or Onco-Surgeon if it turns out to be Cancer,ONLY after doing FNAC Biopsy with Histopath study.Treatment would be according to the cause of the lump-by Simple Excision of the Toungue Cyst / or Radical Excision of the Toungue lump if its Cancerous.Welcome for any further query in this regardWill appreciate writing your feedback review comments,to help the needy patients like you at HCM.Good Day!!Dr.Savaskar,Senior Surgical SpecialistM.S.Genl-CVTS"
},
{
"id": 210230,
"tgt": "What treatment should I go for depression, anxiety,sinus problem?",
"src": "Patient: I was just diagnosed with nasal turbinates but I want to know if that could be related to low estrogen levels. I keep having bloodwork done on hormone levels and they say I'm low on estrogen but normal. I have depression, anxiety, no energy, GI problems and sinus drainage issues. Doctor: Hello and welcome to Healthcare Magic. Thanks for your query.I understand that you are going through a difficult time due to your multiple physical as well as psychological problems. Regarding your question about your nasal / sinus problem, it is not related to your low estrogen levels.Regarding your other psychological problems like depression, anxiety, etc., you need a detailed evaluation to arrive at a definitive diagnosis and initiate appropriate treatment. So, I would advise you to consult a psychiatrist for a detailed evaluation and further treatment. Effective treatment is available in the form of medication or counselling. Wish you all the best.Regards,Dr. Jonas SundarakumarConsultant Psychiatrist"
},
{
"id": 75190,
"tgt": "Worried for warm sensation in chest and stomach while sleeping",
"src": "Patient: I get a warm sensation in my chest and stomach. Usually at night when I am sleeping. I have had heartburn and I know that is not what it is. It's a different kind of feeling. It doesnt hurt...it just feels weird and very uncomfortable. I do have high blood pressure and starting with pre-menapause. Should be worried? Could it be heart related? Doctor: Hi,You are having heart burn history. Hence you can have warm feeling and mild chest pain from acid reflux. For that you can take Pantoprazole tablet for week. Keep your head elevated while sleeping. Smoking avoided as leads lower LES tone. Excess fatty meals restricted and coffee, chocolate like food avoided. Before labelling it as heart burn from acid reflux, it is better to rule out cardiac cause by EKG. Consult physician for examination with keeping all this in mind.Hope I have answered your query. Let me know if I can assist you further. Regards,Dr. Parth Goswami"
},
{
"id": 5313,
"tgt": "Was ovulating. Took duphaston. Trying to conceive. Should I consume it for the next ten days or only until day 25 because of the late intake?",
"src": "Patient: I was ovulating on day 14-15 of my cycle on clomid. I started to take duphaston 10mg daily starting on day 22. Should I consume it for the next ten days or only until day 25 because of the late intake? I am trying to concieve and I am worried that late intake of duphaston will affect the period if the conception this month doesn't work. If the conception does work, should I consume until Day 32?Please advise. Thank you. Doctor: HelloDuphaston is dydrogesterone, a progesterone derivative that has the beneficial effect in implantation of the fertilised ovum in patients with luteal phase defect. It is advised to take the drug for ten days from day 15 to 25 of the cycle.If pregnancy occurs the ovary starts producing progesterone to maitain the pregnancy. Duphaston is not required further.If pregnancy does not occur duphaston does not interfere with menstruation."
},
{
"id": 176805,
"tgt": "What causes cold skin and vomiting in a child?",
"src": "Patient: My son is 9 yrs old. He threw up yesterday out of nowhere. Last night he jad clammy cold skin and stated he didnt feel well. I laid down with him until he fell asleep. This morning he back to normal. Now this afternoon the same thing with him not feeling well cold clammy skin. Doctor: HIWell come to HCMI really appreciate your concern, this could be viral infection and condition need to be kept under observation, for fever and further bout of vomiting, here hydration is matter, else this is nothing to worry, everything would be fine soon, give light diet, take care, have a nice day."
},
{
"id": 171105,
"tgt": "Suggest remedy for fever and loss of appetite in a 11 months old",
"src": "Patient: My baby has an axillary temperature of 37.2, no appetite and no interest in usual activities. He is 11 months and had been off form with sleeping pattern for the past while and i was putting it down to more teeth coming through. What should i do with him? Does he have an infection? Doctor: It could be infection or may be simple Pyrexia of undetermined origin.Till you decide further, pl start giving him water boiled with Holy basil leaves. As if normal water is given. If he can lick little honey mixed with pure turmeric, pl give him 8 hrly. Turmeric is tasteless, so no problem.These are safe, no side effects. Build resistance against infection. Harmless.In two or three days it will start showing results. You may continue for a week or so.Hope it helps."
},
{
"id": 160816,
"tgt": "Suggest treatment for headache",
"src": "Patient: my son aged 6 keep getting severe headache which cause him to get very upset. he then is very sick and will then sleep and be ok when he wakes. the headache always appears sighted over the eye. This also seem to happen when he is outside for a long period of time. can you offer any advise Doctor: Hi,I would like to clarify few more points: Is the head ache associated with nausea / vomiting / photophobia (difficulty looking bright light) / phonophobia (difficulty hearing noises) / any visual disturbances? Also how long he has this and any other members in the family having this?These points, if present, suggests migraine as the cause of headache, which can be treated with analgesics and life style modification, with occasional need of taking regular drugs for prophylaxis if quite often and severe. If these points are not there, it can be a tension headache. Kindly see your doctor for detailed evaluation. Meanwhile few tips for control:1) Give paracetamol for pain relief (I used to add ibuprofen if no response)2) Take a nap in a dark and quite room when having headache3) Keep regular meal times- avoid missing meaks4) Regular sleep is must- sleep deprivation can precipitate5) Avoid much tea / coffee / caffeinated drinks and chocolatesHope I have answered your question. Let me know if I can assist you further. Regards, Dr. Muhammed Aslam TK, Pediatrician"
},
{
"id": 7497,
"tgt": "Prescribed Azithromycin and Clindamycin for acne. Scars due to acne. Suggest treatment",
"src": "Patient: hello , i am 20 years old have problem of acne , i have consulted doctors but i am unable to complete whole course due to lack of time and not stable at one place.. i was given azithromycin tablets, Isotretinoin tablets and Clindamycin 1% gel to be applied on face..should i continue with this medicine or i should stop taking the same. i also have scars on face because of acne ..please suggest some treatment for both problems.please help me..ll be so thank full to you... Doctor: Hello, Acne scars are a result of acne itself so from preventing newer formation of scars we must stop formation of acne first for which you need to take isotretinoin caps for atleast 3 months continuously. Coming to the scarring you can go for either dermarollers or fractional co2 lasers treatment depends on what type of scars you have. take care"
},
{
"id": 28415,
"tgt": "What causes the high heart rate and anxiety?",
"src": "Patient: I took MDMA about 4 hours ago , only little bit , knowing this week I took about 2g of it ( Monday 1G ) Thursday (1G) only took a bit tonight about 4h ago but my heart is beating very fast ( I also have a diagnostic of anxiety ) is it me working myself up ? Doctor: Hi welcome to hcmI understand your query and concern.Your symptoms are suggestive of Sinus tachycardia that can be due to stress and anxiety.I advise you to get an ECG,2 D Echo,lipid profile,chest x ray,CBC to rule out any cardiac pathology.Tachycardia is when heart beats > 100/min (Normal range being 60-100).So your heart rate is in the higher note.It should be evaluated and treated immediately or else it will lead to Arrhythmias of heart.Drugs like Metoprolol will be helpful in your case.Depending on your blood workup you need to use lipid lowering drugs,anti hypertensives..Avoid stress and anxiety.Restrict fatty foods.Exercise regularly.Consult your cardiologist for expert management.Post your further queries if any,Thank you."
},
{
"id": 55701,
"tgt": "Could head ache, yellow color in eyes and fatigue be something related to gallbladder issue?",
"src": "Patient: my son had some bloodwork down in the past three weeks and everytime they are saying his bilibrium is high. He had an ultrasound yesterday and it turned out ok,they said,they will take more blood from him in Feb. I have noticed on and off his eyes were yellow and he has suffered from headaches and fatigue on and off,they are blaming the flu.I am concerned.My daughter had her gallbladder removed in March 2013 after battling that for 2 years and she was a big water drinker then and still on and off now,My neighbor had her gallbladder removed last night as an emergency and is an avid water drinker too,her fianc\u00e9 had his gallbladder removed in 2012 and also drinks a lot of water,am wondering if its the town water to blame or hepatitis or something like that? Doctor: Hi, dearI have gone through your question. I can understand your concern.Your son has high bilirubin level. His ultrasound is normal so no any gall bladder issue is there. he should go for complete liver function test including ALT, AST, ALP, GGT etc. It will give some idea. He may have some viral hepatitis or other cause. His headache, fatigue and yellow colored eyes all are related symptoms and due to same cause. Consult your doctor and take treatment accordingly. Nothing to worry about his gall bladder. it is normal.Hope I have answered your question, if you have any doubts then contact me at bit.ly/Drsanghvihardik, I will be happy to answer you.Thanks for using health care magic.Wish you a very good health."
},
{
"id": 102906,
"tgt": "Inhaled ammonia and bleach. Is it safe to go back to the room if no is coughing or having reactions?",
"src": "Patient: Hi a while ago like maybe five hours ago maybe more I mixed ammonia base cleaner an bleach, it was a complete accident and I may of made a gas in my room, but it only smelt like it in the room I isolated it and opened all my windows all the fans are on but the smell is still there no one is coughing or having reactions, so I'm wondering is it safe to go back into the room I flushed the mixture (it was in the toilet) fairly quickly twice just toke sure it was gone. Thank you. Doctor: in my opinion you should wait for some hours as there can be reaction even after 24 hours and if some ammonia isleft it can harm you for precaution take anti allergic tabs neomycin h ointment in nose and fliuds"
},
{
"id": 136234,
"tgt": "Suggest treatment for severe pain and swelling in shoulder",
"src": "Patient: My wife of 63 had rotatory cuff repair operation two years ago. Last week she woke up with intense pain in her shoulder and a pocket of what seemed like fluid in it on top of the shoulder. Not like a blister under the skin but a bit deeper. As it persisted the shoulder became more red and painful. Antibiotics don t help and a sonar showed fluid in the soft tissue above the joint but none in the joint or tendon. She has severe pain and fever an no one knows what to do. The shoulder is now severely inflamed and swollen and no pain killers help any more. I have resorted to administering Morfeen orally to help her cope with the pain and it helps, but the second course of antibiotics finishes tomorrow and has had no effect. She was also not bitten by something. Doctor: Hello, Thanks for writing to us, There is possibility of internal muscle injury around shoulder.Shoulder problem should be diagnosed with MRI and arthroscopic repair if required will give good results.Physiotherapy will help for early recovery. Physiotherapy like USG and TENS will help.Hope this answers your query. If you have additional questions or follow up queries then please do not hesitate in writing to us. I will be happy to answer your queries. Wishing you good health.Take care"
},
{
"id": 192901,
"tgt": "Suggest remedy for nil sperm count",
"src": "Patient: Hi, my partner and I have been trying for a baby now for the past 3 years. A few months ago he had some test done and they said that his sperm count was nil. He also had a scan to make sure that everything was clear and everything was normal. He is a body builder and has taken steroids in the past which is why his sperm count was nil, but the question im asking is will his sperm count return in time? Doctor: Hi, It can be due to various reasons like diabetes and imbalance in thyroid and testosterone. So based on the cause only treatment can be given. You can take blood test for glucose and hormone levels. Hope I have answered your query. Let me know if I can assist you further. Regards, Dr. S. R. Raveendran, Sexologist"
},
{
"id": 79579,
"tgt": "Suggest treatment for severe chest pain",
"src": "Patient: Hi I am a 48 year old female and I had a EKG done that came back abnormal I had been having some chest pains this is my findings sinus rhythm-normal P axis,V rate 50-99, early precordial r/s transition-QRS area positive in v2, borderline t abnormalities, inferior leads-t flat/neg,2 3 avf, rate78, Pr172, Qrsd81, Qt376, Qtc428, axis P40, Qrs15, T-11 please explain to me and am I going to Die can this be resolved help me. Doctor: Thanks for your question on Health Care Magic. I can understand your situation and problem. Your ecg clearly suggestive of inferior wall ischemia. There are three walls of heart which are supplied by three coronary arteries. Whenever there is obstruction in any of the coronary artery, blood supply to one particular heart wall is reduced and hence patient is having chest pain. So on the basis of your ecg findings, possibility of obstruction in the right coronary artery and hence inferior wall ischemia is more. So better to consult cardiologist and get done 2d echo and coronary Angiography for the diagnosis of this. No need to worry for death as timely treatment in coronary artery disease have excellent prognosis. So don't wait at home, consult cardiologist and discuss all these. You may need angioplasty and blood thinner medicines. Hope I have solved your query. Wish you good health. Thanks."
},
{
"id": 99836,
"tgt": "Are injections part of treatment for dust allergy?",
"src": "Patient: My friend was suffering from cold with slight fever of temp 100.. he is working in cement factory.. so due to dust he got cold.. he got running nose and body pain too.. he went to doctor..He gave tablets and prescribed two injections.. Whether two injections needed for a dust allergy? Doctor: HI, thanks for using healthcare magicThe injections may be for the allergy symptoms or pain medication since he was having pain.It would be best to speak to his doctor to find out what medication he received.I hope this helps"
},
{
"id": 93373,
"tgt": "Black veins on tongue. Normal? Have abdominal discomfort",
"src": "Patient: I have blackish raise veins under my tongue and to yhe side (but under). Is this normal?ive been rather anxious of late with stress and have had some abdominal discomfort (not much) which comes and goes and more around. I saw a Chinese doc today and she said its gas moving around. She saw my tongue and said I'm extremely stressed Doctor: Hi, Thanks for asking the query,The veins which you saw are normally present but become more visible when inflammed or congested nothing is abnormal or worrisome.Most of the time the infection sholud decrease symptomatically and resolve in ten days. If it persists beyond that or if there is worsening of the symptoms consult your Doctor for thorough evaluation and management.Meanwhile you can go for thrice daily warm saline gargles for faster recovery.A daily B complex multivitamin suplements would be beneficial as per Doctors precription only.Hope you find this answer as helpful.Take care!"
},
{
"id": 163949,
"tgt": "What causes recurrent vomiting and fever despite taking Motrin?",
"src": "Patient: My 5 year old daughter hasn t been feeling well since Monday. She threw up Monday morning, 4 times, had no other symptoms. An hour after throwing up, she had a fever of 103. I took her to the doctor and she was tested for strep. She tested negative for strep and was told it was a virus. She had a fever from Monday thru Wed. She was given Motrin every 6 hours to help with the fever. She went to school today, Thursday. Teacher told me she was very tired all day. She came home from school and complained she didn t feel well and threw up 4 times. The throw up is clear in color with little red specs. She doesn t have a fever after throwing up. What should my next step be? Doctor: Hi.... I understand your concern. By what you say I feel that your child is going through a viral illness too. Your doctor is probably right.I will explain you how the viral illness behaves so that you can be more confident.Fever of few days without any localizing signs could as well a viral illness. Usually rather than fever, what is more important is the activity of the child, in between 2 fever episodes on the same day. If the kid is active and playing around when there is no fever, it is probably viral illness and it doesn't require antibiotics at all. Once viral fever comes it will there for 4-7 days. So do not worry about duration if the kid is active.Regards - Dr. Sumanth"
},
{
"id": 102517,
"tgt": "What is the remedy for the headache, sneezing, fever, cough and nasal congestion, taking azithromycin have not helped?",
"src": "Patient: piercing headache, sneezing, high fever, cough, nasal congestion, back ache, can t sleep, chills, sweats, sore throat, can t breathe Have been exposed to subcontinental hospitals for the last one week caring for a family member. Started three day zithromycene antibiotic course. Day three now, still feel awful. Doctor: Hello, Welcome Thanks for consulting HCM, you said that you are being exposed to hospital infection from few days you should continue to azithromycin for last 3days continues this medicine for course for 5 days ok if still you wont get relief then go for blood test and consult physician."
},
{
"id": 45451,
"tgt": "Brown Colored Semen",
"src": "Patient: I am a male 22 year old, From few days i am in great trouble, because of my sperm of brown color, i am worried about this, is this normal? what is the cause and symptom for this..i dont have any pain in penis, i even had urine test ..reports were normal, just worried for brown colored sperm. Doctor: Hi.. Brown colored semer indicates presence of blood in the semen.. This can occur in few conditions of the prostate and in few conditons of the excreting system.. You can wait for a few days and most often they usually settle, if persisting a examination\u00a0 by a doctor..or getting a sperm analysis should help..."
},
{
"id": 85806,
"tgt": "Can Razo 20 and Kinpride be taken together?",
"src": "Patient: hi doctor, This is taniya and i would like to know whether Razo 20 and Kinpride tablets can be taken together before breakfast. I am suffering from abdominal pain from 1 month..I am taking Smarti for more than 3 year.. I have food allergy and took treatment for that also..i.e cetzine,clamist,etc Doctor: Hi, Kinpride with a composition (formula) of Cinitapride 1 mg and Razo 20 (Rabeprazole) do not share any significant drug interactions.They have not been reported and hence can be taken together. Hope I have answered your query. Let me know if I can assist you further. Regards, Dr. Ajeet Singh, General & Family Physician"
},
{
"id": 200608,
"tgt": "What does enlarged prostate with PSA being 91 indicate?",
"src": "Patient: Hi----My father in law has an enlarged prostate and the DR. said his PSA was 91. I actually thought he meant 9.1, so I doubled checked with him to make sure he said 91, and he did indead say 91. I ve never heard of a high number like this. Is there a different chart that they go by for this reading? My father in law is 82 and has no symptoms to be concerned about. Looking forward to your thoughts. Doctor: Hello dear,Thank you for your contact to health care magic.I read and understand your concern. I am Dr Arun Tank answering your concern.No, it can go up to this range. There are some other patients also who has range of above to 100 also.There are two possibly causes of it either it is benign prostatic hypertrophy or it is carcinoma of prostate.By looking at your value of PSA it looks like carcinoma. I advice you check for biopsy of the prostate.If its good than there is no need to worry. But if its carcinoma than planned operation with or without radio therapy and chemotherapy should be done.I will be happy to answer your further concern on bit.ly/DrArun.Thank you,Dr Arun TankInfectious diseases specialist,HCM."
},
{
"id": 139785,
"tgt": "Suggest treatment for Subacute Haemorrhagic Infract",
"src": "Patient: Hi My father is undergoing Dialysis and few weeks backs he was diagonized to have Jaundice. Suddenly he got Seizures as well. MRI Scan shows Late Subacute Haemorrhagic Infract in Right Cerebellar Hemisphere. He is on medicines for Jaundice and Brain problem but his Jaundice level is not getting reduced, we have been informed by the doctor that Jaundice level will decrease with dialysis. Whether Dialysis alone is sufficient or he has to take any other meidicines. He has been given Liv52 Tablets and Udiliv Tablets Doctor: Hello, He can proceed with dialysis. The head injury is not related to his other systemic illness. Consult a physician for expert evaluation. Hope I have answered your query. Let me know if I can assist you further. Take care Regards, Dr Shinas Hussain, General & Family Physician"
},
{
"id": 129613,
"tgt": "Can itching and stomach rashes be related to wound below right elbow with pus?",
"src": "Patient: hi, I got a wound on below my right elbow a week ago , I am clearing the wound daily and applying soframycin cream on it today I suddenly started getting itching and skin rashes on my hand an stomach area are they related to my wound, I can see some pus as well in my wound Doctor: Dear patient thank k you for that question.Itchy rashes in your hands and stomach are most commonly due to allergy to medication.If there isnt any other medication you are taking it might be allergy to soframycin as itching and rashes are a well known side effect.apply fusilin instead. If there is still a lot of irritation apply fucicort for not more than 2 weeksHope this helped"
},
{
"id": 200028,
"tgt": "Does masturbation cause any loss of strength and muscle?",
"src": "Patient: sir i am working on fitness bodybuilding n i workout 4 times a week and i dont masturbate i even tried to avoid it 45 days continuously n i did because i think if i masturbate i will lost my strength n muscle please guide me n brief me with satisfying detail? thank you Doctor: Thanks for asking in healthcaremagic forum Masturbation is harmless if done only when excited and within limits.So, it cannot cause any weakness or loss of strength as you think. You can enjoy yourself and no need to withhold it un-necessarily. All the best."
},
{
"id": 225878,
"tgt": "Condom broke during sex . Took two birth control pills twice. Period not started. Reason?",
"src": "Patient: I've been on birth control for over eight months now a d take them perfectly. Although,my boyfriend and I started having sex last week. Almost a week ago the condom broke and he came inside me. Me, being stupid, I grabbed two birth control pills and swallowed them to be sure. The next day I accidentally took two pills. I know taking more than one pill is dangerous but I wasn't thinking. Anyway, I am on my 6th day of placebo pills and still haven't started my period. Is this because of e extra pills? Or a I pregnant this early? Doctor: Hello dear,The emergency birth control pills are considered effective when taken within 72 hours of unprotected sexual intercourse.These pills have high hormonal content.And since you have taken 4 pills accidentally, it can cause hormonal imbalance in the body, resulting in delay of periods.However, to relieve your apprehension, you can get a Pregnancy test done using a Pregnancy kit.In case of pregnancy, it will give positive result within one week.The reasons for delay in periods and spotting can be many such as hormonal imbalance(as already explained), anovulatory cycles, stress, any infection or pathology in the uterus, ovary, etc.So, if pregnancy test is negative & periods are further delayed, kindly consult a Gynecologist & get a proper clinical examination done.Investigations like complete blood count, Ultrasound scan of abdomen and pelvis will be helpful to rule out any pathological cause for your symptoms.Wishing you a good health.Take care."
},
{
"id": 23969,
"tgt": "Is it to be concern about the high blood pressure?",
"src": "Patient: I am a 30 yr old female, 5'4\" 160lbs...I have PCOS and after metformin treatment I sucessfully conceived. I had preeclampsia while pregnant with my first child, who is almost 3 yrs old. My blood pressure reached 220/125 before I was induced. It returned to a more normal but still elevated 135/70 within a few months. Since January my blood pressure has gone back up to 150-160/95-105 and over the past weekend it reached 189/116 and would only go down to 150/98 when lying on my side. I have had a severe headache that began like a stiff neck on the left side over the fourth of July holiday and the pain continued for a few weeks on that side. I still have headache pressure in a band across my forehead and the back of my head. Last night my neck began to swell on the right side. I have an appointment in 3 weeks, but am wondering if this is something that I need to go to an urgent care center for now as I am fearful of a stroke with the high blood pressure. Doctor: usually blood pressure more than 180/110 is considered as hypertensive emergency and may comprise symptoms like headache,dizziness,blurring of vision,dyspnea and chesst pain .these hypertensive emergency would be treated by iv dugs iv sodium nitroprusside or iv labetalol.as i find that since january your blood pressure has been found elevated for more than 3 times(>140/90) which means you are suffering from essential hypertension and you need to take a daily dose of antihypertensive drug to control your blood preesure.choice of antihypertensive drugs varies according to other health conditions.if you are pregnant and hypertensive you can opt only few following drugs.amlodipinelabetalolmethyldopaif you are having diabetes you can opt for following drugs .captoprillosartanyou must tell your doctor other health related conditions.sometimes in young age group of 30 blood pressure may get elevated due to secondary cause which usually includesrenal artery stenosishormonal disorders use of contraceptives.you should rule out secondary causes after consulting your doctor.you must go for following lifestyle changes.avoid sedentary lifestyle.avoid eating junk and oily foods take diets full of fibres like vegetables and fruitsdo exercise daily atleast for 30 minutes.get your LDL level checked to find if you require drug to control cholesterol level or not. thakyou."
},
{
"id": 612,
"tgt": "Can I conceive if I am bleeding light post abortion?",
"src": "Patient: Hello doctor i want to ask one question, i am 24 years old i had abortion on april 26 2010 and abortion complete but there is bleeding still now not havey some littel and it has brown color. i want to konw that is that possible have baby futurely. Doctor: yes you can conceive after an abortion but make sure you wait for atleast 6 months before you try to conceive again. Nd start taking folic acid 3 months prior to your"
},
{
"id": 206659,
"tgt": "Suggest methods to handle mental illness with loss of memory",
"src": "Patient: Hi, I m 19 weeks pregnant and have a history of depression. My boyfriend is 20 years old and recently started acting strange he s been in the psych ward 3 times now in the past 2 months, he argues a lot about politics, has run red lights and said he owned the world, once he even ran around naked saying the cops were coming for him. He claims he cannot remember certain incidents. He s on zoloft and I can t remember what else. Well, my question is I ve had depression and I m almost 100 % sure this isn t what he has, please help. Doctor: Hi, thank you for using health care magic.I can understand about concern.The symptoms you have described are definitely not of depression.is he on any illicit substance like alcohol, cannabis , etc? if yes then these symptoms can be due to that substance. If not then these symptoms look like of mania , which is a part of bipolar disorder.In bipolar disorder there is two part one is depression and the other is mania in which there is excessive talking, grandiosity, decrease need for sleep etc.This is like a two opposite pole Depression v/s Mania. Many times symptoms switch between this poles , or there may be mix symptoms of both the poles. Zoloft will not be helpful if person is in mania pole.Help of psychiatrist will be needed for managing this disorder.Hope this will help you.Regards."
},
{
"id": 115124,
"tgt": "What causes drop in blood count after dialysis?",
"src": "Patient: My brother, LKK, 75 yr old, a Chinese ethnic, been hospitalized in Mount Elizabeth Hospital, Singapore after 1 week stay in ICU.His Dr has yet to find out why each time after a dialysis his blood count drops.Although a recent bone marrow test has shown a negative.He is a diabetic patient for over 10 ?year and recently has to be dialysis in SG and in home country-Jakarta and Semarang.He was transported to SG in SOS after found effected with Amoeba in Semarang.And it has been cured in MtE.Why his blood count is not stable after dialysis, how to prevent it, what kind of test and medicine he needs to have? Doctor: HiThanks for askingBased on your query my opinion is as follows1. Blood loss results in decrease in counts following dialysis.2. Other common reason being chronic renal failure which causes anemia due to decreased erythropoietin levels which is needed for erythropoiesis and RBC production.3. Moreover nutritional deficiency due to old age and disease causes deficiency of iron, vitamin B 12 and folic acid required for RBC production.4. Amebiasis is intestinal parasite causes bloody stools, blood loss and anemia. Since he had been infected, this attributes to already existing anemia due to renal failure and exacerbate it.5. Since his bone marrow is normal, not to worry.6. He needs to be given supplements for iron, vitamins, erythropoietin along with adequate nutrition for his age.Hope this helpsRegards"
},
{
"id": 81143,
"tgt": "Suggest medication for difficulty in breathing",
"src": "Patient: I can t breathe without being sick. Every time I take a breath my throat is really dry and it makes me gag and then I m sick. I ve been being sick now for the past three hours I m so tired I don t know what to do, literally every time I take a deep breath I m sick. Drinks don t help medicine doesn t help I can t eat anything as it come straight back up, i m literally scared to Breathe. I don t know why this is happening please help. I do have a cold right now but this has never happened before. Doctor: Thanks for your question on HCM.I can understand your problem and situation.In my opinion you are having viral bronchitis.Viral infection is the most common cause for cold. And this can also give rise to bronchitis (inflammation of bronchus). This can lead to bronchospasm. Bronchospasm will cause difficulty in inhaling air.So your breathing problem can be due to viral bronchitis.So consult pulmonologist and get done1. Chest x ray2. PFT (pulmonary function test).Chest x ray is needed to rule out lung infection.PFT is needed to diagnose bronchitis.You may need inhaled bronchodilators too.So better to first diagnose yourself and start appropriate treatment."
},
{
"id": 74953,
"tgt": "Suggest treatment for respiratory infection and back pain",
"src": "Patient: So heres my problem. I have had an upper respritory infection for several months (like more than 3) and wonder if it could be related to my back pain. I have this horrible pain in my back, mainly right under my shoulder blade which hurts far greater if I attempt to eat or drink anything. I could chew my food a million times and as soon as it starts going down my throat my entire back tenses up and I practically double over in pain. This has been happening for about a week just so you have a time frame. I currently take an antibiotic for the URI and also have an inhaler because it has been difficult to breath. I suppose my main question is if this could all be related to the URI or if I need to go ahead and go to like a hospitals prompt care to get xrays and a better diagnosis? Doctor: Hey,I would suggest to consult hospital soon.As you are having back pain and dysphagia.Go for X-Ray and Sos base CT scan."
},
{
"id": 170689,
"tgt": "Suggest remedy for urinary tract infection",
"src": "Patient: 6 months old female child with urinary tract infection, urine culture shows growth of Klebsella sensitive to Amikacin +++,ciprofloxacin+++,Chloramphenicol+++ andNitroforantib++. So which drug you prefer to prescripe for this child and what is the dose per kilogram of wt. Thank you. Talal Doctor: inj amikacin 5-7mg/kg , tab nitrofurantoin 10mg/kg are good drug for this age she will respond within 24 hrs"
},
{
"id": 48211,
"tgt": "How to treat hematuria?",
"src": "Patient: Hi,My name's Jordan. I recently went to the bathroom and had some bloody urine. I have this a couple of times this year and done urine tests and had cultures done and they come back clean. Could this be caused by a prolonged time of masterbation? Blood vessels popping..something of that nature. Doctor: Good Day and thank you for being with Healthcare Magic! Gross blood in the urine is always a red flag for us urologist. You will need to get a kidney bladder ultrasound or a CT scan to rule out Genito-urinary malignancies. I hope I have succeeded in providing the information you were looking for. Please feel free to write back to me for any further clarifications at: http://www.HealthcareMagic.com/doctors/dr-manuel-c-see-iv/66014 I would gladly help you. Best wishes"
},
{
"id": 102000,
"tgt": "How can i reduce my allergy?",
"src": "Patient: how to reduce my allergy. my nose blocks when i travel by bike, when i encounter dust, cold atmosphere, exposed to air conditioners. continuous nasal discharge which changes from white color to green, then cough,...i have seen many doctors nearby. antibiotics give only temporary relief. what is my problem actually? can it be cured? recently by doctors suggestion i have been using foracort 200mg as he said i have turned asthamatic. can u please suggest me something so that i can be relieved? thanks santhi Doctor: Hi Shanthi,With your history kindly look for any nasal polyp or deviated nasal septum. This may cause repeated infection and features that you are presenting.If sputm changes to yellow or green colour with cough then antibiotics with anti inflammatory drugs should be taken. For confirming asthma, get done AEC counts and spirometry once. Because even with chronic sinusitis and repeated throat infection the symptoms will be similar to asthma. Do regular steam inhalation and salt water gargle. These will help and give maximum comfort. Kindly get back with the investigations report for discussing on your management. Hope this helps you. Regards"
},
{
"id": 19890,
"tgt": "How to control high blood pressure?",
"src": "Patient: Hi, I am 31 years old, yesterday I took my blood pressure and the reading was 147/93. I have previously had high blood pressure but am not on any medication. How dangerous is a reading that high, and what are simple ways to help control high blood pressure. Doctor: Hi ThereAfter going through your query I understand your concern.I would like to tell you that high blood pressure in young age should be managed with lifestyle modification initially that includes -low dietary salt intake- stop junk food- stop smoking/alcohol ( if you do )- brisk walk of 40 min daily - decrease your weight if you are obese Make these changes in your routine for 3 month period. If even after this you BP stays more than 140/90 then you will require medicine to control your blood pressure.Hopefully this has answered your query.Kind Regards Dr Navneet Mahajan"
},
{
"id": 6791,
"tgt": "How to improve LH level since i want to get pregnant ?",
"src": "Patient: HI I am 30yrs old and am planning for my pregnancy . I have always been having irregular periods. My LH level seems to be very high (25). Could you suggest necessary steps I am required to take ? Doctor: Hello Welcome to Healthcare Magic forum. Along with the medications prescribed by your doctor I would suggest you to practice 30min of Yoga asanas which are specific to balance the hormone functions such as (Shashanka asana, Vipareetha kaarini, Baddhakonasana, Matsyasana, Upavistakonasana etc). Follow a good nutritious diet which include soya bean, tofu, tender coconut, whole cereals, egg white, fruits and nuts Dr.Sneha Sama Wellness Clinic"
},
{
"id": 152293,
"tgt": "Side effects of lexapro",
"src": "Patient: i have just taking lexapro and i have started to feel wierd and dizzy is that common Doctor: Welcome to Health Care Magic Your symptoms are common initially. But gradually the symptoms might resolve. Regards, Dr. Jagdsih"
},
{
"id": 192237,
"tgt": "What causes body tremor and how can it be treated?",
"src": "Patient: Dear Sir, I am 26 years old man, working as investment banker.Sir from last 10 years i am suffering form shivering (tremor) so many times i have consulted a specialist for the same problem. Apart from yoga they prescribed me some medicines, but nothing happened. Reagarding My family history i am the first such case. sir kindly advise me regarding the same nd if any information is required for further investigation for the same i will be highly obliged to provide that to you vi mail or through this forum. reagrds Itikhar 0000 Doctor: Hello,Without examining you it's very difficult for me to comment however, there are many reasons like stress induced, benign essential tremors or some other organic brain disease. I suggest you to visit your doctor for proper checkup neurologically and imaging of your brain if needed. Meanwhile, stop taking tea and alcohol if any and if you are taking any medicines for any other purpose do mention those in your consultation.Hope I have answered your question. Let me know if I can assist you further. Regards, Dr. Sameen Bin Naeem, General & Family Physician"
},
{
"id": 219813,
"tgt": "Suggest remedy for spotting during pregnancy",
"src": "Patient: Hello Doctor, I am 6 week pregnant, 7 week is going on, my last period date is 16th Jan, I took hucog 5000 IU on 19 Feb & 2nd on 1st March, my doctor suggest me to take 4 injection and out of that i took 2 injection till now and I am taking these medicines ( Duphaston 10 mg daily 1 , Emfolic-dha daily 1, Ecosprin 75mg daily 1 for 1 month) which is sujested by my doctor, and today I got spot bleading one time in the evening, my doctor is not available so could you please sugest me what should I do... NIkita Doctor: hii..if you r having spotting then better to stop ecosprin for 2-3 days and then u can restart..continue to take other medications as advised"
},
{
"id": 226206,
"tgt": "On implants and pills. Do they interact?",
"src": "Patient: Hi there, I recently went to see my local doctor, but its not the doctor I usually see, as he retired and she s taken over.. I m not good with new doctors. I have been on the implant for almost two years now and my periods are how to put this a pain! So she has now put me on the pill as well as my implant! I had concerns as I have heard it could counteract each other.. But she didn t really seem to be bothered.. Can you help me put my mind to rest please.. Doctor: Contraceptive implant is licensed for 3 yrs - have you started getting continuous bleeding / painful periods? Implant does not interact with hormonal pill which is normally given to stabilise the lining of womb so irregular bleeding with implanon stops. If bleeding still continues - other causes need ruling out - infection OR uterine pathology (USS for fibroids)"
},
{
"id": 59221,
"tgt": "Done blood AST, ALT. Taken wine, acetaminophen. Stopped. How long will be for normal?",
"src": "Patient: I am 55 years old female and recently had blood work done. My AST was 65 and ALT was 105. I had just come back from a vacation and was \"cocktailing\" every night. Wine only. I was also taking a sleep aid that unbeknownst to me contained acetaminophen. I would like to know how long it will take for my enzymes to return to normal after abstaining from all alcohol and no more acetaminophen. MMB Doctor: Hi and welcome to HCM. Considering your findings now it will be needed about 4-6 weeks till it normalizes. If elevation persist after this, you should do liver ultrasounds nad find some other causes of elevated liver enzymes. Avoid alcohol, hepatotoxic drugs and food and I am sure everything will be fine. WIsh you good health."
},
{
"id": 125770,
"tgt": "What causes sharp pain in the upper thigh?",
"src": "Patient: I am having an extremely, sharp pain in my upper thigh. It is very quick , and only seems to be helped by standing up and walking around. It s like a cramp but unlike anything I ve ever experienced. It runs up from my knee to my extreme upper thigh. Doctor: Hello, As a first line management, you can take analgesics like Paracetamol or Aceclofenac for pain relief. If symptoms persist it is better to consult an orthopedician to rule out neuropathic causes. Hope I have answered your query. Let me know if I can assist you further. Regards, Dr. Shinas Hussain, General & Family Physician"
},
{
"id": 125837,
"tgt": "Should an X-ray be taken for pain in the knuckles after an injury?",
"src": "Patient: Banged my hand against a piece of furniture on Christmas. Pinky and ring finger knuckles hurt, swollen. Wrapped it and the swelling has gone away. Full range of motion has returned, but still, I have pain and some tenderness in the knuckles. Should I expect any more progress, or do I absolutely need imaging? Doctor: Hello, It might be a contusion. Fracture is unlikely. As a first line management, you can apply ice packs for symptomatic relief. Generally, the symptoms will settle in a couple of days. If persists better to consult an orthopaedic and plan for an X-ray. Hope I have answered your query. Let me know if I can assist you further. Take care Regards, Dr Shinas Hussain, General & Family Physician"
},
{
"id": 32463,
"tgt": "What are the symptoms of mrsa?",
"src": "Patient: A couple of years ago, I had what I thought was an ingrown hair around my bikini line. Except after pulling the hair out, it got worse. Over the next 36 hours, it went from being a pin-head sized sore to a golf-ball sized thing on the inside of my leg filled with green and white pus. Could this have been staph, specifically MRSA? Doctor: Hi, dearI have gone through your question. I can understand your concern. You may have ingrown hair. It leads to abscess formation. Or you may have some other lesion with acute inflammation. You should go for fine needle aspiration cytology and culture and sensitivity test of that pus fluid. It will give you exact diagnosis whether it is MRSA or other infection. Then you should take treatment accordingly. Hope I have answered your question, if you have doubt then I will be happy to answer. Thanks for using health care magic. Wish you a very good health."
},
{
"id": 17458,
"tgt": "What precautions have to be taken for hypertension in an elderly person?",
"src": "Patient: my father is hypertensive, he had a fall which resulted in profuse bleeding, his L.W. scalp was stitched and the wound dressed, his B.P AT THE TIME OF AD WAS 200/120, REST OF THE VITAL SIGNS WERE NORMAL. HE WAS discharged after obervation in the evening, he is on Lipril 5, Cefoclav 200,emanzen d, doxopan, his B.p, stabilized and it is 140/90,,,,,,,what precautions shd one take....he is 77 yrs old Doctor: Hello, I passed carefully through your question and would explain that it is important to closely monitor his blood pressure values frequently during the next days. I agree with the doctor's therapy. But, if blood pressure values remain above the normal ranges (125/85mm Hg), it would be necessary for increasing the dose of lipril to 10mg daily. Bleeding is related to high blood pressure. Reducing salt and caffeine intake will help maintain his blood pressure values under control. Hope I have answered your query. Let me know if I can assist you further. Regards, Dr. Ilir Sharka, Cardiologist"
},
{
"id": 154663,
"tgt": "How can vomiting caused by caxechia be reduced?",
"src": "Patient: 48 yeares old, 170cm, ~42kg right now!! Extremely thin, caxechia we've been told. The stomach cancer was treated through surgery, chemo, radiation than chemo again. We need to get help with what to give him to stop the vomiting or be able to eat at least a bit. Doctor: Hi,Thanks for writing in.Cancer associated vomiting can occur as an acute event or take a chronic course. It looks like your relative is having a chronic vomiting and cachexia course that needs attention. Many patients have vomiting due to the thoughts of previous chemotherapy and radiation treatments. The initiation of controlling the vomiting can be made by giving anti emetic as recommended by his doctors.Once the initial vomiting is stopped then he can be provided support and help in ways including Nutrition changes, Acupuncture and acupressure, Behavior therapy and Relaxation methods. Since a patient of cancer who is taking treatment is under tremendous stress and uncertainty about the curse of his illness and when he will get cured, he needs help to cope up with this situation. It is more of a psychological element that can be modified to control his stress. Please do not worry."
},
{
"id": 178490,
"tgt": "How to cure bubble like blister in hand?",
"src": "Patient: Hi my 4 yr old daughter had a blister on her hand it looked like a bubble and had some kind of liquid in it , i took her to the doctor he said it was some kind of insect bite so he poped the blister, drained the liquid it was clear liquid and told me to just clean it with peroxide daily and use neosporin as well and the next day it came back like the blister reappeared with liquid again then the day after it went down but then turned dark purple thats how it looks now dry, heard and dark purple should i be worried? Im still cleaning it daily Doctor: Hello. I just read through your question.I agree with your pediatricians approach. What you have observed afterwards is not unusual. It's the natural progression. I recommend continuing to keep it clean and applying neosporin. It will take some time, but it will heal."
},
{
"id": 55783,
"tgt": "What does pain in lower quandrant post gallbladder surgery suggest?",
"src": "Patient: I had my gallbladder take out 1 week ago by key hole surgery, after 3 days I developed pain in my right lower quadrant. Went back to hospital and blood test, X Rays and CT Scans were done finding no lung complications, sent back home 2 days ago and I still have the pain Doctor: There may be muscle spasm ,dont worry it will settle down very soon but you can consider to take some muscle relaxant likb tab hifenac-MR twice a day after food for couple of days .And dont do any straining things like lifting heavy objects ,gyming etc atleast for few days till it is settled.thank you"
},
{
"id": 163297,
"tgt": "Can Meftal be given instead of Crocin for fever?",
"src": "Patient: Hi.. My son is 16 months old. He has fever since morning and we have given him Crocin at 6 hours interval. He has been putting fingers in mouth, seems there is pain due to teething. He is not able to sleep properly. Can we give him meftal p, it has been 4.5 hours since we have given him Crocin as Crocin seems ineffective to subside the pain? Please suggest. Thanks Doctor: Hello,In my opinion, you should continue with Crocin (or any other paracetamol syrup) because Meflan drug is not indicated for children especially for young children. Hope you find my answer helpful.Hope I have answered your query. Let me know if I can assist you further.Regards,Dr. Elona Dashi"
},
{
"id": 193127,
"tgt": "Is it safe to have sex during diagnosis for TB?",
"src": "Patient: sir, i am a patient of tuberculosis. my parents pland my marrige in this march but now the delayed coz some people thought sex is not good for me in Tb. i have complete the period one month using medicine. my T.B is not relate to lungs. please suggest me can i sexual relation during T.B.or should wait till complete treatment. Doctor: Hi, You can have sex, after completing your treatment for 6 months.Hope I have answered your question. Let me know if I can assist you further. Regards, Dr. S. R. Raveendran, Sexologist"
},
{
"id": 130764,
"tgt": "How to treat cyst on fibula and swelling around talus?",
"src": "Patient: Dear Doctor, I have a cyst on the right side of my fibula and my talus area is swollen.. the cyst (also swollen) is painless, however when I squeezed it , there is a lot of clear liquid oozing out of it. I will feel very painful (the talus area) when I walk. What should I do to remove this problem? Doctor: Hello and welcome to \u2018Ask A Doctor\u2019 service. I have reviewed your query and here is my advice.If the cyst is oozing you need to get that cultured and evacuated completely before it gets infected and spreads into your joint. You also should clean it daily and keep it covered so bacteria from outside does not contaminate it. See your primary doctor certainly for further recommendations and possibly antibiotics.Hope I have answered your query. Let me know if I can assist you further.Regards, Dr. Samuel Parmar"
},
{
"id": 105368,
"tgt": "Organic hair dye, allergic reaction, swollen lymph nodes. Taking benadryl. Treatment?",
"src": "Patient: I had an allergic reaction to organic hair dye. My lymph nodes had swollen up at the base of my skull . I was able to make the swelling go down but there is an itching pain keeping me from sleeping. I am taking Benadryl to help with the reaction but what can I do to get rid of the itching pain and actually get some sleep? Doctor: take doxycyclyne 100 mgm bd for 3 week tab montair fx twice a day tab flunarazine 5 mgm at night do fo 3 week stop using organic dyes for preventing further allergies"
},
{
"id": 178368,
"tgt": "What causes diarrhea in a toddler?",
"src": "Patient: Hi Doctor, my kid is 8 months old and she s passing watery stool from last 2 days along with urine after every half hour, until she is slept in night. Also, she is having the urine and stool area got affected with rashes and is very red. When she passes urine or stool, she screams out loud. Please help! Doctor: hello, thanks for your query, pain abdomen( screaming) loose stools suggests acute gastroenteritis. several episodes requires hospitalization and iv antibiotics. kindly visit a pediatrician. hope I have answered your query. all the best. take care."
},
{
"id": 32707,
"tgt": "What causes fever, headache and constipation?",
"src": "Patient: Hi, I have fever, headache and feel constipated for the past two days.. I usually pop in a saridon and the headache subsides but this time it has not.. Keeps coming back.. And also to keep the temperatures down I m taking crocin.. Please let me know if need to consult a doctor? Doctor: Hello Welcome to Health Care Magic.headache can be due to high blood pressure or can be due to infection. fever suggests more chances of infection. constipation can be relieved by high fiber diet and lukewarm water. It will be better if you consult the doctor for taking antibiotics. Hope this information helps you.Best wishes."
},
{
"id": 12062,
"tgt": "My daughter has Pimples, Could you please advice",
"src": "Patient: My daughter is 19 years old, and she has very sensitive skin. She has pimples on face, back and chest from the age of 12, and we have tried all sorts of medicines, but of no use. Could you please advice. Doctor: Hi!madhura.jois, Welcome To HealthcareMagic forum, Pimples are multi factorial, with reasons ranging from wrong selection of creams and soaps to internal hormonal disturbances(esp in females).She should avoid all oily stuff on her face right from oily soaps to moisturizing creams. If your daughter has any problem in her periods or her acne aggravates 2 days prior to her periods ,then you should get a hormonal profile done with a ultrasound of lower abdomen. If some thing comes positive then she needs a hormonal treatment along with a anti acne treatment This would mean that her acne is because of her raised hormones. If not then you have two options \u2013either oral retinoids(for sever acne) or oral antibiotics depending upon the severity. They need to be taken for long .Also use a good anti acne soap daily. Its mandatory to use anti acne creams like nadifloxacin or clindamycin cream.Oral Retinoid is a wonder drug for acne but before taking one should discuss all the precautions and side effects of it.(esp in females). All these require a medical prescription and you should show to a dermatologist for this.Drink plenty of water and take a god 7 hr sleep in night .Never squeeze your pimples or else they would leave marks. Acne is 100% curable if you take a good line of treatment. Regards, Dr.Chawda"
},
{
"id": 31292,
"tgt": "What is the treatment for herpes?",
"src": "Patient: I was diagnosed about 30 years ago with Herpes, the sore appears on my lower back. I dont have regular bouts but maybe approx 6 years or so between an outbreak. Would this be only contagious whilst in blister, I have always been very careful with partners & usually know well in advance before the blister appears. Doctor: Antiviral drugs such as acyclovir un low doses of 200 mg regularly can be indicated for recurrent herpes infection."
},
{
"id": 141395,
"tgt": "What are the early signs of Parkinson s disease?",
"src": "Patient: I have severe boasts of nausea and diarrhea with bloating and pain. I m unable to eat and walking is a problem-my legs just won t work. These symptoms last for several days and then go away. However, my legs are still very weak and I am unable to walk up even small steps. I am a type 1 diabete for 40 years but my A1C is consistently 5.1. I m thinking that I might have Parkinson s? E-mail is YYYY@YYYY Doctor: Hello and Welcome to \u2018Ask A Doctor\u2019 service. I have reviewed your query and here is my advice. It is unlikely be Parkinson\u2019s. Parkinson\u2019s disease manifest as tremors and sometimes dementia(memory loss) and it is quite common in elderly with a family history of same illness. In your case it seems more like neuropathy associated weakness. Consult a neurologist and get evaluated. Hope I have answered your query. Let me know if I can assist you further."
},
{
"id": 74770,
"tgt": "What causes cough and accumulation of fat around the neck?",
"src": "Patient: My husband has been ill since Christmas when he started with a cold. He still has a cough, but is so achey that I have to put his socks on. He has accumulated alot of fatty tissue around his neck, has very little range of motion and is in pain. His doctor sent him for a chest x-ray and his nurse called to say he did not have pneumonia. He is 64, retired and has always been healthy. He fishes and I worry about lymes - any thoughts? Doctor: Respected user, hi I evaluated your query thoroughly.* Simply persistent cough & fat around neck is not indicative of Lyme .* X-ray is not showing pneumonia is fine but what about the diagnosis of the patient .* Many conditions may be present which require precise evaluations as Left ventricular failure - Bronchitis - Thyroid disorders - many more . * Your husband requires - hospitalisation - detailed work up including Laboratory data , 2 D echo & further reports as per clinical judgments .Hope this clears your doubts.Welcome for any further assistance.Thanks for using Healthcaremagic.com & giving me an opportunity to assist you.Wishing him fine recovery from the same.Regards dear take care."
},
{
"id": 26960,
"tgt": "What could cause left sided chest pain?",
"src": "Patient: Ive been having pain on my left side under my chest.also on my left back side sometimes the same time as front pain. I took a drink of milk last night and the pain happened again and it still hurts on front and back side . What do u suggest it might be? Doctor: Hello I have gone through your question and appreciate Your concern. U r having left sided pain. U better consult physician and get Ecg done.If cardiac problem is responsible for your pain then u haveTo take medicines. Gastritis and Gerd can sometimes cause chest pain.This should be ruled out.Thanks U can write me back for any query"
},
{
"id": 85767,
"tgt": "Can Deriphylline 2ml injection be administered every time during acute exacerbation?",
"src": "Patient: hi doctor my mther is 45 years soffering from asthma n copd from last 5 years...she right now has severe wheezing and fever...we have given her injection deriphylline2ml im , i want to knw can we administer it everytime she has an acute exacerberation??? Doctor: Hello, Well, it can overdose. It is generally recommended to be given no more than 4 times a day total dose of 300 mg per 24 hrs.; and 2 ml is on the large size of dosing. Funny story: my mother had an overdose of it when she 1) started taking her recommended dose 2) took erythromycini which causes it to accumulate 3) stopped cigarettes which lower its metabolism when you stop 4) had the flu which lowers the metabolism of it but 5) won a casino jackpot and hyperventilated which triggered the seizure. So, a lot of factors can influence this drug. Hope I have answered your query. Let me know if I can assist you further. Take care Regards, Dr Matt Wachsman, Addiction Medicine Specialist"
},
{
"id": 154395,
"tgt": "Can carcinoid stage 4 cancer cause a lump in the crown of head?",
"src": "Patient: My husband was diagnosed 3 1/2 years ago with carcinoid stage 4 cancer. He is doing amazing...still working. Gets a shot once a month BUT I have always cut his hair and this year I feel a large lump in the crown of his head. You can't really see it but it is definitelly different than it ever was. He knows his cancer is not curable and will not go in for test. I haven't mentioned this shape change to him but I do wonder about it. He turns 68 tomorrow and has a wonderful attitude about life. Thanks for listening. Any thoughts? Doctor: Hi, dearI have gone through your question. I can understand your concern. It may be carcinoid tumour or may be somr other pathology. He should go for biopsy of that lump. It will give you exact diagnosis. Then he should take treatment accordingly. Hope I have answered your question, if you have doubt then I will be happy to answer. Thanks for using health care magic. Wish you a very good health."
},
{
"id": 86710,
"tgt": "What causes abdominal pain when suffering from nonalcoholic steatohepatitis?",
"src": "Patient: I have nonalcoholic steatohepatitis. and the past year I have gotten worse I have constant pain in my upper abdomen and lots of pain in my right shoulder blade that is so bad that some days I cant take a deep breat without screaming in pain, I have also lost around 130 pounds this year from being nausea and from vomiting. Im always tired, should I be checked for liver cancer. no drs in my area want to listen to me about these symptoms. I do have diabetes,high blood pressure,high cholesterol. my age is 35 and my weight is 265 Doctor: Hi.Thanks for your query and an elucidate history.You have got a complex of symptoms which are related to each other.Non-Alcoholic Steatosis, constant pain in the upper abdomen and the right shoulder, making it difficult to breath, loosing weight due to nausea and vomiting, fear of liver cancer- known diabetes, high blood pressure, high cholesterol at the young age of 35 and at the weight of 265 lbs.The most probable cause of your pain in the upper abdomen and the right shoulder can be related to the problem in the lung or the pleura as it is making you difficult to breath. I would advise you the following;CT scan of the chest.CT scan of the abdomen related tests of blood, urine and stool. This will give the final diagnosis and a plan for further treatment ."
},
{
"id": 39923,
"tgt": "Do antibiotics needed for long lasting cough?",
"src": "Patient: MY DAUGHTER HAVING COUGH FOR 3 WEEKS NOW,THE FIRST WEEK I BROUGHT HER IN AN DOCTOR AND THE DOCTOR GAVE HER ANTIBIOTIC.SHE FINISHED TAKING THE ANTIBIOTIC FOR 1 WEEK.BUT AFTER THAT SHES STILL HAVING COUGH BUT MAYBE IN ONE DAY SHE WILL JUST HAVING COUGH MAYBE 5 TIME Doctor: Hi,welcome to healthcare magic forum.Usually,we do a sputum AFB to rule out tuberculosis if cough persist more than 2-3 weeks.But,since she responded to treatment,and cough has come down,we can try a cough suppressant like dextromethorphan,and antihistaminics for 5-7 days(I guess she has no fever/yellow sputum).It may help.If cough still persist,you please see your doctor and do sputum examination.Mean while,avoid dusty/cold environment.I hope you will find this information useful.Thank you."
},
{
"id": 24705,
"tgt": "Are faster heart beat and increased pulse a cause for concern after cardiac ablations?",
"src": "Patient: I'm 28 and have 2 cardiac ablations due to a diagnosed SVT in childhood. Now I have been having a very fast heart beat that hurts and my blood pressure is 133/89 and my resting pulse is 106. The last ablation I had was 10 years ago.... Should I be worried? Doctor: Hello there I understand your concernI feel drub therapy like verapamil can be initiated on you and only if that fails should you go ahead for ablationHope that helps"
},
{
"id": 77471,
"tgt": "Suggest treatment for wheezing and cough",
"src": "Patient: Hi I have had a chesty cough for 5 days now, coughing up green phlegm, it is however much looser, I was weezy but last night and today the weezing has eased. However I seem to be very hoarse and losing my voice, my throat was sore 4 days ago and had a lot of nasal congestion. This has all gone, but losing my voice. Have you any advice as I am a teacher and rather need to use my voice, Thanks Andrea Doctor: Thanks for your question on Health Care Magic. I can understand your concern. In my opinion, you are mostly having post infectious bronchitis. It is common after viral upper respiratory tract infection (URTI). You will improve with with following steps. 1. Avoid oily and spicy food. Avoid junk food. 2. Do warm water gargles 5-6 times a day. 3. Combination of antihistamine and anti inflammatory drugs is also beneficial. 4. Inhaled bronchodilators and inhaled corticosteroid (ICS) are also needed for wheezing. Don't worry, you will be alright. Consult your doctor and discuss all these. Hope I have solved your query. I will be happy to help you further. Wish you good health. Thanks."
},
{
"id": 164917,
"tgt": "Suggest treatment for abdominal pain and blood in stools of a child",
"src": "Patient: My son 4 1/2 had prurititus ani. We treated it but since then he is scared to go to the toilet. Says his testicles are sore when he goes and he has blood in his stool - bright red on toilet paper. he also moans sometimes he has pain in his lower abdomen Doctor: Hi, thank you for the question. your child looks constipated. He is having blood in stools and pruritis ani because of the hard stools which while coming out causes small tear in the sensitive anal skin hence the child is scared to go to the toilet. usually when i see such cases i put a bisacodyl suppository and advice them laxatives for a week or two and advice them regarding diet. Thank you"
},
{
"id": 206325,
"tgt": "How can panic attacks, OCD, bronchitis and loss of confidence be treated?",
"src": "Patient: Hello, I have been having problems with my ocd since childhood but it was diagnosed recently and I have been suggested to either go for medication or to go for cognitive behavior therapy. I recently lost my job and had bronchitis which led to a sudden panic attack. Since then I am on franxit/frestina and nebicard 2.5. The frestina doesn't seem to help a lot as I still get panic attacks sometimes. I try to relax and calm myself down. I am afraid of stepping out as I feel I might get an attack and I am scared. Is there any way where my situation could be helped as I am losing confidence slowly. Thanks Doctor: DearWe understand your concernsI went through your details. I suggest you not to worry much. You do not have to loose your confidence. Mental disorders take time to cure. The best thing you can do is to keep yourself calm and work with your psychiatrist and therapist. The treatment method you are prescribed is usually suitable for OCD. You should talk to your therapist about yoga and meditaton in addition to CBT. Exercise, Life style changes, change in thinking pattern, relaxation etc are as essential as medicines. Psychotherapy can help you changing your lifestyle and thinking patterns. Yoga and meditation help you to streamline your metabolism and neurological balance. Please consult your psychologist for further information.Psychotherapy techniques should suit your requirement. If you require more of my help in this aspect, Please post a direct question to me in this URL. http://goo.gl/aYW2pR. Make sure that you include every minute details possible. I shall prescribe the needed psychotherapy techniques.Hope this answers your query. Available for further clarifications.Good luck."
},
{
"id": 203265,
"tgt": "Is it normal to have smear semen when aroused?",
"src": "Patient: hi pls i want to know if its is normal or abnormal to have smear semen from my penis when i am aroused but without intercourse. it happens to me most of the times that when i think of having sex and could not, after shrinking in of my penis, semen is smeared. pls advice me if it is normal or not Doctor: HelloThanks for your query,based on the facts that you have posted it appears that you are getting watery discharge from penis on arousal.Watery discharge that you get on arousal is a secretion of the Bulbourethral glans located in urethra which get stimulated on arousal and secrete a mucus secretion which acts as a lubricant to help in passage of a thick semen during ejaculation.Do not worry this is a normal phenomenon Dr.Patil."
},
{
"id": 9102,
"tgt": "Can Favors of the face be removed by laser ?",
"src": "Patient: hi..female 22 years old...and I want to remove favors from face without surgery ...is these can be done by laser or there is any way to be removed without surgery ? Doctor: hi it is possible to remove it without surgery. laser r there to remove hair permanent u need to take average 8-10 session of laser result depend on u r skin type n machine"
},
{
"id": 45109,
"tgt": "Am i taking the proper treatment for infertility problem with thyroid ?",
"src": "Patient: Hi, m Neha, 28yrs, 5.1 , 62 kgs, n my query is about my pregnancy . M married now for 2yrs, n m trying to conceive for last 1 yr, after my follicle report my doctor said its POD form of ovary , so she suggested me to take Letroz 2.5mg* 2 (3rd-9th day of period, daily), n then for the follicle test, after this my report was normal, after rupturation i took Duphouston 10mg, but then also i coundt conceive,(this was repeated for 5 cycles), then my husband had the test( sperm analysis) n v found tat it was not normal, so doc suggested for Maxoza 5gms*2 times daily, n he continued it for 3 months, it worked in between but now the results r not at all good. Now the doc has changed the medicine to Siphene 25mg*1 daily for 3 weeks n take 1 week gap n again repeat the same. So is it safe to take it? N if yes is there any side effects of it? For how many days should it be consumed? Now m taking Ovanac 600mg*3times (daily) n B-long f*1daily, m also thyroid patient for last 2 1/2yrs,now my thyroid is 11, taking Thyrox 100mg, for 1 month, otherwise 25mg daily. Is our procedure going correct? Doctor: hi Neha, welcome to HCM. read your prob, you have been suffering from PCOD n hypothyroidism, both diease inhibit ovulation , n most common cause of primary infertility, so you can continue the advised treatment , it has no major side effect, but there is drug interaction between that drugs which you taken so you should adjust dose as per advise your doc. as you mention your hubby s report is not normal so get his testosteron leval n start proper treatment. Dr aseem 9982583020 aseemadhuri@gmail.com"
},
{
"id": 196544,
"tgt": "Should a painful head knot caused by trauma need checking?",
"src": "Patient: Hello....I am 42yo, white male, good shape, 6ft tall, 195....I was hit in the head with an errant baseball last Thursday. No real issues afterward except for a headache. However the knot on the side of my head still hurts quite a bit. No blurry vision, no dizziness. Any cause for concern? Doctor: Good day and thank you for being with health care magic!!! For any head trauma, we suggest getting a plain CT scan of the head just to be sure there is no blood formation inside the skull which in turn may cause compression of the brain. There are cases where in patients are asymptomatic for two to three weeks only to find out that the blood inside has been accumulating until the brain reached its threshold and there was sudden loss of consciousness. I would suggest getting a plain CT scan to be safe and sure."
},
{
"id": 136258,
"tgt": "What could freckle type spots on feet suggest?",
"src": "Patient: Hello - I have brown freckle type spots on top of my feel and going up the back of my calf. I have noticed it for almost a year now. It doesn t itch at all, and there is no feeling tot hem. when i push on it, it goes away. I have an appointment with a derm in a week but was curious if you could provide insight. Thank you! Doctor: Hello,The lesions sound like small vascular lesions called spider naevi. These are usually due to venous drainage problems and may be an early sign of varicose veins.All the best."
},
{
"id": 115337,
"tgt": "What causes dizzy spells and tingling in my tongue?",
"src": "Patient: When I work during the day (enter orders sitting all day), occasionally I get dizzy. I can be faxing orders and feel like the need to hold onto the fax machine. Also I don t know if I have swollen tonsils or glands because I sometimes feel like I will chock over my food if I don t stop and take a deep breath. And my tongue at times feels tingly. I brush my teeth regularily and I am about 15-20 lbs overweight. Doctor: Hello and welcome to HCM,You need to get a clinical assessment of the throat to look for enlarged tonsils.Enlarged tonsils can get inflamed and can obstruct the airways and can cause difficulty while swallowing.You are also experiencing similar symptoms so clinical assessment is required.Feeling dizzy and tingling sensation in the tongue are neurological symptoms.You need to get a complete work up because low hemoglobin or anemia can cause weakness, dizziness, etc.Megaloblastic anemia is a type of anemia wherein neurological symptoms predominate.Thus, investigations to rule out anemia are required and further management can be done after result of the investigations are obtained.Thanks and take careDr Shailja Puri"
},
{
"id": 214628,
"tgt": "Suggest natural remedies for Icthyosis",
"src": "Patient: A friend of mine in Thailand has a son with what I strongly suspect is Icthyosis. My friend says he heard of an herbal remedy produced by a company in Connecticut. I have not been able to find anything on this remedy on the net. Do you know of anything that can help. The boy is about 20 and is beginning to have social problems due to the disease. Doctor: I dont aware about the medicine that u mentioned.icthyosis diagnosed as genetic origin little time to take treat.Depending upon the severity and type of ichthyosis, affected individuals may need treatment from about 6 to12 months, in order to significantly improve from this condition.Ayurvedic treatment for this condition includes oral and external medication along with some diet restriction.if needed (yoga also)If u give a clear detail of this patient i can guide u more."
},
{
"id": 174840,
"tgt": "What causes drooling when suffering from cold?",
"src": "Patient: My three year old son woke up with a cold and is drooling excessively. He is still playing with toys but is uninterested in eating or drinking much. He has 9 teeth on top, he is missing a central incisor (suspected to be born without). What would cause him to drool? Doctor: It may due to sore throat, acute epiglottis, or excessive salivation when child not enough drink saliva."
},
{
"id": 180726,
"tgt": "What do swollen blood vessels under the tongue indicate?",
"src": "Patient: The blood vessels under my tongue are bulging and swollen, thick and more bluish/black in color. Last year a black spot was looked at by the ENT surgeon and he said to watch the spot for any changes. It has not changed but the veins under my tongue, below the spot, are 3/4 distended into my mouth, of the bed of the tongue. I have no pain but it feels funny and looks horrible. Doctor: Hello,Veins under the tongue or the lingual veins can appear large and dark, and it can be normal to see large veins under the tongue. At times there can be dilated veins under the tongue. As if there is no pain, you need not worry about it. However, if you have doubts, please send me pictures of the underside of the tongue and the black spot so that I can guide you better. Hope I have answered your query. Let me know if I can assist you further.Regards,Dr. Honey Arora"
},
{
"id": 69593,
"tgt": "What is the cause of swollen lump on the head ?",
"src": "Patient: Hi, I had my hair dyed 2 days ago, i go to the shop all the time, its organic, and I now have a lump on the back of my head which is swollen and hurts when touched but also feels quite numb when I touch it to, should I book an appointment to visit the doctors? Doctor: Hi ! Good morning. I am Dr Shareef answering your query.Yes ! You should show it to your family physician to rule out any infected lump over there which is not very unlikely and have a proper treatment. Good luck.Please do not hesitate to ask in case of any further doubts.Thanks for choosing health care magic to clear doubts on your health problems. Wishing you an early recovery. Dr Shareef."
},
{
"id": 190549,
"tgt": "Had triple surgery. Complications with high INR level. Go for overhaul on teeth?",
"src": "Patient: I had a triple a surgery oct 2011 & have had various complications since including I am told a 5+ inr level, also I am told this is dangerious ? Recently I have had a Quincy lanced + heavy anti-biotics after. I wished to get a major overhaul on my teeth , but was warned that high INR levels can create problems with bleeding ? James Doctor: Hi, If the dental treatment you desire does not involve extractions, any gum surgery (treatment in which bleeding may occur), then you may undergo the treatment. For example if you need crowns, bridges or laminates on your teeth where no bleeding in involved to change the shape of your teeth or the over all look. then you can go ahead with it. Still do have a thorough discussion about your medical problems with your dentist before you decide to go ahead with it. Regards"
},
{
"id": 9651,
"tgt": "How to get rid of dry skin and wrinkles ?",
"src": "Patient: Hello Doctor , Iw ould like to know if there any remedy for Dry Skin ,and also i have wrinkles all over the body Doctor: apply moisturising cream over dry skin. for wrinkles ,you have to eat balanced healthy diet full of fresh fruits & vegetables. if you want quick relief for wrikles, you should go for botox therapy."
},
{
"id": 90356,
"tgt": "Suggest treatment for lower abdominal pain",
"src": "Patient: my boyfreind 50 has been experiencing lower abdomen pain after eating starting about 6 weeks ago,he does have a large gut anyway but says itseems to be more puffy lately. Along with periodic constipation he has been drinking at least 50 oz of pepsi daily for the past 30 years. Doctor: Hi,Its a pain from a long time,History of pain after eating is typical history of gastric ulcer,You will have to undergo some blood tests,abodimal and pelvis ultrasonography,and most propably endoscopy,I recommend you to visit gastroentrologist and take his opinion."
},
{
"id": 147994,
"tgt": "What causes head spinning and dizziness after bruising my chin?",
"src": "Patient: Hi. About 3 weeks ago I got slapped on the right side of my face. I had a quarter size of bruising on the right of my chin and a broken lip. It happened about 7 p.m. at 9 p.m. I went to sleep. When I woke up the next day I could not sit up. My head was spinning. I have not been the same since. For 2 weeks I experienced dizziness as if I were drunk, then 1week ago I developed real bad right ear pains and the right hemisphere of my head is sensitive to touch. I have been taking amoxillin 500 mg 3 times per day, for 6 days. Ear pain stopped, but now is back again. To recap: 1. 1/17/2014. A couple of hard blows in my face, mostly in the right, including my jaw. 2. 1/18/2014 Dizziness the next morning combined with severe vertigo, but no vomiting or naucea. 3. 01/28/14. Ear pain started. Headspinning continues when my head changes position. 4. 01/30/14. Began amoxicillin treatment and neomycin drops. Pain stopped 02/02 5. 02/05/14 Pain came back, still taking amoxicillin orally and neomycin drops (in my ear) and aleve for pain. 6. I ve done the Epley maneuver, 4 times. It has not worked. My questions: Do you think that blow to my face injured something in my ear? What should my next step be? Should my prescription be changed? Doctor: Welcome to hcm!From my clinical experince I can say that you have suffered an injury to nervous system ( due to the blow on your face). It does not apper to be due to infection ( that is why antibiotic will not work properly in subsiding it). You should get an MRI brain to rule out any injury / concussion syndrome. Get visual screening test done. I would advice that you shoud see a neurologist and get your self throughly examined. Any further change in medicine should be done only after complete investigation.Hope the reply is usefulPlease feel free to ask more questionsDr. Manisha GopalMD psychiatry"
},
{
"id": 29926,
"tgt": "Does an intestinal infection cause severe diarrhoea with mild headaches and fever?",
"src": "Patient: does a intestinal virus cause severe dianrrea - six to 8 times per day? I has this for 8 days running...water was all I could hold down, so almost pure watery stool. Occasional mild headaches. Low fever. Slept about 17-20 hours day. YYYY@YYYY Doctor: Hello!I have been through your question. Related your symptoms I think this is an intestinal infection but the cause could be viral(norovirus, rotavirus) or bacterial (Salmonella, shigella, E.coli ect) Intestinal infections can be serious depending on the microbes which have caused the infection.You should know that symptoms of an intestinal infection could be some like watery diarrhea and some time bloody, abdominal pain, fever, headache (sign of dehydration), nausea and if symptoms persist for several days as you have it is important to do a physical examination by an Infectious diseases specialist, do a full blood count, azotemia /creatinemia( to see renal function), electrolytes, and sample stool and stool culture to define better the diagnosis( if is viral or bacterial) and to start the treatment ( with antibiotic if the cause is bacterial like salmonella, E.coli, shigella and ect). Furthermore during this time I suggest to drink a lot of water and tea and take probiotics.I hope my answer help you.I wish you a quick recovery."
},
{
"id": 46683,
"tgt": "What does urine protein report regarding kidney indicate?",
"src": "Patient: My name is abdul jabbar. I have a one kidney. I am 48 years old. My 24 hours urine creatinine and urine protien report is: serum creatinine 1.2 mg/dl, creatinine clearence is 73ml/min, surface area is 2.00 m2, urine creatinine 1.5 gm/24HR, Urine Protein 1189 mg/24HR, 24 HRS Urine Volume is 4100 ml/24HR. Please guide what is the position of my kidney, Doctor: Based on your report protein leakage in urine is little high. Probably you need renal biopsy for planning further treatment. All the best. If you have any questions feel free to contact me."
},
{
"id": 135102,
"tgt": "Suggest treatment for chronic pelvic pain",
"src": "Patient: I went to the gyno today for 2nd opinion. Have had chronic pelvic pain. He said it was urology issue and urologist cleared me and said it was gyno issue. New gyno said it could be interstitial cystitis. I do not have bladder problems (beside occasion infections). No urgency or accidents. Any thoughts? Doctor: Hi Dear,Welcome to HCM.Understanding your concern. As per your query you have chronic pelvic pain . Well there can be many reasons for symptoms you mention in query like endometriosis , muscle spasms , inflammatory disease , sexually transmitted , fibroids , irritable bowel syndrome or depression. I would suggest you to consult gastroenterologist for proper examination . Doctor may order ultrasound , endoscopy along with stool test . Doctor may prescribe drugs like sucralfate , anti inflammatory , omeprazole , ornidazole along with laxative . Doctor may also refer you to orthopedic surgeon for muscle problem . For now eat light and non spicy and take ibuprofen or acetaminophen for pain .Hope your concern has been resolved.Get Well Soon.Best Wishes,Dr. Harry Maheshwari"
},
{
"id": 51635,
"tgt": "Can Pangraf be taken instead of Vingraf by a kidney transplant patient ?",
"src": "Patient: My mother is kidney transplant patient. she is taking vingraf capsules for 4-5 years. can she take pangraf capsules ? which one is better - pangraf or vingraf? Doctor: Hi,Jitimultani, Thanks for query. Both medicines are same,company is different, Both contains Tacrolimus. Always take before meal or after 2-3 hours after maeal. As she is taking vingraf,continue with vingraf. ok and bye."
},
{
"id": 19906,
"tgt": "Can excess tuna fish consumption cause tightness in chest?",
"src": "Patient: my son eats alot of fish...tuna, exercises hard each day, does alot of cardio...once had tighten of chest at gym...his ekg was adnormal...doc said maybe sinus bradycardia..now has to go trough 24 hour heart monitor and echo...never took a mercury poison test...should he do so. never had a problem with heart..only one time short breath pulse is 38-46... Doctor: Hello!Welcome and thank you for asking on HCM!Regarding your concern, I would explain that bradycardia is quite normal in healthy and active persons. It can be related to chronic physical activity in sport persons. I agree with your doctor that a Holter monitoring is necessary to investigate his heart rhythm for a prolonged time. A cardiac ultrasound is also necessary to investigate for the presences of cardiomyopathy. Regarding the fact that he eats a lot of tuna, I agree with you that he may be at risk for mercury intoxication. For this reason, it is necessary performing mercury plasma levels. Hope you will find this answer helpful!Kind regards, Dr. Iliri"
},
{
"id": 79017,
"tgt": "Could birth control cause chest discomfort at night?",
"src": "Patient: Lately I have been having the feeling of pounding in my chest at night when I lie down to sleep. Usually an hour into lying down. I never have heartburn and I have never had symptoms other than possibly this one but I think this might be GERD. I'm 34, female, a healthy weight, eat healthy -lots of fruit, veggies, and work out 3-4 times a weight with cardio and weights and yoga. I believe this is GERD at night because I tested taking zantac a few times and the pounding did not happen. I haven't been eating too close to bed - eat about 2-3 hours before bed, and usually have salad. I think possibly its my birth control. I take birth control right before I go to sleep at night on an empty stomach. i am wondering if maybe switching that to daytime use and having it with lunch with get rid of symptoms. Could the birth control be the cause? I recently had a checkup and I was fully heathy, my blood test had all my vitamins and minerals in the normal range and my heart checked out. Doctor: Possible it is GERD as you are suspecting . Try taking the medication with food as well as take antacid every night for a period of a week. Also try to elevate your head end by extra pillows at night. Might give you some relief."
},
{
"id": 215139,
"tgt": "How can pain be managed in leg, neck, etc.?",
"src": "Patient: I have been taking tramadol 50mg once a day as and when my back and neck pain due to bulging disc became unbearable . Now the muscles in both upper forearms and muscles in both front thighs are going into spasms is this due to pressure on nerves from bulging discs 1 in c6-7 vertebrae and also LS 1 &osteo in 456 or due to side effect of tramadol which is increashed to 2 mornings as pain is worse. I am awaiting electric impulse tests soon as no reflexes found in upper arms also have severe pain in both legs and feet and amround both ankles . Thank you Doctor: Hi, Well, tramadol is a 3 times a day drug. spasm and other nerve signs are treated better with nerve medicines like lidocaine, anti-epileptics, even cold packs and electric impulse generators (TENS). Take care. Hope I have answered your question. Let me know if I can assist you further. Regards, Dr. Matt Wachsman, Addiction Medicine Specialist"
},
{
"id": 33691,
"tgt": "What could be the reason for having dry tongue, swollen tonsils, sinus infection and stomach pain?",
"src": "Patient: These symptoms have been going on for the last few weeks. These symptoms include dry/burnt tongue, Swollen Tonsils, sinus infections, Stomach pains and sharp pains on the left side of my abdomen towards the rib section. Can you give me any ideas of what may be wrong with me? Doctor: thanks for questionyour data is incomplete you didnt mentioned that which problem start initially,your age, sex etc.swollen tonsil, sinus infection followed by stomach pain is indicative that infection initially involve your throat and later on it desends and also involved your abdomen. consult a physician and get it treated soontrulyDr. arvind"
},
{
"id": 80764,
"tgt": "Suggest treatment for mild chest infection",
"src": "Patient: hello for about 4 weeks I have felt breathlessness. I am normally quite a fit person I do taekwondo and at times its tiring, but this is different, I feel that I am not getting enough air and that something is trapped in my throat and chest, at first I thought it might be a mild chest infection, because I have suffered with them before on a few occasions but they have been bad where it felt someone was sat on my chest. but this is different and now it feels like I have trapped air in throat and wanting to burp, its really a second opinion as I have seen doctor twice and had a blood test today. I am 48 and I wondered at first if it was thyroid connected. can you help. Doctor: I would like you to answer me few questions which would help me get to the diagnosis of your problem:-Do you hear whistling sound with breathing when you are breathless- Are you allergic to dust or any strong smells- Do you get similar symptoms with climate change- Does having cold foodstufs lead to cough or sneeing- family history of asthma or hives- do you smoke tobacco or any such addictionsI would opine you to get an Xray chest and a Spirometry (pulmonary function test) done..We need to rule out asthma or a hypereactive airway disease.."
},
{
"id": 169087,
"tgt": "Is irregular heart beat in a new born a matter of concern?",
"src": "Patient: irregular heart beat before baby was born, saw fetal heart doctor-had ekg stress test ect. Doctor said should be fine either before delivery or shortly after. Two weeks old tomorrow and at last visit still slightly ireg. Due to see heart doctor in to months for check up. Is this something I should be worried about? Doctor: irregular heartbeat in relation to breathing is common in infants. please checkup regularly with a cardiologist paediatric. go for ECHO if you are still worried."
},
{
"id": 126157,
"tgt": "What causes swelling in the fingers?",
"src": "Patient: my index finger on my left hand gets swollen and stiff after I play guitar for an extended amount of time. Im a musician and this is starting to worry me. Is there something I can take to decrease the swelling and remedy this situation. can you tell me whats causing this ? Doctor: Hi, It is nothing but a contusion arising out of excessive use and strain. As a first line of management you can apply ice packs and take analgesics/anti-inflammatory combination like Aceclofenac/Seratiopeptidase for symptomatic relief. If symptoms persist better to consult an orthopedics and get an MRI scan done. Hope I have answered your query. Let me know if I can assist you further. Regards, Dr. Shinas Hussain, General & Family Physician"
},
{
"id": 102259,
"tgt": "What does a CPK level of 175 mean?",
"src": "Patient: Hello. What does a CPK level of 175 mean? I have seen a geneticist who ordered this test and it came back high. I have a mitochondrial disease, hypothyroidism, fibromyalgia, tachycardia, restless leg syndrome, asthma, pulmonary hypertension. I am a 44 year old female. I don't know if that matters. Doctor: Hi, thank for posting.Phospho-creatine kinase is an enzyme that takes part in tissue energy production. CPK is found in tissues that consume high energy. It is found in high amounts on skeletal muscle, brain, photoreceptor cells of the retina, hair cells, inner ear, spermatozoa, smooth muscle and on myocard.CPK level of 175 is in its maximal limit.High level of CPK shows tissue damage of the above organs.To determine the organ that is damaged, you need a CPK isoenzymes test.So you should consult your geneticist.Regards.Dr. Behar."
},
{
"id": 162700,
"tgt": "What causes cough in a child when diagnosed with adenotonsillectomy?",
"src": "Patient: my 4 year old daughter had adenotonsillectomy yesterday and has started to cough chronically today. she is on flovent for asthma and the cough had improved tremendously until last night after the surgery. is the cough normal? the nurse at the surgery center told me she removed a ton of yellow fluid yesterday and cleaned her out... Doctor: Hello and Welcome to \u2018Ask A Doctor\u2019 service. I have reviewed your query and here is my advice. This type of cough immediately after surgery is not normal. Some times the ligated vessels might rupture. I suggest that her surgeon or his team needs to see your daughter once. Rarely some anesthetic drugs can cause this. Hope I have answered your query. Let me know if I can assist you further."
},
{
"id": 198537,
"tgt": "Will the frequent urination be due to masturbation?",
"src": "Patient: Hi doctor I am suffering from frequent urination I don t have this before as I used to mastrubate thrice a weak but nw stated mastrubating once a day due to stress in exams also it habituated in holidays even.....was using toilet atleast 30 times a day. Was scared what the problem is??? Is this due to mastrubation??? Or any other else???.....I frequently using toilet had no pain and my urine is clear nd no abnormal smell. Plz give reply soon. Doctor: HelloThanks for query .Frequency for urination after masturbation is mostly due to inflammation of delicate mucous Membrane of Urethra due to rough ,vigorous shaking of penis by firm grip of hand .You need to take anti inflammatory medicine like Diclofenc along with urinary antiseptic Nitrofurantoin twice daily.Drinking more water will help to keep the urine dilute .Restrict and avoid masturbating frequently .This will help you in long term to avoid facing many sexual problems in future .Dr.Patil."
},
{
"id": 203406,
"tgt": "Why did I not find any semen after ejaculation? How much semen is required to impregnate a woman?",
"src": "Patient: 1. i ejaculate before my wife does and in most cases i see no semen after ejaculation? 2. can a man 38yrs with about 94.42 to 100,million sperm counts be able to impregnate a lady of 25yrs? 3. can a man 38yrs with about 84.42milloin sperm counts be able to impregnate a lady of 36yrs? my e-mail ID YYYY@YYYY Doctor: HiThanks for choosing healthcare magicYou need just one active and motile sperm to pregnant a woman, but at time of ejaculation your sperm count should be more than 20 million. It would increase the chances of fertilization. Rest depend upon mobility of sperm. In your case, sperm count is good, but get sperm motility checked. That would help you clear reason of your problem.Thanks"
},
{
"id": 56681,
"tgt": "What are the signs of having gall bladder disease?",
"src": "Patient: I have symptoms of gallbladder disease. Sharp upper right quadrant pain. Doctors checked for stones and negative. Next I m having a Hida scan. Been constipated for a week and now when I tried and wiped I had a pink mucus discharge and a pink drop in the bowl. Could this also be from gallbladder disease? Doctor: Hi,Thanks for posting your query.I am Dr.R.K and I am pleased to assist you.Gallbladder disease can present with right upper quadrant pain, jaundice and fever. The blood and mucus stool you had is not a symptom of gallbladder disease. It indicates you are having colitis. This could be due to infection or inflammatory bowel disease.Stool analysis and if needed a colonoscopy may be done to find the cause.I hope that answers your question.Regards,Dr.R.K."
},
{
"id": 153194,
"tgt": "Is surgery advisable for abdomen cancer?",
"src": "Patient: Hi, My father is suffering from Stomach Cancer. We came to know about this a month ago. Doctors here are suggesting for the operation. As my father's age is 67 and he have diabetes from last 10 years and recently BP is there. We are thinking whether to go for the surgery or not? Is there any better treatment for this cancer apart from surgery? Please help us to take the decesion. If you can provide us the email address, I can send you the scanned copy of the reports. Please give mthe phone no, so that I can better explai. Please help us sir...Thanks in advance... Doctor: Hello! Good Day! Primary treatment for Gastric/Stomach cancer is still resection/surgery, especially for early stage cancers. Tumor needs to be removed in order to control the source, in order for it not to go anywhere else in the body. However, for locally advanced Gastric cancer, in other words unresectable or bulky tumors, chemo and radiation therapy is will be the initial treatment, followed by a resection, if chemo/radiation will be able to decrease the tumor size and converted it to a resectable tumor. For more advanced cancers (Stage IV/ with distant metastasis), chemo/radiation plus other palliative treatments will suffice."
},
{
"id": 107059,
"tgt": "How can severe pain in the lower back be treated?",
"src": "Patient: Hi Doc,I have a terrible lowe back ache-it is almost if something is pricking me,it hurts when I sit down and want to get up-I have not hurt myself,it started on monday-last week,when I stand it is ohk but as soon as i sit and get up I feel this pain -do u think I should drink a lansoloc 30mg-or is for something else Yours sincerely Patricia Doctor: In my opinion it\u2019s best to see an orthopedist.meanwhile one may take parrafonDS twic daily after meals available OTC.Avoid bending,lifting objects. Do hotfomentation and apply Voltaren gel.It should get well with this, but do Consult orthopedist doctor for knowing cause with investigation if needed"
},
{
"id": 198789,
"tgt": "What causes penis pain, urinating small amounts frequently when masturbate/ejaculate daily?",
"src": "Patient: hello my penis has been hurting alot and I have been masturbating everyday and ejaculating also. And I always have this urge to go on porn websites and start masturbating whenever I have the chance. I drink cranberry juice occasionally and last night my penis started hurting when I was trying to urinate and in these past few day I have this feeling as If there is a blockage or something in my penis which is perventing me from urinating. I also have problems urinating because I urinate in small amounts in small time. Doctor: DearWe understand your concernsI went through your details. Please do understand, masturbation in moderation is OK depending on your age. Frequent masturbation can cause inflammation of the glans penis area beneath the foreskin. Such an inflammation could cause burning sensation and other urinary tract disturbances. The blockage you are feeling could be due to your obsession with the problem. I suggest you to stop masturbating immediately for a week and consult a physician for physical examination.If you require more of my help in this aspect, please use this URL. http://goo.gl/aYW2pR. Make sure that you include every minute details possible. Hope this answers your query. Available for further clarifications.Good luck. Take care."
},
{
"id": 97731,
"tgt": "Is herbal medication safe to take?",
"src": "Patient: is it safe to take a herbal med that contains 500mg of Valerian with the prescription drugs I take which are: gabapentin, atacand, asacol, baclofen, crestor, vit D3, vit b complex, vit c, coq10 the hebal med is Somnapure, a natural sleep aid that also contains, passion flower, lemon balm, L-theanine, hops extract, chamomile flower, passion flower and melatonin. Kathleen Doctor: **1. since the said drug 'somnapure' have not been evaluated by the Food & Drug Administration, The FDA only evaluates foods and drugs, not supplements like these products. These products are not intended to diagnose, prevent, treat, or cure any disease. In best Interest I would suggest you to visit a sleep disorders doctor to get some expert help with whatever issues you're having.PS.. Using gabapentin together with valerian may increase side effects such as dizziness, drowsiness, and difficulty concentrating, you may also experience some impairment in thinking and judgment. It is important to tell your doctor about all other MEDICATIONS [as mentioned] you use, including vitamins and herbs. Do not stop using any medications without first talking to your doctor."
},
{
"id": 102767,
"tgt": "Asthamatic. What is the treatment for swollen throat, chest and back pain?",
"src": "Patient: Hello! I was diagnosed with very severe sleep apnea and I have asthma (severe) I have been having pain both in the middle of my chest and the middle of my back and at times I feel like my throat is swollen and it is quite difficult for me to breathe and sleep. How do I seek help and find a doctor that will help me? Doctor: Was your sleep apnea diagnosed on polysomnography if so what is our AHI index.Have you been prescribed home CPAP ventilation.Get your ENT check up for any problem in your tonsils or adenoids or any other obstruction like polyps.You have also not specified the treatment you are taking for asthama.Once your sleep apnea gets proper treatment then your other symptoms would get releived."
},
{
"id": 176126,
"tgt": "Can Epilex treat episodes of senselessness in child?",
"src": "Patient: Hi, I just would like to know that my nephew is suffering from a problem in which he become senseless for 2-3 seconds, 3-4 times in a day, He is 8 years old. He is now taking epilex chrone. He was fit physic wise before he started the medicine, but now a days his physic is also not so good and also there is no changing in his problem. Please let me know is this medicine is ok for this particular problem or any side effects of this medicine. Doctor: Hi..by what you quote I feel that he might be suffering from a type of seizures called absence seizures. Epilex contains Valproate and it is an anti-seizure drug and is a good one for this condition. Side effects include some weight gain and some times rarely hepatotoxicity and hyperammonemia. We need to check for these at least on 3 monthly basis.Regards - Dr. Sumanth"
},
{
"id": 29619,
"tgt": "Is it safe to travel abroad while after treatment for facial cellulitis?",
"src": "Patient: Hi i have facial celulitis..im still in hospital..im in hospitl 9 days now and i am on i.v. 9 days.i was told today another 2 or 3 days stay snd then medication at home orally ..but my question is im going on holidays in 6 weeks ,is it safe to fly abroad and also exposure to sun ... Doctor: you can go if you dont have any signs and symptoms of cellulitis left . if on the end of 6 weeks still you are symptomatic then it is better to delay your holiday . but personally i think 6 weeks are ample of time to heal yourself completely if you follow your doctor very well ."
},
{
"id": 167031,
"tgt": "Could stopping drinking milk in 18 months effect health?",
"src": "Patient: My 16 month old suddenly no longer wants to drink milk. He eats very well and has cheese and yoghurt daily, but would usually also have 2 bottles of milk a day. Should I be concerned that he has stopped drinking milk. He is still active and does not appear unwell. Doctor: Hello. I just read through your question.Based on your description, it seems like he gets his daily dairy requirement each day. As long as he is taking the yogurt and cheese, he is ok."
},
{
"id": 80853,
"tgt": "What causes minimal blunt in left lung in ultrasound scanning?",
"src": "Patient: I suffered from TB 1 years ago. I was cured after medication. I had X-rays at periodic intervals and everytime the report was normal. my ultrasound scanning was also normal, my left lung is minimal blunt what is the cause for it , and I experience mild pain appears often ..... Doctor: Blunt angle can be due to pleural effusion which can be detected on lung ultrasound. Other causes could be thickened pleura and pleural fibrosis which can cause chest pain.you have taken antiTB treatment already so there is a possibility of pleural fibrosis and its a healing process.no need to wo\u0155y for it if there is no fluid on ultrasound."
},
{
"id": 204217,
"tgt": "What is Savella prescribed for?",
"src": "Patient: Hello Dr., thankyou for your time and expertise. I hae been diagnosed with Chronic Fatigue Syndrome and 4 mnths later with s every Chemical senitivities along with non0Aids or other retro-virus related low CD -4 cells (have had a abone marrow and many other expensive ongoing blood test over a few years period. The number has fallen from 550 at first testing to 109 very many years later. My healthhas declined greatly even snince first diaganosese,but my present pain Dr. athe head of duke apin clinic in Nc, gives little attention to CD-4 cells or Chemical sensitivities and is contantly almost forcing me to try yet anti anti-depressant (savella) though I ve tried 14 others with very bad results over the tears. He threatens to stop prescribing properly taking morphine which controls my pain if I don t take a new $410.00 per month drug not even meant for CFS but is an anti-depresant (he agees I; m NOT depressed nor do I have fibromyalgia, the two things the drug is for. Should I go through further misery and take another drug that I react badly to and can t afford (my total income is $1315.00 per month and rent alone is $124.00 per month. or chance drugs that work being taken away? Stressed in Chapel Hill Doctor: Hello and Welcome to \u2018Ask A Doctor\u2019 service. I have reviewed your query and here is my advice. It is true that Savella is not approved for CFS but at the same time it is also true that it is potential medication for CFS. Your doctor is doing his best to protect your health with use of medication which causes least number of side effects in compare to current medication (morphine). In my opinion there is nothing wrong to trey a new medication with potential to treat CFS when 14 other medications failed despite the years of effort. Hope I have answered your query. Let me know if I can assist you further."
},
{
"id": 144142,
"tgt": "Suggest treatment for arachnoid cyst in brain",
"src": "Patient: Hi! I just had an MRI of the neck and brain and was told that I have an arachnoid cyst and would like to know if its ok to take 81mg aspirin on a daily basis even though I have the cyst. I always hear about hemorraging either with the stomach or even the brain with some of the risks. Thank you for your time. Doctor: Hi, I am Dr.Bruno. I have read your question and understand your concerns. Let me try to help you Question: Suggest treatment for arachnoid cyst in brainAnswer : Treatment depends on few factors 1. Location of the cysts 2. Size of the cyst 3. Whether it is growing (to be determined by another Scan after 6 months) 4. Whether it is compressing any vital structures In Most cases, we don't need any treatment as the cyst wont be compressing any vital structuresHope you found the answer helpful.If you need any clarification / have doubts / have additional questions / have follow up questions, then please do not hesitate in asking again. I will be happy to answer your questions.Let me know if I can assist you further.Take care."
},
{
"id": 171362,
"tgt": "Suggest diet which can improve growth metabolism",
"src": "Patient: My friend, Jay has a son, Jimmy, age 8, and is well-nourished so she tries to make sure that besides eating healthy that he take a daily child\u2019s multivitamin/mineral pill that also includes 10 mg of iron. She\u2019s particularly concerned that he get enough iron because she\u2019s read that iron-deficiency anemia is one of the most common nutritional deficiencies in children. Therefore, she also gives him an additional iron supplement containing 30 mg of iron that was given to her at the health clinic to take because her hemoglobin levels were low. At Jimmy\u2019s latest health check-up, the pediatrician notices that his growth rate has decreased since his check-up last year and is now below average for his age, and Mrs. Jones reports that Jimmy\u2019s appetite has decreased lately and he hasn\u2019t been eating as much. When closely questioned by the physician, Mrs. Jones tells him about the supplements she\u2019s been giving her son. The doctor feels that Jimmy is indeed suffering from a nutritional deficiency, but it isn\u2019t iron! In your essay, be sure to address the following questions and points (in essay form, don\u2019t number or list parts of your essay): Doctor: Hi, there is no need to give iron continously, once the hemoglobin levels becomes normal we should stop iron and focus on dietary improvement. Moreover, iron also causes bad taste in mouth and constipated. So, in my opinion you should stop iron immediately. Child should be given food in addition to milk, like fruits: banana, apple, mango, chiku, rice, cereals, paneer, yoghurt, paranthas, eggs, ghee and butter in every meal. I also would like to do some blood investigations like complete blood count with peripheral smear. If I was treating you then i would have given him a deworming syrup and an appetizer syrup and waited for cbc report. I hope this has helped you. If you have any more questions, please don't hesitate to ask me. Wishing your child good health."
},
{
"id": 176692,
"tgt": "Suggest remedy for stomach pain",
"src": "Patient: My 3 year old son has complained of Central tummy pain since Thursday (and on and off for past few weeks). He was sick once Thursday at 5pm, again yesterday at 3pm and again tonight at 5.30pm. Not eating much. I m trying to keep hydrated but am worried now as he is tired and not himself. Doctor: Hi..this is called chronic pain abdomen and the reason could be many like...renal stones/ gall stones/ mesenteric lymphadenitis/ urinary reflux/ or it could just be a functional pain abdomen or sibling rivalry. I suggest you consult your paediatrician with these tips and get him evaluated.Regards - DR. Sumanth"
},
{
"id": 8193,
"tgt": "Good ointment to get rid of pimples and its scars",
"src": "Patient: hi pls suggest me any good ointment to get rid of pimples and its scars i have got pimples from many yrs consulted many doctors and used many creams but dint work. Doctor: hello dear , you havnt mentioned ur age.Any way,u can try nadibact cream locally on acne lesions.Keep a good anti acne soap with you.Avoid pricking the pimples.And if you have scars following pimples then u shd get a chemical peel or a laser done for that from a dermatologist. regards, Dr.Bharat Chawda"
},
{
"id": 88248,
"tgt": "Suggest treatment for left abdominal pain",
"src": "Patient: Hi i have pain in the left side abdominal ribcage under my left pec about 2 and half inches down. i went to the gym earlier and ive never experienced this before when i breath in theres slite pain but not a great deal, also im finding it hard to eat. could you help? Doctor: Hi.Thanks for your query.Read the history and understood it. This is a classical costo-chondritis: I would suggest the following treatment.-Rest to the area by means of strapping with the Adhesive tapes during the deep Expiration phase. This will limit the movement, reduce the pain and give rest for healing to occur.-Anti-inflammatory medicines in full dose as per the weight.-Steroids if required for its strong anti-inflammatory role.-Lasix to reduced the edema.This sort of a treatment helps all my patients. and should help you too."
},
{
"id": 111124,
"tgt": "Suggest remedy for back pain",
"src": "Patient: Sir, I am 36 yr old married and resident of Bangalore. Since last 10 days I am suffering from sevear back pain from buttock to thigh. I am counsult a doctor and adised me to take nevrotin , ibugesic plus and oxalgin gel. but I have no relief . Please advise me further course of action. Doctor: Hello, I had gone through the case and found that the prescribed medicine are right for muscle spasm and nerve trapped.But if pain is as it is then go for X-ray of spine and physiotherapy for back.Also take Vitamin D3 once a week.Hope my answer will be effective for you.Thanks"
},
{
"id": 62426,
"tgt": "What causes painful lump under rib with pain in lower pelvic area?",
"src": "Patient: Hi I am a 40 year old man who has various symptoms which may or may not be related. I have in the last week been tested positive for Chlamydia to my horror. I received the treatment on saturday and have taken the four pills at once (Zithromax) a dull ache under my right rib towards my back and there appears to be a small lump there too. This lump not getting any bigger but the ache is getting worse. I also have a dull ache in my lower pelvic area still. I am worried that the infection has led to complications elsewhere in my body. I have generally been feeling unwell for a few months but have not been sick or have not lost weight. I have recently lost my mum to cancer and have been too scared to go and get checked out for this. Can you help. Doctor: HI,Dear,Welcome to HCM.Based on the facts and data of your query,You seem to have a complex problem of 2 entities -a-Painful lump under the rib- which seems to be sebaceous cyst with reactivated infection of the cyst / or could be due to the fibro-myalgia from the pelvic chlamydial infection which you seem to have over last few days.b-Genitourinary infection causing pelvic unwellness/dull ache indicative of -cystitis/ urethritis/ with prostatitis due to chlamydia-with which you are diagnosed lately.This is STD disease and needs detailed and thorogh follow up treatment with Dermatologist doctor.c-Cancer Fear Psychosis with obsessive anxiety-as your mother died lately with Cancer.Check with your psychiatrist, who would resolve it by proper medicines.In the above scenario, you need to be very cautious and take the needful treatments for the varied health issues you have.Hope this reply would help you to resolve your sever anxiety.Welcome for any further query in this regardWill appreciate writing your feedback review comments,to help the needy patients like you at HCM.Good Day!!Dr.Savaskar,Senior Surgical SpecialistM.S.Genl-CVTS"
},
{
"id": 14678,
"tgt": "Suggest treatment for rash on face",
"src": "Patient: I have had a rash on my face. When I wash it with or without soap etc warm water. Red pinpoint pus spots. But only come up when I was my face in morning and night. Doc said it's acne and I don't believe this! I also found today a small lump under the skin just down past my ear into neck. Like gland maybe ? Doctor: HiWell come to HCMThe infective skin condition may be likely if I would be treating physician of this case then I would treat this case with the Tab Erythromycin 500 mg three time in day with the Erythromycin local cream, with advise to keep the face very much clean, hope this helps."
},
{
"id": 109655,
"tgt": "What causes back pain, shortness in breath and pain while breathing?",
"src": "Patient: I thought maybe it's just a pulled muscle from swimming......but not sure, I'm 33 white female, a smoker, drinker, otherwise healthy......severe lower back pain in lower right side, shortness of breath, sharp pains when taking deep breaths, unable to lay on left or right side only flat when sleeping??? Doctor: Welcome to HCM,I had gone through the case and found that it might be muscle spasm.So avoid swimming for a time. Take bed rest and any mild painkiller with apply muscle relaxant gel and heating pad. Pain will disappear 3-4 days.Hope my answer will be effective for you.Thanks"
},
{
"id": 170726,
"tgt": "How to treat kidney scar in a 11 years old child?",
"src": "Patient: My 11 year old daughter has:- scar on kidney, duplex kidney, spina bifida occulta, autism, very tall compared to peers, very thin, fontenal was large at birth, high arched palette, growing pains (under investigation for JIA), legs not straightening, narrow ear canels, complains about blurred vision (knothing found). Do you think Marfans syndrome could cause all of this? - My husband has stretch marks all over and no muscles yet v strong, v tall, my son and husband have curved backs (hollow back), my husbands dad died from aortic anorism at 44. Doctor: i think it could a kind of genetic connective disorder called ehler danlos syndrome in your husband and son,but your daughter features are more suggestive of marfanoid type...invariably both can be evaluated and diagnosed by genetic workup and approriate management..So please meet the geneticist and community pediatrician who could alliase you with a multi disciplinary team ...Hope this helps"
},
{
"id": 80635,
"tgt": "Rattle in chest with wheezing and green cough, on and off headaches",
"src": "Patient: I have been sick today is the sixth day. I have quite a rattle in my chest, with some wheezing. I am coughing up green. My headaches on and off but the body aches have stopped. My throat was sore but now only hurts when I cough. Have had low grade fever but I don't now. Should I go to the doctor? Or will this resolve on its own? Doctor: Hello dear, thanks for your question on HCM. In my opinion you should consult pulmonologist and get done1. Clinical examination of respiratory system. 2. Chest x ray. 3. PFT ( pulmonary function test ). Green mucus, wheezing and rattling sound , is more suggestive of either lung infection ( pneumonia ) or bronchitis. So chest x ray is needed to rule out pneumonia. PFT is needed to rule out bronchitis. You may need inhaled bronchodilators and antibiotics. So better to consult pulmonologist and first diagnose yourself and then start appropriate treatment."
},
{
"id": 88346,
"tgt": "What is the constant pain in lower abdomen after an abortion?",
"src": "Patient: hi dr...i m 24 years ...i hav 2 years of my marriage after 1 yr of our marriage i was pregnent ...but my in laws abort it ...now we are trying 4 a baby but still i hav pain in my lower left abodment ...this pain is from last 1 yr ...when i had abortion ...can u suggest me why this pain is?...becoz i consult my docter ...i did all my scans .,soab test .....but everything is fine Doctor: Hi ! Good evening. I am Dr Shareef answering your query.From the history of yours, I would presume it to be a PID (pelvic inflammatory disease) or a UTI (urinaty tract infection). If I were your doctor, I would advise you for a complete blood count, a urine routine/microscopic/culture and sensitivity test, a vaginal swab for culture and sensitivity test, and ultrasound of abdomen to rule out any intra pelvic organ pathology. Further management would depend on the results of a physical examination and related investigations as advised.I hope this information would help you in discussing with your family physician/treating doctor in further management of your problem. Please do not hesitate to ask in case of any further doubts.Thanks for choosing health care magic to clear doubts on your health problems. I wish you an early recovery. Dr Shareef."
},
{
"id": 10986,
"tgt": "Can hair thinning be due to medications?",
"src": "Patient: I am on the following medications: Mucinex, potassium, calcium, Evista, Diovan, Singular, HCTZ, Ranitidine, Vit D3, Tramadol, Naprosin, Lyrica, Advair discus. I have been having problems with hair thinning for the last couple of years. Although I am 70 years old, I think that it is medication induced. I have read that Singular can cause the problem. Can any other of my medications be adding to the problem? Doctor: Dr. Hanif warmly welcomes you!Thanks for consulting at my virtual clinic. I have carefully worked through your case, and can well realize your health worries. Being your physician, I want to assure, I will take care all of your medical concerns.No it is not due to medicines. Even Singulair does not cause this issue.Your age is 70 and at this age hair and skin issues start and very common is that. I have researched on the reason amongst 60+ age group of a large number of my patients for years and the reason was Toast and Tea syndrome. It refers to the poor dietary habits in this age group. People don't take care of the diet they have.I suggest you to please improve your diet and keep proteins (skimmed milk, eggs, fish) in your diet in handsome amount. Add green leafy vegetables and salads too. Plenty of water and a good physical activity is a must.I want to add a supplement specially for your needs. Capsule perfectil for skin hair and nails, once daily. It will definitely help you get better hair very soon.Hope I answered all of your questions. Please don't forget to add your precious positive feedback to help me able to assist you better in future too. Have a wonderful time ahead. Keep in touch for a follow-up too.Regards!DR. MUHAMMAD HANIFUSA"
},
{
"id": 170774,
"tgt": "What causes constipation with bleeding?",
"src": "Patient: hi doctor, i have a 6 month old baby girl.in a day she is mostly on breast milk and ragi.Two weeks back i started on stage 1 ceralac. for the past 5 days her stool is very hard she does the process with lot of effort. one day she had even bleeding while putting stool.what can be done so that like before her stool is loose. Doctor: Hi.... she might be having this constipation because of ragi in her diet. Ragi is very rich in calcium which predisposes to constipation. I suggest you reduce the ragi and her diet and also include papaya and banana in her diet. I do not suggest using laxatives at this tender age as it might cause severe diarrhoea or it can become habit. I also suggest you restrict the milk intake to maximum 400 ml per day till this constipation is relieved.Regards - Dr. Sumanth"
},
{
"id": 31506,
"tgt": "Is Condyline, effective for genital warts?",
"src": "Patient: Hi i was recently treated for Genital warts at my GUM Clinic a few months ago and they were frozen and i was given Condyline to put on them twice a day i think it was. Now i have some i have just noticed on my foreskin and i still have the Condyline in my room. I was just wondering if i have to get the small warts frozen first or can i skip going to the GUM Clinic again and just apply the Condyline?? Doctor: Hi thanks for asking question in HCM.According to various studies done, it has shown that some wart resolve spontaneously or it will remain stable in size or increasing in size.And removal of wart is not always necessary.So it is individual preference whether to remove it or not.Here you have reccurance of wart , so it can be removed as usual with freezing technique using liquid nitrogen.It is better to remove it with freezing technique rather then applying condyline first.And dont try to remove wart with any home base remedy as genital skin is very sensitive.I hope i have solved your query.thanks."
},
{
"id": 3881,
"tgt": "How to conceive when diagnosed with hypoechioc intramural myoma?",
"src": "Patient: Iam 26 years old, trying to conceive. Iam being diagonised with a hypoechioc intramural myoma measuring 2.3mm x 2.4 and 1.7mm x 2.1mm along the wall of posterior. I don't usually have a painful menses but i feel waist pain always. I really want to conceive what can i do? pls kindly advice me. Doctor: Hi, thanks for writing..I dont think you will have problem in conceiving with such a small fibroid.. You can conceive without much difficulty provided you are ovulating.. Only if the placenta locates over the fibroid, blood supply might be affected.. But there are many cases where the patient reaches term and delivers normally without any problem.. Hope i have answered your query.. good day.."
},
{
"id": 199260,
"tgt": "Suggest treatment for rashes on penis",
"src": "Patient: I have a rash on my penis and the skin behind the head of penis its red in color it bleeds from the skin cracking I have tried a cream for a yeast infection but it just made it worse and any type of cream like Neosporin or lotion just made it spread I have went to two different health clinics and its not an STD they said it might be fungal I cant remember the cream I was given before but it just made it spread too.? please help. Doctor: HIWell come to HCMI really appreciate your concern, such lesion could be due to hypersensitivity reaction some time may be due to some drugs if you have taken it, condition is almost self limiting but it can be managed with broad spectrum antibiotic ointment and best is \"Povidone Iodine\" keep the area clean, hope this information helps, take care."
},
{
"id": 131972,
"tgt": "Experiencing difficulty in breathing due to an external injury on hip/back bone",
"src": "Patient: I am on a 5 day prescription of prednisone! It is because I injured myself on the job, but the doctor found that the inflammation that was caused by my injury was because I have arthritis in that area. In my hip/back bone. I feel as if someone is sitting on my chest and making it difficult to breath. Is this normal? Doctor: Hi any injury in the back or hip will not cause breathing difficulty unless there is some other cause / Injury in chest. Steroids are given in spinal injury only in rare cases when patient has paraplegia or quadriplegia. In hip arthritis or spinal arthritis Steroids have no role.I suggest that you again see your doctor."
},
{
"id": 179145,
"tgt": "Suggest remedy for sparse hair growth",
"src": "Patient: Dear Concerned,I have an enquiry about my male child who is just 10 months old. His hair is not growing as per his age. He has very less hair inspite of he is 10 months old. So can you suggest me what we can do in home or any medicine to regrow his hair faster. Doctor: Hi Dear welcome to the HCM,Improve his nutrition with high protein diet.Add multivitamins and iron syrups.Vitamin D to be taken care of.Some specific vitamins like Biotin and calcium pantothenate to be supplemented for some long time.Expose the child to sun daily for some time.Hope all these measures will help to grow the hairs faster,thanks"
},
{
"id": 55756,
"tgt": "Suggest remedy for hepatic cirrhosis",
"src": "Patient: What more can I do for hepatic lipidosis/cirrhosis. I m on diuretics, (don t do much), can hardly eat food but am putting on wt. Too tired to do any more exercise than shuffle around the park on good days. That helps more than anything but it s very hot here now so I ve stopped walking unless I get up very early. Doctor: Welcome at HCM I have gone through your query and being your physician i completely understand your health concerns.What is ur age? For how long u are suffering from it? Any other associated problem? like diabetes or hypertension? Any history of TB in past or exposure to TB? Are u running fever? Any other lymph node swelling in ur body? what is the cause of cirrhosis? alcohol? HCV ? HBV? As far as cirrhosis is concerned, u cant reverse it but u can manage it by changing the life styles. Avoid salt , and meat. you can use meat once a week like fish. Use regular ISPAHGOL husk to prevent the constipation. try to eat fresh vegetables along with fresh fruits and juices. use VIT k after every three months. Dont skip your medications like ALDACTONE , INDERAL. You can use multivitamin and liver tonics like SILLLIVER daily. Meet your HEPATOLOGIST , he will adjust the dose of medicines for you and make proper workup. If i am ur attending physcian i will get ur FBC LFTS S, ALBUMIN, PT/INR ,ABDOMINAL ULTRASOUND every six months to estimate the severity of cirrhosis Get your ENDOSCOPY and COLONOSCOPY done every two years Meanwhile stay calm and use acetaminophen to relieve pain Get well soon Hope your query is adequately addressed if you still have any feel free to ask RegardsDr Saad Sultan"
},
{
"id": 161919,
"tgt": "Why does my 15 months old spit the food out always?",
"src": "Patient: hello doctor, my 15-monoth-old baby boy is continuously spitting the food that i give him for about the past month..i took him to the pediatrician and she gave me prescription to deworn him and i did that too...but no improvement. only if he forgets himself by playing with some new toys or playing in tap water, he swallows in..else he spits out..what can be done...any suggestions....i would appreciate your advice Doctor: Hi, This looks like the baby is not liking the food. He might be choosy I guess! I would like to suggest you try different varieties of foods and present the food before him in some attractive ways: colorful dishes and garnish with jems/tomato sauce. Play with him and feed. Invite other kids/friends also and make competition among children for finishing the food! Remember kids usually like eating in little amount but frequently. So give them short and frequent diet. Hope I have answered your query. Let me know if I can assist you further. Regards, Dr. Ajaygupta009, General & Family Physician"
},
{
"id": 206709,
"tgt": "How to control sexual feelings?",
"src": "Patient: I'm 24 years old and i have sexual feelings against my mom love to watch it dressing , love to touch parts of her body and when she in the bathroom she let the door little bit open and when she finish pee she up her panties out of bathroom i used to spy on her and sometimes i want fuck herafter i watch her and touching i go to bathroom and masturbate when i finish i fell shame that i have this feelings against my momi don't know what to do and her in my country we don't have sex until we married , plz help me Doctor: DearWe understand your concernsI went through your details. I suggest you not to worry much. I sincerely opine that the problem you are facing now is of your own creation. These are just obsessive negative thoughts. We all know, we cannot behave in the given way with our mother. You are having sexual urge even when you have nothing to do with your mother. The more you try to escape from the situation, the more obsessed you will be. Therefore, the best method is to ignore this. Promise your self that you will not fantcise with your mother and ignore the urges. Keep yourself busy.If you still need my help, please describe the whole problem in detail and post a direct question to me. I shall definitely help you with psychotherapy techniques to over come your problems.Hope this answers your query. Available for further clarifications.Good luck."
},
{
"id": 225375,
"tgt": "Is there a chance to get pregnant after taking nordette pfizer pills after having unprotected sex?",
"src": "Patient: Hi doctors! I had unprotected sex 4 days after my period at about 11:30pm. I took 4 white pills of Nordette Pfizer 8 hours after the deed and 4 white pills again after 12 hours. Then I took another 4 more pills in the next days and took again after 12 hours. So in all, i have taken 16 pill of nordette.is that ok? Will i get pregnant consume too much pills? What are the chances. Thanks. Doctor: Hi, Welcome to Health care magic forum. Actually the tablets nordette should be taken in 2 doses, of 12 hours gap. If excess of the pills are taken, there may be no use but they may interfere with the effect of the first 2 doses. When pills are taken out of the shedule, the effect may be indefinite, and may depend upon so many factors. Still the posibility if getting pregnant in your case are very less.The day of the sex is in the safe period. The first 10days after the period are safe, because there wont be the release of the ovum to become pregnant. 11th to 20 th day from the day of the period are unsafe, as during these days Ovum may be released on any day and ready for conception for 24 hours. 21st to 30th day again safe, because ovum is not available. If period is late by anxiety or some thing have a home pregnancy test. Wishing a good time. Thank you."
},
{
"id": 176945,
"tgt": "What is the cause for diarrhea for about a month?",
"src": "Patient: Hi I have a six yr old rottie who has had diarrhea for about a month...took her to vet they gave her metronidazole , cainine probiotic and we put her on a boiled chic and rice diet none of this has helped.she eats and drinks but I have no clue what this could be...the vet didn t do blood tests or anything because she seems healthy should I go back and request a blood test? I have also tried a few home remedies such as pumpkin and herb called slippery elm bark to no avail either(should I try pepto?)....shes starting to loose weight but in great spirits... Doctor: diarrhoea persisting for >14 days is termed as chronic diarrhoea. And it can be due to a number of conditions both infectious and non infectious. Get a stool test for ova, cyst or any parasite. Also get a stool routine microscopy, culture and stool for occult blood. He will also have to get a Thyroid profile, Hiv Elisa and pancreatic function assessment. As for its management, put the child on Lactose free diet(avoid milk and milk products). Give syrup Zincolife or any other zinc preparation 5 ml once daily. Give her tab Albendazole 400mg single tablet tonight and then start her on Syp oflox-nitazoxanide combination. If the diarrhoea is not controlled even then, she might have to get an intestinal biopsy done."
},
{
"id": 86485,
"tgt": "Suggest treatment for lower right abdominal pain",
"src": "Patient: Hi am 25 years old. I been having lower right abdominal pain four the last four months I have taken 6 ct scans that show fluid around my colon and on my right lower abdominal. They did a drainage but couldn t get the fluid. They never tell me what it is and why it s there. Different types of cancers run in my family and am just worried,. Doctor: Hi, read and understood the history of pain in the right lower abdominal pain for which you have undergone 6 CT scans.IT shows fluid around the colon. This finding is very classical of retrocecal appendicitis. I would do a diagnostic laparoscopy in such a situation and am sure to get a disease which may not be seen in any of the investigations like Appendicitis, colitis, diverticulitis and so on. Some of these are curable by surgery and I would do it to give you the cure. Please stop doing further CT as you have already taken a high dose of radiation and with the history of cancers in the family , this wrong thing has already been done.Will also get the routine investigations of the blood, urine and stool for ruling out any other problems and for fitness for surgery."
},
{
"id": 211030,
"tgt": "How to treat anxiety and panic attacks with stomach and chest pain,weak leg and frequent bowel movement?",
"src": "Patient: Hi,I am new to anxiety and panic attacks. I have been seen by many specialists who haven't found a physical cy!!!ause for the symptoms I feel. The feelings come on the same way each time and range from chest pain, stomach pain which leads to frequents bowel movements, then weak legs. Not one of my doctors mentioned anxiety!! Could this be the answer? Doctor: HIThank for asking to HCMI can understand your problem, I actually advise my patient if ever come with such complain and found clinically that this are the functional problems, say anxiety and depression them I ask the patient to come out of this, without the help of medicine, because initially patient can do it , as you said, \"\u00cf am new to anxiety\" such patients easily comes out of this, I advise you too to do the same for that you have to keep your moral high always think positively, have words with the people with open heart if have some dispute with them, you can do it, just try it and then every thing will be alright, take care and have good time."
},
{
"id": 217104,
"tgt": "What causes sudden pain in the right side of abdomen?",
"src": "Patient: 56 yr old male 5'9\", 179 lbs has had this 3 time since Dec. starts with excruciating right sided abdominal pain, referred pain to right shoulder, lateral pain across stomach area, difficulty breathing due to pain, the severe pain last 6-8 hours but remains sore fre 3 days. Fever off & on, GERD, Nausea/vomiting, hiccups are severeHe has had a double hernia repair 2008 laparoscopic & in a severe care crash in November with 3 compressed disc. Did not have symptoms until 6 weeks after accident & the flu followed the first time symptoms were present. Doctor: Hi,thanks for asking HCM!I really understand your introduction, he has billiary colic (severe pain from gallblader) due to infection and gallstone.An abdominal ultrasound and elevated white blood cells will confirm both.Hope my answer was helpfull!"
},
{
"id": 96822,
"tgt": "What does high number of bruises mean in small accidents?",
"src": "Patient: My 8yo son has several bruises over his body after attending rugby league training - there is dark red, purplish coloured bruise with a white centre on the inside of this leg in line with his knee cap. This bruise would measure approx 6 to 8 cms in length and has become increasingly worse over the past 2 days. In addition, he has smaller spot like bruises down the same leg as well as bruising on his chest and shoulder. He has always bruised easily but is this normal? Doctor: The bruises depend on the kind of injury. You should get a thorough blood profile done for bleeding and clotting time and coagulation profile since the injury seems to be lesser than the severity of bruises u just explained."
},
{
"id": 143740,
"tgt": "Is generic of Keppra safer than the real brand of Keppra?",
"src": "Patient: Hello, Is taking generic of Keppra ok rather then taking the real brand of Keppra? Seems like I have to keep increasing the generic med. what does the brand name have in it that the generic doesn t have. Perhaps the ingredients of the generic that are missing is what I need? Problem is the real brand is so expensive. Doctor: HiI am Dr Mittal.I have read your message.I think I can help you.While I cannot really comment on the quality of the generic medicines in my current position, considering your opinion that you need to increase the dose of generic kepra continuously, I would suggest avoiding the generic version.Kepra is an anti epileptic medicine that contains levetiracetam. This is a very important drug and you should not miss the dose or take an inadequate dose because you may develop seizures and this may be devastating depending on the circumstances when you develop the seizures. Besides, brain cells die with every seizure that you develop.So if you feel that the generic version is inadequate, avoid it.I agree with the fact that Kepra is expensive. Again, I would like to withhold the reasons why that is so since that will not gain us anything in the current discussion. However, there are many other brands that are now marketing levetiracetam at a much more affordable prices. The brands will depend on your location (that I am unaware about from your current query). A few brands in INDIA are Levipil, Levefree, Levepsy, Levera, Torleva etc. You can look into your pharmacy as to the other options available and buy a medicine that suits your affordability.If all the brands remain expensive for you and you are not satisfied with the generic drug, I suggest meeting your doctor and discuss starting cheaper antiepileptics like phenytoin, phenobarbitone etc. Of course, every antiepileptic has a fixed use, and so you cannot take any random medicine. So you need to meet your treating doctor and discuss the other options with him/her, so an alternative medication can be started as per the type of seizures you have.I have tried to make it as simple and as useful for you as possible. Please feel free to contact us for more information as per your further specific query to my response. Best of luck, Dr Mittal"
},
{
"id": 8951,
"tgt": "How to enlighten your face ?",
"src": "Patient: Hi sir my name is uma , my face became soo dark. my body in normal fair colour, from 3 mnths iam facing this problem please suggest me any creams iam female 23 yrs and recently married Doctor: Hello Welcome to health care magic Dark complexion is not a curse ,it is a beauty in itself.color of skin don`t matter ,but matters is a texture & glow of skin. Wish you speedy recovery Disclaimer"
},
{
"id": 172072,
"tgt": "What could small bump like area on infants forehead be?",
"src": "Patient: my baby is 12 months old and i just got her out of the bath and she has a small area that looks like a honey comb on her forehead. it isnt a rash but more like bumps the whole area is about the size of a dime. what could it be and should I be worried? Doctor: Is it paining or oozing? Did he hurt himself against the bath-tub edge? I guess it could be anything ... you should show him to a doctor in person. Alternatively, you might want to send me the question with his photos, but that might cost you, so it is up to you.In the meantime, why don't you apply some antibacterial ointment and see if the bump resolves?Dr. Taher"
},
{
"id": 81323,
"tgt": "What does the lung CT report indicate?",
"src": "Patient: I had an abnormal lung ct. it showed three spots on my right upper lobe. I went to a pulmonologist and he has put me on a years prescription of Arcapta 75mg neohaler. he gave me what he thought the diagnoses was but I cant remember the name which starts with an M and he said it was in the family of TB but not contagious and is bacterial. could you name the diagnosis so I can look it up? I just didn t want to bother my doctor. Doctor: Hello Spots in lungs may be due to many reasons like infection,autoimmune causes,malignancy etc.Arcapta 75mg neohaler is a bronchodilator and it is prescribed if there is bronchospasm.Your condition need clinical correlation also.It may be due to infection.It is important to know your age and symptoms.Get well soon.Take CareDr.Indu Bhushan"
},
{
"id": 157999,
"tgt": "Drinking excessively. My stool has blood. Also depressed. Help?",
"src": "Patient: Yes please, I m pretty sure I suffer from depression , when I feel down I drink to excess, although I am a reasonably heavy drinker anyway. Also I do take drugs casually, mostly cocaine but most weekends. Anyway I ve had a problem with going to the toilet for a number two. I seem to need to go very regularly, and often there is blood , lots of blood, not in the stool but seemingly afterwards. And if I sit there it will just drip for ages. I ve just split up with my girlfriend and am drinking to massive excess, I ve not eaten for 4 days and this problem is worse than ever. Im pretty sure it s alcohol related and I know I need to go to the doctor, but can you tell me what you think it is? I m scared, both sets of grandparents died from cancer , also my father. Doctor: I would have liked to know your age and what cancer did your father and grandparents die from. Certain types of cancers of the colon and rectum do have a hereditary component and the risk is further compounded by alcohol abuse and poor dietary habits. The good news is that the type of bleeding you have described is most commonly due to hemorrhoids particularly in the young age group. However, there is only one way to find out whether you have cancer or not and that is to show it a surgeon or cancer surgeon who will examine your anus and rectum in the opd and remove your dilemma."
},
{
"id": 200871,
"tgt": "What relieves the pain in the anal region due to sex?",
"src": "Patient: Please write your query here Dear Doctor...m a gay guy of 24 years old so iv bin havin a problem wth my anas 4 3 months nw.m nt having sex anymore n Im craving now.im from da doctor last month he said its paals n gave a salf to applay.so I mustubate every night. Doctor: Hi, the pain is likely due to the sexual activity.You should avoid anal sex as rightly told by your Doctor. You can indulge in other ways of sexual activity which do not involve penetration which can lead to complications as have been in your case.Masturbation is alright if done once to twice a week. More than that can also lead to problems.All the best and hope you recover soon.Take care,Dr Rishi, New Delhi, India."
},
{
"id": 137718,
"tgt": "Is sciatica causing pain from lower back to the foot?",
"src": "Patient: Thank you, I have pain in my lower back and it goes down my right leg all the way to my foot. I can t stand or walk for more than about one hour when it hurts too much . It does not hurt when I sit down. Is this sciatica and what can I do to make this go away? Aleve helps some time but not always. My leg feels tingling also. Sorry, I do not do any financing on the internet ! I ll figure it out! Doctor: Hi,Thanks for your query.From description it looks like siatica. Sciatica refers to pain that radiates along the path of thesciatic nerve \u2014 which branches from your lower back through your hips and buttocks and down each leg. Typically, sciatica affects only one side of your body. MRI is needed to establish the cause of sciatica, so that appropriate treatment can be prescribed. Sciatica do respond well to bed rest and analgesics (pain killer) and is right mode of treatment. The definitive treatment depends upon the cause of sciatica. I would suggest you to consult an orthopediac surgeon who may order MRI to determine cause of sciatica (level and degree in case of disc herniation) and may suggest other mode of treatment. After initial few days of bed rest, you needs physiotherapy (exercises).I do hope that you have found something helpful and I will be glad to answer any further query.Take care"
},
{
"id": 39519,
"tgt": "What causes inflamed tonsils and bumps on tongue after throat sex?",
"src": "Patient: I preformed deep throat sex on my partner, the next day I felt my tonsils were inflamed. I didn't have a sore throat, it was just my tonsils acting up. I also noticed these red little bumps on the back of the side of my tongue. What could this be?? Should I be worried? Doctor: Having oral sex increases the chances of having the oral & throat infection. The symptoms you telling about such as the inflamed tonsils & red bumps on tongue point towards the infection which might be the result of the throat sex you had.It is advisable to have a physician consultation who will have a look over your oral condition & may prescribe the antibiotics accordingly.You can have home remedies such as gargling with lukewarm normal saline water or with the beta-dine ( povidone iodine ) gargling solution."
},
{
"id": 39475,
"tgt": "How to get rid of facial swelling due to mumps?",
"src": "Patient: Hi! I'm 27 years old guy who had mumps for almost 4days now. My fever is gone last 2days ago. but my face is still swelling a bit. it's it okay to me to have travel vacation? i feel strong now. but i am afraid that my mumps will get worst if go out. some advise. Thanks! Doctor: Thanks for your query.If I were the treating physician,I would suggest you for not to travel and to take rest for 5 days after the appearance of swelling.During this time mumps virus can spread to susceptible persons.It is good to hear that you are feeling better.Do not worry about your swelling.It may last about 10 days.You may apply warm pack over the swollen gland which may be helpful.Hope this answers your query.Wish you good health."
},
{
"id": 71045,
"tgt": "How can severe breathing difficulty be treated?",
"src": "Patient: Very bad cpda breathing issues. Taking symbacort, nabulizer with albutrin an just starting symacort. Cannot get air in lunrrg an d purse breath out happened 4 scary times today wth blood pressure at 189/120 pulse 78 then to 151/102 after med filter 235 mg the 141/83 then 142 then 158/87 and really bad cannot get breath in so scary. Suggest a new breathing med or call doc ang go into hospital. Throbbing head aches, frozen left shoulder can t eat well please any idea/r Doctor: Hello and Welcome to \u2018Ask A Doctor\u2019 service. I have reviewed your query and here is my advice. *As per my clinical experience, these much severe symptoms indicate possible cardiac problem giving intense breathlessness. * I strongly recommend to visit an ER and get necessary work up with EKG, X-ray chest for further intensive management. Hope I have answered your query. Let me know if I can assist you further."
},
{
"id": 72983,
"tgt": "What does this chest CT scan test result for blood in saliva indicate?",
"src": "Patient: Dear sir, Since one week my grand father was bleeding blood from mouth with spit and while coughing.we have gone for treatment and they advised for x Ray and CT scan of chest and abdomen.all the reports are positive.so they are not knowing the reason why it is coming.one impression is there for CT scan,Focal fibriotic strands in left lingular lobe.so please explain me the reason sir. Doctor: Hello dear , hiWelcome to Healthcaremagic.comI have evaluated your query thoroughly .* This seems in relation with infection of the lungs either in form of tuberculosis or else .Hope this clears your query .Regards ."
},
{
"id": 36727,
"tgt": "Suggest treatment for cold shivers and hot sweats",
"src": "Patient: My husband has had shingles for going on six weeks and somedays are good and some not so good. lately he has been waking up at night with cold shivers and then hot sweats. what can be done to make it better? also, one day he feels pretty good..he will go out and do things in the yard and seems to get worse. he went to a doctor within 2 weeks of outbreak and was put on 11 days of anti viral med. Doctor: Hello,I understand your concern.I am Dr. Arun Tank, infectious diseases specialist, answering your query.In my opinion it looks like malaria and dengue.The symptoms you have mentioned is looks like the above two condition.Blood investigation can help further confirmation of diagnosis. Till that time you should continue with doctor's prescription.You can take paracetamol for fever under your doctor's guidance..Please take plenty of water it helps your husband recover faster.I will be happy to answer your further concern, you can ask me on bit.ly/DrArun. Thank you.Dr Arun TankInfectious diseases specialist."
},
{
"id": 21325,
"tgt": "Is surgery needed to cure hole in heart?",
"src": "Patient: hi iam Raju so my wife has hole in her heart that she discovered only 2 months ago. She's 19. She told me that the doctor found out because her bleeding from the mouth, The doctor said that the hole will closed by surgery. All I wanna know is that is this common for under 19years old? how many months after she can do the normal work? and is that surgery helpful? Doctor: Sir this is not normal and hole in the heart can be of various types whether your wife require surgery or not only her echo report and symptoms can decide .She will be fine after surgery can say only after seeing her reports.Most of the repairs are successful.Regards"
},
{
"id": 91359,
"tgt": "What is the cause and treatment for lump and left abdominal pain?",
"src": "Patient: Abdominal Pain my stomach hurts a lot and I ve had diarrhea for the past 2 weeks hair and it really hurts after I go to the bathroom my stomach am I go to the bathroom also I ve had paid left side and it s not the appendix so I don t know what it is. I just know I have had a lump right by my stomach on the left side under the skin and the doctor said not to worry about it unless it get big or starts to hurt. and now I m having pain breathing and my left side abdominal hurt and so does my lower. it also grumbles a lot. I refuse to eat. and I can barley breath. Doctor: HI. The history and findings by you are definitely suggestive of a lump in the left colon. You need an admission Colonoscopy to see and get the biopsy done.This can be a mass. You need a CT evaluation too and further management."
},
{
"id": 200656,
"tgt": "Suggest remedy for sperm leakage in urine",
"src": "Patient: please help!!!!!! iam dying my age is 21, my problem is that sperm leakage in urine.the initial stream of urine is whitish it appears to be dense like sperms. It happens when I eat mutton or any meat+drink milk+fried chicken or drink juice even I ate bannana and offten when I sleep after launch. The urine appears cloudy and turbid.it actually starts two years ago one day I was mastarbating about 2,3 hours but holding cum inside.suddenly I feel I edgaculate it self.I donot mastarbate very frequently but about two years I constantly leakae sperms and pass cloudy urine. I already visits urologist and give multiple tests they always came normal. Any help regarding this problem I am very tense and becoming paranoid.and I am about to commit suicide please help!!!!!! Doctor: Thanks for asking in healthcaremagic forum I understand your concern. Urine if infected may become cloudy. Sometimes semen ejaculated may remain in urethra and can be voided whenever you urinate giving it cloudy appearance. So, you do not have any illness/disease, it is completely normal to have this. Please stop thinking of suicide and instead go for counselling. All the best."
},
{
"id": 159440,
"tgt": "Swollen lymph nodes below the chin after getting tonsils removed. On antibiotics. Cancer chances?",
"src": "Patient: I have had swollen lymph nodes right below my chin on both sides. I went to the ent doctor. they put me on an antibiotic for two weeks and they did not change. I have always had enlarged tonsils so we discussed taking them out. he did not feel the lymph nodes were from a cancer type but he did check down my throat to be sure as I was a smoker for 20+ years. I had the tonsils removed 3 weeks ago and the lymph nodes are still swollen and hard...not changing at all and he said they would change by the first of the year. Doctor: Dear Dcschlichting, Swollen lymph nodes under your chin does not necessarily mean that you have cancer. It could be because of your poor orodental hygiene. If your tonsils were removed, have they been subjected for histopathological examination. Calcified lymph nodes will not get any change inspite of taking variety of anti biotics. in case your physician is suspicious about thease nodes you should get fine needle aspiration from the lymph nodes done. Even direct biopsy of the suspicious pathological node is recomonded as there are so many causes of lymph node enlargement and the main cause can be ruled out. Consult your primary physician about it. Dr. Ticku"
},
{
"id": 32670,
"tgt": "What causes enlarged cervical nodes inside the parotid gland?",
"src": "Patient: Hi Doctor, I've had recurring swelling around my right parotid gland for years. Every time I see a doctor, they tell me it's infection. Recently, I noticed that the swelling doesn't go away at all, no matter what anti biotics I am on. In the past 2 months, docs put me on anti biotics for 5 weeks and realized that the swelling isn't subsiding. So an ultra sound was done, and was told that the duct on the right side is enhanced - twice the size of the left one and has a very irregular shape however there's no stone or calculus. Now my MRI says that there are multiple enlarged superior cervical nodes inside the parotid gland... some measure upto 2 cms. What does this all mean and why are they suggesting biopsy? Doctor: Hi..Welcome to HEALTHCARE MAGIC..I have gone through your query and can understand your concern...As per your complain and presentation of symptoms if you are suffering from recurring infection of the parotid gland and the antibiotics that you are taking is not providing you relief that means the infectious organisms are resistant to the antibiotics or the infection is not bacterial in nature and the antibiotics can only treat bacterial infections..The enlarged cervical lumph nodes are due to infection itself and you need to consult an ENT expert and get evaluated and he can advise you to get a blood investigation as well as saliva investigation done to rule out the causative organisms..Culture and sensitivity testing will also be advised to rule out the exact stain of bacteria or virus causing the infection and treatment can be done targeting the same..Biopsy is commonly advised to rule out the exact cause of swollen lymph nodes which can be benign or malignant in nature..Hope this information helps..Thanks and regards..Dr.Honey Nandwani Arora."
},
{
"id": 219773,
"tgt": "What are the symptoms of potential pregnancy?",
"src": "Patient: I'm 27 yrs old, married for 1.5 yrs. I my menstruation is regular but this time i missed my periods for one week & it shows pregnancy test positive. by abdominal USG it shows only thicken endometrium wall 13mm. ans normal sise & shape. no others abnormality found. so what does it mean? am i pregnant? Doctor: Hello, and I hope I can help you today.So, if you had a positive pregnancy test and did the test correctly, you should be pregnant. However, if there is no gestational sac visible on the ultrasound there are a few possibilities of what may be going on.First, the pregnancy may just be too small and early to see on an ultrasound. That can be normal if your period is only a week late.Secondly, the pregnancy may not be growing a fetus, which is a kind of a miscarriage (called a \"chemical pregnancy\") that gives you a positive test without development of a fetus. Most people with this condition will eventually start bleeding and pass the pregnancy.The third possibility, which is the most concerning, is that your pregnancy is located outside of your uterus. This is called an ectopic pregnancy. The most common site for an ectopic pregnancy is in the fallopian tube. The problem with a tubal pregnancy is that, as the fetus grows, the tube is not designed to expand and it actually can burst, resulting in severe internal bleeding and even a risk of death for the mother.Because of this, it is extremely important that you be closely followed by a physician for the early part of your pregnancy. You first need a blood level of your pregnancy hormone taken and then repeated along with another ultrasound in a few days. The hormone level should double every 48 hours in a normal pregnancy, and furthermore, if the hormone level is high and there is no visible intrauterine pregnancy then it is very suspicious for a tubal pregnancy.Signs and symptoms of an ectopic pregnancy include spotting, abdominal pain (especially if it is only on one side), and sometimes right shoulder pain. Dizziness, shortness of breath and fast heartbeat may indicate internal bleeding. If you have any of these symptoms, you should go to an emergency room for evaluation or call an ambulance. So in summary- when you have a positive pregnancy test with an ultrasound that does not show an intrauterine pregnancy, you need to be ruled out for an ectopic pregnancy. A blood pregnancy hormone level (called a bHCG) should be done first, and depending on the result your doctor will be able to determine what is going on with your pregnancy and determine the best course of action.I hope I was able to adequately answer your question today and that this information was helpful.Best wishes for the rest of the pregnancy,Dr. Brown"
},
{
"id": 962,
"tgt": "Could i be pregnant as i have cramps and headaches?",
"src": "Patient: hello, i have pregnancy related question. i have ovulated a weeek ago and the day after i started having cramps till today as well as fever i fell a bit wet down there and i started having headaches this morning. could i be pregnant or is there something else wrong with me? Doctor: Hi, I think you should wait for the periods. if your periods get delayed, do a urine pregnancy test at home. It will clear your doubt that you are pregnant or not. If positive consult a doctor. If negative wait for your periods. Hope I have answered your question. Regards Dr khushboo"
},
{
"id": 217528,
"tgt": "What causes numbness in lower annuls and down left hip/leg?",
"src": "Patient: I have numbness lower annulus and down left hip/leg to toe big. also have numbness cervical neck. I have had a MRI lower back and a MRI contrast . Im numb at back upper neck. They have told me they have found something however im waiting for may10th to discuss with doctor , Could this be MS Doctor: it could be MS. it could also not be MS and be many more diseases like lumbar canal stenosis or some other disease of the peripheral nervous system. what you need is a thorough clinical examination by a neurologist and tests to figure out your diagnosis"
},
{
"id": 58569,
"tgt": "Bloody stools like dark blood diarrhea, vomiting. History of having cirrhosis of liver, had ultrasound. Serious?",
"src": "Patient: My husband has cirrhosis of the liver. He had an ultrasound last week followed by two days of bloody stool. Today he had an MRI and now has bloody stools again. Stool is like dark bloody diarrhea. He is also vomiting. Is this caused by his cirrhosis or by the test? He sees his liver doctor on Monday. Should he go to the hospital? Doctor: Hello,Thank you for the query.I have gone through your query and understood your concern. I dont think that MRI caused bloody stools as it doesnt causes any radiation exposure lie other tests. But the culprit in your case is liver cirrhosis in my opinion. And hospital visit is always needed as increased blood loss will leads to hypovolemic shock which is not an acceptable condition if the blood loss is no much then it is fine and you can wait for the change but if it is more you need to consult your treating doctor for sure. Thank you."
},
{
"id": 73215,
"tgt": "What causes sickness, fainting, dizziness with chest pressure and heart palpitations?",
"src": "Patient: hi i am 22 years old. i just got sick the begining of january. ever since then i am feeling faint, dizzy i cant stand very long and i feel something is sitting on my chest. a couple weeks ago i had a very rapid heart beat it felt like a mini heart attack. my heart was pounding i got light headed, dizzy, was shaking, got hot and cold flashes. ever since then ive been having heart or chest problems, what is wrong with me? oh and the sick thing i got was only coughing and a very sore throat with a fever. my doc wasnt sure what it was, gave me pills that didnt work. Doctor: Thanks for your question on Healthcare Magic.I can understand your concern. In my opinion, we should first rule out heart diseases for your symptoms. So get done blood pressure monitoring, ecg, 2d echo and Holter monitoring (24 hours continuous recording of Ecg).If all these are normal then no need to worry for heart diseases. Sometimes, stress and anxiety can also cause similar symptoms. So avoid stress and tension, be relax and calm. Consult psychiatrist and get done counselling sessions. Try to identify stressor in your life and start working on its solution. You may need anxiolytic drugs too.Don't worry, you will be alright with all these. Hope I have solved your query. I will be happy to help you further. Wish you good health. Thanks."
},
{
"id": 60033,
"tgt": "Member of the family has history of hepatitis C, currently PCR, hepatitis negative. Will exposure to his wounds transmit virus to me?",
"src": "Patient: Hi Doctors, I have a very important question. My father was infected with HEP C 3 yrs ago during his bypass surgery .He was tested for HEP C just after a month and it came positive it was *acute* hep c and was treated with 6 months treatment.he cleared it and now is clean for about 2 yrs .he tests for HEP C every yr now PCR HEP C and hes negative! .My question is for example if anyone in our household get exposed to his blood like into a wound or anything with our blood .are we going to get the virus? even thou he doesnt havea virus anymore but his antibody test would always show positive tht means he will hve the antibodies positive so if someone is exposed or had hep c PCR negative persons blood transmitted somehow tht person test results would come antibodies positve too or negative? and why does he still have positive on the antibody test but negative on PCR? I get myself and my family tested every other 6-7 months we are negative but I am still afraid If I can get this disease by any means and If i do get exposed to his bloodto my blood ..Am i gona have antibody hep c positive too? im so worried Doctor: Hi, Thanks for using the HCM. Hepatitis C virus infection is mainly transmitted through parenteral route, and its a chronic condition, by taking medications we can control further multiplication of virus, can control viral load under control, antibodies will remain positive, antibodies are bodies response to virus, there are chances of viral transmission through blood, you need to take precaution while handling his wound, need to wear gloves, separate and proper disposal of wound dressing wastes,hand washing with antiseptics, by taking all precautions you can avoid viral transmission Hope I have answered your query. If you have any further questions I will be happy to help. Thanks get well soon...."
},
{
"id": 199897,
"tgt": "Any suggestion for premature ejaculation?",
"src": "Patient: hi, im having a little bit erection problems and premature ejaculation sometimes. im 30 years old and this is happening since last year so i feel a big difference in my sexual life. also i had balantis infection which sorted out last year but it appeared to be back recently. please help me. supa Doctor: HIWell come to HCMIn most of the cases such complaints of erection and premature ejaculation are functional in origin and may not be due to any disease, over conciousness, apprehensiveness, depression, anxiety, may be the causes of premature ejaculation and erection problem try to come out this conditions, try some exercise, yoga, relaxation therapy, everything would be fine soon, take care."
},
{
"id": 127634,
"tgt": "What causes pain in the joints despite a normal CT scan?",
"src": "Patient: Hi, My name is Geneva. I have been having severe pain for about a week now. I went to the emergency room on Sunday, July 9th,2017. they did 2 series of blood work, an EKG, and a CT Scan. All test came up clean. they didn t know what else to do for me, so they told me to follow up with my PCP within a week, which I did, but they couldn t get me in before the 24th of July. I realized that the pain I m experiencing is in all the pressure points and joint areas that the diagram shows online, such as : the baseline of the head, the neck , shoulders, back, the elbows, etc. My pain is explosive at the base of my head on the left, my neck, shoulder, back, the back of my left arm, and on the right side of my body , it is painful in my shoulder blade, and under my lung. Should I be checked for Fibromyalgia ? Doctor: Hello and Welcome to \u2018Ask A Doctor\u2019 service. I have reviewed your query and here is my advice. Fibromyalgia is a clinical diagnosis,no testing would diagnose it. Get tested for gouty arthritis, get your serum uric acid done. Alternatively get your Ra factor ,ANA ,Anti CCP done. Hope I have answered your query. Let me know if I can assist you further."
},
{
"id": 166514,
"tgt": "Suggest medication for an ear infection",
"src": "Patient: My grand daughter had ear infection5 times for last 6months.. she has been given antibiotic(Amoxicline, azithromycin) each time when she got infection... Doctor advised that ear tube has to be inserted, so that the fluid accumulated in the ear drum can be drained out...My grand daughter age is 1year and 6 months .. Please advise.. Doctor: Dear parent, the most common cause of recurrent ear infection is enlarged adenoids . enlarged adenoids can be a site of Focal sepsis that spread to the ears leading to. recurrent ear infection. this is an indication of removing the adenoids using surgery to avoid further complications . I recommend you start with removing adenoids before inserting ear tubes"
},
{
"id": 9431,
"tgt": "Suggest treatment for dryness and swelling in upper lip",
"src": "Patient: Every morning for the past 5 days or so my top lip has been very swollen and chapped. I use chap stick and it helps with the dryness but the swelling is still there. I even bought a new chap stick and no change. My lower lip is not swelling, just the top lip in the center. Doctor: HIThank for asking to HCMI really appreciate your concern and looking to the history given here I could say that this could be hypersensitivity reaction and stop using the stick, better Beclomethasone cream, this would give nice result, hope this information helps you, take care and have a nice day,"
},
{
"id": 147207,
"tgt": "Neck, shoulder, chest pain, cervical spondylitis. MRI scan done. Treatable with calcium supplements?",
"src": "Patient: hello sir, Medical history- my mom is suffering frm cervical spondylitis , doctor suggested her to put cervical traction and at once MRI scan was also taken through which we diagnosed the stage of disease-it was 25% doctor decided but this was 2yrs before no such recent activity was done. When she works more she will be getting more pain at backside of neck and sometimes it even reaches to shoulder and chest.......she cant sit in an normal chair continuously........ Now my query is tat she has been used a lot of pain killers, which she is even using now along with some strength giving capsules like BONACT- calcium carbonate- API...........is this not curable she should use tablets for lifelong.........i want her to use calcium supplements like-Calcium Sandoz woman and horlicks woman ............and such other things so that she can avoid the usage of tablets.........what do u say????????? Doctor: She shd do back strengthening exercises.take cap rejunuron od daily once life long.and also with calcium vitamin d 60 k tab once a week for 8weeks"
},
{
"id": 10067,
"tgt": "Suggest remedies for hair fall",
"src": "Patient: Good Day, I desperately need your help please. I use to have healthy long hair but that changed in the past year when I started loosing a considerable amount of hair...and I mean loads of it. Now my hairline is non-existent and my central hair area has hairless patches due to hair loss. This has been causing me a lot of stress and have no idea what to do! I have consulted a homeopathic doctor and he gave me R89...said I should take 15 drops (in my mouth) 3 time a day before meals...please advise if this is an ideal medicine and which other over the counter supplements, foods & shampoo/moisturizers can I take/use to speed up the hair loss remedy process. I am an African lady, 31y/o, with naturally afro hair (well, the little that s left of it). Also, I have been on low carbs/no sugar & salt diet and have been taking Duromines for the past year...is any of the a catalyst in this hair loss problem? Please help...this problem is starting to affect me badly. Kind Regards, Balding Anonymous Doctor: Hi, You seem to suffer from female pattern baldness. Consult the dermatologist for the perfect diagnosis and proper treatment. I would advise you to apply minoxidil 2% solution twice a day for a few years. Take biotin tablet 10 mg daily. Cap vitamin E 400 mg daily may be taken. Keep hygiene of scalp good by cleansing the scalp with ketoconazole-containing shampoo. Keep the mood relaxed. Have a healthy diet. Hope I have answered your query. Let me know if I can assist you further. Take care Regards, Dr Ilyas Patel, Dermatologist"
},
{
"id": 46987,
"tgt": "Can Addison's disease cause a spot on the lung?",
"src": "Patient: My husband is 73 years old and has addison disease. He was just told he has prostate cancer. The doctor took a two chest xrays and sees a spot on his lung. Tomorrow he is having a cat scan. Can this spot on his lung b e caused by his addison disease? Doctor: Hi thanks for posting in HCM I have understood your concern 1.addisons disease is adrenal cortex disease with low levels of steroids and needs steroid supplement. it is not concerned to the chest x ray patches.2.chest x ray showing patches mostly because of metastasis of prostate cancer.I can suggest you to get done 1.LFT to check for liver involvement 2.serum creatinine3. serum electrolytes 4.CT scan of abdomen to look for metastasis 5.PET CT is better one to check for metastasis.depending upon the stage of the cancer he. can be subjected to surgery in initial stage , chemo and radiotherapy in advanced stages.hope this helps you any further questions please let me know thanks"
},
{
"id": 333,
"tgt": "Can i get pregnant if suffering from polycystic ovary?",
"src": "Patient: Hi, I am 29 years old and where trying to have a baby for almost 4 years. My periods will only get normalize when i take duphaston but when i stop I go back to the same problem. I have been taking duphaston and clomiphne for almost a year but I suddenly stop 4 months ago coz it seems that it has no effect to me. And now, I got an ultrasound with my gyne and result was, I have a polycystic right ovary, cystic follicle at the left, slightly thickened endometrium and with intramural myoma at the upper posterior wall measuring 2.2 x 2 cm. What is the best medication for this and should I still have a chance to get pregnant? Is my condition worse? I am badly needed your advise. Thanks Doctor: u can get pregnant ...u should go for gonadotropins wid clomiphene/letroze ...n do regular ovulation study til follicle reach 18 mm..under guidance of gynecologist"
},
{
"id": 12999,
"tgt": "How to get rid of painful rash on butt cheek?",
"src": "Patient: My daughter (14 yrs. old) is doing detasseling and now has a seriously painful rash running down the inside of one of her butt cheeks (crack area). It hurts to walk or sit or run. She has been using gold bond medicated powder but we are wondering if there is something else she can do? Doctor: Hi, I would recommend you to apply topical stress and antibiotic like Neosporin ointment and Cortisone cream on the affected areas twice daily for 10 days. This will improve her condition. Hope I have answered your query. Let me know if I can assist you further. Regards, Dr. Asmeet Kaur Sawhney, Dermatologist"
},
{
"id": 175722,
"tgt": "What causes change in behavioral pattern of child?",
"src": "Patient: Hi, my son is 3 years old and has been potty trained for about 3 months. He was doing great at first. No accidents, now, he wets his pants all the time. From 1 to 3 times a day. He doesn t even care about being wet, if I don t notice it immediately, he will just sit in it. Any suggestions as to if this might be behavioral or medical? Doctor: DearWelcome to HCMWe understand your concernsI went through your details. I can understand your emotions. Toilet training is a delicate issue. Too much rigid or too lazy training has its future bad behavior problems. Therefore, you should tread your course carefully. Reward and good behavior and simple punishment for wrong behavior were always used to cultivate good behavior patterns. Toilet training should also be undertaken with that tempo. I also think, you, as a mother, is busy to take care of your child's toilet training properly. In that case, set alarms every two hours, take your child to toilet, compel him to pee. Continue this even during night. Take this pain and within a week he should be alright.If you require more of my help in this aspect, please use this URL. http://goo.gl/aYW2pR. Make sure that you include every minute details possible. Hope this answers your query. Available for further clarifications.Good luck."
},
{
"id": 20128,
"tgt": "What could ejection fraction of 20% suggest?",
"src": "Patient: i had a heart catheterization done yesterday, with the reult of a ejection fraction of 20% and 2 stenosis of 70%, one year ago after a heart attack, the catheterization showed an ejection fraction of 40%, one stenosis of 50%, how is this possible, i exercise and i am on a strict diat Doctor: Welcome and thank you for asking on HCM!I have gone through you question carefully. Presence of stenosis in heart vessels (CAD) causes decrease in heart function (decrease EF). Treating the blocks by stenting or surgery can result in improvement in blood supply to heart and ejection fraction too.Hope you will find this answer helpful!Wishing all the best, Dr. D Sunil Reddy"
},
{
"id": 149577,
"tgt": "Precautions while taking anti- seizure medicine? Are old people more prone to seizure? How to reduce them?",
"src": "Patient: Hi Can you tell me what patients should definitely avoid taking or doing, while taking an anti-seizure medication? If a person gets seizures during the night while they are sleeping can this be more dangerous? As a person gets older, do they end up having more seizures? Are seizures in adult patients (40+) common? What could have caused the seizures to suddenly come out? What can be done to reduce or stop the number of seizures. Doctor: HI Thank for choosing HCMSeizures is symptom and not the disease it self, and this could happened due to the abnormal electrical activity of neurons inside the brain, often called convulsion in medical terms, It can be Epileptic convulsion, convulsion due to potassium, sodium disturbances (either Grandmal or petit mal) Febrile Convulsion (Pediatric Age group) Brain infection (Meningitis) also induces convulsion, the most common type of convulsion is generalized tonic clonic one (epilepsy), this can be occur at any age groups more common in male patients, newer drugs are available which gives very good results, all convulsions are being treated according to under laying cause, for epileptic seizures life long medicine required, ( Natural remission of seizures often noticed) Hope this will help you. good day."
},
{
"id": 43939,
"tgt": "Taking Clomid. Positive pregnancy test. What does a 54 HCG test result really mean?",
"src": "Patient: I have been on Clomid the last 6 months and this month I finally have a positive preg test. I have been taking 150mg of Clomid the last two months. I had blood work done today 29 days after LMP and my HCG level is 54. My ovulation usually occurs about the 16th day of my cycle. I am a little later than the average woman - according to what I have been told. I read on the internet today and my HCG level of 54 concerns me. I will have another blood test in 2 days. What does the 54 HCG test result really mean? Doctor: HELLO, well , you have done a pregnacy test too earlier.This is a low level HCG, and it could be a early pregnancy. Please repeat after 2 days . The level needs to be more than double in normal intrauterine pregnancy.If it doesn't, then it is a cause of little concern. TAKE CARE"
},
{
"id": 186408,
"tgt": "Any suggestion for swelling and painful gums when having limited mouth opening?",
"src": "Patient: Hi I was chewing guthka due to which I've limited mouth opening. However I've quit the same.Now I've a problem with my wisdom teeth as they are not alligned properly for which I am suffering from swelling gums and pain. Now I want them to be surgically removed but my limited mouth opening is creating problems. Kindly suggest Doctor: Hello, Welcome Thanks for consulting HCM, I have gone through your query, as you have reduced mouth opening due to chewing habit of tobacco , this can be due to fibrosis in buccal mucosa . This condition Oral Submucous fibrosis is treated by medicine and surgical removal of fibrous band You can do Mouth opening excercises by consulting your dentist . Apply ointment Tess on cheek twice daily for 15 days. One Tablet vitamin A once daily for 15 days .One Tablet Antioxidant twice daily .Consult dentist for excercises and oral examinationHope this will help you."
},
{
"id": 174615,
"tgt": "How to deal with spasm in the arm of a one year old?",
"src": "Patient: My one year old has been having what I can only describe as a spasm in his right arm throughout each day for about a week now. The first time he did it, I thought it was an \"I dunno\" kind of one-shoulder shrug but he continued to do it without response to anything. Doctor: Hi...by what you quote I feel this is a tic or a behavioral issue which is developing. But the other possibility could be a stiffness of the particular arm due to a neurological deficit. I suggest you consult your pediatrician or pediatric neurologist for this.Regards - Dr. Sumanth"
},
{
"id": 91530,
"tgt": "What could have led to vomiting sensation along with stomach cramps?",
"src": "Patient: Hi, My boyfriend has been having stomach pains the past few days, he thought he ate something bad, but now hes unsure. He hasn t really been eating the past few days because the food doesn t seem like it wants to stay down. He feels like he may want to throw up but doesn t, its like he may want to use the bathroom but doesn t really get the urge Doctor: Hi ! Good morning. I am Dr Shareef answering your query.If I were your physician, I would advise your boyfriend for a general clinical examination of the abdomen, routine stool test for ova and cyst, a liver function test, and an ultrasound abdomen if considered essential by depending on the clinical and investigative finding. Till that I would advise him some anti emetic, a proton pump inhibitor, a probiotic, and an anti spasmodic if need be. He should also go for a high fiber diet and enough liquids if tolerated by him and completely avoid food from outside. Further management would depend on the clinical findings and investigation reports.I hope this information would help you in discussing with your family physician/treating doctor in further management of your problem. Please do not hesitate to ask in case of any further doubts.Thanks for choosing health care magic to clear doubts on your health problems. Wishing you an early recovery. Dr Shareef."
},
{
"id": 59222,
"tgt": "High bilirubin, SGPT and SGOT levels. Is this serious?",
"src": "Patient: My sister was detected with 6 bilirubin(TOTAL) 1 month ago... with 500 SGPT and 200 SGOT.... Which came down to 3 Bilirubin (T), and 67 SGOT, 119 SPT.. But Now when I tested today.. although the Bilirubin TotaL was 1.2, and direct bilirubin was 0.74, but the SGPT has increased to 135, and SGOT 117.. which increased from the previous check.. Is it a reason to worry?? Doctor: Hello , thanks for \"Q\" to H. C. M.C. High serum billirubin level is associated with some liver ailment i.e. jaundice ( many types). S G O T & S G P T are around 40 but there significance is not much important in jaundice as far as concerned. However their importance is in some heart ailment. So once biillirubin has been under control so in this case please don't bother. Good luck Dr. HET"
},
{
"id": 159813,
"tgt": "What should i do as i am having a number of health conditions after removal of a schwannoma tumor?",
"src": "Patient: Hi, I m a 26 year old female, who has suffered with a number of health conditions in the past. Two years ago I had a large (20cm) benign schwannoma tumour removed surgically from a nerve behind my lung , and I also had blood clots across both lungs, which I was treated with warfarin for for 6 months. I have for the last 4 years been suffering from right lower sided abdominal pain , for which I still don t have a diagnosis. The pain varies from mild to occasionally very severe. My GP is of the belief I have endometriosis , but unfortunately I cannot have a laparoscopy as I am too overweight. I suffer from fatigue regularly, and regularly pick up chest infections and water infections. Just recently (one month ago) I had what my Doctor believed to be a kidney infection, and had a persistant high temperature on 3 different occasions. Following antibiotics and paracetamol it didn t bring down my temperature for 3 weeks. My GP now also considers that this wasn t a kidney infection, but I often have microscopic blood in my urine. The problem that I m having now is that my legs are aching horribly, and I have unexplained bruising on my arms and legs, as well as a chesty, wheezy cough. The pain in my legs is so bad that I cannot pull myself up off the bed/chair. I am suffering fatigue badly at the moment and I also have mild asthma which isn t helping the cough. My worry after having suffered from blood clots is that this may be a blood disorder. I also suffered from a nose bleed about 5 days ago, and it s very rare for me to have them, the last one must have been over 10 years ago. I do get a bit nervous when I am unwell, as when I was diagnosed with a tumour I spent about 4 weeks unable to breathe and with chest pain (and also had calf pain) which went unnoticed by doctors, who just thought it was a chest infection and gave me antibiotics. I m worried about something being missed again. Any advice you can give me would be greatly appreciated. Thank you. Doctor: DEAR VICKY DONOT BE NERVOUS AND LOOSE YOUR SATISFICATION WITH TREATMENT,YOU SHOULD GET ADMITTED IN A CLINIC AND A THOROGH BLOOD INVESTIGATIONS,ULTRASOUND AND CT SCAN EXAMINATION OF ABDOMEN ADVISED FOR A FINAL DIAGNOSIS"
},
{
"id": 127832,
"tgt": "Suggest treatment for chest soreness post an injury",
"src": "Patient: I wrecked my car over two weeks ago my chest were the seat belt crosses your chest is still really sore , you can push on my stermun not really sore can t find any real sore spots but my chest really hurts I don t have any trouble breathing I can take deep breaths and it doesn t really hurt just wondering why it is still so sore it hurts to do any thing at all Doctor: Hello,You have suffered a blunt chest injury. As you have no trouble breathing it's fine. The pain and discomfort will go in six weeks. All you require is to take it easy for the next few weeks. If you have trouble breathing or if the pain suddenly increases, then you can consult your physician.Hope I have answered your query. Let me know if I can assist you further.Regards, Dr. Santosh S Jeevannavar"
},
{
"id": 207965,
"tgt": "What causes slime like substance on tongue and white colour on the edges of the tongue?",
"src": "Patient: My mother is in the late stages of dementia she has for the last three weeks had a slime like substance on her tongue,it is also a white colour on the edges. Her lips also were swollen and very sore it really does look horrific .What do you think it could be? Doctor: HiIt could be anything.May be a part of drug therapy or nutritional deficiency.It would better to consult your treating doctors and get advise.Do symptomatic treatment like Vitamin b complex and folic acid.If it is swallowen then apply chlorihexidine gel over it.But if there is difficulty in eating then consult a doctor.It is difficult to give advise without examine it properly.Thank you."
},
{
"id": 121309,
"tgt": "What could cause right sided pain?",
"src": "Patient: Hi Doc, well my right side in the front was hurting me really bad and the pain just started last night at work. We I stood up fro my chair I had this terrible pain in my side, and I was wondering could it be my appendix. Right now I am deployed in Aghanistan so could you please give me some advice. Doctor: Hello,If you have fever and persistent pain in this region, going to the ER is necessary for a physical exam, an abdominal ultrasound and some blood lab tests. Otherwise, if you do not have fever, I recommend taking an antispasmodic drug (buscopan, dicyclomine, etc).Hope I have answered your query. Let me know if I can assist you further. Regards, Dr. Ilir Sharka, Cardiologist"
},
{
"id": 69591,
"tgt": "Can lumps growing near my fathers ear be a cancer or Tumor?",
"src": "Patient: good evening sir my father is habituated to use tobacco to clean his teeth daily , his age is 70 + , few weeks back a little lumps is growing might be tumour we guess. today it burst automatically we have been consulting with doctors but they are not sure whether it's cancer or any other deases. i am very worried please suggest any better way outmy email address is YYYY@YYYY sorry i forgot to mention the area where the lumps was growing , it's near area behind right ear Doctor: Hi ! Good evening. I am Dr Shareef answering your query.Even though it is good to be concerned about one's health, one should not become frightened or obsessive on cancer for a growth of few weeks duration only without a clinical examination by doctor, and the related investigations if any. Therefore, I would advise you to get your father examined by your family physician, who after a general examination might put you on a short course of broad spectrum antibiotic and anti inflammatory drug. If it was of an inflammatory aetiology, the swelling might subside. Otherwise, you might have to take appointment with a general surgeon in your area to get your father assessed and managed properly.I hope this information would help you in discussing with your family physician/treating doctor in further management of your problem. Please do not hesitate to ask in case of any further doubts.Thanks for choosing health care magic to clear doubts on your health problems. Wishing you an early recovery. Dr Shareef"
},
{
"id": 96649,
"tgt": "Suggest treatment for dizziness, headache and bruising around temple after injury",
"src": "Patient: Hi I had a bit of a clash in playing netball tonight!! I hit my temple quite hard and felt a little dizzy for a few minitues. I continued to play and it was okay just a little tender!! I have a slight headache and bruising around the temple - should I be worried or get this checked out and what symptoms should I look for if there would be any internal damage!! My brother was in a axe accident and became permanently brain damaged so I know how delicate I can be which may be making me paranoid, I am struggling to sleep over thinking so would appreciate some reassurance or advice - thanks Doctor: Hello, I understand that you are worried about this issue. I would advise that you visit your doctor today or tomorrow and get this problem checked out. Seeing as you are having a headache, it is always better to be on the safe side. There is nothing to be paranoid about though, do not worry. Most likely it is a concussion but even if your doctor finds a contusion or some amount of bleeding in the cranium, he would be sure to treat you well, and the condition is very amenable to treatment, without long term sequelae in the vast majority of cases. The symptoms that you should look out for : loss of consciousness, vomiting, unbearable headache, difficulty with your vision. It is always good to know what symptoms to look out for, however it is best to visit your doctor and be on the safe side. All the best."
},
{
"id": 177026,
"tgt": "Suggest remedy for itchy and swollen eyes and lips after eating walnuts",
"src": "Patient: My seven (7) year old daughter was baking with me and ate some walnuts. Her tongue itched her and then her throat and her lip swelled up. Now her right eye area is very swollen. Her left eye is also starting to itch. Should I take her to the hospital or get some over the counter product at my local pharmacy? Doctor: Hello,Thank you for asking at HCM.I went through your daughter's history and would like to make suggestions for her as follows:1. If she has only symptoms limited to eye and lips, I would suggest her to take an antihistamine like hydroxyzine or diphenhydramine. 2. However, if symptoms progress, then there is a risk that she might have swelling in throat and airways, and in that case she might have difficulty in breathing too.So were I treating her, I would suggest you to give her above suggested antihistamine at home and then take her to nearby hospital so further treatment can be done in case her swelling progresses beyond throat. In that case, she might require some injections and steroids also.3. In future, I would suggest her to avoid eating walnut as well as any product that contains walnut until her allergy to walnut is confirmed or ruled out by an Allergist-Immunologist.Hope above suggestions will be helpful to you.Should you have any further query, please feel free to ask at HCM.Wish your daughter the best of the health ahead and a very good recovery.Thank you & Regards."
},
{
"id": 179492,
"tgt": "Suggest treatment for constipation of an infant",
"src": "Patient: Severe constipation for my 11 months old baby after giving nan pro 2...... pls advice what to Do? He z bleeding while passing hard stone like stools...... tried fruits veg water juices etc..... have fever and cold too...... pls help....can t see him crying ... Doctor: Thanks for the query.Do not worry, it is normal to have such problems in infants.Continue lot of water , fruits, veggies.Try some exersices like massaging the feet from lower side to upword.Cold and cough may be due to viral illness.Use nasal saline drops for decongestion.Paracetamol for fever.Hopei help you."
},
{
"id": 12550,
"tgt": "Red spots on back of hand due to bug bites. Solution?",
"src": "Patient: Hi this morning I woke up and I find 3 red spots on the back of my hand ... They aren t bumps, they don t itch they,and all three are pretty much same in size and they are alittle smaller then a eraser on a pencil ... I m 14 male and my bed has no bedframe so it s on the floor so I think it might be bug bites I got from bugs who crawled in my bed but they just appered so thank you for your time and I hope you can help me And if u need more details ask me Gmail: YYYY@YYYY Doctor: hi dear happy new year, it might be a case of bug bite. but bug bite usually itches. its surprising that u dont have itching. regarding treatment use calasoft lotion, this will help u. also u can apply lotion sarna before sleeping, this ll protect u from bites. also try to find some solution to bed bugs. they can be very harmful."
},
{
"id": 26154,
"tgt": "What could cause high BP?",
"src": "Patient: Possible Drug Reaction I just found out yesterday I have High Blood pressure. I was given a prescription and felt fine. I took it again today and now I am having occasional breathing problems and my blood pressure is high and my pulse rate is low. What should I do? Doctor: Hello, I have gone through your query.Thanks for using for using HCM. Basically i should know what was the level of blood pressure when first time detected to be high.I also need to know what medicines were prescribed. I can guid only after knowing this detail. your should give some time to medicines to stabilise the pressure.You low heart rate might be due to Beta blockers if given to you,but it is not cause of worry. At this stage i can only suggest take regular medicine ,monitor BP and maintain contact with your treating doctor, My Best wishes Dr.Rajesh Teli,MD."
},
{
"id": 87466,
"tgt": "Suggest treatment for pain in the lower abdomen",
"src": "Patient: Hello Doctor, I recently suffered from Jaundice (Hepatitis A) was under treatment from 26 August 2014 to 4 September 2014. The SGPT count was increased to 10.25 now it has reduced to 2.8(tested on 13 September 2014). I rejoined my office from 11 September 2014. Although I feel I am recovered from Jaundice but from 14 September 2014 I am experiencing pains in my right side of abdominal below ribs and the pain extends till back. Now the intensity of pain has increased. One more point to note is the pain is not continues, the pain occurs only when I make movements especially after driving bike Is it due to lack of rest or is it some other problem? Doctor: It could be related to your previous hepatitis. Although hepatitis A doesn't lead to chronicity, but incomplete recovery or reinfection cannot be ruled out. You should get a ct scan of abdomen to check around the hepatobiliary system as well as repeat all the liver function tests required for evaluation. But foremost thing, do not delay and visit your doctor. Certain pain relievers like drotaverine for liver related pain and hyoscine for gall bladder related pain are only symptomatic treatmennts. You should have a complete work up done."
},
{
"id": 209628,
"tgt": "Suggest medicines for depression and anxiety without any side effects",
"src": "Patient: Hi, I started the maoi marplan 4 days ago at a dose of 10mgs a day this is the second time I ve restarted this drug and around the 4th or 5th day I notice my skin has a some red discoloration around my face is this common? What should I do I get side effects with every antidepressant and have been struggling with severe anxiety and depression for a long time now... Doctor: Hello,Thanks for choosing health care magic for posting your query.I have gone through your question in detail and I can understand what you are going through.You may consider fluoxetine or escitalopram or mirtazapine. All these medicines are safe and are useful for treating depression and anxiety.Hope I am able to answer your concerns.If you have any further query, I would be glad to help you.In future if you wish to contact me directly, you can use the below mentioned link:bit.ly/dr-srikanth-reddy\u00a0\u00a0\u00a0\u00a0\u00a0\u00a0\u00a0\u00a0\u00a0\u00a0\u00a0\u00a0\u00a0\u00a0\u00a0\u00a0\u00a0\u00a0\u00a0\u00a0\u00a0\u00a0\u00a0\u00a0\u00a0\u00a0\u00a0\u00a0\u00a0\u00a0\u00a0\u00a0\u00a0\u00a0\u00a0\u00a0\u00a0\u00a0\u00a0\u00a0"
},
{
"id": 25454,
"tgt": "Suggest treatment to reduce hypertension",
"src": "Patient: Hi , I have 27 years old. But I have hypertension from past 6years onwards. Blood pressure ratings are continuing 145/90, iam using AtenAm and NatrilicSR tabs. Our doctors were all tests reports checked and finally they told there is no problem in reports. But we require to investigation where the problem happend to raising the Blood presures So please give suggestions to reduce hypertension. Doctor: Hi Thanks for writing to HCMDoctors probably investigated to rule out and secondary cause of your high BP considering you age. But as you said all tests were so that's a welcome news.For controlling BP apart from Medicine you can switch to low salt diet, regular exercise, eating fresh fruits, avoid fried junk foods.Dr. Pinak"
},
{
"id": 198194,
"tgt": "Any suggestion for high glucose content discovered in semen analysis?",
"src": "Patient: I went for a semen analysis and these were the results: Progressive - 13%, Non-progressive - 4%, Total Motile - 17%, Volume - 3.9ml, Sperm Count - 66 million. We have been trying to have a baby for several years now. I am 44 years old, quit smoking 10 years ago and rarely drinks. I recently discovered that I have high Glucose content. Thank you for your assistance. Joe Doctor: Your semen analysis is almost normal except that motility is low. It may be due to some problems inside testes or the sperm conducting passage. You need to confirm your blood glucose level first. Then depending on the fertility factors of your wife. we can plan further therapy."
},
{
"id": 2822,
"tgt": "Am I pregnant if farting, having delayed periods, sore breasts and vaginal discharge?",
"src": "Patient: i am suspecting that i am pregnant my period was due on saturday but still has not come, iam also farting a lot, my breast are very sore especially at night and i am producing a lot of vaginal discharge as if i am ovulating. I did not want to do any pregnancy test this ealy, what do you think? Doctor: Hi, as you have crossed your due date you should do a urine pregnancy test, as symptoms you are telling can be pregnancy symptoms also."
},
{
"id": 147627,
"tgt": "What does mild polypoid mucosal disease in the floor of maxillary sinus mean?",
"src": "Patient: Hi I started loosing tasting food I taste only spicy food and mild garlic tastes..my doctor asked to do MRI for brain and it resulted like that ... There is mild polypoid mucosal disease involving the floor of the maxillary sinuses.. What does it mean.. Am so panic please help..thanks Drss... Doctor: Hi, first off DONT PANIC.Let me see if I can give you some basic information that will put you at ease and help you develop you game plan on how to deal with it.Your nose is a pretty amazing organ. It's not only responsible for the obvious stuff like breathing and smell. It also is very important in filtering out the nasty dust particles and infections that are in the air. It also protects the lungs by moistening and warming the air that you breath in.The maxillary sinuses are naturally occurring air spaces that help warm and moisten the air as you breath and they help your voice carry by resonating when you speak. The thing is... those sinuses are lined with layers of what we call 'MUCOSAL TISSUE'. This mucosal tissue is there basically to keep the sinus clean and make sure no gunk builds up inside of it. What the report seems to indicate is that some of the mucosal tissue in the sinuses has started overgrowing a bit and forming into tiny bumps called polyps. Those polyps normally form when you have allergies or when you have alot of sinus infections. When those polyps grow into the nose , they can mess with your sense of smell.Most of the time the polyps can be dealt with by dealing with the underlying cause (if you've got allergies, treat those, if it's infections, antibiotics, etc). Occasionally you need surgery on the polyps that can remove them. The surgery is done on the inside of the nose using endoscopes and is called a Functional Endoscopic Sinus Surgery (FESS). But that's rare.Overall, I think your best plan here is to take this report back to the ENT and see if they can prescribe you a set of medications that will hopefully make the polyps shrink a bit, or at least stop growing. If that doesn't work, the surgery might be whats needed. I hope this helps. You can get in touch with one of our ENT's through our \"Ask a Specialist\" service. we will get back to you right away.Thanks for coming to HealthCareMagicVin"
},
{
"id": 10558,
"tgt": "Suggest remedy to prevent itching on scalp and hair loss",
"src": "Patient: Hi, My problem is hair loss.From quite sometime I am having hair loss and now whenevery I comb, wash hair, do oiling its hair and hair all over. And sometimes i feely itching my scalp too. Let me know whats the probable reason and should I counsult Dr. ?? Doctor: hi there.1. your concern is telogen effluvium along with seborrhoeic dermatitis most probably.2. you can try anaboom serum once daily bed time, gently massage into the scalp.3. use sebamed anti dandruff shampoo weekly twice with soft drinking water.4. prefer diet rich in proteins like egg.this will help you in controlling hair fall and itching in the scalp."
},
{
"id": 178568,
"tgt": "Could ingestion of water by a toddler be a matter of concern?",
"src": "Patient: I'm just concerned.My toddler fell into our pond today and within seconds I jumped in a grabbed him.My mother was with us and she said I was so fast that I jumped in,grabbed the baby and had him out of the water before she even knew what happened.He coughed once and began crying then I immediately took him and we both jumped in the shower and he continued to cry.He settled down by the time we got out of the shower and then fell asleep,but he did miss him nap time so he was tired.I woke him up early cause I was concerned about this 'dry drowning' or 'delayed drowning'I was reading about.Should I be concerned? He is acting fine and normal now.He ate and is playing and acting fine.There was no continuing coughing or chocking.Reading about this has got me concerned about..24 hours later people have died and they were acting fine prior to ingesting water. Doctor: Thanks for putting up your query at HealthCareMagic. I have gone through the details carefully and do not think that it is a cause of your concern. You may relax. Probably the child was crying because he was not prepared for the sudden situation. Even if the child has ingested a bit of water (if any), all that can happen is some amount of diarrhoea or vomiting. In case it happens, you need to manage it using ORS. I do not see any other major issue. Do not be bothered by jargon like dry 'drowning' or 'delayed drowning'. I do not think that this can be the situation in the case of your child. But take the child to a pediatrician if you see any abnormal behaviour.I hope that helps. Feel free to revert back with further queries if any."
},
{
"id": 64558,
"tgt": "What causes lump and bruise in cheek months after injury?",
"src": "Patient: I fell down the stair over eight months ago on November 2nd, 2009. After falling down the stairs i went to the doctor to see if anything was broken and it was not. I had a large bruise and swelling on my left cheek because that is where i took the hardest part of the fall. However, more than eight months later there is still a bulge where there was once a bruise and in most areas of the lump it is amost numb, but if you poke it in just the right spot there is still a small part of pain. How can you explaine this and what should i do about it? Doctor: Hi..Can understand your concern..As per your complain a lump and bruising occurs after injury or trauma as a result of inflammation and formation of hematoma due to broken blood vessels..A hematoma and inflammation can take a longer time to subside and can take months even but in case if you still have pain in the region along with numb feeling it can be due to secondary infection in the area leading to tenderness and swelling..The numb feeling can occur due to either pus formation in the area due to infection or it can be due to injury to nerve fibres which could have happened due to fall..I would suggest you to consult a doctor and explain the entire medical history and get ruled out for the cause of tender swelling an numbness..In case of infection antibiotics can help in reducing swelling and pain..In case of numb feeling due to nerve injury recovery can take a longer time of months and years even.. In case it is due to pus formation it will get relieved once infection subsides..Hope your query is solved..Thanks and regards..Dr.Honey Nandwani Arora.."
},
{
"id": 224788,
"tgt": "Taking micronor contraceptive pill. Raised blood pressure, getting irregular heavy periods. What's going on?",
"src": "Patient: I have been taking the micronor contraceptive pill for about 4 months but for the bast 2 months have been bleeding heavily every other week! This really isn t ideal as it is ruining my sex life! I was originally on the microgyn pill but it raised my blood pressure . I went to my GP about the bleeding and she just said it might stop completely soon but she didn t seem certain and I really need answers. Doctor: Hello. Thanks for writing to us. The irregular periods that you are having is likely to be due to the icronor pills that you are taking. This irregularity will gradually decrease on its own.I hope this information has been both informative and helpful for you. Regards, Dr. Rakhi Tayal ,drrakhitayal@gmail.com"
},
{
"id": 184735,
"tgt": "Suggest remedy for bleeding mouth sores",
"src": "Patient: I have bleeding mouth sores. I attribute this to my chemo treatment for leukemia. I see my oncologist tomorrow but I fear this evening when I go to bed. It I seems that is when the bleeding starts. I am very careful brushing, drink a lot of ice water, swish my mouth with baking soda and water. Help. Doctor: Thanks for your query, i have gone through your query. The bleeding sores could be secondary to the leukemia. in leukemia the platelet count will come down, that results in increased tendency to bleeding from gums or even minor cuts. consult a oral physician and get your teeth cleaned to remove the deposits because evn deposits will cause bleeding. You can use topical gum astringents like stolin gum paint to prevent bleeding gums. i hope my answer will help you, take care."
},
{
"id": 97509,
"tgt": "Is there any Ayurvedic medicine to increase follicle growth during IVF?",
"src": "Patient: I am 29 yrs, 50 kg and 5 ft tall. I am under going IVF treatment, after day 8 of my cycle my scan suggest no growth or hardly any growth in follicle in both the ovary. However there is one cyst in left ovary which is growing. Is there any Ayurvedic medicine to increase follicle growth. Doctor: Hi Welcome to HCM This is Dr Suchda , NATUROPATH - HOMEOPATH- MAGNATOTHERAPIST going through your query to help remove your problem . your worry is that 'even after 8 days of your IVF treatment follicle not growing. Though it is matter of concern ,even then i will suggest you to have patience . I will suggest you to take care of your physical as well as mental health . Include in your food , Vitamin E in the form of almonds ,eggs ,fish oil . take wheat , black gram , almonds ,eggs ,fish oil , ginger , ,lettuce ,radish cooked with leaves ,milk,refined butter, mango , melon . Out of these , take as many as you can in a day . some of these you include in BF , some of them include lunch and some in dinner .And last but not the least ,take water prepared in copper glass twice every day . fill water in copper glass and keep over night , drink it in the morning . Again fill and keep it for 12 hours and drink in evening , Carry on this regime for 3 month or so . you will see the result .Keep your specialist aware of situation now and then . Don't hesitate to get back if have any further query All the best . God bless you Also do pranayam -Inhale -hold -exhale - hold deep breathing , dont over do . Keep Calm and don't take unnecessary tensions . Take homeopathic Medorrhinum 30 4 pills weekly for 4 weeks"
},
{
"id": 66563,
"tgt": "Suggest remedy for lumps in collarbone",
"src": "Patient: My wife woke up with a three inche long lump on her right collarbone one week ago. So far, antibiotics have not shrunk it. The doctor is taking bioposy on Monday. Should she take multiple samples and is there a high correlation between malignancies and lumps on the right side of the clavicle (as opposed to the left side)? The doctor said it was soft, but it feels quite hard. I am worried that the lump may be a result of a cancer that may have metastasied. She says her stomach feels a bit bloated today. I am worried about stomach cancer. Doctor: HiWelcome to hcmStomach cancer spreads to left side lymph node mostly. Biopsy will confirm the cause. But cancer spreading to right side lymph node is rare. You have to wait till biopsy result. Regards"
},
{
"id": 98587,
"tgt": "How can allergic rhinitis be treated?",
"src": "Patient: Morning, I have had what appears to be an allergy reaction to pollen since moving from One city to another. Runny eyes, runny nose, very bad chest cough, hot and cold flushes, slight temperature elevation. Saw my GP who said it was a reaction to pollen due to area I live in. Gave me Telfast 180mg. I have had this for more than 3months. Very tired, lack of sleep at night due to coughing fits etc. Also take a Multi Vitamin as well. I m 56 yrs and not getting any younger. Starting affect my daily routines, work, and external activities. Any suggestions please. Tony Doctor: Allergic rhinitis is not a rare disease and can effect a people of differernt ages. All the symptoms that you are having are more or less classical for this condition. There are three major ways to go about in treating Allergic Rhinitis1. Prevention - Taking medications to avoid having the symptoms2. Symptomatic - Taking medication to alleviate the symptoms you are experiencing.3. Immunotherapy - Make you immune to the allergens causing the symptoms in the first place.In your case as your symptoms have already started. We need to get your symptoms in control first. You are unfortunately having the full package of symptoms i.e., your eyes, your nose and your chest, all three things are involved. If your symptoms have failed to settle with an anti allergic medication like telfast and are experiencing fatigue and lack of sleep then its better you see your physician sooner than later. I would suggest that you should be on the following drugs initially and then the drugs can be slowly tapered off.1. Steroid nasal spray2. Anti allergic eye drops3. Anti Allergic oral medication4. A Steroid + Beta Agonist inhaler (If the chest on examination reveals narrowing of airways)5. In extreme types of allergic Rhinitis your physician can even give you a few days of oral steroids to get your symptoms under control quickly.Once your symptoms are under control then he can concentrate on referring you to an allergy specialist for immunotherapy which can take weeks to months to get the adequate relief required. Till then your drugs will help keep your symptoms at bay and help you go about your daily work with little or no problems. I hope this helps."
},
{
"id": 151805,
"tgt": "What does compressed vertebra and non replying nerve mean ?",
"src": "Patient: hy sir i m 32 male my doctor is neorologist and orthopadic specialist hy sir i m 32 male my doctor is neorologist and orthopadic specialist after the checking my knee and vertebra x-ray doctor prrescribe me: gabafix 3 times a day. feldren 20 2 times a day. medigesic 2 times a day he cant told me for MRI or CT scan he says nothing to worry he told me ur vertebra compression is the cause of burning sensation in full leg and tingling sensation in many area of body somtimes. tell me how to de-copress the vertebra tell me any excercise name then i m searching on youtube in video can i must doing MRI or CT scan wht do u say doctor says when vertebra is compressed the nerve is not replying normaly it is tru i want ur comments thnx regards Doctor: Hi,Saif, Thanks for query, Unless you have h/o injury on back decompression of vartebrae is not possible at this age. but as MRI and ct scan is normal so nothing to worry. Go for back extension exercise, Do physiotherapy and SWD. ok and bye."
},
{
"id": 127244,
"tgt": "What causes severe pain in the legs?",
"src": "Patient: My name is Brenda I have had this problem for two years now and no answers I thought maybe I might get a fresh perspective. I have had chronic diarrhea to the point of not being able to make it five feet to the bathroom, I thought I had rhymatiod arthritis and the blood test was negative, I have massive pains in my legs to the point that I can t stand or sit in the same position for long, it s not the joints that hurt it s the bones they feel like they are on fire. I do have barrets esophagus and had lower arena upper scopes done last November and everything was normal and un changed with the barretts. Doctor: Hello and Welcome to \u2018Ask A Doctor\u2019 service. I have reviewed your query and here is my advice. Pain in the legs are basically due to neurological dysfunction though vascular and musculo-skeletal problems can also be there. Visit a neurologist and get all the nerve function tests done along with a few imaging investigations. Nerve damage causes a fiery feel at the points of damage. You need medications & endurance for nerve repairs take time. Take calcium & iron rich diet. Take dairy products and stay well hydrated, maintain the electrolyte balance of your body. You can also take vitamin B12 with folic acid & minerals as dietary supplements. Hope I have answered your query. Let me know if I can assist you further."
},
{
"id": 79104,
"tgt": "What causes difficulties in breathing and tight chest?",
"src": "Patient: Hi... I have had difficulties taking deep breaths for the past 3 days. My chest is very tight and on the right side of my chest there is a knot. Also I keep wanting to yawn but can't. Omplete the yawn. I don't feel any anxiety or anything so I don't think that is the problem Doctor: HelloTightness in chest and difficult breathing need immediate medical attention.You need proper clinical examination and few investigations.Investigations include routine hemogram,random blood sugar,liver function test,lipid profile,renal function test,chest X-ray (PA view),ECG,ECHO,TMT,upper GI endoscopy.Further investigations can be done if needed.Treatment depend upon findings.Get well soon.Take CareDr.Indu Bhushan"
},
{
"id": 205589,
"tgt": "Suggest treatment for type 1 bipolar disease",
"src": "Patient: I am not epileptic; I am bipolar, type 1 with psychosis (possibly schizoaffective instead). I stopped taking my medicines on Sunday night. I missed, among others, my keppra (off label), trileptal and lithium. I take 1000 mg keppra BID; 300 mg trileptal AM, 600 mg trileptal PM; 300mg lithium BID. While I have had definite positive effects, my brain just did something weird. It, well, pulsed. I just took all of the above drugs. I see my doc tomorrow. He does not know I did this. Am I OK? Doctor: DearWe understand your concernsI went through your details. I suggest you tell this to your doctor and get his idea. There should not be anything wrong with it.If you require more of my help in this aspect, please use this URL. http://goo.gl/aYW2pR. Make sure that you include every minute details possible. Hope this answers your query. Available for further clarifications.Good luck."
},
{
"id": 60634,
"tgt": "protien and omega3 fatty acid for jaundice recovered patient",
"src": "Patient: Dear Doctor, due to jaundice, the patient has recovered but lost a lot of weight, and wrinkle in the skin. she is 53 yrs old. hence can we give enough protien in diet for the patient and how about omega 3 fatty acid from salmon rgds Doctor: u can include resource (hepat) powder 2 tsp in glass of milk daily at bedtime radish garlic juice will increase the vitality"
},
{
"id": 116248,
"tgt": "Can elevated liver count level cause muscle pain?",
"src": "Patient: Hello, I recently went to the doctor to have a physical and blood work was done. I was called back about a week later and was told my liver count level was too high, and also my thyroid count was low. She says that she wants to do a hepatitis panel test and an ultrasound of the liver. When researching online I found that muscle pain could possibly cause an elevated liver count and the day before my physical the muscle on the right side of my lower back was hurting a lot. 3 days before I also did some heavy drinking. Should I be worried? Could I possibly have elevated levels from taking mucinex and day quil? Doctor: hi, dr. boskey gandhi is here, in liver function test, multiple tests are available. which parameter of liver is affected & how much, its important to know for us,(as u had not mention) for decisions of doing liver penal.but I think that u should go for sonography of abdomen, to rule out any gross pathology. so best luck."
},
{
"id": 92601,
"tgt": "Stomach pain, feeling tired, hot. Feeling push in stomach. Pregnancy test negative. What does it mean?",
"src": "Patient: my girlfriend is having stomach pain, always tired , gets hot , mad at me one moment and then in love with me the next , trips over every little thing. she says she feel something push in her stomach i think she is pregnant but we took a test a while back and was negative but basically these are all the symptoms..... what would you think it is?????? Doctor: Thank you for asking on health care magic.If I was the treating doctor i will do the following:1. Look for other symptoms along with stomach pain like vomiting, diarrhea. If they are present will consider some infection. If there is only stomach pain, cramp like and no other symptoms, i will consider psychological causes.After ruling out any organic illness, the symptoms appear due to psychological causes likely anxiety. Anxiety can cause stomach pains, feeling of heat in the body, irritable mood and sometimes tiredness.I hope you find this helpful.Thanks"
},
{
"id": 200843,
"tgt": "Suggest treatment for yellow greenish discharge from penis",
"src": "Patient: Have yellow greenish discharge from penis burning when I pee only feel it in the tip of penis i don t drink water and I use the drug crystal meth but just smoke it and don t want to see a doctor so what can it be and what can I try to clear it ??? Please need help with out seen a doctor??? Doctor: Thanks for asking in healthcaremagic forumIn Short: Greenish discharge suggest infection.Explanation: Greenish discharge from urethra suggest infection of urinary tract. So, please visit a doctor and get yourself tested for STDs as early as possible."
},
{
"id": 29699,
"tgt": "Suggest remedy for cystic acne on face from medication for ADHD",
"src": "Patient: How do you treat/stop acne caused by Vyvanse (hoping to reduce dose, but continue it for the benefits)? Horrible, deep cystic breakouts on my chin and around my mouth. They break open into multi-pore wounds. Never experienced anything like it. Pubmed has nothing, but there s plenty of anecdotal discussion about at least a correlation on add and adhd forums. Please help, this medication saved me from losing my job, but my face hurts! Doctor: HI, thanks for using healthcare magicAcne can be worsened by some medications. There are 4 factors involved in acne and management is directed towards treating as many of these as possible.(1)inflammation of the skin(2)blockage of the hair follicles of the skin(3)excess production of sebum which is a waxy oily substance(4)the presence of a particular bacteria- propionbacterium acnesThe guidelines suggest that retinoids should be the first type of medications tried. These can be topical or oral medications.Topical include adalpalene, tazarotene, tretinoinOne of these can be combined with a wash or cleanser with benzoyl peroxide- can be obtained over the counterYou may also benefit from a course of antibiotics as wellI hope this helps"
},
{
"id": 73615,
"tgt": "Suggest treatment for difficulty in breathing",
"src": "Patient: Hi, I have hard time breathing sometimes. I wouldn't be doing anything I'd br sitting down watching tv and my heart starts pounding really fast and I feel like I can't breath any more. I get up to get a drink of water and I start feeling light headed, I told my doctor but he didn't say anything was wrong with me. I have low blood and low iron and I'm not sure if that's the problem. I take vitamins and iron pills but I'm not committed to them. I'm 16 turnig 17 next month. Can you please help me? Doctor: Thanks for your question on Healthcare Magic.I can understand your concern.Lethargy, breathing difficulty, palpitations etc are symptoms of anemia.You are not taking iron and multivitamin pills regularly. So your anemia will not improve and your symptoms will persist.So better to take your iron and multivitamin pills regularly as per directions of your doctor.Eat more fruits, green leafy vegetables and pulses.Don't worry, you will be alright once your anemia is cured.Hope I have solved your query. I will be happy to help you further. Wish you good health. Thanks."
},
{
"id": 147350,
"tgt": "Suggest diagnosis for CSF intensity",
"src": "Patient: Hallo sir my brother is 10yrs old suffering csf intensity we'll defined lesion in middle posterior fossa posterior to cerebellum it measure about 39/38/19mm.key hole deformity noted at the interior part of cerebellum.thin septa are seen within.mild scalloping of brain surface noted.possibilty of gaint cisterns magna is more likely than arachnoid cyst. Now there is headache so pls kindly guide us Doctor: Hi,Thank you for posting your query.I have noted your brother's symptoms and MRI brain findings.First of all, I would like to reassure you that MRI findings do not suggest any serious abnormalities and there is no need to worry about that.Giant cistern magna is a normal finding in many people and does not require any treatment. Most likely that is not the cause of headache.The headache could be due to migraine or tension headache. These would respond to medications.I hope my answer helps. Please get back if you have any follow up queries or if you require any additional information.Wishing you good health,Dr Sudhir Kumar MD (Internal Medicine), DM (Neurology)Senior Consultant NeurologistApollo Hospitals, Hyderabad, IndiaClick on this link to ask me a DIRECT QUERY: http://bit.ly/Dr-Sudhir-kumarMy BLOG: http://bestneurodoctor.blogspot.in"
},
{
"id": 6187,
"tgt": "Having cyst in ovary. Had miscarriages. difficulty conception. taking oosure. Why am I not able to conceive?",
"src": "Patient: Hi my name is Neha i have 2 cyst in my left ovary measuring 23 cm and 28 cm.Befor that i had three miscarrge as it was blanded ovum ...now i am not able to concive...right now i am taking oosure medicne as advised by doctor. could u pls advise what eaxctly the problem is and why i am not able to concive is it because of the cyst .. Doctor: Hello. Thanks for writing to us. Cysts in the ovary do sometimes cause hormonal imbalance and causes difficulty in ovulation. You need to get your hormone profile evaluated and ovulation studies done to know if your other ovary is working normally. I hope this information has been both informative and helpful for you. Regards, Dr. Rakhi Tayal drrakhitayal@gmail.com"
},
{
"id": 162412,
"tgt": "Can Longifene-A syrup be used for appetite loss in a toddler?",
"src": "Patient: May baby boy 22months he is not eating food my doc suggest me longifene syrup for food 2.5ml for 2 time bfr half and hour food... Can I used this syrup? It s any side-effects is there? Please suggest me.. Hi, can I answer your health question? Please type your question here... Doctor: Hello,1. Although it is common in children between 2 and 6 years as their appetite fluctuates in accordance with their age and energy levels, but if you notice that loss of appetite is accompanied with weight loss also then consult a Pediatrician immediately to identify the cause and correct it. PS. at his age common causes to look out for are anemia, intestinal worms, and constipation since they are preventable with some extra care and attention. 2. Make sure to exclude temporary causes of loss of appetite like teething, sore throat, growth changes, till then keep following things in mind: . feeding tests the patience of the mother, but it is the most important part of treatment. . small feeds every 1/2 hour. . undiluted milk thickened with 1 tablespoon of skimmed milk powder. . gradually increased solid foods like potato, mashed banana, sweet potato, pulses. . minimum handling of the baby. . multivitamin syrup: 10 drops twice daily. As you are under the observation of physician, thus remain under his regular supervision as babies are delicate and can get worse with infection, also Longifene [Buclizine] is given in motion sickness, allergy and vertigo and sleepiness is the major side effect of the drug, so do not stop/start the medicine without first consulting with your physician. Hope I have answered your query. Let me know if I can assist you further.Regards, \u00a0\u00a0\u00a0\u00a0\u00a0Dr. Munish Sood"
},
{
"id": 181119,
"tgt": "How can toothache be treated?",
"src": "Patient: I had a broken tooth they had to do a root canal on. Now the oral surgeon said the tooth on either side is dying. They did a root canal on one and will probably do the other on Monday. My concern is now a fourth tooth is feeling pain, the next one in line. I wonder if I need to see an MD about nerve damage or bone issues. Doctor: Hello,I completely understand your concern about the toothache. Dental pain surely annoys you.Reading your question, I guess you have a high carries index which is why the dentist has suggested you for a root canal treatment. There would be a deep cavity in your tooth which is why you feel the pain.I do not suspect any bone or neurological damage.Get your dental x-ray done and depending upon the depth of the cavity , the doctor would suggest you to go in for a simple filling or a root canal treatment. The earlier you visit a dentist, the better it is !Hope this helps. Take good care"
},
{
"id": 13224,
"tgt": "Suggest treatment for itchy flaky rash on shin",
"src": "Patient: My husband has had a chronic rash on his lower shins for the past two years. It is itchy raised and flaky. It never quite heals despite antibiotic creams and steroid creams. He has a 30 year history of ankylosing spondylitis. We would like to know what it is. A dermatologist misdiagnosed him with follicultis Doctor: Hi,It may be lichen simplex chronicus most probably. Consult the expert dermatologist for the perfect diagnosis and proper treatment. I would recommend you to take intra dermal triamcinolone 10 mg injection by the dermatologist. And apply potent steroid cream twice a day. Take oral steroid in tapering dose for few weeks. Take antihistaminics daily at night. Avoid soap bath. Avoid worries and tension. Hope this helps..Dr.Ilyas Patel,Dermatologist"
},
{
"id": 5611,
"tgt": "Had sex, missed birth control pill, had short period. Pregnancy chances?",
"src": "Patient: I am on birth control and had a normal period on feb 3rd. Had sex feb 23rd and missed my birth control pill on feb 24th but took 2 birth control pills on feb 25th. Period started on march 3rd only lasted till march 5 not normal for me. march 6th no signs of period then march 13th light brown/tanish spotting on toilet paper. Can I be pregnant? Doctor: Hi, Since you took the additional pill, you were protected for the cycle, so chances of pregnancy are almost nil. Also, you had intercourse during the safe period. The spotting you have now could possibly be a pill breakthrough. If you wish to have confirmation, you may take a HPT or a blood test for pregnancy and see your consultant for further care. Take care."
},
{
"id": 80643,
"tgt": "Suggest treatment for neutropenia and chronic cough",
"src": "Patient: I have had cold like sympyoms for 4 days but yesterday was the beginning of a chronic cough followed by spitting out green or brownish phlem. I have an immune deficiency (Neutropenia) but my thermometer hasn t read any higher than 98.8. I have shortness of breath, and a slight pain in my left lung. Bad cold or Pneumonia? Doctor: Hello dear, thanks for your question on HCM. I can understand your situation and problem. Neutropenic patients are immunocompromised and prone to bacterial infection. Fever is not seen commonly in infection in Neutropenic patients. So in your case even in the absence of fever, green mucus and coughing suggest lung infection ( pneumonia ). So better to consult pulmonologist and get done1. Clinical examination of respiratory system. 2. Chest x ray. 3. CT thorax if required. 4. Sputum culture and sensitivity. Chest x ray and CT thorax are needed to diagnose pneumonia. Sputum culture and sensitivity is needed to isolate the causative organism and guide the appropriate antibiotic therapy. So better to consult pulmonologist and discuss all these."
},
{
"id": 1516,
"tgt": "Will i get pregnant after taking Livogen and Folmine?",
"src": "Patient: i m taking glycomet-850 tablet trice a day, livogen twice a day, folmine twice a day and one ovrall tablet per day with doctor advise. I didnt able to understand what my doctor told. can u please explain clearly?.. can i able to get pregnant after talking these tablets ? Doctor: Hi, ovral is z contraceptive pill, so you can't get pregnant on that. Talk to your doctor regarding this. You may need some medicines for growth of your follicles and track your follicles growth by repeated ultrasound. When follicles reach a size more than 17 to 18 mm, take injection for rupturing the follicles. Be in contact with your husband every 2 to 3 days after your periods stop. Take progesterone for next 2 weeks. Do a urine pregnancy test at home after that. You can try like that for 3 to 6 months. Glycomet can help you in conceiving. Hope I have answered your question. Regards Dr khushboo"
},
{
"id": 59309,
"tgt": "Elevated liver enzymes in reports, just social drinker. What could be the cause ?",
"src": "Patient: Hello. I just went for my yearly physical and my MD told me I had elevated liver enzymes . My AST was 49 (N 10-35) and my ALT was 62 (N6-40). They have never been elevated before. I am only a social drinker, only 1-2 drinks per weekend and not every weekend and I have not been sick. The only symptom I have is a feeling of a sweaty rush a few times a day. My MD repeated my blood work and is also testing for hepatitis . Can you have elevated liver enzymes for no apparent reason or does it always mean some type of liver damage or muscle damage? Doctor: Hello! Thank you for the query. Your liver enzymes are slightly elevated. Such results can appear due to alcohol intake (does not have to be everyday), fatty foods, spicy foods, medicines (like painkillers), liver viral infection and many more. Such slight elevation usually does not indicate any serious pathology and with proper diet and avoiding alcohol it can come back to normal ranges. Hepatitis tests are a very good idea and if negative, just avoid alcohol, fatty foods and repeat this test in a month. Hope this will help. Regards."
},
{
"id": 58775,
"tgt": "Liver tumor increasing in size, spreading. Quantiferon tb gold test positive. Diagnosed primary hepatocellular carcinoma, hcv. Advise?",
"src": "Patient: my faher by MRI IN 2007 diagnosed primary hepatocellular carcinoma.no biopsy done,he has a HCV.all tumour marker -AFP,ALKALINE PHOSPHATASE,CEA,BILIRUBIN,ALL OTHER TEST are normal,exept sgot-126,sgpt-37,GGT-83,ESR-46.before 4 months all lab almost normal,liver tumour increasing in size.now it spread in both adrenal glands & psoas as per MRI.weight reduced -5 kg,anorexia,diarrhoea,weakness start this is almost within 2-3 months.he is treated only with homeopathy,HCV VIRAL LOAD BY PCR -neg.I read in google some cases dx-hcc & then turned liver TB.he has a old lesion of pulmonary tb.now chest x-ray normal but QUANIFERON TB GOLD TEST-positive.this test does not diff. letant tb & active tb. bl.culture neg.can you give us advised please.thanks.waiting for your answer. Doctor: Good morningYou need to urgently see oncologist and herpetologist(liver specialist).Your father has advanced cancer of the liver.The cancer has metastasis(spreading to other organs) sign of bad prognosis.He will also need counseling services by professional counselor."
},
{
"id": 165471,
"tgt": "Suggest medication for neck pain",
"src": "Patient: My daughter is 2yrs 3months old and today when I picked her up from daycare she touched the back of her neck and said it hurts and cried. She has done this now several times and in between the times she hold her head and cries she is playing like normal only she seems to be holding her neck still. Doctor: Hello,As she has been crying and holding back of her neck several times and having static tone of her neck while playing then she might be having some stretching or sprain of ligaments of neck or stretching of muscles. You may use medications like Paracetamol or Ibuprofen for pain relief. But, it would be better to consult orthopedic surgeon.Hope I have answered your query. Let me know if I can assist you further.Regards,Dr. Khan Shoeb Mohammad Sher Mohammad"
},
{
"id": 47729,
"tgt": "Reason for severe pain in the kidney area?",
"src": "Patient: I HAVE BEEN HAVING SEVERE PAIN IN THE AREA OF LEFT KIDNEY. I HAD A CT SCAN AND THEY SAID LEFT KIDNEY WAS ENLARGED. HOWEVER A MRI AND RENAL SCAN HAVE TURNED UP NOTHING BUT I STILL HAVE THE PAIN AND NAUSEA. I HAD KIDNEY STONES AT ONE TIME BUT NONE CAN BE SEEN NOW. ANY IDEAS Doctor: Hello and welcome to HCM.As an Urologist, let me assure you, that left kidney pain can be easily diagnosed, after doing the tests as mentioned by you.If you can send a copy of the CT and MRI scans , i can give an expert opinion about cause of the pain. Hope your urine and creatinine tests are done. Kindly mention your age and genderSend the reports as a direct question, in my name, and i shall give you an expert advice.Dr.Matthew J. Mangat."
},
{
"id": 34579,
"tgt": "Suggest treatment for chronic fungal infection",
"src": "Patient: hi sir iam strugle my unhelty of fungal infection since six years but has been taking of nuforse tablets weekly once but it is not pastly working can say related medican for fast eleminated for fungus inside of body organas like brain stomach joints keens kidnys sevaral parts of my body mouth also can say best treatment for inside of funguls infection Doctor: Hello dear,Thank you for your contact to health care magic.I read and understand your concern. I am Dr Arun Tank answering your concern.Such a severe fungal infection occurs only in the state of immunocomprmise condition.This type of severe infection also requires a long time to cure. I advice you can test yourself for HIV, diabetes. Which is the common cause of the severe fungal infections. Nuforce tablet is a good treatment choice for all the fungal infection. You can continue it under your doctors guidance.Please adopt good living practice that is hygiene, good food and daily exercise. This will equally help in the treatment.I will be happy to answer your further concern on bit.ly/DrArun.Thank you,Dr Arun TankInfectious diseases specialist,HCM."
},
{
"id": 216623,
"tgt": "Suggest remedy for pain in calf muscles",
"src": "Patient: I developed a pain in my calf while running like a charlie horse or cramp, now its 3 days later and the pain is worse, especially when I walk or go up and down stairs, extremely painful. I am concerned that it could be a blood cloth, but there is no redness in the calf that I can detect and it does not hurt at all when I am at rest, sitting or laying down. Do you think its a muscle injury or blood cloth ??? Doctor: hi,thank-you for providing the brief history of you.As you have a doubt if it's a muscle injury or blood clot, we need a thorough clinical examination of you clinically and also some blood routine as well.Since there is no redness or skin discoloration I feel it's completely a muscular injury and can be helped by simple medication and some exercises.But getting an assessment done for ruling out the doubts will add on a safer note to remove many of the unusual thoughts.regards Jay Indravadan Patel"
},
{
"id": 155967,
"tgt": "How is ovarian cyst cancer diagnosed?",
"src": "Patient: I had a cyst on my ovary that grew so big I needed surgery. My doctor tested the fluid in my cyst after they removed it for cancer cells and tests came back positive. But they did a blood test on me for cancer before surgery and it came back negative. I need more answers please help. Doctor: The blood test done was perhaps CA 125. It need not be elevated in all cases of ovarian cancer. Though CA 125 is raised in most of serous ovarian malignancy, it is normal in mucinous tumors. It is not a diagnostic marker and treatment decisions are not made depending on its value. Since the fluid has come as positive for cancer , the diagnosis of cancer is confirmed and you need treatment."
},
{
"id": 152805,
"tgt": "How can urinary incontinence post prostate surgery be treated?",
"src": "Patient: My father sufferred from prostate cancer. He was treated and operated at Canada and effected portion was removed successfully in October 2010. After the operation, he found that his urine is tickling every second and he cant control the same. The doctors at Canada are continously consultated, they advised use of Diapers with some exercies. Now more then six months passed, their is no improvement at all and rather conditions are worsening day by day. His urine is tickling every second and the same is not controllable. Due to cntinous flow of urine, his skin all around the pennis has rotten. Now my question, 1. why urine is tickling continously, 2. is operation was unsuccessful, 3. is by any chance during operation, the operating doctor had damaged part which controls urine. 4. What treatment can be given to control this problem, 5. How this problem can be rectified now, 6. is my father needs to go for some second operation to overcome this problem. Doctor: HiUrinary incontinence after radical prostatectomy for prostate cancer is a known complication. It occurs in more than 50% of patients after surgery but in most cases resolve and in few patients might require intervention -I usually quote 5-10% of patients will require additional surgery to correct the problem. This is due to interference with the muscles controlling the continence during prostate operation. He will need either an artificial sphincter or sling surgery. Please talk with your urologist again."
},
{
"id": 195934,
"tgt": "Suggest treatment for infection on scrotum and penis",
"src": "Patient: Hello Sir, AM a black and I had a skin infection on my scrotum and it is discoloured. I had been to the the hospital on some occasions and I was adviced to use antifungal creams....it helped but now, the colour of my scrotum is pale and the infectin is getting to my penis too Doctor: hellothank you for trusting HCMDear you may be suffering from tenia cruris. use candid b ointment locally,tab. fluconazole 150mg once weekly for 6 weeks if symptoms not improved please consult your doctor he will examine and treat you accordingly.maintain good hygiene,make genital area keep dry,ithink I answered to your question if you have more questions feel free to ask.take care"
},
{
"id": 15584,
"tgt": "Bad rash, has little pus, itching awfully, skin dry and cracking. What is going on ?",
"src": "Patient: Hi My friend has very bad rash looks like a chicken pox but not it has a little puss and iches alful her skin is all very dry and cracking to see blood. she has been to 2 different dermtogiest and they don't know this has been going on 9 months. do you have any suggestons she has had blood work and Mri cat scan and all that looks good. Doctor: HIThanks for choosing, HCM, It seems she might be having, some scabies like infection, you can try anti-scabies, lotion,,Gamma-benzena, would work nicely, apply this lotion for consecutive three days, take second generation anti-histamine,,have nice day"
},
{
"id": 132392,
"tgt": "What is the treatment for swelling around the incision?",
"src": "Patient: I had foot surgery approximately 7 weeks ago to remove a bone spur and cyst on the top of my foot. It went well until 2 weeks ago, 5 weeks post surgery go infected. I am on my second round of antibiotics. It is healing but I still have fluid and swelling around the incision. Do you think this time frame of healing an infection is normal? At what point would you consider getting a second option? Doctor: Hello thank you for writing us here. Well the would healing takes time depending upon the size of incision, your immune level and blood supply to the foot. Antibiotics are given because surgeries like that are more likely to be infected and because while preforming the surgery, the blood supply of bone is cut off and because of that, the antibiotics don't show much effect as they don't reach the bone to that extent when they show their effect. Multiple shots are required for proper effects. God bless youBest regards,Dr Gunjan"
},
{
"id": 164510,
"tgt": "Does spine of an infant affects by sitting instead of sleeping?",
"src": "Patient: Hello. My baby is 3 months and healthy, but I worry that I never put in on his tummy enough. Now when I do he is fuzzy and get panicky because he doesn t know how to relax his head and thinks he needs to hold it up all the time. Also he has been sitting a lot in his baby bjorn bouncy chair since he was 6 weeks. I do have a flat pram though andhe is sleeping in a flat cot half the night and rest of the night in my bed. I am worried that he has been sitting to much in his bouncy chair instead of lying on his tummy. Can this affect his spine? Doctor: normally baby can sit with support with straight back from 4 months age.. there will be no harm to the spine if you make your baby to sit in bouncer. make sure that the baby is comfortable wherever he was placed."
},
{
"id": 91601,
"tgt": "What is the treatment for pain in abdomen?",
"src": "Patient: Hello...Here are my symptoms:Episodic pain in Lower Right Quadrant of Abdomen. Around the area of the inguinal canal / appendixHappens for about 3 hours every day but with no obvious trigger.Pain goes into the tubes running into the testicles as well.The pain is not sharp and agonising but dull, aching, dragging / heavy.Often rises slightly higher into the abdomen.Has been accompanied with nausea after eating (episodic), fatigue, and once I woke up in the night and the lower right quadrant pain had become a sharp throbbing more painful episode. This went in about 15 minutes.The pain goes into my lower back and down my thigh (again dragging, heavy), and into my right testicle / scrotum.My bloods are fine, I have had an abdomen ultrasound on liver, kidneys etc and they are all fine and I am going to have a soft tissue hernia scan.I don't have an STI, but the Sexual Health clinic treated me for epididimitis and put me on a two week antibiotic course. This has actually helped a fair bit, especially the pain into and above the right testicle, but not completely and the symptoms are definitely still there (esp lower right quadrant episodic pain and nausea after eating ). It doesn't hurt to cough and there is no visible hernia bump. Also it often hurts while resting / lying down.I have bilateral plantar fasciitis and am trying to get rid of this problem, so I can continue with my rehabilitation.Also I have had a great deal of anxiety and stress recently, due to my own foot condition and my family problems.Any ideas?!What confuses me is the hospital said it could be epididimitis, but my GP said it doesn't fit in with the symptoms (there is no obvious swelling, and it is episodic and not agonising). My urine and stool tests have been normal. Many thanks. Doctor: Thanks for an elucidate and understanding history.The pain you described , its radiation all suggest me that you may be suffering from more than 1 problem simultaneously. This is definitely a pain of nerve origin , wherein the nerve supplying this area is either entrapped or involved into an inflammatory process. We have to scan from the spinal level by MRI of the thoraco-lumbar region the back of abdomen , the iliac area by clinical examination as you have rightly mentioned for hernia scan and ultrasonography or CT scan of abdomen to see for the psoas muscle problem, appendicitis in the course of the nerves. Once a proper diagnosis is make , a proper treatment is an easy thing to do.As for the planter fascitis, the best way is to have hot fomentation with super-saturated salt solution ( the feet are dipped into a tub with warm salt water.)"
},
{
"id": 94630,
"tgt": "Have mild prostate enlargement. Prescribed Veltam. Have abdominal pain. Reason?",
"src": "Patient: I am a 56 year old male recently diagonsed with Milld Prostate Enlargement. The PSA was 1.9 and prostate weighed 33 g. I have been prescribed Veltam 0.4. I am sometime experiencing abdominal pain . what could be the reason ? I am on this medicine since last one month. This is apart from my regular medication for blood pressure and cholesterol . Doctor: Hello Thank you for getting in touch with us. As I see, you have been diagnosed with mild prostate enlargement. It should also be noted that your PSA levels are normal at the moment. Veltam is a commonly used drug in the elderly population with prostate problems and is highly effective in them. Regular use of this medicine leads to significant improvement in the condition of the patient. Abdominal pain is mostly not associated with Veltam and you need to get it examined to find out the real cause. You should try to keep your BP and cholesterol in control and exercise on a regular basis. Hope this helped Regards Dr. Gagandeep Dhillon"
},
{
"id": 166298,
"tgt": "What causes blood in stools of child?",
"src": "Patient: sir my son age is 16th month.he has not going motion properly .he goes thrice a day that also very hardly sometimes blood also coming.he have so much pain .i consider so much doctors but not use. weight 8.5kg,height i don t know,come to medicines lactogen only motion problem dont have any problems Doctor: hi, blood in stool in a 16 months child can be due to hard stools due to constipation, acute gastroenteritis, worm infestation. constipation can be prevented in child by giving more water to child, giving all type of food not just Milk or any single food. other thing could be acute gastroenteritis this occurs due to sucking of fingers, or taking things in mouth. for this, antibiotic like cefixime should be given. also give a Prebiotic like and enterogermina ampule once a day for 3 days. for worm infestation Albendazole should be given. an examination by doctor should be done so that we may not miss any important finding. Have a good day."
},
{
"id": 60849,
"tgt": "What does a painful lump on the tip of the eyebrow indicate?",
"src": "Patient: Hi. This morning when i woke up i suddenly had a hard tender lump appear on the inner almost tip of my eybrow, and i felt as if my eye was a little swollen feeling (which only lasted a little while). I also noticed a very dry very stuffy nostril on the same side as the lump, the lump only hurt slightly when touched and now has a light persistent pain and a little warmth radiating from it, it does not seem to have a tip or a pore in the center. WWhatcould this be? Doctor: Hello and Welcome to \u2018Ask A Doctor\u2019 service. I have reviewed your query and here is my advice. As per my surgical experience, this painful lump on eyebrow indicate possible stye formation following some bacterial or viral infection. I recommend hot mop application, antibiotic drops, and medicines to cure it faster. Regards, Dr. Bhagyesh V. Patel"
},
{
"id": 203274,
"tgt": "What causes leg pain after masturbation?",
"src": "Patient: Hi doc I am hawk. I have pain in my legs after doing masturbation . Why is this ? I am doing masturbation some 5 to 6 years but now there is a pain in my legs after masturbation..... What should I do ?? And plz answer me quickly I need it immediately Doctor: Hi,There might be strain on leg muscles during hard masturbation giving rise leg pain.Nothing to worry, take high protein diet.Do regular exercise like running, swimming, jogging etc which will keep your body healthy and fit.Ok and take care."
},
{
"id": 62996,
"tgt": "Suggest treatment for small lumps on the arms and shin",
"src": "Patient: Hi, my name is Anita and I have these little lumps forming under my skin, mainly on my arms and shin bone on both of my legs. They are very sensitive and at times, causes extreme pain when I am touched. They are not visible to the eye but can be felt. Should I be concerned? Doctor: hi. based from your description, it could be a fibroma, lipoma or cyst. these lesions can occur anywhere in the body. it is best if you have it clinically examined by a doctor, preferably a general surgeon, for evaluation and management (medical and surgical).hope this helps.good day!!~dr.kaye"
},
{
"id": 215329,
"tgt": "How to cure body pain after exercise?",
"src": "Patient: Overall: Pain on the left side of the neck protruding from the back of my head behind left ear all the way down to lower tip of the left scapulaThe pain started after one exercise in May 2014 and has not gone away. It is not a constant, acute pain, but it is very frequent and bothers me when I turn head from right to left and when I do head stretching and pushups in the gym. Since the time this pain started, I have also experienced mild cracking/clicking sound in the neck. After I have been avoiding one specific exercise for about a month, the pain has subsided a bit, but has not gone away completely. It keeps coming back.Yesterday, I have started to experience moderate chest pain around the heart area most noticeable in the back, not in the chest. I measured my blood pressure and it was around 150/92, which is higher than my typical 120/80. Heart beat was around 52. This morning I could actually identify a tender spot on my above my left spatula, closer to my spineI am scheduled for annual physical with my primary doctor at Kaiser for tomorrow, Monday, at 11:00. Can I wait till tomorrow, or should I go to emergency? Doctor: Hello, The location and movement of pain give clues as to what is going on and often there is a (usually minor) underlying problem associated with pain. Blood pressure that is not consistent and certainly when it is not done with a good 5 min of rest is NOT VALID for risk assessment. The heart rate of 52 in someone who exercises a lot and has no cardiac problems nor symptoms strongly implies good cardiovascular shape. Localized, recurrent pain often means localized damage like (minor) arthritis. and simple x rays often show it. MRI's would be a more sensitive indicator of problems. Hope I have answered your query. Let me know if I can assist you further. Take care Regards, Dr Matt Wachsman, Addiction Medicine Specialist"
},
{
"id": 106476,
"tgt": "What could be the cause for muscle pain close to the hip area?",
"src": "Patient: Hi I have what feels like a muscle pain near my left kidney. Yesterday it was around the right kidney and raidiate around to right side. It feels a bit like a side stitch tryout would get when running It is a sharp pain. Thank you. XXXX Doctor: Hi, If the pain is in loin area urinary tract infection or kidney stones should be ruled out. Urine routine investigation and ultrasound examination will be needed to rule out that. If that is clear then it can be due to back muscle spasm or lumbar spine issue. A short course of muscle relaxants like thiocolchicoside and NSAIDs like Aceclofenac will be helpful. Hope I have answered your query. Let me know if I can assist you further. Take care Regards, Dr Nishad BN, General & Family Physician"
},
{
"id": 64429,
"tgt": "How to treat 3 round lumps on the left side of the spine?",
"src": "Patient: My back started to itch one evening (2 months ago) and I felt three round lumps along the left side if my spine. They are not on the spine, just off to the left. They are tender after probing. I have an under active thyroid and don t know if this info would be relevant. Should I visit my GP. Thank you. Doctor: Hi,Good Morning,Thanks for your query to HCM online Clinic.I have read your query in details.-Dear In My opinion, the lumps on the back mostly are due to the insect bite or bacterial infection leading to boils.-As you have hypo-thyroid,associated Dry skin due to hypothyroid has more tendency to itch and thus could lead to infected boils on the para-spinal area of the back.-This is what I could advise from the data of your query.-It needs further detailed study and so I would advise you to consult a ER Doctor, who would fix the cause for it and would treat it appropriately.-Dont worry.-Remedy-Take Care with your doctor and it would go off.Treatment -Tb NSAIDs,Antibiotics.,would resolve the lu-Hope this would resolve your query.Wellcome for many more queries toHCM from your friends and you in time to come.Have a Good Day."
},
{
"id": 175092,
"tgt": "What causes vomiting, fever, stomach cramps, body aches, dizzy in a child?",
"src": "Patient: My son is 7. He was sent home friday with vomitting, fever. Saturday he had severe stomach cramps,body aches, fever, dizzy,. Sunday same thing., but fever broke at night. Today is Monday and all the same symptoms are present, but with a very white discharge on his tongue and throat. What do you think this is? Doctor: Thank you for follow up.It seems your child has gastroenterocolitis, usually causes by infection.White tongue is mirrow of inflammed stomach.Thats why start to give him cefixime 8 mg/kg .If his weight 15kg*8=220mg once daily after meal during 5 days-Syp .indowel(ondansetron )2ml is helpful to stop vomiting -Enterogermina 1 sashe 2times for normalization flora of intestine 5 day-Charcoal 1 tab 1 time for absorbtion of toxins and gases-Try to keep the baby hydrated by plenty of water and ORS Best regards Dr.Svetlana"
},
{
"id": 15951,
"tgt": "Rashes spreading on the legs, arms and shoulders. Suggested to take prednisone and pepcid. What are these?",
"src": "Patient: Hi , I woke up the other morning with a rash on my feet that look like hives or chicken pox of some sort. They started on my feet and then went to my hands arms and shoulders . I went to the hospital and they couldn t figure it out they told me I need to go see a demetoligst and gave me prednisone and Pepcid . . Please help it s very itchy Doctor: HI..Thanks for posting here.Prednisone is a steroid and it is always advisable that you take it under observation of a Dermatologist,since you are having hives with itchiness you may be suffering from urticaria,You take tab.allegra 180 mg once a day ,it will relive your itch for some time,and better consult a dermatologist for further treatment."
},
{
"id": 47212,
"tgt": "Could sodium chloride drip be the reason for dialysis?",
"src": "Patient: My father went in for a procedure for gaul bladder infection, he is diabetic, has kidney disease has build up of fluid in lungs and in his legs and at the hospital they put him on a sodium chloride drip, 9 days later, he now has to take insulin and has to go to dialysis.....Can the sodium chloride drip for 9 days be the culprit for the now diagnosed dialysis Doctor: HelloThanks for query .Your father who is known diabetic and has kidney problem developed pulmonary edema during post operative period after surgery for gall bladder infection (Cholecystectomy) .Please since note that diabetic patients can not be given intravenous glucose hence has to be given Normal saline to maintain fluid and electrolyte balance during surgery and post operative period .Normal saline never lead to renal failure and pulmonary edema .It is one of the known common complication in elderly patients with diabetis patients with compromised renal function .Do not worry he is rightly advised insulin and dialysis and he will definitely come out of this crisis .Dr.Patil."
},
{
"id": 71629,
"tgt": "What causes tight feeling in chest with increased physical activity?",
"src": "Patient: hi,when ever i do any physical activity like running, i don't neceseraly have shortness of breath, but it feels like i can't take as much air in my lungs as i could normally. Also i have this feeling in my chest which vary's from the middle to the left and sometimes for a brief moment to the right. sometimes i feel it just in my middle-back on the right side. it feels like i have heartproblems but i don't have a high pulse or feel actual pressure in my chest. more just a feeling.. also not painfull but just concerning becouse it is in that area... i don't know if this is anough information but i hope you can ask me some more question regarding what symptoms i should have to get a sence of it being heart problems.. or that it's something like acid reflux which i have red about. i'm an 18 yo man . my father, uncle and grandfather have had hart problems in the past 3 months. (grandfather already 40 years back but now again) Doctor: Hello,Since you are having a strong family history of heart diseases, we should definitely rule out heart diseases for your symptoms. So better to consult a cardiologist, and get done blood pressure monitoring, ECG, 2d echo and stress test (trade mill test).If all these are normal, then no need to worry about heart diseases. Also, get done pulmonary function test (PFT) for the diagnosis of exercise-induced asthma. If PFT is normal, then no need to worry about lung-related diseases. Sometimes undiagnosed stress and anxiety can also cause similar feelings. So avoid stress and tension, be relaxed and calm.Hope I have answered your query. Let me know if I can assist you further. Regards,Dr. Kaushal Bhavsar"
},
{
"id": 176928,
"tgt": "What causes swollen lymph nodes behind the ears?",
"src": "Patient: Hi Dr, I have a son, 1.8 years old, he is a G6PD deficient. I observed swollen lymph nodes at the back of his ears. Is it because he has cough? or is it a symptom of G6PD? Since he always have common colds/cough in the morning, we give him ascorbic acid, is it okay for him to take ascorbic acid? Looking forward for your advice.Thanks Yours truly, Mom Ann Doctor: Hi,No,The swollen lymph nodes are due to some infection in the mouth,or scalp,.Not due to g6 PDF deficiency.God blessDr uma"
},
{
"id": 137401,
"tgt": "Suggest treatment for ganglion cyst within right knee",
"src": "Patient: MRI few yrs back revealed lg ganglian cyst within R knee, asymptomatic. 3 days ago kneeling on floor heard deep POP within R knee, could it have ruptured? No pain unless try to kneel or sit with legs tucked under on floor. No redness, swelling observed. Knee feels very loose & posterior has tightness when standing from sitting & when bending over to touch toes. Doctor: cyst are usually found at posterior part of knee. when poplitues muscle tendon moves over cyst it causes sudden pop . rupture of cysts are usually very painful, which is not in this case.hamstring stretch and static quadriceps muscle exercises will decrease stiffness and discomfort. Hope this helps.Hope this answers your question. If you have additional questions or follow up questions then please do not hesitate in writing to us. I will be happy to answer your questions. Wishing you good health."
},
{
"id": 182739,
"tgt": "Suggest treatment for tooth pain",
"src": "Patient: I am a 45 year old male. I recently had a tooth pulled and a bridge fitted. One of the teeth that the bridge is on has been hurting since the other tooth was removed. I went back to the dentist and the x-rays showed no issues. What could be causing the pain? What are my options, I cannot afford another bridge. Doctor: Thanks for your query, I have gone through your query.The pain in the tooth taken as support can be because of two reasons.The first reason can be because of high points in the bridge that create excessive pressure over the tooth and result in pain.The second cause can be because of the pulpal involvement during crown cutting.If it is a high point get the high point reduced by consulting a dental surgeon.If it is a pulpally involved tooth then get the tooth restore with RCT and the bridge has to be repeated.Regarding your financial issue, I can suggest you to go to a educational dental institute where you can the treatment in nominal price.I hope my answer will help you, take care."
},
{
"id": 35124,
"tgt": "Will accidental contact from poison Ivy affect the hemoglobin level?",
"src": "Patient: I am a 67-year old male and have just contracted poison ivy from gardening trimming bushes, etc. I had a recent hemoglobin level of 12.2 and am due to have a repeat blood test. Will having poison ivy affect my hemoglobin level and is it safe to have the lab test with the poison ivy on my arms. Normally, the blood is drawn from my hand. Thank you. Doctor: Hello and thanks for your question.Having poison ivy will not have any effect on your blood tests. Good day to you and I hope the poison ivy clears up soon."
},
{
"id": 144975,
"tgt": "What does the MRI show?",
"src": "Patient: Hello I had a MRI conclusion is Multival small T2/Flair hyperintence signal areas inbilalateral cerebral white matter with sparing of corpus callosum. Most liklly due to chronic small vessel iscaemic changes.the cerebral white matter prodomenly in frontoparietal lobes with sparing of the corpus callosum and the subcortical U fibres.I have never had blood pressure,smoked a little when young,very little.I eat pretty well ,and do moderate exercise.Will I have a stroke from this,how can I prevent that.Or am I heading foe dementia.I am 60 next yr and am female.I do have truble processing what people say,and can t process too much at once.I have pretty much always had this. Could you kindly give me an answer.Thanks Paula. Doctor: Hello Paula and thanks for using HCM.I have read your query and understand your concerns.MRI results are consistent with small vessels disease of brain.This can result from uncontrolled high blood pressure, Diabetes, high blood fats, and in some cases autoimmune vessels diseases.All these conditions predispose to stroke, so preventive measures include strict control of blood pressure, Diabetes and blood cholesterol control.Using a baby Aspirin once daily helps too in preventing stroke.These measures about preventing stroke are completed by moderate exercises and healthy eating.So, get blood tests about Diabetes and blood lipids first.Hope this helps. Wishing you good health."
},
{
"id": 57512,
"tgt": "What does Gall bladder and Urinary bladder distended mean?",
"src": "Patient: 1. What is the Meaning of Gall Bladder is partially distended & what are its symptoms ?2. Urinary Bladder is adequately distended - Symptoms ?3. Liver is Enlarged - Due to what - Symptoms?4 There is Non-obstructing Calculus seen in lower pole of right kidney - What is Calculus? Doctor: hi, Distension of gall bladder and urinary bladder are physiological events and is normal. Enlarged liver may have many causes and the symptoms are right sided abdominal pain and reduced appetite. calculus means stone. Hope i am clear."
},
{
"id": 31118,
"tgt": "What does this typhoid test result indicate?",
"src": "Patient: I have done my widal test for typhoid the result is To Th PA PB all are coming <1:80...I didnt understand what it means, I m 38 female. I have just completed my Antibiotic for 10 days and the test result showing what I wrote please tell what is the test result means.....is it still there or clear ? Doctor: Please finish the antibiotic course and repeat the WIDAL test after a week. If titres are rising then the treatment hasn't been adequate. Titres more than 1:80 must be taken into consideration."
},
{
"id": 95578,
"tgt": "Is non specific distal colitis a type of Irritable bowel syndrome (IBS) ?",
"src": "Patient: hi my name is aaamir saeed khan some days back i had endosopy and colon scopy in that non specific distal coloitis is come can you tell me ibs is same or this ibd. Doctor: Hi welcome to Healthcaremagic Hi amir, IBS is irritable Bowel syndrome.. IBD is Inflammatory bowel Disease.... The above thing is IBD the cause of the infection must be found and treated, a dose of antibiotics is needed... consult a physician.. Hope I have answered your question.. Takecare..."
},
{
"id": 14499,
"tgt": "What causes skin rash?",
"src": "Patient: I have a skin rash that I have had for two months it has gotten better but I have tried three different cremes prescribed by doctors...I now have a slight red spot and now my skin has turned white..its as if my skin is bleached white...im terrified by it....i started by using panderm creme then Zovirax..finally fluocinonide...i thought it was herpes then i thought it could be a fungal....whyy is my skin turning bleach white ..what should i do...?? Doctor: HIWell come to HCMThis could be due to hypersensitivity reaction and this can be treated with oral medicine now instead of local treatment and the best option is Tab Hydroxyzine 10 mg three times in day, everything would be fine soon, take care, have a nice day."
},
{
"id": 132480,
"tgt": "Is it safe to do surgery for nerve problem in the back?",
"src": "Patient: My father,58 years is suffering from nerves problem in the back. Doctor has advised him three tablets for 1 month.Zymoflam-D, Rablet20 and Fibrogesic. Doctor has told if these medicine works then its fine otherwise only options that would be left is Operation. Is it safe to get operated at this age or is there any after effects. Please respond its urgent. What is the success rate for this opertaion. Doctor: Hello! thank you for writing us here.Well, a possible case of nerve compression at the back region. That's your concern right? If the medications doesn't make any difference, an operation would take a couple of hours to perform.they would remove some part of vertebrae which won't compress the nerves anymore.he'll get quick relief after the operation and success rates are high. Pain would be present for few weeks at the site of operation but it would subside with NSAIDs the doctor would prescribe. Rest the chances of relapse are low. best regards,Dr Gunjan"
},
{
"id": 25642,
"tgt": "What does lateral ventricles appearing mildly dilated about 16mm suggest?",
"src": "Patient: i recenly took my 7 month premuture son to doctor and doctor said that there is some problem in my son brain and we need to do ultrasound of brain, n report said: poor beam penetration due to very small/premutre closure of fontanels.lateral ventricles appear to be mildly dilated each measuring about 16mm. blood evaluation is not possible. so my question is what is the treatment and what are the risk and complications Doctor: Thanks for your question on Health Care Magic. I can understand your concern. Your son is having premature closure of fontanales. These are gaps in skull which allow to grow brain upto the age of 5 years. So these children are prone for poor brain development and delayed milestones. Their motor functions are also reduced. Intelligent quotient (IQ) will also be low. Complications like brain edema, infection, hydrocephalus etc are more in these children. So your son needs to consult pediatric neurologist. He may need VP shunting (Ventricular peritoneal). Hope I have solved your query. I will be happy to help you further. Wishing good health to your son. Thanks."
},
{
"id": 197396,
"tgt": "Will type 2 herpes spread without intercourse?",
"src": "Patient: last night i met with this girl. she gave me a little oral sex. i touched her vagina a little through her underwear. she informed me she has type 2 herpes. how much of a chance is it that i have it?? im kinda scared because im a little itchy around my genetals area. Doctor: Hi thanks for contacting health care magic....You have come into contact with her secretion .....So if you have breach in mucosa then infection can spread to you....It is sexually transmitted illness ...So rule it out by ELISA test or PCR ...Take care ....Consult surgeon for it..."
},
{
"id": 72084,
"tgt": "What does this CT show in the liver and thorax?",
"src": "Patient: ct study of thorax and abdomen SPIRAL CT REPORT this examination demonstrates mild pleural thickening in the right mid hemithorax posteriorly with subpleural bands in the adjacent right lower lobe. Liver ia in normal size .there is diffuse fatty infiltration of liver. please guide what to do in this situation or guide any medicine for it Doctor: Thanks for your question on Healthcare Magic.I can understand your concern. I have gone through the CT report you have mentioned. In thorax, it is showing pleural thickening and bands. These are healed lesions from previous pleural effusion or pleurisy (inflammation of pleura).No treatment is required for this because they are scarred, inactive lesions. In liver, you are having fatty infiltration. This means that you are having higher body fat. So try to reduce weight and stay fit and healthy. Start exercising and dieting.Avoid alcohol Because this can also cause fatty liver. Hope I have solved your query. I will be happy to help you further. Wish you good health. Thanks."
},
{
"id": 121854,
"tgt": "What causes pain in shoulder,back and neck?",
"src": "Patient: I usually have a blood pressure of 124/80, but tonight it is 170/103. I also seem to have a pulled muscle or pinched nerve in my neck, shoulder, back, right arm and right upper chest. It is OK when at rest, but hurts when I move. It started last Wed. I had it when I woke up in the morning. How can you tell a pulled muscle from anything worse?I do have a stent. Doctor: Hello,The symptoms seem to be related to a pulled muscle. The fact that the pain increases when you move,helps through the diagnosis. I suggest using Voltaren gel for local application. I also suggest using anti inflammatory medications such as Acetaminophen to relieve the pain .Hope I have answered your query. Let me know if I can assist you further.Regards,Dr. Dorina GurabardhiGeneral & Family Physician"
},
{
"id": 147330,
"tgt": "What does these CT scan results indicate?",
"src": "Patient: I had a CT scan and the findings state: Mild decreased attenuation in the right posterior periventriicular deep white matter. Post craniotomy changes of the midline occipital region noted. May be related to underlying demyelinating process/multiple sclerosis. Doctor: Hi,Thank you for posting your query.I have noted the CT scan findings. They are non-specific and can be seen in a variety of conditions such as ischemia (lack of blood flow to brain) and multiple sclerosis.We need to correlate the CT scan findings with your symptoms. Please get back with your current symptoms, and also why was surgery done.If needed, an MRI of brain may be required to confirm the diagnosis.I hope my answer helps. Please get back if you have any follow up queries or if you require any additional information.Wishing you good health,Dr Sudhir Kumar MD (Internal Medicine), DM (Neurology)Senior Consultant NeurologistApollo Hospitals, Hyderabad, IndiaClick on this link to ask me a DIRECT QUERY: http://bit.ly/Dr-Sudhir-kumarMy BLOG: http://bestneurodoctor.blogspot.in"
},
{
"id": 212442,
"tgt": "Loss of memory. Used triazepam and diazpan. Have anxiety and depression. Help?",
"src": "Patient: hi i think my freind has date raped me usin triazepam n diazepam together,i have no recollection of this n i certainly wouldnt of slept with this person knowingly but i woke up unsure of what had happened to me the next day ,i also am diagnosed with anxiety n depression n im told these shouldnt be taken why im takin prozac please help me Doctor: Hello Prozac is fluoxetine. It belongs to SSRI group. When this drug is taken it increases serotonin in brain. In depression and anxiety serotonin is assumed to be deficient. So this drug replenishes the serotonin in brain and improve the symptomatology of depression and anxiety. This drug is a safe drug, no sedation with this drug and also no long term side effects, so there is no harm in taking this drug. This will improve your anxiety and depression. You suspect your friend gave you triazepam and diazepam and date raped you. These drugs may cause ante grade amnesia, but I need further details to comment. You can ask again with detailed question. Thanks Dr. Seikhoo Bishnoi"
},
{
"id": 4993,
"tgt": "Missed periods, urine with ammonia smell. History of having protected sex. HPT negative. Possible pregnancy?",
"src": "Patient: I havent had my period in about 2 months and i am sexually active but we always use protection. We took pregnancy tests and they both came out negative (2 of them) In the mornings. Ive always been very irregular and i get dehydrated easily but im afraid because my urine has been smelling more like ammonia. Im planning on getting a blood test soon to make sure. Do you think i could be pregnant? Doctor: Hi Dear, the missed period may be due to te irregularity in your cycles as you told. You are having 2 mnts amenorrhoea and during this time urine test should come out to be positive. Also you were using precaution. However go for B HCG test to clearly rule out pregnancy because every method has its failure rate and moreover delayed conception is a possibility Good luck"
},
{
"id": 55258,
"tgt": "What is the meaning of increased echogenicity in liver?",
"src": "Patient: i have done the sonography it says liver is normal in size and shows generalised increased echogenicity of parenchyma all other reports are normal and i also dont consume alchol and usually take veg diet i am an indian national plz suggest what does it mean and what is the cure of it Doctor: Hi thanks for asking question.Noted increased echogenicity of liver perenchyma.The possibility could be non alcoholic fatty liver by obesity like causes.It could be by hepatitis and sometime cirrhosis like cause.Here you need is to do liver function tests.According to which treatment can be guided.Sometimes increased echogenicity could be by glycogen storage disease or hemochromatosis like condition.If you have symptoms and no cause clear then biopsy can be taken.If liver function test abnornal then just follow few instructions like....-Avoid trans fat-fruits and green leafy veg-Avoid refined food-regular exercise-Avoid smoking-omega 3 fatty acid oil helpful like olive or sunflower like oil-Avoid cheese, meat, butter, chips etc.Repeat USG and liver function test after approx.month and after following above measures.I hope your concern has solved.Dr.parth"
},
{
"id": 34838,
"tgt": "What to do for RSV bronchitis infection?",
"src": "Patient: Mom has had a horrible virus infection RSV bronchitis was hospitalized now is soooo hoarse can barely talk doc suggests voice rest and it should get better her cough was non stop last week now gone down quite a bit minis 87 on abx ice chips I suggested for throat comfort please advise Doctor: Hello..Welcome to HCM..I am Dr Jigar(ID specialist). First of all it is important that this diagnosis is confimed and there is no secondary superimposed infection. For this test on secretions collected by bronchoscopy is advisable.RSV infection is not common in adults but chances increase in patients having decreased immunity and respiratory problems. So all these conditions should be ruled out by appropriate tests.For this infection early administration of ribavirin will lead to rapid recovery. Otherwise it is self limiting illness and symptomatic treatment is the main target.I hope this will help you. Wish you a good health.Regards.Dr Jigar."
},
{
"id": 125505,
"tgt": "What is the cause for incurable lesions on the skull?",
"src": "Patient: I have a family member that is bedfast and they told his wife that he has lesions on his skull that are incureable and will eventually kill him. she couldn t remember the name of what they told her and we are curious as to what he has. Do you know what it could be? Doctor: Hello, Consult a neurologist and get evaluated. Hope I have answered your query. Let me know if I can assist you further. Take care Regards, Dr Shinas Hussain, General & Family Physician"
},
{
"id": 64144,
"tgt": "What causes lymph nodes under the jaw and chin?",
"src": "Patient: Hello,i am a 22 year male. I have nodes under the jaw on both sides, and one under the chin. the one on the right side sometimes get a bit sore. i checked with other guys but didnt find any nodes in the same area as mine Is dat normal. Also i touched right node quite a time after feeling some soreness due to which it got more sore. Doctor: Hi Dear,Welcome to HCM.Understanding your concern.Thanks for your concern . The lumps under your jaw are most probably swollen lymph nodes . Lymph nodes gotswollen due to infection . There could be other reasons also for lumps under jaw like dental abscess or swelling due to blocked duct of submandibular gland .I would suggest you to :-- Apply warm compresses to the lumps 2-3 times a day .- Keep yourself hydrate by drinking at least 8 Glasses of water .- Avoid drinking cold beverages .- Avoid drinking and smoking.Consult ENT specialist (ear,nose,throat) for proper diagnosis . Doctor may palpate it physically or may go for CT scan to find out exact cause or to rule out other causes .Doctor may prescribe antibiotics along with medicine for any underlying disease if there is any .If doctor finds any dental related cause he/she will refer you to dentist . There acn be many treatment options for dental abscess like root canal . extraction or minor surgery.Hope your concern has been resolved.Best Wishes,Dr. Harry Maheshwari"
},
{
"id": 59264,
"tgt": "Been through abdominal hernia surgery. Having stone in gall bladder. Not ready for surgery. Suggest what to do?",
"src": "Patient: i have undergone abdominal hernia surgery in may 2012. i would like to go for a check-p in your hospital. i have also developed a 5mm stone in my gall bladder. i am worried as i am not mentally ready for further surgery. please suggest what to do. Dr. Riju Kakati Sarma, Asstt professor, dept of botany, Pandu College , Guwahati, Assam. Doctor: Hi Gall bladder has to be removed for Gallstone disease when there is pain in the upper part of the abdomen on right side below rib cage. Gallstones are diagnosed incidentally many times.They will cause no symptoms in many people throughout their life. So,if there is no pain,surgery is not needed. Nothing to worry Wish you good health Regards"
},
{
"id": 123667,
"tgt": "Suggest treatment for varicose",
"src": "Patient: My brother is have a leg pain below his left know. We did ultrasound and it was not a varicose. Pain is such that he cant even Walk. Doctors have referred to Spine clinic. When doctor said spine , Iam very worried. My brother is 52 Yrs old, and he is liver transplant patient , ( Transplant was done 5 years back.) Doctor: Hello, Surgery is the only definitive option for varicose veins. Consult a general surgeon and get evaluated. You can opt for newer techniques like RFA( radio frequency abalation) as it has got the best results. Hope I have answered your query. Let me know if I can assist you further. Take care Regards, Dr Shinas Hussain, General & Family Physician"
},
{
"id": 174359,
"tgt": "Suggest treatment for myoclonic jerks in children",
"src": "Patient: My son was diagonized with Infantile spasms when he was 3 months old. Now he is 4yrs old and suffers with myoclonic jerks. His development was assessed to be like a 6month old and he does not have control over his body. Neurologists I have consulted could not assess the reason what the reason is. Wanted to know if there is any cure at all for such diseases? Doctor: Hi,Thank you for asking question on health care magic.There are several medicines for infantile spasms.ACTH is the drug of choice,clobazam, clonazepam, nitrazepam,prednisolone, and sodium valproate may be used.The cure depends on the underlying cause.Hope this answer will serve your purposePlease feel free to ask any more queries if requiredTake careDr.M.V.Subrahmanyam MD;DCHAssociate professor of pediatrics"
},
{
"id": 107829,
"tgt": "Suggest remedy for indigestion with back and shoulder pain",
"src": "Patient: Hi, may I answer your health queries right now ? Please type your query here I suffer with indigestion and this is pretty much all the time. I get a really stabbing pain in my back across my left shoulder blade and in the left side of the chest. This is particularly acute if I am propped up in bed. If I lie flat the problem lessens. I don t take any remedies as I feel they only mask the problem. I eat a healthy diet hardly any alcohol, don t smoke and am not overweight. If I eat or drink milk it seems to go away for a while and then comes on as my stomach empties. Doctor: Dear- I reviewed your question in detail and will give you my medical opinion.Indigestion could have multiples causes including slow digestion, gluten entheropathy, celiac disease or just related to the food that we eat. I understand that you eat healthy, but sometimes vegetables like broccoli and cauliflower can make your digestion slow ,produce a lot of gas and give you indigestion. Try to watch what you eat and when you have your symptoms .Back and shoulder pain can be reflex from the gas.I hope my advise has been helpful.Dr.Sara"
},
{
"id": 16764,
"tgt": "Suggest treatment to bring down blood pressure",
"src": "Patient: 54 y.o. post-menapausal, 140lbs. 5 4 . meds: humira 2x month, toprol (racing heart), lipitor, wellbutrin, prozac & aspirin. my blood pressure has always been normal; the exception was twice over the last 3 years and then again today beginning 3pm and continuing currently. the highs for systolic ranged from 144 to 154, 92 to 129. i took another toprol (low dose) and .25 xanax about an hour ago. currently 141/94. no jaw pain. can i wait until tomorrow th.e speak with my cardiologist? any advice to get these numbers down tonight? thanks Doctor: Hello, I would explain that the situation can wait until tomorrow as your blood pressure values are not very high. I would just recommend having some rest and take a glass of lemon juice. Adding a new antihypertensive drug to your therapy may be needed. But, before coming to this conclusion, I would recommend performing some blood lab tests in order to exclude other metabolic causes underlying high blood pressure values. Hope I have answered your query. Let me know if I can assist you further. Regards, Dr. Ilir Sharka, Cardiologist"
},
{
"id": 9430,
"tgt": "Suggest treatment for dry and wrinkled lips",
"src": "Patient: I'm a 28 yr old female and yesterday I noticed my upper lip felt extremely dry and like maybe a piece of tape was on it. This morning when I woke up I noticed a dark patch in the center of my upper lip and its wrinkly and dry. I drink plenty of water and I keep my lips moisturized Doctor: hello madam,i would like to know if you are currently on any medication like isotretinoin which may be a cause for your dry lips. if you are not on any, then you may use a moisturising lip balm thrice a day for about 3 weeks and a multivitamin tablet twicw a day for about 1 month. Thank you"
},
{
"id": 5455,
"tgt": "Have irregular periods. Not able to conceive. Taking gestafine. What to do?",
"src": "Patient: hi i was married 4years before but iam not pregnat because i suffering irregular periods.from 4years iam using tablets for pregnancy till irregular now iam using gestafine 100mg now but for 15 days iam getting period some times it will not stop for 5 days it is getting 15days 20 continous what i can do please advise me Doctor: Hello, I would be happy to help you with your question. In the setting of abnormal bleeding that is not being easily controlled with hormonal therapy, it is appropriate to undergo a more thorough evaluation that would include: 1. An exam 2. An ultrasound 3. Labs that would include testing for thyroid disease, diabetes and prolactin levels 4. A pregnancy test! I hope that this helps. If you have more questions, please click on \"ASK ME A QUESTION\" link above to directly ask me a question!"
},
{
"id": 62221,
"tgt": "Suggest treatment for persistent sore lumps on the back of the head",
"src": "Patient: My 6 year old son has been ill for over a week now with headaches, and lumps on the back of his head. Been to A&E and was told it was swollen glands and was given calpol. A few days later he was still very poorly and lumps had got bigger. Went to my GP who said it is a scalp infection and gave antibiotics and antihistamine. Finished course of medicine and lumps are still present and sore! What do i do next!!? Doctor: HI,Welcome to HCM.I studied your query.I understood Your health concerns.Based on the facts of your query,you seems to suffer from-Boil with lymphadenitis,with resolved nodes after antibiotics.Remaining lumps indicates Chronic Adenitis with fibrotic changes in it.Reason of antibiotics not giving regression of these lumps is due to the accompanied Chronic furuncle- lumps with its lymphadenitis.FNAC biopsy / or Excision Biopsy / or CT study -of the persistant lumps, would give the exact cause of these persistant/ non-regressive lumps.Other causes like infectious mono, or viral lymphadenitis/or TB lymphadenitis/ NHL/HL lymphoma- needs to be ruled out.As the facts are very scanty,I would also suggest you to take Second opinion from Surgeon,to rule out other causes which would cause such symptom/complaints,after physical/clinical examination.Will appreciate writing your feedback review comments,to help the needy patients like you at HCM.Good Day!! Dr.Savaskar,Senior Surgical SpecialistM.S.Genl-CVTS"
},
{
"id": 212536,
"tgt": "Surgery on leg suggested. Suffer from OCD, raised heartbeat. Caffeine overdose?",
"src": "Patient: I had about 3-4 (maybe 5) cups of regular roast coffee...I ve been informed by a doctor that I need serious surgery on a leg following an injury in December and I have OCD ...I m feeling anxious, uptight, my heart is beating faster than usual and I can t seem to get calmed down...I worry I may have a caffeine overdose...what should I do? Doctor: Hello solsticeboy Due to proposed surgery anyone can become anxious and in OCD anxiousness increases. There is no chances of caffeine overdose. But to avoid anxiety try- -Thought stopping technique immediately in mind say stop whenever obsessive thought comes in mind, this will help to to decrease anxiety. -Relaxation techniques like Progressive Muscle Relaxation techniques will help -Deep breathing exercises also cause immediate relaxation and will help to relieve anxiety. so try these techniques and go for surgery, all the best. Take Care"
},
{
"id": 140899,
"tgt": "How can degenerative changes in the shoulders and spine along with bibasilar atelectasis be treated?",
"src": "Patient: The Cardiomediastinal silhouette is mildly enlarged. Degenerative changes are noted in the spine. Thoracic aorta is tortuous and ectactic. No pleural effusion,focal consolidation, or Pneuothorax. There are also degenerative changes in the shoulders. Mild bibasilar atelectasis is seen. I am 75, and in good physical shape...and do physical work, and workout on weights. Have my kown business. Some things I am aware of, and others not so much. Appreciate some interpretation. Thank you. Mac Doctor: Hello and Welcome to \u2018Ask A Doctor\u2019 service. I have reviewed your query and here is my advice. It is not a serious issue.Hope I have answered your query. Let me know if I can assist you further."
},
{
"id": 68581,
"tgt": "What caused a sore lump in my abdomen?",
"src": "Patient: I have a sore lump, almost feels like a muscle strain in my abdomen, Its about 2 inches in diameter starts in the top of my belly button and goes above it about center. I did some strenuous activities over the weekend and I don't know if I did something bad. Please help. Doctor: Welcome to Health care magic.It could be umbilical hernia, as you describe (some strenuous activities) which can only happens in the hernia when the bowel in the herniated content with peristalsis will feels like that.The pain could be due to the strangulation of the herniated content.Suggest you to see your GP as soon as you can and get it examined and ask for an ultrasound - helps to evaluate the nature of the lesion, its source and extensions and any associated pathologies. Further it cane treated accordingly.Hope it helps you. Wish you a good health.Anything to ask ? do not hesitate. Thank you."
},
{
"id": 106153,
"tgt": "Can I take benylin extra strenght with reactine ?",
"src": "Patient: I have sore throat and some phlegm dripping down the back of my throat. I've already taken a reactine but wanted to know if okay to also take 30ml of benylin extra strength. Doctor: Hi, . What is your age and Gender? Since how long do you have this Problem? Since you said you have a Sore throat, which means you are suffering from Pharyngitis. Kindly get yourself examined by a ENT Specialist so that he can examine and rule out any other health problem. Benylin is precribed for dry cough, but is contra indicated in various health issues. You have not mentioned if you have any other health issue. Kindly do not take any medicine until confirmed by your Doctor. You may require antibiotics if a Infection is confirmed as Benilyn and Reactine will just give you a Symptomatic relief. Get adequate rest and drink plenty of fluids. Do warm water gargles several times a day. Wish you Good Health, Take Care."
},
{
"id": 185822,
"tgt": "What causes red spot between the teeth?",
"src": "Patient: hello doctor,i am a 23 year old male,i am 5'7 and 154 pounds and i dont smoke.I dont floss but i do brush regularly.but a couple days ago i noticed a small red spot appeared on my gums in between two of my front teeth.Its inbetween my bottom teeth and its as high as it can go on my gums down there.no pain but it does bleed easily when i floss there.Any idea what this is?Its scary to look things up online. Doctor: thanks for your query, i have gone through your query. the red point over the gums and bleeding could be be cause of the inflammation of the gums secondary to the deposits over the teeth. consult your oral physician and get your self examined and go for oral prophylaxis(cleaning of your teeth) and maintain oral hygiene after that. do saline gargling or rinse your mouth with water after eating anything. i hope my answer will help you. take care."
},
{
"id": 160997,
"tgt": "Suggest treatment for metabolic propionic acidemia in a child",
"src": "Patient: Hi, my grandson is 14 months old and he has been diagnosed to be having metabolic propionic academia. He is not in severe condition, he is under controlled diet and constant medication and regular medical care. I want to know if there is treatment, genetic treatments and if ALT auxiliary liver transplant is the solution and what would be the risks and benefits and would this be successful solution . Doctor: Hello, Unfortunately, curative treatment is not available. Newer techniques like gene therapy are not routinely available. In severe cases liver transplantation may be required. Consult a pediatrician and he will direct you accordingly. Hope I have answered your query. Let me know if I can assist you further. Take care Regards, Dr Shinas Hussain, General & Family Physician"
},
{
"id": 91836,
"tgt": "How to cure severe abdomen pain ?",
"src": "Patient: Hello my fiance is 22 years old and he has been a sharp abdomen pain in his lower right side but varies from there to a little higher. He has been light headed, dizzy, and has had a slight fever. The pain comes and goes but almost brings tears to his eyes it lasts about 30 seconds. He still has hid appendix, and Gaul bladder but hasn't been vomiting or any nauseous. Has regular bowel movements. A little more information he has had two hernias in his high pelvic area. Please help as much as possible thank you:) Doctor: hello,I undersand your concern and my advice is to go immediately to the nearest emergency room because these symptoms could be a sign of appendicitis and this is a first grade emergency. You mentioned also hernias and these symptoms could also be conected with incarceration which is also an emergency. I hope you will listen to my advice and bring your fiance to ER."
},
{
"id": 207860,
"tgt": "Could anxiety be due to PTSD?",
"src": "Patient: Hi my names Zoe and i m 15 and i suffer from ptsd i am having problems via i would like to know if they are connected. I feel enclosed all the time. I feel stressed out all the time. I feel angry and like i cannot be bothered any more can you please tell me if this is linked to my ptsd thanks Doctor: HII admire you for positive outlook.PTSD is the one of the type of anxiety disorder.In PTSD anxiety related to past traumatic events, flashback, hyper vigilance and avoiding to that situations.Stressed out all times suggest PTSD combine with GAD means generalized anxiety disorder they both are in same spectrum.Sometimes it combines with depression which make you irritable.Proper treatment is must.This kind of condition can be treated with mild dose of benzodiazapine and mild dose of SSRIIn SSRI mild dose of escitalopram, sertaline and paroxetine like drug is very useful. But before taking this medicines you have to consult psychiatrist and evaluate your self.After proper evaluation ,he will guide you further.Get well soon.Thank you."
},
{
"id": 73354,
"tgt": "What causes chest pain?",
"src": "Patient: For over one year I have been experiencing periodic pain in my chest immediately after I swallow any type of meat...chicken or red meat. It is very painful...kind of feels like an air bubble is in there. Yesterday I again experienced it after eating a small bite of chicken breast. I became sick to my stomach...felt nausated...starting sweating...felt like I was going to pass out. I'm thinking I need to get this problem looked at. What type of doctor should I see? Doctor: See a Gastroenterologist. He or she will likely start with a barium x-ray of your esophagus and may proceed to doing endoscopy."
},
{
"id": 172171,
"tgt": "What could cause lumpy pimples on body of 8 month old child?",
"src": "Patient: My 8 mth old has these little pimples on his back that are lumpy.they were weepy then disapeared by themselves.its been about 3 wks but now he has 3 which have formed on his head.he also has1 on the back of his knee.he has had problems with ecema but he drinks presciption formula elecare and has for bout 3mths.he is a twin and i have noticed my other son got afew on his chest afew days ago.any idea wot it could be?? Doctor: Hi dear,I understand your concern. But dehydration due heat wave really give tiredness. I suggest - ORS, it will increase his tonus;- Zincovit 1 tablet daily for 1 month;- Spirulina 1 capsule daily for 1 month.Cardiologist is not required. If his haemoglobin is normal,then don't worry, give him fresh juices, more fruits, give him a rest."
},
{
"id": 30125,
"tgt": "What causes stomach pain and tenderness in neck/jaw after sinuplasty procedure?",
"src": "Patient: I had a sinoplasty procedure done about 60 hrs ago and I felt fine the following morning. As the day went on I became lethargic and felt lots of pains and discomfort.Last night, 1 day post op 24 hrs ago I had a temperature of 102 that went down after hydrocodone I have had no appetite since the surgery. . About 24 hrs ago I had a temperature of 102 that went down after hydrocodone. This morning I had a low temp of 97, and later an elevated temp of 99 I am experiencing some minor pain in my stomach as well as very low energy and tenderness in the back of my neck/jaw below my ear. Should any of this concern me ? never mind I thought this was a free service Doctor: FESS(Functional endoscopic sinus surgery) usually does not cause severe postoperative sinus pain. After the procedure, it is important to keep the nose as free from crust build-up as possible. To achieve this, the surgeon may perform a lengthy cleaning two to three times per week or the patient may perform a simple nasal douching several times a day. Normal function usually reappears after one or two months. In patients with severe sinusitis or polyps, a short course of systemic steroids combined with antibiotics may quicken recovery.(thanks)"
},
{
"id": 106709,
"tgt": "Does fracture at L1 cause a backache?",
"src": "Patient: I was in a serious car accident 2 yrs ago. I crushed my legs. And had a compression fracture on L1 among a few other things. The pain in my back is horrible. Seems to start between lower shoulder blades and radiate outwards as the pain becomes severe. Is this pain from the fracture. Or could it be from something else. Doctor: Hello and Welcome to \u2018Ask A Doctor\u2019 service. I have reviewed your query and here is my advice. If it starts from shoulder blades, L1 injury should not be the cause, you likely have DJD in the cervical area, and also now that you damaged the Lumbar area, that is the cause of your pain. Hope I have answered your query. Let me know if I can assist you further."
},
{
"id": 44143,
"tgt": "PCOS, given Ovafar, asked to have regular contact from 9th day to 19th day. How frequent must the contact be ?",
"src": "Patient: I have just started my treatment for getting pregnant. I have PCO; and I am given Ovofar 50mg for 5 days to be taken from the day 2 of periods. I am asked to have regular contact from the 9th day to the 19th day. I want to know whether we need to contact every 24 hours; because me and my husband will not be together on the 14th day. How frequent should the contact be in these days?? Doctor: Hello The most fertile period of yours fall between 9th to 19th day and since there are maximum chances of pregnancy during this period,you ate advised for sexual contact as many times as is possible for both of you. The more you meet,more are the chances of conception during this period. There is no binding that you have to meet once in 24 hours.You can meet as much as you wish. OVOFAR will help in ovulation induction in you. Thanks"
},
{
"id": 137495,
"tgt": "Suggest treatment for chronic tail bone pain and numbness in the knee",
"src": "Patient: hi doctors, my name is anita and im inquiring regarding chronic pain ( as of yesterday evening ) in my tail bone, i did not fall just almost tripped from behind but caught myself, since then from my knees down i feel numbness or tickling sensation and the pain is not aliviating, can i get your input and do i have to seek immediate help or home excercise is okay ? Doctor: Hello,I have studied your case and most probably you are having disc herniation with nerve compression in the spine. I would recommend you to get some rest. Take hot water fomentation in the back and also kep your leg elevated over pillow. You should also take tablet Pregabalin M one daily.This will decrease nerve edema and also take care of the pain. I hope this answer will be useful for you. Let me know if there is any other followup questions.thanks"
},
{
"id": 2811,
"tgt": "Could anal sex cause pregnancy?",
"src": "Patient: My boyfriend and I tried anal sex, he didn t have a condom. I know about STD s i got tested, it came out negative. My question is, can I be pregnant? It was 16 days after the first day of my last period. He rubbed his tip near the opening of my vagina but he only had pre-cum. What are my odds? I m really scared. Doctor: Hi,I understand your concerns but anal sex cannot cause a pregnancy. it is unlikely that precum causes a pregnancy if the penis was not introduced inside the vagina despite the fact that you had sex at the time of your possible ovulation. I recommend that you take a plan b pill to make sure. Wait until the date of your next period, if it doesn't come get a pregnancy test done.Hope I have answered your query. Let me know if I can assist you further. Regards,Dr. Salah Saad Shoman"
},
{
"id": 191048,
"tgt": "Tingling sensation on my tongue with blood blister on my gum",
"src": "Patient: Hi, My jaw on the right side has been hurting for at least 1 week. My tongue on the right side also had a tingling sensation last week which is now gone. Now this week I noticed a pencil tip size blood blister on my gum directly behind my last tooth on gum. one day its there and the next day its gone .What could be the cause of this I a starting to worry. Doctor: It may either be erupting last molar which is compressing upon the nerve supplying the oral cavity. Kindly let me know your age. Or it may be Folic Acid deficiency. You may take Tab. B-FOLCIN (FOLIC ACID) 5 mg twice a day. Use a mouth wash like HEXIDINE gargles four-five times a day. Cover the blister with pain-killer gel like MUCOPAINE before taking food. Do not worry about MUCOPAINE going into stomach as it is medicine."
},
{
"id": 181263,
"tgt": "What causes discomfort at the site of wisdom tooth removal?",
"src": "Patient: Hi. I got my wisdom removed Friday evening due to an infected wisdom tooth. I'm worried about getting dry sockets. I also noticed that it still looks infected even if my molar was removed. My doctor gave my antibiotics. Is it normal for me to feel/see the stitches move? When will I know if I have dry sockets or not? Doctor: Hi..Welcome to HEALTHCARE MAGIC..I have gone through your query and can understand your concerns..As per your complain it is normal to have some pain and tenderness at the site of wisdom tooth extraction for a period of a week to even ten days..Stitches can at times move or fall off if not placed properly..As it is about a week of extraction, and in case if there is no severe pain then the chances of dry socket are less as the dry socket generally becomes symptomatic after 5 to 6 days of extraction and is a severely painful condition..I would suggest you to go for a clinical examination and get evaluated and if there is tissue filling the socket then there is no dry socket formation and the extraction site is healing well..As of now you should do warm saline gargles, gargle with antiseptic mouthwash and maintain a good oral hygiene and keep the extraction site clean..Avoid putting tongue again and again at the extraction area..Hope this information helps..Thanks and regards.Dr.Honey Nandwani Arora."
},
{
"id": 223174,
"tgt": "What should I do after taking the last pill of minigynon?",
"src": "Patient: hey i got the shot on the 26 of october and started taking minigynon of the 2 of november i started my peroid on the 12th of november and it hasnt stopped,i hvnt stopped taking the pills, today is the last pill and i am not sure what to do nx do i continue on the pill of give a break Doctor: Hello and Welcome to \u2018Ask A Doctor\u2019 service. I have reviewed your query and here is my advice. You should continue your pill and give break as usual. Hope I have answered your query. Let me know if I can assist you further."
},
{
"id": 12873,
"tgt": "Can rashes occur in coxsackie virus infection?",
"src": "Patient: Hello,I was diagnosed with coxsackie virus five days ago. It's main symptom for me was the rash on my hands that was red and painful. Is this aggravated typically by heat? My hands seems to start feeling better until I start to do more with them and then they become very warm and inflamed again. Doctor: Hi,Erythematous rash on skin may occur in coxsackie virus infection. It may vanish in few weeks. It may be aggravated by heat.There is no need to worry as it is not serious condition. Hope this helps.Dr.Ilyas Patel, Dermatologist"
},
{
"id": 76579,
"tgt": "Suggest treatment for chest congestion , ear infection and fever",
"src": "Patient: My 12 year old has been sick for 2 weeks. started with congestion and typical allergy symptoms then he started running a fever. tookto Dr found 2 severe middle ear infections started amoxicillin 7 days later his fever is higher and has a persistent cough. He has now been on z pac for 3 days still has a low grade fever cough and head aches I am considering taking him to the ER as I think there is more happening here. We did blood work to rule out pneumonia, mono, kidney failure and leukemia Doctor: HIWell come to HCMI really appreciate your concern, it must the respiratory infection causing the fever, to confirm this x-ray chest is must, with routine blood test, condition can be managed with antibiotic but it is better to confirm the diagnosis first, looking to age it is not advisable to put the child on antibiotic hope this information helps."
},
{
"id": 35581,
"tgt": "What causes severe mouth ulcers?",
"src": "Patient: Hey, I currently have about 18 mouth ulcers. My mouth keeps getting more and more ulcers but days previous to it happening my mouth felt very dry and strange/ roof of mouth felt itchy as well. Among the ulcers I have another problem where the tissue under my tongue appears to be turning white as if they are covered in multiple ulcers forming together. And this is the most painful part of this.im not sure if they are ulcers as well or something differ It has been four days. Should I see a doctor? Or let it take its course? Doctor: Hi,Welcome to the HCM There can be many causes of mouth ulcers like ,1.gastric infection or ulcer2. vitamin B12 defeciency3.Vitamin C deficiency4.folic acid deficiency5.fungal infectionMouth ulcers to this extent should not be ignored and should be treated properly.My opinion would be first you get blood test done of vitaminB12 if you are vegetarian .Then you may take tab.folvite 5 mg daily and tab limcee .1 tab daily , orasep gel to decrease the ulcer .Since fungal infection and gastric ulcer has also to be ruled out for that ,get checked by physician in person and discuss further about endoscopy and vitamin B12 intramuscular injection course.Regards,Dr.Maheshwari"
},
{
"id": 84319,
"tgt": "Suggest remedy for side effects of disprin",
"src": "Patient: Hi, I had a headache last evening and I took disprin, but it reacted with me. I got swollen puffy eyes and I could not open them properly also. When I went to a doc, she was herself surpised, but she gave an IV injection and now it opened a bit. But, still I need ur suggestion. Doctor: Hello Take it full stomach so that hyperacidity and risk of peptic ulcer can be avoided. Disprin Tablet may increase the risk of bleeding, therefore not recommended in patients with peptic ulcer disease or any gastrointestinal bleeding. Disprin Tablet is not recommended in children with varicella infections or influenza-like illnesses due to the risk of Reye's syndrome. Hgh doses of Disprin Tablet may cause sodium and water retention and increases the risk of heart failure. Therefore not recommended in patients with heart failure and in patients with the known history of aspirin-sensitive asthma. Hope I have answered your query. Let me know if I can assist you further. Take care Regards, Dr. Ajeet Singh, General & Family physician"
},
{
"id": 72192,
"tgt": "Should I be concerned for a breathing problems causing chest pain?",
"src": "Patient: Hello, i am a 14 year old girl and i have breathing problems. I have to breathe by taking in deep breaths and yawning. Now i am unable to do that and when i breathe or try to my chest starts to hurt a little bit the pain lasts a few seconds and then stops i talked to a few doctors and they all concluded that i have allergies and the meds they perscribe do not help at all so they perscribed an inhaler saying it is shortness of breath. Is this a serious matter? Is it possible to cure? Doctor: Thanks for your question on Healthcare Magic.I can understand your concern. At your age, we should definitely rule out asthma for your symptoms. So better to consult pulmonologist and get done clinical examination of respiratory system and PFT (Pulmonary Function Test).PFT will not only diagnose asthma but it will also tell you about severity of the disease and treatment is based on severity only. You will need longer acting inhaled bronchodilators (formoterol or salmeterol) and inhaled corticosteroid (ICS) (budesonide or fluticasone).Oral combination of antihistamine (levocetrizine or fexofenadine) and anti allergic (montelukast) once a day at night is also beneficial.Don't worry, you will be alright with all these in 2-3 months.Avoid stress and tension, be relax and calm. Hope I have solved your query. I will be happy to help you further. Wish you good health. Thanks."
},
{
"id": 177249,
"tgt": "What causes difficulty walking along with fever?",
"src": "Patient: Hi Doc my fren s 4 yr old daughter had fever abt 102 and had given her crocin as well as para suppository and the fever calm down. The next day when she got up from bed she was unable to walk properly. What could be the caused for this ? We are worried but dont know wat to do. The doc in the hospital says its normal. Please help Doctor: Hi, as your doctor told it could be normal. Due to weakness the child might not be able to walk. Again in viral conditions there could be excessive body pains. Since your doctor has seen I think he would be a better judge, because am not examining the kid.Tc"
},
{
"id": 152159,
"tgt": "I am 35 years suffering from brain TB,taking rifampicin,pyrezinamide,solonex. Do they have any side effects ?",
"src": "Patient: Hi docter, i m 35 yr old lady suffering from brain tb ,and takin medince for 5 yrs ,i am taking rifampicin,pyrezinamide ,ethionamide,solonex,selzic, benadon .pls tell me if there any side effect of these drugs,as i want to start my family after stoping these drugs.and pls tell me what to eat right so that i come over from the side effects of these drugs..i m very much worried ..pls help me docter regards Prerna bhatnagar, kanpur Doctor: hi Eat fresh fruits,vegetables, drink plenty of water. exercise regularly, like yoga and pranayam. take sleep enough"
},
{
"id": 39617,
"tgt": "How to handle issues related to snake bite?",
"src": "Patient: Respected Doctor, my sister got her right hand middle finger bitten by the snake about ten days back. presently she has recovered and normal. dorsal side of the finger is blackened fully, and ventral side three fourth. The one third tip of the finger is senseless . Please advice Doctor: Dear Friend.Hi , I am Dr Anshul Varshney , I have read your query in detail , I understand your concern.It seems she has developed Cellulitis if the bitten area.But if that has blackened, and no blood supply and sensation is there, it might require an amputation.Otherwise, she would have spontaneous self amputation.So, get Doppler study of that area, to see for strength of blood supply and share reports with us.This is my personal opinion based on details available here. If you still have any other query, you may please ask me.Stay Healthy. Dr Anshul Varshney , MD"
},
{
"id": 88606,
"tgt": "What causes discomfort and swelling in the abdomen?",
"src": "Patient: 3 days ago my abdomen began to fill swollen and uncomfortable and when I pressed on my left side it was tender...then two days ago I was having pain in the center under my brest bone that radiated to my back...yesterday it was back to the swollen feeling and today I have twinges on my left side. Any ideas? Doctor: Hello and welcome to HCM,Fullness of abdomen and feeling uncomfortable and swelling on left side suggests splenomegaly.However, clinical assessment and ultrasound of the abdomen is required for assessment of abdominal organs.You need to consult your primary health provider for interpretation of ultrasound and thus further management.Enlarged spleen can occur in number of conditions like infections, hematological disorders, storage diseases etc.Thanks and take careDr Shailja P Wahal"
},
{
"id": 51674,
"tgt": "Can you suggest a good kidney doctor in India ?",
"src": "Patient: My cousin has a problem with 1 kidney which is negligibly functional and the other one appears to be shrinked as per the report, please suggest the best doctor/surgeon/hospital in India in a medium cost of budget. And whether it is serious and is curable. Doctor: hi u need to admit him in kidney hospital. there is no 1 kidney hospital in civil campus of ahmedabad. it is trust hospital so it is economy with high standard treatment. its Institute of Kidney Diseases and Research Center, Gate No 3, Civil Hospital Campus, Asarwa, Ahmedabad, 2685600/01"
},
{
"id": 193385,
"tgt": "What are the signs of puberty in men?",
"src": "Patient: I am 16 and i have not gone through puberty yet, however when looking on the internet to see if this is normal i found a chart showing the different stages of puberty. Stage 2 was pubic hair growing at the base of your penis, and i have been at this stage for quite a while now and feel as if i will never go through puberty. Should i go to my doctor and see if there is any problems? Doctor: Hello, Since there is pubertal arrest I feel there is need to look for hormonal causes of delayed puberty. This can be done by testing the sex hormone profile for which you need to visit a doctor. Hope I have answered your query. Let me know if I can assist you further. Take care Regards, Dr Ashok Kumar, Psychiatrist"
},
{
"id": 196080,
"tgt": "What causes pain in the right scrotum?",
"src": "Patient: My right scrotum was in pain since last week but yesterday I was shocked to find a hard lump attached to the ball inside the right sack which pains a lot. It pains even more when touched or pressed. Please help cure this or suggestion on how to sure it. Doctor: Hello Thank you for trusting HCM Dear possibly your suffering from epididymitis. Have you suffering from any Stds?? Other causes for Testicular pain hydrocele, inguinal hernia, kidney stones, spermatocele, orchitis etc. cap. Doxy 100mg twice a day for 1week consult your doctor he will examine and treat you accordingly."
},
{
"id": 36570,
"tgt": "Can alcohol be consumed while on medication for H pylori infection?",
"src": "Patient: Good afternoon, i was diagnosed with H-Pylori infection, and have been taking Omeprazole DR 20 MG Capsule ( twice daily for 10 days) & Pylera Capsules ( 4 times per day for 10 days) today is my 5h day. aAlthough because of my work schedule its been impossible to actually complete the daily dosage in full, i have been taking them at least 3 times per day. Today i only took my morning dosage at 5am. Aside from slight discomfort i really haven t had any other symptoms, I was diagnosed with H- Pylori once before in 2007 and took some antibiotics but I m not sure if it was the same ones. I would like to know if with the dosage I ve had so far or with the dosage I ve had today (1 of Omeprazole & Pylera Capsules) am i able to consume alcohol? and then begin my regular dosage tomorrow? Im 37 years old, 215lbs, 5 foot 8 inches, over all good health aside from this bacteria and the fact i would and should loose few pounds. Doctor: Hi and thank you so much for this query.I am so sorry to hear about this diagnosis. Alcohol accelerates the destruction of drugs from the system. Please. avoid alcohol while on medication for best results. I hope this helps, I wish you well. Thank you so much for using our services and feel free to ask for more information and clarifications if need be."
},
{
"id": 151624,
"tgt": "Myasthenia gravis, continuous weakness in legs. No relief with wysolone 7.5 mg. Alternatives?",
"src": "Patient: Hi I am from India. My daughter is a patient of myasthenia gravis and taking mestinon (60 mg) for the last 4-5 years. As she start complaining about weakness in her legs doctor prescribe her wysolone tablet 7.5 mg a day but the tablet is not doing well and she always use to complaint about weakness in her legs. please suggest what should we do. thanks Doctor: Hi, Welcome to Healthcaremagic. Steroids like Wysolone are one of the best and final resort treatment in Myasthenia.The response to such drugs is seen after a lag period of few weeks. I suggest to continue on the same medication and wait for some time so that their steady concentration is achieved and show effective response. Do not go for any other medications as the treatment she is on are right and suitable to the severity. Wish you good health."
},
{
"id": 220024,
"tgt": "Can pregnancy occur despite having heavy periods?",
"src": "Patient: Hi, ahries right now ? Please type your query here...ok so my bf & i had unprotected sex & he came inside me 3 weeks ago, & then i got a heavy period, but the day before I got my period we had unprotected sex & he came again. Does it matter if I had a heavy period? Could I still be pregnant? Doctor: Hello dear,I understand your concern.In my opinion once you get the periods there is no chance for the presence of pregnancy.But if the period is delayed and then you got the heavy periods then it gives a suspicion of miscarriage so it needs to be checked for the pregnancy.As per your information you got heavy periods on time so it cannot be considered abnormal.And ejaculation inside the vagina the day before the onset of periods might not be the reason for heavy periods .As the day before periods comes under safe period there is no chance for pregnancy.So relax.Avoid stress.There is no scope for the presence of pregnancy.Hope this helps.Best regards......."
},
{
"id": 131057,
"tgt": "What causes pain in shoulder, lower back, wrist, elbow, ankle and knee?",
"src": "Patient: Hi, I am a 16 year-old female of normal weight. I have been having very severe shoulder pain that comes and goes for the last couple of years. It is in the left shoulder, but sometimes in the right as well. Lately, however, I have been getting pain in my lower back, wrist, elbow, ankle, and knee. I got an X-ray done, but nothing came up. Any ideas? Doctor: If cervical spine x-ray was normal, nerve conduction test should be done of axillary nerve. X ray on pelvic should be done to exclude ileosecal joint arthritis. Take Gapapentin 100 oral tablets 3 times daily."
},
{
"id": 39068,
"tgt": "What causes swollen axillary lymph node?",
"src": "Patient: I am concerned about my son's swollen axillary lymph node. It is 1cm in size, movable, soft but has been there for 2 1/2 months. The doctor has said there is no need for biopsy, as it is small ,not getting bigger and there are no other prominent nodes. His CBC shows a normal WBC profile but his RBC is microcytic, hypo chromic. His eosinophils are about 10 times normal . He has been given worm medicine and has been on a course of iron therapy for 2 months but the follow up blood work still shows no change. Should the lymph node be biopsied? Doctor: Hello,Welcome to HCM,The axillary lymph nodes filter the lymph from the arm, chest wall and breast. It is often difficult to feel normal axillary lymph nodes. Not all enlarged axillary lymph nodes feel the same. Axillary lymph nodes may swell from an injury or infection to the arm or hand. A rare cause of axillary swelling may be breast cancer or lymphoma.I would suggest you to take your son to pediatrician for physical examination and to find out the reason for the palpable axillary lymph node.Thank you."
},
{
"id": 222713,
"tgt": "Suggest treatment for hematuria during pregnancy",
"src": "Patient: I m 35 years old and 10 weeks pregnant. 1 month ago: uti culture positive (hematuria). 1 week later repeat culture: negative. Mild symptoms for a month. Today: quick test at obgyn positive for hematuria. Culture sent to lab. Is it possible that i have a low grade UTI? I have had them chronic since 18. I am concern about health of my kidneys, etc. if left untreated. Thank you. Doctor: Hi,Urinary tract infections are very common in pregnancy especially in women who have had them before. These infections are more likely to happen in initial weeks. Since you have had them twice already in this pregnancy, you should get fully evaluated including renal function tests which include serum creatinine, urea and also get a renal ultrasound, which could detect any stone or abnormality .Also you may have to consider long term antibiotics to prevent the infection from recurring. For this a opinion from a renal physician will be required.Please get the infection treated promptly as not treating it can have a negative impact on the pregnancy, lead into preterm,hypertension in pregnancy.Hope this helps.Regards."
},
{
"id": 182118,
"tgt": "Suggest remedy for tickling of tongue",
"src": "Patient: About a month ago the tip of my tongue started tingling at random intervals, maybe once or twice a day. This past week it has been tingling 8 or 10 times per day, lasting about 30 seconds each time. Today my tongue started tingling, and now my left fingertip is tingling. Should I be concerned about this? I have diabetes and I take metformin and insulin. Doctor: Thanks for your query, I have gone through your query.The tingling sensation in the finger tip and tongue can be because of the diabetic neuropathy or secondary to any nerve injury. Nothing to be panic, consult a oral physician and get it evaluated. Mean while you can take nerve regenerating drugs like multivitamin complex like(neurobion). You have to keep the blood sugar level under control. Once the blood sugar level comes to normal, the tingling sensation gradually reduces.I hope my answer will help you, take care."
},
{
"id": 119533,
"tgt": "Suggest remedy for knee, leg and sole pain and varicose veins",
"src": "Patient: Thanks. I am having knee pain and also my leg and soles are paining and burning for the lst five years. I am also suffering from varicose veins. I had consulted nuero surgeon and after a test he was of the opinion that the blood flow in both the legs was hampered and he prescribed me Pregalin M M75, Aldonil 50 and Rejunex OD and was taking these medicines for the last one year but I am not getting any relief. On the contrary, the pain has increased and I am finding difficulttowals even onem. Pl. advise. Doctor: Hello, In this case, I do agree with treatment of your doctor. He has already prescribed you right medicines to help you. But if you are feeling continuous pain which is hampering your routine activities then there are some nerve block procedures which can be done by expert neurosurgeon or a neuro-physician. You may ask this from your doctor. Take care. Hope I have answered your question. Let me know if I can assist you further. Regards, Dr. Mukesh Tiwari, Orthopedic Surgeon"
},
{
"id": 185533,
"tgt": "Why are my gums black and teeth discoloured?",
"src": "Patient: The bottom of my gums are black and the inside of my cheecks are rough and discolored a greenish color, and im curious to know what it is or causing this? But I also noticed when I eat peanut butter the inside of my cheeck swells for awhile and my mouth will slightly itch....could this be the cause of my mouth issues? Doctor: Hello!Welcome to HCM.You did not mention if you use tobacco or any other habits.This discoloration can be familial,which runs in family.You do not need a treatment if this is the reason.Accumulation of calculus and chromogenic bacteria can also cause this.Get a complete scaling done with your dentist.Quit any tobacco related habits.Maintain proper oral hygiene.Peanuts do not cause discoloration of your teeth and gums.You seem to be suffering peanut allergy.Avoid peanuts strictly.Regards."
},
{
"id": 159920,
"tgt": "Is radiotherapy necessary if I have adenoma which is benign ?",
"src": "Patient: i had under gone invasive pituitary macroadnoma decompression on march 27th 2011. now i am taking rest. mri to be taken in june last. the neuro surgeon suggests radio therapy only as the remedy if the remaining adenoma is found growing. the adenoma is benign. not affected endocrine system . please give your valuable suggestions. vijayanvaliyapurayil@yahoo.co.in Doctor: Hi Vijayan, . Since it is a Non Functioning or Benign Tumor there is nothing to be concerned about. You should be getting Radiotherapy done as directed by your Doctor and go for regular follow up as Observation, surgery and medications are the main treatments for pituitary tumors. Wish you Good Health, Take Care."
},
{
"id": 80557,
"tgt": "What is the reason for chest pain?",
"src": "Patient: Been having left side chest pain for over a year. Pain gets worse when laying on my stomach and subsides when standing. Pain will last anywhere between 3 min and 10 hours. Had an EKG, heart ultrasound and holter monitor with no signs of heart issues. Tried many antacids with no response to them. Pain is not influenced with exercise. Have mad multiple blood tests all showing normal. Forcing gas out as what feels like an inside burp seems to relieve some of the pain.Pushing on my left rib cage and breast also seem to help. Doctor: Hello dear, thanks for your question on HCM. Since your extensive cardiac work up is normal, no need to worry for cardiac cause. In my opinion you are having GERD ( gastroesophageal reflux disease ). It is due to laxity of gastroesophageal sphincter. Because of this the acid of the stomach tends to come up in the esophagus and cause the symptoms. Along with antacids, you need to follow these lifestyle modifications for better symptomatic relief. 1. Avoid hot and spicy food. 2. Avoid stress and tension. 3. Avoid large meals, instead take frequent small meals. 4. Go for walk after meals. 5. Keep 2 - 3 pillows under head in bed to prevent reflux. 6. Avoid alcohol and smoking if you have these habits. 7. Loose weight if you are obese. Don't worry, you will be alright."
},
{
"id": 45981,
"tgt": "Can excessmastrubation cause pain in kidney?",
"src": "Patient: hi this is Omar i am having back pain n its been almost 14 days its totaly bearable but still its a pain its just uper side of my hips not inside the bone some time in kidney's place and sometime just down to that place both sides right and left i mean i can walk properly i can sit but during a movent i can feel that its a pain still there yeh i wana ask is it due to masturbating also ? i mean all i can remember i masturbated n i went to bed and again when i wake up my back was already paining and from that day its still there its nt increasing and neither decreasing just some time changes the place as i said some time kidneys place n some time very down to that help me pls? Doctor: Hello and Welcome to \u2018Ask A Doctor\u2019 service. I have reviewed your query and here is my advice. Masturbation is a normal activity and it has no harmful effects though some men may feel guilty at times for they feel they are addicted to it. Sometimes harsh masturbation may cause injury to the penis. Lower back pain should not be a cause for masturbation. Despite the possibility of addiction, there are no harmful side effects. Let go the negativity. You can meet a therapist for your back pain if you are not relieved from it. For back pain you can take some painkiller and check the mattress in which you sleep. If you are still in discomfort you should see a doctor, hope I have answered your query. All the best. Dr Nupur K"
},
{
"id": 184191,
"tgt": "How to cure swollen foliate papillae?",
"src": "Patient: I have swollen foliate papillae. I'm a bit concerned... I have been going to my doctor for him ignore my symptoms. My tongue has been burning, some itching as well on tongue and throat at times. I was treated for yeast due to slimy coating on side of tongue. Now, the side of my tongue is red, burning a bit, have a weird taste and is swollen. At times, I get a swollen side of the tongue, feels bruised and I get a sore throat and ear on that same side that is affected (mostly the right side). I was thinking food intolerance or something. Not sure! Doctor: hellooo...having gone through your query i can say that it can be due to some fungal infection mostly and also vitamin deficiency in the diet....so nothing too much to wory...stop having too much spicy food...then drink lots and lots of water ..take vitamin b complex or multivitamin tablet for two weeks...apply antifungal topical application to your tongue....brush twice daily....do chlorhexidine mouth rinse..above all reduce any stress and tension...tension is cause of most diseases...be happy....have a healthy day!!!"
},
{
"id": 140048,
"tgt": "What causes spinal sinosis?",
"src": "Patient: My twin sister was diagnosed with Spinal sinosis I am a cancer patient was diagnosed with advance stage 3 Breast Cancer in the right breast I have had many complications and I also have a blood disorder Cical cell red blood thelesemia I have gone throw all treatment for the Cancer and I have had Right radical mastectomey removal of all lymph nodes of arm pit and left mastectomey chemotherpy radiation you name it. My question is my bones and spine have been hurting me so much after and with the treatments that I was on...Should I be tested for Spinal Sinosis Doctor: Hello, You mean spinal stenosis right?Since you have been experiencing symptoms such as pain, you should get evaluated further in order to understand correctly the cause.Degenerative disease of the spine, stenosis also, are possible conditions.Get evaluated by your Doctor and by imaging tests for a correct diagnosis. Hope I have answered your question. Let me know if I can assist you further. Regards, Dr. Erion Spaho, Neurologist, Surgical"
},
{
"id": 84543,
"tgt": "Is back pain a side effect of Actonel?",
"src": "Patient: I have chronic back pain, with muscle stiffness, for over a year now ( except for 2 days of a juice fast just this month). I also have lots and lots of yellow-white hard crystals in my stool. It almost looks like I have eaten sesame seeds when I have not eaten any. I ve been so frustrated that today I rinsed the stool and saved the crystals (since my back pain diminishes after having a bowel movement. I move my bowels up to 5 times a day at times so I am not constipated.) I am on Actonel monthly and calcium supplments 1200-1400 mg/day, but have been cutting back lately. My vitamin D levels are normal as per testing. I also drink skim or 2% milk daily for more calcium for my osteoarthritis. I also have muscle spasms in my feet and leg cramps at night on occassion. I am in my greatest pain at night, and wake frequently. An MRI showed some disc bulging last Fall. The muscles spasms are starting to interfere with my ability to bend and do normal routine chores. I have heard that back pain may be a side effect of Actonel. Please help. (Sorry for the description) Doctor: Hello, Yes, it is one of them. Common side effects of Actonel include upset stomach, stomach pain, headache, flu symptoms, muscle pain, diarrhea, constipation and joint or back pain. See your clinician. Hope I have answered your query. Let me know if I can assist you further. Take care Regards, Dr AJEET SINGH, General & Family Physician"
},
{
"id": 162843,
"tgt": "What causes pale skin, fatigue and body tremors?",
"src": "Patient: My 12 year old daughter often gets pale and shaky as well as fatigue. Her dad is a type one diabetic. We have had her tested with the 4 hour glucose test and it came back negative. She eats normally as well as drinks only water. She has had this problem since she was around 4 years old. Now she is involved in school sports as well. This occurrence happens more often when she gets hot. Any ideas? Doctor: Hello and Welcome to \u2018Ask A Doctor\u2019 service. I have reviewed your query and here is my advice. In this case, I will suggest a total blood analyses to have a check up that everything is ok. Especially, I will suggest a screen of thyroid hormones. It may be just her metabolism is itself like this and you don't have to worry. But first, let me have the total check up analyses and then let me know. Hope the best."
},
{
"id": 110430,
"tgt": "How to treat the back pain caused by falling?",
"src": "Patient: hey i'm concern about my lower back because when i was playing basketball i landed on my lower back real bad and i've waited a month for the pain to go away but it haven't. it seems if i do anything physical like working out, or anything my back goes out and its hard for me to stiff my back straight i'm 16 years old an i'm really scared this can cause me not playing basketball anymore Doctor: Good evening. Well you need to consult a spine specialist and get an MRI of the spine done. If the symptoms happened after a fall, one needs to rule out ligament tear or some minor fracture in the vertebra. Try using some analgesics and physiotherapy sessions. but do consult someone at the earliest."
},
{
"id": 6097,
"tgt": "Delayed periods, swollen breasts, white discharge. Taking prenatal vitamins. Negative pregnancy test. Pregnant?",
"src": "Patient: hi i am trying to get pregnant since my last day of my period witch was july 1,2012 it lasted 4 days only and we had unprotected sex since july 5.. i was suppose to get my period but it has not come i am 3 days late and i have a lot of cramping,swollen in my breast and discharge every time i think my period came its not it..its only white discharge or watery. i did a pregnancy test but it keeps coming out negative i did took prenatal vitamins just in case. also i already have a baby his only 1 year old and i thought i could get pregnant faster because of that. what should i do? can i be pregnant? Doctor: Hi, Thanks for query, As your pregnancy test shows negative result the chances of delayed period is much more. Due to hormonal imbalance you get this type of symptoms too. hormonal imbalance can occur due to tension,Anxiety,long working hours,shift duty,ovarian pathology etc. Previous delivery can not increase the chances of being pregnant. Wait for few more days.if you do not bleed repeat pregnancy test. Thanks."
},
{
"id": 188535,
"tgt": "Back tooth removed, no pain. Have bad breath after extraction. Normal?",
"src": "Patient: HiHad back tooth removed yesterday. No pain afterwards. Have white where my tooth was, think this is the blood clot. Area looks healthy, not red or swollen. From previous posts I see dry socket does not appear for a couple of days. What I am asking is straight after the extraction and still today 23 hours later I have bad breath. Is this normal? Doctor: Thanks for sharing your concern in HCM forum. The whitish area at your extraction site could be some food lodgment or some slough, which is not uncommon 24 hours after extraction of a tooth. The dry socket is a phenomenon which is still poorly understood to prevent it. It is a highly painful condition typically appear within 2nd to 4th day after extraction. As you have no pain , so still the area could not be considered as dry socket. You can start chlorhexidine mouth wash 4 to 5 times a day for one week and refrain from smoking for 3 to 4 days if you are a smoker, as a probable prevention of dry socket.Continue medicines prescribed by your doctor, if any. Continue your brushing teeth as usual avoiding injury to extraction site. Bad breath for couple of days after extraction in not uncommon. If you do not have any other oral problem, your bad breath will disappear with proper brushing and chlorhexidine mouth wash, and do not get tensed by thinking about dry socket. Its not a very common incidence. If something like that appears, consult your dentist. Get well soon."
},
{
"id": 204033,
"tgt": "red spot found after the tab test , what is the remedy ?",
"src": "Patient: 8 months ago I had a tab test done because I am a deputy and work inside a jail! There was no effect at all on my skin! I had another done yesterday and I have a red spot were it was injected which didn t happen last time! Could this still be a negative result? There is no bump just redness! Doctor: Hi,I can understand your concern for the tab testing and its different results. YOu should know that it is a screening test and is used to screen certain people who are at high risk for TB exposure.Positive results mean that a person is likely to have been exposed to TB. They may be due to a latent or active TB infection, or occasionally due to a false positive.Negative results for either test may mean that a person has not been exposed to TB, that the person is not infected with tuberculosis, that their immune system has not responded to the antigen in the test, or that it is too early to detect exposure.Hope this information relax you.Take care."
},
{
"id": 171546,
"tgt": "Diagnosed with Clostridium difficile",
"src": "Patient: My 12 month old smells like maple syrup. It is a very strong smell. He has been treated for C Dif twice in the last 45 days and is getting over hand/foot/mouth. He has been off the Flagyl for about 4 days now. I noticed the smell several days ago, but it is getting worse and more noticeable. Doctor: HiWelcome to the HCMI have gone through your question and understand your concerns. It seems that your child may be suffering from an inborn error of metabolism known as Maple syrup urine disease. The intermittent variant of this disease menifests at the times of illness leading to accumulation of branched chain amino acids in the body. These acids give typical odour of maple syrup in urine and body odour.You may take him for plasma amino acids analysis for diagnosis and management accordingly.Hopefully this will help you. I would be happy to help you in any further questions.Take care"
},
{
"id": 134400,
"tgt": "What causes shoulder pain and dry cough?",
"src": "Patient: Hi I went to my local Dr Friday having had pain in my right side and shoulder and was told because I had a fever it was probably infected or inflamed gallstones. I explained to Dr I was going on holiday in the Saturday and she said ok because your not in pain when I examined you and you seem ok go, take these antibiotics, rest loads, go on a strict fat free diet. if I was sick or skin started to turn yellow I needed to go to hospital. so I ve been doing all this,I ve developed a dry cough tonight (the Wednesday night) it started off just now and again but I went to bed about 2 hrs ago and it got really bad. I think the pain in my shoulder was to blame for most of it, but I wondered if I should be worried about the cough? it seems to have pretty much gone now. we are headed home on Friday where a scan letter should be waiting for me but I wondered if it s related? Doctor: hi,I do not think that the cough and shoulder pain are related. Cough , which is for most of the people a dry one is common. For that I will advice to take steam and do deep breathing exercises. Along with this you can continue your physician prescription. Now let's talk about your shoulder pain, which may be due to some other issue. if the pain is too much you can try initially taking hot water fermentation for it and do slow upper limb movements. for this you can simply clench your hands and do lift it up and down slowly and gradually with in your pain limits. let's add some exercise for respiratory system Along with this . try taking deep breaths when you raise you hands up and blow the air out through your mouth when getting the hands down. hope this helps. if the symptoms persists you can meet up a physician for the same."
},
{
"id": 151176,
"tgt": "History of cervical dissectomy. Have ankylosing spondylitis. Disc replacement required?",
"src": "Patient: I have had cervical discectomy/fusion in 2007 knowing due to spinal stenosis I was needing another in future. Which has come. I have found a wonderful neurosurgeon who is suggesting a partial disc replacement with some kind of RD cage...I m totally getting it wrong..but nervous. The fusion worked like a charm..and wandering why something different? I also have Ankylosing Spondylitis . Doctor: Hi, Thank you for posting yor query. I can understand your dilemma, when you are offered another treatment option, after having had a good response with the previous surgery. The choice of surgical procedure depends on the current knowledge, patient's disease condition, surgeon's preference and the costs involved. In 2007, disc replacement was new, and not many surgeons liked it. Now, with more experience, many surgeons prefer it. It is a safe procedure and complication rates are less. However, please discuss the pros and cons of both the procedures before you decide to opt for one. Best wishes, Dr Sudhir Kumar MD DM (Neurology) Senior Consultant Neurologist drsudhirkumar@yahoo.com"
},
{
"id": 161275,
"tgt": "Suggest treatment for cold and cough with wheezing",
"src": "Patient: Hello Dr My son is 10weeks old, had a cold and cough with wheezing and shallow breathing. Dr diagnosed bronchitis and prescribed Alex syrup, ambrodil syrup and started nebulizer initially every 2 hours, then every 4 hours. We went for a follow-up after 2 days, baby seems to be slightly better. But slight wheezing still present. So Dr asked to continue nebulizer and prescribed defcort syrup. I m concerned that this is a steroid and don t want to use it on my 2.5 month old baby. Can I skip the steroid? Doctor: Hi, No. If it is wheezing, steroid is the treatment along with broncho-dilators and antihistamines. Please continue the same treatment and if fever is there, you can defer steroid. Hope I have answered your query. Let me know if I can assist you further. Regards, Dr. Prasanna Lakshmi, Pediatrician"
},
{
"id": 83177,
"tgt": "Will taking detox cause pain and burning in feet and also jaundice?",
"src": "Patient: My fiance was drinking heavily for 6 months, has had diagnoses of hepatitis A and pancreatitis. She began self detox a week ago, and has been very successful at this, with no cravings, a return of appetite and feeling great. Then the bottoms of her feet began to hurt and burn, and has jaundice as well. She is 40, 5 8 , 200 lbs. She did not sleep for 6 days, but now is sleeping soundly and for 12 hours in the last 2 days. Do we need to be concerned with the jaundice or is it a natural reaction to the detox? Doctor: Hi,Is there any signs of diarrhea or vomiting? Is there any signs of fever? I advise you to:1. Do liver function test - LFT blood2. Stop detox and drinks, outside foods3. Consultation with your physician regarding jaundice and undergo treatment accordingly Take care. Hope I have answered your question. Let me know if I can assist you further. Regards, Dr. Indhu Priyadharshini, General & Family Physician"
},
{
"id": 51413,
"tgt": "Kidney condition, schizophrenic patient, unable to do dialysis, removal of infusion from arms. Alternatives?",
"src": "Patient: hi ! i have a brother wich the only way to survive is dialysis for him.unfortunatley his schrizofenic. the doctors consider that is imposibile to take him on dializys beacuse his take out the infusion from his arms.is there a way to get him to sleep while he is on dialysys? regards Doctor: Hi, Welcome to Healthcaremagic. yes, he can be sedated before taking him to dialysis. This has to be decided by the doctor in the dialysis department. well its not impossible . Wish you good health."
},
{
"id": 14939,
"tgt": "Developed rashes with itching and burning all over body after abdominal reconstruction",
"src": "Patient: my wife had major surgery last Tuesday for a complete abdominal reconstruction. she is scheduled to go home from the hospital tomorrow but has now v developed a rash that is spreading over her body. looks like mild hives and is both itchy and burns a but Doctor: Hi\u00a0Welcome to Healthcare Magic Forum\u00a0Looks like you are suffering from\u00a0acute urticaria.This is an itchy condition of skin\u00a0which leads to redness and development of\u00a0hives\u00a0on the skin on itching.Since she had a major surgery, so she must be on some pain killers.Both pain killers and antibiotics are a very important cause of development of hives.You can try if you can find out the pathology.\u00a0For the time being, start with some anti histamines.You might need a\u00a0tapering\u00a0course of these medicines.So if itching persists, consult a dermatologist for proper evaluation.\u00a0Finding out the cause of the itching is very important.Hope it helps\u00a0,Wishing you speedy recovery\u00a0.Dr Geetika Paul"
},
{
"id": 72193,
"tgt": "Is tietze's Syndrome, related with chest tightness?",
"src": "Patient: I am having swelling on my left chest.. along with tightness feeling... It's very annoying.. It's there for almost 6-8 months... Went to 4-5 cardiologists and all told me that heart is alright and my feeling is psychological... But I know how it feels... I have zeroed in on Tietze's Syndrome... can any Doc help me please...I am 40 years old male... Doctor: Thanks for your question on Healthcare Magic.I can understand your concern. First of all no need to worry for heart diseases. Tietz syndrome is costochondritis and it can cause chest pain and chest tightness. So follow these steps for better symptomatic relief in costochondritis. 1. Avoid movements causing pain.2. Avoid heavyweight lifting and strenuous exercise.3. Take painkiller and anti inflammatory drugs like ibuprofen or acetaminophen.4. Apply warm water pad on affected areas of chest.5. Wear rib belt.6. Avoid stress and tension, be relax and calm.Don't worry, you will be alright with all these. Divert your mind from this. Don't pay attention to this pain. Hope I have solved your query. I will be happy to help you further. Wish you good health. Thanks."
},
{
"id": 108527,
"tgt": "Can diarrhea and lower back pain be related?",
"src": "Patient: For the past month I have had light colored diarrhea, on and off. Then things started getting back to normal about 10 days ago. Well last Sunday I started having lower back pain on my left side, my stool is back to normal but it is lighter then normal and today I noticed some blood when I wiped. I have been to the doctor for my back pain and did mention the diarrhea but he said if you have no fever and felt fine, and no blood then it should be ok. So now I noticed the blood and I have another appointment Thursday and will definitly mention the blood, Can these two issues be related? Doctor: HIWell come to HCMCondition could be due to fissures, or haemorrhoid and this need to be ruled out if this is not the case then this could be intestinal infection condition can be treated tentatively with Tab Metrogyl 400 mg twice in day for five days, most of the times this comes to normal and if not then stool need to be analyse, hope this information helps, take care."
},
{
"id": 210435,
"tgt": "What to do for adhd symptoms?",
"src": "Patient: Hi, I was just wondering what to do about my adhd symptoms. I took a questionaire test at my family doctor. And he said I definitly fit the description. He put me on strattera which I couldnt take. He wanted to put me on another name brand add drug. Both were very exspensive even after insurance. He wouldnt even consider stimulants. Even though I know adderall actually helps me concentrate. Not sure what to do but its getting hard for me to focus. Doctor: DearWe understand your concernsI went through your details. I suggest you not to worry much. If your ADHD is new, or the symptoms came within a time frame or 6 months or so, the best way to approach it is with training to gain attention. Although ADHD has most often been treated with medication, medications do not cure ADHD; they are used solely to treat the symptoms associated with this disorder and the symptoms will come back once the medication stops. You might need psychotherapy to change your life style so as to get trained to beat ADHD and that is the perfect cure.Please post a direct question to me in this website. Make sure that you include every minute details possible. I shall prescribe some psychotherapy techniques which should help you cure your condition.Hope this answers your query. Available for further clarifications.Good luck."
},
{
"id": 169518,
"tgt": "Suggest remedy to relieve pain while urinating in 4 year old",
"src": "Patient: my 4yr old has had gas from birth and began haveing severe constipation and then scared of pooping, and know we are haveing off and on severe pain during peeing with drippling and holding, I am at my wits end try to tell docs to give ultrasound, they want uring test and can not get cause he has to go to bath water to try and go. I am just besides my self and can not stand to see him in this pain, it comes and goes. He took antibiotics and it seemed better for just a couple of days then crying and pain he says sticker pain and it can last for hour or so sometimes. please help me!!!!!!!!! Doctor: for burning micturition u can give Syp.citralka and to rule out UTI by doing tests like CUE and c/s.tests.and appropriate antibiotics."
},
{
"id": 1660,
"tgt": "Can sysfol tab help to conceive?",
"src": "Patient: I am trying to get pregnant, my periods are almost regular, scanning and fh lh and thyroid tests are all normal, even then am not able to make it, started off with sysfol 5mg tablets, one per day as advised by one of the doctor. In how many cycles i can expect? how this tab help in conceiving? I had an abortion 2.5 years ago since we were not ready for parenting at that early stage of married life. Kindly help Doctor: Hi there, I have understood your concern. First of all do not panic. Please try to understand that Folic acid tablet as such is nit to help you getting pregnant. I usually suggest to my patients who are planning to get pregnant, to start with Folic acid, Vitamin B 12 and Omega 3 supplements at least 3 months before planning to get pregnant. This helps to prevent many problems during pregnancy and delivery. You can follow the same .Secondly, if you are willing to get pregnant, then please get following 3 things checked. 1 ) Husband's semen analysis, after the abstinence of 3 to 4 days. This helps to know about quality and quantity of sperms. 2 ) Post menstrual HSG - hysterosalpingography, within the first ten days of the cycle. This helps to know about the patency of the fallopian tubes. 3 ) Follicle Study from day 10 to 20 of the cycle to know about the exact day of egg formation. Unprotected sex act or IUI on the day of egg formation will help you to get pre.I hope this answer helps you. Thanks. Dr. Purushottam Neurgaonkar"
},
{
"id": 40785,
"tgt": "How does Yamini tablet help to conceive?",
"src": "Patient: Hello mam I have pcos since 2 years nd ws trying for my second baby got conceived wit the second cycle of clomid 100mg nd got miscarried in the last year nd aft dat continued it for another 4cycles but failed to get pregnant. Nd nw last mont my period ws irregular wit heavy bleeding nd stopped aft takin regesterone tab nd dis month I got my period early as I missed to take some of the regesterone tab nd my doctor has nw suggested me to take yamini tabs for 21 days so pls tel me in wat way dis tab could help me... Doctor: Hello and Welcome to \u2018Ask A Doctor\u2019 service.I have reviewed your query and here is my advice.Yamini tablet is given to you to regulate your hormonal balance.It's a contraception plus hormones regulating medicine. Once your cycle wil get regular, your gynecologist wil ask you to concieve. Don't worry.Hope I have answered your query. Let me know if I can assist you further.Regards,Dr. Sheetal Agarwal"
},
{
"id": 89943,
"tgt": "What is the recurrent upper abdominal pain?",
"src": "Patient: I am a male who has diabetes and then a heart attack a few years ago. Today I am having upper abdominal pain and I had to miss work. I am now 64. The outside temp. is 101. I am still working as a police officer, now for 38 years. Sometimes the upper abdominal pain is severe, then it lessens. I know it could be a lot of things, but what is the most likely cause? Thanks muchly for your help! Doctor: welcome to Health care magic.1.The possibilities are acid peptic disease / cholelithiasis / cholecystitis / bowel pathology.2.The acid peptic disease gives periodic pre, post meal.3.Colelithiais when ever there is any obstruction in the biliary passage.4.Bowel pathology may give colicky pain with features of infections, fever.5.Get an ultrasound abdomen done, which will give near diagnosis and rule out many causes which will narrow down the diagnosis.Hope it helps you. Wish you a good health.Anything to ask ? do not hesitate. Thank you."
},
{
"id": 80937,
"tgt": "What causes chest pain?",
"src": "Patient: I have been having mild chest pain for the last few days. It s been mainly on the right side. It s a dull ache which comes and goes. I do not have a cough, congestion, fever, etc. It does not hurt when I take a deep breath or move around and it actually seems to be relieved when I exercise. Doctor: HIWell come to HCMI really appreciate your concern, such symptoms could be due to muscular spasm or may be due to poor posture, try to maintain good posture while you are exercising even after that symptoms persist then this could be due to hyperacidity, and you can try \"Tab Omeprazole 40 mg once in day for three weeks, hope this information helps, take care."
},
{
"id": 85075,
"tgt": "Is zobone 5 a safe medicine to take?",
"src": "Patient: helo doc, my mum has been prescribd zobone 5 by doc..she has to take this inj..once a year for 4 years...is this drug safe she has olso undergone both tkn (total knee replacement)..n she has been sufferin from back ache last one month plz advice..thx Doctor: Hello dear and Welcome to \u2018Ask A Doctor\u2019 service. I have reviewed your query and here is my advice.Zobone 5 mg Injection is used to treat or prevent osteoporosis in postmenopausal women.It is a safe medication, so you should not be worried.Hope I have answered your query. Let me know if I can assist you further.Kind regards!Dr.Dorina GurabardhiGeneral &Family Physician"
},
{
"id": 162845,
"tgt": "How to treat eczema rashes on the face and body despite using steroid cream?",
"src": "Patient: hi my son (11yrs) woke up Tuesday morning with an itchy red rash on his cheek. by the end of the day, the rash had spread slightly to his forehead, ear, neck, chest and inside elbow crease of one arm. the day before, he said he felt like he was coming down with a cold. about 6 monts ago, the same thing happened - brought him to pediatrician, was told it was most likely eczema, and they gave us a topical steroid cream. after a week, I brought him to dermatologist - said the same, but she gave us a 2.5% hydrocortisone cream (said the steroid is too strong for his face don t use it). anyway - so back to current day, i m assuming its a flare up of this eczema rash again. today, the rash is even worse, his cheek & one ear is swollen and hot. there are little patches of the rash on other parts of his body, stomach & legs in addition to the others I ve listed already. none of them are as bad as his face and ear, yet. I guess my question is, does this sound like eczema? can eczema cause swelling and be warm to the touch? I called the dermatologist, but of course its my luck they are closed until Tuesday for Labor Day holiday. Doctor: Hello and Welcome to \u2018Ask A Doctor\u2019 service. I have reviewed your query and here is my advice. I am indeed sorry that you haven't been able to get urgent attention for your son. It is now over a month since you asked this question. If you are still interested in knowing what to do, I will offer a reply. Yes, this is eczema, and it should be treated with moisturizers, and if necessary, steroids applied topically. The child may also need an antihistaminic to reduce itching and the rash. This last medicine would need to be used orally (by mouth). Hope I have answered your query. Let me know if I can assist you further."
},
{
"id": 108113,
"tgt": "What causes lower back and groin pain?",
"src": "Patient: I noticed some small blood stains on my undewear and when I urinated and wiped myself there was a small amount of fresh blood on the toilet paper. My urine appeared light yellow when I checked. My urethra feels a little bit irrititated. I have a nagging pain near my groin and lower back pain both on right side radiating down my leg which I have had for over 6 months Doctor: Hello.Thanks for writing to HCMI have studied your case and history.From your symptoms there appears to be systemic disease which may be affecting renal system.There can be associated urinary tract infection.You may require good clinical examination from MD orthopaedic and urologist doctor.He may advise you to investigate like Ultrasonography pelvis and abdomen.If required MRI spine will help.Spinal nerve compression can lead to radicular leg pain.You need to consult your treating doctor for further investigation.Hope this answers your query. If you have additional questions or follow up queries then please do not hesitate in writing to us. I will be happy to answer your queries. Wishing you good health.Take care."
},
{
"id": 26534,
"tgt": "What could cause death following a shoulder catch and breathlessness?",
"src": "Patient: I LOST MY HUSBAND JUST 1 MONTH BACK AT 4.30 AM HE GOT AND TOLD ME SHOULDER CATCH AND BREATHLESSNESS AND HE FELT IT IS DUE TO GASTRIC AND TOOK TABLET FOR THAT STARTED TO WALK OUTSIDE AT 4.50 SUDDENLY HE FELL DOWN AND DIED IMMEDIATELY HOW IT HAPPENED MY MAIL ID IS YYYY@YYYY Doctor: HiThanks for posting your query. Very sorry to know about the demise of your husband. Most likely he would have had a massive heart attack which sometimes present with shoulder pain or arm pain instead of chest pain. The other possibility is stroke and rupture of aorta ( large blood vessels ).Any further information needed feel free to contact.Regards."
},
{
"id": 208848,
"tgt": "How to overcome the loneliness and depression?",
"src": "Patient: Hi! I am very lonely all the time........I really don't know why but i end up with wrong people at the wrong time. I have a tendency to slash by body with scalpel. I have tried searching 4 someone to whom ican talk clearly and heart out but no one seems to be there Doctor: DearWe understand your concernsI went through your details. I suggest you not to worry much. This is not loneliness, but pure depression. Negative thinkings and uncontrolled thinkings during loneliness do bring self harming behavior. Please consult a psychiatrist for better treatment and to save your future.If you require more of my help in this aspect, Please post a direct question to me in this website. Make sure that you include every minute details possible. I shall prescribe the needed psychotherapy techniques which should help you cure your condition further.Hope this answers your query. Available for further clarifications.Good luck."
},
{
"id": 41053,
"tgt": "How to get pregnant while suffering from PCOD?",
"src": "Patient: my wife had a problem of polytheistic ovarian syndrome and Dr. has done drilling on both the ovaries. after that four times we have done IUI treatment but no result has come. and she has problem of TSH & Prolactine, just because of these she is taking Thyronorm 75mcg and sicriptin 1.25mg. can you suggest for further to do what? should we go for IVF? and how much possibility will be after IVF? Doctor: HELLO DEARI have gone through your question and understand your concern , In my opinion before going for IVF you can try for ovulation induction with clomephene and timed IUI and one important thing to be kept in mind your thyroid and prolactin should be normal before induction and also to increase success rate you can reduce weight if you are obese by exercise , diet and metformin. Metformin also improves insulin resistance and efficacy of clomephene. After trial given for 5 to 6 cycle if conception not possible then you can go for IVF. The success rate for each IVF cycle is 37 %. Best of luck THANKS"
},
{
"id": 124357,
"tgt": "What is the treatment of pain on right side under ribs ?",
"src": "Patient: I ve been having some dull achy pain on my right side under my ribs. It comes and goes. I stopped alcohol and smoking over 2 weeks ago. I ve got insomnia and have been taking one Tylenol PM every night for sleep. My stool was yellowish, not very solid this morning. What should I do? Doctor: Hello, As mentioned in history about the old habits of alcohol and smoking which is stopped past two weeks and now the symptoms of pain on the right side of the upper abdominal region I would advise you to first get few things done. Get an ultrasound abdomen done along with an x-ray chest, a blood routine and a urine routine. Since there shouldn't be any major stuff to understand by the exterior view. It is always good that a proper examination is done with some diagnostic procedure to rule out the pathology related to the liver or some other organ. Rest assured, once you follow the steps of advice things shouldn't go bad. Hope I have answered your query. Let me know if I can assist you further. Take care Regards, Jay Indravadan Patel, Physical Therapist or Physiotherapist"
},
{
"id": 156906,
"tgt": "Does clear scab inside the ear that keeps forming indicates skin cancer?",
"src": "Patient: I have a clear scab inside my ear that keeps forming. Its not painful or anything but it has been reoccurring for the last 5 months . My husband said it looks discolored and shiny. We think it might be a fungus or maybe dry skin. Could this be skin cancer or something? Doctor: scab in the ear is not cancer. formation of scab is normal phenomenon. but if you think that if it is excess or if there is foul smell or discharge from the ear, then you need to consult ENT specialist immediately. most commonly foreign body in the ear can cause such symptoms"
},
{
"id": 77362,
"tgt": "What causes cough,sore throat and watery eyes?",
"src": "Patient: I have had the following symptoms for approximately one month. I am a 56 year old male, never smoked, maybe three beers a year.a) coughb) a little difficulty swallowing when thinking about itc) a feeling inside my throat (near the Adams Apple) of a small pill not swallowed.d) no sore throate) watery eyesI have a doctors appointment in two days but I am concerned now. Any thoughts would be appreciated.Thank you,Terry Doctor: The symptoms of Cough, with sore throat and watery eyes may be the symptoms of acute viral upper respiratory tract infection, but if that's due to infection with virus, it may not last for 4 weeks usually. So in your case there is another possibility of Allergy of the respiratory tract , that may be causing these symptoms. Anyways Both of these conditions are well manageable and treatable, so I believe there should not be much worry, and you may look forward for an appointment with your doctor."
},
{
"id": 217467,
"tgt": "Suggest treatment for neck pain",
"src": "Patient: I am 38 years old female, working in a private concern... I have neck pain (not severe) due to spondilyties. I have consulted an ortho and doing excercise. For the past few months, I am not able to write. I am finding it very difficult to write even a single line, due to pain in the fingers. My fingers are hard sometimes. Which doctor should I consult for this... Neuro or Ortho? please advise Doctor: Hello. I read your question carefully and I understand your concern.The fact that you've started to feel pain radiating to your fingers and weakness suggests that there is involvement of the cervical nerve roots and/or the cervical spinal cord. The medical treatment prescribed by neuro or ortho I think would be similar with antiinflamatory drugs, muscle relaxants, collar, exercise.However since there is the possibility of involvement of the above structures I think a neurological evaluation is necessary at this point determine whether an MRI is indicated, because depending on the degree of involvement at times a surgical procedure might be required.Hope to have been of help. Feel free to ask further questions."
},
{
"id": 201075,
"tgt": "Does masturbation cause sexual problem?",
"src": "Patient: sir,i am 23 years old.i use mesterbation by hand from 8 years,per week.my penis size is 5 inch.it will continue also now.pl sir tell me,have i face any sectual problem for this?i don t fell any wrong even now.i don t do any sex with girl.so i don t know ,am i do sex in long time.and i want to long time sex.have i any problem for this?and what i can do if any problem? and if there is no problem then what can i do to increase my sex time?pl sir advice me..please ..and i have no cradit card sir...please kindly inform me please as a human Doctor: HelloThanks for query.You had been indulged in masturbation since many years and now worried about problems related to sexual activities.First of all there is myth in a mind of common man that excessive masturbation is the reason of all sexual problems But I would like to state that it has been discussed in scientific forums all over the world and proved scientifically that masturbation does not have any negative effect on any organ or system in the body.All the problems that you are facing now are mind related and due to anxiety.Following measure will help you to boost up your confidence and getting good erection.and delay ejaculation.1) Practice regular exercise for 45 minutes followed by meditation for 1/2 an hour in the morning.2) Take high protein diet rich in vegetables and fruits and Vitamin A,C,D,E.and Zinc3)Take anti oxidants like Almonds 5-6 everyday..4) Avoid alcohol and smoking..Dr.Patil."
},
{
"id": 209512,
"tgt": "Could mental and physical weakness indicate major health issue?",
"src": "Patient: I am 24 years old male. I have quite a few queries regarding my health. I feel that nowadays my concentration level is too low and I am not able to focus on any matters. I am feeling so light and easy going. I am feeling that there s always something wrong with me which I feel is Psychological as all my blood tests are absolutely normal. My family doctor prescribes me to do a lot of exercises which I have started to do. My doubt is are these syndromes for any major health disorder as I feel mentally and physically very weak. Doctor: Hi,If you are feeling extremely weak in spite of having normal blood tests, it is quite likely that this is psychological in origin. Mental stress as well as mental illnesses can cause low mood, fatigue and loss of concentration despite being physically fit. If possible, consult a psychiatrist for proper diagnosis and management.Best wishes."
},
{
"id": 18466,
"tgt": "How risky is the delay in the cardiac catheterization procedure?",
"src": "Patient: Hello and thank you for your help I took a cardio calcium score test yesterday and my number was 77 showing some amount of blockage in on left side in a descending artery they pull that order the Widowmaker my doctor and scheduled me for a heart catheterization on May 24th that s in about 2 weeks from now I was wondering if that is too long to wait he didn t seem too concerned however after reading about it and being told something about it on on the internet I thought maybe I should ask to make the appointment sooner. Doctor: Hello and Welcome to \u2018Ask A Doctor\u2019 service. I have reviewed your query and here is my advice. Usually it's not a problem to wait for this much time period especially if you are started on medicines and do not chest pain at present. So you should be statin medicines like Aspirin, statin like atorvastatin which would reduce the risk of acute coronary syndrome. Also, avoid heavy activities. Hope this helps you and get back if you have any doubts."
},
{
"id": 180887,
"tgt": "What causes purple discoloration on the bottom gum?",
"src": "Patient: Hello. My bottom gums (way down - not visible unless you are looking) - appear red/purple on the bottom of the front 6 teeth. So far, there is no tooth or gum pain - no swelling but it looks like it's bleeding internally? I don't recall getting hit in the face/mouth ;) - so wondering what it could be? I do use whitening mouth wash if that has anything to do with it but it's only on the bottom gums. Thanks! Doctor: Purple discoloration on bottom gum is mainly because of acute necrotizing periodontal disease and due to dead tissue, or necrosis. Improper oral hygiene, stress, smoking, and viral infections, are causes of infection. If this condition left untreated, it can spread to your cartilage and then bone, causing very serious bone loss. I would suggest you to visit dentist and get proper cleaning of this area done. Periodontist is the one specialized for gums for proper scaling, root planning. Maintain hygiene brush three times a day along with flossing. There are surgical and non surgical both methods available to prevent this discoloration. Use antimicrobial mouth wash such as Listerine.Best Wishes,Dr. Harry Maheshwari"
},
{
"id": 193218,
"tgt": "Could the dull pain in scrotum and groin with freckles be due to ageing?",
"src": "Patient: I'm 29,male and I've recently noticed a somewhat increased number of freckles appearing on my body. I have also had some slight dull pains in my groin area, scrotum and lower back/abdominals, although the pain subsides often. Am I just getting older? Doctor: Hi,Sun exposure is the main cause of freckles and dark spots however dull ache in your groin area might be due to number of reasons. All you can do at home is don't wear tight under wears, avoid using tight clothing too and if you area having some sort of discharge with urine go and see your doctor, it might indicate some sort of infection.Hope I have answered your question. Let me know if I can assist you further. Regards, Dr. Sameen Bin Naeem, General & Family Physician"
},
{
"id": 185952,
"tgt": "What could be the reason for pain and swelling under the tongue?",
"src": "Patient: hi i have red swollen pain gum located on tori on inner side of mouth, not cheek side the under side of my toungue, not the tongue itself, however, the connecting area of floor of mouth to tongue is red and swollen as well oral surgeon feels it is trama caused by a tee i had preformed, however, this is the same area of my mouth that i had bone infection two years ago. on antibiotics for 2.5 days and pain is increasing Doctor: Hi With you query, as you mentioned it as tori, actual meaning of tori is literally means extra growth of bone. As such, this growth will not cause any injury or trauma any sort of pathological disturbances except esthetic appearance if it is present on labial side or front side of teeth.In reagrd to your mentioned swelling could be an abscess of either periodontal or periapical origin.If it is tori you can get osteoplasty done by the oral surgeon but if it is an abscess one must rule out the cause and treat it.Avoid uncessary use of antibiotics 'coz even after antibiotics and pain killers you complain of pain which means its not an infection.Secondly, if swelling is on the floor of mouth make sure all the salivary ducts are patent i mean there is no obstruction which may be due to caluculi.If it is a salivary caluculi you will have increase in pain while eating due to exces sproduction saliva.Hope this is informative and please seek your dentist attention."
},
{
"id": 77199,
"tgt": "Suggest treatment for pulmonary embolism, weight gain , fatigue and shortness of breath",
"src": "Patient: I have pulmonary embolisms, Edima, weight gain, severe fatigue, shortness of breath which leaves me light headed. My cardio function is great, blood panel good, but elevated inflammation markers. Inhalers including albutoral showed no pulmonary change. Oxygen saturation typically at 97%. No one has been able to diagnose root cause of emboli. Any thoughts are appreciated! I am on xeralto and diuretics. I am hoping someone has seen these symptoms present before and knows the cause of them. No dvt Doctor: hi,thank you for providing the brief history of you.to come closer look to your symptoms that you have shortness of breath, fatigue, edema and weight gain.Few things to understand here - the symptoms you are facing are all interconnected. As due to the embolism you are not able to have proper utilization of the lung fields, which is needed. Due to which the oxygen is less in the body and you feel fatigue. Also when you feel fatigue later it will restrict your cardiorespiratory endurance and lead to edema. To cut down all the bridges between the whole process, you need to focus on doing regular exercises. As once you will do your most symptoms will come down. As exercise improve the metabolism and also the oxygen in the blood it will help further to recover and provide good functional abilities.I have seen cases with such issues and 99% have recovered well. I wish the same of you. god bless youRegardsJay Indravadan Patel"
},
{
"id": 16502,
"tgt": "What is this bruise in my anal ?",
"src": "Patient: Anal bruise problem for past 5 days, esp when I release stool in the morning. I don t have any constipation problem. Doctor: Hi, Thanks for query, Apply antibiotic cream,take antibiotic course. Avoid constipation,take light laxative at night. Avoid fried,chillies and junk food. Take plenty of water. ok and bye."
},
{
"id": 222571,
"tgt": "Will small cervix cause any problem during pregnancy?",
"src": "Patient: I have completed 17 weeks of pregnency. During my internal examination I was told by my doctor that my cervix is small. Its length is 3.3 cm. The doctor has suggested stitching the cervix For now, I am told to take bed rest and come again after 15 days for examination after which she will decide whether to give the stitch or not. Please tell me if having a small cervix can cause some problem and is stiching/ operation is compulsory. Also what are the risk associated with this condition. Doctor: Hello, and I hope I can help you today.Cervical shortening (it is the length of the cervix that is measured) can be a risk factor for preterm labor as well as preterm delivery. It should be measured accurately with a transvaginal ultrasound not just with a digital examination.However, the cervical length at which intervention is recommended and where we have real concern is a length of less than 2.5 cm.Cerclage (or placement of a stitch to close the cervix) really should only be performed if the length shortens to less than 2.5 cm. The procedure itself has some risks including breaking the water at the time of the cerclage and infection of the cervix.However, the most effective treatment currently available to prevent preterm delivery with a short cervix is vaginal progesterone. This is given as a cream or suppository and does not have the risks associated with cerclage placement.So in summary, I would not recommend any treatment if your cervix is longer than 2.5 cm, as a length of 3.3 is not considered a significant risk factor for preterm birth. If your cervix shortens to 2.5 cm or less, vaginal progesterone is a safer and more effective means of preventing preterm delivery over cerclage.Furthermore, bedrest has not been shown to prevent preterm delivery in any scientific study, especially in your case as you are still in the second trimester and are not dilated.I hope I was able to effectively answer your question today and that my advice was helpful.Best wishes for the rest of the pregnancy, Dr. Brown"
},
{
"id": 35836,
"tgt": "What causes bruise in my neck and chest?",
"src": "Patient: Thanks! Hi, I am just wondering about something. I took a shower tonight and was washing my neck and chest and noticed I felt a bruise, but there is nothing there. I felt this about a month ago and I don't really want to worry, but I don't want to ignore something that has been around for a month. Any ideas? Doctor: Hi, dearI have gone through your question. I can understand your concern. You may have purpura or petechhia. You should go for complete coagulation profile. Then you should take treatment accordingly. Hope I have answered your question, if you have doubt then I will be happy to answer. Thanks for using health care magic. Wish you a very good health."
},
{
"id": 123516,
"tgt": "What causes pain in my shoulder blades with recurring cough?",
"src": "Patient: Hi I have had a recurring tickly cough for about 2 months now. My chest feels tight as though someone was pressing on it. And for a few days I have been having a little pain between my shoulder blades. At first I thought it was from sleeping funny but it s been few days no so not sure this is the reason Doctor: Hello, As the shoulder blades plain indicate the spasm in the Trapezius muscles. This happens due to difficulty in breathing occurred due to cold/cough. With hot water fermentation, myofascial release and kneading massage the spasm of the Trapezius can be reduced. Deep breathing exercises and steam therapy will help reduce the cold/cough as well. Hope I have answered your query. Let me know if I can assist you further. Take care Regards, Jay Indravadan Patel, Physical Therapist or Physiotherapist"
},
{
"id": 110759,
"tgt": "How to treat lower back pain?",
"src": "Patient: I Was In THe Hospital. DiagnosWith This They Got My Calicum Down To Wher It Should BE But Im Very Short Winded And Still Have Lower Back Pain What TO DO Got A Job To Face And Do A Lots Of Walkin Cant Walk That Far Right Now Help! DONT HAVE NO CREDITS CARD FOR PAYMENT HELP THOSE WHO NEED TO BE HELPED ?! Doctor: HiThank you for asking HCMI have gone through your query.In such cases Parathyroid hormone and Vitamin D should be checked.Renal profile also should be checked.I usually give Vitamin D3 and Calcium for such cases.In case of back pain damages to spine should be ruled out by taking X rays of lumbosacral spine.Hope this may help you.Let me know if you have any further query."
},
{
"id": 112394,
"tgt": "Had an abscess on back of spine, soreness and pain. Scheduled operation. Will it benefit?",
"src": "Patient: I had a abscese on back of my spine 30 yrs ago it has never healed properly I went to the doctors this morning and he Sid there is a hole and that it never healed properly it is causing me pain and very sore where the scar is he said I have to have a operation I see a specialist in a week time but I am so scared of them reopening the scar has I had packing inserted and this was so painful please can you advise amanda Doctor: Hello,I have gone through your query and understood your concerns. According to your history it seems like a sinus tract formed and operation is a better idea. Otherwise it will get infected and cause more problems for you so its always better to operate it. As your doctor told you its not sure that it will get treated completely but there will be good effect than bad to you.Thank you."
},
{
"id": 199346,
"tgt": "Suggest treatment for nocturnal emission",
"src": "Patient: Since last year i meant 2014 March. I have been having is problem when i go to bed sometime i will found myself wet.... i was thinking that it s just a sign of heat unknown that is my sperm that comets out every night Apart from that if i want to make love to a woman. sperm equally kick too am highly frustrated in my life .....I m seeking for help and better solution.... I grow up with one scrotum i don t really know it was the causes Doctor: HelloThanks for query.Getting night emission is normal and natural at the age of adolescence and there is nothing to be worried about it.Normally it gets resolved itself spontaneously without any treatment.As regards what you have mentioned about only one testicle from child hood .It is likely that you have only one testis and the other being undescended and located in either inguinal canal or abdomen .You need to consult qualified Urologist for clinical examination and get Ultrasound Scanning of the abdomen and pelvis to locate the position of other testis.He will advise you regarding treatment for the same depending upon the location of testis.Dr.Patil."
},
{
"id": 179126,
"tgt": "How to improve eating habit for my kid",
"src": "Patient: My son is 1.5 years old. He doesn t eat anything with interest. We need to pressurise and push in food into him deliberately. I feel he doesn t feel hungry at all because he doesn t ever come runnign towards food. Have tried all types of food, veg,nonveg,eggs everything. He also has just 4 teeth till now. Can you suggest some medicines to increase his interest in food? Doctor: Hi, I had gone through your question and understand your concerns.I need to know more before I can answer your in a better way . 1. What is weight of your child.2. Is your Child active.Medicines are not going to increase interest in food unless reason for it is known , like :- if child is anemic , iron preparations are going to help.If your child is underweight(less than 10 kg ) , pale , not active , there is abdominal distension , he needs to be evaluated by a pediatrician to find the cause and treat accordingly .If his weight is normal (10-11 kg) growth is OK , baby is active , problem may be simply forced feeding of child leading to aversion of food.Regarding giving or adding medicines you need to consult your doctor .Hope this answers your question. If you have additional questions then please do not hesitate in writing to us.Wishing your baby good health.\u2022DISCLAIMER: - All the information provided here is for information purpose only , it is not a substitute for the advice of a physician after physical examination , it is not intended to replace your relationship with your doctor. This information in no way establishes a doctor-patient relationship. Before acting anything based on this information do consult your doctor. I recommend that online users seek the advice of a physician who can perform an in-person physical examination ."
},
{
"id": 71039,
"tgt": "What causes shortness of breath and tight sensation in the chest?",
"src": "Patient: About a month ago I went to ER for shortness of breath and chest tightness. They did all kinds of major tests on my heart. Everything is fine. I did feel better for a few weeks but now the feeling is back and worse. I m waiting for my pulmonary doctor office to call and make and appointment. Can you help with any ideas to what may be causing this? Doctor: Hello and Welcome to \u2018Ask A Doctor\u2019 service. I have reviewed your query and here is my advice. For chest tightness and difficult breathing heart tests are normal So further work up done in the form of chest X-ray and spirometry to rule out respiratory causes. If respiratory cause also not present than history about muscular strain has to be taken Excess anxiety also can lead to such presentation Take care Hope this will help you."
},
{
"id": 206364,
"tgt": "Suggest treatment for dementia pugilistic",
"src": "Patient: Hello, I was wondering about Dementia Pugilistica. I am wondering about my nephew. He is 17 now and has had alot of concussions from hard blows to his head in football. The last 2yrs hes been acting stranger and stranger. Then he had a truck accident and hit his head again. Is demenor got even worst with memory problems, blackouts with major aggression. He wouldn t listen and played football anyway and started drinking acohol for the heaches. Now he s in trouble big and he doesn t remember if he done what there accusing him of..At first I thought he was lying now I think he really does have issue of the brain. What are the symptoms and how do I find a doctor to check him and help him if he does..Live in Arkansas.. Thanks --Confused Sorry didn t know you had to pay I haven t work in 6 yrs do to muscle spasms in my body that no one can figure out the cause b/c test keep comming back normal.. They still hurt though.. Thank You for your time anyway. Sorry... Doctor: Hello Thanks for askingDementia Pugilistica also called as chronic traumatic encephalopathy is a type of dementia that occur in boxers after repeated trauma on head. The main clinical features are declining memory and features of Parkinson's disease like tremors and a number of other symptoms. He is having forgetfulness but no other symptoms are there that can label him with dementia pugilistica.Multiple trauma to head and then one serious accident are most likely the reason for his personality change. He is acting stranger, have problems of aggression, don't listen any one and has started taking alcohol. He is not remembering the episode of aggression. Such symptoms may occur some time following head trauma because of injury to frontal and temporal lobe of brain. Injury to brain may cause personality changes and may aggravate anti-social behaviour. Loss of short term memory might be due to trauma only. I would advise you to consult a good Psychiatrist for his treatment. To decrease his aggression and post traumatic complications medicines like Mood Stabilisers like Sodium Valproate, Carbamazepine etc can be used. Hope this helps you, thanks"
},
{
"id": 24773,
"tgt": "What does low pulse rate indicate?",
"src": "Patient: I am 47 years old I had a Heart attack six months ago (on valentine's day Ironic) I am female,, I had many symptoms beforehand shortness of breath.. Pain in arms etc..etc.. I have 2 stents to R.C.A. m y damage is on bottom and side of heart and my problem is the more active I am the worse I feel.. My pulse skips around and sometimes is in the 30 range even without beta blocker.. They say I am not a canadate for a pacemaker tell my pulse stays in the 30's but I do not feel well! Doctor: Hello and thank you for using HCM.I carefully read your question and I understand your concern.I will try to explain you something and give you my opinion.You should know that low heart rate is not always an indication for pacemaker.You mention that your heart skippes a beat.You should be carefull and not measure the frequency after a prature heart contaction or a skipping beats, becouse in this cases the heart pause its normal.So, it is important to differentiate this thinks, if you have low frequency during uour dayly live and you feel week, or you just measure this episodes after a skipped beat whith pauses.In my opinion, if I was your treating doctor I will recommend a holter rhythm monitoring.if it register frequency to 30 beat for minute during day and you do have symtoms than it might be an indication for pacemaker becouse symptomatic bradycardia fulfills the criteria.Hope Ia was helpfull.Best regards ,Dr.Ervina."
},
{
"id": 134253,
"tgt": "Does robaxin 5oomg help in feet pain?",
"src": "Patient: last 2 weeks I have mild (tolerable) pin pricking pain below my toe ( not the heel part) on both my feet. I stand 5-6 hours a day at work and it does not help. I am not diabetic and I am 60 years old,male. No other symptoms. I had 2 robaxin 5oo mg left from my back pain (2 years ago) and I took 250 mg twice a day for 2 days and for 3-4 days I did not have any pain at all????? Doctor: hi,Thanks for providing the brief history of youSince you mentioned your symptoms you have and you have to stand for 5-6 hours a day, also you tried self medication which didnt work.What it looks like to me is the nerve getting irritated at the lower spine region for which you are feeling this symptoms of pin pricking. And also you had the back pain issue in the past as well which is making me think this.Since the symptoms are not to big and tolerable for you, i think you can be fine with some exercises for which you can meet a physical therapist and learn from him.Or either you can do some exercises for strengthening the core and the spinal muscles. by this the symptoms should stop.Also, whenever the smallest things starts from the spine, one should keep continuing the core strengthening and spine strengthening exercises for long term to avoid major injuries.feel free for any more queriesRegardsJay Indravadan Patel"
},
{
"id": 166846,
"tgt": "What causes vomiting after having solid food?",
"src": "Patient: Dear Doctor my baby is week in healh she is almost 9- Months but she looks like 3-4 Moths baby . every time after taking solid food .eg Cerelac / Banana / Rice . she excell out the food / Vomating . This is the main reason for not her growth . I am very much worried Doctor: Hi,You should give small amount of food in a day, and slowly increase the quantity of food.Hope I have answered your query. Let me know if I can assist you further.Regards, Dr. Narendra Maurya"
},
{
"id": 41916,
"tgt": "What is the treatment for infertility?",
"src": "Patient: Iam 32 and mu husband 33 - my fsh level is 12 (took for 3 years ago). I am getting IVF treatment but not pregnant yet. What can i do for get pregnant. My eggs are fine and my husband sperm is fine - we match and in earlier ivf my eggs fertilized but they wont be in uterus. what I can I do? Please help me Doctor: Hi welcome to healthcaremagic.I have gone through your question.Your fsh value was within normal limit.Your reports and your husband s semen analysis is normal. Still you are not conceiving then its \"unexplained infertility\". Then only option is ivf.Consult a gynecologist and ivf specialist for this problem.Take care."
},
{
"id": 82622,
"tgt": "What are the symptoms of sle?",
"src": "Patient: I have SLE and suffers from a slight fever permanently. I am in a relationship and my friend stated that he burns when we have intercourse yet I do not itch or burn. He also mentioned that my vagina have a smell. Could this be related to my SLE or should I visit my GP Doctor: No relation with SLE. but can be due to secondary infection that may occur due to immunosuppressive therapy u might be taking for SLE"
},
{
"id": 47878,
"tgt": "Suggest treatment for end stage renal disease other than dialysis",
"src": "Patient: Hi, I was diagnosed to have end stage renal disease sec. to obstructive uropathy sec. to neurogenic bladder. I was once a had spina bifida but corrected with a surgery. I have 4-5 mg/dl crea. doctor advises to undergo dialysis. any more alternatives? thanks! Doctor: Hello, end stage renal disease. This term itself it self explanatory for the condition you are having. This means renal disease has reached an end stage where its not going to function and its mostly irreversible. Dialysis is the best option for patients with ESRD.Only other option for these patientsis undergoing renal transplant. Which again requires series of tests and fitness for undergoing surgery and much more.I will advise you to undergo dialysis as there is no other way to remove the waste products from the body. You can try asking your doctor about renal transplant.Thank you"
},
{
"id": 28886,
"tgt": "How can chicken pox be treated?",
"src": "Patient: My son has chicken pox and is refusing for me to apply the calamine lotion ,does it sting the sores?i thought it would make him feel better but it s a fight to put it on him,Is there another alternative ,we are on day 3 now with it as well how do I know it s healing? Doctor: Hello and Welcome to \u2018Ask A Doctor\u2019 service.I have reviewed your query and here is my advice.The calmine lotion is only for your son's benefit. It does not have healing effects, but it soothes the skin. So if he prefers not using it, that is fine.You could try neem leaves. Besides that paracetamol for the fever is adequate, as the lesions will start resolving on their own. You will start seeing the lesions start to scab and fall off.Ensure your son does scratch the scabs as it could cause scarring.Hope I have answered your query. Let me know if I can assist you further.Regards,Dr. Pranav Balakrishnan"
},
{
"id": 156994,
"tgt": "Is it normal to have swollen foot while taking chemotherapy treatment for Mantle Cell lymphoma?",
"src": "Patient: I have Mantle Cell lymphoma and am being treated with R-CHOP chemotherapy, my platelets are low (27), I am having a problem with my foot, it has pained me for a few days, it has now swollen and has bruised an three toes. The top of my foot is turning red and has a rash - is there anything I can do for myself without the need to consult the hospital hotline Doctor: It is not normal. The cause of swollen foot may be due to infection or lymph channel obstruction with superadded infection which can be verified by clinical examination. At the moment you can not have further chemotherapy as your platelet count is very low. Report immediately to your doctor if you have rash in the form of bleeding spots on any part of your body.Dr. J. Ticku"
},
{
"id": 62637,
"tgt": "What causes bump in the tongue?",
"src": "Patient: so about a year ago i had my tongue pierced and about two to three months afterwards i took it out... now at the bottom part of my tongue where the hole was from the piercing there is now a bump there and it has been a concern to me i am just curious as to what this could be? Doctor: Hi,Welcome to HCM.This could be due to impacted food with granuloma/ or could be mucous inclusion dermoid cyst/ or could be abscess at the pierced site.Hope this would help you to resolve your query.Else Contact with a Followup Premium question to ME.Will appreciate your Hitting thanks and writing excellent review comments to help needy patients like you. Welcome with any other further query in this regard.Good Day!!Dr.Savaskar,Senior Surgical SpecialistM.S.Genl-CVTS"
},
{
"id": 70438,
"tgt": "Is the high fever due to Retropharyngeal abscess in a 5 year old?",
"src": "Patient: My 5yo son's normal temp is around 96. He spent a week in the hospital with a temp of 100-102 sleeping constantly. He had retropharyngeal abscess to which the doctors surgically removed. All the while they stated he never ran a high temp associated with it which grew concern to them all. Doctor: Hi.The concern of the Doctors for not so much rise of temperature is for 2 reasons.1. This can be due to bacteria- gram negative in particular and many times are resistant to many antibiotics making treatment difficult. 2. Possibility of tuberculosis.Well, the drainage of abscess must have given a sample for staining and culture and sensitivity, guideline a bit for further management.I hope the baby is fine now."
},
{
"id": 189324,
"tgt": "Having Cubic Zirconia crowns on molars. It is cracking and breaking constantly. What should I do?",
"src": "Patient: I have had cubic zirconia crowns put on a couple molars over the past 3 years or so. I have had nothing but trouble...cracking, breaking, chipping of these...at least 3 times!! this morning, the crown i had fitted one year ago just cracked/broke. So I am going in to see him this afternoon. I was told its under warranty. I have a new dentist now because my insurance recently changed. Should i be looking into getting a different kind of material? The dentist said he recommended the expensive crowns because i have an immune system health issue. Is he just trying to make more $$? Doctor: Hi, Thanks for aksing the query, Chipping and breaking of the zirconia crown occurs when your biting forces are high. I would suggest you to visit the Prosthodontist and look for a full ceramic or metal fused to ceramic crown, discuss with your Dentist about the treatment options. Zirconia crowns does not have any relation with your immune system so you can definately think of different treatment options. Maintain a good oral hygiene use antiseptic mouthwash rinses twice daily. Hope this helps out. Regards."
},
{
"id": 23374,
"tgt": "Is it possible to have heart attack when scared?",
"src": "Patient: is it possible to have suffered a heart attack after being scared? I walked into a dark room during the night and a family member screamed as I turned on the light, scared me so badly I've had a feeling on constriction thruout my chest and arm ever since. Should I be concerned? I am a female, 48 yrs old, no other heart problems Doctor: hia feeling of heaviness and constriction through out chest can be a stable angina (not heart attack) or it can be because of panic attack alsoa thorough history is neededlike do yopu have family h/o of cardiac ailmentshow long did the chest heaviness lasteddid you ever felt any pain or heaviness in chest on walking which got relieved on resting i would suggest you to go for a ecg test which is a basic testmost likely the episode was because of panic but yet we have to rule out cardiac diseaseif you are a smoker it has to be ruled out at the earliestthanks"
},
{
"id": 118082,
"tgt": "Blood test report shows pernicious anemia. Can b12 affect sugar levels?",
"src": "Patient: got a blood test done today as the doctor told me last week my blood sugar was a bit high,i have pernicious anemia and my doctor told me today not to get my injection untill the results come back, can b12 effect sugar levels or is something to do with my immune system Doctor: No there is not relation or interaction between B12 and anti diabetic drugs , you should administer your drug for DM now so avoid complication of DM Hoping you get well soon Dr.Adel"
},
{
"id": 24980,
"tgt": "Suggest treatment for low BP",
"src": "Patient: Hi! I am 5'1 and 115 pounds...I am 28 years old and a mother of three. Besides my kids keeping me crazy busy, I also highly active/high paced job where I am on my feet straight for at least six hours a night. I do high intensity workouts about five days a week consistantly. My blood pressure has always been normal but these last few months it has been low. My last check up it was 92/70...my doc said it was ok...but im wondering if there are maybe other things i should be watching out for. My heart rate always seems to be fast, even when imaying down?? Doctor: Hello!Welcome and thank you for asking on HCM!I understand your concern and would explain that low blood pressure could be related to different causes: - chronic anemia- dehydration- electrolytes imbalances from malnutrition- orthostatic hypotension, etc.. I would recommend performing some tests to investigate for the possible cause: - a resting ECG (as you refer to have a fast heart rate)- blood lab tests (complete blood count, blood electrolytes, kidney and liver function tests)A head up tilt test would be necessary to exclude possible orthostatic hypotension. If all these tests result normal there is nothing to worry about. Meanwhile, I would recommend avoiding prolonged standing up position and take plenty of fluids (water, juices) to stay well hydrates. An increase in the salt and caffeine intake could help in raising blood pressure. Hope to have been helpful!Greetings!Dr. Iliri"
},
{
"id": 160863,
"tgt": "What causes fever and lethargy in a child?",
"src": "Patient: My daughter is four yrs old and Friday night she came down with a fever and once I was able to get a teaspoon of motrin down her she reminded me of Linda Blair in the Exorcist. Sat. I came home from work and the fever was still prestent. The babysitter told me she had no fever till hour before i picked her up but she simply laid on the couch all day with no energy or appetite. Today being Sunday, she is breathing very fast and lathargic with a fever of 102.under arm dig. thermometer. I am so worried, she is not herself and is to scared to tell me what is wrong. Doctor: Hi, You can give her paracetamol syrup four times per day to lower her temperature. In addition to augmentin syrup 325 mg as this is an infection. Hope I have answered your question. Let me know if I can assist you further. Regards, Dr. Salah Saad Shoman, Internal Medicine Specialist"
},
{
"id": 153649,
"tgt": "Suggest treatment for loss of appetite after surgery for colon surgery",
"src": "Patient: hi,i have a grandmother,she is 80's of age ,she was operated just this year.She a colon a cancer and the doctors remove some part of her intestine to prevent the spread of cancer.She pooh on her belly and not from her anus.bec. the Dr. here in Phils.cut the connection of it. My grandmother lose her appetite and she is now confine.The Doctor said she have a knot intestines.Is it possible?How they will solve the problem?Thank you from a Pharmacy Grad. apple. Doctor: Hi,Thanks for writing in.Your grandmother might have got a colon surgery and permanent colostomy done. It will help to provide her with nutritious food including proteins and fluids. Please know what foods are her favorite and she might take that. Loss of appetite is common in patients fighting cancer. Is it important to know if she is taking chemotherapy to control cancer. This also causes weight loss and difficulty in eating.At her age she should be given a daily 1800 kcal diet mostly containing fluids and fresh juices which are easy to ingest and can be given regularly in small amounts. Please ask her if there is pain which is making her uncomfortable and give her pain relief. Please do not worry."
},
{
"id": 135362,
"tgt": "What causes jaw muscle swelling with neck dysphagia?",
"src": "Patient: hi I seem to have this constant feeling that something may be stuck in my neck. also muscle under my jaw seems to be swollen on the left side (i think its the Mylohyoid muscle)Some people tell me that i might have acid reflux but others say i have tonsils Doctor: HiWelcome to healthcaremagicI have gone through your query and understand your concern.This seems to be due to infection most probably viral infection. You are advised to visit ENT specialist for it. Till that analgesic such as ibuprofen is required.You can discuss with your treating Doctor about it.Hope I have answered your query.If you have any further questions then don't hesitate to writing to us . I will be happy to help you.Wishing you good health.Take care."
},
{
"id": 146174,
"tgt": "What causes recurring dizziness on standing up?",
"src": "Patient: I suffer from dizziness and loss of balance every time I bend over or squat. The dizziness occurs when I stand up. In the morning, getting out of bed causes the same thing. I took my blood pressure when I first wake up. It is normal. I exercise my hands, arms and legs before I get up. My BP stays normal. But when I stand, it goes down and my heart accelerates and I get very dizzy. What kind of doctor do I need? Doctor: I read your question carefully and I understand your concern.The way you describe your symptoms it seems like a case of orthostatic hypotension (excessive fall in blood pressure when you change to an upright position). If it was a case of separate rare episodes it might be normal but since you describe the episodes as happening every time the cause must be looked into.The possible causes could be many. If you are taking certain drugs (like antidepressives, antihypertensives) it could be a side effect of theirs.Otherwise a complete blood count and routine biochemical tests would be the first step in my opinion (to rule out diabetes, electrolyte changes, anemia etc).Regarding the specialist since the possible causes are many, belonging to different specialties being checked by a general practitioner or an internist would be wise. If I had to pick a more specific specialty than since cardiovascular causes are the most frequent ones I would say a cardiologist.Hope to have been of some help. Feel free to ask further questions."
},
{
"id": 116901,
"tgt": "How to treat HLAB27 after delivery?",
"src": "Patient: hi...im 27yr old....got married on 11may2011....jus nw im 3 months pregnant.....i used to have pain in my back n buttock region fm past 5n half yrs....i did all blood test...it cums HLAB27 positive....bt crp was coming neg....nw also i used to have pain...sumtimes in groin region n nw in buttock n stiffness in back.....plz jus tel me wat all drugs i can take nw n after delivery?..can i tk ecosprin75mg til 7th month?my height is 5ft1inch...wt nw is 78kg... Doctor: Hello and welcome to HCM,pain in back and buttock region with positive HLA B-27 is suggestive of ankylosing spondylitis.Ankylosing spondylitis is an autoimmune and degenerative disorder characterized by auto-antibodies directed against the joints of the backbone.The management of ankylosing spondylitis is primarily with steroids.Painkillers can be taken for the back ache.During pregnancy there is restriction in type of medications which can be taken, However, after delivery of the child there is no such restriction.Thanks and take careDr Shailja P Wahal"
},
{
"id": 107514,
"tgt": "What causes low back pain and shortness of breath?",
"src": "Patient: I m a 37 yo female who has experienced chronic low back pain for 1 1/2 years (main complaint), EXTREME fatigue and noticeable joint pain for years. Over the last 8 mths my blood pressure has been a little high, mainly the diastolic # (122/95) I ve had a total of 3 kidney stones, but the last one I had was the largest 1.3cm and required lithotripsy to be removed. It was in my kidney for 2 years, without causing any problems. Due to the other random symptoms I was experiencing, I decided to have it removed to eliminate it as a cause to anything that was going on. Every urine specimen I gave had blood and tissue in it. Also, I will mention that I was diagnosed with asthma after my youngest child was born (2009). I had never had any symptoms prior to that. I never was happy with that diagnosis, because I know people with asthma and my symptoms didn t seem anything like theirs. The meds may have made a little difference but not a significant difference. I remember asking my pulmonary dr if there could be something with my heart causing the shortness of breath, because it felt more like my hear or diaphragm was weak and not strong enough to draw in a deep breath. Not sure if that makes sense to anyone but me though\u2026. About 2 months ago, I started experiencing swelling in my feet. About 3 weeks ago I started experiencing intermittent nausea. Last week I started experiencing intermittent sharp chest pains and shortness of breath. I saw my dr and he did an EKG in his office. It looked good. He examined me and said he heard a possible clicking noise in my heart, so he ordered an abdominal ultrasound and ECHO. The echo came back with 1)Left Ventricular systolic function is borderline reduced 2)The estimated LV Ejection Fraction is 45-50%. 3) There is borderline global hypokinesis of the left ventricle. The ultrasound shows 1)The kidneys demonstrate mild increased echogenicity, which may be represent chronic medical renal disease, but are without evidence of suspicious solid renal mass, hydronephrosis, or shadowing stones. 2)There is a benign and simple-appearing cyst seen at the left kidney, measuring 1.5 X 2.6 X 1.3 CM. The left kidney measures 10.9 in greatest dimension, while the right kidney measures 10.1. I had a CT (w/o contract) in May of this year, just before the lithotripsy and nothing was noted or mentioned about anything with my kidneys, other than the large stone. I m worried about about an abdominal aorta aneurysm, for the cause of my heart issues, as that would explain the low back pain. When the ultrasound was done, she focused only on my upper abdomen and didn t go below my belly button. What are you thoughts on the heart impressions ? Are they symptoms of something else (like an aneurysm) or is this a condition/disease/defect itself? My grandfather died from a massive heart attach. We were told he had a thin place in the main artery to his heart, and that it was genetic, but only the males in our family needed to worry about it. Also, your thoughts on my kidney findings would be great. Does an ultrasound show better images than CT, that would explain why nothing showed on the CT I had a few months ago, or is this something that has developed this fast? :-/ Doctor: All your symptoms suggestive of kidney involvement with systemic symptoms If u have low back ache tell me does it radiate to lower limbs? If yes which limb And how far are u able to walk in terms of metres?"
},
{
"id": 145226,
"tgt": "What causes dizziness, right sided impaired vision, head numbness and hearing loss?",
"src": "Patient: my husband has been experiencing dizziness, numbness to right side of head with hearing loss in right ear and impaired vision is right eye. He has iddm type 2. he has undergone several test including ateriogram. Went and seen vascular specialist yesterday at vanderbilt. no surgical intervention was indicated. what could this be. The doctors localy just dont have a clue. Doctor: Hello ! I read your question and understand your concern. In my opinion the troubles he has may be related to his inner ear. The hearing loss is very typical of this . As you mention in your question , he has done several tests. I am taking into account that the brain MRI has resulted normal, because a tumor of the right pontocerebellary angle would give similar symptomes. Diabetes can cause cranial nerve neuropathy.The impaired vision may be related to cataract. I would recommend an ophthalmologist consult to examine better the problem. If the problem is outside the eye, but in the optic nerve visual evoked potentials would be necessary to locate the problem. The acustic and vestibular nerve can be affected by this disorder.I would recommend brainstem auditory evoked response, which would confirm the diagnosis. The numbness in half of the head may be related to the trigeminal nerve disorder, like the two other nerves mentioned above. Hope to have been helpful!Greetings, Dr. Abaz Quka Hope to have been helpful."
},
{
"id": 39857,
"tgt": "Can gangrene foot be treated without amputation?",
"src": "Patient: gangarene foot can be treated without amputation ? is it possible in any type of medicine like allopathi,ayurveda,homeopathi etc....? if passible please let me know....beacause i dont want to cut my gangarene foot .... my email YYYY@YYYY Doctor: HIThank for asking to HCMI really appreciate your concern the early detection is matter if the Gangrene being detected in very early stage then this can be treated successfully without the amputation but this is happened very rarely and patient need to go for amputation there is no other option in any other pathies take care and have nice day."
},
{
"id": 51929,
"tgt": "What does this report signify and is it enough matched to undergo a transplant operation for CKD patient (stage V) ?",
"src": "Patient: I am a CKD (stage V) patient and now on Hemodialysis since jan,2011. My height is 5 5 , dry Weight 50 kg. My age is 22. I Want to transplant. Here is the report of HLA tissue test and cross match report. I want to mention that doner is the sister of my mother. HLA Tissue Type: Patient: DRB1 : 04,07. Other than DRB1 : DRB4. A Locus : 68,_. B Locus : 44,55. C Locus : 1,_. Others : BW4,BW6. Doner: DRB1 : 07,15. Other than DRB1 : DRB4. A Locus : 1,34. B Locus : 13,44. C Locus : 3,_. Others : BW4,BW6. Cross Match Report: T cell cross match: at 4 degree centigrade : 10%. at 37 degree centigrade : 10%. B cell cross match: at 4 degree centigrade : 5%. at 37 degree centigrade : 15%. What does this report signify and is it enough match to undergo a transplant operation ? Doctor: You have a less than perfect match. If you do not have any other donor,You can still go ahead with transplantation but with the use of an induction agent like Anti thymocytic globulin and higher immunosuppression. You will have an increased risk of graft rejection than others with a better match."
},
{
"id": 185593,
"tgt": "What causes hardness and electric feeling at the tip of the tongue?",
"src": "Patient: ok this seems a little odd to me but my fiance said that last night the tip of his tongue felt like he had stuck it to a very strong 9 volt battery ...... This is the second time in about a week or so I thought he was dreaming the first time but he was actually awake last night. Doctor: Hello, Thanks for your query.Tingling tongue can be caused by a wide variety of conditions. Common causes include stroke, head injury, reduced blood flow, vitamin or mineral deficiencies, and neurologic diseases, such as multiple sclerosis (disease that affects the brain and spinal cord, causing weakness, coordination and balance difficulties, and other problems).I do hope that you have found something helpful and I will be glad to answer any further query.Take care"
},
{
"id": 81773,
"tgt": "What causes sharp pain in my lower rib cage?",
"src": "Patient: when I am in a seated position and bend over I get a stabbing sharp pain just below and under my lower left rib cage with my legs crossed in that position it is even worse. When I bend over from a seated position as if to pick up something off the floor on the right hand side of me no pain when I bend over as to pick up something off the floor on the left side of me I get extreme sharp pain..pancreas? YYYY@YYYY Doctor: Thanks for your question on HCM. In my opinion it is muscular pain only as it is associated with movement. It is aggravated by certain position and movements. So possibility of musculoskeletal pain is more. Pancreatitis pain is usually constant and more in the back.So try to follow these steps for better symptomatic relief in muscular pain. 1. Avoid heavy weight lifting and strenuous exercise. 2. Avoid bad postures in sleep. 3. Avoid movements causing pain.4. Take good painkiller and muscle relaxant. 5. Apply warm water pad on affected site."
},
{
"id": 33465,
"tgt": "Suggest treatment for dog bite",
"src": "Patient: My dog bit my thumb just a few minutes ago. Most of it is just a bruise but it scraped some skin off and it bled slightly. I washed it with anti bacterial soap, poured peroxide over if and then put betadine solution on it. My dog is up to date with vaccines and everything but should I worry about it? Doctor: Hello dear!I have gone through your query and understood your concerns!I would say you to calm down and not to be worried about it.You have done a very good thing that you have washed it enough and disinfected with appropriate solution.If your dog is vaccinated regularly you are protected from severe conditions like rabies.I wish you a good health!Take care!DR ERIOL."
},
{
"id": 170,
"tgt": "Is conception viable from a posthumous sperm?",
"src": "Patient: hi - i want to conceive a child for which i have post humuous sperm from my husband..i haven't seen a fertility doc yet, but the sperm bank is telling me i have a one in a million chance of conceiving. the viability of the sperm is 15% - could the sperm bank be correct or do i still have a chance? Doctor: Hello,No sperm bank result varies plus semen analysis result varies when you give sample in different time, so visit a gynecologist. Do serum TSH and serum prolactin then do HSG on day 8th then do follicular study after taking Clomiphene from day 2 until follicle size increases to 18 mm then rupture and then IUI for early results.Hope I have answered your query. Let me know if I can assist you further.Regards,Dr. Sheetal Agarwal"
},
{
"id": 54646,
"tgt": "Is an MRI suggested to confirm liver cancer?",
"src": "Patient: I was diagnosed with stage 4 liver cancer but my alt is 19 and my ast is 18 all my blood work is normal. Does this mean my liver is funtioning fine i have a 5mm tumor and a 2 mm tumor. Im not sure if i shoud do chemo yet i have started some natural remidies do i ask for another mri to see if its working? Doctor: Thank you for posting your query.Liver functioning maybe reflected by testing your serum albumin and clotting profile (PT and aPTT).MRI is aids in locating the tumor and also indicates any metastases.Histological evaluation (liver biopsy) is required to confirm the benign or malignant nature of the tumor. sure, you may begin with a round of natural remedies but be sure its under the observation of a \"professional herbalist\", otherwise certain herbs may harm. Turmeric powder has anti-cancer properties.however, tumors are small, localized and maybe treated with \"ablation therapy\". Better to consult a liver surgeon for this purpose.anyhow, wish you speedy recovery and good health by any means it comes to be.hope to answer your concern.any further queries are welcomed. Health professionals aim to diagnose properly and manage patients according to their limited knowledge. Cure is blessed by the ONE who Created us, whose power and knowledge is unlimited .wish you good health.regards,Dr Tayyab Malik"
},
{
"id": 117635,
"tgt": "Suggest treatment for glansman disease",
"src": "Patient: hello i want to ask about my sister she has glansman disease about a month a go she had a baby birth cesarean but shes bleeding since then ! she took blood and platelets but its not working and the platelets doesnt work anymore her body is refusing it ! shes talking ciclocapron now and hermons .. is there something else we can do ? Doctor: It is being month that you have not stopped the bleeding. even blood and platelts are not accepting.you should start with the low dose steroids like prednisolone. low dose steroid will help you accepting the other platelets. Thnx"
},
{
"id": 26713,
"tgt": "Suggest treatment to control blood pressure",
"src": "Patient: Hi..I have a fear of having my bp taken because I know it s going to be high , I have (white coat) and now hypertension) not on meds yet because my doc wants to complete his tests. How can I controle this fear and calm myself in the hospital environment and bp monitor? I try so hard . Doctor: Hello!Welcome and thank you for asking on HCM!I understand your concern, and would like to explain that this is a common syndrome affecting up to 2 % of patients. If your blood pressure is high only when monitored by the medical staff or in a medical institution, there is nothing to worry about.White Coat syndrome is a benign type of episodic hypertension, which doesn't cause any higher cardio-vascular risk compared to normal persons and as such no treatment is required. It is really difficult to control your anxiety, but psychotherapy and anxiolytics can help you manage better this situation. I recommend to closely monitor your blood pressure at home different times during the day and perform some tests like: a resting ECG, chest x ray, blood lab tests (complete blood count, fasting sugar, blood lipid profile, thyroid hormone levels, kidney and liver function tests, blood electrolytes, etc.), to exclude other possible causes of secondary hypertension. Hope to have been helpful!Best wishes, Dr. Iliri"
},
{
"id": 25779,
"tgt": "What is causing my high blood pressure?",
"src": "Patient: i just had my blood pressure checked - 152 over 106. i'm a really healthy guy, workout alot - but i do have some stress in my life. I was about to donate blood and could not. Is this fine to wait until tomorrow to call and schedule a doc visit or should i go to an Urgent Care facilitiy? Doctor: Thanks for your question on Health Care Magic. I can understand your concern. No need to urgently visit emergency room. You can wait and visit your doctor tomorrow. Stress is the biggest risk factor for hypertension. Other risk factors are old age, family history, high salt intake, diabetes, dyslipidemia etc. So first of all avoid stress and tension, be relax and calm. Consult your doctor and get done blood tests to rule out diabetes and dyslipidemia. Strict salt restriction in diety is needed. Don't worry, with antihypertensive drug, your blood pressure will be controlled. No need to worry for much. Consult your doctor tomorrow and discuss all these. Hope I have solved your query. I will be happy to help you further. Wish you good health. Thanks."
},
{
"id": 207854,
"tgt": "Recommend help for mental illness",
"src": "Patient: Hi, may I answer your health queries right now ? Please type your query here...My close relative seem mentally sound. She plans everything quite nicely but she suddenly gets off mood and gets really very angry remembering ten different things which must have happened in the past. Some of them don t seem true. Is she mentally sound? Doctor: Brief Answer: consult psychiatristDetail Answer:HI dear,first of all since when she has such complain?is there any other associated symptoms like sleep and appetite disturbance, easy irritable, loss of interest, sadness of mood, fatigue, weakness, negative thoughts etc..?and such mood swings and irritability occurs everyday or in what frequency.?for proper diagnosis detail history and mental status examination is necessary for that you must consult a psychiatrist.Thank you"
},
{
"id": 41289,
"tgt": "Suggest treatment for infertility problem",
"src": "Patient: Hi, I am 31 years Old Lady. My Height is 5.2 feet and weight is 65 Kilo. I have been married since 4 years. Now, Me and My husband are planning a baby. We are trying from last more than six months but no success. I earlier had a regular menstrual cycle of 38 days but from last 4-5 months, my period have become irregular. I consulted 1 doctor (Dr. Himangi Nengi) who advised me to do follicular monitoring. In that monitoring, Ovulation didn't occur, so she gave me ovulation Tablets (Fertomid 50) for five days from Day 3rd of my period to Day 7th. I am regularly following this medication from last 2 months and ovulation also happens but still, I am not able to concieve. Can you please suggest what should I do now and please suggest some best Gyneo/Infertility Doctors. Thanks Kaur Doctor: HIThanks for posting your query. First of all the cause for infertility has to be evaluated . It could be due to some problems in you or your husband or both . So both of you have to get evaluated. I advise you to get the following tests done . Serum TSH, FSH, LH, Prolactin, AMH, Ultrasound Abdomen Pelvis and Serum Vitamin D 3. Also get Semen Analysis and Serum Vitamin D3 for your husband and revert back with reports. If you can tell me from which place you are I can suggest some good Infertility Doctors. Hope this information was useful to you. Any clarifications feel free to ask ."
},
{
"id": 176947,
"tgt": "Will my daughter have only stomach upset after having 1 Carprofen 25 ml?",
"src": "Patient: Hi, my 3 year old (30 lbs) swallowed 1 carprofen 25ml and tonight she has been complaining about her eyes hurting, they are a little bloodshot and shes been rubbing them, this has never happened before. I took her in the shower and washed her from head to toe and rinsed her eyes very well, she seems a bit better, under her eyes are a little swollen. I don t know if maybe its from the pill or if she got something on her hands (what idk) and rubbed her eyes. I called p.control earlier and the said she might a upset tummy and thats about it. What do you think? Doctor: carprofen is a NSAID that has been more offen used by veterarians for animals. It is known to cause a lot of Gastrointestinal problems in form of indigestion, pain, nausea, vomiting and diarrhoea. Your child might have accidently applied something into her eyes which has caused such an event. Now make her drink some water(warm). Give her some anti emetic in form of Syp Ondem. She will require only symptomatic treatment."
},
{
"id": 225826,
"tgt": "Have Mirena coil. Irregular bleeding. Abdominal pain, pain in bottom. What is the problem?",
"src": "Patient: Hi i have had the merina coil for 4 years and my doctor checked it 2 months ago. She said it was still in place. The past month i have had bleeding irregular bleeding. Starting off bright red then turning brown. I have had upper abdominal ache the past two days and have starte bleeding again. I also have a pain in my bottom which also goes down my leg now. It just feels as if somethings caught. Do you know what is the problem? Please A x Doctor: Hi,Thank you for posting your question here, I will try to answer it to the best of my abilities.Have you considered having a voluntary ultrasound done to check if you have any fibroids? I could be wrong but they are known to cause sever pain and bleeding in women, so I think it is worth pursuing.I hope this answered your question."
},
{
"id": 46875,
"tgt": "What causes pain during urination after treating kidney stones?",
"src": "Patient: Hi, I have a month ago I was in intensive care had a stent placed inside me and diagnosed with Kidney stones. Im now out of hospital but for the last five days I have pain when peeing. It does not hurt when I am going but as I finish. My GP has prescribed me antibiotics and im drinking water like its going out of fashion... Doctor: HiIf you still have got the stent inside you then that explains the symptoms. 2 out of 3 patients who have stents have what are called stent symptoms which include urge to urinate, frequency of urination, bleeding in urine now and then, pain/cramps in lower abdomen, pain in the loin area in the sided of stent insertion etc. Not all symptoms occur in all patients.Infection has to be ruled out. Hence submit an urine sample for culture and keep drinking adequate fluids. You could take mild pain killers. If pain persists contact the urologist who inserted the stent."
},
{
"id": 30508,
"tgt": "How long will it take to cure bacterial cold and laryngitis?",
"src": "Patient: I have a bacterial cold and laryngitis and started taking amoxicillin last nightI have been blowing my nose (green stuff) and coughing up phlem. How longwill my voice be gone now that I have started the antibiotics. Also, is it good to go to the gym and sweat or sit in a sauna? Doctor: hiwelcome to HCMi read your history, can understand your concern.you may need to start with Tab Augmentin 650mg B D for 5 days. meet local doctor , get done sputum culture done to identify bacteria and antibiotic sensitivity test done.tips.drink hot waterdo salt water garglingthank youtake care"
},
{
"id": 153283,
"tgt": "Is 5-FU effective in curing cancer?",
"src": "Patient: My mom was diagnosed with stage 4 Cancer of an Unknown Primary with Mets. to the bone. Her bones from her knees to her head and all the way up her spine are full of tumors. We believe she has been living with stage 4 cancer for over a year, but she was officially diagnosed 8 months ago. She started with Cyber Knife to her Clival Tumor, which was ineffective after 8 months. She also has been thru a Cisplatin/Gemzar treatment as well as a Carbo/Taxol treatment that did not effect the cancer in the bones at all. In the 9 weeks that she was on The Carbo/Taxol, spots of cancer showed up on her liver. She is also receiving regular treatments of Zometa as well. She is trying one more avenue now with an oral 5-FU and Zometa. My questions are, if those other chemo s did absolutely nothing to change her cancer, how likely is this 5-FU to work? Also, if she has lived over a year with stage 4 cancer, how long can we expect her to be with us? Weakness and pain is constant for her and I want to know how long she will have to suffer for... Doctor: Hi,Thanks for writing in.The medicine 5FU is 5 fluoro uracil and is an anti metabolite. It kills the cancer cells by interfering in the cell growth cycle and stopping multiplication of cells.Unknown primary with metastatic disease is a difficult condition to treat and usually requires treatment with many approaches and regular modifications to control the disease. Stage 4 cancer is advanced disease and survival is unfortunately up to 2 years after detection in most patients. Hoping that your mother has got a less aggressive cancer and it gets under control and she has an extended survival for years.Oral 5FU and zometa might work where Cisplatin/ Gemzar and Carbo/ Taxol failed to show effective response. Pain relief is to be continuously provided to maintain a comfortable pain free survival. Please do not worry."
},
{
"id": 181764,
"tgt": "Suggest treatment for severe jaw pain",
"src": "Patient: I have just returned from Mexico from a week holiday. I have been swimming and snorkling daily. I have pain inside my ear as well as pain on the side of my face. My teeth and jaw hurt like a tooth ache. I have had alot of mucus discharge from my nose at first thick yellow then green. it is thick then just like water right after and will run out of my nose like water. 10 hours later it was thick and bloody. I started to use polysporin antibiotic ear drops (24 hours now) sympoms seem to be getting better. Swimmers ear is what we think it may be but realy don't know. Doctor: Thanks for your query, I have gone through your query.The pain on the face can be because of the tooth infection, nothing to be panic, consult a oral physician and get it evaluated. mean while you can take a course of antibiotics like amoxicillin and metronidazole(if you are not allergic).The discharge from the nose can be because of the respiratory tract infection, you can consult a ENT surgeon.I hope my answer will help you, take care."
},
{
"id": 18658,
"tgt": "What is the life expectancy of a person with final stages of heart failure?",
"src": "Patient: I am 36 years old and have heart failure with my heart pumping 20% at the moment, my doctors tell me my lungs are what they expect to find on an 80 year old, a few weeks ago I suffered kidney failure and was admitted to ITU for dialysis for several days, can you please give some indication of my life expectancy as my doctors avoid this question Doctor: Hello and Welcome to \u2018Ask A Doctor\u2019 service. I have reviewed your query and here is my advice. No exact answer, I see patients with heart failure with EF 20 living in reasonable conditions. However, unfortunately, you have multiple problems that may complicate your condition and can bring it down to 1-5 years. I hope God's hands touch you and improve your health to go beyond our expectations."
},
{
"id": 117550,
"tgt": "Is the dark blood sign of the anemia?",
"src": "Patient: HIJust come from an appointment with a rhuematologist in which l did an ordered blood test. The blood in the tubes after being drawn was dark red, almost black in colour. It was also quite thick and difficult to pour.I have normocytic anaemia, mild connective tissue disease, P-ANCA positive and proteinuria.Is the dark blood just another sign of the anaemia?ThanksTori Doctor: Hello, Thnx to contact us. If I am your treating doctor I advice you that when you have anemia your blood must be light red in color not a dark red. So you must be having a mild anemia. Even in a condition which you have mentioned dose not show a dark blood. If you have anything else to ask, kindly contact me Dr. Arun Tank"
},
{
"id": 216889,
"tgt": "Suggest treatment for fibromyalgia syndrome",
"src": "Patient: I have been taking 3- 4 hydrocodone tablets per day for approx 2 years (prescribed for fibromyalgia syndrome .) To detox, I went to a pain management clinic and was prescribed 0.1 mg. clonidine HCL TID the first day and was told to start buprenorphine 8-2 mg SL BID the following morning. I had to stop the suboxone after the first day as I developed severe itching (mostly face and scalp). I had two C-sections (the last in 2000) and experienced the same side effect (severe itching) after being given morphine. My questions: How long is typical for someone to detox from the amount of hydrocodone I was taking and length of time I was on it? Is it necessary to take both clonidine and suboxone to detox from hydrocodone? Can I detox on clonidine alone? Is severe itching a side effect of either/both suboxone and/or clonidine? Many thanks. Doctor: There are several stages in detox. For the acute withdrawal symptoms 1-2 weeks. But there are several other features of addiction such as the altered mentation, cravings, triggered use that require a longer period of time. clonidine helps some people with the initial symptoms of withdrawal (such as agitation, and goose flesh). But that is all clonidine does. Suboxone blocks the effects of other narcotics taken legally or illegally along with it. As such it provides a safe haven for evaluating ones relationship to narcotics. Severe itching is not a common reaction to either clonidine nor suboxone, but it is more common with suboxone."
},
{
"id": 146951,
"tgt": "Is talking imaginary things a symptom of Parkinson's disease?",
"src": "Patient: My father is 70 year old and is suffering from Parkinson's disease since 7 years. He used to take Syndropa CR, Ropark 0.5 and Parkin 2mg and Entacom as medicine. For last few days he has development a new symptoms of talking wrong imaginary things. Is this common to parkinson disease. Doctor: there are 2 possibilitiesa. this may be a part of the dementia. the dementia may make the person talk irrelevently. the dementia is present in a large number of patients and often it comes late. b. may be an effect of the medications, the dopamine based medicines may have simlar effects. it is known to cause such symptoms.best to meet a neurologist for the readjustment of the dose.Best of luckDr Mittal"
},
{
"id": 29841,
"tgt": "What causes loss of voice after taking Cefdinir for cough?",
"src": "Patient: I just started on cefdinir for a bad cough and sinusitis...now I totally lost my voice as I did when I was given levaquin a number of years ago...are these drugs including cipro related? I feel I should avoid them altogether...I have no trouble w az pack...thank you Doctor: Hello! I have been through your question.Related your concern I think that you should know that Cefdinir has hoarseness (loss of your voice) as a side effect. Cefdinir belongs class of cephalosporins and has not relation with cipro and levaquin that belong fluoroquinolone class. You should speak with your doctor to decide to stop or not Cefdinir.I hope my answer helps you. I wish you a quick recovery."
},
{
"id": 167616,
"tgt": "What causes reddish and blotchy skin in infants?",
"src": "Patient: My 3 month old baby was kind of fussy and I was holding her trying to feed her and she just kept screaming and when I looked down at her right leg it was bright red so I touched it and it was really warm. She finally calmed down and fell asleep but now her leg is red and splotchy. What is wrong with her? Doctor: Hello. I just read through your question.This could potentially be a skin infection. I recommend consulting with your doctor so the correct diagnosis can be made and the appropriate treatment plan implemented."
},
{
"id": 190044,
"tgt": "Is it possible to have crowding corrected in 8 months?",
"src": "Patient: Dear Dr, i would like your opinion on a braces matter i went for a consultation for ice braces & the doctor told me i will need only 8 months for the braces . i am getting married first of Sep & i told him if i needed longer time i wont be able to do it. he is a very reputable doctor & wont decieve me for the money as he already has a thousands of patients. i am however worried about the 8 months time frame . is it possible to have crowding corrected in 8 months ? thanks alot Doctor: hello and welcome to hcm forum, i would be glad to answer your query, first of all i , with all due respect to you and your dentist i would like to inform you that being a dentist we cannot assure you of the time period a tooth takes to moves from its original position to a new position. you have not mentioned your age here, but i believe you might be older than 20. ideally, orthodontic treatment is done during the growth years (puberty), but in some cases, we do perform orthodontic treatment beyond that age as well, as in your case orthodontic treatment can be done but the time & the results cannot be decided before. As the teeth and jaws have developed fully, therefore to retain them in a new position , you will need permanent retainers which hold the teeth in an appropriate place, do not worry these are not visible to anyone. i would advise you to get the orthodontic treatment done but avoid unrealistic expectations, just have faith in your dentist. i wish you good health, take care."
},
{
"id": 45763,
"tgt": "Suggest health risk with kidney donation",
"src": "Patient: In your opinion, how effective is dialysis in comparison with a kidney transplant? A kidney transplant from a living donor in comparison with a cadaver? What are the health risks of kidney donation , specifically kidney, from the donors prospective? Is it common for donor health to decline after donation? Do you anticipate the number of kidney donors going up or down in the next 10 years? Do you think there will be an end to the shortage of kidney donors in The United States? Doctor: Hi, Donors generally recover well without any complications. However, donors also undergo a major surgical procedure and there is always a risk associated. Hope I have answered your query. Let me know if I can assist you further."
},
{
"id": 1528,
"tgt": "How does Krimson 35 help one in getting pregnant?",
"src": "Patient: Hi Doctor, my age is 26 and I am 82kgs married 2yrs back.I have pcos problem from past 8yrs.I was on contraceptive pill for all these years for regular periods.planning to have kids. gynic prescribed krimson 35 for 3months. how does it help me in getting pregnant...plz advice Doctor: Hi, krimson 35 is also a type of contraceptive pill and you can't be pregnant on this. You have to take medicines like clomiphene for the growth of your follicles and track your follicles growth by repeated ultrasound. When follicles reach a size more than 17 to 18 mm, take injection for rupturing the follicles. Be in contact with your husband for next 2 to 3 days. Take progesterone for next 2 weeks. Do a urine pregnancy test at home after that. You can try like that for 3 to 6 cycles. Talk to your doctor regarding this. Hope I have answered your question. Regards Dr khushboo"
},
{
"id": 174483,
"tgt": "What causes sudden dilation in eyes with pale lips and headache in children?",
"src": "Patient: We just moved and I don t have insurance in this state yet. My 11 yr old daughter just woke up...came into kitchen, eyes dilated to the point of no color, lips pale, said head hurts and felt like she was going to throw up,she couldn t even talk to me. Her eyes rolled in the back of her head and then she fell to the floor.. Eyes still dilated. I got her to stand up and I said Mikayla what s wrong and she said momma I don t know I don t feel good. Please help me. Doctor: HI your child might have a fits like episode based on your description as she felt pale and fallen down,but you didnt mention when she became conscious.If she is able to woke up immediately with your voice , then she might have a syncope due to decreased blood flow to brain due to skipping of food"
},
{
"id": 136565,
"tgt": "What causes pins and needles in right hand?",
"src": "Patient: i have had pins and needles in my right hand , also i get very hot sometimes and also i have had some pains in my chest , which was really bad pain , it lasted for about 5 mins with the pain , the pain went away but even now as i type i have the pins and needles in my right hand , i dont use computers a lot . what should i do ? Doctor: Hi,Thanks for your query.The abnormal sensations that you have such as burning, pins and needles are termed peripheral neuropathy. Peripheral Neuropathy is a condition caused by damage to the peripheral nervous system [most commonly seen in diabetic patients]. Another possibility could be nerve compression in spine due to disc bulge.Change your diet and remove inflammatory foods such as citrus fruits, dairy products and wheat that may aggravate the condition. Simultaneously increase your intake of potassium and calcium-rich foods such as bananas and avocados and green leafy vegetablesVitamin B12 is important in the maintenance of health and normal functioning of the nerve tissue and deficiency of it can lead to numbness. You can increase B12 level by taking meat and fish, or you can take tab methylcobalamine .I do hope that you have found something helpful and I will be glad to answer any further query.Take care"
},
{
"id": 195231,
"tgt": "How safe is taking Humog and pubergen to improve sperm count?",
"src": "Patient: hello doctor my husband count was less than 1 million. i am 31 and he is 36. we went through iui, ivf but all in vain. But once got conceived thro iui but it was a missed abortion :( Now one otehr doctor prescribed Humog and Pubergen injections for him and his count is increased to 11 million. How can I progress further Doctor: Hello and Welcome to \u2018Ask A Doctor\u2019 service. I have reviewed your query and here is my advice. Any medication has its own side effects, then you have to select benefits out weighing the risks. Common side effects are bloating, stomach pain, pelvic pain,nausea or vomiting, rapid weight gain, allergic skin reaction,irritability, enlargement of breasts etc. Hope I have answered your query. Let me know if I can assist you further."
},
{
"id": 53437,
"tgt": "Is gamma test and STPI, STPD of liver showing high values concerning?",
"src": "Patient: Hi, I am Sandeep, 30 yeares old guy. I got fever last one month back and took medicine and got curied. But from 15 days i am getting hot feeling inside. eventhough outside i wont get any hot feeling. Checked all parameters like typhod, malaria, blood cell count and everything ok....but only gamma test and STPI, STPD of liver showing some high values... Please suggest Doctor: Hi,AST (SGOT) and ALT (SGPT) are reasonably sensitive indicators of liver damage or injury from different types of diseases or conditions, and collectively they are termed liver tests or liver blood tests. So the next step is to look for certain liver damage and most common cause is fatty liver disease. It should be confirmed by ultrasound. Other causes are viral hepatitis, cirrhosis, medications, alcohol intake or autoimmune diseases. To verify exact cause, US, bilirubin levels and tumor markers should be additionally done. Then appropriate treatment can be started.I hope I have answered you query. If you have any further questions you can contact us in every time.Kindly regards,Ivan R. Rommstein"
},
{
"id": 221999,
"tgt": "What causes irregularities in menstruation?",
"src": "Patient: Hi, i am 23 years old and have always had a regular period. I recently had unprotected sex and I got my period 2 weeks later but it has now been going for 9 days. This has never happened before. My period started normally but is now very light with some little clots. Could I be pregnant? Doctor: Hello dear,I.understand your concern.In my opinion the delayed period with prolonged bleeding definitely needs to rule out pregnancy.I suggest you to do a urine pregnancy test.If the test is positive the bleeding can be considered as miscarriage.If negative the bleeding might be due to some hormonal imbalance.Avoid stress.Relax.Consider doing a urine pregnancy test.Whatever may be the result consult gynaecologist.Best regards..."
},
{
"id": 192428,
"tgt": "What causes erectile dysfunction?",
"src": "Patient: My husband is not complaining tht he dont feel naythin inside me and he is loosing his erection again and again ..i dont kno what is the problem .Iam 23 yr old and he is 25 years old and we have daughter 18 months old .This started happening first during my pregnancy then it got back to normal after my daughter but now from few months it started again .We use condoms is it because of them? Doctor: Hello If you have started using condoms now then it can be the cause of his loss of erection. Stimulation of penis during sexual intercourse keeps it erect. Condom causes decreased sensation and hence can lead to loss of erection. In addition post child birth the vaginal canal gets lax which further reduces the sensation. You can calculate your fertile period and try having sexual intercourse in early non fertile days without condom. This can be helpful in ascertaining the cause. Hope I have answered your query. Let me know if I can assist you further. Take care Regards, Dr. Rahul Kumar"
},
{
"id": 57111,
"tgt": "Suggest treatment for pancreatic attack",
"src": "Patient: Hi DoctorMy Husband is suffered with cute pancreatic attack he is very unconsciousness due to gas can please suggest us what to do so gas is not make in his stomach and if make it can pass out please suggest me what to do? he is taking Tab Razo at empty stomach and antoxid and Bivon syrup but these medicine cant help him. Doctor: hi sorry for the situation your husband is on. With regards to stomach gas after acute pancreatic, it is common. If I were your treating doctor I would prefer pancreatic enzymes as treatment. you can start to use Creon 3 X 25000 UI per day before meal. Wishing him fast recovery DR.Klerida"
},
{
"id": 168287,
"tgt": "What causes penis swelling and redness in an infant?",
"src": "Patient: hi, when i bathed my 1 yr old son, i noticed his penis was swollen and red, once i tried to examine it, i noticed puss coming out, even more so once i pulled the foreskin back just a little. He didnt like this so must be hurting him as generally he doesnt mind when i clean him usually. what could this be? Doctor: Hello! Thank you for writing us here. Its normally because of infection because of diaper (most likely). I would suggest you to give him plenty of water and good food. Also check if his diaper is changed regularly. He would be alright in 5 to 7 days. Best regards,Dr Gunjan"
},
{
"id": 70010,
"tgt": "What could be the swollen knot right where the legs meet?",
"src": "Patient: Good afternoon,I have a swollen knot right where my leg meets up with pelvis near my scrotum. It has been a little bump for over 2 years but now it is a nice size and it hurts. What could that be? Not sure if it is a cyst or lymph node but I am concerned. Doctor: Hello!Thank you for the query.In this location such lump can be caused by inguinal hernia, enlarged lymph node or some kind of benign skin lesion (like lipoma or sebaceus cyst). As enlarged lymph node may need some diagnostics and inguinal hernia needs to be fixed with surgery, I suggest you to consult a surgeon with this issue.In case of doubts, ultrasound should be done.If its enlarged lymph node, biopsy might be necessary to verify it.Hope this will help.Regards."
},
{
"id": 88617,
"tgt": "Suggest medication for abdominal pain",
"src": "Patient: I had a bowel obtruction 10 year s ago . now i have had a 1 bowel movement after 13 days that is with taking pico salax and lactulose and now i;m on my 11 day again i;m bloated and pain in stomach and tired and having some memorie loss and the doctor has me on senokot-s 2 tablets 4 times a day lax a day17 gram lactulose 2 tbs 3 times a day and dulcolax suppository daily and i still cant go . Im having some gas so that is good but now what should i do and how long should i wait to see doctor again . Doctor: HIWell come to HCMIf you have history of obstruction of bowel (Not exactly given which bowel was obstructed) neither stated what type of treatment you have given at that time, if the constipation is due to the intestine pathology then this need to be treated accordingly no question is there for waiting, because this is chronic history, and this need to be brought for investigation and management as early as possible, hope this information helps, take care."
},
{
"id": 132168,
"tgt": "Suggest remedies for redness and swelling in the bone above ankle after an injury",
"src": "Patient: I got hit by a softball (bounced up and hit me...no line drive) about 6 inches above my ankle bone a week and a half ago. It hurt a little, but nothing terrible. I ran 3 miles the next day and a 5k 4 days later with no pain. 9 days in and it s now red, swollen, warm to the touch and extremely sensitive. I m concerned about cellulitis. Is this possible with no open wound? I am a 33 yr old active woman. However, I did just get diagnosed with pneumonia a week after this injury. What should I do? Doctor: hi you had an injury with softball in your leg about 9 days ago and have been diagnosed with pneumonia a week after this injury . Now you have swelling and redness in the injured area and you are suspecting cellulitis since it is red ,swollen and warm to touch. I would think that probably a hematoma ( Collection of blood) ,had formed at the time of injury which went unnoticed since it did not cause any problem,and mow it is getting infected. I would recommend aspiration to see if any abscess is there and if present would drain it. Antibiotics will continue if pus if there."
},
{
"id": 23838,
"tgt": "Is dizziness symptom of orthostatic blood pressure?",
"src": "Patient: Hi, I have been told that I have orthostatic Blood Pressure. I have been having a series of dizziness and black outs. I stumble and fall a lot. I can be walking and be unstable in my gait. Do any or all of these symptoms come from orthosatic bp? What is a definition of orthostatic bp, and why does it hapen? Doctor: Hi! I had gone through your question and I understand your concern .Yes these symtoms are caused by orthostatic or postural hypotension ( orthostatic blood pressure)It happens when you stand up - gravity makes the blood to pool in your legs so the heart has less blood to pump - low blood pressure ( means less blood will go in your brain and the other part of the body ) .Normally our body has natyral process to fix the hypotension problems in these process ( in heart , nervous system , metabolic problem ) cause orthostatic hypotension. The important thing is to find the main cause of the orthostatic hypotension here are some that may cause : - Dehydration ( diarrhea, vomiting , fever , not drinking enough water .- Heart problem :Slow heart rate (bradychardia ), heart valvue problems )-endocrine problem :low blood sugar ( hypoglicemia ), Addison's dissease .-Nervous system disorder .If I were you treating doctor I would check your heart , ( ECG , Ultrasound ) do some blood test to check your blood sugar . You have to do an appointment with your doctor for that .Meanwhile drink plenty of water . Wish these help! If you have any additional question please feel free to ask me ."
},
{
"id": 18197,
"tgt": "What causes shortness of breath and fatigue after an open heart surgery?",
"src": "Patient: Hi. My father had open heart surgery in March. Up until then, he also had shortness of breath, low oxygen, and fatigue. For a month or so after the surgery, he felt better, but is now experiencing the same symptoms he had before the surgery. He had a nuclear stress test which showed no blockages and a CT scan of his lungs which showed nothing either. He has type 2 diabetes. Years ago he took medication for an under-active thyroid. Doctor: Hello and Welcome to \u2018Ask A Doctor\u2019 service. I have reviewed your query and here is my advice. Shortness of breath, fatigue, may be due to cardiac illness, pulmonary disease, allergic conditions, anemia, infection, all these needs differentiation, beside these shortness of breath and fatigue may be functional in origin, condition demands the clinical examination and some basic tests like routine hemogram, EKG, ECHO. Hope I have answered your query, Let me know for further assistance."
},
{
"id": 12091,
"tgt": "Can i try for a child if i use tetralysal 300 for dark underarms ?",
"src": "Patient: Im a 30 years old female and have had tetralysal lymecycline 300 about 4 yrs ago for dry patches i developed under my underarms i was also told that it cud childause our baby to hav yellow teeth . my dr put me back on as my underarms are now very dark and is spreading which makes it very hard to be confident as it looks very unsightly. we have also just decided to start trying for our 2nd child and how bad is it for conception or for the unbron baby if we were to fall pregnant while on this medication??? Doctor: Welcome to Healthcare Magic The mother needs to stop taking this medication and request Doctor for alternative medication while trying to get pregnant. For what reason are you both taking this, you haven't specified. You probably have developed drug reaction. Do get your blood sugar tested to rue out Diabetes as sometimes it can present with dark thick patches in the armpits. Kindly ask the Doctor to change the medication as tetracyclines can arrest bone growth in children. The mother can take Folic acid tablets under medical supervision once a day from one month before conception for safety from Neural tube defects in the baby."
},
{
"id": 84126,
"tgt": "What are the side effects of metformin 500 mg?",
"src": "Patient: hi doctor,am 27wks pregnant..am on metformin 500mg thrice daily for past two days..bt yesterday i had 3times loose motion,abdominal pain,nausea too..am very tired today,no bowel movement till this noon,cant do any activities which i used to do previously..please give me valuable solution. Doctor: Hello,Your symptoms seem to be related to side effects of Metformin. Metformin\u00a0has a very low risk of birth defects and complications for your baby, making this drug safe to\u00a0take\u00a0before and\u00a0during pregnancy.\u00a0Metformin\u00a0is also safe to\u00a0take while\u00a0breastfeeding. The symptoms will probably go away as the body get used to the treatment. If the symptoms continue, you may consult your doctor for insulin therapy.Hope I have answered your question. Let me know if I can assist you further. Regards, Dr. Dorina Gurabardhi, General & Family Physician"
},
{
"id": 133415,
"tgt": "Suggest treatment for persistent joint pain and swelling in finger and leg",
"src": "Patient: Hello Sir,My wife Chaitali Prasad, age 37 yrs old she is suffering for joint pain since one year. Sometimes his finger and leg are swelling and she does not any work and movement this time. So, I request kindly proper advice us. Thanks & Regards Sanjay Prasad (Husband) Doctor: Hello sir,As per the history u gave consult an orthopaedician and physician.Your wife might require blood investigations,xray of hands and leg.She might be having systemic illness that is affecting joint of her hands and feet. Need to evaluate certain conditions like gout,pseudogout,rheumatoid disease ... Etc.and depending on diagnosis treatment will reduce her problem.i hope she will get well soon.Thank you."
},
{
"id": 210620,
"tgt": "What causes anxiety when on medication for PTSD, depression?",
"src": "Patient: I'm 68 years old, white male, and have been diagnosed with complex PTSD, severe depression (clinical) depression, and am on a plethora of medications.I have psychotic aspects. I have a feeling of impending doom awaiting me, and am doing my best to try to look positively at my life. Why is it that I always feel like I have to wait before I,m free from anxiety to tackle something new?. I feel like my life will be over before I get started. Doctor: Hello,Thanks for choosing health care magic for posting your query.I have gone through your question in detail and I can understand what you are going through.It seems your anxiety disorder has not been completely treated and the symptoms are still persisting. You have not mentioned which drugs you are currently on. Otherwise I could have suggested you some more measures which can be taken up in your medications. Hope I am able to answer your concerns.If you have any further query, I would be glad to help you.In future if you wish to contact me directly, you can use the below mentioned link:bit.ly/dr-srikanth-reddy\u00a0\u00a0\u00a0\u00a0\u00a0\u00a0\u00a0\u00a0\u00a0\u00a0\u00a0\u00a0\u00a0\u00a0\u00a0\u00a0\u00a0\u00a0\u00a0\u00a0\u00a0\u00a0\u00a0\u00a0\u00a0\u00a0\u00a0\u00a0\u00a0\u00a0\u00a0\u00a0\u00a0\u00a0\u00a0\u00a0\u00a0\u00a0\u00a0\u00a0"
},
{
"id": 32201,
"tgt": "Suggest safe medicine to get rid of fever",
"src": "Patient: My wife aged 36 years is suffering from fever for the last 3 days. Her temp has been fluctuating from 100 - 104. After consulting a physician she has been advised to take the below medicine: Dicloran-A (Twice daily); Timo-O (Twice Daily), Cetrizine (Twice Daily) and Pantraz-D (Twice daily). Her state has not improved much excepting that the fever now comes down to normal and after 10 hours again reaches 100/ 101. Please advise. Doctor: Hi Dear,Welcome to HCM.Understanding your concern. As per your query you have high fever since 3 days and it is regressive down an dif regress comes back again which could be due to viral infection. Need not to worry much. I would suggest you to visit general physician once and get it examined. You should get blood tests done to find out exact level of infection in your body. Till now keep going with the course which your physician already prescribed you as it will take 7-10 days for viral fever to come back to normal. Take anti-inflammatory and NSAIDS combination. Drink plenty of fluids. You should take proper rest. Hope your concern has been resolved.Get Well Soon.Best Wishes,Dr. Harry Maheshwari"
},
{
"id": 130690,
"tgt": "What causes tingling sensation in left shoulder blade?",
"src": "Patient: I started feeling this intermittent tingling (\"electric\") sensation in my left shoulder blade area just beneath my skin. It doesn't hurt but I have never had this occur to me before. The symptom started about an hour ago and it comes and goes (and possibly getting stronger each consecutive episode). Otherwise, I did not experience anything peculiar during the day. I am a 24 y/o female, non-smoker, and not on any medications. Is this something I should worry about? Doctor: Hi,It seems your neck is in wrong posture due to which your nerve has got impinged due to which you are getting such electric sensations. Check your neck position and follow correct neck ergonomics and you acn do hot pack over neck and shoulder region. It will relieve any neck muscle spasm if any and reduce the sensation. Use any topical muscle relaxant gel or spray. It will help you out. Hope I have answered your query. Let me know if I can assist you further.Regards, Dr. Harsh Swarup"
},
{
"id": 204785,
"tgt": "How can anxiety and depression be treated?",
"src": "Patient: I m having some anxiety and depression from the past month. I recently broke up with my girlfriend. I always thought that she is the one and I still think that i lost her forever. Maybe this is not the only thing that keeps me worried. I am always worried about my family, because everyone has anger problems and anything can happen. I want to leave my family, stay alone, and i thought would marry my girlfriend after some years. I am stuck. I always feel like im in a box. Doctor: hi sirin my opinion, presently your entering into depressive episode. your separation with your girl friend was the main stressor for you at present. as your unable to cope up with that stress, your slowly entering into depression. Because of that, your having those feelings of being sad and lonely features. if you neglect it, slowly you enter into severe depression. So, i strongly recommend you to consult a psychiatrist. Mostly cognitive behavioral therapy is sufficient at this stage. If needed, anti-depressant medications can also be prescribed."
},
{
"id": 207529,
"tgt": "What causes short term memory loss, depression and mood swings?",
"src": "Patient: Hi. I recently noticed how i am experiencing short term memory loss. It never used to be like this. Personality change is another thing that im also experiencing. Also personality change overs are taking over me too. One moment I may be lively and jolly , a minute later I would be quiet , depressed.Whats going on with me? Doctor: DearWe understand your concernsI went through your details. I suggest you not to worry much. I think you are a teen ager or adolescent from the description. If yes, there is nothing wrong with you. These mood swings are quite natural and there is no memory loss with you. You are disappointed with some thing. That is all.If you have crossed adolescent, there could be some sort of disappointment in your life which caused you to behave in this fashion. Such behavior is momentary. You should be alright within a month or so. Please concentrate on your studies and career.If you require more of my help in this aspect, Please post a direct question to me in this URL. http://goo.gl/aYW2pR. Make sure that you include every minute details possible. I shall prescribe the needed psychotherapy techniques.Hope this answers your query. Available for further clarifications.Good luck."
},
{
"id": 34751,
"tgt": "Can high fever in a teenager need immediate medical attention?",
"src": "Patient: My 16 year old son has fever (39.8 degree celsius) this afternoon around 5:45 PM so 1 tab Paracetamol 500 mg was given to him per orem. Fever was monitored and went down to 38.2. By 8pm, his fever went up to 39.4 so another tablet of Paracetamol was given. It's 9:30 PM but his fever has not subsided. Should I bring him to the ER already? Doctor: Hello dear,Thank you for your contact to health care magic.I read and understand your concern. I am Dr Arun Tank answering your concern.Yes, if the facility of immediate treatment is available than you can go for the treatment immediately.You can do the laboratory testing of blood for the malaria, typhoid, dengue. Also get tested for urine microscopic examination.This will give you catch about the possible pathogen.Till that time I advice you to take the paracetamol to cover the fever. Please drink plenty of water.Please check for any serious sign like high fever, unconscious ness and hemorrhage. If this happens than you can immediately transfer to the hospital.I will be happy to answer your further concern on bit.ly/DrArun.Thank you,Dr Arun TankInfectious diseases specialist,HCM."
},
{
"id": 2859,
"tgt": "Suggest effective procedure for conceiving",
"src": "Patient: Hi Doctor, I am 26 year old. From last 1 year we are trying for child but I am not able to conceive. Finally we visited to doctor in last month. He advised me to take clofert 50 from 2nd of period for 5 days. However there is no positive result. .I got period in this month. This time doctor again advised me for clofert 50 only. Kindly help and please let me know what are steps for to get pregnant and how much time it will take? Doctor: Hi,You and partner require infertility specialist consultation. I would suggest to go for reproductive hormone analysis, USG scan and ovarian follicle study for you, and semen analysis for your partner for better management. Specific treatment will depend on tests results. Clodert will use to induce ovulation. It should be used for 4-6 cycle and regular ovarian follicle study will require to monitor ovarian follicle growth.If there is no maturation of follicle and ovulation after taking it then HCG shot shot should be taken. You can also ask for progesterone pill in second half of period and ovacare to support pregnancy. Avoid stress, take healthy diet, drink plenty of water and do regualr exercise.Hope this may help you, Contact further if follow up needed.Best regards,Dr. Sagar"
},
{
"id": 123531,
"tgt": "How does one stop the progression of osteoarthritis?",
"src": "Patient: I have been diagnosed with severe osteoarthritis at the base of my right thumb and x rays show it in both hands to a lesser degree. It is now bone on bone there and I d like to know what I can do to stop or slow down the progression of it to the rest of my skeletal joints. Thanks! XXXX Doctor: Hello, As the OA has affected the thumb joints bilaterally the first advice will be to stabilize the joint in a maximum number of hours in the wrist brace. Which is widely available at online stores. Also, using hot water fermentation and improving the strength of the muscles of the forearm and hand is advised. We can only avoid the progression of the OA while keeping the muscle stronger to maintain the joint space. In case if we fail to do such then the grinding keeps happening and the inflammation doesn't come down. Hope I have answered your query. Let me know if I can assist you further. Regards, Jay Indravadan Patel, Physical Therapist or Physiotherapist"
},
{
"id": 158502,
"tgt": "Vertigo. Treated for carcinoma multcentric stage III. Administered with Fragmin injections daily and Neupagen shots. Problem?",
"src": "Patient: Hello, Thank you in advance for your response! I am a 40 yr old woman currently being treated for inv.ductal carcinoma multcentric stage III. I had my 3rd cycle of neoadjuvent FEC chemo last Thursday 8 days ago. Chemo nausea subsided and taking no meds other than 0000 IU Fragmin injections daily (past month) for a large Blood Clot running from inner elblow ending at arm pit. -pic line(now removed) - I am also taking (past 6days) Neupagen shots. *** my question or concern today is regarding vertigo. I awoke this morning @ 1 hr ago with loss of balance and blurred vision. Feeling like I had too much to drink on a tug boat in stormy waters. I almost went head first into a wall before catching myself. So naturally I m concerned about a mini stroke precursing a massive stroke or hopefully perhaps low hemoglobin. My symptoms improve when lying or sitting still, but worsen with movement. I m hoping I don t need to make a trip to emerg. for a six hr wait. Thank you Doctor: Looks like you are suffering cancer.Regarding the complains better to do a MRI Whole brain tomorrrow.Take care"
},
{
"id": 83416,
"tgt": "Can Loette cause heavy bleeding and frequent urination?",
"src": "Patient: I ve been taking Loette for two months, and skipped my period inbetween the two packets. I ve now gotten breakthrough bleeding for about 10 days, it s really heavy, just like a normal period and i ve had really bad cramps. is this my period? or is it BTB? is there a chance i m pregnant? when can i expect this to stop? also, i feel a constant need to urinate and when i do, my clitoris burns. I m really worried ): Doctor: HiThis is more likely to be a breakthrough bleeding.Sometimes breakthrough bleeding can occur, even though you are taking the Pill continuously. If the bleeding is light or just spotting that lasts for two or three days, keep taking the hormone pills daily as you have been. If the light bleeding continues for more than three or four days, or is more like a moderate or heavy period that lasts for more than a day, a four-day break from the Pill is recommended. During this time, you should experience a proper period. Start taking the active pills again after four days. Do not use this technique more than once in any four-week cycle. If you miss two or more active tablets, the pill is much less effective as a contraceptive.Rule out any urinary tract infection and adequate hydration is essential.Hope I have answered your query. Let me know if I can assist you further. RegardsDr.Saranya Ramadoss, General and Family Physician"
},
{
"id": 168791,
"tgt": "What causes persistent stomach pain, IBS and constipation in a child?",
"src": "Patient: My daughter is nearly ten and been complaining of tummy pains for the past couple of weeks. There is no temperature, vomitting or diorrhea. She is not going to the toilet regularly and complains that when she does go it hurts or feels like she wants to go, then when she gets to the loo, the sensation stops. She has been prescribed buscopan, first, then movicol, which we had to stop after three days as she was going to the toilet too much and it was diorrhea. Today she went for an ultrasound on her abdomen and some red colour showed up on the screen during the scan. What does this mean? Doctor: SHE REQUIRES COLONOSCOPY AND STOOL EXAMINATION FOR BLOOD IN STOOLS, HAEMOGRAM,LIVER FUNCTION TESTS AND RENAL FUNCTION TESTS. THEN ONLY I CAN HELP .DRUGS USED FOR HER ARE WITHOUT ANY DIAGNOSIS SO HER CONDITION IS WORSENED. PLEASE MEET ME WITH APPOINTMENT AT PUNE"
},
{
"id": 53361,
"tgt": "Is cyst on the adrenal gland be a matter of concern?",
"src": "Patient: While I was pregnant in my third tri mester I was diagnosed with a 8cm liver cyst. I have had three ultra sounds, one ct scan, one mri. I was seen by a liver doc, and noe a surgeon. The surgeon has decided that the cyst may not be on my liver, but on my adrenal gland. I have dicomfort, but they are saying they are 98 percent sure it's benine. I have a history of cancer in my family and myself. The surgeon would like to just leave it. I guess before I can do anything they have to properly diagnose me. Should I leave it alone,cause it disrupt the funtion of my Adrenal gland. Doctor: Hi,Adrenal tumors most are non functional and benign which can just be left for follow up. Functional once and malignant ones which are diagnosed based on symptoms, hormone study and size respectively. Large size tumors are likely to be malignant hence require excision. Preop biopsy is usually not done for adrenal tumors. Tumors of size 4 cm and above requires evaluation for malignancy with adrenelectomy.Hope I have answered your query. Let me know if I can assist you further. Regards,Dr. Safarulla SH"
},
{
"id": 179965,
"tgt": "Is Altris Gel for grey hair treatment safe for children?",
"src": "Patient: Hi Doctor My son is having grey hair. He is 2.5 years old. Our dermatologist prescribed Altris Gel for melanin Improvement & Hitreev oil for strong hair growth.They have suggested to use for 3 months. Is it Altris gel & Hitreev oil is safe 2.5 years old kid? It will not cause any side effect after treatment is over? With Thanks Jagadees Doctor: Thanks for putting up your query at HealthCareMagic. I understand the cause of your concern. Although not very common, it is possible for children of all ages to grow a few gray hairs of their own. One of the leading causes of gray hair in children is a genetically determined. But before I answer your question, let me mention that there are certain simple conditions like malnutrition, vitamin B deficiency or abnormal thyroid hormone levels in the body (either too low or too high) can also cause gray hair. These are correctable causes and need to be considered before you actually try to increase the melanin production.There are some health conditions that can cause gray hair in children. One such condition is alopecia areata, which causes patches of hair loss on the scalp and other parts of the body. As the hair grows back, it may be gray at first. Some children lack pigment cells in a patch of hair follicles at birth. Others lose hair color when the immune system accidentally destroys pigment cells in a localized area. Some forms of poliosis can be inherited. Poliosis may also be associated with certain skin disorders, such as vitiligo, and genetic disorders, such as Marfan's syndrome and Waardenburg's syndrome. Less commonly, gray hair may be associated with childhood disorders such as vitiligo (which also causes a decrease of pigment in the skin), tuberous sclerosis, neurofibromatosis and Waardenburg syndrome. These disorders run in families and have other symptoms such as seizures, tumors under the skin, and hearing loss. If you are concerned about any of these conditions, I encourage you to talk with your child's pediatrician.Protein rich foods like soya, lentils, sprouts, meats, liver, eggs along with biotin and selenium. Foods like carrots, salmon, sardines, nuts cauliflower induce good hair growth as well as prevent further premature greying. Green leafy vegetables, salads fruits are helpful too.Altris gel contains melitane a new compound launched to prevent premature greying. You can use it safely as it would not harm your son's hair. I am not aware of any side effects of Hitreev oil.I hope that helps. Feel free to revert back with further queries if any."
},
{
"id": 222765,
"tgt": "How to control high BP during pregnancy?",
"src": "Patient: Hi all I am from pakistan. My wife have BP problem during pregnancy. Which causing premature birth of babies. 3 Babies died due to prematurity. 2 time operated (Cesarian). Please help me in this regard how to control the PB during pregnancy. I have no children. I got maried in sep-2006. Doctor: HiDr. Purushottam welcomes you to HCM virtual clinic!Thanks for consulting at my virtual clinic. I have carefully gone through your case, and I think I have understood your concern. I will try to address your medical concerns and would suggest you the best of the available treatment options.1] firstly start with B 12, folic acid: at least 3 months before getting pregnant.2] Adopt healthy diet and exercise regimen.3] once she is pregnant , from 14 weeks she can take ASA 76, vitamin C & E. 4] keep close watch on blood pressure every 15 days. She can take appropriate anti hypertensive drug dose at the onset of hypertension.I hope my answer helps you.Thanks."
},
{
"id": 66437,
"tgt": "What causes lump and swollen vein in groin area?",
"src": "Patient: I have a golf ball sized lump in the right side of my groin, where the leg and the pubic area meet, it has been there for almost a week. The past couple days I noticed my pubic bone area has been tender and now seems to have a swollen vein that goes to the clitoris. I was wondering what this could be and if it needs to be treated by a doctor? Doctor: Hi, dearI have gone through your question. I can understand your concern. You may have some mass or hernia. It can be easily differentiated by examination. If it is hernia then you need surgery like meshplasty. If it is mass then you should go for fine needle aspiration cytology of that lump. it can be benign or malignant lesion. Then you should take treatment according to your diagnosis. Consult your doctor and take treatment accordingly. Hope I have answered your question, if you have doubt then I will be happy to answer. Thanks for using health care magic."
},
{
"id": 55545,
"tgt": "Suggest treatment for gall stones and pain in middle of chest",
"src": "Patient: I had a ultrasound 2 weeks ago and it was determined I had gallstones. At this moment I am having pain in the middle of my chest and in the middle of my upper back between my shoulder blades. I have a headache and slight right jaw pain. Is all of this associated with the gallbladder. My doctor is out of town and have an apt with him July 7th. Should I be concerned about a heart attack. I am 52 yrs old and have been under quite a bit of stress lately. Doctor: HelloChest pain may be due to gall stones,however it may be due to other causes like reflux/acidity,cardiopulmonary causes etc.I suggest for definitive treatment of gall bladder stones.Definitive treatment is surgical removal of gall bladder(cholecystectomy).Gall bladder calculus are prone to infection and obstruction,so it is wise to get rid of it.Gall bladder calculus usually doesn't respond to medical management.Empirically you can take proton pump inhibitor like pantoprazole 40 mg once daily half hour before food in the morning.I also suggest for ECG in all leads and cardiac evaluation.Get well soon.Take CareDr.Indu Bhushan"
},
{
"id": 122329,
"tgt": "How to treat pain in the shin area due to an injury?",
"src": "Patient: My husband was lifting weights and the foot rest flung back and hit his shin really hard. He bruised automatically, and now it is swollen like there is a baseball stuffed under his skin. He is icing it, but it hurts. He can walk on it. He hit it around 2 hours ago. Should I get him checked out? Doctor: Hello, It may be due to blunt trauma. For pain you can take tablet acetaminophen. You may require X Ray in case of swelling after orthopedician consultation. In case of fracture you may require surgical intervention after consultation. Use support while walking. Restrict your movements Hope I have answered your query. Let me know if I can assist you further. Regards, Dr. Shyam Kale Family and general physician"
},
{
"id": 202362,
"tgt": "How to treat uncircumcised penis?",
"src": "Patient: My boyfriend is uncircumcised. He s very sensitive when I touch the tip of it.I discovered that at the tip of his penis there were two big dark red bumps.They look like they were filled with blood.He said it hurt where the bumps were located.Also the tip of his penis was bleeding.What could this be. Doctor: Good Day and thank you for being with Healthcare Magic! Your boyfriend might have hemangioma in the head of the penis. Treatment can be done on an outpatient setting and cautery of the hemangioma can be done to treat his medical condition. I hope I have succeeded in providing the information you were looking for. Please feel free to write back to me for any further clarifications at: http://www.HealthcareMagic.com/doctors/dr-manuel-c-see-iv/66014 I would gladly help you. Best wishes.Regards, Manuel C. See IV, M.D. DPBU FPUA"
},
{
"id": 47783,
"tgt": "What are the symptoms of kidney damage?",
"src": "Patient: I have been a diabetic for just over 50 years. I have had signs of early kidney damage. Currently, I have had symptoms such as a heat-like, heavyish sensation in the middle of my chest along with difficulty breathing. I also have more of a lack of energy than normal and find it difficult to go up stairs without my heart rate increasing. I have begun checking my BP and over the past 3-4 days, the highest it has been was 143/ forty-something (don t have my list by the comupter). However most of the time the top reading has been under 115 and the lower in the 40-60 range. Could these symptoms be heart-related? My next appointment is not until spring. Doctor: Hi,Thanks for writing in.If your age is above 70 years then an upper limit of systolic blood pressure of 150 mm Hg is acceptable. This is because the blood pressure increases with age and at elderly age group there is a higher normal range of blood pressure.The symptoms of kidney damage includes the following(i) Swelling of limbs and facial puffiness(ii) Foamy urine due to proteinuriaThis is supplemented by doing kidney function tests like serum creatinine which shows a higher than normal level. Ultrasound scan of kidneys shows increased parenchymal echotexture. In urine test routine urinalysis shows proteinuria and 24 hours urine will show protein excretion above normal.Heart conditions also cause heaviness in the chest with difficulty to breathe. Lung infections are to be ruled out. Please do not worry."
},
{
"id": 48901,
"tgt": "Can creatinine levels affect kidney?",
"src": "Patient: I am 47 yrs old.Before 2 months I had high blood pressure that went upto 220 then my dr. suggested me to have a blood test and i found out that my creatinine level was 6.5 and it remained constant after that.Does it affected my kidney...??size of right kidney-8.5 * 3.4 cm......and left kidney measures 9.4 * 4.0 cm.what is the treatment for it? Doctor: Hai,Creatinine level is an indicator for kidney function.high creatinine level(6.5) shows abnormality in kidney. as you got high blood pressure, it will increase glomerular pressure results in loss of functional nephrons (kidney parts). This may lead to elevated blood urea and creatinine.kindly take anti hypertensive drugs with your doctor advise. and check for your creatinine level and get an opinion from a nephrologist for further management.treatment for high creatinine level mainly with food control and water intake.medication comes second.thank youhope i answered your query"
},
{
"id": 209881,
"tgt": "Suggest medication for depression",
"src": "Patient: Hi, my boyfriend is suffering from depression, I am worried about him but he s sleeping at the moment. I talked with him about finding an on line specialist to talk to him about what bothers him and see if he can figured out what s going on and why he has depression. Is it possible for you to email him? He told me three days ago he would do this and he s avoided it since. Doctor: HiThanks for using healthcare magicIn his case, he needs help of a psychiatrist. He need proper mental evaluation, that would help to figure out the reason of depression. He also needs antidepressant that would help to control his negative thoughts. There are number of online specialist, but you can not get medication without prescription. So, better to consult a psychiatrist nearby your location.Thanks"
},
{
"id": 156015,
"tgt": "Suggest treatment for mouth cancer",
"src": "Patient: Dear sir/madum,Patient: GOWRAMMA, AGE:55 YRS OLD, PLACE : TUMKUR,KARNATAKA. PH.NO: 0000 now i am suffering from mouth cancer ,my treatment started from last 4 months ,major surjery done & radio theraphy treatment completed on 14.10.2011. rightnow i can take cow urine theraphy WITH CHEMOTHERAPHY? if so please advice to me regarding doctors available in karnataka. Doctor: Hello and welcome to HCM,The management of mouth cancer consists of surgery and radiotherapy.A close follow up is required after treatment is instituted.You need to stay in touch with your treating doctor to assess for recurrece.A PET scan be done to look for metastasis.If PET scan is within normal limits, there is nothing to worry.Thanks and take careDr Shailja P Wahal"
},
{
"id": 200993,
"tgt": "Can excessive masturbation cause pain in testicle?",
"src": "Patient: I am feeling pain in my testicle and I am afraid it might be because of masturbation. This is the series: In the beginning I used to masturbate at least once or twice a week at sxul for 3 months but in this holiday I masturbated for after every 3 or 2 days in the first month but using the same routine in the second month I am starting to feel some testicular pain from time to time in either my epididymis or testicle I dont know the difference. So I stopped for a whole week and I still felt some pain from time to time.Please help me Doc. Doctor: Thanks for asking in healthcaremagic forumIn short: Give rest to your little friend for some more timeExplanation: Any thing over used within a short period of time can pose problems. So, it applies to your own body also, so I would like to suggest you to give rest to your little friend for some more time(8-10 days) and if pain persists even after that to visit a doctor. If the pain is severe and continuous at present please visit a doctor immediately.Good luck"
},
{
"id": 184173,
"tgt": "What is the treatment for a gap in the upper front teeth?",
"src": "Patient: Hi Sir, This is vasanthkumar from chennai ...I have gap in my upper front teeth..Is any solution to clear that problem sir,I am doing software sales and i cant smile i looks like so bad if i just smile to others means they are irritating me please give me a solution.This is my mail address \" YYYY@YYYY \" please send me the solution sir..This is my prestige problem Doctor: Thanks for your query, I have gone through your query.The gap can be closed with the help of braces or tooth color composite restorations. Consult a oral physician and get yourself examined todecide whether it has to go for orthodontic treatment or restoration. If the gap is minimal then it can be restored with tooth color restoration. If there are other teeth are also affected then it as to be treated with orthodontic treatment.I hope my answer will help you, take care."
},
{
"id": 203207,
"tgt": "Reason for feeling tired, lightheaded and heart hurting after having self pleasures",
"src": "Patient: Hello doctor. My friend self pleasures himself a lot. He used to do it once or twice a day, but now he does it once a week, if at all. His heart has been hurting after he does it, and even the day after it hurts. He said when he first started to do it, everything was fine, nothing hurt. Then, his heart beat would drop after an orgasm then would hurt, then two times ago, his heart was racing for a long time, then it was normal but hurt, then last time his heart was hurting, beating fast, and the rest of the day and the day after, his heart was still faster than usual. He is reaching out to me for help. I m 14 and he is 13. He gets very tired and kind of light headed after he does it, and even hours later. Please help with what it could be. You can ask questions. He is seeing a doctor soon. Thanks AJ Doctor: DearWe understand your concernsI went through your details. I suggest you not to worry much. I can understand the cause behind your friends experience. It is simple tiredness, stress and anxiety. he gets tired after masturbation. Every children does. But worrying about that tiredness makes him anxious and more tired. Anxiety in turn creates fullness of chest and occassional pain.You might need a physician's assistance. Do consult a physician for further details. Visit http://psychocure.webs.com/Hope this answers your query. Available for further clarifications.Good luck."
},
{
"id": 5329,
"tgt": "Getting some discharge in period. Can I be pregnant. Had first baby",
"src": "Patient: im 25 and trying to get pregnant. my husband has just started taking all kinds of medicine to increase his penis size and help build muscle mass and powder to gain. weight and more but I don t want to put it on here.my cycle is has been regular every since we had our first baby. just this month my cycle has been doing really really crazy stuff. so can all of his medication plus unprotected sex 3-4 times a week b messing with my body? Doctor: Hi,Thanks for the query.Usually the size of genital organs won't influence the fertility.The count and quality of the spermatozoa is more important.Stress and strain and medicines can affect the hormonal levels and can alter the menstrual cycle.So you once consult gynecologist and get examined to find out the cause for irregular periods.After regularization of your periods you can track ovulation and plan intercourse around that time that will increase the possibility of pregnancy.For more details you can ask me through: http://www.healthcaremagic.com/doctors/dr-sree-gouri-sr/63429Take care."
},
{
"id": 148986,
"tgt": "Had spinal stenosis. Having groin pain, headache, fever, nausea, blood in stool. Should i go for medical check up?",
"src": "Patient: Yesterday i had groin pain , this morning i woke up with a headache , pretty sure i had a fever and was nauseous but didn t vomit . Also have a bloody stool that was bright red... I am a dissabled vet with trauma to my spine, i have Spinal Stenosis which the VA has treated with 2 epideral steroid injections recently., so far its not working .. my question is should i go see a Dr. right now or do you think it would be ok to wait until the VA Doc can see me.. right now i feel ok. Doctor: Yes you should go for a check up without waiting because immidiate treatment is warrented for the above symptoms.However these symptoms are not related to spinal stenosis. Due to past trauma to spine it seems you had spinal cord injury with residual neurological deficit. This residual neurological deficit does not get cured by any medical treatment however stem cell research has shown encouraging results in such cases"
},
{
"id": 88196,
"tgt": "What causes abdominal pain?",
"src": "Patient: I am having upper right quad abdominal pain which will radiate to my back from time to time. I have had this pain prev and 4 years ago had my gallbladder removed. I have also had a colonoscopy and it was clear (4 years ago). What could be the cause of this type of pain? Doctor: Hello,Welcome to HCMI am Dr Rakesh Sharma answering your query.I can understand your concern.With gall bladder removed your pain seems to be due to acid peptic disorder,If you have no problem in urine,as urine infection and stone also cause pain in that area.Take pantoprazole for about two weeks.Drink plenty of water.Take small frequent bland diet.Hope this suggestion will help you.Good Luck."
},
{
"id": 213114,
"tgt": "Vomiting, stomachache, mental problem, taking rabonic-20. Is there natural therapy or permanent cure?",
"src": "Patient: my doctor has told me that i have mentaly problem (like stress, over thinking) that s why i have occured problem of stomach (like full of stomach, vomiting , stomach pain , burn in stomach) and he give RABONIC-20 MEDICINE (That contain rebaprazole) so, that medicine is good or bad or what are the natural therapy and is there any perminant solution of this disease. Doctor: hi..well, if you have gastric problems out of stress, its psychosomatic disorder..you should get help from a psychiatrist..also counseling may be needful to counter your stress-factor..and also continue to vistit your gastro-enterologist as before because only rabeprazole may not be sufficient for your problems..an USG or endoscopy may be required..have a good health.."
},
{
"id": 177623,
"tgt": "What causes cough and difficulty in breathing in children?",
"src": "Patient: My 3 almost 4 year old has had a slight cough for a week or so. Then a few days ago I noticed him taking a lot of deep breaths. Then yesterday he started doing it every couple seconds. Doc checked him out and said he was fine, hospital checked him out and said he was fine. I am 28 and have had the same problem a few times in my life, around age 25-28. I was allergic to everything growing up and wondr if it could be allergies making him heard to breath? Doctor: Hi...By what you quote I feel what your kid could be having viral associated wheeze or multi triggered wheeze. Unless the doctor diagnoses wheeze, you need not worry till then.Regards - DR. Sumanth"
},
{
"id": 72502,
"tgt": "Suggest therapy for chest pain",
"src": "Patient: my 31 year old son has presented with chest pain for over a month. went to the primary dr. who did an EKG, normal, blood work, cholesteral 187. dr. recommended echo stress. he is very concerned and seems to be experiencing anxiety, feels like he has to take a deep breath all the time. I am also very concerned. Doctor: Thanks for your question on Healthcare Magic.I can understand your concern. Possibility of stress and anxiety related symptoms is more likely. But better to get done 2d echo and stress test first.If both these are normal then no need to worry for heart diseases. So better to consult psychiatrist and get done counselling sessions. Try to identify stressor in his life and start working on its solution. He may need anxiolytic drugs too. Don't worry, he will be alright. Tell him to avoid stress and tension, be relax and calm. Hope I have solved your query. I will be happy to help you further. Wishing good health to your son. Thanks."
},
{
"id": 50593,
"tgt": "Histroy of kidney stones. Sharp pain in groin, nausea, unable to pee, lots of water intake. Urine test showed blood, no infection. Help",
"src": "Patient: On Monday I went to the dr with pain in the left side of my stomach and nauseous for 3 days. My urine test showed a large amount of blood but no infection. I was sent home with anti- nausea medicine suspecting a kidney stone which I have had several. It is Thursday and the urge to pee is overwhelming. Sharp pain in my groin but I am not passing much ironed. I just purchased azo over the counter and the pan is going away. Still not peeing. What is the danger to me if I am not getting rid of the fluid I am taking in as I am drinking lots is water? Doctor: hi.. you have multiple renal stones.. which is causes colic and obstruction at various levels of ureter and urethra.. since there is blood in urine the stone may be excoriating the ureter and causing it.. see your urologist for the management.. small stone7mm you need to undergo a procedure where the stones are broken into small pieces which will pass through urine.. all the very best.."
},
{
"id": 102634,
"tgt": "How to heal itchy rash on neck with no relief from allergic tablets?",
"src": "Patient: Hi there, I have a rash on my neck, that is slowly getting worse. My GP gave me allergy tablets but they don't seem to be working. The rash is very itching and spreading up my neck. It is no where else on my body. But the first instance it was on my face but has cleared. I have put cream on my neck to try and calm the itching but nothing seems to work. Do I need to see my GP again to get antibotics? Or ?? Any suggestions of what it could be. Doctor: Hi, I have read your query & according to me you should get blood test for sugar ,CBC & allergy (absolute eosinophilc count).If you wear anything in neck then stop wearing it, because this may be the cause of allergy.After blood report you should consult with dermatologist."
},
{
"id": 203080,
"tgt": "Could white colored, egg like substance under top of penis be due to infection?",
"src": "Patient: i am 22 years old male..today i noticed tiny white egg like stuff under the top of my penis..it does not itches there is no rash but i am just worried..is that tiny egg like stuff part of my penis or is it some kind of infection i don't know about ..please help me Doctor: Hi,It seems that you got deposition of smegma on your glans and nothing to worry.Make a habit of cleaning smegma by averting foreskin while taking bath.If you do not clean then deposition of smegma will continue and later on you may get adhere foreskin leading to phymosis.Ok and take care."
},
{
"id": 67600,
"tgt": "What causes lump near jawline and ear lobe along with numb face?",
"src": "Patient: I feel a lump by my jawline and ear lobe, parts of my face feel numb sometimes. You can't really feel it until I turn my head to the opposite side. It doesn't seem to bother me and I am not sure how long it's been there. I noticed it a couple of weeks ago. Although about 1-2 months ago I woke up with extreme neck pain like I had a pinched nerve. I have a feeling it's related to the lump. Doctor: Welcome to health care magic. 1.The most possible cause could be submandibular cystic dilatation - sialadenitis.2.Other possible causes could be submandibular gland lesions, bony cystic lesions.3.Finally lymph nodal enlargement. To conclude the diagnosis - primarily ultrasound examination will help to evaluate. 4.Ultrasound will help to evaluate the nature of the lesion, its source and extensions.5.So i would suggest to request for a ultrasound scan and further depends on the scan result.Good luck.Hope i have answered your query,any thing to ask ? do not hesitate to ask.http://doctor.healthcaremagic.com/doctors/dr-ganesh/62888"
},
{
"id": 29418,
"tgt": "Are itchy lesions on the knee of a child due to skin irritation?",
"src": "Patient: My 7 year old has random lesions on her knee, back of knee, inside of left arm near elbow (one above and one below elbow), one on her butt cheek, and one below her lip. they are between a dime size and a quarter size. We keep them covered, with calamine lotion. They don t seem to be getting worse, but not necessarily getting better. At what point should I take her to the doctor? Note: no other symptoms. Just itchy. Eating and sleeping fine. No fever. Also, I have two concerns. 1. She boogered up her knee at camp three weeks ago, and then played in the lake with the open wound. The lesions look similar tot the wound on her knee (which is healing slowly). 2. A week ago Saturday she played all day in a friends wooded back yard, and came home with a tummy rash that looked like a grass irritation. Benedryl, and it went away. Not sure if any of this is related. Doctor: Hi,She probably has atopic dermatitis (eczema). It's a chronic skin condition caused by an allergic reaction to variable triggers such as climate, food, physical, chemical, emotional factors. Main sign is itching leading to thickened leathery skin (lichenification). Lesion are dry and scaly, affects mostly flexural areas (back of the knee, etc) in children but any part of the body may get involved. Patients with atopic dermatitis usually have other types of allergic conditions. Her tummy grass irritation was possibly contact dermatitis which responded to Benadril (antihistamin). Atopic dermatitis is treated with skin moisturizers (dry skin worsens the condition). Patients should avoid frequent bathing, using washcloths or brush during bathing. Also, should avoid known allergens. Cool compress and antihistamins used to control itchiness. I is advised to wear loose fitting cotton clothes. In severe disease topical or systemic corticosteroids may be prescribed. Patients with atopic dermatitis are increased risk of secondary skin infections. Signs of secondary skin infections are swelling, redness, pain or fever. In such a cases patients should see a doctor. If you have further questions I am happy to help.Thank you."
},
{
"id": 208627,
"tgt": "Suggest treatment for hallucinations",
"src": "Patient: My sister is only 30 years old & married.She got married at the age of 23 years ,when she delivered a baby at the age of 25,she was very serious & from sice then her remembrance power also become weak .sice last 05 years ,her doctor advised her to take medicine oxetol-300 & Respidon-1 & she is also taking these medicines but from last 02 months she always says that some bhoot (ghost) is around her and torchering her. please suggest what to do? Doctor: hii dearsometimes this hallucinations suggest underlying psychopathologyif she has another symptoms like delusion, disorganized speech and behaviorthan it suffice diagnosis of schizophreniaonly 1 mg resperidon cant help1st step is detail evaluation is necessary to stemp diagnosis2nd increase the dose of resperidone up to 6 mg along with trihexyphenydylbut consult your doctor and take proper advisethank you"
},
{
"id": 130946,
"tgt": "What is the treatment for leg pain?",
"src": "Patient: I was in a car wreck a month ago I hit my right below my knee really hard on the dash of the car. So hard in fact I broke the dash. My leg has been swollen and bruised ever since, it feels really hard, but tender to the touch. I can walk fine, but I have noticed that I may be breathing shallower now, not sure if they are related. I have not went to see a doctor yet because I didn't think they would be able to do anything for me, but I am tired of my leg hurting. Doctor: In my opinion its nothing seriousi recommend alphinturn 300 three times a day for 1 week , if you still have pain then you should do an MRI to check your kneeGood Luck"
},
{
"id": 7858,
"tgt": "How can i cure my child's acne on his cheeks ?",
"src": "Patient: This query is for my son. he is 1hr 5 months old, there are pimple like scar on his cheeks ,i am applying Acme ointment/soap and Lotion but not helpful Can you please recommend treatment for this Doctor: hi welcome to healthcare magic forum . acne in child is due to hormonal effect from the mother you need not to worry as they will subside . tell me the content of cream ,if it is clindamycin ,continue it . dont pick ,popped it . better to consult dermatologist if he has infection than he need short course of antibiotic. i hope i have answered your question."
},
{
"id": 108740,
"tgt": "What causes pain and soreness in lower back?",
"src": "Patient: Three months ago I fell while walking down stairs and landed on my back. No major pain at the time, just soreness. Now that area of my back is sore to the touch and sometimes tight, painful in the morning. It s the lower back, on the spine. I have no trouble with movement or lifting. Should I have an x ray? What are possible explanations? Doctor: Welcome to HealthCare Magic. Thank you for posting your question.I can understand your concern.Low back pain can be due to a lot of reasons :1.muscle strain2.muscle spasm3.osteoarthritis4.Disc prolapse5.HLA-B27 associated arthritis including ankylosing spondylitis, reactive arthritis, psoriatic arthritis, and inflammatory bowel disease6.spondylolisthesisThat's all about the information. Do not worry too much.I would suggest you to take combination drug containing Tablet Flupirtine 325 mg and Paracetamol 325 mg once daily X 14 days after foodand apply dry heat (with a warm cloth) over the lower back twice daily.I would also suggest you to visit an orthopaedician. He can guide you on the further managements after proper examination.Hope this helps.Post any other question. We will be happy to help.Have a nice day. Regards."
},
{
"id": 208613,
"tgt": "What are the symptoms of schizophrenia?",
"src": "Patient: someone who i know has major meltdowns of crying and sadness, explosions of anger, extremely unrealistic, and thinks everyone is against her, she has major mood swings, and thinks extremely unrealisticly, and is suicidal at times, and she says strange things, and has lost interest in almost everything, she is 14 years old, and has had these symptoms for many years, has been to mental hospitals and has been on many different medications, she is currently on risperdol, she doesnt have any visual or auditory hallucinations, is there a possibilty that she has schizophrenia? Doctor: hi dear,what ever you described about her symptoms is mostly suggestive of schizophrenia but with out mental status examination it is not possible to stamp schizophrenia.for proper diagnosis detail history from patients and her relative and mental status examination of patient is necessary.and for that consult local psychiatrist.symptoms of schizophrenia is paranoid ideas, referential ideas, infidelity ideas , irrelevant speech and behavior, negative symptoms etc.2 symptoms must be present for 6 month to diagnose schizophrenia Thank you"
},
{
"id": 28681,
"tgt": "What causes chronic vaginal infection?",
"src": "Patient: Hi , February 13 i got the nexplanon Implant. I found out I had chlamydia and I was treated for it March 1st along with it I had a yellow discharge and itching. I was also treated for A yeast infection and a UTI , but i still have Heavy Yellow discharge. and itching occurs rarely only if I sit wrong. What other possibilities could it be ? Doctor: Hello and Welcome to \u2018Ask A Doctor\u2019 service. I have reviewed your query and here is my advice.The yellow discharge you are saying is commonly caused by chlamydia, or yeast infection or bacterial vaginosis.A vaginal secretion culture can be done.chlamydia is well treated with doxycycline and bacterial vaginosis with metronidazole. While yeast with antifungal drugs such clotrimazole.Hope I have answered your query. Let me know if I can assist you further."
},
{
"id": 101982,
"tgt": "What causes swelling of throat on chewing Orbits Strawberry gum?",
"src": "Patient: I chewed a piece of Orbits Strawberry gum. My throat swelled and it became hard to breath. i also got a headache. All this happened within 5 minutes. I took a benedryl. Now, the symtoms are relaxing. What caused the problem? I dont have any food allergies. Doctor: Hi, You maybe allergic to any one of the content in orbit chewing gum. The symptoms appeared just after taking this and relieved after benadryl suggest allergy.you should never take the same content food again as the chance of reactions is more and with more severity. Hope this helps you. Regards"
},
{
"id": 53743,
"tgt": "What does weight gain in person with liver problems indicate?",
"src": "Patient: my boyfriend of 10 years has alcahol cirrohis, he was yellow, bleeding from everywhere, severe drinking for alot of years (a gallon or more a day, for the last few years) swollen everything, urine was bloody, pnumonia, etc. he was hospitalized 2 years ago and was not supposed to live. He stoped drinking, (has had a few slip ups) but the doctor said he would eventually die that his liver is pickled, im not sure what score she was talking about but she said it should be in the 100 s and his was only an 8. He is starting to look really good? he has sick spells off and on during the day (maby 5-10 a day, where he doubles over in pain and nausea) and still takes meds, he has ghal stones but they said they will not remove them because of his liver he could die during operation. He is now starting to gain alot of weight a bit fast. I wonder if his end stage liver is healing or is this a calm before a storm? Doctor: Hello,1) Unfortunately, present condition of your boyfriend is irreversible2) The main reason for weight gain in end-stage liver disease is accumulation of water in body (mainly abdomen and sometimes chest)3) The score you mentioned is CTP score for end-stage liver disease; less the score more chance for survival (Maximum score is 15).4) With score of 8, chance for 2-year survival is 55-60%"
},
{
"id": 96670,
"tgt": "Does swelling and bruise around a cut need medical attention?",
"src": "Patient: My wife drop a broken plate on her arm a few inches from her wrist. There is a very small cut about 1/4\" or less. There was swelling but we put ice on it and got the swelling down. She has use of her arm and hand with pain. There is some bruising around where the plate hit and a little wider around the effected area. Do you see any reason to go to the ER? Doctor: HelloThank You for contacting HCM.Bruising occurred because blood has accumulated at the site of injury. Its color changes with time.You need not to worry. It will resolve itself in few days. i would suggest her following things:> Take rest> Don't touch the wound with dirty hands> Apply polyfax plus ointment thrice a day> Wash hands before and after applying.> take acetaminophen for painHope this answers your question. If you have additional questions or follow up questions then please do not hesitate in writing to us. Wishing her good health."
},
{
"id": 125415,
"tgt": "What could cause feet and hand swelling?",
"src": "Patient: my feet swell and change color, my hands swell and change color, i get dizzy spells, was told i was anemic and now i had just found a lump under my right color bone saw it in the mirror and felt it and it doesnt move and now i am scared what could this be Doctor: Hello, Consult a physician and get evaluated. We have to rule out possible causes like cervical rib as it can cause similar symptoms. Hope I have answered your query. Let me know if I can assist you further. Regards, Dr. Shinas Hussain, General & Family Physician"
},
{
"id": 154203,
"tgt": "Suggest treatment for tumors",
"src": "Patient: my husband had throat cancer in 2004, hes had 2 mri an cat scan an bone scan, now the suregon saying cancer is bk, that he has 2 tumors growing in the same place where his cancer was before, an told him no operation no chemo no radation,, only thing they can do is give him a ultrasound an bipsy 2 see if its fast or slow growing,,,, Doctor: Hi, dearI have gone through your question. I can understand your concern.He has throat cancer. Treatment depends on type, stage and spread of tumour. So first of all he should go for biopsy to know exact type and grade of tumour. Then he should take treatment accordingly. In late stage surgery will not help. Palliative radiotherapy or chemotherapy can be given depending on spread. Consult your doctor and plan accordingly.Hope I have answered your question, if you have any doubts then contact me at bit.ly/Drsanghvihardik, I will be happy to answer you.Thanks for using health care magic.Wish you a very good health."
},
{
"id": 189803,
"tgt": "Have swollen area in jaw. Diagnosed as pocket of air. Had tooth extracted. Normal?",
"src": "Patient: My dentist recently did a wisdom tooth extraction of lower impacted molars. Sutures are good, no possibility of dry socket. He saw me today at the golf course (he is one of my members) and my left lower jaw is still swollen. Feels like a tight area of tissue in my lower jaw/cheek, about golf ball sized. He says that it is a pocket of air, that I can push out and it will make a fart noise through the suture area. So I ve made it home, and I ve attempted to push from outside, pinch from both sides, and flat palm push it, and nothing is coming out. No movement. It feels like a solid ball of something, not air. 1. Is this common to get? Air getting into the area beneath the gum in a pocket after wisdom teeth extraction? 2. Is it really something I can push out? Why, if so, is it not escaping from my suture area? What can I do to help it get out? 3. Should I be concerned? *note* Dentist said he would see me on Monday to put a needle in and extract the air himself if I was unable to get it out. Doctor: hi and welcome , Entrapment of air in the extraction socket is a rare condition , because after extracton of wisdom tooth the socket is curretted to remove all the irritants and compressed which helps in clot formation and faster healing. Though we can expect air embolus to be formed in the jaw bone which leads to pain or as you have mentioned the swelling feels hard it may be a bony spicule or remnant of alveolar bone left unfiled. i would suggest you to get a X RAY done for the affected area which will show the cause of swelling accordingly your dentist can plan the treatment for you . i hope this information helps , take care."
},
{
"id": 218812,
"tgt": "Is pregnancy a possibility while on Regestrone medication?",
"src": "Patient: Hi...my periods was due on 31st Jul...I took regestrone from 21st Jul to 26th Jul...I have not got my periods yet....I had unprotected sex with my husband on 18th 19th & 21st Jul...is there a chance for me to be pregnant and since I had taken regestrone...will it affect my baby...plz suggest Doctor: Hello user.first do urine pregnancy test and confirm.if double line positive.Registrone is unsafe during pregnancy.thanks."
},
{
"id": 11129,
"tgt": "Suffering from Trichotillomania. Name few good supplements for regrowth of hair?",
"src": "Patient: Hi Doctor, i have been suferring from Trichotillomania for more than 9-10 yrs. now it has been reduced 80%. I would like to regrow my hair by using Zevit Tablets. btw i have come acroos many products in internet n got confued to choose right one. please doctor suggest me good supplements.by Shal Age-29. from Bangalore. India. my hairs becoming very thin , dry and Split ends. Its very tough to grow back my hairs. Request you to suggest good supplements to regrow my hairs. Doctor: Save your money! The primary problem is the physical pulling of the hair. The hair will regrow on its own and no supplements will speed that process up considerably."
},
{
"id": 223349,
"tgt": "Are sore breasts a symptom of mercilon?",
"src": "Patient: i have come off my pill it has been about 2 weeks now. we are trying for our second baby and im experiencing itchy sore breasts. Is this a normal symptom of coming off mercilon? I dont remember it with my first child? I caught in the first month with my son and sort of hoping that i may have caught after 2 weeks of coming off the pill. Is this possible? im not due on until 28th sept Doctor: hello user,side effect of mercilon is breast tenderness,but u need not worry,it will go with time.if you keep relationship around your ovulation..there are high chances to get pregnant."
},
{
"id": 104085,
"tgt": "Lumps near the ears. Taking steroid for asthma. Medication related?",
"src": "Patient: i have lumps by both my ears right above my jaw line. I take asthma medicine with steroid each day. My pharmacist said it is probably due to the steroid and there is a name for it but I can,t remember it. Once I stop taking the asthma medicine during a 2 year period it went away, but to be able to have quality of life I have to take every day (Its happened with Advair and Symbycort) Just using the nebulizer does not help enough with my breathing, is there another daily medicine without steroids you could recommend so I can speak to my primary care doctor about? Doctor: Hi ginnm529, Thanks for visiting HCM. Let me tell you that steroids are the backbone of asthma medication. No bronchodilator can be prescribed without steroid combination. So forget about not taking steroid. Now regarding your second query of lump behind ears....These seem to be sebaceous cyst. You must have noticed some cheesy material coming out when they burst. I will advise you to meet a general surgeon and get both these lumps/ cysts excised. Its a simple outdoor procedure done under local anaesthesia. If you leave them as it is, they will frequently recur and many a times will get infected too, requiring antibiotic treatment. So continue your asthma medication and meet a general surgeon. Best wishes"
},
{
"id": 36129,
"tgt": "What causes nausea, vomiting and rashes on chest?",
"src": "Patient: I have a rash down my chest, middle, like an upside-down T, with a straight line between my breasts, and a horizontal line under them. This is the 2nd time I have had this in the last 6 months, and both times I experienced nausea / vomiting and thought I had stomach virus. Doctor: RASH MAY BE DUE TO FUNGAL (YEAST) INFECTION.YOU CAN APPLY MICONAZOLE CREAM TWICE A DAY.TAKE ANTACIDS FOR NAUSEA & VOMITING.AVOID JUNK FOOD.CONSULT YOUR DOCTOR FORPHYSICAL EXAMINATION .HE WILL PRESCRIBE ORAL MEDICINES FOR NAUSEA ,VOMITING,RASH."
},
{
"id": 47266,
"tgt": "What does minimal PC dilation in ultrasound suggest?",
"src": "Patient: hi Dr.i am 30 years old man.i had stones in my kidney from the last 2 years but i take the treatment for it and i don't now if its go outside of or not.yesterday i was so tired and i have make an ultrasound which shows minimal left sided PC dilatation what does this mean?and what should i do regarding this? Doctor: HelloYour findings suggests minimal left PC dilatation,it means there is minimal back pressure changes in left kidney may be due to calculus in kidney etc.Your report doesn't mention about calculus etc. Your condition need to evaluate about underlying cause.You need proper evaluation by a urologist,Treatment of renal calculus depend upon many things like composition,position,size of calculus etc.You may need CT scan of KUB region or IVP(intravenous pyelography)to investigate the underlying cause.Treatment depend upon findings.Get well soon.Take CareDr.Indu Bhushan"
},
{
"id": 51275,
"tgt": "Renal stones, have severe stomach pain on the left side. Is it safe to conceive?",
"src": "Patient: Hello doctor ! I m Priya from chennai age 26 married i m stone problem for the past 2 years at first i have 5 stones so i under went doctor advice and they treated with kmac syrup and cystone table for 2 months then later i get cured . Before 3 months i scanned in that they told me now in that 5 stones 3 were gone away and now 2 were still present in right side of the kidney .But from yesterday onwards i feel severe stomach pain in the left side of my stomach.i dnt know wat to do . pls give me some suggestion .& i have no child . whether i can undergo for the sexual life or not Doctor: Thanks for your query You have 5 small renal stones detected 2 years back for which you have taken tab Cystone and syrup.But I would like to state that there is no medicine on earth that can dissolve the stones Small stone normally pass down through urethra with urine.Now you have two more stones in the kidney. and you are getting pain in abdomen this is an indication that the stone has been impacted in a passage of urine mostly in Ureter.So pl get your Ultra sound scanning done to locate the position of the stone and the degree of obstruction it has caused on the kidney.Depending on the finding you can opt for modality of treatment which will be decided by Urologist.After this gets cured you can think of pregnancy."
},
{
"id": 31910,
"tgt": "Why does one develop mottling while recovering from chicken pox?",
"src": "Patient: Hi. My wife (51) is recovering from chicken pox (pox are starting to scab over). We just noticed mottling of skin over both knees and radiating outward. There's also some mottling on the hands. A doctor prescribed a mild penicillin to help with secondary infection in some blisters. Is this something we should be concerned about? Doctor: Hi Dear,Welcome to HCM.Understanding your concern. As per your query you develop mottling while recovering from chicken pox which is due to compromised immunity and viral infection of body. Recovery depends on severity of lesions and intensity as well . It will take around one week for chicken pox to recover if you are under observation. You should not itch lesions or vesicles at all. Drink plenty of water and take diet rich in multivitamins. Take acetaminophen or ibuprofen if you are having fever. If any itching will be there apply Calamine lotion as that act as soothing agent. Chicken pox don't leave any scars unless you will scratch or itch it badly. Visit dermatologist once if symptoms keeps on persisting and start treatment after proper prescription.Hope your concern has been resolved.Get Well Soon.Best Wishes,Dr. Harry Maheshwari"
},
{
"id": 104269,
"tgt": "Infant with breathing problem. Can i ask doctor online?",
"src": "Patient: Baby born 4 days ago, full term, 9lb 2 oz birth wt. No problems during pregnancy. Problem breathing . Traced to inhalation of fluid from birthing pool. In intensive care for 3 days. Fluids via nasal feed & antibiotics via cannula . O2 levels reduced. Moved from i/c today still experiencing breathing problems. Nasal feed removed. Cannula still in situ. Prognosis? Doctor: Hi welcome to Health care magic forum. Thanks for choosing H.C.M.Forum. Your baby of 4 days had inhalation of fluid from birthing pool or may be inhalation of amniotic fluid. baby is in I.C.U. for three days , now shifted to from I.C.U. it means that the baby is out of danger, and O2 levels must have increased. Residual dyspnea will be there for few days more , due to residual fluids at the base of the lungs. With effecient treatment baby will be fine in few days. If you feel necessary you can ask the doctor to consult a neonatalogist for acute care. As you said baby will be fine. Wishing the baby a quick and complete recovery. Best regards."
},
{
"id": 103797,
"tgt": "Allergic reaction to vagisil medicated wipes. Prescribed with dipropionate and clotrimazole for yeast infection. Chances of allergy?",
"src": "Patient: Hello,I had a terrible reaction to vagisil medicated wipes last week. After being diagnosed with a yeast infection today, I have been prescribed clotrimazole and betamethasone Dipropiaonte Cream and Terconzxole Cream. Do either of these have ingredients similar to those in vagisil? I certainly don't want to make a bad situation - worse. Doctor: NO THESE ARE USECD TO TREAT ALLERGIESAND THESE DONOT CAUSE ALLERGYOU ARE ALLERGIC TO YEAS SO CAN HAVE WITH YEAST IN FOOD TO GET CURE ELIMINATE YEAST DIET COMPLETELY FROM YOUR FOOD AND OTHER APPLICATIO MATERIALS YOU CAN START ANTIALLERGIC ALLEGRA 120 MG OD FOR SOME WEEK ALOMG WITH OTHER MEDICINES YOU ARE PRESCRIBED CAN ADD ANTIFUNGAL FORCAN 150 MG ONCE A WK FOR 4 WK"
},
{
"id": 190049,
"tgt": "Using toothpaste for gum problems, bad taste in lips. No improvement. How to correct it?",
"src": "Patient: hello..hope that all of you will help me,,,im 22 years old,,male,,my concern is that my lips has a bad taste,,,how can it be cured,,,and i think i have gum problems caused it taste also bad,,,,im using toothpaste for gum problems but my gums is not improving...what should i do,,,,i dont know if this gum problem is caused by sinusitis ....please help me..gramercy Doctor: Hi Welcome to HealthcareMagic forum Yes, acute sinusitis can give rise to all the problems mentioned by you. You need to get a CT scan of the paranasal sinuses to look for any complications secondary to sinusitis which is causing lip and gum problems. Meanwhile, stop the paste you are using for the gum problems and drink plenty of warm water. I wish you good health Regards"
},
{
"id": 93046,
"tgt": "Suffering from abdominal and chest pain. Endoscopy shows erosive gastritis, normal stress tests. Advice?",
"src": "Patient: Hi, I had abdominal and chest pain. Had an abdominal cat scan with contrast and did not show erosive gastritis. I had an endiscope and I have been diagnosed with erosive gastritis. Waiting for the biopsy report. Very worried. Why didn't the cat scan show erosive gastritis and what is associated with this diagnoses. I was also admitted into the hospital to rule out a heart attack. Had stress test, echocardiagram and carotid artery test. All tests were normal. Doctor: Hi ! It is good that all tests related to your heart were normal. Erosive gastritis m gives rise to esophageal spasm which may mimic a cardiac problem.CT scan is not the ideal investigation to show erosive gastritis. It can be diagnosed with the help of your history, followed by the endoscopy. You can go for a proton pump inhibitor, a prokinetic agent, and an antacid after consulting your doctor, and at the same time you should go more for bland diet, fresh vegetable and fruits, and avoid fast fried food from outside. Biopsy is a routine procedure whenever an endoscopy is done to exclude many other pathological processes associated with chronic esophasitis. So you do not have to worry about it till the report is ready. Wishing you an early recovery."
},
{
"id": 167638,
"tgt": "Suggest treatment for poor appetite in children",
"src": "Patient: my son is two yr old nnow m pregnent,he only take mother feed n some other food by choice n only 10 kg in weight he z v week his appetite z poor tell how to increas his weight n convince for bottle feed nt mother feed n he dont speak clear words except mama papa just Doctor: Hi...Thank you for consulting in Health Care magic.I take this opportunity to tell you certain scientific facts and relieve your distress -1. An active is a well kid even if she/ he is not eating well.2. Development of a kid is as important as or I would say even more important than growth alone.3. As the kids grow their interest in food decreases and interest in play activities increases so that they eat small quantity and run away to play. As parents we need to be more patient and feed than less quantity but more aliquots per day.4. This age rather than the quantity of the food I suggest you concentrate more on the number of times you feed her and also make whatever you feed her calorie dense by adding good amount of ghee to it.5. I suggest you not to use appetite stimulants on long run as they may cause drowsiness.Regards - Dr. Sumanth"
},
{
"id": 138027,
"tgt": "Is there any relation between rashes and joint pain?",
"src": "Patient: About one week back, suddenly i got small rashes in my face and body. The day before my wife got the same. I thought, it was an allergic problem, and so i took some medicine on allergy. However, after two days, i started getting pain in my hand joints, finger joints. On the very next morning, I found that my joints were swelled up and started pain in all the joints in my body. Few months back, i consulted with a orthopedist because of hip joint pain, which I have been suffering for more than 2 years, and he gave me some medical tests. However, I couldn t manage time to make those tests. Now I made those tests and found that my ESR is 14. Now my question is why suddenly these happened? Is there any relation between rashes and joint pain? Why ESR level rises? Thanks a lot. Doctor: Hello,Welcome, and thanks for sharing your concern I went through your query, and I feel, your symptoms are of reactive arthritis, following any viral infection, this can occur even months after the initial viral fever, so see 1 if you have a history of any such febrile episode in the past, 2 Do you have arthalgia, which simply means tender painfull joints3 Any history of inflammatory arthritis in the family members, like RA.check for these and revert back, we could help you reach the diagnosis, if we can have answers to these questions. I hope my advice would have been useful, in decision making regarding your treatment, still if you have any clarifications or doubts feel free to contact back.Thanks."
},
{
"id": 201777,
"tgt": "How to treat itch on penis?",
"src": "Patient: i have a silly question yesterday i was out side riding an atv in tall grass later the day i came home approx 5 hours later i find a tick on my penile foreskin i removed it an now the skin has a small bump an red an itches really bad what is wrong ive never had a tick itch Doctor: Thanks for contacting HCMI am sorry to hear that you found a tick and the area that you found the tick is now swelling and red. When you removed the tick it is possible the tick released some of its body juices into your skin. I would recommend using a small dose of hydrocortisone. If the area becomes more inflamed I recommend you seek medical attention. Hope I answered your question. Please feel free to contact us again for you medical concerns and questions"
},
{
"id": 132129,
"tgt": "Suggest remedy for pain and tightness in neck",
"src": "Patient: Hi, I have an appointment with a doctor in an hours time but my question to you is, I have been having a nasty dull pain in my back that lasts about 10 minutes I also have hot and cold sweats and light headed to were I feel I am going to faint, yesterday, I also experienced pain and tightness in my neck which lasted for around 10 minutes also I feel tired when I am normally active I weigh 50 kilos but have noticed in the last 4 days my stomach bloating. I experienced joint pains in my left elbow and wrist today Could these be signs of trouble with the heart? Doctor: hi, you have back pain lasting about 10 minutes , feeling of tightness in neck and dizziness with pain in (L) elbow and wrist. With this clinical picture I would suggest that you undergo a heart check up first and if everything is OK with heart then I would investigate for cervical spine pathology."
},
{
"id": 113184,
"tgt": "Twinging pain on left side of lower back, difficulty walking after standing for long. Due to fall from stairs?",
"src": "Patient: Hi, Ive been having pain on my left-side, lower back, like a burning in my left buttock, and lots of twinges . Could I just now be feelimg the effects from falling down my basement stairs. That happened over 3 years ago? After standing on my feet , working 8 hrs, sometimes i can barely walk upright. The pain feels internal. Thanks for your input Doctor: Hello Jody,Your symptoms indicate Sciatica where the person experiences tingling, & shooting pain radiating from back of thigh to back of leg, burning sensation & lower back pain (lumbago). The symptoms get aggravated during walking & bending forward. CT & MRI scan will help to confirm the diagnosis . Jody, though possible, the pain you are experiencing now may not be directly due to the injury you experienced 3 years back. Most common cause of sciatica is slipped disc. Bed rest may help but will not cure the condition. Therapeutic benefit of drugs like Aceclofenac & Cortisone to relieve the pain & inflamation are limited because of their side effects as they need to be used for a prolonged period. Stretching exercises to relax the muscle spasms are very effective."
},
{
"id": 60304,
"tgt": "What can be the reason for pain in the right lower ribs intermittently with SGPT 79U/L and SGOT 53U/L ?",
"src": "Patient: I am a 45 yr old womanwith a height of 5 6 and a weight of 170 lbs My liver analysis shows bilirubin total 0.3mg/dl, SGPT 79U/L and SGOT 53U/L with non- reactive HepatitisB antigen. My cholesterol levels are 140mg/dl and triglycerides 98mg/dl. I am non alcoholic, vegetarian and non diabetic. I experience pain in the right lower ribs intermittantly. There is no family history of any type of cancer . I do not have any other gastro intestinal problems. Can you please suggest the possible indication? Doctor: Hi, Welcome to HealthcareMagic Forum. May be you have a Mild Hepatitis, Kindly get yourself examined by a Gastroenterologist to rule out any pathology and if necessary get a Ultrasound scan done. You are a bit overweight, you need to reduce it by regular exercises and avoiding high calorie foods like junk. Your cholesterol is on the higher side and you will need statins for reducing your Cholesterol. Work out daily like cardio, crunches, swimming and skipping for an hour daily. Wish you Good Health and Take Care."
},
{
"id": 48864,
"tgt": "What is the treatment for Right kidney stenosis and left kidney calculi?",
"src": "Patient: I'm from Bangladesh planning to visit CMC for treatment of my impaired kidneys. Right kidney has stenosis and smaller in size functioning 30% and the left kidney has calculi but functioning 80%. Creatinine is 1.7mg/dl. Kindly Suggest treatment and appointment.-Nijbjnjdin. Doctor: HI THANKS FOR POSTING YOUR QUERY ON HEALTH CARE MAGIC.KIDNEY STENOSIS CAN BE TREATED WITH RENAL ANGIOPLASTY AND STENTING.RENAL CALCULI CAN BE TREATED WITH ESWL;EXTRA CORPOREAL SHOCK WAVE LITHOTRIPSY AND PCNL-PER CUTANEOUS NEPHROLITHOTOMY.BECAUSE OF LOWER FUNCTIONING OF YOUR RT KIDNEY,YOUR CREATININE IS SLIGHTLY HIGH.YOU CAN TAKE AN ONLINE APPOINTMENT AT CMC.CONSULT YOUR DOCTOR FOR FURTHER QUERIES.THANK YOUTAKE CARE"
},
{
"id": 220682,
"tgt": "Suggest natural methods to terminate pregnancy",
"src": "Patient: i am student and i have not had my period for the past 2months ..i don t want to be pregnant.i am just 18 and just started my degree. please i won t be able to afford expensive medications,if any natural ways are there please help ...if there isn t any way i ll commit suicide Doctor: Hello dear,I understand your concern.In my opinion a urine pregnancy test need to be done to confirm the pregnancy.Then depending on the age of pregnancy the termination is planned.Early pregnancy upto 6-7 Weeks can be terminated by medical methods.After 7 Weeks surgical methods are sucessful.Iam sorry but there are no natural methods available for pregnancy termination.First of all I request you to avoid stress and don't get depressed.Mistakes happen but we will need to correct them.Don't think of ending life for this small matter which is correctable and reversible.I suggest you to consult doctor without delay and follow thier instructions.Nothing to worry.Do not try the abortion medicines on own as they are associated with complications like heavy bleeding,incomplete abortion etc.So relax and consult doctor.Hope this helps.Best regards..."
},
{
"id": 198327,
"tgt": "What are the symptoms of hydrocele?",
"src": "Patient: My semen analysis says bacteria detected and has red cell what does it mean?Also my right scrotum is swollen but know pain only I had discomfort , I am told it is hydrocele by putting the torch below the scrotum in dark i can clearly see my scrotum transparent, so its hydrocele or some thing else? Doctor: helloThanks for query .Based on the facts and positive light illumination test of scrotum the swelling of the Scrotum that you have is mostly due to Hydrocele ..However it needs to be confirmed by a clinical examination by a qualified General Surgeon or Urologist and Ultrasound Scanning of Scrotum .If confirmed to be hydrocele ,you will need to get it operated .Dr.Patil."
},
{
"id": 61411,
"tgt": "How can a lump on the wrist be treated?",
"src": "Patient: Fell down and noticed lump on wrist. What could it be?Hi, my 7 year old daughter fell yesterday and now there is a lump on her wrist. She can move her hand just the same, it is not swollen and she says it does not really hurt. What could that be? Thank ...Ask a Doctor Now \u00bb Doctor: Respected user , hiThanks for using Healthcaremagic.comI have evaluated your query thoroughly .* Seems post traumatic hematoma most likely .* Needs clinical assessment or minimum evaluation of high resolution photograph of it for better guidelines .* Present suggestions - keep it clean with antiseptic liquids - ice application 3-4 times a day - keep that hand in arm rest support- in our clinic we recommend penicillin group of antibiotics for better resolution of the lump & prevent infection - analgesic syrup if more pain Hope this clears your query .Welcome for further assistance .Regards ."
},
{
"id": 197895,
"tgt": "Experiencing Varicocele problem in the left testes",
"src": "Patient: i have vacocelene problem in my left tastes , so my quaere is, that i must to surgery for that or not because i m still single, and after surgery there will be any problem regarding sexual intercourse or not, and will any problem of children birth ? thanks Doctor: HelloThanks for query .You have been diagnosed to have Varicocele on left side and have doubts related to Varicocele .The answers to your query are detailed as below point to point 1) Surgery is not mandatory in each and every case of Varicocele Surgery is generally advised in case one has severe pain confirmed to be due to Varicocele .2) Infertility is not primarily due to Varicocele the patient of infertility may have Varicocele but it is not still proved that Varicocele can cause infertility .3) There will not be any problem in having good enjoyable sex after surgery for Varicocele .4) As stated earlier Varicocele can not be cause for infertility .Dr.Patil."
},
{
"id": 142783,
"tgt": "What causes cold and numb extremities?",
"src": "Patient: Is it normal to have cold/numb feeling hands and feet when experiencing bouts of PVCs? I get pvcs frequently and my cardiologist has said they are benign. But I'm laying in bed and the tops of my hands and feet have an odd, almost numb sensation, but they are not totally numb because they still have feeling. Doctor: Hi.I am Dr Mittal.I have read your message.I think I can help you.Having read your symptoms, there are 2 possibilities that I would consider first.One, PVC, or premature ventricular complexes basically cause a reduced blood supply if there are many of them together. It is caled trains of PVCs. If these trains of PVCs are present, then sometimes you may have these symptoms as the blood circulation in the body is affected and the body attempts to supply the blood to the vitals organs first.A second possibility is that you may be developing peripheral neuropathy. There are many causes of this disorder. In this situation, the loss of sensory symptoms in hands and feet, usually symmetrical (classically known as glove and stocking pattern of sensory loss) is highly suggestive of neuropathy. You will need to test a Nerve Conduction Study and meet a neurologist for the same. If it does turn out to be neuropathy, the addition of medicines like pregabalin or gabapentin may help you.I have tried to make it as simple as possible.Best of luck, Dr Mittal"
},
{
"id": 45847,
"tgt": "Suggest treatment for kidney disease",
"src": "Patient: sir my father is ill from this kidney disease . he can not want to exchange the kidney . he face this problem 3 years . he try all the medical line but no response we can found .he has 4.8 creatnine and urea 51 please sir give me a well suggestion . Doctor: Hello and Welcome to \u2018Ask A Doctor\u2019 service. I have reviewed your query and here is my advice. If the creatinine value can not be brought down with medical treatment like diuretics, he might require dialysis once or twice in a week. Consult a nephrologist and he will direct you accordingly. Hope I have answered your query. Let me know if I can assist you further."
},
{
"id": 103514,
"tgt": "Nasal congestion. On allegra, nasonex and antibiotics. Chest x-ray showing granulomas. Taking prednisone. Any suggestions?",
"src": "Patient: I have had bronchial and now nasal congestion on and off since the end of December. I was put on antibiotics which didn t do much. I saw an allergist who performed the skin test and no allergies showed up. It is not asthma. I was put on Allegra and Nasonex , which didn t help much. I had a chest xray which only showed some granulomas but nothing significant. I was most recently put on prednisone and a stronger nasal spray to reduce inflammation and dry up the nasal passages. However, I am done with the prednisone and today I am still congested. My next step is to get a sinus cat scan. WHAT ON EARTH IS THIS??? I often cannot fully speak because there is so much snot and phlegm in my nasal area. It s gross!! Doctor: Hello,Granulomas in the upper airways (nose) or lower airways (lung) needs proper evaluation.The differential diagnoses:1. ANCA positive vasculitides - that require exclusion with ANCA, MPO PR3 blood tests, urea creatinine. If PR3 is positive, Wegener's granulomatosis is a distinct possibility.2. Sarcoidosis - ANCA is negative in this condition where granulomas form, and ACE levels (blood test) can be high in some patients. Steroids with immunosuppressive drugs are the treatment options.I hope that was useful.Best Wishes."
},
{
"id": 98367,
"tgt": "What is the exact dosage and administration guidelines of Methylprednisolone in case of asthma?",
"src": "Patient: I am in the middle of an asthma/upper respiratory infection and was prescribed Methyliprednisolone 4mg. It is a 6 day run sarting with 6 today, 5 tomorrow, 4 the following day etc. I expected more information when it was filled and now I don t know if they expect me to take 6 of these pills tonight or 1 at a time...timed to get 6 in within 24 hours. I am leaning toward the latter. Please advise Doctor: Hello and Welcome to \u2018Ask A Doctor\u2019 service. I have reviewed your query and here is my advice.The dosage of Methylprednisolone prescribed by the doctor is the right for your health problem.You are supposed to take all the daily tablets at the same time, not 1 at a time.It's better to take the tablets in the morning, after breakfast.Hope I have answered your query. Let me know if I can assist you further."
},
{
"id": 199606,
"tgt": "Can any online doctor help me with the following results?",
"src": "Patient: Quantity 5.0 ML ml; Sperm counts (Total count) 60 MILL/ML; Live count 15 MILL/ML, sperm per ejaculate 300 MIL; Motility: Percentage motile sperm 25%, Rapid linear progression 10%; slow non linear prgression 15%, non progressive 75%; sperm morphology 35%. SameerPleae commint Doctor: Hello Your semen analysis findings may indicate decreased sperm motility.Motility should be at least 55 %.Your findings suggests over all 25% motility,so it is low.Sperm count is normal.It should be at least 20 millions/ml.You should do exercise and take nutritious diet.You can also take anti-oxidants.You should avoid alcohol and smoking if you take these things.You also need clinical correlation and few investigations like random blood sugar,ultrasound of scrotumIf findings persists then you may need assisted fertilization technique like IUI.Get well soon. Take Care Dr.Indu Bhushan"
},
{
"id": 134364,
"tgt": "What causes twitching pain in arm post an injury?",
"src": "Patient: My daughter has been complaining about shoulder pain for over a month. Now she has arm pain and twitching in her right arm. She was in a dance competition in a back bend and flipped on her head when her arm gave out. What could be causing this? Muscle injury? What do we do for it. It seems to be aggravated after physical activity. Doctor: hi,thank you for providing a brief history of your daughter. well, as per my understanding by reading your history I can figure out that the fall is the cause for the injury. Well anytime a fall can be unpredictable that what injury it can really lead to. if you say the pain and twitching is in the upper limb I will advice you to meet up a ortho and take his advice. May be an x-ray or an MRI if the ortho feels after assessing your daughter will be guided which will play a key role in understanding that what actually might have got wrong. post that a proper rest, medication and proper care will surely help.with the grace of God I wish your daughter a speedy recovery and back to action."
},
{
"id": 66696,
"tgt": "Suggest cure for lump attached to collarbone",
"src": "Patient: Hi i have found a small pea sized hard lump attached to the top side of my coller bone. Im scared as ive had a persistan cough for 6m - trying steroids to see if its asthma been on them 1.5 weeks feel better but cough not completely gone. had chest xray 2/3 months ago. Get odd pain or two in chest too. Please help Doctor: Hi, this is a lymph node and could be due to some chronic chest/throat infection or even tuberculosis!get your sputum tested and an FNAC test from the nodule and revert back to us.wish you early recovery!"
},
{
"id": 94329,
"tgt": "Pain in the upper abdomen below the rib cage. Pain while breathing",
"src": "Patient: Hello. I woke up yesterday with a pain in my upper abdomen , right under my rib cage. It felt like heartburn , but it didn t burn like acid reflux did, more of an ache. It was sudden and just woke up with it. Its been 24 hours now but the pain is still there. It hurts when I take a deep breath, or move my body quickly Doctor: Dear friend, welcome and thanks for entrusting your query here at HCM! I am Dr. Prasad Akole (Critical Care Expert- http://bit.ly/Dr-Prasad-Akole) and am glad to address to your query here. Since you say it is different than heartburn of reflux disease, the possibility is gall bladder inflammation if on the right side. Bowel, stomach, spleen can be other organs of origin of pain. Sometimes early lower lung pathology can cause pain. Could you describe all the symptoms in details? Like what brings it on, how long does it last? What is it like? Increasing in any position? Relation to food? Did you have fever, nausea, vomiting, breathlessness? Would be glad to advise more precisely after you tell us these details. If the pain persists or worsens, please see your doctor soon. If it points to gall bladder etc. a sonography would help. I hope you are fully satisfied with the guidance I gave. I would be glad to answer any further queries. Please ask for any clarifications before closing and rating this answer. Thank you and Good luck!"
},
{
"id": 150199,
"tgt": "Diagnosed Piriformis syndrome as sciatic nerve pain is increasing, painful leg spasms, frequent headaches. What s going on ?",
"src": "Patient: Hi, had discectomy in Sept. Underwent PT mostly since then. Piriformis syndrome is what I've now been diagonsed since the sciatic nerve pain is once again increasing. Recent MRI of back, hip, knee were negative for sheath tumors. I'm getting painful leg spasms at night and frequently headache on the back of my head / at the base of spine. What is going on? Doctor: Hello,Thanks for the queryYou may be be muscle spams and/or the same problem for which you underwent surgery. I think a detailed neurological examination will clarify the picture. This will follow a test called electromyography which will help reach the diagnosis. The treatment will depend upon the cause identifiedPlease get back if you need any additional informationBest wishes"
},
{
"id": 57825,
"tgt": "Is it necessary for my father who has liver problems to go for ANA and ASMA test?",
"src": "Patient: hello doctor, My father is 62. he is having liver problem and is taking medicines. He is non- alcoholic. we got his hepatitis C test done and it was negative. We recently had his LFT done. His report was Bilurbin1.20, Conjugated Bilurbin - 0.70, Unconjugated Bilurbin - 0.50, Total Protien - 6.70, Alumbin - 3.60, Globolin - 3.10, A/g ratio - 1.16, SGOT -67, SGPT - 81 Alkaline Phosphate -134. From last few months his SGOT and SGPt has increased earlier it was normat SGOT 58 and SGPT 63. No one in our family had this genentic disease. doctor has further advice to go for ANA and ASMA and Coesophonal level test. is it really require. kindly suggest and is there any risk facto if SGOT and SGPT level goes higher Doctor: Dear FriendWelcome to Health care magic. I am Dr Charu , I will shortly try to help you with my opinion.I advise you to get also Hepatitis B testing.If it comes negative , you should get ANA , ASMA , to rule out Auto immune hepatitis. However , in autoimmune hepatitis , Serum Globulin levels are raised.Also , get an ultrasound of liver to see for size and echotexture of liver. Based on the available details, this is my best advice for you. You should discuss with your doctor and share my opinion , I am sure he/she will agree to my opinion.Good Luck."
},
{
"id": 153118,
"tgt": "Suggest treatment for stage 4 liver cancer",
"src": "Patient: My brother has been diagnosed with stage 4 liver cancer and cancer in his omentum. they cannot do surgery and he is not a canidate at this time for clinical trial med s as he has a high liver count which he is being tested for hep B. What do you believe his chances are at beating this? they have started him today on chemo pill form Doctor: Good evening. A stage 4 liver cancer is a bad disease. Moreover he has got HEP B which is the cause of his cancer. The only good treatment available for it is the chemo pill known as sorafenib which is effective in 40 percent of the patients. The average survival will be around 9 months if the patient responds to it."
},
{
"id": 198476,
"tgt": "Any suggestion for having azoospermia, fsh 20.4, lh 6.4, testosterone 1.9?",
"src": "Patient: i am a man of 43yrs,11months ago i was diagonised to have severe oligospermia,4 months later it became cryptozospermia.my fsh 20.4ng,lh 10.6 testoterone 2.1. ecosonography revealed small testis,with cyst and calsification at head,tail and body of left epididymis,and small cyst at the head of the right one.i stopped all medications and after 3months result read azospermia,fsh 20.4,lh 6.4,testosterone 1.9,csyst increased in size.am confused and we are trying to concieve.what your humble suggestion pls Doctor: HelloThanks for query .Based on the facts that you have stated your Azospermia is not related to FSH or Testosterone levels .It is basically due to pathology of the both the testicles and epididymis .In a given situation there is no ray of hope to improve quantity of sperms in semen and having normal and natural conception .You need to consult qualified Infertility specialist for clinical assessment and to confirm whether testicles are producing sperms or not?If there is production of sperms by testicles he may suggest you to go for In Vitro Fertilization (IVF) or ICSI by using sperm being aspirated (Sperm Aspiration) directly from the testis .Dr.Patil."
},
{
"id": 65235,
"tgt": "Suggest remedy for lumps in neck",
"src": "Patient: i just turned 14 a couple of days ago and i had a pain in my left peck when i bumped it. It only hurts when something touches it, i ignored it but tonight i had touched my peck there is a small lump but it does not hurt as much as it did when i first found it. I don't know what it is but I'm a little worried it could be serious. Doctor: Hi! Good evening. i am Dr Shareef answering your query. Even though it needs a clinical examiantion, from the history it seems to be a hematoma (collection of blood) in the subcutaneous tissue resulting from the bump. Usually small hematoma would get absorbed by itself and nothing more might have to be done. If I were your doctor, I would prescribe you with an anti inflammtory along with a proton pump inhibitor and ice-soakages for some time for a symptomatic relief. If it increases, then you might have to see a surgeon for further opinion and management.I hope this information would help you in discussing with your family physician/treating doctor in further management of your problem. Please do not hesitate to ask in case of any further doubts.Thanks for choosing health care magic to clear doubts on your health problems. I wish you an early recovery. Dr Shareef."
},
{
"id": 109080,
"tgt": "What causes severe pain in the lower back?",
"src": "Patient: Hi i have had this pain for a long time now. well ever since i've had my son and it's in the lower part of my back. I know i suffer from a weak back since having him, but everytime i want to poo i suffer from pain in my lower back and its a wierd sensation. Could you tell me of any solutions. thanks Doctor: Low back pain is a very common ailment in females and the cause usually is calcium and vitamine deficiency. Are you taking these supplements? If not start on calcium along with vit D3 and also multivitamin supplements. Also add an antioxidant to it. Also get your general pathology work up done to rule out anaemia and diabetes. If the pain persists even one month after this, then get your back examined by an orthopaedician after doing a xray."
},
{
"id": 46157,
"tgt": "Suggest treatment for increased levels of creatinine",
"src": "Patient: my sister is 35 years old she is suffering from diabetes since 10 years very recently we came to know that she is suffering from kidney problem her creatinine level is 2.6 when her sugar level is under control her creatinine level goes up and vic e versa kindlly suggest what shhould be done in future Doctor: Dear user, I recommend you: step 1) to perform a fundoscopy to diagnose diabetic retinopathy. If this is diagnostic for retinopathy, then it is very likely that your sister has a diabetic nephropathy, although EVERY organic cause should be excluded at least with lab exams (C3,C4, ANA, kappa and lambda free light chains, urine analysis). If your sister has a negative fundoscopy, a renal biopsy should be performed. Avoid every nephrotoxic drug, such NSAIDs.Let me know, Andrea"
},
{
"id": 121075,
"tgt": "What causes a sudden onset of swelling on left side of armpit?",
"src": "Patient: I am a 41 yr. old female with a sudden onset of swelling on my left side below the armpit. My breast is also swollen. No redness, no fever. It is tender if I push on the ribs but I can t feel any lumps. It has been like this almost 2 weeks. I haven t had any injuries but I have been carrying my grandson ( 25 lbs) alot. Should I be worried? I have no insurance and can t afford to go the doctors office right now. Doctor: Hello,I read carefully your query and understand your concern. Your symptoms seem to be related mastitis which is an infection of the breast. The node is related to reactive lymph node which is common in an infection. I suggest using anti inflammatory medications such as Acetaminophen to relieve the inflammation. If the symptoms continue, I suggest you to take a cycle of antibiotics. Hope my answer was helpful.If you have further queries feel free to contact me again.Kind regards! Dr.Dorina Gurabardhi General &Family Physician"
},
{
"id": 214189,
"tgt": "Does using inhalers for toddlers cause side effects?",
"src": "Patient: my daughter is 4 yrs. She was advised to use budate/budecort inhaler regularly from the age of 2 (2puffs 2times a day), non-availability of these two use budamate. and salbair inhaler (2puffs 4/6 hourly) during cough. She has an allergrtic cough and cold problem. I want to know the side effects of this inhalers and whether she has to use it lifelong? Doctor: HelloI have gone through your question and understood your concern.Budamate Inhaler contains Budesonide, and Formoterol Fumerate as active ingredients and it works by relaxing muscles and opening air passages; reducing swelling and inflammation in the nose, throat, lungs or intestine.The most commonly reported side-effects of Budamate Inhaler are Blemishes or pimples on skin, Local hypopigmentation of deeply pigmented skin, Cough, Dizziness, Dry mouth, and Headache.Salbair Inhaler contains Salbutamol as an active ingredient and it works by relaxing the muscles in the walls of the small airways in the lungs.Possible side effects of this drug are:CoughTightness in the chestHoarsenessHyperactivity in childrenShortness of breathDifficulty breathingThis inhalers are used for Asthma,Respiratory disease,Lung disorders.Your doughter has an allergic cough,this is a chronic condition ,so she may need to use this drugs lifelong.Hope i have answered to your questions.If you have further question ,i will be happy to helpThank you for using HCMWish you good health."
},
{
"id": 50426,
"tgt": "Nephrotic syndrome. Have swelling, weight gain, hair fall. Taken wysolone. Side effect?",
"src": "Patient: hi, i am 28 yr old. my name is geeta kapoor. befor 10 mnths i had nephrotic syndrom. I take wysolone 40 mg for at least 45 days. I had lots of side effects at that time like swelling, weight gain. but now i am ok. but at that time i had very heavy hair fall and now my hairs are not growing. tell me what to do. waiting for your reply Doctor: Hi !From your history it appears to be a side effect of wysolone. It usually does not happen if you are taking a short course but can occur if you are taking the drug for a longer duration.You can consult a dermatologist for further evaluation and management."
},
{
"id": 158161,
"tgt": "Take dulcolax, milk of magnesium. Feel bloated and uncomfortable. Unable to use glycolax prior to a colonoscopy. Treatment?",
"src": "Patient: My husband is unable to use the glycolax that most people use prior to a colonoscopy. He took 3 dulcolax last night and 4 tablespoons of milk of magnesium this afternoon. He has gone to the bathroom but is very bloated and uncomfortable. He is to start drinking mirolax tomorrow. Any suggestions on how to help him feel better? Doctor: Hi! Thank you for your question and welcome to our site. As you may already know, these types of medicine help clean the bowels but can induce gas production and bloating through bacteria living in our intestines. As such, these laxatives really do have bloating, abdominal pain and even sweating as their side effects. These medicine usually act within 2-3 days. The most important thing is to follow the instructions on how to take them and to drink plenty of fluids. Taking some milk-products with probiotics like yoghurt and other drinks may help to balance the effect of the bad bacteria. I hope I was able to address your concerns! Please let me know if you have further queries.Take care!"
},
{
"id": 16374,
"tgt": "How can I remove the skin rashes on feet and legs due to wetness ?",
"src": "Patient: I stepped in a puddle in the mountains about a week ago and the inside of my boots got all wet. The next morning I woke up and I had a rash on the inside and soles of my feet that looked like broken blood vessels. It doesn t itch or hurt. It just looks like small broken blood vessels under the skin . Since then (about a week) it has spread up my legs to my groin area. I have a history of eczema so I tried my Vanos creme and it did nothing. I also tried Lotramin thinking it may be fungal and it is also not helping even in the slightest. Doctor: It can be a bruise and progressively going up the Long saphenous vein or may be infected. or you check if you are taking aspirin. In case both are not I wish to see the sin and examine which you see your GP /Surgeon to sor out the diagnosis. Treatment is Antibiotics/Stop warfarin or aspirin. and treat bruise conservatively with hirudoid cream /other. but also rule out any gangrene,"
},
{
"id": 55267,
"tgt": "What causes blood in urine, bloating and pain after eating?",
"src": "Patient: Had recent ct and mri scan only thing showing is cyst in lt renal pole ? I am convinced I have some form of liver disease . I have been in constant pain and discomfort for 2 yrs now urq I bloat and pain after eating. Blood in dark urine tired and my stomach actually feels nodules now how can an mri not detect what I fear is wrong ?? Doctor: Hello! Thanks for putting your query in HCM. I am a Gastroenterologist. When both CT scan and MRI are normal, I dont think there is something to worry. Keep cool. Get your urine examination and liver function tests to confirm if there is anything wrong in the liver and to confirm whether there is blood in urine or not.I hope I have answered your query and this will help you . Wish you a good health"
},
{
"id": 53291,
"tgt": "Suggest treatment for hepatitis-B infection",
"src": "Patient: Sir, I am suffering from hep b virus infection. Had cirhossis of the liver in 2003 and it is chronic case. Things are under control under the guidance of mumbai hepatogist. for three years i was on limuvidine and for two years on adesara(adevefoir) and now on sebivo(telbivudine) - a costly medicine. i am hbeag -ve, anti hbe +ve, hbv dna -ve. Is there any alternate therapy for the problem? Doctor: Hello! Thanks for putting your query in HCM. I am a Gastroenterologist.Since you have cirrhosis antiviral has to be given for long time. So I will suggest you to take Tablet tenofovir, of which no resistance is known till date even after use of more than 10yrs. Medicines which you are using may show resistance and there will be viral breakthrough. Discuss about this with your doctor. I hope I have answered your query and this will help you . Wish you a good health"
},
{
"id": 38118,
"tgt": "How long does corn cap holes take to heal?",
"src": "Patient: I had a corn removed from my toe last week. It must have been pretty deep bc now I have a large hole in my toe. I am using antibiotic cream and taking cephalexin for a week. How long should it take for the hole to close up, and will it really close up to normal skin? Doctor: Hello, Thank you for your contact to health care magic.I understand your cconcern. If I am your doctor I suggest you that as your corn is very deep it requires a long time to heal may be 2 to 3 month. It will come back to normal skin having a mark of incision on it. But till that time continue with your medication.I will be happy to answer your further concernYou can contact me. Dr Arun Tank. Infectious disease specialist. Thank you."
},
{
"id": 195968,
"tgt": "What causes rashes on the shaft of the penis?",
"src": "Patient: My boyfriend appears to have a itchless, painless, petichial-type rash on the shaft of his penis for approx one week. There is no drainage noted from any of the sites and it appears to only be on the top of the shaft. He is uncircumcised and I believe he has erectile dysfunction, because he uses a band made of velcro to help maintain his erection. What could this rash be? Doctor: one should to need to rule out HIV infections ,but,Genital rashes aren\u2019t always a sign of HIV. They can result from a number of other conditions, including:jock itch, which is a fungal infection associated with staying in sweaty clothing for too longa yeast infection, which is an overgrowth of fungusbalanitis, or the swelling of the penis tip or foreskin, which is associated with poor hygienecontact dermatitis, which may result from allergensThey can also indicate the presence of other sexually transmitted infections (STIs), such as:scabiescrabssyphilis"
},
{
"id": 118336,
"tgt": "What is an alternative to aspirin for weakened vein condition?",
"src": "Patient: Hello, my name is Elizabeth Gray, my question is , I am currently taking aspirin for a weakened vein condition to keep my blood from getting to thick but I have a rash like hives on my neck and chest which is driving me crazy and I think it is because of the aspirin as it has happened before, can I safely stop taking the aspirin and maybe go on something else instead. thank you. Doctor: Hi, You started taking aspirin but for which condition you forgot to mention . It is anti platelet drug at lower dose and hence prescribed in conditions which can led to thromboembolic conditions. It has side effect of bleeding which currently you are facing . So ask your doc to shift to other group of drug ( as should not be stopped abruptly ). Thanks n regards"
},
{
"id": 51427,
"tgt": "Have stage 3 kidney disease, general malaise. Could the WBC be from the lupus or from kidney disease?",
"src": "Patient: I have had left radical nephrectomy from kidney cancer and also have stage 3 kidney disease. Recently my UA has repeatedly shown WBC but no culture growth. I do know how to do a clean catch mid-stream collection. I also have SLE and I am concerned that there is a problem with my remaining kidney since I have had general malaise symtoms. Could the WBC be from the lupus or from kidney disease? Doctor: Hello & welcome to HCM forum 1) WBC/ Pus cell in urine up to 3 cells in female & up to 5 cells are normal. 2) If your urine culture is negative then it indicates that you are not suffering from Urinary Tract Infection. 3) Still pus cells beyond normal limit in urine indicate cells arising from glomeulus in kidney. SLE cause glomerulitis in kidney and you may have pus cells in urine. In such cases, urine examination also shows presence of RBC in urine. 4) You should go for kidney biopsy and immuno-fluorescence study of kidney to see the exact present status. Thanks Dr. Mayank Bhargava"
},
{
"id": 47673,
"tgt": "What could be the black spots on kidney shown in ultrasound?",
"src": "Patient: My mother had an ultrasound today. They spent over twenty minutes on one side and took photo s of black spots on one kidney and also the gall bladder. Results have to be reviewed by radiologist then sent to the doctor in a few days. Wondering what the spots could be? Doctor: Hi, welcome to HCM.Looking to your mother's history, it seems that black spot on one kidney on USG may be due to cyst in kidney.But, it may require CT scan with contrast to stage it and decide further plan of action.Don't get worried.You can communicate further with me with above reports. Best wishes. TC.Dr Jay Patel"
},
{
"id": 52259,
"tgt": "What does this ultrasound report indicate?",
"src": "Patient: Hello Doc USG of Liver reveals 18.6*16.6 ecogenicity found. Fatty liver spare. D/D- Haemenagioma\\ fical hyperplasia.... What does it mean that a woman having liver tumor? Her age is 29 yrs.married ,having 2 kids and no habit of alcohole consumptions.... Doctor: Hello, Both the mentioned condition in the liver is Benign tumour and so don't worry about that. But for further workup, you need to get investigated with MRI scan of the abdomen. If needed CT guided biopsy can be planned for confirmation of diagnosis if your doctor found it. You need to consult a gastro surgeon for detailed evaluation. Hope I have answered your query. Let me know if I can assist you further. Regards, Dr. Parth Goswami, General & Family Physician"
},
{
"id": 214413,
"tgt": "Suggest natural remedies for abortion",
"src": "Patient: I m 44 years old. going through miscarriage since may 27 when I first start bleeding .I was 8 week (7 weeks and 6 days as the doctors said it s the baby size) I don t have a sign of progress, no bleeding now. I would like to avoid D&C .is there any natural way you can recommend to me.? my hormone level now 32 last week was 39. Thank you Doctor: Greetings Madam. I am glad that i am able to guide you.As of my opinion is that if you want to avoid getting complication out of trying any such method in home(which is not at all advisable at any age. But you are already having problem getting conceived & your age is more than 30).My sincere advise is fix appointment with OBGYN, express to her your problem & she may help you in getting out of this problem & also suggest best methods to get conceived & relieve your stress so that you could lead a happy life ever after.& Note:Finally it's just a guidance. Not a definitive treatment as i haven't seen you personally. You've to realize that these are temporary remedies. I don't know what your exact condition is unless I see you personally. A doctor after personally examining you could help.Hope it helped you & guided you right."
},
{
"id": 165993,
"tgt": "What causes armpit odor in child?",
"src": "Patient: My 4yr old son has horrible armpit odor. I bathe him every night or every other. I keep a clean house and change his sheets every 10 days. The only problem I can think of is his diet consists of dairy and carbs, which Im working on changing. Any ideas? Doctor: Abnormal skin odour may b due to some foods and sometimes some metabolic disorders may also cause this. You should consult your pediatrician for that."
},
{
"id": 75829,
"tgt": "Can Mantoux cause persistent fever?",
"src": "Patient: Hi DOctor, I donot have maor heatlh issues but last month i had fever....not knowing the reason.It was 102 and would come n go by itself...I have got all test done staring from typoid to malaria to Montaux...all are neg though i was positive to mantaux...doc did not treat me saying i donot have enuf symptoms to treat any disease....although my ESR is 80...and now i feel a hard lump inside my neck on left side...cant see it out side....Help Doctor: Hi, I see you have investigation reports in search of diagnosing fever of ? origin but except Mantoux all are not indicative for any thing. In presence of high ESR- 80mm/hr with a lump on Left side although more hard to feel are enough to suspect some chronic infection with a focus some where but unlikely Mantoux responsible for continued fever since you were looking for clue with Mantoux test included. Without waiting any additional symptom to manifest , I consider it would help you and we doctors if you agree to undergo Aspiration Cytology or better still Histopathology of the hard lump you noted and special culture of possible bacterium at least to rule out Tuberculosis. please also add some info as your present age, occupation, country of stay, any migration, any drug history- present and past along with the discussed investigation asap."
},
{
"id": 178529,
"tgt": "Should stool test be done for child having rice like substance in stool?",
"src": "Patient: Rice like substance in the stool of 3 year boy. Did have stool samples to the lab x2 returning negative however stool at the time was normal and no rice substances were sent. Should new stools samples be taken with the rice substances be collected and resent? Doctor: hi..if child is not having loose stool, no bad smell and not rice watery, not much to worry. just rice particle or vegetable particles are commonly seen in small children. it is due to poor chewing by children.if you are happy with my suggestion kindly rate me.Regards-Dr.surendra.h.S"
},
{
"id": 196644,
"tgt": "What causes recurring fungal growth on penis?",
"src": "Patient: I had a previous bout of suspected fungal growth on my penis about 2 months ago and I am uncircumcised. This is not the first time in my previous years and the solution every time had been a course of anti fungal cream and if its persistent my gp would also prescribe some pills to clear it up. The itch and fungus would go away normally in a week. While its not advisable on my earlier bouts I have also used hand cleaners or gels to clean the area during shower as a an added measure which fortunately have never sparked Any form of skin allergy. However I noticed this year on my first bout , happened in April and I saw the same doctor my penile skin started peeling in thin layers ( white in nature ) that would seem to roll off the surface as you rub and clean it to expose a shinier new skin under. This happens on the forehead as well as the foreskin adjoining it. Tis has never happened. I was thinking if the very strong anti bad hand wash which I used to clean during shower has taken a toll on the skin. In any case the gp prescribed me some diflucan for a 4 week single dose per week course and everything cleared in 2 weeks including the skin peel. Fast forward 3 months in July I had a reoccurence for some reason but this time with the same symptoms I can t get rid of the skin peel. The gp and also a dermatologist I saw mentioned I no longer have a fungal infection looking at the condition of the penis. There is no itch no discharge and looks clear except for some redness at times when it does peel after every 3 to 4 days. She prescribed daktacort that I think made things worse, more reddening, I tried konzert with not much effect for I think the earlier bouts of canestan has cleared all fungal infection. I am beginning to think i have a dermal condition on the skin for it seems to be really really delicate. I tried just applying a little papaya ointment to moisturizer the area that looked fine for a while but the symptoms would always be the same, over time the skin would just start to peel all over again. please help. Doctor: Hi,Recurring fungal infection can be as a result of underlying immunosuppression as in cases of diabetes and HIV Itraconazol pulse therapy is usually required for long duration in cases of resistant infection. Hope it helps. If you have any other question please do not hesitate to contact us.Regards,Dr. Atishay Bukharia"
},
{
"id": 128932,
"tgt": "Suggest treatment for severe leg pain",
"src": "Patient: I have selling of both ankles and feet and as well both legs ache. I do have some diagnosed bulging discs in my cervical spine at C5 and C6. Could this all be related or are the leg aches and feet swelling different. I have had blood work and the only thing it has show is higher than average levels of inflammation. Doctor: Hello,Thank you for using healthcaremagic.I read your question and understood your concern.I do not think that you ankle problem is related with the cervical spine bulging discs.I think this is more of an inflammation so I would reccomend you to see a rheumatology specialist to give you proper medication with antiinflamatory drugs.Also thyroid problems might give swelling of ankle and feet , so you should check the thyroid function with blood tests.I wish you quick recovery.Dr. Selmani"
},
{
"id": 142328,
"tgt": "What causes light headedness and pressure behind eyes?",
"src": "Patient: Everyday, I feel very light headed. The pressure is behind my eyes and feels like it goes behind my ears and to the bottom of my head and down my neck. My muscles in my neck, shoulders, and back also are very tight. When this happens I get very scared, I m afraid I am going to faint. I also have really bad anxiety and panic attacks, I m not sure if that could have anything to do with it. -Gina, 17 years old Doctor: Hello Gina!Welcome on Healthcaremagic! Your symptoms seem to be related to tension type headache and panic attacks. There is nothing to worry about!Coming to this point, I would just recommend you to focus on your anxiety. Yoga and meditation can help in this regard. I would also recommend being active as much as you can (walking, running, sports, etc.). Psychotherapy can also help. I would also recommend checking your complete blood count for anemia and thyroid hormone levels to exclude possible thyroid gland dysfunction. Hope you will find this answer helpful!Best wishes, Dr. Aida"
},
{
"id": 119540,
"tgt": "Suggest remedy for calcification of ribs",
"src": "Patient: on xray of my back(due to pain) my bones have the appearance of being hollow and my entire upper carage or ribs are severly calcified. My ribs at the bottom are not rounded they have jagged points calcified. Leg tightness and stiffiness, lack of energy. I am 42 and have bones of a 90 yr old women...what could cause this Doctor: Hi, As per the signs described by you i confer that its a case of hyperparathyroidism. There are many causes and types of hyperparathyroidism. Some are trivial while other are very critical. I suggest you get a thorough clinical examination along with appropriate investigations. As far as hollow vertebrae are concerned they are an indicator of osteopenia, which is curable, bit we need to rule out other severe concerns first. Take care. Hope I have answered your question. Let me know if I can assist you further. Regards, Dr. Rohan Shanker Tiwari, Orthopedic Surgeon"
},
{
"id": 199406,
"tgt": "I want to increase my intercourse period and want to get my actual penis size. Remedy?",
"src": "Patient: Hello Sir I am Sojib and 26 years old and also married. but I have to face some sexual problem. My semen is watery and when egaculate the amount is very less. Besides it egaculates first time within 40-50 seconds. And now I am feeling that my penis size is seemed to me smaller than my past. I have habit of musterbation. Sir I want to increase my intercourse period and want get my actual penis size that I had. Please give me some direction and suggestion. If medicine needed please tell me about these. Doctor: First you need to adopt healthy lifestyles. If you are obese then you should exercise regularly. Avoid smoking n drinking. N abstain from masterbation. If your problem is mainly premature ejaculation then u shud try Master Johnson technique. It is foreplay without use of genitals. Use your hands n mouth but not your genitals.N u need to be open to your partner as well. Try these n if u still have problems then you should visit a psychiatry clinic with your partner."
},
{
"id": 36675,
"tgt": "Suggest medication for swollen lymph nodes above the elbow",
"src": "Patient: My gf in philippines has has a swollen lymp about 2 inches above her left elbow has reddened now she cant lift her arm she says has other lumps it is about 2 inches long and inch wide is reasonably soft to touch like fluid and has been there intermittant for 6 to 12 months only this few weeks has got red and now reallly painful Doctor: Thanks for your query at HCMYour patient is suffering from lymphadenopathy which is due to infections, inflammations or cancers. But as it is soft in consistency it can be infective in origin either viral or bacterial. You need to treat the underlying cause rather than swelling. I suggest get basic hematological test, ESR done. Undertake USG FNAC and examine for cytology, ZN stain (Tuberculosis) and gram stain/culture & sensitivity for bacterial infections. You can take analgesics for pain. Follow up with investigations to start appropriate treatment.Take care!"
},
{
"id": 50529,
"tgt": "X-ray showing cysts in the kidney, nabothian cyst. Pap smear showing candida presence. STD?",
"src": "Patient: Hello doctor, the following are the results of my X-ray and utz:LSXray- lumbar instabilityKUB- 3 cysts on the left kidney, 1 on the rightTransv utz- All normal except for Nabothian cyst. Cervix is normal with homogeneous echopattern. an anechoic lesion measuring 0.8\u00d70.7\u00d70.7 cm (vol=0.21 cc) is seen within.Pap smear \u2013 moderate inflammation, Organism- consistent with Candida spp.I wanted to freak out because I thought having candida means having std, What meds can I buy over the counter? My schedule of going back to the OB is 3 days from now, for the mean time, what can you suggest, my husband and I just had sex 2 days before I found out bout the candida.. Does this mean he is also infected? Other than using tampons, lubricant gel, douching, re- current UTI, can it be caused by having diabetes insipidus?I am hoping my questions will be answered,I'm getting more paranoid everyday.. Thank you so muchSincerely, Mar Doctor: Hello,Thanks for writing in.Coming to each investigation:1. X ray Lumbar spine shows instability: Means there is some little problem in your in maintaining the back posture.2. KUB 3 cysts in the left kidney: The cysts size is not mentioned. I believe they are simple cysts and something you should not be worried about.3. Transvaginal ultrasound: Nabothian follicle is a small benign follicle which is taken as a normal finding. Nothiong to worry for it.4. Candida infection and diabetes: Yes there appears to be an increasd likelihood of fungal infections in diabetics. In your situation the infection is in the cervix. This may require treatment with anti fungals in tablet or cream (cream is difficult to apply in your condition).Your doctor will give you anti fungals to take for a while.Your husband may not be infected if he has a good immunity. Still he also taking a course of anti fungals will be beneficial to both of you.Hope this helps."
},
{
"id": 190064,
"tgt": "Lump in neck, growing, have dental issues, have flu and cold. What is wrong?",
"src": "Patient: I have a lump on left side of neck , recently getting bigger. I felt around and have smaller one on right side, sort of where the vein would be. I know, I have been having teeth problems because can afford dental any longer and feel sick a lot from that, and sometimes feel like heart is effected by this. I have been getting sicker and sicker lately, but the flu cold has been going around that is quite bad, but lump was there before, now today, noticed a lot bigger. Im a little worried maybe health issues from teeth and then getting sick on top of that, but keep telling people for months feel like someone that would be dying. But just letting it go. Im so beautiful and love my looks but feeling so sick and lump on neck is actually hurting a little tonight, what could this be?? Doctor: Hello there... Problems associated with teeth, nose and sinus (infections arising from them) would drain into the lymph nodes in the neck region....these get enlarged and become painful....there are swellings seen in neck as a result of an abscessed lower tooth- the infection spreads down to the neck and manifests in the form of swelling...swelling in each part of the neck has its own significance. Clinical examination is mandatory. if teeth related consult a Oral and Maxillofacial Surgeon. Otherwise you can consult an ENT surgeon for further management."
},
{
"id": 39694,
"tgt": "How herpes is transmitted?",
"src": "Patient: Hi my girlfriend has a coldsore on her lip she just found out about today. She said she will buy over the counter creams and apply it tommorow. I genereally don't know how the herpes coldsore virus works and how its transferred. Is it through direct contact or can i get it by kissing her cheeks? Doctor: helo thanks for consulting at hcv..herpes is contracted via contact with the mucosal and abraded skin surfaces. Since HSV-2 is tropic for the genital regions, transmission of HSV-2 is usually through sexual contact. HSV-1, which usually infects the mouth and lips, is usually transmitted via kissing or through the exchange of premasticated food,so kindly abstain frm contracting the virus,, she can take topical antiviral and if needed systemic antiviral to reduce the viral sheddings..kindly follow strict personal hygiene,,hope it helps,,tc"
},
{
"id": 40432,
"tgt": "What are the symptoms of staph infection?",
"src": "Patient: hello, i think that i have staph infection but hard to get diagnosed. is there any specialist in la whom I can get tested and treated, please advise. I'm depressed after reading all on line writings. it seems that the germ remains inside of body no matter what. is this a curable illness? Doctor: Hello,Welcome to HCM,Yes, staphylococcal infection can be cured by choosing proper antibiotics. Penicillin group of antibiotics definitely helps to prevent the multiplication of the cocci and helps to prevent symptoms caused by these bacteria.Antibiotics in combination like amoxicillin and clavulanate potassium as got very effective action against this bacteria.You can get the antibiotics by consulting your doctor and get a prescription for the treatment of the infection.Thank you."
},
{
"id": 123205,
"tgt": "Suggest treatment for bone island on left thigh",
"src": "Patient: I have been diagonized with bone island (left thigh)..and its painful.Couple of lesions in the left and right hip. Did a CT scan to evaluation being cancerous and the reports all came out negative on cancer. What is my next step in treating my ailment? which doctor should I see? would cortizone shots do anything? Please help. Doctor: Hello, No steroids- oral or injectable. Consult an orthopedic surgeon. A biopsy must be done. further treatment thereafter. Hope I have answered your query. Let me know if I can assist you further. Take care Regards, Dr Nirmal Chander Gupta, Orthopaedic Surgeon"
},
{
"id": 120188,
"tgt": "What does the MRI mean?",
"src": "Patient: My 13 yo daughter s MRI reads: mottled pattern of abnormal bone marrow signal intensity throughout the hindfoot may represent complex regional pain syndrome ( reflex sympathetic dystrophy) although is nonspecific and could not exclude other entities as described above. Described above was Charcot arthropathy, but there is no evidence of this. What does all of this mean? Doctor: Hello,The exact cause of complex regional pain syndrome isn't well understood but may involve abnormal inflammation or nerve dysfunction. Symptomatic treatment with supportive care must be given.So don't worry in case of charcots joint may be due to nerve damage lead to arthropathy. Medication with regular exercise and physical therapy may useful in some cases. Please consult with your orthopedician he will examine and treat you accordingly.Take care. Hope I have answered your question. Let me know if I can assist you further. Regards, Dr. Penchila Prasad Kandikattu, Internal Medicine Specialist"
},
{
"id": 24385,
"tgt": "What causes heart to beat abnormally?",
"src": "Patient: I'm a 18yr old girl and in a resting state my heart will beat anywhere from 82 to 114bpm and when I consulted a heart doctor he wasn't very professional and said at my age 200bpm is the norm but when my heart randomly beats that fast its not comfortable and its started to worry me. Doctor: Hello 114 heart rate is not that great... Your ecg should be done and need to show there is no rhythm problem... Natural body response to anxiety running exercise leads to increased heart beats... And these many times appreciated as palpitation .. So if your ecg during normal heart rhythm with fast beating then no need for medication... take care"
},
{
"id": 56460,
"tgt": "What treatment is suggested for laceration of liver?",
"src": "Patient: Wife fell out of my tractor trailer and broke 3 ribs #8,#9 #10. She also had a laceration to her liver. This happened 3 weeks ago , been in and out of hospital. They sent her home saying it will heal itself. Her stomach is bloated and she was told she has fluid and blood build up on outside lever which is swollen, they said surgery was not needed. How is the fluid and blood going to go away to allow swollen to go away? Do not trust the doctors at this hosipital. They did not see the laceration on liver on first visit and they found it after she had to go back after she visited her own doctor and found her hemoglobin was low. Doctor: Hello , ill explain the problem thing is that when she had trauma to chest and abdomen on first cosultattion ur doctor detected rib fractur but thing is that they woud have done ultrasono to detect abdomen trauma but it may happen that it is not detected on first usg it can happen another thing is that whta doctor has said that liver lacearation will heal its own . it is right now days for liver trauma conservative line of treatment is given i hope this explanation ll be helpfull to u thank u"
},
{
"id": 50113,
"tgt": "Suffering from Double malaria and jaundice. Recommended dialysis. What does it lead to?",
"src": "Patient: Hello Doctor, My Brother - in - law is admitted to one of the hospital in thane when he complained about the breathing problem and temprature. There we came to know that he is suffering from Double malaria and jaundice . They straight away took him to ICU and did some blood tests which showed that blood platelet count is as low as 0000 so they injected 2 bottles of platelets and on third day the count came to 0000 and today body on its own started developing platelets and today s count is 0000 which i suppose is good. But his kidney is not responding in a normal way now, in 24 hours he has passed only about 300 ml of urine. So to deal with it doctors have recommended dialysis to remove impurities. But after doing the dialysis 3 times in a week still the passage of urine is not upto the mark. I want to take a second opinion and see if the medication what is being given is on right track or not. Currently he is being treated in Titan hospital Thane. For second opinion i need your help or if you can recommend me to some specialist who can give me a second opinion. I can be reached at 0000 Thanks and Regards Sumit Sharma Doctor: Hi,Thanks for writing in.I suppose your brother in law must be suffering from acute renal failure as a complication to his medical illnesses. This condition of 300 ml urine in a day is known as oliguria. Dialysis has been recommended because of raised blood urea and serum creatinine. As you have not mentioned about the drugs and any possible nephrotoxicity arising from it, its difficult to tell the exact problem.You may seek guidance by discussing the case with a nephrologist and taking along with yourself complete treatment details and reports.Hope this helps."
},
{
"id": 133761,
"tgt": "What can be the reason for join pain and swelling?",
"src": "Patient: hi, I m a 59 year old female with joint pain and swelling in my knuckles. also have been bruising very easily and very often. My platlett count came back within normal levels and I tested negative for reumatoid arthritis. what else can cause my symtoms. also I have had some weight loss but my appetite is good. Doctor: hi,thank you for providing the brief history of you.A thorough musculoskeletal assessment is advised.As you have pain in the knucles and you don't have RA it could be a overuse injury of the tendon. which can be helped with the help of a physical therapy.Also, weight losing happens due to various other factor for that a thorough assessment is needed.since the knuckles are getting swollen taking hot water fermentation and performing small exercises of the hand will be helpful.RegardsJay Indravadan Patel"
},
{
"id": 42548,
"tgt": "What causes low BP and brown spotting after an IUI?",
"src": "Patient: Dear me and my wife are trying for baby more than two year. we are suffering for male infertility. count is vary from 4 million to 14 million. we had two unsuccessful IUI. Third IUI already done and waiting for results. after third IUI she is having low blood pressure almost everyday and brown spotting. today is 14 day post IUI. if this IUI is failed. do you recommend to do another IUI or go for IVF. how manu try is advisable.thanks Doctor: HAI WELCOME TO HCM SINCE YOU ARE HAVING VERY LOW SPERM COUNT YOU CAN CHOOSE INVITROFERTILISATION TECHNIQUES TO CONCEIVE.IUI IS NOT AN OPTION FOR THIS SPERM COUNT.CONSULT AN INFERTILITY SPECIALIST.REGARDSDR.VANITHADEVI"
},
{
"id": 108379,
"tgt": "What causes pain in the upper back and neck?",
"src": "Patient: i have had pain in my upper back towards the right side, pain in the neck and also pain from right shoulder down to fingers already had a mri/mra of chest and mri of neck and x-ray of neck and upper spine and a nerve conduction done everything can back normal don't know what it could be any suggestions would help Doctor: Hello, I have studied your case. With normal investigation reason of pain can be due to fibrosis.you may benefit from local injection. It may give pain relief for longer time.For these symptoms analgesic and neurotropic medication like pregabalin can be started consulting your doctor.Till time, avoid lifting weights, Sit with support to back. You can consult physiotherapist for help.Physiotherapy like ultrasound and interferential therapy will give quick relief.I will advise to check your vit B12 and vit D3 level.Another possibility of fibromyalgia.You can take amitryptiline and duloxetin consulting you\u2019re treating doctor.Hope this answers your query. If you have additional questions or follow up queries then please do not hesitate in writing to us. I will be happy to answer your queries. Wishing you good health.Take care"
},
{
"id": 112995,
"tgt": "Pain between rib & lower back when moving. Why does he get pain?",
"src": "Patient: My father in law is experiencing pain on his right side. Somewhat between his rib and lower back. It's very tender and he said it hurts when he moves, coughs, or sneezes. He does not have pain inhaling/exhaling, or when going to the restroom. I have suggested he go his doc as I do not believe its musculoskeletal. Any ideas? Doctor: Hello. Thanks for writing to us. The pain on the right side is likely to be due to musculoskeletal origin. It could be due to an injury to the area or a muscle spasm in the area. He needs rest, hot fomentation , pain killers and muscle relaxants for relief. I hope this information has been both informative and helpful for you. You can consult me again directly through my profile URL http://bit.ly/Dr-Praveen-Tayal Regards, Dr. Praveen Tayal drtayal72@gmail.com"
},
{
"id": 164176,
"tgt": "What causes mottled skin in a 2 year old?",
"src": "Patient: Hi my daughter is 2 and a half and has mottled skin, sometimes it is worse than other times but most of the time always there. She had viral meningitis at 5 weeks old and was in hospital for 2 weeks andwhen she gets a cold or viral infection she always gets it really bad but i just wondered if her mottled skin is ok or would there be a reason behind it. Doctor: Hi.... I feel it could be a - status marmoratus (physiological skin mottling to changes in ambient temperature).This is not related to the meningitis she had as an infant and I appreciate you giving me the old history of medical significance.I suggest you upload an image of the same so that I can guide you better. skin conditions are best diagnosed and treated after seeing them directly.Regards - Dr. Sumanth"
},
{
"id": 109897,
"tgt": "What causes chronic neck and lower back pain?",
"src": "Patient: Hi, I would like to know when I should be worried about my neck pain? The left side of my neck has been hurting for 2 weeks now. I didn't sleep on it wrong, it just hurts. The pain is on the left side of my neck, I can barely turn my head to look left or tilt my head back because it hurts so bad. Also my lower back hurts and pain going into my pelvis to the point I can hardly stand up straight to walk. Please help Doctor: It may be a strain, so you should be on anti inflammatory medication.if it does not help see your physician"
},
{
"id": 164175,
"tgt": "Is constant headache, extremely cold hands and feet after the scan serious?",
"src": "Patient: My grandson age 6 had a severe headache. He said it hurt on the top and the back of his head.He cried for almost three hours with it. Doctor ruled out sinus,flu and strep.Had a ct scan done. right before the scan he was lethargic ,sobbing and crying acout his head. As soon as scan was done he jumped up talking excesssively. When we returned home we noticed his hands and feet were extremely cold for several hours. today he seems fine.Any suggestions? Doctor: Hi...this sort of abnormal sudden behavioural changes and talking excessively can occur in - 1. Migraine with de javu phenomenon2. Frontal lobe tumoursI suggest based on the CT scan brain report you consult your pediatrician and if anything is abnormal he might require an MRI brain.Regards - Dr. Sumanth"
},
{
"id": 20202,
"tgt": "Suggest treatment for uncontrolled hypertension",
"src": "Patient: My mother just turned 90 and we usually monitor her blood pressure 2 - 3 times a week. She is currently taking LOSARTAN/HCTZ 50 12.5 mg tables (generic for HYZAAR 50). All day today she has felt \"off\". When I took her bp this evening it read 155 over 95. Second reading approx 45 minutes later read 168/98. What should we do now? Doctor: Hello, She should have low salt diet and monitor blood pressure regularly thrice a day for one week then once or twice a week and bp should not be persistently more than 140/90 mmhg even whatever below 150 is acceptable. We need to add one more medicine like Tab Amlodipine 5 mg once a day and monitor bp.Hope this helps you and get back if you have any doubts."
},
{
"id": 57249,
"tgt": "How is Hepatitis A transmitted?",
"src": "Patient: Dear Doctor, PLease advise if Hepatitis A is transmitted through vaginal sex. I surfed on net and found that oral contact with Anus of the infected person or anal sex or the infected person does not wash hand properly after stools is when you get infected . Please advise. Doctor: Hi,Hepatitis A is transmitted by feco-oral route only.Stop having anal sex. It can be dangerous.Regards"
},
{
"id": 192752,
"tgt": "How should a single rough red spot be treated?",
"src": "Patient: Me and my girlfriend had unprotected sex, she hadn't been checked and i dont know if she had anything. i stopped seeing her and about 2-3 months after i noticed a single red spot which had a sort of rough look. I try to shower as much as possible but some days i am far to busy to fit one in, sometimes i go 2-3 days not being able to have one. Could this be an std or is it more likely something else and how can i get rid of it? Doctor: Hello,The commonest cause of rough red spots are balanitis, cause due to poor hygiene generally in uncircumcised men. Take showers & wear cotton clothes. Clean the genital area & keep it dry. Fungus & infection spread faster in damp or moist skin. You can apply locally ointment hydrocortisone in the affected area. A course of antifungals or metronidazole can work well.However since you are sexually active. You should get clinically examined by a GP. Get STD tests done to stay on the safer side. If positive results are obtained. You will be treated accordingly.Hope I have answered your query. Let me know if I can assist you further. Regards, Dr. Nupur K, General & Family Physician"
},
{
"id": 190257,
"tgt": "Pain due to infected tooth, taking Amoxicillin. Any way to reduce infection?",
"src": "Patient: Hi, I have an infected lower left molar (2nd last tooth ). I was initially prescribed amoxicillin , but the pain continued to increase. On wednesday I was then prescribed metronidazole , and I am taking both at the same time. The pain re, Mains quite acute. I am seeing dentist this coming Wednesday for either extraction or a root canal . Is there any way to reduce the infection in addition to using the medication. Doctor: Hello Welcome to HCM I would like to tell you that you need to get your tooth treated as soon as possible because it might be a case of periapical abscess.Meanwhile you start with Tab Zerodol-SP thrice a day along with Tab Amclaid twice a day & multivitamin complex along with it. The best way to get rid of this pain is to get its treatment like Root canal or extraction done as soon as possible. Take Care Regards Dr.Neha"
},
{
"id": 217722,
"tgt": "Suggest treatment for upper back and chest pain",
"src": "Patient: Hi ive been experiencin occasional upper back pain and chest pain for months now..and i had my xray chest and spinal no abnormalities noted. My ecg reveals normal sinus ryhthm non specific st-twave changes Ive been diagnosed having UTI could this be related? Help! Im freakin out! All my doctors couldnt find any problem on me. Am i havin anxiety or panic disorder coz im really scared! :( i am 20 yrs old 107 lbs and 5 ft tall. Hope i can get some answers. Doctor: UTI with stone in the upper urinary tract may lead to such pains. You have not mentioned the siude of pain. This could be due to lower RTI or gall bladdet stones."
},
{
"id": 184769,
"tgt": "What causes white glue-type ooze from gums?",
"src": "Patient: My elderly aunt has had a white, glue type texture that pours from her gums and has been doing that for over 3 months. NO one has any ideas what is causing it. She rinses her mouth with hydrogen peroxide and nothing helps. Any ideas or thoughts would help. Doctor: Thanks for your query, i have gone through your query. The white glue type of fluid could be the pus discharge secondary to the gum infection. consult a oral physician and get your teeth examined and get your teeth cleaned. Maintain oral hygiene after getting your teeth cleaned by brushing your teeth twice daily and use mouth washes. i hope my answer will help you, take care."
},
{
"id": 142969,
"tgt": "What causes sensation of fluid in head and short term memory loss?",
"src": "Patient: I feel like I have fluid in my head and I am experiencing a constant brain fog. It seems to be affecting my short term memory call. This is something that is recent and it has been building up within the last year. I have a history of basic allergies, but this is a new sensation for me. Do you have any idea what this could be? Doctor: Hi, Welcome to HealthCareMagic.com I am Dr.J.Mariano Anto Bruno Mascarenhas. I have gone through your query with diligence and would like you to know that I am here to help you.Your conditions can be due to raised ICP. Please consult a Neurosurgeon for Clinical Examination and Fundus ExaminationHope you found the answer helpful.If you need any clarification / have doubts / have additional questions / have follow up questions, then please do not hesitate in asking again. I will be happy to answer your questions. In the future, for continuity of care, I encourage you to contact me directly in HealthCareMagic at http://bit.ly/askdrbruno Best Wishes for Speedy Recovery Let me know if I can assist you further.Take care."
},
{
"id": 128446,
"tgt": "Suggest alternative to pain killers for sciatic nerve pain in lumbar and piriformis area",
"src": "Patient: the sciatic that I have is caused from both piriformis and lower lumbar conditions...I have all the symptoms! and have been seeing doctors. I fear there is NOTHING that can be done for me, since I have had it for years,is there any hope? besides pain killers??? Doctor: Dear patient there is definitely hope. we need to confirm diagnosis first by Mri of the spine and hip. If diagnosis is sciatica due to disc prolapse surgery is the good option which can be done in endoscopic method with minimal incisions. And if piriformis is the cause of pain ultrasound guided incision of corticosteroids will help in pain relief. If that is not helping surgical release of piriformis with neurolysis of sciatic nerve will definitely help. please consult expert in hip and spine surgery and get yourself examined from them and discuss about this options."
},
{
"id": 153863,
"tgt": "Is there any chance of fertility issues due to thyroid cancer?",
"src": "Patient: Hi. I am twenty five. I have been trying to conceive for almost a year now with no luck. I have just recently been diagnosed with papillary thyroid cancer. They have removed the right side of my thyroid where the cancer was and I am returning for surgery again next week when they will remove the left side of my thyroid. They are then going to start me on 100mg of eltroxcin. I am wondering if my fertitilty issues had anything to do with having thyroid cancer or will my fertility chances improve once I am on eltoxcin? Doctor: Hi, dearI have gone through your question. I can understand your concern. You have thyroid cancer. Papillary carcinoma of thyroid will not affect the fertility directly. But you should check your T3, T4, & TSH level. If they are affected then if czn lead to infertility. So consult your doctor and take treatment accordingly. Hope I have answered your question, if you have doubt then I will be happy to answer. Thanks for using health care magic. Wish you a very good health."
},
{
"id": 160702,
"tgt": "How can severe reflux causing cyanosis and breathing trouble be treated?",
"src": "Patient: Hi I have a 6 year old with special needs who suffers from reflux and has been medicated successfully for this for about 3 years. However recently her symptoms have returned severly especially at night with her waking unable to swallow the reflux and turning blue with trouble breathing. Any ideas on what to do please? Doctor: Hi,It is commom for children with special needs to have this problem because of lack of proper coordination of breathing/swalloing muscles and poor tone esophgus. Usually this will respond to acid reducing agents like lansoprazol. Also, few non pharmacological will help- give dinner few hours before bedtime, make him sit upright for 30 minutes after a meal, increase consistency of diet to a more solid / semi solid type, keep the head end of the bed elevated to about 30 degree from horizontal, and avoid caffeinated things like tea / coffee / similar items.If these measures does not help, we rarely need a fundo-plication procedure after ruling out other causes like LPR (laryngo pharyngeal reflex) and pertusis. Your doctor will guide through this.Hope I have answered your question. Let me know if I can assist you further. Regards, Dr. Muhammed Aslam T. K., Pediatrician"
},
{
"id": 188383,
"tgt": "Wisdom tooth extracted, night sweats, cold, after antibiotic stopped, stiff neck, mood swings. Suggestions?",
"src": "Patient: HelloI had my lower left lower wisdom tooth extracted almost 2 weeks ago I have started these extreme night sweats while sleepingI feel fine... actually better than ever in 7+ years of letting the tooth rot in my head. Dumb I know...but since gotten a cold or became ill just after antibiotic have stoppedI'm worried...Have stiffness in my neckAnd mood swingsConstant drainage of left sinus Doctor: Hi, Your symptoms refer to common cold. Have more rest, plenty of fluids, especially water and tea. Take paracetamol or ibuprofen for neck stiffness. If no progress, get checked by your physician."
},
{
"id": 43091,
"tgt": "Should i try IUI again as i am not ovulating with clomid?",
"src": "Patient: Hello Doctor,I'm Banusha. TTC more than 2 yrs ready. I have went through 1 IVF & 1 IUI failed. April 2013, I went for Laproscopy for Endo and did tiub check & ovarian drilling. After lapro, doc gave me injection for hold menses for 3 mths then she induce period after 6 mths. Now, I'm still not ovulating with clomid. What am I suppose to do. Should I try IUI again? Doctor: Hi Banusha,Welcome to HCM.You can go for ovulation induction with IUI at a good center.Wish you good health."
},
{
"id": 196955,
"tgt": "Suggest treatment for low sperm count & motility",
"src": "Patient: hello doctor i am 27 year old i am married but do not have child trying to conceive for last 18 months my main problem is having sperm count 5 millions & motility rate 50% i am taking tab oilgocare adviced me for 3 months i want to know will i am able to make my wife pregnant in near future Doctor: HelloThanks for query .Based on the report of your semen analysis that you have posted you sperm count is 5 millions per ml suggestive of what is called as Oligozoospermia which is very low to result in to conception by natural means.Your Dr has prescribed medication (Oligocare) to be taken for three months .Truly speaking there are many drugs sold in pharmaceutical market and being prescribed by medical fraternity to increase sperm count.However the fact of the matter is that not even a single drug has been proved scientifically effective to prove their claim .Following measure if practiced religiously for 4-6 months will help to increase sperm count 1) Practice regular exercise for 45 minutes followed by meditation for 1/2 an hour in the morning. 2) Take high protein diet rich in vegetables and fruits and Vitamin A,C,D,E.and Zinc 3)Take anti oxidants like Almonds 5-6 everyday4) Avoid alcohol and smoking..Dr.Patil."
},
{
"id": 121561,
"tgt": "What causes severe fever,sweats and body aches?",
"src": "Patient: My husband suffered from a bout of flu commencing July 29th 2011 while in Rome at the start of our holiday, we had been there for 2 days. He had all flu like symptons and we presumed given a few weeks he would return to normal. This has not occured. Ongoing symptions include aches and pains in joints which seem to travel to different spots in his body but regularly in his hands. High temperatures and severe sweats. General tiredness and feeling unwell with the occasional bloating and stomach pains. Unable to perform a normal days activity and spends much of his day laying on a bed. He has had 4 separate sets of blood test, for varies virus including Ross River and Bahman Forest Virus. All blood tests have not provided any indication or resolution to the problem. He spend 2 nights in hospital this week with a suspected heart attack, but his heart is strong and blood pressure excellent. He is under our local GP however there is no help other than to say it takes time . His symptons of fevers, sweats, tiredness and aches seem to be worse now rather than better after nearly 6 months. Please help Doctor: Hello,The flu like feeling can be due to a post viral infection that affects the bones and joints. Rest and mild pain killers with a good vitamin C rich diet will help in his recovery.Hope I have answered your query. Let me know if I can assist you further. Regards, Dr. Praveen Tayal, Orthopaedic Surgeon"
},
{
"id": 32123,
"tgt": "Suggest remedy for difficulty in swallowing",
"src": "Patient: Hi I am a 25 year old healthy male, since monday 24 jan 2011 I can't swallow anything I find it really hard to drink aswell and each day it has got worse. Pain killers not helping at all and now I'm findind it very hard to swallow my serliver. My throat is not swollen on the out side but I can't open my mouth fully wide. I have not eaten In 4 days Doctor: Hi Dear,Welcome to HCM.Understanding your concern. As per your query you have difficulty in swallowing which could be due to stiffness of jaw joints as you not able to open mouth and it could be due to raised pressure in lower end of esophagus. Need not to panic much. I would suggest you to visit gastroenterologist once and get it examined. You should go for endoscopy as well. You can go for surgery of lower end of esophageal sphincter if pressure is really high. Till then you should maintain proper oral hygiene. Take antibiotic and do mouth stretching exercises. You should start treatment after proper prescription from doctor. Take diet rich in soft food and drink plenty of water and liquids. Do warm saline gargles. Hope your concern has been resolved.Get Well Soon.Best Wishes,Dr. Harry Maheshwari"
},
{
"id": 33684,
"tgt": "Should I try interferin and telaprevir to treat Hepatitis C?",
"src": "Patient: I have been diagnosed with Hep-c for Twenty years. Are there any new drugs that I can look forward trying as I have tried telepraver and RIBA virin and interferon and it nearly killed me. Thank You Doctor for your time. My e-mail is YYYY@YYYY Doctor: A new drug combination recently came out that does not require the use of interferon or ribavirin. It's once a day, minimal side effects, and has a very high cure rate. The only problem is that it's extremely expensive, and only approved for treatment of genotype 1. The combination is of 2 drugs called ledipsavir and sofosbuvir. The brand name of the combination is Harvoni. I believe there is another combination coming out soon or may actually be approved by now. But in any case, there are choices to anticipate very soon. Check with a liver specialist or the doctor who treated you before and get started on the improved meds. Hope this helps."
},
{
"id": 18596,
"tgt": "What causes chest discomfort while on Metoprolol?",
"src": "Patient: I been having issues with my bp. taking a new med and stopped on my own today cause it was making me feel like an elephant on my chest it is called metoprol. my bp today is running about 149/81 that s good today tho. can i stop taking the metoprol? Doctor: Hello and Welcome to \u2018Ask A Doctor\u2019 service. I have reviewed your query and here is my advice. I would like to tell you that your systolic blood pressure is on higher side which definitely needs medicine to treat. But if Metoprolol is not suiting you, then you can switch the medicine to another group of anti-hypertensive drug. Kindly stop Metoprolol and start with Telmesarten. Hope I have answered your query. Let me know if I can assist you further."
},
{
"id": 106814,
"tgt": "How can pain in the lower back and legs be treated?",
"src": "Patient: Denzel, a 46-year-old man, is working with his physical therapist on a set of back exercises. Around four weeks ago, Denzel was helping a friend move and attempted to lift a heavy box. Since that incident, he has had severe pain in his lower back that extends into his lower leg. His health care provider prescribed physical therapy three times per week to treat the condition. Doctor: Hello,You have got acute back sprain with radicular pain to.lower limb. This has happened due to lifting the weight with a jerk to back. You need to take rest and stop lifting weight for a week. Avoid forward bending also.Start tablet Myoril twice a day and tablet Ultracet twice a day for 5 days, along with tablet Pregalin X one at bedtime. Myoril is a muscle relaxant, and tramadol drug helps in pain relief. Pregabalin is for the relief of lower limb pain. A visit to the orthopedic specialist is also recommended.Hope I have answered your query. Let me know if I can assist you further.Regards,Dr. Jayesh Vaza"
},
{
"id": 27354,
"tgt": "What could cause chest pain?",
"src": "Patient: I am experiencing a pain in my chest, like a burning stabbing pain, that radiates up my throat to my right ear. my stomach and enitre mid section is swollen also. Celiac, and Hpylorid negative. Seems like my breast swell also when this happens. The other day my right hand was swollen as well. Doctor: Hello. Thank you for your question and welcome to HCM. I carefully read your query. With the information you provided, I would say that we are facing a problem with your stomach. The chest discomfort, as you describe it, is called pyrosis, and is the burning-like sensation in chest which is produced by the returning of the highly acid stomach fluid into the oesophagus. This situation can be produced even if Helicobacter pylori is negative (it is present only in 30% of American population). Hereby, I would recommend you to follow the measures stated below. First, you have to lie down at least 90 minutes after you have had your last meal (the average time that stomach takes to \"push\" its content to the small intestine). Second, I would recommend refraining from some food and beverages that produce more acid and, thereby, irritate the gastric inner lining (mucosa), such as: - coffee, tea (xanthyne products)- chocolate- smoking and alcohol- pickles, grilled and spicy foodThird, I would recommend you to start on a course with a protein pump inhibitor (PPI) drug. For instance, omeprazole 20 mg x 2 tb/d for the first two weeks and 20mg x 1 tb/d for two other weeks. After all these measures are undertaken by you, your symptoms should subside. If they remain present, I would recommend you to perform a upper gastro-intestinal (GI) endoscopy. I hope I was helpful. Take care and feel free to ask again."
},
{
"id": 120649,
"tgt": "Suggest remedy for pain and swelling in the tailbone area",
"src": "Patient: I fell off of a skateboard and landed harshly on my tailbone on cement. I am experiencing pain and swelling in the tailbone area as well as a tingling discomfort in my genitals. Is this the after effects from the fall that in time will go away or should I go to the doctor. Doctor: Hello,I read carefully your query and understand your concern. Your symptoms seem to be related to a the injury after the fall.Commonly it is treated with:-cold compress or ice pack application for 20 to 30 minutes for\u00a0the\u00a048 hours following injury to reduce swelling. -rest. anti inflammatory medications such as Ibuprofen to relieve the pain. Hope my answer was helpful.If you have further queries feel free to contact me again.Kind regards! Dr.Dorina Gurabardhi General &Family Physician"
},
{
"id": 18829,
"tgt": "Suggest high BP medication that does not cause migraine headache",
"src": "Patient: Is there a drug for hypertension that does not cause migraine headaches (triedLisinopril and systolic) or diarrhea (tried benicar) or Reynard ( tried benicar and bystolic). Also, I work outside, am slim, and can t take a diuretic because of excessive sweating. Doctor: Hello,You can safely take calcium channel blockers (CCBs) for a migraine headache and high blood pressure (hypertension). CCBs relax blood vessels and help to control hypertension. These group of medicines is also effectively used for the prevention of migraine headaches. Please, ask your Doctor to prescribe CCBs (verapamil) for hypertension and migraine headaches.Hope I have answered your query. Let me know if I can assist you further.Regards, \u00a0\u00a0\u00a0\u00a0\u00a0 Dr. Malik"
},
{
"id": 101521,
"tgt": "What causes burning & itching sensation in the back?",
"src": "Patient: Arjita Mitra, F 20 years suffered fro read spot like irruption at a small portion of back with burning & itching sensation 3 days before.Applied herpex ointment but no improvement or detoriation. Today same symptom is found at left hand. please advice. Doctor: HiWith you query, it Mau be a viral once infection like chicken pox do you have any other symptoms like fever, body pains them it could be this. Kindly visit your doctor"
},
{
"id": 120245,
"tgt": "Suggest remedy for broken stitch",
"src": "Patient: hey my name is david i broke my ulna and radius bones mid-shaft and had surgery 1 week ago. They put 2 metal plates and 12 screws in my arm. I think one of my stiches broke open last night but can t tell because i m in a splint right now. It is wednesday today and my next check up is Friday. please help Doctor: Hello, If the wounds are not wide apart, nothing much to worry and it will heal by itself. If the wounds are gaped, taping with steristrips must be required. Hope I have answered your query. Let me know if I can assist you further. Take care Regards, Dr. Shinas Hussain"
},
{
"id": 130678,
"tgt": "What could be the reason for having sharp prick like pains in the palms of my hands and soles of my feet?",
"src": "Patient: I am having sharp, prick-like pains in the palms of my hands and the soles of my feet (arches). The pain started a few months ago in my hands and was occasional. Now the pain is constant in both my hands and feet and occasionally in my arms and/or legs. Doctor: Hi,As your symptoms are constantly occuring, as for now I would like to ask your age and a few more other questions. Are you a strict vegetarian? Do you drink alcohol or have any thing like diabetes or hypertension (high bp)? I am asking this because it would help me in knowing or ruling out the exact or common causes of these complaints. As for now I would suggest you to get your sugar levels checked both fasting and post prandial. And would suggest you to take Vit B12 and Vit B6 containing tablets. Neurobion forte injection would help. If any reports from the above suggested ones are abnormal, we would go accordingly.Hope I have answered your query. Let me know if I can assist you further.Regards, Dr. Mohammed Athar"
},
{
"id": 124308,
"tgt": "What is causing burning or cramping in the pelvis?",
"src": "Patient: hello,my husband and I are currently trying for a baby. I am now at day 21 of my cycle and have developed persistant burning/cramping low down in my pelvis. There is no discharge or vaginal pain, its all in the pelvis. I have been taking paracetamol but they have not taking the edge off and the pain is constant. I have been to see my GP this morning who performed an internal examination and could not find anything obviously wrong. I have also had a blood test this morning to see if I am ovulating/pregnant as I also have suspected endomitreocis which I am seeing a specialist about later this month. Can you shed any light on what this pain might be? If it turns out that I am pregnant, does this suggest that there is something wrong and that I am likely to misscarry? Doctor: Hi, As what I can understand from the history there might be a chance of UTI. This is one of the common which most females face. As there is always a need of being hygienic before any sexual activity. Usually people are conscious about being hygienic but sometimes it doesn't happen and leads to UTI. The burning sensation is due to UTI. Also, there will be a weakness of the Pelvic floor muscles. Which is also common in most females. One needs to do simple exercise to the Pelvic floor muscles. There are a bunch of pelvic floor exercises which you can try out. But the most important ones will be the Kegels exercise. It's mainly used for urinary incontinence but it also has other benefits like strengthening the pelvic floor muscles. You mentioned about the blood test? But did you try to get the urinary test done once. As sometimes it gives more inputs. I wish you good luck for becoming mother soon. Hope I have answered your query. Let me know if I can assist you further. Regards, Jay Indravadan Patel, Physical Therapist or Physiotherapist"
},
{
"id": 201393,
"tgt": "How to cure a painful lump on scrotum?",
"src": "Patient: I have a painful lump on my scrotum, just at the base of my penis. I think its an ingrown hair or a large pimple, but im not 100% positive. Is there something more sinister that I should worry about? There are no other symptoms present. Im 29 years old and married, with no reason to suspect an std. I dont have a picture to upload, but it just looks like a red lump roughly the size of a kidney bean. it is freely moveable, and it travels from the top of the scrotum to the bottom of the base of my penis when im erect. Its only particularly painful when I poke or squeeze it. I have no reason to suspect an STD, and Im 29 years old with no other health concerns, sans being overweight. Doctor: HelloThanks for your query,based on the facts that you have posted it appears that you have what is called as Sebaceous Cyst over base of the penis.Scrotal skin is rich in sebaceous glands and hence prone to get more sebaceous cyst due to accumulation of sebum beneath the skin.Normally they fade away without treatment however they need to be treated if increase in size or get infected.Please consult qualified General Surgeon for clinical evaluation and further treatment.If needed some of them which are bigger may need to get excised in Toto (Completely along with the sac)Dr.Patil."
},
{
"id": 125752,
"tgt": "Is sharp pain in the shoulder a symptom of bursitis?",
"src": "Patient: Hi. I have fallen a few times, tripping etc. I have landed on my hands, and hurt my collar bones, felt like greenstick fractures. I never could afford to go to the doctor. I fell again when I stepped in an overgrown hole with both feet, spraining both as well. I fell on my left arm and dislocated my left elbow at a 45 degree angle. I had therapy etc, but the pain of the elbow masked the possible damage to my shoulder.It feels slightly loose in the socket. when my arm hangs down it feels like it shifts and it aches. The other night I rolled over on my left shoulder, arm folded up...and I heard a pop, and felt a sharp pain in my shoulder socket. it s been very sore, and I can barely move my left arm, though heat seems to help. Do you think it might be bursitis? Or something worse. Doctor: Hello, As of now, you can use analgesics/anti-inflammatory combination like Aceclofenac/Serratiopeptidase for symptomatic relief. Most probably it will be a musculoskeletal pain. If symptoms persist you can consult an orthopedician and plan for an MRI scan. Hope I have answered your query. Let me know if I can assist you further. Regards, Dr. Shinas Hussain, General & Family Physician"
},
{
"id": 146676,
"tgt": "Is microvascular ischemic white matter disease dangerous?",
"src": "Patient: I did a MRI and the results is: There are scattered nonspecific of T2 prolongation in the sub cortical white matter representing microvascular ischemic white matter disease. Few scattered nonspecific foci of T2 prolongation in the white most like representing microvascular ischemic white matter disease given the patient s age Please, should I see a doctor, what kind? This is danger? Thanks Doctor: Please mention age of patient and any history of vascular risk factors like hypertension , diabetes. Considering MRI ffindings we should start tab aspirin and tab atirvastatin in adequate doses. Symptoms will depend on vascular ischemic load which will progress gradually. Control hypertension and diabetes if present.Hope my advice will help you. Take care. Don't forget to rate me."
},
{
"id": 47949,
"tgt": "Suggest treatment for chronic kidney disease",
"src": "Patient: goodday my wife has been suffering from renal medical disease grade III. Her serum creatinin level is 2.2 at present. She is on medication as of now. I wnated to know the severity of the disease and precautions to be taken to avoid escalation/degradation. Doctor: Hello and welcome to HCM.As an Urologist, i must advise you to send a copy of the latest reports of urea, uric acid, Potassium, Sodium,Hb, HbA1C, (if diabetic),urine routine, urine micro-albumin, 24 hours urine protein, and an ultrasound scan of the abdomen. kindly write her age, duration of illness, and send a copy of present medication.You can get an expert opinion on the reports,and precautuions, by sending it here, in my name.Wishing her well."
},
{
"id": 223819,
"tgt": "Can contraceptives cause bleeding?",
"src": "Patient: I had an emergency contraceptive for the first time around a week back. Since yesterday, i have been bleeding black blood. I thought it must be my periods, but I m yet to get my periods for another week technically. The bleeding is much lesser than my regular periods and there is not much pain and cramps. I m seriously worried. Can it be a side effect of the contraceptive? or is it something else? please help. Doctor: Hi dear common side effects of emergency contraception is bleeding,cramps, weakness, gastric troubles so that could be the possibility for that use Tab ibuprofen and iron supplements..also keep your self hydrated and take balance diet with green vegetables and fruits..I also advise you do urinary pregnancy test once you miss your periods by more than 2 weeks..as pills give 95% protection..thanks for writinghope this will help youregardsDr.Sohil Takodara"
},
{
"id": 198616,
"tgt": "Suugest treatment for haemmoroids",
"src": "Patient: i have haemmoroids and have been using anusol cream for the past few days. unfortunately they have not shrunk and i feel the constaint need to use the toilet only to find that i can not pass anything. iam in pain all the time which seems to radiate into my scrotum Doctor: Thanks for asking Healthcaremagic. Hemorrhoids occur when the veins of the rectum or anus become inflamed because of too much straining in the area. I would suggest:1. Take NSAID like ibuprofen and paracetamol for pain.2. Apply ice several times a day for 10 minutes at a time. Follow this by placing a warm compress on the anal area for another 10 to 20 minutes.3. Take a sitz bath. Fill your bathtub with just enough warm water to cover the anal area. Do this several times a day, especially after you have had a bowel movement. Soak for about 15 minutes at a time. Be careful! If the water is too warm, it can burn you.4. Eat enough of Salads and fruits along with laxative syrups if needed and avoid straining.If you still feel no relief, you should visit your doctor and get yourself examined. i hope this helps. Thanks and Regards."
},
{
"id": 223583,
"tgt": "Should I be worried about a light colored bleeding after taking i-pill?",
"src": "Patient: Hi, i had sex with my husband ..we used precaution...but when i checked it was torn off...inside...so i was bit scared as i dnt want to get pregnant right now..i took,i-pill..on the same day..and today..its been 7days m hving bleeding...and its very light colored red..,nt like usual periods...m really tensed..till how many days i will get this bleeding Doctor: Hi,Do not worry. You took I pill on same day and it is highly effective in preventing pregnancy. It has just 0-5% failure rate. But it can cause hormonal imbalance as it contain high progesterone. It cause withdrawal bleeding in most off cases after 7 days. It will go by itself with in 3-5 days. Avoid stress, take healthy diet, drink plenty of water, maintain proper pelvic hygiene and do regular exercise. Hope this may help you. Contact further if follow up needed.Best regards,Dr. Sagar"
},
{
"id": 199664,
"tgt": "What causes swelling in the foreskin which could be due to masturbation?",
"src": "Patient: Hi. I have had a swollen foreskin for two days with no other symtoms, which could have been brought on by masturbation...This morning however as the swelling has started to go down I noticed two small (roughly 2mm - 3mm in diameter) lesions under the forskin which for obvious reasons has me concerned. Do you think it is something be be concerned about? there is no pain or itchiness involved. Doctor: HelloYou have developed what is termed as Paraphimosis due to rough and vigorous shakin and tight grip of of your penis while masturbating.which has resulted into small abrasions under foreskin.You need to take broad spectrum antibiotic like Cefixime along with anti inflammatory drug like Diclofenac twice daily.along with topical antibiotic ointment like Neosporin twice daily.Ensure to wash your genitals with warm water twice daily.Dr.Patil."
},
{
"id": 182896,
"tgt": "Suggest remedy for oversensitivity",
"src": "Patient: Hello Took me many eyars to go bakc to the dentist. I did 3 years ago w/sedation. Had a deep cleaning and a tooth pulled. My teeth hurt, but I expected that because it was under the gum work and so on. Since then I have gone back every 6 months for a cleaning. I still have terrible pain in my guns and teeth for days, sometimes a week. I can t get releif. My dentist thinks I am exagerrating. I am supposed to go next week but now.........I just can t go through that again. What is wrong? If I am oversenstive, what can I do? Doctor: HiThanks for writing in.Due to deep cleaning you might have been experiencing pain & it will subside with in few days.You just need to take few measures:--use desensitizing tooth paste containing strontium chloride-do gum massage by gum tone gel twice a day-maintain proper oral hygiene by regular brushing & flossing.Visit your dentist if problem persists.RegardsDr. Neha Sumra"
},
{
"id": 39109,
"tgt": "Suggest treatment for mono disease",
"src": "Patient: No serious pain when swollowing, but since about April I have had little like, red vein looking things (1 or 2 purple) in my throat as well as the skin around my tonsils. I do have allergies and drainage at night. Also found was quite sick at the beginning of the year, and later found that my issues were from an undiagnosed case of mono. Not knowing I had mono, I continued to drink alcohol at the time. I'm a normally healthy 28 year old female, before all this only got sick maybe twice a year Doctor: Hello,Welcome to HCM,Infectious mononucleosis (mono) is a contagious illness caused by the Epstein-Barr virus (EBV).The infection can be spread by saliva.Treatment is directed toward the relief of symptoms.1.Antiviral drugs have no significant effect on the overall outcome of mono and may actually prolong the course of the illness.2.Most of the times strep throat occurs in conjunction with mono and is best treated with penicillin or erythromycin.3.Acetaminophen can be given for fever and any headache or body aches.Thank you."
},
{
"id": 9457,
"tgt": "What is the treatment for itching?",
"src": "Patient: I have one foot that is very dry, itchy. Some times i end up peeling my skin and it starts to bleed, sometimes i peel a bump or two that have clear like subtance that comes out. I also have itchy toes which i was told it was chiggers, not sure if the chiggers clear or itchness is part of what i have on the bottom of my foot. The strange part is i have it bad on one side of my foot and the other side is just my toes. Doctor: Hello and welcome to healthcaremagicDry scaly skin on the feet associated with itching, suggests to me that it could be a fungal infection of the feet(Tinea pedis).I would ask you get a KOH wet mount from the scale, for a confirmatory diagnosis.If KOH wet mount shows fungal elements, then you could apply a topical OTC antifungal cream e.g clotrimazole cream, twice daily for 4-6 weeks.An oral antifungal e.g fluconazole 150mg once weekly for 4-6 weeks is also advisable for a confirmed fungal infection.An OTC antihistamine e,g cetrizine 10mg once daily would help you in providing symptomatic relief from itching.regards"
},
{
"id": 207354,
"tgt": "Suggest remedy for mental health problem",
"src": "Patient: need advice about an injury to head I got hit on my head right above the frontal lobe region by an iron pipe. I suffered concussion for over 3 days and many other symptoms like headache, nausea, tremors, dizziness, imbalance, blurring of vision etc. This happened in 2006. I was taking nootropics at the time. I want to know if this can cause Aboulia. :( I am showing all symptoms of Aboulia and my life has gotten worse after this incident. I was suffer from depression and borderline personality disorder but what I am feeling now after the incident is totally different from before. Even when depressed I never used to be inactive. But now I have totally turned into a doll just sitting there quietly. Please help. If it is Aboulia, will it be treated? What should I do to get well? Doctor: HiThanks for using healthcare magicIn that case, better to take help of a psychiatrist and psychologist. That would help you to come out of the situation. You can also try some antidepressant with relaxation exercise. It would help to control these depressive thoughts. In case, you need further help, you can ask.Thanks"
},
{
"id": 216956,
"tgt": "Suggest remedy for severe back pain and pain and cramping in left leg",
"src": "Patient: I have been diagnosed with a lumbar synovial cyst, slight scoliosis, and the start of degenerative disk disease. I have been having horrible pain in my back that has been causing pain and cramping in my left leg for over 3 weeks now. I have tried taking muscle relaxants, pain meds, anti inflammatory, heat, ice, and stretches with no relief. I need to get this taken care of as I cannot continue to keep taking meds as a cover up. Doctor: Hi there . thanks for your question at HCM.Lumbar synovial cysts are relatively uncommon cause of scitaica kind of pain. The cysta are usually associated with Degeneration if joints in your back, known as facet joints.Rest, pain killer are the first line of therapy. If you don't have significant relief in pain faxte joint injection and epidural injections can be tried based on your MRI findings. The last step in the ladder would be a decompression surgery or fusion surgery again depending on yiu r radiology and pain pattern. As if now it is best for you to try pain medication for 6 weeks and we how you do.Hope this helps.All the best. Regards. Dr.SBK"
},
{
"id": 1414,
"tgt": "What is good day after periods to conceive?",
"src": "Patient: Yes...I am 24 years of age and Me and my husband are trying to have a baby...we have been trying for over 8months now and we are starting to wonder...I have been keeping up with my period cycle which is in between 28-32 days. I started last month (Feb) on the 11th and ended on the 17th.....if it was 28 days then I would have started on the 17th of march...and if it was on a 32 day then I would have started today the 21st so is this normal? what should I do? Doctor: Hi , How are you doing ? Let me give you a list of investigations which can find if any hindrances Husband- Semen analysis at least 2- done 4 weeks apart preferably in an infertility center ( examination done by andrologist), Blood sugarsWife - Blood Sugars, Thyroid profile Ultrasound scans to evaluate Uterus , Ovaries & adnexa Confirm tubal patency- most important- Tube is the connection between uterus & ovaries( the bridge where sperm meets the egg & forms a baby) this is size of hair follicle & cannot be seen on scans. So check with HSG( Xray with dye), or SSG ( Scan with dye) or LaparoscopyIf all this done & found normal , nothing can stop a pregnancy with God's willFor the best fertility period. Have sex daily or alternate days once your bleeding stops. This is to clear the old sperms. (1st day of bleeding is first day of periods) , stop sex on day 10, No sex for 2-3days then again have sex from day 13 for next 5 days. This is time of ovulation. This will definitely get more healthy & young sperms to reach egg. Older sperms will cause lower pregnancy rate & unhealthy pregnancy.Hope I have clarified your queries All the bestDr.Balakrishnan"
},
{
"id": 218399,
"tgt": "Can PCOS cause problems during pregnancy?",
"src": "Patient: hello i was recently diagnosed with PCOS and i have been having some cramping so i went to the hospital and they told me i am like 5 days pregnant.My husband and i are extremely happy but with my condition i am kinda scared. Am i at more of a risk for a miscarriage? Doctor: Hello and Welcome to \u2018Ask A Doctor\u2019 service. I have reviewed your query and here is my advice. 1) PCOS is not a cause for miscarriage. Don't worry. Hope I have answered your query. Let me know if I can assist you further."
},
{
"id": 173538,
"tgt": "What do white patches around the chest and stomach indicate?",
"src": "Patient: My 3 month old daughter has developed some white patches around the lower area of her chest and on tummy from 2 days back , also she has small patches on her right cheek........... Kindly let me know the reason for the same and what has to be done so that it can bve cured Doctor: Hi,Thanks and welcome to healthcare magic.It is difficult to diagnose without seeing the baby.The white spots may be due to vitamin B complex deficiency or it may be fungal infection.Give multivitamin drops daily.You may try local application of clotrimazole ointment for a few daysIf no improvement you consult pediatrician or dermatologist .Hope this reply is OK for you.Please feel free to ask further queries if any.Dr.M.V.Subrahmanyam."
},
{
"id": 103634,
"tgt": "Itchy rashes on the thighs and arms. Pain in the ankles and wrists. Took benadryl. Allergic reaction?",
"src": "Patient: My daughter is 16, she was laying in the grass with shorts on and about an hour later developed a rash on her thighs and arms (looks like an allergic reaction rash) that itches and burns. She washed her thighs with technu extreme medicated scrub and I gave her one tablet of Benadryl and the rash redness seemed to subside but she said she was feeling pain in her ankle and wrist joints. I asked if she had been in the woods, and she was yesterday. She has no rash on her stomach, and the joint pain just started after this rash occurred from being in the grass. Doctor: hi, your problem is the ankle and wrist pain is from allergic reaction ? no this one is not from allergic reaction, is may be due to any other reason, like any injury may be lite, muscular sprain by any reason like fast running etc,. and the skin problem of allergic reaction it will subside within short time. by the medicine which you have been taken so don't worry. and if the pain is more then taken painkiller O.T.C. product. or consult the orthopedic doctor.get well soon."
},
{
"id": 2575,
"tgt": "What is the success rate of iui?",
"src": "Patient: Hi I m aarti n my age is thirty five I hv to come to india to do treatmrnt for my baby my husbanD stays in jakarta sowr deposited his sperm counts r not so gud my husband cnt try intercourse at all nd he is diabetic I hv tried many iuis but it is coming failure I cnt afford ivf cn I get a success frm iui pls advice my reports r ok Dr told Doctor: Hi,It is common to get failed in initial trails with IUI. Try one more time you may get pregnant. Before IUI get thorough examination of your body like all the possible tests that prevent pregnancy from your side like patency of Fallopian tubes and other hormonal imbalances. Hysterosalpingography is the test to know the patency of the tubes. If the tubes are blocked IUI will fail.Hope I have answered your query. Let me know if I can assist you further. Regards,Dr. Vidya Jyothi B"
},
{
"id": 142334,
"tgt": "What is cause for seizures, headaches and black out?",
"src": "Patient: whats wrongwhat is wrong when you have what seems like a seizure and almost like your body was trying to have a coma state seizure? since then im constantly having headache, feeling tired off and on through the day, and off and on feeling like im going to black out. Doctor: Hi, Welcome to HealthCareMagic.com I am Dr.J.Mariano Anto Bruno Mascarenhas. I have gone through your query with diligence and would like you to know that I am here to help you.Your Complaints and Narration of History is suggestive of Increased Intra Cranial Tension This needs to be evaluated further. You need to undergo Clinical Examination and Fundus ExaminationPlease consult a Neurosurgeon near your place for further guidanceHope you found the answer helpful.If you need any clarification / have doubts / have additional questions / have follow up questions, then please do not hesitate in asking again. I will be happy to answer your questions. In the future, for continuity of care, I encourage you to contact me directly in HealthCareMagic at http://bit.ly/askdrbruno Best Wishes for Speedy Recovery Let me know if I can assist you further.Take care."
},
{
"id": 150915,
"tgt": "Brain hemorrhage, weakness, performed EVD. What treatment further?",
"src": "Patient: hi , i want an advise. My dad fell in the bathroom on 30th Nov 2012 and was hospitalized . he was diagnosed for Brain Hemorrhage and right side weakness. we performed EVD extra ventricular drain to drain the excess fluid . according to the doc my dad was not stable enough for a shunt . he was in the hospital for 36 days and the docs requested to take him home . now he has been discharged he is awake but no movement as such .. can you advise what treatment should we go for Doctor: Hi, Thank you for posting your q Urey. At this stage, your dad requires regular exercises under the guidance of a physiotherapist. He should be made to sit, stand and walk with support. He would also benefit from cognitive stimulation. Family members should talk to him, tell him about various events in the family, etc. He should be also made to listen to songs, bhajans, etc. In addition, you should take care of his food and nutrition, with the help of a dietician. Best wishes, Dr Sudhir Kumar MD DM (Neurology) Senior Consultant Neurologist"
},
{
"id": 144898,
"tgt": "How can an occipital ache with bruising due to trauma be treated?",
"src": "Patient: Hi, earlier i was going to the cupboard to get breakfast and then when i turned around, i supposedly blacked out/fainted and fell to the floor hitting my head. I mainly hit the bone above my eye and there is a little bruise nothing else, i sat with an ice pack on it for a while. My mother thinks its because i hadn t eaten anything for about 20 hours and thats why i blacked out because i had no energy. But now i have an ache in the back left of my head and i don t want to go to the doctors/hospital and im very afraid of them, please help. Doctor: Since you have fallen on your forehead, it is likely that the force might have been transmitted to your neck as well as back side of the brain. Occipital headache can arise in both. This needs to be sorted out. For the time being you can try soft cervical collar and voveran 75 mg twice daily after food. But I suggest you should see a doctor. There is nothing to be afraid of doctors. Most are good people."
},
{
"id": 172100,
"tgt": "What causes foul smell from armpits?",
"src": "Patient: Hi, my daughter recently had a eczema breakout and some kind of heat rash also. I took her to the doctor and he gave her a steroid shot and prescribed the medicine Prednisone to take as well. Anyway after the shot her armpits started having a bad odor to them and I want to know how long will this last. It does not matter how many times she puts deodorant under her armpits it doesn t last long and the odor comes back. What can I do about this? She is only 10 and it makes her feel bad. Doctor: Hi...sometimes when eczema has superimposed ichthyosis this can happen. Not only that as she's 10 years old, she might be entering into adolescent phase of glandular development where in the odoriferous sebaceous glands start developing. This might be the reason for the bad odour. The only way to treat eczema is to keep the areas moist and not letting dry.Regards - Dr. Sumanth"
},
{
"id": 173566,
"tgt": "What causes redness around the mouth while feeding cerelac for the 4 month old?",
"src": "Patient: My baby is 4 month n 1 weak old . I started giving her nestle cerelac wheat n milk stage 1frm yesterday. And i noticed that the area around her mouth which come in contact with cerelac while feeding is becoming red. And is disappearing after some time. So shud i continue feeding or stop for a while? Doctor: Hi...by what you quote this looks like a food allergy to some ingredient in the cerelac you are using. I suggest you stop giving it. Rather than artificial preparations like this I suggest you give only exclusive breast feeding till 6 months of age. That's what even the World Health Organization (WHO), recommends. So give only exclusive breast feeding till 6 months age and then we can decide further.Regards - Dr. Sumanth"
},
{
"id": 213553,
"tgt": "Suffered from malaria, not interested in studies, not able to concentrate. Suggestion?",
"src": "Patient: i cannot put intrest in studies and i dont have sleep and also i dont have intrest to go to college because i cont understand class,and my mind is full of absence.this type of things happend from three months from joining in new college aftre diploma into btech.before this i am a well student regarding my studies and all other activities. 15 days ago i have suffered from maleriya and taking treatment also. Doctor: Hello Your problem seems to associated with difficulty to adjust to new environment. This is a usual problem so dont worry at all. make new friends with whom you can share and enjoy your college days. Try to be attentive in classes and ask your doubts to the teachers.this will even help in getting closer to teachers, which will booost your confidence. you always keep in mind that you are a good student and you can do well in your studies. Involve in other activities of college like sports, entertainment,etc. these will help you in adjusting to the new circumstances I am sure you are going to come up with flying colors in future. take care. Thank you"
},
{
"id": 221527,
"tgt": "Suggest treatment for morning sickness during pregnancy",
"src": "Patient: Hi Doctor, My name is Divya. age 25. I am a pregnant & now in 12th week. i am still suffering from morning sickness & vomiting. i feel so tired. but dont feel hungry. if i have anything suddenly i feel like vomiting. pls suggest how to avoid this? and my weight is 60 kg now. i feel that i am over weight, is it the right weight? what should i do to get the normal delivery. pls suggest. Doctor: HiDr. Purushottam welcomes you to HCM virtual clinic!Thanks for consulting at my virtual clinic. I have carefully gone through your case, and I think I have understood your concern. I will try to address your medical concerns and would suggest you the best of the available treatment options.1]Firstly do not panic.2] Usually morning sickness should disappear by 14 weeks.3] i will suggest to have small frequent meals.4]I will suggest indulging in a healthy diet and regular exercise regime.Include plenty of fruits, salads, vegetables in the diet.Avoid deep fried foods, bakery products, and refined sugars.5] Tab DOXINATE twice a day 30 minutes before meals will help you get rid off morning sickness.6] Please find approximate ideal weight as height in cm minus 150,in kilo.7] Just enjoy your pregnancy at present, do not worry about mode of delivery.I hope my answer helps you.Thanks.Wish you great health."
},
{
"id": 25417,
"tgt": "Repeated syncope after by-pass surgery",
"src": "Patient: Dear Doctor, My father (63) had by-pass surgery in 2011 and off late we noticed repeated syncope. After ruling out cardio-related problems we switched now to a potential Neuro issue. EEG was found to be normal. Doppler Carotid was conducted yesterday. It had following statements and along with his other medicines, he has been advised to take Levipil 500 twice a day: 1. Evidence of calcified atheromatous plaque in left distal common carotid artery. Bulb diameter reduction of around 45%. Atheromatous plaque with similar characteristics on the right 2. Normal Flow Velocity and Flow Patterns. No Flow reversal or Disturbance 3. No significant hemodynamic stenosis on both the sides (left and right). Await your advise. Doctor: Hello and thank you for using HCM.I carefully read your question and I will try to explain you something and give you my opinion.When we deal with a syncope we have to evaluate cardiac and neurological problems.According to cardiac issues. There are different cardiac reasons that might cause a syncope. The most important one are rhythm issues like atrio-ventricular block, severe bradicardia, long heart pauses, arrhythmia.All sow it is quite known in cardiology that sometimes several people do heart blocks after cardiac surgeries.To exclude cardiac factors it is mandatory to do some examinations.It is very important to verify in what situations does the syncope happen each time, in what situations.So, if I was your treating doctor i will recommend cardiac echo-cardiography and the most important a holter rhythm monitoring for minimally 48 h. This rises the possibility to catch any problem. You must know that the rhythm monitoring is random, it might happen that you do it but it doesn't happens anything with your rhythm during that day and so we don't catch anything, So it is mandatory to take it as long as possible and repeat it.According to the Carotid result, it doesn't explain the syncope.So my opinion is to continue and do holter rhythm monitoring and if no result, maybe a head CT scan.Hope I was helpful. Wish your father good health.Best regards."
},
{
"id": 60182,
"tgt": "Encopresis, low self esteem, premature liver, any treatment available ?",
"src": "Patient: my grandson is eight years old,has encopresis. He has a big heart and puts others ahead of his self. When there is conflict with other childern he often goes and hinds. I can tell that his self esteem is very low and he has several acciedents daily.When born he had a premature liver and went home on a light with a home nurse comming daily for 2months. As I said he is eight do you have any positive suggestions that might help? Doctor: Hello, Welcome to Healthcare Magic forum. Your grandson is suffering from encopresis and hence may have related psychological problems like low self esteem which is commonly seen in about 20% of children with encopresis. Also,Encopresis can develop from a range of causes apart from medical reasons including simple phobia, poor toilet training experience, painful constipation experience, social immaturity, fear of getting older, as an attempt to regain/exert control etc. Untreated encopresis can pose several risks to a child's health: megacolon- enlarged colon, desensitisation of nerves and muscles, bleeding and cracking of the skin fissures, urinary tract infections and wetting accidents. The psychological concerns include: shame, guilt, loss of self esteem, social ostracism by peers, delayed social development, withdrawal, anxiety and hopelessness. Parents often also feel a range of emotions when their child has encopresis including feelings of embarrassment, and fears of incompetence as a parent. The most important thing needed for your grandson is accurate medical assessment. Once all possible medical avenues have been exhausted psychological causes and maintenance factors need to be explored. Generally, the earlier treatment begins for encopresis, the better. The first step in treatment focuses on clearing the colon of retained, impacted stool. After that, treatment includes encouraging healthy bowel movements. This includes training your child to go to the toilet as soon as reasonably able when the urge to defecate occurs. in cases of encopresis without constipation and simple toileting refusal psychological treatments are often very simple and effective. Psychological assessment will involve a detailed history of development, absences from parents, history of increasing independence, toilet training history and the child's general behaviour . Once the psychological assessment is complete it may be that toileting refusal is not the major concern, there maybe difficulties in the parent child relationship or frequently the child may have low self esteem and poor self confidence. Psychological treatments for encopresis generally tackle a number of areas of concern at once. They usually work on improving the parent child relationship, improving the child's self esteem including their sense of accomplishments and independence. So I would advice you to get your grandson first evaluated by a pediatric gastroenterologist to rule out any medical causes and complications and with his recommendation a psychotherapist can be consulted for appropriate psychotherapy. Please do not worry as with appropriate therapy your grandson can grow into a normal self esteemed individual. Wishing him a speedy recovery and good health forever. Regards"
},
{
"id": 21514,
"tgt": "Is calf pain associated to Cardiac catheterization?",
"src": "Patient: Hi, may I answer your health queries right now ? Please type your query here...Post heart cat my mother was put on lipitor80mg. In therapy while ambulating had left calf pain. Doppler was neg. Thought maybe the lipitor but has been off for three weeks still has tight muscle and calf pain post walking 100 feet . Is there a know calf pain associated with the cat. The cath was in the opposite Doctor: It is unlikely that the calf pain is related to the catheterisation especially since it is in the opposite leg.High dose of statins (Lipitor) could cause muscle pain in some people.The other possibility is peripheral vascular disease given that the pain comes after walking for some time. ( in medical terms known as claudication distance)I would advise you to get her ankle/brachial index to begin with. Also get a thyroid panel if not done previously."
},
{
"id": 87986,
"tgt": "What causes lower abdominal pain?",
"src": "Patient: hi, i have lower right abdominal pain. Its the second time its happening. The first time it lasted for a week long, and that was last month,now this month its happening again. It does not hurt when i lie down but when i move or sneeze its worse ,but the pain is mild and then later worse again. Can you maby give me advise on what it can be? Doctor: Hi,from history it seems that you might be having chronic apppendicitis giving this type of pain and often recurrence.Consult your doctor and get examined.Physical examination while having pain will give more precise clue about your problem.Meanwhile take some antispasmodic or anti-inflammatory medicine to get temporary relief.Ok and take care."
},
{
"id": 212728,
"tgt": "Feeling jittery and dizzy. Could it be due to depression or anxiety? Any relief?",
"src": "Patient: hi i feel jittery everytime i go out.. This is after i got dizzy and puked when i went out one time Seems like i cant get rid of that worry out of my head. What should i do? this is effecting my social status sometimes i dont go out anymore because im scared of getting dizzy.. I know its all in the head but its easy to say. When it slaps me i feel uneasy, my feet feels light and i feel like i need to rush somewhere or rush home. I calm myself with water sometimes and a menthol rub but it seems like its getting worse. Some people say it could be because im depressed or its anxiety .. what should i do? Doctor: Hello.... Thanks for your query. On going through your symptoms, it appears that you are suffering from social anxiety disorder which has adolescent onset. It is characterised by anxiety symptoms only ion social situations. Effective anti-anxiety medication are available in addition to psychotherapy (eg exposure & response prevention). Hence, i would suggest you undergo a thorough psychiatric evaluation so that appropriate management can be initiated. Regards Dr Sundar Psychiatrist"
},
{
"id": 127035,
"tgt": "What causes severe pain below the sternum?",
"src": "Patient: I just had an episode which I had sever pain below the sternum. Then I got the sweats and weakness. I took some Gavinon,because I have acid reflux disease. A short time later I chewed 3 80 mgs of baby asprin. I laid down.about 10 minutes later I felt fine. I drank plenty of water. I did have a lot of diarrhea. This morning. I took my BP and it was good 124 over 74 pulse 59. I fell OK now but a little tired. What do you think Doctor: Hi, Consult a physician and get an EKG done at the earliest to rule out cardiac causes like angina. Cardiac pain can also present with similar symptoms. Wishing you good health. Hope I have answered your query. Let me know if I can assist you further. Regards,\u00a0\u00a0\u00a0\u00a0\u00a0 Dr. Shinas Hussain"
},
{
"id": 60775,
"tgt": "What do facial swelling and a lump under the eye indicate?",
"src": "Patient: I woke up with swelling on one side of my face yesterday. After touching the swollen area, I noticed a rod shaped lump under my skin right between the bag of my eye and my cheek bone. The area where the lump has continued to hurt when touched but the swelling around the lump has gone down after applying a heat compress. I should note that I had popped what I thought was a pimple a couple of days ago near the lump and I wonder if that might have caused an infection. Doctor: Hi, Swelling around the eyes may be due to infection at the local site or it may be due to renal or cardiac failure in case of dependent edema. Until examination is done it is difficult to say what is the cause for it. If you have redness or itching then it may be a local infection. Dependent edema may be due to the renal or cardiac problem. Please consult your physician, he will examine and treat you accordingly. Hope I have answered your query. Let me know if I can assist you further. Regards, Dr. Penchila Prasad Kandikattu, Internal Medicine Specialist"
},
{
"id": 5043,
"tgt": "On IUD, thinking of conceiving. History of having twins, bed rest due to .08 cervix. Possibility of short cervix again?",
"src": "Patient: Hi, I am 28 and my husband I are thinking about having another baby, we have 4 year old twins. I have an IUD. With our twins we had 3 rounds of clomid. But at 20 weeks my cervix was at a .08 and was put on bed rest. I would love to have another baby but I am scared about going thru what we did with the twins. What is the possibly of having a short cervix again? I don't know if it was just because of the twins. Thanks for your advice. Doctor: Hi, thanks for the queryAs per your history, you had cervical incompetence in last pregnancy. Twin pregnancy might have been an aggravating factor but not a root cause of your problem.There are very high chances of having cervical incompetence again in your next pregnancy.But, there are many procedures available now a days to avoid abortion with such pathology. You should plan your pregnancy and take specialist advice regularly so that necessary measures can be taken on time.Hope this information is helpful and informative to you. If you have any further query you can directly reach out to me on the following URL:http://doctor.healthcaremagic.com/doctors/dr-deepti-goyal/65111"
},
{
"id": 178904,
"tgt": "What causes difficulty while speaking?",
"src": "Patient: My 2 year is not really talking yet but he makes alot of sounds. He is doing alot of grunting/growling noises & also tries to say words but it sounds like it's with his throat but with his mouth closed. He has good hearing and I can't see any problems with his tounge. Could there be something to do with his throat that is preventing him from making certain sounds? He says, hey, hi, daddy (\"dede\"), no, but doesn't show alot of jaw movement to say words like \"ball\". (he is seen for speech but only twice a month) Doctor: Hi, thanks for asking. I can understand your concerns. I would like to ask you certain things.How is his non verbal communication?is he making those sounds for communication intent or just like that?How is his socialization?Can he imitate facial movements like sticking out his tongue or puffing his cheeks?Depending on all these he may either have specific language delay or oral motor coordination problems or autism.Evaluation by a developmental and behavioural paediatrician will help you get to an appropriate diagnosis and then get treatment accordingly.I hope I have helped youThank youDr. Meera KotechaDevelopmental and Behavioural Paediatrician"
},
{
"id": 66444,
"tgt": "What is the red lump on the scrotum with only blood and not pus?",
"src": "Patient: Hi, Earlier today i found a red lump on my scrotum. I decided to squeeze it to see wether it would pop or not. It did. But there was no pus , all that came out was blood as it made a sharp popping sound. Can you tell me what this could possibly be? How it may of got there? and if i should pop it or just leave it? Doctor: Hi, thanks for sharing your health concerns with HCM! Sorry to say that this bloody discharge again could be pus only mixed with blood! Anyways, If I were your treating Doctor for this case of bloody scrotal lump, I would come up with three possibilities, these include: 1.a hemangioma /hematoma 2.a scab due to friction / trauma3. an infected sinus tract or a benign small sebaceous cyst\u00a0\u00a0\u00a0\u00a0\u00a0Overall, it is benign and not to worry about this but you could go for FNAC test for confirmation !Hope this answers your question. If you have additional questions or follow up questions then please do not hesitate in writing to us. I will be happy to answer your questions. Wishing you good health."
},
{
"id": 220266,
"tgt": "What are the symptoms of potential pregnancy?",
"src": "Patient: Hello, I am on my first cycle of clomid, my last period started on 4th July it lasted 3 days, my GP says that I anovulate and blood tests show that i have very high Teststerone levels. around the 25th of july i began to have cramps and felt a lot of pressure low down in my belly along with very sore nipple and tender breasts. these symtons have now become much milder but I have never experienced them before. Is this because I am now ovulating properly or could it possibly be signs of early pregnancy (I dare to dream!) I have also had headaches, dizzness around the same time and am tireder than usual. I would love your opinion and if you could tell me the best time to take a test as i m terrified of being dissapointed again x x x x x Doctor: Hello dear,I understand your concern.In my opinion some woman experience muscle cramps,tiredness,sore breasts before the periods.These symptoms are due to hormones and signify that the cycle is ovulatory.In case of anovulatory cycles there might not be symptoms.The clomid is effective in inducing the ovulation and it is the first choice in those with anovulation.The presence of symptoms might suggest ovulation has occurred.The urine pregnancy test needs to be done a week after missed period for accurate results.Test with the first morning sample of urine.The first sign of pregnancy is missed period.Other symptoms of pregnancy like nausea,vomitings, increased urination,sore breasts start 2 weeks after missed period.Hope this helps.Best regards...."
},
{
"id": 11153,
"tgt": "Suggest treatment for hair loss with hypothyroidism",
"src": "Patient: Hi,I 28yr old.weighing 76kgs. Am treated for hypothyroidism (now normal) for last two months with Thyronorm 75mcg & Eldervit injection for 8week to reduce hair loss My hair fall has still not been reduced & the hair thinning is still on...Please let me know when will it stop or is there any perscription to stop thyroid hair loss. Thanks Doctor: Hi,Probably you may be having telogen effluvium.. It may be due to some cause. Most common cause is psychological upsets like anxiety,worries, tension or depression. Other causes may be thyroid dysfunction, vitamin and mineral deficiencies, anaemia, other internal diseases,drugs..etc. and you have thyroid dysfunction which might be responsible in your case.And remember that normal hair fall in adult may be 100 to 150 per day. So, please avoid anxiety related with hair fall.You consult dermatologist for firm diagnosis and treatment. Blood tests may be done to rule out internal diseases.You may take biotin containing tablets for long time along with vitamin E cap. You may apply mild steroid lotion like beclomethasone lotion on scalp at night. Mild herbal shampoo to cleanse the scalp and almond oil as hair oil may be used. Please continue treatment for hypothyroidism with regular check up TSH level.Have patience for the result. I hope you got my answer.Thanks.Dr. Ilyas Patel MD"
},
{
"id": 25298,
"tgt": "Is angioplasty right treatment for diagnosed 80% and 60% blockage in valves?",
"src": "Patient: My dad had a Heart attack on 22.11.11 and was hospitalised and taken the angiogram, and the report was he has block in one valve about 80% and in another valve he had 60% , Doctor suggests to have a angioplast. Please advise we have to do the angiplasty immediately. And also let us know the best hospital and doctor for doing the angipplast with the lower cost and what will be the cost to doing that in bangalore. Doctor: HiFor the blocks ranging from 60 --90% angioplasty is the best solution.Please go ahead with the procedures as early as possible.Best hospital with cheaper rates are narayana hrudalaya centres,still cheaper withbonly stent cost is jayadev hospital.You can discuss the cost at the help desk of the following hospitals.All the best"
},
{
"id": 59614,
"tgt": "Have multiple calculi in gall bladder. Advantages of having it removed?",
"src": "Patient: I have been diagnosed with multiple calculi in the lumen of the gall bladder . I do not have any symptoms currently but would like to know what s the best thing to do: leave it until there is a symptomatic pain or go for a cholesystectomy? What are the advantages and disadvantages of having the GB removed? I am a vegetarian and do not consume too much fat containing foods but still a bit apprehensive about having an organ removed. Doctor: Hello, thewhizzler, You mention that you have been diagnosed with multiple gallstones. I am sure that the reason you had the sonogram of the gallbladder must have been because of some vague digestive sypmtoms or some abdominal pain. Multiple stones are more problematic than a solitary or single stone. One or more of these stones pass through the bile duct or the channel that brings the bile to the intestine, can get stuck and cause obstruction with severe attack of pain, vomiting and yellow jaundice ,fever etc. This would then become an emergency situation. Small single stone (Cholesterol) without calcium can be treated with medication to dissolve this or even laser treated to pulverize it. It is hard to do this when you have multiple stones. I am glad that you are a vegetarian and avoid fatty, greasy foods. If you have a tendency for gallstone formation, you can't prevent it by diet alone. Good news about surgery is, that can be very safely performed by laparocopic approach with small button holes on the belly without a huge scar like it used to be years ago. Uasually, you go in the morning, have the surgery and you are home by afternoon, ofcourse barring any unusual findings or complications. Your liver will continue to produce bile that we need for digestive process. Instead of being stored in the gallbladder and released , the bile will get into the intestine directly. Occasionally, you might experience a little diarrhea sometime after the gallbladder is removed, but this temporary. Hope this answers your concerns. I wish you well."
},
{
"id": 2370,
"tgt": "What is the cure for blocked fallopian tube other than surgery?",
"src": "Patient: Dear sir,iam 30 yrs old .i got married in 2010 feb.till iam not conceive.prob is my left side fallpian tube is blocked .my doctor is suggested laproscopy.iam very fear abt it.i want remedy without any sugeries.pls findout any solution with any medicine Doctor: Hello, By saying that your left tube is blocked I think the RIGHT tube is normal & patent. If so you can try for pregnancy with one tube . No matter which side the egg comes , it will finds it's way through the open tube. But when you get pregnant have an early scan to confirm pregnancy is in uterus, not in tube.There is no medicine to open a blocked tube.If both tubes are blocked you will have to go for Hystero-Laparoscopy. Under hysteroscopy guidance tubal recanalisation can be done . It's not a major or bloody surgery. You can get discharged on the same day.Or else you have to opt for IVF- which doesn't require tubesHope I cleared your doubtAll the bestDr.Balakrishnan"
},
{
"id": 195039,
"tgt": "How to diagnose a mental illness?",
"src": "Patient: Hi my name is Zulma and I have a 29 year old son who is talking to him self all the time ,says there are people telling him to do things like taking his close off and where ever he goes people are bothering him and talking to him. He says the neighbor is in his room and telling him things . Four years ago he burn the whole apartment where we were living in and 2 month ago he burn our patio and the neighbors. We live in Arlington t.x. and do not know what to do since he s not a minor but I see he is escalating, saying he is tire of the situation. Please let me know how can I help him specially when he says they are after him. Doctor: Hi, The symptoms more or less suggestive of schizophrenia. Consult a psychiatrist at the earliest and get evaluated. Hope I have answered your query. Let me know if I can assist you further."
},
{
"id": 209621,
"tgt": "Suggest alternate medication to divalproex",
"src": "Patient: hi! i am taking divalproex and oxycabamazepine for the past 5 months as i got depressed after a psycho deceived me in the name of marriage,now my weight is 5feet 4 and weight is 88kilos , age -38. I want to change medication as i feel dull and sedated at times,please help Doctor: Hello,Thanks for choosing health care magic for posting your query.I have gone through your question in detail and I can understand what you are going through.You have mentioned that you are depressed but unfortunately the medicines that you have mentioned doesn't have anti depressant in it. Both are primarily mood stabilizers. Along with the mood stabilizers you need to be on antidepressants as well. If sedation is a problem then you may choose bupropion or fluoxetine as antidepressants as they are nonsedative anti-depressants.Hope I am able to answer your concerns.If you have any further query, I would be glad to help you.In future if you wish to contact me directly, you can use the below mentioned link:bit.ly/dr-srikanth-reddy\u00a0\u00a0\u00a0\u00a0\u00a0\u00a0\u00a0\u00a0\u00a0\u00a0\u00a0\u00a0\u00a0\u00a0\u00a0\u00a0\u00a0\u00a0\u00a0\u00a0\u00a0\u00a0\u00a0\u00a0\u00a0\u00a0\u00a0\u00a0\u00a0\u00a0\u00a0\u00a0\u00a0\u00a0\u00a0\u00a0\u00a0\u00a0\u00a0\u00a0"
},
{
"id": 86718,
"tgt": "What is the treatment for lower stomach pain?",
"src": "Patient: Hi, may I answer your health queries right now ? Please type your query here... My husband woke up with a pain in his lower stomach. We had sex the night before and he was fine. But at work he felt pain in his power abdomen and pain and sensitivity in his scrotum but a testicle was harder than the other. He also urinated more that day. The next day his lower ab pain went away and he now has less pain in his scrotum but the hard testicle remains but not as sensitive. What could this be to come on all of a sudden then subside the next day? Doctor: Hi.Thanks for your query for your husband.Sex before night, pain in the lower abdomen and the scrotum and the testicle being harder than before, frequency of urination, and now the pain has gone away from lower abdomen, less pain in the scrotum but hard testicle remain but not as sensitive. These all things point to the fact that there might have been either the torsion or the Epididymo-orchitis along with urinary tract infection. I would advise :Urgent color doppler and ultrasonography of the scrotum and abdomen.Tests of urine culture and sensitivity before antibiotics are started to get us a better results / of blood. Final diagnosis will tell us the further plan of management. http://bit.ly/askdrtchandrakant"
},
{
"id": 184225,
"tgt": "Suggest remedy for foul smelling mouth with yellow patches on tongue",
"src": "Patient: my mouth smells very bad whole day,even i brush 2times a day i noticed that i got a weird taste in mouth after some time and generally goes off after eating something but comes back again.Also there is whitish yellow coating on my tongue which i think might be reason for bad breath..please help doc.. Doctor: Yes those white patches suggest u are feverish or suffering from some underlying ailment . Get your blood tests done. Use a tongue cleaner to clean it.use hexidine mouthwash."
},
{
"id": 223364,
"tgt": "Is morning after pill effective if taken the next day of having unprotected sex?",
"src": "Patient: I made a mistake this morning and had unprotected sex but it is the first day of my period also I gave birth only 6 weeks ago I understand I need to get the morning after pill but unable to get it today will it be as effective tomorrow and can I take it when I'm bleeding?Thankyou for your help Doctor: hello user,no need to take morning after pill.because u had sex on your safe period u wont get pregnant.dont worry..thanks.."
},
{
"id": 190039,
"tgt": "Stiff neck, painful, weakness, sore throat. Taking analgesics. Suggest?",
"src": "Patient: I am 52 Male. about three months back I experienced a stiff neck . Thinking it to be cassual pain I overlooked it. Since I was simultaneously under going dental treatment I was on analgesics and hence did not felt the pain. Now even after three months the pain has once again erupted and from last two days i am having a very stiff neck. I am also feeling very week. I have no fever . Yes I do feel that I am experiencing a mild sore throat from last three months. Should I call on an ortho or an ENT Doctor: Hello there...what kind of dental treatment you underwent?based on that i can let you know if your dental treatment has any relevance with your neck pain...Since you are complaining of neck stiffness...you have different reasons for that...though there are few dental conditions that has relevance, i cannot comment until i know the kind of treatment performed... stiffness could be because of severe pharyngitis or tonsillitis or any form of infection in your neck....if it only to the neck region then you can consult an ENT who are experts in it... bacterial infection to the salivary gland (submandibular salivary gland) would also lead to pain in the neck..clinical evaluation is mandatory for final diagnosis...."
},
{
"id": 157504,
"tgt": "Growing number of moles on inner labia. Have undergone LEEP procedure. Is this normal?",
"src": "Patient: I saw my gyn in March 2013 and had pointed out a new mole (light brown, eraser sized w a hair) on my labia - she shrugged her shoulders and said \"it's a simple mole, make sure it doesn't change\" it hasn't changed. But I have since then noticed 2-3 more very Small, similar sized moles beginning to grow on the inner lips. I previously had a leep procedure in April 2013 for CIN 111 cells in cervix. (2 good paps since) I have an appointment to see ,my doc in 2 weeks but am FREAKING out... Please help. Why is this happening? What could be the cause? I am an esthetician for a plastic surgeon and am familiar with what skin cancer can look like, this does not appear to be abnormal in any way. Thanks! Doctor: Hi,Thanks for writing to us.The description in no way sounds alarming. It could be a mole, cherry angioma, warts, molluscum contagiosum, angiokeratoma etc. I do not think it is cancerous. However apart from your gynaecologist, I would advise that you seek an opinion from your dermatologist too.Regards"
},
{
"id": 224783,
"tgt": "Going for robotic laparoscopic surgery for fibroids. Advised to stop birth control pills. Should it be stopped before the surgery?",
"src": "Patient: I will be going in for robotic laproscopic surgery in Sept 2012. I went for 3 different opinions. 1 does open laprarotmy & told me to stop taking my bc pills about a week before surgery as did the 2nd opinion i had with a different dr. The 3rd dr witn whom i have selected to actually do the robotic on both fibroids 9.5 cm & 2.5 cm due to his experience of over 1000 patients has old me it wasnt necessary to stop. A little worried now whether i should just stop on my own- i only take them so I can forcast my cycle since I bleed so much I cant leave the house. Please help! Doctor: Hello and welcome, The birth control pills increase the risk of blood clots. Immobilization and surgery is another reason for increased risk of clots and the third is age. So if you use pills and are immobilized and operated plus the post operative recovery period where a person is relatively less moving about, the risk of blood clots increases a lot. These clots go to the lungs and lodge there so there is trouble in breathing. Having said that every surgery will not lead to clots, but it does increase the risk. So we stop the pills as a safe guard to decrease this risk that can be life threatening. If your surgeon is confident, then he must be mobilizing the patients real fast so that the risk decreases. So just discuss this with your surgeon and then decide with his consent as he is the one who will operate and responsible for post operative care. Hope this satisfies your query. All my best wishes are with you for an early recovery.Thanks for using HCM.\u00a0\u00a0\u00a0\u00a0\u00a0Feel free to ask any more questions that you may have. Dr Madhuri BagdeConsultant Obstetrician and Gynecologist"
},
{
"id": 208043,
"tgt": "Suggest treatment for loneliness and suicidal thoughts",
"src": "Patient: Hello doctor! I am from Sri Lanka . I am from a rich family . I am Enginner .I having every thing in life . but every time I feel lonely . Hate every thing in life . Don't like to go forward in socity . Don't know why . Some time feel to go suiside .Don't know why plz help me Doctor: Hello,I suggest you not to worry much. I assure you that you are not depressed or having any mental disorder. You may be lacking your self-confidence or self-esteem. It could happen due to many disappointing situations happen in everyday life. I suppose that you are just disappointed. Please consult a psychologist for counseling, and you should overcome your present problem.Hope I have answered your query. Let me know if I can assist you further.Regards,Dr. K. V. Anand"
},
{
"id": 16234,
"tgt": "Red itchy rash on both inside of elbows and back of knees. Tried Cortizone, creams. Solution ?",
"src": "Patient: Hi there I have a red rash on both the insides of my elbows an backs of knees, it becomes increasely very itchy after I have a shower, have got a bit sweaty, (as I wear lots of warm clothes as work early morns outside) but doesn t spread stays the same size doesn t spread but can also sting when becomes very itchy, tried Cortazone creams, vit creams an just can t seem to get to te bottom of it, also am trying a liver tone at present please help! Thanks Doctor: Hi there. Your problem requires the attentions of a dermatologist on the ground. It could be any of the papulosquamous skin diseases aka psoriasis (inverse) or atopic dermatitis from your description. It may require a skin biopsy & or pathc tests to confirm & the treatment will be based on the pucca diagnosis. I dont know how much a liver tonic is going to help you but Good luck! Do add an antihistamine like fexofenadine 180mg in the interim. cheers!"
},
{
"id": 51831,
"tgt": "Sharp pain I lower abdomen sometimes continuing to kidney area. any ideas ?",
"src": "Patient: For three months now i have been going to my doctor non stop and am now being referred to the hospital for further tests. im 21 and i am haveing severe pains in my lower abdomen which continues round to my kidney area. I am also always going to the toilet (urinating 20+ times a day) i have done various urine tests etc. i recently had an ultrasound to check for kidney stones or ovarian cysts , but again nothing was found. I have had four different antibiotics incase of infection, and yet still no difference. i also the whole time i have been going to the doctor, have had blood and protein in my urine. any ideas ? Also i dont know if this is helpful or not, but diabetes and endometriosis. thank you Doctor: Hello cass; welcome to HealthcareMagic Diabetes can give shelter to infections which become difficult to treat.Also diabetes can cause the increased frequency of urine but as for the pain it can be due to infection.Presently since you have been referred to a hospital for investigation please get a urine culture and sensitivity test done to find the nature of infection (whether it is bacterial or fungal) and the sensitivity test will guide regarding the indicated antibiotic or antifungal which will hep to treat the infection better.Please get investigated and then the doctor will treat you accordingly. Thanks"
},
{
"id": 91222,
"tgt": "What is the cure for intense pain in lower abdomen?",
"src": "Patient: sharp painI have very sharp pain in my left lower abdomen that keeps intensifying. I know its not menstrual cramps but is keeping me form moving very fast or sleeping very well or for very long. should I go see my doctor? I am 28 female 6 foot tall 288 lbs. Doctor: Hi. This is most probably due to a cause that needs an urgent attention of a Doctor for clinical examination , elicitation of classical clinical tests and ultrasonography to know the reason.The pain is keeping you from moving itself is a symptom, enough to visit a Doctor soon."
},
{
"id": 144758,
"tgt": "What is the treatment for stroke?",
"src": "Patient: Respected Sir, Last year in the month of Jun 2014 my father suffered a stroke, due to which he got numbness in left side of the body. we are regularly medicating him but the numbness is still same. Though he is able to do all his daily works without any help, but he is feeling nervous due to the numbness. please suggest me any exercise to reduce his numbness. thanking you, Regards Bhaskar Rao Tolana Doctor: Hello I think your father is a lucky man to get back to all activities after a stroke. The numbness he is suffering from is probably due to the stroke and more specifically the loss of blood supply to the part of the brain controlling sensations. The brain functions after a stroke take time to recover , helped by physiotherapy etc. Sensory feelings would sometimes take longer than expected . There is nothing to worry in this regard and there is no real drug to improve this except for the continuation of the blood thinners which I think he must be already on .best wishes to him"
},
{
"id": 48357,
"tgt": "What alternative treatments are suggested for sluge in gall bladder?",
"src": "Patient: I was told by gi surgeon my gallbladder has sludge in it and an operation is needed to remove the entire gallbladder. I have Lupus and on Dialysis and did not experience the pain in my stomach but in my lowere back. could this be a case of Celeiac Disease or the tricks my lupus plays with pain in my body. I would love to hear of other avenues other surgery Doctor: Hi,Thanks for writing in.Gallbladder sludge refers to a thick material that is not absorbed by bile in the gallbladder and allowing this to accumulate over time can cause stone formation which will then obstruct flow of bile and can cause further pain and discomfort. Surgery is a standard form of treatment in gall bladder sludge formation. However you might request your doctors for a trial of non surgical treatment with drugs may be given to dissolve gallstones. This does not work in every patient but you can try taking it for a while and adhering to dietary restrictions. Should the medications fail to respond then a surgery is the only form of definitive treatment."
},
{
"id": 154866,
"tgt": "Is cancer hereditary?",
"src": "Patient: Hi Doctor, I am 43yr Caucasian female and 84kg, badly anemic and generally feeling tired - otherwise health is ok. My dad died of stomach cancer at 52 (other organs were effected and he died within 3months of diagnosis) and now my mum has colon cancer (stage-3), - am i a high risk? Thankyou for your time Doctor: Hi, dearI have gone through your question. I can understand your concern. Colon cancer has proved genetic cause and it is hereditary. In stomach cancer few are hereditary. So overall you have chance of gi cancer. You should go for regular colonoscopy and CEA level to rule out possibility of cancer. Hope I have answered your question, if you have doubt then I will be happy to answer. Thanks for using health care magic. Wish you a very good health."
},
{
"id": 91395,
"tgt": "How to treat prolonged cramping in stomach and abdomen?",
"src": "Patient: I have had stomach cramping in my lower abdomen, center and left for 10 days now. Last two days have been painful cramping.. bm normal until today..loose today. Left testical is aching. I was told to drink mylox and take ezo for bladder. It s worse today. Any ideas? Thank you. Doctor: HI. You have concurrent loose BM and left sided pain in abdomen as well as testicle. This can be due to an intestinal infection and due to straining or so there might be a strain on the muscles supporting the testis. Get yourself examined by a Surgeon who on the clinical grounds can tell you the problem and also treat and investigate you accordingly . Till you see a Doctor - drink plenty of oral fluids , support the testis."
},
{
"id": 190856,
"tgt": "How can redness and sore gums be cured caused by keeping whiskey for long hours in mouth ?",
"src": "Patient: About five days ago I was drinking whiskey and for some unknown reason (?) I decided to keep the whiskey in my mouth to kill bacteria , I know I was wrong now. I don t know how long I kept the whiskey in my mouth but I probably did it at least twice for twenty minutes each time. The next day I didn t notice much difference and didn t even remember the incident but as the day went on I noticed that I was having gum pain on eating. The next day things got worst, I look at my gums with a mirror and notice they were burnt looking all on the inside of my mouth where I held the whiskey. White dead skin was coming off and I had very red sore gums around the base of my teeth on the bottom and top insides where the liqueur was. I nursed this condition for four days with warm salt water rinses three times a day and ate only soft non-chewable foods. Now after 5 full days my gums look better but still some red patches that are very sore. The redness is not as vivid as it was a few days ago but the pain is excruciating and has not subsided much. Ibuprofen isn t helping much. Bottom line most of my gums are looking better and slightly less pain except one spot that hurts constantly no matter what I do, it s red in this area covering about three teeth at the gum line, (number 30 and teeth next to it). If I didn t have the non-stop pain I wouldn t be writing, I can t sleep, eat or do much without constant pain. Any suggestions? It s been five full days. I haven t drank any alcohol since and don t plan on it after this episode. Thank you. Doctor: Hello sir Your signs indicate an alcohol burn of gums, which has denuded the superficial layer of the gums causing irritation, pain and redness. You still need to avoid spicy and hot foods for few more days as it takes a couple of weeks for the repair or healing of the gums. so you need to wait till then. dont worry its going to be fine in few days. avoid smoking and alcohol till complete healing and even after for a month to help the tissues growth. Till then you can apply gels containing anesthetic and anti inflammatory like Mucopain. if at all the symptoms dont come down then consult a periodontist for further tratment."
},
{
"id": 153008,
"tgt": "Is cell transplantation necessary for Lymphoma?",
"src": "Patient: My son, aged 25 yrs, was first diagnosed with Hodgkins lymphoma in Jan 2008. He was given 8 cycles of ABVD chemo. just 2 months later, his PET Scan showed and FDG uptake, for which he was given 2 cycles of GDP as salvage, followed by Autologous stem cell transplant. He had taken the transplant quite well and was even out of hospital in just a month. Now, in May 2010, on a routine check, his disease seems to have relapsed. A biopsy also confirmed this. His bone marrow biopsy showed no trace of disease in the marrow. now he has been recommended 3 cycles of ICE chemo, after which an Allogenic Stem Cell transplant has been planned. I am terribly worried about the transplant. Right now, he has completed one cycle of ICE chemo last week and is doing quite well now. his platelet count has started improving steadily. My query is, is the transplant absolutely necessary? what other options do we have ? If we do not take the transplant now., and God forbid, his disease relapses, what other treatment options will we have? what could be his chances of relapse with only the ICE chemos? pls. do let me know. Doctor: hello dear. I have gone through your case. It appears aggressive disease biology and transplant is the only curative option left. If you do not get transplant there will be chances of relapse again. So please go ahead with the transplant.thanks"
},
{
"id": 203896,
"tgt": "Can masturbation cause health problems in future? How to overcome this habit?",
"src": "Patient: hi sir i am loosing sperm wantedly thrice in a week ..is it cause any health problem to me...? am doing this action at the age of 19 years old ...now my age is about 25 years will it leads any dangerous to me ....even i have healthy body after doing this also ..only the problem is am lean....please give me the solution to overcome this habbit Doctor: Hi,Masturbation is a habit and not the disease. Nothing bad happens after the masturbation. Then, why should you worry? Take it easy.Just relax after the event. Still, if you want to quit masturbation, you consult psychiatrist. Psychotherapy and anxiolytic medicine may be given to you by your doctor.I hope. You got my answer.Thanks.Dr. Ilyas Patel MD"
},
{
"id": 172984,
"tgt": "What causes green colored, loose stools in a 7 month baby?",
"src": "Patient: Hi, My 7months old baby is having green and loose stools. Previously she used to have 8 stools every day and now 4 to 5 stools per day. After every breast feed or givng cerelac she is having loose and green stool.. Stool test (O &P) was normal... Please suggest what to do.. Doctor: Hello dear,With such presentation in my clinic I would suggest to. give tests for stool culture and sensitivity to antibiotics. - Enzymes syrup should be given for good absorption of food for 1 month- Econorm 1 sachet daily 3 days ,then Vyzilac 1 capsules for 1 month-Z&D syrupWishing baby good health"
},
{
"id": 100272,
"tgt": "Could eyes itching and swelling be due to a cat allergy?",
"src": "Patient: I was told by another doctor - not my own - that it sounded like my asthma was misdiagnosed. When I was 4 I had Bronchitis/pneumonia (not sure exactly which one) and my doctor told my mom and I that all it was was a little infection. But my mom kept on insisting I had asthma, which made the doctor ask me questions that my mom insisted were true although when I was asked them later the answers we always no . I haven t had any problems with my asthma since I was 9- when I sold my cat. I can run 4 miles without an issue, I am never short of breath, I had an inhaler but it never helped or anything; but if a cat gets in my face my eyes get itchy & I feel swollen almost I Have no problems with any other allergies, I am rarely ever sick, and I have no problems nor have I ever that are typically associated with asthma. When I explained this to a doctor I came across while at a restaurant, they said it sounded like I had a cat allergy, not asthma. Would you say I do have asthma or that it s more likely i just have a cat allergy? Doctor: Hello,Thank you for asking at HCM.I went through your history and would like to make suggestions for you as follows:1. Asthma is a very broad diagnosis, it is not a single disease, it has many types. By knowing details about symptoms, it is only possible to judge (judgement vary from doctor to doctor) a diagnosis.2. Diagnosis of asthma can be reliably made by a test known as \"spirometry with post-bronchodilator testing\". So, I would suggest you to discuss need of the test if diagnosis of asthma is a concern.3. From your history, I would think of cat allergy, which can be also confirmed by allergy testing for cat proteins.4. However, I would like to mention here that asthma itself can be manifestation of allergies and one can have asthma due to cat allergy. So, if diagnosis is a concern, both of the above suggested tests will help you so as a consultation with an Allergist-Immunologist.My personal opinion, may be cat allergy, which caused asthmatic symptoms in your childhood but now causing eye symptoms.Hope above information will be helpful to you.Should you have any further query, please feel free to ask at HCM.Wish you the best of the health ahead.Thank you & Regards."
},
{
"id": 148575,
"tgt": "Severe left milder right foraminal stenosis with asymmetric mass effect on left L5 dorsal ganglion. Indications?",
"src": "Patient: I just got found out that my l5 S1 had a small central caudal extrusion superimposed on mild retrolisthesis of L5 S1 no mass effect on the s1 roots, Severe left milder right foraminal stenosis with asymmetric mass effect on left L5 dorsal ganglion... what does all this mean? Do you think spinal injections will help? Doctor: Hi,Thank you for posting your query.I have noted your MRI findings. These suggest mild compression of nerves in the lumbar region (lower back), which may produce back pain, pain in the legs, and numbness of feet.Treatment would depend on the severity of symptoms.Initial treatment would consist of medications such as pregabalin or gabapentin capsules, along with physiotherapy.If the pain relief is inadequate, then, epidural injections would be helpful.I hope my answer helps. Please get back if you have any follow up queries or if you require any additional information.Wishing you good health,Dr Sudhir Kumar MD (Internal Medicine), DM (Neurology)Senior Consultant NeurologistApollo Hospitals, Hyderabad, IndiaClick on this link to ask me a DIRECT QUERY: http://bit.ly/Dr-Sudhir-kumarMy BLOG: http://bestneurodoctor.blogspot.in"
},
{
"id": 204159,
"tgt": "What causes swelling in the uvula while suffering from panic attacks?",
"src": "Patient: I am curious about something that s happened a few times to me now. This is embarrassing, but I really want answers so I m asking here. My uvula will sometimes swell to the point where it can lay down on my tongue. I don t suffer from any anxiety disorders, but each time this has happened, I ve had a panic attack and hyperventilated. In my head, I KNOW I m not suffocating/ choking, but the constant swollen pressure inside my throat makes me agitated, sweaty, and manic. It s like my body is in terror, but my mind is saying There s nothing to worry about. It feels like I can t breathe (which ironically makes me hyperventilate) and it triggers my gag reflex. It s incredibly stressful. I start to shake, hyperventilate, gag, sweat, and experience panic despite the head knowledge that I m really not in danger. It s hard to explain what it feels like to feel your uvula touch multiple sides of your throat. It feels like my throat is clogged (because the contact is the circumference of the uvula) and I unconsciously swallow to clear the obstruction , which only makes it worse. If it matters, the sensation feels like I ve got thick mucous on my uvula, so I feel the need to repeatedly swallow. I suspect the swallowing inflames my uvula which makes it grow WAY bigger. Then it becomes hard to swallow and the sensation of having something stuck in my throat makes me feel like I m choking. In reality, I can still breathe, but it certainly raises all the alarms physiologically that I am choking/suffocating. I m a normal weight (170 lbs, 6 foot tall, 27 year old male). The panic attacks only ever happen AFTER my uvula swells. The uvula swells over the course of hours then stays swollen for hours more. At a certain critical mass, I feel the sensation of having the uvula down my throat which feels like I m choking. Additionally, my saliva glands go into overdrive. Since I can t really swallow, I m left leaning forward over a bowl and spitting to keep my mouth from filling with spit (which only exacerbates the sensation of drowning/ choking/ suffocating). Leaning forward also keeps the uvula from going down my throat though, feeling it lay on my tongue isn t exactly a huge improvement. This only happened 4-5 times in my life and all within the last 3 years. The first three times this happened, I ve gone to an emergency room. Each time, steroids reduced the swelling. I ve been referred to an Otolaryngologist about surgically reducing the size of my uvula. However, when the swelling goes away, my uvula is quite small and it s not medically necessary to do surgery. I ve had blood panels taken before by the ENT to see if I had some rare condition that caused the repeated swelling. Negative. The doctor suspected that I had strep throat, which can uncommonly cause swelling. However, I ve noticed that 4/5 times these swelling episodes happened, I ve had dry mouth due to Vyvanse or Mydayis. When on these medications, I swallow a lot more and my mouth often feels uncomfortable and a little swollen. Right now, as I write this, I am experiencing that same mucous in the back of my throat that makes me want to swallow sensation. If it becomes necessary, I ll see a doctor if it swells more. Are there any medications I can discuss with my doctor that can alleviate this problem? Final stipulation: my family has a history of drug abuse. With the sole exception of Mydayis (previously Vyvanse), I avoid any addictive medications and ask for alternatives that don t come with the risks. I love my life and I ve seen too many loved ones lives ruined by addiction. So if there are non addictive medications I can take to help, I would love to know what they are so I can consult with my doctor about taking them. Additional information: I have tried taking Advil to reduce the swelling. Any reductions in swelling weren t noticable. I ve tried Benadryl in case I m allergic to something (could be anaphylaxis for all I know), still no alleviation of my symptoms. Each time I ve gone to the doctor, it has taken multiple rounds of steroids (shots, don t remember the brand/name) to relieve the swelling. I suspect if I could take medication when I feel the uvula begin to swell, I could altogether avoid these episodes. However, I can t afford to see the doctor each time I SUSPECT it s going to happen. More often than not, the swelling has been minor no worse than a mildly sore throat. But 4-5 times have progressed to the point I described. Doctor: Hi, Panic attacks come under the anxiety spectrum of disorder and need to be treated fast. Due to anxiety, several neurotransmitters start malfunctioning. Hope I have answered your query. Let me know if I can assist you further."
},
{
"id": 159938,
"tgt": "Can Granulosa cell tumour with CD99 positive be cured by chemotherapy after tumor removal ?",
"src": "Patient: IS Granulosa cell tumour with CD99 positive can be cured by Chemothereapy. Tumour had been removed from the abdomen but still not diagnosed the stage. let me know how to proceed? My Mother was diagnosed with granulosa cell tumour with cd99 positive. Tumour cells are small with oval hyperchromatic nuclei with mild nuclear pleomorphism . Doctor operated and removed it from the body. We are not sure on which stage the cancer is. let me know how to proceed with that. will chemothereapy is required for the same.? Doctor: Hello. Thanks for writing to us. Chemotherapy is recommended for granulosa cell tumor. The types of drugs needed and the number of cycles required depends on the stage of the cancer. After the tumor staging is done, you need to meet a radiotherapist who will guide you further. I hope this information has been both informative and helpful for you. Regards, Dr. Rakhi Tayal drtayalrakhi@gmail.com"
},
{
"id": 47302,
"tgt": "What is the remedy for pain due to kidney stones?",
"src": "Patient: I am a 39 year old lady, have had appendix out, have had 3 cesarians and a full hyrsterectomy. I have a history of small kidney stones for 8 yrs in both kidneys. Been hospitalised twice with the pain on passing. Last one passed was 5mm, very painfull with vomitting. This week, bloody urine for 6 days. My doc picked up UTI & assumed stones again, gave pain relief. Not coping at home, went to ED st Hospital. Catscan revealed a 14mm by 8mm stone in left kidney. Sitting in the pelvis of left kidney. Cant possibly pass. I live in a Regional area of Australia. Cant get it operated on for about a month as I have no private health insurance. My question is: How do I deal with the pain without becoming reliant on the drugs? Do I have to keep treating the UTI's in the meantime with antibiotics? I have to work full time and look after 3 children on my own, what can I do to make this wait easier? Doctor: HelloThanks for query .You are known to get recurrence of urinary stones since many years and now detected to have stone of a size of 14 mm in pelvis of your left kidney .The best treatment option for this stone to be removed is Extra Corporeal Shock Wave Lithotripsy(ESWL).This is a non invasive technique and does not require hospitalization and can be done as out patient procedure so that one can carry on his/her routine work ..It is a non obstructing stone and can be certainly managed for couple of weeks till you decide to go for ESWLTake following treatment 1) Antibiotics to prevent infection 2)Pain killers as on when required (Diclofenac or Cyclopam )3) Drink more water .4) Avoid lifting heavy objects You can go ahead for ESWL once you decide to go for it preferably within a month or two .Dr.patil."
},
{
"id": 107946,
"tgt": "Suggest treatment for middle back pain",
"src": "Patient: Hello Doctor,I am having heavy pain on my middle of the back. The X-ray result and scan result shows nothing. But still i am feeling heavy pain while sleeping and sitting.Not while walking/Jogging. And i can't bend even little bit. What will be the problem Doctor. Thanks in Advance...RegardsKrish Doctor: When there is no problem in x - Ray or scan any intervertebral disc reduction or spondylosis us ruled out. There is no radiating pain reported either.This paves the way to decide in favour of stiffness of lumbar vertebral joints. Best way is to take hot water baths and apply muscle relaxants thereafter. May drink turmeric powder 1/2 tsp twice in milk or hot water. OR take Haridra capsules. It's same thing.Avoid cold drinks and cool atmosphere say A.C. etc. Drink hot water.Avoid pain killers. But my own patients would be given Ayurveda's herbal Dashmularishta liquid twice daily and Tarunikusumakar kusumakar herbal powder 1 tsp at night with warm water every other day. These are safe and side effect less naturals.In four weeks everything is expected to be normal.Hope it helps you."
},
{
"id": 207267,
"tgt": "Suggest treatment for adult ADD",
"src": "Patient: I have recently been informed that I might have Adult ADD by my peers. I didn t really think about it too much, except I think they might be right. I am unorganized, often unable to concentrate for long periods of time, unable to listen to long conversations, and low on self esteem. Im not sure I am hyperactive, but I definitely feel I have ADD through the symptoms subscribed to me. What steps would I need to take to find out if I can get medical help for this supposed ailment? Doctor: DearWe understand your concerns.I went through your details. I suggest you to keep calm. Please do not trust your friends' guidance in this regard. From the symptoms given, ADD is a long shot. Low concentration level could be due to loss of interest and confusion. People are normally unorgnized when there is no real compulsion. Low self esteem could be due to loss of interest level and disappointment. Introspect yourself about disappointments that happened in the recent past. If you want to have correct diagnosis please undergo a psychological test for ADHD. No self diagnosis and worries. Ignore for now.In case if you need more of my services in this regard, please post a direct query. I am happy to reply you.Hope this answers your query. Available for further clarifications.Good luck."
},
{
"id": 29837,
"tgt": "Can rashes be caused by mold and mildew?",
"src": "Patient: The area we live in has much mold & mildew on a ll the homes. I have a few spots (rash type) that no matter what i put on it I cant get rid of it. Have tried Triamcinolone acetonide cream, Nystatin, Betamethasone Dipropionate cream & several over counter creams, clotrimazol & such. Nothing works.I can go out of town a couple or 3 days & it clears up. has t? o have something to do with mold or mildew. Any suggestions at all much appreciated. Thanks.....Jack 73 years of age. Doctor: hello, I also think its a ant or some bugs bite. Other posibility is developing allergy to clothing material. Did you wear a new underwear today made up of synthetic materials? Some synthetic materials in clothing are allergic to skin. So always try to use cotton underwears. You can have antihistamine like loratidine and see whether itching and swelling subsides. allergy or bugs bite both has to subside. Hope this helps. Thank you."
},
{
"id": 27547,
"tgt": "Are lorezepam, Terazaosin, lisinopril, and Cipro normal for high BP, etc.?",
"src": "Patient: Hello, I am concerned about the medications my husbands doctor just prescribed to him. He went to the doctor last Thursday, He went for a check up and came home with three medications. One was for high blood pressure, and the other one was for stress, and one? Well we ended back today a day earlier then hos original fallow up. He was in pain in his lower back, we thought it could be kiddney stones, he is in alot of pain? the Doctor said it was normal gave him another perscription ,Cipro we just got back home and he is tiered , so he goes to lay down and passes out on his way I am worried can you give me some kind of answer on whats going on? The medications are lorezepam,Terazaosin,lisinopril and now cipro Doctor: Hello!Thank you for asking on HCM!Regarding your concern I would like to explain that there are some interactions between the drugs he is taking. It has been shown that lisinopril and terazosin can cause an exaggerated first dose hypotensive response. This means that when combined together in the first days they can cause an extreme decrease in the blood pressure. Another drug that can cause fatigue and sleepiness is lorazepam, which is a drug which main effect is anxiolytic and sedative. I would recommend you to closely monitor his blood pressure. If it is low, I recommend to consult with the GP and lower the doses of lisinopril and lorazepam. Hope to have been helpful!Greetings! Dr. Iliri"
},
{
"id": 116348,
"tgt": "Suggest treatment for reactive hypoglycemia,low iron and vitamin D levels",
"src": "Patient: I have reactive hypoglycemia and I was just told my iron level is 3 should be 15-30 and I have low vitamin D . I am taking 3,000 IU a day and still came out low. I have fibromyalgia for 20 years. These others things have just happened now. I am 50 years old. Doctor: Hi, dear. I have gone through your question. I can understand your concern. You are suffering from iron deficiency. You should take iron rich food like jeggary, dates, green leafy vegetables, dry fruits etc. You can take oral or injectable iron supplements. For vit D3 you should take cholecalcitriol and calcium tablets. Ypu should search for the cause of reactive hypoglycaemia. You should keep sugar with you in emergency. Consult your doctor and take treatment accordingly. Hope I have answered your question, if you have doubt then I will be happy to answer. Thanks for using health care magic. Wish you a very good health."
},
{
"id": 162588,
"tgt": "What causes red blotches all over the body accompanied with temper tantrums?",
"src": "Patient: Our 7 yr old grand daughter and her daddy, our son, flew up from Virginia Beach for the weekend to visit and we went to Disney on Ice.....of course bought her lots of Frozen items and settled in for the show, ice level. after exiting from the arena, she began to show moodiness on the way to the parking lot....shortly after being in the car she began crying violently, tearing at her clothing, yelling get me out of this city, out of Pennsylvania, I hate myself, nobody loves me, developed red blotches all over her body while crying and carrying on....did not want anyone to touch her to soothe her....lasted probably 45 minutes. She seemed exhausted as were we, wanted to give her a nice bath upon arriving home, but she wouldn t have it. It was a heart wrenching situation and as grandparents, we feel so badly for her. Her mom has said that she has meltdowns , but we never realized it was like this. She had one after going last Tuesday after going halloweening in her neighborhood. Doctor: Hello. Welcome to 'Ask A Doctor'. I have read your query and here is my advice. You must have been frightened when you witnessed your granddaughter's behavior. You did the right thing coming for help. I want to give you insight about her problem. Temper tantrums can occur between one year and five years of age.Children who aren't able to express their feelings with words are more likely to continue having tantrums. These types of children include those with speech and language delays and those with autism spectrum disorders. Tantrums that are severe, frequent, or persist after the age of five years can be a sign of depression, low self-esteem, ADHD. Children who hurt themselves, destroy property, or utter phases which express low self-esteem like, \"I hate myself, no one loves me,\"may have significant emotional problems. Tantrums occurring frequently in school can be a sign of learning disabilities or speech and hearing problems. From what you have shared with me I feel she is not having the usual kind of temper tantrum. Your granddaughter should be evaluated by a child psychiatrist who can provide talk therapy and prescribe medication should the latter be necessary. Hope I have answered your query. If you have any further questions,I will be happy to help. Arnold Zedd, MD, FAAP"
},
{
"id": 84003,
"tgt": "Can losartan cause high pulse rate?",
"src": "Patient: Can losartan cause a high pulse rate? This is a new medication for my high blood pressure, 100 mg. My blood pressure is much better on this, however my pulse rate is elevated to a little over 100 resting. I m actually aware of my heart beating, which worries me. Doctor: Hi,Losartan is an angiotensin receptor blocker commonly prescribed to treat high blood pressure and other conditions. Its common side effects include low blood pressure, increased blood potassium and sometime dry cough. It is not known to cause high pulse rate which could be due to other causes for which you are advised to consult your doctor.Hope I have answered your question. Let me know if I can assist you further. Regards, Dr. Mohammed Taher Ali, General & Family Physician"
},
{
"id": 34200,
"tgt": "What are the symptoms of flu?",
"src": "Patient: Hi, my husband is 58 years old. He is in good health and on Wednesday came down with fever and cold-like symptoms. He took some over the counter meds like Advil Cold & Sinus and Nyquil. Yesterday he developed a red rash all over his body. He went to the clinic and an FNP gave him a steroid shot and an rx for steroid pills. The rash is better today but his jaw is hurting badly and he can hardly talk. Should he go back to the clinic? These are weird symptoms! Oh, they tested him for flu and it was negative. And the fever is gone. thanks. Doctor: Hello and thank you for your questionFlu symptoms are cough, fever and body aches. The rash was probably from the viral infection that was causing his cold like symptoms. If his jaw is hurting so badly that he can hardly talk he needs to go back to the clinic and be seen again. That is not a normal symptom of a viral infection.I hope he is better soonRegards,Dr. Robinson"
},
{
"id": 128719,
"tgt": "Suggest treatment for spondylolisthesis",
"src": "Patient: I was found to have spondylolisthesis at around 12. Fast forward to 2012, and it was found at L4-L5 level, there is a minor disc bulge with minimal flattening of the ventral aspect of the thecal sac with no compromise of the exiting nerve roots. At the L5-S1 level, there is a minor anterolisthesis of L5 relative to S1. The disc bulge extends to the foramen on the left, without any significant impingement of the exiting root. There is an unusual appearance the facet joints bilaterally at the L5-S1 level. No pars defect on either side at L5. There is a small well-corticated bone fragment seen, related to the facet joints bilaterally at this level, larger on the left than the right. Possibly developmental. There was also some superimposed degenerative change at the facet joints. Anterior Spondylilolisthesis of L5-S1(as reported on MRI, CT scan, and from Surgeon Dr. Ganesh Swamy in Calgary, Alberta). Fast forward to now, I moved back to Ontario, I ve had 5 children(yes, you read that right FIVE children), and my pain is significantly worse. I go to my nurse practitioner I m on lyrica(6-9 75mg caps.per day), 8 extra strength Advil, 12 Tylenol 1 s daily to try and manage pain. I got sent to North Bay, Ontario for an MRI, to be told that the only thing wrong with my back now is an ever so slightly slipped disc at L5. None of the other problems exist.I also had an X Ray in 2013 that stated the same. Can someone explain to me how my back got miraculously better, my pain got significantly worse, and maybe an explanation as to what s wrong. I just don t want to be in pain anymore. I want to be able to play with my kids again. I also have had 2 ultrasounds on my liver saying it s large n comparison to my body, and yet when it was felt by my specialist, he sad it felt just fine it doesn t seem large at all. Please, i m begging you, help me. I have nowhere else to turn and Im in pain nearly every day Doctor: Dear patient spondylolisthesis at 2 levels L4l5 and l5s1 back in 2012 and constant increase in pain since then may be due to increase in lishesis and degeneration in facet joints. You are taking high dose of analgesics which itself suggests significant diability. first of all I would like to advise digital of xray of lumbo sacral spine anteroposterior and lateral views. Lateral views should be taken in full flexion and full extension position also known as dynamic views. This gives idea about instability due to lishesis.I would also advise recent mri of lumbo sacral spine with screening of whole spine to know about condition of disc and facet joints. please do it now. please consult expert spine surgeon around you. if xray is showing dynamic instability surgery may be indicated. thanks."
},
{
"id": 158470,
"tgt": "Have hard bumps behind ear. History of breast cancer. Cancer free now. Guide",
"src": "Patient: I have two hard boney lumps behind my right ear along the bone line(marable size). I went to my primary care. She did not know what it was .Definitely not swollen glands. And told me to just watch it. I am a stage 3 Breast Cancer survivor. I had some positive nodes, I have been cancer free for eight years. I don't know what to think. Or do I need to get a second opinion or notify my Breast Oncologist. Your thoughts. Thanks Doctor: Welcome to HCM.You have to be careful for this,Regular follow up check up is required.There are always chances of metastasis.go for certain investigations after consultation of your treating doctor."
},
{
"id": 7243,
"tgt": "I was done fasting insulin test which was 156.3(after 100 gms). will dimet500 help in ovulating ?",
"src": "Patient: my doctor adviced me dimet 500 thrice a day. I was done fasting insulin test which was 156.3(after 100 gms) for one hour insulin and my 2 hour insulin was 199.8 which was twice the normal range. I am having pco and doctor told i have problem in ovulation. will dimet 500 help in ovulating ? I am 27 years old and 57 kg with normal height. having this tablet for past 2 months but could not conceive. I had a miscarriage 7 months back..anticipating your answer. Doctor: Hi Welcome to HealthcareMagic. Dimet helps in regulating glucose levels as well as in promoting conception.Please go ahead with the treatment regimen and have patience.Pregnancy in PCO is not impossible but needs lot of hormonal management.If needed your doctor may advise you to take ovarian stimulating drugs to enhance ovulation and so increase chances of pregnancy. Take care"
},
{
"id": 156328,
"tgt": "What causes tumor in the sinus area?",
"src": "Patient: My 28 year old husband was told he had a tumor in the sinus area. He's been having bleeding from the nose, loss of taste/smell, sinus headaches, congestion, swelling of the eye and it's all been on the left side. What is the prognosis? We are worried and don't see a specialist until Wednesday January 11th. Is there any more information you can give us? Doctor: Hi and welcome of HCM. It is hard to predict prognosis but such tumors are usually very problematic since it is hard to remove it completely and it may infiltrate surrounding tissue very fast. but if this is early stage then it can be removed surgically and chemotherapy and radiotherpy can lead to complete cure.Thanks for the query. Regards"
},
{
"id": 208552,
"tgt": "Can stress cause rapid heart rate and dizziness?",
"src": "Patient: I have rapid heart rate and am dizzy lately. My heart rate goes up to 132 when I am walking around. I am dizzy when I sit. I went to Meijer and it happened there my blood pressure was 146/93. 108 heart rate. I have never had high blood pressure. I have had the same thing before and went to the E..R. where they said I was fine. I am under a ton of stress right now, could that be the reason Doctor: hi..thanks for the query. May I know what is your age & whether you are on any long term medications or received a short course of medications recently? Assuming you have neither of these, first thing we need is to conclusively rule out a condition named arrhythmia which occurs due to dysregulated firing of hearts rhythm generator or pace maker resulting in symptoms of sudden onset pounding heart, lightheadedness or dizziness. This can be ruled out by a ecg. Once arrhythmia is ruled out; what we are seeing is essentially anxiety either situational or occurring suddenly. This can be easily taken care of by giving sos clonazepam/etizolam (benzodizepines) and depending upon cause of your anxiety SSRI group drug like sertraline, escitalram which will take care of your anxiety symptoms altogether. Hence dont worry, consult your primary care doctor & I am sure you will be fine in no time!! good luck"
},
{
"id": 126592,
"tgt": "What can cause swelling and bruises on the ankles?",
"src": "Patient: Hi, why are my ankles so swollen and brusied, some parts are hard, and they hurt a lot plus their warm like I have a fever. They first told me I wasn t getting enough oxigent from my head to my feet, but I think its much more? I also have bionions, they hurt a lot as well. Doctor: Hello, Swelling and bruising in the legs may be due to varicose veins or eczema or deep vein thrombosis or peripheral vascular disease etc. Until examination is done, it is difficult to say what it is. Elevate the limbs while sleeping. Use moisturizer cream locally. Check for kidney and renal problems. If symptoms not improved please consult your doctor, he will examine and treat you accordingly. Hope I have answered your query. Regards, Dr. Penchila Prasad Kandikattu , Internal Medicine Specialist"
},
{
"id": 170606,
"tgt": "Suggest treatment for fever,cough and vomiting in a child",
"src": "Patient: my child she is 4-years-old from past two days she has high fever with temperature of sometimes 103 degrees and sometimes less. she has three episodes of vomiting. she is not taking food. but drinks lots of fluids. she has consistent cough. today she vomited green to light brown then coffee colored . Doctor: Dear friend as a pediatrician I would like to examine the patient clinically... Wanna do USG abdomen, blood test, urine and stool test... Kindly get the patient admitted to local pediatrician and do all needed investigations..."
},
{
"id": 122298,
"tgt": "Could severe leg pain, night sweats, bloating and headaches be due to stress?",
"src": "Patient: having severe leg pain when I get up in the mornings, night sweats, bleeding gums, headaches and a blaaoted tummy. I also am never hungry and have a bruise on my are that just showed up from no where as well as a red pin prick on my other arm. Could this just be stress or should I be worried? Doctor: Hello, As a first line management you can take analgesics like paracetamol or Aceclofenac for pain relief. We have to rule out possible causes like fibromyalgia. Hope I have answered your query. Let me know if I can assist you further. Wishing you all the best. Regards, Dr. Shinas Hussain, General & Family Physician"
},
{
"id": 11315,
"tgt": "Suggest treatment for hair fall with dandruff",
"src": "Patient: Hi, I am Deepthy. I am having much dandruff and couldn't get ridoff from the sever hair fall.. Hair is falling frm the roots.. Please advise what can I do to get ridoff from the fall and keep maintaining my hair. Please advise the good diet which keeps me healthy aswell since I look lean (43 kgs - 26 yrs - 5 feet). Doctor: Hello. Thanks for writing to us at healthcaremagic.In dandruff Or Seborrheic Dermatitis, there is an overgrowth of yeast, Malassezia furfur. This yeast has been implicated in dandruff.Dandruff is a common cause of increased hair shedding. Controlling dandruff effectively would control excessive hair fall too.Various antifungal agents (e.g ketoconazole, sertaconazole, ciclopirox olamine, piroctone olamine) are available either as shampoo or as overnight leave-on lotions, for dandruff.You may try using an OTC antifungal shampoo e.g Nizoral shampoo (contains ketoconazole), every alternate days.However, in some cases shampoo alone may not be totally effective. In such cases, along with the antidandruff shampoo, I usually ask my patients to use a topical steroid lotion (Mometasone furoate Or Clobetasol propionate lotion) once daily at night for a short period of 2 weeks Or i might switch them to an overnight leave-on antidandruff preparation.Seborrheic dermatitis is a steroid responsive condition, therefore, topical steroids are usually effective in this condition. Topical steroid lotions are prescription products.A two weeks course of topical steroids brings about remarkable improvement, which can then be maintained with just an anti-dandruff shampoo, every alternate days.However, if your dandruff does not respond as desired by OTC shampoo, you may visit a dermatologist in your region. Your dermatologist might choose to prescribe a topical steroid lotion for a short duration of 2-4 weeks.Your dermatologist may also want to switch to a different antifungal shampoo Or choose to prescribe an overnight leave-on antidandruff lotion.Regards"
},
{
"id": 57613,
"tgt": "What is Coarsened Hepatic Echogenicity, compatible with a component of Hepatic steatosis in a scan report?",
"src": "Patient: Hi, can you please help me to understand what this means it came up on a scan after being test for my Gallbladder to be removed. What does this mean Coarsened Hepatic Echogenicity, compatible with a component of Hepatic steatosis or other Hepatocellular disease Doctor: Hello Thanks for writing to HCMLiver is normally homogeneous in echotexture.Coarse hepatic echotexture indicates liver parenchymal disease.It may be due many reasons like excessive fatty deposition,cirrhosis etc.You need proper clinical examination and investigations.Investigations include routine hemogram,RBS,LFT,RFT,Lipid profile,elastography. Further investigations can be done if needed.Proper treatment depend upon clinical examination and investigation reports.Hope i have answered your query.Take CareDr.Indu Bhushan"
},
{
"id": 178151,
"tgt": "How to treat loose motion in a 6 months old child?",
"src": "Patient: Hi Dr, My baby is 6 months old and is suffering with loose motions since 5-6 days. His stools are some times watery and sometimes semi solid. Color changed to green since last two days. Medication prescribed by pead is Z n D drops for 10 day and sporlac powder for 3 days. After the stool routine, it is said that the infection is very mild and no need to try on anti boitics at this stage. (pus cells 8-9/hdf is the report result remaining all nil). After giving those medicines (sporlac and z n d given for one day till now), his stool became still darker green color and morw watery. Is that because of the zinc drops? Or sholud i start with anti boitics right now or should wait for 3 more days and decide as prescribed by the pead ? Kindly reply. Doctor: your babby is probbaly suffering from teething diarroeah most of the time no treatment is required except giving ORS .Zn drops will be help to certain extent pre &pro biotics will also help please do not give any antibiotics as on now"
},
{
"id": 970,
"tgt": "How to conceive with PCOD and irregular periods?",
"src": "Patient: hi, i am 25 year old married from 2.5 year from last 8 months i m trying to concieve but not getting the result.i have pcod and very irregular periods.i had an ovalution checking ultrasound done which was normal size of growth of my eggs were normal.my periods are always 8 to 10 days late and during that late period phase i have urine burning and constipation.ive done urine test which was also normal no infection Doctor: Hi, I think you should take some medicines like clomiphene for the growth of your follicles and track your follicles growth by repeated ultrasound and when your follicles is more than 17 to 18 mm, take injection for rupturing the follicles. Be in contact with your husband every 2 to 3 days after your periods stop. Take progesterone for next 2 weeks. Do a urine pregnancy test at home after that. You can try like that for 3 months. Discuss with your doctor regarding this. If it doesn't work then you can go for HSG to see for tubal patency and if it is normal you can try for IUI. Do a semen analysis of your husband. Hope I have answered your question. Regards Dr khushboo"
},
{
"id": 181295,
"tgt": "Suggest remedies for an extra skin growth inside mouth",
"src": "Patient: I have this flap on the inside of my mouth that I've had for about 2 days.- it doesn't HURT excruciatingly when left alone but when touched with my tongue it hurts about a 2 out of 10, so it's minor pain- it's coloration is the same as my cheek, although it does appear to have a white/foggy tiny spot on it-its not a bite because I don't recall me biting my cheek and plus its too high to be a bite -Ive been putting salt water on it before i go to sleep since yesterday -its annoying but I can't feel it in my mouth unless touched with my tongue Doctor: Hi..Welcome to HEALTHCARE MAGIC..I have gone through your query and can understand your concerns..As per your complain a flap of skin over the cheek that is mildly painful and does not cause any botheration can be most probably a Fibroma that occurs due to soft tissue overgrowth and is a harmless soft tissue or fibrous overgrowth..Other causes can be a Wart, skin tag etc..I would suggest you to consult an Oral Physician and get evaluated as if it is a fibroma it will not resolve with salt water and you need to get it removed surgically..So first get a clinical examination done and get it treated as per the cause..Avoid touching it again and again with tongue as it can become irritated, inflamed and sore..Hope this information helps..Thanks and regards.Dr.Honey Nandwani Arora."
},
{
"id": 94372,
"tgt": "History of hernia surgery. Have weight gain, bruise in side, abdominal cramps",
"src": "Patient: I have had hernia surgery 2 times; I ve since gained a significant amount of weight so I think maybe the work is coming undone,it seems as if I never had the corrective surgery because it is just as (if not more) painful; after work I was changing my clothes ( my pants were a little tight in the waist ) and I noticed bruising in my lower abs but not where the waistband would be. I had a relatively large bruise on the lower right side and on the left side and around to the center i had two or three smaller bruises. None of the bruises hurt to touch or even to push on, the only pain was the pain of the hernia feeling as if it were pulling apart (or tearing ) and in my lower abdomen it feels as if I have minor menstral cramps; I have no insurance so avoiding an unneccesary trip to the doctor is ideal for me, if there is anything that one might suggest as to what may be going onandif I can take care of it myself or if it is abso;utely neccessary to seek out a doctor s help Doctor: In cases of unexplained bruises without any trauma one needs to investigate the coagulation status of the blood. Usually these kind of bruises occur due to subcutaneous tiny hemorrhages.There can be various causes which can cause such hemorrhages.I think you should see a physician to get clinical examination done.The doctor may initially want to check your platelet count, bleeding and clotting time. There is always risk of internal bleeding such cases, which can be serious.There is nothing one can do at home. Regards."
},
{
"id": 101009,
"tgt": "Suggest treatment for nasal allergy",
"src": "Patient: Hi Allergy I have been suffering from nasal allergy since my second pregnancy last year - from April 2013( I was 4 months pregnant then )I had severe allergy that time. I was unable to sleep in the nights due to breathing difficulty.I used Allegra, Sudafed, Claritin on and off ( never used as a course as I was pregnant)After delivery, the allergy continued but not as severe as when I was pregnant.I went for allergy testing ( skin pricking test ) in USA in december,2013 and I was diagnosed as allergic to DUST MITES and some other environmental factors too. I was given a Fluticasone nasal spray to use. I used it for 3 weeks and stopped it as they are steroids.I came to India for 2 months ( January and February) . I visited a general doctor and was diagnosed as Allergic Rhinitis. \u00a0I tested CBC ( Complete Blood Count ) test and thyroid and eosinophilia - all were negative.\u00a0 The \u00a0doctor said it is pure allergy caused due to dust. He prescribed me to use Montair LC for 20 days. Iwas feeling better while using the medicine. After stopping the medicine, the symptoms came back. Then I visited doctor again he told me to use the medicine Montair LC for 15 more days and check it. I used it and was fine and the symptoms returned when I stopped using the medicine. Then, I visited an Ayurvedic doctor in India and was asked to use chywanprash, \u00a0SwAsari pravasi, tribhuvankriti Ras and SwAsari ras for 3 months and use Chyawanprash only for 3 years. I started using these Ayurvedic medicines on and off as they have high heat potency. Then, whenever I get severe allergy reactions, I take Montair LC for few days for relief and stop it.\u00a0 ---- what kind of diet is required for this allergy type??? ---- what foods should I ignore???? \u00a0 \u00a0 \u00a0 \u00a0 \u00a0 I used to take lemon, honey and apple cider vinegar mixed with water fora Few days in the early mornings for increasing my resistance. But as I am not supposed to take citrus fruits I stopped taking this mix. I used to eat oranges and orange juice earlier for increasing the vitamin c levels. Now I stopped taking this as they r citrus too. What should I do to have vitamin C \u00a0- any supplements ?\u00a0I used amlaki ( for vitamin c - it has 500 mg) on and off........should I continue this for vitamin c...???? ----- vitamin d - I tested vitamin D levels here last month - my levels are 38.6 ( my multivitamin has 1000 IU ) ----- I am taking prenatal multi vitamin tablets daily since pregnancy. Can I eat wheat bread as it has GLUTEN in it????? ----- instead of milk, can I drink soy milk or Almond milk with cereal or oatmeal in the mornings for breakfast.... ( for calcium intake ) ???? ( earlier when I drank milk at night after eating food, I had nasal congestion ) - then slowly after I get burping and gases go away, the nasal block relieves again ---- sometimes even if I take heavy meals my nose is blocking and the block relieves once my gases pass away. ----- Nuts ( badam, pecans, walnuts, pista, dates, raisins, peanuts ) \u00a0- should I stop eating all these nuts??? ----- I started doing pranayama since the last few days. ----- for omega 3 , I am taking chia seeds one tablespoon (2250 mg) daily is it enough or Do I need fish oil or cod liver oil supplement or algal DHA\u00a0???? -----Please tell me how to increase my immunity levels for combating these allergies My cousin , ent specialist told me to use Xaria Plus for 3 months course and said it will cure my allergies. Is it safe to use it this way? Please clarify these doubts....... Doctor: Allergy can be managed not treated. Keeping this in mind, try to relieve your stress and just avoid dust. Sometimes , high anxiety about your condition leads to a high level of stress leading to symptoms of GERD. Get a course of de-worming in consultation with your doctor.Just try to be as cool as possible"
},
{
"id": 197640,
"tgt": "How to cure brown patch at the base of the penis?",
"src": "Patient: I found a light brown patch of skin at the base of my penis along the hair line , i don't know how long it has been there. it isn't raised not quiet a 1/4\" in size. it soesen't look like a mole more like a birth mark. Is this something i should be concerned about? Doctor: HiWelcome.I have gone through your query.It is difficult to comment on it without examination, so I would either suggest you to see doctor for examination and for exact diagnosis or upload the pictures here, I'll go through them, and provide you with specific advice.Since when you have noted it?Might be fungal infectionHope this helps.Take care"
},
{
"id": 151251,
"tgt": "Internal bleeding in head, headache. What does this sound like?",
"src": "Patient: Hello my name is Kayla, and my husband has been diagnose with having internal bleeding in his head and was put into the hospital.They then sent him home because the bleeding had stopped; and they could not find where the bleeding was comming from inside of his head. He has been back to the hospital multiple times with complaints of horrible head aches along wtih other sysmtoms. What could this sound like to you? Doctor: Hi, welcome to HCM. It IS hard to guess, but a CT or MRI should have picked up the bleed. Kindly consult a neurophysician / neurosurgeon and get well soon. regards"
},
{
"id": 191464,
"tgt": "What causes chapped lops and dry skin while suffering from diabetes?",
"src": "Patient: I am a type 1 diabetic and my sugars have been pretty good but I am experiencing some dehydration. My lips are chapped, the inside of my nose is raw, skin is splitting and bleeding on my fingers and I have overall dry itchy skin. I live in the Midwest and the air is dry but is there anything else that could be contributing to these issues? Doctor: Welcome to Healthcare Magic, Please add multivitamin antioxidant combination once a day in addition to your diabetes medicines. If you have any further query then you can come back to Healthcare Magic, I will definitely help you and guide you appropriately With Regards, Dr Varinder Joshi"
},
{
"id": 74183,
"tgt": "What can cause trouble in breathing in kids?",
"src": "Patient: my five year old has been having trouble breathing at night for the past 5 months and its not a stuffy nose. We have visited with her pediatrician on several occasions and she has been prescribed with claritin and zirtec, I have also tried many otc medications along with humidifiers and air purifiers with no relief from any. my daughter and I are both frustrated and restless what can this be and what can i do to help my daughter? Doctor: Hello dearThanks for using Healthcaremagic.comI have evaluated your query thoroughly .* This seems more of stress , anxiety related and less likely to be pathological issue .Hope this will help you .Welcome for any further guidance .Regards ."
},
{
"id": 145782,
"tgt": "What does sensation in one side of brain with making sounds in sleep imply?",
"src": "Patient: Hello. I fee burning (warm) sensation in the left side of the brain with nick stiffness. Also Yesterday I walk up speaking jeberesh (sounds not words) not sure if there is an correlation. Could you please help me decide if there is something to be concerned about? Doctor: Hello. I have been through your question and understand your concern.According to my point of view you are experiencing some episodes of confusional state. It has many causes with most of them being benign but you should see your GP to start at least with a complete blood count and have you have a objective exam.As I said these are generally benign situation and you should not worry, but neither you should neglect this.Hope this helps. Please feel free using HCM again."
},
{
"id": 42034,
"tgt": "Suggest treatment for infertility problems",
"src": "Patient: My partner and I are undergoing tests for infertility at the moment and his semen test has come back 0% morphology and they were expecting to see at least 4%. My partner has to take another test but now thinks he will never be able to father any children, is this the case? Doctor: welcomeatleast 4 % normal morphology is required to tell normal...but its not always that you will not become parent..there are many options availabe.so dont worry..consult your doctorthanks"
},
{
"id": 84765,
"tgt": "What should i do as my mother got allergic to medicines?",
"src": "Patient: Hello sir, my mother (age around 58) had been to a doctor for regular check up and got to know that her sugar level is 125 before food and 218 after food.On 6th Sept 2012 she went to a diabetes specialist.Doctor prescribed her volibo 0.2 (1 tablet before lunch).My mother body started itching and resulted in big allergy.Later we went to a skin specialist,neither sugar has reduced nor allergy , now complete body of my mother is with white pimples and its itching.Following are the tables , she is currently taking Cream 1. RITCH creamy lotion - 50 ml ( CURATIO ) 2. HALOX cream ( Ranbaxy ) Halobetasol Propionatae 0.05% w/w Tablets: 1. Polaramine 2mg (MSD) Dexchlorapheniamine maelate 2. Averzine-25 ( Canixa) Hydroxyzine Hydrochloride USP 25mg Please suggest ... Doctor: Hello,First of all stop taking the medicines that led to the allergy. Sometimes it takes time to recover. I would suggest you consult your doctor regarding the allergic reaction so that he will note it down and change the medicines for better sugar control. Life style changes asking with exercising daily helps in diabetic patients.Hope I have answered your query. Let me know if I can assist you further. Regards, Dr. Manisha Auradkar, General & Family Physician"
},
{
"id": 115795,
"tgt": "Does Herbalife formula 1 shake elevate uric acid levels in blood?",
"src": "Patient: Hi dr. I am taking the herbalife formula 1 shake for more than a month now.I had my labtest for my uric acid and it seems that the result is elevated.I have been taking medicine for gout since i have gout arthritis for one month and Im already done taking it? I am expecting that the lab result will be normal since I lost also 10 lbs because of Formula 1 shake substitute of my two meals.Do you think the high result caused by the formula shake since I am careful of eating food with high in uric.My uric acid result is 581 umol/L Doctor: Hi, dearI have gone through your question. I can understand your concern. You have gouty arthritis. You should avoid protein rich food and herbal formula. Protein is metabolised in uric acid. You should take drugs like allopurinol according to your doctor's advice. Hope I have answered your question, if you have doubt then I will be happy to answer. Thanks for using health care magic. Wish you a very good health."
},
{
"id": 150732,
"tgt": "Seizures, headaches. Taking eption. EEG, head X-ray done. Suggest?",
"src": "Patient: Dear Doctor, I need your kind advice and for following issue of mine. Following is my shorty summary of treatment. I belongs to Allahabad, during my study days around 21 year ago I was suffered from seizures and approached to Nuro Specialist in Allahabad. After Head Xray and EEG Doctor start giving me AED Tageratol 300 mg but Seizure not fully controlled later on he changed the medicine to Phenytal 30 mg - 3 tablets a day. When again seizure not fully recovered he advice Contrast CT SCAN Brain and reports diagnose me with neurocysticercosis, then Doctor advice me 2 Tab Albendazole for 3 months every morning along with Eption 100 Mg - 3 Tablets in night after 3 months Doctor advice to stop taking Tab Albendazole and keep continue Eption 100 Mg - 3 Tablets in night. Later on after 3 years [Seizure Free] I shifted to Delhi but Eption 100 mg - 3 Tablets continue taking in night. Then I have contact to another Nurologist in Delhi, he again advice the CT Scan and reports were normal hence he advised to reduced the Eption gradually by 3 Tablets to 2 Tablets and I continue taking 2 Tablets of Eption 100 mg then after 1 year Tablet reduced to 1 Tablet [Seizure Free]. Later on again after 2 years I shifted to Mumbai due to my service transfer but now I continue the Eption 100 Mg - 1 Tablet every night. Since I am living seizure free from past 18 years apx and having much active life (Doing Driving 100 Km a day, Swimming, Horse Riding ,Rock Climbing etc, also taking Hard Drinks twice in week ), but still taking Eption 100 Mg - 1 Tablet every night , hence to stop medicine again I contact to Nuro Specialist in Mumbai he gone through my medical history and and advice Test, after according to latest MRI / SCAN reports , which are normal, Doctor me to stop taking Eption 100 MG One Tablet straightaway. So regarding this I need your suggestion and opinion. Now almost 10 days passed , since I stopped taking one Tablet of Eption 100 Mg, and feeling comfortable but little bit worrying seizure recurrence and and feeling headche. Is any chance of recurrence of fits after stoping medicine, why am having slight headache . Doctor: Hello Mohit Sharma Welcome to Health Care Magic As per guidelines usually after a single episode or in some cases two episodes of seizures, patient is advised to take AEDs. As you were also having Neurocysticercosis which was the cause of your seizure, and you had finished the treatment of Neurocysticercosis also. Following which you took phenatoin 300 mg. per day, and presently you are seizure free for about 17-18 years. Usually AED drugs are given for at least 3 years in any epilepsy patient, after 3 years of seizure free period the neurologist taper the drug. In you case you are seizure free for 17 yrs, your scan is also normal so you have to stop taking the medicine as per your Neurologist advise. The anxiety and headache you are having is nothing to do with pharmacological action of drug, it is just the anticipatory anxiety of any untoward episode. Be assured and follow proper seizure prophylaxis like- - For few days avoid driving in night, avoid looking to bright lights. - Try to take meals on time - Don't take alcohol or any other psychoactive substance - Late night awakening to be avoided for some time Get an EEG done and take opinion of neurologist for any residual electrical discharge. Hope I cleared your doubt. Take Care Dr. Seikhoo Bishnoi Psychiatrist"
},
{
"id": 206153,
"tgt": "Suggest treatment for auditory hallucinations",
"src": "Patient: I was getting out of the shower and I looked down and saw a bug. Asked my dad to come and kill it and when he came into the room he said theres nothing there, he checked around the entire room. I looked it up and it said it could be a form of anxiety? but I'm not too sure, my friend told me to try this website, I hope you can help me. Doctor: Hi.I had gone through your query.Hallucination either physiological or pathological.But visual hallucination that occur in fully conscious mind indicate pathological.You have visual hallucination along with fearfulness.It may indicate underlying psychopathology.Proper evaluation will give more confirmatory diagnosis.It can be treated once proper diagnosis will be made. Anti psychotic can help.It can be result of anxiety but it is secondary to that hallucinations.So consult psychiatrist and get help.Happy to help you further.Thank you."
},
{
"id": 110227,
"tgt": "Suggest treatment for back pain",
"src": "Patient: i am 38yrs female. 1 am 5'3\" and my weight is 57kgs. i have pain in my lower back for last 1 year.my mri findings are a)diffuse disc bulge and thickened ligamentum flavum at L5- S1 impinging on thecal sacb)left postero-lateral disc bulge at L3-4 mildly compromising left neural canal Doctor: HIWell come to HCMThe MRI could be suggestive of degenerative disease of spine and in my opinion this can be best treated with complete bed rest for 4 to 6 weak and physiotherapy would be much helpful rather than the medicines, hope this helps."
},
{
"id": 188639,
"tgt": "Suffer from bleeding gums, loose teeth and swelling. Any permanent solution?",
"src": "Patient: Hi Doctor,I am having following problem with my teeth and gums.. please let me know if there is any permanent solution for same.bleeding gums (gums bleed when you brush or floss)bad breathhave an unpleasant taste in your mouthgums are tender, red or swollenloose teeth.feeling pain or discomfort when chewingteeth are sensitive to hot or cold foods or beverageshealth has declined Doctor: Hello, Bleeding gums could be due to poor oral hygiene. You should get your teeth scaled once in 6 months. Brush your teeth twice daily, floss your teeth once daily. Use a mouth wash twice daily. Massage your teeth by finger after brushing. Loose teeth could be due to bone loss around the teeth. You should get an OPG xray done to determine the extent of bone loss. A flap surgery along with placement of bone graft will help in solving the problem. Sensitivity is associated with your periodontal problems. Consult a Periodontist immediatly. Regards"
},
{
"id": 86525,
"tgt": "What are the symptoms of appendicitis?",
"src": "Patient: Hi my 23 ya daughter has had lower abdonimal pain since Monday night, she visited our doctor on Tuesday morning and he prescribed pain killers, these did not assist and we took her to the emergency dept they did a blood test and said that something was going on but the particular test they did was \" non specific\" they ruled out pregnancy and did an inernal for ovarian cysts, they thought it might be early stage appendicitis but sent her home as it is too early to do anything else. Doctor: Hello. I just read through your question. Appendicitis can start as generalized abdominal pain that, over time, localizes to the right lower part of the abdomen. It is typically accompanied by fever and vomiting. From the time of the onset of symptoms until the appendix ruptures is approximately 36-48 hours. So if the symptoms have been going on for longer than this, it is not appendicitis and another diagnosis should be considered."
},
{
"id": 198210,
"tgt": "What does PSA test with history of prostate cancer indicate?",
"src": "Patient: My husband had prostate cancer 4 years ago, he just had a PSA and everything is normal. Is the PSA test the first indicator that something may be going on in his body. He had is whole prostate taken out and also radiation. He had no involement of lymph nodes Doctor: HelloThanks for query .Your husband has Cancer of the prostate and has been successfully treated with Radical Prostatectomy and Radiation 4 years ago.PSA is an enzyme secreted by cells in the prostate .One has to estimate PSA levels frequently even after Prostatectomy to rule out any recurrence of cancer process as there are incidences of recurrence of cancer in residual prostatic tissue in body ..It is one of the most important tools to asses and rue out recurrence of prostate cancer .Dr.Patil."
},
{
"id": 16384,
"tgt": "Fungal Infection Treatment",
"src": "Patient: HI, for the last few months I am suffering from fungal infection. I have taken AF 400 twice a week for almost two and a half months, Gris O.D for almost one and half months, terbiforce once a day for almost one month and now taking zinafate once a day. I have applied creams like Ciprolar, HHZOLE etc. My itching at the groins is not lessening. My whole family had also been suffering, but they appear to be recovering, except my wife, who also taking similar medication. Kindly help why it has occurred and how to get rid of it. Doctor: hi friend, our question is not clear ,first of all -Are you diabetic?because diabetic patients poorly respond to antifungals and secondly you explain about lesions of groin...Is it reddish papules or ring worm like lesions..?Is it limited to groin only..or extended to any other part of body..?I advice go for KOH preparation of skin lesion and get consulted with an experienced Dermatologists.DR."
},
{
"id": 214360,
"tgt": "Suggest natural remedy for irregular period",
"src": "Patient: lcan i use sauna belt to reduce my weight my periods are irregular my fsh level is high periods are delayed last 3 months i checked it is negative iwant to conceiive pls advice what are the other natural remedies i will be very thankful waiiting for the replay`- Doctor: Hello Madam As you mentioned that you are using belt to reduce weight i.e. OBESITY .Irregular BLEEDING.Unable to conceive .Altered FSH level .You want some natural remedy for conception .But one thing I want to tell you about the diagnosis .All these disorder , which you mentioned in your query may be due to Poly Cystic Ovarian Disease ( PCOD or PCOS ) .I recommend you , please get in ultrasound of bot ovaries to rule out the root cause of all these symptoms .If everything comes fine and within normal limit , thereafter think about some NATURAL REMEDY .So consult a gynae and get his opinion .Hope this will help you.Good luck."
},
{
"id": 121431,
"tgt": "How long does a leg fracture take to heal?",
"src": "Patient: my husband suffered D12 compression frature due to aminor car jerk. He is osteoartheritic 78 was active for 6 days before it was diagnosed. after a bed rest of 15 days with a help of hyper extension belt he performes the toilet fution with difficulty and eats normally if seated on a chair.It has been a month now.How long it will take to walk with out assistance.Now he has started walking with a walker. thanks Ragini Doctor: Hello,He is recovering from a D12 compression fracture or a spinal vertebra fracture. This fracture depending upon the severity of the fracture takes about 6 weeks to 12 weeks to completely heal. The mainstay of treatment is to avoid sudden jerky movements to spine and take rest till the fracture heals. Your doctor has already given you the hyper extension grace which will protect the fractured bone till it heals up. Usually within 10-12 weeks he can slowly walk without support depending upon the severity of the fracture. Just follow the doctors advise and he will be better in another 4 weeks.Wish him a speedy recovery. Hope I have answered your query. Let me know if I can assist you further. Regards, Dr. Santosh S Jeevannavar, Orthopedic Surgeon"
},
{
"id": 117852,
"tgt": "Are hemoglobin transfusions effective for low IgG?",
"src": "Patient: My 10 year daughter was recently diagnosised with low igg numbers. We have been working with an immunologist for a couple of years and he has recently recommended hemoglobin transfussons. How helpful are these? are the pain and side effects worth the overall benefits of the treatment? Doctor: Hi, Thanks for asking.After going through clinical history and treatment advised, my opinion is as follows:1. Hemoglobin transfusions are not indicated. 2. However, intravenous immunoglobulin can be given. 3. Immune reactions are common and can be life threatening.4. It is beneficial and risks needs to be titrated.Any further queries, happy to help again."
},
{
"id": 209066,
"tgt": "Are loss of appetite, sleeping trouble and being distant signs of depression?",
"src": "Patient: my husband has been acting different lately. he told me he is not tired of being married but he is not happy. he can not tell me why he is unhappy, he just says he does not know. could he be suffering from a mild case of depression? he had a stroke 4 years ago, could that lead to some sort of depression now? he has not been eating much lately, is distant, and has trouble sleeping good ( but he works graveyard shift). any advise would greatly be appreciated. I don t want my marriage to end over this. Doctor: Hi,I can understand your concern regarding your husband's symptoms. from what you have mentioned, it is quite possible that your husband is having a depressive episode. To link it with stroke is a bit difficult. Usually, post stroke depression starts within 2 years. In your husband's case, it has been 4 years. It may be possible but less likely.In any case, he would need treatment. You should seek a psychiatric consultation for him. Treatment with anti-depressant like escitalopram will be effective.Hope this information was helpful. Best wishes."
},
{
"id": 41866,
"tgt": "Suggest treatment for infertility",
"src": "Patient: hi am 29 years old and trying to b pregnant since past 2 years my doctor priscribed for me letrozole after my result been normal also my husband's result were normal bt after taking letrozole still am not getting pregnant and beside whenever i get my periods i get acidity problems..pls help Doctor: Hi welcome to healthcaremagic.I have gone through your question.Letrozole is nonsteroidal aromatase inhibitor. Have better result compared to clomiphene. Letrzole has lesser chances of multiple gestation than clomiphene.I would advise You should continue with letrozole for atlest 3/4 cycle then can swithover to other medicines.To control acidity, you can take proton pump inhibitors once a day like omeprazole or pantoprazole.Hope i answered your question.Would be happy to help further if needed.Take care."
},
{
"id": 81488,
"tgt": "Suggest remedy for sharp chest pains",
"src": "Patient: Hello I m a 23 year old female been having sharp chest pains. Started a few months ago I would get them randomly and then it would go away. Just last week it started but hasn t went away went the to the er last Friday and they said it was pleurisy never gave me any type of x ray or ct scan just diagnosed me n sent me home w a shot and prescription for naprosyn500mg. I have yet to get the medicine due to money issues at the moment but the pain hurts so bad when I breath and I just took 4 200mg ibprof. N still feel the pain Doctor: Thanks for your question on HCM.In my opinion, you should rule out cardiac and pulmonary causes first for your intermittent chest pain.So get done ECG to rule out heart related causes.Get done chest x ray to rule out pleurisy (inflammation of pleura) and lung related causes.If everything is normal then mostly you have anxiety and related chest discomfort.So better to consult psychiatrist and get done counselling sessions. Try to identify stressors in your life and start working on it. Avoid stress and anxiety. Be relax and calm."
},
{
"id": 10530,
"tgt": "Suggest remedy to prevent hair loss while suffering from hypothyroidism",
"src": "Patient: Hello Doctor, I am 29 yrs old, i have hypothyroid but in normal range, i have severe hairloss from 6-7 yrs, i had been using Biominox 2%, eltroxin 5mcg & Dcal500, but the hairloss has become worst, I feel very weak, fatigue, my gums bleed, my legs especially calf muscles & bones ache a lot even after waking up, now my doctor has put Me on proanagen, my scalp is visible, I ve even done 12 sets of mesotherapy, but hairfall resumed after the treament, Please advice. My Hb was 10 Doctor: Hello and Welcome to \u2018Ask A Doctor\u2019 service. I have reviewed your query and here is my advice. As per your case history of hair fall due to hypothyroidism, my treatment advice is - 1. Take good nutritious diet full of green leafy vegetables and milk. 2. Use a good herbal hair oil and shampoo for routine use. 3. Take an iron supplement and vitamin B12 supplement. Hope I have answered your query. Let me know if I can assist you further."
},
{
"id": 123692,
"tgt": "Does disc removal of collar bone weaken the collar?",
"src": "Patient: I have a broken collar bone that has to be reset. which will need to be plated or just screwed. they have said they will remove the plates or screws once the bone has healed , but my question is, once the srews or plate is removed will this weaken the collar bone where the screws have been or will it heal up Doctor: Hello, Implant removal is a routine procedure and does not worsen or weaken the bone. Hope I have answered your query. Let me know if I can assist you further. Take care Regards, Dr Shinas Hussain, General & Family Physician"
},
{
"id": 209397,
"tgt": "Suggest treatment for psychological issues",
"src": "Patient: hi i m gagandeep singh 19/m wgt:45 sir in my mind every time something is going on some say i have sychology problem but i n't verify my problem doctor i start thinking about what is going around me and mainthing is that when ever there is a pain in body or any part of it a start thinking about it and get nervous so plz tell me Doctor: Hi,Thanks for writingWhat you are explaining might just be a variation of normal human nature, especially at your age. However, in order to be sure about the same, I suggest you consult a psychiatrist once face to face and get evaluated in detail. Hope that helps,Dr A Rao"
},
{
"id": 138939,
"tgt": "Suggest treatment for severe leg pain",
"src": "Patient: HI. After I have been sitting for a while my legs hurt when I stand. I am 45 but I feel as if I were an unhealthy 90 year old. In fact I feel a lot of muscle fatigue. It is mostly upon standing. I am hunched over and barely walk until I get used to moving and then I am fine. Doctor: Hi, you may be having arthritis of the knee joints, or deficiency of essential Vit D3 and calcium.you will need an X Ray of both the knee joint, start with calcium suppliments and Vit D3.Revert back with an x ray."
},
{
"id": 34109,
"tgt": "What causes night sweats after having influenza?",
"src": "Patient: Are night sweats common after the flu? I had flu symptoms - fever, aches, chills, headache and cough for four days. Last night my fever finally subsided and was 99.1 at 8pm. I was and am still battling, a now worse, cough (now terrible chest cold), but thankfully fever is gone. I woke up four times drenched in sweat, flipping my pillows and moving to a diff section of the bed. The 4th I had to get up and change the sheets and pillows. I've had the flu, but it's been years. I must say, I don't recall ever experiencing anything like this afterwards. Is this normal? How many nights could this potentially last? Thank you! Doctor: Hello dear,Thank you for your contact to health care magic.I read and understand your concern. I am Dr Arun Tank answering your concern.Sweating is the bodies response to cover up the temperature in the fever.So whenever you have fever in the end of fever you will have sweating this will counteract the body temperature or you can say it will bring down the temperature.So this will continue until you are cured with the infection or in other word without infection cured this cycle will continue.If you are having sputum culture and sensitivity can be helpful for diagnosis and treatment of the organisms isolated.Please start doing the respiratory exercise. This will be helpful in clearing the chest rapidly.Please maintian good local hygiene this will equally helpful in curing the infections as antibiotics do.I will be happy to answer your further concern on bit.ly/DrArun.Thank you,Dr Arun TankInfectious diseases specialist,HCM."
},
{
"id": 200146,
"tgt": "What should be the minimum sperm count for pregnancy?",
"src": "Patient: Hi, my name is Nithya... We are married for 4 s years and have no child. A test done 2 years back revealed that my husband s sperm count was too less which was 9 million/mL. But now, when we again checked, my husband s sperm count has increased to 40 million/mL. With this count, will I be able to conceive? Doctor: HelloThanks for query.A sperm count of 40 millions /ml is considered to be normal for conception provided other parameters like sperm motility,percentage of active grade one motile sperms and morphology of the sperms are also within normal range of WHO standards.Dr.Patil."
},
{
"id": 85251,
"tgt": "Does sodium monofluorophosphate cause neurotoxicity?",
"src": "Patient: is sodium monofluorophosphate toxic (found in toothpaste) I realize by conventional standards there may be considered a low-toxicity rate but I have heard even small amounts may cause neurotoxicity. People do use toothpaste every day and often multiple times per day. Doctor: Hello,Let me tell you that like other drugs sodium monofluoro-phosphate has some side effects and your doctor has prescribed this medication because he or she has judged that the benefit to you is greater than the risk of side effects. Sodium monofluoro-phosphate can cause gum irritation but it does not cause neurotoxicity.Hope I have answered your query. Let me know if I can assist you further. Regards, Dr. Prabhash Verma, General & Family Physician"
},
{
"id": 84081,
"tgt": "What are the side effects of metformin 1000 mg?",
"src": "Patient: my periods hav been delayed by 5 days.usually i have regular cycles.i m taking metformin 1000mg from the last 6 months,after i know that i hav PCOS problem.i did a pregnancy test today morning but it showed a negative result...i tensed why my priods are late? Doctor: HiMetformin does not affect your fertility in any way. Gastrointestinal symptoms such as cramps,diarrhoea,bloating are common with the drug.Fatigue,low blood sugar,vitamin b12 deficiency are the other side effects of the drug.Repeat your urine pregnancy test 10 days after missing your period.Hope I have answered your query. Let me know if I can assist you further. RegardsDr.Saranya Ramadoss, General and Family Physician"
},
{
"id": 169196,
"tgt": "Suggest treatment for passing stool with white film with stomach cramps",
"src": "Patient: I have a 6 year old girl who the last few days has passed stools which had a whitish film around them. The last two stools she had looked mouldy. She has not complained about stomach cramps and has been eating and behaving normally. She weighs around 20 kilo, and in good health Doctor: Hi...Thank you for consulting in Health Care magic.It seems your kid is having viral diarrhoea. Once it starts it will take 5-7 days to completely get better. Unless the kid's having low urine output or very dull or excessively sleepy or blood in motion or green bilious vomiting...you need not worry. I suggest you use zinc supplements (Z&D drops 1ml once daily for 14 days) & ORS (Each small packet mixed in 200ml of potable water and keep giving sip by sip) as hydration is very important and crucial part of treatment.Regards - Dr. Sumanth"
},
{
"id": 114651,
"tgt": "What causes low ferritin and low Vitamin B-12 levels in the blood?",
"src": "Patient: Hi I recently had a full blood count done at the doctors less than 24 hours I was called back in to repeat the blood test as my blood had abnormal markers low ferritin, vitamin b12 , low blood level I am awaiting second blood results why are my bloods like this and what is the treatment I should also mention that I have been taking iron tablets since April 2016 as ferritin level low then but hasent got better just worse Doctor: hello, Low vitamin b12 occurs quite commonly due inadequate diet or worm infestation. So you should increase milk and dairy products, meat consumption. Also, need deworming with Albendazole. Low ferritin is due to iron deficiency, either due to inadequate intake or loss. So need to stop iron supplements for 3 days and get one stool for occult blood testing. If positive then need further investigation to confirm the site of bleeding."
},
{
"id": 22058,
"tgt": "What is EECP?",
"src": "Patient: I am suffering from Heart problem and doctor advised me anjioplasty.My age is 63 years,height is 5 feet,weight is 55 kg.I am also a patient of Diabities,high blood pressure.I got a heart attack in last july,2010.Please advise me. What is EECP ?is it available in AIIMS and how would be the cost? Doctor: hello, Eecp is basically a method to relieve anginal pain in patients who are not a candidate for angioplasty or bypass. It's benefit is plus minus only. It's not available in AIIMS but in Medanta. If you are advised angioplasty, then you should undergo it and eecp is not a alternative to it."
},
{
"id": 11294,
"tgt": "Is there any side effects of using Finpecia tablets and MX5 for hair loss problems?",
"src": "Patient: I have been facing hair loss for the last 3+ years. When I recently visited a doctor for treatment. He suggested me Finpecia tablets and MX5 or Tugain 5%. Are these good medicines to use? Would there be any side effects of using them? I am worried if these can cause more fall than actually solving the problem. Doctor: Hello. Thank you for writing to us at healthcaremagicYou seem to be suffering from Androgenetic alopecia; Hair thinning and hair fall in males is usually hereditary in origin, known as Androgenetic Alopecia.This type of hair loss in Androgenetic Alopecia is gradual.It has a particular pattern to it; hair fall and thinning is most pronounced at the front and top of the scalp. This may also be associated with gradually progressive recession of anterior hair line.Usually there is a family history in father, siblings, cousins etcMinoxidil and Finasteride are approved remedies for Androgenetic Alopecia. They arrest further progression of thinning and hair fall and even reverse early lossMinoxidil is absolutely safe; it may cause a dull headache if used in more than the recommended amount because it is a vasodilator. The normal recommended amount to use is 1 ml.Finasteride may cause sexual side effects like decreased libido in 1-2% of patients, however, this is a temporary side effects and subsides with continued treatment.Regards"
},
{
"id": 31139,
"tgt": "What causes swollen lymph node in posterior cervical region?",
"src": "Patient: I have a swollen lymph node in my posterior cervial region... A straight line from my jaw to about 1\" behind my ear on the left side of my neck. It swells and goes down just about daily. In the morning its hard and maybe the size of a penny around but you can't see it threw my skin. Then in the afternoon it's barley there... I can still find it but its small, about the size of a pencil eraser around and I'm wondering what that could be from, maybe sleeping wrong? Also I'm a hypochondirac and I have 2 small children and don't get enough sleep. Could stress be a cause? My head itches a lot and has since I had my kids and I have hypothyroidism. I just don't know what to make of it and my doctor rolls her eyes every time I ask about it. THanks for you help! Doctor: hi thanks for posting on healthcaremagic.com after thoroughly reading about your concern, I concluded that swelling in region described could be of several reasons it could be possibly a throat and mouth infection, tuberculous, dermoid or thyroglossal cyst or related to your past history, in all cases it need to be proper examined and investigated by a competent physician which can help you out with proper history of your disease.thanks and take care.Dr Jawad Alam"
},
{
"id": 27472,
"tgt": "Are there any tests to check the heart?",
"src": "Patient: I had a high on a c-reactive protein test that came with a group of tests. This was last summer. Not concerned about my heart till I had a strange shimmer in my heart last night and this morning. Is there more blood tests or other test to check my heart or should I be concerned at all? My father had a heart attack about my age. Doctor: Hello. Thank your for contacting HCM. I carefully read and understood your concerns. The C-reactive protein (CRP) is an established indicator of coronary artery disease risk. People who are higher on CRP have a greater risk for an eventual cardiac event than people who have in within normal ranges. It is, in common words, an indicator of the presence of inflammation within the atherosclerotic plaque (fat build-up in coronary arteries). Other test that can show this phenomenon, although not specific, is a recent discovered factor that is linked with this kind of inflammation - the red blood cell distribution width (RDW). Also, I recommend you to check your blood levels of total cholesterol, LDL fraction and triglycerides.Chest pain, indicative of coronary heart disease - angina, is more like a tenderness, heaviness on the midden part of the chest, it is triggered by physical activity (although it can happen while resting), radiates to the both sides of chest, jaw, left shoulder and left arm. According to your characteristics, you have a positive familial history for cardiac events. Do you smoke? What is your age? Besides blood test, I would recommend you to have a full cardiac check-up with an EKG, echocardiogram and a cardiac stress test.Keep in mind that a series of changes in diet and lifestyle will help improve your risk factors for cardiac disease, such as a diet rich in fibres, poor in salt and a regular, 60 minutes, physical activity a day.I hope I was helpful. Feel free to contact me again. Good day."
},
{
"id": 214284,
"tgt": "Suggest natural remedies for pain in whole body",
"src": "Patient: Hi doctor! I am 18 years old. Almost from last three years, I m suffering from bone aches. Most oftenly my backbone, knees, condyle (left one mostly), pelvis, scapulas,\u00a0metatarsals and ankles hurt very badly. As I m quite young so I believe I can over come these pains very easily without using any Madison. So please suggest me natural remedies for it. Thank you. Doctor: Take proteins and vitamin reached diet. Take one glass of milk in morning. Sleep for at least 8 hours a day."
},
{
"id": 204328,
"tgt": "What causes memory loss and confusion while on Keppra?",
"src": "Patient: Hi my wife who is 51 years and has been up until 2014 very healthy. On the 13/11/14 she got a type of blankness which we brought her to hospital and after consultation with the medical team and after seeing an neouroligist she got diagnosed with epilepsy .....but every month between the 9th and 15th she got this blankness which she gets forgetfulness....after the tablets Kepra @1000/day she went back to probably every two months getting these episodes/forgetfulness, but now they want to increase by 500/day. The common denominator is every time she is ovulation //periods she gets these episodes....any ideas Doctor: Hello and Welcome to \u2018Ask A Doctor\u2019 service. I have reviewed your query and here is my advice. From the available description it appears to me that she is suffering with catamenial epilepsy, a kind of epilepsy with change in pattern according to menstrual cycle. The treatment of this particular form of epilepsy is similar to other epilepsies although it may be less rewarding. Hope I have answered your query. Let me know if I can assist you further."
},
{
"id": 181151,
"tgt": "Suggest treatment for coating on teeth",
"src": "Patient: My mother who is 89 has a coating on her teeth that drives her crazy. We have been to 2 dentists. Both say dry mouth. She does not believe them. Is ther anything that can help her? She has tried special tooth paste,mouth wash,gum,drops,spray. Any ideas? Doctor: Hi..Thanks for the query..Yes, it is true that if she has a dry mouth she can have a false feeling of having a coating on her teeth..But it would not be actually present..So the first priority is to treat dry mouth for which you need to get the exact cause ruled out.Treating the underlying cause can help in resolution of symptoms..The basic treatment is the same what she is given but if it still did not work then medications like Pilocarpine can be advised as it will help in stimulating saliva flow and relieve dry mouth..Also once get her teeth cleaned by scaling procedure to check if she finds any improvement..Hope this helps..Regards.."
},
{
"id": 52710,
"tgt": "Suggest treatment for liver cirrhosis",
"src": "Patient: I had a lobectomy 4yrs ago. They removed the right love of my liver and a 4.3 x 5.0 tumor. Needless to say recovery has been long and difficult with a lot of pain. Naturally I have a left lobe which is fully involved with cirrhosis. So far my left lobe has not shown signs of failure. I would like to nurture any function and regrowth possible. I have taken methadone 120 mg in 3 divided doses per day. Is this the optimal pain reliever or would something else be as effective but gentler on my lobe? Doctor: Hi and welcome to Healthcaremagic. Thank you for your query. I understand your concerns and I will try to help you as much as I can.This is serious disease if cirrhosis is present and quitting alcohol is the most important thing that should be done. Also, medications which are hematotoxic should be avoided. This is necessary to prevent disease progression which is lethal in most cases. At early stages it can be treated with these measures but but in case of progression.Diet should be balanced and healthy and get regular exercise. Limit high-carb foods such as bread, grits, rice, potatoes, and corn. And cut down on drinks with lots of sugar like sports drinks and juice. If there is viral hepatitis as underlying cause then antiviral medications are required.I hope I have answered you query. If you have any further questions you can contact us in every time.Kindly regards. Wish you a good health."
},
{
"id": 7807,
"tgt": "How to get a sure cure of acne ?",
"src": "Patient: hi, if you have been to the doctors before to get something checked out and they tell you that you have something similar to acne what can you ask for to get rid of them ? Doctor: Hi,thanks for query.Acne is very common problem of growing age.The reasons are hormonal changes,oily skin and enviormental effect.Please avoid too much of junk food,take good care of your skin after coming from out side.In case of infected acne you need to take a course of antibiotics to get rid of it.Some local creams also help.In case of severe problem,get in touch with a dermatologist. wishing you good health."
},
{
"id": 185841,
"tgt": "Suggest treatment for swollen jaw line",
"src": "Patient: Hi. My daughter (15 with Aspergers so medical descriptions are sometimes cloudy) has a \"weird\" feeling under her jawline (both sides) which is a bit swollen.. This came about suddenly. She has not eaten anything unusual. Earlier she was complaining that her knees both hurt. They seem swollen too - just below the kneecaps. She says it hurts when she walks. What do I make of all this swelling and whatnot? Doctor: Hello and welcome.Thanks for sharing your concern.Please visit your dentist once and get examined clinically. There is nothing to worry.There might be food debris accumulation and causing pain to her.Inadequate oral hygiene can aggravate her gum problems too.Therefore ask her to do warm saline rinses and please see your dentist.for knee swelling please see your orthopadedics .Dental problems and knee swelling are nowhere related.Hope it helps.Thanks.Take care."
},
{
"id": 11781,
"tgt": "Reddish dry patches on the hand. Due to cold weather? Numbness in the hands",
"src": "Patient: Hi, I have these dry red patches on the back of my hands that basically look like a fingerless glove. What exactly is this? I m thinking it might be caused by the cold weather recently ( 40 F) because, thinking back, I would sometimes really feel numbness in my hands whenever I had to carry something outside. It doesn t hurt or itch , but just feels really dry and uncomfortable if I stretch the skin. Doctor: Hi Thanks for posting your query. Your problem looks like contact dermatitis due to detergents rather than due to clod weather as you have mentioned that is is dry. But your numbness points out to skin manifestation of cold weather too. This has to be confirmed by a dermatologist nearby as the treatment for these two conditions vary from each other. Good day"
},
{
"id": 189872,
"tgt": "Severe pain in tooth upper 5, advised root canal, antibiotic Ciploxtz. Any alternatives to root canal ?",
"src": "Patient: i had seere pain in tooth upper 5_|.the dentist advised root canal . and antibiotic ciploxtz.my question is is the root canal only alternative? can t I go for extraction? direct. I am aged 68 and I have consume ciplox in tons during my life as I suffer from frequent colitis . would it help? I have sever abscess also. or can I use oflaxcin + ornidazole which suits me. Doctor: choice is yours. both rct and extraction will solve your problem, but extraction would leave you with one less tooth . yes you may use ofloxacin ornidazole combination. i can't find any reason except monetary to prfer extraction over root canal treatment thanks"
},
{
"id": 159396,
"tgt": "Suffering from stage 4 small cell cancer. Spread to lungs, liver, brain. Currently at nursing home. Shift to hospital?",
"src": "Patient: My sister has stage 4 small cell cancer . Her cancer is in the lungs, liver , and brain. She has now a fever of 101. She is at a nursing home. The doctor had lab test done (BNT & UA). The nursing home is asking her daughter and I if we want my sister to go to the hospital. Would the hospital be a better choice for my sister or not? Doctor: Dear loretta, You have rightly said that your sister has stage 4 cancer. shifting her to nursing home or to the hospital will be in her best interest as she will get better palliative and terminal care which you canot provide her at home. you will be also comfortable and she will also be comfortable. truly, Dr. j. Ticku"
},
{
"id": 26966,
"tgt": "Could stress be the cause of rapid heart beat and breathlessness?",
"src": "Patient: My 20 year old grand daughter has had 3 episodes of rapid heart beat and breathlessness. She said she's had it before but only periodically and not in a cluster. What could cause this. She says she's not under undue stress right now.e mail YYYY@YYYY Doctor: Hello and thanks for writing.I can understand your concern and would try to help you in the best possible way. Yes, stress, anxiety or panic attacks could be a cause of these symptoms especially in young patients. However if the problem is recurrent then other conditions like hyperthyroidism may be considered.I suggest you consult a doctor who would enquire for her full medical history and perform relevant investigations to rule out secondary causes.In the mean time try to engage her in physical activity, yoga and other relaxation techniques and have family talk sessions to reduce any stress she may have."
},
{
"id": 63624,
"tgt": "What is the hard movable lump on my Jaw?",
"src": "Patient: I have a lump on my jaw about 3/4 of an inch away from my teeth where my jaw starts curving upward. It's hard, but will wiggle side to side. It doesn't hurt at all even when I put pressure on it. Should I go to my dentist? My next cleaning isn't until May. Doctor: Hi dear ,The symptoms you mention in query like hard lump , movable and do not having pain are pointing towards a condition known as ' lipoma ' . A lipoma is a growth of fat cells in a thin, fibrous capsule usually found just below the skin. It doesn't seems to be due to dental infection. Usually lipoma is harmless . It become a course of concern if it increase in size , cause pain or get infected . I would suggest you to consult dermatologist for proper diagnosis and rule out cause like dental infection . Doctor may recommend its surgical removal for aesthetic purpose .Hope your concern has been resolved.Get Well Soon.Best Wishes,Dr. Harry Maheshwari"
},
{
"id": 123133,
"tgt": "What is the treatment for the injury in the tibial plateau?",
"src": "Patient: hello , i had a motor accident before 45 days and i diagnosed with open tibial plateau fracture grade 6 and maybe over that ! I am 24 years old so i would like to ask you which would be the best possible treatment for my injury ! Also what is the prognosis ? thank you in advance ! Doctor: Hello, Consult an orthopedic and a get evaluated.In most cases we can manage by conservative measures. Surgical correction may not be required in most of the cases. Hope I have answered your query. Let me know if I can assist you further. Regards, Dr. Shinas Hussain, General & Family Physician"
},
{
"id": 9561,
"tgt": "Eczema around nose due to dryness. Sugar makes it worse. How to treat it?",
"src": "Patient: hey doc I ve been having dryness around my nose ever since 8th grade due to I think was excessive proactiv. until recently I ve been getting eczema around my nose and have eliminated sugars since I think makes it worse. it s now turned into hot and red nose after I eat some sugar. is this just hereditary and I have to live with it or is there a treatment? Doctor: HiConrad, Thanks for writing in. The condition you are suffering from can be acne rosacea or contact dermatitis. etc. Chances of rosacea are more.Rosacea is treatable with drugs.It is a chronic but treatable condition. Consider visiting your GP or dermatologist for proper diagnosis and treatment. You can apply Hydrocortisone cream over the area till you visit a doctor. Hope this helps. Regards Dr Sudarshan MD Dermatology."
},
{
"id": 184233,
"tgt": "What causes extra sensitivity on the gum line?",
"src": "Patient: I just had a gingivectomy done a week ago. My mouth is all healed, but there is one spot on my gum line that is just extra sensitive. It I touch it, it hurts. If i drink or eat anything and it hits this certain spot it hurts as well. Do you know what the cause could be? Doctor: hey...in my opinion there is nothing much to worry in that..its on the process of healing....all areas wont heal together sometimes...so try not to touch that side for few more days.....do warm salt water gargle twice....and moreover be happy and cool.....have a nice day!!!!"
},
{
"id": 126827,
"tgt": "How can severe shoulder pain be treated?",
"src": "Patient: My husband was on glimepiride 4 mg (2 tblts daily) and pioglitazone Hal 30 mg (1 tblt daily) both together in morning for a month. He stopped taking them due to swollen feet and hands, shortness of breath and severe shoulder pain that he cannot raise his arms, also hips are in pain. He cannot sleep due to pain in shoulders when he lays down. He s up all night walking around the house. What should he do. His doctor said give it time. This Friday will be 3 weeks. Thank you, YYYY@YYYY Doctor: Hi, The shoulder pain is not related to his routine drugs. As a first line of management you can take analgesics like ibuprofen or diclofenac for painrelief. If symptoms persist, you can consult an orthopaedician and get evaluated. Hope I have answered your query. Let me know if I can assist you further."
},
{
"id": 60211,
"tgt": "What precautions should one take during jaundice ?",
"src": "Patient: i have jaundies from last 3 week and my blood report show billbilum level above 2000 and my age is 18 please suggest me whoar should i eat and step that should i take Doctor: Hello will try to help you, It would have been better if you would have given me full LFT report, to differentiate which type of jaundice it is. Cosidering it as viral hepatitis i would like to give you few advices; . Avoid food rich in proteins and fats . Consume easily digestable food. .Sugar cane juice and other fruit juices. . Regular medications as prescribed by your doctor. Hope my advices will help you"
},
{
"id": 49122,
"tgt": "Constant pain in rib, back and shoulder. Ultrasound shows liver cyst and pelviectasis",
"src": "Patient: I have had constant URQ pain (under rib, radiating to back and shoulder) for 10 days. An ultrasound 5 days ago showed only a smaller benign liver cyst and pelviectasis. For the past couple of days I have had pretty constant back pain in both left and right kidney areas. Doctor: hi, kidney cysts can cause pain in loin but smaller sized cysts usually are asymptomatic. your back ache can be mechanical back ache due to sprain or disc problem and is less likely to be due to liver or kidney cysts. RUQ pain can be due to gall stones or gastric pain. as you don't seem to have stones, you may try antacids like rabeprazole along with ibuprofen 400 mg twice a day for 5 days and see the response.hope this helped."
},
{
"id": 52047,
"tgt": "i have WBC and RBC in my urine. Is this something to concern ?",
"src": "Patient: Hi, I am having pain while passing urine. Did my urine test and the result says there are traces of protien and WBC is 20-25,RBC 40-50. Doctor told to do a scaning after a week. Is this anything serious. I am 35 yr old and mother of two. Doctor: hi madam, nothing serious but it needs to be treated adequately. please follow as the doctor told u. get a scan done to see if there is any problem with your kidneys. you will hve to start on treatment with the antibiotic as soon as possible & drink plenty of water. get urine culture slao done so that your doctor can have an ide of what type of organism is growing."
},
{
"id": 112354,
"tgt": "Pain increased after spinal fusion, MRI shows herniated, bulging discs. X-ray shows Anterolisthesis. Solution?",
"src": "Patient: I've had spinal fusion of the L5-S1 6 years ago after deployment to Iraq (spondylilothesis, sorry about spelling), since then pain is increasing and prior MRI shows herniated discs, bulging discs, (L2-3, L3-4) and recent Xrays shows Anterolisthesis of L5 on S1 and mild spurring at L2-L3 and L3-L4. No frank hardware failureor loosening is noted. Doctor: HIThank for asking to HCM I usually put my patient on conservative treatment, which included lumber support, some pain killer medicine, little counseling, if come such spinal problems like you have this gives much better result, and it could be long lasting too, I would love to advise you the same thing, have lumbosecral belt, with any analgesic drug, Diclophenic would be best, have nice day"
},
{
"id": 159130,
"tgt": "Stage 4 colon cancer spreading to liver. Lymph nodes in mensetary swelling. Should I be worried?",
"src": "Patient: i have stage 4 colon cancer that spread to my liver the irenotecan avastin, ect.cocktail has shrunmken the tumors significantly i have been on this regement since march 2010 however lately two pota hepatic lymph nodes in my mensetary have been swelling . my cea,s are 6.8 i am 54 in good health otherwise and recover from my treatments well. i have my infustion every two weeks.should this be a great concern and if so why . thank-you T.S. Doctor: Hi, Enlarged node at porta hehatis signifies distant metastasis. You have been doing well with your colon cancer treatment, that is a positive issue . Probably you may do well again with chemotherapy. In case of disease progression generally next line of chemo has to be selected. If ienotecan has already been used now possibilities are combination of oxaliplatin and irenotecan like FOLFOXIRI regime."
},
{
"id": 64567,
"tgt": "What causes a white color bump on the arm?",
"src": "Patient: I had what looked like a oversized whitehead on the top top of my forearm(about the size of a pencil eraser. I did pop it yesterday and today the skin is very mushy and is draining clear liquid. It is a deep red color and kinda puffy. I am washing it with peroxide. It just doesn't seem to want to scab over. what should I do? Doctor: Hi,thanks for your query to my HCM clinic.I understand the concerns .In my opinion the white lump on the fore- arm-is a well ripe boil on the forearm with necrosed white head discharging after its bursting.Its not scabbing as the its not drained adequately.Hence You need Surgeons help , who would Incise and drain adequately, so that it would heal fast.So I would advise you to consult ER-Surgeon.Hope you would be relieved of your query. with this reply.I would love to help and Wellcome to HCM for more queries."
},
{
"id": 154788,
"tgt": "What does high ESR and high CRP levels mean?",
"src": "Patient: MY MOM IS 64 YRS. SHE HAD LEG PAIN AND SLIGHT SWELLING REDNESS PAIN AND temperature around 100.2F. swelling & redness decreased though. she has 7000 COUNT FOR WBC which is normal and 9.6 for CRP which is supposed to be only 6.0. she has ESR 92.because she has normal WBC and high ESR & HIGH CRP does this mean she has cancer? Doctor: Hi, dearI have gone through your question. I can understand your concern. She has very high esr and slightly high crp. Both ofcthem are suggestive of some inflammatory disease. High esr can also be seen in tuberculosis,anemia, autoimmune disease, paraprotienemia, malignancies etc. However chances of malignancy are low. She should search for the cause of high esr and then take treatment accordingly. Hope I have answered your question, if you have doubt then I will be happy to answer. Thanks for using health care magic. Wish you a very good health."
},
{
"id": 82485,
"tgt": "What causes chest discomfort and shortness of breath after smoking hookah?",
"src": "Patient: Well I smoke regularly from hookah and I don t usually experience symptoms, but today after a session, I laid down and I feel short of breath like the breaths I take are deeper now and if I cough, you can feel a sort of swollen feeling in the throat leading to the chest area. I m actually considering quitting the hookah. Is this a temporary symptom due to the tobacco smoke from The hookah? Doctor: Thanks for your question on HCM. In my opinion you should get done chest x ray and PFT (Pulmonary Function Test) to rule out chronic bronchitis. Chronic smokers are prone to have this and breathless is the first symptom of it.So possibility of chronic bronchitis is more in your case.So better to consult pulmonologist and get done chest x ray and PFT.And quit your smoking habit. It is not good."
},
{
"id": 6288,
"tgt": "Trying to conceive, purpose cervical flushing, previously caesarian delivery",
"src": "Patient: i want to get pregnant again since my child is already nine years old.but my obgyne told me that i have to undergone cervical flushing so that my intestine and ovary will separate.maybe because i was a caesarian before that is why my intestine and ovary are together.what is the purpose of cervical flushing?please help me with my problem. Doctor: Hi welcome to HCMForum. So far my knowledge is concerned cervical flushing is a term for in vitro fertilization. {test tube baby} but the chances of succes are questionable. Intestinal adhesions are not advisible to retain. I advise you to consult a gastroenterologist for confermation of any other sites also after having c.t.scan"
},
{
"id": 187404,
"tgt": "What causes blisters and sore in mouth after neck surgery?",
"src": "Patient: Hello about a week before I had surgery on my neck which involved three levels a plate and screw put in I started with my mouth like a little blood blister in it after surgery which is just little over a week and half my mouth is soar with more blisters I brush wash with warm salt water and use non mint ,or scope mouth wash what can this be ??? I can not go out yet and move around too much please help Thanks. Doctor: Hello, Welcome Thanks for consulting HCM, I have gone through your query, as you have mentioned that you have undergone surgery of neck now you have blood blister and sore mouth, dont worry you first consult your doctor for this , if he said it is not due to neck surgery then should consult dentist and go for Oral Examination and investigations if necessary As you are asking what it can be so it can be due to viral infection may be oral Herpes infection , or may be allergy induced or may be cancer sore or Angina bullosa haemorrhagicaSo consult dentist for Oral examination as early as possible,Hope this will help you. Wishing you good health."
},
{
"id": 87904,
"tgt": "What causes stomach pain and shortness of breath?",
"src": "Patient: i have been experiencing stomach pains, nausea, dizziness and breathlessness for a few months, i am a fifteen year old girl and have started my periods even though they are irregular. I often feel out of breath even when talking like i have just done exercise. whats wrong with me? Doctor: hello. my name is dr waleed. actually it is not possible from the history you have given to reach a diagnosis but i can tell you that you must see a doctor very soon so that he can take a complete history and do a physical examination and then do necessary investigations. in this way it would be possible to reach the diagnosis. From whatever history you have given, i can tell you this. you may possibly be losing too much blood that can make you anemic. Severe anemia can make you breathless and cause you dizziness(light-headedness) but that will not explain the nausea though. Also, if you meant \"vertigo ie the world spins around\" by dizziness that would be more alarming though. a benign pituitary tumour can be a possible cause too that cause the nausea,vertigo, and irregular menses. if you have vision problems too, then it can be more likely. A pituitary tumour would not explain not the abdominal pain unless unrelated.another possibility is that you are bleeding into your stomach or intestine (is your stool colour blacK?, if so think of bleeding into stomach or early part of small intestine). it will explain the abdominal pain nausea lightheadedness and breathlessness.Polyarteritis nodosa can explain all these symptoms except the irregular vaginal bleeding.Conclusion: please go to your doctor soon for a full workup"
},
{
"id": 43523,
"tgt": "Trying to conceive. Taken semen analysis, USG pelvis. Why am i not getting pregnant? Suggestions?",
"src": "Patient: hello doctor .my name is maheen..age 20, weight 51,i am married and it is being one year age I am trying to conceive but I am not getting pregnant...I also visit doctor they suggest my huband semen analysiswhich report comes normal then she take ultrasound of me which name is usg pelvis...report is here endommetium appear normal measuring 3mm in thickness..multiple follicles of vaiable size appear in both ovaries follicle measuring 17*19mm in right sided...can you guide me why I am not getting pregnant whats wrong with me..any suggestion you want to recommend me Doctor: Hello welcome to health care magic madam maheen you have told that your husband semen analysis report came normal,,it shoukd include normal count and also normal motility..if motility is not there..there lies a problem.. secondly,your USG report,,ovarian follicular size are normal... What about your menses whether they are regular or not..if not go for hormonal studies.. If your menses are normal and normal flow,,,then go for tubal(fallopian tubes) patency tests using hysterosalpingography after consulting infertilty specialist.. Hope you understand my suggestion and answer.. Hope you become pregnant soon thank you Dr.siddartha"
},
{
"id": 207783,
"tgt": "Suggest remedy to overcome insecurity, panic, anxiety and increased sex drive",
"src": "Patient: I AM FACING MENTAL PROBLEM SINCE 5 YRS. MY PROBLE IS UNSECURITY, FEARFUL THOUGHTS,PANIC ANXITY AND HIGH SEXUAL THOUGHT TO OVERCOME FROM THIS PROBLEM.TAKING MEDICINES .PL. SUGGEST HOW TO OVERCOME FROM THIS PROBLEMS WITHOUT MEDICINES? I AM DOING SUDARSHANKRIYA, LISTNING BRAHMAHA KUMARIES PROGREM DAILY FOR 30 MIN.ALSO READING BOOKS ON THIS PROBLEM, I AM CONFUSED WHETHER TO DO ART OF LIVING OR ONLY WALKING OR OM SHANTI OR HAPPY THOUGHT OR MANSHAKTI OR REKI OR HIPNOTHEROPATHY OR ANY OTHER PATHY.PL GIVE CONSULTATION. Doctor: DearWe understand your concernsI went through your desciption. I suggest you not to worry too much. Please understand that psychiatry / psychology is a deep health science. Prolonged treatment may be necessary. Proper assessment is required to have proper diagnosis and treatment. Please consult a psychologist for psychometric assessment and advise on further treatment methodology. Yoga, meditation and psychotherapy are proven to assist the recovery procedure. But they cannot cure you if used alone. You must also work with your psychiatrist so that he can help you. Getting anxious about your mental conditions aggrevate symptoms and those meditation methods cnnot help you in that case. If you need more of my help in this regard, please post a direct query on this portal. I am happy to help you.Hope this answers your query. Available for further clarifications.Good luck."
},
{
"id": 154482,
"tgt": "How to treat ovarian metastases?",
"src": "Patient: very gud afternoon sir, my mother she is 58 years of old, the problem for my mother for last year october 2010, started first with stomac ache then immediately we are under gon efor diagnosis with biopsy, later they told it is HB positive and symtoms are like cancer, imediately we approached oncology specialists and they said very clear 3 chemeo therpahys forst then surgery ( uterus cut) after that again 3 chemo after that form last 3 month she is ok, but form last one week she gets again stomach ache immediately we met doctor and he said mild water is formed on ovarian ad he proposed to do test, the cytalogy report is positive metastas. please advice and help out in this Doctor: Hi, Your mother has started accumulating fluid in the peritoneal cavity which is otherwise called as ascites. The treatment for this is chemotherapy. Those drugs which were not given during the previous chemo will be given now. Also they can attempt intra peritoneal chemo in which chemo drugs are directly instilled in the peritoneal cavity."
},
{
"id": 108777,
"tgt": "Suggest remedy for pain in back",
"src": "Patient: hallo,Am 29yrs old with a height of 166.I have this pain which start in my right lower back,then to my hips and now going down to my legs and to both legs.when i sit the pain is not there at all,when standing the pain is worse and now i cant walk for more than 5 minutes without sitting down not even standing for a while and problem when sleeping or lifting amything.It started 4 months after having my baby then I thought it was epidural and the doctors says no.Some told me I have disk location,vain inflamation ,lumbago which they claim was to go in six month naturally.they just send me to the physiotherapy which just worsen it even more and give me alvedon and diclo which never release any pain .when sent to mri after 18 months they said that it was something that they have never seen before in the whole sweden.then they did all the test including the tb and hiv which were both negative.then they did the punchture on the affected area on the back and they say that it was TB in my lower back.now am on TH medication but walking is still like going to hell.my legs get to much numbs ,restless ,weak and so heavy until I have to sit and rest.cant go to stairs or even just a slightly hill.Will I ever get well and be able to walk again normally?.And is it true this thing was never caused but epidural?And can you have pain in the back and not in any other part of the body?because they say i dont have in the chest and that it is TB that i cant spread to any other person. Doctor: Hi,The pain you are experiencing is from that of a pinched nerve. It could be due a number of causes like bulged or herniated disc, wedge compression, vertebral collapse fracture or TB of spine or other cancers. You also mentioned that it was diagnosed to be from TB of spine. Has it been confirmed by a tuberculin skin test or a biopsy of the vertebrae affected? If on taking TB medications, the pain or numbness do not begin to resolve with in 2-3 months then it is most probably not TB spine. Yes it is true that TB spine is nontransmissible unless there is direct exposure to the infected tissue without proper gear. You should first consult with a spine specialist and get the diagnosis confirmed before going ahead with taking medications. Radiological confirmation is not the gold standard. Culture of the tuberculous bacteria is the gold standard test."
},
{
"id": 202003,
"tgt": "Suggest treatment for penile infection",
"src": "Patient: Hi Doc,Good PM, I have a Pus discharge from my Penis, through Gram Stain/Culture Sensitivity the Urethral Discharge Specimen showed that I have the following:Organism = Streptococcus AgalactiaeCeftiaxone - SClindamycin - RPenicilin G - SLinezoid - SChloramphenicol - SErthromycin - IVancomycin - SFrom the choices what Antibiotic should I take? Doctor: Hello Thanks for writing to usAccording to reports you are suffering from urinary tract infection due to bacteria streptococcus agalactiae and it is sensitive to cetriaxone.It is sensitive to other drugs also.You can take ceftriaxone 1 gram intravenous twice daily for seven days after skin test.You should also take alkalizing syrup and drink lot of water.Hope i have answered your query.Take Care Dr.Indu Bhushan"
},
{
"id": 120160,
"tgt": "What causes vein in bicep to ball up to a size of large bean?",
"src": "Patient: My right Bicep vein seems to ball up to the size of a large lima bean at the location of the bicep vein where it meets and disappears under the shoulder muscle. No pain. No redness or swelling of the arm. No unusual sensations at all. Seems to have become bigger over the last 2 months and only occurs when working out (lifting weights/ cardio). When i am at rest it is not visible. Afraid its a clog or clot or maybe the blood is pooling up here and could burst the vein? Doctor: Hello, The vein at Biceps is known as cephalic vein.This is one of major draining vessel from upper limb.During exercises output from limb increases,this is reason why this vein swelled up during exercise.It is a normal phenomenon. Veins do not burst during exercises instead they open other channels in hard workers.So please do not bother about that. Hope I have answered your query. Let me know if I can assist you further. Take care Regards, Dr. Mukesh Tiwari"
},
{
"id": 115756,
"tgt": "Why does one need frequent blood transfusions?",
"src": "Patient: a friend of mine is getting blood transfusion 5 times a week because he says there is an imortant ingredient in his blood missing. he doesnt disclose anything. i apprehend that could it be some kind of cancer? why else would you need transfusion five times a week? Doctor: Hello,I understand your concern.I am Dr. Arun Tank, infectious diseases specialist, answering your query.In my opinion your friend is having thalassemia.It is the condition where haemoglobin formation is not completed. So always there is deficiency of haemoglobin.In severe case of it this causes very low haemoglobin. This require compulsory blood transfusion.I think your friend is in this stage so it requires a blood transfusion. It is not contagious disease so you do. Of have to worry for it.I will be happy to answer your further concern, you can ask me on bit.ly/DrArun. Thank you.Dr Arun TankInfectious diseases specialist."
},
{
"id": 209723,
"tgt": "Suggest medication to control anxiety",
"src": "Patient: AnxietyLately I have been having Anxiety really bad I'm going through a divorce. I have moments when I feel like I cant breathe and feel like I'm having a heart attack. Once I sorta calm down it starts to get better. Is there any medication that can help? Doctor: DearWe understand your concernsI went through your details. I suggest you not to worry much. You are going through a period of disappointment and dissatisfaction. The series of events points to that. Going through the process of divorce definitely is a anxiety provoking circumstance. But, just for this purpose, why you want to take medicines to cure anxiety. That anxiety process is over. Only thing you have to understand is that due to anxiety you are never going to have heart attack or any other physical problems. Try to calm down. Exercise, breath well and have sufficient water.If you really want to have medicines, please visit a psychiatrist. Psychiatrist should assess you well before prescribing medication. You cannot be properly diagnosed online with the given symptoms. Self medication and diagnosis can be dangerous. If you require more of my help in this aspect,Please post a direct question to me in this website. Make sure that you include every minute details possible. I shall prescribe some psychotherapy techniques which should help you cure your condition further.Hope this answers your query. Available for further clarifications.Good luck."
},
{
"id": 105496,
"tgt": "Child having recurring allergy, has early morning sneezes, continuous congestion and nasal blockage. What treatment should be taken?",
"src": "Patient: Hi Doctor, My son has been sufferring from the tendency of allergy and this has been right from his birth. He had in his childhood shown sighns of brethlessness on multiple occations , some doctors termed this as Asthma , while others told it is Rhinitis . He does have tendency to sneeze aerly morning , especially during the monsoons and sneezes around 10 -12 times after getting up before he gets normal. I have had him treated with Homoeopathy , allopathy and Ayurvedic medicines but the effects seems to be very short lived. Could you please help us ? He is already eigth years old but problems don t seem to disappear. Although , breathless complains have reduced, he seems to be a manufacturing house of mucous inside, that block his nasal passage , when the problem happens. A change of weather is a good trigger. Many thanks for the much required help in advance. Doctor: Hello, I am sorry to note that your son has been suffering from possibly allergic rhinitis/asthma and the condition seems to be worsening. At the onset, it is important to note that allergic rhinitis when remains untreated for long eventually leads to asthma-like complications and full blown asthma. There is a lot of research to support this concept and hence treatment of both rhinitis and asthma may be required depending on the stage of presentation. You need your son to be seen by an Allergist for either skin prick tests to elucidate the actual allergenic triggers or blood tests (specific IgE) where the same can be identified. A strict avoidance together with optimal treatment for both rhinitis and asthma is then required. The final option of an almost 'cure' for allergy is desensitization and either injection immunotherapy or sublingual immunotherapy over 3 years is required."
},
{
"id": 176064,
"tgt": "What causes breathing difficulty and coughing?",
"src": "Patient: My son aryam is 5 years and 8 months weighs 18 kg. he is having fever since Friday and also have cough since Wednesday. I have taken him to a paediatrician and he has diagnosed it as viral infection. he is quiet active and playful when there is no fever. but have increased and noticeable coughing especially while eating and during sleeping. the existing doc says that its due to the viral infection. he had severe cough in the during 24-25th december 2013 and was administrated with antibiotic distaclor -125mg for 8 days. further was adviced for pentids -400 for next 30 days with asthalin and cetzene. he gets burning sensation in chest while coughing. there is some discomfort while breathing due to congestion. there is no flowing nose. Doctor: Hi,From history it seems that there might be having viral infection or mixed with bacterial infection.It is advisable to give him one course of antibiotic medicine for 5-7 days.If viral infection is there, gradually it will be alright but if there is mixed or bacterial infection is there there might be having risk of producing pneumonia if antibiotic not given.Give him cough suppressant syrup as well.Ok and take care."
},
{
"id": 111129,
"tgt": "What causes back pain?",
"src": "Patient: Hello Doctor , I am feeling lot of lower and up back pain. As per my doctor Advice , I am using Gemcal D3 and Neuben capsules. Please let me know if I would get any side effects and let me know which tablets to use to decrese the pain. I am living in Mexico now. Doctor: Hi,As per your question-there is no side effect reported till date for the drug mentioned. To alleviate pain you can take dicofenac sodium only when pain becomes unbearable. I have told you the salt name .you can take it of any company convenient to you.try to do some physio to decrease the intake of pain killer.Best of luck! :)"
},
{
"id": 68198,
"tgt": "Reason for lump just below the hip bone?",
"src": "Patient: hi, i have a hard lump just below my hip bone. i am a male, 44 yr old. i get bumps from time to time, but haven't one in about 5 yrs. tis one seems slightly tho, as it has to head... but when i squeeze it, a little bit of white pus comes out. also about an inch right around the 'head' there is a hard lump underneath the skin. i suppose it is where the pus has gathered. can you tell me what this might be? Doctor: Hi, dear. I have gone through your question. I can understand your concern. You may have infected cyst or abscess formation. So pus comes out from it. You should go for examination. If it is abscess then go for incision and drainage. If it is some cyst or other lesion then go for removal of it. Then do hp examination. Hope I have answered your question, if you have doubt then I will be happy to answer. Thanks for using health care magic. Wish you a very good health."
},
{
"id": 95061,
"tgt": "Intermittent upper abdominal pain, frequent bloating, belching, foul smelling flatulence, shoulder pain post spleen removal. Cause?",
"src": "Patient: Hello Dr. Ghosh, I am kobby. About 6 years ago a had my spleen removed. After sometime, I have been feeling intermitent pains in my upper right abdomain, just below my ribs, with frequent bloating , belching as well as pains in my shoulder muscles. I often have flatulence with foul smell as well.Most often it seems to get worse after eating. Thank you. Doctor: Hello isdontwi pain (Myalgias) associated with Sore Throat, Diarrhea &Vomiting with Rigor- need urgent treatment. Advice-Vaccination aganist Pneumoniae, Haemophilus Influnze,& Meningitis. simple pain -non specific frequent bloating, belching & flatulence with foul smelling- due to constipation ,amoebic colitis& aerophagia Advice- correct consption, colitis & prevent air during swalloing . hope you will enjoy positive health. thank you."
},
{
"id": 77980,
"tgt": "What causes pain in my left side of my valves?",
"src": "Patient: hi, i'm 30 years old,female. i have problems with one of my valves,but not on medication,the docter said it is not that big of a problem. i have a pain in my left side for years and now that i am using duromine i get it more and then i don't feel so good my heart beets very fast. must i stop using duromine. Doctor: Thanks for your question on Health Care Magic. I can understand your concern. Yes, you should definitely stop duromine tablets. Duromine is having phentermine. It is sympathomimetic drug. It stimulates heart and cause increase in heart rate (tachycardia). So your fast heart beats are due to duromine only. You already have valvular problem. And tachycardia can actually exaggerate and worsen the ongoing valvular problem. So better to consult cardiologist and get repeat ecg and 2d echo to rule worsening of valvular heart disease. And don't take duromine or any drugs without consulting doctor. Hope I have solved your query. I will be happy to help you further. Wish you good health. Thanks."
},
{
"id": 213376,
"tgt": "How can I control exam anxiety? What medication?",
"src": "Patient: hello doctor.... i m a physiotherapist and i have just completed my last year...and unfortunately i hv nt scored upto my mark due to some unexpected things that took place during my exams.... because of my hostel batch mates, who came on previous day of my final exam and told me certain things due to which i wz very disturbed....even when i wz knowing that my final year university exams are imp bt i wz nt able to get control on my anxiety ..... and whn i m in such situation i m nt able to defend my self even if i m right....with such anxiety and depression problem i went to a doctor who gave me some medicine (xet CR 12.5 and etilaam 0.25) bt sir i m really scared to take this medicines as they are addictive and i might not able to go through its withdrawal symtoms.....sir i hv to again go to my hospital for internship n face all this situation again...And i hv just taken 1 tablet of xet CR 12.5....plz help me sir Doctor: hi u should not be scared as these medicines are not addictive if taken on advice of a psychiatrist. u have to be in regular followup as directed.dont just panic my dear.have faith on ur doctor.if u r not satisfied u can take a second opinion with another psychiatrist."
},
{
"id": 153340,
"tgt": "What are the symptoms of liver cancer?",
"src": "Patient: Hi my father is already in hospital but doctors don't know what he have because they don't are experienced doctorsmy father have yellow eyes and yellow skin he don't eat anything he have the symptom of liver disease but he don't have pain in the abdomen or anywhere we think he have liver cancer or somethink like this can you give me a reply to understand what he actually have because i'm in a very hard situation! Doctor: hi.your father's clinical picture has varied possibilities, from infectious to a cancerous pathology, based from your description alone. several diagnostic examinations - blood examinations (esp liver function testing and bilirubins) and imaging should be done to further evaluate your father's condition. if it's available in your place, CT-scan will be of great help.hope this helps.good day!!~dr.kaye"
},
{
"id": 81830,
"tgt": "Interpret results of needle biopsy",
"src": "Patient: I was hospitalized with chest pain and pneumonia with a quarter size lesion on my right lung. First a bronchoscopy was performed and I was told my lung was too inflamed to take a biopsy. 2 days later a needle biopsy was performed, but I still do not understand the results. I am still having chest pain and don t follow up with my pulmonary doc for weeks. I m hoping someone can help me better understand the findings which I have copied and are as follows: Sections show an acute lung injury pattern with reactive type 2 cells. They are associated with acute and chronic inflammatory cells infiltrating including some eosinophils, and as well as neutrophils, some of which are in alveolar walls and raise the possibility of so-called capillaritis. In addition, there is some airspace fibrin and very focal organizing pneumonia(it is thought to be the dominant pattern). It is hard to know what these findings might mean. I think infection is certainly a differential. I did note that the BAL also grew candida, and although this really does look very much like Candida pneumonia, I would still do at least a GMS stain on the tissue. The differential also includes aspiration, although I do not see definitive foreign material. The presence of an acute lung injury pattern with areas suspicious for capillaritis, also raises the possibility of this to be a manifestation of granulomatosis with polyangiitis (wegener s granulomatosis), or at least likely another vasculitic process. A GMS stain is performed and demonstrates no fungul or yeast like organisms. I am undergoing further blood tests and follow up with my pulmonary doc in a few weeks. I really am desperate to better understand these results. Doctor: Can u Share your CT scan plates and reports to know exactly how much of the lung is involved and how many lesions."
},
{
"id": 174142,
"tgt": "What is the medication for cold and cough during winter?",
"src": "Patient: hello sir, My son is 1.5 yrs old and he is suffering from cough and cold. Last year also in winters he got the problem of bronchitis and this year from starting only he is suffering from cold. can u suggest me something which i can use the whole winter to avoid cough and cold and also the problem of bronchitis? Doctor: Hi,Thank you for asking question on health care magic.Stem inhalation at once daily if possible.Local instillation of saline nasal drops 2-3 times daily.Zn-20 syrup 5 ml daily may prevent repeated colds and cough.You may try Flucold syrup 3 times daily for relief of cold.Karvol applicap to be squeezed on a hand kerchief and pin it to his garments so that he will be inhaling the smell continuously.Hope this answer will serve your purpose.Keep the child warm and avoid exposure to cold atmosphere,cold foods and cold drinks.Please feel free to ask any more queries if requiredTake careDr.M.V.Subrahmanyam MD;DCHAssociate professor of pediatrics"
},
{
"id": 107350,
"tgt": "What causes continuous pain and inflammation in the lower back?",
"src": "Patient: Hi doctor, I am a 28 year old lady I been having a some pains with burning sensation on my lower back for 6 months now very uncomfortable, have been to so many doctors , when it first started doctors gave me antibiotics thinking is an infection and that didn t work later went for CT scan and ultra scan and all was normal now it seems all the doctors can t find what is exactly wrong with me and this worries me. Doctor: Hello, I just read your query. If ct scan and other investigations are normal then it might be due to muscle strain which may be due to lifting heavy object again and again, twisting, excessive physical work which may lead to restlessness or due to sudden jerky movements.* Take complete rest for some days as it will help your body to overcome strain if any.* You can also use heat or ice packs as both these helps in reducing inflammation and relieves pain.* You can also try this natural remedy. Take 50 to 60 ml of coconut oil or sesame oil and fry 7 to 8 cloves of garlic in it and fry till the garlic turns brown. Apply this oil when lukewarm on affected area and do gentle massage for 5 to 10 minutes. It will help you in relieving pain.* Try all these things for 10 to 20 days and you will feel positive results.* In case of any emergency you can consult a nearby orthopedic physician.* I hope you find my answer helpful.Thank you."
},
{
"id": 82705,
"tgt": "What is mixed connective tissue disease?",
"src": "Patient: I have mixed connective tissue disease. I am 44 years old. And weight 205 this morning. I am on leflunomide and prednisone for the MCTD as well as 2 diauretics, elavil, crestor, lisinopril potassium, high dose vit.D and lotab as needed for pain. I had swelling in the soft tissue in the hollows of both shoulders which were negative under MRI. Now the collarbones on both sides hurt and the pain gets worse if I run my fingers over them. The pain radiates into both shoulders. Is this related to the MCTD or is it something I need to get checked out tonight? It started about 6:00 pm my time and has been getting worse. Doctor: It could be related to MCTD , get your CPK checked . A possibility of cervical spondylosis also exists ."
},
{
"id": 70387,
"tgt": "Does blood donation cause a lump on the elbow,and if not,what could it be?",
"src": "Patient: My daughter has a very small round lump in the inner part of her elbow. She says it feels bruised in that area. No sign of bruising on the skin though. Its on the same place where she donated blood beginning of last year. What could this be? Regards Tessa Doctor: Hello!Thank you for the query.Yes, skin irritation with needle can cause a lump. Usually it is a fibroma, benign, painless lump which slowly grows. It does not turn into cancer.The best way to find out exactly what she is dealing with, is to remove if and send to histopathology.Please consult a surgeon.Hope this will help.Regards."
},
{
"id": 151050,
"tgt": "Severe pain from neck to chest, breathing issues, no improvement with medicines. Remedial measures?",
"src": "Patient: Hello, this is with respect to my wife health. My wife has been facing severe pain starting from left side of her neck till left lower part of the chest. She faces breathing issues too. We have been consulting different doctors since last 4-5 months with no great success. The last survical CT scan report showed some disbalance in C4-C5. She has been taking tablets such as Skelebenz , Estocrine , few pain killer medicines as well. Please suggest what should be done to reduce her neck pain and remove her breathing issue. Her current physical status is as below: Height: 5.6 Weight: 84 kgs Note: We have done test for Thyroid , Sugar and the report seems OK. She has also been taking medicines related to High Blood Pressure . Doctor: Hello, Welcome to health care magic forum, CT scan report showed some imbalance in C4-C5, and all problems are due to that. so, consult to near by orthopedic surgeon, he will guide according to problem, like some medicine, physiotherapy, cervical belt, etc. For more inquiry you may contact on \u201csubhashponkiya@gmail.com\u201d Have a good health"
},
{
"id": 142912,
"tgt": "What causes tingling on lips with blurred vision and headaches?",
"src": "Patient: My sons symptoms are (he is 30 Yrs old)... constant tingling, peeling, burning, swollen lips, he said, it feels like his lips are raw and have been cut constantly. Other symptoms right eye becomes blurry, feels pressure behing eye. throat is always swollen, and red... headaches often, not all the time.. upset stomach occasionally.. he is constantly tired.... never feels rested , just wants to sleep all the time.... (not like him normally).. he has much anxiety, dread, worry , fear, anxious, tightness in chest... Any thoughts? His insurance kicks in in a few weeks... he has been suffering since May of 2013... Doctor: Hello!Welcome and thank you for asking on Healthcaremagic!Your son symptoms could be related to an allergic reaction. A chronic infection or a metabolic disorder can not be excluded either. For this reason, I would recommend consulting with his doctor for a careful physical check up and some tests: - a sinus X ray study- complete blood count- PCR, ESR for inflammation- thyroid hormone levels- vitamin B12 and Vitamin D levels for possible deficiency- blood electrolytes. Further tests may be needed ( allergy tests, etc.). Hope to have been helpful!Best wishes, Dr. Aida"
},
{
"id": 5963,
"tgt": "Trying to conceive, low sperm count, side effects of fertyl f and fertyl m",
"src": "Patient: I HAVE DAUGHTER OF 3 YEARS OLD, WE R PLANNING FOR SECON CHILD,,, MY PERIODS WERE IRREGULAR THEN I DIAGNOSED AS PCOD I fertyl f,,TOOK 5 CYCLES,,,,,, STILL DID NT WORKED THEN WE CAME TO KNW MY HUSBAND SPERM COUNT IS LOW,,,,,,,,,,THEN DOC GAVE FERTYL TO HIM PLZ ADVISE HW MANY CYCLES I SHLD TAKE WAT ARE SIDE EFFECTS OF FERTYL F AND FERTYLM Doctor: Hello. Thanks for writing to us. Frtyl F and Fretyl M can sometimes cause overstimulation of ovaries leading to multiple pregnancies. Fertyl M helps in restoring the sperm motility to some extent if used for more than 3 months continuously. I hope this information has been both informative and helpful for you. Regards, Dr. Rakhi Tayal drrakhitayal@gmail.com"
},
{
"id": 22166,
"tgt": "Suggest remedy for heart ailment",
"src": "Patient: I am a 35 yr old female, I have a pacemaker, recently found out that i have a bad lead wire, my dr is doing some adjustments so that we dont have to go through the dangerous surgery, but since the last adjustment my bp is high and my pulse rate is high as well, I have bradycardia, sick sinus syndrome and rate drop. my heart rate has never benn this high before. is this something that i should be concerned about Doctor: hello, First, high bp is most probably a age related change in you because pacemaker adjustment will not lead to this. Secondly, high pulse rate is most probably a part of sick sinus syndrome, because these patients have bradycardia and tachycardia. We need one ecg to confirm confirm the type of high heart rate. If Normal we may need holter as well. Also, you should get one hemoglobin level and thyroid test done. You should have a healthy lifestyle like avoiding fatty, oily and high calorie diet. Have low salt diet and monitor blood pressure regularly thrice a day for one week then once or twice a week. If bp remains high more than 140/90 then you should be started on bp medicine. Regular exercises like brisk walking, jogging according your capacity atleast 30 min a day and 5 days a week. Lots of green leafy vegetables, fruits, fish once or twice a week, avoid meat. Avoid smoking and alcohol if any. There shouldn't abdominal fat deposition or obesity. Get your lipid profile and sugars tested once.."
},
{
"id": 128437,
"tgt": "Can Diclofenac Sodium topical gel cause fluid retention?",
"src": "Patient: Can Diclofenac sodium topical gel cause someone to retain fluid? A friend of mine has pedal edema with pain and someone gave her some of her prescription gel to help with the pain, but I m afraid that due to the sodiium aspect, it could increase the edema. Doctor: Dear patient Diclofenac gel absorption is very less through the skin. even if absorbed short term use will not lead to kidney damage. So you need not to worry with topical use of diclofenac gel."
},
{
"id": 101893,
"tgt": "Could excessive tears be produced as a result of severe allergies?",
"src": "Patient: My eyes are overly producing tears, and I'm wondering if it could be related to my weight. I'm 5' 5\" and 300 pounds, so I'm wondering if it could be a part of that... I also have severe allergies, but it's worse than just simple allergies, they're practically just constantly draining. When I keep my eyes from fans, or air, it lessens. Doctor: Hi, thanks for using healthcare magicObesity is not linked to excess watery sensation of the eyes though it can affect other eye diseases such as age related cataract, glaucoma, diabetic eye disease.Excess tears can be due to: (1)chemical irritation of the eye(2)allergies(3)eye infection(4)eye injury(5)abnormal turning of the eyelidIf it continues, you may want to consider visiting your doctor for an assessment to determine the cause and to initiate treatment.I hope this helps"
},
{
"id": 216045,
"tgt": "Suggest medicine for pain management",
"src": "Patient: Hi Dr. I have a question and I am stuck. I came to Florida with my 84 year old grandmother. I wanted to surprise her with a vocation. I didn t check her medication s before we left from home. She has been seeing a pain management doctor for over 3 years now. I forgot to check her medications and she is out of her pain meds. Her pain meds can only be giving throw a pain management doctor. Who and where can I take her too see a doctor to be able to help her with her pain meds while on vacation? Doctor: Hello and Welcome to \u2018Ask A Doctor\u2019 service. I have reviewed your query and here is my advice. Your grandma is suffering from what kind of pain? Is she had any kind of disease? Usually it's better to get her once examined and then start on any pain medicine. Any practitioner can start her on Ibuprofen or Tramadol if liver and renal functioning are normal. I don't know the status of her. So share more details to probably find out the actual condition of patient to treat her better. Hope I have answered your query. Let me know if I can assist you further. Regards, Dr. Archana"
},
{
"id": 222647,
"tgt": "Is Duphaston effective for conceiving?",
"src": "Patient: I have under gone laproscopic myomectomy in july 2010. Now we are trying to conceive. I am suggested to take tab. Siphene twice a day from day 3 to day 5. From day 16th I have to take tab.Duphaston twice a day till day 25th. Kindly tell me how is it going to help? Doctor: Hi,For pregnancy, you need to have addressed male factor, ovarian factor, tubal factor and uterine factor.I am assuming that all these have been checked and are OK.Siphene (Clomiphene) is given when we are anticipating ovarian problems, if you have not been diagnosed with PCOS or ovulation problem, taking Clomiphene will not increase your chances of pregnancy. Duphaston can be given to help implantation, but as such has no role.Please consult a infertility specialist and do not take siphene empirically without a clear diagnosis. As every medicine has side effects which have to be discuuused.Hope this helps.Regards."
},
{
"id": 184026,
"tgt": "Suggest treatment for tooth pain",
"src": "Patient: I am in Costa Rica and have had tooth pain for 2 months, the doctor found/treated and infection and replaced filling, but I still have pain. Now my eye on that side is red and swollen. How serious is this? Should I drive 4 hours to ER or can I wait until my Wednesday appt with specialist? I am on Penicillan starting yesterday. Doctor: Hi,Thanks for posting the query, Toothache indicates infection in tooth such type of infected tooth needs to be root canal treated, i would suggest you to get a checkup done take an x-ray of the tooth and plan for its treatment, take tab amoxicillin and tab aceclofenac BD for 3-5 days.Take care!"
},
{
"id": 25163,
"tgt": "Should i meet doctor for the BP variations?",
"src": "Patient: Good morning Doctor,I'm 54 years old male. I've been taking Atenolol 25 mg for 15 yrs and this last three days my bp has been 140/90-146/96 before taking meds in the am. After taking my morning meds my bp goes down to 125/88-135/89. Do I need to call my Doctor and have him adjust my medication or wait and observe my bp for a week by limiting sodium intake and exercise more in the holidays when food are in abundance? Help!Thanks,Edwin Manabat Doctor: Thanks for your question on Healthcare Magic. I can understand your concern. In my opinion, you should wait for few days before going tu doctor. Since you are eating in abundance, this is the likely cause four your fluctuations in blood pressure. So follow strict salt restrictions, avoid fatty and oily food, do regular aerobic exercise like running, cycling, swimming etc for few days (10-15 days) and then again do blood pressure monitoring. Your fluctuations will mostly reduce with these in 1-2 weeks. If still there are fluctuations even after 1-2 weeks then consult your doctor and discuss about change in medication. Hope I have solved your query. I will be happy to help you further. Wish you good health. Thanks."
},
{
"id": 179110,
"tgt": "What causes fever,cough and runny nose after hitting the head?",
"src": "Patient: My two year old had a low grade fever 3 days ago and 2 days ago he walk into the door and hit the side of his head and a little knot was the but I but ice on it right away. Now his fever going up and down. Coughing and runny nose. What should I do. I did take him to urgent care the day it happen. What should I do. Doctor: Hi, thank you for posting your question. As your child already had fever before he hit his head, the running nose and cough could be attributed to a simple viral infection which has nothing to do with the injury. In this case all you have to do is keep a watch on his temperature, give him medicines for fever and cold/cough as your doctor would prescribe. The injury and bump on the head seems to be an unrelated issue here, for which some cold fomentation and pain killers should do."
},
{
"id": 8152,
"tgt": "I have had a dense concentration of blackheads on my nose. How can I get rid of them ?",
"src": "Patient: For over 11 years I have had a dense concentration of blackheads on my nose. There is not a square centimeter on my nose that is not covered with them. I have tried creams, oils, witch-hazel, and prescription medicines. I wash and scrub my face at least twice a day. I drink plenty of water. I jog every other day. I do not share pillow cases or towels. They are not scars, because if I pinch any where on my nose...well, you know. These are all the of the solutions I have ever read on this or any website. How can I get rid of them ? I do wear makeup over it (not when exercising, though) but they are still slightly visible. It is humiliating when children ask me What are those dots on your nose ? Doctor: Hello. Welcome to HealthcareMagic forum. Black heads on the nose are notorious for reappearing again and again. Either you keep on removing them on a regular basis.Or for permanent removal, you can opt for laser treatment but multiple sittings will be required. Dr. Rakhi Tayal"
},
{
"id": 42299,
"tgt": "Can HCG levels be too low to show on a HPT?",
"src": "Patient: I have PCOS and ttc. My dr had me take clomid and progetrone at the same time (first time taking clomid) after I had 2 days of light brown spotting. 2 days after I finished the clomid I had 2 positive opt tests 2 days in a row. The next day I had an inconclusive hpt. So I waited a couple days finished the progetrone, which usually makes me start my peroid the very next day and took another hpt; it was negative. Fastforward 2 weeks, I still haven't gotten my peroid, my breasts are swollen and nipples tender and I have become extremely sensitive to smells. I took another hpt today and it was negative, could my hcg be too low to read on a hpt? Doctor: Hello,thanks for using health care magicI have read your query with a lot of concern. The first thing i find is a little ambiguity as to why your hpt were positive and two weeks later negative.It most likely scenario is that the hpt gave false positive results. Normally, pregnancy test becomes positive at two weeks after fertilisation i.e. about the time the next menses was about to start and the first symptom thus is absent of menses in most cases.If truly, the tests were positive, then you might have been pregnant long before you even started taking the drugs and you must have had a miscarriage that has resulted in negative hpt. However, this is also ambiguous because it requires averagely three weeks for the test to become negative after termination of pregnancy.If you recently became pregnant and its less than 4 weeks(you have not yet missed your period), the hpt will be negative.Best regardsDr Achuo"
},
{
"id": 70830,
"tgt": "Suggest remedy for dry, chesty cough, pain in chest and loss of voice?",
"src": "Patient: started of with a dry cough last saturday and my voice has gradually gone my cough has now turned into a chesty cough..Ive been havin chest pains and coughing that hard that i am being sick, i have woken up this morning and i have like a achey burning sensation running up and down the back of my legs and my eyes are hurting me when i look around or to the sides? do you know what this could be Doctor: Hello, As you explain the history this is a common viral situation. You can take painkillers a lot of liquids and if you can do a rest for some days. Hope I have answered your query. Let me know if I can assist you further. Take care Regards, Dr Jnikolla, Pulmonologist"
},
{
"id": 183334,
"tgt": "Suggest treatment for tooth ache",
"src": "Patient: 36 year healthy female, went to dentist yesterday complained of intermittent pain for last six days, no cavities no discernible problem, suggested endontist which I scheduled for Tuesday. Trouble sleeping last, but did fall asleep after several hours with help on Advil and ice pack. Today pain excruciating and constant but relieved with ice water every two minutes. Help Doctor: Thanks for your query, I have gone through your query.The pain in the tooth can be because of the tooth infection secondary to the decayed tooth. The pain during night time is a charecteristic feature of pulpitis. The decay might not be visible clinically, it can be present in between two teeth. So consult a oral physician and take a radiograph to rule out the decay and tooth infection. If it is tooth infection You can take a course of antibiotics like amoxicillin 500mg and metronidazole 400mg tid for 5 days(if you are not allergic). After taking antibiotics you can get the tooth treated with RCT(if bone support is fine) for tooth infection.I hope my answer will help you, take care."
},
{
"id": 29803,
"tgt": "Suggest remedy for persistent body acne",
"src": "Patient: Hi, I am 25 years old and suffering from a few skin diseases..I have consulted so many doctors but none of them guided me to the right path or helped me to find the right cause ..I have itchy skin, body acne and any marks on my body does fade even after months and months.. Doctor: thanks for posting your query to hcm.Fungus is a common cause of itchy skin lesion .If you have not undergone any investigation for fungal skin infection First go for skin scrapping examination to fungal microscopy .If it is found positive we can treat it with antifungal drug. If found negative then we will think about other causes of itchy skin lesion like eczema , psoriasis etc . these kind of lesion are genetic in origion and require long duration of treatment . Review me with investigation report .hope it will help you ."
},
{
"id": 32459,
"tgt": "Suggest treatment for throat pain and vomiting",
"src": "Patient: Hi Sir, I had a fever last month. The symptoms where headache, Shivering, & my body feels so much cold . I took medicines from a clinic also done blood check up for Malaria but everthing was normal as per report. After a10 days continues medication I came to normal life. Now exactly on the same day next month this again started. Now my problems are throat pain, when ever i take food i feel I am going to vomit, Also my body seems so cold. Can you pls suggest how to proceed on this..... Doctor: Hi,Welcome to health care magic,All this symptoms of throat pain,fever with nausea or vomiting could be because of upper respiratory infection (pharyngitis, laryngitis, epiglotitis) or bronchitis or sinusitis but should be evaluated further.It is mostly due to viral infection like EB virus,adenovirus but some time could be bacterial or even parasitic infection.You may have some routine blood investigations with sputum tests as if you have cough and Chest X- ray to get correct diagnosis first.You may require higher antibiotics (quinolones or macrolides) to prevent super infection with anti-inflammatory and other supportive treatment like bronchodilators if there is any breathing problem due to throat infection.It would be advisable to see ENT specilist and start treatment after that.Regards,"
},
{
"id": 96502,
"tgt": "Acid peptic disease treatment which will offer cure",
"src": "Patient: three years from now i have been diagnosed as suffering from GERD. and i have to take a tablet Ganaton 30min before meals. more so many restrictions like not to lie down after food, not to eat full stomach. still it is not getting cured. is there any better treatment which will offer cure can some body help me in this regard Doctor: even i had this problem, i took gelusil syrup, rantac and so many tablets i was not relived. but a ayurvedic doctor prescribed me a chooran which helped me a lot. even i advice yu to go for it. tablets is only for symptomatic treatment."
},
{
"id": 132324,
"tgt": "Suggest remedy for sore shoulder muscles",
"src": "Patient: Hi, my husband has sore shoulder muscles after being off of work for a few month s, he recently went back and for a few day s had to do work that requires using his upper shoulder muscles, he normally drives forklift and will be returning to that Monday. Can he take this drug for just a couple of nights? Doctor: Respected sir/ madam, I have understood your concern, see pain in shoulder joint is due to injury at it as you mentioned. pain may be due to muscular tightness, inflammation in muscles and tendons, or in capsular strains may arise. drugs can be taken to minimise pain in shoulder, but more power ful ways are available for reduction of pain. in PHYSIOTHERAPY we have IFT modalities and ultrasound therapy which relive pain situated deeper. apply ice for short term pain. but for proper range in joint and powerful muscles, exercises are must, and can be started after 15 day of modalities treatment. I hope you have satisfied with the answer. kindly provide feedback"
},
{
"id": 127966,
"tgt": "What causes pain and numbness in the neck?",
"src": "Patient: Hi. Good day i am kevin. And im experiencing a feeling of tired face and sometimes numbness in the cheeks down to my neck. Doctor said that im stressed. My throat also have a problem it feels that there is something stucked in it that i cant blow out. Please help me its affecting my way of life.. thanks Doctor: Hello, I have studied your case.Due to compression of nerve root there can be tingling numbness in your face and neck.I will advise you to MRI cervical spine for better diagnosis.For these symptoms analgesic and neurotropic medication can be started.Till time, avoid lifting weights, Sit with support to back. You can consult physiotherapist for help.Physiotherapy like ultrasound and interferential therapy will give quick relief.I will advise to check your vit B12 and vit D3 level.Hope this answers your query. If you have additional questions or follow up queries then please do not hesitate in writing to us. I will be happy to answer your queries. If you are satisfied with answer do not hesitate to rate this answer at end of discussion. Wishing you good health.Take care."
},
{
"id": 123146,
"tgt": "What causes knee pain and ankle swelling after a fall?",
"src": "Patient: I scraped my leg up falling down some steps. Now I have a sore squishy lump and my ankle still swells. i did not twist anything. Also knee hurts when even material touches it. I tripped about a month ago. I start out my day with just a little swelling. it does not swell a lot. But it does swell above ankle. Doctor: Hello, As this looks like a ligament sprain within the knee joint leading to swelling and pain. I will advise for a knee MRI. Also, using knee and ankle brace will be helpful to stabilize the joints. Post which doing hot water fermentation will help vascular supply to be good. Also, keep the leg elevated over a pillow while lying down this will help boost the swelling to come down more. Post the pain comes down a little. Start exercises like - ankle toe movements, straight leg raise, static quadriceps, static hamstring, hip muscle strengthening etc to improve the muscle strength and improve the joint space to avoid inflammation to arise. Hope I have answered your query. Let me know if I can assist you further. Regards, Jay Indravadan Patel, Physical Therapist or Physiotherapist"
},
{
"id": 149888,
"tgt": "Hearing loss in both ears, heart patient, have MRI report. Can any online doctor help me to read it?",
"src": "Patient: Sir, I am 75 year old retired executive. After retirement I was keeping myself busy with training programs for employees in several cos. Suddenly six months back I had loss of hearing in both my ears. As no treatment could help, I had to go for hearing aid (though hearing aid also not very effective). Due to sudden hearing loss I had to stop my training activities and thus I went into deep depression . Started feeling heaviness in the head. I am also a heart patient of irregular and low pulse rate (heart beat around 50 p.m.). On advice of a Dr. I went for my MRI . The concluding remark of MRI report is: B/L Few Old Ischemic Foci, Gross Generalised Brain Atrophy and Right Maxillary Polyp as described. I am taking Carvistar, Isonorm, ECOSPIRIN AV 150 for my heart problem and CENSPRAM PLUS for depression. Kindly advice the meaning of my MRI report (i.e.B/L FEW OLD ISCHEMIC FOCI,GROSS GENERALISED BRAIN ATROPHY AND RIGHT MAXILLARY POLYP AS DESCRIBED. ) Is there any serious problem in the brain. Am I taking right medicines. Pl. help. Rajendra Prasad (75 years), Lucknow. Doctor: Hi, The M.R.I. reads that there are old ischemic foci means loss of blood supply. and atrophy secondary to the ischemia, but both can't produce, sudden loss of hearing. The maxillry polip is there and it can have some relation to the heahing. I advise you to consult an E.N.T.surgeon for diagnosis and treatment. polepectomy may help you to some extent. Thank you."
},
{
"id": 95400,
"tgt": "Having solar plexus pain causing extreme weakness and taking antacid but no relief. Suggest a possible cause and course of action ?",
"src": "Patient: My mother aged about 56 use to have severe pain and heaviness at solar plexus region. Any exertion or walking brings the pain and the solar plexus region (2 inches below the sternum) gets stiff. Whenever this pain starts, there comes symptoms like high BP, heat flushes alternating with chills, faintness, sudden loss of strength, extreme weakness etc. USG of abdomen revealed atrophy of the uterus. She is taking medicines for high BP and heart . The doctor prescribed antacid for solar plexus region, but it is not providing any relief. Could you please suggest the possible cause for this pain and probable course of action? Doctor: Hi, Thanks for query, Your mother might have some cardiac problem. Consult cardiologist and get investigated. Second possibility is she might have manopausal syndrome and I think she is having this problem.Due to hormonal imbalance this type of symptoms are common. Consult gynaec and go for treatment. She can attend manopausal clinic and take guidence. Ok and bye."
},
{
"id": 20851,
"tgt": "Suggest treatment for increased heart rate",
"src": "Patient: i am taking bisoprolol for svt i am overweight and really want to get fit but each time i go to the gym the treadmill machine slows down to a walking pace after about two minutes because it says my heart rate is too high what type pf wxercise should i do? Doctor: Hello Thabks for posting here. Your heart rate tends to be fast as soon as you exercise. Bisoprolol keeps the heart rate under control. You need to walk on the treadmill at slow speeds initially then gradually progress to faster speeds. Due to lack of exercise, the heart is not accustomed to it, hence heart rate increases rapidly when you exercise. Once you do it regularly, the heart rate will start getting under control and your treadmill will not lower its speed automatically. Continue taking bisoprolol. Regards"
},
{
"id": 54344,
"tgt": "What does HbsAg positive suggest?",
"src": "Patient: hi i want to ask about hepatitis B.if the test show that HbsAg positif, it means always infected?but i dont have problem or symptoms related to hepatitis such as jaundice, etc.my mother also show that she has HbsAg positif but my father does not.and also my brother has HbsAg positif but his wife is not positif. is it means that im infectious to other person especially my husband?what should i do know?thank you Doctor: Thank you for posting query at HCM.HBsAg positive means that your have been infected with HepB virus. you need to be chacked for HbeAg and viral load to know your \"infectious state\".ONLY positive or high HBeAg and/or viral load will mean that your highly infectious.It is possible that the virus remains in dormant phase throughout and causes no harm to liver. Only one fourth of patients infected with HepB virus develop liver cirrhosis. therefore, one cannot be sure about your status. the viral load should be monitored and he needs to repeat test every 6 month or atleast once a year.without further evaluation , you can not be adviced treatment. liver enzymes (LFT), HBeAg , and viral load (PCR test) need to be examined. Take results to treating physician or post them at HCM for further advice.Results of all these together with HBsAg will give a clear picture for treatment.meanwhile, people close to your friend needs vaccinated and also test for HBsAg.if any further questions, feel free to ask.Health professionals aim to diagnose properly and manage patients according to their limited knowledge. Cure is blessed by the ONE who Created us, whose power and knowledge is unlimited .wish you good health.regards,Dr Tayyab Malik"
},
{
"id": 58156,
"tgt": "Had removed my bladder. Have severe stabbing. On HCG injections. Immediately after eating the stabbing pain starts. Is this a sign of an illness?",
"src": "Patient: I had my gall bladder removed a couple of years ago. My main symptom was a severe stabbing pain whenever I ate food. Since then that pain in the shoulder disappeared until recently I had a couple of mild pains in my shoulder in that same spots within 30 minutes to an hour. But since it was only twice I thought I was fine. I currently am using HCG injections for the first time & I'm on my 3rd week. For the past 3 days in a row, almost immediately after I eat that severe jabbing pain in my shoulder is occurring. Is this a sign that has to do with previously having gall stones or is this signs of maybe developing pancreatitis? What's going on? Doctor: Greetings of the dayWelcome to Healthcare-MagicShoulder pain is unlikely to be due to pancreatitis. One of the possibility could be post cholecystectomy syndrome, which could have similar manifestation of gall bladder disease even though it has been removed. I would suggest you to avoid oily and spicy food. Another possible cause of your symptoms could be duodenal ulcer. Management includes long term use of proton pump inhibitor and helicobacter eradication treatment to prevent recurrence.Wishing you a healthy life.Get back to me if you have any other queryTake careRegardsDr T ShobhaMBBS MD"
},
{
"id": 188480,
"tgt": "Having frequent mouth ulcers, used B complex but no use. Not able to eat food. Suggest?",
"src": "Patient: I am having frequent mouth ulcers for LAST 7 years. In a month I suffer at least 2 weeks. Used B.complex and other methods but no use. Can any one please suggest which medicine can I use for this? really i am facing big problem with this mouth ulcers i am not able to eat food even if at least water also., please guide me to come out from this problem.. Doctor: Hello,Thanks for writing to us.From the details produced its clear that you are c/o recurrent Aphthous ulcers.I would advice you to apply topical steroid at the affected area.If the ulcers are aggressive,oral corticosteroids has to be administered.Maintain oral hygiene well.Avoid spicy foods.Smoking as well as tobacco chewing has to be avoided.Plenty of fluids as well as nutritious diet has to be administered.Avoid anxiety and stress.Hope this helps."
},
{
"id": 37407,
"tgt": "Suggest remedy for a swollen lymph node",
"src": "Patient: I have a swollen lymph node that is progressively getting bigger and more tender under my jaw I have had a complete cbc work up and have a high platelet count and a low htc count also have a fatty liver dr ordered a ct scan but did elaborate over what she could be looking for.. I also have a normal wbc she did say she didn t think it was a infection Doctor: HiThanks for consulting HCMA lymph node must be examine for FNAC ( fine neddle aspiration cytology)which will clearify either the origin of lymph node is tubercular or cancerous.even in tuberculosis wbc is normalOn ultrasound grade 1 fatty liver doesnot require medicine and need blood test LFTafter physical examination of body , lymph node and blood report / ultra sound report i only can comment about ct . so kindly disscuss with your doctor about FNAC by a good Diagnostic centre"
},
{
"id": 86672,
"tgt": "Can crystal meth cause stomach rupture?",
"src": "Patient: Hi I m. Really concerned for my little brother who is 30 now. He has has been taking crystal meth along with methadone for over a year now. He has been hospitalised with severe pain n stomach. After having a scan doctors have found a whole in his stomach due to this fluid has spilled into his body. He was unable to drink also and was told that as fluid couldn t enter his stomach it has gone into his lungs also. He is on a breathing machine and will be for a few days after having surgery to his stomach. Doctors say that he is going to be in a very bad way for the next few days as his body fights this infection. My question is will he recover from this and pull through what are the chances and also can the crystal meth have caused this rupture in his stomach also. I forgot to mention he had a gastric bypass op a few years ago too please help. Doctor: Hi.Thanks for your query. Noted the history about your brother, gastric bypass surgery a few years ago, use of Crystal meth and methadione.That now he is on Ventilator for a rupture of the stomach and lung involvement, it will be very difficult patch for his life. The chances are always meager for life as these patients can go into multi-organ failure. Pray the God, help the Doctors to give then the full liberty so that they can try for the best."
},
{
"id": 98105,
"tgt": "yuyutyu",
"src": "Patient: zzxczxc Doctor: Hello, Thanks for posting, When you experience pelvic pain/lower abdominal pain associated more or less to lower back pain it is usually as a result of an affection at the level of your pelvis or kidneys/UT. Now first of all, since you have missed period, take a pregnancy test, and if positive consult a gynobs so he can run a pelvic scan to determine if any abnormal pregnancy. Some series of tests include screening for PID (which is very likely given the presentation) and also urine tests + more or less of some blood tests could determine the cause of your problem and henceforth an appropriate treatment can be established. Hope this helps Dr Nsah"
},
{
"id": 135566,
"tgt": "What causes pain in legs, thigh and back after vomiting?",
"src": "Patient: I have thrown up twice today. I noticed after the first time that my legs from my knees up to my lower back became very achy, especially in my front upper thigh. Could this just be from the force of throwing up? What can I do to ease the pain? It hurts more than my stomach. Also, I feel like I have a low grade fever because I keep getting chills n then hot again. Doctor: U might be having some sort of lumbar disc problem which causes pain in legs due to nerve compression. . An LS spine MRI wud b helpful Thanks"
},
{
"id": 135959,
"tgt": "What does Osteophytes in the c1/c2 region suggest?",
"src": "Patient: I have a friend who suffers from a unilateral c1 c2 problem; she is in her late 60s, a small boned person; there was a traumatic injury some years back, and gradually this small spot of pain in her upper neck settled in and increased. She has no other neurological symptoms. The radiography to date shows osteophytes in the c1/c2 joint, and I think a swollen capsule. vertebral artery trajectory is normal. It was eventually noted there is a small sliver of bone near the site. I think posterior fusion with rods and screws is the best total treatment, but, if she fears to undergo that (the risks), could that sliver BE the source of her sharp, localized pain and COULD it be removed in a less invasive manner, perhaps solving the pain issue, do you think? Doctor: hiif osteophytes are causing only root pains they may be excised alone without need for fusion surgery r any implant.this could be undertaken laproscopically by a trained spine surgeon, or open excision.If pain beearable, then conservative treatment, physical therapy, cervical collar and isometric neck exercise may be done.consult spine surgeonthanks"
},
{
"id": 175922,
"tgt": "Suggest remedy for frequent migraines",
"src": "Patient: my son gets migraines about 1 to 2 a week. He is only 10 - could it be caused by growth spurts? We have tried everything and have brought him to several specialist. His migraines were under control up until June of this year - and since have been frequent. Doctor: Hi...Thank you for consulting in Health Care magic.Headache in kids need to be considered serious only when the below symptoms are there - 1. Vomiting continuously2. Seizures/ Fits3. Watery of bloody discharge from ears and nose4. Unconsciousness5. Altered sensorium or behaviour. Without the above red flag symptoms, just feeling low energy levels points out towards Migraine and other sorts of tension headaches which are on the rise in current scenario due to excessive academic and peer stress the kids are experiencing.I have a few questions for you -1. How long has the headache been there?2. Does it occur always on forehead or keeps changing?3. Is it associated with blurring of eyes or double vision or giddiness?4. Is there any family history of migraine or single side headache?5. Is he very angry when he gets headache and does the ache get better after a good sleep?6. Does he vomit when there is headache?You can approach me at the following link. Please find the link below - www.healthcaremagic.com/doctors/dr-sumanth-amperayani/67696"
},
{
"id": 199091,
"tgt": "What causes red,itchy area on the bottom of the penis head?",
"src": "Patient: I have a red itchy area on the bottom side of my penis head. it looks like little small bumps that move when i move the skin. I had unprotected sex with a girl who said she had a yeast infection a few weeks prior. There is no pain, no blistery looking places, no type of painful urination or discharge Doctor: HelloThanks for query.Red itchy area on bottom of glans penis are most likely to be due to infection (Ballanitis) following unprotected sex Take antibiotic like Cefixime and topical antibiotic ointment like Neosporin for a week .Ensure to wash genitals with warm water twice daily..Dr.Patil."
},
{
"id": 207922,
"tgt": "Does anesthesia cause quick temper and irritation?",
"src": "Patient: Hello, I had two surgeries on my leg and my lungs over a year ago after a car accidentand have another one surgery ahead, and the anestisia really affected ,e untill this day, I easily get annoyed and loose my temper. Is it because of an anestisia and when i will have my next surgery is the anestisia going to affect me even more? Doctor: HI Well come to HCMAfter the induction of general anesthesia if halothane used then such delirious condition is just likely and this is temporary condition and this comes around on its own without the medications no need to worry about this, take care."
},
{
"id": 28732,
"tgt": "How can painful UTI be treated?",
"src": "Patient: I have been on 3 different antibiotics in the last 3 weeks...started with feeling of bladder infection...severe pressure and burning with some pinkish blood and a little bit of pink tissue... Went to Dr..urinalysis showed infection with trace of blood..white blood cells...went on Keflex..didn t really help switched to Macribid... Slightly Less pressure and burning no blood but still feel the pressure and constant urge to go with terrible pain in my groin and hip area...Did another culture it showed no bacteria this time but still white blood cells.. She gave me 3 days of Cipro...still bothering me... I have dip sticks which show still white blood cells...no nitrates..so uncomfortable.. Doctor: Hello and Welcome to \u2018Ask A Doctor\u2019 service. I have reviewed your query and here is my advice. At this point your infection is gone but you still have some inflammation. Your culture showed no further bacteria so that is not the cause. Persistent inflammation can be caused by a number of problems including interstitial cystitis, stones, and other things. At this point you should not have further antibiotics (as what would you be treating?) but have a full urinalysis sent to the lab rather than a quick dipstick urinalysis. You may also need imaging studies and referral to a urologist if your primary care doctor can't sort it out from a full urinalysis and imaging. Hope I have answered your query. Let me know if I can assist you further."
},
{
"id": 102071,
"tgt": "What test to do for checking asthma?",
"src": "Patient: hi, iam lakshmi my eosinophils are 498 in absolute count and the normal range is 40-440 and basophils are 0. this should be a cause of concern. according to pft report i have asthema but my chest was always clear. suggest me some test for clearing my doughts. Doctor: HI, do you have any symptoms like shortness of breath, forced respiration, tightness of chest, wheezing and coughing, bronchoconstriction releaved by bronchodalator like salbutamol , these symptoms may be present through out the year in reduced intencity and aggravate in special situation or they may menifest themselves only in special situation like after moderate or severe exercise, cold weather , exposure to smoke, air pollen or fumes, formaldehyde, cigarete smoke, or even house dust. these symptoms with suggestive PFT IS HIGHLY INDICATIVE OF ASTHMA, and if these symptoms and also PFT results improves with bronchodilator (salbutamol) inhalation then it almost confirms the diagnosis. even though for your clarifiation you can go for methacholine challenge test, chest X RAY, complete pannel allergen test, and salbutamol response test. i suggest you to avoid those conditions which precipetate your symptoms, have regular spirometry to follow up this condition, take your drug properly and do some yoga regularly. wish you all the best."
},
{
"id": 119324,
"tgt": "Fatigue, weight loss, yellowish eyes and skin due to low hemoglobin. Its adverse effect?",
"src": "Patient: I am 25 year old male from india, I have some health issues like fatigue , weight loss . My eyes and skin seems like yellowish. I have undergone blood test last week, it shows that my haemoglobin is lower than the normal range. Haemoglobin - 12.2 RBC - 4.12 Does it mean anemia? What would be the severity of this measures? Doctor: Hello, Hemoglobin of 12.2 does not indicate severe anemia. This is not the cause for your sypmtoms of fatigue and weight loss. I am a little concerned about your skin and eyes being yellowish in color. This might indicate liver problems such as Hepatitis. There are different types of viral hepatitis. Are you consuminng significant amounts of alcohol such as beer or whiskey on a regular basis ? This could also cause liver damage. You need to have a complete blood work to include Hepatitis screen,also liver enzyme determination along with other liver tests. Wish you the best of health."
},
{
"id": 21580,
"tgt": "How to treat low rate rate condition?",
"src": "Patient: My daughter is 30 years old and has a low heart rate. She just had a holter done 24HR and her heart rate went down to 38. She is not athletic. I don't know if this was recorded in the night but her daytime heart rate has been recorded at 44 to 48. What advice can you give me. I am worried about heart failure. Should I be?? She had an echo done and came back normal. Could it be an over active vagal nerve. Thank you very concerned Dad Doctor: Hello SirThe medical details which you have mentioned of your daughter I have gone through them and first of all I would like to assure you that low rate in a young adult does not signifies HEART FAILURE so please take take that stress out of your mind.Yes heart rate often goes below 40 beats per min during deep sleep so that's alright. The only thing we need to check in your daughters case is that whether this low heart rate is sinus or not ( it means every P wave on EKG IN HOLTER is followed by a QRS or not). If it is sinus then you need not worry. For this its advisable that you should consult a cardiologist personally and discuss with the doctor.There is no need to treat Sinus bradycardia until unless the person is symptomatic.I wish her good health"
},
{
"id": 217168,
"tgt": "What could be the cause of pain in cheeks?",
"src": "Patient: i ve pain in left check,,little uneasy,while pressing left i feel comfortable.this uneasy not much with big trouble just slightly something wrong g going on that i experinced.what is this infact is it harmfull?i went lab two times a year fo blood chech whether any problem in my heart but bood chech find nothing Doctor: Pain in cheek comes when there is a pressure on fascial nerve of same side or when the muscles are tired and weak. In your case you have only light uneasy feeling and that is also only on one side cheek not on same side eyebrow and lip as well chin area that means it can not be related with fascial nerve, so that means muscle of that side covering the cheek area are weak. But Dont worry just use a warm hot pack on same side and it will be all right.At any time if you feel one part of face muscle is not working then do visit a doctor immediately. Till then relax. Take care."
},
{
"id": 100568,
"tgt": "How can skin rashes with itching caused by bites be treated?",
"src": "Patient: Hi, I have been getting rashes on my skin for past 3 weeks. I visited a physician and she recommended Vozet 5mg (glaxo) for four days. Once I started that tablet, the itching disappeared. But started again after 2 days. I took one more course for 4 days the result is the same. I have never had any allergy in my life (43 years male). The itching started while returning from a foresty region. I remember I had few bites of mosquito likes - -. What should I do? Doctor: Hello.Thank you for asking at HCM.I went through your history and would like to make following suggestions to you:1. I usually suggest my such patients regular montelukast-levocetirizine for at least 2 weeks.2. I also suggest them to apply a lotion like calamine/moisturizer over itchy areas immediately after bath.3. As your problems starter after a trip, I would suggest you to take a course of antiparasite drugs. (parasitic infections can commonly cause such hives).Hope above suggestions will be helpful to you.Should you have any further query, please feel free to ask at HCM.Wish you the best of the health and a very good recovery.Thank you & Regards."
},
{
"id": 225855,
"tgt": "Changed birth control pills. History of endometriosis. Missed periods. Pregnancy chances?",
"src": "Patient: I'm 20 years old and I have been on birth control for almost four years but my doctor recently changed my pills so that I take them continuously because I have a slight case of endometriosis and the first month I missed my period I have never missed one since I've been on the pill. I randomly have symptoms that feel as if I am having some type of hot flash. I have taken a pregnancy test but it came back negative and I took it after I had missed my period, is there anyway I could still be pregnant? Doctor: hi.. thanks for using HCM..you are taking ocp's.. chances of pregnancy is remote..since the urine pregnancy test is negative, get one serum HCG levels.. which is confirmatory..see the obstetrician in any case..all the very best.."
},
{
"id": 102659,
"tgt": "Does sinus have any relation with allergy and cold?",
"src": "Patient: I read that Allergens can cause sinus infections. I keep getting head colds, sinus infections, last was upper respitory. Now another sinus thing. I've come to believe I have an allergy to sewing fabric. I worked on a project over the weekend and got a little itchy so I washed the second project. Wondering if my sinus thing is related to this type of allergy? I would think not that would only cause a rash, but once I had an allergic reaction so bad I was in the hospital multiple times and they said it was so sever that I must have ingested whatever was causing it. Only later did I realize it had to be the quilt I was continually working on. Doctor: HI, sinus infection is secondary to repeated common cold attacks getting secondary infection,if person has low resistance. So, allergy being one of the instigating factor in repeated colds ,may be blamed for sinusitis. Yes one can be allergic to fabric/quilt dust fibers -inhaled & manifested as allergic rhinitis ( common cold)allergy . You need to- - get allergy testing, followed by avoiding allergens/ a desensatization treatment for removing allergy. - have a nutritive diet,exercises,respiratory exercises,vitamin,iron,supplements( if advised by your doctor) Thanks."
},
{
"id": 152062,
"tgt": "What treatment should i give to my 29 days old baby for Erb's palsy ?",
"src": "Patient: Dear Doctor , My daughter (age 29 days) is patient of erb s palsy . The problem identified on next day of birth . No moment or sensation observed by doctor. X-ray report is normal. The doctor at Raipur(Chhattisgarh) checked and identified the problem as compression of C5, C6 & C7 and prescribed for physiotherapy at home. Initially there is no strength in hand but now some power is appearing in forearm , but still there is no strength in wrist. No any other test was carried out. should i wait for six month as advised by doctor. What i can do for betterment of my baby. Kindly advise. Sunil Soni Korba (chhattisgarh) Doctor: hi well come to hcm Usually it can be diagnosed clinically as well.If at all you are intereste in gettin investi getting it you may submit her for X-cervical spine and EMG[Electro myography}.As far as Erbs palsy is concerned physio therapy is best trearment.So if you wish You may continue with physitherapy and wait for the out come. Thanks Better luck"
},
{
"id": 74254,
"tgt": "Could chest pain be due to a bony lump near the sternum?",
"src": "Patient: I am a 42-year-old female who weighs 120 lbs. I normally weigh 128 lbs. but I lost 8 lbs. over the last fours weeks because I'm on an elimination diet (basically a vegan diet) to try and identify food triggers for my chronic migraines. Three weeks ago I had horrible pains in my upper chest and back that I attributed to heartburn and that were somewhat relieved by TUMS. Since then I have continued to have sensations of discomfort in my upper chest, something like your chest feels when doing exercise at high altitudes. The discomfort is pretty constant and doesn't seem to coincide with eating. A couple days ago I discovered a bony lump on the right side of my sternum, an inch or so below my collarbone. It's about the size of a marble and it feels uncomfortable when pressed but not painful otherwise. Is the lump related to the chest pain and should I have it checked? When I breath in deeply, I feel the most discomfort at the top of my sternum and so I'm wondering if the lump is related? Doctor: It could possibly be related to this mass. Sometimes, we can get bony non-malignant growths - osteomas (which is a new piece of bone growing from another bone). This is usually benign, but can cause discomfort. I would recommend a CT scan of your chest, and I'd have them scan your abdomen at the same time. The other test I would recommend is an endoscopy, which can be done by a gastroenterologist. Finally, a right upper quadrant ultrasound can look at your gallbladder and liver, and see if there are abnormalities (this would be lower on the list, as gallbladder pain is usually related to eating). Hope this is helpful."
},
{
"id": 41031,
"tgt": "Can Ovafin cause amenorrhea?",
"src": "Patient: hello doctor i am 26 years married. i am trying to conceive but nothing happened. my FSH is 15.77 LH and prolactin is in range. i have also taken cerophene 50 mg for 5 days from day 2. and then i took ovafin for 5 days in next cycle. on 12 day i went for ultrasound and my doctor said you have to take ovafin 200 mg per day again for 5 days in your next cycle. but i am not having my periods from two months so i cant start the treatment again or i can take the treatment without waiting for the menses. could absence of menses be a side effect of ovafin or what. i am very much depreessed. plz help me what should i do. Doctor: HiDr. Purushottam welcomes you to HCM virtual clinic.I have gone through your query. I think I have understood your concern, I will try to suggest you the best possible treatment options.I will suggest you that do not start the medicines unless you start your periods. Secondly FSH levels that you are stating need close attention. Get AMH levels of blood done, your treating doctor will be aware of its importance. These AMH levels can be done on any day of the cycle. They indicate eggs reserve of the ovary. So before going ahead with furher treatment please investigate as suggested.I hope my answer helps you.Thanks."
},
{
"id": 213139,
"tgt": "Depression, signs of schizophrenia, has severe mood swings, suspicious thoughts, does not continue the medicine course. Help?",
"src": "Patient: Hello Dr Bodhale, My elder brother is ur patient. U treated him around 4 yrs back. But he didnt continue his medicines. he is depressed and have according to my knowledge have some signs of pysiphrenia. like, 1. He doesnt sleep in night and keep calling everyone whole night. 2. he feels that some or other is planning a conspiracy against him. 3. he is very very suspicious. 4. he abuses people for things that not really happen or to the people who have nothing to do with him. 5. he feel so damn depressed i cant explain. 6. he is not at all confident. 7. his mood swings within minutes. 8. he smiles sitting alone. these are only some signs. we have his reports and want him to meet you. Now even he feels that something is wrong with him and my mom has told him that he needs medication . bbut he takes medicines for somedays and again stop taking medicines. Doctor: Hello and welcome to Healthcare Magic. Thanks for your query. If you would like to consult a specific doctor or a specialist, please use the premium service. You brother seems to be suffering from a psychiatric disorder called Schizophrenia. This is a chronic disorder, which needs regular and long term treatment. Stopping medication or irregularly taking medication will lead to relapse of symptoms. Please take him to his psychiatrist for further treatment at the earliest. If he is refusing to take medication or co-operate, then other treatment options such as in-patient treatment or long acting (depot) injections have to be considered. Please discuss these options with his psychiatrist. Wish you all the best. - Dr. Jonas Sundarakumar Consultant Psychiatrist"
},
{
"id": 36148,
"tgt": "Suggest treatment for water infection and sore breasts",
"src": "Patient: Hi I m just wondering if you can help I have been to the toilet and noticed that there has been some brown clumps/mass that s had a funny smell to it I have recently being suffering from a water infection could it have something to do with that or do I need to see my GP, also my breast have been extremely sore over the last week or so ??? Doctor: TAKE PLENTY OF LIQUID TO COVER DEHYDRATION.MAINTAIN GOOD HYGIENE.WASH YOUR HANDS WITH SOAP & WATER SEVERAL TIMES.FOR SORE BREAST APPLY 'BETADIN' TWICE A DAY.CONSULT YOUR DOCTOR FOR PHYSICAL EXAMINATION & FOR ANTIBIOTIC."
},
{
"id": 177771,
"tgt": "Suggest treatment for diarrhea and vomiting in a child",
"src": "Patient: My daughter has got 1 time vomting and 3 or4 times loose motion yesterday morning, only liquids(coconut water, electrol water, butter milk) were given to her as diet no medicines have given.to day morning she said a mild stomuch pain, then meftal spas 1/2 tablet is given with butter milk. Later coconut water is given, as of now no pain she said. What further medication, diet is necessary for her? Doctor: You are rightly managing her with plenty of fluids and ORS (electral). Meftal spas is ok to relieve her abdominal discomfort. Such episodes of acute gastroenteritis generally get corrected in 3-5 days irrespective of use of antibiotics. Giving Zinconia syrup (containing zinc) for a few days should be helpful. Also give plenty of curd or yogurt. No other medicines required. Normal diet to be given as tolerated. Giving protein enhanced diet is also a good option."
},
{
"id": 58282,
"tgt": "Gall stones found in ultrasound. Had voimitings, upset stomach, back ache. Treatment options?",
"src": "Patient: i just had a gallbladder ultra sound and found out that i have several gall stones. I have an appt with dr 11/06 to determine what way to go. Several friends have said that they would probably remove the gallbladder. I have had issues for about 1 1/2 years with vomiting, upset stomach, back ache, cold and hot sweats. What is the normal time in the hospital and down time. I am raising my 2 year old granddaughter. Doctor: Hi,Thanks for using Healthcaremagic,You have gall stones and having symptoms if medical treatment is not giving enough relief surgery is an alternative called cholecystectomy that is removal of gall bladder.This is simple surgery in expert's hand and can be performed with the help of laparoscopy hence hospital stay is short and recovery is fast maybe 2-3 day in hospital.This also depends on your age and associated medical condition.Hope this information is useful.RegardsDr.Akhilesh Dubey M.D."
},
{
"id": 201590,
"tgt": "How long does a torn urinary opening take to heal?",
"src": "Patient: 27male. One week ago I was sexually engaged and felt a pain. The next morning I found that my urinary opening had torn toward the frenulum. I ve been applying neosporin to avoid infection and so far so good. The head is less hypersensitive than before, but there still is an opening that hurts when I m erect, which I avoid, but it just happens. How long do you think this should take to heal? As in how long do you think I need to avoid sex? Is there anything else I should be doing? My girlfriend is going nuts. Doctor: Hi,Thanks for writing in.The tear in your penis may get better without treatment. Avoid sexual activity until the tear has healed. Once it has healed, you can try using a lubricant during sex to prevent the problem from happening again.It is not a serious issue and might take 2 weeks to heal. Right now you are required to keep your genital clean and dry. Please wash with warm water twice a day to avoid any infection. You may apply neosporin ointment over the torn frenulum as you are doing. Please take oral pain killers like Tylenol for pain control.Since the bleeding seems to have stopped, you do not need surgical suturing. It may happen that a scar might form in the area, if this happens then there is shortening of frenulum and might be released by minor surgery . This cannot be known now and will be obvious only after one month.Please wear loose inner clothes and maintain genital hygiene. Avoid masturbation or sex for two weeks. Please explain this in detail to your girlfriend, I am sure she will understand that your penis wound needs to heal to avoid complications."
},
{
"id": 160961,
"tgt": "Suggest treatment for constipation in a child",
"src": "Patient: My daughter is 19months old and extremely constipated. Currently we are giving her 225ml Pediasure in the morings and 225ml Novalac 3 in the evenings. She is not eating bread and gets a fruit every day. She is a very picky eater. She currently weighs 10kg and is very active. We stopped giving her Creche Guard and Viral Guard, it seems as if the iron is making her more constipated. We are giving her Dupholac to help her with the constipation (5ml once a day). Will it help if we change her formula to Lactogen 3? Doctor: Hello, Laxative syrups can be tried empirically. Most importantly conditions like hypothyroidism must be ruled out as it is a major cause for constipation in children. Hope I have answered your query. Let me know if I can assist you further. Regards, Dr. Shinas Hussain, General & Family Physician"
},
{
"id": 200152,
"tgt": "Suggest treatment for erectile dysfunction",
"src": "Patient: hi, i used to use hand to masterbate 4-5 time everyday when i was 15-16. then i used to watch porn and masterbate on bed with pushing pelow and sometime as keeping my chest and panis on the bed and giving pressure and pushing on my panis . first time i tried to have sex with my gf and i saw my panis not really enough strong. and it was masterbated with a minute....now im 24 years old.. i really want to get ride of this problem and want your help and suggesion to get in normal. my panis about 4-5 inch long and a litlle bit bend in right turn in the front... thank u Doctor: Thanks for asking in healthcaremagic forum Masturbation is not a disease, done only when excited. But making it a habit doing 4-5 times can harm you. Please divert your mind for outdoor games, reading novels, socializing and by not being alone in your room. All the best. Size does not matter if you have normal erection and know the art of satisfying your partner."
},
{
"id": 52520,
"tgt": "Does large amount of pain killers during pregnancy cause liver problem?",
"src": "Patient: I am a 35 year old female who has a slightly enlarged liver and elevated liver enzymes following a pregnancy complicated by severe bilateral hydronephrosis. I recently had litho twice to remove stones on left side ( one 13 mm, one 10 mm and one 5 mm). I am having problems with random severe cramps on the right front side of my lower rib cage. My dr. gave me muscles relaxants to carry with me all the time but they make me too tired to function. Do you have any suggestions as to the cause of the pain and treatment for it? Could the large amounts of pain medications taken during my pregnancy be the cause of my liver problems? Doctor: Hello and Welcome to \u2018Ask A Doctor\u2019 service. I have reviewed your query and here is my advice. Prolonged use of pain killers may cause adverse effects on your liver and kidney. It may also affect the health of your baby also. Better to avoid pain killers especially in pregnancy period. You can take paracetamol if required. Analgesics like Diclofenac and ibuprofen should not be used in pregnancy. Hope I have answered your query. Let me know if I can assist you further."
},
{
"id": 92305,
"tgt": "Experiencing intermittent pain near bulge in belly area very near to liver, gallbladder and colon",
"src": "Patient: I am a male 65, 5'10\" 310# I have a slight bulg in my belly area. It is just below my right ribs about five inches from the point of my sternum. I have experienced intermittent pain at this location for the past year. I am concerned that it is close to my liver and gallbladder as well as my colon. It doesn't react when I have gas and pressed on by my fingertips. Doctor: Hi.You should get one ultrasound whole abdomen done at fasting level to see the cause of this bulge .It is already 1 year now.By that time I will advise you not to take spicy, fatty meal .Take some antispasmodic for pain and a proton pump inhibitor, like Pantiprazole.thanks."
},
{
"id": 39466,
"tgt": "How to get rid of anal itching?",
"src": "Patient: hello doctor,I had an itching problem in and around anus . After 3 days suspecting it as a case of piles I consulted a surgeon . He after examination with proctoscope found nothing of piles. problem of pinworms and fungal infection. He prescribed Allworm one tob taken at night and Flucose150 one tablet to be taken for a day.along with Tribenplus skin cream to be aplied in the area.aafte 2 days of application since i didnt get much relief I tried Zole F , which gives some better relief but not completely cured. My question is should i continue Zole F alone or should I take Flucose 150. If so how many more doses i should take to get rid of the fungal infection completely ?Kindly advise me. Doctor: Taking tablets without proper diagnosis is like throwing arrows in dark.My advice that as you have consulted the surgeon & surgical cause is ruled out, you must take a proper dermatologic consultation as many dermatological problems can not be diagnosed unless and until visual impression is made. The causes of itching in said region may be fungal infection or the any worm infestation most common of which is round worm.Considering round worm infestation as you have already taken all worm tablet my advice would be wait for days or two as the clearing of worm may require few days after having tablets. If there is relief in few days You should repeat the same tablet ( albendazole chewable Would be better choice ) after one week.If the itching is not relieved in a day or two you must consult a dermatologist he would prescribe you proper antifungal for oral as well as topical use."
},
{
"id": 161147,
"tgt": "Why would a child be physically violent with his siblings and act out his anger?",
"src": "Patient: My nephew is 7 yrs old and has been acting out his anger. He is on meds but this evening while at daycare, he kicked his 3 yr old brother in the side of his head, threw his 6 yr old brother across the floor and hit the daycare owners son. I am not sure of all meds he is on, but he needs help. Doctor: Hi, Tantrums may happen when kids are tired, hungry, or uncomfortable; or because they can't get something (for example, an object or a parent) to do what they want. Learning to deal with frustration is a skill that children gain over time. Tantrums are common during the second year of life, a time when language skills are starting to develop. Because toddlers can't yet say what they want, feel, or need, a frustrating experience may cause a tantrum. As language skills improve, tantrums tend to decrease. This is what you can do to avoid or prevent tantrums - 1. Give plenty of positive attention 2. Try to give toddlers some control over little things 3. Keep off-limits objects out of sight and out of reach 4. Distract your child 5. Help kids learn new skills and succeed 6. Consider the request carefully when your child wants something 7. Know your child's limits. Hope I have answered your query. Let me know if I can assist you further. Take care Regards, Dr Sumanth Amperayani, Pediatrician, Pulmonology"
},
{
"id": 155174,
"tgt": "Is extreme thirst after chemotherapy normal?",
"src": "Patient: Hi, My dad just finished his 4th round of chemo (Alimta and Carboplatin) for his lung cancer following pneumonectomy. However, since his 3rd round of chemo, he has been excessively thirsty. First we thought its good that he s drinking lots of water but now its getting too much especially when he ends up going to bathroom way many times. Plz help! Doctor: HiThirst after chemotherapy is very common.I would advise you to drink at least 3.5 litres of water per day.RegardsDr de"
},
{
"id": 132492,
"tgt": "Would taking Simivastin cause bad muscle pain in legs?",
"src": "Patient: What is a good over the counter med I can take for chloresterol? I took simivastin for 2.5 months and developed really bad muscle pain in my legs. My doctor gave me Zetia to try. My research shows major side affect for muscles with Zetia as well. Any suggestions? I walk 3-4 miles 5 days a week. I m not overweight. Thank you Doctor: please dont try to take medications for cholesterol without a prescription from a doctor because it will do more harm for you than benifits. actually there are no over the counter drugs for cholesterol. your leg pain may has causesd by skeletal muscle damage as a side effect of simvastatin.so it is not suitable drugs belongs to statin group such as atorvastatin , rosuvastatin and simvastatin. drugs belongs to fibrinate group such as gemfibrosil or fenofibate may be helpful with diet control with low fat diet. so first do a lipid profile after 14 hours fasting .then show the report to your family physician or general physician.tell him what happened to you when take statin. he will decide what is the most suitable management for your cholosterol."
},
{
"id": 124416,
"tgt": "How to treat sharp pain in my shin and knee bone poat horse riding injury?",
"src": "Patient: I was riding my horse at a trot and she ran me into a fence. The end of which hit me hard the shin bone. It s swollen and hot, I can walk on it but I can t put weight on it for a long period of time. I get sharp pains every now and then. Since I hit it farther up (6 inches from top of the knee) I feel pain in my knee too. Should I go to the ER or wait it out? Doctor: Hi, It could be a contusion only. As a first line management you can take analgesics like Paracetamol or Aceclofenac for pain relief. You can also apply ice packs for faster recovery. Hope I have answered your query. Let me know if I can assist you further. Regards, Dr. Shinas Hussain, General & Family Physician"
},
{
"id": 209746,
"tgt": "What causes anger,fatigue and depression?",
"src": "Patient: Im 16, healthy (eat very healthily and exercise regularly, get 8hrs+ sleep) but always feel angry, fatigued and frustrated. I fall asleep at the same time every day, around 12pm. I went to the doctor about it and he said i may have slight depression, but im not sure. Doctor: Hi,I read you query and do understand your concern. The symptoms that you have mentioned are suggestive of depression. However, it is also important to rule some medical disorders like thyroid dysfunction, anaemia. Complete blood counts and T3, T4, TSH levels will be needed. You ca discuss regarding the same with your physician.In case your reports are normal, I would advise you to consult a psychiatrist for starting anti-depressants like sertraline/ escitalopram for your condition. These medications are safe and effective. In addition, also start exercising on a daily basis.Hope this information was helpful. Best wishes."
},
{
"id": 127814,
"tgt": "Is soreness and black discoloration on the knee post an injury a concern?",
"src": "Patient: I experienced acute pain in left knee due to arthritis and bone on bone.in ER doctor numbed knee then removed fluid from under cap to test. Results showed no inflammation, no infection, no gout, . Knee replacement distinct possibility. I made appt. With surgeon for 1week from now. However my knee is very sore, and black and blue all around it and above and below. Am concerned if this is normal for problem experiencing. I take Coumadin for blood thinner and have for quite a few years. Last INR a day ago was 2.7. Goal is 2-3. Should I see a doc now or can I safely wait the week before I see the surgeon? Thank you. Doctor: Hello,What I can infer from your complaint is that you have had an episode of knee effusion, and as you are on blood thinning medications the blood collection and time for the hematoma to get absorbed will take time. It\u2019s safe to wait for a few more days and then consult your surgeon.Hope I have answered your query. Let me know if I can assist you further.Regards, Dr. Santosh S Jeevannavar"
},
{
"id": 63431,
"tgt": "How to get rid of lump in front of my ear?",
"src": "Patient: I have a pea sized lump infront of my ear next to my side burn. It goes smaller then bigger then smaller and so on. It is movable and kind of firm but sometimes it goes softer. Ive had a cyst before on my right eyebrow, do you have any ideas about what this lump could be? Doctor: Hi, dearI have gone through your question. I can understand your concern.You may have lump with change is size. It can be due to enlarged lymphnode due to reactive hyperplasia or infection in parotid gland. You should go for fine needle aspiration cytology of that lump. It will give you exact diagnosis. Then you should take treatment accordingly.Hope I have answered your question, if you have any doubts then contact me at bit.ly/Drsanghvihardik, I will be happy to answer you.Thanks for using health care magic.Wish you a very good health."
},
{
"id": 86992,
"tgt": "What causes dull pain in my abdomen?",
"src": "Patient: last night I had severe pain in my right side mid abdomen. It hurt while using restroom and continued while I walked to my bed. It was painful to walk and to try to stand straight. It lasted about 3-5 minutes then I was able to get back into bed. Today I have dull pain in the same area. Slight body ache, headache, and slight nausea. Doctor: Hi. Thanks for your query. You have got a classical history of pain due to a stone in the mid-ureter. Yoiur history of sudden pain while going for restroom and then a dull ache indicates the stone in the ureter. I would advise you to have urgent ultrasonography, Urine tests : Routine, microscopy, culture and sensitivity.Once the diagnosis is made you can have treatment as per the the situation which can be a lithotripsy, ureteroscopy or open surgery"
},
{
"id": 189264,
"tgt": "Chapped lips, really red with tiny blisters. Can I get ir rid of it? Exfoliate my lips?",
"src": "Patient: I have badly chapped lips, that are really red and they seen to have tiny blisters. I've tried vaseline which has helped a bit but then it ends up getting worse. Is there a brand of chapstick or lip therapy or medication that would heal this? I've been drinking water and I don't lick my lips. I'm really trying to get rid of it since the weather has been getting hotter and I don't want it to get worse. Also I don\\t want to exfoliate my lips because they hurt even when water touches it. What can I do to make it better? Doctor: Hello, The bilsters on lips may be due to aphthous stomatitis. This is usually caused due to- Dehydration. Nutritional deficiencies. Allergic to weather,food items etc. I would suggest you to drink plenty of water and fluids too keep yourself nourished. Take nutritional rich diet. Avoid exposure to too cold/hot weather. TAKE VIT.C,FOLIC ACID,iron supplements. Plenty of fruits and vegetables has to be taken. Hope this helps."
},
{
"id": 197551,
"tgt": "Is there any remedy to reduce FSH levels in a male and to concieve?",
"src": "Patient: Hai..Dis is Victoria...Im having a query for my husband..He is 28 years old,recently we came to know that the FSH level is very high..Doctors are suggesting to go ICSI..But we dont know what decision to take...Is there any way to reduce the FSH level? Do we have any chance to concieve? Plz reply... Doctor: Higreetings. Your husband's FSH is high means his testis may not be functioning properly.To know that we will need LH,TOTAL TESTOSTERONE Prolactin values.He may be having hypergonadotrophic hopogonadism meaning in spite of stimulation by the pituitary gland testis is not functioning.After analysing and confirming we should find out the cause for testicular problem and if correctable should be treated. what about his semen analysis.? If the count is low and testicular problem cannot be treated then IVF _ICSI can be considered.Hope my answer helps you. Regards"
},
{
"id": 210018,
"tgt": "How can one deal with obsessive behaviour?",
"src": "Patient: HI, Doctors I want consultancy for my nephew,his behavior is obsessive now these days. I need a visit to a Doctor in my nearby area, . We live at Bhilwara (Rajasthan)-India.Please advise me who is the doctor of these matters at Udaipur, Ajmer or at Jaipur in India. Doctor: HiThanks for using healthcare magicIn case of OCD, he needs antiobsessional drugs. Best doctor in this field in Jaipur is Dr Shiv Gautam. You can consult me. In case, you need any further help, you can consult Dr. Abhishek Kapoor in Gurgaon. He is also well known in field of OCD and knows behavioral therapy in OCD. You can also take online consultation.Thanks"
},
{
"id": 177594,
"tgt": "What causes cough, cold and heavy breathing?",
"src": "Patient: i want to know exactly what it is wrong with my 2 and half year old son has he has been occasionally having cough and cold and is unable to sleep at night. my doc presribed ventolin syrup sugar free for him and it relieved him we had an xray done and his chest was clear. doe it means my son has asthma or we are preventing it.Doc also said we should have the drug handy at home. when he cant sleep his breathing is fast and you could see his chest /stomach really rasing up and down very fast. Doctor: Hello,Thank you for asking at HCM.I went through your son's history and would like to make suggestions for him as follows:1. I would like to know his symptoms in great details before considering whether he has asthma. I would like to know more about him like - a. frequency of his respiratory symptoms b. since what age c. requirement of nebulization d. any gastric symptoms e. any other allergies f. family history of allergies/asthma etc etc2. At present, I would suggest you to give him combination of montelukast and levocetirizine for 1 months. 3. It is true that for fast breathing and wheezing (whistling sound from chest), some drugs should be handy at home. Were I treating you, I would suggest you to have salbutamol in inhaler form with spacer and mask device or nebulizer at home, as it becomes effective earlier than an oral drug.4. Childhood asthma is very different from adult asthma and it requires very detailed history before diagnosing asthma. 5. In general, I would suggest your child avoidance of smoke, air pollution and dusts as much as possible and a healthy diet rich in vitamin and nutrients to improve immunity.Hope above suggestions will be helpful to you.Should you have any further query, please feel free to ask at HCM.Wish your child the best of the health ahead.Thank you & Regards."
},
{
"id": 115251,
"tgt": "What does the following CBC result suggest?",
"src": "Patient: Pls interpret my CBC result: Hemoglobin 123gm/L,Hct 0.37, Leucocyte 7.4, neutrophils 0.71. myelocytes segmenters 0.71, lymphocytes 0.29... I had also URINALYSIS :PH-6.0, SG 1.020, Sugar(-), Protein Trace, Epithelial cells MANY, Mucus Thread MODERATE, Bacteria FEW, RBC 0-3, PUS CELLS 15-20, Urates FEW.....Thanks RADZ 37y.o., female from philippines Doctor: Hi!Welcome to HCM!I can understand your problem, nothing to worry!As per the laboratory shown in your question, you have infection of urinary tract. Do you get fever, back ache, or burning sensation while passing urine?Take Ciprofloxacin tablets 500 mg twice daily for 7-14 days. If you have fever, you can Paracetamol tablets 500 mg 3-4 times daily till fever subsides.Besides you should take lot of oral fluids, juice etc so as to maintain a good urine output.Take care!"
},
{
"id": 119474,
"tgt": "Why do I have numbness in palms?",
"src": "Patient: hi im maria, i just want to know whats going on with my left and right palm ang fingers, this past 2 weeks i always felt numbness, everytime i hold something for just a minute , suddenly my hands get numbs. im 40 years old , 53kls. 5 1 . i had an operation in 1996 , right thyroid removal. Doctor: Hi, Your symptoms and your personal profile points towards carpal tunnel syndrome. Carpal tunnel syndrome is the condition in which the space underneath the carpal ligament is reduced causing pressure over the passing median nerve resulting the symptoms of numbness, paresthesia and pain. It is very common in females of more then 40 yrs of age, who are hypothyroid. To confirm the diagnosis you need to get a USG or MRI (wrist) is required. As far as treatment is concerned it ranges from conservative with splints and nerve strengthening drugs to operative release of carpal ligament. Take care. Hope I have answered your question. Let me know if I can assist you further. Regards, Dr. Rohan Shanker Tiwari, Orthopedic Surgeon"
},
{
"id": 159710,
"tgt": "Reoccurring hard and red bumps on side of neck. Concerned about cancer?",
"src": "Patient: I have about 6 bumps on the left side of my neck located near my ear and neck. They have come and gone over the past year, but I now have 6, when usually the most I ever found at one time was two or three bumps. They are hard and red and tingly. I am going to the doctor this week, but I am concerned it may be cancer . Please advise and give me your opinions. Thank you Doctor: Hai, Thanks for writing in. 1. Seems they are lymph nodes. Red, painful nodes indicates kind of inflammation in oral cavity / ear-nose-throat / head / scalp. 2. Do you have caries of tooth? discharge in your ears? dandruff? These can cause such lymphadenitis. 3. Non-tender (painless) nodes may be a sign of cancer. But such nodes don't wax & wane. So mostly you are not suffering from any cancer. 4. It is advisable to get an ENT consultation to get your throat & oral cavity examined, which are the usual causes for such nodes & proceed further. 5. Salt water gargling, good oral hygiene, dandruff control, may be short course of antibiotics will be helpful to you. Get well soon, Regards, Dr.Sathyadharan"
},
{
"id": 185225,
"tgt": "Can root canals cause pain in the upper cheek bone?",
"src": "Patient: Daily aching pain in upper left cheek bone. Had an apicoectomy on number #13 8 months ago after having the root canal reworked twice over the lsat 4 years. Since then, #11 spontaneously died and I've had to have a root canal in that tooth along with #14 and #12. Now, I have no roots in my upper left-side teeth but, I'm still having constant aching to the point where I've been taking ibuprofen daily and sometimes something stronger. According to the dentist x-rays there's no longer any infection and everything is healing fine. Any suggestions to what might be going on? Doctor: Hello!Thank you for posting here.Pain in the area even after 8 months of treatment is not very common.You did not mention your age and if you have any pain while chewing.Make sure you do not have sinusitis as it may mimic dental pain at times. In that case, you must get it treated first.Your dentist tells its all normal in terms of healing.Do you have pain while chewing on that teeth?If yes, then it is a dental issue.Get a orthopantomogram done on your jaws.You do have roots in your teeth.Only the last part of the root will be removed in apicectomy.Prolonged doses of painkillers is not advised as it can have side effects.I advice you to see a oral surgeon immediately.This can be atypical facial pain.However, i suggest you to rule out any dental cause.Regards."
},
{
"id": 36456,
"tgt": "Suggest a diet plan for a TB patient",
"src": "Patient: Hi my mother she is a tb patient and she grew thin. She is still coughing...She has been taken doses for tb for 2 months and she was advised to tk medicine for 4 extra months..Cud u suggest bettr supplement fruits n veggies to gain health.. She is feeling too weak.. Doctor: Hi,Tuberculosis is a wasting disease so to gin muscle mass she requires high protein diet like milk, cheese, panier, eggs, puddings, pulses and cereals.Give her fresh vegetables.Give her fresh fruits and juices.Give her dry fruits as well.Give her moderate amount of fatty and carbohydrate food.Along with anti-TB drug she should take B.complex and multivitamine medicine.Go for regular checking of her weight.Ok and take care."
},
{
"id": 180915,
"tgt": "What does a blister on the lower gum indicate?",
"src": "Patient: I have a very sore spot on my bottom lower gums in front. Not red or bleeding. lt Started about three days ago. I just had a cleaning about two weeks ago and bite wing xrays. No issues at that time. I do have receding gums but the hygienist told me that they were stable. I have been using warm salt rinse with just a have received just a small about of help. Doctor: Hi..Thanks for the query..A sore spot in the gum can be due to injured gum in that area.It can cause inflammation and soreness of gum and at times formation of an abscess..It can occur due to tooth brush injury, any hard food, foreign body or food particle impaction in the gum etc..So for now you can apply Chlorhexidine ointment over the gum.Apply Lidocaine gel to reduce pain..Do warm saline gargles and antiseptic mouthwash gargles..In case if there is no improvement consult an Oral Physician and get evaluated..Localised cleaning and a short course of antibiotics can help in improving..Hope this helps..Regards.."
},
{
"id": 151755,
"tgt": "Is there a treatment for my brother who forgets an event that just happened 1-2 hours back ?",
"src": "Patient: Dear Sir, My brother who is a Doctor aged 55, stays in France. In 2004 he had a heart attack and he went into a coma for some 4-5 days. Upon regaining conciousness, he looked fine with no physical abnormality. Then Doctors concluded that he had a problem to remember things that just happened. For example he forgets an event that happened just 1-2 hours back.But he remembers everything about his past, his childhood, his marriage, etc. Doctors have condemed him that he will never recover. He is actually staying in a \\ Doctor: yes,it happens,In such case recent memory loss is there but past memory is o.k."
},
{
"id": 92753,
"tgt": "Abdominal pain. Negative for hernia and appendix infection. Help",
"src": "Patient: Hi, over the past four weeks I have been suffering right abdomen pain. It started off on the lower part of the right abdomen. Have been tested for hernia and also a infection in the appendix , they both came back negative. I was then requested to take time off from physical activities - as I am a very active person. This was for a strained or tear muscle. I took nearly three weeks off. Over the past three weeks I have been suffering upper right abdomen pain as also as the lower right abdomen pain. I feel that the upper right abdomen pain gets worst immediately after eating food. Doctor: Welcome to HCM.You have to go for to rule out any stone in urinary system,inflammation in testis and urinary tract infection by proper USG study.Otherwise it might be due to indigestion pattern to some food,disturbances in diet habit,stress.There might be possibility of irritable bowel syndrome.Avoid stress,take simple diet,drink more water and buttermilk,fruit juices,vegetable soups.Consult your physician for better guidance and proper medical management."
},
{
"id": 209460,
"tgt": "What causes a feeling that someone is standing behind?",
"src": "Patient: Everytime im not doing anything and there is a corner or something behind my, i have a really powerful urge to turn around and look at it for 5 seconds. This happens every 5 seconds and if i dont i start hallucinating something is behind me and the feeling starts destroying my brain mentally. What would this be because i cant stand it anymore i want to live life without looking behind me every 5 seconds.Thanks Doctor: Hello,Thanks for choosing health care magic for posting your query.I have gone through your question in detail and I can understand what you are going through.It could be a presentation of OCD. This will require treatment with fluvoxamine or clomipramine or fluoxetine etc. Hope I am able to answer your concerns.If you have any further query, I would be glad to help you.In future if you wish to contact me directly, you can use the below mentioned link:bit.ly/dr-srikanth-reddy\u00a0\u00a0\u00a0\u00a0\u00a0\u00a0\u00a0\u00a0\u00a0\u00a0\u00a0\u00a0\u00a0\u00a0\u00a0\u00a0\u00a0\u00a0\u00a0\u00a0\u00a0\u00a0\u00a0\u00a0\u00a0\u00a0\u00a0\u00a0\u00a0\u00a0\u00a0\u00a0\u00a0\u00a0\u00a0\u00a0\u00a0\u00a0\u00a0\u00a0"
},
{
"id": 136588,
"tgt": "Suggest treatment for swollen and painful ankle",
"src": "Patient: Metal adjuster on a water hose hanging from the ceiling was swinging and smacked into the inside of my right ankle on the bone. It has been two days I still have shooting pain dull throbbing pain little swollen achy sometimes can t bear weight on it. Should I see a Dr? Doctor: Hi there. Thanks for your question.Rest, Analgesics, Ice and bandage application can help relieve the pain and swelling associated with your ankle injury.48 hours after injury if you have pain while standing or walking it needs further evaluation and to be seen by a physician in person.Hope this helps. All the best. Regards. Dr.SBK"
},
{
"id": 92806,
"tgt": "Vomiting, fever. USG scan done shows minimal free fluid in abdomen and pelvis, thickened omentum. Anything to worry?",
"src": "Patient: sir , i was having fever for for last 4 days now i am alright but before 2 days i started vomiting whatever i am taking including liquid items . so today i have taken usg scan & in that report they have mentioned minimal free fluid noted in abdomen & pelvis . minimal free fluid noted in right pleural cavity .& thickened omentum with minimal arcitis . all other thinga are normal. pls suggest your comments thank you Doctor: WElcome to HCMGreetings of the the dayYou could be having abdominal tuberculosis. Get an Erect X-Ray Of Abdomen done to rule out intestinal obstruction. Pain in abdomen, distension of abdomen, not able to pass stools could be the other features which suggest obstruction. Consult a general surgeon. Don't be worried things can be taken care.Regards"
},
{
"id": 197861,
"tgt": "What causes pain in my chest, hands,and legs after masturbation?",
"src": "Patient: hi doctor,am having unbearable pain in my legs after masturbation and also having pain in my chest and hands.are these due to increase in masturbation or sumthing else ?also having sharp pain at upper left chest which vanishes after flashing 5,8 times. Doctor: Dear, We understand your concernsI went through your details. I assure you, the present symptoms, pain in your legs and the chest pain, are not associated with the current masturbation. Masturbation consumes a good amount of energy. If you masturbate in an already tired condition, it is quite natural that your body will show the fatigue and pain. Due to frequent masturbation, your energy will be used much more without replenishing energy. This could cause fatigue, tiredness and thereby depression and anxiety. Depression, stress and anxiety are known to produce physical symptoms such as pain. Still, as the chest pain is involved here, I suggest you to consult a physician for investigation.If you require more of my help in this aspect, please use this URL. http://goo.gl/aYW2pR. Make sure that you include every minute detail possible. Hope this answers your query. Further clarifications are welcome.Good luck. Take care."
},
{
"id": 113295,
"tgt": "Burning pain in the back radiating to the front side. Pulled muscle?",
"src": "Patient: Hi i have this back pain in the middle area on my left side and it reallh bothers me the burning and achy pains radiate to my front side which makes it difficult to sit. The pain is not brought on by any paticular activity one day its hurtd after just sitting another folding laundry or just vacumming i have had went to the doc more than once and just told its a pullex muscle this has been going on for.over two years what else could it be? Doctor: Hi Madisonsmommy, Thanks for writing your query. After reading your post, it seems that your pain is either muscular in origin or has origin in kidney. Still, you should get yourself examined by an Orthopedician and physician and get the following investigations done :- 1. Kidney Function Test. 2. Ultrasound KUB region. 3. MRI lumbosacral region. 4. Urine examination. If there is no abnormality in the kidney, you can take pain killer like Diclofenac 50 mg twice a day after meals along with an antacid like Omeprazole 20 mg and a muscle relaxant like chloraxazone. Physiotherapy will also be beneficial in relief from pain due to muscle spasm. I hope this is helpful to you. Thanks."
},
{
"id": 136439,
"tgt": "what causes pain in the hip joint?",
"src": "Patient: I injured my hip while golfing about two months ago. The pain seems to radiate from my hip joint, and travels down my thigh muscle and into my shin. Can you tell me what I have injured. and will the pain go away without a visit to the doctor? Thank you Pat. Doctor: Hi welcome to HCMI have gone thru your query regarding pain in your hip joint .Dear ,To know what you have injured you have to under go an Xray . It is obvious , when you get injury in some part of your body , surrounding areas get affected , you feel pain and swelling all around your injured part . It heals gragually , takes time to heal fully . Modification in Recreational and Daily Activities will help accelarate process of healing : While in pain , rest gives intant relief . Lie down comforting your hip and knee with a soft pillow Apply hot pack on painfull part after applying analgesic ointment Do not work through pain.When walking or running, pay attention to pain. If the discomfort accompanies running, then stop. If it occurs with walking, lessen your stride length by taking smaller steps. A core strength exercise won't harm and can actually help to \u201cre-educate\u201d the lower abdominals so that they provide the best support.Lie with knee bent up.Maintain a neutral/comfortable spine position. .Gently , slowly raise your hip and your lower back , make sure if it hurts .Stop and come back slowly and rest Take a T spoon of Turmeric powder + Almond oil with a cup of hot milk thrice a day for 3 days . It is a natural antibiotic and has no adverse reaction , alleys pain and inflammation fast .sun bath for vit D and C,in natural form as lemon juice Bannana If problem persist , it is advisable to consult an ortho expert .Hope this helps solves your query .Take care , All the best .Don't hesitate to get back if have any further query"
},
{
"id": 136718,
"tgt": "What causes pain and swelling on ankle after hitting on a metal?",
"src": "Patient: When walking I hit upper outside of knee on a metal part of a trailer. It hurt really bad when it happened and was sensitive, it has continuously hurt more but I can walk on it as long as I don t bend it. If I bend it, it takes my breath away and hurts. It is swelling, not hugely but a noticeable amount. I m wondering if it s possible to have fractured or hurt something more than just I thermal bruising. It s been less than 24 hours, but hurts more as time goes on. Should I go to the doctor or just rest it? Thanks!! Doctor: Hi,Thanks for your query.An initial xray of the injured part is mandatory to rule out any bony injury (fracture). If the xray is negative for a fracture, then the other possibility is soft tissue injury either to the ligaments, tendons etc.The initial management for your condition is as follows -- Give adequate rest to the injured part and avoid any movements that produces pain- Apply ice wrapped in a damp towel to the injured area for 15 to 20 minutes every two to three hours during the day- Compress or bandage the injured area to limit any swelling and movement that could damage it further. You can use a crepe bandage or a simple elastic bandage - Keep the injured area raised and supported on a pillow to help reduce swelling. If your leg is injured, avoid having long periods of time where your leg is not raised.I do hope that you have found something helpful and I will be glad to answer any further query.Take care"
},
{
"id": 114254,
"tgt": "I have been pooping many times with headache. Kindly suggest",
"src": "Patient: I have been pooping none stop with very bad back pain and a head ack aswell. I also feel like I'm going to trowup what should I do Doctor: How did all this start? How many times di you vomit? If the vomiting is not getting controlled go to ER and take medical help."
},
{
"id": 13402,
"tgt": "What could rashes on hand with sore throat suggest?",
"src": "Patient: I have a rash on my hands and around my toes nose and chin. Its very itchy painful to walk with pins and needles in hands and feet. The spots break and weap if scratched. I was also ill yesterday with a slight temp sore throat and headache blocked ears and blurry vision and dizziness. What can it be? Doctor: Hi, It may be a viral disease. Most probably hand, foot and mouth disease. Still, consult the dermatologist for the perfect diagnosis and proper treatment. Antiviral like Acyclovir may be taken. Take rest. Antipyretic and antiinflammatory drugs may be taken under observation of doctor. It may vanish in two to here weeks. Hope I have answered your query. Let me know if I can assist you further. Regards, Dr. Ilyas Patel, Dermatologist"
},
{
"id": 23249,
"tgt": "Can Levothyroxine cause arrhythmia and hypotension?",
"src": "Patient: I was diagnosed with hypothyroidism in April. My doctor prescribed me 125 mcg of Levothyroxine to start, and after 2 weeks reduced dosage to half. After 2 weeks on this dosage, my blood pressure dropped severely and I was syncope and admitted to the hospital. Dr. took me off Levothyroxine as he said I developed an arrhythmia, hypotension and sick sinus, and now has me on fludrocortisone to stabilize my heart rate. My question is can the levothyroxine have caused my heart problems? I never had any heart problems in the past and am very healthy & active. Doctor: DEAR USERTHANKS FOR CONSULTING AT HCMYES LEVOTHYROXINE CAN CAUSE ARRYTHMIA ONLY IF YOU HAVE TAKEN EXCESS DOSE OF THYROXINE MISTAKENLY.. BUT YOUR DOSAGE APPEARS TO BE NORMAL FOR HYPOTHYROIDISMHOWEVER IF YOU COULD SEND THE TSH AND T4 REPORTS V COULD TELL YOU ABOUT YOUR HYPOTHYROIDISM..YOU ALSO REQUIRE A ECG AND A CARDIOLOGIST CONSULTATION FOR THE ARRYTHMIAHOPE I HAVE ANSWERED YOUR QUESTION.. YOU CAN MESSAGE ME FOR ANY FURTHER CONCERNS"
},
{
"id": 9912,
"tgt": "What causes hair loss and itchy scalp?",
"src": "Patient: I am 22 and my front hair is thinning. This started maybe two weeks ago, up until then all was fine but i have begun noticing hair falling off (10 strands minimum a day).. my scalp is itchier than usual but not at the areas where my hair is falling off? is this an allergy to a shampoo or hair gel? Doctor: Hi Dear, Understanding your concern. As per your query you have symptoms of hair loss and itchy scalp which seems to be due to improper cleanliness, sudden change in environmental factors, hormonal changes, fungal infection and could be due to stress. Need not to worry. I would suggest you to find the exact cause and start treatment as per that. Avoid excessive stress and drink plenty of water to stay hydrated. Maintain proper hygiene of your scalp and wash hairs every alternative day and take Indian gooseberry powder half spoon. Consult dermatologist once for proper examination. Doctor may prescribe Minoxidil preparations and along with that multivitamin preparations as well.Hope your concern has been resolved.Best Wishes,Dr. Harry Maheshwari"
},
{
"id": 57954,
"tgt": "Does large belly indication of fluid buildup due to drinking or is it liver damage ?",
"src": "Patient: My brother is 58 years old and a long time drinker. He has stopped drinking beer, but his belly has recently gotten very large (looks very pregnant), while his arms, legs and face are very thin. I know he probably has liver damage, but does the large belly indicate fluid buildup as well, and if so, would draining the fluid help him? Doctor: Hi and welcome to HCM.Yes,it seems like there is fluid accumualtion in abdomen called ascites and this is sign of serious liver damage.he should do liver ultarsound and LFT as son as possible and then the right cause should be established. Fludi can be drained also if necessary. Thanks for the query. Regards"
},
{
"id": 120884,
"tgt": "Suggest remedy for pain while moving thumb",
"src": "Patient: I fell while on crutches - my fall was broken by my hand - specifically the thenar area of my hand. I can move my thumb, but it is extremely painfull. I just had bone fusion surgery on my foot and the pain wasn t as bad as this hand injury. If I can move all my fingers am I ok? Doctor: Hello,I read carefully your query and understand your concern. The symptoms seem to be related to a thumb sprain.I suggest to rest. Try not use your hand for at least 48 hours.Apply ice immediately after the injury to keep the swelling down. Wear an elastic compression bandage to reduce swelling.Hope my answer was helpful.If you have further queries feel free to contact me again.Kind regards! Dr.Dorina Gurabardhi General &Family Physician"
},
{
"id": 54637,
"tgt": "Suggest treatment for mild chronic hepatitis",
"src": "Patient: i am a patient of liver diseases. I nee some help for treatment. I am Mizanur Rahman 37 years old, Sex- Male, from Bangladesh.Investigation of liver disease report are as follows:SGPT (ALT) -79 (05-08-2011), SGPT-94 (11-07-2011) SGPT always up and down 60 to 94 from 2008.Liver Biopsy- Mild chronic hepatitis, HAI- 7, Knodell Score : 8Microscopic appearance: Sections of liver reveal moderate piecemeal necrosis with a few tiny foci of lobular necrosis. The portal areas contain increased number of chronic inflammatory cells and show periportal fibrosis.Anti-smooth muscle antibody : NegativeCeruloplasmin Blood Serum: 462 Normal value (280-570)Anti Nuclear antibody (ANA) : Negative by ELISALipid Profile : Cholesterol (Total): 180 mg/dl, Normal value (120-200)HDL-Cholesterol: 37mg/dl normal value > 35.00LDL-Cholesterol : 116.40mg/dl Triglycerides : 133mg/dl Normal value fasting 10-190hTSH : 2.61 ulU/ml Normal value adults 0.47-5.01Ferritin : 142.80 ng/ml Normal male adult 20-300I as also a patient of Muscular dystrophy (Myopathy):Muscle, calf (biopsy): Features are compatible with myopathy.I have take Treatment : Medicine 1. Laevadosin 2 + 0 + 2 (The medicine was made by Austria, but I cannot manage it)Medicine 2. BIO-E (Vitamin E) : continued.Please suggest me.Thanking you. Doctor: Hello! Thanks for putting your query in HCM. I am a Gastroenterologist. Elevation in liver enzymes has many causes apart from which you have already got. Like Hepatitis B, C, Metabolic disorders, NAFLD, alcohol intake etc. So I will suggest you to visit a Hepatologist for evaluation who will advise tests and then manage accordingly. Meanwhile there are few medicines which I found very useful in my patients are ursodeoxycholic acid and vitamin E. Do regular exercise and reduce your weight if you are overweight, avoid alcohol if you drink.I hope I have answered your query and this will help you . Wish you a good health"
},
{
"id": 44976,
"tgt": "What all foods are helpful to maintain the follicle during pregnancy ?",
"src": "Patient: what all food helpful to maintain or increase the follicle during follicle study for prengency..naturally. doctor , i want to ask what all food products to be taken to maintain or increase the size of the follicle during follicle study..naturally. Doctor: Hi,Deepa ,thanks for query.There are no special food or diet for this.Just take healthy diet comprising of fruits,vegetables and salads,that helps in general.In case of some particular deficiency like iron ,you need iron supplements.As you are trying to conceive,please ask your doctor about folic acid supplements,which is good for you. wishing you good health."
},
{
"id": 75365,
"tgt": "Suggest cure for dull, achy pain in chest",
"src": "Patient: My question has to do with having dull, achy pain in my chest for the last 2 1/2 years, but has gotten slowly worse over that time. It started very mild and is now everyday, constant, with a little coughing here and there and it also feels like a mild chest cold sometimes. I started coughing up blood 3 months ago. I had a broncoscopy done and all that was found was a mild staph infection in my lungs. I took antibiotics and the coughing up blood is much more mild, every 3 to 5 days but very little blood is in my mucus. My chest still feels the same. it never feels good and my voice gets hoarse every day, which used to only happen 2 or 3 days a week. I had a chest ct scan, nothing was found except a very small 3mm nodule on my left lung, which means nothing I guess. They did find that my spleen has multiple leisons, one is big but I m not sure the size. They have looked at that 4 times and aren t sure about it so I am going to a G.I. specialist. I m guessing that it has nothing to do with my chest but I thought I would throw it in there. I am female, 40, 5 5 , 120 lbs. I have never smoked but grew up with smokers in the home. Diabetes is in my family. I ve always had good health but that last 2 1/2 years have been weird. I have also had a lot of problems with my bladder for the same amount of time but with no bladder infection. It is doing better but still there. I had shingles about a 1 1/2 years ago and a bad hive break out 9 months ago. Thank you Doctor: HelloThank you for asking in HCMYou explain that for lung problems you have done the lung Ct and the bronchoscopy which were negative for lung disease.This is a good news for you.There are others causative agents extra pulmonary that can cause sputum with blood.Some of them might be hematologic pathologies.So the G.I specialist combine with the hematologist maybe,will discuss together for further examination.Thank youDr.Jolanda"
},
{
"id": 107041,
"tgt": "What causes pain in the lower back after receiving an Epinephrine shot?",
"src": "Patient: I was in a operating room and was givin an epadermal and the anesthesiologist emptied a vile of epinephrine into my spine instead of what he was supposed to use, I spent two days in the hospital after he knee surgeon called off the operation after the mix up, I was in so much pain in my lower back and my legs down to my knees, i had to see a neurologist to get cleared for my knee operation and told him about I had numbing in my feet and it feels like I m wearing socks all the time, I had a EMG done yesterday for the problem and the said I had neuropathy ,I m not diabetic nor have high blood pressure or colestrol, all my blood work is fine,I did not have this numbing until that njection but no one seems to listen or care about my concern, could that injection have caused it because it changed something Doctor: hi sir/madam,Let me advice you for your problem.Low back ache may be a pre-monitory (prodromal) symptom in fistula in ano, sciatica, gouty arthritis, hernia, emaciation disorder etc. Pain is confined to lower part of the spine (back bone) especially lumbar region or lumbo-sacral area (rarely sacro-iliac region also). If it is secondary, earlier history of fall or injury may be associated. Rarely, radiating pain may be complained by the patient towards lower limbs. But it is quite common in low back aches if the defect is in the discs between vertebrae (back bones). Often the movements of lumbo-sacral region like flexion and rotation are hampered either partially or completely.Low back pain may be found in mild form in case of anemia, sciatica, rheumatoid arthritis, hemorrhoids, urinary calculi, uterine disorders etc also.Line of treatment as per Ayurveda:-The general principles of treatment of vata dosha are adopted in case of katishoola (low back pain). It includes various measures to suit its varied clinical entities, stages and associated complaints.1. Snehana (oleation) \u2013 by sneha dhara (pouring oil), abhyanga (oil massage), avagaha (tub bath with oil or oleaginous medicaments), kati basti (retaining medicaments on the back) etc.2. Swedana (sudation) \u2013avagaha sweda (sweating treatment with tub bath), pizichil (kayaseka), nadi sweda (sudation through a tubular device \u2013 local sudation), panda sweda (sudation through medicated paste or powder) etc.3. Mridu Samshodhana -mild purgation.4. Basti (medicated enema) like eranda basti, vaitarana basti, pippalyadianuvasana basti etc.Formulations indicated in Low back ache as per Ayurveda:-1. Dashamoola kwatha.2.Maharasnadi kashaya.3. Rasnaerandadi kashaya.4. Sahacharadi kashaya.5. Gandharvahastadi kashaya.6. Trayodashanga guggulu.And for local application these oils will help:-1. Ksheerabala tail.2. Mahanarayana tail.3. Dhanwantaram tail.4. Maha narayan tail.Avoid these for better results:-1.Bitter, astringent and pungent food2.Cold water3.Fear4.Exhaustion5.Standing6.Driving7.Cold food and beverages etc.Hope i was helpful.Have a healthy day."
},
{
"id": 37102,
"tgt": "Suggest treatment for fever and body aches",
"src": "Patient: i had fever started from yesterday 6.00pm, initially I feel body ache and had fever of 100. after this I had fever of 101 I had taken medicine, but no benefit I found within whole night. right now I am having body ache I am taking Flexon is this right decision. Doctor: Hi,It seems that you might be having viral infection producing body ache and fever.This will remain for 2-3 days.Mean while you can take Flexon or any other analgesic medicine as and when required.Take plenty of water.Take light diet.Ok and take care."
},
{
"id": 101587,
"tgt": "Suggest permanent solution for asthma problem",
"src": "Patient: my name is remo age 29 i am suffering from asthma. during these days i cant breath properly in night. i feel like hell, i am taking tab. asthalin-4mg still also i cant feel well. many times i had to take injection named derrifilin & dexona mixed to get relief, is there any permanent treat to this? Doctor: Thanks for your query.I understand your concern regarding breathing difficulty.Brief answer: No,there is no permanent cure of asthma but there is good treatment options in modern medicine by which you can have a very good asthma control.Detailed answer:Modern medicine has improved in the asthma management to such extent that one can remain symptoms free if he/she follows physician's advice and take medications properly.I see similar cases in my clinic.After diagnosing asthma by clinical features and objective test ,I suggest step care management.If I were the treating physician,I would advise you to :>Avoid triggers(eg.dust,cold,emotions etc) so far it is possible.>Steroid inhaler twice daily>Bronchodilator (Salbutamol) inhaler as required.For asthma management,medications via inhalation is the best method because it ensures more entry of drugs into airway with less side effects.So my suggestion is :Please meet your local doctor/respiratory medicine specialist to confirm the diagnosis and get treated thereby.Hope this helps you.If you have additional questions or follow up queries,please do not hesitate in writing us.Wish you good health."
},
{
"id": 199941,
"tgt": "What causes groin pain with a hard pinching feeling?",
"src": "Patient: Hey I m Morgan I m 16 years old don t play any sports or anything and I m not on any meds never had a surgery in my life and I have a sponge kidney about a week ago I was in bed and moved my right leg to get off the bed then in the crease where my leg starts there was a very hard pinching feeling in my groin area its been happening I can t walk or move it I can t even move I just scream it hurts and I ve had trouble going to the restroom it hurts to push idk what it is or what it could be I really need some help I hope its not my kidney messing up and I ve had a lil fever Doctor: elloThanks for query .Severe pain in groin could be due to either Inguinal Hernia or acute infection of the spermatic cord (Funicullitis)or Epididymis (Epididymitis)However this can be confirmed only on clinical examination by a qualified general Surgeon and ultrasound scanning of abdomen and pelvis In the mean while take broad spectrum antibiotic like Cefexime along with anti inflammatory drug like Diclofenac twice daily.Dr.Patil..."
},
{
"id": 8899,
"tgt": "How can I grow my beard ? I am 28 years old ?",
"src": "Patient: i m 28year old and have very little bear, please tell me how it can grow properly? Doctor: Hello. Thanks for writing to us. A small bear wants a complete care, proper diet at right temperature, maintenance of appropriate tempaerature of the surroundings and clean living area. The diet depends on its body weight and age. You can get the appropriate food packets for its age at the pet stores. I hope this information has been both informative and helpful for you. Regards, Dr. Rakhi Tayal drtayalrakhi@gmail.com"
},
{
"id": 42910,
"tgt": "What is the treatment for infertility?",
"src": "Patient: I had d & c last year march, now unable to conceived. Pelvic scan revealed free fluid in the cul-de-sac indicative of PID, I went for HVS, I was diagnose having Staphylococcus aureus and I treated it with ciprotab & ceftriaxone, still infertile. Pls what next? Doctor: hi,welcome to healthcare magic forum,as u had PID so this is an imp cause for infertility.as your history din't give other clue for the cause of infertility so better you consult an infertility expert next id no other cause has been rulled out till now.all the best."
},
{
"id": 34327,
"tgt": "Suggest treatment for yeast infection",
"src": "Patient: I have been experiencing recurring BV and yeast infections for the last 3 months. My doctor says my PH is off due to cycle because that s always when it occurs. I was put on 7 day dose of Flagyl pills followed by twice weekly gel for 4 months. After second dose and third I have been noticing dime size clumps of wet paper looking yeast (I assume). Very little itching on outside and no discomfort or other discharge. Is this common and since I have 3 1/2 months of gel to go, should I be on Diflucan long term? This is all very frustrating! Doctor: Hello dear,Thank you for your contact to health care magic.I read and understand your concern. I am Dr Arun Tank answering your concern.There is need for confirmation of the present diagnosis. I advice you should do Grams staining from the discharge. Grams staining can confirm the diagnosis whether it is bacterial, yeast or other infections.Bacterial vaginosis is bacterial infections and requires a Doxycycline and metronidazole combination.While in fungal infections requires fluconazole tablet.There are some vaginal tablets available, which requires to be inserted in the vagina. This acts locally and cab help treat the infections.Please maintain good local hygiene. Avoid sexual intercourse or can do protected intercourse.If the male partner have the same infections than both requires the same concurrent treatment.I will be happy to answer your further concern on bit.ly/DrArun.Thank you,Dr Arun TankInfectious diseases specialist,HCM."
},
{
"id": 203456,
"tgt": "What is the reason for watery semen and pain in testicles?",
"src": "Patient: I am a young man of 30 yrs. Lately I noticed my sperm is very watery. I have not had intercourse with my wife for over 2 weeks. I had some aches in my testis some weeks back and I was prescribed some antibiotics which I took judiciously. Need help badly as my wife is expecting us to have our next child. Doctor: DearWe understand your concernsI went through your details. I suggest you not to worry much. The viscosity of semen depends on so many matters like water intake, how much time you take before sex for fore play, your physical and mental state during the sexual intercourse etc. You must have noticed that the viscosity of semen is always more after masturbation than after sexual intercourse.You do not have to worry about anything for now. Conception depends on your sperm count and motility, not on semen viscosity. Don't get confused and anxious.For psychological and sexological counseling visit http://psychocure.blogspot.com/Hope this answers your query. Available for further clarifications.Good luck."
},
{
"id": 123406,
"tgt": "Suggest treatment for joint and muscle pain along with tingling in legs",
"src": "Patient: Hi, it goes like this: eating processed food, in 30 minutes get drowsy, if when taking a nap 15 minutes later waking up to heart flutter, then headache, tingling of legs and arms. Within hours having joint and muscle pain and may smell food from urine. Thanks. Doctor: Hello, I will advise about avoiding processed food. As due to this you are getting acidity and due to which abnormal reactions. Feeling drowsy is a common thing after having processed food is the metabolism is slow. Tingling in legs and arms aren't related to the processed food. It may be due to other factors and needs a separate examination and assessment. Joint pains can be due to inflammation and not related to processed food. As we aren't knowing your age and any underlying past medical history we can not connect the processed food with the current mentioned symptoms. Do some simple general exercise so the metabolism improves and you will also avoid processed food for good health. Hope I have answered your query. Let me know if I can assist you further. Regards, Jay Indravadan Patel, Physical Therapist or Physiotherapist"
},
{
"id": 210615,
"tgt": "How should we act around a 19 year old son suffering from depression?",
"src": "Patient: I have a son 19 years old and suddenly I have notice he is not well, he tells me he regrets things he did in past and cry a lot he is being seen by a therapist but he might have to talk medication as well. what me and his dad wants to know how do we have act around him, because he is the only child always in private school and as far as money he never have any problem, he is a sweet, intelligent guy, it hurts us so much to see him like that, depression runs in my family but smoking pot didn't help either, he did stop smoking just recently. please tell us how we should be around him. thanks Doctor: HelloThis is a very difficult time for the family . You can overcome this by patience . He should be seen by a psychiatrist who will start antidepressant medicine. It takes 3-4 weeks to see some improvement . You should be supportive, non demanding and try to make sure he takes adequate diet. He will be all right soon.DR SAATIISH JHUNTRRAA"
},
{
"id": 19686,
"tgt": "What causes hot flashes, fluttering heartbeat and tightness in chest?",
"src": "Patient: i think i have GAD. I get hot flashes, my heartbeat is fluttering, i have a feeling like i might die of a heart attack or a anyrsym and it occurs just about everyday at night. i feel weak, and dizzy. my chest is tight and im nervous. ive had alot of stress what do you think? Doctor: Hello there. thank you for choosing health magic. i can see that you are suffering from a lot of anxiety. All your symptoms may be a result of anxiety. But it is advisable to rule out any cardiac causes. i would advise you to get an EKG done to rule out arrythmia. If your EKG is normal then you would be requires to take some tranquillizers to reduce your anxiety. But these are prescription drugs which are habit forming so I would suggest you to discuss it with your physician. I hope I was able to answer your question. I f you have any other queries please feel free to write."
},
{
"id": 166481,
"tgt": "What causes cough?",
"src": "Patient: my 3 and 1/2 yr old daughter is having cough since last 1 and a 1/2 mths with a gap of 5-6 days(during thius time she was better) she is been kept on inhalers (2 types) but this is worrying me as she is taking a long time to improve and relapsing again n again Doctor: Hello,It's called reactive airway problem. It is very common nowadays with so many allergens and weather changes around. In general, inhaled are safe. Try to identify allergen in food or the environment.Lucking childhood problems cease on its own with age as immunity improves by age of 10-12. You can use preventive medicine called Montair LC kid (5 mg) one tablet once daily for three months to prevent repeated attacks.Hope I have answered your query. Let me know if I can assist you further.Regards,Dr. Kalindi R Shah"
},
{
"id": 129347,
"tgt": "Having noticed pain on the knees",
"src": "Patient: My 16 year old son hurt his knee in football a few weeks ago, it instantly swelled, there is a hard spot in the middle with fluid around it and feels mushy, he was able to rub the knot down a week ago, however, he came home this evening and it's bigger and he can't put any pressure on it, any suggestions? Doctor: Hello,The most frequent and probable problem of the knee injury with a mechanism and signs like you are describing is anterior cruciate ligament tear. Anterior cruciate ligament is the ligament that links the tibia bone and femur bone in the knee.When it is tear, the knee swells up immediately and you cannot continue the game. It causes the knee to \"give away\" often. If that is the case I would recommend to do an MRI of the knee and consult an orthopedic surgeon for possible treatment. The second option is a fracture of the bones around the knee that usually happen with a more high energy mechanism such as fall from height or car accident. Third option might be meniscus tear. Usually, the swelling of the knee here is not immediately like anterior cruciate ligament tear right away, but later in the evening or next morning. Again here an MRI of the knee would diagnose the problem and orthopedic consult is needed.Hope I have answered your query. Let me know if I can assist you further.Regards,Dr. Edvin Selmani"
},
{
"id": 67519,
"tgt": "Is numbness of tongue common with Alleve mouthwash?",
"src": "Patient: I had a lump (I think a traumatic fibroma) removed from the edge of my tongue 2 weeks ago. Biopsy showed it was benign.But it still is quite painful; site of lesion removal was cauterized, but it keeps splitting open...I can eat few foods.I m using Alleve and a mouthwash containing benydryl, maloox and Lidocaine. Wash helps for about 15 minutes only; makes my mouth temporarily numb.Is this healing process normal?(68-year-old woman; no smoking history.) Doctor: Hello Thanks for consulting HCMRead your query as you have undergone minor surgery healing of lesion takes time be relaxed dont take stress , as your mouthwash contains lignocaine this is anaesthetic agent causing numbness on tongue . I will also suggest you to consult your doctor for follow up .Hope this will help you."
},
{
"id": 201671,
"tgt": "Is impotency related to baldness?",
"src": "Patient: Thank you. Is there a connection between early male baldness and impotence especially if the baldness takes in the whole top of the head with hair remaining on the sides and back or is that just an old wive s tale? Someone we know in his early 30 s had a very short term marriage (2 months). He is interested in remarrying but we were wondering if there is any connection?Thank you. Doctor: Hello,There is no connection between impotency and baldness. This might be just a misonception. There is no need to worry.Thanks."
},
{
"id": 107299,
"tgt": "What causes pain in the flank region?",
"src": "Patient: I have dull flank pain, both sides of my back just under rib cage. Went to emergency room the other night and Urine sample and bloodwork turned out good and xrays were clear. made doctors appointment and just trying to do some research ahead of visit. Doctor: Hi,This may be caused by tiny urinary / renal calculi.You need to get ultrasound abdomen done.Drink plenty of water and liquids.Thank you."
},
{
"id": 151244,
"tgt": "Headache, upper lip vibration, heaviness in cheek, normal MRI. Help",
"src": "Patient: hi i m sheetal. i m 29years old. i m not married. before two months i feel pain in my left side headache . from the very first week my left upper lip vibrate when i felt pain.. now it stop... now my upper left cheek feel heavyness.. i m too worried... i get my MRI but its normal. i got check my eyesight. its also perfect.. plz help me... one of the doctore prescribe me nulokind more and neurobion forte... Doctor: Hi Sheetal, Thanks for writing your query. One sided headaches are usually Migraine headaches. Migraine can be triggered by many factors, common of them are:- 1. Lack of sleep 2. Stress 3. Missed meal 4. Heavy exercise 5. Menstural periods 6. Dairy products 7. Alcohol and smoking You can look out which triggers your attack and can avoid it. MRI is usually normal in migraine. Taking Paracetamol along with Domperidone is helpful. Please get yourself examined by a Neurophysician. I hope this is helpful to you. Thanks."
},
{
"id": 135551,
"tgt": "Suggest treatment for swollen and painful neck with history of broken disc",
"src": "Patient: In 2008 I fell off a wall and broke c-5 and c-6 in my neck yesterday I woke to my neck swollen and very painful I tried ice pack 15 mins on then heat pad 15 mins and 600 mg ibuprophen today the swelling continues and the pain is worse should I go to the emergency room? I have Medicaid coverage. I am male 57 y.o. Otherwise in good health. My b.p. 184 sys 96dia heart rate80 bpm. I am experiencing tingling and some numbness in left arm and hand Doctor: HiU might be having acute disc prolapse with nerve compression If d pain is worse u shud consult and get an MRI doneTill then take analgesics. .rest n use a cevical collar.. local hot fomentation Have a good dayTake care"
},
{
"id": 38870,
"tgt": "Does pus from an external boil drain into the body?",
"src": "Patient: I had a boil cut and drained on the back of my neck around 2 years ago> last week it cam back in the same spot. I have not been able to get to the doctor yet and it has started to go down on its own with no draining. should i be worried that the pus is draining into my system Doctor: Hello,Welcome to HCM,I think it is a small abscess,as it is draining by itself nothing to worry. I would suggest you to undergo lab investigations like CBC, blood culture, FBS,PPBS and routine urine examination.To fasten up the drainage of the abscess you can apply hot fermentation which will soften the abscess and helps in drainage of the pus.I would suggest you to take a course of antibiotics like Tab Bactoclav, 625 mg, twice daily for at least five days to prevent the secondary bacterial infection.Thank you."
},
{
"id": 84058,
"tgt": "What are the uses and side effects of Etova- ER 600?",
"src": "Patient: My mother, who is 82, has been advised to take Etova - ER 600. She suffers from acute pain in her legs and is unable to sleep at night. She is also under medication for BP, which is under control. Could you please tell me the effect and purpose of Etova - ER 600? Rajendran Doctor: Hi,Etova-ER is a non-steroidal anti-inflammatory drug commonly prescribed to treat painful conditions like arthritis, muscle pain, and toothache. Its common side effects include vomiting, heartburn, indigestion, stomach acidity, stomach ulcer and bleeding. It may also cause dizziness, drowsiness or visual disturbances.Hope I have answered your question. Let me know if I can assist you further. Regards, Dr. Mohammed Taher Ali, General & Family Physician"
},
{
"id": 180163,
"tgt": "Is insertion of rectal suppository into vagina harmful?",
"src": "Patient: I inserted a rectal suppository into my daughter s vagina by mistake..She screamed and then it fell down. But then a transparent fluid came out of her vagina. Did I harm her? Do you think she is still a virgin? She is four years old. She has been complaining that it is hurting her down there. She didn t bleed though Doctor: Perhaps you have ruptured her hymen.Fluid and pain suggest that. Apply some antibiotic cream to prevent infection.Accidental rupture of hymen as in sports or other injuries is not related to sex hence person still remains a virgin. two things are not synonymous."
},
{
"id": 145086,
"tgt": "Suggest treatment for bilateral posterior osteophytes",
"src": "Patient: I have had a c6 corpectomy with cage spacer. Now at C4/C5 I have developed posterior bilateral osteophytes which have mass effect on the left and right nerve roots. At this time there are no symptoms in my right arm but my left gets pretty bad. What is the likely hood of needing more surgery? andrew. Doctor: Hi and thanks for using HCM,We understand your concern you should know that surgery depends on your neurological examination and symptom alleviation from treatment.If you are experiencing pain or numbness treatment management like painkillers and physiotherapy are recommended. If there is no symptom alleviation than surgery is an option to take in consideration.Please feel free for any other information."
},
{
"id": 58682,
"tgt": "Severe abdominal pain. Blood test shows raised bilirubin levels. Causes and treatment?",
"src": "Patient: Thanks Dr ,i was having severe abdominal pain .Atfer test for blood we got raised level of s. bilirubin .indirect -3.02mg/dl ,direct -1.52mg/dl ,total 4.54mg/dl and total wbc count 0000/cumm sir i am strictly non drinker then what could be caused this ,what is treatment for normal level of bilirubin ? please answer. Doctor: hi....alkohlism is not the only cause of derange liver function.....this high billirubin level suggest hepato billiary pathology.....indirect billirubin indicates obstruction of biliay system by stone,worm,tumor etc....so further evaluation is needed...."
},
{
"id": 50383,
"tgt": "Non obstructive stone in kidney. Is it fatty tumor? Consider lithotripsy?",
"src": "Patient: 57 female, good health, no history of kidney stones . I was diagnosed with 9mm non obstructive stone in right kidney by ultrasound. Urologist wants a CT bc he states ultrasounds aren t definitive and it cound be a fatty tumor . Also he will only consider lithotripsy as treatment if stone confirmed. Is it reasonble to request that the stone be surgically removed instead to insure nothing is left behind. Thank you. Doctor: Hi, many thanks for the query!You need to do- RFT, Urine (R/M), Random blood sugar level, USG KUB.If its a Ca-oxalate (most common variety) stone, it will not break by ESWL.9 mm stone may pass out with medicines.Till your reports come, take antispasmodics, diuretics withyour Urologist's opinion.Drink plenty of water so that at least 2 litres of urine is voided in 24 hrs.If diabetic, blood sugar levels must be in good control. Wish you a good health. Take care. Regards."
},
{
"id": 194410,
"tgt": "What causes white flaky patches on penis, calf muscles and forearms?",
"src": "Patient: HI I have some white patches on my penis.It is very itchy and when I rub ,it comes out like flakes(like large dandruff) but after that the area becomes red and starts burning.If water drops on it ,then it burns like an acid drop.The patches are even found in my calf muscle,forearms and sometimes in head (very minor) I am not sure...this is fungal or somethin else...Please advise Doctor: Hi, It may be scaly dermatosis most probably psoriasis vulgaris. It is an autoimmune skin condition of unknown cause. Stress may be the factor to exacerbate the condition. Consult the dermatologist and get thorough examination and investigations to get perfect management. A keratolytic ointment containing salicylic acid may be applied on the scaly patches on extremities and mild steroid cream on penile skin may be considered. There are many oral treatment modalities also. Coal tar-containing shampoo may be used to cleanse the scalp. All the treatment should be taken under observation of the dermatologist. Hope I have answered your query. Let me know if I can assist you further. Regards, Dr. Ilyas Patel, Dermatologist"
},
{
"id": 180370,
"tgt": "How long does Clotrimazole take to cure oral thrush?",
"src": "Patient: I am taking clotrimazole, mouth thrush, use steriod inhaler, the cause of it I believe, i't s been 2 days, every 5 hours, it takes a while i guess, some relief, how long should this go on till fungus is gone completely? I have no problem with regime, was given 35 tablets, which is. 7 day at 5 a day Doctor: Hi, Clotrimazole is effective in the treatment of oral thrush and it looks like you are advised lozenges for the same. As you take steroid inhaler, yes, it is a common cause of oral thrush. It can take a week for oral thrush to subside, however, it depends on the causative factor too if it is present or eliminated. So my suggestion is that after the use of inhaler gargle thoroughly. Eat more of probiotics like yogurt and garlic. Drink plenty of water. Use Nystatin mouthwash gargles. Clean your tongue daily using a tongue scraper. Hope I have answered your query. Let me know if I can assist you further. Regards, Dr. Honey Arora, Dentist"
},
{
"id": 159816,
"tgt": "gamma rays scan",
"src": "Patient: i have been working as a gamma rays scanner for the last 9 months, i work on it for a total of 4-6 hrs per day, i am worried if i am prone to cancer due to working on gamma rays scanner. pl. advice. Doctor: Hi. To begin with i would like to tell you that even i work in nuclear medicine, about 8 hours a day for the past 5 years. I am doing just fine. :). I known people who worked for over 40 years. Coming to scientific explanation, you are exposed to radiation because of your occupation. (Occupational radiation exposure). You must be provided with TLD badge, which is changed every 3 months. You must also be taught the safe handling of radioactivity. If all the safety precautions are followed, people exposed to radiation during their occupation, dont have increased risk of cancer compared to others. You must also remember that almost everyone in this world is exposed to radiation (natural background radiation) which is in the scale of 3-5 mSv. Because of your occupation, you might receive an additional 1 to 1.5 mSv per year which is not dangerous."
},
{
"id": 10009,
"tgt": "How can severe hair loss after hair smoothening treatment be managed?",
"src": "Patient: Severe hair fall.. Hi I m 30yr/f.i had hair smoothening in my hairs 4-5 months back.i m having severe hairfall since then.i m having keraglo Eva tablet once daily n foliglo shampoo twice a week since 1 month.i m not getting any change.plz suggest me some treatment.plz Doctor: HiThank you for asking HCMI have gone through your query. Blood count should be done to rule out anemia and thyroid profile to rule out hypothyroidism as these can be the cause of hair fall also. Medicated shampoos like salicylic acid with ketoconazole will be helpful. Minoxidil can be used also.Thanks."
},
{
"id": 173382,
"tgt": "Is cefi with novamax recommended for fever, cough and loose motion?",
"src": "Patient: My son 15 months old has running fever, cough and loose stomach for last two days, we have given Meftal along with CEFI now doctor changed the CEFI with Novamox 30ml, please advise if it is safe and will be effective as earlier combination of meftal & CEFI not worked Doctor: Hi...Thank you for consulting in Health Care magic.Fever of few days without any localizing signs could as well a viral illness. Usually rather than fever, what is more important is the activity of the child, in between 2 fever episodes on the same day.If the kid is active and playing around when there is no fever, it is probably viral illness and it doesn't require antibiotics at all. Once viral fever comes it will there for 4-7 days. So do not worry about duration if the kid is active.Paracetamol can be given in the dose of 15mg/kg/dose (maximum ceiling dose of 500mg) every 4-6th hourly that too only if fever is more than 100F. I suggest not using combination medicines for fever, especially with Paracetamol.Meftal-P contains Mefenamic acid which is a NSAID. It carries the risk of gastritis and renal problems in the long run. Please do not use Meftal-P. Hope my answer was helpful for you. I am happy to help any time. Further clarifications and consultations on Health care magic are welcome. If you do not have any clarifications, you can close the discussion and rate the answer. Wish your kid good health.Dr. Sumanth MBBS., DCH., DNB (Paed).,"
},
{
"id": 42604,
"tgt": "Suggest treatment for infertility",
"src": "Patient: hai sir i am mallika i have one child of 8 years and i got abortioned two years in the month of 5th starting. after that i have undergone laproscopy surgery for endometriosis . as per doctors advise letrozel tablets for 3 months is also over. and all lab tests are normal including my husband. recently consulted another doctor she suggested IUI and i have undergone. doctor prescribed miprogen-400 capsules daily one for 15 days. kindly advise me in this regard. thank you Doctor: Haiwelcome to hcm you can try iui for 6 months in order to get good results from iui. You can take mi progen tab to support your luteal phase .In stimulated cycles progesterone formation is less compared to normal menstrual cycle. So take progesterone continuously .Regards Dr .Vanitha devi."
},
{
"id": 134669,
"tgt": "What causes hip, back and ribs pain with headache after falling down?",
"src": "Patient: Yesterday I slipped and fell pretty hard on my butt and back. I immediately got a headache, a little dazed and was a little achey but took ibuprofen and went about my day. Today my right upper hip hurts and up the right side of my back along my ribs up to my shoulder blade. Also a slight headache as well. Is this something that can be taken care of with Otc meds and time or do I need to Go get it checked out. I m not one to run to the dr over every ache and illness but it was a pretty hard fall. Doctor: Hello ,Thanks for your query and welcome to Healthcare magic.In your case it appears that there is a soft tissue injury along with back strain and since you have been able to walk and ambulate after the injury, there definitely is no bony injury.So in your case my advise will be for ice pack application 3-4 times in a day on painful areas and areas with swelling.Also pain medications like Tab Ibuprofen, Tab Paracetamol or even Diclofenac if it is available OTC can be used on SOS basis.The cause of concern will be there if your symptoms persists after 2-3 weeks because this is the time taken by soft tissues to heal normally.Kindly get back to me should you need any further information/SOSThanks & Regards,Dr Akshay Kumar Saxena"
},
{
"id": 76713,
"tgt": "What causes no respiration in COPD patient after placing oxygen mask?",
"src": "Patient: A patient was admitted to the hospital with chronic obstructive pulmonary disease. His PO2 was 55 and PCO2 was 65. A new resident orders 54% oxygen via the venturi mask. One hour later, after the oxygen was placed, the nurse finds the patient with no respiration or pulse. She calls for a Code Blue and begins CPR. Explain why the patient stopped breathing. Doctor: Thanks for your question on Healthcare Magic. I can understand your concern. COPD patients are used to have low oxygen saturation as compare to normal individual. This is due to their poor lung functions.Brain is having respiratory center which controls breathing. This center is under influence of oxygen and carbon dioxide levels of body. In COPD patients, this respiratory drive is maintained through low oxygen and high carbon dioxide level.. So when COPD patients receive high oxygen, this respiratory drive is demolished. So their respiration will gradually stop. This is happened to your patient because he was given high flow oxygen and because of underlying COPD, this high oxygen diminishes his respiratory drive with no respiration. Hope I have solved your query. I will be happy to help you further. Thanks."
},
{
"id": 86612,
"tgt": "Suggest treatment for severe pain in the abdominal region",
"src": "Patient: I went to the emergency room last night with severe pains on the right side of my back and stomach. The doctor said I was constipated and gave my a prescription for Gavilyte. After almost finishing, I threw most of it up and now I am hurting worse than before. Should I go back to the doctor or does it take some time? Doctor: hello..do you have any addictions?age?did he advise any antispasmodic(pain medicine)..?take that for pain relief if he dint, take a buscopan for pain relief.occasionally nausea and vomiting are side effects of Gavilyte..nothing to worry if it happens for once..if the problem persists visit your doctor..take plenty of water..hope my answer helps youif you have any more queries i would be happy to answer.."
},
{
"id": 211468,
"tgt": "Emotional numbness. Feverish symptoms, nausea and rapid heartbeat. No relief even after taking antibiotics",
"src": "Patient: For about 7 or 8 days now , I ve been feeling completely empty and just pure emotional numbness , I can t seem to get excited or passionate about anything anymore and haven t felt myself for over a week , it s very frustrating and I really don t want to feel like this anymore , I miss having feelings and being able to think about things and go out and socialize with my friends and my natural excitement and enthusiasm about things. I went to the hospital , because the first few days I had really bad feverish symptoms and felt horrible and nauseous and a really rapid heartbeat, they said they found blood and a weird type of protein in my urine that spread and may of possibly caused an infection, so they put me on a drip and injected an anti sickness drug into my body and then gave me a course of anti biotics. But I ve been taking these anti biotics and I still have the complete same emptiness and feeling disconnected. Doctor: DearWe understand your concernsI went through your description. I suggest you not to worry too much. I don't understand why thinks negatively where such thinking is not at all advisable. Everyone goes through periods of illness. Everyone feels as you. Emotional numbness or some sort of uneasy feelings. Body and mind is related. If body is ill mind also feels the brunt and it is applicable in reverse also. Thinking also needs energy and body wants to conserve energy to have more effective work inside the body.Understand the body chemistry and it answers your questions. Worrying increases the worry itself. You will be cured for sure. Your doctors are doing their best. Also god is with you. God cannot leave you. Be happy.You might need psychological counseling., I am more than happy to provide telephone psychological counseling through healthcaremagic. Contact me through customer care, www.healthcaremagic.com.Hope this answers your query. Available for further clarifications.Good luck."
},
{
"id": 16565,
"tgt": "What causes heart palpitations while suffering from insomnia?",
"src": "Patient: Hi. I was unable to sleep most of the night last night. My heart seemed to be beating hard and quickly. I have been suffering from insomnia over the last week or two. When I did wake up this morning from a couple hours of sleep my heart felt like it was pounding out of my chest and my right arm was numb and tingling, then it moved to both arms. I feel extremely weak too. This is the second time this has happened to me in the last 6 months. I ate an egg and an apple hoping some protein and carbs would help. My arms are feeling somewhat better after 45 minutes but heart is still rapid and pounding. Heartbeat is around 100 beats per minute. Doctor: Hello ThereAfter going through your medical query I understand your concern and I would like to tell you that rapid heart rate can be because of restlessness which is why you are facing insomnia as well, an ECG is recommended for you and take an anxiolytic and sleep well .Hopefully this information will guide you properly.Kind Regards Dr Bhanu Partap"
},
{
"id": 21957,
"tgt": "What causes chest tightness and breathing difficulty?",
"src": "Patient: I'm a 30 year old female. Chest tightness and slight difficulty breathing. Happens during the day and night. I've been also having dull to sharp nagging pain in my right lower back rib area, like in my ribs. What could this be? Medical History: 30 Female, asthma since child, 1 pack cigarettes per day, no alcohol, 99 pounds, 5'2\", Doctor: Hello Ma'am Am sorry to say this but you doing total injustice to yourself, as you are a known asthmatic since childhood and above that you are smoking that to 10 cigs a day. You must be knowing that cig contains high amount of nicotene and nicotene causes constriction of your airways that leads to chest tightness and pain. It can also lead to acute exacerbation of asthma that is a medical emergency. My advise is to cut down the smoking and finally stop it.Good Luck"
},
{
"id": 67984,
"tgt": "How to get rid of lump on the side of the feet?",
"src": "Patient: On the side of my right foot I have a bump /lump that I ve had for the past few years. I recently had a kidney transplant in April . The lump does not hurt to stand on it . It is sensitive when i roll my foot on a baseball ( for example ) . My pedicure lady has recently started questioning me about it. I m wondering if I should go to my foot doc. I am a type 1 diabetic also Doctor: Hi, dear. I have gone through your question. I can understand your concern. You may have some soft tissue tumor like lipoma or neurofibroma or may have some cystic lesion. You should go for fine needle aspiration cytology or excision biopsy of that lump. Go for that and then take treatment according to your diagnosis. Hope I have answered your question, if you have doubt then I will be happy to answer. Thanks for using health care magic. Wish you a very good health."
},
{
"id": 211204,
"tgt": "How to overcome my depression and get my appetite?",
"src": "Patient: hi..my name is kiran.and am a male patient of 21 years..i have been suffering from depression from last one year..i am on mirtazepine 30 mg untill last month now on 45 mg..but still i dont have appetite..but there is an inmprovement ..from last year..when ever am eating wheat products ..caffiene my mind is getting negative thoughts..and appetite is a major problem for me..am 60 kgs..plz help me when i will get my appetite and out of this depreesion Doctor: hithanks for your queryResponse depend upon improvement with drugs and its vary from patient to patient. Exactly no one say in how many days u will become absolutely fine, but definitely u will get improvement with drugs. If u r not getting sufficient improvement with mirtazapine, that means u need one more antidepressant in ur treatment. Discuss this with ur psychiatrist.Thanks"
},
{
"id": 4292,
"tgt": "Trying to conceive and started elevit. Period stopped, had spotting but pregnancy teat negative",
"src": "Patient: Hi. I'm not on any contraception at the moment but i had one shot of the depo when my son was 4 wks old now he is 20mnths old and i was not on anything else since then. My period only came back May last year and has been normal since then. Now i'm wanting another baby and started on elevit last month and it seem like my period is now stopped. Didn't come at all. My last one was 18/01/2014 then i got brownish spotting on the 1st of Feb which i believed it was ovulation and nothing else since then..I took 2 urine test so far and results came bck negative. Can u tell me if there is a hope for me! Doctor: Dear member,Thanks for writing to healthcare magic.If your pregnancy test is negative then you can consult a doctor to induce a period.Don't worry.As you have had a baby and your cycles are regular there is no need to worry.Try home ovulation detection kits and time intercourse. If after 1 year of this there is no pregnancy then you can consult for further investigation and treatment.Dr Bhagyashree."
},
{
"id": 59093,
"tgt": "Hepatomegaly with grade 2 fatty infiltration, high SGOT and SGPT levels. Need treatment?",
"src": "Patient: Dear DoctorI have hepatomegaly with grade II fatty infilteration, 15.8 cm span of liver showing increased parenchymal echotexture suggestive of fatty changes, SGPT (ALT) 79 and SGOT(AST) 45 as per ultrasound and blodd test. Kindly tell how serious is this condition and treatment/lifestyle change etc. I am a non alcoholic vegetarian men of 44 year. Doctor: Hello! Thank you for the query. Fatty liver becomes quite common disease in modern society. This condition is caused by liver cells degeneration and liver overload by the fat. Fatty liver can be caused by alcohol intake, severe medicines intake or due to bad diet (fatty foods). Luckily it is reversible condition and it is possible to regenerate the liver. However, fatty liver can lead to liver cirrhosis which is not reversible. I suggest you to change your dietetic habits. Avoid fatty foods, spicy foods, soft drinks, alcohol. Increase fiber rich products, vegetables, fishes intake. You should also have checked blood level of cholesterol and triglycerides and if increased, statins should be prescribed by your doctor. Ultrasound and liver tests should be repeated in a 3 months or so. Hope this will help. Regards."
},
{
"id": 214428,
"tgt": "Suggest natural way to get rid of pimples and freckles",
"src": "Patient: hi doc, i'm a filipina but already moved in here in u.s. i got a skin problem, i always had a pimples and freckles, is there a natural way to get rid those skin problems, right now i am using proactive just to minimimize the pimples occurence but there's an acne scar left on my face..... Doctor: hello,there is no complete way to cure scars by natural methods.do try washing your face regularly.use a good quality face wash.do not squeeze the acnetry using turmeric containing cream like Vicco Turmeric"
},
{
"id": 158223,
"tgt": "Diagnosed with prostrate cancer, removed testicles in surgery, chemo done as spread to bones. Hb reducing. Any ways to increase it ?",
"src": "Patient: My dad is 70yrs old and was diagnosed with ca of prostate in 2007. Surgery was done to remove the testicals in order to reduce the production of testosterone. From bone biopsy we found that it has spread to bones and chemo was done for 2 cycles out of 3. the last one could not be completed because of severe side effectsHis hb keeps reducing every 3-4 months and he requires transfusion. Is there a way to avoid this and increase the hb naturally? He eats a very healthy diet rick in iron but effect of Hb is slow Doctor: We Use in our patients Erythropoitin 10000 units every 2 weeks. You can consult your doctor who can guide you the form and route of administration.Truly,Dr. J. Ticku"
},
{
"id": 220775,
"tgt": "Suggest remedy for numbness in back during pregnancy",
"src": "Patient: I am 18 weeks pregnant and I have been having a wide spread feeling of numbness that started in my lower back about 2 weeks ago now it has spread down both legs and is starting to go into my left arm I had an MRI come back normal but this doesn t feel normal this is my 4 th pregnancy Doctor: Hi, Dr Purushottam Neurgaonkar here. I welcome you to HCM VIRTUAL CLINIC. I have gone through your question, and I think I have understood your concern. I will suggest you the best possible treatment options. I will suggest use of calcium supplements and via min D 3 along with good quality protein. Many times as pregnancy advances deficiency of these items and changes of posture during the pregnancy lead to nerve Compression and numbness.I will suggest to adopt correct posture . This will help to minimise numbness .I hope this answer helps you. Thanks Dr Purushottam Neurgaonkar."
},
{
"id": 161588,
"tgt": "What is the treatment for vaginal bumps in a child?",
"src": "Patient: My 4 year old daughter has a small bumps on her vagina and about 4 larger ones on the cheeks of her buttocks that have opened and bled and then looked really dry and went away. They do not seem to bother her unless I try to put some sort of cream on them. What is this and am I doing something wrong? Doctor: Hello, Skin conditions are best diagnosed only after seeing directly. I suggest you to upload photographs of the same on this website so that I can guide you scientifically. Please revert back to me with images so that I can guide you better. Hope I have answered your query. Let me know if I can assist you further. Take care Regards, Dr Sumanth Amperayani, Pediatrician, Pulmonology"
},
{
"id": 26094,
"tgt": "Will Losarton 25 mg control my blood pressure?",
"src": "Patient: I am 45 years old . My BP used to be normal but due to some financial problem ,I thinks a lot and reason thet my BP reads 150/100 , Today i went to the physician she prescribed me losarton 25 mg once a day .Will my BP will reduce to the normal reading . I also dont have sound sleep. Suggest me what to do/ Doctor: Hello!Thank you for asking on HCM!I read carefully your question and would explain that this dose of losartan is a really low dose and I don't think that it will bring your blood pressure to the normal values. But I think your doctor has prescribed this dose, in order to start with a low dose and increase it slowly until your blood pressure reaches the normal values. You should know that every modification to your antihypertensive therapy (even starting) needs a certain period of time (up to a week) to show its effects on your blood pressure. So you should be patient, closely monitor your blood pressure different times during the day for the first week on this dose and refer those values to your doctor, in order to make the proper changes to your actual therapy (increase the dose and/or ad other antihypertensive drugs). Hope to have been helpful!Feel free to ask me directly at any time whenever you need!Greetings!Dr. Iliri"
},
{
"id": 105344,
"tgt": "Allergy like irritation. Taken R-cinex. High reading of urine and creatinine. What is the problem?",
"src": "Patient: My dad is suffering from serious allergy like irritation..He is taking R-cinex tablets for about 15 days...He is also taking treatment for kidney problems like high readings of urea , creatinine in both english and also in siddha medicine..Now he s suffering from serious allergy for the past week..What is his problem?Please tell me..I am totally depressed.. Doctor: if the allergy occured after taking the medicines it might be due to medicines and mismatching of salts because you are taking allopathic and sidhi medicines because of high creatinine and kidney problem we cant give medicine high dose of or even low dose of medicines because of kidneys malfunction urea and creatnine itself can cause allergy just apply local topical ointments like histocalamine mometasone and petrolleum jelly if skin irritates because you have not specified particularly the skin if any other form of allergies you can take tab piriton and try to treat route cause for which you are giving rcinex and stop sihi medicine"
},
{
"id": 56408,
"tgt": "What does hepatic metastatic disease suggest?",
"src": "Patient: After a CT Scan of chest and abdomen- several masses were found in my Mother s lungs as large as 5.5 x4.6 x 6.6 as well as in the liver. The report read wide spread hepatic metastic disease. A liver biopsy was done 3 samples were taken and the results were inconclusive. Now a biopsy of the lungs is scheduled. I was told the biopsy of the liver would tell me everything I needed to know. It was never explained to us that there was a possibility of it being inconclusive. The Dr. that delivered the results was the partner of my mother s regular pulmonologist. We have no rapport with him and he has no bed side manner, didn t answer any questions or even make eye contact with us when he spoke. We will now have to wait until Tuesday for her regular Dr. to return to have theses answers. Can you shed any light on this? Doctor: Good morning I am sorry to hear about her ill health Masses in the lung and liver and due to cancer in almost all cases and I regret to say that I seriously suspect this in your mother For this to be treated, we need to confirm it by doing a biopsy from the mass lesion either from the liver or lung or sometimes from both organs. After we confirm that this is cancer, then its very important to note the origin of this cancer because choice of medicines varyThe common sites of origin are lung, breast, stomach and colon. So the final answer is from the pathology lab for an cancer diagnosis and in this process, unfortunately some patients need to undergo multiple biopsy proceduresHope this would answer your questionWish her a speedy recovery"
},
{
"id": 110422,
"tgt": "What does pinch feeling in back indicate?",
"src": "Patient: i got kicked in the kidney in soccer about two years ago and recently i been feeling a pinch in my back where the kidney is located and ive been super tired and out of breath , also i have to urinate a whole lot more, and i've missed my period. whats wrong with me? Doctor: Hello ,Thank you for writing to HCM,Brief answer -A pinched nerve occurs when there is \"compression\" (pressure) on a nerve.Explained answer -Nerves extend from your brain and spinal cord, sending important messages throughout your body. If you have a pinched nerve (nerve compression) your body may send you warning signals such as pain. Don't ignore these warning signals.Damage from a pinched nerve may be minor or severe. It may cause temporary or long-lasting problems. The earlier you get a diagnosis and treatment for nerve compression, the more quickly you'll find relief.In some cases, you can't reverse the damage from a pinched nerve. But treatment usually relieves pain and other symptoms.Hope You will find it out helpful.Thanks And Regards,Dr. Gaurav Prajapati"
},
{
"id": 85777,
"tgt": "Can Risperidone cause nerve pain in neck?",
"src": "Patient: I recently developed a neralgia (sic) in my neck/ jaw which is being treated with gabapentin. I have been prescribed risperidone for a few months. I havent taken he risperidone for 3 days and the pain in my jaw seems to have subsided, could risperidone have caused the nerve pain in my neck/jaw Doctor: Hello, Risperidone does not cause nerve pain as a side effects based on evidence studies. So, in my opinion the nerve pain is not related to Risperidone. In my opinion, it can be a pinched nerve in the neck. I suggest using anti inflammatory medications such as Acetaminophen to relieve the symptoms. Hope my answer was helpful. If you have further queries feel free to contact me again. Regards, Dr. Dorina Gurabardhi, General & Family Physician"
},
{
"id": 209774,
"tgt": "What makes me feel that the world is a cheater and a liar?",
"src": "Patient: my name is gnknar. i need help becoz i dont know what happing with me. i like changes but not in my things, every morning i think '' today i do something in my study but its not happening ''.i feel something bad . i think every people is liear in this world all r cheater . so i cant belive in any people Doctor: HiThanks for using healthcare magicI think, you are in depression. You have paranoid ideas that are part of depression. In that case, you should consult a psychiatrist for proper diagnosis. if there is no underline psychotic symptoms in you, then you can take antidepressant and you would improve with medication. In case, you need further help, you can ask.Thanks"
},
{
"id": 148107,
"tgt": "Lumbar region vertebra impinging upon thecal sac, moderate spinal stenosis, degenerative disease, L/2 vertebral body hemangioma",
"src": "Patient: L1/2 disc bulge which impinges upon the thecal sacL2/3and L3/4 disc bulges causing small ventral impressions upon the thecal sac and partially compromising the adjacent neural foramina bilaterally. ModerateL2/3 spinal stenosis. L 4/5 and L5/S1 disc bulges which impinge upon thecal sac. grade II spondylolisthesis involving L5 on S1 with associated severe bilateral L5/S1 neural foraminal stenosis. The L/5 nerve roots are compressed within the L5/S1 neural foramina bilaterally.minimal scoliosis. Degenerative disease. L/2 vertebral body hemangioma. Doctor: you have mentioned about reports but no symptoms. Are you having back pain and rediculopathy or severe pain on standing. Than I would advice you to take opinion from orthopaedic surgeon. For the time being take pregabalin and methylcobal tablets. Also wear lumbosacral belt."
},
{
"id": 19847,
"tgt": "What causes elevated BP, tachycardia with discomfort in chest?",
"src": "Patient: Hi, I have elevated blood pressure, tachycardia when standing, squeezing sensation in my chest around left shoulder and neck and a chronic cough that increases with activity. I have also noticed some bright colored blood in my sputum. I have a clean chest X-ray, clean echocardiogram, normal ECG and a recent CAT-scan of my aorta is also non-alarming. The chest discomfort intensifies at rest and is not constant. Additional symptoms: Headache, enlarged blood vessels in my eyes, intermittent kidney discomfort, bluish fingernails, intermittent tingling of left hand fingers and feet. Doctor: Hypertension is alifestyle disease which needs a lifestyle solution.Please rule out secondary causes of hypertension especially Pheo chromo cytomapleaase doa TMT to rule out IHD.You have not mentioned about your work stress and home stress Your occupation ,whether you are a smokerwas Vitamin D , Magnesium and HS CRP TEST DONE and their results .A more detailed hsitory required to give more specific advice"
},
{
"id": 169419,
"tgt": "Suggest remedy for nasal congestion",
"src": "Patient: Hi Doctor,my baby is 11weeks old. She has flu like symtoms. Nasal Congestion and phelm build up in her chest. I have tried saline drops to relieve the congestion and bisolvent to loosen the phelm. Her breathing is very heavy especially after a feed and at night. She doesn t have a fever and is having her feeds despite her difficulty breathing. However, now she has a slight wheeze,I can feel and hear it, whilst burping her. How can I rid my baby of the phelm. I have her on a course of anti-biotics for the past 3 days. She is passing good stool and has over 6 wet diapers a day,but the difficulty breathing is frustrating her. I have her on lactogen formula and breast. She also has at least 100ml water a day. I ve been forced to delay her 10week vacines because of her cold. Please help me,help my baby. Regards, Pam Reddy. Doctor: Hello. Welcome to HCM. 1st of all its not good to give a baby water until she starts solid foods, because you risk Water Intoxication. At your baby's age only saline drops can be given for relieving her congestion. Just drip 2 drops of saline in every nose opening and try to suck the secretions with a baby nose suction. For night sleeping try elevating her mattress a little bit so the p helm will not disturb her. Green Stools may be a side effect of the antibiotics."
},
{
"id": 192363,
"tgt": "What is the safe time to ejaculate after testicular surgery?",
"src": "Patient: I received surgery for testicular torsion about 4 days ago. I have stitches on both testicles; my right one was torted but my left one was operated on also for preventative measures. I am wondering how long I have to wait before it is safe to ejaculate, whether by masturbation or by sexual activity. Thank you. Doctor: Hello, After surgery 1 -2 weeks you have to wait . Even after that you have show the lesion to your surgeon if it is healed then only you may ejaculate. Hope I have answered your query. Let me know if I can assist you further. Take care Regards, Dr. Penchila Prasad Kandikattu"
},
{
"id": 176944,
"tgt": "What causes fever along with dry cough and sore throat in a child?",
"src": "Patient: Hi. My 6 year old daughter has fever last night. She has running nose and with dry cough. She chills and complains of sore throat. It started with fever. She is not picking her nose. She is just blowing her nose normally. Please advise. Maria Doctor: the symptoms suggest that your child has got viral infection. Treatment is symptomatic in form of anti inflammatory like Paracetamol, antihistaminic for running nose like Cetrizine and a cough syrup. Hydrate the child well, give her good amount of fluids. She should do fine in a couple of days. Viral fever usually subsides in 3-7 days."
},
{
"id": 190840,
"tgt": "What is the cause and remedy for redness and pain over uvula and palatoglossal arch on eating ?",
"src": "Patient: hi my uvula and palatoglossal arch hurts and its red. Everytime i eat and food passes by i hurts like a mother! what is the problem? and how can i treat it? thank you Doctor: Hi, There might be ulcer on the uvula and palatoglossal arch. thats why it is hurting you when you eat some thing or food passes through it. I advice you to take healthy and nutritious diet, take vitamin A & C supplements, avoid smoking stress and anxiety. If the situation persist please visit to the doctor. Thanks, Dr. Vishal"
},
{
"id": 36379,
"tgt": "Could scabies be infected even after using hibiclens?",
"src": "Patient: Hello, I have recently found out my brother's girlfriend has scabies, and has been in and around our house. I myself have been using the hibiclens soap for a while due to my own issue lately. But my question is, with the use of hibiclens, could I still have gotten scabies? Doctor: HelloWelcome to HCMThanks for your query.Hibiclens Soap is a topical antiseptic antimicrobial used in topical treatment of skin infections including Pseudomonas aeruginosa, Staphylococcus aureus, Methicillin Resistant Staphylococcus Aureus. It is a powerful antiseptic antimicrobial topical soap, it contains 4% Hibitane, which is a combination of chlorhexidine gluconate.So,it acts against bacterias not against parasites.Scabies is a contagious skin infection caused by the mite Sarcoptes scabiei.So.there are chances of you getting scabies.Thank you"
},
{
"id": 13805,
"tgt": "What causes bruises to reappear?",
"src": "Patient: I have a lot of bruises on my thighs all over back front sides n one side of my rear end .. I have also a rash of Tiny little bumps where ever I have bruises at . I have night sweats always feeling tired out of breath alot of times in not diabetic nor anemic what cld it be? N bruises start off small as a quarter then become bigger Doctor: Hi, Detailed evaluation is required to find out what exactly causing these bruises. Vasculitis and associated diseases can present like this. You can consult a dermatologist and get evaluated. Hope I have answered your query. Let me know if I can assist you further."
},
{
"id": 198277,
"tgt": "Will masturbation cause problems in gaining weight?",
"src": "Patient: hello sir. im 20 yrs old man.i started gym 2 weeks ago and i want to put mass on me. m talking multivitamin , post workout and tribuls terestris. and i also mastubate after two days. i jus wanted to know if masturbate plays role in big muscle? does masturbate cann not let yu gain lean mass?? is this true? Doctor: DearWe understand your concernsI went through your details. Masturbation do not cause any health problems because it is natural and normal. But you should keep in mind that masturbation do consumes energy. Frequent masturbation can cause drain in energy which may be problematic for your gymnasium exercises. Except this problem, masturbation cannot cause any hurdles in developing your muscles. Go ahead. Restrict masturbation to those days on which you don't go to gymnasium.If you require more of my help in this aspect, please use this URL. http://goo.gl/aYW2pR. Make sure that you include every minute detail possible. Hope this answers your query. Available for further clarifications.Good luck. Take care."
},
{
"id": 45672,
"tgt": "Suggest treatment for fever, dark urine and lower back pain",
"src": "Patient: Light periods lasting 2 days, bloated stomach. Feverish occasionally and dark urine sometimes. Middle back ache and had pain on right side kidney area that went away after taking antibiotics for water infection eve though no pain on urinating and no frequent urinating. Kidney pain seems to be returning and feaverish again Doctor: Hello, I got your history and it us due to persistent urinary tract infection. Though you have taken medications but infection has not cleared. You require urine culture and after that proper and selective antibiotic can be tried.There is nothing to worry ,report to emergency room for examination and tests so that drug therapy can be initiated. Hope I have answered your query. Let me know if I can assist you further. Regards, Dr. Soheel Hussain Zargar, Dentist"
},
{
"id": 29451,
"tgt": "What causes recurrent yeast infection?",
"src": "Patient: Hi, I just got over a yeast infection about two weeks ago. It came back a few days ago, and I ve started another 3 day over the counter treatment. I am positive that it is a yeast infection because I ve had the symptoms before. Why would the yeast infection return? Is it possible to get BV from the treatment? Doctor: Hello, yeast infection can recur due to several reasons. If you havnt taken the drug in correct dose and for the correct time duration infection recur within short period of time. Sometimes if you have diabetes also. You would get recurrent yeast infections. If you havnt checked your fast blood sugar with your 1st yeast infection episode you'd better this time. Yeast infection can be spread by sexual contact. All your sexual partners/ partner has to be examined an treated simultaneously. Other wise even if you were cured you would get the reinfection from sexual contact. Yeast infection sensitive to ketoconazole, meconazole, cotrimoxozole. They are available as creams, vaginal dosits. And in oral forms. Hope this helps. Thank you."
},
{
"id": 62999,
"tgt": "Suggest treatment for an abscess in the operated area",
"src": "Patient: MY FATHER IS 57 YRS OLD HE MET WITH THE ACCIDENT AND PARCIAL HIP REPLACEMENT CEMENTED OF THE LEFT WAS DONE IN THE MONTH OF MARCH 2010 OPERATION WAS FINE HE RECOVERED BUT IN THE MID OG JULY HE GOT INFECTED AND ABSUS WAS SEEN ON THE STICHES,DOCTOR NICKED THE ABSUS AND AFTER THAT TILL DEC HE GOT NEW ABSUS ONE AFTER THE OTHER,NOW HE IS NOT GEETING THE NEW ONE BUT STILL HEALING,FROM80KGS HE HAS FALLEN TO 44KG BECAME VER WEAK.HE IS UNDER THE MEDICAL TREATMENT WE CULTURED THE PUS SOME BACTERIAL INFECTIONS AS THE HOSPITAL WAS NOT CLEAN & UNHYGIENIC.I WANT TO KNOW ARE THERE THE CASES WHICH TAKES TIME TO HEAL,AFTER THE DOCTOR GAVE HIM TOXIM 200 TBLET Doctor: hi.is your father diabetic, immunocompromised or nutritionally challenged? i asked because these factors are very important in his wound healing.. if there are underlying medical conditions other than his wound, it could slow down his healing process. consultation and regular follow-up with your doctor is best. daily wound cleaning for the abscess and the surrounding area is also important, along with the proper antibiotic regimen.hope this helps.good day!!~dr.kaye"
},
{
"id": 105958,
"tgt": "I am troubled by cough and cold, please suggest",
"src": "Patient: hallo sir i am ram verma my wife having allergy.she is troubled by cough and cold.so what can i do. Doctor: Dear Mr.Ram Verma, If your wife is having cough and cold very often,then it is due to some allergy,either food or pollution.I suggest you give her the following medication:- 1]Tab.Levocet-D 1-0-1 2]cap.Doxycycline 1-0-1 3]Syr.Phensydyl linctus--5ml 3 times. Kindly visit your family physician if there is wheezing along with cold & cough."
},
{
"id": 225915,
"tgt": "Raking Errin birth control. Missed a pill. Will that make a difference?",
"src": "Patient: Hi there,I began taking Errin birth control and am about halfway through my first pack. Yesterday morning I woke up late and realized that I didn't wake up to my alarm to take it. I took it as soon as I realized it just like the directions said to. This morning I woke up and took my regaular pill at the normal time and I then realized that I had woken up yesterday and took a pill but it was a Wednesday pill from two weeks from now. I had taken two pills in one day and now have a missing pill. What should I do? Doctor: Hi, thanks for the query.Taking two pills in a day will not affect the efficacy of pills and you will be equally protected. If you have taken a missed pill within 3 hrs of due time, it will not change anything. But if it was after more than 3 hrs, then you should use backup method(condom) for next 48 hrs.As you have one less pill now, your pack will end a day earlier. In my opinion you should start your next pack very next day.Hope this helps. For any further queries you can directly reach out to me on the following URL:http://doctor.healthcaremagic.com/doctors/dr-deepti-goyal/65111"
},
{
"id": 5983,
"tgt": "Miscarriage irregular period. Can Hyponidd lead to pregnancy ?",
"src": "Patient: HI,I HAD A MISCARRIAGE IN DEC,AND I GOT MY PERIODS LAST MONTH ON 1ST MARCH.tHIS MONTH AGAIN MY DATE GOT DELAYED AND I GOT MY PERIODS ON 8TH MARCH.I TOOK THE ADVICE FROM MY GYNAECOLOGIST , AND SHE HAD JUST SUGGESTED ME TO TAKE HYPONNID-2 TABLETS DAILY MORNING AND EVENING.WILL THIS TABLET HELP ME TO CONCIEVE AS ITS BEEN TWO YEARS I M TRYING TO GET PRGNANT BUT STILL NOT ABLE TO CONCEIVE.PLEASE SUGGEST. Doctor: Hello. Thanks for writing to us. Hyponnid is an ayurvedic preparation which helps in regularizing the periods in case of delayed periods. This can help you in conception upto some extent but it is best to try ovulation inducing drugs like clomiphene after consulting your gynecologist. I hope this information has been both informative and helpful for you. Regards, Dr. Rakhi Tayal drrakhitayal@gmail.com"
},
{
"id": 137661,
"tgt": "How can bruising, skin abrasion and lumps on the buttock be treated?",
"src": "Patient: a week and a half ago i fell 7 feet when my ladder slid from under me i landed on the ladder the doctors said they could not see any signs of broken bones. i had some bad bruising on mt left lower back and buttocks, the bruising is going away but i have quite a few lumps of different sizes, i have lost of feeling to the surface of the skin and it is painful to the slightest pressure. Is this normal of hematoma or is it possible it could be something else? Doctor: Hello,I think you have developed multiple tinny hematomas and these are appearing like bumps. I would suggest you to visit a general surgeon and hot water fomentation. Take antibiotics like cefuroxime and NSAID for five to six days.thanks"
},
{
"id": 96708,
"tgt": "How to cure foot injury after a fall?",
"src": "Patient: Hi, on saturday i tripped over and badly hurt my foot, i already have permenant ligament damage in the foot. i went to see a doctor today and they said i have pulled the surface ligaments. I can't really walk with the painful foot and there is a large amount of brusing around my big toe area and down that bone. So i was wondering if i could have a second opinion? Doctor: Dear friend ,a large amount of bruise tells that you have badly injured that foot.and there is every chance that you have injured the ligament of that foot.because in violent sprain,that happened in this case,ligaments are the worst sufferer.also your ligaments are already in bad shape as you have said.so there is hardly any reason that you should not listen your clinicians advice."
},
{
"id": 205680,
"tgt": "Suggest treatment for adult speech impairment in condition like stroke",
"src": "Patient: My mom has suffered from TIA strokes in the past. She is taking blood pressure medicine as well as a few others: Atorvastatin 20mg, Amlodipine Besylate 5mg, and Levothyroxine 75 mcg. Today she is not showing signs of stroke but can not get her words out. Like her thoughts cant be completed. What do you suggest that I do? Her doctor is an hour away.Thank you, Terri My email is: thamilton206@gmail Doctor: dear,the speech impairment appears secondary to the brain ischemia. Hypertension further adds to the risk.The impairment is generally reversible, however it depends Upon the severity of infarcts and its duration.Cnsutation to neurologist is must here, besides investigations like MRI or NCCT brain is required.Regards"
},
{
"id": 30519,
"tgt": "Can Crocin be given for fever?",
"src": "Patient: Hi sir,My dad M.Balram Reddy,age-55,male had blood count 5 in the month of February 2010.The treatment started on May this year under the supervision of NIRNI S.S of Indo American Cancer Hospital in Hyderabad.He asked us to transfuse blood and doing so the blood count went on to 10, later it gradually got down and currently he has haemoglobin-6.1.He suggested to take tonic .I felt Doctor was checking for the symptoms of blood cancer instead of diagnosing!!From past two days dad is suffering from fever ranging 102-104c and he says he is not hungry and has become weak...Even we consulted an Ayurveda doctor he gave medicines for tuning up the liver. will try sending the scan of the medical tests too.Could you please provide us a suggestion what can be done! Can we give some crocin to the fever to subside? My mail YYYY@YYYY you..:) Doctor: hi sir, welcome to HCG, i understand ur problem, yes sir, crocin can given for fever. but why dont u use dolo 650 tab thrice a day with antibiotic extracef plus200 twice a day after food... maybe ur dad have gastritis also, plz use pantafol dsr once before breakfast... and ur dad heamoglobin is low, use iron sucrose IV inj weekly once, it will increase his blood, and make him stronger.. he need 10 inj weekly once... hope u satisfy with my answer, thank u"
},
{
"id": 26264,
"tgt": "What causes the tingly sensation with high BP?",
"src": "Patient: My blood pressure is currently high and my lips and face are tingly. I also have a headache. I started taking blood pressure medicine yesterday. What is causing the tingly sensation and what can I do to relieve the headache? My address is YYYY@YYYY . Thanks Doctor: Hi,You should take medications for longer period, at least for five days, to see the effect. If you started medication day before, don't worry, you should wait for couple of days. And your complaints can be caused either by high blood pressure or blood pressure fluctuations.Hope I could help youWishing you good healthIn case of further questions don't hesitate to askRegads,"
},
{
"id": 126376,
"tgt": "Suggest treatment for edema in the legs post a knee replacement surgery",
"src": "Patient: My husband is 2 weeks post knee replacement surgery. The surgical site is fine but he has edema in bot( of his lower legs. He just had an ultrasound to rule out a clot and he\u2019s been prescribed Keflex to stem inf3ction from any cellulitis that might develop. Other than elevating his legs above his heart what other actions can we take to help eliminate the edema. His PT has been delayed for a week and we are fearful of delayed rehab causing other issues. Any recommendations or suggestions? Doctor: Hello, Just try to keep legs elevated. active calf pump exercises. Check if he has a fever. Usually after knee surgery oedema occurs due to lack of calf pump. so he needs to do that. learn it from YouTube. Hope I have answered your query. Let me know if I can assist you further. Take care Regards, Dr Anuj Gupta, Spine Surgeon"
},
{
"id": 81400,
"tgt": "What causes rattle in left lung?",
"src": "Patient: I have a rattle in my left lung when I breath in and sometimes when I breath out and sometimes it comes on the right side but mostly on my left, I ve got phlegm as I ve had a cold and when I cough it up most the time it s white but it has been green I ve been on a corse if antibiotics I m due to finish them Tuesday I ve got a cough that comes and goes as well Doctor: Thanks for your question on HCM.In my opinion you should consult pulmonologist and get done chest x ray to rule out chest infection (pneumonia). As rattles (crepitation) in chest is mostly heard due to retained secretions. And secretions are seen in pneumonia. So we need to rule out this.If chest x ray is normal than get done PFT (pulmonary function test) to rule out Bronchitis. As rattles are also heard in bronchitis.Treatment of both are totally different. Pneumonia needs antibiotics and bronchitis needs inhaled bronchodilators.So better to consult pulmonologist and discuss all these."
},
{
"id": 75202,
"tgt": "Does recovery after myasthenic respiratory crisis take long?",
"src": "Patient: My husband experienced a myasthenic respiratory crisis, and 13 days later discharged to rehab but seems as weak or weaker than ever. He is able to sit himself up but not rise to a standing position alone. Is this a normal for recovering from such a serious crisis? Doctor: Hello and Welcome to \u2018Ask A Doctor\u2019 service. I have reviewed your query and here is my advice. I feel sorry for your husband. Yes recovering from myasthenia crisis may take that long as he is already weak, though good thing is that he is out of any immediate danger and will recover with time. However it should be kept in mind that crisis like this may come again. Continue with the treatment recommended by doctor and very soon we will see him walking.Hope this was helpful do let me know if you have any more questions.Regards,Dr. Akash Kumar"
},
{
"id": 211383,
"tgt": "Took ashwaganda brahmi combo, having intense anxiety symptoms. Can this be due to those supplements?",
"src": "Patient: I ve tried ashwaganda brahmi combo and the first week I felt so much better,had more energy ...etc,then when I got my period had very intense anxiety simptoms.Do you think this might be do to those supplements.I started those to help with stress!Thank you Doctor: HiTo be honest being an allopathic doctor psychistrist it will not be good to comment on a drug of other system. However the compound mentioned by you have effects on brain heart and CNS and will cause these symptoms as side effects in some as these also have effect on cvs and BPConsult a doctor self medication to be discouragedGet well soonDr LalPsychiatrist"
},
{
"id": 72960,
"tgt": "What causes shortness of breath?",
"src": "Patient: hello dr.i have a sleepless 4 nigths ago i couldnt even lie down on my bed due to difficulty of breathing and accompanied by nonstop burping,i thought i could be able to breath if i burp but still i always reposition to able to sleep please i need your help..thanks a lot...waiting edith Doctor: Hello dear , hiWarm welcome to Healthcaremagic.comI have evaluated your query thoroughly .* There are different causes for the same as - lung pathology in form of pneumonia , effusion or severe congestion - severe GERD related - anxiety , stress associated - diaphragm problems - liver pathology - other issues .* Needs proper evaluation with lab tests , x-ray , ultrasound examination to decide the cause and further management .Hope this clears your query .Regards ."
},
{
"id": 58341,
"tgt": "Fatty liver, pain in upper quadrant, high cholesterol, ldl. MRI done. Reason for pain?",
"src": "Patient: Hi. I am 35 years old , 5 feet 9 inches tall ,80 kgs, I have a fatty liver which got diagonosed during ultrasound. My alt is 102 and ast 65. I have persistent dull to moderate pain in Right upper quadrent also felt in upper right side of back. My lipids are also high total cholestrol 265 ,ldl 185 , triglycerides 260. I am a daily drinker since last 12 -13 years , per day consumption 300 -350 ml alcohol. Got my mri , ct scan, mrcp done , they all suggest fatty infilteration.. during all these years of drinking I have been giving gaps of 30 days atleast twice or thrice an year that is 60 to 90 days in an year.. kindly advice ... where do I stand ...and reason for this pain.. although tests have ruled out pancreatitis , do u think this could be mild initial pancreatitis not sowing in imagery tests .... or is it related to liver or gall bladder or is it some ulcer or just ibs...please guide... Doctor: HelloThanks for writing to HCMDefinitely pain is due to liver problem.You have to quit alcohol completely.Fatty liver is reversible state.You can take tablet ursodeoxycholic acid 150 mg twice daily for three months. It will help in regeneration of cells.Pancreas involvement can be confirmed by blood amylase and lipase level test.So,get it done.Also you need medicines for altered lipid profile.Get well soon.Take CareDr.Indu Bhushan"
},
{
"id": 78255,
"tgt": "Is it a matter of concern if I suffer from severe chest pains a few minutes after consuming alcohol?",
"src": "Patient: Hi. I am 23 years old, and very rarely drink alcohol (every 2-3 months) the last couple of times I have consumed alcohol, within 15 minutes I have suffered severe upper right chest pains. This happened last night after half a glass of wine - and the chest pains lasted through the night. Along with this I was unable to sleep as suffered a serious chill. Should I be worried? Doctor: Thanks for your question on Health Care Magic. I can understand your concern. You are mostly having alcohol induced GERD (gastroesophageal reflux disease). Alcohol causes laxity of gastroesophageal sphincter. Because of this the acid of the stomach tends to come up in the esophagus and cause the symptoms of chest pain. So better to avoid alcohol. Start proton pump inhibitors and prokinetic drugs. Avoid stress and tension. Avoid hot and spicy food. Avoid junk food. Don't worry, you will be alright. Hope I have solved your query. I will be happy to help you further. Wish you good health. Thanks."
},
{
"id": 80178,
"tgt": "Could the swollen throat and swelling be due to Laryngitis?",
"src": "Patient: My son( 6 yrs old.) woke up with a swollen throat, I noticed he couldn't sleep last nite, he woke up to clear his throat a few times, and when he did it wasn't easy, and he sounded like he was wedding because of the phlegm. When I looked in his throat, his gums were swollen, his back thoat was red, and swollen, and has bloody light patches on each side, and hes also saying it feels like a lump is in the back when he swollows, his voice sounds tight, and hoarsish. Could it be strep, or layngitis? Doctor: Hi,Thanks for the question.It looks like your child is having enlarged tonsils which may either be because of viral or strep infection. In laryngitis usually there is no swelling or lump sensation. So we can definitely rule it out.Hope it helps.Thanks"
},
{
"id": 130,
"tgt": "When should pregnancy test be taken after missing a period?",
"src": "Patient: I stopped taking the pill after my last period which started on the 9th Jan 2011 and I had previosly only been on the pill for 3 months! Me and my partner have been having unprotected sex since then at least every 2nd or even 3rd night! We already have an 6 month d daughter and want another one but I thought it would take a cpl of months to concieve? When do u think I will be due for my period so I can take a test? I have had dull lower abdominal and back aching, headaches, moody and tired!but preg tests r saying neg still even early ones.. My last pregnancy I had no noticeable symptoms until about 6 weeks! Do u think I am? An do u think I'd be due for my period around 7th feb? Or could it be different cos I came off the pill! Doctor: Hello,Around 9th of February you will get your periods and if you miss your period do UPT on 11th of February and see the result. As you stop taking pill and have sex every third day then you may get pregnant.Hope I have answered your query. Let me know if I can assist you further.Regards,Dr. Sheetal Agarwal"
},
{
"id": 17932,
"tgt": "Suggest treatment for head pressure from temple to temple",
"src": "Patient: i am cipro for a bladder infection....on about the last day i started to feel an odd head pressure from temple to temple...3.5 days after cipro i still had the inf. so am on cipro another 10 days...i still have this head pressure on and off all day....had a headache for 2 days while on the 2nd dose but my period started so not sure if that headache was from that because i get bad headaches every month while on my cycle...but i did have this headache and had nausea and vomiting from it.....i still have about 5 days left of cipr. I am concerned the cipro is causing my head pressure....i have told my dr. about it..she told me to get my eyes checked (because my vision gets funny with the head pressure) and she told me to take a decongestant (i don t have a cold or anything). I am concerned about intracranial hypertension that a freind mentioned to me. Doctor: Hi, I would explain that your symptoms seem to be related to a tension-type headache. In my opinion, you should not worry about intracranial hypertension. I would just recommend taking Ibuprofen or Acetaminophen and closely monitor your blood pressure values. Hope I have answered your query. Let me know if I can assist you further. Regards, Dr. Ilir Sharka, Cardiologist"
},
{
"id": 217322,
"tgt": "Suggest remedy for pain in the elbow",
"src": "Patient: Hello, 3 weeks ago i suffered an injury to my forearm area that caused me to be admitted following my initial ER visit where an xray of bone was negative but kept due to comparment syndrome in the affected area. I was painting a rather large steel job storage box when i bumped the box and the lid crashed down on my right forearm just below the elbow. I dont really know how to describe the injury; to my knowledge it was way worse than a typical bruise but not a strain or pull because typically i associate those type injuries as occuring during motion and activity, such as running and pulling a hammy. I have begun to have motion back in my wrist and fingers but am at a loss as to where i go from here. Is there anything i should be worried about regarding damage to the muscle itself. Know im really having more pain in my elbow while the impact area remains hard and feels like it may be deformed. Doctor: I think u should go and get yourself exam8Ned by an orthopaedician. Even if there is no bony injury, there may be muscle or ligament trauma which may require some treatment."
},
{
"id": 93924,
"tgt": "Lower abdominal pain. Eggs raptures. Injected HMG. Given susten and duphaston. Help?",
"src": "Patient: Hello Doctor, My wife was being injected with HMG 150 on day 3rd till day 7, she had produced more eggs than a normal no's of eggs a female should produce. Thereafter her follicular study was done, my wife's doctor had injected her for the eggs get raptured. After that he has advised to take duphaston, Progonova & Susten VT100. she still gets cramps in her lover abdominal part Is she Pregnant. Today she is on her 29th day of her cycle. when should i do my pregnancy test. Doctor: Hi and welcome to HCM. Cramps arent usual symptoms of pregnancy. If she didnt have period you should wait 2 weeks after it and then do pregnancy tests. It is hard to say now is she pregnant. SOmetimes is more than 6 months needed for hormonal stimulated ovulation and succesful fecundation. I wish you luck,just be patient and tenacious,there is no need to hurry. This therapy can cause unplesant symptoms such as cramps, vaginal bleeding etc. but this cant be avoided. Wish you good health."
},
{
"id": 54075,
"tgt": "What does absence of fluid in drain tube of liver indicate?",
"src": "Patient: I have a drain installed in my liver. recently, the drain tube began leaking under the dressing when I flushed it. now there is no fluid at all in the drain tube. I went to the VA ER on Friday, but no surgeon was available. they sent me home with extra dressings and instructions to apply a new dressing every time I flushed the drain. it was just Thursday when a fresh scan of my liver was conducted and determined the drain had more work to do. why would there be no fluid at all in the drain tube? No surgeons at the VA on weekends. Doctor: Hi and welcome to Healthcaremagic. Thank you for your query. I am Dr. Rommstein and I will try to help you as much as I can.There are few possible causes, drain may be closed with old hematoma or pus or kinked inside abdomen. If there is leaking behind drain then it means that there is intrabdominal collection that still persists. YOu need to do ultrasound and reposition of drain.I hope I have answered you query. If you have any further questions you can contact us.Kindly regards. Wish you a good health."
},
{
"id": 129882,
"tgt": "Suggest treatment for severe knee pain while bending",
"src": "Patient: I am suffering from severe knee pain just below the knee cap. The pain increases while bending my leg.. I was advised to take altraday (if needed) , uniwalk and calcired plus tablets. 4 days gone still the pain exists if altraday is not taken regularly. Doctor: Most probably u r suffering from.osteoarthritis . . Ado one knee xray Ap an lateral and consult orthopedic surgeon . ."
},
{
"id": 63895,
"tgt": "Suggest treatment for bump on shoulder in kid",
"src": "Patient: Im trying to figure out what to do about my 6 year old daughter has an open wound on her head on the top i have put peroxide and triple antibiotic ointment on it and it was getting better, but I looked today and it looks worse then it did. It almost looks like a hole and it is got mucous and blood. Then she has a bump on her shoulder that looked like a pimple so i squeezed the puss out and its getting bigger and redder and i keep having to squeeze the puss out. Doctor: Hi,Dear,Good Morning and Thanks for your query to my virtual Clinic At HCM.I studied your query in depth and understood your concerns .Causes for lump on shoulder in your kid as follows-Mostly they are -Chronic Scalp wound infection with boil on shoulder with pus discharge.Treatment-Tab-NSAIDs -like VoltarenTab-Amoxycycline 250 mg x three times a days Dressing for the Scalp infection andSurgical I and D for Boil with abscess on Shoulder.Check with ER Surgeon and check your fbs and pps,and plan for Excision Biopsy of the lump to resolve the worry.Don't worry of them,as they could be relieved with proper treatment.Hope above reply would help you to relieve of the issue you have.Welcome for any further query to ME and HCM.Wishing you a Fast Recovery.Write Good review for the benefit of my new patients.Click thanks ASAP after this.Have A Good Day.With RegardsDr.Savaskar"
},
{
"id": 195614,
"tgt": "Is total sperm concentration of 92 million with actively motile 85% normal?",
"src": "Patient: Dear sir my semen report is volume-2.5ml, colour-grayish white , viscosity-present , ph-alkaline, total sperm concentration 92 million, actively motile-85%, sluggishly motile-10%, non-motile-05%, head defects-10%, neck&midpiece-10%, tail-defects80%, epithelial cells-nil, red blood cells-nil, pus cell-2-4/HPF, I WANT TO KNOW my Report is right or worng Doctor: Hello and Welcome to \u2018Ask A Doctor\u2019 service. I have reviewed your query and here is my advice. Your semen analysis report is perfectly normal and nothing to worry in it. The sperm count and motility(proportion of healthy sperms) are very well with in normal range. As per your report your semen status does not affect your fertility. Hope I have answered your query. Let me know if I can assist you further."
},
{
"id": 7860,
"tgt": "What skin care regime must I follow to control acne ?",
"src": "Patient: Hi, I have a combination skin, which means oily on my t-zone and dry on cheeks . I take care of my face real good. I wash it every night with a gentle cleanser , I moisturize it, and whenever I get pimple or acne I use the acne medicine and it will disappear. But in the few past days I got some pimples and acnes which just dont go away! And whenever they disappear new ones come out! Why is that? What should I do? p.s. I do exercises, and I eat healthy. And I m 18. Doctor: Hi!Sue, Welcome to HealthcareMagic forum, Sue you wrote that you clean your skin properly..thats good ,but you also mentioned that you moisturize it ....well that not good atall...since you have acne prone skin you should not moisturize it.It an irony that you have a combination skin and still you get pimples..but that happens..So start with using a oil free face wash...you need to take tablets like azithromycin 500mg once daily for 3 days for acne..apply anti acne gels regularly like nadifloxacin gel ..Along with this you need to check your hormonal profile too,as imbalance in you periods can also be the cause of pimples...Hormone profile like LH/FSH ratio,SERUM TESTOSTERON,DHEA-S needs to be done...this would give the clear picture of your hormones.At time one gets pimples two days prior to periods ,which also means that hormones are responsible for pimples.If not you need a conventional treatment.Recently available is one of the best medicines for acne is isotretinoin which needs a dermatologists prescription.Its a wonder drug for pimples but needs some precautions and cautions before taking this. Kindly consult a dermatologist for the same. take care, Dr.chawda"
},
{
"id": 105305,
"tgt": "Suffering from skin allergy, skin gets patchy, itchy, no relief with several treatments. Guidance?",
"src": "Patient: Sir, My age about 60 year is suffering from skin allergy or skin problem since last 8 years in which her skin become patchy and ( starts severe khujali in her arms and face) I have shown to many doctors, some say that it is allergy with gajar ghas and some say it is internal problem it can,t be healed and some say it is life long and it cannot be cured . In Jabalpur I also shown to Skin Specialist ,he gave treatment but after some relief it again reappear and the problem is as it. So ,i request to sir kindly guide us for the treatment of the same. Doctor: Dear Mr Mehra, If your skin comes up in hives or circular red itchy lesions (with central clearing) that disappears on its own or after medications (usually antihistamines) in a few hours, you are most likely suffering from a condition called urticaria. This can also manifest with swelling under the skin or swelling on lips and tongue called angioedema. This is due to histamine release that may be spontaneous or a secondary trigger like scratching (physical), cold or heat (temperature change) or related to hormonal imbalance. Conditions such as diabetes, underactive thyroid, or other autoimmune diseases make it worse when they are uncontrolled and a proportion of patients with urticaria have 'autoimmune urticaria' while in others it may be idiopathic. High-dose long-acting antihistamines for 3 months (minimum duration) may provide extended relief when stopped but may need to be continued for longer. Fexofenadine 180mg and cetirizine 10mg twice daily is the usual choice. Thanks."
},
{
"id": 200712,
"tgt": "What causes difficulty in getting erection with pain in groin?",
"src": "Patient: My husband is complaining of pain in his groin area, like its pulling his penis inside of his body. He has issues getting an erection, and if he does its not fully erect or last long. I ve noticed that his penis is even shortening, and the gerth isn t there anymore. Can you please let me know what I need to do to help my husband. Doctor: Thanks for asking in healthcaremagic forumIn short: His pain due to some other reason may be the cause for erectile dysfunctionExplanation: If there is any pain/infection nearby , he can have difficulty in getting and maintaining erection. So, please take him to a doctor for examination of the same. Good luck."
},
{
"id": 67122,
"tgt": "What causes white lumps on elbows/knees?",
"src": "Patient: My 13 year old daughter has developed these white lumps on her elbows and knees and another type of small raised bumps on her arms and legs. those on her arms and legs look like a razor has been on it but she hasnt shaved her arms. but the lumps on elbow and knees loook different and wondering is its all related or different Doctor: HiWelcome to hcmAs per your discription those are unerrupted hair follicles on the skin which generally more prominant on elbow and knee. But at least once show it to a dermatologist/skin specialist actually. For these small bumps only proper scrubing of that skin with scrub and mechanically scrubing it is adequate in most of the cases.Regards."
},
{
"id": 197241,
"tgt": "Suggest treatment for nocturnal emmision",
"src": "Patient: hi! i am suffering from nightfall from last 6 yrs .it occurs 2-3 times a week and sometimes not occur for the whole month also .i feel very depressed because of this and it affects my studies also .i am 21 yrs old please suggest some medicine so that i can completely get rid of it. i do masturbation very less only do it when it feels uncontrollable Doctor: Hello dearUnderstand your concernDo not depress and just relax because nocturnal emission is common problem in men of your age group. It mostly seen in sexually inactive person or those who masturbate very less. Sperm and seminal fluid production process is continuous. If you do not remove it by masturbation, it will come out naturally in the night as it production is overflow.II advice you to masturbate regularly (3-4 times per week). as maturation is normal sexual behavior and meets your sexual demand. It does not cause side effect if done in moderate pattern.Once you become sexual active either by masturbation or with partner, it will disappear.Take healthy diet with high protein, fresh fruits and regular exercise and medication in the early morning.Hope this may help youBest regardsDr. Sagar"
},
{
"id": 186766,
"tgt": "How to treat gum bleed?",
"src": "Patient: My gums bleed treble and there is no pain. i took amoxilicin moxikin cv 625.... now my head aches all the time. i feel like im fainting. i have tried all the possible remedies. mouthwashes salt gargle limewater but the swelling just insnt going n its gettin worse. i cant work this way. plz suggest me a way. tx Doctor: Hello and welcome.Thanks for sharing your concern.Please try to meet your dentist to the earliest.your gums requires a small surgery called as periodontal surgery.Delay might land up in mobility of teeth and finally loss of all teeth.Therefore its nice that you have taken a course of antibiotic,simultaneously see your dentist too.Hope it helps.Thanks.Take care."
},
{
"id": 151375,
"tgt": "Difficulty walking, sharp pain in feet, burning in lower spine. Spinal fusions done. Treatment?",
"src": "Patient: For the past three months, it has been very hard to walk. I am only 38 years old. I have had two spinal fusions in 2005 and 2007. Someone rear ended me leading to these surgeries. I am now experiencing very sharp pains in my feet . Burning from my lower spine to my thighs, and it feels like the tendons are super tight to where my legs just don t seem like they want to work. Standing from a laying down or sitting position is almost unbareable. Once I get walking it all calms down, but it takes about a minute or two. Doctor: Hi Yours is a post spinal fusion status ( operated twice) and now have developed sharp pain , burning sensation and spasticity of lower limbs. At the first place were you operated upon at the same spinal segment twice or it was different segment? What the spinal segment? What was the cause - Intervertebral disc prolapse or tumor? Surgeries usually are taken up in cases of intervertebral disc prolapse when all other conservative managements fails or when there is a severe spinal canal stenosis. It is also not uncommon to land up with a 'failed back syndrome' in post operative cases. Whatever the case now you need to get the following investigations done: 1. MRI Lumbo-sacral spine (plain). 2. X-Ray lumbo-sacral spine (AP and Lateral) See a neurosurgeon with the report and get a proper neurological evaluation done. As for the present treatment options, it is possible only after getting a detailed history regarding the previous surgery , seeing present scan reports and doing a clinical evaluation . Thanks."
},
{
"id": 153251,
"tgt": "Suggest treatment for severe shoulder pain and breast cancer",
"src": "Patient: My husband has moderate to severe pain under his left shoulder a little towards the right. This has been going on for approximately 3 months. It is some what swollen and is also tender to the touch. Motrin 800 takes the edge off the pain, cold compresses do not help but heat seems to give him some relief. He has seen a physician and was given muscle relaxers and narcotic pain medication that really just puts him to sleep. He also has taken steroids which did seem to help him. It is also important to note that on December 5, 2014 he was diagnosed with breast cancer in his right breast and is scheduled for surgery this Thursday 1/22/15. Pet scan, CT scan, MRI, Ultrasound have all been negative for any cancer in the area where he is experiencing the pain in his back. What are your thoughts? Y. Williams Doctor: Hi,Thanks for writing in.If not the cancer then it is probable that he is having pinching on nerves taking place along the nerve pathway. The nerves take origin from roots at the spine. Then the nerves travel through exiting nerve root foramina and take the formation of brachial plexus which proceeds to supply the skin, mucles and joints.It is possible that a nerve entrapment is occurring at some place and the pain is felt at the shoulder. Cancer is less likely the cause of this pain. Please do not worry."
},
{
"id": 140992,
"tgt": "What are the side effects of ridlin intake in females for ADD?",
"src": "Patient: Hi there I am 55 years old female and have been treated for ADD for 15 years now. I take 20mg of ridlin five days a week . I have tried other options but nothing seems to of the job. I am a very active person and I am worried about what my body is going through with taking Ridlin for all those years ... thank you joann Doctor: Hi, You mean Ritalin right? In such a case, the major risk of chronic use may lead to tolerance and dependence. So, try to use the smallest dose possible and avoid abuse. Follow up correctly with your Doctor. Hope I have answered your query. Let me know if I can assist you further. Regards, Dr. Erion Spaho, Neurologist, Surgical"
},
{
"id": 35573,
"tgt": "Should I be worried for watery discharge from the staples incision?",
"src": "Patient: I recently had staples put in my leg after surgery. After they were removed one of my cuts started to swell from underneath. I determined it was fluid build up. I took a needle and drained a clear/red fluid from the location. Do I need to be concerned? Doctor: Hi welcome to hcmYes it could underlying infection leading to abscess..Consult your surgeon immediately and start using antibiotic course and dressing of your wound .thank you"
},
{
"id": 107092,
"tgt": "What causes lower mid back pain when diagnosed with kidney infection?",
"src": "Patient: Hello. I had been diagnosed with a kidney infection. I had pain in my lower left back. Pain had subsided for most part but now has traveled to lower middle back. Hurts most when turning over at night but is now hurting during the day. Same pattern during kidney infection, only in different part of back now. What could this possibly be? Doctor: dear patient pain in middle lower back can be due to kidney stone or back problem. since you had taken treatment for kidney infection possibility of renal inflection is there and needs to be ruled out. Do u have urine problem like burning sensation ? is urine frequency Is increased and there is need for urgency? if yes then renal infection is likely. please do not neglect this and visit your urology as soon as possible. thanks."
},
{
"id": 86960,
"tgt": "Suggest remedy for constant pain in the lower abdomen",
"src": "Patient: Hi, I seem to have a constant pain the last couple of days, lower abdomen, left side, above my hip. Tender to touch. I am female, 50, in good health except for iron deficient anemic to the point of having transfusions, and iron IVs, plus monthly B-12 shots. I am having blood work done by hematologist tomorrow and would like to at least be educated enough to describe this properly and perhaps know some of the possibilities. Thanks Doctor: Hi there,Thanks for using HCM.Pain in the left lower quadrant usually occurs due to colitis or diverticulitis.If you have a history of constipation, the diverticulitis is the most likely cause.A CT scan of the abdomen may be done to confirm it.Iron deficiency anemia that you have could be due to inadequate intake of iron or increased loss of iron. This loss could be from the gastrointestinal tract or genitourinary tract.An upper GI endoscopy and a stool occult blood may be done as the first step to see if there is any gastrointestinal loss of iron.Is this answer helpful?"
},
{
"id": 167935,
"tgt": "Suggest treatment for vomiting and fever in kid",
"src": "Patient: Good morning sir\\ma. I have a six months old girl, she has been sick for some time now. She stool, vomiting when taking oral dispensation, hot temperature and cough. This did allow to sleep. She wakes after some naps. Pls, kindly advise on what to do.r Doctor: probably your child is having gastroenteritis which is usually a viral infection of the intestine , usually associated with vomiting, fever and diarrhoea I recommend you to visit her doctor to assess her if she is dehydrated or not and based on that he may request stool samples and suggest oral rehydration solutions I hope this helps"
},
{
"id": 15900,
"tgt": "Have rashes on abdomen along with itching. No medication till now. What should I do?",
"src": "Patient: Hello I've developed a rash of sorts. It started with a red bump on my abdomen that has now proceeded to become a flat dark dry patch (10mm x 20mm) that is still itchy. Other similar lesions have cropped up on my thighs and inner arms though smaller.The only thing that has changed recently is that I've cut out a lot of processed foods and am drinking green juice regularly (kale cucumber celery and raw plant protein powder). Im a vegetarian. I used a topical over the counter steroid cream which helped temporarily. The rashes have been there for two weeks now. Doctor: Hi Welcome to Healthcare Magic Forum Seems like you have developed Pityriasis Rosea(PR) PR manifests as an acute, self-limiting, papulosquamous eruption with a duration of 6-8 weeks. PR has been linked to upper tract respiratory infections. The disease typically begins with a solitary patch, usually salmon-colored, that heralds the eruption and thus is commonly referred to as the herald patch or spot. This initial lesion enlarges over a few days to become a patch with a collarette of fine scale just inside the well-demarcated border. Within the next 1-2 weeks, a generalized exanthem usually appears, although it may occur from hours to months after the herald patch. This secondary phase consists of bilateral and symmetric lesions with scales .This phase tends to resolve over the next 6 weeks, but variability is common. Pruritus is commonly evident in 25-75% of patients and usually of mild-to-moderate severity. The eruption is symmetric and most commonly involves the thorax, back, abdomen, and adjoining areas of the neck and extremities. As is a self-limited disease; treatment is supportive. Topical zinc oxide and calamine lotion are useful for pruritus. If the disease is severe or widespread, topical or oral steroids may be used. The skin rash may take 6-8 weeks to clear up completely..You can take oral antihistamines like levocetrizine for pruritus. Hope it helps Dr Geetika Paul"
},
{
"id": 9003,
"tgt": "Changes in skin color",
"src": "Patient: when i was child my face colour was white but now i m age of 18 my coloure is black why how i can white? Doctor: Hello Mr .Pathan; welcome to HealthcareMagic Since you are 18 yrs now the colour of the skin of face will change because in young age there is tanning of skin due to excessive exposure to sun while travel;play;driving bike and other out door activities.To make it a little better or fair please try to reduce exposure to sun if it is not necessary to do so.Try applying a sunscreen before going out then after that for care of your facial skin you can get a clean up or even facial done and things will improve.If the problem continues or worsens then contact a Dermatologist for further advice and treatment. Thanks"
},
{
"id": 177856,
"tgt": "What causes loose motions in a baby?",
"src": "Patient: Hello Doc.. My baby is 12 months+ old.. She hv been suffering frm loose motions since 10 days. I hv given her homoeopathic medications.. By that stool frequency hs reduced frm 8 motions to 5 motions per day.. Character of stool is smetmes yellowish frothy or wid mucous or smtmes greenish.. Painless... Stool after evey feed Doctor: Hi....It seems your kid is having viral diarrhoea. Once it starts it will take 5-7 days to completely get better. Unless the kid's having low urine output or very dull or excessively sleepy or blood in motion or green bilious vomiting...you need not worry. I feel it is settling down now, as the stool is gradually getting semisolid.Regards - Dr. Sumanth"
},
{
"id": 116359,
"tgt": "Is blood sugar test result of 98mg/dl cause for concern?",
"src": "Patient: Hi, may I answer your health queries right now ? Please type your query here... Hi Dr. This is Rajesh I am 46yrs. I have got my Blood Sugar Test : Result 98mg/dl Post meal 151mg/ dl S. Cholestrol 201mg/dl Are my results, Ok I have Hypertension from 2001 , Taking losartin 25mg twice. I am not on Sugar medicine. Guide.... Doctor: Hello,Based on your query, my opinion is as follows.1. You fasting level is good. But your post-meal is showing a mild increase.2. Serum cholesterol mildly elevated.3. You need not start on any medication for diabetes or increased cholesterol. Weight reduction, aerobic exercises, and diet control are necessary.4. Also, continue medications for hypertension.Hope I have answered your query. Let me know if I can assist you further.Regards, Dr. Prakash H Muddegowda"
},
{
"id": 162563,
"tgt": "What causes ear ache and fever in a child?",
"src": "Patient: My 8 year old daughter has been experiencing pain in her ear as well as in front of the ear and behind the jaw bone x 3 days. If I press on her ear or touch it by accident it hurts her. She had 99.6 degree fever the other day but not now. She had been swimming, but has no other symptoms except her neck on that side is tender if she tries to bend it to her shoulder and her neck popped a few times while checking ROM. I don t feel any swollen lymph nodes. Doctor: Hello and Welcome to \u2018Ask A Doctor\u2019 service. I have reviewed your query and here is my advice. By what you say I feel that it could be an otitis media or middle ear effusion. I suggest you take her to the pediatrician or ENT surgeon - otoscopy will be done and then appropriate antibiotics or pain medications will be prescribed. Hope I have answered your query. Let me know if I can assist you further."
},
{
"id": 222401,
"tgt": "What are the symptoms of pregnancy?",
"src": "Patient: I had my periods on December 23rd .I am 30yrs old.For the past one week,I have symptoms of Vommiting,tiredness,Legs paining as if I am going to get my periods,morning sickness.Feels very hungry.getting Distrubed from sleep,Getting so irritated.Is it signs of pregnancy,if so,when should I test . Doctor: Hi dear, I have gone through your question and understand your concerns.The symptoms you have described can be present both in early pregnancy as well as in premenstrual period.I will suggest you to wait for the periods, if delayed then get a urine pregnancy test done to confirm the diagnosis.Hope you found the answer helpful.Wishing you good health.Dr Deepti Verma"
},
{
"id": 125600,
"tgt": "What causes intermittent unusual bruise-like pain in the legs and hips?",
"src": "Patient: Weird bruise-like pains that come and go...they sorta increase in intensity for a few minutes then leave...happens for a few days about every three months and there is never any visible sign of whatever it is...generally in the hip, buttocks, and leg regions...any thoughts? Doctor: Hi, These are called paresthesia. It could be due to deficiency of vitamin B12 or nerve pressure (if they are occurring in one area only). Apart from these, there are many other causes. Get serum Vitamin B12 tested, and to see if it is normal go for a neurologist consultation. Hope I have answered your query. Let me know if I can assist you further. Thanks. Regards, Dr. Gopal Goel Orthopaedic Surgeon"
},
{
"id": 114749,
"tgt": "What is the proper procedure for taking my blood pressure?",
"src": "Patient: Hello,I was recently turned away from a colonoscopy because my blood pressure was too high (170/110). I understand there is a proper way to take blood pressure readings and improper ways to take them.I am taking my blood pressure daily and logging it for my next doctors visit. What is the proper procedure for taking my blood pressure?Thank you, Doctor: Hi welcome to HCM Dear ,You may need to take your BP at home if have hypertension (high BP) or hypotension (low BP). High BP increases your risk for stroke, heart attack, or kidney disease. Low BP may cause you to feel dizzy or like you are going to pass out. This usually happens when you stand up quickly. You may need to take medicine to keep your BP at a normal level . Start the procedure with few do's and dont's for accurate reading Do not check your blood pressure within 30 minutes of smoking, drinking coffee, or exercising. These may affect your BP reading.Do not take a BP reading in an arm that is injured or has an IV or a shunt.Sit and rest for 5 minutes before you take your BP. Extend your arm and support it on a flat surface. Your arm should be at the same level as your heart. Both of your feet should be flat on the floor.The device has a built-in pump that inflates the cuff. Put the cuff about 1 inch (2.5 cm) above your elbow. Wrap the cuff snugly around your arm. The BP reading may not be correct if the cuff is too loose.Turn on the BP monitor and follow the directions.Write down your BP, the date, the time, and which arm you used to take the BP. If possible, take your blood pressure twice and write down both readings. These BP readings can be 1 minute apart. Let the air out of the cuff. Turn off the monitor and take off the BP cuff.Take your BP at least twice a day. Take your BP at the same times each day, such as the morning and evening for accurate reading . Hope this helps solve your query . All the best .Don't hesitate to get back if have any further quer"
},
{
"id": 166772,
"tgt": "Is it normal symptoms to have earache and painful rashes with chicken pox?",
"src": "Patient: my four year old has chickenpox, she came out in the rash yesterday, i have given her calpol and priton, which was advised by the chemist, also camiline cream, but she is complaing that the spots hurt, she also has earache and feels sick, is this normal? Doctor: Hi,I suggest you to get back to us with an image of the child with the so-called chicken pox skin lesions. I am telling this because we need to first ascertain the diagnosis of chickenpox and then suggest you accordingly. Please do not apply any cream or lotion over the body as it might cause of the skin lesions to spread fast. Earache and sensation of nausea are quite common in any viral illness and chickenpox is also a viral illness.Hope I have answered your query. Let me know if I can assist you further. Regards,Dr. Sumanth"
},
{
"id": 35241,
"tgt": "What is the healing period of fungal infection on leg?",
"src": "Patient: I have a fungal infection on my right leg due which lot of liquid has been deposited...i visited Apollo hospital and they provided with dressing and prescribed few medicines...However i am taking medicines and also applying mycoderma antifungal powder which has somewhat dried up but still not completely.can u tell me how much time it will take to completely heal up? Doctor: Hello dear,Thank you for your contact to health care magic.I read and understand your concern. I am Dr Arun Tank answering your concern.Fungal infection usually requires a long time therapy. Duration or treatment varies according to severity of the infection. But average three to dour month treatment is required to completely clear the infection.In addition to treatment good hygiene keeping by frequent cleaning and dressing is utmost important as taking drugs. Please maintain strict glycemic control. If you are diabetic than strict sugar control can help you in good treatment.Avoid injury and wearing good quality cotton cloth over the infefted area can help you in treatment.I will be happy to answer your further concern on bit.ly/DrArun.Thank you,Dr Arun TankInfectious diseases specialist,HCM"
},
{
"id": 85581,
"tgt": "Can formonide cause numbness in shoulder?",
"src": "Patient: I am 32 year old and asthama patient and where I am staying is cold weather -5 deg C temperature I am taking formonide 400mg and since last one month my left side shoulder is sensless, Last year I fell on left shoulder during winter. please let me know is that because of this medicine or something else Doctor: Hello, Very unlikely. Formonide does contain a steroid. It also contains a bronchodilator. Although steroids can cause weight gain, when they are inhaled as Foracort is, the risk of side effects like that or ant shoulder numbness are pretty rare. Hope I have answered your query. Let me know if I can assist you further. Take care Regards, Dr AJEET SINGH, General & Family Physician"
},
{
"id": 178050,
"tgt": "Suggest treatment for small water blisters between toes of a child",
"src": "Patient: My 8 year old son has been having small water blisters between his toes. It is not healing and has been giving him lots of pain and itches a lot. How should I treat this? Should I pop the blisters? Is this something I should take him to the dermatologist for? Doctor: Hello. I just read through your question.This is not uncommon. For now you can apply any antibiotic ointment (ie: bacitracin, neosporin, etc.) three times per day. This will be absorbed into the blisters and kill whatever bacteria is in there. if the blisters pop, continue to apply the same ointment to prevent bacteria form infecting it again."
},
{
"id": 118128,
"tgt": "Why is my blood pressure high in the mornings?",
"src": "Patient: My blood pressure is 117/90. The bottom is consistently high when I wake up in the morning. I'm pretty sure I have sleep apnea, which I believe is what brought this on. Sometimes when it is real high, my mom or dad will give me one of their blood pressure pills. Doctor: Hi,Your blood pressure appears to be within the normal range. You have not mentioned the blood pressure, which you say is consistently high. However, I would like to inform you that sudden arousal of the sympathetic system you blood pressure will be high in mornings compared to other times. It will be higher at around 10 am and peak till the noon and then drop. This is normal diurnal variation in all individuals.Avoid taking unregulated treatment. Your dosage might be low, but other added factors might lead to hypo-tension and accidental injuries.If you still have doubts, you can take ambulatory blood pressure monitoring, which helps in checking your 24 hr circadian blood pressure variation. If you think you have sleep apnea, meet the doctor and try to get it treated as it increases the risk of stroke, vascular diseases and congestive cardiac failure.If you have further doubts, you can get back with the exact high blood pressure in mornings and I will try to further help you.Hope i have answered your query."
},
{
"id": 162711,
"tgt": "What treatment is recommeded for enlargement of heart on one side?",
"src": "Patient: I m a nervous grandma. My 4 year old granddaughter is having a procedure to fix a leaky heart valve which has been being monitored since she was very young in hopes it would seal. Consultant now says one side of heart is becoming enlarged so time to do something. Day surgery, through the groin. Is this a common procedure, what exactly do they do, and will the enlarged heart return to normal after and she will be 100% over time. Is currently active/healthy/happy but does sweat unusually and last weekend was very red-cheeked although still happy/active as usual. Doctor: Hello and Welcome to \u2018Ask A Doctor\u2019 service. I have reviewed your query and here is my advice. As you described sweating it indicates heart isn't able to pump enough blood to meet the needs of body. Certainly time to do something. Every operative procedure has its risks involved. But if not treated risk of increasing current heart failure is more. Usually after procedure the symptoms resolve and growth of child improves. The heart comes to normal size in some weeks. So need to understand risks involved in treatment and leaving it alone. For more detailed counseling meet your cardiologist. Explain your expectations and listen about what is to be expected after the procedure. Hope I have answered your query. Let me know if I can assist you further."
},
{
"id": 49724,
"tgt": "Done bronchospy, had stomach pain and pneumonia. Has a shadow on kidney and lung. Meaning?",
"src": "Patient: My mother has a shadow on her kidney and lung what does that mean and what should be done next. She is 80 and had a bronchospy done 2 weeks ago going back to consultant today ... What questions should I be asking. Also as a follow up from a few days in hospital 3 months ago she is having an ultrasound of her kidney and will be meeting a different consultant for that ... What should I be asking him. My mother is completely strung out about all these tests not having been healthy all her life she had a severe pain in her stomach 3 months ago and was hospitalised but they found nothing diagnosed she had atypical pneumonia these are the follow up tests now .. Thanks Doctor: Hello, Thanks for the query to H.C.M. Forum. In my opinion the black shadow in the kidney area may be due to tumor or solid mass . Diagnosis can be confirmed by biopsy of the part. 2nd thing the shadow in the lungs is due to solid mass ( may be secondaries due to kidney mass ) in kidney area. Diagnosis will find out the exact nature and type of this mass .3rd point is pain in the stomach , this may be due to peptic ulcer disease . Diagnosis can be confirmed by upper G I endoscopy . However try raising the head of bead 4 inches with blocks help. Avoid greasy , spicy food . In ,my opinion first of all consult an Urologist and get his opinion regarding kidney shadow and there after deal according to situation . If kidney shadow normal as due to cyst then consult a pulmonologist and get his opinion and deal accordingly . Only question is whether this black shadow is cancer or not.Good luck for your mother . Dr. HET"
},
{
"id": 115787,
"tgt": "How to improve WBC count?",
"src": "Patient: i am 59 years old male very active run 7-8 miles every other day now for the last 4 weeks my health is declining i cannot even run a mile without short of breath exhausted and getting dizziness, yest. i have my cbc done and my wbc 3(L) now whats my next move ? how can i improve my wbc? thanks RON. Doctor: Hello and welcome to HCM,There is no means of improving the white blood cells.White blood cells are disease fighting cells of our body and they increase or decrease in case of infections or conditions wherein the white blood cells are exhausted.Thus, it is important to know the differential count of the white blood cells to assess which component of white blood cells in depressed.Further management can be planned after clinical assessment and relevant investigations.Thanks and take careDr Shailja P Wahal"
},
{
"id": 124999,
"tgt": "What causes sore painful neck?",
"src": "Patient: Ok so I woke up this morring and my neck was fine then about an hour later my left side of my. Neck was sore sore, I couldn t move it to the left back forward or even turn it, if I did so my neck will kill and I wouldn t.be able to lift my left arm because its gone dead! What is it?? Doctor: Hello, It could be due to conditions like cervical spondylitis. Consult an orthopaedician and plan for an MRI neck for further evaluation. Hope I have answered your query. Let me know if I can assist you further. Take care Regards, Dr Shinas Hussain, General & Family Physician"
},
{
"id": 206620,
"tgt": "Suggest treatment for bipolar disorder",
"src": "Patient: My husband has had bipolar for over 20 years with minor ups and downs. Recently he had a sever episode with extreme paranoid and manic symptoms, rapid cycling and long mixed episodes and was not reacting well to any types of meds. This lasted for 6 months. after being in the hospital he has stabalized but is not the person he once was. He has extreme cognitive issues. Can bipolar that was so sever cause actual brain damage? He is seeing a neurologist and just had eeg's of brain and MRi and we are waiting results Doctor: HIThanks for using healthcare magicBipolar disorder is a psychological disorder and in long term, it could lead to cognitive dysfunction. It does not lead brain damage. Brain damage could be due to medicine or age factor. In case, you have further query, you can ask.Thanks"
},
{
"id": 196832,
"tgt": "Suggest treatment for itching in the scrotum",
"src": "Patient: My situation is that I have ever present dampness in my scrotum area with a persistent itch........ I have tried the jock itch creams available OTC with a degree of relief, but after i discontinue use the itch comes back..... I walk around looking like a hip hop star always grabbing my crotch area........ Doctor: Hi, you need oral antifungal treatment like Fluconazole 50mg once a day for a week, followed by 150 mg once a week for 8 weeks to cure the fungal infection that you have--regards"
},
{
"id": 172884,
"tgt": "Can an infant be administered with oflox drops for cold?",
"src": "Patient: Hi I'm a mother of an 8 month old baby girl, the peadiatrician prescribed to use oflox drops in her nose for her cold to reduce the bacteria. I would like to know if it's safe to do so I'm a bit worried as the drug is only used for the ear and eyes. Please help. Concernd mom. Doctor: HiWelcome to the HCMIt's not advisable to go for ofloxacin drops for such treatment. Most of the cold in this age group are due to self limiting viral infections. They respond very well to just symptomatic management. So, I would recommend you to give her an antihistamine drops at recommended dose as per weight and age along with regular feeds and fluids to maintain hydration.Also, you may use normal saline nasal drops regularly as they are the safest for clearing blocked nose.Use a room humidifier while sleeping if the weather is dry.Hopefully this will help you.I will be happy to help you in any further questions.Take care"
},
{
"id": 202841,
"tgt": "Blood through penis on masturbation, no pain or itching. Cause?",
"src": "Patient: i am 19 and from last 2-3 months i am having a problem while masturbation some amount of blood used to come out from my penis along with the semen .... but 2 days back when i masturbarate only dark red blood come out from my penis..... I am really very worried what to do for it.... I am not having any pane or inching in my penis..... plz help . Doctor: HelloThanks for your query,based on the facts that you have posted it appears that you have what is called as Hematospermia.This could be most likely to be due to infection of the either Seminal Vesicle or Prostate.Please get your semen culture done and consult qualified Urologist for clinical and digital rectal examination.and get Ultrasound scanning of the abdomen and scanning with trans rectal probe done to confirm the diagnosis. You need to take broad spectrum antibiotic like Cefexine along with anti inflammatory drug like Diclofenac twice daily.for a long period ( 4-6 weeks .)Dr.Patil."
},
{
"id": 159676,
"tgt": "Blood in urine, tender lump on right kidney. Urine test, kidney scan, cystoscopy done. Cancer?",
"src": "Patient: hello. Over the last 8 weeks i have had my urine tested 5 times, each time traces of blood have been found. My urologist is going to test me for Cancer . I am having a kidney scan and Cystoscopy in 2 weeks time. I have noticed for quite some time a lump on my right kidney which is tender to the touch, it doesnt appear to have got any bigger and hasnt worried me until now, i just assumed it was a gland. Thank you Karen Doctor: hi happy to help you that it may be hydronephrosis? may be stone in ureter? so you dont wory and take excess intake of water"
},
{
"id": 66424,
"tgt": "How to get rid of growing painful lump on labia minora?",
"src": "Patient: hi Doctor, i have a questions to ask. i have a lump on the left side of my labia minora lower side. it hurts when i touch and wipe when i pee. it started last week with small and i thought it is pimple so i think it will go away soon. but it is bigger than last week and i am so worried. can you help what is this? Doctor: Hi, dearI have gone through your question.I can understand your concern.You may have some benign cyst. It's common to have benign cyst. You may have some soft tissue tumour. You need examination for exact site, size and nature of lesion. It will help to reach the correct diagnosis. Then you should take treatment accordingly. Hope I have answered your question. If you have any doubts then feel free to ask me. Thanks for using health care magic.Wish you a very good health."
},
{
"id": 128845,
"tgt": "What causes pain in the armpit?",
"src": "Patient: i have a like a achey pain in what i feel is a random place, its like above my left breast , below my collor bone and just in from my armpit! just a general pain, been there on and off for a few weeks, i cant figure out what brings it on or anything, not sure if its a muscle thing? any advice? Doctor: Hi, I had gone through your question and understand your concernsWith such presentations in my clinic, I would first reassure you as this pain is mostly muscle spasm in pectoralis major muscle.I advise you to use warm fomentation,muscle relaxant and topical analgesic cream."
},
{
"id": 158852,
"tgt": "Mouth ulcer caused by broken teeth, took medicines, no relief. Cancer?",
"src": "Patient: hi can write in bengali? i am suffering from mouth ulcer(right corner) caused by broken wisdom teeth of my right side (upper portion) since 6days, i have taken mox500mg (15 cap for 5 days). it was ok, but now from yesterday i m again feeling pain, i saw the ulcer its whitish and can see little bit swelling there.i m very tensed that after taking amoxyciline it was ok, then why i m feeling pain again? if its a cancerous ulcer then antibiotic can work? Doctor: Hi, Thanks for asking the query, Due to trauma from broken tooth you have developed an ulceration on the on the cheek mucosa. I would suggest you to visit the Dentist and you can get the sharp bordres of the wisdom tooth removed by selective grinding,or else take an x-ray of the tooth and you can go for extraction of the tooth. Wisdom tooth does not have nay significant role in mastication and occlussion so its better to get it extracted if causing trouble. Apply Dologel oral ointment topically over the affected area. Maintain a good oral hygiene use antiseptic mouthwash gargles twice daily. Take multivitamin tablets. Hope this helps out. Regards...."
},
{
"id": 128262,
"tgt": "Will copper supplements help with tendinitis?",
"src": "Patient: i suffer from tendionitis in my bicep and sholder does copper supplements help ? do you have any information on causes of this condition through diet or how i can modify my diet to help to treat / prevent this. I am a weight lifter but i suffer from stiff joints and short muscles my e-mail is YYYY@YYYY Doctor: i recommend rest for the part ice application and avoid lifting weights for a week for tendinitis,since you are weight lifter by occupation, wrong position or improper handling or insufficient warm up might cause tendon to get inflamed.once tendinitis develops it is highly recommended to give rest to the part.warm up exercises are thought for prevention of such episodes."
},
{
"id": 53286,
"tgt": "What causes elevated GGT levels?",
"src": "Patient: Hello, I am 30 years old, live in Brazil and more than 10 years living with rates of high GGT and ALT, and they come and go frequently. The GGT reaches 700 and 500 the ALT. Actually I was already in various medical and match any of them can tell what it is! What do you think about it? Doctor: Hi and welcome to Healthcaremagic. Thank you for your query. I am Dr. Rommstein, I understand your concerns and I will try to help you as much as I can.GGT i sliver enzymes usually increased in liver damage, especially alcoholoc liver damage. If other findings bilirubin and AST and ALT are fine then alcohol is most common cause. So the next step is to look for certain liver damage and most common cause is fatty liver disease beside alcohol. It should be confirmed by ultrasound. Other causes are viral hepatitis,cirrhosis, medications, alcohol intake or autoimmune diseases. To verify exact cause, US, bilirubin levels and tumor markers should be additionaly done Then appropriate treatment can be started.I hope I have answered you query. If you have any further questions you can contact us in every time.Kindly regards. Wish you a good health."
},
{
"id": 709,
"tgt": "Can an initial level of sperm count cause delay in pregnancy?",
"src": "Patient: hi doctor i married before 10 months we r not using any precotin from start but still my wife is not become pregnetas per my seaman analysis i have on initial level of sperm count it may be a reason , please guide me.and tell me the proper medical solution Doctor: I am really sorry for your health condition.. Hi and welcome to HCM.. Thank you for posting your query.. I read your question completely and have understood it well.. I being a surgeon, we encounter so many such cases. You say, you have initial level of sperm Count, that means? Anyway, I will guide you what to do.. Generally, we term infertility only after 1 year of unprotected sex between couple. So Cheer up, you still have few more months.. Sperm counts lesser than 48 million per ejaculation as per recent guidelines are called oligopermia. That doesn't mean that, a male with 30 million sperm cannot have children, definitely not. And hence I need the exact sperm count of yours per ejaculation. Well, I will still try to solve your issue. 1. If your counts are more than the number I've mentioned then you need not worry Do these things A. Be positive, positive attitude gives good results B. Make sure, your female partner is normal. C. Have sexual intercourse, around the time of ovulation D. You can use, Tablet Carnisure 500mg twice a day for 2 to 3 months. It helps. 2. If your counts are below the normal. If it's more than 20 million, then you still have pretty good chance. Lesser than 20 million, you need to go ahead with artificial insemination techniques. Hope I have cleared your doubts, in case you need any further assistance let me know.. Take care and God bless."
},
{
"id": 118814,
"tgt": "Checked my blood and it shows Hb, WBC. Feel so tired, have digestion & motion problems. Suggestions for iron rich food, medicament?",
"src": "Patient: dear sir i check my blood and my Hb is 7.9% and wbc is 8100cu.mm and poly morphs is 57% lyphocytes 40% E.S.R - 30 MINS - 17 MM AND 60 MINS - 31MM.SUGAR - 80MGS .SIR MY PROBLEMS IS FEEL SO TIRED WEEK.AND DIGESTION AND MOTION PROBLEMS.KINDLY SUGGEST IRON ENRICHED FOODS AND ANY MEDICINE.CURRENTLY IM TAKING TAB ZELAC B, ZIFI, AULOC RD,T.P.-500. Doctor: hiThanks for your queryi can feel your concernslow haemoglobin is cause of tiredness,weakness and lethargydo have fever or any other infection?ESR is a nonspecific test it can be raised due to anaemia or infectioni advise you to take iron,multivitamin and folic acid supplementsimprove your dietadd meat,green leafy vegetables and fruitsavoid tea and coffeetake vitamin C and citrus fruitswishing him a speedy recoveryHope i have ansewered your queryi am available for your help regardsDr.imran"
},
{
"id": 52184,
"tgt": "Should a liver biopsy be done for elevated liver enzymes and fatty liver?",
"src": "Patient: I am a 63 yr old woman. My doctor has been checking my LFT s for a few months since only my ALT went up to 43 after two months of taking more tylenol then I usually take but not even considered a lot. After staying off Tylenol and alcohol for 3 wks, we retested and my ALT was down to 40 but she recommended a liver ultrasound. Results today show mild fatty liver. I am normal BMI and all other testing on blood for other causes is negative. All of my blood tests were normal in Feb, 2018. I also take 4mg of Testoterone daily and Fosomax 70mg once a week. Have started Fosomax in March, 2018, Testosterone taking for over a year. Should I have a liver biopsy considering there is not a logical reason for a fatty liver? I usually drink 7 drinks a week but am willing to cut back or stop. My doc wants to recheck blood in three months and said I can drink my 7 drinks a week. Thoughts? Doctor: Hi, I don't think that biopsy is required. it would be better to do MRI Scan. Hope I have answered your query. Let me know if I can assist you further. Take care Regards, Dr Iven Romic Rommstein, General Surgeon"
},
{
"id": 124202,
"tgt": "Why is my son having flat feet complain of knee pain?",
"src": "Patient: what type of doctor would you suggest can look at my son 14 years old with flat feet who has been wearing orthotics for over 5 years and is now complaining of knee and ankle pain and is sad he cannot play football anymore due to the pain in the knees. We are based in Dubai and would ideally like to consult with someone in the UAE or India or any great doctor elsewhere. Help please. thank you my email address is YYYY@YYYY Doctor: Hello, With the history, it sounds that due to the usage of orthotics within the shoe for flat feet there is the cause of pain in the knee. This is one of the most common problems people face when they have flat feet. Using orthotics is only when there is walking. When a person is trying for sport the orthotics need to be avoided as it will apply high pressure over the joints of the lower limbs. The pain must have caused due to changing in the biomechanical forces in the joints due to this. What I will suggest as per my experience of 12 years in sports physiotherapy is that, avoid orthotics while playing sports. Use orthotics only when just walking. try doing exercises like lower limb strengthening and balance board rehabilitation. This should give good outcome scores for the patient. Hope I have answered your query. Let me know if I can assist you further. Regards, Jay Indravadan Patel, Physical Therapist or Physiotherapist"
},
{
"id": 50495,
"tgt": "Enlarged kidneys with cortical calcifications and left renal cortical cyst-bosniak. Any advise?",
"src": "Patient: Hi my husband is 68 yrs old he has kub ultrasound last week right kidney measures 12.6 * 5.8 cms with cortical thickness of 1.5 cms left kidney measures 14.1 *5.3 cms with cortical thickness of 2.0 cms. Echo pattern is homogenous.calyces are not dilated. Negative for solid nor fluid filled masses. Few tiny cortical calcifications measuring 2-3mm in diameter are still seen in both kidneys.cortical cyst with wall calcification in the left lower pole now measures 1.4*1.4 cms urinary bladder is well distensible with anechoic lumen. Wall is not thickened no evidence of local mass lesions nor mucosal irregularities seen negative for urinary retention. IMPRESSION:ENLARGED KIDNEYS WITH CORTICAL CALCIFICATIONS AND LEFT RENAL CORTICAL CYST-BOSNIAK 2 UNREMARKABLE URINARY BLADDER. I need some advice our primary doctor says we should see urologist im concern my husband health Doctor: HI Mam I really appreciate your concern, It some time becomes very difficult when some one ask for some advise without clinical complain, just sending a usg report without chief complain wont clear any thing, for what your husband advised for sonography, it is not stated here, your said report is indicative of some geriatric changes until & unless having some clinical symptoms, for better advise please send the clinical history, looking forward Good day"
},
{
"id": 2047,
"tgt": "Suggest remedy for getting pregnant",
"src": "Patient: hello doctor, I have irregular periods. i m trying to concieve from march. last month i ent to doctor. he did folicle test & gave me injection. i have taken duphaston for 10 days. My periods is like to come on 2nd august & my doctor told me to do preg test on 3rd.i did it comes negative. now what next Doctor: Hi I think you should wait for one more week. If periods don't come repeat the test. If it's negative then your periods may come in another 1 week. Hope I have answered your question."
},
{
"id": 209526,
"tgt": "How to cure anxiety and depression?",
"src": "Patient: hai iam 36 years male for past 15 years years igot anxiety and depression at that i was young and dont know what to do .in 2004 imet a psychatric he said me tht iam a petient of anxiety and depression .doctor has adviced me to the tretment .after few years iwas o.k . Doctor: Hello,The question appears to be incomplete but still I think you want to ask if this illness can be cured and what to do in case of relapse.When one drug had been effective in controlling the symptoms initially, then in case the symptoms reemerge on stopping the drug- it is recommeded to strat with a low dose of the previous medicine.Secondly, psychiatric illnesses are consedered to be treatable and not curable- like diabetes or hypertension. Only infectious diseases are consedered to be curable So, it may happen that symptom reappear after drug is stopped.Good luck.Dr. Manisha GopalMD Psychiatry"
},
{
"id": 170554,
"tgt": "Suggest treatment for itchy rash with low grade fever",
"src": "Patient: My 5 yr old daughter started with a rash on her face which in a day spread to her entire body head to finger tips. Within 24 hrs she had common cold symptoms. On the third day took her to her Pediatrician who tested her for strep stating it could be scarlet fever or allergic reaction to soap or lotion. She began feeling itchy so I took her to the hospital where they said it was a prickly heat rash. Suggested I give her benadryl and keep her cool. The rash looks almost like goose bumps that don t go away. Day 5 she has a low grade fever of 100. Please help Doctor: Hi,Welcome to Hcm,I understand your concern but unfortunately without having a look at the rash, just by description a clear cut diagnosis cannot be arrived at. However, description makes me think of possibility of scarlet fever especially with fever coming up. I would suggest you get another consult with a pediatrician and sought treatment. If it looks like scarlet fever then child needs to be started on antibiotic . hope child recovers soon. Take care."
},
{
"id": 170190,
"tgt": "Suggest remedy for persistent vomiting",
"src": "Patient: Well the story goes i bought some of the Organic Milk from Walmart about 2 days ago. My son is 16 months old and i made about 4 bottles out of the jug and he started to throwup badly. I was concerned and inspected the milk and it was sour. I started to give him emtrol and pedyliate and he has only thrown up once since them and about a total of 6 times now. Should i take him tot he emergency room or wait it out Doctor: Hi, Welcome to HCM. I have read your question in detail and I know that you are very concerned about your child but don't worry. The symptoms that you have described are suggestive of food poisoning. In my opinion, you should give syrup ofloxacin to the child along with pedialyte and emtrol. An examination by doctor is a must to look for dehydration and vital monitoring. I hope this will help you. Wishing your child good health. Take care."
},
{
"id": 43104,
"tgt": "What kind of treatment will help me in getting pregnant?",
"src": "Patient: i am saima yousaf from pakistan i am married since 6 years and taking a lot medicines clomid, ivfc injection and puregon inj, also, my all test report are normal, like hsg are normal, period cycle is regular husband reports is also ok but still not conceive, tvs is also normal plz help me Doctor: HiYou seems to be suffering from unexplained infertility. The best option would be to go ahead with testtube bababy or IVF treatment.If you cannot afford, go for IUI treatment.Regards"
},
{
"id": 103299,
"tgt": "Suffering from allergic bronchitis, redness in skin. Applying botonica, no improvement. Suggestions?",
"src": "Patient: Hello sir,last year i had allergic bronchitis and in slow run after it i started oberving my skin is now started developing allergy (redness on my face)i tried n no. Of products but of no cure.Consulted dermatologist where she prescribe me Botonica but that too dint suit me and it causes redness again, m bugged up now and want a proper intervention. Doctor: Welcome to HCM.This is most likely some allergic reaction.You have to find out any allergy your self.It might be due to photo sensitivity after sun exposure or it usually occurs after quinolone group antibiotic.Ask your doctor about your medication and explain your complain.You can apply calamine lotion.Take short course of steroid with levocetrizine.avoid unnecessary sun exposure or cover your face during exposure.Keep in touch with your doctor."
},
{
"id": 137589,
"tgt": "What causes intermittent appearance of a bruise with knot on left forearm?",
"src": "Patient: I noticed a nickel sized bruise on my left forearm about 3 weeks ago. It was only slightly discolored when I noticed it and there also seems to be a small knot under the skin. It hasn t gone away completely and the coloring seems to change. A few days after I noticed it the blue/green tint was gone but it still felt like a normal bruise. The next week the blue/green tint was back and then a few days later it was gone but felt the same. Now the color is bac again. I plan on making an appointment to have it checked, but wanted to make sure I didn t need to do something immediately. Thanks! Doctor: Hi,Thanks for your query.Unexplained bruising once in while is understandable but if occurs frequently should be investigated thoroughly since it can indicate an underlying medical condition or ailment.It has been observed that bruises are often observed more on women than men. Women often complain of bruising on legs, thighs, buttocks and upper arms. Thinner people are more prone to bruising as they lack the fat cushion that helps prevent injury. It may also be due to deficiency of vitamin C, vitamin B12, folic acid and vitamin K.Many times taking medications like aspirin, ibuprofen, non steroidal anti-inflammatory drugs, and even birth control pills increases chances of sudden bruising. Topical and systemic corticosteroids cause skin thinning, making one prone to bruises. Many a time bruising can be caused due to liver ailments, kidney diseases and blood disorder. I do hope that you have found something helpful and I will be glad to answer any further query.Take care"
},
{
"id": 44882,
"tgt": "Does Vitex Agnus Castus 1000mg solve infertility problem ?",
"src": "Patient: infertility since long. dr recommended vitex agnus castus 1000 mg with vitamin b6 100 mg will it work your views Doctor: Hello.Welcome.I can't answer this question as you have not shown your semen report.True modality of your treatment can be decided only after going through your semen analysis report.Good luck."
},
{
"id": 57141,
"tgt": "Is increased AFP in liver cirrhosis patient dangerous?",
"src": "Patient: My father is a Liver cirrhosis patient.It is started 4 years back & it was minor.He used to go to doctor & do all the test.this time doctor said AFP is more(208)& increasing AFP is not good.Please suggest is it a dangerous situation.what&where the best treatment available. Doctor: Hi,How are you? I am sorry to hear about your father's diagnosis of cirrhosis.AFP (alpha feto protein) is a substance normally produced by the fetal liver when it is in the uterus. Once the baby is born, the liver stops producing this substance so that in an adult the levels of AFP in the blood will be very low. Unfortunately, if there is a liver tumour growing, then this tumor starts to produce AFP and hence its level in the blood increases. So if there is a sudden increase in the blood levels of AFP, especially in a cirrhotic patient, it could potentially point to the development of a tumor in the liver. This needs to be evaluated with imaging studies, an ultrasound to start with, and then with a Tri-phasic contrast enhanced CT scan of the liver. The treatment options will depend on size and number of tumors and also the general condition of the patient. The best treatment for patients with cirrhosis and liver tumor is Liver transplant provided they meet some criteria. If the tumor is small, then percutanous ablative techniques or even resection of a small segment of liver in which the tumor is located can also be done.Please do let me know where you are located so I can guide you to the best center for treatment.Hope this helps and hope your dad feels better. Please do not hesitate to contact me for further details - rxsuresh@gmail.com"
},
{
"id": 213359,
"tgt": "Have post partum toxemia after last delivery. Treat kidney infection, high BP and delusion?",
"src": "Patient: I am 35 years old and I have a two month old baby and a six year old daughter who is in first grade. Since the birth of my youngest daughter I have had post partum toxemia and also a severe kidney infection . I have been on high blood pressure medicine and also antibiotics for the kidney infection. I am so tired and forgetful and sometimes I feel disillusional. What could be wrong with me? Doctor: Hi, In sever infection in any part of the body tiredness and abnormal perceptions can occur till infection is relieved, For treatment monitor BP, continue antibiotics, take proper rest, adequate hydration, morning walk and healthy diet will be helpful. If symptoms become severe then immediately visit your physician. I hope this information has been both informative and helpful for you. Wish you Good Health. Regards, Dr. Ashish Mittal www.99doctor.com"
},
{
"id": 192024,
"tgt": "Suggest treatment for type 1 diabetes",
"src": "Patient: My name is Maria Penkowa. I am 22 years-old girl from Bourgas, Bulgaria.For several months I am suffering from diabetes type 1. My daily insulin dose is 11 units \u2013 4times a day. In my case as in each young person this hinders the development of my career, relationships with friends, and depresses me. Recently I was informed that the only way for full recovery is through the implantation of stem cells. Please give me more detailed information concerning the procedures, documentation and possible price for the manipulation in your clinic. Sincerely yours: Maria Doctor: HI, thanks for using healthcare magicStem cell implantation is an experimental procedure at the moment. It is not used wide spread in the treatment of diabetes at the moment.This unfortunately means that it is not likely an option at this time but you would have to speak to your local doctor to find out if there are any centers in your area that are currently experimenting.You may be able to better manage if your regimen in altered to suit your lifestyle.A basal insulin can be used once daily eg lantus, peglispro, detimirShort acting insulin can then be given with your meals. The amount of short acting insulin would depend on the amount of carbohydrates with your meal. If it contains no carbs- no insulin, if it has a high amount would need to enough insulin to cover the meal.This would allow some flexibility with dose and time. You would need to visit a nutritionist to learn to count the carbs.I hope this helps"
},
{
"id": 7064,
"tgt": "My 2nd periods after D&C is delayed and I have had irregular periods always. When can I possibly conceive again ?",
"src": "Patient: Hi, I am 34yrs of age. I had a D C for missed abortion at 9weeks of pregnancy , on 19th July'10. I got my Ist period post D C on 8th Sept. It is 9th Oct today, but am still waiting for my next period. I was on treatment for high prolactin when I conceived and have had irregular cycles since teenage. Should this be a cause for concern as I want to plan pregnancy as soon as possible. Thanks PBS Doctor: Hi, You need not worry as you had periods 4 weeks back, and if have high prolactin level you require treatment as it my leads irregular periods, amenrrhoea and infertility. Take care"
},
{
"id": 142511,
"tgt": "What causes vomiting and vertigo with WBC count of 14.5?",
"src": "Patient: My mother-in-law has a WBC of 14.5. She went in to the hospital last week with severe vomiting from vertigo. She had a UTI (e coli) and some pneumonia from aspirating. She has had a full course of antibiotics and has no fever. She is 92. Prior to the vertigo she could do all ADLs but now she can t even walk or do anything. SHe was to go to rehab but her WBCs are still up. Doctor: Hello!Welcome on Healthcaremagic!Her symptoms seem to be related to an infection. The increased WBC count is indicative of a bacterial infection. Anyway, considering her age, her symptoms (vertigo and vomiting), I would recommend performing a brain CT scan to investigate for a possible stroke.Hope you will find this answer helpful!Kind regards, Dr. Aida"
},
{
"id": 64287,
"tgt": "Suggest treatment for a lump in throat developed during medication for bladder infection",
"src": "Patient: I am 49. I have had a bladder infection and 2 other woman infections. Only sleep with my husband. So not an STD. Had been on 3 different antibiotics for this. The last one was flagyl. I think. Half way thru taking it I developed a lump in my throat. At same time my gut is a mess. Have now been on prilosec for three months and it doesn t seem to be getting better. Could it be candida vs. gerd? Had barium swallow and it showed a little reflux but nothing excessive the doctor said. I think I am not to the bottom of this. I have been misserable now for 3 months. Doctor: Hi,Good Morning.This is Dr.Savaskar from India,attending your query to my online HCM Clinic.-I studied your query in depth and Understood your health concerns.-Accordingly on the facts given by you-Treatment I would suggest is as follows-a-Your query facts are incomplete for the demands put by you.So I would advise you to consult a Surgeon-as the facts given for replies are insufficient to comment-on the type of the lump of throat?which could be due to the oral sex and STD related lumps....Like Gonorrheal Tonsillitis ? LGV?infectious Mono?/ and bladder infection.b-Still-AS your have taken treatments with 3 antibiotics of which-while taking flagyl you developed throat lump.Gynaec-Surgeon-Doctor would treat it after fixing out of Candida or / GERD.I would suggest the oral smear and or a scrapping of the lump to be tested under MICRSOCOPY, for fixing the cause of the throat lump.b-It would need FNAC- Biopsy before advising any specific treatment.FNAC BIOPSY would fix its cause and Gynaec-Surgeon-would treat it according to the causes.YOu have multiple symptoms and hence I would advise you to consult with Gynaec-Surgeon and STD -specialist,who as a team would treat your complex health problems on the lines suggested as above.c-Before treating the - GERD as confirmed from -Barium swallow-I would suggest OGD to confirm it.I would advise Proton-Pump Inhibitors with Domeperidol for the Reflux control.d-Beside this to avoid any further missery-follow normal Sex-practices with your partners with advise of the Gynaec Doctor, here on-wards,as yours is a complex health issue and you need to be serious about it on the lines suggested herewith.Wishing you healthy living and early recovery of this query.Wellcome for more queries till you get the satisfied.Thanks for your query to HCM Clinic.Have a Good Day...!!Dr.SAVASKAR M.N.M.S.GENL-CVTS,Super specialist and Senior Consultant-and Expert in Non-Curable-Disease therapy for Cancer,Asthama,etc,Rejuvenation therapy and Tissue failure -reversal therapies."
},
{
"id": 90700,
"tgt": "What causes stomach pain and blood in stools?",
"src": "Patient: Hi I have been having stomach pain on my left lower side and today have had a fishy smelly light yellow mucus when passing stools. I have had this on and off for the past year. i have had small amounts of blood in this mucus coming from my anus after just passing a stool. My stools vary between soft and very hard which is is pretty frequent. Doctor: Hi.Thanks for your query and an elucidate history.Your history is very classical:1 yearleft power pain fishy smelling mucus and blood in stool.changing bowel habit.I would advise you to undergo a colonoscopy as soon as possible along with ultrasonography to start with and then CT scan if required by your Gastroenterologist. Also needed are routine blood , urine tests; stool - routine, microscopy, staining for bacteria, AFB, cancer cells and occult blood, All this will lead to a proper diagnosis whether this is due to ulcer/ polyp / malignancy or what.The treatment will be best and guided - step by step approach to give you the best results."
},
{
"id": 202367,
"tgt": "Is it normal to have pain after penis masturbating?",
"src": "Patient: Hi Doctor, I'm Ashwin 31 years old. From the past 8 years, I experienced pain in penis after masturbation. After I urinate, the pain reduces slowly. Not all the times it pains; if I don't masturbate more than 3 days and do after that, I experience pain. Does it normal? Doctor: HIThank for asking to HCMI really appreciate your concern looking to the history given here I could say that because of the masturbation there must be trauma to the urethra and this causing pain in my opinion stop the act and better to get done the clinical examination if the symptoms does not improved even after stopping the masturbation, hope this information helps you, have a nice day."
},
{
"id": 160366,
"tgt": "What causes black stool after treated chest and ear infection?",
"src": "Patient: Hi, may I answer your health queries right now ? Please type your query here...my 5 year old girl was with the doctor few days ago ear infection,chest infection,but she wasgetting up black sick and still black stools like tar what would it be?? Doctor: Hello, Your child recently had chest and ear Infections for which she was treated. Now she is heaving black stools. black stools commonly suggest bleeding in stomach. As you said she is getting up black, sick that might be caused by loss of blood in stool. Your child need medical attention. She might need I.V. infusions in case of low B.P. and more tests need to be done as she had chest infection. Hope I have answered your query. Let me know if I can assist you further. Take care Regards, Dr. Shashank Verma"
},
{
"id": 143877,
"tgt": "What causes episode of seizure?",
"src": "Patient: My 74 father had an episode this weekend while out in the yard. Felt nauseous and like he was going to throw up.. Then his eyes rolled back in his head, and head went back. Fell to the ground like a tree ... Not like a collapse with fainting had kind of a moan and foam around his mouth. Doctor: symptoms u r describing are suggestive of complex partial seizures.....considering his age at presentation there should be secondary cause ......most likely structural.....for such seizures...get his evaluation done in form of MRI brain with contrast, EEG, serum Na , K and Ca, blood glucose. start tab levipil regards"
},
{
"id": 158561,
"tgt": "Nipping on labia, dark patch. Can it be a cancer?",
"src": "Patient: hi i am 38 and for a few months now i have noticed a nipping o my labia. on looking i found a fleshy what i thought was a skin tag. Stupidly i pulled it off. It has left an open sore and lloks like more could be coming back..There is also a dark patch which isnt on the other side. I have been looking online and rightenied myself that this is cancer and i have been ignoring it.Im now wooried sick as the doctors arent open till mondayi thought it was just my scar from being cut when i was in labour but its worrying me as its adark patch Doctor: Hi, It is difficult to say without clinical examination. Many a time there are some fungal infection with similar features. You should not worry at all. Dark patch not only be due to cancer.There are other possibilities too. Do not get nervous . Have a gynaecologist's appointment. He/she can guide you better. Nothing to think about cancer right now. There are more common diagnosis to be ruled out first."
},
{
"id": 159333,
"tgt": "Recovering from flu and fever. Started sweating at night. Is this cancer related?",
"src": "Patient: I had recently had a fever and now have the flu but I am slowly recovering from it. And week, recently I go to sleep and I ll wake up in the middle of the night and only my neck is soaking wet all around, this has happened for two nights now. If you could tell me what the problem is, that would be lovely. I did some research and I just really hope it isn t cancer related. Doctor: Hi, cancer is not a common diagnosis, there are many cause of fever. after excluding all the possible diagnosis cancer can be thought. long term fever and night sweat are commonly seen in tuberculosis. it is important to know about your geographical location and occupation history. Consult with your physician.take care."
},
{
"id": 192153,
"tgt": "Suggest treatment for varicocele",
"src": "Patient: Hi Sir, As per Reports of semens test and sonography, i am suffering from Varicocele. my right testis is small than normal size. Thats why my sperm count is low. in first report it was 50.in second report it was 20. Doctor suggest 1-2 month medicin Q-gold , after then surgery if needed. Please suggest the solution. Thanks Regards Umesh Doctor: Hi, Varicocele is not serious condition but it affects mans fertility. Your doctor suggested right thing. Go for surgery at a appropriate time as per surgeons advise. Hope I have answered your queries. Let me know if I can assist you further. Take care Regards, Dr. PRAMOD KOKARE"
},
{
"id": 170315,
"tgt": "Suggest treatment for enursis",
"src": "Patient: I have a son who suffers with enursis. We have used the usual methods of the alarm and desmopressin. It has been suggested to me to look at his diet and foods which he may be intolerant to. Please could you offer some advice on this and am I able to get any food allergy tests carried out anywhere. Thankyou Doctor: Hi.... I have just gone through your query. There is no evidence in medical literature that enuresis is related to food allergy. Any specific food will not induce enuresis in an otherwise normal kid. I do not think for this reason you need to take up a food allergy panel testing. You are already on treatment for enuresis for which you can continue the same. I again reiterate the fact that enuresis is not related to food allergy.Regards - Dr. Sumanth"
},
{
"id": 142469,
"tgt": "Suggest treatment for weakness and numbness in the left arm, face and tongue",
"src": "Patient: Hello, my wife has been dealing with left leg pain and weakness for the past 15 years. Recently my wife developed left arm, face and tongue weakness and numbness. She was diagnosed with a 5mm Chiari malformation and was given steroids to relieve migraines. After seeing a neurosurgeon she was told she was ineligible for surgery and has been told that she may be making up the symptoms in her head. We don t believe that, basically her left leg, arm and now face experiences pain and weakness and we have no idea why. She was tested for MS but with no lesions and multiple side symptoms that was rules out. Any advice you can provide would be appreciated. Doctor: Hello!Thank you for asking on Healthcaremagic!I read carefully your question and understand your concern. Coming to this point, I would recommend performing a cervico-thoracic spine MRI study to exclude possible syringomyelia, which could cause her symptoms. This is a common complication of Chiary Malformation. Anyway, it would not explain the numbness in her face or tongue. Another option would be starting an antidepressant, especially considering the fact tha she suffered from migraines. These drugs can help improve both the migraine and her symptoms. Hope you will find this answer helfpul!Best wishes, Dr. Aida"
},
{
"id": 203770,
"tgt": "What is the remedy for painful and tender penis head when it extendes out of foreskin during sexual act?",
"src": "Patient: I had unprotected sex the other night and during the intercourse the head of my penis enlarged and the foreskin would not retract over the head. This was extremely painful, I cannot be erect without it hurting. When the head extends out of the foreskin it is very tender and painful. This happens with every erection since and I never noticed it \"poking out\" before. Doctor: you need circumcision to solve your problem permanently.please contact surgeon for your treatment and further management"
},
{
"id": 41004,
"tgt": "Suggest treatment for infertility problem",
"src": "Patient: I am 27 yrs old , married and longing to have a baby.I have PCOS. I got a periods on Sept 6,2010. Then on Dec 8th 2010 i jus got very little trace of it . Just 2 - 3 drops when I pee ( this was there for 2days only). Then I had Meprate (progestrone) 10mg to get my periods from 9th - 14th of Jan . I got my periods on 21st jan 2011. But the bleeding is heavy. Today is my 5th day but still i have bleeding.. Can this be an abortion?? i am not experiencing any pains. I am not on any medication for my PCOS from Oct 2010. I stopped all my medicines for PCOS from Oct 2010. Please advise.. Thanks Doctor: The main treatment of PCOS is weight control. So, try it hard. Then we need to check your cetain hormone levels (LH, FSH, Prolactin, TSH) and may need to correct them if needed. After that we need to check other fertility factors of you (tubal patency test) and your partner (semen analysis) and accordingly we can plan treatment. But with weight control, majority of the women conceive."
},
{
"id": 57410,
"tgt": "Digestion problem, fatty liver, elevated liver enzymes, low plasma level, HCV +ve, blood in stool. Acute hepatitis can cause melena?",
"src": "Patient: hi my husband had no thing before in his liver and HCV was -ve just last year. He has some digestion problems and he did parium meal. They discover fatty liver , elevated liver enzymes , low plasma level and HCV +ve. Yesterday he experienced tarry stool with blood clots. I want to ask is acute hepatitis can cause melena ?? we are shocked and can t know what to do. Doctor: Dear Friend.Welcome to HCM. I am Dr Anshul Varshney. I have read your query in detail. I understand your concern.He might be having Acute or Chronic Hepatitis C.Melena can occur due to coagulopathy.Melena can occur in acute or decompensated CLD.He required PT/INR, and might require Fresh Frozen plasma TransfusionThis is my best advice for you with the available details, if you have any further query, please ask us.Stay Healthy"
},
{
"id": 141103,
"tgt": "What causes nerve pain in the legs at night?",
"src": "Patient: I suffered a Tibia/fibula break in a skiing accident 4 weeks ago. To do or not to do surgery was right on the line I elected no surgery and have been in a recliner for all this time with no weight bearing. I return to the orthopedic doctor in 2 more weeks. Recently I have been experiencing the nerves jumping in my legs for most of the night, resulting in lack of sleep and discomfort. Is there anything I can do to help this situation? I am normally an upbeat person but am beginning to feel a little anxious. Doctor: Hi, If only the leg with fracture is affected, symptoms are related to the local damage (swelling, changes in blood circulation, etc.). If both legs are affected, you should be checked about spinal injuries, since you had a skiing accident. At least, an x-ray of your spine may give valuable information, even to exclude any traumatic damage of your spine. Discuss with your Doctor about these issues. Hope I have answered your query. Let me know if I can assist you further."
},
{
"id": 212115,
"tgt": "Teenager. Sensation of pins and needles on arms, hands and legs. Fasting blood test clear. History of poor posture, smelling dirty nappies and burning them. Advice",
"src": "Patient: Dear Doctor, My fourteen year old daughter has for the past three weeks reported the sensation of pins and needles in both her arms, hands and legs. My husband took her to our GP as we thought she may have a trapped nerve. We've in the past paid for chiropractic treatment to help with her poor posture. The GP requested a check of her bloods with a fasting blood test and I'm releived to report that these are clear. My daughter has since reported periodically smelling dirty nappies nad burning, often asking friends and family if they can smell this. The GP has asked that we keep a diary for two weeks. We haven't made her aware of the these smalls so I will do that shortly. Any advice would be very much appreciated. Kind regards Claire Cooper Doctor: HelloThe symptoms like pin needle sensation in hands and legs can occur due to a number of conditions. These symptoms may be seen in low Vitamin B12 levels or in low thyroid hormone levels. Please get these investigations done to rule out these conditions.If everything come normal then we can think of some neuritis or nerve entrapment but as she is also complaining of abnormal smells so the possibility of these conditions are less.Such symptoms may occur due to some anxiety disorder but have a close watch on her symptoms because if she show symptoms of suspiciousness and problem of abnormal smells persist then consult a psychiatrist for evaluation of hallucination or any psychotic disorder if any. Thanks"
},
{
"id": 206631,
"tgt": "What causes short breathing, shaking and sweathing during stress?",
"src": "Patient: My friend, when upset, falls down, clutching his chest, breathing in rapid, short breaths. After the episode, which can last from half a minute to a couple minutes, he is shaking, sweating and noticeably much weaker. A warm shower seems to help him recover, but what else can I do ? Doctor: Hi , thank you for using health care magic.I can understand your concern about your friend's health.From the symptoms you describe it seem your friend suffer from anxiety disorder.But there are some other possibilities need to be rule out before going forward. If your friend loose consciousness or have incontinence of urine or stool during episode ,tongue bite, then better to consult a doctor because it can be a convulsion. Anxiety disorder is treated with medicine and psychotherapy , medicine like sertraline peroxetine will help in reducing this kind of episode.Your friend also need to be taught some relaxation exercise like deep breathing and muscle relaxation , this method when applied while having stress reduce the episode. Hope this will help you.Regards."
},
{
"id": 156307,
"tgt": "Is BPT and aloe arborescens effective for cancers caused by pathogenic microorganisms?",
"src": "Patient: Is Dr Isaac Goiz Duran biomagnetic pair therapy an effective therapy for cancers caused by pathogenic microorganisms? The same question for the aloe arborescens. And How more effective would they be if applied at the same time. I will greatly appreciate your profesional input. Thank you very much. Doctor: treatment is allopathy. must be supported by randomised trials. no data is of any value without scientific evidence. they are no randomised trials that are practise changing on the herbs you mentioned. they might have medicinal value but are of no use in cancer if used alone"
},
{
"id": 147304,
"tgt": "What causes regular headaches, blurred vision and loss of vision with seizures?",
"src": "Patient: my friend's CT scan report states - calcified granuloma in left occipital peri lesion edema. she have symptoms of sezuries [thrice ] ,regular headache , blurred vision and loss of vision for few seconds 3 days back .what is her diagnosis , is it a life threatning problem ? how can it be cured ? is sugery recommended. Doctor: HIThank for asking to HCMI really appreciate your concern for your friend looking to the history given here for your friend the CT report is not confirmative of any disease even the given report is also not clear in the sense in my opinion she must be in need to EEG and MRI test these tests could clear something, I hope this information may help you, have a nice day."
},
{
"id": 1931,
"tgt": "Are there pregnancy chances without ejaculating inside?",
"src": "Patient: hi. um, i want to know if i can get pregnant, my last period started on july 27th. my boyfriend and i had sex today, and he didn't ejaculate inside of me or even came close to orgasm, i asked him to pull out waay before. is there still a possibility to get pregnant? or am i panicking for nothing? Doctor: Hi I think there is no chance of pregnancy if there was no preejaculate or ejaculate. So don't worry."
},
{
"id": 78729,
"tgt": "What causes breathing difficulty while raising hands?",
"src": "Patient: Hello, for the past couple of days now, i ve been having breathing complications whenever i would lift my hands above my head. It would seem as if my throat closes up, and my airway is cut off. i begin to chock and cough until i put my hands down by side. could this be something serious? Doctor: Thanks for your question on Health Care Magic. I can understand your concern. Cough with breathlessness are commonly seen in bronchitis and lung infection. Since your chest x ray is normal, no need to worry for lung infection. Possibility of bronchitis is more in your case. So better to consult pulmonologist and get done clinical examination of respiratory system and PFT (Pulmonary Function Test). PFT is needed for the diagnosis of bronchitis. It will also tell you about severity of the disease and treatment of bronchitis is based on severity only. You may need inhaled bronchodilators and inhaled corticosteroid (ICS). Don't worry, you will be alright. Hope I have solved your query. Wish you good health. Thanks."
},
{
"id": 202639,
"tgt": "What is the cause of ejaculating red semen?",
"src": "Patient: Hello docs n experts, Since yesterday I m ejaculating red semen. I noticed it yesterday while masturbating, even today I seen same thing when I was trying to confirm was it a normal one day case or a serious issue. Now I think it is a serious problem for me. Please help me. Doctor: HIThank for asking to HCMI really appreciate your problem, what the history you gave you here of red semen the possibility of blood stain semen could be there that might be happening due to infection or trauma to the system(From urethra to testicle region) this need to be investigated and the first test would be complete urine test, x-ray, and scanning study, hope this information helps you, have nice day."
},
{
"id": 117074,
"tgt": "What causes elevated blood counts?",
"src": "Patient: my daughter cbc came back showing high wbc 15.4, absolute monocytes high and absolute neturophil. He prescribed augmentin 875mg bid for 14 days she had to stop taking after 7 days,as it gave her serve stomache pain. He the prescribe keflex for 10 days. She completed it and he ran another cbc her WBC still high 13.7 along with her absolute monocytes and absolute neturophil. She has really not been feeling well for a couple of months. Getting pains in her chest along with pressure EKG normal, pain in her back near her kidneys, these pains are everyday. She is also have frequent upset stomache. Visited ER yesterday as she has really just not feeling well. Thought may her gallbladder ultrasound should nothing. Still has elavated blood counts. Seeing Hematologist on tuesday as per MD recommendations. Doctor: Hi, dear I have gone through your question. I can understand your concern. You have some bacterial infection. It is the most common cause of high wbc count and high neutrophil count. She may have some hidden foci of infection in her body. search for that and take treatment accordingly.Hope I have answered your question. Thanks for using health care magic.Wish you a very good health."
},
{
"id": 215099,
"tgt": "What could be the reason of having a high body temperature ?",
"src": "Patient: hi i am rishi 25 yrs old, male. weight is 65kg. my body doesnt have normal temperature. little more, in all time. if i eat egg a pox is coming on my eye lid. if i break this pus is coming. and not feeling pain. this is rarely only. i dont have bp, diabetics, thyroid. though what is the reason for this. if u can help me i will be thankfull Doctor: Dear Rishi,you are absolutely fine.This is skin condition in which due to some reasons your skin pores gets blocked and you get infection there.Take advise of your family doctor and you will be fine.Avoid eating too much of oily foods specially junk foods.bye and take care."
},
{
"id": 144192,
"tgt": "Suggest medication for hearing problem and having been diagonised with bell s palsy",
"src": "Patient: What is the average length of time off from work for bells palsy? I was diagnosed 6/11 & I see very slight improvement. I was prescribed 20 mg prednisone, 2 a day for 5 days,800 ml acyclovir, 5 a day for 7 days & 600 mg ibuprofen for pain, but my effected side (right) is having hearing problems ( feels like something is moving in my ear & popping sounds) have gotten worst & I have heavy pressure besides the paralysis & vertigo. Doctor: Hi, I am Dr.Bruno. I have read your question and understand your concerns. Let me try to help you Bell's Palsy usually refers to Lesion of the Facial Nerve (7th Cranial Nerve) outside the Skull But, in your case, you say that you also have also Hearing Problems +Vertigo. These are rare in Bell's Palsy and more common when the Facial Nerve (7th Cranial Nerve) is affected inside the Skull and the 8th Cranial Nerve also is affected I would suggest you to consult a Neurosurgeon at once Hope you found the answer helpful.If you need any clarification / have doubts / have additional questions / have follow up questions, then please do not hesitate in asking again. I will be happy to answer your questions.Let me know if I can assist you further.Take care."
},
{
"id": 32895,
"tgt": "Suggest treatment to get rid of shingles infection",
"src": "Patient: I am 52 years and was diagnosed on Tuesday with Shingles. After receiving multiple prescriptions, I asked about getting the vaccine to prevent another bout of this. My inquiry was met with, You are not sixty years old so no use in discussing it . If this would or will help me why not? This is the worst pain I have ever felt, doesn t go away, just constant severe pain. Thank You, Doctor: Dear Sir/Ma'am, Hi & Welcome.I appreciate your concern for the treatment of Shingles infection and the pain following the infection.Though most people experience only one episode of shingles during their lifetime. To prevent the recurrent episodes of shingles, individuals can receive the zoster vaccine (Zostavax) however this vaccine is not effective in relieving the severe pain following the infection.For the severe and sharp pain that occurs following the infection \"post herpetic neuralgia\" medicines like Gabapentine or Pregablin are effective.Hope to have answered your query.A feed back is appreciated."
},
{
"id": 22381,
"tgt": "Is it ok to miss blood pressure tablets?",
"src": "Patient: i forgot to take my blood pressure meds this morning and can't leave work to go get them-i will not be able to take it until 4pm-is there anything i can do to prevent any possible problems? I'm 51, 5'2\" weight is 183 and i take 10 mg lisinopril daily. Doctor: Hello, It's not wise to miss the tablets because Your BP may rise and may create lots of problem. So better not to forget or you can buy a strip from local shop. You should get your BP checked in such instances as increased BP is asymptomatic but dangerous. Anyway You should have a healthy lifestyle like avoiding fatty, oily and high calorie diet. Have low salt diet and monitor blood pressure regularly thrice a day for one week then once or twice a week. Regular exercises like brisk walking, jogging according your capacity at least 30 min a day and 5 days a week. Eat lots of green leafy vegetables, fruits, fish once or twice a week, avoid meat. Avoid smoking and alcohol if any. There shouldn't abdominal fat deposition or obesity. Get your lipid profile and sugars tested once.Hope I have answered your query. Let me know if I can assist you further. Regards,Dr Sagar Makode"
},
{
"id": 131393,
"tgt": "Suggest treatment for chills and muscle pains",
"src": "Patient: over the past 2 months my son has experienced getting a quick on set of chills and muscle pains - it seems to only last about a day - but it has occurred about 5 times. I asked him if he had a fever but he hadn t taken his temperature so he wasn t sure. He works out often I was wondering if it is maybe some mineral deficiency etc. he eats well and drinks a lot of water but I thought the muscle pains might be the need for some potassium etc. Doctor: hello I have studied your case and I think that sells and fatigue is not possible due to deficiency of any mineral only cause I can think of is due to deficiency of calcium and sometime D3 I would suggest you to check Vitamin D3 and Vitamin B12 level of your son this will confirm if he is having any mineral deficiency thanks"
},
{
"id": 13682,
"tgt": "What causes red heat rash on the body?",
"src": "Patient: Thank you, yes. I have a red heat rash (I think that is what it is) which deveoloped in June when I wore an old swimsuit to the beach and returned thinking when I took off the suit (two hours later) that it was sunburn because I had neglected to put sunscreen on the unexposed (I thought) area around my navel, near my hips and around the panty line area above my thighs. My question is since the rash has not gone away, could it be a skin infection instead? Doctor: Hi,If you have a sore and it oozes out fluid or plus, then it could be a sign of infection. Also, infection is associated with fever and fatigue. I recommend you to consult your Dermatologist for ruling out infection and to get treated.Hope I have answered your query. Let me know if I can assist you further."
},
{
"id": 90678,
"tgt": "What causes abdominal pain after uterus removal?",
"src": "Patient: Hi Sir, My mother is 52 year old, last march 2011 she had a operation of uterus in dehradun, and the doctor removed uterus. now she had still pain in stomach, while the Ultrasound and X-ray KUV reports are fine. what should i do please suggest me and we did the thyroid test in which her STH is high may be due to that fat is incresing especially in her stomach. she is taking the tablet for thyroid but the fat is not in control. We are not able to find out the reason of the pain.I would really thankful if you please suggest me the test which i should go, or the specialist of this field RegardsPavan Doctor: Hi Pawan,Welcome to HCM.If usg and kub xray are normal then pain could be due to postoperative adhesions.For that you need to show to a laparoscopic surgeon for a diagnostic laparoscopy and adhesinolysis if adhesions are found.But before that her thyroid has to get normal. And for that you need to repeat the TSH levels every two months to adjust the dose.Hope i have been helpful.RegardsDr. Ashish Verma"
},
{
"id": 41578,
"tgt": "Suggest treatment for infertility",
"src": "Patient: Respected Sir, We are married for last 2 and half years and live at Ahmedabad.We are trying for conceiving and took treatment of infertility since then,but no positive outcome.All test came OK including semen test and HSG test. We came to delhi (our home town) and as per Gynac advise, we took D&C and PCR test and found PCR test +ve(NONTUBERCOLOUS MYCOBACTERIUM detected POSITIVE). Dr. Animesh Arya (MD medicine and speciliast in Tuberculousis diseases has advised wife 3 month medicine course for treatment of NTM of uterus. 2. Please advise the success rate of pregnancy after treatment of NTM of uterus. What precuations we need to take during treatment?(Can we make relation or we need to use protection)? Is there SPECIAL DIET regime to be followed ? 3. Please advise Sir. Thanks & Regards Doctor: hellonon tuberculous myobacterium detected in menstrual blood have become common these days and with effective treatment it can be completely cured.Though I personally suggest the course of treatment to be 6 months.You can keep relations during the period but you should use protection as conception is not advised when under treatment.As such there is no specific diet instructions but you should follow a healthy diet and lifestyle.In case of being overweight you should try to loose some weight as it helps in conception.People usually conceive within 6 months to 1 year after treatment.Please keep in touch with your local doctor."
},
{
"id": 64755,
"tgt": "What causes swelling on one side of the neck?",
"src": "Patient: I have a swelling on one side of my neck gland when I eat a meal. Hopefully, I am having a CAT scan tomorrow but I would like some peace of mind for the next 24 hours. With this problem, does it always lead to an operation? I am going overseas in two weeks time for a five week period. I am 63 year old woman with up till now perfect health. I take no medication. Doctor: Hi,DEar,thanks for your query to HCM.I studied your query in depth.1-In My opinion the swelling on one side of neck gland-thyroid, especially when you eat,is due to -Sialeadenitis with Sialolithiasis with stone in duct obstructing the submandibular gland,.2-This may need urgent surgery -as it increases with eating a meal , indicating that the swelling is due to the stone obstructing salivary duct of the submandibular gland.3-If you are going out-station-better have a -on-travel -health insurance to cover any emergency -which may need Urgent Surgery if need be.4-Hope this would solve your query.5-Wellcome for any more questions on new or this subject also...Have A Good Day..!!"
},
{
"id": 7217,
"tgt": "I am on birth control pill but i am not getting my period, why ?",
"src": "Patient: i finished my 21 tablet then its my 4th pill free days but still i dont have my period yet, we had unprotected sex on my 1st and 2nd pill free days.am i pregnant? Doctor: welcome to healthcaremagic if you have taken the pills regularly ,chance of pregnancy is usually not there may be you will have periods in 2 to 3 days if you miss the period go for pregnancy test from morning sample of urine and consult gynecologist"
},
{
"id": 77687,
"tgt": "Suggest treatment for chest congestion , nasal congestion and shortness of breath",
"src": "Patient: Yes I have head chest and nasal congestion the first day I had no chest congestion the next day I woke up my chest is in a lot of pain I cannot hardly breathe what I cough up is neon green from the chest congestion even when I m blowing my nose can you tell me what sickness can cause this and what I need to get to get over the sickness Doctor: Thanks for your question on Health Care Magic. I can understand your concern. In my opinion, you are mostly having post infectious bronchitis. It is common after viral upper respiratory tract infection (URTI). So your nasal congestion is due to viral URTI and Chest congestion, breathing difficulty, green sputum etc are mostly due to bronchitis. So better to consult pulmonologist and get done clinical examination of respiratory system and PFT(Pulmonary Function Test). PFT will also tell you about severity of the disease and treatment of bronchitis is based on severity only. You may need inhaled bronchodilators and inhaled corticosteroid (ICS). Antibiotic and antihistamine drugs are also helpful. So consult pulmonologist and discuss all these. Don't worry, you will be alright. Hope I have solved your query. I will be happy to help you further. Wish you good health. Thanks."
},
{
"id": 32853,
"tgt": "Suggest treatment for bugs inside body",
"src": "Patient: hello, i have a friend of the family that has an issue with bugs inside their body. Mostly coming from inside her eyes and anus. They are small specs, that you can not see with the regular eye. Do you have any ideas what could be the cause and how to cure? Doctor: Hello,Welcome to Health Care Magic. I have gone through your query and I understand your concern. In order to be more helpful for your friends case, I would need more details, like for example the symptoms your friend is suffering from. As a first impression from the details you are giving, I would suspect for a loa loa filariasis, but, I cannot give any diagnosis or treatment without other details. I would recommend that your friend visits his/her local infectionist, in order to have a clear diagnosis. After that I could help you with an option of treatment. If the disease I mentioned above is diagnosed, it could be treated with the drug diethylcarbamazine (DEC), and when appropriate, surgical methods may be employed to remove adult worms from the conjunct.Hope I answered to your question. Take care.Kind Regards,Dr. Ina Zaimi"
},
{
"id": 161045,
"tgt": "What causes prominent boney lump formation outside the leg?",
"src": "Patient: My daughter, age 5, has a prominent boney lump on the outside of her leg (at the top of her shin bone). I think there is one on her other leg but not as obvious. She is very slight and her knee joints look big but I put this down to her skinny legs. Are these lumps normal? Doctor: Hello, It could be an osteoma. However other causes must be ruled out. Consult an orthopaedic and get evaluated. An X-ray is required for further assessment. Hope I have answered your query. Let me know if I can assist you further. Take care Regards, Dr. Shinas Hussain, General & Family Physician"
},
{
"id": 57216,
"tgt": "What causes fatty liver?",
"src": "Patient: yes I was having pressure up the right side of my throat and pain in my liver area so I had an ultra sound. I was told I have a mild fatty liver, it does not seem to be a mild case with the itchy skin etc.. But Im not wishing the worse. Also my blood work was fine liver enzymes etc. Doctor: Hi,Fatty liver can be caused by excessive alcohol intake (Alcoholic steato hepatitis, ASH) or due to excessive fatty food intake and sedentary life style (Non-alcoholic steato hepatitis, NASH). ASH is usually the beginning stages of alcohol induced liver damage and is completely reversible if the patient refrains from alcohol. NASH is being much more commonly diagnosed currently owing to poor dietary habits and sedentary life styles of people. High caloric intake, high fatty food intake and poor exercise regimens lead to deposition of fat goblets in the liver tissue leading to fatty liver. This is also completely reversible in the initial stages if caloric intake is limited and exercise regimens are adhered to. If left un-checked, patients usually become obese, diabetic and in final stages may lead to cirrhosis of the liver which will necessitate a liver transplant."
},
{
"id": 180778,
"tgt": "What causes pain in the molar gums?",
"src": "Patient: Good morning Dr. Andrew. I am Deborah. Recently I had some pains in my molar gum, I went to see the dentist and was given some anti-biotic Flucamox for 5 days. but to my worry on the fifth day, very smelly mucus started coming from my right nose and later I realized must be pus. it is now 5 days and I am getting worried what abt what to do. pls advise Doctor: Hello,It looks like\u00a0sinus infection\u00a0causing the discharge of infected fluid from the nose. As the sinuses drain through the nose, any infectious fluid accumulation in the sinuses can come out through the nose.It can also cause pain in the upper jaw, gums, and teeth as the sinus lies in close proximity with upper jaw teeth. So my suggestion is to once get a sinus x-ray done to confirm a sinus infection.If the sinus infection is ruled out, you advised another course of antibiotics like\u00a0Augmentin\u00a0and along with it, anti-inflammatory painkiller like Advil advised. Decongestant\u00a0nasal spray\u00a0like\u00a0Oxymetazoline\u00a0and saline\u00a0nasal irrigation\u00a0advised. Do steam inhalation.Hope I have answered your query. Let me know if I can assist you further.Regards,\u00a0Dr. Honey Arora"
},
{
"id": 122615,
"tgt": "Suggest remedy for tightening of calf muscles",
"src": "Patient: I played basketball for an hour but then we had a break for 20 min while we listened to a speaker. I presumed playing and then started to feel my calf muscle tighten and I stopped. Not sure if this is a cramp or something else? Calf is still tight after playing 2 hours ago and had iced my calf. Doctor: Hello, The symptoms seem to be related to overuse of the muscles. I suggest using anti-inflammatory medication such as Ibuprofen. I also suggest continuing using cold compresses in the area of pain. I recommend resting the muscles for a few days. Hope I have answered your query. Let me know if I can assist you further. Regards, Dr. Dorina Gurabardhi, General & Family Physician"
},
{
"id": 93098,
"tgt": "Abdominal pain, had unprotected sex, white discharge, chills, weight loss. What is wrong?",
"src": "Patient: Hi, I had a question that been in mind a couple days now. So first of all on June 25th my right adomin hurts. Not for long it feels like a stabe. One at a time. I dont know if this mean i was ovalating or not but when i count its the 14 or 15 days before i start my period. Anyways me and my bf had sex that night which is the 25th. He yes came in me and after that i lay down and fell asleep. Next morning when i got up and went to pee and wipe there was like a lump of white dischager. It not super white kind the white tan i dont know how to explain it. I didnt touch it just flusj it down. Two day later i feel chiles but not sick. And about 7 days after i feel tired but now im ok. Also i having been eating lately and notice that i had lost weight all my old shirt fits me now but my jeans are not fit prefectly now. What wrong me? Can i be pregnat? Doctor: Hello,There is always a chance of pregnancy with unprotected sex during the unsafe period of the menstrual cycle.Get a Urine pregnancy test done to confirm it.If pregnancy is excluded then we can think in term of PID(Pelvic inflammatory disease).You may need a Viginal swab and ultrasound of the pelvic for diagnosis."
},
{
"id": 33208,
"tgt": "What causes itchy, red colored,swollen lymph nodes on the armpit?",
"src": "Patient: I have had recent swollen lymph nodes in my arm pit monthly the week before and during my menstrual cycle. My doctor seems to think that this is normal due to the increased bodily fluid and hormone activity. The swelling is red, and very painful; when my cycle is over, it subsides until the next month. It generally starts out itching badly and progresses to red and painful. Is this normal? I'm 5'7\", 150#, 43 y/o, basically healthy, I have a great deal of stress at the moment, I'm prone to cysts, foliculitis, and benign tumors. Doctor: Thank you for sharing your query on Healthcaremagic, I have carefully worked through your case and of course is not normal you probably have an Hidradenitis that is an inflammation of sweat glands, as you do not do the right treatment the process persist( hidden ) and when you have your period is exacerbated with the hormonal changes, I think you need to take antibiotics and some OTC ( anti-inflammatory and pain killers) treatment, another thing I advice you to aply a light deodorant ( no grease ) wash your armpits 2-3 times a day ( you can use wipes) because doing this you are avoiding the obstruction in your glands ducts. I hope that my advice has been helpful, take care"
},
{
"id": 197765,
"tgt": "Why my nipples are inflamed and sore?",
"src": "Patient: Quick question.. I am a 19 year old male and I'm healthy. Both of my nipples are inflamed and sore. I was just wondering what this could be. I know it's probably nothing serious. Oh and no one in my family has had cancer of any sort so I know it's not that. Doctor: Hi,It seems that this problem might be due to some allergy to synthetic bra or ill fitted bra or friction of bra with nipples producing some irritation, soreness and some infection on the part.There might be having possibility of having yeast infection or some local skin condition like dermatitis.Clean local part and apply antibiotic cream for 2-3 days.Wear cotton and well fitted bra.Ok and take care."
},
{
"id": 132447,
"tgt": "What causes burning in the knees and the treatment for the same?",
"src": "Patient: Hello I have been suffering from knee burning since beginning of 2013. I can tell you it is not pain, it feel like burning and I feel bad when I move. I am working long time for Sainsbury s for more than 5 years. For this job I need to run all the time and need to stand for long time. My question is where can I go for proper diagnosis. I want to know my disease. Could you help me please. Doctor: you first need to do an x-ray you need to rule out if it is osteoarthritis then do esr , crp to see if any inflammation +nt or not then you may also do uric acid and rheumatic fector to rule out these disease"
},
{
"id": 107939,
"tgt": "What causes back pain with history of polio?",
"src": "Patient: I had Polio when I was young I m now 70 and have been going through a lot of pain in the back my right side had an operation on my back shots nothing helps when I had polio it was from the waist down and this is where all this pain is coming from I m on oxicoden for a long time and I m still going through this pain I m so tired of pain and hurting I don t know what to do and nothing seems to help I don t think any one understands this polio business it is always over looked on me. Doctor: polio causes severe weakness of the muscle which are helpful in walking and making posture . due to weakness of these muscles more strains develops on the spine to correct posture and it causes pain. there may be advanced grade degenerative changes in your spine . you should start with hot fomentation of back and back strengthening exercises with help of you physiotherapist. it will give you relief from pain and may decrease dose of analgesics.Hope this answers your question. If you have additional questions or follow up questions then please do not hesitate in writing to us. I will be happy to answer your questions. Wishing you good health."
},
{
"id": 98268,
"tgt": "Do symptoms such as sneezing, clogged nose and itchiness in the ears and nose indicate any type of allergy?",
"src": "Patient: hello i really bad allergies, i get what i call an allergy attack. Meaning all out of nowhere i start sneezing uncontrollably, my nose gets completely clogged to where no air can get through, my eyes get very itchy and so do my ears, and i start to wheeze while breathing. my face recently started swelling up during this attack. I have no idea why or what starts it. It comes at any time of the day. Doctor: HelloAs you explain the situation yes these are signs of allergy for sureFirst you can take antihistamines such as Telfast orally twice for some days and it will improve your simptoms but for longer follow up you need to see an alergologist specialistRegardsDr Jolanda"
},
{
"id": 42826,
"tgt": "What does the ultrasonography of the follicular study indicate?",
"src": "Patient: Dear sir, my ultrasonography of follicular study reports DAY OF CYCLE RIGHT OVARY LEFT OVARY ENDOMETRIUM THICKNESS 10th day 12x10mm 11x08mm 06mm12th day 12x11mm 15x09mm 6.7mm14th day 12x07mm 17x12mm 7.3mm16th day 12x07mm 10x06mm 11mm minimal free fluid is seen in POD. pregnancy & not pregnancy . my reports. Doctor: Hi,Thanks for writing to HCM .According to your report your right ovary follicle has not grown properly and not ruptured. But your left ovary follicle has grown properly and ruptured. With this report we cant tell you are pregnant or not because if you conceive also it won't be detectable by scan till 5 wks. Your just 16th day. The first sign of pregnancy will be increase in serum b-hcg . It is positive after 12 to 15 days of conception. After 15 to 18days that b-hcg seen in urine and your pregnancy kit will come positive..So you have to wait for 2 wks to know the result. Hope I have been helpful .RegardsDr.Deepika Patil"
},
{
"id": 36351,
"tgt": "Suggest treatment for swollen, painful neck lymph node",
"src": "Patient: Over the past two days I noticed my lymph node on the right side of my neck is swollen and hurts to touch. It hurts to swallow yawn and cough. Also my urine has a strong smell to it. It has no burn when goin however I tried a uti medicine still it was an OTC. should I go to the emergency room? Doctor: Hello ,I understand your concern. I am Dr. Arun Tank, infectious disease specialist, answering your concern.FNAC of the lymph node will guide you for further treatment. It might be possible to have TB in you.TB can cause such symptoms mentioned above. Please do not take OTC drugs because it can cause resistant in TB bacilli.Please do drug sensitivity and culture testing in you. once this report is available you can get perfect treatment. Please complete the TB treatment otherwise it will cause resistant in the TB bacteria.I will be happy to answer your further concern you can contact me here or you can contact me on bit.ly/DrArunWe wish you a best health at healthcare magic. Thank you,Dr. Arun Tank"
},
{
"id": 183431,
"tgt": "What causes a painful hard white spot with ear and neck pain?",
"src": "Patient: When I was 22 I had 3 wisdom teeth pulled.They said I only had 3. Well I am almost 40, is it possible to have that 4th wisdom tooth coming in now? Or is this pain with a white spot that is hard and hurts when touched, also ear and neck pain something else. Thank you! Doctor: Thanks for your query, I have gone through your query.The pain over the white spot could be because of the erupting wisdom tooth or it can be because of the trauma from the opposing tooth. Nothing worry, consult a oral physician and get a radiograph done to rule out any wisdom tooth. If there is a wisdom tooth, get it removed. If there is pressure from the opposing tooth, get it rounded off.I hope my answer will help you, take care."
},
{
"id": 39011,
"tgt": "What is the permanent cure for fungal type infection in groin area other than Quadriderm ?",
"src": "Patient: im having some fungal type infection in my groin area for the last 2 months.. initiallly i tried Quadriderm cream... It worked... But it has again reoccured and the condition has worsened... It has now spread to my belly region .... i have tried various combinations of clotrimazole, beclamethasone and neomycin but to no avail.. now the area appears slightly swollen with a typical ring pattern.... Im a Veterinarian.... So i tried these combiantion of drugs.... Im nt getting cured.... Plz advise. Doctor: Hi,Welcome to HCM.I understand your concern regarding recurrence of ringworm infection.The most common causes of recurring fungal infection include inadequate treatment, not maintaining personal hygiene and increased sweating in those areas which predispose to repeated infection.I am also suspecting antifungal resistance in your case which could be the reason for recurrence.I suggest you undergo a fungal culture and sensitivity for a skin sample from the groin area and get a prescription for an appropriate antifungal based on the test results. This treatment has to be continued till 6 weeks at least to avoid recurrence.Keep those areas clean and dry and maintenance of personal hygiene is a must.Avoid tight underwear and clothes to decrease the sweating in those regions.Thanks."
},
{
"id": 63903,
"tgt": "What causes lump in arms?",
"src": "Patient: I got a swollen vein this past tuesday. I didn't go to the doctor I just put ice on it and thought the swelling would go down. Well it did but it made my arm worst then it was. I now have a huge dark bruise on my left arm. When I touch it I can feel a lump and it hurts. I want to know whay is causing this bruise to not go away... Doctor: i,Dear,Good Morning and Thanks for your query to my virtual Clinic At HCM.I studied your query in depth and understood your concerns .Causes for the lumps in the arms are as follows-a-Mostly they are-Thrombophlebitis with bruise in left arm.Dont worry of them,as they could be relieved with proper treatment.-Treatment -a- Tab-NDAIDs/b-Hot fomentations,c-Rest and Elevation of left armc-Antibiotics if does not respond in a day to earlier treatment. d-Surgeon consultation and needful further treatment with him is advisablee-To remove that lump-I would advise Thromphobe oint locally x 1 wks time.Hope above reply would help you to relieve of the issue you have.Welcome for any further query to ME and HCM.Write Good review for the benefit of my new patients.Click thanks ASAP after this.Have A Good Day.With RegardsDr.Savaskar"
},
{
"id": 41840,
"tgt": "Should IUI be delayed until TSH is normal?",
"src": "Patient: Hello, my tsh is at a 4.21 and I am doing IUI procedure to get pregnant. Should I wait to try until my TSH is below a 2? I am 29 years old, 5.4 and weigh 175 lbs. I have normal periods and ovulate (I tracked it twice - April and July using a OPK) other than the hyperthyroid issue which was a 5.98 when the blood work was read originally I have a clean bill of health. Doctor: yes if we can wait that is always better.Dr. Prerna Gupta MBBS (AIIMS), MD (AIIMS), (Gold Medalist)DNB, MNAMS, MRCOG (UK)Commonwealth Fellow in Reproductive Medicine (BCRM, UK)consultant Moolchand prernagadodia@gmail.com"
},
{
"id": 192064,
"tgt": "What causes low blood sugar levels followed by dizziness?",
"src": "Patient: My mother had a problem last night and her blood sugar had dropped to 40. The medics had gotten her back to normal and we got her a hearty meal to get some real sugar. Now this morning she felt a little dizzy and her blood sugar is up to 360. She does have high blood pressure and is 52 years old. Is this a cause for concern? Doctor: HIWelcome to HCMI am Dr B M GUPTA answering youThe cause of Blood Sugar only 40 mg /dl would beEither she took double dose or on fastingI advise- Blood for HbA1c .The cause for blood sugar 360 mg/dl heavy sugar given Do not worryKeep B P atTake careYour mother is at risk -hypertention plus diabetes plus menopause consult your doctorcheck life stylethanks"
},
{
"id": 36661,
"tgt": "What are the side effects of insect bite?",
"src": "Patient: my 2 yearold had, about 4 days ago, what looked like typical uk midge/gnat insect bites, 2 on one hand and one on his forhead, after a few days they have got smaller but developed a red ring around them. they dont seem to be irritating him.... any ideas? Doctor: Hi,It seems that this is local allergic reaction due to insect bite.Give antihistamine medicine like Benadryl or Cetrizine for 2-3 days.Apply triple actioncream locally.Ok and take care."
},
{
"id": 155251,
"tgt": "Is there any alternative medicines other than levitra for prostrate cancer?",
"src": "Patient: As a result of prostrate cancer treatment (bracheotherapy) I have had a prescription for levitra to assist me when needed. It gives me headaches so I use about 1/3 of a 20 mg tablet. Lately, it does not work as well. Is there something else that would be better. Should I try different alternatives to find the best? Do I need a prescription? Doctor: Most of the side effects of radiotherapy, including erectile dysfunction improve with time, although the time needed for this to happen is variable and unpredictable and sometimes quite long. Also, there is not much that can be done for this besides using the drug that you already are using. To avoid the headaches you can take plenty of water half an hour before taking the drug and then increase the dose. That should suffice for the moment, till the time you completely recover. All the best."
},
{
"id": 109802,
"tgt": "Will intake of neurobion ease numbness in left thigh,back pain?",
"src": "Patient: I have numbness in my upper left thigh since last 4-5 yrs i always feel tired less energetic then earlier no symptoms for thyroid or sugar got checked weight 84 kg hight 170 cms, always having back pain and many places in back accumulation of some thing nmay neurobion fort or pls help me this numbness and pain Doctor: HiThank you for asking HCM. I have gone through your query. Your problem can be osteochondrosis or disc damage of lumbo sacral spine or muscle strain. For such cases i advice NSAIDs like aceclofenac muscle relaxants like thiocolchicoside and neurotropics like neurobion. Neurobion is a nerve vitamin which helps in nourishing nerve and enhances regeneration. Physiotherapy muscle strengthening exercises shoul ortd be done along with this. You can consult an orthopedician to rule out the condition . Hope this may help you. Let me know if you have any further queries."
},
{
"id": 93210,
"tgt": "Having pain in lower stomach, bloating. CT scan shows fluid in pelvic. Should I get worried?",
"src": "Patient: Hello I am in china. I had some pain in my lower stomach . I though that I may eat some thing wrong that y it is bloating so finally I just went to had a C T scan today in china. The result came back show that I had a fluid in my pelvic . Is it some thing I have to worry about or not please help. I really don t trust DR in china Doctor: Hello,Thanks for writing in.Free fluid in pelvis can mean many things.1. Little amount of fluid can be normal in females.2. Fluid can result from any infection in the bowel.3. Fluid in abdomen can also be due to serious conditions.4. Trauma to abdomen can lead to fluid.5. In diseases of liver, kidney and heart fluid accumulates in abdomen.Please get yourself examined properly by internal medicine specialist."
},
{
"id": 57207,
"tgt": "Is it possible to perform cholecystectomy after findings suggest tumour attached to gallstones?",
"src": "Patient: Hi there....have quick question.....i have an aunt whom recently was going to have her gall bladder removed due to having gall stones, but when the surgeon opened her up saw that there was a tumor attached and said he could not remove it....would you know why that was the case? Doctor: Hi,I am sorry to hear about your aunt's diagnosis. But it is fairly common to have a \"surgical surprise\" of seeing a tumor when operation is being carried out for gall stones. Your aunt's surgeon did the absolute right thing in aborting the operation. This is because your aunt needs to be investigated to evaluate the extent of tumor - whether it is limited to the gall bladder or spread to local lymph nodes or the liver. She will also need additional blood tests for checking for tumor markers. once all this is done, then her treatment can be tailored according to the stage of her disease and can include surgery, chemotherapy with or without radiotherapy.Hope this helped and please do not hesitate to ask for clarifications"
},
{
"id": 19851,
"tgt": "What causes sudden fluctuations in the BP level?",
"src": "Patient: I am 64 yrs. old i have had blood pressure of 120/80 my entire life & it seems suddenly it has gone down to 103/62 some times a little bit higher sometimes as low as 93/59 outside of that i feel fine i just wonder if i should take it seriously or monitor it for a couple of weeks. Doctor: Hello!Welcome and thank you for asking on HCM!Regarding your concern, I would explain that these low blood pressure values could be related to different possible causes: dehydration, anemia, electrolyte imbalance, etc.. For this reason, I would recommend consulting with your attending physician for a careful physical exam and some tests: -a resting ECG-a chest X ray study- complete blood count for anemia- blood electrolytes for possible imbalance. In the meantime, I recommend having some rest and take plenty of water to stay well hydrated. Increasing salt and coffee intake will improve you situation. Hope you will find this answer helpful!Wishing all the best, Dr. Iliri"
},
{
"id": 80815,
"tgt": "Why do I have dull chest pain in the xiphoid process area?",
"src": "Patient: while working, i laid my chest on a stack of boxes to lift a rather heavy box, now, i have dull chest pain in the xiphoid process area, it has been a few days now, when i eat i get a full feeling after just a small portion of what i would normally eat at a meal, turning/twisting causes pain in my back and a self exam of the xiphoid process, the right side seems firmly attached to the rib cage while the left side there appears to be about a 1/4 to 3/8 gap and the xiphoid appears to be lower on this side Doctor: Thanks for your question on HCM. I can understand your situation and problem. In my opinion you are having both muscularpain and GERD gastroesophageal reflux disease ). You have history of heavyweight lifting. This cause muscular pain on bending and movements. You have symptoms of early satiety (small portioning of food gives you fullness) , which is seen in GERD. So for GERD, start Proton pump inhibitors. Avoid stress and tension. Avoid hot and spicy food. Avoid large meals, instead take frequent small meals. For muscular pain start painkiller and muscle relaxant. Avoid strenuous exercise. Avoid heavyweight lifting. Apply warm water pad on affected site. Don't worry, you will be alright."
},
{
"id": 214103,
"tgt": "Suffered with low self esteem",
"src": "Patient: Id just like peoples opinions on how lack of this can affect an individual?.Like to know peoples experiences , who suffer/ have suffered with low self esteem... what did you do to overcome it( if you did) Doctor: Hello. Low esteem is just a psychological feeling of one\u2019s own performance or worth. So all you need do is, just train your mind in order cope up better.Here are some tips to help you deal with it.1) Look for a model (someone who has self confidence) among your friends circle or a public figure for inspiration and learn from them.2) Prepare thoroughly for any task that you are going to do, so that you can be sure you are ready and confident.3) Stop comparing yourself to other people in negative way and you will end up having low self esteem.4) Always think positive about yourself; you can\u2019t develop high self esteem if you constantly repeat negative comments about your skills and abilities.5) Healthy body, healthy mind; Take adequate and proper nutrition, which includes all vitamins and minerals.6) Mix with positive and supportive people; it will help you to raise your self esteem. You can follow me twitter, drarunprakashmd."
},
{
"id": 56816,
"tgt": "Can chewing and swallowing tobacco cause damage to liver?",
"src": "Patient: my liver enzyme level is slightly elevated and I have had liver sonograhm and hepititis screening. The sonograhm was normal and the hepititus tested negative. My doctor is having me fast from sugar and no alcohol for a week and then check liver enzyme again. What I am wondering about is that I chew tobacco and do not spit, I also drink 2 or 3 drinks almost everyday. I know about the risk of alcohol but am wondering about my tobacco use. Is it possible that my chewing and swallowing habit is causing damage to my liver? Doctor: Ya chewing of tobacco effects liver also,it causes your liver shrunken, avoid chewing, but alcohol cause more dangerous effect than tobacco"
},
{
"id": 97474,
"tgt": "Suggest alternative method of treatment for dialysis",
"src": "Patient: My creatinine is 7.4 and BUN is 159, Doctors advised to be ready to go in for dylyssis, please advise any other cure such as accupressure theropy which will be a better alternate. Living at Borivali east, Mumbai. Age 58, 162cm, 67kg. Left kidney operated for one inch calculus in 1992. Doctor: No pathy can offer cure in such cases as there is shrinking of kidney size thereby reducing its functioning. This can not be regained. But the condition can be maintained under stable state for a long period. as to maintain BP and Sugar it is advisable to have regular allopathic medicines. To improve kidney functioning even at this stage I have seen good results with tab. punarnava mandur, punarnava arishth and varunadi vati.dr_lakhanpaul@yahoo.co.in"
},
{
"id": 81461,
"tgt": "Is there possibility of curing mdr?",
"src": "Patient: i am mdr patitent i m taking canamycine inject hencypran 100 combutol 800 and moxy isuffer of jondice so stop all med for one month now jondice nearly clear i start inject 05 ten days befor moxy start 7 days a go will b start other medicine by time iafferaid my tb will be cure Doctor: Thanks for your question on HCM.MDR tuberculosis needs atleast 24 months treatment with at least 6-7 drugs.The drugs are as follows1. Kanamycin2. Levofloxacin3. Ethambutol4. Ethionamide5. Cycloserine6. Pyrazinamide7. Para Amino Salicylic acid (PAS).So your treatment is not according to the standard guidelines for MDR tuberculosis.So better to consult tuberculosis specialist and discuss all these.Moxifloxacin and capreomycin are used in XDR (extremely drug resistant) tuberculosis not in MDR."
},
{
"id": 39076,
"tgt": "What is diet for typhoid?",
"src": "Patient: Hi Doc, my mom is suffering from typhoid... she is 50 years of age. the typhoid has been detected today itself.. its been almost 20 days now that she is having fever... what can we give her to eat so that she can digest it easily... she is vomiting frequently.... Doctor: Hello1] It takes @ 2 weeks to cure Typhoid completely. 2] Give very soft, well cooked, bland foods like- Rice gruel, Ganji, Khichadis, soups, fruits without seeds,non oily & non spicy foods, but there is no food to make her cure fast only by food. 3] She should be on this diet for 2 weeks at least. She can eat well cooked , boiled / grilled Non-veg food after 2 weeksDiscuss this with your doctor and then decideHope this helpsTake Care."
},
{
"id": 209462,
"tgt": "Suggest treatment for mental attachment disorder",
"src": "Patient: Hi, im a 16 year old girl. I am looking for help with my mental attachment problems. It s a big problem for me and i can explain it further if you d want. I want therapy but if i ask my parents, they ll start stalking my everyday life and not leave me alone until i give them a good reason . Doctor: Hello,Thanks for choosing health care magic for posting your query.I have gone through your question in detail and I can understand what you are going through.You will certainly require psychotherapy preferably psychodynamic psychotherapy. Hope I am able to answer your concerns.If you have any further query, I would be glad to help you.In future if you wish to contact me directly, you can use the below mentioned link:bit.ly/dr-srikanth-reddy\u00a0\u00a0\u00a0\u00a0\u00a0\u00a0\u00a0\u00a0\u00a0\u00a0\u00a0\u00a0\u00a0\u00a0\u00a0\u00a0\u00a0\u00a0\u00a0\u00a0\u00a0\u00a0\u00a0\u00a0\u00a0\u00a0\u00a0\u00a0\u00a0\u00a0\u00a0\u00a0\u00a0\u00a0\u00a0\u00a0\u00a0\u00a0\u00a0\u00a0"
},
{
"id": 48495,
"tgt": "How can cysts in the spleen and kidney be treated?",
"src": "Patient: I have had Multiple Myeloma 1999 with a stem cell transplant. At this time I have 16mm cyst on my spleen and 16mm cyst on my left kidney with 2 smaller cyst on the same kidney. How should I get treatment and what does this mean, my Igm levels are high again? Doctor: Helloyou dont need to worry about these cysts because these r harmless n cysts can be found in many health people"
},
{
"id": 170623,
"tgt": "What is the red colour lump near the rib cage of an infant?",
"src": "Patient: I have a 2 month old boy and I just noticed a lump the size of a nickle, raised and reddish color to the skin area at the bottom of the right side of his rib cage. I have an appt with the pediatrician in the morning but am anxious about a thought of what it could possibly be? any help would be appreciated. Doctor: Hi.... by what you quote I feel that it could be a hemangioma or cluster of small blood vessels, which you are observing. The Other possibility is that it could be a bleeding disorder, but it would have been bluish black and not red lump like this. I suggest you either post an image of the same on this website or get it exam examined by your pediatrician.Regards - Dr. Sumanth"
},
{
"id": 63033,
"tgt": "Suggest remedies for a painful lump on the shin",
"src": "Patient: about two months ago I noticed a hard lump the size of a half dollar on my right shin, it was tender and did hurt a little when touched or rub against something...now it is now a knot the size of a chick pea. My whole leg hurts from that spot down..not to bad but it still hurts..also my foot really hurts almost as if something fell on it..it mainly hurts when I stand and walk but its always hurting. I can't remember any injury to make the lump appear and I haven't suffered any injury to make my leg and foot hurt...Please help, I am getting worried. Doctor: hi.it is best if you consult with a doctor, preferably a general or an orthopedic surgeon, for medical and physical examination. based from your description, it could be a cyst (a sebaceous or a keratinous type), a fibroma or a lipoma. these lesions can occur anywhere in the body and have the tendency to recur. it could also be a tumor which needs further evaluation such as MRI and tissue diagnosis. management (medical and/or surgical if indications are found) will be directed accordingly.hope this helps.good day!!~dr.kaye"
},
{
"id": 76014,
"tgt": "Will it be safe to take Diprofos injection for rhinitis and oedamatous sinusitis?",
"src": "Patient: I am a tenor, and because of seasonal rhinitis, and oedamatous sinusitis, my voice physician has recommended diprofos injection. Whilst it usually has an immediate healing effect, nasal congestion persists. What are your professional recommendations considering this scenario? Doctor: Hi thanks for contacting HCM...Here you are having allergic infection of nose mucosa and sinuses....As it is needed to get immediate relief your doctor recommended dexomethasone injection for that...Steroid having antiinflammatory action so useful for it ....Steam inhalation taken two times daily for few days....Excess spicy and fried food restricted...More water and fruit juice taken....So steroid can taken to reduce inflammatory swelling but frequent use of it avoided...Alternately inhaled steroid intranasal spray also useful....Salt water gargles done ....Strepsil like lozenges useful ...Take care"
},
{
"id": 218674,
"tgt": "How can accurate pregnancy test results in weeks be identified?",
"src": "Patient: Hi my name is K my children's father came home from jail on the 22nd of last month i took a test 2days ago and 2 lines came up for positive pregnancy The line was very light but noticeable First day of my last period was August12th 2017 It lasted seven days as usual We had intercourse my highest ovulating days soon as he got home Question is am I 2 weeks and 6 days or how far? Doctor: Hello,As you might be aware that in a woman with regular cycle, day 10 to 20 of the cycle is the most fertile period of the cycle. In your case, if you happened to have unprotected sex activity in this period, then you stand chance to get pregnant.Usually, pregnancy duration or age of pregnancy in weeks is calculated from the first day of last menstrual period. So, you can calculate the weeks from August 12th, and that will help to know about gestational age in weeks. This can be confirmed by Ultrasonography (USG).Hope I have answered your query. Let me know if I can assist you further.Regards,Dr. Purushottam Neurgaonkar"
},
{
"id": 147814,
"tgt": "Suggest me treatment for paralysis",
"src": "Patient: Dear sirMy brother is patient of paralysis from 2.5 yr ago.only some fingers problem are left in paralytic condition.He is only 25 yr old. apporex 1.5 yr ago, he started getting fits for 15 minute then after 1yr of first fits, second time fits came for only 2min and now only 5 months later again fits came only for 1 min fits and after fits nothing is remembered that somthing is happendwith his body.Only he feel weakness in whole day. Please suggest me that what is treatment of this disease and plz tell me that its will come in whole life? Doctor: HIThank for asking to HCMI really appreciate your concern looking to the history given here it could be said that this could be a epileptic seizure, and this disease easily being treated and with the proper drugs the result is excellent once the treatment start then fit stops immediately unless the drug not being taken in time, I think if the fit is still there even after taking the medicine then investigation would be needed, MRI has to be done, for the paralysis under lying cause has to be treated, hope this information helps have nice day."
},
{
"id": 210691,
"tgt": "Is it safe to take Ativan for anxiety and panic attacks?",
"src": "Patient: hi my daughter is and has been suffering from anxity and panic attacks now for 4 days it is coming in wave crying episodes and points where she feels like she cant breathe...however I was wondering if giving her Ativan is safe of a low dose of 1mg...she is 14yrs old..will this be safe? Doctor: HIThanks for using healthcare magicAtivan is a safe benzodiazepine, but better to give it on sos basis to avoid dependence. If she has repeated attacks of anxiety, that she needs a antidepressant. You should consult a psychiatrist in her case for proper management.Thanks"
},
{
"id": 26386,
"tgt": "Suggest treatment for left sided chest pain and numbness",
"src": "Patient: Hello... uhm my name is Connie.. and I am a freshman... i just wanna know something please... the left side of my heart tightens and my left upper body paralizes or numbs for few seconds when it do... my left side of my chest hurts... and the heart pinching occurs more frequently now... please help :'( Doctor: Welcome at HCM I have gone through your query and being your physician i completely understand your health concerns.What is your age? For how long u are suffering from it? Any other associated problem? like diabetes or hypertension? Any history of TB in past or exposure to TB? Are u running fever? Any other lymph node swelling in ur body? do u have cough? are u smoker? Chest heaviness or discomfort in elderly patients should not be taken lightly. You need to be investigated for proper management. If i am ur attending physician i will get your BP reading along iwth ECG ECHO CHEST xRAY to find the cause.. Meet your doctor so that proper intervention can be done in time. Meanwhile stay calm and use acetaminophen to relieve pain Get well soon Hope your query is adequately addressed if you still have any feel free to ask RegardsDr Saad Sultan"
},
{
"id": 103331,
"tgt": "Rashes all over scalp, behind ears, spreading to face, swelling ears. Cure for symptoms?",
"src": "Patient: I need to figure out the type of allergy I'm having, started with rash all over the scalp, a few days later down behind the ears, again a few days later It go extended to the face so the face and ears were swelling and now it is still on the face, neck and chest. I'm afraid its going to reach the chest and tighten up my breathing. Doctor: Hi, Thanks for using HCM.Since the rashes are spreading and you have not mentioned about its appearance and associated symptoms, it should be evaluated for the cause. Usually skin rashes does not affect systemic if it is due to derma problem. If any systemic association or the rashes appearde due to systemic problem then there are chances of other organ involvement. It may be due to allergy or due to infection or due to any drug induced rashes. Kindly let us know about above mentioned charecteristics, investigations you have, drugs you are taking and any other disease you are suffering from so that we can give you better picture for the management. Consult your doctor for examination and management.Hope I answered your question. Feel free to ask me if you have any further queries. Wish you good health. Take care.RegardsDr. Lohit"
},
{
"id": 195256,
"tgt": "What causes sperm leakage during day time?",
"src": "Patient: Hi,is it possible for my partner to leak semen while he is at work,eventhough we have had intercourse that morning.He has had a vasectomy and says that semen still comes out after intercourse.I not sure what to believe as his underwear most days have deposits of semen on them after work.Is it possible as he claims he is not cheating or masturbating. Doctor: Hello and Welcome to \u2018Ask A Doctor\u2019 service. I have reviewed your query and here is my advice. Usually vasectomy won't cause semen leakage. After vasectomy only precum or prostatic secretions may be ejaculated. But semen leakage may be seen in pathological conditions like 1. Nocturnal emissions. 2. Medication side effects. 3. Prostate problems. 4. Nerve injury. Please consult your doctor, he will examine and check for it then treat you accordingly. Take care."
},
{
"id": 68609,
"tgt": "What causes lump in between the breasts?",
"src": "Patient: Just yesterday I realized I have a lump in the middle of my chest. Its right in between my breasts. It hurts when i press it and I even get pain in my back directly behind it sometimes. I did not have the lump before but that area did hurt starting last sunday and then I was even short of breath or whenever I would breath it would hurt, and the deeper I would breathe in the worse it hurt. Im not sure what is going on. Doctor: welcome to Health care magic.1.Lump with pain radiating to the back and shortness of breath seems like a infective aetiology ( bacterial infection - Abscess, infective cyst).2.If you are mu patient i would have examined and ask for the ultrasound lump / lesion. ( helps in assessing the nature of lesion, its source and extensions)3.After that depending on the nature - an FNAC - fine needle aspiration cytology to evaluate the kind of cells ( benign / malignant / infective)4.If infective a small incision and drainage is the treatment of choice.5.If mass lesion - depending on extension removal will be planned.Hope it helps you. Wish you a good health.Anything to ask ? do not hesitate. Thank you."
},
{
"id": 147357,
"tgt": "How can a person with a blood clot in brain and breathing through a tube recover?",
"src": "Patient: hello doctor ,my friend s dad is suffering from blood clot in brain nerve..........now he is unconscious......he is breathing through oxygen tube........plzzzzzzzzzz help me to recover her dad my email : YYYY@YYYY plzzz help me immediately Doctor: HIThank for asking to HCMI really appreciate your concern for your friend's father looking to the history given here he must be in coma and having life supports so this is question of intensive care and I guess he must be in same unit and there he is on all the necessary treatment, in my opinion it is advisable to have a word with treating physician for prognosis, hope this information helps you have a nice day."
},
{
"id": 88655,
"tgt": "Suggest remedy to relieve pain in abdomen",
"src": "Patient: Hi I'm a 26 year old Female and for the past week now i have been having a lot of upper and lower abdominal pains in my stomach. My upper and lower stomach is sore, has sharp pains, and feels like I have been punched in the stomach. I am also feeling very very gassy burping more often and farting as well. I've never had these issues therefore I don't know what to do. The other night i had so much pain I couldnt sleep. Only after taking alka seltzer I got sleep for a few hrs then had to take some more alka seltzer. I went to the doctors and she didnt seem to think there was nothing wrong and told me to wait another week! This pain is not going away and I am very worried. What over the counter drugs should i take? should i wait another week before going bsck to my doctor? I really needyour help. Thank you so much! Doctor: HI.In such a case you should take another opinion of another Doctor as you have pain and all other symtoms. I would suggest you the following. I would advise all such patients the following:Routine blood (CBC, blood sugar, Creatinine) , urine and stool tests.Ultrasonography of the abdomen.I trial with a course of an antibiotic, metronidazole for 5 days.Activated charcoal tablets, Prebiotics and probiotics, multivitamins.Avoid foods and beverages (like coca cola and soda), which you must have observed by this time that cause more gas problem.Eat less, walk around after food, sleep early, get up early , moderate exercises.Avoid stress and anxiety at any cost. This is in your hands to decide. If you have gall bladder stones, you will need to get operated to remove the gall bladder.Once the diagnosis has been made, you can certainly get a targeted treatment to get a cure."
},
{
"id": 158115,
"tgt": "Fibromyalgia, pain in arms, hands. Taking lyrica. Family history of pancreatic cancer. Treatment?",
"src": "Patient: Hi Doctor, Ihave been told that I could have fibromyalgia ,but not really diagnosed properly.I have seen the chart just now that shows the areas that are affected and that is exactly where I have the pain,although my pain extendsto my arms and hands. I have been taking lyrica for this and I find this helpfull.What exactly is lyrica for.My brother passed away last November from pancratic cancer and he was also on lyrica.Thank You. Doctor: Lyrica tab contains the ingredient Pregabalin.Pregabalin is an anticonvulsant drug used for neuropathic pain and as an adjunct therapy for partial seizures with or without secondary generalization in adults.Pregabalin is effective at treating chronic pain in disorders such as fibromyalgia nd spinal cord injury.If it is very painful then u can take Tab lyrica 150 mg twice daily after food for 1 month."
},
{
"id": 221147,
"tgt": "What is the best treatment to get pregnant?",
"src": "Patient: me and my fiance have been together for three years and we never used a condom. he pulls out usually but we have had alot of times when we did not pull out and i was glad i did not get pregnant but for the last month we have been tryin to get pregnant and i just started my period so should i be worried that i cant get pregnant or is it normal to take a while to get pregnant? Doctor: Hello, and I hope I can help you today.When trying to conceive, 80% of couples will conceive within one year. Even though you used the withdrawal method previously, there still is no evidence that you would have any trouble conceiving as withdrawal does reduce the risk of pregnancy by about 75%.Many women assume that if they have not had an accidental pregnancy in their life that they are infertile. This is actually untrue, because contraceptive failure, even with natural methods, is still relatively unusual.So if you and your partner continue to have regular, unprotected sex during the fertile time of your cycle (which is the time between the end of your period until the time you start to feel PMS symptoms) you should be able to conceive eventually.While you are trying to become pregnant, it is recommended that you begin taking prenatal vitamins or of folic acid supplement to help prevent certain kinds of birth defects.I hope that I was able to adequately answer your question today, and that my advice was helpful.Best wishes, Dr. Brown"
},
{
"id": 105769,
"tgt": "Extreme sore throat,cough and chest pain while swallowing",
"src": "Patient: I have been on and off sick for about 2 months now. I have an extremely sore throat , so bad to the point that I can t eat for days. My throat swells up and I have white sores all over the back of my throat. I have been to 3 different doctors and it s not strep or mono or any other type of bacterial infection . On top of this about 4 weeks ago I developed a cough, which I originally thought that maybe it was allergies but now I have pain in my chest when I swallow or cough and I have the taste of mucus and copper when I cough. Doctor: Since you are suffering for last 3 months, you need to undergo detail evaluation. Chronic diseases like TB needs to be ruled out. White spots in the throat may be chronic bacterial infection or fungal infection. A routine blood test, sputum test, HIV test and a xray chest should be done."
},
{
"id": 197347,
"tgt": "Is my sperm count normal?",
"src": "Patient: My semen densuty(count) is 48mill/ml. My total motilityof sperms is 90%.Rapid linear progressive motility is 85%. Slow linear progessive motaliy is 5%.Non progressive moyility is 5%.Immmotile is 10%.Vitality is 90%. My sperm volume 2ml, pH value is 7.8, viscosity High. Is my sperm count Normal? Doctor: Yes it is normal. According to World Health Organisation criteria of year 2010, lower limit of normal are Volume - 1.5 ml, Concentration - 15 million/ml, Progressive motility 32%, Normal forms 4%"
},
{
"id": 197829,
"tgt": "Will stopping masturbation help in getting back to normal physical structure?",
"src": "Patient: well, i read about that masturbation changes body when you do it as an addiction, and i've notice my right chest shrinked and looks smaller than my left chest. i have masturbated for a year and im addicted to it. i just wanted to ask you i i can get back to normal if i dont do it anymore? Doctor: Hey..See dear, Masturbation does not cause any physical injury or harm to the body and can be performed in moderation.Yes, you should not make it a addiction or habit.In my opinion Masturbation has nothing to do with your shrinked and smaller left chest.If your left chest is abnormally small when compared to the right one, then get it examined by a local doctor near you.Also if you are feeling little weak, then concentrate on your diet, drink plenty of water and exercise daily. This will keep your body fit and well.Hope I have answered your question. If you have any questions, then I will be happy to answer them.Take care.Regards.Dr. Surender sharma."
},
{
"id": 125260,
"tgt": "Suggest treatment for pain in my hand when it moves out",
"src": "Patient: Hi, sometimes a bone in my wrist moves out of place when I workout. I have had my chiropractor put it back in place in the past and he showed me how, but I am struggling to push it back in. When I bend my wrist down, the bone pops up just behind the hollow near the base of my index finger metacarpal. What can I do to relieve my hand pain please? Doctor: Hello, As first line management, you can take analgesics like Paracetamol or Aceclofenac for pain relief. If symptoms persist you can consult an orthopaedician and get evaluated. Hope I have answered your query. Let me know if I can assist you further. Take care Regards, Dr Shinas Hussain, General & Family Physician"
},
{
"id": 194067,
"tgt": "What does enlarged prostate gland mean?",
"src": "Patient: I am male of 58. suffering from diabetes for last 20 years. undergone angioplasty with 3 stents in 2005 and taking all medicines prescribed by Doctor. present average level of sugar is 150. recently, i frequently urinate with 2 -3 hours interval.i got my check up. doctor says that prostate is enlarged (BPH) for which tests e.g uroflamatory, S.P.SA,USG Kvb + prostate PvR, Urine (Routine & Cr ) are recommended. I want to know what these each test mean and indicates Doctor: Hi, BPH is most common among males above age 50. It usually causes hyperplasia of the prostate which usually causes the prostate to enlarge. It thereby compresses the bladder above & then it leads to urinary retention. Due to urinary retention, the bladder fills quickly and causes frequent urination. There is a specific test for BPH known as PSA. Which is known as Prostate-specific antigen instead of doing those you can do this alone. Hope I have answered your query. Let me know if I can assist you further. Take care Regards, Dr S.R.Raveendran, Sexologist"
},
{
"id": 106453,
"tgt": "I have pneumonia and a low White Blood",
"src": "Patient: MY Doctor sad i have pneumonia and a low White Blood Cell Count please read ? i hope its not HIV ? My doctor sad i have pneumonia and my WBC is a little low, the Range was 3.8-10-8 of my White blood cells. what could be causing, the only condition i have Seizure, Asthma, Muscle Aches and HSV1. Could it be HIV i heard it lowers your WBC? Doctor: Have you tested for HIV? Get one done. You have HSV1, which could also be the cause for it."
},
{
"id": 82381,
"tgt": "Suggest treatment for pleural effusion",
"src": "Patient: Dear Sir My Father is suffering from Plural Effusion for the past few months. We have taken him to different hospitals and the reason for Plural Effusion is unknown. We have taken all the relavant test and all the results are negative for TB and Cancer. We have drained the plural fluid few times but still there is no sign of stoppage. Now a tube has been attached to Lungs to drain the water. Since the reason is unknown the treatment is in effective. Is there any specilaist who can help in diagonisis ? I live in Chennai-TamilNadu, India Doctor: Thanks for your question on HCM.In my opinion you should consult pulmonologist and get done pleuroscopy and biopsy.Get done fresh CT thorax and plan cor pleuroscopy.In chennai, many good hospitals are there. Go to apollo hospitals, they have dedicated team of pulmonologist.Possibility in your case ir either drug resistant TB infection or mesothelioma.So consult pulmonologist and discuss all these."
},
{
"id": 146253,
"tgt": "What causes facial paralysis early in the morning?",
"src": "Patient: wife woke up with right side of face paralyzed -- unable to blink right eye, move right side of mouth...rest of body seems ok. she is type 2 on insulin. she refuses to go to doctor. she looked up MILD STROKE and says she will be ok. I say she should have gone immediately. what can happen? Doctor: Hello dear,What your wife calls mild stroke is most probably not a stroke.It is Bell's palsy rather.In stroke mostly(unless it is a pontine stroke which also impair eye movements),there is only facial deviation and blinking of eye is not affected.Please take her soon to a neurologist as early treatment with antivirals and short course of steroids enhance the recovery significantly.Even if it is stroke,early treatment is must for recovery and to prevent recurrence.I hope it helped."
},
{
"id": 91168,
"tgt": "How to treat abdominal pain after an abortion?",
"src": "Patient: Hi, I had a abortion about two weeks ago and I was under a lot of stress so about for days later I took a drink and had sex and I went to the doctors and they said everything is fine and my pain should go away when I come on my period but the abdominal pain hurts really bad, what does this mean??? Doctor: Hi.Well you should not have sex and drinks for minimum 3 weeks. The best way to know about pain is an internal examination by Gynecologist and an ultrasonography to help the clinical diagnosis. The possible reason can be retained products/ Perforation of uterus with peritonitis ( if history of surgical evacuation) / Pelvis inflammatory disease."
},
{
"id": 139617,
"tgt": "What causes indentations on both sides of head?",
"src": "Patient: im 50 yrs old. im 5ft 6 weight 103 lbs. i smoke. dont drink or do drugs. i think im healthy. except i get alot of headaches. have for yrs. about 3 wks ago i felt indentations on both sides of my head, on top along both sides. very noticable. i went to dr yesterday he ordered xrays and for me to see a rheumatoligist. thinks it could be osteporosis. it doesnt hurt. just really scarey. do you think my dr is going in the right direction Doctor: Hello and welcome to \"Ask a Doctor \" service.I have read your query.It may be osteoporosis, but also it may be muscle atrophy.I think that CT scan of the head is more appropriate than x-rays in your case.Bone density also should be measured and blood tests for hematologycal problems should be carried out.Hope this helps.Let me know if I can assist you further."
},
{
"id": 67173,
"tgt": "What relieves the soreness at the groin due to a lump?",
"src": "Patient: I had a spot on my groin, where the crease is which I squeezed . Blood and puss came out and now there is a little hole, probably size ofa pen tip, its a little sore still. I have been sore there for couple of weeks before I squeezed spot today. Should I be concerned? Doctor: HiWelcome to hcmThis is mostly a skin infection near hair follicles in groin. You should take antibiotics after showing to surgeon. Locally you can apply ointment like soframycin or Tbact.Regards"
},
{
"id": 205267,
"tgt": "Does anti-depressants interact alcohol?",
"src": "Patient: My husband has been an alcoholic for many years. His drinking, coupled with taking anti-depressants, led to a bipolar manic episode recently, resulting in a 2 week stay in a psych unit. He immediately returned to drinking upon discharge. He is now complaining of moderate edema in his ankles and calves. Could this be a reaction to his new meds or could it reflect possible kidney failure? Doctor: Hi, your husband was taking antidepressants so it might be antidepressants induce manic episode.alcohol dependence and sleep disturbance may precipitate mood episode in future so it is better for him to avoid alcohol and get better sleep in night.edema feet might be due to antipsychotic medicine which he is receiving for manic symptoms, it might be due to other cause. Better you discuss it with your treating doctor he will find out the cause and manage it properly.hope this is helpful for you.Thank Dr Ramashanker YadavMD PsychiatryAhmedabad"
},
{
"id": 55367,
"tgt": "Suggest diet to avoid gall stones",
"src": "Patient: Is there a site I can go to where it tells me exactly what foods to avoid because I am having gall bladder problems. I know there are some sites that say to avoid red meat, and fatty things but I just want to know which fruits and vegetables I can eat. For instance, my sister-n-law says I should avoid bananas? Is this a true statement? I am no currently off all dairy products, red meat and bread and tortillas. Doctor: Hello! Thanks for putting your query in HCM. I am Dr.Amit Jain (DM, Gastroenterologist).Foods to be avoided are:Fried foodsHighly processed foods (doughnuts, pie, cookies)Whole-milk dairy products (cheese, ice cream, butter)Fatty red meat Diet to be taken is:Fresh fruits and vegetablesWhole grains (whole-wheat bread, brown rice, oats, bran cereal)Lean meat, poultry, and fishLow-fat dairy productsBanana is not contraindicated and all fruits can be takenI hope I have answered your query. If you have any further query I will be happy to answer that too. Wish you a good health"
},
{
"id": 225220,
"tgt": "What does it mean if I do not have withdrawal bleeding after stopping microgynon?",
"src": "Patient: Hi, i ve been off microgyn for two days and i haven t had any withdrawal bleeds nor is my period seem to starting. What does this mean? does this mean i m pregnant? (ps i took microgyn for the first time two months ago, and i took two packets consecutively before stopping) Doctor: Hello ma'am and welcome.No this does not mean you are pregnant. Normally it should take around 3-7 days to experience the wirhdrawal blled, but it can take longer in some women as well.Hope this helps.Best wishes."
},
{
"id": 44519,
"tgt": "bicornuate uterus and vigenal septem, diabetes, IVF, conception",
"src": "Patient: my name is madhavi latha age of 32 i was married in the year of 2002 as of know i have no baby i went for many doctors, two times i was concive and aborshan , my problem is bicornuate uterus and vigenal septem the operation has completed successfully in CMC hospital vellore in 2011 but no concive as of know we are planning to go for ivf what is the success rate of this ivf my husbend age is 39 he is dayabatic patient but it is control what is your suggestion Doctor: Hi, Since you are already under treatment of gynecologist it will be best you discuss it with your doctor . Evaluation of your uterus and tubes should be done by H S G to know the the tubes are patent . your husband to be evaluated also doing semen analysis . Bicornuate uterus is known to have recurrent abortions and you will have to be under care of your doctor throughout from conception when it occurs expecting a best outcome . In case both the partners having normal parameters and normal sexual activities I V F may not be advised for conception. Hope you get result with treatment and pregnancy continues to full term or at least to viable age when a premature baby can be taken care of in well equipped I U C attended by efficient doctors and nurses. treatment to enhance fertlity will be helpful for conception ."
},
{
"id": 55094,
"tgt": "What does having borderline liver function test mean?",
"src": "Patient: What does it mean when I have a borderline liver function test? btw I'm 17 and I have thalassemia minor (hemoglobin 8) and a low level of vitamin D. I'm 5'7 (171 cm) and I weigh 168 lb (76 kg). Doctor recommended that I take vitamin D 5000. Do I have to change my diet? Maybe loose weight? Doctor: Hi you have borderline liver function test mean it is not above limit range but if you will not take care then it might raised above limit value.You might probably have fatty liver if you are obese.So loose weight if needed.Take minimal oil in diet.Use only good oil like sunflower oil or safflower or pea nut oil.Junk food, butter, cheese, meat taken very less.Regular exercise done.Refined food avoided with more fruits taken.Take vitamin d as you have lower value of it.Milk taken more.Daily also early in morning do some yoga or exercise in sunlight.I hope your concern solved.Take care.Dr.Parth Goswami"
},
{
"id": 171818,
"tgt": "Why my kid is afraid to interact with some guests?",
"src": "Patient: sir, my daughter is 4and a half years old, when some guests comes in our house she weeps a lot, she never allow her father to talk to anyone when with her, but if someone comes whom likes like any servant or any hawker she plays with them. we are very upset Doctor: DearWelcome to HCMWe understand your concernsI went through your details. At the age period of 2 to 5, children experience something called \"stranger anxiety\". At this age period, children are more attracted towards people who \"care\". For them care is usually given by mother, father, other near family members. Second aspect is their playful attitude. Because of this second attitude they mingle easily with even stranger children. But, the case of strange adults are different. Stranger anxiety coupled with \"he is not my play partner\"- attitude triggers the given behavior. Don't worry. The problem will be solved within some months.If you require more of my help in this aspect, please use this URL. http://goo.gl/aYW2pR. Make sure that you include every minute details possible. Hope this answers your query. Available for further clarifications.Good luck. Take care."
},
{
"id": 63582,
"tgt": "What is the lump in my earlobe?",
"src": "Patient: Hi I have this little lump in my earlobe, it only hurts if squeezed andI don't know what it is, I had an odd cut there and that like liquid type pus would come out of it so I am thinking it may be broken cartilage, and that ball is kind of the same thing as colliflower ear, that is just my guess.. Doctor: Hi,Dear,Thanks for the query to HCM. I studied your problem in depth and I understood your concerns. Cause and plan of Treatment-In my opinion on the given data,you seem to have infected wound on the earlobe, with pus in the earlobe.Don't guess more. I would advise ER Surgeon Consultation who would treat it with antibiotics / anti-inflammatory for 5 days time.Hope this would resolve your issues.Act fast.So keep cool and Consult your ER Surgeon, if need be , who would treat it accordingly.So don't build up wrong concepts and create more psychic complications in you which would increase risks and costs to you, but just ask a query to HCM and be comfortable to resolve your health issues.Welcome for any more query in this regard to HCM.Write good reviews and Click thanks if you feel satisfied with my advise.Have a Good Day.Dr.Savaskar M.N."
},
{
"id": 149372,
"tgt": "Painful area on spine, tender to touch. No limb weakness. No known injury",
"src": "Patient: Dear Doctor, I have a spot on my spine ( Thoracic area) that is painful when bending forward or hunching over. The spot where the pain radiates from also is painful and tender to the touch. The pain is not debilating, just very aggregating. I have no weakness in arms or legs. The pain has been around about a month now. I ve had no injury to that area that I know of. The pain hasn t gotten worst, but hasn t gone away. I m 33, male no history of health problems. Doctor: This type of problem appears in a person who is using Mattress since long time and the place where u r lying definitely there will be a sagging of matress so first u have to change ur matress and use Firm bed then follow Physiotherapy advices like UST, Maitland mobilization some extension exercises & Ergonomic advices if u r habitual to work in a slouch posture, be attentive for ur posture."
},
{
"id": 92508,
"tgt": "Stomach pain, pressure in heart, faster heart beat, headache. Will taking advil help?",
"src": "Patient: I woke with a stomach pain down below cause I'm about to get my period, but I usually don't get sharp pain like this. Then my heart felt a lot of pressure and started racing fast for a few minutes and it happened a few times. And now my head hurts cause I was at a family birthday and there were lots of conversation. I just took an Advil so hope it'll help. Doctor: HIThank for asking to HCMWhen there is any kind of pain one is experiencing and bearing it hard then heart beat increases, and later on headache started, dizzy feel these all symptoms are just consequences of tolerating the pain and goes away soon, any symptomatic medicine gives nice results, rest is also equally important, no need to worry, have nice day."
},
{
"id": 223925,
"tgt": "Can birth control pills cause irregular periods with brown discharge?",
"src": "Patient: Hi. I am 32 around 5 4 220 lbs female. my period is not due for about a week and I am on birth control pills. for the last few days (same thing happened last month) I have been having a brown fibrous discharge. it is not overly heavy and am having no pain...should I be concerned? is ita issue with my birth control pills? Doctor: Hi Hope this message finds u in good health.I have gone through ur msg and understand your concern.it may be due to some sort of hormonal imbalances or variations in the body,though there may be other reasons as wellNothing to worry about, You should eventually get back to normal.Take multi vitamin and calcium supplements Do consult a gynecologist if symptoms worsenGet back to me for any FOLLOW UP QUERIES anytime.Regards,Dr Mahaveer Patil...(MBBS,MS,Mch)"
},
{
"id": 159575,
"tgt": "Lump in throat, weight loss, healthy appetite. Blood test shows low haemoglobin. Cancer?",
"src": "Patient: Husband has a lump in throat ,non-palpable, and lab results today came back w/low hemoglobin . In the last year he has lost about 10 lbs. he s very thin, eats healthy, and is . He s never lost weight in the about 20 yrs. we ve been together. He s 46 yrs. old. Exercises about 1 hr/day. Former triathlete. Doesn t sit right w/my gut. Many neighbors around 50 yrs. old have had cancer in this area. Live in an active, median income $100,000+ community. Am I overly concerned? Doctor: No, I think you are not overly concerned. Your husband is a young patient (46 years old) with a recent history of significant weight loss (without a diet to loose weight), with anemia and a lump in the throat. He should have a complete evaluation: He should be observed urgently by a physician, with a complete blood analysis (CBC and chemistry) and probably he should do a CT scan to detect possible lesions, which can explain these symptoms. Hope this answer was helpful."
},
{
"id": 216151,
"tgt": "What causes abdominal pain after umbilical hernia repair?",
"src": "Patient: Mymother-in-law is 5 or 6 yrs post op umbilical hernia repair with mesh implant. She has started wxperiencing abdominal pain that started out as a \"twinge\" toward the right of the area and has progressed to the point of going to the emergency room d/t the pain. She is a strong woman and handles pain well. She rated her pain an 8 on a scale of 0-10. A CT scan revealed nothing. Labs revealed nothing. They told her it most likely was a pulled muscle. Gave her muscle relaxers and sent her home. The pain lessened, as she did nothing but what absolutely had to be done. Decreased movements or muscle relaxers offered the relief...not sure. Now up and going and no meds and the pain is back with avengence. UA was negative. Could this be a mesh issue? Who should she consult as her surgen is no longer inthe area and her PCP is not real worried. Doctor: Mesh have become notorious for problems. Basically, they are not normal tissue. They do not adapt, change, move, repair as normal tissue does. There can be plain inflammation around them(rare) but certainly scar and not moving iwth the surrounding area causes twinges. There isn't really easy fixes for it."
},
{
"id": 140332,
"tgt": "What causes groin pain radiating to back and episodes of stuttering?",
"src": "Patient: I had a car accident feb. 2 this year and initially had head fuzzyness, discomfort in my neck and pain at base of my skull. I also slammed my hand on steering wheel. The front of my car t-boned a car that failed to stop at a stop sign. My car is damaged considerably. I did not hit my head or lose consciousness. CT and hand xray in ER were negative. I was sent home with neck collar and hand brace. Two days later I had groin discomfort on the right which radiated to my right hip and lower back. Two days after that I noticed word finding difficulties at times and frequent episodes of stuttering. I saw my doctor yesterday who is having me stay off work for a week and I am to follow up with him on 2/14 to see if I am ready to return to work. Should I be waiting this long? Doctor: Hi, Based on the description, the problem could be minor such as concussion and mild spine injury. However, I would not wait long and get an MRI of brain and spine done to look for any injury. This is better than waiting, as early detection of the problem and prompt treatment could avoid long term neurological dysfunction. Hope I have answered your query. Let me know if I can assist you further. Regards, Dr. Sudhir Kumar, Neurologist"
},
{
"id": 155729,
"tgt": "What could cause redness, varied areola, thickening of breast with family history of breast cancer?",
"src": "Patient: mammogram, ultrasound and punch biopsies of breast all came back inconclusive. but still experiencing symptoms of redness, areola different from other breast, puckering, thickening. family history of inflammatory breast cancer. dk what to do next. please advise. Doctor: Thanks for your question on HCM.Your all symptoms like puckering, thickening, reddened areola are suggestive of malignancy mostly. And you are also having positive family history. Which makes you suspect for breast cancer.I advice you to get done1. PET SCAN 2. Biopsy from PET positive lesion.3. Get done ER (Estrogen Receptor) and PR (Progesterone Receptor) study from biopsy material.This will give you exact idea about you are having malignancy or not.If PET SCAN is negative than no need to worry much."
},
{
"id": 30128,
"tgt": "What causes infection post placement of screws in the ankle?",
"src": "Patient: On 27-05-2016, I got operated my broken left ankle and its supporting small bone.Doctor has inserted screws at the ankle and plate with screws to the small bone. Everything is under control,but the place where the bones tear the skin is infected with pus and little painful. I am 58 years and diabetic under control. At the time of discharge and 2nd review orthopedician prescribed me with (i) Cal Q Alfa only and the General physician with (i) Agriplim M2 ,(ii) tenebite for Sugar and (iii) Nicardia Retard for BP. Since, I am not able consult them please advise with for the infection with pus. Doctor: HI, thanks for using healthcare magicThe infection would need to be treated with oral or iv antibiotics. IV antibiotics would be needed if the infection is assessed as severe.You need to see your doctor, either your GP or orthopedic doctor , for an assessment as soon as possible to determine which type of antibiotic would be best for you to use.Early treatment would prevent spread to the bone , if it is not there at this point.I hope this helps"
},
{
"id": 197846,
"tgt": "How long does the remaining sperm stay in the penis post ejaculation?",
"src": "Patient: how long does sperm stay in the penis when ejaculating twice?well the other day i was masturbating and i ejaculated and right after i ejaculated i started to masturbate again then i ejaculated again so my question is since i ejaculated twice how long does the remaining sperm stay in your penis? Doctor: Hi,Thanks for writing in.The penis contains a long thin tube called the urethra. The semen passes through the urethra and is ejected out of the penis. The sperm can get stuck in small amount within the urethra and this can stay for about 15 minutes. After that the sperm again gets liquefied and comes out during the next act of urination.Therefore if you ejaculated twice in succession then the semen would have been in the penis for 15 minutes and then it gets dissolved and is passed out in urine. Please do not worry."
},
{
"id": 52185,
"tgt": "Does gallbladder dysfunction cause constipation and warm sensation in the stomach?",
"src": "Patient: Hi Doctor, a few weeks ago, I felt a flutter in my stomach after eating and immediately was short of breathe with Tachycardia, I didn t lose consciousness but couldn t catch my breathe and went pale. I went to my GP, had labs done, and everything came back in order and I was told it was nervous stomach from stress, I was prescribed Zantac. I have always suffered from constipation, but now after that episode, I get the same sensation in my stomach after I eat anything, I feel full, constipated, and like throwing up, and as if there was a ball in my stomach that feels warm. I also have the hard time breathing and my heart starts to race. Could this be a gallbladder symptom or something else? How can I have my GP give me a full check up to see if there s something wrong with my digestive tract? I m a 38 year old female and this is very scary. Thank you! Doctor: Hi, Yes, it may be caused by gallbladder dysfunction. I suggest to have echolecytsectomy. Hope I have answered your query. Let me know if I can assist you further. Take care Regards, Dr Iven Romic Rommstein, General Surgeon"
},
{
"id": 152169,
"tgt": "Which treatment i should take for right back side of my head pain, when mri report is normal ?",
"src": "Patient: sir my age is 24 yr. i have pain on the right back side of my head the pain increases after eating food the pain is continuous i had done mri but the report is normal i feel heaviness of head which diseases is this and which tests i have to do? Doctor: Hi,Laxman thanks for query.Normal MRI rules out major structural problems so there is no need to worry.The possible causes that can lead to this are ,stress headache and indigestion.Take light diet and avoid stress.Try to avoid too much of spicy food ,take foods regularly.You can try with antacids.bye."
},
{
"id": 48807,
"tgt": "Suggest treatment for kidney stones and pain",
"src": "Patient: hello doctor . ..I hav pain in my left kidney n there r small stones have occured . .even though i have taken a tablet called Uripas , the pain is not reducing .. .please help me out to which doc should i consult in Kidney foundation Padmanabhanagar. Doctor: Hi,Management of kidney stones depend upon the size and location of stone,Your USG findings is besseray to comment on it,Tab ultracet and meftalspas will help you reduce pain,Your treating doctor will be knowing dose,Visit any urologist over there as they are expert in managing it."
},
{
"id": 45941,
"tgt": "Suggest treatment for internal injury in the right kidney",
"src": "Patient: My nephew, 8years old had an internal injury on his right kidney. Doctors did a surgery on his tummy to remove the blood from inside. after 2 weeks the kid started complaining on pain in the tummy. after an ultra sound examination, doctors found that haematoma around the right kidney had increased & there was free fluid around the haematoma and in the pelvis. - Is there any reason to remove the kidney? - What treatment should he be given? Rgds, Esmail Doctor: Hello and Welcome to \u2018Ask A Doctor\u2019 service. I have reviewed your query and here is my advice. Kidney hematoma are very common after kidney surgery or biopsy, treatment of renal hematoma depends upon the size of hematoma, if it is big enough causing the pressure symptoms over kidney then it should be removed and it can be easily removed by transcutaneous aspiration of hematoma( by passing a large-bore needle through skin). Before removing the kidney a doctor should make sure that the kidney function is lost or damaged beyond repair and keeping the kidney in the body would be harmful, once these questions are answered only then the doctor should go for kidney removal. Hope I have answered your query. Let me know if I can assist you further."
},
{
"id": 219697,
"tgt": "Is pregnancy possible through non penetrative sex?",
"src": "Patient: Hello Doctor Me and my girlfriend had sex on 7th day from her starting day of period. During sex she din t have her pantie but I had my under wear on me. But my under wear got wet and its near to my girl friend vegina. Is there any chance of pregnancy for her? Doctor: Hallow Dear,Firstly, your body play took place on the 7th day of menstruation. There is no egg present on this day. The egg is released 14 days prior to next expected menses and has life of only 24 hours. The sperms are active for 72 hours. Hence a period on about a week to 10 days around the day of ovulation is fertile period. This is roughly from day 10 to day 20. So there is no possibility of conception by intercourse on day 7. Moreover, body play with clothes on and no vaginal penetration by penis has no chance of pregnancy even if the ejaculation takes place in the clothes. However, to avoid such stressful situations in future, I would advise you to practise some reliable contraceptive methods like condom, birth control pills or intrauterine contraceptive device. I hope this helps you.Dr. Nishikant Shrotri"
},
{
"id": 6249,
"tgt": "Trying to conceive, brown spotting, queasy stomach. Could I be pregnant?",
"src": "Patient: Hello, I have been trying to conceive a child now for 17 months now. This past month my husband and I have decided to stop trying. Now I am 3 days late with very light spotting brown not red at all. And it keep stopping no cramps just very tired and my stomach is very uneasey having to use the restroom quite often. Could I be pregnant? Doctor: Hello, Thanks for writing to us. Your symptoms may be related to early pregnancy. You can get urine pregnancy test done using early morning urine sample. If any doubt persists, can get blood B HCG levels done. Consult a gynaecologist. Take care."
},
{
"id": 88535,
"tgt": "Suggest cure for sickness in the stomach",
"src": "Patient: My mom has hada sudden onset of mottling on her abdomen. I am curious what the cause is. Her back story is as follows: she is 3 weeks post op from a modified Whipple( removed body and tail of pancreas, instead of the head) she went to surgeon s office on a Wednesday to have JP drain removed. Started vomiting about 4-5 hrs after getting home. Continued vomiting until she went to ER sunday. Saturday night her abdomen was normal. Sunday when she was in ER her abdomen now has mottling across the whole thing. I didn t see her surgeon in ER. When I did see him on Monday, I asked about and he has no idea what has caused it. Any ideas???? Doctor: Hi.Thanks for your query. Read the history about your Mother and understood it. The cause of sudden mottling on the stomach after the drain is removed after modified Whipple's surgery can be due to following reasons:-Commonest reason is some sort of allergy, for which a detailed history is need. -There might have been leakage of pancreatic enzymes, which can cause this. -The spread of infection in the subcutaneous planes can also cause this. If the vomiting is persistent, if the other signs or symptoms develop like continuation of vomiting, distension of abdomen, constipation or obstipation, fever an so on, she needs to be admitted; Nasogastric tube passed and kept on intravenous fluids and antibiotics. Blood and urine investigations will be done. CT scan of the abdomen will help to diagnose a few more things and the course of action will depend upon the progress of the symptoms / signs / vitals and complications if they develop. Take care, get her admitted and do not take your Mother home till she is cured well."
},
{
"id": 65116,
"tgt": "How can a lump on the neck be treated?",
"src": "Patient: My 3 year old daughter woke up with a lump on the side of her neck . it is hard and close to the size of a apple. The other side is slighty enlarged to the size of ping pong ball. She is otherwise well. The Dr checked her and ordered bloodwork and a chest xray. She has had no illiness since January(6 months ago) How concerned should I be Doctor: Hi, dearI have gone through your question. I can understand your concern. She may have enlarged lymphnode. It can be due to reactive hyperplasia, tuberculosis or lymphoma. She should go for blood test, x ray chest and fine needle aspiration cytology of lump. It will give you exact diagnosis. Then you should take treatment accordingly. Hope I have answered your question, if you have doubt then I will be happy to answer. Thanks for using health care magic. Wish you a very good health."
},
{
"id": 213452,
"tgt": "Suffering with bipolar disorder. Which tests will confirm it?",
"src": "Patient: i am fourteen [in 2 days] and i think i may have bipolar but i dont want to go to my gp or tell any friends or family. how can i tell if i have it for certain, i have done an online test and i will include the link and my score but please could you help me. thankyou. [ WWW.WWWW.WW ] _score is 39- Doctor: Hi, Thanks for using the HCM. I am happy to address your questions, bipolar disorder (also known as manic depression) causes serious shifts in mood, energy, thinking, and behaviour from the highs of mania on one extreme, to the lows of depression on the other. More than just a fleeting good or bad mood, the cycles of bipolar disorder last for days, weeks, or months. And unlike ordinary mood swings, the mood changes of bipolar disorder are so intense that they interfere with your ability to function. During a manic episode, a person might impulsively quit a job, charge up huge amounts on credit cards, or feel rested after sleeping two hours. During a depressive episode, the same person might be too tired to get out of bed and full of self-loathing and hopelessness over being unemployed and in debt. The causes of bipolar disorder aren\u2019t completely understood, but it often runs in families. The first manic or depressive episode of bipolar disorder usually occurs in the teenage years or early adulthood. The symptoms can be subtle and confusing, so many people with bipolar disorder are overlooked or misdiagnosed resulting in unnecessary suffering. But with proper treatment and support, you can lead a rich and fulfilling life. Many factors are involved in diagnosing bipolar disorder. Your clinician may need to ask about your family history, your recent mood swings and for how long you've had them, perform a thorough examination to look for illnesses that may be causing the symptoms, run laboratory tests to check for thyroid problems or drug levels, talk to your family members about your behaviour, take a medical history, including any medical problems you have and any medications you taken, watch your behavior and mood. Hope I have answered your query. If you have any further questions I will be happy to help. Thanks"
},
{
"id": 35399,
"tgt": "Does inhaling mold cause bleeding in the nose?",
"src": "Patient: My boyfriend was recently cleaning out his mother's basement and it has a lot of mold down there. He said that he inhaled it but it would be fine. Well soon after his nose was bleeding orange. This has never happened before. He's had some headache but not severe ones, and he's been feeling a little dizzy. He's been sleeping a lot and felt a little sick yesterday. I'm very worried because it's not normal. Any reasons why this could be happening? From inhaling mold or something else. Is it something to be very concerned about? Please help. Doctor: Hellow there,I have gone through your problem,In particular of your case i feel that your boyfriend have allergy of mould. After inhalation of mould it can cause varied symptoms like from only itching to bleeding from the nose. It is very severe type of allergy. U need to go for allergy testing. By allergy testing you can know against how many type of substance you have allergy.Please gone through this allergy testing."
},
{
"id": 51614,
"tgt": "What medicine should I take for Nephro syndrome ?",
"src": "Patient: Hello Doctor , I am a patient suffering from Nephro Syndrome and has been advised for the following dose of Alphadopa 250, Olmesar 40, Cilacar 20, Omnacortel 40, Clodrel 75, Lamino, & Renodapth S 360 for a day.Sir, my B.P. remains 160/110 on an average and Sr. Creatine 2.14 as well TLC 14000. Please suggest me if the medication is going in right direction or i should change the same. Also how long should i continue with this treatment. The size of kidney is also reduced, Left 8.8cm & Right 9.1cm. Expecting a lot from you, Thanks & Regards. Doctor: hi Mr Bhushan read your prob, you have suffering from nehrotic syndrom . n you have high BP.TLC is high shows presence of infection. first you have to control your BP, it should be 120 80 not more than it. as high BP again affact on kidney. treatment should be continu for 8 to 10 month. after then by diet control n controlling BP you can prevent kidney for further damag. Dr aseem 9982583020 aseemadhuri@gmail.com"
},
{
"id": 110863,
"tgt": "Suggest treatment for back pain",
"src": "Patient: Hello I have a constant pain in my lower left back. It feels like I was hit with a line drive from a baseball. The area of the pain is the size of my fist. I d feel the pain during walking, sitting, lying down it never goes away and has been hurting for 3 weeks with no improvement. There is no bruise I have had no trauma or impact to that area. It hurts worse if you put pressure on the area. I have no other symptoms? Doctor: Hello, Thanks for your query.From description , it seems you might developed muscular or ligamentous strain over back .Now after 3 weeks if pain is severe ,I would suggest getting this evaluated by an orthopedician for an accurate diagnosis and appropriate management. You may get the MRI of the area affected under his/her guidance.Meanwhile following measures will help with the pain\u2022 Lie on a hard bed.\u2022 Anti -inflammatory drugs like Tablet Motrin 1 tablet as and when required \u2022 Avoid forward bending.\u2022 Avoid strenuous activity & lifting heavy weights.\u2022 Apply diclofenac gel on the affected area. Warm compresses will also help.I do hope that you have found something helpful and I will be glad to answer any further query.Take care"
},
{
"id": 56940,
"tgt": "What causes elevated SGOT and SGPT with normal health conditions?",
"src": "Patient: m a 32 yrs old male...previously i had a calculas in ureter,during ultrasound i found condition hydrouretronephrosis alongwith grade 2 fatty liver changes.the problem of stone was solved by URS.then i tsted for liver test like sgot--41u/l and sgpt--77u/l.then after5days again tested found sgot--53 and sgpt99. then in third final test value after 7days is sgot 51 and sgpt 98, so whats the reason behind it yet m with normal health and appetite.whats the critical value of sgot and sgpt are? is this limit is curable or not?are there others lab tests to be performed by me?so provide me the clear suggestion. thanks.. Doctor: Hello in the absence of symptoms abnormal sgot and sgpt could be caused byAlcohol or vir hepatitisI would suggest you get tested for hepatitis B and hepatitis CIn case you don't drink and the above thests are negative then I would repeat the sgot and sgpt levels after 3 months if they are still raised raised you might need further investigations. Fat deposition in the liver can also cause the rise but you need to rule out the conditions mentioned above.Hope this helpsDo get back in case you have further queriesDr Samir Patil"
},
{
"id": 38807,
"tgt": "Does Epstein-Barr virus cause nausea, vomiting and diarrhea?",
"src": "Patient: My 8 year daughter has had a fever off and on for 6 days. She has seen the doctor twice. Her first symptom was a sore throat. Since then she has had nausea, vomitting and diarrhea. She also complans of a pain in her side. The doctor kmows of of this but since she tested positive for the Epstein-Barr virus (we are waiting blood results), nothing else is being done. I am more than concerned. The doctor says EBV would explain all her syptoms but why is the fever almost continuous. She has definitely loss weight and now stays in bed. Should I seek a second opinion? I have seen two different doctors in the same practice. PLease help! Doctor: Hello,Welcome to HCM,Infection with Epstein-Barr virus (EBV) is very common and usually occurs in childhood or early adulthood. EBV is a contagious infection that spreads from person to person.EBV is the cause of infectious mononucleosis and the symptoms arefever, sore throat, swollen lymph nodes in the neck, and enlarged spleen. The symptoms like nausea, vomiting and diarrhea are very much rare with EBV infection.Individuals with EBV infections requires1.Adequate rest 2.Plenty of Fluids3.Corticosteroids to treat significant swelling in the throat4.There is no specific medicine to treat EBV infections.Thank you."
},
{
"id": 36530,
"tgt": "How to treat urine infection?",
"src": "Patient: Hi started trimethoprim but my infection seemed to get worse ! Now I m been treated with nitrofurition antibiotic . The doctors didnt take a sample of urine to culture ! Should they have done ? And is the second antibiotic stronger than the first . I am still in a lot of pain . Doctor: Hello and thanks for your question. Doctors usually take a urine sample and send it for culture to make sure they have chosen the correct antibiotic. If you are still having symptoms you need to go back to the doctor and ask them to send a urine culture. Good luck to you."
},
{
"id": 182562,
"tgt": "Suggest remedy for severe tooth ache",
"src": "Patient: Hi im really in need of a Dentist, I have had this really bad tooth ache that feels as if someone is pulling my tooth and its a pain that is so hard to expalin that it leaves me kicking and screaming and crying and i dont know what to do. I cant seem to get an appointment till the middle of June and i cant wait that long. Doctor: Thanks for your query, I have gone through your query.The severe pain you are describing is because of the tooth infection (acute apical periodontitis). Nothing to be panic, Consult a oral physician and get the tooth treated with root canal treatment or extraction of the tooth. Get a radiograph done to see the extent of the infection and suitability of the tooth for the treatment. If i am your treating doctor, i would have prescribed, a course of antibiotics like amoxicillin 500mg and metronidazole 400mg tid for 5 days (if you are not allergic). Along with antibiotics, you can an analgesics like diclofenac 50mg BID for 5 days. But these medicines will provide a temporary relief. Once you complete the course, try to get the tooth treated as early as possible.I hope my answer will help you, take care."
},
{
"id": 190927,
"tgt": "How to remove nervous pain in the teeth and gums ?",
"src": "Patient: hi my mother has nervous pain in in her mouth gum s her age 54 years dose she use artificial teeth (removable). this pain is from 12 years and now she removed her all teeth. and she had problem of bed breath this happens when she sleep how this nervous pain goes... Doctor: Hi Mudit, Welcome to HealthcareMagic Forum. Is she Diabetic or Hypertensive? She may be having Gum Sensitivity, you should get her examined by a Orthodontist, so that he can examine her thoroughly and Prescribe medications. Since she is having Bad Oral Breath, which is called Halitosis, she is not taking good care of her artificial teeth and also her oral hygiene. Kindly take her to the Orthodontist at the earliest. Good Luck."
},
{
"id": 178457,
"tgt": "What does redness and slight bleeding in 3 year old s clitoris indicate?",
"src": "Patient: My 3 ur old daughter clitorous appears to be red and slighty bleeding? She has just been potty trained and using it on her own now for a few months we let her wipe etc. Not sure if that has anything to do with it? We shower 4 times a week. She complained it hurts when trying to wash her private Doctor: Trauma mild injuries are common when children are wiping due to faulty wiping technique as they are very new and after that irritation of clitoris make them scratch with finger and causes further trauma with mild bleeding and nothing to worry clean the area and use fuscidic cream if u see any raw dended area"
},
{
"id": 575,
"tgt": "Can pregnancy happen after non penetrative sex?",
"src": "Patient: Hi Dr.,me and my girlfriend was masturbating each other and before i touched her vagina,i pulled down my underwear so im worried that some pre ejaculate sperm may get on my fingers,so i used a hand sanitizer and rub it,and then wipe it on a plastic bag.No penetrations are occurred here,just rubbing,help can by any chance she get pregnant and what are the chances Doctor: Hello, I just read your query. Pregnancy can not occur without penetrative sex. When penis penetrates into vagina and semen ejaculated in female partner,only then pregnancy occurs. *The ovulation time period of your female partner also matters.*But if you are still in doubt,then wait for the periods to come. If they come on regular time then there is nothing to worry. Otherwise you can go for a urine pregnancy test if periods does not come regularly.* I hope you find my answer helpful.Thank you."
},
{
"id": 91236,
"tgt": "What causes dizziness,cramps and pain in lower abdomen?",
"src": "Patient: I am in my 16th week of pregnancy and went in for blood work today, since then I have been experiencing dizziness, lightheadedness, cramps in my lower abdomen and very strong headache. I'm currently on amoxocillin. For a ear infection and cold. Should I go see mmy doctor? Oh and two of my household members have bronchialitis. Doctor: hi your syptoms seems to be related with side effect of amoxicillin. i strongly suggest to start probiotics and buscopan. Wishing you all the best Dr.Klerida"
},
{
"id": 148122,
"tgt": "Is having giddiness, head spinning and frequent cold related to fits ?",
"src": "Patient: My daughter is 13years old she is having giddiness, head spinning for past one month and she is also having cold frequently whenever she cries, dust allergies and even whe we wrap onion, we went to ENT specialist he gave T.Verting 8mg for one week and he told that may be in ear fluid will be less or it may be fits, i am scared on hearing this now i want to know whether its fits and why its coming Doctor: Hi,Had it been due to fits, the symptoms would have been episodic, lasting for only a few minutes. The description that you have given of symptoms persistent for last one month and associated with cold are more suggestive of inner ear dysfunction. The inner ear structures contain a fluid and they are necessary for maintaining the balance of the body. Any dysfunction results in dizziness and vertigo. It should improve with Vertin (betahistine) tablets. Please do not worry. It does not appear to be fits.Best wishes."
},
{
"id": 40805,
"tgt": "Suggest treatment to conceive after IVF treatment produced negative results",
"src": "Patient: hi I am 32 year old female. my height is 5 4 and weight is 140lbs. I got married in 2008. I am trying to conceive but no luck yet. All tests are normal for me and my husband. In july 2010 we went to jalandhar india for IVF which came negetive. please advise what should be my next step Doctor: Haiwelcome to hcm Since you are 32 yr in vitro fertilisation techniques are the best way to conceive. Check your endometrium per status. If your embryos are good I'm first ivf you can try with your egg .Consult your infertility specialist. Regards Dr. Vanitha devi."
},
{
"id": 3994,
"tgt": "Will i be able to conceive after taking 'regesterone'?",
"src": "Patient: hello doctor i missed my period for 2 monthd i.e for dec and jan, I consulted doctor she asked me to take regestrone for 5 days and now i have got my cycles also. As i am 24, 5.3 and 52 kg and never ever missed my period before for the first time I have take the meidicine. Well also I am married and plannig for kids I want to know that by taking regesterone will i be able to concieve or will be facinh any problem, plz help Doctor: dear,first if u have missed ur periods for 2 months then check for ur pregnancy by doing a home pregnancy test.and if that is negative then only u may take registerone for withdrawl bleeding.otherwise u may be pregnant if u have not checked yet.if test negative then ur medicine is right for u.it will not create any problem in conceiving"
},
{
"id": 42108,
"tgt": "Suggest treatment for infertility",
"src": "Patient: hello my husband and I are trying for a baby for the past 8 month and he is 22 yrs old I'm 36 his sperm is always watery and leaks out of my vagina after intercourse what can we do? We going to see the GP but my husband works he never have the time to go. Doctor: Hello,Welcome to healthcare Magic.I have gone through your query and Would like to reply in detail as follows:1. Watery semen is not a concern.2. Get semen analysis doneI hope your doubt is clarified. Let me know if you wish any other information.Regards,Dr. Mahesh"
},
{
"id": 52645,
"tgt": "How can gallbladder stones be effectively treated?",
"src": "Patient: I have a 1.7 cm. gall bladder stone which is causing many attacks and much discomfort with many symptoms. Should I have just the stone removed, or the whole bladder. I am receiving different opinions from different doctors. Thank you so much for your advise. Doctor: Hello,Symptomatic gall bladder stone has to be treated. As you have much symptomatic cholecystitis attack, cholecystectomy that is the removal of gall bladder operation has to be done. So your whole gall bladder needs to be removed. Consult surgeon for examination and discuss all these.Hope I have answered your query. Let me know if I can assist you further.Regards, Dr. Parth Goswami"
},
{
"id": 3868,
"tgt": "How to conceive?",
"src": "Patient: hi,I am 25 year old i am trying to concive last 7-8 month my husband test normal then doc. say me follicur study then on 14th day doctor say ovaculation is the my reports good or bad? can i conciving this time? but this time only one time sexwith us. Doctor: Yes your report is normal since follicular study on 14th day showed ovulation. The sperm lives up to 3-5 days in the female genital tract. The lifespan of ovum is 24 hours. So if you have had sex within 5 days before the date of ovulation or on the day of ovulation you can conceive this time.To increase your chances of conception you need to have healthy life style i.e. stop smoking if you are a smoker. Should not drink more than 1-2 units of alcohol per week. Your BMI should be around 25. If it is more you need to reduce weight. One more important thing is you should have regular intercourse 2-3 times/ week.If you don't conceive in 3-4 months time period consult your doctor and get the fallopian tube patancy test done.If you have any further queries feel free to ask me. Good luck"
},
{
"id": 83371,
"tgt": "Shall i do only physiotherapy by giving up neugaba medicine?",
"src": "Patient: I M SUFFERING FROM MILD ROTATOR CUFF TEAR IN MY RIGHT SHOULDER. . JUST NOW TAKING NEUGABA 75, PAIN IS SUBSIDED, TAKING IFT & ULTRA SONIC RAY FROM A PHYSIOTHERAPIST. NOW I WANT TO GIVE UP THE MEDICINE NEU GABA..??..IS PHYSIO..OK.?? WHAT SHOULD I DO...?/ Doctor: Hi,Neugaba 75 is a medicine used to relieve pain caused by nerve damage due to diabetes, widespread muscle pain and stiffness in people with fibromyalgia. It acts by increasing inhibitory chemicals like GABA in brain which inhibits pain.You can stop the drug and continue with physiotherapy and check for any discomfort. Eventually when there is no pain or discomfort, you can continue with only physiotherapy. Kindly rule out diabetes since it can also cause shoulder pain.Hope I have answered your question. Let me know if I can assist you further. Regards, Dr. Saranya Ramadoss, General & Family Physician"
},
{
"id": 82131,
"tgt": "Can multiple clots in lungs cause severe pain in legs?",
"src": "Patient: I have had previous multiple blood clots in my lungs on two separate occasions,they said they traveled from my legs.This was about 4 years ago.I am not on anything for those anymore.But I have just started experiencing,excruiating pain in my lower right leg,and it feels like someone is shocking me.Its sharp and constant. Doctor: Thanks for your question on HCM.In my opinion you should get done bilateral venous doppler study of lower limbs to rule out Deep Vein Thrombosis (DVT).As most common cause for lung clots (pulmonary embolism ) is DVT.And you need to take blood thinner medicines after diagnosis of pulmonary embolism. You are having constant shocking pain also. All these favours possibility of DVT.So better to consult vascular surgeon and get done venous doppler. In this test flow in veins is checked and any obstruction due tothrombosis is there or not can be picked up. So get done this."
},
{
"id": 113777,
"tgt": "Having back and hip pain, cramping and odourless discharge, bloated stomach. What's going on?",
"src": "Patient: I ve been experiencing lower back pain and hip pain, female cramping which produces a thick white odorless discharge , tired all the time. Stomach is bloated. I m 29, I receive my period on time every month. I exercise and eat very healthy and have been maintaining the same weight yet my clothes keep getting tighter and tighter. Doctor: Hello. Thanks for writing to us. The thick white odorless discharge without any itching might be a normal variation. Pain in the lower abdomen needs to be evaluated after a physical examination and investigations like ultrasound scan. I hope this information has been both informative and helpful for you. Regards, Dr. Praveen Tayal drtayal72@gmail.com"
},
{
"id": 146006,
"tgt": "Can strong emotional response be common during vertigo?",
"src": "Patient: My husband has been struggling with vertigo. He has one symptom that seems particularly strange. When he is doing the manuevers that are supposed to help him, he has a strong emotional response and cries. Is that a common thing when doing these movements? Doctor: HIWell come to HCMGiven history is suggestive of some functional condition and may not be having any pathological reason, so just nothing to worry, he might be in a need of some moral support, and that would be okay for him, hope this information helps, have a nice day."
},
{
"id": 176708,
"tgt": "What causes poor weight gain in an infant?",
"src": "Patient: brest fed is not enough so i am giving farex formula to my baby 5 month old but she is not gaining weight iam giving 60ml for 2 hrs once. she is not awake in night time what shold i do for this iam worry about my baby weight. born weight is 2450 kg anf now 5 kg Doctor: Hello. I just read through your question.I understand your frustration with the feeding of your baby. I want to reassure you, however, that the weight that you report is not abnormal. We expect babies to double their birth weight in the first four to five months of life. The numbers you report indicate that your baby's birth weight has doubled. Typically, we recommend beginning baby foods at this time in a baby's life. Adding baby foods can potentially have a positive effect on your baby's weight. I recommend consulting with your doctor about your baby's diet. However, I again reassure you that the weights that you report are normal."
},
{
"id": 12826,
"tgt": "Suggest remedy for vaginal rash in children",
"src": "Patient: my daughter is one year and 1/2 she has has a ugly rash in her outer part of the vagina and her butt a little in her legs and about four in her chin shes had it for four days and it hasnt gone away they say its something like shingles her vaccines are up to date and that rash is red a few looked like if they had water in them and popped Doctor: Hello,I read carefully your query and understand your concern. The symptoms of your daughter are very likely related to shingles.I suggest using an anti-inflammatory medications such as acetaminophen to relieve the pain.I also suggest to use an antiviral cream for local application such as Acyclovir cream.I recommend to maintain a good hygiene.Hope my answer was helpful.If you have further queries feel free to contact me again.Kind regards! Dr.Dorina Gurabardhi General &Family Physician"
},
{
"id": 59216,
"tgt": "Lab report showing renal mass due to hydroureteronephrosis, renal enlargement and lymphadenopathy. Suggestive of hepatic metastasis",
"src": "Patient: Hi Doctors,My dad result from his lab exams, impression/solid left renal mass suggestive of neoplastic process with secondary mild to moderate left hydroureteronephrosis, non specific right renal enlargement, right suprarenal solid mas suggestive of adrenal neoplasm presumably metastatic..paraaortic and peripancreatic lymphadenopathy.. nodal metastasis is considered, right hipatic lobe solid nodule suggestive of hepatic metastasis.. cholecystolithiasis. Here is no evidence of biliary ductal obstruction, grade 1 prostatic enlargement....my dad is in the philippines and my sister just texted me the result, what is this all about? Are these curable? Thanks from Canada Doctor: Hi and welcome to HCM. Unfortunately this sounds very serious. This is expanded malignant disease and prognosis and treatment are hard to predict, but I cant tell details without more info. It is always possible that all these masses and changes are benign, but CT scan is usually accurate about this and this looks like primary kindey cancer with multiple metastasis. This is hardly curable,almost impossible. First the surgical procedure should be performed and then chemo or radiotherapy or both. All in all long and difficult treatment is neccessary,but I hope that everything can still be well controled. Wish you good health."
},
{
"id": 50401,
"tgt": "Changed from Crestor to Lipitor. Having pain in kidneys when blowing nose. Kidney damage?",
"src": "Patient: I was taking Crestor for high colesterol med for about 2 yrs. when I began to feel extreme muscle fatigue and pain, my Dr. took me off and changed me to Lipitor . It has been about 1 yr. and starting to feel it again. I looked up the symptom and realize that this is very severe. And could be kidney damage So I stop taking my meds about 2-3 days ago. I feel much better. I want to know is when I blow my nose I am feeling a pain in my kidneys... Is this something to worry about?. I have a phone call in... But no one has called me back yet! Doctor: Hi, long term use of statins (Lipitor,crestor etc) can lead to various musculoskeletal symptoms. Usually they subside with dose reduction or changing to other molecules. They rarely produce any direct problem to kidneys. If you have pain while blowing, it is better to get completely ckecked up by nephrologist. Wish you good health."
},
{
"id": 146406,
"tgt": "Should I take Nediclon for pinched nerve causing knee discomfort?",
"src": "Patient: getting over a pinched nerve. little to no discomfort in left knee (finally). Been icing, stretching and massaging. A friend gave me 6 Nediclon (round yellow pills with no markings - but packet says Nediclon - 100mgs. Should I take them...or trash? thank you, Harry Doctor: Hi, I had gone through your question and understand your concerns. You don't have to use Nediclon ( Diclofenac), since you feel better and it is a NSAID drug with possible effects. Previous therapies that you mentioned gave you improvement, so I encourage you to continue them. Hope this answers your question."
},
{
"id": 4613,
"tgt": "Any chance of getting pregnant when on antibiotics and birth control pills. Any need of taking plan B?",
"src": "Patient: I had sex two days ago while on antibiotics and birth control I used a condom too but it skipped down about an inch I was wondering if I could get pregnant and if I should take plan b ? Also I started antibiotics and birth control on the same day, does antibiotics actually effect birth control ? Doctor: Hi,Thanks for the query. Sometimes antibiotics can affect the function of birth control pills by disturbing their metabolism. But if you used the contraceptive pills as per the guidelines the possibility of pregnancy is less. For more details : http://srsree.blogspot.com/2013/11/general-guidelines-of-using-combined . telAs, you used condom also, if there was no ejaculation, there is less possibility for pregnancy. So, continue to pills and wait till your expected date of withdrawal bleeding. Possibly you will get withdrawal bleeding as expected. If not consult gynecologist once and go for pregnancy test. As the possibility of pregnancy is less, emergency pill may be needed. Heavy dose of hormones can lead to menstrual irregularity. Take care."
},
{
"id": 206687,
"tgt": "How to overcome the depression and anxiety?",
"src": "Patient: hello, i am only 13, my height is around 158cm and i recently had a sinus infection which has nothing to do with my problem . ( i am also allergic to cats and seafood - except for fish ) but my problem is, i think im going through a depression. i know i sound like an emo but to be honest with you, i have though how committing suicide and the reason to this is i am going through a lot. I had a friend blackmailing me not long and i cried every day for a week. and ive been having arguments with my mom every week which has cause me to literally wanting punch her so badly ever since. you can t imagine things that she would say to me. she would swear at me like im some kind of beast and i know she loves me but she gets pissed off so easily. and it drives me crazy. i also think that my family has some anger issues after seeing how mad people in my family get ( including me ) but just need your advice to tell me what to do in times like this. and im still not talking to my mom. and im glad im not. because despite her good mother advice, she can t help to shout at me about everything i say. and she never appreciate anything i do, she would ignore me all the time to be honest. she doesn t even talk to me. i hate her. i do. and i was planning on moving with my brother ( in an other country ) so i can have my own freaking freedom for once. but AGAIN, she starts going around and telling people how its a wrong decision and how i am making a fool of myself. she never realize the good things i do and always mentioning how bad i am all the time. please help me before i do something bad. Doctor: DearWe understand your concernsI went through your details. I suggest you not to worry much. I sincerely suggest that you are victim of your own negative thinking. You cannot term your current state of mind as clinical depression. Rather, this is just disappointment. Disappointment which arises out of unforeseen life events do bring such symptoms like sadness, irritation, anger, lethargy, lack of interest, lack of appetite etc. Please understand this fact and make sure to introspect. Find the reason for your disappointment and consult a psychologist if need be. If you require more of my help in this aspect, Please post a direct question to me in this URL. http://goo.gl/aYW2pR. Make sure that you include every minute details possible. I shall prescribe the needed psychotherapy techniques.Hope this answers your query. Available for further clarifications.Good luck."
},
{
"id": 56090,
"tgt": "What could be the reason for elevated liver enzyme levels?",
"src": "Patient: 50yoa caucasion male, non smoker, non drinker. 72 tall and weight is 245. only health problem is blood pressure, I just started taking 20mg of Lisinopil. as part of my yearly exam I got a liver functions test..my AST is 32 and my alt is 61. (test indicates 9-46 U/L is normal). I have recently taken to consuming apx 500-700mg of ibuprofen for a tooth ache. can you tell what is going on with my liver? is this something I should worry about. My doc has me set up for hepatitis series next week to check that. Doctor: HelloIncreased ALT may indicate liver injury.It may be due to many reasons like hepatitis,alcohol intake,altered lipid profile,auto immune causes etc.You may need few more investigations like routine hemogram,random blood sugar,lipid profile,ANA estimation,viral markers,urine RE/ME,ultrasound of abdomen.I suggest tablet ursodeoxycholic acid 300 mg twice daily for three months to my patients.It helps in regeneration of liver cells.Further requirement of medicines depend upon findings.Findings are not related to recent intake of ibuprofen.Get well soon.Take CareDr.Indu Bhushan"
},
{
"id": 217846,
"tgt": "What is the solution for pain in lower back, shoulder and arm?",
"src": "Patient: I have pain the lower back and at times it feels strained. Pls advise on how to deal with this problem... the pain started with the scaiatic nerve and though I have taken treatment, the weariness in the back continues. I also have pain the right shoulder and arm. Pls advise. Melba Doctor: Hai.pain in the back, sciatica are related pains.u should not do vigorous exercises like bending, lifting weights. Pain in the shoulder can also be due to radiation from the back. Can go for relaxing physiotherapy exercises, opioid therapy, drugs like pregablin added with methylcobal"
},
{
"id": 92541,
"tgt": "Sharp pain in stomach when pressed. No other symptoms. Reason for pain?",
"src": "Patient: hi, i'm in a kinda hurt right now. Sinc yesterday when i press on it my stomach hurts. It's not a really extremely pain but it hurts when i press on it and it's also kinda sharp. I went to the hospital today, they said that everything is fine and that i should wait for it but i'm not really comfortable with that and i don't want to hear something bad about it cause i'm already kinda panicked. I have a normal poo colour, normal yellow color of my pee and have no any type of illness or sickness. I would love to hear what the problem could be. Doctor: Hi,From history it seems that you might be having gastritis or abdominal colic.Take anti spasmodic tablet like Dicyclomine and Omiprazole tablet for 1-2 days.Avoid fried and junk food.Take light diet.Ok and take care."
},
{
"id": 220984,
"tgt": "What causes fluctuation in platelet counts?",
"src": "Patient: i have fluctuation in platelet counts...it goes as much less as 50% of the minimum count and becomes normal without any medication after a week or so. now i am in trimester (6th week). wl this have any complications in my pregnancy. what diet should i take to increase my platelet count Doctor: Hi there,More than the fluctuations in platelet count, we need to make sure that it doesn't get to be less than 1 lakh/dl. Platelets are known to fluctuate in pregnancy. But the lowest level should not be less than one lakh. As this will need precautions during delivery. And platelet levels less than 50,000 do sometimes require platelet transfusions. But do not worry as most low platelets in pregnancy are gestational and revert back to normal themselves after delivery. There is not much relation of platelets to diet and there is no recommended diet.Hope this helps.Regards."
},
{
"id": 142667,
"tgt": "What causes a bump on the back of the head?",
"src": "Patient: I have a hard bump on the back of my head that is kind of moveable, I guess is how I'd say it. Its on the back of my head on the right, its about two inches behind my ear.It doesn't hurt. I haven not hit my head on anything to cause a goose egg. I'm just wondering what it could be. Doctor: That will be lipoma the collection of fat at that region have consult with general surgeon who will define that whether it is a lipoma or something else??"
},
{
"id": 18958,
"tgt": "Suggest treatment for T wave abnormality",
"src": "Patient: Dear Dr.,My father aged 71 yrs has undergone an ECG and the findings are as under:1. Sinus rhythm with marked sinus arrhythmia2. T wave abnormality, consider lateral ischemia3. Prolonged QTThe measurements are as follows:QRS:96msQT/QTcB:452/462msPR:166msP:128msRR/PP:1088/880msP/QRS/T:56/26/100degrees.Pls advice Doctor: Hello,A doctor can't say much without seeing the ECG. These changes in T wave might be just from chronic hypertension, or maybe a serious problem like coronary artery disease. I suggest that if your father has any chest pain or difficulty in breathing or other cardiac symptoms to go to a cardiologist. He can see himself the ECG and can make echocardiography to see more things and to give you specific advice.Hope I have answered your query. Let me know if I can assist you further.Regard,Dr. Anila Skenderi"
},
{
"id": 153180,
"tgt": "Is MX3 helpful in esophageal mass cancer?",
"src": "Patient: my husband is diagnosed with esophageal mass cancer last mo.,,then we were told by our dr.tohave ct scan in preparation for the radiationtherapy,,we followed him to have a ct scan,,but we did not go back yet to give the result,,right after i heard of mx3,,last april 15 until now,,he is taking this meds 3x a day,,and im hoping we will succeed,,,he is 5'8 in height,160lbs.,,Before he was diagnosed of cancer,,he vomitted blood and rushed to a nearby hospital in the province,his bp was 70/60 so right after he was released ,,i brought him to Manila for endoscopy,,there he was find out to have a mass in his esophagus junction,,and had a biopsy,,with that impression,malignant,,,,so,,before we will be referred to onco dr.,i told my husband to give a try with mx3. Doctor: Hello! I read your concerns and feel sorry for your husband. I have researched about MX3, it is only a food supplement and does not have any therapeutic effects. The primary treatment for Esophageal cancer is still resection/surgery. However, for locally advanced esophageal cancer (unresectable tumor), chemotherapy plus radiation can be done to decrease the size and to increase the resectability of the tumor. For more advance stage (with distant metastasis), palliative stenting can be done together with chemotherapy.I hope your husband gets well soon."
},
{
"id": 161433,
"tgt": "How can fecal incontinence be prevented in a child while on Movicol for encopresis?",
"src": "Patient: Hi my 8year old son has been diagnosed with encropesis and the GPS we go to has given us movicol for it and said there is not much more we can do about it... well its is getting worse is there anything I can give him to solidify the stuff that s leaking which is causing him to spoil everything and get the hard stool that is stuck out Doctor: Hi, Well, wait for final results. In the start, it will look like a worsening condition but it will take time. Eventually, the bowel that is stuck would be defecated. Hope my answer was helpful. If you have further queries feel free to contact me again. Regards, Dr. Maimoona Saeed, General & Family Physician"
},
{
"id": 10700,
"tgt": "Suggest remedy for hair loss",
"src": "Patient: I'm 19 years old & suffering from right-sided hair loss for 2 years. I'm a poor sufferer after using a bad & cheap hair dye. My hair from that side is very weak,unhealthy & falls so easily. can u people help me? I'm scared! I'm 5 feet tall,weight-63kg.I'm tired because doctors said my hair fall is normal,I'm just obsessive!They don't try to believe the truth. I spent lots of money for the purpose but got no result. I'm so frustrated now & even stopped my study for this frustration.please help! Doctor: hi there.1. your concern may be telogen effluvium leading to androgenic alopecia.2. you can try neutriderm antihairloss lotion.3. apply enough quantity into the roots, 3 hours before bath.4. then rinse off with sebamed antihair loss shampoo.5. repeat it daily until hair fall controlled.this may help you in controlling hair fall."
},
{
"id": 61407,
"tgt": "What causes a painful lump below the clavicle?",
"src": "Patient: Hi, Someone was on top of me and had all their body weight from their palm on my chest right below my left clavicle and now there is a golf ball size lump. I have super sharp pain in my back and it hurts to breathe deep and coughing is almost unbearable. It's been about a week and a half and I need to see if this is dangerous to not see a doctor. It feels like a muscle strain in my back though. even though it was caused in the front from being pressed on way to hard. Any advice? Doctor: Respected user , hiThanks for using Healthcaremagic.comI have evaluated your query thoroughly .* Seems hematoma formation most likely with referred neuritic pain over back side .* Should immediately seek medical attention without delay as the symptoms severity is much more according to the narration , take tab.tylenol (500)mg stat dose for instant relief .Hope this clears your query .Welcome for further assistance .Regards ."
},
{
"id": 150579,
"tgt": "MRI showing mild to moderate chronic ischemic changes, mild frontal leukomalacia. What does it mean? Numbness in the arm",
"src": "Patient: What does mild to moderate left greater than right pontine chronic ischemic changes, and mild right frontal leukomalacia which is related to chronic small vessel ischemic disease but no acute infarction or mass on MRI really mean. My 65 year old mother has had numbness and decrease in grip in left arm and hand since going to ER Doctor: Hi, Thank you for posting your query. MRI brain findings as described, in your mother, are suggestive of decrease in blood supply to certain parts of brain, also referred to as brain stroke. Her neurological deficits such as numbness and weakness are a result of that. Treatment would consist of blood thinners such as aspirin, control of risk factors such as high BP and sugars, and physiotherapy. Please get back if you require any additional information. Best wishes, Dr Sudhir Kumar MD (Internal Medicine), DM (Neurology) Senior Consultant Neurologist Apollo Hospitals, Hyderabad, My personal URL on this website: http://bit.ly/Dr-Sudhir-kumar My email: drsudhirkumar@yahoo.com"
},
{
"id": 137232,
"tgt": "Suggest treatment for pain in medial side of feet",
"src": "Patient: I am a female 77 years of age. For the past 3 years I have been bothered by foot pain in the instep of my feet. I have had cortizone shots during the past year in the ankle and haveI am overweight also been diagnosed with fibroid tumors in the arches of both feet. At the present time I am really getting worse. It seems that I can t wear any of my shoes and if I am on my feet for any length of time I get sharp shooting pain in the insteps. I am overweight by about 60 lbs. Doctor: Hi there.You could be suffering from Tarsal tunnel syndrome or Flatfoot. Avoid tight fitting footwear. Apply warm soaks to the painful area. You can take pain killer like Tab. Ibuprofen 200mg twice a day. Consult an Orthopaedic surgeon for a physical examination and management accordingly"
},
{
"id": 188538,
"tgt": "Have toothache. Dentist gave antibiotics. On metronidazole, dihydrocodeine. Have temperature. Suggestions? Medicament?",
"src": "Patient: I've had a toothache for over two weeks. Been to the dentist twice and have been prescribed antibiotics. The latest course is for metronidazole. I also got painkillers, dihydrocodeine, for pain relief. At the moment my temperature has gone up and I'm feeling rather poorly. Took my last dose of tablets at midnight. Should I take some paracetamol to bring my temperature down? Doctor: Hi,As antibiotics are continued for long time and problem is still there, there might having abscess in root of tooth.Consult your dentist again and discuss.At present to control fever you may take paracetamol.Ok and take care."
},
{
"id": 214540,
"tgt": "Suggest home remedies for infection in uterus",
"src": "Patient: my sis has gone through pcr test since she is trying for a test tube baby, but it has come positive so she again has to have medicine for 2 or more months to get it cured.......she is 33 yrs..........seems she has some infection in her uterus...........what are the home remedies to cure this ASAP Doctor: Hai,As there is no proven home remedy for uterus infection.Kindly go through a endocervical(female genital tract) culture to rule out the cause for the infection.kindly visit a gynecologist for proper medication and treatment with the help of culture report.ruling out the organism which causing infection and proper treatment will be the idea way.thank youhope i answered your query."
},
{
"id": 117039,
"tgt": "How to treat DVT?",
"src": "Patient: Hi i suffered a dvt in 1998 aged 23. Suspected contraceptive pill as cause. No dvt for 15 years, despite surgery, long haul flights and a previous pregnancy terminated as I had a coil fitted. However in September this year got pregnant and suffered severe anxiety, depression and was immobile for a few weeks. Doctors, husband and myself all agreed that termination of pregnancy was sadly required as i was suffering mentally and phsically. Had termination and despite a preventative dose of clexane (heparin) I suffered a dvt later that week. There is a family history of dvt in grandparents. I don't want to go on warfarin for life and I'm depressed I may have to. Normally I'm fit and active. I feel hormones are to blame. Please do you have any advice? Doctor: Hi,Thanks for asking.Based on your query, my opinion is as follows.1. One of the causes of thrombosis is a hypercoagulability, i.e. increased risk of coagulation.2. As you have a family history, and already suffered one episode and future risk is present, you need to be on anti-coagulants.3. Hormones are responsible, yes. However, if you try to reduce the hormones, other complications will happen. You need to be on warfarin for life to avoid risks. Hope it helps.Any further queries, happy to help again."
},
{
"id": 189461,
"tgt": "Have blister tip of tongue, painful, irritating, clear naturally, reappear. No diagnosis made",
"src": "Patient: i have a blister on the bottom tip of my tongue and one on top. i've been getting these since i was about 8, i am now 14. i get them every so often and they are quite painful and irritating ! i dont know anyway to get rid of them so i let them clear up naturally.. this usually takes 7-10 days. i dont know why i get them and i have been to the doctors they do not know either. please help me Doctor: Hello and welcome to hcm forum, The blisters can be of aphthous ulcers which is recurring in nature. I would suggest to include nutritious food and fluids into your diet. Vitamin-becosules,folic acid and iron supplements has to be taken. Avoid spicy/salty food . Intake of allergic food items has to be suspended. Rinse your mouth after every meals. Floss your tongue daily without vigorous force. In severe cases,steroids has to be taken after consulting with your dentist. Take care."
},
{
"id": 137473,
"tgt": "What causes joint pain and headache?",
"src": "Patient: hi I had an sex ex poser with a csw in China . I was on 5th of May 2013 . I use condom also but later she told condom is torn .I had gone for test on 24th August 2013 it came negative. And I again went for test on 20th Oct again it was also negative (non reactive). Then again I went for test in Delhi on 24th Dec 2014 . And it was also non reactive . Should I now leave worrying about it . I feel joint pain and headache too much. Rgd V Doctor: Hello,I have studied your case and I would recommend you to get P 24 antigen test.This test will prove if you have HIV or not. All other tests takes long time to become positive and some time it may be up to 6 months to years. Still chances are very less but there is no harm in getting p 24 assay. I hope this answer will be usefil for you.Let me know if there is any other followup questions.thanks"
},
{
"id": 49056,
"tgt": "How can i reduce my creatinine level?",
"src": "Patient: I AM 67, I AM DIABETIC FROM LAST 35 YEARS, MY CREATININE LEVEL IS 1.6, HAVING SOME CARDIC PROBLEMS ALSO, DR PRESCRIBED C T SCAN ANGIOGRAPHY, DUE TO HIGH CREATININE TEST CAN NOT BE DONE, YOUR VALUABLE ADVICE IS REQUESTED TO REDUCE CREATININE LEVEL. THANKS AND BEST REGARDS, MRS FARIDA NAZEER, EMAIL: YYYY@YYYY Doctor: HIThank for asking to HCMI really appreciate your concern let me explain you something that creatinine is not the independent chemical that can be regulate with some exogenous drugs but the underlying cause is matter and that should be treated like the renal function should be corrected if this is due to the infection then that should be treated in short kidney function should be brought to normal, then the creatinine value would comes to normal hope this information helps you, have nice day."
},
{
"id": 185530,
"tgt": "Can stretched nerve while wisdom teeth removal cause numbness in tongue?",
"src": "Patient: back in feb I had 4 wisdom teeth removed and still have tingling burning and numb 2/3 tongue altered taste like metalic.Both sides of jaw including chin and bottom lip numb.Hard to find the right mri image for nerve damage.I believe its from injection in the corner of my jaw because if you touch this area feels like glass shooting through my tongue.Dentist says he may have streatched the nerve.Does he know something he really not telling me? Doctor: Hello, Read your query, as you have numbness in tongue, altered taste, after extraction of wisdom tooth there is nothing to be worried so much this happens when there is nerve injury or you can say stretched nerve this happen when there is traumatic extraction also , for this you can Do warm saline gargle two - three times a day, you can take Neurobion tablet and vitamin B supplements with folic acid by consulting with your dentist for month , You can take supplement protenix also . Take vitamin , protein rich diet, Dont be so much stressful be relaxed take proper sleep of 8 hours . Take healthy nutritious diet avoid junk food. Hope this will help you. If you have further query I will be happy to help.Regards, Dr. Priyanka tiwari"
},
{
"id": 2932,
"tgt": "Can one get pregnant by accidentally rubbing semen containing hands on vagina?",
"src": "Patient: I was out with boy friend he musterbsted I got huge amount if semen on my hands I wipe it off and after 45 I felt I need to go toilet, I went and I washed my Virginia and rubb it from out side . 1.is it possible to have semens under my nail and it didn t dry after 45 minutes? 2.So I can get pregnant? 3. Incase if it was under my nails is that amount enough for pregnancy? I was done from my period and I was on day 9 after period I got period on17 feb and this happened on 25 feb , is it possible ? Doctor: Hi,I understand your concern. Do not worry. There is no chance of pregnancy.1. Semen will not remain under the nail. It will dry in 45 minutes. If it will be present still it can not pass in the vagina without seminal fluid. 2. You can not become pregnant.3. If it will remain in the nail still can not pass into vagina and quantity is not enough to become pregnant. Again you did this act 5 day after period and it is completely safe period of cycle. To become pregnant, penetrative sexual intercourse with or without ejaculation during fertile period of cycle is required. Avoid stress, take healthy diet, drink plenty of water and do regular exercise.Hope this may help you. Contact further if follow up needed.Best regards,Dr. Sagar"
},
{
"id": 79721,
"tgt": "What causes severe chest and back pain?",
"src": "Patient: Hi I m having real severe chest and back pains went to several docs they can t find the reason I m in an old house could it be black mold my aunt died over it my 11 year old grandson has the same symptoms and this house leaks bad what are the symptoms of mold please tell me no one else can sincerely Barbara I m 59 yrs old Doctor: Thanks for your question on Health Care Magic. I can understand your concern. Molds can cause variety of Pulmonary symptoms when exposed for many years. Upon inhalation, these molds can cause constant irritation, inflammatory reaction in airways. So bronchitis and asthma like diseases are common. Molds can also weakens lung defense. So chronic fungal infection can also be seen. So chronic cough, chest pain, breathlessness, wheezing etc are the common symptoms. So better to consult pulmonologist and get done 1. Clinical examination of respiratory system 2. Chest x ray 3. PFT (Pulmonary Function Test) to rule out mold related Pulmonary diseases. Hope I have solved your query. Wish you good health. Thanks."
},
{
"id": 132598,
"tgt": "Suggest medication for spondylolisthesis?",
"src": "Patient: I have spondylolisthesis, scoliosis,with severe bilateral neural foraminal stenosis. When I walk and step with my right leg and my knee starts to bend it is like I have no leg there at all. I do not have any loss of strength in my legs that I have noticed. At times it is worse than other times. Am I going to lose the use of this leg all together? I have been trying to get help for 4 years now and I am still waiting for help. How long do I have to wait? Doctor: Hi Hope this message finds you in good health.I have gone through your complaints and understand your concern.SPONDYLOLISTHESIS NEEDS TO B TREATED WITH A good lumbosacral corset,rest ,analgesics,physio and exercises.If symptoms aggravate,needs to be treated with surgery.Nothing to worry about.\u00a0\u00a0\u00a0\u00a0\u00a0I hope your question has been answered.If you have any follow-up queries,feel free to consult me anytime.Thanks,Take care,God bless."
},
{
"id": 112035,
"tgt": "What to do for chronic back pain other than taking exalgo er and cyclobenzaprine?",
"src": "Patient: Hi. I m a patient with chronic back pain. I m in remission from kidney cancer, Thank God. Right now all i have is exalgo er 12 mg tablets presrcibed to me, but that is not helping. I also have cyclobenzaprine 10 mg tablets prescribed to me, but it s not helping either. Just for the night to get some relief, what should I do? Doctor: Hi there .Since you have kidney disease I would advice you to either take tramodol or tapentadol ....whichever is freely available for the night .Take some hot fomentation for the back and apply some analgesic gel .Sleep on a firm mattress. This should get you through the night.I hope my advice has helpedGood luck"
},
{
"id": 45268,
"tgt": "How to decrease proclactin level & will it affect fertility ?",
"src": "Patient: Hi. I am 24 years old..Last year i have got married.We dont have babies till now.I have undergone hormone tests last week and found my prolactin level is 28.5. may I know how to decrease it?will it affect fertility ? Doctor: hi .. i m also 24 ..n have a high level of prolactin hormone it was 30.16 on day 4th of the cycle but i donot know how high is this... n what is thee normal range ....Dear Sowmiya, will u help me what treatment dr . gave you"
},
{
"id": 126501,
"tgt": "What causes joints pain despite taking vitamin D supplements?",
"src": "Patient: My vit D is at 11. I m on 50,000iu\u00d71wk. My bones hurt my muscles hurt. My hands are cold and feet my arms are very heavy and weak. I m so tiered. My thyroid is fine. I feel like I have some neuropathy going on. Is it protacal to add magnisium with D3. Doctor: Hello, Consult a rheumatologist and get evaluated. It may be an early phase of arthritis. As of now analgesics like Acetaminophen or Tramadol can be taken for pain relief. Hope I have answered your query. Let me know if I can assist you further. Take care Regards, Dr Shinas Hussain, General & Family Physician"
},
{
"id": 18713,
"tgt": "Is dizziness an indicative of low BP?",
"src": "Patient: Recently had a dizziness and lightheadedness experience after standing, then standing after sitting. I am taking Flomax, and Lipitor for cholesterol control. My systolic is in the 140 s and my diastolic ranges from 84 to 67 when lying down. Is this problem related to low or high blood pressure and should I ask my doctor about discontinuing Flomax or adding a high blood pressure med? I am confused here. Doctor: Hello and Welcome to \u2018Ask A Doctor\u2019 service. I have reviewed your query and here is my advice. You should have a healthy lifestyle like avoiding fatty, oily and high calorie diet. Have low salt diet and monitor blood pressure regularly thrice a day for one week then once or twice a week. If BP is persistently more than 140/90 mmhg, then you should be on medicines for it. The likely cause for this is, flomax. It can cause postural hypotension and your symptoms. So you have plenty of water and avoid dehydration. Also, avoid sudden change in postures and prolonged standing. If symptoms are troublesome then ask for change of flomax. Hope I have answered your query. Let me know if I can assist you further."
},
{
"id": 24483,
"tgt": "Can I climb stairs with diastolic dysfunction?",
"src": "Patient: I am male 62. Had CABG in Dec 2008. Recent 2D echo shows mild diastolic dysfunction, LVEF 60%. Other parameters normal. BP less than 110, diabetes controlled with diet and a little medication, taking metolar and rampril. Can climb stairs, slope without problem. Rest HR 75. Doctor: Thanks for your question on Healthcare Magic. I can understand your concern. I have gone through the 2d echo report you have mentioned. At the age of 62, mild diastolic dysfunction is common finding. You aloo had CABG (Coronary artery bypass grafting). So nothing toh worry about mild diastolic dysfunction. Your ejection fraction (EF) is 60% which is very good at this age. So your heart pumping is excellent. So you car definitely climb stairs or slope without any worry or fear. But please done do it fast and take proper rest after climbing 2-3 stairs. Hope I have solved your query. I will be happy to help you further. Wish you good health. Thanks."
},
{
"id": 44896,
"tgt": "Can any online doctor tell me whether my semen report is normal or not ?",
"src": "Patient: Are the following report normal , please advise.. The semen volume 1.5 ml, liuifaction time 30 mintues, total count 25 mil1/ml, active motile: 20%, sluggish motile 10%, non motile 70%, pus cells 4-6/hpf, epithelial cells 3-5/hpf Doctor: hi,thanks for query,the motility of sperms is less and there is some minor infection.Please repeat the test after 3 days of abstinence as in this case of semen normally single report is not conclusive.In case the second report is same then you need to consult a urologist who can do proper examination and give advise. wishing you good health."
},
{
"id": 92934,
"tgt": "Gastroscopy shows stomach ulcers. On antibiotics. Had severe swelling in ankles, feet, eyelids. Inter-related?",
"src": "Patient: I had a gastroscopy and was diagnosed with the beginning of stomach ulcers , I was put onto a 2 week course of antibiotics (Moxypen & Klarithran) which I m still taking as well as Topzole, for the last 3 days I have had severe swelling of my ankles and feet and even my eyelids are swollen and I feel flushed all the time, could there be a relation between the symptoms and the medication?? Doctor: Hi,Thanks for posting your query.I am Dr.Ratnakar Kini and I am pleased to assist you.The swelling you have in the ankles, feet and around the eye lids is not related to the stomach ulcers or the medications you tak.Swelling like these occurs due to either heart, liver or kidney diseases. Puffiness around the eyes usually occurs in kidney disease.You need to get tests done to see which of these organs is affected causing the swelling of the ankle, feet and around eyes.I hope that answers your question.Regards,Dr.Ratnakar Kini"
},
{
"id": 122357,
"tgt": "Suggest treatment for pain in leg",
"src": "Patient: sir my wife having pain in her left leg. she was x-rays in surya ortho in Faridabad . The Doctor said that there is less gap in hip due to that the nerves pressed and left backside pain occur. He also done Physiotherpy for IFT & ILT . He also give the PONDEX(TRAMADOL &PARACETAMOL), ALPHALIPOIC,VITAMINS &FOLIC ACIDS CAPSULE. She take this dose for two month but no any relief from pain .sir pl advice us Doctor: Hi, Physiotherapy is a good and harmless option. You can also use Pregabalin 75 (if there is no underlying contraindication). Hope I have answered your query. Let me know if I can assist you further. Take care. Regards, Dr. Tanmoy Roy, General & Family Physician"
},
{
"id": 201903,
"tgt": "What are the precautions for aids after sex?",
"src": "Patient: hii my name is famaan i am a boy 22 years old.... i am straight guy....i had protected sex with a gay but when i was fuckung the condom breaks and i didnt came to know that it s broken inside....i didnt cum inside his ass max to max Fucking lasts for 45 secs or 1 minute he say s he is clean he wanted me to trust him... but i am scared what should i do now to prevent from aids ... ???.. i am really very scared and parents dont know about this...... : ( Doctor: Hi, if you have a strong suspicion you can have a course of Anti-retroviral medicines for Post-exposure prophylaxis(PEP). It is a regimen of a week or so and can prevent the infection in a large number of cases if started soon after exposure to the infecting agent. You should consult your doctor for the same and start it if he advises. Please be careful in the future, you have a long life ahead. Make it healthy. Feel free to contact me directly on 'Ask a Specialist' section of the website.Dr Rishi, New Delhi, India."
},
{
"id": 30928,
"tgt": "Suggest treatment for tuberculosis",
"src": "Patient: HI I am Dr Shamshad Alam ...I would like to know about a patient who is suffering from cox A female of age 35 was suffering from tuberculosis and medication was given and dr advise her to close the medicine...but she is still seuffering from same condition like dysnea.fever .cough,headche,weight loss etc what should I adivse to her? Doctor: hello, speaking of pulmonary tuberculosis whether it was sputum positive or negative treatment was started of suspicionor basec on CT scan was not mentioned. if treatment was initiated after sputum AFB positivity treatment duration will be of 6 months provided patient has drug sensitive bacilli. if sputum remains positive at the end of intensive phase then suspect MDR TB and send sputum for culture and sensitivity. if patient not improved on ATT suspect other causes of sputum AFB positivity like nocardia and atypical mycobacteria. kindly review the initial diagnosis if it was confirm tuberculosis, check for compliance with treatment and then proceed with sputum culture and sensitivity testing."
},
{
"id": 49263,
"tgt": "On medication for UTI and blood and mucous in the urine in a person born with chronic disease in one kidney",
"src": "Patient: I have a uti and i have gotten medication for it. I am now bleeding and have mucus in my urine. I do have one kidney that has chronic disease , (was born with one). The doctor said to give it three days but the bleeding has gotten worse. Now the stringy mucus is more present. Doctor: HelloThanks for your query,based on the facts that you have posted it appears that you have Please get your routine urine test and urine culture done to find out the organisms causing this infection and antibiotics to which they are sensitive to.Please take broad spectrum antibiotics like Cefexine along with urinary antiseptic like Nitrofurantoin twice daily and urine alkaliser thrice daily.Later on switch on to appropriate antibiotics as per culture report.Get the prescription of medicines from your family Physician.Ensure to drink more water.To keep your urine dilute This will help to control hemauria.Dr.Patil."
},
{
"id": 175843,
"tgt": "Suggest treatment for loose motions in a child",
"src": "Patient: Hi, My son is 20 month old.Heis suffering from loose motions since 2 days(especially day time like 7 stools p.d ).I gave him pedialyte so that he doesn t dehydrate. But its not controlled. It seems like my child is getting some more teeth.can u pls help Thanks shwetha Doctor: Hi...Thank you for consulting in Health Care magic.It seems your kid is having viral diarrhoea. Once it starts it will take 5-7 days to completely get better. Unless the kid's having low urine output or very dull or excessively sleepy or blood in motion or green bilious vomiting...you need not worry. There is no need to use antibiotics unless there is blood in the motion. Antibiotics might worsen if unnecessarily used causing antibiotic associated diarrhoea.I suggest you use zinc supplements (Z&D drops 1ml once daily for 14 days) & ORS (Each small packet mixed in 200ml of potable water and keep giving sip by sip) as hydration is very important and crucial part of treatment. If there is vomiting you can use Syrup Ondansetron as prescribed by your paediatrician.Hope my answer was helpful for you. I am happy to help any time. Further clarifications and consultations on Health care magic are welcome. If you do not have any clarifications, you can close the discussion and rate the answer. Wish your kid good health.Dr. Sumanth MBBS., DCH., DNB (Paed).,"
},
{
"id": 180205,
"tgt": "Is dent on head after an injury an indication of brain damage?",
"src": "Patient: Hi, at 3 weeks old my baby fell off of the bed while laying on my chest when I fell asleep, his cries woke me up but when I picked him up from the tiled floor he stopped crying after about 15 seconds. I checked him for bumps and bruises and he had none. I tried to keep him awake for awhile and kept checking on him, all appeared to be fine but now he is 6 weeks old and I noticed he now has a dent in the back of the right side of his head. What Should I do? Does this mean his skull or brain may be damaged? He still acts the same so does that mean he s ok? Doctor: Hi...do not worry. Your kid is fine.1. Skull or brain damaged kid would not behave like this (you say your kid is fine and this dent is an incidental finding).2. His is ok as he is acting normally and developmentally normal.Hope my answer was helpful for you. I am happy to help any time. Further clarifications and consultations on Health care magic are welcome. If you do not have any clarifications, you can close the discussion and rate the answer. Wish your kid good health.Dr. Sumanth MBBS., DCH., DNB (Paed).,"
},
{
"id": 51966,
"tgt": "Feel pain in lower left abdomen. What could it be?",
"src": "Patient: i feel pain in my lower left abdomen, what could it be? Doctor: ITcould be due to CONSTIPATION OR ELSE KIDNEY STONES IN YOUR CASE,GET DONE ULTRASOUND -ABDOMEN&CONSULT EITHER GENERAL SURGEON OR GASTROENTROLOGIST FOR MORE."
},
{
"id": 9476,
"tgt": "What is the cause for redness around my mouth and eyebrows which is very dry and flaky?",
"src": "Patient: Yes thank you!I have redness around my mouth,above my eyebrows, and the top of my nose bridge. These areas are very dry and flaky. I try washing or wiping the areas but as soon as the flakes are removed it returns. I have very sensitive skin. Sometimes the redness goes away for a few days or weeks but it does return. Not sure if something I am allergic too or using wrong skin care products. Doctor: Hello,Welcome to healthcare magic.From your description, you could be having seborrhoeic dermatitis.It is a condition similar to dandruff on the scalp. It will benefit with a 2% ketoconazole cream used twice a day for 10-15 days. It could recur, hence use a ketoconazole containing shampoo on alternate days during bath. Leave it on the affected areas for 5 minutes and wash off. Use mild cleansers like cetaphil cleanser. Hope this helps you.Take care."
},
{
"id": 45604,
"tgt": "What causes constant pain around the kidney area?",
"src": "Patient: I ve had bouts with kidney stones, having to have one removed surgically a month ago. Starting 3/4 days ago that same kidney hurts all the time, as opposed to the sporadic pain I experienced with the previous stone issue. Why am I having constant pain? Doctor: Hi, This maybe is caused by urinary infection including kidneys infection or urinary stones so you should do urine analysis, urine culture and kidneys ultrasound to rule these out which are most common causes. You should drink raspberry tea and plenty of water until you do tests. You should reduce salt in your diet as well, alcohol and carbonated drinks. In most cases this is self-limiting but sometimes antibiotics are required and more invasive tests such as cystoscopy or pyelography or CT scan. Less common causes include tumors, glomerulonephritis, and metabolic diseases. Hope I have answered your query. Let me know if I can assist you further. Take care Regards, Dr Ivan R. Rommstein, General Surgeon"
},
{
"id": 75030,
"tgt": "Suggest treatment for bronchitis",
"src": "Patient: I m a 50 yr woman, had total hysterectomy 7 years ago. I was sick in December with asthmatic bronchitis and again in Jauary. Both time I spent a week in hospital. Was put on prednisone and was off it by March. Right after I got ou, I started to have stomach issuess, reflux, I iad thrush all the way down my throat, I also had a pounding heart for months. They sent me to cardiologist he said MVP with mild regergitation wanted to use Beta Blockers but I can t take because of asthma so he s doing nothing say s he dosen t think I should be feeling my syptoms. I have a lacey kind of look to my skin, very fatigued sweat profusley at times even with out doing anything and in front of AC. I also had a few night of extrem tingling in hands, face and my hands swelled very badley left more than right. Went to make sure I wasn t having stroke or something. I have gone to a Rheumatologist and all my tests are coming back normal. The fatigue and sweating are so bad I almost can t work. They did Brain MRI and Bone scan to rule out MS, brain cancer and the bone scan didn t show polyarthritis. I also found I have a mottled look to my skin and when I went to tweeze my eyebrows after skin around eyebrows broke out into hives.I don t feel like I m in the right place, but I do know since I was sick I have not returned back to myself and now anxiety is starting to take over any clues or ideas? Doctor: Respected madam, hi sorry to hear about your issue.Most probably this is (1)steroid induced g.i.problems(2)later on effects of steroid withdrawl.Many simple tiny measures you have to take to come back to normal which will take time.* Maintain your hydration.* Maintain BP , Blood Sugar levels.* Maintain your weight.* Regular walking in fresh air with concentration exercises and YOGA 30 min. a day without failure.* Diet rich in protein , fibers, iron.* Multivitamins, multiminerals tablets daily once.* Bath tub bath in leukwarm water with added 10 gm ipsom salt and 5 ml of Lavender oil.* Avoid smoking / alcohol / any abuse substance if using.* Have patience have confidence in yourself , nobody is going to cure you except your innner counciousness.* Engage yourself in more activities of your choice.* Laugh more which you are not doing at all at present.* BE POSITIVE every morning brings new rays of hopes in everybody`s life it`s how your eyes look at them.* Change your perspectives towards the situations and situations will change for you. Hope this helps you to any extent . Thanks for using Health Care Magic & reviewing my answer carefully. Regards Wishing you happy healthy life ahead . Take care Dear Bye"
},
{
"id": 29493,
"tgt": "How to treat extreme fatigue while having mono EBV in the elderly?",
"src": "Patient: Hi! i had the first symptoms of mono in late may.Went to doctor 3 time one for antibiotics next steroids and the third time had complete physical and lab results showing I had mono EBV. I AM 64 YEARS and the fatigue is still here with me. yes my pancreas was swollen and it show an air bobble in my chest between ribs and under my sterium. what can it do for the fatigue, sleeping 12 or more hours a day........... Doctor: Hello,Mononucleosis is a viral infection so antibiotics do not help. Antibiotics only treat bacterial infections. The fatigue from mononucleosis may last for up to 6-8 months. There is no real way to speed this up but there are some things you can do that will help. Make sure you get plenty of sleep at night, at least 8 hours. Exercise daily, at least walking for 10-15 minutes. Eat healthy foods, mostly vegetables with a minimum of red meat and fried foods. Do not smoke or drink alcohol.I hope you have a recovery of your energy very soon."
},
{
"id": 199667,
"tgt": "Noticing painless white discharge from penis while defecating",
"src": "Patient: Whenever I deafecate I observe a painless whitish discharge from my penis, I have complained to a doctor and have used antibiotics it will stop and later return. The discharge has no smell sometimes it is thick like sperm sometimes it is watery white. Doctor: HelloSperm leakage may be due to reasons like straining,over excited state etc.White discharge of semen is common during defecation.It is important to evaluate for constipation etc.You should try relaxation technique and avoid provocative literature and videos.You should try to be calm and go for yoga,mediation.You may need routine blood and urine investigations.Treatment depend upon findings.Get well soon.Take CareDr.Indu Bhushan"
},
{
"id": 197206,
"tgt": "Does kidney stone cause bleeding from penis or is it STD?",
"src": "Patient: my boy friend is having bleeding problems from his penis.sunday night while sitting atthe table he had a pain shoot from his kidney down when he got up to go pee he was drippig blood from penis and could not pee because it hurt that bad.i took him to the hospital they said he has a std and thats what they are treating him for,thats not whats wrong,he had kidney stones a couple months ago,last night when he went pee he passed a long worm like in shape blood clot.can you help me? Doctor: helloThanks for query .With previous history of kidney stone disease Bleeding from penis associated with severe pain in urethra is due to impacted stone in urethra stone rubbing against the delicate urethral mucosa to cause bleeding and not due to STD .This has been confirmed itself by passing a stone in urine next day morning .Ask him to take antibiotics for couple of days .Ensure he drinks more water .Dr.Patil."
},
{
"id": 175406,
"tgt": "Is there any side effect for using Cremaffin to a 1 year old girl?",
"src": "Patient: hi dear doctors, my 1 year old girl is sufferign from constipation. and now i m using cremaffin plus 7.5ml once in a day. and she is doing fine now. my question is there any side effect if i will cremaffin plus for one year daily? my email id is \" YYYY@YYYY \" Doctor: You can give to child cremaffin during 2weeks-1 month, but she can not drink all time. This is just symptomatic therapia. If you were give for a long time a fat -soluble vitamins, like as A, D, E,K would not absorb properly.It also gives anal irritation. Ypu should give to her laxative food like as Dahi, boiled rubber beetroots with 1 teaspoon of olive or sunflower oil. You can also give tomato soup with beetroots. Give to your baby -girl fresh juice 100ml from carrots, beetroots, orange , apple or rubber apple. Pumpkin soup or kheer from pumpkin is very helpfull. Kadu, salad from cucumber and tomato and onions with sunflower or olive oil. Corn porridge. You can give green tea with ginger and turmeric powder1/2 teaspoon for 2 glasses. She should eat more liquid.Thanks, best regards"
},
{
"id": 43975,
"tgt": "On IVF treatment, cyst on left ovary. Prescribed microgynon pill. Will it reduce cyst size?",
"src": "Patient: hi, i have been in ivf treatment and at the point of starting the medicine doctor realized that i have a 4cm cyst in my left ovary . he told me i can not start ivf treatment and he gave me Microgynon pill to take for a month. i am wondering if it helps me to reduce the cyst or not?i was waiting for starting ivf treatment for a long time and now this happened to me and i couldnt start that. i am really upset and worry about this situation. Doctor: hello madam, well since you have got a 4 cm cyst in the one of the ovaries ,getting gonadotrophins now for IVF may increase the size of the cyst. Well this may be a follicular cyst that may dissapear by the next cycle. you can just leave it and do a scan on day 2 of menses next cycle.alternatively you can also take the pill prescribed by your doc for 1 or 2 cycles and then do a day 2 scan to find whether the cysts persists or has dissapeared.either will help.the latter helps by suppressing your ovarian cycle and thus the impact of hormones on your ovary. i think you have got your answer. good luck"
},
{
"id": 36708,
"tgt": "What do bumps in buttock area indicate?",
"src": "Patient: hi Doctor, i have these hard lumps suddenly surfaced on my buttocks, a few are inside my buttocks. tried using alcohol to remove it. Some used to be bigger but with continuous application of alcohol it has reduced. It is not painful but smaller bumps used to be so itchy until it grow a little bigger and then there is no itch nor pain. I dont know that causes this as I am obsessive in my cleaning habits. I have bactroban here but I haven't used it yet. Hope you can help. thanks Doctor: Hi,These bumps might be due to having perspiration, friction and comparatively due to poor approach for cleaning.You can apply Bactroban locally after proper cleaning.Keep local part clean and dry.Ok and take care."
},
{
"id": 66977,
"tgt": "What does a lump on shin indicate?",
"src": "Patient: I have a lump on my shin that has been there for a couple weeks. I bruised it playing soccer - it swelled up slightly and I had a huge bruise right away. The bruise is now gone, but I still have a lump/knot on the injury site that is only painful when I touch it. Is this an issue, or will it heal on its own? Doctor: it could be an infected sebaceous cyst or just hair infection or an abscessa course of antibiotics is recommendedin worse case you need to consult a surgeonall the best!"
},
{
"id": 58144,
"tgt": "Liver psoriasis. Endoscopy done bands placed. Liver biopsy came back negative. BP dropped. What is the next step in treatment?",
"src": "Patient: My husband has cirrohois of the liver which is severe. He had an upper GI bleed 10 days ago and an endoscopy was performed and two bands placed. A couple of days later his first parenthentisis was performed followed the next day by a liver biopsy (was negative). He was released from the hospital on Nov. 7 (was admitted Oct.30). On Nov. 11 and Nov. 14 we returned for more parenthentis treatments. When he had the procedure done on Nov. 11 they took 5 liters out and his blood pressure dropped so our PC doc ordered only 2 liters removed on the 14th. I have been measuring his abdomen twice a day and is increasing again. My question is how many times is it possible to repeat this procedure and what could be the next step. He is 60 years old. Thank you! Doctor: Welcome to Healthcare-MagicGreetings of the dayParacentesis can be done as much time as possible.It just provides symptomatic relief from distension off abdomen and respiratory distress. Next step would be doing other serology test to know the etiology/cause of cirrhosis.Take careRegardsDr T ShobhaMBBS MD"
},
{
"id": 97337,
"tgt": "Does homeopathic medicine help in treating a hip labral tear?",
"src": "Patient: I have been told by my orthopedic after looking at my XRAY that I have labral tear or bruise in my right hip. This is due to undercoverage of the femoral head in the right in conjunction with a mobile right hip. My question is what homeathic medicine I can take for fast healing along with regular physio therapy. Doctor: Dear sirThanks for choosing healthcaremagic.you can take Arnica 200.It will give you fast relief.For further inquiry you can contect me."
},
{
"id": 13763,
"tgt": "What causes red colored line on the wrist?",
"src": "Patient: I ve noticed a red rash like line starting on my left wrist, it goes nearly half way around, it itches and is swollen. it stops on the center of my hand, perpendicular with my ring finger. the width is nearly half an inch. and it s about 2-3 inches long. Doctor: Hi, Yours symptoms seem to be related to contact dermatitis.I suggest you to use calamine lotion for local relief at the affected area.I also suggest you to use antihistamines daily. Hope I have answered your query. Let me know if I can assist you further."
},
{
"id": 31562,
"tgt": "Can a person have rigors without fever?",
"src": "Patient: Can a person have rigors without fever? Twice now I have been woken in the middle of the night with violent, intense chills or rigors which lasted for over an hour involving my whole body. I am not sick nor do I have a fever. What could be causing this? Doctor: if you take my opinion then I would suggest you not to get worried and have all your investigations done for blood reports and blood count ,rigors can be psychological too, so don't get worried"
},
{
"id": 136169,
"tgt": "Suggest treatment for hip injury",
"src": "Patient: I fell down the stairs carrying my little girl the other day in the rain. I held her up in the air so I landed quite hard on my hip. It happened about a week ago. Although I can get around better now there is a large circle on my hip and upper thigh that is sore to the touch and is reAl red with fever in it? I hate going to the dr but do you think it could be serious? Doctor: HiWelcome to healthcaremagicI have gone through your query and understand your concern.This is hematoma ( collection of blood in and beneath skin). This is likely to resolve gradually with passage of time but as you have got fever so it is advisable to visit a doctor. You need analgesic such as ibuprofen for pain relief and antibiotics to control the infection. Vitamin B and C help in healing. You can discuss with your doctor about it. Hope your query get answered. If you have any clarification then don't hesitate to write to us. I will be happy to help you.Wishing you a good health.Take care."
},
{
"id": 33807,
"tgt": "Suggest treatment for candidiasis infection",
"src": "Patient: Hi, Dr.i am 21yrs with a height of 5.4feet tall, weighing 50kg, i have been having a worm like movement all over my body and it is accompanied by abdominal pain, but went i to the hospital a test was carried out and it was moderate growth of staph, i took treatment but symtoms still persist. I went again, a test was carried out, it was candidiasis infection of which i took the treatment but the symtoms still persist. What do u think is the possible treatment? Doctor: Hello. Thanks for writing at Healthcaremagic.com.I understand your problem and try my level best to solve it.for candidiasis you have to take Tab. Fluconazole 200mg for atleast 2 week.i hope my review will help you.Thanks and Regards.Dr Vijay"
},
{
"id": 185229,
"tgt": "Suggest remedy for white spots on gumline",
"src": "Patient: i get white spots on my gumline. lately i've getting them twice in to months and before now i think it lastest about two weeks i don't know what it is or how to treat it. each time they come up my hold mouth hurts bad.what is going on i am afraid. to ask but i need to fix this bad. Doctor: Hello!Thank you for posting in HCM.You did not mention if you smoke or have any other habits.This can be a candidal infection as you say it comes and goes. You did not mention your age,general health status and if you suffer diabetes.Check if you are low on immunity.This can also be a oral lesion. However, you need a clinical examination with your dentist for definitive diagnosis.Maintain a good oral hygiene routine.Gargle warm saline thrice daily.Have a stress free life style.Please see a oral pathologist immediately for they can help you with a biopsy.Hope this helps."
},
{
"id": 49693,
"tgt": "Pain in rib. Urinating a lot, blood in urine, some time dark some time light. Kidney problem?",
"src": "Patient: Hello, and thank you, I have had a pain in my left side kind of hard to explain where it is just under my left rib, I have been uriniating allot and one day I had a small amount of blood in my urine, not allot just a little bit on the paper. It is a sharp pain and sometimes it is really painful. Generally I feel well, I keep active but when the pain comes it is powerful and very painful, I urinate allot at night although i dont drink anything after 8 pm, sometimes my urine is darker yellow and sometimes does not look like it has any colour, sometimes not allow comes out, I am thinking it can be my kidneys? Doctor: How old are you?does the pain radiates to your that side thigh?Do you feel any dragging sensation in your left hypochondrium(below rib)Do you have pale appearance(anemia and jaundice)how is you stool.any constipation?diarrhoea? cramps? sounds with pain(borborygmi?i suspect two things1)spleen for darknening of urine explains but for that hemolysis needs to be ruled out, so do Blood smear , Full blood count, Retic count , serum LDH. serum haptoglobin, coombs test. MP test if you have any recent travel history.and must do an ultrasound for spleenomegaly or renal calculi as it will help my both suspicions(next down narrated)2)now the blood in the urine with painful micturition is a hallmark sign of renal stones. Boy you should clear out that possibility by an ultrasound, simple Digital x ray KUB. Intravenous pyelography, do a simple Urine Routine Exam too. i am sure it will helpmean while treat symptoms. Get antispasmodics, Analgesics, Drink alot of water, get some urine alkalizers, be on a soft diet.get some multivitamins if you are anemic and do the investigations to be out of the woods,I will be here if you need me. RegardsDr Shafi Ullah Khan"
},
{
"id": 107298,
"tgt": "What causes lower right back pain?",
"src": "Patient: Very painful lower right back duting most movement. No pain when I lay level on my left side. Advile and icy hot tens therapy are helping but not much. Lifted light weights yesterday with no pain. This is very painful today and is also felt on the outside of my right hip. No fever or sweats. Doctor: Hello! Thank you for writing! Is your pain associated with numbing, or tingling on your leg, or is the pain associated with other signs? How is its time decours? It's more painful in the morning or at night? What is the character of the pain? When it begins, is it constant or not? What are the factors that make it worse? What is your job, please? Is it related with lifting heavy weights? Have had a car accident or falling from height? I suggest you to visit an Orthoped. You may need to do a lumbar MRI, but better first visit an orthoped and he will tell you. Wish you good health. Best regards."
},
{
"id": 19794,
"tgt": "Suggest treatment for heart palpitations",
"src": "Patient: Hi, i am a 26 year old female and i have been having strong heart palpatations once in a while. My dr has done an echocardiogram,stress test, holter monitor and other blood work and everything is normal, so why am i still having these palpatations that feel like a flutter and hard pounding? Doctor: It is good that your cardiac evaluation is normal.have you ruled out hyperthyroidism if not get your thyroid function tests done.check your vitamin d levels as wellas RED BLOOD CELL Magnesium levels reduce intake of coffee/ tea to 2 small cups per day.reduce your stress at work and home relax with yoga and meditation socialize , volunteer and go out and recharge you should be fine"
},
{
"id": 110255,
"tgt": "Suggest treatment for lower back pain",
"src": "Patient: My mother 70 yrs old,is suffering from lower back pain for last 2 months. we consulted and took medicine from various doc. and ortho specilities after that we switch to another. and he told us that it is TB and pus is there ,after checking MRI & various blood tests. (where as another ortho doc. couldnot informed us about it even after Digital Xray) Now she is feeling improvement but still feel pain.Are we on right track??? Doctor: Hi,Welcome to healthcare magic.After going through your query .Some tuberculosis is not seen even in digital x-ray. As it is seen in MRI AND FURTHER MORE PATIENT IS RESPONDING TO TREATMENT SO YOU are on right tract, that is she is suffering from tuberculosis. There should be significant improvement in pain after nearly 2.5 months of anti tubercular treatment.I think You are on right tract. I think your query answered.Welcome to any follow up query."
},
{
"id": 105586,
"tgt": "Experienced pricking, itchiness and swelling on both palms. Conditions persisted after taking medicines for allergic reaction",
"src": "Patient: My daughter who is 16 told me the other day she experienced some prickling and tingling on both palms and the wrist . She also complained of itchiness and became panicky. At first look her palms were a little swollen but later that disappeared and I thought she had an allergy reaction and gave her Loratadine and Antarax each, but the episode persisted. What to do? Doctor: Hello. Thanks for writing to us. The symptoms your daughter is having could be related to an allergic reaction or vitamin B complex deficiency or electrolyte disturbances. Giving her adequate supplements and a healthy diet will take care of the symptoms in few days. I hope this information has been both informative and helpful for you. Regards, Dr. Praveen Tayal drtayal72@gmail.com"
},
{
"id": 84428,
"tgt": "Is taking rexipra and nizoral together recommended?",
"src": "Patient: Hi, I am taking Rexipra 10mg daily at night for severe anxiety. Due to scalp infection and severe hair loss my dermatologist has prescribed me to take Nizoral 200mg once daily with meal. Can I take both the medicines or will I have some side effect with the same? Thank You, Deepa ( Age 42) Doctor: Hi,It is not safe to take these medications together. Taking rexipra together with nizoral can increase the risk of cardiac arrhythmia (irregular heart rhythm) which may be serious and potentially life-threatening. You should seek immediate medical attention if you develop sudden dizziness, lightheadedness, fainting, shortness of breath, or palpitations during treatment with these medications, whether together or alone.Hope I have answered your question. Let me know if I can assist you further. Regards, Dr. Mohammed Taher Ali, General & Family Physician"
},
{
"id": 169391,
"tgt": "What are the treatment for impetigo in a 21 month old?",
"src": "Patient: what are some treatments for imetago, for a healthy 21month old. A small red spot with what looked like a white head started on her chin 8 days ago. Now she has several more around her mouth some were draining pus but are now scabbed over. But does not look any better. Have been applying neosporin with no releif Doctor: Hello there,Impetigo is very easily treated by applying an antibiotic ointment that is stronger than anything over the counter. Of course is does have to be prescribed by a doctor so I would encourage you to at least call the doctor. Bactroban, Bactrim and Cleocin are the main drugs to treat impetigo. Staph and strep are the 2 diseases that can spread from it so it is very important to start treatment right away. Good luck!"
},
{
"id": 176234,
"tgt": "Is it safe to give S-26 Gold as milk for a 2 month old G6PD child ?",
"src": "Patient: HI, i m a mother of a 2 months old baby with G6PD....we we re able to determine her condition after the newborn screening....her Pedia knows about it and she recommended to take S-26 Gold as his milk since she was 0 months old until now....but in my researches, i found out that soy lecithin foods must not be taken by a G6PD deficient. Should i stop giving him S-26 Gold? Doctor: HelloG6PD enzyme level deficiency can be precipitated by various food products and medications. You can get that list from your physician or from the website .These products must be avoided by the parents and you should also teach the baby about all this as she grows.Soy product is one of those. So I would suggest you to discontinue the present preparation and consult your pediatrician about this matter.Regards"
},
{
"id": 179597,
"tgt": "What causes loose motion in baby?",
"src": "Patient: Hi my 6 month old is on nutramagin since she was 3mts old. Her stools have been slightly loose since, however since I started weaning her stools are very loose and she has really bad nappy rash. Prob due to veg and fruit. My question is should I change her milk and if so what will I change to?? Doctor: HiThanks for writing to health care magic.Diarrhea is usually viral and may last for 5 days.Give ors and zinc.you can change to lactose free formula like isomilWishing your child good healthRegardsDr Arun"
},
{
"id": 49049,
"tgt": "Is a biopsy done on kidney too risky?",
"src": "Patient: My cousin has had several blood and urine test to determine the reason for or changes to her kidney function. Her Dr is now recommending a biopsy be done. Should they do some other testing before this is done, as I have read that it is a bit risky. We are all new to this issue Doctor: Kidney biopsy is an invasive procedure in which a small amount of kidney tissue is retrieved with the help of a biopsy needle/biopsy gun and microscopical examination is done. This is done under local anesthesia. This is usually done when clinical examination, blood investigations (routine and specific) and imaging investigations are not enough to give a definitive diagnosis.A few investigations may be done before the procedure like Platelet, Prothrombin Time, APTT and bleeding time, which indicates the coagulation profile. Some nephrologists prefer the subject to be off medications like Aspirin and Warfarin. Post procedure, the subjects lie flat on their back for 4\u20136 hours to minimise the risk of bleeding. Vital parameters and urine are frequently monitored to ensure the patient is not suffering any bleeding complications.Complications are uncommon and may consist of minor bleeding from the biopsy site and in some cases there there may be haematuria (blood in urine). Pain at the biopsy site may occur, which may be relieved with anti-inflammatory agents like Paracetamol. Very rarely, severe complications like perinephric hematoma, Urinary retention (due to blood clot) and shock (due to sudden brisk bleeding) may occur.Your Nephrologist will give an overview about the procedure and the pre-procedure tests required, and the patient's NOK may have to sign a consent form for the procedure."
},
{
"id": 188963,
"tgt": "Child having viral fever treated with antibiotics. Gums red, sensitive, unable to chew. Cure?",
"src": "Patient: Hello Doctor..My 21 month old sn was suffering from Viral fever since last 5 days only yesterday the fever finally got over in the due course of the medication we gave him antibiotics changed the antibiotic to Cefpodoxime Proxetil dispensible tablets. Although he doesnt have any fever but his gums are very red and sensitive plus he is not able to chew much and screams with even a small amount of chilli. Please suggest how do we cure this. Doctor: hello pooja.. welcome to healthcaremagic.. wel itz good that u took care of ur son but i would suggest in future dont overload your son with higher antibiotics like cefpodoxime proxetil as they are used only in very severe infections... it wil lead to resistance if u use higher antibiotics frequently.. his gums may be red due to either of three reasons, 1) allergy to the antibiotic or any other medication which u have given.. 2)it may be due to vitamin c and vitamin b complex deficiency 3)or it may be due to granulomatous inflammtion.. wel the diagnosis can be made on examination and investigations.. so i would suggest u to go to nearby paediatricain and get consulted.. i wish a speedy recovery of ur son.."
},
{
"id": 181953,
"tgt": "Is it compulsory to take NSAIDS after a dental procedure?",
"src": "Patient: I just had a tooth prepped for a crown. No pain. I do not like to take NSAIDS My dentist suggested I take 2 now and 2 6 hours later to help reduce inflammation. Is not my body capable of doing that on its own and wouldn t the NSAIDS inhibit the body s natural healing processes? Doctor: Hi The imfamation needs to be reduced to hasten proper healing . though you are right that your body has its own mechanism to heal but a healthier envoirnment fastens body healing process . since even instrumentation and high speed aerotars used for crown cutting are forms of external irritation a mild dose of NSAID has been corrrectly prescribed by your dentist to negate the summative effect of all procedures as well as the imfamation present before the procedure . rest be assured just two doses will have more benefits than do any harm . Take care"
},
{
"id": 98304,
"tgt": "How can allergic rhinitis be treated?",
"src": "Patient: Started with small type hives. Then runny Nose cough & itchy eyes.. it still seems to be progessing. Even after taking just about all types of over the counter allergy . EVERYTHING. Benadryl, Allerest ,Allegra and about 4 other off brand types, Flonase and a Nasel Rinse because of the pressure. I can t get it to go away or even ease up. What else can I do? Doctor: Hello and Welcome to \u2018Ask A Doctor\u2019 service. I have reviewed your query and here is my advice. At this point you can use oral corticosteroids to help you get relief. Hope I have answered your query. Let me know if I can assist you further."
},
{
"id": 145108,
"tgt": "What causes numbness,dizziness and pain in head?",
"src": "Patient: Hi, i am 24 years old about a year ago it happened every time i go out for few drinks not alot the day after i feel very strange i have no power in my legs, its like they are numb my head feels very strange i dont have pain more i feel dizzy and numbness in my head few stings in my head my eyes are feeling weird, is this normal after drinking or should i go see a doctor ? Doctor: Hello!Thank you for your question ! I understand your concern. I think that the symptoms you describe are normal after a heavy drinking. If this happens to you after not many drinks , it means that you are a person more sensible to alcohol than the others. Different persons have different reactions to the amount of alcohol. It depends on your liver function , which is different in every one. If your liver function tests are ok, there is nothing to worry about. Hope to have been of help!Greetings!Dr. Abaz Quka"
},
{
"id": 39248,
"tgt": "What causes vaginal itching?",
"src": "Patient: The outer lips of my vagina is itching and bothersome. I am diabetic and have allergies. Oh, I am also obese. I have to use a pubic bathroom at work. Currently, I am taking about 5 diabetic medications. Could the itching be from consuming sweets or allergies? Doctor: Hello,Welcome to HCM,Known case of diabetes with the history of itching in the vagina suggests me that you may be having fungal infection. As you have mentioned that you are obese which is also an important risk factor for the fungal infection.The fungus are the normal commensals of the genitals which are kept under control by the helpful organisms. whenever there is change in the environment these fungus will multiply and produce these symptoms.I would suggest you to follow1.Maintain good hygiene by keeping the area clean and dry.2.Sugar level should be kept under control by taking appropriate anti diabetic drug.3.Apply antifungal cream over the itching area.4.Oral antihistamines to reduce itching.Thank you."
},
{
"id": 84627,
"tgt": "Is there any side effects for coq forte tablet and cernos gel?",
"src": "Patient: helo i am 34 yrs and i have azoospermia my height is 6feet and i am slim my weight is 70kg and i dont hav any family background of azoospermia.dr has suggested to take coq forte tablet and cernos gel for 3 months.does it hav any side effects.plz suggest. Doctor: Hi,Generally they are well tolerated. COQ-10 is an anti-oxidant which may cause heartburn, nausea, loss of appetite and diarrhea. Cernos Gel contains male sex hormone (testosterone) which is known to cause breast enlargement (males), weight gain, acne and hair loss.Hope I have answered your query. Let me know if I can assist you further. Regards, Dr. Mohammed Taher Ali, General & Family Physician"
},
{
"id": 223195,
"tgt": "Does BC pills Minigyon cause irregular menstrual flow?",
"src": "Patient: Good day, i am having a problem since lately i start taking a contraceptive (minigynon) i took it on my first period my period always finishes within 4-5 days but ever since i start taking this contraceptive i notice that 8 days has past and i am still seeing this dark color liquid. I stop taking the pill for 2 days to figure out what is causing the problem but i was still seeing the dark liqiud appearing i took back the pills in other same way and it is still that way.. So please i am urgently seeking a reply from you i am so very concern about this situation and thank you in responce.. Doctor: Hello,Minigynon pills are an effective birth control method. It has both female hormones (oestrogen + progesterone). From the way you have described, it appears that you started the pills in the right way - from the first day of the period.The period that one gets after starting pills is not the same that one gets when not using it. The dark coloured blood could have been old blood which took time to come out of your body. This should not change the way you take pills.You have to take it continuously as instructed. Stopping in between interferes with the blood levels of the hormone in body. It can produce irregular bleed. It can also lead to failure of method and pregnancy can occur.Hope I have answered your query. Let me know if I can assist you further.Regards,Dr. Sabina Hussain"
},
{
"id": 67349,
"tgt": "What are the painful lumps on the back of my head?",
"src": "Patient: I just discovered lumps on the back of my head. They did not hurt initally, however now after 5 days they are getting painful when i touch them. I am also feeling numbness and tingling on my head. My doctor thinks they may be cysts and has given me an antibiotic. If not cleared up in 2 weeks or becomes painful I should go back. I am just wondering what you think it may be. Both areas are on the left side of the head. one on the back side and one on the top left side of head almost like an L shape. Doctor: Hi! Good evening. I am Dr Shareef answering your query.Yes, possibly your doctor was right in labeling them as cysts. In the starting, they were not painful, but these got secondarily infected later on giving rise to the pain. These could subside with antibiotics, but could recurrently get infected. Therefore, I would advise you to get these assessed by a general surgeon and get these removed once the infective phase was over on consuming the antibiotic and anti inflammatory drug along with a proton pump inhibitor.I hope this information would help you in discussing with your family physician/treating doctor in further management of your problem. Please do not hesitate to ask in case of any further doubts.Thanks for choosing health care magic to clear doubts on your health problems. I wish you an early recovery. Dr Shareef."
},
{
"id": 50185,
"tgt": "On palliative treatment for relapses tumour, on cathereter for urinary problems. Creatine level 2.3, Advised to consult nephrologist. Suggest",
"src": "Patient: My dad was diagnossed for malignant melonoma in 2009 and APR performed .It relapsed in Dec2012.Tumour is beyond cure hance palliative treatment going on .He has a urinary problem wherein urine is uncontrolled doctors in TATA hospital Mumbai performed compreesion of urine and presently he is on a cathereter ,creatine level is 2.3 .doctor adviced for nephorologists opinion PLEASE GUIDE Doctor: Hello dear,Creatinine is a marker of Renal functions.Normal range varies from 0.5 mg/dl to 1.2 mg/dlIn that case, a value of 2.3 mg/dl can be considered high, which is indicative of underlying damage to the renal tubules.Based on the above mentioned fact, the doctor has advised a Nephrologist opinion.So, that the underlying damage can be detected at the earliest & further progression can be prevented.Take care."
},
{
"id": 4651,
"tgt": "Trying to conceive, having pregnancy symptoms and short period. Took thyrox-50. PCOS not found. Reasons for these symptoms?",
"src": "Patient: i'm trying to conceive. Last month I felt pregnant symptoms like bloating, relieving lot of gas, indigestion, light-headed, slight headache, but got my period on the actual period date. The period only lasted for 1.5 days, which is usually 5 days. My TSH was 8.13 during august, Took thyrox-50 and now 3.22. after periods also i felt the pregnancy symptoms. So tested for beta HCG and came 1.2 (negative). Checked for PCOs and none found. What to know why i got those symptoms. Am i normal? Doctor: Hello and welcome to HCM,Bloating, indigestion, headache are not symptoms of pregnancy.Pregnancy can only confirmed by pregnancy tests during early pregnancy days.Hypothyroidism or low thyroid hormones can cause menstrual abnormalities and/or difficulty in conceiving.However, treatment of hypothyroidism corrects all menstrual abnormalities.Conception takes place at the time of ovulation.Ovulation can be determined by basal body temperature and examining the cervical mucus.Bloating, indigestion and light headedness are caused by problems of the gastro-intestinal tract.Thanks and take careDr Shailja P Wahal"
},
{
"id": 29357,
"tgt": "Suggest treatment for dengue with a low platelet count in males",
"src": "Patient: Hi, my father was suffering from dengue. He was admitted to the nearby nursing home and was treated but after some days his stomach bloated abnormally including his eyes, though he passed 7 days in this situation the platelates haven t come to the preferable count, but he has no pain in his stomach along with no appetite for food, please suggest me what should I do. His recent platelates count is 20,000 Doctor: dengue is a viral infection and treatment is mainly supportve ie plenty of fluids ,paracetamol for fever and antihistamincs for itching.......it cures by itself within 7 -10 days .but if there is a bleeding manisfestations eg nasal bleed etc ...platetlet count less than 10000 ,then prophlactic platelet transfusion is doNE.FOR ur father if he has no bleeding tendency then he only needs symptomatic treatment ....if platelets goes down to 10000 thn he might need platelet transfusion"
},
{
"id": 144907,
"tgt": "What causes facial paralysis after biting tongue?",
"src": "Patient: I bit my tongue while brushing my teeth and it became numb yesterday morning. By the end of the night my left eye was getting air inside and when i blink it wouldnt shut right. This morning when i woke up My face is paralyzed on the left should I see a doctor. Doctor: You are having Bell's palsy. You need to see a physician or a neurologist for confirmation. You will have to be started on steroids as treatment followed by rehabilitation."
},
{
"id": 128065,
"tgt": "Can a cast be used on a blistered proximal phalanx fracture?",
"src": "Patient: My daughter broke her 3rd proximal phalanx 4 days ago, the hospital have put it in a shoe, removable, that is too big, giving her blisters and she trips over. Took her back to the hospital this morning and was advised to \u201cput a sock on the foot to help with the blistering\u201d which obviously is actually impossible due to the pain and bruising. Is it possible to cast this fracture? She\u2019s 9 Doctor: Dear sir /madam Buddy strapping of 3rd and 2nd toe for 3 weeks also would serve the purpose, if casing or boot is cumbersome and difficult, At this age most fractures unite and because of growth potential at this age bone remodels."
},
{
"id": 122089,
"tgt": "What causes pain under breast at top of rib cage?",
"src": "Patient: I have pain under my breast at top of rib cage when I blow my nose or sneeze. the discomfort started on the left side first. The discomfort feels like I have been using muscles to lift stuff but I have not. I have cold symptoms as well with a periodic cough not consistent. Doctor: Hello, Your symptoms are suggestive of local musculoskeletal inflammation or costochondritis. For this reason, I would recommend performing a chest X-ray study and inflammation tests (complete blood count, PCR, ESR). In the meantime, naproxen or ibuprofen can help improve your situation. Hope I have answered your query. Let me know if I can assist you further. Take care Regards, Dr Ilir Sharka, Cardiologist"
},
{
"id": 145697,
"tgt": "How to treat a tender spot on head caused due to injury?",
"src": "Patient: I hit my head on my windshield in an accident and went to the E.R. Had a Ct scan that was negative, however almost a week later I still have a little tenderness and there is a spot that when I press down on it feels like my finger sinks into it. Does this warrant a visit to the doctor? Doctor: Hi,The CT brain is the gold standard for any head injury. If it has shown a normal study, you needn't be worried. The bump on head is probably soft tissue swelling and it should recede in a week or two. Applying an ice-pack will certainly help in decreasing it further and the pain too should go away. You could also take an over-the-counter Tylenol for the pain. If however the symptoms don't go away and you get any new symptoms you would need to visit your doctor."
},
{
"id": 189710,
"tgt": "White patch on cheek due to biting, bad breath, heaviness in cheek, swollen face. Is it cancer?",
"src": "Patient: firstly i bited my cheek once due to which white patch developed and its not healing , also i started bad breath, feel heaviness on left cheek , sometimes feel left face is swallen...dont know but only thing coming in my mind is i am having cancer ....i have shown to ENT and dentist and they said its ok and nothing of that sort but still i have doubts and its not going from my mind Doctor: Dear friend. Thanks for sharing your concern. when you have already visited ENT doctor and dentist, and they have suggested you that it is not cancer then you should be relaxed,that it is not cancer. Now i would like to ask you why do you feel it is cancer? it has no such signs and symptoms,which could be correlated to cancer. you have already mentioned that it has arised from the self induced injury to cheek. now the symptoms, that you are experiencing like swelling and heaviness that is just due to tissue injury and inflammation. once the tissue repairs or heals you will feel absolutely normal.Also you must avoid this habit further. Regarding your bad breath, that is due to improper oral hygiene or chronic sinusitis.You must rule out these too. use antiseptic mouth wash for 15 days twice daily and look for the changes. I am sure you will be fine. Hope it helps. Thanks. Take care."
},
{
"id": 102555,
"tgt": "How to treat bronchitis with noisy breathing while on zithromax, albuterol inhaler and cough syrup?",
"src": "Patient: I am recovering from bronchitis , took zithromax, albuterol inhaler and cough syrup. After over a week of recovery I still feel very weak and have now started to notice noisy breathing ( crackling and sometimes whining sounds) when I breath out. This is especially notable when I lay down. I am otherwise feeling better. Should I be concerned?? Doctor: hi, you may experience those symptoms as an episode of bronchitis may take upto 4 weeks to recede completely. you may continue the medication, but i suggest you to add mucolytic agent like N-Acetyl Cystene nebulisations which can help in clearing secretions. other symptoms will resolve gradually. hope this helped."
},
{
"id": 148711,
"tgt": "Swirling sensation near temples at night. Normal blood test. What is this?",
"src": "Patient: Hi there, probably an unusual system and quite challenging to describe, but I keep getting a swirling sensation around both sides of my head near my temples, I often try to rub it in order for it to stop. I notice it mainly at night and when I go to bed. I recently had a blood test done for everything, heart monitors test and other than a subclinical hypothyrodiasm (which I kind of new already) everything seemed fine. I have also had an eye test and that was all fine. I haven t yet had an MRI . What could this be symptomatic off? I am gettng a bit absent minded as well and very tired alot of the time but that I think is due to the thyroid issue. Could it be a tumor? But I don t really get headaches . many thanks Doctor: Hiplease get an MRI The problem does not seem to because of thyroid but get the test done Ger ENT check upAfter every test is normal what is left is psychological sensation for which you need to see a psychistrist who may provide reliefGey riding of your problem by advice of your psychiatrist Dr Lal Psychiatrist"
},
{
"id": 41416,
"tgt": "Suggest treatment for infertility",
"src": "Patient: AA doctor i am married since 2 years i took tablet clomid for three month regular in every periods of my cycle but i did not conceive so my husband for semen analysis which shows advance motailty 10% motality 20% and immotality 30% what would you suggest me i consult with a doctor she advised to take capsule evione 400mg daily for me and my husband both plz answer Doctor: HelloI can understand your concerns.If possible kindly reply back with the complete semen analysis report of your husband.Tablet evion is vitamin E. It acts as antioxidant and is said to improve sperm count. I think you both should continue with it.I would also suggest your husband to take tablet paternia. It is said to improve sperm motility.Kindly persuade him to stop smoking and alcohol, if he does so.As you have been taking clomid with no success, i would suggest you to go for hormonal assays (FSH, LH, PROGESTERONE).Do consult your nearest infertility specialist.Hope this helps.Thanks."
},
{
"id": 88801,
"tgt": "What causes abdominal and waist pain?",
"src": "Patient: I have a pain which move around my abdomen an waist no pain outside my body only inside and mostly on now the both side of abdomen, i did sonography test they say its normal but the pain is there night an day it wake me up , my urine good , my stools not to good i goes of sometimes normal an somtimes it like in a week. Doctor: Thanks for your queryAs per your complaint of abdominal pain and ur sonography report is normal that suggest that there is no pathology with ur urinary tract. As per your stool complaint there might be 1 gastro intestinal tract infection or 2 muscular pain So u should go for stool culture and blood count.Get well soon"
},
{
"id": 29793,
"tgt": "What causes frequent urination with bladder pain?",
"src": "Patient: I am constantly having to go to the bathroom to pee. About ever ten minutes I have the urge to go and sometimes my bladder still hurts and I still need to go. It doesn't hurt when I pee but it's getting old real fast having to go this often. The only things that have changed lately is with me is I started taking methadone a little over a week ago and I have been drinking a little more fluids throughout the day. Doctor: HIWell come to HCMI really appreciate your concern, it is the symptom of poly urea and most likely cause of this could be infection, bladder stone, and diabetes all these needs to be ruled out, some time it could be due to too much water intake so I would advise to reduce the water intake if this is too much, second get done the urine test just to rule out the infection and diabetes (Presence of sugar) some time it may needed x-ray of lower abdomen hope this information helps."
},
{
"id": 115881,
"tgt": "Is triglycerides level of 220, HDL 47 and LDL 77 concerning?",
"src": "Patient: 41 year old female triglycerides level at 220 but my HDL is 47 LDL 77 MCV-100.5 MCH-34.1 RDW-SD-46.6 MPV-11.8 AST-46 Doctor is only worried about the Triglycerides,and started me on fish oil. Should I be concerned with the other high numbers? Any thing else I can do to lower my Tri's Doctor: Hi, dearI have gone through your question. I can understand your concern. Your triglycerides level is high. You should take low fat diet with high amount of polyunsaturated fatty acids. Your MCV is slightly higher. You should go for serum B12 level once. You may have vit b12 deficiency. Your rest all reports are normal. No need to worry. Hope I have answered your question, if you have doubt then I will be happy to answer. Thanks for using health care magic. Wish you a very good health."
},
{
"id": 148253,
"tgt": "What to do if I can't find a doctor who would prescribe Methadone which helped a lot for pain relief after spinal fusion?",
"src": "Patient: Hi i am 53 year old mail 7 years ago had a bi level spinal fusion the operation consisted of incisions in my stomach ,back and left hip my pain since the surgery has been so bad there are days i cant walk . I was at a pain clinic where for the 6 years i was being treated with 100 mitagram Fentynal patch and 7-15miligram oxycodone a day it got to the point that i was on so much medication i felt sick but it got to the point the meds were not helping . I decided to wean off of the meds to find alternative medications found a pain doctor that suggested trying 20-30mg of methadone twice a day amazingly it helped greatly and for the last year i have felt better physically and mentally. Now the problem my doctor died suddenly at the end of November 2013 the other doctor in the office filled in for a couple of weeks on December 20th he gave me a prescription for 30 days a week and a half later a got a call saying he did not want to deal with prescribing methadone that i needed to find a new doctor . I found a new doctor who my first visit was Jan 13th new doctor wont prescribe pain meds wants me to go to pain clinic cant get appointment for a consult until the 24th of Jan and have no pain meds . Not sure what to do have been on pain medications for 12 years and I honestly cant find anyone to help or with any advise. Called original doctor today at the advise of new promary care to ask for a two week prescription until pain management what are the doctors obligation to treat me in the interim. Any help would be appreciated . Doctor: Hi,Thank you for posting your query.I have noted your symptoms and it is unfortunate that you are unable to get pain medications from your current doctor and your appointment with pain specialist is a few days later.In the meanwhile, you can request your current doctor to give you a few days of prescription of pain medications.You can also take OTC pain medications.Tramadol and flupirtine are other medications that could help reduce your pain.I hope my answer helps. Please get back if you have any follow up queries or if you require any additional information.Wishing you good health,Dr Sudhir Kumar MD (Internal Medicine), DM (Neurology)Senior Consultant NeurologistApollo Hospitals, Hyderabad, IndiaClick on this link to ask me a DIRECT QUERY: http://bit.ly/Dr-Sudhir-kumarMy BLOG: http://bestneurodoctor.blogspot.in"
},
{
"id": 153200,
"tgt": "What causes difficulty in ejaculation after the treatment for testicular cancer?",
"src": "Patient: my boyfriend has had alot of operations froma kidney transplant to testicular cancer when we have sex he can cum but it doesnt come out of his penis he says his penis is to tight wll this effect our chances of having children and is there anything we can do? Doctor: Hi,Thanks for writing in.Patients might have difficulty in ejaculation after taking treatment for testicular cancer. This is usually for a short duration and patients will improve with time.If you both plan to have children then it is best to first get a semen analysis done and then consult a fertility clinic to know the probabilities of have a child in the natural way. If on few attempts you are not able to have a pregnancy with your partner then artificial reproductive techniques might help. In present days there is a lot of medical advancements which might benefit you."
},
{
"id": 189516,
"tgt": "Face swollen after major 4- hour dental surgery, bumps inside lips. Is this normal ?",
"src": "Patient: Ok,I had major 4- hour dental surgery ,Mr face swelled aftierwards,and my lips ,swelled I just noticed the inside of my lips,little whit bumps that really hurt bad.just trying to figured out if that s an infection,or is that normal . I did have a fever for 4- days after dental surgery,and developed a small cold sore in the corner of my mouth on the right side. Doctor: having this sequelae is a common occurrence considering the length of the surgery. bone might also have been removed and soft tissue cut or incised, which results in such swelling. you could have some steroids such as dexamethasone along with antibiotics and painkillers. also use hexidine mouthwash and do warm saline gargles if possible thanks take care."
},
{
"id": 114708,
"tgt": "Suggest treatment for anemia and vitiligo",
"src": "Patient: Hlw, I m 19(unmarried). Normally,I have this problem of likoria(no itch in the area) but for last three days ,I m observing thick white yellowish likoria.I have anemia ,taking medicine.l also have vitiligo slightly in my face ,I m taking medicines for that.I have a habit of drinking less water.do I have any infection / do I need to go to doctor for this problem(likoria) Doctor: Hi welcome to HCMI have gone thru your query regarding anemia and vitiligo . Dear , all your problems are outcome of increase in the levels of toxins in the body and poor eating habits . In other words , your low immunity is the root cause of all your problems . Anemia occurs because your body doesn't make enough red blood cells due to poor eating habits and faulty life style .Bleeding causes you to lose red blood cells more quickly than they can be replacedYour body destroys red blood cells causing you anemia and anemia causing vitiligo , leuchorrhoea and others .Vitiligo is a disease in which the pigment cells of the skin, melanocytes, are destroyed in certain areas. Is also a life style desease .Leuchorrhoea also known as vaginal discharge , is a very common condition that has been experienced by most women of all ages, at some time or the other. It is thick white yellowish color most of the time . is mainly experienced during puberty, when sexual organs are developing in a woman. It can be very irritating and damaging if left untreated This is because of the fact that the female genitals are highly prone to infections because they are moist and covered at most times. Moreover, women do tend to sweat a lot in that area, which increases the chances of infections and inflammation. Some of the most common causes are Anemia and Hormonal disturbances and increase in the levels of toxins in the body and poor eating habits .Modification in your life style and food habits -including regular exercise ,yoga , pranayam and meditation for proper blood circulation and balanced diet having essential nutrients - fiber , Vitamins ,minerals antyoxidants in natural form - lemon juice , turmeric powder , honey , aloe vera juice , amla regular intake will help gain heallth in 6 months .All the best"
},
{
"id": 74762,
"tgt": "What causes chest discomfort around diaphragm area?",
"src": "Patient: Hi, my 14 yr old son has been have some occasional chest discomfort around his diaphragm area. He is not obese 5'4\" 120lb. If the weather is hot he has shortness of breath. He has just started puberty this year. Tylenol and Motrin take care of the discomfort. Could he have a heart condition? Doctor: Hey there, Mostly its puberty effect.Still go for EKG and chest x-ray and rule out other causes related to lung and heart."
},
{
"id": 127866,
"tgt": "How to treat a possible ACL knee injury?",
"src": "Patient: I have what appears to be ACL knee injury according to Web MD symptom checker. The area is just below the knee and to the inside. I had a previous injury to this same area about 9 mos. ago and the affected area was exactly the location and the area of discomfort was about the size of a quarter dollar. It s been about a month ago that I d strained the same area after doing some unexpected/more strenuous stair climbing etc. This time the area was much greater than a quarter dollar size, it s closer to the size of a silver dollar coin. Doctor: I suggest you to take Ayurvedic medicine it will cure your ligament muscle and even Bone diseasetab mahayogaraj gugglu 2bd cap had jod vati 2bd . for 1 month you will get relief."
},
{
"id": 147885,
"tgt": "Meaning of spine MRI that shows disseminated spinal metastasis?",
"src": "Patient: MRI Report for dorso lumbar spine ( plain & contrast) Findings.Defuse involvement of the dorsolumbar vertebrae present showing focal disease as well as involment of the entire vertebra. These appeared hypointens on T1W and T2W images involvement of the proximal sacral segment specially S1 and S2 also present including the body and ala and shows contrast enhancement. Findings are consistent with disseminated spinal meastasis.At L5/S1 level, moderate spondylolisthesis of L5 over S1 noted causing anterior displacement of L5.Impression. Diffuse in envolvement of the dorsolumbar vertebrae present showing focal disease as well as involvment of entire vertebra.Findings are consistent with disseminated spinal metastasis. Doctor: Hi,Thanks for writing in.The above findings are pointing to a possible spread of cancer to multiple bones on the spine. This report must be discussed keeping in mind the medical history of the patient and the related symptoms."
},
{
"id": 188755,
"tgt": "Why do I feel a steel like sensation when my tongue rubs against the inner side of the lower lip ?",
"src": "Patient: I am a 53 year old female. For the last year I have had a wierd sensation when my tongue rubs against the inside of my bottom lip. Feels like steel. I have asked my Dentist to check but when he does he says everything looks normal. Doctor: Hello,Thanx for choosing HCM,Your query is not clear to me because you have not given detail...I think you feel something hard like steel inside your lower lip when you rub with your tongue..right?If its so then might be some pathological leison has been developed...Proper radiographic examination should be done...If its not so then it can be psychological....Hope this will help you...Get well soon..Regards...."
},
{
"id": 128656,
"tgt": "Need treatment for neck pain",
"src": "Patient: Previously my doctor prescribed me flexoril (cyclobenzaprene) for my muscle spams. Ive been having this neck pain for about a week now and I was wondering how up to how long these can last for? I don't have any headaches or any other side affects. My neck is tender a bit at the base of my skull in the back of my head and thats all. Should i be concerned or should this subside? Doctor: it will subside. Just a advise change your posture. In case you are working much on computer or using excessive mobile don't use continuously. change in posture and neck extension exercise will prevent pain happening again"
},
{
"id": 14293,
"tgt": "Can itchy bump on arm be heat rash or eczema?",
"src": "Patient: I have smale bumps that itch in the crease of my arm where it bends... I notice it bothers me more when it gets hot and gets worst at night... But this summer I have been exposed to the sun and heat more then usual ... And it started on my left arm now it s staying on my right arm . could be a heat rash or eczema ? Doctor: Hi,It may be atopic dermatitis.Kindly consult the dermatologist for the perfect diagnosis and proper treatment.Antihistaminics like desloratidine 5 mg twice a day, steroid in the tapering dose for few weeks and application of tacrolimus oint 0.03% might improve the condition.Avoid contact with suspected allergens.Avoid eggs,fish,prawns,cashew nut,peas,soya,fermented food in diet.Avoid soap during bath.Keep the mind cool.I hope this might help you.Thanks.Dr.Ilyas Patel MD"
},
{
"id": 53082,
"tgt": "Noticed liver lesions and shadows in ultrasound and ct scan",
"src": "Patient: I am in hospital and 34 years if age. In good enough health. I ve had an untra sound and a ct scan yesterday which shows liver lesions and shadows. They want to get me to do another ultrasound on my tesicles. I m a young just married father and so worried help Doctor: Hi, I had gone through your question and understand your concerns.This findings is definitely pathological but still not enough specific since it may indicate various diseases. Some of them are benign and some malignant and may include even serious issues such as cancer. However, in your age this is rarely seen and it is more likely something benign such as hemangioma, adenoma, fatty liver or some other benign tumor. However, it should be ruled out and it is good that you will do l necessary tests as soon as possible. Even serious issues can be managed surgically so you dont need to be worried.just follow doctors recommendations.Hope this answers your question. If you have additional questions or follow up questions then please do not hesitate in writing to us. I will be happy to answer your questions. Wishing you good health."
},
{
"id": 24288,
"tgt": "What causes low heart rate with dizziness and weakness?",
"src": "Patient: Hi, I was overweight about 6 months ago, I weighed around 136 KG, I then changed my life around and started eating right and exercising and I now currently weight 94 KG.Sometimes without even exercising I often feel quite light headed, dizzy, and weak, even after having something to eat.and then there's instances where I will fuel and hydrate pre-workout and within 2 minutes of even light jogging I start to feel weak and dizzy and light headed and need to stop.Today I went to the Clinic and got an ECG and they found a very low heart rate, was around 48BPM she said and she said it could be to do with fitness but I don't know, I really don't feel very good and fit when I fule and hydrate and then within 2 minutes of light jogging feel weak and dizzy and need to stop..I am going to the doctors about this tomorrow but, my question is, I wanted to play soccer next year, and obviously feeling like this after a slight workout isn't very practical, I was wondering if there is anything you know of that can stop me feeling like this when I work out?Is there anything I can take such as even just a bit of caffeine to get my heart rate up or anything?Thanks Doctor: Hello!Thank you for asking on HCM!I carefully passed through your question and would explain that a low heart rate could be related to regular physical activity. But, based on your symptoms, it is important performing further tests to investigate for the possible causes: - a cardiac ultrasound to examine your cardiac function and exclude possible cardiomyopathy or heart failure- a cardiac stress test to investigate for possible coronary artery disease- some blood lab tests (complete blood count for anemia, thyroid hormone levels for possible dysfunction, cortisol plasma levels for possible adrenal gland dysfunction, blood electrolytes for possible imbalance). Meanwhile, I recommend you to closely monitor your blood pressure and refer those values to your attending physician. Hope to have been helpful!Feel free to ask any other questions whenever you need!Kind regards, Dr. Iliri"
},
{
"id": 217855,
"tgt": "Suggest remedy to relieve pain in heel and hip?",
"src": "Patient: i LANDED ON MY LEFT FOOT FROM AN 8 FOOT JUMP THE EMERGENCY ROOM SAID IT WAS A SPRAINED ANKLE. i HAVE LITTLE TO NO PAIN IN MY ANKLE BUT CANNOT PUT ANY WEIGHT ON MY FOOT DUE TO SEVERE PAIN IN MY HEEL. THE PAIN EXTENDS ALL THE WAY UP TO ALMOST MY HIP. wHAT CAN i DO TO FEEL BETTER AND HOW LONG TILL I CAN WALK AGAIN? Doctor: Hai.u can apply crepe bandage, elevation of the limb will help. U cam take tabs like ultracet (paracetamol and tramadol) to control the pain . immobilise the limb at least for a week. ? ? Dont walk on tat injured limb."
},
{
"id": 25824,
"tgt": "Should i take BP medication for borderline BP reading taken 12 months back?",
"src": "Patient: Hi,I had my blood pressure taken by wearing the apparatus overnight,that was 12 months ago. I went to my doctor only now for results,he said I was borderline and gave me a prescription for Lestace 2.5 should he have taken my blood pressure again as it was a long time since it was done. Or do you think I should get my prescription. Doctor: Thanks for your question on Health Care Magic. I can understand your concern. In my opinion, you should not start the treatment at present. You should definitely get done repeat blood pressure monitoring before starting any antihypertensive drugs. You had high blood pressure 12 months ago. So this is very long duration and you had only mild elevation. So you might have normal blood pressure at present. So better not to take this antihypertensive drug without fresh measurement of blood pressure. Hope I have solved your query. I will be happy to help you further. Wish you good health. Thanks."
},
{
"id": 149128,
"tgt": "Have cervical spondylosis, vertigo, migraines. Done ECG. On holter monitor. On nystagmus and vertin. Continue medication?",
"src": "Patient: Hi,I am an Indian female, 50 years old. in geerally good health. Exervise daily for about an hour. I have had cervical spondylosis for over 12 years now. Also have a history of vertigo (for 5 years) and migraines. Consulted a neurologist. I took a nystagmus test which confimred his diagnosis of a damaged inner right ear and the balancing nerve. Also, ECG with a Holtor monitor revealed a mitral valve prolapse. Have been advised a beta blocker and Vertin 48 mg a day to be taken in the morning. Also Fenofibric 105 once a day, Dizitac twice a day and Inderal 10 mg twice a day.The Vertin makes me feel extremely drowsy all the time and also seems to slow my speech.Is 48 mg a too-high dose? I weigh 60 kg and am 157 cm tall. Doctor: Hi,Thank you for posting your query.Vertin (betahistine) is a common drug used for treating vertigo and is a relatively safe drug. Drowsiness and sleepiness is a common side effect of this drug. A total dose of 48 mg of vertin is routinely used, however, if you feel drowsy, please reduce the dose by half to 24 mg total. You can either take vertin 8 mg three times daily or 16 mg twice daily.In addition, you should also do vestibular rehabilitation or adaptation exercises.You can expect to get better in a few weeks.I hope it helps. Please get back if you require any additional information.Wishing you good health,Dr Sudhir Kumar MD (Internal Medicine), DM (Neurology)Senior Consultant NeurologistApollo Hospitals, Hyderabad, IndiaClick on this link to ask me a DIRECT QUERY: http://bit.ly/Dr-Sudhir-kumarMy BLOG: http://bestneurodoctor.blogspot.in"
},
{
"id": 151141,
"tgt": "Lower back pain, pain in shin. MRI shows sciatic nerve problem. Tingling limbs after taking steroid. Cause for concern?",
"src": "Patient: Hello. Ive had lower back pain for a few days now. It s hurting in my lower back, butt and into my shin on my left leg. I went to the ER yesterday because I couldn t stand the pain anymore. He didn t do any exrays or MRI but he said it appeared to be my sciatic nerve and said I was having muscle spasms because my thigh muscle was jumping. He gave me a steroid shot and sent me home. I woke up an hour or so ago and both my legs and both my arms are tingling . Is this cause for concern? Do I need to go back to the ER? Doctor: Hi, Thank you for posting your query. The initial problem with which you went to the ER is suggestive of nerve compression in the lumbar (lower back) region. This is commonly caused by the prolapse (bulging) of the disc. Common symptoms include lower back pain, pain in the legs, tingling, numbness, etc. It can be diagnosed with the help of MRI. If symptoms persist, you can get the MRI done. Your second problem of tingling of arms is probably not serious and is due to compression of nerves while sleeping. It should recover in a short span of time and there is no need to go back to the ER. Best wishes, Dr Sudhir Kumar MD DM (Neurology) Senior Consultant Neurologist"
},
{
"id": 158094,
"tgt": "Stage four stomach cancer. Taken one round of chemotherapy. Necessary to take six rounds?",
"src": "Patient: My sister who is 33 years has cancer in the stomach stage 4. The doctors say she has 5 more months to live. She is not in pain and is as active as she was before the diagnosis in June this year. Except for a swollen stomach (the cancer has spread to the right ovary ) no one can tell she is unwell. She took the 1st round of the chemo and the doctors have recommended she takes 6 rounds taken after 21days intervals. if she has 5 months to live is it wise then she continues with treatment which she is taking in XXXXX. We are XXXXXXX. Are there known alternative therapies that she can use to pro-long her life. Doctor: Unfortunately, about 8 out of 10 (80%) people diagnosed with stomach cancer are stage four, meaning the cancer has already spread when they are diagnosed. Understandably, the survival statistics are lower than for stage 3 stomach cancers. Fewer than 1 in 20 people (5%) live for at least 5 years if they have stage 4 stomach cancer when they are diagnosed.She has to take atleast 3 cycles of chemotherapy first followed by CT scan of whole abdomen to find out the response assessment.If after 3 cycles it is stable disease or partial response then 3 more cycles need to be taken followed by reasssesment again.If it is stable disease or partial response then CT scan whole abdomen after 3 months.If progressive disease then plan for second line chemotherapy."
},
{
"id": 23142,
"tgt": "Could the pain in the arm be related to heart?",
"src": "Patient: I am 60 years of age I have had a pain down my right arm when this happens I am unable to liftor carry anything it a sharpe pain and lasts for a few seconds I only have this symptom now and again do you think it has any thing to do with my heart. Doctor: kind of description you have given, it's appears to non cardiac pain. it's probably neurological in origin. Age ?Any history of trauma or neck pain?any tingling sensation or numbness?cardiac usually on left side and dull ache type"
},
{
"id": 201937,
"tgt": "How to control sexual drive?",
"src": "Patient: i am 27 years male. i never had girl friend. i could not control my sex feelingx. i earning very less money only. so i cant go for marriage also. i dont want to go for wrong affairs too. so i just want to reduce or stop my sexual feeling. is there any medicines or drugs or operation to stop my sex harmones? Doctor: Hi,Thanks for writing in.The best way to release sexual feelings is by doing masturbation about 2 to 3 times a week. Please do not allow sexual thoughts to come to your mind at other times. This can be practised by getting involved in your work and spending time with family and good friends. There is no need to take medicines or get operations done. Please try to meditate and go for walking or running when you have free time."
},
{
"id": 70617,
"tgt": "Why would someone have persistent cough for several years despite administering Allegra-D and nasal spray?",
"src": "Patient: ENt says I have esophageal reflux prescribed 4 or 5 prescriptions. Pulmonologist says it is allergies and prescribed nose spray and prescription Allegra. D. Have had persistent cough for more than several years. Cough all night and good portion of day.. It is life altering. Nothing has helped. Where do I go from here? Thank you Doctor: Hi Dear,Understanding your concern. As per your query you have persistent cough which is due to chronic lung infection or bronchitis. Need not to worry much. I would suggest you to visit pulmonologist once and get it examined. You should go for pulmonary function tests and x-ray chest as well to check exact level of infection. Start treatment only after proper prescription. Drink plenty of water. Do Not lay in bed immediately after having food as it will leads to formation of excess acid. Take hot milk rich in turmeric as it will boost immunity. You need inhaled bronchodilators and antihistamine drug. You should use mist humidifiers in room. You should take cough expectorants as well.Hope your concern has been resolved.Get Well Soon.Best Wishes,Dr. Harry Maheshwari"
},
{
"id": 11991,
"tgt": "Is there any medicene or supplement to cure white eyelashes ?",
"src": "Patient: I have Vitiligo from last 10 years, just few spots on the body, however I have few white eyelashes on my left eye n use mascara to cover it up, Is there any medicene or supplement to cure white eyelashes. Please suggest. I have Vitiligo from last 10 years, just few spots on the body, however I have few white eyelashes on my left eye n use mascara to cover it up, Is there any medicene or supplement to cure white eyelashes. Please suggest. Doctor: hi,thanks for query.The white patches are part of generalized vitiligo.There is no special cream which can help.Please take treatment for dermatologist from vitiliogo and stick to one doctor as treatment is long and takes time to respond. wishing you good health."
},
{
"id": 213171,
"tgt": "Suffering from OCD. Not taking medicines consciously. How to cure?",
"src": "Patient: My unmarried daughter aged 24 yrs has been suffering from OCD such as playing brainvita game, surfing facebook repeatedly, listening to ipod frequently/listening to songs on player aloud, when in the bedroom, she sits with her arms resting on hip, gets up and then stands and then with a jerk sits on the bed. she also has been browsing through one particular photo album relating to us including herself, refuses to part with the album. she does not venture outside our balcony leave alone outside. she has been abusing us (parents) whenever v ask a general query getting aggressive. She used to blame her state of affairs such as short hair/ skin infection , menstrual problem and other aliments on some of our close relatives. she does not do daily tasks such as cooking, cleaning the house, washing clothes, leaves all of them too her mummy (aged 59 yrs). She started getting depressed somewhere in 2004 when she failed to perform to her expectations on XII Std scoring only 52% thereby denied getting admitted to some college. BCom Correspondence Course of Delhi University she took up; result was the same, got compartments in the I and III Yr course. She did complete the course after much pursuasion with a gap of one year. Simultaneously, she tried to pursue Company Secretary Course, completed only Foundation Course and in the first yr after that, the assignments which sent was returned to her advising her to focus on the questions in it and reply accordingly. From then on, she felt quite dejected. She did apply for NIIT Computer Course without our concurrence, forced mummy to shell out cheque for tuition fee, but after delivering it to the institution, stopped attending classes. v told her there was no need to apply because vr already having computer; she did not listen to us. She has been applying for job at times but without confiding with us. She used to venture out voluntarily for interview but failed to get anything. Thereafter, she did join a 2 yr course in South Delhi Women Polytechnic but after completing a successful one year, she remitted tuition fee for Rs 8,000/- by forcing us to part with for the 2nd yr but after a day she decided to stop attending once for all. Ultimately, v did take her to a psychiatrist in Safdarjung Hospital and after counselling, began to take risperidone and pari tablet for a month but failed to turn up to see the concerned doctor. v had to indirectly administer the tablets by mixing with dough. v have settled down here after retirement in Oct 2011 and as of now giving her sezodon liquid 2 ml with Pacitane on advise by a private psychiatrist here. No visible impovement has been noticed. In this scenario, is it advisable to search for matrimonial alliance? Please advise as to how to make her take tablets/liquid consciously because even now, v had to mix the liquid with lime water/salad/buttermilk without her knowledge. she rarely talks to us on any subject, excepting that she makes tea for self, eats whatever was served to her, but not a trace of sympathy, affection towards us. Awaiting ur advice. Doctor: Hello, Sir..from the detailed history you have provided it is clear that she had been diagnosed correctly and she has been given right medication. But as you have mentioned she is not showing much improvement , it is clear that she need trial of other drugs as well. There are many options available and you can give those medication. But for that you need to visits a near by psychiatrist and get the required prescription. As far as your desire to give her medication with her knowledge , than you need to admit her in a institutional setup as in her illness patient consider them self absolutely fine and think that there is no need of treatment. Another question you asked is about her marriage. Mostly people advice that get such people married and change in place will make a difference. But I would suggest you to think many times before taking such step, as marriage does not cure such illness and it may even act as a stressor and aggravate it. Also you should let the other party aware of her condition pre marriage because after marriage they will eventually get to know her condition and you may have to go through a lot of legal hassle . So overall consult a good psychiatrist, do regular follow up and she can be better with that."
},
{
"id": 43217,
"tgt": "Infertility problem. Right tub be blocked and left partially open. Menstrual blood test done which was negative. Prolactin is high. Suggestion?",
"src": "Patient: hello dr i am having infertility problem .my right tube is block and left one is partially open . i had my tb test done by menstrual blood test which was negative .now my doctor had prescribed me to go for tb treatment as he think tb to be the reason of tubal blockage .my prolactin is little high .what should i do .please suggest. Doctor: Hi,Thanks for consulting us.This is true that Tuberculosis is very common in certain parts of the world. When this involves the genital tract, subfertility may result.I don't think it would be justifiable to start ATT ( anti-tuberculous treatment ) without confirming the diagnosis first. I think you should take a second opinion before starting ATT, as this is not a standard practice to start treatment without lab evidence of the disease & there are better treatment options to help women with tubal blockage. Women with borderline raised prolactin do not need treatment. So, better search for a good fertility specialist in your area.Wish you good health.Best of luck."
},
{
"id": 173810,
"tgt": "Will swallowing of dologel ct gel by a 5 year cause any problems?",
"src": "Patient: My son is 5 years old. he swallowed about 0.5 cc of DOLOGEL-CT gel. On it , is mentioned that it should be kept away from children. After that he had 100 cc of vomitus. please let me know , will he suffer from any problem? I am very much worried. please provide me the action if required. Doctor: Hello dear,Thank you for your contact to hcm. Dologel contains choline salicylate .It is for external use, you should give to son Activated Charcoal -1 tablet/10 kg/day for absorption gel and toxins during 2-3days. Then everything will be ok. Kind regards Dr.Svetlana"
},
{
"id": 26637,
"tgt": "Suggest treatment for congestive heart failure",
"src": "Patient: Name: Diane Deaton Email:Comment: My dad is 88 years old and just admitted to hospice with congestive heart failure. I was reading about heart pumps and would like to see if he would be eligible for this and if he could get an appointment. I know his age is not good but I would rather take the chance of doing this than doing nothing except waiting for him to die. Please email me in regards to this. Me and my dad have grown closer in the past two years and I am not ready to let that go if there is any hope at all. Thank you so much.Phone: 859-912-1154 Doctor: Hello and welcome to HCM, madame. I carefully read your query. If you are referring to a left ventricular assist device (LVAD), they are usually used as a bridge to heart transplant. I am not sure how this works in your country, but I am quite confident that an 88-year old is not a candidate for transplant and, I am afraid, does not pass in the heart transplant committee. However, there are other precautions that can be undertaken in cases of congestive heart failure with low ejection fraction. It would be helpful for me, if you had provided the ejection fraction of the left ventricle, assessed with an echocardiogram. Nevertheless, the therapy should be strict and adhered to. Besides other drugs, a good outcome accompanies the addition of an angiotensinogen converting enzyme inhibitor (ACE-I) or an angiotensinogen II receptor blocker (ARB), a potassium-sparing diuretic (spironolactone) and a loop diuretic (furosemide), to both oppose the compensatory mechanism which, when acting longer than needed, are dangerous to the organism, and, also to reduce the heart workload. Besides this, salt should be totally withheld from his diet, because it is estimated that 1 g of salt retains 10 g of water. Also, you can discuss with his cardiologist about the possibility of implanting an ICD (implanted cardioverter-defibrillator, to automatically convert the potentially life-threatening ventricular arrhythmias, should they happen, which have a high predilection in the damaged heart muscle) and of a CRT (cardiac resynchronization therapy, basically a biventricular pacemaker, which helps both ventricles to contract closer in time). I hope this helps and answers your questions. Take care. Regards, Dr. Meriton"
},
{
"id": 218335,
"tgt": "Is it safe to take Ecosprin, Clexane, Susten and Thyronorm in the first trimester of pregnancy?",
"src": "Patient: I was termed lupus anticoagulant after 1st miscarriage.Now iv conceived and have been advised to take ecosprin150+daily clexane 0.4mg+susten25 mg twice weekly + lobet 100mg daily+ thyronorm100mg because I have thyroid and I was a high BP patient too. Currently the BP stays Normal. Current test of lupus is also negative but doc still keeping me on these medicines. Is it safe in first trimester.i am more worried of Aspirin dose. Doctor: Hi, The four medications you are taking is absolutely safe during pregnancy. Ecosprin and Clexane will prevent the formation of blood clots which is very important in individuals diagnosed with lupus anticoagulant. Formation of micro-thrombus (small blood clots) in the placenta will lead to miscarriage. So please be advised to continue the medications. Susten is a progesterone which helps maintain pregnancy. Thyronorm is thyroid hormone and it is very important to continue taking it as prescribed. Normal thyroid hormone levels are very important for healthy pregnancy outcomes. Please address any more questions or concerns here. Hope I have answered your query. Let me know if I can assist you further."
},
{
"id": 183220,
"tgt": "What causes scattered white spots in site after tooth extraction?",
"src": "Patient: Hi I just had my bottom right back molar extracted on friday morning. I was in a good amount of pain but Its now Monday (72 hours after tooth extraction) and the pain is minimal just a lot of soreness and swelling. I heard swelling can peak at 48 to 72 hours post extraction so I'm not too worried about that. What I am worried about is how today when I woke up I noticed these white spots scattered around the site where the tooth was extracted and also a few on my inside cheek. They are not painful but I'm scared because I don't know what they are. What couls these white spots be? Dead tissue? Is this part of the healing process because I can't find anything on internet which worries me more. Doctor: Hello, Read your query as you have white spot on extraction site this can be deposition of food debris due to difficult in maintain proper oral hygiene , it can be periodontal flap also , dont be worried so much I will suggest you to follow post operative instruction given by your dentist , do warm saline rinses , take proper course of medication prescribed by your dentist . Visit dentist for follow up.Hope this will help you."
},
{
"id": 41960,
"tgt": "Suggest treatment for low sperm count and ovarian cyst",
"src": "Patient: iam nearing age 31 & my husband nearing 30. we try to conceive past 16 months but not we checked my husband not having mortial sperm (0.01) & my overies having cyst & fobroid also. we are very disappoint, my doctor advice my husband to take medicine to increase count, & advice to do iui after my spouse treatment completed, it is ok ? pls advice me. Doctor: Hi thanks for writing to health care magic.I have gone through your question.As your husband have low sperm count and low percent of motile sperm then you should investigate for him. U should consult urologist and do color doppler ultrasound to find vascularity of testes and to rule out varicocele.There are supplementary medicine for increase sperm count.For you fibroid's size location matter for pregnancy and you should consult a gynecologist for the same.Iui is best option available.Have a good life and success..."
},
{
"id": 122740,
"tgt": "What is the difference between broken and sprained finger?",
"src": "Patient: how can you u tell the difference between a break and a sprained finger?y my husband was working outside on the house. then he hit something wrong he felt it turn but didnt here a pop but it hurts to bend and is swollen it is his ring finger. should he go see a doctor Doctor: Hi, Sprained finger means there is no fracture in the bone and only injury to soft tissue is there, however, broken finger means there is a fracture in the bone. Get done the x-ray to rule out any fracture.and yes you should go to hospital.till than give cold fomentation, over the counter painkillers. Hope I have answered your query. Let me know if I can assist you further. Regards, Dr. Jaideep Gaver, Orthopaedic Surgeon"
},
{
"id": 198060,
"tgt": "Suggest treatment for a painful lump on the penis",
"src": "Patient: I have a lyph node like lump under the skin of my penis. It appeared after masturbation and this is the first time i've experienced this. There is no dicoloration and it only looks like something's underneath the skin. It's right under the head of the penis on the right hand side and it moves with the skin. For example, if i pull the skin down, it goes down with it. It does not hurt unless i put pressure on it. I can move it around and pinch it to \"pick it up\". I have done some reading and i think it is lymphocele. I'm not positive and a doctors opinion would be nice Doctor: HelloThanks for query .Freely movable lump in foreskin is most likely sebaceous cyst however it can be confirmed only on clinical examination by a qualified General surgeon. The genital skin is rich in sebaceous glands and hence prone to get more sebaceous cyst due to accumulation of sebum beneath the skin.Normally it fades away without treatment however they need to be treated if increase in size or get infected.Please consult qualified General Surgeon for clinical evaluation and further treatment.If needed it may need to get excised in Toto (Completely along with the sac).Dr.Patil."
},
{
"id": 134373,
"tgt": "What causes uncomfortable feeling in fingers?",
"src": "Patient: Hello my left side of my arm near my fingers and around the heart feel uncomfortable like something is there. I went to a cardiologist and he said that my heart is fine and nothing is wrong and that it can be stress from my excessive typing on the computer. Is there anyway to solve this issue. Doctor: hi,thank you for providing a brief history. well as you mentioned in your message about you feeling uncomfortable on left arm and hands. well ofcourse one will get panic and look for a cardiac symptom. it is really to appreciate that you took immediate step and met a cardio. now since the cardio has mentioned about not to worry with the anything as nothing is wrong let's look up to the other factors.it may be a stress or work related issue. Since you mentioned you do a lot of typing that means you use a computer and there will be a lot of stress on the muscles around the neck and the chest muscles. now taking this into consideration about your job profile I suspect it to be a issue with your repetitive stress injury. that means you have pain and aches due to work related issues. constant sitting, long hours working, position of your neck and arms. why don't you try doing some regular breathing exercises, meditation, simple neck exercises and upper limb exercises. kindly see to even this that you take regular frequent breaks during your working time. as it will help to avoid stress on muscles and avoid work related injuries.hope you will be fine."
},
{
"id": 220359,
"tgt": "Can anal sex be performed during pregnancy?",
"src": "Patient: hii i m 25 years married and i m in my 5 month of pregnancy and till time we used to have sex but we avoid missionary posit.and we used to have anal sex so is it right enough to have sexual relationship during 5th month of pregnancy and is it right to have anal sex .and also please tell me till which we can have sex i order to avoid any discripancies.. Doctor: Hallow Dear,Let me congratulate you at the outset on your pregnancy. During pregnancy, sex is not forbidden. During first three months, since the pregnancy is at a precarious stage, we advise to observe sexual abstinence. However, after completion of three months, you can have sex till you complete 7 months. During sexual intercourse, your pregnancy will be more safe if you practise sex with incomplete insertion of the penis in to the vagina. Intercourse in lateral (side) position will facilitate this type of intercourse. After 7 months, since the abdomen has started protruding substantially, man on woman or from the front of woman may be bothersome and causing pressure on the abdomen. There is a risk of complete penile insertion in the reverse position. Hence, vaginal sex with man behind the women will facilitate such sex. If you refer to Kamsutra by Vatsyayan, there are some specific positions explained for sexual intercourse during pregnancy. So with such allowance for intercourse during pregnancy, I do not feel there is any necessity to have an alternative option of anal sex. However, if that is your choice, you have to take precautions mentioned for the vaginal sex. The choice is yours. I hope this helps you for a safe happy marital relations. Dr. Nishikant"
},
{
"id": 222417,
"tgt": "What causes inconsistencies in menstruation?",
"src": "Patient: Hi, I ve been trying to conceive. I had intercourse 3 days before ovulation (day 14) and again during ovulation (day 17). My period was 4 days late and when it did show up the first 2 days were very light and bright colored, my period stopped completely on the 3rd day and was back again on the 4th day but lighter and shorter than the previous days. I have also been feeling nauseous and have now noticed a weird rash on both sides of my neck. I have had 4 home test that were all negative. I am just wondering if, based on the symptoms above, I have conceived. Doctor: chances are less that you are pregnant because your tests are negative and you had bleeding almost on due date. still wait for 1 week and do repeat test. rash could be because of some other reason."
},
{
"id": 129963,
"tgt": "Why does my knee pop when I walk?",
"src": "Patient: My knee usually pops when i walk. Doesn't hurt just makes a popping noise that others can hear. Last night when I was sleeping, my knee started to hurt and almost felt like it needed to pop like you would with your knuckles. Every once in awhile it does happen but not as often as my knee popping. Any ideas? Doctor: Hi...The popping noise in your knee is due to friction developed in the knee due to muscle imbalance.The bone slides over the other and move, at the end of movement give rise to pop while releasing the frictional pressure..I suggest you to kindly do lot of stretching of the muscles around the knee especially Quadriceps Hamstrings and calf...Do lot of icing around the knee...Do glute strengthening exercises like... Bridging clamshell..step ups...squats...Do calf raises...Hope this is helpful for you...Kindly revert back in case you need any further clarification..."
},
{
"id": 170784,
"tgt": "Suggest medicine for constipation",
"src": "Patient: my 1.5 year old daughter is having difficuly passing stools..she is 10.5 kg and avg height, and is now fed on Lactose free milk as suggested by PD.But still no change.she has solids, dal-rice,banana squash, all fruits, and drinks good amount of water. Doctor: Hi...Thank you for consulting in Health Care magic. I think your kid is having habitual constipation. I have certain questions and suggestions for you.Questions:1. Did your kid pass motion or meconium on day one of life?2. Since how long is the kid constipated?3. Does the kid have any bleeding along with hard stools?4. How much milk does the kid consume per day?5. Does the kid eat fruits and vegetables (fibre diet) appropriately?You can get back with answers at the following link - www.healthcaremagic.com/doctors/dr-sumanth-amperayani/67696Suggestions:1. Natural methods are the best to relieve constipation.2. Constipation is a risk factor for UTI3. Maximum milk consumption per day should not exceed 300-400ml4. Minimum 3-4 cups of fruits and vegetables to be consumed per day5. Toilet training - that is - sitting in Indian type of lavatory daily (I mean in squatting position) at the same time will help a lot.Hope my answer was helpful for you. I am happy to help any time. Further clarifications and consultations on Health care magic are welcome. If you do not have any clarifications, you can close the discussion and rate the answer. Wish your kid good health.Dr. Sumanth MBBS., DCH., DNB (Paed).,"
},
{
"id": 15004,
"tgt": "Can taking high dosage of niaspan result in red rashes and swelling on skin?",
"src": "Patient: I have been taking Niaspan er 1000mg for a few years. Last summer I came down with a terrible rash (redness on my face and neck) Went to dermatologist who gave me meds and it subsided (cleared.) Also went to allergist for tests and info at the same time. Now, this condition has returned with a vengeance. My face is red/swollen somewhat and burning. The skin flakes on my face and this rash has moved to include my right arm and hand. Before I go for my next doctor(s) visits: could this possibly be a reaction of taking Niaspan in such a high dose and for so long? I do take an aspirin as prescribed for the \"flushing\" but this is an unusual rash. Thank you.Pauline Witriol (age 77) Doctor: Hi,After reading your query the closest possibility of your problem appears to be \"allergic dermatitis\". This is a condition caused by bursting of allergy producing cells known as \"mast cells\" due to contact of certain allergens. As the chemical inside them is released causes the kind of rash which you mentioned. There are many factors inducing the increased number and activity of these cells.Kindly follow the suggestions below:-Apply fluticasone cream 2 times a day.-You can do cold water compresses to reduce the intensity of itching.-Do not scratch or rub the skin as it can lead to further aggravation.You can take oral anti histamine like loratadine or levocetrizine under medial advise. Despite above measures if the problem is not getting better then it is better to consult a dermatologist for best guidance.Hope these informations will help."
},
{
"id": 209570,
"tgt": "Can Xanax be used for panic attacks and anxiety over small things?",
"src": "Patient: Do you absolutely need to have panic attacks to take xanax ? I am anxious everyday over very little things, mostly about things that has happened in the past and somehow worries me( not big things, just things like wow I shouldn t have said this to that person 2 years ago. ), but when this happens, I want to scream and punch on something, or just nervously bite my nails or hit myself. I have gotten panic attacks before, but they re not my main concern at all, my main concern is that anxiety feeling I have everyday over very silly things, and if it would be appropriate to use xanax ? Doctor: DearWe understand your concernsI went through your details. I suggest you not to worry much. If you know the reason for your silly and small panicky thoughts, why do you want to ask about anxiety medication. Why don't you treat the root cause, the cause itself? Is it not necessary?Anxiety, worries and panick attaks come when you are being idle. Remember, \"IDLE MIND IS DEVIL'S WORKSHOP\". You are actually fearing the fear itself. Past incidents are just incidents. They gave you positive lessons and experiences which you are supposed to carry forward and use in your future. Instead you are carrying \"just\" incidents. How can incidents worry you? A snake can startle you, but can a picture of a snake? A picture can startle you if you are worried about it, you start fearing the picture also. I think I am making myself clear. Your treatment is psychotherapy. Not anxiety medicines. Start with these steps:Step 1 - Practice Deep and full breathing whenever possibleStep 2 - Drink plenty of water and fruit juices. Step 3 - A Tomato a day keeps anxiety at bayStep 4 - Exercise regularlyStart these and follow strictly. After 7 days please post a direct question to me in this website. Make sure that you include every minute details possible. I shall prescribe some psychotherapy techniques which should help you cure your condition further.Hope this answers your query. Available for further clarifications.Good luck."
},
{
"id": 208968,
"tgt": "Why does tongue become black during seizures?",
"src": "Patient: Hi my 9yo son Has HIE and is on a vent with a trach, he has a Hx of febrrile seizures for about 4 yrs, he has been having epiosodes where his tongue turns black and his body shakes and he stares off into space more than usaul, it isn t like hisnormal seizures, they have been under good control latlely, but it seems like a seizures, I m going to call his neurologist on Monday but worried in the meantime. Doctor: Hello,Thanks for choosing health care magic for posting your query.I have gone through your question in detail and I can understand what you are going through.It is cause of the trauma to the tongue and internal bleed. Hope I am able to answer your concerns.If you have any further query, I would be glad to help you.In future if you wish to contact me directly, you can use the below mentioned link:bit.ly/dr-srikanth-reddy\u00a0\u00a0\u00a0\u00a0\u00a0\u00a0\u00a0\u00a0\u00a0\u00a0\u00a0\u00a0\u00a0\u00a0\u00a0\u00a0\u00a0\u00a0\u00a0\u00a0\u00a0\u00a0\u00a0\u00a0\u00a0\u00a0\u00a0\u00a0\u00a0\u00a0\u00a0\u00a0\u00a0\u00a0\u00a0\u00a0\u00a0\u00a0\u00a0\u00a0Wish you good health,Kind regardsDr. Srikanth Reddy M.D"
},
{
"id": 70853,
"tgt": "What can cause shortness of breath along with dizziness?",
"src": "Patient: I can't get enough air in my lungs and I feel dizzy at the third attempt, I feel heavy when breathing.This thing comes for several days and goes, but it continues, I think it's started from two or a year and a half. you will say why you don't go to the hospital? Because I thought it for psychological reasons but now it get worst(I have 17 years and half)( no drink, no smoke)( I don't take any medication)(I don't have allergic)(height :167 cm, weight: 54 kg)(female)\" excuse my bad English \" Doctor: Hi, * As per my clinical experience, the combination of short breath and dizziness sound to be low haemoglobin, the altered immune status of the lungs or others. * You require primary evaluation with basic lab tests, x-ray chest with your family doctor. Hope I have answered your query. Let me know if I can assist you further. Regards, Dr Bhagyesh V. Patel, General Surgeon"
},
{
"id": 127173,
"tgt": "How can swelling and pain in the ankles and knees be treated?",
"src": "Patient: my ankle is swolen actually it s my whole foot which swells up like a football, is red and very painful. This foot is deformed and been that way since birth. My elbows, shoulders and knees hurt as bad as my ankle and are swollen also. In emergency room yesterday but they could not help me = no doctors here in my immediate area treats feet. I also have a syrinx at the base of my brain stem which connects the spine. YYYY@YYYY Doctor: Hello dear,welcome to Ask a doctor service. I reviewed your query and here is my advice. I think you may be suffering from rheumatoid arthritis. You should consult a rheumatologist for treatment options. Some biological drugs are very effective and can help you. Hope I have answered the question. Let me know if I can assist you further."
},
{
"id": 38568,
"tgt": "Suggest treatment for swollen face, lips, eyes & cheeks",
"src": "Patient: I keep waking up with a really swollen face, lips, eyes, nose, cheeks - everything! My face has also become very dry and so is often numb or has a tingling sensation. For the past few nights when I am trying to go to sleep, I also am finding it hard to breathe and have been suddenly waking up gasping for air. What could be the problem? Doctor: HiThanks for posting your query . There are few causes for swelling of face . Most common among them is kidney problems , loss of proteins in urine , thyroid disease , adrenal disease , heart problems . Associated with Breathing difficulty it might be due to kidney or heart problem . I advise you to get these tests done and revert back : Complete Hemogram , Serum Creatinine , Urine routine , Ultrasound Abdomen pelvis , 2D echo . You can start taking TAb Lasix after consulting your doctor . Will be looking forward for your follow up . Regards ."
},
{
"id": 170506,
"tgt": "What causes poor concentration in children?",
"src": "Patient: My son who is 9 years old was diagnosed with mono yesterday. He s had trouble concentrating in school and has recently failed a couple tests over simple material the teacher said 90 % of the class got A s on. My son is normally an A or B student and doesn t do poorly on tests. Is this something I should worry about or is simply fatigue from the mono causing lack of concentration in school? Doctor: DearWelcome to HCMWe understand your concernsI went through your details. Signs and symptoms of (mono) mononucleosis may include: Fatigue. General feeling of unwellness (malaise) Sore throat, perhaps a strep throat that doesn't get better with antibiotic use. Fever. Swollen lymph nodes in your neck and armpits. Swollen tonsils. Headache. Skin rash.Mono usually do not possess any continuous concentration problems. The concentration problems could be present while the child is having acute symptoms of mono. If you and teachers are noticing a change in the level of concentration or school performance in your child, you should first consult a psychologist to determine the cause. The psychologist will be able to provide proper guideline and treatment including behavior therapy. If you require more of my help in this aspect, please use this URL. http://goo.gl/aYW2pR. Make sure that you include every minute detail possible. Hope this answers your query. Further clarifications are welcome.Good luck. Take care."
},
{
"id": 129519,
"tgt": "What causes pain in hands and shoulder?",
"src": "Patient: Started with pain in hands and wrist with the first three digits in my right hand going numb while driving. Over time got more pain in arm then in shoulder and neck. At night arm would go numb and be very painful. Recently On two different occasion I felt first pain on the inside of my palm and shortly after a vein popes leafing my palm blue and the same happened to one of my digits. Doctor: Hi Hope this message finds you in good health.I have gone through your query in detail and understand your concern.You are suffering from a slipped cervical disc causing compression of the spinal nerve in the neck.This causes pain in the upper limb and numbness in the hands and finger.I would suggest you to get a MRI of the cervical spine done.Methycobal and pregabalin tablets with use of cervical collar will help.Avoid using a pillow while sleeping.Get a MRI of neck done and consult an orthopedician.Hope your query has been answeredNothing to worry about, You should eventually get back to normal.Get back to me for any FOLLOW UP QUERIES anytime.Kind Regards,"
},
{
"id": 139512,
"tgt": "What could partial facial numbness indicate, neurological tests are inconclusive?",
"src": "Patient: Hello, My mother have freezing temptation on her left side of face, she had gone through different kind of scanning and other releated test recomended by nurologist. but Dr. said all the test result have normal. and they given some medicine for 2 months, but still theres no differents. it s been more that 8 months. stiil that freezing is there. any body can suggest very best doctor in Kerala. Doctor: Hello,Numbness is a very minor and nonspecific problems and common in many individuals. If the extent of numbness is the same and problem is not worsening in the background of normal scans I recommend to continue on current medications. Don't be worried about problem not improving.Hope I have answered your question. Let me know if I can assist you further. Regards, Dr. Shiva Kumar R., Neurologist"
},
{
"id": 146780,
"tgt": "How to treat severe dizziness?",
"src": "Patient: I woke up this morning rather dizzy when I opened my eyes and got out of bed. Not terribly dizzy, but unsteady. I have prescription for Meclizine 25 mg. 3 times daily. Have taken only one so far and due another in about an hour. I also took one zyrtec and two motrin along with my other meds. Losartan, Toprol XL50 and metformin 1000mg. Does this all sound like it is compatible and will hopefully relieve the dizziness? PLEASE CANCEL THIS REQUEST...DID NOT REALIZE THERE WAS A CHARGE. THANK YOU VERY MUCH. Doctor: Hello dear,The symptoms as mentioned in your post can be attributed to pathology in the Vestibular Apparatus in inner ear (it is associated with maintenance of body posture & balance) most probably Benign Paroxysmal Positional Vertigo.Symptomatic relief can be obtained with intake of Vestibular sedatives like Betahistine or Cinnarizine preparations (to be taken only under the guidance of a Physician).However, given the medical history that you are taking anti hypertensives and oral hypoglycemics, the role of sudden fluctuation of blood pressure and blood sugar levels also need to be evaluated.So, kindly consult your Physician/ ENT Specialist & get a complete clinical examination done.Investigations like complete blood count, estimation of blood pressure, blood sugar levels, serum electrolytes & vestibular function tests will be required to rule out any pathological cause for the symptoms.There is no need to worry, you will be fine.Till then, maintain adequate hydration & proper nutrition status and avoid stress.And also take precautions of getting up from bed slowly & avoid sudden head movements.Wishing you a Good Health.Take care."
},
{
"id": 63488,
"tgt": "What causes lumps in neck,chest and back?",
"src": "Patient: My 22-yr. old son has had lumps in his neck, chest and now back that appear then disappear within several months. Recently he had 2 lymph nodes removed from his armpit. The frozen sample was inconclusive. We re currently awaiting a pathology report. We ve heard everything from dysplasia to lymphoma. What are the chances we re dealing with Hodgkin s? Doctor: Hi, dearI have gone through your question. I can understand your concern. He has lump and enlarged lymphnode. It can be due to some malignancy or soft tissue tumor. In malignant lesion it can be hodgkin's lymphoma or some carcinoma. Or soft tissue tumor like multiple lipoma or neurofibroma with reactive lymphnode enlargement. So you should wait for biopsy report. It will give you exact diagnosis. Then you should take treatment according to diagnosis. Hope I have answered your question, if you have doubt then I will be happy to answer. Thanks for using health care magic. Wish you a very good health."
},
{
"id": 77387,
"tgt": "What causes the left side of the chest to feel heavy and painful?",
"src": "Patient: Hello, I've been having alot of problems with my chest.. for example when I wake up in the morning and sit up the whole left side of my chest is incredibly heavy and hurts alot. I get sharp pains at the same time. I get heart palpitations nearly everyday and my heart seems to thump agaisnt my chest alot of the time. If i put my shoulders back i feel pressure on the left side of my chest. Im not entirely sure but my heart seems to go from beating quickly then slow down alot of the time. Im always fatigued, weak etc. I get sudden dull aches in my pressure points in my wrist/hands/feet that come out of nowhere and leaves me with no energy afterwards. Doctor: Hi thanks for your question.Let me know you do you have hypertension or positive cardiac history ??First to rule out heart problems do ECG of chest there might be fibrillation present that can lead it.The palpitation could also be physiological.In increased metabolic activity palpitation can occured like fever, hyperthyroid etc.After rulling out cardiac problem move toward musculoskeletal pain.Note you have pain esp after getting up.So it might be by improper sleep posture.sleep in supine position rather then lateral.By heavy exercise or heavy weight lifting muscular strain can occur.simple analgesic taken.If ribs are tender to touch then costochondritis or ribs osteoporosis like changes secondary to calcium or vitamin D deficiency might be suspected.Try to avoid stress.I hope these suggestions will help you a lot.Dr.Parth goswami"
},
{
"id": 91535,
"tgt": "What is causing pain below sternum followed by bloated and hard stomach?",
"src": "Patient: Hi, My husband presented at the hospital last weekend with a pain just below his sternum. They confirmed it wasn't heart trouble but he hasn't been able to consult with a GP this week because of work commitments. He had an appt today with his Dr, but unfortunately he was sick and couldn't see him. He has had a bit of pain all week but his stomach is awfully bloated and hard. The Dr at the hospital that the pain could be from an ulcer, or GERD. He is of normal colour he hasn't been unwell and his bowel habits have been much the same. Doctor: Hi.The symptoms and the findings you have noted are not matching to each other.In spite of bloated and hard stomach your husband is working. He has only a bit of pain. GERD can not cause this . Ulcer can cause this only if it is perforated and healed immediately ( very very rare). There is a rare chance of of an epigastric hernia protruded with fat only to cause discomfort and hardness. I would advise to consult a GP ASAP and undergo all the relevant tests as advised. Bloated and hard abdomen can be due to subclinical form of intestinal infection not giving much of bowel symptoms."
},
{
"id": 78663,
"tgt": "Suggestions for symptoms showing hyperinflated lungs",
"src": "Patient: Hi, I have GERD for the past 13 years and I\u2019m taking Nexium. It really helps. Before I was diagnosed with GERD I coughed a lot, especially at night. Recently, I started to yawn and sigh very often. I had a chest x-ray and it showed hyperinflated lungs. Any suggestions? Doctor: Thanks for your question on Health Care Magic. I can understand your concern. In my opinion, you should definitely get done PFT (Pulmonary Function Test) to rule out bronchitis. GERD is common risk factor for bronchitis. Your symptoms like cough, yawning, hyper inflated lungs etc are seen in bronchitis. So better to consult pulmonologist and get done 1. Clinical examination of respiratory system 2. PFT (Pulmonary Function Test). You may need inhaled bronchodilators and inhaled corticosteroid (ICS). Don't worry, you will be alright. Hope I have solved your query. Wish you good health. Thanks."
},
{
"id": 190440,
"tgt": "Soreness on tip of tongue, pain on swallowing followed by burning, no fever, eases with cold water. What could it be?",
"src": "Patient: Hello, It started a few days ago, I noticed tip of tongue sore,almost like a canker sore, by next day swallowing was semi painful. Day three portions of tongue feel like there on fire,and its feels like im swallowing lumps . I am currently working as a nurse in long term care. I do not wish to pass this on of its strep . I don t have a fever , very uncomfortable,and taste buds are shot. It feels good if I drink ice water, or something scratchy going down my throat . I have a slight pink tongue. Thank.you so much for your time. Doctor: hello and welcome,this can be an ulcerous condition called aphthous ulcer.it can be treated with tab.flanzen-10mg;3 times daily for 3 days and tab.wysalone-5mg-3 times daily for 3 days.please discontinue any oral habits like tobacco or pan chewing if you have any;as it can lead to burning mouth and limited mouth opening called oral submucous fibrosis-osmf.please add vitamin-b containing food to your diet.hope you get well soon.always clean your mouth after every food intake.use soft-bristled tooth brush and tongue cleaner to clean your teeh."
},
{
"id": 15030,
"tgt": "Any remedy for rashes all over arms, neck and face with swollen eyes?",
"src": "Patient: I am currently working for a tree service putting Christmas lights up and developed a skin rash, I have had the rash for about 4 days. My eyes are swollen, and crusty in the am. I have red spots(rash) all over my arms, my neck and some on my face. Also my rash itches, a lot. I have never had an allergic reaction from pine trees or trees in general, but I just moved to CO from MI. I do wear a face mask, my arms are covered, and its been very cold while working. Help? Doctor: HIThank for asking to HCMThis could be urticarial rash and this will go away with the help of long acting anti-histamine and the best drug would be \"Levocetirizine\" have it three times in day and if does not go away then try \"Dexamethasone\" tab. 8 mg three times in day this will surely cure the symptoms have nice day."
},
{
"id": 185787,
"tgt": "What causes expulsion of worm like thing from a gum after a root canal?",
"src": "Patient: Hi, I went for my six month dental cleaning yesterday and a root canal gum (1990's) was inflamed for the first time and was irrigated with beta dine. I thought I felt something moving at the time expecting an almond skin but nothing cleared, not even tissue. This evening I checked my gum and pressed on it and out came a thin 5mm long deep pink colored piece of tissue I suspect is a worm? \u2026 this thing was 1 to 2mm wide and deep pink in color. I will alert my doc tomorrow since he suspects my canal has cracked and I'm to go to a specialist in month. I have frozen the little worm like thing. I was a wildlife rehabber of mostly birds for a decade, but that was from 1990 to 2001. What do you think? \u2026 I suspect a worm from the shape and color. Doctor: Hello:)Welcome to HCM.This can be pus from the infection.A thorough clinical and radiological examination is necessary to help you.You can do warm saline gargle till you see your dentist.Regards."
},
{
"id": 176206,
"tgt": "Suggest treatment for fever and dry cough in kid",
"src": "Patient: hi my 3 year old daughter has this fever for almost an week, and very dry cough, her throat is itching she said that's why she coughed every now and then. i gave her a mucolytic, then she has taken an antibiotic already, at first cough wasn't that persistent but lately it worsen...pls help Doctor: Children often suffer from viral infections that manifest with fever and cough. Generally it lasts for 5-10 days irrespective of the use of antibiotics which are only effective in case there is a secondary bacterial infection. But since you have begun the course of antibiotics, you should complete the full course to avoid development of antibiotic resistance.You have done the right thing by giving a mucolytic. Give some antiallergic like cetirizine along as that should relieve the allergic component that often remains associated. Giving steam inhalation should also help. Water in the form of vapour would reach deep into the lungs and help in diluting the cough."
},
{
"id": 40577,
"tgt": "How can infertility be treated?",
"src": "Patient: its been a year since we tried for a baby (was trying naturally)its only last month that I visited a doctor and gave me fertomid tablets which i used from first day of my cycle which goes for 5 days and then when i tested for ovulation the results were weak positive after i had sex on the 10 th day.What might be the cause? Doctor: Hello,When you have been trying naturally for one year and succeeded to get pregnant, that means it\u2019s the time to get evaluated. All that we need is1) Husband semen analysis2) Ultrasound to see if ovaries and uterus are normal and most importantly the antral follicular count, which gives us an idea about the eggs you have. 3) Hormones FSH, LH, TSH (prolactin if cycles r irregular).If the above evaluation is normal then as your doctor suggested you need to take your Fertomid tablet and have intercourse from day ten alternate days up to the twentieth day of cycle provided your cycles are regular and yes knowing when you ovulated and the ovulation tests would give us more accurate days to have sex. Usually, in a regular twenty-eight days cycle, women ovulate on day fourteen, and even after trying this for three months you don\u2019t get pregnant I would suggest you get fallopian tubes evaluated. Hope I have answered your query. Let me know if I can assist you further.Regards, Dr. Avanti Sathineedi"
},
{
"id": 94802,
"tgt": "Have burning, pulling sensation after appendectomy. Why is the pain throbbing? Cause?",
"src": "Patient: I am having burning, pulling, pain sensation after appendectomy a week ago. The pain is constantly throbbing and increases when I cough , laugh, bend over, sit up, or move a certain direction. I have no idea what is causing this . I contacted the doctor on call and he said it was nothing to worry about if i pulled a stitch or muscle. Doctor: Hi Dear thanks for question,you have under gone appendectomy,and now your having post operative pain,you have not mentioned in the history when your appendectomy was done,as if your in early postoperative period then infection can give rise to throbbing pain ,other cause could be damage to small nerve twig that lie in the incision line.or if your in immediate postoperative period, then it can be pain of surgery itself.i hope i answered your question"
},
{
"id": 190430,
"tgt": "Cheek has tingling sensation. No inflammation. But dizziness and headaches. Why?",
"src": "Patient: hello, A 50 yr. old patient reported to the dental clinic with tingling sensation over the right cheek region for 2 months. No h/o trauma, dizziness , severe headache, paraesthesia, running nose , URTI, blurring of vision. No sign of inflammation, no tenderness over maxillary sinus region. NO intraoral pathology detected which could explain the symptom. please kindly give your opinion on the probable causes of the symptoms with treatment plan. Doctor: Hello according to the dental history pf patient it seem that patients nerves are affected.i would like to know about patients oral habits like smoking or tobacco chewing because ingredients present in it irritates the mucosa causing tingling sensation.i would advice to give neurobian forte for relief and refer to neurologist. In case of smoking or any hazardous oral habits kindly dont continue.avoid rubbing your cheeks.mantain oral hygiene.rinse mouth with warm saline water twice daily. Regards dr.sapna"
},
{
"id": 164523,
"tgt": "What could constant pain in head, ears and stomach of a child mean?",
"src": "Patient: Hi, may I answer your health queries right now ? Please type your query here...My 4 year old daughter is complaining frequently of her head, ears and stomach hurting for the past month now. She just had Strep again in Jan. and and ear infection that followed about a week later. Have taken her doctor, ENT doctor and ER. They say she seems fine. I want to think she is going through a stage, but at times I wonder if it is really something. What would u recommend I do now? Doctor: if all examinations are fine then just do a antibiotics course of three days if any underlying infection it will cure , careful about hygiene."
},
{
"id": 157842,
"tgt": "Ongoing ear infection. Mucus in throat. Had bladder infection. Brain and cancer ailment likely?",
"src": "Patient: hi iv had ongoing ear infections for 2 years 2 months ago a er doctor was worried cause im in there all the time and he got me a appointment to a ent when i got there he said i had diseased mastiod and i had 2 cysts under the skin behind my ear he cut me scalp open and removed the cysts i never made it back cause i missed my appointment and he wanted 40 to reschedual and i started feeling better but latley my problems have retured and are worse for the last 2 yrs i just drained my ear and took antibiotics for most of those 2yrs now im misrable and feel hopeless my ent is on vacation and i have medical but cant get a appointment for 2weeks to get a referal to another ent i feel like im gonna end up dying i have a lot of stuff coming out my ear and ve been twisting my ears so it comes out and shaking my trying to swing out the fluid and it now feels like something is pulling from my throat like maybe mucas has come from my throat in my ear then trying to go out causing pulling i dont know im just misrable and my eyes feel like they are gonna pop out and my head feels like its drowning the er in my town isnt gonna help me i have no fever or they would but my vital sighns are fine i dont know what to do oh and i think i now have a bladder infection on top of it im so tierd of this ive been thinking about just killing myself cause im hopeless i wont do anything to myself i just feel like it please if you have any advice to help me id appreciate it cause my brain is thinking horrible things like i have cancer or brain tumor i even think i might b being poisoned by someone helpppppp Doctor: Do not worry, you do not have cancer from the symptoms you have described. You seem to have an infective condition of ear most probably chronic suppurative otitis media which responds to antibiotics. There os usually a perforation in ear drum which usually heals if it is small or if large can be corrected with surgery. Just take antibiotics, oral decongestants and antibiotic ear drops for few days and when your appointment date comes, consult your ENT. Take care."
},
{
"id": 217749,
"tgt": "What causes severe pain above the abdomen?",
"src": "Patient: Hi, this pradeep i am suffering from stomuch pain from 9-10 year. in initialy pain is right above abdomen but after 3-4 year it is at the novel position.but at the time it is again the above the right abdomen position. i have done the three time ultrasound and two time liver blood test one time urine test it is normal. i have also done the ct scan it have find the only apendix 12.3 mm diameter. and there are more gas to make in my stomach. i have consult the doctor one saying is pain from apendix and another doctor is saying pain in stomuch from alsar. i want to check the alsar . what i should do. please give the suggestion to me. Doctor: To check the ulcer u must meet a gastroenterologist and get sn upper GI endoscopy done. Till then avoid snoking, tea coffe and spicy food"
},
{
"id": 59790,
"tgt": "Elevated levels of MCV. Will it reduce after quitting alcohol and doing liver flush?",
"src": "Patient: I am a 45 year healthy female, non smoker and recently my blood test results showed a MCV of 105 (enlarged red blood cells) I have been drinking alot of red wine lately and I am wondering if that could be the cause? Also, if this is the case, how long for me my MCV count to get back to normal once I have ceased my alcohol consumption and doing a liver flush? Whatever information you can provide I really appreciate. Thank you. Doctor: It can take couple of months for MCV to get into normal range. It is really encouraging that you have decided to stop alcohol. I hope your doctor have prescribed Folic Acid and Vitamin B12 with other multivitamins."
},
{
"id": 74112,
"tgt": "Suggest remedy for difficulty in breathing",
"src": "Patient: Hi, may I answer your health queries right now ? Please type your query here...I was working in -35 temp and my breathing was rapid for extended periods of time. Since then my chest is wheezy and and I have the urge to cough. Is there anything I can do to relive this .... thanks.. Doctor: HiThis is a normal reflection of the organism due to the environment conditions. Try to avoid working in these conditions Dr.Jolanda"
},
{
"id": 179861,
"tgt": "Suggest treatment for throat pain,difficult swallowing and chest pain in a child",
"src": "Patient: I recently developed this habit of my that thing in the middle of my throat into like an S movement. Idk how it started or how long but I have stopped. But sometimes I do do it on accident. Ever since then I ve had pains in my throat and recently parts of my food don t go down and I can feel it kind of just sitting there or trying to come back up. I can feel and hear a clicking when I swallow. Most of my pain has recently been on my left side and I even have a chest pain kind of by my heart. I m 14. I m just getting kind of concerned because the symptoms have gotten worse. I sometimes get pains between my shoulder blades and the back of my neck. I m not sure if that s related though. Doctor: Thank you for choosing Healthcare Magic. When do you usually feel the difficulty in swallowing? Does this happen to both solids and liquids? Is there a specific time in a day when you feel this. Based on your description, it seems that you can have Gastroesophageal reflux disease. There are several food that can make your stomach produce more acid like spicy food, carbonated drinks, caffeine, chocolates. Try to stay away from these food. Stress can also contribute to the hyperacidity. But of course, it is still best to consult your Physician to rule out more serious problems. Hope you feel better.Sincerely, Hannalae Dulay-See, M.D.Pediatrician"
},
{
"id": 192379,
"tgt": "What does curd like substance under fore skin indicate?",
"src": "Patient: Its a white fine curd substance that i get under my fore skin and will reapeer in a day or two after i have a shower and some times has a strong odor --I read one time that this could be caused from too much sugar -I use about 4 spoons a day [coffe] --Thanks Doctor: Hello, It can be smegma. Which is nothing but a dead skin and mucus. It is a sign of poor personal hygiene. As of now you can clean the area with soap and water regularly. Generally it will settle by itself. If recurrent infection, circumcision may be required. Hope I have answered your query. Let me know if I can assist you further. Take care Regards, Dr. Shinas Hussain"
},
{
"id": 53412,
"tgt": "What causes flatulence building up in liver area?",
"src": "Patient: I am male, nearly 53, have had fatty liver for ten yrs or so, i drink regularly but not excessively, mostly wines, and eat healthily, salads, cereals, fruits, fish, very low meat and fats. Exercise at least 4ties a week, mostly swimming, live in warm climate. Have flatulence that feels like it is building up in liver area. What should I do? Doctor: Hi,For flatulence you can take a proton pump inhibitor like Pantoprazole. Try to walk daily for 30 minutes. Eat less fibres in diet, as fibres in diet increase flatulence. A normal diet should contain no more then 30 gram of fibres per day.Hope I have answered your query. Let me know if I can assist you further.Regards,Dr. Ramesh Kumar"
},
{
"id": 86108,
"tgt": "What causes abdominal pain and itching on the roof of the mouth?",
"src": "Patient: I have chronic intestinal pain. I have flare ups with the roof of my mouth covered in a rough, itchy, and sometimes painful film, increased abdominal pain, anal itching and a feeling of fatigue. Doctor: Hello and Welcome to \u2018Ask A Doctor\u2019 service. I have reviewed your query and here is my advice. The narrated symptoms indicate possible hormone imbalance (thyroid level or others) or nutrient deficiency of certain types with gut flora disturbances. I would recommend to get it evaluated with basic lab tests, supportive correction with necessary probiotics, deficient trace elements and supportive care. Thanks."
},
{
"id": 126047,
"tgt": "Suggest treatment for knee pain",
"src": "Patient: My wife is having pain in the back of her knee that is keeping her from being able to walk without severe pain. . .the only thing that helps make her mobile is flexeril but it makes her to sleepy to do her job. Metaxalone doesn t help at all. Any other suggestions to help her deal with this? Doctor: Hi, Most probably she will be having conditions like osteoarthritis or ligament related problems. Consult an orthopedician and get an MRI scan done. As of now, analgesics like Aceclofenac or Tramadol can be tried for pain relief. Hope I have answered your query. Let me know if I can assist you further. Regards, Dr. Shinas Hussain, General & Family Physician"
},
{
"id": 71271,
"tgt": "Suggest remedy for chest pain post TB treatment",
"src": "Patient: Hi, I had TB 7-8 years back and it was fully treated and eventually I got normal X- ray. I still feel pain in my chest when I take and hold long breaths. My doctor told me that its fibrosis and I ve nothing to worry. Is there any cure for this? Will this affect my chance of getting a work abroad (US, Australia)? I`m an Systems Engineer in an IT company in India and had good offers working aboard but I`m afraid that I might turn down for health reasons if they`ll find out my previous record. Please advise what would be the best thing to do? Doctor: Hello,First of all, no need to worry about this. TB lesions in lung heal by fibrosis. These fibrotic lesions are sometimes painful when there is a movement of pleura like deep breathing, laughing, sneezing etcetera. No treatment is required hence available for these scars. But for work visa purpose, you need to prove that these are old healed fibrotic scars and not active infections lesions. For this, you need to consult pulmonologist and get done CT thorax and bronchoscopy. If both these are negative for active infection then you can definitely opt for a work visa. Hope I have answered your query. Let me know if I can assist you further.Regards,Dr. Kaushal Bhavsar"
},
{
"id": 215710,
"tgt": "What causes severe leg pain at night?",
"src": "Patient: I am having severe pain in my right leg from hip to calf...Keeps me up at night. Have seen an orthopedic doctor 3 times with no improvement. Had cortisone shot in right hip, put on motrin, and muscle relaxers. Not a real big pill person. then say it MAY be bursitis but I have mentioned 3 times that I have lumps on my thigh that is sore to the touch as well a few on my side of my right leg and they never bothered to even look at them let alone touch them, My concern is that I fear a blood clot somewhere. Any advice Doctor: Hello, The location and movement of a pain give clues as to what is going on. and it's usually pretty obvious if it is the joint or the muscle or the butt or the hip joint. There can be irritation of the sciatic nerve with nerve type pain the goes from the back down the buttock and down the back of the leg. There can be muscle tone changes and this is the most common cause of nocturnal leg pain. Sometimes potassium and/or magnesium supplements may help but muscle relaxants or Parkinson's disease drugs are more commonly useful. Hope I have answered your query. Let me know if I can assist you further. Take care Regards, Dr Matt Wachsman, Addiction Medicine Specialist"
},
{
"id": 212898,
"tgt": "Anger, nausea, lack of sleep, excessive crying due to loss of job. Cure for poor mental health?",
"src": "Patient: Hello doctor, my name is XXXXXX, and something is going very boring and that bothers me, doctor I do not stop to pull hair and everything is more frequent after I left my job. Doctor can not just start crying my hair. I am very angry with the people can not take or hear a noise, I do not know what to do because I m also feeling a very strong feeling of nausea and sleep well. Doctor helps me, tells me what I have to do because I m desisperada. Doctor: Hello, Thanks for writing your query to us. The symptoms which you had mentioned suggest that you are in depression. Depression is a mental state characterized by a pessimistic sense of inadequacy and a despondent lack of activity. It is an emotional disorder, and nothing to be ashamed of. But often, people who are in depression are suffering from pessimistic thoughts about themselves. These pessimistic thoughts lead to even further depression making a cycle that gets hard to stop. In depression a person loses his energy to cope with a challenge. I would suggest that you should meet a psychologist who will be able to help you. They will listen to your problem & start counseling sessions. You are really stressed out & you need to take this initiative Take care."
},
{
"id": 213659,
"tgt": "What treatment can I take for ADD ?",
"src": "Patient: 28,male,minor depression treatment history,recently recieved A.D.D meds and cant commit to a doc in specific town caused by my job I m a business man that travels alot. Does healthcare magic docs prescribe adderall if not any suggestions? Doctor: hiwelcome to healthcare magic forum here we can give you advise related to your problem and medicine but prescribe any medicine is not possible addeall is amphetamine and has addictive propertyso dr can only prescribe you after examine you there are other medicine which are more safe and effective better to consult psychiatricsi hope i answered your question"
},
{
"id": 84156,
"tgt": "How long can we take Desval ER 250 - 2 tabs, Sizodon LS - 2 tabs?",
"src": "Patient: Respected Sir, I have been treated now with Desval ER 250 - 2 tabs, Sizodon LS - 2 tabs, and Lonazep .5 -1 tab a day, i have been taking mood stablizer for more than 3 years, now what i have to do and how severe my illness. when can i stop taking medicine. Doctor: Hello, These are psychotropic medications. The decision about stopping medications would be of your treating doctor. Otherwise there would be release of the symptoms. Hope I have answered your question. Let me know if I can assist you further. Regards, Dr. Shubham Mehta, Psychiatrist"
},
{
"id": 63271,
"tgt": "Suggest treatment for bumps on tongue",
"src": "Patient: hi! it has been a week and i have noticed some bumps on my tongue which are red! i felt my tongue sore but its a lillte bit better now. now there is a bitter taste in my mouth and my tongue is very white but the bumps are still there. am a smoker but i quited since i noticed the bumps...what should i do? thank u Doctor: Hi, dear I have gone through your question. I can understand your concern. You may have leukoplakia or some bump. It can be due to benign squamous hyperplasia or squamous cell carcinoma. You should go for biopsy of that area. It will give you exact diagnosis. Then you should take treatment accordingly. Hope I have answered your question, if you have doubt then I will be happy to answer. Thanks for using health care magic. Wish you a very good health."
},
{
"id": 196589,
"tgt": "What causes reddish marks below the rim of the penis?",
"src": "Patient: I am noticing redish/clearish but painless marks just below the rim of penis that seem to go all around the penis. Im also noticing the tiny red dots on the head of the penis, however i do remember this going away with steroid cream before. Im also noticing that my semen is a yellowish color. And sometimes i feel like im picking up a smell. I am a very worried person and am wondering what i should do Doctor: Hi,Reddish colored rim on penis might be related to allergic reaction, Infection ( like STD), insect bite or injury either during masturbation or tight undergarments. If possible please upload me a photograph and relevant information like your age, whether you are sexually active or not? In my opinion you can apply steroid ointment if you are suspecting a insect bite. Otherwise maintain good hygiene and apply some antibacterial or antiseptic ointment.Hope this answers your question. Please feel free to ask for follow up question, I will be gladly answer you.Best wishes,Dr. BR Hudda"
},
{
"id": 76571,
"tgt": "What causes pain in chest and upper back with itching?",
"src": "Patient: Hi there, i am a bit worried. My symptoms are the following. I started having pain in my chest and upper back, for more than 3 months, I went to our famliy doctor and he eventually diagnosed Tietze's Syndrome. After almost 4 months of pain ( now also my lower back, jaw, and headaches, he reffered me to a internist for further tests. What could this be. I am slo itching all over my body 24/7. Thanks Doctor: Hi thanks for contacting HCM...Tietz syndrome you have mentioned is swelling of costal cartilage....As your symptom of chest pain and upper back pain stable since 3 month cardiac cause less like....Here you might have muscle inflammation or myositis like condition...Sometime muscle tendon might tear and producing pain ...So history of trauma ruled out..For detail grading of muscle tear or tendon rupture CT scan helpful...Your detail history and examination needed and accordingly investigated ..If history suggestive connective tissue disorder then ruled out by molecular diagnosis.Take care.Dr.Parth"
},
{
"id": 126478,
"tgt": "What causes upper thigh pain while standing?",
"src": "Patient: For 4 days, I have had pain in my upper thigh near my groin, mostly when I walk or stand up after sitting. Late this afternoon after taking my usual 25-30 minute walk in the neighborhood, my left upper thigh near my hip started to hurt, and is throbbing now. I m thinking of calling my primary care dr. tomorrow. Otherwise, I am in excellent health and enjoy walking every day. Doctor: Hello, Most probably it is a musculoskeletal pain. We have to rule out other possibilities like neuropathic pain and pain due to poor blood supply.As a first management you can take analgesics like Acetaminophen or Diclofenac for pain relief.If symptoms persist, you can consult a physician and get evaluated. Hope I have answered your query. Let me know if I can assist you further. Regards, Dr. Shinas Hussain, General & Family Physician"
},
{
"id": 142393,
"tgt": "Why is this medication given for tingling sensation in thigh?",
"src": "Patient: My mother has been advised to take Shelcal-CT as she feels tingling sensation in her right thigh...Also she was diagnoised with Diabetes recently which was above normal limits but not by huge margin... Can you help with why Shechal was prescribed & will it be okk to take it.. Doctor: Hello!Welcome on Healthcaremagic!You should know that Shelcal-CT is a drug containing calcium and vitamin D supplements. It is really helpful in case of osteoporosis or vitamin D deficiency. But, I don't think that the tingling in her right thigh is related to such disorder. As she suffers from diabetes, it is very likely that her symptoms are caused by a peripheral nerve damage, probably a mononeuropathy or a pinched nerve. For this reason, I would recommend performing a nerve conduction study. If the diagnosis is confirmed, I would advice starting treatment with gabapentine or pregabaline, to help improve her situation. Hope you will find this answer helpful!Best wishes, Dr. Aida"
},
{
"id": 180750,
"tgt": "Is a white tissue in the middle of the tooth socket a concern?",
"src": "Patient: Had 3rd tooth from back on bottom left side removed Friday, because so much of the tooth had chipped off. It is now Wednesday, and I only have a little ache, but the clot is gone, and now there is something white in the middle. My stitches are doing good also. Should I worry about the white stuff in middle of socket? Doctor: Hello,If there is white stuff filling up the extraction socket, then it is probably the healing tissue that continues to remodel so that the extraction socket gets healed completely. So, if the extraction socket has white tissue, it is a good sign and as the pain has also been nominal now, this is an indication that the extraction site is healing well.So, my suggestion is to maintain a good oral hygiene and let the extraction site heal the way it is healing. Get the stitches removed after a week. Do not let food accumulate in the extraction site to avoid any chances of infection.Hope I have answered your query. Let me know if I can assist you further.Regards,Dr. Honey Arora"
},
{
"id": 113330,
"tgt": "Back and abdominal pain after eating certain fruits, vegetables. What's the problem ?",
"src": "Patient: After eating certain fruits and vegtables (raw carrots, corn on the cob, blueberries, etc) I sometimes have back pain (in the middle right wear my bra strap is ) and upper abdominal pain . It will go straight from my middle back to the middle front. I also have a gas build up and it causes me to burp. It is quite painful and will last for a few hours and gradually ease of . I have had this problem for about ten years. I get these attacks maybe two or three times a year and it always seem to be brought on by the foods I mentioned above. thank you for any suggestions as to what my problem Doctor: Hello. Thanks for writing to us. The pain in the back that you are having is likely to be related to gas and indigestion which gets precipitated by the particular foods. Most of the times these symptoms are self limiting and subside spontaneously. Taking peppermint oil will also be helpful. I hope this information has been both informative and helpful for you. You can consult me again directly through my profile URL http://bit.ly/Dr-Praveen-Tayal Regards, Dr. Praveen Tayal drtayal72@gmail.com"
},
{
"id": 15434,
"tgt": "Red bumps on inner thighs, no itching. Had hysterectomy and C-sections. Have chest infection",
"src": "Patient: I am a 31 year old female and I hope I have a simple heat rash , Both my inner thighs are really irritated red with little red bumps. The area burns, no itching whatsoever, I put a baby wipe on it thinking the cool would help and had mixed feelings because that burned some too but took a lot of the heat out of the area. At the moment I do have a chest infection. I had an abdominal hysterectomy September of 2011 and have struggled with the incision ever since. I had two c sections prior and had no problem whatsoever. It will be normal for a week or two then it is red, and not the entire incision mostly left side, becomes red and inflamed looking like a sun burn, patchy like a sun burn I guess is what I m trying to say. I have nystatin in powder form and I want to put that there to soak up moisture and I think it will help. I probably should also add that I have posture problems, scheuermann s disease where between my shoulder blades are hunching. In the past I have dealt with blackheads that my doctor said were from clothes rubbing more than likely because of the way I walk or that they don t fit properly. I appreciate any information you have to offer. Doctor: Thanks for posting the qwery.Your lesion seems like intertrigo I.e. Inflammation due to friction between opposing surface.Please investigate for routine blood test along with blood sugar levels. Keep the area dry and apply proper antimicrobial creams and antifungal preparations on those lesions after proper dermatological evaluation.RegardsDr Sandeep"
},
{
"id": 164463,
"tgt": "What causes cough with a runny nose?",
"src": "Patient: My 15month old baby suddenly developed a nasty cough sunday, and didn t sleep well of this cough. he cried after coughing. He almost always has a running nose. he did not indicate that his ears were sore and even his through is not red. He just have this awful cough. He is getting a new set of teeth. My husband took him to the doctor and among coughing syrup, antibiotics and something for fever he gave us Celestamine syrup. I had asthma as a child. Is it possible that my baby may have it too? Doctor: Hi...Thank you for consulting in Health Care magic. Greetings from Chennai.By what you quote I feel what your kid could be having viral associated wheeze or multi triggered wheeze. I have a few questions for you -Questions:1. How many days per month does she cough or feel breathless?2. How many nights per month does her sleep get disturbed due to above symptoms?3. Does she feel breathless when she runs around or plays with other kids?4. Are the symptoms when there are seasonal changes?5. Is there any family history of asthma or any other sort of allergies like skin allergy etc.?6. Is the cough always associated with fever?Please revert back to me with answers so that I can guide you better.You can approach me at the following link.Once the page opens there will be an option below my image as \u2013 ASK ME A QUESTION \u2013 click on it.Please find the link below -www.healthcaremagic.com/doctors/dr-sumanth-amperayani/67696Regards - Dr. Sumanth"
},
{
"id": 121163,
"tgt": "What causes knot in the arm which is growing in size and has a bruise?",
"src": "Patient: My son has a knot that started small on his left upper arm (the under side) the knot has grown in size and now has a large bruise around it. He plays football and has started weight lifting and the knot has become very painful. What could be the cause of the knot? Doctor: Hi, It could be hemangioma, get your MRI done to know exactly the cause of the swelling. Hope I have answered your question. Let me know if I can assist you further. Regards, Dr. Jaideep Gaver, Orthopedic Surgeon"
},
{
"id": 223003,
"tgt": "When is the due date according to the US report?",
"src": "Patient: My son girlfriend had an ultrsound on03/03/11. Based on the information listed below when would this baby have approx conceived: CAL 0.712cm CGA 6w4d EDD 10-23-2011 Please help us this very important...thank you Doctor: The report itself says that EDD (expected due date of delivery is 10-23-2011. What is your doubt then? They have already calculated and informed you the due date. :)I am sure I have solved your query."
},
{
"id": 69009,
"tgt": "What causes a lump under the skin?",
"src": "Patient: I have a painful, purple-colored lump that is under the skin and does not have a head as of yet. I first noticed it 5 days ago as a sore red bump, but now it looks more purple in color and is sore to the touch. I have heard of MRSA and Staph, but do not know exactly how they present. Any info is appreciated. Doctor: Hello!Thank you for the query.Please do not think about MRSA. This kind of infection is usually caused by long stay at a hospital. Healthy people or never hospitalized people usually never have any MRSA problem.Of this lump is on the leg, varicose veins is the most probable reason. If it hurts, blood clot can be inside.If it is on the other location, sebaceus cyst which has gotten infected can be the reason.I suggest you to consult your doctor with this issue.Hope this will help.Regards."
},
{
"id": 189480,
"tgt": "Infant had fever, blisters on lips, teeth swollen, gums infected. What it is?",
"src": "Patient: my baby is 14 months she had a fever and woke up with what looked to be fever blisters on her top lip when i looked in her mouth her top two teeth looked very swollen with yellow pus ;( she also has spots on her tongue now today her bottom teeth and gums look infected and swollen can u help me i just want to know what it is Doctor: Hi, Thanks for asking the query, Fever along with blisters needs to be evaluated, a basic blood count will help. For fever you can give paracetomol in correct dosages, please take your baby to the Pediatrician. Do not brush the teeth till the blister heals as it may worsen the condition. Take your child to the Pedodontist and get the thorough clinical evaluation done. Swollen gums along with pus formation can be due to improper oral hygiene, scurvy, bacterial and fungal infections. It can be a serious sepsis. If your child is taking some antibiotics for fever it can be due to reaction to antibiotics and other drugs leading to ulcers on lips, gums and tongue known as steven johnson syndrome. Make sure that your child drinks plenty of drinks and water, milk, fruit juices. Hope this helps out. Wishing your child a healthy recovery! Regards..."
},
{
"id": 23767,
"tgt": "Is it normal to have reflux and coughs in 5 months old after a surgery for floppy larynx?",
"src": "Patient: Hello, My 5 month old had DS and a brain abnormality called HPE and she has a heart defect TOF. She just had surgery to correct her floppy larynx. Dr said IF she aspirated before she might afterwards....I m not seeing aspiration, we are seeing reflux...again! We can actually see her struggle and watch from her body language that it s rising up. She coughs and struggles. Is this normal and drs said it will go away within 2 weeks. We are so nurses. What do u think Doctor: Yes its normal...it may take longer than 2 weeks to go away...but it will disappear at the end , you can check with her doctor to give her something to relieve the cough"
},
{
"id": 194361,
"tgt": "Suggest treatment for testicular pain",
"src": "Patient: Hello i am now having pain in my left testicul for abour 2 months it is heating higher each time and i feal the same thing in my penis i almost past out 3time by now the pain cause up to my stomic and belly please tell me wath to do i went 4 time to the doctor i got on medication 2time for infection but its stegesal not working Doctor: Hello, It can be due to: 1.\u00a0\u00a0\u00a0\u00a0\u00a0Epididymitis \u2013 is Inflammation of the tube, back of the testicle that carries sperm. Due to infection (both sexually and non sexually transmitted ).diagnosed by urine Complete, Ultrasound pelvis. 2.\u00a0\u00a0\u00a0\u00a0\u00a0Hydrocele - can cause swelling 3.\u00a0\u00a0\u00a0\u00a0\u00a0Hernia \u2013 can also cause swelling but it will be an off 4.\u00a0\u00a0\u00a0\u00a0\u00a0Kidney stones can cause abdominal pain too 5.\u00a0\u00a0\u00a0\u00a0\u00a0Infections \u2013 can cause fever, discharge from the penis 6.\u00a0\u00a0\u00a0\u00a0\u00a0Testicular torsion- 7.\u00a0\u00a0\u00a0\u00a0\u00a0Epididymal Hypertension (EH) \u2013 Also called as BLUE BALLS, its caused due to long sex without ejaculation (delayed Orgasm). Letting out the sperm by masturbation, oral sex or regular sex will ease out your pain. 8.\u00a0\u00a0\u00a0\u00a0\u00a0Urinary tract infection (UTI) 9.\u00a0\u00a0\u00a0\u00a0\u00a0Any previous surgical procedure 10.\u00a0\u00a0\u00a0\u00a0\u00a0Diabetic neuropathy 11.\u00a0\u00a0\u00a0\u00a0\u00a0Benign or malignant lesion. If you haven't done an ultrasound scrotum, need to do and also ultrasound pelvis and prostate examination. Hope I have answered your query. Let me know if I can assist you further. Take care Regards, Dr S.R.Raveendran, Sexologist"
},
{
"id": 73302,
"tgt": "What causes persistent cough?",
"src": "Patient: my son who is 25 had been having a persistent cough since he left his trip to the Philippines 3 weeks ago it was actually beginning to hurt me when he cough, but he said only his throat hurt a little from coughing so much. I decided to offer him the Bromphenir-pseudoephed DM the Doctor had prescribed for my granddaughter in July this year even though she is 12 Years old it seem to work great but now we are out and we need to know what over-the=counter is similar Thanks Doctor: Respected user , HiWarm welcome to Healthcaremagic.comI have evaluated your query thoroughly .* The probable reason is allergic bronchitis .* Suggestions for better recovery- Gargles with salted lukewarm water added peppermint oil 3 times a day .- OTC molecules as tessalon pearls and mucinix od can help .- Avoid exposure to pollen , dust particles .- Refrain from smoking , alcohol .Hope this will help you for sure .Welcome for any further guidance .Regards dear take care ."
},
{
"id": 207270,
"tgt": "Suggest treatment for schizophrenia",
"src": "Patient: Is there any treatment to cure scziphrenia? Where can I avail good treatment? At present I am using Risdone-2 mg but it is hiding the problem temporarily. Is there any method of meditation or hypnotism to remove the disease entirely? I am great need of help. Please advice. Doctor: DearWe understand your concerns.I went through your details. I suggest you to keep calm. Schizophrenia are of various types. Early detection and proper diagnosis and medication often provides good results in the management and cure. From your description, the details are not clear. There are no other scientific treatment methods to cure schizophrenia other than psychitry and psychology. Hypnotism is a strict NO. Hypnotherapy and Cognitive Behavior Therapy should be of help to you. Do work with your psychiatrist. Trust him.In case if you need more of my services in this regard, please post a direct query. I am happy to reply you.Hope this answers your query. Available for further clarifications.Good luck."
},
{
"id": 112004,
"tgt": "How to treat severe lower back pain? No relief with heating, ice pack and soothanol",
"src": "Patient: Horrible low back pain that won't let up. 5th day. It is centered above my buttocks. Dead center and nothing helps it. I've done heat, ice, bio freeze, soothanol x2, flex aril, 10 mg vicodin x2 at a time, soma? I have tried it all. It is horrendous. Scale of 1-10 a definite 9 Doctor: HiDont worry avoid sitting on hard surface.Have complete bed rest, followed by ice application and lumbar traction. Along with this if you are not a diabetic patient then short course of steroid in tapering dose will surely help you. Avoid weight lifting and forward bending.Thank youGet well soon."
},
{
"id": 12378,
"tgt": "How to treat psoriasis?",
"src": "Patient: My mother is suffering from psoriasis from last 4months i have taken her to doctor after a long treatment from specialist their is no positive come back. I am looking for specialist or u can suggest me medicine. my Email id is YYYY@YYYY & my cell number is 0000. Awaiting for reply Doctor: Hi,As you said your mother suffers from psoriasis...I would like to say few things about psoriasis... . Psoriasis is a chronic relapsing and autoimmune disease. You may be having scaly plaques on various parts of the body,scaly lesions on the scalp. Exact cause is not known. Genetic tendency, autoimmunitity,stress,dry wheather...etc may be responsible for precipitation and exacerbation of psoriasis. You consult dermatologist for firm diagnosis and treatment.I usually recommend cap acitretin 10 mg twice a day along with methotraxate 15 mg weekly in three devided doses 12 hourly...for psoriasis.This may be taken for long time till good response. Blood tests may be done to monitor the treatment. For itching, you may take antihistamnics like levocetirizin...Avoid steroid ,as there may be severe flare up after stopping it.She may apply mild steroid cream or calcipotriol oint on the lesions on the psoriatic lesions. She may apply moisturiser to improve the skin texture. She should avoid soap bath for few weeks..And she should be free of stress and worries as that might worsen psoriasis. She might get good result.I hope you got my answer.Thanks.Dr. Ilyas Patel MD"
},
{
"id": 26906,
"tgt": "Can I take amlogard and veltam for hypertension?",
"src": "Patient: I am a patiant of hypertension for the last17 years. I have a problem of prostrate glands also. My doctor has prescribed me following medicines : ( I am 72 years old-Male) 1) Amlogard 5mg (2) Telsarten 80 active (3) Tab Atocor 10 mg (4) Tab Tab Veltam 0.4 mg Pl advise me if the medicines prescribed are in order. My Lipid test results are within range. V G Joshi Doctor: Hello Mr Joshi. Your medicine are perfectly in order. Amlogard and telmisartan are for your blood pressure, and veltum is for prostate. Though veltum is known to cause decrease in blood pressure in general population it is nit clinically significant. If your lipids are in order and no other risk factors you may decrease atacor dose and completely move over it. Rest wishing you the best of Health. Regards Dr Priyank Mody"
},
{
"id": 48721,
"tgt": "What does \"obstruction from distal ureteral lithiasis and nephrolithiasis\" in ultrasound suggest?",
"src": "Patient: hi, my ultrasound result says: mild fullness of the left pelvocalyceal structures, consider obstruction from distal ureteral lithiasis vs atony from infection or recent passage of stone non-obstructing nephrolithiasis, right sediments/crystals, left kidney Doctor: Hi,Thanks for writing in.The ultrasound report means that the collecting system of the kidney on left side is swelled up mildly. This can happen due to two main causes.1. There is a stone in the lower end of left ureter (tube connecting kidney to bladder). When this happens there is obstruction to flow of urine and the kidney swells up. There is severe pain in lower abdomen.2. Another possibility is due to infection in the ureter or a recently stone which was initially stuck in the proximal ureter and has subsequently been washed out. The stone in this case can scratch the ureter and cause inflammation leading to swelling of the kidney."
},
{
"id": 167520,
"tgt": "What causes painful and swollen nipple in a child?",
"src": "Patient: Hi, may I answer your health queries right now ? Please type your query here...my 9 year old daughter s nipples are swollen and she says they hurt her. I noticed them about 2 days ago, but thought it was normal. I did not know they were hurting her. what could this be? Doctor: hi! this could be a normal part of puberty since breast development is one of the first signs of puberty. although if you notice any redness around the nipple this could be an infection for which you should see a doctor."
},
{
"id": 3713,
"tgt": "Does the delayed periods mean pregnancy?",
"src": "Patient: Hi am a girl of 24 years.am using pertogyn, then i discovered that I had discharge, went to doctor she gave me pills(flegyl) after be on periods immidiatly for 5 days. Then after I hade an unprotected sex then I suppose to go to my next appointment on the18/05/2015 I didn't do that continuing doing sex.so now since then I don't go to my period.Does that mean am pregnant? Doctor: HI, A normally menstruating lady in fertile age.. having un protected sex & getting delayed periods.. the first diagnosis is pregnancy. Yes, you have possibility of pregnancy. But there are many other factors which cause delay in periods like- Hormonal imbalance due to psychological stress/ illness & medication/ thyroid or ovarian problems. The pregnancy can be confirmed by blood HCG test ( as urine test may have false result due to infection you had )/ USG scan of pelvis ( after 5 weeks ) Thanks."
},
{
"id": 63774,
"tgt": "What causes lumps on scalp with a head cold?",
"src": "Patient: I m a 64 year old male suffering from a head cold for the past 3 days. I showered this afternoon and noticed several lumps under my scalp, each 3/4 to 1 in diameter (the shapes varried and aren t symetrical). Should I be concerned? I ve had sinus problems for years and had emergency surgery on one side while serving in the military as a pilot. Doctor: Hi, dearI have gone through your question. I can understand your concern. You may have some enlarged lymphnodes due to reactive hyperplasia or some cyst or skin adenexal tumor. You should take a course of antibiotics. If it doesn't respond to treatment then biopsy diagnosis is advisable. Consult your doctor and take treatment accordingly. Hope I have answered your question, if you have doubt then I will be happy to answer. Thanks for using health care magic. Wish you a very good health."
},
{
"id": 178008,
"tgt": "Are red spots on body and jaundice in a newborn after being in incubator concerning?",
"src": "Patient: my baby is 12 days old. His got jaundice was in the incubator for 12 hours then from 300 went to 258 we were sent home and was advised to take baby to the sun. baby eat well do wee and poo regularly. but jaundice still there and there are red spots in his body is there something that i need to worry about his condition? Your advise is much appreciated Doctor: HI...the jaundice will take at least another 10-14 days to settle completely and the red spots you are seeing could be phototherapy rash and you need not worry if the baby is feeding well and passing urine and motion well. An active baby is a well baby.Regards - Dr. Sumanth"
},
{
"id": 84260,
"tgt": "Is there any side effects for OLMEZEST H 20?",
"src": "Patient: Sir, I am dibetic patient with good control for the last five years and I am hypertensive also. Doctor advised me to to take metapolol50mg daily one. But the BP remains to be 140/97 now doctor advised me to OLMEZEST H 20 in addition to metaprolol. Will it have any side efffects in long run Doctor: Hi,Yes, it can cause certain side effects. Olmezest-H 20 is a combination medicine (olmesartan + hydrochlorothiazide) commonly prescribed for the treatment of high blood pressure. Generally, it is well tolerated by most of our patients however it may cause dry cough, low blood pressure, and swelling of the lips / eyes / feet.Hope I have answered your question. Let me know if I can assist you further. Regards, Dr. Mohammed Taher Ali, General & Family Physician"
},
{
"id": 215373,
"tgt": "What causes pain under the ribcage?",
"src": "Patient: I get a pain under my ribcage, in the middle, every few weeks that lasts from anywhere between 20 mins to 12 hours. It sometimes makes me vomit. My doctors don t know what is wrong with me. Have you any ideas? I m 23, 5ft and I think I m 13 stone. Doctor: Hello, As per your history, it may be due to muscle sprain. For that you can take tablet acetaminophen. Apply diclofenac gel or ointment locally. Heating pads will also help. Restrict your movements. In case of breathing difficulties you may require chest X Ray after chest physician consultation. Hope I have answered your query. Let me know if I can assist you further. Regards, Dr. Shyam B. Kale, General & Family Physician"
},
{
"id": 214137,
"tgt": "Severe Social anxiety",
"src": "Patient: Hi I suffer from severe social anxiety. I have read that CBT is helpful. Please advice me on how to go about it and whom to contact. I live in bangalore. Doctor: Hi.. CBT is done at any rehabilitation centres.. NIMHANS would be the best place for this. But it would take some time to get an appointment there as it is a central institute.. You may contact a clinical psychologist near your locality and check with him.. If you do not have one then you may search for one near your place.. I shall give you a link in\u00a0 helping you do this.. http://www.netexpress.co.in/yellowpages/display/Doctors/Psychologist/Karnataka/Bangalore/1/display.aspx Hope you find it useful"
},
{
"id": 203193,
"tgt": "What could cause stinging and inflammed scrotum with skin turning bright red?",
"src": "Patient: Hi, I am a 70 year old male and for several months now have been suffering from stinging and inflamed scrotum which at times the skin turns a bright red colour, it is most uncomfortable. I would appreciate any advise.Many thanksI wish to withdraw my question as I thought it was for free.i can't afford the costs, many thanks for your time. Doctor: Hi and thank you so much for this queryI am so sorry to hear about this inflamed scrotum. The symptoms suggest and infection. However, there are several different germ types that can cause this infection like fungal, bacteria, parasites, etc. It would be a great idea to get it examined so as to identify the exact cause and treat for it. Given a generic treatment would most likely not be helpful because we would not know what to target.I hope this addresses your query fully. Thank you so much for patronizing our services and please do feel free to ask for follow up clarifications and information if need be. I wish you the best of health.Dr. Ditah, MD."
},
{
"id": 107649,
"tgt": "Experiencing back pain on the right side of my back",
"src": "Patient: back pain on right side of back in kidney area, age 78, COPD, on oxygen ,3 back surgeries 15 years ago, had staff infection from surgery. on IV for 6months. Saw doctor last Thursday for a follow up after blood draws and general check up. On aricept . Doctor: U might suffer from spondylosis or spondylolisthesis or spondylolysis. Better take one dynamic flexion-extension view of LS spine. U got multiple comorbidities. Better to get treated conservatively"
},
{
"id": 674,
"tgt": "What causes a missed period with stomach cramps?",
"src": "Patient: my first day of last period was 5th june 2010 and did an IUI on the 19th and 22nd of june.i still didnt get my period for this month and 41 days late uptoday.sometimes i have cramps and vomitish feeling.leg pains and back pain also.sometimes i feel like going to have periods. And in april also my period late for 41 days.am i pregnant. Doctor: yes chances are there for your pregnancy so you have to go for your pregnancy test to confirm it.=========================="
},
{
"id": 14308,
"tgt": "What causes itchy rash on the arm?",
"src": "Patient: About a month ago I developed a small rash in the antecubital region of my left arm. I applied some Aquaphor for a few days and it went away. Well, yesterday it reappeared. I have kept Nystatin on it and it seems to be going away. It is only slightly itchy but the first day was really red. What is this? Doctor: Hi! Thank you for your question!As you mention nystatin, that is an antifungal agent, and your rash seems to disappear it could also be a dermatophytosis (fungal infection). I would recommend you continue with Nystatin txice a day for at least 2 weeks, and also a week after the lesion disappears.It could also be a dermatitis, and that`s why it went away with Aquaphor. Sometimes this two conditions can present together, that`s why I also recommend you use a topical OTC corticosteroid once a day for 1-2 weeks. you can also mix the two medications together. You can continue with Aquaphor, several times a day to hydrate the skin.I hope you get better. Please feel free to contact me again if anything is unclear.Dr. Migena GEGA"
},
{
"id": 118922,
"tgt": "Suffering from hemorrhage, suspecting to be pregnant. Tried home remedies. Treatment?",
"src": "Patient: Hai I have haemorrage for the past 40 days.Now I have a doubt that I'm pregnant .Do this affect my pregnancy?Is this case severe or extreme piles.? Do it require any surgery? I've tried many remedies in home.Drinking water,taking fruits and vegetables,using stool softner and also applying ointment.I,m better now but still I've light bleeding .What should I do?Please answer me. Doctor: Hello,Thanks for posting your query.During pregnancy there are increased chances of constipation that aggravates haemorrhoids.So now you should be much more careful by increasing water intake, fruits & vegetables in your diet.Get repeatedly investigated for haemoglobin if it keeps on decreasing even after taking haematinics then you would have to go for surgery.Take care."
},
{
"id": 201788,
"tgt": "Suggest treatment for fungal infection on penis",
"src": "Patient: I have a fungle infection on penis for two years I have use many english medicine but jab tak unka use karta hua tab tak aaram lagta hai quit the medicine same reasult so please tell is any medicine is useful in amwayplease tell me name and how take it Doctor: , fungal infection in the penis has to be continued for 3-4 months for compleate recovery if you stop the treatment inbetween it will relapse itr again i advice to clean the genital area nicely before anti fungal application and to change your undergarments every dayhope these things will give better resultshope this explainwith regardsDr.Amarnath"
},
{
"id": 94606,
"tgt": "Dull abdominal pain on the lower right side, no pain on pressing, good appetite. Cure?",
"src": "Patient: I have a slight abdomen pain very dull on a scale of 1 to 10 may be a 2. I think i lifted something 2 days ago..and it happens to be on the lower right side of the abdomen may be once like every 5 minutes...I can jump and run without any problem....I have good appetite ...I can press all the way down on the lower side without any problem wondering whether it is appendicitis or a cramp Doctor: Dear sir/madam, it definitely doesnt sounds like appendicitis, you would have constant pain in DLQ increasing on pressure, also associated with fever,nausea and vomiting. So it can be cramp or u simply injured some of abdominal muscles. You shouldnt be worried except this pain persists and become worse. If you want to be sure you can do abdominal ultrasound but i dont think its necessary. Wish you good health.Ivan Romich"
},
{
"id": 107399,
"tgt": "What causes weakness in the lower left side and legs?",
"src": "Patient: I ve been experiencing a weakness on my lower left side, legs, after having lower left side back pain, it moved to my left leg. That s been going on for 4 days, no other symptoms. Today, I got very numb and started sweating and felt very out of it. Any ideas? Doctor: Hi i am Dr Ahmed Aly thanks for using healthcaremagic site ,I had gone through your question and understand your concerns .. That could be a muscular spasm or a kind of nerve compression due to heavy lifting , vigorous movements , trauma or injury , bad sitting or sleeping postures . These given symptoms may show some sciatica due to neurovascular inflammation which may subsides simultaneously within days or extend to chronic pain if un treated properly . For now i may suggest advil tab when needed , hot massages with topical gels , muscle relaxants , B12 vitamin supplementation will be effective for mild cases else i recommend MRI on spine for proper evaluation and management .Please click and consider a 5 star rating with some positive feedback if the information was helpful. Wish you good health,Any further clarifications feel free to ask"
},
{
"id": 41293,
"tgt": "What does the semen analysis test indicate?",
"src": "Patient: hi, i'm looking at some results and i want to get some answers. its a semen analysis test. what is that kinetic motilit characterized as fair ? also, appearence opaque? turbidity 2? seminal epithel as none? ph-8? is anything of this good or bad? Doctor: HelloLimited information with your query regarding semen analysis appears normal.pH of semen is normal.Motility of sperms are reported as fair,it means it is normal.Other findings like sperm count,morphology etc are not known.These are important parameters for semen analysis.So,findings are good.Get well soon.Take CareDr.Indu Bhushan"
},
{
"id": 162301,
"tgt": "Suggest remedy for blocked nose with dry cough",
"src": "Patient: Hi, My daughter is 3 yrs 5 months old. she had adenoitis when she was 3 and was treated with anibiotics. However, since then, she is constantly having blocked nose, cough which at times is dry and other times productive. She was again suffering from blocked nose and snoring for the last 3 days so I gave her flomist spray. one puff in each nostril last night, which really helped to relieve the nose. For how many days can I give her the spray? Also is it safe? Doctor: Hello, A blocked nose is most likely due to the nasal congestion of a cold. For this, you can put 1-2 drops of normal saline (Over the counter at the pharmacy) into each nostril while she is lying down. This will stimulate her to sneeze and then you can suction out the nasal discharge with nasal bulb syringe. This can be done as often as needed. I believe Flomist is for hay fever(nasal congestion, sneezing due to allergy). You may use this for 5-7days. Twice a day. It will be safe like this. Hope I have answered your query. Let me know if I can assist you further. Take care Regards, Dr Arnold Zedd, Pediatrician"
},
{
"id": 78360,
"tgt": "How long will it take to clear fluid from lungs?",
"src": "Patient: My 69 year old Mother was just discharged from the hospital after spending 5 days there for pneumonia. She has no spleen and was admitted Sunday because of fluid in her lung and was almost septic Monday. How long should she wait before getting on an airplane? Doctor: Thanks for your question on Health Care Magic. I can understand your concern. It is advisable to take at least 2-3 weeks of rest after pneumonia. She is also having pleural effusion (fluid in lung) and septic, so possibility of severe pneumonia is more. In this situation, it takes almost 2-3 weeks for complete resolution. Air travel in plane is not safe because of low oxygen saturation at that height. So possibility of breathing difficulty is high in pneumonia patient in the plane. So better to take proper rest at home for 2-2 weeks atleast. Get done repeat chest x ray and clinical evaluation after 2-3 weeks. Hope I have solved your query. Wishing good health to your mother. Thanks."
},
{
"id": 147525,
"tgt": "Completed epidural shots for spine, pain. Taken betamethasone dipropionate and kenalog. Are many procedures required?",
"src": "Patient: I just completed four epidural shots for my spine between the L4 & L5. the doctor used kenalog. I am still in some pain . Two of my friends had the same procedure done one time to the L4 & L5 using betamethasone dipropionate. They have been without pain for five to six months. Why did my doctor do this? I have also read that kenalog is only 1/5 to 1/6 the steroid potency of betamethasone/ dipropionate. I am thinking it was just the fact that he does these four procedures instead of one just to make more money. I would love to hear your opinion Doctor: HIThank for asking to HCMIn my opinion such invasive therapy some time causes danger out come because this could be just symptomatic treatment and not curative if this is true then why to go for such invasive procedure when the same result one can get with the oral drugs, better to avoid such therapy and have opinion form good spine surgeon, hope this information helps you have a nice day."
},
{
"id": 186418,
"tgt": "Suggest ways to get braces without much expenditure",
"src": "Patient: [Brenda] Im wil be 21 in a few days therefore my medical won't work for dental I am in desperate need of braces but I have no job and no support from parents I've been very insecure and hate my smile Im just hoping for a change is there anything you may do to help me? Doctor: Hello, Welcome Thanks for consulting HCM,I have gone through your query, as you have less money cant afford braces then you can consult dentist and go for atleast Appliance for your treatment if possible . Hope this will help you."
},
{
"id": 167827,
"tgt": "Suggest treatment for problem in eye of baby after premature birth",
"src": "Patient: Dears, I have problem with my kids. They are twins, I gave a birth when was on 29th week of pregnancy. Long time we, meaning myself and my kids were staying in hospital for rehabilitation. They wre under oxygen more than month, Now both of them have problem in one eye, it doesn\u2019t\\\\\\ t see at all, mikroftalm, fibroglioz in inside, doubts on partial detachment of retina, and no chance to see as I was said. Can u pls advise if there is any chance for them? or can you treat this? I very much hope to receive yes, but if it is not can you refer me somewhere where it is treatable. I got already NO from dunyagioz. Thanks a lot in advance, Bahar Doctor: this condition is called retinopathy of prematurity , and it's risk increases in cases where this high demand of oxygen therapy , more than 21% needed to maintain saturation above 92% .it's very helpful to identify the problem early with regular examination by the ophthalmologist. some cases require early Lazer therapy to prevent retinal detachment and permanent blindness. I recommend seeking other opinion of an expert pediatric ophthalmologist ."
},
{
"id": 207385,
"tgt": "Suggest remedy for sleep masterbation",
"src": "Patient: In the last few months I ve been woken from my sleep by my husband masterbating in his sleep. I have talked to him about it and he is totally unaware that he s doing it. He says that he hasn t been having any errotic dreams and we are both baffled as to why this is happening. He s now really conscious about it and is worried about going to sleep incase it happens again. What is causing this to happen? We ve been very happily married with a very healthy sex life for 18 years. Doctor: DearWe understand your concernsI went through your details. I suggest you not to worry much. Such incidents are very rare. You have not given much of the details here. How many times this happened? what type of masturbation it was? Was he using his hand all the time or was lying facing down and moving his pelvic area?When nocturnal emission happen, males do show some movements of pelvic area. If that is the case, nothing to worry and is normal. If that is not the case, please be descriptive and you may approach a psychologist.If you require more of my help in this aspect, Please post a direct question to me in this URL. http://goo.gl/aYW2pR. Make sure that you include every minute details possible. I shall prescribe the needed psychotherapy techniques.Hope this answers your query. Available for further clarifications.Good luck."
},
{
"id": 181745,
"tgt": "Suggest treatment for tooth abscess",
"src": "Patient: I am showing symptoms of have an abscess and so I am visiting the dentist as soon as possible. If my tooth needs to be removed, would I be able to be put to sleep? I have a massive phobia of needles and drills so I don't want to turn to root canal treatment. Also, if my tooth is removed, will I be able to get braces to close the gaps? I have had braces before and the abscess has made me unable to wear my retainer therefore my teeth have moved back to their origanally place. I am a child so I am not sure Doctor: Thank for your query. I have gone through your query. As far as the abscesses tooth is concerned, it is better to save the tooth with root canal treatment rather than extraction. Because once you get the tooth removed it's very difficult to close the gap with braces. Nothing to worry about injections and drilling, it's a simple procedure. Get it done. I hope my answer will help you. Take care"
},
{
"id": 2306,
"tgt": "Could intake of aspirin cause difficulty in conceiving?",
"src": "Patient: HI, I am 30 years old and my husband is the same age. We have been trying to conceive for almost two years. I had a missed miscarriage last year in September and had a septum removal (disovered during the D&C) in February. Since March we are trying to conceive but nothing happens. I also take low dose aspirin every day prescribed by a doctor as they have discovered that my blood clots a bit more than usual. My husband is alergic to aspirin and now I am worried that maybe that might stop us from conceiving. I also have very irregular periods, but the doctors keep telling me that since I have conceived once it will happen again. So what test should I do to get pregnant faster and should I continue taking the aspirin. Thank You Doctor: Hallow Dear, Low dose aspirin will not cause infertility. You are having irregular menses; so it is most likely that you may not be ovulating regularly or at all. Please submit yourself for ultrasonography for ovulation monitoring. If you are not ovulating, you may need some treatment for ovulation induction. If you are ovulating, then try to plan your sexual relations on and around the day of ovulation. Also get your tubal patency and uterine cavity tested by Hysterosalpingography and Laparoscopy. Any block and/or adhesions may require surgical intervention. Husband's sperm quantity and quality should be assessed by semen examination. Please report to an infertility consultant. Dr. Nishikant Shrotri"
},
{
"id": 120401,
"tgt": "Suggest remedy for thoracic scoliosis",
"src": "Patient: my son is 22 yrs old and is suffering from thoraic scoliosis convex on the left side with cobb s angle of 18 degrees and lumbabdegrees. no bone destruction, disc space narrowing or paraspinal ossfications. the gravity line is anterior to S1. what should be the medication to this, can the curves be corrected to normal position? my husband is also suffering from cervical osteophytes, he s treated with muscles relaxants only as no serious bad condition to suffer from according to doctors. which is which? Doctor: Hello,A 18 degree cobs curve is not a matter of worry in scoliosis at age of 22. In our routine OPD we advise physiotherapy in such cases to strengthen the back muscles. A scoliosis of more then 20 -40 degree need surgical interventions. At present physiotherapy and electro-therapy are sufficient for your son as a treatment. Hope I have answered your question. Let me know if I can assist you further. Regards, Dr. Mukesh Tiwari, Orthopedic Surgeon"
},
{
"id": 159403,
"tgt": "In remission for clear cell ovarian cancer, done all treatments, have blood in stools. Anal cancer or hemorrhoids?",
"src": "Patient: i am a 44 yr old female who is in remission for clear cell ovarian cancer stage 2c. it had spread to the rectum . i have had had all the treatments done. now i have blood in the stool that started just before christmas. i thought it was probably hemmoroids. but now i have had blood in my stools for over three days now. whats the difference between hemmoroids and cancer of the anal maybe reoccuring. i have no pain or itching or straining when i have a bowel movement . thanks Doctor: Hello, Welcome to HCM, I am Dr. Das Look, from your description, it is clear that you are suffering from clear cell carcinoma of ovary which has metastasis in anal region. So, it is obvious that you may experience bleeding. Metastasis means a cancer occurring in that particular region that is similar with that of its ovarian counterpart. So, you should be operated for the metastasis to prevent further disease progression. Regards."
},
{
"id": 45669,
"tgt": "What causes tenderness in the lower abdomen?",
"src": "Patient: I had kidney stones a couple years ago and yesterday I started have a lot of pain on my right side both in the front and back - the back it seems to be about the waist level midway from the spine to my side - the front seems to be again about the waist area but maybe a little lower - both areas are even tender to touch and hurts when I stand up and sit down - it really hurts to bend over as well - could this beginning kidney stones again? Doctor: Hi, As you say it indicates there is a possibility of kidney stones and pain from which radiates to the backside. Let me tell you a few questions: Do you feel nausea, fever? Is there any pain in the thigh region or testes? Is your bowel ok? Is there any problem in urination? Let me know so that you can be precisely guided well. I hope you will update in feedback. You can also get an appointment with a physician where examination can be carried properly and accordingly cause can be precisely identified and managed. Hope I have answered your query. Let me know if I can assist you further. Regards, Dr. Soheel Hussain Zargar, Dentist"
},
{
"id": 63845,
"tgt": "Suggest treatment for bump on shoulder",
"src": "Patient: I have had a bump on my shoulder right by my neck for several moths now. It started out the size of a bb and I thought it was a pimple. Didn't give it much thought till today because it hurts and is about as big as a pea. It also looks to have like a blister head in the middle of it. Should I be seaking medical attention? Doctor: YES you should seek doctors check.as the lump has grown and is soar also with blister head. Else it would complicate moreHit thanks. Gd day."
},
{
"id": 216212,
"tgt": "Can taking Tramadol and Oxycontin for MVA cause nausea?",
"src": "Patient: I was prescribed Tramadol for pain related to an MVA over 5 years ago. I only take it PRN and occasionally will take an Oxycontin, but only with significant pain. I have noticed since I started using these I will have bouts of nausea even when I am not taking the drugs. Since I use these perhaps once/week or less, can the nausea still be related to the drugs? Thank you for your help! Doctor: Hello and Welcome to \u2018Ask A Doctor\u2019 service.I have reviewed your query and here is my advice.Tramadol and oxycontin are both opioids and nausea and vomiting is one of the common side effects of both drugs. Tablet domperione 10 mg needed to be taken half an hour before taking tramadol to prevent nausea and vomiting.Hope I have answered your query. Let me know if I can assist you further.Regards,Dr. Jayesh Vaza"
},
{
"id": 26424,
"tgt": "How to reduce BP?",
"src": "Patient: hi my BP is 170/120 as of now doctor advised telista AM everyday morning (one daily) permanant am 43 yrs old with ht 5'9\" wt 94kg. normal eating habits vegetarian doesnt smoke. How do i reduce my BP No other major illness no diabetes so far.Eats well,sleeps for 9 hrs at night. Thnks Doctor: Hello, as your blood pressure is very high, it would not be controlled with lifestyle changes alone, hence you need to start on medication at present, one can slowly consider decreasing dose after a good control is a achieved. Meanwhile you should start Exercise 30-45 mins a day atleast 5 times a week Decrease salt in diet to less than 2-3 gms a day Consider losing weight and try achieving ideal weight of around 80. Regards Dr Priyank Mody"
},
{
"id": 22318,
"tgt": "What causes persistent elevated blood pressure?",
"src": "Patient: 48 year old male, 215 lbs, exercise several times per week, take blood pressure reducing supplements such as garlic, coq10, olive leaf extract, vitamin e and many others, but blood pressure still remains around 140-170 over 90 - 105. No history of kidney or liver problems and in fairly good physical shape. Had bad accident almost 20 years ago with rods, plates, pins, wire wraps, etc. in legs & arms and had a direct drip into heart while in hospital. Could any of that be the problem? Doctor: Hello,Most of the times it's an age related problem, and along with healthy lifestyle you'll need medicine like telmisartan 40 mg. So you can these prescribed from local doctor. You should have a healthy lifestyle like avoiding fatty, oily and high calorie diet. Have low salt diet and monitor blood pressure regularly thrice a day for one week then once or twice a week. Regular exercises like brisk walking, jogging according your capacity at least 30 min a day and 5 days a week. Eat lots of green leafy vegetables, fruits, fish once or twice a week, avoid meat. Avoid smoking and alcohol if any. There shouldn't abdominal fat deposition or obesity. Get your lipid profile and sugars tested once.Hope I have answered your query. Let me know if I can assist you further. Regards,Dr. Sagar Makode"
},
{
"id": 159792,
"tgt": "Lymphoma B cancer, sticky skin around my genital areas, armpits, eye lids under arms and genitals",
"src": "Patient: I am recovering from Lymphoma B cancer and in my second remission period, I had a lump the size of a tennis ball removed from my neck here in Portugal. Last summer as chemo finished I got sticky skin around my genital areas and armpits, it went away but about three months ago it began to return and now it is on my eye lids under arms and genitals, I see on the internet many sufferes but no advice or cure, would you have any ideas please? Richard Doctor: Treatment of lymphoma by chemotherapy may be cause of dry skin but your complaint is stickiness around genitals unlikely a chemotherapy effect. You must maintain good hydration by taking enough fluids daily and maintain hygiene around the genitals. Avoid high calorie and fatty foods.Use any nonabsorbent Lotion to apply around genitals and this problem will subside."
},
{
"id": 151421,
"tgt": "Have severe headaches and increased heart rate after having a lumbar epidural steroid injection. Should I be worried?",
"src": "Patient: Yesterday I had a lower lumbar epidural steroid injection. I ve had the worse headache ever & my heartbeat is racing since then. I ve had 3 already in my upper neck area. This is my 2nd one in the lower lumbar. I ve never felt this after the others. Should I be worried? I had to cancel plans due this horrible headache & racing heartbeat. It has me a little worried. Doctor: Hi welcome to HCMForum. it appears that the symptoms have nothing to do with the injection as you have already taken them. it may due to infection of the tissues at the site of injection . but dont worry by administering proper drugs you can be relieved of the problem.i advise you to consult your doctor who gave the injection, he will be success ful to provide the solution. Besides i advise you some diet recomendations as avoid taking spices chillies junk foods oily foods. thanks for calling."
},
{
"id": 56447,
"tgt": "Is it dangerous to have GGT level of 160?",
"src": "Patient: I have raised levels of ggt of 160 my doctor tells me it has been raised for over 2 years, I drink very little alcohol and have a connective tissue disorder I take paracodine medication 45/500 mgs 4 times a day is this a dangerous level? my doctors says it is a 'fatty liver' and need to lose weight I an 54 years of age, female and weigh 10 stone Doctor: HelloHigh GGT may indicate liver injury.It may be due to many reasons like hepatitis,alcohol intake,altered lipid profile,medicines,auto immune causes etc.It is important to know other parameters like SGPT,bilirubin,alkaline phosphatase level.You may need few investigations like routine hemogram,random blood sugar,lipid profile,complete liver function test,ultrasound of abdomen.You should avoid alcohol completely.I suggest tablet ursodeoxycholic acid 300 mg twice daily for three months to my patients.It helps in regeneration of liver cells.You may need to take few more medicines after full investigations.Get well soon.Take CareDr.Indu Bhushan"
},
{
"id": 185522,
"tgt": "What causes boil filled with puss and blood below the tooth?",
"src": "Patient: Hi, I have gone through a root cannel for one of my tooth 3 years ago, after 3 years i am facing a boil filled with puss and bllod in my jaw just below the tooth (which has undergone root cannel). the issue is there for 6 weeks, please advice on possible cause . is it infection ??? Doctor: Hello, Thank you for consulting with HCM.This boil of puss, shows that again there is an infection in the same tooth or the adjacent tooth in which you got the root canal treatment done. Because in case of infection an abscess forms below the tooth which is the collection of pus only.So you should visit your dentist and get an x-ray done of that area to show the infection and get the treatment accordingly. Hope it will help you."
},
{
"id": 222663,
"tgt": "What causes intense pain in shoulder and hand during pregnancy?",
"src": "Patient: I am 12 weeks pregnant. While I was sleeping last night, I woke up with an intense and throbbing pain in my left arm. This pain radiates all the way down to my hand. My left shoulder and chest (primarily on the left side) are also painful. I am a bit weak this morning, though this could simply be from lack of sleep, but other than that, I have no other symptoms. What should I do? Doctor: Hi dear, I have gone through your question and understand your concerns.Pain in the arm and chest can be due to angina, or due to muscular or neurological pain.I will suggest you to consult a doctor immediately as angina requires immediate treatment.Hope you found the answer helpful.Wishing you good health.Dr Deepti Verma"
},
{
"id": 1133,
"tgt": "Can i take medication during pregnancy?",
"src": "Patient: Hi,I currently take 2.5mg of ramipril daily as I have moderate heart valve regurgitation and will one day have to undergo heart valve surgery. I also take 10mg of propranolol twice a day to prevent migraines, though my cardiologist said this has benefits for my heart too. I'm looking to start a family, and have been told that I will need to 'wean off' ramipril before getting pregnant. Do you know how long it will take to wean off 2.5mg of ramipril? Also, can I still take propranolol during pregnancy? I know there is a lot of risk involved; my partner and are continuously discussing the pros / cons! I weigh 9 stones, I', 5\"7 and I'm 25yrs old. Thank you. Doctor: It's not safe to take Ramipril in pregnancy. it causes decreased fluid in womb and kidney problem in the unborn child. Propranolol on the other hand is safe. Talk to your doctor and change your medication before attempting conception"
},
{
"id": 161229,
"tgt": "Can Noworm be given for lack of appetite in the child?",
"src": "Patient: Hi Doctor, My son is 11 years old. Few months back his pediatrician prescribed noworm suspension though he did not seem to have any symptoms of worms..... Only thing is he looks lean and was falling sick frequently at that time. But now he is OK. But I feel I should give him NOWORM as he is not eating well. Please advise. Can there be any side affects in case I give him this even if he did not have worms. Doctor: Hello, If deworming medication is given in the past 6 months there is no need to repeat. If he is not active and repeatedly falling sick with pneumonia or ear infections, with growth faultering child needs blood investigations. If he is active otherwise no need to do anything. Just give him a protein-rich diet. Hope I have answered your query. Let me know if I can assist you further. Take care Regards, Dr Prasanna Lakshmi, Pediatrician"
},
{
"id": 60330,
"tgt": "What treatment or further tests should be conducted for analytical sensitivity of 33.6 IU/ml ?",
"src": "Patient: HCV RNA detected with the analytical sensitivity of this assay is 33.6 IU/ml Hello Sir, I have just received the HCV-RNA blood test report of my mother and i am really worry after watching the results. Report saying : HCV RNA detected with the analytical sensitivity of this assay is 33.6 IU/ml ..Kindly suggest me the further tests, treatment or any suggestion for further treatment / tests.. I will be very very thankful to you.. Doctor: Hello. Thanks for choosing HealthcareMagic forum. Your mother is showing the presence of hepatitis C virus in her body. It causes jaundice and is a water borne disease. There is nothing to worry. The disease is fully curable and all its effects are reversible.You can consult your family physician for its management. Dr. Rakhi Tayal drrakhitayal@gmail.com"
},
{
"id": 123358,
"tgt": "What causes injury in bones?",
"src": "Patient: I totalled my car last Wed night. The bruising was very bad. As the color is turning to yellow, there is an area that is approximately 1 inch by 6 inches where the seat belt would have restrained me that is now hard to the touch. Is this normal or should I be worried? Doctor: Hi, As this is normal and happens due to friction and pressure. Allow the healing time to occur like 3-4 days and also use icing to reduce the inflammation. You should be fine soon. Hope I have answered your query. Let me know if I can assist you further. Regards, Jay Indravadan Patel, Physical Therapist or Physiotherapist"
},
{
"id": 164186,
"tgt": "Is betnesol oral drops safe for 24days old?",
"src": "Patient: SIR my daughter 24 days old she is sneezing from the day of birth.we consulted childeran doctor on 12 th day he recommended t-minic drops and saline nasal spray,we used it for one week but no use again we contacted the doctor to day ,he said notting to worry . he saked to contuniue the above medicane addition he recommended betnesol oral drops.is this medicen safe to use please advice ble Doctor: Hi... this is a long acting steroid and should not be used without a proper indication. What you are saying is not at all a indication for using such a steroid medication.I suggest you not to use Betnesol drops in your young baby.Regards - Dr. Sumanth"
},
{
"id": 163344,
"tgt": "Suggest remedy for fever, blotchy skin and lethargy in toddler",
"src": "Patient: we went to urgent care this morning, they couldn t find anything wrong with my daughter. She is 15 months old and 22 lbs. She has had a fever of 103-104 all day. She has blotchy skin all over her body, she s lethargic, and is too young to take tamiflu. is there any point in taking her to the er, or should i just stay home and ride it out? Doctor: Hello,At high-grade fever, you should rush to ER as baby can get fits any time. It could be viral or bacterial. Do CBC test. Also, improve the feeding status of the baby. And consult again with the test report.Hope I have answered your query. Let me know if I can assist you further. Regards,Dr. Hina Javed"
},
{
"id": 2763,
"tgt": "How to get pregnant?",
"src": "Patient: Hi how are you ,I m trying to get pregnant for the year I went a Fertility&Genetics we did some test and he stated that cervix is health,but I m 39 so I know my chances is very low he wanted to start fertility but I wanted to try on my own and still no thing Doctor: Hi,I understand your concerns but I agree with your doctor that you should start considering an infertility treatment as IUI or IVF or ICSI.Hope I have answered your query. Let me know if I can assist you further. Regards,Dr. Salah Saad Shoman"
},
{
"id": 23695,
"tgt": "Can premental pause be the cause for palpitation?",
"src": "Patient: Im 42 year old woman. had two healthy sons. normal delivery. developed type 2 diabeties have come off all meds and insulin for 1and 7 months. I have palputations when I'm relaxing. went and had Ekg echo and stress test. my cholesterol is well in rang and so is my A1c. I have a stressful life. owning and managing two businesses. Am I going thru premental pause Doctor: Hello, Thank you for your query. No, I do think that you are going through menopause. I think the stress is getting too much for you to handle. It is vital to manage this stress by breathing and Meditative techniques.. Please seek expert help if you are able to manage on your own*were you ekg, Echo and stress test normal? *have you had panic attacks in the past? *any family history of heart disease? *have you checked your thyroid levels? *do the palpitations persist through the day? *how much caffeine do you take in a day? *have you ever passed out? I recommend :- frequent walks during the day (after your physician gives you the go ahead). This acts as a stress buster. -quiet time for 20 minutes twice a day (no phones, meetings,computers.music helps. -limit caffeine to two cups a day. -drink plenty of water-make a conscious effort to breathe deeply in and out. It may take a while for it to become habitual. Managing two businesses can't be easy and you would have to be in the best form to keep doing that. Take my suggestions seriously and it will help. Hope this helps. Please let me know if there is anything else I can help you with. Wish you good health."
},
{
"id": 79543,
"tgt": "What treatment is suggested for cough & wheezing problem?",
"src": "Patient: Hai sir, suffering with cough and wheezing since from last three months back, telekast-l tablet 10 mg prescribed for me daily once, for 1 month ,after stopping the tablet, cough persists so iam taking same tablet whether it is safe for long duration. please guide what to do Doctor: thanks for asking your questionI completely understand your question cough of long duration and associated rhinitis can be seen in diseases like allergic rhinitis , sinusitis , post nasal drip and cough alone for longbduration can be seen in tuberculosis and cough with wheeze can be aeen in bronchial asthmaso now u need to consult a pulmonologist who can request for a sputum afb , chest xray and a spirometry if needed .this will render you safe and all yhe diseases which are a possibility can be ruled out no need to panic continue with the treatment , the drug you mentioned can easily taken for a long duration , dont worrythanks / regards feel free to ask more questions may god bless you with good health"
},
{
"id": 194563,
"tgt": "Can excessive masturbation cause erectile dysfunction?",
"src": "Patient: Hi I'm 21 years old I have been masturbating 5 To 6 days a week since I was 14, when I have sex with girls it's rare for my penis to get completely hard, I feel that my penis doesn't get as big and hard as it use too specially the head it doenst get hard as when i was 16, I was wondering what can be the problem causing this, too much masturbation? Doctor: Hi, Excessive masturbation may cause erectile dysfunction if you are still continuing with excessive masturbation. it can be treated and cured. Hope I have answered your query. Let me know if I can assist you further. Regards, Dr. K. V. Anand, Psychologist"
},
{
"id": 25709,
"tgt": "What causes pain in back and arms with BP of 158/98?",
"src": "Patient: I have pain in the back , shoulders and arms which began when I rose from my armchair to go to bed. No shortness of breath but my personal blood pressure machine shows blood pressure of 158/98 and pulse rate of 96. I always have a fairly high pulse rate - at least 80. Doctor: Hello!Welcome and thank you for asking on HCM!I understand your concern and would explain that a comprehensive differential diagnosis should be done to differentiate between different causes of this clinical scenario: - a backbone disorder. It could lead to pain in the back and shoulders, and high blood pressure due to the physical stress. - an aortic dissection could lead to this clinical scenario- anxiety could also cause similar symptoms. I recommend going to the ER for a careful physical exam and some tests: - a resting ECG and cardiac ultrasound- a chest X ray study - a cervic-thoracic column X ray study- some blood lab tests ( CPK for possible myalgia, a complete blood count, inflammation tests). You should discuss with your doctor on the above issues. Hope you will find this answer helpful!Kind regards, Dr. Iliri"
},
{
"id": 205287,
"tgt": "What causes episodes of inability to respond and unexplained anger?",
"src": "Patient: i have moments where i am unable to respond to circumstances surrounding me. i have been told that i have seizures that are short lived. I have neither noticed these events as i was not really conscience of them most of my life. Recently i was able to remember the events after they occurred, but not any memory during them. also, i feel a sense of anger without reason.is there any help for this problem? Doctor: Thanks for asking in hcM.Yes your illness is treatable. Do you have unconsciousness, tongue bite or rolling of eyes during seizure with involuntary jerky body movements??It means you are suffering from true seizures and need medications to control them. Some people with epilepsy may have some behavioral problems like anger outburst low mood etc. In such cases there should be augmentation of your anti epileptic drugs like Risperidone to control your anger. I will suggest you to visit nearby psychiatrist for counselling and medications.Dont worry after taking these medicines you will be fine in couple of weeks. hope my answer helped you. Regards"
},
{
"id": 153374,
"tgt": "Suggest treatment for low HB and platelet count",
"src": "Patient: Gd mng sir, I m from haryana kaithal distt. I belongs to a poor family, My bhabhi ( Elder brother's wife) is suffered from leukemia(A.M.L) When we go to pgi chandigarh, there doctors said dat if u hav 30 lakhs then v can start treatment, othervise there is not any solution. My family come back home wid patient, We can't afford more than 50,000 rs. At dis time, Abt patient, She is of 22 yrs old, Have 2 child one of 8 month n 2nd of 2.5 years. Husband working as a tailor in village, I m lab technician wid 4500 rs. Salary per mnth. We have no option at dat time What we can do? Today I do blood tests of patient, Hb. 5.9 g/dl(5 BT before 5dayS) Tlc 46000 Blast cell 70% Platelet count 12000/cmm I cant see dis situation. Pls sir tell me what we do? Pls try to help my family. Pls reply soon Doctor: Hi, dearI have gone through your question. I can understand your concern.Treatment of acute myeloid leukemia depends on type of leukemia and blast percentage. If you can not afford bone marrow transplantation then chemotherapy should be given it is the treatment of choice and freely available at many center. Consult your doctor and start chemotherapy according to his advise.Hope I have answered your question, if you have any doubts then contact me at bit.ly/Drsanghvihardik, I will be happy to answer you.Thanks for using health care magic.Wish you a very good health."
},
{
"id": 160219,
"tgt": "Removal of Appendix. Is this safe?",
"src": "Patient: Hi, I had my appendix removed on Feb. 14. I was released from the hospital the following day. I was released to go back to work by the surgeon one week prior to surgery . My step son same home from his mom s house with strep during my recovery. I became very sick and went to the doctor and was diagnosed with pharyengitis. I am on a Z pac and seem to be doing better. However, it is now allergy season. I teach horticulture and have a terrible problem with allergies. I generally take raw local honey. Is this safe? Also, when am I going to feel GOOD again? Doctor: Hi, Welcome to Healthcare Magic Forum. You should recover from Appendectomy surgery pretty soon if you take your medicines regularly and go for the follow up. For Pharyngitis, you may need antibiotics, speak to your Doctor so that he may examine you. Having honey is not harmful, if you are not a Diabetic. Good Luck and Take Care."
},
{
"id": 205581,
"tgt": "Suggest treatment for dementia and vomiting in a 70 years old female",
"src": "Patient: My friends mother who is in her 70 s and has dementia has had diarrhea for approx 2 weeks with little to now appetite, has seen doctors and gone to the ER 4 + x s however she is now vomiting, is this cause for another ER visit and how can she convince the doctors to admit her mom? Doctor: Hai, Dementia is a degenerative neurological illness characterized by loss of memory and deteriorating personality the rate of progression of the disease varies from individual to individual. There are medications currently available which can slow down the progression of the disease, I would advise you to see a psychiatrist at the earliest. The vomiting that she has been having could be due to a lot of reasons, taking her age in to consideration, she is in high risk of going in for dehydration which can inturn lead to acute renal shut down. Considering her age her illness, I'm sure if I was her treating physician I will surely suggest in hospital treatment."
},
{
"id": 192080,
"tgt": "Can Freego be taken when suffering from diabetes?",
"src": "Patient: Hi, my father is 52 years old and has high diabetes. He is also having liver problems and doctor has prescribed Freego (Lacitol monohydrate & Ispaghul Husk granules). It is quite sweet to eat? What is the purpose and wouldn't it be harmful for a diabetes patient? Doctor: Hello,Welcome to HCMThanks for posting your query in HCM.I understand your concerns.Freego contains Lactitol and Ispaghul Husk granules as you know already.Lactitol is a sugar alcohol.It is used in sugar-free candies, cookies (biscuits), chocolate, and ice cream. Lactitol also promotes colon health as a prebiotic.Lactitol in combination with Ispaghula husk is an approved combination for idiopathic constipation as a laxative and is used to prevent or treat constipation.It is the best Osmo-Bulk laxative in Constipation associated withIrritable bowel syndromeFissuresHaemorrhoidsPregnancy andPre and post-operative conditions.Your father may be suffering from some type of constipation.So,it has been prescribed.Lacitol doesnt cause increase in blood glucose levels.So it can be taken by your father if he is diabetic also .Hope you are happy with the answer.Thank you"
},
{
"id": 118860,
"tgt": "Vein in the shoulder seems to be fluttering fast, no pain. Cause?",
"src": "Patient: Hi i am a 16 year old female. I weigh 125 pounds and have no other health issues that i know of, but a vein or artery in my left shoulder is fluttering really fast, i can not only feel it but see it doing it, and then it will beat really hard about every 4 seconds and it looks like its jumping. Its not painful, but i am rather worried. Doctor: hiThanks for your queryi can feel your concernsyou need not to worry,usually it is artery which is a bit superficial and you feel blood flowing in to it.it is painless and not associated with any problem.hopr i have ansewered your querywishing you good healthregardsDr.AMNA"
},
{
"id": 212506,
"tgt": "Mild head tremors, major depression, feeling flushed due to anxiety. History of rear end collisions. Causes?",
"src": "Patient: Hi, I just began having slight head tremors. I have history of heart disese in my family and will be seeing a heart doctor soon. I also have major depression . Sometimes I feel flushed when I am anxious and could this be part of the tremors. I also had 2 rear end collisions in October. Any ideas on the tremors? Thanks! Doctor: Hi there, thanks for asking. It could be related to your depression and anxiety or it could be related your neck injury following the collision. If it has caused some ligament injury, that may trigger the tremors and it is better to ask your doctor to assess it further. It may also be an early sign of Parkinson's disease so it is important to ask your doctor to have a full exam to see what the reason might be. It could also be a part of symptoms related to post concussion syndrome (after head injury) which is very important to avoid accurate mental activities if it is diagnosed. I wish you a good health."
},
{
"id": 107376,
"tgt": "What causes chronic right sided mid back pain?",
"src": "Patient: I am having some severe mid back pain on the right side. Like a muscle spasm although I have not done anything strenuous. It interferes with breathing and movement. Just trying to get an idea of what it might be or anything I can do to minimize the pain. I am afraid to move Doctor: Hi i am Dr Ahmed Aly thanks for using healthcaremagic site ,I had gone through your question and understand your concerns .. In my opinion you have a sort of a muscular spasm or radiating pain due to nerve compression . in mild cases of sciatica low back pain may be resolved simultaneously or with relaxation . For my patient i may suggest advil tab when needed , hot massages with topical gels , B12 vitamin supplementation with some physiotherapy and spinal straightening exercises and even yoga will be helpful. If pain persists i recommend MRI to rule any spinal disc herniation for proper evaluation and management . Please click and consider a 5 star rating with some positive feedback if the information was helpful. Wish you good health,Any further clarifications feel free to ask."
},
{
"id": 111518,
"tgt": "Need treatment for lower back pain",
"src": "Patient: Hi my wife is facing severe lower back ache since past 6 months. As per doctor, the arc of her lower back has deformed slightly due to muscle spasm. She has hypothyroidism and as per recent reports she has vitamin b12 & vitamin D deficiency. Doctor has prescribed her mecobalamin injection 500mcg. Need your 2nd opinion. Doctor: Hello,I had gone through the case and found that for muscle spasm take mild painkiller with local application of muscle relaxant gel.Mecobalamin injection is good for that.For vitamin B12 and vitamin D take proper diet rich in calcium and take egg and fish for vitamin b12.Also take VITAMIN d3 60,0000IP UNIT once a week with milk.Hope my answer will be effective for you.Thanks"
},
{
"id": 25747,
"tgt": "Is it likely for the heart functioning to decrease after undergoing multiple heart attacks?",
"src": "Patient: My father who is 74 had a cardiac arrest and was revived 3 times. He has a 10% heart function. And was told several months ago that he was at stage 4 heart failure. He suffered a mild heart attack back in 1996 or so and he has been on high blood medicine, water pills, and medication for his heart. He is being released today from the hospital to go home with hospice care. What is the possibility of him dying within the next weeks or months? Doctor: Thanks for your question on Health Care Magic. I can understand your concern. Yes, heart attack reduces heart muscle strength and cause permanent damage to heart tissue. So heart functions tend two reduce with every heart attack. At present he is having only 10 % functioning of heart. This is very very low for survival. So he is carrying very poor prognosis. According to my clinical experience, he is having life expectancy of less than 6-8 weeks. Hope I have solved your query. I will be happy to help you further. Wishing good health to your father. Thanks."
},
{
"id": 103612,
"tgt": "Phlegm in chest keeps recurring. Took antibiotics. Could it be allergy?",
"src": "Patient: Hi, just a question about phlegm in my chest.It seems to keep coming back. I've been to the doctor and he gave me antibiotics the second time, it went away, but then every few weeks it comes back and there is a sore throat in the morning and then phlegm in my lunchs and throat again. it does not hurt, just annoying. I'm wondering if it's something in my home that's setting it off. an allergy to dust? something i'm eating? Doctor: Hi,Yes it could be allergy.u can undergo allergen test to see for allergen or change ur living place for a while to see the effects.byeee"
},
{
"id": 22085,
"tgt": "Does open heart surgery have risks of stroke and other complications?",
"src": "Patient: My 82 yr old mother has severe aortic stenosis, she has had two previous open heart surgeries mitral valve repair and then mitral valve replaced. Her drs do no feel she can endure another surgery and have suggested balloon valuoplasty of her aortic. Ihave real concerns with the risk of stroke as well as possible other complications resulting in having to do open-heart or possible death. Any advice would be appreciated Doctor: Hello , 3rd open heart surgery at 82 years would surely be a risky proposition particularly if she is flail . There is definitive risk of stroke however that is not a major and only concern . It's her probability of sustaining and surgery and weaning off from the Ventilator and Bypass machine . Balloon aortic valvulaoplasy would be decent palliative procedure which can curtail her symptoms to a great extent . I would hereby suggest an alternative of Transcatheter aortic valve replacement ( TAVR ) if the facility for same is available. Do discuss the same with your doctor . Regards Dr. Mody"
},
{
"id": 74579,
"tgt": "What causes a tingling feeling on right side of chest with trouble sleeping?",
"src": "Patient: I am a 20 year old male. I am a chain smoker and a little over weight. I have trouble sleeping. When i'm in bed it feels like something is wrong and i suddenly sit up. I also get a tingling feeling on the right side of my chest. Sometimes i have pain in my arms. It feels like muscular pain but i am not sure. I am a chain smoker and a non-alcoholic besides that i have never used drugs. At rest my heart rate is usually 76 beats per minute. Can you determine what the problem is? Doctor: Respected user , hi I have evaluated your query thoroughly .@ These symptoms suggest psychological manifestation of the underlying stress or anxiety disorder .Hope this will help you .Regards ."
},
{
"id": 167,
"tgt": "When is it safe to conceive again after an abortion?",
"src": "Patient: sir, my last period date was 27 April 11. On may 23rd i confirmed i m pregnent but due to some circumstances i took medicine on 8th May to abort the baby. Now my date has overed on 29th May 11. Pls tell me now which period is best for conceive the baby again. thanks Doctor: Hello,Firstly, regular ovulation must be established. Fertile period or ovulation period is 2 weeks prior to the onset of the next period. Regular unprotected intercourse during this period is needed for conception. Rest flat for 30 minutes after the intercourse to help proper ascent of the sperm. Rest on your abdomen if you have a retroverted uterus.Hope I have answered your query. Let me know if I can assist you further.Regards,Dr. Shanti Vennam"
},
{
"id": 99051,
"tgt": "Is extensive swimming while having asthma safe?",
"src": "Patient: i am a swimmer with asthma. today in practice we did a set called the lung crusher. 1 does that even sound like a set i should do? 2 the work out is basiclly that i have to swim 30 yards with no breath. i can feel my lungs getting tight and breath when im not suppose to but i'd rather not die and my coach wont and doesnt believe me when i say it is acting up my asthma, he just says it is all mental. what can i tell him to believe me or do? Doctor: HI, thanks for using healthcare magicYou would need to get a certificate or letter from your doctor discussing your asthma and any possible risks that this particular training exercise may have for you.Your doctor would be able to assess you and help you however possible.I hope this helps"
},
{
"id": 148634,
"tgt": "What symptoms should we expect from my dad who is suffering from carbon monoxide poisoning brain damage?",
"src": "Patient: My dad is suffering from carbon monoxide poisoning brain damage. . The doctors told us 60-70% of people that get this delayed brain damage from CO do recover.. But could take about a year or so. What should we expect his symptoms to be this next year? Doctor: Hi,Thank you for posting your query.The extent of brain damage depends on the amount of carbon monoxide exposure and also its duration of exposure.PET scan of brain and EEG tests would help to quantify the brain dysfunction (in addition to the clinical neurological examination).Recovery may be delayed, and also it may not be complete.I hope my reply has helped you.I would be pleased to answer, if you have any follow up queries or if you require any further information.\u00a0\u00a0\u00a0\u00a0\u00a0Best wishes,Dr Sudhir Kumar MD (Internal Medicine), DM (Neurology)Senior Consultant NeurologistApollo Hospitals, Hyderabad,For DIRECT QUERY to me: http://bit.ly/Dr-Sudhir-kumar My blog: http://bestneurodoctor.blogspot.com/"
},
{
"id": 122611,
"tgt": "How long will it take for uric acid levels to be normal?",
"src": "Patient: Hi, I am 34 years old, suffering from High Uric Acid. Uric Acid Serum Test gave 6.9-7.0 values. I have started consulting Ayurveda Doctor. He advised me Kaishore Guggul, Udarmrit Vati, Punranvadi Mandur after meals. its been a month now on above drugs. I still feel my Knee getting locked and I feel the knee joint got stiffness. The Alopathic alternative, suggeste by Dr is Febutaez 40. It helped crackinlg and stiffness. But, my Question is How long it will take to recover from metabolim issue which is causing High Uric Acid? Doctor: Dear siruric acid level 6 or 7 is just on the higher side and cannot said that it is increased, your symptoms of knee pain and locking of knee might be due to other reasons.get your complete check up done including MRI of knee, X rays of knee, thyroid profile and other investigations for joint pain like CRP ,RA factor, ANTICCP.however regarding your query the uric acid level of 7 come down within a month after taking tablet febutaz for one month and avoiding the food products which increases the uric acid.thanks and regards."
},
{
"id": 166695,
"tgt": "What causes pain in leg calf while having flu?",
"src": "Patient: Hello. My 8 year old son had the flu this week, and this morning woke with pain in the calf area of his legs, more concentrated on one leg, but some pain in the other calf. Is there cause for concern, or could the pain be associated with growing pains and not necessarily connected with the flu? Doctor: Hello,If the calf area is swollen and red along with the pain, then you should take him to the hospital right away as it might be a deep vein thrombosis.Hope I have answered your query. Let me know if I can assist you further.Regards,Dr. Salah Saad Shoman"
},
{
"id": 460,
"tgt": "How to get pregnant while suffering from irregular periods?",
"src": "Patient: sir iam 30 yrs male last year my marrage happened.iam apharmacist so i have full knowlege of sex with my wife.but she not to be known of that much knowledge about sex whatever ..,i teach her for the geting good baby. but she is having some fears and most probley iritations in her points.so i releieve her in some seconds,but i am doing well and ejaculate about 2mi t0 3sperm in her cervix area..,but not yet she got preganant.every month iam expecting but she shows irrugular and delayed periods what i can i do get good baby for me Doctor: Hi, Thanks for the query.I understand your concern. Regular menstruation with ovulation is essential for conception as is good quality of semen. Considering your age&wish to con eive earliest... I would suggest for her usg of uterus/ovaries & ovulatuon study... and also your semen analysis . Get needful treatment &then olan for a baby. Having sex 2 days before &two days after the ovulation day (bybasal body temprature or LHSurge test)would help in conception. In case there is no positive sign with 3 months trial..you should consult a gynecologist for detailed investigations &needful aid. Thanks."
},
{
"id": 178719,
"tgt": "What is the cure for severe cough and nose bleeding in a 7 month old?",
"src": "Patient: Hi my 7 month old baby is struggeling with a cough, its not a wet cough, she had the rota virus for a week and is beter now, but now she started to cough and when she cough she starting to cry like its sore. She is still on moxypen antibiotics, she s also sneezing and at times her nose bleeds a little. What can I do to help my baby feel beter? Doctor: Hi...you can try any anti cold medicine if your doctor has given any (as these are not over the counter at this age) and for stuffy nose use any saline nasal drops. Mild bleeding occasionally is common in sever cold and if the bleed is recurrent see a pediatrician. regards - Dr. Sumanth"
},
{
"id": 163234,
"tgt": "Does Omnacortil given for three days have any side effects?",
"src": "Patient: Hi, my 4 yr daughter gets cold n cough very frequently.along with medicines like asthalin for nebulisation n cough syrup n antibiotic doctor has also given her OMNACORTIL which is steroid for 3 days. i kno side effects of omnacortil but i want to kno if given for a period of 3 days what will be its side effects? pls help me out Doctor: Hello and Welcome to \u2018Ask A Doctor\u2019 service.I have reviewed your query and here is my advice.Omnacortil will give minimal effect if given in small doses for few days as 3 or 5 days. Many doctors prescribe for various infections under the cover of antibiotics.Low dose steriods upto 2 weeks are safe. Earliest symptoms are weight gain and high BP. Hope I have answered your query. Let me know if I can assist you further.Regards,Dr. Varinder Joshi"
},
{
"id": 128557,
"tgt": "How to identify a bone fracture in the neck caused by an injury?",
"src": "Patient: About three weeks ago I had a nasty fall in which I landed on the back of my neck and the weight of the rest of my body came down on top of me. St the time I heard a horribling cracking sound and has a burning sensation. It was able to get up and move around but my neck felt weird. Later that evening my neck felt stiff- was visibly swollen. Had pin and needle in my little finger and shooting pains in my arm. Had limited movement for a week but has got nearly full movement back now. But it still hurts especially at night, nipping and burning sensation. Pins and needles sometimes. I feel that running, exercise, going up and down stairs all exaggerates it and causes pain. Should I be worried, is it possible I could have fractured a bone in my neck? A silly question sorry. Doctor: Dear patient since you are able to move your neck without reasonable ke pain possibility of neck vertebra is less likely. pins and needles in upper limb and burning sensation can be caused by nerve compression due to doc injury. That can be ruled out by Mri of cervical spine. Meanwhile start tab pregalin x 75 mg one at bedtime. If tour report is abnormal please consult neurophyysician nearby you."
},
{
"id": 116179,
"tgt": "What causes elevated neutrophil count?",
"src": "Patient: My absolute neutrophil count has been greater than 8000 less than 9000 for the past four months. All other blood results have been within the normal range. I have a recheck in December. How worried should I be about these levels? Is there anything dietary that I can change to help lower them? Doctor: HiThanks for askingFrom your history, findings and query, my opinion is as follows1. High neutrophii counts signify acute infection mostly bacterial, acute inflammation.2. If you are asymptomatic now, there is no need for concern as counts settle to normal range when infection subsides.Hope this helpsRegards"
},
{
"id": 220922,
"tgt": "What causes brownish vaginal discharge during pregnancy?",
"src": "Patient: Hello I am 7weeks today. For a few days off and on when I go to the restroom I have small brownish tissue on the tissue when I wipe. I dont see it on my underwear just sometimes when I wipe here and there. The last time I had my hcg numbers tested they jumped from 21,00 to 42,000. In the past i have had a few miscarriages due to lupus anticoagulat. Is brownish tissue always a bad thing or could it mean something else? Doctor: Hello dear,I understand your concern.In my opinion the presence of brownish spotting might come under threatened miscarriage where in its just a indication that miscarriage might happen if the proper care is not taken.Also as you have lupus anticoagulant which might be a cause for recurrent miscarriage the treatment might be needed for that.Avoid physical strain and intercourse.Take good rest.Also ultrasound might be helpful in knowing the pregnancy status.Its good that HCG levels have doubled up.Dont worry many cases with lupus anticoagulant also progress and will have good outcome.Relax.Best regards..."
},
{
"id": 223644,
"tgt": "Suggest treatment to avoid pregnancy after intercourse",
"src": "Patient: hi....this is sandya m a student.........i had sex wit my boyfriend on my 9th day of period started but nw this month 23rd has to get my period bt still dint get my period so wt cn i do get.......if i m conciev how can i get abort myself in home....plz suggest me Doctor: Hello dear.understand your concerns.first of all, you have sex on 9th day of period which is safe period and have less chance to become pregnant.if you still worried then you can check your pregnancy at home by urine pregnancy test . after 7 days delayed period you can check it.or you can check it by blood HCG test.if you are pregnant then up to 3 month intrauterine pregnancy,you can ABORTION by pills like pregout kit.This kit has 1 mifepristone tab.600 mg ,take it OrallyAfter 48 hours,take another 4 tablets of misoprostol 200ug placed in vagina one by one.I advise,di it under supervision of Gynecologist.I hope this will help you.ThanksDr.sagar"
},
{
"id": 212473,
"tgt": "Pins and needles in leg, back pain and pain at base of skull. Is it Stress?",
"src": "Patient: iI was recently told i had ms now afterfive months told never had ms. i have feeling like walking on broken glass and pins needles in legs all times back pain sleep feeling in arms and several times have pain at base of skull . the neurlogist said it could be stress and axiety related. help me please i have had enough of uncertain what is wrong thank you svcory , rainy river ontario canada Doctor: Hello and welcome to Healthcare Magic. Thanks for your query. There can be a variety of causes for having a sensation of pins and needles on your legs. There can be medical causes like peripheral neuropathy or psychological causes like anxiety / stress. Since you have had an evaluation by a neurologist, it is unlikely that your symptoms are due to any neurological or medical cause. Such sensations and pains are commonly seen in anxuety disorders. It is important to understand that anxiety can manifest with not only psychological symptoms, but with a variety of physical symptoms also. So, I suggest that it would be worthwhile to consult a psychiatrist for a detailed psychological assessment and further treatment. Wish you all the best. Regards, Dr. Jonas Sundarakumar Consultant Psychiatrist"
},
{
"id": 125373,
"tgt": "Suggest treatment for slip disc",
"src": "Patient: Respected doctor, I have been suffering from slip disc, disc3. Right now I have been getting treatment for the last one month. I am doing Yoga . I Have been suffering from back pain for the last three months and consulted the doctor when I felt pain in right leg. I am doing Yoga suggested by the physio. I want to recover fast kindly guide me . Thanks Deep (39yrs) Doctor: Hello, Consult an orthopedician and get evaluated. If the symptoms are mild, you will improve with physiotherapy and lifestyle modifications. In severe cases, surgical correction will be needed and it is unlikely in your case. Hope I have answered your query. Let me know if I can assist you further. Regards, Dr. Shinas Hussain, General & Family Physician"
},
{
"id": 217071,
"tgt": "What causes sharp pain in the lower back while getting up from chair?",
"src": "Patient: Yes about an hour ago I got up from sitting in a chair and all of a sudden as I was getting up a really sharp pain hit me in my lower left side The sharp pain isn t now what it was earlier but it hurts and can t walk hardly without limping What s wrong with me ??? Doctor: Thanks for keeping trust with us..In my opinion nothing to worry about it. ..as u don't have limping ; most likely it is muscle spasm ....just avoid weight bearing over it...take a tab diclofenic 50 mg sos that will be enough ...If it still persist go for a orthopedic consultation"
},
{
"id": 108223,
"tgt": "What causes back pain?",
"src": "Patient: MY HUSBAND HAD AN ILIAC STENT PUT IN THAT WAS PERFORATED BY HARD CALCIUM. HE LOST ABOUT 2 TO 3 PINTS OF BLOOD IN BODY CAVITY. THE DOCTOR WENT BACK IN AND INSERTED MESH TO STOP THE BLEED. HOSPITAL STAY FOR 4 DAYS. HE IS HOME NOW BUT EXPERIENCING A LOT OF BACK PAIN. BLOOD PRESSURE IS OKAY. WHAT IS CAUSING THE BACK PAIN YYYY@YYYY HOW LONG WILL IT TAKE FOR THE BODY TO ABSORB THE BLOOD IN HIS ABDOMEN? Doctor: Dear friend, I understood your concerns, see this is called as peptic ulcer or you May have ulcerative collitis, then only you may have bleeding in stomach or intestine, as your ulcers heals your bleeding stops and pain also decrease... and pain in back may be due to weakness or spasm in muscles so we need to take ice treatment for pain reduction. Within 15days pain will decrease. You have to improve your posture also.."
},
{
"id": 223517,
"tgt": "Should i be worried about getting pregnant after using contraception?",
"src": "Patient: Hi im just really nervous my boyfriend and i had sex april 19th and the condom broke, he didn t cum or anything yet it was just what people call Prejizz He pulled out right after he felt it break. But we still freaked out and i took the plan b pill 45 mins later. Then the next day we were having sex and the condom broke AGAIN with the same prejizz and just to be safe i took the plan b pill again but within 13 hours. im due for my period i think april 26 or 27 im just extremely nervous the only side effects i have had are tiny headaches every now and then and im super emotional my mood swings are everywhere. But i dont know if thats just due to the fact of stressing out about this. I know the plan b pill is very effective from the reviews ive read. But i just need a professionals opinion. we used a condom both times and they broke both times but everytime it broke he pulled out right after and didnt cum it was just prejizz and i took the plan b pill before 24 hrs both times. should I be worried?? please get back to me as soon as possible. The worrying is killing me. Thank you for your time. Doctor: Hello,Welcome to Healthcare Magic.I have read your question fully and understand your concerns.It is very unlikely that you could become pregnant. Your boyfriend did not ejaculate, and the pre-semen technically does not have any sperms in it. But a few sperms could have escaped from his previous ejaculate and could be present now. But it is still highly unlikely that you could become pregnant as you used emergency pill on both the episodes.As you have taken two emergency pills, your period may be a bit delayed though.The symptoms that you are experiencing are more because of the stress.I hope this answer has helped you. Thank youRegardsDr Sunita Sayammagaru"
},
{
"id": 162293,
"tgt": "Suggest remedy and diet food, for loose motion in infants",
"src": "Patient: hello sir my baby is one year and 5 months old but she prefers only milk and she has loose motion for the past two days as we travel long, please give me some tips to make her eat other food and some remedies to solve her loose motion thanks in advance sudha Doctor: Hello, A child of this age generally needs to eat three large meals and three small snacks with adequate emphasis on fruits and vegetables. Generally, milk consumption without solid food reduces the child's appetite and may cause loose stools. In the case of diarrhoea while travelling it could be because of altered diet pattern or the milk itself. I would suggest you give adequate fluids, may be oral re-hydration fluids to replenish the ongoing fluid losses. The rule is one glass of Oral Rehydration solution of 200ml for every loose stool. Then observe her appetite, urine output. There is no medicine to improve appetite. Hope I have answered your query. Let me know if I can assist you further. Take care Regards, Dr ASHWIN BALIGA, Pediatrician"
},
{
"id": 64625,
"tgt": "Suggest treatment for a large lump on the head",
"src": "Patient: Hi, I m a 50 year old male. I fell off my bike when I was 6 hitting my head hard on the sidewalk. I had a large bump which took a while to go down, I still have a small remnant of it. My question is if there are long lasting effects that impact cognitive abilities as you age? Doctor: Hi,Dear,Good evening.Thanks for your query to my virtual HCM Clinic.I studied your query in-depth.In My opinion-diagnosis is-Resolved ? Calsified ?Fibrosed-Heamtoma -post trauma.AS you never had head injury signs-like gidiness,vomiting, No need to worry about the Cognitive losses after 44 yrs of hit.Still if u worry of the brain damages-MRI study of the brain-parts would resolve your worry,after Consulting with NEURO-SURGEION and neuro-physician.Hope this would resolve your worrying query.Wellcome for more query from you.Good day."
},
{
"id": 69152,
"tgt": "Suggest remedy for lumps in belly button",
"src": "Patient: hi i have had a lump on the right side of my belly button for a wile now it has caused me NO pain but recently i have another on the top of my belly button wich causes me pain if touched or some times when i am sat down please could you help me to find out what this problem is ?? Doctor: Hello!Thank you for the query.This lumps are most likely caused by umbilical hernia. Due to small hole in the abdominal wall fatty tissue gets out from the abdominal cavity. It may give pain if the fat stuck in the hernia.I suggest you to consult a surgeon with it. Ultrasound can be done to confirm it.Surgery is necessary to fix it.Hope this will help.Regards."
},
{
"id": 37480,
"tgt": "Is ivy poison, related with red bumps, on foot and fingers?",
"src": "Patient: A few days ago I noticed a small red bump on the top of my left foot that itched a little. It grew in size and a few days later I had a few more bumps on my left foot and on my fingers. Also a few on my right calf. The ones that I scratched weep a clear fluid when I rub them with an alcohol wipe. Those I don t scratch are a hard looking people that only itch if I rub them. I recently started using Cortizone Cream.Is this poison ivy or what? Doctor: Hello,Thank you for your contact to healthcare magic.I understand your health concern, if I am your doctor I suggest you that hard bump without fluids are not related to ivy poison and those coming out with clear fluid is related to poison ivy. For hard bump cortisone is not right cream. You should consult doctor and take appropriate treatment for it. Because visual examinations will only help in proper treatment.I will be happy to answer all your future concern. Thank you,Dr Arun TankInfectious disease specialist.Wish you a best health at health care magic."
},
{
"id": 138636,
"tgt": "Suggest treatment for sharp pain in ribs",
"src": "Patient: My x-ray result say that there is mild unfolding of the aortic arch and there is poor inspirating effort,there is normal cardiac size and shape.what could be wrong whit me ,i have sharp radiating pains in my right ribs andat the back pls suggest treatment Doctor: Here as per your x ray report the heart size and shape is normal but still you have sharp pain in your rib and radiating to you back. It can be related to couple of things.It can be due to vitamine D and calcium deficiency so that there could be weak bone profile.It can also be problem at spinal level which is paining at spine as well as at rib.I suggest to visit a orthopedic doctor for proper physical examination. And doctor may ask you to do some tests like ECG, tredmile test as well MRI scan which ever is needed. Here doctor will suggest you treatment with few medications like vitamine D, and calcium tablets. Along with this i also suggest to take a exercise program from physio for improving cardiac functioning as well for spinal strengthening and improving lung capacity.In case any thing major gets diagnose in tests then you may need a surgical correction too. But as per your symptoms given by you does not looks any thing major.Here I have given you general guidelines as per your symptoms because physical examination and check is not possible. How ever it will be useful for you if you follow step by step.Take care."
},
{
"id": 110228,
"tgt": "Suggest treatment for back pain",
"src": "Patient: My Mom is having chronic back pain .. we have just visisted orthopedics doctor . He adivised to get MRI L-S Spine and he is suspecting that there must be bone displacement issue. What is the best approach to recover from the back pain? does it need any surgery Doctor: Hi, Welcome to healthcare magic. After going through your query I think your mother is suffering from chronic backache.Treatment of back pain is exercises, NUROKIND GOLD ONCE DAILY and analgesics (diclofenac 100mgSR). Sometimes vitamin D deficiency is the cause so get your vitamin D checked .If it is low then vitamin D supplementation (Bon DK 60K weekly with milk) can be taken. Avoid long continuous standing. Sit in a straight posture. Eat milk, fruits and green leafy vegetables daily. To further investigate MRI of the region is advised. It sometimes needs surgery such as in case of spinal stenosis and spondylolisthesis .I think your query answered. Welcome to any follow up query."
},
{
"id": 172544,
"tgt": "Suggest treatment for persistent pain in left abdomen",
"src": "Patient: Hi doctor,I have 8 year old daughter who gets pain on her left abdomen 3-4 times already now, mostly at night. It s located under her left ribs. Usually putting some rub oil on the area and taking panadol, would comfort her but last night, it took almost 2 hours to get the pains away.We have checked once to a GP once, and the doctor said that it might be just cramps. But it just happened again last night and she really was in a lot of pain. And I don t know what to do.Can I have your opinion, please..... Doctor: HiWelcome to the HCMI have gone through your question and understand your concerns. Pain in your daughter seems to be due to some musculoskeletal problem, pulmonary illness or gastritis. For gastritis, you may give her antacid such as Lansoprazole or Ranitidine for next 3-4 days along with healthy, non spicy diet with plenty of fluids to see for the response. In case the pain worsens at any time or doesn't improve, take her to a pediatrician for proper clinical evaluation and laboratory investigation if required. They may advice you for a chest x Ray and ultrasound abdomen to come to a definitive diagnosis.Hopefully this will help you. I would be happy to help you in any further questions.Take care"
},
{
"id": 42306,
"tgt": "What does the sperm count in my semen analysis test indicate?",
"src": "Patient: hi Dr. I need your help to know that we are planning to have baby for last six months but did not get success.so Dr prescribed Semen analysis for me And the results are likeMacroscopy: Quantity 3.5 ml Viscosity Less viscous Colour Normal pH 8.0 Liquefaction time 30 minutesMicroscopy: Sperm Count 4.5 millions/ml Motility ; Active 25% sluggish 10% Nonmotile 65% Abnormal forms About 30% Pus Cells 2-4/HPF Others ----Please tell me about this report what does it mean Doctor: Hi,I have gone through your query.You semen analysis report is abnormal. Sperm count is very low along with less number of active sperms. This is the cause of infertility. You need to visit infertility expert for proper treatment.Hope i answered your query.You can contact me on my page http://bit.ly/DrGauravWish you a good health."
},
{
"id": 148582,
"tgt": "Scaling and tingling fingers, touched zucchini",
"src": "Patient: i got home about an hour and a half ago, and probably 15-30 minutes after i got home, i noticed the fingers on my left hand were scaling and tingly/numb. the only thing i ve touched is the zucchini i was making for dinner, which i had rinsed off with fit . now i m noticing tingliness and numbness all up and down my left side, including my leg and foot . i m starting to panic. Doctor: Dear best things you can do is cnsulting a neurologist as soon as possible.If you suffer from arterial hypertension or/and diabet mellitus is time change treatment.You should do a carotid ultrasound but you neurologist will explain it to you.Wish you all the best"
},
{
"id": 30015,
"tgt": "Suggest treatment for severe chest infection",
"src": "Patient: HiMy daughter is 11 months old she has acute respiratory infection and also a chest infection and my paediatrician prescribe orelox 40mg/5ml suspension for 5 days and ventoline with normal saline nebulise... I want to know that this antibiotics suspension is cure my daughter's chest infection or not?? Doctor: Thanks for posting your query to hcm. your paediatrician start treatment in a correct manner as we need to start treatment with a antibiotic to clear chest infection .hopefully it will cure your daughter."
},
{
"id": 18145,
"tgt": "How can plaque build up inside the heart arteries be treated?",
"src": "Patient: my cardiologist says my heart has pockets of plaque treatment to be either nitro(doesn t work!) or medicine?i m 77 female had 2 mild heart attacks.i m out of breath when i do something other than sit. he ruled out by-pass surgery,(*lungs& kidneys.)what course should i take?i m still robust! carole sciallo YYYY@YYYY Doctor: Hello and Welcome to \u2018Ask A Doctor\u2019 service. I have reviewed your query and here is my advice. Bypass surgery and angioplasty are the best options. It may be possible because of your age and health issues you have. You can start on thrombolytics like Aspirin or Clopidogrel as advised by your cardiologist. It will help dissolve the clots and prevent further clot formation. Wishing you good health. Thanks."
},
{
"id": 179806,
"tgt": "What causes brown,sticky and dry stools in a child?",
"src": "Patient: My four year old daughter has been having dark brown stool that is dry, somewhat sticky and flaky. The consistency is a lot like dry mud. It sticks to her bottom, so I have to wipe her and as I do, it falls off in dry flakes. She has been also dealing with constipation since about October of 2013. I ve visited doctors multiple times, they gave her an enema (which did nothing at all), and about four x-rays-- but I have yet to receive any answers. I ve given her MiraLax and cut constipating food out of her diet, I ve also increased her water intake. I just don t know what it could be. I am desperate for answers. Thanks! Doctor: Thanks for the query.Sometimes, older babies become constipated because they are trying to avoid pain. For example, they may have a tear in the skin around the opening of their bottom (anal fissure). This can become a vicious cycle. Your baby holds on and gets more constipated, and then the pain is even worse when she does eventually go. Sometimes some medication, viral infection may ause this problem.Regular stooltiming will help, also try glycerin suppositories.Hope i helped you."
},
{
"id": 2650,
"tgt": "What precautions need to be taken in order to get pregnant?",
"src": "Patient: I had my baby after 2 abortions (1st in 5months 2nd in 2months) .....I have still irregular cycles.. ~45 days sometimes up to 3months with no cycle I consulted many gynecologists treated with OCPs then metformin and Norprolac for 3months and lastly Clomid with duphaston ......with no pregnancy!! At begening l had PCOS by U/S Now....no cysts.... Just 3follicle (after Clomid) My expected date of ovulation..28-29-30/April-1-2/May Is the is any social precautions or advice for me as my hasband will travel after 2months for 6months and l wish to get pregnant before that I started duphaston 2tabs per day but I don t know for how long?? Plz help me Thanks. Doctor: Hi,I guess your concern is about precaution need to be taken in order to get pregnant. Firstly I want to tell you that your main problem is irregular menses, and because of this you are not getting pregnant yet. But RELAX, ITS NOT A BIG DEAL, its a treatable thing. It is most common thing in reproductive age womens (reproductive age women are 15-45 yrs in which chances of getting pregnant is high). But you did not give information about your age. So I don't give any comment on this. You told that your gynecologist treated you with medicines ( ocps, metformin, norprolac). These medicines are given to the patients who are having PCOS. So don't worry you are treated with right medicines.Hope I have answered your query. Let me know if I can assist you further. Regards,Dr. Sudha Rani Panagar"
},
{
"id": 70026,
"tgt": "What could cause on bones especially near ribs?",
"src": "Patient: I've been sick for 10 months now with no diagnosis yet. I have anemia of unknown origin and transfusion dependent. My body is forming lumps on several of my bones. They feel fixed and not painful but the lumps on my ribs front and back are painful? Any idea? Doctor: Hi.This is impossible in days of modern medicine that the cause of your anemia is not diagnosed.Meet the MD Doctor or preferably a hematologist to get the correct diagnosis. You will get one.Correct diagnosis only can cure you .The bone lumps must be biopsied at least go for an FNAC. See the correct Doctor. By telling the probable causes of un-diagnosed anemia, I do not want to make any frightening statement. Please go for a Proper Doctor and get teh diagnosis."
},
{
"id": 36932,
"tgt": "What causes feeling like having infection in genital area?",
"src": "Patient: I am a 16 year old male and i think that i have an infection in my genital area, it has been like this for quite some time now and is sometimes inflamed. It is concerning me a lot because I have never had sex before and I am worried as to how i got it.I am nervous to tell my parents, what can i do about it? Doctor: Hi,It is not clear that what type of infection is present on your genital area.This might be some local causes like yeast infection, dermatitis or ingrown hair follicle infection.This is normal if there is lack of personal hygiene and do not hesitate telling your parents.Consult your doctor and get examined.After proper diagnosis he will go for treatment accordingly. Keep local hygiene clean, well shaved and dry.Ok and take care."
},
{
"id": 207732,
"tgt": "Suggest remedy for fear and nightmares",
"src": "Patient: Hi, I need help with a problem, I'm 16 years old but I suffer every night from terrible nightmares, and I shake with fear at night, and even some times during the day. I always feel like theres someone in my house watching me or waiting to harm me. This is all really confusing to me, I think I have paranoia, what would you say? Doctor: DearWe understand your concernsI went through your details. I suggest you not to worry much. You are aware of paranoia, are you? If the description you are projecting here is true to the best of your knowledge, you definitely have paranoia. But, there is another way of seeing this. For a person who feels that he may have paranoia, could be obsessed with such a thought, and may feel paranoia like symptoms. you need to visit a psychologist / psychiatrist for clarification. Please don't guide the therapist. Just tell the symptoms and let them diagnose.If you require more of my help in this aspect, Please post a direct question to me in this website. Make sure that you include every minute details possible. I shall prescribe the needed psychotherapy techniques which should help you cure your condition further.Hope this answers your query. Available for further clarifications.Good luck."
},
{
"id": 151179,
"tgt": "Pain in calf and ankle, worsens in the morning. Taken cortisone injection. Any ideas?",
"src": "Patient: Hi, I have a calf/ankle nerve ache that has been with me for almost a year. It is worse in the mornings and normally eases during the afternoon. Have had cortisone injection in my back and there was no change. Any ideas? Gets to the point some days that I need to sit down to try and get some relief and I am getting very frustrated as I tend not to do things either due to the pain or in fear that it will create pain. Cheers, Tim Doctor: hello, the pain in ur ankle or calf could be due to posture (long standing), or eve if u r obese in ur stature could cause u pain, or if there is calcium deficiency can cause pain, or if there is any history of sprain or injury. also if the pain is worse on waking up in morning, advise you to start with calcium supplements under an expert advise. hot fomentation of legs, slight elevation of legs while sleeping at night, sitting cross legged while having ur meals, and few mild exercises may help u. take care. for further queries contact aashirwadhomoeopathyclinic@gmail.com"
},
{
"id": 148053,
"tgt": "What is the treatment for red line in spinal region after injury?",
"src": "Patient: I went skydiving yesterday and landed on my tailbone at quite high speed on landing....the impact was too great for my legs (fierce stinging pain) so without thought I sat the landing....I m moving better but I now have a promonite red line down my crack from the spine region of my lumbar to the tailbone.... Doctor: Hi,Thank you for posting your query.I have noted your symptoms and based on the description, your injury seems to be minor and no important structures seem to have got injured.The red line is possibly due to skin trauma/rupture and would heal on its own in a few days.If there is pain in that region, an MRI scan may be done to look for any slipped disc.I hope my answer helps. Please get back if you have any follow up queries or if you require any additional information.Wishing you good health,Dr Sudhir Kumar MD (Internal Medicine), DM (Neurology)Senior Consultant NeurologistApollo Hospitals, Hyderabad, IndiaClick on this link to ask me a DIRECT QUERY: http://bit.ly/Dr-Sudhir-kumarMy BLOG: http://bestneurodoctor.blogspot.in"
},
{
"id": 8261,
"tgt": "Suggest good shampoo and conditioner for dry and unhealthy hair",
"src": "Patient: Hi Doctor, My hair is dry and unhealthy and thats very obvious by its look. I have tried many shampoos to fix my hair and it gets worse. Can you help me with a good shampoo and conditioner which can fix my problems on hair. After reading a lot on hair products most dermatologist have suggested triflow , my doubt on that is that the triclenz shampoo contains silicon and wouldn t that just coat the hair to good texture rather repairing hair? Please help Me and clarify my doubts. Doctor: Hello, Welcomewell before i discuss everything about hair i just want to say that shampoo and hair oil, condition does not affect health of and growth of hair. These products only affect the appearance and shinyness. So do not spend unnecessarily.1. Up to 150 hair per day fall normally. Old hair go off and new hairs grow in their place. 2. Generalized hair fall in female is mostly due to stress, poor sleep, poor nutrition and excessive blood loss due to menstruation3.Straightening and other procedures have poor effect on hair health and make the hair week in long term.4. Long duration disease like malaria, ptyphoid, or other chronic disease can cause poor growth of hair. Specially after few weeks of starting of disease.Treatment1. Have good sleep. food., green vegetables , fruits2. Take iron and multivitamines like follihair atleast 3 month.3. Check you blood hemoglobine if it is less iron and folic acid supplements should start.4. Check thyroid and diabetes to rule out other cause."
},
{
"id": 66610,
"tgt": "What causes a painless lump at the back of neck?",
"src": "Patient: I have had a small painless lump on the back of my neck tendon area for about 3 weeks. At firstt i thought it was a bug bite but it does not itch and has not gone away. People think im just paranoid and laugh but im ttc and do not want to have any complications if i succeed. Should i be concerned abouy this. Had bloodwork about 4 months ago with my gp and whatever they were testing i think cholesterol ect was all good. Doctor: Hi, If I were your treating Doctor for this case of a nape of neck lump, I would come up with only one and one possibility that is a sebaceous cyst! Nothing to worry about this and try not to get infected there!I suggest you to go for an FNAC test of the lump for confirmation and to relieve your concerns!Hope this answers your question. If you have additional questions or follow up questions then please do not hesitate in writing to us. I will be happy to answer your questions. Wishing you good health."
},
{
"id": 199567,
"tgt": "What causes testicular pain and burning sensation in the penis?",
"src": "Patient: I have pain in my testicles with redness, and a feeling of tightness, and occasionally burning and tenderness in the tip of the penis. This happened once before about six months ago but went away on its own. This may be unrelated, but I also experience tingling and slightly chapped lips both times this occured. I have been to the clinic twice for urine tests to treat a bladder infection and also std tests, came back negative so I don`t know what to do next. Doctor: HelloYou need proper clinical examination and routine investigations.Investigations include routine hemogram,random blood sugar,ultrasound and colour doppler of scrotum,urine RE/ME.Your symptoms may be due to many causes like epididymo-orchitis,UTI etc.Ultrasound of scrotum is important due to redness and tightness.It can assess many conditions in testis.Proper treatment depend upon findings.You may need to take antibiotics.Get well soon.Take CareDr.Indu Bhushan"
},
{
"id": 126547,
"tgt": "What causes insomnia, frequent urination and weight loss?",
"src": "Patient: I as 66 and losing muscle mass, can not sleep,insomnia, up 4 to 5 times a night to pee, losing wait, and turning gray very fast. I walk very slow in baby steps also. Any idea whats going on? I have diabetes, high blood pressure ,psoriatic arthritis and Erectile Dysfunction? Doctor: Hi, This much of information is not complete to answer your question. You could have some prostate problem because of which you have an increased frequency in urination. You could have some chronic infection or some malignancy because of which there is weight loss. All the above points are just a wide guess on the basis of the amount of information you have given. So don't get frightened. Get yourself investigated. Get done USG KUB, CBC, KFT, LFT, blood sugar levels and serum PSA. Hope I have answered your query. Let me know if I can assist you further. Regards, Dr. Anuj Gupta, Spine Surgeon"
},
{
"id": 90512,
"tgt": "What causes abdomen and back pain with white discharge and blood?",
"src": "Patient: Hello doctor, I am Simi, 30 years old. I am 5.4\" tall and weigh 67kgs. I am a spondyloarthritis patient. Recently I hadd my urine test done whihc revealed WBC-10-12/hpf, RBC 30-35/hpf, Blood ++, Leukocytes +. Doctor, what does this report state. I feel very tired all the time. I also get abdomen and back pain at times. When I get these pain, I find white discharge along with some blood. I have copper-t inserted. Thank you. Doctor: Hello! Welcome to HCM.Blood & leukocytes in urine indicate urinary tract infection.As you have mentioned about abdominal & back pain, there can be a stone causing this and acting as nidus for infection.Stone in ureter causes obstruction to urine outflow, smooth muscles in ureter contract excessively to overcome this obstruction & so is the pain.Ca-oxalate stones are quite common & these stones have spikes on their surface. These spikes cause mucosal trauma in urinary tract producing blood in urine.To such patients in my clinic, I advise ultrasound abdomen & pelvis, X-ray KUB, renal function tests, CBC, urine routine & BSL.Treatment is done accordingly.For temporary pain relief, smooth muscle relaxants like drotaverine, pain killer..diclofenac, tamsulosin are prescribed.Hope this helps.Wish you speedy recovery.Thanks.Regards."
},
{
"id": 186962,
"tgt": "Can I get braces that could be removed occasionally?",
"src": "Patient: i hav a gap in my front teeth of upper jaw. I want to cure it with braces but not a permanent braces. I want a braces which can be removed when i hav to attend a party. FYI my none teeth are missing in upper jaw.The gap is almost of 0.2 cm. I am 25 yrs old? How long it will take to cure? Doctor: Hi. Welcome to Healthcaremagic.I read your query. In case of spacing or irregular teeth, they are not treated individually but as entire dentition. I suggest you get an OPG and Lateral cephalogram done and visit an Orthodontist. If rest of your dentition is fine and the gap is the only thing and if it can be closed by their movement only, then your treatmentvwont require permanent braces.But usually this is not feasable as moving a tooth ultimately require whole dentition adjustment. You can go for capping or venners for only gap closure. Or there are transparent braces these days which are less visible. Treatment with fixed braces can last from 6 months upto a year varying from case to cas. And you may need to have a retention wire on inner sidd of your teeth to prevent relapse. This all needs to be decided after proper case study only.Hope the answer helps you. Thank you!"
},
{
"id": 197657,
"tgt": "Suggest remedy for skin problem in penis and testicles",
"src": "Patient: i had a viral flu and was treated with amacxolene[500 mg x 3] and clevimox [625 mg x2 per day]for 10 days. now after 3 weeks little ok. but the testical skin has got shrinked upward and now the penis skin is bulging down under the head of the penis. What am i to do?Wije Galagedera Sri Lanka Doctor: it's likely to be allergic reaction to the drug. so stop the drug. maintain sexual hygiene. if it persists pks get it checked by doctor"
},
{
"id": 223228,
"tgt": "What to do if having continuous periods after getting Nexplanon inserted?",
"src": "Patient: Hi I was implanon for 3 years with no period for almost the entire time, then I had my implant swapped to a nexplanon which I was told would be the same. I had it inserted in September and have had a period ever since- cant stand it anymore. Blood goes from brown/stringy to dark brown/black back to red again. Doctor gave me pills (microygen) which I have been taking for a week no change. Should I just have the thing out?? Thanks Doctor: Hello,Thanks for sharing your health concern with us. It is true that the hormone contained is similar, but, the bio-availability of the drug varies for each preparation. This leads to breakthrough bleeding/spotting which you seem to be experiencing. You may also use styptics to arrest the bleeding. If you find no relief with any medication, if the bleeding is heavy and persistent, you may have to change the contraceptive. In addition to medication, take adequate rest and maintain proper nutrition. Hope this helps."
},
{
"id": 5990,
"tgt": "Trying to conceive, implantation pain, periods started, light headed and drowsy, pregnancy test negative",
"src": "Patient: Hi, I am a 30 year old female trying to conceive. We started trying last month while I thought I was ovulating. The last day we tried was March 28th. On March 31st I started feeling what I assumed was implantation pain. It s been two weeks now, and I still feel it sometimes, although I started my period three days ago. The pain is always in the same spot, towards the left side of my uterus (not in my fallopian tubes ). I was so surprised when my period started because I really felt like I was pregnant . Aside from the uterine pain, I was also feeling lightheaded and drowsy all the time. I took a pregnancy test the day I started bleeding 3 days ago and it was negative. Any idea what could be going on? Doctor: Hello. Thanks for writing to us. An implantation takes place at seven days after a successful fertilisation. The symptoms you had are only presumptive of pregnancy and are not confirmatory. Since your pregnancy test was negative, pregnancy is ruled out. I hope this information has been both informative and helpful for you. Regards, Dr. Rakhi Tayal drrakhitayal@gmail.com"
},
{
"id": 63105,
"tgt": "What causes painful lump in neck?",
"src": "Patient: Hi, I ve had an under the surface lump in my neck for about 7 years now. I went to a doctor when it first appeared and he told me not to worry about it unless it got bigger, then to go back to a doctor to get tests done. So, I just figured since he didn t think it was anything to worry about it wouldn t be anything.. Anyways, I m 20 now and it has grown a significant amount since i ve seen that doctor but I didn t think anything of it until now. It s been hurting on and off for a few months now and sometimes feels like it s blocking my ability to breathe. I ve also started getting a spotanteous, tense pain the in the base of my skull at the back of my neck. As well, If I star at something for a while the object starts to shake. I ve also had a bruise on my shin for about 4 years now that just wont go away. I m not sure if any of these things are connected but if you could help me with any or all of these issues I would greatly appreciate it. I m not the type of person who really likes to go to the doctor as I feel like i m just wasteing their time.. But these things are really starting to worry me and I don t want to be worrying over nothing. Thank you so so much. Samantha Doctor: hi.it is best if you consult with a doctor now, preferably a general surgeon or an ENT doctor. medical and physical examination will be done. diagnostics (neck ultrasound or ct-scan and a biopsy) will be requested if the lesion needs further investigation. management (medical or surgical if indications are found) will be done accordingly.hope this helps.good day!!~dr.kaye"
},
{
"id": 209085,
"tgt": "Suggest remedy for obsessive-compulsive disorder",
"src": "Patient: my younger has been suffering from obsessive-compulsive disorder for the last 10 years. he becomes averse to his compulsory medicine once in a year. we were somehow managing it and restoring him to normalsy. now it is becoming tough to manage him. could you pl. suggest the place where we can permanantly admit him so that he can be taken care of amongst the people suffering from similar ailments. Doctor: My dear friend, ocd is indeed a very difficult disease but it is treatable with medicines and behaviour therapy. Medicines help only upto 20 30 percent but behaviour therapy helps to control the problems. For these treatment you need to visit any mental institution like pgi chandigarh or nimhans bangalore. His problem will take time but he will be managable once treatment starts properly."
},
{
"id": 63173,
"tgt": "What is the treatment for fibroadenosis?",
"src": "Patient: when i was 18(2yeaqrs ago) i had some pain, so i went to the dr, i was adviced a FNAC, upon getting it done the reports showed, fiberadenosis, the dr said it would dissapear in a year or two, these lumps are still present n are very prominent upon touch everyday. what do i do????? what is it? how do i get rid of this?????? i am now 20! Doctor: Hi, dearI have gone through your question. I can understand your concern. You have fibroadenosis. If your lump remains persistent and painful then hormonal treatment like tamoxifen can help you. Surgical excision is the last option. Consult your doctor and plan accordingly. Hope I have answered your question, if you have doubt then I will be happy to answer. Thanks for using health care magic. Wish you a very good health."
},
{
"id": 157100,
"tgt": "How to cure painful bump close to throat roof in mouth? Is it mouth cancer?",
"src": "Patient: I had a sore throat about 4 weeks ago. Actually lost my voice and a lot of mucus. Sore throat has been on and off last 3 weeks and now I have a painful bump closer to my throat roof if my mouth. It s not on the hat palate but close to the end if it. Worried!! Is it mouth cancer? Doctor: HiThanks for your query.it is most likely an inflamed gland in throat mucosa. You should get it examined by ENT so that necessary treatment be prescribed.Hope this helps.Regards"
},
{
"id": 180023,
"tgt": "What should be done after a crocin overdose?",
"src": "Patient: Hi doctor, I ve given my baby with high fever, appr. 15ml of crocin drops in 24 hours as I read the dosage info on the bottle wrongly. He is 11 months old and 11 kgs.... m sure it is overdose...as of now he seems ok but please tell me the signs I ve to look for and also let me know if he should be admitted in the hospital immediately. Doctor: I am glad that you have put your query on HealthCareMagic. I understand your concern about your child. The standard dose of crocin, which is the brandname of paracetamol, is 15 mg/kg/dose. Paracetamol is a very safe drug. The dose of paracetamol in crocin drops is 100 mg/ml and comes in 15 ml bottles. On the other hand crocin syrup/suspension is 120 mg/ml and comes in 60 ml bottles. That means for for your baby of 11 kg, the appropriate dose would have been 165 mg which is present in 1.65 ml of crocin drops or about 7.5 ml of crocin syrup. Please check whether you have used crocin syrup or drops. If you had used the syrup, I do not think there is much to be worried. But in case you have used the crocin DROPS and consumed the entire bottle of 15ml, I think that instead of trying to assess on your own, it would be advisible to take the child to a hospital with a paediatric setup wherein they would be able to assess the condition better.If paracetamol overdose occurs, initially have nonspecific complaints such as vague abdominal pain and nausea. With progressive disease, signs of liver failure may develop; these include low blood sugar, low blood pH, easy bleeding, and hepatic encephalopathy. Some will spontaneously resolve, although untreated cases may result in death. So after confirming whether you have used crocin drops or suspension, i.e. if there is indeed an overdose, take the child to a hospital as urgent attention might be required."
},
{
"id": 123040,
"tgt": "Is replacement surgery necessary in elderly with severe knee pain?",
"src": "Patient: Hello, my father has severe knee pain ( RH & LH) when walking, he is 86 about 30 lbs overweight, but other than that in good shape physically and mentally. His quality of life is declining due to the pain, whould he be a candidate for replacement surgery? Doctor: Hi, The indication of knee replacement is when the person is not able to continue with his daily activities. By this criteria I would strongly recommend knee replacement. Hope I have answered your query. Let me know if I can assist you further. Thank you. Regards, Dr. Gopal Goel Orthopedic Surgeon"
},
{
"id": 113414,
"tgt": "Having severe back pain. Took motrin. Feels dizzy, weak, headache. Biopsy of a thyroid done. What is going on?",
"src": "Patient: this morning i woke up with severe back pain .... the hemoglobin in my last urine was 3+. im thinking the pain was my kidneys, i took motrin and it went a way.. now i feel dizzy and have a headache. feel really weak. what could be going on. im also having a biopsy on my thyroid tomorrow because of a not good looking nodule . i just started synthoid 50mg a day Doctor: Hello, your pain appears to be from the kidney region in view of your 3+ haemoglobin in urine. please update us regarding your thyroid nodule as well"
},
{
"id": 64014,
"tgt": "Suggest remedy for vaginal lumps",
"src": "Patient: i have a round ball shape lump which hangs out my vagina. i can touch it but after its painfull also intercourse is painfull adter and makes me bleed the next day? iam 43 5 foot5 12stone. have bowl incontinence and very weak bladder. my periods are on shorter cycle than usual intercourse will make me bleed the next day . if i touch my vagina it painful after. Doctor: Hi,Dear .Good noon from INDIA.Thanks for the query to HCM.I studied it in all the details put forth by you and understood your concerns.@Remedy for vaginal lumps-In your case I would advise as follows-a-The possibility of Cervical polyp / uterine fibroid hanging from cervix down in vagina-needs to be fixed by FNAC Biopsy and clinical colposcopy of vagina by your ER Gynaec doctor."
},
{
"id": 139088,
"tgt": "What causes subtle crunching sounds from my knee?",
"src": "Patient: Both my knees make very subtle crunching sounds when I climb stairs, although I have no pain whatsoever. I m 60 y.o. and in great shape, engaging in weight training three times a week and riding a stationary bike a mile each of the three days. Should I be concerned about my knees? Doctor: Hi, if you have no pain, than do not be concerned with sounds, these are normal for joints of many people, and are due to the close fluid compartment of the joint and since knee is a big and superficial joint it is appreciated more here"
},
{
"id": 147396,
"tgt": "Braintubercloma with partial seizures. On R-cinex, valparin 500, lacoset 100 with inital dose of trac4. Are these safe?",
"src": "Patient: Sir,my wife is suffering from braintubercloma with partial sizure only two/three times .she is taking R-cinex,valparin 500and lacoset 100 about six month having inital dose of trac4 for four months. Are the antisizure medicine are safe, further there is no sizure after the date of admission to the hospital. name-sarat chandra nayak (orissa) Doctor: In brain tuberculoma anti seizure medicines are usually given along with anti tubercular drugs ,particularly if there is History of seizures.However small the seizure may be,it is always safe to give anti seizure drugs for a long period at least for a minimum of 3 years after the last seizure.With proper dosage and careful monitoring these drugs are usually safe."
},
{
"id": 197493,
"tgt": "What causes nightfall?",
"src": "Patient: Hi, My name is J.K and I am 22 years old. i have been suffering from a strange kind of disease. it has been 3 years, when i wake up early in the morning, everyday I see sperm spots on My paint without having any sex or dreaming such things. there hasn't been a single day without having those sperm marks while i wake up at morning. what is reason i facing this and how can i get well. Does it up any kind of effect after marriage? Doctor: DearWe understand your concernsI went through your details. Please do not worry about nocturnal emission because it is a normal process of your body. This happens because your body throw away the stored semen. Worrying will bring anxiety disorder. Please go for psychological counselling. If you still need my assistance in this regard, please use this link. http://goo.gl/aYW2pR. Please remember to describe the whole problem with full detail.Hope this answers your query. Please feel free to post follow up queries. Available for further clarifications.Good luck. Take care."
},
{
"id": 112195,
"tgt": "Should i be concerned about the sharp pain located in middle of my back?",
"src": "Patient: Hi Dr. Grief... I have been having frequent episodes of sharp pain located in the middle of my back. This happens when I straighten up and sometimes when I inhale. I don't really relate the pain to the inhaling... It just so happens that I inhale after trying to straighten my back. This happens when I'm sitting down or standing up... but more so when I'm standing. Should I be concerned? Doctor: HIThank for asking to HCMI can understand your problem, this may be a muscular pain initially I would advise you to manage it with the muscle relaxant and the drug of choice for this are \"chlorzoxazone with Acetaminophen\" you can even try topical gel for this and \"Diclofenac\" will give good results I sure this will come around with this and it fails to respond then it has to be investigated, hope this will help you have nice day."
},
{
"id": 201237,
"tgt": "Can small hernia style tear on pelvic area cause squeezing pain in testicle?",
"src": "Patient: Asking this question for my husband. He was told he has a small hernia style tear lower pelvic area. No symptoms then. He is experiencing a squeezing pain in his left testicle. No pain urinating. Has a history of Kidney stones. Is any of this related to this pain or if not what else may be causing it. No recent accidents. Doctor: Hi,From history it seems that he might be having indirect hernia entering in scrotal sac giving this squeezing pain in scrotal sac.Consult surgeon and get physically examined.This will give clear idea of having this problem.Ok and take care."
},
{
"id": 11946,
"tgt": "Hyperpigmentation on hands and legs",
"src": "Patient: i have big black patch on left hand on upper part the same on left thigh at middle part. it was not my birth mark but when i was 5 yrs old it started with a small pigment and spread to large extent. different doctors have different views,,,,,some say its incurable,its GOD s gift...some say it can be treated... some say its blue skin disorder . till now there is no such ill effect other than pimples on patched hand. could u tell what is it? i can send u my sample pic if needed..... Doctor: hello, i would appreciate it if you can send a picture. its most likely what we call becker's nevus but i cant tell you for sure till i see it. does it have hair on it too?"
},
{
"id": 129695,
"tgt": "Suggest treatment for injury mediated head bump",
"src": "Patient: Hi, I hit my forehead on a pole today while I was changing for pe. I didn t get any concussion symptoms except some pain right after I hit it, but the pain went away in 2 minutes. There s a small but noticeable bump on my head, what do I do to make it go away faster? Doctor: Hi, there!Immediate ice aplication is the best way to avoid haematoma (bump) formation! You can apply heparin based gel now.Take a rest for 3 days to monitore Yourself, because 2nd day after injury reveals true condition.Avoid warm on injured skin.I hope this helps"
},
{
"id": 30635,
"tgt": "Is it advisable to use a lipstick that has been exposed to cold sores?",
"src": "Patient: Hi, i want to know if i should dispose of my makeup like lipstick and foundation and facial powder that has come in contact with exposed cold sores as to not spread the virus or have it to be continuous although being treated or reoccuring. What do you think? Should i purchase new makeup or is it safe to use what i already have? Doctor: Hello,I can understand your concern. The lipstick and make up that may have come in contact with the sores of herpes simplex virus may be contaminated with the virus that may infect you again after you have been cured from the disease.Thus, my advise to you is to not to use the make up that may have come in contact with the cold sores of herpes simplex. In addition, if possible, do not use make up on the sores as the chemicals may irritate the sores.I hope this answer helps you. Thank you for choosing HealthcareMagic. Take care.Best,Dr. Viraj Shah"
},
{
"id": 104178,
"tgt": "Have rash on back, neck, itchy, goes taking antihistamines. Have various allergies, but nothing introduced",
"src": "Patient: Hi - for the past 2 weeks, I have on 4 occasions woken up to a rash on my back and the back of the neck. It is slightly itchy and goes away quite quickly with antihistamine. I have several food and airborne allergies but have not had any new foods or introduced any new to my bedroom so I don't know what causes this occasion rash. Doctor: can be dampness of new bedroom there are fungal increases on exposure to damp environment apply candid ointment bd for 4 week take ebastine 10 mg night for 3 week see the result anf if recurs again and agin take consultation fom allergeologist and can ask again in this form keep skin dry no use of oils on area keep airy"
},
{
"id": 124193,
"tgt": "How long does it take for foot-recovery post bunionectomy?",
"src": "Patient: Hello i am going into two months now after Bunionectomy surgery everything seems fine and I have a appt tomorrow I am excited to see the xray because the last one the bone was not yet close up enough but I am just curious to know about how long does it take before are at least your foot is normal again Doctor: Hello, As the surgery is not that big, you should be discharged 2 days post surgery and dressing for up to a week depending upon the wound healing and that's it. You should do well. To regain full function of the foot after the wound healing is completed maybe 10-15 days it may take as the pain will take some time to come down and muscle strength will return. Hope I have answered your query. Let me know if I can assist you further. Take care Regards, Jay Indravadan Patel, Physical Therapist or Physiotherapist"
},
{
"id": 11739,
"tgt": "Had cystic pimple on forehead. Suffering from hyperpigmentation scar. Tried microdermabrasions, chemical pill, hydroquinone. Suggest best treatment?",
"src": "Patient: Hi I am 36 years. I had a cystic pimple on my forhead which I don't remember picking. But after the pus, some blood oozed out and left a dark brown hyperpigmentation scar. The brown scar is slightly raised. I have a tendency to peel thinking the skin underneath will be of regular colour but each time it darkens again with slightly elevated skin. Ive tried microdermabrasions, chemical peels, hydroquinone since 10 months in vain. Whats the best method to treat hyperpigmentation pimple raised brown scar? Doctor: Dear Lola, Hi and welcome to HCM. It appears from your query that you are overdoing your efforts to get rid of the scar. You know our skin needs bit of rest to get its normal texture. when we are doing a lot of things then it actually cause s more of post inflammatory hypermelanosis/ post inflammatory hyperpigmenation (PIH). Other wise also from your query it appears to be two problems- the one is Scar and the other one is Pigmentation. You might get some relief in pigmentation but to get rid of scar is really difficult one. The very first thing you should do is - kindly stop doing any thing for a period of at least 3-4 week. As it will come out i its natural shape so would be better to be analyzed for pigmentation. You can try use azelaic acid cream, Kojic acid cream, glycolic acid cream, licorice extract or arbutine under medical supervision. The last option would be Fraxel Laser by an expertize hands who can judge your situation and can go for it accordingly. Hope this information will help you out of your problem. With best skin wishes, Dr Sanjay K Kanodia"
},
{
"id": 45585,
"tgt": "Are high levels of creatinine and blood sugar signs of renal insufficiency?",
"src": "Patient: Hello, I am 52 y.o. white female. Recent lab results show my Creatinine level at .93 mg/dl. My Dr. is saying I have renal insufficiency; my a1c is 5.4 and fasting blood sugar was 100, he says I am pre-diabetic, I feel he is being a little too aggressive with diagnoses? Doctor: Hi, If blood sugar level goes above 100 and below 126 than it is called pre diabetic. I am not in favour to label your case as pre diabetic. However you need to get checked your fasting and post prandial blood sugar again after few weeks to check control in sugar. Meanwhile take low fat and low sugar diet with active exercise. Your creatinine is within limit. Hope I have answered your question. Let me know if I can assist you further. Regards, Dr. Parth Goswami, General & Family Physician"
},
{
"id": 118370,
"tgt": "What are the non-serious causes of elevated levels of B12 and Folate?",
"src": "Patient: My blood tests showed elevated levels of B12 and Folate. what are some non serious causes of this? Everything I read says leukemia or liver disease. This sucks that only after the patient has posed the query does she discover there is a fee. Individuals, as you should know, who use this sight are already fearful and anxious. Doctor: Serum b 12 and folic acids both are water soluble and usually can not be deposited unless they are produced in very high rate such as in leukemia. But reports can be wrong too. Never hear any other medical cause which can cause increase in these two drugs."
},
{
"id": 180625,
"tgt": "What do soreness and swelling on the tongue indicate?",
"src": "Patient: In the last couple of days my tongue has become sore and swollen and has now has swollen sections that are whitish yellow. The interior of my mouth is now becoming more sore and sensitive and I have been unable to eat and now drink.I need to know what this is and how to rid myself of it Doctor: Hello and Welcome to \u2018Ask A Doctor\u2019 service.I have reviewed your query and here is my advice.Sore swollen tongue with yellow white patches can be due to yeast infection or oral thrush. Consult an oral physician and get evaluated. A microscopy of scraping from tongue can help in confirmation of the diagnosis. If it is thrush then antifungal medicine like Nystatin mouthwash, Clotrimazole lozenges and oral medicine like Fluconzole can be advised for 15 days. Eat plenty of probiotics like yogurt and maintain a good oral hygiene.Hope I have answered your query. Let me know if I can assist you further.Regards,Dr. Honey Arora"
},
{
"id": 117932,
"tgt": "How to control ESR levels?",
"src": "Patient: Hi, I am female and when I got my blood tested it is found that, ESR level is 30 where as the normal level mentioned is 0 to 20. I want to know the problems of excess ESR and remedies for the same. Thanks and regards, my email address is YYYY@YYYY Doctor: your esr value is higher then normal. but there are many cause for that. they are simple infection or inflammation, anemia, tb, autoimmune disorder and many more.so just by rasied esr we can not know the defitite cause. so first search for the causee. pproperly investtigate and then take treatment accordingly"
},
{
"id": 178126,
"tgt": "What causes frequent stools?",
"src": "Patient: Hi Doctor,My daughter is 5 months 26 days old. she is on Enfamil A+ stage 1 since she was four months. 10 days back she passed semi solid d stool 7 times. Our doc OFM susp BDPC for 5 days and Zerolac for two weeks. But she refused to drink Zerolac summarily. So, after consultation with our doc we restarted Enfamil A+ stage 1. But, today she is again passing frequent stools as before.Kindly advise what to do...Thanking in advance... Doctor: Hi , I'm Dr Suresh K Yadav M D (paediatrics) , I have gone through your question and understand your concerns,Passing stools 7 times a day can be normal in a 6 month baby , if she is active, accepting feed , passing urine adequately , gaining appropriate weight . She is almost 6 months you can start weaning foods like curd , meshed banana , khichdi, suzi with low lactose burden that help in less stools .Hope this answers your question. If you have additional questions or follow up questions then please do not hesitate in writing to us. I will be happy to answer your questions.Take care.Thanks."
},
{
"id": 207043,
"tgt": "What causes fatigue,tremors and irritability?",
"src": "Patient: Hi, for about 2 months now ive been constantly weak, shaky and very irritable. i went to the doctors last month and i was sent for a blood test. i was tested for 5 different things, cant remember them all but i know i was tested for diabetes, thyroid, bone?cant remeber the rest but everything came back fine. Doctor: Hello,If all the investigations are normal, the problems you have mentioned could be the part of anxiety disorder or depressive disorder. You need treatment on the lines of these disorders.Thanks."
},
{
"id": 210081,
"tgt": "Should I be worried for increased heart rate and anxiety attacks?",
"src": "Patient: I'm a 33 year old female, 5'2\" and 145 lbs. I was diagnosed with MS in December 2009. Since a month after having my third son in January, I've been having panic/anxiety like attacks. My resting heart rate right now is about 94. Should I be worried? I don't normally check my pulse rate but with the attack I'm having now, my heart feels like it's racing. Doctor: HIThanks for using healthcare magicPalpitation and increase heart rate is part of anxiety. If you take tension about these symptoms, then it would increase more. In that case, better to stay relax and take few antidepressants after psychiatrist consultation. You can also try relaxation exercise that would keep you relax.Thanks"
},
{
"id": 47379,
"tgt": "What causes pain in kidney area inspite of removed catheter?",
"src": "Patient: my husband just had his catheter removed today. Now, when he urinates, he has extreme, deep pain in his right kidney area (only while urinating). Not sure if he is urinating enough. He has a stent in his ureter and the doctor says that because the ureter is not able to close because of the stent, that the urine is pushed back up into the kidney creating the pain. Wondering if he should have kept the catheter in while he has the stent? Doctor: HelloThanks for query .Your husband has undergone surgery (Probably for stone in his ureter ) and a D.J Stent has been put in his ureter .It is common to get pain in kidney area as the urine flows backwards through stent in to kidney causing pain in kidney area while urinating .Since the stent has to be in the ureter for 2 weeks the patient can not be put on per urethral catheter to drain the bladder as there is high risk of getting UTI due to catheter itself .The catheter in urethra is more cumbersome than pain due to having stent in ureter .Normally this gets resolved within a week till then take pain killers and antibiotics .Dr.Patil."
},
{
"id": 111094,
"tgt": "What to do for spondylosis and muscular chronic back pain?",
"src": "Patient: Hi, I am a 51 year old female and I have spondylosis without myelopathy and muscular chronic back pain, can I claim for any assistance like DLA as I find the simplest of things very painful, i.e shower bathing, drying my hair and cutting food ? I would be very grateful for your advice, thank you Doctor: Hello,I had gone through the case and found that you must go for proper blood test like ESR, CRP, RA Factor ane uric acid.Go for MRI of spine and shoulder joint and fingers.After the proper diagnosis take proper treatment.Along with do yoga and exercise.Take Vitamin D3 and calcium rich diet.Hope my answer will be effective for you.Thanks"
},
{
"id": 67831,
"tgt": "What does a small lump on head indicate?",
"src": "Patient: My mom has a small lump on her head underneath the skin. It doesn t move on it s own but if you take two fingers and press a tiny bit, you can move it. It s like a little rock under her skin. What is it? Does it sound like she needs medical care immediately? Doctor: Hi,This is most likely to be a sebaceous cyst. They are commonly found on the scalp and are not serious. She does not need to seek medical attention unless it becomes painful or swollen which would indicate infection. Regards,Dr K A Pottinger"
},
{
"id": 163635,
"tgt": "What causes red spots on the body of a child?",
"src": "Patient: hello,My son who is truning 5 next month has these red spots under his arm has been there for 3 days now at first i thought it s a rash but today when i had look at it it has increased and more down towards his body?just wanna make sure is it chicken pox? Doctor: you need to consult doctor and show him red spots..chickenpox, measles, and many other disease can be present like this."
},
{
"id": 53832,
"tgt": "What causes vomiting in case of liver cirrhosis?",
"src": "Patient: my husband was diagnoise with cirrosis liver but now hasd thombosis swelling in the legs wel a vasculr doc said it appears he did have a large blood clot at his ankle on right leg that moved up and has caused severe damage to veins. that it explode in his stomach So we don;t know if it is cirrosis or the blod clot making him throw up but they said he was almost dead reaching the hos, from loosing so much blood can you help me at all? Thankyou very much my number is 678-545-0893 if you can call Doctor: cirrhotic patients do have a prothrombotoc state, meaning they have a tendency to develop blood clots. But the clot in his leg can NOT explode!What happens is, as the liver shrinks, the flow of blood from veins from the stomach slows down, causing them to swell up. This is the site of bleeding in such patients."
},
{
"id": 78053,
"tgt": "What causes feeling suffocation around people?",
"src": "Patient: hello, my age is 24 years old and my name is dipika.Whenever i will be in crowd i feel like suffocation, i don't understand if its my crowd fear or some breathing problem....whenever i will be in crowd i feel like blood is circulating very fat in my body.can it solve by medicines??which medicines? Doctor: Thanks for your question on Health Care Magic. I can understand your concern. By your history and description, possibility of Enochophobia. It is fear of crowd, crowded space, social gatherings etc. So better to consult psychiatrist and get done counselling sessions. Discuss your problem with him. Along with counselling sessions, you may need drugs like anxiolytic drugs, beta blockers etc. Don't worry, you will be alright. First consult psychiatrist and discuss all these. Hope I have solved your query. I will be happy to help you further. Wish you good health. Thanks."
},
{
"id": 204249,
"tgt": "What causes low pulse rate while on an anti depressant?",
"src": "Patient: I am currently on 2 bp meds and an anti depressant. I am also morbidly obese. Two months ago I started weight watchers and have lost close to 20 lbs. I also got a fitness tracker since I started going to the gym about 3 weeks ago. My question is it shows my resting pulse to be 52 bpm. Is this to low and should I talk to my doctor about maybe taking me off one of the bp meds? Doctor: Hello and Welcome to \u2018Ask A Doctor\u2019 service. I have reviewed your query and here is my advice. Its not very common with commonly used antidepressants to cause such low pulse rate. This could be due to blood pressure medicine. Propranolol, Atenolol etc are known to reduce the pulse rate. You should talk with your doctor for re-evaluation. Hope I have answered your query. Let me know if I can assist you further."
},
{
"id": 205352,
"tgt": "Can stress cause black eye?",
"src": "Patient: Can stress cause a black eye? woke up with a black eye I thought it was due to new medication so i stopped it and black eye went away. Now it looks like another black eye is starting on other eye. Involves the eye lid and radiates to outside eye lid to inside near nose. Should i go to an eye doctor? Doctor: Hi, it was not due to any medication. u shall consult any ophthalmologist for blacking of eye, eyelid and nose.thank u"
},
{
"id": 60654,
"tgt": "How to get back the normal level of SGOT and SGPT ? I was taking antibiotics for chicken pox",
"src": "Patient: Dear Dr. I am a 31 yrs old male.Last week i went for a medical check up and my SGOT is 78 and SGPT is 102.I was taking antibiotics for chicken pox one month back.Could you please help me how can i bring down them to normal level?or whether i need to go for an ultra scan?Thank u very much... Doctor: unless the spot and spgt are 3 times the normal range you have nthing to worry about including hepatitis. Kindly keep a check on yellowish discolouration of skin and urine. Get yourself tested for hepatitis B."
},
{
"id": 142228,
"tgt": "Suggest treatment for getting shock after touching metal railing",
"src": "Patient: this is going to sound stupid but im very worried i was eletricuted the other day i recvied help but today i touched a metal railing and a ark of light shocked me i was knocked to the floor and after the railing was boiling my friend toched it and got shocked Doctor: Hello!Welcome on Healthcaremagic!I understand your concern and would explain that it seems that you have been in contact with an electrical shortcut. As you are feeling better now, there is nothing to worry about! It is quite normal to faint after an electrical shock, but it seems to have caused no damage as you have not physical signs now. So, just try not to think about it any more!Hope to have been helpful!Best wishes, Dr. Aida"
},
{
"id": 221650,
"tgt": "Why am I not pregnant as yet?",
"src": "Patient: age 23 years female married respected mam 6 months have been passed being married .still im not pragnant.my menstrual cycle from april is below. 27 april 6 june 19 july 7 sep 12 oct am i suffering through irregular cycles? plz help me how can i be pragnant Doctor: Hello dear,I understand your concern.In my opinion after seeing your dates of your cycles they seem to be irregular.The causes for irregular cycles like thyroid abnormalities,PCOD,overweight or obesity or stress need to be ruled out.Ultrasound and thyroid profile are helpful to diagnose the above conditions.Weight reduction in case of overweight or obesity is helpful.Avoid stress.Best regards..."
},
{
"id": 168966,
"tgt": "What causes pain while urinating?",
"src": "Patient: My son is about 7 weeks old, he does not cry when urinating or fuss when he does. He did however wake up this morning with what looked like a tiny bit of blood in his urine. Should I be taking him to see his pediatrician or to the ER? Or will he be fine? Doctor: a blood in urine could be urethritis as long as it's fresh blood. check his penis for any injury- retract the prepuce to see and if it recurs only see the doctor and not to get panic."
},
{
"id": 110094,
"tgt": "What pain relief should be taken for chronic back pain?",
"src": "Patient: I thought this was a free service My script for hydrocone 10/325 and xanax .1 were ready for refill today. I called office and they said I was Fired, thus they won t refill. Said they weren t medically nesnesary. I have been on hydros for 20+ years. Saw a rhemetologist 2 weeks ago and he said I needed paain relief for chronic back pain. I have herniated and bulging disc in lumbar. Thay can do this the Eat they were due for refill? I aaalso bave R.A. Both shoulders have had rotator cuff surgery. Both knees needing surgery according to gov t Dr. who claimed me disabled. WOW Doctor: Hello, I have studied your case.Your MRI says disc protrusion which causes compression of exiting nerve root which later on supply right lower limb [leg].Due to compression of this nerve root there is tingling numbness in your leg and pain associated with it.Medication like methylcobalamine with muscle relaxant and analgesic will reduce pain; you can take them consulting your treating doctor.You may consult physiotherapist for further guidance. He may start TENS, or ultrasound which is helpful in your case.I will advise to check your vit B12 and vit D3 level.You can take hydrocodone from online pharmacy.Hope this answers your query. If you have additional questions or follow up queries then please do not hesitate in writing to us. I will be happy to answer your queries. Take care."
},
{
"id": 95311,
"tgt": "Why do I have stomach pain while passing motions ?",
"src": "Patient: When I poop it burns... And having stomach pain... And my stool is like a yellowish color ... I really don t know what to do about this... Should i go to the ER? Doctor: Welcome to Healthcare Magic You are probably having a bowel infection. Have you noticed any blood in the stools. It would be best to go to ER and get your stool tested for infection. Also get examined for anal fissure or similar pathology which can cause burning besides the acidic nature of the stools which can also irritate the anus causing burning. Drink plenty of water. Keep hydrated. Eat well washed fresh fruits and vegetables."
},
{
"id": 113639,
"tgt": "Periodic backache, difficulty in moving leg due to stiffness, temporary relief with Etoshine, done MRI, spondylosis and arthritis test. Suggestion?",
"src": "Patient: Hi I am suffering from Back Ache for quite a long time. This pain comes and goes on its own. Some times this pain gets so severe that I am unable to meve my leg an inch because of this pain and stiffness . I feel joint pain in between my back reebs also. Doctor has prescribed me ETOSHINE 60 for pain relief which is working well but on stopping this medicine pain comes back. Conducted MRI , Spondylitis Test and Arthrities test but all result was good. There is no sleep Disc also. Please suggest Deborshi Chatterjee S/W Engineer Doctor: hello Deborshi, welcome to HCM as u have mention about ur pain its due to spasm of muscles.. ur all the tests are also normal.. ur profession is software engineer which makes you to sit for a long period in front of desktop.. 1st you need some ergonomically change at your workstation.. 2nd you need to do some exercises to keep ur back strong enough to avoid the back pain.. so that u do not need to take pain killers.. stay fit Deborshi.. consult a good physiotherapist in ur locality to learn proper exercises and few precautions... u can mail me on sidharthphysio15@gmail.com so that i can guide u about ergonomical changes in ur workstation and exercises also.. take cae thanks"
},
{
"id": 7313,
"tgt": "why is still my foliculos increased?",
"src": "Patient: I am taking humog 450 but still my folicules are not increased. why? Doctor: hello, Follicular development is judged by the concentration of oestrogen, measured in blood or urine. Clinical assessment of the response including pelvic examination and cervical mucus studies should also be performed. Humog administration should continue until an adequate oestrogen level is achieved. did your doctor prescribed you hucog. it is given on the day of the urinary oestogen peak or the day after the plasma 17B -oestradiol peak.If the oestrogen values are less than either 180 nmol/24 hr. (50ug/24 hr) for tested urinary oestrogen or 1100pmol/L (300pg/ml) for plasma 17B-Oestradiol follicular development may be inadequate. its better to consult your doctor for the cause of follicular indevelopement.take care"
},
{
"id": 106624,
"tgt": "How can pain in the lower back and hips be treated?",
"src": "Patient: I have lower back and hip pain. I have been to the chiropractor several times this past year and still have the pain. My wife is a physical therapists and thinks it might be Sciatica. I have been doing some stretches but only obtain temporary relief. Doctor: Hello and Welcome to \u2018Ask A Doctor\u2019 service. I have reviewed your query and here is my advice. Yes, it may be sciatica. You should get an X-ray of lumbar spine done. Maintain proper posture while sitting, standing and sleeping. Sleep on hard bed. You may take analgesic as tablet aceclofenic and muscle relaxant as thiocolchicoside or tizanidine. As these are prescription drugs please contact your local doctor he will prescribe these. Hope I have answered your query. Let me know if I can assist you further. Regards, Dr. Varinder Joshi"
},
{
"id": 102067,
"tgt": "How to treat whiskey mediated red patches on hips?",
"src": "Patient: Hello doctor, I have a concern about about red long patches that appear on the two sides of my hips. They are not extremely itchy but yes some times I feel like scratching a bit. Also I have observed that these patches occur only when I have had alcohol, specifically whiskey. It's happened thrice. I was wondering if this might be a serious condition or just an allergy to whiskey. Also only this time I'm feeling dizzy. Though I tthink it might be due to lack of sleep after drinking. I've slept for 3 hours only today. I would really like to know the opinion &precautions to be taken or advice you may have about future alcohol consumption in parties. Thank you. Doctor: as per your symptoms you have allergy to whiskeythe symptoms are allergic dizziness is due to patchesthe medicine to control it is to use anti allergic tab like fexofenadine apply anti allergic ointment like bello methadonethis will clear it answer to second question is yes you should avoid drinking in parties as elimination of allergen is best option to prevent allergies"
},
{
"id": 128964,
"tgt": "How to treat swelling and pain in the left arm caused due to sprain?",
"src": "Patient: i was doing a a lot of digging in the yard lat week. the muscles on the top of my left arm began to swell and hurt its been aa week and now the pain is much worse and when i hold the area with my other hand and move the arm that hurts it feels like a rubber band or rubber ball moving around Doctor: Take proper rest, you can take analgesics. Better to have a sling arm Support so that your joints get rest and therby forearm."
},
{
"id": 40503,
"tgt": "How can infertility be treated?",
"src": "Patient: Hi Doctor am beza, my baby is 4 years old ,naw i need 2nd chance baby, but i cant get pregnancy i try for more than two years, but there is nothing, i take pregnancy treatment clomad 50gm by my doctor for 2 times but the 2nd clomd delay my ministration for tow month until know what shall i do? doctor Doctor: Hello and Welcome to \u2018Ask A Doctor\u2019 service. I have reviewed your query and here is my advice. Clomid medication would help to induce ovulation. Since your period has been delayed, I would advise you to take a urine pregnancy test first. In case it is negative, then you should be evaluated to check for patency of Fallopian tubes. Kindly consult obstetrician in this regard for clinical evaluation and appropriate management. Hope the information provided helps. All the best. Regards, Dr.Ashakiran.S."
},
{
"id": 100066,
"tgt": "What causes severe asthmatic attacks post Symbicort withdrawal?",
"src": "Patient: I was taking Symbicort for 2 months, and my asthma was the best it has ever been ever. Prior to that I used to wake up every night from asthma having to take my ventolin. I never needed to use my ventolin since I was on the Symbicort which was great. But I was a bit silly and didnt realise my symbicrt inhaler ran out, so my asthma got really bad to the point where I couldn't go to work, and when i took my ventolin it appeared to no longer work on me. Could I be having withdrawals from the Symbicort? Or could it be that I hadn't taken the ventolin for so long that my body doesn't want it anymore? Doctor: Hello,Thank you for asking at HCM.I went through your history and would like to make suggestions for you as follows:1. To address your main queries - withdrawals from Symbicort? - one can say that in common man's terms, but actually it is not withdrawal in scientific sense. (as explained in No 2).Body does not need Ventolin anymore - it is not a correct interpretation.2. Asthma is a chronic disorder. Most of the \"controller drugs\" for asthma (such as Symbicort) actually only \"control\" the symptoms of asthma but cannot \"cure\" asthma. So when one stops such controlled medications, one experiences return of asthma symptoms.3. Ventolin is a \"rescue\" type of drug. It is to be used only for symptom relief when symptoms occur. However, frequent intake of such medications is not desirable, so in that case one needs to take \"controller\" type of inhalers on regular basis. This is as happened in your case.4. I usually my such patients to take minimum dose of \"controller\" medications regularly that controls all their asthma symptoms. For example, I would suggest my patient regular inhaler (like Symbicort) and montelukast. In addition, I would suggest him to take salbutamol inhaler on as-and-when-needed basis.5. Were I treating you, I would also suggest you allergy testing for common air-borne allergens like house dust mites, molds, pollens, insect proteins, pet dander (if you have pet) etc. This will help you identify substances causing allergies to you as well as to know the measures to avoid them.6. Based on allergy testing, an Allergist-Immunologist may prescribe you allergen specific immunotherapy which works gradually on immune system to improve allergy symptoms on a long term basis.7. Regular breathing exercises and a healthy nutrition will also help you in a long run by improving your lung capacity and immunity respectively.Hope above suggestions will be helpful to you.Should you have any further query, please feel free to ask at HCM.Wish you the best of the health ahead.Thank you & Regards."
},
{
"id": 174050,
"tgt": "What do hard lumps on forehead of an infant indicate?",
"src": "Patient: Hi today I found 2 small hard lumps on my 22 month olds head. One is at the front on his hair line just right of the middle and the other one is just to the left of his crown. The one at the front is hard and the one at the back is a little softer. I m really worried as they was not there 4 weeks ago when he had his hair cut. Any advice? Doctor: Hi...by what you quote I feel that they cold be dermoids. The other possibility is that they could be lymphnodes. Unless they are bothering them, nothing to worry about it.Skin conditions are best diagnosed only after seeing directly. I suggest you to upload photographs of the same on this website, so that I can guide you scientifically.Hope my answer was helpful for you. I am happy to help any time. Further clarifications and consultations on Health care magic are welcome. If you do not have any clarifications, you can close the discussion and rate the answer. Wish your kid good health.Dr. Sumanth MBBS., DCH., DNB (Paed).,"
},
{
"id": 49833,
"tgt": "Having colon infection. Not responding to antibiotics. Swelling on the feet due to kidney failure. Will the swelling spread to the upper body?",
"src": "Patient: My Aunt went in Hospital a month ago for a colon infection. Antibiotics did not work. She was put on La six for the retaining of fluid. She is now had here kidneys shut down. Glassy eyed, very small amount of modeling on her feet . She is given 10 mgs of morphine every 4 hours for the pain. We were told she had 24 hours, but that was 2 days ago. How much longer can she linger like this and if she is showing some modeling how fast will it take to move up her body? Doctor: Hello madam/sir,Your problem of swelling of the feet needs evaluation. It could be due to protein leak in the urine. The chances that the edema will progress upwards would depend on the amount of protein leak or degree of renal failure.Kindly get a serum creatinine, 24 hr urine protein, urine routine examination done and revisit your physician.Hope i was of help."
},
{
"id": 153731,
"tgt": "What is the life expectancy with kidney failure and spinal tumor?",
"src": "Patient: Hi, my friend has klippel feil syndrome with a lot of complications including ongoing acute kidney failure, heart abnormalities, spinal tumor, and water retention issues. Im afraid her life will be cut short if the doctors cant do something, she is only 40. Please help Doctor: Hi,Thanks for writing in.1. The ongoing kidney failure requires to be analyzed from the stage of failure she is in. If it is stage 1 or 2 then she can maintain kidney functions for few years. Since she is 40 years old, a renal transplant might be thought of but this needs to be discussion with her other health conditions.2. Heart abnormalities are important to be corrected. There can be heart abnormalities due to valve defects or electrical rhythm disturbances. Some of these might be treated with medicines and when there is severe disease then surgical treatment might be suggested depending on the cause of heart failure.3. Spinal tumor must be investigated in detail to know if that is a benign or malignant tumor and its location. Tumors localized to a particular area and causing pressure symptoms need to be surgically operated for treatment of associated discomfort. Water retention might be due to kidney or heart disorder requiring treatment. Please do not worry."
},
{
"id": 173219,
"tgt": "What causes severe leg pain while urinating?",
"src": "Patient: My 8 year old daughter is complaining of severe leg pain when she urinates, left upper leg. She can feel it burning at the onset of the stream and escalates from there to a point where she needs help from the toilet. As soon as she stands up she feels better.... Doctor: From your concern, I would initially ask for a urine Routine examination to see if there is urinary infection. and do culture to start proper antibiotic. But it is also possible to think of ( though uncommon ) a small ureteric calculus ( stone ) could be lodging in the ureter and it would be usually passed on while urinating. I hope I have explained with reasonable information I got fro your history. You could discuss with your doctor and decide. Regards."
},
{
"id": 173553,
"tgt": "Suggest treatment for vomiting in an infant",
"src": "Patient: My 5 month old son has been projectile vomiting for over 30 hours now after every time he eats. We have given him pedialyte as we were told to do by the ER Dr yesterday evening when I took him to the children's hospital. He is even puking that up. He just puked up a yellowish green mucus looking substance, and the past 5 times he has vomited he has gone limp. He seems to have less muscle control to hold his body upright. What should I do? Doctor: HiWelcome to the HCMI understand your concerns. Persistent vomiting episodes can be due to viral gastroenteritis but it may be something serious such as meningitis as well.Give him Syp Domperidone or ondansetrone for symptomatic relief. If you feel that he is limp, poorly responsive and not accepting anything orally, then immediately go to ER in your trusted and near by hospital which has pediatric emergency services available. They will give him injections to stop vomiting and improve dehydration. Also, laboratory work up will be done if required after complete physical examination.Hopefully this will help you. In case of any further questions, you may contact me.Take care"
},
{
"id": 185642,
"tgt": "How to get rid of soreness and white coating like on my tongue?",
"src": "Patient: I've been snorting and occasionally smoking meth for about a month or so and I've noticed that I have sores and a coating of white on my tongue and my tongue is actually sore. Also I think I've been biting my tongue maybe when I sleep cus I grind my teeth really bad.. Anyways I want to stop the sores and the biting are there any ways to make things better on my mouth? Oh and the fucking acne... I've never had acne ever my body is smoother than a babies butt and I've been breaking out on my back and arms and even on my legs a little. What does that mean and how do I stop that? Doctor: Hello:)Thankyou for posting in HCM.Quit smoking immediately.This burning can be due to smoking as well.You did not mention if the white coating on tongue is scrapable.I suggest you to consult your dentist for a clinical evaluation.If necessary, you must go for a biopsy, to check if it is a lesion or just bacterial growth.Maintain good oral care.Brush,floss and mouthwash.You did not mention if you are stressed. stress and worm infestation most commonly leads to night grinding. You can use albendazole if you suspect worm infestation.Ask your dentist for a night guard.This will dissepate the grinding forces.Acne can be because of hormonal imbalance.See your physician reg. this.Apply quadragel ointment thrice daily on the tongue ulcers for relief.Regards."
},
{
"id": 43245,
"tgt": "Not conceiving, advised for ETS 200 to continue for conception. Should have sex or not?",
"src": "Patient: Hello sir, My wife is not conceiving. I am treating her to a gynaecologist , Her periods circle is not properly after 30 days I went to gynaecologist to see her. She advised for ETS 200 to continue for 7 days for conception During 7 days. May I regular sex or not with my wife. Please suggest me. Thanking you. I am from kanpur Nagar. Doctor: Hi,You can have intercourse during these days. No problem. You dont have to worry about any side effects of the same.Regards"
},
{
"id": 118053,
"tgt": "How to confirm of being hepatitic?",
"src": "Patient: i have a blood in the emergency room and the physician test me for complete blood count +automated diff the sickle screen was negative,basic metabolic panel was good and my hepatic panel is normal the lipase serum is 23 and it range was [<=61 u/l] and finally acetone,serum qualitative is negative can i still have hepatitic Doctor: If Hepatic penal like ast, alt, alkaline phosphatase and bilirubin is normal then most probably you are not suffering from liver disease"
},
{
"id": 213080,
"tgt": "Starting with encorate chrono for epilepsy. Why am I shivering and unable to concentrate on work?",
"src": "Patient: Hi sir, I am a degree student,i had started my medicine course from 2007..starting with encorate chrono 200 for epilepsy ,but gradually my condition got worse,and started to get shaked up..then i started having encorate 300 along with levipil 250..after 2-3 years..i started getting shivering problems with ma hands ,my hands do shiver when i do each and every work..I dont know y this shivering came..and i had found that..its more seen when iam concentearting and working..when i get tensed ..its found morre..what should i do..should i continue with the same medicine??is ther any side effects for these medicines..please help me out..?? Doctor: hi..well, this may be a form of drug-induced tremor possibly arising out of valproate..anxiety may also be a cause..whatever is the case, you should consult with your neurologist regarding proper dose-adjustment of the drugs you are taking..you can also repeat your EEG again if you are seizure-free for a considerable period..have a good health..thanks.."
},
{
"id": 21714,
"tgt": "What is the treatment for low Blood Pressure?",
"src": "Patient: Is there any thing you can do to raise low blood pressure? I know someone who looked as if they were having a seizure but turned out to be the body's response to low blood pressure and the fact she needed to lie down. She passed out and eventually came to. Doctor: Hi ThereLow blood pressure if causing symptoms can be handled by making the patient LIE DOWN with his/her legs slightly elevated, give lots of water to drink, if possible give a cup of strong coffee to drink as it contains caffeine which helps increase the BP and also dietary salt can be increased.Once the condition improves a physician consultation should be taken to find out the underlying cause.Good LUCK"
},
{
"id": 199143,
"tgt": "Suggest treatment for erectile dysfunction",
"src": "Patient: Hi I m justin and im 16 years old. Since 2 days now (On February 7,2015) I ve been having a hard time getting a full erection.. I do masterbate a lot like 2 or 3 times a day. If I stop masterbating would it help? I don t know what is happening! I m so scared to:( Doctor: DearWe understand your concernsI went through your details. You are just 16 and you should not worry about these problems and should concentration your education. You are masturbating twoce or thrice daily. Reduce it to one per day or alternate days. Now, for a week, do not masturbate at all. Your emotional negativity about masturbation will reduce and then you will be alright. If you still need my assistance in this regard, please use this link. http://goo.gl/aYW2pR. Please remember to describe the whole problem with full detail.Hope this answers your query. Available for further clarifications.Good luck."
},
{
"id": 72887,
"tgt": "How to treat pain below the chest due to tuberculosis?",
"src": "Patient: Hi..i am vijay rana i am feeling pain right upper side of my stomach below the chest.i was suffering from tuberculosis and i finished my course in nov-2010 after that i drink alchohal some time.i am very worry about this pain what step i have to take? Doctor: Pain can be due to gastritis from drinking alcohol. You should stop drinking alcohol or take antacid."
},
{
"id": 173121,
"tgt": "What should I do for the sore tummy of my 5 year old?",
"src": "Patient: My 5 year old girl has had a bit of a cold for the past week Yesterday she came down with a fever which is controlled with advil and tylenol, she vomited when she gets the onset of the fever again but that is normal for her. Now she is complaining of a sore tummy even to touch she says it hurts.. What should I do? Doctor: Hi....Fever of few days without any localizing signs could as well a viral illness. Usually rather than fever, what is more important is the activity of the child, in between 2 fever episodes on the same day. If the kid is active and playing around when there is no fever, it is probably viral illness and it doesn't require antibiotics at all. Once viral fever comes it will there for 4-7 days. So do not worry about duration if the kid is active.Sore tummy could be due to viral illness as such due to mild gastritis. This is quite common.Paracetamol can be given in the dose of 15mg/kg/dose (maximum ceiling dose of 500mg) every 4-6th hourly that too only if fever is more than 100F. I suggest not using combination medicines for fever, especially with Paracetamol. Hope my answer was helpful for you. I am happy to help any time. Further clarifications and consultations on Health care magic are welcome. If you do not have any clarifications, you can close the discussion and rate the answer. Wish your kid good health.Dr. Sumanth MBBS., DCH., DNB (Paed)."
},
{
"id": 112019,
"tgt": "How to get relief from lower back pain which is getting worse after two c-section delivery?",
"src": "Patient: Hi there, I've had ongoing lower back pain since the birth of my first child (via emerg. C-section) 2 years, it continued throughout my 2nd pregnancy. Again I had an emerg. C-section with my 2nd child, which was 3 months ago. Pain unfortunately getting worse & am now getting tingling/numbness/pain in both my heels. Have seen an osteopath for last few months but has not given me any relief.Thanks Doctor: Hi there. Your symptoms correlate to disease called Prolapsed Intervertebral Disc...... most probably at the L5-S1 level. Pressure on the nerve is causing the tingling sensation. Kindly get an immediate Xray and MRI of the lumbosacral spine region done. Get it evaluated by an orthopedician.In the meantime follow this protocol :1. Bed rest.....as much as possible2. Avoid bending forward and avoid lifting heavy objects3. Local heat massage and analgesic gel application4. Painkiller and muscle relaxant medication.5. Pregablin to suppress the tingling srnsationsI hope my advice has helpedGood luck"
},
{
"id": 186640,
"tgt": "What cause the skin tag in my mouth?",
"src": "Patient: I have what looks Lila a skin tag on the right upper side of my mouth inside. I think several days ago I bit my gum but not sure if that is the cause. I noticed it about three days ago and it's just sore and unsightly. What is it and how long will it take to go away. Doctor: Hello, thank you for consulting with healthcaremagic. It can be because of the trauma only which was caused by bitting. It is called as fibroma. You just wait and observe the lesion , if it starts creating problem then you have to visit a dentist to get it removed, otherwise just apply a topical ointment named mucopain on it for symptomatic relief. Hope it will help you."
},
{
"id": 94494,
"tgt": "Had stomach pains after eating lunch, gets severe and then reduces. Causes ?",
"src": "Patient: I started having stomach pains yesterday after eating lunch. It hurt very bad and with all the way to my back. A hot bath took all the pain away, but maybe 30 mnutes later it began again. It would be very severe for about 3 minutes and then I woud be fine for 15 - 20 minutes. After I went to bed, it was good for the night. I woke up feeling great. No pain. I ate a piece of toast and eveything was still good. After I eat anything larger I get the pain. I plan to keep a diary of foods to see it I have developed an allergy . I am 57 and never thought about developing one this late in life. Doctor: thank you. have you vomiting or diarrhea ? it may be a one type of acute inflammation of the stomach. now you will take small meal with frequent interval. avoid large fatty meal. within 2-3 days it will be fine."
},
{
"id": 148697,
"tgt": "Recurred painful lump at the base on skull. X-ray showed no dental infections. Is this serious?",
"src": "Patient: One month agao I had an enlarged painful lump on my neck next to my carotid. Dr. treated with antibiotics told me to have my dentist do xrays two weeks later during my routine visit he couldn't see any infection CBC was goo so he thought maybe dental. The lump went away with antibiotics, nothing found at dentist, he even did the full 360 xray just to make sure. So I didn't worry much after that. For the past two days I am having same syptoms, same side only this time lump is on base of skull. Sore, hurts when I bend head down. Should I be concerned? Doctor: This may be due to cervical lymphadenitis.do u have any ulcers in the mouth? Did you ever suffer from tonsillitis ?kindly have a check up with Ent specialist toknow if there is any cause."
},
{
"id": 152464,
"tgt": "What are the symptoms of bone cancer in the legs?",
"src": "Patient: my brother had leg pain he was heading to restroom in house. all of a sudden his leg snapped two or thjree inches above knee. he went to hosp. in ambulance excrutating pain. they put a rod in leg to samples bone marrow from broken bones. that was two weeks ago hes in rehab now. im worried about bone cancer. Doctor: Hello and Welcome to \u2018Ask A Doctor\u2019 service. I have reviewed your query and here is my advice. Signs and symptoms of bone cancer include: Bone pain, swelling and tenderness near the affected area and weakened bone, leading to fracture and fatigue. Hope I have answered your query. Let me know if I can assist you further."
},
{
"id": 91583,
"tgt": "What causes an overnight lower abdominal pain?",
"src": "Patient: I am a 55 yr old reasonably fit male. I weight 87kgs at 173 cms tall. Overnight I have developed a lower abdomen pain - below the stomach button. I have now had it 16-17 hours. It is deeply seated. I haven't had the much to eat since I am doing the 5/2 diet to reduce my weight to a goal of 83kgs as I am an active rugby referee in New Zealand. I ate the same as my family last evening (i.e. a small piece of fried pork, 120 gms boiled potatoes and 100 gms mixed green vegetables). I have been able pass urine after drinking 1 cup of tea and 2 cups of coffee and probably 2 litres of water. I thought it was a bowel problem but I was able to go about 12-14 hrs ago. I have suffered from lots of gas over the last few years and that can be really uncomfortable and again I thought that was the problem but it is a sharper pain. I had a small dinner 3 hours ago and the pain was immediately worse. I have been having raspberries, blueberries and cherries over the last week and wonder if a cherry stone could a blockage or could there be another issue/ Doctor: Hi. This ca not be due to berries as there would have been distension, vomiting, constipation if here would have been an obstruction..This may be an intestinal infection because your pain is almost constant. A course of an antibiotic , metronidazole and probiotic should be of help. It is better to get checked by a Doctor as you need a clinical treatment and a prescription too. On a safe side go for an ultrasonograpy to rule out any other organic lesion/."
},
{
"id": 25561,
"tgt": "Does anxiety cause increase in triglycerides?",
"src": "Patient: Hi..I have prescripted Tonact TG at night to control triglycerides which was at 400. Within a weak the levels were 110...can i stop tonact tg now..what are the side effects. Am going through lot of anxiety..wll anxiety cause increase in triglycerides? Doctor: Hello!Welcome and thank you for asking on HCM!Regarding your concern, I would explain that anxiety is not related to high levels of triglycerides. High levels of triglycerides are usually related to your diet and low physical activity. Alcohol intake can lead to increased triglyceride levels. Your triglyceride levels have decreased because of Tonact TG therapy. But if you stop it, they will probably raise again. I recommend making some diet modifications (mediterranian diet, fish, olive oil, nuts and avoid fat and sugar) and a lot of physical activity. After these lifestyle modifications you can discuss with your doctor on the possibility of reducing the dose of Tonact TG. Hope you will find this answer helpful!Greetings!Dr. Iliri"
},
{
"id": 76707,
"tgt": "What does chest pain with bp 124/82 suggest?",
"src": "Patient: yes iam 24 years old and weigh about 300 lbs. For the last two weeks i have been having a little pain in my left chest. So i went to my doctor and he ran EKG and blood test and he said everything was fine. But i have been known to experience some anxiety . my blood pressure was 124/82 and heart rate was 88. The pain isnt real painful just a little tight sometimes do you think im okay? Doctor: Thanks for your question on Healthcare Magic. I can understand your concern. First of all no need to worry for major heart related diseases for your chest pain because your ecg is normal and your age (24 years) is too young for heart diseases. Sometimes, uncontrolled anxiety can also cause chest pain and chest tightness. So better to consult psychiatrist and get done counselling sessions. Try to identify stressor in your life and start working on it's solution. You may need anxiolytic drugs too. Don't worry, you will be alright. Avoid stress and tension, be relax and calm. Hope I have solved your query. I will be happy to help you further. Wish you good health. Thanks."
},
{
"id": 11942,
"tgt": "Brown spots, lomela cream, sun ban lotion, solution for pigmentation ?",
"src": "Patient: I had severe pigmentation i.e brown spots in my face I had severe pigmentation i.e brown spots in my face for the past three years. I have been advised to use lomela cream and sun ban lotion by a dermatologist . But no use. The pigmentation is still there. Ofcourse it is not spreading. I am wearing specs and under the spex, both side of my chin Doctor: Hi..dear Sheela.., Thanks for choosing HCM.., Brown spots on the face is called ..FRECKLES.., It is not respond to normal lightening creams.., So go for.., 1) Morning times use...SUNSCREEN LOTION.., 2) Night times use...Kojic acid and Vit C.., on night times.., 3) But best removal by using.., RF..Radiofrequency...it has to be done by...Dermatologist.., ok thanQ"
},
{
"id": 46234,
"tgt": "Is it ok to have sex while kidney stent fitted ?",
"src": "Patient: hi, just got home after having a kidney stent fitted as i have a stone stuck in my left tube going to the bladder. Starting to feel a bitv sick and have startedv get mild stomach ache and vwhen i pee it stings really bad i presume this is normal am i right, o and is having sex ok if so when can i expect to be able to do it and will i have to be gentle Doctor: Hi and welcome to HCM.As an Urologist, i can understand your anxiety.The stent is placed is placed endoscopically and hence will not affect daily routines. You don't have to worry about any physical changes externally.Neither worry about the stent or any adverse effect on kidney.Drink plenty of fluids daily.For pain relief,take the pain killers given by your Urologist.Have the review visit as advised by your doctor for stent removal.Dr.Matthew J. Mangat."
},
{
"id": 210083,
"tgt": "What can be the reason as can hear light ultrasound?",
"src": "Patient: I can sometimes hear a light ultrasound in my right year like when I read or in front of a computer screen. Never when I walk, work, train. It is not a major problem for me since it is weak and only when I am not active. At night, I dont have the problem...luckyly Doctor: HiThanks for using healthcare magicI think, you have tinnitus. In that case, better to consult a ENT specialist and get your ear properly checked. Sometime, due to wax, tinnitus happens. It is not hallucination, so there is nothing to worry about it. With cleaning of ear, such sound would subside.Thanks"
},
{
"id": 7717,
"tgt": "Acne on back,home remedies tried,now using Glyco-6 creme.any advice ?",
"src": "Patient: greetings doctor........ iam suffering from back acne from a few months......iam taking much personal cleanliness yet i see no decrease in acne......first i tried natural remedies like baking soda ,lemon, honey etc....the real problem is the acne left scars.....i am very fair and so those brown scars are highly visible on my back....finally iam trying Glyco-6 cream from a couple of days.....is it enough or shall i go for higher concentrated glycolic acid....also suggest me other efficient remedies to remove the scars.......i hope ur suggestions make my back scar-free..... Doctor: hi there. acne of the back is difficult to get rid of easily. what you should do is 1. use salicylic acid body wash twice a day 2. session of IPL every 3 to 4 weeks 3. microdermabrasions for dark spots 4. avoid oily junk food 5.no oil over body 6. red acne spots need cream clobetasole+ momet combination 7. may need to take doxy 200 twice a day for 2 weeks. then see the effect. all the best."
},
{
"id": 17922,
"tgt": "Should a cardiologist be consulted for low heart rate while on a blood thinning medication?",
"src": "Patient: I ha e a fib and take a blood thinner. I do not feel anything when I have had a fib attacks. I was diagnosed with while in the hospital 18 months ago following surgery. I have just purchased an Apple Watch. I notice with the heart rate app today while sitting watching tv my heart rate during an hour period was low at one point, 39, and within the hour as high as 151. Is this something I should see my cardiologist about? Thank you Doctor: Hi, Are you on any drugs other than blood thinner medication? If your symptomatic while on bradycardia then you have to consult your Cardiologist. Usually during rest or sinus arrhythmia may present like this. Get it done 24 hour ECG monitoring, echocardiogram, serum electrolytes and thyroid profile. If symptoms not improved please consult your doctor he will examine and treat you accordingly. Hope I have answered your query. Let me know if I can assist you further. Regards, Dr. Penchila Prasad Kandikattu, Internal Medicine Specialist"
},
{
"id": 166563,
"tgt": "Why does a child speak louder than others?",
"src": "Patient: Hi, I have a 4 year old daughter which usually speaks louder than others, even in the daycare they told me she speaks loud and I keep telling her you don't need to talk that loud. I was thinking to ask her doctor for a hearing test. other than that she is a very healthy kid. Doctor: Dear parent , speaking loudly could be a sign of hearing impairement . you should get a hearing test done"
},
{
"id": 136594,
"tgt": "Suggest treatment for swollen and painful knee",
"src": "Patient: I recently (6 weeks ago) sustained a crushing injury to my knee. The usual swelling, black and blue and pain. All that has gone. I still have slight swelling down to my foot, but no pain. A small area on the left side of my knee has no feeling. There is also a walnut sized blood clot just below the skin that down from egg sized. In the last week I have been getting heart palpatrations, (actually, missing a beat). Just for a short time when I get up in the morning or ready for bed at night. I take medications for low potassium, under active throid and high blood pressure. The question is: How serious does this sound? Doctor: Hi there, thanks for your question. I think your knee is fine.No need to worry about that, since you have no pain at present. Where as new onset palpitations and swelling down th leg are more worrying. I believe you should meet your physician and get it worked up. Hope this helps. All the best.Regards. Dr.SBK."
},
{
"id": 210736,
"tgt": "Could depression be the side effect of taking minocycline?",
"src": "Patient: My 17 year old son is taking Minocycline - 100mg for acne and has been taking it for years. He has also been suffering from mild to moderate depression for years but has recently been suffering from severe depression. Could there be a correlation between the two? Doctor: HiThanks for using healthcare magicMinocycline is a antibiotics and it does not lead to depression rather there are study, which shown that it help in reducing depression. In you son case, depression may be due to some underline stress. Better to consult a psychiatrist and get him treated.Thanks"
},
{
"id": 81717,
"tgt": "What causes frequent cough along with tiredness?",
"src": "Patient: Non smoker, 44 yr old female. Last 2 years have had difficulty exercising and feel tired. Frequent light cough and feels weird when I exhale. Pulmary fibrosis killed my father and mother has asthma, however pulmonary tests show nothing. I am concerned. I know my lungs are not right Doctor: If the pulmonary function test is normal and you have a very light cough, I believe there must be nothing wrong with the lungs. If you are obese it could be related to that however if not then u need to rule out any cardiac problems."
},
{
"id": 17061,
"tgt": "What causes sudden chest discomfort and leg weakness?",
"src": "Patient: I have been experiencing sudden chest discomfort such as pressure, fullness or squeezing sensation in the chest, and leg weakness and cramps and fatigue with a high blood pressure of 161/86 and pulse rate of 92 in the last 24 hours, should I go to the Emergency Room or wait to see a private doctor? Doctor: Hi, Any chest pain should never be underestimated and should be evaluated in detail.The symptoms you are telling can be of cardiac origin and I advise you to immediately see a doctor in an emergency and not wait.You should also get an ECG and 2D Echo done. If both are normal go for a stress test. Hope I have answered your query. Let me know if I can assist you further. Regards, Dr. Sameer Maheshwari, Cardiologist"
},
{
"id": 13835,
"tgt": "What are the red rashes on the back of the neck?",
"src": "Patient: hi I have a question, my husband has a rash on the back of his neck that just started out real small... he thought it was a pimple at 1st, he scratched it and he said it kinda stung a little and was sore right at 1st, well he didnt think much of it then the following day he ended up getting more spots and he said that they oozed out with stuff when he went to scratch them or rub them.. hes never had this before... we are wondering if it could be shingles or possibly something else that might be contagious.. nobody else has this in our household except for him... we arent sure what it is could you tell me your input please,,, thanks! Doctor: Hi, It could be a folliculitis or a herpes infection. Both the conditions are contagious. But the treatment differs. So, the diagnosis has to be confirmed by a Dermatologist. Please do consult your Dermatologist to confirm the diagnosis and to initiate proper treatment. Hope I have answered your query. Let me know if I can assist you further."
},
{
"id": 28384,
"tgt": "Suggest treatment to control blood pressure",
"src": "Patient: check Hi I am 62 year young lady and now I have high blood pressure,My daughter gave me anpron machine to check my pressure,but now I am nervous as my pressure keeps fluctuating.My daughter is a Doctor and now I am nervous.I am currently in care with my own Doctor ,but as I read thede didital machines are not that accurrate.Please advise. Doctor: Hello! Thank you for asking on HCM! You are right my lady, when you say digital BP measuring machines are sometimes incorrect ( they may lack appropriate calibration, or their battery may be almost expired and the machine doesn't properly work). I would not recommend digital machines, they are comfortable for solitary people, but they lack accuracy. Instead i advice a simple sphygmomanometer device. Only after using a correct measuring device, one can be sure in judging the presence of high BP.Hope to have been helpful to you! Greetings! Dr. Iliri"
},
{
"id": 105575,
"tgt": "Allergic to dust, weather changes, starts with irritation on the upper side of mouth, throat, chest causing breathing trouble. Improvement with Montair LC. Is it safe for long term?",
"src": "Patient: hello sir my problem is that i am allergic to lot of things like weather change saw dust, taking something cold , the reaction is i start getting uncomfortable there is a littile irritation upper side in my mouth then this passes to my throat and then my chest, shortness in breathing , breath heavily after climbing stars, i had some relief after taking allegra , then i consulted a md he prescirbed montair lc this give me complete relief for one day i feel very comfortable recently i found that after eating rice i felt this is something which is creating this problem. i also suffer from digestion problem is it safe to take montair lc for a long period what will be its long term side effects Doctor: As is apparent from your description, you are prone to develop allergy to various substances. If you are feeling better with Montair LC, you may continue it as it`s a very safe medicine with hardly any serious side effects. It can easily be taken continuously for 6 to 8 weeks. Moreover if you have breathing trouble, you may take inhaler medicines like aerocort, seroflo, foracort etc. Avoid things you are allergic to as much as possible."
},
{
"id": 77684,
"tgt": "Due to abnormal painful snouts experiencing difficulty in breathing",
"src": "Patient: My dad has abnormally huge painful boogers. He says they make it impossible to breath cause headaches that he can't get rid of and there is also blood almost every time he blows his nose and even more blood when he can finally get them out! They cause him a lot of pain! I was wondering what is causing this, if its something to be worried about, and if at all possible how to make it a little more bearable or make it go away completely?? I've googled this countless times and can't find anything! Doctor: Hi,Dear,Thanks for your query to HCM.Dear I read your query and reviewed it with context to your query facts.I understood your health concerns and feel Concerned about them.Based on the facts of your query, you seem to suffer from-Hard,Crusty Nasal dried out discharge.Don't disturb them by removal.Warm water with bicarbonate douches would release them easily for you.If you physically remove them,they could createheavy bleeding from nasal bleeders placed at that location.Adequate hydration and timely maintenance would reduce their hard encrusted formations.Hope this would resolve your health issues in the best way possible.Welcome for any further query in this regard.Good Day!!Dr.Savaskar M.N.Senior Surgical SpecialistM.S.Genl-CVTS"
},
{
"id": 131613,
"tgt": "Could consistent inflammation in thumb with sharp, painful area be bone spur caused due to constant massage work?",
"src": "Patient: I m having consistent inflammation in my thumb, which has been sorely overused due to years of doing deep tissue massage work. There s an area that is hard, fairly sharp, and painful from the knuckle to the corner of the thumbnail, on the lateral side. Is an x-ray the only means to determine whether this is a bone spur? Doctor: HiYes, x-ray can be helpful .hot fomentations and application of volini ,may help.if severe pain, motrin tabsConsult a orthopedic doctor"
},
{
"id": 218413,
"tgt": "Should the ER be visited for fever during pregnancy?",
"src": "Patient: Hello the question is about my pregnant daughter she has the flu and she\u2019s running a fever of 101.4 I know a fever can be dangerous to the baby but I\u2019m just wondering if you can tell me if I should take her to the hospital she\u2019s drinking and keeping fluids down Doctor: Hello and Welcome to \u2018Ask A Doctor\u2019 service. I have reviewed your query and here is my advice. Fever in pregnancy needs evaluation. You have to take her to the obstetrician at the earliest to find out the cause of pregnancy and viability of fetus. Hope I have answered your query. Let me know if I can assist you further."
},
{
"id": 28117,
"tgt": "What causes dizziness and shortness of breath with low blood pressure?",
"src": "Patient: I m male, 51-years-old, 5 8 , and 230 pounds. I take 10 mg of lisinopril and 12.5 mg of coreg 1 x day. For the past week I ve been lightheaded and short of breath. My blood pressure reading 10 minutes ago was 118/72 and my resting heart rate was 111. Should I be concerned and go to the ER? I have an appointment with my doctor on Wednesday morning. Doctor: Hi welcome to healthcare magic,I understand your query and concern.Your symptoms are suggestive of heart failure secondary to ischemic changes in the heart.I advise you to get an ECG,2 D Echo,lipid profile,hsCRP immediately to confirm the diagnosis.Drugs like clopilet,ecosprin,atorvas ,inotropes like digoxin will be helpful.Coronary angiogram with or without stenting will be the procedure of choice.Consult your cardiologist for expert management.Hope I have answered your query at the moment,Post your further queries if any,Thank you."
},
{
"id": 58116,
"tgt": "Elevated bilrubin, average BP, take good diet. How worried should I be?",
"src": "Patient: Just had my yearly blood test . All came back normal except for a high bilirubin reading of 70. I have no adverse symptoms. Apparrently though last year it was 80. Based on the reading of 70 however I have had to provide another blood sample for further analysis. I now need to see my GP to discuss. Just wondered how worried I should be. I am 49. BP averages about 138/78. I am 6 and 12 stone 12. I run 3 miles 3 times per week to keep fit. My diet is pretty good but I am probably boarderline on my alcohol units. Doctor: Dear FriendElevated bilirubin levels can be because of many causes.We need to know your direct and indirect bilirubin levels.We also need to know your ALT / AST levels.I advise you to get an USG Abdomen to see for liver size, texture.Get Hepatitis panel.Avoid alcohol.Stay Healthy"
},
{
"id": 210403,
"tgt": "How to treat anxiety and depression with history of brain aneurysm?",
"src": "Patient: I had a brain aneurysm in 1997, and now I am dealing with anxiety and depression. I have been to mental health services and seen psychiatry. That keeps me more disturb saying things that I am making up things and that I am hyper and I monopolies about my past. I am suffering now and need suggestions to control of head again. I look normal on the outside but inside my head I fell very sick. Doctor: DearWe understand your concernsI went through your details. I suggest you not to worry much. Your mental health services people could be right. Let me explain. they said that because they could not find any organic relationship between your current problem and brain aneurysm in 1997. So possible reason is hypochondriasis. Hypochondriasis or hypochondria (sometimes referred to as health phobia or health anxiety) refers to excessive preoccupancy or worry about having a serious illness. This debilitating condition is the result of an inaccurate perception of the condition of body or mind despite the absence of an actual medical condition.In your case, you probably are worried about the previous illness and that causes you the discomfort. Such a discomfort and state is possible. The best possible treatment include psychotherapy and relaxation techniques.Please post a direct question to me in this website. Make sure that you include every minute details possible. I shall prescribe some psychotherapy techniques which should help you cure your condition.Hope this answers your query. Available for further clarifications.Good luck."
},
{
"id": 74118,
"tgt": "Suggest treatment for gas poisoning",
"src": "Patient: Hi, I just had a new gas boiler fitted, but the installer made a very bad gas leak that he didn't repair, so after a few days I suffered gas poison, the effects were very bad, now 2 weeks after it happened I have very little energy, and I can't get interested in anything, is there something that my doctor can give me to make me feel in the land of the living, because Doc I really feel like death warmed up. Many thanks.Alan. Doctor: HiBetter to do a check up to examine better your actual situation of the organs.Take careDr.Jolanda"
},
{
"id": 83977,
"tgt": "What are the side effects of hydrocortisone cream?",
"src": "Patient: Hi my name is Denise I am 17 year old girl and I have a medicine problem. I supposedly have eczema the doctor prescribed me Hydrocortisone but every time I use the cream I have an intence burning sensation and I feel I am going to cry. What have I done wrong and how can I fix this problem? Doctor: Hi,Your symptoms might be due to the over-usage of hydrocortisone cream. Common side effects of hydrocortisone includes:- Skin blistering, burning, crusting, dryness- Also, irritation, itching, scaling, severe redness, soreness, or swelling of the skin- If used on the face it might cause redness, scaling around the mouth, easy bruisingI advise you to report to your doctor and stop using the cream. You can try other alternatives like:- Fluticasone cream at night - Tab. Prednisolone 5mg for 10 days on tapering doseHope I have answered your question. Let me know if I can assist you further. Regards, Dr. Yogapriya Vasudevan, General & Family Physician"
},
{
"id": 201934,
"tgt": "What is the treatment for low sperm count?",
"src": "Patient: Good morning, I have been married fot almost two years now but my wife has never missed her menstruation. She therefore accussed me of having a low sperm count. We went to see a doctor last friday for a LSC TEST of which the doctor confirmed that I am having a low sperm count. The results indicated that I am having 16million sperms instead of 25million and that some of the sperms a having big heads and some double tail, I want to know if this can be treated. My name is Jonas and I am 37 years old. I am about 1.7meters tall Doctor: Hi Jonas. Your count is low but not very low. Another problem is the abnormal forms. I suggest you get a Semen culture done to rule out any infection which might be leading to the abnormal forms. If it picks up infection you should take antibiotics as shown sensitive by the same report. Also take Auurvedic preparations like Speman or similar medicines for three months which have known to increase sperm count. Do let me know if there are any further queries. You can contact me directly by using 'Ask a Specialist' icon. Take care. Dr Rishi, New Delhi, India."
},
{
"id": 115385,
"tgt": "What do elevated white and red blood cells suggest?",
"src": "Patient: Hi my blood work was perfect last year but I had it re taken yesterday and it came back elevated white and red blood cells as well as elevated albumin and calcium and elevated protein and mpv and hgb and hct!! Rest seems good! I had cervical cancer 16 years ago and last pap was fine! This doctor just wants to re check in two months but don t these mean cancer ?? Doctor: Hi welcome to HCMI have gone thru your query regarding elevated white and red blood cells .I can understand your concern .Dear ,you have not given your age nor your blood count of your test . Abnormal , high or low , reading of counts can be indication of some grave problem and gives us warning against diseases So beware and mind it 'Pervention is better than cure' Magnitude of the survival advantage from improved nutrition, can even be greater than the magnitude of the treatment effects being targeted in current clinical drug trials . your doctor has given you to recheck in 2 months time .I wiould like to suggest you to modify your life style for enhancmen of your immune system , side by side your physician's guidance to fight the imbalaced blood counts , is one of the most effective treatment options in my view ..Avoid processed foods carbs , sugar , fried ,fast foods , Tea , coffee ,alcohol and smoking all will produce biochemical and metabolic conditions in your body that will decrease your immunity, so avoiding is definitely the first step in the right direction.Detoxing the body, boosting the immune system .One of the important strategies to achieve healthy body is a diet rich in fresh, raw whole foods .More of high Fiber, protein , fruit, egg fish ,fish oil for Omega 3, green leafy veges .To keep metabolism on right path to increase the strength of immune system , take all supplements full of antioxidant serve as antibiotics .Raw garlik ,gingerCoconut water ,Bittergourd juice 25-30 ml Lemon juice with water twice before meals , Turmeric powder ,+a spoon of almond oil level spoon with with a cup of hot milk with B/F & at bed time , goat's milk preferButtermilk with lunch Aloevera juice 25 ml +amla juice 2 spoons Physical activity in order to renew healthy cell and rebuild itself, so you should make exercise a lifelong commitment, but never overdo.Do 30 mins walk/ exercise - from head to toe ,yoga ,pranayam - deep breatihing , Kapalbhatti / Laughing aloud . proper rest , meditation & positive thinking , to detoxify mind & body & give immunity to your system , to accelerate the process of recovery . The condition is REGULARITYAbove regimen will surely gradualy , help you to lead happy, worry free ,healthy life ahead Many have been benefitted by These therapies which slow the growth of bacterias and kill on mass bases . Get an apointment with YOUR DOCTOR AFTER 2 MONTHS ANS SEE THE DIFFERENCE and review your reports please .HOPE THIS HELPS SOLVE YOUR QUERY Take care All the best . If any doubt mail at drsuchda@gmail.comDon't hesitate TO COME for futher query if anyWish you early recovery"
},
{
"id": 124834,
"tgt": "What causes a swollen muscle in buttock that hurts?",
"src": "Patient: I have a what feels like a swollen muscle in my left buttock that hurts to sit or stand. I had a hemrroidectomy 6 years ago (both internal and external) and since the surgery struggle to go to the bathroom with pushing and straining. Even with loose stools. Doctor: Hi, These problems are not common after hemorrhoidectomy. In my opinion, you could be suffering from some other problem in gluteal muscles. Please consult orthopaedic surgeon for examination, investigation and treatment. Hope I have answered your query. Let me know if I can assist you further. Take care Regards, Dr Gopal Goel, Orthopaedic Surgeon"
},
{
"id": 137420,
"tgt": "What causes muscle fatigue in the legs and arms?",
"src": "Patient: My husband (55 y.o.) started experiencing muscle fatigue in arms/legs two months ago. It has progressed from arms and legs to head to toe, including needing to exert increased effort to speak, breath and swallow. Symptoms are mild in the mornings and increase through the day till he feels totally exhausted in the evening and describes feeling like every move he makes is through chest deep water. He denies any pain or weakness. He also complains of occasional light tingling and slight numbness of his lips. All blood work results are normal. Doctor: Hello,I have studied your case and I think that he might be having vitamin D and calcium deficiency along with low salt in the body. I would suggest you to get someblood test for him.It includes vitamin D3 , and serum B 12 test along with blood sugar and thyroid testing. If there is deficinecy of any thing then he needs supplementation. he should take plenty of salts and electrolites in diet with green leafy vegetables. I hope this answer will be useful for you. Let me know if there is any other followup questions.thanks"
},
{
"id": 203851,
"tgt": "What could it be if I had a painful rash in the groin which is discharging fluid?",
"src": "Patient: I have a rash that is bad enough to be discharging fluid. It has a bad smell to it and is only on one side of my groin between the scrotum and groin. It is painful and looks like raw skin. It appeared after having trimmed my pubi hair with an electric trimmer. I may have nicked the skin. My wife and I had intercourse the next day and then the rash began about two days later. Not sure if this is all tied together. The infection seems to be bad enough now that my inner thigh has swollen. Doctor: Hi ! Good afternoon and a Happy New Year 2014. I am Dr shareef answering your query.With your history it seems that you have got a fungal infection with super added bacterial infection. The infection might have come by the way of trimming the pubic hair or might have come from the vaginal discharge of your wife during intercourse.In both the cases, you have to go for a topical antiseptic and antifungal ointments, and also a broad spectrum antibiotic for the cellulitis/infection in the thigh.I hope this information would help you in discussing with your family physician/treating doctor in planning your treatment. Thanks for using the health care magic forum for your query on health. Wishing you an early recovery. Dr Shareef."
},
{
"id": 154828,
"tgt": "Can severe pain in the abdomen symptoms for carcinoid cancer?",
"src": "Patient: I am currentlly battling stage 4 carcinoid cancer with mets to my liver. In the past few months I have experineced charlie horse pain in my abdomen (upper left side and some on the right). It balls up in ball and leaves me screaming trying to straighten out and rub out the not. It just happened 3 times 20 minutes ago while I was using the bathroom. The pain is a 10. I did go to my Gastro and the NP patted me on the head and said it was my stomach and sent me on my way. This is NOT something that I can live with. Doctor: Hi, dearI have gone through your question. I can understand your concern. Your pain is mostly due to carcinoid tumour. So you should take treatment of carcinoid cancer. Treatment depends on the location and stage of cancer. In terminal stage chemotherapy is the treatment of choice. For terminal pain you can take morphine for pain relief. Consult your doctor and take treatment accordingly. Hope I have answered your question, if you have doubt then I will be happy to answer. Thanks for using health care magic. Wish you a very good health."
},
{
"id": 43181,
"tgt": "Polycystic syndrome. Have spotting. On genette, dosme. Will there be problems with conceiving?",
"src": "Patient: hii... last month i had spotted istead of bleeding..i was adviced to take orgamed by doctor i took it and i bleeded normally...and ddr told me that i have polysytic ovaries after my seeing my scan report..and she told to come on 5th day of my period and i went on my day 5..and sheadviced me some other drugs like ginette,dosme,metdml...and to come on day 2 of my next period...due to some reasons im unable to go on 2nd day and i left the place...now this month i had bleeded normally on the day when i spotted on last month i felt hpy that my periods are regular but again im spotting on the day when i bleeded last month....now we are planning for pregnency...is there any problem to get concieve ....im so tensed about this.... Doctor: Hi, polycystic ovaries is not a problem for conception. You have to undergo regular follicular growth scan to check follicle size and best time to have intercourse. Regards, Dr. Mahesh Koregol"
},
{
"id": 76191,
"tgt": "Does pressure in chest with lightheadedness be a cause for concern?",
"src": "Patient: 52 woman, Obesse, Once in awhile, resting, driving, I have a build-up feeling in my chest, and know what is coming, skipped heartbeat(s) then rapid beats that catch up. This creates some fear that I might be having a heartattack, but I feel fine afterwards. Today I had one and felt light headed off and on for two hour now, and have had a headache for about an hour now. There is a tiny bit of pressure in my chest. Should I be concerned? Today is a holiday, should I go to the Dr.'s tomorrow? Doctor: Thanks for your question on Healthcare Magic. I can understand your concern. Yes, you should definitely consult doctor tomorrow. Your symptoms like chest tightness, heaviness, skipped beats, rapid pulse rate, lightheadedness etc are more suggestive of heart diseases. You are also approaching menopause. So hormonal changes (low estrogen and progesterone) are likely in your case. And these changes are risky factors for heart disease. So consult doctor and get done 1. Blood pressure monitoring 2. Ecg 3. 2d echo 4. Stress test (trade mill test). Don't worry, you will be alright with appropriate medicines. Hope I have solved your query. I will be happy to help you further. Wish you good health. Thanks."
},
{
"id": 152252,
"tgt": "Is it possible to have worms living in the sinus nose ?",
"src": "Patient: Is it possible to have worms living in the sinus nose ? when I blow my nose it looks like to me stringy, and as they always seem to be the same size im sure they are. if they are how in lords name can i kill them. 1-2 cm long. Doctor: Hello Welcome to HealthcareMagic As per your history it may be any sinus growth as polyp or other. After ent examintion one may decide it if a worm and how to treat. Have a Healthful Day.."
},
{
"id": 99335,
"tgt": "How do steroids help asthma?",
"src": "Patient: Does we have antidose for Steroid.actually ,my wife taken some ayurvedic/traditional drugs for cure of Asthma,after two months she found her total body getting swelling,so discontinued the medicine.upon enquiry the person who'd given those medicine admitted to used some steroids.though for Asthma she take the help of inhaler and under control but,her body is in swelling condition for last one month and feels severe pen while walking.plz advise the remedy. Doctor: Hello,Thank you for asking at HCM.I went through your wife's history and would like to make suggestions for her as follows:1. Side effects of steroids are more common if they are taken orally or injections. I would first think that she has had side effects due to oral steroids which were mixed in medicine.Common side effects of oral steroids are weight gain, diabetes, high blood pressure, gastritis, and many others...2. Many of the inhalers for asthma also contain steroids, but they are in inhaled form and they are minimally absorbed in blood. Hence they usually do not cause such side effects. Hence, they are much more safer for long term use as compared to oral steroids.3. As she has already had side effects, unfortunately there is no antidote. Side effects of oral steroids can decrease with time (a few months) after discontinuation. Though she will be able to use inhalers as per her doctor's advice.4. For better asthma management, I would suggest her to consult a pulmonologist or allergy-asthma specialist who will assess her symptoms in detail and will prescribe necessary treatment.5. I would also suggest her regular yoga & pranayama, regular exercise like walking which increases immunity and lung capacity over long time.6. I would also suggest her a healthy diet rich in vitamins & minerals which will gradually improve her immunity over long time.7. Regarding pain while walking, it could be due to overweight, but it is important to know exactly which part of leg is aching to guide further. An orthopedic consultation may be helpful if pain is severe/of long duration.Hope above suggestions will be helpful to her.Should you have any further query, please feel free to ask at HCM.Wish her the best of the health ahead.Thank you & Regards."
},
{
"id": 4250,
"tgt": "Is it possible to get pregnant after having had an abortion and receiving depo injection?",
"src": "Patient: hi i had a abortion dec 21st and i also had the depo injection the same day and dec 25th i had unprotected sex and then i also had unprotected sex about 3 or 4 more times after that now im feeling pregnant again im very fatigued my bones are relly weak my back butt and legs are very crampy and so is my stomach Doctor: Hello dear, just do a urine pregnancy test and if negative there is nothing to worry. But remember the contraceptive effect of injection is for 3 months so plan for further contraception after its duration is over. Take care and all the best."
},
{
"id": 27891,
"tgt": "Suggest treatment for heart palpitation and shortness of breath",
"src": "Patient: I recently took my daughter to the doctor because she was having what seemed to be heart palpitations. They were very speratic in timing being several days or even a week or more in between episodes. our doctored performed a EKG that seemed to be normal. The doctor asked if she had shortness of breath or pain. the answer was no. My daughter does not drink or smoke. Caffeine consumption is low due to she does not like soda as taste is distorted by her topramate that she takes for optic migraines. Our doctor did not seemed alarmed and explained these may be some sort of electical palpitations. But the past three days she has had a total of ten which is way more than usual in such a short time. A couple instances there were three episodes within 30 minutes. Other times they were hours apart. We called our doctor to report these episodes but she does not seem alarmed. Should we be alarmed? Doctor: I have gone through your question and appreciate Your concern. According to your history there are repeated episodes of palpitations. I think it is matter of concern as she maybe you suffering from Rhythm disturbance. Like atrial fibrillation or ventricular tachycardia. If Ecg is normal holter monitoring can be done to reach diagnosis. Before labelling it due to anxiety better to rule out arrhythmia. Thanks"
},
{
"id": 154822,
"tgt": "Suggest treatment for BLDCL type Lymphoma cancer",
"src": "Patient: My father whose age is 58 yrs . suffering from BLDCL type Lymphoma cancer. He had undergone through 6 chemo of R-chop along with Autologous BMT but not found positive result after duration of 8-9 months . Again 3 therapies had done...now they doctor has recommended for Allogenic BMT . Can you please suggest what to do now ?? Doctor: Hi, dearI have gone through your question. I can understand your concern.Your father is suffering from diffuse Large B cell lymphoma( DLBCL). It is a type of non hodgkins lymphoma. Treatment depends on stage of lymphoma. Overall chemotherapy is the treatment of choice. However if your father does not respond to chemotherapy then Allogenic bone marrow transplantation may help him. Consult your doctor and take treatment accordingly.Hope I have answered your question, if you have any doubts then contact me at bit.ly/Drsanghvihardik, I will be happy to answer you.Thanks for using health care magic.Wish you a very good health"
},
{
"id": 166535,
"tgt": "What is the treatment for cough and vomiting in a child?",
"src": "Patient: My son has had cold symptons for several days. This morning both of his eyes were matted shut and his eyes have ran with green gunk all day. He has a terrible cough that results in the vomiting of phlegm. He has no fever, but does have a nasty nose. Should we ride it out or is it time to see a doctor? Doctor: dear parent, for his eyes you should use an antibiotic eye drops as topradex. for cough you should give a cough suppressent as dexomethorphan, together with an expectorant as mucinex. for the running nose you should give an antihistaminic as citrizine once daily before going to bed"
},
{
"id": 196753,
"tgt": "Can there be any sperm in pre-ejaculation ?",
"src": "Patient: Hi, I'm 16, 5'3, 96lbs, female. I was wondering, I know that some sperm stays behind from a previous ejaculation and can come out with pre-ejaculation. But, if you have never ejaculated before, could there be any sperm in your pre-ejaculation than? Doctor: Dear user,Thanks for writing to HCM.I can understand your concern about query regarding the presence of sperm in preejaculate.It is not so that preejaculate is due to remaining part of previous ejaculations.Pre ejaculate may contain live sperms irrespective of the age and number of ejaculations.It may be there in the first ejaculation also.Regards."
},
{
"id": 131424,
"tgt": "How to treat partial humeral avulsion of the inferior glenohumeral ligament(HAGHL)?",
"src": "Patient: I have a partial humeral avulsion of the inferior glenohumeral ligament (HAGHL) and there is a small locular subcoracoid bursal effusion. My question is are there any other treatments out there beside surgery? Right now I have very little use of my Right arm without pain. Doctor: HiThe gleno humeral ligament tear has to be repaired,there's no alternative.The sub coracoid bursa may be infiltrated with depomedrol injection for now, but if open repair of glenohumeral ligament is undertaken, this bura may be excised in same sitting once the area is exposed in surgery.Support arm in a sling and avoid all movements of shoulder and plan for earliest repair.Take tramadol100mg sos for severe pain and ace -proxyvon tablets1 BD for 5 days .Apply voltaren gel locally on shoulder."
},
{
"id": 106782,
"tgt": "How can lower backache and pain in the buttocks be treated?",
"src": "Patient: I have lower back issues from lifting people while in and out of there mobility equipment, I have a continuous pain in my right buttock but gets worse when I m laying in bed or trying to get out of bed along with painful knees but becomes less painful the more I walk around, why do you think that is or could be I m sorry for wasting your time, I have no extra funds to pay, I live by myself with my mentally challenged sister, just spent my tax return to pay off bills and catch up on my mortgage. The only income we will have now is her disability. The ER just sent me a bill from my visit totaling $927.00 Doctor: Hello and Welcome to \u2018Ask A Doctor\u2019 service.I have reviewed your query and here is my advice.Lower back muscle spasm can be the most likely reason. Taking over the counter NSAID drugs like Ibuprofen along with Ranitidine daily for 5 days will be helpful.You can also do hot fomentation on back. If no improvement then x-ray should be taken to rule out spine problems.Hope I have answered your query. Let me know if I can assist you further.Regards,Dr. Nishad BN"
},
{
"id": 205540,
"tgt": "Suggest medication for exhausted feeling and loss of concentration",
"src": "Patient: I am so exhausted all of the time and have a problem concentrating and not being scatter brained but I tried phentermine and was not as exhausted and was very alert and my mind was quite a bit sharper than usual. Could I obtain a prescription for this medication? Doctor: Hello thanks for asking from HCM for your health related queriesI can understand your concern. The symptoms like exhausted feeling, poor concentration etc can occur due to persistent stress or mild depression. But some medical causes should be ruled out to make diagnosis of stress as cause of these symptoms. Low vitamin B12 levels, poor nutrition, iron deficiency anaemia, hypothyroidism, hyper cortisol levels etc are common medical causes that should be ruled out for these symptoms. Please consult a doctor for proper evaluation and investigations.Vitamin B12 supplements, Omega 3 Fatty Acids tabs help to reduce these symptoms and are very useful. If symptoms persist then visit a psychiatrist for treatment. Thanks, Hope this helps you. Take careDr. Seikhoo BishnoiPsychiatrist."
},
{
"id": 38149,
"tgt": "Is carrington antifungal cream right treatment for yeast infection?",
"src": "Patient: I have a yeast infection and also my face and touge are have red spots on them and my face is chap and itching and I m starting to itch other spots on my body, at this time I am using monistat 7 and AZO yeast for treatment should I go to the hospital or just apply carrington antifungal cream to the other spots on my face and body Doctor: Hello, Thnx to contact us. I understand your concern. If I am your treating doctor I advice you that in your case taking the antifungal treatment is not enough. Please consult the nearby doctor near to you for proper diagnosis. I suspect it is not a fungal infection and something else. Visual examination is necessary for diagnosis and than proper treatment.If you have anything else to ask, kindly contact me.Wish you a very good health at health care magic. Dr. Arun Tank. Infectious Disease."
},
{
"id": 120643,
"tgt": "Suggest treatment for pain in Achilles",
"src": "Patient: Hi I ve had pain in my left Achilles for the past 2 weeks, I ve been using ibuprofen gel which has helped but it s still painful especially going up and down stairs and if I walk on it for a length of time. I ve now also noticed a bump on it to the the side and have a fluttering feeling in it. I wondered whether it could be tendonitis or whether it could be something else? I don t think it s ruptured as I still have full flexion of my ankle and toes. Doctor: Hello, Based on your information it's appears to Achilles tendonitis.However, accurate diagnosis could be arrive only after clinical examination and relevant investigation. If it's Achilles tendonitis,Course of physical therapy and shoe modification with heel raise and soft posterior countor would be quite helpful. If no relief afte 6 weeks of above treatment,local injection could tried. Hope I have answered your query. Let me know if I can assist you further. Take care Regards, Dr. Rajesh Gayakwad"
},
{
"id": 63049,
"tgt": "What causes lump upper middle chest?",
"src": "Patient: about 3 days ago I noticed a lump on my upper, middle chest, pretty well above my right breast. It is non moving and hard. There is no pain or tenderness upon touching it, It feels as big as 1-1 1/2 inches, and about the size of 2 quarters stacked.I am a 21 year old female. I smoke very rarely, about a pack a month. Don't drink or do drugs. I take prescribed percocet as needed for back pain (1 or 2 10mg pills every day). I exercise regularly and eat healthy though I am slightly overweight since i suffer from PCOS and hyper and hyper thyroid. I also suffer from fibromyalgia I just went to the doctor today due to the fact that I have been sick for around 3 days ( I noticed the lump when I started feeling sick) The P.A had no bedside manner and didn't even tell me the diagnosis however, from past experience, this feels like an Upper Respiratory Infection since the cough, pleghm, everything are mostly in my chest. I don't know if any of this information is even relevant but i figured I would put it down in case you need it.I just want to know if this could possibly have anything to do with my sickness or possibly my fibromyalgia. Or is this something I should be worried about? Also, if it is something worrisome, what kind of doctor would I see?Thank you so much! Doctor: hi.it is best if you consult with a doctor, preferably a general surgeon, for medical and physical examination. based from your description, it could be a cyst, a fibroma, a fibrocystic lesion or other breast tumors. these lesions are best evaluated clinically. further diagnostic examinations, such as breast ultrasound, mammography and tissue diagnosis or biopsy, will be requested as needed. regular breast examination and follow-up with your doctor is highly recommended in these cases. management (medical and/or surgical) will be directed accordingly.hope this helps.good day!!~dr.kaye"
},
{
"id": 21060,
"tgt": "What is the treatment for hypertension?",
"src": "Patient: I was in the hospital for hypertension and the doctor gave me amlodipine bestlayte 5m and I have been feeling the same, I lose my appite and I dont feel like talking to anyone, and My saliva feels like water should I stop taking the pill or wait and continue taking them and make an appoiment with my doctor? or is this normal? Doctor: Hello, thanks for posting a question on HCM. I understand what you are going through. When I usually prescribe amlodipine for my patients, some complaint of nausea and other symptoms. A majority of them report that the symptom goes away after few weeks of taking continuously taking the medication.I will advise you consult your physician for an alternative medication if you continue to have this symptom. Wishing you a good health."
},
{
"id": 98685,
"tgt": "Suggest treatment for asthma and chronic bronchitis",
"src": "Patient: I am 63 years old obese woman with history of asthma and chronic bronchitis. Yesterday I started coughing which by late last night developed a deep sound. My neck feels tight when I cough and my head. Face.ears,and throats hurts when I cough. Will taking nebulizer with solution that expired nov 16 hurt or help me. I already have a call in to my doctor for my symptoms and for solution and new nebulized. Doctor: If u didnt open the solution bottle and it is sealed then u can take it in emergency. It sounds though u are suffering from acute bronchitis and need nebulisation immediatly. Since u are obese u need to sleep with ur head raised at a higher level so that ur respiratory tract pathway is clear and u can breathe easyTry to take early morning walks regularly and take high protein foods like egg whites or lean meats and chicken without skin and moderate amounts of carbs. This will directly stimulate ur immune system and decrease recurrent attacks of bronchitis. Avoid all heavy and fat foods."
},
{
"id": 210905,
"tgt": "How to control over obsessed dad keeping regular watch on me?",
"src": "Patient: I have a 70 year old stepdad that is extremely obsessed with me! He digs through the trash when I throw something away, no matter what it is. If I bring food home from work and put it in the microwave, he waits until he thinks nobody is around and has to open the microwave to see what it is. He is always watching everything I do. I could just be sitting in my chair and he will go out of his way to look at what I m doing, even if I m just sitting .One day when I was eating at the dinning room table, I noticed that he had moved some stuffed animals in my moms china hutch to the side, so that he could watch me in the mirror. If he can t see me, he will get up and stare at what Im eating. My mom and I were wondering why he is so obsessed with me and what I m doing? Doctor: DearWe understand your concernsI went through your details. I suggest you not to worry much. You have not given your age or gender here. Yes. The behavior of your step father do correspond to obsession. But why? From the description you gave, it is not possible to identify the cause behind it. But you can find. Just go back some days to that day when it all started. Then make a chart with all the events happened in one week time. Most probably you should get your answer. If not, it is better to ask your father itself. Do not shy away and keep the doubt to yourself.Hope this answers your query. Available for further clarifications.For more visit Http://psychocure.webs.com/Good luck."
},
{
"id": 126304,
"tgt": "How can anterior hip pain which has progressively gotten worse be treated?",
"src": "Patient: I have had what I think is anterior hip pain which has progressively gotten worse. I can t tell if it s my hip or upper outer thigh. More recently I ve started having buttocks pain more on the right side. There is no radiating pain. This all gets worse as the day goes on because I sit all day at my job. Doctor: Exercise is one of the treatment to help to reduce your pain it helps to maintain the range of motion and strengthening the muscles that support s your hip stretching the muscles that surrounds the joints may help to ease pain may reduce the risk of hip problems and prevent them from injuries"
},
{
"id": 84412,
"tgt": "IS pain in back and knee due to reduction in steroid intake?",
"src": "Patient: I have pmr my steroids have been reduced and I am experiencing pains in the back of my left knee that is persistent. I always had pain there when I was first diagnosed with this. My dosage of steroids now is 3 for one day then 2 then 3 again which is what my specialist advised. Doctor: Hello, Long term use of steroid is not recommended. Consult a spine surgeon and he will direct you accordingly. Hope I have answered your query. Let me know if I can assist you further. Take care Regards, Dr. Shinas Hussain, General & Family physician"
},
{
"id": 143418,
"tgt": "What does dizziness, vomiting, and heavy head indicate?",
"src": "Patient: hi, what does dizziness and heavy head indicate..constant vomiting after getting up from rest and the local doctor said that it is nothing to do with vertigo or with the inner ear.......when walking you feel like your about to fall and patient very fatigue and unable to do anything. Doctor: Hello!Thank you for asking on HCM!Regarding your concern, I would explain that your symptoms seem to be related to an inner ear disorder. Coming to this point, I would recommend consulting with an ENT specialist for a physical check up and labyrinthine tests to investigate for an inner ear disorder. Some blood work is also needed, considering the fact that you keep vomiting (complete blood count, blood electrolytes). Meanwhile, I would recommend having some rest, avoid abrupt head movements and take plenty of fluids to stay well hydrated. Cinnarizine or betahistine may be helpful. Kind regards, Dr. Aida"
},
{
"id": 119922,
"tgt": "What causes severe pain on the foot after falling down?",
"src": "Patient: 2 months ago I fell and twisted my ankle. No broken bones from the x-rays. Pain was getting worse so Dr had an MRI done showing Compression fracture to the ankle and several other problems. I have been in an air boot for over 6 weeks the last 2 weeks 24/7. I have severe pain when I recline and at night when I m in bed.It that normal? I thought by know that the pain would have eased up. I have put no pressure on the foot for 6 weeks. I m a 62 year old woman. Thank you for listening Doctor: Hello,The symptoms seem to be related to to a twisted ankle. I suggest using anti-inflammatory medications such as Acetaminophen to relieve the pain. I also suggest to use a splint to maintain the ankle immobilized. Cold compresses can also be helpful.Hope I have answered your question. Let me know if I can assist you further. Regards, Dr. Dorina Gurabardhi, General & Family Physician"
},
{
"id": 122568,
"tgt": "What causes strain in the back?",
"src": "Patient: Ok thank u for.taken the time out for me .fri we had a big snow storm n from shoveling i strain my back n I went to the doc off to see if its worse than tht. N they told me not to lift all wk end m take sme advil to help out so I did tht n tht I feel better my pt job employe is asking me to get a note befor I go back to wrk is thtvtru even though its nit wrk related ? Doctor: Hello, As per your history, it may be due to muscle sprain. For further assessment you may require MRI back and nerve conduction studies after consultation. For pain you can take tablet acetaminophen. You may require tablet Pregabalin after consultation. Do regular physical therapy. Hope I have answered your query. Let me know if I can assist you further. Regards, Dr. Shyam B. Kale, General & Family Physician"
},
{
"id": 1836,
"tgt": "Does Duphaston help in getting pregnant?",
"src": "Patient: good eve doc..im 26 yrs old 54kg,5\"3 in height..im irregular since before..i go to the doctor then they tell me i have polystic ovarian syndrome..do i have chance to get pregnant,,the doctor now is telling me to take duphaston..duphaston will help me to get pregnant?what the good time to have sex if i take duphaston,,i hope you can help me doc for my problem.im wating your answer,thank a lot.. Doctor: Hi, if you have pcod problem with irregular periods, then I think you should take some medicines like clomiphene for the growth of your follicles. Track your follicles growth by repeated ultrasound. When follicles reach a size more than 17 mm, take injection for rupturing the follicles. Be in contact with your husband for next 2 to 3 days. Take progesterone for next 2 weeks. Do a urine pregnancy test after that. So, the duphaston which you have been told to take is the progesterone. It helps in supporting the pregnancy.But alone it will not help. Hope I have answered your question. If you have any other query I will be happy to help.Regards Dr khushboo"
},
{
"id": 64384,
"tgt": "Suggest remedy for lumps",
"src": "Patient: hi! for three days now i noticed i have a lump on my face.first, it was located just above the right side of my eyebrow.i thought it was an insect bite after touching it for several times i felt it moves to the center of my eyebrows above my nose area.it disappear overnight, but a lump appears on my right forehead.for the second time around it disappear overnight.on the third time i noticed a small lump above the chin area on the left side below my lips.it's slightly painful when i touched it, what could it be? pls help me thanks! Doctor: HI,Good Evening.Thanks for your query.I studied the query in-depth.I understand your concerns.My diagnosis-?Acne /? mild regressing-pimples / or? Urticaria--Treatment for this multiple moving and re-appearing lumps on your face--Tb-Antihistaminic- / with Hetrazan-Antibiotic for 5 days.-Tb-NSAIDs.Hope this would releive your query.-Wishing you early recovery and good health.Wellcome again."
},
{
"id": 115623,
"tgt": "What causes bleeding & a cramp like feeling?",
"src": "Patient: Hi, I had a lot of blood, now I have cramp like feelings, & ongoing, pulsating. It hurts right in my anus hole now. I had surgery a couple months ago, laporoscopic, removed scare tissue & an ovary. I am on a diet that GI specialist suggested, more fiber & such, I'll start w/that Doctor: Hello,Thank you for contacting HCMI understand your concern, I am Dr Arun Tank, Infectious Disease Specialist answering your query.Such problems are common when on fresh surgery. I advice you to continue the diet prescribed by the GI specialist. You can use Drotaverin as you are having cramp, please take the medication under your doctors guidance.Hurting on anus side is the reffered pain which you are getting at the site of the ovary.Please start some walking on whole day, because once you start walking the pain you are getting is reduced.You can ask for further queries here on bit.ly/DrArunThank you,Dr Arun TankInfectious Disease specialistWe wish you a best health at health care magic."
},
{
"id": 74826,
"tgt": "Can phlegm cause nausea?",
"src": "Patient: I am nauseous, last night I had to make myself puke, and a solid substance that floated, but hit the water like a rock with a sploosh; from there strands sank off it giving it a jelly fish appearance... I felt better after this was out, but only for a few hours and am back to where I started 24 hours later... Doctor: Hi welcome to the health care magic Thick mucus or phlegm can lead nausea feeling Whenever there is damage to respiratory epithelium or inflammation, cough production occurs as reflex mechanism and because of inflammatory mediators excess mucus production occurs Cause for phlegm should be ruled out like.... -Allergy inflammation -bronchitis -pneumonia (if fever, chest pain present) Investigations needed like CBC, Auscultation, chest x ray etc... If needed spirometry done for further work up (only if needed) If these presentation because of infection antibiotic course prescribed.. For excess wet cough expulsion Ambroxol containing syrup can be prescribed Hope your concern solved Consult physician or pulmonologoist for examination and discuss these information with him Take care"
},
{
"id": 8480,
"tgt": "Please suggest treatment for facial spots",
"src": "Patient: hi , i m kaushik 23 yrs old. I had chicken pox 3 weeks ago . I got some spots in my face.I consulted a doctor .He gave me triglow cream ,drantox cap and one sunscreen lotion. Those spots are now becoming reddish . i am fearing that the products might have any side effects or they work like this only.......Plz help me out... Doctor: Hello Kaushik,Welcome to HCM.It seems you have got local irritation to the cream (TRIGLOW) you have used. It called Contact irritant dermatitis.Nothing to worry as your skin will be fine soon.I would advise you a weel course of oral prednisolone in low dose (5-10mg).Apply a mild cortisosteroid like desonide and antibiotic like fusidc acid twice a day.After good quality moisturiser like cetaphil DAM twice a day over full face.Avoid use of any cosmetics/facewash for few days.Avoid direct sun exposure and use water-based sunscreen before stepping out in sun.Hope this was useful.Take care Dr Hardik Pitroda"
},
{
"id": 158514,
"tgt": "Stomach pain, pancreas valve releases insulin three times normal size. Done X Rays and scan. Is it cancer?",
"src": "Patient: Hi Dr Last January I was in the Hospital for stomach pain. It turned out that my pancreas valve that releases yhe insulin was three times the normal size. He was very thorough. He had a sonogram. xrays,rscp scan I think that's what it was. And he put the endoscope ( tube down my throat 0 and said it was fine. Well i've been reading and finding out that pancreatic is almost always fatal. What should I do$ i lost my son to a rare type of brain cancer he got at 14 followed by non hodghkins lymphomia I'm really terrified that this is bad. Im really scared. I have no insurance And don'y know Doctor: Hi, Based on the information provided it seems \" like a benign insulinoma\" which is a cancer of - Beta cells that produces the insulin, and the surgical removal of those cell mass will be a cure of your problem. Its not a malignant pancreatic cancer you have been reading about, just a simple tumor that can be removed and 100% cure is obtained. Now for the insurance issues, some hospital will allow a cheap surgical options, NEEDY MEDS website will help you to locate such cheap location.Hope it helps.Dr. JAY PATEL"
},
{
"id": 224390,
"tgt": "Is there any problem for taking morning after pill?",
"src": "Patient: hi,I just have a quick question...it may sound silly. But end of Jan i had sex with a friend it was protected, i was a bit confused so i went out and got the morning after pill, and my period came on time. I just need to here from a doctor that everything will be ok Doctor: Hi There is absolutely no reason to be worried.The hormones in the pill are washed off from your body in few days time. It has no impact on your future menstrual cycles or fertility."
},
{
"id": 28478,
"tgt": "Suggest treatment for bradycardia",
"src": "Patient: Hi, my heart rate is 46 right now. It has been as low as 38. I have Bradycardia. I had a heart ablation Feb 12 for Atrial Fib. Now I am taking Sotalol. I see my heart surgeon next Friday. How low is too low? I am 70 years old. I have been considered athletic. Before the ablation, my normal low limit on my sport watch was set to 45. Doctor: Hi welcome to HCM.I understand your query and concern.Bradycardia has to be dealt properly through adequate investigations.Its can be treated through procedures like implantable cardiac pacemakers.Drugs like Isoproterenol will help to up the cardiac pace rate.I advise you to have a baseline 2 dimensional echocardiography,ECG and lipid profile to assess the basic cardiac reserve of your heart.Restrict the intake of salt to less than 6g/day.Regular physical exercise in the form of brisk walk for 20 min a day for 5 days a week is pretty useful.Drugs like antihypertensives and antiarrhythmics will help.Reduce the intake of fatty and fried food.One pomegranate will help to keep your heart at good pace without clot formation. Consult a Cardiologist for further expert management.Post your further queries if any.Thank you."
},
{
"id": 84055,
"tgt": "Is overdose of Folsafe harmful?",
"src": "Patient: Dear Madam, I have done a blunder. I have been prescribed Folsafe tablets for 1 at a day. but by mistake i have taken it 3 daily. Is there any problem? I have not conceived and I had gone for treatment that is my periods were irregular. I am married. I had not still planned for a baby. But after the treatment i would like to conceive being healthy. My mother and grand mother are diabetic and I am 25 yrs old. Doctor: Hi,Usually it is not harmful. Folsafe-L contains L-methylfolate, methylcobalamin and pyridoxal phosphate which are water soluble vitamins commonly prescribed to treat their deficiency states causing anemia including during pregnancy.Since these are water soluble vitamins so even if they are taken in larger doses they are eliminated from the body without causing any harmful effects. It will not affect your pregnancy.Hope I have answered your question. Let me know if I can assist you further. Regards, Dr. Mohammed Taher Ali, General & Family Physician"
},
{
"id": 195022,
"tgt": "Suggest treatment for erection problem and low libido",
"src": "Patient: Good afternoon, I m a 21 year old male and I ve been having a very embarrassing problem. Recently the tip of my penis has become numb to stimulation. Sex and masturbatjon both get me 3/4 of the way hard but I feel little to no sensation in the tip of my penis. It has become a problem for me In the bedroom and I don t want my girlfriend to continue to think it s her fault. I also have low libido and rarely wake up with mornin erections like I used to. My balls are often shriveled up and don t hang like they used to. I drink 3-4 times a week and smoke casually but other than that I work out and my diet is fairly good. Please let me know of any remedies to my problem I am becoming desperate for a solution and I m too young for this to be a consistent problem. Doctor: Hi, Numbness at the tip of the penis has to be treated. But there is nothing to worry. Do consult an urologist in person for evaluation of the cause and effect. Hope I have answered your query. Let me know if I can assist you further. Regards, Dr. K. V. Anand, Psychologist"
},
{
"id": 223444,
"tgt": "Can i miss inactive birth control pills to avoid a period?",
"src": "Patient: Hello! So I have been on Tri Sprintec for a year or so now and I recently had breakthrough bleeding about 2 weeks into my birth control pack and it has been a good 8 days of bleeding and now my pack is over today. Can i just start my new pack on sunday without taking the inactive pills for seven days? i really dont want another full 7 days of a period. Doctor: ya..u can miss ur inactive pills..but u wil get your period...as dese inactive pills contain either fe tablets etc...dese r non hormonal...infact if u dont want to get period start another pack without gap...n visit ur gynec"
},
{
"id": 143733,
"tgt": "Suggest medications for chiari malformation",
"src": "Patient: I was DX with chiari malformation 2 weeks ago and have been advised to see a neurosurgeon right away, but when my pcp called they where given an appointment for July 7th. I am having several sx and I m getting worried about my work and traveling in my car. Doctor: Hi,chiari malformations cause a variety of symptoms like neckpain, headache ,cerebellar ataxia ,lower cranial nerve palsies and brainstem and cervical cord compression.For this I would like to examine you and see your MRI films .Medicines to be given are pregabalin , steroids and analgesics .Thanks"
},
{
"id": 27524,
"tgt": "What causes shortness of breath and nerve pain along with fatigue?",
"src": "Patient: I have been having shortness of breath, nerve pain, chest pain, and fatigue. I took samples of my saliva and found these: https://www.flickr.com/photos/0000@N00/sets/0000/ Can you tell me what these are and if they may be contributing to my symptoms? Doctor: Hi,sorry the link you shared shows \"bad link\" address.Anyways, your complains are suggestive of some cardio/pulmonary or other general problem, diagnosis of it would need your brief history, because your age, habits, other disease, predispose your health and current symptoms.For now, i would recommend you to contact your primary care physician and get evaluated by basic cardiopulmonary assessment plans.Regards, and best wishes"
},
{
"id": 57269,
"tgt": "What medication is suggested for Cirrhosis of liver?",
"src": "Patient: I was dedected for Cirrhosis of the Liver ( due to alcohol ) 2 and half yrs back. Conditon is now normal vis a visblood reports, no asities, good USG , good digestion - NO alcohol sinnce diagnosis, no red meats , pleanty ofexcercise etc. Height 6 feet 1 inch, wt 98 kgs ( went down to 85 kgs during illness ) . Does the damaged liver regenerate ? If not then can one lead a normal life with the changes in life style OR does the damage process ( node formation / damage to cells ) continue ? Doctor: The only two things which will help to improve survival is abstinence from alcohol and adequate nutrition. The liver does regenerate but it is a slow process. In some case the liver damage can prrogress inspite of stopping alcohol."
},
{
"id": 13695,
"tgt": "What causes body rashes with burning sensations in the ears?",
"src": "Patient: I was talking to my son in TN today and we were discussing allegies. He has had about 3 episodes of breaking out in a rash but first he has a burning sensation in his ears then gets dizzy and feels like he will pass out. one time it happened while in line at a convenience store another time in a restaurant. He stated that he had not had any thing unusual to eat before his occurence. Any info would be appreciated. Doctor: Hi, Perhaps he had hives. Hives are itchy welts with surrounding flare. There can be associated dizziness and feeling of passing out in a severe reaction. Hives can happen as an allergic reaction to either food or food preservatives, drugs and alcohol. I suggest him to take an oral antihistamine e.g Tab Cetrizine once a day for a few days. Topically he may use a soothing lotion e.g calamine lotion to calm down the rashes. Hope I have answered your query. Let me know if I can assist you further."
},
{
"id": 12044,
"tgt": "Dark pigmentation on face particularly below eyes",
"src": "Patient: Hi my mother,45yrs old has dark pigmentation on face particularly below eyes n on cheeks ,it developed after marriage. even her sisters have same kind of pigmentation so which amway product will reduce it n give good results? Doctor: Hello and welcome to healthcare magic!!! There are a few things to think about before using a cosmetic product. Do your mother or her sisters have any joint pains or any other complaints along with this dark pigmentation? it might be a sign of connective tissue disorder which presents with what is called a butterfly rash seen on the cheeks below the eyes in this fashion. Consult a doctor and get some tests done. take care!!"
},
{
"id": 155072,
"tgt": "Will bladder cancer be cured without surgery?",
"src": "Patient: hello....I have a friend with bladder cancer and refused to let a doctor(S) do any type of surgery he has a license to grow medical weed - he states he is very tired and has friends coming over to his house whon serve as care takers. and still refuses to go to the hospital /do you think he will be okay? He has had it for about 7 years Doctor: HiBladder cancer if detected at an early stage can be cured by surgery.Surgery followed by chemotherapy is a good option.RegardsDr de"
},
{
"id": 147825,
"tgt": "What does mild generalised cerebral and cerebellar volume loss with sulcal prominence at vertex of both hemispheres mean?",
"src": "Patient: My brain scan says this: There is patchy abnormal signal within the deep and subcortical white matter of both cerebral hemispheres especially within both frontal lobes in the anterior watershed distribution. The appearances are in keeping with chronic small vessel disease. the abnormal white matter lesions do not have the morphology of inflammatory lesions. Mild generalised cerebral and cerebellar volume loss with sulcal prominence at the vertex of both cerebral hemispheres. What does this mean please. Doctor: Hi,Thank you for posting your query.I have noted your MRI brain findings. First of all, I would like to reassure you that these findings do not represent any serious brain disease.I need to know your age and clinical symptoms to explain you the significance of your MRI findings.Chronic small vessel disease suggests lack of blood supply to certain parts of brain. The underlying risk factors could be high blood pressure, sugar or cholesterol.Volume loss refers decrease in size of brain. Both these findings are common in older people and tend to worsen with age.I hope my answer helps. Please get back if you have any follow up queries or if you require any additional information.Wishing you good health,Dr Sudhir Kumar MD (Internal Medicine), DM (Neurology)Senior Consultant NeurologistApollo Hospitals, Hyderabad, IndiaClick on this link to ask me a DIRECT QUERY: http://bit.ly/Dr-Sudhir-kumarMy BLOG: http://bestneurodoctor.blogspot.in"
},
{
"id": 6523,
"tgt": "What treatment should one with irregular periods and pcod take to get pregnant ?",
"src": "Patient: pcod irregular periods 37x31mm cyst in LT ovary on11july. 7x7mm inRTovary and 11x12mm inLT ovary on22 july. 11x12mm inRTovary and 8x9mm inLT ovary on27july. what does this mean? IS ovulation happen this time Doctor: Hello Thanks for your query.\u00a0\u00a0\u00a0\u00a0\u00a0 This study is known as follicular study ,it done to monitor the formation of follicle that must be more than 17-20mm in diameter ,before an injection is given for the release of ovum ,necessary for fertilization and pregnancy .As you don`t have dominant follicle ,so chances of pregnancy are poor. \u2018Hope I have answered your query, I will be available to answer your follow up queries, \u201cWish you Good Health and trouble free speedy recovery\u201d"
},
{
"id": 43264,
"tgt": "Have PCOS, irregular menstruation, normal BMI, tiredness, headaches. On folic acid, metformin. Can I get pregnant?",
"src": "Patient: hello doctor owen. pls help me by answering my question. 4 years ago I got pregnant and was aborted because Im not yet ready to have a baby that time ( I was 21 when I got pregnant ). When I reached 25 years old I got married and decided to have a baby..but for few months we got no luck and we seek for a help from a local doctor. I was diagnosed that I have PCOS..My menstruation is very irregular. My BMI is just normal but was advised by my doctor to reduce my weight. I was given few tablets like utrogestan\u201a ova-mit clomifene\u201a metformin\u201a folic acid and vit.b12..I just finished my 2nd cycle of ovamit and I failed to get pregnant. For the 3rd cycle I did not take utrogestan hoping that my cycle will now follow the exact date but sad to say it didn't. My last period was on July 25\u201a2013 but until now I do not have my menstruation. Pregnancy tests every week but negative results.I feel tired most of the times. Light headed and headaches. Do I need to take utrogestan again to induce my menstruation? do I still have the chance to get pregnant? i hope to hear from you at your earliest convenience. thank you very much and God bless. Doctor: Polycystic ovary is a condition in women that can lead to difficulty with menstrual irregularity, infertility, blood sugar control, weight management and increased body hair growth. Many women need to use a medication like utrogestan to induce their menstrual period. If women go without their menses for long periods of time, they have a higher risk of uterine cancer. Taking a medication to induce menstruation is helpful for knowing when to start clomiphene on months that pregnancy is attempted. Clomiphene is used to help women ovulate, and this can be a problem in PCOS, leading to the irregular menstrual periods. The good news is that with proper treatment, the fertility rate for women with PCOS is approximately 80% if no other factors are involved. Metformin helps regulate blood sugar in women with PCOS, helps regulate menstrual cycles, and actually improves fertility as well. I hope this answers your question. I am available for further consultation if needed."
},
{
"id": 36550,
"tgt": "How can blisters be treated?",
"src": "Patient: Hi. Last week I entered a home that was infested with fleas and ended up getting well over 50 bites all over my body. Well now I have blisters in addition to the bites: one on my left knee and 5 on my right thigh. I'm freaked out and terribly itchy/uncomfortable. Help? Doctor: HIWell come to HCMI really appreciate your concern this can be treated with local application of Diphenhydramine lotion and oral anti-biotic, Tab Amoxicillin would be best option 500 mg three times in day, take care and have a nice day,"
},
{
"id": 6678,
"tgt": "What precautions can I take in 2nd pregnancy post d and c due to no appreciation of heart beat ?",
"src": "Patient: Heart beat not yet appreciated for my child in my first pregnancy and after DC we are trying for pregnancy for a year with No successAfter D C periods are delayed and its 47 th day with negative pregnancy testCan u suggest what can we do now Heart beat not yet appreciated for my child in my first pregnancy and after D&C we are trying for pregnancy for a year with No success.After D&C periods are getting delayed and its 47th day with negative pregnancy test. Can u suggest what can we do now? Doctor: Hi, After D&C abortion it's common to get irregular periods for a couple of following months.Iif tthe irregularity persists even after that -you should get your sonography( ovulation study) done .Sometimes ovulation induction is needed after sonography with gynac consultation. thanks"
},
{
"id": 63085,
"tgt": "What is the soft painful lump under the skin on labia major?",
"src": "Patient: I have a softish marble size lump under the skin under the labia major. It's growing in size rather quickly, isn't very painful and feels as though it can move around a little bit (like when I'm checking it out to see if changes are being made). What can this be? Doctor: hi.based from your description, it could be a cyst (possibly a Bartholin's). it is best if you consult with a gynecologist for physical examination and possible medical and surgical management.hope this helps.good day!!~dr.kaye"
},
{
"id": 160357,
"tgt": "Up to what age vaccine can taken for cervical cancer ?",
"src": "Patient: I am 36 years old. I want to know if I should be taking vaccine for cervical cancer . There seems to be lot of confusion, as some doctors and web-sites say it is only for ladies in age group 9-27. Please clarify. Doctor: The vaccine for cervical cancer has recently been introduced. It can be taken from age 10 to 45 years. Three doses are given. At 0, 1 and 6 months. Please consult your doctor."
},
{
"id": 86419,
"tgt": "What causes pain in lower stomach and right testicles?",
"src": "Patient: i haave this pain in the right lower area of my stomach, and it makes mee feel like i have to got to bath room. as well has right testical hurts at times. this also makes me confusied and buzzing in my ears. my ears used to get real hot at night. if I eat something it makes pain all most go completlly away. Doctor: Hi! Good evening. I am Dr Shareef answering your query. If I were your doctor, after a clinical assessment of your abdomen and testicles, I would advise you for a urine routine/microscopic and culture sensitivity test, and treat it with appropriate antibiotic if positive. The UTI sometimes makes the testes inflammed secondarily giving rise to orchitis causing pain. So, I would also advise you with some anti inflammtory along with a proton pump inhibitor drug, and a good scrotal support with advice for proper rest till you are relieved of the pain. Also consume enough liquids in your diet.I hope this information would help you in discussing with your family physician/treating doctor in further management of your problem. Please do not hesitate to ask in case of any further doubts.Thanks for choosing health care magic to clear doubts on your health problems. I wish you an early recovery. Dr Shareef."
},
{
"id": 187849,
"tgt": "How to treat painful throbbing gums with bleeding and having calcium spots on them along with swollen left jaw and white oozing bump?",
"src": "Patient: I have had 3 Children and while pregant and even after i started losing my beautiful heathy cavity free teeth they started hurting,throbbing,gums bleeding and having calcium spots on them but worst of all the starting chipping and falling out.I have had dentist look at my teeth and they said theyve never saw anything luke it befor where extreme vomitting caused dental damage like this so they refused to work on my teeth unless i admitted i did drugs i told i wasnt on nor have i ever dont any illegal drugs but even after my medical records were faxed they called the police on me and refused to let me leave tilk cops arrived the police told they i wasnt on any drugs and that i could sue them then they said we will qork on your teeth for $5,225 if i didnt sue them and if they could use photos of ny teeth for conferences but now my teeth are worse and im scared because i can not afford dental work to restore my teeth and i have a swollen left jaw because of a white ozzing bump on my gum and what feels like a pulp inside my gum around my root and it bleeds swells and has my jaw sticking out swollen and really sore please help me. Doctor: Hello, Thank you for consulting with HCM.Actually while pregnancy there are some hormonal changes which affects gums and tooth and make them infection prone.Because of which there are swelling of gums and tooth also become loose and prone for caries.That is why you experienced these throbbing and bleeding gums. Now as you are feeling that there is swelling on left side, it looks that the infection of tooth has spread to periapical region and an abscess has been formed.You have to visit another dentist to get it all treated as this condition can not be healed only by medication.Hope it will help youThank you"
},
{
"id": 29253,
"tgt": "Do body ache, fever and fatigue need medical attention?",
"src": "Patient: I have began the following symptoms 5 days ago; minor body aches, low grade temp, extreme fatigue, loss of appetite. The fever has subsided but have also developed redness and skin irritation on my face, neck and hands and inside my mouth. I returned one week ago from a weeklong trip to Cuba. Should I seek med attention or assume it is a virus? My health is excellent. Doctor: HI, thanks for using healthcare magicViral infection is the most common reason for these symptoms. You can consider monitoring them for now unless they increase in severity or do not appear to improve.Fluids and rest along with acetaminophen (paracetamol or tylenol), would help.I hope this helps"
},
{
"id": 132722,
"tgt": "What is the cause for tingling and numbness in the right arm and the right side of the face?",
"src": "Patient: I am experiencing tingling in both of my legs and arms. I have tingling and slight numbness in my right arm and the right side of my face that sometimes radiate down my whole right side. But lately, as of a day or two ago i began to also experience tingling now in my left arm and now lightly on both legs. can you tell what cause it and i should do?? Doctor: Hi Hope this message finds you in good health.I have gone through your complaints and understand your concern.Tingling sensations are caused by sort of hormonal,calcium or vitamin deficiency. Another possibility is neuropathy,that can be cured by pregabalin tablets.You can get an Xray and Nerve Conduction studies to find out whats wrong.Nothing to worry about.I hope your question has been answered.If you have any follow-up queries,feel free to consult me anytime.Thanks,Take care,God bless."
},
{
"id": 94012,
"tgt": "Dizziness, intense abdominal pain, sweating. Can a blocked carotid artery cause abdominal pain ?",
"src": "Patient: My dad was recently seen by a physician for dizziness, intense abdominal pain (for 2-3 minutes), then sweating and feeling he was going to fall. Dr. took x-rays of some sort, saw calcification along his spine and sent him to his cardiologist for testing. They have discovered his right carotid artery is completely blocked (don t know whether internal or external) and are going to do a CT scan today to determine the percentage of blockage in other arteries. Why would a blocked carotid artery cause abdominal pain? Should we be looking for something else (abdominal aneurysm , etc.)? Doctor: Hi welcome to Health care magic forum. Thanks for calling H.C.M.F. Your father consulted the doctor for dizziness, abdominal pain, and sweating. hi is diagnosed with calcification along his spine, and cardiologist diagnosed blocked carotid artery, and ordered for C.T.scan for looking for the block of other arteries. He may need to have a color doplor for exact diagnosis of blocks of other arteries. If necessary your cardiologist may order latter. If there is a block of the abdominal arteries, one may get abdominal pain, due to ischemia of the abdominal tissues. Wishing for a quick recovery . Best regards."
},
{
"id": 86723,
"tgt": "What causes pain in upper abdomen and headaches?",
"src": "Patient: Hi, may I answer your health queries right now ? Please type your query here... \u00a0why does my high abdomen hurt i dont have a period but i have a headache sometimes when i see the sun,i have mood swings and all but why do i feel like this? Doctor: Hi.Thanks for your query. Your poosting detailed and relevant history helps the Doctor to help you more. Anyways, with the history you have provided, of pain in high (upper abdomen),. no periods yet, mood swings, headache when you see the sun. The probable causes of mood swings is obviously hormonal. The pain in teh upper abdomen can be due to acidity , anxiety or stress, Get PPI , Domperidone, Medicines for anxiety . should help ."
},
{
"id": 151134,
"tgt": "Paralysis in right leg, hand. Brain angiography shows haemorrhagic infract in left hemipons. Cure?",
"src": "Patient: my father is 65 years old .He is suffering from paralysis on right leg and right hand since five months. Recent MR Angiography of brain and neck vessels showed acute haemorrhagic infract in left hemipons. please suggest to clear the infract in left hempions.Due to this he is still not able to do his actvities by himself .He walks with difficulty using a walker and is not able to use his right hand . Doctor: Hi, Thank you for posting your query. Infarct in the brain occurs due to lack of blood flow to that part. Any attempt to clear it successfully can only be done in the first 4-6 hours after the onset of paralysis. After that, it is not possible to clear the clot. However, recovery and improvement can occur slowly over time, with good physiotherapy and exercises. Best wishes, Dr Sudhir Kumar MD DM (Neurology) Senior Consultant Neurologist"
},
{
"id": 122413,
"tgt": "What can cause severe pain in the shoulder after swimming for long?",
"src": "Patient: I have a lot of pain in my left shoulder.... I work at the great wolf lodge as a lifeguard.. for our certification test we had to swim 10 laps in our pool... I have never swam that much in my life... I\u2019m concerned I did something, I feel if I was sore it would be both of them Doctor: Hello, The symptoms seem to be related to a muscle strain. I suggest using anti-inflammatory medications such as Acetaminophen to relieve the symptoms. I also suggest using anti-inflammatory cream for local application such as Voltaren cream. Hope I have answered your query. Let me know if I can assist you further. Regards, Dr.Dorina Gurabardhi, General &Family Physician"
},
{
"id": 108070,
"tgt": "Suggest treatment for back pain and stomach pain",
"src": "Patient: stomach pain, back pain. i got my first depo provera birthcontrol shot in june22,2011 a week after receiving the shot. i started to have stomach pains and back pains. its now october 21,2011 and its gotten worse my obgyn gave me antibiotics. and not happened. i dont know what too do its getting worse everyday. i can barley get to sleep. Doctor: I have gone through your question and appreciate Your concern. First don't neglect your helth.Consult orthopaedic spine specialist And Get examined if required Mri of spine Should be done to rule out any pathology.For pain in abdomen do usg of abdomen.General measures which will help u areAvoid bending forward and lifting heavy Weights. Hot fomentation thrice a day. Local Analgesic gel for local application Muscle relaxants and analgesics will help you. After relief of pain muscle Strengthening exercises Will help u strengthen your back muscles. Bed rest and lie on hard bed.Thanks U can write me back for any query."
},
{
"id": 85395,
"tgt": "Is it safe to take opana er while on other medications?",
"src": "Patient: My mom was recently perscribed Opana ER and I read online about the serious side effects of this drug. She s on other medications and I ve read that she shouldn t be on them while taking this drug. I m concerned can someone please explain this more to me? Doctor: Hello,Drugs may interact with one another and these interactions can be from mild to severe ones. I do not have the names of the other medications that your mom is having together with Opana ER so that I can say specifically which are the possible interactions and risks.Kindly reply and let us know of all the medications that she is using so we can further assist you.I hope I have answered your query. I remain at your disposal in case further medical assistance is needed.Regards,Dr. Antoneta Zotaj,General and Family Physician"
},
{
"id": 119436,
"tgt": "Can vitamin D deficiency lead to knee pain?",
"src": "Patient: Hello Doctor I am namrata I m 31 years old post my delivery in 2009, my knee pain started and gradually increased and when I had and X-ray I got to know the knee cartilage shrink in both knees; but I never had any major or minor accident before. Also I had Vitamin d deficiency for which i have already taken injections. i am taking treatment for this since 1 year butnot getting relief. Please suggest me any remedies thru which I can restore this back as I don t need to go for surgery at this early stage Doctor: Hello,At your age thinning of cartilage is not a common condition. If you are having knee pain then most common reason at your age can be overweight and lack of exercise. You may start exercises, vitamin D deficiency can lead to knee pain but for osteoarthritis, it is not a established reason. You may do warm fomentation with water bag over your knee with exercises for at least 3 times a day. There are some supplements which can also be helpful like glucosamine, chondroitin and collagen supplements. Take care. Hope I have answered your question. Let me know if I can assist you further. Regards, Dr. Mukesh Tiwari, Orthopedic Surgeon"
},
{
"id": 35885,
"tgt": "What is the treatment for weakness and 1-3 pus cells after treated typhoid?",
"src": "Patient: hi my mothers age is 58 yrs old.She is having 99 to 99.5 degree temp in the morning and this is continuing forlast 15 Days. she has typhoid in 1/80. And she has tAken medicine for 10 days. she is also feeling weak.In urine examination report pus cells are 1-3/Hpf. how to get her allright Doctor: Dear i can understand that your concern . I would like to inform you that urinary tract infection is very common specially in females . As you have mentioned your mother has temperature more than 99 in morning this is suggestive of fever and this shows she has some infection that need to be searched and adequately treated . Get urine culture report done and treat that if any organism grows in that.Look for any other cause of fever as well such as any infection in lung or any painful site.Discuss this with your physician and get expert opinion . Hope this will help you . If you have any other concerns please feel free to message me . Thanks for contacting HCM ."
},
{
"id": 115036,
"tgt": "What does sudden elevation in BP during night mean?",
"src": "Patient: my blood pressure suddenly rose to 170/110 for the last few days. It happened at midnight and when i was not in stress. I am taking metaprolol 100mg twice a day and nifedipine 60 mg a day. My day time pressure is around 145/90. Please let me know why it spike at night? Thank you. Doctor: HII UNDERSTAND YOUR CONCERNS.LATE NIGHT OR EARLY MORNING SURGE IN BLOOD PRESSURE IS BECAUSE OF CIRCADIAN VARIATIONS IN BP IN HUMAN BEINGS.EVEN IN NORMAL HUMAN BEINGS BLOOD PRESSURE IS SLIGHTLY IN UPPER RANGE OF NORMAL AT LATE MIDNIGHT AND THIS EFFECT IS PRONOUNCED IN HYPERTENSIVES.I WILL STRESS UPON YOU THAT THIS RISE IN BLOOD PRESSURE NEEDS TO BE CONTROLLED TO PREVENT FURTHER COMPLICATIONS.SO I WILL RECOMMENT YOU HOME MONITORING OF BP .YOU CAN BUY ONE AUTOMATIC ELECTRONIC BP APPARATUS.THEN VISIT YOUR PHYSICIAN WITH THESE BP READINGS AND GET YOUR DOSAGES OF ANTIHYPERTENSIVES ADJUSTED.I HOPE MY ADVISE WILL HELP YOUTHANKSDR PRANAY BUDKULE"
},
{
"id": 216348,
"tgt": "Suggest medication for pain prior to carpel tunnel",
"src": "Patient: Good evening. My 73 year old husband will be having carpal tunnel surgery in 4 days. He was told to not take any ibuprofen or aspirin in the week prior to this surgery. We understand that this is due to the blood thinning problem, however his pain is intense. Tylenol does not seem to help, and we were wondering if he could take a 1/2 hydrocodone tablet. (10-325 T) This were prescribed for him when he previously had his other wrist operated on. Thank you for your time. Doctor: HelloThanks for using Hcm.You can take the hydrocodone tablet if it is available with you.This has no contraindications pre operatively.It can reduce pain as well as anxiety for surgical procedure.All the best.Take care."
},
{
"id": 165192,
"tgt": "What causes worm like particles in feces?",
"src": "Patient: Hi, may I answer your health queries right now ? Please type your query here... My 15 month old has clear particles shaped like a grain of rice(slightly larger) worm like appearance in his feces. They do not move. Appear to be jelly substance. What is this? Doctor: Dear Parent,The jelly like substance in your child's stool seems to be mucus which keeps our intestines moist and lubricated. Small amounts of mucus in stools is normal. However you will need to consult a pediatrician if the amount of mucus in stools increases or is associated with blood and abdomen pain."
},
{
"id": 16454,
"tgt": "What should I do for the tenderness and burning sensation in the breasts planted with silicone bags and saline filler ?",
"src": "Patient: Ive had saline breast implants for ten years with no problems In august i was in a car accident that the air bag deployed and hit my right breast it did not rupture my implant but it was advised to have them replaced My surgery was March 28 replaced with silicone bags and saline filler ive taken my antibiotics and worn my surgical bra The right breast incision is in crease that did get infected plus i was bitten by a spider on my hip the purpose of this email is because my right breast is insanely tender and occasionially has a burning sensation as well as my skin being red under my nipple appears to be red where the bag is what do you think I am a 37 white female. Health good, not allergic to anything. i was taken keflex after surgery and the cream i was using for spider bit was bactroban cream. i was given scar fade cream that i put on my right incision probably twice. i just read the back of the scar fade cream that says if causes redness stop using. Doctor: hi you need to stop scar fade cream as it may cause such reaction over skin. dont rub over that area during bath. wear cotton cloths bactroban is enough for bite. if u dont get response ,better to consult doctor"
},
{
"id": 217775,
"tgt": "What causes severe pain in the upper quadrant after eating?",
"src": "Patient: My mother is 67 y/o and having right upper quadrant pain, had gallbladder removed 10 years ago and a history of bile duct stones. After she eats, pain worsens. LFT's were drawn yesterday and Alk Phos was 111, ALT was 318, AST 261, Total bili 1.6, Direct Bili was 1.0. Her eyes and skin have a slightly yellow tint. Would another bile duct stone cause this elevation in LFT's and should she go to the ER, her GI specialist is out of town? Doctor: This surely shows that she still has some problem with bile duct with obstruction due to stones. Its better to grt in touch with a gastroenterologist and he may do an ERCP to remove such stones. If nit done problem mey increase in severety."
},
{
"id": 30791,
"tgt": "Suggest treatment for infertility",
"src": "Patient: my age is 36,height-5'2, MENS CYCLE 26 DAYS,trying for kid last 3yrs back .this month folliculars stdy report-day 10,date-19/11/10,Rtovery-12.8.Ltovery-18.4mm ,,,day11,date20/11/10,Rtovery-20.1mm,Ltovery-24.3mm,ET -8.1,,TAKEN THE INJECTION-HUCOG 10000 AT NIGHT.,,DAY-13,DATE-22/11/10,Rtovery-25.3mm,Ltovery-30.1mm,ET-9.8,Again taken the injection Hucog 10000 at noon,STILL ITS NOT RUPTERD.what shall i hope for this mont kid Doctor: Hi, Thanks for posting in HCM. I have read your description and understand your concern. HCG is a hormone, which maintains the pregnancy. If its levels are low during especially in the 1st trimester, there are more chances of abortion. Hence, it is recommended to take injections regularly, especially during 1st 3 months. Based on the follicular study, it appears that the maturity of follicles seem to be good and Hucog has been administered at right time. Follicles would be ready for rupture in about 24 to 48 hours following injection.Kindly have intercourse for 3 to 5 days following injection, which should improve the chance for conceiving. Kindly follow the advise as given by your doctor and hope for the best. Hope the information provided would be helpful. All the best."
},
{
"id": 154460,
"tgt": "Suggest treatment for endometrial cancer",
"src": "Patient: It has been 9 months since I had a total hysterectomy for endometrial cancer. Stage 1, Vigo grade1/2. I have been experiencing pelvic pain intermittently these last few weeks, and they seem to be getting a bit more frequent. Is this normal? Should I be concerned? They are on the right and left sides, in the groin area. Thank you Doctor: Hi Thanks for posting. Endometrial cancer removed 1 year back does not require any treatment now. However, the new onset pelvic pain needs further evaluation. You need a gyne-onc evaluation and imaging of the pelvis with CT/MRI to rule out recurrence. If these are normal, then only symptomatic treatment is required. Hope this helps Regards"
},
{
"id": 144842,
"tgt": "What causes a pop in the head with acute headaches?",
"src": "Patient: I know this sounds weird.... I am 42 years old with low blood pressure, in great health but last night I got really mad and felt a pop in the top of my head and have had a headache for the past 24 hours with no relief from advil. I called my Dr office this afternoon but she is on vacation this week. The pain has yet to subside but I don t have any other symptoms that would indicate a brain bleed or that this could be life threatening... thoughts? Should I get a CT or MRI to be on the safe side? Doctor: Thank you for asking Healthcare majic. My name is Dr Ehsan Ullah & I have gone through your query. well sudden onset and severe headache has got many causes like SAH,migraine,trigeminal neuralgia...As the history is not much clear i will like to suggest you to get visit as soon as possible to medical consultant to find out the underlying cause of your symptoms.. If this is worrying to you then definitely go to ER and seek medical advice for CT or MRI..Meanwhile kindly get some pain killers and relaxing exercise should be learnt from psychologist...If you are having chronic headache this exercise will provide you much relief.Don't delay to visit your doctor. Hope this may help you. Let me know if anything not clear. Thanks."
},
{
"id": 190745,
"tgt": "Painful ulcer on gums, taking becosules, cause. Can I use smile gel?",
"src": "Patient: Hi, I am a 27 year old female. I developed an ulcer on the tongue last week which healed after applying smile gel. Now a small white circle has formed on my lower front gum and on the skin joining my lower front gum and lip. It is extremely painful. Will smile gel heal this as well? Also are canker sores or gums sores the same as mouth / tongue ulcers? I had started taking becosules a week before? Could the becosules have caused the tongue ulcer and gum sores, as I very rarely get them otherwise? Thank you for the help. Doctor: Hi,Neerajaji, Welcome to HCM, It seems that you might have apthous ulcer. Only smile gel is not enough. Go for antibiotic medicine and B.complex and lactobacillus sachets. Do salt water gargle frequently. Ok and bye."
},
{
"id": 6892,
"tgt": "I am not able to conceive for the past 1 year. Does it mean I have fertility problems ?",
"src": "Patient: hi. Ive been of birth control pills for 8 months and my boyfriend in I are trying to conceive. We have been trying since feb. Could this be a fertile problem??? In can I have sex and get pregnant the same day I ovulate?? I have a 6 year old son ( not his child ). We would of have a child in 2009 but had a miscarriage . Doctor: Hi,Jasmon,thanks for query.I am sorry but what I understand that you are on birth control pills and now wants to become pregnant.I hope that you have stop birth control pills.If not please stop it for at least 3 months and then try to conceive.Normal ovulation is must for conception.Try in the middle of cycle so that you have more chances of becoming pregnant.bye."
},
{
"id": 99149,
"tgt": "Is the fatty area above collar bone due to allergy?",
"src": "Patient: Ihave had itchy arms legs to point of scratching til bleed over2years Dr did rast test ,showed high allergiy but no exact substance,notisced fatty swelling above centr of collar bone last week,not painful just looks as if put on weight,forgot to mention to GPas was distracted getting rast test result,he has put me on 5day steroid course for itching and stopped my statins for a week to see if this could be cause of itching could this fatty area be related to my unknown allergy Doctor: Could be. But the most suitable to such of my patients have been Ayurveda's Haridra khand, manjishthadi kwath liquids and some natural application on body while bathing such as Fuller's earth or multani mitti.These can be used without any risk and on your own. Contents are natural. May be slow in action but would be better acting than your history given above. No side effects and are economic too.May meet Ayurveda doctor for further guidance.Hope it helps you."
},
{
"id": 25844,
"tgt": "Can vomiting of blood in patients with portal vein thrombosis curable?",
"src": "Patient: i am 27 years old. i have portal vein thrombosis since 2000 when i vommited blood first time, i got treatment through barley, olive oil and honey, then in 2006 again i vomitted blood 2nd time and treated and recovered. now once again in 2012 i have got a blood test which shows low Leukcoyte and platelets count i-e internal bleeding.....this time i did not vomit blood. what is should to do...is this curable or just controlable. can i get rid of it permanently. Thanks for your valuable advice. Doctor: hello,I have gone through your query.Thanks for using HCM.You are developing portal hypertension secondary to portal vein thrombosis.You may be developing hypersplenism proucing low WBC and Low platelet counts.You muat consult a gastroenterologist for further evaluation and definitive treatment.My best wishesDr.Rajesh Teli,MD."
},
{
"id": 48332,
"tgt": "Suggest remedy for pain due to kidney stones",
"src": "Patient: My mother is 77 years old and suffering with a lot of pain from kidney stones. The stones are calicified. She cannot have surgery due to heart. She is given pain meds but they just help her sleep. The doc says her kidney is full of stones. What I want to know is what is having this many stones with very little urination do to her? Can I do anything to help her? Doctor: Hi, I am sorry for the situation your mother is in. Apart the medications her doctor are prescribing to her, I advise to:- encourage her to drink plenty of fluids (especially water)- give her fern stone tea to help ease her symptoms Wish fast recovery!"
},
{
"id": 3088,
"tgt": "What chance of getting pregnant?",
"src": "Patient: Hi, i just had sex with my partner but he ddnt come inside of me but after a few hours he urinated and washed his penis with soap and we had sex for the second time around and he did come but didnt put it inside of my vagina , my question is, can i get pregnant with that situation? Is there sperm left inside of his penis even after he urinated and is sperm present in pre ejaculated fluid? Please answer thanks. Doctor: sometimes there are chances that you may get pregnant while the chance is very less.few sperms may be present in preejaculte fluid."
},
{
"id": 58461,
"tgt": "CT scan shows cyst in liver. Had abdomen pain. Have a cyst wall irregularity. Have Vascular flow. Treatment?",
"src": "Patient: I had a recent CT scan without contrast a few weeks ago of abdomen due to extreme pain- there was a simple cyst in liver detected measuring 2.8 x 2.8 x 3.0cm. I had a follow up ultrasound that showed hepatic cystic lesion within right lobe. There is thick septation within the cyst. There is cyst wall irregularity which may reflect additional septations which are not visualized in their entirety. Vascular flow is noted adjacent to the cyst. The radiologist is recommending tat I consider tissue sample due to complex character. Doctor: Hello, Thanks for the query to H.C.M. Forum. As in C T SCAN diagnosis , confirmed so only option is type of cyst and its cause. This is cyst with many septum. Most probably this type of cyst is known as Hydatid cyst and this is the most common cause of cyst in the liver. 2nd reason may be traumatic cyst ,but this type of cyst is usually large in size and without septum. Hydatid cyst usually due to a parasite either from pet dog or unripe salad . As you mentioned only cyst so very difficult to say something about the cause of cyst. Any how only liver biopsy will find out the exact cause, so consult a surgeon and get it done. Good luck. Dr. HET"
},
{
"id": 197888,
"tgt": "Why do I get so many wet dreams?",
"src": "Patient: i am a boy of 23 year.i have a very big problem that i have suffering from excess wet dream.i have lost my weight,semen,strenthand memory.i have consulted many docters and specialists .but i could get any permanent solution for it .please guess me ,what is the proper causes of nightfall or wet dream.and please give me solution for it.thanks Doctor: Hi,Dear,Thanks for your query to HCM.Dear I read your query and reviewed it with context to your query facts.I understood your health concerns and feel Concerned about them.Based on the facts of your query, you seem to suffer from-High testosterone activity(High Blood levels of testosterones),with Excessive desire without any option for natural outlet for ejaculation in males,is the cause of Wet dreams.Lack of knowledge,fantasies about sex acts and excessive involvment may cause this in and around puberty years.Besides this, GI diseases with Gases and Hyper-urination / increased frequency of urine,may cause these wet dreams.With lack of more information from your side, I would suggest to have Sex Counselling from psychiatrists or clinical psychiatric assistants.Hope this would resolve your health issues in the best way possible. Welcome for any further query in this regard.Good Day!!Dr.Savaskar M.N.Senior Surgical SpecialistM.S.Genl-CVTS"
},
{
"id": 8228,
"tgt": "How to get rid of pimple scars?",
"src": "Patient: I have lot of pimple scars,how to get rid of it? I have been having lot of pimples from my teenage. It used to be big pus filled ones. I used to pick the pimples and now as a result iam having lot of scars on my face. I have tried all kind of creams and lotions. I have used many packs on my face but still the scars are more. Doctor: HI\u00a0 That is not a big problem, not need to anything what to do is ill tell you one lotion use it regular after your bath apply as a creams call me this is my number 9703005423, 100% Use I hv Used ready tht product dont worry Be Happy"
},
{
"id": 106456,
"tgt": "Respiratory tiredness with shortness of breath",
"src": "Patient: Im 17 years old & I've got respiratory tideness & shortness of breath.And I was suffering from the cold,hayfever associated wheezing or asthma from about 10 years old period Till july 21st this year.And it was not an chronic condition,it got cured by one or two doses of medicine.But now I'm suffering from chronic chest tideness from about 3 months.I took broncholidators but it doesn't work.The doctors are telling that I've got no problem by taking X-rays & ECO scans & my spirometeral rate is at about 600.But I struggle in day today activities like walking,talking,exercising.I don't have any allergens.But I smoked about 6 months before 2 months I got this symptoms Doctor: The symptoms that you are typical of Respiratory Disease. We advice you to see a Pulmonologist for the correct diagnosis of your condition. It could be an Obstructive or Restrictive variety which can be decided by Spirometry and later to diagnose the correct condition, a diagnostic steroid therapy is put or a Brochoscopic biopsy can be done. The treatment could be longterm but will show results and put the attacks under control."
},
{
"id": 157663,
"tgt": "Had vomiting, infection, constipation, kidney issues. Done endoscopy. Had bladder cancer, received Bcg",
"src": "Patient: My father is in the hospital for over a week. He is unable to digest food, had vomiting, infection that can t be identified(White blood cell count was extremely high - but has now dropped), kidney issues-possible infection, constipation, etc. Today they did an endoscopy and said they could not see much because there was so much undigested food. Tomorrow, they are going to give him a barium test. They also said they were looking for some sort of blockage in the stomach. What could he have? My father had bladder cancer last year and was cleared and received BCG treatments just to be safe. Doctor: Hi and thanks for the query,The problems in the stomach could be explained by a blockage generally called an intestinal obstruction. The past history could suggest the fact that maybe due to spread of a tumor, an intestinal region was affected, and the tumor mass is blocking the flow of food. It is also possible that abnormalities in the electrolyte metabolism, generally with low potassium levels could lead to constipation. Endoscopy and serum potassium should be checked first. thanks and hope this helps. Kind regards"
},
{
"id": 215157,
"tgt": "Suggest remedy for pain below rib cage",
"src": "Patient: Hi I m 22 and been getting really bad pains below my right rib cage for around 3 week s now it got that bad last week that I got rushed into hospital with it, the doctors did a ecg and checked for blood clots and I also had a xray nothing showed on results so they put it down to a chest infection and gave me some antibiotics, I completed the course but the pains still there for the past 3 days I ve been hearing a loud growling noise from my tummy and now I have pale and mucusy stools with blood and advice please as I m getting worried ans have 3 children to look after and I m struggling with day to day activities thanks, Leanne Doctor: Hello,I read carefully your query and understand your concern. Your pain seem to be related to the presence of gas in the colon. I suggest using Simethicone medications three times a day. I also suggest using a spasmolytic to relieve the pain.I recommend to drink liquids and eat a plain diet.Hope my answer was helpful.If you have further queries feel free to contact me again.Kind regards! Dr.Dorina Gurabardhi General &Family Physician"
},
{
"id": 53219,
"tgt": "Does the use of Fentanyl patch improve food intake and alertness?",
"src": "Patient: Hi my sister in law is end stage liver disease. She has been incoherent and unable to eat, speak. However, they put her on a fentanyl patch and she is now more alert and eating somewhat? is this due the the patch or what the call predeath rallying (if there is such a thing)? Doctor: Hi and welcome to Healthcaremagic. I understand your concerns..Well, it may only indirectly lead to these activities that you mention. Because fentanyl parch will reduce pain and anxiety and in this was it may help patient to eat better and be more relaxed in less pain. In every case, she should continue with fentanyl pacth if there is visible improvement. I dont think this is predeath rallyng definitely, this should not concern you. This is serious disease, but these are not sign of terminal days so I am sure it will be fine.I hope I have answered you queryKindly regards. Wish you a good health."
},
{
"id": 119499,
"tgt": "What causes swelling and pain in elbow after an injury?",
"src": "Patient: I had a football game last night, i had my arm fully extended reaching for a player then another player tackled the guy i was extending for land on my arm and caused a lot of pain. It is now swollen with pain at times. What do you think i did to my elbow? Doctor: Hello, It looks that you have a outstretched fall on your arm. There can be injury to your bones around your elbow joint. I shall advice you to at least have a x ray of elbow to rule out a bony injury. Till then you may have a good analgesic to have relief from pain. Take care. Hope I have answered your question. Let me know if I can assist you further. Regards, Dr. Mukesh Tiwari, Orthopedic Surgeon"
},
{
"id": 99678,
"tgt": "Suggest remedy for allergic rashes & hives",
"src": "Patient: Hello. I am a 26 year old female (5'2, 125 lbs). I have a severe bee allergy (honeybee, yellow jacket, and white-faced hornets). I am also allergic to cephalexin. Two days ago I broke out in an allergic reaction (rash and hives) from the waist up around 4:00 PM. I was treated at the ER, and my rash went away over night. Yesterday, the same reaction happened aroud the same time. I thought it could have been from mold in the library of the school where I work (where I was on the days of both reactions), but today the reaction happened again, and I was not in the school building. The reactions are happening at the same time each day. I have not eaten anything new, and I am not using any new products of any kind. What could be causing a reaction at the same time everyday? Doctor: Hello,Thank you for asking at HCM.I went through your history and would like to know more about you as follows: a. In past, have you ever had such hives? b. How your allergies to insects and cephalexin were diagnosed? c. Do you have any other complaints along with hives, like respiratory complaints, nose complaints, stomach complaints, etc? d. Do you have any repeated digestive complaints? e. Do you have any other medical conditions? f. Do you need to take any other medications frequently/repeatedly? Above information would help me to understand your complaints better and therefore to make suggestions more specifically. However, at present, I would like to make suggestions for you as follows:1. I usually suggest my such patients taking regular antihistamines like cetirizine/levocetirizine in morning and hydroxyzine in evening for 1-2 weeks depending upon response.2. I would also suggest you to apply calamine lotion over the itchy areas/rash.3. Please do not scratch the itchy areas as it would aggravate itching.4. To identify/suspect the cause, in addition to above asked details, some investigations such as complete blood counts, ESR, stool examination, urine examination, etc would also help.Hope above suggestions will be helpful to you.Should you have any further query, please feel free to ask at HCM.Wish you a very good recovery and the best of the health ahead.Thank you & Regards."
},
{
"id": 10538,
"tgt": "Suggest treatment to reduce hair fall",
"src": "Patient: Hi doctor i am 22 yr old my name is swati. My hair fall incrasing in day to day ,plz suggest me hair effective hair oil to prvent my hair fall and scalp.Doctor recently im using Livon hair gain tonic,but it is not effective for me.plz doctor solve my hair gain problem Doctor: Hello and Welcome to \u2018Ask A Doctor\u2019 service.I have reviewed your query and here is my advice.I advise my patients of diffuse hair loss to undergo 2 blood investigations: serum ferritin and thyroid function test because abnormalities in these are a common cause for diffuse hair loss. If this turns out to be normal, start with a hair supplement like Folihair tablet once a day after meal for 30 days, change your shampoo to a milder one and use conditioner on hair shaft post washing for 5 minutes and rinse. Livon hair gain tonic changes hair shaft texture and has no effect on hair root and hence, no effect on hair fall.Hope I have answered your query. Let me know if I can assist you further.Regards, Dr. Vanya Narayan"
},
{
"id": 98509,
"tgt": "Suggest medication that can be taken with Singulair for allergy",
"src": "Patient: hi , I am on singulair for allergies and my doc wants me to take an otc along with it. currently taking the generic of Claritin, but not working as much as I would like to relieve symptoms, mainly nasal congestion. would you have a recommendation? would like to stay with 24 hour pill. also, I cant smell anything, would something stronger help that? thank you Doctor: You can try allegra allergy 24 hour tablet(otc) once a day and watch for relief in such cases.IF you are also having blocked nostrils,instill xylometazoline nasal drops only for 3 days and stop."
},
{
"id": 72313,
"tgt": "What causes pain while taking breath?",
"src": "Patient: Good evening,I drank last night about 3-4 liters of coke, barely ate anything today, but now i got extreme pain when i breath deeper. I looked it up on the internet and i saw trapped gas. I tried the big burp but it didn't work at all. I have a family history of heart problems. What can it be. Doctor: Thanks for your question on Healthcare Magic.I can understand your concern. Aerated soft drinks like Coke can cause acute gastritis. And this in turn cause bloating of stomach and burning chest pain. So better to take pantoprazole (antacid) twice a day on empty stomach.Avoid coke or other aerated softdrinks in future. Avoid hot and spicy food. Avoid junk food. Don't worry, you will be alright with all these in 2-3 days. Hope I have solved your query. I will be happy to help you further. Wish you good health. Thanks."
},
{
"id": 5871,
"tgt": "What are the chances of getting pregnant after having a laparotomy?",
"src": "Patient: Im 29 years old and just wanted to know What are the chances of getting pregnant after having a laparotomy ... I had the surgery on March 2010 when I was 3 months pregnant! My son is now 2 and my husband and I have been trying for the past 6 months but no luck yet. I m scared to think that I won t be able to conceive anymore Doctor: Hi, Thanks for your query, To answer your query, we will need details of operation. What was exactly done and what structures were removed. If i presume it was done for ovarian cyst/tumour which is a common cause of laparotomy at this time of pregnancy. In this case also even if one ovary or its part is retained during operation, then chances of pregnancy is there. Go through the operation notes. Best of luck. Hope it helps.."
},
{
"id": 163844,
"tgt": "What causes lack of appetite in a child?",
"src": "Patient: Dear doctor, I am chandra shekar from bangalore. Sir my 1yrs old child is not eating properly at all he will take atleast 1 or 2 hours time to eat & in some times he wont eat anything pls tell any solution which makes every children eat s time to time properly. Thank you Doctor: dear userabout your question i would like to tell you that loss of appetite in infants has some causes and we should find your possible cause of them....we have two common possibilities of that....first is psychological factor ..if the infant has psychological or emotional troubles that will lead to loss of appetite examples of psychological troubles is being punished for eating...if you tryed to force him to eat his desire for eating will decrease as there is someone always try to feed him even if he want and so his sense of appetite decreased because of that......2nd cause is your infant may has iron deficiency anemiaand this anemia causes always loss of appetite ...this anemia is common at that age as iron administration not enough in milk so we always give prophylactic iron therapy to infant at age of 10 month as the anemia will begin.........wish your baby healthy life"
},
{
"id": 52310,
"tgt": "What can cause elevated ALT and AST levels?",
"src": "Patient: Why does my ALT & AST keep going up I have lung cancer but its been shrinking. I have been doing well, my numbers have been up since I ve had the last 3 months of this amuno therapy they drop a little and yet they can t keep it down in order to give me a new med, so they put me on Prednisone and that does t seem to be working all that good is there a natural solution I thought I ate pretty good fish veggies fruit lots of water I just don t want this cancer to start to grow again now that I have been through all the treatments of chemo down to the amuno therapy they won t give me any meds unless my liver enzymes drop, its my ALT & AST Doctor: Hello, Liver enzymes may be raised because of various causes. In your case, some may be more possible but in every case it requires evaluation of the cause. Most common causes of such liver derangement's are Alcohol, Viral hepatitis, Non-alcoholic fatty liver disease, Drugs. Gamma GT is most commonly raised due to alcohol intake and in case you take alcohol, you need to keep a control over it. If you don't take it, you need following tests: hepatitis virus markers, Ultrasound Abdomen and HIDA scan. A lot many times, liver dysfunction is due to Non-Alcoholic Fatty Liver Disease (NAFLD). So, if you have any evidence of fatty liver in Ultrasound and your other tests are negative, this rise is liver enzyme would be attributed to NAFLD. For this you need to reduce your weight, reduce fat in diet. Hope I have answered your query. Let me know if I can assist you further. Take care Regards, Dr Ivan R. Rommstein, General Surgeon"
},
{
"id": 3040,
"tgt": "Will pregnancy be complicated due to irregular periods?",
"src": "Patient: Hi I got married 4 months back.For the last 2 months I had irregular periods and i checked for HPT and it was negative.For this month i hav not got my periods and checked negative this time too.I visited a gyno and she has prescribed Regestrone for 3 days .Then she has prescribed Folinz tablet and Siocare Syrup for 30 days.I am worried is there any complicacy in me getting pregnant.Before marriage I had regular periods. I am not overweight and I look normal. Doctor: Hallow Dear,To start with, let me discuss about the medicines you have been prescribed. Registrone is a retroprogestational medicine which helps controlling the excessive bleeding or inducing the withdrawal bleeding when taken for short period. If taken continually cyclically, it helps regularising the menses without affecting the ovulation. Folinz tablets are tablets of Folic Acid and Zinc. They help the healthy growth of the tissues. Siocare Syrup is an Ayurvedic medicine which is used for the treatment of Menstrual problems, Bleeding Gynaecological conditions, Reproductive system problems, Bleeding disorders, Diarrhoea, Eye disorders and other conditions. Siocare Syrup contains Ashwagandha, Asoka, Chandan, Daruharidra, Dashmoola, Dhataki, Guduchi, Jeeraka, Lodhra, Musta, Shatavari, and Udumbara as active ingredients. Now let us turn to your main issue of fertility and the irregular menstruation. By and large, the women with irregular menstruation are known to face problems in egg release; however, it is not a rule. If you are overweight also, having hairiness at the unwanted places of the body along with irregular menses, please check for Polycystic Ovarian Syndrome which is known to have ovulation problem and result in infertility. At this stage, I would advise you to monitor your ovulation. Ultrasonography would be option of choice for this. If you find evidence of egg release, you should not be concerned about conception problems. Try to plan your pregnancy at and around the day of egg release to achieve the pregnancy. If ultrasonography reveals the failure of ovulation, you will have to undergo treatment of ovulation induction. Please report to your Gynaecologist. Dr. Nishikant Shrotri"
},
{
"id": 52831,
"tgt": "Why did my post partum check up show AST 62 and ALT 92?",
"src": "Patient: Hello doctorI just delivered a baby befor 6 week.During Postpartum checkup everything bilirubin alp Alubimin shows normal except my AST is 62 and ALT is 92. I don't have any liver disease and i don't have any problem in pregnancy. I took progesterone tab until 12 weeks of pregnant. Doctor: Hi and welcome to Healthcaremagic. Thank you for your query. I am Dr. Rommstein, I understand your concerns and I will try to help you as much as I can.SGOT and SGPT are sensitive indicators of liver damage or injury from different types of diseases or conditions, and collectively they are termed liver tests or liver blood tests. So the next step is to look for certain liver damage and most common casue is fatty liver disease. It should be confirmed by ultrasound. Other causes are viral hepatitis,cirrhosis, medications, alcohol intake or autoimmune diseases. To verify exact cause, US, bilirubin levels and tumor markers should be additionaly done Then appropriate treatment can be started.I hope I have answered you query. If you have any further questions you can contact us in every time.Kindly regards. Wish you a good health."
},
{
"id": 161393,
"tgt": "Are frequent stomach aches, bladder infections and headaches in a child a matter of concern?",
"src": "Patient: My 6 year old daughter frequently experiences stomach aches, bladder infections, pain under her ribs on the left side and occasional headaches. She s often pale and tired, has a loss of appetite, weak and shaky in the mornings and isn t gaining weight. Her regular family doctor doesn t seem to concerned besides treating the bladder infections but I m not convinced she s ok. Doctor: Hello, By what you say, we need to think about other possibilities like recurrent viral illness or chronic viral illness like epstein-barr virus infection. We should also think about upper urinary tract infection like pyelonephritis which may be recurrent. Hope I have answered your query. Let me know if I can assist you further. Take care Regards, Dr Sumanth Amperayani, Pediatrician, Pulmonology"
},
{
"id": 125713,
"tgt": "How can pain and tingling sensation in the leg after a back surgery be treated?",
"src": "Patient: HI, I HAVE A PROBLEM WITH MY RIGHT LEG, I HAD SURGERY ON MY BACK ABOUT A YEAR AGO, AND SINCE THAN I HAD A PROBLEM WITH MY RIGHT THIGH,IT S VERY TIGHT AND IT HURTS A LOT, NOW I GET TINGLES THAT GO DOWN TO MY KNEE, AND MY LOWER BACK IS ALWAYS SORE. I AM 75 YEARS OLD I AM IN DECENT SHAPE, BUT NOW I DON T KNOW HOW TO HANDLE THIS PROBLEM, I AM GOING TO SEE A SPECIALIST NEXT WEEK AND HOPEFULLY HE WILL BE ABLE TO HELP ME OUT,SOME DAY IS WORSE THAN OTHER. ANY SUGGESTION PLEASE? Doctor: Hello, It could be due to spine related problems like disc prolapse or nerve compression. Consult an orthopedician and plan for an MRI scan. Hope I have answered your query. Let me know if I can assist you further. Regards, Dr. Shinas Hussain, General & Family Physician"
},
{
"id": 38902,
"tgt": "Can low white blood cell count cause thrush?",
"src": "Patient: i have been experiencing re-occuring constant thrush for over a year now. the skin down below is very red and it burns and swells during and after sex, making it almost impossible. i have been given various treatments but nothing clears it up. my blood test just came back as having a low white blood cell count. can this be the cause of or be affecting the thrush? Doctor: Hi,Welcome to HCM.There are chances of low immunity if your white cell count is low, which could be predisposing you to opportunistic infection like thrush. But there are other factors also which play a role. I suggest you get a vaginal swab tested for fungal culture and sensitivity and then get appropriate antifungal treatment based on the test results. I would like to stress here that antifungal therapy has to be continued for atleast 6 weeks to prevent recurrence.Thanks."
},
{
"id": 213422,
"tgt": "May have Asperger's syndrome. Any advice?",
"src": "Patient: Good Afternoon, We believe after talking to some people that our 29 year old son may be suffering from a mild form of Asbergus symdrome. Reading about it online he seems to have many of the signs. He lives with his girlfriend who has been very supportive & sympathetic to him so far! He has eventually (but not happy about it) agreed to go see a doctor with her. Have you any advise..is this the first step..Thank you.... Breda Hayes Doctor: Hello As per me Asperger syndrome should be diagnosed Only after detailed evaluation , for this an expert is required. So it is a good decision that his girlfriend has planned to see a doctor for it. It would be best if she consult a psychiatrist specifically."
},
{
"id": 79341,
"tgt": "What causes constant chest pain?",
"src": "Patient: i have almost constant chest pain . Sometimes light sometimes extreme sharp pain in either side of my chest. I also get an ache in the lower left side of my neck, not sore but uncomfortable. I also notice it seems to worsen or be brought on by the slightest strain when using the toilet. I have had a camera down my throat and have been placed on nexiam 40mg for approx 2 months now. Doctor: Thanks for your question on Health Care Magic. I can understand your concern.By your history and description, possibility of GERD (gastroesophageal reflux disease) related chest pain is more. But better to first rule out cardiac cause for chest pain. So get done ecg and 2d echo. If both are normal than no need to worry for cardiac diseases. You are taking nexium. It is having pantoprazole. Better symptomatic relief, along with pantoprazole, you need to follow these lifestyle modifications. 1. Avoid stress and tension. 2. Avoid hot and spicy food. Avoid junk food. 3. Avoid large meals, instead take frequent small meals. 4. Go for walk after meals. Keep 2-3 pillows under head in the bed to prevent reflux. 5. Avoid smoking and alcohol, if you have these habits. 6. Loose weight if you are obese. Don't worry, you will be alright. Hope I have solved your query. Wish you good health. Thanks."
},
{
"id": 117523,
"tgt": "What causes addiction to mint when suffering form anemia?",
"src": "Patient: I have all symptoms of intrinsec factor anemia I got iron inyections for long time and it did not help me much now I am getting so bad that I can not function I have ann appt with an hematologist en few days my concern is ... Why I am so attracted to mint that is becoming like an addiction? Is there some spetial prpperty in the mint leaves that could cure me Thank you Doctor: Hi,Thanks for asking.Based on your query, my opinion is as follows.1. Iron deficiency is known to cause pica.2. In this condition, patients are known to ingest paint, varnishes, mud etc.3. Mint could be due to pica. 4. On correction of deficiency, mint addiction should go.Any further queries, happy to help again."
},
{
"id": 180158,
"tgt": "What is the ideal treatment for typhoid fever - Zanocin or injection?",
"src": "Patient: My daughter is 5yr old.she suffering typhoid fever. doctor advice me to give zanocin 100 liquid 6ml for 10days 2twice daily. or I give typhoid injection to her.what is better for her? using zanocin 100 any side effects? which one is better for her injection or zanocin 100.plz advice me. Doctor: Injection Typhoid is only for prevention.For treatment she needs Antibiotics.It's fine to continue with Zanocin. If child is active, eating and alert antibiotic injections are also not needed."
},
{
"id": 53417,
"tgt": "Suggest treatment for elevated levels of GGT",
"src": "Patient: I had a GGT test 5 years before and it was as low as 13. Today I have repeated the test and it is 67. I need the result not to be higher than 50 because I have to submit it to the embassy. I have been taking nexium for over 4 years. so, I stopped it for 9 days and then measured the GGT. I was surprised that it increased to 90. Could Nexium be causing the problem ? and how long I have to stop it in order to the GGT level return to normal? Doctor: Hi,GGT is a marker of liver disease. Neksium is a proton pump inhibitor, there is no documented evidence of interaction of Neksium with liver causing effect on levels of alkaline phosphatase. However if you feel like you feel better on Neksium, then you can continue with it. However, I would advise you tests for liver to find out the reason for rise of the liver enzymes. PPIs have no significant effect on it.Hope I have answered your query. Let me know if I can assist you further.Regards, Dr. Ramesh Kumar"
},
{
"id": 84732,
"tgt": "Does Cartigen forte,Diacerein and Deflazacort have any side effects?",
"src": "Patient: Hi Doctor,My mother have knee pain and we have consulted one doctor and he prescribed some medicines. The medicines include Cartigen forte ,Diacerein and Deflazacort. Please can you tell me for what purpose we have to intake these tablets.And whether there are any side effects having these. Doctor: Hi,1. Cartigen forte is a nutritional supplement used in osteoarthritic patients. It reduces pain, disease progression and improves joint mobility.It also timulates cartilage synthesis & delays cartilage degeneration and breakdown. It is common side effects include bloating sensation, constipation or stomach upset.2. Diacerein is used to relieve pain and swelling associated with osteoarthritis. Its common side effects include stomach upset, diarrhea, vomiting, liver dysfunction and allergic skin reaction3. Deflazacort is a steroid used in the treatment of inflammatory conditions of arthritis. Its common side effects include gastritis, peptic ulcer, high blood glucose, high blood pressure, weight gain, osteoporosis, and cataract.Hope I have answered your query. Let me know if I can assist you further. Regards, Dr. Mohammed Taher Ali, General & Family Physician"
},
{
"id": 131547,
"tgt": "What causes frequent bruising with lightheadedness and lack of energy?",
"src": "Patient: Recently started bruising easily and feel somewhat flat and lacking in energy. Occasionally light headed but still have plenty of strength when weight training.. I take a vit B complex and also added an iron supp in the past week but still bruising easily. No other symptoms. Added some red meat to diet also..no change. What is going on?! Doctor: HiYou must see a clinical hematologist.A blood test for liver function, Prothrombin time, bleeding,clotting time may be done along with CBC with total platelets count.If any deficiency is found it can be managed"
},
{
"id": 45675,
"tgt": "What causes excessive fatigue and loss of appetite during kidney dialysis?",
"src": "Patient: My mother, 79, is undergoing kidney dialysis, has other issues (primarily diabetes-related issues), has recently become moodier, more lethargic, has no appetite at all, and is showing signs of Alzheimer s. Her meds change often. Would there be any benefit in getting her to a Gerontologist? Thank you. Doctor: Hello, I work in a hemodialysis center. Based on these symptoms, first I would check for anemia (Hemoglobin level, hematocrit and iron studies) and second for dialysis adequacy (blood urea nitrogen pre and post dialysis). If the results are acceptable, then your mother should consult a Gerontologist. Hope I have answered your query. Let me know if I can assist you further. Regards, Dr. Elton Halili, General & Family Physician"
},
{
"id": 184115,
"tgt": "Is white discharge from the tooth sockets infectious?",
"src": "Patient: I had all my teeth extracted about 3 weeks ago for dentures. There is a white discharge from one of the sockets and it's still painful-is this infection?I'm 52, 160 lbs, and received antibiotics immediately after extraction because there was some type of infection already. In general I feel better but some of the site is still extremely painful. Doctor: Thanks for using Health Care Magic.Read your query.Multiple extraction sites takes time to heal.The white discharge as you have mentioned suggests infection in the socket involved.I would advice you to visit the oral surgeon and get it reviewed ,followed which they can drain the pus in the socket which will induce the healing process.You may require a course of antibiotics including amoxicillin 500mg and metrolag 200mg( if you are not allergic to any medicines) on prescription by your dentist.Continue salt water gargling or any antiseptic mouth wash gargling.Hope this was useful and a have a speedy recovery.Thanks and regards."
},
{
"id": 75512,
"tgt": "What are symptoms of pneumonia?",
"src": "Patient: My 16 years old breathed in her spit by accident and has since (over the last three days) developed a chesty cough (no cold) which is getting progressively worse. My elderly father once did this and it turned to pneumonia. Should I be concerned about it turning to an infection in her lung/bronchial tubes in my daughter or is the cough coincidental? Thank you. Doctor: Hi welcome to HCM....Here chest pain , fever , dyspnea like symptom not present....So chance of pneumonia less. ..But it could be upper respiratory tract infection or mild bronchitis...Investigate her with .....1.CBC with absolute differential count 2.Auscultation 3.Chest x ray only if needed Here decongestent like phenylephrine given for relieving congestion and cough ...Water drinking more Excess fried food avoided ...If cough worsen than sputum cytology and throat swab along with aso titre done Advise ; pulmonologist consultation..."
},
{
"id": 221663,
"tgt": "What are the symptoms of potential pregnancy?",
"src": "Patient: I had Unprotected Sex The Day I Got Off Of My Period (9.12.2010).. A Week And a Half Later My Teeth Are Very Sensitive When i Chew On Anything And My nipples Are Very Sore And Erect. On Top Of This I'm Constantly Sleepy During The Day But Wide Awake At Night. I Also Have Been Urinating A lot More Often And I Have Excessive Discharge.. Are These Early Signs Of Pregnancy.? Or Is It To Early To Tell.? Doctor: Hi dear, I have gone through your question and understand your concerns. Unprotected intercourse can lead to pregnancy. The symptoms of excessive tiredness, sleeping, engorged breasts are symptoms of early pregnancy and can start appearing10 days after conception.Hope you found the answer helpful. Wishing you good health.Dr Deepti Verma"
},
{
"id": 88315,
"tgt": "What causes abdominal pain and nausea?",
"src": "Patient: I have been having stomach pains for too long. I went to the doctor about a month ago and they told me I had chronic indigestion. But now, I can t have sex because my stomach pain increases and on top of that I get a bad nausea. after I eat I get the feeling of going to the restroom. After I drink water is the same thing. I lost weight because I haven t been eating all the fried foods I used too. What should I do?? Doctor: HI.Thanks for your history and query. Having loss of sex-life in addition to the problems you have explained is a serious matter . The most probable causes are :Irritable Bowel Syndrome.Abdominal tuberculosis. Nonspecific Gastroenteritis. I would advise the following investigations and treatment in a such a case:Investigations:*Routine blood investigations like Complete blood picture, blood sugar, urea, creatinine, liver function tests, thyroid function tests.*Urine -routine and microscopy*Stool: routine, microscopy, occult blood, culture and sensitivity.*Ultrasonography of abdomen, and CT scan of the abdomen.X-ray of the chest to see for TB or other problems.*Colonoscopy All these investigations should be within normal limits for one to say you have IBS.Once a proper diagnosis is made , it will be easy to get a proper treatment, that will cure your problem, improve your health and sex-life."
},
{
"id": 26128,
"tgt": "Is it alright to have sex while having SVT?",
"src": "Patient: is it ok to have sex with svt? Hello I have recently started having irregular heartbeat as high as 180 . I have had it about about 10 times in a year. sometimes it goes away fast sometimes it last hours. My cardio thinks i have svt and would benifit from an ablation. I am not really wanting to do it. I would rather control this with meds and stress diet controlling. Most of my episodes come from stress. Is it ok to have sex with my wife or should i play it safe. Also do some of you agree ablation is risky? Im 33 4 kids and a wife who need me . I cannot take a chance to leave thhem. Doctor: This SVt is a very symptomatic you did not tell me if you are on any medication you should be on betablocker to prevent auto control the rate if you are in beta blockers then you may be safe to have sex ablation is not that dangerous and SVT is a very easily corrected by it if you are not controlled by medications then you should definitely get oblation disc is very low"
},
{
"id": 173798,
"tgt": "Is it safe to use fluzet syrup, meftal and muco-salvin syrups for fever?",
"src": "Patient: hi my 3.6 years old son is suffering from cough and high fever.earlier i was giving him crocin and ibujesic syrups alternetively betweer 6 hours .i went to the another doctor and he gave 1.fluzet syrup 2.meftal p syrup 3.muco-solvin syrup.i want to know is it safe for him? Doctor: Hi , Thank you for your query. Do not worry. It is usually don't give serious negative effects. Fluzet contains Antiallergic medication(Clorpheniramin) is known to cause hyperactivity in children. Kindly put him also some nasal drops in his nose and ask him to gargle 4 times a day with a medicated gargle.That should take care of the problem. Fluzet can give nausea,allergic reaction,skin rash. If your baby will have the following side effects then you should stop giving this medication. Meftal is more effective for fever.As any remedy it can give skin rash, urticaria, and edema. After overdose dizziness and other Central Nervous System manifestations such as drowsiness, visual disturbances and convulsions can occur. It may be useful to take Muco-salvin during the day and Fluzet at night, to help get an undisturbed sleep. Muco-salvin rarely can give stomach upset, diarrhea, constipation, heartburn, nausea, indigestion and vomiting.Hope this answers your query.Wishing your child a speedy recovery.Have a nice day."
},
{
"id": 144230,
"tgt": "Is the artifact within a sulcus in the frontal lobe causing vertigo and dizziness?",
"src": "Patient: hi, I had an mri report and it shows a small blooming artifact along the left surface of the frontal lobe-within a sulcus ?? thiscould represent an old small focus of bleed-cavernoma but could represent a calcification. Ct was recommended to establish the difference. Do you think it is necessary to do the ct scan ? I had 2 episodes of severe vertigo and vomiting and was taken to the hospital.I am still dizzy and I try to find out what is wrong with me .The conclusion of the mri report was:no explanation for vertigo and vomiting.What do you think? thank you for taking my question ! Doctor: Hi, I am Dr.Bruno and Let me answer your query.The MRI Findings in Left Frontal Lobe may not be not related with Vertigo and Vomiting I would suggest you to do a CT Scan Brain as Suggested to fix the diagnosis If you need any clarification / have doubts / have additional questions / have follow up questions, then please do not hesitate in asking again. I will be happy to answer your questions."
},
{
"id": 8409,
"tgt": "Suggest methods to bring the buttocks to a good shape",
"src": "Patient: I have an unusual shaped buttocks. I have been self conscience about my body because my butt looks like a hump on top and it has like dents on the side. I don t remember it being this shape when I was younger. What can I do to change this besides cosmetic surgery? Do you think it s a medical issue? Doctor: hi..there are different ways to improve buttocks shape..but unfortunatly except exercise there is no other way to do this..there are surgical and non-surgical alternatives to give fuller, more rounded buttocks:Exercises like squats will build the gluteus muscle and tone the buttocks.Surgical butt enhancement treatments include fat transfer, butt implants, and the Brazilian Butt Lift treatment.Implants have the advantage of adding permanent volume to the buttocks, but has higher risks for infection, seromas and malpositioning.The fat transfer procedures (including Brazilian Butt Lift) is less invasive than implant augmentation, and the required liposculpture helps contour the gluteal area. A disadvantage is that the amount of volume that remains after a year varies from person to person.hope you got your answer,,,thanksDR IHSAN"
},
{
"id": 44914,
"tgt": "Can a person have a child with the following semen report : volume 2.5ml colour gray white reaction alkaline motility 60% ?",
"src": "Patient: i have a person with me whose test report for semen is as follows.He has not been able to have a child for a long time. His report states volume 2.5ml colour gray white reaction alkaline motility 60% viscosity thin sluggish 10% dead 30% total count 80 millions/cumm others few puscell s/hpf millions/cumm Doctor: Hi binu; welcome to HealthcareMagic The report you have posted is not normal;ask your friend to consult a Urologist/Andrologist for examination ;he will require treatment to increase the motility of the sperms.After treatment if the motility improves to more than 50% and the reports of his wife are normal then he will have a chance to become a father. Thanks"
},
{
"id": 12324,
"tgt": "What is the treatment for eczema and psoriasis?",
"src": "Patient: Hello I have scabs on my forehead which looks like eczema or psoriasis I also have it around my war and on my elbows.. I've seen 2 doctors who don't think it's either. I was given daktakort which worked in the summer but its now came back and it's no longer working Doctor: HIWell come to HCMLet me tell you something skin condition some time needs longer duration of treatment and drug regime goes on changing from time to time some times this become difficult for patient to follow this and eventually disease does not cured, in my opinion if this is chronic condition then better to continue the treatment what ever given by the dermatologist, again skin condition demands the clinical examination for proper diagnosis, till then you can try \"Miconazole cream with Mometasone \" apply this three times in day, hope this information helps, take care."
},
{
"id": 141721,
"tgt": "What is the treatment of dizziness if had hit head ?",
"src": "Patient: i banged the top part of my head really hard about2 hours ago there is a small blood mark where i hit it and i went very dizzy with a thumping head, i now feel very drowsy and a heavy left arm and heavy head and keep thinking it is sunday when in fact it is saturday Doctor: If you are having these symptoms, you probably should be checked out in person, and have a CT of the head performed.While in most cases things will be OK, it is better to be safe than sorry."
},
{
"id": 100721,
"tgt": "Are steroids suggested in case of skin allergy?",
"src": "Patient: Hi, My relative has the below problems: 1. skin allergy (rashes and heavy itching) since 2 years 2. suddenly last month his BP dropped to 30/60. Was unconscious but things were under control after 2 days. since then no BP problem now 3. After this currently he is facing the problem of ulcers inside his mouth since last one month. He is not able to eat anything spicy and hard. 4. he is also under stress due to some family problems We have consulted 7 doctors but no luck, recently one skin doctor suggested steroids and one tablet to calm him down. Want to know the consequences and also about the disease he is suffering from. Please help... Thanks in advance Doctor: Hello.Thank you for asking at HCM.I went through your relative's history and would like to know moredetails about him like:a. What is his age? b. I would like to know about the episode of reduced BP - how it happened, what was the diagnosis. c. I would like to know any other medical conditions he may have as well as any other medications he may be taking on frequent/regular basis. From your given history, I would like to make suggestions for him as follows:1. Were I treating him, I would prescribe him regular anti0histamines like levocetirizine and hydroxyzine. I may add montelukast to his regular treatment if no response with above.2. I usually prescribe a short course of corticosteroids if his itching & rashes are very severe and do not respond to above treatment. Once they are controlled with corticosteroids, they may be safely withdrawn under doctor's guidance. 3. I would also suggest him a good multivitamin supplement containing adequate vitamin B complex. 4. I would also suggest him to apply lotion like calamine over itchy areas.5. Stress can aggravate allergies, hence I usually suggest my stressed patients regular meditation & yoga to reduce stress.Diagnosis requires detailed evaluation of all his symptoms including his past medical history and family history. Hope above suggestions will be helpful to you.Should you have any further query, please feel free to ask at HCM.Wish him the best of the health.Thank you & Regards."
},
{
"id": 210824,
"tgt": "Suggest medication for depression with nose bleed",
"src": "Patient: ive never really felt depressed before, but last week for two days, i felt really depressed and tried to feel better, and little things that shouldnt bother me bothered me. but i had this wicked nose bleed about two days into my depressed feeling, and it felt like i wasnt depressed at all, and felt very relieved. Doctor: Hello,Thanks for choosing health care magic for posting your query.I have gone through your question in detail and I can understand what you are going through.To diagnose a depressive episode the illness has to last for at least 2 weeks. Two days is not sufficient to diagnose depression. Probably that was somestress reaction and the fact that you are fine now, suggests that you dont need any therapy as of now. Regarding the nose bleed you need to see a psychiatrist for the same. Hope I am able to answer your concerns.If you have any further query, I would be glad to help you.In future if you wish to contact me directly, you can use the below mentioned link:bit.ly/dr-srikanth-reddy\u00a0\u00a0\u00a0\u00a0\u00a0\u00a0\u00a0\u00a0\u00a0\u00a0\u00a0\u00a0\u00a0\u00a0\u00a0\u00a0\u00a0\u00a0\u00a0\u00a0\u00a0\u00a0\u00a0\u00a0\u00a0\u00a0\u00a0\u00a0\u00a0\u00a0\u00a0\u00a0\u00a0\u00a0\u00a0\u00a0\u00a0\u00a0\u00a0\u00a0"
},
{
"id": 161959,
"tgt": "What causes drowsiness, sore throat and stomach ache following fever in children?",
"src": "Patient: hi, my 4yr old was awake all night with a temp it has gone down since last night.but he seems very dosey + is complaning of a sore troat and stomach pains.2 nights ago he had a bit of a temp again went down with calpol but was complaning about his teeth what could be wrong with him in your opinion any help would be greatly appreciated. Doctor: Hi, Look all these are symptoms suggesting of infection. He might be having Pharyngitis/Tonsillitis? You can do the following things: 1. Just give lukewarm water, vegetable soups. Thus maintain hydration. 2. Need to start giving a course of Antibiotics, Paracetamol, B-Complex Syp., Antihistamines. 3. Proper Rest & healthy diet shall be helpful. 4. Monitor temperature 4 hourly will be helpful to assess prognosis. 5. If required basis investigations: CBC, BMP, RBS can be done. Hope I have answered your query. Let me know if I can assist you further. Regards, Dr. Ajaygupta009, General & Family Physician"
},
{
"id": 177921,
"tgt": "Suggest remedy for redness and cracks in labia in toddler",
"src": "Patient: My daughter is 2 years old, and she has eczema on her inner elbows and behind her knees. She never had a diaper rash at all except for one time when she had diarrhea. I usually used the Desitin rapid relief every so often as a preventative measure. When she got red in her labia area and her crack, I asked my husband to buy some more Desitin. He bought the extra strength kind. I did not notice a difference other than the texture of it was not as creamy. I kept putting it on to help, but it seemed to get worse or at least not get better. At the store, we looked at the other kind of Desitin I used to buy and saw that it said it was unscented. We ve always used unscented bath wash, lotion and detergent for her. I thought that must be it, so we bought the unscented kind and threw the other one away. It s been 3 days now, and it still looks bright red in her labia area! I don t know what to do! Doctor: If eczema u may use CLOBATE-GM skin cream combination of steroid and antibiotic and wet dressings...do not apply steroid on genitalia...hydration followed by application of emollients like petrolatum ...syp levocetrizine 1.25 mg (2.5ml ) once un 24 hours if itching is severe otherwise no"
},
{
"id": 125553,
"tgt": "How can colitis be treated while suffering from neuropathic pain?",
"src": "Patient: Hi I am in hospital with suspected colitis, I have sle lupus, Sj\u00f6gren s syndrome, intersetional cystitis and fibromalgia, it started 3 weeks ago with worst fibro flare I have ever had, then neuropathic pain, then ic flare with my bladder then 2 weeks ago started having what they said was over flow have lost just under a stone in 2 weeks from passing green, yellow water from bowel had to wear adult nappies as kept messing myself, then yesterday 3 times passed no poo just about a cup full of orange jelly and blood, the pain was severe, this morning was sick but hardly anything in me to bring up, X-ray at hospital today shows I m jam packed full of poo they said bloods showed no inflammation markers awaiting ct scan results haven t passed anything since last lot if jelly last night with blood, on morphine to control the pain in my abdomen, loin and lower back what could this be ? Please any suggestions welcome, I have had long history of blockages and inflammation in my bowel x Doctor: Hello, Consult a gastroenterologist and get evaluated. Probably you will improve with antibiotics. Hope I have answered your query. Let me know if I can assist you further. Regards, Dr. Shinas Hussain, General & Family Physician"
},
{
"id": 76714,
"tgt": "Suggest medication for strep throat and cough",
"src": "Patient: Hi my name is Karolina I'm 19 years old,my weight is 52kg and I do not have any big health disorders in history. Last week I got a flu, I had a temperature and got sore throat. After 2-3 days it went away. I took Nurofen Cold&Flu tablets twice a day and Strepsils tablets. But now I got this strange continuous coughing ,it increases at night time especially! Whenever I inhale I feel a light pain in the middle of my chest which forces me to cough. Should I be worried? Doctor: Hi thanks for contacting HCM...Here you could have viral or allergic respiratory infection...As mild chest pain associated possibilities of bronchitis present ...CBC examination done to rule out lymphocytosis as in viral and eosinophilia as in allergic condition...Symptomatic management done....Excess fatty , fried food avoided.Take antihistaminic drug for inflammation and congestion relief....If blocked nose present then oxymetazoline nasal spray useful ..Steam inhalation also help in soothing effect....Take ginger juice with honey one tsp....If fever more with continuous chest pain then chest x ray needed to look for consolidation....I am giving you here advise according to history but it will not replace the auscultation that is needed .....Take care .Don't worry....Hope my suggestion will helpful.Dr.Parth"
},
{
"id": 19476,
"tgt": "What causes pain in the sternum and left arm?",
"src": "Patient: I have a hx of MI with stents; last night I had sternum pain radiating to left, light headedness, left arm pain down to elbow; after SL NTG I was able to lie down and go to sleep Today, my sternum is still uncomfortable with pain in left shoulder blade intermittently and light headed again. Could this be another MI or just angina? Doctor: Hi,Given your history this can be MI or angina again.I would advise please go and see an ED physician.It would be of worth doing an ECG + troponin during an episode of chest pain. Wishing you good health.Regards."
},
{
"id": 181022,
"tgt": "How to treat a small skin tag on the gum line?",
"src": "Patient: I have a small flap like those discussed below, but unlike these, this one is above the gumline and near my canine. It is also not small (like one I have on the opposite side of my mouth in the same spot). One is about 1/4 of an inch long but the larger one is about 1/2 inch long and 1/4 inch across. The larger one is sore, but I don't see how this could be from a bite. I do use an electric toothbrush and it is also possible that I cut it on a food item at some point, since trauma seems to be the cause according to other posts. It doesn't really both me other than it is sore, so I'm wondering if it will just go away to some degree or if I really need to get it removed. Because it is above the gumline, it isn't really in a place that would be bitten. Is there any danger to just leaving them? Doctor: Hi..Thanks for the query. Well, you should see a specialist to address the issue in the near future. While it very well could be a normal finding, it is difficult to say anything without more information, and especially an examination and possible biopsy. I would like to know, if you use or abuse any substances such as alcohol or tobacco, have been exposed to human papillomavirus, the medical history of your family, and whether or not there is a family history of rare disorders or head and neck cancer). Also it is important to know that the mucosal tags are painful, growing, bleeding, changing colour.If the answer are no then nothing to worry about it..Whaetever your answers to those questions are, you should be seen by a physician who can look at and feel the lesion to see if there is anything more ominous present. Some cancers will begin with small skin growths similar to what you are describing. It is also common to have small papillomas or other benign swellings of the oral mucosa. The best way to know for sure and to avoid something serious is to see a physician soon.Hope this helps..Regards.."
},
{
"id": 88400,
"tgt": "What causes upper abdomen pressure?",
"src": "Patient: Upper abdomen pressure constant in the midlle worse after eating even small meals...not sharp pain...dull pressure feeling that radiates to back...lower joints hurt when at its worst...abdomen becomes warm to the touch....not Hausas or die aria....no change with that.....do have fateige and light headed at its worse Doctor: HelloThere is a dull pain in the abdomen , even after small quantity meal and sometimes radiate in the back , also feeling hot flushes , fatigue and lightheadedness.All these symptoms may be due to these possibilities , these include:1 The first possibility is of hyper acidity or gastritis ( chronic , since pain is dull ) . Diagnosis can be confirmed by upper G I endoscopy . Try raising the head of your bed about 4 inches with block. It might also help to avoid eating or drinking for 2 hours before you lie down. To control stomach acid ( gastritis ) one should not drink alcohol or drinks with caffeine in them or eat chocolate or spicy or greasy food. Also take some antacid but if severe pain you may need drugs like pantoprazole-D once or twice in day .2 The next possibility of such symptoms may be of pancreatitis , as it may cause upper left abdominal pain which is worse when bending over or sitting.Pancreatic enzyme like serum amylase & lipase should be measured to rule out any pancreatic cause.3 The next possibility is of due to over eating ( however , you mentioned even a small meal produces this pain ) or due to tension or anxiety .In my opinion you should consult a physician and get his opinion because physical examination , clinical history is important in such cases as you are also having fatigue and lighheadedness ( if your age is >50 years high blood pressure may develop such symptoms ).Good luck"
},
{
"id": 160653,
"tgt": "Suggest remedy for rashes and bumps on legs and thigh",
"src": "Patient: my 28 month old daughter came home from the in-house day care yesterday with a fever, was fine when i dropped her off. her nose was runny and dried on her face, after her bath i gave her childrens benedryl 5ml and childrens motrin 5 ml she barely wanted to drink anything, she had a small snack and went to sleep while i ran erands. she was not warm when we got home. while sleep she woke up with a fever at 330am i dont have a thermometer gave motrin she woke up at 830am ready to play and very happy all day til went to bed at 8 for 2 hours warm gave motrin and benedryl she ate now she is sleep for the night. i noticed a rash on her legs looks like a little bite on the front of her right thigh, thighs and legs have small raised bumps??? Doctor: Hi,This as a viral upper respiratory tract infection (cold with rash is mostly viral origin). Nothing to worry. Continue benadryl (this will also relieve any itching from the rash) and motrin (more aptly paracetamol) for 3 days. You can apply calamine lotion over the rash 2 times a day to soothe the area. Most probably she will be fine by then.If fever is not coming down by then or if she develops breathing difficulty, kindly take her to doctor.Hope I have answered your question. Let me know if I can assist you further. Regards, Dr. Muhammed Aslam T. K., Pediatrician"
},
{
"id": 26021,
"tgt": "What causes increase in blood pressure?",
"src": "Patient: My blood pressure is 147 over 103 with a 65 pulse. Yesterday about this time it was 106 over 65 with a 56 pulse, I was yawning and had to nap about 30 minutes. It remained low but I was able to function again. What is going on? I am 58 years old, female. Doctor: Hi.. blood pressure also dEpends on the physical activity.. if one is at rest the bp may be normal... similary after any sternous activity bp may rise.. these variations in bp are normal... for ideal blood pressure it should be measured after 15 min of rest... so please check ur bp after rest.. and please avoid using digital bp instrumsnt as it is not 100% accurate.. take care"
},
{
"id": 206487,
"tgt": "Does anxiety increase blood pressure?",
"src": "Patient: Dear sir/madam, i have a veyy high blood pressure from last 15 years, my age is 38, i was not taking any medicine,and i got used to the blood presure, further investigation says its anxiety, does anxiety causes blood presure to increase,and my creatinine levels shows 1.7,that bothers me a lot, but no matter how much i try my presure is not coming down from 180, all this check ups and test where done by general family doctor, and local pathology ,actually i want to consult you concerning this. YYYY@YYYY mob - 0000 kindly suggest thanks Doctor: Hi.I had gone through your query.I understand your concern.Anxiety can result of increase sympathetic outflow from brain.Neurotransmitters like nor epinephrine and adrenaline discharge play important role.So it may also elevate blood pressure. But control of anxiety can also improve in blood pressure.Anxiolytic like etizolam along with anti hypersensitive medicine can help.Regular exercise and meditations can help.Daily work schedule with stress free activity can be beneficial.Still if you have query then feel free to ask.Get well soon.Thank you."
},
{
"id": 181798,
"tgt": "What causes sore mouth and lumps around the lips?",
"src": "Patient: Hi my mouth has become very sore with various lumps on and around my lips. My lips have also swelled to double there size and they are very painful and restricting the movement of my move and causing lack of sleep. One of the lumps has a large black dot in the middle could you please tell me what it could possibly be. Thankyou Doctor: well these could be due to Herpes infection or some benign cystic lesions getting infected.please go for an FNAC/Cytology microscopical test for confirmation!all the best............."
},
{
"id": 33965,
"tgt": "Am I having fullness of stomach due to para-pelvic cyst?",
"src": "Patient: My stomach always feel full and uncomfortable with eating a meal or on an empty stomach. Also my lower back hurts and I am short of breath all the time. A recent CT scan showed a parapelvic cyst. Is the cyst associated with my stomach and breathing issues. Doctor: Thanks for posting you query to health care magic.cyst are fluid filled structure coverd with a membrane and occupy space wheeever formed. A large cyst in abdominal cavity may cause compression of nebighouring viscera and on stomach leads to discomfort in abdomen . this compressive produces pressure over diagphram and produces shortness of breath .treatment of cyst is its complete surgical removal . Hope you would be satisfied with my answer . Feel free to communicate if any query .regards,Dr.Manish PurohitInfectious disease specialist"
},
{
"id": 20377,
"tgt": "Is BP reading of 136/80 normal?",
"src": "Patient: I ve been feeling a vibration in my left ear that happens about every 3 minutes and lasts about 5 seconds. I ve been watching my blood pressure which is usually 110/80 but today it is 136/80, 168/63 and a message that says irregular heartbeat. I m a 61 yr old female. Doctor: The symptom that your are describing is very, very common. A source is almost never discovered. It usually passes over 2-3 days."
},
{
"id": 32935,
"tgt": "Suggest treatment for gonnococcal infection",
"src": "Patient: Sir, for gonorrhea , how long should I take the VDM-kit, actually I have taken it once and it seems to reducing, but now it has occurred again, so what should I do, I am married, is it the may be for my wife? another question is that whether VDM -kit also recommended for my wife too? Doctor: Hello You had taken VDM-Kit ( fluconazole 150 mg+ azithromycin 1000 mg +secnidazole 2 tablet , 1 gm each ) .Read this carefully and you will get answer:Sex partners of patients with N. gonorrhoeae within 60 days before symptoms or diagnosis should be evaluated and treated for N. gonorrhoeae .No sexual intercourse until therapy is completed and until patients and their sex partners have no symptoms.Recommended Regime :Any of the followingCeftriazone 1 gm or cefotaxim 1 gm ( I/M or I/V) +Azithromycin 1 gm orally or doxycycline 100 mg orally twice in a day for 7 days .If pelvic inflammatory disease also associated , take metronidazole ( in your case secnidazole ) .VDM-KIT is effective or not effective , I don't know .The most advanced treatment after scientific studies is , as mentioned .(Gantz's manual of clinical infectious disease ).Hope this will be useful .Further , if any question I will reply."
},
{
"id": 144215,
"tgt": "What does paralysis certifico mean?",
"src": "Patient: yes one of my friends great great great grandads died back in 1841 or therabouts i noticed on the death certificate that the cause of death was ,paralysis certifico ? and as far as i know paralysis means a stroke but i cant seem to find out what the certifico means ,can u help? Doctor: Hi, I am Dr.Bruno. Question : What does paralysis certifico mean?Answer : It does not mean anything Certifico is the next word of the certificate which means the person who certified. The Cause of Death is just Paralysis Hope you found the answer helpful.If you need any clarification / have doubts / have additional questions / have follow up questions, then please do not hesitate in asking again. I will be happy to answer your questions.Let me know if I can assist you further.Take care."
},
{
"id": 127830,
"tgt": "What causes pain in the lower back and buttock?",
"src": "Patient: I have pain in my lower back on the right side I also have pain in my right but cheek and sometimes down my leg I have had it for over a year I don t like to go to the dr but the pain is getting worse and it is making me not even get up and walk because I know its going to hurt what do u think it would be. Doctor: Hello,It could be a just lumbosacral strain or lumbar disc injury. I suggest you undergo physiotherapy for the same. The physiotherapist will be able to teach you stretching exercises and proper posture. You can apply local pain relieving balm or gel twice a day. If the pain and radiation get worse or persists after six weeks, then you can consult your doctor. Hope I have answered your query. Let me know if I can assist you further.Regards, Dr. Santosh S Jeevannavar"
},
{
"id": 130935,
"tgt": "What to do if having severe pain in leg after working heavily and undergoing mouth surgery?",
"src": "Patient: I had surgery on my mouth about three weeks ago. I was unable to eat the first two days, then I was only able to eat yogert and pudding for the next three days. I lost 15lbs. In less than a week. I was laid up for 10 days. I returned to work for a week and had to work 50 hrs. Then the following week i went back to work on Monday and was feeling good when I got home. Then Tuesday morning I woke up and had some extreme pain in my right leg. I went to my doctor wednesday and he said he thought it was bursitis and gave me a prescription for ibuprofen. Things were getting better until today, Monday I tried to go back to work with the use of an ortho type knee brace. Within 15 minutes of walking around I had severe pain in my knee, my thigh and my calf. I had to remove the knee brace, but it was to late the pain has gotten worse. What is causing my leg pain and should I seek immediate attention? Doctor: In my opinion you could have knee osteoarthritis or lumber spondylitis i recommend an x ray on knee and back , it is common after long time of disuse to noticr them Good Luck"
},
{
"id": 19702,
"tgt": "Suggest treatment for uncontrolled hypertension",
"src": "Patient: I recently had a pacemaker installed.I have gout (I khow it is not associated with the pacemaker) I have been taking a 0.6Mg Colchicine since then my blood pressure is always very high.Currently it is 201/107.It stays around this reading even though I take a 300Mg Diltiazem and a 320 Mg Diovan.daily.These medication seem to be ineffective in binging down my pressure. Your advice will be greatly appreciated. Doctor: In addition to medication try lifestyle modification like Salt restriction to 1.5 gms per day Do not eat processed food.Do meditation and simple yoga Exercise daily Sleep for 8 hours dailyTake a Magnesium supplement at bedtime .You can try vegetable juicing using beets , parsley and celery these veggies have shown to reduce blood pressure .Try them one by one .reduce weight if you are overweightStop smoking and alcohol since Alcohol precipitates Gouty attack.all these measures will get your BP to the required level .After 1 week of trying these if you still have high bp a new drug belonging to a different drug group needs to be added .hope this helps you."
},
{
"id": 194577,
"tgt": "How long will it take Eltroxin and cabgolin to increase the sperm motility?",
"src": "Patient: hi This is Pallavi ,. I am 26 yrs married girl. with 5.7 height and my weight is approx 65. i got married in 2009. Now we both are planning to have kid ASAP. We consulted a gynecologist , also gone through my Tyroied checkup and my husband's semens analysis. My report says my free T3,serum by CMIA is 3.06 and TSH(ultrasensitive)serum by CMIA is 7.03 and prolactin total is 26. And my husband's Grade 1 (Immotility is 95% grade 2 (non progressive motility ) is 05% grade 3(slow progressive motillity) is 00 % grade 4 rapid progressive motillity is 00% and sperms count is 37 millions /ml . he took this test on 5th may. So our doc suggest me to take T.Eltroxin ,T . cabgolin and Letroz and for my husband Maxiza,Hey Forte . So my querry is how many days require to balance my hormone and to get increase his spearms motillity. How much time require to concieve ( to have baby). Also is there any side effect of the medicine which i mentioned above ? Is it the case of infertility? Please reply me soon , waiting for your precious reply . Thanks Pallavi Doctor: Hi, I don't think letrozole is indicated now as your husband's semen analysis shows very poor motility. He should consult an Andrologist or urologist at the earliest to find out the cause of decreased motility. Hope I have answered your query. Let me know if I can assist you further. Take care Regards, Dr B. Radhakrishnan. Nair, OBGYN"
},
{
"id": 3611,
"tgt": "Suggest treatment for infertility due to tubal blockage",
"src": "Patient: Hi There! I am 30 yrs old, married for last 2.4 yrs, trying to conceive since two yrs by now, but not able to do so. Yesterday I went for a HSG test and the report says that there is irrgularity and beading , more marked distally near fimbrial end, suggestive of bilateral tubal block.All my previous reports are quite normal. Like my uterus is normal, ovaries are normal, cervical canal is normal, hormone levels are normal but still infertile. Please suggest. Doctor: Hallow Dear,Unless the tubes are patent, there is no chance of natural conception. However, your tubes are not just blocked, there is irregularity and beading in the tubes noted in Hysterosalpingography. This finding is highly suggestive of genital tuberculosis wherein the tubes are affected. Please get yourself investigated from a Gynaecologist for genital tuberculosis. Also look for presence of tuberculosis elsewhere in the body, particularly the lungs, by X-ray. Hysteroscopy will help to have a look at the endometrium for similar lesions. Laparoscopy will throw more light on complete pelvic health. If tuberculosis is confirmed, you will have to undergo antitubercular line of treatment. Tuberculosis can definitely be treated completely by treatment under observation; however treatment of tuberculosis is a prolonged treatment and for genital tuberculosis, you may have to take the medicines for at least 9 months. Along with the treatment, healthy high protein and high calcium diet with adequate Vitamins is necessary. Fresh fruits, fresh green leafy vegetables and fresh air to breath help speedy recovery. If tubes are affected by tuberculosis, the chances of natural conception are rather poor since though by surgery the tubal patency is regained, the tubal function is damaged. So after completion of the antitubercular treatment, you may consider Artificial Reproductory Technique for conception and pregnancy. One more suggestion, as per the WHO suggestions and our experience, every case of tuberculosis should be tested for HIV since tuberculosis increases the possibility of HIV infection six fold; so also HIV increases the possibility of tuberculosis six times. I feel you should concentrate first and more on getting healthy. Then later, attempts for pregnancy may be considered. Dr. Nishikant Shrotri"
},
{
"id": 162945,
"tgt": "What causes pain in penis of a toddler?",
"src": "Patient: I have a 5 year old child complaining of penis pain. I have been to the doctors three times now and they have told me its a habit. School have mentioned this to me quite a number of times. Today they approached me telling me my son had been in discomfort all day and has been touching his penis much more than ususal. My son has had a urine test three times and these have been fine. His penis and everything around it looks absolutely normal and there is no redness. I m at my wits end could this be a serious condition? or is there anything that could be living on the penis for example thrush that could be causing this? Please help Doctor: Hello and Welcome to \u2018Ask A Doctor\u2019 service. I have reviewed your query and here is my advice. Could he have put something into his urethra(this is the tube-like structure that takes urine from the bladder to the tip of the penis)? When urethral obstruction is a possibility, it is a good idea to observe your child's urine stream. Thrush would be cheesy white on his penis and readily apparent. Has he been seen by a pediatric urologist? I hope I have been able to give you some help. Please return if you have additional questions. Arnold Zedd, MD, FAAP"
},
{
"id": 262,
"tgt": "How to conceive despite being hiv positive?",
"src": "Patient: Hi dr im belina 28yrs old me and my husband are hiv pos we dont have children at all we realy want a child so badly i cant conceive.i hav a problem evry month when im menstruating i sleep bcoz of bad pains on my abdomen and worse on my backache and even during i have pains inside my abdomen and i cant barn to take something down if i can barn when i stand up i'll feel pains that makes me unable to move i resigned where i was working bcoz of this pls help dr i realy need help me and my husband are happily married help dr. 88kg Doctor: Hello and Welcome to \u2018Ask A Doctor\u2019 service.I have reviewed your query and here is my advice.If you don't have any pregnancy yet than the pain during menstruation is mostly due to spasm of cervix (mouth of uterus). So, you have to take antispasmodic tablets or injection for pain relief. You can have that tablet every time in your menses. Secondly, if you both are HIV positive that doesn't mean you cannot conceive. Yes it is true that there is a chance of transfer to the baby. And also it is a high risk pregnancy. You just have to continue your treatment medicines and continue pregnancy also. For detail you have to consult a nearby gynecologist and may be pediatrician for chances of pregnancy, chances of transfer to baby, and other pros and cons of that. Now for your pain, Dicyclomine like Antispasmodic will do. And if it remains for long time and you don't want pregnancy at all than cervical dilatation is one option after which your pain will be very less and bearable. It is a minor operation you can ask your doctor about it. And if you are planning for pregnancy then one pregnancy happens and the problem is gone so no pain. Once delivery or abortion takes place the source of pain is also gone due to natural cervical dilatation.Hope I have answered your query. Let me know if I can assist you further.Regards,Dr. Jimesh H Mavani"
},
{
"id": 206701,
"tgt": "Suggest treatment for anxiety and stress in a HIV positive patient",
"src": "Patient: I am homeless nfor 3 weeks, i am tired,fitigued, light headed,aometimes I feel as though im going to faint, a little trembly at times. I probably dont eat enough or drink enough .,Situtation is temp......I am HIV poz, Diebetes, and have depression. I have a big load bang that happens in my head and it makes me jump,as i never know when its going to happen, i went to the emergency room, cat scan,ekg, upper cheat xray,blood and urine did not show anything wrong but it persists. Could this just be stress ? Doctor: Hello,Thanks for choosing health care magic for posting your query.He will require cognitive behavioural therapy to understand his thoughts in a better way.In future if you wish to contact me directly, you can use the below mentioned link:bit.ly/dr-srikanth-reddy\u00a0\u00a0\u00a0\u00a0\u00a0\u00a0\u00a0\u00a0\u00a0\u00a0\u00a0\u00a0\u00a0\u00a0\u00a0\u00a0\u00a0\u00a0\u00a0\u00a0\u00a0\u00a0\u00a0\u00a0\u00a0\u00a0\u00a0\u00a0\u00a0\u00a0\u00a0\u00a0\u00a0\u00a0\u00a0\u00a0\u00a0\u00a0\u00a0\u00a0\u00a0\u00a0\u00a0\u00a0\u00a0\u00a0\u00a0\u00a0\u00a0\u00a0\u00a0\u00a0\u00a0\u00a0\u00a0\u00a0\u00a0\u00a0\u00a0\u00a0\u00a0\u00a0\u00a0\u00a0\u00a0\u00a0\u00a0\u00a0\u00a0\u00a0\u00a0\u00a0\u00a0\u00a0\u00a0\u00a0\u00a0\u00a0\u00a0\u00a0\u00a0\u00a0\u00a0\u00a0\u00a0\u00a0\u00a0\u00a0\u00a0\u00a0\u00a0\u00a0\u00a0\u00a0\u00a0\u00a0\u00a0\u00a0\u00a0\u00a0\u00a0\u00a0\u00a0\u00a0\u00a0\u00a0\u00a0\u00a0\u00a0\u00a0\u00a0\u00a0\u00a0\u00a0\u00a0\u00a0\u00a0\u00a0\u00a0\u00a0\u00a0\u00a0\u00a0\u00a0\u00a0\u00a0\u00a0\u00a0\u00a0\u00a0\u00a0\u00a0\u00a0\u00a0\u00a0\u00a0\u00a0\u00a0\u00a0\u00a0\u00a0\u00a0\u00a0\u00a0\u00a0\u00a0\u00a0\u00a0\u00a0\u00a0\u00a0\u00a0\u00a0\u00a0\u00a0\u00a0\u00a0\u00a0\u00a0\u00a0\u00a0\u00a0\u00a0\u00a0\u00a0\u00a0\u00a0\u00a0\u00a0\u00a0\u00a0\u00a0\u00a0\u00a0\u00a0\u00a0\u00a0\u00a0\u00a0\u00a0\u00a0\u00a0\u00a0\u00a0\u00a0\u00a0\u00a0\u00a0\u00a0\u00a0\u00a0\u00a0\u00a0\u00a0\u00a0\u00a0\u00a0\u00a0\u00a0\u00a0\u00a0\u00a0\u00a0\u00a0\u00a0\u00a0\u00a0\u00a0\u00a0\u00a0\u00a0\u00a0\u00a0\u00a0\u00a0\u00a0\u00a0\u00a0\u00a0\u00a0\u00a0\u00a0\u00a0\u00a0\u00a0\u00a0\u00a0\u00a0\u00a0\u00a0\u00a0\u00a0\u00a0\u00a0\u00a0\u00a0\u00a0\u00a0\u00a0\u00a0\u00a0\u00a0\u00a0\u00a0\u00a0\u00a0\u00a0\u00a0\u00a0\u00a0\u00a0\u00a0\u00a0\u00a0\u00a0\u00a0\u00a0\u00a0\u00a0\u00a0\u00a0\u00a0\u00a0\u00a0\u00a0\u00a0\u00a0\u00a0\u00a0\u00a0\u00a0\u00a0\u00a0\u00a0\u00a0\u00a0\u00a0\u00a0\u00a0\u00a0\u00a0\u00a0\u00a0\u00a0\u00a0\u00a0\u00a0\u00a0\u00a0\u00a0\u00a0\u00a0\u00a0\u00a0\u00a0\u00a0\u00a0\u00a0\u00a0\u00a0\u00a0\u00a0\u00a0\u00a0\u00a0\u00a0\u00a0\u00a0\u00a0\u00a0\u00a0\u00a0\u00a0\u00a0\u00a0\u00a0\u00a0\u00a0\u00a0\u00a0\u00a0\u00a0\u00a0\u00a0\u00a0\u00a0\u00a0\u00a0\u00a0\u00a0\u00a0\u00a0\u00a0\u00a0\u00a0"
},
{
"id": 143732,
"tgt": "Suggest treatments of pain in lower back spine",
"src": "Patient: i have two back surgeries in the last two years for pain in tthe lower back spine .in with they inserted a spine prosts devise. still have severe pain 10 month later while taking strong pain meds.looking to find asecond opion .hoping clevend clinic might be able to help. Doctor: Hi as you told that you had two back surgeries in the past and having severe low bachache.I would advise you MRI of LS spine to see cause for pain and further treatment will be decided by that only.However , physiotherapy and analgesics can be started .Thanks"
},
{
"id": 199169,
"tgt": "What does growing extra skin inside penis head mean?",
"src": "Patient: hi, my penis condition worries me. about half a year ago i noticed some extra skin inside my penis head when i opened it with fingers. it kinda looked like a little tongue, but the skin goes deeper inside and is connected with the wall of urethra. sometimes it bleeded if i pulled my foreskin too hard, so i though i have done it myself. for last 3 months i ve had a lot of sex and my girlfriend noticed it, because it has grown a little (1 or 2 mm) for last 2 weeks it has been out of my penis head, looked just like a tongue between lips. today i found out that the tip of it looks like 3 little bubbles, i could easily tear them off with fingers. no pain, just a little bleeding, like teared skin. it looks so much better now, though the question is. is it normal to have this kind of skin inside penis? is there anything else i need to know? Doctor: DearWe understand your concernsI went through your details. What you are supposed to do now is to stop worrying. The skin growth is OK and is found with many other individuals. You are not supposed to prick or pinch those because it may cause infection. If you have any doubts or irritation, please consult a physician.If you still need my assistance in this regard, please use this link. http://goo.gl/aYW2pR. Please remember to describe the whole problem with full detail.Hope this answers your query. Available for further clarifications.Good luck."
},
{
"id": 143152,
"tgt": "Possibility of having encephalitis",
"src": "Patient: i think i have encephalitis. some symptoms i have are severe headache with no change after 1000 mg tylenol 3x a day, low grade fever, agitation, lymph pain, neck pain and malaise im wondering if i should wait for my appointment with a doctor at my clinic or go into the emergency room? Doctor: Hello!Welcome on HCM!I carefully read your question and would explain that your symptoms seem to be related to an infection. I can not determine if it is encephalitis, because your symptoms are not really typical of such disorder. Anyway, considering your medical condition, I would recommend going to the ER for a physical exam and some tests ( a brain CT scan, complete blood count, chest X Ray study, ESR, PCR). Further tests may be needed to determine the cause. Hope you will find this answer helpful!Kind regards, Dr. Aida"
},
{
"id": 152498,
"tgt": "Is nausea at night a symptom of stomach cancer?",
"src": "Patient: I have a history of barretts and need recheck in two yrs but ent doc sending me to gastro doc.. My question is getting up in middle of night about to vomit but cant. I eat late like 7pm and have incline pillow but still have to throw up. Not actually doing it though. I am terrified its cancer of stomach or esaphogus. Am I worried for nothing? 62 yrs female white Doctor: Hello and Welcome to \u2018Ask A Doctor\u2019 service. I have reviewed your query and here is my advice. Yes it might be a symptom of cancer. You need to get an endoscopy done to rule it out. Hope I have answered your query. Let me know if I can assist you further."
},
{
"id": 136501,
"tgt": "What is the difference between Telma 80 and Telma AMH?",
"src": "Patient: What is the difference between telma 80 and telma am h and telma h Initially I was taking Telma AMH 40 MG later I found Swelling in my foot doctor has changed the medicine telma h Feeling very much uneasy after taking the second telma h 80 mg then doctor changed to telma 80 mg Doctor: hi ,i m happy to see your curiocity about the drug .telma 80 contains telmisartan which is antihypertansive,working as ARB,wher as telma AMH is combination of thyzide and amlopidin.amlopidin known to cause ankle swelling.thanks. any future quary is welcomed."
},
{
"id": 185760,
"tgt": "Can a chronically blocked eustachian tube cause a cyst?",
"src": "Patient: It turns out I had a huge infection beneath a crown in one of my molars. I just had the ccrown and tooh removed today. There was a 6 mm cyst on one root. In the meantime, I have had a chronically blocked right eustachian tube (same side as the bad tooth). Could there be any link? Doctor: Hello:)Welcome to HCM.Yes,infection and pain can be radiated towards the ear.You got the tooth removed and the infection source is removed.You must have a full course of antibiotics prescribed by your dentist.Also,consult your physician regarding this.Ask your dentist if the removed tooth can be replaced for a better chewing ability.Regards."
},
{
"id": 51067,
"tgt": "Kidney stones, large calculi. Staghorn in renal mid means?",
"src": "Patient: Hello Sir, I have one large size stone of 33 mm in left kidney and 2 stones of 16mm and 6 mm in right kedney.... i am concern about my left kidney...please suggest......both the kidney are normal and fine in size... Large Calculi, posibly staghorn seen in left renal mid and lower pole calyces the largest one measures 33 mm in size. Doctor: Hi Welcome to HCM. YOU NEED to be concerned about both kidneys, actually. The stones most probably need removal by endoscopic or open method. The decision depends on factors like kidney function and so on. Please contact nearby urologist. regards DR GS"
},
{
"id": 166669,
"tgt": "What is the treatment for severe stomach cramps in a baby?",
"src": "Patient: hello there! my baby is just 1 week old and have severe stomach cramps or at least that s what we believe he has as he cries intensely at certain times, has his belly contracted and his legs as well!we ve heard is not common for such young baby to have colic! Doctor: Hi,It is very common and completely normal in the first three months of age for the baby to experience colicky pain. You can use gripe water or Dentinox baby drop when needed to relieve the colics. Also holding the baby upright with his tummy against your shoulder and massaging the back till the baby burbs after feeds will help decrease the colicky pain. Hope I have answered your query. Let me know if I can assist you further. Regards,Dr. Salah Saad Shoman"
},
{
"id": 132716,
"tgt": "Suggest remedy for bumps in arm",
"src": "Patient: Hello, I was working in the yard on Sunday and I scratched my arm on a fence . No cuts or abrasions at the time. Monday I noticed that in the area where I hit my arm there is a cluster of bumps. No color and they itch if I scratch them. They look as if they could be popped and release fluid. Since I did not know what it was, I have not popped them. The bumps are raised off the skin. The area is about this size of a nickel. Again, it does not itch or hurt. Prior to the branch I was in an area heavy with poison oak. But the rash has not spread. Any ideas of what I m dealing with? Thank you Doctor: Hi Hope this message finds you in good health.I have gone through your complaints and understand your concern.It seems to be a allergic reaction to something that might have caused the bumps.Though the symptoms may vary in severity,they usually last for a few weeks. Taking anti-allergic medications should help.Nothing to worry about.\u00a0\u00a0\u00a0\u00a0\u00a0I hope your question has been answered.If you have any follow-up queries,feel free to consult me anytime.Thanks,Take care,God bless."
},
{
"id": 149212,
"tgt": "Diagnosed with Parkinsons disease. Reason? Symptoms in advanced stage?",
"src": "Patient: What is Parkinson's and where does it come from?I was diagnosed this year and the symptoms seem to advance when I least expect them. No one on either side has it (my mom or dad). I am on Requip 2 mg. and that is doing something but my Neurologist isn't clear what is going to happen to me. Where can I go to find information to read about it? I like to be informed. Dena Ketterer Doctor: hi i had gone through your query and understand your concerns.-Parkinson\u2019s disease (PD) is the most common example of a family of neurodegenerative disorders characterized by a neuronal accumulation of the presynaptic protein -synuclein and by variable degrees of parkinsonism, de\ufb01ned as a paucity and slowness of movement (bradyki- nesia), tremor at rest, rigidity, shuf\ufb02ing gait, and \ufb02exed posture. Nearl all forms of parkinsonism result from a reduction of dopaminergic transmission within the basal ganglia. Sporadic and idiopathic PD account for 75% of all cases of parkinsonism; the remaining 25% result from genetically de\ufb01ned etiologies and other causes including other neurodegenerative disorders, cerebrovascular disease, and drugs. treatment option; i advise you taking Homeopathic medicines gives safe and permanent cure for your complaints consult your local homeopathic physician for correct diagnosis of the case and remedy to fit your complaints I hope this is helpful for you, thank you"
},
{
"id": 214774,
"tgt": "What are the home remedies for treating acne?",
"src": "Patient: hi,I am 23 years old.i have been suffering from acne since 5 yrs,but still cant over come tem.right now em using medimix soap, cetaphill cleansing lotion and benzoyl peroxide cream for acne.my mensural cycle is of 1 n hlf or 2 mnths..i found this as one my reason for acne..n also because my blood is not purified.plz suggest me a syrup which can purify my blood.Also plz suggest me a day or night cream for fairer skin which can b used for a long time and without side effect..can i use himalaya herbal fairness cream instead? Doctor: **1. acne at your age is physiological [since this is the age of Pitta] in Ayurveda which is concerned with Acne outbursts, and also Pathological because of Hormonal imbalance as well as Pelvic problem, so in best interest first get a gynecologist checkup [pelvic examination] along with necessary Investigations [Thyroid Profile, Hemoglobin, Ultrasonography, Estrogen-Progesterone ratio]2. Avoid : oily, spicy, fried, and left over food, too much exposure to heat, dry and windy climate, stress, newly harvested cereals, black gram, radish, sour and heavy substances, milk, curd, sugarcane products, and jaggery, sleeping during day right after meals. Take old barley, wheat, brown rice, green vegetables, boiled and filtered water to drink, green gram, pigeon pea, masura, patola, kakamachi, garlic, cucumber, bitter dietary substances, old ghee, sesame oil, nimba oil, cow\u2019s urine, good sleep, along with regular morning walk of 45 minutes.. Don\u2019t allow to dehydrate yourself, thus avoid alcohol and caffeine containing beverages.. According to Ayurveda:i. Triphala helps balance all the doshas and clean up the digestive system [take 5gm with lukewarm water at bed time [night]ii. Avocado is good source of Vitamin E and Vitamin C: thus natural moisturizer and anti inflammatory fruit.iii. Clean the Pimple area with Apple vinegar, Leave it for one hour and then wash with lukewarm wateriv. Spinach: contains Vitamin A [most important vitamin for the skin] and also helps to stop the production of sebum [ who suffer from oily skin]v. Kale contains high amount of antioxidants which can reverse skin damage and provide with calcium, fiber, omega-3 fatty acids which help to reduce inflammation.vi. Use face mask with multani mitti, chandan, neem leaves powder, haridra with lemon and honey. vii. prescription drugs available are: Maha Thiktakam kashayam, Patoladi kashayam, Maha Thiktaka Gritam, Guggulu Thiktaka kashayam / Gritam, Saptasaram kashayam, Khadira Arishtam, Kumari Asavam Sarivadi Asavam / Arishtam, Avipattikar Churna, Haridra Khanda, Chandanadi Lepa, Multani MittiPS. i. wash your face with Besan and lukewarm warm water after coming from outside (best natural bleaching agent), removing excess sebum and dead skin cells, likewise Neem,acts as a deep cleanser.. Use of fresh vegetables and fruits, stay hydrated (drink 10-15 glasses of water/ day). Avoid: Sugar (in any form), caffeine, cheese, oily-spicy-dry-fried food, chocolates, cakes, iodized salt, soft drinks.. Avoid exposure of the skin to severe climatic conditions.. Friction makes PIMPLES more likely to erupt, so do not over-scrub or touch the affected area whenever possible.PS: because Skin & Nervous system has a common Embryological origin (thus acting as an initiating or aggravating factor), thus treatment is directed as \"Psychosomatic Treatment\""
},
{
"id": 154531,
"tgt": "Suggest remedy for burning sensation with blood in the urine",
"src": "Patient: Hello, my father is suffering from prostate cancer with bone mets and undergoing hormone therapy. The psa initially was 28 and after 15 months of hormone therapy its ranging between 0 and 1. Recenly it was 1.4. He is on acitrom due to mitral valve replacement. Due to recurrent uti he is on antibiotics which are fluctuating his inr. Now from last two days he is having blood mixed urine at the start of stream and a little pain nd burning during urination. My doctor has told me to wait for 1-2 days. Inr is 2.0 now. Please suggest any remedy Doctor: Hi, dearI have gone through your question. I can understand your concern. He has prostate cancer under control. But his urinary tract infection with acitrom may be responsible for bleeding and pain in urination. You should go for culture and sensitivity test of urine and regularly monitor your INR. If needed change the antibiotics accordingly. Drink plenty of water. Hope I have answered your question, if you have doubt then I will be happy to answer. Thanks for using health care magic. Wish you a very good health."
},
{
"id": 152634,
"tgt": "What is the prognosis for papillary cancer post thyroid surgery?",
"src": "Patient: My son is having thyroid surgery now. It is half over and they have said he has multiple lymph nodes involved. It is papillary cancer. One of my daughters and my husband have also had this cancer. They are fine. No lymph node involvement. What is his prognosis as compared to theirs? And thank you, Audrey Doctor: She needs a radioiodine scan after surgery. Initially a diagnostic scan. If the scan shows uptake, then therapeutic radioiodine scan needs to be done. This will ablate all the remnant tissues. once this is done, she is put on follow up. Papillary carcinoma thyroid has a very good prognosis. Most do not die with this disease. In one series cancer related mortality in non metastatic disease is only 6% after 16years."
},
{
"id": 124148,
"tgt": "How to cure the pain in my left leg?",
"src": "Patient: i hurt my left leg between knee and ankle. since then i have throbing pain in left leg. it is extremely difficult for me to lift left leg in order to start walking. i vertually limp while trying to walk. i feel extreme pain in trying walking. suggest treatment. thanks Doctor: Hi, There is a strong possibility that you have injured one of your major tendons like Achilles Tendon. I suggest Immediate consultation with orthopaedic surgeon. Hope I have answered your query. Let me know if I can assist you further. Take care Regards, Dr Gopal Goel, Orthopaedic Surgeon"
},
{
"id": 161823,
"tgt": "Suggest treatment for child having swollen bruise on lower leg",
"src": "Patient: My 7 year old daughter had what looked like a pimple on herMy 7 year old daughter had what looked like a pimple on her lower leg (just above her ankle)... it had a white head and I of course popped it. Lots of puss came out. Now its the size of a penny, hard and swollen... even looks slightly bruised under the skin. What can I do to treat this? Doctor: Hi, It would be better if you upload an image - but by what you say I feel that it might develop into cellulitis and the kid might require antibiotic therapy with Clindamycin. Hope I have answered your query. Let me know if I can assist you further. Thank you. Regards, Dr. Sumanth Amperayani Pediatrician, Pulmonology"
},
{
"id": 11105,
"tgt": "Suggest treatment for hair loss",
"src": "Patient: Hi, I am suffering from hair loss...... it is increased from past 1 week. I have very dry hair and flaky scalp. If i use any dandruff shampoo it makes my hair more dry. I loose more hair when i apply oil and during shampooing. Please suggest me any good/mild shampoo. Doctor: HIWell come to HCMyou have not mentioned your age and gender here, if you are male person then this could be Male Patron Baldness and nothing much can be done, but Minoxidile 5 % solution can be tried, and this can be used in either genders for the dandruff \"Povidone Iodine \" shampoo is the best option, hope this information helps, take care."
},
{
"id": 162902,
"tgt": "Are frequent UTIs in a 4 year old a concern?",
"src": "Patient: My Grandaughter is experiencing a lot of urinary track infections. She is only turning 4 in Jan. My Daughter - in - law told me that she might have some kind of reflex disease. They are or will be running tests. Is this rare for this age group and what causes this to happen? Doctor: Hello and Welcome to \u2018Ask A Doctor\u2019 service. I have reviewed your query and here is my advice. Less that ten % of young girls can have vesicoureteral reflux. This is when urine in the bladder refluxes, or travels out of bladder into the ureter, and back up toward the kidney. The ureter is the tube-like structure that normally takes urine from the kidney down into the bladder. In these young girls the attachment of the bottom of the ureter into the bladder is abnormal and allows the urine to leave the bladder and go back up into the ureter. Depending how severe the reflux, the urine can go back up into the ureter and into the kidney. Hope I have answered your query. Let me know if I can assist you further."
},
{
"id": 97469,
"tgt": "Suggest alternative medicine to stop chewing of tobacco",
"src": "Patient: hello sir, my husband and brother both are addicted to gutka..since 10 years..they stopped taking proper diet and they look pale day by day..please suggest which medicine should i give which help them to leave this habit..one more thing both of them don't want to get rid of it that is why i am asking for the medicine.Husband details:- Age - 35 yrs.,Height - 5.7\"weight - approx 48 kgs.no medical history till now.Brother Details:-Age - 27 yrs.,Height - 5.9\"weight - approx 60 kgs.no medical history till now. Doctor: Hi there, thanks for posting your query on this forum.Gutka addiction is equally dangerous as any other addiction as it contains carcinogens which lead to oral cancer if taken for many years.Taking gutka may cause symptoms like loss of sleep, loss of concentration, acidity, etc.Submucosal fibrosis is another important condition seen in gutka addicts where they are unable to open their mouth completely.Your concern is genuine but unless your husband and brother don't feel like stopping their habit nothing can be achieved.Irrespective of giving any medication or any counseling one cannot get rid of an addiction unless the person himself feels like giving up the habit.Regards,Dr. Kunal Lokare"
},
{
"id": 72795,
"tgt": "What causes a red and sore spot below the shoulder blade?",
"src": "Patient: I have a red rough sore spot ( slightly raised) that islocated just below my left shoulder blade above chest. It appeared about 4 or five days ago is about 1/2 inch in diameter and does not seem to be healing. Should I be concerned? I don't know where it came from...no injury. Seems like it might be infected as it stings when I touch it. Doctor: Hello dear , hiWelcome to Healthcaremagic.comI have evaluated your query thoroughly .* This seems in relation with possible infected sebaceous cyst most likely .Must get confirmation with clinical examination or a photo pic attachment isstrongly recommended .Hope this will help you for sure .Regards ."
},
{
"id": 121421,
"tgt": "What causes severe cramping in thenar muscles on clenching fist?",
"src": "Patient: When I use implements that are held in a clenched fist, like a spatula or hammer, with a firm grip, I get severe cramps in the thenar muscles. What is going on, and what are some good massage or stretching techniques to relieve the cramps and is there anything I can do to make this occur less frequently? Doctor: Hello,Any medication you are on, I can infer that you get frequent cramping. I would suggest you to get your VIT D and VIT B12 levels done. If theses are low then to take supplements to normalize these levels. If even after taking these supplementation if the symptoms persist then to consult a orthopedic doctor.Wishing you a speedy recovery. Hope I have answered your query. Let me know if I can assist you further. Regards, Dr. Santosh S Jeevannavar, Orthopedic Surgeon"
},
{
"id": 79646,
"tgt": "Suggest treatment for shortness of breath and black dots on the breast",
"src": "Patient: hye, My aunt is having shortness of breath and she is on vent now. Her breast showed some kind of infection that apparently turned out to have black blisters. I could she those spreading. When we went for lungs x-ray doctor said she has accumulation of water in her lungs Doctor: Thanks for your question on Health Care Magic. I can understand your aunt's situation and problem. By your history and description, possibility of bacterial infection especially staphylococcus is more in her case. She is having pleural effusion and infective skin lesions on breast. Staphylococcus can cause pleural effusion and blister formation on skin. So chances of staphylococcal infection is more in her case. Better to send pleural fluid culture and sensitivity for the diagnosis of staphylococcal infection. This will also tell about effective antibiotic therapy. With appropriate antibiotics and Care, this infection can be treated. Hope I have solved your query. Wishing good health to your aunt. Thanks."
},
{
"id": 209648,
"tgt": "How to deal with paranoid feelings?",
"src": "Patient: I am a 15 year old girl and have been drinking monster energy everyday for a few weeks,not healthy i know.Lastnight i had a can in bed and then i mastrbated,this was so scary,my blood pressure went straight up and i began to feel extremely paranoid,i have asperger's syndrome so that contributes to me feeling quite paranoid,anyway,i began to feel a hot feeling in my belly (when i get paranoid or nervous) and i had a breakdown and had to walk around.I woke my mum up and told her,it was really embaressing but i though it was the best thing to do.The next day (today) i am still feeling really paranoid please help,i know i sound like a creep but i'm seriously worried,thanks. Doctor: HIThanks for using healthcare magicI think, you have anxiety attack and in that case, you need low dose benzodiazepine. That would to control these anxiety symptoms. This drug is not available over the counter, so you should consult a psychiatrist for proper management. You can also try some relaxation exercise that would keep you calm. In case, you need further help, you can ask.Thanks"
},
{
"id": 66625,
"tgt": "What causes a small lump in collar bone?",
"src": "Patient: I have a small lump in my neck above my collar bone, on my right hand side. It s about the size of pea and seems to be quite hard when I squeeze it, it s not on the other side either. my grandma had cancer of the lymthnodes when she passed so wondering if I should get it checked out? Doctor: Hi, dearI have gone through your question. I can understand your concern. You may have some enlarged lymphnode. It can be due to reactive hyperplasia, tuberculosis, lymphoma or metastatic cancer. You should go for fine needle aspiration cytology or biopsy of that lump. It will give you exact idea regarding cause. Then you should take treatment accordingly. Hope I have answered your question, if you have doubt then I will be happy to answer. Thanks for using health care magic. Wish you a very good health."
},
{
"id": 185112,
"tgt": "Is it normal to have little holes after pulling wisdom teeth?",
"src": "Patient: Hello Dr, I just got my wisdom teeth pulled, all four. I have been fine sense I got them pulled. It will be a week tomorrow sense they were pulled. But I was looking at my teeth and noticed little holes where the teeth were removed. Is that normal or are those dry sockets? Doctor: hi .... in case of dry socket you will have lot of pain .. so if pain is not there dont worry . as it has been only one week . so it will take time to heal. you will fell small depression but gradualy it will be filled . even for few days you may fell food is deposited there. but with time it will be ok. untill there is no pain dnt worry.... still you have any doubts free to ask"
},
{
"id": 118742,
"tgt": "Pain in back, left central portoin. Why yellow urine? Why tired? Why biliribin exceeds?",
"src": "Patient: hi Dr.. i am 29 years old. i feel pain on my left central portion of back side ..i mean almost netral of back bone but in left side... befor my weight 65kg.. now 52 only. also i fell tiredness & my urine yellowish colour..i made some lab test .. isee total bilurubin level is exeed.. ie 33.9 umol/ litter.. please advice me what the problems for me & what the further treatment... iam attaching my reports. Doctor: Hello and welcome to HCM,Raised levels of bilirubin and yellow colored urine suggest jaundice.The next step is to determine the level of direct and indirect bilirubin levels.Direct and indirect bilirubin are raised in different states.Direct bilirubin is raised in obstruction of the biliary tract and indirect bilirubin is raised in hemolytic anemias to name a few conditions.The other liver function tests should also be done - SGOT, SGPT, alkaline phosphatase, and 5- nucleotidase.These investigations will throw more light on the causation of jaundice and hence its management.Consult your physician ordering the investigations.Thanks and take careDr Shailja P Wahal"
},
{
"id": 81754,
"tgt": "Suggest treatment for sharp pain between ribs",
"src": "Patient: Sharp pain between ribs. I has a surgical abortion yesterday and I had no pain medication during or after besides antibiotics, I had normal cramps after the abortion and bled for like a half hour after and with a few clots. Now I have brown discharge and it smells fine. I have no cramping or anything just very tender between the ribs and when I touch its a sharp pain. Please help. Very worried Doctor: Thanks for your question on HCM. In my opinion you are having muscular pain mostly. But better to rule out lower respiratory tract infection ( LRTI ) first, as LRTI can be seen as infective complication of any gynecological procedure. So get done chest x ray to rule out LRTI. If this is normal than your rib pain is mostly due to musculoskeletal cause.Try to follow these steps for better symptomatic relief.1. Avoid heavy weight lifting and strenuous exercise. 2. Avoid bad postures in sleep. 3. Take good painkiller and muscle relaxant. 4. Apply warm water pad on affected site."
},
{
"id": 131666,
"tgt": "Is unusual painful cramps and inordinate stiffness due to PMR?",
"src": "Patient: several years ago I was diagnosed as having PMR. I took Prednisone for a period of time but am not on any medication at this time. I do however, have inordinate stiffness and at night shortly after laying down and following exercise I predictably experience unusually painful cramps. Is this possibly a further symptom of PMR? Doctor: PMR can cause muscle and joint pain/stiffness but not commonly causing muscle cramps..it can be due to low potassium. drink plenty plenty of water and eat bananas on daily basis ."
},
{
"id": 202282,
"tgt": "How to get stronger and longer erections?",
"src": "Patient: I am 55, and have had infrequent sex for the past ten years. My wife is staying away to take care of kids' education, and my job is 1200 km away. So we meet hardly for a week or a fortnight -- twice a year. We enjoy sex. That apart, I do not hve sex. So is attrition taking its toll. Moreover, my nutrition level has gone down. I do not get erection at all. Now we are staying together. Please advise on how to get a stiff erectgion that lasts some time. lifestyle changes, food changes. I am total vegetarian. Thanks. YYYY@YYYY Doctor: Viagra is a medicine that you need to start taking.It will allow you erections to be hard and you can perform better.You can get this from your primary doctor.Please leave 5 star rating!"
},
{
"id": 224183,
"tgt": "Is it normal to experience headaches, cloudy urine and dizziness after getting depo shot?",
"src": "Patient: I had my second depo shot in march, 1 day after the first day I could. Recently I've had headaches, cloudy urine, dizzy spells, strange cravings and an assortment of other seemingly pregnancy related symptoms. Is a pregnancy possible while using the depo shot, or are these all side effects? Doctor: Hello,The efficacy of the depo shot dwindles as it approaches the third month or the 90th day. This is enhanced when there is weight gain during usage of hormone medication. Also stress, concomitant use of certain antibiotic and other medications can reduce the contraceptive effect of hormones and can lead to failures and hormone disturbances leading to several symptoms. Your present symptoms can also be due to the hormone effect. To clarify, you can get a blood test for pregnancy. Hope this helps."
},
{
"id": 82262,
"tgt": "What causes coughing with phlegm in the evening?",
"src": "Patient: Every fall I start caughing,with phlem and in the evening I wheeze sometime, and the coughing last, sometime for a couple of months, my stomach, lungs, and isophagus are ok, they all have been checked,and it s not asthma, I think it s bronchitis, what do you think? Doctor: Thanks for your question on HCM.I want to ask you few question.1. Are you smoker?2. Do you have exposure to smoke or chemical at work place?So In my opinion you should consult pulmonologist and rule out bronchitis.As smokers and workplace exposure can cause bronchitis. You need chest x ray and PFT (Pulmonary Function Test) to diagnose bronchitis. PFT will also give you idea about severity of the disease. And depends on this treatment is directed. So get done chest x ray and PFT."
},
{
"id": 159321,
"tgt": "Undergoing chemotherapy and radiotherapy for small cell lung cancer. Smell in scalp. Result of chemotherapy?",
"src": "Patient: i am currently undergoing chemo and radiation for small cell lung cancer i am almost done with radiation and have two more ruonds of chemo. i have lost most of my hair an do use a gentle shampoo and conditioner daily and when at home wear a cap, however i have noticed that i can only wear the caps one time because they seem to smell - i was wondering if this is as a result of the chemo. i do not have any other bodily odors none of my clothes or nightclothes have any odors. Doctor: Hi, chemo not really a causative factor of scalp smell. you may have catch any infection. this is a uncommon feature you are pointing about. you better consult a dermatologist."
},
{
"id": 135130,
"tgt": "What causes sudden onset of swelling of middle finger and purple discoloration?",
"src": "Patient: Sudden onset of swelling of the middle finger, especially from joint to hand, slight purple discoloration. Slight tenderness. No pitting. No fever. No bite marks or openings in skin. Swelling is not hot. No diabetes. History of borderline to high blood pressure. This is the first episode of this happening. Doctor: Thanks for your query. I have gone through your query.The purple discoloration over the finger can be because of the ecchymosis secondary to the drugs you are taking for the hypertension particularly the blood thinners like aspirin or clopidrogel. Nothing to be panic, consult your physician and get it evaluated once. I hope i have answered your query. Take care."
},
{
"id": 208947,
"tgt": "Suggest treatment for depression",
"src": "Patient: I've read descriptions of depression and I recognize some symptoms in myself but I'm not so drastic that I can't get out of bed. I just seem to slow down and fizzle out each day. I experience low feelings, more so than I used to. How can I know if it's mild depression? Is there such a thing and what's the treatment that's not involving drugs? Doctor: hi dear,for depression to diagnose there certain criteria are there and for that you should consult psychiatrist.sometimes when any stress comes in life feeling of depression is there and it is temporary.there are other method except medication to overcome depression.like psychotherapy also do yoga, exercise meditation etc thank you"
},
{
"id": 39620,
"tgt": "What could cause IgM reactive after intake of ciprofloxacin for two weeks?",
"src": "Patient: i had fever and went for typhidot test, that came positive. IgM was reactive and IgG was non reactive. i took ciprofloxcocin for 15 days on advice of my family physician...yesterday i repeated my typhidot test in which again IgM was reactive... i want to ask that is it possible that after having anti-biotic for 15 days again IgM comes reactive? Doctor: Dear Friend.Hi , I am Dr Anshul Varshney , I have read your query in detail , I understand your concern.Usually Widal and IgM Typhi DOT remains positive for about 1 month, infact sometimes even longer.They are non specific tests.You don't need to worry.Only rising levels of Widal are considered significant.Share with us all your reports and symptoms, i would tell you you have Typhoid or not.Many a times, Widal and IgM Positive positive, even when we don't have a Typhoid fever.This is my personal opinion based on details available here. If you still have any other query, you may please ask me.Stay Healthy. Dr Anshul Varshney , MD"
},
{
"id": 43895,
"tgt": "PCOS, high prolactin level, on medication for irregular periods. IUI done, susten prescribed. Suggestions ?",
"src": "Patient: AAAAA,25,165 cm,65 kg,for the past 6 months,i had poly cystic ovary problem,high prolactin level,they prescribed tablets for regular periods,metaformin,yasmine 21 days,during second day of periods they prescribed purgeon injection and after that follicle study has been done,after successful ovulation, IUI has been done,after that doctor prescribed to use susten 400. Doctor: Hi, Your treatment is on right track. Just continue. It will give chance of pregnancy of 25-35% per cycle if tubes & semen are fine. Wish you good luck."
},
{
"id": 117352,
"tgt": "What is the treatment for headache and chest pain?",
"src": "Patient: I had a blood transfusion in august of 2012 because my hemo was at 5.5. Tests were ran but nothing found. My hemo is at a 6.5. Now. What could be going on. Im tired, eat ice day n night. Some light chest pains and headaches. I have no insurance. But im feeling like crap. Should i go to emergency Doctor: hemoglobin is still low to account for your symptoms.you need to get evaluated forcause of anemia and treated suitably.Sans insurance may go to ER. meanwhile take haematinics,eat rich green vegetables rich in iron and high protein diet and arrange to consult physician in hospital"
},
{
"id": 220773,
"tgt": "Could the tenderness in vagina affect the fetus?",
"src": "Patient: hello i had a question my friend is 17 weeks pregnant and she lets her dog lick her vagina. she just recently told me this because she was starting to worry because from what she says she feels extremely tender and she was also wondering if it could hurt the baby I didn't know what to tell her so i came to see if i could get an answer if you don't mind Doctor: HiDr. Purushottam welcomes you to HCM virtual clinic!Thanks for consulting at my virtual clinic. I have carefully gone through your case, and I think I have understood your concern. I will try to address your medical concerns and would suggest you the best of the available treatment options.It appears a bit weird about that habit.I will suggest to avoid this licking habit. As dog saliva can contain viruses and other pathogens, which can get transmitted even with a minor bruise in and around vagina. And nay such infection is harmful to both - mother and the baby.For her tenderness she can use Tab CALPOL 650 mg as needed and CLINGEN vaginal tablets for a week can be of help.I hope my answer helps you.Thanks.Wish you great health.Dr Purushottam"
},
{
"id": 8366,
"tgt": "Is there any problem for using melawash face wash gel?",
"src": "Patient: dear sir i m using melawash face wash gel for past 3 months (daily 1 time for face 1ly) and also i m using Venusiya moisturising cream(Dr.Reddys) i got good result..I have oily sensitive face..now good improvement..face wash gel contain 6% glycolic acid any problem wil come in feture Doctor: Hello. Thanks or writing to us at healthcaremagicThe face wash that you mention is quite good and can be used as long as you desire. It contains 6% glycolic acid.Glycolic acid is an exfoliating agent as well as a skin lightening agent. It helps with Oil control as well. Glycolic acid is an excellent peeling agent in those who have sensitive skin. It has no negative long term side effects on skin. In fact glycolic acid is a natural product and is derived from sugarcane.Glycolic acid has many advantages for maintaining the youthful appearance of the skin by its antiageing, skin rejuvenating actionsRegards"
},
{
"id": 127027,
"tgt": "What could be causing severe pain in my neck and shoulders?",
"src": "Patient: Hi for the past three days I am having severe neck pain and heavy shoulders and getting tired in a min went to the hospital they did my ekg it came normal I am in severe neck pain and if I try to run I feel I have walked miles Doctor: Hi, It can be cervical spine related problems like spondylitis or contusion. As a first line of management you can take analgesics like Acetaminophen or Diclofenac for pain. If symptoms persist better to consult an orthopedic and get evaluated. It is better to immobilize the neck in a soft cervical collar. Wishing you good health. Hope I have answered your query. Let me know if I can assist you further. Regards,\u00a0\u00a0\u00a0\u00a0\u00a0 Dr. Shinas Hussain"
},
{
"id": 170955,
"tgt": "Suggest remedy to avoid diarrhea",
"src": "Patient: Hi, my son is suffering in dieria last 5 days. the loose motion ration is 7 to 8 but last 2 days motion ratio is 4 to 5. i want to ask you how many days is required to recover from dieria and also guide me what is the safety precaution to avoid the dieria Doctor: HiThe frequency of poops improve gradually over a week. You may give banana and home made hygienic diet with plenty of fluids. Avoid sugars. Also, start him on probiotics such as enterogermina twice daily and zinc supplement such as Syp ZnD or Zinconia.Hopefully this will help you.Take care"
},
{
"id": 91151,
"tgt": "Suggest treatment for lower abdominal pain?",
"src": "Patient: I am having pain in my uterus an lower abdominal area. It feels like its stretching or pulling something when I take deep breaths or bend. I am not pregnant that I am aware of yet I havent had a period in a month in a half. That's usually normal for me to not get them all the time. Plus i have changed my diet. Doctor: Get your urine pregnancy test done to rule out pregnancy. The pain most probably is due to spasm. It will get relieved by taking Tan Buscopan twice a day.Also i would advise you to have an ultrasound done and get a gynecologist opinion."
},
{
"id": 195459,
"tgt": "How to stop masturbation addiction?",
"src": "Patient: Hello, I am a 38 yo white heathly attractive male who masturbates at least twice a day or more. I am not promiscuous, not as in my younger days...now I just watch internet porn or think about it. I Love sex and have since I can remember back to five years old playing with girls. I expected the intesity to lessen with age, but I have come to find out it is not and I am more and more beginning to embrace my nature as it seems this is how I am wired. Will the fire ever subside? Doctor: Hello and Welcome to \u2018Ask A Doctor\u2019 service. I have reviewed your query and here is my advice. For sexual desire masturbation is safe and pleasure giving procedure. You should try to avoid less porn if masturbation is leading fatigue like bad effect on body. Yoga and meditation will be beneficial. So it can be controllable but you should have desire to control it. Make short and long term goals. Consult psychiatrist for your counselling and examination. Hope I have answered your query. Let me know if I can assist you further."
},
{
"id": 54202,
"tgt": "Suggest treatment for stage 4 liver disease",
"src": "Patient: My husband has been diagnosed with stage 4 liver desease. He has not touched alcohol ij over 20 yrs. He contracted hep c while in the army from a blood transfusion. He had a liver tumor removed 3 months ago it was small and localizd.since then he has been in the hospit for acites and gi bleed. No active bleeding in the throat stomach or colonhowever they did band off a grade 3 varices column. Where do we go from here Doctor: HiI understand your concern.When ascites develop, disease becomes decompensated. Treatment includes symptomatic treatment and some dietary management. Avoid constipation to reduce effects of liver disease in brain. For this syrup lactulose can be used. Avoid protein rich diet. Although albumin decreases with liver failure but high protein content can cause upper GI bleed. Avoid salt intake to avoid ascites. Certain drugs to reduce ascites are used too like tablet spironolactoneHope my answer helps."
},
{
"id": 66723,
"tgt": "What causes small lump at the top of my buttocks?",
"src": "Patient: Ihave a small lump at the top of my buttocks which is extremely painful to sit on. It is quite hard to touch with a small blackish area on it. I am going away tomorrow and despite it hurting I am not too worried but I wondered if theres anything I can do to calm it. I wondered if it could be a splinter or just some kind of cyst or spot? Doctor: Hi,It seems that there might be having some ingrown skin infection and now forming abscess is likely.go for one antibiotic and anti-inflammatory medicine course for 3-5 days.Apply antibiotic cream locally.Avoid perspiration on the part.Ok and take care."
},
{
"id": 195601,
"tgt": "How can a hard bump on the penis shaft be treated?",
"src": "Patient: I have a oval shaped bump on my penis shaft. Its hard at touch. It does not hurt nor does it itch. I noticed it back in march 2017 and it has not went away. I had unprotected sex with 2 women at different times of course. Im starting to notice 2 more smaller red/skin colored bumps in different spots on my penis shaft. I masturbate daily and often(idk if that has anything to do with it)... i have not been to the doctor at all. Im very nervous and embarrassed. To the point im rejecting sex with my girlfriend. Doctor: Hello and Welcome to \u2018Ask A Doctor\u2019 service. I have reviewed your query and here is my advice. The condition which you are suffering from is peyronie's disease which is the formation of fibrous plaques within the shaft of the penis i would recommend you to visit the urologist as this disease me grow over the period of time making it difficult for you to maintain an erection or it might cause painful erections. Hope I have answered your query. Let me know if I can assist you further."
},
{
"id": 195284,
"tgt": "Suggest treatment for ingrown hair at the bottom of the penis",
"src": "Patient: Hello, I have an ingrown hair toward the bottom of my penis, I can't move the hair to the follicle to pull it out, It will not seem to grow out or to an area where I can use utensils to pull it. Since, it's been about 3 weeks since I noticed the bump, I've had one before in the same spot or close to it, but was able to pull out the hair. What Can I do? Doctor: Hello and Welcome to \u2018Ask A Doctor\u2019 service. I have reviewed your query and here is my advice. Clean the area, apply worm compression daily. Use hydrocortisone cream to reduce inflammation. Inspect the area daily if you observe the end of ingrown hair you can remove it gently. If the hair is so deep that it's created a cyst, see a dermatologist. If it's a severe case, the doctor can surgically remove the cyst. Take care."
},
{
"id": 173562,
"tgt": "Suggest remedy for stomach upset, vomiting and diarrhea in 2.6 years",
"src": "Patient: HI doctor, my baby(2.6 yrs) had vomiting in the morning. I took her to the pediatrician doctor suggested Junior Lanzol. but after giving this tablet by mixing in 3 mil of water after her break fast she was fine for 2-3 hrs but after that her stomach is upset. Please suggest. She passed motions 2-3 time with in 1 hr of time. Doctor: Hello!Thank you for asking Healthcaremagic!Answer:From your description, it seems like a gastroenteritis.Most cases of gastroenteritis in children are mild and pass within five to seven days without any specific treatment. It is very important to drink plenty of fluids to prevent dehydration and use oral rehydration salts.If your child has pain and a high temperature, this can be relieved by giving liquid paracetamol. Medication to prevent vomiting and antidiarrhoeal medication are not usually recommended for children with gastroenteritis.I would suggest:1. Juices, fruits and vegetables2. Probiotic yoghurts are also available 4. Zincovit should help in recovery if diarrhoea is continuingIf he is running fever, stools contain blood/mucus/very offensive then it needs to be tested and may need antibiotics.I suggest you to consult the pediatrician again if the symptoms get worse.Hope my answer was helpful! Dr Suela."
},
{
"id": 1521,
"tgt": "What is the appropriate time period for increasing the likelihood of conception?",
"src": "Patient: hi doctor iam married.iam trying to get pregnant. but i have irregular periods.i consulted doctor.she gave me deviry,siphene to get irregular period as well as to get pregnant.this month i got regular period.but how can we know when will ovum is released.we concieve from 10th 20th of my last month period but we failed.so i want to know when will egg produce if we use deviry and siphene tablets Doctor: Hi, I think you should go for follicular monitoring. In this, you can take siphene and track your follicles growth by repeated ultrasound and when your follicles is more than 17 to 18 mm, take injection for rupturing the follicles. Be in contact with your husband for 2 to 3 days after injection. Take progesterone for next 2 weeks. I think deviry is not a good option in progesterone. You can take some other progesterone like duphaston or susten. Talk to your doctor regarding this. Do a urine pregnancy test at home after taking 2 weeks of progesterone. You can try like that for 3 to 6 months. Hope I have answered your question. Regards Dr khushboo"
},
{
"id": 124980,
"tgt": "Is it normal to have pain after surgery in arm?",
"src": "Patient: i broke my wrist three weeks ago and had surgery 2.5 weeks ago to align the radius bone. I have plates and screws in my arm and its now in a hard cast just below my elbow. The top part of my wrist has been aching for the past 2 days. Is this normal? Is this just a feeling from the healing or it being constricted in the cast? Doctor: Hello, You should immobilize the wrist after you had a surgery for proper recovery. Pain & inflammation are normal after surgery, not to bother much. Take the medications as prescribed, if you have been suggested to do wrist exercises after surgery, do it under a physiotherapist. If you are in unbearable discomfort, visit ER for follow-up examination. Sometimes, pain can be felt 6-8 weeks post surgery. Hope I have answered your query. Let me know if I can assist you further. Regards, Dr. Nupur K, General & Family Physician"
},
{
"id": 31947,
"tgt": "What causes an increase in the eosinophils count?",
"src": "Patient: sir/madam,i am from hyderabad,andhra pradesh. My mother is having a high count of eosinoplills and was administered various anti-biotics by different doctors. She is also coughing a lot. Could this be related to any blood disorder or any infection related to lungs. She went through all the tests and her chest x-ray is clear, but her CBP shows a high count of eosinophills. Doctor: Hi, dearI have gone through your question. I can understand your concern. She has eosinophilia. There are many causes of eosinophilia. Asthma, allergy, hypersensitivity, parasitic infection etc are the common cause. She should investigate for this. Fund out the exact cause and take treatment accordingly. Till that antihistaminics like chlorpheneremine malate and DEC can be used. If needed steroid can be used as last resort. Thes all are prescription based medicine so consult your doctor and take treatment accordingly. Hope I have answered your question, if you have doubt then I will be happy to answer. Thanks for using health care magic. Wish you a very good health."
},
{
"id": 200150,
"tgt": "What could feeling of releasing fluid during ejaculation from leg on cast due to broken ankle suggest?",
"src": "Patient: I recently ejaculated and when i was finishing , it felt like my leg was releasing fluids, or fluids where being taken out of it. I also had surgery for a broken ankle and have a cast on. I have 3 open areas on the foot in the cast if that might be where the moist feeling or fluids were coming from. Doctor: Thanks for asking in healthcaremagic forum If the cast has opening then it may be due to open wound. So, your ejaculation and fluid from wound is not related. Please visit your doctor who advised you cast for this, if you have too much of discharge/pus discharge etc. All the best."
},
{
"id": 111619,
"tgt": "Experiencing back pain after sports activity",
"src": "Patient: Hello Doctor, I am 26 years age and my weight is around 70 Kg. I am having a back pain since a year, i don t feel it every time but when i walk for a long and sit or play cricket my back hurts a lot for 15-20 mins. While sleeping on the bed i can feel it too. I came to know that it could happen when a person is over weight. just would like to know is it a normal behavior or do i need to take it seriously. Thanks -Kiran Doctor: Hi, Thanks for using HCMIn my opinion you are not overweight at all so stop worrying. All you need to do is give you back a little rest. For immediate relief have some pain killer along with a muscle relaxant and have proper bed rest for 2 weeks. Do not lie down on soft mattress use hard one instead, do not sit for longer duration and do not travel long distance for 2 weeks . You can also apply some good anti inflammatory pain killer gel on your back. If the pain increases do some hot fermentation locally and make a routine of doing some back strengthening exercise daily . Start with 20 mins twice a day to 40 mins twice a day and walk for 30-40 mins a day at least. It will help you build you back muscle and you will be able to continue with you cricket in few weeks but do not over look at the symptoms right now."
},
{
"id": 203635,
"tgt": "What could be the reason for the problems facing while masturbating such as nerves swelling up and giving pain in the penis?",
"src": "Patient: Hi Doctor I am Nag, from past 3 months I am facing problems while I masturbate, the nerves on my penis are getting swelled and giving pain, initially when I used to do twice or thrice my penis used to become weak penis, but now though I don t do masturbate frequently still my penis seems like I did masturbate more than twice, not sure what is wrong, its always a weak penis and not recovering from that state, initially after an hour its used to become normal the nerves on my penis used to become normal, but now its not the case, now its always in a state wher I feel like I had sex multiple times, sometimes its painful when the penis gets erection, now I am afraid that I can go for marriage or not, as I can ruin someone life with my defects, please help me know what is the problem Doctor: Hi, Welcome to Health care magic forum. Mastrubation is not bad for health, nor for marriage life. But for you it is not good, because you started thinking negatively. So i advise you to consult a physician for treatment of the psychological status. and physical build up. Take more of green leafy vegetables, pulses,sprouts, and protein rich foods to have a good health. Wishing for a quick and complete recovery. Thank you."
},
{
"id": 209384,
"tgt": "How do I overcome my stress?",
"src": "Patient: Hi,I have recently been diagnosed with chronic narcolepsy. I am extremely worried due to the fact that I have recently taken out a mortgage on a new property and as a result of my condition I have been forced to cut my working hours substantially. My main worry is that I can no longer afford the repayments and I have been told that stress makes my condition worse. When I took the mortgage 12 months ago I did take a life and critcal illness policy and I have spoken to them in great detail. Apparently I am not covered by this policy as my condition is not considered to be life threatening. I am at my wits end. Any advice or feedback on this would be much appreciated.Many Thanks Doctor: HIThanks for using healthcare magicI think, you are in stress and due to that reason, you are facing problem. In case, you are feeling tiredness, sleepiness or low mood, you can try some antidepressant like paroxetine or some stimulant like modafinil that would keep you active and help you to come out of current problem. You can also try some relaxation exercise. In case, you need further help, you can ask.Thanks"
},
{
"id": 16251,
"tgt": "Patches in the beard, head, itchy. What can this be?",
"src": "Patient: Hi, about 6 months ago I was working in Queensland, we towed a barge in a tugboat all the way from mackay to Brisbane, this took in all 6 days and when we arrived at our hotel room to my surprise I had two round ten cent like circles in my beard, I went to my local gp and he referred me to a specialist that was 4 months ago, I have slowly noticed it get bigger and bigger so I booked the specialist and still have a two week wait, I shaved my hair off two weeks ago not for any reason except a change and soon realized I had two more patches but these have massively expanded and now theres easy a dozen or more and I can pretty much tell by sight they are getting bigger every day, even with a completely shaved head they are extremely noticeable because the skin is a pale white smooth and also I have noticed my head is becoming more and more itchy, I am really starting to worry have you any ideas what this could be.. Doctor: Hello. Thanks for using this forum. By the description of your clinical symptoms this seems to be some sought of infection perhaps of fungal origin, as these tend to spread fast and be more itchy. However as you have already booked with the specialist, he/she can determine its cause by taking even a small skin biopsy. Usually for such conditions antifungal creams along with antifungal shampoos are usually prescribed. Feel free to write in. Regards."
},
{
"id": 177057,
"tgt": "Suggest treatment for accidental consumption of clorox by infant",
"src": "Patient: Hi. My 2 year old little sister drank from a cup with a little bit of clorox in it. I know she didn t drink a lot, considering the cup was nearly empty. But she started coughing a lot and threw up a little, so I gave her milk to drink, then she drank warm water ..then more milk. She seems fine for 2-3 hours and eventually fell asleep. There s no sign of her in pain, or bleeding, should I be worried? Doctor: it is chemical compound which was causing problem. she coughing because of this . it may causing burning in mouth, u should take her to nearest hospital and do check up of her . u not have to wait till something to happened"
},
{
"id": 81633,
"tgt": "Suggest treatment for fever and chest pain",
"src": "Patient: I have been sick with bronchitis and sinus infection for 2 months, have been through 2 courses of antibiotics and steroids but seem to be going downhill again with fluctuating temps, terrible body aches especially in chest, back and joints, feel like I m going to die Doctor: Thanks for your question on HCM. In my opinion you should consult pulmonologist and get done chest x ray to rule out lower respiratory tract infection ( LRTI ). LRTI is very common after upper Respiratory tract infection ( URTI ) especially sinusitis. Since you have taken 2 courses of antibiotics but not improving, possibility of resistant infection is more.So get done chest x ray and sputum culture and sensitivity to guide antibiotic therapy. You may require longer treatment."
},
{
"id": 94144,
"tgt": "Had pain in left side of stomach and pain while breathing for 20 minutes. Cause?",
"src": "Patient: Sharp stabbing pain in lower left side of stomach , happened very suddenly and felt like I might pass out, also hurt to breathe, stomach and chest felt tight, like i could not take a deep breathe, it lasted about 20 mins, I had to sit down because of the pain but then it just went away as quickly as it came, still don t feel well but the extreme sharp pain is gone, very confused about what this is, what caused it and why it came and left so suddenly Doctor: Hi, Thanks for posting your query. Do you have fever/ watery diarrhea/ constipation/ spasmodic abdominal pain/ nausea or vomiting/ weight loss/ mucus discharge in stools? Do you have any mass palpable in abdomen? Is there any aggravating or relieving factor for abdominal pain? With the available described symptoms, there could b possibility of peritonitis or muscular ache. You should consult with internal medicine specialist/ gastroenterologist and should go for thorough check up. You should go for complete blood count, urine routine microscopy, renal function test, serum amylase and lipase, colonoscopy, UGI endoscopy for better clarification. You should take low fat, spicy soft diet. Meanwhile, you should take antispasmodics along with proton pump inhibitors. Treatment depends on exact diagnosis. Take care, Dr. Mayank Bhargava"
},
{
"id": 62867,
"tgt": "What causes raised lump on stomach while doing sit ups?",
"src": "Patient: When I do sit ups I have noticed a raised lump on my stomach when I am pulling myself up. When I lay back down and relax my stomach it doesn't seem to show. I have been to my own doctor and was told that this was common in a lot of men. Could it be a hernia? Doctor: Hello,Thanks for posting in HCM.Yes sir your suspect is right, any lump coming up during touting of rectus abdominal muscles like when doing sit ups shows that there is some weakness in anterior abdominal wall. But however if the size is very small then it is not a matter of concern. But if you feel it is large enough or increasing in size gradually then you might need consultation of general surgeon to do a detailed clinical examination and recommend treatment after examination. Also an USG scan of abdomen can show the exact size of defect helping to plan out further line of management.So, yes it could be hernia (umbilical or paraumbilical) and it needs to be examined clinically.I hope I have answered your query well. If you have any more questions kindly directly write back to me, will be happy to help you.Thank you"
},
{
"id": 63749,
"tgt": "What causes lump on left side of lower abdomen?",
"src": "Patient: There's a still lump in the left side of my lower abdomen sort of by my hipbone, above/to the left of my uterus. The lump doesn't extend past my adominal wall, so it's not visible, but I can easily feel it and it's different from my right side. The same stiffness/mass/whatever it is can be felt in my upper vaginal wall. Doctor: Hi, dearI have gone through your question. I can understand your concern. You an feel mass inside left lower abdomen. You may have enlarged spleen or ovarian or uterine mass. You should go for ultrasound abdomen. It will give you exact diagnosis. Then you should take treatment accordingly. Hope I have answered your question, if you have doubt then I will be happy to answer. Thanks for using health care magic. Wish you a very good health."
},
{
"id": 175818,
"tgt": "How to treat chest congestion in a 1 years old child?",
"src": "Patient: My son is 1 year old and after every 20 days he fall sick with congestion in chest and then the doctor recommends him the dose of budecort .05 mg+ Levolin .63 twice a day and once only levolin .63 . I need to know as they are steroids can they harm my son in long run. Doctor: Hi...Suggestions:1. You cannot use oral steroids are associated with complications.2. There are certainly alternative management therapies in allopathy now a days. medicine has advanced a lot and not asthma is 100% controllable.3. Inhalers are the newest management strategies for this - RATHER THAN NEBULIZATION - METERED DOSE INHALERS ARE BEST.4. Triggers can be environmental changes/ dust/ talcum powder/ seasonal changes/ un-cleaned a/c vents/ cold weather etc....we can specifically say this is the cause - unless we observe the kid closely - best person is the parent.5. I suggest you shift over to metered dose inhalers of Budecort and Levolin rather than nebulizationHope my answer was helpful for you. I am happy to help any time. Further clarifications and consultations on Health care magic are welcome. If you do not have any clarifications, you can close the discussion and rate the answer. Wish your kid good health.Dr. Sumanth MBBS., DCH., DNB (Paed).,"
},
{
"id": 110937,
"tgt": "Suggest remedy for back pain due to bulging disc",
"src": "Patient: i have a bulging disc as well as rls and go for days without sleep sometimes and then have to deal with the back pain all of the time im tired of going to the doctors office and know that fairly strong pain meds are the only thing that helps both dont know what else to do ? Doctor: Hello, I have studied your case. Due to compression of this nerve root there is pain in your back and leg.For these symptoms analgesic and neurotropic medication can be started.Till time, avoid lifting weights, Sit with support to back. You can consult physiotherapist for help.Physiotherapy like ultrasound and interferential therapy will give quick relief.I will advise to check your vit B12 and vit D3 level.Hope this answers your query. If you have additional questions or follow up queries then please do not hesitate in writing to us. I will be happy to answer your queries. Wishing you good health.Take care."
},
{
"id": 168484,
"tgt": "What causes red rashes around the mouth?",
"src": "Patient: I am frightened my son has impetigo. His child care centre had a case yesterday. He has had a little rash around his mouth for a few weeks now, from his dummy keeping the area moist i think but the rash seems more red today and slightly bigger. Is he ok to go to care tomorrow so long as there are no blisters? Doctor: dear madam, sorry to hear that.if you suspect impetigo better get medical treatment as the lesion tends to spread and it is contagious.so dont send him to child care centre till his lesions heal.have a good day"
},
{
"id": 43578,
"tgt": "Have PCOS. Tried IUI and IVF. Gaining weight. Cure?",
"src": "Patient: I m a 30 yrs old married woman. My marriage has gone 7 yrs. I am having problem of PCOS . therefore, I have problem of irregular periods, infertility etc. Now, my weight is 69kg and my height is only 5ft. Two times IVF/ICSI and two times IUI treatment have been done with me. In IUI I didn t get any positive result but in IVF I got positive result but in my first IVF it was miscourage due to lack of heart-bit and in second IVF it had been an ectopic case and I had to loose my left tube. Now still I am at the position from where I have started but with weigh gain. I have also taken Glycomet, Obimet, Birth control pills, Duphastan etc many times but I couldn t get any permanent result and also not get any issue. What shold I do for issue and to get rid from this irregular period problem and to lose my weight? Doctor: Hello, The first and foremost important thing in PCOS management is weight reduction. According to BMI calculations, you are overweight. So, first thing i would suggest you is weight reduction by diet and exercise. You can consult a dietician and follow the diet advise. you need to have regular physical activity. Along with this the tablets like glycomet, OCpills, and others will help you to regularise periods. May be after a gap of 3-6 months, you can restart infertility treatment. Alongwith this, also evaluate any other factor which needs treatment. Dont take stress. Hope this information helps. Dr. Mrs. Sumeet Baheti"
},
{
"id": 80250,
"tgt": "What causes heaviness/tightness in lungs and breathing difficulty?",
"src": "Patient: Hi there, for the past couple weeks i ve been having some breathing difficulties. 1) its hard to breathe through my nose, feels like its blocked but theres mucus 2) lungs sometimes feel heavy or tight . I did have a cold about a month ago and cough up a bunch of phlegm. When i breathe i can feel a rumble in the middle of my chest, sometimes it goes away. I am a smoker and i also have anxiety disorder. Ive been to the ER 3 times last year because i felt like i couldnt breathe. They gave me an ekg and xray, said everything is fine. Saw my primary doctor in May and told her about the rumbling in my chest and heavy breathing, she said my lungs sounded fine... It feels like my breathing has been getting more difficult and just wanted to ask on her for advice before going back to see a doctor. My lungs dont hurt, its easier to breathe laying down. Could this just be my anxiety? Doctor: Thanks for your question on HCM. I can understand your situation and problem. In my opinion, we should first rule out bronchitis in your case. Bronchitis is common after cold (viral infection) and show similar symptoms like breathing difficulty, wheezing etc. So better to consult pulmonologist and get done 1. Clinical examination of respiratory system 2. PFT (Pulmonary Function Test). If PFT is normal then anxiety is the most probable cause for your symptoms. So better to consult psychiatrist and get done counseling sessions, try to identify stressor in your life and start working on its solution. Avoid stress and tension. Be relax and calm. You may need anxiolytic drugs too. Don't worry, you will be alright."
},
{
"id": 15563,
"tgt": "Few small itchy lumps on both sides of upper thigh near vagina. Tried hydrocortisone cream, no help. Solution ?",
"src": "Patient: I have a few small itchy lumps on both sides of my upper thigh near the vagina . The lumps are similar to a mosquito bite and dark red. They get very itchy. when I itch the area, the lumps seem to get bigger and more will pop up (i guess inflammation?). Sometimes the skins dry and it looks like a scab. I ve had it for 3-4 days now. I tried hydrocortisone cream 0.5% on it but not sure if it did anything. I don t sweat or exercise so it s definitely not heat rash . Please shed some light! Doctor: Hello,Thanks for the query.You might be suffering from a fungal infection,Probably candida infection.Dont apply hydrocortisone , it may worsen the situation.You can apply an antifungal cream like clotrimazole.Also take an anti histamine tablet for the itch.Dont worry you may get relief with this.Please meet a dermatologist and discuss regarding this.Let me know if you have any other doubt.you can ask a direct question to me on this forum, following the below link.https://urldefense.com/v3/__http://www.healthcaremagic.com/doctors/dr-rahul-kumar/64818Wishing__;!!Mih3wA!SBzm6_kI6hCZ58EPH6N_05MFfiPbxWXT0a2TJCdFQObRWm5mV5ur7hUOMa8clQ$ you a good health.Thank you"
},
{
"id": 160046,
"tgt": "Baseball size infection of arm. Had breast cancer",
"src": "Patient: My mom was admitted into the hospital tonight with a baseball size infection in her left arm by her armpit. She had breast cancer 2 years ago and a left myscetomy of the breast, also had about 27 lymph . Nodes removed from the same arm. Hospital said this infection is so bad it is life threatening, and could be an abcess... what is your opion? Doctor: Hi, Thanks for the query. Infection is invariably very common in cancer because of decrease in immunity. Lymph nodes could be due to infection or metastasis. This can be confirmed by investigations, antibiotics need to be given on the basis of microbiological studies. Your Physician will look into all these and prescribe appropriate antibiotics. These infections are treatable and hence not life threatening. Kindly take the opinion of treating doctor and give proper medication. The surrounding area of infection needs to be cleaned using either Spirit or Dettol/Hydrogen peroxide. Thanks"
},
{
"id": 137632,
"tgt": "What causes cramps on both hands?",
"src": "Patient: Hi Doctor, I get cramps on both of my hands. Got my blood test done nothing related to magnesium, or cholestrol or heart issue as I got my stress test with other tests, checked blood pressure. I am not sure what cause the cramps in my arms. I get the cramps when I wear something tight or lift weight. The cramp comes on my triceps and goes to my back ribs as well. It lasts few minutes. I mostly happens when I am anxious or stressed. Not sure what the cure is Doctor: Hi,Thanks for your query.After going through your suffering, it seems you have developed muscle spasm (cramp).The cramps are caused by hyper excitability of the nerves that stimulate the muscles. Drugs given for asthma, angina and Parkinsonism also lead to cramp. Infact, till date there is no clear cut evidence for the cause of muscle cramps. But the most common cited theories include -- Altered neuromuscular control- Dehydration- Electrolyte depletion- Poor conditioning- Muscle fatigue- Doing a new activityFollowing measure will help-*Drink fluid-An electrolyte imbalance can be the cause of muscle spasms, so rehydrating with an electrolyte drink can help restore balance.*Stretch the affected muscle with a mild, static stretch.*Use ice to numb the pain in a cramp, reducing inflammation and allowing the muscle to relax.*Bring the affected muscle into a relaxed position and apply direct compression to the cramp.I do hope that you have found something helpful and I will be glad to answer any further query.Take care"
},
{
"id": 59983,
"tgt": "Had cholecystectomy. Taking Amoxicillin for infection. Have PCOS. Now have hot flashes, tiredness and pain. Whom should I see?",
"src": "Patient: Hi there. I had a cholystectemy back in March and as of 4-5 days ago the same pain I had prior to surgery has resurfaced. I am rather confused, as I have had multiple other medical issues going on and am currently on amoxicillan for a viral turned bacterial infection (just a cold that lingered, leaving swelling and ent symptoms, also chest junkiness). I also was diagnosed with pcos a few years back and gained a substantial amount of weight with my daughter (now almost 3) that I have yet to lose. I have been feeling pretty sluggish and tired (particularly in te morning) and have hot flashes periodically throughout the day (mostly toward the evening). Usually temp is in the 97 s and it has been running in the 99 s on average over the past 10 days. I just want to know who I should consult to get help--and I mean help--my surgeon for the gallbladder to reevaluate, an endo, a reproductive endo or an osteopath, etc. I need someone to take me seriously--I m absolutely over being second guessed. Any thoughts or advice would be greatly appreciated. Doctor: Hello You seem to be suffering from post cholecystectomy syndrome and it describes the presence of abdominal symptoms after surgical removal of gall bladder. The symptoms are right quadrant pain,nausea,vomiting,bloating etc. In 50% cases the cause is biliary and due to remaining stone,choledococyst etc while in remaining 50% cases,it is due to non biliary causes. You must consult your surgeon and get yourself evaluated. Your temperature is due to residual viral infection and shall be all right soon. For PCOS you must take METFORMIN 500 mg twice a day to reduce your insulin resistance and must start a regular exercises schedule to reduce your weight. Get yourself tested for hypothyroidism. Consult a gynecologist,if you are planning a pregnancy,otherwise consult a endocrinologist for the management. Thanks"
},
{
"id": 134819,
"tgt": "Suggest remedy for complex tear med meniscus posterior horn at root junction",
"src": "Patient: never had knee pain till 4 weeks ago, 55 yo. It went away after 5 days. Then running to car one week ago feltva pop and severe pain. Could hardly walk for 2 days. It is sore but much better, MRI shows complex tear med meniscus posterior horn at root junction. Chondromalacia severe at medial compartment. Resting it now, no golf just work. I m a podiatrist and can t afford to be off work, any suggestion. Oath appt next monday Doctor: Brief Answer:Requires arthroscopic repair or partial menisectomy by an orthopedic surgeon.Detailed Answer:Warm Welcome. I appreciate your trust in HCM and assure you are on best portal on the globe for expert online medical services.I have thoroughly reviewed health query you raised, and well realize all of those concerns. Being your physician, I will take care all of them today and will extend my expertise to solve the problems.Based on what you said, if you want to regain your knee functionality, I would suggest surgery preferably via arthroscopic repair or partial menisectomy. It is up to your treating surgeon that what method he applies. After your surgery, you will enter a rehab program and at the end of it, you will have sufficient efficacy to continue your work.If you don't opt out for surgery, then bed rest and knee brace can be an option followed by physiotherapy but you may not get satisfactory knee function.Hope I addressed all the concerns you have. Let me know if you have further questions. I will be more than happy to clear any doubts. Keep a follow up to see how you doing.Regards!Dr. Ali Raza"
},
{
"id": 46808,
"tgt": "What does high protein level in kids mean?",
"src": "Patient: my 4 yr old son, has high protien in his urine. we took a sample of his urine to his doctor. is this normal for children to have high protien levels and should we be worried that he may have some type of \"kidney\" disease??? We have not received his lab results yet. He is also adopted, and we do not know if biological parents have any kidney problems... What should we be expecting and is this painful for him?? Doctor: Hello,Mostly the problem is due to bephrotic syndrome which is very common in kids and right tretment with right nephrologist wont harm him. Early diagnosis and treatment is always the best option.Regards Dr.Alekhya"
},
{
"id": 137822,
"tgt": "What causes swelling and redness behind the knee?",
"src": "Patient: Hi I have a very sore leg above my calf,and behind my knee. It is extremely hard to walk on! No pain when sitting. Behind the knee is swollen and slightly red. I have not knocked or banged this. I am 52 woman and just started statins a week ago . Any ideas what this could be ? Thanks sue Doctor: Hi,Thanks for your query.According to your description , pain over your knee seems to be inflammatory in nature. It could be an acute attack of gout.I advice you to give you rest to the part affected, take anti-inflammatory drug like motrin 1tablet with food as and when required (upto 4 tablets daily) to reduce pain and inflammation and consult your doctor for thorough examination and rule out any infective cause.I do hope that you have found something helpful and I will be glad to answer any further query.Take care"
},
{
"id": 188863,
"tgt": "uit smoking, thrush on tongue, high IgE level. Worried",
"src": "Patient: Hello, doctor..i have recently quit smoking but i have started to note thrush on my tongue , and on the sides, as well. I have very i IgE levels, last tested were 1100 u/ml but i am very worried about the oral thrush.I did a course on 20 days, of Nizral tablets, as i am also suffering from Seb Derm.Currently i am on AF 150 ( Fluconazole ) weekly. Doctor: dear karan , hi. oral thrush is a fungal infection of mucus membrane of mouth. it is mainly due to -poor health - diabetes -HIV. since your IgE levels seems to be raised which can occur in Immunodeficiency, please get the following tests done:- 1.CBC 2.Blood Sugar ( Fasting and PP ) 3. Elisa tet for HIV. Treatment you are taking is antifungal which will help in curing this fungal infection. thanks."
},
{
"id": 2881,
"tgt": "Are there any side effects of taking Fertyl ?",
"src": "Patient: i have PCOD and planning for baby.currently my doctor prescribed fertyl 150 mg and dhuphaston .After Ovulation i am getting injection for egg release.this is the fourth cycle of fertyl. first 2 months with 50 mg, and 3rd cycle with 100mg and fourth is 150 mg.what are the chances with fertyl and is there any side effects.my age is 27 and weight 58 Doctor: Hi,Fertyl contain clomiphene. It causes following side effects:- abnormal vaginal/uterine bleeding, breast tenderness or - discomfort, headache, nausea, vomiting, diarrhea, flushing, blurred vision or other visual disturbances, or - ovarian enlargement: presenting as abdominal or pelvic pain, tenderness, pressure, or swelling. Twins in 8-10% cases, ovarian hyperstimulation syndrome.But all these symptoms does not occur in all cases. Do not use more than 6 cycle and take it under advise of gynecologist. Contact your doctor immediately if you have shortness of breath, seizures, stroke, chest pain, and decreased urination. It will increase chance of ovulation by 80% and increase fertility in 45-50% cases. Along with Clomiphene you can also use HCG injection for release mature egg from follicle. Avoid stress, take healthy diet, drink plenty of water and do regular exercise.Hope this may help you. Contact further if follow up needed. Best regards,Dr. Sagar"
},
{
"id": 146407,
"tgt": "Suggest non surgical treatment for craniopharyngioma",
"src": "Patient: respected sir my father 63old. he is posted surgery two times due to craniopheningioma in jubilee mission hs in thrissur. then Dr decide to radiotherappy.. at last now father is totally dis order and he is almost deppressed.. now 12/08/14 father is admitted in same hospital... sir any solution nonsurgery....? what can I do sir...? any possibility in mediation.... ?please help me sir... Iam waiting for your valuable information.. Doctor: Hi, I had gone through your question and understand your concerns. A good alternative to treat Craniopharyngioma is Stereotactic Radiosurgery. This can be done in highly specialized neurosurgical centers. It is a single session treatment and short hospitalization ( usualy one day). Hope this helps."
},
{
"id": 59549,
"tgt": "Abdominal pain, fatigue, weakness, headaches, bloating. Elevated liver enzymes. Cure for symptoms?",
"src": "Patient: Hi, I have a question, I have lower abdomen pain, also, have upper right abdomen pain, weak, fatigue , headache . Went to the E.R., they told me that I had pain and my liver enzymes are elevated. I have a doctor appt. this week to recheck my liver and so my doctor can find out what is wrong with me. But now my pain is getting worse and my abdomen is swollen. I need to know what is going on? If you can help me with some information regarding this, because doctors don t know what is going on? Doctor: Hi. Thanks for your query Liver will be beneath the Right rib cage in the upper abdomen.So,pain there may be due to some abnormality in the Liver.It may be simple enlargement,infection,abscess,lesion etc. As your liver enzymes are elevated,you should be further investigated like Blood tests for Hepatitis A/B/C, Ultrasound abdomen,Repeat Liver function tests and even CT scan of the abdomen. Swollen abdomen indicates that there may be some collection in the abdominal cavity.It may be simple fluid or pus.You can ask your doctor to aspirate fluid from abdomen and send it for biochemical analysis. Nothing to worry. There is treatment for majority of liver disorders. Wish you good health"
},
{
"id": 115460,
"tgt": "What is meaning of c1 esterase deficiency?",
"src": "Patient: I have recently been tested for C1 Esterase deficiency and have had conflicting test results, however now seems it may be a true diagnosis. I do not suffer uticaria at all.Can you please explain in laymans terms what this is? And the difference between acquired and genetic if any?I don\u2019t really know what the diagnosis means for my future or what to expect if anything? I suffer from Hypertension and have had weight loss\u2026 could this be related? Doctor: Hello,Thank you for contacting HCMI understand your concern, I am Dr Arun Tank, Infectious Disease Specialist answering your query.C1 esterase is the enzyme present in our body which prevents the cascade of complement system. This system is also a part of the immune system. But to control this system C1 esterase is essential. As you are deficient for this enzyme there is likely to cause exaggerated response of complement system.The symptoms is angioedema means swelling around neck and face. which may cause chocking. It is mostly a genetic one not a acquired one.Hypertension and weight loss in not associated with this deficiency but it may be associated with some syndrome. I advice you to visit nearby multispeciality hospital and get your complete physical examination done this will give you idea reagrding the syndrome you have.You can ask for further queries here on bit.ly/DrArun.Thank you,Dr Arun TankInfectious Disease specialistWe wish you a best health at health care magic."
},
{
"id": 180469,
"tgt": "What causes pain in the gums?",
"src": "Patient: I have a dental question, is that something you can help with? Over the last 4 days I have been experiencing a mild/dull ache. When I apply pressure it source seems to be from my gums. On occasion I have a light bleeding in my mouth when I brush my teeth (I brush my teeth twice a day). I can still chew food on this side of my mouth if required but I don't attempt to chew anything too hard. The ache seems to be more prominent in the evening. I have a huge dental phobia/anxiety and haven't been to see one in approx. 15 years. I cannot remember a time I have had an ache like this before and I hope you can give me possibilities of what this could be and possible home treatments so I do not need to attend a dentist. Doctor: Hello and Welcome to \u2018Ask A Doctor\u2019 service. I have reviewed your query and here is my advice. Your explanation is pointing towards gun infection or Gingivitis.It is the cause for gum pain and bleeding from gums.So my suggestion is to consult an Oral Physician and get evaluated and a thorough clinical examination should be done.A thorough scaling followed by antibiotics like Metronidazole and gum paint massage can help in improvement.Do warm saline gargles and antiseptic mouthwash gargles.Hope I have answered your query. Let me know if I can assist you further. Regards, Dr. Honey Arora"
},
{
"id": 128831,
"tgt": "What causes pain on the top of the feet after a back surgery?",
"src": "Patient: I have my mother in law in the hospital that is recovering from back surgery 1 month ago. Her feet hurt on the top if you touch them and she cannot stand on them. By the arch she has redness like a bruise on the sides of the arch. Her potassium level is low so they kept her overnight. Her potassium is still low after 24 hrs. Her uric acid was on the high side and they thought she had gout. They are doing an MRI tomorrow on her feet. Doctor: Hi, I had gone through your question and understand your concernsWith such presentations in my clinic, two possibilities are common for foot pain:first possibility:Hyperuricemia, Gouty arthritis is one of the common causes of foot pain which could be treated by urate lowering drugs.second possibility: Radicular pain from back problem due to Nerve root injury ,for this possibility you have to wait and I suggest using neurotonics ."
},
{
"id": 42639,
"tgt": "Will depo shot cause infertility?",
"src": "Patient: hi, i was on depo and i had my last injection in december, me and my fiancee have decided we would like to start a family. I have started having my period its been normal for about 8-9 months now but still nothing, could of the depo made me infertile? Doctor: Dear member,Thanks for writing to healthcare magic.The return to fertility is delayed after depot preparation compared to other contraception. After a year if you have not conceived then you will need detailed infertility analysis and your partner semen analysis.Usually most patients conceive in a year. Use ovulation detector kits from day 12 of cycle for 3-4 consecutive daysThus timed intercourse can give better success.ThanksDr Bhagyashree"
},
{
"id": 157435,
"tgt": "Have lung nodule in lower lung. Is doing needle biopsy only way to find out cancer?",
"src": "Patient: Hello, Doctors have found a lung nodule in my mothers right lower lung. They want to do a biopsy because they don't know if it's lung cancer, or from breast cancer or from colon cancer. Is this the only way to find out is by a needle biopsy? Also does a needle biopsy spread cancer cells? Doctor: though imaging can suggest malignancy based on certain characteristics, the only way to confirm cancer is by performing fine needle aspiration cytology (FNAC) or biopsy. They are safe and do not spread cancer cells"
},
{
"id": 124403,
"tgt": "What causes warm sensation on leg?",
"src": "Patient: hello doctor. i have a warm sensation every now and again on the outside of my right leg it goes away after a few seconds i had this about 2 years ago on my right leg . i do have a sciatica problem and thats causes my leg and back to ache. not sure if it is connected. hope you can help me on this. thank you sue. Doctor: Hi, Yes this is connected to sciatica. As the nerve carries two function - sensory and motor. When motor is involved there is weakness. When sensory is involved you will have touch, temperature, pain etc as symptoms. The change in feeling of the hot and cold sensation is carried by nerve and so as you mentioned you had sciatica than it is one of the reason. Kindly do regular exercise which you may have been doing in past for your sciatica. Symptoms should subside sooner. Hope I have answered your query. Let me know if I can assist you further. Regards, Jay Indravadan Patel, Physical Therapist or Physiotherapist"
},
{
"id": 210753,
"tgt": "Noticing sleeplessness and irritable nature",
"src": "Patient: I have a friend who is a professional I can't tell what is wrong with him. He is usually late to work but works 7 days a week. He seems out of it sometimes and gets very focused on trivial things and has difficulty following things through. He is very messy and hardly ever sleeps. Always on energy drinks do not touch alcohol. At times very abusive, irritated, irrational and has spending sprees. Buys a lot of boy toys like helicopters etc. Always thinks he is superior to others. He went through a marraige break up which the ex says he is very manipulative and physical abused but not proven. Doctor: Hi, Thanks for sharing. Your friend probably has a personality disorder. He need behavior and psycho therapy. His irritability is because of excessive use of energy drinks and sleep depreciation. You should take him in confidence and help him by taking him to some good psychiatrist and be assured that he is treatable and he will thank you if you will fulfill a real friend's job."
},
{
"id": 222885,
"tgt": "What is the pain in thigh and stomach during pregnancy?",
"src": "Patient: Hi, I m 36 weeks and 3 days, been feeling pain in my right upper thigh. I am suffering from a hemmoroid and using cream for it. But also I m feeling a bit sick and achy. I ve been feeling a lot of pressure and tummy tightenings all wEek is this something to worry about?. Doctor: Hi dear, I have gone through your question and understand your concerns.Pain in upper thigh at the gestation of around36 weeks is most likely due to onset of labour pains.It has no association with the hemorrhoids.I will suggest you to consult an OBGYN specialist to get properly examined, and get appropriate treatment.Hope you found the answer helpful.Wishing you good health.Dr Deepti Verma"
},
{
"id": 183645,
"tgt": "What causes gum bleeding when brushing in kids?",
"src": "Patient: My 6 years old cute little daughter when was brushing her teeth. There were bleeding gums that I noticed. This has happened like the second time in 2 months. I am terrified to think something wrong is happening to her. I am feeling really paranoid. She is healthy otherwise with occasional cold/flus as all kids probably get in her age. How common is this in kids to have bleeding gums. I am also her dentist today Doctor: Hello,Bleeding of gums in the kids at this age is common so dont worry, but be careful with her diet reduce sticky food and fruit juices and all if at all given clean it that time itself, and you should properly check her to do brushing twice daily though children at this age tend to cry for not doing brushing. Maintain good hygiene. Bleeding is simple gingivitis common in this age group children.Hope I have answered your query. Let me know if I can assist you further. Regards,Dr. Rohit S Menon"
},
{
"id": 205394,
"tgt": "How to deal with Heroin addiction and paranoid behaviour in a partner?",
"src": "Patient: I am concerned about my grandsons living with my daughter who is exhibiting bipolar symptoms and her boyfriend that has told me that he has been diagnosed paranoid schazephrenic. neither are under a dr care for these conditions. her boyfriend was born with as a heroine baby and has had serious drug problems from street drugs. recently in August he beat her up and held her captive trying to gauge her eyes out and kicking and punching her. a friend of hers husband beat him up and has post concusion disorder sadly she returned to live with him despite having a restraining order and DA speaking up against having it revoked. he is still on probation and seems to be well behaved lately. my concern is how dangerous is he and is it this mental illness that causes this behavior. he is extremely controlling and paranoid and fuels this belief in my daughter Doctor: Hello,Thanks for your query. I can understand that the situation with your daughter's family must be quite distressing to you. You are indeed right in being concerned about the safety of your grandsons and your daughter.You have mentioned that your daughter's boyfriend has been diagnosed to have Paranoid Schizophrenia and that he has serious drug problems as well. Now, Paranoid Schizophrenia is a serious mental illness and requires regular treatment and follow up. Unfortunately, it seems that he is not on any medication or monitoring, which increases the risk of relapse of symptoms, that can lead to aggressive behaviour. Using street drugs like heroin, etc. further worsens the illness and increases the risk of violence. So, considering the above factors and the fact that he has already exhibited very violent behaviour in the recent past, I would say that his risk of violence / danger to your daughter and your grandchildren is very high. The situation seems to be further complicated by the fact that your daughter has mental health concerns herself. From the situation that you have described, I have concerns whether your daughter's decision-making / judgement capacity is being clouded by the mental health problems that she may be undergoing.From your side, I would suggest that you consider the following two things:1) You can inform Child Services that you have serious concerns about the safety of your grandsons. They will take appropriate steps to do a risk assessment and put in safety measures in place.2) You can take your daughter and also her boyfriend to a mental health professional to ensure that they are started on appropriate treatment and followed-up regularly.I hope I have addressed your concerns and do sincerely hope that this difficult situation resolves as the earliest.Kind regards,Dr. Jonas Sundarakumar MBBS., DPM., MRCPsych.(U.K.), MBA.(Hosp. Mangmt.)Consultant Psychiatrist"
},
{
"id": 90758,
"tgt": "What to do for travelers stomach?",
"src": "Patient: Good Evening I bought some antinal-nifuroxazide for travellers stomack in Nov of 2010 in Egypt. I am going on a trip to China in Feb and am hoping the shelf life is still good. Would you be kind enough to give me an idea of the shelf life. Appreciate your help Daryle Doctor: Hi thank you asking HCM to my opinion you can use nifuroxazide ore ciprofloxacine. wishig you nice trip dr.klerida"
},
{
"id": 25619,
"tgt": "Is IRBBB dangerous?",
"src": "Patient: my 11 yr old son was told he has IRBBB. Naturally as parents we are so stressed. He is VERY athletic and loves sports. Sometimes gets a little short of breath with endurance sports like Basketball.. we have a dr's appt next week. Hopefully will get news... but how serious is IRBBB? Doctor: Thanks for your question on Health Care Magic. I can understand your concern. IRBBB is Incomplete Right Bundle Branch Block. First of all, you need to understand basis of bundle branch to understand IRBBB. There are two bundle branches in heart which transmit the impulse from atria to ventricle. Right and left. Any block in this transmission, can cause problem in transmission of impulse from atria to ventricle and hence ventricle will not function properly. In pediatric patient, most common cause for this is developmental defect. So in your son's case, his right bundle branch is blocked incompletely. So right ventricle is not functioning well. Right ventricle is connected to lungs, so he feels shortness of breath on exertion. Honestly speaking, this is serious health issue and this can worsen with the age. So consult cardiologist as soon as possible for this IRBBB. Hope I have solved your query. I will be happy to help you further. Wishing good health to your son. Thanks."
},
{
"id": 32943,
"tgt": "Suggest remedy for swelling and tingling in lips",
"src": "Patient: Three days ago my bottom lip started to tingle and had numbness. The next day I woke up and where my lip was tingling was now swollen twice the size. The same day my upper lip started to tingle and now woke up and both lips are swollen. There are little bubbles inside my lips on the swollen area. I have taken benadryl both days to see if that helps. Doctor: HiThanks for your query at HCM.It appears to be due to allergic reactions or side effects of a drug. If you are on drug kindly tell your physician regarding this. If not it is likely due to allergic reactions. I would suggest you that drugs like cetrizine, lorataine or desloratidine will be beneficial. Be observant and if any difficulty in breathing then visit your health care provider immediately.Take careHope I answered your query"
},
{
"id": 43207,
"tgt": "Trying to conceive. Irregular period, depression. Primary infertility. On ossur plus and glycomet. Negative PT. What can I do?",
"src": "Patient: Hi Doc, i am 29 and my husband is 30, we had being trying for a baby since past 1 and half year. i met one gyne according to her its a primary infertility she gave me few medicine like oosure plus and glycomate 500 and 5 days tablet for the irregular periods after that i got my periods but now again it has become irregular. last was in the month of August she has also suggested me to lose weight which i am doing. but cant find a way out, kindly help me and suggest me what we should do. pregnancy test came negative i checked last week only.i am going under severe depression. Doctor: Hi, to be fertile, both the partners should be tested and use medicines. You have got irregular periods, it might be due to the P.S.O.S. or anemia. the doctor gave medicines for P.S.O.S. it is also to be confirmed by trans veginal U.S.scanning, and confirm the cure. I advise you to take more of green leafy vegetables, pulses, sprouts, and proteine rich foods. Thank you."
},
{
"id": 191471,
"tgt": "What causes re-occurrence of leg weakness after discontinuing Glucosamine shots?",
"src": "Patient: Hello, I have been experiencing leg weakness as well as minor spasms in my legs. At night, my legs are restless as I attempt to sleep. I have had lower back pain as well for a couple of years, which the doctors have ruled out any issues with my kidneys. I was placed on an injectable B-6, B-12, Glucosamine compound a few years ago for joint mobility and swelling. I recently got back on the injections after being off for about a year. I thought that maybe the weakness was a result of resuming the injections because that s when the weakness began, but I m not sure. I also have Type 2 diabetes. Doctor: Hello Thanks for posting to HCM Take tab. Neurobion forte twice a day. Do routine test for Sugar level. with Regards"
},
{
"id": 95271,
"tgt": "Could my abdominal pain be due to ulcers ? Can I take perloc 40 mg tablets to treat my condition ?",
"src": "Patient: hi..i complain from abdominal pain and i took alot of drugs for neurities and arthirities is these drugs cause pain??? female 56 years old complain from abdominal pain i goto doctor and tell me that s due to medicatin you take it which cause poor digestion and give me perloc 40 mg tablets and these drug for ulcer and because i trust in all doctors of these site and i didn t take these drug is these good for me or i worry that i have already ulcer and doctor didn t tell me..please help me????? Doctor: Hi.. First relax mam, your abdominal pain may be simple gastritis, may be caused by the medications u have taken, to diagnose ulcer u have to get endoscopy done, for now u can take the tablet doctor has prescribed it wil help u. If pain persists consult gastroenterologist."
},
{
"id": 117704,
"tgt": "Does a heavy period mean a normal haemoglobin level?",
"src": "Patient: My iron levels and hemoglobin have been diagnosed as being low. I have been taking 3iron pills a day for last three months. My period is super heavy this month so I was wondering if that could be due to my iron and hemoglobin going back up to normal range Doctor: Hi,Thanks for asking.Based on your query, my opinion is as follows:1. Normal hemoglobin level does not cause heavy periods.2. Usually abnormal hormone levels possibly leading to endometrial hyperplasia is commoner one leading to heavy periods.3. If it repeats, get a endometrial biopsy done.Continue iron pills. Normal hemoglobin does not lead to heavy periods. If it repeats, meet your doctor for biopsy.Any further queries, happy to help again."
},
{
"id": 81180,
"tgt": "What causes sharp pain in chest and dry cough?",
"src": "Patient: hello i ve been having chest pains my chest was feeling heavy and sharp pains, but the chest pains are gone now is just that i am having very dry cough for 2 weeks now and is very nasty cough at night time and i get out of breath as wel what can dis be ? Doctor: Thanks for your question on HCM.In my opinion, we should first rule out cardiac cause for your chest pain.So get done ECG to rule out cardiac cause.If ECG is normal than no need to worry much for cardiac cause.Coughing for 2 weeks with chest pain can be seen in lung infection (pneumonia). So get done chest x ray to rule out lung infection.If chest x ray is normal than no need to worry much lung infection.You may have upper respiratory tract infection mostly.So drink plenty of fluids. Avoid hot and spicy food. Warm water gargles 5-6 times a day. Take antihistamine and anti inflammatory drugs.You will be alright."
},
{
"id": 29539,
"tgt": "What causes persistent vomiting and tiredness?",
"src": "Patient: My Husband has no fever, but has not got out of bed all day. 63 yr old male, bit overweight, at 3am last night he ate a whole strawberry cream pie and 8 breakfast bars right before bed. 5'11\" 250 lbs he has vomited anything he put in his tummy today, be it water, juice, anything. Heaving vomiting. Still in bed at 9:30 pm. Is this dangerous?? Doctor: Hi, I have gone through the medical history of your husband and understood your concerns. Your husband is going through an acute gastroenteritis (bowel infection) most probably viral. That is not serious unless his symptoms will get more and more aggravated. I advise to:- give him plenty of fluids (water with oral rehydration salts (ORS), soups, juice, yogurt)- if can't keep anything, just give plain water with ORS- if he is no vomiting anymore, give tasectan and probiotics to stabilize intestinal floraIf still vomiting, then, should send to ER for iv fluid replacement.Wish fast recovery."
},
{
"id": 208615,
"tgt": "What causes depression?",
"src": "Patient: i feel low these days cant enjoy life like i used to but i dont feel like depressed as im trying to fight..i cant speak or behave like an extrovert as i used to..can u figure it out .i am 21yrz old,height 5.7 inch weight 55kgs and i have long ago faced this type of situation about 3yrz bfr since then i was alrt and i have never consulted a doctor for a remedy as i was ok till now Doctor: Hi..Thanks for the query. Can you tell me how is your sleep & appetite over last 15 days or so? when you tell that you are feeling ''depressed'' thus this mean that you feel sad/low all the time irrespective of events going as per your likings or desires & whether this feeling of sadness has persisted over last 2 weeks or so? Simply putting depressive episode or a syndromal diagnosis of depression in Psychiatry requires a minimum of 2 weeks period of persistent sadness, decreased energy levels, decreased self confidence/esteem which remain irrespective of positive/negative events. Additionally there are associated symptoms of sleep disturbances like sleep onset & maintenance difficulties or excessive sleepiness,disturbed appetite (under or over eating), disturbed sexual drive (libido) etc. If untreated these symptoms may progress to ideas of guilt, feelings of hopelessness, suicidal ideas & attempts. You mentioned that you had similar symptoms 3 years ago; so how long these symptoms lasted then? Did you take any medical help at that point for these symptoms? It will be a good idea to visit your primary care doctor once for a detailed evaluation. Good luck"
},
{
"id": 168615,
"tgt": "Suggest treatment for fever in babies",
"src": "Patient: My son is 10 months old and he had a fever of 101-102 for the past 4 days and now that it is gone he started having red little spots on his face, down the neck and back and some on his legs. It got worsen since morning. And I was also giving him baby tylenol because he is teething and had fever. Do you know what could it be and should I go see his pediatrician? Doctor: Hi...Thank you for consulting in Health Care magic.Fever of few days without any localizing signs could as well a viral illness. Usually rather than fever, what is more important is the activity of the child, in between 2 fever episodes on the same day. If the kid is active and playing around when there is no fever, it is probably viral illness and it doesn't require antibiotics at all. Once viral fever comes it will there for 4-7 days. So do not worry about duration if the kid is active.Paracetamol can be given in the dose of 15mg/kg/dose (maximum ceiling dose of 500mg) every 4-6th hourly that too only if fever is more than 100F. I suggest not using combination medicines for fever, especially with Paracetamol.Hope my answer was helpful for you. I am happy to help any time. Further clarifications and consultations on Health care magic are welcome. If you do not have any clarifications, you can close the discussion and rate the answer. Wish your kid good health.Dr. Sumanth MBBS., DCH., DNB (Paed).,"
},
{
"id": 59907,
"tgt": "Stomach swelling, gastric issues, heavy head. Why does this happen?",
"src": "Patient: Hi, i am 28 y male,from last 2 year i facing problem like stomach swelling , gastric problem,heavy head digestive problem, some time increase heart beet, A treat from 4-5 dr some say its liver problem, some say stomach pgm,some say its acid problem, some say its my brain problem, i have also go through CT-scan, normal scan, endoscopy ,and many blood test ,in blood test only SGOT is increase, and in endoscopy some type infection, i have treated all tease problem,But my all this problem is hear,i feel my swelling all 24 hour, i confuse so Plz give me right advice what i do for that, what type body test will be do.............thank you.. E-mail YYYY@YYYY Doctor: hi, good morning, i feel the scan you are mentioning is brain scan.if your ultrasound scan of abdomen is normal you need not worry about the liver problem ,even with slight increase in SGOT(may be because of alcohol).the swelling of your abdomen is bloating due to acidity problem. please take omeprazole with prokinetic (domperidone) for 2 months.so the best test at this stage is abdomen scan. bye."
},
{
"id": 10729,
"tgt": "What causes white scaly patches on forehead at the beginning of hairline?",
"src": "Patient: i am puja studying mbbs , i am having white scaly patches ( medium size)on starting of my forehead where hair starts ,its from 1 week only i am thinking dandruff (because i am having it) .recently i changed my shampoo 4weeks back i think that may be the reason please help me .i am fearing may i have seborrhic dermatitis so plz help me Doctor: Hi puja and welcome to HCMBefore going into the diagnosis, just wanted to know if you have had similar complaints any time in the past, anywhere else on the body, on elbows, knees or on the trunk, soles and palms?Do you generally have any other skin condition or allergies/eczema?is there a family history of similar complaints?As per your description and short duration, you could have seborrheic dermatitis which is also a type of dandruff with flaky skin and itching. It is nothing to worry about. It may also affect other oily areas like face, chest, upper back. Change in climate, change in shampoo or anything causing oiliness on the scalp could cause an increase. Make sure not to apply more greasy gels, styling products and to wash off conditioner well as this can also dry up and look flaky. Keep your scalp clean and oil free.Another important diagnosis is scalp psoriasis along the hairline. So it is necessary to consult a dermatologist to examine and rule it out. It maybe seasonal as well and may spread.For now apply Salisia-KT or similar shampoo twice a week, avoid prolonged oil application. Take an antihistamine like Levocet for itching if needed.Eat healthy and cut down on oily, fried foods. Further supplements can be given after examination.See a dermatologist as advised.Hope this helps you."
},
{
"id": 57157,
"tgt": "Suggest natural remedy for liver hemangioma",
"src": "Patient: Hi, I have a liver Hemangioma, is there any way to cure it by natural medicine or diet ? I am 51yrs, 5'3, 159 pounds. I'm taking Synthroid 0.05, since January 2010, also taking Nexium 20mg for reflux. In the past I had duodenal ulcer and my gastroenterologist treated it with Prevpac for 14 days. My doctor ordered a MRI for liver and gallbladder because there is suspect for fatty liver. I will appreciate all your help. From Puerto Rico Waleska Rosado ( female) Thanks. Doctor: Hello Ms Waleska,How are you? A hemangioma of the liver is a fairly common finding on scans of the abdomen. Not all hemangiomas need treatment. Only large hemangiomas and symptomatic hemangiomas need any treatment. Some hemangiomas can cause a dull achy pain in the upper belly, especially the bigger ones. sometimes the hemangioma may clot and cause severe pain, or some may cause bleeding. Very rarely, they can also decrease in platelet counts. So in summary, if the size is small and you do not have any symptoms, I would just wait and watch for now.Hope this helped and please do not hesitate to contact me for further details - rxsuresh@gmail.com"
},
{
"id": 187230,
"tgt": "How to apply dologel for pain in mouth at the upper jaw?",
"src": "Patient: Sir, i have pain in my mouth at the upper jaw, but am not sure whether the pain is of the teeth or the gum but when i touch the teeth i feel and pain and am quite facing this pain over a week and its the first teeth(paired teeth) that starts just after the single set of teeth, to be clear. I have consulted a dentist and checked with him, he said that there is no damage in the teeth and gum and suggested dologel which is an analgesic and antiseptic gel, i have applied it overnight and felt better till morning but later again faced a little pain, i feel that's working and may take little time, I need help to know how to apply the gel and how many times a day??? Like Can I apply the gel for the teeth too that is painful and how many times should I apply it for a day and how long should I massage the gum with the gel??? Will be thank you for answering me Sir....!! Doctor: Hello,Thanks for writing in healthcare magic.First and foremost we need to rule out if the pain is from the tooth. If the tooth is having decay, or fracture pain can be due to that.A x ray can rule any abnormality. Dologel can help if there is any ulceration or soreness in the gums.It acts as a topical anesthetic agent thus numbing the sensation and reducing pain. You can apply it 2 to 3 times a day with your finger on the affected area. I would suggest that if the pain is not coming down or worsens go back to your dentist and get a thorough examination done. Many times dental pain are referred and a correct diagnosis is important.Hope that answers your query. Let me know if you have any doubts.Thanks,Dr. Sushma"
},
{
"id": 132120,
"tgt": "Suggest treatment for pain due to achilles tendinitis",
"src": "Patient: i have had achilles tendonitis and have done physical therapy before, the pain went away but came back a year later. I am a dance major in college therefore overuse is unavoidable.should i have an mri done to see what is left of my tendon and do pt again? Doctor: Hi Achilles Tendinitis is chronic recurring disease caused by overuse . If you have problem after one year , go for the therapy + anti inflammatory pain killers . You may have to restrict dancing for few weeks. MRI helps in diagnosis and planning of surgical treatment , So i would recommend MRI only if I am contemplating a surgical procedure or confirm my diagnosis."
},
{
"id": 87542,
"tgt": "What causes pain in the upper abdomen and nausea after meals?",
"src": "Patient: i lacerated my spleen 3 months ago, and have just started back to work - i am in a physical job.. after 5 days at work, i felt a tearing feeling in my left side, which won't go away, the left side of my upper abdomen is painful to touch. I feel very bloated and sick after I eat. could this be related to my previous injury? Doctor: Hi! Good evening. I am Dr Shareef, a general surgeon answering your query.With a history of splenic laceration three months back, I would advise you to go for a review ultrasound of abdomen to rule out any hematoma formation in the spleen, and if so, to see its progression by serial ultrasounds. If there is no such issue, then I would advise you for an anti inflammatory along with a proton pump inhibitor drug for a symptomatic relief. Also I would advise you for a routine stool test for ova and cyst and get it treated if positive.I hope this information would help you in discussing with your family physician/treating doctor in further management of your problem. Please do not hesitate to ask in case of any further doubts.Thanks for choosing health care magic to clear doubts on your health problems. I wish you an early recovery. Dr Shareef."
},
{
"id": 27426,
"tgt": "What causes fatigue along with poor vision?",
"src": "Patient: I have been feeling funny lately. Not so much energy...I am 57 and on 25 mg of toprol xl I have complained for years of fast heartbeats (sorta racing) .My doctors says its ok. I went to get my pap test a 2 weeks ago and my doctor said we need to talk about blood pressure 154/ 95 ...it was the 95 he was worried about. I told him I always watch the top number ....he explained to me. I have been watch every day taking pressure some times 140/79 I feel funny today ....eye sight has been getting bad the last month. My father had major heart issues. Today my pressure 194/ 94 than later 149/88 Doctor: The symptoms you describe may be due to fluctuations in blood pressure with it being on the higher side most of the times. I would recommend you to take regular reading at different times of day on different days, if persistently high start an anti hypertensive medication as per your doctor. Also cut down salt in your diet, no frozen and foods with preservatives. Try some exercise, 30-45 mins a day for atleast 5 days a week. If there are no palpitations and things well controlled with toprol, changing it is not justified. Toprol is not the first line anti hypertensive and adding a better med is justified at present."
},
{
"id": 210504,
"tgt": "Which is the best medicine for major depression and generalized anxiety?",
"src": "Patient: Hi.....I have been diagnosed with major depression and generalized anxiety disorder. I tried 4 ant-depressants but all made me slightly manic....I also tried trileptal and one similar which made me very aggressive.... now I have been just taking ativan as needed and think my depression is creeping back in... have been off meds for 3 months... when weaning , I felt pretty good on 5mg Paxil... should I go back to my psychiatrist and ask for that? What would be good since nothing works? Doctor: Hello,Thanks for choosing health care magic for posting your query.I have gone through your question in detail and I can understand what you are going through.Paxil is a very good drug for anxiety and depression, and I use it as a drug of choice. Although 5 mg dose is on the lower side but it can be gradually increased. You should visit your psychiatrist. The fact that you had improvement earlier with paxil suggest that there could be improvement with paxil even now. Hope I am able to answer your concerns.If you have any further query, I would be glad to help you.In future if you wish to contact me directly, you can use the below mentioned link:bit.ly/dr-srikanth-reddy\u00a0\u00a0\u00a0\u00a0\u00a0\u00a0\u00a0\u00a0\u00a0\u00a0\u00a0\u00a0\u00a0\u00a0\u00a0\u00a0\u00a0\u00a0\u00a0\u00a0\u00a0\u00a0\u00a0\u00a0\u00a0\u00a0\u00a0\u00a0\u00a0\u00a0\u00a0\u00a0\u00a0\u00a0\u00a0\u00a0\u00a0\u00a0\u00a0\u00a0"
},
{
"id": 51679,
"tgt": "Does the tab wysolone for glomerulus nephritis nephritic syndrome have any side effects ?",
"src": "Patient: iam rajkumar suffering from glomerulus nephritis nephrotic syndrometaking predisolone since 10 years now my age is 18 years i had left wysolone for 2 yearswill this tabwysolone has any side effects like retardation in growthcan i grow taller now my height is 46ft can i extend to 53 to 55ftwhat should be done to grow can i grow Doctor: Hi, Normally height can be increased up to the age of 20 years but as you are using steroid since long it might have adverse effect on your bones. Secondly the diseases you are suffering may cause lack of nourishing stuff,minerals etc will disturb your borne growth. So I am doubtful regarding your increasing height."
},
{
"id": 65819,
"tgt": "What is the lump on the forehead?",
"src": "Patient: I'm a 34 year old female, 6 feet tall weighing 300 pounds. I just noticed a large lump in the center of my forehead right behind the hairline that goes straight back like almost a retangular like lump for about an inch. I've been having migraines lately but I don't know if that's related. I don't know what this is, I didn't hit my head or anything and it has me worried. Doctor: Hi, dearI have gone through your question. I can understand your concern. You may have some soft tissue tumor like lipoma or neurofibroma. Or you may have some skin adenexal tumor. You should go for fine needle aspiration cytology or biopsy of that lump. It will give you exact diagnosis. Then you should take treatment accordingly. Hope I have answered your question, if you have doubt then I will be happy to answer. Thanks for using health care magic. Wish you a very good health."
},
{
"id": 113896,
"tgt": "What can be done for back pain alleviation ?",
"src": "Patient: Pain What to do for back pain alleviation? Doctor: Hi, Welcome to Healthcare magic forum, Lower back pain radiating to lower limbs suggests inter vertebral lumbar disc prolapse or bulge. The best way to diagnose is to get MRI LS spine. Precaution to take: Avoid riding two wheeler. Dont lift heavy weight. Sleep on hard mattress and on ur back and side, dont sleep in ur abdomen since the muscles will get more strained. Avoid sudden bending and turning. Take hot fomentation daily twice. Sit in a proper potion, dont hunch back when u sit. U can wear LS belt while travelling. Consult ortho specialist for further treatment. Hope I have answered ur question, Regards"
},
{
"id": 122395,
"tgt": "What would be the reason for leg pain and ankle swelling after recovering from typhoid fever?",
"src": "Patient: Helo sir/mam.. My mom had typoid fever a month back and later leg pain started severely and now her ankles gets swollen if she travels or does work for sometime.. what would be the the reason.. please do guide us.. consulted to doc but swelling up is still continuing and makes her very tired.. XXXX. Doctor: Hello, It is quite common after a bout of typhoid. Nothing much to worry and it will settle by itself. No definite treatment is required. If there is pain or fever, you can take paracetamol for symptomatic relief. Hope I have answered your query. Let me know if I can assist you further. Take care Regards, Dr Shinas Hussain, General & Family Physician"
},
{
"id": 143719,
"tgt": "Suggest remedy for hematogenous metastasis",
"src": "Patient: hi my sisters breast cancer has metastasized and is now she has a tumor in her hypothalamus and it is affecting the fluid in her spine...she has been through so much...and now we are fighting for her life...what are the treatments for this type of condition Doctor: Hi, breast cancer with metastasis with tumor in hypothalamus can be managed with palliative chemotherapy and radiotherapy.Advised to consult medical oncologistThanks"
},
{
"id": 217742,
"tgt": "Is pain in the neck related to heart diseases?",
"src": "Patient: Hello doctor, few days before My uncle felt that some critinias and some pain in heart.he checked with the doctor..He told that some cholestarl content is in the heart..chances are there to affect the heart and he gave medicine.in between my uncle is feeling some pain in neck. what are the test we need to take...Why this neck pain is coming..is it related with heart...pls advise us. Doctor: Hai.chest pain or angina can present with pains like left or right sided chest pain, shoulder pain, neckpain, jaw pain,pain in the hand, pain mimicking gastritis. If cholesterol levels are high please get evaluated totally for the cardiac problems.U can go for an ecg, echocardiography, sr creatinine, fasting cholesterol and sugar levels, tmt,Based on the tmt test u can go for angiogram.If the neck pain is still persistent pls go for an xray neck or mri neck (cervical spine).gooluck take care."
},
{
"id": 8939,
"tgt": "What cream should I apply for suntan on my face ?",
"src": "Patient: hi doctor , i am priya i am getting marriage soon,in us but i got suntan my face got black because of roaming in the sun is there any cream to help getting rid of suntan please doctor suggest me any us products please ,please help me Doctor: Hi !Priya, Welcome To HealthcareMagic forum, Priya ,it is very important to use a good spf medicated sunscreen daily every three hours. Along with this you can use a hydroquinone based cream at night whole face .Use this little cautiously so that it doesn't cause any burning sensation or redness. You can also consider using kojic acid creams with vitamin c cream in the morning over the sunscreen . But this would take a while to get the original color back. Hope this answers your query,if you have any further questions ,kindly write back to us. regards, Dr.Bharat chawda"
},
{
"id": 197238,
"tgt": "IS it possible to regain zinc energy lost due to excessive masturbating?",
"src": "Patient: I have a huge concern of my excessive masturbation and have STOPPED myself from doing is as of yesterday. I started masturbating at the age of 13, and I stopped at 22. I found out today that it causes me to lose zinc energy needed to help my immune system function, as well as sexual impotency and prostate cancer, and that's what urges me to reverse the issue. I also found myself going to the bathroom excessively today from masturbating. I did research on replacing my zinc and etheric energy by eating high zinc foods and the Roy's Exercise which I need the definition of. I also did a 5 month research on astral projection/out-of-body experience which I found out I won't be able to do until I regain my energy. I want to know, will I be able to regain my energy and reverse prostate? How will I do that? Your help is extremely important to me. Thank you for taking the time to read my concern. Doctor: HelloThanks for query .You have been indulged in masturbation since many years and now facing problems related to sexual activities and have multiple myths and doubts about your general health . First of all there is myth in a mind of common man that excessive masturbation is the reason of all sexual problems But I would like to state that it has been discussed in scientific forums all over the world and proved scientifically that masturbation does not have any negative effect on any organ or system in the body .All the problems that you are facing now are mind related and due to anxiety.Following measure will help you to boost up your confidence and getting good erection and delay ejaculation. 1) Practice regular exercise for 45 minutes followed by meditation for 1/2 an hour in the morning. 2) Take high protein diet rich in vegetables and fruits and Vitamin A,C,D,E.and Zinc 3)Take anti oxidants like Almonds 5-6 everyday 4) Avoid alcohol and smoking..Dr.Patil."
},
{
"id": 56762,
"tgt": "Suggest medication for jaundice",
"src": "Patient: Hi . I had my gallbladder removed due to gallstones. After one month of operation i got severe pain in my stomach . My blood tests showed jaundice at very high level . Doctors advised me for more tests like mri and various blood tests but everthing came normal. All the symptoms are of obstuctive jaundice but doctors are not able to find any obstruction . Now they are suggesting for hida scan . Its been one months since i have diagnosed with jaundice . My liver is working fine but my serum bilirubin is 23 mg% . Please tell me what should i do ? Doctor: Hello,Welcome to HCM.Please consult hepatologist for proper cause.Treatment of jaundice depends on cause.If hepatitis, it should be cleared.Gallstones obstruction is also a cause but it is not possible in your case.Alcohol and some drugs also cause jaundice.Avoid alcohol and drugs causing it.Do USG, X ray or liver biopsy to identify cause.Do liver functions tests once again.Hope this will help you.Thanks for contacting.Take care..."
},
{
"id": 210060,
"tgt": "Why having no interest in studies but very active in other activities?",
"src": "Patient: my cousin sister is showing nill performance in her studies. her age is 11. she shows quick response in other worldly things but when comes to studies she stays quiet. i want to know why she is doing like this? do she really need a psychiatrist. plz do reply me i need your suggestion.thank you, Doctor: Hello,Thanks for choosing health care magic for posting your query.I have gone through your question in detail and I can understand what you are going through.The fact that she is quick in all other activities except for the studies (reading/writing) suggests that she certainly needs in detail assessment of her reading and writing capabilities. She could be suffereing from a condition called dyslexia or learning disorder and this could be impairing her scholastic learning abilities. You need to take her to a psychiatrist who can evaluate in detail.Hope I am able to answer your concerns.If you have any further query, I would be glad to help you.In future if you wish to contact me directly, you can use the below mentioned link:bit.ly/dr-srikanth-reddy\u00a0\u00a0\u00a0\u00a0\u00a0\u00a0\u00a0\u00a0\u00a0\u00a0\u00a0\u00a0\u00a0\u00a0\u00a0\u00a0\u00a0\u00a0\u00a0\u00a0\u00a0\u00a0\u00a0\u00a0\u00a0\u00a0\u00a0\u00a0\u00a0\u00a0\u00a0\u00a0\u00a0\u00a0\u00a0\u00a0\u00a0\u00a0\u00a0\u00a0"
},
{
"id": 126697,
"tgt": "Is leg stiffness post an injury a concern?",
"src": "Patient: My husband slipped and fell hard on a wood floor landing on his but. He can walk slowly but that leg is stiff. He is using a walker to get around. No bruises at all. He is 87 years old in fairly good health. He is a computer geek and does not exercise. Can he have fractured a hip? Doctor: Hello, At his age chances of fracture are high due to osteoporosis and even minor injury may lead to fracture. I would like to advise X-ray of pelvis with both hips anteroposterior and frog leg views to rule out fracture. Please get it done from radiology centre nearby you. Meanwhile Advise strict bed rest and tab Ultracet twice a day can be taken for pain relief. Visit to orthopedic surgeon for detailed clinical Examination is recommended. Hope I have answered your query. Let me know if I can assist you further. Regards, Dr. Jayesh Vaza, Orthopaedic Surgeon"
},
{
"id": 31762,
"tgt": "Suggest treatment for infection in my navel",
"src": "Patient: I have a sac filled with blood and pus in my belly button. if i push down a little bit just about it - whitish-yellow pus spews out. it is very itchy and this morning I woke up with it full of dried blood. I dont see a cut or anything, but I dont know where the pus is coming out from. It started hurting 5 days ago and it started pusing 3 days ago. I amwondering what it is caused by and what to put on it. Doctor: Hi..Welcome to HEALTHCARE MAGIC..I have gone through your query and can understand your concern..As per your complain it seems to be bacterial infection in the navel leading to formation of a pus filled abscess..As the navel is the area where there can be accumulation of moisture, there can be bacterial accumulation and it leads to formation of abscess..I would suggest you to clean the area with antiseptic solution and apply triple antibiotic ointment and steroid ointment over it..You can take oral antibiotics like Augmentin for relief..You can apply mild pressure over the navel and try to drain off pus for faster relief from infection..In case if still there is no relief consult a General Physician or Surgeon and get evaluated so that a proper clinical evaluation can help in diagnosis and treatment can be planned accordingly..Hope this information helps..Thanks and regards.Dr.Honey Nandwani Arora.."
},
{
"id": 83400,
"tgt": "Can folic acid and histussin be taken together?",
"src": "Patient: taking folic acid andhistussin together does it cause any harm to the body and i want to know that my period came on the 5th of december last year and on the 31st of december of the same yearand ended on the 4th of january in 2012 can my period come on the 31st of this month Doctor: Hi,There is no documented any harmful interactions between these two medicines hence they may be taken together as prescribed. If your cycles are regular yes, your next period may come on 31st as you expect.Hope I have answered your query. Let me know if I can assist you further. Take care.Regards,Dr. Mohammed Taher Ali, General & Family Physician"
},
{
"id": 51292,
"tgt": "Bleeding in urine, abdominal, stomach, testicle pain. History of kidney stones. Treatment?",
"src": "Patient: i have blood still in urine .. been taking anitbiotics now for 14 days... pain in abdomin, stomach and testicals comes and goes and is very uncomfortable at times.. (i had a stone crushed in april, the stindt didnt pull out so i had a second surgery to have removed from back of kidney .) when i exercise and move around alot it bleeds bad, all parts of stone have passed, had a ultrasound two weeks ago of kidneys and there is nothing in there... sometimes when i urinate there are blood scab like pieces that pass.. Doctor: Hello, Welcome to Healthcaremagic. Do you bleed from your mouth or nose ? If you did a recent urine analysis, kindly attach its result. Regards Dr Ahmad"
},
{
"id": 30627,
"tgt": "Suggest treatment for flu with severe night sweating",
"src": "Patient: Got the flu, last thurs. PCP prescribed taking Tamiflu 75mg last thrus. fever = 103.8. fever broke by sat. usual symptoms except vomiting within a few hours of 1st dose of tamiflu (non since) and now 2 days after fever has gone away, sever night sweating. I mean pillow, sheets and comforter soaked. And the heat register in my room is closed. Is this a side effect of tamiflu? Doctor: Hi, dearI have gone through your question. I can understand your concern.You have flu. Night sweating, vomiting, fever all are common in flu. Don't worry. just continue your tamiflu. Take plenty of fluid orally. If you have fever then you can take paracetamol as and when required. Hope I have answered your question. If you have any doubts then feel free to ask me. I will be happy to answer.Thanks for using health care magic. Wish you a very good health."
},
{
"id": 214368,
"tgt": "Suggest natural alternatives for treating blockage in left ventricle",
"src": "Patient: I had a blockage in my left ventricle in October of 2012. I was given a bunch of medicine, that I have been taking ever since - Metoprolol 12.5 mg, Clopidogrel 75 mg., Atorvastatin 80 mg., and Lisinopril Tab 2.5 mg. I was taking Aspirin also, however, I have recently replaced that with Bilberry tablets & have felt the better results of switching. My question is this: I have gained 20 lbs., in this same time that is almost impossible to lose, my skin is sagging & leathery. I want to get off all of these chemicals, are there natural alternatives that I can use instead? Doctor: HelloYou are taking metoprolol ( atenolol ), lipid lowering atorvastatin, antihypertensive lisinopril and clopidogrel . You gained 20 lbs , that is a cause of concern.Avoid fried , fatty oily greasy foods .This measure may help you , this is not scientifically proved but I ahve tried on so many patients having Coronary artery disease ( blockage in arteries supplying blood to heart).Take CREEPER POMPION ( GHIYA in India ) , extract its juice in juicer , add 10 leaves of basil ( tulsi ) , 10 black pepper , add black salt . Total juice should be 300 ml , drink on empty stomach . For first 4-5 days you may develop loose motion ( diarrhea ) , after this it will adjust in body.From day one you feel difference . This is advised to take it for 6 months at least for the best result . Creeper pompion should be FRESH . Bilbeery tablet will effect or not I don't know .Further if any question regarding this , I will reply .Good luck."
},
{
"id": 172278,
"tgt": "Is there any problem for taking higher dosage of Refzil-o?",
"src": "Patient: Hello Doctor, My Son 6 and half year old. he his prescribed to give Refzil-o 250 mg, but I din't noticed that i got 125mg from the medical shop. So inspite of giving 5ml I gave him 8ml. I couldn't able to contact doctor. Can you give me a suggestion. Doctor: Hi...it you have got a medicine at half strength of what your doctor has prescribed then you have been right. You can double the dose of 125mg strength when the kid has been prescribed a 250mg strength basically by the pediatrician.Regards - Dr. Sumanth"
},
{
"id": 196959,
"tgt": "How to confirm the condition of bell clapper deformity of penis?",
"src": "Patient: i am a 17 year old boy and since a couple of months i feel that my testicles rotate 180 degrees. i feel that the epidermis has come on the front. is this possible . this goes back to normal in some time. there is no pain. i had a ultrasound done and it was all normal. but i feel i have a problem. is this some thing called the bell clapper deformity. will this cause a torision. i am very worried of ruining my life if that happens. what is wrong with me. please help. Doctor: HelloThanks for query Based on the facts that you have posted you have noticed your testicles rotate about 180 degree around itself (Bell Clapper Deformity)and have fear of getting Torsion of testis .Since Bell Clapper Deformity is easily diagnosed on ultrasound scanning, and your ultrasound Scanning being reported to be you need not worry about getting Torsion of the testis .More over it requires testicle to rotate around itself with 3-4 twist to get torsion of testis .Dr.Patil."
},
{
"id": 41774,
"tgt": "Suggest treatment for low sperm count",
"src": "Patient: Sir i m 26 yrs old male.i have been masterbating since 12 yrs.now i feel my semen has losen its quality & perfectness.after a little excitement it comes out & often in sleeping i lose my semen sir plz hlp me.giv me ans to my mail YYYY@YYYY . Doctor: Hi welcome to healthcaremagic.I have gone through your question.As you are 26 and masturbating since 12 years is absolutely no reason to worry.Is absolutely normal.Ejaculate early is called premature ejaculation.Is also not a medically abnormal.Lost semen at nighr sometimes it happens normally.So no need to worry regarding anything.Hope i answered your question.Would be happy to help you further.Take care."
},
{
"id": 63824,
"tgt": "Suggest remedy for itchy fluid filled lumps on scrotum and thigh",
"src": "Patient: Hi, may I answer your health queries right now ? Please type your query here...inner thighs both sides and scrotum have a lump full of fluid it itches, and is leakin the fluid, it started as little rash, used lamasil, but it is not working, it itches and is leakin slowly a clear odorles but greasy fluid Doctor: Hi, dear I have gone through your question. I can understand your concern. You may have multiple sebaceous cyst or some other lump. You should go for examination first. Then ypu should take treatment accordingly. Hope I have answered your question, if you have doubt then I will be happy to answer. Thanks for using health care magic. Wish you a very good health."
},
{
"id": 120641,
"tgt": "What causes tremors ?",
"src": "Patient: My 32 yr old son has bad tremors/ involuntary movements while drinking alchohol. I have heard of tremors the next day but never heard of anyone shaking while actually drinking. It is quite bad and lasts fot a few minutes. Why is this happening. He has been a heavy drinker for a number of years but not on a daily basis, also he has used different types of drugs. Doctor: Hello, What appears to be bad tremors or involuntary movements can be seizures. The cause could be anything from alcohol, illicit drug abuse to some underlying brain pathology, electrolyte imbalance to list a few. To make a definite diagnosis it is important to take detailed medical history along with the details of events that happened when the patient experienced tremor; like loss of consciousness, loss of bladder control, twitching of limbs etc. I would suggest the patient to see a general physician to get examined at the earliest; and take help from anti-addiction clinic/program available in your country. It is crucial to have family and/or friends to encourage, support, help a person fight his/her addiction. Hope I have answered your query. Let me know if I can assist you further. Take care Regards, Dr. Ayesha Shareef"
},
{
"id": 138775,
"tgt": "Suggest treatment for foot fracture",
"src": "Patient: my wife fell down the stairs a while back and went to hospital and ended up on crutches for 6weeks and found that she factored her foot but now he metertasus is sticking out of her left foot, bearing in mind she didn t have a cast or a boot to support her foot Doctor: Hello,I have studied your case and I think that she needs to see a doctor as soon as possible. If she did not receive any plaster or splint than I think there can be two possibilities.1.There is no union of the fracture and that is why metatarsal is coming out of the skin.2.Bone is united but not in proper position so you are seeing such protrubarance. I would recommend you to get x ray of the foot and see what is happening. Accordingly we can treat this condition. I hope this answer will be useful for you. Let me know if there is any other followup questions.thanks"
},
{
"id": 58064,
"tgt": "What is the meaning of liver ultrasound results that show fatty liver?",
"src": "Patient: Hi, My ultrasound result shows about my liver as; Normal size with sharp edge and smooth contour, Has brighter parenchymal echo, No evidence of focal liver lesion seen, and finally conclude as fatty liver. What does it mean? What should I do? what are the causes Doctor: Hello.Fatty liver is a chronic pathology.The pathology is an excessive accumulation of fat in the liver.Causes related to fatty liver are obesity, alcohol consumption and high cholesterol levels. People with fatty liver must control your weight, avoid alcohol, reduce the intake of fatty foods and practice physical exercise. If you follow the referred recommendations and your doctor controls your cholesterol, GOT and GPT levels in blood ,probably you will avoid the appearance of fatty liver complications.I wish you good health.(If the answer has helped you, please indicate this)"
},
{
"id": 32161,
"tgt": "Is it advisable to take rectal temperature?",
"src": "Patient: Hello.My question is in relation to rectal temperature taking. I compete at a very high level of sport. I work out and train regurally. I do not smoke, drink alcohal or stay up late at night.I stick to strict healthy eating diet also. As a result of this I am 100% healthy. Whenever am under the weather I take my own temperature rectally to insure the most accurate reading.I am a complete health fanatic so this procedure doesn't bother me in the slightest. The few times I visit my doctor when an illness usually cold or flu is taking hold of me my temperature is taken orally. I know rectal is more accurate and I feel more at ease knowing im been as accurate as possible. In my field of sport there is liltte time to be out ill.My question is , is this something that I could ask my doctor about, taking a rectal temperature? Or would I be looked at as if I had two heads? Doctor: Hi, Welcome to the HCM,I understand your concern, regarding body temperature measurements, there are many ways such as reactal, oral, axillary or tympanic membrane etc. The rectal measurements will be usually 0.5\u00b0F (0.3\u00b0C) to 1\u00b0F (0.6\u00b0C) higher than an oral temperature. Rectal temperatures are generally thought to be the most accurate for checking a young child's temperature not for the adults. A rectal temperature is generally considered to be the most accurate, and the standard for monitoring the core body temperature, but it is generally not recommended for use in patients, due to the risk of rectal perforation and tears with this method and also in relation with hygiene and sanitation rectal is not preferred in clinics for adults. I hope this answers your doubts. Have a nice day and take care."
},
{
"id": 165082,
"tgt": "Suggest remedies for fluctuating temperature in a baby",
"src": "Patient: im having alot of questions and i need an answer asap, i have 29 month baby and she recently had a fever for a week she was taking antibiotics and theirs still fever , she was takin amoxillin amoxicillin and clavulanic acid, and that produce diareah and also she had shots of Ceftriaxone and now she has low body temparature (35.6) and it goes up and down please in need a answer! Doctor: Hi, this is probably sepsis. Child needs admission in hospital. Blood culture needs to be done. CRP levels, CBC should be done. Examination is necessary here. Take care."
},
{
"id": 25627,
"tgt": "Suggest treatment for high BP and extreme stress",
"src": "Patient: I am under extreme stress and my bp is very high...210/110. I can't get it to come down even though my dr told me to triple my meds. It has been over 8 days since I visited the ER. The dr there told me to chil out. My fam practice dr said to triple the meds. I am away from home right now. Doctor: Hi thanks for asking question.Just now your blood pressure is very high.so as doctor advise you dose increased.And still needed intravenous sodium nitropruside can be given.First avoid stress. Meanwhile start regular exercise like jogging and running.you are away from home , so nearby Jim can be joined instead of running.Take very low salt diet.Drink more water .take green leafy vegetables.Take fruits more.After blood pressure comes down to below 180 / 100 , maintain it with regular exercise and diet restriction.I hope my suggestion will help you."
},
{
"id": 78977,
"tgt": "Suggest possible medication for cough",
"src": "Patient: Hello Doctor,Good morning!My daughter is 2 yrs4 months old. She is suffering cough last 4 days. I took her for check up. Doc has subscribed Asthalin syrup, Singmlair-4mg, Nasivion-mini as well as Omnacortil faste syrup.Are these medicine safe for my daughter?Thanking you.Mr. A. Bhuyan Doctor: The doctor you have consulted must have carried out a detailed examination before prescribing the same. So kindly trust your doctor and get back to him/her if you find any difficulties after using the same."
},
{
"id": 113615,
"tgt": "Sharp upper back pain, near shoulder blades, pain on coughing, taking deep breaths, sneezing. Treatment?",
"src": "Patient: A couple days ago my upper back started hurting. It felt almost like a back spasm . We have been on vacation so I attributed it to bad matresses. We have since returned but the pain has worsened and is primarily on the right side around my lower shoulder blade. Pain is sharp when I cough , take deep breathes, and sneezing brings me to my knees. Any ideas? I have a chiropactor appointment today. Doctor: Good morning. I will be very glad if i could have know ur age /sex. What type of built. But still can judge that ur upper back pain is due to severe spasm of ur back muscles and respiratory muscles. I advice u to lie flat on flat surface and take analgesic and muscle relaxant. Need not worry it gets relieved soon"
},
{
"id": 12214,
"tgt": "Is melasma genetic ?",
"src": "Patient: hi. i am 27 year old and deliver a baby boy (4 MONTH) . I have malasma on my face this is present before pregnency also and even these were started to flareup from when i was 24 year old. I have consult to dermatolosist, he advise me (Hydroquenone and suncross sunscream) but due to hydroq. i feel redness & more flare then after i used Kojic acid since 2 year i feel some better and i havedone peeling by glycolic acid(35%) after every 2 month these were all befor pregnancy but now what can i do & plz tell me becoz my elder sis ,elder brother and younger brither also have same pattern of pigmentation and started in almost 24-25 year. i cant face sun exposer even for 5 minut cholasma become prominant. lz give me suggestion and permanent treatment. thanks Delhi Doctor: \u2018Welcome to HealthcareMagic Forum\u2019 Moth spots Or chloasma is common after pregnacy. it can be treated effectively with homeopathy. Sepia 30 6pills thrice will give you benefit with in a month wish you good health!"
},
{
"id": 155551,
"tgt": "Is chemo advisable for cancerous mass around the Mesenteric artery and veins?",
"src": "Patient: My husband had pancreatic cancer over 5 years ago. Two years later he had adhesions surgery. Tests and Scans because his legs are swollen have shown a cancerous mass around the Mesenteric artery and veins. Is chemo advisable? Thank you for any information that would be be helpful for us to make a wise decision Doctor: Thanks for your question on HCM. If tumour mass encasing vessels than chemotherapy is the treatment of choice. Surgical resection is contraindicated due to encasement. So chemotherapy and radiotherapy are the only options available. Targated radiotherapy should be tried to decrease the tumour bulk. Systemic chemotherapy will useful in prevention of distal metastases and also reduce tumour size.So chemotherapy should be given in this case."
},
{
"id": 34386,
"tgt": "Suggest treatment for infected sebaceous cyst on lower back",
"src": "Patient: Hi I am 17 weeks pregnant and last week was diagnosed with an infected sebaceous cyst on my lower back. I was given an antibiotic to clear the infection but this does not seem to have worked as the area is still tender but has now turned purple. What will be the next stage of treatment ? Doctor: Hello mem,Good evening, First of all congratulations for your pregnancy...Now coming to your question about sebaceous cyst infected sebaceous cyst...Generally sebaceous cyst is quit common n if it is there n yreatment of choice is incision n drainage...Now you have infected sebaceous cyst then treatment of choice first control your infection n then incision n drainage...So don't worry mem you are on right path of treatment...If you have any problem then just raise question to your surgon he/she ll give you ans after examining that part n cyst perfactly as without examination I can not give any comment for surgery...Thank you,Regards,Dr.Ravi Kotecha"
},
{
"id": 92035,
"tgt": "Why do I have flutters in lower left side of stomach after exercising on planks ?",
"src": "Patient: I had a tummy tuck about 11 months ago. I work out 5 days a week. I started doing a new excercise which is planks. its been 3 days i have been experiencing flutters in my lower left side of stomach. It's annoying and freaking me out. Not pregnant had a tubligation in 1992. what is happening to me Doctor: Hi. Flutters can be due to irritation of the entrapped nerves, or sort of a over-use in the form of exercises. Give rest to the part- stop work-outs for the time being. Wait and watch, If the problems persist even after 3 days visit a Doctor."
},
{
"id": 69314,
"tgt": "Need advice on the large bump behind the head with radiating pain to back",
"src": "Patient: I have a large bump on the back of my head, my family doctor thinks it just a swollen lymph node, although it is very painful, espically after I have smoke pot, although I when I put to and to together i stoped, while normally if i hadnt smoked in a fewdays or a week, it would bairly hurt, and the swelling would almost be gone but now it feels swollen more than ever and hurts very badly with a shotting pain down my back raiding from the lymph node, and I have a headach that hurts very badly sitting right behind my forhead. Doctor: Hi.There are no lymph nodes in the situation/ area you explained. This is either a lipoma or a sebaceous cyst. It is better to get this removed (excised) for 2 reasons.1> your get rid of the disease and 2> you get the histopathological diagnosis- makes you tension-free and symptoms free. It is possible that the nerves supplying the upper and lower area are going through the nearby area so you are getting a typical referred pain ."
},
{
"id": 164809,
"tgt": "Suggest treatment for pain in the hands and arms",
"src": "Patient: My 2 yo grandson, who will be 3 on June 13th, is experiencing both right and left arm and hand pain. Last month he was experiencing leg pain. Both times low grade fever is present. Last month my daughter, his mother, took him to the pediatrician but no diagnosis was made other than there was a thought he might have the flu. Now he is experiencing the arm and hand pain and I feel something is not right. What is your thought doctor? Doctor: Dear Gran,Your concern seems right. Even though it did not turn out to be any thing last time, but I feel you shall take him to a pediatrician. Also pay attention if there are other symptoms as well. Keep a watch."
},
{
"id": 170836,
"tgt": "Suggest treatment for a low grade running fever",
"src": "Patient: My daughter has been running a low grade fever (between 101-103) for three days (since Tuesday afternoon). This morning, she has a temperature of 98.8. However, she woke up with a very sort jaw line only on the left side. Any chance these things are related. The only other symptom she has had this week is vomiting (for the first 24 hours). Last week, on Monday, I took her to the doctor and she was diagnosed with strep throat and put on Amoxicillan. Her 10th day would have been WEdnesday, but we stopped on Tuesday night since she was vomiting. She doesn t have any kind of rash on her body anywhere so I don t think it s Scarlet Fever. What do you think could be causing her jaw pain? I don t feel any swollen glands under her chin/neck area. Doctor: Hi.... the jaw pain your daughter is experiencing could be referred pain from the inflamed and enlarged tonsils or congested pharynx. I suggest you continue the antibiotic therapy for a total of 10 days which is the appropriate duration of treatment for streptococcal sore throat. The jaw pain can be managed with appropriate dose of Paracetamol, but if it is so severe as to restrict the movements of the jaw then it requires evaluation by physician.Regards - Dr. Sumanth"
},
{
"id": 134243,
"tgt": "What is causing the sharp pain in my hip?",
"src": "Patient: I was working out and I fell backwards onto the ground from an elevated area (5ft fall) . There was a metal pole that my left lower back fell on and I hit my head, which caused a medical concussion that a doctor verified. Now, when I move my left hip or even slightly switch positions I have an extremely sharp pain in the inside of my hip. Could this be bruise or something else. Also, where my left lower back hit the metal pole a laceration formed and also tissue damage caused a lump in my back Doctor: hi,thanks for providing the brief history of you.as you mentioned you had contusion and have been treated for the same. Also, have you undergone any MRI or CT of your lower back since the injury was directly on to the region of the pole. also have there been any x-ray take for the hip region.usually the nature of the mentioning may be due to the nerve getting pinch either by the gluetus muscle or may be at the spinal level. For the confirmation of it an MRI is advise. post that , a physical therapy is advised and you will be put on a mild to moderate exrrcises which will help you strengthen the muscles and within a short time . All you need to focus is to strengthen the muscles.with the grace of God I Wish you a good health.regards"
},
{
"id": 4931,
"tgt": "Forgot to take pills after sex during fertile period. However, took pills next time. Chances of pregnancy?",
"src": "Patient: Hello. I'm 28 years old. I'm taking althea pills for almost 2 years now. I would like to ask if there is a tendency if I am pregnant. My last period was August 09,2013. And by August 14, I should be starting my another box of pills, yet I decided not to, since i wanted to take a rest of taking pills just for a month or two. However, last August 24, we made love. I know I am fertile that time (and I am not taking pills already), before we make love that day I am having this white discharge. And right after that day, the discharge was gone. Since I am not ready to be pregnant, the day after we made love I decided to take pills and until now I am taking it waiting and praying for my menstruation to happen. (Hoping it will help to discontinue the conception just in case...) With that situation, Is there a tendency that I am pregnant for 1 week now???? Doctor: Hi,Thank you for choosing Healthcaremagic. After analyzing your query as you have stopped your pills during fertile period, you could become pregnant, it would have been much useful if you had taken emergency pill rather than continuing the pills. Presently you wait for your periods, if you miss your periods, then you need to get a urine pregnancy test done and rule out pregnancy.I hope this information has been both informative and helpful for you. In case of any doubt, I will be available for follow-ups. If you like my answers kindly rate it, and write a review as well. Please do not forget to accept it.Thank you,Wish you good health.Regards,Dr ArifYou can consult me again directly through my profilehttp://www.healthcaremagic.com/doctors/dr-arif-n-khan/65133"
},
{
"id": 4,
"tgt": "How can one conceive successfully?",
"src": "Patient: helo i am 25 years old. i was married last year(May 2010)after 5 months i concieve pregnancy but unfortunately i cant countinue my pregnancy.light spotting was started and in the begining of 3 month i face DNC but now i cant concieve plz help me and guide me? my doctor has to given me fertab tablets but now i m feeling nausea? Doctor: Hi dear!I have read your question carefully and understand your health concern.If your cycle is regular ,you should try to conceive at least one year and than if during this time you will not get pregnant you should contact with your gynecologist.During the time you try to conceive you should :be calmeat a healthy balanced dietexercise dailyHope I have been helpful .Regards"
},
{
"id": 216102,
"tgt": "Will I be tested positive for mouth swab drug test if taking Norco for pain management?",
"src": "Patient: HI. I AM A PAIN MANAGEMENT PATIENT AND CURRENTLY AM PRESCRIBED THE MEDICATION, NORCO, BY MY PHYSICIAN. I HAVE A JOB INTERVIEW TOMORROW AND THEY WILL DO A MOUTH SWAB DRUG TEST ON ME. MY QUESTION IS WILL MY MEDICATION CAUSE THE TEST TO BE POSITIVE AND IF SO WILL THE TEST BE ABLE TO TELL WHAT KIND OF DRUG THAT IS IN MY SYSTEM (NORCO) OR WILL IT JUST READ POSITIVE? Doctor: Hello and Welcome to \u2018Ask A Doctor\u2019 service.I have reviewed your query and here is my advice.Yes, it is likely it will be positive.The mouth swab test usually is done right in my experience and is not just an immune test to see what reacts with antibodies to heroin (like a cheap urine dipstick) but the more sensitive and accurate mass spectroscopy test. This tells the specific drug.Hope I have answered your query. Let me know if I can assist you further."
},
{
"id": 216173,
"tgt": "Suggest treatment for pain in right side of waist",
"src": "Patient: I am suffering from waist pain which goes down the leg particularly on right leg. I wake up each night from sleep because of pain on the right side of waist & leg when I sleep by turning on right side.It is difficult to walk because of pain. I cannot tolerate Gabapentine because of my hyperacidity & swelling of feet. I am on pacemaker. Please advise. B Basak Kolkata, India Doctor: Dear patient your symptoms indicate diagnosis of nerve compression at the level of lumbar spine with radicular pain in the right lower limb. Reason may be disc prolapse. Diagnosis can be confirmed by doing MRI of the lumbo sacral spine with screening of whole spine. Please get it done from radiology centre nearby you. Gabapentin is drug effective in radicular pain. if you are getting hyperacidity please take tab pantocid 40 mg twice a day before meals along with gabapentin. You need to consult spine surgeon with report."
},
{
"id": 156491,
"tgt": "What causes fatigue and chest pain after having chemo treatment?",
"src": "Patient: My husband had 2 MI's in 2005, 4 stents placed, in 2009, he was diagnosed with head & neck cancer, 3 cycles of chemo and 35 sessions of raditation, in Nov, of that yr, he had bilateral neck dissection.. it was touch & go but we made it thru it. Now for the last couple days he has bee very fatigued and some pains in his chest... do I need to seek medical attention> Doctor: He definitely needs medical attention as he has a very strong history of ischemic heart disease with 4 stents in place. Chest pain and fatigue should not be taken lightly. Please consult your doctor."
},
{
"id": 208827,
"tgt": "How to treat short term memory loss?",
"src": "Patient: help please. im a 23 year old with short term memory loss. not like I forget where I left my keyz.. more like i cant remember things me and my boyfriend do or the last movie I watched and I just watched it a few days before. any idea what it could be? Doctor: DearWe understand your concerns. I went through your details.I suggest you not to worry much. The symptom you are mentioning does not point to short term memory loss. It could just be selective forgeting. Your brain hs the capacity to forget something which you do not want to remember. The same thing happens in the case of undue anxiety and restlessness. The case of forgetting keys is one example and the other one what you are experiencing now. Therefore, unnecessary worries may trigger selective forgetting.You can post a direct query to me in case you further need my services in this case. Hope this answers your query. Available for further clarifications.Good luck."
},
{
"id": 131930,
"tgt": "What causes pain and weakness in calves?",
"src": "Patient: thank you. i recently arrived home from a long road trip. the calves of both legs are now sore and weak, but no swelling. i have had problems in the past with my calves getting cold and painful while doing my walks but i was using lipitor at that time. this is the first time i ve had this problem after a road trip.any advise that you may offer will be appreciated. Doctor: Hello,There is something called intermittent claudication that may be what you have. It causes calf pain after exercise. It is caused by blockages of the blood flow to the legs. There is a simple test your doctor can order called an ankle brachial index to screen for this condition. See your doctor soon.Dr. Robinson"
},
{
"id": 94633,
"tgt": "Right sided abdominal pain, diagnosed as abdominal strain. Treatment options?",
"src": "Patient: I have pain in my lower right side abdomen just above my pubic hair . It wakes me up at night when I roll over to my left side. When I lay on my right side it is fine. When I get up in the morning my first few steps are very painful and then throughtout the day the pain goes away unless I move very quickly to the left with my leg. I have been to the doctor any he said it was an abdominal strain but it seems to be getting worse. I work out 5 days a week and have stopped for 6 days but it does not seem to be getting any better. Doctor: Dear ktirney, when doctors find nothing then they says its abdonial strain. It can be but if you really suffer cause of this pain then some serious disorders should be ruled out. You could have inguinal or ventral hernia, do you feel some mass when you cough on this place? if its intensity changes due to position than it can mean that gas in the colon and position of intestines trigger the pain. In that case you can have an intraadbominal lesion or infection. Also you can have some kind of superficial neuropathy which can cause such intermitent pain. So if pain progresses I advice you to do some of imaging tests and pelvic/hip rtg. And neurological examination... Wish you good health, Dr. Ivan Romic (Rominho)"
},
{
"id": 223088,
"tgt": "What causes hypertension and pressure, cramping, vomiting and pain during a pregnancy?",
"src": "Patient: I am 15 and 38 weeks pregnant...I m asking on here bc my doctor is a horrible one, I have been having a lot of pressure, cramps and pain for the past few days... this morning I started vomitting and can t keep anything down I. Also have pregnancy induced hypertension what could be wrong w me Doctor: Hi dear, I have gone through your question and understand your concerns. Cramps and pressure in the abdomen are the signs that the process of labor is starting.Vomiting can be due to labor or gastroenteritis.I will suggest you to consult an OBGYN specialist as soon as possible so that you can be properly examined,and treated accordingly.Hope you found the answer helpful. RegardsDr Deepti Verma"
},
{
"id": 47688,
"tgt": "Does my father s prostate and kidney condition need an operation?",
"src": "Patient: my father had his ultrasound this morning.based from it his prostate gland is enlarged 6.68x6.94x4.44cm volume is 107.7cm. impression indicated the following: infected cyst vs nodule,left kidney, cystitis and prostatomegaly grade 5 with concretions and suspicicous nodules. may i ask if he needs an operation? Doctor: HelloThanks for query .Ultrasound scanning of your father has revealed 1) Gross Prostatomegaly and suspicious nodule in prostate2) Infected cyst or nodule in kidney .Considering the findings in report He has prostatomegaly with nodule which could be a cancerous nodule 2) The cyst in the kidney (Infected ) In a given situation he has to undergo few more tests to establish diagnosis .Please consult qualified Urologist for clinical and digital rectal examination and get following basic tests done to confirm the diagnosis.1) Urine routine and culture.2) Ultrasound scanning of abdomen and pelvis3) Serum PSA.4) Ct Scan of abdomen and pelvis .Further treatment will depend upon result of these tests and final diagnosis.Looking at the large sized prostate he will need to undergo surgery /Dr.Patil."
},
{
"id": 37964,
"tgt": "Suggest treatment for shingles",
"src": "Patient: I have a rash which I have had for 2 years now first they said it was shingles, then I was hospitalized and they did some biospy and said I have klebsiella and I had a pic line and Invance tx for 2 weeks. This rash is relentless and itches to high heavens. I am taking Atarax which does nothing for it and I have such pain I want to die I am serious. What can I do because the quacks here want to put me under uv light for 50 bucks a pop and that seems like robbery to me? Help.... Doctor: Hello, Thank you for your contact to health care magic. I understand your concern. If I am your doctor I suggest you that you should do culture and sensitivity report from the lesion in renowned laboratory in your area. Take the report and than go to doctor, take treatment accordingly. If it is Klebsiella pneumonia it will be treated without need of UV light.I will be happy to answer your further concernYou can contact me. Dr Arun Tank. Infectious disease specialist. Thank you."
},
{
"id": 73478,
"tgt": "What causes dull chest pain while exercising?",
"src": "Patient: Hi- I had done Mibi test recently and Rest tomographic slice show a small to moderate sized defeat involving the inferolateral Wall with normal wall motion. However, the result shows that Mibi shows the inferolateral rest defect was mostly corrected by with the CT attenuation correction. I have sometime dull chest pain when I excercise. CAD test should blockage less than 50%. I am 44 yrs old with family history of heart problem. I do NOT have high blood pressure. Here are the questions. 1.\u00a0\u00a0\u00a0\u00a0\u00a0SHOULD QUITE EXERCISE or CONTINUE? Or should change the exercise intensity \u2013 walk instead of run? How long? Etc. 2.\u00a0\u00a0\u00a0\u00a0\u00a0Should go for the Catherization and coronary Angiograms? Concerned due to invasive nature of it. 3.\u00a0\u00a0\u00a0\u00a0\u00a0 Should repeat MIBI \u2013 Mibi shows the inferolateral rest defect was mostly corrected by with the CT attenuation correction. 4.\u00a0\u00a0\u00a0\u00a0\u00a0Should repeat the CAD test? Doctor: Thanks for your question on Healthcare Magic.I can understand your concern.No need to worry much for inferolateral defect on MiBi scan.Continue with running and other cardio workouts.No need for invasive angiography.Get done regularly, CAD and MiBi scans to know if there is any worsening of defect.Hope I have solved your query. I will be happy to help you further. Wish you good health. Thanks."
},
{
"id": 117937,
"tgt": "How to detect HCV?",
"src": "Patient: My husband and I have applied for new health insurance. After nornal lab results from a life insurance blood chemistry the week prior, the health insurance company states there results indicated a positive HCV result. His AST is 36U/L, his ALT 30U/L. How could they make this claim? What other result would indicate HCV? My husband is age 59, 175 pounds and in excellent health. Doctor: Your liver function test is normal. It must be chronic condition. You dont have any problems then its ok.no needed to take any treatment. If you want to claim then go for elisa test it will give perfect results. And on basis of that you can claim."
},
{
"id": 93037,
"tgt": "Abdominal cramping, burning sensation, heartburn, heavy breathing. Why the pain?",
"src": "Patient: I have some abdominal cramping and my stomach is burning this has been going on off and on for about 2 maybe three weeks. At first i thought it might be heart burn its in my stomach area, and the cramping is off and on as well . It feels like a menstrual cramp but worse this comes and goes and im just not sure what to do. I would just like some kind of input on what this might be. When this does happen I need to sit down breath heavy and then the pain goes away it seems as though its worse at night i dont know. Doctor: Hello Good Evening,You might be suffered from gall stone, which cause similar symptoms as you described.Symptoms commonly begin to appear once the stones reach a certain size (>8 mm).A characteristic symptom of gallstones is a \"gallstone attack\", in which a person may experience intense pain in the upper-right side of the abdomen, often accompanied by nausea and vomiting, that steadily increases for approximately 30 minutes to several hours. These symptoms may resemble those of a \"kidney stone attack\". Often, attacks occur after a particularly fatty meal and almost always happen at night, and after drink.But you should also go for Gastric ulcer/duodenal ulcers and cardiac evaluation.Please get evaluated by following investigation:-Blood tests-X-ray Chest PA-X-ray FPA-UGI Endoscopy-USG Whole AbdomenOnce we reach the final diagnosis, management will be easy.Regards,Dr. Naresh Jain."
},
{
"id": 141627,
"tgt": "What does MRI showing brain positioned low mean?",
"src": "Patient: Hi! Yes. What are the possible meanings for an MRI finding to show that the brain is positioned low. That was all the oncologist said ( breast cancer survivor and I had been experiencing dizziness since chemo) and said to go to a neurologist after the MRI showed that the brain was positioned low. Any thoughts?? Doctor: Hello and Welcome to \u2018Ask A Doctor\u2019 service. I have reviewed your query and here is my advice. Of course it would be useful to have the report, but my guess would be that it refers to a malformation in the brain called the Chiari malformation, in which the brain stem or cerebellum are placed lower than normally, in the foramen magnum which is the canal connecting the skull to the spine. It is a very common malformation. In many cases it causes no symptoms and one can lead a normal life. In some cases though when the displacement is too severe there can be headaches or neurological deficits from brain stem compression. As I said without the report one can't say how many millimeters the displacement is. If it's not marked and the neurologist doesn't find any neurological deficit than it is a coincidental finding and nothing needs to be done about it. Hope I have answered your query. Let me know if I can assist you further."
},
{
"id": 9312,
"tgt": "Suggest treatment for dry and discolored patches around pubic area",
"src": "Patient: Hello there, For the last few months, I have been noticing some discolored patches around my bikini and pubic area. The multiple patches are only a little darker and browner than my normal skin, and every once in awhile they get very dry but not itchy. I am sexually active but have always used protection. I use lotion to fix the dryness but the coloration and patches remain. Advice please? Doctor: Hello, I can understand your concern for dry discolored patches around your bikini and pubic area.After analyzing your query I can make out that you have \u201cFungal infection\u201d of the area causing discolored patches around my bikini and pubic area. These are common type of infections due to fungus which more commonly affects the areas which are more prone for sweating or moist areas.These can also be related to the hygiene and environmental factors along with hormonal problem such as diabetes.I suggest my patients to apply a cream containing anti-fungal creams such as clotrimazole for two times. I also suggest my patients to take antihistamine like Levo-cetrizine.I suggest you to prevent conditions of moistness or humidity to prevent fungal growth.Hope the information helps."
},
{
"id": 66091,
"tgt": "Suggest treatment for lump in neck that pains",
"src": "Patient: I have a small marble sized lump to the left of the center of the back of my neck right below my hair line. Its on the left side, and when i push on it, i feel a sharp pain on the top right side of my head. I noticed it about a week ago. what could it be?? Doctor: Hi! this is a common complaint in our general OPD and we come to a conclusive diagnosis only after a needle biopsy!Anyways, we find different possibilities as follows:1. a neurofibroma, considering sharp pain on the top right side my head; 40% cases2. an enlarged lymph node due to infection in the scalp, ear, throat, or dandruff etc. 30% case3. a sebaceous cyst 25% case4. other benign tumors like lipoma 5% casesTherefore, the possibility is non-serious or benign but needs confirmation to relieve your tension; you need to see a physician for an examination and relevant management!regards,"
},
{
"id": 99874,
"tgt": "Is coughing/getting congested from laughing related to asthma?",
"src": "Patient: im wondering if i have exercise induced asthma...i was playing bball with a friend a while ago and after maybe 5/10 minutes (i was playing hard) he made me stop because the way i was breathing scared him. He saidi was breathing really deep/fast andhe thought i was going to pass out. I didnt notice because im usually like that when i exercise (bball,skateboarding,biking). depending on how long/hard i exercise ill get a tight chest so its kinda harder to breathe (sometimes if i just push myself harder my chest will burn) and end up feeling congested and ill be coughing a lot. I always thought it was normal, but my friend asked if i had asthma...and so i looked stuff up and found out about EIA. Im wondering if i should go to a doctor, cus i have some people telling me im just out of shape...but i dunno cus i rememeber coughing a lot during exercise for the last two years or so. Its a deep harsh cough from the chest. One person told me i might have just normal asthma. I never had allergies before but in the last year or so i have noticed that especially in the summer ill have what feels like sinus problems...just a feeling of pressure around my nose/eyes and a headache. I never used to get headaches much before either. I dunno if im imagining things now cus im wondering wat my symptoms are, but it feels like im either getting sick (cus my sister had a cold recently) or something else..cus my chest feels a little tight, ive got that sort of sinus pressure feeling and a little bit of a headache...Does coughing/getting congested from laughing have anything to do with asthma? cus i dont even have to laugh hard to get that. In a conversation i can laugh several times off and on and then start coughing (with phlegm) Doctor: Hello,Thank you for asking at HCM.I went through your history and would like to knoe more about you like your age, nose symptoms, weight, height, family history, etc etc.However, from your provided details, at present I would like to make suggestions for you as follows:1. Yes, bouts of coughing and breathing difficulty/fast breathing/wheezing (whistle like sound from chest) while exercise and laughing suggest asthma.Asthma is a very broad diagnosis, it can be due to allergies or other non-allergic reasons to. Sinusitis (a sinus disorder) and obesity also worsen asthma symptoms.2. As you have above symptoms and also sinus problems, I would suggest you to consult a doctor who may suggest you investigations for diagnosis of asthma and allergies also.3. I usually suggest my such patients spirometry with post-bronchodilator reversibility testing and post-exercise testing to diagnose/rule out asthma. 4. Sometimes exercise related breathing issues can also be non-asthmatic, so it is important that you see a doctor and get diagnosed by him/her.5. If you are obese, weight reduction will also improve your symptoms to some extent.6. Control of sinus symptoms with antihistamines (cetirizine or levocetirizine or fexofenadine etc) and regular steam inhalation will also gradually improve you symptoms.7. Regular physical exercise including breathing exercise and healthy diet rich in vitamins & minerals will gradually improve your lung capacity and immunity respectively in a long run.8. Please avoid exposure to dusts, smokes and air pollution as much as possible.9. For breathing difficulty/wheezing/asthma symptoms, your doctor suggest you to use salbutamol inhaler on as-and-when-needed basis or inhaled corticosteroids on regular basis depending upon your spirometry report and detailed examination.Hope above suggestions will be helpful to you.Should you have any further query, please feel free to ask at HCM.Wish you the best of the health ahead.Thank you & Regards."
},
{
"id": 84985,
"tgt": "What are loprin, amlopine and arpitor side effects for long term usage?",
"src": "Patient: i am using 47 years old woman and has been prescribe to take Arpitor 10MG, Loprin 75 and Amlopine 5MG. I have been taking these drugs from last five years.But I do not know the good effects of these drugs and how long i need to continue taking this drugs.please suggest me any precautions that i need to take care. Doctor: Hello,these drugs prevent cardiovascular disorders. Arpitor will lower your cholesterol, loprin prevents clotting and amlopine reduces the blood pressure. Although you haven't mentioned much, it seems that you have arterial hypertension and hyperlipidemia and perhaps also a cardiovascular disorder or you are at high risk for cardiovascular disorders. These drugs can rarely be stopped completely unless the initial indications were not very strong. Sometimes a dosage reduction can be tried but you already take the lowest possible doses. As I've already pointed out, these drugs reduce the risk of cardiovascular disorders. Regarding their potential side effects or precautions you need to take: they may cause a bunch of side effects but they're too many to list here. I'll mention some of the most common ones though... The antihypertensive medications may cause hypotension or postural hypotension, so you should always check your blood pressure when you feel weak, dizzy or feel you're going to faint. The antiplatelet drug (loprin) may increase the risk for bleeding, so you should avoid injuries particulary of the head and you should always check your stool for signs of gastrointestinal bleeding (black stool). Finally the lipid-lowering drug may increase your blood sugar, may cause muscle aches and liver toxicity. Blood tests will reveal most of these disorders. I hope I've answered your question. Please let me know if you need further assistance.Kind Regards,Dr Panagiotis Zografakis,Internal Medicine Specialist"
},
{
"id": 176336,
"tgt": "Suggest treatment for soft lump on back of head in an infant",
"src": "Patient: Hi my ten montb daughter was un settled and layin next to me in my bed, she fell asleep soo i just went to turn off the light when she rolled and she fell offthe bed but hit the corner or my bedside table she cries for a long time but fell back asleep after cuddles, i notice she has a soft lump at the back off her her head and now im worried she is fast asleep but if i try to move her she does respond do i need to take her to the emergency room or see how she is inthe mornjng Doctor: HI...nothing to worry about. The soft lump you are noticing is a boggy swelling due to localised oedema in the hit area. I will suggest you danger signs of head injury -1. Vomiting continuously2. Seizures3. Watery of bloody discharge from ears and nose4. Unconsciousness5. Altered sensorium or behaviour. If none of them are present, I don't think you should worry about his trivial head injury. For pain you can give oral Paracetamol in the dose of 15mg/kg/dose(max ceiling dose 500mg) every 4-6th hourly. Regards - Dr. Sumanth"
},
{
"id": 123031,
"tgt": "What is causing numbness and pain feels like to the bone?",
"src": "Patient: I have a deep ache kinda pain almost feels like to the bone,also on this left same side is my foot feeling numbness,but I check my sugars and it always at about 98 even after 2 hours after a meal or first thing in the morning,I have arthritus in the knee on the same side, Doctor: Hello, Pain along with numbness is a sign of nerve involvement. Firstly I will advise for the uric acid check. If that turns normal than we need to look out for the nerve involvement. Usually, when there is arthritis of the knee there will be a change in the gait and that may lead to change in the muscle physiology in the lumbar spine leading to spasm and deviation of the knee and can lead to nerve impingement. I will advise to try this out if it helps you. Hot water fermentation for the lumbar spine, core stability exercise, hip muscle strengthening exercises, static hamstring, static Quadriceps, straight leg raise and ankle toe movements. Wait and see if this exercise prescription is helping you for reducing the symptoms. Hope I have answered your query. Let me know if I can assist you further. Regards, Jay Indravadan Patel, Physical Therapist or Physiotherapist"
},
{
"id": 87305,
"tgt": "What causes abdominal pain and delay in menstruation?",
"src": "Patient: Hello I am 23 years old and have been on Nuvaring for 6 months now- My current period was 4-5 days late and now that it has came I am experiencing extremely bad abdominal pain from below my sternum to my pelvis. I am also very nauseous and have thrown up a few times, and have had diarrhea. 1 or 2 times there has been what looks to be a small amount of blood in my stool. I would appreciate any help as I am hurting quite badly.. Doctor: HIGreetings from Dr.Divakara.PThanks for posting your query. Your symptoms looks like Intestinal Infection . It can happen due to bacterial or viral infection. But after going through your query I feel it is more of a bacterial infection than Viral . Still I would recommend a Stool Microscopy test which will help us to know what kind of infection you are having . Also kindly get an Ultrasound abdomen pelvis scan done. Also till you get your reports , contact a doctor and get an Antibiotic prescription and Anti spasmodic drug prescription. These medicines will relieve your symptoms. Hope this information is useful to you. Any clarification feel free to ask."
},
{
"id": 103347,
"tgt": "Have hive like itchy red bumps on skin after going outdoor. Is it sun or grass allergy?",
"src": "Patient: Hey I am a 15 year old boy, who loves to be outside. I recently have been going to volleyball camps 10-2 and each week I return the exposed skin develops hive- like ichy, red bumps. mainly concentrated on my knee, bottom of my forearm, after about 8 hours after I get home. Is this some sort of grass or sun allergy? Also this is just a recently occurring issue, I did cross country running in the fall and nothing like this happened. I cover myself with spf 50 sunscreen before I went to play. Can I get a diagnosis, also how can I continue to attend outdoor activities while avoiding this issue? A reply would be much appreciated. Thank you,William Doctor: might be grass or pollen allergy as sun triggers allergies and also exercise triggers it sunscren can prevent you can take antiallergic tab regularly and apply calamine bd till your camps and can find out cause by consulting allergy specialists"
},
{
"id": 196555,
"tgt": "How often can one masturbate?",
"src": "Patient: I have read on many websites that masturbation can change the sensitivity of a mans penis. I also read that stopping masturbation for awhile can bring back sensitivity making it easier for a man to ejaculate during intercourse is this true? If so can i still masturbate? Which would probably be healthy every other day,once a week,,maybe three times a month? Doctor: Welcome to healthcaremagic.com and myself Dr. Atishay Bukharia would be answering your questions.. various studies have suggested that twice weekly is ideal frequency for masturbation. masturbation has nothing to do with increase or decrease sensitivity. Hope it helpsIf you have any other question please do not hesitate to contact usRegards."
},
{
"id": 106526,
"tgt": "How can pain in the upper back be treated?",
"src": "Patient: for two days now, i have dull aching pain in upper left back (shoulder area)hurts to turn head to the left,can not raise my left arm more than aprox. 8 inches away from my body without dull pain all the way down to my mid forearm. i have taken 2 tylenol every 4 hours without any relief. could you help me ? Doctor: Hi, You may be suffering from neck muscle spasm. Neck movements with pain to left indicates this diagnosis. Treatment is hot water fomentation and local application of Dynapar gel. Start Neorelax MR twice a day with tablet Ultracet once at night. Use soft pillow of low height. Avoid jerky movements of neck. You will be fine within few days. Hope I have answered your query. Let me know if I can assist you further. Regards, Dr. Jayesh Vaza"
},
{
"id": 152887,
"tgt": "Noticed the symptoms of lyme disease after a complete hysterectomy",
"src": "Patient: I am 55 years old & it has been 3 years since my complete hysterectomy due to stage 1 uterine cancer. I was told my sex life would eventually be normal again (in the sense of getting pleasure like I used to before the surgery ...specifically orgasms). Unfortunately, in the 3 years after surgery I have had only one slight orgasm (nothing like it used to be). I am also still so tired during the day & am told to try harder.....is it all psychological (depression or still repercussions from the lyme disease treatment that ceased last October) or do othersshare my problem too. Thanks! Doctor: Hi, dearI have gone through your question. I can understand your concern. You had utrine cancer and you have gone for hysterectomy. Sexual pleasure and orgasam are not affected by this. However age, mental stress, depression all can affect the orgasam sensation. So in my opinion its not due to your surgery. You should consult your doctor for counselling. Hope I have answered your question, if you have doubt then I will be happy to answer. Thanks for using health care magic. Wish you a very good health."
},
{
"id": 29751,
"tgt": "What causes redness,pain and rashes around the belly button?",
"src": "Patient: My wife had gall bladder surgery in Sept 2016. This morning, she woke up with redness around an in her belly button incision site, and a small rash there. She s got some pain in the area also that feels like a burning. It started last night and worried her enough to say something this morning? Doctor: Hi,Thanks for your question on HCMI do understand your pain and discomfortsAs per your clinical history is concerned please do a physical examination by your local doctors may be due to minor skin infections so you may use any antibiotic cream like [framycetin] and if oral antibiotic is required take a full course after consultation by your doctor.Hope that helps"
},
{
"id": 183346,
"tgt": "Suggest remedy for toothache",
"src": "Patient: Hello,I've had terrible toothache last night from the tooth, on which I'll go in a month or so, for a root canal treatment. Last night it pained terribly. I somehow remember, during my childhood, my father gave us potassium permanganate solution to get rid of pain. Is it worth giving a try. I'm 29 now. Doctor: Thank you for your query,The toothache is usually due to caries exposed tooth in which the infection has reached the pulp tissue (living part of tooth) and hence causing inflammation of the same. Due to inflammation you are experiencing excruciating pain, along with sensitivity to touch, cold and hot things. It is advisable to take proper antibiotics (to control infection) along with analgesic (to control pain). I am not sure whether potassium permanganate solution will work!!!"
},
{
"id": 86367,
"tgt": "Suggest treatment for persistent right sided abdominal pain",
"src": "Patient: I am having extreme abdominal pain on the right side. I have had this for serveral months now. I have had a ct scan and am scheduled for a colonoscopy in two weeks....my doctor will do nothing about my pain and I think it is a hernia or complications from the hernia repair with mesh ten years ago. I also have had umbilical hernia repair. Doctor: is the pain directly below the operation site?are you male or femaledo you get fever with pain?nausea?constipation or diarrhea?burning feeling when you urinateis the pain more on the front side or on the back?"
},
{
"id": 74362,
"tgt": "How to remove tuberculosis scar on lungs?",
"src": "Patient: Good day! I got Pulmonary tuberculosis when i was in grade school and every time i went to xray for job or any reason the result shows some densities, my doctor says its a scar but i just want to ask is there any way to remove it or appear my lungs clear? Doctor: Thanks for your question on Healthcare Magic. I can understand your concern. No, you can not remove scars from lung. Any lung insult (tuberculosis, pneumonia) heal by fibrosis and calcification. They form Scar on healing. And these scars are permanent. Nothing should be done as they are harmless. For your job purpose, you need to prove that that these scars are noninfectious and you are not suffering from active disease. For this, you need to consult pulmonologist and get done CT thorax and bronchoscopy. If both these are normal then you can argue with job authorities. Hope I have solved your query. I will be happy to help you further. Wish you good health. Thanks."
},
{
"id": 6710,
"tgt": "Why am I unable to conceive after a miscarriage ?",
"src": "Patient: Iam 30 years old and my height is 5feet 1 inch ,my weight is 55kg.I got married since twoyears back . till now I dint conceive ,Iam doing pcos treatement During the medical treatment , I conceived , but unfortunately I had misscarriage . After my misscarriage my periods become irregular, on taking duphaston only ,I get my period.will there is a chance to become pregnant on pcos Doctor: Hi You will have to undergo further treatment with ovulation induction drugs etc to get pregnant.See a fertility specialist also.More tests need to be done."
},
{
"id": 185161,
"tgt": "Why do I have sores on tongue and gums?",
"src": "Patient: I have a mouth sore under my bottom partial ( under my tongue) but not on my tongue. It's more on the bottom part of my gums where my partial fits right on it. Very painful....can't hardly stand to put my partial in & I need it to eat. Had this happen once before & it took a long time for it to heal. Is there anything I can do to speed up the healing & do I need to see a dr.?? I'm a 54 year old female weighing about 155 & I'm 5' 3\". Doctor: Thanks for using Health Care Magic.Read your query.The ulcer can be due to the denture injury , which is causing the pain .I would advice you to apply mucopain / mucogel on the lesion.Avoid wearing the denture for a day or two till the symptoms are relieved.If still persisting consult the dentist to check if the surface or edge of the denture is rough and get it smoothened to avoid more injury.If the pain is severe , take a Panadol.Do betadine mouth wash gargling.Thanks and warm regards."
},
{
"id": 143710,
"tgt": "What causes headache and memory loss ?",
"src": "Patient: I had sudden attack for ten sec a loat of pain on left side my head was so swear it remains up to 40mnts I lost my memory for ten mnts I went asleep for two hrs low pain was still in my head memory arrives but power of my body was very down it happened yesterday Today I took lobazam10 mg and emotion 100mg at 9.30 now I fill some reflex Should I go to PGI for further consultancy or continue this previous medicine this Medicean was stopped after taking 4years Doctor: Hi,this is due to seizure episode followed by headache and loss of memory.You continue this medicine .If I would have been your doctor then I would advise you to do EEG and neurology consultation.Thanks"
},
{
"id": 30402,
"tgt": "Suggest treatment for fatigue, headache and joint pains",
"src": "Patient: I have been diagnosed with lyme disease and have had symptoms for over two months that include severe fatigue, headaches, knee and hip joint pain, nausea, stiff neck and back. Yet I am being told by doctors that the lyme disease is no longer active. I have been on 2 cycles of antibiotics already. What else can i do or what else could it be? Doctor: HiSorry to hear you suffered from lyme disease. It is a disease transmitted by the black-legged tick.Symptoms are as you described.Tretment of lyme disease is with antibiotics like doxycycline, cefuroxime or amoxicillin for 10 to 21 days. In a small percentage of cases, the symptoms can last for over 6 months.This condition is known as post-treatment lyme disease syndrome.So persistence of symptoms does not mean that you still have the active disease.You will progressively get over it. Just rest well and eat healthy.Hope my answer helps to reassure youBest regards"
},
{
"id": 13125,
"tgt": "Remedy for severe rash after consuming medicine for facial herpes",
"src": "Patient: Hi. I am currently in bali. I got what i thought was a heat rash and allergic reation to sunscreen. As i have been told by doctors in australia i am allefgic and i get a similar rash. but i went to thr doctors to get a cream. The doc told me it was facial herpes which i have never had before. I have taken the medicine for facial herpes for one day and the rash has gotten much worse. I do not know if i should keep taking the medici three daysne as i will b Doctor: Hello, The symptoms can be related to an allergic reaction. If you are an allergic person, I suggest avoiding contact with the allergens that cause the symptoms. Meanwhile, I suggest using Loratadine. I also suggest using Calamine lotion for local application. Hope I have answered your query. Let me know if I can assist you further. Regards, Dr. Dorina Gurabardhi, General & Family Physician"
},
{
"id": 486,
"tgt": "Is there a chance of pregnancy if the tip of penis touches the vagina?",
"src": "Patient: my girlfriend and i tried anal for the first time without a condom, she said it hurt so we stopped, but im worried that my penis touched her vagina, but did not go inside it, could she get pregnant if the tip of my penis only touches her vagina Doctor: Hi, Thanks for the query. I understand your concern. Conception needs deposition of semen in female genitals So just touching of the genitals (without e\u00f1ering or a\u00f1y discharge) can not cause conception. Thanks"
},
{
"id": 6890,
"tgt": "When is the ovulation time after taking HCG injection ?",
"src": "Patient: hi,I am trying to concieve for past 3 months.My cycle is 27 days and it is regular.I have hypothyrodism and doctor had put me on to eltroxin100mcg for past 5 months.now TSHlevel is normal.Recently i had my HSG done.both tubes are patent.doctor had put me on to letros tablet from 3 day of my cycle(3-8day of cycle).on 10 thday of my cycle i had under gone follicular scan and left ovary has follicule of 18mm*14mm size .on 12 th day the scan was again done and follicle size was 20mm*16mm.doctor advised for HCG injection;5000IU.Now could you please tell me when can i expect ovulation Doctor: Hi Wewrer, In 27 days cycle ovulation will be on day 13th. Ovulation occurs 14 days priar to oncoming menstrual bleeding."
},
{
"id": 35826,
"tgt": "Please suggest treatment for throat sinus infection",
"src": "Patient: i have been on 3 different antibiotics over 2 months for throat sinus infection. My throat started hurting again on right side. have pus coming from roof of my mouth behind uvula. sore throat keeps coming back before i finish antibiotic. i am now on my last day of biaxin. 55 years ols Doctor: Hello dear,Thank you for your contact to health care magic.I read and understand your concern. I am Dr Arun Tank answering your concern.I advice you to do culture and sensitivity from the pus that coming out of the back of your mouth.It will guide you regarding the probable organisms and it's sensitivity pattern.Once the report is available to you, you can take the drugs according to it.You might be having resistant infection that is why it is not responding to the drugs and it is recurring there after.Please maintain the life style more hygienic and healthier. This will equally affect the disease as antibiotics do.I will be happy to answer your further concern on bit.ly/DrArun.Thank you,Dr Arun TankInfectious diseases specialist,HCM."
},
{
"id": 22130,
"tgt": "Suggest treatment for chest pain",
"src": "Patient: for the past couple of weeks ive been having chest pain on my right side. The pain goes down my arm, i alot of cramping on my stomache and diarrhia for two days, i also have to catch my breath most of the time and i have no energy and i feel faint please help im 29 yrs old and ive been feeling like i have the flu every day Doctor: hello, Mostly it's a acidity and reflux disease. Do you also have nausea, vomiting, bloating, burping, increase in pain on food, sour water feeling in throat or chest burning, if yes it further supports the diagnosis. You should avoid fatty, oily and high calorie diet. Have some walk after having food instead of taking rest. Have multiple small meals instead of heavy meals. Have regular sleep habits and avoid stress. Lots of green leafy vegetables, fruits. Avoid smoking and alcohol if any. You can get prescribed tab Pan DSR 40 mg beforebreakfast once a day for 2 weeks. For cramps you can tab dicyclomine 20 or hyoscyamine 10 mg whenever needed basis prescribed from local doctor. If it doesn't respond, you should get yourself examined and need some investigation like Ultrasound and stool examination. One ECG to rule out heart problem although doesn't appear to be one. Also, may need deworming with Albendazole etc."
},
{
"id": 84197,
"tgt": "What impact does clamoxyl duo posses on baby?",
"src": "Patient: after having an emergency c sec 9 weeks ago to deliver a healthy baby boy i have now been given - clamoxyl duo 500/125 along with METROGYL 400. to cure an infection that they noticed today when i had my check / pap smear im a breast feeding mum and i want to know how these drugs would effect my baby? Doctor: Hello, Both metrogyl and clamoxyl are safe during breast feeding as it is not secreted in the breast milk and no major adverse events are reported. You can safely continue the course of antibiotics. Hope I have answered your question. Let me know if I can assist you further. Regards, Dr. Shinas Hussain, General & Family Physician"
},
{
"id": 59240,
"tgt": "Been through gall bladder surgery. Suffering from gastric and back pain. Ultrasound test reported 20cc fluid. Prescribed toradol, norco, prevacid, miralax. Remedy?",
"src": "Patient: I had gallbladder surgery 1 week ago.My surgeon had to staple during my surgery. I had a lot of pain from trapped gas pockets from scope and I have to sleep elevated. I am having pain in right side and towards back when I breathe as if a stabbing pain.I had ultrasound done and lab work ultrasound showed 20cc of fluid where gallbladder used to be but lab work is ok.I am now on toradol and norcoAnd Prevacid, miralax everyday,I have severe reflux nowConstant epi gastric pain radiating thru back Doctor: A small amount of pain post surgery is normal also a small amount of fluid in the gallbladder area post surgery is a normal sequalae of surgery and usually this fluid gets absorbed over time. If your pain persists or increases and also if the fluid persists or increases in volume then is of concern and needs to be looked into. Take care"
},
{
"id": 46803,
"tgt": "What causes knots on arms while undergoing kidney dialysis?",
"src": "Patient: I have been on Dialysis since June, 2011. (female, 55 years old, 5'11\" 185 lbs) Last test showed kidneys functioning at 4%. I have noticed most of the long time patients have huge, round protrusions (knots) about the size of a ping-pong ball on their arms. I find that very disturbing. I developed a swollen, hard, area two weeks ago that is still tender , after an infiltration. What causes the \"knots\" I have seen and is there any way to prevent them. Doctor: Hello and welcome to HCM.As an Urologist and transplant surgeon,i can understand your anxiety.Your kidney function at 4%,is like a non-functioning kidney.Dialysis or a kidney transplant are the only options available.A transplant is the best option. All people on weekly dialysis,will have a fistula on their forearms,for access. That should be the knots you see.Once the A-V fistula has matured,the veins get swollen up and prominent.If they're hard or tender,you must show it your nephrologist.Thrombophlebitis is a possibility.You'll need treatment for it.If you've any doubts,send all reports to me,as a direct question.Dr.Matthew J. Mangat."
},
{
"id": 35997,
"tgt": "Suggest precautionary measures post typhoid infection",
"src": "Patient: rESPECTED sIR mADAM i have recently recovered from typhoid and was on the bed for more than 15 days after that i have also taken a weeks rest currnetly my temperature is normal what precautions should i take so that i would neither harm my self nor my family and environment Doctor: Hello dear,Thank you for your contact to health care magic.I read and understand your concern. I am Dr Arun Tank answering your concern.Wish you a very happy recovery from the typhoid.You should take the moderate rest now after a weeks rest.You should start the exercise early in the morning. It should no be hard exercise. Only mild exercise like walking to start with. Please keep glucose ready with you while walking.Avoid eating heavy meal at once instead you can take the frequent small meals. Because there's chance of intestinal ulcer. Which on heavy diet can cause perforation.You can take high protein, high sugar meals while low fat and more bulky foods are not allowed.Please live hygienic life as it is the most important part in your life particularly in convelacence period.I will be happy to answer your further concern on bit.ly/DrArun.Thank you,Dr Arun TankInfectious diseases specialist,HCM."
},
{
"id": 208225,
"tgt": "Suggest treatment for lack of concentration",
"src": "Patient: I'm a college student and I have trouble paying attention and I make horrible grades. Everyone tells me that I have ADD and should get tested but i can't afford it because of how expensive it is and because of my college budget. Is there anyother way for a doctor to diagnose me and put me on medication without having to actually go through all the testing because that's just finantialy not an option for me? Doctor: DearWe understand your concernsI went through your details. I suggest you not to worry much. You do not have any concentration problem if you:1. are able to watch an interesting movie for one and half hour.2. are able to play a game without loosing your concentration for an hour.In these given situations, you were concentrating on those given tasks because of your interest on those subjects. Therefore, if you are unable to concentrate on your studies, it is because of your non-interest. Make your studies or learning interesting.If you require more of my help in this aspect, Please post a direct question to me in this website. Make sure that you include every minute details possible. I shall prescribe the needed psychotherapy techniques which should help you cure your condition further.Hope this answers your query. Available for further clarifications.Good luck."
},
{
"id": 37487,
"tgt": "Suggest treatment for food poisoning",
"src": "Patient: Hi Dr. I ve just recovered from mild food poisoning that started early Sunday morning after reheating some take away food from the night before. I was sick at bout 1am until 5am after which I rested and went into work about 2pm(I work in the middle east and work Sundays) I have not been sick since 5am on Sunday but still feel a bit off however my main worry is every so often my chest starts to flutter for a few minutes (I ve done a bit of internet research and it sounds like these are heart palpitations) Should I worry or is this just a result of the food poisoning (the reheated food was a mutton curry with rice). Many thanks Andrew Doctor: Hello,Thank you for your contact to healthcare magic.I understand your health concern, if I am your doctor I suggest you to check for your blood pressure. Because of diarrhoea and vomiting associated with it there is probability of dehydration and low blood pressure. Because of it heart beats faster for which you feel palpitation.I will be happy to answer all your future concern. Thank you,Dr Arun TankInfectious disease specialist.Wish you a best health at health care magic."
},
{
"id": 157123,
"tgt": "Should I be concerned about lymphoma with enlarged lymph nodes found in CT scan?",
"src": "Patient: I have paraesophageal lymph node x2. I have no symptoms. I have had endosopy x2 done within the past 6 months. no lymph node found on the first endo. On 2nd endo nonbleeding angioectasiis found in the duodenum, No finding to account for abdominal lymphadehopathy.Have had ct done whereas an enlarged lymph node was found. have had pet scan/ct done whereas showed a 2nd enarged node, size of 2 cm. Have had dcis a year ago, treated with partial mastectomy and radiation. Will have follow up with another endo ultrasound and biopy. Should I be concerned about lymphoma? Doctor: Hi and welcome to HCM, thanks for the query.at this point you should not be worried since most such findings are not indicating malignancy. enlarged lymoh nodes can be found in healthy people or as as reaction to any systemic infection. You need just regular follow up and thats all. Wish you good health.Regards"
},
{
"id": 141952,
"tgt": "Is it possible to find typhoid fever through CT scan?",
"src": "Patient: My nephew is sufferring from some probelm related to Brain. Fluids were taken out and sent for test. After that the CT Scan reports shows typhoid fever. Is that possible that CT Scsn shows such results, beacuse when blood samples were taken nothing were detected, but after CT Scan, the doctor was telling as typhoid. Plesse let me what can be problem Doctor: Hello!My name is Dr. Aida and I am glad to attend you on Healthcaremagic!I would explain that the brain CT scan is not a specific test for typhoid, as the brain changes should be correlated with the blood lab tests and fluids exam. The CT scan changes may be suggestive of a brain disorder, but other tests are needed to confirm the diagnosis in case of typhoid. You should discuss with his doctor on the above issues. Hope you will find this information helpful!Best wishes!"
},
{
"id": 144241,
"tgt": "What causes tingling sensation in upper body?",
"src": "Patient: About 4 times a week I have a very strong tingling sensation in my upper body. It is so strong it stops me in my tracks. The best way I can describe it is when you squeeze something real tight for a long period of time and then there is a strong tingling sensation. It is in my chest, my back, neck and arms. It last under a minute and then goes away. I have been experiencing them for about two years but in the past few months they have become more intense and instead of 10 seconds it lasts closer to a minute. Doctor: Hi, I am Dr.Bruno and Let me answer your query.Very strong tingling sensation in my upper body can be an early sign of Syringomyelia. I would suggest that you consult a Neurosurgeon and undergo clinical evaluation and X Ray / CT / MRI Cervical Spine with CV Jn and other tests as suggested by the Neurosurgeon If you need any clarification / have doubts / have additional questions / have follow up questions, then please do not hesitate in asking again. I will be happy to answer your questions."
},
{
"id": 49973,
"tgt": "On Pravochol for 15 years. Have increasing pain in the kidney area. Cannot lie on that side. Stopping the medicine helped. Suggestions?",
"src": "Patient: have been taking Pravochol for 15 years ,am now 68.Have had increasing pain in the upper back near the kidney area and in my right side under the ribs, so much so I cannot lie on my right side at night and have been requiring Panodol for the pain every night.However on stopping the medication symptoms have decreased considerably- should I continue off the Pravochol Doctor: hithank you for the queryI have gone through your query understood your concerns...sir upper back pain in kidney area I would like to give you some causes of pain muscle pull vertebral column diseaseliver and gall bladder infectionrenal pathologypravachol has got muscle pull as the rare side effect which I don't think is the cause of your pain after taking so many long years but still takingthe benefit of doubt you can consult your GP and should continue taking pravachol..and next I would like to suggest you to get an ultra sound abdomen done to rule out liver gall bladder and renal pathologies if any which is causing your pain at kidney area, clinically pain because of renal pathology will be associated with nausea vomittings and radiation to groin and urinary symptoms like dysuria hematuria so I hope that pain you are suffering from will not be cause of kidney problemhope I have answered your querythank youregardsDR Alekhya"
},
{
"id": 50206,
"tgt": "Suffering with CKD kidney infection, taking medicines. Could the medicines be causing increase in blood urea, creatinine?",
"src": "Patient: Dear Sir My mother is suffering from CKD Kidney infection . As per report dated 10-01-2013 Her Hemoglobin=7.60 Urea=228 Creatinine=10.05 Uric Acid=5.10 Protein=6.50 Albumin=3 A:G Ratio=0.86 Alkaline Phosphatase=94 Calcium=8.50 Phosporus=4 Sodium=130 Potassium= 5.30 Chloride=104 Allopathic treatment has started wef 24-11- 2012 Allopathic medicines given as under, Pantocid DSR=anti gastic, Thyrox-25= thyrod ( both in morning empty stomach ), Phostat 667mg= calcium tab, Ferium XT=Iron, Starpress XL-50= metoprolol succinate extended release tab, Maxid D3 0000 IU= calcium, Zyloric 100 mg= uric acid, Tide-10 mg (1/2 tablet), Ketosteril, amlong 5 mg= blood pressure , Injuction Espogen-4000 IU kindly suggest if any of these medicines are causing increase in blood urea and creatinine and also suggest their substitute, if any. Kindly suggest line of treatment as soon as possible thanks Doctor: HiThanks for the query.The blood results of urea and creatinine suggest an advanced stage of kidney failure.The list of medications you have mentioned are commonly used to treat the various complications of kidney failure.At this stage your nephrologist may suggest initiation of dialysis.Hope this helpsGood luck"
},
{
"id": 64454,
"tgt": "What is the large lump above my collar bone?",
"src": "Patient: Hi I have a large lump on my left side right above my collar bone next to my neck, it is soft. Went to the doc and he said soft tissue should disappear in 2 weeks it has been 3 still have it. I have been icing it no results. I am 66, 5' 8 164 lbs. Thanks Doctor: Welcome to Healthcare magicI am Dr Fahim, and I will help you with your problem I have gone through your question. What I comprehend are, there is a large swelling above left collar bone, for the last 3 weeks. More information is needed for better understanding of the swelling- How this lump developed, gradual or sudden?- Any association with trauma?- Is it painful?- What is its consistency, soft/firm or hard?- Whether it is mobile or fixed?- Any recent history of throat infection?- Any history of fever, weight loss? With available information it appears to be enlarge inferior cervical lymph nodes. If there is no recent history of throat infection or cough and it is not disappearing with time,You will need an excisional biopsy of this swelling to be performed by general surgeon under local anesthesia and send for histopathological studies to get a definitive diagnosis.I hope my answer will help, do communicate back with answer of above questions if problem persist. Do rate it if you like it.Regards"
},
{
"id": 90421,
"tgt": "Is abdominal pain with urge to urinate dangerous?",
"src": "Patient: hello Dr. Chandan. i m vinay i have some query, i felt abdominal pain and testicle pain when i was with my gf, and at the same time i felt like i have to pee. is there any serious problem.. feeling of abdominal pain happened when i sit for 2-3 hours with her. and i had to go for pee many time. Doctor: Hi.Thanks for your query.Interesting history indeed.This looks to be more of an anxiety disorder than a disease. It looks over-excitation has caused this problem.On a lighter vain:: keep the anxiety or the girlfriend away.Do not worry. This needs no special treatment. This will improve with time."
},
{
"id": 57729,
"tgt": "Any food suggestion for SGPT level of 475 that has reduced from 1180 after taking udiliv?",
"src": "Patient: My daughter, 9 years old, had SGPT count as1180 in the month of Aug 2013 and after one month the SGPT level comes down to 492. 2 days back again test is done and now the SGPT level is 475.She is taking Udiliv*300 twice a day. I am very much worried about this level. Suggest food to be taken Doctor: Hello.Thank you for the query.Such high liver enzyme level indicates acute liver disease. It can be caused by Hepatitis viruses. This can not be healed with diet. It needs to be diagnosed and treated by hepatologist.That is why I suggest you to visit your doctor and start with viral tests done.Hope this will help.Regards"
},
{
"id": 191901,
"tgt": "Suggest treatment for diabetes",
"src": "Patient: I am suffering from high blood pressure last 20 years now I am I am after age of 35 I am suffering from severe diabetes I am after age of 35 I am suffering from when I become nervous my heart beat become very fast Now I take one tablet of calvix 75 which prescribed by dr dandaiya now I what to do please suggest me Doctor: HiDiabetes is a disorder in which either your pancreas does not produce insulin (TYPE 1 ) or insulin is produced in less quantity or insulin not utilized by the tissues/organs(Type 2)More than 90% of the diabetes patients are Type 2. If your blood sugar levels are high eg. Fasting > 150 HBA1c > 7.5 then you need to discuss with your diabetic consultant for further course(either add another oral drug or insulin) depending on the caseOne of the good way to increase the insulin secretion is exercise/walking for 45 minutes daily or atleast six days per week. This will help secretion of the insulin aswell as utilization of the insulin by muscles/organs etc. Also exercise/walking will help to maintain your blood pressureYou can continue using Calvix tablet however if you have gastritis then you can take antacid tablets like Ranitidine before foodTry to meditation daily which will help controlling your anxiety and heart beats to normalIf you still have any query please get backWish You Good HealthRegardsDr Sridhar K"
},
{
"id": 26454,
"tgt": "What causes feeling like chest going to explode with hypertension history?",
"src": "Patient: I am a physician and have not been able to get a straight answer on thses symptoms myself. I will wake up from a nightmare where my chest feels like it is going to explode. I normally have a fast heart rate in the low 90's. When I wake up I am in the 60's and by 30 seconds am back to a normal rate and the chest pain abates. I have no history other than HTN and have had these symtoms over a decade. I have been on anti-hypertensives for about 4-5 years so it is not the medication. Doctor: Hello!Thank you for asking on HCM!I read your question carefully and understand your concern. Your symptoms seem to be related to nightmares and anxiety. This is a common sleeping disorder, associated with autonomic nervous system activity. Have you measured your blood pressure during these episodes?I recommend consulting with your GP for a careful physical examination, a resting ECG, a chest x ray and some blood lab tests : - complete blood count- fasting glucose- thyroid hormone levels (thyroid dysfunction could explain some of your symptoms)- kidney and liver function tests - blood electrolytes, etc. An ambulatory 24-48 hour monitoring would be necessary to exclude a possible cardiac arrhythmia. Hope to have been of help!Best wishes, Dr. Iliri"
},
{
"id": 175584,
"tgt": "What causes pimples on face and bumps on forehead of a child?",
"src": "Patient: My 5 year old has pimples on his face, mostly on chin, but one one his cheek and looks like starting a few more bumps on forehead. It seems to get better over night, and then by the time I pick him up from school it looks worse. Wondering what it could be? No fever. Eating well. Doctor: HelloPimples on face in a child may be due to pyoderma,seborrhic dermatitis with secondary infection. In both the cases patient may not have fever.Atopic dermatitis itself may cause red bumps/rashes on the face.In case of pyoderma or secondary infection treatment with antibiotic is needed. In atopic dermatitis treatment of seborrhea along with application of emollient/mild steroid is required.I would advise you to consult a pediatrician for clinical examination and management accordingly.Regards"
},
{
"id": 94013,
"tgt": "Abdomen pain after menstrual period is complete. Acidity and constipation. What can i do?",
"src": "Patient: Hye doc. I am 27 year old. I have sweet and liight pain in left lowe abdomen . I feel light swelling of left lowe abdomen. I have this pain afet my menstrual period is compete.I went to gynacologist theythey do some check up and lell every thing is normalbut i still have pain in left abdomeb .i also suffer from acidity and constpation .pls suggest me what can i do Doctor: Hi welcome to Health care magic forum. You had a sweet and light pain in the left lower abdomen. Gynacologist said that every thing is normal, but still pain is continuing. Left lower abdominal pain usually due to urinary tract infection, stones in the urinary tract, Inflammation of the descending colon or rectum. I advise you to consult a surgeon for diagnosis and treatment. You may need to have M.R.I. colonoscopy, besides other routine tests for confirmation. Wishing for a quick and complete recovery, and thanks for calling H.C.M.F. Best regards."
},
{
"id": 103545,
"tgt": "Taking levosiz 10 for allergy. What are the side effect and precaution to be taken if continuously consumed levosiz 10?",
"src": "Patient: Hello Doctor, I am suffering from allergy & as per the our family doctor advice I have started consuming Levosiz 10 1 tablet in a day since last one year and feeling comfortable now. Please let me know, how harmful to an individual health when he consinuously consumes Levosiz 10 & what pre-cautions should be taken to over come from any side effects. Doctor: the anti allergics are usually harmless and there are no side effects even you take lifelong as they are made like this so have no sideeffects long termthe medicines like antihypertensives diabetic allergies are madelike this as they have to be taken lifelongtake lot ofwarm water and can continue off and on when you requre on long term basis"
},
{
"id": 116758,
"tgt": "How to reduce eosinophils in blood?",
"src": "Patient: hi doctor my name is sathish. my age is 21 yr and 9 month. from my childhood more often i affected by running nose and cold. for that i simply considered a doctor and took medicine however which recovers for few days. and finally i tested my blood found that eosinphils 9% polymorphs 55% lymphocytes 35% monocytes 1% doctor said that you have allergy ...may i know what is diet should i follow to decrease the eosinophile in my W.B.C Doctor: Hi,Thanks for asking.Based on your query, my opinion is as follows.1. Allergy is the main cause for eosinophils increase.2. Avoiding allergen is the best method. However, your eosinophils is just mildly raised and not to worry.3. Diet is not important. Identifying the allergen and avoiding it is the only method, which will be helpful.Hope it helps.Any further queries, happy to help again."
},
{
"id": 145239,
"tgt": "Suggest treatment for sleep disorder after ischemic stroke of left thalamus",
"src": "Patient: My husband 55 years old had an ischemic stroke of the left thalamus 6 months ago. He is unable to regulate his sleep. Sleeps up to 16 hours a day, and can t wake himself though he tries in the morning with alarm clocks and phone calls he requests from his sons. He s getting frustrated with it, and knows he ll need to be able to wake himself if he is ever to return to work. His memory and motor skills are functional - the sleep is the biggest issue now. Any ideas? Doctor: Hello ! I read your question and understand your concern. In my opinion he is suffering for hyipersomnia. It is more common in thalamic stroke. Unfortunately there is not a curable therapy to this problem. I would give some advises to try to regulate his sleep cycle: -Sleep a few hours longer at night to avoid excessive sleepiness during the day.-Exercise daily in the morning or early afternoon, if possible.-Engage in enjoyable activities that require your full attention, such as spending time with friends, writing letters, or playing with a pet.-Try to go to sleep and wake up at the same time every day.-Get out of bed and stay out of bed until bedtime.-Avoid foods that make you sleepy and heavy meals during the day.-Avoid alcohol and caffeine.I would also try Modafinil , which is a drug effective in primary hipersomnia. This drug must be prescribed under a neurologist prescription. Hope to have been helpful!Best wishes, Dr. Abaz Quka"
},
{
"id": 64726,
"tgt": "Suggest treatment for swollen lump on forehead after injury",
"src": "Patient: My daughter hit the ground with her forehead 2 days ago. A huge lump formed above her eye and has since gone down but swelling has moved to over half her forehead, her nose, and one eye is somewhat swollen but both eyes are becoming bruised. No nausea or dizziness, just a bad headache and swollen face. Normal? Doctor: Hi,Dear,Thanks for the query to HCM .I studied your query in depth.I feel concerned about it.In my opinion ,these symptom complex and indicates --Severe Head Injury-with bad bruising and headache.Your daughter certainly needs Observation in ER-under Surgeons care,for critical signs of head-injury needs control by specific medications-under close observation.The head injury is not too serious but certainly needs special care.Treatment would be-a-Observation with Liquids- By Mouth for 24-48 hrs more,b-Tb-NSAIDs,c-Tb-serropeptidase -for caring- extensive swelling of Headd-Observation for vomiting,nausea , pupilary signs, and others is needed for another 48 hrs-under Surgeons Care-in ER.d-Antibiotics if required.Investigations urgtly required are-Skull Xray in different angles,CAT-scan for the Skull Anatomy-as the facial look is distorted much.A opinion of the Neuro-Surgeon as per Surgeons need and advise.All this care would take care of your daughter and I hope she should come out without any Un-eventuality.Hope this would help you.But first thing you should do is to shift the patient to ER-and start the treatment Urgtly.I pray fast recovery for your daughter.Wellcome to HCM for any more queries."
},
{
"id": 158836,
"tgt": "Diagnosed with liver and bile duct cancer. Fatigue, back pain and cough. Survival chances?",
"src": "Patient: my sister was diagnosed with liver/bile duct cancer in Feb. She is very very tired, no appetite, pain radiates around to her back and as of yesterday I noticed a persistent cough. She was diagnosed as terminal but is going to participate in a clinical trial next week. She feels so \"stuffed\" in the chest area and coughs while she is talking.One dr said a yr but another hints several months. Should we be preparing for her to leave us soon. She also has some cancer in the cervical area but the dr said we won't worry about that now. Doctor: Hello From your description , your sister seems to have advanced stage cancer. For cancer in hepato-biliary area the treatment are not that good and outcome is also not so good. Lets hope she benefit from clinical trial. Often overlooked treatment in advanced stage is palliative care.It helps relieve symptoms, but is not expected to cure the disease. It can be given along with cancer treatment, or can even be cancer treatment. The aim of palliative care is to improve the quality of your life, or help you feel as good as you can for as long as you can. Sometimes this means using drugs or other measures like physiotherapy etc to help with symptoms like caugh pain fatigue etc. Regards"
},
{
"id": 74868,
"tgt": "Suggest treatment for chest pain,shortness of breath and swollen neck lymph nodes",
"src": "Patient: Age:47 Height:5 8 weight: 135 Sex: Male - I have been having lung pain with Shortness of breath, swollen lymph node/s above right collar bone, Pale face color, some weight loss (10 pounds in last 2 months), odd headaches and woozee feeling, with ongoing low blood sodium (136 down to 125) for the past 2 years. I have had 3 chest CT s. First 2 showed mild lymph node swelling (no masses or malignancy) with the 3rd showing what the radiologist called residual thymic tissue with it s own blood supply (Not present on first 2 CT s). 3rd CT also showed inflammation of Rugal folds, Dodoneum, with contracted Gallbladder, and partially collapsed stomach. Have smoked 1 pack for 35 years (but am quitting due to the above symptoms and chest pain). My Primary Care says that my CBC is fine, CT s show nothing significant and other deficiencies aren t serious (low Vitamin D, elevated CRP, Elevated RH Factor) so it can t be anything serious and to quit smoking change my eating habits (I don t really eat badly). Symptoms have become substantially worse over the last several months. I know what it sounds like and I am worried due to worsening symptoms. Is there anything that stands out given the tests and other items above? Thank You for your time Doctor: Hello Thanks for posting at HCM. I see here that you underwent multiple CT scans, blood tests. But you have not mentioned anything about the lymph nodes which you have above your collar bone or in the chest. havent you ever been adviced a lymph node biopsy? I recommend a lymph node biopsy or aspiration from the collar bone lymph node under USG guidance. The aspirate can be sent for further tests. In view of your symptoms like shortness of breath, weight loss and swollen lymph nodes, tuberculosis must be ruled out. Please go ahead with a biopsy from the lymph nodes. Wishing you good healthRegards"
},
{
"id": 45980,
"tgt": "What cause drowsiness and confusion while on dialysis?",
"src": "Patient: my mother is a chronic kidney patient since 2006, was doing good with medicines, 19 Dec she was hospitalized to treat high creatinine, urea and low HB, given blood, gets little better, then dialysis started, looks even better, then done CAPD process for home dialysis on 1 Jan, the same night she had respiration problem, went to ventilator, last 10 days shw is suffering from CO2 narcosis, the dialysis and regular medicines are ongoing, she is drowsy and confused Doctor: Hello and Welcome to \u2018Ask A Doctor\u2019 service. I have reviewed your query and here is my advice. I had gone through your question and understand your concern. As per the patient's history seems she has uremic symptoms. That is her blood urea level is increased and affecting the brain function. When such patient comes to me I advice hemodialysis. Hemodialysis is much effective in control blood urea and Creatinine. Hope I have answered your query. Let me know if I can assist you further."
},
{
"id": 127670,
"tgt": "What causes throbbing pain in knee and ankles?",
"src": "Patient: Hi I have throbbing pain in my knee and ankles so bad that at times it just about takes me off my feet I work on concert all day in a diesel shop and the pain started two days ago when I woke up and could not walk right away thats how bad its herts thank you for your help on this matter Doctor: Hello and Welcome to \u2018Ask A Doctor\u2019 service. I have reviewed your query and here is my advice.Sudden onset throbbing pain in both knee and ankles,without any local cause is most likely to be a radiating pain from back( sometimes there is no pain in the back). I suggest pain killers , muscle relaxants and MRI . If pain does not reduce in few days then I would advice orthopedic consultation.Thanks"
},
{
"id": 150378,
"tgt": "Pain in the buttocks and legs. Disc protrusion and annular tear in the MRI. Need surgery?",
"src": "Patient: Do i need surgery? I have been having terrible pain and can't feel my buttocks sharp pain in legs. Had MRI and central disc protrusion with superimposed annular tear at the L4-L5 level leads to mild effacement of ventral thecal sac. Mild bilateral neuroforaminal encroachment at this level. Small central disc protrusion with superimposed annular tear at the L1-L2 level. Doctor: Hi, Your symptoms are suggestive of bilateral sciatica.MRI explains the cause for it.This due to disc prolapse as you have mentioned.You need evaluation by Neurolgist for same.Symtoms can be releived with neuralgic drugs like pregabalin 75mg twice daily along with amitryptilline combination You require rest for next 2 weeks and Lumbo sacral belt for next 3 months.Avoid lifting heavy weight objects and avoid bending forward.You also require short course of pain killers, and calcium supplementation.If your symtoms are not relived by medical treatment as mentioned above (8 weeks of treatment)or you develop any weekness of limbs then surgery is indicated"
},
{
"id": 180686,
"tgt": "What causes itching in the gums?",
"src": "Patient: I am 38 weeks pregnant and had my bottom left wisdom tooth removed 2 weeks ago. I was prescribed antibiotics but never took them. This week my bottom right side (gums) were throbbing so I took the antibiotics thinking it would help. The pain is gone but now my gums on the lower right side are itchy terribly. Could it be the antibiotics? Doctor: Hello,It looks like there is infection in the gums that has caused itching due to inflammation.Many a times there is inflammation of the gums in pregnancy that is due to hormonal changes and the condition becomes worse if there is infection and deposits in the gums. You should consult an oral physician and get evaluated.For now, you should do:- Warm saline gargles- Do ice packs over the gums- Avoid spicy and acidic foods- Massage the gums with Chlorohexidine gel but do not swallow and keep spittingHope I have answered your query. Let me know if I can assist you further.Regards,Dr. Honey Arora"
},
{
"id": 156112,
"tgt": "Suggest treatment for ulcerated growth in entire body",
"src": "Patient: Hello doctor,I am Deepak from pune, India. Its regarding my father as we get his endoscopy done. And impresion on report says that he got an infiltrating and ulcerated growth in the entire body and the antrum of the stomach. And biopsy report says --- Moderately differentiated adenocarcinoma. Sections shows gastric mucosa, with three of the biopsies showing tumor cells in nests and a typical grandular structures infiltrating the submucosa, with surrounding desmoplasia. Fragments of necrotic slough colonised by bacteria and fungal pseudohyphae with exudate are also included. Want to know the best doctors in Pune. And what treatment we should go for? And whether it's curable at this stage? Doctor: Hi Welcome to HCM I have gone through your query and concerned about his condition . But dear don't get disheartened . Where there is a will there is a way . strengthen his will power by giving him hope of recovery , It is a great thing that helps a person to recover . Meanwhile , you can administer him course of antioxidants as below -Mixture of extract of Lemon , Ginger ,Garlic , Vineyger of coconut water ,Bark of Moringa ( sahajana ) tree , Honey all in equal quantity in a dose of 25 ml , twice a day , before meals for 31 or 41 days as per requirement . Is good healer and also preventive . Give a level t spoon of turmeric powder twice a day , in a cup of hot milk after meals , Meals should be simple nourishing and easily digestible .Take care he doesn't have constipation . Let him do pranayam - Deep breathing exercises & Walk for resistance against diseases and to strengthen the vital part of the body . And many heart and cancer patients have been benefited from this mixtureThe above regimen is to be carried on even side by side of his treatment I suggest you to Find out a good renowned hospital and show his reports ASAP , which will refer the case to specialist , who after studying the case will start treatment . I suggest you should not delay now .. Hope this gives reply to your query Take care .All the best & good healthDont hesitate to get back if have any further query"
},
{
"id": 70607,
"tgt": "Suggest treatment for persistent cough and coughing fits which lead to pricking pain in the shoulders, chest and arms",
"src": "Patient: I keep getting a nasty cough, feels like a dry spot but last night i was coughing so hard all of a sudden when i coughed i got this very painful prickly feeling in shoulders, chest and down arms..It hurt so sad and happened every time i coughed..Someone told me it might be a pinched nerve in my neck from coughing..What do u think?n Doctor: Respected user, hi I evaluated your query thoroughly .* Symptoms narrated are the systemic manifestation of underlying lung pathology only .* Recommendations are - avoid exacerbating factors as dusty atmosphere / pollens , if required use face mask- avoid exposure to excess cold - prefer semi reclining position when ever possible to support lungs - avoid exposure to cooler air , air conditioner till possible- avoid smoking / alcohol / any abuse substances if used- chilled materials , ice creams , cold drinks, beverages, oily , spicy , non vegetarian stuffs should be avoided by en large - garlges with salted lukewarm water & added peppermint oil- maintain oral hygiene- balance nutritious diet to combat external pathogens- need of systemic molecules in form of antibiotics , decongestants, broncho dilators is to be evaluated by physical examination of the patient with necessary Lab tests & X-ray chest / pulmonary function test Hope this helps you.Welcome for any further assistance.Thanks for using Healthcaremagic.com & giving me an opportunity to assist Wishing you a happy healthy life ahead .Regards dear take care."
},
{
"id": 200816,
"tgt": "What causes pain during erection after inguinal hernia?",
"src": "Patient: I had an inguinal hernia and after surgery my right testis and testicular artery have become hard and stiffened. It also has lots of fluid around it. Its been almost two weeks since the operation and it hurts when I get an erection and the sack is up tight. Cause ? Solution ? Is it something serious? Doctor: Thanks for asking in healthcaremagic forumIn Short: It takes time to heal and to gain normal ability to have erection.Explanation: As you have undergone surgery recently,it takes time for healing. Pain may be there for sometime. If pain is there then erection may cause further pain. So,take anti inflammatory drugs as prescribed by your doctor and wait for some more days, you will be back to normal. If you have still swelling or any discharge visit your doctor immediately."
},
{
"id": 220081,
"tgt": "How long can a baby survive inside the womb?",
"src": "Patient: ok so if a woman is pregnant, and shes supposed to give birth, but cant for some reason, and the baby just stays inside her, what happens? does she die, does the womans body reabsorb the baby? and what happens to the baby? how long can a baby live inside a woman if she never gives birth? Doctor: Hi, I have understood your concern. Basically nature has its own mechanism. Usually after 40 weeks if pregnancy, labour process starts .Strong contractions of the muscles of the uterus push the baby out of mother's body through birth canal. So eventually baby comes out of mothers body. It can not remain in side or ddoesn't get dissolved or absorbed into mother's body. I hope this answer helps you. Thanks Dr Purushottam Neurgaonkar"
},
{
"id": 159650,
"tgt": "Bleeding in the anal region after passing stools, blood tests, camera exam normal. Chances of bowel cancer?",
"src": "Patient: Hi my husband is experiencing bleeding when going to the toilet? ive asked him if its in his stools or just on the toilet paper? he thinks just the paper but, cant be sure. He went to the doctors yesterday and the doctor has done blood tests and sent for a camera examination, he did an internal examination and found nothing irregular but, is looking on the chance of bowl cancer? He has had no pain? Doctor: Hi, Thanks for query, First of all we have to decide that blood in stool is there or not? You need stool examination to confirm it. As camera & internal examination does not reveals any abnormality the chances of blood coming from Large intestine arises.There were several condition causes bleeding from large intestine like IBS ,Amoebiasis,Enteritis any pathological lesion etc. So go for stool examination first. thanks."
},
{
"id": 222251,
"tgt": "Will candid v3 vaginal tablets affect the unborn baby?",
"src": "Patient: Hi i am 33weeks pregnant n was prescribed wit candid v3 vaginal tablets and the cream . I have been using the cream but have avoided the tablets because of the babies position and according to the instructions i should not be using it at this stage what should i do ? Will it affect the baby ? Doctor: Hello dear,I understand your concern.In my opinion the candid V 3 vaginal tablets are safe to use in pregnancy.The candidiasis is most common infection seen in pregnancy.Untreated vaginal infections also are one of the causes for preterm pains.So candid v3 tablets can be used to treat the infection.Nothing to worry.It doesn't harm the baby.There will be local action of drug on the fungus over there and no any systemic effects or on the fetus.Best regards..."
},
{
"id": 209475,
"tgt": "What is the suggested medicine to control frequent fits?",
"src": "Patient: Respected Doctor, My daughter was born as a normal child but at her 11th Month was suffered with meningitis fever and shunt process has been done due to several cell damage.All these 15 years Now she is Bedridden who couldn t speak and all her foods in liquid state only. Sir the problem is all these 15 years she had a frequent fits and it was more than 25 times a day till last 6/7 years. After consulted with another Neurologist here in Salem. he adviced us following medicines:- somiel 1mg thrice a day, encorate chrono 500mg twice a day, levipil xr 500mg twice aday & oxetol 300 mg twice a day. Even she suffer 4/5 time a day by fits. Now our major issue is in recent time of 2 months she make noise by shaking her head as well opening mouth too. The same doctor said this is due to behaviour problem and suggest to increase oxetol 450mg from 300mg and feeding accordingly past two weeks and now i can say that she is some what ok. Dear sir, is there any other medicine to control her fits. please advice as our whole family was desperate all these years. Please advice. thanks. Joseph. Doctor: Hello,Thanks for choosing health care magic for posting your query.I have gone through your question in detail and I can understand what you are going through.Taking regular antiepileptic medications is the only treatment. The good antiepileptics are phenytoin, lamotrigine, levetiracetam etc/.Hope I am able to answer your concerns.If you have any further query, I would be glad to help you.In future if you wish to contact me directly, you can use the below mentioned link:bit.ly/dr-srikanth-reddy\u00a0\u00a0\u00a0\u00a0\u00a0\u00a0\u00a0\u00a0\u00a0\u00a0\u00a0\u00a0\u00a0\u00a0\u00a0\u00a0\u00a0\u00a0\u00a0\u00a0\u00a0\u00a0\u00a0\u00a0\u00a0\u00a0\u00a0\u00a0\u00a0\u00a0\u00a0\u00a0\u00a0\u00a0\u00a0\u00a0\u00a0\u00a0\u00a0\u00a0"
},
{
"id": 218444,
"tgt": "Is a nasal growth during pregnancy a matter of worry?",
"src": "Patient: Hi there, I am Seven months pregnant and we just had our house painted earlier in the week. Since the past 4 days I have had nose bleeds in the mornings although and sometimes throughout the day. I went to the A and E and there said there is a growth in my nostrils which is common in pregnancy should I be worried. Doctor: Hello and Welcome to \u2018Ask A Doctor\u2019 service. I have reviewed your query and here is my advice.I think a grouth is always something to worry unless the biopsy shows that it is nothing to worry about. Sometimes in pregnancy nose capillaries become more fragile due to increased hormones and may bleed. This is nothing to worry about. Hope I have answered your query. Let me know if I can assist you further."
},
{
"id": 203105,
"tgt": "Cause of pus filled swollen nipples which on being popped also release blood in a man?",
"src": "Patient: Woke Up A Couple Days Ago, NoticeD My Nipple Was Tender. HurT And Burned Like When I Shave TheM And Cut It. But I Haven't Shaved TheM In Months. JustThe Rubbing Of My Shirt Would Be Painful. Last Night I Finally Looked At It And Realized My Nipple Was ThreE Times The Normal Size I Squeezed It And WhitE Pus Came Out Like When You Pop A Zit. FollowEd By Clear Liquid Then Blood. I Just Looked And Its Still The Same. Popped It Again Same Thing As Before Doctor: Hi,Thanks for writing in.It appears that you are having a sub areolar abscess (nipple abscess). It is due to acute infection and usually of bacterial origin. I suggest that you consult your doctor and get pus culture and sensitivity test done. This will identify the organism causing infection and the desirable antibiotic to be taken to kill the bacteria. You may also apply antiseptic ointment and take oral pain killers for pain management."
},
{
"id": 210977,
"tgt": "Are Diclofenac and Baclofen similar to tramadol?",
"src": "Patient: Hi, my doctor prescribed me to diclofenac sodium 75mg and baclofen 10mg together.. I have previously been on tramadol 100mg once every 12-24 hours as needed but due to my new antidepressant she switched me to these two.. are these two similar to tramadol? And/or what should I expect. I got these for back pain and cramping and I'm curious for answers as to there not being any reviews of the two online. Tramadol works wonders for my pain so I'm hoping there's not much of a difference? Doctor: Hi and . Baclofen tablets is useful for the alleviation of signs and symptoms of spasticity, particularly for the relief of flexor spasms and concomitant pain, clonus, and muscular rigidity. Basically its a muscle relaxant. Tramadol is indicated for the management of moderate to moderately severe pain in adults. Tramadol is a strong analgesic, that's why u will get relief from pain fast. You have not mentioned which antideppresants u are taking. Your doctor has changed to diclofenac (which is pain killer) and baclofen because he/she might have foreseen some drug interactions between tramadol and antideppresants which is not safe. I advice u to follow your docs advice and be safe. All the best. Regards"
},
{
"id": 191111,
"tgt": "Jaw pain when I drink something cold",
"src": "Patient: My problem: Whenever I drink anything cold (even semi-cold), the bone under my gums on my right side become very cold and hurt. Here is some background info (i do not know how relevant this might be): I got both of my right wisdom teeth pulled over 2 weeks ago. my dentist put in dissolvable stiches. Because of my TMJ, my jaw was basically locked after that. When I went back to the the dentist, he just told me to keep prying my jaw open. My jaw still hurts, although it can now move. It is constantly in pain. Doctor: Hi There, Your condition is called post-surgical TRISMUS. This generally is a complication after impactions & extensive bone removal. It shd last for max 3-4 weeks till the healing is complete. The open wound exposes the jaw bone & also the root of the adjacent tooth,hence the sensitivity to cold. You need to do the following; a) Make sure the procedure was properly done: this can be achieved by a post-op X-ray or a 2nd opinion by an oral & maxillo-facial surgeon. b) Take a muscle-relaxant along with pain medication to unlock the jaw muscles like IBUGESIC-MR. c) Chew on a sugar-free gum like orbit/wriggley for 15min every 3hrs. This is physiotherapy for your jaw muscles. d) Gargle with warm salt water for 5min alternate hourly Thanks & take care"
},
{
"id": 12599,
"tgt": "What shall i do to cure Psoriasis suffering since 3 years ?",
"src": "Patient: what to do 3 years and no results only the sunbath tratment left i have psosisas down right leg some on left leg belyy button and feet have tried al sorts but is getiing worse Doctor: Hi!dennis, Welcome To HealthcareMagic forum, Psoriasis is definitely a chronic and a frustrating disease .There are various treatments available ,but it is important to understand that we should try to suppress the lesions with these and then try to maintain it further. Over the years you might need something or the other to maintain yourself.Initially you might require aggressive therapy. You have not mentioned your age and marital status ,kindly do so. Various options are as follows: 1) oral methotrexate ,cyclosporin ,acetretin etc. 2) topical steroids , tazarotene, vitamin D3 derivatives, tacrolimus,tar ,antralin etc. 3) phototherapy - PUVA ,NB-UVB(latest) 4) Biologics (injectable-latest). The result of combination of all this is excellent. Apart from this one should understand that there are certain aggravating factors also which one remember. For example ,anti hypertensive medicines ,ibuprofen ,anti psychotic medicines can aggravate psoriasis. Since I am not aware of what all treatmnmet you have taken and with what results ,i may not be able to guide you specifically.For details you can visit my website https://urldefense.com/v3/__http://www.skinhairclinic.com__;!!Mih3wA!SBzm6_kI6hCZ58EPH6N_05MFfiPbxWXT0a2TJCdFQObRWm5mV5ur7hVjHTVz8g$ . I hope I've answered your query ,if you have any further questions you can write back to us."
},
{
"id": 200596,
"tgt": "What causes intermittent pain in groin and testicles radiating to abdomen?",
"src": "Patient: Hello, I have been having groin pain on and off for a few years. the pain feels like it comes from the testicles like behind them maybe in the epididymis area and shoots up either side of my abdomen. Also have been having trouble urinating, trouble starting and weak stream. I m starting to get a little worried. What could this be? Doctor: Thanks for asking in healthcaremagic forum Your symptoms are suggestive of either Urinary tract infection or Urinary system calculus(stone). So, please visit doctor for investigation. Till then please drink plenty of fluids(water). All the best."
},
{
"id": 209807,
"tgt": "Can a panic attack last off and on for days?",
"src": "Patient: I had a panic attack two days ago, i think. I seem to have all the classic symptoms. Woke up to use the bathroom, heart started racing, sweating, scared, shaking. I have had the same feeling a few months back. I have seen a cardiologist and neurologist where all tests were normal. I am still feeling light headed today and feel like I could faint but have not. I did have chest pain but That seems to have gone away. Can a panic attack last off and on for days or should I go back to see my cardiologist again? Doctor: HiThanks for using healthcare magicI think, you have generalized anxiety disorder and panic attack. In that case, it could last for days. You need low dose antidepressant and benzodiazepine that would help to control anxiety as well as panic attacks. You can also try relaxation exercise like JPMR or deep breathing exercise. In case, you need further help, you can ask.Thanks"
},
{
"id": 158628,
"tgt": "Noticed little lumps on skin. Appear after shower nad then disappear. Family has history of cancer. Guide",
"src": "Patient: Hi, I am a 23 years old female. About 6 months ago I first noticed after a shower that these little lumps under my skin on my shoulder appeared. These are painless, and have only gotten a bit longer and thicker since the first appearance. They only appear after a shower or after putting body lotionon, for a few seconds or minutes and then disappear again. They feel swollen like lymph nodes. My family has a history for cancer, however after doing some research could these be parasites? Any help or direction with this would greatly be appreciated. Thanks Doctor: Hi, You have mentioned this lesion appeared after using some lotion then this may be due to hypersensitivity reaction. Little swelling at the shoulder which disappears sometime surely not related to any sort of cancer. At this region there are no major lymph nodes.Parasitic infection may be there but without clinical examination of the lesion it is difficult to comment. You should visit a dermatologist's office for better guidance."
},
{
"id": 47177,
"tgt": "Is sea diving safe post steroid treatment ?",
"src": "Patient: Hi, I am 24 years old and had membranous glomerulonephritis completed treatment with steriod and antihypertensives. Now taking only \"Diovan\" no problems any more.I am planning to attend a deep sea diving ciurse. Is there any problems or limitation with my condition? Thank you Doctor: Hi,Thanks for writing in.Taking steroids causes reversible side effects like weakness and increased tendency to fall sick. It is important to have a complete evaluation of your general health including blood counts to know if your immunity is good and to confirm the absence of infections. Considering deep sea diving, a depth of more than 80 to 100 feet is to be undertaken only after getting examined and taking a fitness permit from your doctor.Diovan contains valsartan and this is a good anti hypertensive medicine. People who have high blood pressure require a medical fitness permit from the doctor.In conclusion, a general health examination and medical fitness permit is required which you can get from your doctor attached to the deep sea diving institute. Please take precautions when diving deeper than 80 to 100 feet under water. Please do not worry."
},
{
"id": 9433,
"tgt": "What causes whole body itching and dry scabs?",
"src": "Patient: I ve been itching like crazy all over my body I could be sitting there and all of a sudden I itch. Ive been getting scabs for the last two weeks that look kind of like dry broken skin. Under my boob and my torso I usually where theyre located now its moving down to my legs this first started to happen when I felt kind of a big welt on my side im scared and dont know what to do I am not sexually active and have no diseases Doctor: helloThere is so many causes of generalized pruritus.Scabies is one of them.If there is night aggrevation of itching it can be scabies.treatment - permethrin 5% lotion to be applied all over body except face overnight nd take a lukewater bath next morning."
},
{
"id": 208207,
"tgt": "Suggest treatment for psychotic episode in woman",
"src": "Patient: I am a psychotherapist. I work in a crisis clinic. Recently a 50 year old Mexican National woman had a psychotic episode after her father (99 years old) died of natural causes. She has no family or personal hx of psychiatric illness and has worked and been basically healthy up to now. She stopped menstrating 2 months ago. Doctor: Hi, thanks for the query. Considering that you are well aware of the symptoms of psychosis such as reality distortion, delusions, hallucinations, formal thought disorder, behavioral disorganization etc. & presuming that the psychotic episode has occurred in response to stressor (loss of loved one); I would like to enquire whether the lady has fluctuating symptoms varying in intensity & severity over a day to day- hour to hour basis & whether their are predominant affective symptoms like emotional turmoil? I am suspecting this to be acute & transient psychotic disorder & for first episode of Psychosis; Risperidone/Olanzapine shall be my preference presuming there are no contraindications of health, lifestyle. Benzodizepine group drugs like clonazepam, lorazepam etc. can be used as adjuvents to reduce agitation. She should respond as early as within 1 week & may require drug treatment for next 6-8 months or so. Kindly get in touch with a Psychiatrist. Good Luck"
},
{
"id": 55842,
"tgt": "I have sharp sudden pain right above and around my navel inspite of having abdomal hysterectomy and cholecystectomy",
"src": "Patient: Hi, I am a 35 year old women. Have had total abdomal hysterectomy and cholecystectomy. I am having sharp sudden pain right above and around my navel. Also it radiates straight down to my pelvis with pelvic pain. Also pain during urination. Please help. Thank you. Doctor: Hi, Thanks for your question. After going through your question, I can understand your concern. The pain is most likely related to urinary system. Most likely a right ureteric calculus (stone in tube connecting kidney to urinary bladder). But other causes like pain of colitis should be considered. You should consult your doctor for detailed clinical examination. Depending upon it following investigations help in making appropriate diagnosis. - Complete Blood Count. - Urine examination.- Ultrasonography of abdomen and pelvis. - X-ray abdomen in standing position. Depending upon clinical examination and above test results further evaluation can be planned. Till that time use some pain killer with antispasmodic like diclofenac and Buscopan. Hope this helps you, if so do vote as helpful."
},
{
"id": 166733,
"tgt": "What is the treatment for swollen areola in a child?",
"src": "Patient: Hello, I have a 3 week old baby boy and the areola around his nipples on both sides seems swollen and hard... Is this something that will go away or should I be concerned and get his pediatrician to check this out? He does not seem to be in pain but it does not look normal. Doctor: Hello,It is the effect of the mother's hormones on the baby. These hormones pass to the baby while still in the mother's womb. The effect of these hormones will subside eventually, so no need to worry.Hope I have answered your query. Let me know if I can assist you further.Regards,Dr. Srilatha Myaka"
},
{
"id": 132484,
"tgt": "What causes left sided rib pain?",
"src": "Patient: I am a 40yr female. I have had sharp pains to my left side at the bottom of my ribs for some months now. There seems to be no trigger, this happens at all different times and during different activities, often when I m not even moving. I feel truly exhausted but often can t sleep. I never feel refreshed. I also keep getting a sharp pain at the top of my head on the right. I haven t been to docs as don t want to waste time when I don t have anything specific it s all a bit general. Any advise/idea? Doctor: hello thank you for writing us here.Your complaint suggest of chronic costochondritis which is the infection of the rib cartilage. A minor infection could be the reason caused by virus/ bacteria. i would suggest you to go for an ultrasound of your abdomen to know the cause and the treatment of your condition. Without investigations nothing could be done. You can still take paracetamol tablets to get relief from pain. I highly suggest you to see a doctor and get the treatment after proper investigations. Best regards,Dr Gunjan"
},
{
"id": 208705,
"tgt": "How to treat depression?",
"src": "Patient: I had a vaginal hysterectomy in Feb. 2014 . I m not the same person since. I have no appetite for life any more. I m on antidepressants and estrogen pills. Nothing is helping. On my days off I don t even want to leave my house. I want my life back. To be the person I once was. I ve gained 15 lbs since . I need some help and advice PLEASE!!?? Doctor: Hi dear,on which anti depressant you are?what are the symptoms you have?there are many mediaction now a days which helps you like SSRI(selective serotonin re uptake inhibitor), SNRI(serotonin and nor epinephrine re uptake inhibitor) NASA (nor epinephrine and serotonin antagonist).discuss above medication with your doctor.are you having thyroid problem?whether you test for thyroid any time?Thank you"
},
{
"id": 220576,
"tgt": "Suggest cure for urinary infection",
"src": "Patient: I m 26 years old n 26weeks pregnant, last week my urine was tested and there was a trace of protein...i was later prescribed amoxicillin caps which I started taking yesterday.. my questions are; (1)will it affect my baby in any way and (2)is it safe 4 me to still have intercourse with my husband? Doctor: Hello dear,I understand your concern.In my opinion when there is albumin in urine the high BP is to be ruled out.If the BP is normal there is nothing to worry.You are prescribed amoxicillin capsules may be for urinary tract infection(UTI).It does not cause any harm to the baby.And yes you can have intercourse with your husband.Usually intercourse is best avoided during the first 3 months and in last months of pregnancy.I just want to say a few things regarding UTI.UTI is the most common infection seen in pregnancy.It should be treated because untreated UTI cause irritation of uterus leading to preterm pains.And taking adequate fluids upto 3 litres per day prevent UTI.So I suggest you to take adequate fluids and also rule out any high BP.As such nothing to worry.Hope this helps.Best regards..."
},
{
"id": 205168,
"tgt": "How can anxiety be treated?",
"src": "Patient: Started taking an all natural herbal formula for my nervousness, worrying, and inability to relax, but wondering if this will react to Benicar hct, and lorazep? The supplement facts are Vitamin D 600 iu, thiamine B1 300 mcg, niacin B3 20 mcg, b6 15 mcg, calcium 21mg, magnesium 15 mg, and a proprietary blend of passion flower, GABA, KL-tryptophan, ashwagandha, root powder, ashwagandha extract, Bacopa monnier, L-theanine, green tea extract. please advise. thanks Doctor: U can carry on with the relaxation...aling with it take propranalol 10mgs in the morning time and take half tablet of clonazepam 0.25mgs when ever you feel very jittery"
},
{
"id": 919,
"tgt": "Does the follicular study report indicate the chance of conceiving this month?",
"src": "Patient: hello doctor, I m 30 yrs, married since last 2 yrs, earlier I had two spontaneous abortion of 6 wks. now i m on follicular study for past 2 cycles, this is my 3rd follicular study sitting. on 15th day my LO is 14.5*12mm, RO is small and ET is 7mm. please suggest me what to do next, if this cycle i couldn't conceive? Doctor: Hi, I think you should repeat a scan after 3 days. If follicle is more than 17 mm, take injection for rupturing the follicles. Be in contact with your husband for 2 to 3 days after injection. Take progesterone for next 2 weeks. Do a urine pregnancy test at home after that. You can try like that for 3 to 6 months. If it doesn't work then you can go for IUI. Discuss with your doctor regarding this. Hope I have answered your question. Regards Dr khushboo"
},
{
"id": 145741,
"tgt": "What does hemangioma on left lower side of L2 vertebra mean?",
"src": "Patient: On my lumbar MRI it is showing a hemangioma on the left lower side of the L2 vertebra on the right image of the L2 it is showing the hemangoima to be presenting superior at the center of the L2 vertebra body, does this mean that the hemangoima has grown throught he vertebral body? Doctor: HelloHemangioma on left lower side of L2 vertebra mean hemangioma in left side of L2 vertebra in lower region.It is present inside the vertebral body.Vertebral haemangiomas are most common benign vertebral neoplasms. They are usually asymptomatic and incidentally detected during imaging for other reasons.It usually doesn't require any treatment. Treatment is indicated if there is neurological deficits or severe pain.In symptomatic lesions,treatment options are radiotherapy,balloon kyphoplasty or transarterial embolisation.Take CareDr.Indu Bhushan"
},
{
"id": 59911,
"tgt": "Small child, bloated stomach. Liver test done, no blood disorder. Treatment?",
"src": "Patient: my son who is 2 years 9 months , he had bloted stomach for which we did do test and found his liver which is 13.5cm and spleen is 11.8 cm , we had already done thelesmia test and found no thelismia . but he is leading a very healthy life with no energy down at any given point of time . in test even the Portal veain is also clear - no blockage, doctor is suggesting biopsy of liver - i am confused kindly guide Doctor: hi, welcome to HCM I am Dr. Das There are many causes for hepatospleenomegally in a child including some congenital diseases. If all other tests like complete blood count, LFT and others, liver biopsy is suggested. Before going to do a liver biopsy, you have to do coagulation profile which indicates whether liver biopsy can be done or not."
},
{
"id": 115239,
"tgt": "What causes clots in the elbow, arm and neck?",
"src": "Patient: Developed pulmonary embolism following shoulder surgery. They found clots in my surgery elbow, arm, neck, and both lungs. I ve been on Coumadin and Lovenox injections for 9 days now. INR levels still not in therapeutic range (1.3). Doc doubled the Coumadin/Warfarin to 10mg for the past few days. Yesterday I noticed an itching on the inside of my wrist. Sometimes I see 5 or 6 small red dots, but mostly it s invisible. The area is always damp and noticeably different in temp from rest of my arm. I m concerned reading about Warfarin Rash/Necrosis . I also know that my PCP is going to wave this away as nothing because you can t SEE it. He has not yet demonstrated a knowledgeable grasp of Embolisms and Coumadin protocol so i dont trust him to address this properly. Is this serious enough to pay another visit to the ER tonight? Doctor: Hi, dearI have gone through your question. I can understand your concern. You have pulmonary embolism. Ypu have high chance of blood clotting. So you need blood thinner like warferrin as prophylactic treatment.Dosage is very important in warferrin treatment. You should take warferrin to maintain your INR between 2 to 3. If it goes above that then there is chance of bleeding. If it goes below then there is chance of clotting. So keep this thing in mind. Consult your doctor and set the dosage accordingly. Hope I have answered your question, if you have doubt then I will be happy to answer. Thanks for using health care magic. Wish you a very good health."
},
{
"id": 92585,
"tgt": "Had umbilical hernia, bumps around belly button, smelly fluid. Had a surgery. What is going on?",
"src": "Patient: I had an umbilical hernia and about a year ago, I had surgery. For the past few weeks I've had two bumps on my belly button, they're not big, but noticeable. There is also clear, smelly fluid coming out. I have some sharp pain around where the hernia was/still is. I don't know if I need it looked at right away or what to do. Doctor: Hi, welcome to Health care magic forum. You had umbilical hernia surgery, for the past few weeks you had 2 small bumps, with clear and smelling discharge. The causes may be the sepsis of the sutures in side, or may be the recurance of the hernia. Any how it important to consult your doctor for treatment, to prevent the recurrence, if it is not recurrence. Wishing for a quick and complete recovery. Thank you."
},
{
"id": 53909,
"tgt": "What causes elevated liver enzymes?",
"src": "Patient: my husband is 34years old.he is 103 kg and 179cm today he makes liver enzyme test and they are high but he didn't have any symptoms nor signs.is it possible because of overweight?we have made also tests for hepatitis A B and C and waiting the results.iam really worried Doctor: Hi.Thanks for posting query at HCM.Usually ALT or AST values higher than \"two times the upper normal limit\", is considered abnormal ( in some countries, ALT or AST values of more than 100 are considered abnormal). Value of AST or ALT greater than 85 or above maybe investigated further.Alcohol ingestion and \"obesity\" are common causes of elevated liver enzymes in developed countries. however, HepB and C should be ruled out.advice for fatty liver:- abstinence from \"Alcohol\" - LOW fat diet should be followed, AVOID junk food and beverages- decreased oil consumption (oily food)- NO red meat- green vegetables should be ingested daily- use lemon juice (lemonade) once in a day- reduce weight if overweight/obese-\"recheck liver enzymes after 6 to 8 weeks\" and/or ultrasound.any further questions are welcomed.hope to answer your concern.wish you good health.regards,Dr Tayyab Malik"
},
{
"id": 74451,
"tgt": "What causes pain in sternum after squeezing ribcage?",
"src": "Patient: after seeing a chiropractor for neck and back issues he did a bear hug on me...after seeing him and being adjusted I started having pain in the sternum and it was sore to touch the sternum I could hardly bend forward I wondered if the squeezing of the ribcage and sternum during the bear hugs caused costochondritis? Doctor: Thanks for your question on Healthcare Magic. I can understand your concern. Yes, tight hugging can cause costochondritis. And your symptoms are suggestive of the same. So apply warm water pad on affected areas. Avoid heavyweight lifting and strenuous exercise. Take simple painkiller like paracetamol and diclofenac. Avoid sudden jerky movements. Don't worry, you will be alright with all these in 7 days. Hope I have solved your query. I will be happy to help you further. Wish you good health. Thanks."
},
{
"id": 5099,
"tgt": "Delayed periods, frequent urination, gas, constipation, fatigue. On m2-tone and meprate. Possible pregnancy?",
"src": "Patient: I M trying to conceive ,my period was irregular so a gyn. suggested to me to take m2-tone and meprate for 10 days which i took.My last period was on 25th of may,and it is 28th of june and my period has not started yet and I have following symptoms -- frequent urination ,headche,backache,gastric problem,constipation ,fatigue.am i pregnant? Doctor: Hello. Thanks for writing to us. The symptoms that you have are non specific and can be due to pregnancy. But this needs to be confirmed with a urine pregnancy test available over the counter.I hope this information has been both informative and helpful for you. Regards, Dr. Rakhi Tayal ,drrakhitayal@gmail.com"
},
{
"id": 74506,
"tgt": "What causes shortness of breath while suffering from stomach flu?",
"src": "Patient: I went to the hospital day before yesterday for dehydration due to stomach flu. I am able to keep down water and gatorade and crackers and toast. 3 days ago I was sick from both ends(sorry for graphics) no more vomiting but diarrhea in the am today. I am continuing to have shortness of breath...is this normal? What should I do?? Doctor: Respected user , HiThanks for using Healthcaremagic.comI have evaluated your query thoroughly .* This is in relation with reflux of stomach acid due to stomach flu , electrolyte imbalance giving rise to short breath .* Recommendations for better recovery - Deep breathing exercises , YOGA- Remain ambulatory with walking in fresh air .- Avoid oily , spicy stuffs .- No smoking .Hope this will help you .Welcome for further doubts .Regards ."
},
{
"id": 37249,
"tgt": "How to remove thorn stucked in knuckle?",
"src": "Patient: On Friday I was pulling weeds and a blackberry thorn from a bush punctured my knuckle. Now it is stuck in my knuckle and is swelling up. I tried getting it out but it wont budge and now it s hard for me to even move my finger. What do I do to get rid of it? Doctor: Hi,It seems that thorn stacked in knuckle producing infection swelling and pain.consult your doctor and get this thorn removed.Afterwards go for regular dressing.Go for one antibiotic, anti-inflammatory medicine course for 3 days.Ok and take care."
},
{
"id": 68143,
"tgt": "Suggest treatment for red lump on tongue frenulum",
"src": "Patient: I found a large red lump on my tongue frenulum this morning. I cant remember hurting myself brushing or eating. there is no pain just red and swollen a little. should i get this checked. I remember seeing this in a picture regarding mouth cancer in the past? Doctor: Hi, dear. I have gone through your question. I can understand your concern. It may be some benign cyst like renula or other cystic lesion. You should go for excision biopsy and histopathological examination. Hope I have answered your question, if you have doubt then I will be happy to answer. Thanks for using health care magic. Wish you a very good health."
},
{
"id": 61282,
"tgt": "What does a lump on the forehead indicate?",
"src": "Patient: My 1 yr old fell down our basement steps yesterday, apps 10 wooden ones. He cried for a few seconds then he was fine. Bump to the forehead which is fine today, unstable walking, but he is just learning. No LOC or vomiting at the time of injury. Today he seems to want to nap a lot more then usual. When he does nap, he is easily aroused. Am I just being paranoid? Doctor: Hello dearWarm welcome to Healthcaremagic.comI have evaluated your query thoroughly .* The external swelling is hematoma , what is concerning matters is desire to nap more may warrant possible internal damage .* Strongly recommended to undertake CT scan brain to rule out any underlying brain insult .Hope this will help you for sure .Regards ."
},
{
"id": 54061,
"tgt": "Does high SGPT level lead to infertility?",
"src": "Patient: Hi Doctor,I am stephen, I am a 28 years old, As per my last test, I found that my SGPT value is 149 which is 4 times higher that normal as per the consultant. I married last year and trying or a child but she is not concieved. Do this Fatty liver cause any infertility doctor... thanks in advance... Doctor: Hi.Thanks for posting query at HCM.direct liver to injury does not lead to infertility BUT liver inflammation should be properly investigated.Rule out hepatitis B and C as a cause of increased liver enzymes.stop taking any medication that maybe injurious to liver.Usually ALT (SGPT) or AST (SGOT) values higher than \"two times the upper normal limit\", is considered abnormal ( in some countries, ALT or AST values of more than 100 are considered abnormal). Value of AST or ALT greater than 85 or above maybe investigated further.Alcohol ingestion and obesity are common causes of fatty liver disease.advice:- abstinence from \"Alcohol\" altogether- LOW fat diet should be followed, AVOID junk food and beverages- decreased oil consumption (oily food)- NO red meat- green vegetables should be ingested daily- use lemon juice (lemonade) once in a day- reduce weight if overweight/obese-\"recheck liver enzymes after 6 to 8 weeks\" and/or ultrasound.hope to answer your concern. for further queries, you may post a direct question to me via HCM portal.http://doctor.healthcaremagic.com/Funnelpage=askDoctorDirectly&docId=71638regardsDr Tayyab Malik"
},
{
"id": 58721,
"tgt": "Aneurysm on branching arteries to liver. Suggested ateriogram after getting stronger. Second opinion required?",
"src": "Patient: My uncle has a 2 inch aneuysm on one of the branching arteries to his liver . The doctor sent him home and told him that he wanted him to get stronger before they did anything. He told him to walk 2 miles a day and do at least 25 sit ups a day and come back in 6 weeks. then they will do an ateriogram to better determine the next step. This does not sound logical to me. What do you think. I think he needs a second oppinion!! Doctor: Hi and welcome to Healthcare magic.Thank you for the query.I dont think so. small arterial aneursysms ar oftenly treated conservatively and it is always advisable not to do surgery if there is no clear indications and if he has not symptoms. Such arteries rarely rupture and he should just do frequent follow up and see is there any progression in size.Wish you good health. Regards"
},
{
"id": 2442,
"tgt": "What are the chances of successful conception after undergoing treatment for TB?",
"src": "Patient: hi,i m 25 year old my right cornual is blocked bt left is visualized peritoneal spillage seen on the left side i get married before 18 months i got miscarriage my almost one month baby before 5 month & i am suffering from tb igG 241 i started treatment for tb before 5 month can i get pregnent i m so tensed pls doctor pls tell me the truth Doctor: Hi , If you had one pregnancy , your chances for one more is very high. TB damages inner lining of Uterus & tube. Maybe that's why one tube is blocked.Your Uterus inner lining where baby attaches may not be very healthy. So you aborted with reduced blood flow to baby.After you finish Medicines for TB, you can get pregnant, but will need lot of hormonal support to maintain a healthy pregnancy. Next time you get pregnant have an early scan to rule out pregnancy in tube alsoAll the best"
},
{
"id": 160652,
"tgt": "How should urinary and bowel movement retention due to pain be treated?",
"src": "Patient: Hi, may I answer your health queries right now ? Please type your query here... my daughter is with holding urine since this afternoon. she is saying that it hurts when she passes urine. she is also with holding bowel moment. please advice. she is 3 1/2 years old. Doctor: Hi,Is she having fever? Any history of increased frequency of urination preceding this? How was her bowel habits before- normal or constipated?Pain while passing urine usually occur when there is urine infection. Urine retention can occur secondary to severe constipation too.You can give her paracetamol or ibuprofen syrup 5ml. This will relieve pain in 1 hour. If she is not passing even after that, kindly take her to doctor to see what is happening. Urine infection will need antibiotics and constipation needs unloading of rectum with suppository, followed by laxatives for few days.Hope I have answered your question. Let me know if I can assist you further. Regards, Dr. Muhammed Aslam T. K., Pediatrician"
},
{
"id": 136261,
"tgt": "What causes left hand shoulder pain and numbness?",
"src": "Patient: I am the mother of a 2 month old baby , I do a lot of lifting the baby , rocking movements with the baby in my arms , it began as an uncomfortable pain in my left shoulder, now kind of gives me electric numbness kind if feeling at the area between my base of mech and both shoulders , a hot bath or hot water bag gives me temporary relief , but the pain or numbness returns back when I hold the baby etc .. I reckon this has happened after prolonged carrying / lifting baby in the same arm , please advice remedies.what could this condition be? Doctor: Hello, I have studied your case.It can be shoulder rotator cuff injury which may need MRI for confirmation.Medication like methylcobalamine with muscle relaxant and analgesic will reduce pain, you can take them consulting your treating doctor.You may consult physiotherapist for further guidance. He may start TENS ,or ultrasound which is helpful in your case.You may need to take rest for early healing.Also check your vit B12 and vit D3 in your blood.Hope this answers your query. If you have additional questions or follow up queries then please do not hesitate in writing to us. I will be happy to answer your queries. Wishing you good health.Take careRegards Dr Vaibhav"
},
{
"id": 73722,
"tgt": "Suggest treatments for shortness of breath and stomach burning",
"src": "Patient: i am 45 years old. i have blood pressure,allergy,gas trouble,isnophelia,and body pains. for this i use different different pills for these diseases.but now i am getting problem in breathing.i am feeling sleepiness and burnings in my stomach.once i got kidney infection also. i am feared of eating medicines because i have used so many medicines and suffering so many problems because of that like confusion, memory loss, weakness and etc....how i can recover well from all these problems.what type of checkups i should do? and what precautions i have to take..... Doctor: Respected user , HiWarm welcome to Healthcaremagic.comI have evaluated your query thoroughly .* Guidelines for all remedies - Always maintain hydration levels with plenty of liquids .- Soft , light diet .- Avoid oily , spicy , non veg , hot beverages .- Quit smoking or alcohol consumption if doing .- Regular walking in fresh air , deep breathing exercise , YOGA .- Have adequate sound sleep of 8 hrs .- Keep your BP , blood sugar , cholesterol levels under control .Hope this will help you for sure .Regards ."
},
{
"id": 212280,
"tgt": "Fatigue, no social interaction. Depression?",
"src": "Patient: How can I know if I'm depressed? And what should I do if I think I am? I'm a 21 year old Swedish male who've just gotten enough of being tired, I've barely went outdoor at all the past year or so, except for buy groceries and stuff, and even that I can find myself without mostly of the time. The times I've seen my friends the past year is to be counted on 2 hands. Doctor: Hello You have symptoms like easy fatiguability, poor social interaction, less number of friends. If these are only symptoms present then it can not be labelled as a depressive disorder. But depression in milder from may present with lack of energy, easy fatiguability, pervasive sadness of mood, sleep dysfunction, poor or increased appetite, pains in body etc symptoms. Still if you want to know about your symptoms you can visit a Psychiatrist for evaluation. Thanks Dr. Seikhoo Bishnoi, MD"
},
{
"id": 128363,
"tgt": "What is the significant level of ESR?",
"src": "Patient: please and thank u doctors----29 years old female with thrombcytopenia 8 months ago that developed after last labor--diagnosed as slE-- she is on cortico steroid theraby ---she developed intractable bone pain and ESR 1st hr----50 2nd hr-----85 is this significant level? Doctor: yes it is significant. esr is a prognostic tool. intractable bone pain, in a 29 year old female with significant esr is to be thought,can you please provide where is the pain, does she have fever,since when was she ill"
},
{
"id": 1922,
"tgt": "Suggest remedy for problem in getting pregnant",
"src": "Patient: I have one child. I was planning to concieve my second since in March 2010. I noticed I had a fibroid which was 10cm*9(intramural).. I had a surgery in July 2010 and been trying from December 2010.During operation i asked her if it was near fallopian tube and she said no.My doctor put me on 3 months duphaston. After that she did 2 rounds of ovulation using fertyl. First round it showed 10mm follicle on both sides on day15.The second round she put fertyl for 7 days(CD2-CD9) and I seemed to ovulate by 13thday which scan said. She then said to meet a infertilty doctor. On first visit she said I have bilateral PCOS. I have not proceeded with any treatment.I am curious to know what happened when fertyl dosage was increased and should I do HSG and then proceed with PCos treatment. Doctor: Hi I think you should go for HSG first to see condition of your tubes. If normal you can try fertyl tablets again. Sometimes higher dose is required in PCO patients to make ovulation happen. This might have happened with you."
},
{
"id": 81009,
"tgt": "Does positive QUANTIFERON-TB GOLD test confirm TB positive?",
"src": "Patient: I have undergone MONTOUX test it resulted 35 mm reaction.After this i had taken QUANTIFERON-TB GOLD test result is positive. My chest X-RAY is normal (as per doctor). I dont cough, My doubt 1. Is QUANTIFERON-TB GOLD test reliable,do i have TB? 2. How to use FORECOX tablets..doctor said one in the morning and one in the evening. Please kindly answer my questions.Thanks in advance. Doctor: Thanks for your question on HCM.I can understand your situation and problem.Quentiferon TB gold test measures IgG antibodies against mycobacteria tuberculosis (MTB). So positive test does not always means you are suffering from active TB disease.It is positive in patients who are living in TB endemic areas,had past TB infection etc.So just on the basis of positive quentiferone TB gold test, one can not diagnosed as having tuberculosis and should not be started treatment.Your chest x ray is also normal and you are not having coughing,so no role of positive quentiferone TB gold test in you. And thus you should not take anti TB drugs.So, I advice you to consult pulmonologist and discuss all these before starting anti TB drugs.And forecox is anti TB drug. It should be taken simultaneously. So don't take in morning and evening. Take both tablet in the morning.But first consult pulmonologist before starting forecox."
},
{
"id": 143729,
"tgt": "What are the symptoms for asperger s syndrome?",
"src": "Patient: Hello, am a female, 5 3 , 145 lbs, and 21 years of age. I feel that I have asperger s syndrome. I have always felt normal for me but also different, awkward, and maybe even sad. I am reluctant to seek help because I am not sure weather I really do have asperger s or if I m convincing myself. What do you recommend I do? Doctor: Asperger syndrome is type of autism disorder .It is a disorder in which there is poor social interaction,language problems and behavioral problemd.It is managed by psychiatrist.You may be having .Advised to take psychiatry consultation."
},
{
"id": 151493,
"tgt": "Tingling and pins and needles sensation in arm. MRI done. Prescribed amitriptyline. Suggestions?",
"src": "Patient: I am a 66 yr old female and I have been having tingling and pins and needles down my left arm intermittently for the last 8 months. I have been to a chiropracter and an osteopath, have had neck and back x-rays (which just showed wear and tear), and been to a spinal specialist mainly to get an MRI scan which he did not think I needed. ( I also had sciatica last year which I still get in my left leg at night.) I can get rid of the tingling in my left arm by turning my head to the right or raising my left arm above my head. My GP has prescribed Amitriptyline to help me sleep and I have also had anti inflammatory tablets. Any suggestions or advice or exercises I can do to help would be greatly appreciated. Doctor: Hi. at your age it is common to have changes of wear and tear on X-rays. Some times these changes of aging bone can cause compression of the nerves which pass between neck vertebral bones. MRI scan may definitely help in pin pointing the location of compression. Neck collars to restrict movements at the neck and provide support may be useful. Use of hard bed with soft thin pillow may be helpful. Physical therapist can teach you some neck exercises. Good luck."
},
{
"id": 129816,
"tgt": "What causes pain under the right rib?",
"src": "Patient: i am having a pain jus under the right rib. it somewhat pains or giving some uneasy when i am breathing. is this is a ulcer prob.? or any symptoms for any other problem. is it good to take gelusil tablet for this.? whether this will give recovery for my symptoms.? Doctor: Hello!In my opinion You shouldnt use Gelusil on Your own idea!Visit primary doctor first, the causes can be related to other organs.Ultrasonography of abdomen (gall bladder) is useful"
},
{
"id": 58703,
"tgt": "Itchy rash and small open sores all over body. History of Probable RA and Humira. Liver enzymes elevated. Suggestion?",
"src": "Patient: Diagnosed 2-3 yrs. ago with Mixed connective disease/probable RA - on Humira blood work came back last week with elevated sed / crp / ast / alt ( liver enzymes) will do blood work again in 4 weeks however - just came back from long weekend in FL, broke out in very itchy rash of small open sores all over hands, arms, lower legs?? Doctor: Hello! Thanks for putting your query in HCM. I am a Gastroenterologist (DM).Development of rash over the body and elevated liver enzymes mayall be the side effect of Humira itself. Please consult your doctor regarding this.I hope I have answered your query and this will help you. Remain in touch and get-well soon."
},
{
"id": 165676,
"tgt": "What causes purple spots on the scrotum in a child?",
"src": "Patient: Hi, may I answer your health queries right now ? Please type your query here... My son has purple spots on his scrotum. It started when he was in 7th grade with a small one. Two years later, the small one has grown to the size of a dime and he has two. Any idea of what it is? Doctor: Hi...Thank you for consulting in Health Care magic. Skin conditions are best diagnosed only after seeing directly. I suggest you to upload photographs of the same on this website, so that I can guide you scientifically. Hope my answer was helpful for you. I am happy to help any time. Further clarifications and consultations on Health care magic are welcome. If you do not have any clarifications, you can close the discussion and rate the answer. Wish your kid good health.Dr. Sumanth MBBS., DCH., DNB (Paed).,"
},
{
"id": 131941,
"tgt": "Does loud cracking sound behind my knee while exercising mean a knee sprain or tear?",
"src": "Patient: Hi Doctor, Thank you for your time. Approx. 3 weeks ago I was doing a circuit training in the gym and while jumping sideways I felt and heard a loud crack behind my knee. Suddenly I was unable to continue and was in pain. I struggled to walk on this leg. Three weeks later the pain has eased, am able to walk but still feeling pain and weakness in my knee. Was wondering if perhaps I have a tear of some type or just a knee sprain? Doctor: Hi I have seen your injury description . In my opinion you have most likely injury of Posterior Cruciate ligament of the knee .Second possibility is hamstring strain. In post Cruciate ligament injury the knee gives problem in future while hamstring injury normally heals well. I suggest you get an MRI Knee done to R/O Post Cruciate ligament Injury."
},
{
"id": 208456,
"tgt": "Does coconut oil help in improving the memory?",
"src": "Patient: does taking coconut oll help improve memory My husband and I are 75 and 77 and noticed we tend to forget things more then earlier.. we are in fairly good health and are active in our church and community. I (75)still work and am a GED instructor and site computer operator. Husband is retired but mows yards and helps with everything he possibly can for friends and neighbors, even in triple digit heat...It is frustrating to forget where we put things and are not able to find them....I read an article on the internet that Coconut oil could help with memory loss....Is this for real??? Doctor: DearWe understand your concernsI went through your descriptions. I suggest you not to worry much. Very glad to converse with people like you. God has bestowed all kinds of blessings on you and I see god through people like you. God is there. Available, taking care of needy. Madam, mild memory troubles are a common feature of the old age people across the world and is due to degeneration. But I am sure that you are immune to that type of memory loss. I don't see any memory loss. What ever you forgetting, every people are forgetting. Means you are experiencing a type of memory loss which every one have. Busy people, working people, family people, even idle people forget. At this age, if you can remember ninety five percent, you have the strong memory power across the world. I guarantee it.Hope this answers your query. Available for further clarifications.Good luck.We understand your concernsI went through youVery ou which should clear your problems within weeks.Hope this answers your query. Available for further clarifications.Good luck."
},
{
"id": 173749,
"tgt": "How to get rid the foreign object that is swallowed?",
"src": "Patient: My child swallowed a foreign object 2 weeks ago. It is still in him. The drs don't know what it is. They said to watch and wait. Yesterday his left eye swelled shut and then last night the right eye did the same thing. Can it be related to the object in his stomach Doctor: Hi.....I don't think the swelling of eyes is related to the object that's swallowed. The alternate swelling of eyes could be due to his position during sleep. But if the object swallowed is high in lead content due to its coating its not good. If it's already 1 week then I suggest consultants a pediatric gastroenterologist and getting it removed endoscopically.Regards - Dr. Sumanth"
},
{
"id": 28422,
"tgt": "Is it safe to take water pill and hydro chloride for high BP?",
"src": "Patient: I just started buspar 15 mg twicedaily, morning and night for anxiety.I started it only two days ago. My heart feels like its beating differently a little faster and I feel slightly dizzy......i havehistoryofhigh blood pressure controlled by meds that work good and keep me and my physician happy. Are these symptoms because i new to this med.? I don t take any other meds...except hydrochloride and a water pill to control high blood pressure....enalpril..should I continue or stop? Doctor: HI Thanks for posting your query. Yes the symptoms ( Faster heart beat, dizziness ) are the side effects of the tablet Buspar. But these side effects wear off on continued usage of the medicine. My advice for you is to take half the dose of Buspar for a week and then increase it to full dose . Your symptoms will reduce and disappear once you follow this regime. Also continue Enalapril and the water pill . Hope this information was useful to you . Any clarification feel free to ask."
},
{
"id": 117463,
"tgt": "What causes elevated blood homocystiane?",
"src": "Patient: dear docter my doughter saniya 7yrs 8months old. today i tested blood homocystiane level test. Her blood level result is 69.56 umo1/L. Last five year she take tablet epilepsy sydrome or a diffuse brain disorder. please sugest me what can i do. thanks. Doctor: For elevated blood homocysteine, the good treatment is give her supplement of Folic Acid 5 mg. It will help to control the homocysteine level. Also being a vitamin only, it will have least side effects."
},
{
"id": 40371,
"tgt": "Is it water infection to have smelly urine?",
"src": "Patient: Hi, my son is nearly 19 months old. Over the last few days I have noticed that my son has very smelly urine, he is drinking plenty and I am getting a lot more wet nappies off him. He is also crying a lot which can be very tiring! I have given him Calpol as a precaution. Could this be a water infection? Doctor: HII really appreciate your concern for your son this is not the infection, if the urine being left over in nappies then it could start stinking badly, but still if you have doubt, get the test done for urine examination this will clear the doubts, without the indication never ever give the Acetaminophen (Calpol) hope this information helps you, have good time."
},
{
"id": 178898,
"tgt": "What causes dry lips with purple color?",
"src": "Patient: hi my two and half yr old lips has been dry and purple looking for the past 2 weeks we visited our family physician and he did the basic check and said everything seems fine, that is with his heart and lungs but to monitor it, might be something he ate. Can i get your advise? Doctor: Bluish discolouration of lips occurs due to heart , lung or blood problems. If the bluish discolouration continues you should get an Echocardiography, blood for Methemoglobinemia done. If the problem would have been in the lung the child would have respiratory distress which i think is absent. hope this helps."
},
{
"id": 199762,
"tgt": "Can masturbation causes non motility of sperms?",
"src": "Patient: Hi, i am Male 34...i have been masturbating since last ten years almost twice a day.....Now married since last 4 yrs...my semens analysis shows less motality( total 30-35%),count being fine....can masturbating be the reason for this non motality? we are planning for baby...pls advice... Doctor: HelloDecreased motility is not related to masturbation.Low sperm motility may be due to many reasons.You need proper clinical examination and investigations.Investigations include routine hemogram,random blood sugar,ultrasound of scrotum.It is important to exclude any surgical reason for low motility.You should do exercise and take nutritious diet.Avoid smoking and alcohol if you take it.Take anti-oxidants also.It also helps in increasing motility.If low motility persists then assisted fertilization techniques like IUI can be considered.Get well soon.Take CareDr.Indu Bhushan"
},
{
"id": 200943,
"tgt": "What causes circular dry spot on foreskin of penis?",
"src": "Patient: Hi I have a circular dry spot on the foreskin of my penis. With no pain, discharge, ulceration, and no tenderness. I Had unprotected sex with a relative stranger 6 months ago. Did all the std testing and everything came back clean, including follow up blood testing. However in Canada we have no blood test for herpes. Because of this I am quite concerned. Could it be from masturbation? I am quite regular at 2-3 times daily. Always dry masturbation as well. Doctor: Thanks for asking in healthcaremagic forumIN short: Dryness could have caused this.Explanation: If you are negative for all STDs then be happy. You need not go for testing herpes. You would have got pain if it was. Masturbating whenever excited is not bad but should not become like habit(like 2-3 times a day). Try to divert your mind for some other activity. Good luck"
},
{
"id": 81768,
"tgt": "Can dyspnoea cause risk to heart?",
"src": "Patient: im 36 had surgery for pectus excavatum when i was around 5 yrs old. i am verry short winded. im concerned about my heart and lungs, i get worn out very fast. i cant even keep up with my children. i cant play with them and have to make excuses because i get winded so fast. i push my self to keep up, but im afraid my heart is gonna exploid. will my heart last? Doctor: Thanks for your question on HCM. By your history I think you are having uncorrected pectus excavatum. It Is a kind of rib cage anomaly. With age this anomaly can cause restrictive lung disease. In this lungs are restricted to expand due to rib cage anomaly. So lungs become weak and it further damage heart.So right sided heart failure with cor pulmonale and pulmonary hypertension can be seen in these patients. Better to consult pulmonologist and get done PFT ( Pulmonary Function Test ) and 2D Echo to rule out all these. Start chest physiotherapy and deep breathing exercise."
},
{
"id": 45278,
"tgt": "suffering from azoospermia",
"src": "Patient: hi, i am 32 and recently came to know that i suffer from azoospermia. is there a treatment, how the cause or type of azoospermia which i have can be properly diagnosed? Doctor: hi, welcome to health care magic azoospermia are two types 1.obstructive 2. non obstructive in obstructive types there will not be any sperms in semen but it will be present in testis den you can do ICSI(intra cytoplsmic sperm injection).but if it is non obstructive then sperms will be absent both in semen & testis.there wll not be any treatment for this,then it is better to go for an early adoption"
},
{
"id": 98761,
"tgt": "What could pressure in chest and swelling in tongue indicate?",
"src": "Patient: I have a DVT factor 8, had this condition for 11 years. I have had pressure in my chest. I have a greenfeather filter. Today I went to the emergency room. They did test and stated after testing INR result was 0.97 low. No clots. They upped my coumadin after being on only 2.5 to 5.0. That was Wednesday. MY INR has not raised, I am worried about why it has not raised. I was sent home told to contact my doctor on Monday. Nothing was given to me to raise it. They did cat scan and testing and found nothing including a blood clot. I have been on 2.5 Mg. for a year. I have not had my INR checked until two weeks ago because the doctor I use to have said it was ok for six months to not have it checked. My new primary said that was not correct. Two week s ago I had a blood clot that the filter did catch. Was taken to Freeman Hospital where they did test, and even a echo of my heart, while doing a stress test. They said it was heart burn!!!! My primary said that was ridiculous that it sounded like I had passed a clot that the filter did catch. Never had this close of a call. Today the right side of my tongue swelled for no reason. The emergency doctor said it (the tongue swelling) was probably something due to an allergic reaction. I am not allergic to anything!!! They found nothing sent me home saying to talk to my primary. I am 42 years of age female never smoked, been in great health. Now these items are coming up. I am scared these doctors are not finding what is wrong even with my condition. Why would my right side of my tongue swell during this? Doctor: Hello,I can understand your concern. It seems that the swelling of tongue that you have developed has nothing to do with the DVT and pressure in chest you are having. I agree with your doctor that it seems to be an allergic reaction. I would advise you to take Antihistamine such as Levocetrizine 5 mg (Levocet) or Loratidine 10 mg (Lorfast) once a day for at least 5 days. If it would be an allergic reaction, you should find immediate improvement in the swelling in 1-2 days. Otherwise, the cause is not allergic. The allergy could be due to anything that you have been exposed to through air, water or food. If the episodes are frequent, you should go for a blood test for allergy to find out what substance you are allergic to and you can avoid exposure to that in future. However, if the swelling does not decrease with the antihistamines or if it increases, then you should consult a dentist to examine the oral cavity and investigate further regarding the cause.I hope this information helps you. Thank you for choosing HCM. I wish you feel better soon.Best,Dr. Viraj Shah"
},
{
"id": 109218,
"tgt": "Suggest treatment for headache and back pain",
"src": "Patient: I ve been having headaches everyday for over a month now. I think I may have a swollen lymph node of the left side of the back of my head. Only for a couple days I ve noticed when I pee it burns and was painful the other day. Today it just burns without even peeing. I ve always had really bad back pain mainly in my lower back. My last two periods have always been really adnormal. Last moth it was a week late and when it came it was really bright red blood and really heavy lasted over 5 days. This month it lasted 2-3 days and was very dark but very little bleeding the whole time. I didn t see any red which is very uncommon for me. I ve also had pains in my lower stomach as if I was on my period but I m not. Today I had a really sharp pain in my lower stomach area and had to bend over it was pretty bad. I m really wondering what this could mean? And if I should see a doctor? Doctor: Hi,1, headache might be due to enlarged painful occiputal lymph node giving headache.This might be due to having some infection on the scalp.2, having pain in the back and abdomen might be due to having acute urinary tract infection.Go for one antibiotic medicine course for 5 days which might cure your both the problems.TaKE PLENTY OF WATER."
},
{
"id": 210061,
"tgt": "What are the symptoms of strokes?",
"src": "Patient: my mother had a stroke 2 years ago and has been getting very tired lately, sleeping alot, and at times just staring off in to a trance or some thing. we put her to bed and after she wakes up she is better. She hasn't been eatting as much lately either. Could she be having miny strokes. Doctor: Hello,Thanks for choosing health care magic for posting your query.I have gone through your question in detail and I can understand what you are going through.There is a possibility that multiple lacunar infarcts can lead to these symptoms and especially the fact that your mother has had a stroke in the past puts her at a higher risk. The other possibility is that these infarcts might have led to cognitive impairment and dementia which can also present with similar features. If it is dementia then there has to be significant impairment in the memory.You need to take your mother to a psychiatrist or a neurologist who can order and MRI and also assess the cognitive functions using MMSE. That will give a better idea. One thing is clear that there is no need to panic and things can go on their own pace. Hope I am able to answer your concerns.If you have any further query, I would be glad to help you.In future if you wish to contact me directly, you can use the below mentioned link:bit.ly/dr-srikanth-reddy\u00a0\u00a0\u00a0\u00a0\u00a0\u00a0\u00a0\u00a0\u00a0\u00a0\u00a0\u00a0\u00a0\u00a0\u00a0\u00a0\u00a0\u00a0\u00a0\u00a0\u00a0\u00a0\u00a0\u00a0\u00a0\u00a0\u00a0\u00a0\u00a0\u00a0\u00a0\u00a0\u00a0\u00a0\u00a0\u00a0\u00a0\u00a0\u00a0\u00a0"
},
{
"id": 187102,
"tgt": "Is the pain all around the ear and down to chin related to arthritis?",
"src": "Patient: Over 20 years ago I had broken jaw, complete break. Plates and wires put in. Teeth wired for 6 weeks. For the last year if I lay on that side, even for a few minutes I have horrible pain all around the ear and down to my chin. It subsides after awhile but the whole side of my face stays very tender to the touch. I am 65 now. Can plates shift or could it be arthritis caused from plates. If I fall asleep on that side for to long, as soon as I lift my face the pain is horiffic for about 5 minutes and then whole face is so tender,it hurts to try an eat for hours!!7 HELP!!! Also, the pain covers the whole side of my face.Thanks Doctor: hello thanks for consulting at hcm..it could be either infection of bone in that region or it could be nerve problm,, either ways get an iopar, opg and cbct..plz consult a oral medicine/ oral surgeon specialist at the earliest,, u can take analgesics ,,if problm persists plz take a consult with ur specialist,,hope it helps,,tc"
},
{
"id": 93550,
"tgt": "Have pain in abdomen and vaginal discharge. Took medication but no use. What to do?",
"src": "Patient: i am experiencing sharp pain in my lower right abdomen and in my right shoulder which aches whenever i inhale deep. my urinalysis and blood test says that i have high bacterial rate, i also have yellowish vaginal discharge and i can't bear the pain anymore. what should I do? please help me. the meds that the doctor prescribed arent taking the pain away. please help Doctor: Hi thanks for posting your query on Healthcare MagicYour complaints and investigations are suggestive of urinary tract infection, that can be treated with antibiotics but I would suggest you to get urine culture after discussion with your treating doctor and take antibiotic as per his/her advice based of culture report. This infection can spread and lead to pelvic infection and result into vaginal discharge, should be evaluated by gynecologist.Please make sure you should continue treatment as per doctor adviceFor your lower abdominal pain there is possibility of stone in urinary tract again require clinical examination and further imaging can be done if requiredI hope my advice may be helpful for you. Take care"
},
{
"id": 174328,
"tgt": "What causes constant goose bumps in skin?",
"src": "Patient: My 2 year old daughter is warm but has goosebumps.. I have covered her with her blanket but she still continues to have goosebumps a half hour has passed.. she is sleeping.. her breathing seems to be normal she will snore at times.. she is asthmatic.. I am very paranoid when it comes to her.. I am afraid I will miss something to do with her asthma.. Doctor: HiYour concern is genuine But believe me there is no connection of these two things asthma and goose bumpsIf she is asthmatic just ensure that she is not having fever and as you mentioned that her respiration is normal look for some other cause of feverIf it is possible get her urine examination done as girls are always at risk of urinary infection and may go unnoticed if at sub clinical levelI hope u got ur answerWelcomed if need further clarification."
},
{
"id": 149072,
"tgt": "Heavy head and tongue, neck pain, nausea, vertigo. Want expert opinion",
"src": "Patient: Hi Dr Mintz I am Jasmine. I am about 38 years old and at present going through some health problem. For the last 2 weeks months I have a weird feeling. My head suddenly starts getting heavy and I feel my tongue is also getting heavy and feel it being pulled back. It all starts with slight pain that starts from the neck region. Before this my problem started with neck pain , heavy head with nausea and vertigo . After medication the vertigo part and nausea part is better but for the complaint that i have mentioned above. Request your expert opinion. Best regards Doctor: MAI I KNOW YOUR PROFESSION PLEASE?.BECAUSE MOST SOFTWARE COMPUTRER PERSONNEL HAVE H/O PROLONGED SITTING BEFORE THE COMPUTER WHICH LEAD TO STRAIN OVER THE PARASPINAL MUSCLES OF THE NECK LEADING TO WIERD FEELING.GET A X RAY CERVICAL SPINE DONE N GET CHECKED FOR CERVICAL SPONDYLOSIS."
},
{
"id": 203067,
"tgt": "Any suggestion for weak erection with quick discharge?",
"src": "Patient: Hi, my name is jaan from UAE. i am 37 years old married person. i have some weakness in my sexual life. 1st my pines size is 4.5 inches. 2nd, my pines is not come hard during intercourse. 3rd, i discharge very quickly. please give me some advise to get my site back. thanks Doctor: HelloThanks for your query,based on the facts that you have posted it appears that you have problem of Erectile Dysfunction and Premature Ejaculation .In absence of any major health issue cause of ED and Premature Ejaculation is mind related and due to what is a called as Performance anxiety.Following measure will help you to boost up your confidence and getting good erection.and delay ejaculation.1) Practice regular exercise for 45 minutes followed by meditation for 1/2 an hour in the morning.2) Take high protein diet rich in vegetables and fruits and Vitamin A,C,D,E.and Zinc3)Take anti oxidants like Almonds 5-6 everyday..4) Avoid alcohol and smoking..Taking medications Sildenafil( Viagra ) on demand will help to get good hard erection and delay ejaculation .Please consult your family Physician to get prescription and to know timing and dosage of the drugs to be taken.Dr.Patil."
},
{
"id": 210687,
"tgt": "Could difficulty in paying attention, lack of concentration, forgetfulness and trouble organizing be due to ADHD?",
"src": "Patient: Hi there, I have trouble regulating my attention/concentration on a task, and I constantly shifting from task to tasks without ever finishing the task I set to do. I have trouble organising things and keeping in touch with time. I m always forgetful and never have quite managed to get things done since I procrastinate and always distracted. I suspect I have ADHD inattention. Doctor: HIThanks for using healthcare magicThe way, you have explained your symptoms, it is towards ADHD, but treatment depend upon the severity of problems. In case of mild symptoms, there is no need of medication. If you are feeling, ADHD is causing problem in your day to day activity, then better to consult a psychiatrist for proper medication.Thanks"
},
{
"id": 15978,
"tgt": "Taking prednisone for itchy rashes. Swollen lymph nodes, pain in the neck. Need to be concerned?",
"src": "Patient: Im on prednisone for a rash , Ive also been on anti biotics (z pACK) I itch all over and have swollen lymph nodes..on my neck, i get achy and seem to get cold much more then I ever have.I was on prednisone and a week later the rash came back and I was very itchy..went back and was put back on prednisone again. should I be worried Doctor: Hi and thanks for the query, Itchy rashes and lymph nodes around the neck region might most of the time be due to viral infections. It could be necessary to actually trace what is causing the rash. A history of a recent change in the environment, diet, sleeping room, job site should worth considering. Other accompanying symptoms like fever, wheezing, rhinitis are important to consider to know if its an allergic reaction or not. An initial response to anti biotherapy could indicate an infectious cause. Persistence of symptoms should warrant an appointment with the dermatologist for a proper evaluation. Thanks and best regards, Luchuo, MD."
},
{
"id": 32227,
"tgt": "Suggest treatment for severe typhoid fever",
"src": "Patient: Hiii doc..my mother hv typhoid she is in icu..she has never any improvement by 4 days in the hospital.she suffer the typhoid from the 6or7 days.she has vry complicate situation...what cn i do for her....every doc try there best but....i am helpless to my mom Doctor: You have not mentioned what were the tests done to confirm typhoid. Widal alone is not sufficient. Please check with your doctor if \u2018blood culture\u2019 was done. This tests not only confirms typhoid, but also provides details on the list of antibiotics that are capable of working against Salmonella bacteria. Instead of administering any antibiotic, it is crucial that only the antibiotics that were sensitive are administered. If you do not have the time for this, at least check if I.V Ceftriaxone is on the drug list. Please also watch out for any gastrointestinal and other complications that can occur due to typhoid."
},
{
"id": 77065,
"tgt": "Suggest remedy for chest pain",
"src": "Patient: I have had intermittent chest pain for the last 3 days. This morning I have developed a slight wheezing sensation and when I try to take a deep breath it feels like my lungs are freezing and the deep breath doesn't seem to work. The chest pain seems to go across my chest and ends in my left arm pit. I do also have a swollen Lymphnode under my arm as well. Help! Would rather avoid the hospital but I am getting nervous. Doctor: Hi I can understand your concern...You are complaining about wheezing with chest pain..So here there might be some obstructive cause...First rhonchi confirmed by auscultation ..Then for confirmation of asthma or chronic obstructive lung disease pulmonary function test done with spirometry.....Chest x ray also useful to rule out obstructive mass if present...If it is present biopsy taken....I am suspecting this as you have enlarge lymph node....According to cause your treatment guided ..Like bronchodilator and steroid like drug.You can keep all these in mind and consult doctor without worry for your respi examination.You can take further opinion from HCM by sending follow up report....Dr.Parth"
},
{
"id": 2919,
"tgt": "What are the chances of getting pregnant by taking plan B after sex?",
"src": "Patient: Hi, Long story but to make it short, I had my period on June 14 through the 18th. Never on birth control; Had sex on the 27th. Boyfriend ejaculated in me three times. Pretty much unprotected sex. (The sperm might be very strong because my boyfriend hasn t been sexually active for about 2 weeks. ) Took Plan B on the 28th. After 24 hours..What are the chances of me being pregnant? No side effects yet but my vagina is a little itchy. Please HELP! Doctor: Hi,I understand your concern. You have unprotected sex in fertile phase of period. You took plan B pill after 24 hours. It has up to 85% effective in preventing pregnancy. It has 0-15% failure rate in your case. But pill can cause earlier period, delay in period by 1 to 2 week and excessive bleeding can occur. I would suggest to wait for period. If period will delay by more than 2 weeks then go for urine pregnancy test and or blood HCG test to confirm pregnancy. If you will be pregnant then you can still terminate intrauterine pregnancy by abortion pill under advise of gynecologist. Meanwhile avoid stress, take healthy diet, drink plenty of water and do regular exercise.Hope this may help you. Contact further if follow up needed.Best regards,Dr. Sagar"
},
{
"id": 56043,
"tgt": "Suggest medication for chronic hepatitis B",
"src": "Patient: Can i take liv 52 to cure hepatitis B chronic i find it from past four years Are is there any medicine to get rid from this disease And my life partener is not affected with hepatitis b so is there any booster medicine that can make live together. Plz reply Doctor: Hi there,Thanks for posting in HCM.Liv 52 does not cure hepatitis B.In fact there is no medicine that can completely eradicate hepatitis B infection.There are antiviral medications like tenofovir, entecavir etc which can keep the infection under control.Your partner can get vaccinated with three doses (0,1 and 6 months) of hepatitis B vaccine and she will protected from hepatitis B.Regards."
},
{
"id": 715,
"tgt": "What causes weakly positive pregnancy test?",
"src": "Patient: i was diagnosed as weekly positive for pregnancy earlier this month.however when my beta hcg was tested it was very low.first time it was 63 miu/ ml and the second time it went down to 22miu/ ml when i went for an ultrasound the doctor said that he could see an intra uterine gestation sac.i am really confused as to to what is happening because my gynaecologist said that nothing is visible on an ultrasound until the beta hcg reaches above 1000.also my tsh was 7.92. my home pregnancy test was negetive today.my lmp was 9 april and i was spotting from 23 april to 18 may .please help Doctor: Hi, Thanks for the query. I understand your concern. Weakly +ve pregnancy test with decending values of beta hcg levels &continuous spotting for about 21days sstill usg showing a gestational sac. indicates a blighted ovum missed abortion. As the hcg level is decreasing.. it's clear that pregnancy is not viable &you should go for evacuation of uterus to remove whatever remnents are still inside &is causing bleeding&weakness. Thanks."
},
{
"id": 71602,
"tgt": "Could the pain near the heart be due to discontinuation of TB treatment?",
"src": "Patient: Hi,a may I answer your health queries right now ? Please type your query here... am a tb patient. had medicines only for 5 days that to interrupted noe its been 2 months not taking any medicines so is my tb has evolved to \" MDR or XDR \" FORMS . and heard that this also effects heart and brain cause am having massive pain near my heart area but it feels like you have the pain during heart attack. so whats wrong? Doctor: Hello,Yes, you are right. If you don't take proper treatment for tuberculosis, it can lead to drug resistance (MDR - Multidrug resistance or XDR - Total drug resistance). So you should definitely restart your anti-tuberculosis drugs, and take them for at least six to eight months.For your chest pain, it is better to first rule out heart diseases by doing ECG and 2d echo. If both these are normal, then no need to worry about heart diseases. You just finish your tuberculosis treatment, and your chest pain will also subside.Hope I have answered your query. Let me know if I can assist you further.Regards,\u00a0\u00a0Dr. Kaushal Bhavsar"
},
{
"id": 201884,
"tgt": "Recommend treatment for low sperm count",
"src": "Patient: It takes about three years that my sperm has been reduced to some very few drops. I have never had any sexual sickness or, any related problem. I just drink a lot of beers (addicted) and also have hypertension (completely controlled), I am 53, and never made prostate test. Can you please help me? Doctor: HIWell come to HCMlooking to your age the quantity of the sperm would come low and this can not be increased not even with the medicines although there is such medicine that could increase the semen volume hope this helps"
},
{
"id": 172188,
"tgt": "Suggest treatment for stomach pain and irregular bowel movement",
"src": "Patient: Hi, For the past 2 days my son is having stomach pain, and his bowel movement is kind of foam..i gave Albandazole suspension(given bya paediatrician)....but still his bowel movement look like a foam..can u suggest me any medication to regular the bowel movement? Doctor: Hi...Thank you for consulting in Health Care magic.It seems your kid is having viral diarrhoea. Once it starts it will take 5-7 days to completely get better. Unless the kid's having low urine output or very dull or excessively sleepy or blood in motion or green bilious vomiting...you need not worry. There is no need to use antibiotics unless there is blood in the motion. Antibiotics might worsen if unnecessarily used causing antibiotic associated diarrhoea.I suggest you use zinc supplements (Z&D drops 1ml once daily for 14 days) & ORS (Each small packet mixed in 200ml of potable water and keep giving sip by sip) as hydration is very important and crucial part of treatment. If there is vomiting you can use Syrup Ondansetron (as prescribed by your paediatrician).Dr. Sumanth MBBS., DCH., DNB (Paed).,"
},
{
"id": 115321,
"tgt": "Should Pradaxa be continued to treat blood thinning?",
"src": "Patient: I am on pradaxa - the new blood thinner. I was diagnosed with erosive gastritis a year ago and spit up bloody sputum every morning when I awake. It gathers in my mouth when I sleep, and shows only in am. It seems to be minor. The pradaxa keeps me from having angina. I feel that I need to take it. What should I do? Doctor: Hi,Thanks for asking.Based on your query, my opinion is as follows.1. Bloody sputum usually signifies, problems with oral hygiene.2. Meet a dentist for evaluation of gums, teeth or any ulcers.3. Pradaxa, if it causes gastritis, and bleeding, it would have showed in stools, as dark colored stools. 4. Presently Pradaxa is not causing any erosive gastritis. Causes of bloody sputum appears to be in mouth. ENT and dentist evaluation will identify cause. Need to look for tumor too.Hope it helps.Any further queries, happy to help again."
},
{
"id": 22162,
"tgt": "What is the treatment for panic attack and heart murmur?",
"src": "Patient: I just checked my blood pressure at a grocery store and the machine said 168 systolic and 68 diastolic. I'm 20 years old! I'm not over weight or have any medical conditions except for a slight heart murmur I was born with. But I did suffer a pretty bad panic attack a few days ago, because of finals and stuff. I'm still kind of stressed, because I'm worried about my health and such. Should I be worried? Doctor: hello, You shouldn't be worried at this stage, you should buy one bp apparatus and monitor your blood pressure. Of your blood pressure is high persistently then you will some investigation. yes stress and exertion can certainly raise blood pressure. You should have a healthy lifestyle like avoiding fatty, oily and high calorie diet. Have low salt diet and monitor blood pressure regularly thrice a day for one week then once or twice a week. Regular exercises like brisk walking, jogging according your capacity atleast 30 min a day and 5 days a week. Lots of green leafy vegetables, fruits, fish once or twice a week, avoid meat. Avoid smoking and alcohol if any. There shouldn't abdominal fat deposition or obesity. If bp is more than 140/90 persistently then You should undergo test like kidney function test, renal artery Doppler, Sr cortisol and 24 hr Urinary metanephrines."
},
{
"id": 96827,
"tgt": "What causes pain in ribs after an accident?",
"src": "Patient: I am 39 years old. I was in a car wreck 10 years ago and the hospital did MRIs CT scans, etc. They said that they were surprised that my neck wasn t broken because I wasn t wearing a seat belt. I do not wear seat belts because they hurt my shoulder and cause my arm and hand to go numb. The hospital sent me a report stating that I have a birth defect that caused me to have two cervical ribs. My mother said that the hospital has to be wrong that I was born with no birth defects. When I was in third grade I was tested and taken out of regular classroom because I tested 190 on the IQ test. I am 5 8 and weigh 110 lbs. I have never had the flu, chilhood illnesses, fever blisters, or bruises. My body temperature is always low. I have always had headaches. I have always been very active and never feel tired to the extent that other people ask me what I take to have so much energy. I have a lot of hair so you can t see the big lobes on each side of my skull. I now had a bump at the base of my left thumb. I had a small birthmark that looked like a scar just above my ankle. I have never had a leg cramps before seven monts ago. However, seven month ago when I fell to floor in spasms on the right side of my body. I couldn t walk for three months. There are no words for this kind of pain. I had no cuts on my leg but now have a scar that runs from my ankle all the way up my right leg. My left cervical rib is out of place now and I need to know how to get it back in place. The rib is causing severe neck and head pain. Who can help me? Doctor: HelloI am Dr Adil Thank you for asking the question.Having cervical ribs isn't a major deformity or disease. and these are the cervical ribs that are causing all the numbness of your hands and arms when you wear a seat belt. The problem is, cervical ribs tend to compress the nerves of brachial plexus, that control the sensations of your arms and hands. Compression of brachial plexus by cervical ribs, become more severe by wearing seat belt. So this is one problem. You may consider removal of cervical ribs by surgery, consult a surgeon.Moreover, as far as your other problems are concerned, I must say that, one can't give you a better opinion or advice without asking more questions and examining you, especially you motor and sensory system of all the limbs. And one would also ask for your old MRI and CT scan reports. . I would suggest you to see a specialist and get his advice. If you have more questions, feel free to ask. Regards. Dr Adil"
},
{
"id": 122142,
"tgt": "What causes loss of 1.5 inches of height in less than 1 year s time?",
"src": "Patient: what would cause me to lose 1.5 inches of height in less than 1 year s time? I am 54. Female, in menapause. I have bone density tests every few years (the last one being Dec. 2011) and always pass with flying colors results with lowest possible risk factors. Doctor: Hello, Your symptoms could be related to chronic degeneration of the intervertebral disks or the vertebral column bones. For this reason, I recommend performing an X-ray study of the vertebral column. Hope I have answered your query. Let me know if I can assist you further. Take care Regards, Dr Ilir Sharka, Cardiologist"
},
{
"id": 53191,
"tgt": "What could AST & ALT levels of 107 & 149 suggest?",
"src": "Patient: HI I am 54 yr old male - my BP is 117 over 79 - no meds. I am about 20 pounds over wieght. I drank a six pack of beer a day for about 30 years. My the AST& ALT levels comes out to be 107 &149 respectively about a month ago. I quit drinking about 2 months ago. I feel great and my wife says I am more calmer . Even lost some wiegh. Doctor: Hi.Thanks for posting query at HCM.Usually ALT or AST values higher than \"two times the upper normal limit\", is considered abnormal ( in some countries, ALT or AST values of more than 100 are considered abnormal). Value of AST or ALT greater than 85 or above maybe investigated further.Alcohol ingestion and obesity are common causes of fatty liver disease.advice :- abstinence from \"Alcohol\" - LOW fat diet should be followed, AVOID junk food and beverages- decreased oil consumption (oily food)- NO red meat- green vegetables should be ingested daily- use lemon juice (lemonade) once in a day- reduce weight if overweight/obese-\"recheck liver enzymes after 6 to 8 weeks\" and/or ultrasound.any further questions are welcomed.hope to answer your concern.wish you good health.regards,Dr Tayyab Malik"
},
{
"id": 46260,
"tgt": "Is radiating pain from abdomen to testicle sign of nutcracker syndrome?",
"src": "Patient: Age: 33 Height: 69'' Weight: 180lbs Pain in left testicle fro apprx 2-3yrs. Ultrasound showed no lump or tumor. Vericocele was found to be cause for pain. Received series of cord block injections and anti-inflammatory meds. Pain began to radiate from left lower abdomen and into the left testicle. Ultrasound shows no hernia, however, physical exam led my current physician to believe I have a hernia. I have recently read about \"NutCracker Syndrome\". Is this a common issue, mis-diagnosis of this syndrome. I ask because my urologist wants to go straight into shaving my spermatic cord...??? Doctor: Hi and welcome to HCM. As an Urologist, i can understand your anxiety.Your symptoms aren't due to ' nut cracker ' syndrome.Your ultrasound scan shows a varicocele,which should be it's cause.Hernia is well seen on clinical examination and accurately on the scan.As it's not reported on the scan,forget it.If your pain is recurrent and you need analgesics,undergo varicocele surgery.That should give you good relief.Dr.Matthew J. Mangat."
},
{
"id": 61004,
"tgt": "What does a lump on the left shoulder indicate?",
"src": "Patient: My son is 21 years old and noticed a lump on top of his left shoulder today after golfing. He said it was not painful or warm to the touch and that he had not been bitten by any insects or anything like that. What do you think this is? A cyst or a lipoma possibly? Doctor: Hello,This is possible soft tissue lesion which may be lipoma, neurofibroma, sebaceous cyst or others.You need clinical examination and ultrasound examination for the confirmation of the diagnosis for further management.Hope I have answered your query. Let me know if I can assist you further.Regards,Dr. Bhagyesh V. Patel"
},
{
"id": 144771,
"tgt": "What causes hair fall and fatigue while suffering from degenerative spinal disease?",
"src": "Patient: I have severe degenerative spinal disease (every disc in my cervical spine except for two which are fused and pinned are herniated or bulged, every lumbar disc from L2 to S1 as well), bursitis in all four hip bursa s and have been undergoing injections and RFA s to treat the chronic pain. A few weeks ago my hair started falling out and I have been experiencing malaise and fatigue. Could there be any connection? (I just had blood work and my thyroid came back normal) Doctor: Hi there, thank for your question. Hair fall has no one to one co relation with degenerative spinal disease. I suggest you to meet a dermatologist for your hair fall and be rest assured that it has nothing to do with your spinal pain. Though both are age related, they are independent of each other. On the other hand fatigue can be associated with pinched nerves in cervical and lumbar canal stenosis.Hope this answers your question. All the best. Regards. Dr.SBK"
},
{
"id": 189567,
"tgt": "Is there any treatment to fill the gaps between front teeth ?",
"src": "Patient: HELLO DOCTOR , I AM AMIT, AGE 26 YEARS. I HAVE A BIG GAP BETWEEN MY FRONT UPPER TWO TEETHS. WHAT ARE THE TECNIQUE AND APPROX COST FOR FILLING THIS GAP? THANK YOU. Doctor: Hello, Thanks for writing to us. A diastema is a space or gap between two teeth. It appears most often between the two upper front teeth. However, gaps can occur between any two teeth. Sometimes, a diastema is part of a set of problems that require orthodontic treatment. If you want it done in one visit, then bonding is the way to go. Bonding is considered a permanent material, but that doesn't mean it will never need to be redone, it most likely will at some point. Orthodontics might be the best choice, but it takes longer and will be more expensive, people get braces, which move the teeth together. Often, no matter where the diastema is, you must wear a full set of braces \u2014 on both your upper and lower teeth. That's because moving any teeth affects your entire mouth. Wishing you good health... Regards."
},
{
"id": 113940,
"tgt": "Is there any medical reason as to why my result showed there is no pain killer medication in my system inspite of taking the medicines ?",
"src": "Patient: I am a male 34 years and currently going to a pain management doctor for severe pain in my back. I have had two surgeries on my back. Recently they give me a urine test and they said the results came back saying that there was no pain killer medication in my system. I take the medicine every day as prescribed by my doctor and the results should have showed it. Can you tell me if the is any medical reason why my results would come back like that? I drink Mountain Dew and I pee alot, my mom says that maybe that has something to do with it. Please help me because the doctor took me off of the medication because he said it s a state law that he has to do that if it doesn t show up in my urine test. I need the medicine to help get rid of my pain. Doctor: see mr,what for you got done two surgeries?i hope you went for lower back pain only&see it is quite difficult to detect pain killers in urine since they undergo metabolism(catabolism=break down into short chain prouducts&appear in urine),NOT ALL SOME MEDICATION EXCREATED IN URINE UNCHANGED. BUT problem here is why do you want to know in urine,at last see,IN HUMAN BODY NATURAL PAIN KILLERS LIKE ENDORPHONES WILL PRODUCED IN RESPONCE TO PAIN,THEIR LEVEL MIGHT HAVE COME DOWN DUE TO CONSTANT worried abt bak pain,which eventually increases pain&visa versa.go for ''YOGAASANAS'' CONTROL YOUR MIND FIRST OTHERWISE IT ACTS LIKE ENEMY&EXPAND YOUR MIND&ENLARGE YOUR WORLD,THEN ONLY MEDICINES GIVEN OR SURGIERS CONDUCTED WILL HELP.GOOD LUCK."
},
{
"id": 408,
"tgt": "Suggest treatment for difficulty in getting pregnant despite having regular periods",
"src": "Patient: my brother got married 4yrs ago. After marriage he was affected by TB so child birth has been postponed. now he finished his treatment. for the last 6 months he tried for child but failed. his wife and his reports are normal. what kind of treatment he has to undergo kindly clarify Doctor: Hello welcome to the health care magic Your brother and his wife have to do frequent intercourse around 14 day of menstrual cycle because it is time where ovulation occurs So frequent intercourse between 12 to 18 day of cycle needed Investigate with following reports..... For your brother...1.Semen analysis 2.Blood Glucose 3.Serum testosterone 4.Serum LH, FSH estimation For your brother wife.... 1.cbc 2.chest x ray 3.serum prolactin 4.serum TSH, T45.Endometrial biopsy 6.HSG and laproscopy if needed If specific defect found then further treatment guided If no cause present and can't conceive then assisted reproductive techniques has to be adopted Hope your concern solved Take care Consult infertility specialist and discuss all these"
},
{
"id": 47504,
"tgt": "What does this sonography report of kidney indicate?",
"src": "Patient: Dear sir I am 28 Year male . My height is 5.5\" and weight approximate 65 kg . i am suffering from kidney stone. My sonography report says \" RT Kidney:- it is normal in size shape, Position & outline. parenchymal echogenicity is normal with the corticomedullary differentiation are well defined. pelvicalyceal system & upper ureter are dilated with a calculus of =12 mm in upper ureter two calculi noted and mid lower poles (Larger =7 mm )Left Kidney:- is normal in it is normal in size shape, Position & outline. parenchymal echogenicity is normal with the corticomedullary differentiation are well defined.No evidence of hydronephrosis in left kidney. A small calculus of 6.5 MM at upper pole.Impression :-# Fatty Liver # Right Hydronephrosis & Upper Hydroureter due to calculus in upper ureter # B/L nephrolithiasis Doctor: Hello and welcome to HCM.As an Urologist, let me opine, that your scan report shows, 1.A 7mm stone in lower part of Right kidney.2. A 6.5 mm stone in the upper part of Left kidney.3. The right upper part of tube draining the kidney, shows 2 stones, which are obstructing it. This will need treatment, as they're large in size. The right ureteric stones must be cleared first, as that side kidney is blocked, and kidney function will be affected. This can be done with endoscopic surgery.(URS-lithotripsy). If you've any other doubts, you may contact me with a direct question.Dr.Matthew J. Mangat."
},
{
"id": 210665,
"tgt": "Need medication for cold numb and painful fingers",
"src": "Patient: everynight i wake with cold numb painful fingers the tips of 2 fingers are numbish all day my middle finger nuckle is big i fold clothes for 75 boys for last 3 years before that i was a cleaner been and seen chiro i know dirty word but he is family so but to no releif dont have the best diet do drink a little say 3 vodkas 4 times a week do smoke cigarettes its really getting me depressed and tired can you give any help Doctor: HiThanks for using healthcare magicI think, you have underline depression and need low dose antidepressant. That would help to control underline anxiety and depressive symptoms. For pain in fingers, you can take pain killer on sos basis. Better to consult a psychiatrist for proper mental status consultation and further management.Thanks"
},
{
"id": 106857,
"tgt": "How can pain in the lower back be treated?",
"src": "Patient: Hello, I was inline skating, fell on my bum on Friday evening. At the time, I also had pain in my lower back. Over the past two days, the pain has be come more and more in my lower back and finding it very difficult to sleep at night. Is there anything to recommend? Or should I go in and see a doctor? Doctor: Hello,A history of trauma to the back in your case a fall can cause pain due to a variety of reasons from a strain to your muscles to a fracture of the bone.I would highly recommend you to go get yourself checked out by taking an x-ray.Hope I have answered your query. Let me know if I can assist you further.Regards,Dr. Fahim Sheik"
},
{
"id": 49536,
"tgt": "Increased total bilirubin and recommended phospholipides multivitamins. What is the diet I should follow?",
"src": "Patient: hi good morning, Doc, my i ask how can i reduce my total bilirubin, 1st i get met my medical annd i got 1.89 tb then after they told me to take silymarine (legalon) 140mg and phospolhiphides multivitamins (essentiale) for 1 week and 3 x day, then after a week they try it again but instead to make it lower it make higher and make it 2.29, i dont know what happen i dont eat meat anything with oil i only eat fish, may i ask what should i take and what should i avoid to eat i'll try to oatmeal and banana only for everyday to make it lower it is ok how about medicine should i take the silymarine lower mg instead of 140 mg i just want to make it to 1.0 level, thank you very much Doctor: Hello!Thank you for the query.Diet or silimarine wont rather help with elevated bilirubin level. You should rather be well diagnosed to find out what is the reason of such elevated bilirubin. There is many reasons of bilirubin problem including gallstones in bile ducts, pancreas cancer (this can be the first symptom), liver issues (viral infection, cirrhosis, tumors), blood hemolysis. I suggest you to do not wait any longer with it. You should have blood work, liver tests (AST,ALT,GGTP,AP), liver virus tests (HbS-Ag, Anti-HCV), abdominal CT.Hope this will help.Regards."
},
{
"id": 141373,
"tgt": "What causes ribs pain, numbness in the fingers and unconsciousness?",
"src": "Patient: Hi I was in several serious trauma accidents and at one point had a few broken bones and TBI. I was driving 2 nights ago and have had some constant pain around the ribs that were broken several years back but never sought treatment for them, I started breathing extremely rapidly and the felt as if I was going to pass out ....everything started to go really slow motion then I lost feeling in my fingers then arms, then up my neck and literally almost crashed the car. I then felt as if I could not hold my head up, as if it was being pulled forward.....my vision got extremely blurred and my eyelids started to twitch. I could not feel my face or make a smile .....it last for about 15 mins and dissolved to a minimum but it is 2 days later and I still feel tingling and off balance and still have pain & shortness of breath. Doctor: Hello and Welcome to \u2018Ask A Doctor\u2019 service. I have reviewed your query and here is my advice. I would explain that your symptoms could be related to a seizure or a TIA (transient ischemic attack). A panic attack cannot be excluded either. Coming to this point, I would recommend consulting with your attending physician for a physical exam and some tests: - a resting ECG and a cardiac ultrasound - a Doppler ultrasound of the cervical arteries - an EEG - a brain MRI - complete blood count - blood electrolytes - fasting glucose. Hope I have answered your query. Let me know if I can assist you further."
},
{
"id": 219669,
"tgt": "When to take pregnancy test?",
"src": "Patient: I have done 3 home pregnancy test all positive I than went to the doctors they did a test it come back negative than they took bloods to make sure it come back positive but my hcg levels were low so I had to come back to have more bloods so I did so 3 days later they had risen but not much so again I had to do this process 3 days later and the same thing levels had risen but not much so they now think I have a cyst I feel normal just sore boobs and tired could it be possible that im not pregnant and that it is a cyst Doctor: Yes and no. A simple cyst should not cause an elevated hCG. However, your hCG is not rising appropriately. What does that mean? Well, early in pregnancy we expect the hCG to double every 2 days, approximately. If it consistently does not double correctly, the pregnancy is abnormal and will likely end in a miscarriage. Your doctor is tracking your hCG level with blood tests- there is no reason to continue the urine home pregnancy tests. I'm sorry to tell you that this is not a normal pregnancy. There is a high chance that you will not end up with a baby in 9 months. Good luck!! If you were trying to get pregnant, try again when all this is over!"
},
{
"id": 208244,
"tgt": "Suggest remedy for mental health problem",
"src": "Patient: I think my friend may have somethin wrong mentally. He talks about hearing voices that ask him to do bad things. He's distant. He cried when I toldhim a joke. He stayed up an entire 24 hours staring at the ceiling. Something isn't right. Please, help. Doctor: Hello, thanks for the query & I must congratulate you for your vigilance. Based on your description, hearing voices commanding to do something bad, inappropriate emotional responses, social withdrawal & aloofness are suggestive of Psychosis/Schizophrenia. Since when are these symptoms emerging? Your friend needs emergency attention from a Psychiatrist as long duration of untreated Psychotic symptoms leads to worse long term disease outcome. He can be treated by antipsychotic drugs like Olanzapine, Risperidone, Aripirazole, Quetiapine etc. based on his clinical profile & physical health. Once treatment is started, he will start to get better in as early as 1-2 weeks period. He may need to take these drugs on a long term basis depending upon his clinical profile. One last thing is please rule out whether he is on any drugs like cannabis, ecstasy, heroin, cocaine, amphetamine etc. As these drugs may also cause Psychosis during intoxication phase. Do rever back for any further help. Good luck"
},
{
"id": 175792,
"tgt": "How to improve weight of an infant?",
"src": "Patient: Hi, I have a baby of 3 and half months old. I since birth gave her first pre nan and now nestle pro nan 1. I want to increase her weight. Her current weight is 4 kg. only. I don t breast feed her, as I don t have enough quantity of milk in me. pl. tell how do I increase her weight. Rgds., Deepa Doctor: Hi...a 3 and half month old baby weighing 4 kg is doing extremely well. You need to worry to increase her weight even more. If she is regularly gaining weight and smiling at you now and her development is normal, you need to worry. Please continue whatever you are giving her currently.Regards - Dr. Sumanth"
},
{
"id": 75785,
"tgt": "Suggest treatment for chest pains",
"src": "Patient: My partner has been taken into hospital with chest pains. Her cardiac enzymes are slightly elevated and BP 174/93. The nurse said one cardiac enzyme was +55 and her CRP is +11 which indicated an infection. Is all of this serious as they are keeping her in hospital but say it may be heartburn. They are repeating tests in 6 hours Thanks Doctor: Hi thanks for contacting HCM....You have chest pain complaint ....Here your cardiac enzymes are high ...CRP is non specific that can elevate in inflammation or infection both ...So first rule out ischemic heart disease by EKG and ECHOCARDIOGRAPHY .....If ischemic heart disease there treated accordingly...Your blood pressure high which needs to be corrected by sodium nitropruside like drug infusion ...If no specific cardiac cause present and you have acidity problem than chest pain can be reffered pain ....For which simple acidity reducing drug given like omeprazole on empty stomach before meal ...Take care ....Consult physician for this."
},
{
"id": 101914,
"tgt": "Is frequent urination connected to an impending asthma crisis?",
"src": "Patient: Hello Doctor, am an asthmatic and want to know if frequent urination is connected to an impending asthma crisis. I notice I always have this occurrence just towards a major attack (about 2-3weeks).I am 30years old, male, African, and been diagnosed with asthma for 15years now.Thank you. Doctor: Hi,Frequent urination is not a known precipitating factor for asthma.If the frequent urination is a regular occurence the it needs to be evaluated by a physician to rule out other causes like UTI, Diabetes, etc.RegardsDr. Mishra"
},
{
"id": 171605,
"tgt": "Are swaying head and clenching fists normal for baby?",
"src": "Patient: My 10 month old granddaughter sways her head from side to side and at time also clenches her fists (not at the same time). She will especially do it when there s a favourite song of her s playing, but will also do it with no music. Is this something to be concerned about? Doctor: Hi,Welcome to Hcm,I dont really think that as a problem. Behavior can be varying from child to child. As long as she is not having any such persistent activities and she is otherwise behaving well, feeding well and developing well, you needn't worry. If it gets worse, the only thing that needs to be done is an eeg test to check if any seizure like activity on. Hope I cleared your query. Take care."
},
{
"id": 164436,
"tgt": "What causes abdomen pain with blotchy skin in a 4 year old?",
"src": "Patient: My daughter is 4 years old and has had severe stomach pains for 4 days which usually starts after lunch and lasts until she goes to bed at 8:00. No fever, she has red blotchy skin, slight cough, burps, vomiting the first two days and has pale skin. We ve been to the hospital twice and they say they can t do anything. Please help. Doctor: it might be due to bacterial infection or ulcerative gastritis,, use antacid therapy for 5days with soft diet and plenty of oral fluids. in ulcerative gastritis the children will look pale and lethargic.. get routine blood counts with hemoglobin. use iron and folic acid if the hemoglobin is low."
},
{
"id": 38831,
"tgt": "What causes a growth noticed at the thigh area which has pain and pus?",
"src": "Patient: Hi, I have what feels like a growth the size of an acorn on the inner thigh where the leg meets the body. Just noticed it. It isn't very painful, but when pushed some pus comes out of somewhere. Female, 175 lbs, 5'4. I'm guessing an infection? How can I get rid of it? Doctor: Hello,Welcome to HCM,The history and symptoms suggest that you may be having a small boil but it should be confirmed by physical examination.For the present symptoms I would suggest you to follow1. Give hot fermentation to the area.2. Apply topical T Bact cream.3. Oral antibiotics like tab augmentin for 5 days. 4. Oral NSAIDs to relieve the pain.Thank you."
},
{
"id": 74665,
"tgt": "How to treat chest problem called pneumothorax at left side?",
"src": "Patient: I am Ratan, Male, 27 yrs old and weight 55kg. I have a chest problem which is called Pneumothorax at left side .My sputum report is negative.Still I take a 6 moth course of CAT 3 medicine by doctor advise.After 5 month it was cured up. But after 6 month it reoccur. Then I start to take Rcinex 600, Supradyne and Stimulive.But my problem is still same. What should I do? Doctor: Hello,For pneumothorax, ideal treatment is ICD. If the pneumothorax is minimal, high flow of oxygen for 1 week will be sufficient. I don't think you need cat3 drugs for pneumothorax unless you have an underlying cause of TB.Hope I have answered your query. Let me know if I can assist you further.Regards,Dr. Siva Kumar Reddy"
},
{
"id": 180772,
"tgt": "What does an ulcer on the tongue indicate?",
"src": "Patient: I am 64 y/o white female with type2 diabetes. Currently have a small whitish lesion on ulcer under my tongue on one side near the last molar. At first thought maybe had bitten my tongue but lesion isn't raised, its sunk like an ulcer maybe. Really painful esp when swallowing. Doctor: Hello,As per your explanation, it looks like a Canker sore or an Apthous ulcer only. As it is inflamed it is painful and can take for about a week to get resolved.For now, you can apply Xylocaine containing gel over it or Choline Salicylate gel over it. You can take anti-inflammatory painkiller like Ibuprofen for relief.Suck ice or do cool compresses. You can also take Vitamin B complex or Lysine supplement. Do gargle with specialized mouthwash like Magic mouthwash. You should also avoid spicy or irritant foods.Hope I have answered your query. Let me know if I can assist you further.Regards,Dr. Honey Arora"
},
{
"id": 58785,
"tgt": "Pain in the left side of body. Tests shows elevated liver enzymes. Experiencing fatigue. Suggestions?",
"src": "Patient: Hi. I ve been having pain in my neck (left side), my left shoulder , and I have had some pain in my chest and under my left armpit and down my left arm. My head has also been hurting some. I also had blood work done recently that showed I have an elevated enzyme count in my liver . A normal count is 52, and mine is 500 (just included this in case it would have anything to do with my symptoms). I ve been experiencing fatigue as well...more than usual (I have a TBI). Doctor: Hi and welcome to Healthcare magic.Thank you for the query.There are severalcondition which can cause such pain including heart and lung disease. Most common abdominla causes are gastritis and gallbladder issue. SO you need to rule out these 2 first by doing ultrasounds or CT scan. Wish you good health. Regards"
},
{
"id": 57477,
"tgt": "Pyelonephritis, elevated liver enzymes, diarrhea, yellow bubbly stools, fever, back pain",
"src": "Patient: My daughter is 29,,she has pyelonephsis,,also she has elevated liver enzymes ,,diarrea yellow and bubbly,,,,low grade fever to none,,, back pain ,,,,,,got a clue what it could be? Doctor: Hi, Welcome to Health care magic forum. As you describe, it appears to be the hepatitis, with high serum bilirubin. The eyes and the urine also expected to be yellow. I advise you to consult a gastroenterologistfor diagnosis and treatment. Avoid taking nonvegetarian foods, oilyfoods, spicyfoods. Take more of curd,buttermilk, and soft foods. Wishing for a quick and complete recovery. Thank you."
},
{
"id": 103363,
"tgt": "Have fever, URI, sinusitis and bronchitis. On keflex, albuterol. Now have large lumps on the neck. Suggestions?",
"src": "Patient: Good evening Dr. Prasad I have had fever since the 17th and URI sinusitis and bronchitis went to quick care started keflex, albuterol, lortab, lidocaine viscous, solu-medrol shot, Flonase alternating APAP and ibuprofen for fever which runs low grade while on meds around 100.1 woke up this morning with large lump and several small ones on back of my neck up into my hairline have an apt again with my GP on wed also severe headaches however I suffer from migraine HA any idea about the lumps and yes they hurt if pressed no exudate in the lumps keep in mind I've been taking antibiotics Doctor: Hi thanks for your question.The big lump in along with several small lumps in the neck could be lymph nodes. Lymph nodes are normally present in body and get enlarged when there is infection in their vicinity. You are having sinusitis, upper respiratory tract infection and bronchitis. These lumps could be cervical lymph nodes. You are taking antibiotics along with plenty of symptomatic medicine. Continue with these medicines. But you a good physical examination along with investigations like, chest X Ray, blood count and fine needle aspiration cytology of these lymph node is necessary to establish the diagnosis. After accurate diagnosis treatment can be prescribed to you. For migraine continue the medicine prescribed to you by your G p. Re consult your G P with these base line facts in your mind, so that he may re evaluate you for your problems.Hope this answers your question"
},
{
"id": 181790,
"tgt": "Suggest treatment for painful teeth post tooth removal",
"src": "Patient: I had a tooth pulled 5 days ago. The pain was lessening up until last night it began to hurt again. It's controled by regular aspirin but is uncomfortable. I had to work the day of extraction and three days after long shifts. Is this pain normal. I can still see something on the hole (was red then a white covering and now is like a grey) Doctor: Hello, Thank you for consulting with HCM.Actually when a tooth is removed you should continue with the antibiotics and painkillers for 5 days and even follow the instructions given by the doctor. if you have followed all the instructions then pain might get relieved after 5 days.But as in your case , it looks that due to exertion the socket is not healing properly and that is why pain is present, there is a possibility of dry socket. Better to visit your dentist again and get it treated.Hope it will help you."
},
{
"id": 11020,
"tgt": "Suggest possible treatment for hair loss and dandruff",
"src": "Patient: I am suffering from hair loss & dandruff problem and have even developed baldness in some areas I am using one ayurvedic hair oil which have helped me to reduce my dandruff problem but the hair loss is is still persisting. kindly advice some tablets or capsules to regrow my hair thanks Doctor: HIWell come to HCMI really appreciate your concern, if this is the dandruff then hair loss could be due to this only and this need to be managed first and for that try \"Povidone Iodine\" apply this on scalp prior to bath at least 30 minute before and this is nothing to worry, hope this information helps, take care and have a nice day."
},
{
"id": 111739,
"tgt": "What causes pain in back/bicep when having degenerative disc with shoulder replacement?",
"src": "Patient: i have degenerative disc at L2,L3,L4L5,S1 i have also had a pain across my back for the past 4 weeks and it is bad. seems like it in the scapula area from left to right. and on the right side some pain goes down the right bicep. Also keep in mind that I have both shoulders totally replaced. Doctor: It looks to me that apart from disc protrusion at your back you are also having cervical disc protrusion. This leads to nerve compression and it causes pain in shoulder, biceps and in scapular region. My advice to you is to get cervical spine MRI. Treatment for this will be intermittent cervical traction exercises and neck muscle strengthening exercises. Drugs which will help you are Myospaz and Myoril. Also you should take Pregablin and Methylcobal."
},
{
"id": 221362,
"tgt": "What causes bleeding and cramps during pregnancy after injury?",
"src": "Patient: So i just found out i was pregnant. and Me and my brother do boxing.. i was hit in the stomach repeatedly... directly after thast i had a gush of blood come out.. i have been having mild cramping.. and the other day i had a brown thick gross discharge just come at a rush. what is this? Doctor: Hello,I have gone through your query and understood your concern. Heavy exercise is completely contraindicated during early pregnancy and if possible, throughout pregnancy. Your history suggests that you may have had a threatened abortion where there is no pain but bleeding is observed as evidenced by the old bloody discharge that you had. The current cramping pain with bleeding suggests a possible inevitable abortion. Please see a specialist immediately for further care. If the pregnancy is still viable, further management aims at salvaging the pregnancy. Hope this helps."
},
{
"id": 74382,
"tgt": "What causes sharp pain in the upper chest?",
"src": "Patient: feeling a dull achy and sometime sharp pain in the upper chest below the neck in the middle area. Not heart related (have seen cardiologist recently for normal check up). Has been occurring for about 3 days on and off. I have also had other joint pain. Taking high blood pressure medicine and started a statin this week. I am 51 years old. Doctor: Thanks for your question on Healthcare Magic. I can understand your concern. No need to worry for heart diseases. Possibility of musculoskeletal pain is more likely. So apply warm water pad on affected areas. Take simple painkiller and muscle relaxant drugs. Avoid heavyweight lifting and strenuous exercise. Avoid movements causing pain. You will mostly improve with all these in 1-2 weeks. If not improving get done chest x ray to rule out lung related diseases. Hope I have solved your query. I will be happy to help you further. Wish you good health. Thanks."
},
{
"id": 209541,
"tgt": "What could a sense of lack of energy and concentration?",
"src": "Patient: Hi there,I was wondering about the following:I am a 20 yr old female student, for the past 3 or so months I've noticed the lack of energy I've been having despite having 7-8hrs of sleep a day, eating decently, getting some exercise here and there, with minimum to moderate amounts of stress. I find that concentration is difficult, mostly at school, I've been feeling depressed and moody also. Would you happen to know why things are this way? Doctor: Hello,Thanks for consult,Brief answer,DepressionExplanation,1. Symptoms which you are mentioned like difficulty in concentration, depression and moody suggest that you are suffering from pyschological disorder called Depression- a mood disorder2. To prevent depression behavioral therapy or medications belongs to the class of antidepressants are used which include a) Selective Serotonin Reuptake Inhibitors-SSRI b) Serotonin-norepinephrine reuptake inhibitors- SNRI c) Tricyclic antidepressants- TA d) Monoamine oxidase -MAO are usedHope this information helps"
},
{
"id": 152517,
"tgt": "Is a large growth on the top of the head a concern?",
"src": "Patient: My Husband has a growth on the top of his head. Has been there awhile, just scaly but the past 4-6 wks it has grown up out of his head and gotten about the size of a dime or larger. I showed a pict of it to a Dr friend she thinks its a basil cell crater. My husband is in an Alzheimer s facility Does this sound like what my friend said? And if its a crater does that indicate it is growing inward? I m not sure how invasive to take this What are the pros and cons to treating this? Doctor: Hello and Welcome to \u2018Ask A Doctor\u2019 service. I have reviewed your query and here is my advice I would advise you to do a CT scan of head. If non-malignant in biopsy test then the growth on head can be removed by surgery. Hope I have answered your query. Let me know if I can assist you further."
},
{
"id": 148065,
"tgt": "Can the feeling of ice cold water running down the leg be associated with the herniated discs?",
"src": "Patient: I have been told by my orto surgeon that I need lumbar microdiscectomy surgery to correct a herniated disc that is pressing on a nerve. A couple of days ago I started to feel ice cold water running down my left leg. Could this be associated with the herniated discs? Doctor: Hi !I read carefully your query.You should run to do vascular doppler ultrasound to exclude any vascular pathology and than than to do lumbar spine MRI to evaluate for disc herniated and evaluated nerve pathology.Depend from the pathology you should treatment from specialist for the first from angiologyst and second from neurologist or rheumatologist but it depend a lot of you because you should do sport activity like swimming in quiet phase.Wish you fast recovery.Dr Ilirian Laci"
},
{
"id": 145092,
"tgt": "What is the treatment for throbbing pain in the temples?",
"src": "Patient: hi ive had a throbbing pain in my temples right above my ear. once the pain/ popping feeling happend on the left side i thought i had a stroke or anyursm and then it didnt happen for a while then last night i had a throbbing pain in the right side of my head and i paniked and ran to the bathroom and thats when it went from a throbbing pain to a sudden popping feeling above my right ear. i have not gone to a doctor yet but can you please give me an idea of whats happening to me. it scares me i know that i also propably have anxiety im a hypocondriate but i really need to know that im gunna be alright please help let me know what you think it is please and thankyou! Doctor: Hello!I read carefully your question and understand your concern.I think that this may be a primary stabbing headache, if it lasts for seconds. It is a benogn headache without a known cause. It may also be related to the muscles. It may also be caused by inflammation like a temporal arteritis, but it is also associated with vision problems which you don't refer.The fact that it has occurred in both sides makes is a good indicator excluding an aneurysm.I recommend you to consult with your neurologist ,have a neurological examination and a full blood work for inflammation.Hope to have been of help!Greetings!Dr. Abaz Quka"
},
{
"id": 177732,
"tgt": "Suggest treatment for child suffering from itchy rashes",
"src": "Patient: Hello Doctor, My three year son has developed small itchy rashes one months back. It started as a small patch behind neck but slowly spread to his shoulders and elbow and lower back. Consulted a dermatologist. She advised applying momate and atrax for five days. the rashes seemed to have almost dried up. On second consultation she gave lycosole and another cream. But soon after the second visit the rashes have reappeared and dont seem to go away but are instead spreading. please advise Doctor: The cause behind the rashes needs to be looked into. The medicines given by your dermatologist are proper and please continue the medicines without worrying. Had I been able to take a look, I would have been better able to comment. Get a blood count including the platelet count and leucocyte differential count done. Please get back with the reports and I will try to help further."
},
{
"id": 184787,
"tgt": "How to improve a speech clarity?",
"src": "Patient: I will be applying for admission to various B-Schools in the upcoming few months. I sometimes have problem with my speech. problems such as slurring when speaking too quickly and getting stuck on pronounciation of some words. I feel this can let me down in my admission. Where can i find a good speech therapist in Kolkata? Please advice. Doctor: HiWith your query, as such we need to examine your oral cavity for any tongue tie presence since this is the most common reason if it is present is present you may have to go for excision which is minor procedure and other reasons are macroglossia etc... kindly visit your dentist for the needful.sometimes even the alignment of teeth may also result in slipping of the words you pronounce"
},
{
"id": 223326,
"tgt": "Can pregnancy happen even after consuming Yasmin and trimethoprim?",
"src": "Patient: I am on the contraceptive pill - Yasmin pill and was recently taking a 3 day course of trimethoprim, on the second day of taking this antibiotic, and at the end of my pill packet, me and my boyfriend had intercourse with no additional contraception. Is there a risk I could fall pregnant by this? Doctor: hello user,no pregnancy wont happen after consuming both yasmin and trimethoprim.Trimethoprim does not reduce the effectiveness of yasmin..yasmin provide 99 percent contraception.thanks"
},
{
"id": 1880,
"tgt": "Suggest treatment to normalize the hormonal level and conceive",
"src": "Patient: Hello, I am 28 yrs old,my height is 5.1' and weight is 60.5kg.I am suffering from irregular periods and so i consulted a gynac .She suggested me to go for an hormonal test.The test showed my LH level as 16.42mlU/ml,which is high as per my doctor's knowledge.She had given me dimet 500 and asked to take it twice a day and also asked me to reduce my weight.I want to know whether my hormonal problem will be solved and can i conceive without any complications? Doctor: Hi, your problem can be solved through medicines. You can have a normal pregnancy. For that, you will have to take some medicines apart from dimet. You can take some medicines for growth of your eggs like clomiphene. You can keep a track of your follicles growth and be in contact with your husband every 2 to 3 days after your periods stop. It will increase chances of your pregnancy. You may need a injection for rupturing the follicles. Hope I have answered your question. If you have any other query I will be happy to help. Regards Dr khushboo"
},
{
"id": 130440,
"tgt": "What causes fibula pain after an ankle injury?",
"src": "Patient: A week ago i slipped in my friends bathroom and rolled my ankle it stung a little right of hand. I just figured it was sprained, so i treated it as such. But last night and all day today I've barely been able to walk on my left foot. But its not my ankle that hurts its my fibula. And now as im putting heat on it, it is tingling from my toes to right under my femur. It hurt now more than ever, any idea's? I've only ever had shin stress fracture in marine corps boot camp 6 yrs ago. Doctor: Hi,Putting heat is not a good idea. I think that is why you have all that pain. You should consider putting ice and take same anti-inflammatory drugs such as ibuprofen. No weight bearing for three weeks or at least as pain permit. I recommend doing a x-ray and to be seen by a specialist.Hope I have answered your query. Let me know if I can assist you further. Regards,Dr. Rezart Cipi"
},
{
"id": 210347,
"tgt": "What causes irritation when asked for something and behave in an abusive manner?",
"src": "Patient: I would like to know what type of disorder a man of 43 could have if he get angry when asked questions and instead of being able to talk about things he get blood shot eyes and talks in an abusive manner that does not relate at all to the questions, sometimes physically threatens when in these eposides as well. Doctor: HiThanks for using healthcare magicIn broad term, i can say that i may be in psychosis and he needs antipsychotic treatment. Before starting this drug, better to consult a psychiatrist for proper mental status examination. That would help to make proper diagnosis and plan treatment in his case.Thanks"
},
{
"id": 165235,
"tgt": "What does on and off loose motions with gas and foam suggest?",
"src": "Patient: My son is 5 month old, yesterday he passed motions 5 times, sometimes in qty, other times very little foam/gas bubles type. we started cerelac wheat giving only 1-2 teapsoons twice a day. he is also having a calcium/iron/vit d/zinc syrup since 3 days..... today he has done it twice. Is it normal, what to do? pls help Doctor: Hi... it is very common once you start introducing solids. Its nothing to worry about.Unless the kid's having low urine output or very dull or excessively sleepy or blood in motion or green bilious vomiting...you need not worry.But I suggest that you start complementary feeding only after the baby completes 6 months of age.Regards - Dr. Sumanth"
},
{
"id": 63108,
"tgt": "Suggest treatment for ulcerative bump on tongue",
"src": "Patient: Hi, may I answer your health queries right now ? Please type your query here...I have allover rash on both legs and sore knees...also sore throat...strep and endoscopy ruled out cancer..had bump on tongue removed...no cancer found...now just found ulcerative bump on tongue...some suggestions pleasemarsha Doctor: Hi Dear,Welcome to HCM.Understanding your concern. As per your query ulcerative bump on tongue could be a non healing ulcer or recurrent aphthous ulcer. Need not to worry about it, it will resolve its own. I would suggest you to maintain proper oral hygiene. Gargle with warm salt water 2-3 times a day. Visit oral pathologist once and go for biopsy of the non healing ulcer and also rule out any trauma. Apply apple cedar vinegar with the help of cotton. Drink plenty of water and fluids.Hope your concern has been resolved.Get Well Soon.Best Wishes,Dr. Harry Maheshwari"
},
{
"id": 137393,
"tgt": "Suggest treatment for grade-III chondromalacia of cartilage over the lateral tibial plateau",
"src": "Patient: A recent MRI shows Grade III chondromalacia of the cartilage over the lateral tibial plateau with high-grade thinning and mild irregulaity and a Grade II chondromalacia of the cartilage over the lateral femoral condyle with mild thinning. Are there corrective procedures available? Doctor: Hello,Chondromalacia is a condition with thinning and damage to cartilage.Usually it is associated with difficulty in walking and squatting. it is a self limiting procedure and usually patient responds well with physiotherapy and medicinal treatment. If there is no response with treatment then you can try for arthroscopy or key hole surgery. In this surgery damage cartilage is removed.it is called as chondroplasty. I hope this will completely heal your problem. I hope this answer will be useful for you. Let me know if there is any other followup questions.thanks"
},
{
"id": 64264,
"tgt": "How can a hard painful lump on the shin be treated?",
"src": "Patient: About two weeks ago in baseball practice, I overran a really fast grounder so the ball bounced and smack my lower shin(slightly above my ankle). It swelled alright but died down though there is still a small bump. It feels hard like its part of my bone. I've been able to walk just fine but when I run I feel slight discomfort from where the bump is(no pain at all). I don't know if I fractured it, I just don't feel pain unless I poke the hard lump. Should I consider an x-ray? What to do about it? Doctor: Hi,Good Evening.Thanks for the query to HCM.I studied it in depth and I understood your health concerns.-Treatment-1-I would advise orthopaedic surgeon consultation before starting any treatment.2-As you are anxious and as you had the tender hard bump on the shin.3-But by the facts told by you , I dont think that you would have fracture of the shin -Tibial bone.4-Still as you are worried get the Xray of the shin done and show it to ortho-surgeon.5-This bump is mostly due to the Boney Contusion with cellulitis with resolved fibrosed hematoma and traumatic periosteal cellulitis and induration.6-I would treat it as follows-a-Cold compresses and b-Rest and Elevation,c-Tb-NSAIDs for 5 days to expedite the recovery.7-Dont get scared and uneasy,but be cautious and act fast.Hope this would help you a lot to relieve you.Wish you fast recovery-and healthy life.Wellcome to HCM with more queries till you are satisfied.Have a Good Day...!!Dr.SAVASKAR M.N.M.S.GENL-CVTS,Super specialist and Senior Consultant-and Expert in Non-Curable-Disease therapy for Cancer,Asthma,etc,Rejuvenation therapy and Tissue failure -reversal therapies.a"
},
{
"id": 138565,
"tgt": "What causes pain above hip with reddish bruises?",
"src": "Patient: I have had severe pain when I cough for around 3 weeks now. It happens on my left side about elbow length down. So severe it has taken me to my knees in front of co-workers! I have NOW noticed severe reddish looking bruises on my left side, just above my hip!! What am I dealing with here??? Please help!!! Doctor: Dear Sir/MadamI have gone through your query and read your symptoms.In my opinion, pain with coughing is a sign of pressure on nerves when pressure in abdomen increases as a result of coughing.consult your neurologist and explain your symptoms his examination will help in the diagnosis.I hope that answers your query. If you want any more clarification, contact me back."
},
{
"id": 64876,
"tgt": "What does a lump on cheekbone indicate?",
"src": "Patient: I have a soft lump that just appeared suddenly tonight under my eye, right on my cheekbone. I doubt it is a spider bite because I think I would have noticed a spider on me as it s evening. It seems fluid filled,like someone inserted water under my skin. It doesn t hurt or itch. Doctor: Thanks for your question on Health Care Magic. I can understand your concern. By your history and description, in my opinion it is mostly due to bite only. Another possibility is skin infection or allergy. So better to consult dermatologist and get done clinical examination. You need removal of fluid to avoid infection in it. So get done proper surgical dressing. Don't worry, you will be alright. Hope I have solved your query. Wish you good health. Thanks."
},
{
"id": 82254,
"tgt": "Suggest treatment for bronchitis",
"src": "Patient: I have had a cold for over a week now and a bad cough bronchitis like with lots of thick mucus coming out, but now my back hurts on the right side near the shoulder blade. It seems to be worse when I cough or blow my nose so I think it is muscle related. I put Biofreeze on it and that helped for a while but it really hurts. Should I see my dr about it? Or will it heal when I am not coughing so much? Doctor: Thanks for your question on HCM.In my opinion you should consult pulmonologist and get done chest x ray and Pulmonary Function Test (PFT) to rule out chest infection and bronchitis. The shoulder pain is most likely due to muscle pull due to excessive coughing.Treatment of bronchitis is inhaked bronchodilator and antibiotics. You may require antihistamines for coughing. Painkillers and muscle relaxant for pain."
},
{
"id": 65986,
"tgt": "What does growing itchy bump near areola indicate?",
"src": "Patient: i woke up about a hour ago and felt a stingy bump like a size of a pimple on my right areola that only hurt when touched. I walked to the mirror and I have a thin pink line that starts a that bump and circles around my breast. Its been almost 45min now and the line is getting wider and the bump is getting a little bigger and itches. Doctor: Hi,From history it seems that there might be having something poisonous bite causing local allergic reaction.there is also a possibility of having some allergy due to synthetic bra.Take antihistamine like Cetrizine or Levo-cetrizine or Benadryl for 2-3 days.Applt triple cream locally.Ok and take care."
},
{
"id": 96429,
"tgt": "I have aleays stomach problem",
"src": "Patient: i m always stomach problem my food is not digest im eating lunch or dinner and immediately im going to toilet Doctor: u better consult a gastroenterologist and find out the cause of the problem........"
},
{
"id": 37116,
"tgt": "Could weakness, fever and tiredness be aftereffects of having suffered from typhoid?",
"src": "Patient: HII THIS IS DEEPA HERE FROM INDIAI GOT MARRIED BEFORE 2MONTHS. AFTER GOT MARRY I ALWAYS LIVES ILL.1ST MONTH I GOT TYPHOID& NOW FROM 10 DAYS M SUFFERING FROM FEVER BUT REPORT ARE NORMAL.MEDICINES ARE NOT EFFECTING ON ME. I ALWAYS FEEL TIRED, VERY WEEK INTERNALLY, HEADACH & UNFRESH PLS HELP ME WHAT TO DO? Doctor: Hi,It seems that you might be having some infection in your body giving recurrent bouts of fever.Mostly after marriage urinary tract infection is common.Go for,Complete blood test like TC, DC, platelet counts.complete urine routine and microscopic.After reports go for treatment accordingly.Take light diet.Ok and take care."
},
{
"id": 146056,
"tgt": "Does the twitching of little finger indicate parkinsons?",
"src": "Patient: Hi , my name is Scot , i m a 48 yr old male , I have MS ( was diagnosed back in 2001 ) , but as of Friday morning when I woke up I noticed that the pinky finger on my left hand was twitching , it stopped then started again , and as been doing it all day today , its not painful just a bit odd .... I do get tremors from my MS often , but this is different than what I normally have , it seems to have a mind of its own ... i am being checked for Parkinson s , and have an MRI next week ... Doctor: dear Scot,Twitching of a finger can have varied etiology.Our body movements are been controlled by certain areas of the brain, which is governed by various chemicals called neurotransmitters. Any change in the local or generalised levels of these chemicals in brain areas , can give rise to associated symptoms. Similarly, movement of little finger is governed by part of brain which involves a chemical \"dopamine\", essential for proper movements( smooth movements) of body. Variation in the levels of dopamine can lead to twitching of fingers, as experienced by you.Now Parkinsonism is a disorder, which consist of generalised dysfunction and set of symptoms clinically, and need to be confirmed on investigations like MRI,etc. It predominantly is due to depletion of this chemical of dopamine in brain.Therefore, there is possibility of it being due to Parkinsonism, however it is difficult to confirm as I can't see any associated symptoms or no investigation done as yet.I suggest you to follow the advice of your treating doctor.I hope to have satisfied your query.welcome for further information.thank you. stay healthy."
},
{
"id": 59833,
"tgt": "Stomach pain, fever, vomiting, done blood test. Help",
"src": "Patient: Hi, My Elder brother is feeling pain in his lower stomach and a fever as well. Whatever I am taking is coming out through vomit , even if its water. Its late night here and we do not have any medium to travel. Blood Test Results which are marked by the lab attender: Polymorphs 27 Lymphocytes 70 Platelet count 0000 Leaver Test results marked by the lab attender: Unit is U/L and Reference Range is 5 -34 SGOT (AST) 153 SGPT ( ALT ) 164 Please advise. Thanks and regards, Vishal Pachori Doctor: Hello, Welcome to Healthcaremagic. For now, stop any oral intake until you consult a surgeon. Try to consult a surgeon as soon as you can. Regards Dr Ahmad"
},
{
"id": 111930,
"tgt": "Does it make sense to have musculoskeltal for UTI/back pain symptoms?",
"src": "Patient: Thanks! I have UTI/ back pain symptoms. Yesterday's urine dip was neg. & dr. sent me for kidney ultrasound, which showed left kidney hydronephrosis; then they did a CT scan which was negative so dr. put me on ciproflaxin & said it's musculoskeletal. Does this diagnosis make sense? Thanks! Doctor: Hi,I have read your query.Yes some times back pain due to musculoskeletal reason can mimic UTI. So if patient UTI and back pain we usually recommend him/her urine culture sensitivity examination and if needed further scans etc.Your doctor has rightly prescribe you about tests and scans etc.Your pain is due to muscular reason if tests for UTI are negative.I hope your have get your answer.Thanks.Dr.Singh."
},
{
"id": 93960,
"tgt": "Have incision, stabbing pain. Had perforated ulcer. Gallbladder removed. Help?",
"src": "Patient: My name is Amanda and I had my Gallbladder out in May of 09 just 2 weeks after giving birth to my daughter. I had to be cut open because I had a perforated(sp) ulcer. I did pretty good for about 6 months and then started having stabbing pain on my right side at the end of my incision. Any idea what this could be? Or what type or Doctor I should go to? Doctor: Hi, Based from the information is seem that pain is mostly due to the stretching of the fibrous tissue around the area. The healing occurs with fibrous tissues which are tough then normal cell, so that the joint / healed portion can stay connected to the main part. But in some cases the over formation of such fibrous tissues may lead to sensation of pain as they are bit rough, or may have some other connection and leading to stretch causing feeling of pain.ADVICE: 1) Consult your surgeon and explain what is going on. 2) Ultrasound of abdomen may help to find out any abnormal connection formation after the operation was done. Hope i have addressed you concern satisfactorily. Dr. JAY PATEL."
},
{
"id": 174850,
"tgt": "Need medication for cough",
"src": "Patient: chronic coughing my daughter is 3 years old and weighs 50lbs and has been coughing for 2 1/2 months now and everytime I take her to doctor they don't hear weezing in her chest So they don't do anything. they say it's drainage from her stuffy/runny nose. and it's weird cause she runs a fever one day (103) and then money the next. it's worse at night that she coughs So hard she throws up. I have a neubulizer at home with prescribed medicine for it. they said it would be good to give her treatments and I have for 2 1/2 months but still has This cough. I feel as though they don't believe me. and when she has a fever she breaks out in hives. they put her on an antibiotics /steroids for the hives cause of the itching until they are gone. I did but then they come right back as soon as I stop giving it to her. then she is complaining of ear pain now, So it's back to the doctor. I don't know what to do or help the doctors understand what is going on help.worried and distressed mother Doctor: Hi,From the history it seems that she requires some investigations to find out the cause of her chronic cough problem.She requires,1, complete blood checking like TC, DC, ESR,2, X-ray chest,3, Montoux test to rule out primary complex.4, stool test for ova and cyst.Many a time Eosinophilia does cause this type of chronic cough.Consult pediatrician and discuss about these tests.Ok and take care."
},
{
"id": 10932,
"tgt": "Does Hair 4 u and Biotin tablets taken for hairloss have any side effects?",
"src": "Patient: Hello Doctor, I am 25 years old male guy.I am suffering from hair loss from the past 3-4 years and now it can be seen clearly as my hair line is slowly receding from the forehead. I have been suggested Hair 4 u 10% lotion & Biotin tablets. Can you pls let me know whether these do not have any side effects on the body Doctor: Hi..Welcome to HEALTHCARE MAGIC..I have gone through your query and can understand your concerns.As per your complain Hair 4 U 10% contains Minoxidil 10% and it is commonly given in patients with Androgenic Aloepaecia which seems to be in your case and it is mostly due to hormonal cause as well as genetic in origin..It is safer as well as helpful in hairgrowth but can have side effects like growth of thick and dark hairs on scalp..But it is not a very common finding..Biotin is a nutritional supplement and although will not cause any severe side effect but should be taken only after proper consultation with a Trichologist..You can also be advised Finesteride to be taken orally..Take multivitamins.Take more of green leafy vegetables and lots of water..Keep the scalp clean and dry..Hope this information helps..Thanks and regards.Dr.Honey Nandwani Arora."
},
{
"id": 36999,
"tgt": "Suggest treatment for spider bite",
"src": "Patient: I had gotten a spider bite a few day ago on my finger. It had swollen and a red line started tracking up my arm. The doctors dripped antibiotics in me through I v and prescribed me Clindamycin. This morning I got another bite which is as well starting to track. Will the Antibiotics I am currently taking be affective on this new bite or will I need to go to back to the hospital ? Doctor: HelloWelcome to HCM,Spider bites are actually rare occurrences and bites from most spiders cause local redness, irritation and pain that usually can be treated at home.These local reactions usually resolve without treatment over a period of 7-10 days.I would suggest you to followWash the site of the spider bite well with soap and water.Apply a cool compress or ice pack over the spider bite location.Over the counter pain relievers may be used to relieve symptoms.Thank you."
},
{
"id": 84883,
"tgt": "What is the side effect of using R Cinix EZ?",
"src": "Patient: Hi, i am Rahul from Delhi. I am using R Cinix EZ on a Doctor s prescription. I could not able to accustom with it s side effects if I use it with empty stomach. I have to drive car to reach to office for 60 KMs. Is there any other timing other than morning that I can use this medicine. YYYY@YYYY Doctor: Hi,R-Cinex EZ is a combination of four drugs (rifampicin, isoniazid, ethambutol and pyrazinamide) which is commonly prescribed for the treatment of tuberculosis. Generally it is well tolerated by most of our patients, however it may cause abdominal pain, fever, orange discoloration of the urine or saliva, flu like symptoms, inflammation of the nerves, visual disturbances and liver injury.If you develop fever, jaundice and upper abdominal pain you must report to your doctor after discontinuing the medications. To ensure maximum effectiveness this medicine should be taken on an empty stomach either one hour before or 2 hours after the meals. The absorption of this medicine may be reduced if it is taken along with food.Hope I have answered your query. Let me know if I can assist you further. Regards, Dr. Mohammed Taher Ali, General & Family Physician"
},
{
"id": 100133,
"tgt": "Suggest treatment for cold with asthma and cough",
"src": "Patient: I have had a cold for 5 weeks(also have asthma that flares up with colds). Have a bad cough & greenish yellow mucus in nose & cough. I also have occasional black flecks in my mucus....I have done a course of Cefdinir & now am on Augmentin. All this with neb treatments & a steroid inhaler. What to do next? Doctor: HelloYou are an asthmatic patient and taking many drugs like ( cefdinir, , now augmentine ) steroid and nebulizer .In spite of this all symptoms present with greenish phlegm with black flecks in mucus. The possibilities of these symptoms may be , these include:1 Bronchiectais , as this is most common cause . A physical examination and sputum culture examination is important.2 Asthmatic allergic bronchitis , as proper antibiotics is not working , so get in sputum culture and drug sensitivity test to find out exact type of bacteria.3 Tuberculosis , as black flecks may be due to tuberculosis . Diagnosis can be confirmed by x ray chest as well sputum and blood examination.Since you are an old asthmatic patient and not responding to treatment so consult a PULMONOLOGIST and get his opinion as physical , clinical examination is important in this case.Hope this will help you."
},
{
"id": 101093,
"tgt": "Suggest remedy for child asthma with breathing problems",
"src": "Patient: hi i am sanjay from India my daughter having child Asthma, when weather changes she become ill and she face problem while breathing due to heavy cough.Actually i can t afford her treatment and regular nebulization charges. Is there any alternative for this Doctor: Hello Sanjay.Thank you for asking at HCM.I went through your daughter's history.I would like to ask you a few more questions to make better suggestions like - What is the age of your daughter? Since what age she has been having breathing problems? How many episodes does she get in a year? How is she in between the episodes? Does she have nose symptoms like nasal congestion, running nose, sneezing etc? Does anyone from home smoke? Does anyone from her family have allergies or asthma or both?However, from your given history, I would like to make following suggestions to her:1. Were I treating her, I would prescribe her montelukast for at least 7-14 days when she has an episode.2. If she has nose symptoms also, I would add an antihistamine like levocetirizine or cetirizine.3. If with above treatment, she is getting wheezing, I would suggest you inhaled salbutamol or levosalbutamol for her in appropriate device. However, suggestion for device can only be made after knowing her age.4. Please try to avoid exposure to dusts, smokes and air pollution for her.5. A healthy dietary habits (adequate green leafy vegetables, fruits, sprouts, etc, minimizing cold drinks, iced foods, junk foods, etc) will help improve her immunity in a long run.Hope above suggestions will be helpful to you.Should you have any further query, please feel free to ask at HCM.Wish your daughter the best of the health.Thank you & Regards.Dr Parin N Parmar"
},
{
"id": 144699,
"tgt": "What causes excessive sleep, stiff neck and blurred vision?",
"src": "Patient: What should one do when experiencing, first, uncontrollable sleepiness, followed by a stiff neck and severely blurring eyes, in addition to left shoulder pain extending down into the left upper arm and elbow. These symptoms combined with a mild pain occurring in the left mandable, and faintly projecting down the left side of the neck. Could I have been experiencing simultaneously both a heart attack and a stroke? Doctor: You could be having cervical spondylosis. Get Your blood sugar levels tested to rule out Diabetes. Consult your Orthopaedic surgeon."
},
{
"id": 159450,
"tgt": "Suffering with cancer, has constipation. Heavy bloating on taking duphalac. Remedy?",
"src": "Patient: my moter is 62 years old suffering from cancer , shes on pain management since last 8 months and using fentanyl patches . She suffering from constipation at times. This time doctor advised to take duphalac for constipation treatment. i have given her two doses 15 ml today and her stomach is bloated heavily, kindly advise Doctor: Hello, Sorry to hear about your mother's condition. Constipation can occur in these patients and if it does not respond to duphalac, you can use a suppository to stimulate bowel movement. Also, keep in mind that adequate water intake is a must while using laxatives for best action. The bloating should hopefully resolve with the bowel motion but if it persists, then you can contact your doctor again for agents that can reduce bloating."
},
{
"id": 55553,
"tgt": "What precautions to be taken to lower liver enzymes?",
"src": "Patient: I am 35 yrs old and just found out that my blood tests revealed liver enzymes of 241- I do like to drink, but am not an alcoholic, and do not drink every day. How good are my chances of lowering these enzyme levels to within normal limits? will I have liver disease for the rest of my life, or will it resolve? Doctor: Hello! Thanks for putting your query in HCM. I am a Gastroenterologist (DM). Elevation of liver enzymes suggest injury to liver cells.There are many other causes for elevation of liver enzymes apart from alcohol intake like Hepatitis B and C virus, wilson disease, autoimmune liver disease for which you need to be tested. So you may visit your doctor for these tests. Also meanwhile what you should do is:1. Avoid alcohol2. If you are overweight or obese then do weight reduction by diet and exercise3. take tablet ursodeoxycholic acid 300mg tds and Cap evion 400mg odI hope I have answered your query and this will help you. If you have any further query I will be happy to answer that too. Remain in touch and get-well soon."
},
{
"id": 34238,
"tgt": "Suggest treatment for sinus infection and nausea",
"src": "Patient: bout once per yr, I have what I call sinus with discharge of phelm from mouth and nose. I experienced an eposide where I found it difficult to stand or coordinate, room spinning, sweatinf, nauseaetc. I had to throw up. After about 45 minutes, except for a bad taste in my mouth, I was back to normal. Doctor: hi.For sinus headaches, treatment starts with decongestants, pain relievers, and nasal irrigation to ease sinus pressure and congestion and help drainage.Antibiotics or nasal steroids are often used as a second line of attack to treat the underlying infection or chronic disease.A sinus headache caused by an infection should go away soon after starting treatment.Migraine treatment isn't just about stopping a migraine once it starts. It's also about preventing them and reducing their frequency, severity, and duration.Drugs called triptans are used during a migraine attack to reduce pain and restore function.thank you for your query."
},
{
"id": 126970,
"tgt": "What can cause swelling in the foot and ankle?",
"src": "Patient: I ve experienced swelling in the foot and ankle area for about a week. Elevating the feet seems to greatly reduce the swelling. It seems to be a fluid buildup, not an injury. Today, my right knee is also swollen. I ve been wearing compression stockings for several years although I haven t had any swelling before. Doctor: Hi, It can be due to contusion or ankle sprain. As of now apply ice packs and you can take anti-inflammatory drugs like serratiopeptidase for symptomatic relief. If symptoms persist, better to consult an orthopaedician and get evaluated. Hope I have answered your query. Let me know if I can assist you further."
},
{
"id": 141641,
"tgt": "Suggest treatments for fissure",
"src": "Patient: Hi Sir, I have been suffering from fissure . before marriage it was not so massive and painful but after having 2 deliveries after marriage, it has become worse. Its nearly 10 years and I am not finding any proper treatment as I could not find any lady doctor specialized in this and I hesitate to go to male doctors. Please suggest appropriately Doctor: Firstly avoid food that cause constipation.Your diet must contain good amount of water,roughage and green leafy vegetables.start taking hot seitz bath twice a day at least.To do this take lukewarm water in a tub and sit in it for 20-25 minutes.Also apply lignocaine jelly over fissure and anal orifice just before passing stools.Start syrup lactulose 6 tsf bedtime to soften stools.follow up after a week.."
},
{
"id": 200711,
"tgt": "What does itchy, puss like white heads on shaft of penis suggest?",
"src": "Patient: Have white little puss like white heads on my shaft I popped them now they are taking forever to heal they itch and I don t know exactly what it is. Please help I hope it s not herpes.. They are blistering also I think I m not sure but it looks like it s going to bleed when I pop them but no blood comes out Doctor: Thanks for asking in healthcaremagic forumIN short: DO not meddle it with if it is not causing you any problemExplanation: Usually visible sebaceous glands can be seen on penis like this. They are harmless. So, please visit your doctor for examination of the lesions and further suggestion. Hope this helps you. Good luck."
},
{
"id": 104211,
"tgt": "Have environmental allergies, nose dripping, breathing problems. Temporary relief with medication. Permanent solution?",
"src": "Patient: I was told I have allergies to things out side & I was taking allergy medicine.it helped till about 3 months ago.now my nose drips every day nothing is working! I have nose sprays ,i have done salt water & medicine . I dont know what to do now .I also have breathing problems now too.I cant work because I am to sick. I have no life because of this . Doctor: i give you temporary solution of medicines you can take off and on for as many number of days and as many times without any side effect but eventually you have to consult allergeologist to get allergy tests to find the cause and desensitisation therapy for permanent cure tab montair fx(combination of montelucast 10 mg and fexofenadine 120 mg) apply neosporin h eye ointment in nose bd put sea water drops 2 drops at night in each nostril syp tossex(combination of cpm and codeine) 1/2 tsf night this will protect you fully against allergies"
},
{
"id": 87034,
"tgt": "What causes pain in upper abdomen and upper back area?",
"src": "Patient: I have pain in my upper abdomen above my belly button,pain in my upper back between my shoulder blades and acid reflux regurgitation that tastes like vomit. I have Systemic Lupus,Psoriatic Arthritis and Fibromyalgia. What could the symptoms I named be? Doctor: Hi.Thanks for your query.The symptoms of pain in the upper abdomen above the belly button, between the shoulder blades and acid reflux regurgitation that taste like vomit is called GERD.GERD means Gatsroesophagral reflux disease. This may be due to the medicines you are taking for Systemic Lupus, Psoriatic Arthritis and Fibromyalgis.I would advise you the following:-First of all get the upper GI endoscopy done to see the exact problem , to see if there is Hiatus Hernia or any other associated finding.-PPI or Ranitidine -Antacid gel 3 hourly interval-Domperidone 8 hourly interval Soft bland dietSmall frequent mealsNo tobacco in any formNo alcohol or aerated beveragesNo late nights. All these things will help you for the other problems also."
},
{
"id": 165330,
"tgt": "Suggest remedy for injury on eyebrow bone",
"src": "Patient: Hi, may I answer your health queries right now ? Please type your query here...my five year old ran into a door knob which resulted in a huge contusion on the outside of his eyebrow bone. We applied an ice pack immediately which seemed to reduce the swelling some. How do we know whether he needs medical attention for it, and what can be done about healing other than time? Doctor: Huge contusion outside eyebrow bone needs to be clinically evaluated by a doctor.Consult your doctor. He will do detailed clinical evaluation and accordingly may prescribe some pain killer medication."
},
{
"id": 101887,
"tgt": "How to cure allergy from sweat?",
"src": "Patient: Hello Doctorfrom 2-3 Years during summer i am suffering from arms allergy when am sweating. Please tell me some precautions on this............also my skin is very sensitive as no artificial things like earings, bracelets, necklace etc. not suit me. I am also not able to wear watch and belt also.Thanking You Doctor: hi, you might be suffering from cholinergic urticaria also known as autologus sweat hypersensitivity.for its management you should avoid hot environment, heavy exercise , hot bath , sun exposure as much as possible. you should stay in AC room if possible to avoid sweating, rather you can use colt water spray or apply wet towel regularly to avoid sweating.you should go for total and differential count of white blood cells for ruling out basophilia.you can use antihistaminic like levocetrizine or loratidine or zileuton an antileukotrenes.. steroidal drugs and omalizumab an anti IgE therapy can be initiated but for that you should consult an immunologist. its other mode of treatment include desensitisation, steroid pulse therapy , danazol and propranol but consult a physician for their proper application. thanks for consulting HCM."
},
{
"id": 84730,
"tgt": "Suggest medication for amoebiasis",
"src": "Patient: Hi my baby is positive of g6pd. He suffer fro amoebiasis right now, can he take these medicines? metronidazole, nifuroxaxide, zinc sulfate and ornix. My email is YYYY@YYYY Thanks and your reply would be a great help to me and my baby.. Doctor: Hi,Of the listed medications, Nifuroxaide is not safe for G6PD deficient patients and hence it is not recommended. Rest of other medications are considered to be safe and hence can be given for the treatment of amoebiasis.Hope I have answered your query. Let me know if I can assist you further. Regards, Dr. Mohammed Taher Ali, General & Family Physician"
},
{
"id": 204622,
"tgt": "Is Ativan the right medicine for anxiety?",
"src": "Patient: My daughter was diagnosed with general anxiety disorder. She started taking 5mg lexepro 3/2. At the moment she is very distraught and crying. She was also given a prescription for Ativan? Generic is lorazepam. She just got this prescription and has not yet taken it. At the moment she said she doesn\u2019t need to take it because she doesn\u2019t feel anxious but feels depressed and doesn\u2019t think it will help. As it is very late and we are both tired, should I try to convince her to take it or see how tonight goes? Doctor: Hello,Ativan drug is for anxiety and sleep. It can be taken when required and not daily. Lexepro drug will take care of her depression symptoms.Hope I have answered your query. Let me know if I can assist you further.Regards,Dr. Rohit Kothari"
},
{
"id": 37009,
"tgt": "What does the testicular biopsy report indicate?",
"src": "Patient: sir i m azoospermic...i have gone for testicular biopsy,the result was germ cell aplasia and my doctor said to me that isnt any chance for us.... when i went for biopsy my harmonal reports were normal but i found my FSH level higher...what would be the beeter option for us...PLEASE HELP!!!! Doctor: HIThanks for posting your query to Healthcaremagic. Germ cell aplasia means the cells is your tested have not matured and cannot produce Sperm . I am very sorry to tell that nothing can be done for that . But I would like to go through the biopsy report and also the blood reports so that I can advise you if anything that can be done to help you . Kindly upload the reports. Contact me directly on this link . http://doctor.healthcaremagic.com/doctors/dr-divakara/68412Will be looking forward for your reply."
},
{
"id": 208819,
"tgt": "Suggest remedy for change in attitude and behavior during exams",
"src": "Patient: Gud afternoon sir,my sister is studying 1st btech.she score good marks in all internal exams.during final exams she said that i didnot remember anything what i read earlier,iam unable to read and understand,some times i got headache.We observe a changecin her attitude and behaviour.she said that iam feeling unwell.we r even unable to find the problem.please kindly suggest any solution. Doctor: HiThanks for using healthcare magicI think, she is in depression. May be due to study or stress, she is getting depressive symptoms. In that case, she needs some antidepressant with low dose benzodiazepine that would help her to come out of that situation. She can also try some relaxation exercise like JPMR or deep breathing exercise. Better to consult a psychiatrist for proper evaluation and management. In case, she needs further help, you can ask.Thanks"
},
{
"id": 127345,
"tgt": "Should an X-ray be taken for severe pain in the shin after a injury?",
"src": "Patient: Hi my daughter banged her shin on a metal cupboard at school (she is a 31 year old teacher!). She says the pain was excruciating and ice was applied. That was 9am today. She drive home. Now the pain has returned. Her shin is painful and there is a swelling. She has elevated her leg and applied ice. Should she be xrayed? Doctor: Hello and Welcome to \u2018Ask A Doctor\u2019 service. I have reviewed your query and here is my advice. You can go for an X-ray even though the chances for a fracture is unlikely. Most probably she might have suffered some kind of contusion or sprain. It will resolve with conservative management. You can give her analgesics like Diclofenac for pain. Apply ice packs and take adequate bed rest and do limb elevation. Generally symptoms will subside with in a week with conservative management. If pain persist despite all these, better to consult an orthopaedic and get an MRI scan done. Wishing you good health. Thanks."
},
{
"id": 140824,
"tgt": "What causes nerve catch sensation after stenting?",
"src": "Patient: after angiogrm undergon for two stunts on the left side of the chest used to feel nerve catch with out pain before stunt same feeling after stunting.suffered with leftside peralysys three years back leftside body pain still exists this nerve catch sensation due to heart related Doctor: Hi, If I understand your question I believe that the \"nerve catch sensation\" which is usually brief and sharply described pain is not likely heart-related simply based on your history that it existed prior to & after stent placement though it's not absolutely clear from what you've written that the catch ever went away after the 2 stents were placed. If you have any doubts about this pain being related to ischemia that is going on in the heart muscle itself then, you should mention this to the cardiologist who placed the stents. They may wish to give you a stress test or perform some other manoeuvres to make sure that blood flow to your heart is being adequately maintained. Hope I have answered your query. Let me know if I can assist you further. Regards, Dr. Dariush Saghafi, Neurologist"
},
{
"id": 171687,
"tgt": "What causes dark lips in a baby?",
"src": "Patient: Helo dr my baby girl is 4 weeks old and I am concerned about her dark lips and she was very jaundice too. She is up for a fourth blood test as her jaundice improving but they still want a double chek but is that why she has dark lips bcoz my other kids do t have it and I am Pakistani Doctor: Hi,Welcome to HCM,I understand you concern and I must assure you that your baby sounds to be totally normal. Dark lips can be there in neonates which usually resolves as the child grows. As long as baby is otherwise active, feeding well, rest body is pink in colour and not pale or bluish, passing urine and stools well, nothing to be concerned about. Hope I answered your query well. Let me know if there is any other clarification. Enjoy your baby."
},
{
"id": 59841,
"tgt": "Enlarged spleen, damaged liver, low WBC, platelet count, cold. Liver transplant done. Prognosis?",
"src": "Patient: I had a liver tranplant 12 years ago but the last 3 months my spleen is enlarged and I am waiting for my biopsy to come back I had a light and the dr told me that my liver is really damaged. The reason for my first transplant was I had primary billiary scherios they think it is back and if so I will not be going on a tranplant list. My stomach is enlarged and my white cells and platets are very low and I am cold all the time. I just got a cold can I fight this and how long can I live with this disease? I really need some answers Doctor: Hi your conditions is really is very bad. you stomach is enlarged due to ascities, spleen due to blood cells damage. you cannot go for the transplant anymore. you have to live with the current condition to remaining for your life. don't how much you could live. take the supportive treatment as prescribed by your doctor. Thanks"
},
{
"id": 207549,
"tgt": "Suggest treatment for feelings of anger and frustration",
"src": "Patient: I feel angry most of the time. Maybe disconnected. My stomach feels quite sick when I'm cross or angry. I don't understand, the little things really upset me. I feel lost and that nobody around me supports me. My fiance says I'm highly strung, I don't feel he listens to me and this frustrates me more. I don't feel heard and I can't release these feelings. What do or can I do?? Doctor: DearWe understand your concernsI went throughh your description. From the given symptoms, I can clearly see low self confidence and low self esteem. You should know the reason. Are you expecting more? Are you expecting more than what is your worth. If you are average and expecting to score 90%, may disappoint you. Trying to do a job which you do not like at all can also disappoint you. These disappointments together irritate you and you automtically become short tempered. Please understand you and your abilities. Act accordingly. Do not underestimate you. Do not try to see problems within you. Disappointment in life do create these symptoms and these are momentary. You should be able to overcome these within a month. If you want more of my services, please post a direct query to me in this website. Hope this answers your query. Available for further clarifications.Good luck."
},
{
"id": 199969,
"tgt": "Suggest treatment for itchy blister on the scrotum",
"src": "Patient: Hello I'm 18 y/o and I just noticed a single red blister on the side of my scrotum it's mildly itchy and gets irritated easy when it rubs on the side of my leg. I've had sex once but it was more than half a year ago so I don't think it's an std but I'm not for sure. I am pretty active at my job so I though it may be from sweat, heat, or even something with the hair rubbing or pulling. Please help I'd really appreciate it. Doctor: it is likely to be jock itch. if is assosiated with intense itch and no to mild pain.... probable it is jock itch.... you can over the counter medications for it. your active life and so probabily of excessive sweat can be the reason for it.it does not look like an STD to me."
},
{
"id": 165217,
"tgt": "Is slimy stools with chunks of yellow or green normal in a bottle fed baby?",
"src": "Patient: My son has had slimmy pool with white chunks its sometimes yellow or green. He pools after every feeding and it is very loud. He has had sores on his bottom from the poop. Is this normal for a bottle fed baby. It was not like this before and he has had this for almost a month now. Doctor: the botl feeding infant his lotion more than normal breast feeding infant so don't be warry according the colour of stool you must do stool analysis only"
},
{
"id": 179315,
"tgt": "How can abdominal pain with fever, vomiting and loose stools be treated?",
"src": "Patient: hello sir, my child is 3 yrs old and she is having abdominal pain and fever since last 2 days. she vomited once 2 days back and has had 3 to 4 loose stools. fever is mild but she is having intermittent recurrent abdominal pain off and on. she is taking her feeds but complains of abdominal pain...please suggest a medicine. Doctor: Give syr crocin if temp is more than 100F. Normal diet has to continue. Give plenty of fluids including ORS. Syr ondem 5 ml if vomiting is severe. Syr decolic 5 ml if pain is severe. Syr zinconia 5 ml for 14days. No antibiotics required, as high chance that its viral infection."
},
{
"id": 72301,
"tgt": "Suggest treatment for chronic cough",
"src": "Patient: Hi, my mother is diabeti and has hpertension, she is taking tablets on a daily basis and she started coughing after she was diagonised with these chronic diseases and has ben coughing ery hard ever since. What type of cough syrups can help her please advise Doctor: Thanks for your question on Healthcare Magic.I can understand your concern. Chronic cough in diabetic patients is mostly due to lung infection (pneumonia or tuberculosis) and bronchitis.So better to take her to pulmonologist and get done clinical examination of respiratory system, chest x ray and PFT (Pulmonary Function Test).Chest x ray is needed to rule out lung infection. Pft is must for the diagnosis of bronchitis. She may need antibiotics, inhaled bronchodilators and other supportive treatment. Don't start treatment without diagnosis. Hope I have solved your query. I will be happy to help you further. Wishing good health to your mother. Thanks."
},
{
"id": 69353,
"tgt": "How to treat a lump near the upper corner of eyebrow?",
"src": "Patient: I was hit on the head from a fist fight around august. i had a huge lump on my head, that is gone now but sometimes the area where the lump was at feels sensitive when i make facial expressions. Its on the right upper corner by my eyebrow and it feels like theres a slight lump. could it be scar tissue ? Doctor: Yes, it can be a scar tissue only.Local examination by your doctor will be helpful.In my clinic, I give intralesional steroid injection if its a keloid.Thanks. Regards."
},
{
"id": 78261,
"tgt": "Difficulty in breathing post septoplasty",
"src": "Patient: hello dr,sumit i am dr. pradosh from jipmer. i have grossly deviated nasal septum to left.. i hav undergone septoplasty 10 yrs back. still i dont feel any improvement. i even cant breathe properly through left nostril.. frequently i am having fever and worse headache for this. sir i want a solution for this. my consultant told me to again go for septoplasty. but i am afraid whether it will help. plz give ur expert opinion..thank u ... Doctor: Thanks for your question on Health Care Magic. I can understand your concern. Best treatment for deviated nasal septum (DNS) is surgical correction (septoplasty). DNS is fixed mechanical obstruction. Without correcting it, your nose will not be opened. So repeat surgery is needed. Better to get it done through experienced ENT surgeon. Also start 1. Plenty of fluids orally and keep yourself hydrated 2. Steam inhalation 5-6 times a day. 3. Intranasal spray containing steroid (fluticasone).4. Oral combination of antihistamine and anti allergic drugs. All these and septoplasty by experienced ENT doctor will definitely help you. Hope I have solved your query. I will be happy to help you further. Wish you good health. Thanks."
},
{
"id": 71470,
"tgt": "Suggest treatment for high globulin and alkaline phophatase levels",
"src": "Patient: I AM ON AKT4 FOR LAST 11 WEEKS AND HAVE TO UNDERTAKE IT FOR ANOTHER 13 WEEKS (TOTAL 24 WEEKS PROGRAMME). AKT-4 IS TAKEN EMPTY STOMATCH. I GOT MY LFT DONE RECENTLY AND THE UNFAVOURABLE RESULTS WERE AS UNDER :- Globulin 4 (1-3)- A/G Ratio .95 (1.2-2.0)- Alkaline Phasphatase 191 (60-170)Other results in the LFT tests were favourable.I am taking Pantocid as antacid with AKT-4. Kindly suggest whether the treatment is going in a right direction or not.-HARI SINGH BISHT (FARIDABAD) Doctor: Hello,AKT 4 is having rifampicin, ethambutol, pyrazinamide and isoniazid. Amongst these four drugs, rifampicin, isoniazid and pyrazinamide are hepatotoxic (damages liver). Main hepatotoxic effects reflect as raised Bilirubin and Liver enzymes (SGPT and SGOT).Since, you are these reports are normal, no need to worry for AKT 4 induced hepatitis. Raised alkaline phosphatase (ALP) is seen with obstruction in biliary tract. So, it is better to get done ultrasound examination of abdomen to find out cause for raised ALP.Hope I have answered your query. Let me know if I can assist you further.Regards,Dr. Kaushal Bhavsar"
},
{
"id": 91864,
"tgt": "What could on and off sharp, pressured feeling in lower stomach going to tailbone indicate?",
"src": "Patient: Hi I am having a lot of pain. It starts at my lower tummy close to my croch pretty much and it feels like pressure building then it gets really painful like sharp pain then the pain is still there but goes to close to my tailbone. It hurts so bad. It goes on for about 30 minutes repeating the cycle them it will stop for 2 hours then start back up. Can you help me with this? Doctor: There can be two reason for such pain. First it may be that you are having back muscle spasm, i mean your spine muscles are tight. It can be due to kidney stone also. So my first advice to you is to get a ultrasound and a spine x ray.If stone is present you need to show to a urologist. If there is no stone than do few things1.Avoid heavy weight lifting and forward bending.2. Sleep on hard bard.3.apply dicofenec cream4. sit in bat tub with gentle warm water so your tail bone get some heat.5. take muscle relxant like myoril and myospaz.6. learn back muscle strengthening exercises from physiotherapist."
},
{
"id": 166554,
"tgt": "Suggest remedy for diarrhea and cold",
"src": "Patient: Hi i took my 5 old month baby to see the doctor today as he has had dioreah since wednesday, also a cold which she gave me sme antibotics to use with him. but im worried beccause she said it will make him have more dioreah. i really dnt want him to get worse with his dioreah as all he is doing is sleeping all the time or crying the other. He also hardly taking any of his feed and when he does it comes straight back out. is there anything i can do to help? thanxs Doctor: Dear parent, for the diarrhea you should use an antidiarrheal medication together with oral rehydration solution 5 tea spoons for every pound of child's weight with every time he passes stool. make sure the child is adequately hydrated with enough fluids and milk. use an antiemetic to stop vomiting ( other than primperane as contraindicated in children) . try to offer only soft easily swallowed food as soup, youghurt and jelly . following these measures will not make the antibiotic worsen the diarrhea so continue on the antibiotic as prescribed by your doctor"
},
{
"id": 225956,
"tgt": "Bleeding after intercourse. Have implant. Could this be due to the birth control?",
"src": "Patient: can you think of any other reasons why this would be happening?. When the implant was first inserted, everything appeared fine. Then the problems started building up. Sometimes i'd finish menstrating, then the normal dis-charging would follow. After this i'd have sex for an example then i'd bleed again almost instantly and sometimes this would cause a mild discomfort.I'm not very good at explaining the symptoms i am currently experiencing , but it does bother me and has become rather embarassing during those times. Doctor: Hi, Based on the information you are having bleeding after intercourse and are having implant for the birth control. Most of the time the bleeding is from the trauma caused by implant, due to the thrust applied during intercourse. Advice:- please consult gynecologist and get it checked, due to risk of having infections.hope it helps.Dr.JAY PATEL"
},
{
"id": 110273,
"tgt": "What to do for the severe back pain?",
"src": "Patient: Hi! I am 30 Years old(Male) . I am suffering from Back pain. I did MRI and as per MRI report below : ( Impression: The Suggestive of degenerative changes in lumbosseral spine with lumbarization of S1. Postero-lateral disc herniations at L4-L5, L5-S1 and S1-S2 are compressing the thecal sac, couda equina nurve roots and both neurals foramina. There is secondary central canal stenosis at L5-S1 and S1-S2 with border line central canal stenosis at L4-L5. ). As per Neoro surgeon in Kolkata, it requires operation. Please suggest where is the best place to make operation of this. Please suggest Doctor: Hi ,Welcome to healthcare magic. YOU ARE SUFFERING FROM PROLAPSED DISC.Primary treatment of this is bed rest , analgesics (Diclofenac sodium50mg three times a day) methlcobalmine one tablet daily,but it takes longer time to relief . If one develop neurological symptoms then operation is required . This operation is everywhere in india . In my opinion you should done it locally because frequent follow up will be required so get operated by orthopedic surgeon of your choice ."
},
{
"id": 191211,
"tgt": "Is Glycomet advisable for diabetes in a child who is underweight?",
"src": "Patient: My son 10 years old was recenyly diagonosed with Diabetes initially we had given him insulin for 1 month but from past 5 months he is not requiring insulin but his sugar levels are not normal 7 not high basically levels of pre-diabetes Now some doctor has recommended Glycomet 250 mg tablet one daily with half an hour of jogging. He says it works as a supplement for Pancreas to rejuvenate and decrease chances fo Diabetes to re-occur & normalise our sugar levels .My concern is whether this is true does it help to revive Pancreas .The other concern I am having is that as this tablet is also given to children who are obese but my son is already underweight as his weight is 27 Kgs & height is 136 cms and ideally he should be 32-33 Kgs So please advise accordingly Doctor: Hello, First, Glycomet is safe but it does not affect pancreas in any mean, its action is through decrease appetite and absorption of food in the stomach. And as your child is already underweight, I do not recommend it for him. he may instead to keep on healthy diet and exercise. Hope I have answered your query. Let me know if I can assist you further. Take care Regards, Dr Salah Saad Shoman, Internal Medicine Specialist"
},
{
"id": 40903,
"tgt": "How to stop vomitings, restlessness and severe headache after taking injections for IVF treatment?",
"src": "Patient: my name is xyz i am undergoing IVF treatment now, i am taking gonapeptyl injection and nugon -450 injection, i am getting vomitting, i feel restless, severe headache also,my doctor asked me have plenty of water, but also it is not reduced, plz suggest me what to do for these problems Doctor: HiGreetings. These are hormones given for multiple egg retrieval for ivf or ICSI.Your symptoms are common to most of the patients. It is better to discuss with your specialist since she will be knowing your details of associated problems.Drinking more water is good , Taking medicines for headache is optional .There are medicines for headache and nausea,but should be taken after discussing with your doctor. hope you are convinced. Regards"
},
{
"id": 176730,
"tgt": "Suggest treatment for urinary incontinence and cough in a child",
"src": "Patient: My son is 22 months and been generally irritable, clingy, tantrums for past couple of weeks now. He has had a cough, now is urinating far more often ie every 20-25 mins, drinking a lot. He vomited yesterday and tonight ge is cold and clammy with very low temp 34.3, help Doctor: Hi...by what you quote I feel that he might be having a urinary tract infection (UTI). I suggest you get a normal urine routine analysis done and see your paediatrician with it. If it is suggestive of UTI, you may need to go ahead with a urine culture and oral antibiotics too.Regards - Dr. Sumanth"
},
{
"id": 145306,
"tgt": "What causes dizziness and vomiting?",
"src": "Patient: I was totally ok before went to bed last night. Woke up 2:30 to go to bathroom , felt the room was turning around. I walked back to bed,continue sleeping. Woke up 7:50 am , still dizzy and felt like throwing up. Been vomited some fluid till now, still weak n felt the same way. But not that dizzy. I m a runner, I did get dehydrated couple times before, but not dizzy, only vomiting n diarrhea. Doctor: Hello!I think your symptoms may be related to a vertiginous syndrome. It may be caused by a dysfunction of your inner ear or the brain. I would recommend to consult with a doctor and have a physical examination and a full blood work.You can start cinarizine or betahistine.If the problem persists I would recommend a brain CT scan or MRI to rule out possible cerebral lesions.Hope to have been helpful!Best wishes Dr. Abaz Quka"
},
{
"id": 216125,
"tgt": "What causes sharp shooting pains in anus?",
"src": "Patient: Hi, I keep getting sudden sharp shooting pains in my anus when standing. It s so bad that I can only describe it as a knife up there! When I sit down I get the same feeling but as though it s in my vagina. My tummy feels swollen & It comes and goes. What could it be? Doctor: Hello,As a physician, as you have sudden sharp shooting pain in the anus, my probable diagnosis is anal fissure or fistula or hemorrhoids or trauma.Can I come to know whether you have blood in stool or complaints of constipation?I will suggest you to consult with surgeon for per rectal examination. That will give a clear cut diagnosis. I will suggest you to apply Anovate ointment locally and tablet Dolo.I will also suggest you to review here with detailed history or consult with the surgeon.Hope I have answered your query. Let me know if I can assist you further."
},
{
"id": 469,
"tgt": "What is the treatment to make follicle grow?",
"src": "Patient: Age:27 Height 5.1 weight 64kg hi , i was trying for baby for 11 months and i have no growth in my follicle (12mm) in current follicle cycle.i was asked to use ovrall g for 3 periods cycle. Doctor says this will suppress the ovulation for 3 months and there will be suddent rebound in follicle development in 4th month WIll it make my follicle grow?. Doctor: Hi, I think taking 3 months of ovral g can help in increased response of follicles in next cycle. But you should also get yourself evaluated properly. Do a thyroid profile and prolactin level and a ultrasound done to see uterus and ovaries. It will rule out any thyroid disorder or polycystic ovarian syndrome. If follicles are not forming, you can take some medicines like clomiphene for the growth of your follicles. Talk to your doctor regarding this."
},
{
"id": 217648,
"tgt": "What is prognosis for necrotic chronic pancreatitis with 40% scarring?",
"src": "Patient: I have been diagnosed with necrotic chronic pancreatitis but I have never drunk alchohol. I have now lost 4 stone in weight over the year and have chronic pain. What would help these symptoms? They are saying my pancreas is 40% scarred so what prognosis do you think I have? Doctor: A serious concern especially if the cause is not known.Gallstones can irritate, but do not often cause any scarring. Elevated triglycerides are problaby the most common cause after alcohol or illegal drugs and are both highly treatable/controllable, and likley to have significant healing with treatment. Prescription drugs like common blood pressure medications (HCTZ) sometimes cause pancreatitis. So can minor viruses. But they shouldn't be present long enough to cause scarring."
},
{
"id": 169766,
"tgt": "What causes severe pain in the leg of a 3 year old?",
"src": "Patient: My 3 year old son has been complaining off leg pain at night specific to his right leg on and off for the last few months. We brought him to the hospital and he has been referred to an endocrinologist and an orthopaedic dept although we were told in A&E they coudln t identify any specific problem at the time. The appointments are not for a couple of months and in the past week he has started to have nose bleeds, Should I be concerned and if so why? Doctor: Hi, I don't think that nose bleed has any relation to leg pain. Nose bleed can occur occasionally in children. Advice child to stop nose picking. It will help. Take care."
},
{
"id": 24221,
"tgt": "What causes chest pains with blood in mucus?",
"src": "Patient: hi,i am 22 years old,female and i am smoking from 1 year and from 2 weeks i was suffering form cold and cough,sometime my chest also starts paining while coughing violently and sometime i throw while coughing but to my surprise today while coughing i throw again and when i spit over my vomit i see two drops of blood on there,i am so afraid please help me!and i have no medical history too as far as i know!thanks! Doctor: Brief Answer:Your symptoms are more likely caused by acute bronchitis.Even though, you should see a pneumologist.Detailed Answer:Hello,Thank you for using HCM. I understand your concern.A cough is a common cold symptom. It's the body's way of getting rid of phlegm or mucus. But as your cough persists and is associated with dyspnea (respiratory difficulties) and coughing up blood, you need to see pneumologist. He will make a accurate physical examination of the lungs and will guide you into further examinations needed.You may need a chest radiography, full blood count and sputum examination.There are different causes of coughing up blood such as: bronchitis, bronchiectasis, pneumonia, some reumatologic disorders, pulmonary thromboembolism ect. Bronchitis is the most common cause and the most probable in your case. Coughing up blood due to bronchitis is rarely life-threatening. So don't be to much worried. Your symptoms can be made worse by smoking. So, don't smoke and don't allow others to smoke in your home.Stay away from or reduce your time around things that irritate your nose, throat, and lungs, such as dust or pets.Hope this is helpful.Let me know if you need more clarifications. Otherwise please close and rate the answer.Kind Regards, Dr. Juarda"
},
{
"id": 82668,
"tgt": "What causes itchy lumps on thighs with history of lupus?",
"src": "Patient: Hi, I have these lumps on my quads. Years ago my doctor said they re just fatty tissue, nothing to worry about, however, they hurt and itch. The lumps were first on my left thigh and now there growing on my right thigh. I am 51 years old and am very active. I had tested positive years ago (1994.95) for Lupus as it runs in my mom s side and I was told I have FMS, but no testing was done for that one. Doctor: Your history is not suggestive of lupus. However in view of positive family history and ANA positivity, Kindly get FNAC of lumps done to rule out panniculitis."
},
{
"id": 139976,
"tgt": "What is the treatment of dizziness ?",
"src": "Patient: sir, i am 35 years old women three days before i had a new experience with diziness and vomitting i never get this experience in my life.just newly married for 6 months ago(late marriage). idon t know the reason.now i ma taking the tablet vertin 16 mg.i wnat to know the reaon why it came. Doctor: Hello, I would explain that your symptoms could be related to an inner ear disorder. I agree with the treatment with Vertin. But, you should try to be patient as this situation may last up to a month. Consulting with an ENT specialist is necessary. Hope I have answered your query. Let me know if I can assist you further. Take care Regards, Dr Ilir Sharka, Cardiologist"
},
{
"id": 44760,
"tgt": "How can I convert pmol/L ng/mL and what are the normal values in the 1st trimester ?",
"src": "Patient: how to convert pmol/L to ng/mL?What is the normal values of first trimester? Doctor: Hello.Welcome.I have read your question three times but can't make out what you are asking.Please be clear in what you ask for.Good luck."
},
{
"id": 160100,
"tgt": "I have low SGOT, WBC, SGPT, what does it mean?",
"src": "Patient: I receive my blood work back today and it reads my SGOT (AST) a low 3 WBC a low 4.3 SGPT a low 39 I am 37 year old mother of 4 children. I had the blood work done for my insurance. I am 5 6 and 115lbs What does this mean? Doctor: Thanks for the query Low wbc might be caused by a recent viral infection. You can help the liver function by taking supplements like tab hepamerz or tab livopill ds once or twice a day. I dont think u have something to be worried about. Repeat the tests after two weeks, including the entire CBC Have a healthy living"
},
{
"id": 121894,
"tgt": "What are the complications of hemorrhagic contusion ?",
"src": "Patient: A small hemorrhagic contusion is noted in the left high frontal parafalcine region. This is what the ct scan report says for my brother who had met with a road accident. He had fallen down from a bike. Please clarify how serious this is and would it have further complications in future. Thanks. Sanjiv Doctor: Hello, The hemorrhagic contusion occurs in 20 or 30 %of the injuries. The contusion will recover gradually. Soft tissue\u00a0contusions\u00a0can\u00a0take\u00a0anywhere from a few days to a couple of weeks to\u00a0heal properly. Meanwhile, I suggest to rest, use ice compresses and use Ibuprofen in case of pain. Hope I have answered your query. Let me know if I can assist you further. Take care Regards, Dr Dorina Gurabardhi, General & Family Physician"
},
{
"id": 118781,
"tgt": "Elevated hemoglobin. History of radium implant exposure. Sign of leukemia?",
"src": "Patient: I have a wbc of 12.6. It was 11.5 6 months ago. my hemoglobin is elevated also.6.5. Is this a sign of leukiemia? I am 71 years old female and have had thyroid issues for some time. It was burned out with liquid Iodine several years ago.I was also exposed to a raduim implant for 2 days 41 years ago.Thank you so much for your time. Doctor: Hi and thanks for the query,A proper clinical review is first important. Diagnosis of leukemia requires a thin blood film and a bone marrow aspirate for examination a pathologist or a hemotalogist. Indirect measures like raised white blood cells and raised uric acid levels could also be used. A clinically important sign is actually fatigue, easy bleeding and chest pain.The opinion of a hematologist and an internist could be of great help. Thanks and kind regards,Bain LE, MD."
},
{
"id": 199486,
"tgt": "What should be the normal value for semen analysis?",
"src": "Patient: I m 31 , married 4 a year now wt no child,I did a sperm analysis test n I av over 20million sperm wt traces of staph aures but d doctor gave m an injection and Augmentin tablet,but we still tryn 4 a child and I m living a. Healthy live wt fruits,suppliments n exercises.pls doc wot do I do? Doctor: Hello You may need recent semen analysis.Findings suggests normal sperm count.It should be at least 20 millions/ml.You were rightly treated with antibiotics for semen infection.It is important know other parameters of semen analysis like sperm motility,morphology etc.You may need investigations like routine hemogram,random blood sugar,semen culture and sensitivity,ultrasound of scrotum.You are rightly taking nutritious diet and doing excercise.Fertility is expected to increase with above measures.Your wife also need proper evaluation.You should consult renowned infertility center. Get well soon. Take Care Dr.Indu Bhushan"
},
{
"id": 195764,
"tgt": "What causes semen discharge after bowel movement?",
"src": "Patient: I experience a semenal discharge from my penis after a bowel movement, more-so when excessive wiping is needed. There is no pain in urination. Not sexually active. After the discharge often I will feel \"blue balls\" for an extended period of time (5 minutes to an hour). Doctor: Hello and Welcome to \u2018Ask A Doctor\u2019 service. I have reviewed your query and here is my advice. If the frequency of masturbation or sexual intercourse is less, there is a possibility of night emissions or discharge while passing motion as motion can squeeze the prostate partly.This is not abnormal and I don't thing you need to worry about it. Hope I have answered your query. Let me know if I can assist you further."
},
{
"id": 41966,
"tgt": "What causes difficulty in becoming pregnant?",
"src": "Patient: hi! my name is ria my age is 23 yr,my height is 5'2' ' .my question to you is i may married for 3 year i a,m trying for a baby since three yr ultimatly i am unsucessful my weight is 64 kg ihave nothing such medical problem can u plz tell me what is my problem before i visited to a doctor he suggest for weight reduction so plz help me? Doctor: Hi, thanks for writing to health care magic.I have gone through your question.You are trying for last 3 years to conceive. Have you done any kind of reports?? If no, then should consult a gynecologist and do some simple reports like 1. Ovulation profile 2. Hsg histo salpingo graphy if needed. 3.thyroid profile. These reports help to find any abnormality if it is there.If needed then you should do semen analysis of your partner.As far as your weight is concerned 64kg is not a matter of concern.Have a healthy life..."
},
{
"id": 120493,
"tgt": "What could cause spasms under the left ribs?",
"src": "Patient: I have been having this strange feeling as well, the butterfly, twitch, spasim what ever that feeling is under my left ribs. I have had a ultra sound done just days prior to the sensation comming on for a routine check up and everything was fine. I had blood work done also and all was well, I also had a colonoscopy and the other end too, again all was fine. My Doc says that the feeling is do to the high level of Anxiety that I have, but I have been close to Anxiety free for a week or so now and the flutter is still there. I went and got a 2nd opinion and he looked at my chart and said that it started about the same time I started Buspar(for anxiety) and Kapidex(for acid reflux), so he is having me stop the Kapidex for a week to see if anything changes, I guess he thinks that because it is such a weird symptom that it could be a side effect from meds. I feel what you are all feeling and am very supprised that for as long as this post is going no one has came back and shared the wealth of how the Dr. visits went or a diagnoses. On one hand I am glad to see that there are others like me so its just not one more thing ill feel weird about and alone, but on the otherhand I feel bad that you guys have this CRAP too. So if any one has info or a diag please feel free to share... Doctor: Hello,I read carefully your query and understand your concern. The symptoms seem to a muscle spasm in the ribs area.I suggest using a spasmolytic such as Baclofen three times a day. I also suggest using magnesium supplements for muscle relaxation. I recommend using warm compresses for local application. Hope my answer was helpful.If you have further queries feel free to contact me again.Kind regards! Dr.Dorina Gurabardhi General &Family Physician"
},
{
"id": 205451,
"tgt": "Suggest remedy for insomnia",
"src": "Patient: Hi i developed insomia 2 1/2 months ago. Physiciatrist does not want to give me antidepressants he believes i can get out of my anxiety alone. I went to mexico and im taking cinatrizina in spanish as a sleep aid. He said that should help the flow in my brain due to stress. Sonething like that. Do you believe i should continue like that. Its been a week only. Doctor: DearWe understand your concernsI went through your details. If your insomnia is due to stress and anxiety and obsession, there is no need for you take sleep aid medicines. You just are suffering from this problem for two and half months of so. That is reasonable time period. Instead of worrying, you should take this on its stride. It is not that you are unable to sleep whole night. You do sleep for three or four hours per day, but you don't seem to be fulfilled. Not having sound sleep. You also feel tired during day time. If yo do suffer from sleeplessness for two days, automatically on the third day you will sleep sound. You should understand this pattern and follow it. Slowly body will also understand.Physical exercise, being creative and entertaining, meditation etc can help you regain your sleep. Please consult a psychologist for counseling and necessary therapy.If you require more of my help in this aspect, please use this URL. http://goo.gl/aYW2pR. Make sure that you include every minute detail possible. Hope this answers your query. Further clarifications are welcome.Good luck. Take care."
},
{
"id": 7456,
"tgt": "Pimples, acne, sensitive skin. Regular periods. Want lotion to remove dark spots and improve skin quality",
"src": "Patient: Hello Doctor,I am suffering from pimples and acne . My skin is also vey sensitive . Which soap can i use and please tell me any lotion also which helps to remove dark spots and can improve skin quality also. I am suffering from pimples from my teenage. Now I am 25 yeras old. My periods are regular and i am married . Sometimes i get stressed out due to which pimples come on my face. I drink lots of water. Please help. Doctor: Hi, You should use Cetaphil cleansing lotion for face wash. For your marks and skin texture, you should meet a qualified dermatologist. Any creams containing tretinoin, hydroquinone, glycolic acid, azelaic acid, kojic acid would be good for you. But if you are still getting pimples, then you should treat them also so as to prevent any future marks. Always use a broad spectrum sunscreen as long as there is daylight outside. Happy new year Ha"
},
{
"id": 187473,
"tgt": "Can I take pain killers, pull my teeth and then seal it with a hot wire?",
"src": "Patient: Hi Dr Sabmbl I'm Glmmn , I have a broken tooth in my top right 3 from the back. I have no money to pay 4 help. So I am planning to take a lot of pain killers. & try to pull it out ,with what I have in my . Tools. & then seal the nerve with a peace of hot wire. Not good I do know this but need to do some thing. Regards Glenn. Doctor: hello,welcome to HCMif you dont have money then you can go to govt hospital and get the tooth extracted.dont follow the procedure you are talking,its not only painful but harmful also. taking out tooth by doctor will not cause you pain thank you"
},
{
"id": 205378,
"tgt": "How to overcome severe depression caused due to tragic loss?",
"src": "Patient: My daughter is 40 years old and has experienced a tragic loss in the last year. Her loss was her spouse of fourteen years. I thought that it was heart wrenching grief as occurs in such loss, but it has escalated into something else. She thinks all of her family members want only her inherited finances that are supposed to be used for her and her sons livelihood. She has been spending and giving away her finances and then she explodes over the money spent. She violently accuses us of wanting to destroy her. We want only to help her but she insists we are just using her till the money is gone. Is this a form of severe depression? Bipolar? Schizophrenia ? I could use some advice on how to help her. Doctor: DearWe understand your concernsI went through your details. This has to be Post Traumatic Stress disorder (PTSD). Depression and associated symptoms are common with PTSD. Usually calming medicines, psychotherapy, relaxation therapy, yoga and meditation are used combine to treat such cases. still for your confirmation you can approach a psychologist for a detailed assessment of the case.If you require more of my help in this aspect, please use this URL. http://goo.gl/aYW2pR. Make sure that you include every minute details possible. Hope this answers your query. Available for further clarifications.Good luck."
},
{
"id": 190289,
"tgt": "Taking Gabapentin for trigeminal neuralgia, lump on gum. Which doctor should I visit?",
"src": "Patient: Hi I was diagnosed with Trigeminal neuralgia 2 years ago. It went into remission after several months of Gabapentin . So when I thought it came back I began the gaba again with my neurologist . I just found a lump on the top of the gum on the inside of my mouth on that same side. I think it is the cause of my pain. What kind of Dr should I see. A dentist? A periodontist or a MD? By the way my maiden name is Greiff. It wasn t easy by at least it wasn t grief. Sorry. Doctor: drugs causing gingival overgrowth are - calcium channel blockers, nifedipine, amlodipine , cyclosporine and phenobarbitone. so check out with your earlier physician as to whether you were prescribed any of these drugs. and do not just leave gabapentin at once mr. greiff. consult your physician before you stop it or start it."
},
{
"id": 668,
"tgt": "How long after abortion is trying to get pregnant again safe?",
"src": "Patient: Hi, I am 43 years old, I conceived in Dec-Jan 2010. However, it was a missed abortion on Mar 9th as the heart beats had stopped in the 6th week of pregnancy. I have delivered a boy 18 years back (normal delivery). I wish to conceive again. When should I try again?? Thanks. Doctor: Dear,Chance are less but you can try as your periods are still going on. why chance are less because it age factor. we can try as your periods are still going on, so you can take medicine to conceive -tab- fertomid 50 mg, 2 tab per day.(on 5th day of your periods till 10th day)after that you have to intimate with your husband for 1 week daily.Thanks"
},
{
"id": 133920,
"tgt": "What can be the reason for the pain under the rib cage?",
"src": "Patient: I have pain when I touch the area just under my rib cage on the right side. I went to a clinic in February because I was shovelling and jammed the handle into my stomach and the doctor said he didn t think it was anything serious and should get better, sometimes I don t notice as much until I push on it and it still hurts- any suggestions would be great Doctor: hi,thank-you for providing the brief history of you.A thorough musculoskeletal assessment is advised.As mentioned the pain is in the right side of the rib and your physician it is not a serious one, I still feel that you can take x-ray of the ribs and see if hairline fracture of the ribs is there. As this is common at times and recovery gets delayed. Also, post this, performing proper breathing exercises your muscles will become stronger and bone will also heal. As hairline fracture is not a big thing and bery little ortho can do due to the anatomy of the bone.Also, breathing exercises to perform are -* Deep breathing exercises* Purselip breathing exercises* diaphragmatic breathing exercises* abdominal breathing exercises* costal breathing exercises.To note the important thing - while performing breathing exercises kindly place the hand over the paigul region to avoid pain triggering.In case, in 5-7 days you don't feel improvement than taking an abdominal scan is advised since you injured your abdomen region to see any systemic illness. usually in my clinical practice patient recover with breathing exercises in 5-7 days and continue same for 4 weeks further.RegardsJay Indravadan Patel"
},
{
"id": 78230,
"tgt": "Is there any need of INH in case of borderline TB igm?",
"src": "Patient: hi.. m a doc with tb lymph on treatment.. my spouse is keeping unwell for quite some time..to b precise he complains of chills at night... got all possible prelims done..the pulmonologist suggested.. cbc,esr,crp,widal,malarial smear,urine routine,lft... all normal.. but the mountoux came in strongly positive an induration of 15mm..he isnt having any cough,phlegm, etc..chest x rays look perfectly right too.. lymph nodes normal..so the microbiologist suggested a tb igm.. which gav a borderline 0.8 result(normal is 0.0 -0.8).....hes maintaining a temperature chart.. temps in normal range sinc a week...wondering whether to unnesseraily put him on INH?.....MEANTIME I AM NEARING THE END OF MY INH TREATMENT.. AND AM KEEPING PRETTY WELL.. Doctor: Hi,Welcome to HCM,TB Igm borderline reactivity with MT positive strongly in countries where TB is endemic than no treatment is required but in countries where incidence of TB is low wld require inh treatment."
},
{
"id": 18839,
"tgt": "What does this EKG report indicate?",
"src": "Patient: I went in today for a pre-surgery check with my primary care doctor (family practice). I am scheduled to have a C3 through C7 anterior disc fusion on April 5. After my doctor checked me out, he indicated that he cannot clear me for the surgery until I see a cardiologist. He gave me a printout of my ekg that says st junctional depression non specific. I am supposed to get the clearance for the fusion two weeks prior to the surgery, but an even bigger concern is the abnormal ekg. I am a 50 year old white male who is treated with medication for high blood pressure and very high cholesterol. Is the st depression indication something that should alarm me? Or is it a common incident in someone of my general health? Doctor: Hello,Since this is the major spine surgery you're scheduled for and your family doctor wants to make sure that you don't have any underlying cardiac issues. Though ST-segment depression >1 mm is significant and may indicate compromised blood flow to a specific portion of your heart. But before jumping to any conclusion it's better to visit your cardiologist. He may advise some more cardiac-related tests including Echo and Excercise/Persantine Stress test to get a better view of your heart. And will clear you for your surgery based on the results. Good Luck with the procedure.Hope I have answered your query. Let me know if I can assist you further.Regards,Dr. Ameet Kumar"
},
{
"id": 187760,
"tgt": "Will smoking have any effect on the teeth extraction surgery?",
"src": "Patient: Hello, I am having 4 wisdom teeth pulled tomorrow morning at 9 am. I have smoked cannabis and tobacco 4 days ago (last Thursday). I just wanted to make sure this will have no affect on my surgery. Also I was wondering how long after surgery until I can resume cannabis use. Doctor: Dear user,Thanks for using healthcare magic.I had gone through your query and understand your concerns.Smoke with cannabis and tobacco is not effective after 4 days,if you are abstinent,Cannabis inter fare with wound healing,increasing the infection risk,So you should discontinuing the cannabis until wound heal proper,Wish your good health,'Hope I have answered your query. If you have any further questions I will be happy to help\". Thanks."
},
{
"id": 203431,
"tgt": "What do you suggest for my scrotum that has grown to a size of a grapefruit?",
"src": "Patient: my scrotum has grown to a size of a grapefruit,i am 79 years old and my doctor seems to think that at my ageI shouldn't worry about it unless it bothers me.it is starting to bother me a bit but it is not painful I would like a second opioned on this matter. Doctor: Hi,having this problem at this age requires evaluation of your case.There are possibilities of having,1, Hydrocele,2, indirect inguinal hernia,3, some space occupying lesion.Go for Ultra sound and get it diagnosed.Consult urologist and get examined, physical examination will good information about your problem.Ok and take care."
},
{
"id": 198534,
"tgt": "What causes enlargement of scrotum with water retention?",
"src": "Patient: My husband, age 60, diabetic, obese, amputee RBK, and partial left foot, has an enlarged scrotum. It started rapidly growing about 5 weeks ago. it is now bigger than a bowling ball! It is retaining fluid. Went to hospital ER 2 weeks ago. They did blood tests. Negative for infection. They did Ultra-sound. Male organs are fine, but It has fluid around them. Growing about 1/2 inch per day. On-call Urologist said he couldn t do anythings for him. Regular doctor gave more water pills, but had to back off due to leg cramps and blood test that showed kidneys were working hard. Is it related to Lymph Nodes? What causes this? What can we do about it? PLEASE HELP! Doctor: Hello, Thanks for posting in HCM.From what history you are giving me it look slike he has hydrocele. It is abnormal collection of fluid in scrotum due to various reasons.The only treatment for it is aspiration of fluid and surgery so as to reduce the size and depending on the reason for fluid accumulation it can be repaired.As you already have got done a ultrasound you need to consult a general surgeon for this problem.I hope i have answered your query well. Thank you"
},
{
"id": 209911,
"tgt": "Suggest remedy for anger management",
"src": "Patient: hi have a roommate easy angry, he light turn on all the light, forget turn off the stove and he cook burning many time, i keep eye on him when i at home. i don't know how can i test him does he have mental problem. what number can i call, and how can i find out he get mental problem. he told me he have medical problem. i d Doctor: Hi..Based on your provided information, your roommate is seemingly having difficulty in attention-concentration.Plus his irritability is seemingly more due to sense of feeling overwhelmed/unable to cope to life's demands. He might be possibly suffering from major depressive episode. Hence, it is prudent on your part that you should encourage him to talk about his difficulties, convince him to visit a Doctor preferably a Psychiatrist for getting a formal evaluation & treatment.Lastly I would like to congratulate you both for your concerns & your vigilance for well being of your roommate. Good luck"
},
{
"id": 186844,
"tgt": "What could the hard ridge on the roof of mouth causing pain indicate?",
"src": "Patient: I FEEL A HARD RIDGE IN THE CENTER OF THE ROOF OF MY MOUTH. IT HURTS WHEN I EAT. NOT TERRIBLY BUT BOTHERSOME. NO DRAINAGE. NO FEVER. NO OTHER SYMPTOMS. WENT TO DENTIST LAST WEEK AND WAS TOLD EVERYTHING WAS OK. DID NOT MENTION THE ROOF OF MY MOUTH BECAUSE IT DID NOT BOTHER ME. WHAT IS IT? WHAT SHOULD I DO? Doctor: Hi,Thanks for posting the query, I would suggest you to take an x-ray of the region this could be due to infection in any nearby tooth, any bony protuberence causing discomfort in the region , any beningn lesion. I would suggest you to visit to a Oral Surgeon and get a second opinion.Take care!"
},
{
"id": 49813,
"tgt": "Elevated uric acid levels. Is it advisable to take Formula 1 shake and Personalized protein powder from herbalife?",
"src": "Patient: i am herbalife distributor. I have one customer who is having uric acid levels high and she is restricted from protein. Can I suggest her to go for Formula 1 shake (one shake contains 9 gms protein) along with Personalised Protein powder(1 spoon contains 5 gms protein it is a combination of Soy protein isolate & whey protein ). Pl advise Doctor: Hi. First you have to check how much is the uric acid elevated. If it is pathological it needs to be evaluated for the possible cause. There is no need to completely restrict the protein. She can take her daily requirem Of protein which is 1 gm/kg of body weight. She can take that amount through any means. Please consult her physician before offering her any treatment.Thank you"
},
{
"id": 167143,
"tgt": "What causes constant coughing with burning and swollen eyes?",
"src": "Patient: Hi, may I answer your health queries right now ? Please type your query here...My 5 yr old son is not well with a few days, constant coughing, he s burning up, hot to touch and red watery swollen eyes, but his temp is normal 37.4 and 37.2.....does he require a doctors visit??? Doctor: HiI understand your concerns but don't worry. I would recommend you to visit a paediatrician for clinical evaluation as the illness can be due to several reasons which needs proper clinical evaluation. Take care"
},
{
"id": 224428,
"tgt": "Taking Loestrin, bulimia. How long does BC pill take to dissolve?",
"src": "Patient: i am taking loestren 24 at about 12 pm everyday but I also struggle with being bulimic at times. I do not do it as often now but usually how long does it take for the birth control pill to dissolve? it has been a few weeks since I have thrown up but I want to know and to make sure I am never throwing up the pill if i do have a relapse Doctor: hi dear and thanks for your query.If you are diagnosed with Bulimia taking oral birth control is not for you.Hence contact with your doctor to change birth control.The best for you is from intradermal patches to IUD.ALL THE BEST"
},
{
"id": 82864,
"tgt": "What tests are required to confirm the diagnosis of lupus?",
"src": "Patient: LET Lupus Tumidus. Just received diagnosis few minutes ago. However there is an amount of uncertainity. -CWhich kind of blood tests should i do in order to have diagnosis confirmed?br -What are the chances this LET will evolve into LES. I read there are very few chances but what s Yr opinion? Thanks ( 55 Y.O / male) Doctor: Higo for ANA, If negative, means you donot have lupus. If it comes positive, than probably you have some chance of lupus, but it is confirmed by further test and clinical exam.I advised you to see a rheumatologisttake care"
},
{
"id": 136918,
"tgt": "Suggest treatment for swollen and painful feet and ankle",
"src": "Patient: I am 39 years old 5 foot tall and weigh 265 lbs. I have gained over 50 lbs since August 2014. I am tired all of the time. I recently have swelling in my feet and ankles. I wake up several times a night. I fall asleep at the drop of a hat. I have shortness of breath. I am wondering what may be wrong with me. Doctor: Hello, Thanks for writing to us, I have studied your case.I will like to ask few questions so that I can help you betterSwelling is on both feet?Swelling is reducing after lying down?Any history of injury?Both feet involvement differential can be swelling due to exertion; swelling can be cardiac /renal origin, early inflammatory arthritis.You may need to do X ray and blood investigation after clinical examination from your doctor.swelling can be cardiac or renal disease related.Hope this answers your query. If you have additional questions or follow up queries then please do not hesitate in writing to us. I will be happy to answer your queries. Wishing you good health.Take care."
},
{
"id": 135354,
"tgt": "What causes hip and shin pain in an elderly woman?",
"src": "Patient: Recently I have had problems after walking short didtance with hip and shin pain this affects right hip and shin and sometimes I wake up with pain in hip and shin. I am used to exercise and have always done plenty of walking. Could this be the beginning of an arthritic condition. I am 58 years old. Doctor: HiWelcome to healthcaremagicI have gone through your query and understand your concern. Yes this can be beginning of arthritis problem. But more likely it can be weakness of bone i.e osteoporosis. You need to get your investigation of knee and hip x rays to see for arthritis. BMD and vitamin D test can further needed to investigate for osteoporosis. Till then you can take analgesic such as ibuprofen for pain relief. Vitamin D3 can be of use to fulfill vitamin D deficiency. You can You can discuss with your treating Doctor about it.Hope I have answered your query.If you have any further questions then don't hesitate to writing to us . I will be happy to help you.Wishing you good health.Take care."
},
{
"id": 55650,
"tgt": "What does my USG report mean?",
"src": "Patient: I am a 48 years married woman. My USG shows liver is enlarged with heterogenous parenchymal echopattern. Intra hepatic billary channels are not dialated. No focal solid or cystic SOL seen. Lever span at mid clavicular line is 15.3 cm. what does it mean? Doctor: HelloYour findings suggests mildly enlarged liver and heterogeneous parenchymal echopattern may indicate liver parenchymal disease.It may be due to many causes like hepatitis,metabolic disorder etc.You may need routine hemogram,viral markers,random blood sugar,liver and renal function test,lipid profile,urine RE/ME.Liver biopsy may be needed after evaluation.You may need few medicines after evaluation.Get well soon.Take CareDr.Indu Bhushan"
},
{
"id": 175082,
"tgt": "What causes vomiting in babies?",
"src": "Patient: My 10 1/2 month old recently started with loose grayish colored stool 3-4 times/day, and is not eating as well as usual. We recently weaned from breastfeeding. He has all of a sudden stopped wanting oatmeal/rice cereal that he previously was getting three times a day. When he will take it, he vomits the entire meal. The only thing we can get him to keep down is some formula and yogurt. Doctor: Thanks for asking on HealthcareMagic.Grayish colored stools can occur due to obstruction in the flow of bile. But it can also occur due to change in food habits. I would not suggest stopping breast milk before 1 year of age provided that there is adequate flow. In case of inadequacy, the situation is different and there is no other way out. Semi solids need to be given. But if the child is vomiting it out, I think that you should get a medical consultation in order to assess for conditions like liver pathology or intestinal obstruction that could cause such vomiting and gray stools. In the meantime, also ive ORS (Oral Rehydration Solution) and plenty of other fluids. It comes in ready made form or may be prepared by mixing a full sachet in the designated amount of water."
},
{
"id": 108119,
"tgt": "What is the cause of intense pain from neck to lower back?",
"src": "Patient: I am a 24 year old man. When i sneeze i get this intense pain from my neck down to my lower back and my shoulders down to my upper right arm. Sometimes this pain lasts for 15 plus minutes. I have to stop what ever i am doing because it feels like my bones are crushed. This started about a week ago when i got a cold. Doctor: dear friend thanx for asking question on hcm... see according to your complain, i understood that you have pure muscular spasm due to continuous coughing or might be you had done some intense exercise at gym... or any heavy work done like sudden lifting of heavy weight or else so that your muscle of back may went in to severe spasm so if you do any work like sneezing coughing or dedication or yawning so you may have sensations of pain due to muscular spasm.... but do not worry... this will be temporary in nature within a week you will be alright.... take aceclofenac, paracetamol and chlorzoxazone combination tablet and apply ice for instant relief.... after 4 days go to physiotherapist take ift therapy for 4 more days... do not apply any warm modality...."
},
{
"id": 148500,
"tgt": "Tingling in arm and leg, patches of goosebumps. What is this?",
"src": "Patient: Hi, I am 20 years old. Yesterday I noticed a tingle in my left arm and leg . When I looked there were random patches of what looked like goose bumps . They stay for only a few seconds then disappear. This has happened a hand full of times since then and it is only on my left arm and leg. I m starting to get really worried because no one seems to know what this is? Please help! Doctor: DearWelcome to HCMWe understand your concernsI went through your description. I suggest you not to worry much. You noted, \"no one seems to know what this is?\" That means, you had been to so many physicians for this problem. I suggest stop worrying. When the physician says that you have nothing to worry, why doubt their capability? They are capable and that is the reason they are in that position. You are just 20 and should have some more better things to do than sitting and thinking and worrying. You must not have anxiety over this.You might need psychotherapy and other cognitive behavioral therapies. I can help you through telephone. Please do contact me through customer care of www.healthcaremagic.com for a telephonic consultation session. Hope this answers your query. Available for further clarifications.Good luck."
},
{
"id": 23697,
"tgt": "What causes pressure in chest and heart flutters?",
"src": "Patient: My heart flutters, I feel VERY slight pressure in my chest, I feel an odd sensation in my head and a hear a \"sush-sush\" in my ears. I notice it more when standing. Should I be concerned? Just started today. I am 49, I weight 167 lbs (5 feet 9 inches) no history of heart disease. No medications. Have had prolonged leg pain that is gradually worsening and swelling in ankles. Doctor: Hello, Thank you for your query. I urge you to seek medical attention right away. This could be an ongoing heart attack or a stage which may lead to it. Your symptoms are indicative of cardiovascular compromise and needs evaluation. Caught early, treatment can prevent heart muscle damage and related complications. -any family history of heart disease, diabetes, hypertension, sudden death, arrhythmia (heart rhythm disturbances)? -do you smoke or consume alcohol? I recommend :-if symptoms persist or worsen , go to the ER right away. Do not drive. Take someone along. -a physician consult. -an electrocardiogram, echocardiogram. -blood tests (complete blood count, lipid profile, Thyroid panel, blood glucose levels, cardiac enzymes to check for heart muscle damage, kidney and liver function tests) -it may just be gas but it is vital to rule out heart pathology. (even if you have no prior illness). Hope this helps. Please let me know if there is anything else I can help you with. Wish you good health"
},
{
"id": 222044,
"tgt": "Why is one unable to conceive?",
"src": "Patient: I AM 35 YRS OLD SINCE LAST YEAR i have bleeding only for 1st 2days & from 3rd day it is negligible.we are trying for child i had one miscarriage 3years ago .after that i had taken pills .since 1 year i have stoped pills but unable to conceive alreday done 1 iui all of my reports are normal Doctor: Hello dear,I understand your concern.In my opinion the scanty menstruation could be due to hormonal abnormalities due to thyroid abnormalities,overweight or obesity.Kindly consult a gynaecologist and rule out the above said causes by doing thyroid profile.Weight reduction in case of overweight or obesity is important.Dont worry.Avoid anxiety and stress regarding pregnancy as they delay fertility further by causing hormonal imbalance.Also get your partners semen analysis done.If everything is normal there are definitely scope for pregnancy.Best regards..."
},
{
"id": 84755,
"tgt": "Does Crisanta taken for irregular periods have any side effects?",
"src": "Patient: hi, i do have irregular periods and i consulted a doctor she gave crisanta tablets and asked me to take it on first day till 21 day of my periods. she aske dme ot continue for 2-3 months,are there any side effects for this?please let me know either weight loss or gain or any other problems?? Doctor: Hello,Crisanta has been advised to you by your doctor considering its benefits more than its adverse effects as she is aware of your medical history. There are few adverse effects which are commonly observed and weight gain is one of those. You can counter it with regular exercise. If you still have doubts you can talk to your doctor for guidance.Hope I have answered your query. Let me know if I can assist you further. Regards, Dr. Manisha Auradkar, General & Family Physician"
},
{
"id": 122182,
"tgt": "Is it safe to take Zerodol for neck pain?",
"src": "Patient: hi i m having a neck pain. it s a case of neck stiffness i suppose. the reason was that i was playing FIFA 12 strday non-stop for 6 hours sitting in the same position with insufficient lighting. I m prescribed zerodol for it. is it a good choice? i have this feeling that zerodol is a bit too much for neck stiffness. kindly advise. Doctor: Hello, You can safely continue Zerodol. It is quite effective for stiff neck and neck spasm. You have to immobilize the neck for a few days. In severe cases you can use a soft cervical collar also. Hope I have answered your query. Let me know if I can assist you further. Take care Regards, Dr Shinas Hussain, General & Family Physician"
},
{
"id": 186444,
"tgt": "What causes under eye pressure after root canal treatment?",
"src": "Patient: I had a root canal redone 3 days ago due to extreme pain caused by reinfection. Today I noticed pressure under my eye on the same side and a foul smell whenever I blow my nose. Is this related? Will this go away on its own? I am currently taking amoxicillin. Doctor: thanks for your query, i have gone through your query, the pressure over the eyes could be because of the periapical abscess even the foul smell. once the infection is treated through RCT the problem will be solved. you can take metronidazole along with amoxicillin. do saline gargling. i hope my answer will helpyou. take care."
},
{
"id": 126012,
"tgt": "How can pain and swelling in the shoulder be treated while having difficulty in moving the arm?",
"src": "Patient: My father fell on his shoulder & is in a lot of pain. He can t move it at all & has difficulty getting up, moving & using one arm. Could it be broken, sprained or rotator cuff? There i=s a bruise on the arm. It may be a little swollen, but doesn t look like it. Arielle Doctor: Hello there I have read through your question and would like to tell you that pain and swelling along with inability to move the shoulder joint all point towards a diagnosis of fracture. I would advise you to get his XRay shoulder joint done as soon as possible as delay may lead to complications. Meanwhile it would be better that he doesn't try to move the shoulder too much and keep it restricted with the help of a strap. I hope that answers your question. If you have any further questions please feel free to write. Thank you for choosing healthcare magic."
},
{
"id": 40565,
"tgt": "How can infertility be treated?",
"src": "Patient: Gm,Doctor My period was irregular but since from this January i was having treatment and Doctor had given me 3 months fertilization injection from Jan till June it was coming perfectly. My last period was on 29th June 2017 and till now i have no periods and yesterday at around 12:30 pm i had check pregnancy through kit but it was negative. and in between i was having watery some times milky watery discharge. Am i pregnant, i want to be pregnant. Doctor: Hello,For that, you should be checked thoroughly and run sometimes which can tell about your problem definitely.So, your report and your partner's report are necessary and then we can find the cause of infertility and treat accordingly.Hope I have answered your query. Let me know if I can assist you further.Regards,Dr. Bhadresh Lakhani"
},
{
"id": 23261,
"tgt": "Should hypertension medication be continued when blood pressure becomes low?",
"src": "Patient: I am 54 yrs, 5'4\". I am taking three medications: Atenolol, Norvasc and Hydrochlorothiazide. In january 2011 I had medical insurance, always had blood pressure readings of 183/90 with pulse at 45. Now I don't have medical insurance, went on a protein diet went from 205 lbs. (January 2011) to 170 (May 2011). Now my blood pressure usually is 116/65, pulse at 48. Only when I exercise on the treadmill does my pulse go up to 65 (exercise every day for 30 minutes). Should i still be taking all three medications even though my blood pressure is now low? Doctor: DEAR USER,THANKS FOR CONSULTING WITH HCMI UNDERSTAND YOUR CONCERN.. DEAR USER YOUR HEART RATE IS TOO LOW ITS PROBABLY DUE TO USE OF ATENOLOLHEART RATE BELOW 60 IN A PATIENT ON ATENOLOL IS OF PARTICULAR CONCERNI ADVISE YOU TO STOP THE TABLET AND CONSULTE YOUR PHYSICIAN AS EARLY AS POSSIBLEHOWEVER YOU CAN CONTINUE WITH NORVASC AND HHYDROCHLOROTHIAZIDEYOU NEED TO CONSULT A PHYSICIAN AS EARLY AS POSSIBLEHOPE I ANSWERED YOUR QUERY. YOU CAN MESSAGE ME FOR ANY FURTHER CONCERNS"
},
{
"id": 178464,
"tgt": "What causes red welts on legs?",
"src": "Patient: Baby wakes up with welt on legs,red about 1/2 inch. During the day They turn a yellow color, then ooze and scab. Happened about three times in last month. First two times only one, last week three on same leg. Took her to doctor first time, told us could be bug bite and gave her Benadryl. Cleaned room, sheets, everything, second one about a week later. This time a couple of weeks between. Any suggestions? Doctor: Hi...I feel that your doctor is 100% right. This looks like an insect bite allergy only. Anti- allergic medication like Benadryl is appropriate. Once this starts there can be a phenomenon called second crop where in the allergy may manifest again with 1-2 weeks time. But the ooze which you are describing is due to secondary bacterial infection which your kid is getting. She might require a oral antibiotic like Cephalexin or Clindamycin. I suggest you to consult your doctor for this prescription after he feels this suggestion is right.Regards - Dr. Sumanth"
},
{
"id": 213555,
"tgt": "Duration for lithium tablets for bipolar disorder patients to get normal",
"src": "Patient: sir for how much time a bipolar disorder patient should take lithium tablets so that he should not triggered to manic episodes again n again n should stable like a normal person with regular routine Doctor: Unfortunately, bipolar disorder must be treated at all times for the duration of the patient's life. Once a person goes off of their medications, the bipolar symptoms return regardless of how long the person has been on medications. All medication should be taken at the same time everyday to ensure maximum coverage. Be well, Dr. Kimberly"
},
{
"id": 136761,
"tgt": "What causes pins and needles down my whole left arm?",
"src": "Patient: Hi each night I am waking up with pins and needles down my whole left arm. it usually goes away during the day but comes back at night. it wakes me a few times each night for an hour or so. I work in a physically demanding job , with heavy lifting and using machinery. I am 40 years old and I smoke. I am 80 kg and 6ft1. I do regular sport 3 to 4 times a week Doctor: Hi,Thanks for your query.Your symptoms look like the features of nerve compression. You may take pain killers and if need be, tablet gabapentin, for this numbness. It is a stronger pain killer for neuropathies. Your doctor will know the doses.Also an X-ray cervical spine may help in diagnosis of any pathology there. Thus I would recommend that you see an orthopedic surgeon and get appropriate investigations done including vit B12 levels in blood.I do hope that you have found something helpful and I will be glad to answer any further query.Take care"
},
{
"id": 98474,
"tgt": "What causes wheezing while suffering from bronchitis and asthma?",
"src": "Patient: I have had what I thought was Bronchitis and I have asthma. I've taken a z pack a steroid pack on ad air and pro air and still have significant coughing wheezing and today my cough sounded very croup like. I think I have pneumonia. What are your thoughts? Doctor: HelloYou should do a spirometry test and a chest x ray for detecting if there is a pneumonia or not.RegardsDr.Jolanda"
},
{
"id": 31257,
"tgt": "Suggest treatment for bacterial vaginosis",
"src": "Patient: hi,i have been diagnosed with Bacterial Vaginosis twice in different clinics and even after the medication-Metronidazole it doesnt seem to clear,or sometimes it goes for afew days and the discharge is back,and it gets worse(amount and smell) when my periods are about to come like now.Could it be a misdiagnosis? Doctor: there is a possibility but a vaginal smear should be done before. because different antibiotics are specific for organism involved in vaginal discharge it could be bacterial or fungal."
},
{
"id": 171155,
"tgt": "What causes cold and cough in a 6 month old?",
"src": "Patient: Hi, My daughter is six and half months old due to bad weather condition from last two she is suffering from cold and cough , initially she was coughing two or three times a day but from yesterday night she coughing like sometimes she vomiting also ,so that lead me consult with pediatric and got consulted and was referred to give soventus jr. cough syrup 2.5 ml as single dose three time a day for at least three days. So should I go with this or Is their any better treatment possible. Rgds. Sudi Doctor: Hi... By what you quote it seems that initially your child has cold but now she has developed wheezing. In wheezing there is increased cough episodes at night and early morning, usually child requires nebulisation in wheezing. I suggest you to go to the pediatrician again and get the chest examined. Till then continue the cough syrup. I hope this has helped you. Take care. Regards - Dr Deepak Patel, MD Pediatrics"
},
{
"id": 99452,
"tgt": "Suggest treatment for urticaria",
"src": "Patient: I take Quetiapine at night for sleep. Right now I am experiencing Urticaria from a food allergy. I have to take the Benadryl all day long. Is it safe to take the Quetiapine at night after taking Benadryl during the day? Benadryl does not make me sleepy at all, the contrary, so it doesn t replace the Quetiapine. Doctor: HI, thanks for using healthcare magicYes, you can take the benadryl to help with your allergic reaction or you can use a non drowsy antihistamine such as loratidine, cetirzine or fexofenadine.If you do feel very drowsy from the use of the anti histamine then you may want to consider with holding the dose of the sleep aid.I hope this helps"
},
{
"id": 62922,
"tgt": "Suggest treatment for a hard lump in the face",
"src": "Patient: My newborn baby has a hard blue lump under the skin of her face. Could this lump be cancerous? She is red all over her body and she is highly constipated. Her tummy is painful to the touch. Could this all be symptoms of cancer in a baby? She was born with the lump and its about 2mm in diameter. My husband and I both have a history of cancer in our family. Doctor: Hi, dearI have gone through your question. I can understand your concern. She may have hemangioma or other vascular tumour. You should go for examination. If needed go for ultrasound study. It will give you exact diagnosis. Then you should take treatment accordingly. Hope I have answered your question, if you have doubt then I will be happy to answer. Thanks for using health care magic. Wish you a very good health."
},
{
"id": 5568,
"tgt": "Suggested to take clarithrimycin for a bladder infection. Blood and WBC in urine. IS it safe to get pregnant now?",
"src": "Patient: Hi. I have been described 500mg clarithromycin for a bladder infection yesterday. there were some blood and white blood cells in my urine . I have read the leaflet but there is no mentioning of bladder infection. also Iam worried as we are currently trying for a baby how safe they are. I did mention this to my doctors yesterday. thank you Kind regards Doctor: You can complete the course of the antibiotics. You can also try for pregnancy. The effects of the antibiotic is short lived so it wont harm you now. But its better to avoid certain antibiotics after you get pregnant."
},
{
"id": 171422,
"tgt": "Is it normal for a 4 year old to have an over active behaviour?",
"src": "Patient: hi , my four and half year old son is very active and intelligent, but i am concerned that he may be a hyperactive child. he is excellent in academics and can focus in all activities. can communicate very well and is highly analytical. extremely quick, body is flexible, likes to climb and run. please tell me if i have to be very concerned about his over-active behavior. it is difficult for him to sit still unless he is watching tv or occupied with activity Doctor: DearWelcome to HCMWe understand your concernsI went through your details. According to you your son is excellent in academics and can focus in all activities, can communicate very well and is highly analytical. That is certainly good for a 4 year old child. I don't see anything wrong in the behavior of the child. At the age of 4, any child is supposed to be a bit hyper mainly because of the playful attitude. You may search internet for the signs and symptoms of hyper activity and compare with your child. It's natural to wonder if ADHD (attention deficit hyperactivity disorder) could be the cause of your child's high energy. But just being hyper doesn't necessarily mean your child has the condition. Watch out for these other signs of ADHD:Does he interrupt frequently?Does he have trouble following instructions and organizing tasks?Is he forgetful?Is he impatient?Does he frequently speak out of turn?If your child has ADHD, these problems will happen over a long period of time, and usually at home as well as school. If you say Yes for three out of these questions, you may need to consult a psychologist for ADHD assessment and further guidelines.If you require more of my help in this aspect, please use this URL. http://goo.gl/aYW2pR. Make sure that you include every minute details possible. Hope this answers your query. Available for further clarifications.Good luck. Take care."
},
{
"id": 163625,
"tgt": "Suggest an effective alternative for breast milk for a 3 month old",
"src": "Patient: Hi Sir, Good Morning My baby is having 3 months as weight of 6.4 kg. i would like to ask you to my working hour is have 10:30am to 04:30pm between this time I m not able to feed milk to my baby. So, kindly requesting you to advice me what should i have to provide my baby. waiting for your positive response. Thanks Doctor: Hello,First of all, I want to congratulate you to have a baby and at 3 months the weight is very good. Please let me know the birth weight too.Now, coming to your question, I suggest you to express breast milk sufficient enough for feeding four times (in between 10:30 and 4:30). If you see that expressed milk is not that enough, then you can use formula milk stage 1, till 6 months of age.You can give formula through bowl and palady or bottle but in that case bottle hygiene need to be maintained properly.Hope I have answered your query. Let me know if I can assist you further.Regards,Dr. Raktima Chakrabarti"
},
{
"id": 98318,
"tgt": "What causes painfully swollen neck glands along with enlarged thyroid gland?",
"src": "Patient: I was seen at an urgent care clinic for an allergy flare up. I was given steroid injection and instructed to continue with allergy medicine. My neck glands are swollen and very sore. The back of my neck hurts as well. It was noticed my thyroid was enlarged. I have had nodules in the past. I also have a small rash where my dog scratched me about a week ago. Not sure if any of these issues are related, Was also found to be anemic with no known cause. Made an appointment with new primary care for this week for follow up. Feeling tired. Doctor: Hello and welcome to \u2018Ask A Doctor\u2019 service. I have reviewed your query and here is my advice. As you explain the history it might be an infection of the thyroid gland or the area around.I recommend you to be seen by your doctor for probably further follow up i mean some lab tests. Hope I have answered your query. Let me know if I can assist you further."
},
{
"id": 168470,
"tgt": "How to treat congenital defect on foot of the right leg?",
"src": "Patient: hi doctors, hope you are doing well. my baby is congenital defected of her right leg club foot. now is under treatment in Bangladesh Apollo hospital & running serial plaster. so doctors adv us need for minor surgery pls adv what will do. she is 7month running & height around 2.5 . i need your adv. Doctor: Hi...by what you quote I feel that your doctors are right and the treatment is going on in the right direction. It is not only surgery, the kid might also need special shoes after the surgery.Regards - Dr. Sumanth"
},
{
"id": 206173,
"tgt": "Suggest treatment for depression and sleep disorders",
"src": "Patient: my husband takes cymbalta and has for more than 10 years. He also takes three different meds for high blood pressure and two different blood thinners. Is there significant drug interactions that would cause him to stop taking the cymbalta or at the very least switch to something else. His depression has returned and he is having sleep issues. Doctor: Hi.I understand your concern .Cymbalta contains duloxetine and it is one of the good medicines for depression.If you are taking anti hypertensive medicine then need to do proper evaluation of class of that medicines . There are interaction with duloxetine but still it is safe to take it under observations .SSRI sertraline / escitalopram are safely used in hypertensive patients.Sertraline is the safest among all in cardio vascular compromised patients .Lorazepam is safest benzodiazapine for sleep problems.You can discuss all these options with your treating doctor .Still if you have a query then feel free to ask .Thank you ."
},
{
"id": 206387,
"tgt": "How long the medication for schizophrenia be taken?",
"src": "Patient: I am 40 years male. 10 years back I had an severe episode and the neuropsychiatrist had diagnosed me with schizophrenia and I have been treated. I had continued the medicine for more than one year and stopped. subsequents years I get episode on a yearly basis and had to consult psychiatrist and medication taken. For more than three years now I am not getting episode and since then I am on 2 mg respiredone daily. My psychiatrist whom I consult every two to three months told me to continue with the medication. I don't have the fear or anxiety anymore. how long should I continue the medication and what are the side effects if any with this medicine? Doctor: Hi I had gone through your query.Schizophrenia is psychotic illness and require long term treatment.It has poor prognosis so it may chance to get worse if you stopped down medicines.Usually 1st episode of schizophrenia may require treatment up to 2 to 5 years.2nd episode may require life long treatment.2 mg of resperidone is ideal maintenance dose and do not have such side effect.So it is good to continue it or do changes as per your treating psychiatrist advise.I hope i have answered your query.Still if you have query then feel free to ask.Thank you."
},
{
"id": 124344,
"tgt": "What is the cause for the feeling of something stuck under the right rib?",
"src": "Patient: what is it when you feeling tired, i have the feeling that something is stuck under my right rib, i have had 5 blood tests, and they are all normal, i can t understand why i am feeling like this, could it be my gallbladder, and i feel bloated, i av been taking buscapan they any good. Doctor: Hello, As you have done all the blood routine which turned out to be normal. Why don't you opt for some simple breathing exercises and gentle massage over the area. As you didn't mention about under which rib it is practically difficult to answer it. But then if it is the rib cage then you can take an x-ray to figure out the status of the ribs positioning and also the lungs tissue. If it is below the rib cage that is below the diaphragm level than you should definitely opt for Ultrasound scanning of the abdomen. At times small discomforts are made unnoticed by a physician just by the blood routine, but on a safer side taking one of the above as required should reveal if there is something really wrong or not. Hope I have answered your query. Let me know if I can assist you further. Take care Regards, Jay Indravadan Patel, Physical Therapist or Physiotherapist"
},
{
"id": 69230,
"tgt": "Suggest remedy for lumps in the neck",
"src": "Patient: Hi. I have a spongy lump in the middle of the back of my neck which appears when I put my head back. Shen I straighten my neck it disappears. It's like I can push it into the muscles. My neck hurts when I extend it back but the lump does not hurt. Do you know what this is? Doctor: Hello!Thank you for the query.It may be a lymph nodes or some benign skin lesion. Its hard to tell not being able to examine it. The best you can do is to have an ultrasound of this area. If the lymph nodes will be diagnosed as enlarged, biopsy should be done.Benign lesion (like lipoma) can be removed with small surgical procedure.Hope this will help.Regards."
},
{
"id": 71721,
"tgt": "Suggest treatment for bronchitis and vomiting",
"src": "Patient: Dear Doctor, I am Anand from Mumbai India.The purpose of writing you this email is to understand for what ailment are the following medicines prescribed. My wife is consuming these medicines .The symptoms like vomiting on regular basis,cant digest rich food,while sleeping she folds her legs and bends,cant stand and travel .Food consumption is also very slow at times she swallows food with water.These Facts are creating a doubt that she is suffering from some major ailment.In past she has been diognised of Bronchitics.Also plz send me your email id so that i can email you the reports.My id is YYYY@YYYY . Following are the medicines: 1. Diprovate RD 2. Ciplox 3. Softovac 4. Clindamycin Phosphate-gel usp 5. Excylin-civ 625 duo 6. Flagyl 400 7. Mefal (Analgesic\\antispasmodic) 8. Folica Hair Tonic 9. Cap Becadepamine 10. Cap Immuncace Forte Kindly Advice. Regards Anand Doctor: Hello! Welcome to HealthcareMagic! Medicine doesn't always mean severe disease. Doctors usually thing two or more diseases after your history and write medications accordingly. Hope this clarifies your concerns. Thank you for the query. Take care varanasichestclinic.com"
},
{
"id": 143434,
"tgt": "Suggest treatment for numbness and tingling sensation on face and extremities",
"src": "Patient: I ve had my SED rate running in the high 60-75 since last summer when I was supposedly diagnosed with an infection in my stomach. I ve had several tests and ruled out RA, no diabetes, and several other diseases. I keep going to my Dr for multiple complaints of pain, swelling, numbness tingling in extremeties and face, weakness, stumbling and a fall, difficulty with memory and speech, involuntary leg jerking at rest, pain in lower back and knees, and pain in arms and legs/knees. My Dr said I was just depressed and had anxiety. Now I m having vision problems and there have been days where I couldn t get out of bed or walk? I don t know what to do! Doctor: Hi, Thanks for your question. I do understand your pain and discomfort. I have seen many cases with similar complaints. Please find my advice below-:1)Do a MRI scan to ruled out neurological deficit2)Check B-12 level correct it if necessary3)ESR and Antipolymer antibody assay to ruled out fibromyalgiaI hope I was able to address your query. If you have any further questions, please do not hesitate to write to me. Wishing you all the best. Thanks,"
},
{
"id": 188398,
"tgt": "Had teeth extractions, another tooth broke, infection in gum, no pain. What can I take to relieve pain?",
"src": "Patient: hi i had two teeth extractions on wedensday 2nd one didint come out easy :( by friday i went back had an infection i had the site cleaned and was given antibiotics and advised to rinse with salt water.Today the tooth next to the extraction has broke half way down horizontally along gum line i can see the brown inner of my tooth upper part is still intact and healthy that tooth was filled on wed it was a small hole..I am very concerened as if i haft to get it pulled it will effect my smile..Is their any hope for my tooth i have no pain from that tooth the infection on my gum is still their im day 2 of my 3 day course i am calling dentist on monday am i am an nhs patient aswell just looking for reassurance (starting to reslly dislike the dentist ) regards carol Doctor: Hello,Thanks for writing to us.Tooth broken next to the extracted tooth has to be replaced or extracted for permanent relief.I would advice you to get a thorough clinical evaluation done.If the fracture involves pulp,tooth has to be root canal treated and crown has to be placed.If tooth cannot be saved,get it extracted.Antibiotics as well as analgesics will help to relieve pain.Meanwhile,avoid chewing on hard objects.Take care."
},
{
"id": 214534,
"tgt": "Suggest home remedy for sore tongue",
"src": "Patient: I have a very sore tongue. It started a couple of weeks ago and at times goes away but when it comes back it is worse than ever. It hurts to eat, drink and brush my teeth.. Do have any home remedies that might work for me? Could this be an aspartame allergy? Doctor: Thank you for the query. I would like to know your age and gender and whether you suffer from any comorbidities like diabetes mellitus. Sore tongue can occur due to number of reasons. Allergy to any food ingredient can cause it. Vitamin deficiency, is however the commonest cause, and you should immediately start on Vitamin B-complex supplements. You can apply commercial preparations in your mouth for symptomatic relief. Chlorhexidine mouth wash can help. Avoid hot and spicy food. Occasionally fungal infection may be a cause, in which case clotrimazole mouth paint application is advised. Apthous ulcer is another common cause for soreness in the mouth, specially in young adults.If the problem persists please do consult your doctor. Hope this helps."
},
{
"id": 28676,
"tgt": "What causes an infected cyst in the underarm?",
"src": "Patient: I've just had a painful cyst removed from under my arm the doctor has sent swabs and the capsule for testing, i thought it wasnt too big, id only had it a week, but according to my doctor it was infected and very deep,:what are the possible diagnosis for this problem? Doctor: Hello and Welcome to \u2018Ask A Doctor\u2019 service. I have reviewed your query and here is my advice. I have gone through your query regarding Infected sebaceous cyst. Sebaceous cysts are common non cancerous cysts of the skin. Common tests used for a sebaceous cyst include:\u25cf\u00a0\u00a0\u00a0\u00a0\u00a0Physical examination\u25cf\u00a0\u00a0\u00a0\u00a0\u00a0Medical history\u25cf\u00a0\u00a0\u00a0\u00a0\u00a0CT scans, helps to find the best route for surgery and to spot abnormalities\u25cf\u00a0\u00a0\u00a0\u00a0\u00a0ultrasounds, which identify the contents of the cyst\u25cf\u00a0\u00a0\u00a0\u00a0\u00a0punch biopsy, which involves removal of a small amount of tissue from the cyst to be examined in a laboratory for signs of cancer A sebaceous cyst is treated by draining it or by surgically removing it. Normally, cysts are removed. This isn\u2019t because they\u2019re dangerous but rather for cosmetic reasons. Since most cysts aren\u2019t harmful to your health, you can choose the treatment option that works for you. It\u2019s important to remember that without surgical removal, your cyst will usually come back. The best treatment is to ensure complete removal through surgery. Some people do decide against surgery, however, because it can cause scarring. One of these methods are followed for cyst removal:1.\u00a0\u00a0\u00a0\u00a0\u00a0Conventional wide excision, which completely removes a cyst but can leave a long scar.2.\u00a0\u00a0\u00a0\u00a0\u00a0Minimal excision, which causes minimal scarring but carries a risk that the cyst will return.3.\u00a0\u00a0\u00a0\u00a0\u00a0Laser with punch biopsy excision, which uses a laser to make a small hole to drain the cyst of its contents After your cyst is removed, you can use antibiotic ointment to prevent infection. You should use this until the healing process is complete. You can also use a scar cream to reduce the appearance of any surgical scars. Hope I have answered your query. Let me know if I can assist you further."
},
{
"id": 191985,
"tgt": "Is it safe to not have any medications while having diabetes?",
"src": "Patient: I AM NEWLY DIAGNOSED WITH DIABETES. MY ALC LEVEL WAS 6.9 WHEN MY DOCTOR INFORMED ME THAT I WAS DIABETIC. HOWEVER, MY DOCTOR HAS NOT PUT ME ON ANY MEDICATIONS AT THIS POINT. IS THIS COMMON? TO NOT BE ON ANY MEDICATIONS ONCE DIAGNOSED? SHOULD I CONTACT MY DOCTOR ONCE AGAIN TO INSURE THAT THIS IS ACCURATE???? I'VE BEEN WATCHING MY CARBS AND CUTTING SUGAR DRASTICALLY FROM MY DIET. I AM LOSING WEIGHT. BUT AM JUST CONCERNED IF I SHOULD BE TAKING MEDICATIONS ALONG WITH MY EXERCISE AND DIET CHANGES? Doctor: thanks for your query in HCM,AS per your narration you are newly diagnosed with diabetes. your HbA1c is 6.9. that means your sugar is under poor control since last 3 months. what is your age and please mention FBS and PPBS recent lab values. the first thing advised for newly diagnosed diabetic is diet control and regular excercise. if the glucose levels doesn't come down with diet and excercise then medication needs to be started. please get FBS, PPBS, HbA1c tested after 3 months. if they are still elevated your physician will put you on medicationsthank you"
},
{
"id": 23078,
"tgt": "What causes the shooting pain in left carotid artery area?",
"src": "Patient: i have been experiencing off and on for a couple months random shooting pain in my left coratid artery area. yesterday i jog/walked my first 10k and about 15 minutes afterward well after i had cooled off and my breathing was back to normal my heart sped up i broke out into a sweat felt dizzy and my entire left arm tightened i thought i was going to pass out. after taking a few deep breathes and drinking some water it disappeared but it scared me. do you think i should see a dr? heart problems run in my family do you think this is heart related? oh i m 35 female, 5 4 165. had gastric bypass was 269 4 years ago. had gall bladder removed last year. Doctor: certainly you should get yourself evaluated.first get your routine test like lipid profile and sugar test, ekg.you should also get tmt done if ekg is normal. avoid strenuous exercise till heart disease is ruled out."
},
{
"id": 87552,
"tgt": "What causes upper abdominal pain and cold?",
"src": "Patient: Last night i started feeling some pain around my upper abordmen( assuming relating to Kindey or overies) then today i have been having a feeling cold and mucles are arching for the whole day. I haven't been to the doctor as yet could you please give my an indication of what could be happening Doctor: Hi,From history it seems that you might be having some infection in your Genito-urinary tract giving rise to cold feeling and might be having some temperature.consult your doctor and get examined.You might require one antibiotic medicine course for 3-5 days.Meanwhile take some antispasmodic medicine with paracetamol like Meftal spas to get relief from pain and cold feeling.Take plenty of water.Ok and take care."
},
{
"id": 14739,
"tgt": "What could itchy rashes between legs and pubic area indicate?",
"src": "Patient: I have an itch rash near the crease between my leg and pubic area And also have tiny red bumps on near belly button. Some are flat now but started out raised with a tiny whitehead. A few on lower stomach and hip too. The rash itches but the bumps on stomach and hip dont. Doctor: Hello and welcome to healthcaremagicI would keep a possibility of Fungal infection of the groin folds (Tinea cruris) along with bacterial folliculitis of the lower abdomen and buttocks. Groin folds are a common site for fungal infection specially in hot and humid weather in summer, which is conducive for fungus growth.If I was the treating doctor I would suggest you an OTC topical antifungal cream e.g clotrimazole cream, to be applied twice daily in the groin folds.An OTC antihistamine e.g cetrizine 10 mg once daily would provide symptomatic relief from itching.Moreover, the perineal and pubic region is rich in resident bacterial flora. Sweating and humidity together with friction and rubbing provide a portal of entry into the skin, through the hair follicles, thus giving rise to Folliculitis.For the folliculitis, I would suggest an empirical course of an oral antibiotic for a week.My choice of oral antibiotic would have been either:Amoxycillin 500mg in combination with clavulanic acid 125 mg, thrice daily for a weekORCephalexin 500 mg thrice daily for a weekORCefadroxil 500 mg twice daily for a week.An OTC topical antibacterial e.g 2% Mupirocin, is to be applied over folliculitis lesions, twice daily.Bathing twice daily, wearing loose fitting cotton clothes, weight control and using an antifungal dusting powder regularly to make the area unfavourable for fungal growth are few general measures that would help in preventing recurrence.Regards"
},
{
"id": 185250,
"tgt": "Suggest remedy for ear & jaw pain",
"src": "Patient: saturday my right ear was very achy, not clogged just achy. today monday my right jaw is swollen and achy making one of my teeth achy and its getting painful with a right side of the head head ache. i went to the dentist for a cleaning about 30 days ago. Doctor: HiWith your query, since your chief complaint is regarding the ear ache i would suggest you to consult your ENT specialist then take his opinion and consult yor dentist for further treatment.I hope this helps you."
},
{
"id": 193617,
"tgt": "Suggest remedy penis erection problems",
"src": "Patient: Dear Doctor,I am 20 years Old. My weight is 55 kg and hight is around 5.3\". I have masturbated alot. Even for last 8 years i am masturbating. Now my penis doesn't erect. And one thing more i have never discharge while sleeping. I am really worried about my situation now. Kindly let me know treatment for this problem. Doctor: Hello, You are having a problem in the erection of penis. It can be related to psychological causes more at such a young age. You need to restrict masturbation frequency 2-3 times a week in the future. I suggest you watch porn videos or magazines if it can help in an erection. If within a few days the penis can't get erect than rule out hypertension, diabetes, high cholesterol etc as primary workup. You can Consult a psychiatrist or sexologist for counseling. Hope I have answered your query. Let me know if I can assist you further. Take care Regards, Dr Parth Goswami, General & Family Physician"
},
{
"id": 211980,
"tgt": "Suffering from vertigo, depression, BP. Medication not helping. Suggestions?",
"src": "Patient: Hello doctor, Iam working in desret area in oman with single status. 3 months before i suffered from panic, vertigo, unbalacing, not able to eat & sleep well.So during vacation , doctor checked the pressure & found 180/120 { high BP}. so prescribed hyper tension reducing tablet {proprancial hydrocholoride and sleeping dose at night.Also all tests including CT scan, blood count, thyroid & every thing thing is normal. finally it was concluded and cleared during vacation. When I return back from my family without sleeping dose { clonotril 0.5mg} at night problem persist again. Now Iam taking paraxetine hydrochloride 12.5 mg for the past one week and still problem not cleared. How many days required to recover? Whether i am going in the right way? because four years before I suffered cevere vertigo & depression, That time I took nexito 10mg for one month in the alternative days and it was cleared. Doctor: while you present with both anxiety and depression symptoms , you need to be rational in the way you take medication . you need to be on longer time on antidepressant whether its nexito or paroxetine (you may need a larger dose) , taking the drug on alternate day is not going to benefit you. along with these drugs you need to see a local psychiatrist for regular monitoring . and you also can ask him for cbt or other psychotherapy for anxiety and depression. also go for regular exercising and brisk walk . regularly check the B P so as to see if the rise was organic or just due to anxiety"
},
{
"id": 39058,
"tgt": "Suggest treatment for open bleeding sore under breast",
"src": "Patient: I found an open, bleeding sore, the size of a pea under my right breast after showering the other day. It doesn't hurt and have no idea what it could have been from. I've kept Neosporin on it and kept it covered but it seems to have grown a bit in diameter. Any idea what this may be? Doctor: It could be anything from a minor boil to a tumor. Are you married and with children? Also are you diabetic? Do not keep the wound covered (by wearing lose clothing over it). How long is it taking to heal? You must get your blood sugar levels checked along with a mammogram done as soon as possible. You could get back to me with all your reports. I 'd be glad to help. Till then keep dressing it.Dr. (Major) Rakesh Sharma"
},
{
"id": 168694,
"tgt": "Suggest treatment for accumulation of white material inside penile foreskin",
"src": "Patient: Hi im almost 13 and i found out what circumsizing is and that i am not circumsized and i thought that you only had to wash the outside skin until recently when i got in the shower to wash the inside skin and not only did it hurt (like not badly but just uncomfortable hurt) but i found white stuff on the inside skin on my penis Doctor: Hi,In uncircumcised person there is collection of smegma under foreskin and if not cleaned it deposited there producing collection of white substance or white layer.If does not clean regularly one gets difficulty in retracting fore skin.Make a habit of cleaning fore skin by averting foreskin while taking shower.Oki and take care."
},
{
"id": 158049,
"tgt": "History of completion of radiation therapy and chemo for breast cancer. Suggested biopsy after pap test. On Herceptin. Chances of recurring cancer?",
"src": "Patient: iv just finished chemo and radiation therapy at the end of april 2013 i was diagonised with breast cancer trible postitive her2 positive as well, i just had a pap test. dr called to say i need a biopsy. i though i was safe for a while at least due to the fact that im still undregoing therapy, Herceptin every three weeks, im scared and wondering what are my chances of it being cancer again \\ Doctor: Hi! WELCOME TO HCM! You have finished your chemo and radiation therapy.I can understand that you may think that your abnormal pap smear is related to breast cancer . The fact is that cervical screening is designed to pick up minor changes before any problems develop. Having an abnormal result is not unusual. Approximately 1:20 women have test results that show some abnormality and it is extremely rare that an abnormal change found on screening is actually cancer. An abnormal change shows small changes in cells which may be an early warning sign that over time could develop into cervical cancer. normally you get referred to colposcopy so they can look at your cervix under a microscope and can take a biopsy if they think its required. Hope your biopsy will be normal !take care!"
},
{
"id": 66061,
"tgt": "Suggest remedy for lumps on neck of cat skin",
"src": "Patient: hi I am a young girl 13 and my cat has a huge lump on his neck it is under the skin and you cant see eany thing uther then hes neck is the size of a bace ball this is my first cat and we have no money to take him to the vet seeming as we just got in to a new place a week ago I was wondering if u could help me to find out what this could be I am very worried. Doctor: Thanks for asking in healthcaremagic forum Sorry I have not studies veterinary science to treat animals. But if the lump is growing too fast and is symptomatic take it to animal hospital. All the best."
},
{
"id": 16081,
"tgt": "Rashes on neck, shoulders, severe pain. Tried hydrocortisone, benadryl capsules. Due to shingles?",
"src": "Patient: I m a 68 year old male. I have a rash on left neck and shoulder which literally hurts. I ve tried hydrocortisone 1% cr. chlortrimazole cream, Benadryl caps, Prednisone 20 mg/daily x 5 days and it won t go away. I m thinking it might be shingles but I ve been dealing with it for several months. Numbness and tingling up face and ear. Left side only. Doctor: Hello Welcome! Thanks for your answer . I would be pleased to answer your question . If it is Shingles , You will need to take an anti viral course for a period of 10 days . You will need a prescription for the same from your Dermatologist or Primary care physician for Anti Viral Acyclovir or Valacyclovir . Please do not apply steroids or take steroids orally for a long time as it will not make much difference . Thanks and take care"
},
{
"id": 57368,
"tgt": "Elevated liver enzymes, had ulcerative colitis, severe blood drawn. How long medicine should be stopped to get good results?",
"src": "Patient: My 18 year old son has significantly elevated liver enzymes without any symptoms. He was diagnosed with Ulcerative Colitis in Oct 2011..He has had his blood drawn over 30 times since Oct. 2011. We were finally told to stop his meds to see if they were causing the elevated liver enzymes..he repeats his blood work today. We see a Hepatologist Thursday. What are the indicators of meds being the culprit of elevated liver enzymes and how long do you have to stay off meds to get a true result? Doctor: That depends on the medicines taken.One week is a reasonable time to wash off the effect of most of the drugs as for as the readings of liver enzymes are concerned."
},
{
"id": 95362,
"tgt": "What can do I for vomiting sensation that I have after every meal ?",
"src": "Patient: why every time i eat my first meal . i feel like i have to throw up after or my stomach gets in pain i am in perfect health i just feel i have pains and feel like throwing up after wach meal. Doctor: Hello; welcome to HealthcareMagic During the first meal the stomach is empty and the symptom you are telling is due to acidity or even gastritis which may be causing you to throw up.Please take antacid like ranitidine-300 mg and if required for nausea you can take domperidone 10mg.Both 1 tablets each before every meal three times a day for 1 week then you can reduce for 2 times a day and continue for 2 weeks and the problem will settle. Thanks"
},
{
"id": 8788,
"tgt": "How can I get rid of a mole on the face ?",
"src": "Patient: I have a mole of size nearly 1 mm on the left side of my nose i want to get it removed Doctor: Hi...dear lok.., Thanks for choosing HCM.., Mole...1 mm in size.,left of the nose., Not safe to remove surgically.., First asses the type of Mole.., That means..Is it...epidermal, dermal or Junctional.., Epidermal Nevi is safe to remove.., But my option is ...If U paricular about this.., go for LASER....treatment!., Here ERBIUM-YAG Laser with negligible scaring is treatment f choice.., and another one is FRACTIONAL CO2 LASER.., it is also good .,minimal scarring is noticeable.., Preferable , Bcoz cost is economic compare to ERBIUM...thanQ"
},
{
"id": 105610,
"tgt": "Anxiety disorder, allergy, have chest tightness, sensitive teeth, dizziness, weakness. Cause?",
"src": "Patient: I have a generalized anxiety disorder and take wellbutrin and trazadone for it. I also have bad allergies and take claritin. Lately my chest has been tight and feels like when I have a chest cold and my lower teeth are really sensitive. I feel very dizzy and weak and want to sleep all the time. Does it sound like I am sick or is it something else? Doctor: Hi, Thanks for writing in. The dizziness and weakness could be due to side effects of wellbutrin and trazodone. The tightness in the chest and sensitivity could be symptoms of your anxiety and it is unlikely that you are actually suffering from any other illness. I would suggest that you speak with your physician if the symptoms persist. Hope this helps."
},
{
"id": 143308,
"tgt": "Suggest treatment for sudden dizziness, headache",
"src": "Patient: i got up this am and went to the bathroom came back to bed and had the biggest spinning feeling it was harsh the room spun fast and hard as if i was on a roller coaster it stopped sat up then laid back down and it was back. now i just am not right slight headache shoulders and neck hurt and just not right Doctor: I read your question carefully and understand your concern. Vertigo with head movements are very suggestive of Benign Paroxysmal Positional Vertigo (BPPV). You should see a ENT doctor. He can perform specific maneuvers which can cure you within seconds. Hope you will find this answer helpful. Kind regards,Dr. Wu"
},
{
"id": 77630,
"tgt": "What causes fluttering feeling in chest?",
"src": "Patient: My husband is complaining of these symptoms: fluttering feeling in middle of chest follwed by feeling of flushing up the back of his neck over his head to his forehead, then feeling stoned for a few seconds. Happened several times last night. He does have CHF, diabetes, kidney failure, has had two CABG s (these are the worst of a myriad of other things). Doctor: Thanks for your question on Health Care Magic. I can understand your concern. In my opinion, your husband is having arrhythmia. Arrhythmia is irregular heart beats. So this can cause fluttering feeling in chest. He is also having heart and kidney disease. He is also diabetic, so all these also favour arrhythmia more. So better to consult cardiologist and get done 1. Ecg 2. 2 d echo. 3. Holter monitoring (24 hour continuous monitoring of ecg). He may need anti arrhythmia drugs to stabilize his irregular heart beats. Hope I have solved your query. I will be happy to help you further. Wishing good health to your husband. Thanks."
},
{
"id": 223374,
"tgt": "Can IUD mirena causes nausea, pink discharge and abdominal pain?",
"src": "Patient: Hi my name is Cindy. i had my son 08/09/2012 and early Oct i got an IUD Mirena. I have been nauseous i have abd pain especially when i sit and get up and i bleed for 2 weeks straight and i still have light pink when i wipe. i feel like im eating alot more and also i feel like i have more mucous when i would insert a tampon i noticed more white mucous than usual. Doctor: hello,yes mirena causes nausea,discharge,abdominal pain,alternation in menstural flow,headache,acne,weight gain,breast pain etc..nothing to worry.."
},
{
"id": 206026,
"tgt": "Suggest treatment for hallucinations",
"src": "Patient: My daughter is 20 Has had many diagnosis' - depression, mood disorder, anxiety, ODD. She is now having hallucinations and talking nonsense at random times. She has been admitted into Lauralwood hospital for 72 hours, What are some diagnosis' I could look into? Doctor: Hello thanks for asking from HCMYour daughter of 20 years have depression, anxiety disorder and OCD. She is now having hallucinations and she has started talking nonsense at random times. Such type of symptoms can occur due to some psychotic disorder or due to some mood disorder most likely due to bipolar disorder. If she is having violent abusive behaviour, demandingness, risk taking behaviour etc symptoms apart from what you have mentioned then most likely the diagnosis is bipolar disorder. In psychotic disorders hallucinations especially auditory type, suspiciousness, fear etc symptoms are see. Visit a psychiatrist for evaluation and treatment. Medicines like anti-psychotics as Olanzapine, Risperidone, Qutiapine etc can improve her hallucinations and will also help in decreasing her abnormal behaviour. Consult a psychiatrist for prescription of these drugs.Thanks, hope this helps you."
},
{
"id": 76924,
"tgt": "How to cure discomfort and pain in chest?",
"src": "Patient: hello, I had a car accident one week ago, and I crashed into a concrete wall with about 65-70mph. I didn`t have x-ray, but two days after the accident I am in pain (chest pain) when getting up, doing something I need to apply some force or laughing,ect, accomplined with pain in the right ear and righ side of the head ,which doesn`t stop.Doctors said my pain is from the seatbelt ,but it is getting worse and I do know whether is possible to be something else.Thanks Rumyana Petrakieva Doctor: Hi thanks for contacting HCM...You had car accident history ....So here first for ear pain get your ear examination with otoscope ....Look for tympanic membrane congestion...Second here your chest pain can be by.....-muscular strain-musculoskeletal pain -costochondritis -rib fracture...Chest xray done for it....Rest needed...Analgesic can be given...Compression bandage if needed applied.Avoid movement that cause pain....Physical therapy stretching exercise useful.Hot compress also useful.Heavy work avoided.Take care.Dr.Parth"
},
{
"id": 207071,
"tgt": "Suggest remedy for hallucinations, day dreaming and lack of concentration",
"src": "Patient: hello sir, i need ur help , there is always a story or a play running in my mind. like if i m watching tv show then the story of tv start running in my mind nd i start associating myself with them,,, i feel i m talking with them,,,,,v r doing fun togather..v r going to college nd party...but in real i m in home nd all this is just creation of my mind.. i tried a lot to get rid of these stupdity. i tried to concentrate on my work to make myself busy. but when i m free. all these absurd thing started again in my mind....my memory is alo become very weak now. i forget things where i placed just a second before... i feel very hesitated to this to my parents ...plz help me what to do Doctor: Hello,I think your problem is OCD. In this person knows that his thoughts/actions are absurd and he tries to resist or stop, but is unable to do so.You need treatment with anti-obsessional drugs.Once your thoughts are in your control, your memory and concentration would improve.Thanks."
},
{
"id": 179808,
"tgt": "Would taking flexon cause heat boils ?",
"src": "Patient: My daughter is 12 months old and has got heat boil from last 2.5 days. she has caught fever because of boil. Her ped. has given her antibiotics and asked to give flexon only if having fever only once per day. after every 6-7 hrs her body temp becomes 99.2.-99.9 F. Should i give her flexon after evry 6-7 hrs. or i should give other medicine like coldman?? Please help Doctor: Thanks for the query.For boils you need to take the complete course of antibiotics.The boils are not due to heat , but they are due to infection of hair follicle.Flexon can be given every 6 hourly for fever more than 100 F.For fever less than 100F , you need not to take any antipyretic.Coldman is anticold , you need not to give it for boils.Flexon will reduce the pain and also fever.Hope i helped you."
},
{
"id": 53867,
"tgt": "Is 301 level of sgpt dangerous in post liver transplant?",
"src": "Patient: yes sir i am 38 year old ,i got liver transplant sugary from india,now i have completed 4 month after treatment of my surgery i get blood test all test is clear just my SGPT is 301,and ggt become raised,so i worried about my blood test ?sir please help and advice me what should i do? thanks Doctor: Hello and thanks you for your query. I am Dr. Rommstein and I will try to help you as much as I can with my answer.This is not dangerous if there are no signs of liver failure such as jaundice, bleeding or pain. It may be sign of liver fibrosis or hepatitis so tests should be done and include ultrasound with viral markers.I hope I have answered you query. If you have any further questions you can contact us.Kindly regards. Wish you a good health."
},
{
"id": 221353,
"tgt": "What causes evening sickness,bloating and weight with a negative HPT?",
"src": "Patient: Hello i have evening sickness and all the classic signs of pregnancy including a swollen stomach when i have never had weight gain starting from the lower regions i have negative pregnancy test and this has been happening two months could it be possible that even so i could be pregnant and how can i know for sure? Doctor: Hello dear,I understand your concern.In my opinion as the pregnancy test is negative there might be no chance of pregnancy.The symptoms of pregnancy are nausea,vomitings,increased urination,sore breasts etc which start 2 weeks after missed period .So the bloating ,weight gain might be due to some other problem as the pregnancy test is negative.Even the stress regarding the thought of pregnancy also makes one to feel the pregnancy symptoms.So relax and avoid stress.And consider repeating the test after a week.Usually the pregnancy test gives accurate results a week after missed period.Best regards..."
},
{
"id": 198403,
"tgt": "Suggest treatment for nocturnal emission",
"src": "Patient: Sir , i am suffering from night falls (leakage) ,, Recently in one week 5times happen . I use tablet cystone (a product of himalaya) But i doesnt get better result .i tell you that i m suffering from it about 2years . Please sir help me . I am totally frustaded Doctor: HelloThanks for query .It is normal and natural to get night emission at the age of adolescence .It gets resolved spontaneously over a period of time and no treatment is required .Take high protein diet and have a moderate exercise every day so as to get sound sleep .This will definitely help to reduce the incidence of night emission .Dr.patil."
},
{
"id": 23493,
"tgt": "What could Echocardiogram findings be suggestive of?",
"src": "Patient: I am waiting for results of my Echocardiogram taken 2 week ago. I was shown a picture of my heart with red & blue, the blood pumping, it also had a 2 thick lines of green going down. Does green mean normal?In the last 2 years I have had multiple PI's twice, on warfarin for life now. Scan was done sfter ECG as my pulse does odd things Doctor: hi blue red mozaic greenish tinge all depens on the direction of flow of bloodafter capturing images of your heart by machine flow of blood is analysed by experts dont worry let the reports comered blue green mosaic alll colors atre significant and for various heart disorders colors are differentso there is nio golden rule to predict the pumping of heart just by seeing colorsvarious parameter like wall motions valve leakage etc are to be seen before making a final diagnosis thank youwish you a goood luck"
},
{
"id": 156951,
"tgt": "What natural treatment apart from MSM, Vitamin C & d-ribose should be done to treat swelling from cachexia due to lung cancer?",
"src": "Patient: My wife is suffering from swelling from cachexia due to her lung cancer..We are trying to heal her with natural treatments. We have been giving her 1000mg MSM, 500mg vitamin c powder, and d-ribose twice a day. Is there anything else you would recommend. Thanks Doctor: cancer cachexia is belt relieved by treating cancer. As long as primary is untouched, it progresses. Other way is by giving her good nutrition. Without good nutrition no amount of vitamins and minerals will help her. She requires high calorie diet along with vitamins which you are giving."
},
{
"id": 177219,
"tgt": "What causes yellow coloration of skin,white stuff in nappies and mucus and stringy bits in stool?",
"src": "Patient: Hi, my 5 month old is definitely not right. He is slightly yellow in colour but his eyes are still white. More worryingly he is having loose nappies with white stuff in them. It looks like bits of tissue paper. There are also stringy bits in it and bits of mucus. Should I be worried? Doctor: Hi...by what you quote it could be an evolving viral hepatitis or a viral diarrhea. Right now as his eyes are still white you need not worry. Coming to viral diarrhea - Once it starts it will take 5-7 days to completely get better. Unless the kid's having low urine output or very dull or excessively sleepy or blood in motion or green bilious vomiting...you need not worry. I suggest you use zinc supplements (Z&D drops 1ml once daily for 14 days) & ORS (Each small packet mixed in 200ml of potable water and keep giving sip by sip) as hydration is very important and crucial part of treatment. Regards - Dr. Sumanth"
},
{
"id": 122891,
"tgt": "What causes throbbing or fluttering sensation in the thigh?",
"src": "Patient: I m experiencing a light throbbing or fluttering sensation in my right thigh. It s not painful just annoying, sometimes it s occurs a few times with in 20 minutes, and other times it may occur a couple of times within 5 minutes.. This started about three or four days ago. Doctor: Hello, This could be due to compression of a nerve or could be a referred symptom of the sacroiliac joint. I need to ask a few more questions to come to the conclusion. Hope I have answered your query. Let me know if I can assist you further. Take care Regards, Dr Anuj Gupta, Spine Surgeon"
},
{
"id": 114014,
"tgt": "Can Indocap plain give permanent solution for back pain ?",
"src": "Patient: My father is 75 years old. He is taking Indocap plain for a long days,but it gives him a temporary relief. Can you provide any permanent solution? Doctor: hello indocap is a pain killer. would need to know for which disease is he taking this drug to know and discuss permanent solutions. how long has he been on this drug. indocap can cause lots of gastritis. so please provide more details to be able to discuss."
},
{
"id": 194992,
"tgt": "Suggest treatment for erectile dysfunction while trying to conceive",
"src": "Patient: Dear sir, i am having a very big problem and that is why i am trying to do sex with my wife in night my penis doesn t stand and in case if it stands it just goes down in 1 minute and we just unable to do sex and this is very disgusting. we are having this problem for a long time and this is so bad. So please help me sir its been 1 years since i have got married and we trying to have a baby but without sex how could we able to have a baby so please help sir Doctor: Hello and Welcome to \u2018Ask A Doctor\u2019 service. I have reviewed your query and here is my advice. 1. If the main can obtain an erection on his own by masturbation, but not with his wife during intercourse or cannot sustain it, then the cause is definitely psychogenic. 2. Other causes which need to get evaluated as the probable or hidden cause of ED are: - Any use of hypertensive or psychiatric drugs - Any medical condition like overweight, diabetes, high blood pressure/cholesterol] - Any psychological issue [stress/depression/anxiety] - Any substance abuse [adddiction] Please consult a Urologist for necessary examination Serum Testosterone and FSH/LH level] and investigation, till then: (i) start with perineal exercises: while urinating, stop the flow of urine, start again, stop & start again 4-5 times. (ii) Take: milk and milk products, ghee, cheese, curd, black grapes, pears, raisins, dates, almonds, walnut, pomegranate, carrot, banana, apricots, muskmelon, amla, coconut, pistachio, turnip, potato, onion, lady finger, radish, old brown rice, peas,garlic, bottle gourd. (iii) Avoid sour and cold substances, tobacco [in any form], brinjal, tandoori roti, red chilies, heavy and food which require long hours to digest, jaggery and its items, deep fried food. Before prescribing medication especially for ED, the patient should be evaluated for any cardiovascular diseases, thus have an appointment with a Cardiologist for any potential risk of a cardiovascular event. Hope I have answered your query. Let me know if I can assist you further. Regards, Dr. Munish Sood"
},
{
"id": 176948,
"tgt": "Suggest treatment for loose motions and high fever",
"src": "Patient: My 3 and 10 month old baby is suffering from loose motions and high fever since yesterday. Our pediatrician suggested Zinconia syrup and Nutrolina B for the loose motion and also calpol syrup for fever. There is no improvement in fever or motions so far Doctor: Diarrhoea in infants is a very common condition most commonly caused by rota viral infection, sometimes gram negative bacteria. It lasts for 5-7 days and then goes off. So just treat your child symptomatically with syp calpol , give syp zinc 5 ml once daily for 14 days and hydrate the child adequately.give ORS solution to drink and replace every loose stool with 5-10 ml of ORS."
},
{
"id": 174272,
"tgt": "What causes large labia to a 9 year old?",
"src": "Patient: Hi, My 9 year old daughter (just 9) has a very large Labia- she is quiet petite for her age. Is that normal? The nurse today said it isn't! She's going to see a female doctor next week. What could it be caused through and what might the next steps be? Thank you Doctor: Hi,Thank you for asking question on health care magic.Big sized labia is not a problem,but any other developmental defects associated with it should be ruled out as early as possible.Better consult gynoecologist.Hope this answer will serve your purposePlease feel free to ask any more queries if requiredTake careDr.M.V.Subrahmanyam MD;DCHAssociate professor of pediatrics"
},
{
"id": 190578,
"tgt": "Stitches on tongue have come out, no bleeding, pus oozing. What should I do?",
"src": "Patient: I am a 60 yr old male 5 10 205lbs. During a dental procedure, I moved my tongue and my dentist caught it with his drill. He put in dissolvable stitches (under my tongue, close to a vein . It has been 48 hrs and some stitches have come out. There is no bleeding, but there is some pus and I feel like I can see my vein through the open slit. what should I do? Doctor: Hi Sharocw, this is unfortunate that your tongue got into the drill. It is good that he sutured it immediately. See normally we use non nylon MERSILK suture in such situation, which we remove in 6-7 days time period. As 48 hrs has passed, the primary healing of the area is finished. but, you had written that some pus is there, please dont ignore it as it is a danger area, infection from here can spread very fast to the neck region and take a shape of ludwig angina, which can be fatal. I dont want to scare you but i want you should visit to your dentist immediately and take proper treatment for it. Thanks, Dr. Vishal Jain."
},
{
"id": 224317,
"tgt": "What causes delay in period after having i pill?",
"src": "Patient: Hai..mam i had sex with my gf just 5 days before and she had taken ipil after 24 hours and today 22dec is her period date does ipil casues the delay of the period and she had not get the periods today plzzz mam by ur valuable sugastn we r based plz mam hlp us from dis tensn plz mam Doctor: HiFrom your notes it is not clear when was your girl friend's last menstrual period?The first week and the last week of menstrual cycle are safe period. If you had sex during this time then there is no possibility of pregnancy. If you have had sex on any other day then you need to take I PILL if you wish to avoid pregnancy. I PILL is most effective when taken within 24 hours of intercourse. If I pill is taken in the second half of menstrual cycle it can delay the period. If period is delayed by more than a week then you need to do pregnancy test as there is a possibility of method failure ( I PILL is not 100% effective).I have an advise for you. If you wish to avoid pregnancy in future, your girl friend should consult doctor and start using regular contraception like birth control pills or intrauterine contraceptive devise as they are more effective than emergency contraception in preventing pregnancy. In addition please use barrier contraception like condom to protect yourself from sexually transmitted diseases.I hope I have answered to your satisfaction. If you have any further queries do contact me through Healthcare Magic."
},
{
"id": 47978,
"tgt": "Should hemodialysis be started due to skin swelling and itching?",
"src": "Patient: Hi dr i am 55 years old ,i have a kidney problem and i was using hemodialisis but now i m using drugs to take the water from my body i was okay but my skin starts swolen and itchy then i went to my dr and he told me your blood is dirty and he told me to start the hemodialisis . Should i start hemodialisis please i m so worried and sick. Thank you Doctor: Hello and welcome to HCM.Your concern is understandable.Your kidney problem is called CKD-Chronic Kidney Disease.,also Chronic kidney failure. In this stage, dialysis is done, until you opt for a kidney transplant.Hemodialysis is better and much safer, than a peritoneal dialysis.(CAPD).Both have side-effects, but hemodialysis is done under medical supervision.Don't worry, as hemo is done, if there are some complications with CAPD.If you need more expert advise, you may contact me by name."
},
{
"id": 79018,
"tgt": "What causes episodes of tingling in hands,tightness in chest and numbness?",
"src": "Patient: Starting out with numbness and tingling in both arms, and then tightness in my chest, went to ER, everything turned out fine, followed up with my dr, said it was anxiety and put me on xanax, I keep having these spells, they come on sudden, longest one lasted about 7 minutes, now I'm having it in my back as well Doctor: If these attacks are hampering your daily routine frequently the it would be advisable to consult a psychiatrist if all other workup and investigations are normal."
},
{
"id": 203576,
"tgt": "Blister like bumps on the penis and under the foreskin without pain / itch in a sexually active man",
"src": "Patient: Hello, so I am sexually active with my girlfriend who I have been with for a year now, im her first and she definitely has never slept with another guy, 3 days ago I had blister like bumps on the head of my penis and under the foreskin, they were in patches containing 4-6 in the one spot about 1mm diameter, they were hazy in colour and very easy to pop, it was a clear liquid and there was no red surrounding them nor any swelling and no pain even while and after popping them, no itching or anything, they left little red patches that just looked like I lost a layer of skin in the areas effected, up to date I have had no scabbing or open wounds, just peeling of the skin creating that while looking dick cheese that some people call it (sorry I don t know how else to refer it) and as day 3 it is barely noticeable that there is any red and the peeling of skin had basically stopped thanks for your help! Doctor: hithanks for using healthcare magicAlthough there are multiple causes for blister on penis, but i think in ur case it may be due to some external injury or due to sweating. In case of infection like herpes, blister must have been painful, reddish in nature which was no their in ur case. Do not worry about it, In future be cautious. In case u get these blister again, then consult a doctor to rule out herpes.Thanks"
},
{
"id": 44285,
"tgt": "Done semen test. Prescribed Maxoza and Redguard. Any chance of conception?",
"src": "Patient: hello doc my husband had a semen test in which sperm count was 148mill/ml but active motility was 15% doctor has prescribed to take maxoza powder and redguard capsules I just want to ask is there any chance of conception with low motility and can he take maxoza tablets in place of powder. hoping for a positive response Doctor: Hi MansiWelcome to HealthcareMagicSperm count, motility and morphology are most important parameters of semen analysis. According to WHO,total motility of 40% out of which progressive motility should be at least 32% is considered normal and natural conception can be expected if there is no other contributing factor to infertility. Low motility could be due to various reasons and important one is infection. Get a semen culture done to rule out infection and if found should be treated by course of antibiotics.Get your husband evaluated by an andrologist to rule out Varicocele which may also cause low motility. Multivitamins and other antioxidants will be of help( Tablet or powder anything can be taken ).Take care."
},
{
"id": 179442,
"tgt": "Is ambrolite advisable for cough in child?",
"src": "Patient: Hi Doctor , My son is 22 months old and weighs 11kgs . Recently he has been coughing a lot (not dry cough) and had slight temperature last night. I called the pediatrician and he advised us to give 2.5ml of Ambrolite-D, 3 times a day . Will that be ok. What is the difference between Ambrolite S and D ? Regards, Gokul Doctor: Thanks for posting your query at HealthCareMagic. Your son has a respiratory tract infection. Usually it is caused due to a virus and has an allergic component associated. Both Ambrolite D and Ambrolite S are safe to be given. Dose is ok. Ambrolite S contains ambroxol, guaiphenesin and salbutamol whereas Ambrolite D contains Cetirizine, Dextromethorphan, Pseudoephedrine and Menthol. The actions are similar. The main difference is due to the salbutamol which is a bronchodilator that helps in widening the airway tracts when there are spasms that cause wheezing.Hope that I have been able to address your queries. Feel free to revert back in case of further questions if any."
},
{
"id": 55310,
"tgt": "Are broken blood vessels with rashes related to liver issues/damage?",
"src": "Patient: I have what appears to be broken blood vessels just under the skin in my lower legs. It s creating a splotchy rash appearance. Went to PCP today. They ordered blood work, labs, etc. They seem to be concerned it may be a liver issue, but they had no other information to give me...any thoughts? Doctor: Hi thanks for asking question.Your doctor want to rule out chronic liver disease in your case like cirrhosis.In such case telengectic dilated blood vessels may present with rash over body due to hyperestrogen.For that ultrasound exination and viral markers done.Second you are also adviced your blood examination.If platelet counts are low then also rash can occur.Some allergic reaction and with advancing age also purpura can occur.Vitamin c deficiency also can lead it.I hope I have solved your query."
},
{
"id": 77727,
"tgt": "Suggest treatment for pulmonary embolism",
"src": "Patient: My 88 1/2 year old father was diagnosed about 3 weeks ago with a pulmonary embolism. He was on coumadin, (sp?), for about 3-4 years then went off of it in late January/early February, and, even with this diagnosis, does not want to return to it. He lives in a wonderful assisted living community, does not smoke and is not over weight at all. Fully aware that all situations are different, is there any kind of forecast you would be willing to share as far as what to expect as far as his quality of life and life expectancy? Thank you! Doctor: Thanks for your question on Health Care Magic. I can understand your concern. Pulmonary embolism is common in old age due to atherosclerosis and stiffening of arteries. Severity, life expectancy, prognosis etc depends on 1. Size of embolism (massive embolism can cause sudden death). 2. Involvement of pulmonary artery. If main Pulmonary artery is involved then prognosis is poor. But if distal, smaller arteries are involved then prognosis is good and hence life expectancy is more. 3. Severity of pulmonary hypertension. Pulmonary embolism (PE) almost always cause Pulmonary hypertension (PH). So if PH is severe then prognosis is poor. So please let me know these details, so that I can guide you more about life expectancy in your father's case. Hope I have solved your query. I will be happy to help you further. Wishing good health to your father. Thanks."
},
{
"id": 12687,
"tgt": "Suggest remedies for a rash and acne after a spider bite",
"src": "Patient: my daughter was bitten by a black widow and ever since she gets a terrible rash, also during this time she also got a tatoo, she's been to several doctor's and nothing has worked ,she also developed allegy's to hair dye. Her rash is raised, very painful and acne like in appearience Doctor: Hi.I've gone through your query and I can understand your anxiety. From the history, your symptoms might be an outcome of contact or allergic dermatitis (allergy to hair dye), Atopic dermatitis or eczema. - If contact or allergic dermatitis, the main treatment is to stop further exposure to allergen, hair dye or tattoos - Use antihistamines, Moisturizer and lotions- Topical steroids Hope I have answered your query. Let me know if I can assist you further. Take care. Regards, Dr. Yogapriya V, General and family physician."
},
{
"id": 115844,
"tgt": "Suggest treatment for low potassium level",
"src": "Patient: my mom had a potassium level of 2.4, she was hospitalized and given potassium intravenously. She still has a low level, please provide me some reasons for the low level and decrease in her levels & what can be done to help her maintain a healthly level? Doctor: Hello,it depends on the cause. Potassium replacement that fails to raise potassium levels is a finding commonly encountered when low magnesium levels coexist. She needs a magnesium measurement and potential replacement. The intravenous is the most efficient way to rapidly replenish potassium losses. If she's capable of taking drugs by mouth then she could also take potassium orally.The most important thing though is to treat the initial problem that caused the low potassium levels.I hope I've helped!If you'd like more information, please let me know. I'll be glad to answer.Kind Regards!"
},
{
"id": 152171,
"tgt": "My father got MRI Brain result.All are OK but he had Lung Cancer",
"src": "Patient: Hello, My dad has gotten his MRI Brain results back .. all is normal, save for a \u201cworsening chronic microvasular disease\u201d. \u201cMild to Modertate Hight T2 signal lesions of chronic ischemia\u201d All else is normal. He had a lung cancer diagnosis 5 years ago, received chemo and radiation. All seems \u2018ok\u2019 there, save for the hacking cough. I am concerned about these recent MRI findings .. he complains of dizziness more than ever these days, has pronounced mood swings, is tired, not too hungry. Has trouble hearing, but his results are that his hearing is fine. He\u2019ll see his doc in a few days, but I wanted to possibly provide him with a sense of how to engage the doc \u2013 my dad is a great repressor, does not speak English well, and will certainly not report what may well be cognitive issues. How might I read the MRI results? Thanks for any help you might give, Frank Doctor: Hi,thanks for query.Chronic ischemia means there are certain areas of brain which are not getting proper blood supply since long.the symptoms that he has like dizziness and other problem might be due to this.He will need some more investigation like lipid profile,kidney reports,sugar etc.Talk to his doctor about problems he will correlate things together on his own.bye."
},
{
"id": 139838,
"tgt": "What is the fluttering sensation am I having in my head?",
"src": "Patient: Feels like a fluttering sensation, like a running sensation (flowing) like gigaling - on the right side of my head above my ear approximately 2 to 2 1/2 inches. Doesn t hurt, just feels Not constant, but maybe every few minutes without warning it just happens. When I touch my skull and push on the area, it stops. Doctor: Hello, Your symptoms do not seem to be related to any serious medical condition. Nummular headache (which is a benign type of headache) could mimic this clinical situation. Anyway, I recommend consulting with a neurologist for a physical exam. Hope I have answered your query. Let me know if I can assist you further. Take care Regards, Dr Ilir Sharka, Cardiologist"
},
{
"id": 198899,
"tgt": "Is there any abnormality if the sperm count is 14.5 million?",
"src": "Patient: hi sir myself vijay age 32,men,during semen analysis report shows i have sperm count 14.5 million per ml,viscosity normal,appearance narmal,fructose present,motility rapid pressive 55%,sluggish 25%,non progressive 5%,non mortile(dead)15%,pus cell 20 hpf,i am capable to become father to child? Doctor: HelloYour semen analysis findings may indicate semen infection with low sperm count.Sperm count is low.Normally it should be at least 20 millions/ml.Your findings indicate 14.5 millions/ml,so it is low.There are also excess pus cells in semen.You may need investigations like routine hemogram,random blood sugar,semen culture and sensitivity,ultrasound of scrotum.You may need to take antibiotics.Motility is normal.Normally it should be at least 55 %.Other findings are also normal.Fertility and sperm count are expected to increase with control of infection.If findings persists then you may need assisted fertilization technique.Get well soon.Take CareDr.Indu Bhushan"
},
{
"id": 142557,
"tgt": "What causes dizziness when diagnosed with haematoma at the back of the head?",
"src": "Patient: my Aunt fell at the airport this am and had a big haematoma at the back of her head, I wasn t there so did not witness it but my cousins said that she was dizzy for a bit and was shaken up, took her to he first aid area, iced her head but she was able to make it to the flight 3 hours later. It is a long 14 hour flight, I am worried about stroke, can you pls tell me anything about the situation? thx Doctor: Hello!Welcome on Healthcaremagic!I would explain that dizziness is quite common after a head concussion, even in the absence of stroke. But, we can not exclude intracranial bleeding like a delayed subdural hemorhage. For this reason, I would recommend having some rest and avoiding physical activity. If the dizziness does not improve, or even worsens, or she experience symptoms like gait distubances, confusion or nausea, I would recommend bringing her to the ER for a brain CT scan, to exclude intracranial bleeding. Hope you will find this answer helpful!Kind regards, Dr. Aida"
},
{
"id": 61863,
"tgt": "What is the treatment for a lump in the head?",
"src": "Patient: I have a lump on right side of my head about an inch above my ear. its about the size of a nickle and soft doesnt hurt. was there out of nowhere for about three weeks. now im getting headaches and feeling pressure at times where the lump was. it went away and now is hard and the size of a pea. What is this? Doctor: hi.it is best if you consult with a doctor for physical examination and clinical evaluation. diagnostics, such as imaging (x-ray, ct-scan or MRI) and biopsy, may be suggested for further evaluation of the mass, which could be any of the benign lesions (cystic formation, lipoma or hemangioma). other tumor types (such as malignancy) must also be ruled-out.management (medical and/or surgical) will be directed accordingly.hope this helps.good day!!~dr.kaye"
},
{
"id": 200237,
"tgt": "What causes blood in sperms during intercourse?",
"src": "Patient: I am 66 years old I take Viagra once a week with my wife the rest of the time I masturbate daily this last week I notice that when I come I have some blood on my sperm, I have no pain or symptom ,I would like to know if it is the pill or maybe I just masturbate too much, could you please let me know something, thank-you,joe Doctor: Thanks for asking in healthcaremagic forum Please provide details regarding any frenulum injury while having intercourse/masturbation and also give details about bleeding, whether the blood was mixed with semen/seen separate/was it bright red/black etc for further suggestion. All the best."
},
{
"id": 107210,
"tgt": "How can spinal disc herniation be treated?",
"src": "Patient: Hi I was in a car accident a year ago. I rear ended someone on an angle going about 50mph. My head hit the steering wheel and the airbag did not deploy. I did have a concussion, a punctured lung, and whiplash. I thought I healed well from the accident, but a year later I am in worse shape. I have muscle spasms, a torn Lattisimus dorsus, and constant next stiffness and pain. I am having an MRI next week and a CT scan showed I had a slight herniation of t12-t11. I am wondering if there is hope for me to fully recover with physical therapy? I am 26. Doctor: Hello,Thank you for using Healthcaremagic.I read your question and understood your concern.I think you can fully recover since the disc herniation is slight.You may need a lot of physical therapy exercise to strengthen your back muscles and abdominal muscles to prevent more slippage of the disc.Wish you quick recovery.Dr. Selmani"
},
{
"id": 177857,
"tgt": "What causes crying in a child after a fall?",
"src": "Patient: Hi, my bb is 12 days old and fall down from my arms while bf her and asleep. Its about 2 1/2 ft , facing down to the tiles. I panicked and immediately pick her up and she cried about 30 second, stop and sleep back. She looked pale, no bruises on head and found out her cheek is swollen. Bring to ER, she no longer pale and I bf her. DR examine her and says everything is fine. No CT scan done as she is too young to go through it.Advised to monitor her for 3 days. Now it past 3 days no vomiting. But my bb shocked during sleep until her hand and feet lifted. She sometimes cry loudly and stop after i held her. Sometimes her tongue is placed between lips during sleep. I wonder if it is normal after the incident. I still worried of her development later on. I couldnt rely on my own judgement if she is fine or not because its hard to monitor any changes in newborn. Seems everything is fine but I still worried a lot.tq Doctor: Hi...putting tongue in-between lips while sleeping is not related to head injury. I understand your concern. First be reassured that your kid is normal. I will suggest you danger signs of head injury -1. Vomiting continuously2. Seizures3. Watery of bloody discharge from ears and nose4. Unconsciousness5. Altered sensorium or behaviour. If none of them are present, I don't think you should worry about her trivial head injury. Regards - Dr. Sumanth"
},
{
"id": 12614,
"tgt": "Which tube can be used for psoriasis?",
"src": "Patient: which tube can be used for psorasis Doctor: hi psoriasis is chronic condition u need to apply some cream for long time and regularly depend upon the severity and grade u need medicine in some case oral medicine are required so better to consult dermatologist . cream with steroid and salicylic do best u need to use shampoo with tar keep u r skin moist by applying good moisturizer . dont let us skin to dry as it will increase ur problem."
},
{
"id": 79143,
"tgt": "What causes a painful chest after drinking fruity sugary drinks?",
"src": "Patient: Sometimes when I drink fruity sugary drinks (apple cider, or regular pop) or eat watermelon, I get a constricted almost painful feeling in my chest. It is directly connected to the sugar intake, and always passes after 15 to 20 minutes. Could it be related to a metabolic problem? Doctor: Thanks for your question on Health Care Magic. I can understand your concern. Your chest pain is associated with intake of certain food. So possibility of GERD (gastroesophageal reflux disease) is more. It is due to laxity of gastroesophageal sphincter. Because of this the acid of the stomach tends to come up in the esophagus and cause the symptoms of Central chest pain, chest tightness, pressure etc. So better to start proton pump inhibitors. Avoid foods which cause you trouble. Drink plenty of fluids orally. Avoid stress and tension. Avoid alcohol and smoking if you have these habits. Don't worry, you will be alright with all these things. Hope I have solved your query. Wish you good health. Thanks."
},
{
"id": 35729,
"tgt": "What causes fever, stomach pain, vomiting and red spots on hands?",
"src": "Patient: i have 99-100 F fever in the evening only for last 3 days....stomachache is also there... i feel weakness also...after taking food...i feel like vomiting..and today morning there are red small spots on my hands which disappears after an hour....... what should i do. Doctor: Hello dear,Thank you for your contact to health care magic.I read and understand your concern. I am Dr Arun Tank answering your concern.It is the case of food poisoning.I suggest you to maintain your optimum hydration it is the first priority. Once your hydration is maintained you are protected by the vomiting and diarrhoea.I suggest you to take the ORS or Oral Rehydration Therapy for maintenance of hydration.Please do culture and sensitivity from the stool sample, taking treatment according to the report will cure you early and cheaply.Please eat curd as much as one can eat. It will replace the gut flora of bacteria and can help you recover from food poisoning.You can take cetrizine tablet for the tiny rashes on the hand.I will be happy to answer your further concern on bit.ly/DrArun.Thank you,Dr Arun TankInfectious diseases specialist,HCM."
},
{
"id": 187771,
"tgt": "What needs to be to treat a lump inside the tooth?",
"src": "Patient: i had a root canal done years ago and my crown and posts fell out about 4 years ago - i have never done anything but now there is a lump inside what is left of my original tooth- doesn't hurt but i can't chew and the lump fills up the remaining space of the tooth. is this an abcess? Doctor: Dear User,Thanks for using health care magic.From the available description it appears that lump is not a abscess.This may be a cyst.The second possibility is of a tumor. it may be benign or malignant. If it is rapidly progressive than chance of being malignant is much more.In my view you need to see your dentist so he can check and evaluate you accordingly.'Hope I have answered your query. If you have any further questions I will be happy to help\".Thanks"
},
{
"id": 56340,
"tgt": "Should the gall stones be removed having severe pain and chest burning?",
"src": "Patient: I ve got gull stones that I was diagnosed with dobtbknow if I should get gull bladder removed bc of the pain I ve been having from it but on top of that I ve got burning sensation UN chest and bad pains in middle of chest as well as Ichoke up white foam right when I first lay down what is going on Doctor: Hi, dearI have gone through your question. I can understand your concern. You may have cholecystitis with acidity. You should go for ultrasound abdomen to know exact size of gall bladder and confirm the diagnosis of cholecystitis. If needed go for cholecystectomy. You should also take Proton pump inhibitors like pentoprazol or rabeprazole. Consult your doctor and take treatment accordingly. Hope I have answered your question, if you have doubt then I will be happy to answer. Thanks for using health care magic. Wish you a very good health."
},
{
"id": 81347,
"tgt": "What causes painful knot in chest?",
"src": "Patient: What does it mean if you have a knot in your chest ? my chest been hurting me so bad I can barley move , & I been had this little hole under my left breast & now it s a knot in it & it hurt really bad when I touch it or lay down on my stomach .. & I always get knots under my right arm pit .. Can you please tell me what all this stuff means? Doctor: Thanks for your question on HCM.In my opinion you are having enlarged Lymphnodes.Your history and description favours possibility of enlarged Lymphnodes only.Lymphnodes are lymph draining organs and can be enlarged in infection.You are also feeling pain, which again favours infective enlargement of Lymphnodes.But better to consult doctor and get done clinical examination of swellings.You may need biopsy of the lesion if doctor feels so.Otherwise broad spectrum antibiotics are sufficient to treat enlarged Lymphnodes.Sometimes lung infection can also cause enlarged Lymphnodes, so get done chest x ray to rule out this."
},
{
"id": 112688,
"tgt": "Recurring sharp shooting pain in lower right back. Have scoliosis with lower curve on left side. No tests or X-ray done. Help",
"src": "Patient: I have been having ONLY lower right back pain for months now and the past couple of months it has gotten worse, the past week it has gotten so bad that I have become dependent on pain medication, and today it is so bad that it hurts to walk and pain medication is having little effect. It is specifically in my lower back area and it is a dull aching pain unless I move it and then it is a sharp shooting pain. I am a 21 year old female with scoliosis(I have a S Curve) but my lower curve is on the left side of my body and all of my pain is on the right side. I have seen my doctor twice and he has not taken any tests or done an X-Ray. I have been taking physical therapy but that seems to be doing very little to help the situation. Doctor: Hi, Thanks for using HCM.The pain you are suffering may be due to musculoskeletal pain, disc problem or Spinal injury due to nerve compression.Usually spinal or nerve compression afects other structure (shooting pain/numbness/tingling). If disc is affected and musculoskeletal pain only localised severe pain will be there. So take Tab. Aceclofenac with muscle relaxant chlorzoxazone twice daily and stretching exercises for lower back. Get done one CT scan for diagnosing any problem in disc or compression of nerve. Discuss with your doctor for further management. Hope this will help you. Hope I answered your question. Feel free to ask me if you have any further queries. Wish you good health. Take care.RegardsDr.Lohit"
},
{
"id": 16566,
"tgt": "What causes atherosclerosis?",
"src": "Patient: Mr. T., a 45-year-old black male employed as a midlevel corporate manager was seeking a physical examination. He appeared somewhat overweight. He denied taking any medications or smoking, but admitted drinking alcohol. His father and older brother have hypertension, and his paternal grandfather experienced an MI and a CVA at a young age. Mr. T. stated, \u201cA year ago at a health fair, my cholesterol was tested. I was told later by mail that my cholesterol was 250.\u201d What lab tests are indicated? What other questions should be asked of this patient? What are Mr. T\u2019s risk factors for hypertension and coronary artery disease? If Mr. T. showed a high level of intellectual curiosity, and kept asking questions, how would you explain the pathophysiology of hypertension and coronary artery disease to him? How does atherosclerosis develop? Doctor: Hello!Welcome to Ask a Doctor service!I understand your concern and would explain that atherosclerosis is a gradual process. Arterial hypertension and dyslipidemia are independent risk factors for atherosclerosis, which increase its speed of development. From the other hand, atherosclerosis can lead to gradual blockage or the coronary arteries and brain arteries and increase the risk for coronary artery disease or stroke. That it why it is necessary to periodically check your blood lipid profile, a resting ECG and Doppler ultrasound of the carotid arteries. Hope to have been helpful!Wishing good health, Dr. Ilir Sharka, Cardiologist"
},
{
"id": 94868,
"tgt": "Occasional sharp pains in lower abdomen after drinking cold water in the morning post breakfast. Cause?",
"src": "Patient: I ocasionally have a sharp pain in my lower left abdomen after I drink a glass of cold water in the morning, usually after breakfast. The pain goes away rapidly and I continue to drink water as well as small amounts of food a day with no reoccurance. I consume alot of water and well as healthy foods with raw fruits and vegetables daily to prevent constipation . What is causing ocasional pain? Doctor: Hello,brandtphoen, Most often the occasional abdominal pain after eating or drinking occurs due to contraction in the intestines called \"Peristalsis\". This happens in some more often than others. Cold beverages tend to cause more contractions than hot or warm drinks. Try to chew your food slowly and take the drink after you are done eating. Drinking during the middle of the meal sometimes you swallow more air which can add to the bloating or distension. I am glad that you are trying to get enough fiber in your diet by eating more vegetables and fruits. Keep that up. If your bowel movements are regular then there is less chance for gas getting trapped in the intestine and giving you pain. Wish you good health."
},
{
"id": 92661,
"tgt": "Sick after drinking whiskey, beer. Stomach pain, bowel movement painful, threw up clear liquid. Solution?",
"src": "Patient: I went to a party last night. I drank half a beer and a Shit of whiskey and later I began to feel sick. Late at night I woke up feeling I was going to throw up but instead I had a bowel movement that stung and it hurt. I felt somewhat better and went back to sleep but the same thing happened again. I had another rush to the bathroom and the same things happened. Later I threw up clear white liquid and I tried to make my self throw up more. I couldn't. So I just took gravol and tried to sleep.I didn't get out of bed today, just to go to the bathroom and get water. But it's now 9 at night andmy stomach STILL hurts. What do I do? Doctor: Hi,It appears that the alcohol you took has brought on an episode of gastritis. Just taking Gravol will not help. You shall need strong medicines to stop the acid production in the stomach, like proton pump inhibitors (omeprazole, pantoprazole etc), so that acid does not further inflame the already damaged stomach lining. Antacids with cytoprotective agents like sucralfate will also help. In my opinion, you should see a doctor, as you may need certain tests and also since there may be bleeding if this condition worsens. Till you get to see a doctor, avoid hot and spicy foods and take over the counter antacids.Hope this helps you. Best wishes."
},
{
"id": 124018,
"tgt": "What causes pain while breathing after an injury?",
"src": "Patient: i fell on the ground & for a second couldnt breathe but after that i was fine the next day i started feeling pain where my ribs are i only really feel it when i take a deep breathe yawn or sneeze is this a bruise fracture or break & will this be able to heal on its own & how long will it take Doctor: Hello, As with the history to clear the confusion out whether it is a bruise or a fracture of the rib, a simple chest x-ray will be good enough. If it was a big fracture there might be much more changes. Any direct injury to the rib cage can lead to pain and difficulty in breathing, sneezing and coughing. Most rib fractures, if they are hairline, heals on its own, but x-ray will be a good choice still to take it for safer side. Also, whenever you are sneezing, yawning, or doing deep breathing, I will advise you to put the hand over the painful area so the pain doesn't increase. In my clinical practice of over 12 years, most rib fracture cases or pain in ribs due to falling recover well with conservative therapy and preventive measures. Hope I have answered your query. Let me know if I can assist you further. Regards, Jay Indravadan Patel, Physical Therapist or Physiotherapist"
},
{
"id": 108847,
"tgt": "Suggest treatment for back pain",
"src": "Patient: I have a block that keeps getting darker on my lower back the skin douse not hurt but my back douse right where it is right down to my hip bone on only my left side and a pulling stranging pain when I lean to my right and my back gives way on me some times Doctor: Hello, I have studied your case. At old age there will be age related spondylitis but if there is any nerve compression then that may need specific treatment.I will advise you to do MRI spineTill time, avoid lifting weights, Sit with support to back. You can consult physiotherapist for help.Physiotherapy like ultrasound and interferential therapy will give quick relief.Hope this answers your query. If you have additional questions or follow up queries then please do not hesitate in writing to us. I will be happy to answer your queries. Wishing you good health.Take care."
},
{
"id": 53033,
"tgt": "Suggest treatment for deranged liver function tests",
"src": "Patient: I AM 36 YRS MAN DURING MY RECENT HEALTH CHECKUP,THE RESULTS ARE AS FOLLOWS: SGOT 48 SGPT 51 ALKALINE PHOSPATASE 279 SERUM URIC ACID 9.2 SERUM CALCIUM 8.6 RAM BLOOD SUGER 80 HAMEOGLOBIN 14.5 KINDLY ADVICE AND EXPLAIN STATUS OF MY LIVER PROBLEM AND TREATMENT FOR WHICH I SHALL BE GREATFUL TO YOU.PLESE SUGGEST WHAT KIND OF DITE I HAVE TAKEN. Doctor: Hi and welcome to Healthcaremagic. Thank you for your query. I understand your concerns and I will try to help you as much as I can.As I see from these findings, there is only slightly elevated SGOT and SGPT and if you dont have any other symptoms and signs, this is not concerning finding. Pthe findings are fine.SGOT and SGPT are sensitive indicators of liver damage or injury from different types of diseases or conditions but only when elevated more than 60. This still can be normal finding or indicate mild fatty liver or medications intaek that damage liver slighty. So I would not recommended any further diagnostic tests. All you need to do is to repeat test in 4 weeks and if there is increasing trend of LFts then ultrasound should be done. Also, have easier dier and avoid alcohol. if such findings persist then beside fatty liver, other causes are viral hepatitis,cirrhosis, medications, alcohol intake or autoimmune diseases may be considered but as I said, at this point this is not concerning sign.I hope I have answered you query. If you have any further questions you can contact us in every time.Kindly regards. Wish you a good health."
},
{
"id": 35813,
"tgt": "How to treat folliculitis with MRSA infections?",
"src": "Patient: I was just diagnosed with B12 anemia. level 146. I also have been battling with folliculitis for 1and 1/2 years with over 21 MRSA infections in these wounds. does anemia have anything to do with this and what can I do to get rid of this folliculitis? I have been on ABT therapy for 1 1/2 yrs also. Doctor: Hello dear,Thank you for your contact to health care magic.I read and understand your concern. I am Dr Arun Tank answering your concern.No, there is nothing to be related with anemia. But prevalence of infection is more in anemic person than usual ones.I advice you to start treatment with the vancomycin. It is the drug of choice for MRSA but please take this treatment under your doctors guidance.You also have to take care for the anemia. Anemia in you is of vitamin B12 origin. So you need to take both vitamin B12, folic acid and iron. Please take the tablet of the above medicine under your doctors guidance.Please maintain good life style habit. This can be achieved by good hygiene and exercise. This minimum effort will keep you away from the infections.I will be happy to answer your further concern on bit.ly/DrArun.Thank you,Dr Arun TankInfectious diseases specialist,HCM."
},
{
"id": 205658,
"tgt": "Suggest treatment for social anxiety disorder and depression",
"src": "Patient: My 22 year old son is going to be seeing a doctor in a week for headaches and a possible seizure. I m wondering if I sent his doctor an email to inform him of his high anxiety levels, social anxiety, depression, etc. That he s suffered since middle school if she would keep it confidential and not tell him I contacted her. It would do some real damage if he knew I contacted his doctor. I don t think he would lie to her but he doesn t see himself as the family does and may not bring up these issues. This is the first time he s agreed to go to a doctor and I really just want to be sure she knows. Should I let it go or will it be kept between us? ?? Thank you. Cathy Doctor: DearWe understand your concernsI went through your details. You do mention that your son agreed to see a doctor for the first time. In that case it is also sure he realizes the seriousness of the issue. he understands his problem. This is the correct time for you to take your son into confidence. I suggest you to be with your son when the consultation takes place. You may chip in whenever opportunity arises. that would be helpful. Your son may not know the details of his childhood and only you will be able to tell those. Nothing wrong in telling facts to the doctor. Be truthful and faithful. Don't exaggerate. You may email the details to the doctor. But usually doctors do not attend cases with presumption. go ahead. Everything will be alright. If you require more of my help in this aspect, please use this URL. http://goo.gl/aYW2pR. Make sure that you include every minute details possible. Hope this answers your query. Available for further clarifications.Good luck."
},
{
"id": 225968,
"tgt": "Had unprotected sex, taken Gynaecosid. Will it work?",
"src": "Patient: Hello, I just had unprotected sex with my husband about three hour ago. After an hour I took one tablet of Gynaecosid and am yet to take the second one. Please I will like to know if it will work. I am within the 12 day after my mensuration. I had unprotected sex eight day after my period but I felt am save so didn't use any drugs but today is the 12th - 12th day so I decided to take one tablet of Gynaecosid. I hope it works. Doctor: Hello,Thanks for posting your query.Fertile period is 3 days before & 2 days after ovulation & you are in mid of your cycle.To avoid pregnancy you should take emergency contraceptive pills.Gynaecosid is used in secondary amenorrhea excluding pregnancy for withdrawal bleeding & is not an emergency pill.I hope your query has been answered.Regards,"
},
{
"id": 151377,
"tgt": "Have secretion from head, weight loss, sore throat, tonsillitis. Medicines prescribed. Possibility of tubercular meningitis?",
"src": "Patient: im 46 years old ... since the past 3-5 years , i have been facing the problem of getting secretion from the upper side of head ... i am experiencing weight loss of approx 7 kilos , sore throat and tonsilitis frequently ... doctor , ENT specialist advised me levocetrizine dihydrochloride and montelukast sodium tablets ... azelastine hydrochloride and fluticasone propionate nasal spray .... i am afraid if im developing tubercular meningitis ...my mother recovered from TB meningitis 3 years ago ..... please help with your valuable advice .. Doctor: Dear Rajesh, Getting treatment for your current symptoms from an ENT specialist is different matter but for your weight loss of 7 kg, you must see a good physician. The physician will rule out many conditions which cause weight loss in a person who is not trying weight management intentionally. Common conditions being diabetes, hyperthyrodism, HIV infection leading to AIDS, TB, hypocortisolism, chronic diarrhea which deprives the body of nutrition from the eaten food etc. Certainly your family history of TBM will help them take decision by having look at you whether you also need to be tested for any kind of TB as it is also one of the causes for weight loss. He/she will also see if the discharge is treated appropriately or not. So see the doctor and ask what all investigations are necessary. Wish you all the best."
},
{
"id": 166828,
"tgt": "Can Piriton syrup be given to a child for nasal congestion?",
"src": "Patient: Hi my almost 1 year old (He s 1 in 2 weeks time) has had a severely congested nose, running constantly with bright yellow/green mucus for almost 6 weeks now. Had him to doctor about 5 weeks ago who prescribed antibiotics for upper respiratory infection (didn t really make much difference if I m honest), took him back to doctor yesterday who said he had double ear infection and prescribed more antibiotics for this. However, he said that he thought his nose congestion/mucus was most likely due to an allergy. I didn t think at the time but I m now wondering if I could try piriton syrup to see if it helps his nose if it is an allergy? I noticed it said not suitable for under 1 but since he s almost one and large for his age (29lb) I wondered if I could try him with some? Doctor: Hi,Don't give medicine that is not recommended for kids under 1 year. I agree with the doctor said it seems to be respiratory allergy. Any parents or brother have? If yes it is a one more point to be that. Besides that kind of drug will not help him. Try an Otolaryngologist doctor he could make more tests and help your child."
},
{
"id": 29822,
"tgt": "Suggest remedy for persistent diarrhea post treatment for sinus infection",
"src": "Patient: Good Morning Dr. Rynne, I have had chronic diarrhea for the past 4 days following a sinus infection that I took the antibiotic Ceftin. cannot keep food down and have been drinking propel water containing electrolytes which also just pours out of me not long after. what foods are best for this and what to avoid. thank you, S Doctor: HI, thanks for using healthcare magicAt the moment, since you have diarrhea, it would be best to avoid any solid food till there is a 4 hour window with no symptoms.The colon is still inflamed/irritated , this is the reason for the persisting diarrhea. If solid food is used then it would provoke more diarrhea and may delay healing since the intestines are not ready for it.Treatment: (1) holding off solids as mentioned, till there is a 4 hour window with no symptoms.When this occurs, start small amounts of dry food such as biscuits, toast , plain rice(2)continue the fluids (3)probiotics either on their own or with plain yogurt, would help to speed up healingI hope this helps"
},
{
"id": 45558,
"tgt": "Is weight gain normal post dialysis?",
"src": "Patient: i know a friend who gets dialysis. His weight before dialysis was 114.7 pounds. After dialysis it was 116.8 lbs. He does not understand with how much weight is considered normal to gain or loose after dialysis. Please explain. When does he need to contact his kidney doctor when it comes to weight gain or loss? Doctor: Hello, Possible causes like fluid over load must be ruled out. Dialysis per day does not cause weight gain. If symptoms persist, it is better to consult a physician and get evaluated. Hope I have answered your query. Let me know if I can assist you further. Take care Regards, Dr. Shinas Hussain, General & Family physician"
},
{
"id": 178169,
"tgt": "Can applying olive inside the baby s ear cause any problem?",
"src": "Patient: Hi,sir These is Sharmila and I have question about my baby.my baby she is 5 month old baby.My mother in law she put olive oil on my baby ear continue 4and half month everyday and now she is not here.After my mother in law left I stop to apply oil on my baby ear and my baby start to rubbing her ear every night.p/z sir provide me some guide lines which was help for my baby and last which my mother put oil my baby ear effect my baby ear or not.oil apply ear is good or not.its affect baby ear or not.I am wondering about my baby. Thanks sir for online. Doctor: hello, thanks for your query, as such it's a old concepts from old people , with out proof. there is no medicinal value in local application. better to stop.hope I have answered your query, all the best, take care."
},
{
"id": 104496,
"tgt": "Recovering from asthma. On steroids. Altered menstruation. Medication side effect?",
"src": "Patient: Hi dr.. I just had a whole 2 weeks of treating quite a bad of case of asthma .. And am just recovering now. I have been on steroids (pred) for more than a week. Yesterday I got my period, with a 9 day delay, which I don t usually have...but it s not bleeding normally, it s just spots with my period pain. I usually have a very heavy flow on my 2nd day, but this time it s literally a big spot. What could the cause of it be? I had 2 miscarriages before, and am concerned. Thank u so much . Doctor: Hello, Corticosteroids are known to upset the normal hormonal balance, as well as menstruation. Menstrual irregularities are quite common in people who take steroids, more so for those who have to take a big dose for flare ups of asthma, eczema, and other disorders. If the flow is very heavy, then tranexamic acid will control it. But usually the flow settles down, and the periods normalise by the next month or soon after. There is no need to worry about it. Best Wishes."
},
{
"id": 125839,
"tgt": "Suggest treatment for pain in the hip and thigh",
"src": "Patient: I ve developed a painful sensation in my right hip and thigh pain associated with walking and with entry into my car when stepping into the driver s side of the car with the right leg extended first into the car. I notice a muscle pain in the top of the right thigh when ascending stairs and some hip discomfort with movements of the right leg only. I thought initially it was a muscle pull from excessive walking exercise, but I m not sure now and am taking a week off of my walking routine. Doctor: Hello, Consult an orthopaedic and get evaluated. As a first line management, you can try analgesics like Aceclofenac or Tramadol for pain relief. Hope I have answered your query. Let me know if I can assist you further. Take care Regards, Dr Shinas Hussain, General & Family Physician"
},
{
"id": 39709,
"tgt": "What is the cause of yeast infection when on atrophy?",
"src": "Patient: I am 58 years old and had a hysterectomy 30 years ago. I weaned myself off estrogen 18 mos ago. My GYN doctor recently started me on a estrogen vaginal cream because of atrophy. I have had a bad yeast infection since I started using the cream. Is the cream the problem? Doctor: HI, thanks for using healthcare magicEstrogen vaginal creams can sometimes cause thick white discharge in some persons.Your doctor can consider providing your with oral or topical creams to help to treat it effectively.She may also consider a swab to send to the lab for analysis.I hope this helps"
},
{
"id": 146642,
"tgt": "What is the treatment for hematoma on the right side of the brain?",
"src": "Patient: Right now after doing ct scan,MRI and angiography i am diagnos having 10 by 10 cm hematoma at my right brain. Then my doctor said i need to do major operation at my right brain. But there s a risk that my left brain might be affected. Its not cleared to me right now what are the possible risk of my operation if ever i said yes to do the operation. And if i didnt want to do the operation what will happen to me? Please help me doing the right decision. Doctor: Surgical decision depends on size, site, duration of haematoma and cause of haematoma. Please mention your age and vascular risk factors. Also mention MRA findings. If there is underlying AVM or aneurysm it must be operated. Considering right brain involvement, which is a non dominant hemisphere neurodeficit will be minimalHope my advice will help you. Take care. Don't forget to rate me."
},
{
"id": 11116,
"tgt": "Can MX-5 lotion and KZ Lotion cause serious side effects?",
"src": "Patient: Hi doctor.I am 30 years old. I am facing hairloss problem. I have consulted a dermatologist and he prescribed me to use MX-5 lotion twice in a day and Foliglo tablets daily one and KZ Lotion/Shampoo twice in a week. I heard MX-5 lotion and KZ Lotion/Shampoo may cause serious side effects. Is it true? Also want to know about Foliglo. Doctor: Hi, MX 5 lotion (Minoxidil) is a local vasodilator, means it will increase blood flow to the area it is applied, but it is known to get systemically absorbed, so you have to be cautious while applying it, like do not rub it much on the scalp, wash your hands after the application and it will work like wonder for you.Few other side effects are slight burning and redness at the application area but they are minor once. and kz lotion (ketoconazole) is an antifungal agent. Don't worry about it, it doesn't have any serious side effects. You can use both of them very effectively.I hope this was helpful to you."
},
{
"id": 12045,
"tgt": "Black spots in between the legs in upper thigh",
"src": "Patient: Hi, i have black spots in the upper region of my thigh. Suddenly the skin in those areas of both sides, in between the legs turned black . I m 22 yrs old lady & still unmarried. so it is quite embarrassing fr me. I want to get rid of it. Can u pls suggest me any suitable treatment or product which I can apply. it wud b very wise of u if u can help. thank u Doctor: hi apply candid powder in morning and canesten cream at night.for 15 day. better to consult dermatologist. dont rub spot."
},
{
"id": 129199,
"tgt": "Suggest treatment for inability to speak clearly",
"src": "Patient: My mother is not able to speak clearly, nor can she move so easily, there is no myophagism except the little part between her thumb and forefinger, she had all kinds of examinations in China but nothing was found. She has been like this for 2.5 years, she used to be a normal person before this. She is 48 now. Doctor: Hello,Thank you for using Healthcare magic.I read your question and understood your concern.I do not know if your mother had Brain MRI or CT scan , and if not I would suggest you to do so , as these signs that you are telling might have the origine in the head ( bleeding or stroke or tumor).Wish you all the bestDr. Selmani"
},
{
"id": 201925,
"tgt": "Should I go for prolieve procedure for enlarged prostrate?",
"src": "Patient: I have an enlarged prostate and the Dr. wants to do a procedure called Prolieve, I have concerns about this procedure, am not really having any uncomfortable symptoms, but after a cystoscopy the Dr recommended I have this procedure. Some people are saying I should get a second opinion and that I should not have this done. Could have more serious problems after. Doctor: Hi,Thanks for writing in.The Prolieve is a medical device that both heats the prostate and dilates the prostatic urethra and treats prostate enlargement.During the procedure, the physician will insert a catheter into your urethra.When microwave energy is turned on, it will be transmitted from a special antenna inside the catheter to provide heat to your prostate.The heat will reduce the enlarged tissue of your prostate.A small balloon that is also part of the catheter will inflate within the section of the urethra close to where the prostate is located.During the entire 45-minute procedure, water will be circulating inside the catheter through your urethra.This is a new technology and you must discuss with your doctor before getting treated."
},
{
"id": 122342,
"tgt": "What causes constant static electric shocks on body?",
"src": "Patient: My problem is : If touch any thing or any one, some times I am attacked by current for 2 days. I am from India. Here there is winter runnng. Is it possible because of winter season or may be other types deseas? Plz reply me fast as possible.. Doctor: Hello, Consult a neurologist and get evaluated. Detailed evaluation is required to find out exact cause. A nerve conduction study and MRI spine has to be done. Hope I have answered your query. Let me know if I can assist you further. Take care Regards, Dr Shinas Hussain, General & Family Physician"
},
{
"id": 58624,
"tgt": "Taking Simvastatin for elevated cholesterol. Have diabetes type 2. Medication causing elevated liver enzymes?",
"src": "Patient: I was dx with elevated cholesterol and placed on simvastatin. About six months later my liver enzymes were and still are climbing, and I have been dx'ed with diabetes type two. Could this medication cause such sx? Along with these problems, i keep having urinary infections, dry and peeling skin that does not respond well to moisturizers. If I eat, my blood sugar goes up, but falls quickly and causes clammy sweats and nervousness. Anything I should do ? I went to the ER because of pain in mt back and anxiety like nervousness with rapid heartrate. They said it was because my serum potassium was low and prescribed 500 mg of pot,chloride bid. Last labwork one month ago shows my K levels normal, but liver enzymes still climbing. I don't know what to do because my Dr. Keeps saying he is going to research my problem, but I have not heard from him. Doctor: Hello,Statins like SIMVASTATIN is known to cause elevation of liver enzymes.It rarely causes problem to the patient or liver damage.Although you have not mentioned the levels of your liver nzymes,It is very rare that drug has to be withdrawn because of liver toxicity.Mild elevations are ignored but if the values are above 10 times of normal upper value or if patient develops symptoms,the drug is withdrawn.Either a different statin like pravastatin or fluvastatin is given or a entirely different drug belonging to other class like EZETIMIBE can be given.Other most important factor is the control of your diabetes.An uncontrolled diabetes can also result in high liver enzymes.Stop your SIMVASTATIN and see th response in next 1 month.If liver enzymes become normal,then we know the culprit and can think of starting EZETIMIBE.Keep control of your sugar level by medications,appropriate diet and regular aerobic exercises.I hope it helpsThanks"
},
{
"id": 184770,
"tgt": "What causes difficulty in speaking after having artificial teeth?",
"src": "Patient: I am 66-yr 0ld fit male, still working as a Director in an Educational Institution. Recently, I got three artificial teeth for better chewing and one artificial tooth in front to fill the small gap. I am 5 ft 3 in & last weighed about 72 kgs. Ihave no medical history. The problem I face is: For the recent few months, my speech is affected & proper words do not come to speak. However, my thinking is absolutely fine. Please tell the cause & the remedy. Doctor: Hello,Thank you for contacting Healthcare Magic. I need to confirm that you are calling a removable partial denture artificial teeth? Do you have a plate that fits on the roof of your mouth? New denture wearers have an adjustment period, but I do not expect this to continue for months. Common issues that affect speech include:-excess saliva-tongue movement-adjustments needed to teeth and biteYou will have excess saliva until you get used to something in your mouth. Your tongue has to get used to the plate covering the palate. If the tongue feels restricted, ask if an adjustment can be made to reduce the amount of material or thickness of material on the plate. If the position of the teeth affect your speech, the bite can be adjusted. This is often a simple reshaping to improve how your teeth meet or a slight reduction to the edge of teeth to shorten the length.Most often, time and practice will resolve your speech difficulties. Try an astringent mouthwash to reduce saliva. Return to your dentist for an adjustment if you do not feel your speech improve, but be patient. If you do not notice an improvement in a couple of weeks schedule a visit. Also, if you develop sore spots, schedule a visit for a simple adjustment. a less common occurance is an allergic reaction to the materials used.If you have a fixed bridge and continue with speech difficulties, you will need the dentist to adjust your bite. Call for an evaluation follow up visit.I am available to answer additional questions and hope you will not continue with speech difficulties after following my suggestions. I hope you find my reply helpful."
},
{
"id": 115376,
"tgt": "What causes low platelet count?",
"src": "Patient: In my blood test today, the platelet count is 133 instead of normal range 150-400. What does this mean and how can I improve it? The report also mentions anisocytosis. Can you please elaborate. Blood pressure is low like 90/60 for last couple of days. Doctor: Hi, thanks for writing to us sharing your health concerns!In the present case of mild thrombocytopenia, if I were your consulting physician, I would just say you, 'nothing to worry!'As per your description, you have platelets adequate for one's normal health but a count of 133 could be low to some hematologists and in some methods! Manual platelet count is necessary for confirmation. Anyways, there is nothing to worry about this as platelet count is so variable and might be reduced due to some flu-like conditions; once you are in good health within few hours/days the count will be normal.In case this is constantly low or decreasing then please write to us again.Anisocytosis relates to RBC morphology and some degree of anisocytosis is seen in normal health but when in severe degree, it might suggest some kind / cause of anemia. Please mention your Hb% level also!Low BP is treatable with good protein diets, high fluid intake and salt in foods/drinks...Hope you got your answer. Please feel free to write to us if any more queries.Wishing you the best health!"
},
{
"id": 89187,
"tgt": "What causes intense pain in lower abdomen?",
"src": "Patient: I'm a 30 year old female and I've been having intense pain in my lower abdomin which radiates to my upper left leg and my back I've had the pain for about a week and I'm due for my menstrual cycle soon but I've never had pain like this before what could it be Doctor: Hi,Such type of pain referring to thigh and back can be due to:Ovarian cyst or cancer growing very fast. Salpingitis severe PID.The best way is to get ultrasonography done and a gynecologist examination. If the cause is surgical , you need to undergo surgery.May not be related to menstrual period , ."
},
{
"id": 133367,
"tgt": "Suggest remedy for vascular dementia",
"src": "Patient: My Husband has been on prednisone for some years he suffers from vascular dementia poliysmyalga,COPD,anemia .His base is 15 mg.Lat week I had to go away so he went into care, and had his meds blister packed by the Phamacy.They didn t pack his prednisone, so when I got home I realised he hadn t had any for 3 days.My GP said to start them again at 20mg.he seems to be more ancouse than normal very tired I am quite concerned that this has happened Doctor: Hi,thank you for providing the brief history of your husband.As he is suffering from vascular dementia, i will advice to undergo a clinical neurological examination. May be an MRI will be required to understand the brain areas better.As medicine helps on a symptomatic relief and too much medication will not be good for the general health of the kidneys. Prednisone on a longer run also has some negative effects and the GP may not prescribe anymore.Also, in my clinical practice most patients with neurological illness, responds well to physical therapy along with medication. As with physical therapy exercises the metabolism is improved and so the vascular blood flow to the brain by improved cardio respiratory endurance.RegardsJay Indravadan Patel"
},
{
"id": 43510,
"tgt": "Suffering from infertility. Should i go for IVF?",
"src": "Patient: hi,i m 39 years with primary infertility with 13 years of married life..now i m diagnosed with left fallopian tube blockage by fibroid agglutination after HSG and diagnostic laproscopy ,,,cannulation donebut unsuccessful..no TB and carcinoma...right tube is normal..polypoid endometrium at anterior and posterior wall of uterus mainly on posterior wall....blood reports r normal n my husband reports r also normal......should i go for ivf??????i want ur good advice plzzzzzzzzzzzz Doctor: Hi welcome to Health care magic forum. Thanks for asking a question to H.C.M.F. You are 39 years old, case of infertility ,married since 13 years. You had left fallopian tube blockage by fibroid agglutination. confirmed by H.S.G., and diagnostic laproscopy. Polypoid endomatrium at ant, and posterior wall of uterus.Blood tests and your husbands reports are normal. right falopian tube is normal. As it is I.V.F. may not suit you due to endomatrium. as the right tube is patient, after treatment of endomatrium and your doctor says normal you can try normal way. I advise you to avoid sex in first 10 days, compulsorily have sex in 11th to 20th days of the cycle. have sex in female down position and stay lying for some time after sex. Wishing for a good luck. Best regards."
},
{
"id": 82703,
"tgt": "Suggest treatment for lupus",
"src": "Patient: Hi I have been running a low grade fever for a couple of months almost everyday. I have been tested for lupus, r.a, thyroid, infection and mono. All tests come back good. Due to go back and get a sonogram and check my hormone levels in a week. I m so frustrated! Really just want it to go away so I will feel like myself again. Does anyone have any ideas other than these? Thank u. Doctor: If every things are negative than it might be lupus flare, take steroid after discuss with your doctor"
},
{
"id": 130273,
"tgt": "What is the treatment for the marginal asteothytosis in D12 and L1 vertebral bodies?",
"src": "Patient: hi i am vikas . i am suffering back-pain from last 1 year and doctor tell me that marginal asteothytosis are seen in visualized vertebror mind enterrior wedging in D12 and L1 vertebral bodies . what are the solutions for this disease? Please advice me Thanks vikas Doctor: Hi Vikas,You have marginal osteophytosis with anterior wedging in D12-L1. The probable causes are osteoporosis,thyroid disease,spinal infection,trauma.It would be good if you rule out the causes to prevent trauma to other vertebrae.Depending upon the cause you would need to take medicine course .These are prescription medicines so it would be good if you visit a nearby orthopedic.Coming on to the treatment part ,depending upon the severity of the fracture(wedging) it will be decided if it can be conservatively treated or not.For conservative treatment following options are availablePhysiotherapy(includes rest,strengthening of limbs and abdominal,ergonomic advises)Lumbar braceAnd the operative procedures areVertebroplastyspinal fusionHope you get well soon and this helps youThank you"
},
{
"id": 219605,
"tgt": "What causes chest and throat tightness with breathing difficulties during a pregnancy?",
"src": "Patient: hi i am 5 weeks pregnant and for the last few days i have found it difficult to breath tightness in my chest and throat especially at night.i have had post nasal drip on and off for the last couple of mths and at first put it down to that but ive never had tightness in my chest before.this is my second pregnancy Doctor: Hi.Your 1st trimester can present with some really unexplained symptoms, but the breathlessness could possibly be in relation to the ribcage which moves upwards and outwards in order to accustom itself to the changes taking place and also to allow easier breathing during the later stages of pregnancy. This is absolutely normal and should not be a cause for concern.Best wishes."
},
{
"id": 155909,
"tgt": "What are chances of survival for stage 3 cancer patient after removal of lung?",
"src": "Patient: hello im inquiring about lung cancer and truthfully how long could a person live if they have lung cancer but has had surgery to remove left lung? well or should i ask whats the chances of the cancer already spreaded some where else but just not showing up yet.this person is in stage 3 also. Doctor: unfortunately chances are high, and lung cancer is one of the worse. you can hope for the best, but statistically, survival rate after surgerry in stage 4 is 20%. wish you good health. Regards"
},
{
"id": 86605,
"tgt": "Suggest treatment for severe abdominal pain",
"src": "Patient: For the past 3 days I have been having sharp pains in my upper and lower abdomen but mainly lower ( behind my belly button and off to the right of it and lower) and I ve been really gas ( feeling like I have to pass gas ) but when I pass gas it doesn t seem to help. And it hurt when I eat or don t eat. Each one makes it worse. Any suggestions of what to do or what it could be? Doctor: Dear thanks for the question. I share your concerns. There are several possibility in your case. It can be gastritis, hepatitis, appendicitis or stone in urinary system. If you have fever also, then I would go for appendicitis, as it requires urgent treatment. Gas related pain is second possibility, it may not always releave by gas and it may not have relation with food always. Hepatitis is in upper right side.I would advise you to visit a surgeon and under go ultrasound scan , as it will help in coming to a diagnosis or rule out some diagnosis. Blood work up is an additional important investigation. I hope I have clarified your queries. Please write for follow up queries. I wish you good health. Please provide a feedback. Thank you"
},
{
"id": 61854,
"tgt": "What causes a raised area around the depressed area due to a bite?",
"src": "Patient: i have a single bite (i think its an insect bite), on the top of my foot....about 1/2 inch in diameter... i see what looks like a single puncture mark, sitting on a raised area, surrounded by an annular depressed area, which is finally surrounded by an annular raised area.... Doctor: Hi,It seems that there might be having some local allergic reaction after insect bite.Take Benadryl or Cetrizine for 2-3 days.Apply Caladryl lotion on affected part.Ok and take care."
},
{
"id": 58616,
"tgt": "Have hepatitis C infection. Done liver function test. X-ray show endplate degenerative changes. Reason?",
"src": "Patient: i have hep c just got a liver function test showing that my ast sgot r 60 between 15-37 what is that ? also i got a xray taken of my lumbar spine due to hep c it states that three views of the spind have endplate degenerative changes . why is that cause of my hep c . last year it was in the 8 million but levels were find Doctor: HI, thanks for using healthcare magicHepatitis C is a viral infection involving the liver. It can resolve or progress to chronic infection of the liver.SGOT is one of the liver enzymes, it is also known as AST. There are 4 enzymes used to assess the liver: SGOT, alkaline phosphatase, GGT and SGPT (also known as ALT).These enzymes can indicate if the liver is not working normally though they are also found in other parts of the body.Your SGOT result is 60 and you quoted the lab range of 15 to 37, this means that it is above the upper limit of 37.You would have to look at your previous values to see if this is increasing, decreasing or remaining stable.In terms of the x ray- degenerative changes mean break down. It may refer to arthritis.It is not likely due to the hep c virus.YOu may want to discuss the result with your doctor.I hope this helps"
},
{
"id": 168487,
"tgt": "What causes reddish and inflamed penis in a child?",
"src": "Patient: Dear Doc, My son 2 years old ,when he woke up today morning, his penis was reddish and inflammed.I consulted a peadeatrician in the afternoon who said it might be an insect bite.He took the first dose of syrup and I also applied the ointment but the swelling has not gone . Could you please let me know what might be the reason for this reddishness and inflammation and is it something to be worried. Doctor: dear madam, sorry to hear about your son.as suggested by your doc,it could be insect bite.usually it takes 48 hours for any swelling to subside completely.no problem if there is no difficulty in urination. have a good day."
},
{
"id": 44977,
"tgt": "Can Evion 400 be taken by men ?",
"src": "Patient: how a male overcome his sexual weakness Doctor: Welcome to HCM. You can take evion 400 but other vitamins are also needed to overcome it. Take good healthy diet rich in protein.,fruit juices and salads . Spend some more time in foreplay.Don't concentrate your mind in sexual activity while doing it. Exercise,yoga and pranayam will be helpful. Consult your physician for better guidance."
},
{
"id": 89793,
"tgt": "What causes pain in lower abdomen with normal x-ray?",
"src": "Patient: I am having pain in my Lowe abs from the left side of my stomach extending it to my lower back. I went in today to see a doctor and he ordered an X-ray,according to him he did not see anything wrong with me but prescribed naproxen to reduce inflammation, I came home with no diagnosis but still in pain. How can you help me? Thanks Doctor: Because of the limitted information you provided, provisional diagnosis can not be made, if such patient com to my clinic i will askWhere did the pain start and where it goes thenIs there any association with foodHow much it is severe, is it aching dull or sharp painAre you feel naseated.Your bowel habits are normal?Then i would suggestUltrasound abdominFull blood countUrine analysis.It is better to consult general surgeon in your area."
},
{
"id": 3669,
"tgt": "Will straining after sex effect badly if i get pregnant?",
"src": "Patient: we had sex last week, and hence it is not confirmed, but they say we should not strain ourself for 1st 3 months, so if i strain myself in the 1st 2 weeks after sex and if i am going to be pregnant will it affect me from being pregnant, i mean will the baby get aborted? Doctor: Hello dear,i understand your concern.In my opinion straining after sex might not effect the conception.Almost most of the women dont know they are going to get pregnant and do their works normally till the pregnancy is confirmed.Once the pregnancy is confirmed one has to be careful like avoid physical strain,avoid lifting heavy weights,avoid intercourse etc.So heavy stress might lead to miscarriage during the initial weeks or months of preganncy but not before conception.Best regards...Dr.Srilatha"
},
{
"id": 22970,
"tgt": "What causes red patches on legs/face and racing heart rate after drinking?",
"src": "Patient: Hi I'm not a heavy drink maybe 2 beers or a glass a wine a week been drink for 10 years. I'm now 28 years (female) over the past few months whenever I drink I get red patches on my legs or face and my heart rate goes from 69 bpm to 95 and will stay like this for a while. Is it possible Ive become allergic my blood pressure ranges from 95/69 and 111/72. I'm not on any medications. Thanks Jane Doctor: It's physiological effect of alcohol, it's not allergy but heighten sensitive to the effect of alcohol, it causes blood vessels to dilate and your symptoms. So consume alcohol in less amount and effect probably wean after sometime."
},
{
"id": 172627,
"tgt": "Reason for child s ear lobe which is swollen?",
"src": "Patient: my baby s ear lobe is swollen. She sleeps mostly on that side, but not sure why it s swollen. The only thing I can think of us that I wiped her ears too much or too hard. It looks like there s a little bit of sticky stuff on her skin. She also has baby acne, anything i can do for that? Doctor: Hi... I sympathize with the kid and understand your concern. The trick of the management is keeping the skin moist and never allowing it to be dry. For this oil massage and lotions like Oilatum will help. Regarding cure options - eczema is a sort of allergy...allergies can only be controlled and can never be cured. But usually kids grow out of it...that is as the age progresses, they become better starting from 6 months.But, skin conditions are best diagnosed only after seeing directly. I suggest you to upload photographs of the same on this website, so that I can guide you scientifically. Hope my answer was helpful for you. I am happy to help any time. Further clarifications and consultations on Health care magic are welcome. If you do not have any clarifications, you can close the discussion and rate the answer. Wish your kid good health.Dr. Sumanth MBBS., DCH., DNB (Paed).,"
},
{
"id": 93902,
"tgt": "Severe abdominal pain after DNC for retained placenta. Hypothyroid, advised HIS for poss bicornuate uterus. Causes of pain ?",
"src": "Patient: i gave birth feb 26th 2012 at 35 weeks gestation. 4 weeks later i had a dnc for retained placenta . since giving birth, i have had sever abdominal pain that radiates from gallbladder area to appendix area to heart area. no known allerhies to foods, recently diagnosed after having abby with hypothyroidism and hosimotos, all cat scans, mri s, hida scans, bloodwork, and u/s came back ok. i was told today that i need to get a HIS for poss bicornuate uterus??? could this be the cause for all my abdominal pain that i am having. i also have yet to have a period . not breastfeeding . i did bleed at 13 weekd gestation and was told that i had a subchorionic hemmorage and a tilted uterus....any advise???? Doctor: Hi welcome to Health care magic forum. Thanks for choosing H.C.M.Forum. You had delevery on 26- 2- 2012. at 35 weeks gestation. 4weeks latter you had D.&.C. for retained placenta. You had bleeding at 13 weeks and told that you had sub chorionic hemorrhage, and tilted uterus. Doctors suspecting bicornuate uterus, and ordered for hysterosalpingogram. You had retained placenta due to some reason, and resulted in infection, which in turn would have caused pain. After D&C also infection is to be cured to get the pain relief. I advise you to consult a general surgeon to find out any other non gynacological cause. Wishing for a quick and complete recovery. Best regards."
},
{
"id": 199301,
"tgt": "What is the remedy for hard erection due to anxiety and premature ejaculation?",
"src": "Patient: Hi doctor. I have a very severe premature ejaculation. I went to Urologist and gave me zoloft and I had very bad side effect from it. I went back to him for the 2nd time and he gave me paxil which didn t do anything at all. I didn t go back to see the Urologist after that. I m having a problem now with having a hard erection which I think caused by the anxiety I have knowing that once she touch me ill ejaculate rite in there. It s kinda 2 problems now. Any advice from you please ?? George G Doctor: DearWe understand your concernsI went through your details. I can understand. You said you have severe premature ejaculation. Psychology based premature ejaculation can be treated with the help of awareness, psychotherapy and relaxing techniques. You should also know that, you can engage in sexual activities for a second time, after you experienced premature ejaculation. Go for the second time. You will be alright.If you require more of my help in this aspect, please use this URL. http://goo.gl/aYW2pR. Make sure that you include every minute details possible. Hope this answers your query. Available for further clarifications.Good luck."
},
{
"id": 172333,
"tgt": "Suggest alternative of isomil milk powder for an infant",
"src": "Patient: My baby is 1yr 5mts old when she was 2mts old we wr giving her ISOMIL MILK POWDER we tried to change her milk many times but she refuses to take anything besides ISOMIL now it is not available any were can u suggest a replacement. I would be very greatfull if i could find a solution. Thanking you SUNIL GOA Doctor: Hi Read ur question 1 yr 5 month old baby can take any thing u are give Isomil is powder milk , u try coe milk or baffelow milk instead of powder milk If any taken the use lectogen of Nestl\u00e9 company My advice to u that try direct milk instead off powder milk Any if not taken then add some sugar or health powder lik horlicks or bornvita"
},
{
"id": 223881,
"tgt": "What are the chances of getting pregnant after taking ipills?",
"src": "Patient: Hi , I was wondering am I most likely okay if I had sex with a condom, it slipped off but my partner didnt ejaculate in me. I also took the morning after pill the next morning. Also can the pill make you late, because i am a week late and am a very paranoid person and Im freaking out although my friends say I shouldnt be ! Doctor: Hi dear without ejaculation inside there are no chances of getting pregnant also you have taken I pill so no need to worry..if you missed your periods by 2 weeks then do the urinary pregnancy test,blood hCG levels and USG for final confirmation..that's my advise as more anxious you be it will late the periods..anxiety is one of the cause of delayed periods..stay strong and be relax..regards Dr.Sohil Takodara"
},
{
"id": 114869,
"tgt": "What does positive c-reactive protein blood test indicate?",
"src": "Patient: InvestigationResults 27.03.10Results 06/04/10Results 28/04/10Results 14/05/10Results 05/06/10Given valuesHb-13.114.714.114.913-18TLC12600106008300870096004000-11000Lymphocytes21.322.131.73322.525-50ESR102110454340RDW-11.613.9-16.711.6-14SGOT(AST) 20.4132.4717.5519.42SGPT(ALT)31.2348.5775.9425.8331.13S. TSH-2.0203.520--0.27-4.2S. Creatine---0.620.690.7-1.2S. Uric acid---13.4812.143.4-7S. Calcium----10.928.6-10.2C Reactive Protien----positive Titre----12 Dr Palash above are his blood test reports latest being on 05 jun.can u pls advice me on following doubtshis Lymphocytes has again fallen down, reasons? his ESR although has come down but insignificantly. reasons? his RDW has risen. reasons? his uric acid values are worrying me a lot. any suggestion to bring it down pls? reason for S calcium raised values? C Reactive Protien is positive. what it means? Titre is 12 what it means what overall u see ATT medicines are effective?his TB is under control in last 02 months or not yet?regardsrajni Doctor: Hello, First of all, lab tests without clinical information are practically useless. If the doctor cannot use clinical info to filter the differential diagnosis then the list of potential diagnoses becomes impractical to use, because of its huge size.I understand that tuberculosis is the main problem. Tuberculosis may cause any disorder in the complete blood count. The closet the values return to normal the better for the efficacy of the treatment. There is use in commenting on values like the lymphocytes or the Rdw. Perhaps if I knew the complete medical history, there could be a reason for that.The qualitative crp is also useless. A quantitave measurement would be much more useful. The esr is a better alternative for tuberculosis and it seems to get better. His last values are almost normal which is a very encouraging sign about the efficacy of the treatment. The uric acid values are probably related to pyrazinamide use. If he's excretion a lot of it in his urine then he should take drugs to lower uric acid.Regarding treatment efficacy although the tests seem to get better, These tests are not sufficient to judge it. Radiological and microbiological tests are more important, like a chest x-ray and sputum tests.I hope I've helped! Kind regards!"
},
{
"id": 97480,
"tgt": "Suggest ayurvedic treatment for psoriasis",
"src": "Patient: I am 27 years old, weight :60 kg. I never had any problems with my skin nor other diseases. I have psoriasis since 4 jears. In the begining I had it only on mij head but now I have it almost everywhere on my body. Every time it gets hot it gets worse. If I use cortici steroids it calms but doesnt get better.Is there any thing that I can do to make it go away. Perhaps something ayurvedic Doctor: Hi Welcome to HCMThis is Dr Suchda Naturopath - Homeopath - MagnetoTherapist taking your query to help you . Psoriasis is an obstinate spoilt condition of skin . as you are suffering since 4 years ,I need not go into details of ,how it happens and all I want to know if scab formation is there ?I would suggest you to use Neem or Bakayan ( is tree like Neem )Decotion Prepared with 4 cups of water and 25-30 leaves . Boil till 1/4 of water remains . Apply on affected areas , daily. keep till 2 hours . Wash it with multani mitti . Don't use any soaps and drink same water 2 table spoons 3 times a day . Apply neem oil if dryness there . continue till 1 month . Take Homeopathic Radium Brom 200 1 dose fortnightly .3 times Last but not the lease ,I must say make nature your friend .It will save you all untoward diseases and now you will recover fast .We have power within our self . we have to explore and use it for our benefit .Adopt exercise ,Yoga & Pranayam -Deep breathing exercises which give us resistance to our body by burning toxins and strengthen us against diseasesDo this for 30 minutes . and feel the difference . It will surely help your recover fast .Diet also need proper consideration . Have Balanced diet -more of fiber than fine fast foods , veges , frut ,soups etc, which helps keep constipation at bay the root cause of most of the illnessesTake care All The bestDo come back if have any further query ."
},
{
"id": 74856,
"tgt": "Could the episodes of cold followed by muscle ache be Fybromyalgia?",
"src": "Patient: I'm pretty sure I have fybromyalgia, my doc. has not diagnosed it as such but I have talked to him about it. I try not to consume certain foods or drinks, so I think my case is pretty mild now. However, when I get cold (cold feet or hands - get out of shower and leave hair wet for a while) I get cold and then all of my muscles start to ache. Could this be part of fybromyalgia? The only way to feel better for me, is to get a heating pad any move it around different muscles until I'm warmed again. Doctor: Respected user, hi I evaluated your query thoroughly.* Fibromyalgia is characterised by chronic wide spread pain & heightened pain response to pressure , with other symptoms as feeling tired, affecting normal activities , sleep problems and troubles with memory also in many cases.* Diagnosis is usually after exclusion of other causes.* Many more information are not available here in your briefing , but till the information you have provided I don`t agree it to be fibromyalgia.Hope this clears your query.Thanks for using Health care magic & giving me an opportunity to assist youWelcome for any further queries.Regards dear take care."
},
{
"id": 89137,
"tgt": "What causes severe pain in abdomen?",
"src": "Patient: Hi, my stomach has been painful since this morning. I felt hungry this morning first, because of the pain, then I ate some food, but the pain continues. Then I felt full. I took the medicine called soothe compare to pepto-bismol. 2 doses, but didn t seem help. Doctor: HI.Your history is saying you had pain in the morning and you got hungry due to it.This suggests the possibility of an Acid Peptic Disease, which can be gastritis or duodenitis or so. The non-response to the food and the medicines you took indicate a possibility of a Gastric or Duodenal ulcer or pancreatitis or cholecystitis.I would advise you to be nil orally- meaning do not take anything orally as you need to undergo ultrasonography of the abdomen, Upper GI Endoscopy and other blood examination including Liver functions and Lipase , Amylase enzymes, sugar, kidney functions and so on. Get admitted in the hospital."
},
{
"id": 29776,
"tgt": "What causes severe chest pain from persistent dry cough?",
"src": "Patient: I have been coughing heavily for a week now. Went to the doctor and was prescribed Antibiotics, prednisone and an inhaler after 2 breathing treatments in the office. Although I feel a little better, My lungs (chest) hurt and I still cough without much production. The coughing is like I can t catch my breath.. I was told it was acute bronchitis but the pain when I cough seems to be getting worse not better. Doctor: Hi, I have gone through your question and understand your concerns. Chest pain after respiratory tract infection is usually due to pleurisy (inflammation of pleura) , you can use anti inflammatory and pain killers ( dicloran)along with other medication you are taking. Hope I have answered your question ,if you have any other query feel free to ask.Wish you good health."
},
{
"id": 193146,
"tgt": "Suggest treatment for blood in urine after masturbation",
"src": "Patient: Sir , three months back I had blood in my urine (ONLY ONCE ) , it was totally painless . I had it in the morning after mastuburating . Throughout the day I drank a lot of water . At that time I used to take a lot tension . After that day I never had blood in my urine , it never repeated . At present I drink a lot of water . Can you please tell me what was the cause behind it . I drink tonnes of water , but recently in the net I read drinking too many litres of water can cause water intoxication . What should be the exact dosage of water every day ? Doctor: Hi, The blood in your urine can be due to renal calculi, prostate infection and urinary tract infection. Test for those conditions and take respective antibiotics. You can take approximately 3 liters of water per day, there will be no intoxication in this dosage.Hope I have answered your query. Let me know if I can assist you further. Regards, Dr. S. R. Raveendran, Sexologist"
},
{
"id": 39929,
"tgt": "How to treat the cough related to a chest infection?",
"src": "Patient: i have had a chest infection for the last 3 weeks i have just finished a 7 day course of anitbiotics but i am still coughing upgreen gunk . the only prob is now all i can taste is salt . my saliva is salt the gunk im coughing up is salt , everything i eat . its driving me mad Doctor: Dear FriendWelcome to HCM. I am Dr Anshul Varshney. I have read your query in detail. I understand your concern.I would advise you to first investigate the cough.You should get the following as you have cough from last 3 weeks:1. Xray Chest.2. Hemogram with ESR3. Sputum for AFB.4. Sputum for Gram stainThis is my best advise for you based on available details. If you have any further query, please ask me.Stay Healthy."
},
{
"id": 165663,
"tgt": "How serious is ear infection and kidney infection in a 2 year old?",
"src": "Patient: hi Dr,my 2yr and 8 months old baby girl was suffering from high temp and continuous vomiting few days ago.we consulted a Dr and he told she is having ear infection ( dark reddening deep inside the ear) and kidney infection.when he touched he back side,she yelled in pain.she is having head ache also.today morn her leg also aching.antibiotic treatment started.i want to know how severe is this.is it completely curable.she is 12.5kg .but born as premature weighing only 2 kg.she had 3 times malaria since her first Bday.and more than 5 times sorethroat.please give me ur valuable answer. Doctor: Good afternoon..!!Ear and kidney infections occurring simultaneously could mean that your 2 year 8 month old daughter has low immunity levels indicated by frequent sore throats and malaria three times.Yes, they are completely curable with proper treatment. Nothing to worry much.So if the problem is recurring, it's better to visit pediatric ENT surgeon and pediatric nephrologist for further evaluation and management.Thank you, have a nice day..!!!"
},
{
"id": 65283,
"tgt": "How to treat an injection site swelling?",
"src": "Patient: I had a flu Jab last October, after one week I had swalling under my left hand where I took the flu jab. after two to three months this swalling of the glands come and go then progress to another two places under the right hand and under the neck. I did blood test and they were fine but the swalling still. is their any treatment. Doctor: Thanks for your question on HCM. The most common cause for injection site swelling is infection and inflammation of vein (thrombophlebitis). If not properly treated, this can spread to surrounding skin tissue and form large swelling type lesion. The treatment of this is as follows. 1. Apply ice packs on the swelling. 2. Start anti inflammatory and painkiller drugs. 3. Keep that hand up for reduction of swelling. 4. Apply thrombophobe cream on swelling. It usually takes 4-5 days for resolution."
},
{
"id": 146511,
"tgt": "What does the following CT scan report mean?",
"src": "Patient: I had a CT of the brain without contrast. The findings: NO infarct, hemorrhage, mass effect, extra-axial fluid collection or white matter abnormality is noted. I need to know if this normal. I have lupus, I fall a lot. My head feels like it squishes , I can hear it. Doctor: The reporting finds no abnormalities in your brain, it is a normal report. More subtle changes could be found with an MRI, but even if that was the case they shouldn't be causing your falls."
},
{
"id": 114941,
"tgt": "Can low hemoglobin level lead to blur vision and dizziness?",
"src": "Patient: in hospital 5 weeks ago hymaglobin down to 5.4 had 2 bags of blood and iron injections for 2wks. now on iron tabs bloods taken every 2 weeks. esr 22, hymaglobin up to 12, feel better than i did b4 i went into hospital, but feel dizzy and pain in temporal area, blurred vision, pain in right hip have to lift my leg to get in car because of pain. blood pressure ok, but pulse 110. pain in left upper arm and back of head and neck. fed up with my GP he offered me antidepressants, said i will feel better, but i refused the antidepressants. what is wrong with me. Doctor: low hemoglobin levels can cause blurred vision and diziness . But now as your Hb has increased to 12.0 this should no longer be the cause. Please get a CT done ."
},
{
"id": 221569,
"tgt": "Suggest treatment for elevated uric acid levels during pregnancy",
"src": "Patient: hi doc, my wife is pregnant now & at hospital.. she had HPT & uric acid level was 374. fetal weight 1.2 kg. the doc said he want to monitor the fetus until 1.5 kg. always said its dangerous for the fetus for raised of uric acid. if it is dangerous why keeping it & not deliver early.? i worry both wife n the baby. hope u can help us...tq Doctor: HiDr. Purushottam welcomes you to HCM virtual clinic!Thanks for consulting at my virtual clinic. I have carefully gone through your case, and I think I have understood your concern. I will try to address your medical concerns and would suggest you the best of the available treatment options.1] First of all do not panic.2] It is not clear from your query , what is the duration of pregnancy and unit of Uric acid.3] Usually uric acid is expected to be less than 3.5 mg%in pregnant women.4] History of high blood pressure and rise in uric acid is indicator of obstructions in the blood flow to baby.5] If baby is less than 34 weeks , mother is given Injection BETNESOL 12mg 2 shots 12 hours apart; to help get baby's lung maturity.6] If USG shows less amniotic fluid around baby, obstruction to blood flow of uterus, umbilical cord or baby's brain; then decision to deliver the baby needs to be taken.7] In good neonatal care units baby weighing above 1 kg stands good chances of recovery.Remember the treating doctor always tries to give viable outcome without compromising mother's health.Put full faith in your treating doctor.Rest will be all right.May god bless you with Bundle of JOY.I hope my answer helps you.Thanks.Wish you great health."
},
{
"id": 38761,
"tgt": "What is the oral alternative of amikacin injection?",
"src": "Patient: good day doctor, I am 29 yr old female.I am suffering from a UTI which is a reoccurence after5 months. the symptoms are frequent urge to urinate, pain while urinating,little vomiting sensation.Four months ago i got a urine culture & a vaginal swab test done.while the urine culture didn't show any bacterial infection swab test did come up with E-Coli infection. was adivsed by the consulting doctor to take Amikacin for 5 days but due to certain conditions i was unable to take it.at present i am on board a ship in vietnam as my husband is marine engineer.so i am unable to take amikacin at this moment as well as it comes only in the form of injection. Plz advise alternate medication which can be taken orally.plz advise more than one trade name .Also plz advise if URISPAS will be helpful or not?Thank you.KIM Doctor: Hello I hope you are not pregnant now. After observing your test reports , I suspect you were suffering with uncomplicated Urinary tract infections .if you are my patient I would advice you to take Macrobid 100mg twice daily for 7daysMacrodantin100mg 4times foe 7daysFuradantin 100mg 4times for 7days. Also use urispas tablet for 7days twice daily.it will relieve the spasmic pain during voiding.I hope you are satisfied with my clarification. Wish you good health. Rate my answer."
},
{
"id": 43196,
"tgt": "Pelvic ultrasound shows PCOS. Taking Ovamit while trying to conceive. Want better medicine",
"src": "Patient: i am 27 years old now.. recently i had undergone a pelvic ultrasound and the doctor diagnosed me that i have PCOS. Me and my husband are trying to have a baby now.. the OB give me an OVAMIT.. i had take it once.. ahmm, actually we already have one child.. a baby girl.. and shes already 4 years old now.. buy that long years we dont have any baby yet.. even a sign of pregnancy.. i dont even taking pills or any family planning.. my hubby and i are usually do the natural method... not even wasting any sperm cells just to be get pregnant... for that year long we really miss to have a little boy... pls help us...pls give me any medicine that would be work better than OVAMIT... i am begging... plsssss.... best regards Doctor: Hello, Thanks for the query to H.C.M. Forum. Ovamit ( clomifene ) is used in anovulatory infertility . As you are having pcos and already there is one daughter so you are a patient of secondary infertility . Causes of infertility may be . 1 anovulation , diagnosis can be confirmed by study of follicles and also study of dominant follicle. 2 uterus is unable to receive fertilized ovum . Diagnosis can be confirmed by study of endometrium thickness , if this thickness is less then 10 mm , no chance of pregnancy.3 Hormonal imbalance may be the another reason of secondary infertility . Diagnosis can be confirmed by blood examination of estrogen , androgen , F S H / L H ratio. In my opinion first of all consult a gynecologist and after all above mentioned measures , deal accordingly . Good luck. Dr. HET"
},
{
"id": 206545,
"tgt": "Suggest treatment for tokophobia",
"src": "Patient: he is 40 years has no problem about money and yet scared to get me pregnant I know he is a shy type. He loves me so much and font want me to break up with him too.we have been dating for 3 years. I asked his brother if there is a reason but he assured me there is none. What can I do please Doctor: Hello Tokophobia is fear of getting pregnant in females or fear of childbirth in males. It is seen in a lot of urban couples due to occupational pressure. You have mentioned that there is no money problem. He don't want to get a child and this is causing problem in your personal life.I would advise both of you to consult a Psychiatrist or a marital counsellor for expert management. Proper marital counselling will tend to increase his confidence. If condition doesn't improve the there is need to take medicines. Anti-anxiety medicines will help in early improvement.Thanks, hope this helps you"
},
{
"id": 153279,
"tgt": "Suggest prognosis of invasive squamous cell carcinoma with wide excision",
"src": "Patient: A biopsy for a bump in the middle of my back came back invasive squamous cell carcinoma. A wide excision is scheduled for 12/9. My question: Should the fact that it's \"invasive\" affect the removal procedure? Dr is proposing a wide excision. Am wondering if this should be addressed using MOHS Surgery? How exactly does \"invasive\" affect prognosis? Doctor: Hi,Thanks for writing in,An invasive tumor is one which has infiltrated to surrounding tissues. Such a tumor requires wide excision and a margin of healthy tissue is also removed at surgery. This will ensure that the margins are tumor free and there is less scope fr micro invasion of the cancer.From the prognosis point of view, this might be a slightly advanced or aggressive cancer and requires a planned treatment and management. You should be completing dedicated treatment and follow up with the doctor regularly. Mohs surgery might be discussed with your doctor depending on the extent of the cancer. Please do not worry."
},
{
"id": 111052,
"tgt": "Suggest treatment for severe back pain",
"src": "Patient: I am 30 year old Female, weighing around 68 kg with a height of 163cm and currently working in an IT company . I am married and have a 3 year old daughter. My problem is back pain. I am suffering from back pain almost 10 years back. I had taken medicines which will be effective till the period i consume later it persists. recently my daughter while playing in my bed fall on by back ( opposite side of by stomach) and i was suffering from a heavy back .took medicnes for 3 weeks which was a relief. Later after 1 week the same pain is repeating . I feel more painful while doing house hold activities and while sitting and sleeping . I am unable to lie in the bed facing by back as it pains me. Ploease suggest me what to be done Doctor: Hi...I can sense how problematic this pain is to you.As you have a long history of back pain of about 10 years, I will advise you to strengthen your back muscles first. Pain killers can relieve the pain only for the time being.you are a working women, must have busy schedule. Excessive work, faulty posture all can contribute to the chronicity of pain. Do some muscle strengthening exercises . initially start with the lighter ones. You did not mention about any test. I will asvise you to go for consultation to an orthopaedician and have some test to rule out any internal problem.for immediate relief, you can take analgesics like ibuprofen pr other non steroidal anti inflammatory drugs.Get well soon"
},
{
"id": 216645,
"tgt": "What causes the swelling and pain in the scar?",
"src": "Patient: Good evening. I have an old lateral scar (approx 5 inchces which required stiches back then but we never got them) on my upper thigh, which is about 20 years old. While getting dressed tonight I noticed that 2/3 of it was swollen and raised off of my leg. it s not too sore. I was wearing jeans and was mowing the lawn for three hours tonight. Thoughts??? Doctor: hi,thank-you for providing the brief history of you As you have a chronic scar which is getting bigger and bulging I feel you should try out physical therapy.As when the scar becomes chronic there leads to weakness of the muscle and the accumulation of the fluid in the area. Regaining the muscle strength will help reduce the swelling appearance and also taking therapeutic ultrasound therapy will help losen the scar as well.In my clinical practice most scar are treated with Therapeutic ultrasound therapy and exercises which helps reduce the symptoms.Regards Jay Indravadan Patel"
},
{
"id": 147428,
"tgt": "Suggest treatment for hot sensation in the head and knee pain",
"src": "Patient: i want my body feels like heat not fever specialy back of my head and slight faint like doctors said that you have no problem but weakness but its long time now from last 1 and half month this problem is going on the body is not feeling fresh joint specially both knees are paining. Doctor: HIThank for asking to HCMI really appreciate your concern, this is a functional complain and some psychological elements plays the role behind this, you need to stop thinking about the bizarre ideas, in such cases I initially try \"Tranquilizer\" and this gives excellent result, but important is you need to keep your stress level low, and the best option is \" Tab Chlordiazepoxide three time in day\" hope this information helps you, have a nice day."
},
{
"id": 201056,
"tgt": "What causes wrinkles on foreskin of penis after applying tea tree oil?",
"src": "Patient: Doctor, I had this genital looking warts on my penis and I applied 100% tea tree oil at night for over a week. Lately, I have been realizing that my penis looks wrinkly on the left side where I had put on the tea tree oil. However, my penis seems to look normal when it is erected. Is there anything I can do about it so that the wrinkle is gone even when I am not erected? Thx Doctor: Thanks for asking in healthcaremagic forumIn short: Allergy to your favourite tea tree oil might have caused this.Explanation: Do not experiment with these oils and other chemicals for your wart. Go and visit a dermatologist and get your wart removed if present. Usually the skin of penis is wrinkled when it is flaccid and little stretched when erect. This is normal in every male. Stop seeing your penis and visit your doctor for the solution. Stop putting your oil till then. Good luck"
},
{
"id": 182583,
"tgt": "Suggest treatment for jaw pain after wisdom tooth extraction",
"src": "Patient: I had my wisdom teeth extracted 4 yrs ago this September and in the past couple of days I have experienced excruciating pain in the lower left side of my mouth behind my back molar where one of my wisdom teeth were.. The pain radiates through to up and under my jaw. It hurts if anything touches it. It hurts to breathe through my mouth, eat and drink. I have no insurance and no money to go see anyone to get an opinion or pay for diagnostic treatment so if i can get an idea that would be great! Thank you very much for your time. Doctor: Hi, I have gone through your query, as u complaints of pain in the extracted region,either it may be due to inflammation in that region or due to incomplete extraction of wisdom tooth.You should visit to dental clinic for Consultation. Thank you - Dr.Bhuwanesh Sharma BDS, MIDA (Oral and Dental Surgeon)"
},
{
"id": 218947,
"tgt": "How to confirm if delay in periods is caused due to pregnancy?",
"src": "Patient: hello I just stop having a period bout 2 months ago I have had my period every month my whoel life and I never missed one accept when I got pregentt 10 years ago this is freaking me out I recently got married and my husband does not have any kids I am worried.. I wonder will I ever get pregnett we have been trying over a year now butt I m wonder watt happend before my period stop 2 months ago the month prior before that my period last for a month then it completely stop Doctor: Hi, I think you should do a urine pregnancy test to rule out pregnancy first. If negative, take some medicines like meprate or deviry for inducing periods. Also do some blood tests like thyroid profile and prolactin level and a ultrasound done to see uterus and ovaries. Also do a AMH level to see the number of Antral follicles in ovaries. If everything is fine, it can happen due to stress factor sometimes. You can try naturally for 6 months. Be in contact with your husband every 2 to 3 days after your periods stop. It will increase chance of pregnancy. Hope it helps."
},
{
"id": 111208,
"tgt": "What causes frequent urination and lower back pain?",
"src": "Patient: I am experiencing frequent urination. I have checked with my a specialist but everything seemed perfect. I suspect it has something to do with my lower back ache. Or could it be due to bacteria? or does it soemthing to do with my lower back ache? Tq. my email: YYYY@YYYY Doctor: Hello,I had gone through the case and found that there might be many cause like urinary tract infection, diabetes or any pressure on bladder.So go for ultrasound of lower abdomen, blood sugar and urine culture.After that take proper treatment.Hope my answer will be effective for you.Thanks"
},
{
"id": 112357,
"tgt": "Left side back pain, problems with eating, weakness, nausea. Taking tylenol. Suggestions?",
"src": "Patient: I have a left side back pain, having trouble sitting straight, also having problem with eating and if I touch below my rib cage in the front and the back pain seem to be at the same point. Feeling weak and nausea, especially when eating, back pain is strong, does not go away with tylenol.What could it be assoicated to? What should I do? Doctor: Hi, Thanks for your question at HCMI understood your problem and from the ways symptoms are described there are two things that bothers you1. back pain2. pain below the rib cage in the front.Firstly, I think you are having gastrits which can cause nausea and also pain when you press below your rib cage in the front.Secondly, back pain on the left side can be caused by either wrong position while sleeping or sitting or could be a cause of any problem in your neck bones. to know exactly what is the reason for the back pain better show to orthopedic surgeon. For your gastritis you could take Pantodac 40 mg tab, 1 tab 1 hr before food on empty stomach and avoid taking spicy foods.Hope this answers your question"
},
{
"id": 57555,
"tgt": "Could pain in upper stomach be due to gall stones and what is the remedy?",
"src": "Patient: I have been diagnosed of having gallstgones about a tear ago but no medicine is prescribed for it. now my upper stomach is always aching and i suspected it was my gallbstone that is attacking me, Do u think im right with my suspicion. if it is what is the best pain remedy? Doctor: Dear Friend.Welcome to HCM. Yes Gall Bladder Stone can cause pain in abdomen. It can also cause Cholecystitis causing pain.However a pain in upper abdomen can also be due to acid peptic disease.I would advice you to get ultrasound of abdomen to see for Gall Bladder and stone status.If there is a problem a surgical removal of gall bladder might be required.If you have any further query please ask me.Stay Healthy."
},
{
"id": 179922,
"tgt": "How can a raised itchy lump caused by a bite be treated?",
"src": "Patient: Returned home from Mallorca two weeks ago. My 6 year old has a bite that had just appeared, it is a perfect red circle with a raised itchy bump in the centre. Could this be a reaction to mosquito bite after two weeks? Or something else. Does not looked infected. I am a nurse. Doctor: Thanks for posting your query at HealthCareMagic. It could be a mosquito bite but unless a doctor can directly see for himself, it is hard for him to comment whether it is a tick bite or something else. If he does not have fever or associated symptoms you may sit back and watch if it goes away. If not, consult a paediatrician.I hope that helps. Feel free to revert back in case of further queries."
},
{
"id": 16208,
"tgt": "Non-itchy rash on inner part of left leg, red discoloration, raised. Treatment?",
"src": "Patient: I have a non itchy rash on the inner part of my left leg. It does not itch, and it is spoltchy, but not raised. It s red and purplish. It comes and goes. It seems to come on with a hot/warm shower or by the sun. The day it first always appears, the rash is hot. I am having it biopsied tomorrow, but any thoughts as to what it might be? This has been going on for months! Thanks Doctor: MAY BE FUNGAL OR VIRAL IT INCREASES IN HUMIDITY AND HOT IT IS BETTER TO GET IT CHECKED TO DIAGNOSE AND START FOR PARTICULAR TREATMENTS"
},
{
"id": 204558,
"tgt": "Can Prozac be taken while on pain medication?",
"src": "Patient: I have terrible lower back pain. I took 1/3 of 10 mg. norco at noon today and another 1/3 at 5pm. Had 3 beers, and developed acute respiratory stress over the last 4 1/2 hours. While it s getting easier to breathe, should I take a prozac? Thank you. Doctor: Hello and Welcome to \u2018Ask A Doctor\u2019 service.I have reviewed your query and here is my advice.No, don't take any drug combination without doctor\u2019s advice. Prozac i.e., Fluoxetine oral will increase the level of serotonin and combination will increase the risk of serotonin syndrome. Whenever you visit take all your prescription to the doctor and then take advice. Hope I have answered your query. Let me know if I can assist you further.Regards,Dr. Rohit Kothari"
},
{
"id": 41608,
"tgt": "Why I am not getting pregnant with unprotected intercourse?",
"src": "Patient: Hiya, i am 23 and have been having unprotected sex since i was 19. I was in a long term relationship at the time and we were young and wanted a family. Now, I am greatful i didn't get pregnant then as i was still very much a child but, now 4 years later and i have still not gotten pregnant. At the beginning of the year, I was 3 & 1/2 weeks late, and thn when it did come, it was very very light and went for about9-10 days(instead of my usual 3-5). I was scared and didnt go to a doctor until a month later. Test was negative by then. Doctor said it was a spontaneous miscarriage that we'd completely missed it, but im not sure what the truth is bcos another dr. told me i was fine and it was just an irregular period. Now I am thinking i may be infertile as 4 yrs unprotected sex and still not pregnant. Can you help? Doctor: Hi welcome to healthcaremagic.I have gone through your question.As you are not sure if miscarriage or its delayed period, and not conceiving since 4 years then i would suggest some tests to get done for both of you.For you 1 ovulation profile to see whether follicles are developing it not.2 thyroid profile to rule out hypothyroidism3 histosalpingography HsG to rule out tubal blockageFor your husband semen analysis.After these consult a gynecologist if anything is abnormal.Hope i answered your question.Would be happy to help you further.Take care."
},
{
"id": 8673,
"tgt": "Smoking addict, scar seems to be red even after plastic surgery. Does smoking hamper healing?",
"src": "Patient: hello doctor thanks for help...... i had a plastic surgery for my scar 7 months ago and my scar is still red is it normal ? and isit true that smoking stop scar from healing or it just delay ....... i didnt stop smoking before or after surgery and if it stops healing would my scar heal if i quit the scar is on my face thanks alot Doctor: Smoking is a well known cause of delayed healing. It generally affects healing in the immediate post operative period as smoking leads to decreased oxygen supply to the wound by increased carbon dioxide levels and constriction of blood vessels leads to decreased blood supply to the wound. Apart from that various other chemicals in the smoke cause cell damage hence affects healing. Any scar after either operation or trauma takes at least 9 months to one year to completely mature up. In your case the redness of scar after 7 months of operation may be due to many factors, most common of them is that it may be due to scar is still immature and active healing process is still taking place but at abnormally high pace hence it is still red due to increased healing activity. Smoking delays the maturing of scars. hence I suggest that you should stop smoking, at least till the scar become mature, whitish and fades away. If the scar starts becoming thick and itchy it means abnormal healing is going on and must consult your operating surgeon. Meanwhile you should massage three to four time per day the scar by cold cream. Dr Rajesh Kaushash Cosmetic & Plastic Surgeon"
},
{
"id": 154770,
"tgt": "Can spots in kidney & ovary suggest cancer?",
"src": "Patient: had ct scan they found spot on left kiddney , left overy and multipule on my appendix now my left side hurts alot getting worse, cancer is really bad on my fathers side, its been a week since i went for another scan where they put dye in the intervenous should i be worried? Doctor: Hi,Thanks for writing in.There can be many spots that can be seen in kidneys, ovary and appendix. Most of them are from benign causes and only some have a malignant cause. Without knowing the details of the spots with respect to size, appearance and location, it is difficult to confirm the health concern. This is the reason you had to get another scan done with contrast dye injected through intravenous route and this will help delineate the characteristic features of the spots under evaluation. It will be better if you can include details of your symptoms and its duration. Findings of ultrasound scan and CT scan are also important when discussing the probable conditions. Please also include a noted on the cancer type prevalent in your family. Please do not worry."
},
{
"id": 223762,
"tgt": "Does consumption of Lo Estrin Fe cause irregularity/reduced flow in periods?",
"src": "Patient: Hi. I am on my third month of birth control pills. I was prescribed Lo Estrin Fe and have been taking them since February. I started the pack right when I got them, after my cycle. That month I had a period, however it was in the third week of the pills. My doctor suggested that was normal, and that my next period should come around the end of the pack- near the white and brown pills. This month, it did not. I had very little cramping, and a tad bit of blood, but nothing like I am used to. Also, I had a gelatinous brown substance that I saw when using the restroom. Should I be worried? Doctor: Hello,I have gone through your query and understood the concern. It is common to observe menstrual irregularities during the first three cycles after the initiation of the contraception. This is because the body takes time to adjust to the altered hormone milieu. If the breakthrough bleeding is persistent or heavy, please contact your health care provider for further management. Also, you need to use additional protection like condom or spermicide during the initial couple of months after the pill. If not, you carry a higher risk of failure. I would ask you to get a proper clinical appraisal and also exclude pregnancy. Take care."
},
{
"id": 214439,
"tgt": "Suggest natural remedy for frothy, green mucus in stools in an infant",
"src": "Patient: hi sir, My daughter[ 5 months old] is having green frothy mucus stool...and she poops after each feeding since a week ..she also strains a lot ...got her stool report it say mucus present ... pus cell5-10 hpf + in mucus and reducing substance in trace.. occult blood trace... plz help me with what i sould do as i dont believe in giving medicine.. thank you. Doctor: HIWell come to HCMI really appreciate your concern, the report given here is suggestive of Dysentery, and this can be treated with Syrup Metronidazole according to weight twice I day, for five days, Lactobacillus lyophilized powder can be used, hope this helps."
},
{
"id": 41294,
"tgt": "How can asthenozoospermia be treated?",
"src": "Patient: Hi I am 31yrs old and am a bit worried as my wife and I have been trying for a baby..hmmm, I would say for the last 6months with no joy of conceiving. I asked my doctor to allow me to go for a semen analys last year november - 11,2009. The following is a summary of the analys: Days of Abstinence: 2.5 weeks Liquefaction time: 10 mins volume: 2.8ml Viscousity: Normal Motility: 18% Progression: good/fair colour: Normal Morphology (%) Normals: 23 Headpeice defects: 58 Mid-piece defects: 18 Tail defects: 0 Immature forms/cells: 1 Comments: Asthenozoospermia (reduced sperm motility) however, note long abstinence period. I would appreciate if you could advice me on the posible problems and what can be done to improve my sperm motility so my wife and I can stand a chance to have a baby and raise a family. Thank you Doctor: Hello Your semen analysis findings may indicate decreased sperm motility.Motility should be at least 55 %.Your findings suggests over all 18% motility,so it is low.Sperm morphology is normal.Only 16 % sperms should have normal morphology for natural fertilization.You should do exercise and take nutritious diet.You can also take anti-oxidants.You should avoid alcohol and smoking if you take these things.You also need clinical correlation and few investigations like random blood sugar,ultrasound of scrotum.If findings persists then you may need assisted fertilization technique like IUI.Get well soon. Take Care Dr.Indu Bhushan"
},
{
"id": 160623,
"tgt": "Suggest treatment for collar bone fracture in a child",
"src": "Patient: My daughter (4.5 YRS) got her collar bone fractured from middle portion. Orthopedician suggested 8 figure sling. 15 d post injury, when it was reexamined by X-ray, no rejoining by then and broken portions were a bit more apart, in comparison to the time of injury. How much time it may take to heal properly and bone rejoining ? Doctor: Hi, Collar bone fracture unites gradually even if it is displaced. It needs surgery almost never. Any delay or mal-alignment of union will not cause any functional issues to shoulder. Figure of 8 helps initial immobilisation and pain relief. Healing usually occur by 4-6 weeks. Your orthopedist will discuss in details. Hope I have answered your query. Let me know if I can assist you further. Take care Regards, Dr. Muhammed Aslam TK"
},
{
"id": 113052,
"tgt": "Having lower back ache, over weight. How to reduce weight?",
"src": "Patient: i have lower back ache problem.from a very long time.its unavoidable.i am overweight also.70kg.age 31 height 5 feet 3 inch.how to reduce weight.what is ideal weight for me. my problem is since i was just 55 kg. thank you! Doctor: Good morming. I think if it is 50 - 55 kgs is an ideal weiht for ur height.do a lot of weight reducing exercises. For back pain keep a good postres while u sleep or sit. And do back exercises. U will be pain free very soon. Thank u"
},
{
"id": 189972,
"tgt": "Have epiphora. Wisdom teeth removed. Watering stopped. Are they related?",
"src": "Patient: I have had epiphora for over 2 years now and I just had my wisdom teeth removed today. I ve noticed that my watering has gone down immensely yet they are still watering a bit. Could the two be connected? As a side note, I also have bad allergies, but I ve yet to see the connection between the watering eyes and the allergies Doctor: Hi, Thank you for the query. Epiphora may be because of block in the tear ducts, narrowed tear ducts resulting in swelling due to inflammation,irritants causing over tears because of infective conjunctivitis, allergic conjunctivitis,trichiasis,ectropion (lower eyelid turned upwards). I would suggest you to see an ophthamologist and ENT to rule out the exact cause for the watering eyes . wisdom teeth with pain can give you tears sometimes but definitly continuous watering may be due to some allergy or inflammation in and around the eyes. Hope this information helps you."
},
{
"id": 45081,
"tgt": "Should i continue with folic acid 5mg during pregnancy ?",
"src": "Patient: Hi doctor , I am PCOD patient. We tried 2 and half years for pregnancy . I underwent laparoscopy operation two months back in India. My Gynecologist prescribed Folvite 5 mg Folic acid tablets. I came to UK after my operation. Now I am 6 weeks pregnant. Should I continue taking Folic acid 5mg? I was having Thyroid also. Currently I am taking Thyroid tablets Thyronom 25mg and Folic Acid 5mg. Please suggest whether I should continue these tablets? Do I need to take any other medicines? Thank you. Doctor: Hi Welcome to healthcareMagic Yes you should continue taking folic acid tablet till 12 weeks of pregnancy at least. There is no harm if you continue taking folic acid even after 12 weeks. Thyronorm can be continued after consultation with your doctor and reviewing reports of thyroid profile. You can start taking iron n calcium supplements after 2 months. Take care."
},
{
"id": 138269,
"tgt": "Can Sjogren s syndrome cause goose bumps on the legs?",
"src": "Patient: Hi I ve been getting goose bumps on my legs even if is not cold and I was told by my rheumatologist that I might had sjogrens because I had a positive ssa I wanted to know if the goose bumps had anything to do with the sjogrens I Also had a c4 level on 14 which he said it was ok also every time is hot or if I am in the sun my hands get red and swollen and in the morning when I wake up they also a little swollen and the swelling usually goes down in like an hour or so also I can t bath with hot water because they also get really red Please help Thank you Doctor: Dear Sir/MadamI have gone through your query and read your symptoms.In my opinion, you would be knowing that this syndrome can cause such problems and they have to be treated under a good dermatologist.I hope that answers your query. If you want any more clarification, contact me back."
},
{
"id": 110925,
"tgt": "How to treat the condition of herniated disc ?",
"src": "Patient: I just had my second MRI and now they said I need to be referred to a spine surgeon, the first MRI should I had a herniated disc, now when I asked for the results they said abnormalities and I need to see and spine surgeon? They faxed me the report but of course trying to read it is almost impossible unless you are a dr. Doctor: Hello, Thanks for your query.IN herniated disc( PIVD) - the nerves get compressed by the protruded disc.There are several treatment modalities with medicines, physiotherapy and with injections in the back called epidural injections and root/facet blocks.If you don\u2019t get better with these treatment ,I suggest you undergo surgery, by which the nerves are decompressed by removing the protruded disc (discectomy), which would significantly improve your symptomsI do hope that you have found something helpful and I will be glad to answer any further query.Take care"
},
{
"id": 102544,
"tgt": "Whats causing itchy rashes on the neck, shoulders after working out in the gym and which reduces after taking allergy tabs?",
"src": "Patient: I have recently been getting a rash on my neck after I work out at the gym and it itches. then I go to the Jacuzzi and sauna. Afterwards I notice the rash is on my shoulders and near my bra straps and it looks like large mosquito bites. It itches like crazy even after I shower. I've taken two allergy pills when it happens and that takes care of it. Today, my palms started to itch really bad. It finally went away with washing and cr\u00e8me. Then I noticed a large mosquito bite on my wrist, but didn't itch as bad. It stopped and not sure what caused that? Doctor: Hello.You are suffering a skin reaction that may be related to physical exercise, sweating, contact allergy to some substance ... Hydrocortisone cream twice a day + Ebastine 10 mg twice a day by mouth are widely used in the treatment of this condition.Please,go to your doctor to establish diagnosis and for prescription.I wish you good health.(If the answer has helped you, please indicate this)"
},
{
"id": 61311,
"tgt": "How to treat a non developing bump which has been for long time?",
"src": "Patient: Hi! I am a 33 year old female, about 40 pounds overweight, and my only health issue has been pulmonary embolism due to birth control that I no longer take. My vitals are always perfect, and I'm in good health. I have had two pencil eraser sized purple/blue bumps on my forearm since at least the age of 12. Sometimes they are flat and cause no pain; other times they are puffed up and tender to the touch. They haven't grown larger or moved. I have searched many times for similar bumps online but haven't seen anything like these. Any idea what they are? Doctor: Hello dearWelcome to Healthcaremagic.comI have evaluated your query thoroughly .* This seems most likely to be sebaceous cyst type soft tissue lesion .* Still better insight can be possible with photo pic as an attachment .Regards ."
},
{
"id": 89671,
"tgt": "Suggest remedy for abdominal pain",
"src": "Patient: hi, i m 25 years female weight 54 kg, i have pain in right abdominal area and also am suffering from mld loose motion, in the past i have been diagonised with high temperature due to typhoid and there were swellings in the abdomin as reflected in the ct scan report...but doctors suggested these swellings are due to typhoid...do u think i m infected by abdominal tb Doctor: No. Abdominal TB will have different findings on CT scan. You can repeat CT scan after one and half months to ascertain this. You can also go for Gold Quantiferon test of TB now.. If this is positive your suspicion of Tuberculosis can be considered. First get well from Typhoid."
},
{
"id": 196168,
"tgt": "Is there a chance of HIV from anal sex?",
"src": "Patient: Sir i really fear and confused to know fact about hiv sir because in 4 years back one homosexual person forced me sir. it is my childhood sir.after i tested. the test was given negative sir.i did two times hiv tri dot test.and i donate blood in two times.i can not realized .next again i went to that person to know about his mental attitude.this is my mistaken sir.i think and later i came to knew he was a homosexual person.but unfortunately while i met second time.he forced me.i rejected .but he touched my penis and bum (back hole) with their hands.later i very confused and i feel very sad about that situation.after did n't meet him and i can't dare to go any test center. r any doctor.(my question is if the sperm is in his fingers then any chances of getting hiv(it is my assumption and my doubt)?(in last 4 years i said answer to my mind and heart for that question ,is the virus is not living in atmosphere is n't correct sir)?so please give me a correct answer sir please because i very confused nd sad please . . . . Doctor: hiv spreads by doing intercourse,sharing needles,blood transfusion etc.,it wont spread by sharing vessels,hand to hand contact,mosquitoes,using same food plates etc.,dont fear.kindly do your elisa test 3 months after in order to get rid of fear.hope my answer will help you."
},
{
"id": 173872,
"tgt": "Suggest treatment for headache and eye pain in a child",
"src": "Patient: 3 year old. Complaining of head hurting and eyes hurting. Seems a bit lethargic, but still talking and somewhat active. Do I just do a tylenol/Ibprofun switch off or take her to the after hours clinic? Won't really let me take the temperature, but the ear thermometer said 101. Doctor: Hello dear,Thank you for your contact to HCM.The way a child exhibits a headache may be related to many factors, such as beginning of viral or bacterial infection, genetics, hormones, stress, diet, medications, and dehydration. Recurrent headaches of any type can cause school problems, behavioral problems, and/or depression. Actually, you an use the rescue medication purchased over-the-counter, such as analgesics (pain relievers), to stop the headache-Tylenol,Ibuprofen is enough. Keep your baby hydrated with plenty of water.Tylenol an be given in the dose 15 mg/kg every 4-6 hours,that too only if fever more ,than 100F. The common site of infection is throat. That's why pay attention to her gurgle. Take care! Kind regards Dr.Svetlana"
},
{
"id": 220562,
"tgt": "Will previous sexual abuse affect the pregnancy?",
"src": "Patient: When I was at the age of 7 or 8 years(i dont know exactly) i was forced for sex with a boy his age may be 15 to 18( dont know exactly)... now i am 25 years old i got married now will it affect my pregancy now? I am not getting period reguarly after marriage.... i am having black dots on my breast.......I am very worried now..... Doctor: Hello dear,I understand your concern.Iam sorry for your condition.In my opinion the sexual abuse in past is neither going to affect your future pregnancy nor it will prevent the pregnancy.There are various causes for irregular periods like thyroid abnormalities, polycystic ovarian disease (PCOD),high prolactin level, anaemia, overweight, obesity,stress etc.So the above causes should be ruled out by necessary investigations like ultrasound,thyroid profile,HB,serum prolactin etc.Weight reduction in case if you are overweight or obesity will be helpful.So adapt healthy lifestyle like exercise daily and healthy nutritional diet.Avoidance of stress and anxiety regarding pregnancy or any is very much important as they cause hormonal imbalance and cause irregular periods and delay fertility.Don't worry periods become regular once the cause is treated.Also regular intercourse increases the conception chances.Nothing to worry.Hope this helps.Best regards..."
},
{
"id": 151715,
"tgt": "What measures should an obese take to get rid of tingling in thighs and palms ?",
"src": "Patient: I AM OVERWEIGHT . I WEIGH 120 KG. MY RIGHT THIGHS HAS TINGLING SENSATION AND BURNING SENSATION CONTINUOUSLY. AND THAT PORTION BECOMES VERY HOT ALSO. THIS HAPPENS MOST OF THE TIME WHEN I AM SITTING AND SLEEPING. WORST IT BECOMES WHEN I SIT IN CAR. AND ALSO MY PALMS OF BOTH HANDS START TINGLING IN THE NIGHT. I CANT SLEEP. I WOULD ALSO LIKE TO SAY THAT DOCTORS KEEP SAYING REDUCE YOUR WEIGHT. BUT NOBODY GIVES EXACT PROCEDURE FOR WEIGHT LOSS . AND OVERWEIGHT PERSON ABOVE 100 KGS CANNOT PERFORM THE EXERCISES IN THE GYM OR TAKE A WALK BECAUSE AS IT IS THEIR BODY IS IN PAIN. THEIR MOVEMENT IS VERY LIMITED. Doctor: Hi i am very much concern with your weight. The main reason for your all the complaine is your weight. For a best suggestion can you plz revert me with your hight weight and age. Mind you dont get depress you can carry out your brisk walk even with this weight as i have lot of patient more than this weight who do there walk for more then 1hour. You plz also get me your blood test for lipid and kidney with complete thyroid test along with ecg. bye and take care."
},
{
"id": 126330,
"tgt": "What causes cramps in the feet at night?",
"src": "Patient: I am having cramping like charlie horses in the arches of my feet at night and sometimes have to get out of bed and stand on the floor until they stop. Also sometimes when I am talking on the phone and hang up, my fingers get stuck in a holding position and I have to straighten them out with the other hand. Am I afraid I may be getting MS or other serious ailment. Doctor: Hello, The most probable cause will be peripheral vascular disease due to poor blood supply to the feet. Consult a general surgeon and get a vascular Doppler to assess the blood flow. As of now you can take analgesics like Aceclofenac or Tramadol for pain relief. We have to rule out other possibilities like neuropathy also. Hope I have answered your query. Let me know if I can assist you further. Regards, Dr. Shinas Hussain, General & Family Physician"
},
{
"id": 17880,
"tgt": "Is surgery essential for the blockage in arteries?",
"src": "Patient: My wife, aged 56, had a heart attack (angina) last saturday and after observation in Vasai Hospital for 2 days had angiography in S.R.Mehta Cardiac Clinic, Kings Circle on 7 Jan 11 and diagnosed with 4 blockages with one major to be operated within a week. Cost may exceed 1 lac. Do you have any solution. Thanking you.Sincerely Suresh - YYYY@YYYY Doctor: Hello, After going through your medical query I understand your concern and I would like to tell you that an urgent treatment for blocked arteries is must especially after and heart attack. Major blockage should be taken care early to prevent any further cardiac Event and to improve cardiac efficiency. It\u2019s recommended for you to have a word with your treating cardiologist to proceed further. Hope I have answered your query. Let me know if I can assist you further. Take care Regards, Dr Bhanu Partap, Cardiologist"
},
{
"id": 91871,
"tgt": "Why do I feel left lower side of my stomach swollen and tight?",
"src": "Patient: My left lower side of my stomach feels swollen and tight across from my belly button, when I push down there is pain and I feel like I m going to vomit, my heads hot and headaches and my throat is swollen and makes it hard to swallow.. I m sorry I cant afford to pay thank you anyway Doctor: HI. Your history is suggestive of a problem with the descending part of the large bowel called colon,. This can be twisted or have a swelling causing you pain and other symptoms on pressure. Headache and throat problem may be altogether be a different problem"
},
{
"id": 8738,
"tgt": "I have a stitch mark above my upper lip",
"src": "Patient: i have a stitch mark above my upper lip on the moustache area. I want to get it removed using plastic surgery. can you please let me know how painful will it be and how much approx can it cost ? Doctor: Hi thanks for your query on healthcare forum.Any troublesome scar over the face can be revised and the procedure is known as scar revision .The treatment is done once the scar is mature ie. more than atleast 9 to 12 months old with no redness or blanching in the scar .The treatment depends upon the site , colour , texture , elevation or depression characterstics and the direction of the scar.Any treatment done has just one intention : to make the scar as inconspicuous as possible.The surgical treatment may be supplimented with Laser treatment or some ointments or injection therapy.If you are a male and keep your mustache we can just implant few hair follicles using FUE technique which camouflages the scar.What treatment is best for you can only be determined once the exact picture of the scar is seen. God Bless"
},
{
"id": 221854,
"tgt": "Is pregnancy possible without ejaculating in vagina?",
"src": "Patient: Hi, may I answer your health queries right now ? Please type your query here... i did intercourse with my girlfriend 13 days ago. Firstly i did intercourse with her in lying position but not ejaculate myself and after that did intercourse in standing position with the help of wall but also not ejaculate, is there any possibility of pregnancy guide me accordingly as I am tense of fear. Doctor: I would suggest you not to get worried as ejaculation into the vagina is essential for getting pregnant ,but still get a urine pregnancy test done to confirm pregnancy"
},
{
"id": 5875,
"tgt": "Smoker, drinker, drug abuse, had unprotected sex. If concieved, will all that affect the baby?",
"src": "Patient: hey there I m trying to help my cousin she had sex with this guy 2weeks ago she told me she had a feeling that she could be pregnant but she ate edibles a week ago and a couple days ago she smoked weed, drank beer, and snorted a line of crystal meth is it possible when she takes the pregnancy test the results could come out false ?and, could all that still affect the baby if she ends up pregnant? what would you suggest me to do to help her? sorry I m asking so much but I m really worried for her she don t want to go to the doctors and i really wonna have a talk with her about this topic but need to know these answers so i could know what to tell her. Doctor: Hi and welcome to the site. For the entire first 7 days after conception, there is ZERO connection of the mothers blood with the babies. So no chance that anything she took by way of drugs or substances will pass to the baby during that time. During the next 7 days after that, the embryo is just starting to implant into the wall of the uterus. SO only during this time slowly is there a connection to the mothers blood stream. So it isn;t until 14 days after conception that there is a meaningful connection between the mothers blood stream and the baby. I hope this is reassuring."
},
{
"id": 212062,
"tgt": "Taking lonazep and prodep for drinking, suffering from depression, mood swings. Help",
"src": "Patient: Hi I hv been taking lonazep and prodep since 5 years as prescribed by doc ......I stopped I between didn't go go to doc ......I hv been drinking hv mood swings now past 6 months Self medicated and taking lonazep and prodep and suffer from depression .... 2md morning and night with 20 mg prodep......trying to quit drinking but cannot help........please help Doctor: HelloFirst of all I would advise you to stop taking alcohol immediately because alcohol is a CNS depressant and taking alcohol with another CNS depressant like benzodiazepine may result in severe CNS depression. Now as you left medicines without the advise of doctor and again you starting taking it again without consulting doctor. I would advise you to consult a Psychiatrist regarding proper dosing of Prodep or fluoxetine. Dont self medicate yourself because clonazepam is a benzodiazepine and it has high abuse potential especially when taken without the supervision of a doctor.Thanks"
},
{
"id": 100856,
"tgt": "Suggest treatment for asthma with wheeze and sneez",
"src": "Patient: Hi, this is Badri, im suffering from asthma since my childhood around 10 years but from last two years it converted in to \"Bronchial asthma\", but from the last two years it became more my problems are:1) An allergic reaction associated with extreme breathing difficulties and wheezing. 2) Thick mucus is usually produced in large quantities and the airways become irritated. 3) Most of the attacks are triggered by hyper-sensitivity to air-borne particles(dust, pollen) or any sort of allergy to foods.4) when the climate is little bit cool my throat is getting very reddish and followed by sneezing and it automatically producing more mucus in my lungs then asthma is getting started This is the way my problem is getting more and more now my age is 24 male i got married.. Please give me the solution for my asthma problems i have used so many medicines but no use.... Doctor: Hello Badri,Thank you for asking at HCM.I went through your history and would like to know more about you like - what is the frequency of your asthma symptoms? Are there any particular foods that trigger your asthma? What is the frequency of your nose symptoms - like running nose, nasal congestion, sneezing, etc?From your given history, I can understand your concern and would like to ake suggestions for you as follows:1. Were I treating you, i would suggest you daily montelukast for at least 3 months and an inhaler like salbutamol/levosalbutamol to be used on as-and-when-needed basis for asthma symptoms.Should you have significant nose symptoms, I would also add an antihistamine like levocetirizine/fexofenadine to your daily treatment.2. I would suggest you allergy testing which will help you identify the substances causing troubles to you and also to know how to avoid them.Based on the report, an Allergist-Immunologist may prescribe you allergen specific immunotherapy which will work on your immune system to gradually improve your allergy symptoms.3. Please avoid exposure to dusts,smokes and air pollution as much as possible.4. Regular breathing exercises like Yoga & Pranayama and healthy diet rich in vitamins and minerals (adequate amounts of green leafy vegetables, fruits, sprouts) will improve your lung capacity and immunity respectively and will be helpful to you.Bronchial asthma is very much manageable disease. With combination of appropriate medications, immunotherapy (if needed), nutrition and lifestyle changes, one can live a life just like other normal people. So please do not get stressed about it.Hope above suggestions will be helpful to you.Should you have any other query, please feel free to ask at HCM.Wish you the best of the health.Thank you & Regards."
},
{
"id": 148133,
"tgt": "Which specialist should be consulted for epilepsy?",
"src": "Patient: Hi I am looking for a GP with good knowledge of Epilepsy. Our Neurologist is an Epilepsy Speicialist but there are only 3 short appt's per year and the waiting time withing these clinics are approx 2hours. Any other support is only through a very busy and frightening Emergency Department at Fremantle Hospital, were we have waited for approx 6hours or sometimes into Day 2 and sent home without any support, as the seizures have stopped and told to carry on as usual. Finding regular supportive medical support has been impossible. Doctor: Hi, welcome to our site. I am Dr Saumya Mittal.Read your query. That is a very significant question and i appreciate your problem. I will try my best to answer your querysorry about what you facei would suggest you look for a md medicine doctor at least if the neurologist is unable to give you ample time.it would be most necessary to have that done. the idea is that a md medicine would be able to give you a better advicemeanwhile, if the seizures are under control, you need not worry too muchi do hope an mri brain has been done at least inceand you do check for lft once in a whileI hope this helps you. Inform the reports mentioned above so i can be of help further. Best of luck.I have given you the answer to the maximum considering the information provided. The results of the tests could further enhance my answer to you.Please do understand that some details could be extracted from a detailed history and examination.Looking forward to your return query with the details asked so that I can help you further.(If the answer has helped you, please indicate this)"
},
{
"id": 128798,
"tgt": "What causes sudden pain in the left arm?",
"src": "Patient: 52 year old female with sudden pain in left arm and hand. came out of nowhere, first realized this while picking up a cup; had no control didn t scare me just pissed me off. what is your professional input plz respond now. with all due respect, thank u for your time. Doctor: Hello,This sounds like some sort of nerve irritation. Any of us can have such a pain for unknown reasons. If the problem persists then there may be something in the arm or shoulder that is impinging upon a nerve. You would need an exam and testing to figure out the exact cause. If it does not happen again then I would not worry about it.Regards"
},
{
"id": 15801,
"tgt": "Rashes on the face spreading to the neck and shoulder. Do not want to take antibiotics. Any other treatment?",
"src": "Patient: i am growing and not feeling the way i use to feel. i have this rashes on my face that started in 2011 and refused to go. i went to the hospital and i was put on docicycline for three months which i refused to take cos i felt the antibiotics will dry my blood. earlt this year the rashes decended to my neck and upper part of my body. what can i take to make me get back to shape. Doctor: HiThanks for your querySeems like you have developed Acne vulgaris and Corporis lesions..Sometimes acne lesions can occur over the face and body esp upper back, chest and upper armsAcne vulgaris is a common human skin disease, characterized by areas of skin with seborrhea (scaly red skin), comedones (blackheads and whiteheads), papules (pinheads), pustules (pimples), nodules (large papules) and possibly scarring. Acne affects mostly skin with the densest population of sebaceous follicles; these areas include the face, the upper part of the chest, and the back. In your case, you seem to develop acne lesions only over chest and upper arms,..This is a common condition especially in adults,...Some general advise : Avoid using any kind of moisturising creams/ lotions or oils over the affected area. Wear cotton clothes as much as possible Do not use any liquids like dettol/savlon over the affected area You can however continue to use antibacterial soaps You might need to apply some antiacne medications like Clindamycin gel over the affected area in the morning along with another anti acne agent like benzoyl peroxide 2.5% at night. But if inspite of the above measures, you continue to develop the lesions, then you might need a course of oral antibioticsOral antibiotics have very good results in acne patients esp in those not responding only to topical agents.So i would recommend that you visit a dermatologist in that case for proper examination and treatment wishing you speedy recoveryDr Geetika Paul"
},
{
"id": 16171,
"tgt": "Pink rash along rib cage, spread to stomach, inner thighs. Causes for rashes?",
"src": "Patient: Hi, I have had a pink, bumpy, but not itchy, rash along my rib cage for about a week. But just recently, it has spread to my stomach and inner thighs. I am on no meds and have not changed location. I have never been allergic to anything and have had no previous medical problems. I am a 20 year old female, height 5 5 , weight 180. Doctor: hi Phoenix , Thanks for writing in. The rash you are suffering can be pityriasis rosea.It is a type of viral infection. It is self limiting and there is nothing to be worried about. You can apply calamine lotion over the area . Avoid any other drugs or medications. I encourage you to visit a dermatologist for proper diagnosis and examination. Hope this helps. Regards. dr Sudarshan. MD dermatology."
},
{
"id": 49273,
"tgt": "What is the prognosis for dialysis done on child?",
"src": "Patient: My niece is in the ICU for 3 weeks now and everyone is worried about her condition because the nurses and doctor at the hospital refused to take out the tub in her mouth out so that she can at least eat. My niece had two procedure done, one last week and one yesterday (dialysis). My niece is very cold and her hands and feet are turning dark purple. My niece will be 18 years old in March of this year. I am very sad because she is innocent just like all children around the world. She has not yet see what the future holds. Thank you very for helping. Doctor: Dear Madam,Thank you for posting your query on HCM.Well looking into your problem IT will not be possible to say anything because dialysis can be done for a variety of reasons , if it is done for acute renal failure she will improve but in case chronic kidney disease is there prognosis is worse .Dr. Shruti"
},
{
"id": 143748,
"tgt": "What are the symptoms of having Lebers neuropathy?",
"src": "Patient: My husband is totally blind from lebers optic neurothpathy and recently has had symtoms of constant lip licking espceially when he is tired which he isn t aware he is doing . He also has essential familinal tremors which is increased as he has got older He sometimes has some mild difficulty swallowing, Are these symptoms related to his Lebers neuropathy. Doctor: Hi, yes these other symptoms may be present in LHON Vision loss is typically the only symptom of LHON; however, some families with additional signs and symptoms have been reported. In these individuals, the condition is described as \"LHON plus.\" In addition to vision loss, the features of LHON plus can include movement disorders, tremors, and abnormalities of the electrical signals that control the heartbeat (cardiac conduction defects). Some affected individuals develop features similar to multiple sclerosis, which is a chronic disorder characterized by muscle weakness, poor coordination, numbness, and a variety of other health problems.Thanks"
},
{
"id": 216381,
"tgt": "Suggest treatment for terrible headache and body ache",
"src": "Patient: Hi, I cleaned my grandmas garage and there were rat dropping everywhere.. Today, the day after, I feel terrible, headache body ache, sort of like a fever. In not sure what could cause this but I do know you can get sick from inhaling rat droppings in dust. Which I was doing Doctor: Hello Thanks for Healthcaremagic. I have read your query and understand your concern . As far as terrible headache and bodyache is concerned for this you take proper rest and you can take medicine like Paracetamol with citrezine by consulting with local doctor . Hope this will help you ."
},
{
"id": 58725,
"tgt": "Liver abcess following whipple procedure for neuroendocrine tumor of pancreas. How long should the antibiotic therapy taken?",
"src": "Patient: if a liver abscess occurs after a whipple procedure,which was done for neuroendocrine tumor of pancreas with mets to liver, and leads to septic shock , what should the duration of antibiotic therapy be. icu was one week, followed by medical floor for a week, and now inpatient rehab for two and half weeks. drain is still in, being followed by the surgeon. should a gi specialist be involved in care? Doctor: Hi and welcome to Healthcare magic.Thank you for the query.It should be treated there is radiological confirmation that there is no abscess anymore. antibiotic commonly arent enough for full recovery and you should cosult doctor who can perform percutaneous drainage if there is no improvement on antibiotic th. Such abscesses can cause serious consequances so it must be treated aggresively,even by open surgical procedure,sometimes.Wish you good health. Regards"
},
{
"id": 166169,
"tgt": "What causes difficulty in bowel moments in baby with sticky stools?",
"src": "Patient: my little grandaughter is 20 months old...when she needs to poo she seems to be in so much pain, her little body shakes. Sometimes the poo is very sticky and she has developed a rash. This has been going on for about a month. She cries when we try to wipe her. My heart breaks every time she has to poo (which is often because she doesn t do ot all at once). She is a very good appetite and eats everything. She drinks milk and water ...some juice. Is this normal for children her age to go through this type of pain? Doctor: Hello. I just read through your question.it is not unusual for babies to display this type of bowel behavior from time to time. I recommend adding prunes or prune juice to her diet. This will stimulate her digestive tract to work a bit more efficiently thereby decreasing her discomfort during bowel movements."
},
{
"id": 112734,
"tgt": "Fell down. Head injury healing, right chest and back area painful. Advice?",
"src": "Patient: hi, last monday 3rd June I slipped on a wet area in bathrrom falling sideways onto the outside hob of shower, not drunk either. My head is cut but healing but my right chest & back area is still severely painful. Breathing is slowly improving but difficult to lay comfortably in bed without 6 pillows, propping my left side high to take the weight of my arm off my chest & back area. I doubt i have broken a rib but it sure feels like it. I am better walking around slowly than lying which makes me ache & stiffen . the 1st 3 dys i laid in bed but now i am walking around slowly. could this till be a rib repairing or bruise??? If i went to hospital & managed to get an xray which they are always very hesitant to do usually only with insistence, i still prob would only be told to rest as no treatment for broken ribs. Yes I have had b4 many years ago, the lower 3ribs in a car accident. any advice would be appreciated. Doctor: Hi and thanks for this query.i a sorry to hear about this bath room accident you suffered.Knowing the exact problem is always a plus. The management of broken ribs if often rest and pain killers but not ALWAYS. if the Xray should suggest a broken rib and reasons to operate upon, like the edge is pointing to the skin or the heart or lungs, then operating upon to align it is possible. So, please go and get the xray done. At worse, it will provide answers and reassurance/assurance.If you should not feel better soon, then going to the hospital is a better option. Hope this helps and answers your question.Feel free to come back to us for more answers.Cheers"
},
{
"id": 56359,
"tgt": "Is there any effects on body after gallbladder removal?",
"src": "Patient: My Pita (i think it is known as gall bladder) is removed after surgery . doctors advised me to use less oil in food but not mentioned what precaution should i take . can u plz plz tell me the consequences and effects on my body by such removal . what are its alternatives. Doctor: Hi there,Thanks for using HCM.After removing gallbladder, you may get loose stools for a few days which will stop on its own. Other than that there will not be any serious consequences after gallbladder is removed.There is no need to take ant special precautions.Is this answer helpful?"
},
{
"id": 52569,
"tgt": "What causes persistent pain in the abdominal area and fatigue?",
"src": "Patient: I have right moderate constant pain in my liver area. All labs are normal, MRI and ultra sound normal and yet I feel fatigue and continued pain in the liver area, especially upon palpation. I don t know what to do next , all the attending Dr. are also puzzled. What do you think I should do ? Doctor: Hello and Welcome to \u2018Ask A Doctor\u2019 service. I have reviewed your query and here is my advice. As we can not make a diagnosis with ultrasound. Get a CT scan abdomen to look for any lesions. You can also opt for an endoscopy in case of diagnostic difficulty. Hope I have answered your query. Let me know if I can assist you further."
},
{
"id": 54148,
"tgt": "Suggest treatment for abdominal pain, nausea and constipation along with inflamed bile duct",
"src": "Patient: hi i am a 48 year ol female, 202lbs. I have debilibtating pain in my upper abdomen that radiates to my back. I have nausea, constipation (with intermittent diarehhea and vomitting). I also have very dark urine (i drink alot of water) and my feces is a strange tan color. I was taken to my doctors office to the hospital last week because the pain was so bad. I had a ct scan which show inflammed bile duct,no stone. gallbladder was removed at age 21. My My liver enzymes are elevated. I also had an MRI which then showed a small cyst in my spleen. I don't know what to make of any of this. Please help. Doctor: eg:cyct in spleen must do fine needle biopsy to diagnosis and take a tinflamatory and liver support"
},
{
"id": 177930,
"tgt": "Suggest medicine for child suffering from recurring upper respiratory infection",
"src": "Patient: Hi, My Son is 4.5 yr old. Since 1 year, he started catching cold & cough every second month. He was some time prescribed for Prednisol. Then He was prescribed for QVR, Albutorol puffs. Since we are his parents, we actually understand, he does not have asthma but Upper resperitory infection, which triggers cold, could and some time breathing issue. We don;t know, why he prescribed for such medicine. Can t be there any medicine which can just stop upper Resperotary infection? Doctor: All the medicines prescribed are ment for asthma probably the child is having COPD . The child requires further investigation and long-term treatment ..along with medicines change of place(environment)&change in diet & also breathing excerise after 6 yrs"
},
{
"id": 129897,
"tgt": "What causes swelling on ankles?",
"src": "Patient: fell and twisted ankle in 3/2012 and diagnosed with unspecified sprain/strain left foot. Xrays showed nothing broke, MRI shows severe edema. 12 PT treatments, that helped a month ago. I continue to wrap wth ace bandage, walk and still my foot and ankle are extremely swollen, unable to get left shoes on and the ball of my foot hurts now. What could ths be??????? Doctor: Hi..The problem in your ankle is due to sprain in inferior talo fibular joint...termed as High ankle sprain...It will take a bit longer time to heal...Don't worry...I recommend you to continue Icing frequently...Keep your ankle wrapped as you do now...Gentle massage over calf muscle...and outer muscles of leg...once pain permits gently stretch the calf and outer keg muscles..glute muscles....Don't bear weight on the injured leg as this would disturb the healing process...Start glute stability and ankle rehab immediately...Hope this is helpful for you...Kindly revert back in case you need any further help in this regard"
},
{
"id": 200868,
"tgt": "Is it possible for the reduction in size of the penis after a surgery for cyst?",
"src": "Patient: Hello, I would like to ask you something concerning a dermatology problem. I underwent a surgery before one month. I had a bladder/cyst on my penis and after the surgery i think that my penis has been redused a little bit, is it possible? And if it is, is there anything i can do? Doctor: Thanks for asking in healthcaremagic forumIn short: Only a cyst can be removed not your penisExplanation: If it was only for the cyst. Cyst might have been removed which inturn appears like your penis' size has been reduced. I dont think so. Stop thinking of size, if you are having good erection that all required. Be happy."
},
{
"id": 218439,
"tgt": "Should a pregnancy test be done when experiencing dizziness and frequent urination?",
"src": "Patient: Hi there. This is really stupid but, about a month ago my boyfriend and I were having sex, and I thought he was wearing a condom when he wasn\u2019t. I don\u2019t think any sperm got in me, but it was close. I have irregular periods where I don\u2019t know when i\u2019ll Have a next one but, I am experiencing some different symptoms such as bloating, heartburn, having to urinate a lot more frequently, dizziness, and a general irritability has me worried that I might be pregnant. Should I take a test? Doctor: Hello and Welcome to \u2018Ask A Doctor\u2019 service. I have reviewed your query and here is my advice.Of course you should take a home pregnancy test but remember that you have to take it five days past your expected period. Hope I have answered your query. Let me know if I can assist you further."
},
{
"id": 155582,
"tgt": "What can be the cause of coughing up blood?",
"src": "Patient: my mom had a video barium yesterday and was told the epiglottis is not closing properly. she also wakes up at night coughing/choking with blood \"pooled\" in her throat, not actually \"coughing it up\". she has been doing this off and on for three years and no one can find a reason for the blood. Doctor: Thanks for your question on HCM. In my opinion you should first consult pulmonologist and get done chest x ray and Bronchoscopy. Coughing out blood can be due to1. Infection like pneumonia ot TB2. Chronic bronchitis3. MalignancySo tu rule out all these chest x ray and Bronchoscopy is needed.You may need CT THORAX if x ray findings are inconclusive."
},
{
"id": 187890,
"tgt": "What could be the reason for having a lump under my jaw?",
"src": "Patient: I am a 57 year old male and have just developed a lump under my left side under my jaw and when I eat my salyiva is very sensitive and seems to feel like it has a drain under my toungue in the front of my mouth and has been up and down for about 2 days , no fever , but some pain trying to swallow ?? sorry I am not able to pay I was told this was a free web-site sorry for your time >> Good bye !!!! please delete my question !!!! Doctor: Hi,Thanks for asking the query,Lump under the jaw can be due to salivary gland blockage i would suggest you to get a thorough examination done by a Dentist also get an OPG radiograph done.Take care!"
},
{
"id": 29435,
"tgt": "Suggest medication for recurrent fever",
"src": "Patient: Hi Doctor, My daughter having high Fever .Not Regularly, between 1 hr interval, she is getting the fever with his head and feet.Has consulted few doctors in my home town .But the fever is not reduced.Can you please help us to get some medicines to reduces the fever. Doctor: Follow health advises given below:Measure your temperature and tell me reading 1. Avoid exertion 2. Take tablet paracetamol 500 mg after food ( check drug allergy) as and when required for fever more than 99 f (maximum 3 tablets with gap of 8 hr can be taken in a day) for 3 days 3. Lot of fluids to be taken 4. Take proper diet homemade food like moong dal dalia, chapati etc. 4. Avoid wearing synthetic and tight cloths. We have to evaluate for other symptom like cough/ throat pain/ burning urine/ stomach pain so that we can confirm cause of feverAnd get Followng test done and review with reportsComplete Urine Analysis (CUE), Complete Blood Count (CBC), Erythrocyte Sedimentation Rate (ESR), Malarial Parasite Identification, Typhidot"
},
{
"id": 124662,
"tgt": "How long will it take to hold the displaced sacrum in place?",
"src": "Patient: I ve been diagnosed with a displaced sacrum by a physical therapist. He realigned the bone and had me start doing daily exercises to strengthen the muscles with the intention of holding the sacrum in place. I slept on my wrong side day before yesterday and woke up in a lot of pain. I can tell the sacrum is displaced, again. How long does it take to permanently correct the problem? Doctor: Hello, Two to three months of immobilization is required for best results. Consult an orthopedician and he will direct you accordingly. Hope I have answered your query. Let me know if I can assist you further. Regards, Dr. Shinas Hussain, General & Family Physician"
},
{
"id": 20406,
"tgt": "What causes heaviness in the chest and breathing difficulty while having heart disease?",
"src": "Patient: Hi! I m a 28years old male, and since last year I m having a heart problem( I think...).It started when I was on a diet .(In 3 months I ve lost 15kg.From 85 to 75 )During physical exercises or sometimes without any reason my heart suddenly begun to race even do I wasnt tired and as quickly as it started after couple minutes the symptoms were going away. So I went to a doctor I ve made some tests (ECG,Echocardiogram,blood,trademill) and everything came up normal . After that I didint have any palpitations but I was not feeling so good. Dizzines, felt like my blood pressure was jumping up (for example..when crouching) and down and heavy, slow beat of my heart . Now I m having tiny, needle like chest pains from each side and very cold hands, feet which I ve never had before. And on top off that for last couple days most of the time I feel like something heavy pressing on my chest, causing difficulties with breathing properly. I would be very grateful for any help or advice on this problem. Luka Doctor: Hi Luka ... If i had a consult like yours .. i would first reassure that there is nothing serious ... As your echo ecg and treadmill are reported normal .. Just get your thyroid test done . If its normal .. Be assured u are healthy . Hope u dont smoke .."
},
{
"id": 137326,
"tgt": "What causes severe body pain after taking vitamin d supplements?",
"src": "Patient: Because of the result of blood tests it was determined that I needed vitamin D. I have been taking 5 pills daily. I have been on this dosage for five weeks. Since that time I have been in constant pain. It feels as if I have arthritis all over my body. I am up at 2 am because the pain is so bad. I did not have this type of pain prior to taking vitamin D. I had this problem when I tried to take calcium supplements and finally had to stop. The pain was not as severe as it is now. What is happening? Doctor: HelloI have studied your case. There is possibility of associated arthritis leading to pain. vitamin D supplements will not lead to pain. I will advise you further investigation like MRI and clinical examination. I hope I have answered your questions. If you have further questions please feel free to contact us. I will be happy to answer. Take care."
},
{
"id": 170417,
"tgt": "Suggest treatment for dermatitis in a 11 years old child",
"src": "Patient: My 11 year old son most likely has perioral dermatitis from over use of steriod on his rash on the sides of his nose, elidel is not helping much and we have been using it for like 6 months...family doctor gave us mipirocin to try but its redder and crusty lookign now. Doctor: Hi...Thank you for consulting in Health Care magic. Skin conditions are best diagnosed only after seeing directly. I suggest you to upload photographs of the same on this website, so that I can guide you scientifically. Hope my answer was helpful for you. I am happy to help any time. Further clarifications and consultations on Health care magic are welcome. If you do not have any clarifications, you can close the discussion and rate the answer. Wish your kid good health.Dr. Sumanth MBBS., DCH., DNB (Paed).,"
},
{
"id": 162849,
"tgt": "Suggest treatment for stomach pain and cough in children",
"src": "Patient: My five yo has recurring influenza symptoms after being pain free, fever free, motrin free all day. This is his 2nd round of flu like symptoms in 3 wks. He also is complaining of stomach pain, cough, conception. Randomly throws up phlegm. Thoughts on meds, RX, viral, bacterial? YYYY@YYYY Doctor: Hello and Welcome to \u2018Ask A Doctor\u2019 service. I have reviewed your query and here is my advice. As your child has repeated infections, it may be due to reexposure to infection or decreased immunity. For that you can give cough syp. containing Guaifenesin and Ambroxol. He may require antibiotic like Ofloxacin and Mefenamic Acid for stomach ache after consultation. Keep him hydrated. Give syp. Paracetamol for fever. Take care. Hope I have answered your query. Let me know if I can assist you further."
},
{
"id": 67929,
"tgt": "Can red, swollen and painful lump on neck be serious?",
"src": "Patient: I have a hard lump on front of neck which is red and swollen. It is very painful and making harder to breath. I ve been to urgent care twice and told to put warm compresses. I ve been doing that for three days. Should I go to emergency or wait till tomorrow and call my doctor? Doctor: Thanks for asking your question here in this forum...red swollen and painful lump ,all are signs of infection...sometimesnthere is bacterial thhroiditis which is like this...i will advise you to see your surgeon..if its an abcess it needs to be drain...if its some other cause that needs the proper management...hope thises informations are useful for you...thanksDr ihsan"
},
{
"id": 104817,
"tgt": "Taken antibiotics for chest infection. Have flu virus. Taken Plumicort for asthma. Is it okay to take Symbicort?",
"src": "Patient: i have the flu virus now for 5 days . The day before i finished an antibiotic for chest infection . i suffer from allergies and when i get a chest cold i get asthma . i take plumicort turbo inhaler 200 micrograms 2 puff twice a day and when i need too i take salamol 100 micrograms , 2 puffs 3 times a day . i dont always have to take the salomol . on day 3 of the flu i started to cough up mucous which had traces of blood for the day .proior to this i hab been coughing up green mucous after finishing the antibiotoc. by late evening day 3 i started wheezing very badly snd couldnt breath very well so i took the salomol but after taking the 3 doses to no avail i decided to try my sons symbicort turbo inhaler 100 micrograms 4 puffs which helped a lot . is it ok to continue to use this as my gp is not availabe Doctor: hi, i think your chest infection must have cleared by now, some times traces of blood do come in mucus when there is severe infection, if you get again you can visit your doctor for check up. if it has stopped after your infection is cured then you can stop worrying about it. now about your asthma, plumicort 200 which you take is budesonide, which is steroid. and symbicort which your son uses is budesonide plus a long acting broncho dilator. ideally when inhalers are given it is adjusted to minimum dose at which patient is comfortable. since you where comfortable on pulmicort 200 you should continue the same thing. but for now until you get medical attention you can take symbicort 100 ,4 puffs 2 times , if you have heart problem or you are old you should go to your doctor before starting your sons inhaler. your symptoms of asthma increased due to your lung infection after 2-3 days you will be fine like before, and you will be comfortable on your inhalers itself thank you dr.mukesh"
},
{
"id": 126249,
"tgt": "Suggest treatment for ribs pain despite taking Naproxen",
"src": "Patient: Hi there.A few weeks ago I went toboganning with my daughter and I ended up wiping out pretty hard and landed on my left side.Felt like I was hit by a truck or something as the impact of landing was severe and I felt like I had the wind knocked out of me.I have roto-scoliosis so my left side rib cage has always protruded more than my right side.Took me a few minutes before I was able to get myself back up and I noticed it immediately hurt to inhale.I thought well maybe I just bruised my ribs or something and I took pain meds at home and iced the area but still was in a lot of pain so I went to my family doctor and she believes I may have several cracked ribs and prescribed me Naproxen and said she could order an xray but felt it was pointless as there s nothing that can be done anyhow.The breathing in has gotten easier but now I m finding myself in different pain that s almost constant but sometimes worse at times then others. Almost like I have a constant pressure..like I have a cement block sitting on me or something.I ve also developed a new symptom where I can feel a shifting or popping in there. I have no bruising but I do have swelling.This accident happened on family day(Feb 19th).The pain I have is not far below my left breast.Is this normal or should I go back to the doctor? Doctor: Hello, The symptoms are most likely suggest the possibility of costochondritis. As a first line of management you can take analgesics like ibuprofen or diclofenac for pain relief. Anti inflammatory drugs like seratiopeptidase is also effective. Hope I have answered your query. Let me know if I can assist you further. Take care Regards, Dr. Shinas Hussain, General & Family Physician"
},
{
"id": 150420,
"tgt": "Seizure. Valparin chrono 500 and 300 taken previously. History of fits. Advise?",
"src": "Patient: I have had fits in the past (only 2-3 occasions) and I was advised to take Valparine Chrono 500 and 300 (day time 500 and night time 300). I had it for many many years and stopped taking it as advised by my neurologist in 2010-11. My last fit occurred in 1996-97. I was completely well and continued with my daily tasks with no difficulties at all. Today I again had a seizure and this surprised me. Should I take my medicine life long now? Also will these put any restrictions in my life? I go to the gym and play cricket, will these be effected at all? Doctor: Hi friend, Welcome to Health Care Magic Seizures are not always curable / yet most of them are manageable. You have not stated your age, gender, personal habits, family history or co-morbidities You should see a Neurologist again You may have to take drugs for several years again / and then try for gradual withdrawal again.../ may be life time medications are necessary. The restriction are minimal / usually aimed at preventing injury Take care Wishing speedy recovery God bless Good luck"
},
{
"id": 174411,
"tgt": "What are the symptoms of brain lesion in a one year old?",
"src": "Patient: what would be some warning signs or symptoms for a one year old girl with problematic brain lesions? the lesion is located in the vision/hearing part of her brain. she is extremely clumsy and has a very hard time keeping her balance. I am worried that the lesion is not benign like the dr seems to think. Doctor: Hi,Thank you for asking question on health care magic.Brain damage in less than one year old child will have delayed mile stones and speech delay ans some times mental retardation.some may have hearing and vision problems.Better consult pediatrician or neurophysician.Hope this answer will serve your purposePlease feel free to ask any more queries if requiredTake careDr.M.V.Subrahmanyam MD;DCHAssociate professor of pediatrics"
},
{
"id": 220041,
"tgt": "What causes painful rectal bleeding during a pregnancy?",
"src": "Patient: hi i am 28+6 weeks pregnant and for the last 2 days everytime i go for a poo it is really painful and i bleed quite abit of fresh blood from my bum after i am really worried that something might be wrong with my baby is all this normal as it leaves me saw for the day afterwards Doctor: Hello dear,I understand your concern.In my opinion the bleeding during passing motion suggests the bleeding is from rectum.It might be due to anal fissure or piles.It needs physical examination for accurate diagnosis.Passing hard stools or constipation will provoke fissure or piles.And constipation is a common complaint faced during pregnancy.But nothing to worry as it will subside with treatment.Take adequate fluids upto 3 litres per day,high fibre diet and laxatives to prevent constipation.Once the constipation is treated the bleeding subsides spontaneously.But kindly consult doctor to check for any piles.Topical ointment can be applied to decrease the pain.Also take iron rich diet and iron supplements to combat anaemia caused due to blood loss.Sometimes iron supplements also cause constipation so check if they are the culprit and in that case the preparation need to be changed.Relax.Avoid stress.Hope this helps.Best regards....."
},
{
"id": 196578,
"tgt": "What causes uncomfortable feeling if I don't ejaculate?",
"src": "Patient: hello sir, i am 20 years old guy and i m facing prob that i ejaculate every day and if i don't i m feeling very uncomfortable i can't concentrate on any thing my mind is totally diverting me to see bluefilm's and ejaculate.Some times i decide 2 see clips but nt 2 ejaculate but when i start seeing clips i cant control myself 2 ejaculate. i wanted 2 know tht is it safe 4 me 4 future. Can i able 2 satisfy my wife in future Doctor: if you believe in Ayurveda, believe me it's 'veeryaksheenta' - the condition related to debilitated 'shukra dhatu' (semen).it can be cured by simple diet management of light and 'satvic' food, deep breathing exercises (pranayam) and meditation and few herbal combos called 'vrushya' herbs resulting in semen generation and spermatogenesis to the required level. Semen is very important element as per principles of ayurveda.By effects of 'vrushya'herbs u will have check on uncontrollable passion and feeling to ejaculate. For the time being you may start simple 'Shatavari' and 'Ashwagandha' herbs in powder form or tablet or capsules form with milk twice a day. Pl report after 15 days, of response or no response, so that I direct you required combos ahead. treatment may be required for six months. believe me it's totally safe n effective. pure natural, thats why slow. for any other questions pl ask me or any classic Ayurveda doctor."
},
{
"id": 169768,
"tgt": "Suggest remedy for swelling in penis in a child",
"src": "Patient: My 6 year old son put his penis by the hot tub jet. The heat wasn t on just the jets. I didn t realize he was doing that but he got out to change and showed me and it was super swollen on the shaft and hasn t gone down for about an hour. I wasn t sure if I should take him to the doctor or if the swelling will go away on it s own. It s normal coloring and it doesn t seem to be bothering him too much just a little sensitive. But it s pretty swollen especially right under the tip Doctor: Hi.... I suggest you put some cold water in the region and see if there is any betterment of the Swelling. But you need not worry if he is not feeling any discomfort or if there is no problem in passing urine. If these two things appear then I suggest you get him evaluated by a physician or pediatrician.Regards - Dr. Sumanth"
},
{
"id": 187005,
"tgt": "How does braces on teeth work?",
"src": "Patient: i went to the orthodontist the other day and she said i need to get 6 baby teeth pulled out since the adult teeth are growing on my pallet. she said i need minor surgery and to have these teeth brought down by braces. how will they bring the teeth down? i am 14 Doctor: Hi. Welcome to healthcaremagic.I read your query. Primary teeth are pushed out by growing permanent teeth. But at times the primary teeth do not shed at time and permanent teeth deviate from normal path. Orthodontist will study your dention and OPG. He will pull out primary teeth which are not shedding naturally. Braces are applied to permanent teeth and tighten so it exerts a required amountvof pressure on them.Slowly your permanent teeth will move into their destined allignment. Entire process will need 6months to a year.You may have to wear retainer after that for maintainence.Hope the answer helps you. Thank you!"
},
{
"id": 79047,
"tgt": "How to treat bronchitis?",
"src": "Patient: I went to the Dr today and was diagnosed with bronchitis. I thought I was getting better because I haven't had a fever today just a horrible cough. Until, tonight, when I all of a sudden got severe pain in the back left side of my neck and head. I feel dizzy and nauseous. What's going on? Doctor: HIWell come to HCMI really appreciate your concern, in bout of cough such dizziness could be likely while coughing care need to be taking, better not to exert any force because it may cause circulatory disturbances, hope this information helps, take care and have a nice day."
},
{
"id": 166195,
"tgt": "What causes pain in forehead in a child?",
"src": "Patient: MY 4 YEAR OLD WOKE UP IN THE MIDDLE OF THE NIGHT SCREAMING AT THE TOP OF HER LUNGS THAT HER FOREHEAD HURT AND AGAIN THIS MORNING SCREAMING THAT HER FOREHEAD HURTS TYLENOL SEEMS TO HELP BUT ONLY FOR A SHORT PERIOD OF TIME SHOULD I TAKE HER TO URGENT CARE? Doctor: hi, pain in forehead could be due to migraine, space occupying lesion in brain, weak eyesight. If the pain is very acute then child should be taken immediately to emergency and imaging like CT scan or MRI needs to be done. If pain is not that much then proper investigation should be done like a checkup should be done for refraction, ENT check up should be done to rule out migraine, is symptom persist then CT scan or MRI should be done. you can give paracetamol syrup 15 milligram per kg at the time of headache. Take care."
},
{
"id": 173917,
"tgt": "What could cause constant crying and being stubborn in a 3 yr old child suggest?",
"src": "Patient: My three year old cries all the time. Once she starts, it can last an hour or more. Ignoring her upsets her more and so does putting her in time out. We've tried distracting her with something else but that usually gets her more upset too. Family members are starting to notice her fits and are asking me how long this has been going on and encouraging me to have her evaluated. I'm not sure where to go from here. Doctor: Hi,Thank you for asking question on health care magic.Your history indicates that she may be suffering from temper tantrums.Normally with advancing age it will disappear.If you like you may see a child psychologist.Hope this answer will serve your purposePlease feel free to ask any more queries if requiredTake careDr.M.V.Subrahmanyam MD;DCHAssociate professor of pediatrics"
},
{
"id": 69047,
"tgt": "Suggest treatment for a lump on the outer part of thigh",
"src": "Patient: Hi, I have a lump on the outer of my thigh invbetween the back of my leg and knee cap.. It feels like it has roots Coming off it.. It is a little bigger that pea size... I tryed to squeze it but nothing has. Come out.., it has been there about 3 eeeks Doctor: Hello! Welcome to HCM.This can be a Morrant Baker's cyst, as you said its coming from knee.Its a benign cyst communicating with the knee joint.Also it can be a lipoma or sebaceous cyst rarely.In my clinic, after clinical examination, I advise FNAC - fine needle aspiration cytology for confirmation of the diagnosis, if in doubt.Treatment is marsupialization for Baker's cyst & for any other benign symptomatic lump, excision biopsy is done i.e. swelling is removed under local anaesthesia & sent for histopathological examination.Complete removal along with whole of the cyst wall is important.Hope this helps.Wish you healthy life.Thanks.Regards."
},
{
"id": 148752,
"tgt": "Excruciating pain all over the body after lumbar puncture hit sciatic nerve. Suggestions?",
"src": "Patient: Hi: I had a lumbar puncture with flouroscopy Friday afternoon. Now, two days later, I have excruciating pain all over my body, especially my legs. There is also pain in right flank at diaphragm, and hurts when I breath. Upon puncturing the lumbar area, I experienced an electric strong pain in my right sciatic nerve that shot down radiating to my right foot causing my leg to kick once uncontrollably. I said oh I think you hit my sciatic nerve! When the Dr pulled back on the needle, the pain stopped, but the leg felt numb. The pain was not this bad yesterday, but now even with my very potent meds for chronic back pain, the pain is very bad. I am stuck in bed and starting to get head and neck pain too. I have some arachnoiditis in the lumbar area from four spinal surgeries at L5 - S1. The radiology Dr said my cord area is very narrow as it was very difficult to obtain CSF. The puncture was done prone for the xray and then I had to roll onto left side for collection of CSF. It was barely dripping out. It took about 30 min. to get 6 cc. Is this pain normal?? I see many people on the site talking about the needle hitting the sciatic nerve, and the xray tech said it happened a lot at the facility I went to....Is it common that the sciatic nerve is hit upon needle entry? I would appreciate any advise as it is Sunday..... everything closed but ER. Doctor: Hi,Your symptoms suggest that you have experienced two side-effects of the lumbar puncture. Firstly, you have experienced some nerve irritation or damage during the procedure. This is likely to recover but may take a few days to show an improvement. It is a recognised occurrence during a lumbar puncture. Secondly, the head, neck and body pain that has started is also due to the puncture but is not nerve damage. It is due to leakage of CSF through the hole made by the needle and, again, is a recognised problem with a lumbar puncture. My suggestion is that you continue to take your pain meds. Lie down and rest as this may relieve the head and neck ache to some degree. Keep well hydrated by drinking plenty of water. Caffeinated drinks will also help with the pain. These measures are likely to ease the pains.If you do not feel significantly better in 24 hours, I suggest you contact the clinic where you had the procedure done for further advice. I hope you feel better soon.Regards,Dr K A Pottinger,MBChB FRCA"
},
{
"id": 17340,
"tgt": "Suggest treatment for high blood pressure",
"src": "Patient: I have had 2 coronary bi-pass surgerys. My blood pressures have been running: 196/96, 177/105, 171/93, 169/92, etc for the last several days. The last check was 138/94. I have an appointment with my cardiologist on 6/2/10. Should I contact him sooner? Doctor: Hi, Yes, definitely your BP reading is high your medicine needs to be optimized. You should definitely see your doctor soon. Hope I have answered your query. Let me know if I can assist you further. Regards, Dr. Sameer Maheshwari, Cardiologist"
},
{
"id": 191400,
"tgt": "What is the lethal dosage of Insulin?",
"src": "Patient: Just started keeping track of my dad s sugar levels. It is exceedingly high. How much insulin is too much in a day? He had 55 units this morning and I gave him another 10 about an hour ago. His sugar before the 10 units was 208, I just took it and it s 299??? Doctor: Hello,Usual insulin dosage is one unit per kg per day for regular insulin. But insulin can cause low sugar coma or death in doses which are excessively high. This high dose depends on person to person. Fatal doses record as 400 to 900 units for a person. Hope I have answered your query. Let me know if I can assist you further.Regards, Dr. Varinder Joshi"
},
{
"id": 143070,
"tgt": "What causes right sided numbness inspite of normal scans?",
"src": "Patient: ive had knumbness in my rightside body for over a year maybe longer my right side of my face is getting worse and my tinitus has also gone to an extreme its so painful also headaches on same side i went for an mri and got letter today it says my scan is essentially normal and that there is a none specific dot in the white matter of the right parietal lobe but says dought it is of significance im at my wits end to be honest whats does it mean Doctor: Hi, Welcome to HealthCareMagic.com I am Dr.J.Mariano Anto Bruno Mascarenhas. I have gone through your query with diligence and would like you to know that I am here to help you.Right Sided Numbness, Tinnitus and headaches are suggestive of hemianaesthetic migraine. You need to consult a Neurosurgeon or Neurologist who has experience in treating this condition Hope you found the answer helpful.If you need any clarification / have doubts / have additional questions / have follow up questions, then please do not hesitate in asking again. I will be happy to answer your questions. In the future, for continuity of care, I encourage you to contact me directly in HealthCareMagic at http://bit.ly/askdrbruno Best Wishes for Speedy Recovery Let me know if I can assist you further.Take care."
},
{
"id": 167304,
"tgt": "What causes vomiting in baby?",
"src": "Patient: My 10 months old baby boy has only finely pasted rice/dal and vomits a lot he is lean, Iam worried about him. I still feed him but it is very low. shall I feed him lactogen milk or general buffello milk that we get at home. Iam worried about his vomits. but he is active. Doctor we consulted said its normal till now but noe he says we need to do scanning and urine tests to know the reason about his vomits. He said he is anemic too. please suggest me what do I need to do. Thanks in advance Doctor: Hi...by what you quote I feel that the infant is having severe form of GER (gastroesophageal reflux). Your son will need - 1. Antireflux therapy - Proton pump inhibitors + antiemetics2. Always make him lie down in a propped up position - as in the care seat. She should not be sleeping completely flatly.3. Avoid heavy feeding and feeding more times and in small aliquots.He needs expert gastroenterology consultation.Regards - Dr. Sumanth"
},
{
"id": 104980,
"tgt": "Itchy dry rashes on the body. Difficulty in breathing, swollen feet. Responds to zyrtec. Is this life threatening?",
"src": "Patient: hi, my name is jawaria and since september i started to get these rashes that looked like bruises, very itchy, i scratched them a hell lot and now they are at my back, arms, neck , chest and also now on my torso, i m very scared as i dont know what it is, they are dry but don t go away, they are faded purple and the ones on my chest are faded red, i do not have any food allergies , once in a while i get difficulty breathing and my feet swell up, for that i take zyrtec and it goes away, please do help me as i do not know what this is or whether this is life threatening or not, i don t even have insurance and i m scared how will i treat this Doctor: Hello Jawaria, I am sorry to note your present state. The condition that you describe is consistent with urticaria, and very itchy red hives that fade away but reappear within a few hours. Some attacks on the skin are raised and known as angioedema due to the chemical called histamine released from mast cells. These cells are present in the lips, tongue, skin, chest, gut etc and therefore wherever histamine gets released, you get the specific organ related symptoms. Another condition called urticarial vasculitis definitely needs to be excluded and moreso when there is no response to antihistamines. 80% of people with urticaria do not have any allergies which is the case with you. Long-acting non-sedating antihistamines like fexofenadine 180mg once daily plus/minus cetirizine 10mg at lunchtime/evening is required for 3 months. Occasionally the treatment may need to be longer if there is a recurrence. Tests such as CBC, LFTs, renal profile, TSH TPOabs are standard investigations to exclude a secondary cause. Most people go into remission so do not worry. Thanks."
},
{
"id": 10808,
"tgt": "Suggest medication for hair loss",
"src": "Patient: Hi I am 28 years old woman...I am suffering from severe hair loss..my hair is long till my hip but very very thin...I have been using Himalaya herbal shampoo for the past six months, but still no effect in the sense there is no regrowth of hair..Please suggest me a remedy Doctor: Hi ma'am and welcome to HCMFirstly we need to consider the reason for hair loss. Do note if there were any factors before the hair loss started. Most common are dandruff, hormonal(menstrual,thyroid etc),pregnancy, any lifestyle or residence change, stress, any history of recent illness like typhoid,low iron or nutrition levels, any chemical procedures, etc.Females also may be affected by a genetic type if it runs in the family.Also do note if you have hair loss anywhere else in the body.These causes need to be considered and treated first as they affect hair growth.You need to take a nutritious diet, well balanced, with enough fluids and rest. Avoid stress.Consult a dermatologist regarding the type of hair loss you are facing, and also get some tests to check your hemoglobin, iron and thyroid levels.For now you may start hair supplements daily containing biotin. Also apply a hair serum like Recute or Keratex. They need to be continued at least for 2-3 months.You will notice the hair fall reducing and new hair growth slowly. It depends on your hair cycle.Be sure to see your doctor regarding the possible cause of your hair fall.Further treatments will be advised by your consulting doctor after observation, depending on your results.Hope this guides you"
},
{
"id": 181802,
"tgt": "Suggest treatment for swollen lymph nodes and tongue",
"src": "Patient: I've been sick for roughly 3 months. My symptoms have increased in severity to the point that I have obvious swollen lymph nodes under my tongue, my tongue is swollen to the point that eating food and even swallowing and often breathing is difficult. I'm now having massive headaches, pains in my mouth, jaw, face, etc. My face, eyes, and even hands, feet and joints are all swollen. The headaches and pressure in my chest are growing increasingly difficult to handle. Ive been to the ER before and they treated me for essentially a sore throat possible brought on by acid. Told me to take Malox and sent me home. I truthfully feel like I'm dying. My equilibrium is off to such an extent that I walk as though I'm intoxicated even though I don't drink alcohol at all. So many things wrong. how do I get help? A dr friend told me that he is afraid that I have a lymph node blockage or possibly more. How do I get the ER dr's to listen to me? Doctor: severity of symptoms persistent for 3 months is definitely alarming. kindly do describe any pre existing ailments. you need blood investigation (CBC) and an x-Ray to know extent of the infection, based on these investigation results further investigation can advised and appropriate treatment will be instituted."
},
{
"id": 52322,
"tgt": "What causes nausea and headache while having fluctuations in the SGPT and bilirubin levels?",
"src": "Patient: My SGPT levels deceased from 68 to 53 but my bilrubin levels are elevated from 2.34 to 3.10,should I be worried,AST & ALT are all normal & m not feeling nauseous, fever or headache it s just from 4 days m also suffering from loose motion but it s only for 1 time in a day. Doctor: Hello, The bilirubin level can fluctuate in case of liver hepatitis. You need to investigate with a viral profile for hepatitis and ultrasonography abdomen to guide you further. Kindly revert back with that two reports. Meanwhile, avoid high fatty foods and take more fruit juice. The sugar cane juice, apple juice, lemon water etc beneficial in reducing your jaundice. Hope I have answered your query. Let me know if I can assist you further. Regards, Dr. Parth Goswami, General & Family Physician"
},
{
"id": 200825,
"tgt": "What causes large painful boil on scrotum?",
"src": "Patient: My husband has had a very large and painful boil on his scrotum for about a week now. We went to the urgent care on Wednesday last week. They said it looked as though it was ready to pop, so they gave him anti-biotics and something for the pain. The next day it did pop, and some pus/blood came out. But now for the past 4 days, he has been draining consisitantly. So much that he has to wear a maxi pad in his boxer shorts to prevent his pants from getting wet. However, today, the blood and pus is thick and clotting. So much so that if he squeezes the boil, he literally has to pull the pus out like a string. It is very thick and long. The boil itself is still there, very large and hard. About 3 inches long. Is this actually a boil and will this improve or should we go back to the doctor. He is being stubborn today and wouldn t go to urgent care but I am very concerned. Doctor: Thanks for asking in healthcaremagic forumIn short: Daily dressing with removal of pus is necessaryExplanation: A boil can cause swelling and pus formation which can burst open discharging pus. AS you are taking antibiotics, it is better to continue it for further 2-3 days and daily dressing by yourself or by medical personnel is necessary for removal of pus and for early healing. Do not shy away, visit a doctor again."
},
{
"id": 116875,
"tgt": "Should I be worried about low WBC and lymphocyte count?",
"src": "Patient: My recent blood work ..white blood count 3.4. Absolute Lymphocytes 646 both low .....What does this mean as I am never sick! I work in healthcare and my staff is sick a few times every year n I never catch anything. Should I ve worried??? My white blood cwt. has been low for 20 yrs at least. Doctor: Hi, dear. I have gone through your question. I can understand your concern. You have low wbc count. There are many causes of low wbc count. Even simple viral infection can leads to this. However if you don't have any symptoms then no need to worry. Because many healthy person having this count as normal variation. So don't worry. Just be relaxed. If you have any symptoms then consult your doctor and take treatment accordingly. Otherwise no treatment is required. Hope I have answered your question, if you have doubt then I will be happy to answer. Thanks for using health care magic. Wish you a very good health."
},
{
"id": 135475,
"tgt": "Suggest remedy for neuopathy",
"src": "Patient: YES HI Im 45 and have what my doctor beleives to be Neuopathy Drop everything, I have hot, burning, loss of feeling in both hands,Its so diffacult to hold on to anything, Even my writing has become scribble, sould I adsk about a tutot to help me with this handy cap? And how long will I have to live like this, is this perment? Thank you Doctor: You could be Diabetic. Get tested for the same. Take Tablet Pregabalin 75mg at night for a month. Other causes could be multiple sclerosis, slip disc in the cervical spine of the neck. You need to get a spine MRI. Consult your Neurosurgeon."
},
{
"id": 198722,
"tgt": "Why my semen is yellow and thin?",
"src": "Patient: Ok I recently have noticed for the last two weeks or so I've had dark yellow, thin semen. I've never had this before and am wondering what it is most likely. I keeping reading threads saying it could be an infection but I have yet to feel pain. What are your thoughts? Doctor: HelloThanks for query.Yellow and thin semen is mostly due to infection of either Seminal Vesicles or Prostate .You need to consult qualified Urologist for clinical and digital Rectal examination and get following basic tests done 1) Routine urine and urine culture 2) Semen examination and semen culture .3) Ultrasound Scanning of abdomen and pelvis take antibiotics like Doxicycline and anti inflammatory drugs like Diclofenac twice daily later on switch on to appropriate antibiotics as per culture report.Ensure to drink more water .Ensure to avoid sexual activities till it gets cured .Dr.Patil."
},
{
"id": 185552,
"tgt": "How to cure pimple on the left side of the mouth?",
"src": "Patient: I have a pimple on the left inside of my mouth cheek, It only noticed it this morning when I was brushing my teeth.It is otherwise not sore unless touched.it looks like a 1-2cm narrow piece of skin, but its hard when touched.I am 30 and have neer had things/sores in my mouth except for self biting.the pimple is actually where I usually find that I bite myself & get sores. Doctor: HelloI have gone through your query.it might seem like a case of oral fibroma.oral traumatic fibroma is usually due to chronic irritation such as: cheek or lip biting or rubbing from a rough tooth. The common location for an oral fibroma is on the inside of the cheek where the upper and lower teeth meet. It is usually due to chronic irritation such as:cheek or lip bitingirritation from a rough/sharp toothdentures or other dental prosthesesOral fibromas do not develop into oral cancer.A biopsy may be taken to exclude other conditions or to remove the lesion.The only option is surgical excision of the fibroma and eliminate the source of the irritation.if i were your treating doctor i wud suggest you to visit a dentist for the exact diagnosis, cause and treatment of the lesion like smoothning of any rough tooth surface and removal of the fibroma.Hope this ans helpsRegardsDr. Shesh"
},
{
"id": 59233,
"tgt": "Pain and strange sensation in chest, back pain, had a gall stone. Gall bladder removed. Further?",
"src": "Patient: Hi strange pain /sensation in middle of chest (upper stomach?) it feels like I have either been punched or my insides are being sucked in in that area..I also have pain sensation in my back area....it sort of reminds me of when I have had a gall stone attack but not full blown as I had my gall bladed removed 5 yrs ago...any ideas? Doctor: Hi and welcome to HCM. It sounds like stomach issue. All this is caused by high acid secretion which is found in gastritis, GERD or hiatus hernia. These disorders can cause such difficulties and you should do gastroscopy to evaluate this. Symptoms that are experienced in such conditions are pain and tightness in upper stomach, bloating, breathing difficulties, heartburn, feeling of lump and sore in throat, gurgling sound in throat, fatigue and loss of apetite. Symptoms are aggravated after meal, long sitting, leaning forward,stress and anxiety. In the beginning this is treated by changing dietary habbits. You should avoid alcohol,coffee,smoking,fats and fast food, carbonated drinks and eat more milk and milk products, fruit and vegetables,boiled food and natural beverages and teas. 1 hour after meal you should not sit or lay. Also lay with elevated upper body. If you are overweight you should do often exercise. Secont option is pancreatic disorders, you should do certain tests to verify is tehre any kind of pancreatitis. Wish you good health."
},
{
"id": 37165,
"tgt": "How to treat pneumonia?",
"src": "Patient: I am worried that I might have pneumonia . I have been on antibiotics since Thursday, but my cough is getting harsher And causing My chest to tighten up and be painful. I am short of breath that could be compared to how u feel when u ran a race and I am just laying in bed. I Do feel rundown and weak. Doctor: Hello,I understand your concern.I am Dr. Arun Tank, infectious dissensitivityialist, answering your query.In my opinion you should start with antibiotics like cefixime under your doctors guidance.Also take the sputum out and sent it to the doctors for culture and sensitivity.Thereafter switch the antibiotics to the newer sensitive antibiotics.Taking the full course frees you from the infections.Please do respiratory exercise also, it will help you clear up the infections early.I will be happy to answer your further concern, you can ask me on bit.ly/DrArun. Thank you.Dr Arun TankInfectious diseases specialist."
},
{
"id": 192110,
"tgt": "Can masturbation affect sexual health?",
"src": "Patient: Dear im masturbating since i was 15 years old and now im 29 mean im doing this since last 15 yearsnow i feel that im sexually week because of thiscan u suggest that how can i control my premature masturbation does masturbation effect our ejaculation period Doctor: Hi, Masturbation is an innate response to increased libido. But when we overdo it, it causes negative effects on our body like, reduced testosterone levels, weakened central nervous system which together leads to premature ejaculation and erectile dysfunctions. It's advisable to completely quit masturbation to reverse its negative effects on your body and a healthy lifestyle that includes a nutritious diet full of milk nuts fruits and vegetables , and regular exercise and a good amount of sleep will get you back on track. You should expect a gradual improvement in your sex health as 15 years of damage does take some time to restore. Take care. Hope I have answered your question. Let me know if I can assist you further."
},
{
"id": 125970,
"tgt": "How can severe pain and swelling in the knees after injury be treated?",
"src": "Patient: Yesterday I tripped on the sidewalk and crashed onto the concrete. The first thing to hit was my knee cap it hurt like crazy but I was able to get up and walk home. It swelled pretty significantly and I could hardly walking on it within a few hours. I took two Alene and have iced it. Today is better but am limping and if I torque it a little there is shooting pain. Still icing it and I have braces from a former acl tear (years ago) and a knee wrap. Should I give it more time? Is it important to see a doc sooner rather than later? Doctor: Hello,The severity of the symptoms you are experiencing, suggest that you may have suffered a fracture due to the fall. Since your pain and swelling have increased, and you have not felt any relief even after icing and rest it is advisable that you get yourself checked by an orthopedic surgeon as soon as possible to prevent any further damage. It can also be a case of ligament or meniscal injury and can diagnose with the help of an x-ray and MRI of the knee joint. Treatment will depend on the diagnosis.Hope I have answered your query. Let me know if I can assist you further.Regards,Dr. Nazma.amman"
},
{
"id": 18855,
"tgt": "Suggest treatment for rapid heart rate",
"src": "Patient: I'm a 50 yr. old woman. Last night after going to bed, my heart just wouldn't calm down. It was racing and felt like it was going to pound out of my chest continuously. It was very hard for me to go to sleep. This morning on my way to work, I thought I was going to black out.....started getting very dizzy and had to pull over. I still don't feel right. Doctor: Hello and Welcome to \u2018Ask A Doctor\u2019 service.I have reviewed your query and here is my advice.Fast-beating, racing or pounding heart is highly likely was an episode of heart palpitation. A wide variety of factors may cause heart palpitation. Certain medications, excessive exercise, stress or medical conditions (heart problems, endocrine diseases) may trigger heart palpitations.If the episode of palpitation is a few seconds to a few minutes duration without any other symptoms, then it does not need medical attention. But your heart palpitation is longer in duration and accompanying with dizziness which requires emergent medical evaluation.For the diagnosis purpose, your doctor may prescribe an Electrocardiogram (ECG), and/or continuous heart monitoring (Holter monitoring). The treatment is based on the underlying cause of the disorder. Please, contact your Doctor or visit ER for medical evaluation of your condition urgently.Hope I have answered your query. Let me know if I can assist you further.Regards,Dr. Malik Amonov"
},
{
"id": 180319,
"tgt": "What causes yawning while crying or participating in some activity?",
"src": "Patient: Hi there, why does my 4yr old yawn when she cries or is at a swimming class? She gets 11.5hrs sleep and doesn t behave in tired manner. Sometimes she says she s tired when she needs to participate in an activity such as tennis or playball, which I might add her instructors say she s incredibly talented for her age. Is she just being lazy, seeking attention, has a medical condition or being a typical 4yr old? Doctor: hi this is quite normal in kids to yawn during activities, there is nothing to worry about. just see that she gets continuous sleep in the night from 8pm onwards and don't allow them to watch tv in the night.hope this is helpful to you, rate it if you like it. take care"
},
{
"id": 122012,
"tgt": "Suggest treatment for swollen ankles and calves with deep pink rashes",
"src": "Patient: Hi I am homeless and living I sit in a mostly unheated car at night. Then spend times in libraries during the day my ankles and lower calf are swollen and I have a deep red or pink rash with some pain what is it and what do I do no insurance dr or money XXXX Doctor: Hello, I would explain that your symptoms could be related to the hanging position of your legs during all the day. Another possible cause would be a fungal infection, as your hygiene is not the best lately. for this reason, I recommend trying to hold your legs upright position as often as you can and try to wear compression stocks, and see if they help improve your situation. Hope I have answered your query. Let me know if I can assist you further. Regards, Dr. Ilir Sharka, Cardiologist"
},
{
"id": 57479,
"tgt": "23b weeks pregnant, itching all over the body, intense at night, udiliv300, swelling of liver, non-functional gall-balder, effect of medicine on fetus",
"src": "Patient: Hi! im 23 weeks pregnant and im suffering from itching all over the body ,it gets intense at night. Doctor advise me to take udiliv300 one tablet at night because my liver is swelled and gall bladder is not working well.i want to know that is this problem effect my baby,s growth or in anyway. Thanks Doctor: HIThank for asking to HCMI really appreciate your concern, just stop taking the Udiliv tab, your itching problem could be due to this, again it is not good to take such medicine during the pregnancy, for the swelling of liver and non-functional gall bladder, the Udiliv is questionable, hope this information helps you have good day."
},
{
"id": 208178,
"tgt": "How to recover from akathisia?",
"src": "Patient: Daughter has akathesia caused by reglan and compozine. This has been on going for one month. it has caused disassociation, anxiety, memory loss, tremors, out bursts of anger. looking for help on how to treat her. quality of life poor, never had issues until those meds were given. Doctor: Hi,Thanks for choosing Healthcaremagic.I have read your query carefully and understood that your daughter had taken Reglan (Metoclopramide) and Compazine (Prochlorperazine) (??Dose and duration) and now she is having symptoms suggestive of akathisia since last one month. I would like to let you know that both the drugs are dopamine antagonist and are commonly used for nausea and vomiting. And akathisia is a rare side effect of both of them. It can be treated with B-blockers (like propranolol 20-40mg/day) and a benzodiazepine (like clonazepam 0.5-1mg/day).But the patient needs to be evaluated by a psychiatrist before starting those medications to identify the cause for its onset in the patient and the cause for its prolonged duration of side effect.Hope I answered your query, let me know if you need any further information.RegardsDR. ASHUTOSH SINGH"
},
{
"id": 155704,
"tgt": "Is bone cancer a common complication of prostate cancer?",
"src": "Patient: My 90 year old uncle and fallen and fractured the tibia and fibula. He also has advanced prostrate ca which is not being treated. He had surgery on the leg and it is casted . Now I m worried about poor circulation and infection. Is bone cancer a common complication of prostrate cancer? Doctor: Thanks for your question on HCM.Prostate cancer can commonly metastatize to bones especially thoracic vertebrae and long bones like femur, tibia, humerus etc.Metastases from prostate causes osteoblastic secondaries in bones.This makes bone weaker. They causes osteoporosis and demineralisation of bone. So bones become highly fragile and prone to fracture. Even slightest force can cause fracture. Bone metastases are very common after untreated prostate cancer."
},
{
"id": 142580,
"tgt": "What causes goose bumps when diagnosed with chronic spine pain?",
"src": "Patient: I have almost every disc is almost completely degenerated.... Stenosis, foramina Noramal (cant spell it) and other diagnosis regarding messed up back..... starts at base of skull and goes all the way to tail bone..... X race car driver..... I have recently been getting goose bumps from pain in upper back / neck region.... Is this normal..... I have been chronic pain patient over 15 years Doctor: Hello!Welcome on Healthcaremagic!Your complaints are related to chronic degeneration of the cervical spine. Unfortunately there is not too much to be done in this clinical situation. But, I would recommend performing a cervical spine MRI study to exclude possible spinal canal stenosis in this region, which may need surgery. A cervical collar can help reduce the pain. Hope you will find this answer helpful !Best wishes, Dr. Aida"
},
{
"id": 12752,
"tgt": "What causes recurrent red bumps on a different area each time?",
"src": "Patient: My daughter, seven years old, has been getting red bumps on different areas of her body, mostly torso and legs that look like small mosquito bites but they do not itch her. She has not had a fever and they are not painful. However she breaks out in one area with several, and by the time those start to fade she breaks out in another area. They tend to break out in a localized area then move to another part of her body. Doctor: Hi Dear,Understanding your concern. As per your query your daughter have symptoms of recurrent red bumps which seems to be due to folliculitis and it could be due to contact dermatitis. Need not to worry. You should avoid touching or pricking bumps. I would suggest you to apply topical antibacterial ointment. You should wear loose clothes and avoid sharing them with anyone. You should apply betnovate ointment on them 2-3 times a day. You should take course of oral antibiotics. Visit dermatologist once if symptoms keeps on persisting. Start treatment after complete examination and proper prescription. Hope your concern has been resolved.Best Wishes,Dr. Harry Maheshwari"
},
{
"id": 77213,
"tgt": "Could severe cough with yellow mucus be due to Bronchitis?",
"src": "Patient: Hi my condition is once or twice in a year particularly during September end i will start severe cough and cold and i will start to split yellow thick mucus and cough will be prolonged for 2-3 months and it will lead to difficulty in breathing and start astama. earlier my doctor had prescribed to take telecast plus tablet for this he told take it daily one tablet during these months.I am confused what is my condition?it is astama or bronchitis.Because after january onwards i dont have any of these syptoms Doctor: Thanks for your question on Healthcare Magic. I can understand your concern. By your history and description, possibility of allergic bronchitis (seasonal) is more. In my opinion, you should take combination of antihistamine (levocetrizine or fexofenadine) and anti allergic (montelukast), 15 days prior to season starts and continue till season ends. Along with this, also take inhaled bronchodilator (formoterol or salmeterol) and inhaled corticosteroids (ICS) (budesonide or fluticasone) for the same period. You will mostly improve with all these. Since all these are prescribed drugs, you should consult pulmonologist, discuss all these and get done prescription. Hope I have solved your query. I will be happy to help you further. Wish you good health. Thanks."
},
{
"id": 225287,
"tgt": "Will the contraceptive still be effective if I stopped taking last 4 pills of Qlaira and start Cilest?",
"src": "Patient: I am on Qlaira, currently on day 22 of 28 but i want to skip my period. I am in the process of changing my pill to Cilest but i am waiting untill I finish this month of Qlaira to change. I understand on Qlaira that if you do not wish to have your period you should not take the last 4 pills (2 dark red and 2 white) and start a new pack instead. If I were to not take the last 4 pills and start immediately on Cilest would it still work as contraception or should other contraceptive methods be used? Your assistance will be greatly appreciated. Doctor: Hi, Welcome to Health care magic forum. If you use the the new pack form the 1st day, you will definitely have protection, but to be on safe side it is better to follow the other methods, for 7days. from the next mont you can follow the plan as described,on the leaf let. Take more of green leafy vegetables, pulses, sprouts, and protein rich foods. Wishing for a quick and complete recovery."
},
{
"id": 38455,
"tgt": "What causes an abscess after an incisional hernia repair?",
"src": "Patient: In oct had incsional hernia repair with mesh implantIn nov had aspiration of 1000ccs of fluid In dec had 2 nd surgery due to abcess of fluid filled cyst is this common, how long will it continueam still wearing binder and drain tube Doctor: Hello,the answer is no. This is not common. You were unlucky enough to suffer from a serious complication of surgery. The skin protects the interior of our body from bacteria and other pathogens. Skin cuts may invite pathogens inside the body. That's why surgeons should be very careful and meticulous when preparing the patient's skin for incisions.Hollow organs like the intestine also harbor potential pathogens. Injury of the intestine may release those pathogens to the peritoneal cavity and cause infections. I don't know what happened in your case but peritoneal infections can be hard to eradicate. I can't tell how much time is required. Only your surgeon can estimate that. When the fluid stops coming out through the tube, you'll be very close. I hope you get better soon!Kind Regards!"
},
{
"id": 164366,
"tgt": "Is long term usage of Foracort inhaler safe?",
"src": "Patient: My 4 yr old daughter is allergy prone. She gets frequent cold, cough and associated wheezing. Especially every time we go outside. Her doctor has suggested to give her foracort inhaler for a few weeks. But is it safe for long term use as it s a steroid Doctor: Hi, welcome to HCM. Can understand your concerns. Long term steroid inhaler are given if symptoms of asthma are chronic. Take care."
},
{
"id": 196978,
"tgt": "How to confirm fertility?",
"src": "Patient: Home|FAQ\u2019s|Search Private AreaPasswordLogout [ Back to Listing ] Name [ Age ] / Sex:Mr VIGNESH.D.V [ 27 Y 1 M ] / MaleCustomer ID:44579 / 72372Sample Recieved on:13/01/2012 09:44Report Date:13/01/12 12:52Referred By:Dr. CHITRALocation:T Nagar TEST NAMERESULTREFERENCE RANGECLINICAL PATHOLOGYSEMEN ANALYSIS (WHO 1992)Period of Abstinence 4 days Mode of Collection Self .Liquefaction time > 90 MinutesColour Whitish Opaque Volume 4.2 ml> 2mlViscosity Highly Reaction Alkaline MICROSCOPIC CHARACTERSTICS Sperm Concentration 67 > 20 millionsMotility 50 %50 or moreMOTILITY GRADING (GR I +II=50%)Gr I Fast & forward progression 20 %Gr II Slow /Sluggish forward progression 20 %Gr III Non Progressive 5 %Gr IV Non motile 55 %SUPRA-VITAL STAINING (EOSIN Y)VIABILITY 65 %(Viable >75%)Agglutination if any Absent NilFructose (Qualitative) Positive PositiveSPERM MORPHOLOGY (WHO 1992) (Normal 83 %Head Defects 10 %Neck / Midpiece defects 4 %Tail Defects 3 %Amorphous Matter Nil Pus Cells Occasional /hpfRBCs Nil /hpfEpithelial cells Nil /hpfIMPRESSION Spermatogenesis in acceptable range with reduced motility REMARK -NA- Time of Collection 9.01 am ** END OF REPORT **\u00a92010 Primex Healthcare & Research Pvt Ltd. Home|FAQ\u2019s|Search Private AreaPasswordLogout [ Back to Listing ] Name [ Age ] / Sex:Mr VIGNESH.D.V [ 27 Y 1 M ] / MaleCustomer ID:44579 / 72372Sample Recieved on:13/01/2012 09:44Report Date:13/01/12 12:52Referred By:Dr. CHITRALocation:T Nagar TEST NAMERESULTREFERENCE RANGECLINICAL PATHOLOGYSEMEN ANALYSIS (WHO 1992)Period of Abstinence 4 days Mode of Collection Self .Liquefaction time > 90 MinutesColour Whitish Opaque Volume 4.2 ml> 2mlViscosity Highly Reaction Alkaline MICROSCOPIC CHARACTERSTICS Sperm Concentration 67 > 20 millionsMotility 50 %50 or moreMOTILITY GRADING (GR I +II=50%)Gr I Fast & forward progression 20 %Gr II Slow /Sluggish forward progression 20 %Gr III Non Progressive 5 %Gr IV Non motile 55 %SUPRA-VITAL STAINING (EOSIN Y)VIABILITY 65 %(Viable >75%)Agglutination if any Absent NilFructose (Qualitative) Positive PositiveSPERM MORPHOLOGY (WHO 1992) (Normal 83 %Head Defects 10 %Neck / Midpiece defects 4 %Tail Defects 3 %Amorphous Matter Nil Pus Cells Occasional /hpfRBCs Nil /hpfEpithelial cells Nil /hpfIMPRESSION Spermatogenesis in acceptable range with reduced motility REMARK -NA- Time of Collection 9.01 am ** END OF REPORT **\u00a92010 Primex Healthcare & Research Pvt Ltd. My husband went for sperm analysis ans the report came to be as follows - liquifaction time - >90mincolour - whitish opaquevolume- 4.2ml (done after 4 days)viscosity - highlyreaction - alkalinesperm concentration - 67motility - 50%viability - 65%GR I fast / fwd progression- 20%GR II slow/sluggish fwd progression - 20% GR III non progressive - 5%GR IV non motile - 55%agglutination - N/Afructose (qualitative) - positivenormal - 83%head defects - 10%neck - 4%tail - 3%pus cells - ocasional / hpfNow i want to know about the analysis. My husband drinks. He has hernia problem. He is diabetic patient, has BP. He is aged 27.I want to know detailed explanation about this analysis. whether he is fertile or not. Regards, Vaishnavimail me @ YYYY@YYYY Doctor: yes he is fertile,this test is normal.. the count is 67 mil/ml and the normal level starts from 15 mil/ml.. motility is considered normal starting from 40%.. and normal forms is considered ok if above 4% and he has it above 80%"
},
{
"id": 164389,
"tgt": "How to treat buttock pain in a 4 years old child?",
"src": "Patient: hello my daughter is nearly 4 years old and recently keeps complaining of pain in her bum during the day and at nite and theres no sign of redness or anything wrong, looks normal but its drivin me mad now not knowing wat 2 do with her only been putting on sudocrem, wat do u think it mite b? or is there anything else i cud do 2 help her with the discomfort. Doctor: Hi...by what you say, I feel that this could be a worm infestation. This is even more possible as you say that the local examination is normal.You can use Albendazole at the appropriate dose. This is a prescription drug and you will require a doctor's prescription for this.Regards - Dr. Sumanth"
},
{
"id": 60402,
"tgt": "I have a spot on my pancreas, what does it mean?",
"src": "Patient: It was discovered I had a spot on my pancreas during a CT.I had HepC-ws in treatment w Pegasus for 4 mos. They wanted me to do a MRI-I didn t have health insurance and frankly I ignored theor advice aboit MRI. I have no pain or any unusual symptoms-I am now rescheduled for MRI-applying for health insurance. I am NOW VERY worried. Doctor: Hi, Welcome to HealthcareMagic Forum. Since you do not have any symptoms like pain or fever it can not be an abscess. MRI has to be done to rule out any benign or malignant condition. Please do not neglect and let the doctors do their job. Since spot on pancreas is an incidental finding on CT scan and there is no clear cut relation between HCV infection and pancreatic cancer I advice you to undergo MRI and follow the advice of your doctor. Take care. Wish you good health."
},
{
"id": 170564,
"tgt": "What causes red rashes on the abdomen of a 2 year old?",
"src": "Patient: Our two year old had a pediatrician visit yesterday and was given 100% health. Yesterday afternoon she was complaining about her tummy hurting. Upon viewing her upper stomach there appeared to be to red marks about index and middle finger long. We thought she had bumped something or scraped it. Gave her a gel pack in paper towel to ice it. Then at bed time when we viewed again it was bold red and appeared to be like burn marks. We have nothing hot she could have gotten into. Unsure what this is or how she got it. Neosporin a day later no visible change. Help??? Doctor: Brief answer:Appears to be a insect bite. Detailed answer:Hi, welcome to HCM. Your child was perfectly normal a day ago as examined by pediatrician. Now, suddenly he has developed rash marks over abdomen. This appears to be insect bite which has trail marks. In my opinion, you should give hydroxizine syrup PO 5 ml three times a day for 3 days and apply neosporin powder locally. I hope this will help you. Wishing your child good health. Take care. Regards:Dr Deepak Patel, MD Pediatrics"
},
{
"id": 73843,
"tgt": "What causes severe chest pain in case of chest contusions?",
"src": "Patient: Good Afternoon,I was hit by a car 3 days ago on my left side( was walking) X rays were taken and they show chest and upper extremity contusions. I just feel the pain is getting worse in the last 1 day in my chest . It is hard to sit or lay down, standing seems to be the most comfortable position. Sneezing cause sharp impossible pain. Is this normal? I do not want to rush to the doctor or emergency room. Thank you for your advice in advance! Doctor: Hello dearWarm welcome to Healthcaremagic.comI have evaluated your query thoroughly .* There are different possibilities for this pain as - muscle pulling - ligament inflammation or irritation due to contusion - nerve endings irritation - hematoma related positional pain - secondary rib inflammation or pleural involvement .* Recommendations for better recovery- Alternate application of ice and heating pad .- Local application of analgesic ointment or spray will help a lot .- Tab. advil or other pain killer 3 times a day after meals .- Avoid sudden bending or weight lifting .Hope this will help you for sure .Regards ."
},
{
"id": 63742,
"tgt": "What is the lump between the buttocks?",
"src": "Patient: Hi, i recently found a lump in between my buttock over a week ago. its almost like skin hanging out , it feels swollen and is painful. i tried pushing it back in but it didnt work, im really scared , is this some form of cancer? thank you for your help Doctor: Hi, dear I have gone through your question. I can understand your concern. You may have some hemorrhoid ( piles ). You should go for examination. Take plenty of water and high fiber diet. Avoid constipation. For pain relief local anesthetic gel like lignocain can be used. permanent treatment of choice is surgery. Consult your doctor and take treatment accordingly. Hope I have answered your question, if you have doubt then I will be happy to answer. Thanks for using health care magic. Wish you a very good health."
},
{
"id": 165102,
"tgt": "How can fever, chest congestion and stomach pain in a child be treated?",
"src": "Patient: I have a 4 year old little girl who has a high fever, chest congestion, bad cough, she was vomiting yesterday, complained of cheek and ear pain, but the pain has gone, has terrible stomach ache, won t eat anything, won t walk anywhere, and she refuses to take anykind of medicine, I ve slipped some grape tylenol into some juice and she drank a little but not enough to be effictive I don t think. Is there anything I can do/give her to make her feel better? or does this sound like something serious that needs to be seen right away? Doctor: Dear parent,by the description it seems your daughter is suffering from respiratory infection. If she is able to take steam then give her steam three to four times, preferably by nebulizer. If she is hyper active don't try steam.Otherwise also by description of her condition its better to take a medical advise."
},
{
"id": 93366,
"tgt": "Stomach ache, light headed, fatigue, nausea, swollen veins. What is happening?",
"src": "Patient: helloi am feeling pain in a stomach all over mostly in my diaphragm and outer stomach and also in my pelivc region and on the bottom i feel very tight pain and i am having symptoms such as light headed, weak fatigued, nausea. i also have very swollen veins in my ball sack and a bunch of viens when my sack is warm and hangs a liitle pain i feel in the bottom of my sack i have felt my sack for lumps and i have found none.when i poop it is diarrea and some times normal have not bleed when going poop or pee. dont know whats going on pain is very discomforting. Doctor: Hi ! With the history you have given, it seems that you have got an intestinal infection possibly some kind of food poisoning. This is why you have got diarrhoea, nausea, and pain in the abdomen. You have to take appropriate antibiotics on consultation with your family physician.Another cause for the pain in the abdomen could be due some infection of the testis in yous sack which causes some localised pain as per your history. The bunch of veins what you are feeling is a condition known as Varicocele of the testicular veins, which may cause heaviness in your sacks. I am not sure if you have already completed your family or not. Vacocele is sometimes incorporated as one of the causative factors for sterility. I would suggest you to see a Surgeon/Urologist who may advise you to go for a doppler ultrasound of the testes for proceeding further in the treatment.Wishing you an early recovery."
},
{
"id": 30415,
"tgt": "Suggest medicine for penile infection",
"src": "Patient: Hi, may I answer your health queries right now ? Please type your query here... i have a bag of worms on my leftside of my penis i had so much pain yesterday everytime i move my body its hurts but today this morning i woke up my underwear was wet with some i think its blood+water smells pretty bad wasnt really red though, and pain had much relieves. what medicine i should take doctor Doctor: HiA bag of worm on the left side of your penis could suggest varicosis or a local skin infection in that area.A smelling watery/bloody liquid from it suggest an infection that needs a course of antibiotics to go.I advise you get it examined by a Doctor immediately to know the next step to take.Wish you a quick recovery"
},
{
"id": 85520,
"tgt": "Am I having side effects from Oflacin and Ornidazole?",
"src": "Patient: I have been diagnosed with amoebasis. given oflacin and ornidazole tablet for 2times for 3 days. For 1 day everything was fine. 2nd day stomach pain and loose stools started. what could be the cause. Could this be side effect and can I stop taking medicines. Doctor: Hello, The symptoms seem to be related to side effect of Ornidazole. I suggest using an over the counter option such as Imodium for the diarrhea. I also suggest using daily probiotics and eating a healthy diet.Hope I have answered your query. Let me know if I can assist you further.Regards,Dr. Dorina GurabardhiGeneral & Family Physician"
},
{
"id": 10269,
"tgt": "Suggest better treatment for hair thinning",
"src": "Patient: I am 37 yr old male. Suffering from hair thinning on top-rear and top. I fear that if it goes like this I may have bald patches on these area after 2-3 years. I have read about Minoxidil 5%. Is it right for me ? What is better a solution or gel ? which brand is available in India ? Pls help. Doctor: Hello and Welcome to \u2018Ask A Doctor\u2019 service. I have reviewed your query and here is my advice. I have gone through your complaints and Minoxidil 5 percent will be effective in your hair loss. You can go for Tugain gel on the affected areas twice daily. Hope I have answered your query. Let me know if I can assist you further."
},
{
"id": 179964,
"tgt": "Suggest suitable medication for deworming in children under 5 years",
"src": "Patient: Hi, both my 2 yr daughter and my 3 and a half yr son are due for the regular deworming (& Vit. A). i have just procured Albendazole, for them, but have noticed that the enclosed info sheet contains a precaution that the drug has not been studied in children under 6 yrs of age. what should i do? Is there a more suitable drug for their age? Doctor: Albendazole can be used freely in any child above the age of 1 year as there are no serious safety issues. Each of your children will need 10 ml once."
},
{
"id": 102532,
"tgt": "Is it safe to take montair lc for long duration of time?",
"src": "Patient: My daughter who is 12 years old has been asked to take montair lc (cipla)(Montair-Lc Montelukast 10 mg, Levocetirizine hydrochloride 5 mg) once a day for 2 months and after that when needed for her allergy problem. Is it safe to take the drug for such a longer duration? Are there any side affects of the same? Doctor: Hello,It is my pleasure to help you,You can give this medicine as long as recommended by your doctor even if you feel better.This drug is considered as medication for long-term control of asthma.Like any other medicine it also has side effects, but the advantage over weighs the side effects.Most common side effects are increased bleeding behaviour, Mood related changes, diarrhoea, dyspepsia, nausea, vomiting, dizziness and drowsiness.Hope I have answered your query.Thank you."
},
{
"id": 45851,
"tgt": "Suggest risk with kidney reflux with scars",
"src": "Patient: My daughter has kidney reflux in her left kidney, she is 5 years old. She has severe kidney scarring and her kidney was functioning at 32 % last August, since then she has had about 5 kidney infections - should we be concerened about her future kidney functioning? Will this result in problems later in life? Doctor: Hello and Welcome to \u2018Ask A Doctor\u2019 service. I have reviewed your query and here is my advice. Long standing reflux can result in scarring of the kidney and later kidney failure. It has to be corrected surgically to prevent further complications including scarring and kidney failure. Consult a urologist and most probably she will require surgical correction like pyeloplasty. Hope I have answered your query. Let me know if I can assist you further."
},
{
"id": 140749,
"tgt": "What is the treatment if half of the face cant feel anything ?",
"src": "Patient: hello, My name is Daniel, I have been having problems, with my right eye and low jaw, just half of my face, sorta of can t feel it, the right side I tried to whistle like I always do, but now I can t my lips like goes to the side is there something I can do, I have cure or treatment... Doctor: Hello, Judging from your description it sounds like facial nerve palsy, otherwise called Bell's palsy. In most cases it is benign and most patients improve spontaneously over several weeks. Medication with steroids and antivirals may help regeneration if started in the first few days. However, in some cases, it may be in the setting of a neurological condition or metabolic issues like diabetes. So you must be checked by your primary physician or a neurologist for other possible signs which would warrant MRI imaging as well as testing for diabetes. As I said in most cases it is isolated and improves spontaneously with tests turning out normal, but you should be seen to confirm the diagnosis. Hope I have answered your query. Let me know if I can assist you further. Take care Regards, Dr Olsi Taka, Neurologist"
},
{
"id": 120609,
"tgt": "What does the esr regarding multiple myeloma indicate?",
"src": "Patient: I am 36, 5 feet 4 inches, 55 KG, male and was diognosed with Multiple Myeloma in February 2011. Before treatment my ESR was 92mm and after six cycles of Chemo therephy (with Valcade), it came down to 4mm. That was three months back. I got a ESR test done yesterday and it is 13 mm. Please advice. Doctor: Hello, You should not worry at present.Your present ESR mark is under normal limit.If it continue to rise then you should be concerned.I will advice you to get it repeated after about 20 days. Hope I have answered your query. Let me know if I can assist you further. Take care Regards, Dr. Mukesh Tiwari"
},
{
"id": 137703,
"tgt": "Suggest treatment for a blood clot in the arm",
"src": "Patient: I have a blood clot in my upper arm that I got as the result of an intervenous hook up. It is in a surface vein. I am 64 years old and have no health issues at all. I need to have rotator cuff surgery. They put me on blood thinners for the clot. Now a blood Dr. said I cannot take my permpro because she said it will interfere with the blood thinner. Is this true? I have been taking prempro for 10 years. I have tried to quit taking it a couple of times, but I have break through bleeding when I do. Is this normal? Doctor: Dear Patient,Are you taking blood thinning agents just for a clot in your superficial vein, rather than giving your body systemic bad effects of drugs that can be surgically removed. if your body has a propensity of making regular clots than off course you need to take these blood thinning drugs. So be certain of that.secondly your doctor has rightly advised you to stop permpro, along with blood thinning agents.Hope that answers your queries,ThanksDR Narender Saini"
},
{
"id": 29743,
"tgt": "What causes itchy scalp in spite of good hygiene?",
"src": "Patient: we recently movedinto our new home.its very nice buti haveitchyscalp and skin found little coffee grounds here and there . had the termite guy come back but , no termites in house/ but my scalp is raw/ I wash hair every day but I cant get rid of this Doctor: Hi,Thanks for your question.Itchy scalp with dandruff mostly due to fungal infection.you can use ,ketoconazole shampoo3times a week for 4 wks.Hope your problem will solve,wishing your good health."
},
{
"id": 63671,
"tgt": "Suggest remedy for lumps on testicles",
"src": "Patient: Hi I have a lump which appears to be connected to my testicles via some tube(?), 3 days ago I was hit in the testis with a ball and this appeared a day later. I plan on getting it looked at in a few days but I was wondering if you could offer any speculation to ease my mind? Doctor: Hi,Dear,Thanks for the query to HCM. I studied your problem in depth and I understood your concerns.Causes and Remedy-In my opinion your testicular lump is mostly epididymal cyst -post traumatic,which occurred after ball hit to testes.I would suggest you to wait and watch for 2-3 days time.If the swelling subsides on its own,you need not worry but keep a watch.If not-Consult ER Doctor and take anti-inflammatory and antibiotics to reduce the swelling.If it grows in size consult ER Surgeon.So consult your doctor and don't worry.So don't build up wrong concepts and create more psychic complications in you which would increase risks and costs to you, but just ask a query to HCM and be comfortable to resolve your health issues.Welcome for any more query in this regard to HCM.Write good resume and Click thanks if you feel satisfied with my advise.Have a Good Day.Dr.Savaskar M.N."
},
{
"id": 220926,
"tgt": "What are chances of normal delivery after 4 preterm deliveries?",
"src": "Patient: hi i been looking for an anser for a long time...i have 4 kids and they were all premature and the first one was 27weeks the second was 33weeks the third was 35weeks and the fourth was 35weeks. now im 30 weeks and i want to know the potentials of me delivering early again? Doctor: Hello dear,I understand your concern.In my opinion it appears from your history that there is recurrence of preterm labour.As it is recurrent the cause for preterm labour like short cervix should be ruled out.Actually in case of short cervix a cervical stitch can be applied around 15-22 weeks.Once the cause is treated the preterm delivery can be prevented.So relax.Nothing to worry.Best regards..."
},
{
"id": 150582,
"tgt": "Intermittent numbness in the foot after a surgery for stenosis and discectomy. Pain in the hip and knee. What can be done?",
"src": "Patient: I am 40 years old. I had surgery 4 months ago on the left side of my back to remove stenosis at L4 L5, L5 S1, and discectomy at L4 l5. I v had intermittent numbness in my left foot since the surgery. In the last 2 weeks my right foot has become numb. I also have pain in my hip and knee (right side). Just to be clear, my left foot is still numb as well. I am an active person. However, I have not over done it with exercise since my surgery. I stretch and walk every day. What should I do? Doctor: Hi, Thank you for posting your query. Intermittent numbness could be related to your problem prior to the surgery. It is possible that the leg nerves that arise at L4-5 and L5-S1 were compressed prior to the surgery and were partially damaged. Recovery would occur over time. Please continue with physiotherapy. However, if symptoms worsen over time, you may need to consult your doctor. Please get back if you require any additional information. Best wishes, Dr Sudhir Kumar MD (Internal Medicine), DM (Neurology) Senior Consultant Neurologist Apollo Hospitals, Hyderabad, My personal URL on this website: http://bit.ly/Dr-Sudhir-kumar My email: drsudhirkumar@yahoo.com"
},
{
"id": 103840,
"tgt": "Intense itching on the skin with raised bumps. Allergy?",
"src": "Patient: Hi for the past three weeks I have had intense itchy skin. once I scratch I get raised bumped like a big mosquito bite that last for several days and itch intensely. they wake me up at night and I scratch until I break the skin. they are on my hands, fingers, around my wrists, around elbows, around my shoulders, around my knees, and arround my ankles, and on my feet and toes. My husband has no rash or itches and my cats are not scratching themselves. I have found no bed bugs or fleas. I live in Arizona. What could this be? Doctor: Hello,History of intense pruritis.This may be Urticaria. Your skin has come in contact with come allergen for which it is hyperreactive.Itching and rash are initial forms of hyper reaction.Try to find out the allergen and avoid further exposure.Take anti histamines and apply moisturizers to the area of skin involved."
},
{
"id": 9372,
"tgt": "Suggest treatment for a dry and wrinkled upper lip",
"src": "Patient: Hello doctor, My upper lip feels like it is taped. It feels very dry. When I try to smile, it is very uncomfortable to move my upper lip because of the dryness, I believe. It started in the last few months ago. The symptom lasts about one week, but comes back constantly. This is my third times having this symptom. Every night I apply lip balm before I go to bed. Currently, I am using Aquaphor lip protectant/sunscreen, but it doesn t seem to work. My upper lip is wrinkled from the dryness and it peels a little on the edge, but not severly. There is no bump or spot, it just feels uncomfortable and dry like a tape is on there. What can I do to cure this problem? Doctor: HIWell come to HCMI really appreciate your concern, better to stop applying what ever you are using for this, and try medicated moisturizing agent with steroid, like Urea, Lactic acid, paraffin, Beclomethasone, drink more water, have more leafy vegetable in diet, hope this information helps."
},
{
"id": 107015,
"tgt": "What causes pain in the lower back during pregnancy?",
"src": "Patient: Hello, I am 24 weeks pregnant and I am having some pinkish brown colored discharge with mild pain in my lower back. This is my third pregnancy and I have never had this happen before. My baby is still moving a lot and I am home alone with my four year old. Is this normal? Or something that can wait til the morning to go the hospital? Doctor: Dear patient Lower back pain is common during pregnancy. This happens due to increased weight of the baby and uterus which lies anterior to spine and hence increased stress on the spine. Pinkish brown discharge is not normal and consultation with obstetrician would be worthwhile. it may be sign of placenta previa. all the best."
},
{
"id": 74329,
"tgt": "What causes rattling sound in the chest of a child?",
"src": "Patient: my 2 yr old daughter has rattling n her chest and breathing heavy like its hard to breath was at hosp monday doc did xray she has bronchitis went bk yesterday for check up. she didnt start rattling till we got home yest. is this leading to something worse? whay should i do? Doctor: Respected user , HiThanks for using Healthcaremagic.comI have evaluated your query thoroughly .* This is due to bronchospasm caused by bronchitis giving rattling sound .* Additional suggestions for better recovery- Continue all medicines as per pediatrician advise .- Give balanced nutritious diet with plenty of liquids to maintain hydration- Maintain room temperature .- Avoid exposure to excess cold .- Avoid all dairy products , excess sugar , sweets , ghee , cheese .- Keep watch on urine output and fever .Hope this clears your query .Welcome for further guidance .Regards dear take care ."
},
{
"id": 152898,
"tgt": "What are options of treating adenocarcinoma apart from carboplatin and taxol chemotherapy?",
"src": "Patient: I had a bartholin adenocarcinoma removed in March 2008. I had a radical vulvectomy. No cancer in the lymph nodes. This year four nodules were found in my lungs that indicate the same cancer. I have had chemo consisting of carboplatin and taxol. The first series decreased the nodules. The second did not. Have you heard on any other treatment? Doctor: Thank you for posting your queries. You had a vulval carcinoma which is now spread to lung. Paclitaxel (Taxol) and Carboplatin are commonly used used chemotherapy regimen in this scenario.Ideally response should be assessed after 3-4 cycles which guides continuation or change of regimen. According to you probably your disease is not responding well with the above regimen.Actually there is no other treatment option except chemotherapy in metastatic vulval cancer.You oncologist may change the chemotherapy regimen with some other drugs which may help.But there is no role of surgery,radiotherapy or any other medication except supportive drugs."
},
{
"id": 163478,
"tgt": "What causes unexplained fever and abnormal breathing pattern in a child?",
"src": "Patient: 5 year female child has ataxia,nystagmus,difficulty swallowing and chocking,dysartheria and abnormal breathing pattern.All these saymptoms heve been developed gradually over the lastyear.Apart from delayed motor mile stones where she started walking in her 3rd birthday ,she has otherwise normal developmental history.Nowadays patient is in PICU on mechanical ventilation because of the desaturation that develops as aresult of her abnormal breathing pattern and chocking with her own saliva.other significant findings is her unexplained fever,hepatomegally that increase with days .what could be the case and iam wondering if you could give me a plan to work on ,in order to reach to the diagnosis. (if you want any additional information ,you can ask me) Doctor: Hello,It could be a case of muscular dystrophy or atrophy, as an only motor milestone is delayed. Some form of brain dysfunction is also seen, most likely smooth muscle atrophy, it usually progresses with age. Fever can occur due to aspiration pneumonia, infection as the baby is not a ventilator.Hope I have answered your query. Let me know if I can assist you further.Regards, Dr. Sachin Kumar Agarwal"
},
{
"id": 159395,
"tgt": "Sudden knee pain, swelling, pain on walking. MRI done, referred to orthopaedic specialist. Cancer?",
"src": "Patient: My mum suddenly had knee pain and it swelled up. She had pain walking so had x ray which showed a large shadow on the outer leg just above the knee. After about a week her knee got better and she could walk again. She had an MRI and has now been referred to an orthopaedic hospital. I m really worried it could be cancer . Is there anything else it could be? Doctor: Dear michellewrench, Good afternoon, You did not mention the findings of MRI reports. Also it is difficult to comment without seeing the X-ray. By history it appears she may have have developed ligament injury or meniscal injury (a soft bone in joint). It is very less likely to be a cancer. Kindly go ahead with the treatment/ consultation by orthopedic surgeon. Take care. I wish your mum a fast recovery & good health. Regards."
},
{
"id": 165912,
"tgt": "What causes cough after the treatment for bronchitis?",
"src": "Patient: my 27 month old son was treated for bronchitis about 2 months ago and the following week he had an ear infection followed by a fever. He was treated and he is fine now except that he still coughs when he runs around. He also sweats a lot especially when its bedtime. Could this be another disease? Doctor: looks like bronchitis is stil remaining its traces use nebulisationsgive monteleukast and levocetrizine combinations Syrup for two weeks"
},
{
"id": 218042,
"tgt": "What is the pain in my left calf?",
"src": "Patient: For several months, maybe even as long as a year, I have pain deep in my left calf that seems to be getting worse. When I walk and try to stretch it it hurts the most. It s just odd that it has lasted this long and getting worse. I don t feel warmth but it does seem to me that this calf feels thicker than my other calf. What could this be? Doctor: Hi there, Thanks for your query. The probable causes of pain can be inflammation of the calf-muscle-tendon, contusion/spasm of the calf muscle, underlying incompetent veins or, Vitamin B12 deficiency. Get a Doppler study to assess blood flow in the calf muscles veins. An X-ray of the heel bone will show if there is a partial detachment of the tendon. A course of a combination of anti-inflammatory pain killers, such as ibuprofen and, a muscle relaxant, such as, Chlorzoxazone should give relief in case of inflammation of tendon or muscle contusion/spasm. Consult your doctor and apprise him of my opinion. I am certain that he will agree with me and will prescribe the advised drugs in appropriate doses. If you find my response helpful and informative, do not forget an \u201cexcellent\u201d (5-star rating) to my answer, to ENCOURAGE ALL doctors- engaged in social service- to render sound advice to the FREE queries. Take care Dr. Rakesh Karanwal"
},
{
"id": 37278,
"tgt": "Suggest remedy for fungal infections",
"src": "Patient: HI ,I am suffering with fungal infection on my ties from last 3 year. doctor suggested tab :Gris -OD(daily one for 45 days) and cream KETO to apply on surface of skin.my question is i need to use it 45 days mandatory r any thing else to heal in 10 are 15 days.Regards Vijay Kumar Doctor: Although not mandatory to use for 45 days, I would insist on prolonged use as your doctor has advised. This is because, some fungal hyphae remain there for long and often germinate after the course is completed. This may lead to failure of treatment which can be avoided with prolonged treatment with topical ketoconazole (keto cream) and oral griseofulvin (Gris OD). Your doctor has given you the right medicines, so please continue it."
},
{
"id": 120322,
"tgt": "What causes severe leg pain and suggest treatment?",
"src": "Patient: Hi My father is in his 60 s and suffering badly from pains in his legs, he is on a waiting list for steriod inections to help relieve the pain but until that date comes about his in lots of pain is there anything else he can do until the appt comes ? :( Doctor: Hello,I read carefully your query and understand your concern. The leg pains can be related to many medical conditions. Beside steroid shots, your father can use other alternative treatment. I suggest to try painkillers such as Ibuprofen 400 mg to relieve the pain. It can be taken three times a day. Hope my answer was helpful.If you have further queries feel free to contact me again.Kind regards! Dr.Dorina Gurabardhi General &Family Physician"
},
{
"id": 183883,
"tgt": "Suggest treatment for lower jaw pain",
"src": "Patient: I am 30, am 5'7\", weigh 150 lbs. I have severe lower left jaw pain and severe throbbing in a lower left molar. I have been wearing a tap appliance for about 4/5 months. The pain is continuous and aches and throbs constantly. It has been getting worse daily. I had a filling done on the tooth that hurts a little over a month ago. I don't know what is causing the pain or what to do to ease the pain. Any suggestions would be very much appreciated. Thank you! Doctor: Hello and welcome to HCM forum,I would like to inform you that as mentioned in your query, you recently got a filling done on your tooth and that what creates a suspicion. Sometimes, filling in your tooth is slightly high than ideally it should be.Sometimes, people tend to grind down the filling bringing it to the level of other teeth.In your case, i will advise you to see your dentist and get the filling recheckes for high points. I am sure your dentist will know better.Until then avoid chewing from that side and take soft diet if possible.I hope i answered your query.I wish you good healthTake care ."
},
{
"id": 97258,
"tgt": "What is the recovery period for accident injuries and fractures?",
"src": "Patient: Hello sir I had a bike accident on Saturday . Got 14 stiches on upper lips . And full road rash scars on face and fracture on right elbow with 3 broken teeth . And many road rashes on the body . Please tell me the time taken to recover and some precautions to be taken Doctor: Hi and welcome to HCM. Thanks for the query.the only problematic part is elbow fracture and recovery for it may last up to 6-8 weeks if there isnt complciasted fatcture. everything else will be beter within 2 weeks. you should rest and keep your wounds clean. take standard analgesia,there is no other specific treatment.Wish you good health. Regards"
},
{
"id": 12783,
"tgt": "Can a pinched nerve in my spine cause a skin rashes?",
"src": "Patient: Can a pinched nerve in my spine cause a skin rash. I had a blistery rash. Looked like poison ivy. That has been ruled out. Then changed meds for maybe scabies. Blisters dried up but still have a rash. I also, prior to the rash, had a spine out of alignment. No real bad pain with it. Just couldn't raise my arms in front. The other day I had that sensation again through my collar bone area. But the feeling only lasted 5 or 10 minutes. It is wierd having an unexplained rash. The rash is mainly my arms but has spread to other areas once the blisters dried up. Doctor: Hi, According to your complaints it seems to be an eczematous rash. I have recommend you to take antihistaminic like tablet cetrizine 10 mg twice daily and apply topical steroid & antibiotic combination on affected areas twice daily for 10 days. Hope I have answered your question. Let me know if I can assist you further. Regards, Dr. Asmeet Kaur Sawhney, Dermatologist"
},
{
"id": 35050,
"tgt": "Does the body become to chicken pox if had once?",
"src": "Patient: Hi there I am about to go for frozen embryo transfer sometime next week (depending upon when I ovulate) and have just been exposed to chicken pox - my niece has just developed spots today, my nephew has crusted spots. I had chicken pox when I was a child, so does this mean I am immune, and if so, is it safe to go ahead next week? Thanks, Carolyn age 34 Doctor: Hello,Welcome to Healthcare Magic,Chicken pox is a highly communicable 1-2 days before the appearance of rash and 4-5 days thereafter. After which virus tends to die out and looses the capacity to transmit disease to normal individuals. The scabbing usually begins 4-7 days after the rash and completed by 30 days. During the scabbing period you are child will be sterile and will not transmit disease to others.The disease will cause dehydration because of fever, the symptoms will come down once the rashes starts fading off. Your child should drink water as much as possible atleast 100ml/kg body wt./day.Thank you."
},
{
"id": 173032,
"tgt": "Would physio therapy be effective for problem in movement of right hand?",
"src": "Patient: Dear Doctor, My kid 12 months old. she is not using her Right hand as effectively as left hand. She crawls with left hand and plays with left hand only. Doing physio therapy exercises regularly still we are not much difference with that. kindly suggest. Doctor: Thanks for asking I gone through your question. your 12 month old kid use less right hand then left hand. you not mentioned since when he/she doing this. if i was your attending pediatrician would like to ask some questionig1. since when your kid is having weakness inright hand2.was there any problem during birth or there after (like fever with fits)3. any history of hospitalization4.about his diet he received since birth. 5. i would like to go for some test for detect cause of weakness in right hand. like (MRI brain.)so you need to meet with either pediatrician and or pediatrics neurologist for better out comehope i able to answer your queryhope your kid recover some"
},
{
"id": 89194,
"tgt": "Can medicines such as Ibuprofen and erythromycin result on stomach cramps?",
"src": "Patient: Hello, My age is 28, height is 5'5\", weight is 50Kgs and I am by and large unable to take medicines like ibuprofen and even antibiotics like erythromycin etc as I develop severe stomach cramps and often diarrhea tooI would like to start the oral contraceptive and have been recommended yaminiI have read a lot about the side effects like mood swings and can corelate that to vitamin deplition etcCan you advice me about the potency of this pill and if any lower dosage ones exist that i may start with? Doctor: hello madam,your stomach problem are due to ibuprofen which is a known drugs to cause stomach painsdo stop it and substitute it with aceclofnec.for oral contraceptives do vists a gynaecologist"
},
{
"id": 94614,
"tgt": "Pain in lower abdomen. Cannot bend, twist body. Having stomach pain which relieves after using the bathroom. Reason?",
"src": "Patient: I woke up this morning and got out of bed, while getti out of bed I felt a sharp pain in my lower abdominal side. Ignored it. But when I tried I get back in bed and lay down, or lean to my right or left, I had a excruciating sharp stabbing pain on my left abdominal side. I can sit on a couch but I have to sit up staight and not twist or bend to avoid the pain. I lay on my right side majority if the time when I sleep. I have been noticing some pain in my belly a few nights and mornings. It usually goes away after I use the bathroom. It feels like an Iran is twisted maybe. My stomach is also bloated. Doctor: Hi, Thanks for posting your query. There may be possibility of acute gastroenteritis along with muscle ache. Presence of bloating abdomen along with relief after using washroom aggravating on movement suggests the diagnosis. You should consult with internal medicine specialist and should go for thorough check up. You should take proper antibiotic course along with anti-spasmodics. You should also go for ultrasound imaging along with x ray abdomen. Take care, Dr. Mayank Bhargava"
},
{
"id": 59232,
"tgt": "Suffering from liver abscess. On rebicip. Done US. What disease is this?",
"src": "Patient: my son age 25 is suffering from liver abscess. Dr. diagonised on 22nd march and started treatment, now he is feeling good but he has to take medicine rebicip before eating and after eating metrogyl tap 3 times in a day. in the ultra sound, the it is showing 170 mm (<30), what is the best remedy to cure from this type of disease. Doctor: Hi and welcome to HCM.Thank you for your querry. Zhis isnt very common in his age. Usually abscesses are found after sepsis or some serious surgical procedure. It is important to be sure that his isnt adenoma, or ehinococal cyst. The best treatment is surgical, but sometimes doctors try with antibiotics first and see is there improvement. 170mm is really small abscess ant I think it can be managed nonoperatively. This shouldnt be anythng serious and will heal without consequances. Wish you good health."
},
{
"id": 87118,
"tgt": "What could abdomen pain with burping and dry mouth suggest?",
"src": "Patient: Hi just a query,I get random pain in the left side of my abdomen about 2 inches left of my belly button. The pain is slightly sharp but is more of a stitch or contraction, and only lasts about 30 seconds to a minute.This pain is random and i can go hours without having them and eating doesnt seem to affect it.Although i around the same time as this pain comes i get reflux issues such as burping, a really dry mouth after sleeping.A few weeks after i get the pain i lose my appetite and turn off food.Any help would be greatly appreciated! Doctor: Hi.Thanks for your query.Pain in the left side of the abdomen just 2 inches left of belly button lasting 30 seconds, not affected by eating, but have reflux and burping with pain, dry mouth after sleeping, with loss of appetite indicate the following reasons:Acid peptic disease with GERD. I would advise you to undergo Upper GI endoscopy and ultrasonography.Once the diagnosis is made go for the proper treatment of PPI, Antacid, soft bland diet. for 3 months and you may be fine.If no relief, CT scan and Diagnostic laparoscopy may be required."
},
{
"id": 205238,
"tgt": "Suggest treatment for borderline personality disorder",
"src": "Patient: My wife was diagnosed with BPD last August and I think I m beginning to see how she has emotionally manipulated me all our relationship which has grown worse since then. In the past I ve believed everything she feeds me about how much she was mistreated by me though I didn t see things the same.how should I proceed from here knowing she absolutely refuses to believe that her thoughts and actions are being driven by the illness? Doctor: hi, i will suggest u that not diagnose by your self and from internet. first of consult any psychiatrist if she will not ready than call psychiatrist or clinical psychologist at your home for consultation. treatment will start after consultation."
},
{
"id": 178257,
"tgt": "What causes excessive hiccups with vomiting in an infant?",
"src": "Patient: my 28 months daughter hiccups alot , recently she started throwing up randomly without a pattern not everyday not even every week , she just vomit out of nowhere especially at night around sleeping time i tried to find any connection between these days but nothing everytime she d a different thing at dinner so what could be the reason Doctor: Thank you for posting your question.Hiccups occur due to diaphragmatic irritation and this could be due to a variety of reasons one of them being over distention of the stomach.Here hiccups and vomiting could be two unrelated symptoms or could be due to a condition called gastro oesophageal reflux.In this conditions the contents of the stomach regurgitate back to the food pipe manifesting occasionally as frank vomiting. This may be preceded and caused by gastric over distention leading to hiccups. This may also be due to dicreased tone of the lower oesophageal opening leading to recurrent regurgitation.I would suggest, you try some changes like putting him to sleep atleast half an hour after his meal, keeping the head end high or putting her to sleep on his left side.If there is no change then I suggest you take a medical consultation and consider taking medicines like proton pump inhibitors to decrease the acidity of the stomach and thus the reflux symptoms. If you have any more doubts, do write back."
},
{
"id": 34081,
"tgt": "Suggest treatment to follow after dog bite",
"src": "Patient: Hi my name is ming. 29 yrs of age.my question is,what should one do when bitten by a dog.i was told that one should get TT injection and that i should check on the dog for seven days.if the dog is normal than i do not have to get antis-rabies injection. is that true.what if the dog has had rabies vaccination. Doctor: Hi Ming!Post dog bite treatment consists of Local treatment of the wound, Inj Tetanus toxoid,Vaccine therapy - FIVE doses of Inj Antirabies vaccine given in the deltoid muscle (on 0, 3, 7, 14 & 30 days schedule) should be initiated immediately with the bites of categories II and III.For category III bites immediate vaccination and administration of rabies immune globulin ( along with the first dose of vaccine) should be given.Types of dog bite are:Category I \u2013 touching or feeding animals, licks on the skinCategory II - nibbling of uncovered skin, minor scratches or abrasions without bleeding, licks on broken skinCategory III \u2013 single or multiple deep bites or scratches, contamination of mucous membrane with saliva from licks.Treatment may be discontinued if the dog or cat remains healthy throughout an observation period of 10 days;Regarding the treatment of bite inflicted by a vaccinated dog.A vaccinated dog is not always a guarantee that it is not rabid.Vaccine failures may occur because of improper administration or poor quality of the vaccine, poor health status of the dog, and the fact that one vaccine dose does not always provide long-lasting protection against rabies infection in dogs. Hence, appropriate documentation of vaccination status of dog and proper history should be elicited before deciding to defer post-exposure prophylaxis after bite by vaccinated dog.Hope the suggestions given above could be of great help in getting the prophylactic treatment following a dog bite.A feed back is highly appreciated."
},
{
"id": 5219,
"tgt": "Trying to get pregnant. Taking Fertab 50. Semen test normal. Advice?",
"src": "Patient: hi doctor,i m manisha from dubai,i m trying to be preganant from last 5 months,actully i married 3 years ago,but i did not take any treatment in my previos marriage,i have remarried about 5 montha ago, now my husband take care of me very well, and we are trying to get baby,we met gyn.doc in dubai,doctor given me fertab 50,i m using from last 3 days ,after 2 days she call me to hospital for checking,so what she wii eximine only after 5 days to use of fertab 50,my husband also given semen test and its normal report,i m 37 year old ,thanks Doctor: Hi,Thanks for the query.Fertab 50 contains clomiphene citrate.That helps in ovulation induction.So after using the tablets for five days, doctor can suggest you follicular study.By this the ovulation can be tracked and intercourse can be planned around those days, which will increase the possibility of pregnancy.Start taking folic acid tablets from now-onwards.For more details you can ask me through: http://www.healthcaremagic.com/doctors/dr-sree-gouri-sr/63429Take care."
},
{
"id": 81226,
"tgt": "What causes chest pain while sleeping?",
"src": "Patient: Hello, i get pain in my chest from past 3 - 4 only while sleeping (after 2 - 3 hours of sleeping in the bed) and also it s difficult to turn around on the bed. the pain is vanish after 20 - 30 minutes after i wake up from the sleep or if i leave the bed . should i consult a cardiologist ? Doctor: Thanks for your question on HCM.I can understand your problem and situation.Yes, you should definitely consult cardiologist and rule out cardiac cause for your chest pain.Chest pain in night is commonly seen in cardiac causes like angina, early congestive heart failure and myocardial ischemia.So better to get done 1. ECG2. 2d echoTo rule out above mentioned possible causes.If both are negative than no need to worry much about cardiac disease.You may have anxiety related or muscular pain.Do don't worry, be relax and calm."
},
{
"id": 142815,
"tgt": "Suggest possible medication for high fever & body pain",
"src": "Patient: I have a 103 fever again. Had 104 2 weeks ago, but got better. Now takind Ibupren 1000 mg only brings it down to 101 Chivering, body aches, back hurts, no naucias, but brain fog. Fever on and off 24 hours Urgent care 10 days ago dx hand, foot, mouth virus. Took benadryl and pregnazone for 2 days and swelling and rash went away What do i do next? Doctor: Hi, Welcome to HealthCareMagic.com I am Dr.J.Mariano Anto Bruno Mascarenhas. I have gone through your query with diligence and would like you to know that I am here to help you.you should undergo blood for malaria parasiteblood for Widalblood C/sand sputum for AFBHope you found the answer helpful.If you need any clarification / have doubts / have additional questions / have follow up questions, then please do not hesitate in asking again. I will be happy to answer your questions. In the future, for continuity of care, I encourage you to contact me directly in HealthCareMagic at http://bit.ly/askdrbruno Best Wishes for Speedy Recovery Let me know if I can assist you further.Take care."
},
{
"id": 202748,
"tgt": "Slightly bent penis, do not have swelling or pain. Cause of curvature?",
"src": "Patient: I m 19 years old , recently I observed that I have slight bent penis towards right from the very base. So I go over the internet in order to find some answers and came across Peyronie s Disease and of course without any doubt saw some shocking images. But by studying a bit more I came to realize that I don t suffer any symptoms like swelling or pain other than a turn which is definitely not that kind of a bent like the one in disease. So should I be concerned or it s just natural shape or due to the way I sleep or put my penis during clothing ? Doctor: HelloThanks for your query.You have bending of your penis towards Rt side.This could be due to scarring in the Tunica Albugenia an envelop that covers erectile tissue .Normally it gets cured itself without any treatment and does require treatment if the curvature is more than 30 degree and hinders penetration of vagina during intercourse.Please monitor it carefully and consult qualified Urologist for clinical examination and further treatment if required.Dr.Patil."
},
{
"id": 47727,
"tgt": "I have a stent placed, and have large kidney stone, what should i do?",
"src": "Patient: I had a stent placed to help me urinate while a large kidney stone is present. I was referred to a Urological center for stone and stent removal but due to the high costs of the previous procedure and hospital visit, I am now broke and cannot afford the drive/co-pay for this new one. What should I do Doctor: Hello and welcome to HCM.As an Urologist, i must advise you to go to a government medical college hospital, where treatment is free of cost.A large stone, (??size), if affecting the kidney, must come out, because it damages the kidney function.If you can send me the details, i can guide you better.You may send the reports as a direct question in my name, to get an expert opinion.Dr.Matthew J. Mangat."
},
{
"id": 160766,
"tgt": "Could the lump after a DPT vaccination be a matter of concern?",
"src": "Patient: my 4 month old baby had his DPT vaccine last week.He had fever that lasted for 1 day.The next day he had lumps on the area where the vaccine was done.I put warm compress on it and go to nearby doctor.He gave ointment and said to continue putting warm compress.After 4 days,the lumps is still there.They said it s dangerous or the effect will be seen when my son grow up.I m so worried. Doctor: Hi,It's quite common for children to have some lumpiness and swelling at vaccination site. If it is not painful, there is no limping on walking and no overlying skin redness, no need of any worry. It will disappear in 6-8 weeks. It is neither dangerous nor troublesome in the future. Be confident.Hope I have answered your question. Let me know if I can assist you further. Regards, Dr. Muhammed Aslam TK, Pediatrician"
},
{
"id": 158420,
"tgt": "Gray freckles on tongue. Done tongue biopsy. What is it? Cancer?",
"src": "Patient: I had a tongue biopsy yesterday. On the right side of my tongue, I had a grayish spot with little freckles around it. A week later the left side had some discoloration as well. I m scared and can t sleep much waiting on the pathology report to come back. Since yesterday, the swelling went down, I m able to speak and my tongue has white stuff on it and theres lots of bruising . I m afraid it might be cancer. Doctor: Hi there,I understand your anxiety and concern, but nothing short of biopsy report can conclusively tell if it is a cancer or not. I'm happy that you got biopsy done and did the right thing. Now we just wait for the report and in the mean time try to develop a healthy lifestyle, do some light exercise or try walking for 1 mile or so evening or morning. Improve your diet. Take care"
},
{
"id": 224035,
"tgt": "Suggest dosage of contraceptive pills",
"src": "Patient: hi doctor, iam getting married next month and my fiancee is not in urgency to have a baby for a year and he is not willing to use condoms for having sex but instead asking me to take contraceptive pills for precaution.please suggest me a contraceptive pill and the procedure of how and when it should be used? Also tell me will it affect when we plan to start a family by stopping the pills? Doctor: HiDr. Purushottam welcomes you to HCM virtual clinic!Thanks for consulting at my virtual clinic. I have carefully gone through your query, and I think I have understood your concern. I will try to address your medical concerns and would suggest you the best of the available treatment options.1)First of all do not panic.2) The oral contraceptive pills that are available now a days are 3rd generation pills. They have most effective minimal dose of medicines in them. They are safe and effective. And do not have effect on your fertility once you stop taking them.3) I usually suggest Tab OVRAL L . For the first time pack is to be started on day 2 of the period. Finish 21 tablets , one per day at bedtime. Keep a 7 day pill free interval,and restart nest pack for 21 days. Follow 21 days on pills and 7 days off pill regime.4) I will also suggest to take Cap FOLINEXT-D daily along with OC pill.I will suggest indulging in a healthy diet and regular exercise regime.Include plenty of fruits, salads, vegetables in the diet. Have adequate water intake , say 8 \u2013 10 glasses per day. Avoid deep fried foods, bakery products, and refined sugars.Avoid substance abuse, if any.I hope my answer helps you.Thanks.Wish you great health.Dr Purushottam S Neurgaonkar"
},
{
"id": 168925,
"tgt": "What causes rashes under arm of a 12 year old for years?",
"src": "Patient: My son aged 12 has rash down one side under arm along side of back. He has had this for ages and 2 lots of antibiotics have not got rid of it. The spots are red and sometimes angry looking where he scratches them but they are not itchy all the time. They have yellowheads and they scar. He has had them for almost a year and the blood test was non conclusive. Can you help thankyou. Doctor: Hi, can understand your concerns. Underarm Rash can be due to many causes like fungal infection, allergic Rash, underarm sweat Rash, early signs of HIV infection, Contact dermatitis or worm infection. I suggest you to do HIV test of child. There are some home remedies which you can follow like we are loose clothes, apply ice packs to reduce itching, apply Vitamin E oil on aloe vera oil, increase Vitamin C intake to reduce skin infection, swipe slice of lemon under your armpit to help kill bacteria. I hope this will help you. Take care."
},
{
"id": 25200,
"tgt": "What causes pain in upper left chest and above the breast?",
"src": "Patient: For over a week now I've been having pain in my upper left chest above my breast when my heart starts to beat really fast. It usually happens when I'm relaxed. My dr said my blood pressure was high in november 120 over 94 so he has me on Lisiniprol and also doubled my metformin dosage to 2000 mg per day. Are these symptons of a stroke or heart attack? Ive also been very bloated and gassy and the pain I have in my chest I've had for years. I just have never had such rapid heart beats and its causing me to freak out. I'm due to go back to my Dr in feb. I also have shoulder pain but again, i have a bone spur there. The upper central part of my spine feels sore as well. I have a tons of issues going on in my life right now but I cant tell if its anxiety or truly heart attack/stroke related. Doctor: Hello!Welcome and thank you for asking on HCM!I carefully passed through your question and would explain that it is important peforming a comprehensive differential diagnosis of different causes that may lead to these symptoms: - gastro-esophageal reflux or hiatal hernia- a musculo-skeletal pain, including costochondritis. - a backbone disorderl - coronary artery disease, although the symptoms occurring at rest are not typical of this disorder- anxiety could mimic this clinical scenario I would recommend performing some tests: - a chest X ray study- a resting ECG and cardiac ultrasound to examine your cardiac function and structure - an ambulatory 24-48 hours ECG monitoring to examine your heart rhythm through the day- some blood lab tests (complete blood count, PCR, sedimentation rate) for inflammationA fibrogastroscopy may be necessary if all the above tests result normal, to help rule in/out possible gastro-esophageal reflux. If all the above tests result normal, you should consider anxiety as the main cause of your symptoms. In such case you should consult with a specialist of the field. Hope to have been helpful!Best wishes, Dr. Iliri"
},
{
"id": 122172,
"tgt": "Suggest treatment for sciatica pain in the right leg?",
"src": "Patient: Hello Sir, My wife 44 years old. She got a cytika pain in her right leg (below knee, thigh) since last three weeks. She is continuously taking medicines from general physian. But she could not get relief from this pain. From last week she has started homeopathic treatement is started. But she could not get relief. She could not have night sleep due to this pain. Please kindly advice on this. -Subhash Wavikar Doctor: Hello, The symptoms seem to be related to sciatica pain. I suggest using anti inflammatory medications such as Ibuprofen to relieve the pain. I also suggest using gentle stretching exercises for relieve. If the symptoms continue, I suggest using prescription painkillers such as Pregabalin. Hope I have answered your query. Let me know if I can assist you further. Regards, Dr. Dorina Gurabardhi, General & Family Physician"
},
{
"id": 141675,
"tgt": "What causes foot numbness after undergoing hysterectomy?",
"src": "Patient: I had a hysterectomy a week ago Im doing great but after 2 days of surgery I start feeling my right foot numb, the doc see me yesterday and he said he doesnt know, he doesnt think it is anything to do with the surgery, he is gonna see me on friday again to remove my stitches and see how im doing with my foot but i feel the same and, Im walking im been really active Im doing good with the surgery except this problem with my foot numbness> Im really worry about it is really uncomfortable. Doctor: Foot numbness should not be related to your recent surgery.I would want your doctor to look at the lumbar spine for possible \"pinched\" nerves, and the peroneal nerve (which can cause numbness at the top of the foot)."
},
{
"id": 132483,
"tgt": "What is the cause for tingling numbness in the right arm?",
"src": "Patient: I m a 37 year old male, 5 10 , 185, and I m having numbness on the right shoulder, arm and hand for about 48 hours. I have been clenching the right side of my jaw for about 2 months. About 2 months ago I finally stopped a cough that I had for three months. Should I see a doctor for this tingling numbness in my right arm? Doctor: Hello.Yes this needs to be evaluated and it's obviously a pinched nerve problem, most likely from a disc problem in your neck.I would start with an MRI scan of your cervical spine. This may respond to proper physical therapy and not need a surgical approach"
},
{
"id": 197944,
"tgt": "What causes weird sensation in right testicle?",
"src": "Patient: My right testicle doesnt hurt, It just feels wierd. kind of like it has dropped down. and i have a wierd sensation from my testicle towards my leg on my rt side. It has an odd sensation in the tubing that comes our of my testy if that makes any sense.. what is wrong with me? Doctor: Dear, We understand your concernsI went through your details. It is quite normal that the testicles shrink. Each testicles could shrink its own or shrink one by one. Shrinkage is attributable to many things including outside temperature or dryness. Nothing to worry because it is common. You must be worried about the shrinking and therefore you are having the weird feeling. Unless there is pain or drawing feeling, nothing to worry at all. You may consult an urologist for confirmation.If you require more of my help in this aspect, please use this URL. http://goo.gl/aYW2pR. Make sure that you include every minute detail possible. Hope this answers your query. Further clarifications are welcome.Good luck. Happy New Year. Take care."
},
{
"id": 24449,
"tgt": "Can working in developing countries cause cardiac issues?",
"src": "Patient: I am a 48 yr old female nurse and in June I visited my GP with chest pain brought on by cardio exercise. He referred me to a cardiologist who found an ejection systolic murmur. I was referred for an echocardiogram and CT angiogram. Whilst waiting for these tests I developed chest pain at rest. The CT angiogram is still pending but the echocardiogram showed aortic stenosis and mitral valve stenosis. The doctor doing the scan asked me if I had ever worked in a developing country which I had 7 years ago when my then boyfriend died sussenly of myocarditis? cause. Could you plese tell me the relevence of being in Namibia 7 years ago and my cardiac problems now? and what is the likely prognosis and treatment? Doctor: Thanks for your question on Healthcare Magic. I can understand your concern. Your all current symptoms are due to valvular heart disease (aortic stenosis and mitral valve stenosis). Valvular heart diseases ateare commonly due to rheumatic heart disease (RHD). RHD is caused by streptococcal infection of heart valves. This kind of infection is commonly seen in developing and under developed countries. So when you stayed in Namibia, you might have infected your heart valves which is causing RHD at present. You will need surgical valvular replacement of aortic and mitral valves. Surgical correction is the only treatment available for severe valvular defects. Hope I have solved your query. I will be happy to help you further. Wish you good health. Thanks."
},
{
"id": 135795,
"tgt": "Suggest treatment for severe knee pain",
"src": "Patient: I have had posterior knee pain since April, it s pretty uncomfortable. I can squat or bring in my leg with it all the way flexed if that makes sense. I was working out when it started to hurt. So far MRI has come up clear for my knee and x-Rays are fine. Two orthopedist and still have no idea??? Doctor: HiWelcome to healthcaremagicI have gone through your query and understand your concern. You are likely to be having osteoarthritis of knee joint.You are likely to get benefitted by Diacerine and glucosamine. Quadriceps excercises are of help. You can take analgesic such as ibuprofen for pain relief. You can discuss with your doctor about it. Hope your query get answered. If you have any clarification then don't hesitate to write to us. I will be happy to help you.Wishing you a good health.Take care."
},
{
"id": 57439,
"tgt": "Increasing heart beat when exercising. History of chronic hepatitis C. Ideas?",
"src": "Patient: I am 49 years old , female. The last month I have had rapid heart beats, lack of appetite , very tired (even after a long nights sleep), body aches all over. I have been dx with chronic Hep C , but all bloodwork in July 2012 has come back undetected for viral load . My calcium is now at 10.4 after going back and forth within weeks from 11.0 to 9.9 to now 10.4. Parathyroidism has been ruled out. I have been referred to a Endocrologist, which I have an appt in Aug. Today I have been having a needle /pin like feeling in my upper chest area. I am 5 6 going from 105 to now 102lbs. I eat because I know I need to but am not actually hungry. I also have a bad colon with history of benign polyps so far, last colonoscopy in 8-2011 had two small sessile polyps, one was precancerous. Both removed and next colonoscopy not to be done until 2014. I also went from being in great shape when I was 18 to only sitting at jobs for over 30 years and not exercising regularly. Now that I am trying to exercise, my heart goes nuts .... soooo needless to say... I am trying to take babysteps to exercise even though I do not know what is going on. Thank you for any ideas you may come up with. Doctor: hi Thank you asking HCM with regards to your sympoms I strongly suggest to consult a cardiologist to make further examination about your heard beats. it is not associated with chronic hepatitsi C. Another reason may be anemia so please do a blood work. wishing you fats recovery DR.Klerida"
},
{
"id": 22393,
"tgt": "Could heart pressure lead to swelling in face?",
"src": "Patient: An 8 year old boy from my church was taken to the hospital today with chest pains. They found that he has a mass putting pressure on his heart and are transporting him, via helicoptor, to a children's hospital. I had asked his mother last week about his face looking a bit swollen. Do you have any idea what the mass might be? Doctor: Hi,Yes, it's possible if mass is causing compression over and surrounding blood vessels it may lead swelling of face. Mass can have number of causes. Common cause is lymphoma is a type of blood cancer. However we cannot say the unless we do biopsy of the mass. So it's unwise to discuss regarding diagnosis until CT scan and biopsy is done.Hope I have answered your query. Let me know if I can assist you further. Regards,Dr. Sagar Makode"
},
{
"id": 64007,
"tgt": "Are lumps in labia common post c section?",
"src": "Patient: Hello. I am 34 years old with average height and weight. I want to know if its normal to have a hard lump or boil on my labia minor and labia majora about 1 week after a repeat cesarean section? Is it serioius? What caused it and how can I treat it? Thanks for your help. Doctor: HI,Dear,Good Morning From INDIA.I studied your query with all its details and understood your health concerns.Causes of lumps in labia-minor and majora-a-As they have appeared after C-section-catheterisation and neglect for hygien over last few days there after could be reason for it.b-Reason-AS this has lead to the infection of the hair follicles and sweat glands of labia minor and majorac-Its serious if not treated by a doctor.d-Treat it by-Tb NSAIDs / Antibiotics x 5 days/ hygiene of the private parts.These health tips would help your urgency for the lumps.So consult your doctor and wish you fast recovery.Hope this will help you to recover your urgency.Wishing your fast recovery.Wellcome for any further query in this regard.IF you get relief-write a good review and recommend my HCM clinic to your foes and friends.Have a good Day..With Regards"
},
{
"id": 66772,
"tgt": "Suggest treatment for swollen lymph nodes in groin area",
"src": "Patient: I have had a swollen lymph node in right groin for about 18 months. Recently I discovered swelling in left groin and more node swellings in right groin. I also have a swelling on left side of neck, above collar bone which has been growing for nearly a year. I also have a pea sized swelling on the back left side of neck, just below skull. I have mentioned it to my GP several times, but he dismisses it and doesn t consider it important enough to investigate. Sometimes my legs, arms and body itches. I am concerned about lymphoma. I am due to see my GP this week. Please advise. Doctor: thanks for sharing your health concerns with us!you should have mentioned your age and TLC/DC of your blood.however, multiple lymph nodes or any kind of nodules in your body is a real thing for concern and one should think about tuberculosis, viral infection or even malignancy like lymphoma!an FNAC TEST IS THE BEST WAY TO CONFIRM IT!all the best..................."
},
{
"id": 224688,
"tgt": "When can i take the medication?",
"src": "Patient: I was just wondering when is the best time to start taking Logynon Ed I am on the 2nd day of my period am I better off waiting till next month to start or can I start now. Also how long should I wait before starting to skip my period after starting to take Logynon ed Doctor: Hello!Thanks for your query.Better is to start in the first day of your menstrual cycle because seven days before you are unprotected.You need to use another form of contraception.You should wait 7 days.Take care of your self"
},
{
"id": 176813,
"tgt": "What causes neonatal seizures?",
"src": "Patient: My baby had neonatal seizures and is in recovery now. The MRI report has this to say: Multiple small foci of of restricted diffusion in both hemispheres, Periventricular and probably in the paras a chital area including the genus of corpus callosum. The signal is abnormal in the bifrontal cortex. Basal ganglia is unaffected. Small amount of subarachnoid blood and small amount of blood in occipital horns. We are awaiting a formal meeting with the neurologist and neonatal radiologist. However, I d like to know more on this so that I am aware of what my child has gone through and what, if any, steps I need to take to help her out of this unfortunate juncture. Could you help me understand this a bit more and suggest a few questions to ask to gain more clarity? Many thanks. Arun Doctor: neonatal seizures occur most commonly due to Hypoxic insult to brain(during delivery or post delivery if child does not immediately cry), or due to metabolic causes like low glucose, low calcium or low magnesium. Seizures can be due to infections acquired during or after delivery. Small blood in Subarachnoid space signifies trauma to head during delivery which can be normal. Periventricular involvement in the brain is serious. Seizures can be controlled as of now with AED like Phenobarbitone but the effect of the condition on the neonatal brain cant be judged at present. What symptoms the child might develop in the future cant be judged right now."
},
{
"id": 38501,
"tgt": "Are CRP and sputum results worrisome in terms of a viral infection?",
"src": "Patient: My husband is admitted in hospital for severe viral infection. Various teast has been done.two things I am worried about are C reactive protein high sensitivity is 6.664 lab range is 0.748 His sputum gram s stain test says: occasional gram negative bacilli nd few gram positive cocci in pairs nd chains seen. 10-12/lpf Pus cells present. 3-5/lpf Epithelial cells present. Doctor: Hi, thanks for using healthcare magicC reactive protein is an inflammatory marker. This means that it increases when there is any inflammation such as infection in the body.It is high because your husband is currently fighting an infection. As the infection resolves, it would decrease.The sputum lab test was done to determine which bacteria is causing his infection. This would allow his doctors to choose the appropriate antibiotics.I hope this helps"
},
{
"id": 119067,
"tgt": "Fever and shivering. Platelet count 175.000. Treated with anitbiotics. Recurrence of platelet. Causes and cure",
"src": "Patient: Male 79 got fever and shivering 3-4times in the last 3-4 months which last about a day. Last time the doctor tested his blood for platelet count and was found 175.000 and admiited him in the hospital. Treated him with anitbiotics, the count decreased to 150,000 the next day but brought back to 250, 000 after a few days and was released from the hospital and stopped medicines. He again got fever, shivering and tested for platelet and it is 150 000 now. He is otherwise in general good health or his age and active. what could be the underlying cause? Doctor: Dear Ms Jacob It is essential to understand that we need to treat the symptoms and identify the cause. Your concern for the fever is very understandable, but it is important to identify the cause by further tests such as a urine examination. In elderly patients urine infections are very common and can be confirmed only by a urine examination. The variation in the platelet readings is not worrisome and could just be a normal fluctuation or a change primarily due to the infection elsewhere in the body. So for now I would suggest you to get a urine test done and also get a complete blood picture. In cases of infection the white blood cell count or WBC count would be more helpful to guide the treatment. Wish your patient good health."
},
{
"id": 111520,
"tgt": "What need to do for lower back pain?",
"src": "Patient: My husband has had and now is having the same issue of severe pain in his lower back... He was told to take something to create a bowel movement which helped last year. This early morning he is having this same pain but even more severe. What can we do before calling the doctor? Doctor: Hello,I had gone through the case and if pain is unbearable then give him Injection Voveron for palliative treatment.Immediate go for ultrasound of abdomen to find the exact cause of pain. It might be constipation, renal stone or urinary infection.Hope my answer will be helpful for him.Thanks"
},
{
"id": 87007,
"tgt": "How can chronic stomach pain be treated?",
"src": "Patient: I have a 5 year old daughter who has been having chronic stomach pain for the last two years. They come and go. Usually in the middle of the night. they last for an hour or so. She throws up and then she is fine. Done all the test and eliminated suspected food. Nothing has helped. Any suggestions? Doctor: Hi.Thanks for your query and proper history. I can understand the plight the child and you must be going through.Your daughter is just 5 years old and hence the causes of pain in the abdomen for the past 2 years can be as follows. The most probable cause of pain in a child -Tummy tonsils- meaning enlargement of the lymph nodes of the abdomen.-Appendicitis- by default this has to be ruled out in every case of pain in -abdomen of a child.-Worms.The best diagnosis is a high resolution ultrasonography. some blood, urine and stool tests.A therapeutic trial with a course of an antibiotic, metronidazole and probiotics under the guidance of a pediatrician / Surgeon always helps to solve the problem and to get a diagnosis.Also given is a dose of medicines active against worms in intestine.I hope a consultation with a Pediatric Surgeon, investigations and a course of an antibiotic and metronidazole should solve the problem.If this is indicative of appendicitis , please go for the surgery of appendectomy."
},
{
"id": 21698,
"tgt": "What is the treatment for Left ventricular hypertrophy?",
"src": "Patient: Sir/Madam,I've been diagnosed as having mild lvh. My ldl/vdl ration is just a point below normal, that is, 4.8 against the normal level of 5.10. I'm using ismo 20mg and losar h in the morning, atrva 20 mg in the afternoon and stamla beta along with ismo 20 mg before going to sleep. I've been doing this for a fortnight now. My othe blood and urine reports are quite normal. I'm walking a bit (3 km in the morning and evening). My diet has highly restrict or nil salt levels. I'm drinking a glass of buttermilk made of milk containing 1.5 fat level. I feel ok with this regimen but when BP is fluctuating between 140 & 160 sometime in the evening i get abit scared. When I take Stamla beta I feell ok in half an or so. Ido a clerical job on an eight hour shift. It'll be a great help if a cardiologist can be kind enough to reply. I want o be anynymous. Doctor: Hi ThereIts a pleasure answering your query, i have gone through the medical details provided by you and i want to tell you that all the routine you are following is pretty good, it only need some minor adjustments.As you have mentioned your Blood pressure go high generally in the evening so i would like to suggest to start taking your Stamlo beta and ismo in the evening around 6 pm instead of taking it in the night and YOU will be good.Left Ventricular Hypertrophy is not a disease its is a sign that patient has Hypertension.Good Luck"
},
{
"id": 189400,
"tgt": "Have smell even after bowels, senokot liquid subsided. Have bad breath, told nothing gave mouthwashes",
"src": "Patient: Hello , although I go to the toilet every morning (poo), by about 10am to 11am or after I eat anything, I can still smell myself it doesn't help that I work with a colleague with an over active sense of smell which has consequently made me very concious. As a result I have resorted to taking senokot liquid 10mls every night but not sure if this is sustainable but the odour has gone. (before this I was taking epsom salts but I found that rather too drastic to take on a regular basis) Also for the past few years I have had terrible terrible bad breath, I have spoken to my dentist who said there is nothing I can do just use mouth washes. Do you have any suggestions.I will be back in 10 minutes have to collect my daughter from gym club Doctor: Hello, Thanks for posting your query. Bad breath is usually occurs due to poor oral hygiene maintenance. Plaque and calculus deposition can cause ginigivitis and thus bad smell from mouth. Please do visit a dentist and get scaling done. Use soft-interdental toothbrush and clean your teeth after every meals. Frequent warm saline gargle may help. Also get your blood sugar level checked. Get your medicines replaced if causing difficultis. Also consult a physician regarding your physical health is concerned. Hope this helps."
},
{
"id": 118527,
"tgt": "Drug induced bone marrow suppression has caused low platelet counts. Undergoing platelet transfusion. Any homeopathic treatment for this?",
"src": "Patient: Hi,my brother 26 suffered from drug induced bone marrow suppression which has been diagnosed by bone marrow aspiration.he was high temp and rashes so he was initially suspected for Dengue but on ELISA it was negative. His platelets nearly 30,000 and white cells 15,00 only while hematocrit normal and liver functions normal. In hospital been transfusing platelets and homoeopathically using Crotallus hor 30, Phosphorus 200, Ars alb 30, F. Phos 6x,eupatorium perf 200 and Ipecac. Please if some homeopathic meds which can activate bone marrow and increase platelets and white cells to normal values. ThanksAliLondon Doctor: hi Alinow i am not sure about homeopathic medicines but there certainly is a medicine that may help youtab revolade 50 once dailycheck liver functions and platelet counts regularly while on itNote: revolade is not specifically indicated for this. but i know hematologsts who have tried this drug for your kind of condition with relative success"
},
{
"id": 354,
"tgt": "Can pregnancy happen if condom falls off during intercourse?",
"src": "Patient: Hi I have 2 kids. I am not on any BC and I just finished nursing and got my period back for the 1st time. My husband and i have never had this happen on the 30th of dec we had a conom fall off I was excpected to ovulate yesterday. what are my chaces of being pregnant? Doctor: Hello,Thanks for posting your query. If ejaculation occurred in the condom and there was a dislocation of the condom, there is a risk of spill into the vagina. Since you are in the fertile period, chances of conception are high. If you do not want another pregnancy now, you can use the emergency pill or go for the insertion of an IUD. Hope this helps."
},
{
"id": 170099,
"tgt": "How can autistic children be managed?",
"src": "Patient: My child is a othistic clild,he is 5 years old and so much sleam, he donot do anything corectly and all time be afred.he know the math,meaning,translet but donot tell without pressure. I want to treatment him. Pls give me the instruction and medicine name. Doctor: DearWelcome to HCMWe understand your concernsI went through your details. Autistic children are always treated as special children. They need care. The goal of autism treatment should be to treat all of your child's symptoms and needs. This often requires a combined treatment approach that takes advantage of many different types of therapy.Common autism treatments include behavior therapy, speech-language therapy, play-based therapy, physical therapy, occupational therapy, and nutritional therapy.Caring for a child with an autism spectrum disorder can demand a lot of energy and time. There may be days when you feel overwhelmed, stressed, or discouraged. Parenting isn\u2019t ever easy, and raising a child with special needs is even more challenging. In order to be the best parent you can be, it\u2019s essential that you take care of yourself.Don\u2019t try to do everything on your own. You don\u2019t have to! There are many places that families of autistic kids can turn to for advice, a helping hand, advocacy, and support. Go through many articles about autism and treatment. If you require more of my help in this aspect, please use this URL. http://goo.gl/aYW2pR. Make sure that you include every minute detail possible. Hope this answers your query. Further clarifications are welcome.Good luck. Take care."
},
{
"id": 219419,
"tgt": "What are the symptoms of potential pregnancy?",
"src": "Patient: Well, I used to be on the depo shot. Only received 2 injections. My last was the beginning of december. I started on a regular cycle (for me) in march. At the start of may me and my partner have started to have unprotected sex. At the end of may (26th) I got what should have been my period. It was more of a brownish pink discharge and really only saw it when I wiped. I had that for about 4 days. Two weeks later I had some more of that discharge which lasted 3 days. With that one I started to get weird symptoms. My breasts feel sensitive, nausea, light cramping, back ache, uncomfortable to sleep on stomach, can t even wear tighter jeans around the waste, weird feeling in uterus, and some pulling more in the upper stomache area. I took a hpt last tuesday (negative. Then did a blood one at the doc (negative). Now the 21st close to when I m suppose to start my period, I m getting the same discharge. Still having those symptoms. But now with gas, and an increased appetite. I don t want to say I m pregnant, that would just get my hopes up. So what do you think? Doctor: Hi,I have gone through your query and understood the concern. Your symptoms can occur during a very early pregnancy or during the premenstrual phase of the period due to progesterone effect. The vaginal discharge however, is abnormal and most likely due to hormone imbalance. Since you had both blood and urine pregnancy tests negative, pregnancy can be ruled out. A trans-vaginal sonogram will help to know the status of the pelvic organs. Weight disturbances, infections, stress, polycystic ovaries and certain medications also can create this clinical picture. Please get further help from a specialist in person. Wish you good health."
},
{
"id": 98161,
"tgt": "u",
"src": "Patient: j Doctor: hi it may be due to oil retention. just like pimple on face u develop it over vagina its normal ,u need to keep it dry and clean apply antibiotic cream over it dont pick ,scratch or pop it"
},
{
"id": 150600,
"tgt": "Hard lump on c6-c7, nerve pain, muscle spasms in area. X-ray normal. Treatment?",
"src": "Patient: I have a hard lump at the back and to the Left Of my neck at c 6 to c7 . My doctor has referred me for an ultrasound. I have had a normal exray with no evidence of abnormality. It flares up from time to time and causes nerve type pain and all my muscles around the area to spasm . I am having an ultrasound. I have had this for 2 years now. What are your thoughts in what it could be. Regards Doctor: Welcome to HCM. Most probably it is swelling in the subcutaneous fat. It is also known as hump. Sometimes it is painful. If possible upload a clinical picture of your neck and upper back. Usually local measures such as physiotherapy may help to relieve the pain. We need to see your posture and if you have protruding neck with abnormal spine curves. Is it painful? is it just looking bad? whats your concern pain or cosmesis? Thanks."
},
{
"id": 17817,
"tgt": "Suggest medication for choking,rapid heart beat and pain in right arm",
"src": "Patient: hey doctor i woke up 1 morning and felt as if i am chocking and i couldnt take deep breath and then my heart started beating faster than ever...and now i am very weak i have pain in my right arm and i get very tired and feel sleepy all the time..my head is heavy all the time and i dont feel like doing anything..apart from that i am having awful trapped gas inside me and i hear bubbling sound under my left breast please help me doctor its been 2months and still i am in a bad position..my doctor told me that i am going throught bad anxiety which is out of control..and yes i forgot to mention that i am also having pain in my neck and shoulders and also in my back.. Doctor: Hello, I passed carefully through your question and would explain that your symptoms could be related to gastro-esophageal reflux. For this reason, I recommend performing fibrogastroscopy. Anxiety may be playing an important role in all this clinical scenario. For this reason, I recommend try to focus on your anxiety. Yoga, meditation and regular physical activity can help in this regard. Hope I have answered your query. Let me know if I can assist you further. Regards, Dr. Ilir Sharka, Cardiologist"
},
{
"id": 216266,
"tgt": "Does the mild angina curable with medication?",
"src": "Patient: Hi, I have been diagnosed mild hypertension a year ago &put on medication viz: Ridaq 25mg daily po. I used this medication, then I experienced sharp throbbing pain radiating to the left arm, which I explained to my physician same day when consulted, tht I have something like angina symptoms which he confirmed is a mild angina. I m currently using atenolol,amtas5, disprin 150mg daily together with Ridaq 12,5 mg daily, the pain is still there but am also using Isordil prn,which has a good effect on pain. I want to know, is the angina going to stop/ is it like the chronic hypertension? Doctor: Hard to say. New onset angina would be a reason for a lot of interventions like cardiac catheterization to see if there are blockages. Sometimes the pain isn't from the heart but mostly it would be and mostly it would be chronic.The situation is dynamic though. and will tend to get better or worse with a significant risk of heart attack and heart problems."
},
{
"id": 205423,
"tgt": "Suggest remedy for vomiting while on anti-depressants",
"src": "Patient: I HAVE A FRIEND who is getting her tattoo removed on her lower wrist it seems every time she is doing this with in 24-48 hours she is vomiting and running to the bathroom she is also on anti- depressants should she be worried as she is unable to eat anything or to keep it down is this a side affect of the treatment Doctor: Dear, We understand your concernsI went through your details. This could be natural allergy reaction to the \"removal-of-tattoo\" process. These nausea and vomiting do happen in such allergy conditions. You should have mentioned what is the anti-depressant your friend is taking. Usually anti-depressants do not act negative in 24 - 48 hours. Do consult a psychiatrist.If you require more of my help in this aspect, please use this URL. http://goo.gl/aYW2pR. Make sure that you include every minute detail possible. Hope this answers your query. Further clarifications are welcome.Good luck. Take care."
},
{
"id": 56648,
"tgt": "Suggest treatment for a gall bladder condition",
"src": "Patient: I had my only gall bladder flare up last week. I had an ultra sound and cat scan that shows inflammation along with sludge within and along the walls. The pain was intense and lasted around 35 hours. My GP wants to remove the gall bladder but the internist that performed the endoscopy is not as pro active. What do you say? Do the pros outweigh the cons? Doctor: Hi,Thanks for posting your query.I am Dr.R.K and I am pleased to assist you.You are having signs and symptoms of acute calculous cholecystitis. The ideal treatment is to get the gallbladder removed. Though you have recovered completely from it, there is 75% chance of a recurrence within one year. So I would advise you to get your gallbladder removed.Gallbladder can be removed laparoscopically.I hope that answers your question.Regards,Dr.R.K."
},
{
"id": 226024,
"tgt": "Birth control Pills, not had periods even after stopping. Breasts are sore and full. Pregnancy test negative. Advise?",
"src": "Patient: hey, i just have a few symptoms im concerned about, im 22years old and have one child, first concern is i had been on the depo injection for 3years i stopped and last injection was in october last year (2012) it is now june (2013) and i havent been on any other contraceptive and i still havent had a period, but i have had white discharge since i havent been on it aswell. Second concern is my breast have been very tender and sore (i have taken pregnancy tests and all come back negative). on my areola i have got a pimple like lump its red and white discharge keeps coming out of it aswell. For about 2-3weeks my breasts had swollen up a cup size and were full and heavier but have now gone back down. Any answer to my concerns please?? Thanks. Doctor: hi i am dr usmanfirst of all you should know that prolonged use of oral contraceptives can alter the hormonal mechanisms and periods are controlled by hormones. if you are breast feeding then periods can be delayed. also have an ultrasound done to look into uterine status and get a blood pregnancy test. if all negative consult a gynaecologist which can prescribe medicines to start periods thanks. breast tenderness is generally due to hormanal irregularites as well. thanks"
},
{
"id": 127384,
"tgt": "What causes sharp pain behind the shoulder blade?",
"src": "Patient: Hi. I have had a sharp pain that is located behind my left shoulder blade and sometimes travels down my left arm. It acts up the most when I am sitting at my desk at work and gets worse as the day goes on until I get home to rest. It does not hurt when I am sleeping. I m thinking a pinched nerve. I had this problem 2 years ago. I went to get checked and x-rays showed nothing. It went away after a few months and last year I was fine. Now it s back. Please help Doctor: Hello and Welcome to 'Ask A Doctor' service. I have reviewed your query and here is my advice.Considering the fact that you had this shoulder pain two years ago, that you have it during your work sitting at the table, and it gets worse as the day goes on. In my opinion your complaints might be related to a cervical pinched nerve due to a disk herniation, or it might be related to rotator cuff problem on the shoulder joint.These problems both are commonly accompained by shoulder pain that radiates to the arm or hand, and it gets worse by bad posture.To put the right diagnosis you should consult a doctor for a physical examination, and MRI is the best examination to diagnose this problems.For pain relief you can try taking Naproxen , (antiinflammatory drug) daily.Exercises and physical therapy also help and try to keep a good posture during your work.Hope I have answered your query. Let me know if I can assist you further."
},
{
"id": 203211,
"tgt": "Is it a nerval disorder to have less semen?",
"src": "Patient: hi doctor actually am 26 male I started masturbating from 14 approximately am leaned body usually I eat once a day or rare irrelevant in night sleep as well I cannot suppose to stop masterbating any side effects or nerval disorder or malfunctions ,actually I have less semen release from day one I started its comes like 1 or 2 drops when it feels erect I do 4 times a day every day Doctor: There is no side effect for mastrubation. It is phychological. You requere counselling. You have nodisorder or malfunction. All are phchycological disrder, it requeres proper counselling."
},
{
"id": 136921,
"tgt": "What causes numbness in the arm?",
"src": "Patient: During the evening I am awaken by a numbness in my arm, or sometimes both. It eventually goes away if I squeeze a rubber ball or just hang it over the side of the bed or hold it to my chest. Most of time when it wakes me my arm is above my head. What is causing this? Doctor: Hello, I have studied your case. Due to compression of nerve root there can be numbness in arm.Also there can be large disc bulge which is compressing over spinal cord.For these symptoms analgesic and neurotropic medication can be started.Till time, avoid lifting weights. You can consult physiotherapist for help.Physiotherapy like ultrasound and interferential therapy will give you relief.I will advise to check your vit B12 and vit D3 level.If nerve compression becomes symptomatically severe then surgical decompression will give permanent relief.Hope this answers your query. If you have additional questions or follow up queries then please do not hesitate in writing to us. I will be happy to answer your queries. Wishing you good health."
},
{
"id": 1830,
"tgt": "When do i need to take home pregnancy test after delaying period?",
"src": "Patient: I have been trying to conceive since march of this year. My last period was on the 13 of june and i was suppose to start on the 11th of this month (july) so im now 2-3 days late. I have cramping its not as bad as it usually is when im on my period. Ive gotten a lil bit of acne which is a normal sign for me during my menstrual cycle. I was wondering if i could maybe be pregnant and is it too early to get an accurate at home test result Doctor: Hi, I think you can do a urine pregnancy test at home. It can tell you if you are pregnant or not. Test shows the results as early as 2 days after you miss your periods. Hope I have answered your question. Regards Dr khushboo"
},
{
"id": 54053,
"tgt": "Does Crestor affect SGOT and SGPT level?",
"src": "Patient: I have high cholesterol and I have taken Crestor for 1 month. After lipid panel, my SGOT and SGPT doubled. My doctor wants me to come back for retest in 1 week. Can I expect much of a drop in scores in that time. I am a 65 year old woman. 5'9 and 150 lbs. Doctor: Hello. Statins like Crestor can elevate liver enzymes, and a small elevation is nothing serious, but in your case, were I your doctor, after rechecking your levels, if they were correctly reported, I would discontinue the Crestor"
},
{
"id": 124035,
"tgt": "Is there a treatment that will help polio affected leg into normal size?",
"src": "Patient: Hi Doc, My wife has gone through an operation in her polio effected leg. and now her leg is in shape but very thin. but she can walk without any support cant use stairs much. means able to walk up to 2 stair building. can u suggest us wat kaind of treatment will help her leg to be as same size of normal leg. Doctor: Hello, If she is polio and operated then you should hire a physiotherapist and start exercising every day to have a good quality of life. Usually, people with polio tends to develop muscle weakness and some orthopedic injuries in future, as the body is pathologically applying the pressure without even the knowledge of individual on soft tissues. In my practice, I have seen cases with polio and post-polio syndrome and most do well with physiotherapy. Hope I have answered your query. Let me know if I can assist you further. Regards, Jay Indravadan Patel, Physical Therapist or Physiotherapist"
},
{
"id": 126308,
"tgt": "How can increasing pain in the hips that is causing difficulty in walking be treated?",
"src": "Patient: I fell on the ground 3 weeks ago landing on my Lt hip. The problem hip was not bad and I got up without a problem. The pain has increased in the hip making it difficult to walk. If is worse at end of the and i heard a loud poefand acute of pain for about 1 min. I have been walking better today. Could I have dislocated my hip before, Doctor: Hi.Thanks for the query in HCM.It could be just a soft tissue injury around the hip.Kindly take rest and a simple analgesic for two to three days,If no relief. Kindly meet an orthopaedician. He ill suggest u X ray/ MRI of hip. Which will diagnose the problem and further treatment can be considered. In pain is severe, u need to consider a simple fracture and meet an orthopaedician.Thank u"
},
{
"id": 109552,
"tgt": "What causes upper back pain and pressure with slight itching?",
"src": "Patient: Hi I have question yesterday I felt like a lil pain on my upper back it's sharpy and I felt so much pressure after a while I started feeling lil bit of fever n itchy so I notice a red/ pink bump and it started hurting more I could feel it and still feel it if I lain back could it be an absess or what I'm scare of this Doctor: Hi,As this development occurred for one day all of a sudden indicates that it might be due to something insect bite causing allergic reaction to surrounding tissue producing this pain and itchiness.Another possibility of having some ingrown hair follicle producing boil or forming abscess.Wait for a day or two, if problem increases then go for one antibiotic medicine course for 3-5 days.Take some antihistamine as well.Ok and take care."
},
{
"id": 126881,
"tgt": "How can pain and swelling in the legs and ankles be treated?",
"src": "Patient: I fell last week twisting both ankles on the way down. I felt a lot of pulling and tearing. Now I have a lot of bruising on both ankles, swelling and a lot of pain when I stand or walk plus I have bruising and a lot of swelling in both knees. The ortho doctor looked at briefly and said that I sprained both knees and both ankles and there was nothing that can be done but time. I\u2019m now experiencing radiating pain up my legs and pain in my toes and arches. Doctor: Hi, It can be due to sprain or minor contusion developed following that injury. As of now, you can take analgesics like Acetaminophen or Tramadol for pain relief. Apply ice packs. If symptoms persist, better to consult an orthopedician and get evaluated. Hope I have answered your query. Let me know if I can assist you further."
},
{
"id": 46716,
"tgt": "How can pus in the kidney be treated?",
"src": "Patient: hi..i'm 25years old and 53kg. 2 month ago i've been diagnosed with puss in the right kidney after doing CT Scan. 1.how puss formation happen in kidney? 2.what the causes of puss formation? 3.can a antibiotics treatment heal this puss formation..what the other treatment? Doctor: Hi1) Pus formation in kidney can be due to blockage to urinry tract, urinary tract stones or sometimes due to infection from blood stream in patients with diabetes or immunosuppressed or due to tuberculosis. 2) Pus formation is due to infection3) If there is obstruction this needs to be relieved along with antibiotics. Occasionally the pus have to be drained by operation by a radiologist by placing a needle into the kidney. Specific treatment will be based on underlying cause."
},
{
"id": 222089,
"tgt": "What are the risks of being conceived in 3months after still birth?",
"src": "Patient: I lost my baby girl 5 months ago at 34 weeks and 5 days pregnant. After delivering a stillborn, cause unknown. My husband and I wanted to fill our empty arms. We are pregnant and conceived about 3 months after delivering our daughter. Yet I only had one cycle in between these pregnancys. My question is have I put this child at a greater risk by not waiting the 3 cycles suggested? What are the chances that this child will live. Honest factual answers only please. Thank you for your time. Doctor: HI, I understand your concern. Actually it is desired to get rid of all the possible causes for the mishap & have 3 normal cycles before next conception. BUT any way you have already conceived.. so now instead of being stressed, you should consult a gynecologist earliest-, get some investigations done ,.. have a guess for the cause & try to prevent repetition again by timely medication. In some conditions complete irradiation of the flaw is essential, in that case medical abortion may be advised. Please don't panic, go ahead with positivity.. Thanks."
},
{
"id": 212584,
"tgt": "Feel depressed, tired, lonely, hate going to school, people around. How to deal with it?",
"src": "Patient: Hi, I hate school.Not the education but the institution;that big place with many stupid people I have to deal with over and over again.Going to highschool is HELL for. I just can t do it anymore.I m depresive and tired and there s too much pressure.I think I may have social anxiety?I read about this but I m not sure.All I can say is,I feel so lonely.BUT somehow is better to be on my own.I can t stand people / classmates /teachers.I hate everything and everyone ,I even hate the people I love(does it make any sense?).I hate myself.I don t know how to deal with my life anymore. Doctor: Hello. Thanks for writing to us. What you are going through right now seems to be a behavioral disorder. You need to attend a few sessions with your psychotherapist for some behavioral therapy. The symptoms will improve gradually with proper medicines. I hope this information has been both informative and helpful for you. You can consult me again directly through my profile URL http://bit.ly/Dr-Rakhi-Tayal Regards, Dr. RakhiTayal drrakhitayal@gmail.com"
},
{
"id": 48922,
"tgt": "Do I have to see an urologist if having calcification in kidney?",
"src": "Patient: Hi do i have to see a urologist just found out on ultrasound that herez a 8mm cortical calcification in my upper pole of right kidney(i went in for a liver and abdominal ultrasound originally cuz i am a hep b carrier so my hepatologist wanted the ultrasound to make sure everyhing was ok) i have right flank and rib and right lower thoracic pain around the kidney area that wakes me up but i had a hockey related injury about 1 1/2 yrs ago where i sublixed my last 2 ribs on same side so ive always attributed my pain to that injury and didnt think anything of it unitl now Doctor: hai,from your relevant history possibility of the injury may cause the calcification in the right kidney. pain may be because of that too(calcification in kidney.)correlating with your associated symptoms will help in ruling out the cause.however ,i do suggest a urologist for your ultrasound report for further opinion and management.thank youhope i answered your query"
},
{
"id": 169202,
"tgt": "Suggest remedy for aggressive behaviour in a child",
"src": "Patient: Hi I have a 5 yr old nephew that has aggression issues. He hits children in school and has been in trouble numerous time.He also is delayed in some areas like holding a pencil tracing letters and can not stay focused for more than a couple seconds. He also doesnt understand emotions. I was wondering if autism is a possibility. Doctor: Hello. Thank you for your query. Yes it might be a possibility for autism, but you can't know for sure if you don't test your nephew for this problem. Best think you can do is to take him to a doctor and see what is going on with your nephew."
},
{
"id": 154191,
"tgt": "How to confirm cancer?",
"src": "Patient: I had a lission about the size of little finger nail removed for biopsy, it is irrgularly shaped and slightly raised. it has been a weeknow waiting for the results, they said the results are taking longer becouse the Oncologist is looking at a \" deeper cube or slide\" of the lission. first if i can send you a pic of this lission can you tell me what you think. and second why would he want to look deeper Cube or slide\" should that scare me or set me more at ease. Thank you Greg Cant sleep or eat Doctor: Hi, dearI have gone through your question. I can understand your concern. You may have some benign lesion or cancer. You should wait for your biopsy results. If you send me pictures of your histopathology images then I can tell you whether it is cancer or not. From picture of lesion we can't say that. They are taking deeper section. It is to search invasion in to deeper structure or sone cancer foci. Don't worry much about that. Just be relaxed. Wait for your biopsy results. Then you should take treatment accordingly. Hope I have answered your question, if you have doubt then I will be happy to answer. Thanks for using health care magic. Wish you a very good health."
},
{
"id": 44965,
"tgt": "Is high SHPRL curable by medicine ?",
"src": "Patient: I have SHPRL level 40.90.I am nonpregnant women. Infertility problem is there.Will it be cured by medicine? Doctor: THanks for the query Levels of 430 to 750 is considered normal U r suffering from hypoprolactinemia but this exactly is not known to cause infertility issues but none the less it should be considered as one of the causes of nothing else could be isolated. Treatment is available, u will have to go to an endocrinologist and he might prescribe u Carbagolin tablets to be taken for sometime/life long Have a healthy life"
},
{
"id": 15580,
"tgt": "Rashes around all over the body, itchy. Started after taking antibiotic for epidural. Taking benadryl. Remedy?",
"src": "Patient: Hi my name is leah, i just give birth last july 1, and i had develop a rash from one of the antibiotics that they gave for my epidoral.... i had a rashes around my tommy they gave me a benadril prescription from the hospital. The rash is gone on my tommy but i had another rashes again all over my body now its redesh and super itchy what can i do with this? Doctor: Hello,Thanks for the query.You have developed a drug allergy.Keep taking benadryl, this will help in controlling itch.For the rash you can apply a mild steroid cream.Mild steroids can be used during breast feeding, so dont worry regarding that.Let me know if you have any other doubt.you can ask a direct question to me on this forum, following the below link.https://urldefense.com/v3/__http://www.healthcaremagic.com/doctors/dr-rahul-kumar/64818Wishing__;!!Mih3wA!SBzm6_kI6hCZ58EPH6N_05MFfiPbxWXT0a2TJCdFQObRWm5mV5ur7hUOMa8clQ$ you a good health.Thank you"
},
{
"id": 202343,
"tgt": "What to do if having sensation of extra pressure in the glans and painful ejaculation after masturbating?",
"src": "Patient: HiDue to a sressful few months i have been masterbating 2 or 3 times a night. Over the past couple of nights i have felt a sensation of extra pressure in the glans and painful ejaculation. I am 32 and have not had sex, protected, for nearly three weeks. Is this something to be worried about? Doctor: Good Day and thank you for being with Healthcare Magic! I believe you may have strained your penis and testicles from over masturbation. this condition usually resolves in a few days to weeks with abstinence and rest. Just take it easy and there will be no permanent side effect due to over straining. I hope I have succeeded in providing the information you were looking for. Please feel free to write back to me for any further clarifications at: http://www.HealthcareMagic.com/doctors/dr-manuel-c-see-iv/66014 I would gladly help you. Best wishes.Regards, Manuel C. See IV, M.D. DPBU FPUA"
},
{
"id": 30404,
"tgt": "What causes bumps on stomach and breast?",
"src": "Patient: Hello i am really confused about what it is i have.it started with boils under my armpits iwent to the doctor and was given antibiotics.then i developed 2 ringworms and which i started using blue star.now i have alot of bumps on my stomach and breast.what could this be? Doctor: Hi, I understand your concern. 1. The boils you had under your armpits are due to possible skin/hair infection. That's why your doctor prescribed antibiotics. 2. Ringworms are fungal infection of the skin. Here, there is a case for discussion. As the ringworms skin eruptions started immediately after starting antibiotics, my suspect go for possible allergy reaction to that antibiotic. As you have developed a lot of bumps on your stomach and breast, then, this for me, confirms allergy. I suggest to stop that antibiotic, and to report to your doctor as side effect. 3. You can also provide a photo of the bumps to me if you need further specific opinion. Hope it helped!Dr.Albana"
},
{
"id": 1309,
"tgt": "What are the effective ways to get pregnant or any pill will work?",
"src": "Patient: Hi, I'm trying to get pregnant and the last time I got my periods was on 30th septI have taken several pregnancy tests and they all ate negative butI'm not getting my periods. I wanted to ask should I induce periods with somePill. Please give me the name of the pill . Doctor: Hi, I think you can take some time like progesterone to induce the periods. But before that you should go for an ultrasound to reconfirm the pregnancy status. Better consult a doctor. Don't take medicine by yourself. If you are planning to get pregnant, get yourself evaluated. Do a thyroid profile and prolactin levels. Also do a semen analysis of your husband. If everything is fine, you can take some medicines like clomiphene for the growth of your follicles and track your follicles growth by repeated ultrasound and when your follicles is more than 17 to 18 mm, take injection for rupturing the follicles. Be in contact with your husband for 2 to 3 days after injection. Take progesterone for next 2 weeks. Do a urine pregnancy test at home after that. You can try like that for 3 to 6 months. Hope I have answered your question. Regards Dr khushboo"
},
{
"id": 197978,
"tgt": "Are painful erection, burning sensation and frequent urination signs of prostate infection?",
"src": "Patient: I ami tajine antibiotics for what my urologist suspects may be a low grade prostate infection. My prostate is normal as well as my bladder. No cancer either. However after one week the pain and frequent urination is still occurring. Also painful erection and ejaculation burning sensation. Penis also has developed an hourglass Appearance intermittently. What else can this problem be? Doctor: HelloThanks for query .Frequency for urination ,painful ejaculation .burning urination that you have strongly suggest active infection of Prostate (prostatitis) Please consult qualified Urologist for clinical and digital examination and get following basic tests done to confirm the diagnosis.1) Urine routine and culture.2) Ultrasound scanning of abdomen and pelvis3) Serum PSA.4) Semen Examination and culture 5) Swab test of urethral discharge obtained after digital prostate massage .In the mean while start taking antibiotic like Doxicycline along with anti inflammatory medicine like Diclofenac twice daily .You will need to take these drugs for 4-6 weeks Further treatment will depend upon result of these tests and final diagnosis.Dr.Patil."
},
{
"id": 117669,
"tgt": "How to control low RBC and high MPV levels?",
"src": "Patient: Hello Doctor, I am 35 years Old, Male, 5ft,2 inc Tall, 74KG, 36 Inches waist, I want to check if the attatched report has any concerns. I am concerned about the Low RBC count and High MPV. Also i am having Dry Cough for more than 3 weeks now. Thanks and Regards Manoj M Doctor: hi, your low rbc count has no relation with high mpv. you should search for the cause of your low RBC. it may be due to low iron, low vit b12 or may be due to chronic blood loss or any other cause. search the cause and take treatment accordingly. and don't worry regarding high mpv. its ok. and also take treatment for your chronic cough,thanks for using health care magic."
},
{
"id": 99520,
"tgt": "What causes red spots on chest?",
"src": "Patient: I have red spots on my chest that have spread and become more. I am not sure what it is. They are not itchy at all. It started with one then another and another and now it is getting to be a lot. It is between my breasts on but spots are starting to appear on the insides of my breast and rising up my chest. Do you know what this may be? Doctor: Hi,This red spots could be urticaria, eczema,tinea versicolor or corporis,psoriasis,shingles etc.If there is no itching then chances of fungal infection (tinea) would be less.It will require to get diagnosed first by doctors inspection.Treatment is mainly depend on the the diagnosis.So,better to consult dermatologist immediately rather than waiting as it is spreading continuously."
},
{
"id": 21467,
"tgt": "What causes muscles in the heart to get thick?",
"src": "Patient: my father 64 years, suffering from cardiomyopathy...in which case heart muscles get thickened. he is under medicine (solet) for the same for last 3 years. however inbetween he has suffered from cerebral stroke and now almost every two months lungs infection, congestion and acute breeting trouble is there. Is there any treatment in ayurveda for this? Doctor: Hi ThereAfter going through he medical details of your father i would like to tell you that Hypertrophic cardiomyopathy (HCM) usually is inherited (Genetical) . It is caused by a change in some of the genes in heart muscle proteins. HCM also can develop over time because of high blood pressure or aging. Diseases such as diabetes or thyroid disease can cause hypertrophic cardiomyopathy.No Ayurveda do have treatment for disorders like hypertension, thyroid, diabetes but not for HCM. Although in modern medicine it can be corrected with Surgical Shaving off of the heart muscles or by Alcohol Septal Abaltion.I wish him good health"
},
{
"id": 27573,
"tgt": "Suggest treatment to control blood pressure",
"src": "Patient: last summer I blacked out I was taken to the er before that I was having eposides of getting very hot wanting to faint feeling like my clothes are squezzing me my blood pressure will drop to example 80 over 60 at some point they put a heart monitor on me. I was told my heart stops 3-4 seconds at a time when I saw a heart doctor he said it was NO BIG DEAL is Doctor: Hello!Thank you for asking on HCM! I understand your concern. Your symptoms may have been caused by a low blood pressure related to dehydration and hot temperatures. If you have not had any symptoms since that period, there is nothing to worry about. I recommend you to take plenty of water and juices and avoid staying under extreme weather conditions for a long time. If you have symptoms like fatigue or fainting, I recommend you to consult with your GP and perform an ECG and an ambulatory 24 hour blood pressure monitoring.A Head Up tilt test would be necessary to exclude orthostatic hypotension. Hope to have been helpful!Greetings! Dr. Iliri"
},
{
"id": 5783,
"tgt": "Anxious to conceive, PCOD, undergone laproscopic surgery, ovarian drilling. Can I conceive naturally, or injections required?",
"src": "Patient: Hi this Pooja from India. Age 28 anxious to conceive but suffering from PCOD recently undergone laproscopic surgery and ovarian drilling. My question to you is for anovulatory cycles, if I stop taking birth control pills will I ovulate naturally and can conceive or I should go for injections to enhance egg growth and IUI treatment.pls help Doctor: Hi Pooja, with PCOS and the hormonal imbalance it is less likely that you could ovulate naturally. For infertility treatment,protocol is to start with tablets for ovulation and if you don't respond to them then switch over to injection for ovulation n then IUI so I would advice you to go in a step wise manner and not directly jump to iui or injections consult your doctor and seek advice."
},
{
"id": 169558,
"tgt": "Suggest treatment for bloody diarrhea in kids",
"src": "Patient: My 5 years old girl have bloody diarrhea since friday and today is the 4th day, took her to the dr yesterday and she put her on septra but the bloody diarrhea is worst and she has tommy ache don t now what to do pls help (ps) the dr sent her stool to the lab but it won t be ready until 72 hours. Doctor: Hi Dear,Welcome to HCM.Understanding your concern. As per your query your daughter have symptoms of blood diarrhea which seems to be due to inflammation of gastrointestinal tract. Need not to worry. I would suggest you not to panic at all. You should start treatment with combination of fluroquinolones and ornidazole.You should give child rice with curd or buttermilk. You should get intravenous fluid administration as well. Avoid taking Domperidone. you should avoid taking dairy products in excess. Give child Walamycin syrup for controlling infection. Avoid hot, sharp and spicy food. You can take banana on daily basis and light food. Drink plenty of fluids to rehydrate yourself. Visit gastroenterologist once and get it examined and start treatment after prescription.Hope your concern has been resolved.Get Well Soon.Best Wishes,Dr. Harry Maheshwari"
},
{
"id": 58489,
"tgt": "Had gastric bypass, gall bladder removed. Had internal bleeding, assumed as ulcer. Chances of bleeding again?",
"src": "Patient: I had gastric bypass in Jan 2012 had my gall bladder removed dec 2012 and 5 days later was in hospital with internal bleeding . they assumed ulcer that they never found...I have had little to no answers. My doc thought ulcer was in old stomach. At time I was given 6 units of blood two bags of plasma . they were able to stop bleed through protonex and i have been put on that and a proton pump inhibator both to be takin 2x a day. What are the chances of this bleeding out again? and y did this happen Doctor: Hi and welcome to HCM .Thanks for the query. this is called stress ulcus and this is known complication of any surgical procedure. this can be very serioud condition but it is over now and it is not likely that bleeding will occur again especailly if you take antiacid medicines. So dont worry and follow doctors instructions. Wish you good health. Regards"
},
{
"id": 27378,
"tgt": "What causes shortness of breath , heart palpitations and dizziness?",
"src": "Patient: Hello I ve had 3 ekg and all came back normal..buy of course when it s time to.go to the dr what you go for it never happens when you get there. But I ve been having heavy heart palpitations and shortness of breath, and dizziness. I have low blood pressure. I don t smoke,drink, do drugs. I m not on any medication never have been. Two years ago my lab test came back high cholesterol Doctor: Hi, if the symptoms occurs everyday than the beat is to get a 24 hour holter, which will monitor your ecg for 24 hours and detect the pattern during the time of palpitations. Accordingly it can be treated. Also avoid caffeine, aerated beverage and taking too much of any stress"
},
{
"id": 82036,
"tgt": "What causes chronic and frequent breathlessness?",
"src": "Patient: Symptoms: 1.\u00a0\u00a0\u00a0\u00a0\u00a0Chronic and frequent breathlessness \u2013 feeling of taking deep breaths still not sufficient; anxiety 2.\u00a0\u00a0\u00a0\u00a0\u00a0Pain in: \uf0e0 upper chest on left/right, sometimes both together, \uf0e0 left shoulder near the neck and \uf0e0 rib bone turning @ left armpit \u2013 on full breathing and pressing with finger on certain points near collar bone and little below it \u2013 mostly inside the gap between two rib bones: felt mostly only bone-deep or muscle-deep 3.\u00a0\u00a0\u00a0\u00a0\u00a0Cracking of bones or \u0939\u0921\u094d\u0921\u0940 \u091a\u091f\u0915\u0928\u093e near shoulder and chest @ stretching etc 4.\u00a0\u00a0\u00a0\u00a0\u00a0Small swelling just below the left collar bone 5.\u00a0\u00a0\u00a0\u00a0\u00a0Pain and little numbness propagating in the arms \u2013 mostly left arm 6.\u00a0\u00a0\u00a0\u00a0\u00a0Pain in/near deep RIGHT lower rib and same area in the back @getting up in the morning 7.\u00a0\u00a0\u00a0\u00a0\u00a0Feeling of contraction (\u091c\u0915\u095c\u0928) in almost complete chest and upper back @ breathing 8.\u00a0\u00a0\u00a0\u00a0\u00a0Frequent bloating, incomplete evacuation, feeling of fullness, hyperacidity 9.\u00a0\u00a0\u00a0\u00a0\u00a0Pain and burning in complete chest and back starting sometime after upper body exercise 10.\u00a0\u00a0\u00a0\u00a0\u00a0This condition since more than a year \u2013 fluctuating. Goes away and comes back, mostly when stomach problems are more. Diagnosis done by other doctors: 1.\u00a0\u00a0\u00a0\u00a0\u00a0Costochondritis\u2013 X-ray done for 1st costochondral junction \u2013 told it was okay; ECG okay 2.\u00a0\u00a0\u00a0\u00a0\u00a0Doctor explained that the Pleura might be damaged because of some reason that is causing pain while breathing and it is propagating 3.\u00a0\u00a0\u00a0\u00a0\u00a0Tests done: CBC,ESR, FBSL, Lipid profile, LFT, Bl Urica, Serum Creatinine \u2013 pointers highlighted: \uf0e8\u00a0\u00a0\u00a0\u00a0\u00a0S. Alkaline Phosphatase in LFT = 235 \uf0e8\u00a0\u00a0\u00a0\u00a0\u00a0S. Bilirubin Direct in LFT = 0.40 \uf0e8\u00a0\u00a0\u00a0\u00a0\u00a0Blood glucose fasting = 114.5 Medications given: Tab Muskel, Tab Satrogyl, tab Raciper, Liq Pepticame, Tab Gembax, Tab Chymoral, Tab Neurokind Next, Tab moxclav, Cap Nexbro, Cap Paxidep, Tab Colospa Retard I still have pain intensity of which is fluctuating. Can you diagnose or help me/ suggest test so as to get the disease diagnosed . Doctor: First thing i wasn't to ask you here is, Are you into heavy workouts? That can explain the pain at multiple points on chest. Though I would like to review the chest Xray, just to rule out pleural, lung or bony pathology. Do you have a past history of pleural effusion (fluid collected around the lungs, most commonly due to tuberculosis)? that can make one think of pleural thickening as a cause of this pain. USG chest will be helpful if your physician doubts so.As for your breathlessness, I strongly advise you to get a Pulmonary Function Test (PFT/ Spirometry) done. It will be be helping in knowing if there is any restriction to lung expansion that is causing you to feel breathless and then it can be managed accordingly.Consider analgesics like Diclofenac to relieve pain as well as Pantoprazole to take care of your acidity.Any more queries are welcome.Regards."
},
{
"id": 130088,
"tgt": "How does carpel tunnel and flexor synovectomy work?",
"src": "Patient: i have had surgery for carpel tunnel and flexor synovectomy following a wrist injury in may i do not see the consultant till mid jan but do not know what this surgery involved as not had full explanation yet and wondered if you could explain what it may have been as surdeon did not know what he was going to do untill operation commenced. thank you Doctor: Hi i am Dr Ahmed Aly thanks for using HealthcareMagic site ,I had gone through your question and understand your concerns .. Carpal tunnel release operation means that the nerves passing to your fingers mainly the median nerve was irritated or obstructed during its passage to your fingers through the flexor retinaculum at the level of the wrist . the flexor retinaculum is a kind of sheath that sorrounds some nerves and structures passing through your wrist region so this sheath is cut to free your median nerve . Flexor synovectomy means removal or excision of a peace of the synovium which is a thin layer surrounding your wrist joint that allows flexion . may be it had a minor fissure or fracture that neede to be removed to immobilise your wrist joint . Hope you feel better after your operation and follow your surgeon instructions and you may need physiotherapy sessions with a chiropractor to manage your injured joint to fully recover . Please click and consider a 5 star rating with some positive feedback if the information was helpful. Wish you good health,Any further clarifications feel free to ask."
},
{
"id": 3066,
"tgt": "Suggest remedy for getting pregnant",
"src": "Patient: Hi we are trying a baby from this mnth. My last period is June 6th. Now we are trying a baby. My cycle is regular cycle only. Every month I got a white layer in 17 to 22 days after my periods. This month also coming. So it s a symptoms for my periods will coming this month also ah? And I feel over heat in my body Doctor: HiGREETINGS ARE YOU HAVING regular periods.? Done any ultrasound scan or hormonal analysis.? If not first get a semen analysis for your husband and ultrasound scan for you.Your thyroud hormones also need to be evaluated .Hope my answer helps you.Regards"
},
{
"id": 1927,
"tgt": "Could mirena cause any problems in getting pregnant?",
"src": "Patient: Hi Dr, I'm 32 years old. I had the marena in for 2 and a half years. (it was removed a week ago) I had a lot of problems. I google and did research and found that I have Candida (for a looooong time). I want to get pregnant, can this causes problems for my unborn child and conceiving? Doctor: Dear Ms, the Mirena is a medicated loop that releases progesterone into your body. this leads to decrease the aecreation of endogenous progesterone. I will take some time before your body starts to secreate progesterone again . there will be a delay in getting pregnant of average 6 months after removing the loop"
},
{
"id": 25410,
"tgt": "What causes increased heart beat after dinner?",
"src": "Patient: hi.im a female.age 26.since last two nights,after having dinner, after about 10-15minutes, my heart beat increases and i feel like my heart is under some pressure and some one is squeezing it very hardly, due to which i have to breath hardly. but when i sit and lie down,it goes away in 5-7minutes.. im married and trying to conceive,so i am having Pregnacare(multi Vitamins) past 2months. just informing you on the medication.im really worried.please guide me. Doctor: Hello and thank you for using HCM.I carefully read you question and I understand your concern.I will try to explain you something.There are different reasons that might cause increased heart beats. Simple physical or emotional stress, caffeine products might cause increased heart frequency so its mandatory to avoid them.After eating, our metabolism increases so it might be normal to have slight high frequencies for a while. Or it may be another problem. When you eat, the abdominal distention rises up the diaphragm and in a secondary way might trigger the heart for higher frequencies. A simple gastro-ezofageal reflux or the food that comes back from stomach to esophagus all saw might be the cause.There are different pathology's like anemia, thyroid gland problems like hyperthyroidism that might be the cause of fast heart beats. When you experience a high frequency, it might be normal to experience palpitation, chest discomfort, pressure and breathing difficulties.So, if I was your treating doctor, as you allays connect this symptoms with eating i will recommend a gastroenterologist consult, a fibrous-gastroscopi maybe to exclude any problem like re-flux, diaphragmatic hernia ect.After this I will recommend an electrocardiogram, a cardiac echo-cardiography and of course a rhythm holter monitoring for 24 h. This is necessary to have a full view of your mean frequencies during day and night.After these, a full blood chek up to exclude anemia or thyroid problems as you want to have a baby soon.Doing this examinations helps us understand if it is necessary to treat this high frequencies or not. You should not be worried, because this is nothing serious.Hope I was helpful. Wish you good health.Best regards."
},
{
"id": 160636,
"tgt": "What causes high fever and diarrhea in a child?",
"src": "Patient: My seven year old son has had a high fever for two days. 104. He also has a terrible stomach ache with nausea, but not diahrrea. He has had a loss of appetite for several days before and still does. He has also complained of a terrrible headache even before the fever. Also, he has complained of his legs aching. He is on cefdnir 1 1/2 tsp. once daily. started this five days ago. He is very clammy and can t even sleep well. His doctor ordered blood work today, but won t have it bavk till tommorrow around noon. Also his neck bothers him when the motrin is wearing off, He has had some grapes and a small grilled cheese and kept this down, but the stomach ache is still present. He will eat a little for me before I give motrin. I m scared I need to take him to the ER. Any suggestions? Also, three weeks ago he had a terrible congested cough and sinus issues. He was prescribed augmentin for that and the cough did go away, but he clears his throat constantly. The doc did look at his tonsils and they are swollen, but not pussey he said. He also did a flu nose swab and it was negative. Should I go to the ER? He is really sick.--THANK YOU FOR ANY ADVICE Doctor: Hello, You are a good historian- so your son has high fever, headache and stomach pain, poor appetite, and pain in many parts of body. In addition to his head and stomach the other pains are probably muscular. Putting all of these symptoms together makes me highly I suspicious this is caused by a virus. You feel he is very sick.Take him to ER. Hope I have answered your question. Let me know if I can assist you further. Regards, Dr. Arnold Zedd, Pediatrician"
},
{
"id": 139453,
"tgt": "What causes a sudden feel of dizziness?",
"src": "Patient: my son is 15 and complains that he feels dizzy whenever he stands up suddenly, even just from a sitting position. This happens on a daily basis. He has taken quite a growth spurt and is always hungry. His diet is resonably okay although fruit and vegetable intake is not as good as it should be but he eats a lot of protein and fortified cereals Doctor: Hello and welcome to \" Ask a Doctor\" service.I have read your query and here is my advice.Orthostatic hypotension may be a condition to consider.Lower blood sugar levels also is a possibility.Discuss with the Pediatricin for these issues.Hope this helps.Feel free to ask if you have further questions."
},
{
"id": 32880,
"tgt": "Suggest treatment for dog bite",
"src": "Patient: my son has been bitten by a dog on his bum on the 19th of may. he has been given tetanus and one injection costing rs. 340 on the 20th of may in the morning. the dog is a pet dog n has last been given vaccine in 2010. now when should i give the next injection to my son. Doctor: Hi & Welcome to HCM.I appreciate your concern for your beloved son who is being bitten by your pet dog.Your son must receive a total of 4-5 doses of rabies vaccine.Since he has already received one dose on May 20th (which should be counted as \"Day 0\"), he must take the next dose on days 3, 7, 14 and 28 as well.Meanwhile your pet dog should be observed 10 days for the following signs and symptoms of rabies like biting without any provocation, eating abnormal items such as sticks, nails, feces, running for no apparent reason, hoarse barking and growling or inability to make a sound excessive salivation or foaming at the angles of the mouth.If your pet dog remains healthy and does not show up the signs and symptoms as described above the remaining doses of the vaccine can be stopped.Hope to have answered your query.A feed back is appreciated."
},
{
"id": 82360,
"tgt": "What cause shortness of breath, chest pain and numbness in arm?",
"src": "Patient: Hi - i ve had numbness/tingling/aching in my left arm for quite a few weeks now..sometimes my hand shakes so badly i need to sit on it to still it - i carry a lot of tension in my shoulders though.. i have bronchitus at the moment and woke up today with a burning pain through my left arm, starting in my armpit - along with shortness of breath and chest pain i m just a little concerned Doctor: Thanks for your question on HCM. In my opinion we should first rule out cardiac causes as you are having left sided chest pain and left arm pain.So first get done ECG and 2D ECHO to rule out cardiac cause.If both of these are normal than get done chest x ray to rule out lung causes. Also get done PFT (Pulmonary Function Test) to rule out bronchitis as you are having it in the past.So if all the above are normal than no need to worry much. Is is due to stress and anxiety. So avoid stress and anxiety. Take good painkillers and muscle relaxant. Avoud heavy exercise and weight lifting."
},
{
"id": 40340,
"tgt": "What could on and off flu, breaking out hives on body after being treated for viral infection suggest?",
"src": "Patient: I had a viral infection of some sort since January 1. Diagnosed with flu, bronchitis, costrochondritis, low grade fevers, fatigue. I also have fibromyalgia. I broke out in a rash for about 2 weeks, and was given medrol dose pack and topical steroid cream. I still feel flu like off and on, and am still breaking out it hives - although it does not get on my face. Mostly my neck chest, back, and arms. Some areas on my lower legs. What else could it be? What else should I do? Doctor: Hello,welcome to HCM,The repeated flu episodes is may be due to allergic reactions following viral infection. The virus itself can stimulate the mast cells to release histamines to produce the symptoms of generalized hives.The symptoms are suggesting me that you may be having allergic reactions.I would suggest you 1.Cold Packs over the hives2.Oral Antihistamines with Mast cell stabilizers3.Topical Antihistamines and Steroids.If there is a secondary bacterial infection it should be treated with oral antibiotics.Thank you."
},
{
"id": 110319,
"tgt": "What causes lower backache while on treatment for urticaria?",
"src": "Patient: Hi, I have suffered from chronic urticaria for the last 30 years, & have been on prednisone. I have every test & drug known to man, & have been seen at Stanford. As I m sure you ve guessed by now, it s an autoimmune thing. However, I also have a ton of lower back issues, all the usual suspects . I was doing some research, & just saw that the T9 vertebrae being out I guess you could call it, can cause hives. Do you think this is possible, & could be what s been causing my hives all these years? Doctor: hives are common with allergy/hypersensitivity.it has nothing to do with vertebrae.you must consult an orthopedician for this problem.xray/MRI should be done for complete picture and cause of this be explored first to begin treatment for this"
},
{
"id": 210140,
"tgt": "Suggest treatments for mood swings and pain in upper ribs",
"src": "Patient: i am having extreme mood swings, i am crying all the time, so mad i could hurt someone, upper left side ounder ribs pain, this has all started in the last 6 weeks. I had my gallbladder removed about a month ago, and since then i am either ready to rip someones head off, or am crying. I hardly ever cryed before this. Doctor: DearWe understand your concernsI went through your details. I suggest you not to worry much. Gall bladder removal does not warrant mood swings. Your anxiety and depressive thoughts regarding the removed gall bladder could have become the reason for the mood swings. In fact, gall bladder removal has become so common a surgical procedure. Negative thinking or apprehensions about body ailments could trigger mood swings. The pain under your ribs could possibly be due to acid reflux. Intake of garlic along with meal should cure it.But for mood swings, you should exercise, keep yourself engaged, do your duties properly, concentrate in your education or career or both, drink plenty of water etc. You may need psychotherapy intervention. Post a direct question to me including every details. I shall prescribe some psychotherapy techniques which should cure you.Hope this answers your query. Available for further clarifications.Good luck."
},
{
"id": 22430,
"tgt": "Suggest remedies for moderately high blood pressure",
"src": "Patient: Hi, what's the best ways to reduce moderately high BP? - Mine is 151/90 last time it was checked (last Wednesday). My HDL cholesterol was measured as 1.63mmol/L. I'm 49 years old and need to loose about 2 Kgs, so Im not too overweight. many Thanks, Nick in London UK Doctor: Hi,You should try healthy lifestyle first then if doesn't respond then you will need medicine. Avoid fatty, oily and high calorie diet. Have low salt diet and monitor blood pressure regularly thrice a day for one week then once or twice a week. Regular exercises like brisk walking, jogging according your capacity at least 30 min a day and 5 days a week. Lots of green leafy vegetables, fruits, fish once or twice a week, avoid meat. Avoid smoking and alcohol if any. There shouldn't abdominal fat deposition or obesity. Get your lipid profile and sugars tested once.Hope I have answered your query. Let me know if I can assist you further. Regards,Dr. Sagar Makode"
},
{
"id": 110958,
"tgt": "What causes lower back pain with normal X-ray report?",
"src": "Patient: Hi, I am a lady with 35 years of age and 56 kg weight. I am suffering from lower back pain since last 3 - 4 days. Upon going to a doctor, he suggested me to get a L S x- ray. He says that x - ray is normal but he doubts that it is slip disc problem and wants me to do an MRI. I do not have any history of BP or diabetes. Please advice. Doctor: Hi,Please proceed to MRI, and confirm the diagnosis. In few conditions X-ray might look normal, but an MRI can give you the right diagnosis. So, it is advised to get it done.Thanks."
},
{
"id": 108686,
"tgt": "What causes shooting pain all over the body and lower back pain?",
"src": "Patient: I am a 39 year old female 145 pounds and 5 4 One I think I may be a hypercondriac as I always think I have cancer. I do suffer from anxiety and have IBS. Lately I have been having shooting pains all over my body everywhere and bad aching lower back also I get a tighening feeling in my right side under my rib and above my hip. Last I have this tiny bump under my skin beside my nose that is hard. Not really painful just kind numb feeling? Please help if you can and ease my mind Doctor: thank you for your question. I would suggest you to take elavil 25 mg two times daily and dolo-neurobion also twice daily. Apply voltaren gel locally to back after hot fomentation.Avoid lifting heavy objects.supplement Evion 400 mg also once a day.you will be alright soon"
},
{
"id": 126086,
"tgt": "Suggest treatment for burning sensation from the hip down to the thigh and knee",
"src": "Patient: I have burning from my left hip or buttock down my left thigh to my knee. I can t sleep because of the pain. I ve rubbed pain oils, creams, and spray pain relievers and none work. I take Naproxen tabs that don t help and my Dr. changed my pain med to Meloxicam but that doesn t help either. I has gotten worse in the last 4 days. I have even tried drinking a little pickle juice but no help. I just want to be able to sleep and walk without the burning pain. I feel like an 80 yr.old lady, and i m only 56. Doctor: Hi, The possibilities are a neuropathic pain as in sciatica. As a first line management, you can try analgesics like Aceclofenac or Tramadol for pain relief. If symptoms persist better to consult a neurologist and get evaluated. Hope I have answered your query. Let me know if I can assist you further. Regards, Dr. Shinas Hussain, General & Family Physician"
},
{
"id": 160148,
"tgt": "Can cancer be contagious?",
"src": "Patient: If a person with cancer has sores on their mouth, can eating out of their plate be dangerous? Doctor: No Cancer is not contagious, but few of them can be heridatory, that is if you have had any of the family members having the cancer, you might stand a chance of getting it due to their heriditary nature."
},
{
"id": 88186,
"tgt": "What causes pain in lower abdominal region?",
"src": "Patient: I had to get something that fell into a dumpster and I was leaning over the side with my stomach area balanced on the edge. I have had pain in my lower ab area for the past 3 days. I have been taking pain meds to ease it but wanted to know if this will go away. Doctor: Hi,From history it seems that due to fall giving heavy stretching of abdominal muscles there might be having spasm of the muscles giving this problem.Continue with analgesic medicine as and when required.Within a day or two it will be alright.Ok and take care."
},
{
"id": 120583,
"tgt": "What causes soreness of clavicle bones post pacemaker implant?",
"src": "Patient: I have my second pacemaker never had a problem with the first one and had the second one implated for over a year now. Over the past 2 weeks I have a lot of tenderness where the lead wires goes up under my clavical bone. It aches all the time but is especially sore if I try to touch the skin near it. Doctor: Hello,The pain in the clavicle bones is normal post peacemaker implant. It will relieve gradually. Meanwhile, I suggest using anti inflammatory medications such as Acetaminophen to relieve the pain. Hope I have answered your question. Let me know if I can assist you further. Regards, Dr. Dorina Gurabardhi, General & Family Physician"
},
{
"id": 165246,
"tgt": "Suggest treatment for autism and impulsive nature at the age of 11",
"src": "Patient: hello, my son has autism and is 11yrs old. he is very hyperactive and impulsive. he also gets very anxious and cries loudly,runs,does not listen at times for his demands like eatable,going out, escaping studies etc. i understand about behaviour modification, at that time he is only controlled by anger. he is been on resperidone since 6 years now. last year he had problem with bladder control. recently doctor advised me to give lonazep. he sleeps very less so i am scared to give fludac. please guide Doctor: hithanks for using HCM.as you mention the child is hyperactive and implusive have you ever tried Atomoxetine or Methylphenidate as these drug are indicated for same. these are prescription only drug so consult your psychiatrist for the same if they are worthy to your child or there are any contradiction for same. hope this will help you.thanks"
},
{
"id": 215753,
"tgt": "Suggest remedy for pain in ovary when having kidney stones",
"src": "Patient: sir i had the kidney stone so i was under treatment but suddenly i felt pain in my right ovary. ultrasound revealed enlargement in right ovary.two days before i went to see gynecolgist and she gave me some medicines. last day my periods started and today i felt something white thin net like substance with blood,hardly visible.what is that sir ,do i have a bigger problem. plizzzz answer Doctor: Hi, The kidney stones and ovarian enlargement is not related. You can continue the medicines prescribed by your gynecologist and you can take analgesics like Mefenemic acid for pain relief. If symptoms are more severe, you can consult your gynecologist again. Hope I have answered your query. Let me know if I can assist you further."
},
{
"id": 184903,
"tgt": "Will this extraction cause complications later on?",
"src": "Patient: Hi all and one, I just had a tooth extraction done Monday just gone. Was glad to find that I don't have a dry socket but several things are worrying me: Firstly it looks like 2 roots have been left in there, can this be right? Will it have complications later on? As the tooth was suppose to be taken out due to pus being developed on the gum and was infecting everything around it,..? Secondly, the tooth next to it it's gum split...one side is flopping around? Will it grow back together? Shall I be rebooking to go and see the dentist? Thank you in advance Doctor: hello thanks for consulting at hcm.. Tooth extraction includes removal of tooth in total,,so if the roots are left back.. I suggest you take a course of antibiotics since you had pus or abscessed tooth,,then review back with the dentist for extraction of those 2 roots left behind..do salt water gargle thrice daily, drink warm water. The neighboring gum has split, so i suggest you review back with your dentist for removal of the flap or if it could be saved by suturing the split gums..hope it helps..take care.."
},
{
"id": 118519,
"tgt": "Diagnosed with thalassemia, blood count normal, gets tired, taking Celexa, Lyrica. What to do?",
"src": "Patient: My red blood count total is fine, although it takes more red blood cells to reach the proper total. My white blood cells are regular size and numerous. Although I have about half the symptoms, I have been diagnosed with thalycemia. I am often extremely tired, and sometimes feel like fainting as I feel like all the blood has been drained from my body. I am taking Vitamin B, Vitamin D and a multivitamin as supplements, as well as Celexa, Lyrica, Xanax and a very old tri-cyclic, Imipramine. These medications are overseen by my psychiatrist and rheumatologist. Doctor: HIThank for asking to HCMIf I would be your Doctor then I would certainly advise you to cut down some medicine particularly Tricyclic anti-depressant, if you have history of thalassemia then it necessary to have less drug therapy, multidrug therapy or say polytherapy may some time causes blood dyscrasia and this could be dangerous for condition like Thalassemia take care of your self and have nice day."
},
{
"id": 103826,
"tgt": "Heaviness in breathing, asthma diagnosed, exacerbation dyspnea pleurisy. Steroids taken. PT for reversal cervical lordosis done. Advise?",
"src": "Patient: Im a 31 female with no previous breathing problems. I went to ER Diagnosis was Asthma w/ exacerbation dyspnea pleurisy. X-RAY clear blood work good. Gave steroid pack 6 day with treatments. Today last day & still have heaviness in breathing laying down & sitting up. Why is this happening to me know. Started when I started PT for reversal cervical lordosis. Doctor: THE ALLERGY CAN DEVELOP AT ANY ADE AT ANY TIME WITH ANY SUBSTANCESOME MEDICINES YOU TOOK OR LORDOSIS MAY HAVE CAUSED ALLERGIESYOU CAN TAKE ANTI ALLERGICS FOR 4 - 6 WKLIKE SINGULAR 10 MG TDSSYP ASTHALIN 1 TSF TDSSYP GELUSIL 1 TSF DSI THINK IF YOU DONT TAKE ANY PAINKILLER IT MAY MAKE YOU COMFIRTABLE IN 3 WK TIME"
},
{
"id": 52984,
"tgt": "Suggest treatment for increase in billirubin value after treated jaundice",
"src": "Patient: Sir Im suffering from jaundice for the past 4 mths ,The billurubin level varies between 2.5 and 2.1 for a long time ,Last month when i had done a test it is 1.4 and it is normal ,again it got increased to 2.5 last week.I had undergone treat ment for the past three mths ,All medicines which i had taken are related to gastric ulcer,I didnt have symptomps of jaundice ,Only discomfort i have is a uneasyness at the right side of the heart that too due to gastric problem but it occurs most of the time ,Kindly suggest what i have to do as i have allready taken lot of medicines Doctor: Hello, I hope you are well. It is important to know all the parameters of your liver chemistry, since you should consider the type of bilirubin elevation, because it can be either of direct or indirect predominance and the diagnostic protocol and handling of the two cases differs .In addition, isolated hyperbilirubinemia may occur in combination with elevation of your liver enzymes and / or alkaline phosphatase or GGT levels to consider a possible obstructive condition in the biliary tract or some hepatolytic factor. That is why I suggest you perform a complete liver chemistry and from there make the right decisions regarding more studies and / or image."
},
{
"id": 219238,
"tgt": "Suggest treatment for swollen and painful vagina during pregnancy",
"src": "Patient: I have hemroids since pregnancy, for a total of 5 years. Within the past year they bleed every time I have bowl movement. It is very painful and I dread going #2. However, I know I can take care of this with surgery. Within the past week I am having vaginal pain in the back of my vagina. It feels dry, swollen, and extremely sensitive (like s bad bruise.) It hurts to wipe, even when I pee. The pressure can feel very uncomfortable too! My Vagina even hurts to sit and walk. I know I need to see a doctor a.s.a.p. but I am looking forgone advice on what could be causing the vaginal pain and discomfort. Doctor: Hi.It could be your bowels just applying pressure on the uterine walls because of the increased size of the uterus, the pressure falling on the bowels can cause some amount of counter pressure on the uterus. This could be causing the discomfort, but it is still best to visit your doctor and go about an abdominal scan to confirm/rule out the cause.Best wishes."
},
{
"id": 85797,
"tgt": "What is the side effect of medicine saaz for spondylitis along with being on BP and diabetes medicines?",
"src": "Patient: i have been prescribed medicin saaz by safderjung phicytion for anko lysing spondoliytic problem having since last 25 years where as iam taking medicine fo bp and suger both what side effect may cause taking saaz along with my age is50 and weight is 87 kg please answer. Doctor: Hi, Saaz (Sulfasalazine) is used for treatment of ankylosing spondylitis. It can have drug interaction with Sulfonylureas: Increased sulfonylurea half-lives and hypoglycemia have occurred. If you are taking any anti-diabetic of Sulfonylurea group ( e.g. Glimepiride, Glipizide, Glyburide or Gliclazide), you run the risk of hypoglycemia. Hope I have answered your query. Let me know if I can assist you further. Regards, Dr. Tushar Kanti Biswas, Internal Medicine Specialist"
},
{
"id": 159334,
"tgt": "Suffering with stage 4 bile duct cancer, no improvement with chemo. What is the prognosis?",
"src": "Patient: Hi there my mom has been diagnosed with stage 4 bile duct cancer she has undergone chemo but it hasn t worked so chemo has been stopped she has no symptoms with her liver as yet she does not want to know how long she has left which I respect fully. I live in New Zealand and myself and my family are planning a trip back to the Uk to spend quality physical time with my parents my worry is when will be a good time for my moms health to do this at the moment we are all dealing with the prognosis setting a date is a sorry as we don t want it to be too late! Doctor: Hi, Bile duct cancer are very aggressive in nature. In advanced stage they very often fail to response to chemotherapy, response rate is 20%. As your mom has stage IV cancer it means it has been metastasized. Exact survival period can not be commented on. one year survival probability is not very good. Oral chemotherapy may be an option but if not responding it will be better to maintain a conservative management. Take care."
},
{
"id": 35796,
"tgt": "Suggest treatment for pancreatitis",
"src": "Patient: Hello My name is Rodney. I have been in the hospital for 6 weeks suffering from pancreatiuas. I had a acute case I believe the surgen accidentley hitmy pancreas causing it to inflam. Since I was in so long and couldn't eat or drink foods on a regular, my blood level went down. I am home now and its still low 11. I am taking iron pill once a day and Hema plex twice a day and B12.My BP seem to be high when I would a short distant, but it does come down after I sit for a while the bottom number is still high 138/90. Also, my pulse seem to run 115 on average. Do you think this is because my blood level is low causing my other vitals to be off Doctor: Thanks for your query at HCM!I am Infectious Disease Specialist! I went through your query!Hi Rodney!I realize you went through a tragic phase.Your hemoglobin will increase with time just take haematinics.As far as your BP and tachycardia is concerned you need proper evaluation from a cardiologist you might be suffering from essential hypertension.May have to take anti-hypertensives.Also get blood culture, USG abdomen and procalcitonin test done to see for infections if any.Happy to take more queries! You can also write a review for me. If you would like some more information, I will be happy to provide. You can take a follow-up query.Take care!Dr. Sheetal VermaInfectious Disease Specialist"
},
{
"id": 75925,
"tgt": "What causes chest pain and varying heart beats?",
"src": "Patient: I have been having extreme chest pains semi- often, and dull aching at other times. I had to wear a 24 hour heart monitor and was told that my heart rate jumps all over, it got as low as 36 at one point and jumped much higher than normal afterwards. What could be wrong? Doctor: Hi thanks for contacting HCM....Normally heart having regular systol and diastole phase and contract and relaxation of heart chamber regularly ....And so normal heart rhythm is there ....If disturbance occurs in heart rhythm it is called arrhythmia.....So heart beats irregular ....For perfect diagnosis of arrhythmia whether it is premature ventricular contraction , atrial arrhythmia or ventricular one etc....Holter monitoring done for 24 hour ...ECG and ECHo also useful....According to cause treatment given like for ex.beta blocker ...Your blood pressure also should be checked ...Consult cardiologist for this....Hope your concern solved...Take care"
},
{
"id": 168029,
"tgt": "What causes chronic runny nose in a child?",
"src": "Patient: Hi there, my 21 month old has had a runny nose (changing from clear to green) for going on 6 months now... he isn t sick, has no temperatures and is a happy little man. For the first 7 months of his life he was very congested in the nose and every night I tried everything to aliviate his discomfort (e.g. elevate his matress, use Karvol, saline drops) but that all stopped at around 7 months old. The runny nose is an annoyance more than anything and actually doesn t seem to bother him but I m just worried that he might have adenoid or sinus problems. Doctor: Hi...by what you quote it looks like a mild intermittent allergic rhinitis. If I were your pediatrician I would suggest giving a course of Levocetrizine for a period for 4 weeks daily as single dose in the night.This is a prescription drug and you would require one for it.Regards - Dr. Sumanth"
},
{
"id": 212242,
"tgt": "Going crazy, lie to family & friends. Lost my husband. How to change?",
"src": "Patient: I think I am going crazy. I lie to my family and friends . I am not sure why. My family was just here and said I dont let them into my thoughts. I have lost my husband to kidney failure last week. My family said I don't show emotion until I get \"upset\". I think they are right. I fear for my future. Even my 97 year old mother told them I am too busy doing what I want to come to visit with her. She is right. I am sorry but the truth does hurt. I want to change but dont know how to start. Doctor: Hello Welcome to health care magic I can understand your situation. I appreciate your effort to seek online support. As per your details, you are possibly having underlying anxiety/depression/grief reaction. For this consult a psychiatrist for detailed evaluation. Following treatment strategies would be helpful- 1) Antidepressants like SSRIs (escitalopram, sertarline, paroxetine etc) 2) Antianxiety agents like clonazepam, lorazepam, alprazolam etc 3) Psychotherapy/ Counselling sessions 4) Relaxation techniques like yoga, meditation, deep breathing etc Hope it helps Good luck Regards Dr. Gourav Monga Consultant Psychiatrist"
},
{
"id": 62959,
"tgt": "What does a knot on neck indicate?",
"src": "Patient: Hi, my mother recently had a herniated disc and she got the surgery to replace it about a year ago. The disc was located near her neck. We went to a ski resort a few days ago and she went down a path on a sled that really hurt her neck. Earlier today she asked me to look at a growth near her neck and it kind of feels like a knot. She told me that it s numb and that she can t feel it. Do you know what is happening? Doctor: hi.it is best if you bring your mother to a doctor for clinical evaluation. imaging studies (i.e. cervical x-ray, neck ct-scan or mri) may be requested to further investigate her condition. management will be directed accordingly.hope this helps.good day!!~dr.kaye"
},
{
"id": 5572,
"tgt": "Had 2 miscarriages. Trying to concieve. Prescribed medicines. Reason?",
"src": "Patient: Hi am 33 yrs old. I have a 4 years old child. last year i planned for other child but got a miscarriage after 8 weeks, Doctor told me that sack was there but heart beats were not there, after two month my miscarriage i again conceived. This time heart beats were there but yolk was absent and after 7 weeks heart beast also gone. I had all the test told my doctor to find the reason for the same but all test were clear . After taking a gap of around 9 months again i want to plan and i visited a gynecologist he prescribed me Placentrex IM (twice a week till 3 week), Biorox 150 for( 5 days),satrogyl 200 for (3 days,Folinex daily) and Neogest 200 for (10 days from 18th day of my monthly cycle ). Now i am confused that if any complications have not been deucted then why i have to take all these medicine. Please help if you have any answer. Thanking you Suman Doctor: Hello suman thank you for your query. Firstly, without knowing the cause of your previous miscarriages, please do not plan a pregnancy. Please consult an expert gynecologist for thorough preconceptional counselling. Commonest cause of miscarriage are genetic reasons. Ideally, karyotyping of products of conception should have been done. You and your partner should undergo genetic evaluation. Intra uterine infections should be ruled out, and treated if present. Uterine and cervical abnormalities should be screened out. Autoimmune causes of abortions should be evaluated by appropriate tests. Also, your hormonal profile to check for hormonal imbalance as a cause of abortions should be done. Take regular folic acid supplements. Only after detailed testing, once you have the cause in hand, you should plan your next pregnancy. take care."
},
{
"id": 158467,
"tgt": "Have multiple myeloma. Can Ayurvedic treatment be done along with chemotherapy?",
"src": "Patient: My Father, aged 73 years suffering from Multiple Myeloma , diagnosed 1 months back. Doctors say it is in 3rd and last stage. Doctor Suggest treatment through chemothearpy but there are lots of side effect. Is Ayurveda give hope along with conventional treatment is possible. Revert, if any supported reports(CT, PET-CT , MRI etc) are required. Doctor: Dear,Ayurvedic physicians do claim the treatment for the disease but till this day there is no definite evidence of its effect. many latest drugs have come in the market called targeted and antiangiogenic drugs which give some hope.Radiotherapy is needed only when there is a gross single leision in the bone or there are compression signs of the spinal cord.If you still feel that there is some logic in the Ayurvedic treatment there is no harm in continuing itwith chemotherapy.Truly,Dr. J. Ticku"
},
{
"id": 50428,
"tgt": "77 years. Lower back pain, cold, asleep during conversations. History of open heart surgery and 20% functioning kidneys. What tests should be done?",
"src": "Patient: Hi I am worried about my Dad who is 77 years old. He had open heart surgery 20 months ago and came through it great. Before he went into surgery his kidneys were only functioning at 20%. Over the past two months he has been complaining about lower back pain, being cold and has started to fall asleep during a conversation. My question is when he goes to see his family doctor what types of testing should he have done to see if his kidney has failed any worse than it was when he had his surgery. We live in a very small community and it is hard to get a referral to see a specialist. Doctor: hii appreciate the care and concern for your dad. when you see your family doctor next time to know about the kidney function, he will definitely ask you to do some renal function tests like serum creatinine, blood urea and serum electrolytes. these tests will reflect the kidney function. if they are abnormal you have to consult a nephrologist for further treatment as it needs dialysishope my reply has answered your doubtstake care"
},
{
"id": 183747,
"tgt": "What causes weak gum?",
"src": "Patient: hi. i am a smoker and 22yrs old. my gums bleed when i brush my teeth and sometimes one part will bleed alot. but it always stops. when i rub my finger on my gums and smell it. it is smelly. i went to dentist before and he said i have weak gums. is it because of smoking? Doctor: Thanks for using Health Care Magic.Read your query.Bleeding gums suggests inflammation of the gums which is usually caused by many factors including improper maintenance of the oral cavity,smoking,systemic conditions being a few .I would advice you to first stop smoking completely as it will affect the whole general health. Visit your local dentist and get your teeth cleaned and polished followed by correct post dental instructions.Use chlorhexidine mouth wash for a while.If the bleeding is still continuous as before or more visit a general physician and get it evaluated for any systemic conditions.Hope this was useful.Thanks and regards."
},
{
"id": 134100,
"tgt": "What precautions needs to be taken post surgery for meniscus?",
"src": "Patient: Good evening I am a police officer back in 2011 I had surgery on my meniscus, prior to having another MRI four weeks ago on the same knee, I have been sent to physical therapy three times. After reading the latest MRI, I am now back in physical therapy for 7 weeks. My doctor has informed me not to walk up stairs if possible and no running as well. As a police officer that is impossible. I m taking aleve per my doctors instructions but still have inflammation in the knee after walking, what would you suggest. Doctor: hi,as you mentioned you had an meniscus injury and also undergone the surgery for the same. Well, i dont think you need any limitation of the activity as surgery was performed to get you close to normal functioning. Also, undergoing a proper rehabilitation of the knee - just like the soccer players. If they can play soccer post their ligament injury why not you have a normal functioning of your joint.Focus on not only the injured knee, but also the whole body conditioning. Strengthening the quadriceps and hamstring are the prior thing which will be followed. But extend your protocol to 12 weeks as by that time the knee is ready to fight back.Focus on strengthening the core, pelvic floor, upper limb, lower limb, spine muscles. Undergo the balance retraining using the balance board. Also, get the torque in the muscles of both the thigh bone. Make the body lighter so it can take the jumps and runs.Usually what i see patients they all can perform 99% of their physical activity like - jumping, running, climbing stairs, squatting and sprinting.Any further questions you can surely ask meRegardsJay Indravadan Patel"
},
{
"id": 73905,
"tgt": "Suggest treatment for pleurisy",
"src": "Patient: i went to hospital with chest pain on saturday, they done an ECG and the results were normal. they then did some blood tests, which come back 0.42 for a blood clot, i was sent for a chest x-ray which showed a dark lining around my lung which the doctor said was pleurisy. i was given injections to take home to have in my stomache. i have a chest scan booked for next tuesday, at the moment i am not having any pain when i am breathing in like i did before just pain when i move my chest. can pleurisy go away on its on? and if it was that bad when i went to the hospital would they of kept me in? and what are the ways of getting rid of pleurisy? Doctor: Hello! Thanks for posting your query . The course and treatment for pleurisy depends on the cause of pleurisy. Pleurisy due to some causes like viral infection do resolve with just supportive medications and time. With only chest x ray it s sometimes difficult to differentiate pleurisy from pleural effusion (where fluid gets collected around your lungs) Hence, it is advisable to get an ultrasound of your chest done just to ensure there is no effusion. For the pain due to pleurisy ,short course of simple analgesics, chest physiotherapy and breathing exercises will give symptomatic relief . Just give it time until it resolves. Hope my reply is of help . Do drop in your valuable feedback."
},
{
"id": 37506,
"tgt": "Noticed discoloration & irritation of skin due to insect bite",
"src": "Patient: hi, I was bitten by an unknown insect almost two weeks ago and the inflammation of the bite is still there along with tissue bruising, leaving my skin around the bite discoloured. I have noticed over the past few days after the bite hole if you wish, has raised into a almost puss like spot, however around the skin is still discoloured, inflamed and sore burst and left my skin slightly torn. I have this continuous tingling sensation which gradually increases in pain, almost like a really bad case of pins and needles. the pain starts from the bite down to the sole of my feet. I have no idea what to do about it, and how to treat it. all I have dine thus far is take ibuprofen for the pain and apply an ice pack and on occasion Vaseline around the bite. however nothing seems to be working. Doctor: HiWelcome to Health care magic.I went through the history you have provided here. It seems from the history that you are having irritation of skin following a insect bite.These symptoms indicate secondary infection in the skin. I usually treat these conditions by the use of antibiotics and anti-inflammatory drugs. Application of local antiseptic ointments and hot compression provides rapid relief.Do consult your doctor for examination to confirm the diagnosis and prescription of drugs.Hope this is helpful to you.Thank you for writing to us."
},
{
"id": 51816,
"tgt": "will thyroid or dialysis cause blood vomiting?",
"src": "Patient: HI , My mom is under dialysis and she takes thyroid tablet . when get up in the morning she gets 1 spoon of blood . every day morning . Please tell me is it because of thyroid or dialysis . her TSH level is 400 . Doctor: Thanks for the query No thyroid tablets won't cause blood in sputum or vomit. Tsh is very high I hope u r taking appropriate medications for it Meet an internal medicine doctor first and then endocrinologist Have a healthy living"
},
{
"id": 158091,
"tgt": "75 years. On chemo and radiation therapy for lung cancer. On 100mcg fentanyl patches along with 30mg morphine every 2 hours. Normal?",
"src": "Patient: my father has been taking chemo and radiation therapy for lung cancer. he is 75 yrs old and has stage IIA cancer he is stopped taking treatment due to hoapitalization for confusion but my question is that he is currently being administered (3) 100mcg fentanyl patches at the same time and new ones are put on daily. is it unusual for someone to be given such a large dos of fentanyl along with 30mg of morphine every 2 hrs Doctor: Fentanyl is approximately 100-150 times more potent than oral morphine;Fentanyl patch 100 mcg/hr is equivalent to 360 mg of oral morphine dose in 24 hrs.Oral morphine is preferred when an opioid is required for severe chronic painbecause of its familiarity, availability and the ease of dose adjustment.Reserve fentanyl patches for use in opioid-tolerant patients with chronic pain and established opioid needs who cannot take oral morphine, for example in severe renal impairment .Fentanyl is also useful when oral morphine cannot be used due to vomiting or difficulty in swallowing."
},
{
"id": 135451,
"tgt": "What causes muscle pain in left hamstring with ripping feeling?",
"src": "Patient: I had the bariatric sleeve done n November. I am having success with it as I have lost 120lbs. I had experience for muscle pain n my stomach that felt like my muscles were ripping when I truned a certain way. Those have gone away but now my left hamstring is dong this when I stand or stretch too fast. I am wondering what vitamin or mineral deficiency may cause this. I drink plenty of water each day also. email: YYYY@YYYY Doctor: hello,I think you are having vitamin d 3 and vitamin b 12 deficiency. I would suggest you to get these blood test .If there is any deficiency then you need to take 60000 unit vitamin D per week basis and 1000 microgram b 12 daily. Also take high protein diet to take care of muscle building. I hope these drugs will be helpful for you. Let me know if there is any other followup questions.thanks"
},
{
"id": 193627,
"tgt": "What does the sperm count in my sperm analysis report indicate?",
"src": "Patient: Hi, My sperm Analysis report shows as follows: Voulme: 4.o ml, Liquefication time: Minutes, Sperm Count: 82.0 million, Motility - Grade 1: 10 percent, Grade II: 20 percent, Grade III: 5 percent, Non Motile: 65 percent, Morphology- Normal 95 percent, Tapering 4 percent, small 1 percent, Sperm vitality, live 44 percent, dead 56 percent, Others: Pus cells: 1-2/hpf. Can you let me know if this is normal. Thank you Doctor: Hi, You have 65% non motile sperm. It is not normal, at least 50% of sperm should be motile. Cause can be due to 1.\u00a0\u00a0\u00a0\u00a0\u00a0Infection due to sexually and non sexually transmitted infection 2.\u00a0\u00a0\u00a0\u00a0\u00a0Injury 3.\u00a0\u00a0\u00a0\u00a0\u00a0Inborn errors 4.\u00a0\u00a0\u00a0\u00a0\u00a0Un-descended testicles I hope I have answered your query. If you have further doubts, I would be happy to help you. Regards, Dr. S. R. Raveendran, Sexologist"
},
{
"id": 132602,
"tgt": "How to get rid the osteoarthritis?",
"src": "Patient: thought I was having heart or anxiety problems,have taken zanax for 15 years,keep having severe pain in lower throat and bones or tissue,can t tell which ,pain gets so bad.i have osteroarthritis and fibro,many bone issues.pain is getting worse every day.lymph nodes in neck and pain in other areas I can t explain.pain now in all bones in chest area coming and going?????? Doctor: Hi Hope this message finds you in good health.I have gone through your complaints and understand your concern.osteoarthritis is a natural ageing and causes pain in joints,have high calcium diet,calcium supplements,rest,analgesics,physio.Nothing to worry about.\u00a0\u00a0\u00a0\u00a0\u00a0I hope your question has been answered.If you have any follow-up queries,feel free to consult me anytime.Thanks,Take care,God bless."
},
{
"id": 149136,
"tgt": "Had hemorrhagic stroke while bleeding. No surgery as bleeding stopped, numbness, tingling in foot. Nerve test done. Solution?",
"src": "Patient: Had a hemmoragic stroke on my right side and at the time while still bleeding rushed to have it drained out, fortunately the bleeding had stopped by the time of arrival to the neurologist so their was no surgery. June 10 2011..through rehab learned to use my left side again however numbness is absolutely extreme especially in my foot as if it was in a vice while someone continually tightening it. My arm/hand is mostly extreme tingling ..it is by the grace of God that I can walk and drive and do just about everything I could before it happened however the extreme numbness doesn't stop. Had nerve test before and was told I had some damage ..if I didn't have any that damn tester sure felt like it did some damage. Please help with any info that will help me!! Been taking my kids to school , picking them up and everything else cause I'm a natural worker and can not stand to be just sitting doing nothing. Thalamus is the area where the bleeding occured Doctor: HIThank for asking to HCMSorry to hear that! look numbness after the brain stroke is common phenomenon and does not cause any harm to you only thing is it may disturb you, if you are too sensitive to it, once you became a use to it you wont be disturbed any more, but it takes time to be used to it, If I would be your Doctor then I would definitely advise you, not to pay any attention toward this feeling as it is not at all danger, have nice life bye"
},
{
"id": 22493,
"tgt": "Suggest treatment for shortness of breath and headaches",
"src": "Patient: My husband is 54 y/o. He has had symptoms of shortness of breath and headaches. We determined the source of his headaches were blood pressure related. He has since been put on Lisinipril 20 with a diuretic. He has problems staying hydrated due to his work. It is usual for him to have cramps at night in his legs but drinks alot of gator aid and water throughout the day. The last few nights his legs have cramped consistently resulting in no rest and he is using the bathroom constantly. He continues to have shortness of breath and at this time his bp was 102/73 and his heart rate is 119. He had a heart scan this morning which came back clear. He is re-hydrating today and the doctor has called in the same medication without a diuretic. He has shortness of breath and a headache. I have explained that these are symptoms of low blood pressure. Any thoughts about this. Doctor: Hi,Have you got him examined? If everything is fine from cardiology side then this could be related to lung or blood. Is his hemoglobin normal, have you got his chest x-ray done, does he have any wheezing, what are the factors which increases or decreases shortness of breath, does he smoke, have cough with sputum production, is is having to exposure to particulate matter at work, these are the multiple questions I want to ask. We need to rule out , his lungs as a cause of shortness of breath. So get his x ray and if needed CT scan done. Get back with the answers of the above questions and with exact reports if possible. Till then follow healthy lifestyle like exercises, yoga and avoid smoking if any. He should protect himself at work if there is any exposure.Hope I have answered your query. Let me know if I can assist you further.Regards, Dr. Sagar Makode"
},
{
"id": 53829,
"tgt": "What does this altered liver profile indicate?",
"src": "Patient: I am 54 year old female and just had my annual physical recently. and my AST level is 65, but my ALT is fine, it s 32. This is my concern, about 22 years ago I had rocky mountain spotted fever and as a result of this illness developed hepititus, which was classified as c. With that said, should I be concerned? Should I see a liver specialist or have more test done? Doctor: I suggest you consult your liver specialist.Having rocky mountain spotted fever, and having hepatitis that is classified as C, are two separate entities. The elevation of AST to values greater than ALT, makes me question, if alcohol is being consumed in more than recommended amounts?kindly get in touch with your liver specialist."
},
{
"id": 182366,
"tgt": "What causes pink band around teeth causing pain and sensitivity?",
"src": "Patient: I am on coumadin, am pacemaker dependent and recently had a lead extractation. Before the extraction was done, they did a t.e.e. as I have a history of blood clots. Unfortunantly, a bottom front tooth was bumped pretty hard, it hurt and was a little loose, it's now getting worse, instead of healing. It has a pink band that goes around it and is very painful and sensitive. It looks like blood and is more pinkish each day. My INR is high, but is it possible for your teeth to have blood in them? Doctor: heloo...Do you have mouth pain confined to your gums, with bleeding or swollen gums? Can you see a 1mm. wide, deeper pink line of gum along its' edge up against your teeth?There are many cause of pink band around teeth.....Bleeding is likely to be obvious if you use floss between your teeth.Flossing is useful to get out bits of meat caught between teeth, but may not adequately clean away plaque.You will be able to cure mouth pain from gingivitis. Gum disease responds well to careful cleaning. This is also true if the apparent cause is drugs for epilepsy.Severe lack of vitamin C causes scurvy, with bleeding gums from gingivitis. One's food should include raw items, as vitamin C is destroyed by cooking.Peridontal disease isn'tjust a mouth pain problem. It's a strong risk factor for later developement of cardiovascular disease. It increases silent systemic inflammation believed to underlie many chronic degenerative diseases.If someone has valvular heart disease, it increases the risk of bacterial endocarditis. This is infection on the heart valves, a very nasty thing to get."
},
{
"id": 68693,
"tgt": "Suggest remedy for small lump at the back of upper neck",
"src": "Patient: hello Doc I have a very small ball on the back of my upper neck, on the right side I'm pretty sure its not a cyst. One day I turned my head to the left and felt a horrible pain on my right side, and now I have a small ball that you can only feel if i stretch my neck....what can i do, I've had it for over a year and it bothers me. Doctor: welcome to Health care magic.1.Lump with pain most possible cause could be infective aetiology.2.Othere possible causes of lump in the neck regions could be vascular malformations, lipomas and lymphomas.3.If you are my patient i would have examined the lump and ask for a ultrasound lump - to find out the nature of lump, its origin and its extensions.4.Depending on size the treatment will be a course of antibiotics to small incision and drainage.Hope it helps you. Wish you a good health.Anything to ask ? do not hesitate. Thank you."
},
{
"id": 26856,
"tgt": "Suggest remedy for side effects of Diltiazem medicine for hypertension",
"src": "Patient: yes, I just started taking the drug diltiazem hc 60 mg for hypertension every 12 hrs. the side effect is herendous...put me like in coma state, and could not get out of bed, no matter how I tried. I stop taking it after three days, and only took one in a 24 hour period. I have been taking my blood pressure for the past three days an stable at 117/62 highest was 127/67. I feel a short of breath at times like I do not take in enough air to fill my lungs w/air...I have to make an effort to take in air to fill my lungs. I do not feel I can talk w/out feeling winded. I had phneumonia a couple of years ago which put me in the hospital for 2 days. I caught it as a virus, I never smoked, nor had any kind of lung condition. perhaps nasal allergy but nothing as bad as that. I am a 64yr old femaie. I never feel this shortness of breath when I am exercising at the gym a few days per wk for 1 hour. While I was on the meds I did not have that problem, but then I was asleep most of the time. Do you think I should see a cardiologist? or what could be causing this condition? My email YYYY@YYYY Doctor: hi, diltiazem does not cause side effects that you have mentioned, and your bp readings are normal which do not require treatment, you should see a cardiologist for shortness of breath and a neurologist for sleepy feeling."
},
{
"id": 34294,
"tgt": "Can i get infected by getting poked?",
"src": "Patient: i was walking on the road n some guy poked me with something on my bicep. I m not sure what it was but any chance I can get infected. I was wearing full sleeve shirt and the contact was just for a second. I see no blood coming out ... i am little scared.. can u please suggest Doctor: Hello dear,Thank you for your contact to health care magic.I read and understand your concern. I am Dr Arun Tank answering your concern.No, there is no chance of any infections.Usually infections occurs only when you have bleeding with the Pocking.You will be further protected as you have wear full sleeve T shirt. If the T shirt is short sleeved you will be in doubt of having infections.But please take good care of the Pocking area. You can do good care by frequent dressing and Cleaning. I will be happy to answer your further concern on bit.ly/DrArun.Thank you,Dr Arun TankInfectious diseases specialist,HCM."
},
{
"id": 44123,
"tgt": "Suffering from thyroid, low endometrial thickness, failed IUI. Treatment for infertility?",
"src": "Patient: DEAR SIR/ MADAM I WANT KNOW THE CHANCES OF MY PREGNENCY, I HAVE A THYROID PROBLEM AND MY ENDOMETRIAL THICHNESS IS LESS, I HAVE TRIED FOR 5 TIMES OF IUI BUT THERE IS NO POSSITIVE RESULT. CAN I HAVE TAKE CHANCE FOR NEXT IUI. FOR IVF WE HAVE NO SUFFICIANT MONEY ALSO , MY HUSBAND HAVE NO SPERM COUNT. PLEASE GIVE ME THE CORRECT ADVISE. Doctor: Hi SuryaprabhaWelcome to HealthcareMagicI would like to know your endometrial thickness on the day of ovulation. For successful pregnancy it should be atleast 7 mm. If endo thickness is persistently less during IUI cycles then you should undergo diagnostic hysteroscopy ( where uterine cavity is directly visualized using a camera ) and also endometrial biopsy need to be taken to rule out tuberculosis. If everything comes out to be normal then there are certain medicines which can be used to improve endo thickness with mixed success like estradiol valerate, arginine, vit E , pentoxyphylline , sildenafil. You can discuss options with your treating doctor. You can proceed for one or two more cycle of IUI with stimulation by gonadotropin injections instead of clomiphene . If these treatment also fail then IVF and surrogacy are last options. For husband's problem IUI with donor sperms can be done. All the best ."
},
{
"id": 119420,
"tgt": "Is knee replacement surgery recommended in case of cold and cough?",
"src": "Patient: I am due to have a total knee replacement tomorrow. I have a cold at present which is resolving slowly although I have started to cough ove the last 3 hours. I am 68 and otherwise in good health. BP 145/85. Cholesterol 5.1. Should I go ahead and have the surgery? Doctor: Hi, Knee replacements are done under spinal or epidural anaesthesia,cough and cold doesn't pose a problem for these type of anaesthesia but as a precaution we avoid doing replacement while the patient is having any sort of respiratory infection, so as to prevent the situation in which we need to intubate the patient with respiratory issues in case of complications arises. Secondly, there are high chances of lodging of infective agent at knee implant so it is always better to postpone surgery till the time infection subsides. Take care. Hope I have answered your question. Let me know if I can assist you further. Regards, Dr. Rohan Shanker Tiwari, Orthopedic Surgeon"
},
{
"id": 160586,
"tgt": "How to bring underdeveloped scrotum to normal size in an infant?",
"src": "Patient: My son six-month old and both undescended testicles. his scrotum is very small.underdeveloped. The pediatrician said, if the scrotum is too small,the surgery results will not be good. So suggested us to stretch the baby s scrotum 3 hours a day to make the skin bag large. We want to ask, are there any else methods that can make the scrotum be normal size??? We are very worried. Thanks a lot!!! Doctor: Hi, If the testes are really undescended, this needs a surgery to bring back the testes into scrotal sac. Otherwise this will cause trouble in future including infertility and abdominal pain. In my opinion, stretching of an empty scrotum will not be helpful. Kindly take him to a pediatric surgeon. If both testes are undescended with very small scrotum, you may be asked for an ultrasound scan and few blood tests including karyotyping to ascertain the sex. Hope I have answered your query. Let me know if I can assist you further. Take care Regards, Dr. Muhammed Aslam TK"
},
{
"id": 131090,
"tgt": "How to treat nipple tenderness and chest pain?",
"src": "Patient: A few days ago my husband complained of having nipple pain and tenderness in his chest and rib area. At that time there was what appeared to be a slight discoloration over his ribs. Tonight he says its bruising and darker and the his chest is burning and feels worse. He had not had any known injury to this area, what could cause these symptoms? Thank you Doctor: These are symptoms of infection (bacterial or fungal ) you are describing specifically candida i recommeng he see a dermatologist Good Luck"
},
{
"id": 99226,
"tgt": "What causes itching all over the body?",
"src": "Patient: 46 days ago i quit smoking. Which had become a habit of 10 years. I used the Nicoderm patch for step 1 and 2 and did not feel the need to use step 3. 2 weeks ago i went to the gym for the first time and when i finished my work out i got home experiencing an intense and infuriating itch all over my body, no signs of rashes other then where i was scratching, the itiching has persisted since and have been using benadryl with little to no relief. any idea what may be causeing this or what i may use to find some relief? Doctor: Hi,Welcome to health care magic,Itching all over body skin could be due to eczema,allergic hives,atopic dermatitis, tinea corporis etc.The important thing is to diagnose first rather than treatment.You can go to skin specialist for detailed history and inspection of itchy skin to have a clear diagnosis.You might require topical corticosteroid like beclomethasone, clobetasol or mometasome with antifungal cream like miconazole or itraconazole with antihistamines orally to reduce itching.Some time oral antifungal once dosage with prednisolone are also effective.So,you must take him to the doctor.Regards,"
},
{
"id": 54414,
"tgt": "What is the remedy for vomiting and cough due to liver problem?",
"src": "Patient: hi sir, myself is shubhendu banerjee my father hasbeen sufferinf from liver problem for 7 days. with vomiting tendency whenever he wants to take some food. two day onwards he is not even able to take some solid food. he has also a deep cough inside. what to do sir? i am in hyderabad and he resides in calcutta. please reffer some good gastroenterologist in kolkata. Doctor: Hi I have gone through your query....In liver hepatitis there can be anorexia , vomitting , fever , muscleache etc ...These are some constitutional symptoms that can occur in hepatitis....Here you can have sub acute or early stage hepatitis....Following investigation done....1.Serum bilirubin for jaundice 2.USG for hepatomegaly3.liver enzymes sgpt and sgot raised in hepatitis 4.Urine urobilinogen and bile pigment for distinguish obstructive and hepatocellular jaundice....5.if needed viral marker....Meanwhile for jaundice low fat diet.Fruits more...There is anorexia in hepatitis but try to take semisolid food and apple juice or carrot juice .....Gradually solid diet taken...Avoid alcohol.....One tsp licorice powder with honey can be taken....Consult gastroentetologist in your city with keeping my advise in mind....Take care....Dr.Parth"
},
{
"id": 3099,
"tgt": "Are there pregnancy chances with follicular size of 10 mm?",
"src": "Patient: Hi, I had my follicular reading done ,on 19th day my follicule size is 10mm . It s was constant from 10th day to 19th day and didn t increased a bit, my doc has prescribed me with metaformin 500mg (bd) and duphaston10 mg (bd). Can I get pregnant this month ? I have pcos. Doctor: Hi, Thanks for your question. I do understand your pain and discomfort. I have seen many cases with similar complaints.As per your history, Please find my advice below-:1)The size is very small so it is impossible for fertilisation average size is (18-35MM)2)Do proper treatment for PCOS as per AACE guidelines only. I hope I was able to address your query. If you have any further questions, please do not hesitate to write to me. Wishing you all the best. Thanks,"
},
{
"id": 106038,
"tgt": "Asthma Specialist in Navi Mumbai",
"src": "Patient: Hi, I am suffering from allergic asthama sience last one month. I had undergone repetative cources of antibiotics/ asthaline/ prednosoline /derifyline etc. Also seince last one week taking regular nebulisation. But getting temporary effect. I am staying in New Panvel. Can anybody suggest me the asthma specialist (a senior doctor ) in Navi Mumbai Area please Doctor: since u r already on allopathic drugs, y don't u try naturopathy which has no side effects. u can contact dr. kishore punjabi in chembur (e). he has been practising for over 19 years. dr. punjabi's contact details are as follows: phone = 9821660400/ add: 15 charisma centre 19th road near general education school chembur (e) mumbai 71. hope u feel well soon. rgds."
},
{
"id": 76084,
"tgt": "What causes breathlessness after a bee sting?",
"src": "Patient: My daughter was stung by a bee a week ago. She had a momentary lack of breathing and swallowing. The stinger was pulled out by another student. Then she said she felt okay. Slight swelling and redness that went right away. Now after one week later, she is still complaining of shortness of breath. Is this from the bee sting? Doctor: Bee sting venom causes trouble breathing, chest tightness, itching, swelling etc so may be because of this may be she has shortness of breath it will settle, still persists means you should consult near physician"
},
{
"id": 206471,
"tgt": "Suggest treatment for mood swings and depression",
"src": "Patient: Hello sir, my elder sister gets depressed suddenly and starts creating scene. She says she has become alone and has no true friend. I am her brother and she says that I have separated her from our parents. She is persuing her Masters degree presently. We have tried to console her every time she does like this. For some days she becomes normal but again after a few days, she starts getting upset. She even cries in loneliness in her room. It has become very difficult to control her. We can hardly satisfy what she wants. She always blames me of ruining her life which is not at all true. This is not a new problem. I have seen her doing all these from my very childhood. Any help would be appreciated Doctor: Dear brother,The phenomenon of displaying these sorts of mood swings and blaming you for the loneliness and sufferings are the classic symptoms of \"SIBLING RIVALRY\" which is emanating from her inner mind.As a matter of fact, the elder offspring gets a sort of insecurity feeling, when the second one is taken birth since all attention is diverted towards the new born which is beyond the comprehension of the first born. In this context, the child(your sister) construed that she was being avoided by her parents on your arrival, which hurts her in her inner mind.The inner mind of the first child is not so developed at that stage and is not expected to reason or rationalize the core fact.This could have been avoided, had her been given enough attention, care and a reassurance of the unconditional love of her parents in the midst of all your birth and growth. Unknowingly, she displaces her anger towards you which was originally towards the innocent parents.The elder sister, all through her growth compares the love, care, attention, things, events etc which you enjoy that of her and confirms her conviction in her innerworld. These are all the unconscious process started from her childhood since your birth. The lingering thoughts emanated from the harboured emotions are the causes mood swings. Whatever happened has happened. Need not worry. Everything will become alright. Take her to a Psychologist for intervention and for the mood swings, a Psychiatrist can effectively prescribe some medication for a short duration. The psychological counselling and the unconditional family support will invariably help her mind to come in terms with the reality and she can lead a peaceful life.In case you need any further clarification, please do check if you can contact me directly. I am from Erode, Tamil Nadu.Best of luckGreat days ahead.Dr Rajesh Thottingal KalamRehabilitaion Psychologist from India"
},
{
"id": 67996,
"tgt": "What causes a lump between the pelvic bone and upper thigh?",
"src": "Patient: I feel a lump under my skin between my pelvic area and upper thigh. I m scared it might be something serious. I asked my big sister about it and she said it might either be ingrown hair or a cyst. Should I need any antibiotics or surgery, or is it nothing to worry about? Please help me. Doctor: Hi, dear. I have gone through your question. I can understand your concern. You may have some cyst or some soft tissue tumor like lipoma or neurofibroma or others. You may have infective pathology. You should go for fine needle aspiration cytology of your lump. It will give you exact diagnosis. Then you should take treatment accordingly. Hope I have answered your question, if you have doubt then I will be happy to answer. Thanks for using health care magic. Wish you a very good health."
},
{
"id": 193981,
"tgt": "How to impregnate my wife with zero sperm count?",
"src": "Patient: i am 34 years old after one and half year of merrige we have no babay three before i have no sample for semen analysis after ayurvedic treat ment i got the sample for the test but there is nil sperm count in my sample my wife is normal there is scope for become a father please advise me for further treatment Doctor: Hi, Testicular sperm aspiration (TESA) can be done.To find any active sperm available. Hope I have answered your query. Let me know if I can assist you further. Regards, Dr. S.R.Raveendran, Sexologist"
},
{
"id": 161371,
"tgt": "Suggest treatment for problem with speaking and loss of memory due to encephalitis",
"src": "Patient: ok, thanks for that My daughter (of 5 years and 4 months)suffered from high degree fever on November 11, 2010 which induced fits and diagnosed an attack of Encephalitis as disclosed the EEG test conducted at a reputed Hospital. She had also undergone MRI test and an other test meant to assess her auditory system. Presently, She has problems of speaking, recognizing, loss of memory, but she walks ceaselessy while laughing or making noise which has become a source of headache for us but can\u2019t utter a sensible word. She touches everything with her right hand and try to put it into her mouth. Prior to this, she has the ailment of fits since the age of 08 months. She was taking Lumark tab 500mg 1+1 and Epival Syp 1+1 Spoon daily. Any kindhearted physician, who can treat this ailment, please help us. I can send her MRI, EEG and Auditory Test Reports at given e-mail address. Doctor: Hello, By what you say I feel that your kid is suffering from post-encephalitic sequelae being seizure disorder and attention deficit hyperactivity disorder. She will also require workup for autistic spectrum disorder with a clinical psychologist. She will require oral medications to come down along with occupational therapy and cognitive behavioral therapy. May I suggest you discuss these possibilities with her pediatrician and get her appropriately evaluated and managed, please. Divulging personal information and emails is prohibited on this website. Hope I have answered your query. Let me know if I can assist you further. Take care Regards, Dr Sumanth Amperayani, Pediatrician, Pulmonology"
},
{
"id": 192034,
"tgt": "Is marijuana an alternative therapy for temporal lobe epilepsy and diabetes?",
"src": "Patient: hi, My girlfriend suffers from temporal lobe epilepsy and diabetes. She wants to try marijuana, but I m doing research on the effects of it on her conditions. a lot of reports I have found suggest that it has anti convulsant properties and has helped a lot of sufferers to control the symptoms of tle. is this true? and what are the risks involved? Doctor: dear friend I can understand your concernin my opinion untill now there are no approved indications of marijuna to be used in epilepsy and diabetesI suggest you to follow your treating doc adviseregards"
},
{
"id": 211997,
"tgt": "Facial swelling, bruising, ringing in ears. Had stressful family situation. What is going on?",
"src": "Patient: I was in on both sides of my face early Sunday morning in the eye and cheek area after finding my daughters acting strangely / aggresively at around 2:00 in the am. We did call the police one is in jail and the other is no longer in the house they're both 21. My ears started ringing earlier today and I have some facial swelling and bruising. Should I be concerned? Doctor: hi,thanx for writing.i can understand your emotions.firstlt it could be just stress and anxiety which is showing up.secondly it might be some cardiac problem.u need to consult a doctor .might be hypertension tooo.byeeee"
},
{
"id": 178423,
"tgt": "Suggest treatment for hyperthyroidism and motion sickness in kids",
"src": "Patient: Hello sir! My four and half year daughter have thyroid by birth. She is currently on thyroxin 50. Inspite of taking the medicine regularly her TSH is still high. Around a week back when I got it checked it was 21. Sir she also has motion sickness and has issues with vomitting. She is always coughing and in the process vomitts whatever she eats. With slight change in weather she always catches cold. Sir please advice me on the further course of action and also let me know which hospital I should take her. Doctor: Dear Sir/ Madam,Thank you for posting your query at healthcaremagic.comI understand the concern of your daughter from birth. It appears that the dose thyroxin is not sufficient. I advice you to get further blood test and nuclear scanning of the thyroid done and consult pediatric endocrinologist for further course of action.Regarding motion sickness she can take domeperidon syrup (SOS). Modify food habits, for eg. multiple small meal in a day.IF you have any further query please revert back to me, I wil be happy to assist you.With best wishes.Dr. Vishwanath Patil"
},
{
"id": 142888,
"tgt": "How can fits caused by swelling in brain be treated?",
"src": "Patient: hi! my sister is 20 years old.she was totally healthy but suddenly she started having fits from last week after MRI it is detected that it is due to swelling in brain because of cabbage worm.we are all worried. if there is any permanent solution?plz help Doctor: Hi, Welcome to HealthCareMagic.com I am Dr.J.Mariano Anto Bruno Mascarenhas. I have gone through your query with diligence and would like you to know that I am here to help you.Q : swelling in brain because of cabbage worm.we are all worried. if there is any permanent solution?plz helpA : There is a permanent solution. We need to first treat the edema and fits with anti edema measures and anti epilepticsThen we need to give drugs to kill the wormPlease consult a Neurosurgeon and take proper treatment. Hope you found the answer helpful.If you need any clarification / have doubts / have additional questions / have follow up questions, then please do not hesitate in asking again. I will be happy to answer your questions. In the future, for continuity of care, I encourage you to contact me directly in HealthCareMagic at http://bit.ly/askdrbruno Best Wishes for Speedy Recovery Let me know if I can assist you further.Take care."
},
{
"id": 130333,
"tgt": "What is the treatment for foot deformity?",
"src": "Patient: Hi, so I have this weirdly shaped bump on the outter lateral side of my right foot. It's not an obvious big bump. But it's slightly obvious when I palpate it or when I turn my foot to the side. I acquired this last year while I was on vacation. I rode a boat and hopped down a bit fast from a high surface and ended up landing hard on the side of my foot on the wooden surface of the boat. It hurt, but because we were on vacation and it wasn't bleeding nor swelling, I thought nothing of it.After the vacation, i noticed the slight deformity or bump days later. It only hurts when I walk for a long time in slippers. I know it has something to do with the bump because I never used to get foot pains on the side of my foot until the incident.What should I do? Is this bump permanent? Is my foot bone deformed? Do I need surgery for this? Can massaging the side of my foot help or worsen the situation?:O? Thank you sincerely! Doctor: no your bone has no deformity,,, so be relaxed , but yes the bump may remain for a longer time , go for a foot x-ray to confirm whether there's a old fracture , first palpate whether the bump is soft or it appears bony n hard ,if it's soft and tender it represents ligament tearif it's bony and hard it may be a bone damage, check it.if it's soft it will heal with time and immobilization, if it's hard and bony you may need surgery, if ligaments do not heal within 7 months and pain doesn't subsides you may need surgery for that too."
},
{
"id": 10073,
"tgt": "Suggest remedy for dizziness with hair loss",
"src": "Patient: Hello I am 20 years old female seeking your advice concerning the complications I ve been having lately. I am constantly lightheaded and I ve been feeling dizzy. I have an extreme hair loss and my menstrual cycle seems to be irregular. I have lost concentration and I seem to get tired easily. I ve been put under a lot of pressure lately knowing that I had exams so I was worried! Please I need an expert advice. And thank you Doctor: Hello, Dizziness is due to hypoglycemia or hypotension. Kindly check blood sugar level and blood pressure. For hair loss, you can apply Minoxidil lotion. Treatment for dizziness is mainly depended upon the underlying condition. Hope I have answered your query. Let me know if I can assist you further. Regards, Dr. Shyam B. Kale, General & Family Physician"
},
{
"id": 52100,
"tgt": "My mom has swollen kidney and stones in it",
"src": "Patient: my mom is suffering with the kidney problem.. having swollen kidney and stones in it please give me suggestion in finding a good doctor please Doctor: your mom is suffering from hydro-Nephrosis a condition usually caused by partial obstruction of urinary tract by calculus [stone] or any other pathology when this calculus is removed this hydro-Nephrosis is usually reversed there is very good treatment of Renal and urinari calculies [stones of kidney ,ureters and Bladder ] these drugs are called Ashmari bhedya or Lithiolytic in Ayurveda you can go for Ayurvedic consultant or write at drnaithani@gmail.com"
},
{
"id": 173619,
"tgt": "What is the reddish patch a the back of the neck of an infant child?",
"src": "Patient: HI I Am african american and i have a 10 month old baby. a few weeks after her birth, she developed a reddish patch a the back ofher neck. she later developed these rashes on her cheeks. they seem to appear out of nowhere. My doctor gave me a steroid cream but it hasn't helped. he did not tell me what the condition was either. i would be grateful for your assistance Doctor: Hello dear,Welcome to HCM. Your baby has allergy-atopic dermatitis. Bathing tips:-Bathe your child in warm,better boiled water \u2014 not hot \u2014 water.-Limit your child\u2019s time in the bath to 5 or 10 minutes.-Use cleanser only when needed and make sure the cleanser is mild and fragrance-free. Do not use bubble bath. -After bathing, gently pat your child\u2019s skin partially dry.-If your child has medicine that you apply to the skin, apply medicine when your child\u2019s skin is almost dry and use the medicine as directed. -Apply moisturizer on the skin: better with fragrance-free products, so consider petroleum jelly ;-Using a laundry detergent made for sensitive skin may be beneficial. Remedies:1. Fenistil 8 drops thrice a day for 10-14 days, apply fenistil gel to affected area.2. Probiotics - Darolac 1 capsule once for 1 month.3. Enzymes syrup-for 1 month Recovery soon"
},
{
"id": 123919,
"tgt": "What cause pain in face under the eye starting from cheek bone?",
"src": "Patient: i have a sharp striking pain in my face just under my left eye , but starts on my check bone near my nose and runs down just a small bit. but the pain is sharp and last just a second for each strike. comes and goes, been doing this all afternoon and is still doing it now at 1:30 am. Doctor: Hello, Your symptoms might be the early onset of TRIGEMINAL NEURALGIA Disease. Trigeminal neuralgia is a painful condition that affects the trigeminal nerve, which carries sensation from your face to your brain. You may initially experience short, mild attacks. But trigeminal neuralgia can progress and cause longer, more frequent bouts of searing pain. It's advisable to go and visit a Neurologist who might ask you to go for MRI Brain for a confirmed diagnosis and treatment. Hope I have answered your query. Let me know if I can assist you further. Take care Regards, Dr Saurabh Singh Rajan, Sports Medicine Specialist"
},
{
"id": 164525,
"tgt": "How long should baby take Alimentum for reaction towards milk?",
"src": "Patient: Hi, my 5 months old baby is on Alimentum for the last 2 weeks after reaction to milk ingredient of rice cereal introduced 2 wks ago, she still has very loose green stool 2-3 X a day, but no mucous , is it ok? or we should see pediatrician again? and how long should she be on Alimentum before switching back to Good start she was on before? Thanks. Doctor: normally, we recommend to start give carbohydrates and proteins from 6 months of age, but in few children who have difficulty with lactose intolerance can start on normal carbohydrates diet by 5 months..better to add rice dhal, ghee , fruits bananas apples, leafy vegetables, potatoes and carrots boiled and smashed along with one egg daily. start giving these,, which will be enough for the baby growth..start using zinc syrup for 14 days 20mg."
},
{
"id": 73153,
"tgt": "What could croup like coughing suggest?",
"src": "Patient: Hi, may I answer your health queries right now ? Please type your query here.my 4 year old daughter has to undergo a procedure to check her trachea as she has had croup like cough three time this year. I am nervous....is this a serious condition Doctor: Thanks for your question on Healthcare Magic.I can understand your concern. Croup like cough is barking type of very loud, irritable cough. It is mostly seen with Tracheal pathologies like Tracheomalacia, Tracheal obstructive lesions like mass or cyst or nodule. So better to get done bronchoscopy for evaluation of Tracheal diseases. If this is normal then no need to worry much. But if any of the above mentioned disease is found then surgical correction is needed. Hope I have solved your query. I will be happy to help you further. Wishing good health to your daughter. Thanks."
},
{
"id": 180913,
"tgt": "How can gum bleeding be treated?",
"src": "Patient: I am suffering from gumbleeding since 3 months after using ecosprin tablet.i have consulted dental surgeon and he has cleaned teeth after 3 days use of antibiotic.and prescribed beta dine gorgle and c vitamin and Novotone tooth paste.still the gum bleeding is not stopped.is it due to ecosprin or any other reasons.kindly tell me the remedies. Doctor: Hi..Thanks for the query.Bleeding from gums is commonly due to deep gum infection causing inflammation and bleeding..Any inflamed tissues contains abundance of premature blood vessels and even slight stimulation can cause bleeding..In case if you take medication that can thin your blood like Aspirin that has an anti-platelet activity then it can cause increased tendency of bleeding..So my suggestion is to consult your Physician and get evaluated and if needed your dose of Aspirin should be reduced..Also if there is gum infection that is the cause of bleeding then another course of antibiotics and gum paint massage can help..Along with it do warm saline gargles and antiseptic mouthwash gargles.Do cool compresses over the gums..Use an ultrasoft toothbrush to brush your teeth..Hope this helps..Regards."
},
{
"id": 73825,
"tgt": "What is the treatment for chest tightness?",
"src": "Patient: I have been suffering with a tight chest that feels as if I have a heavy weight on it. This started last year with me waking up occassionally with a shortage of breath and an indegestion type pain. The tightness has become more regular up until a few weeks ago when I did some physical work in the garden. I suffered heavier than normal pain which eased after a rest, pain killers and a hot bath. The tightness is now with me nearly all of the time and I now get a twinge across my chest a few times a day with pain or numbness in my left palm. Over the past month my lower eyelids have begun twitching, starting with the right and now affecting both eyes and is very irritating. I probably don't hacve the best diet in the world and do have quite a stressful job and 2 small children. Is this something that I should worry about or do you think that it will clear up in its own time? Doctor: Hello dearWarm welcome to Healthcaremagic.comI have evaluated your query thoroughly .* Suggestions for better recovery- Maintain hydration with plenty of liquids .- Avoid smoking or alcohol if using .- Regular walking in fresh air , deep breathing exercises , YOGA .- Avoid stress , anxiety .- Regular sound sleep of 8 hours .- Consult later doctor if required .Hope this will help you for sure .Regards ."
},
{
"id": 21057,
"tgt": "What is the normal blood pressure at the age of 54?",
"src": "Patient: I go to a specialist every 4months to get meds for 3 bulged discs in my neck (vicoprofin & chlorzoxazone) i've been able to get by without a operation for 10 years during the last year & a half my blood pressure has been in the 150-160/over88-95 & the Dr. keeps telling me to go see a G.P. & get my blood pressure checked. I'm 54 & about 230lbs. what should a normal blood pressure be? Doctor: for your age of 54 yrs, your normal BP should be hence it is advicable to consult a GP"
},
{
"id": 207924,
"tgt": "Suggest treatment for borderline personality disorder",
"src": "Patient: Hi, My family member appears to have Borderline Personality Disorder since last 4-6 months. The has become very agressive, always ranting negative thoughts and does not trust anyone. There is no way to stop even by accepting everthing she says or wants. I wanted to know if DCT will help and how long this will continue, when and how she can recover fully to take care of her child. Doctor: HIThanks for using healthcare magicIt is difficult to get result in borderline personality disorder. In such cases, DBT is helpful, but getting 100% improvement is a tough job. Duration of therapy would depend upon the severity of problem. Better to consult a psychologist. In case, you need further help, you can ask.Thanks"
},
{
"id": 146950,
"tgt": "Dizziness, difficulty in learning, memory problems. Help",
"src": "Patient: hi first i want to introduce my self,i m ahmed i m 17 years old from tunisia and i will be so greatful if i get help as soon as possible to this huge problem the thing is that i spend a lot of time alone which causes me other problems such as difficulty in communication, lack of friends ( in mater fact i have none) and sometimes i have some suicider idea (nothing to worry about right now but it might be a huge problem after few years ) and what anxious me most is that feeling of dizziness moreover i find some difficulty in learning and memory . the solution looks so easy (getting socialize) but i just can t it take so much courage to talk to someone beside my family. please doctor anyone help me i don t get so much back up from my family and you are my only hope Doctor: My dear friend,please feel free to contact a psychiatrist and a behavioral therapist in your area.sooner the better.I would really suggest that you relax a bit.Relaxation techniques like yoga, meditation and even music may helpSpend time with someone you trust.You need not necessarily talk about things that you dont want to share.But just hanging out with this person may help you.I am hesitant in suggesting medicines because these group of medicines have a huge number of side effects, and it is always better that a qualified person does that properly and review your condition accordinglyBest of luckDr Mittal"
},
{
"id": 99529,
"tgt": "Can dust allergy be cured completely?",
"src": "Patient: hello doctor.. im 26 F suffering from dust allergy from my childhood... symptoms are wheezing, running nose, itching eyes, cant go outside during summber. yesterday i went on allergy test...i get this dust allergy ly in summer from feb tto may . i wanted to know 1) will this allergy get cured permanently 2) will it affect to other parts of our body after consuming medicine prescribed from allergy center3) will it continuous to next generation also plz reply doctor i wanted to get ride of this dust allergy Doctor: Hello,Thank you for asking at HCM.I went through your queries and would like to make suggestions for you as follows:1) Will allergy get cured permanently:In modern science, allergen specific immunotherapy is a hope for long term relief from allergy symptoms as it works on immune system. Allergen specific immunotherapy is usually prescribed by Allergists-Immunologists.2) Will it affect other parts of body:As you have wheezing, nose & eye symptoms, you are allergic from air-borne allergens. Aeroallergens usually cause allergic rhinitis, allergic conjunctivitis, allergic asthma etc. It important that they are treated promptly. For example, if allergy rhinitis is not treated promptly, it could lead to long standing nose obstruction and can affect hearing, sinuses, etc.Rarely aeroallergens can cause dermatitis or urticaria.However, if you are allergic, it is possible that you develop allergies to other allergens with progression of time. Allergen specific immunotherapy can halt this process of progression of allergy.3) Not all, but 40-60% of children of allergic parents can have allergies. It is difficult to say which children will get allergy from parents. The chance will be higher if both parents are allergic.Hope above information will solve your queries.Should you have any further query, please feel free to ask at HCM.Wish you the best of the health ahead.Thank you & Regards."
},
{
"id": 125773,
"tgt": "How can swelling and pain in the wrist be treated?",
"src": "Patient: My fiance s wrist was red and started swelling this morning. The wrist was sore to the touch. When she arrived home this afternoon, the redness had spread and, the soreness is still there. She also said that her wrist was stiff. She experienced a similar situation with her ankle about 7 months ago. We had her blood tested, sonogram, ultrasound tests all showed nothing. It took about a month and, it cleared up. Any suggestions? Doctor: Hello, As of now, you can use analgesics/anti-inflammatory combination like Aceclofenac/Serratiopeptidase for symptomatic relief. If symptoms persist better to consult an orthopedician and get evaluated. In that case, an MRI scan might be required. Hope I have answered your query. Let me know if I can assist you further. Regards, Dr. Shinas Hussain, General & Family Physician"
},
{
"id": 107396,
"tgt": "Suggest treatment for chronic back pain due to disc osteophyte complex",
"src": "Patient: I ve been suffering back pain for a long time now. I ve had several MRIs done on my back, C5 through C6 demonstrates moderate midline ventral defect extending to the right and left of the midline representing disc osteophyte complex can you please help me with my problem? Doctor: Hello,Regarding your condition insuggest you to have some seances of physical therapy to strenthen your muscles and some antiinflamatory drugs such as meloxicam to relief the pain.Best,Anisa"
},
{
"id": 93713,
"tgt": "Abdominal pain, frequent urination, urge to urinate, tingling in hands and feet, thirsty, fatigue, irregular heart beat. Cause?",
"src": "Patient: I have been having lower to upper abdominal pain for 4years now. This year I have experienced frequent urination along with urgency and have had 4 UTI treatments in 6 months. Urologist thought I had ICS but that came back as normal after he looked at my bladder. My brother was diagnosed with primary amyloidosis recently and I m wondering if I could have this. I have tingling in my hands and feet at times, always thirsty, very fatigued all the time, constant abdominal pain, irregular heart beat the list keeps going. I m not sure what to do from here. Docs keep telling me everything is normal when tests get run. Doctor: Hi welcome to H.C.M.Forum. Thanks for calling H.CM.F. You have got pain in upper and lower abdomen, since 4 years, and now you got frequency, urgency and got treatment for U.T.I.4 times in 6 months. You also have thirsty, tingling in hands and feet, fatigue, constant abdominal pain, irregular heart beat, Your brother got amyloidosis , you had a doubt whether you also got the same. your problem appears to be urinary tract infection, or stones in the urinary tract. I advise you to consult an urologist for diagnosis and treatment. you may need to have urithroscopy, and culture and sensitivity of the urine, besides other routine tests for confirmation. Wishing for a quick and complete recovery. Best regards."
},
{
"id": 87998,
"tgt": "What causes headaches and abdominal cramps?",
"src": "Patient: Hello I am a 29 yr old single mother of three. I have been having headaches the past week while feeling very tired. I have also noticed some tingling in my right hand and today noticed some severe adominal cramping. The cramping is gone but other symptoms still persist. My stress level has decreased tremendously over the last year so i don't feel that is the issue. Please help. Doctor: Hi.Thanks for your query. The symptoms of headache and abdominal cramps for more than a week are indicative of a general disease like Typhoid or so. Please get the blood checked for Widal (Typhoid) and sugar and other relevant tests as ordered by your Doctor on clinical examination. Get a course of an antibiotic and symptomatic for headache and other symptoms if you have."
},
{
"id": 37455,
"tgt": "Suggest treatment for the stomach burning sensation due to viral infection",
"src": "Patient: I have had a stomach virus for 3 days with dry heaving. I now am having trouble swallowing due to the dry and acid burning feeling in my throat. It feels as if I have no saliva there, and it burns terribly. I cannot sleep, and am sill occasionally vomiting. Whenever I vomit it is excruciating due to the acid burn...what can I do to relieve this pain? Doctor: Hello,I understand your concern.I am Dr Arun Tank, Infectious diseases specialists, responding to you.All your problem is related to acidity. I advice you to take pantoprazole and domperidone as antiacidity and anti vomiting agent respectivelyCertain lifestyle changes like eating less spicy foods and stress free life, complete sleep is all necessary. I will be glad to answer your further query. Thank you for your contact to health care magic.Wish you a best health.Thank you,Dr Arun Tank."
},
{
"id": 189958,
"tgt": "Damaged teeth post accident, slight protrusion. Root canal done, metal fused porcelain bridge. Permanent cure?",
"src": "Patient: hello doctor i am 35 .at the age of 30 i have an accident and my 3 front teeths are damaged. before the accident my front teeths are slightly protruded and i am very shy on that.my dentist done root canal and a metal fused porcelain 3unit bridge is done.he said that there is a chance for protrusion in the future and this type of bridge can withstand that.after five years i dont feel any problem .but now a days i feel anxious that did this brige protrude ?did i look goofy in the future.i did dental check ups every year and dont find any problems.please give me your kind answer. Doctor: Hello & welcome, Protruded teeth are more to get traumatized or broken as compared to normally aligned teeth specially in children. But as you had an accident leading to breakage of the front teeth, Root canal treatment followed by placement of three unit bridge was correct decision. If natural teeth were protruded, then while making bridge this alignment can slightly be improved by adjusting the placement of the bridge. But this carries physiological limits with it. If the bridge is in the place properly, then never think that it will let the underneath teeth to protrude more than present. This is because bridge itself is an obstruction for teeth movement. But still movement can occur only if the abutment teeth are peridontically weak or patient presents with poor oral hygiene. Keep your cleaning habits proper. Visit your dentist regularly for clinical examination so that any slight movement can be anticipated & treated immediately. Take care."
},
{
"id": 197745,
"tgt": "Suggest me natural remedy for the semen leak out",
"src": "Patient: Respected Doctor(s), My age is 21. I've been masturbating repeatedly since I was 14. I now observe that My semen continues to leak out even When I am not under the state of arousal. Perhaps, As a result of the same, I'm very weak in spite of my diet being quite rich and nutritious. Please help me out with some natural remedy with the help of which I could cure this problem. Doctor: Dear, We understand your concernsI went through your details. Semen leakage is a natural process and is neither related to your masturbation nor can create any physical health conditions. Please consult a psychologist for counseling.If you require more of my help in this aspect, please use this URL. http://goo.gl/aYW2pR. Make sure that you include every minute detail possible. Hope this answers your query. Further clarifications are welcome.Good luck. Take care."
},
{
"id": 50234,
"tgt": "Suffering from frequent urges to urine, back pain, increased creatinine. Could it be kidney infection?",
"src": "Patient: i have just had a blood test and urine analysis which has shown my blood creatinine level to be 73 with estimated GFR of 75 previous reading have shown between 73-85 - however my urine analysis showed up an albumin/creatinine ratio to be 6.26 with urine creatinine of 1756 -a week ago it showed up at 4. About 30 years ago i got nephrotic syndrome from strep infection i have never had a recurrence but my filtration was slightly damaged but the kidney function has been performing ok to now . over the last week i have recently been suffering from frequent urges to go pass urine, pain in my lower back, fatigue and generally feeling unwell i have noticed slight frothy consistency to my urine - i did think that i may have UTI but the dip stick showed up negative on all counts of protein, blood and white cells - the culture is awaited. i am very concerned that i may have a recurrence of nyphretis as i was fluey recently but i had a throat swab which was negative for strep . Up to now recently i have been symptom free from kidney problems i am monitored by a nephrologistDo you think that i could have a kidney infection /uti which is driving increase in creatinine and albumin ? many thanks for your advice Doctor: I must appreciate that you under the supervision of your Nephrologist are taking care of your kidneys.The symptoms you have described suggest an 'Over Active Bladder' which can be taken care of by Tab Oxybutynin 2.5mg OD.You may also start Tab Sodabicarb(500mg) 1 tab ODYou GFR is slightly reduced but can be maintained by taking care & avoiding NSAIDS & other nephrotoxic drugs & diet items.All the best"
},
{
"id": 166645,
"tgt": "What causes body rash, swollen and red tongue?",
"src": "Patient: My 3 year old daughter had a temperature a few days ago, a very white fury tongue and smelly breath, today she is fine as in no temperature but her tongue is red, with all her taste buds raised or pronounced or swollen ..... she complained this pm that it was painful, also i noticed she has a slight rash on her body ...could this be a result of the temperature? Please could you advise me what the tongue complaint could be aswell. Thank you Doctor: Hi,Welcome to HealthcareMagic.Thank you for your question. Symptoms of rash, temperature and swollen tongue maybe due to an allergic reaction. It maybe a mild reactions. I would suggest you to try and recall if she had some food/medicine before the reaction and be careful or avoid it in future. It would be better if you can meet your family doctor/paediatrician for further evaluation.I hope this answers your query. Good luck.With best regards,"
},
{
"id": 58097,
"tgt": "Have fatty liver. Suffering from back pain and abdominal pressure. What could be causing this?",
"src": "Patient: Hello I am a 35 year old male I had a recent blood test that showed high liver enzymes but it is not the first time that it has happened. I have been diagnosed with fatty liver after a CT scan 6 years ago. And again 3 years ago after an ultrasound. This time I was taking Tylonel and alot of advil for back pain prior to blood test. I was also taking omparozole for reflux . I have gotten on a diet and have lost 15 pound I am starting to feel much better but still had some abdominal pressure in my upper stomach near my sternum after some meals and a lack of appetite. I started to take omparizole two days ago and last night had some back pain while sleeping on both of my flanks. I had a bowl movement this mourning and it was very light color. I look at my bowl movement every day and this is the fist I have seen that color. What could be the cause of this? Doctor: Hi,Thanks for using Healthcaremagic,After going through all the available details, You certainly have fatty liver and your liver enzymes are elevated.Your enzymes could be high due to fatty liver, or toxicity to paracetamol or ?alcohol.Even Omeprazole can cause it I presume you are taking treatment under supervision of Physician.It is good that you are losing weight and taking care of diet. It will help to even reduce fatty liver.Please visit a Physician and get your Liver functions,Lipid profile, Ultrasonography of abdomen to see state of liver.You should continue to diet an and avoid alcohol. Please do not take medicines without proper prescription of doctor.Light colour of stool indicates cholestasis or obstruction to bile flow which can be detected by liver function and USG.Hope this information helps.TAke care.Good Luck.Dr.Akhilesh Dubey M.D."
},
{
"id": 183339,
"tgt": "Suggest treatments for painful sores on the tongue",
"src": "Patient: I quit smoking 23 days ago - about 7 days ago - I woke up with a very sore tounge - thought I bit it overnite - but actually I have two large cracks near the center rear of my tounge - very sore especially when eating - is there any treatment for this? Doctor: Thanks for your query, I have gone through your query.The painful soreness on the tongue is because of the major aphthous ulcer or fissured tongue. This aphthous ulcer occurs secondary to the quitting the habit of smoking. This will go off gradually. Nothing to be panic. Consult a oral physician and get it evaluated. Take topical anesthetic and analgesics like anabel gel. apply 3-4 times daily before food. If the lesion does not after taking this, then you can topical steroid like triamcinolone acetonide 0.1% 4-5times daily after food. Do saline gargling. Maintain the oral hygiene.I hope my answer will help you, take care."
},
{
"id": 118213,
"tgt": "Suggest treatment for hemolytic anemia",
"src": "Patient: I am 49 year old female. Underwent Aortic Valve replacement surgery 1 year ago.Due to low HGB Iron etc. over the course of this year I have had 10 IV iron infusions.HGB is finally up to normal (13.2); however platelets remain high 564; RDW 20.4; LDH 227; Hepataglobin less than 17 and Iron Saturation at 13. Have seen hematologist - say's hemolytic anemia.My question would be can this improve or should it worsen, what am I looking at long term? Doctor: Treatment of hemolytic anemia depends on the type and cause of the hemolytic anemia. In emergencies, a blood transfusion may be needed.For hemolytic anemia caused by an overactive immune system, drugs that suppress the immune system may be used.When blood cells are being destroyed at a fast pace, the body may need extra folic acid and iron supplements to replace what is being lost.Rarely, the spleen may need to be removed, because it acts as a filter, removing abnormal cells from the blood.RegardsDr De"
},
{
"id": 61820,
"tgt": "What causes fluid filled lump in groin area?",
"src": "Patient: I have a lump/pocket that feels like fluid is in it . Near my groin. It s uner my testicles and on my butt cheek. I went and popped it and blood came out it filled up a few times with blood and the pain has stopwd. I noticed it looked like clotted blood came out. Doctor: Hi..Welcome to HEALTHCARE MAGIC..I have gone through your query and can understand your concerns..As per your complain a fluid filled lump in groin area over the butt cheek seems to be an abscess or a boil..It can also be Hair follicle infection causing Folliculitis..I would suggest you to consult a General Surgeon and get evaluated and he can advise you Oral antibiotics like Sulfamethoxazole and Trimethoprim, anti inflammatory painkiller like Ibuprofen and topical application of Triple Antibiotic Ointment and steroid ointment..Hope this information helps..Thanks and regards.Dr.Honey Nandwani Arora."
},
{
"id": 217830,
"tgt": "Suggest treatment for swollen and pain in legs",
"src": "Patient: at night , i was stretching and my leg muscles tightened up , it happens a lot and it hurts for a while , but this time i woke up in the morning and it still hurted a lot , now i cant walk on it right and its swollen and the pain went up to my thighs , and my legs feel a little heavy , what do i do Doctor: Hai, acute pain, sudden onset could be due to sciatica or any other cause,pls check out for any dilated veins on the leg.keep ur limb elevated at least for 30 degrees, at restdo not keep standing posture for long timeu can go for tab ultracet (pcm+tramadol) for pain.pls check out for ur sugar levels, thyroid profile, venous doppler of ur leg.do slow physiotherapy to release the spasm."
},
{
"id": 87462,
"tgt": "Suggest treatment for right abdominal pain",
"src": "Patient: I have been experiencing right abdominal pain like sharp shooting pains to the side of my navel and a bit lower than that for the past five hours. Last night I had pain in the middle of my tummy just under my ribs before this pain began. When I bend, turn or move about too much I feel the pain in my abdomen. If I touch the area it seems to help but then the pain comes back as soon as I let go and sometimes hurts a bit more. I am 29 years old, weight 54kg and 159cm tall. I am currently on the Auto-Immune Paleo Protocol diet to aid my rheumatoid arthritis. Doctor: Pain can be because of Stones, either gallstones or renal stones Appendicitis Gastritis, Diet modification, usg whole abdomen, rantac /pantocid SOS for acidity To review with reports"
},
{
"id": 84483,
"tgt": "What is the use of susten 100 injections?",
"src": "Patient: I am 30yer I am 30 years old and pregnant for 7weeks . On the 7th week I had some blood coming out from my vagina. After seeing doctors she advised me to take 8 doses of susten 100 inj daily. Could anybody advise me what is the use and suspection? Doctor: Hi,It is used to support the pregnancy. As per the history you seem to have vaginal bleeding suspected to be due to missed abortion for which your doctor advised you to take injection susten. Injection susten contains natural micronised progesterone which is commonly prescribed during pregnancy when progesterone deficiency is suspected. It helps to maintain early pregnancy until the placenta completely takes over, hence found to be useful in preventing the miscarriage.Hope I have answered your query. Let me know if I can assist you further. Regards, Dr. Mohammed Taher Ali, General & Family Physician"
},
{
"id": 62096,
"tgt": "What causes bump on testicles behind the scrotum?",
"src": "Patient: hi i have a bump on my testis behind the scrotum it pains when touched personally speaking i sort of like it and thought of squeezing it like right now but is this lump normal could someone had injected me i plan to do medical testing should i remove bump for a second or two i seem to like the pain whe n i go to the spot and squeze it Doctor: Hi,Dear,Thanks for your query to HCM.Studied your query in full depth of its details.Reviewed it in context of your health concerns.Based On the facts,You seem to suffer from-Epididymitis / or from Epididymal cyst/ or cyst of the Appendix of the Testes.USG of the Bump on the testicle behind the scrotum would resolve the cause of such a lump.This reply would help you to plan further treatment soon with your treating doctors.Best of Luck and early recovery.Welcome any further query in this regard,which would be replied in next session.Good Day!!Dr.Savaskar M.N.Senior Surgical SpecialistM.S.Genl-CVTS"
},
{
"id": 63837,
"tgt": "What causes a lump above the thyroid gland?",
"src": "Patient: hi I have an oblong lump above my thyroid on the right side, I noticed it when i was rubbing cream on my neck but initially it was tiny but it has gradually got bigger. Could it be a lymph node? i am feeling fine but have been quite tired over this past 6 weeks. I have had this lump now for roughly 3 months. Do I wait longer or visit doctor? And what do i say? Doctor: Hi,You need to consult a Surgeon urgtly.Its mostly a lymphadenitis ? above rt thyroid gland ? Jugulo-Diagastric lymphnode is the possible lump.Fnac Biopsy of the lump would fix the diagnosis in your case.This reply would help you to plan treatment with your ER doctor at earliest.Welcome for any further query to Me at my virtual HCM Clinic.Wishing you early recovery.Hit Thanks if this reply is helpful to you.With Regards."
},
{
"id": 213512,
"tgt": "Silent behavior, not taking bath, depression, lethargy, recurring condition. Treatment for poor mental health?",
"src": "Patient: hi, my brother age is 28 yrs old, since he was 16yrs old, he use to go to silence behavior, he does not talk to any body, he does not take bath, he most of time lying in room, he talks himself some times, in a month mostly 15-20 days behave like this, when he comes out of his silence behavior, he talks more to the people and when he goes to take bath, first he wash all the area including wall & floor. Please advise me where have to go for treatment . Doctor: hallo thanks for the concern your brother seems to have some psychological problem thats bothering him alot. he need moral support and counselling from all the famoly members, and you need to consult a psychiatrist for treatment approaches."
},
{
"id": 177974,
"tgt": "How should nasal bleeding caused by trauma be treated?",
"src": "Patient: My 3 year old son was climbing on a child chair, the chair flipped and he hit his mouth. He cried and whined and fell asleep. I just noticed some blood in his nostril. Do I continue to let him sleep or wake him up? I am assuming he bumped his nose when the chair hit his mouth. Doctor: a fall of this nature can cause nasal bleeding please keep a cold pac ,if bleeding continues please take to ENT surgeon"
},
{
"id": 32121,
"tgt": "Can polio be treated to be able to walk without caliper?",
"src": "Patient: i m 28yrs from punjab amritsar. i m suffer from polio when i m only 3 years old with reaction of injection in fever. i operater from visakhapatnam in 1998. after operation i walk with caliper. my back lower portion & upper portion of knee are too weak. can i walk properly with treatment. Doctor: Hi dear , I have gone through your question and i understand your concern.I feel very sad to inform that there are currently no effective pharmaceutical treatments that can stop deterioration or reverse the deficits caused by the post polio syndrome .However, a number of controlled studies have demonstrated that non fatiguing exercises may improve muscle strength and reduce tiredness. Non fatiguing means,exercise that don't let you fatigue - mild to moderate exercise.My dear friend i want to share here that exercise is safe and effective when carefully prescribed and monitored by experienced health professionals and you must also consult nearby physiotherapist.Exercise is more likely to benefit those muscle groups that were LEAST affected by polio. Cardiopulmonary endurance training is usually more effective than strengthening exercises, especially when activities are paced to allow for frequent breaks and strategies are used to conserve energy - means do not fatigue yourself while exercising.Heavy or intense resistive exercise and weight-lifting using polio-affected muscles is not advised, as this can further WEAKEN rather than strengthen these muscles. Hope you find the answer helpful, please do get back for further queries I will be happy to assist you.Please rate 5 stars as you find some benefits with this information, it will ignite me to help more.Wishing your good health.RegardsDr.Vigyan Mishra"
},
{
"id": 49752,
"tgt": "Had operation for kidney stone. Having weird pelvic pain, getting early periods. Cause for the pain?",
"src": "Patient: Hi, I am writing beacuse i have been having weird pelvic pain . I was in the hospital 2 months ago for kidney stones and after a few weeks the pain had not gone away. I went back for serveral tests, and i have been told that the stones are out. I have extremely sharp burts of pain in my lower pelvic on my left hand side. Then last week, i got my period 5 days early and it only lasted 2 days, not heavy, but i still have pain. Would you have any ideas as to what would be causing my pain? Thank you in advance. Megan Savage Doctor: HelloThanks for your query.Based on the facts that you have posted it appears that you have been treated very recently for kidney stones and have pain in pelvic region since three days.This is most likely to be due to residual stone( small Fragment of stone) in ureter .Please get your ultrasound scanning of the abdomen done to rule out the same and consult qualified Urologist for clinical assessment and further treatment.I think you should be fine with antibiotics and anti spasmodic drugs for a short period of 5 days.Dr.Patil."
},
{
"id": 78374,
"tgt": "What could lead to rise in BP in Chronic obstructive pulmonary disease?",
"src": "Patient: I have COPD and have not been feeling well for several days. Today I have taken my b/p and when lying down the diastolic was 41. That was over an hour ago. I have been sitting in a chair for approx 1 hour and now my b/p reads 108/46. Should I be concerned? Doctor: Thanks for your question on Health Care Magic. I can understand your concern. Your diastolic pressure is very low. Normal diastolic pressure is between 70-90mmhg.So you are having diastolic hypotension. And it is common in COPD patients due to right sided heart failure. So possibility of right sided heart failure is more in your case. So consult your doctor and get done 2d echo to rule out heart failure. You may need diuretic and cardiac inotropic drugs too. Better control of COPD is also important. So take your inhalers timely. Hope I have solved your query. Wish you good health. Thanks."
},
{
"id": 87121,
"tgt": "What causes pain in the abdomen?",
"src": "Patient: Dear doctor, I am 34 year female, married, have 2 sons aged 20 and 17. i work in a corporate office. some 12 years back i had a acute on my right abdomen and doctor doubted appendicitis but the pain went away. Then aftersome years the pain on the same abdomen occurs sometimes and goes, but slowly my montly period became heay lasting for 7 days and some kind of piece of blood comes, feel thirsty, no appetite, feel like eaten too much, gas formation, some whitesh discharge comes sometimes, back pain, some giddiness while geting up from chair/bed. The monthly period com on time. after the delivery of two sons tubotomy was done as both the delivery was ceaserean. Recently about 20 days back again i had a acute pain on the same abdomen with feeling of something is struct inside, feeling like vomiting sometimes, overeating, thirsty, no appetite. This time one doctor after doing all the checks and ultrosound says tumor in ovary, need surgery and i referred to second doctor, who conducted all the test and check says not tumor but cyst in ovary, need not surgery. The pain is better but still there little. The pepsimer and malingness test is noram.no medicine given. advised to review after 6 months I am confuesed with two opinion. If doctor could give some advise on this and also your suggestion to diet and other means to control or reduce the pain. Doctor: Hi I am sorry to know about your illnessI can understand your concernat this point of time you need not to worryif you can upload your ultrasound reports I would be able to help you bettermeanwhile have plenty of fluids avoid fatty and fried foodPhysical exercise dailyhave tab pantoprazole 40 mg half an hour before breakfast to avoid hyperacidity and related abdominal symptomsHope this helps you you can ask for further query if anythanks for using health care magicwish you good health"
},
{
"id": 212282,
"tgt": "Unstable mental condition. History of using meth and other drugs. Mental breakdown?",
"src": "Patient: my daughter, who has used meth and other drugs, believes that we (her family) are evil all powerful people who are trying to kill her. She believes all manner of strange things she sees or tv or in books. She has been so unstable in recent visits that we are getting a restraining order for her children. Could this behavior be from the drugs she has used or a mental breakdown? Doctor: Hi,thanks for using health care magic. Her symptoms may be due to 1) effects of mehtamphetamine particularly if she taking it presently. 2) Psychiatric illness like schizophrenia 3) personality disorder As she is harmful to self, she needs urgent psychiatric help and hospitalization too, if necessary. You should consult psychiatrist as soon as possible for detailed evaluation and treatment. thanks"
},
{
"id": 182490,
"tgt": "What can be the cause of swelling and pain in gums and teeth?",
"src": "Patient: I am a 19 year old female. I take good care of my teeth everyday. Three days ago my gums began hurting and swelling on one side of my mouth it has now spread the entire way across my top gums and is starting on the bottom ones. What could cause this and do I need to see a doctor? Doctor: Do you have complains of food lodgement in your teeth and gums and have your gums ever bled? The reason for you gums to flare up like this can either be due to an infection, allergy or hereditary reasons. If you have been taking good care and have had no complains before then it may be infectious and hereditary presents with painless growth and so i feel it could be an allergic cause which may have started due to some food which you had in recent past or just in case if you have tried a new tooth paste or a mouth freshener. You can take a simple anti- allergic like cetirizine which wont harm in either case and report to a dental specialist to get it examined."
},
{
"id": 126367,
"tgt": "What causes tightness in the hamstring when I sit down?",
"src": "Patient: My symptoms are: My left hamstring started tightening up a few weeks ago, especially when I sit down. This has been followed by pain in my knees (burning at times on sides) and tightening of my calfs. Sometime stiffening up so much that it is hard to walk. I just turned 56, but pretty muscular for my age. My only medical condition I had was after my physical a few weeks ago my TSH level was 7.5 so Dr. has increased my thyroid medicine from .05 to .1. Hopefully I do another thyroid panel in a week or two. I have been on thyroid meds for 4 years. Not sure this is cause although I do know one symptom of hypothyroidism is muscle pains. Doctor: Hello, It can be due to muscular pain as in pulled muscles. As a first line of management you can take analgesics like paracetamol or Diclofenac for pain relief. If symptoms persist you can consult an orthopaedician and get evaluated. Hope I have answered your query. Let me know if I can assist you further. Regards, Dr. Shinas Hussain, General & Family Physician"
},
{
"id": 164903,
"tgt": "Are there chances of puberty after spotting blood in 9 year old?",
"src": "Patient: Hi, may I answer your health queries right now ? Please type your query here...please help me sir/madam my daughter 9-year-old having spotting blood since four days then i consulted gynecologist then she advised to get urine complete analysis the result was pus cells 1-2, epithelial cells 1-2, and acidic reaction. i doubted that she may have puberty or not. Doctor: Hi, thanks for the question. your childs urine analysis is normal. yes your child might be having puberty"
},
{
"id": 137955,
"tgt": "Suggest treatment for pulled groin muscle and low grade fever",
"src": "Patient: I have what feels like a pulled groin muscle(s), but I also have a low grade fever. I exercise but there is not a specific incident that I believe would have caused me to strain the muscle this much. It hurts to walk. What type of doctor should I go to Doctor: Hello,Thank you for contacting me at \"Ask a Doctor\". I will try to answer your question to the best of my ability.I would check in with your primary care physician. They should be able to treat simple problems like a groin strain. The low-grade fever may be completely unrelated and probably needs the primary care physician to check for causes of infection.Take some simple over-the-counter medications such as Tylenol. This will help with the pain in the groin and also reduce the fever.I hope this information is useful for you. Please do not hesitate to message us back if you have any further questions.Best wishes,Adrian Rawlinson MD"
},
{
"id": 199720,
"tgt": "Could masturbating 4 to 5 times a day effect health?",
"src": "Patient: Hello, Doctor! My name is Mark and I;m from the Philippines. I would like to ask you about this sickness that I ve been having lately. You, see I ve been separated w/ my wife for almost two years now. Sometimes I masturbate as much as 4 to 5 times a day. But, of course, this doesn t happen everyday. Say...this number of times per week. The problem is that sometimes I do it all (4 - 5 times) in just one day. I m already 44. Will this affect my health? Thank you. Doctor: Hi,Masturbation has no bad effect in your health or sexual life or getting weakness.It is just like you go for intercourse.If you do not loose your sperms through masturbation or intercourse, it is naturally removed from the body through kidney after entering into blood flow.But keep in mind that every thing in excess, always to be avoided.Ok and take care."
},
{
"id": 154703,
"tgt": "Is surgery needed for tumour in pituitary gland?",
"src": "Patient: Hi. i am a pilot and there has been an incidental find of tumour in my pitutary gland. I have no other symptoms as headaches etc. at present the growth is about 9mm. i started wearing glasse about 5 years ago. there is alight reduction in my left eye range vision . what do you suggest. should i go for surgery. Aijaz Doctor: Hello dear, thanks for your question on HCM. I can understand your situation and problem. In my opinion you should definitely get it removed surgically. You are symptomatic with glasses and difficulty in vision. And your tumour is still very small. So prognosis is good if you removed it early.It will slowly grow and can damage eyesight more. So better to remove it at present and avoid future complications. Another point which favours early removal is your job.You are pilot and your eyes are everything for you. So better not to wait, get done surgery."
},
{
"id": 146904,
"tgt": "What causes headache , nausea ,numbness in arm and legs?",
"src": "Patient: my father has been diagnosed with a infarct in his right side of his brain on 30 april was getting treatment and was feeling better but fron last 20 days he is started hving problems severe headache, nosia, uneasyness , numbness in left arm, left leg and ribbs. Doctor: I think you will need to get him to the hospital againThere is a strong chance that he has developed another complication in the brainAnd this can be tested by an MRIOf course there are other possibilities. One of the main ones being a low sodium level. This can have all the symptoms that he has, and may in fact feel like he is having another stroke.So testing that would be as important. And can be done in a hospitalBest of luckDr Mittal"
},
{
"id": 61957,
"tgt": "Suggest medication for a lump in left armpit",
"src": "Patient: My daughter has a lump about the size of a golf ball in her left armpit. It is semi soft but does grow bigger and harder and then it proceeds to leak milky fluid out and goes down in size. She is pregnant now and had it happen 4 years ago when she was pregnant with my other grandchild. She has been to the doctor and they said it was not lymph nodes but something to do with ducts. As I am not there with her to go to the doctor, do you have any thoughts on what it could be? Doctor: Hi, dearI have gone through your question. I can understand your concern. She has lump in armpit with milky discharge. It increase in size at time of pregnancy. So it can be pregnancy induced changes in axillary breast or galactocele. She should go for fine needle aspiration cytology and if needed go for mammography. It will give you exact diagnosis. Then you should take treatment accordingly. Hope I have answered your question, if you have doubt then I will be happy to answer. Thanks for using health care magic. Wish you a very good health."
},
{
"id": 64131,
"tgt": "Suggest remedy for lumps on arm",
"src": "Patient: I have a pea size lump in the crease of my arm (where you would get blood from) I have had it for years. It got a little bigger initially but then stopped and has been the same for years. Its not sore and and I can wiggle and move it around slightly Doctor: Hi,Dear,Thanks for the query to HCM.-I went through your query in-depth and Understood your health concernsRemedy and Cause-In my opinion lump on arm being for yrs and has grown not much lately-indicates its a BENIGN tumour?-LIPOMA -solitare ? or could be a Sebaceous Cyst ? Lipofibroma? Fibroma.-You should not be concerned about these painless and HARMLESS lump,unless they grow out of proportion or cause some pain.Remedy is by-Excision Biopsy -as it worries you.If not ,just NEGLECT if your ER Surgeon says after the check up -which is a must STEP.Hope this would help you to work a plan with your doctor.Wishing you early and fast recovery.Wellcome to my HCM Clinic for further queries in this matter.Wishing you Good Health ASAP.Have a Good Day .Regards,Dr.Savaskar ,M.S.Genl-Thorasic CVTSSuper specialist in Asthma/Cancer and non-curable diseases."
},
{
"id": 48578,
"tgt": "Why ferrous sulfate is prescribed for diabetic patients?",
"src": "Patient: My hemoglobin is 9.7. Tsat is 3% and ferritin is 1. I have been short of breath and fatigued and now I feel like my vision is being affected. I am an RN/BSN working in dialysis so I am very familiar with these labs. My doctor started me on ferrous sulfate TID but I am not feeling any better. Is this blurry vision normal? Should I be getting other treatment? Doctor: Hello and welcome...Yes you should continue this drug...you are anemicYour hemoglobin is low.hemoglobin synthesis needs iron and ferrous sulphate us iron...blurred vision is not associated with ferrous sulphate..it has other causes like DM,eye I nfections,trauma optic neuritis etc.Hope yiy would understand..."
},
{
"id": 74629,
"tgt": "What causes chest pain and breathing disorder?",
"src": "Patient: I have anhidrosis, dry skin, sculiosis and trouble breathing at age 21. Im a female, and I was wondering if any of these things can cause pain in the chest and breathing disorders. A recent blood test for life insurance shows I'm okay in all ranges, cholestrol etc... Doctor: Respected user , hiThanks for using Healthcaremagic.comI have evaluated your query thoroughly .* Scoliosis if severe can lead to chest pain & breathing disorder as chest is deformed in shape Minor scoliosis is not related with that , in which case you have to be evaluated for the reason clinically and if needed x-ray , pulmonary function test or other Hope this clears your query .Welcome for any further assistance .Regards ."
},
{
"id": 66367,
"tgt": "How can bump on fore arms be cured ?",
"src": "Patient: I have hundred of small bumps on my fore arms that have come and gone for 20 years. They are slightly darker than my skin. I recently read using duct tape for a week will evoke an immune response, and get rid of these. Also read soak in very hot water for a matter of days. Anything internal I can take to switch it off or make them go away. Sink? Doctor: Hi, dearI have gone through your question. I can understand your concern. You may have either multilple lipomatosis or neurofibromatosis. Biopsy will give you exact diagnosis. Only treatment is surgery. If you want then you can go for surgery otherwise no need to worry about that. No any treatment is required. Hope I have answered your question, if you have doubt then I will be happy to answer. Thanks for using health care magic. Wish you a very good health."
},
{
"id": 80482,
"tgt": "How to cure pneumonia in one lobe, coughing pus and oxygen level going down?",
"src": "Patient: My 24 year old son has been sick for a month coughing he had pheumonia in one lobe of one lung took antibioics seveqal times now hes coughing up pus tgjat smells really bad whats wrong tgjat he cant get bettes his oxygen goes down to 61at night Doctor: Hello dear, thanks for your question on HCM. I can understand your son's situation and problem. Pneumonia if uncontrolled can lead to mucopurulent sputum and fall in oxygen level.The best treatment for pneumonia is to identify the causative organism and then start effective antibiotic against it.So get done sputum culture and sensitivity report. Culture will isolate the causative organism and sensitive will tell you about the antibiotics effective against that particular organism.By these, we can achieve fasten recovery and prevent haphazard use of antibiotics. So consult pulmonologist and discuss all these."
},
{
"id": 133606,
"tgt": "Suggest treatment for ankle pain",
"src": "Patient: my ankle hurts but i dont no y i was sitting down on the floor and when i got up it started hurting i ignored the pain till i got home when i got home in was swolen i hurts wen i walk i put ice hot on it then the next day it was still hurting y is it hurting still Doctor: hi.Thank you for providing the brief history of you.A thorough musculoskeletal assessment is advised.As you were sitting on a floor, the position of the feet must have led to some ligament sprain so you have swelling now. By the application of the ice and keeping the leg elevated should reduce the pain and swelling.RegardsJay Indravadan Patel"
},
{
"id": 167952,
"tgt": "What causes a hard lump on the knee of a 7 year old?",
"src": "Patient: Hi, My 7 year old daughter has a hard lump the size of a golf ball on her knee she has had it for about a month. Originally she complained of knee pain with no known injury the pain went away quickly and now she just has the lump with no pain at all. What do you think could be causing that ? Doctor: Hi... by what you could I feel that it could be a fibrosis or Tendinitis. But as it is not causing any pain now it is not an emergency. But I suggest that you see an orthopaedic surgeon regarding this.Regards - Dr. Sumanth"
},
{
"id": 179225,
"tgt": "Suggest remedies for fever in a child",
"src": "Patient: hi dear doc my baby is twenty months old she was having fever since last two days cough since last five days ,her cbp shows 27% of polymorphs and 62% of lymphocytes ,over all lymphocyte count is 13900 and her crp level is 0.10 ,main worrying part is she is not taking medications orally ,is there any other alternative for me rather than iv injectables pls kindly let mee know sir Doctor: Hello. I just read through your question. This is very common in pediatrics and there are many babies who refuse to take medication orally. To reduce fever, there is a rectal suppository form of acetaminophen that can be use. It is very effective. Additionally, cool compresses to the forehead can also often reduce fever. I suggest trying these methods to help make your daughter more comfortable."
},
{
"id": 79690,
"tgt": "What is the cause of chest pain?",
"src": "Patient: On several occassions the last few years I have had a stinging pain run across the top of my chest and down both arms. I'm 42 yrs old, do not smoke, and drink lightly. Weight 165, height 5' 9\". Low exercise. Eating habits have pluses and minuses. What is this feeling? It really hurts for a few seconds then it's gone. Doctor: thanks for asking your question I completely understand your question it seems like a nerve associated pain but if it is a chest pain u should rule out cardiac causes consult a pulmonologist , he can have an exg done and rule out angina or other causes of chest painthanks/ regards feel free to ask more questions may god bless you with good health"
},
{
"id": 42421,
"tgt": "What are the chances of conceiving with polycystic ovaries and thyroid?",
"src": "Patient: Hi Doctor, I am 29 years old with weight 56 KG and height 5.1. I recently got my Pelvic scan done. I am trying to get pregnant for last 3 months and by advise of my doctor, i went for this scan. The result shows that both the ovaries are mildly bulky in size with polycystic changes. Also, my THYROID test result shows TSH 3rd generation as 8.52 which i high. Please advise if I will be able to get pregnant.Thanks Doctor: Hi,Welcome to HealthcareMagic . Yes you can conceive but after your thyroid problem and pcod is treated. For thyroid you need to consult doctor and start thyronorm. For pcod you need to start metformin and reduce weight. Once both are controlled which takes 3 to 4 months pregnancy may be planned.Hope I have been helpful .RegardsDr.Deepika Patil"
},
{
"id": 163227,
"tgt": "What causes stomach churning with lack of appetite in a child?",
"src": "Patient: My child is 10 yrs old, and has had stomach churning for 4 days. No signs of fever, diarrhea, loose stools, pain, tenderness, emotional stress, nor vomiting. We are in a normal routine, so no physical/emotional upsets are involved. However she does not seem to have an appetite and has been eating less. No other family member displays these symptoms. What could be the culprit? Doctor: Hello and Welcome to \u2018Ask A Doctor\u2019 service.I have reviewed your query and here is my advice.Your concern is right. Your child seems to be suffering from subclinical infection of intestine. You may try prebiotics and probiotics available OTC.If the child develops any Frank symptoms then seek medical advice. This shall help the child mostly.Hope I have answered your query. Let me know if I can assist you further.Regards,Dr. Ratna Mulay"
},
{
"id": 183464,
"tgt": "Should root canal with crown lengthening or implantation be done?",
"src": "Patient: hi i would like to get a suggestion from you. I have got root canal treatment six years back and now i am getting problem with that tooth. food particle are stucking in between the teeth where the crown is placed and getting gum pain. i visited doctor here in boston and the doctor gave me 2 choices. one is to redo the rootcanal, crown lengthening,post followed by a crown and the second chioce is go for implantation. can you suggest me which one is good for me? thank you Doctor: Hi,Thanks for posting the query, I would suggest you to get your full mouth scaling and [polishing done also see if there is any problem with the crown fixed if so then get it removed or refixed.. Of the above two options ofcourse the first one will be good because it does not require the tooth to be removed, for the last you can think for the implant placement......Hope you find this as helpful,Take care!"
},
{
"id": 38380,
"tgt": "Suggest remedy for recurring boils on skin grafts",
"src": "Patient: I have full thickness skin grafts on about 40% of my body for about 3 years now. I have since had several boils on the skin grafts on my neck. Because they re on grafted skin I always have to go to the ER to have them lanced because no one at the urgent care will touch them. Why does this keep happening and how can I prevent it from happening? Doctor: HI, thanks for using healthcare magicSince the infection is recurring, it is possible that not all of the pus or discharge is being removed when it is incised.If a small amount of discharge remains then it would cause re infection to occur.It is also possible that the infection is not completely responding to the antibiotics used, there may be some resistance to the medication.A swab can determine the appropriate antibiotic.If the infection recurs, you may want to consider visiting an infection control specialist who may be able to initiate appropriate treatment.I hope this helps"
},
{
"id": 21184,
"tgt": "Suggest treatment for bubbling sensation on the body along with high blood pressure",
"src": "Patient: hi, I am constantly worried about a bubbling sensation I seem to have very often lately. At first i thought it was palpitations, but it is lower down and I can feel a strange sensation throughout all of my body. I am on blood pressure tablets and water tablets to control my blood pressure but i never seem to feel well, I know i suffer from panic like attacks and anxiety...is there anything I can take to remedy this...it really does get me down Doctor: your symptoms looks like they are not due to high blood pressure they may be hallucinations plz consult a psychiatrist as soon as possible"
},
{
"id": 71112,
"tgt": "What causes shortness of breath despite a normal EKG report?",
"src": "Patient: The past few years, I have been experiencing a progressively worsening shortness of breath, syncope, and recently, and many other symptoms that were diagnosed as pulmonary hypertension. I was examined by a cardiologist and a pulmonologist including a couple of EKGs, an electrocardiogram, a sonogram (right term?), etc., but an angiograph proved normal blood pressure in my heart and pulmonary artery and no other heart defects. These two experts are at a loss. What else might it be? I didn t know what PH was, let alone what all of these other conditions might be. Doctor: Hello and Welcome to \u2018Ask A Doctor\u2019 service. I have reviewed your query and here is my advice. According to the history you provide there are other causes of breathlessness despite heart problems such as lung diseases. For his you should discuss with your doctor. Regards Dr.Jolanda"
},
{
"id": 157128,
"tgt": "Suffering from bone and prostate cancer, how to reduce back pain and swelling in limbs?",
"src": "Patient: My husband is in the hospital right with vary swelen feet hanbs legs and stoumick and his back hurts to he has bone cancer from prostreat cancer thay have beentreating him with steroids and he is also a diabetic and his sugar has bee vary high so they have been giving him more and more metforman. And he said the have. Ot given him any for four days and he is now nt hurting in his Ba k and the swelling is starting to go away Doctor: HiThanks for your query.Treating bone metastases with only steroids will not be helpful. There are other medications like hormonal therapy, bisphosphonates, analgesics, local radiotherapy, etc which may be used in various combinations to give him some relief. Please talk to your doctor regarding these options.Hope this helps.Regards"
},
{
"id": 136661,
"tgt": "What causes torn ligaments?",
"src": "Patient: I have had dequervein tendonitis and it has lasted for a whole year and not doctors say that it may be torn ligaments. In the process I also had done cupping combined with acupuncture. I wander if that caused the damaged ligaments. what do I need to do to heal? Doctor: Hello, Thanks for writing to us, I have studied your case with diligence.Tenosynovitis is due inflammation and swelling of tendon .swelling on tendon may not reduce with medication alone.Local steroid injection can give good results added with physiotherapy protocol. If not relieved with conservative treatment surgical release may be required.MRI may require to confirm diagnosis.Hope this answers your query. If you have additional questions or follow up queries then please do not hesitate in writing to us. I will be happy to answer your queries. Wishing you good health.Take care"
},
{
"id": 71053,
"tgt": "Is coughing up of black phlegm a matter of worry?",
"src": "Patient: Hello there, Before bed last night I put on the electric heater for the first time this year. The heater started to smell like some thing was burning off of it like dust or some thing so I shut it off. When I woke up I had a sore throat, headache, runny nose, and a dry cough. My saliva was very thick and when I tried to cough up mucus there was not much but it was dark (almost black) and thick. My symptoms persisted through out the day. Is this some thing I should worry about? Thank you, Jeff Doctor: Hello and Welcome to \u2018Ask A Doctor\u2019 service. I have reviewed your query and here is my advice. You should be having colourless sputum. Black may indicate fungal infection. Please get a culture sputum test done. Chest X-ray is also must. Hope I have answered your query. Let me know if I can assist you further."
},
{
"id": 135728,
"tgt": "Suggest remedy for bruised knee",
"src": "Patient: my daughter is 15 and plays varsity soccer and basketball. She complained about her right knee hurting and then it switched to her left. I think that s just compensation from one to the other. We took her to an orthopedist and he said she had a bruised knee and that ice, ibuprophen an rest. He also said she should play on it as much as she could bear. does she need another opinion or maybe an mri?her knee is somewhat swollen Doctor: hitry the treatment advised for few days, and make her rest more for a week so that pain subsides totally.If this doesnt help, then your doctor would advise x ray?MRI if requiredthanks"
},
{
"id": 38472,
"tgt": "Suggest remedy for infection in breast",
"src": "Patient: hi I have a breast surgery 2x ( mastitis) but my infection and discharged keep coming back. my dr said he will do(a surgery) a permanent milk duct block. why is my infection not clearing after all the antibiotics I ve taken?im worried. all biopsies are negative. Doctor: HI, thanks for using healthcare magicIt is possible that the bacteria causing the infection is resistant to the antibiotics used.This is a common cause of repeated infection.Performing a procedure directed towards the milk duct that is the likely source of the infection should definitively prevent any further infections.I hope this helps"
},
{
"id": 25248,
"tgt": "What caused gasps for air, eyes rolling back and foaming of mouth during death?",
"src": "Patient: MY HUSBAND RECENTLY DIED - HE LET OUT VERY STRANGE GASPS FOR AIR , EYES ROLLED BACK IN HIS HEAD, AND FOAMING FROM MOUTH. I DON'T HAVE THE AUTOPSY REPORT YET BUT HE DID HAVE A THORACOABDOMINAL ANEURYSM. THEY SAID HE HAD A MASSIVE CORONARY YET HIS CARDIAC ENZYMES WERE NORMAL UPON ADMISSION TO THE ER. WHAT MAY HAVE HAPPENED? Doctor: Thanks for your question on Healthcare Magic. I can understand your concern. In my opinion, your husband might had hypoxic seizure (epilepsy) during his last breaths. Gasp of air, eyes rolling up, frothing from mouth etc are classical symptoms of epilepsy. He might have ruptured his aneurysm. So there might be great internal blood loss. This might have caused decreased blood supply in the brain. And this can give rise to hypoxic seizures and death. But all these will confirm only by autopsy report. Hope I have solved your query. I will be happy to help you further. May your husband's soul rest in peace. Thanks."
},
{
"id": 103672,
"tgt": "Child has knock knees. Has allergy to coconuts and antibiotics. What can be done to correct deformity?",
"src": "Patient: Hello doc: My 2 y old son is having knock knees, I began noticing this since he was18 month,, he his other wise fine and in good health since he was born,, he doesn t have any health issues apart from allergy to coconuts and some antibiotics .. I am afraid his condition advances and become disfiguring,, is there any thing to be done to correct this deformity?? Doctor: this DEFORMITY CAN BESOLVED BY WORKING ON ILK THEORYCHILD NEED BREAST MILK ONLY TILL 2 YEARS AND AFTER THA NO MILKIF WE ADD MILK AND DIARY PROTEINS IN DIET SOME ALLERGIES START DEVELOPING AND ALSO THE DEFORMITIESWITHDRAW MILK AND DIARY COMPLETELY FROM DIET ADD VEGETABLES FRUITS CEREALSTHE DEFORMITY WILL START REVERSING HAVE FAITH IN THIS THEORYAS MILK IS ANIMAL PROTEIN AND ANIMAL PROTEINS ARE NOT COMPETIBLE WITH HUMAN PROTEINS"
},
{
"id": 171717,
"tgt": "Is infant cereal enough for a 2 year old?",
"src": "Patient: Is this correct or cerelac is enough for her, please advice Hi, My 2 year old child is eating only cerelac -baby food. she is not at all taking rice ,fruits ... growth she needs normal food ....is this correct or cerelac is enough for her ..please advice .. Doctor: HiWelcome to the HCMI have gone through your question and understand your concerns. But for a 2 years old kid, it's not at all sufficient to sustain only on cerelac baby food. Food fads develop in this age and unless and until you start offering her all your staple diet, she won't nourish well on just baby foods.Offer your child the same healthy foods that you and the rest of your family enjoy. Give your child foods with different flavours and textures from all 4 food groups. The 4 food groups are:-Vegetables and fruit-Grain products-Milk and alternatives-Meat and alternativesMake sure that the foods you offer are prepared with little or no added salt or sugar. Include healthy foods that are higher in fat like salmon, avocado, cheese, and nut butters.Hopefully this will help you. I would be happy to help you with any further questions.Take care"
},
{
"id": 196597,
"tgt": "Having noticed the urge to urinate frequently",
"src": "Patient: hello , i am 26 year old boy and i am masturbating for past 10 years .earlier it was daily but know 4 to 5 times a month.the problem is that for past few months after urination some drops comes out frequently.and when i drink water even a half glass i need to go to toilet immediately.please help me to cure this problem. Doctor: Hi,You might be suffering from urine urgency is a common feature of diabetes mellitus, prostatitis and bladder infection. I would suggest you to go for routine and culture examination of urine in order to rule out any gross Pathology. Hope it helps. If you have any other question please do not hesitate to contact us.Regards,Dr. Atishay Bukharia"
},
{
"id": 32985,
"tgt": "What causes vomiting and shortness of breath during medication for an infection?",
"src": "Patient: iv had surgery on my ankle to have metal work removed and a few days after surgery its became infected ,swollen, my hole foot is red and feels under pressure iv been to a & e they said it was boarder line infected and sent me on my way .iv been put on antibiotics and im now vomiting i hav stomach pains and dissiness also shortness of breath can u shed some light on this please Doctor: Hello You are passing through a trauma , initially you operated for metal work removal , got infection , took antibiotics . These all developed hyperacidity or gastritis and that is main reason of vomiting and stomach pain .Since you are also complaining of shortness of breath ,this may be due to hyper acidity or forceful vomits . You should take some proton pump inhibitors like pantoprazole-d ,once or twice in day ( although normal dose is once in a day ). Also take some antacid .If you got relief in infection stop antibiotics . You can use local antiseptic solution ,if infection is localized and minor as it will not produce any hyper acidity or gastritis.Still feel problem or vomiting then consult a physician and get his opinion.Good luck."
},
{
"id": 81828,
"tgt": "How to treat fibromatosis of the lung?",
"src": "Patient: A family member is experiencing fibro matosis of the lung. The left lung is collapsed. The next organ that could be affected it the heart. The patient is not strong enough for chemotherapy or surgery. Is there another therapy that could slow the growth of this substance? Doctor: Yes radiotherapy can be tried however chemotherapy will be the best thing to do. Speak to your oncologist for a better individualized treatment"
},
{
"id": 96734,
"tgt": "Suggest treatment for left thigh bone injury",
"src": "Patient: i am pooja kamath amboli. My son abhishek,5 years met with an accident in June this year injuring his left thigh bone. He had a plastered leg till Sept. after which his plaster was removed and is able to walk properly. However, during since plaster was from his abdomine till the left toe, he was bed riden and due to which he gained weight of around 10-12 kgs. Now, he is 36 kgs. My question to you is how to reduce his weight and maintain it too? Doctor: Welcome to hcm, he has been treated for thigh bone injury. And is recovering so his weight reduction has to be made like this Avoiding fatty items in foodAvoiding oily items and fried itemsIncreased intake of resins and dried grapes which are rich in cacimTaking fat free milk everydayTaking fruits and vegetablesAnaerobic exercises which can be done without straining the legAnd by sittingPhysiotherapy exercises. With proper guidance from a physiotherapist.Hope this explains.With regardsDr.Amarnath"
},
{
"id": 167147,
"tgt": "What causes diarrhea with yellow or green stool with stringy bits?",
"src": "Patient: Hi my daughter is nearly 7 months and has had dioreah for about a month now. First of all it was very wateryy with yellow stringy like bits, now it has turned dark green with the yellow bits. I have been to my doctor but she said she is fine, i took a sample of stool to the hospital and it came back negative for everything??? I dont know wot is wrong with her....she also wont sleep through the night, at the most 4 hrs. She used to sleep the whole night through? Im very worried because its been going on for so long and must be so uncomfortable for her. Doctor: It is not clear whether the baby is on breast milk or top feeds. We might be facing an issue of lastose intolerance. Please consult a paediatrician and take advixe regarding change of feeds."
},
{
"id": 205042,
"tgt": "How can bi-polar disorder be treated?",
"src": "Patient: Hello, I have been diagnosed with bi-polar type 2 and I might have border line personality disorder, I take prescribed medication. I m a 20 year old female. In the morning I take 1 valproic acid 250mg pill , 1 and 1/2 escitalopram 10mg pill, and 1 synthroid 0.05mg pill. In the evening I take 2 valproic acid 250mg pills. I stopped taking my medication a few days ago, maybe 3 days or 4, when I wasn t supposed to, but I started taking my medication tonight because I was having physical discomfort because of the withdrawal symptoms. I was wondering how long it will take to feel normal again (no more withdrawal symptoms, or symptoms or re-starting my medication)? Doctor: bipolar mood disorder is episodic mood disorder,with mood stabilizers like valproate and lithium it can be controlled."
},
{
"id": 16115,
"tgt": "Itchy rashes on the arm after childbirth. Is it hormonal? Need medication?",
"src": "Patient: hi, i am Renea. for 2 months after i gave birth to my son, i have developed a rash on my arms. Sometimes it itches on my legs and back. It seems to itch worse at night or when i get hot or alittle flustered. I am online looking for answers and all these people going to the dr. s about it and them not getting any answers. I even called my obgyn and she has never heard of it, which is hard to believe. I just want to know what is the cause of it, is it hormonal or what and how long does it last. Do i need to go to the dr for medication to get rid of it cause some people said medication did not work for them. I just want it to be over with it makes me irritable. Doctor: Hello, If you could please describe the precise lesions you see or how you feel them, estimate the size by comparing to a known object. Locations of lesions, clustering or not. The dermatosis you are having might not even be directly related to your pregnancy. But if they are pregnancy related, then you should allow sometime for hormonal balance to be established in your system and suppress the symptoms by either applying hydrocortisone 0.05% cream on the lesions or taking oral antihistamines. Another possibility is if the above treatment doesn't work, you might be having scabies ( strange right? but it is quite possible given the locations you have mentioned). If that is the case, you'll need to take appropriate treatment with anti-scabies lotions. Your daily skin lotion too, should be re-checked as it might be the cause and by stopping and switching your soap might help your situation. I will suggest you pay a visit to a dermatologist for close examination and proper management if above do not work. Not able to examine the rashes myself makes it difficult to give you exact management. Best wishes and regards"
},
{
"id": 167132,
"tgt": "Suggest remedy for brain fever",
"src": "Patient: my neighbour hood 1year baby went unconcious and no breath b4 a week then went took her to hospital and doctor said its bran fever....just b4 3 days only she got dischare and doctor said that she shouldnt get fixs or fever for 5 years but yesterday she got 100 deg fever....after consulting doctor too the fever is still there wat should we do to avoid fever and the first aid pease..... 1yr baby brain fever.doctor is confused on her wat is happened to her exactly Doctor: Brain fever in medical terms is called Meningitis. The usual course of treatment for Meningitis is IV antibiotics for at least 2 weeks. The persistent fever may be due to incorrect choice of antibiotic or insufficient duration of treatment. Please go for a follow up and continue the treatment."
},
{
"id": 159421,
"tgt": "Suffering with COPD, H-plyori, has profuse sweating, fatigue, gall bladder issues. Family history of stomach cancer. Suggestion?",
"src": "Patient: My Fiance is a 53 year old male with COPD/Emphasema, in the past two months he was diagnosed with H-pylori ,he had 2 months of chronic diarrhea .This was cleared up. He has had some gall bladder problems,having had nausea, dizziness reflux and have had a few episodes of vomiting but for the past month he has been very SOB on exertion and fatigue in the past few days has woken up to profuse sweating with no fever enough to change the sheets. His father Passed with stomach cancer about 10 years ago and I am beginning to be concerned . This is not your typical night sweats.could this be signs of cancer? Doctor: Hello! Thank you for the query. Night sweats can always indicate a cancer. However it does not has to be stomach cancer. As she has COPD I assume she smokes. Nausea and reflux indicates some digestive tract problems. And she is 53. Such age gives increased risk of colon cancer. He should have chest CT, abdominal CT (pancreas should be checked), upper GI endoscopy and colonoscopy performed. Full blood work, CEA, CA 19.9 CA 125 should be done as well. Hope this will help. Regards."
},
{
"id": 66327,
"tgt": "What causes yellow lumps underneath labia majora or between majora and clitoris?",
"src": "Patient: i have a small yellow lumps or bumps underneath the labia majora or in between the labia majora and clitoris hood. i didnt feel any pain. im just worried. please help me to these matter. im afraid of that. thank you. no need. i thought it is free. thank u. Doctor: Hi, thanks for writing to HCM.Well, If I were your treating Doctor for this case of multiple yellowish lump lumps underneath labia majora or between majora and clitoris , I would come up with three possibilities, these include: 1.\u00a0\u00a0\u00a0\u00a0\u00a0prominent glands o the skin/mucous membrane2.\u00a0\u00a0\u00a0\u00a0\u00a0The second possibility is of benign and simple conditions like skin papilloma or fibroepithelial polyps!\u00a0\u00a0\u00a0\u00a0\u00a03.\u00a0\u00a0\u00a0\u00a0\u00a0The last possibility is of some cysts like retention-type cysts of skin adenexa or inclusion cysts ! Nothing to worry about these also!I suggest you to go for a physical examination by a gynecologist for a definitive opinion to relieve your concerns!Hope this answers your question. If you have additional questions or follow up questions then please do not hesitate in writing to us. I will be happy to answer your questions. Wishing you good health."
},
{
"id": 2031,
"tgt": "How to get pregnant?",
"src": "Patient: hi, im a 21 year old female and happily married. We have been TTC for over 3 years now. My GP says my progesterone levels are border line, but i now cant do anything about it until im 23, then i can be refferred for IVF...is there nothing i can do until then? We are both desperate for a baby, and our parents are now into their 60's with health issues thereselves and we really would like them to meet their grandchild Doctor: Hi I don't think you should think about IVF now. Progesterone levels can be increased with help of medicines and injections. You should get yourself evaluated first. Do a thyroid profile and prolactin levels and a ultrasound for your uterus and ovaries. Also do a semen analysis of your husband. If everything is fine you can try medicines for ovulation and also some medicines to support implantation."
},
{
"id": 107377,
"tgt": "Suggest treatment for severe pain in lower back and hips",
"src": "Patient: I just finished 10 days of levaquin for a sever sinus and lung infection. I have had a lot of pain in my low back and both hips that started about 7 days after starting it. can that be caused by the medicine aqnd if so, what is the treatment. I am having trouble walkind from the pain. I stayed out of work for last 3 days and rested but still have a lot of pain. Doctor: Hi i am Dr Ahmed Aly thanks for using healthcaremagic site ,I had gone through your question and understand your concerns .. Actually rarely levaquin can cause tendinitis and pain in knees and hips but that is very rare and happens only on chronic using or long duration of administration . In my opinion some times severe lung infection causes bone aches that may subside simultaneously with bed rest and relaxation . For now you may use advil tab when needed , hot massages with topical gels , B12 vitamin supplementation , muscle relaxants , spinal straightening exercises , physiotherapy will be effective for most of cases . If pain persists i recommend a visit to your physician and MRI to exclude any spinal injuries or disc herniation for proper evaluation and management . Please click and consider a 5 star rating with some positive feedback if the information was helpful. Wish you good health,Any further clarifications feel free to ask."
},
{
"id": 213588,
"tgt": "Fits disease, encorate medicine, loss of appetite, can't communicate properly, likes to play with slippers or shoes",
"src": "Patient: my sister s daughter is now 3 years when she was a year old she caught fits disease dr prescribed encoret medicine along with other medicines ,the course was for 2 years . now she is in her 3 rd year she is thin and eats any food with no interest and only drinks milk she does not communicate properly she speaks some words only but not communicate fluently. and she cries when she ask for any thing for example when she needs water or milk etc. so long it will take her to cure completely, and how does she will get of encoret s side effects ? now she has fits at all, but she like to play with chappals and shoes while she has many toys, and she also lick and chews chappals and any rough things . so please give your valuable suggestions and prescribe some medicines to make healthy, her liver has become sluggish . Doctor: Hello. Thanks for writing to us. The time taken for her recovery depends on the extent of damage her brain has suffered. All her symptoms are not due to the side effect of the medicines. Playing with shoes is a common pastime for children and is not abnormal. A clinical assessment is necessary before suggesting anything in such cases. I hope this information has been both informative and helpful for you. Regards, Dr. Praveen Tayal drtayal72@gmail.com"
},
{
"id": 104434,
"tgt": "Blocked nose during weather changes. Prescribed xylomist nasal drops, lorfast anti allergic tablets. Cure?",
"src": "Patient: My nose severally blocks when weather changes..my doctor prescribe xylomist nasal drop with lorfast anti alergic tablets,with romilust l10 .But the problem is after taking those medicine i improve for few days ,after that same thing happens.And to continue my work i have to take xylomist drop in a day till now for some relief from nose blocking.now what will i do? Doctor: You are having chronic sinusitis You can see Ent specalist After Getting Xray pns Water View You need fUll treatment of sinuses and allergies For temporary measure You can take Fexofenadine !20 mg once a day morning and singular once a day evening You can continue off and on till you go ahead for treatment Dont use xylomist too much Can apply neosporin H eye ointment in nose twice a day Can put Sea water Drops 2 drops in each nostril at night"
},
{
"id": 157241,
"tgt": "Can anti-depressants or other medicines cause deterioration?",
"src": "Patient: My aunt has been on various anti-depressants for over 30 years. She has a history of cancer, has many digestive problems, and is 95 y/o. She has become increasingly agitated, and lately paranoid about friends stealing from her. Can anti-depressants or other meds cause deterioration or is it more likely just her age/Alzheimer s symptoms? Doctor: Thanks for query .all though antidepressants in fact all mood changing drugs have side effects and her age also has adding effects on it.it is better to take a regular follow up with psychiatrist as you mention she is being paranoid."
},
{
"id": 23842,
"tgt": "What causes palpitation and breathlessness?",
"src": "Patient: Hi Doc, my wife having WPW in the past and she gone through some procedure. Few months after, she is still continue having extra hearth pulputation and difficult in breathing. She went for a 24hrs ECG monitoring and found that she is having high hearth pulputation upto 120 per min in rest situation. And she went to untra-sound to check the condition of the hearth, but the Doc said that the heart is strong and normal, and the high pulputation is normal as well.... If everthing is normal, why the extra pulputation is there and she continue suffer short in breathing few times a day now? Please help. Thank you. Doctor: probably your wife has undergone Electrophysiological study and ablation of accessory pathway in the past for WPW syndrome. now she is again having palpitation which may occur in recurrence of the disease.breathlessness can also occur during palpitation episodes. So again Electrophysiological study and if required radio frequency ablation may have to be done."
},
{
"id": 224712,
"tgt": "Does HCG diet affect Implanon birth control?",
"src": "Patient: I have had the implanon for a yr & just started the hcg diet. I started some spotting 2night & didn't know if I should be worried? I have not had a period while on the implanon & my husband & I are not ready 2 have kids. Should I be worried? Can the hcg diet nul my implanon birth control? Doctor: first get pregnancy test.consult your doctor to go for-change in family planing method like copper t. Even your husband can use condom. hcg diet has nothing to do with pregnancy."
},
{
"id": 85308,
"tgt": "Is exhaustion and nausea a side effect of medication?",
"src": "Patient: Hi, I started taking shelcal 500 since last week under doctor s prescription (since i was having severe leg pain for past one month). From the time im taking shelcal (alt days) im having trouble sleeping, having nightmares if i sleep in the afternoon, feeling tired, exhausted, and sever nausea and uneasiness in my stomach. Can this be triggered by shelcal since nothing else has changed in past 1 week. Doctor: Hi, It can't be caused by Shelcal, it can be due to allergic reaction of your body to that drug. Change in diet pattern can help which contains more calcium.Hope I have answered your query. Let me know if I can assist you further. Regards, Dr. S. R. Raveendran, Sexologist"
},
{
"id": 85433,
"tgt": "What are the side effects of i-pill?",
"src": "Patient: i am 18 year old girl.. it was my 6th day of period i had sex with my boyfriend for the first time n bad luck it was unprotected.. i took an i pill.. now i m worried about its side effects and its effects on my menstrual cycle.. will i gain a lot weight coz of it?? Doctor: Hi, Side effects of the morning-after pill, which typically last only a few days, might include: - Nausea or vomiting - Dizziness - Fatigue - Headache - Breast tenderness - Bleeding between periods or heavier menstrual bleeding - Lower abdominal pain or cramps Hope I have answered your query. Let me know if I can assist you further. Regards, Dr. Ajeet Singh, General & Family Physician"
},
{
"id": 215249,
"tgt": "Suggest ways to get relief from reflex sympathetic dystrophy syndrome in leg",
"src": "Patient: Hi my name is Craig and I have R.S.D have had this since 2002 through a fall to my left leg have been on a lot of meds and now weened my self of them and only take small amount of pain killers now .Have just had in last month Ketamine infusion over seven days in hospital this did knock me about for about a week and a half . and pain leaves have go ten down to about 50 to 60 percent .is there any thing els I can be doing to help this further .Thanks Craig Doctor: Hello, Well, if the ketamine worked, that is better than usual. Sometimes knocking out the sympathetic nervous system either with drugs or with surgical procedures can be helpful Or with botox. I cannot say in your particular case, but generally, there are several therapies for it. Hope I have answered your query. Let me know if I can assist you further. Take care Regards, Dr Matt Wachsman, Addiction Medicine Specialist"
},
{
"id": 196470,
"tgt": "Suggest treatment for gynaecomastia and abdominal fat",
"src": "Patient: I am the son of the perennial sports i'm thin but i have lots of fat at the Abdomen and flank and some part the other my body and they are smooth like woman and muscle maker my body is very bad what do i do pleas help me?by the way i have gynocomastia ,gynexin can help me? Doctor: Hello,Thanks for trusting Healthcaremagic!i have gone through your query and I can understand your concerns.It could be due to food supplements.Consult a general surgeon and get evaluated. Sometimes surgical removal of the involved breast may be needed.Thanks"
},
{
"id": 58030,
"tgt": "Have blood tests report. Can any online doctor help?",
"src": "Patient: 4 months since an aortic valve replacement along with CABGx5. Recent blood tests show ALK of 139, ALT of 65, and AST of 43. Could any of these high readings be a result of the past surgery? My family doctor wants to have an ultra sound done of my liver and abdomen. Doctor: Hi,I read your query and would like to say that yes this readings may be related with the past intervention. But to determine this you should perform an abdominal ultrasound, HBsAg ant-HCV (to rule out any hepatic problem) and a heart echo (to rule out heart failure).Hope I answered to your query.Dr. Klerida"
},
{
"id": 222930,
"tgt": "Does cramping without spotting after taking follistum tablet mean pregnancy?",
"src": "Patient: i took the follistum shot at the begining of feb. i was to ovulate feb 14 and take a test on feb. 28 i wanted to wait to take it til i was pretty sure i was pregnant. i have had some slight cramping going on but no bleeding or spotting at all. am i pregnant? Doctor: No cramping is not a sign of pregnancy. Wait for few days if no periods go for upt tests. If negative still wait for few days before consulting your gynaecologist."
},
{
"id": 14503,
"tgt": "What causes itchy rash around the dressing after removal of picc line?",
"src": "Patient: I had a picc line removed last Saturday and now have a rash around the dressing sight that itches and will not go away. I also am starting to get a knot under the picc line insertion spot. I have an appt. this afternoon to see my doctor. Should I be concerned? Doctor: Hello. Thank you for writing to us at healthcaremagicThe rash that you notice at the dressing site is most likely due to allergic contact dermatitis either to the topical antiseptic/ antibacterial that may have been part of the dressing Or it could be due to allergy to adhesive used to hold the dressing at the site.If I was the treating doctor I would suggest you to use a moderately potent topical steroid e.g fluticasone propionate 0.05% cream, twice daily, at the site of rash.Steroids are antiinflammatory, immunosuppressive in action and would take care of the allergic rash.Regarding the knot, it is most likely due to thrombophlebitis of the vein thorough which the picc line was accessed.The knot would resolve on its own in a few days. An Oral antiinflammatory e.g ibuprofen would help decrease the pain, swelling.Topically you may use a heparin sodium gel, which would act as an antithrombotic.Regards"
},
{
"id": 138390,
"tgt": "What causes pain and swelling in the knee?",
"src": "Patient: Actually my husband has had 2 artho knee surgeries and 3 full replacements...#1-parts too big, #2 not enough parts in, made it too loose, #3 all hardware and stability good...but he still has severe pain and swelling on the top outer side of his knee with skin discoloration... all has been done within 3 years..last surgery was April 15th....the ortho Dr says everything is fine and he just has to push through the pain.... Doctor: There should be some reason for the pain. But such things ate only understood once we see and examine the patient. I would recommend u to seek an opinion from other ortho."
},
{
"id": 141630,
"tgt": "Could the swelling in the spine around the surgical area be causing pain in the hip?",
"src": "Patient: I had decompression surgery last October, and even though I was told I was told I would be in and out of hospital in three days, I ended up not being able to stand right after surgery. I was sent to physio therapy for 5 days, and was able to go home with a walker. It took six weeks before I was free to stand and walk on my own. Two months later, after returning from the bathroom,I got a horrible pain in the sacrum area. The surgery was done on L-4,L-5 area, and they felt after a CT scan and an MRI that I had a pinched muscle in the hip area. That was just before Christmas, and I ended up once again in Rehab for 5 days. The pain has gotten better, but now I am getting a burning in my left hip radiating down into my knee. Could I still have swelling along the spine in the surgical area that could be causing this. They did see a disc that had fluid around it on my MRI. Doctor: Hello and Welcome to \u2018Ask A Doctor\u2019 service. I have reviewed your query and here is my advice. I feel that the prolapsed disc is causing all these symptoms by compressing the exiting nerve root.All you need is a fresh MRI lumbosacral spine and look for the status of prolapsed disc or any osteophyte or any other bony displacement (listhesis). MRI will decide further course of action whether by medicines or operative. Meanwhile take Tab Voveran twice and tab Gabapentin twice a day. Hope I have answered your query. Let me know if I can assist you further."
},
{
"id": 117345,
"tgt": "Is blood transfusion a solution for low hemoglobin?",
"src": "Patient: I (male) have been diagnoised with low hemoglobin (9.0) due to a recent loss of blood (ulcer). I am feeling really tired and was told by my family Doctor that it would be a slow process to regaining my normal degree (12.0 -14.0) Can I request a blood tranfusion? Doctor: Hi, dear. I have gone through your question. I can understand your concern. But till possible you should avoid blood transfusion. Hb level 9 is low but not an indication for blood transfusion. You should take oral or injectable iron. Because there are many risks of blood transfusion. So consult your doctor and take treatment accordingly. Hope I have answered your question, if you have doubt then I will be happy to answer. Thanks for using health care magic. wish you a very good health."
},
{
"id": 25134,
"tgt": "Can rapid heartbeat be due to stress?",
"src": "Patient: My heart beat has been jumping or doing a hicup ever two beats for the last couple of days off and on went to the emergency room and they said id be fine it was from stress, i made it quite aware to them that i had wpw syndrome and had surgery for it and its been ten years since i had problems like this. I feel like this is dangerous but they said id be fine unfortunately my heart leveled out at the top or bottome of the ekg every two heart beats instead of making a point after every beat. This is very uncomfortable and i dont have inusurance no family doctor and i feel like this is unhealthy. Doctor: Thanks for your question on Healthcare Magic. I can understand your concern. Yes, stress can cause rapid heart rate. But since you had WPW (wolf Parkinson while) syndrome in the past, better to rule out arrhythmia (rhythm disturbances in heart) for your symptoms. So get done ecg, 2d echo and Holter monitoring (24 hours continuous recording of ecg). If all these are normal then no need to worry for major heart related diseases. Your rapid heart rate is mostly due to stress. So consult psychiatrist and get done counselling sessions. Try to identify stressor in your life and start working on it's solution. You may need anxiolytic drugs too. Don't worry, you will be alright. Hope I have solved your query. I will be happy to help you further. Wish you good health. Thanks."
},
{
"id": 124696,
"tgt": "What causes swollen cubital lymph node in elbow?",
"src": "Patient: I am a 28 yr old female and have what I believe is a swollen cubital lymph node in the inside of my elbow where it creases. It appeared 2 days ago and seems to be getting larger. I have not injured it and have had muscle aches and pains as well as nausea and diarrea that lasted for about a week. Doctor: Hello, Consult a general surgeon and plan for an ultrasound scan. You can go for a fine needle biopsy in case of diagnostic confusion. Hope I have answered your query. Let me know if I can assist you further. Regards, Dr. Shinas Hussain, General & Family Physician"
},
{
"id": 140644,
"tgt": "Are seizures hereditary?",
"src": "Patient: Dear Dr, my son have Jacksonian left sided seizures since he was 19 months old. He is 3.5 yrs old now. He had seizure attack 4 times since then. He is on Keppra n Trioptal medication... since last year may he is fine with the medications. My hubby who is 34yr old now, had 1 attack of seizure in last november. My question is is seizures hereditary? If we have another child, will that child also tends to get the seizure? If so means how much percentage of chances are there the other child also getting the seizures? Please waiting for the reply... Doctor: Hi, Seizures are hereditary in about 50% of cases. Another child also would have a higher chance of getting seizures. The exact percentage can not be predicted. Hope I have answered your query. Let me know if I can assist you further. Regards, Dr. Sudhir Kumar, Neurologist"
},
{
"id": 173369,
"tgt": "Suggest treatment for itchy rashes all over the body of a child",
"src": "Patient: our otherwise healthy 11 year old daughter has had a severe rash from head to toe covered with welts and complaints of profuse itching. This has been going on almost continuously for over two months and we are just now finally coming to the end of our wait for appt with an allergist-appt still 5 days away. Before this, she had NKDA NKA. She has been treated around 6 times now at intervals when her medication regimens would come to an end and/or at height of symptoms during this nightmarish 2-3 month time period. None of the anti-histamines, steroids, etc seem to work completely with significant relief of her symptoms (welts may fade somewhat but do not go away and itching reduced only minimally) and any small amount of relief of symptoms does not last long. This is really taking a toll on the entire family (and oh, yes, we have changed detergents, soaps, etc, etc, etc made lists trying to track possible causes) . . . please HELP!!!! Doctor: Hi, If I were your treating Doctor for this case of severe pain under the left rib cage, I would come up with three possibilities, these include:1.Hypoallergenic diet,elimination of allergens.Diagnosis can be confirmed by allergenic food tests.2.Absorption of allergens by giving Charcoal 1 tablet trice a day for 10 days.3. Stool culture for flora to exclude dysbiosis. For normal flora you need to give Darolac 1 capsule twice a day for 1 month, enzyme's syrup during meal for 2 months, albendazole-for elimination of helminths. In addition try Allegra-the strongest antihistamines 1 tablet for night for 14 days.Hope this answers your question. If you have additional questions or follow up questions then please do not hesitate in writing to us. I will be happy to answer your questions. Wishing your child good health."
},
{
"id": 190191,
"tgt": "Tooth and jaw pain, swelling, tenderness. Artificial tooth fell out while eating. Pain management?",
"src": "Patient: I had a porcelain tooth and it recently fell out while I was eating at a restaurant. I placed it down and I lost it. It has been almost 2 weeks since I have not had it. I am waiting for my insurance be effective, which is on the 1st of december. This past sunday I had a really bad tooth ache. Now it feels like the pain has moved to my jaw. My jaw looks swollen and its really tender! what can I do??? Doctor: Hello...why was the ceramic crown placed?was it a decayed tooth?was rootcanal performed on that tooth? these questions needs to be answered before i could answer your question."
},
{
"id": 139626,
"tgt": "Suggest medication to bring back sensation in body and brain coordination",
"src": "Patient: hi i am 26 years old 175 cms tall, weight-72 ... i was under depression in 2009 when i was 24 and was under medication... my psychiatrist had prescribed lithosun and oxetol to calm my brain ...that has made my brain completely inactive now i have lost sensation completly in my body and feel that there is no coordination in my brain . its been almost two years now .... please advice. Doctor: Hi, First of all, you should measure the level of lithosun in the blood every 3 months in order to adjust the dose of the drug during the treatment. The normal level should be 0.4 to 0.8 mEq/L for maintenance therapy and 1 to 1.5 mEq/L in the start of the treatment. Second, you should do thyroid function tests every 3 months, because the lithosun interfere with the Iodine re-absorption and use by the thyroid gland. Third, you should ask your doctor to adjust the dose of the drugs. Finally, the treatment for depression needs the medical treatment and psychological treatment in order to achieve good result. Hope I have answered your query. Let me know if I can assist you further. Regards, Dr. Mustafa, Neurologist"
},
{
"id": 157189,
"tgt": "What is your opinion about taking Tamoxifen after having left breast DCIS and discontinuing Aromasin due to neuropathy in both hands?",
"src": "Patient: Age 59 dcis left breast 5 /2012 clear margins nodes clear, had tah 9 /13 for abnormal pap due to hpv' plus felt surgery would be beneficial for overall health . Took aromasin for about 6 months , early signs of neuropathy in both hands, was discontinued, have to follow up with oncologist to see if tamoxifen would be necessary now. Have read differing views not sure what will be decided...thought? Doctor: according to many studies, taking hormone therapy will help in not coming back of the cancer(Recurrence) of the breast. The main side effect of the tamoxifen is on the uterus.As you have undergone already TAH, and also the effects of aromatic drugs, it is useful to continue on taking tamoxifen. But discussion with your Oncologist and come to a decision and follow it."
},
{
"id": 207193,
"tgt": "Suggest remedy for mental health problem",
"src": "Patient: I have really awful panic attacks that come and go. They make me feel scared and make it hard for me to breathe. I even break out into sweats and feel as if I could cry when they are finished. They seem to happen more often when I am in public or around large groups of people like resturaunts, malls, etc. What kind of medication can help me? UGGGGG. Im so tired of feeling like this!!!!!!!! Doctor: Hi,Thanks for writing to healthcaremagic.Keep one thing in your mind that no matter how much intense panic attack is , you never gonna die due to it.Panic attacks can occur even to normal persons but when there are multiple attacks with anticipatory anxiety and when it interferes with your daily routine ,it's called as panic disorder.There are two ways for treatment:pharmacotherapy and psychotherapy.Neurotransmitter changes occur in panic disorder specially serotonin and norepinephrine. SSRIs(Serotonin Selective reuptake Inhibitor) are drug of choice and benzodiazapine use for short term to ameliorate panic attacks.Behaviour therapy also play important part specifically Cognitive behaviour Therapy(CBT).Hope I have answered your query,"
},
{
"id": 19475,
"tgt": "Why is an echocardiogram to be repeated for mild heart enlargement?",
"src": "Patient: Hello doctor since from last one month iam suffering with chest pain and i have done master health checkup it was all normal but in x-ray they have showed tht mild heart enlargement is there nd suggested me to go for 2decho test wht would be the reason for this? can u please give some information. Doctor: Hi,ECHO provides best results in terms of heart enlargement. Please do it if your chest xray shows enlarged heart. Regards."
},
{
"id": 78674,
"tgt": "What is the reason of nodules seen on chest x-ray?",
"src": "Patient: Had a quick question. Had a chest X-ray which showed three nodules. Then I had a ct scan which did not show the nodules. I know a ct scan can show more, however I'm not sure which one I believe now. I am breast cancer survivor, so I get chest X-rays every year and have never had a problem, let alone find three nodules. Should I repeat these tests, or get a bone scan to be on the safe side, or do you think everything is fine? Doctor: Thanks for your question on Health Care Magic. I can understand your concern.Since you are cancer survivor any lung nodule is a cause of concern. CT is definitely more reliable then x ray. It would be advisable to consult your oncologist to rule out any secondaries. Don't worry, you will be alright. Hope I have solved your query. Wish you good health. Thanks."
},
{
"id": 50019,
"tgt": "Sore right side of lower back above kidney. Taking prozac for a year. Strain? Any ideas?",
"src": "Patient: Hi their i am a 41 year old mum of one and I have had a sore right side of lower back above kidney area. It has been like this for about five weeks and although it isn't getting any worse, it doesn't appear to be improving either. Sometimes a hot water bottle helps me to get comfortable when lying in bed. I'm not sure if it is some sort of a strain? Any advice would be greatly appreciated. I forgot to mention that I have been taking 20 mg of prozac a day for the last year. RegardsMelanie Doctor: Hi, thanks for using healthcare magicIt is possible that it is a muscle strain particularly if you find it worsens with movement in a particular direction.In addition to the application of the heat pad, you should consider the use of an anti inflammatory pain killer such as aspirin, ibuprofen, diclofenac, naproxen.These medications should be used on a full stomach.You should also reduce any movements or lifting activities that aggravate the area to allow it to heal.I hope this helps"
},
{
"id": 207168,
"tgt": "Suggest treatment for avoidant personality disorder",
"src": "Patient: Hi, my name is Ansar, I am from Pakistan, I have avoidant personality disorder, I am taking, Revalp, Depin, nimopro, notens and redeem medicine for almost 8 months, I am feeling better and my confidence level is also better, I would like to know that how long it will take time to become a normal happy and full confident person, I am professional photographer and wants to be normal in people crowd. I get nervous and feel eyes on me in crowd. Doctor: Hi Ansar,I suppose the diagnosis of avoidant personality has been made by a psychiatrist. Your symptoms have been improving for last 8 months. I will recommend you to take them until you feel the improvement line is on the raise. Once you feel there is no further improvement or feeling completely OK, maintain it at the same dose for 4-6 more months. Then start gradually decreasing the does by 5mg every fortnightly. If you doesn't feel that you are again getting the symptoms back then you can safely stop the medicines.I would recommend you to go for cognitive behaviour therapy sessions, it will help you to learn skills so that the improvement persists even after you stop taking medicines"
},
{
"id": 29167,
"tgt": "Is Combiflam an effective alternative for Dolo 650?",
"src": "Patient: I m running with high fever since 29th September, 2017.Average temperature reading 101 - 102.doctor said it s viral and will continue for 4 days. So he prescribed dolo 650.After having dolo 650 temperature come down at 100 to 101.Dose effect last till 5-6 hrs only.He also said this viral fever will last for 4 days. If 4th day nothing changes then he will do my blood test so that he can confirm that whtr it s dengue or not.Doctor can I switch my medicine from dolo650 to COMBLIFAME just to see the result With regards Omi Doctor: hi dear.....you can use combiflam as alternative to dolo 650 safely ...both of them are analgesic and antipyretic..both of them reduce the fever ...but take care if you have history of bronchial asthma or allergy to ibubrofen as combiflam contain ibubrofen with paracetamol...dolo 650 contain paracetamol only...ibubrofen contraindicated if you have history of bronchial asthma.......if you havenot asthma or allergy...you can use combiflam safely.....wish you healthy life....thanks for contacting us"
},
{
"id": 79894,
"tgt": "Suggest treatment for bilateral apical scarring in the lungs",
"src": "Patient: I had a CT Scan, and have two questions: what is bilateral apical scarring in the lungs and what does is come from also, it showed atherosclerotic coronary artery calcification Is that something to worry about, how can I make it get better what causes it (other than age, I am 76) Doctor: HelloFindings suggests scarring(fibrosis) in apical region of both lungs.It may be due to past healed infection.Fibrosis and calcification indicate healed past infection.However other causes may be occupation related etc.It is not a serious finding and patient usually doesn't require any treatment for it.It is a benign finding.Atherosclerotic calcification is also age related finding.It is more seen in hypertensive,diabetic patient.Get well soon.Take CareDr.Indu Bhushan"
},
{
"id": 32079,
"tgt": "Why do scratching causes bleeding in the head?",
"src": "Patient: when i scratch my head, i get white stuff on my head, and when i scratch a little TOO hard, i get blood under my nails.. why is this happening to me?? and i don't know why, but i'm addicted to scratching my hair ! but when i wash my hair i don't dandruff and as soon as i touch it, they appear ! Doctor: Hi,From history it seems that there might be having some scalp skin infection like eczematous dermatitis or dandruff infection giving rise itching and soreness.Consult dermatologist and get examined.After proper diagnosis plan of treatment can be decided.O)k and take care."
},
{
"id": 94533,
"tgt": "Got operated for tb node in chest. Feeling slight pain in abdomen. Chances of mittelschmerz?",
"src": "Patient: Hi, I am Sarita from Mumbai, In the month of March 2011 I had a tb node in my chest & got operated. I was continuing my medications for 2 months & left . I started it again & it almost 6 months completed till now. Now I am feeling a slight pain in my right & left lower abdomen . would this be a mittelschmerz or stomach tb. Kindly advise. Doctor: Hi Indirakonar, As you told that you have been operated for the TB node in the chest and after taking medication for 2 months you left it and again completed the course for 6 months. From this I understand that this is TB of lymphe nodes and not the lung and you have to surgically remove the node also. It is common that TB medication for TB of other part of the body apart from the lung is for long duration and should be stopped only after consultation with your doctor only. Now the pain in the abdomen can be because of recurrence of this or because of some other thing. I would suggest you to consult your doctor to make the confirmation and have the investigations done and keep monitoring yourself. Thanks and take care"
},
{
"id": 23782,
"tgt": "Is it safe to use augmentin for hypertension?",
"src": "Patient: I have been suffering from hypertension for over 10years and have been advised nebicard 10 and 150 ecosprin at night and covance D in the morning. was doing quite ok till about 5days ago when I had tooth extraction and had to be given quite a few local anesthetic jabs as it was a difficult extraction. am on augmentin 625 thrice a day and also combiflam twice a day. my BP shot up and was at 180/110 for a couple days and didnt come down ispite of my taking extra nebicard tabs. then my doc prescribed snumilo 5 which I have been on for last 3 days. now my BP stays at 140/90 most times but goes up to 160/100 when its nearing tab time. what do i do? a little worried. I am 55years old. thank you. Doctor: Hey,The rise in bp after any intervention including dental extraction is a normal body response to stress condition. Taking nebicard and snumlo must be enough for controlling the bp.The use of augmentin ( antibiotic) alongwith generally do not rise your bp. So yoy can safely use augmentin for the duration prescribed by your dentist.If the bp remains high i.e. >150/90 mm Hg, you can change nebicard to Nebicard - H in the morning which consist one additional drug for bp control. Rest dont worry the bp will get controlled. You also have to decrease the salt intake in diet for the optimal bp control.ThanksDr Sameer"
},
{
"id": 135991,
"tgt": "What causes low cholesterol levels during arthritis?",
"src": "Patient: I was recently screened for RA and the result came back less than 20. I know I have osteoarthritis, but I also ache throughout the day, not just in the morning. Also, I have lost 42 pounds in the past year...my cholesterol level was going down, but even without regaining any weight, my cholesterol level is going back up. Any ideas? Doctor: hicholesterol level rise may not be related to osteoarthritis, it depeds more on nutritional factors and liver metabolism and medicines to control levels are there including dietary measures. For arthritic pains, physical therapy and pain medicines may be sought from doctorsbest wishes"
},
{
"id": 18519,
"tgt": "Does high BP cause headache?",
"src": "Patient: My boss is 1401/97. It has been as high as 172/135 today. Have dr appt in 4 days. Could this b causing the horrible headache Should I go to the er before dr appt. I called n they said to rest unless I feel dizzy I should b ok. I\u2019m a little worried. I am 58 yo white female w COPD n irregular heart rhythm unknown cause. No bp medication Doctor: Hello and Welcome to \u2018Ask A Doctor\u2019 service. I have reviewed your query and here is my advice. I would like to tell you that headache can cause high blood pressure in almost every individual but high blood pressure do not result in headaches. Considering hypertension and irregular heartbeat you need to get a Cardiac Evaluation done to prevent risk of stroke which is very common with atrial fibrillation. Kindly get an ECG, Echocardiography done and consult a Cardiologist for further treatment. I hope this information will answer your query. Kind Regards Dr. Bhanu Partap"
},
{
"id": 116085,
"tgt": "Is it dangerous to have low blood pressure?",
"src": "Patient: Hi, my blood pressure was just measured today and it was 90/40. I am an 18 yr old male that is 135 lbs and 6 ft tall. I get some exercise, but not enough. Is this low pressure something to be worried about? I sometimes get lightheaded when I stand up and my vision goes almost completely black for 1-2 seconds after standing. This doesn't happen often. Doctor: Hi,Thanks for asking.Based on your query, my opinion is as follows.1. Anything below 90/60 is worrisome.2. It can lead onto postural hypotension, risk of syncope and any fall could be dangerous.3. You are already having few symptoms, and blood pressure needs to be increased. Increased fluid intake with salts and venous stockings of lower limbs is beneficial. Mild corticosteroids can also increase blood pressure. Detailed evaluation necessary to assess the cause of low blood pressure. More details necessary for further opinion.Hope it helps.Any further queries, happy to help again."
},
{
"id": 150677,
"tgt": "Taking keppra for nocturnal seizures. Is there a permanent cure?",
"src": "Patient: Hi there, my bf has a nocturnal seizure for few years now currently taking keppra 2000mg a day..lately he is been seizure free..he just seen his neurologist last week and was hoping to lower his dose but according to his neaurologist wait in 6mos..just need a second a opinion if there s any cure for nocturnal seizure..appreciate your help...thank you Doctor: Hello Welcome to HCM He has nocturnal seizure for few years and is non Levetricetam 2000 mg per day. He is seizure free for some time. Usually in any seizure or epilepsy Neurologists advocate that medicines should be taken for at least 3 years after last episode of seizure. 3 year seizure free period is duration as per guidelines. As you visited his neurologist and you were told to wait for 6 months more. I will advise to wait and then discontinue medicines with advise of doctor only. Nocturnal seizures can be cured with medicines if taken properly. Also follow proper precautions of seizure as Proper nutrition fix the time of dinner no late night awakening avoiding bright lights avoiding loud music never miss a dose of drug, if dose missed then take the dose when ever remembered avoid undue stress both physical and mental. thanks Dr. Seikhoo Bishnoi Dr. Seikhoo Bishnoi"
},
{
"id": 66436,
"tgt": "What causes lumps inside upper lip and angular cheilitis?",
"src": "Patient: I ve recently developed Angular cheilitis and in the last 12 hours two swollen lumps inside my upper lip (near the corners of my mouth) have appeared. They re fairly large, and feel slightly numb. I don t know if the Angular cheilitis and lumps are linked but I m wondering what the lumps are exactly. It seems that they may be mucocele but those are normally on the lower lip and the lumps aren t moveable in my case. Please help! Doctor: angular chilosis is due to deficits of essential vitamins and lump in lower lips are mostly due to bite followed by mucous retention cyst. treatment of this condition is multivitamins and excion of cyst under local anaesthesia."
},
{
"id": 161652,
"tgt": "What causes red and inflamed testicles in infant?",
"src": "Patient: Hi, I was changing my babies nappy when i noticed that his testicles look very swollen and red. He is 7 months old, and is on amoxocillin at the moment for a chest infection. Do you know why his testicles are inflamed and what could be causing it? Should i be worried? Doctor: Hi, Redness in the testicular region alone should never be ignored.It doesn't appear like Amoxicillin induced rash.It may be torsion.Please take him to the nearest emergency room. Hope I have answered your query. Let me know if I can assist you further. Regards, Dr. Sumanth Amperayani, Pediatrician, Pulmonology"
},
{
"id": 88174,
"tgt": "What causes stomach pain during bowel movement?",
"src": "Patient: hello my name is pawan , my problem is i m suffering from the stomach pain during the time when i m going to toilet...this problem im facing from a year after an regular interval .. i also consult to an doctor who prescribe me normaxin tab..my ultrsound report is also fine. Doctor: Hi,From history it seems that you might be having chronic GI infection giving this problem.Go for one course of Ofloxacilin, tinidazole combination medicine for 5 days.Avoid junk food.Take plenty of water.Ok and take care."
},
{
"id": 96329,
"tgt": "29 years old has internal bleeding inside large intestine. what is wrong inside me ?",
"src": "Patient: hello my name is nasir I am 29 years old I am student and came 2 months ago from pakistan and recently shifted from peckham to sudsbury london my problem is that I am not registered with any gp in my area and I got a serious problem. from two weeks I am gone down with loss of blood in my stool. when ever i use toilet i can see all the flush turned red. There is some kind of internal bleeding inside my large intestine it is also a little painfull for me to attend the toilet and i am also very shy person i cant tell a doctor to see my. to find whats wrong inside me. please please help me some how i will pray for ur kindness and never ever forget your kindness. Doctor: It seems that you are having some local problem in that area.Either it could be fissure means a cut in the skin on lower side,or piles means swollen veins which bleeds on friction means passing stool.First of all stop taking spicy food and heavy foods like bread,tandoori roti etc.Take some light laxative mostly this will be available over the counter.If this does not address your problem and bleeding continuous you have to see a doctor for examination and further treatment."
},
{
"id": 217249,
"tgt": "Suggest remedy pain in left arm",
"src": "Patient: My left arm has this strangle pain that comes and goes. Sometimes it hurts a lot sometimes it just feels like my arm is asleep. I thought maybe it s cause I was sleeping on it so I stopped and it still hurts. It started a week ago when I stopped smoking meth. Doctor: Here it can be due to withdrawal symptoms of smoking. But in that case it should be more multiple part of body. For this kind of patient I first check from where the pain starts when it is starting from neck and radiating to the arm usually I have seen there may be possibility of cervical can disc issue or cervical curve loss too. In this case you need to check with mri of cervical spine for proper evaluation and then visit ortho as well physio for further treatment with medication and physical therapy. You can also use hot and cold pack frequently at your symptomatic area like neck and arm to have more relief. Take care."
},
{
"id": 208766,
"tgt": "Suggest remedy to quit masturbating",
"src": "Patient: my age is 19, height is 160 above, weight 70.. sir i started hand job before 2 years . i wanna leave it. cause i wanna study with full concentration. but it effects on sitting ability. after do it i feel sleepy and weak due to which i do not able to exercise better like before, which results in increasing my weight cause i do not burn fat in me.also now excessive fat is resulting in breasty shape of my chest. sir please tell me some ideas and ways to leave it. Doctor: Hello and Welcome to \u2018Ask A Doctor\u2019 service.I have reviewed your query and here is my advice.I suggest you not to worry much. Masturbation is not a sin or any negative behavior. It does not create health problem if done moderately. You can masturbate thrice or upto 7 times in a week at your age.If you feel anxious about it, that is the reason for your troubles. Masturbation cannot hinder your concentration. You have the capacity to take all these together and be energetic and happy. Hope I have answered your query. Let me know if I can assist you further.Regards,Dr. K. V. Anand"
},
{
"id": 150416,
"tgt": "Had a pilonidal cyst, infected with harden tissue, large sore. What to do?",
"src": "Patient: I had a pilonidal cyst last week. I had it lanced in the ER on Thursday. They said the cyst was surprisingly very small - less than a pea. However it was fairly infected and had large harden tissue surrounding it on both side of the navel cleft. Now the lance wound is basically healed but the skin on the harden tissue has open sores. I am taking my antibiotic, but the discomfort from this hardened tissue and now the large sore are horrible. What can I do? Doctor: Hi, Thank you for posting your query. As of now, continue the antibiotics. You may also take analgesics such as ibuprofen for pain relief. If the symptoms are persisting, then you should re-visit your surgeon, you may need to perform a minor surgery to remove the sinus. Please get back if you require any additional information. Best wishes, Dr Sudhir Kumar MD (Internal Medicine), DM (Neurology) Senior Consultant Neurologist Apollo Hospitals, Hyderabad, My personal URL on this website: http://bit.ly/Dr-Sudhir-kumar My email: drsudhirkumar@yahoo.com"
},
{
"id": 166153,
"tgt": "What does blood in urine of a child indicate?",
"src": "Patient: Hi, may I answer your health queries right now ? type your query Please here... I have a year old son who has had a cold for more than a week. 2 days ago, he had a really high temperature 40 degrees, was taken to hospital and then discharged. Yesterday his urine became brown, took him to the doctor who then sent him to hospital for a check up. He was sent home with antibiotics. Today he has been getting blood in the urine what does that indicate Doctor: Hello, Brown or black color in urine in a one-year-old child can be due to urinary tract infection, urethritis, acute glomerulonephritis, or renal stones. Kindly do complete blood count, kidney function test, urine routine microscopy, and culture test, and ultrasound abdomen.-Give more fluids to the child.-An examination by the doctor should be done so that we may not miss any important findings.-Review back with the above reports.Hope I have answered your query. Let me know if I can assist you further.Regards,Dr. Deepak Patel"
},
{
"id": 66342,
"tgt": "What would be the square shaped bump on hand?",
"src": "Patient: My son has had a square shaped rash? I really don't know what it is it's not red just raised skin tone bumps in the shape of a square. And one on his hand but cluttered. He will be seeing his physician in a couple of weeks but wanted to get another opinion. Doctor: Hi, thanks for writing to HCM and sharing your son's health problem! If I were his consulting Doctor for this case of the square shaped bump on hand/rash, I would consider only one and one possibility and that is a nevus or congenital mole! However, it could be related to hormones and infections also. He needs a physical examination especially by a dermatologist for confirmation.Overall, its nature is of congenital and benign and not to worry at all! Hope this answers your question. If you have additional questions or follow up questions then please do not hesitate in writing to us. I will be happy to answer your questions. Wishing him good health."
},
{
"id": 111436,
"tgt": "Why am i having back pain and abdomen bloating?",
"src": "Patient: I was wondering after having my c section 7 months ago. I have recently started a new job thar requires heavy lifting since i started i developed symptoms back pain my incision hurts and abdomen bloating could the heavy lifting cause me to injure my insides. Doctor: Hello,I had gone through the case and found that it might be due to complication in cesarian.Normally we advise to avoid heavy weight lifting till 6 months.But before doing anything consult to gymecologist and go for proper checkup.Back pain might be due to spinal anasthesia.So take calcium once a day.Hope my answer will be effective for you.Thanks"
},
{
"id": 207692,
"tgt": "Why i not able to use bathroom?",
"src": "Patient: Hello, I have not been able to use the bathroom for a few days now, my next door neighbor is a nurse and she told me to check myself to see how far up it was. I did that but i dont feel anything. Its stuck for some reason all the way up by my lower stomach. What should i do Doctor: DearWe understand your concernsI went through your details. I suggest you not to worry much. This symptoms never is related to mental health. But to answer your query, if your symptoms are correct, please visit a physician for further corrective measures. If you require more of my help in this aspect, Please post a direct question to me in this website. Make sure that you include every minute details possible. I shall prescribe the needed psychotherapy techniques which should help you cure your condition further.Hope this answers your query. Available for further clarifications.Good luck."
},
{
"id": 219714,
"tgt": "What are my chances of my girl friend getting pregnant?",
"src": "Patient: As I have only rubbed on her vagina and also dip a little bit there is no intersourse hardly there is only a little and my sperm dsnt appear fully Inside her it will a little and she will dsnt appear her sperm ....den she does masturbation den sperm appears ....and she take unwanted 72 on 6 day....is there any chances of pregnancy...pls repply Doctor: Hallow Dear,From your history, I understand that you had not inserted your penis inside the vagina, nor did you ejaculate on het vaginal opening. However, pre-ejaculation secretions come out unknowingly and contain some sperms which are capable of fertilizing an ovum. Though she has taken unwanted 72, post coital emergency contraceptive pills, she has consumed them rather late - after 6 days. These pills are very effective emergency contraceptive pills if taken within 72 hours of the unprotected sex. Now the key factor is when did you have this venture of unprotected sex! The egg is released (ovulation)14 days prior to the next expected menses. The egg has life of 24 hours while the sperms are active for 72 hours. Therefore a period of about one week around the day of ovulation is fertile period. If you had your sex during fertile period, the chance of pregnancy would be high. So do your calculation of the days to find out the risk of pregnancy. Now you have to wait expectantly for her menses to appear. One hitch in this also is that after Unwanted 72 pills, there is a possibility of delaying period by about a week or so. So the best option for you is if at all she misses her period, about 3 weeks after this incidence of sex, get her pregnancy test done on the overnight first morning urine sample; done earlier, these test may give false negative results. Alternatively, you may opt for Beta hCG test which gives reliable results even few days before missing a period. If she is pregnant, you have an option of Medical termination of pregnancy till 9 weeks of gestation.To avoid such stressful situations in future, please practise reliable contraceptive methods like birth control pills, condom or intrauterine device. I hope this helps you.Dr. Nishikant Shrotri"
},
{
"id": 29209,
"tgt": "Do headache and dry mouth indicate lyme disease?",
"src": "Patient: Hi, My son has been experiencing headaches, dry mouth and dry eyes. He is a busy student athlete. I am wondering if I should be concerned or just make sure that he is properly hydrated. We live in an area with a high concentration of lyme disease. Can any of these things be symptoms of lyme? Thank you! Doctor: yes but we need to know some signs from u ..is there any circular redness and if he exposed to any thing like tick bite ,mosquito bite"
},
{
"id": 195955,
"tgt": "How can a lump on the testicle be treated?",
"src": "Patient: Hi, This is a very serious issue & I ve had tons of anxiety about it. I recently did a self examination & noticed something off. There s no lump growing on my testicule but behind my testicule I feel something swollen/inflamed or growing... I don t know what it is & I haven t felt any pain. I m working in get insurance and idk what to do. Doctor: A mass growing from the testis could be a benign cyst. Please do not become anxious about it till you get a biopsy report of the same. You would have to consult a Urologist, and depending on doctors discretion, an excision biopsy or a needle biopsy would be done. Once the report is obtained, the appropriate management can be started by the doctor."
},
{
"id": 16741,
"tgt": "Suggest medication for low BP",
"src": "Patient: I was having hypertension and one the medicine Stamlo T for 3 years. But from some time my pulse rate was very high and my doctor prescribed Nebistar 5 and Temsan 40. But after taking both these medicines my pulse rate was normal but B.P started lowering very much. Now I have been prescribed Nebistar 2.5 mg only once and am taking for ten days and feeling very relaxed. Should I continue this medicine as I have heard that it has serious side effects like impotence. I am only 37 yrs. old. Doctor: Hello There After going through your medical query I understand your concern and I would like to tell you that in case of low blood pressure it is better to withdraw medication one by one . It\u2019s recommended for you to stop Telangana and continue with Nebistar and monitor your blood pressure.Hopefully this information will guide you properly.Kind Regards Dr Bhanu Partap"
},
{
"id": 112596,
"tgt": "Pain in the back after lifting weight. Swelling. Is this serious?",
"src": "Patient: I'm a 37 year old female and always lift too much wait for my 5'2 106 pound frame, anyhow I was lifting 2 cases of champagne today and my lower left side of my back hurts ( more annoying than anything) but I have just noticed the dimple on my lower left side of my back is swollen ( where the pain is to be exact) is this serious? Doctor: Hi,Back muscle / paramuscle gets strained easily while bending and lifting weights. The swollen areas most likely reflect the same. However many incidents of spine disc problems have also occurred while lifting weights. Therefore my suggestions is to watch the symptoms with caution. Rest adequately; use pain balms/heat compresses and oral pain killers. If it hurts too much or if it doesn't improve in the next 3-5 days, consider visiting your surgeon for detailed evaluation and radiological tests.Hope this line of treatment helps you. God Bless!!"
},
{
"id": 204118,
"tgt": "What type of specialist doctor should a person affected with dementia be taken to?",
"src": "Patient: I have. neighbor whose mother has dementia that was diagnosed by a family physician. The family would like to see a specialist that can answer more specific answers and make sure this is the correct diagnosis. If the mother does definitely have dementia, they want a plan of care. What type of specialist should they take their mother to? Thank you very much. Doctor: Hello, You can take the patient who has been diagnosed with dementia to a Geriatrician for better management. You can even consult Geriatric Psychiatrist, Neurologists or Neuropsychologist's for tests related to memory & thinking. Hope I have answered your query. Let me know if I can assist you further. Regards, Dr. Nupur K, General & Family Physician"
},
{
"id": 119030,
"tgt": "Increasing WBC count, concluded hyper eosinophilic syndrome, advised hydroxyl urea+. How to overcome side effects ?",
"src": "Patient: Dear Sir, I need to consult a kind of problem related to white blood cells( WBC ) in blood.... this problem is related to my sister who is 31 years old...from last 3 months her WBCs are increasing in number....we had cunsulted with many doctors and done with her proper diagonosis also...report of every test was normal...after dat doctor suggested to go for the bone marrow test...after the bone marrow test doctors concluded dat due to a kind of hyper eosinophilic syndrome her eosinophils are increasing and that causing the increasing number of WBCs also...doctors told us dat this problem can t bu cured permanently...this can be controlled only by medecines....but a kind of medecine called hydroxyl urea 500mg (prescribed by the doctor) giving lot of side effects to her body...due to this problem and such a high dose, her body is getting weak day by day and suffering from other side effects also...i jst want to know is there any permanent, specific and a proper treatment to this disease is available or not..???? pllzzz help me sir... Doctor: Hellowelcome to health care magicRegarding hyper eosinophilic syndrome,it is a myeloproliferative disorder..In the treatment 1st line of treatment is corticosteriods like predinsolone..Hydroxyl urea is the 2nd line of treatment..If you have more side effects,then another drug is there interferon therapy like imitinibConsult hematologist and try this medicine..hope you understand my suggestionthank youDr.siddartha"
},
{
"id": 162174,
"tgt": "How often can you switch between Tylenol and Ibuprofen for treating high fever in a 4-year-old?",
"src": "Patient: My granddaughter is 4. She had a temp of 104.2. I gave her iburprofen. 45 mins later it went up to 104.4. I put her in the tub. Temp dropped to 100.6 she has been out for over an hour. Her temp is going up. 101.6 Can i give her Tylenol also? How long do I wait after the ibuprofen Doctor: Hi, I have gone through your question and I understand your concerns. You can give Tylenol right away together with cold compresses. Ibuprofen doses should not exceed four times daily and you can give Tylenol for another four doses also. Hope I have answered your query. Let me know if I can assist you further. Regards, Dr. Salah Saad Shoman, Internal Medicine Specialist"
},
{
"id": 120888,
"tgt": "Can a corn be treated using salicylic acid?",
"src": "Patient: i m having a corn i applied salicyclic acid actually i dunt exactly know that its a corn or bacterial infection. i applied salicyclic acid n it peeled off the skin it got clear but still i could see small hemolysis underneath which made me think it of a bacterial infection confused ? Doctor: Hello,I read carefully your query and understand your concern. Corns\u00a0will continue to cause pain and discomfort until they are removed, but they can be successfully treated with nonprescription products in many instances. The only ingredient proven\u00a0safe\u00a0and effective for unsupervised self-use\u00a0is salicylic acid.So,you can continue using salicylic acid to treat the corn.Hope my answer was helpful.If you have further queries feel free to contact me again.Kind regards! Dr.Dorina Gurabardhi General &Family Physician"
},
{
"id": 117943,
"tgt": "How to control the BP levels?",
"src": "Patient: II have issues with uncontrooable b/p it goes up reallyhigh 140/105 then drops to 104/68 causing me to almost ass out. My doctor has been trying to regulate it for sometime now. I can not work because it changes what should i do ? I do Have other health issues too Thank U Barb Doctor: you need to set the dosage of your anti hypertensive medicine. dont take it yourself always ask consultant doctors when you change the dose of drugs. and you have not mentioned what other health problems you have so without knowing we cant prescribe you any medicine. Because each have some contraindication and side effects. you better you continiu with your doctors advise and follow it strictly. many times it take time to set the dose. so dont worry."
},
{
"id": 121236,
"tgt": "Does etova p help to treat painful and swollen knees and ankles?",
"src": "Patient: hello sir, my mother is 70 yrs old and she is suffering from severe pain and swelling in both of her knees and ankles. Doctor has advised her to take etova p. kindly let me know whether it will help her or not. thanks Mrs Fatima from Allahabad INDIA Doctor: Hello, Etova-P is an analgesic and give symptomatic the relief from long standing pain. Nothing much to worry and your treatment is in the right track. Underlying causes like arthritis must be ruled out. Hope I have answered your query. Let me know if I can assist you further. Take care Regards, Dr. Shinas Hussain, General & Family physician"
},
{
"id": 97353,
"tgt": "Can acupressure treat recurring itchy rash on the upper part of the body?",
"src": "Patient: I have recently acquired a rash on my upper extremities and torso and went to the doctor and was given a week of prednisone. The rash went away but after 3 days off the medication is now coming back. It is very itchy and is spreading. Will acupressure help me? Doctor: **1. since rash is recurring, make sure to check with Dermatologist for further investigation and examination.2. since persistent [chronic] urticarial tends to comes and goes thus make sure that common stimuli like heat, cold, menstrual periods, stress, or emotion may make the rash flare up worse than usual.3. steroids is not a usual treatment due to side effects related with them, thus anti histamine tablet orally and calamine lotion for local application is suitable 4. since the site involved is locally thus contact allergic dermatitis and/or lichen simplex could be possibility.5. Acupuncture is based on belief that fluids flow through channels and inserting tiny needles open the channels, allowing fluids to flow better thereby might decrease pain and improve healing. Always see a caregiver for acupuncture. Do not try to give this treatment to yourself."
},
{
"id": 61109,
"tgt": "How can a painful lump on the wrist be treated?",
"src": "Patient: I did some PT exercises yesterday for a Rotator Cuff injury. I was using a band that was probably too heavy ( I have only been to PT once so far) At 2 in the morning I woke up with terrible pain in my wrist. Nothing helped. This morning the pain was awful. I started taking Ibuprofin and icing it. I now have a large soft lump on the outside of my wrist and I still have a painful weak wrist. Should I ice it or use heat? Do I need to see a doctor or can I just go to the PT tomorrow as scheduled? Thank you. Doctor: Hello dearWarm welcome to Healthcaremagic.comI have evaluated your query in details .* This is possible inflammation at the wrist , must see the doctor before going to PT tomorrow .* Alternate application of heating pad and ice pack would be of immense help .Hope this will help you for sure .Wishing you fine recovery .Welcome for any further guidance .Regards ."
},
{
"id": 208083,
"tgt": "Can dementia be cured without medications?",
"src": "Patient: Hi with stage 4 aids which it has been only discovered when the toxoplasmosis and tumors in the nose and pancreas were found, the cd4 cells are now at 150. will they go up again once in one month they will operate on the pancreas cancer? the nose tumor has been removed alreadyand its fine. the lesions provocked by the toxo are still there they menaged to get rid of the infection but the demages are hard to cure , will they cure? without medications, dementia takes over the mind, will that cure? movements walking is very slow. will this go back to normal?? thank you in advance. i know it is a lot of questions i hope u can help me Doctor: DearWe understand your concernsI went through your details. I suggest you not to worry much. Many researches and researchers confirm that medicines alone cannot cure mental disorders. Life style changes, change in thinking pattern, relaxation etc are as essential as medicines. Psychotherapy can help you changing your lifestyle and thinking patterns. Yoga and meditation help you to streamline your metabolism and neurological balance. Please consult a psychologist for further information.If you require more of my help in this aspect, Please post a direct question to me in this website. Make sure that you include every minute details possible. I shall prescribe the needed psychotherapy techniques which should help you cure your condition further.Hope this answers your query. Available for further clarifications.Good luck."
},
{
"id": 18400,
"tgt": "What causes high BP and fatigue?",
"src": "Patient: Husband has recently developed high b/p. Also having chills without fever. shakiness, and overall feels lousy. Previously this man liked chopping wood and working around the house. Now he has no energy for anything. Our doctor ruled out viral possibilities. took losartan 25mg one week, increased to 50 mg and b/p still160/100. Doctor: Hello and Welcome to \u2018Ask A Doctor\u2019 service. I have reviewed your query and here is my advice. High blood pressure uncontrolled may be due to Renal problems or salt consumption or CKD or anxiety or Septicemia etc. Until examination is done it is difficult to say what it is. Avoid spicy food and alcohol avoid smoking and caffeine drinks. Daily exercise and yoga may helpful. If symptoms not improved please consult your doctor he will examine and treat you accordingly. Hope I have answered your query. Let me know if I can assist you further."
},
{
"id": 16466,
"tgt": "What are these rashes in my sons body ?",
"src": "Patient: hi my son is 6 years old, he keeps coming out in rash that are in clusters like blisters but not watery. when they go they scar Doctor: hi well come to hcm. veryfy it could be chicken pox, it appears first on trunk and then goes to limb and other part of the body.They are pleomorphic ie different shape and stage in different parts of the body at atime.May or may not be associated with cough and fever.Usually may or may not be pain or itching.So better you confirm it with your Doctor. Thanks"
},
{
"id": 173391,
"tgt": "What can cause shooting stomach pain in a child?",
"src": "Patient: Hi my daughter is 10 yrs old and has complained for months of stomachache. She always has stomach pain from stress. It comes and goes. She is otherwise very healthy and athletic. This month first two weeks had stress relate pain from sports tryouts. When that was done it seemed to get better. Now this week every day is complaining of stomach pain, especially at night. Last evening she complained of shooting pains that made her wince. I massaged her belly and found a hard mass very tender when touched. About 3 above navel, and about 2 diameter. Does not protrude past flat belly wall ( fairly deep). I have booked her with family dr tomorrow. She can eat and go to the bathroom ok, no fever, just constant ache and sometimes shooting pain. What could it be? Doctor: It is advisable to get an ultrasound whole abdomen done on her to ascertain the origin of the mass. Since you mentioned it is very tender, it could be inflammatory, but then the child has no fever.At her age conditions like imperforate hymen could cause a painful mass, without fever as menarche could have set in. This can be treated with simple surgery.If the origin of the mass is elsewhere, more advanced investigations like CT scan may be required and may require more aggressive treatment.It would help to keep us posted on her investigation findings for further help."
},
{
"id": 33876,
"tgt": "Suggest treatment for blackening of fingers in a diabetic patient",
"src": "Patient: My Mom has been diagnosed with Lupus. Acutally in Dec her 2 fingers of the right had had turned cold and then black. It then had vasculitis and has dried. She is been given Omnacotil 5 mg and Zilast 100 twice a day. She was doing well. But from Yesterdaty her 2 fingers of the left hand has started getting cold and dark ( lack of blood supply ). She is daibetic and is on 3 time insulin. What should we do to save her fingers and life. She is 64 yrs old. Please help we are in Vadodara Gujarat Doctor: Hello dear,Thank you for your contact to health care magic.I read and understand your concern. I am Dr Arun Tank answering your concern.It looks like your mothers limb turn into necrotic changes.This changes are more common on diabetic patient.Steroids should not be adviced to the diabetics as this can further complicate the problem.In my opinion you should be examined at nearby doctors. They will guide you regarding whether this blackening will revert or not. Whether there is need for surgery or not.But atmost care should be taken in therapy. As this blackening can be the cause of limb amputation and can be a life threatening condition.Please advice her to maintain good sugar level as this will be preventive in further worsening of condition.I will be happy to answer your further concern on bit.ly/DrArun.Thank you,Dr Arun TankInfectious diseases specialist,HCM."
},
{
"id": 201678,
"tgt": "What causes pea sized bump on scrotum?",
"src": "Patient: I am a teenager, and I have a larger than pea sized bump on my scrotum (the outside near the left leg below the left testicle). I ve had it for a little while, its grown and I just squeezed it and white liquid came out. Kinda looked like semen but not really. Help, really concerned. Doctor: Good Day and thank you for being with Healthcare Magic! I think what you had is a sebaceous cyst in your scrotum. It is filled with oily whitish substance because of the sebaceous glands in the scrotum which is normally present. It may or may not recur and it will pop on its own. Just keep a healthy hygiene and wash with soap and water. I hope I have succeeded in providing the information you were looking for. Please feel free to write back to me for any further clarifications at: http://www.HealthcareMagic.com/doctors/dr-manuel-c-see-iv/66014 I would gladly help you. Best wishes."
},
{
"id": 180990,
"tgt": "How safe is anesthesia in dental procedures?",
"src": "Patient: Is it safe to have the wisdom teeth removed in the dentist office under anesthesia. I saw a story on TV that indicated it could be unsafe if the anesthesia isn't administered by a qualified anesthesiologist Would it be safer to have the surgery done by an oral surgeon. The student is 18 years old. Doctor: Hi...Welcome to healthcare magic. As you mentioned you want to remove wisdom tooth extraction under anaesthesia.Correct technic of nerve anaesthesia is safe .Unwanted injury during nerve block lead to complication. During wisdom tooth extraction anaesthesia given by nerve block ...Dental surgeon give a anaesthesia. Wisdom tooth extraction is a minor dental surgery. For nerve block no need for anesthesiologist...this one given by dental surgeon. I hope this will help you. Still you have any questions than you can ask. Get well soon.. Thank you."
},
{
"id": 155181,
"tgt": "Does joint pains, headache and soreness/knot in throat indicate throat cancer?",
"src": "Patient: hi i was just trying to convince myself that at 32 years old in November I do not have throat cancer. I was very ill for the past few days symptoms were:- joint pains headaches sore throat feeling cold in my body but not having a fever on Sunday i led worship and I sing soprano, i poured my heart and my voice out now I m having a difficulty swallowing and i have this consistently sore spot . It feels like a knot in my throat,my throat also feels very dry. My grandmother died from throat cancer in her fifty s back in 1995. She was an alcoholic.How high are the possibilities of this being a tumor (cancerous) I sing alot that s my ministry so my voice is always hoarse and seeking recovery after every sunday serbice Doctor: HiI would advise you to do a pet scan.You can take 1 week of antibiotic. If it does not resolve then biopsy has to be done."
},
{
"id": 45050,
"tgt": "Shall i get my period based on my follicle report ?",
"src": "Patient: i had follicle study on 12th day . endometric is 5.1 and in right ovary multipal follicle occur but the largest is 1.26cm ... and no fluid found in pod.... ow my doctor asked me to come on 14th day for further follicle study . is this helpful...... now is this mean that i will definitely have periods this time Doctor: Hi Swati,Welcome to HCM.First of all your base line status of follicles in the ovary has to be assessed by 7th day of your period. If there is increase in size of follicle from that base line study, it is informative. Starting the follicle growth study on 12th day may be misleading and tend to be misinterpreted.From the data provided by you the follicle growth( if it did increase from it's base line status ) is not satisfactory.Endometrium thickness is also on the lower side.From this single study nothing much can be predicted.As suggested by your doctor, repeat the scan and see if there is any progress in the follicle and endometrial maturity.Wish you all the best."
},
{
"id": 107333,
"tgt": "What causes loss of appetite and back pain along with muscle pain?",
"src": "Patient: I have a loss of appetite,when I do eat it is extremely small portions, lack of good sleep due to my husband snoring, some muscle pain (without any strenuous activity) constant back pain (usually I blame arthritis) I can t wear any clothes tight at the waist without back pain, Doctor: Hi. It's quite possibly due to arthritis plus the fact that u don't sleep well may make u tired in the morning,irritable and cause loss of appetiteI suggest 1 tablet of Muscle relaxant e.g Flexon every day at night just before bed. Start off with 500mg 3-4 times a day and then taper to 250 mg after 2 weeks.Max dosage allowed is 750 mg 3-4 times a dayPlease make sure u don't have liver disease or porphyria before taking the medication"
},
{
"id": 51390,
"tgt": "80 years old, renal failure. History of diabetes, hypertension. Treatment other than kidney dialysis?",
"src": "Patient: My father is 80 yrs old, he has end stage renal failure and is reccomended for dialysis in the near future. Are there any medicines or treatments he can take to stall the progression of the disease or improve the health of the kidneys? Because of his heart condition he may not be able to tolerate dialysis. He is a diabetic and has mild hypertension . Doctor: if diaysis is recommended then the kidney actualy is not able to support the life of the person. dialysis should be done such situation. whether your patient is suitable for hemodialysis because of cardiac condition can only be judged after detailed evaluation. but if not suitable you can go for peritoneal dialysis. in any case there is no way to avoid dialysis once it is clinically indicated."
},
{
"id": 223553,
"tgt": "Can marijuana lead to spotting?",
"src": "Patient: Hello, I have had the implanon inserted for about 2 years now, I had a lot of spotting and so a couple of months ago I took birth control pills and that made the spotting stop. A couple of days ago I smoked marijuana and suddenly my spotting returned, could the marijuana have caused the spotting? Doctor: Thank you for your question and I will try to help.The most common complaint with these implants (they are progesterone implants) is breakthrough spotting. Early on after their insertion, this is very common. Later, such as after 2 years, the lining of the uterus becomes very thin (which is normal) and prone to bleeding. This can once again be treated with birth control pills. This has nothing to do with marijuana use. Finally, make sure that you check a pregnancy test now and in 1-2 weeks.I hope that this helps and good luck!Dr. Tim"
},
{
"id": 160706,
"tgt": "What causes children to not gain weight and have constant loose motions?",
"src": "Patient: hello, We have two boys ages 22 months and 10 months. They have not gained any weight in many months and are both well below the chart. They both also suffer from soft bowel movements around 6 times a day. They have been tested for CF and Cilliac and both were negative. They are developmentally on track and very happy non lethargic children. We live in a rural idaho town with no specialists. We see the Dr again today and wondering what else he should be looking for? Doctor: Hi,Are the stool mucus/blood stained / contains lot of gases? Is the perineum excoriated? Any ulceration or discolouration in and around mouth? Few conditions causing this trouble are- Vitamin / mineral deficiency like zinc / niacin, worm infestations are rarely malabsorption syndromes. Apart from the said tests you mentioned, a stool routine examination (for worms and reducing substances) may give some clue. In such cases, I used to deworm children and give a course of zinc therapy to see the response, and proceed accordingly. Your doctor will discuss in detail. Meanwhile avoid / restrict sugary items / juices / soft drinks / candies- these can cause loose stools because of osmotic and other mechanisms.Hope I have answered your question. Let me know if I can assist you further. Regards, Dr. Muhammed Aslam T. K., Pediatrician"
},
{
"id": 149517,
"tgt": "Dark and thick vomit after having seizures. On Keppra. Detected nocturnal seizures. Advice",
"src": "Patient: My son is 22 and has been having seizers (grand mall) since age 10, he is under doctors care and on Keppra. However, in the last year after a night of seizers he will through up for days, dark and thick. He has what the doctor calls nauturanl seizers because they would only happen right before he work up in the morning and was still asleep. However, over the last few years he could have them while taking a nap even. Doctor: Hi,Thank you for posting your query.Vomiting is not related directly to seizures or Keppra (levetiracetam). However, in some cases, after vomiting and severe retching, gastritis or esopahgitis (inflammation of esophagus and stomach) may occur. So, it would be useful to get an evaluation done by a gastroenterologist.I hope it helps.Best wishes,Dr Sudhir Kumar MD DM (Neurology)Senior Consultant Neurologist"
},
{
"id": 19608,
"tgt": "What causes walking imbalance after stroke?",
"src": "Patient: My query is related to stroke patient......... my mother had stroke nr abt 6 yrs ago in which her right side totally affected.....now she can walk imbalancely. and right hand is totally paralytic....... dibities is another problem... which now affect her eye sight........ wt treatment shd we try....can it recoverd some % Doctor: hiii for proper walking ...both lower limbs should be free from disease ...as your mother had stroke ...her one leg is having defective motor nerve supply to muscles ...as one side muscles are not working properly ..so there is imbalance ...you can take help of physiotherapist he will help ur mother"
},
{
"id": 202422,
"tgt": "Semen analysis done. Coagulation present. Few pus cells found. Need to continue medicines?",
"src": "Patient: quantity 2 ml, liquefaction up to 30 min,viscosity thick, color opaque grey,reaction(ph) 7.5, coagulation present,microscopic exam. 20%,sluggish 20% dead 60, pus cells 4-6, epith cells nil,r.b.c nil total spermatozoa count 45 million.please suggest me Is it normal report ? presently i am taking the medicine Endro Gardian,c-c 50 and lyco capsul Is their any need to continue it ? Doctor: yes i would continue to help your semen parameters further.Please rate 5 stars. I strive to provide you with the best answer possible to your question!"
},
{
"id": 18760,
"tgt": "Suggest treatment to control blood pressure",
"src": "Patient: hi i have a problem. went to my doctor today and my bp was 200/90 so he gave me another type of bp med but because i took my other bp med before going to dr i did not take other med but tonight i took my bp and my rt arm read 174/113 and left arm 233/138 plz advise Doctor: Hello and Welcome to \u2018Ask A Doctor\u2019 service. I have reviewed your query and here is my advice. I would like to tell you that if you have significant difference between the blood pressure measures indifferent arms then you may require a physical evaluation for the same. A big difference in blood pressure measurements between your arms could be indicating a health problem like blocked arteries in your arms, kidney disease, diabetes, or heart defects which requires proper evaluation for a proper treatment. It's recommended for you to consult your physician personally and get yourself evaluated. Hope I have answered your query. Let me know if I can assist you further. Regards, Dr. Bhanu Pratap"
},
{
"id": 79533,
"tgt": "Why getting positive for koch infection in apicolordotic view only?",
"src": "Patient: im 26 yrs. male, old,5'8 in height and weighing 79kgs...i have kochs infection in a past 9yrs and taking a medication for 6months and in past 3yrs. i have a xray result of essentially normal chest but when i try to know my xray result again lately my result is kochs infction on right apical area..doc how can it be happen?my result appears in a apicolordotic view only?and why my disease cames back? Doctor: thanks for your question i completely understand your question sometimes tubercular consolidation happens in the hidden areas like posterior to clavicle or post to the heart .these places have to be covered by lordotic view if the symptoms are present or other method to scan all the suraface area of lungs is ct scanbut tubercuosis can be confirmed by sputum afb smear examination which is a better specific marker.gold standard to makeout active tb infection is culture of sputum for tb bacilli.if your tuberculosis is confirmed by smear examination then you should consult with a pulmonologist who can request for a sputum culture by mgit or line probe assay that would lead to the type of tb because now. a days treatment for tuberculosis is different for normal tb , mdr tb , xdr tb.as you have taken many courses of anti tb treatment you need to get these tests , you can consult the nearest dots centre for the detailsthanksfeel free to ask more questions may god bless you with good health"
},
{
"id": 33596,
"tgt": "What are the precaution to be followed after taking Rabipur injection?",
"src": "Patient: I am taking Rabipur injection against street dog bite on 09.04.2011 and I am to go to Chennai i.e.32 hrs train journey. What precautions should be taken on (1) Diet-whether veg. food or non-veg. food should be taken? (2) whether can come out in noon of summer? Doctor: HiThank you for asking HCM.I have gone through your query. There is no restrictions in food or life style when taking rabipur (antirabies prophylaxis) or after that. You can have whatever food you like vegetarian or non vegetarian. Physicians usually tell to avoid alcohol and smoking just for health of patients. But this is also not related to antirabies vaccine actually. You can come out in noon during summer also.Hope this may help you. Let me know if anything not clearThanks."
},
{
"id": 17624,
"tgt": "What does triglyceride-345 and cholesterol-240 from lipid profile indicate?",
"src": "Patient: My age is 40,weight is 76kgs and height is 5 feet and 11 inches. My heartbeat is 82 and blood pressure is 130/80. When ever I try to exercise, after someday my blood pressure becomes high and I feel problem in breathing. I did lung x-ray and it shows o.kay. I tested my lipid profile and hemoglobin test and eeg[color doppler] test. Av is slightly enlarged. In lipid profile triglyceride is 345 and cholesterol is 240. I checked my diet and took medicines. The problem is less but not totally cured. Doctor: Hello, The increase in blood pressure values is quite normal during physical activity and emotional stress in every healthy individual. But, if your blood pressure values are persistently above the normal ranges (125/85mm Hg) you may need to start anti-hypertensive therapy. I would also recommend starting statins considering your high cholesterol and triglyceride levels. In the meantime, I recommend reducing caffeine and salt intake. Hope I have answered your query. Let me know if I can assist you further. Take care Regards, Dr Ilir Sharka, Cardiologist"
},
{
"id": 11081,
"tgt": "Suggest remedy for regrowth of hair in forehead and center part of head",
"src": "Patient: hello, dr. sir my name jatinder m from punjab. sir, i have lost near about all the haur from.my forehead n centre part .my age is 26 sir how can i regrow my hairs .sir m also feeling heat producing from my centre part of the head.how can i cure. these problems.? plz reply at YYYY@YYYY Doctor: Dear Jatinder,The symptoms & case history mentioned in your post suggest that you might be having Androgenetic alopecia.Also known as Male pattern baldness, it usually follows a pattern of receding hairline & is related to genetic makeup and male sex hormones.Management includes:1. Minoxidil preparation to be applied directly to the scalp to stimulate the hair follicles.2. Finasteride preparations that interferes with the production of male hormone linked to baldness.3. Multivitamins+ mineral supplements.But these medications have to be used regularly under the guidance of a Dermatologist.Wishing you a Good Health.Take care."
},
{
"id": 12019,
"tgt": "What is the remedy for dark birth mark on my thigh ?",
"src": "Patient: i have a dark birth mark across my lower right back and all the way upto groin area and right thigh....Will it be a turn off for my wife during love making as i am very fair complexion otherwise. Also i dont know if i could ever visit a naturist camp/nude beach due to this mark? is there any remedy for this? As i have absolutely no medical porblems due to this birth mark Doctor: Hello Welcome to HealthcareMagic The birth mark if Port wine stain it is difficult to get removed. Please visit a dermatologist and get advise. Have a Healthful Day.."
},
{
"id": 52019,
"tgt": "I have kidney stone. Is it curable by medicine?",
"src": "Patient: Hi Doc, a CT Scan have shown 1.2cm of stone in my right ureter , pls. suggest if it can dissolve thru medicine Doctor: Berberis Vulgaris-Q homeopathic mother tincture helps to dissolve kidney stones. But I am doubtful whether it will work for a 12 mm stone or not. Drink 20 to 25 drops in warm water 3 to 4 times a day. Also drink lemonade soda daily. Ayurveda has dissolved stones up to 18 mm. Contact Dr.M.S.Zambre, BAMS-AMA, Shree Gajanan Clinic, Near Datta Mandir, Dhayari,, Pune \u2013 411041, Phone: 09423533301"
},
{
"id": 50925,
"tgt": "Red blood cells found in urine, hernia, cyst in kidney. How to treat them?",
"src": "Patient: I am 56 years old, male. I got my urine tested after seeing that my that urine colour is dark yellow previously never had this problem. Any ways Urine DR indicates occassional Red Blood Cells. I also have a lump sort of protusion on my front upper thigh portion leading to the scrotum which I believe is the hernia . I had an ultrasound of whole upper abdomen in October 2012 tiny cysts were discovered in my kidneys. I am regularly jogging and doing physical exercises no weight training just plain flexibility workouts. My height is 5 feet 9 inches and weight is 60 Kg I dont have diabetes and hypertension . Do let me know what should I do? Is it something serious. Doctor: Hi, Welcome to HCM, After going through your history,you seem to have inguinal hernia and for this you should consult a General Surgeon, please understand there is no medical treatment for hernia and you should get it operated to repair it if your muscles are weak a mesh of Dacron may be used. Your urine showing RBCs this may be due to UTI, or an kidney stone if there commonly. You can repeat a ultra sonography to see any abnormality in prostate or kidneys and bladder. your surgeon may give course of urinary specific antibiotic for 7 to 10 days, you can repeat urine R&M. Please continue to follow healthy lifestyle. Take care Good luck."
},
{
"id": 117108,
"tgt": "How to confirm the requirement for iron supplements?",
"src": "Patient: Got blood test results cholestrol 160 LDL 70 hdl 79 triglycerides 56 Vitamin D 59 blood sugar 97 CBC 6.7 red 4.10 hemoglobin 11.9 hematocrit 34.8 I am a 53 year old active woman with no health issues but nurse practicitioner thinks I need iron supplements....I m not so convinced ...you suggestion Doctor: Hi, dear. I have gone through your question. I can understand your concern. You have anemia but from this report only we can not say its always due to iron deficiency. Thete are many othet causes. Even vitamin B12 deficiency may produce same. Sp you should go for peripherals smear examination and serum iron and ferritine level. If ypur iron and ferritine level comes low tjen you should take iron treatment. Otherwise tale treatment according to cause.Hope I have answered your question, if you have doubt then I will be happy to answer. Thanks for using health care magic. Wish you a very good health."
},
{
"id": 92205,
"tgt": "What could be the reasons for recurring nausea, pain in abdomen and below belly button?",
"src": "Patient: nausea on and off for the last couple of days. Took pregnancy test- negative ( but have IUD) occasional pain in abdominal area. But not today. Pain is below belly button. thought it was ovarian pain. A little more in the middle than an ovary. what so you think? Doctor: Hi. The pain below the belly button with nausea can most probably be due to intestinal infection. Ovarian pains can cause nausea only if he pain is severe enough for this reflex. I would like to know what are the other associated symptoms ."
},
{
"id": 191489,
"tgt": "What causes low blood sugar levels?",
"src": "Patient: Hello Doctor, I am a 59 year old female, over the last few months I have been feeling like I am shaking inside about once a fortnight, no real pattern to the feeling. A friend suggested I test my blood sugar levels, on one morning it was only 2.3, but was 4.7 on the other three occasions I tested it. Yesterday I felt shaky so when I did the test and it was only 1.7, I had eaten about three hours previously and did not feel hungry. what could be causing my levels to drop so low?. Thank you. Doctor: welcome to hcm...mam as your blood sugar level are coming low ..there are some problems which need to be taken care of ...please get ultrasound abdomen to know status of liver pancreas and adrenal gland ..as there may be problem with any of these ..on the lab test go for liver function test ...thyroid status ...serum cortisol level ...insulin and glucugon level ...epinephrine level ..."
},
{
"id": 46709,
"tgt": "What is the best medicine for kidney stones?",
"src": "Patient: hello, i am 20 years old and am dealing with kidney stones. i just had a 6mm stone removed about 10 days ago and my stent removed about 5 days ago. i still have small 1-3mm kidney stones in both kidneys and am urinating a lot of blood and blood clots, is this normal? i am worried. Doctor: Hi welcome to HCM....You are having small kidney stones of one and three mm....For their flush out in urine more water drinking done ...Up to 1.5 to 2 litre per day. .If needed alpha blocker that relax ureter muscle can be prescribed ...Diuretic also help in flush stone in urine ...To prevent infection antibiotic course given...Passing stone leads pain , so ibuprofen can be taken....If stone large in size to pass than only surgical or ESWL management needed Advise : as your stone bleeding better to consult nephrologist...Take care"
},
{
"id": 223624,
"tgt": "What are the side effects of postinor 2?",
"src": "Patient: Hi doctor I m Paula by name, 22yrs of age and I kinda have a problem which is how many intake of postinor2 can be harmful to my body ? Why I am asking this question is because in one month I have taken 3packs of postinor2 and I m scared I might be at risk, pls doctor kindly enlighten me plus is it true that the intake of salt solution after sex can prevent pregnancy? Doctor: Hello and Welcome to \u2018Ask A Doctor\u2019 service.I have reviewed your query and here is my advice.There is a high chance of the hormonal imbalance as you took 3 packs of the postinor 2. It cause earlier/delayed period and excessive bleeding. Other side effects are: nausea or vomiting, dizziness, fatigue including headache, breast tenderness, lower abdominal pain or cramps, and ovarian dysfunction. So, do not use postinor 2 as regular contraceptive.Salt solution after sex does not prevent pregnancy. If you get pregnant then you can terminate the pregnancy by abortion pill containing mifepristone plus and misoprostol under the supervision of a gynecologist.Avoid stress, take healthy diet, drink plenty of water, and do regular exercise.Hope I have answered your query. Let me know if I can assist you further.Regards,Dr. Dholariya Sagar Jayantilal"
},
{
"id": 118741,
"tgt": "Anemic, low iron levels, body not absorbing iron. Suggested iron infusion. Any risks associated with this?",
"src": "Patient: My husband was tested for iron absorption & it came out that his body doesn't absorb iron. He is anemic & has had low iron levels before, but this is the first time he has been tested by a blood test for the absorption, took an iron pill afterwards, and hour later another blood test. Now he is to have iron infusions. Are there risks to this ? He is tired all the time, has no energy at all & never feels good. He is 76 years old. Doctor: Hi, Some people will have the problem of non absorption of iron, or some ingredients. Tired ness, weekness, etc are due to the anemia, that is lack of hemoglobin. For non absorption you can try giving green leafy vegetables, in plenty for supply of natural iron. Infusion may not have any problem, and once desired level is attained he will be given intramuscular injections of iron. Thank you."
},
{
"id": 173115,
"tgt": "What could cause a red spot on the right inner thigh?",
"src": "Patient: I brought my son home from nursery this evening and noticed a red spot on his right leg inner thigh since then i have changed my son again and have noticed there are several more appearing some with yellow heads on theyve grown quiet big n look angry red Doctor: HI...by what you quote I feel that it might be a n allergy like a simple urticaria or a rapidly spreading pyoderma as you quote that there are lots of yellow heads appearing. But I want to stress on something very important - Skin conditions are best diagnosed only after seeing directly. I suggest you to upload photographs of the same on this website, so that I can guide you scientifically.Hope my answer was helpful for you. I am happy to help any time. Further clarifications and consultations on Health care magic are welcome. If you do not have any clarifications, you can close the discussion and rate the answer. Wish your kid good health.Dr. Sumanth MBBS., DCH., DNB (Paed).,"
},
{
"id": 59919,
"tgt": "Fatty liver, pain in LHC. Taking rosuvastatin, amledipine. History of high cholesterol. Treatment?",
"src": "Patient: hello , i have high cholesterol level and for that treatment doctor prescribe me Rosuvastatin 10 and Amledipine 2.5 After the 1 month, now i am suffering from the pain in Epi. and LHC, and in USG (abdomen+ Pelvis ) there was found the Fatty liver , Is it due to the Rosuva and Amlodipine ? Plz give me suggestion for treatment (SGPT -169, SGOT- 74) Doctor: Hi, Welcome to HCM, I am Dr. Das Amlodipine and Rosuvastatin does not cause fatty liver. In our day to day practice fatty liver is the modt common finding. But in your case the SGOT and SGPT both are elevated it indicates any hepatocellular insult like infection, alcohol ingestion. What about the other reports of liver function test? Do you have any yellowish discoloration of eye or urine? If you did not do a Liver function test, first go for it and review the report. Till then drink more wter and avoid rich food."
},
{
"id": 85960,
"tgt": "How can recurrent abdominal pain, the source of which cannot be identified, be treated?",
"src": "Patient: I have been in pain for about 6 months now. It started as a little pain in my lower stomach, over the course of about a week it slowly localized to my lower right abdomen. I was constantly peeing and there was no sense of relief after peeing like there normally is and I also wasn t letting out a lot of urine. Eventually the pain was constant but sometimes it wasn t as bad as others. The pain eventually got so bad I had to go to the hospital, the doctor finally got around to me after waiting for about 5 hours, she pushed on my side and just by the way I jumped when she pushed she told me it was most likely appendicitis. The nurse told me I would have to stay on an if overnight and get an ultra sound in the morning. The ultra sound didn t show anything but they said my appendix might just have been behind something so they decided to do a cat scan even though they normally don t do them on someone as young as me because they wanted to be sure. After the cat scan the doctor told me I most likely had mesenteric lymphadenitis and if he were to magnify my cat scan results he would see swollen lyphnodes and he basically said there was nothing that would get rid of it, but it would eventually go away on it s own. He said they would normally have someone with this stay on iv fluids for a few days but they just didn t have the room. The doctor suggested drinking Pepsi because it has a lot of sodium and that would help almost like how you gargle warm water with salt when the lyphnodes in your throat are swollen. So I took his advice and also booked an appointment with my doctor for about a month later. I told my doctor what the doctor at the hospital said it most likely was and he said that it could be that but brushed it off saying it was probably just menstrual pains, I m no doctor but I don t think menstrual pains are a permanent pain. He also said it could be a very small ovarian cyst which is too small to see on an ultra sound and therefore too small it worry about. Five months later I m still in the same amount of pain as when I went to the hospital. Doctor: Hello and Welcome to \u2018Ask A Doctor\u2019 service. I have reviewed your query and here is my advice. There sounds possible gravels or calculus in the urinary tract which can be radio lucent, not shown up on x-ray. I would suggest to get an IVP with CT scan for final diagnosis and further management guidelines. Hope I have answered your query. Let me know if I can assist you further."
},
{
"id": 1305,
"tgt": "Suggest remedy for conceiving",
"src": "Patient: hi,i m 25 and one and a half year married....initially i used contraceptives.But now i m trying to conceive since 14 months but didnt suceed.This month my doc recommend me frtyl from day 2nd for 5 days and memarin 0.625g three tyms a day for three days with folvite...secoundly during intercrose the liquid that pass through virgina is not passing in me.....wat should i have to do......ur advise is precious and helpfull to me.......plz help.... Doctor: Hi.Yes, fertyl should help increasing your chances of conception, but make sure that all your hormone levels are normal before the initiation of fertyl, and also most importantly make sure your menstrual cycle is regular enough to initiate the process of trying to conceive.Also, when the male partner ejaculates in the female, the sperm from the semen passes through the female reproductive tract, the fluid that leaves through the vagina later is mostly just fluid and dead sperm, this is not the entire sperm and so it does not mean the sperm did not enter.Best wishes."
},
{
"id": 37646,
"tgt": "Suggest treatment for lyme disease",
"src": "Patient: Yes I am currently looking for the top of the line in the Speciality of Lyme Disease, I currently am a white female, 50 years of age, experiencing Chronic symptoms of this condition. In 1980 at age 15 I was Dx with a serve exsposure to Deer Ticks on the Kentucky Land between the Lakes. I was hospitalized due to the extreme case. I have a long history of symptoms that have been put in the category of autoimmune issues (MS, Lupus, Etc). Today I am very symptomatic and am curious if this could be a contiributing factors to which I now suffer. I am trying to reach the best research in this field of studyThank you Doctor: Hello,Welcome to HCM,As you were diagnosed to have lyme disease which is an infectious disease caused by Borrelia bacteria and it is a tick borne disease transmitted to humans by the bite of infected ticks.The symptoms are suggestive of the lyme disease.I would suggest oral doxycycline, if you have taken this and if there is resistant to this you can take either cefuroxime or azithromycin. These medications should be taken for atleast one to four weeks.The drugs used to treat this condition is not known to cause any cancerous conditionsThank you."
},
{
"id": 73530,
"tgt": "How to treat the coughing spells?",
"src": "Patient: I have had a cough for 2 months...it started with what seemed to be a common cold. But I can't get rid of this cough...it comes in spells...It can be triggered by just speaking. The doctor gave me a scrip for cough syrup in the beginning....it hasn't worked....help Doctor: Thanks for your question on Healthcare Magic.I can understand your concern.You are having chronic cough (cough for more than 2 weeks).So we should definitely rule out lung infection and bronchitis in your case.So better to consult pulmonologist and get done clinical examination of respiratory system, chest x ray and PFT (Pulmonary Function Test).Chest x ray is needed to rule out lung infection. PFT is must for the diagnosis of bronchitis.You may need antibiotics, inhaled bronchodilators (formoterol or salmeterol) and inhaled corticosteroid (ICS) (budesonide or fluticasone).Don't worry, you will be alright with all these.Hope I have solved your query. I will be happy to help you further. Wish you good health. Thanks."
},
{
"id": 134599,
"tgt": "What causes heaviness in right side of the body?",
"src": "Patient: Hi Doctor, I have uncomfortable feeling like heaviness on the right side of the body; I have complete strength on the right side of the body but feel tightness on right side of the body from head to toe. I have had problem of acidity and am taking some anti-acidity medicine. It gets worse especially when am hungry and I feel comfortable when I eat something. It\u2019s goes and comes over and over. Can you advise any test or is this a serious medical condition. Doctor: this doesn't look a serious issue. kindly have a proper diet, proper exercise and proper sleep. if the symptoms persist kindly meet a physician who can assess you in person and look ahead for further management."
},
{
"id": 54017,
"tgt": "What are the symptoms of gallbladder problems?",
"src": "Patient: Can you advise me what would be the symptoms of gall bladder problems or gall stone problems. Would an X-Ray or CT Scan reveal either of these problems. I am currently experiencing intense pain under my left hand rib cage and some discomfort under the right hand side. I do not have diareaha or constipation and have regular bowel movements. Doctor: Hi welcome to health care magic.... I can understand your concern.... In gall stone chronic discomfort can be present at right side abdomen... Sometime it might be asymptomatic.... For it's diagnosis USG will be sufficient.. CT scan done only if needed.... If stone is there than udiliv can be prescribed.... If it leads acute cholecystitis attack in future than cholecystectomy done.... If left side pain predominant than for gastrtitis pantoprazole taken... Take careAdvise : physician consultation for examination..."
},
{
"id": 173281,
"tgt": "Is dark green stool normal after starting nan pro1?",
"src": "Patient: Hi Doctor, My son is 4 months old and from 2 weeks we started nan pro1 then after he stopped taking my milk, from few days he was having greenis..but today he had dark green color stool having little bit yellow spots and it was lil bit greasy.....Please advise what to do...Is it normal or some intestinal. Doctor: Hi...it is quite normal after starting a new formula feeding. Some times it can even result in constipation too. It seems your kid is having viral diarrhoea. Once it starts it will take 5-7 days to completely get better. Unless the kid's having low urine output or very dull or excessively sleepy or blood in motion or green bilious vomiting...you need not worry. Regards - Dr. Sumanth"
},
{
"id": 131976,
"tgt": "What could it be if leg is sore, painful and swollen after falling?",
"src": "Patient: Hi, I have a question regarding my recent accident, I slipped in the bath it was my right foot I put in and both my legs split. On the side of my leg I got a bruise it is quite big and painful and I realised it was swollen and compared to my other leg sticking out. And now it has turned red and is very painful and it s like pulling my leg. Do you know why this may be? Thank you Doctor: Hi welcome to the health care magic I can understand your concern You have to consult orthopedic surgeon without delay This could be fracture swelling or it can be compartment syndrome Local part x ray done first and if needed needle inserted and pressure can be monitored In compartment syndrome pressure within muscle compartment increases and immediate fasciotomy needed Meanwhile Ibuprofen also given for pain and inflammation relief Avoid any work and consult doctor for examination Hope you will recover fast Take care"
},
{
"id": 111993,
"tgt": "Is it possible to have scoliosis after an accident indicated by swelling in hip area?",
"src": "Patient: my 21 yr. old was in a dirt bike accident July 14, 2013 and broke his pelvis. he now has swelling around the hip area don t know why, he had a nerve conducting test and there is no nerve damage. thank god. but when he started physical therapy (only been 2x, first was eval. the next was warm up cause the bldg. had a water main break and has not been back) they said he may have scoliosis. so i don t know what is the problem. Doctor: Hello, I have studied your case history.Traumatic injury can lead hairline fracture which may not be visible on x rayYou will need X ray of spine and hip and if required MRI.Clinical examination is important to rule out any ligament injury.For these symptoms analgesic and medication can be started.Till time take medication like analgesic and use supportive splint. Do ice fomentation and take rest.Hope this answers your query. If you have additional questions or follow up queries then please do not hesitate in writing to us. I will be happy to answer your queries. Wishing you good health.Take care."
},
{
"id": 194116,
"tgt": "What is the treatment to increase the size of the testicles?",
"src": "Patient: i have small testies,they are not capable to make sperms, so what shold i do make them enlargeage 38 height 5.5 Doctor: Hi, You can't enlarge the size of testes. Find out the reason for small testis like hypogonadism.Get a direct check up with a urologist. Hope I have answered your query. Let me know if I can assist you further. Regards, Dr. B. Radhakrishnan. Nair, OBGYN"
},
{
"id": 28525,
"tgt": "Can Polyfax Ointment cure painful and swollen nose caused after ear piercing?",
"src": "Patient: I have my nose piercing done last Friday I just started having pain and swelling in my nose today I am just cleaning it with salt and water twice a day and then put polyfax ointment on it twice a day Why is this swelling and pain and what shall I do ? Shall a I stop using poly fax ? Doctor: Hello, Any type of body piercing results in swelling and pain initially and it gradually decreases and heals the wound with time with more care and cleanliness of the pierced area. Generally, when the pain and swelling last for a longer duration, there are high chances of the wound is infected. The ointment you are using contains antibiotics and usually used externally for skin infection. I would suggest you, use the ointment with your doctor's advice because if you are allergic to any contents of the ointment it will result in rash, itching or other serious side effects. Another thing excess or inadequate use can lead to undesirable results so to avoid all these things, it's always better to consult a dermatologist or your doctor before using the ointment. Also, your doctor needs to examine the pierced area for any infection because if there is some infection he might advise you some oral antibiotics for a quick recovery, analgesics, if needed, will be prescribed to you by the doctor for relief. Hope I have answered your query. Let me know if I can assist you further. Take care Regards, Dr Manisha Auradkar, General & Family Physician"
},
{
"id": 58455,
"tgt": "Enlarged liver, elevated liver enzymes. Blood test showing hepatitis A. Altered immunoglobulin levels. What does it mean?",
"src": "Patient: I have had elevating liver enzimes for past 2-3 months, my liver is enlarged. The Dr. said he could feel that it was enlarged plus an Ultra Sound confirmed it was enlarged an excessively fatty. I have a lot of pain, and blood tests just came back with hepatitis A Ab, TOTAL. REACTIVE The only other abnormal results was IMMUNOGLOBULIN A 4 L . My PHENOTYPE IS PI*MM. What does this indicate? I do not drink at all, try and eat healthy, am happily married, and 70.9 years of age. I feel fatigued and am in pain most the time. I had a completely normal ultra sound 16 months ago. Doctor: Hi and welcome to HCM.these findings are typical for hepatitis a infection and it shouldnt worry you. it should recover when virus is eliminated from your body. all this is sign of ystemic infection and liver inflammation. you should treat it regularly,avoid hepatotoxic agents and alcohol,rest and follow hepatic diet. also some medicines may be prescribed for liver protection so consult your doctor about it.Wish you good health. Regards"
},
{
"id": 112751,
"tgt": "Have back pain. Had TB earlier which was treated and cured. No medication till now. What can be the cause?",
"src": "Patient: i suffered frm tb in left lower sacroilliac joint .dr say im cured after 1 yr being on akt4 but bone being damaged i hav pain with lower back.visited physio .pain does not go.second opinion dr say it may take yrs to get normal n completely relieved frm pain..pain increase in sciatica nerve n lower back with little more stress of work..want to know how will i ever get normal? im 39 not married female.will i b able to marry and have kids? or in this condition will i fall more sick? what risks involve in future and what care shud i take?thanking you.plz guide Doctor: Hello friend,I am Dr. Vaibhav Gandhi, I am Orthopaedic surgeon and spine specialist.I have studied your case. Residual pain after TB is common ,you need to work to reduce itit may long couple of month moreExercises-Do spine extension exercises,lying on your stomach lift leg,then lift only head and chest,Do stretches and rotation of all joints Here are my other useful suggestions to you:Avoid lifting weights Sit with a support on your back. spine extension exercises.You may need to do X ray spine and if needed, MRI spine.Stress also increases back pain so I advice: Get ample sleep, good amount of water and fluids .eat fresh fruits. Tackle stress at right time, don\u2019t drive bike for long time,keep your body weight in normal range.yes you will get normal ,and you will be able to marry and have kidsyou need to maintain your general health for further query you can contact on my profile directly"
},
{
"id": 197134,
"tgt": "How to improve flexibility in scrotum after having epididymal cyst removal?",
"src": "Patient: Hello, One year I had bilateral varicocele grade 1 and and epidimal cyst. cyst was removed through surgery. However, I m still having pain in both sides of my scrotum. Sometime i feel like scrotum got congested and it is not flexible as it was earlier before sugery was made. How can the scrotum become flexible as it was earlier. Thanks Doctor: Hi Dear !! Thanks for your query to HCM.I reviewed facts of your query in context to your health issues and feel concerned about them.My Opinion-In My opinion, in the given situation you seem to suffer from-Delayed revovery of scrotal dartos function after bilateral Varicocele repair with excision of epididymal cyst.This might take another 6 mths,as the fibrotic process needs more time for it to become supple and normal.Do's Suggested-Regular oiling during baths.Tab Cymoral forte x 6 weeksDiclofenac gel locally at scrotum/ and spermatic cords on both sides.Usg Study with Colour Doppler, to assess recurrence of Varicocele with failed surgery needs to be ruled out.Consults with Urologist after USG and Doppler of Scrotal tissue and CT study of Scrotal/spermatic cord areas would fix the cause of the non-flexibility of scrotum in 1 yrs time.Other causes need to be ruled out,after getting more details from you.Next appointment-Asap to review and plan the treatment for existing problem .If you update more information/ details upon these suggestions, I would give further suggestions on medicines to correct your health issues.If need be, update any health issue 24 x 7 by a direct question to ME, at following HCM link-http://doctor.healthcaremagic.com/Funnel?page=askDoctorDirectly&docId=70229 Dear, if you don't have any further query in this regard, Do close this query with YOUR pleasing and feedback comments to rate this reply and service, to boost the morale of incoming Emergency patients like YOU, at HCM services. If you want to update more details and ask more update queries , You are most Welcome herewith !! Good Day!! Wishing Good Healthy Life in time to come!! Dr.Savaskar M.N. Senior Surgical Specialist M.S.Genl-CVTS"
},
{
"id": 217835,
"tgt": "What causes severe pain in lower central right side of ribs?",
"src": "Patient: Ive been suffering severe pain/bruising to the lower central right side of my ribs, 2 or 3 ribs feel swollen, I suffer when walking, sitting, standing, laying down, coughing, bathing an especially sleeping are issues?I woke 10 days ago with the pain, I gave up drinking 6yrs ago so I know thats not the cause Doctor: Hai. Thissort of pain, especially in the ribs I is very painful, due to the multiple vnerve innervation. Breathing is really painful.pls take good pain relief tabs like tramadol or dicloofenac, donot restrict ur breath. Go for good deep breathing exercise s, chest physiotherapy, incentive spirometry.u can apply local pain relief gel , it gives you good relief from pain"
},
{
"id": 64066,
"tgt": "What could a lump near sternum be?",
"src": "Patient: My 3 year old son has some kind of knot on his chest on the left side, its closer to his sternum and about the same level as his nipple... no bruise or anything, but it s hard and it s been there since Monday.I left it thinking it d go down, like he bumped it or something, but it s not going down but it s not red or anything. He says it doesn t hurt. What could it possibly be? Doctor: Hi, dearI have gone through your question. I can understand your concern. He may have some soft tissue tumor like lipoma, neurofibroma or dermatofibroma. Or it may be some other benign lesion. He should go for biopsy of that lump. It will give you exact diagnosis. Then you should take treatment accordingly. Hope I have answered your question, if you have doubt then I will be happy to answer. Thanks for using health care magic. Wish you a very good health."
},
{
"id": 144019,
"tgt": "What does mild mucosal thickening in the maxillary sinus mean?",
"src": "Patient: May you please explain to me what this is: Exam: MRI brain and sella with and without contrast History: hyperprolactinemia galactorrhea irregular menses Findings: the pituitary gland is slightly heterogeneous in signal. There may be a tiny, 1mm hypoenhancing lesion in the posterior pituitary gland just right of midline. Microadenoma is a possibility given the history the pituitary gland is otherwise unremarkable there is no parasellar or suprasellar mass lesion the pituitary stalk is midline The ventricles are normal in size and morphology there is no hemorrhage infarction and demyelination there is no midline shift or extra axial collection Mild mucosal thickening is noted in the left maxillary sinus Doctor: Hi, I am Dr.Bruno. I have read your question with care and understand your concerns. Let me try to help you Question : What does mild mucosal thickening in the maxillary sinus mean?Answer : For your Complaints of hyperprolactinemia galactorrhea irregular menses , the mild mucosal thickening in the maxillary sinus does not mean anything. So you need not worry about that However, what is important for you is the \"tiny, 1mm hypo enhancing lesion in the posterior pituitary gland just right of midline\" which can well be the cause of galactorrhea and irregular menses. I would suggest that you evaulate this furtherHope you found the answer helpful.If you need any clarification / have doubts / have additional questions / have follow up questions, then please do not hesitate in asking again. I will be happy to answer your questions. Let me know if I can assist you further.Take care !"
},
{
"id": 138265,
"tgt": "Suggest treatment for undiagnosed movement disorder",
"src": "Patient: I have an undiagnosed movement disorder along with motor and non-motor symptoms and have a good response to l-dopa. i experience cold hands as a warning signal that my medication is wearing off. my hands go blue to the touch. I also have tingling and numbness my arm from time to time. I am under a consultant neurologist at the National hospital in london and wondered this coldness /tingling can be treated ? Doctor: Hello,Welcome to the magical world of health care, I went through your query, and since you are responding to L dopa means you have parkinsonism. may i know what are your concerns as already u are under treatment of a neurologist.I hope my advice would have been useful, in decision making regarding your treatment, still if you have any clarifications or doubts feel free to contact back.Thanks."
},
{
"id": 57073,
"tgt": "Suggest remedy for jaundice",
"src": "Patient: I HAVE A PROBLEM OF JAUNDICE ONCE ONE DOCTOR PRESCRIBED ME TABLETS FOR JAUNDICE. AND THATS NOT EXACTLY LIV.52 CAN U TEL ME THE TABLETS WHICH ARE PREFERRED FOR JAUNDICE AND I DONT KNOW WHAT DOCTOR HAS PRESCRIBED TO ME SO I WANT TO CONTINUE THE SAME SO CAN U PLEASE HELP ME OUT FROM THIS. THANK U......................... Doctor: Jaundice is not a disease. It is only a symptom of many disease such as viral hepatitis, serum hepatitis, alcoholic hepatitis,stone in the gall bladder etc., Further rarely it may be also due to cancer of head of pancreas. Hence jaundice has to be investigated properly and the appropriate cause should be treated. Therefore kindly consult a gastroenterologist."
},
{
"id": 61599,
"tgt": "What causes a lump in the hip area?",
"src": "Patient: i have a lump that developed in my right hip to the front i am 42 year old male it is sensitive with area spazams of light to medium pain i am fearful of this should i go to the emergency room right away a little over a week its been there and spazamy type pains in that region for months now Doctor: Hi and welcome to Healthcaremagic. Thank you for your query. I am Dr. Rommstein, I understand your concerns and I will try to help you as much as I can.You should go to RR since it may be femoral hernia incarceration and it required urgent treatemnt. You should take risk for this. However, there are also some less serious conditions such as lipona, ateroma or lymph node enlargement but if this happened suddenly then this required medical attention.I hope I have answered you query. If you have any further questions you can contact us in every time.Kindly regards. Wish you a good health.DR. Ivan Rommstein"
},
{
"id": 115605,
"tgt": "What causes problem with haemoglobin level?",
"src": "Patient: my 14 year old son had a kidney transplant 4 years ago. his hemogloblin has not been good since then. he has had quite a few blood transflusion. the last one was last week 4 units of blood. and it has not been getting any better. the doctors have no idea what the problem is. but i do have a hemotologist appointment set up for thursday. As a mother I am so worried and dont know what to think. if u have any answers or advice please let me know. Nandanee shafer 815-463-0253 Doctor: Hello ,I understand your concern. I am Dr. Arun Tank, infectious disease specialist, answering your concern.It may be because of the deficiency of the enzyme erythropoetin.It is produced by the interstitium of the kidney. As your child has transfused the kidney, may be donors kidney has stopped producing the enzyme.Exact cause of the anemia can only be diagnosed after the proper laboratory investigation.But till that time you can take the blood transfusion done to maintain the normal level of hemoglobin.I advice you to take erythropoetin level of your child done before you meet at appointment.I will be happy to answer your further concern you can contact me here or you can contact me on bit.ly/DrArunWe wish you a best health at healthcare magic. Thank you,Dr. Arun Tank"
},
{
"id": 74817,
"tgt": "How to get rid of chest pain?",
"src": "Patient: Sir, My name is D.Rama Rao, 63 year old. Last year i.e Feb.2010 I am suffering with chest pain. Immediately. I went to hospital and Cath Diagnosis CAD SINGLE VESSEL DISEASE NORMAL LV FUNCTION (BY ECHO) I used the following medicines Deplatt-A 150, Cardivas 3.125 mg. Storvas 20 mg. Olmezest-40mg and Glyzid 80mg. Today My Blood Sugar Fasting 152mg/dl Fasting Urine Sugar Trace, Post Break Fast Blood Sugar 140mg/dl. Total Cholesterol 125 mg./dl HDL Cholestrol 50mg/dl, LDL Cholostrol 59 mg., VLDL Cholestrol 16 mg/dl, Chol/HDL - Ratio 2.5 and Tryglycerides 80 mg/dl. Kindly advise me thea bove medicines how long continue and is there any side effects for long time using. Thanking you sir, Doctor: Hey there,According to your reports your blood values are under control.I would suggest continue same treatment.Of course you are supposed to take treatment for longer time.Only prescribing doctor would advice you to discontinue treatment or not."
},
{
"id": 89181,
"tgt": "Are scans recommended for abdominal and back pain after falling off a horse?",
"src": "Patient: Hello, just over a week ago I had a pretty bad fall from my horse where I was bucked into the air and landed on my back! I was knocked unconscious for a few seconds but had sever pain in my lower back and butt especially to the left. I went to A&E, they gave me morphine for the pain and took a urine sample as they were worried about bruising to the kidney, there was no blood in my urine and they told me it was more than likely muscle bruising and should ease in a week! I took a week off work as the pain of walking and sitting was just too bad! However since the accident it doesn t seem to feel any better at all, I have now started to get low pains in the front of my tummy as well as still sharp burning pains in my lower back. It is very painful to walk and sit! I am also noticing abdominal pain when I go to the toilet. There has been no visible bruising since the accident and the area with the most pain (left lower back, around kidney area) is slightly numb to touch. I am taking inflammatory tablets and pain killers which do help slightly but I am in pain again as soon as they wear off! What would you suggest? No scans were done while at the hospital. Thank you for you help Jodie Doctor: Hi.With a history of a fall from the horse SCANs are certainly indicated to prove or to rule out any internal injury. Consult a General Surgeon for a clinical examination to determine which scans you need to undergo to confirm or to rule out internal injury of any sort. The further treatment will depend upon the findings."
},
{
"id": 186556,
"tgt": "What causes swelling in upper lip?",
"src": "Patient: Good morning. I woke up to some nasal congestion as I have been sick with a cold for over a week. As my sinuses were clearing I noticed a tender \"pop\" or \"snap\" like feeling. My upper lip, right side, is now swollen and numb, as if I have received a lidocaine injection for dental work. Should I be concerened? Doctor: thanks for your query, i have gone through your question. the lip swelling could be secondary to the tooth or , take care.um infection resulting in pus formation causing cellulitis or it can be allergic reaction to some drug or food items. the local anesthesia effect will be there for 1 hour after injection during that that time lip will be numb and you will be feeling like its swollen but it will not be swollen. consult a oral physician to rule out these two. if im your treating doctor i would have prescribesd a course of antibiotics and analgesics along with a antihistamine. i hope my answer will help you"
},
{
"id": 157274,
"tgt": "Got dry patch with small bumps at the back of the tongue with no pain, just a metal like taste. Have been smoking for 4 years. Is it oral cancer?",
"src": "Patient: I have a dry weird patch on the very back of my tongue with small bumps starting to go down my tongue into my throat. It doesn't hurt, just annoying. I can feel it on my uvula (dangly thing) Sometimes I can taste metal on that spot, or just an uncomfortability with it, but no pain. I just turned 20, I'm female and I'm a smoker and have been smoking for about 4 years. I am very scared if it being oral cancer, but I've been reading into it and its said that oral cancer creates with patches, and I do not have that. I think I may be dehydrated as I haven't been drinking very much lately other than pop, or it could be a deficiency in something because we haven't been able to afford much food, so I dont eat all day until supper which is usually microwavable, and afterwards just little snacks. Unhealthy ones. I know I am very unhealthy and I think that may be the cause to this. I'm only 5'4 and weigh 90 pounds. PLEASE I would be very greatful to hear back from you with what you think it may be, if I should wait untill its gone or go see my doctor ASAP. PLEASE HELP!! Doctor: HI, THANX FOR YOUR QUERY. NOTHING TO WORRY, SMOKING CAUSES DISCOLOURATION OF PAPILLA ON TONGUE AND EVEN PALATE. MAINTAINANCE OF PROPER ORAL HYGIENE , CESSATION OF SMOKING GRADUALLY , HAVING WELL BALANCED DIET ARE MORE IMPORTANT. NICORETTE CHEWABLE TABLETS ARE HELPFUL IN SMOKE CESSATION. HOPE THIS IS HELPFUL."
},
{
"id": 103118,
"tgt": "Using Aerocoat rotocaps during cold conditions at night. In summers no problem. How to get rid of it?",
"src": "Patient: Hello Doctor,I have been using Aerocoat rotocaps for more than 7 years but please note its not daily nor even monthly.I use it only during cold conditions and that too only during night time. During summer I dont have any problem.There was a year where I havent used aerocoat rotocaps at all.Please suggest me a way of how to get rid of this for ever.Thanks Doctor: Hello,You would need allergy tests (Specific IgE blood test) to figure out what your allergies are.Desensitization may then be required for seasonal only, that will help prevent the use of steroid inhalers.Also you need to get a CT Paranasal sinus scan to ensure you do not have rhinosinusitis.Best Wishes."
},
{
"id": 40667,
"tgt": "Suggest treatment for a missed period despite taking Duphaston",
"src": "Patient: I am trying to conceive but my utra sound reports say there was no follicles shown while it was done on 14th day of my periods cycle. My periods cycle is 28 to 32 days and they are never late but once in the month of june it was 6 days late then I got my periods now I have missed my period of september and still missing till it is October 18th , the strip test is negative. I went to Dr. She gave me some blood tests and gave myfol tablet and duphaston she said if there will be a baby it will protect it but if there is nothing then my periods will come so I want to ask you how many days would it be taken by these tablets to get back periods because I am still having no periods and delayed my tests due to some reason . I am very confused to know that instead of having myfol tablet for five days my periods did not come yet ? Is it because of pregnancy ? Doctor: Hi there, I will suggest you the best possible treatment options.Please do not worry. Your treating doctor has rightly suggested you to take Duphastone tablets. I will suggest you to take them for ten days. If there is no pregnancy, you will get your periods within 3 to 10 days after finishing the tablets. Myfol tablets I'd Folic acid supplementation, it has nothing to do with your periods. Please continue Folic acid and vitamin B 12 supplements. It helps to prevent various complications during conception, pregnancy and delivery .I hope this answer helps you.Thanks.Dr. Purushottam Neurgaonkar"
},
{
"id": 66485,
"tgt": "What causes a lump on fractured metacarpal with the cast?",
"src": "Patient: about a week and a half back i had a fracture to my fifth metacarpal and a cast back slab was installed. yesterday had if remove for cleaning and found that their is a huge lump feel like bone and it still swollen and very painful when touching it. is the bone set right and hear noise grinding moving my small finger back and fort. Doctor: Hi ! Good evening. I am Dr Shareef answering your query.Usually this kind of metacarpal fractures heal well with splintages. However, in case you are getting pain along with a swelling, it could be a major displaced fracture, and might need an expert opinion and management by an orthopedic surgeon with whom you should take an appointment to get it fixed. Till then, you could go for an anti inflammatory drug along with a proton pump inhibitor accompanied by elevation of the hand.I hope this information would help you in discussing with your family physician/treating doctor in further management of your problem. Please do not hesitate to ask in case of any further doubts.Thanks for choosing health care magic to clear doubts on your health problems. I wish you an early recovery. Dr Shareef."
},
{
"id": 97507,
"tgt": "Is homeopathy treatment advisable for poly cystic ovaries?",
"src": "Patient: Good evening I am 35 yrs married male - married since last 16yr. waiting for child, my wife have Poly-cystic ovarian disorder treated and getting periods not regular.we have done IVF- but failed. My friend suggested that i should take homeo treatment before next cycle, so my question is shall I get help from homeopathy Doctor: Welcome to HCM This is Dr Suchda , NATUROPATH - HOMEOPATH- MAGNATOTHERAPIST going through your query to help remove your problem . I can under stand your Concern & worry is that 'even after 16 years of your marriage, your wife could not conceive IVF treatment also failed . Though it is matter of concern ,even then I will suggest you to have patience & take care of your physical as well as mental health as well as your wife's .To increase the chances of conception, IVF should be done at least 3 times. The chances of conception are 25% to 30% first time, 30% to 45% second time and approximately 60% third time. Before that ,I want to suggest you as Below , if it helps Both should do few Yogic exercises daily-- Do butterfly , Kandharasan , Pavanmuktasan ,Dheerg - Noukasan ,Gomukhasan , Bhujangaasan Shalabhasan till Conception . Balasasan for 4-10 seconds in between every Asan in essential to recoup & 5 mins Shavasan for rest .Take Guidance from expert if any doubt .Also do pranayam -Inhale -hold -exhale - hold deep breathing , Kapalbhatti don't ever over do . Keep Calm and don't take unnecessary tensions . Include in your food , Vitamin E in the form of almonds ,eggs ,fish oil . take wheat , black gram , almonds ,eggs ,fish oil , ginger , ,lettuce ,radish cooked with leaves ,milk,refined butter, mango , melon .Out of these , take as many as you can in a day . some of these you include in BF , some of them include lunch and some in dinner .And ,take water prepared in copper glass twice every day . fill water in copper glass and keep over night , drink it in the morning . Again fill and keep it for 12 hours and drink in evening , Carry on this regime for 3 month or so . Lady to take Homeopathic Medorrhinum 30 / 4 pills daily for 4 days / after that Weekly for 6 weeks you will see the result . God willing . Leave rest to godKeep your specialist aware of situation now and then .Don't hesitate to get back if have any further query All the best . God bless you"
},
{
"id": 8765,
"tgt": "How are small green pus filled boils on the shoulder and stomach treated ?",
"src": "Patient: I have had a very tiny green dot on my left shoulder for at least 5 years now & now i see one near to my belly button the same size. the one on my shoulder used to be slightly raised & i thought it was a small flat bump at first. If you squeeze it hard it stays in place like regular skin would if you pinch it but then a very thick small amount of white discharge comes out. The smell is unexplainable. It smells really bad. The white discharge comes out but the green dot stays out. Only a little discharge comes out & if you squeeze it later in the day youre likely to not get anything out. The one on my shoulder has gotten smaller over the years though it was never been noticeable. The one on my stomach does not give out any discharge. Well, it hurts to keep trying after ive pinched too hard. I would like to know what it is. It is very weird. It has the worst smell ever.Its not doing any harm to me just a smelly little dot on my skin. Doctor: Dear PS, thanks for visiting healthcaremagic as per your history it seems to be sebacious cysts and you need to visit your plastic surgeon for getting them removed with minimal scar. however better judgement about your disease can be made only after examining youe pics if you can post some.."
},
{
"id": 34485,
"tgt": "Can IVF treatment be undertaken when suffering from TB?",
"src": "Patient: My name is Poonam. My age is 36 and weight is 58 Kg. I have taken ATT treatment at the age of 32 years. Now, I am going for IVF treatment from AIMS Hospital. Doctor advised to go for Blood Test of various Tuberclosis tests but they were negative. Then as per the advise of the Dr., II went for PCR for M. Tuberculosis (Sample-EA) from AIMS Hospital. The result of this test is Positive. Now, sir, please advise me whether to go for IVF Treatment or first to take course of medicines for Tuberclulosis treatment? Doctor: Hi Poonam, i will be answering your question, hope this will be helpful to you.First you should finish the ATT and then go for IVF, i will not suggest you to undergo IVF while you will be taking ATT as there will be risk of failure of IVF or else there might be some defect due to ATT.Take proper nutritious diet and take medicines at time. dont miss ATT, even a single tab.Get well soon.Thanks"
},
{
"id": 130149,
"tgt": "Is it normal to have discomfort while jogging for bruised ribs?",
"src": "Patient: Hi, I was in a car accident 4 months ago and bruised my ribs. The xray performed by the ER showed no broken ribs or punctured lung. So the doctor said they are probably bruised. After 4 months, I still feel a noticeable discomfort whenever I jog, lay on my side to sleep, run, etc. Is this normal for bruised ribs? Is there any steps I can take to lessen the discomfort or heal the ribs? Thank you Doctor: Hi there must have been some tissue damage, without a fracture that has not yet healed. Sometimes it can take a long time because the chest cannot be strapped like other parts of body. However you can try gently massaging anti inflammatory cream locally to help healing--regards"
},
{
"id": 108215,
"tgt": "What is the solution for severe pain in back, hip and groin?",
"src": "Patient: I m a 62 year old woman and having severe pain in back, hip, groan area, Have been told I have pelvic fractures , sacram fractures, I have had MRI, ct scans, blood work, dexa scan, bone scan, been close to a year, I m taking blood pressure meds, amlodipine prinzide, need answers, why I can t get believe, thanks Doctor: HIWell come to HCMAppreciate thatWhether these are the really fracture and confirmed or just suspected these, if these are the fractures then these need to be managed accordingly or if this is just query then this need to be ruled outAdvise: pain can be managed with NSIAD (Non-Steroid Anti-Inflammatory Drug )Treatment: Try to low the stress level Medication;Tab Ibuprofen 400 mg three times in dayWhat ever the clinical condition is there you suppose to face this and if you did this soon you would be out of this conditions, hope this helps, take care."
},
{
"id": 5602,
"tgt": "Planning pregnancy, started bleeding but the color is brown. Is it periods?",
"src": "Patient: I had my period on the following days for the last 5 months: 12/11/2012, 13/12/2012, 13/01/2013, 11/02/2013. I am trying to fall pregnant. We had intercourse before my ovulation date and on ovulation, but I started bleeding on 12/03/2013. I am a bit confused as I thought I would definitely be pregnant. The blood changed during the day to a dark brown discharge . Today I still have the brown discharge. Had very light cramping on the first day and did a hpt , but it showed negative result. Is this just my period I m having? Did I maybe do the test too soon? Doctor: Hello. Thanks for writing to us. The bleeding that you are having is likely to be due to your period itself. Since the urine test for pregnancy is negative, pregnancy is unlikely. You can be more sure by getting a blood test for serum HCG levels done. I hope this information has been both informative and helpful for you. You can consult me again directly through my profile URL http://bit.ly/Dr-Rakhi-Tayal Regards, Dr. RakhiTayal drrakhitayal@gmail.com"
},
{
"id": 148057,
"tgt": "What is the treatment for stiffness in my neck and headache with a history of back pain and disc problems?",
"src": "Patient: On Friday I fell down some icy steps, I hit my back, the left side above my hip and landed on my left arm pretty hard. I was holding my little girls hand so I tried to not hurt her, which made me fall awkwardly. I have had severe pain since, but it's gotten worse. My neck is stiff and I have a headache that won't go away. In worried that maybe I should go see a doctor, but I'm not sure if this is normal? I have a history of back pain due to disc problems and degenerative arthritis, so I am not new to pain, however the fatigue and headache are worrying me. Doctor: HIThank for asking to HCMI can understands your problem, you must be developed soft tissue inflammation because of fall, if I would be your doctor then I would treat you with the following treatment,1) Tab. Ibuprofen 200 mg three times in day 2) Tab. Acetaminophen three times in day I hope you would feel much better with this drugs, no need to worry about this take care and have nice day."
},
{
"id": 91489,
"tgt": "What is the cause and treatment for pulsating sensation in abdomen?",
"src": "Patient: Pulsating sensation in upper abdomin I have been feeling a strong pulse in my left side. It is located right next to my left rib cage. It comes and goes, and does not hurt. I also have noticed a since of fullness and have been very bloated. I have a small pain when I lay on my back. Its more uncomfortable than anything. I have not had any sort of appitite at all. My obgyn told me about a month ago that I had IBS, however, I have never felt this \"pulse\" before. When I put pressure on the pulsing spot I can feel that it is pulsing at the same pace as my heartbeat. Occasionally I feel like it moves, but not far, just a slight movement. Is this something I should be worried about? I have read up on blogs, from people with the same symptoms, and have started to worry. I just want to know if I need to see a doctor asap, or if this can wait till my next check up. These symptoms have only been noticeable for two days. Doctor: HI. This can really be a serious matter. This can be due to transmitted pulsation- meaning there can be a lump , which is sitting on a normal vessel and transmitting the pulsation / The other possibility is a lump developed from the blood vessel itself. You need an urgent color doppler study and an admission to the hospital."
},
{
"id": 83860,
"tgt": "What are the side effects of normet tablet?",
"src": "Patient: I started on NORMET tablets due to an intestinal disturbance and mild fever. Have had two doses so far. Am experiencing a lot of discomfort and tightness just below the sternum. Have had antacids and donstal but does not seem to give any relief. Is it a side effect of normet or something else? Doctor: Hi, Common side effects of normet include stomach upset, abdominal cramps, metallic taste, heartburn, and flatulence. Based on the history you seem to have developed gastritis apparently caused by this medicine. I suggest you to take a proton pump inhibitor for the treatment of gastritis however if the symptoms persist consult your doctor to rule out other causes. Hope I have answered your query. Let me know if I can assist you further. Take care Regards, Dr. Mohammed Taher Ali"
},
{
"id": 81696,
"tgt": "Need medication for pain & pulsation just under the ribcage",
"src": "Patient: I have been having pulsating just under my left ribcage towards my chest it doesnt hurt but slightly takes my breath away its been going on for a week but getting more frequent each day ive taken buscopan but thats made no difference should I be concerned. Doctor: Thanks for your question on HCM. Left sided chest pain with feeling of pulsation is seen mostly in cardiac cause.So we need rule out cardiac cause first. Get done 1. ECG2. 2D Echo To rule out this.If both are normal, then no need to worry for cardiac cause. Anemia can cause chest pain and palpation. So get done CBC ( Complete Blood Count ) to rule out anemia. If this is also normal than mostly you are having anxiety related chest pain and palpation. So avoid stress and tension. Be relax and calm."
},
{
"id": 21131,
"tgt": "Is the blood pressure normal despite headaches and dizziness?",
"src": "Patient: im 25 years old female my weight is 151.i have painfull headace and i feel dizzy. i found out my blood pressure is 109/69 is it normal or little low Doctor: hellothanks for posting here. your blood pressure readings are absolutely normal. please dont worry about it.but the cause of your complaints should be determined. you can also have migraine. issues. please consult your doctor about the same. thank you"
},
{
"id": 102114,
"tgt": "What is the medication for allergy related asthma?",
"src": "Patient: I have allergy related asthma, girlfriend bought a horse and I get really short of breath almost gasping if I go near her or the horse, I have an inhaler but is there any other medications that could help me more as the inhaler only works for a bit then seems to stop working? Doctor: anti allergic medicine the combination of fexofenadine and montelucast is besttwice a dose is sufficient to control the disea you can take for long also without side effectsfoe permanent relief you can opt for sublingual immunotherapy after consulting allergeologist"
},
{
"id": 139435,
"tgt": "Should sexual life be avoided for the lumbar spine stenosis?",
"src": "Patient: Dear sir, I am 58 yeras old,weight 95kg; height 1.65m, Type 2 diabetic patient since 15 years, but under control. I am having lumbar spine stenosis at three levels, by doing special exercises I don t have back pain but my doctor advised me not to have intercourse in future. My penis size is 1.5 inches, when erected 3.5 inches without any hardness and immediately gets ejeculation at this loose stage when I try to enter in vagina, thereby cannot satisy my wife any more. I am working in Abu Dhabi, UAE since last 30 years and used to go to India on vacation once in every two years. Last week I rejoined duty and my problem still continues even after taking soft VIAGRA - 100 mg. pills many times before each intercourse and sometimes other unani herbal medicines, but of no use. Please advice. My doctor says because of my age, weight, diabetes and lumbar stenosis, this problem will never be solved permanenetly. Kindly help me to solve this long lasting problem once for all. I request you to write me in detail. I appreciate your quick response in this regard. Thanks. Doctor: Hi, Well if your power of will to find a solution then with personalized therapeutic plan under close medical monitoring you should be able to overcome the issue. This won't be easy. This requires long detailed therapeutic plan to help. I will gladly give a short glimpse of such a plan. Feel free to contact me for detailed plan and step-by-step action. First, your first goal would be to lose weight. It is easier said than done. This can be done using caloric deficit, that will result in weight reduction without suffering. Second, you need to include daily exercise. I know this will be hard due to the stenosis. At start this could be simple gradual exercise plan. In a period of 1-3 months you will start noticing difference. During this period, If I were your treating physician I would prescribe you Metformin, medications that can help you will premature ejaculations. I would monitor your testosterone levels closely too. Prescribe supplements if needed. From what I read, It is great that the stenosis pain responded to physiotherapy. With aforementioned therapeutic intervention you will not only add years to your life but also quality to your life. Hope I have answered your query. Let me know if I can assist you further. Take care Regards, Dr. Ahmad Nazzal"
},
{
"id": 134792,
"tgt": "Suggest remedy for sharp pain in arm",
"src": "Patient: I have pain in my left arm,feels like muscle when I rotate my arm to the right and lift up its sharp. Shoveling snow today didn t help and now its a steady throb. not sure if a xray will help or if I m going to have to have a mri . it s been going on for at least a month or longer. Doctor: helloYou must attend a orthopedic clinic for proer examination and investigations.it could be supraspinatus tendinitis or biceps tendinitis (inflammation of tendons which attach muscle to bone) or even shoulder joint problem.Do hot fomentation and take advil tabs after meals thrice a day and rest arm in a broad shoulder-arm sling till you attend orthopedics clinic.A weeks rest to 10 days would be good in sling alongwith medicines.The final treatment can be decided after examination and xrays if necessary, MRI ,if needed onlythanks"
},
{
"id": 175623,
"tgt": "Is aggressive behavior a symptom of autism in a 3 years old child?",
"src": "Patient: HI Doctor Samuel, My daughters foster son displays aggressive behaviour and I wonder if he could be autistic.He screams at my daughter in the mornings to wake up and if she doesn t respond quickly he urinates on the floor.He throws things at her 12 year old daughter and he gets upset if things are not done in the same order.He is three years old and both his biological parents have mental problems. Thankyou Valerie Hille Doctor: DearWelcome to HCMWe understand your concernsI went through your details. Aggressive behavior pattern may not be necessarily autism. There are so many childhood behavioral disorders which highlights aggressiveness. You need to consult a child psychologist who will be able to diagnose the problem properly. If you require more of my help in this aspect, please use this URL. http://goo.gl/aYW2pR. Make sure that you include every minute details possible. Hope this answers your query. Available for further clarifications.Good luck."
},
{
"id": 188082,
"tgt": "Is tongue yellow due to swine flu infection?",
"src": "Patient: Good Afternoon, I was in the Defence Force 2007-2013. When I got chosen to go on deployment overseas to Iraq, a pre requiset was to have a H1N1 Swine Flu Vaccination, I didn t eventually go to Iraq. Vaccinated November 2009. 6 Months Later April 2010 - Admitted to Base Hospital Symptoms Sleepy, Ears Blocked, throat sore, cough, runny nose, lethargy, diziness, allover body limb soreness, gastrointeritis, diarrhea like a tap, vommiting, dehydrated, cold shivers with a high fever and eyes are light sensitive. Cannot keep down food or water. Administered pain relief paracetomol and had to consume water or be placed on a drip. Discharged after 2 days. March 2011 - Admitted to Base Hospital Symptoms Sleepy, Ears Blocked, throat sore, cough, runny nose, lethargy, diziness, allover body limb soreness, gastrointeritis, diarrhea like a tap, vommiting, dehydrated, cold shivers with a high fever and eyes are light sensitive. Cannot keep down food or water. Administered pain relief paracetomol and had to consume water or be placed on a drip. Discharged after 2 days. May 2011 - Involved in a car accident out the front of base by a negligent driver, sustained some physical injuries and mild concussion. I was rear ended at 80km/h, my car was stationary. Taken back into base hospital for observation and released later that day. October 2011 Diagnosed by sleep physician to have Narcolepsy Cataplexy and started tratment with Modafinil, which I felt all side effects of drug. Later changed to ritalin with better results, up to 50mg a day if required (10mg tabs). May 2012 Admitted myself to public Hospital after midnight with an all over body rash. Red in colour, rash was risen a few millimeters above the skin like a welt, was that itchy couldn t stop scratching it, started only as 2 small circles on my stomach, 4 hours earlier. Was given a anti histamene and allowed to sleep for 6 hours. Rash settled down, inflamation dropped back to skin level and was released from Hospital. October 2013 Medically Discharged from Defence Force December 2013 Saturday 14/12 /13 Symptoms When tongue is flared out, it has an all over yellow/ like puss apperance also, Sleepy, Ears Blocked, throat sore, cough, runny nose, lethargy, diziness, allover body limb soreness, gastrointeritis, diarrhea like a tap, vommiting, dehydrated, cold shivers with a high fever and eyes are light sensitive. Cannot keep down food or water. Only starting to come good today, Mon16/12/13, but still have headache and diziness, vomiting and diorrhea has stopped for now. Questions 1. Cause my Narcolepsy/ Cataplexy symptoms started in 2010, is it possible that the H1N1 injection was to blame? I say this because I had not complained of being tired or falling asleep after consuming a can of coke etc, prior to NOVEMBER 2009, I was in the best shape of my life!!!!!!! 2. Is it possible that I am a carrier of or have caught the Swine Flu through the Imminisation H1N1 injection, as both symptoms for the Narcolepsy/Cataplxy & Swine Flu have very Parrallel running symptoms? 3. Why is my tongue yellow when I put it out of my mouth and widen it, (flex the muscle, puss yellow). Doctor: Hi,Thanks for asking the query,The various symptoms of swine flu includes fever, chills, coughing and sore throat.I would suggest you to get a thorough physical and dental examination done.A thorough clinical examination is required for appropriate diagnosis & treatment.At home take multivitamin suplements and lukewarm saline rinses.Hope this helps out,Regards..."
},
{
"id": 8971,
"tgt": "Why is there swelling on my face after using neutrogena face wash ?",
"src": "Patient: Hello doctor , A couple of weeks before I went to a dematologist for acne treatment.He adviced with to apply Faceclin-a gel for 2 weeks,He told me to dilute the gel with Cetaphil moisturising lotion and lessen the dilution gradually.First week was fine,my skin became clearer and acne also disappread but in the second week, twice or thrice i used neutrogena face wash and in the next morning my whole face turned Red with bit swelling. I stopped using neutogena face wash as i felt it could be coz of that.Now swelling has disappread but the completed face ia still red and darkened specially the area around the lips is darker than the rest of the face. Kindly advice whether i should discontinue using faceclin OR please advice any medicine. Thanks. Doctor: Hi, the color will persist for some time and there is nothing to worry about it.I will recommend you to stop using chemical for acne.Use natural methods to treat acne.Drink lots of water and eat food containing more fiber and fruits.Apply honey on your face to remove acne and black color. conact me dr.riyaz@ayurwellness.in for more natural treatments for your acne and pimples"
},
{
"id": 100964,
"tgt": "How to deal with seasonal asthma?",
"src": "Patient: i am 21 years old weight 45 kg height 5'3\" & i have problem of seasonal asthma since 5 years and taking seroflo inhaler when attack precipitates. i want to ask whether it is harmful to take only inhaler inhaler whether i take any other medication or not Doctor: Hello.Thank you for asking at HCM.I went through your history and would like to suggest you as follows:1. Basically seroflo is a \"preventer\" type of inhaler. As you have seasonal type of asthma, I would suggest you to start regular Seroflo inhaler 2 weeks before anticipated bad season and continue it daily during the same season.2. In my patients, I prescribe salbutamol/levosalbutamol inhaler (these are \"rescue\" type of inhalers) to be used on as-and-when-needed basis at time of symptoms of asthma (wheezing, breathing difficulty, etc)3. I would also suggest you to start taking montelukast 2 weeks before anticipated bad season and continue it during the same season. This may reduce your requirement for inhalers.4. Were I treating you, I would advise you allergy testing to identify the substances you are allergic to and avoid them. Avoidance is the best strategy for allergies.5. Please try to avoid exposure to dusts, smokes & air pollution as much as possible.6. Regular breathing exercises and a healthy diet rich in vitamins & minerals (adequate amounts of green leafy vegetables, fruits, sprouts, etc) will improve your lung capacity and immunity respectively and will help you in a long run.Hope above suggestions will be helpful to you.Should you have any further query, please feel free to ask at HCM.Wish you the best of the health.Thank you & Regards."
},
{
"id": 87617,
"tgt": "What causes abdominal pain and watery stools?",
"src": "Patient: I'm recently experiencing a stomach ache, and my shit is watery. This has been happening for three days now. I'm guessing this will stop eventually but I'm just wondering if there's a way to make mayself feel more comfortable while experiencing this. Also, is there a proper way of saying 'watery shit Doctor: Hi,From history it seems that you might be having gastro-enteritis giving this problem.Take antispasmodic medicine like Meftal spas.Take Ofloxacilin, tinidazole combination medicine for 3 days.Take light diet like curd, rice, butter milk and apple, orange juices.Take plenty of water.Ok and take care."
},
{
"id": 180066,
"tgt": "What causes vomiting with diarrhea?",
"src": "Patient: My son has been up all night vomiting and a very smelly watery discharge, in the nappy. Was keen on food this morning but soon after brought it up.... he is having trouble vomiting at times...prob due to an empty stomach.....what can i give him if he will not drink the electrolyte drink from the doctors? Doctor: Thank you for asking querry.It seems that he has acquired infection from somewhere.For vomitting you can give him ondensatron half hour before feeding.Please let me know weight of the baby so dosage can be told.You can give him lime water,lassi but no coldrinks.If the vomitting is persistent,not taking orally,not passing adequate urine then he may need iv fluids.Hope you find this helpful.Rate the above answer if you like it.Regards,Dr rushikesh kute"
},
{
"id": 217138,
"tgt": "Suggest treatment for pain and discomfort on the neck",
"src": "Patient: my 80 yr old mother caught a crick in her neck this morning. she is diabetic, has high bp, and was given a 600 mg ibuprofen for the pain/discomfort in her neck. an hr in she started feeling loopy. 2 hrs later upon arriving home from an out of town trip, safer to be in ur own state if we had to take to the hospital, we checked her bP and bs. bp was over 200/109. we started out for the urgent care unit and she seemed to begin feeling some better. we decided to bring her back home and monitor her, hydrate her and have her rest. recent checks of bp still shows elevation. the latest, 191/90 pulse 93. should we take her to the urgent care unit... Doctor: Here I suggest for blood pressure you talk to your doctor and take some extra medication for lowering it. Ideal is 120-130/70-80. As per her age and recent fall. If she is able to maintain it after taking some rest and next dose of medication for BP then its okey. No need to take her to urgent care.Now for her neck pain I suggest to give her hot pack frequently for 15 minute each time with light heat. And post use of hot pack apply some pain relieving ointment. If this together give her relief then its fine. If not then you need to visit a orthopedic or physiotherapist take x ray and see there is nothing wrong. Here the doctor will guide you further for medication for pain and inflammation and will also suggest for physiotherapy treatment. Physiotherapy treatment will help to give relaxing and healing effect in soft tissue and will help further for pain relief.At any time if she complains of headache and not having control on hand or leg with high blood pressure then I suggest to shift her for emergency care unit at hospital. Till then nothing to worry take care."
},
{
"id": 127465,
"tgt": "How can a painful heel spur be treated?",
"src": "Patient: I have had a painful L heel spur for over 3 months; can t get into see my podiatrist till first week in June. Trying different shoes and supports and even orthotics isn t helping! So unable to do my fast walks as before! I am also dealing with bulging discs in lower back so the two really don t go together but create more pain. What do you suggest? Doctor: Hello and Welcome to \u2018Ask A Doctor\u2019 service. I have reviewed your query and here is my advice. Heel spur can cause intense pain. Since you have tried almost all non surgical measures, it is better to go for surgery now. Nothing to worry as it is a simple procedure and complications are very minimal. You can consult an orthopedic and he will explain the procedure in detail. Hope I have answered your query. Let me know if I can assist you further."
},
{
"id": 29880,
"tgt": "Suggest remedies for the following results of a urinalysis",
"src": "Patient: What to do if the urinalysis result have pus cells of more than 100? What kind of antibiotic may prescribe? Is it highly dangerous? She don't have a fever but warm to touch temp of 36.5\u00b0c. Can we suggest to the attending doctor to perform.KUB exam.? Doctor: Hello!Thank you for writing . I understand your concern . Does she urinate frequently? Does she have a burning sensation during urination ? From the urinalysis it seems a low urinary tract infection .If she doesn't have signs it is called asymptomatic bacteriuria. The most common cause is escericia coli. The most common treatment is nitrofurantoin.You can do uroculture to find the type of bacteria and among this the type of medication .Best regards ."
},
{
"id": 195031,
"tgt": "Is it normal for the foreskin to not retract over the glans penis?",
"src": "Patient: I am 22 yrs old. My foreskin doesnt retract over glans. If i do it forcefully, i am able to stretch it till half of the glans, when i leave it, returns back to the tip of glans. Is it normal? Will this affect me later in life during intercourse? Please help me. My friend had tje same things. He did circumcision. Is there any need need to do it? Dont we have any non surgical methods? Should i forcefully stretch it? Doctor: Hi, You have to apply significant force to retract foreskin over glans. Hence it is suggestive of phimosis condition. For that topical steroid can be prescribed which is helpful in relieving mild phimosis. But if problem not solved then you have to undergo circumcision operation. Circumcision is the minor surgical procedure and doesn't worry about that. I suggest you consult the urologist for an examination. Hope I have answered your query. Let me know if I can assist you further. Regards, Dr Parth Goswami, General & Family Physician"
},
{
"id": 92535,
"tgt": "Misdiagnosed ulcer resulted in sepsis, bleeding from rectum, tenderness of ribs, painful. Why is this happening?",
"src": "Patient: I had mis diagnosed ulcer 2008 resulted in sepshis had 7 lapootimies near death survived long itu stay now have bleeding from rectum very tender under right rib alot of painful episodes now thickening found on the stomach wall will see gastric surgeon soon also going for endoscopy soon what could it be be honest prepared for worst thanks so much Doctor: hello there! i read your query and i want you to know that Misdiagnosed ulcers lead to perforations and pain under right hypochondrium means it involves duodenum biliary tract with or with out secondary pancreatic involvement.As you already have sepsis and alot of surgical procedures so there are alot of adhesions and post surgical unavoidable complications making your bowel very sensitive and explains all the symptoms you are facing.rectal bleeding needs work up.Colonoscopy would be a good start with complete blood panel.Endoscopy for gastric mass is to rule out any gastric malignancy odds as they increase in cases like yours.If God forbid there is some malignant mass it would be better to intervene early. So thats what all this plan is about. Hope for the least try for the best and get prepared for the worst. I am sure you will be fineTake some special care of yourselfregardsDr S Khan"
},
{
"id": 38947,
"tgt": "Does staph infection cause seizures and ostiomolitis?",
"src": "Patient: My husband had a grand mal seizure in April, 1999 and was brought to the hospital. He was let go with them telling him is will never happen again and that he did have an ear infection. He was given antibotics. In July 1999 he go ostiomolitis in his right clavical and was given a pick line for 6 weeks. In November 1999 he had another seizure. He has had 2 surgeries on his ear drum that was blown out playing hockey during high school. His seizures continue and so has his draining, ringing and poping in his right year for the last 13 years. The ear doctor finally did a biopsy on the drainage that is behind his ear drum. It is a staph infection. Could this infection be the main cause of all his other problems? Doctor: The staph infection could have caused the osteomyelitis. It's unlikely to cause the seizures, unless he's developed an infection or abscess in the brain. However, he would be having other serious problems in that case, so I don't think that's likely. If he is continuing to have seizures, he could have epilepsy. Has he had a workup for a seizure disorder? Other problems in the brain, such as scarring or damage from a blood clot, could result in seizures. The best specialist to see for the seizures is a neurologist. They can evaluate for possible underlying causes and initiate appropriate treatment for a seizure disorder. Hope this answers your query. If you have further questions, I would be happy to answer them."
},
{
"id": 146862,
"tgt": "How to treat excess spinal fluid on spinal cord?",
"src": "Patient: My daughter is 16 with back pain, hip pain and some numbness in feet and hands. MRI shows excess spinal fluid on her spinal cord. She also has a vertebra that that did not separate. We are being referred children s hospital. Just want to know what could be wrong and what to ask dr. Only lower back MRI was done and X-ray Doctor: Hi, welcome to our site. I am Dr Saumya Mittal, MD.Read your query. That is a very significant question and i appreciate your problem. I will try my best to answer your queryI will suggest an MRI Brain with spine screening as well. Here is why-The fluid is produced in the brain. Ideally it is meant for the cushioning effect. In certain conditions called hydrocephalus, 1 of the 2 things may happen (Or both)-1. the fluid produced is too much2. the fluid is not sufficiently absorbed backThere is a possibility that the same high levels of fluids may be present in the brain. Now you have said she has upper limb issues as wellSo there may be a problem in her cervical bones too.So an MRI brain with screening of spine would be my choice of investigationI hope this helps you. Inform the reports mentioned above/if any other so I can be of help further. I have given you the answer to the maximum considering the information provided. The results of the tests could further enhance my answer to you.Please do understand that some details could be extracted from a detailed history and examination.Please feel free to ask another query. I would be glad to help you. Looking forward to your return query with the details asked so that I can help you further. (If the answer has helped you, please indicate this)Best of luck.Dr Mittal.MBBS, MD (Internal Medicine), CC (Diabetes Mellitus), DNB (Neurology)Consultant Physician and DiabetologistJS HospitalEx Apollo Hospital, DelhiEx Kailash Hospital, Noida"
},
{
"id": 110155,
"tgt": "Suggest treatment for lower back pain",
"src": "Patient: i wonder if you can advise? i have a long history of back problems. i have been a midwife for 10 yrs and recently changed working areas after being placed in an area with reduced clinical input due to my back. in april i was told i had to return to the ward area and by may i was again experiencing severe back pain, leg pain and what i can best describe as sharp toothache continually. i have not slept in months and the medication doesnt work if i done have enough and when i take it i am in the equivalent of la la land and cannot function. i had an mri last week and will find out what the results mean in 2/52. the report says as follows, what would this mean? i am struglling on the ward area and in antenatal clinic with partially clerical duties i have coped well with no problems in 3 yrs. the report says Clinical history: Previous discectomy in 1997. Recurrence of severe low back pain in May 2010 with right lower leg symptoms and suspicion of altered bladder function. Sequences: Sagittal T1 and T2 weighted images of the lumbar spine have been acquired with axial T1 and T2 weighted images. Post gadolinium sagittal and axial images have also been acquired. For the purposes of this report the L5/S1 disc is assumed to be at image 701-03. Findings: The L1/L2, L2/L3, L3/L4 and L4/L5 discs are within normal limits. Mild facet joint hypertrophy is seen at L4/L5 but there is no evidence of significant canal or neural compromise at this level. At L5/S1 the disc is degenerate with significant loss of height. There is a small annular disc bulge and there is minor narrowing of the lateral recesses but no evidence of significant root compression. No significant canal narrowing is seen. The visualised spinal cord returns a normal signal and the conus terminates at L1. Conclusion: Degenerate disc at L5/S1 but no evidence of significant neural compromise. please would you be able to explain and even advise what a reasonable plan of care, treatment and options i may consider. many thanks denise Doctor: HiThank you for asking HCMI have gone through your query.Your MRI gives impression of intervertebral disc herniation with out neural compression.For such problem in patients i treat with NSAIDs like aceclofenac and muscle relaxants like cyclobenzaprine and neurotropic drugs.I also send them for regular physiotherapy and muscle strengthening exercise along with this.Wearing lumbar supporting belts also will be helpful.Hope this may help you.Let me know if you have any further query."
},
{
"id": 115765,
"tgt": "Suggest remedy for large RBCs and tingling sensation from head to toe",
"src": "Patient: my red bllod cells have been large throughout pregnancy my daughter is 17weeks old and still have large blood cells.B12 ad Folate levels are fine I feel weak and its getting worse sometimes a tingling sensation from head to toe just had more bloods took to check my liver function .I am breastfeeding and do not have any alchohol Doctor: Hi, dearI have gone through your question. I can understand your concern.There are many causes of large rbcs. B12 or folic acid deficiency is common cause. Alcoholism or liver disease may leads to large rbcs. Go for complete liver function test. Many people have large rbcs without any known causes. You should also check your sugar level. Consult your doctor and take treatment accordingly.Hope I have answered your question, if you have any doubts then contact me at bit.ly/Drsanghvihardik, I will be happy to answer you.Thanks for using health care magic.Wish you a very good health"
},
{
"id": 167798,
"tgt": "What are the effect of hip replacement surgery in child?",
"src": "Patient: Dear Doctors, there s an 8-year-old girl with arthrogryposis diagnosis. She needs hip surgery, but many doctors say it should be done after she completes 18 years. Only one Doctor said it must be done immediately. Her parents are hesitating whether to risk or not. Please look through her web-site ( WWW.WWWW.WW and give us an idea. Contact Boghos at YYYY@YYYY . Thanks in advance. Doctor: pwrioperative managment is difficult in patients with arthrogryposis and that because there are certain difficulties with venous access , pulmonary care post operatively, cervical instability , and intra operative hyperthermia .benefits and risks should be weighted after complete physical exam by her health care personel, because early surgery between age 3-12 can be helpful in later stages . I hope this helps"
},
{
"id": 108538,
"tgt": "What is the recurrent lower back pain?",
"src": "Patient: I have been getting aches in the bottom of my back for a period of time on and off it seems to happen when I become thirsty its the feeling of the muscle in my lower back( or what ever is causing the problem ) are tensing and it causes me pain , Iam a female but It never happens on my period or near my period , I also suck in my stomach alot because i skip breakfast and it make really loud noises when Iam in work i think this may add to the problem , Can anyone tell me what might be worng with me ? Doctor: HelloLow back ache may be due to many reasons like genito-urinary causes,pelvic causes,musculoskeletal reasons etc.You need proper clinical examination and routine investigations.Investigations include routine hemogram,random blood sugar,liver function test,renal function test,urine RE/ME,ultrasound of abdomen,X-ray L/S spine (AP/Lateral view),Thyroid profile,serum calcium and vitamin D level.It may also be due to low calcium level or muscle spasm.MRI of L/S spine can be done after evaluation.Proper treatment depend upon findings.Get well soon.Take CareDr.Indu Bhushan"
},
{
"id": 169552,
"tgt": "Is stomach bloating with diarrhea and flatulence worrisome?",
"src": "Patient: hi my 19 month old baby had a stomach bug and after some bouts of vomitting he has been having diarrhea for the past 2 weeks. I have been to the doctor thrice and they say he is well hydrated and normal and hence we should just wait. He has nt had any fever or blood in stools. But I am still worried as its been quite some time. Also his stomach gets bloated often due to the gas which urges to pass motion. Pls advise... Doctor: these symptoms are common during diarrhoea episodes.treatment for diarrhoea usually relieved these symptoms."
},
{
"id": 224406,
"tgt": "Is consuming diane contraceptive pill the reason for delay in period?",
"src": "Patient: Sir . Am 23 yrs old. Un married. My LMP was jan 24 ... I usually get my periods 3 days before the last month cycle date. I.e 26-27 days of cycle.. I had sex by feb 3 with my boy frd. .. Thought i shd take contraceptive pill (DIANE35) so started to take frm feb 4 th and it continued till feb28.. today is march 5 .. its been 6 days after complete 21 tablets of pill.. till now i dint get my periods.. wat would be the reason.. twice i took pregnancy kit test while i was under contraceptice pill course....it showed negative.. wat would be the reasin behind this.. Feel confused and frustated. Doctor: Hi,thanks for writing..OCP's were not taken on time, so the effectiveness of pill is not valid.. Pill should be taken from the 1st day of periods.. Get a pregnancy test now.. Second reason for delayed periods can be due to pill itself.. After stoppage of pill within 7 days you will get your periods.. Hope I have answered your query.. Good day.."
},
{
"id": 75915,
"tgt": "What causes acute breathlessness with low blood pressure?",
"src": "Patient: sir, my mother is 60. She is suffering from arthritis and asthma for long 30 years for which she has taken steroids for all these years. For last 5 years she has been suffering from diabetes also and her asthma revived a year back as it had been silent for all these years may be due to intake of steroids which she has left. And now recently she is diagnosed with Lvef 35% only. Now she is not taking any arthritis steroids but now she is badly suffering from asthma (beathlessness), severe cough, and low blood pressure(84/64). She has been hospitalised twice recently due to low blood pressure and acute breathlessness and recovered quite well both the times but again health deteriorates after few days.Although she is taking tretment from chest doctor as well as heart doctor but her condition is not improving especialy breathlessness. Please advise. Doctor: Thanks for your question on Healthcare Magic. I can understand your concern. Since she is having ejection fraction (EF) (heart function) only 35%, her heart is failing. She is going in recurrent heart failure. She is also having chronic, steroid dependent asthma. This also causes respiratory failure. So combination of heart and lung failure is causing her frequent episodes of breathlessness. Honestly speaking, these are not good diseases for prognosis. She is having poor prognosis. She needs diuretics, cardiac inotropic drugs and other cardiac supportive drugs along with strict control of diabetes and asthma. Hope I have solved your query. I will be happy to help you further. Wishing good health to your mother. Thanks."
},
{
"id": 18542,
"tgt": "What causes pain in the chest and below the shoulder?",
"src": "Patient: I have daily pain on both side of chest recently after months of painting in over my head area I acquired deep pain in area right below my shoulder. might of been stress due to divorce. also arm on right been hurting, which is arm painted with. I just thought might be whatfather had, because of chest pain and other syptoms I described, which he had persiotes-sorry not spelling correctly. now mainly just chest hurts occasionly. I have had rhematic fever which was followed by cardiotes. please tell me what is my possible problem. Doctor: Hello and Welcome to \u2018Ask A Doctor\u2019 service. I have reviewed your query and here is my advice. After going through your medical details I understand your concern and I would like to tell you that considering your specific case, history of rheumatic fever with pericarditis and now chest pain with radiating pain in arms can be cardiac which requires cardiac evaluation. It's recommended for you to get an ECG and Echocardiography done and consult a cardiologist for physical evaluation and treatment. Hope I have answered your query. Let me know if I can assist you further."
},
{
"id": 188277,
"tgt": "Swollen glands by jaw line, mild swelling and soreness. Advise for relief?",
"src": "Patient: I woke up yesterday morning with swollen glands by my jaw line, mild swelling and soreness. I the evening I was relaxing and starting to become more painful, a hot press and cold press did not seem to work to relieve the tension and stress. I work up this morning at 2am to the swelling getting larger, moving into my neck, creating it hard to breathe when laying down, and swallow. I look like i have gains 20 pounds in my face. I am planning a Dr. Appt. for today, but any ideas for relief or symptoms? Doctor: Hello,Thanks for writing to us.I would advice you to get a thorough clinical evaluation to rule out the cause of swollen glands below jaw line.Decayed or periodontally infected tooth has to be root canal treated or extracted.Abscess has to be drained and curretted.Complete course of antibiotics-analgesics has to be administered.Maintain oral hygiene well.Please do visit a dentist soon.Take care."
},
{
"id": 192621,
"tgt": "What causes bend in penis while erected?",
"src": "Patient: Hi, may I answer your health queries right now ? Please type your query here...Dear sir,i am 26..my problem is my penis is bend down side..normal time its ok..but when i get mood that time only automaticaly is movng to down side. its length is 6'is this any serious problem? will it affect my future sex life?plz explain me Doctor: Hello, It could be a chordae arising due to thickening in the shaft of penis. Consult a urologist and get evaluated. Surgical correction may be required after confirming the diagnosis. Hope I have answered your query. Let me know if I can assist you further. Take care Regards, Dr. Shinas Hussain, General & Family physician"
},
{
"id": 85136,
"tgt": "Does thyoxin have any side effects?",
"src": "Patient: i am taking 150 mg thyroxin everyday morning on empty stomach,followed by after breakfast 50 mg ASA and multivitamin.On alternate days evening one roseday5 mg. This is going on for about a year now. My HDL/LDL is under control.I seemed to be loosing weight recently,and tired. What will be the side effect of thyroxin please. Doctor: Hello, Thyroxine is a safe drug and side effects will be minimal. However, make sure that the hormone status will be within the normal range. You can go for a thyroid function test ( TFT) and make necessary adjustments in the dose. Hyperthyroidism (excess thyroid hormones) can cause symptoms similar in your case. Hope I have answered your query. Let me know if I can assist you further. Take care Regards, Dr. Shinas Hussain, General & Family Physician"
},
{
"id": 98595,
"tgt": "How can allergic itching all over the body be treated?",
"src": "Patient: I got bit by a bunch of mosquitos, and black and yellow longhorn beetles and now my entire forearms and calves are covered from top to bottom in bites, and they re relentlessly itchy. I can t sleep, patches of my face are red, and my lips got chapped very suddenly. Please explain possible causes. Thanks! Doctor: Hi i do care for your concern. Your history suggest allergic reaction to the insect bites. This is a type of hypersensitivity reaction against substance released by these insect. I would advise To consult your physician and get a shot of dexa if needed. Taking medication such as benadryl will help to reduce the itching. application of calamine lotion over skin will add to the help. Hope i have answered your question, if you have more feel free to ask. Thank you."
},
{
"id": 161413,
"tgt": "What does temperature of 90.3 in a child mean?",
"src": "Patient: my son s temp went up and down from 99 - 90.3 we gave him chicken soup and tylenol, he had an epileptic seizure in dec 08 as well as hospitalized in 2000 for pneumonia, i am confused because i never seen it go won that low before. can you please help? Doctor: Hello, If the kid is very active even with such a low temperature then there may be a malfunction of the thermometer. But if the kid is dull and lethargic this is a medical emergency and I suggest you take him to the nearest emergency room as soon as possible. Hope I have answered your query. Let me know if I can assist you further. Take care Regards, Dr Sumanth Amperayani, Pediatrician, Pulmonology"
},
{
"id": 219411,
"tgt": "Can TORCH findings of rubella and cytomegalovirus be treated with minimal risk?",
"src": "Patient: My wife is Pregent for 12 weeks and find in TORCH: (A) Rubella IgG 76.59 (B) Cytomegalovirus IgG 10.60 Baby is OK in Ultrasound at 10th week. Dr. said this is a big problem, but i want to know which period this problem can be solve or minumam risk. Regards Doctor: Hello,I have gone through your query and understood the concern. IgG antibody presence indicates past exposure to the infection or vaccination, while IgM antibody presence signifies recent or current infection. In this scenario, the test may be repeated after 2-3 weeks and if there is a rise in the IgG antibody titres, recent infection must be suspected and further management planned accordingly. If there is no rise in tires, regular follow up is to be done with periodic sonograms to detect any anomalies early on. Vaccination should not be done during pregnancy. The baby is also tested at birth for evidence of infection and appropriate management done. Hope this helps."
},
{
"id": 37382,
"tgt": "Suggest treatment for pilonidal cyst",
"src": "Patient: Hello, I have done some research and I seem to have had either a hair root infection or pilonidal cyst on the upper part of my intergluteal cleft. It burst and oozed dark pus and blood. I have cleaned it and have bandaged it, but when I remove the bandage to replace it, it continues to bleed. I am wondering if I can continue to clean and bandage it to let it heal and not have it come back or if I need antibiotics. I am in college so I would rather not have to go an get a prescription if possible, Doctor: Hi,Welcome to HCM,I had gone through your history and understand your concern,A pilonidal cyst or sinus is a cyst in the lowest part of the vertebral column or in tailbone area. It may contain inside hair or skin remnants. Treatment is surgery under local anesthesia. Usualy there is no connection between the cyst and neurological symptoms. However there are isolated cases reported of sacral or lumbar osteomyelitis and epidural abscess as a complication of pilonidal cyst.I suggest you to discus with your Doctor the possibility of plain radiographs of your lumbosacral column. I also suggest to treat the cyst surgically, it is a simple procedure under local anesthesia.Thank you."
},
{
"id": 15965,
"tgt": "Suffering from rashes all over body, high fever. Blood test done. Treatment for symptoms?",
"src": "Patient: Hello doc! my husband had high fever for 3 days (38-39C), on the 4th day temp is 39-40 with rashes ... blood test result platelet count is 175, hematocrit 0.48. After 12 hours, platelet=161 hematocrit=0.48, after 24 hours , temp is 37.4-37.8, rashes begin to fade, platelet=157 hematocrit=0.46. May I ask for your opinion if he still needs another blood test or do you think he is on his way to recovery? Thank you. Doctor: hello welcome to healthcare magic forum thank you for your query it looks that he is improving ,but should do blood test after 1 day . for rash apply calamine lotion twice a day if itching is there ,antihistaminic is advisable continue all medicine which are prescribed by your doctor there may be exfoliation of skin in next 2-3 days i hope i answered your question"
},
{
"id": 118655,
"tgt": "Flushed, hot, chapped lips. BP 128/105. Concerning?",
"src": "Patient: Just went to the dentist and my bp was 128/105 when I got there, 121/81 when I left - about 45 min later. Knew I had very flushed lips and that has meant high bp for me before so I wasn't surprised but the dental assistant was highly concerned. Been having hot, flushed lips that are staying a bit chapped for a few months now. Doctor: Hello, Thanks for the query to H.C.M. Forum. The normal B P of a young person usually around 120/80 mm of Hg. The systolic Blood pressure ( upper one ) usually elevated due to anxiety or sudden emotions or fear etc. As far as concerned about lower B P ( as in your case 105 ) this is due to any disease in heart or other disorder . Diastolic B P i.e. lower one ( high ) may be ,1 Any heart disorder , can be confirmed by E C H O of heart and consult a cardiologist and get his opinion.2 Hypercholesterolaemia ,so get blood for lipid profile . 3 If all above tests come within normal limit then need not to worry and this may be due to tension or anxiety as this may be your nature. Good luck. Dr. HET"
},
{
"id": 157170,
"tgt": "What do you mean by cluster in arm pit, the mammogram showing three lumps in the left breast and a cluster and was advised for a Biopsy?",
"src": "Patient: My daughter had a mammogram and they found 3 lumps on left breast and a cluster under her armpit. She is 35, went on birth control approx 6 months ago. They are going to do a biopsy on the cluster. She also has a tendency to get kidney stones. I'm worried about the cluster. Can you give me a better explanation of what a cluster is Doctor: Hi, I have read your query and according to my opinion these cluster are enlargement of lymphnode ,that's occurs due to any type of infection in there draining area.That infection may be bacterial or malignant.Thanks for using HCM."
},
{
"id": 4808,
"tgt": "Had tubal ligation, irregular period, cramps, on tampon, stopped bleeding, headache, nausea. Pregnant?",
"src": "Patient: I had a tubal ligation 9 years ago I have my cycle is always on time ever month never late never early this month for some reason it was 4days early I had a period on the 4th of September and period wasn't due until October the 4th on the 30 of September I had no cramps went to the bathroom and I was spotting then went I go up I had a shape pain mild cramping free about 10 min then no more cramps I always get extreme cramps when I have my period so I put on a tampon thinking it was my period changed it no much on the tampon it was just when I wiped for about 2 1/2 days and my period is always 5 days always the bleeding stop 3 days ago now im having headaches mood swings nausea I feel pregnant I had a chemical pregnancy about 6 months ago I want to test but haven't yet Doctor: Hi,As you got tubal ligation there are no chances of having pregnancy.The change of your period pattern and bleeding might be due to some hormonal changes or some stress.If you are near pre-menopause age this is quite common.Ok and take care."
},
{
"id": 22801,
"tgt": "Should i take medicine for 150/100 level BP?",
"src": "Patient: Hello sir, i am a boy with age 23,height 5.6feet,and weigh 69 kilograms, for the last 2 weeks i had severe head ache,yesterday i checked my blood pressure and it was 150/100, my parents also are hypertensive..pls help sir, should i need to take medicine or diet control is enough...pls suggest a better solution Doctor: hi,First you need to monitor your blood pressure and make a chart of it. bp should be taken at rest of atleast 10 min.Also consume low salt diet, avoid smoking if any.if bp is still high we need to find the cause.Following investigation should be done.1. ultrasound Doppler of renal artery 2. Sr cortisol3. 24 hr Urinary metanephrines 4. renal function test"
},
{
"id": 115981,
"tgt": "Are pain in joints and pain during intercourse be the symptoms of thalassemia?",
"src": "Patient: my boyfriend has thalassemia he was just diagnosted with it yesterday. the hemotoligist said it was rare he only a few other cases? his mother has it which is obvioulsy where he got it. he has severe joint pain and swelling in his hands and joint areas. And also it affects intercourse it is extremely painful and basically takes all the energy from him to the point of trouble breathing and it's like it sucks the life out of him. are these things normal, and what should he do about the pain? anything would help as the doctor yesterday didn't explain anything just said this is what you have and took more blood. thank you Doctor: HIWell come to HCBIn Thalassemia such pain may be possible some time after some exertion because this demands more oxygen while patient is not having enough of oxygen in blood and this could cause pain in tissues but this manifestation is not common still one has to be careful, hope this information helps, take care."
},
{
"id": 48351,
"tgt": "How to treat knee pain after using cipro?",
"src": "Patient: I was treated for a bladder and kidney infection with cipro. All that time I had pain in my lower right side of back. The infection was treated but the pain didn t go away. I was diagnosed with two 1mm kidney stones on Friday. The pain is awful and is radiating down to my knee and up my ribs to my elbow. My knee pain feels slightly worse than my lower back pain now. What should I do? Doctor: HIWell come to HCMIf you do have urinary tract infection, this may cause abdominal pain, if it is stone so this could cause frequent infection and pain too, but this may not cause pain in joints, if you have joints pain then this could be due to arthritis, for the stone and urinary tract infection, get done the urine culture sensitivity test, for the stone site and size of stone is important, find it out, for the joint pain this can be managed with Tab Diclofenac 50 mg sustain release once in day, take care have a nice day."
},
{
"id": 173353,
"tgt": "How to treat chiari 1 malformation in a 13 years old child?",
"src": "Patient: My son 13 year old son has been diagnosed with a chiari 1 malformation. His symptoms are, severe lethargy, head aches, blurred vision, dizziness, ringing in his ears, and chest pains. The doctors we have seen are still running lots of tests as they do not think the chiari is responsible for all of his symptoms. From research I have done, all his symptoms are consistent with a chiari malformation. I am feeling very frustrated as it breaks my heart to see my son in pain and not living a normal teenage life. I would appreciate your opinion Doctor: Hi, If I were your treating Doctor for this case, I would come up with three surgical possibilities for relief pressure on the brain and spinal cord, these include:1.Posterior fossa decompression surgery. This involves the removal of a small portion of the bottom of the skull and sometimes part of the spinal column to correct the irregular bony structure. The surgeon also may open and widen the dura matter(hard membrane of brain). That's the firm covering of the brain and spinal cord tissues. This creates additional space for the cerebrospinal fluid to circulate.2.Electrocautery. This uses high-frequency electrical currents to shrink the lower part of the cerebellum.3.Spinal laminectomy. This is the removal of part of the arched, bony roof of the spinal canal. This increases the canal's size and reduces pressure on the spinal cord and nerve roots.Hope this answers your question. If you have additional questions or follow up questions then please do not hesitate in writing to us. I will be happy to answer your questions. Wishing your son good health."
},
{
"id": 161431,
"tgt": "What causes a blister on the neck in a child?",
"src": "Patient: Hi, I have genital herpes that was diagnosed 18yrs ago. i have a genital outbreak about once every 18mos or so and last week I had a lesion on the back of my leg for the 1st time ever. I am concerned about my 9mo old daughter....she had a single blister on her neck/under her jaw that never spread, but is still present, yet smaller now, after 5 days. On day 3 of me watching that one closely another one popped up on her leg, just above her ankle. I am terrified, (and maybe a little paranoid), that this is herpes. I covered the one on her leg with a waterproof bandaid to prevent spread of any juice if it pops. It, like the one on her neck, hasn t spread but has a reddened periphery and is definitely a little blister. Because of the new location of MY leg lesion, I didn t notice it as early as I do my genital outbreaks and am afraid I spread it to her somehow. BTW, I had a normal vaginal delivery 9 1/2 months ago, no outbreak at that time, and was taking a prophylactic dose of valtrex daily for a week prior. Doctor: Hello, Skin conditions are best diagnosed and treated after seeing them directly. May I suggest you upload some images of what your your telling so that we can guide you better. Hope I have answered your query. Let me know if I can assist you further. Take care. Regards, Dr. Sumanth Amperayani, Pediatrician, Pulmonology"
},
{
"id": 173362,
"tgt": "What does blood test reports indicate?",
"src": "Patient: HI MY DAUGHTER 3.5 YRS OLD HAS BEEN DIAGNOSED WITH S.TYPHI'O' +VE 1:80 DIL.S.TYPHI'H' +VE 1:80 DIL .POLYMORPHS 49%,LYMPHOCYTES 43%,EOSINOPHILS 08%,MONOCYTES 00%,BASOPHILS 00%,E.S.R1/2HR 12MM,E.S.R1HR 26MM,PERPHERAL SMEAR MP:NEGATIVE,PERIPHERALSMEAR MF :NEGATIVE.COULD YOU PLEASE HELP UNDERSTAND SERIOUSNESS OF THIS? Doctor: Hi, I had gone through your question and understand your concerns. You should be worried but not too much. Your cholesterol is not that high to be very worried. You may easily understand from analysis that she has Salmonella typhi infection, which usually causes gastroenterocolitis at kids. I see similar cases amongst children who visit my clinic. From blood analysis you should pay attention to high level of eosinophils, it points to allergical process or helmintiasis. You can discuss this with your treating Doctor. ESR is also increased moderately due inflamed process,after treatment with antibiotics it will get back to normal range.Hope this answers your question. If you have additional questions or follow up questions then please do not hesitate in writing to us. I will be happy to answer your questions. Wishing your baby speedy recovery."
},
{
"id": 211628,
"tgt": "Old aged, taking Cymbalta, having severe memory loss, crying episodes. Could the Cymbalta be causing this?",
"src": "Patient: My 92 yr. old mother was started on Cymbalta 30 mg about one year ago when her prior antidepressant , Zoloft , appeared to stop working. Her memory has declined even more when an additional 30 mg of Cymbalta was added for a total of 60 mg Cymbalta. She is having severe memory loss now, crying episodes which are uncontrollable, hallucinations of events that never happened. Could the Cymbalta be causing these symptoms or is it just a progression of dementia? Doctor: Hello,Dementia is unlikely if the symptoms you have mentioned appeared recently specially after increasing the dose of Cymbalta.Considering her age, 90 mg of Cymbalta per day can be considered to be a very high dose for her in my view. Many a times, antidepressant like Cymbalta can also destabilize the mood of patients resulting in symptoms like uncontrollable crying, irritability, insomnia, memory loss, hallucinations etc and in such cases, the antidepressant need to be discontinued and or replaced by a mood stabilizer.You should discuss with her doctor about reducing the dose of Cymbalta to 60 mg per day or less and see if this helps.Kind regardsVikas"
},
{
"id": 60928,
"tgt": "How can painful lipomas in the armpit be treated?",
"src": "Patient: Hi, My husband has multiple lipomas, he just had one removed, but he has them under his armpit`s,and they are hurting him. We have some good Dr.`s but most of them won`t do anything for him. He has fallen,(he has type 2 Diabeties and has it under control ) .When he has fallen,I`ve seen that he stops him self with his arms and knees. I do understand that trauma to the sites I mentioned,can cause these tumers. But how can he have them removed,they are growing... they aren`t small, they have been growing for the last 15yrs. He is 67, and is strong willed, also is there liposuction avaible? Doctor: Hi,The lipoma requires surgical excision only. There is no liposuction available for the same. Hope I have answered your query. Let me know if I can assist you further.Regards,Dr. Bhagyesh V. Patel"
},
{
"id": 21384,
"tgt": "What are the brand equivalents of Bisocor, Salospir and Tulip?",
"src": "Patient: Hello, I have a question regarding some medications i have bought for my heart condition. I wanted to know what brand name of medication is equivalent to (Bisocor, Salospir, Tulip)? PS these medications have been prescribed and bought in Europe. Thanks, Deana Doctor: Hi The brand nameo f the medicines are as followTablet Biselect 5 mg Salazoprin 7.5 mgAtorlip 10 mg In case you need any other help ,i would be more then glad to help you."
},
{
"id": 83309,
"tgt": "Could headaches be side effect of taking cyclacur?",
"src": "Patient: my wife is taking cyclacur to start to manage her period cycle. Since she swtiched from the white tablets to the brown tablets in the box, she seems to have started getting headaches in the morning after waking up ? is this being caused by cyclacur ? she is 42 years old Doctor: Hi,Yes, headaches are a common side effect of taking oral contraceptives pills like cyclacur which usually improve over time. The white tablets contain hormone estradiol and the brown tablets contain a combination of estradiol and norgestrel (progestogen). In case if headaches do not subside your doctor would prescribe a different type of pill with a different dose of estradiol and/or a different type of progestogen.Take care. Hope I have answered your question. Let me know if I can assist you further. Regards, Dr. Mohammed Taher Ali, General & Family Physician"
},
{
"id": 175051,
"tgt": "What is the cause of a swollen face?",
"src": "Patient: Hi Doctor, my 19 month old baby girl woke up with one side of her face swollen this morning, the swelling is noticeable on the chick and the one side of her lower lip. She doesnt seem to be in pain though she is her normal hyper self. Is there reason for me to panic? Doctor: Thank you for consulting in Health care Magic. You should take your child to dentist surgeon and get him examined especially with X-ray, CT scan. There are can be different causes: lipoma,neoplasm or etc. Best regards Dr.Svetlana"
},
{
"id": 132036,
"tgt": "What causes pain in bones?",
"src": "Patient: i m suffering from multiple issues and symptoms rash on face when in sun, locking of the joints, pain in some bones, when i m going to star menstruating symptoms worsen also get before I start my period swelling in the brain, off balance, dizziness, cant remember things, cant think right, severe pain , kidneys seem to not work I fill up with 10lbs of fluid in my body, kidney pain, inflammation all over including stomach, chest, gums, joints , neck, back, blurred vision, pain in bladder, incontienanceof urine sometimes stool, cant have a bowel movement , heart palpatations, plus more Doctor: HiThank you for asking HCMI have gone through your query. A proper diagnostic work out is necessary to rule out the main cause of your problem. You should get a blood routine to rule out anaemia , thyroid function test to rule out hypothyroidism and urine routine to find out whether kidneys functioning well. Blood routine will be helpful to find out any raised ESR which points to any inflammatory process running in the body also. Hypothyroidism or anaemia can give the symptoms you mentioned. In case of anaemia iron supplementation and in case of hypothyroidism thyroxine supplementation will be helpful to clear your problem.Hope this may help you. Let me know if anything not clear.Thanks."
},
{
"id": 24271,
"tgt": "What causes dizziness while on medication for blood pressure and thyroid?",
"src": "Patient: I have vertigo from an inner problem and I wear hearing aids. I have high blood pressure and am taking 100 mg's of Losartan. In the last several months I have experience severe dizziness when I do a lot of work on the computer, read, rocking in a rocking chair, even when I am eating. The only medication I take is this blood pressure medication and a thyroid medication. I need to find an answer to this problem because it is affecting my work. I am so tried of being so dizzy and the room spins and I can get phyically sick. HELP! Doctor: Hello!Welcome and thank you for asking on HCM!I carefully passed through your question and would explain that your symptoms could be related to different causes: 1- inner ear disorder2- thyroid dysfunction3- orthostatic hypotension. 4- cervical spine chronic degenerations For this reason, I recommend consulting with your attending physician and performing some tests: - complete blood count for anemia- thyroid hormone levels- a cervical spine X ray study- labyrinthine tests performed by the ENT specialist. - a Head Up Tilt test if suspicions of orthostatic hypotension are raised. Meanwhile, I would recommend taking cinnarizine. You should discuss with your doctor on the above issues. Hope you will find this answer helpful!Kind regards, Dr. Iliri"
},
{
"id": 207405,
"tgt": "What causes loss of appetite while on treatment for depression/multiple sclerosis?",
"src": "Patient: i feel hungry but donot feel like eating anything. i have been obese all my life. i am a multiple sclerosis patient and also suffer from depression. i am taking treatment for both. i am 27 yrs old and weigh 96kgs 5 6 tall. i am not feeling like having anything from almost a year. first i gave up vegetables and started having only chicken. but now i dont feel like having that too. what to do? Doctor: Yes depression causes change in eating habbit. Dont worry take some appetizer like livzuyme syrups. It will help you a lot. So dont worry. Hope my suggestions helpful and thankful to you. Take care."
},
{
"id": 58825,
"tgt": "Solar plexus has rashes, red spots and itchiness in legs, irregular heartbeat, anxiety problems, liver problems. Suggest?",
"src": "Patient: My solar plexus has a rash of itchy red spots, my sides and underarms are itchy as well. My legs are only itchy after a hot shower, most likely due to dry skin. I don't have a fever, I do have irregular heartbeat issues (since birth with no treatment), I have anxiety problems, I have had liver problems, I drink nightly. I had ringworm, treated with anti-fungal cream, thinking it might be rheumatic, not sure, any suggestions? Doctor: Hi, it appears to be the fungal infection, as you have used only creams it is cured incompletely and recurred. I to my patients wit such symptoms prescribe, fluconazole, grisofulvin, ans ketoconazole soap. Thank you."
},
{
"id": 75309,
"tgt": "What causes chest pain and shortness of breath while smoking?",
"src": "Patient: when i was on my cigarette break, my chest begins to tighten. it's like there are heavy bricks on my chest. Then after that i have this shortness of breath. i can't breath in deeply. My chest became more heavier that i feel dizzy. I go straight to the hospital after my shift, my heart rate is normal. ECG is normal. What could this be? But the doctor advice me not quit smoking ASAP. Is this an asthma attack? Even now i still have this heavy chest feeling. I'm really paranoid. I can't sleep. Please help. Thank you. Doctor: Hi welcome to the health care magic Here your work up apart from EKG done as follow..... (according to your complaint)... -Auscultation -Chest x ray -pulmonary function test -CBC This investigation will reveal COPD or any other restrictive lung disease if present According to cause specific treatment given If pulmonary fibrosis is the cause than steroid might prescribed If copd present than ipratropium bromide like inhaler therapy given Avoid smoking as it damage lung parenchyma by elastase release Take care Consult pulmonologoist for your examination and subsequent work up"
},
{
"id": 206629,
"tgt": "What causes brain fog for long time and unclear speech?",
"src": "Patient: I have had brain fog for nearly 7 years. I didn't quite know what it was until a friend got me to smoke weed and \"relax.\" I felt the exact same under the influence of the herb as I did normally, just amplified. My speech is very impaired. I used to have some of the best memory of all the people I knew, and now I have the worst. I have trouble becoming angry, I usually just feel sadness. (Which is completely different from when I was a child) My thyroid looks normal with the exception of an elevated TSH (4.8). Blood pressure was 110/48 and my BPM was 61. Drinking more water raised my blood pressure, but the overall feeling stayed the same. Doctor: HiThanks for using healthcare magicIn that case, you need to consult a neurologist. It could be vascular dementia or CVA. Better to consult a neurologist. It would help to get a proper diagnosis. In case, you need further help, you can ask.Thanks"
},
{
"id": 116937,
"tgt": "What causes headache after taking Amdepin for BP?",
"src": "Patient: Morning Doctor. I am 52 years old working women also I am a BP patient. Some times it will go up, then I am very very week and also head ache. Now I am taking BP medicine \"Amdopin 2.5Mg as instructed my doctor once a day. please give me a reply. Yours faithfully, jnjm Doctor: Head ache is one of the feature of hypertension. But head ache can also be caused by Amdopin tablet since this drug produces dilatation of blood vessels. However this head ache will gradually diminish when the drug is taken continously. Hence there is no need for concern. However if your head ache is very severe and frequently occurs, you have to be investigated for other serious causes of head ache such as sub arachnoid hemorrhage, migraine,etc.,"
},
{
"id": 182706,
"tgt": "Suggest treatment for a chipped front tooth",
"src": "Patient: Hi, may I answer your health queries right now ? Please type your query here...I chipped my front tooth and cant get to the dentist til tuesday. I have a water polo tournament this weekend, is there anything I can put on it til then to keep it covered Doctor: Thanks for your query, I have gone through your query.The chipped tooth cannot be covered with something by your own, you need to consult a dentist and get it treated, Mean while if you have sensitivity in the tooth, you can use desensitizers like strontium chloride(sensoform or sensodyne).After your tournament you can get the tooth restored either with composite if the fracture is not involving pulp. If it is involving pulp, then get it treated with root canal treatment and crown.I hope my answer will help you, take care."
},
{
"id": 67421,
"tgt": "What causes recurring painful lumps on the buttocks?",
"src": "Patient: Hi, I m a 23yr old man, and I m having this soft, dark red coloured lump on my lower buttocks (the cheeck itself). I have no known allergies, nor a history of severe acne or other related conditions. It is rather painful and annoying, and especially painful when pressure is applied. This makes it very hard to sit on as well. It feels very soft like it is filled with pus, it has no hard part like a cyst would have a pea-like head. This lump comes for a few days to a week then goes away for a while, only to return at a given point. It can occur on both the left and the right cheeck. I doubt it be some hygi\u00ebne problem, as I ve always paid attention to it, especially now since these lumps have been appearing. Could this be something like an ingrown hair? Most likely not a pimple that has gone crazy, as I would have noticed the pimple itself. Would you suggest to just wait until it disappears again as it has done before? Thanks Doctor: Hi! Good morning. i am Dr Shareef answering your query.As you get pain in the lump when it appears, it seems to be an inflammatory/infective origin. I would advise you to take appointment with a general surgeon in your area, and get it assessed clinically with related investigations to rule out a recurrent perianal abscess which could lead to a fistula in ano. Another possibility of a pilonidal cyst would have to be ruled out which could also result in a pilonidal sinus. All these could be diagnosed by a clinical examination and so you would have to see your doctor for an appropriate management. Till then, you could go for some anti inflammatory along with a proton pump inhibitor for a symptomatic relief.I hope this information would help you in discussing with your family physician/treating doctor in further management of your problem. Please do not hesitate to ask in case of any further doubts.Thanks for choosing health care magic to clear doubts on your health problems. I wish you an early recovery. Dr Shareef."
},
{
"id": 76843,
"tgt": "Suggest treatment for whooping pneumonia",
"src": "Patient: I am 63 5'7\"and 198 pounds, and have been diagnosed with, as the nurse practitioner told me, a \"whopping case of pneumonia.\" I'm on my second day taking levofloxicin and using ventolin HFA. I think my fever has broken ..it was 102.5 for a long time, but now I can't stop facial and body sweating. I'm changing nightgowns hourly. Is this something to call my doctor about. Doctor: Hi thanks for contacting HCM...Noted you have pneumonia and taking levoflixacin for it...Continue using drug without worry.Your sweating could be by drugs you are taking ....Healthy and balanced diet taken for early improvement in pneumonia...Fruit juice like orange juice taken more.Avoid smoking exposure active as well as passively....Keep away from dust and environmental pollutions....Plantain leaf help in soothing irritated respiratory tract and decrease mucous production....Oregano capsule Helpful for respiratory tract....Take care.Dr.Parth"
},
{
"id": 43367,
"tgt": "Side effects of Redotil and Biophene to cure infertility",
"src": "Patient: Hi While consulting for Infertility , my doc has prescribed 1. Ubi Q 300 2. Carvi 500 3. Lenova M 4. Biophene and 5. Redotil While the first 3 seem to be Nutritional Supplements , Could you please tell me what Biophene and Redotil are for? I ve been taking all except Redotil (as it was not available) for 2 weeks now and taking Redotil from today. Do any of these tablets have any side effects, especially with sweating, urinating or dizziness (I d say some state of confused/disturbed thoughts than just dizziness)? Thanks in advance for your anwers Doctor: Hi, For low motility in the sperm count and so was given Biophene tablets for that.Redotil is an anti- diarrheal tablet. Side effects of Redotil are nausea, vomiting,sleeplessness and migraine."
},
{
"id": 8785,
"tgt": "Which medication should I use to treat the scar on my upper lip which got burned while hair removing through waxing ?",
"src": "Patient: i removed my upper lip hair through waxing and my skin got burnt. i didnt apply anything it got healed by itself but now i have perminant darker skin of my upper lip than rest of my face. i have bought webber vitamin e cream but i m not sure if i can apply that. what should i use to remove that scar please help! Doctor: Hello, thanks for writing in. You may apply vitamin E cream in morning and Vitamin C cream at night. Gradually it should decrease the pigmentation."
},
{
"id": 139279,
"tgt": "How long does it take to recover after core decompression surgery?",
"src": "Patient: I am undergoing core decompression on the 25th of august my question when can i start work at the earliest. Just googled and noticed i need to be on crutches for 6 weeks. Does it mean I cant go to work for 6 weeks. Can I drive or is the same recovery process as THR which I had for my left hip. Doctor: Dear Sir/MadamI have gone through your query and read your symptoms.In my opinion, you need to understand two things, your avascular head of the femur needs offloading to reshape itself or to prevent from loosing shape, till it gets revascularized. the core surgery is to help it revascularize fast, 6 weeks is just empirical and must, but in some cases it can be extended based on the fact on your x ray changes.I hope that answers your query. If your want any more clarification, contact me back.Dr Narender Saini"
},
{
"id": 52012,
"tgt": "How to get kidney?",
"src": "Patient: hi sir .my mother kidney has failed.i am very sad.but my blood group is no match.but a man want to donate kidney.but doctor no ready and he demand paper.if my mother will not well i will also die.doctor are god for patient.please sir wat do me?please sir help me and save my life. Doctor: Hi, Welcome to Healthcare Magic Forum, Kidney transplant cannot be done only when the Donor is ready, there are few protocols to be followed before proceeding. So, discuss it with your family, friends and take a opinion from other family Doctors as well, they all will help and lastly, don't lose hope. Discuss your issues with the concerned Doctor as well. Your mother will be alright. All the Best, Take Care."
},
{
"id": 225288,
"tgt": "On birth control and azithromycin for strep. Could antibiotics negate the effects of birth control?",
"src": "Patient: I was taking azithromycin for strep earlier last week. I am also on the implanon birth control and It s due for removal tomorrow. Could the antibiotics negate the birth control? Because I started heavily bleeding a couple days ago. Could I get pregnant with all of this occurring at once? Doctor: Antibiotics dont negatevthe effect of birth control especially if you are taking them short term...nothing to worry. ...although few medicine have effect on the metabolism and they r known to cause effect on other medicines but there is no strong drug interactions between azithromycin tsken for short periods and birth control ...provided that you are not skipping your pills"
},
{
"id": 216037,
"tgt": "Ia drinking cocktail daily affect the consumption of methadone hcl 5mg?",
"src": "Patient: My 91 year old aunt was given a prescription for methadone hcl 5mg tablets for pain in her knee and hip, since replacement surgery is not an option. She was previously managing the pain with Advil. She has a half of a martini every day and wants to know if she must stop having her daily cocktail if she is going to take this medicine. Thank you. Doctor: Hello and Welcome to \u2018Ask A Doctor\u2019 service. I have reviewed your query and here is my advice. Alcohol may increase the sedative action of methadone. No others interaction reported between methadone and alcohol and alcohol doesn\u2019t affect the efficacy of methadone. However daily alcohol consumption is not good especially at this age! Hope I have answered your query. Let me know if I can assist you further. Regards, Dr. Shinas Hussain"
},
{
"id": 36056,
"tgt": "What is the medication for jaundice,malaria and typhoid at the same time?",
"src": "Patient: my brother is suffering from jaundice,phelsepherum malaria and typhoid at a same time.. and he has infection in liver too... the haemoglobin level is less as well.. he s admitted in a city government hospital.. can u please suggest me.. what to do now.. Doctor: HelloYour brother is having typhoid , falciparum malaria ( also known as algid malaria 0 , jaundice and anemia .As you mentioned that he is in the hospital , so frankly I would recommend you need not to go anywhere , rather take indoor treatment in a hospital under strict medical supervision.Without proper physical examination and report ,it is not possible to advise any treatment .If condition is not improving shift your brother to any other hospital.Good luck. Although treatment is very simple but not possible without physical examination and reports."
},
{
"id": 53964,
"tgt": "Is surgery recommendable for cyst in liver?",
"src": "Patient: .the cyst appears to be in left frontal lobe of the liver ....the patient doesn't have any pain or problems....but its been detected in the CT scan and the cyst may grow in future...Is it dangerous to go for surgery and get it removed now....?the cyst is 7.09cms in length and 5cms in breadth......please do assist me.... Doctor: Hi and welcome to Healthcaremagic. Thank you for your query. I am Dr. Rommstein, I understand your concerns and I will try to help you as much as I can.If there is no features of cancer and no symptoms then this should not be removed unless it grows to more than 10cm in size or symptoms occur.If there is doubt about malignancy then MRI scan should be done for more accurate diagnosis. I hope I have answered you query. If you have any further questions you can contact us in every time.Kindly regards. Wish you a good health.DR. Ivan Rommstein"
},
{
"id": 28975,
"tgt": "Suggest treatment for recurrent flu",
"src": "Patient: I was diagnosed with the flu last Thursday, was outbid on tamiflu and felt better and returned to my normal activities on Monday. However yesterday evening I started running a fever again and it has continued into today. Should I go to see my doctor again or is this still the flu just running its course? Doctor: Hello,Some advice that I can give to you is:- Take Ibuprofen drug for the fever three times a day- Drink liquid- Drink fruit juicesThe symptoms should continue for two to three days.Hope I have answered your query. Let me know if I can assist you further.Regards,Dr. Dorina Gurabardhi"
},
{
"id": 3945,
"tgt": "Will 'utovlan' help in getting pregnant?",
"src": "Patient: hi i was on the contraceptive injection depo provera for 3 months only had one injection i have been off it for 7 months now i was bleeding for 5 of them so i went to the doctors and he gave me utovlan i have finished taking my 3 weeks course and have had my first normal period will utovlan help with my fertility returning as i am trying for a baby now Doctor: Hi,Utovan alone may not help in conception. Get semen analysis of husband HSG for yourself.Regards"
},
{
"id": 124126,
"tgt": "How to treat a large bruise around my knee?",
"src": "Patient: I got kicked by a horse on Sunday. I have a huge long bruise taking up the majority of the outside of my femur and on the side of my knee. On my knee I have a large lump and very dark colored bruise except in the middle is white. And it s super tender. What is this? Doctor: Hello, As I can understand from the history, I feel this is a blood clot happened due to muscle injury over the thigh. Since you mention the knee I will recommend you to get it examined and also an MRI on a safety basis as sometimes the inflammation in the tissues get septic and needs proper medical attention. For now, you can do quick icing to reduce pain and inflammation. Any ignored injury leads to problems in the latter stages of life. Hope I have answered your query. Let me know if I can assist you further. Regards, Jay Indravadan Patel, Physical Therapist or Physiotherapist"
},
{
"id": 111490,
"tgt": "Suggest medications for chronic back pain",
"src": "Patient: i have had this chronic back pain for a period of 5 years now, x-ray and ultra sound scan was done and was told there is no problem. was given some treatment but the pain has persisted, i have been taking pain relief medication but the pain still comes back. I have consulted several doctors but the only thing i get is pain relievers.. am frustrated, the pain is just too excruciating.... Doctor: Hello, I had gone through the case and found that you must go for MRI of back and Vitamin D3 blood test.Hope you will get the diagnosis. Take treatment according to that.Meanwhile if pain is severe then go for physiotherapy massage.Hope my answer will be effective for you.Thanks"
},
{
"id": 53760,
"tgt": "How to treat high elevated liver enzymes?",
"src": "Patient: I have been diagnosed by a doctor with high elevated liver enzymes. Tests show it is not fatty liver, diabetes and ultrasound didn't display any tumors but did show my spleen is 5cm oversize. Last week while at football training I became very disoriented and had \"tunnel vision\" describing lack of peripheral awareness, although I have not excercises in a while. Just now I noticed my urine was cloudy aswell and has been consistently weird colors. I also have tested negative to all hepatitis. What is your opinion? Doctor: Hi welcome to the health care magic Your liver enzymes elevated but mention what is exact value of sgot and sgpt... If they are elevated more than 80 that is approx two times of normal than it is highly significant.. Viral infections like infectious mononucleosis or cytomegalovirus can lead elevated value... Hence your detail clinical history and examination needed Urine culture and blood culture done along with CBC and peripheral smear examination.... According to cause further specific treatment given Celiac disease, muscular disorder, heart problem Also can be caused but as pus in urine and splenomegaly present , infective etiology most likely Take care Consult physician for examination"
},
{
"id": 137481,
"tgt": "Suggest treatment for dizziness and sharp pain in the arm",
"src": "Patient: I played football on Sunday and twice I headed a ball from a goalkick and became slightly dizzy and got a tingling/ slight pins and needles in my right hand and arm. The hand is still feeling slightly weird when I fully contract it. Any idea what this could be? Doctor: Hi,Thanks for your query.Seems that you are suffering from cervical spondylitis, the giddiness & the sensation of loss of balance in her case is termed as CERVICAL VERTIGO. This is generally a benign condition and is usually self limiting.The general treatment guidelines for her condition is as follows -Use a soft cervical Collar: Soft collar allow the muscles of the neck to rest and limit neck motion. This can help decrease pinching of nerve roots with movement. Soft collars should only be worn for short periods of time, because long-term wear can decrease the strength of neck muscles.Medicines: Analgesic are needed in the acute phase (Ibuprofen, Diclofenac, Naproxen etc). Muscle relaxants are essential for relieving muscle spasm. Neurotropic vitamins like METHYLCOBALAMINE orPREGABALIN will help in alleviating the neuropathic pain.Cervical traction: may enlarge the disc space, permitting the disc protrusions to regress back. Intermittent cervical traction for not more than 30 minutes at a time will suffice.Physiotherapy: Once the acute phase of pain has subsided, Isometric strengthening exercises of the paravertebral muscles are started, under the supervision of a qualified physiotherapist . I do hope that you have found something helpful and I will be glad to answer any further query.Take care"
},
{
"id": 93230,
"tgt": "Had laparoscopic abdominal surgery for insulinoma tumor. Abdominal tension after being hit on the stomach. Is this due to adhesions?",
"src": "Patient: Hello my name is Brandon. I had laproscopic abdominal surgery for an insulinoma tumor 7/21/2011. The recovery went well and I was able to continue my running as I am an avid runner. I avoided lifting and physical contact. After a party in November where I was hit (lightly) in the stomach and slept on my stomach I have experienced abdominal tension when running and standing. CT, colonscopy, and endoscopy inconclusive tests taken in February. Was told to try abdominal exercises and stretches. Those did not help, and followed up with massage therapy which is said to help adhesions. Tension has stayed. Are these adhesions? Would adhesions have arisen after one night? Any reason for this pain which has not gotten any better since November? Doctor: If CT, Colonoscopy and Endoscopy are normal it means there is a very small likelihood and anatomical issue. Yes adhesions do form after any sort of surgery and no adhesions do not form in a day (except immediately after surgery when they form). Your GP/Surgeon would be the best person to examine you and suggest a relief.Take care"
},
{
"id": 63859,
"tgt": "What causes painful lump on ear?",
"src": "Patient: i ve had a large lump about 2 inches below my left ear and about an inch toward the back of my neck from my ear...i was in a car accident last year and noticed it right after...none of the doctors i ve seen have been too worried about it, but i would like to know wut it is...now i have pain that radiates throughout my body sometimes my ankles or elbows or feet or hands or knees...please help ; ; Doctor: HI,Thanks for the query to HCM.Causes for painful ear lump-1 yr after car accident,is abscess in scalp hairline behind ear and -is due to the boil/ furuncle.Treatment -would be Antibiotics / Anti-inflammatory from ER doctor.Hope this would help you to plan treatment with ER doctor.Welcome for any further query to HCM.Hit thanks and write good review if this reply helps your health concerns, for the benefit of new and old patients at my virtual clinic at HCM.Good Day.Dr.Savaskar M.N."
},
{
"id": 121502,
"tgt": "What causes pain in shoulder with increased ESR?",
"src": "Patient: Age:35 height: 6 Hi Dr, my husband is having high ESR 85 frm few months with high fevr and bad shoulder pain ..after dignose we gt some lipomas in his sgoulder opreatd nd he is feeling wel aftr that we did some more checkups HB levl is 10 and ESR still high at 110..am very upset plzz tell me wht can v do why his ESR is high plzz help me plz Doctor: Hello,His symptoms are suggestive of arthritis or bursitis. The high ESR are indicative of present inflammation. Coming to this point, I would recommend performing a shoulder X-ray study. A steroid injection in his shoulder may be needed. You should discuss with his doctor on the above issues.Hope I have answered your query. Let me know if I can assist you further. Regards, Dr. Ilir Sharka, Cardiologist"
},
{
"id": 187988,
"tgt": "What could be the reason for scalloped and red sore around the tongue?",
"src": "Patient: Hello, I have an issue with my tongue. Both sides appear to be scalloped and also both sides has a small red sore. On of the sores on my left has a red border around it and white in the middle. It s not terribly painful but it is irritating. What do you think I might have? Doctor: Hello,The type of sore you are mentioning it looks like that it is an aphthous ulcer which appears because of stress,blood deficiancy, immune deficiancy or other problems, but there is no need to worry as it heal within 1 week.You just apply an topical ointment named mucopain or pansorel to get relieved from symptoms.If it takes time more than that then in that case you should visit a dentist and get it examined.Thank youRegards"
},
{
"id": 12343,
"tgt": "Suggest remedy for psoriasis on head with hairfall",
"src": "Patient: Hello Doctor !!! I am 29 year old i affected by psoriasis for the past 7 years i advised to take sorfil oinment and it worked in my body better and 90% improved but in my head it not worked much especially on my center of the head and hair fall also happened due to that so please advice what to do for the head and hair fall??? and the hair will get regrow or it wouldnt????? Doctor: Hello. Thanks for writing to us at healthcaremagicPsoriasis can affect the scalp as well as other body areas. Psoriatic patches on scalp have thick scaling usually not controlled by OTC antidandruff shampoo.In my patients of psoriasis I usually advice a coal tar+ salicylic acid based shampoo, to be used every alternate days.Since psoriasis is a steroid responsive condition, therefore I also usually prescribe topical potent steroid+salicylic acid based lotions, in my patients of scalp psoriasis.Apart from steroid lotions Vitamin D analogues are also options that can be tried for scalp involvement with psoriasis.These are all prescription medicines. I would suggest that you visit a dermatologist in your region for assessment of the severity of scalp involvement and appropriate treatment.Regards"
},
{
"id": 93424,
"tgt": "Have stomach pain, hair fall and medical history, everything went away during pregnancy. What could be the reason?",
"src": "Patient: I am a 39 year old female. I have had problems with chronic pain, periodically my hair falls out. When I was younger I pealed from head to toe like a snake. Now I am having problems with my stomach swelling so much at time that it pushes against my rib cage. I use to have very little blood flow when I was young, but know I am bleeding so heavy that I have to wear to pads at a time and change my pad every hour. When I was younger I gave blood to the red cross and they said I had hepitis in my blood. When I was tested later I was told I wasn't positive for this. I have been diagnosis with anemia and calcium defiencey. I have a very poor apetite but I still am very over weight. I have had a few 24 hour urines because I have had a high level of protein in my urine. While watching mystery diagnosis I saw the diagnosis Hashimoto disease. This matches almost all my issues. I have seen alot of doctors in my day and I still feel crapy most of the time. My problems all went away when I was pregnant. What do you think? Doctor: this is food interefrenceas your history suggest you may be allergic to milk or wheat as tim increases the systems ae nvolved like stomach reproductive systems and othersget blood srum test for food specific antibiotics for milk wheat potato and other foods you take in rutinego for elimination diet according to test and the diseases will stat decreasing in 3 wk and you will be much confirtable in 3 monthstill the tests you can do sympatomatic rx to relieve the stmptops"
},
{
"id": 73760,
"tgt": "What causes blood discharge while coughing?",
"src": "Patient: Hi, my fiance has had a really presistent cough for the last week now, to the extent he has coughed up blood becuase he ahs been coughing so hard. This morning he has said his hands have started to swell and have gone a pinkish colour..with pain too! do you know what this could be.? Doctor: Thanks for your question on Healthcare Magic.I can understand your concern.He is having hemoptysis (blood in sputum).Common causes for hemoptysis are lung infection, bronchitis and lung cancer.So better to take him to pulmonologist and get done clinical examination of respiratory system, chest x ray and PFT (Pulmonary Function Test).He may need antibiotics, inhaled bronchodilators and inhaled corticosteroid (ICS) on the basis of diagnosis.Telling him to quit smoking as soon as possible if he had this habit.Hope I have solved your query. I will be happy to help you further. Wishing good health to your fiance. Thanks."
},
{
"id": 164981,
"tgt": "Should water be given for 5 month old having pus cells in urine?",
"src": "Patient: My baby will complete 5 months this March 5th. she s having fever and her urine test shows 5-7/hpf pus cells. Her Dr has prescribed 1ml MOX drops every 8 hrs and 8 drops paracetamol drops every 6 hrs in case of fever. He s also asked to do cold sponge with tap water. She was passing mustardy to green foamy stool past 2 days. Have not yet got her stool test.What else am I supposed to take care of. I have less breast milk so she has formula milk and cerelac too. Should i give her plain water every now and then? Is there anything else I need to do. Have no Doctor: Thanks for consulting at Healthcare Magic.Your baby is suffering from urinary tract infection (UTI). Continue antibiotics and paracetamol as prescribed by your doctor.You can add formula with proper dilution if breast milk is not sufficient. Amoxicillin may cause loose motion in some patients but it doesn't warrant it withdrawal. Continue antibiotics as it is crucial to treat UTI completely in infants to avoid future complications.If your baby is passing loose stools and getting dehydrated then you may add ORS solution. You may also add fluids or fruit juices.If fever doesn't subside within 5 days your doctor may advice some investigation like urine culture and sensitivity, USG, CBC etc. to decide further management."
},
{
"id": 105755,
"tgt": "Milk and dust allergy, asthalin inhaler prescribed ,chronic cough",
"src": "Patient: Hi, my daughter is 19 months old and she has got allergy from milk n dust.she use to suffer from cough every 15-20 days...i m very much disturbed from her this frequent cough....today my pediat. Suggested me asthalin inhaler for her...i m lil bit hasitating abt it and i just wanted tp know how long her frequent cough problem will continue..... N ya is this problem is common among children?thnx in advance Doctor: Allergy is common inchildren especially allergy to milk.They will outgrow it by 5 yrs mostly. For the cough, i would suggest budecort 50 via spacer to control the lung inflammation. Asthalin inhalers should not be used for more than 3 days during an acute periods."
},
{
"id": 100105,
"tgt": "What causes itchy red rashes?",
"src": "Patient: Hi. I have recently had blood work that showed a sed rate of 40. I have SLE and am currently experiencing a rash with red bumps and itchiness. Topical creams and antibiotic are not working. I had been swimming in an indoor swimming pool at the gym.Thank you Doctor: Hello,Thank you for asking at HCM.I went through your history and would like to make suggestions for you as follows:1. I would like to know more about your complaints like - where you have rash, did it develop after swimming, does it burn, die you develop such rash for the first time or this was a recurrence, which creams and antibiotics you have tried, which other medications you are currently taking for SLE or any other condition, etc.However, at present, I am replying you as per your given history:2. I would suggest you to apply a moisturizing lotion over the rash, especially immediately after bathing to retain moisture within the skin. 3. I would suggest you to take an antihistamine like hydroxyzine or cetirizine or leocetirizine for at least 1-2 weeks (depending upon response) for itching.4. Please do not scratch the itchy areas as scratching would aggravate itching.5. Please avoid exposure to sun while you have rash because some rashes in SLE can be sensitive to sunlight. You could apply sunscreen (SPF 15 minimum) and wear full covering clothes to protect from sunlight.6. If swimming aggravates itching and rash, please avoid swimming as long as you have rash. Many times disinfectants in swimming pool water are irritants and they can aggravate such rash.Hope above suggestions will be helpful to you.Should you have any further query, please feel free to ask at HCM.Wish you the best of the health ahead.Thank you & Regards."
},
{
"id": 49717,
"tgt": "After renal ultrasound diagnosed parapelvic cyst in kidney, fullness of right renal system, no stones. Suggestion?",
"src": "Patient: After doing renal ultrasound, my doctor told me that he find a parapelvic cyst in my left kidney measuring 3.3 by 2.7 by 3.2 ( cm) and a very slight fullness of the right renal collecting system raising the question of minimal hydronephrosis. No stone is seen. There is no evidence of renal mass.I' men 49 older, I'm warry a little be, so what do you think doctor? Doctor: HIThank for asking to HCMIf the ultra sounds detected cyst in your kidney, then for further evaluation MRI test has to be done for management purpose, or else you will have to keep watch on your sign and symptoms if ever happens, moreover you better get done your renal function test, and go for it periodically, have words with nephrologist, have good day."
},
{
"id": 168802,
"tgt": "What causes severe pain in the back in a 12 year old?",
"src": "Patient: my 12 year old son has had lower back pain for 6 months and recently had an mri i have just recieved a letter saying the mri showed evidence of a chronic problem in his lower back bones and that he has been referred to an orthopaedic surgeon. what could be the problem with his back Doctor: hi, Welcome to this forum.Can understand your concerns.Pain at lower back region on the backside for which NRI has been done and chronic problem is reported could be due to tuberculosis of spine, and old injury which might have small fracture of vertebra,Disc prolapse.Since mri has already been done and it has been advised to visit a Orthopaedic doctor , the child should be examined by the doctor.In tuberculosis, there is history of weight loss, mild persistent fever, persistent cough. While in other 2 cases there is no history of fever.I hope this will help you.Take care.Regards, dr Deepak Patel"
},
{
"id": 146726,
"tgt": "How to manage the extreme pain, numbness in leg, fever and back stiffness caused by disc hernia?",
"src": "Patient: I have been diagnosed with a disc hernia toon through MRI L4L5. I have an apt. to see an ortho Dr. on te 27th but have developed new symptoms. Please advice, Fever, body aches and stiffness in my back, numbness down my left leg accompanied by extreme pain. I am crying as I am in sooo much pain. What should I do? Doctor: Hi,Your symptoms are due to pinched nerve in the lower back region. For relief of pain, you can take pregabalin or gabapentin capsules. Also, you should be on complete bed rest.If pain does not subside, you may benefit from taking epidural injections.Best wishes,Dr Sudhir Kumar MD DM (Neurology)"
},
{
"id": 23513,
"tgt": "How long does it take a dose of Atenolol to have an effect on BP?",
"src": "Patient: How long does it take a dose of atenolol to have an effect on BP? My MD says that my elevated am is from anxiety and that I do not have high BP. I need to get my BP down in order to do pulmonary rehab. I was fine for 19 visits but turned away yesterday. I am to take 1/2 mg. atenolol. I am 74, 128 lbs, 5'5\" Doctor: firstly sir you should have mentioned wjat was your blood pressure without medicine as blood pressure is catagorised in four catagoriesless then 120/80 is normalpres hypertensive stage 120-139/80-89( life style modification is tried)stage 1 htn 140-159/90-99( medications are added plus life style modification)stage 2 htn 160-179/100-109 agressive management is required with two or more anti htn medsatenolol is a antii hypertensive drug and blood pressure will start lowering within 1 hour of intake of the drug,though to maintain yopur blood pressure dose of atenolol to optimum you have to keep a record of your bp every third day for 5 times....dose of atenolol will be inreased or decreased according to your blood pressure records"
},
{
"id": 13278,
"tgt": "Suggest remedy for itchy rashes in knees",
"src": "Patient: Morning Doctor, am a young woman aged 39 years, positive and recently i had rushes around knees which was itchy went to my doctor prescribed Nac Fluconazole and a micorva crea to apply on the affected ares but the rush seems not to be diasappearing at all this is the 7th day taking medication, how long can i expect the rush to disaapear Doctor: Hi,It may not be fungal infection...so not responding to antifungal treatment given by your doctor. It may be some allergy leading to eczematous dermatitis. Consult the dermatologist for the perfect diagnosis and proper treatment. I would recommend you to apply mild steroid cream. And take oral steroids in tapering dose with antihistaminics. Avoid the contact with suspected allergens. Avoid soap bath.Hope this helps.Dr.Ilyas Patel,Dermatologist"
},
{
"id": 138013,
"tgt": "WWhat could sharp pain in rib cage suggest?",
"src": "Patient: Hi, I had a very sharp pain on the right side, under my rib cage when I bent down to pick something up, as soon as I stood up and rubbed my side, I immediately felt like I was going to vomit, and had to rush to restroom, then I felt like I dry heaved but nothing came up except some saliva? It seems ok for now, except a tiny slight pain, however, I m wondering what might this be and if I should be worried? I don t feel like vomiting anymore. Doctor: Hi,Thanks for your query.The symptoms you have described are more likely to be symptoms of muscles strain.To get relief from the pain and for quick healing:1. Do hot fomatation at the site.2. Apply local anti-inflammatory gel 2-3 times a day.3. Take anti-inflammatory drugs like Ibuprofen to reduce pain and local swelling or inflammation.4. Do not lift weight or try to do exercise that involve painful muscles. Give proper time to heal.I do hope that you have found something helpful and I will be glad to answer any further query.Take care"
},
{
"id": 2593,
"tgt": "How soon can I get pregnant after a miscarriage due to cervical incompetence?",
"src": "Patient: as soon as possible i want to b pregnant...when can i got it?just 3 month ego i've lost my 1st pregnancy in 18 week...due to cervical incompetance.my emai add tanushree.sudeshna2gmail.com WWW.WWWW.WW Doctor: Hi,You can conceive now but you have to do cervical encirclage at 16 weeks just fourth month of pregnancy completed.Hope I have answered your query. Let me know if I can assist you further. Regards,Dr. Sujata Kute"
},
{
"id": 163487,
"tgt": "What causes nausea, blurred vision and dizziness in children?",
"src": "Patient: My 12 yearold son regularly feels light headed, complains of feeling nauseous, blurred vision and dtinks atleast 2 to 3 750ml bottles of water each day. When he s playing footy he often looks pail and lacks energy, i give him oranges, water and a couple of jellhh beans for energy. He doesn t urinate to an extensive degree. Do you know what this could be? Doctor: Hello,Water intake very less, improve water intake, liquid intake. Due to less intake of fluid, urine output will also be less, also another symptom you mentioned correlating with it.Hope I have answered your query. Let me know if I can assist you further.Regards, Dr. Sachin Kumar Agarwal"
},
{
"id": 133132,
"tgt": "Suggest treatment for pain on the back and legs",
"src": "Patient: I recently had xrays of my lower back and have been told I have arthiritis. Besides a back op there is not much else that can be done except pain meds. I do not know what type it is but causes pain in lower back and at times pins and needles in my legs and at times I cannot feel my legs. Do you have suggestions. Doctor: dear Sir, I have understood your concern, see as you mentioned, your has told arthritis by seeing x-ray suggest degenerative changes happening to your spine. now pain in lower back is due to lack of movement in spinal segments and pain in leg and foot is due to irritation of nerve in back at root level. so now decompression need to do by no pharmacological and non surgical way by physical therapy in the form of lumbar traction which relive pressure in between disc segments then take ultrasound and IFT modalities for instant pain relief. for 15days continue twice a day. followed by back strengthening exercises and mobilization of segments by skilled physical therapist. you can apply warm and cold water pouring at home and Apply analgesic cream at sleeping time. I hope you have satisfied with the answer."
},
{
"id": 128105,
"tgt": "What causes severe pain in the fingers and fingernails?",
"src": "Patient: on my left hand my 2 middle fingers are going crooked and 1st joint closest to fingernnail hurts. I am 55 and there was no injury although I do have neuropathy, prediabetes, fibromyalgia, and osteoporosis but the finger pain is within the last few months. Doctor: Hello, I have studied your case.Due to compression of this nerve root there is tingling numbness in your fingers pain associated with it.I will advise you to MRI cervical spine for better diagnosis.It can be due to various causes like sudden jerk to spine, due to facet arthritis, degenerative, due to chronic systemic disease like diabetes or hypothyroidism.For these symptoms analgesic and neurotropic medication can be started.Till time, avoid lifting weights, Sit with support to back. You can consult physiotherapist for help.Physiotherapy like ultrasound and interferential therapy will give quick relief.I will advise to check your vit B12 and vit D3 level.Hope this answers your query. If you have additional questions or follow up queries then please do not hesitate in writing to us. I will be happy to answer your queries. If you are satisfied with answer do not hesitate to rate this answer at end of discussion. Wishing you good health.Take care."
},
{
"id": 86446,
"tgt": "What causes abdominal pain and back pain?",
"src": "Patient: All night last night I kept waking up to upper abdominal pain. I felt like I needed to burp or get sick. I vomited a few times but haven't since. Nothing seems to make it better or worse. I'll be fine for a little bit then all of a sudden my stomach starts to hurt again. it's causing my back to hurt right behind the abdominal pain. Any idea? Doctor: Hi ! Good morning. I am Dr Shareef answering your query. If I were your doctor, after a clinical assesment of your abdomen and back along with a general physical examination, I would advise you for some routine blood tests like a cbc,blood sugar,serum amylase and lipase, an LFT, blood urea, serum creatinine, an ECG and some radiological investigation like an ultrasound of whole abdomen, X ray chest and abdomen in upright posture, followed by a CT scan if need be. In the mean while, I would relieve your pain with some anti spasmodic and a proton pump inhibitor drug and advise you to abstain completely from alcohol and smoking in case you do. Further management would depend on the result of clinical assessement and the investigation reports.I hope this information would help you in discussing with your family physician/treating doctor in further management of your problem. Please do not hesitate to ask in case of any further doubts.Thanks for choosing health care magic to clear doubts on your health problems. I wish you an early recovery. Dr Shareef"
},
{
"id": 11867,
"tgt": "Light white patches on face, itching. Prescribed premethrin soap. Cure for patches?",
"src": "Patient: hello doctor, this is XXXXXXX. for the past few months i have light white patches on my face and when checked with skin doctors here, they said it is scables and prescribed a soap called premethrin. i daily go out in my scooty and the dust and air is affecting my skin. before the white patches appeared, my facial skin was itching so much and after that only slight white patches appeared. the itching has stopped but the slight which patch is still there. kindly advise me as to what to do to get rid of this. Doctor: Hi there XXXX. Scabies does not usually involve the face. White patches on the face can be one of many entities. I dont advocate Permethrin soap for my patients usually but each doctor has his/her own rationale & am sure yours meant well. I would advice using a good quality sunscreen depending on your face type et al & for this one would require further descritpion of your skin type et al It would be best if you reported to another dermatologist to get a second opinion & take it from there. If from Blore, you are most welcome to my set-up or a number of colleagues i could recommend you depending on your area. Else feel free to use the HCM directory. Good luck & hope you soon have a blemish free skin Dr Praveen Rodrigues MD Dermatologist, Cosmetologist, Venereologist Vikram Hospital, Bangalore"
},
{
"id": 147453,
"tgt": "Is it possible to be completely cured from viral Meningitis ?",
"src": "Patient: In March of 2013 I got Viral Meningitis. It normally goes away in two weeks but I am still having headaches and very sensitive to light. I was given medication for headaches (Topiramate). Is it possible to be completely cured from t Meningitas or will this be an ongoing issue? Doctor: HIThank for asking to HCMI really appreciate your concern the given history is not suggestive of any viral meningitis you said you had viral meningitis so this is almost year gone since you had meningitis, right now the symptoms that you stated is not because of the meningitis, but could be something else and this is nothing to worry, forget the viral meningitis now, hope this information helps you take care and have nice day."
},
{
"id": 138804,
"tgt": "What causes sudden tremors in left thumb without pain?",
"src": "Patient: For the past few hours, my left thumb has been shaking when I bend it. There is no pain, and it seems to be excluded to just my left thumb. I ve been looking around for an answer, but I haven t been able to find one. Could someone please tell me what s wrong? Doctor: Hi,Dear you could be suffering from a condition known as writers cramps, its a focal dystonia. Such tremors usually are action dependent is they occur when u work, but if they are involuntary ie always present then Parkinson needs to be ruled out.You should consult a neurologist."
},
{
"id": 155475,
"tgt": "Can one color hair after chemotherapy?",
"src": "Patient: Greetings I finished Chemo a couple of weeks ago; Three rounds with 2 wks. off in between for sm. cell lung I am 55 5ft. 3 , 109lbs. I juice 2 x s a day fruit for am liquid cheif salad @ din din I eat very well seafood etc very balanced lots of meat ,not much surger I don t drink. My Query is;..Re: my boob length hair in that my onc. is amaxed that I still have 48% of my hair so,...How soon can I color it as I am a brunet with grey in the front it truley ages me & what should I use ? Doctor: Thanks for your question on HCM. Yes, you can definitely use colour in your hair. No harm in it.But better to use standard manufacturer's colour.And these artificial colours can sometimes produce skin reaction if you are allergic to it.So better to first apply some colour on your skin and if you are not developing any rash, itching etc than it is safe to apply on hair.So no need to worry much you can apply hair colour."
},
{
"id": 205737,
"tgt": "What causes disorganized thinking and behavior?",
"src": "Patient: I have been dating a 59 year old man for nearly a year and have experienced multiple episodes where he says one thing one time, another thing another time--usually the opposite thing. An example is that one time, in an attempt to get me to remain in the relationship, he said he would propose to me in six months. A couple of weeks latter, he denied ever saying that to me, and any time I ve mentioned, hoping for an explanation, he remains silent. There were other times he said things like I ve been single for 14 years and tired of it, want to get married . A short time later, he said I ve been single for 14 years and do not want to get married . He also seems often unable to make decisions, unless either I or one of his family (usually his sister) urge him to decide. He recently spent over 6 months trying to get some very simple repairs done to his house. He insisted he couldn t get anyone to do them. I finally intervened and got a contractor I know to do the work. I do know that he has, in the past, one time 15 years ago, and another 1 year ago, had what the family refers to a mental breakdowns, the last time spending a month in a psychiatric unit, with no recovery, until seen by a psychiatrist, after he left the hospital. Not being a family member, I am not privy to his diagnosis. I just know that he takes a large number of medications. Another thing he does is sometimes come out with totally inappropriate, and what I consider bizarre, comments. A couple of examples: At the end of a regular checkup by his family Dr., after discovering they d both graduated H.S. in the same year, he said to her, Too bad we didn t know each other in high school, we might have got together . He couldn t understand why she just turned and left the room, not answering. He also said recently to a grocery store check out person (a girl): You re pretty good. You re pretty and you re good. Now, maybe that o.k., but it seemed strange to me, the way he later went around, boasting of what he d said, as if it was a really brilliant remark. The latest version of his lack of planning or whatever this suggests, is that he s been off-on re. marrying me for many months, in a very confusing way. So, I gave him the date of May 31 as a time for him to make a decision. The above described self-contradictory remarks re. marriage caused me to try to break-up with him several times, but I truly love the guy--the part that is sweet, kind, very lovable/affectionate. Anyway, with May 31 still a couple of months away, and without ever proposing to me or telling me of his decision, last night, at dinner, his sister said he remarked that She (meaning me) will sell her house here and move to ____ (where he has a house). I will rent out my house. We will buy a house there . There s nothing wrong with all he said, except we ve never made the decision to marry. I sure could use some professional help now. He really is nearly driving me nuts, and I myself am hovering close to depression (which I ve had in the past) due to his disorganized thinking and behavior. What would be the possible cause(s) of his disorganized thinking? Doctor: Thanks for being concerned about your partners health. I would directly answer your question by stating that your partner has a thinking problem. You mention that he was hospitalized a long time back, for a mental breakdown. There is only one possibility. I will politely inform you that, given the graphic descriptions of your partners behaviour, he may be suffering from Schizophrenia, Paranoid sub type. He might also be suffering from OCD (Obsessive Compulsive Disorder), in tandem with schizophrenia or as a distinct illness. I have answered your query by making a diagnosis based on your detailed description of your partners behavioural patterns, thoughts, speech, memory and actions as a schizophrenic with OCD, or as a person who suffers from Obsessions alone. Please take care of your partner. He's dilly dallying the marriage, but he's not doing these things to you on purpose. He just cannot help it. He needs professional help and medication. You must regularly give him the medication and make sure that he takes it in front of you.If you can accomplish this, he'll become \"normal\" and take the right decision to marry you. I really hope this helps you.Please feel free to re-post and take care of your health too."
},
{
"id": 161257,
"tgt": "Are spots on face of a child be due to use of medication for asthma?",
"src": "Patient: My daughter ,she is 6 years old, has Asthma problem, She is taking MomtekLC , Seroflo125according to need. From last few days I have observed some spot are developed on her face thgese are also touchable. Are this reaction of taking medicine? What I will do now please suggest me. Doctor: Hi, What spots white color. If candid patch is developing your child needs Candiderma lotion. Everytime after giving Seroflo please spit with water after so to ensure medication is not lodged in mouth. Hope I have answered your query. Let me know if I can assist you further. Regards, Dr. Prasanna Lakshmi, Pediatrician"
},
{
"id": 83936,
"tgt": "Is there any side effect of swallowing hydrocortisone ointment?",
"src": "Patient: Been having bad allergies in the middle of the night. Accidentally snorted hydrocortisone ointment. It was on a tissue which is used to blow my nose and a tiny bit went up my nose and I swallowed it. I had induced vomiting but am still concerned. Should I go to the hospital? Doctor: Hi,Do not worry. A little amount is not of major concern if you do not have any symptoms or discomfort. In case you have any sneezing or cough, you may visit your doctor.Hope I have answered your question. Let me know if I can assist you further. Regards, Dr. Saranya Ramadoss, General and Family Physician"
},
{
"id": 141436,
"tgt": "What causes tingling sensation in the lower extremities?",
"src": "Patient: Hi, I\u2019ve been diagnosed with spinal stenonis, within the past 10 weeks, I have gone thru rehab for 30 years of chemical dependency from opioids, benzos, muscle relaxants & antidepressants. Consicuently, I am experiencing increased pain lately. My concern is that I am feeling sensations and tingling in my lower extremities. Should I be concerned? Doctor: Hello and Welcome to \u2018Ask A Doctor\u2019 service. I have reviewed your query and here is my advice. That is the result of the spinal stenosis which is usually due to arthritic and common degenerative changes in the skeletal structure which is an aging process and not likely to be strongly associated with the chemical substance abuse issues. You should see a neurologist for a full scale examination and participate in aquatherapy and/or land physical therapy as recommended in order to reduce or slow down the pain and other sensations you describe. Osteoarthritis (likely problem you have) is not curable but it is definitely manageable with proper diet, exercise, and other precautions. Hope I have answered your query. Let me know if I can assist you further."
},
{
"id": 163347,
"tgt": "Suggest treatment for weakness in right arm",
"src": "Patient: my 8 month old has some minor right arm weakness. he uses his right arm but will not lead with it unless he has to. dr. has refered us to a neurologist for further workup. Both legs are strong and equal in balance sits up well with no other physical issues. any suggestions. Doctor: Hello,Any focal lesion can cause the problem if the condition is improving give it time to completely recover if condition getting worse then see a neurologist for any demyelination nerve conduction studies is necessary.Hope I have answered your query. Let me know if I can assist you further. Regards, Dr. Hina Javed"
},
{
"id": 209672,
"tgt": "Does masturbation cause poor concentration and memory?",
"src": "Patient: Hello sir , i am masterbuting last 4 years . I do usually 2 or 3 times in a week . Now i am feeling that my concertration and memory power is gone away .nothing is remembred after reading . What to do now sir . Please help me sir . I am loosing my carrier .amit ,india , Doctor: Hi,I read your query and do understand your concerns. Since you have mentioned about masturbation, it seems that you feel your symptoms are due to masturbation. Let me clear some misconceptions first. Masturbation is a completely normal physiological process. Once a person becomes sexually mature, semen production starts which is a continuous process. A person the either has sex, or masturbates and has night falls. Masturbation doesn't lead to any physical or mental health problems. It also doesn't cause any problem with sexual performance, and potency. So you can masturbate without worrying too much. That brings us to the symptoms of impaired concentration and memory . It might be the result of anxiety and being preoccupied too much about certain things. I would suggest you consult a psychiatrist for detailed evaluation. Treatment with medications like sertraline or escitalopram might help you with your symptoms.Hope this information was helpful. Best wishes."
},
{
"id": 67305,
"tgt": "Suggest treatment for painful bump and bruise on the head",
"src": "Patient: About 3 days ago I was hit in the head with a golf club, I did not lose consciousness I was not dizzy nor did I have a headache I have a slight bump and bruise and the area is slightly painful. I did not go to the emergency room but I was not sure if at any point I should I am nervous and usually like to precautions however becuase there were no complications I didn t take action and I m not sure now if I should . Thank you very much Doctor: Hi,Nothing to worry as now three days passed having hit on the head.due to hard hit there might be having some stiff muscles and muscular strain causing this trouble,Apply ice pack for few days and you will be alright.Ok and take care."
},
{
"id": 72891,
"tgt": "What causes cloudy vision while on antibiotics for URI?",
"src": "Patient: For the last few weeks I have been waking up with my eyes matted shut even though I have been though I was on an Antibiotic for an URI so today I can barely see my eyes feel like they have a thick cloud over them. I have blurred vision or Cloudy Vision. I take Allergy meds every day and I put drops in my eyes at night but nothing has been helping. Doctor: Hello dear , hiWelcome to Healthcaremagic.comI have evaluated your query thoroughly .* This seems in relation with side effect of medicine ( though better insight would have been possible if the name of medicine was available ) most probably , immediately stop its further doses .* Also withheld the intake of allergy meds as well .* Maintain hydration with plenty of liquids .Hope this will help you for sure .Regards ."
},
{
"id": 61048,
"tgt": "Is a lump encircled by redness on the clavicle malignant?",
"src": "Patient: I\u2019m a healthy 66 yr old female who has a suspicious bump with white center surrounded by redness, 1/2 millimeter in size, about 1/2 \u201c below my right clavicle. I\u2019m interested in seeing pictures of pre-cancerous spots to see if it is anything to be concerned with. I do not identify this as a mole. First noticed middle of December & has grown since. Doctor: Hello dear Warm welcome to Healthcaremagic.comI have evaluated your query in details .* Possible sebaceous cyst which is getting inflammation or infection more likely than cancer .* Needs clinical confirmation with examination by consultant surgeon .Wishing you fine recovery .Feel free to ask any further doubts .Regards .Dr. Bhagyesh ( MS , FMAS - consultant surgeon )"
},
{
"id": 11921,
"tgt": "White patches on legs, thighs. Prescribed melgain, exposure to sunlight required. Vitiligo?",
"src": "Patient: Hi,,some white patches are developed on my legs and thighs,over the last two months..I have consulted a dermantologist..he advised me to apply melgain in night and advised me to expose my legs to sunshine in the morning..Is this white patch is vitilgo?..whether it will spread to other body parts also?..I am so tensed..thank you Doctor: Hi...dear Harinarayana..., Thanks for choosing HCM.., Yes....follow the treatment.., It is vitilligo.....If treatment starts..., Spreading of disease....will control.., But it will take more time to respond.., Now very good update treatment is available.., thanQ"
},
{
"id": 1557,
"tgt": "What are the chances of conceiving with sperm motality index of 260?",
"src": "Patient: Hello doctor! i(32) and my husband(38) married for 7 yrs and trying to conceive for last 2-3 years but not success yet.I had series of check up with the normal report like HSG study having normal uterine cavity, smooth period in 29-31 days but 2 ultrasound in between 2 yrs period shows that i have fibroid of 49*39*55 mm on uterine body.Hypoechoic area of size 55*53mm seen in the anterior side of uterus and my husband has semen analysis which shows that mortile sperm of 235.8 million, Appearance Grey white, pH 8, Concentration 92 million/ml, Mortile sperm conc. 52.4 million, Functional sperm conc.36 million, Sperm mortility index 260 but have infection most of the time with Staphylococccus species with Ciprofloxacin sensitive and now according to urologist he is having Cifran 500mg twice a day for two weeks. Can i get pregnant?we are so excited to have baby.Please help nd suggest for further treatment. Doctor: Hi, I think if your fibroid is intramural or submucosal, it needs to be removed as it is large in size. Your husband semen parameters are normal. He can take a long course of antibiotics and antioxidant also. Hope I have answered your question. Regards Dr khushboo"
},
{
"id": 189153,
"tgt": "Upper lip sore and irritable. Why is it swollen now? What can it be?",
"src": "Patient: Yes concerned about under the inside of my upper lip right where it comes down toward my lower lip was sore around the edge last night felt sore and irritated, this morning it is slightly swollen and sore in that area? never had this before appears there may be a tiny sore spot under my top lip right where it would touch my left front tooth if my mouth is shut. I have regular dental check ups and very good oral health as a rule what might this be? Doctor: Hi, Thanks for asking the query, According to your clinical symptoms i suppose that white spot on the inner surface of the tongue can be due to aphthous ulceration, this can occur due to trauma, viatmin deficiencies, immunologic disorders. Apply cortisone acetate topically over te affected area. Start taking multivitamin suplements. Avoid eating of hot and spicy food stuffs and carbonated drinks. Take complete balanced vitamin rich diet, drink lots of water and keep yourself hydrated. Take lukewarm saline and antiseptic mouthwash rinses. Hope this helps out. Regards..."
},
{
"id": 22387,
"tgt": "What causes rapid heart rate with tightness in chest of a 14 year old?",
"src": "Patient: I have a 14 year old girl that has a occasional fast heart beat. It suddenly starts and stops. Took her to ER, they ran EKG, xray and blood work said everything is normal. Now she has chest pains in the middle of her chest with some tightness. What is going on? Doctor: Hi,Increased heart rate and palpitations can have a lot of causes. It can be normal sinus tachycardia due to anxiety or stress or it could be abnormal supra ventricular tachycardia. For the diagnosis, you need to do ECG at the time of palpitation and tachycardia. If the duration of palpitations are short enough to reach a nearby hospital, we put a 24hr ECG monitoring (Holter Monitor). You also need to do a 2D Echo and thyroid function test to see for any associated abnormality if any.Hope I have answered your query. Let me know if I can assist you further. Thanks,Dr. Sameer Maheshwari"
},
{
"id": 24210,
"tgt": "Suggest treatment for blocked artery in heart",
"src": "Patient: The person Iam talking about, The main artery to heart has blockage dr. said like a . (dot) in the middle, That the blood is moving but as he has pains it will close.. cant have surgery cause of 1 kidney and dye will shut it down.. The 1 kidney is 2.7 and on oxygen 24/7 ...Do you think his head will and neck will start to hurt..because of the heart? Doctor: Hello!Welcome and thank you for asking on HCM!Regarding your concern, I would explain that there are also other possible treatment options besides surgery (or coronary bypass) for coronary artery disease. Coronary catheterization coupled with stent implantation is the main treatment option in such cases. You should discuss with his doctor on the above possibility. Hope you will find this answer helpful!Kind regards, Dr. Iliri"
},
{
"id": 96826,
"tgt": "What is the next treatment for the patient in coma?",
"src": "Patient: hi i am looking for a answer about one patient who has history of mitral valve stenosis and repaired with prosthetic,h/o lacunar stroke,recently became unconscious and taken to the hospital where she is in coma GCS E2V2M2, developed subdural and subarachnoid hematoma with hydrocephalus ....mri is done but they can not do angiography due to her prosthetic valve,she is in mechanical ventilation.what is her next treatment line would be and what her chance to survive or recover Doctor: HelloThanks for asking the question.I am Dr Adil.After going through the details of your question, i noticed that your patient has many problems, she has mitral valve stenosis, prosthetic valve, i presume that mitral valve stenosis may also caused her atrial dilatation and atrial fibrillation that had lead to the embolic stroke. Now she also has sub-dural and sub arachanoid hematoma. So these are so many problems, one correct one problem at a time, and all these problems require surgery and for surgeries and anesthesia one needs a good and sound heart and she is also on mechanical ventilation, and long term mechanical ventilation also compromise the function of lungs. I am not seeing a good chance of her recovery. But in medical sciences, miracle do happens. I wish her well and hope, may she wake up soon. Regards."
},
{
"id": 161269,
"tgt": "Does nuchal fold in fetus at 19 weeks cause Downs syndrome?",
"src": "Patient: Hi my name is sarah I have a few questions. At 19 weeks I was told my son had a nuchal fold of 5.1mm and that I was at high risk for down syndrome. I had my blood taking to see if my child bad any abnormalities and everything came back fine. I jus had another ultrasound done at 23 weeks and they told my son had to much fluid in his kidneys. I was told that was another soft maker. What are my chances of having a baby with down syndrome? I m 22 and my son had a nasal bone at my 19 week check up. Doctor: Hi, Nasal bone is not confirmatory. If suspicion of downs is present, get an amniocentesis and karyotyping. If it is suggestive of 21 trisomy, it is confirmed Down\u2019s syndrome. After that, you have to get yourself and partner checked for balance translocation to avoid in the next pregnancy. Hope I have answered your query. Let me know if I can assist you further. Regards, Dr. Prasanna Lakshmi, Pediatrician"
},
{
"id": 36091,
"tgt": "What is the cause for vomiting during fever?",
"src": "Patient: hi i m 25 yr old male suffering from fever from five days... nobody is der in home i m alone... yesterday contacted with a doctor... two sline n few injections were put in by him... he suggested 3 medicine viz mahacef plus lumerax80 n crocin... in my blood report its written paracite mp/mf ict negative.. i hvnt eaten from five days wenever try to eat... its come out as vomitting.... now wat should i do next? Doctor: Hello dear,Thank you for your contact to health care magic.I read and understand your concern. I am Dr Arun Tank answering your concern.As per your report nalaria is negative. But you haven't tested for the dengue.I advice you to test yourself for the dengue. If you are having dengue fever and you received such a high dose antibiotics than it is a acidity which is causing a vomiting to you.I advice you to take the pantoprazole and domperidone tablet under your doctors guidance. It will clear up the acidity and helps reduce vomiting.Please take plenty of water and ORS as it can help you balance your fluid balance.I will be happy to answer your further concern on bit.ly/DrArun.Please hit thank you for me if you are helped.Thank you,Dr Arun TankInfectious diseases specialist."
},
{
"id": 67739,
"tgt": "Suggest remedy for a hard lump on the clavicle bone",
"src": "Patient: I noticed a hard lump on my clavicle bone at the inside of the right side. It did not hurt but is noticeable. I m not sure if I m imagining some soreness in right shoulder and neck area because of thinking about it, but I do believe I have some minor soreness. What could be the cause of the knot and are the two related? Doctor: Hi! Good afternoon. I am Dr Shareef answering your query.If I were your doctor, I would examine the lump physically to see if it was originating from the clavicle itself, or it was a lump just adjacent to the clavicle. In the second case , it could be a lymph node swelling common in the cervical region and there has to be a primary focus of some infection or pathology for the lymph node to enlarge. So a detailed physical examination would be needed before deciding on the plan of management. Therefore, I would advise you to please take an appointment for a general surgeon in your area and get it assessed by him. Till then you could go for an anti inflammtory drug along with a proton pump inhibitor for a symptomatic relief.I hope this information would help you in discussing with your family physician/treating doctor in further management of your problem. Please do not hesitate to ask in case of any further doubts.Thanks for choosing health care magic to clear doubts on your health problems. I wish you an early recovery. Dr Shareef."
},
{
"id": 204041,
"tgt": "Male having hot flashes, back pain, diarrhea, headache, loss of appetite and blood in stool. What is wrong?",
"src": "Patient: Hi I am a 39 year old male and recently started having hot flashes, bad lower back pain , change in stool (not solid, more soft, diarrhea sometimes), shakes, loss of appetite to where I can hardly eat anything, no energy at all, recently started having blood in stool and coughing up blood which is bright red not coffee ground like, fatigue to the point I am sleeping more then I am awake. There are more symptoms to which I am having as well. I am getting very worried as to what is going on and need some advise and a little more knowledge on what could be the cause of all these symptoms. This has happened before about 3 years ago and has now started again but with much worse and intense symptoms. I am also having severe headaches and at one point had one about everyday for almost a week or so. Please help me as I am very worried as to what is going on. Thank you Doctor: Hi. Your age and symptoms are suggestive of a tumor, may be a carcinoid tumor for which you must consult a Gastro surgeon , undergo CT scan of abdomen and routine as well as specific blood tests like % HIAA and as advised by your doctor."
},
{
"id": 2451,
"tgt": "What are the chances of pregnancy after taking ovacet and ubiphene for PCOS?",
"src": "Patient: hello doctor. my name is mridula. I am 25y old. I have pcod from 3 years. I want to conceive and its getting delayed. my follicles don't develop to the required size. I am advised to take ovacet and ubiphene once in a day. today is D8, and I am really worried. last cycle I got iui done but it didn't work for me. what are my chances this time? Doctor: You must take clomiphene for five days and then follicular study on tenth day.. Dose of clomiphene 50mg"
},
{
"id": 51120,
"tgt": "Removed kidney stone. Have diabetes. Swollen face after ESWL. Cause?",
"src": "Patient: My mother just has ESWL for removing the kidney stone in her left kidney. Just to let you know that she has diabetic and has performed conventional surgery and URS two years ago for the same problem. Unlike those previous therapies, after ESWL my mom s face looks little bit swollen. Could you please let me know what is the possible cause of this? Doctor: Hi Thanks for the query. ESWL by itself is unlikely to cause facial swelling. There are a number of possible reasons to get facial swelling like excess fluid administration, drug allergy or low serum protein levels to name a few. If the symptoms persist consult a physician who will examine her and suggest further investigations and treatment. Good luck."
},
{
"id": 116402,
"tgt": "Can low platelet count cause burning in chest?",
"src": "Patient: 2 months ago I was diagnosed with low platelet count. was sent to the hematologist and all they keep doing is rechecking my platelet count and not doing any treatment. last month started having chest pain thought It could be due to the low platelet. went to ER they could not find anything. my regular doctor sent me to have stress test the first one came back slightly abnormal in the nucular stress test came back normal. did not have no chest pain when having both tests done. today I have had burning in my chest off and on and everytime Iget the burning in chestI also get a pain close to my left wrist Doctor: Hi,Thanks for asking.Based on your query, my opinion is as follows.1. Low Platelet count would not cause burning chest pain.2. Possibility of internal bleeding secondary to gastritis/ GERD or angina pain are usual suspects.3. During internal bleeding, thrombocytosis is commonly seen. If the nuclear stress test is abnormal, further angiography is necessary to evaluate for any blockage. You can start on proton pump inhibitors like pantoprazole for a week and see if the pain reduces.Hope it helps.Any further queries, happy to help again."
},
{
"id": 37087,
"tgt": "What do these ALT and AST levels indicate?",
"src": "Patient: After having viral pneumonia for several weeks my dr ordered blood tests. My alt was 122 and ast 45. Two weeks later my alt is now 244 and ast is 90. He has scheduled an ultrasound. Btw all my other blood tests are all in normal range. I am concerned what all this means Doctor: Hello,Welcome to HCM ALT and AST are liver enzymes.This can be because of a viral infection of the liver. The most common causes of viral hepatitis are the five unrelated hepatotropic viruses Hepatitis A, Hepatitis B, Hepatitis C, Hepatitis D, and Hepatitis E. In addition to the nominal hepatitis viruses, other viruses that can also cause liver inflammation include Herpes simplex, Cytomegalovirus, Epstein\u2013Barr virus, and Yellow fever.Hepatitis can also be caused by medicines , alcohol, supplements and some chemicals . The inflammation usually settles once the cause is removed.If you have more queries, I am availableTake CareDr Noble Zachariah"
},
{
"id": 224458,
"tgt": "Why am i having pregnancy symptoms while on nuvaring?",
"src": "Patient: Hi I an on nuvaring and I use it where I font have my period at all. Lately I've been having alot of symptoms of pregnancy and I've even gained 15 pounds and I can tell my stomach is larger but all my pregnancy tests have been negative.... What could be wrong with me???? Doctor: welcomegainig weight is not the symptomp of pregnancy...only urine pregnancy test and sonography can surely tell about your pregnancy...symptomps ae nonspecific..so dont rely on symptomps...nuvaring itself has sideeffect of weight gain...proper use of nuvaring will not have such symptom...thanks"
},
{
"id": 79625,
"tgt": "How to treat thick cough?",
"src": "Patient: I have a thickly cough but am experiencing a burning sensation in upper part of my back to the left of my spine I saw my GP about the burning sensation 2 weeks ago ( didn't have the cough then) but he said its nothing to worry about but I am worrying because he didn't examine me, I've asked him to check but he thinksim depressed so won't examine me! What should I do? Doctor: Thanks for your question on Health Care Magic. I can understand your situation and problem. Since you are not improving and having cough with thick phlegm, possibility of lower respiratory tract infection (LRTI) is more. So better to consult pulmonologist and get done clinical examination of respiratory system and chest x ray. Chest x ray is needed to rule out lung infection. You may need antibiotic, mucolytic and expectorant drugs. Mucolytic drugs will dilute the thick mucus and makes it easy to cough out. So first diagnose yourself and then start appropriate treatment. Hope I have solved your query. I will be happy to help you further. Wish you good health. Thanks."
},
{
"id": 209864,
"tgt": "Need treatment for paranoid disorder",
"src": "Patient: A highly intellegent 16 that was attending an advanced schooling program started to begin acting inappropiate with classmates: laughing at unusual times, staying up for long periods of time, drinking lots of caffiene and no sleep, when he was getting ready to have his pictures taken for school he could not emotionally take it and crawling into a fetal position and began crying, paranoid with classmates Doctor: Hi,The behaviour of the person described by you is highly suggestive of a psychiatric disorder, most probably some psychotic condition. However, for a definitive diagnosis he would need complete evaluation with more details like duration of illness, aggressive behavior, etc. It would be advisable to consult a psychiatrist for the same. Investigations like brain scanning may be needed to rule out any organic brain lesion.Assuming he might be suffering from psychotic disorder, he would need treatment with anti-psychotics like risperidone, olanzapine, etc. Hope this information was helpful. Best wishes."
},
{
"id": 8888,
"tgt": "Is sperm in urine the cause for pimple on my face ?",
"src": "Patient: hiiii dr. im tarun age 17.. sperm in urine cause pimple on face .??? im getting too many its because of this reason?? plz suggest me wat should i do Doctor: hi tarun dont stress much sperm in urine has no relation with ur acne as you grow younger sperm production start its natural process acne is due to hormonal changes . so dont worry. consult dermatologist nearby you , u will be ok"
},
{
"id": 202904,
"tgt": "Have small testicals, erectile dysfunction, pain in testes. Concerned",
"src": "Patient: im 27years sir my testis are both very small and left was too small compare to right testicle and and my penis not getting good erections even in sleeping also. and my scrotum is hanging too much and my testes are paining frequently particularly left which is too hanging of testicle. and i want to know my sexual strength and sperm count with action positions etc... Doctor: HelloThanks for your query,based on the facts that you have posted it appears that you have small testicles with problem of erectile dysfunction.This could be due to hypogonadism .However this has to be ruled out by clinical examination by a qualified Endocrinologist and following tests.1) Detail Hormonal assessment like Serum Testosterone ,Growth hormone ,LH etc .2) Ultrasound scanning of Scrotum.3) Colour Doppler study of scrotum.(To rue out varicocele on Lt side)As regards your sperm count get your semen analysis done to know more about your sperm count and morphology of sperms.Dr.Patil."
},
{
"id": 151871,
"tgt": "Which neurologist or hospital should we contact for brain embolization ?",
"src": "Patient: Friend has a cerebral attack, he was in a coma , now he is paralized on the right side hi , my name is maria and i am from romania i have a question, i have a frind who is now on a bed in hospital he has a cerebral attack, he was in a coma, now he is paralized on the right side, the doctor frome here tell us that he need an embolization. here in romania is very hard to do this , we have good doctors but we don`t have performance technology. we whant to fine a good doctor and performance technology out of our country, where do you think i can finde this? regards sweet_mady2003@yahoo.com Doctor: You can come to india and take treatment here lot of inter-nation hospitals are there like Columbia Asia hospital"
},
{
"id": 127827,
"tgt": "How to treat painful hematoma on the upper arm, elbow and shoulder area?",
"src": "Patient: Yes sir, please.....I was in an ATV accident and have a horrible hematoma on my upper arm from elbow to shoulder.....I had 7oz. Of blood removed from it last Tuesday. It it is still causing me terrible pain. What can I be doing while at home to help ease any pain Doctor: Hello,I can suggest rest to the arm. Ice packs and tablets containing proteolytic enzymes to decrease the swelling. Please don't do any hot fomentation. If you are on any blood thinning tablets, then it will take still more time. The usual time for full recovery is about three to four weeks. Hope I have answered your query. Let me know if I can assist you further.Regards, Dr. Santosh S Jeevannavar"
},
{
"id": 209950,
"tgt": "Why am i unable to walk and feel weak?",
"src": "Patient: Thank you fore this chance.My name is Misrak Paulos.I am male in sex.I am 50 years old.80 kg weight and 175 C.m. height. I am now almost unable to walk.There is no reason for this.It started before five years.Now it seems progressive and gradually increasing.My right leg is very weak and senseless.My left leg is not weak but more senseless than my right leg.When I test my sense organs using hot water,the left leg is senseless unless the water is very hot than the right leg.My right hand is also weak.I feel high burning feeling on my knee and under my feet.I cann't resist cold whether but when i walk,I sweet easily and highly.Some times i cann't control my west but when i thought about my urine,I cann't control it completely. Doctor: HIThanks for using healthcare magicI think, you have some neurological disorder, may be spinal cord compression. In that case, better to get your MRI scan done and try to find out the severity of compression. Nerve usually irreversibly gets damage, in that case, chances of recovery is less. Better to consult a neurologist or neurosurgeon for proper diagnosis and management.Thanks"
},
{
"id": 64660,
"tgt": "What causes small hard bumps on the head?",
"src": "Patient: HI, I have a problem or so think. I have several small bumps on the head. They do not hurt or cause me any discomfort at all. However, they top of them becomes hardened after a while and peels off and when this happens a little bit of blood come out. I was wondering if you (someone) could give me an idea of what this is? Doctor: Hi,Dear,thanks for the query to my HCM virtual Clinic.I studied your query in depth.IN My opinion -these lumps are of Chronic Acne ? keratosis Pilaris.I would advise you to Skin doctor -who would treat it on lines as above.Hope this would resolve your query.I would love to help you for more queries.Good day."
},
{
"id": 49443,
"tgt": "Why has the person who had the drug for induced coma have not woken up after stopping medication and on dialysis?",
"src": "Patient: my brtother had complcations after a heart surgery kidneys shut down they used a turbo lung resperator which caused the lungs to be very inflamed.They put him in a drug induced coma , stopped the drugs 5 days a go but he has not woke up??? why??? still can not breathe on own , and on diallies keeping vitals stable with meds.. any answers would be a great comfort ty Marie Doctor: Hi,Recovery following heart surgery can be complicated by many setbacks. Some degree of kidney impairment is relatively common after major surgery. Most drugs are passed out of the body through the kidneys so if the kidneys shut down, the drugs remain in the body for longer. This may be partly the cause of his prolonged coma. Another contributing cause may be related to neurological complications. There is a possibility he may have had a stroke and this will delay his recovery.I suggest you speak to his doctors are they will be able to explain the important details to you more fully.I hope your brother makes a speedy recovery.Regards,Dr K A Pottinger,MBChB FRCA"
},
{
"id": 170992,
"tgt": "Is it to be concerned about the lumps due to insect bite in child?",
"src": "Patient: big, red, swollen bug bites. they are hard to the touch and getting bigger, my son is 4 years old and he was playing in the durt and mud in the garden, he had a few little bites looked like mosqutos bites and in the morning he is covered by this huge and very itchy red lumps, what is it?? Doctor: HiMost likely it is due to allergic reaction to bug bites. It happens due to release of histamines and other inflammatory mediators body due to insect bites. Don't worry. You may give him antihistamine such as Chlorpheniramine or fexofenadine for relief. Apply lactocalamine lotion on the lesions for early recovery.In case he develops any breathing difficulty, then visit an ER for emergency treatment. He may require steroids and adrenaline for treatment.Hopefully this will help you.Take care"
},
{
"id": 9135,
"tgt": "I am worried about my 2 yrs old kid that my thoughts and behaviour may effect his future life",
"src": "Patient: I am 32. I am an Engineering grad but a house wife. There s always waxing and waning in my thoughts. Sometimes i feel very confident and someother time i feel very low. most of the times i feel worthless and feel like dying. I hate my mom and the way she brought me up. she always restricts me from mingling with others which i like very much. again i feel that i am thinking wrong about my mom. I have 2 year old kid. My thoughts and behaviour may effect his further life. I am totally worried. very confused about myself Doctor: Hi I hope this answer finds you in better health than when you wrote it. Low mood, negative pattern of thinking, mood swings etc are suggestive of depression amongst many problems. There are many ways in which this can be helped (Medication and formal taking therapies for confiremd clincial depression ) and if it is not , then as well, talking about your problems itself can help to a large extent. Do consult a mental health care professional. GoodLuck"
},
{
"id": 92675,
"tgt": "Abdominal pain, watery bowel movements, hurts to sit up. Negative pregnancy test. Help",
"src": "Patient: Hi, I've been expericeing sharp abdominal pains for over 24 hours. It lower stomach area like right above pelvis/privates the flat part before gut. I took pregnancy test negative. I took laxatives and felt like I was dying and I've had several watery bowel movements. I still don't feel any better. It hurst to sit up, sneeze, cough, have sex, have a bowel movement. Please help. Doctor: Namaste Welcome to Healthcare-Magic Greetings of the day Consume plenty of water, avoid dehydration by taking oral rehydration solution. Tablet Metronidazole taken thrice a day will provide much needed relief. If the pain worsens or fever occurs then don't hesitate to consult your physician. Take care Regards"
},
{
"id": 83789,
"tgt": "Is loss of hearing a side effect of folitax intake for RA?",
"src": "Patient: hi sir. my mother in law is taking folitrax since 3 months. she is sufeering from rhematoid arthtritis.. she had taken it 2005-2008.. now she is complaining of stuffed ears. since 2 weeks.. she cant hear if we talk from bit far away.. is it a side effect of folitax Doctor: Hi, The common side effects of folitax include ulcers in the oral cavity, loss of hairs, liver injury, megaloblastic anemia, increased risk of infection, diarrhea and infertility. Based on the history she seems to be suffering from infection for which her doctor should be consulted for further evaluation and treatment. Hope I have answered your query. Let me know if I can assist you further. Take care Regards, Dr. Mohammed Taher Ali"
},
{
"id": 76572,
"tgt": "What does chest pain cough with asthma issues suggest?",
"src": "Patient: Hi, My Son complaint of sever pain in his chest. He said he had the worst cough so far and following that he had this pain and then itching. He has had asthma issues and we observe there is some complication with him lung which no doctor has yet diagnosed.He is 14 years and 82KG. I am just worried what could be wrong and how this could be treated? Doctor: Hi welcome to HCM...Noted you have diagnose with asthma ...It is allergic condition in which patient having hypersensitivity to certain allergen...For confirmation pulmonary function test done...If repeated asthma attack occurs then Metered dose inhaler like long acting salmetrol or formeterol with steroid needed....As asthma allergic condition itching can occur by histamin when exposed to allergen...Try to identify allergen or do allergy test ad keep exposure to it minimum...Your child also overweight ...Reduce his weight ...As obesity create prolem in breathing...Passive smoking exposure avoided...Onion having antiinflammatory action .So take cooked onion ...Take care."
},
{
"id": 39736,
"tgt": "Suggest treatment for sore toe",
"src": "Patient: Hi my 17 yr old is off sking tomorrow night and last night started c/o sore toe. Its red and inflammed, no puss but sore to touch. She has been bathing it an antiseption liquid. Any adice re treatment and having her comfortable sking for the next 8 days. Diane Doctor: Hello,Welcome to HCM,As you are having a sore toe which is evident by redness, pain and swelling, which should be manged thoroughly. Physical examination is must before the initiation of the treatment. The cause for the sore toe should be found out and then it should be managed appropriately.For your symptoms I would suggest you to follow1.Tab Augmentin, 675mg, twice daily for 5 days.2.Topical T Bact ointment 3.Oral Analgesics to relive the pain.These medicines can be taken only after consulting your personal doctor.Thank you."
},
{
"id": 196881,
"tgt": "What causes redness and pus pocket at the back of the foreskin?",
"src": "Patient: my son is 4 qnd woke up this morning his penis was all red sent him to school picked him up at lunch qnd brought him home went to go chqnge him and noticed it had gotten worwe when i push the skin bqck he had a puss pocket right where the skin is still attached now its still red and leaking puss and i dont know weather to take him to the er or wait till mornin Doctor: HelloThanks for query .Based on the facts that you have posted your son seems to have developed infection of Foreskin (Posthatitis).You need to give him broad spectrum antibiotic like Cefotaxime along with anti inflammatory drug like Diclofenac twice daily along with topical antibiotic ointment like Neosporin twice daily.Ensure to wash his genitals with warm water twice daily.Dr.Patil."
},
{
"id": 33013,
"tgt": "Is anti rabies injection required after dog bite?",
"src": "Patient: yesterday a three month old dog bite my child(3year old).The baby dog is now not eating nothing ,but it is alive. There are 3-4 scratches in my child's leg.They are not deep.I washed the wound immediately in running water and applied dettol (liquid).Now shall I give him anti rabies injection ? Doctor: HelloWelcome to Health care Magic.I reviewed your history.Antirabies vaccine is needed after any dog bite, hence I advice you to consult your doctor for antirabies vaccine.Thank you"
},
{
"id": 76133,
"tgt": "What could possibly cause 1cm nodule on right lung?",
"src": "Patient: hi i am wondering if a 1cm lung nodule in the right lung and also shadows on the lungs my husband was took very ill in march with h1n1 and doube phneouma which quickley developed in to ards he was on a vent in intesive care for 4 weeks did not know if he was going to pull through but he did .he is now at home gets out of breath very easily still under the hospital having further tests for lung cancer do u think this could be some type of cancer we are all so worried thank you joanne Doctor: Thanks for your question on Healthcare Magic. I can understand your concern. It is very difficult to understand conclude about having cancer or not. But 1 cm nodule in lung should definitely be investigated for lung cancer first. Lung cancer like squamous cell carcinoma and adeno carcinoma can present as nodule and shadows in lungs. So better to consult pulmonologist and get done 1. CT thorax 2. Bronchoscopy and BAL (bronchoalveolar lavage) analysis 3. CT guided Biopsy from the nodule. 4. Blood tumour markers. Hope I have solved your query. I will be happy to help you further. Wishing good health to your husband. Thanks."
},
{
"id": 139992,
"tgt": "What causes prickling in neck,tingling in arms,dizziness and unconsciousness?",
"src": "Patient: I am having strange prickling in my neck that move up I feel dizzy and nausea 3 weeks ago was so severe I lost consciousness. My arms also have sort off electric like feelings my neck and face get red also. Every day after I have been up awhile These symptoms start and progressively get worst. I have had a neuclar stress test and was fine also have had ct scan but have not gotten results as of yet. I get very tired easily and am not functioning well at all. I am 60 female, have had 3 stents in LAD and have left internal occluded temporal artery. Ex smoker for over 4 years have excercised and tried to eat well and just keep have one ironic problem after another. But have never passed out until recently. My weight is 126 at 5 ft 2. What kinda of test do I need to be seeking? Doctor: Hi, First of all, you should measure your blood pressure and if it were high, you should receive a medical treatment and do an ECG and an ECHO on your heart to assess the ejection fraction. Second, if you took oral contraceptive pills or a hormonal replacement therapy,you should ask a gynecologist for help. Second, you should do ESR to assess the left temporal artery occlusion. Besides taking medical treatment for your heart condition, you should change your lifestyle by stop eating too much fat and salt in your food. Hope I have answered your query. Let me know if I can assist you further. Regards, Dr. Mustafa, Neurologist"
},
{
"id": 35234,
"tgt": "Is it dangerous to have mild temperature after typhoid treatment?",
"src": "Patient: i m zeba /19 yrs was suffering from typhoid from past 1 month even i was hospitalize within one week every thing was okay and my antibiotics course is also complete. but after 2-3 days wen i again check ma body temprature it was 99-100. so doctor consulted me for Wedal test but the report is negative..i still have mild temprature. so what shall i do now...?? Doctor: Hi Do you have lost weight recently? When is the temperature more? During the morning or evening? Kindly get your complete blood counts with ESR which might point towards a diagnosis. Consult your doctor with the reports."
},
{
"id": 53578,
"tgt": "Is blood alt level of 60 dangerous?",
"src": "Patient: I had a alt reading about 2+ months ago of 43 then retested today and got a 60 alt reading my other readings came back normal should I be concerned?Im 40 years 5ft 3 and weigh 192albs have a history of chronic vertigo and insomnia, and chronic sinus problems. I take valium 5 mg 4 times a day, fluticansone 2 puffs each nose once a day, qvair two puffs twice a day to prevent weasing, claritin for allergies, and I do sinus rinses as needed. I am female.My doctor says none of the medicine has liver side effects, and I don't drink alcohol but a sip on the rare occasion. He thinks it could be do my weight and Im working on this with gradual exercise when Im able too. Should I be worried with this test result? I had one other result that was high my triglcyerides read 198 ? WWW.WWWW.WW Doctor: Hello and thank you for asking HCM.I have read your report,and I understand your concerns.First i would say to not worry.Normal Alt levels are 7 to 56 unit (depending from the kits that laboratories use)In one of your tests,your Alt was 60. This level is almost normal and it doesn't always mean a serious condition of your liver. Here are some reasons why a healthy person might have changes in the levels of liver enzymes:-alcohol-exercise -some medications like aspirin,paracetamol,antibiotics, Cholesterol lowering drugs such as statins, -stress-weightTo help your liver you can use these advice:-have fresh fruits and vegetables-minimize your alcohol consumption-drink plenty of water-limit junk food (fast food,pizza ect)-Whole grains and seeds are excellent source of fiber . Fibers pushes toxins through your digestive system faster so you get clean faster-lose waight if needed-Exercise regularly. It improves your health, regular cardiovascular/aerobic exercise can also help improve your liver health and ALT levels.If you have any other question,feel free to ask me.Dr.Endri Katro"
},
{
"id": 137402,
"tgt": "What causes pain in my instep on my left foot after stress fracture?",
"src": "Patient: I have consideral pain in my instep on my left foot. It goes around to the right of my ankle. I have had an Xray but no result yet, the doctor was wondering if it is a stress fracture, but thinks not. could it be something serious? I have had the pain since Dec but iut has become steadily worse. Doctor: your symptoms suggest that you are suffering from palntar fascitis .its a inflammation of muscles that supports the arches of foot. toe stretching and hot fomentation at early morning just after awakening and analgesics such as ibuprofen will be helpful to you. if pain does not subside in 5-7 days then you should consult to rehab specialist and physical therapist .Hope this helps.Hope this answers your question. If you have additional questions or follow up questions then please do not hesitate in writing to us. I will be happy to answer your questions. Wishing you good health"
},
{
"id": 170010,
"tgt": "Suggest remedy for bleeding from ears in a child",
"src": "Patient: Hi, I was cleaning my 7 year olds ears, with an earbud, I put it in too far. She cried for a minute but was ok, but when she was lying down I checked it & there was blood in here ear. I wiped it with a wet cloth, it has not bled again since & she says it s ok & not sore, should I get it checked? I m worried it will bleed through the night? Doctor: Hi.... I suggest you get the consultation of an ENT surgeon regarding this. I suspect that there could have been an ear drum perforation or it could be an injury to the external ear canal. Whatever it is, to diagnose the condition it needs a simple procedure called otoscopy where the ENT surgeon will look inside the ear and decide what to do.Regards - Dr. Sumanth"
},
{
"id": 1751,
"tgt": "Is there any chance of pregnancy?",
"src": "Patient: Hello Doctor , I am 47 years old , I tried twice to IVF with my own eggs . Three eggs each time , and after retrieval of eggs , doctors did not make the transfer , because the eggs were not good enough. Is there any chance in my age to get pregnant? Doctor: Hi, you can go for donor eggs- IVF. It will increase the success rates. Your age is high so your eggs quality is not good. Hope I have answered your question. if you have any other query I will be happy to help.Regards Dr khushboo"
},
{
"id": 64434,
"tgt": "Suggest remedy for lumps",
"src": "Patient: I have a lump on my right side of my neck near my collar bone...no larger than 1 cm... its been there for years and its not growing in size.... went to the er they said it was nothing and its too small to do a biopsy on.... I am curious what it could be. I have neck pain and its hurts to swalow...could this be a sinus infection? I have always had the lump but now..odd things are happening...would appreciate any info! Thanks Doctor: Hi,Dear,Good Evening and Thanks for your query to HCM.I went through your query deails indepth.-In my opinion,in relation to the facts told by you,its-Cervicle Lymphadenitis with Acute pharyngitis ? bacterial-TB sore throat ?-Dont worry for the sore throat with the painful lump..Consult ER doctor- who would do-Mountaux test or PPD test with IGRA blood test to confirm the TB lymphadenitis.-Excision biopsy would confirm TB diagnosis.-Remedy-With antibiotics treatment with persistant symptoms and lump would lead to above tests and only after confirmation of TB diagnosis,the anti-TB drugs treatment is started for 6-18 mths.Hope this would resolve your worries about the lump .Wellcome again for many more queries to HCM online Clinic-from you and your freinds"
},
{
"id": 62135,
"tgt": "Suggest treatment for a hard painful lump on the groin area",
"src": "Patient: I have a big lump in my groin/upper thigh and it was a hard pea shaped ball, but I couldn t move it and was painful, now it s gotten bigger and more tender in the middle and also turned from skin coloured to really red and has a stinging sensation when touched, I don t know what it is? Doctor: Hi,It seems that there might be having enlarged tender inguinal lymph node giving rise this problem.It indicates that there might be having some infection in near by area.Consult your doctor and get examined.A course of antibiotic medicine for 5 days will heal the lesion.Ok and take care."
},
{
"id": 172413,
"tgt": "Should i be worried about drowsiness in my child after being hit in eye?",
"src": "Patient: are babyjust got hit in his eye with a toy gun . he cant talk his eye went side ways and he just stopped crying andfeel asleep is it something to worry about an see dr there is no blood some swelling around te eye but not the eye as far as i can tell.s Doctor: Thank you for sharing your query on Healthcaremagic, I have carefully worked through your case, although your son doesn't have any local signs like swelling, bruises or pain complaint, he is drowsy due the trauma and any chance is really important, I don't suggest to you go inmediatly to the hospital just pay attention and watch carefully if the drowsiness is increasing or others neurological symptoms start to show up, like blurry vision, nausea, vomiting, motor imbalance or something abnormal then you can take him to the emergency room Please kindly rate if my advise has been helpful Wishing you a good health Dr Tania Portelles-Driggs"
},
{
"id": 22828,
"tgt": "Is it advisable to conceive having diagnosed with congenital cardiac anomaly?",
"src": "Patient: Hello I don't know whether you will answer this e-mail or not but this is really important for me:( I am from Turkey. a 25 year old female and married. I have a bicuspid aorta and consider pregnancy.It is a congenital cardiac anomaly. Of course in Turkey I asked lots of cardiologist whether I should do it or not but they have said me that there are risks but noone can't know what may happen during the pregnancy because there isn't lots of person like me. so I can't decide:( After describing my situation briefly I will try to write something from my last echocardiography test. Peak Gradient(mm Hg):65.00 Mean Gradient(mm Hg):37.00 Now I will try to translate my doctor's comment on my test. I don't know lots of the terms. Heart blanks are normal. Aorta is thick and bicuspid. It's expansion is limited. Aort radix is 2.5 cm. The other valves are normal. Light-middle aorta insufficiency and tricuspid insufficiency. I don't whether I could explain it. What do you advise me? I am looking forward to hearing from you. Doctor: hi, it looks like you are bicuspid aortic valve and other other abnormalities in your body.There is 9% risk of similar disease in first degree relatives. So there is 1 in 10 probability of your child having the same.I think it's worth to this much risk and go ahead with pregnancy but pregnancy being a stressful condition you should discuss with your gynaecologist whether you ll be able to tolerate it or not."
},
{
"id": 21978,
"tgt": "Can withdrawal of blood pressure medication cause breathlessness and profuse sweating?",
"src": "Patient: A few weeks ago, I forgot to take my high blood pressure medicine one morning and had an episode of not being able to catch my breath and sweating profusely. Since then I ve been feeling a since of vertigo. I don t feel dizzy more just light headed. Is this something to be concerned about? Doctor: yes ...it is....skipping a dose of antihypertensives can lead to rebound hypertension means increase in your blood pressure.....which can cause an feeling of dizziness....but if you are continuously getting sweating and giddiness...get your echo,ECG done."
},
{
"id": 85203,
"tgt": "Could the Hemo rage black ultra concentrate cause false positive in the drug test?",
"src": "Patient: Hi my name is Cpl Evan Gilless (USMC) I m currently taking Hemo Rage Black Ultra Concentrate , and I was wondering would this make me a positive in a drug test done by the Marines. They have been saying a lot of preworkouts make them positive for meth and such drugs. A false positive. Doctor: Hello,False positive test is unlikely with hemorrhage. You can safely proceed with the test.Hope I have answered your query. Let me know if I can assist you further. Regards, Dr. Shinas Hussain, General & Family Physician"
},
{
"id": 152688,
"tgt": "What do these blood and CT scan reports for breast cancer indicate?",
"src": "Patient: Hello, My sister has been diagnosed with Breast cancer in the last few weeks. The reports are attached-she has also had normal blood work and a normal CT scan. She has seen an oncologist who is recommending that she completes Chemo before having the breast removed. They refer to this as neoadjuvant therapy. The course of the chemo will last 16 weeks and the surgery will follow about a month after chemo concludes. My questions to you: 1-What are there benefits/concerns with neoadjuvant therapy vs adjuvant therapy (having the surgery now and determining the exact stage of the cancer to establish the appropriate treatment)? 2-If she proceeds with the neoadjuvant therapy, should she have a lymph node biopsy now? If not, when? 3-Her pathology report shows HER-2 Negative. From what I read this is a good thing. How accurate is the process of determining HER2 positive or negative in a pathology report vs the fish test? 4-Can you tell me if an Oncotype DX test is important? From what I read it can tell you the likelihood of reoccurrence, however, since she will have Chemo, is it still important? It s very hard to get any clear answers and some guidance would really help! Doctor: In stage 3 cancer and in some stage 2 cancer it is recommended to give neoadjuvant therapy first followed by surgery.it will not only give the patient the benefit of conserving breast in some cases but also in cases where there is complete pathological complete response of the tumour to neoadjuvant chemo treatment it has been proven that it has led to decreased chances of recurrence and increase overall survival rates.FISH for her2 is generally performed in doubtful cases like her2 2 positive.her2negative is definitely better than her2 positive.oncotype DX is generally recommended for small rumours of size less than 1cm with lymph nodes negative cases though there is slight variation in recommendation in case to case basis.liklihoid of recurrence depends on staged and response to chemotherapy in a patient. lymphnode biopsy is generally recommended before neoadjuvant chemo"
},
{
"id": 102367,
"tgt": "What is the remedy for severe cough with Phlegm and wheezing?",
"src": "Patient: Hi Doctor,My son 8 and half years old is suffering from cough for the last days and it triggers wheezing. He has history of bronchial asthma and he has been ok for quite sometime now. We are giving him Bronchipret syrup 5 ml twice a day and nebuliser to control wheezing. There is no fever. The cough is not stopping. Should we continue the same medicine for 5 days or any change required to bring his cough under control. He has phlegm which is not coming out..pl advise. Doctor: Hello, Welcome to HCM, You can continue the medicine for anoither 5 days, the syrup helps to control the cough and the nebulization helps to control your son wheeze. In adddition to this he may require a course of oral antibiotics in the form of syrup and oral antihistamine. I would have suggested my patients with 1. Syrup Bactoclav 375mg twice daily for 5 days 2.Syrup T Minic thrice daily for 5 days. You can take these medicines and the medicine prescribed your doctor and inform him regarding these medicines. Thank You."
},
{
"id": 121891,
"tgt": "Suggest treatment for stiffness in arm after an elbow dislocation",
"src": "Patient: I had an anterior elbow dislocation five weeks ago. I can touch my shoulders with my injured hand right now but I still can t fully straighten my arm. I mean, I can make it horizontal (0 degrees). But I cant make it straighten like my other hand does. ( -10degrees). Is it normal? Will the stiffness go away? What should I do? Doctor: Hello,The stiffness is normal after elbow dislocation. I suggest physical therapy to treat the stiffness. Physical therapy will perform exercises and strengthening techniques to gain flexion, control and motion. Rest, anti-inflammatory medications, and splinting may also be used to help treat the condition.Hope I have answered your query. Let me know if I can assist you further.Regards,Dr. Dorina GurabardhiGeneral & Family Physician"
},
{
"id": 34578,
"tgt": "What causes fever and shivering while on treatment for enteric fever?",
"src": "Patient: Hi My father (64 yrs age) is being treated for typhoid/enteric fever. The widal test yesterday showed -ve values for the typhi bacteria. Today all of a sudden in the afternoon he had very heavy shivering, he was having mild fever in the morning (99 deg F). What could be the reason for this renewed shivering? he's currenlty being treated for urinal infection. Doctor: Hello dear,Thank you for your contact to health care magic.I read and understand your concern. I am Dr Arun Tank answering your concern.Negative widal test is suggestive of cured typhoid infection.As you are saying that he is currently on the treatment for urine infection, this spike of fever is because of the urinary tract infection.I advice you should go for urine culture and sensitivity. This will guide us regarding the possible pathogen and its treatment protocol. Take the treatment under your doctors guidance.Please take plenty of water, this will flush the organisms from the system. Please maintian good local hygiene as this can be help ful in the treatment.I will be happy to answer your further concern on bit.ly/DrArun.Thank you,Dr Arun TankInfectious diseases specialist,HCM."
},
{
"id": 66865,
"tgt": "Suggest treatment for a wrist lump with pus",
"src": "Patient: my son visited pediatrician today has a puss filled bump on inside of wrist which also has a red line going half way up forearm. she said for him to go home and put hydrocortisone but if it doesnt get better in couple hours to begin antibiotics. it didnt get better so i began antibiotics but now the red line is even higher past inner elbow. i paged the doctor awaiting her call, is this bad?? Doctor: thanks for writing to us and sharing your son's health problem.as per your description, it is nothing but an infective disease and therefore potentially curable and not to worry too much but seek surgical opinion.there are several possibilities in this case like an abscess, in infected bursa, ganglion or a sebaceous cyst; in worse case it could be related to underlying osteomyelitis also.in any case it is necessary to detect the cause and the infective organism by culture sensitivity test.all the best on this!"
},
{
"id": 60074,
"tgt": "Diagnosed as grade 2 fatty liver. High bilirubin level in blood. Suggest some precaution to be taken",
"src": "Patient: Hi, I an 29 years male and weigh 87.5 kgs. I have been diagnosed with a Grade 2 Fatty Liver , while the liver is in the normal shape in the ultrasound . Also, my Bilirubin level (Indirect) is high to 2.1. However, other parameters are in the normal range. My alchohol intake has been once a week on average. Please advise some strict do s and dont s so that fatty liver can be reversed and also how much time does it take to reverse the liver to its normal self. Doctor: If you indirect bilirubin is high I wouldnt worry much. You might be having a benign condition called Gilbert's syndrome which can cause transient increase of indirect bilirubin with normal other liver function test. What is more concerning is the fatty liver. You need to do the following to prevent further worsening of the fatty liver 1. Loose weight 2. Stop alcohol 3. Does diabetes run in your family ? - if yes have your sugars checked. Vitamin E has shown to reduce inflammation in a fatty liver however benefit is not conclusively proven. A good healthy alcohol free lifestyle would reverse the fatty liver in due time..."
},
{
"id": 213525,
"tgt": "Severe bouts of hypochondria post remembering a tragic moment. How can I overcome this?",
"src": "Patient: my father suffered from schitsophrenia and took his own life when i was six years old,from my teens until now i go through bouts of sever hypercondria about this , i desperately want to get on top of this and would like to know the best way to go thanxs . Doctor: First you accept that such thing happened in your life, and you have no control over it. Undoubtedly such incidence in anybodies life is traumatic and may come into mind frequently. Best part is you survived of such adverse situation which already made you enough strong to fight even bigger problem in life."
},
{
"id": 161622,
"tgt": "What causes cough and skin rash in a child?",
"src": "Patient: Hi,since the 21st December inside my home has been a smell of Kerosene the insurance company are relocating us tomorrow,myself and my wife have had flu like symptoms but my 3 year old asthmatic son has been coughing a lot and is very pale with loss of appetite,he also has a rash on his chest..... could this be related Doctor: Hello, It might be. It could even be mold or other fungal infections.Kindly get him evaluated for the same. Hope I have answered your query. Let me know if I can assist you further. Regards, Dr. Sumanth Amperayani, Pediatrician, Pulmonology"
},
{
"id": 87701,
"tgt": "What causes throbbing pain on lower right side of navel?",
"src": "Patient: i have very light throbbing pain on my lower right side of my belly button i have GERD but never had pain in that area before due to it. i have no other symptoms of appendicitis so i don't know if it's that or not. the pain increases when i sit in a chair, and fades a small amount when i lay down. Doctor: HI Greetings from Dr.Divakara.PThanks for posting your query. I need more information regarding your problem . Since how long you are having this problem? Is the pain continuous or intermittent? Does the pain increases after eating food?Your sex ?Any burning sensation or pain while passing urine? Kindly revert back with these details. The possibilities could be Kidney Stones, Appendicitis , Liver infection , Infection of gut , Pelvic Inflammatory diseases ( infection of Ovary , fallopian tubes or uterus ) , Partial intestinal obstruction, Kidney infection. I advise you to get Ultrasound Abdomen pelvis , Urine routine test and revert back . Hope this was useful to you. Any further queries feel free to ask . Regards."
},
{
"id": 195473,
"tgt": "Suggest treatment to have stronger ejaculation",
"src": "Patient: hello doctor,i am 37 years male,during intercourse with my wife,i am unable to ejaculate,but after removing my penis outside i will have little ejaculation,we have been married for 2 years,this is hapening from day one.i am not able to get orgasm at all,but i enjoy and ejaculate in masturbation,we are yet to have a child,please advice Doctor: Hello and Welcome to \u2018Ask A Doctor\u2019 service. I have reviewed your query and here is my advice. This is a common condition among males, sometime the sexual pleasure from masturbation is much more then actual intercourse. Masturbation often leads to low libido because the body gets used to get all the pleasure from masturbation rather then sexual intercourse. I would advise you to stop doing masturbation for at least one week and then do sexual intercourse you will feel better and if still you do not feel better then I would refer you to psychiatrist for sexual counselling. Hope I have answered your query. Let me know if I can assist you further."
},
{
"id": 63089,
"tgt": "Does lump on breast near armpit indicate sign of cancer?",
"src": "Patient: Hi, i am a female, 18 years old. i started to develop a lump on my rigght breast near the armpit, i does not hurt. it hasent changed size and i dont have any weird symtoms nor my breast have changed. i am a little scared and would like to know is this is breast cancer? Doctor: hi.it is best if you consult with a doctor, preferably a general surgeon, with regards to your concern. a physical examination is needed. breast ultrasound and a possible tissue diagnosis of the lump on your breast will be done if indications are found. one cannot tell if it is cancer or not without doing those things.hope this helps.good day!!~dr.kaye"
},
{
"id": 71857,
"tgt": "What causes fever and cough after getting bronchoscopy?",
"src": "Patient: Hi My mother recently went through broncho-scopy to get her lungs checked. Doctors suspected TB. Did a CT Scan too but they were not sure since she has had TB before ..around 2 times atleast. Since bronchoscopy, she is getting high fever all the time and coughing badly.. sometimes feels nausea... what should we do now? Please respond urgently.. Thanks Doctor: HelloUsually cough is a normal lung reaction due to bronchoscopy.Regarding the fever it has nothing to do with however her doctor knows better her conditions.RegardsDr.Jolanda"
},
{
"id": 215128,
"tgt": "Itchy nipples",
"src": "Patient: Hi, I am male and 31. my nipples have been itching me almost every evening for about 2 weeks now. Looking closer at them,I see some blisters around the nipple and am trying not to touch it, but its very itchy. Doctor: Apply Ointment pentaphyte P5 of Dr.Palep also apply coconut oil daily twice"
},
{
"id": 76441,
"tgt": "What causes 'woven bone' on chest x-ray after being diagnosed for broken ribs?",
"src": "Patient: This year 30 Marcht my wife(63), heigt 1,74 meter weight 74 kilo fell backwards from a higth of 8 feet.She took an X ray. Her diagnosis was 2 broken ribs 8th rib front leftside and one rib right backside. After about 5 weeks her pain diminished to zero. Now since a week she experiences continually pain.Not especially by touch or breathing. She consulted her doctor and took an X ray again. Diagnoses on 4 ribs left there was surprisingly so called woven bone. Is there an explanation? Doctor: hi,thank you for providing the brief history of your wife.As she is 63, and had a fall which lead to fracture of the 2 ribs at that time and now she is having woven bone on cest x-ray for 4 ribs, this makes us understand the weakness of the rib bones. As per the anatomy of the rib bone, it is a flat bone and healing for it is little slower. It is very less the ortho technology can really do as the location of the bones. What I feel is the weakness in the bones due to less of calcium has lead to this woven bony appearance in x-ray. Also, due to her age, the healing process can be delayed.What I would suggest is to check the calcium and vitamin D3 levels. Based on it there will be a supplements of calcium and also vitamin D3 injection too boost the physiological mechanism. Performing regular breathing exercises like costal will be helpful to boost the strength in the muscles and the bones.Since the rib bones are flat and cannot be made exercise by weight bearing of something there is only a simpler way to deal with it.In my clinical practice we treat such patients with woven bone with calcium supplement, vitamin D3 injection and breathing exercises. The process is slow but functions well even in ageing individual.Regards Jay Indravadan Patel"
},
{
"id": 34766,
"tgt": "Recommend treatment for sinus infections",
"src": "Patient: im a 23 yrs lady, a third year student in degree of doctor of medicine , I have sneezing problem for almost 9 months now more marked in the morning and late evening or in cold times and sometimes on smelling strong smells as perfumes. The sneezing have been of so intense on acute onset to the extent of having flu like syndrome and irritation in ears.I have been using cetrizine as drug for allergies on times of acute onset which are infrequent.I have been taking the condition as allergy to cold and smells for the whole time now.I would like to know the prolonged consequences of this condition and may be what to be done to treat the condition, an if its not an allergy, what likely is this condition Doctor: hi,as you know allergy is provoked by some allergen..try to find that and avoid it..do you have any headache or tenderness over sinus areas associated with occassional fevers?if that is the case get a clinical evaluation done by ent surgeon,xrays etc..an antibiotic course of clarithromycin and sterid resolves if it is sinusitis..for permanent relief undergo FESS..if it is only allergic best possible treatment is avoid allergen.take montelukast and levocetrizine..that would suffice."
},
{
"id": 133611,
"tgt": "How to treat upper arm pain?",
"src": "Patient: Hello, I was prescribed Levofloxacin Feb. 17th. I lost the use of my upper arms.It was extremely painful. My arms have improved to a point,but my shoulders and upper arm sill hut. I can hear the bones in my shoulders crunch. Its very hard to lift my arms. WHAT AM I SUPPOSE TO DO??? HOW LONG WILL THIS LAST??? I am 66 years old. Thank you Doctor: hi,thank-you for providing the brief history of you.A thorough musculoskeletal and Neuromuscular assessment is advised.As you have pain in shoulder and upper arm along with difficulty in the movement of the shoulder joint a thorough musculoskeletal and Neuromuscular assessment is needed. This symptoms can be due to 2 reasons1. Shoulder complex pathology2. Cervical spine pathology.Undergoing an assessment post which an MRI of shoulder or the cervical spine will be needed to see what area is involved and what is the soft tissue status.Treatment of choice will be physical therapy like therapeutic ultrasound therapy and TENS therapy for pain management . For strength and ROM exercises will be taught by physical therapists.In my clinical practice majority of patients respond well to physical therapy and recovers in 2-3 weeks of time.RegardsJay Indravadan Patel"
},
{
"id": 208196,
"tgt": "Suggest remedy for mental health problem",
"src": "Patient: i took marijuana(bhang) in the month of march thats too raw......at that time i had to go through a hard situation which is very hard to explain....actualy everything repeats all the emotions...sadness happiness anger...guiltyness...and even sometimes felt like killing my self....but after few hours i became alright....and after a month and half after taking fenny(a local wine) and thinking about it the same feeling came in my mind bt at slower rate...and nw the effect is still there but the frequency is less....is it happening because of the marijuana that might not be digested fully....or it is something else??? Doctor: hi dear,what ever you described may occur with any substance use.previously you are having such symptoms due to marijuana and at this time due to alcohol.if possible quit all substance .not to worry much.consult psychiatrist if any query is there.Thank you"
},
{
"id": 10568,
"tgt": "Suggest remedy for hair loss",
"src": "Patient: hi i am 19 years of age bt my head because of my hair looks like i'm of 30 (u may say it's just my feeling bt its not)..it has been 3 years i'm having hair loss....first i thought it was normal bt now i am turning bald..both on the front and on the top.... i am happy with my life so no signs of stress....think i take balanced diet...my father is 43 and even he hasnt baldnes...pls help.. Doctor: hi there.1. your concern is androgenic alopecia.2. its a male pattern baldness.3. there are few molecules like finasteride, deutrastride,minoxidil, capixyl and few procedures like PRP,which will help in controlling the hair fall and slowing down the process of baldness.4. consult a dermatologist near by you for starting the prescription as concentrations depends on grading of the baldness.5. prefer protein rich diet.this will help you."
},
{
"id": 224978,
"tgt": "Is it normal to constantly bleed, discharge & clot, back ache 5 months after fitting copper coil?",
"src": "Patient: I had the copper coil fitted 5 months ago and everything was normal had heavy periods and cramping but were manageable, but the last 3 months I hav constantly bled sometime red sometimes brown heavy and light and also discharge and small clotting, I get constant back ache and just feel weak all the time and still getthe butterfly feeling in my tummy is this normal Doctor: Hi, Thanks for the query.I understand your issue. Bleeding after copper T is common and is usually in the form of heavy bleeding during periods. White discharge is also common. Continuous heavy discharge may occur in some women but it needs to be investigated to rule out other causes. Also backache and heavy discharge indicate an infection. I suggest that you get a gynecologic examination done and also check to see if the copper T is in place. If it is an infection, it will get treated by antibiotics.Hope this satisfies your query. Thanks for using HCM.Feel free to ask any more questions that you may have. Dr Madhuri BagdeConsultant Obstetrician and Gynecologist"
},
{
"id": 197655,
"tgt": "What is the remedy for darkness due to jock itchiness?",
"src": "Patient: Hi dr , I m 30 years old man ,I had a infection 2years ago and I got jock itch ,I went to doctor and he gave me some pills and cream , I took pill and infection gone and I applied cream on my groin area , the fungal is gone from my penis and groin area but my glance is not smoot and still darkness , i don t know how to bleache the darkness on my penis and groin area .? Thanks Doctor: some blackeNing is considered normal. but if it persists you shoulf chrck it. also maintain hygiene and check fir sugar in blood."
},
{
"id": 174610,
"tgt": "What causes cyst in bladder?",
"src": "Patient: hello I am Alexis. I have went to my babies ultrasound at 20 week and it showed that my baby had a small cyst in or on his bladder. I went to his 26 week follow up ultrasound and was told that it is smaller. What is going on? Can a cyst get smaller? Is everything okay? Doctor: Hi, welcome to healthcare magic. Cyst in or on the bladder is most probably a ureterocele and may lead to block the passage of urine from the kidneys to the bladder leading to that side renal dysfunction. You can show the problem to a paediatric surgeon for either fetal intervention if possible or intervention just after birth. Also, to diagnose the lesion, you can go for a fetal MRI.Hope, the information helps you."
},
{
"id": 159152,
"tgt": "Mass present in both breast, in lymphnodes. If not cancer what could it be?",
"src": "Patient: I have a mass in my right breast, I did a sonogram and a mamogram and the finding where abnormal. My Primary Care physician believes it might be cancer after reading the report from the radiologist. He reffered me to a surgeon, I met with him yesterday and he found another smaller mass in my left breast, but told me not to worry, it is small and he found a relatively large mass in the lymphnode under my right under arm. I asked him if it could be cancer and he said he doesn t think so. I am scheduled for a surgical biopsy this Friday. I asked him if it is not cancer what could it be and he gave me no answer. I nipple secretion , my nipple is inverted, the mass does not hurt and now I have another mass in the left breast (small) and my lymphnodes have a mass. If it is not cancer then what could it possibly be? Any ideas? Doctor: Dear Ma'am, Sorry to hear about your problem but the management plan decided by your surgeon is correct. Would it possible for you to tell me the BIRADS score mentioned in your mammography report? It is a good indicator whether the lump is benign or malignant. a biopsy will give a definitive answer and you will have to wait for 5-7 days for the result. Was you nipple inverted since childhood or is it of recent onset. Recent onset nipple retraction is a warning sign. As you have not mentioned your age, it is difficult for me to tell you the various differentials but the main thing is to rule out cancer. If you would like to clarify any other point, please feel free to ask me. Rohan Khandelwal Breast Surgeon"
},
{
"id": 3450,
"tgt": "Suggest treatment for getting pregnant",
"src": "Patient: I am 44 and trying to get pregnant. My cycles have been off for the past six months. My daughter had a triple organ transplant and is fighting for her life, so i have a lot of stress. I don t know if i am starting to go through mentopause or if it is stress related. I have a new partner, we have never had children together but he has had three and i have had five. I have been trying for eight months. Do u suggest clomid? Doctor: Hi. Your situation is a little bit delicate. You are 44 and you have menstrual delay for 6 months. Do you have hot flashes? The greatest possibility is that you may be in the perimenopause, which is the phase around menopause. Anyway, what would give an accurate answer regarding the desire of getting pregnant is a single test which is AMH (antiMullerian Hormone). It could tell us your current situation of the ovary. If you want to take clomid, first you need to have menstruations and you should try Progesteron to take them back."
},
{
"id": 217410,
"tgt": "What causes pain around the kidneys, frequent urination and burping?",
"src": "Patient: I'm a 35 year old mother. I'm experiencing pain around my kindneys (sort of the middle of my back), I fell like I've got to wee all the time (getting up at least 3 times during the night) and have started burping and basically I'm feeling pretty flat. I've seen the doc a couple of times and they've ruled out a uti. Doctor: hello memi think your doctor is right. u are suffering from urinary tract infection.here are the perfect treatments for u-drink lots of water in a day as much as possible approximately 3 liters in a day-take antibiotic such as mixture of ornidazole and ofloxacin for five days before meal (250 mg two times in a day)-take ibuprofen for pain relief and relieve inflammation-take paracetamole if you are suffering from fever due to UTI-wash genital area with vaginal wash which keeps urethra clean and bacteria free-use tea tree oil as topically applied over urethra which kills the bacteriai think this may help a lotthanks"
},
{
"id": 186804,
"tgt": "What causes whitish substance at the corner of the mouth?",
"src": "Patient: Hi,I do more of talking in Church as a Teen's Teacher but i discovered whenever i am talking, there is always a whitish substance at the corner of my mouth. please what can i do to stop this. You can reply me through this email YYYY@YYYY .Thank you. Doctor: thanks for your query, i have gone through your query, the white substance near the corner of the mouth could be the dried products of saliva or foreign body..nothing to worry about this you have to swallow the saliva properly and the white substance occumulation disappears... i hope my answer will help you.."
},
{
"id": 157818,
"tgt": "Had gold seeds in prostate, had 37 treatments, tired, weak legs. How long does it last?",
"src": "Patient: I am wondering if you have come across men who have had radium treatment for prostate cancer suffering from tired legs after treatment-if so why does this happen, how long does it last, what can be done to improve thisMY husband had gold seeds in the prostate with his treatment and had 37 treatments. finished 2 months ago, still tired and has weak tired legs Doctor: HIThank fro asking to HCMIt is nuclear medicine, much has to be known about this medicine, I think that the concern nuclear physician is the best person for answer of this question, some time the signs and symptoms of cancer patient may not always be related to the disease, have nice day."
},
{
"id": 13586,
"tgt": "Suggest treatment for an itchy rash in groin region",
"src": "Patient: I have had this very itchy rash up on the inside crease of my leg for about a 2 weeks. it doesnt really bother me until I touch it then it starts to itch so bad! There are tiny bumps in a patch where it is itching and it kind of flares up when im scratching it. When im not messing with it I dont feel any bumps. I hate to have to go to the doctor. Oh there is no burning when i urinate or redness or itchiness in my vagina just on the crease of my inner thighs. And the itchiness comes and goes only when I scratch. I often wear spanx and have picked up some weight. And I ve had a weak bladder recently making it difficult to get to the bathroom fast enough. I find that this region has been moist recently due to sweat. Doctor: Hi, The rash could be a tinea infection (fungal infection of groin) or intertrigo (Inflammation of skin folds which could be due to a variety of causes). Tinea infection is usually treated with antifungal medications and intertrigo is usually treated with mild steroid medications. Steroids can worsen tinea infection. So, it is advisable to consult your Dermatologist to arrive at a proper diagnosis and for initiating the treatment. Hope I have answered your query. Let me know if I can assist you further."
},
{
"id": 202258,
"tgt": "Suggest treatment for erection problems",
"src": "Patient: age 27 year, height 5'1'' ,weight 42.5 kg no medical history... i am trying to conceive for last 10 months. but did not.. my husband has some time erection problem.. and some time after intercourse he do not have ejaculation.. i want to conceive but how?? Doctor: HelloThanks for writing to HCMTrue erectile dysfunction is unlikely in your husband's age.His problem may be due to stress or psychological causes.He should try to be calm and before penetration spend sufficient time in foreplay.This is very important in intercourse.He should avoid stressful situation.Don't take alcohol or cigarettes if he take these things.Last but not the least,he need your support and confidence.He should take nutritious food and do exercise.If symptoms persists then he need proper evaluation and investigations.Investigation include routine hemogram,RBS,lipid profile,urine RE/ME,semen examination,penile doppler,hormone estimation,ultrasound of scrotum etc.Hope i have answered your query.Take CareDr.Indu Bhushan"
},
{
"id": 55154,
"tgt": "Is there something to worry if globulin is level 4.8?",
"src": "Patient: in my liver funciton test i got globulin level 4.8 ( ref values given are 2.5-3.5) and my A/G Ratio is 0.7 ( ref is 1.0-2.1). the doctor told liver is normal. i want to know that if less a/g ratio is there any liver disease is present in my body. please suggest me.thanksramesh Doctor: Hi thanks for asking question.The albumin produce from liver.So if liver disease is present then albumin low and compensatory globulin elevated so a/g ratio less.So rule out cirrhosis or other liver problem by USG and viral markers.If excess excretion of albumin occur in urine then also albumin decreased like in nephritic syndrome ...Do urine albumin and RFT for that.Sometimes protein loosing enteropathy can lead hypoalbuminemia.So rule out gastrointestinal disorders.Thyrotoxicosis, prolonged chronic infections, prolong fever can lead to hypoalbumin.So yes liver problem can lead low albumin .first rule out it then if liver normal rule out other condition.Take care"
},
{
"id": 150356,
"tgt": "Episodes of dizziness, blurry vision, spaced out, sharp headache, feeling of walking slanted. Reason?",
"src": "Patient: Hi, I am a 25 year old female. I have been experiencing episodes of dizziness, sharp headaches, blurry vision, and sometime i feel spaced out like a robot. I sometime feel like i'm walking slanted. It comes and goes. One minute this is happening the next I have my energy back. This has been happening for two weeks now. What could this be? Doctor: Hi, Thank you for posting your query. The possibilities in your case includes a peripheral vertigo and vertiginous epilepsy. You should consult a neurologist for detailed evaluation, including MRI brain and EEG. As of now, there is no need to worry. Both the conditions mentioned above responds well to medical treatment. Please get back if you require any additional information. Best wishes, Dr Sudhir Kumar MD (Internal Medicine), DM (Neurology) Senior Consultant Neurologist Apollo Hospitals, Hyderabad, My personal URL on this website: http://bit.ly/Dr-Sudhir-kumar My email: drsudhirkumar@yahoo.com"
},
{
"id": 144517,
"tgt": "What causes delay in recovering from concussion?",
"src": "Patient: I am a 50 year woman. i fell and got a concussion with no brain bleed(2 ct scans) still having headaches ,was put on deepakote and adjusting dosage...still losing words and feel not 100 percent me again...momentsof frustrations and bad mood....q1 will i ever recover and q2 can having previous head injuries that were probably concusions but undiagnosed prolong length it takes to recover? Doctor: Hello. Thanks for asking from HCM.I can understand your concern. It is good that there was no blood clot in brain. Now coming to your first question, recovery from concussion always occurs. Sometimes it may take few days to months ( in case of multiple concussion). Regarding you second question, yes it is true that it takes more time to recover in case of past history of multiple episodes of concussion. But do not worry. For relief, you should: take bed rest: take diclofenac sodium for pain relief after proper prescription: take mood stabiliser like escitalopram after proper prescription: Try avoiding any head injury: avoid mental and physical stressThanks. Hope it will help you. Take care."
},
{
"id": 92522,
"tgt": "Abdominal pain, hardness, feeling round object, offensive vaginal odor during sex. Is it fibroid?",
"src": "Patient: Hi doctor, early this morning, in my usual foreplay with my wife, but unusually I placed my hand in her lower abdomen, I slightly held it tight with a little press. I discovered the place was hard, then she made a mild painful groan. I stopped, five minutes later, i slightly squizzed the left and right of the same place, there and then I confirmed round object. I did not say anything to scare her. About 5 months back i noticed on regular bases, virginal odour slightly offensive both during and out of sex. With this Doctor might she have fibroid? If yes what is the solution? She looks very healthy, and hardly complain of any ailment. I have told her my discover.Thanks. Doctor: Hi,Thanks for writing in.The hard swelling felt by you might be the distended bladder itself or and fibroid like mass as you say. It is important that she is examined by a gynecologist in empty bladder and then an ultrasound scan is done if felt necessary. For foul odor also consultation with gynecologist is needed.Hope this helps."
},
{
"id": 224218,
"tgt": "Is it normal to have back and lower abdomen pain after taking plan b pill?",
"src": "Patient: Hi me and my girl friend had sex last night and the condom got broken so we decide that she take a plan b pill so she took it after ten hours so now she has back pain and in the lower part of her stomach is that normal ? She is really worried. For how long will she have this pain for? Doctor: Hi dear, I have gone through your question and understand your concerns. Plan B pill is emergency contraceptive pill which has known side effects of pain lower abdomen, spotting and bleeding.The pain, if mild, is not to be worried about. However, if the pain is severe, then she should consult an OBGYN specialist to get properly examined, investigated including ultrasound and treated accordingly.Hope you found the answer helpful.Wishing you good health.Dr Deepti Verma"
},
{
"id": 124011,
"tgt": "How to get cured from tibia fracture with food nutrients?",
"src": "Patient: my wife is gone for tibia bone fracture on left leg on 25.04.2012 & got surgery on 30.04.2012. now she started walking with walker but due to poor bone density & feeling pain while walking she is very much depressed. so how long she will take to recover. what nutrient or food help us in recovering from the injury Doctor: Hello, As it looks that the mineralisation of the bone is too weak. Having good milk and milk products in a day and taking sunbath will be useful. As both calcium and vitamin D is needed for bones to become stronger post-surgery. Also, I think that the surgeon who operated must have provided some Vitamin D3 supplement along with the calcium supplement to help boost the healing. I will even advice to start doing some simple exercise for the upper limbs, core and sound limb. So once she starts going to walk, it should be comfortable and confident. Also, exercise will help her not put on too much weight post surgery. Hope I have answered your query. Let me know if I can assist you further. Regards, Jay Indravadan Patel, Physical Therapist or Physiotherapist"
},
{
"id": 221800,
"tgt": "What are the symptoms and treatment for ectopic pregnancy?",
"src": "Patient: i have a one year old just found out 3weeks ago and won't find out how far long i am until next week since i didn't have a period this whole time because of breastfeeding. i really feel like i feel the baby moving but i estimate maybe 6 to 8 weeks but i guess i could be more yet not showing at all. how early would be normal to feel baby movements? and is it ok to only feel a bump on mostly my right side and not somucj on left. like i can press down my belly on lower left side but not right. i don't remember last pregnancy lke that. afraid of ectopic pregnancy or something. Doctor: Hi,Baby movements can be perceived by the mother only after 16 weeks in the second pregnancy. And if you are perceiving baby movements it is very very unlikely that it is ectopic, as ectopic pregnancy does not continue into 2nd trimester without major complications. Get an ultrasound done to know how far long you are, it will also confirm the location of the pregnancy. The expected due date will also be calculated by the scan. Hope this helps.Regards."
},
{
"id": 205462,
"tgt": "Suggest remedy for depression",
"src": "Patient: I just got broken up with by my ex girlfriend of 1 and a half years. She left me for another girl. And I have cut off all ties with her. And recently I have been feeling very sad and not being hungry much and feeling very sleepy.? Is it depression. Or is it just because of the situation Doctor: DearWe understand your concernsI went through your details. You should understand the difference between disappointment and depression. Disappointment, frustration, disgust etc are emotional problems which can provide symptoms just like those of depression. The silver lining is, these symptoms will be of short duration compared to long lasting and chronic symptoms of depression.In your case, your girl friend of one and a half year left you for another. You are deeply hurt, disappointed, disgusted and therefore emotional. Anger, aggression, stress, lethargy, lack of appetite, lack of thirst, lack of sleep, being lonely, obsession etc are part of this. You will feel and experience these symptoms for some days if you don;t keep yourself engaged. Then, slowly, these symptoms should vanish.If you require more of my help in this aspect, please use this URL. http://goo.gl/aYW2pR. Make sure that you include every minute detail possible. Hope this answers your query. Further clarifications are welcome.Good luck. Take care."
},
{
"id": 51279,
"tgt": "Difficulty urinating, bladder pain, dark yellow urine. History of diabetes. Kidney problems?",
"src": "Patient: HI My husband woke up this morning and he said that he was having a hard time peeing well when he was able to go he said that it almost feels dry inside and it hurts when the pee is coming from his bladder out the end also the pee is really yellow. What does that mean can guy get a UTI? He has never had kidney problems but disbetes ryns in his family. Thank you Doctor: All the symptoms are suggestive of U.T.I..Pl get routine examination and ultrasound scanning of abdomen done and if it is a simple UTI take antibiotics and urine alkaliser..Increase intake of water By the way what is his age."
},
{
"id": 179582,
"tgt": "What causes bright red blood in a child s stool?",
"src": "Patient: My 3 mo old nephew was born with a heart murmur. He had bright red blood in his stool last week and now has an increase of dirty diapers with a lot of mucus in the stool. He had a fever of 102.3 but is now 98.6 diaper rash, a little irritable. He is exclusively breast milk sometimes from a bottle with a #1 slow nipple still Doctor: HiThanks for writing to health care magic.Bright red blood could be bacterial dysentery or intusussception.Urgent ultrasound is required, consult your pediatrician immediately.Wishing your child good healthRegardsDr Arun"
},
{
"id": 15904,
"tgt": "Rashes developing into bumps with itching. Hair like thing coming out of the rashes. What could it be?",
"src": "Patient: My husband has small bumps on his skin, that started as a rash and progressed to bumps that itch . He told me today that while scratching one, he noticed a very fine white hair, that was coming out of the bump, he pulled it and by the time he finished pulling it, he said it looked like a very small cotton ball, and immediately after he pulled it out, the bump started to itch. my email address is YYYY@YYYY Doctor: nothing to worry about"
},
{
"id": 112272,
"tgt": "Twisted vertebrae and have constant back pain. Can I take medical marijuana for this problem?",
"src": "Patient: Hi I am 16 years old an am going to be 17 in 10 days and live in the state of colorado i have twisted vertebrae in my back and have constant back pain, also i have problems eating and sleeping, I was curious to know if i could get a medical marijuana license for these issues, marijuana seems to be the only thing that helps me, im just not sure if i would be able to get it for these issues. Thank you for your time Doctor: Hi, You are too young to start such drugs use although under strict medical supervision. However, you can discuss with the neurologist to determine the level of your pain and getting the right pain-killer for you. Usually, NSAIDs are prescribed, but in your case, if they do not work, I recommend to take pain killers of opioid drug class. Hope it helped!"
},
{
"id": 37553,
"tgt": "Suggest treatment for dog bite",
"src": "Patient: Hello Doctor, A street dog bite me after that i have taken vaccine of Tetanus ( Abhay Tox) and Rabis( Abhay Rab) with in 2 hrs. The wound area now healed after 1 day. i have applied beta-dine ointment over it. Let me know now any other things i need to me done . Doctor suggested me 5 dose of rabis vaccine. What type of food i need to taken and which i need to avoid. Doctor: Hello,Welcome to HCM,As you were bit by a stray dog which has led to punctured wound, dog is known to transmit rabies,which is 100 % fatal but it is 100% preventable by proper and adequate treatment.As there was a bleeding from the site, according WHO categorization it is categorized into Cat III, Which requires following treatment.1. Wash the wound thoroughly with soap and water to washout the rabies virus if it is present in and around the wound.2.Active immunization with anti rabies vaccine on days 0,3,7,14 and 28.3.Passive immunization with rabies immunoglobulin in and around the wound.4.Inj TT, if you have not received in last five years.5.Oral Tab Augmentin, 625 mg, twice daily for five days.There is no food restriction during the treatment. You can have normal diet what your consuming daily. If you are a alcoholic, don't consume it during the treatment as it may interfere with the antibody protection.Thank you."
},
{
"id": 96702,
"tgt": "How to deal with tingling sensation in arm after electric shock?",
"src": "Patient: About an hour ago I had an incredibly small shock from the mains. There was no burn of any sort and I withdrew my hand quickly. It didn't feel like much more than a small static shock, but since then my arm has tingled a lot. I'm worried about my heart - I don't know how I would spot any arrhythmia as it is naturally ectopic. Should I be worried about this or does that only apply to shocks with burns, longer duration, physical jolts etc? I live in the UK. Other than the tingling arm there are no noticeable physical symptoms. Thank you. Doctor: Welcome to HCM,Hello,as you have sustained minor shock there is nothing to worry about ,the tingling sensation will disappear in a few days ,you can spot arrhythmia if you are felling your heart beat aloud ,sudden skip of beat,to rule out any other problem visit a physician who may advice you an ECG if required For now you can drink lots of water as there may be dehydration and relaxThanksRegardsDr.Krupa"
},
{
"id": 15761,
"tgt": "Reddish stripes on the body before menstruation. Rashes with irritation and pain in the hand. Treatment?",
"src": "Patient: I am facing red strips as cut marks on body before 2-3 days of menstruation period. from two years earlier it was on above backbone then last time it was on my chick and face. I have consulted to a skin doctor and took medicines. But this time I have different type of red rashes with little irritation and pain in a the same hand from shoulder on hand above elbow. Please advise me any home or other treatment.Thanking you,with regards,seema Doctor: Hello,Thanks for the query,Rash on the body in association of menses might be related to hormones.Near menses, levels of progesterone is high in body There is an entity called as autoimmune progesterone dermatitis.This is associated with skin rash near to menses.Let me know if you have any other doubts.Please meet a dermatologistyou can ask a direct question to me on this forum, following the below link.https://urldefense.com/v3/__http://www.healthcaremagic.com/doctors/dr-rahul-kumar/64818Wishing__;!!Mih3wA!SBzm6_kI6hCZ58EPH6N_05MFfiPbxWXT0a2TJCdFQObRWm5mV5ur7hUOMa8clQ$ you good health.Thank you"
},
{
"id": 166849,
"tgt": "What causes infant to have big head compared to the body?",
"src": "Patient: My wife took our four month old for his regular check up. The doctor told her that he was a little concerned about the size of his soft spot. He also said that his head was in the 50% and his body was in the 10%. My son has hit all of his milestones and we had a high risk doctor look at him carefully before birth bc my wife had preclamsia(spelling) and he came alittle early. His behavior has not changed and when my wife was a baby she had an big head. The doc said to wait another month and see if his body catches up with his head. My wife and I are obviously freaking out bc weve had no issues till now. Should we get the CT scan now or wait? I just dont know what to do? Doctor: Hi,As the baby is developmentally normal and as your wife also had big head as a baby, you need not worry. If I were your pediatrician, I would follow your son up for developmental pattern and if this is normal I would not go ahead with a neuroimaging. Even if you and your pediatrician have agreed upon doing a scan, I suggest doing an MRI and not a CT scan brain as CT has the risk of radiation.Hope I have answered your query. Let me know if I can assist you further.Regards,Dr. Sumanth"
},
{
"id": 98626,
"tgt": "What causes excessive mucus in the throat while having asthma?",
"src": "Patient: I am a 83 year old woman. I have asthma and for l0 days I have had so much mucus It rattled and I couldn t get it up. Urgent care gave me antibodic, predsone and now over the counter Mucus Relief . They said when you can t get up the mucus it forms an infection. it has been l0 days and only slightly better. Could this be a form of pneumonia Doctor: Yes it can be pneumonia you chest xray will differentiate which pneumonia it is. You must follow up with your doctor and ask him/her to have your xray done"
},
{
"id": 180973,
"tgt": "What causes headaches and sinusitis after a dental procedure?",
"src": "Patient: I had extreme dental work in march. I've had headaches and sinus issues ever since. In may I began to have shortness of breathe , pain under right shoulder blade and burning down my chest. My doctor gave me a steroid shot last week but my symptoms are still there. Doctor: Hi..Thanks for the query..Well, sinus related issues can occur after dental procedure in case if there is perforation of the sinus floor during the dental procedure..It can even happen in long standing cases of dental Infection..Headache can be secondary to sinus Infection as there are large sinuses present behind eyes as well as forehead area..But the symptoms of shortness of breath, pain under shoulder and burning in chest is not related to these problems, so it is better to consult an Emergency room and get evaluated as it can be related to Cardiac issues..So it is better to get examined soon..Hope this helps..Regards"
},
{
"id": 111920,
"tgt": "Can chicken pox cause pain on sides and back?",
"src": "Patient: I am just starting to get over chicken pox. I am 54 years old and now I have numbness and bloating along with some pain on my sides and back. Is this cause for concern. It is just over 2 weeks since I came down with the chicken pox. I got them after my husband had shingles. Doctor: Any viral disease can cause arthralgia and joint pain. This will take some time to recover may be months. Avoid cold weather, apply heat to that side and back. you can use local applicant."
},
{
"id": 78211,
"tgt": "Suggest treatment for chest pain",
"src": "Patient: this is suhaib actually 2 days back i have taken chicken in dinner at that time maybe one or 2 piece of chicken bone went inside . but i m not sure it was happen or not but i m having pain in my chest can u give me suggesting what medicine i take ? Doctor: Hi. I can understand your concern. It would be advisable for you to get a chest x ray done.Consult a pulmonologist for the same. GERD also can cause similar symptoms.Also eat small frequent meals instead of large meals. Exercise regularly, do not lift heavy weights, keep your head end elevated by 2-3 blocks or pillows,give a gap of 2-3 hours before you sleep after your dinnerDon't worry, you will be alright. Hope I have solved your query. Wish you good health. Thanks."
},
{
"id": 112018,
"tgt": "What is the reason for recurring pain in left side of back?",
"src": "Patient: HI....WELL EVERY DAY I HAVE PAIN IN MY BACK BUT IS MORE IN THE LEFT SIDE........WHEN I WALK TO MUCH O STAY STAND UP FOR A WHILE.......THE FIRST TIME I GO TO EMERGENCY BECAUSE I CAN MOVE THE DR TOUCH MY BACK AND WITH OUT EXAMEN HE TELL ME IS THE ASIATIC NERVOUS Doctor: Hi,I have read your query.Usually back pain is managed with pain-killers till 1 month period. If it persists beyond 1 month, then further investigations are done. Since your husband has a bad pain I will suggest the following: - Back pain can be due to many reasons. You should get x-ray lumbo-sacral spine done in AP and Lateral done send us report or discuss it with your doctor. Only then further management can be advised. - Avoid heavy weight lifting pushing and pulling heavy objects, because it will further increase back pain. - You can take medicine like painkillers + muscle relaxants by discussing it with your doctor. Your husband may need stronger pain-killers which are available on prescription only. He can try ibuprofen or naproxen till he sees his doctor. - Apply hot packs for 30 min in morning and evening . I hope it helps."
},
{
"id": 51262,
"tgt": "Why is BP high? Is it related to kidney infection? Opinion?",
"src": "Patient: I have no idea why my blood pressure is high. I went to the doctors yesterday at it was 158/100. They put me on water pills for 3 weeks and a low sodium diet . The thing is, I think maybe it could be an underlying issue. Six months ago when i went, it was 112/86. My diet hasnt change too much (I eat fairly healthy), but i only exercise once or twice every two weeks. I am not overweight .. 5 125lb. I have been on birth control for about 3 years. I also have experienced a kidney infection (of both kidneys) and a few uti s. I want to make my bp isn t because of some kind of kidney problem, or possibly diabetes (my grandfather has type 1). What are your opinions? Doctor: Hi, Wecome to HCM, You are recently detected Hypertension so you will be required it to moniter carefully,as you pointed out at least regular use of birth control pill for 3 years? may contribute to high BP if not the cause of high BP,another thing is kidney problems so you must check renal functions and a Renal ultrasonographic examination. Use of lot of salt or analgesic may contribute to high BP. Lack of physical activity may be another. Salt restricted diet and regular walking exercise 3=4 km /day should improve it. Family history of Diabetes so do not forget to check Blood sugar level twice a year. Stop taking Oral contraceptive pills with consultation of your treating Physician. Good Luck"
},
{
"id": 78392,
"tgt": "What causes difficulty in breathing?",
"src": "Patient: In the past few months I have been having a hard time breathing and have to gasp for air and this is through my day it becomes worse when I eat. It does not burn Just feels like I can not breath normally like I use to. I have asthma But I don't feel its that because I am not wheezing Doctor: Hi. I can understand your concern. Asthma is most common allergic disease which is not a curable disease but definitely controllable, by controlling the same you can lead a symptom free life. Usually presents with cough, wheezing, difficulty breathing and chest tightness. If not treated appropriately can be life threatening.So better to consult pulmonologist and get done clinical examination of respiratory system and PFT (Pulmonary Function Test). It will also tell you about severity of the disease and treatment is based on severity only. You may need inhaled bronchodilators and inhaled corticosteroid (ICS).GERD (gastroesophageal reflux disease) can cause similar kind of chest pain. GERD is due to laxity of gastroesophageal sphincter. Because of this the acid of the stomach tends to come up in the esophagus and cause central chest pain and nausea. You can take proton pump inhibitors. But along with drugs you need to follow certain below mentioned lifestyle modifications for better symptomatic relief. Avoid stress and tension. Avoid hot and spicy food. Avoid junk food. Avoid large meals, instead take frequent small meals. Quit smoking and alcohol if you have these habits. Go for walk after meals. Keep 2-3 pillows under head in the bed to prevent reflux. Loose weight if you are obese"
},
{
"id": 136871,
"tgt": "Suggest remedy for swollen painful toe",
"src": "Patient: Hello,I woke up yesterday to find my little left toe swollen and it hurts. I have not hit it anywhere and there is no sign of bites. The swelling is to the inside of the toe closer to the other toes.I am a 28 years old female and not diabetic nor a smoker .Thanks Doctor: Hi,Thanks for your query.According to your description , pain over your seems to be inflammatory in nature. It could be an acute attack of gout.I advice you to give you rest to the part affected, take anti-inflammatory drug like motrin 1tablet with food as and when required (upto 4 tablets daily) to reduce pain and inflammation and consult your doctor for thorough examination and rule out any infective cause.I do hope that you have found something helpful and I will be glad to answer any further query.Take care"
},
{
"id": 191220,
"tgt": "What is the likelihood of diabetes reversal in a person who is exercising regularly and maintaining a healthy weight?",
"src": "Patient: Hello-I am under my dr\u2019s care for a recent (February) type 2 diagnosis. I am not overweight and I do not drink alcohol. I changed to a LOW carb diet, no sugar. I take 1000 mgs of metformin per day. I am two weeks in on a everyday lap swimming regimen of 2/3 mile. My blood sugar 3 hours after a meal is 116. Is it possible to soon go off meds? Before starting exercise I got my a1c down to 6.5 from a February number of 10.5. What is the likelihood of diabetes reversal?Thanks for your help. Doctor: Hi, Diabetes mellitus was initially thought to be an irreversible condition, however recent knowledge has shown that weight loss can reverse the changes. Data has shown that losing 10 kg of weight reverses diabetes in 2/3rd of patients if done earlier. So you are right, you should focus on diet and exercise to reduce weight. And if your sugar levels are good, you can always stop drugs. However regular follow-up of blood sugars is necessary. Please do not stop medication without the advice from your doctor. Hope I have answered your query. Let me know if I can assist you further. Take care Regards, Dr Muhammad Zahir Shah, Internal Medicine Specialist"
},
{
"id": 14193,
"tgt": "Suggest treatment for dark red bumps on butt cheek",
"src": "Patient: I had a group of small red bumps on my left butt cheek about an inch or two to the left of the rectal are and I had a small patch in between my left thumb and finger. They did not itch and did not hurt except after a few days rubbing on my jeans. I noticed the one on my thumb area had a few little spots that looked like some fluid might be in it but it did not drain or anything. I did not feel drainage on the butt area. I put antibiotic ointment on them and the one on my hand is almost gone and the one on the butt cheek is almost gone but I was worried because it looks dark red, almost purple. What could it be?tay Doctor: Hi.As per your case history you are having furuncles.My treatment advice is \u2013 1. Don\u2019t squeeze the lesions as it may worsen the condition.2. Apply a mild antibiotic cream like fusiderm cream twice daily on it.3. Take a NSAID like ibuprofen for 5-7 days.4. Other treatment options are oral levofloxacin, cefadroxil and azithromycin given only after consulting a dermatologist.Thanks.Dr.Harshit Bhachech.MBBS, DDVL."
},
{
"id": 150675,
"tgt": "Had headache, bloody nose since childhood. Had spinal tap done. Still having headaches, jaw pain, neckpain. Suggestions?",
"src": "Patient: I have a question I am 23 years old and all my life I have had head aches and bloody noses. have had a spinal tap done when i was 14 they said it came back ok. Still having head aches and recently have had blood work done still they say its all ok. I don t know what to do right now I m having bad pains on left side of my head and back of head my jaw hurts all the way down my neck . Any idea what it could be? Doctor: hi, (A) The NECK PAINS may be associated with some nerve impingement, may be some cervical vertebra inflamation/swelling due to repeated friction , MRI would be helpful to get it checked. (B) NOSE BLEED - nose bleed is mostly occurs due to the nasal mucosa (inner surface of nose) gets dry mostly from the winter, trauma to the nose triggers a nosebleed. Trauma to the outside of the nose, such as a blow to the face, or trauma inside the nose, such as nose picking. Rarely : Some anticoagulant drugs like coumadin may be the cause , some bleeding disorder may be the cause like low platelet count but this rare things are dumped down in your case since the blood test and CSF were normal , So for nosebleed stop picking and when ever it bleed you can put a cotton gauge and can be stopped by pressure. (C) There are different type of HEADACHES as below and I will try to explain it in simple terms 1) MIGRAINE - its a head ache which most has some kind of prodrome (factor/symptoms/feeling) which indicates the person that he will be going to have headache soon, mostly people say some kind of aura like lights/ nausea kind of feeling is mostly precedes the onset of headache 2) CLUSTER HEADACHE:- its a headache which is associated with watering in eyes/ nose , pain in eyes , pain associate with some intense light exposure like headache,Sweaty, pale skin (pallor) on the face, one sided headache may be on right or left side. 3) TENSION HEADACHE:- its a headache with band like headache means the hurting area seems like some has tied a band around the head and is very tight. Mostly from the description it seems like a \"CLUSTER HEADACHE\" Now following are the cause that can lead to this kind of headache and needs to checked. Headache NORMALLY DO NOT have any UNDERLYING DISEASE (90% case have no issues), BUT in 10% cases headaches may occasionally indicate a serious underlying medical condition, a) brain tumor or b) rupture of a weakened blood vessel (aneurysm) Well the best thing is to attain a piece of mind, and for that ADVICE: Consult NEUROLOGIST (brain doctor) Here is mostly they will do 1) do physical exam 2) will probably ask for MRI SCAN OF BRAIN, 3) If nothing serious mostly 90% of case are with out danger they will give pain medications that is special for the type of cause (e.g- sumatraptin for migranes , so it will be different from normal over the counter medication (available with out prescription) like ibuprofen) so necessary to consult the physician PLEASE do not be scared , there are 90% of headache and have no issue with any kind of brain problems but for the sake of safety and piece of mind MRI / CT Will be need. Hope it helps."
},
{
"id": 25896,
"tgt": "Suggest remedy for high blood pressure",
"src": "Patient: Waiting to see if I can get in to a Cardiologist today, vs appt tomorrow. Stayed home from work feeling anxious. Couldn't sleep last night. BP at 345am was 142/107 bpm 97, at 6am was 155/110 bpm 77, 9am was 163/110 bpm 73. Can I wait to hear if I can get in today, or does this warrant going to the ER? Doctor: Hello,I have gone through your query.Thanks for using HCM.If you have symptoms like severe headache,chest pain or shortness of breath along with this BP reading then you should go to ER.But if you have no symptoms then you may wait for appointment tomorrow,My best wishesDr.Rajesh Teli,MD."
},
{
"id": 103558,
"tgt": "Mold found in coconut. Allergic to mold. Can it be eaten after removing mold?",
"src": "Patient: okay I ve heard that some molds are safe to eat and kind of douted this answer will be a yes but i got a coconut from a wholefoods today and it had what looked like a small portion of blueish black, pink and yellowy mold in a small ring alound where a cut area was placed for easy opening i already threw out the milk but was wondering if I was able to salvage the rest of the coconut or shouldnt because i am allergic to mold Doctor: i think better avoid the mouldas we dont know the time since how long it is with coconut as this mould grows itcan effext internal parts of coconutan allergic patient can react to even minimum amount of exposureso dont take chance"
},
{
"id": 155318,
"tgt": "What causes dark black spots on inner labia and anal region?",
"src": "Patient: Hi There was only one biopsy taken fron one spot just inside the vagina. No other spots were taken Can you please explain the above? Doctor: If all the spots are similar looking then it makes sense to biopsy any one of them to reach a diagnosis. You dont need to biopsy all of them. However if they are different and all have a suspicion of cancer based on clinical examination then all need to be biopsied. This only your clinician can tell. The treatment of course will depend on the biopsy result which you havent mentioned."
},
{
"id": 81434,
"tgt": "Suggest treatment for breathing problems",
"src": "Patient: I had a pacemaker put in in July and have had no energy and breathing problem. I have had a chest stay blood work up everything is okay. I did find out today my hormone patches were to much, so I will be using a lower patch .could my hormones be making me tired. Doctor: Thanks for your question on HCM.Yes, hormonal imbalance can cause mood swings,tiredness, low energy etc.So get done hormonal study and consult your doctor.Another possibility in your case is problem with pace maker device.If pace maker function is not proper, heart pumping will be affected.So blood circulation is deranged and can cause low energy and tiredness.So also consult your cardiologist and get checked pace maker function."
},
{
"id": 80899,
"tgt": "What is causing weird sensations around chest causing pain while deep breathing?",
"src": "Patient: For a few weeks now I ve had weird sensations around the chest and down the middle of my chest bone. It isn t a pain but a strange feeling. Sometimes it feels like a burning sensation. Sometimes when I take a deep breath I might get a sharp pain by a pec or in my ribs. I feel OK taking a deep breath other than the burn/warm sensation down the middle of my chest. I ve had the full heart scan and that was clear and also doctor has listened to my heart and chest in the usual dozen places. I ve now had blood taken and waiting on results. I m also waiting for an xray appointment. I have Addison disease, don t drink or smoke. I m a 36 year old male. It has been like this for about 3 weeks now. Doctor: Thanks for your question on HCM.I can understand your situation and problem.Since your cardiac and pulmonary examination is normal, no need to worry much for major problem.In my opinion you are having GERD (gastroesophageal reflux disease) mostly. It is due to laxity of gastroesophageal sphincter.Because of this the acid of the stomach tends to come up in the esophagus and cause the burning central chest pain.It is also common in Addison's disease.So GERD is the likely cause in your case.Try to follow these steps for better symptomatic relief.1. Avoid hot and spicy food.2. Avoid stress and tension.3. Avoid large meals, instead take frequent small meals.4. Start proton pump inhibitor.5. Go for walk after meals.6. Keep 2-3 pillows under head in bed to prevent acid reflux.Don't worry, you will be alright with above mentioned lifestyle modifications."
},
{
"id": 10137,
"tgt": "Can placentrex injection with oil cure hair fall?",
"src": "Patient: hi doctor, Iam 26 year old and suffer from severer hair loss. The hair fall is from the roots of my hair. If this continues i will become bald soon. One of my friend suggested to use placentrex injection to mix with coconut oil. Will this be of any use? Doctor: Hello and Welcome to \u2018Ask A Doctor\u2019 service. I have reviewed your query and here is my advice. Placentrex is a human placental extract and it is widely used for hair growth. It shows good results especially among younger age groups. Consult a dermatologist and he will explain in details and the various options available. Hope I have answered your query. Let me know if I can assist you further. Regards, Dr. Shinas Hussain"
},
{
"id": 79645,
"tgt": "Suggest treatment for anxiety and shortness of breath",
"src": "Patient: HI IM HAVING SHORTNESS OF BREATH FOR A COUPLE OF WEeks now it used to be only when lying down now I also have it when talking. Im dealing with anxiety so I kept on blaming it on that and x checked it out. my insurance is not good for another 10 days till I can see a doctor please advise if this is an emergency or it can wait. in addition my period is now late can shortness of breath be a sign of early pregnancy? kindly awaiting ur response amy Doctor: Thanks for your question on Health Care Magic. I can understand your situation and problem. In my opinion, you are mostly having anxiety related symptoms. Stress and anxiety can also cause menstrual abnormality. But better to rule out Pulmonary causes for your breathlessness. So get done PFT (Pulmonary Function Test). If this is normal then no need to worry for lung disease. Your all symptoms are due to anxiety. Better to consult psychiatrist and get done counseling sessions. Try to identify stressor in your life and start working on its solution. You may need anxiolytic drugs too. So don't worry, you will be alright. Avoid stress and tension,be relax and calm. Hope I have solved your query. Wish you good health. Thanks."
},
{
"id": 104786,
"tgt": "Having asthma, coughing up blood. Chest hurts while taking deep breaths. Solution?",
"src": "Patient: I have asthma and am coughing up some blood . Not when I m coughing hard, just a one time cough. This has happened 3 times in the last 6 months. I had a chest xray last Aug. when the first time I did this. This last week, the small amout of blood that I coughed up was a bright red color. The first two were a dark, rusty color. My chest hurts when I take deep breaths a lot of the time. I am a 74 year old female. Doctor: Hi, Coughing up of blood isnot a good sign. You should take care and had to do certain labs that include workup for Tuberculosis and Bronchopulmonary carcinoma. You should do CT scan of the chest and Sputum culture and Mantoux test. If needed further investigation you should have Biopsy of lung for diagnostic purpose. Repeat your all labs that include Blood CP, PT INR, and workup for Tuberculosis and cancer. I hope I am sucessful in solving your query If u have further you can ask me Get well soon Take care Regards, Dr. Azhar Sattar"
},
{
"id": 223270,
"tgt": "What causes spotting, swollen breast and back pain after taking Unwanted 72?",
"src": "Patient: Doctor please help me I don t know what to do.... We had a sex on 11 October during my periods by using condom and I had taken unwanted 72 but at 18 October some brownish red spotting came out from my vagina and I m feeling pain in my back and I fainted today spotting happen 2-3 days my shoulder is also paining and swollen breast plz doctor help me what should I do is this is the symptoms of pregnancy please doctor help me I m in trouble how to prevent this...?? Doctor: Hello,Thanks for trusting us with your health concern. I have gone through your query and would like to offer some information. Firstly, conception cannot happen due to a sexual intercourse during the menstrual period, as this is the safest phase. Secondly, the emergency contraceptive pill has high hormone content and as such may have caused some withdrawal bleeding. However, the shoulder pain, history of loss of consciousness and backache need further evaluation. Breast engorgement can be an effect of the hormone. Please get further help from your health care provider. Hope this helps."
},
{
"id": 27368,
"tgt": "What is Inducible ischemia?",
"src": "Patient: Hi My father is 59...He recently had body pain, throat infection and complained mild chest pain...we showed to doc and he recommended ECG and then TMT test. TMT test mentioned that Inducible ischemia is positive. what it means? What should be next step? Doctor: Hi, Thanks your question. Inducible ischemia on TMT means - There is insufficient blood supply to heart when there is increased work load on heart. This may be due to narrowing of blood vessels that supply blood to heart (coronary arteries). In such situations during resting state blood flowing through coronary arteries is sufficient to meet metabolic need of heart. But when there is increased work load on heart as during any type of physical or mental exertion, (there is increase in demand of oxygen and blood supply of exercising body to meet this demand heart has to work more) the blood flowing through coronary arteries is not sufficient to maintain metabolic need of heart. In that case there may be development of changes of heart attack in heart. In such situation it is advisable to do CAG (Coronary Angiography) to know- Whether there is blockage in coronary arteries or not. - Degree of blockage if present. - Number of blockage. Depending upon it appropriate treatment can be advised. Hope this helps you, if so do vote as helpful."
},
{
"id": 139089,
"tgt": "What causes stiffness and cracking noise in knees?",
"src": "Patient: I have been working on semi trucks for about four months now and sense i started my knees have begun to pop. More so when im getting out from the truck (whether it be the engine, the back, the cab ect.) Also occationally they are minorly stiff. The poping doesnt hurt though. What might it be? Doctor: If you mean a bit of instability,then you will need a thorough examination and MRI to see the internal structure of your knee as tear in internal ligaments can cause such problems"
},
{
"id": 218321,
"tgt": "When should a double marker test be done after talking Hucog injection?",
"src": "Patient: Hello Sir/Madam, My wife is 11 week pregnant now. Dr. has suggested NT scan and Double Marker test between 11-14 weeks. However she is also taking HuccoG injection weekly and Enaxoprin daily. My question is, after how many days we should go for double marker test after talking HuccoG injection? She has taken last HuccoG on Sunday (19th August 2018). Please advise as early as possible. Best regards, VK Doctor: Hi, Bitest results are not compromised by Hucog injection neither by Ecosprin intake. Bitest together with ultrasound report gives a probability number for down syndrome risk of the foetus. Hope I have answered your query. Let me know if I can assist you further."
},
{
"id": 211033,
"tgt": "Is sudden feeling of hit on chest with heavy thump, sweaty body while sleeping concerning?",
"src": "Patient: occasionally I drift off to sleep nice and relaxed and wake up from a deep sleep feeling like someone has hit me in the chest with a sledge hammer and can feel my body jump from my bed and fall back to the bed with a heavy thump and I wake wide awake with my body dripping wet in sweat from head to toe and realise something serious has just happened Doctor: Hi, Welcome to Health care magic forum. The condition you are facing may be called the narcolepsy,caused by the lacking in hypocretin, a chemical in the brain that activates arousal and regulates sleep. . Narcoleptics generally do not have as many Hcrt cells to produce hypercretin, which accounts for his or her tendency to fall asleep. The investigations to diagnosis are Nocturnalpolysomnogram,Multiple sleep latency test,and Spinal fluid analysis . I advise you to consult a neurologist for diagnosis and treatment. You may need to have a C.T.scan for diagnosis. The treatment comprises are 1.For arousal Modafinil,and sodium oxybade.For antidepresant activity-fluoxetine,and sertralin. Wishing for a quick and complete recovery. Thank you."
},
{
"id": 202716,
"tgt": "On and off nightmares, screams and cries, affects sleep schedule and school performance. Opinion?",
"src": "Patient: My daughter is 13 and for about a year now her nightmares have been coming back off and on. At first it was just a rude awakening for her and she would go right back to sleep but now they have become so real to her (as she says). She screams and cries like it is really happening and it is affecting her sleep schedule and performance in school along with her other activities such as girl scouts and band. I have looked for therapists but Im just not sure yet if I should take her or just help her at home. Doctor: Hello,I understand that you are worried about your daughter's sleep problems. Now, everyone experiences nightmares sometimes, bit they can be quite distressing if they happen very frequently.Frequent and distressing nightmares in children can indicate stress or anxiety-related disorders. In your daughter's case, since her symptoms are resulting in significant dysfunction and affecting her school and social life, I would advisr you to seek professional help from a psychiatrist or child psychologist.Best wishes."
},
{
"id": 157042,
"tgt": "What do you suggest for major depression and PTSD?",
"src": "Patient: I am drug resistant, major depression and PTSD. You would not know it to look at me. I have a great husband, six figure income and I my Rx drugs are pooping out- my prescriber refuses to stop listening to the drug reps, the medicine to 'augment' the cymbalta helped me gain 30 pounds and now I have high blood pressure. That company of Seroquel has paid over a half a billion in settlements, but she thinks since the Rep says it's ok, it must be. Then, I say, you take the stuff and see what happens to your body, dang it. I am sorry, but I can't wait for my children to finish school so I can end my pain myself. I am an intelligent woman, but when cancer patients have no hope, they can go to Oregon, or Switzerland or Denmark. Why can't people have compassion and just let me..go. It would be a mercy to me. Doctor: HelloI can understand what you are going through but to get better and enjoy life is much better than ending your life. There are a large number of options to get better like medicines, psychotherapy ,deep brain stimulation,rTMS etc. So give yourself another chance , may be consultation with some other doctor help.Dr Saatiish Jhunttraa"
},
{
"id": 47008,
"tgt": "What does depressed creatinine level signify?",
"src": "Patient: My Name is Siddharth Mehta, I am 17, and my question is in my routine blood tests, everything was perfect, just my serum creatinine was 0.66mg/dL, and the reference range of the lab was 0.8-1.2, but my doctor said low creatinine is nothing to worry about, is this true or should i consult a Nephrologist. Also I have severe muscle stiffness and my muscles are achy and weak, I consulted an orthopaedic, and my x-rays were fine, so he said there is no bone problem, and exercise would help reduce my muscle stiffness, should I visit a Rheumatologist, will he be able to help me out. And lastly can my muscle ache could be because of poor blood circulation, but my CBC and ESR are completely perfect, so should I also visit a hematologist. Doctor: hithanks for posting in HCMI have understood your concern1.As your doctor already said low creatinine is nothing to worry and no need to consult a nephrologist2. muscle aches may be due to no exercise or electrolytes imbalance.no need for rheumatologist opinion.I can suggest you to get doneserum calcium, phosphate, sodium and potassium. if they are in normal range, you are perfectly alright.I can suggest youdaily exercise atleast for 30 mins as walking or jogging.hope this helps youany further questions please let me knowthanks"
},
{
"id": 40951,
"tgt": "Suggest remedy for infertility problem",
"src": "Patient: Date : 25/03/2011 Sir,We are the married couple of last 6 years and in which last years we trying for a baby. Due to some unknown fertility problem still now she is ( My Wife ) not conceive.Because of that we wants your help. We hereby enclose all the reports for your kind inspection along with the following medicine which we have already taken :For Mrs. Arpita Sarkar Age 28 years : ( Medicine )\u00a0\u00a0\u00a0\u00a0\u00a0\u00a0\u00a0\u00a0\u00a0Medicine\u00a0\u00a0\u00a0\u00a0\u00a0\u00a0\u00a0\u00a0\u00a0\u00a0Date of Medicine taken\u00a0\u00a0\u00a0\u00a0\u00a01. Folinal\u00a0\u00a0\u00a0\u00a0\u00a0\u00a0\u00a0\u00a0\u00a0\u00a0\u00a0\u00a0\u00a0\u00a0\u00a0\u00a0\u00a0\u00a0\u00a0\u00a006.02.2008\u00a0\u00a0\u00a0\u00a0\u00a02. Ovofar (500)\u00a0\u00a0\u00a0\u00a0\u00a0\u00a0\u00a0\u00a0\u00a0\u00a0\u00a0\u00a0\u00a0\u00a0\u00a006.02.2008\u00a0\u00a0\u00a0\u00a0\u00a03. Duphaston (100)\u00a0\u00a0\u00a0\u00a0\u00a0\u00a0\u00a0\u00a0\u00a0\u00a0\u00a0\u00a0\u00a0\u00a0\u00a006.02.2008\u00a0\u00a0\u00a0\u00a0\u00a04. Fertibex\u00a0\u00a0\u00a0\u00a0\u00a0\u00a0\u00a0\u00a0\u00a0\u00a0\u00a0\u00a0\u00a0\u00a0\u00a0\u00a0\u00a0\u00a0\u00a0\u00a031.05.2008\u00a0\u00a0\u00a0\u00a0\u00a05. Letoval (2.5)\u00a0\u00a0\u00a0\u00a0\u00a0\u00a0\u00a0\u00a0\u00a0\u00a0\u00a0\u00a0\u00a0\u00a0\u00a031.05.2008\u00a0\u00a0\u00a0\u00a0\u00a06. Clome (100)\u00a0\u00a0\u00a0\u00a0\u00a0\u00a0\u00a0\u00a0\u00a0\u00a0\u00a0\u00a0\u00a0\u00a0\u00a015.02.2009\u00a0\u00a0\u00a0\u00a0\u00a07. Duphastone\u00a0\u00a0\u00a0\u00a0\u00a0\u00a0\u00a0\u00a0\u00a0\u00a0\u00a0\u00a0\u00a0\u00a0\u00a0\u00a0\u00a0\u00a0\u00a0\u00a015.02.2009\u00a0\u00a0\u00a0\u00a0\u00a08. Azithral (500)\u00a0\u00a0\u00a0\u00a0\u00a0\u00a0\u00a0\u00a0\u00a0\u00a0\u00a0\u00a0\u00a0\u00a0\u00a001.08.2009\u00a0\u00a0\u00a0\u00a0\u00a09. LAA (100)\u00a0\u00a0\u00a0\u00a0\u00a0\u00a0\u00a0\u00a0\u00a0\u00a0\u00a0\u00a0\u00a0\u00a0\u00a0\u00a0\u00a0\u00a0\u00a0\u00a001.08.2009\u00a0\u00a0\u00a0\u00a0\u00a010.Fertily (150)\u00a0\u00a0\u00a0\u00a0\u00a0\u00a0\u00a0\u00a0\u00a0\u00a0\u00a0\u00a0\u00a0\u00a0\u00a001.08.2009\u00a0\u00a0\u00a0\u00a0\u00a011.Susten (200)\u00a0\u00a0\u00a0\u00a0\u00a0\u00a0\u00a0\u00a0\u00a0\u00a0\u00a0\u00a0\u00a0\u00a0\u00a001.08.2009\u00a0\u00a0\u00a0\u00a0\u00a012.Evion ( 400 )\u00a0\u00a0\u00a0\u00a0\u00a0\u00a0\u00a0\u00a0\u00a0\u00a0\u00a0\u00a0\u00a0\u00a0\u00a001.08.2009\u00a0\u00a0\u00a0\u00a0\u00a013.Eltroxine\u00a0\u00a0\u00a0\u00a0\u00a0\u00a0\u00a0\u00a0\u00a0\u00a0\u00a0\u00a0\u00a0\u00a0\u00a0\u00a0\u00a0\u00a0\u00a0\u00a001.08.2009\u00a0\u00a0\u00a0\u00a0\u00a014.Folinin\u00a0\u00a0\u00a0\u00a0\u00a0\u00a0\u00a0\u00a0\u00a0\u00a0\u00a0\u00a0\u00a0\u00a0\u00a0\u00a0\u00a0\u00a0\u00a0\u00a001.08.2009\u00a0\u00a0\u00a0\u00a0\u00a015.Ebexid \u00a0\u00a0\u00a0\u00a0\u00a0\u00a0\u00a0\u00a0\u00a0\u00a0\u00a0\u00a0\u00a0\u00a0\u00a0\u00a0\u00a0\u00a0\u00a0\u00a030.03.2010\u00a0\u00a0\u00a0\u00a016.Ovofar\u00a0\u00a0\u00a0\u00a0\u00a0\u00a0\u00a0\u00a0\u00a0\u00a0\u00a0\u00a0\u00a0\u00a0\u00a0\u00a0\u00a0\u00a0\u00a0\u00a012.04.2010\u00a0\u00a0\u00a0\u00a017.Ovidac 10000 injection\u00a0\u00a0\u00a0\u00a0\u00a0\u00a0\u00a0\u00a0\u00a0\u00a010.06.2010\u00a0\u00a0\u00a0\u00a0\u00a018.Ovofar 100\u00a0\u00a0\u00a0\u00a0\u00a0\u00a0\u00a0\u00a0\u00a0\u00a0\u00a0\u00a0\u00a0\u00a0\u00a0\u00a0\u00a0\u00a0\u00a0\u00a006.08.2010\u00a0\u00a0\u00a0\u00a0\u00a019.Zolam 0.25\u00a0\u00a0\u00a0\u00a0\u00a0\u00a0\u00a0\u00a0\u00a0\u00a0\u00a0\u00a0\u00a0\u00a0\u00a0\u00a0\u00a0\u00a0\u00a0\u00a006.08.2010\u00a0\u00a0\u00a0\u00a0\u00a010.Letoval 2.5\u00a0\u00a0\u00a0\u00a0\u00a0\u00a0\u00a0\u00a0\u00a0\u00a0\u00a0\u00a0\u00a0\u00a0\u00a0\u00a0\u00a0\u00a0\u00a0\u00a020.11.2010\u00a0\u00a0\u00a0\u00a0\u00a011.Stimufol 5 \u00a0\u00a0\u00a0\u00a0\u00a0\u00a0\u00a0\u00a0\u00a0\u00a0\u00a0\u00a0\u00a0\u00a0\u00a0\u00a0\u00a0\u00a0\u00a0\u00a021.02.2011\u00a0\u00a0\u00a0\u00a0\u00a012.Ebexid\u00a0\u00a0\u00a0\u00a0\u00a0\u00a0\u00a0\u00a0\u00a0\u00a0\u00a0\u00a0\u00a0\u00a0\u00a0\u00a0\u00a0\u00a0\u00a0\u00a021.02.2011 For Mr. Sukanta Sarkar Age 35 Medicine\u00a0\u00a0\u00a0\u00a0\u00a0\u00a0\u00a0\u00a0\u00a0\u00a0Date of Medicine taken\u00a0\u00a0\u00a0\u00a0\u00a01. Ariwok 500\u00a0\u00a0\u00a0\u00a0\u00a0\u00a0\u00a0\u00a0\u00a0\u00a0\u00a0\u00a0\u00a0\u00a0\u00a0\u00a0\u00a0\u00a0\u00a0\u00a015.02.2009\u00a0\u00a0\u00a02. Doxt 100\u00a0\u00a0\u00a0\u00a0\u00a0\u00a0\u00a0\u00a0\u00a0\u00a0\u00a0\u00a0\u00a0\u00a0\u00a0\u00a0\u00a0\u00a0\u00a0\u00a015.02.2009\u00a0\u00a0\u00a0\u00a03. Becozymo\u00a0\u00a0\u00a0\u00a0\u00a0\u00a0\u00a0\u00a0\u00a0\u00a0\u00a0\u00a0\u00a0\u00a0\u00a0\u00a0\u00a0\u00a0\u00a0\u00a015.02.2009\u00a0\u00a0\u00a0\u00a0\u00a04. Speman 2\u00a0\u00a0\u00a0\u00a0\u00a0\u00a0\u00a0\u00a0\u00a0\u00a0\u00a0\u00a0\u00a0\u00a0\u00a0\u00a0\u00a0\u00a0\u00a0\u00a001.08.2009\u00a0\u00a0\u00a0\u00a0\u00a05. Genco Plus\u00a0\u00a0\u00a0\u00a0\u00a0\u00a0\u00a0\u00a0\u00a0\u00a0\u00a0\u00a0\u00a0\u00a0\u00a0\u00a0\u00a0\u00a0\u00a0\u00a001.08.2009\u00a0\u00a0\u00a0\u00a0\u00a06. Lyco Q 100\u00a0\u00a0\u00a0\u00a0\u00a0\u00a0\u00a0\u00a0\u00a0\u00a0\u00a0\u00a0\u00a0\u00a0\u00a0\u00a0\u00a0\u00a0\u00a0\u00a005.02.2011\u00a0\u00a0\u00a0\u00a0\u00a07. Carni Q\u00a0\u00a0\u00a0\u00a0\u00a0\u00a0\u00a0\u00a0\u00a0\u00a0\u00a0\u00a0\u00a0\u00a0\u00a0\u00a0\u00a0\u00a0\u00a0\u00a005.02.2011 As you see that we have already taken lots of medicine and want some advance treatment. We are very much grateful to you if you please guide us.Waiting for your reply.Regards Sukanta and Arpita Sarkar 0000 Doctor: Hello,In the couple who are finding difficulty in conception, following investigations need to be done:1. Establish the egg release (Ovulation): It can be done bya. Study of the uterine lining by hysteromicroscopy just before the menses.b. Ultrasonography ovulation monitoring: It will even help to locate the day of ovulation and thus you can plan the intercourse accordingly.c. Ovulation monitoring kit.d. Study of the hormones like FSH, LH, prolactin in the blood.2. Find out the patency of woman's genital tract. For this information:a. Hysterosalpingography: X-ray of the uterus after filling in the radio-opaque dye.b. Laparoscopy: It will give the information about the normalcy of the genital organs and any other relevant information from the pelvis.3. Semen examination after four days abstinence.Once all this information is available, further planning of the management can be done.Currently, most of the medicines Mrs. Arpita is receiving are to induce the ovulation. Their need can be opined only after seeing all the reports. Please visit a gynecologist and get all these investigations done. Hope I have answered your query. Let me know if I can assist you further.Regards, Dr. Nishikant Shrotri"
},
{
"id": 213057,
"tgt": "Suffering from nightmares, irregular heart beats, chest pain after starting Atripla treatment, suspicion of stress. Is it due to HIV treatment?",
"src": "Patient: I stared my Atripla treatment on October 23 2011. After 2 months on I suffer from nightmares , irregular heart beat, and chest pain . After visiting my doctor and had some blood work and heart test. On my heart test they found some liquid around my heart and the doctor tome is not danger. They tink is stress related. Today is June 24 my HIV meeds is working perfect and I m undetectable. The only problem I m having is I continue to have disco ford on my left side lower chest, back and under arm. Is this do to the HIV treatment ? Doctor: Hello and welcome to Healthcare Magic. Thanks for your query. It is possible that your symptoms could be stress / anxiety-related since anxiety can manifest itself with both psychological as well as physical symptoms. However, it is also possible that they could be due to the HIV treatment also and Artipla is known to produce a variety of side effects, including psychiatric symptoms. If you are having recurrent such symptoms or having difficulty managing your stress, then I would advise you consult a psychiatrist for a detailed psychological assessment and further treatment. There are effective treatment options - in the form of medication or counselling / psychotherapy which will help you overcome your problems. Wish you all the best. - Dr. Jonas Sundarakumar Consultant Psychiatrist"
},
{
"id": 102348,
"tgt": "What does 'faint nodular opacity in upper right apex' indicate?",
"src": "Patient: I had a chest scan done at the VA a couple months ago. I was looking over the files online - they were checking for asthma, which they found - and there was a note on the file which I don't understandIt says \"faint nodular opacity in upper right apex - recheck six weeks\". Well they never said a word and they never rechecked. Doctor: If I was working with a report of my patient that reads as such then I'll think as follows: Faint nodular opacity is likely an incidental finding on the scan. This could be a benign nodule or a malignancy (if the radiologist was highly suspicious for a malignancy, then it'd have probably been mentioned to you). Talk with your PCP, if they're concerned, a follow up chest X-ray would help; typically in 3-6months.I'd advise my patient to quit smoking (if they are) Hope this helps."
},
{
"id": 52973,
"tgt": "Explain USG study showing fatty liver changes with cholelithiasis",
"src": "Patient: liver is normal in size and shows hyperechoic echotexture . No focal SOL or IHBD dilatation, but gall bladder is distended with few calculi seen within the lumen. No pericholecytic fluid collection. IMPRESSION: USG study reveals fatty liver changes. CHOLELITHIASIS Doctor: Hi welcome to the health care magic You are having fatty liver with gall stone For these conditions ursodeoxycholic acid 300 mg dose tablet can be prescribed (it will help in cholesterol dissolution in bile and reduce cholesterol absorption) Take low fat diet.. Use less oil in cooking Regular exercise done like jogging, swimming etc. Loose weight if obese.. Green leafy vegetables, fruits consumed more Refined food avoided Alcohol intake limited Some doctor also prescribes vitamin E for fatty liver as having antioxidants property Hope your concern solved Take care Regularly check liver profile... If having further query feel free to ask"
},
{
"id": 219937,
"tgt": "What are the early signs and symptoms of pregnancy?",
"src": "Patient: I am confused as to differences between PMS & pregnancy symptoms. I normally have my period regularly and light for 2 days and hardly any symptoms. My period is 2 days late, extremely sore breasts, headaches and low stomach cramp. Am I experiencing PMS symptoms? Thank you Doctor: Hi there,,PMS symptoms are caused due you excess sensitivity to progesterone hormone which causes sore breasts, heaviness, bloating, headaches and depression. Even in pregnancy the progesterone hormone is on the rise and can cause similar symptoms too.The best way to tell if you are experiencing these due to pregnancy or PMS is by testing for Beta HCG which is the specific pregnancy hormone. Doing a home pregnancy test will detect it. And we can known the cause of your symptoms.The treatment of PMS, in case that this delayed period is due to PMS is to have a healthy diet and exercise and if the symptoms are too bothersome , medications such as SSRI( Fluoxetine, Paroxetine) in the second half of the cycle can be considered.If the urine pregnancy test comes positive contact a doctor for pregnancy care.Hope this helps.Regards."
},
{
"id": 199016,
"tgt": "What could bright red color in the rim of head of penis indicate?",
"src": "Patient: I am a 74 year-old very healthy male (eat healthily, work out regularly). I just noticed that the rim of the head of my penis is red..almost bright red. If it had always been like that I think I would have noticed. I m cicrumsized. I m single and masturbate three or four times a week. When I have sex it s always with a condom; I haven t had unprotected sex in years, in my last steady relationship. The rim isn t sore but it sure is red. Is this something to be conerned about? Doctor: Hello,Welcome to healthcaremagic.comRedness around the rim of penis, is definitely a matter of concern. Redness is the sign of localised inflammation, which may arise as a result of infection, trauma. Check for any other sign of inflammation such as localised pain, any discharge from penis, burning sensation in and around penis. Advice for you is to consult a venereologist as soon as possible.Thank you."
},
{
"id": 154760,
"tgt": "Does sinus cold with congestion affect the Ca-125 levels?",
"src": "Patient: My wife was diagnosed 3 years ago with Stage 3 Ovarian Cancer. Things have been going well. Her recent ca-125 done yesterday was 58, up from 38 in late Feb. She has been fighting a major sinus cold, with congestion amd mucus, could this affect the Ca-125 level Doctor: Hi,Thanks for writing in.Cancer antigen 125 is a substance that is produced in the ovary, fallopian tubes, uterus, cervix, and the lining the lining of the chest and abdominal cavities (the pleura and the peritoneum).The CA 125 level measures a sugar protein that may be released when cells are inflamed or damaged. CA 125 levels under 35 kU/ml are considered normal.Since the CA 125 rise can occur in inflammation therefore a common cold and sinusitis can certainly cause a slight rise in the level. I suggest that she takes treatment for the sinusitis and repeats the CA 125 level after a month when she is free of symptoms. This will help guide her during the next follow up visit with the oncologist. Please do not worry."
},
{
"id": 77751,
"tgt": "What can i do for chest infection?",
"src": "Patient: i've got a chest infection for the second time in 2 months.i have blood cloths in my nose and feel pretty weak with a bad cough.I've not gone to the doctor and stayed away from antibiotics but feel i need some medication as my sympthons have been with me for 5 days. Doctor: Hi. I can understand your concern. It would be advisable for you to get a chest x ray done. You could be having pneumonia. So better consult a doctor at the earliest.Don't worry, you will be alright. Hope I have solved your query. Wish you good health. Thanks."
},
{
"id": 88522,
"tgt": "What causes abdominal pain with nausea?",
"src": "Patient: Hi, I get upper abdominal pain occasionally (maybe every couple of months?) and it lasts for 2-4 days at a time. It is right at the bottom of my ribcage, pretty much in the center. It only bothers me when I m standing or walking, to the point that I start feeling miserable or even nauseous. However, as soon as I sit down it pretty much goes away instantly. I also seem to feel like I need to belch more when this happens. I have tried everything, antacids, gas-x, pepto, pain relievers, nothing helps, it just has to go away on it s own after a couple days. I have gone to my local clinic after symptoms stopped and was seen by the RN, but nothing was done since the pain had stopped. What could this be?? Doctor: HIWell come to HCMChances of hyperacidity would be more, may associated with some infection, on tentative basis this can be managed with the following drugs, if symptoms does not improved then this need to brought to investigations, that may includes plain x-ray abdomen, CT Abdomen, till then this can be treated with following medicines 1) Tab Omeprazole 40 mg once on day at evening time for six weeks 2) Tab Domperidone 30 mg once in day for six weeks 3) Tab Clinidium with ChlordiazepoxideHope this information helps, take care and have a nice day."
},
{
"id": 173096,
"tgt": "What are the side effects of taking expired Fluzet syrup?",
"src": "Patient: My 3 year old son,was given 5 ml,of fluzet syrup which was expired frm last 6 months..,soon an,anti infection dose was given to him but he got ulcers in,his mouth... M a bit worried... Wanna ask wat r d side effects of taking expired fluzet syrup..,its manufactured by cipla. Doctor: Hi dear,It is not experied for so long time that cause trush in the mouth. It can easily treated with soda water and Pantosal ointment. I suggest also to apply Vitamin A for healing .Hope it is answer to your question. If your have more queries,then don't hesitate to write to us .I will happy to help you.Wish your baby speedy recovery recovery and good health."
},
{
"id": 201567,
"tgt": "What are the side effects of excessive masturbation?",
"src": "Patient: Hello doctor, I Donot masturbate like that much on the daily basis. But I have been masturbating from long time ago I can barely remember. I am 30 years old am not married yet and I feel my penis is so week myWalsall seminal is not like that a lot as it was before.. Doctor: Hello dear,First of all, remove all myth and misconception from your mind regarding masturbation.There is no any harmful effects of masturbation as such.However, if it is done regularly then it will result in thinning of semen with low sperm count.Since you are feeling that you are addicted to masturbation and it is interfering with your normal functioning, you can try out the following measures:1. Improve your social life by spending more time with friends and relatives.2. Keep your mind active and busy at all times either through curricular or extra-curricular activities.3. Have a diet rich in fruits, vegetables, fish, nuts & honey.4. Avoid fast foods & lipid containing diet.5. Avoid smoking & alcohol.6. Stay away from pornographic materials.7. Keep away stress, think positive.8. Exercise regularly & practice meditation.Wishing you a healthy lifeTake care."
},
{
"id": 123052,
"tgt": "What causes numbness in legs?",
"src": "Patient: my legs have been feeling dead for most of the afternoon and all evening. it feels like a circulation problem but i m not sure if it is or not. i walk a lot for my work and it doesn t happen very often, say every 6 months or so, but today it has lasted since lunch time Doctor: Hello, As numbness in the legs can be due to the nerve entrapment. It can occur anywhere in its course or the root of it. After a clinical examination if needed then an MRI will be advised to rule out any possible soft tissue injury in the spine. Because numbness in the legs is usually considered to be an altered sensation as well. Doing simple exercises like - core stability, spinal muscle strengthening exercises, static hamstring, static Quadriceps and straight leg raise will help regain the normalcy. Hope I have answered your query. Let me know if I can assist you further. Regards, Jay Indravadan Patel, Physical Therapist or Physiotherapist"
},
{
"id": 32310,
"tgt": "Suggest treatment for dry cough and chest congestion",
"src": "Patient: I have had a dry cough for a few days and when I started coughing earlier today, my heart started beating really fast and I felt like I was going to faint and I had to sit down. I also started seeing blury for a few seconds. What may cause this? I also feel congested in my chest. Doctor: Hello,Welcome to HCM.I have read your query and understand the concern.This could be acute bronchitis with palpitations and tachycardia as you are having persistent dry cough without expectoration.The fainting spell could be because of low blood pressure or vasovagal which should be evaluated further by various tests.However,It can be diagnosed clinically but you must undergo routine blood investigation with EKG,chest x- ray to rule out any other pathology of lungs and to rule out any cardiac issue.You might require long course of antibiotics (amoxyllin with chavulanic acid or cephalosporins) with other supportive treatment like antihistamines (montelukast and levocetrizine) or nebulizers with steam inhalation frequently.Consult physician or pulmonologist immediately and take advise further.Regards,"
},
{
"id": 168072,
"tgt": "What causes bright red inflamed gums in an infant?",
"src": "Patient: My 10 month old baby has bright red inflamed gums. His top four teeth have broken through but are not completely out yet. It has been several weeks. He has had a fever of 101.3 the last few days with no other symptoms. Could it be in an infection in his gums? Doctor: Hello,I can understand your concern. When teeth erupt in an infant's mouth, the gums appear sore and inflamed. Till the tooth completely erupts into the oral cavity, seeing the red inflamed gums is normal and should not be worried about. However, if it takes more than a week now on to erupt completely into the oral cavity, a dentist should be consulted.In addition, eruption process is not associated with infection. Thus, if the child is having fever, he might be suffering from other infection like viral infection. You should consult a pediatrician for fever and infection as it does not seem related to inflamed gums.I hope this information helps you. Thank you for choosing HealthcareMagic. I wish your child feels better soon.Best,Dr. Viraj Shah"
},
{
"id": 129034,
"tgt": "What to do for the pain in my hamstring and lower leg?",
"src": "Patient: I have been diagnosed with a bulging disc. I have had this for 3 months and just noticed blood in my stools. Should I go back to doctor? The pain is still occurring daily, but has improved. I pain shoots down into hamstring and lower leg. It is affecting my daily life. Thank you. Doctor: Hello,Thank you for using Healthcaremagic.I read your question and understood your concern.The blood in the stool is not related directly with the bulging disc but maybe with medications that you are taking. You need to go to the doctor for this bleeding.After you clear the bleeding problem then you can take care for the bulging disc problem as it is not so emergent as the bleeding in stool.I wish you quick recovery.Dr. Selmani"
},
{
"id": 128655,
"tgt": "What causes leg pain with lightheadedness?",
"src": "Patient: my mom is 83 and her back of her legs hurt and has been feeling lightheaded for about 6 days now. she went to her doctor Wednesday and he said it was because she was knock kneed! she does take blood pressure medication and is still feeling the same. Doctor: Blood pressure medication might have side effects in form of light headedness also known as postural hypotension when ever she will walk. Back of leg hurts mainly degenerative changes in knees due to which increase stress on calf muscles. for light headedness her dose need to be adjusted or meds may need to be changed"
},
{
"id": 188836,
"tgt": "Tooth ache due to the improper alignment of wisdom tooth. Surgery necessary? Can it be filled?",
"src": "Patient: Hi Doctor, I have a mild tooth ache from last 4-5 months, i consulted a doctor and she said i need to operate my wisdom tooth as it is not in the proper position and needs to be taken out. The wisdom tooth on both the sides are in same inclined position but i have pain on only right side. This is creating a gap in between chew and wisdom teeth and causing cavity. Is it necessary to operte it or can it be done with filling and medicines? Doctor: Hithanks for the query ,As your wisdom teeth is not properly aligned and is causing pain so extraction is advisable.Wisdom teeth are supposed to be the extra teeth which serve no role .So better consult your dentist for extraction of it because taking pain killers and antibiotics is not the definitive treatment.maintain oral hygiene.take care"
},
{
"id": 50250,
"tgt": "Only 1 kidney working, will need dialysis, on Furosemide, Risperidone, has learning difficulties. Any advise ?",
"src": "Patient: My sister in law has only 1 kidney working at about 30-40% and she is expected to need dialysis in the future. She also has an unmedicated but monitored goitre . Among her medication are the drugs Furosemide for water retention and Risperidone for Psychosis . She is due to have a CT Scan soon and has been given both Gastrografin and Picolax to take ready for the procedure. She has learning difficulties and family have to take care of her medical requirements/appointments. No advice has been given as to whether she should have stopped the Furosemide and her renal condition doesn t seem to have been taken into consideration. We have researched the Gastrografin and Picolax and there seems to be a very high risk that these may cause additional renal failure and the possibility of dialysis being needed after the procedure. What is your advise as to the safety of this procedure and the medication required to perform it given her medical history. Doctor: Ideally IV contrast is not to be given in patients ok CKD if alternatives are available.However if in your sister in law case if contrast is already administered then only solution is to be on a look out for rapid worsening of kidney function & administer dialysis if required.My advice would be to avoid it altogether or do it with a non-ionic contrast when it can't be avoided as in case of Coronary Angiography."
},
{
"id": 27983,
"tgt": "Could anaemia cause high blood pressure in a 19 year old?",
"src": "Patient: I am a 19 year old female who went in for a check-up a few months ago and was told my blood pressure was high. I am a college athlete, have no history of blood pressure problems, and I think I eat fairly healthy. Since then I have had my blood pressure checked 6 times and it was high 5 of those times. I gave blood a few times last year and my blood pressure was ok each time. However the last time I went to give blood, they told me my iron was too low. I have since been taking a multivitamin with iron, but I am not consistent with those. My doctor has ordered blood tests and an echocardiogram and ultrasound. Could I be anemic and could that be causing the high blood pressure? I am sometimes tired, but I have a lot on my plate. Doctor: well if you have high blood pressure recorded on many occasions,you have to be evaluated for secondary hypertension,meaning there by at your age there should be some detectable cause of high blood pressure.anemia directly does not lead to high blood pressure.but it may be another manifestation of disease which has lead to hypertension e.g.chronic kidney disease.you have been rightly advised ultrasound of kidneys and other investigations ..."
},
{
"id": 14610,
"tgt": "How to treat armpit rashes?",
"src": "Patient: hi doc, I have a pinkish rash on my armpit,looks likeskin been peeled off and its been there like 3 months now ,about a shape of quarter the palm size,can you suggest me what could it be?Im afraid is it anything to do with HIv as i had some encouters like 8 months back,pls advice Doctor: HIWell come to HCMHypersensitive reaction may be likely and this can be best treated with Mometasone cream, for the oral medication this can be treated with Tab cetirizine three times in day, take care for hygiene, hope this information helps."
},
{
"id": 223877,
"tgt": "Do I need extra protection after switching birth control pills?",
"src": "Patient: I have been on Levora 28 tablets for 20 yrs. I recently switched to Chateal because of my insurance situation. Both are Levonorestrel & Ethinyl Estradiol Tablets. Do I need extra protection during the first week or month of switching? Do I need to worry about pregnancy during this time? Doctor: Hello dearUnderstand your concernYes you can change the birth control pill form one bran to another brand safely.You are on the the levora pill since last 20 year and it contain 0.15 mg levonogestrol and 0.03 mg ethinyl estradiol. Same composition in the cheateal brand. new brand has the same composition as the previous one and you have nor require any protection for next 7 days.And take it regularly same as previous one.Hope this may help youContact HCM for any health issue.Best regardsDr. Sagar"
},
{
"id": 205920,
"tgt": "What causes difficulty breathing in a person with anxiety?",
"src": "Patient: Hi. I have a history of anxiety and the way it got diagnosed is I go through periods where I constantly feel like I'm having trouble breathing. It just feels like I'm breathing through a straw that has a hole in it and I'm not getting all the air. This caused a panic attack and I got put on medication. Well, about a year ago, I came off the medication because I thought I was fine. Just now this breathing thing started again like last week. Today has been the worst. I don't feel anxious. Could this really just be anxiety? Doctor: Hello.You have been diagnosed with panic disorder in past. Now again you are having your breathing problems which were initially considered to be a part of anxiety disorder.I think you are having a relapse of panic disorder. For treatment you should practice relaxation exercises like deep breathing and meditation. Biofeedback therapy under a psychologist's guidence can also prove useful.If you still have doubts, you can go for pulmonary function tests to test lung function and check for any breathing disorder.Hope this helps you.Thanks."
},
{
"id": 97933,
"tgt": "I am 17 years old. How to increase height ?",
"src": "Patient: hello sir, my name is suman magar. Im a college student. Im 5.2 inch, and im 17 years old boy.and im from nepal. I want to increase my height to 6 feet. Is this possible that i can be 6 feet tall? And also i dont do any physical excercise, is there any medicine for help me to increase my height? Please help me sir. Doctor: Hello dear, Height of an individual is determined by Genetics, Environmental & Nutritional factors and Hormonal factors (most important the Growth Hormone secreted by pituitary gland in brain) An individual stops growing in height after the fusion of the Epiphyses (growth plates) of the long bones, which occurs by the age of 17-21 yrs in males. So...you may still have a chance of increasing your height. Make sure that you are taking a healthy & balanced diet, exercise regularly & avoid stress. Protein rich diet like soya, groundnut, pulses, etc should be incorporated in your diet plan. One more thing...avoid consuming any over the counter medication which claim to boost your growth hormone & increase height...it may be harmful. Wishing you Good Health. Thanks and take care."
},
{
"id": 65154,
"tgt": "What is the lump like rash on my bikini area?",
"src": "Patient: Hey, I have this ball about a quarter inch in diameter about 3-4 mm under the skin very near to the crease in the bikini area. It is firm but seemingly a little soft as well. I can mover it around some, but it seems to be fixed in one area. It doesn't hurt at all either. I am not sure if it has been there all along and I am freaking out about nothing or not. But, also for the last few days I have had a rash in the area, I did some research and asked some questions and found it was only an irritation.So, what is the lump? Am I freaking out over nothing, or is something serious going on here.PS: I am 17, female, have a bf, and I am a virgin. Doctor: hi,From history it seems that there might be having enlarged inguinal lymph node producing this lump.This can be due to having some infection or irritation as you got.Apply triple action cream on the rash.If there is some infection, you may go for one antibiotic medicine course for 3 days.Keep local hygiene clean, dry and airy.Ok and take care."
},
{
"id": 170921,
"tgt": "What could cause choking of throat?",
"src": "Patient: my 6 month old sounds like she has something stuck in her throat. it started when i gave her a vitamin D tablet today. she started coughing and since then sounds like there is something stuck. should i be concerned? she is breathing fine but coughs occasionaly. it does not soud like a cold. thanks. Doctor: Hi, welcome to HCM. A 6 month child should not be given vitamin d tablet, instead you can give syrup. If tablet has been stuck there would be constant coughing and difficulty in breathing. I don't think that tablet has stuck. Take care."
},
{
"id": 49044,
"tgt": "What causes back pain and brown colored urine?",
"src": "Patient: Hi Doctor, I have a chronic pain left back and pass urine brown & the same happening since last 10 days, did USG which shows Kidney size normal with Mild pelvicalyceal fullness in left kidney. Please suggest what should I do, I am worried about my health Doctor: I suggest you to consult a urologist n for back pain u can apply diclofenac ointment. The pain Is radiating pain of renal triangle area."
},
{
"id": 216561,
"tgt": "Suggest treatment for pain arms after an injury",
"src": "Patient: Hello any doc here? Pls I have this problem.. I cant pull up my arm sideways (neck level).. as in if I fold my arm like a chicken wing and try bring it up (shoulder forming a 45\u00b0 ) IT HURTS me so bad.. and I keep hearing a squeaking sound when taking it back down.. pls reply Doctor: hi,thank-you for providing the brief history of you As the mechanism of injury indicates a shoulder pathology and needs a thorough clinical examination and an x-ray or MRI. If it is a bony issue which found during examination than x-ray should be fine but in case if we find something like capsular issue of the shoulder joint then an MRI might be needed.based on the clinical findings and correlation with the MRI or x-ray will help an accurate diagnosis. treatment will be simple pain killers and more of physical therapy as this looks more of a muscular weakness and needs a thorough strengthening protocol.In my clinical practice such cases undergo a thorough clinical examination and an MRI post which physical therapy strengthening protocol helps them alot to recovery.Regards Jay Indravadan Patel"
},
{
"id": 104994,
"tgt": "Allergy after using Dependal. Not suppressed by Avil or Citizen. Serious Problem?",
"src": "Patient: HI Doctor , I have always been getting allergy (Body Rashes ) after using Dependal Tablet for Loose motion, i always kill allergy with Avil Tablet. Now at 37 years age i got allergy with dependal which did not supress by Avil tablet for more than 4 days , then again came back in every 3 days . Later i used tablets of Citizen 1 Tab every 4 days for around 20 days to kill allergy, but after 30 Days it has come back. AM I HAVING A SERIOUS PROBLEM?? Doctor: Hi friend, Welcome to Health Care Magic The first principle in treating allergy is to stop he medicine causing it. NEVER TAKE DEPENDAL AGAIN. It may even end up seriously. Be careful. Taking a drug to cause allergy and then taking another drug to treat allergy is bad approach - it is simply inviting more trouble See a Gastro-enterologist and get investigated for loose stools.. You may need colonoscopy for proper diagnosis. It may be Irritable Bowel Syndrome. Never take self medication. Take care Wishing speedy recovery God bless Good luck"
},
{
"id": 217253,
"tgt": "Suggest remedy for stomach pain due to enlarged appendix",
"src": "Patient: My 5 yr old son keeps complaining of his stomach hurting right around his belly button. When his stomach pain gets really severe he doubles over and complains of it also hurting on his right lower quadrant. He never goes without pain. He was recently in the hospital for the same thing and had a ct the ct scan showed an enlarged appendix measuring 7 mm. He is nauseated all the time has vomited and runs fever periodically. The ct scan that was done a week before the last did not show an enlarged appendix. I am at my wits end trying to help my son should I take him to the er at a childrens hospital? Doctor: Hi. Thanks for choosing HCM. I can understand your concern. Your son is getting frequent attacks of appendicitis. Usually acute attack gets relieved with antibiotics. But if he gets it frequently then he should undergo surgery. If untreated it may land up in appendicular perforation which can lead to multiple complications. So plan accordingly and get an elective appendectomy done instead of running in emergency surgery. Now a days this surgery is done by laproscopy approach . So its quiet a safe procedure. Pl. consult a laptoscopy surgeon and take a definitive decision. If you have any further doubts pl. feel free to ask. Take care"
},
{
"id": 174512,
"tgt": "What is the cause of bit of blood in the urinary tract?",
"src": "Patient: My son is 3.5 years old and healthy, he has just started Epilum as they suspect epilepsay. But today when changing his nappy today i noticed that it had a bit of blood in it and it had a rather revolting smell, mixture of of rotten fish and anomia. i have also noticed that his breath smells just as bad, despite washing his teeth daily. Any ideas Doctor: Hi having blood in the urine is abnormal and he also had a foul smell needs further evaluation to rule out the cause-infection/stones. Please also get a ultrasound of abdomen to see for any kidney or bladder abnormality.consult a child specialist who will properly guide you and treat based on the cause.Hope it answers your query and ready to help you further if you have any queries as I am a specialist in children kidneys"
},
{
"id": 177786,
"tgt": "What causes uncontrollable urination during sleep to a 2 year old?",
"src": "Patient: My two year old doberman pinscher (Cocoa) pees uncontrollably when she sleeps. She pees on her bed, the floor, outside. But she doesn t squat in the house. I m pretty sure she s unaware this is going on because shes also peeing on herself. This has been happening since she was a puppy and we don t know what is wrong. Doctor: Thanks for asking on HealthCareMagic. But it would really be appreciable if you take her to a veternary doctor and talk about this. He would be able to look into the possible cause and suggest proper intervention. Since we are not into that speciality, it is hard for us to comment."
},
{
"id": 104608,
"tgt": "Had a Asthma attack, taking Montelukas and levozetrizine . Any medication to avoid asthma?",
"src": "Patient: I have some type of asthma attacks. My work is at night shift. If asthma starts, it will cured with the use of some antibiotics and inhalers. Is there is any medication for the disease not come. Is montelukast 10 mg is useful. Give me an answer. Now I am using Monelukast 10 mg + levozetrizine 5 mg (over the counter) and taking every evening, is that helpful. Doctor: Hi and thanks for the query, Asthma is a disease classified as a chronic obstructive bronchopneumopathy. This means that it stays for a long time, and also has an inflammatory and allergic aspect. Family history and specific diseases like acute bronchiolitis in childhood could predispose one to getting ashtma. Once it is there, there is no magic one drug or anti biotic to treat it. However, there is a particular ascernal of drugs and lifestyle changes that could avoid the frequency of these attacks. Asthma triggers differ from person to person and need to be avoid. In your case, I suggest you should actually keep very warm, and it would be great if your work schedule could be modified if possible. Your nasal spray to use during attacks should always be with you where ever you go. Consult a pulmonologist for a proper treatment plan. Success in treatment relies on avoiding asthma triggers and complying to the treatment plan. Thanks and best regards, Luchuo, MD."
},
{
"id": 140317,
"tgt": "Suggest treatment for seizure disorder where the patient talks to oneself, hears and sees things",
"src": "Patient: I am having seizures but they are not epileptic what kind would they be? My Dr is having an Epileptic EEG test ordered for me but I don t want the epileptic test ordered. I want the test ordered for the other kind of seizures . I also have MS. Do you think that an EEG hour and a half is long enough to determine what is wrong or long enough to make a dx. I am having seizures where I stare for 4 to 5 min. I talk to myself I hear things and see things. I talk to my self and then I pass out for 5-6 min. I shake pretty severely sometimes. What is the kind of seizure test I need to have? My tongue is not coming out of my mouth, it is not epileptic. Right? Please help me I am very scared. I am 38 and I live with my mother. I need to be able to live on my own. I used to be a branch manager of a Library District and the program manager. I have a 17 yr. old son and I want to be a mom! I cant be living in fear of having dissociative disorders, hallucinations, psychotic like episodes. Please help me. My name is XXXX: YYYY@YYYY , 000.000.0000 Doctor: Hi, I am Dr Mittal. I have read your message. I think I can help you. First, relax. there is no need to be worried. Seizures and epilepsy are usually both well controlled with simple medicines.Second epilepsy is basically repeated seizures. So please relax.Doing eeg is not a problem. It will only show If something more than medicines need to be done. I think it would be better if you let the doctors do the tests.I have tried to make it as simple as possible. I hope that the information contained in this message will be able to help you find the best solution for your problem. please feel free to contact us for more information .Best of luck. Dr Saumya Mittal."
},
{
"id": 80840,
"tgt": "What is the treatment procedure for Pneumonia tuberculosis?",
"src": "Patient: Hi i m arozli capili of philippines, i have an active pneumonial tubercolosis as of today. what are the medicine should be taken and how is the processing of the medication?is there a posibilities that after the medication or tretment done that the scar on my lungs will be clear in x-ray so that i can pursue my career at any international countries? Doctor: Thanks for your question on HCM. I can understand your situation and problem. Treatment of tuberculosis is in two phases. 1. Intensive phase for 2 months.2. Continuation phase for 4 months.In intensive phase following four drugs are given.1. Rifampicin (R)2. Ethambutol (E)3. Isoniazid (I)4. Pyrazinamide (P)And in continuation phase 2drugs Rifampicin and isoniazid are continued. Total treatment duration is of 6 months atleast.So better to consult pulmonologist and discuss all these and start appropriate treatment. After appropriate treatment Tb lesions will be healed by either fibrosis or calcification. And these scar lesions will remain through out the life. Since these are scar, inactive lessions, no treatment is required for this. Sp on completion of treatment again consult pulmonologist and discuss your visa problems."
},
{
"id": 90149,
"tgt": "What causes weird feeling in the stomach with unprotected sex?",
"src": "Patient: I had a unprotected sex with my boyfriend. I get a weird feeling in my stomach. I still get periods. I usually have heavy periods all the time but not for the past 6 months. I'm always feel really hungry and when I'm eating my stomach growls. there's this one time I was with my friend and my stomach growled real loud and felt like it came from my lower right stomach... can I get an answer? what's wrong with me? Doctor: This may be relate to stress and irritable bowel syndrome as per your history of weird feeling. The stress increases the intestinal motility.It would be better to get the blood tested for :: Complete blood picture, blood sugar and thyroid function tests as you have increased hunger and growling If these are normal , you can go for an ultrasonography and enteroclysis can help."
},
{
"id": 158573,
"tgt": "Suffer from internal hemorrhoids. Took radiation for prostate cancer. Itching in the rectum, blood in stool, low platelet count",
"src": "Patient: I have internal Hemerrhiods that flare up from having external Radiation treatments for Prostate Cancer last June. The radiation brought my PSA down to .02 They bleed from time to time.. I have a itch in the rectum when fecal matter gets stuck and I don't want to strain ,because I know it will start to bleed. Suppositories help some ,but not always.My DR. wants to do a anoscope exam. but says my Platelete count is to low at this time. I am taking Nplate shots to rise the count. Their is blood on my stools a lot of the time. What else can I to help this out. I am not constapaited .Stools are soft .It is like the last of the fecalmater just won't come out easiely Doctor: HiThe treatment of thrombocytopenia is a must for you. You may also consider other options like recombinant human interleukin II, platelet transfusions, etc. Recombinant human interleukin II has some side-effects like fluid retention, dyspnea, etc.Also do not strain for the last bit of fecal matter to come out. Try over the counter medicines to apply at the anal sphincter. Discuss with your physician about this option.Dr Vaishalee"
},
{
"id": 173575,
"tgt": "What causes stammering in children?",
"src": "Patient: Sir, My daughter age 2 and half year old stammers just 2 weeks agao but she can sing well .. she sometimes take time to say... She talks fluently earlier but now she talk less. she is very afraid of sound of some vehicles sometimes.. Sir what can i do. Doctor: DearWelcome to HCMWe understand your concernsI went through your details. I can understand. If your child can sing well without interruption, earlier was fluent and not now, then the problem should be stress and anxiety related. You must talk to her teachers and fellow students. She must have messed with something at school which caused anxiety and some phobia. You could also talk to a psychologist for diagnosis and cure.If you require more of my help in this aspect, please use this URL. http://goo.gl/aYW2pR.Make sure that you include every minute details possible. Hope this answers your query. Available for further clarifications.Good luck."
},
{
"id": 206651,
"tgt": "What causes loss of coordination and imbalance?",
"src": "Patient: i take three different anti psychoticts, plus lithium and an anti depressant. within the past five months ive had problems with balance, coordination and thought process. i have been psych on meds for over twenty years. is it my illness or could i be having adverse effects from to much medication? Doctor: Hello,Three antipsychotics are not recommended for any of the illness. Your problem imbalance and loss of coordination are most probably due to the side effects of antipsychotics. These drugs can cause extrapyramidal side effects which include tremors, imbalance, incoordination, numb face etc.Please talk to your treating doctor about this.Thanks."
},
{
"id": 113035,
"tgt": "Suffer from back pain, shortness of breathe and lightheadedness. Kidney problems?",
"src": "Patient: I am having back pain primarily on my left side about 3-4 inches above waistline and some shortness of breath, and now feeling a little light-headed and achy, but no fever (or change to urine color, blood, etc.). Based on a good article I read on the site, I don't think it's kidney related (but was concerned originally given some of the symptoms if it could involve any of my internal organs). I have had some disk issues over the years, but those usually subside within 3 days (I am going on 4 now without improvement, and have been traveling so hard to get to a doctor's office). Doctor: Hello. Thanks for writing to us. The pain on the left side that you have described can be related to a renal cause or a muscular pain. It is best to get an ultrasound scan done to rule out any internal organ damage. Mild pain killers and hot fomentation should help relieve the pain. I hope this information has been both informative and helpful for you. Regards, Dr. Praveen Tayal drtayal72@gmail.com"
},
{
"id": 121860,
"tgt": "How to cure pain in neck,back and stomach?",
"src": "Patient: Sir I am 23 years old (male). I have a regular pian in my neck, back, and a stomache pain. back pain and neck pain is severe and unbearable that i start to take painkillers and anit-inflamtion drugs daily. i also have a weight gaining difficulties. stomache pain is a kind of inflamation and so.. please, kindly advice! Doctor: Hello, About back pain, see back pain is due to weakness and tightness of back and thigh muscles. I advise you to go for Physiotherapy treatment and ultrasound therapy and IFT therapy will resolve pain and exercises will improve muscles performance. Hope I have answered your query. Let me know if I can assist you further. Take care Regards, Hemang S Jani, Physical Therapist or Physiotherapist"
},
{
"id": 32446,
"tgt": "What is the duration of typhoid fever?",
"src": "Patient: Hello Doctor, my son who is 8 years has been detected with typhoid on Monday ie. 07/03/11. Since then he has been on medication (Secef and Azithral twice a day), but his fever is not going down from 102 or 103 or 104. Please suggest how may days it should normally take to get the fever down. Should we get some other checkups done. Doctor: Hi, you are taking correct treatment for typhoid typhoid is a major disease and it may take upto 10 days to treat. In my opinion, you should take the drugs properly, do temperature charting and wait for 2 days. If any complications like persistent vomiting, pain abdomen, jaundice occurs then the child will need admission. I advise you to look for these complications, give drugs regularly and if any complications arise then admit the child. I hope this will help you. Take care."
},
{
"id": 75303,
"tgt": "Suggest treatment for cough and back pain along with fever",
"src": "Patient: My husband has a bad cough and back and leg pain. He never complains or is sick. About a month ago he did get the flu pretty bad missed a week of work. He has never missed work in 25 yrs. He also is working nights . Today he woke up and asked for motrin because he can't get comfortable he is very uncomfortable. I am worried he may have phnemonia or something worse. He doesnt want to go to doctor. should I be worried? Doctor: HelloI read your concern and i think that your husband should see a lung specialist.If i were his doctor i would do some blood testsBlood countCRPChest X Ray and if he is a smoker or ex smoker maybe a chest CT and than a Spirometry test.PPD testSputum smear for TB.EKGIf it is again a pneumonia it is very important to know the reason it is come back after one month?The pulmonologist will help you further.Thank youDr.JolandaPulmonologist"
},
{
"id": 21906,
"tgt": "Is placement of stent enough instead of open heart surgery after heart stroke?",
"src": "Patient: Sir My Father in law -50 got heart stroke after tests doctor told us that with in 4 weeks need to do the open heart surgery.We taken that report and consult another 2 specilist they suggested that Stunt is sufficient no need heart surgery.We are unable to decide please suggest Doctor: Hi ThereI understand your concern as it's a big decision to make. As you have consulted many cardiologist and they have a different opinion so in this case it's important to see your father's angiogram first so as to see how many blockages are there and how critical they are and it's not wise to give my opinion with reviewing angiogram.However in general I can tell you that if the blockages are at multiple locations and have a diffuse course or if the Left main coronary artery ( also called widow maker)is affected it's advised to go for a bypass surgery. I know it's sounds dangerous to go for a heart surgery but studies have shown that bypass surgery gives slightly better long term survival rates in good surgeon's hands. Hope I have made my point clear that will help you decide.Good Luck"
},
{
"id": 137467,
"tgt": "Why does the foot keep hurting after twisting the ankle years ago?",
"src": "Patient: I twisted my ankle & both sides of my foot still hurts... Did this August 2013. I am currently on steroid pack but doesn t seem to be helping . Any suggestions. I have had an MRI & showed fluid around the lateral part of the ankle so I just try & put ice on it to help with swelling . So my question is why is it still bothering me. Doctor: You have chronic instability of ankle resultated in effusion of lateral sideDefinitely your movement of affected ankle would b less than the other side.so you must see a physio they will mobilize your ankle and perform glides and accessory glides which will normalize the range of motion further they will strengthen your ankle muscle in midrange and last they will give you balance board and wobel board to improve proprioception and condition your ankle to cope up injury and sudden twist happen in daily living.For right now as home program you must do iceing 10 to 15 minutes 4 time a day adding to rest in elevation and range of motion exercises and circular kneading with finger pad to mobilze swelling .and then keep you leg in elevated position and just do active ankle pump exercise."
},
{
"id": 199425,
"tgt": "What causes tenderness in the penis after masturbation?",
"src": "Patient: Hello, Last night, I was Masterbating and it was pretty much the same as normal. But when I woke up this morning, I noticed a discomfort it my penis. I can t really explain it but it was it was very tender and it burned. So when I looked at it I noticed it was swollen. As the day went by (on a school day) it didn t really bother me until I had to use the bathroom. Then when I got home it s still how it was the same as from the morning except this time it Is itching. Please help me! I m 15 years old. Also at one point last night I masterbated when my penis was soft. And I did it kind of rough. Could it be like this because of that? Please help me! Doctor: Good Day and thank you for being with Healthcare Magic! Yes it was because of excessive roughness during masturbation. Let it rest for a week without masturbating and drink plenty of water and vitamin c to let your body heal. If symptoms persists please a your local urologist so he can do a physical examination on your penis. I hope I have succeeded in providing the information you were looking for. Please feel free to write back to me for any further clarifications at: http://www.HealthcareMagic.com/doctors/dr-manuel-c-see-iv/66014 I would gladly help you. Best wishes"
},
{
"id": 21062,
"tgt": "What is the treatment for Blood Pressure?",
"src": "Patient: helloi am 31 yrs hypertensive for past 1yr under medication (tragit 40), for past 20 days i find difficult in sleeping for even 4 hr a day because i can feel my heart beat. i had consulted physician he had prescribed propanolol, sedative and asked me to continue targit 40 and my BP is under control. whenever i take sedative i'll my sleep but my problem persist when i stop sedative, i am not an smoker or alcoholic. pls give me suggestion. Doctor: Hello Sir/ma'am.your problem at the outset seems to me like an anxiety disorder. Either you are worried for some reason or there is something in your subconscious mind that is bothering you . So to me what i would suggest is ..you go for some relaxation therapies like yoga or meditation or some relaxing techniques like body massages or reiki that would cool down your mind and in case you are unable to do it all by yourself then take the help of a clinical psychologist to guide you"
},
{
"id": 48918,
"tgt": "What does this ultrasound report signify?",
"src": "Patient: my helper's ultrasound result: normal sized kidneys with cortical cyst and peluiectasia on the right. Fluid filled anechoic focus measuring 1.7x1.5x1.6cm is seen in the inferior aspect of the right kidney. Is this something serious? How is this treated? Doctor: You have renal cyst and thus looks like a simple cyst which usually does not cause any issue unless the size is really big, and at this stage I suggest you need not take any active treatment for the cyst but have a followup every year with an ultrasound test and intervene if it gets larger or you have pain or fever due to an infected cyst.Hope I have been able Yp allay your fears."
},
{
"id": 209196,
"tgt": "What causes depression after the finance s death?",
"src": "Patient: Hey Dr. Samuel.. not sure where to start but my mother is 62. she was hospitalized on the 9th of May. she had been suffering from diarreah, nausea vomitting and loss of apatitie for almost a month before i got her to agree to let me take her to the er. they said she had a stomach infection and was treating her for that..they did tons of tests..finally said they thought she was bleeding somewhere in her stomach and gave her blood. on the 13 while she was still in the hospital her husband passed away. he had a heartattack at home..she left the hospital so we could make funeral arrangements. when i took her back to the er a week later for them to readmit her..of course she was still ill and grieving on top of that. they said they could find nothing wrong with her and sent her home. she has gotten better none of the symptoms she had before only now she has begun to talk crazy..out of her mind stuff. she doesnt know the day or time. she knows who i am but talks as though her husband is still alive..she asks me all the time if he has made it home, or did jay come home last night, where is jay...etc...looking for the school bus like she is waiting on me to come home from school...im 39yrs old...what do i do? should i wait to see if she comes around..or do i take her to the er or specialist? im afraid if they send her for mental evaluation she will get hurt because she has to be assisted and watched all day..please help me.. sincerelly Liz Doctor: HiYes she is might be suffering from depressiin but appropiate term is grief reaction.In grief reaction the stages goes like first deniel that the person is still alive thn bargening n shockThn acceptance of decased person n finally filloe depression Yes defenetly she requires treatment periculary psychotherapy. U and ur relarive try to evaluate all these things to talk with her n never let her aloneGrief therapy is necessary in this case if require than mild dose of medicine can be addedConsult psychiatrist n he will further do treatment in form psychotherapyHope u get bestThank u"
},
{
"id": 65816,
"tgt": "How can cysts be treated?",
"src": "Patient: Hi dear 2 month ago I gon through surgery It was cysts. It also send for test bt it wasn't simple cyst bt after month of my surgery again I went for altra sound nd doc said again cysts he said it is chocolate cyst. Plz do u help me nd my ca125 is 19.40 now bt before surgery it was 4856 Doctor: Hi, dearI have gone through your question. I can understand your concern. chocolate cyst can be treated by oophorectomy. It means removal of that ovary. Your ca 125 is very high previously it is suggestive of ovarian carcinoma. You should check your histopathological report. Now your CA 125 is ok. Jyst consult your doctor and plan for surgery accordingly. Hope I have answered your question, if you have doubt then I will be happy to answer. Thanks for using health care magic. Wish you a very good health."
},
{
"id": 162758,
"tgt": "Is medical intervention required for speech impairment in a toddler?",
"src": "Patient: My 2 1/2 year old grandson rarely looks at us in the eye. His speech is somewhat less than other children his age. He could stay with an electronic device 24/7. To get him to eat his parents give him an iPad and feed him while he s to distracted to not eat. He bites his nails to the point they are bleeding. I m am a teacher and his tantrums among other things resembles autistic children I have worked with. Do you see a need for professional intervention and if so what? Doctor: Hello and Welcome to \u2018Ask A Doctor\u2019 service. I have reviewed your query and here is my advice. Depending in your description I think it is necessary to as for professional help. It looks as a development problem that need a diagnosis. And of course the first thing to do is to eliminate all the electronic device from him because they don't bring nothing good in this case. Hope I have answered your query. Let me know if I can assist you further."
},
{
"id": 15380,
"tgt": "Rashes in groin area. Taking warfarin to treat blood clots in legs. Causes for symptoms?",
"src": "Patient: I have been taking Warfarin for about a month to treat blood clots in my leg. About a week ago I got a rash in my groin area. I first treated it with triple antibiotic. That didn t work so I thought it was possibly jock itch. I have been treating it with Lotrimin Ultra. I still haven t mush of a change. What could be causing this? Doctor: Hi.rashes in the grion is usually fungal infection.lotrimol will take lot of time .hence try to use sebifin ointment along with sebifin tablets it gets cured"
},
{
"id": 91350,
"tgt": "What is causing stomach pain, tiredness and vomitings after viral fever?",
"src": "Patient: Hi My 11 yr old son had got viral fever for a week. For last 2 days he didnt have fever, but he complains stomach pain and tiredness. Today he had couple of vomittings and motions. He doesnt have fever, and is eating well. Its just that he is feeling very tired and stomach pain and feeling of vomitting. pls advice! Doctor: Hi.Thank you for your question.Let me assur you that your child is fine and you need not panic.Your Child is suffering from Viral fever which explains all the symptoms like bodyache,tiredness,vomiting,Stomach ache.Dehydration caused by the illness can in turn increase the symptoms like tiredness.I generally advice to my patients the following:1)Take Rest2)The illness will subside on its own.No need to go for unnecessary Antibiotics.3)Oral Rehydration Salt4)Proper Diet with Vitamin supplementation.5)Antipyretics if necessay.Hope i answered your questions.Wish you and your family a healthy life.Regards,Dr.Arun Prasad"
},
{
"id": 153055,
"tgt": "Is ovarian cancer suspected with the following test report?",
"src": "Patient: Hello. N December I went to gyncologist for light pelvic pain.. Everything looked fine. They said I had a follicle on my overy...no big deal. I have been so worried about overian cancer I just did a ca-125 blood test.i looked at my results online and noticed by bun/creatin level was 25.....normal range 7-20...could this be a sign of overian cancer. I'm only 28 Doctor: Greetings.BUN/CREAT level has nothing to do with ovarian cancer and you need not to worry at all. You can consult a primary physician for raised BUN. Most likely it may be due to dehydration. Regards"
},
{
"id": 41886,
"tgt": "How can semen liquefaction time be reduced?",
"src": "Patient: Hello Doctor,Please check my semen analysis report and tell me it is okPHYSICAL EXAMINATION--------------------------------Collection Time : 5.25 PMReceiving Time : 5.40 PMVolume : 1.5 MLLiquefaction Time : 45 MinutespH : 7.5Color : White OpaqueMICROSCOPIC EXAMINATION-------------------------------------Sperm Count : 115 MillionMOTILITY DURING FIRST HOUR-----------------------------------------Rapid Progressive : 35Slow Progressive : 25Very Slow Progressive : 10Non Motile : 30MORPHOLOGY--------------------Normal Spermatoza : 70Abnormal Spermatozoa : 30Viability : 35Leucocytes : 1-2 / hpfErythrocytes : 0-1 / hpfSperm Cells : Occassional / hpfEpithelial Cells : Occassional / hpfSperm Agglutination : +Please tell me this report is normal for fertilization. Do I need to consult a doctor ? We were trying for the past 5 months, but not succeeded. How can reduce semen liquefaction time from 45 minutes to below 30 minutes ?\\Sperm Agglutination : + is normal ?Semen Volume can be increased from 1.5 ml to 2 and above ?What should I do Doctor ? Please help me Doctor: Hi welcome to healthcaremagic.I have gone through your question.Semen liquefaction time should be 20-30 mins.Higher time suggest glandular dysfunction.I would suggest to do post coital test to clinically signify the problem. In pct if sperms are capable to reach cervical mucus then no treatment needed.Otherwise there homeopathic medicines which helpful to decrese liquefaction time which needs 2-3 months to get result.Semen vol can be increased by 2/3 days sexual abstenance. Sperm agglutination + is normal.Hope i have answered your question.Would be happy to help you further.Take care."
},
{
"id": 99614,
"tgt": "What causes back and lung pain with dizziness and headaches?",
"src": "Patient: 22 year old, 5 foot 5 about 10 stone, have asthma bought on by allergies. This weekend been getting back pains/ lung pains- although not linked to any kind of cough and doesnt sound wheezy, my sister a trainy doctor said my lungs sound crackly rather than wheezy- have needed to take my sabutmol more than usual. Also been feeling dizzy, weak, tired and getting headaches (never usually get headaches!) Feels a bit like im not in my body properly- constantly light headed. Have had a weekend of doing nothing particularly stressful, no alcohol, no cigarettes etc. Doctor: Hii have gone through your questioni appreciate your concernsince you haven't mentioned anything about breathlessness and duration of illnessin my opinion you should do steam inhalation and deep breathing exercisesyou can go for swimmingof you can update me with more information like blood profile and an X ray chestI will be happy to help you Thanks for using health care magicgood luck"
},
{
"id": 40771,
"tgt": "How can infertility be treated?",
"src": "Patient: Hi doctor, we r planing for 1 st kid , we did ultrasound 8th day , 10, 12,14 , 16th of her cycle, 14 the day her follicles size. Was 17*17 doctor given injunction to rupture( as my wife having pcod , last cycle follicles dint rupture it own size became very big ) , 16 th day of ultrasound result was it ruptured , since the time she given injunction (14 July Friday ) Friday , Saturday, Sunday , Monday we had intercourse daily an average of 2 times , how much percentage we have chance to get positive report ?? As we r eagerly waiting for baby , pls do me a favour pls text me your number for more questions 0000 Doctor: Hi I think there is a chance of pregnancy. Do a urine pregnancy test to see if pregnancy is there or not. If positive, consult your doctor. If negative, wait for one more week and repeat the test if periods don't come. The chance to get pregnant in one cycle of timed intercourse is 7 to 8 percent only. So, try atleast 3 to 6 cycles. Hope it helps."
},
{
"id": 17053,
"tgt": "Suggest remedy for high BP",
"src": "Patient: I there, my mother just came back from the doctors office with a blood pressure reading of 280 over 130, he told her to go home and take her medicine because she has not taken it in 4 days, do you think I should take her to the ER for them to get it under control Doctor: Hi, Usually, high blood pressure patients start tolerating high BP. If your mother is not having any major symptom besides high BP, let her take her medication and physical rest. She must avoid all physical and mental exertions for the next 3 or 4 days. Even morning walk should be avoided for the next 3 or 4 days. Ask her to avoid tea, coffee and alcohol also. Ask her to relax. Hope I have answered your query. Let me know if I can assist you further. Regards, Dr. Varinder Joshi, General & Family Physician"
},
{
"id": 6396,
"tgt": "Should I take prenatal for difficulty in conceiving ?",
"src": "Patient: i want to get pregnant and i dont know if i should start by taking prenatal my first daughter i got pregnant 8 yrs ago coarse i went off birth control its not happening soon enough and i feel its getting late Doctor: Hi Welcome to healthcaremagic When did you stop taking birth control pill? Consult your gynecologist for evaluation and management. take care."
},
{
"id": 151883,
"tgt": "What could be discoloration in spine resulted from MRI ?",
"src": "Patient: My mother just had an MRI; she had alot of back pain and thought she d get it checked out. They found discoloration (darkened brown areas) from what was described in areas down her spine. She was instructed to get a CAT which she s doing. I m desperate to know what the possibilities of this could be? Any help is much appreciated, thanks, Mike Doctor: Welcome to Healthcare Magic There can be many reasons for back pain and the MRI picture you presented. It could be hemangioma in the vertebral column compression the spinal cord/nerves causing pain. It can also be cancer which has spread to spine from source elsewhere. You need to give full history of your mother. Does she have any other problem. Her age, any bleeding from vagina, any breast problems, abdominal problems as well."
},
{
"id": 104599,
"tgt": "Severe itching in eyes, running nose, headaches, stiffness. Medication to treat allergies?",
"src": "Patient: Dear Sir, I have a kind of alegerie, basically in the morning i have itching in the eyes, running nose with the headache and stiffness in the entire body, for past five years. Please let me know, to whom do i need to consult and contact in XXXX(Doctor) or what kind of medicines do i need to take?? Thanks. Doctor: Hello Welcome to Health Care Magic You are having symptoms of itching in eyes in morning, running nose, headache and stiffness in entire body. This is happening since last 5 years. It may be due to a number of reasons. Most likely reason is infection in sinus mainly frontal sinus. Chronic sinusitis may cause heaviness in head, running nose, congestion in nose and throat, pain in eyes, and headache in frontal region of head. To rule out this opinion of a ENT doctor is needed. X ray of sinuses need to be done to rule out sinusitis. Secondly as it is for about five years, continuously and in mild grade. It may be due to somatic symptoms due to mild depression. So if situation persist then opinion of a psychiatrist is must. Thanks Dr. Seikhoo Bishnoi"
},
{
"id": 201049,
"tgt": "Suggest treatment for injury on penis head",
"src": "Patient: I recently had protected sex and during oral still protected I noticed that the condom had filled with blood after closer inspection it looks as if I been pinched on the head of my penis and it had burst is this something that I need to go to the emergency room about or just apply anti bac cream and let it heal? Doctor: Thanks for asking in healthcaremagic forum.In short: Frenulum tear might have caused this.Explanation: Bleeding from male genital organ while sex can occur after the tear of frenulum. So, I would like to suggest you to visit a doctor as early as possible for your problem. Small tear heals by itself after sometime if given rest. Otherwise sometimes need to be checked if it has to be sutured or not. Good luck"
},
{
"id": 123811,
"tgt": "How long does fibromyalgia take to be cured?",
"src": "Patient: I ave fibromyalgia, but for te last 5 monts it as been really bad. I affected bot nees and arem to te point it affected my walking AND USING MY ARMS.JUST WEN I TINK IRS BETTER IT GETS WORSE IN ANOTER LIMB. OW LONG DO YOU TINK IT WILL LAST TIS BAD AND WAT CAN I DO TO ELP? Doctor: Hello, It is nothing but a musculoskeletal problem, maybe the cause is unknown. But genetic causes, emotional factors, nutritional and stress factors may play a role. First, the triggering factors should be avoided. Do regular exercise and yoga may useful. Avoid the intake of alcohol and smoking. Antidepressants and pain relievers may help. With drugs and lifestyle changes you may keep it under control. Hope I have answered your query. Let me know if I can assist you further. Regards, Dr. Penchila Prasad Kandikattu, Internal Medicine Specialist"
},
{
"id": 131425,
"tgt": "What is the remedy for the pain and tingling in the arms with swelling in veins?",
"src": "Patient: Hi, I fell hard on my forearm last night and I haven't had an X-ray but it is very painful and both my arms are tingling. My spidery veins in both arms looked burst in little river patterns even though I didn't hit my other arm the veins reacted the same way as the impact arm Doctor: HiYou must go to ER for x rays evaluation to rule out fracture.some splinting or temporary plaster slab must be applied to immobilize the forearm.Take advil and Tylenol tabs thrice a day after meals and serratiopeptidase tabs thrice a day for 5 days.keep forearm in a broad shoulder arm sling and do finger exercises.consult an orthopaedic surgeon for further evaluation"
},
{
"id": 186771,
"tgt": "How to cure a large soft bump on the roof of the mouth?",
"src": "Patient: I am seeing a debtist for a large soft bump that came up suddenly on roof of mouth I can move around with tongue. The dr. has cut into it but I dont think anything has come out he has injected it with antibiotics why is it taking so long to go away three weeks should it be surgically be removed thank you Doctor: Hello and welcome.Thanks for sharing your concern.It appears there was an abscess developed palatally(that is in the roof of mouth.your dentist must have drained it out.Allow it to heal.It is more important to treat the underlying cause first.Please discuss it with your dentist it might require a root canal too.Antibiotics should help in recovering fast.Hope it helps.Thanks.Take care."
},
{
"id": 211706,
"tgt": "Stress, vertigo, stomach problems. Friend having same symptoms. Virus?",
"src": "Patient: I have been under a lot of stress for the last three weeks. Yesterdy I had vertigo to the point of really being sick at my stomach. I took some mecklazine (spelling probably wrong) this morning and am feeling some better. I really thought it was stress, but a friend has the same syptoms that started yesterday. Could this be a virus? Doctor: DearWe understand your concernsI went through your problem description. Feeling sick in stomach and having vertigo is too common to clearly diagnose the problem. But I do think that you might be safe, because now you are feeling well.Stress is everybody's state of mind. Whenever we are compelled to do anything which is beyond our actual limit, we get stressed. Such stress feeling and body tiredness should go away after you take proper rest, body as well as mind. That is it. Don't worry unnecessarily and take unwanted medicines. Self diagnosis can be dangerous.Hope this clears your doubts. Available for further clarifications.Good luck."
},
{
"id": 56605,
"tgt": "What is the treatment for gallstones and sludge?",
"src": "Patient: hi, I m 27 male and having pain in my abdomen, it s 5th consecutive day of pain as well as constipation is on it s hike, doc in my locale asked for ultrasound in which I received echogenic sludge is seen in gall bladder lumen. I was prescribed Domstal, Zinetac, Drotin-M and Merizyme syp. none is helping please guide me what to do.. Doctor: Hello Definitive treatment of gall bladder calculus with sludge is surgical removal of gall bladder.Now a days,operation is done laproscopically.Recovery is fast by this method.It is not wise to keep gall bladder calculus for long time.It is prone to get obstruction and infection.It may also be cancerous if kept for long time.You are rightly prescribed symptomatic treatment but you should go for definitive treatment.Take CareDr.Indu Bhushan"
},
{
"id": 220323,
"tgt": "What causes stoppage in periods?",
"src": "Patient: I am 21 with a 21 month old and i haven t had my period it is like almost 2 days late but i took a cheap dollar general $1.00 one and it came back that i am not preg but my boobs hurt and i don t have any sytomes of be having or starting my period is there like way that the cheap test couldn t read it. Couse i use the same one when i found out i was preg with my son please help me Danielle Doctor: Hello dear,I understand your concern.In my opinion the urine pregnancy test gives accurate results a week after missed period.So i suggest you to repeat the test after a week.The sore breasts are one of the symptoms of pregnancy.But the pregnancy is confirmed by urine pregnancy test and ultrasound.There is nothing like the cheaper kits give negative results.So relax and consider repeating the test.If the repeat test is also negative the pregnancy is unlikely.Hope this helps.Best regards...."
},
{
"id": 77541,
"tgt": "What causes heavy & burning of chest with",
"src": "Patient: tired all the time, chest and arms feel heavy. take a deep breath and my chest hurts. get a burning feeling in my chest on left side. have been to the emergency room, to my doctor and to the cleveland clinic. all enzymes, ekg's, stress tests, nuclear stress tests all normal. Doctor: Hi thanks for asking question.First you have to rule out is anemia.Estimate your hemoglobin level for it.If hemoglobin comes low then chances of weakness might be there.For that iron tablet has to be taken.Peripheral smear study also helpful.Second possibilty is of simple musculoskeletal pain,that might be because of heavy exercise or improper sleep bad posture.Simple analgesic with proper sleep posture will be helpful for that.Third this pain may be as part of viral infection or respiratory cause.Do CBC with chest x ray for that.Maintain healthy nutrition with more water.Mostly with time in few days you will be alright.If still have problem then further work up will be done according to physical examination.I hope i have solve your query"
},
{
"id": 98080,
"tgt": "Suffering from rheumatoid arthritis. Taking allopathy. Can I take steroids?",
"src": "Patient: Hi My mom is suffering from rheumatoid arthritis from 2 and half years, she started taking allopathy medicines initially but couldn t find any relief at all from the pain killers.. so she shifted to ayurvedam medicines..but since 6months pains got vey severe and she is unable to do basic day to day work too.. doctor prescribed zempred and some pain killers for 10 days ..is it ok to take steroids ..I heard that these medicines have side effects which tamper eye sight and other major health issues.. she is suffering a lot please suggest ? Doctor: Hello! Thanks for being with us. Rheumatoid arthritis (RA) is a chronic, systemic inflammatory disorder that may affect many tissues and organs, but principally attacks joints. It can be a disabling and painful condition, which can lead to substantial loss of functioning and mobility if not adequately treated. Various treatments are available: Non-pharmacological treatment includes physical therapy, occupational therapy and nutritional therapy but these do not stop the progression of joint destruction. Analgesia (NSAIDs) and anti-inflammatory drugs, including steroids, are used to suppress the symptoms, while disease-modifying antirheumatic drugs (DMARDs) are required to inhibit or halt the underlying immune process and prevent long-term damage. In recent times the newer group of biologicals has increased treatment options. Biologicals like infliximab,rituximab, etanercept, adalimumab, certolizumab,golimumab,interleukin 1 blockers such as anakinra can tried if 3 DMARDs in combination fail to respond. I would like suggest to start her on low dose steroids e.g. prednisolone 5mg OD and 1 or 2 DMARDs like methotrexate,hydroxychloroquine ,leflunimide,sulfasalazine or azathioprine. Low dose steroids does not harm much.Its benefit will be more than its harmful effects. Aceclofenac or indomethacin can be given for pain relief. Consult with a Rheumatologist for better management. Hope this will help you. Wish your mom good health."
},
{
"id": 133416,
"tgt": "Suggest remedy for redness, pain and swelling in toe",
"src": "Patient: Second toe next to great toe gets red, swollen, pain with the blood seeming to settle on the bottom of the toe. Soon it will break open stay runny like a blister. This happens only and always to the same toe around the end of October and stays like this until January or after. Very painful can hardly wear a shoe. This has been happening now for several years. I have seen 5 or 6 doctors and no one has a clue what it is or the cause or how to treat it. Please help. Doctor: Hello sir,Your complaint is suggestive of presence of sinus near ur toe ,It could be infected corn or it might also had involved bone as osteomyelitis.Consult an orthopaedician,he might advice some blood investigation, xray and might require surgical treatment.Hope u get well soon.Thank you"
},
{
"id": 222305,
"tgt": "What are the early signs and symptoms of pregnancy?",
"src": "Patient: I am 26 years old. It is going to be two months with out menstrual period. I feel a drastic change in my body. I feel tired, dizzy with inconsistent light headaches, breasts are sensative and my appetite has increased. Also, I have cravings for sweet and spicy food. 3 weeks ago I saw my doctor. I had a urine and blood test. Both tests were negative. However, 3 days before visiting the doctor, I took a home pregnancy test. The results were one defined line and one faded line. I am very confused and concerned. What should I do at this point. Doctor: Hallow Dear,By completion of two months after missed period, urine pregnancy test does give reliable results. If your test is faint (weak) positive, there is a chance that your pregnancy may be growing out side the uterus, may be in the tubes (Ectopic pregnancy). Ectopic pregnancy is a very serious condition which when ruptured leads to alarming bleeding and requires surgical intervention with blood transfusions. Please report to the Obstetrician without any delay to get the condition diagnosed/ruled out. If ectopic pregnancy, get it treated before you land into any emergency. Wish you the best,Dr. Nishikant Shrotri"
},
{
"id": 120784,
"tgt": "What causes painful and swollen labia minora?",
"src": "Patient: Hi I am a 29yrs old female, I have a 4 months old baby. No relavant medical history. Present complain: I have noticed yesterday a marble sized swallow on my right labia minor ( on the inside) red, quite painfull and seems to increase in size. What to you advise me to do about it! Thank you Doctor: Hello,I read carefully your query and understand your concern. Your symptoms seem to be related to Bartholini cyst.I suggest soaking in a few inches of warm water either in a tub or sitz bath, four times a day for a few days may resolve even an infected Bartholin cyst.Taking over-the-counter painkillers such as Tylenol may help with discomfort.If the symptoms continue, I suggest to take a cycle of antibiotics. Hope my answer was helpful.If you have further queries feel free to contact me again.Kind regards! Dr.Dorina Gurabardhi General &Family Physician"
},
{
"id": 182109,
"tgt": "Suggest treatment for painful sore on tongue",
"src": "Patient: I HAVE A SORE ON THE SIDE OF MY TONGUE,I THOUGHT T WOULD GO AWAY ON IT'S OWN IT'S VERY PAINFUL IT IS SPREADING TO THE BACK OF MY THROAT,ONLY HALF OF MY THROAT IS SORE,AND ON THE SAME SIDE IT HAS TRAVELED TO MY EAR WHICH IS ALSO VERY PAINFUL NOW WHAT ARE THE POSSIBLE REASONS FOR THIS?? Doctor: Hello Ulcer on the side is normal if it subsides within 2 weeks... if you have it for more than two weeks then you need to show to a dentist or ENT SURGEON to go for further examination.Use saline gargle and avoid spicy and hot food.if you are sure that it is increasing in size and is more than two centimetres, immediately show to a dentist.thank you."
},
{
"id": 48333,
"tgt": "When should the following medicines to be taken Amlodipine besylate and Lisinopril ?",
"src": "Patient: I am taking medications for heart and kidney issues .Can these meds be taken at same time in morning or should I very schedule Amlodipine Besylate\u00a0\u00a0\u00a0\u00a0\u00a010mg Lisinopril\u00a0\u00a0\u00a0\u00a0\u00a040 mg Metoprolol Succinate Er \u00a0\u00a0\u00a0\u00a0\u00a0100 mg Digoxin\u00a0\u00a0\u00a0\u00a0\u00a00.125mg Furosimde tabs\u00a0\u00a0\u00a0\u00a0\u00a060mg Losartan potassium \u00a0\u00a0\u00a0\u00a0\u00a0100mg Doctor: HIThanks for posting your query to Healthcaremagic. Furosemide should be taken in morning only . Digoxin any time of the day ( but timings should not vary ) Rest all that is Amlodepine , Lisinopril , Metoprolol , Losartan are all anti hypertensive medicines and my recommendation is to take 2 of them in morning and 2 of them in night . For example Amlodepine and Lisinopril in Night and Metoprolol and Losartan in Morning . Also these anti hypertensive medicines comes in combination . Kindly ask your doctor for combination medicines so that the pill burden comes down. Hope this information was useful to you. Any clarifications feel free to ask . Regards."
},
{
"id": 215943,
"tgt": "What causes pain in head 6 months after injury?",
"src": "Patient: HI, I fell down from bike and hit my head back to road. at that time, i ddnt get any sense of pain. i went to office. after 2 hours, i got pain in my head and vomiting sensation. i was admitted to hospital and was treated. after 6 months, now, i am getting pain in my head, whenever, i am thinking. i am not able to view my desktop screen continuously. can plz suggest me, is there anything wrong. Doctor: Hello and Welcome to \u2018Ask A Doctor\u2019 service. I have reviewed your query and here is my advice. It may not be linked to previous injury. However, you can go for a repeat CT scan to rule out any delayed brain injury or concussion. As of now, you can take analgesics, like Ibuprofen or Diclofenac for pain relief. Wishing you good health."
},
{
"id": 225273,
"tgt": "Will Norethisterone taken for heavy bleeding caused by iud removal affect my fetility?",
"src": "Patient: i had iud removed within 2 months of insertion due to prolonged bleeding of 40 days.with next cycle i has to use i pill. Following that i had to use tid norethisterone due to heavy bleed.now i am have no period past 4 months. I have been feeding my 14 m baby only nights. Please help me. Does it affect my fertility Doctor: HelloThanks for writing to us with your health concern.Absence of periods can affect your fertility.You need to see a gynecologist.Firstly have a pelvic ultrasound scan done transvaginally.This will assess the status of the endometrium.Also get following blood measurements done - FSH, LH, TSH, Prolactin, OGTT, Lipid Profile, DHEAS, free testosterone, AMH.It is wise to have endometrial biopsy also and to rule out tuberculosis if you are living in an endemic country.Absent period could be due to various reasons - you need to have an evaluation.Take care."
},
{
"id": 140436,
"tgt": "What causes vomiting with dizziness?",
"src": "Patient: hi, My dad got sick last week while at work and vomited several times. Although the vomiting has stopped he still feels like his head is in a viscript (spelling)? He also had a toothache before vomiting but no longer has that and does not have a cavity. Any ideas? Doctor: Hello, In my opinion, the first thing to consider is food intoxication. An infection also (stomach flu for example) may be a possibility. Rest for a few days and staying well hydrated should improve your father's condition. In case of prolonged dizziness or vomiting, an evaluation by the primary care doctor is necessary. Hope you found the answer helpful. Let me know if I can assist you further. Thank you. Regards, Dr. Erion Spaho Neurologist, Surgical"
},
{
"id": 34041,
"tgt": "What causes rotten egg body odor and bad breath?",
"src": "Patient: Hi I am experiencing rotten egg body odor and breath. It worsens whenever I eat. I produce a lot of gas whenever I eat anything. I have been researching leaky gut because I also experience flatulence smells without actually passing gas. Nurse said I could have a sulfur overgrowth, but didn t give much info or a resolution to get rid of this problem. What could this be? I also get reoccurant sinus infections & I can barely smell unless I inhale hard Doctor: Hello.I suggest you do swabs of nose and mouth at your doctor. Maybe you have some bacterial of fungal infection. And if you find the live patoghen, antibiotics should help you.All the best."
},
{
"id": 217822,
"tgt": "What cause sharp pains in my husbands head?",
"src": "Patient: My husbandMy husband has been having sharp pains in his head. He has alcohol enduced seizures. But he has been having them without drinking. Now for the last 3 days he has had a headache, and NOTHING he does or takes will help. Even when he is asleep he wakes up from the pain in his head. Also about a week or so ago he was riding his bicycle and passed out and woke up laying on the ground with a bloody nose. he had had a few days in a row that he had bloody noses. I am really worried that it may be a tumor. Please help Doctor: Hai.beimg chronic alcoholic he is more prone for all complications like diabetes, bp, atherosclerosis, high cholesterol, which can all be the cause for cerebrovascular accidents and cardiac problems. Complaints of headache, epistaxis (bleeding nose) all this could be due to any problems in the brain , like sub arachnoid haemorrhage, high bp,.pls do a ct brain, or mri brain, sugar levels, kidney test, liver function test by checking all his haemodynamic parameters, and coagulation profile, u can either meet a neurophysician on euro surgeon, they will surely help"
},
{
"id": 197588,
"tgt": "Can any online doctor help me with the following results?",
"src": "Patient: hi! i would like to know what is the main problem of my husband's semen analysis and what would be the solution. we are trying to conceive for the past 2 years.. here are the results:Volume - 3mlReaction - alkalineViscosity - hyperviscousTotal Count/ejaculation - 55,200,000Liquefaction - >180minSperm Vitality - 35%Motility: after 30mins (PR+NP) - 30% after 1 hour - 25% after 2 hours - 20% after 3 hours - 15%Sperm Type of Motility:1. Immotile - 0% 2. Total Motility (PR+NP) - 5% Progressive motility - 25% Non progressive motility - 70%Total Abnormal Forms (Kruger's Clasification) - 30% a. Abnormal Head forms - 15% b. Mid-portion abnormalities - 10% c. Tail Abnormalities - 5%Spermatogenic cells - 1-2/mlPus cells - 1-2/mlRBC - 3-4/hpf Doctor: Higreetings. Mainly 3 important parameters. 1. sperm density is normal .we need only 16million /ml2.Motility.Rapid progressive motility is below normal but moderate is ok3.morphology of sperms seems to be normal. Another findings is prolonged liquefaction. I suggest your husband to consult an andrologist and get him examined for any infections or varicocele. If so treatment can be taken snd motility and liquefaction might improve.Need not worry about single report. There will be variations always. A repeat anslysis after 2 months can be done.Hope my answer helps you. Regards"
},
{
"id": 51149,
"tgt": "Suffering from Ig nephropathy, increased creatinine, urea level. Kidney transplant done. Treatment?",
"src": "Patient: My brother is a doctor he is suffered from Ig Nephropathy and underwent kidney transplant seven years back.Now the level of creatinine and urea is increasing.The nephrologist suggessted to give methyl prednisolone I.V.But after adminstration of first dose the level of creatinine and urea increased immediately.can u give sugestion. we dont want further transplant.if u have any traetment for this pl suggest. Doctor: Hi The appropriate investigation at this point would be a renal biopsy. This will tell us the cause of increase in creatinine which will guide further therapy. Good luck."
},
{
"id": 91270,
"tgt": "What could it be if having pain in abdomen, pus in stools, deep coloured urine, backpain?",
"src": "Patient: I have had pain in my lower stomach for sometime and had been treated of the UTI sometime back. I did stool test and it show pus in the stool. The pain is too much and the urine is deep coloured. The back of the waist is also paining. The endorscopy did not show anything,what could be the problem. Jay P Doctor: Hi.The pus cells are just indicating that you had / have gastro-enteritis and the endoscopy can be absolutely normal. And if it is normal , this is a good thing that you do not have any mass or ulcer , which can precipitate worst attacks.Due to gastro Problem you can loose lots of fluid and in turn can cause cause dehydration to cause dark colored stools.Both the infections can give one the back-pain and other symptoms . A correct dose and course of an antibiotic and metronidazole can help you come out of this problem."
},
{
"id": 172175,
"tgt": "Suggest treatment for sinus infection in a child",
"src": "Patient: My 2 year old had a sinus infection that didn't respond to courses of Augmentin, after completing the second course his symptoms worsened as well as getting high fevers, and stomach aches, we took him to the ER and he was admitted for pneumonia. During his hospital stay he was on IV Rocephin and Clyndamyacin, his CRP and WBC continued to elevate despite being on the antibiotics, 2 days before discharge they put him on Zithromax and the levels came down some but were still high, he was discharged but was sent for follow up blood work the other day and his values on both of these tests still came back high. His pediatrician wants to wait an additional 2 weeks and have the labs repeated. He is doing somewhat better, he continues to have a runny nose, sneezing and the occasional complaint of a stomach ache. We are concerned that his labs have yet to improve. He was given the remainder of the Zithromax to come home on but has since completed it and is currently not on any anitibiotics. Doctor: Hi dear,I had gone through your question and understand your concern. You should perform nose and throat culture and sensitivity to antibiotics. I advice: - Wash nose with Nasoclear 4 times.-Drop nose with Cyprofloxacin drops 3 drops 3 times.- Take antihistamines: Cetrizine.-Soda gurgling.-Soda steam inhalation (1 teaspoon for 2 glasses of boiled water) during 5 min.- Multivitamins-Zincovit 5 ml for 1 monthHope it helps"
},
{
"id": 184910,
"tgt": "What is the reason for severe tooth ache?",
"src": "Patient: My daughter has just been to the dentist with tooth ache. The dentist says her tooth is loose and X-ray shows a shadow in the tooth but the dentist said there were no holes visible. She has an urgent opg tomorrow and has to see specialist dentist the day after. Any ideas what this is? It's her permanent tooth! Doctor: Thanks for your query, i have gone through your query. The pain in the tooth could be because of tooth infection or gum infection. the tooth can become loose secondary to bone loss or any cyst or benign tumor. consult a oral physician and get the OPG done to rule out the above said causes. if it is a tooth infection get it treated by root canal treatment.if it is gum infection get your teeth cleaned once.if it is a cyst or tumor you need to get it removed surgically.i hope my answer will help you.. take care"
},
{
"id": 216364,
"tgt": "Suggest treatment for pain in the leg after streches",
"src": "Patient: I am a cheerleader so I am flexible and do many different stretches and flexibility training daily, so today during a tryout I went into my right splits and heard three very loud pops and then very bad pain! I continued with my tryout which had several different jumps and kicks and with each I felt even more pain and now I am feeling constant pain on minimal movements. What could be going on? Doctor: Hi...As per your narration you had injured your muscle...there could be snapping of muscle over the bone or could be a partial tear in the muscle..Nothing to worry..If I were you...I would wait for a while and....Apply lot if ice in the injured area... frequently for every 2 hours once for 15mins...No stretching to the injured area for next 48 hours...Don't load for next couple of days..Support the injured area with compression bandage...Once movements becomes pain-free then do strengthening exercises for gluteus and then slowly and progressively load the injured area...like moving your leg from out to inside...etcHope this is helpful for you...Kindly revert back.in case you need any further help in this regard..."
},
{
"id": 150165,
"tgt": "Back pain due to injured lumbar sacral joint. Pain radiating from the buttocks to the leg. Do I have sciatica?",
"src": "Patient: Hi . I have back pain. I injured my lumbar sacral joint when I was 18. I am now 74. I now have pain which radiates down my left buttock and down my leg. I also have pain in my right groin which is always there. I had a fall some 10 years ago and I believe my lower discs fractured and have now fused. I believe I have sciatica. I am hoping that the pain in my groin is associated with this and not a hip problem. Doctor: Hi You have back pain, after injury to lumar sacral joint, when you was 18, you are now 74, you have pain which radiates down to the left buttock and leg, also have pain in rt groin, you had fall 10 years ago. Dont try to make your own diagnosis, if you have severe pain consult an orthopedic surgeon. dont neglect orthopedic problems. Avoid taking tubers, tomato, egg, chicken, and seafoods. Thank you."
},
{
"id": 35758,
"tgt": "Suggest treatment for yeast infection",
"src": "Patient: My daughter is about 10 weeks pregnant. Her tongue has a yellow tint to it She did have an infection about 3 weeks ago (yeast). Took an antibotic. but, the las week she can't get rid of the yellow tongue. She has no other problems.Thank you,Lori Doctor: Hello dear,Thank you for your contact to health care magic.I read and understand your concern. I am Dr Arun Tank answering your concern.I advice you to do Grams staining from the yellow coloured tint.If you receive report stating that you have candida infection than we can start the antifungal medication.Pregnancy is the condition of immunosuppression. So there is chance of yeast infection in it.But if you have fungal infection than you can take the fluconazole oral tablet under your doctors guidance to clear the infection.Please try to maintain the optimum sugar level. Because it is also important in maintaining the fungal infection.Please also maintain good hygiene level as it can also help in clearing infection.I will be happy to answer your further concern on bit.ly/DrArun.Thank you,Dr Arun TankInfectious diseases specialist,HCM."
},
{
"id": 41917,
"tgt": "Suggest tablets to increase sperm count",
"src": "Patient: good morning sir, i am vinay kumar my age is 32 and my sperm count is 02 millions, & i had grade 2 varicocele on left testes. 5 months before my count was 15 million with oligospermia. doctor adviced me 2 take fertyl m, spermi Q, doxypal.is this tablets can increase my sperm count plz advice me. thanku Doctor: Hi wel come to healthcaremagic.I have gone theough your question.Your sperm count is 2 million, and you are having grade 2 varicocele.The medicines you described definately helps to increase sperm count.But according to my opinion you should worry about varicocele also. Consult a urologist and should operate for varicocele after his advise.Have a good health."
},
{
"id": 11145,
"tgt": "Suggest treatment for severe hair fall",
"src": "Patient: hi I am suffering from severe hair loss,I am on niftran,and take eltroxin 50( which was recently changed from 100mmg I had recently a PCNL done for my kidney stones.I just cnat figure what is the reason, I also take afood supplement(cranpac) I am 49 years . Please help Doctor: Hi, welcome to HCM,As per your age and other medical problem hair fall is probably due to Telogen effluvium.Dont get panic, try to control your other medical problem first.Take tab biotin 5 mg once daily.Apply capixyl hair serum daily.Wash your hair with volumizing shampoo.hope this will help you.Thanks"
},
{
"id": 207003,
"tgt": "Suggest therapy for sleeping disorder",
"src": "Patient: I wake up in the morning not feeling fresh. I have a very disturbed sleep cycle. I wake up at night from sleep around 4-5 times on an average. Finally when I wake up in the morning I feel like sleeping more, I feel exhausted. But when I m out drinking with my buddies, the next morning I just feel fine. Sometimes I have sleeping bags under my eyes. But as the day progresses I start feeling fine. So at midnight, I would say I m a lot more productive. Doctor: hiI understand your query.sleep wake cycle disturbances can lead to various problem in daily routines.You have sleep fragmentation and require strict sleep hygiene management.Certain routines and lifestyle modification will help you likeFix sleep schedule and stick on it,no day time sleepExercise that makes you fatigueStress free activities and meditationsAvoid alcohol and smokingAvoid heavy meal in dinnerAvoid tea coffee soft drink at nightUse bed when you feel sleepy,read up to when you will sleepyNo talking or use mobile in bed or watching Tv or playing games while you are in bed.Calm and cool bed roomAll of this will help you to restore good sleepBesides this there are some medicines like benzodiazapine and zolpidame which are useful in treat sleep problems but before that you have to do consultation and get help.I hope i have answered your query.happy to help you further.Thank you"
},
{
"id": 209875,
"tgt": "How to handle short tempered person?",
"src": "Patient: my wife is short tempered, and myself being not so strong( could not control my wife ) I do not know how to treat her. she behaves exactly according to the definitions of how short tempered people behave, Even i could not explain, her that she is a short tempered. Please advise me and guide me Doctor: DearWe understand your concernsI went through your details. I suggest you not to worry much. Leading life with short tempered people are not so easy. You should never say, when or for what purpose, they get angry. Therefore, you cannot act according to her behavior. You being strong is also futile.You can approach her in a different way. You should make her aware that she is short tempered. Then take her to a behavioral therapist and get the therapist's suggestion. Then she shall take care of her own short temper. you do not have to worry about it.You can post a direct question to me in this website. Include every detail as much as you can. I shall prescribe some psychotherapy techniques to control your condition. I am sure that the techniques should be a success.Hope this answers your query. Available for further clarifications.Good luck."
},
{
"id": 125840,
"tgt": "Suggest alternative for Hydrocodone to treat knee pain",
"src": "Patient: Hi, my name is Lillian. I was given the name of this med by my remuotologist. The name of the med is Gralise I just found out that it is the same as Gabapentin, which i m already taking. Also I was given the name of another med: Horizant. What can you tell me about this med? I have been diagnosis with Osteo Arthritis. I ve had 3 epidermal for the back pain and i ts has helped. But after the back got better the knees began to have shooting pain, burning and stinging Constantly! This has been going on since July 2016. I am practically living on Hydrocodon and Gabapentin. I take Hydrocodon every 4-6 hour for the knee pain, it never stops but it eases enough to function. It is 7.5-325. I take the Gabapentin twice a day, 100mg. And 300mg each nite at bedtime. I m VERY CAREFUL with the latter because it s hard to wake up!!! I am looking for a medication that will ease my KNEE PAIN without having to live on the Hydrocodon. (It s a liver spoiler!!!). Doctor: Hello, You can try analgesics like tramadol. If symptoms are severe you can consider knee replacement also. Hope I have answered your query. Let me know if I can assist you further. Take care Regards, Dr Shinas Hussain, General & Family Physician"
},
{
"id": 147532,
"tgt": "My husband, who has Parkinson's disease has dark bruising on side of heel and foot. Should he see a doctor?",
"src": "Patient: My husband, who has Parkinson's disease (since 1998) has unexplained dark bruising on side of heel, less dark round his ankle and foot, but there is no tenderness. Should he see a doctor. The email address is YYYY@YYYY . Would be grateful for a reply. Doctor: Hi,Thank you for posting your query.I have noted your husband's symptoms.The bruising is not related to Parkinson's disease. He needs to be seen by a doctor to find out the cause for the same.It could be due to medications such as aspirin or blood disorders such as low platelet count, etc.It can be easily treated if the cause is found out.I hope my reply has helped you.I would be pleased to answer, if you have any follow up queries or if you require any further information.\u00a0\u00a0\u00a0\u00a0\u00a0Best wishes,Dr Sudhir Kumar MD (Internal Medicine), DM (Neurology)Senior Consultant NeurologistApollo Hospitals, Hyderabad,For DIRECT QUERY to me: http://bit.ly/Dr-Sudhir-kumar My blog: http://bestneurodoctor.blogspot.com/"
},
{
"id": 7789,
"tgt": "How can I stop getting pimples whenever I masturbate ?",
"src": "Patient: hi i am 23 male. hwen ever i matrubate i get pimple on my face . how to stop pimple. Doctor: hi there is no clear relation between this two. dont masturbate for 15 days still u will have pimple. its due to hormonal effect. apply clear gel twice a day dont take muck stress it will increase acne"
},
{
"id": 60987,
"tgt": "What does a red bump on an injection site indicate?",
"src": "Patient: Hi, I donated plasma for the 5th in little over 3 weeks. I get blood drawn from the same arm each time. The last couple of times I have only had an injection mark which healed. I never had a problem with bruising. The healing process was pretty quick.This time a small pea sized red bump/ knot has formed at the injection site about 2 days post of donating. It has been 4-5 days since donating and the little sucker is still there. The bump feels a little warm at times but I would have to say it is more of an itch to want to scratch around the area. I did have work the next day as a dishwasher which can involve mild lifting but knowing me I probably lifted too much. I\u2019m not sure though. What are your thoughts? Doctor: Hello,This is thrombophlebitis after insertion of vein for the plasma donation.Guidelines for better outcome are:- Ice application 3 to 4 times a day- In our clinic, we recommend anti-inflammatory drugs along with antibiotics as per the clinical evaluation- Local application of ointment with hyaluronic acid will dissolve it fasterHope I have answered your query. Let me know if I can assist you further.Regards,Dr. Bhagyesh V. Patel"
},
{
"id": 196200,
"tgt": "What causes the inability to retract the foreskin?",
"src": "Patient: I am 40 years old, in very good health with no real medical problems. All my life, I have never been able to pull my foreskin back to expose the glans. I have always been self concious of it - knowing that it is not normal. I have had an ok sex life so far- but have not known any different so I wonder if this problem is detracting from pleasure during sexual intercourse. Also my current partner, although understanding - finds it a little odd. I would like to get something done so I can be \"normal\" but not sure what my options are. Have always been too ashamed to show my doctor - please help!! Doctor: Good day and thank you for being with healthcare magic!Choices are circumcision or frenuloplasty. This would enable to open and expose head of the penis."
},
{
"id": 90903,
"tgt": "What causes squishy ball in the belly button?",
"src": "Patient: Hello Doc, I am 33 year old male. I just recently noticed a squishy ball in my belly button toward the top of the button. It does not hurt real bad but is very uncomfortable. The ball squishes in and out and I feel a squishy feeling going in and out of my stomache. The ball is about the size of a nickle. Half of the ball is in the belly button, the other half is above the belly button. The ball is not protruded outward just a little from the top. The pain I feel is like if I were to stick my finger in my belly button and keep on playing with it.. That tickly anoying feeling. I do exercise now and then. I thought it may be ahernia but there does not seem to be major pain. I am thinking it may be a tumor or cancer or something...Can this simply be my belly button??? Doctor: Hi.Thanks for your query and an elucidate history.From your very good presentation I can tell you this is called a ''para-umbilical hernia''. IT is a small surgery to correct this as it is small one. It may not progress further as the fat-pad gets stuck inside and does not allow its outgrowth. It is upto you - if you kike to be tension-free get this operated to close the defect. Untill then you can wait and watch ( this is a medical terminology). Get operated if there is a single sign of obstruction anytime."
},
{
"id": 103961,
"tgt": "Swollen lips, rashes, bumps on hands. Reason?",
"src": "Patient: Hi, I have AEC count of 850 and as per doctors prescription we are taking medication since 3 days, but today when i woke up i see my lips swollen and lot of red rashes and little bumps on my hands and body, hesitating to go to office. Please advise if this is a cause of concern or will it goes off. We are scared that this should not result into unexpected situation, is having AEC around 850 is threatneing one, does it result in anyting else. Thanks in advance. Doctor: Hi, Thanks for writing your query. AEC count indicates Absolute Eosinophils Count. Eosinophils are type of cells which increase in allergic phenomenon in the body. They are also increased in presence of worms in the body. Normal value should be less than 350 cells/microL. The symptoms you have mentioned are due to allergic reaction and needs urgent medical attention. Antihistaminic medicines are needed immediately and if clinical examination suggests, steroids in injectable form can be needed. I would suggest you that you should go to the Emergency Department and get yourself clinically examined. I hope this is helpful to you. Thanks."
},
{
"id": 61580,
"tgt": "What causes a lump between testicles and rectum?",
"src": "Patient: My husband has a lump the size of a golf ball in between his testicles and rectum. It has no color,and seems to be under the skin. He says it s very painful. He said he had it about a month ago and it went away and now its retuned and he says the pain is worried. Please advise. Doctor: Hi,There are mainly 2 possibilities. A peri anal abscess (connected to the rectum or anus) or could be pariurethral abscess (connected to urethra). Or sometime could be just an abscess arising from skin or subcutaneous tissue. Both case requires initial coarse of antibiotics and if not responding would require a formal incision and drainage procedure. The complication with Peri anal abscess following spontaneous rupture or drainage procedure is a formation of a Peri anal fistula which requires further surgical care. And a special precaution which you need to take with peri urethral abscess is to drain abscess while on a catheter to prevent iatrogenic urethral injury. Hope this helps. Let me know if I can assist you further.Regards,Dr. Safarulla SH"
},
{
"id": 181302,
"tgt": "What causes soreness in the gum along with throbbing pain?",
"src": "Patient: My gum is very sore, throbbing with pain behind the last upper tooth on the right side of my mouth. I can't see it but it feels like a hole. I thought I scratched my gum but now I don't think so. There is also a small little knot at the back of my throat on that side and my ear is hurting and it's hard to swallow. Not sure what it is. Doctor: Hi..Welcome to HCM..As per your complain there is inflammation of gum flap around your last molar tooth that seems to be probably wisdom tooth and the gum flap around it is inflamed leading to PERICORONITIS..Pain in ear and throat and ear is radiating pain from the site of inflammation..It is causing inflammation in throat and it is leading to painful swallowing..I would suggest you to consult an Oral Physician and get evaluated and a thorough clinical evaluation and investigation like x ray can help in diagnosis and treatment can be done accordingly..In case if the wisdom tooth is impacted and will not completely erupt due to lack of space or obstruction with jaw bone then in that case removal of the wisdom tooth is the permanent solution..In case if wisdom tooth is not impacted then the gum flap will be removed by a minor surgical procedure to relieve the symptoms and will be followed by antibiotics and painkillers..As of now you should start doing warm saline gargles..Take anti-inflammatory painkillers like Ibuprofen..Gargle with antiseptic mouthwash..You should avoid any food accumulation in that area that can further flare up the infection..Hope this helps..Regards."
},
{
"id": 189546,
"tgt": "Have puss pocket at gum line that bleeds slightly. Had crown put on teeth. Related?",
"src": "Patient: About 3 months ago I had a root canal and then a few weeks later a temporary and then finally a crown put on one of my bottom back teeth . There is a little puss pocket at the gum line that bleeds slightly. I was at my detist last week and he took a new xray . There was nothing on the xray that he was concerned about but said we would keep an eye on the gums. Would an xray show if the tooth under the crown is infected? Or is this an issue with my gums? Doctor: Hi, Thanks for asking the query, After successful root canal treatment th etooth is completely devitalised, disinfected and sealed to the apex , so there are no chances of pain. Sometimes due to food lodgement between the tooth and the crown surface there is accumulation od bacteria , plague, calculus and other irritants in the pocket leading to infection and abscess formation. As the x-ray is normal there is no problem with the root canal treatment. I would suggest you to go for deep scaling and polishing followed by curettage. Maintain a good oral hygiene, use chlorhexidine mouthwash gargles twice daily. Hope this helps out. Regards.."
},
{
"id": 49240,
"tgt": "Will stones in kidney cause urine to be red in colour?",
"src": "Patient: Hi Doctor,My sister is 19 years old. She had the problem of Kidney stones one year back and she had got this treatment of Flush theraphy.3 days back she found that urine is red in colour. The microscopic examination of her urine analysis has pus cells 6-9/ hpf and epithelial cells 9-10 hpf.Is this because of kidney stones or because of infection? Doctor: Hello sir what u asking about the red coloured urine It may be due to either infection alone or stone with secondary infection so better get an ultrasound of abdomen for renal stones You did not specify if urine is homogenous red or at the end of micturition slightly red or initial flow was red as all these are significantanyways it's better to show it to your doctor"
},
{
"id": 168907,
"tgt": "What causes memory loss?",
"src": "Patient: my daughter is 7 years old and recently informed me that for a year now she has experienced episodes of temporary memory loss. during these episodes she can not recall where she is or why. It is happening quite frequently, maybe once a day now. what is happening to my child? Doctor: Get an EEG done and rule out absence seizures and if it's normal then forget about it at all as she will get over it soon."
},
{
"id": 174400,
"tgt": "What is the cause for itchy bumps at the injection site?",
"src": "Patient: My son has a bunch of itchy bumps at the injection site from his flu shot that he got on Thursday. Should I be concerned? He isn t having trouble breathing nor is he allergic to eggs. He is just itchy. I put some hydrocortisone cream on bumps. Just concerned. Doctor: greetings,doesn't look like something that you should be concerned about.But be watchful,incase it increases better to consult a doctor.thanks and regards."
},
{
"id": 169832,
"tgt": "Is it safe to sleep after a bee sting?",
"src": "Patient: My almost 3 year old daughter was stung by a bee. I know that I am allergic, but I am not noticing any reaction in her. I gave her some allergy med and now she would like to take a nap is this safe or should I wait for an hour to see if a sudden reaction occurs? Doctor: Hello, depends on the allergy medicine you administered. May be side effect of thatDisclaimer\"This provisional advice provided by me stands subject to the patient undergoing a physical examination and is based entirely on inputs provided by patient/attendants. The patient is advised for physical examination at earliest\""
},
{
"id": 138275,
"tgt": "What causes pain on hip and joints that goes down to leg?",
"src": "Patient: I can not sleep or lye down on my left side My hip and joint by the last lumbar feels inflamed. I can t walk without sharp stabbing pain going down my leg. I have tried Ice and heat with Stop pain Ointment. I don t know what going on with this. I was just at a Chiropractor for an adjustment and he told me my lengths were not off by far, This was yesterday and today I can hardly walk at all. I m in a lot of pain and don t know what to do or try. Should I go to the ER? Can this be something serious? Doctor: Hi.You should visit the ER. As this could be pain of disc origin that is radiating to the leg. These need special care and investigation like an MRI."
},
{
"id": 208929,
"tgt": "Can alcohol or chemo trigger bipolar tendencies?",
"src": "Patient: My ex-fiance went on a binge drunk all day, and turned into someone else. He's survived esophageal cancer w/chemo. He's turned into a monster, threatening to hit me so I left, but I do care about him and I'm very worried for him-he's gone to Las Vegas and gambled a huge amount of money away...we don't talk much anymore but I worry about him. Is it possible that the chemo triggered the bipolar or whatever this is...and alcohol just pushes him over the edge? Thank you very much Doctor: Which chemo is given that is not known so cant predict about its effects.However normally chemo don't worsen bipolar. But yes alcohol of course can worsen bipolar features. You are not aware about his current status of treatment. Another part to be evaluated that is he taking mood stabilizer medicines for bipolar disorder or not. If not He is not in control without medicines also which is another reason more than that you describe."
},
{
"id": 29020,
"tgt": "How to treat broken toe nail oozing green discharge?",
"src": "Patient: I badly broke the toenail on the same leg as my 7 week old hip pinning. No red streaks, or other discoloration, no pain or swelling, but some light green discharge. After soaking in epsom salt it was soft enough to cut most of the nail away. I ve since been soaking daily with epsom salt, drying with paper towels and covering with a clean white cotton sock & usually a fabric bandaid. I was already on Amocycillin for a respiritory problem. It s been 8 days and it s very lightly still draining thin watery green puss. Is there anything else I can or should not be doing? I m worried about the new hip hardware getting contaminated. I m a 32 year hemodialysis patient, also with a 25 y/o aortic prosthesis(Saint Jude valve). Doctor: Hello,This type of discharge from your toenail is indicative of an infection. You are doing the correct thing in keeping the nail clean and soaking it. However, you need more serious antibiotics than Amoxicillin to treat this infection. Since you already have a prosthetic valve and are on dialysis it is much more important for you to eliminate infections than the average person. There could be a much deeper infection than you can see grossly with your own eyes. You need to have your toe evaluated by a doctor. He will most likely put you on a much stronger antibiotic than Amoxicillin to treat this infection.Hope I have answered your query. Let me know if I can assist you further.Regards, Dr. Alexander Underwood"
},
{
"id": 179649,
"tgt": "Suggest medication for vomiting & loose motions",
"src": "Patient: Sir my 1 & half year son suffering from the problem of vomiting when I give him milk he vomit and now he started loose motion and before 3 days he had fever now I give him zinc syrup + ors and ondem and crocin in little amount is it fine can I give all these medicine together or I should give metrogyl in place of zinc please advise me. Doctor: HIWell come to HCMMost of the diarrhea and vomiting in pediatric age group is self limiting and hydration is the matter no need to give any medicine, if even after a week condition does not improved then this can be brought to test, hope this information helps."
},
{
"id": 82220,
"tgt": "Why my chest hurts while coughing?",
"src": "Patient: Hi, I have been sick for the past couple of days, but today is the worst. I now am coughing and feels like it s coming from my chest and it hurts. I just drank some room temperature water and when I swallowed, it really hurt my chest. I have never had this happen before. Not sure what this means. Please let me know! Doctor: Thanks for your question on HCM.In my opinion you are having upper respiratory tract infection (URTI). So better to start this treatment. 1. Avoid oily and spicy food.2. Keep yourself hydrated.3. Start macrolude group of antibiotic. 4. Start antihistamine drugs.5. Start anti inflammatory drugs.6. Warm water gargles 5-6 times daily.Take these for 5 days. If not improve than get done chest x ray to rule out Lower Respiratory Tract Infection (LRTI) and bronchitis."
},
{
"id": 3379,
"tgt": "What causes difficulty in getting pregnant after having Depo shot?",
"src": "Patient: Hi, My name is Kelly. I was just wondering if anyone could help me understand why it s taking so long to get pregnant after the depo shot. I ve been off of it for 9 months now and I have recently seen dark brown spotting but only once and it hasn t come back. I was on the shot for about 4 years. My husband and I would really love to have a kid. Please helps. Tia (: Doctor: Hello, and I hope I can help you today.Unfortunately, Depo-Provera is a very long-lasting contraceptive. For convenience sake, having to only worry about birth control every three months is extremely helpful. However, in some women, normal ovulation can take many months to come back some even up to one year. The reason for that is that the hormone that is in Depo-Provera is designed to be long-lasting and different people metabolize the drug differently. Also, the fact that you had been on Depo for four years and likely you did not get your period during that time is a sign that your estrogen levels also had been suppressed by the medication.If at one year since stopping the Depo-Provera you do not have any sign of real menstruation, I would recommend that you and your partner see a gynecologist for evaluation. They could do an analysis of your hormones, and medication to help stimulate you to ovulate again may be a possibility.So I know it is difficult, please try to be patient if at the end of three months you still have not resumed normal menstrual cycle, I would recommend evaluation by a gynecologist or endocrinologist to evaluate why you are not menstruating. It may be recommended also for your partner to do a semen analysis, as most doctors want to know that there is no male factor involved in your fertility issues.I hope I was able to adequately answer your question today, and that this information was helpful.Best wishes,Dr. Brown"
},
{
"id": 187059,
"tgt": "What causes nausea with red swollen grains on the back of the tongue with white coating?",
"src": "Patient: i got tested for HIV about 4 days ago it was negative i went because i started feeling dizzy with nausea now i have white coating on the back of my tongue and red swollen grains on back also my tongue is swollen i feel a gag reflex im very scared also i get a tingly sensation on my feet Doctor: hello thanks for consulting at hcm..u could have oral thrush,a fungal infection causing white coating,,,also papillitis of tongue at the back of tongue,,clean ur tongue with asoft brush,, and could take topical antifungals..with the prescription of u r oral medicine/oral surgeon ..take acourse of multivitamins..do salt water gargle,,either ways consult a oral medicine/oral surgeon specialist..hope it helps,,tc"
},
{
"id": 83515,
"tgt": "Is it safe to take solonex during pregnancy?",
"src": "Patient: Hi, I am 32 years old having 3 yrs old daughter and from last 3 months i am taking medicine solonex. Doctor advised me to take medicine for 6 months. I want to know that can take this medicine during the pregnancy? Right now i am pregnant and 2nd month is running. Doctor: Hi, Yes, it is safe during pregnancy. Solonex is an antibiotic commonly used to prevent or treat tuberculosis. There is no documented evidence of it causing any harmful effects during pregnancy hence it may be taken as prescribed. Hope I have answered your query. Let me know if I can assist you further. Take care Regards, Dr. Mohammed Taher Ali"
},
{
"id": 134937,
"tgt": "Suggest remedy for tingling sensation in toes",
"src": "Patient: Hi, I had a knee placement just over 3 months ago and it s coming along quite well. In the last couple of days if I move it the wrong way or lie down i have excruciating pain on the side of my knee like a pinched nerve but I can feel it tingling in my toes. Cheers, Kylie. Doctor: hi kylienothing to worry about it happens a few months post op due to nerve stretching knee exercises can helpthanks Take care"
},
{
"id": 12106,
"tgt": "Do PHOTODERMA SPF30 a very effective remedy for brown sport and skin lightning ?",
"src": "Patient: Do PHOTODERMA SPF30 a very effective remedy for removal of brown spots caused due to acne and sun burn. if yes, please tell me the direction of its use,its effectiveness ,in how many days it will start showing results,how many months of its usage help me to get rid of these brown spots and also does it helps in skin face color toning and lightning. Doctor: hi it depend on the spot and its depth u need to consult dermatologist for this photoderm has sun protective and moisturizing effect it has little effect on brown spot so dont try such product without advise better and safer treatment are there in dermatologist's tool"
},
{
"id": 194890,
"tgt": "What is the cause for patch near the butt and itchy red area near scrotum?",
"src": "Patient: I have a very sensitive patch (burning itch) @ the base of my spine, top of butt crack. There is a rough \"basketball skin\" patch running down inside my butt cheek. Also between my thigh & scrotum & above the penis in the pubic hair is very red & itchy.Blue star ointment gives the only releif but burns terribly.Have tried otc antifungals, benadryl, neosporin, hydrocortizone...you name it, at wits end. Dr. said yeast infection use otc antifungal creme, already know that dont help. In agony please help. Doctor: Hi, It might be due to fungal infections. As the first line of management, you can apply topical antifungal like Clotrimazole after cleaning with soap and water. If symptoms persist better to consult a dermatologist and get evaluated. Hope I have answered your query. Let me know if I can assist you further. Regards, Dr. Shinas Hussain, General & Family Physician"
},
{
"id": 134264,
"tgt": "What causes pain in inner thigh?",
"src": "Patient: Hi. I m having left inner thigh pain for about 3 weeks. I was put on antibiotics because it was thought to be due to sensitive lymph nodes, but it did not get better. It is not related to any exercise or activity\u2026I didn t hurt myself. It just started on its own. I m 20 years old. Otherwise healthy. The pain is just getting worse. No lower leg or foot pain. No back pain. Doctor: hi,thanks for the brief history of you.since you mentioned the pain in inner thigh and ruled out all spinal and other issue I want to ask if you have any trouble passing urine?it could be a urinary infection as well. which can be assessed by a simple blood and urine test.also to mention that certain times we really do not come close to the mechanism of injury like how this pain has all started. because all pain builts up over a period of time and as how banks give Interest on Fixed deposit the body gives it all back in the forms of symptoms.kindly have a meet to a physician get yourself checked once and later on using a hot water fermentation or icing will help you lessen the pain.post that some simple inner thigh muscle exercises can be guided by a physical therapist.all the experts does perform well to help the patient to the fullest and taking their advice is never a mere waste.with the grace of God I wish you a speedy recovery."
},
{
"id": 76525,
"tgt": "Suggest medication for cough",
"src": "Patient: 43 years 5.8 weight 65 kg one years ago light cough about seven doctors during this peried he gave a often sterrized medicines now i am patient of dr aslam in mh hospital rawalpindi he advise differint test including x r pns x ray chest esr resiult o7 fungus test fungs smer /koh prep result absoulate eosinphils 1450 result serum ig result 479iu/ml from agha khan hospital karachi no he advising me on st scen on hr what should i do now Doctor: Thanks for your question on Healthcare Magic. I can understand your concern. You are having eosinophilia (high eosinophil count) with positive fungal report. So possibility of ABPA (allergic bronchopulmonary aspergillosis) is more in your case. CT scan of thorax is needed for the diagnosis of ABPA. This is the reason why CT scan is advised by your doctor. You will need antifungal (itraconazole), anti allergic and anti histamine drugs. Inhaled corticosteroid (fluticasone or budesonide) and inhaled bronchodilator (formoterol or salmeterol) are also needed. Don't worry, you will be alright with all these. Hope I have solved your query. I will be happy to help you further. Wish you good health. Thanks."
},
{
"id": 45808,
"tgt": "Are chest pain and kidney stones be related?",
"src": "Patient: Hello Sir, I am very unhappy, because I have two kidney stones in the left and right kidney. In the left kidney there is 6mm size of calculli in the midle calyx. Yet Sir, In the Right kidney there is 12mm size of calcully in the Lower calyx. Actually Sir, I have found pain in the area of abdoment and upper chest of right side. Firstly I want take out the kidney stone from right kidney. problems in here, Please help me Sir , How to removed stone in this condition. Write me the nessecery process, God bless you sir. Doctor: Hello, You can consult a urologist and get the stones removed. You can go for ESWL(Extra Corporal Short Wave Lithotripsy) where the stones are removed using high frequency sound wave with out making an incision. Sometimes pain related or kidney stones may radiate upwards due to common nerve supply. As of now you can take analgesics like paracetamol or Ibuprofen. Hope I have answered your query. Let me know if I can assist you further."
},
{
"id": 202200,
"tgt": "Suggest treatment for watery semen and to improve health",
"src": "Patient: My semen are like water and m very thin boy. i want to make good health and thick semen Sir, I am dev sharma 30 years old boy from Delhi. I have a very critical problem. My semen are like water it is not white it is transparent like water and always come out even i think about sex and it is embarrassing situation some time when i am among people and if i erect my semen will come out like water and it is shameful. plz help me it is a major problem m facing and due to this my health is also affecting am very thin weight 50kg and height 5feet 6inch plz guid me..... i want good health and solve this watery semen problem. plz plz. help Doctor: HelloThanks for your query,based on the facts that you have posted it appears that you have mucus discharge from .Please note that it not a semen but mucus secreted by glands located in urethra.This is due to stimulation of mucus glands in urethra which get stimulated by sexual thoughts producing more precum..There is nothing to be worried about it.,It will get resolved off its own without any treatment.Following measure will help you to boost up your confidence and help to improve your general health.1) Practice regular exercise for 45 minutes followed by meditation for 1/2 an hour in the morning.2) Take high protein diet rich in vegetables and fruits and Vitamin A,C,D,E.and Zinc3)Take anti oxidants like Almonds 5-6 everyday..4) Avoid alcohol and smoking..Dr.Patil."
},
{
"id": 189139,
"tgt": "Have undergone rct. Swollen around the gum, pain and heaviness around the affected area. Medical cure for this?",
"src": "Patient: i have undergone rct in one tooth but due to repeated infection in the tooth, the tooth was extracted. Now after the extraction of the tooth there is mild swelling around the gum and down, little pain and heaviness around the affected area. 10 days have gone after extraction of tooth. Can u tell me what are these symptoms and treatment Doctor: Hi, Thanks for asking the query, Extraction of the tooth is often associated with post operative complications like pain and swelling. Dont worry about the symptoms as they will subside after sometimes. Follow the post operative instruction properly. According to your clinical history i suppose that the infection in your tooth was deep after extraction a curettage and rinsing of socket with antiseptic saline rinses was required. I would suggest you to take a course of antibiotics and analgesics for 3-5 days. If the symptoms persists or worsen visit the Dentist and get the checkup done. Maintain a good oral hygiene. Hope this helps out. Regards...."
},
{
"id": 21119,
"tgt": "What could cause prickly hands,feet,vibration sensitivity in a pacemaker patient?",
"src": "Patient: i had a pacemaker put in april last year, heart had slowed to 20bpm. i am on 10mg lisenopril and 3.125 mg 2day of carvedilol. since september , I have been experiencing prickly hands and feet and vibration sensitivity. I usually weight 152=154 have gained substantial now weight 175. I went got pacemaker tested, I thought maybe a lead was undone. They said everything fine. is this serious, or just something to get used to, I wasn t having this problem before september, I used a pr hairclippers to cut my sons hair, and it seems to have stayed. If you have any suggestions it would be helpful. Doctor: Dear, your problems seem unlikely related to pacemaker!This is more of symptoms related to neuropathy that means weakness of the nerves. This could be due to multiple reasons ranging from dietary deficiencies of micro or macro nutrients to structural damage of the nerves.I would advice you to consult with your physician and get the basic blood investigations done like Blood sugar fasting and PP, vit B12 and folate levels. And accordingly may need vitamin supplements and other nerve strengthening agents."
},
{
"id": 39465,
"tgt": "What is the medicine for ring worm infection?",
"src": "Patient: My husband is under physical therapy after a knee replacement, he is a diabetic, and the therapist said she thinks he has ringworm and it is spreading. One of the answers on your website said to apply deluted bleach - how much bleach to how much water? We are trying to get an appointment with our doctor to confirm that it is ringworm and also what they suggest to do for it. We realize it is contagious. Doctor: First of all you have to confirm it is ring worm because unless we have proper diagnosis there no any use of unnecessary treatment.So my opinion is that take appointment of doctor on priority basis & confirm the thing is ringworm.If it is ringworm then it is transferred to other person only by intimate contact & for getting infected patient must be susceptible. It is not transferred by air born route & if proper hygienic precautions are taken.If the ring worm is confirmed only topical therapy ( local application ) will not be sufficient & you should have to take oral anti fungal such as Terbinafine plus local anti fungal application such as itraconazole. ( for detail therapy consult your dermatologist) My advice is that do not use bleach as it may not be helpful in ringworm infestation instead use one of the above said anti fungal cream."
},
{
"id": 89339,
"tgt": "What cause abdominal pain, sweating and pounding heartbeat?",
"src": "Patient: I just got a stomachache like I was going to have diarrhea. While going to the bathroom I started feeling weird - the roof of my mouth got tingly, heart was pounding, started sweating and feeling a little gray. Got thru that a few minutes ago. Still feel a little weird. Roof of my mouth feels a little numb and tingly, and the back of my throat. Doctor: HI.This is a severe form of Gastroenteritis wherein one looses all he important water and electrolytes in the dead-space or also called as third space - here in the intestines . This is drained out of the body blood hence all the symptoms of tingly mouth pounding heart, sweating and all. Rush to ER to get electrolyte and water imbalance treated, investigations done and to get appropriate antibiotics and supportive therapy."
},
{
"id": 201152,
"tgt": "What causes fishy smelling semen?",
"src": "Patient: My husband s semen smells fishy. After we have sex I seem to start having the same smell. What could this be caused by. He HAS NOT had a vasectomy. I am very concerned about this. Paranoid that he s been with someone else. The only real change he s had is that he has to take prednisone. Doctor: Hello,.Odor may also come from vaginal secretions during sex. There is no need to be paranoid or suspicious. The main reason for this is infection specially candidiasis, which affect both partner. In that case both partner will need treatment with antibiotics after evaluation.Take care,www.99doctor.com"
},
{
"id": 43121,
"tgt": "Taken IUI treatment after which I had severe cramps. Should I take Duphaston to strengthen the Pregnancy?",
"src": "Patient: hi there...a very good day to you. I have a quiestion about IUI treament.I just had it on 05/12/13(thursday) and have severe cramping symptoms since that..nut now had been reduce a little.. what im wondering the doctor did not give me any tablets such as duphaston(10mg) for strengthen my pregnancy stays...and so i did call up and ask him wheteher can i consume the tablets and HE says YES...But right now i've read and herad from some other source this duphaston might or might not for strengthen the pregnancy... doctor please advise me whether i should take it or not)p/s : (this will be my 5th IUI treatment and i have ovarian tube block on right, and irregular periods) Doctor: You may take Tab Drotin tds for the cramps, buy consult your obgn dr. for further management, he can better tell than me, bcs he has examined you clinically."
},
{
"id": 149238,
"tgt": "Is cerebral atrophy curable? Is life span restricted?",
"src": "Patient: Thanks for replying. I am very much concerned about the disease. So far I have collected datas from internet about the disease it is evident that cerebral atrophy is a progressive disease, there is no cure for the disease, and the life span of the patient is restricted. is it true? i am enough strong to hear the truth. pls tell me the truth. Doctor: Dear friend,Thanks for posting your query on this website. I fully appreciate your concerns.I agree with you that cerebral atrophy is incurable. Degenerated brain tissue does NOT regenerate. Besides, the condition is gradually progressive but its complications are the cause of curtailed longevity. I am sure that you will benefit from the advice.Please feel free to revert back to me in case of further clarifications, if any.Have a good health.Fond Regards,Dr.(Col) R KaranwalM.D. (Internal Medicine)PLEASE NOTE :- My opinion/advice is based SOLELY on the details provided by you."
},
{
"id": 204537,
"tgt": "Suggest treatment for hallucinations when diagnosed with schizophrenia",
"src": "Patient: My niece suffer from ezquizoprenia and she has not been taking the medication for a couple of months. She is 37 years old and she is having allucinations right now. She has a 7 year old girl and I am nervous about her condition. What can I do? She refuse to take the medication Doctor: Hello and Welcome to \u2018Ask A Doctor\u2019 service. I have reviewed your query and here is my advice. I can understand the problem you are facing. She is hearing voices and may harm herself or anyone else so be vigilant, keep away things which can harm. You have to understand she have no touch with the reality and so she is avoiding medication once she gets better she will start taking medications. She can be started on long acting injections which are to be given once a month. There are liquid medication preparation available in the market which is colorless, odorless, and tasteless which can be mixed in water or juices easily. Moreover there are various medication available which melts in mouth. Hope I have answered your query. Let me know if I can assist you further. Regards, Dr. Rohit Kothari"
},
{
"id": 34938,
"tgt": "How does meningitis attack and spread?",
"src": "Patient: What if the most common victums to prey opon for meningitis? How does meningitis attack and spread? is meningitis armed and dangerous? what is the degree rate of damage caused by Meningitis? What is the most commoun injury done to the victum of Meningitus? can u anwer any of thses Doctor: Hello dear,Thank you for your contact to health care magic.I read and understand your concern. I am Dr Arun Tank answering your concern.Meningitis is the inflammation of the meninges covering of the brain.There are many cause of it may be viral, bacteria, fungal and parasitic. Investigation can reveal the further etiological agents.Various etiological agent has various route of spread but the most common spreading method is via blood circulation.Meningitis is very dangerous and can cause death if seriously not treated. It can lead to unconsciousness to the infected person makes him eligible for ICU admission.You can assume that if the person is unconscious that the infection is very serious.Many of the person can recover from the infection. But some may not be recovered may lead to paralysis of the victim.But all in one, if the person is diagnosed with meningitis than immediate hospitalisation with the proper and good treatment can cure the victim.I will be happy to answer your further concern on bit.ly/DrArun.Thank you,Dr Arun TankInfectious diseases specialist,HCM."
},
{
"id": 93752,
"tgt": "Constant stomach bloating and pain due to Intestinal contractions. Taking probiotics, vegan diet. Suggestions?",
"src": "Patient: I have constant stomach bloating and pain due to Intestional contractions. I have had pretty much every test except for a stomach emptying test and gallbladder function test. I have been taking probiotics (Align) for the past 8 weeks and experienced no improvement so far. I try and stick to a gluten free and vegan diet to reduce flare ups and bloating/swelling. I also exercise about 12 hours a week. Is there anything else I can try to eliminate the constant abdominal bloating and swelling I experience. I am beyond frustrated, thank you. Doctor: Hi, Thanks for posting your query. With the available describes symptoms, there appears to be high possibility of decreased gastrointestinal movements. Presence of bloating with intestinal pain suggests the diagnosis. Intestinal contraction could be due to intestinal obstruction or due to impaction of fecoliths. You should consult with internal medicine specialist/ general surgeon and should go for thorough check up. You should also go for complete blood count, blood sugar, serum electrolytes, and thyroid profile. You should also go for ultrasound imaging and x ray abdomen. You should take plenty of water, high fibre diet and laxatives containing lactitol and isapgula. You should perform increased physical activity to maintain gastrointestinal mobility. You should also go for endoscopy for better clarification of upper gastrointestinal tract. You should also take proton pump inhibitors along with prokinetics. Take care, Dr. Mayank Bhargava"
},
{
"id": 39148,
"tgt": "Can novaclox be given for mild infection and swelling?",
"src": "Patient: My mother aged 82 has a mild infection and swelling behind the ear.The Dr prescribed cepodoxime xp 325 twice a day for 7 days.After the first two tablets she has ben very nauseous and uncomfortable.Earlier I have seen once the vomiting starts she becomes weak for a week or more.She has tolerated Novaclox in the past well.Can the medicine be switched? Doctor: Hello,Welcome to HCM,As your mother is having some infection with swelling behind the ears, which may be due to infection of the external auditory canal. As your doctor has prescribed Cepodoxime xp 325 mg, twice a day for 7 days. But she has not able to tolerate this medicine.As she can tolerate Tab Novaclox, it can be given as it also covers wide range of microorganisms. This is also an excellent drug in case of infection of the ear and the skin. Before giving this drug, give her Tab Rantac as it can prevent acute gastritis.Thank you."
},
{
"id": 20623,
"tgt": "Suggest treatment for high BP and tingling in head and legs",
"src": "Patient: Hi. My blood pressure has been high for 2 weeks. At first the Dr. told me to follow a low sodium diet and decrease my caffeine intake, I have done both and went back to the Dr. yesterday. It was still reading 153 over 99. I have been on atentolol since Saturday 50 mg once a day, and yesterday they increased it to 100 mg a day. I do now have the omron BP machine, and my readings are still about the same, When will my BP start going down? I am not over weight and am pretty much a healthy 36 year old female. Whenever it is elevated I also feel tingling in my head and my legs. Is this Normal? Thank you! Doctor: Hello!Welcome and thank you for asking on HCM! Regarding your concern, I would explain that it is normal to feel tingling in your head and legs while your blood pressure values are high. From the other hand, you should know that every change in antihypertensive therapy needs some days to achieve a new balance in blood pressure values. So, you need to be patient and wait some days for your blood pressure values to lower. If after a week, your blood pressure values are still high, you should discuss with your doctor on the possibility of making further changes to your therapy (add new drugs). Hope you will find this answer helpful!Wishing good health, Dr. Iliri"
},
{
"id": 58069,
"tgt": "Painful back and jelly like secretion in urine. Have undergone gall bladder and appendix removal. What does it indicate?",
"src": "Patient: Hi, my mom at the beginning of october had her gal bladder taken out. About four weeks ago she started having tarry stools with blood in them. Along with that she was passing this jelly substance that ranged from brownish in color to clear whenever she urinates. The bloody stools have stopped for a couple of weeks now but the gel has persisted. Last Wednesday she had her appendix taken out. The hospital refused to run a test on the gel and just took her appendix out. Also throughout this whole time she has been having a very painful back pain that she describes as a knee in her back. With each surgery it has moved a little and will decreased for a short period of time after the surgeries but then comes back in varying pain levels ranging from like 4 to 7 on the pain scale. Thanks. Doctor: Hello there! Thank you for askingpainful back with gealtinous urine indicates UTI and is explained by recent interventions and surgical history. ALos surgery does this too, So a simple urine routine exam would be a good start.Mean while use ciprofloxacin 500 mg with hygiene therapies, cranberry juice, garlic uses, Sitz baths , Saline washes, And anti spasmodics like phloroglucinols etc for the relief after getting prescribed with. And follow it with an ultrasound abdomen and pelvis to work up furtherGood LuckS Khan"
},
{
"id": 27477,
"tgt": "What causes high BPM rate during nights?",
"src": "Patient: For the past 4 nights i woke up around the same time with a rapid heart beat. I've been to the cardiologist about this. My EKG and Echo were normal. I also wore a holter but it didn't happen that night but it did show my heart beat per minute as high as 120. The doctor says there is nothing wrong and my heart is healthy but to follow up in a month if it doesn't go away. What do you think is wrong. I am 24 yrs old, 5ft 11 in and weigh 212 lbs. Doctor: Hello. Thank you for asking on HCM. I understand your concern.During sleep, there is a predominance of the parasympathetic system acting on the body. However, there is a phase - rapid eye movement phase - which is characterized by a flush of catecholamines (adrenalin) in the system, causing your heart rate to go up. You said that you tried a 24-hour Holter rhythm monitoring. My opinion is that you have nothing to worry about. However, If you continue to be worried, I would recommend you to be put on a episode rhythm monitor, which allows you to turn the machine on right at the moment when your symptoms happen. Then you can go at your physician's office and read the EKG registered at the moment of your symptoms. I hope I was helpful. Take care."
},
{
"id": 12819,
"tgt": "Suggest a remedy for rash",
"src": "Patient: My 9 week old woke up from her morning nap ate a 4 ounces then threw it up went to change her and she threw up chest and chin are covered in a red, warm rash with raised bumps not to mention when she tried to cry she barely had a voice it's extremely raspy no fever Doctor: Hello,I read carefully your query and understand your concern.The symptoms of the baby seem to be related to a viral infection.I suggest to check the body temperature regularly.Meanwhile,I suggest to use use Calamine lotion for local application at the rash area.Hope my answer was helpful.If you have further queries feel free to contact me again.Kind regards! Dr.Dorina Gurabardhi General &Family Physician"
},
{
"id": 101803,
"tgt": "Any suggestion for viral fever?",
"src": "Patient: Hi Doc,I am having fever for past 3 days,i went to a doc,he said its viral, and gave me some medicine(Paracetamol-Diclofenac, Ciprofloxacin and Omperazole), he said take it thrice. What is happening whenever i m taking these medicine,fever comes down but again after 6hrs it comes back. Can u pls. help Doctor: HIThank for asking to HCMI really appreciate your concern looking to the history given here I would like to say that if this is viral infection then no need to take the antibiotic this wont work here only symptomatic medicine would be enough, and this would take at least 4 to 5 days and no need to worry about this, hope this information helps you, take care and have a nice day."
},
{
"id": 135252,
"tgt": "How to treat the pain in upper right arm muscle?",
"src": "Patient: I fell on my arm and hurt my shoulder and upper arm I had xrays nothing s broken the doctor said it was fine she said give it time to heal if it doesn t get better to come back well at this point I can rotate my arm so I assume the rotator cuff is OK I have trouble just picking up a glass you bringing a fork to my mouth the muscle in my upper right arm aches should I be exercising that muscle or massaging it what should I be doing with it Doctor: Hello,I have studied your case and I think that you may be having rotator cuff tear. In this condition there is pain and inability to lift shoulder above head.I would suggest you to get MRI of the shoulder.That will confirm this diagnosis. If there is cuff tear and it is involving more then 50 percent thickness then you will need surgery to repair it but if it is less then that then you can be treated with rest and physiotherapy. I hope this answer will be useful for you.Let me know if there is any other followup questions,thanks"
},
{
"id": 50484,
"tgt": "Slight pain below the ribs radiating to the back, severe while running. Ultrasound shows fatty liver, bilateral kidney stones. What to do ?",
"src": "Patient: Hello sir,I have slight pain in the below the ribs which radiating to the back,actually it is not pain it is like a discomfort.the pain below the ribs gets severe when I run.I have this pain for almost one year.I took a ultrasound scan about 7 mnthys ago.the impression was fatty liver and bilateral kidney stones (a few in both the kidneys,diameter 2-3 mm).I have this discomfort for almost all the day.I am 26 years old.please advice me what to do.I am really fed-up with this.... Doctor: Hi!. Thanks for your query. I do not think that Fatty liver can give rise to your problem of discomfort under the ribs. Two stones in both kidneys are quite small, but you have not mentioned where actually they are located in kidneys. You may see a Urologist for proper advice. Take care."
},
{
"id": 185702,
"tgt": "Is Listerine Total Care mouth wash safe for administration?",
"src": "Patient: Each time I use Listerine Total Care, about an hour or so later I have white, stringy, filmy stuff in my mouth lining my gums and hanging in between my teeth. I don't know what it is but I read in a product review in which the user was complaining of a very similar side-effect that the stringy stuff was the mucous membranes peeling. Does this sound correct - mucous membranes peeling being a side-effect of using Listerine Total Care - and if so is this normal, and is the peeling harmless or ruining my mouth? Thank you for your help! Doctor: Hello dear,I have thoroughly gone through your query. No need to worry much. In my opinion that white, stringy, filmy stuff may be the thickened saliva rich in mucous(thick component of saliva). And of course mucous membranes do not shed off or peel like that.I think that the mouthwash may be causing dryness of the mouth due to which the saliva which is rich in mucous, sticks like a thin film lining over gums and in between the teeth.- You can switch over to some other mouth wash (as there are plenty of chlorhexidine mouthwashes are available).If the problem continues then visit dental surgeon for further investigations.Hope this will help youThank You Take CareRegards"
},
{
"id": 91763,
"tgt": "What medication is advisable in case of severe lower abdominal pain?",
"src": "Patient: I am on several pain meds r/t RSD & I told my pain Managent Dr. He took me of of any drugs that has Tonyl sorry for spelling. My abdomen can become inlarged sometimes, day- day, & other days it stays the same. The last 3 days I am having severe pain in my lower abdomen, on both sides back & front. Doctor: Hi. Enlarged abdomen needs investigations to see the reason.I would advise patients like this to consult a Gastro Surgeon and have minimum of X-ray of whole abdomen in standing position to see if there is any obstruction to the intestines. ( the commonest cause of distension).Withdrawal of some medicines may cause abdominal disturbance but not so much of distension.You have pain on both sides also."
},
{
"id": 124545,
"tgt": "What are the symptoms of fracture ?",
"src": "Patient: I fell two weeks ago and hurt my knee. They told me I bruised it and nothing was broke but it s still hurting and I can t stretch my leg out all the way. Also it s making a popping and cracking sound almost everytime I walk or bend my leg. Could there be something torn? Doctor: Hello, Thank you for using \u201cAsk a doctor\u201d to post your health query.I read carefully your question and I understand your concern.A fracture would make almost impossible to walk or bare weight on the leg. So if the doctors told you that there is no fracture, then most probably it is not the case.Knee injuries need more then 2 weeks to get well. You may have blood or fluids inside the knee cap so that is why you cannot bend it, and the cracking that you hear.To tell for sure if something is torn, would be if you do an MRI, which will indicate very specifically what is happening in your knee. But I would say give it some more time to heal, follow the doctors advice, do not overuse it so will allow it to heal. If not better in 4-6 weeks, or if you feel any instability, then consider seeing the doctor again so they can recommend the MRI or do another physical exam of the knee. Hope this is helpful.Let me know if you need more clarifications.Regards,\u00a0\u00a0\u00a0\u00a0\u00a0Dr. Papaqako"
},
{
"id": 12241,
"tgt": "Lumpy itching patch inside leg indicate psoriasis?",
"src": "Patient: i have a had a small spot on y lower leg about 6 weeks ago , it has now grown fast into a white dry lump patch and is itching all the time , it fells now that it is really deep inside of my leg , when i scratch it it flakes ...could this be cancer or psorisis Doctor: Be assured it is not skin cancer . it is likely to be psoriasis or dermatitis. you can apply topical steroid like Mometasone cream ( Momate)locally twice daily along with oral antihistaminic like levocetrizine 5 mg daily for 2 weeks and see the response"
},
{
"id": 217320,
"tgt": "Suggest medication for pain in leg",
"src": "Patient: My 90 year old aunt has pain in her left lower leg, and has had this pain for some time, she has been to several different Drs, along with a pain management team, she is on oxycodone 7.5 mg q6hrs and continues with pain, I think she has PAD, her feet are always cold, it s hard to obtain pedal pulses, the Dr has her on a medication for neuropathy that is not working, should she go to a vascular surgeon? Doctor: Hello, the pain can be due to neuropathy, or radicular pain or even varicose veins. U can go to a vascular surgeon to rule out varicose veins. If that is not the cause, then you should go to a pain clinician who may suggest a nerve blocking injection which might block the pain, initialky temporarily and then permanantly."
},
{
"id": 224324,
"tgt": "Is spotting normal after intake of ginet-84?",
"src": "Patient: i have just started my first pack of ginet-84, started in the red area like it says on the first day of my period which was around about on time, but a week n 2days later i started spotting, thought i had my period again but it stopped, then it started again and stopped now started again...is that normal? Doctor: Hi,Initially it may take a few weeks to regularise your cycle. Spotting may occur for the few weeks, before your hormone levels stabilise. Just keep taking it on time and dont miss."
},
{
"id": 192616,
"tgt": "What causes fishy smelling sperms?",
"src": "Patient: my boy friiends sperms smells fishy. I have been to the dr. and have been checked for stds. and results are always negative. what could cause his sperm to smell. It hasn't always been like that. prostate cancer does run in his family. could it be prostate cancer? Doctor: Hello, It is not a sign of prostate cancer. Foul smelling semen is a sign of infective causes like urethritis. You can go for a semen analysis to check for any signs of infection. If there is any signs of infection, antibiotics like ciprofloxacin may be required. Hope I have answered your query. Let me know if I can assist you further. Take care Regards, Dr. Shinas Hussain, General & Family physician"
},
{
"id": 194574,
"tgt": "What causes swelling in my left testicle?",
"src": "Patient: I have problem with my left testis. Both of my testis were of equal size but suddenly one day my left testis become swollen and have lot of pain but slowley the pain get reduced but size didn't. Sometimes i feel that size has reduced and after sometimes i feel the it has been increased not sure why is it happening? The problem persist from a long time. It would be around from last 10 years. Not sure how to approach a doctor because i am very shy kind of guy. IF you can help me thru online then it would be great. Doctor: Hi, You need to consult a urologist directly and get a direct check-up at the earliest. In torsion of testis, you may lose your testis. The second reason could be an infection. Hope I have answered your query. Let me know if I can assist you further. Regards, Dr. B. Radhakrishnan. Nair, OBGYN"
},
{
"id": 216190,
"tgt": "Suggest medication to ease the pain due to breakage of bones",
"src": "Patient: I broke my leg 3 weeks ago (broke both bones below my knee), when I fell, my foot twisted 360 degrees which is what I believe is causing severe nerve pain now. I was given Lyrica but am reluctant to start taking that medication. Yesterday, I was also prescribed a cream mix (gaba 5% / keto 4%) which I applied twice last night and 3 times today so far, however, I still have alot of pain, would you have any advice on anything else I could take to ease the pain? thanks. Denise Doctor: Dear patient how was the fracture diagnosed? Was the xray done? What other treatment is given apart from medications? Normally whenever there is fracture of both bones of leg which is to be managed conservatively Above knee cast is given. If cast is given pain should not be there. Pain at the fracture site occurs due to movements of the bone which is prevented by cast immobilisation. If you are getting pain even after cast yiu need to consult your orthopaedic surgeon at the earliest possible time. Meanwhile you can take ibuprofen 400 mg thrice a day with tab ultracet twice a day for pain relief.keep limb elevated over two pillows and avoid weight bearing on the fractured leg."
},
{
"id": 153190,
"tgt": "Suggest treatment for stage IV lung cancer",
"src": "Patient: Hi All, My Father is suffering from Lung Cancer Stage IV - Non small cell due to which the cancer cells are spread over the lungs to other parts of the body. Due to this, my Father is suffering from severe pain in body parts, weekness and start loosing weight. Please suggest us how to cure it? Doctor: Hello! I've read your concerns and feel sorry for your father. Lung Cancers commonly spreads to the bones -- which might be the cause of pain of your father. Stage IV lung cancers are treated by chemotherapy, to slow down and control the spread. Other symptoms such as pain will be relieved by analgesics. Weight loss and weakness is part and parcel of symptoms of cancer, your oncologist can give your prescription for dietary supplements."
},
{
"id": 37797,
"tgt": "How to cure enteric fever?",
"src": "Patient: Sir I also a medical practitioner and usually overlook my own health. I got Typhoid last year on 18th or 19th of July 2013 . Got cures with Ofloxacin 1g for 10 days. This time the sympots appeared on 24th july and Labs confirmed the Enteric....Is there some periodic relationship ? How to prevent it next ? Thanks and regards. Dr Abbas Raza Syed YYYY@YYYY Doctor: Hello,Welcome o HCM,As you are have had typhoid fever for last one month it may be a fever of unknown origin (FUO) which should be thoroughly evaluated to find out the causes for the fever. As she is having for last one month all the relevant investigations should be done to rule out the cause of fever.To confirm it has typhoid after having fever for last one month you should undergo stool and urine culture to confirm it. There are many conditions which can lead to fever of more than one month which should be evaluated thoroughly.Thank you."
},
{
"id": 88810,
"tgt": "What is the cause of fever, abdominal pain and diarrhea?",
"src": "Patient: Hi! I was supposed to get my period the 24th and I had intercourse the next day and there was light bright red blood on my partners penis. Almost like blood from a cut and how fresh it is. Yesterday, I started feeling really weird and took 3 pregnancy tests and they all came out negative. I took a nap and woke up and puked 4 times. All throughout yesterday I had diarrhea. Last night, I had a fever of 102 F and I ve been very tired and light headed. I m still feeling lower abdominal pain as well. It feels like a stabbing feeling. Doctor: Hi. The symptoms of fever, diarrhea getting tired light headedness and all are due to gastroenteritis. Get a proper examination done by a Surgeon and get an appropriate prescription based medicines. Take plenty of oral fluids, soft diet, curd and banana, probiotics.Bleeding after sex could have been due to the normal menstrual period as your pregnancy tests are negative. This can be enhanced by the congestion caused by gastroenteritis. The water-borne infection has to be recognized so that you can avoid this in near future by using boiled and cooled water. Consult a Doctor to see for the vitals like pulse, blood pressure, temperature , level of dehydration, abdominal palpation for clinical evaluation and prescription based treatment."
},
{
"id": 41779,
"tgt": "How can conception be achieved?",
"src": "Patient: i had a stillborn baby almost three years ago and i am worried i may not be able to concieve again as me and my partner have been trying for a baby for the past year or so but it is not happening. we are both under 30 and are generally fit. what is your advice? Doctor: Hi welcome to healthcaremagic.I have gone through your question.As you had pregnancy last year and still birth already. Then i would advise to do histosalpingography HSG to rule out tubal blockage, ovulation profile, thyroid profile.Also do semen analysis for your husband.if reports come normal then plan for baby in natural way, do intercourse in fertile period on alternate days.Do this method for 5/6 months as your age are less than 30.If not get success then consult gynecologist. Hope i answered your question. Would be happy to help you further.Take care."
},
{
"id": 92191,
"tgt": "What could be the reason for pain and visible swelling in abdomen between waist and ribs?",
"src": "Patient: back pain on right-side, lower back next to spine, severe two weeks ago, now lessening, but pain in right abdomen, and swelling midway between waist and ribs. Dr. prescribed pain pills. Had cat-scan 2 weeks ago which showed nothing. swelling is now visually apparent. Doctor: Do an ultrasonography of the affected region, and please detail your history regarding the swelling like onset, duration, any discharge etc."
},
{
"id": 80116,
"tgt": "Will Ibrupropen help relieve cold and flu?",
"src": "Patient: Have had a cold the past few says. Drainage in the throat. Lots of blowing my nose. This morning after I woke up, I blew my nose. It was stuffed, so I blew it kind of hard. Well, my left ear popped. Pain happened next. A few minutes later, the pain decreased, but now I am afraid that I hurt that ear. I am afraid that if I sneeze or blow my nose again, it will hurt more or cause damage. What should I do? I took an ibrupropen. What should I be experiencing if real damage occurred? Doctor: Thanks for your question on Health Care Magic. I can understand your concern. By your history and description, in my opinion you are mostly having severe cold. It is mostly viral infection. Ibuprofen is good for anti inflammatory action. But you need antihistamine drug for sneezing. No need to worry for ear damage. It will clear once your cold is improving. Also start steam inhalation 5-6 times a day. This will help you in stuffed nose. Also drink plenty of fluids orally and keep yourself hydrated. Avoid hard to chew food. You will mostly improve in 3-4 days. Hope I have solved your query. Wish you good health. Thanks."
},
{
"id": 76760,
"tgt": "Suggest treatment for chronic dry cough",
"src": "Patient: I am a 32 Yrs old male. I have been suffering from chronic dry cough which occurs periodically specially whenever i am exposed to chilled water or after excercise for 7 years.Earlier whenever i used to take some antibiotic plus cough syrup it was going off but this month when again it started after taking the chilled water it is not going even after taking medications for more than ten days. The doctor has prescribed me brochodilator, antibiotic, prednisolone Doctor: Hi thanks for contacting HCM...Noted you have here allergic cough....Allergen or by cold you can have histamin like mediator release and they can lead congestive symptom...This time you can have bronchitis....Chest x ray useful for it...Bronchodilator and antibiotic given...Here you might have inflammatory swelling in respi tract so steroid given for it .....If living in cold you can use humidifier...Avoid smoking.....Steam inhalation helpful in decongestion.Avoid excess spicy and fried food....Take drugs for prescribed duration...You can boil milk and add 2 garlic cloves in it ....cool it and consumed .....It will help in relieving congestion...If still no benefit I would like to investigate with spirometry for pulmonary function test.....Take care Hope your concern solved....Dr.Parth"
},
{
"id": 125586,
"tgt": "What causes leg pain?",
"src": "Patient: My legs are very heavy and they pain throughout the day and while getting down from the bed in the morning it takes lot of time to get started whom should I contact this is happening for past about an year or so doctors are not able to diagnose the problem and I am suffering if I take Mr I am fine I m 52 years female Doctor: Hi, You should be evaluated for a varicose vein, any fracture etc. Also, undergo colour doppler if there is any discoloration or swelling. Hope I have answered your query. Let me know if I can assist you further. Regards, Dr. Varinder Joshi, General & Family Physician"
},
{
"id": 53802,
"tgt": "What does this liver triphasic CT report indicate?",
"src": "Patient: my father(50) is suffering from liver cancer (HCC) since 5 months. multicentric lesion are present.chemoembolisation was done. triphasic CT report-the CT revels a large mass exophytically arising from the caudate lobe which shows subtile arterial enhancement but does not show any rapid wash out pattern.the other lesions in the liver does not show any arterial enhancement. thrombus in the main portal vein with portosynthemic collaterals are seen. give me advice as early as possible Doctor: Hello and thanks you for your query. I am Dr. Rommstein and I will try to help you as much as I can with my answer.unfortunately,it looks like extensive and late stage of liver cancer and I am afraid that surgical resection is not possible. What can help is chemotherapy and transarterial chemo embolization. This may reduce tumor size and make possible surgery in future.I hope I have answered you query. If you have any further questions you can contact us.Kindly regards. Wish you a good health."
},
{
"id": 159147,
"tgt": "Biopsy of pancreatic mass showing cytomorphic findings with mucinous neoplasma with mild atypia. Is it pancreatic cancer? Treatment options?",
"src": "Patient: Hi. My mom is 74 years old and the recent biopsy results of pancreatic head mass showed that cytomorphologic findings consistent with mucinous cystic neoplasma with mild atypia. Is this classified as pancreatic cancer? What procedure/s are recommended for this condition, taking into consideration her age? What is the success rate of people at this age surviving or recuperating well after such procedures? Hope you can elaborate. Doctor: Dear What are no. of atypical cells/hpf. If more than 5 then it seems it is a Ca. T/t options at this age is 1) Surgery - if operable & GC / co-morbidities allow. 2) Con. CTRT 3) Chemotherapy only after seeing all records. Actually, Ca. pancreas is usually diagnosed in locally advanced stages, in which case the cure rate is very less. All the Best."
},
{
"id": 107706,
"tgt": "What causes pain in the lower back radiating to the spine and neck?",
"src": "Patient: Hi I have problems walking, lower back pain which flows the outer sides of my spine and into my neck. My tailbone feels slightly higher than it used to be and when I try to do a sit-up it feels like I am rolling over bones as if they are all sticking out. The tail bone is enlarged as if the bone has actually grown into what feels like a cube in my back. I also have wobbly walking and I stumble somewhat when walking making it very uncomfortable and embarrassing. Physios do not seem to have any idea what is wrong and my GP keeps sending me for blood tests and x-rays which show nothing. Doctor: Coccydynia... take one sacrococcygeal spine xray AP and Lateral view... Sitz bath,Ring pillow, sleep in lateral position"
},
{
"id": 24180,
"tgt": "Can Supermet-XL cause erectile dysfunction?",
"src": "Patient: hi ,i m 23 yr old with 63 kg wt. .i hv been taking supermet xl 25 for 2 yr . I hv been ablated 4 ventricular techycardia before 2 yr. . . My doc prescribe me this drug as a prophaylaxis. .i wanna a knw that is that any side effect on penile erecti\u00f6n yet i hvnt experiencd it. Doctor: Hai,Supermet XL (25 mg)- Metoprolol is known to cause sexual dysfunction on long term use.sexual dysfunction/impotence are one of the side effects of supermet.if you experience erectile dysfunction kindly consult your physician for further action.RegardsDr.S.Senthilnathan"
},
{
"id": 60640,
"tgt": "I had a liver function test which showed high bilirubin but no jaundice",
"src": "Patient: hi i recently had a liver function test, the results showed increased level of bilirubin (3.4 md/dl then 2.4 mg/dl). my liver enzymes (AST and ALT) were very normal. my blood test was normal as well. i have no jaundice at all. i feel very fine but am a bit concerned about the bilirubin levels. Doctor: my bilirubin total is 1.4 mg/dl and bilirubin direct is 0.8 mg/dl . is it jaundice. wht type of treatment do i have to take"
},
{
"id": 44248,
"tgt": "Can I get pregnant if I have cystic glandular endometrial hyperplasia ?",
"src": "Patient: hi, my endometrium wall was 16mm thick in trans vaginal test done a day before i got my period, i have small follicles in both ovaries and small cystic areas on endometrial wall.. having irregular period , diagnoised with cystic glandular endometrial hyperlasia.. can i get pregnant? Doctor: Hello. Thanks for writing to us. Cystic glandular endometrial hyperplasia is usually seen in case of increased estrogen levels. It does not interfere with your chances of getting pregnant. The small cysts in both ovaries need to be evaluated as it might be indicative of polycystic ovaries. I hope this information has been both informative and helpful for you. Regards, Dr. Rakhi Tayal drrakhitayal@gmail.com"
},
{
"id": 104709,
"tgt": "Sneezing on exposure to dust, allergens, allergic rhinitis. High risk of lung diseases?",
"src": "Patient: hi..i m always experiencing sneezing when i m exposed to dust or any any allergens..i ve consulted a doctor then he told me i m having an allergic rhinitis ...will i have the tendency to develop any lung diseases or any lung condition in the future?..i do seldom use antihistamine as my medicine..what else can i do to prevent this? Doctor: Hello dear... I went through your post in which you have mentioned that you are having Allergic rhinitis. Management includes: 1. Antihistamine preparations, corticosteroids & nasal decongestants to provide symptomatic relief during acute attacks. 2. Montelukast preparations may be used as a maintenance therapy to relieve symptoms of seasonal allergies. Lung involvement in the form of asthma can be seen in the presence Hyper-responsiveness of air passages to an allergen. To prevent these symptoms, make sure that you are well protected from cold, dust & other allergens. Wishing you a good health. Thanks & take care."
},
{
"id": 16232,
"tgt": "Reddish rashes on the shins. No itching. Overweight. Causes?",
"src": "Patient: Got dressed this morning...everything was normal. Came home at 4p.m. Changed clothes and when i took off my pants there is a red rash on both of my shins. Looks like rosacea sort of but it does not itch or anything and the skin in that area is not hot or anything. Never happened before in my life. I am 51 years old and very over weight. The rash is NOT raised or bumpy at all rather it is completely smooth Doctor: Hi there. Thanks for writing in. Any history of fever? Any significant medical history? Any history of drug intake/ medications? Any history of joint pains, decreased urine output etc? In the absence of the above four, it might be an allergic condition. Try an over the counter anti histamine drug for 2 days. If no improvement, please visit a dermatologist. Hope this helps. Regards, Dr. Divya Kuttikrishnan"
},
{
"id": 4778,
"tgt": "Bloated, tired, negative pregnancy test, irregular periods. Have taken the test too early?",
"src": "Patient: I have been cramping for the last 3 days, i am bloated and feel tired all the time. I took a pregnancy test today (first response) and it was negative even though my period is not until the 15th of this month. Could i have taken the test too early? And should i expect more symptoms? I ovulated on the 1st and my husband and I were intimate. Doctor: Hi,It is too early to say about pregnancy.You have gone for test too early.If do not get period on time, go for test again after 7-8 days passed of your expected date of period.Ok and take care."
},
{
"id": 3857,
"tgt": "Based on my ultrasound can I conceive?",
"src": "Patient: on the 8th day of my menstural i had my first tvs and in reports.The endometrium measures 4.3mm and shows smooth apperence right ovarian dominant follicle measures 15mmleft ovary measures 21*14mmi m 28 and plz tell is there any major complication to concieve Doctor: Hi, thanks for writing..Get a scan repeated after 2 days and repeat every alternate day till follicle size reaches 20 mm.. this is time ovulation occurs.. Endometrial thickness should become 8mm for conception to occur.. After follicle size reaches, repeat scan after 2 days to see for signs of ovulation.. Hope I have answered your query.. Good day.."
},
{
"id": 72239,
"tgt": "Suggest remedy for cough due to water in lungs",
"src": "Patient: My husband has been coughing a lot. He has had a steriod shot and been on anibiotic for 4 days.He had am x-ray which shows fluid on the lungs (both) but is still deteriating. His pallor is grey. Should he seek urgent care or just give the antibiotics more time. He doesn't follow-up with his PCP for over a week. Doctor: Thanks for your question on Healthcare Magic.I can understand your concern. Yes, you should definitely take him to emergency room (ER).By your history and description, possibility of pulmonary edema (fluid accumulation in both lungs) is more likely.Most common cause for this is congestive heart failure (CHF).So he will need ecg, 2d echo urgently. Treatment is oxygen and diuretics. Don't worry, he will be alright but first diagnose him and start appropriate treatment. Hope I have solved your query. I will be happy to help you further. Wishing good health to your husband. Thanks."
},
{
"id": 27067,
"tgt": "What causes left sided chest pain?",
"src": "Patient: Hi, my dad is not feeling good. He has chest pains and the pain is coming from left side of his chest. He feels that his left hand is warming up alot and alot of pressure and even his left side of his chest he feels like it is swelling. What do you think he has? Does he need to go to the hospital? Doctor: Hello. Thank you for your question and welcome to HCM. I understand your concern. I think that if this is a pain that started while resting, he should go to the hospital. There an EKG should be performed and blood tests should be run, such as troponin and creatine phosphokinase (CK) - MB fraction. If they result all normal, then a cardiac stress test, 48 hours after he has been without pain, should be performed, to check if there is an indication of a coronary artery narrowing or blockage. If there is a positive finding, then a coronary angiography would be the reasonable approach. I hope I was helpful with my answer. Take care."
},
{
"id": 54876,
"tgt": "Suggest a safe level of blood pressure in the event of a lever disease",
"src": "Patient: Is blood pressure of 104/51, heart rate of 95 in 42 year old male with end stage liver disease a bad sign. He had a bilirubin level of 43.9 and INR of 2.7 with his last labs 3 weeks ago. He is very sleepy and weak, and his skin coloring is yellow-gray. He is on pentoxyfilline, lasix, spironolactone and prednisolone. Doctor: Hi thanks for asking question..Noted here main concern is your end stage liver disease.He also feeling sleepy .So hepatic encephalopathy should be investigated by estimating blood ammonia level.If present lactulose with rifaximine like antibiotic needed....You should consult surgeon for treatment option like liver transplantation...Bilirubin level is very high.So avoid alcohol and take low fat diet.Prognosis here seems not to be good...Surgeon consultation strongly needed.Hope your concern solved.Take care"
},
{
"id": 155189,
"tgt": "What is the life expectancy in a person having a heart tumor wrapped around the aorta?",
"src": "Patient: My girlfriend is 20 years old and was just diagnosed with a benign inoperable heart tumor wrapped around her aorta... The tumor is apparently imbedded in the walls of her heart... The doctor said she was a ticking time bomb but she also mentioned something about ex vivo surgery.... She has seen 3 doctors all of which say inoperable .... How long does she have to live? If we can find a doctor to operate- what are the chances she survives the surgery? Doctor: The only curative option that seems remotely possible in your case is a heart transplant and that too I cannot be sure since i dont have all the reports in hand. But it will be worthwhile considering and discussing this option with your treating doctor and preferably a heart transplant surgeon. Otherwise, there seems to be no option and she will die with progression of the disease. Ex-vivo surgery is where they take out the heart for a short while, remove the tumor, repair the heart and then put it back. Its a distant possibility considering the extent of the disease."
},
{
"id": 77633,
"tgt": "What causes continuous cough which has lead to chest pain?",
"src": "Patient: I have had a cough for about 5 mnths now. I seen a doctor last Friday. She said I have underlying asthma issues. I am still coughing alot. For the past hour I have kinda had pain in my chest. Could something be strained from coughing or is it possibly something else? Doctor: Hi thanks for asking .Here long duration cough is there.if associated history of fever and weight loss then sputum for TB done.You do your pulmonary function test to rule out ashrhma and about restrictive or obstructive lung lesion.Avoid smoke strictly.Don't take spicy food.Here cough can be because of chronic bronchitis.So better to do sputum for TB and PFT first.Then according to report further work up done.If dry cough troublesome dextrose thorp han tablet or syrup taken.Don't worry.you will be alright.Thank you"
},
{
"id": 24097,
"tgt": "Suggest treatment for orthostatic hypotension",
"src": "Patient: I have been diagnosed with Orthostatic hypo tension. I get so dizzy that I can barely walk in a straight line and have passed out. They have given me midodrine and florineff. The Florineff I stopped taking because of the high blood sugars (I am diabetic). These drugs really do not help. Some days are ok and I can maintain balance although still dizzy and others are terrible. I ve been to the cardiologist, a neurologist have tried all of their suggestions, but nothing helps. I also have an increased salty taste in my mouth, not to everything it seems only things that our sugary, if I eat sweet I taste extreme salt. I did have a CT scan but no MRI.......is it possible that this is something else? I did have Ketoacidosis about a year and a half ago that I was not hospitalized for which is when this started.....I am a type II diabetic. Doctor: Hey, You did not mention your age. If it is above 50, than risks of heart block causing these dizzy spells & loss of consciousness has to be ruled out. For that you need to have a holer done which is 24 hr ecg monitoring. Secondarily orthostatic hypotension is defined as fall in bp with change in position from sitting or lying down to standing. You just have to increase your fluid intake for that & get up from lying down gradually.Thanks Dr Sameer"
},
{
"id": 158707,
"tgt": "Lump on the left and right side of esophagus. Symptoms of thyroid cancer?",
"src": "Patient: Hello, I have a question about a lump on the left and right side of my esophagus . I noticed the first one on the left of my neck a few months ago. It was strange because if I pushed on the lump it would almost click back and then return to the place it was. I have also noticed that I have had a sore throat for the last week or two. This worries me because it sounds like the symptoms of thyroid cancer . Doctor: Dear Sir/Ma'am,Sorry to hear about your problem. From your description it looks like you are suffering from a thyroid swelling. Thyroid swellings can be benign or cancerous and the only way to differentiate between the two is by getting a needle test (FNAC) of the swelling done.You should visit a surgeon and have your swelling evaluated.Hoping for the best."
},
{
"id": 166031,
"tgt": "What cause child to lay back neck while being lifted?",
"src": "Patient: Hi my daughter has been for her 8 week check up today and returning was great until she pulled her up from being lay down with her arms, she seems to keep her head back, she can hold her head up when sitting up and can arch it bk when on he front, they said if it dissent improve (the lying down neck bk et) by 12 weeks they will refer her to a paediatrician what could the problem be, I m really worried!!! Doctor: In pulling the baby start control after 8 wks and achieve full control till 20 wks of age. so no need to worry. Just wait and watch."
},
{
"id": 154014,
"tgt": "What are chances of survival of 70-year-old with small lung cancer?",
"src": "Patient: as a non smoker i had radiation treatment for mouth cancer and part removal of tongue i am very active and eat well but have lost a lot of weight l was told i was cancer free now im told months later that i need surgery for cancer on lungs the doctors say it is small and treatable im 70 and scared what do i expect to happen Doctor: Hi,Thanks for writing in.A small cancer in the lungs needs to be confirmed and then treated. It is important to first get complete cancer work up done to be sure that the cancer in lung is small cell cancer if still not confirmed.Once it is confirmed then it is important to get it treated by surgery and then get chemotherapy and radiation therapy to kill the left over cells in the lungs. Please do not worry about the surgery. Lung is an accessible part of the body and since your doctor has examined you in detail therefore a surgical decision must not be delayed. There is a good chance of complete recovery and normal survival. Please do not worry."
},
{
"id": 152817,
"tgt": "Suggest treatment for nocturia in case of Ca prostate",
"src": "Patient: I am 70 yr old male diagnosed as ca prostate T2aN0M0 adenocarcinoma in January 2014.intermediate risk.Gleason score 4+3=7 I had external beam radiation-igrt-v. mat with androgen deprivation therapy with2 doses of inj pamorelin 12.5 in3 months intervel . R T started on 10-03-2024 and completed on 17 04 2014 without any interruption. Total dose, 7000cGy in 28 fractions.IPforSS score at the end of RT was 20. My urinary symptoms especially nocturia is distressing. My PSA level 16 12 2013 -- 7.1 28-04-2014- 0.15 25-07-2014- 0.02 Recently i have constipation too. What is the advise for the distressing nocturia? How to know the cancer is cured? Doctor: hello, The best remedy for nocturia will be to cut down fluid intake before 3 hours of sleep time. Empty the bladder befre bed. if both does not help than adding a drug which act on bladder tone can be added"
},
{
"id": 7037,
"tgt": "Can you conceive with PCOD while on folic acid ?",
"src": "Patient: Hi.. My last period was 25th October 2011... I have pcod. I am taking metformin,folic acid for three month... 2nd day of my period doctor prescribe me clomid for 5 days... On 15th day my follicle is 20mm... So doctor put hcg injection 10000iu and ask me to intercourse on next two days... Then doctor provide me duphaston from 20th day of my period... Uterus thickness measures 9mm.. My question is when to check home pregnancy test.. And will I conceive this time?... Doctor: Hi Amutha, You can get your urine pregnancy test done around 5th of December.All the best till then.Dont worry,you may become pregnant this time."
},
{
"id": 95603,
"tgt": "I have got problem in sting stool,fat in stool, dark circle around eyes, kindly help me",
"src": "Patient: hi, i am 24 years old guy, i have got problem in my stool from the last 2 years..it has awful smell and some fat floats in the water, too much gas hard to digest all foods..and dark circle around my eyes.. Doctor: Hi well come to HCm. Your dark circle around your eyes has nothing to do with this.It seems to be simple indigestion with acid peptic disorder.Improve life style and food habits. Donot take Junk food Too much of fatty food spicy food etc tobacco or smoking and avoid unnecessary stress. Take some digestive enzyme containing lipase, pantoprazole with domperidone may help you. Thanks"
},
{
"id": 145965,
"tgt": "How can disc protrusion with spinal stenosis be treated?",
"src": "Patient: i am trying to understand mri stating a large central disc protrusion measuring 8mm in ap dimension at l 4-5 which causes significant spinal stenosis.there is also prominent facet artrhropathy at this level which contributes to spinal stenosis with effacement of the thecal sac.tis is just one of many protrusions of the back and neck Doctor: Hi,Thanks for writing in.The lower back is made up of an alignment of small bones. These bones have sponge like discs between them and the spinal cord passes along the back of these bones through the spinal canal. You seem to be having one of the discs protruding between the bones and this protrusion is backwards and causing narrowing of the spinal canal. A 8 mm disc protrusion is a large one and can cause significant pinching of the nerve roots at L 4 - 5 level leading to severe pain in your back and legs.There are patients who get relief by doing physiotherapy and taking neurogenic pain relievers. If your pain is severe and not responding to medical treatment then the disc protrusion needs to be removed by doing a small surgery at the lower back region. Presently, laser assisted surgical techniques are also available."
},
{
"id": 62464,
"tgt": "What is the painful lump on my head along with dizziness and fever?",
"src": "Patient: Hi, my name is Jayme Edwards. I have been experiencing dizziness- which also comes with extreme weakness, a small fever and confusion- Other than dizziness I have been having trouble remembering things. I get sharp, short, painfull, pains in my head. About four weeks ago I found a small bump on my skull on the back/top of my skull. And I have been getting more pains in my head since, and I get them right by the bump, but in my head. Please get back to me, this scares me. YYYY@YYYY Please email me if this ends up costing me money and you think thise might be severe, Thanks. Doctor: HI,Dear,Welcome to HCM.Based on the facts and data of your query,you Seem to have infected painful lump on back of Skull.This is causing fever/confusion/dizziness etc over last few weeks or so.But don't worry its not Cancer-Prima Facie,though it needs further clinical and lab/ radiological investigations from Surgeon , who would rule out the various other causes of it.So unless Surgeon gives his final opinion,DON'T worry at all.It could be treated after confirming the diagnosis,by medicines and by surgery if need be.So RELAX and act fast.Hope this would help you to resolve your sever anxiety.Welcome for any further query in this regardWill appreciate writing your feedback review comments,to help the needy patients like you at HCM.Good Day!! Dr.Savaskar,Senior Surgical SpecialistM.S.Genl-CVTS"
},
{
"id": 189291,
"tgt": "Had a root canal. Swelling on cheeks opposite to cavity. What to do?",
"src": "Patient: hi i am manzoor..recently i had a root canal on one of my tooth.. before root canal i had swellings on one of my cheeks.. but the problem is the swelling is on opposite side to the cavity.. My dentist told that it is because i was eating food oneside for a longtime... The problem is still there after my root canal.. kindly suggest me something..... Doctor: hi manzoor, Thanks for asking query, As the swelling is on other side there is very little chance of the swelling due to that tooth cavity , so it is not corrected even after the root canal treatment. There are many other reasons for cheeck swelling like inflamation due to tuberculosis,syphilis ,infected cyst,celulitis,perotitis,so should go for complete clinical and radiographic checkup. Until then take the antibiotic , analgesic course. Hope it will help you"
},
{
"id": 217472,
"tgt": "How to treat the persisting knee pain followed by surgery?",
"src": "Patient: I had knee surgury in 2011 in november. I still hv lots of pain because i hv scar tissue. It hurt me to stand,bend the knee. When i lay down in bed and tried to get some sleep it hurt.When i go to stand up sometime feel like there is pins in my knee. I no there is scar tissue there. What can i do? Doctor: hi you can use moist heat use wet wash cloth and make sure that it's not too hot that you can burn yourself and you can use hot water pack to relief pain use kneel carp or crap to support knee and use pillow below ankle that will be helpful to you and do stretching and strengthening exercises static quadriceps and hamstring that will be helpful to you you can also take physiotherapy that will also help you to relief pain and strictly follow do and don't of surgery for do and dont you can ask me for list and do this treatment this will help you wish you good health"
},
{
"id": 176338,
"tgt": "What causes purple discoloration of lips when suffering from high fever?",
"src": "Patient: 9 month old girl fever has been up to 103 and back down then back up. Went to dr they said its a virus. Came home she napped then woke up with a fever but her hands feet and lips are a little purple. Gave her a bath and she began to shake a lot. She has stopped shaking but still has a fever over 100 with slightly purple hands deer and lips. Any idea? Doctor: HI...purplish discoloration and duskiness of lips occurs in high due to peripheral vasoconstriction. If the kid is active otherwise and well when the kid is not having fever, you need not worry.Usually rather than fever, what is more important is the activity of the child, in between 2 fever episodes on the same day. If the kid is active and playing around when there is no fever, it is probably viral illness and it doesn't require antibiotics at all. Once viral fever comes it will there for 4-7 days. So do not worry about duration if the kid is active.Regards - Dr. Sumanth"
},
{
"id": 16728,
"tgt": "Do I need to maintain my blood pressure within normal range to get my tooth extracted?",
"src": "Patient: I went to the dentist this morning and when they took my blood pressure, it was 188/111. They told me they cannot perform the extraction and I needed to get my blood pressure and A1C in check before they can do any work. I called my Primary Care Physician and they don t have any openings. I m not certain what I should do or if I should wait until my scheduled appointment with my new Primary Care about a month away. Doctor: Hello, A single spike in blood pressure values could be related to anxiety. Anyway, I recommend you to closely monitor your blood pressure values frequently during the day in the next days. If your blood pressure values are persistently high, I would recommend consulting with your attending physician for some blood lab tests (complete blood count, PCR, ESR, kidney and liver function tests, blood electrolytes, thyroid hormone levels, etc.) and considering starting antihypertensive therapy. Hope I have answered your query. Let me know if I can assist you further. Take care Regards, Dr Ilir Sharka, Cardiologist"
},
{
"id": 2893,
"tgt": "Can pregnancy happen after having unprotected sex?",
"src": "Patient: I don t keep up with my menstrual cycle because it s irregular but I thinks it s about 30 days. My last period started on September 12 and lasted 4-5 days. I had sex for the first time on October 4, unprotected but he didn t finish. Could I be pregnant? Doctor: Hi,Precum may enter into vagina unknowingly before ejaculation and it contain sufficient sperm concentration that cause pregnancy if sex was done in fertile phase of period. You got last period on 12th September, did sex on 4th October. So you did sex in the safe period of cycle if we consider 30 day of cycle. Ovulation will occur at 27th or 28th September and it will live only for 12 to 24 hours. But you did sex after ovulation so no chance of pregnancy. But you told you have irregular cycle. So if it has sometime long period like 35 days or more than pregnancy is possible. In this case you can take I pill with in 72 hours of pill to prevent unwanted pregnancy. Avoid stress, take healthy diet and drink plenty of water. Use safe sex practice like condom in future to prevent unwanted pregnancy. Hope this may help you. Contact further if follow up needed. Best regards,Dr. Sagar"
},
{
"id": 26276,
"tgt": "Suggest treatment for severe arm pain and rapid heart rate",
"src": "Patient: I had an immense amount of pain in my left arm with an increase in heart rate then decrease. It bounced back and forth I also had shivers then increase in body temperature. Is it because of my birth control? Maybe too much inbreprofun? I was outside a lot today moving into my new house. I also have a metallic, almost irony smell in my nose. What should I do? I feel better now that I ve calmed myself. Doctor: hello,I have gone through your query.Thanks for using HCM.I need to know your age,relation of chest pain with exertion,history of smoking and family history before guiding properly.Your current symptoms appear to be due to over exertion and fever related.Over use of ibuprofen certainly lead to acidity and risk of ulcer.You must take rest and simple paracetamol 500 mg.If not relived you shouldconsult a physician for evaluation,My best wishesDr.Rajesh Teli,MD."
},
{
"id": 189246,
"tgt": "Dark red spot on old tooth after having teeth removed. Is it blood clot? What can be done?",
"src": "Patient: i think i have dry sockes i just got 4 teeth removed today at like 9;30am and its 10;52pm i kept the gauze in my mouth tel my face was not num at like 10;00pm i looked in the mirror and saw something dark red over the spot ware the old tooth was so i moved it with my tongue and it came off i spit it in my hand and it wan a clump of blood i was looking online to see what it was and WWW.WWWW.WW said it was a blood clot and it said you get 1 on the spot of the gum after the tooth is gone and after i was online looking i looked in the mirror and it was a nother one in the same place and the other spotd of my mouth dont look like that just that 1 out of the 4 so i need to know if i have dry socket and if i need to go back to the dentist Doctor: hello there, Dry socket is a extremely painful condition seen 3 to 4 days post extraction. red spots are may be due to local anesthetic solution injecting points or the ruptured tissue. you need not remove the blood clot as it helps in healing and wound closure. nothing to worry about wait for a day or two if the extraction socket has not closed,pain and bleeding persists than you got to see a dentist. Follow your course of antibiotic to prevent infection and analgesics,take care."
},
{
"id": 199363,
"tgt": "How to treat lump on my balls with puss in it ?",
"src": "Patient: I had a small lump on my balls when I applied pressure it moved so I slid my fingers together and it moved up eventually releasing thin puss from a hole near the base of my penis . What is that and why is it doing that? It was a good bit the first time I did it but it seem to be just a small amount every couple day . Doctor: HelloI appreciate your concern This could be infected sebaceous cyst of the scrotum or sign of STIIf you can upload me a photograph I will be able to guide you better I would like further information to help you better Your age and relevant medical history like diabetes or Past incidence of STIAny recent exposure to unprotected sex?I would advise you to wash and clean it with antiseptic soap or solution and apply antibacterial ointmentMaintain good hygieneIf it doesn't heal than consult your doctor for examination and battery of tests if need be, to confirm the diagnosis and accordingly treatment.Thanks and regards"
},
{
"id": 23814,
"tgt": "Can I go for a roller coaster ride after suffering from a stroke?",
"src": "Patient: I had a stroke last year. I am a 37 year old female. The cause of the stroke is still undetermined. My heart was checked and healthy, and my arteries and blood was tested and fine. Never been on birth control. Anyways, would it be safe for me to go ride roller coasters Doctor: Hello!Thank you for asking on HCM!Regarding your concern, I would explain that roller coaster is not a very safe sport in this clinical situation (as it may cause changes in blood pressure). As no possible cause of stroke was found, we can not confirm that it is safe for you to undergo such extreme sports. I recommend you to closely monitor your blood pressure, continue on a healthy lifestyle and perform periodically scheduled follow ups (which blood lipid profile, resting ECG and cardiac ultrasound, complete blood count, fasting glucose, etc.) every 6 months to check if everything is OK. Hope to have been helpful!Kind regards, Dr. Iliri"
},
{
"id": 151837,
"tgt": "What tests and treatment can be taken for Spinal Ganglion ?",
"src": "Patient: my wife is 26 and has had pain and numbness in her leg.She does get back pain and also in her shoulder .She has been told she might have Spinal Ganglia? what are the tests for this and possible treatment? Doctor: Welcome to Healthcare Magic Good Day It is possible she has intervertebral disc prolapse which is compressing on her spinal nerves which can cause numbness, weakness of her legs. She needs to get an X-ray or best an MRI as requested by her Orthopaedician to clearly see where the nerves are being compressed. Once the cause and site of compression is seen, appropriate treatment can be started, surgery may be required to remove the obstruction."
},
{
"id": 56805,
"tgt": "What are the sign and symptom for gallstones?",
"src": "Patient: i am a 19 year old female who is having signs of gallstones i have had a gall stone before when i was pregnant and i thought it went away. but now on top of the gall stone symptoms i am also very hot i cant cool down it feels like i am boiling on the inside and thats what is concerning me the most. Doctor: Gall stone present as indigestion burping abdominal pain colicky in nature . What you are mentioning is not related to gall stone"
},
{
"id": 77386,
"tgt": "What causes chest pain and watery saliva?",
"src": "Patient: Hi while i was sleeping my mouth kept on producing large amounts of watery saliva and then i had really bad chest pain in where my chest felt very very tight as if something was stuck there, right now i still feel a little bit of tension in my chest but it comes and goes Doctor: Hi thanks for asking question.Here are few possibilities in your case.You might have chest pain because of respiratory infection.It could be simple cold ,or sore throat like viral infection.Because of excess mucus and secretions blockage may feel.chest pain can occur as constitutional symptom.Second bronchitis like obstructive lung disease also has to be ruled out if smoking history present.chest x ray or spirometry for pulmonary function test useful.CBC also done.if show neutrophilia then bacterial infection could be probability and antibiotic needed.Here if you have acid reflux or ulcer like condition then reffered chest pain can occur.omeprazole needed for it.If congestive symptoms present decongestant or antihistaminic taken.steam inhalation useful with eucalyptus.Drink more water.To rule out cardiac cause ECG done.Finally although here it seems to be less chance , muscular strain can lead to chest pain.simple analgesic useful for it.I hope my suggestion will helpful to you.Dr.parth"
},
{
"id": 34999,
"tgt": "Suggest remedy for painful sores on forehead",
"src": "Patient: i wear eyeglasses and have developed a pair of angry sores behind both temple pieces...repleat with blood & pus discharge. they're getting increasingly painful. I clean them with alcohol & peroxide and apply neosporin but nothing helps. suggestions please? Doctor: Hello dear,Thank you for your contact to health care magic.I read and understand your concern. I am Dr Arun Tank answering your concern.It look like allergy reaction and not a infection.I advice you should first clean it with sterile normal saline followed by application of clobetasol ointment over the wound. This will prevent the allergic reaction.You can also do frequent cleaning and washing followed by dressing with dettol, betadine followed by polysporin applications.Please avoid using sunglasses for the time your wound become clear.You can search for the sunglasses which has material consisting of non allergic material. This will avoid your future allergic reaction.I will be happy to answer your further concern on bit.ly/DrArun.Thank you,Dr Arun TankInfectious diseases specialist,HCM."
},
{
"id": 57859,
"tgt": "Could being overweight cause fluctuating SGPT levels with other liver function levels normal?",
"src": "Patient: Hi, I am 26 years old and I weight 103kgs... my height is 5 ft 7 in... I was tested some time ago and had elevated SGPT... It was 165... SGOT, GGT, Billirubin and all other Liver indicators were normal. After a month, my SGPT was 83... And in a few months SGPT was 27 and recently it was 16 with rest of the LFT normal. What could be the reason for this elevated SGPT? Doctor: Thanks for query.elevated ALT(sgpt) may be due to different reasons as you mention it was high and gradually decreased probable cause could be alcohol abuse if you had a binge or Drugs (ie, statins, aspirin, barbiturates, HIV medication, herbs etc if you had them any.you needed not worry much about their levels unless they are persistently elevated .regards"
},
{
"id": 61806,
"tgt": "Suggest remedy for lump in shoulder blade",
"src": "Patient: Hi i have just found a lump just near my right shoulder blade it gets itchy and stings sometimes also find my selve out of breath more than usual, and a little weezy when i lie down, just to let you know 2years ag i had a spontaneous hymaphorax and havent been right since realy worried. Doctor: hi.it could be any of the following lesions - cystic formation (sebaceous or keratinous type), lipoma (fatty deposit) or fibroma (fibrous tissue formation). other lesion types must also be ruled-out, such as malignancy. it is best if you consult with a doctor, preferably a general surgeon, for clinical evaluation. diagnostics (imaging) and management (medical and/or surgical) will be directed accordingly.hope this helps.good day!!~dr.kaye"
},
{
"id": 221966,
"tgt": "Is Amitriptyline safe during pregnancy?",
"src": "Patient: I m 12 weeks pregnant. I used to take amitriptyline 25 mg /day for chronic back pain (sciatic nerve pain) before the pregnancy. I was OK until the pain started to be worst few days ago. Can I keep taking amitriptyline during the pregnancy? Is it safe the the baby ? thanks Doctor: Hi dear, I have gone through your question and understand your concerns .Amitriptyline is known to cause limb defects and developmental delay in the the fetus, if the mother takes it during the pregnancy.I suggest you to avoid using it during pregnancy and consult your doctor for an alternative medicine, which is safer in pregnancy.Hope you found the answer helpful.Wishing you good health.Dr Deepti Verma"
},
{
"id": 198542,
"tgt": "Does herpes affect sperm count?",
"src": "Patient: I don t know how I got Herpes....as per information in internet it should be sexually transmitted I didn t have sex with another women other than my wife. But as per previous medicine gave me to control cold sore I have Herpes. Is it affecting sperm count...Can I be a farter? Im confused how can I explain this to my wife? Doctor: DearWe understand your concernsI went through your details. Herpes is caused by a virus that affects the genital area (the vulvas and vagina in women and the penis in men). But, in medical practice, it does not seem to cross over into other reproductive areas and has little or no effect on a man's sperm production nor a woman's ability to conceive. The information you read on internet could be false and misleading. Please consult a STD specialist before confusing yourself and work with him.If you require more of my help in this aspect, please use this URL. http://goo.gl/aYW2pR. Make sure that you include every minute details possible. Hope this answers your query. Available for further clarifications.Good luck. Take care."
},
{
"id": 127009,
"tgt": "Are pain and stiffness in the hips symptoms of fibromyalgia or arthritis?",
"src": "Patient: Is it sciatica or fibromyalgia or arthritis. Horrible pain when getting up so stiff even if I did just sitting for a couple minutes. The one side of rear down to the bone is very painful but not all the time to change shifts to my hip feels like it catches and then my thigh muscle gets like twisted. I ve broken my tailbone about 40 years ago when I was in Elementary school slipped off the high dive and landed on cement. Never had any major problems. Chiropractors can t help me anymore he didn t know what else to do. It s been going on for about five years and just gets worse as time goes by. Any any information would be appreciated. Doctor: Hi, The symptoms are due to arthritis. You can consult an orthopaedician and get evaluated. Hope I have answered your query. Let me know if I can assist you further."
},
{
"id": 15018,
"tgt": "What could be the subtle rash on left cheek?",
"src": "Patient: I am 52 yr old female. Have a very subtle rash on left cheek . A few little spots no puss just slight ...But the area feels warm and slight irritated ... I have had it about a month. It seem to be better in the morning Left side only spots seem more red when I'm warm Doctor: Hello,Thanks for writing here.Rash seems to be due to allergy.Avoid sun exposure.Avoid soaps and other cosmetics over face.A mild steroid like hydrocortisone and allegra will help.It will be better to consult a dermatologist if there is no response.Hope this will help you.Take care."
},
{
"id": 204939,
"tgt": "Are Folic Acid tablets advisable for a bipolar disorder person?",
"src": "Patient: I have bipolar and am interested in taking folic acid to help with the condition. Do you think this is a good idea and how much should I take? I am not planning on having children and I dont have a folic acid deficiency. I know that my B12 levels are normal. YYYY@YYYY Doctor: hifolic acid for bipolar depression is skeptical decision. Lmethylfolate might still help. but in my clinical experience there is no point to start folic acid instead of mood stabilizers. bipolar ia an episodic condition requiring a close monitoring of mood. so better to go for mood stabilizer or CBT thank u"
},
{
"id": 166750,
"tgt": "What causes forehead pain in kid?",
"src": "Patient: Hi, may I answer your health queries right now ? Please type your query here.. 4 and a half year old girl having pain in her forehead lasting a few seconds. She s scratching at her forehead trying to scratch the pain away. Happens maybe once or twice a day. Doctor: Hi,Short interval pain doesn't have any clinical significance. If a pain in for head is persistent than at this age group first thing you have to do is to see a ophthalmologist, for her eye checkup. May be she is too young to define persistent pain, my advise is to see a eye specialist to rule out any vision error. Secondly it can be because of sinuses problem, but she should have fever for that. My advise is to see a eye doctor and make sure her sleep should be good. Hope I have answered your query. Let me know if I can assist you further. Best Regards, Dr Zeeshan"
},
{
"id": 125184,
"tgt": "Suggest treatment for burning and pain in hips and neck",
"src": "Patient: HI, I hav been suffering from very painful pain and burning in the hips, neck and shoulders. My condition has only progressed large bumps on back and side of neck (I was diagnosed with bone spurs in all bones of neck), and now also very painful hard bump on outside of left knee. Doctor: Hello, As a first line management you can take analgesics like paracetamol or aceclofenac for pain relief. If symptoms persist, it is better to consult a physician and get evaluated. Hope I have answered your query. Let me know if I can assist you further. Regards, Dr. Shinas Hussain, General & Family Physician"
},
{
"id": 144775,
"tgt": "Does this MRI report of spine indicate the need for surgery?",
"src": "Patient: I have had really bad pains in neck, shoulder and headaches for over 6 months. I got an MRI done and it shows the following: Multilevel disc osteophyte complexes cause no significant central stenosis. There is severe right neural foramina narrowing at C3- C4 from prominent facet degenerative change and a right foramina disc osteophyte complex. There is severe right neural foramina narrowing at C5-C6 from uncovertebral joint degenerative changes. Mild neural foramina narrowing is present at other levels. Mild grade 1 anterolisthesis of C3 on C4 and mild grade 1 retrolisthesis of C5 on C6. Do you think this would call for surgery? Doctor: Hi there, thanks for your question at HCM. Cervical spine related problems are usually more manageable than lumbar spine issues. If your pain is manageable and not disturbing your daily routine. I think you can wait. I suggest you to try physiotherapy, isometric cervical exercises, pain medications in a step wise fashion. Injection shots also can give considerable relief. In the spine pain ladder there are a lot of steps to be taken before you land at the step of surgery. Surgery should be the last resort. Your MRI findings look manageable. Urgent surgery would be indicated in case of any weakness, un relenting pain, no response to the steps and balance related issues.Hope this helps. All the best. Regards. Dr.SBK"
},
{
"id": 45414,
"tgt": "My Sperm count is 50 Million, what is the reason behind this ?",
"src": "Patient: I am Murali from chennai, I got married before 1 year 7 months ago, But my wife is not yet conceived, My Sperm count is 50 Million, Normal count, what is the reason behind this ? Doctor: For this I recommend you to read a lady who after lot of treatments, a long research and struggling hard, Discoverd the easiest way to conceive and become pregnant at \"\"41\"\" of her age. To Read her story just go to the below link mentioned http://f8ce1fl9k3n31153vqib3m3k7a.hop.clickbank.net/ If not clickable ??Copy the link and paste in the browser's address bar? and press Enter"
},
{
"id": 93178,
"tgt": "Precautions to be taken when diagnosed with low amniotic fluid? Why am I vomiting with cramps?",
"src": "Patient: What precautions should be taken when you are diagnosed with low amniotic fluid? My midwife told me that my amniotic fluid was low at my 20 week ultrasound . I asked her how low it was. She said she did not know because they did not take any measurements. I asked for some kind of estimation as to if it is on the normal side or severely low, and she could not tell me anything. She scheduled another ultrasound for me a month out, and I think that is way too long when we don t know anything.(I am switching to a different practice because of this, but my appointment is not for another week) This is my third pregnancy. I had low amniotic fluid with my son at 36 weeks, and had to be induced at 38 weeks. I have been drinking a lot of water, and have not been as active as I had been. I have also cut out caffeine completely. I have consumed two liters of water in a two hour time frame daily, based on the study that supports this. I have noticed much more movement from my baby this week than I had felt previously. I also had severe ( pain scale 7) , recurrent contraction like cramping(with vomiting at the peak of the most intense one) that has not come back since I have been drinking so much water. What precautions should I take to be on the safest side possible? What should I be allowed to do and not allowed to do. Should I have been given more information at my 20 week ultrasound than they gave me? Doctor: hello, thanks for putting up your worry.reduction in the quantity of amniotic fluid is referred to as oligohydramnios. this condition is defined medically as as an amniotic fluid index of 5cm or less. the most common cause is decrease production (ranal agenesis or dyslpasia, urinary obstruction). research has shown that 15-25% are associated with foetal abnormalities.oligohydramnios is also an important sign of chronic placenta insufficiency.thus, i thick you should be admitted and investigated or better see a gynecologist who can re-examine you to confirm the diagnosis.hope this helps. goodluck"
},
{
"id": 156388,
"tgt": "My husband had 12 polyps removed in a colonoscopy. What is the treatment plan if it is cancer?",
"src": "Patient: Sure my husband just has had 12 polyps removed in a colonoscopy 8. mm up to 2.8 cm in the transverse and descending colon. . Strong family colon cancer . Either location especially troubling. Worst case what would the likely treatment plan be if it is cancer Doctor: If it turns out that the polyps are malignant (cancer), investigations would be required including CAT scan of abdomen and chest radiograph to stage the disease, and serum CEA levels. If the disease is is resectable and not metastatic on investigations, surgical resection would be done followed by chemotherapy depending upon pathology report.If disease turns out to be metastatic then chemotherapy would be given.You can discuss with your doctor in detail once biopsy histopath becomes available."
},
{
"id": 150819,
"tgt": "MRI shows acute annular tear, disc protrusion indenting thecal sac, contracting bilateral traversing s1 root. Serious?",
"src": "Patient: my mri report says that I have an acute annular tear at l5-s1 disc posteriorly towards right. posterocentral disc protrusion at l5-s1 level indenting thecal sac and contracting bilateral traversing s1 root. How serious is that since the pain is still there after 10 sessions of traction in physiotherapy and what is the cure Doctor: Hello, I have noted the MRI data If the pain is still persisting despite physiotherapy, you can visit your neurologist and get evaluated. This is especially to look for the signs of nerve root compression that is is being described in MRI. If there are no clinical signs of nerve root compression, i feel that you can still continue conservative treatment with medicines like Gabapentine, pregabaline. Surgery should not be done for pain alone. But if all conservative treatment fails you may opt for surgical management. This is nothing serious. There are options available to deal the problem. Just don't worry and get reevaluated as i discussed Best wishes"
},
{
"id": 32791,
"tgt": "Suggest treatment for cold and fever",
"src": "Patient: hi.... I am a girl of 20yrs from bangladesh...I had bad cold and fever (101-102) last todays and from last night my heart is beating faster than usual rate and so my whole body is shivering... is it anything serious ? What should I do to get back in a normal state ? Doctor: Hello As you mentioned that this fever is just for 2 days . You are also having rapid heart beat ( as this is natural due to fever ) . Also having shivering , this may be due to throat infection or flu ( common cold ) .All these symptoms are just for 2 days ,so most probable cause may be flu , so don't take any tension .Only take paracetamol 10 -12 mg / kg body weight 3-4 times in day for 3-5 days .If symptoms decrease then well and good , if condition worsen then consult a physician and take his treatment .For relief , you will have to wait for 5-7 days ."
},
{
"id": 57347,
"tgt": "Sarcoid condition, infected liver, prednisolone, alkaline phosphatase 263, alanine aminotransferase 131, GAMMA GT 356",
"src": "Patient: my Gastroenterologist has just sent me a letter after examination. I have a sarcoid condition which she says has infected my liver. I am currently on 20mg of Prednisolone for the sarcoid. Blood tests results below: Alkaline Phosphatase 263 Alanine Aminotransferase 131 GAMMA GT 356 ( Was 500 ) An ultrasound shows a normal Doppler of the portal vein although slightly dampened Can you explain the above ?. Thank you Doctor: Hi Dear. I think it's Hepatic involvement of sarcoidosis .Although hepatic involvement usually is asymptomatic, however ones LFT could be deranged coz of chronic cholestatic disease, increase in pressure inside vein that carries blood to liver and cirrhosis .Glucocorticoid treatment is first-line therapy for hepatic sarcoidosis, that's is why your doctor has put you on prednisolone it will improve your general symptoms and abnormal laboratory values . just keep your medications regular and visit your concerning Doctor regularly ."
},
{
"id": 29373,
"tgt": "Suggest treatment for persistent headache, fatigue and headache",
"src": "Patient: I m am a 69 year old female, started having a headache on Thursday it intensified and by Friday evening I became nauseous and with fever. Saturday still had headache but not as severe fever broke around 6:30 this morning but I m still feeling weak, achy, sore throat and headache. Doctor: hiii.welcome to our site.my opinion is that you are having migraine type of headache.avoid stress,because stress is the main precipitating factor for headache.have sound sleep of 8 hours a day.be relaxed always.you can take tablet ibuprofen twice daily for headache.take c.omeprazole twice daily with ibuprofen,as ibuprofen taken alone may cause gastric ulcers.take tablet domstal twice daily for vomiting.thank you.hope you get good health."
},
{
"id": 62109,
"tgt": "What could burning lumps on penis suggest?",
"src": "Patient: on my penis inside of it i feel like little lumps and they seem to spread round in a circle just underneath the helmet or sorta at the bottom of my helmet it never really botherd me but it hurt today and still hurts and spread even more when i urinate it doesnt hurt or burn but when i masterbate it burns the lump thing burns what is this Doctor: Hi,Dear,Welcome to HCM. Based on the facts of your query,You seem to suffer from-Post-Friction Balanoposthitis from the burst out -HPV papules on the glans/coronal sulcus and around.Second Opinion from Dermatologist-With Gardasil anti-HPV Vaccine Shot-With Acyclovir Tabs for 4 weeks would take care of it.Tab Combiflam with Antibiotic-Septran / Moxacilin would recoup it fast.Proper Ano-Genital hygiene with Dettol Sponding and Coconut-Oil smearing would reduce its recurrence.Contact with a Followup Premium question to ME. Will appreciate your Hitting thanks and writing excellent review comments to help needy patients like you. suggestsGood Day!! Dr.Savaskar, Senior Surgical Specialist M.S.Genl-CVTS"
},
{
"id": 64052,
"tgt": "Is bump on head lasting years after a fall harmful?",
"src": "Patient: A few years ago my Son was ice skating fell and hit the back of his head. We went to the emergency room, he was released shortly after. They said he didn't suffer a concussion. He was left with a small bump that we thought would go away in time. He eventually forgot about it, but has noticed a bump in the same area. Possibly it never went away, should we be concerned about this? It doesn't hurt to the touch, he doesn't suffer from headaches. Doctor: Hi,Dear Good Evening and thanks for the query to HCM.I studied your query and understood your health concerns.The lump your son had is not harmfull as its not with any concussion then and not painful to touch even now.-Its hard and still there-indicates -that the hematoma formed then has resolved and fibrosed bump without any pain in it.Hope this would resolve your query.Welcome for any further query in this regard.Plz write a Good Review-without fail,as it would help other patients coming to HCM.With Regards.Dr.Savaskar M.N."
},
{
"id": 16582,
"tgt": "Suggest treatment for head pain and rapid heart rate",
"src": "Patient: Last night I was laying in bed and my head started hurting, then I noticed my heart was beating pretty fast and hard. I could feel it like it was going to bounce out of my chest. Now my head is killing me this morning but my heartbeat is fine. Do you know what might have happened? Doctor: Hello, Regarding your concern, I would explain that your symptoms could be related to a viral infection. Do you have fever? I recommend having some rest and taking acetaminophen for headache. Hope I have answered your query. Let me know if I can assist you further. Wish you good health. Regards, Dr. Ilir Sharka, Cardiologist"
},
{
"id": 194759,
"tgt": "What could a lump on penis be?",
"src": "Patient: My 17 year old brother says he has a lump on his penis and is very afraid its malignant however from what he is describing it seems like lipoma. He says its about the size of a corn piece and has some veins on the side showing. He said it was painless for a while but that lately he is feeling a pinching pain every now and then and that for some reason the area between his testicles and anus seems to hurt when he sits at a certain position. He is 5' 8\" and a bit over weight (I'm guessing 110 kg) Doctor: Hello, Whatever description you give here in words is not sufficient. In every case, you should consult a surgeon for a physical check-up of the lump on your brothers' penis. Hope I have answered your query. Let me know if I can assist you further. Take care Regards, Dr K. V. Anand, Psychologist"
},
{
"id": 16365,
"tgt": "How treatment should I take to get rid of jock itch and regain the original skin color ?",
"src": "Patient: Hi I suffered from jock itch ( tinea cruris ) and still am. I have been to four doctors seeking their medical help. Non of the medication ( oral medication, antifungal creams etc.) I received from them is working. I have also tried home remedies ( selenium sulphide,head and shoulders,tea tree oil etc.) and here as well nothing is working. What should I do to get rid of the fungus and get my skin (area that is affected by the fungus) back to its normal colour? (the fungus is all over my inner thighs,groin and stretches right down into the buttocks ) Doctor: Hi..dear Lockhat.., Thanks for choosing HCM.., According to your complaint..it could be.., TINEA CRURIS...it is fungal disease, recurrence more., Course of treatment is advisable.., Here U follow ..., 1) Every day 2 times bath with Ketaconazole (KETO soap)., 2) Apply cream...morning QUADRIDERM.,(Fulford).., Night times ....EBERCONAZOLE 1% (Reddy's lab-EBERNET)., 3) Tab..TERBINAFINE 1% ( ZIMIG-gsk) daily for 15 dys.., 4) Clotrimazole Dusting powder...(CANDID-Glenmark)., thanQ"
},
{
"id": 72897,
"tgt": "What causes right-sided chest numbness and back pain?",
"src": "Patient: Hi, I am Neichuvoto Kiso from Dimapur, Nagaland ; India and am 41 years old. I have an inflamation type/numbness on the right side of my chest just below the ribs, this sometimes led to back pain. I have visited the doctor recently and he advised for endoscopy which was done and found that there is an ulcer in my stomach (entry part) for this he has prescribed medicine (Pylokit and Rekool) for 1 and half month. Can I know what is my actual sickness and what should be the best medicine for its treatment. It has already been 5 days but still that feeling of inflamation/numbness is there. Or is it that i go for further treatment. If so which hospital/doctor will be the best as per my place or Neighbouring states. Kindly advice. Doctor: Thanks for your question on Healthcare Magic.I can understand your concern. Stomach ulcer can not cause right sided chest pain and numbness. By your history and description, possibility of costochondritis (inflammation of rib cartilage) is more likely.So avoid movements causing pain. Avoid heavyweight lifting and strenuous exercise. Apply warm water pad on affected areas. Take simple painkiller like paracetamol or ibuprofen. Don't worry, you will be alright with all these. Hope I have solved your query. I will be happy to help you further. Wish you good health. Thanks."
},
{
"id": 155526,
"tgt": "What happens when mood stabilizers and anti-psychotic medication are stopped?",
"src": "Patient: What happens when you stop taking the mood stabilizers and anti-psychotic medication and starting menopause with no hormone trxeatment due to breast cancer (ductal incarcineria) and on ahigh doses of pain medication. I have stop taking my bi-polar due to loss of health insurance and can not afford to pay full cost of bi-polar meds Doctor: Thanks for your question on HCM. If you are on mood stabiliser and antipsychotic drugs and you suddenly stop all these, there will be withdrawal symptoms. The symptoms are1. Mood swings from hypo mania to hyper mania2. Hallucinations3. Negative thoughts4. Palpitations5. Headache and giddiness6. Too much hunger or loss of hunger7. Involuntary bowel habits.8. Violent behaviour etc.And you are also in menopause. So this can also affect your cognitive function if you stop these drugs suddenly. So better to continue these drugs. Consult psychiatrist and ask him for cheaper alternatives."
},
{
"id": 148699,
"tgt": "Bumps at the base of skull on either side of opening. Cause and treatment?",
"src": "Patient: I have two swollen like sore bumps at the base of my skull on eiher side of the spine opening. I get stiff and soreness in my next with unbearable pressure and pain. If I have my husband rub these bumps real HARD with real force - almost unbearable, after several minutes of this the pain subsides for a little while, but it will be back. What can be causing this? Is my course of treatment dangerous? Please help. Doctor: Hi I would have advised not to follow this course of treatment. Unless reason of swellig is known it is better to avoid rubbingPlease go for MRI of local area including brain and take the opinion of neurosurgeon Take care Dr LalPsychiatrist"
},
{
"id": 112433,
"tgt": "back and shoulder problem, left pinched nerve, done evasive and fusion surgery",
"src": "Patient: hello,I have had back problems for the past 7 to 8 years. I also have had problems with both my shoulders for the past 10 years. I hurt my shoulders playing softball when I dove for a ball and came down hard fully extended with my arms and hurt both shoulders. my back problems are from a degenerative disc on L5 S1. I have had two minimal evasive surgeries on my back but to no avail, so recently I had a back fusion surgery done which fully resolved my problems with pinched nerves on the left side but now I am having worst pinched nerve symptoms on my right side. my question is could there be any way possible that my pinched nerve problems in my back are related to my pinched nerve symptoms in my shoulders? Doctor: hi i had gone through your query and understand your concerns. i would come up with the possibility of CERVICO BRACHIAL NEURALGIA due to CERVICAL SPONDYLOSIS.yes the pain in the shoulders due to pinched nerves in the cervical spine like lumbar spondylosis.the problem caused by degenerative changes.treatment option;- i advise you taking Homeopathic medicines gives safe and permanent cure for your complaints -without treating the underlying cause even number of surgeries for lumbar or cervical regions cant give you cure without stopping the degenerative changes.-consult your local homeopathic physician for correct diagnosis of the case and remedy to fit your complaints I hope this is helpful for you, thank you"
},
{
"id": 63486,
"tgt": "What causes a lump on the forehead?",
"src": "Patient: I am 18 and I have a lump on my forehead. On the right side and the left side. t feels like a bone. I never had it before and it doesn t hurt or anything but it s very noticeable. I wanted to know what it is and how I could get rid of it and get my old flat forehead back? Doctor: Hi, dearI have gone through your question. I can understand your concern. You may have some soft tissue tumor like lipoma or neurofibroma. Or you may have dermoid cyst. You should go for fine needle aspiration cytology or biopsy of that lump. It will give you exact diagnosis. Then you should take treatment accordingly. Hope I have answered your question, if you have doubt then I will be happy to answer. Thanks for using health care magic. Wish you a very good health."
},
{
"id": 159999,
"tgt": "Black dot on neck with a red bump under it",
"src": "Patient: Hi I just noticed like a day or two ago I have a black dot on my neck.It has a red bump under it though but it doesn't itch.I'm a little worried.I do have freckles and have been in the sun the last few weeks... Doctor: Hi Colleen, Welcome to HCM. Most likely it is nothing to worry about, since it is not causing any problem such as itching or pain. Just wait for about a Weeks time and see if it goes away of it's own. If it is persisting it would be a better idea to consult a dermatologist and let him examine the lesion and advise proper treatment for that condition. Wishing you all the best."
},
{
"id": 7562,
"tgt": "Acne scars, loss of self-confidence, hesitant nature, communication difficulty. Treatment options?",
"src": "Patient: Hey , i m 16 years old , i have a problem communicating with other people . I don t talk to people a lot , i m a very shy person , i don t go out a lot .. I usually sit at home alone in front of my computer .. This wasn t serious for me until now , i lost my self confidence (i have acne scars which makes me even less confident about myself) , i became very hesitant , i m afraid of going out and talking to people and showing my face .. I m not sure if what i m saying is clear , because i have an idea but i don t know how to tell anyone ! Doctor: Hi, Thanks for writing in. Acne is caused by hormonal imbalance. Hormonal work up may be necessary. Blockage of sebaceous glands causes papules and pustules.White substance is secretion of sebaceous glands. Acne is treatable now.With correct treatment all your pimples , scars and dark spots can be treated. To start with you have to apply Brevoxyl cream twice daily . Also you can use anti acne face wash like Brevoxyl face wash. Avoid any hair oil to hairs.Avoid any other moisturizers or cosmetics. Use good quality sunscreen like SUNCOTE gel daily while going out. Discuss the option of Isotretinoin with your dermatologist.It will have rapid effect. There is nothing to be depressed about.Once you start medication there will be rapid effect and you can mix with people with confidence. Hope this helps. Best of luck. Regards. dr Sudarshan. MD dermatology."
},
{
"id": 192674,
"tgt": "What does sperm count nil mean in an semen report?",
"src": "Patient: hi my semen report says. Volume 2.0ml Reaction ALKALINE Liquefaction Time NOT IQUEFIED WITHIN 30 MIN. INCUBATION. SPERMS COUNT NIL ACTIVE (Semen) No MOTILE OR NON MOTILE SPERMS SEEN. PUC CELLS 3-4/HPF rbc Semen NOT SEEN. IS MY REPORT NORMAL if no then plz provide the solution Doctor: Hello, There are no sperm cells detected in the study. You might be having a condition called azoospermia. You can repeat the test one more time before confirming the diagnosis. Conditions like retrograde ejaculation and spinal problems must be ruled out. A detailed evaluation is required. Consult a urologist and he will direct you accordingly. Hope I have answered your query. Let me know if I can assist you further. Take care Regards, Dr Shinas Hussain, General & Family Physician"
},
{
"id": 77148,
"tgt": "What causes chest pain and breathing difficulty?",
"src": "Patient: Hi, i'm 33 yrs old male, my heart beat is 90, having a little chest pain for 2 days now, my blood preasure is 105/75, and today i had difficulty breathing well in the morning, i wake up breathing very fast like i was running, i'm overweighted, is this normal? Doctor: Hi welcome to health care magic....Noted chest pain since two days...You are overweight ...It can lead sleep apnea and dyspnea sometime....Weight should be reduced...But let me know you do you have fever???Cold history ???? Constricting pain ???Smoking pain ???Anyhow here if constricting pain with sweating ECG or eco donectp rule out ischemic heart disease....Chest x ray done to rule out respi pathology like bronchitis or pneumonia if fever present...Spirometry PFT useful for diagnosing unknown asthma or other restrictive disease...If no above causes present rest done and simple analgesic taken.....As it might muscular strain.....Take care.......Dr.Parth goswami"
},
{
"id": 164424,
"tgt": "Does Azithromycin help child suffering from enlarged tonsils and fever?",
"src": "Patient: Hi ! My daughter aged 6 yrs weighing 25kgs, suffered from otitis media on 29th jan 2011. The doctor prescribed Secef which I gave for 6 days, but her tonsils were still enlarged and red. I took a second opinion and the new doctor(another ENT specialist) prescribed Augmentin DDS (5 ml twice a day for 6 days). This medicine I gave her from 2/Feb to 8/Feb but her tonsils are still enlarged. Now she has put her on Syp Allegra for 10 day(5 ml twice a day) Despite all this her cough is persistant, tonsils are still enlarged and today she developed fever(100.9F) Now the doctor wants me to start Azithromycin 200. What should I do?? How many antibiotics am I to give?? Doctor: azithromycin is useful in tonsillitis, but use salt warm water gargle or betadine mouth wash, which will help in reducing swelling, some atypical cases might take some time to respond to antibiotics."
},
{
"id": 90351,
"tgt": "What causes stomach cramps?",
"src": "Patient: Hi,I'm 36 weeks and have woken up in the middleOf the night a few times over the last couple of days With stomach cramps not like period pains but likeWhen u have a banana and go to bed,My baby is still moving around happily.It doesn't happen all the time , could this be braxton hicksContractions? Or is it something to worry about Doctor: Hi.Thanks for your query.This is obviously the contractions as it is 36 weeks.I think you should report to the hospital. There is nothing to worry about. All the best for Motherhood."
},
{
"id": 188294,
"tgt": "Child has painful white spots with pain around the dental caps. Prescribed Tylenol\\codeine but no relief. Suggestions?",
"src": "Patient: On Monday my 5 yr daughter had caps put on her teeth. Since yesterday shes has been complaining of pain. So last night I looked at them and they are swollen with white spots. I called the dentist and he said it sounded like ulcers and he just refilled her medicine from when she first had the procedure done. that just stops the pain and makes her drowsy (Tylenol\\codeine) She only complains and cries when she has to eat or drink something so she barley eats... What can do to help her? I've tried salt water. But nothing seems to help ease the pain. Doctor: Hello and welcome to HCM,Appearance of painful swollen lesion with white spots after putting on the dental caps is suggestive of ulcers in the oral mucosa.Drowsiness may be due to codeine that your dentist has prescribed for the pain.Painkillers will relieve the pain for some time, however an examination of the oral mucosa is required.The dental cap could be causing the problems.Re-fitting of the cap may be required to prevent the ulcers.In the mean time, you can apply local anesthetic eg lignocaine ointment to prevent the pain during eating food.Thanks and take careDr Shailja P Wahal"
},
{
"id": 183692,
"tgt": "Suggest remedy for cyst on tongue",
"src": "Patient: I have a cist in my tough and it keeps getting bigger now I cant eat and I am now having trouble drinking liquids I went to a urgent care but all they did was give me some pills I do have appointment next week with a ENT Doctor but I don t think this thing is going to wait Any help would be great Thanks Greg McCormick Doctor: Thanks for your query, I have gone through your query.The lesion on the tongue could not be a cyst, it can be a traumatic fibroma or lipoma. Consult a oral physician and get it ruled out. If it is fibroma or lipoma, then consult a oral maxillofacial surgeon and get it surgically removed. If the lesion is on the floor of the mouth it can be a mucocele, then it can be surgically enucleated.I hope my answer will help you, take care."
},
{
"id": 76913,
"tgt": "What causes hot flushes with chest pain?",
"src": "Patient: I was diagnosed with blood pressure last week. I am for a week on bilocor 5mg and still get headaches. I get like hotflushes all of a sudden as well since the use of the pills. Two days later I had a terrible pressured like pain in my chest, which lasted for a few minutes. This is the 4th time I have been having these pains in this year. An ECG shows nothing. Please advise what you think it could be. Doctor: Hi and thank you for choosing HCM to post your question.I read carefully your question and understand your concern.It would be better to know about your gender,age,past medical history,smoking status,if you are using other medications ,ect.Bilocor is a beta blocker used for hypertension and hotflashes is a less frequently side effect of it.Just one week of treatment is not enough, so you need to continue treatment and see the results.During this time you need to check frequently blood pressure.If your high blood pressure persists, you need to talk to your doctor and do changes in treatment.Chest pain can occur as a result of cardiovascular,pulmonary,pleural,musculosceletal disease or anxiety states.In the setting of high blood pressure ,you need to exclude cardiac origin of your chest pain.If I was your treating doctord I would suggest to do graded exercise stress testing in conjunction with echocardiography to exclude coronary artery disease and other cardiac causes.A chest x ray is necessary to exclude pulmonary and pleural causes of chest pain.If you suffer from asthma or chronic obstructive pulmonary disease,bilocor can agravate your condition and can cause chest pain.In this case betablocker such as bilocor are contraindicated.I hope my opinion is helpful to you.Wish you good health.Kind RegardsDr .Dushi"
},
{
"id": 204729,
"tgt": "How can PTSD along with severe depression be treated?",
"src": "Patient: What is a good choice of medication for a teen newly dx w ADD who has been previously dx w PTSD and major depression and is currently taking 100mg Zoloft and 2mg Abilify? Also is there a non-stimulant available for a 10 y/o (parents are divorced and father is a former addict)? Doctor: Hi,Welcome to health care magic,I have gone through your query and understand your concern towards your symptoms.I would like to advise you to start atomoxetine, which is a non stimulant, and can escalate the dose of zoloft along with this.But, discuss with your doctor regarding these drugs.Thank you."
},
{
"id": 37236,
"tgt": "What causes dampness with foul odor in the infected thumb?",
"src": "Patient: I have a friend who broke his thumb. He is into the second cast and it has been over three weeks now. It has gotten damp/wet inside and has a foul smell. He refuses to have it replaced, despite the recommendation of his caregivers. He is concerned with the addition copay of $50.00. What will/has happened inside the cast that would cause more damage to his skin or the broken bone, if anything? Is it really necessary to have a new cast. Doctor: Hi,Case of your friend requires evaluation of his case as soon as possible.There might be having some bacterial infection in his thumb and most important is risk of having infection to bone can give rise to osteomyelitis of thumb phalanx.consult his doctor and get examined after removing cast.Ok and take care."
},
{
"id": 187695,
"tgt": "Will wearing retainers cause teeth to buck once again?",
"src": "Patient: So, I had braces for approximately 3 and a half years, the reason why I got said braces was because of my tonsils.Well, when I was about..10 or so, I got them removed and got braces due to the overbite because of the tonsils causing me to breathe wrong, and I also had a small jaw, as a child.Well, after I had gotten my braces removed, I wore my retainer for a month (it was lost, and never found), and also had about four to five teeth removed before I even got braces.Will this cause my teeth to buck once again? Or should I not be worried? A small overbite is fine, but I'd rather not look like a \"beaver\" again.Will my teeth shift back to the Doctor: Hello, Welcome Thanks for consulting HCM, You are wearing braces and then retainer from few years , and for that Orthodontic treatment you had been undergone through extractions of your few teeth , fine You have mentioned that you have problem of Overbite, and small jaw , dont worry much , I will suggest you that if you are wearing braces, had extractions have retainer also then go for Complete Orthodontic treatment , dont leave it in between latter on if you have increased Overbite then you can have problem of attrition , senstivity, many more so it is better you go for orthodontic treatment. Hope this will help you."
},
{
"id": 214942,
"tgt": "Do pudina leaves help in shrinking of uterus fibroid ?",
"src": "Patient: Pudina leaves helps in shrinking of utres fibroids Doctor: Hi, Welcome to Healthcare magic forum, Medically there are no studies done for pudina shrinking fibriods, My suggestion is consult ur gynaecologist if u have problem in menstrul cycle, planning to get pregnant. The effect of fibriod depends on the location and size of it. If it is small fibriods not giving troble to ur periods then u can ignore it. Hope I have answered ur question, Regards"
},
{
"id": 130932,
"tgt": "What causes pain in my outer thigh when i stand up?",
"src": "Patient: I have s sharp pain in a muscle in my outer thigh. It feels like a bruise but I know that I havn t bumped or hurt it. I ts not painful when I exercise but I can feel it when I stand up. I am not aware of doing anything that has damaged it. despite pain medication and heat it is getting worse. 2-3 days now. Doctor: In my opinion you should do hip x ray to check for hip osteoarthritiship pain show on the side and front of hip as a painfull pointGood Luck"
},
{
"id": 106790,
"tgt": "What causes severe backache?",
"src": "Patient: Hello, my 46 year old husband has been having posterior flank pain in the thoracic area. He recently has had some kidney issues , high creatinine due to probable excessive ibuprofen use( due to shoulder pain). He is a construction worker who is in good shape for his age. He had seen a urologist and a nephrologist during this time. They did diagnose crystals and some stones but no treatment other then stopping the ibuprofen use. He also has had a laminectomy about 15 years ago followed by the following year of spinal decompression. This pain now is higher and unlike the type of back pain he has experienced. He hasn\u2019t seen a chiropractor for a few years and is scheduled tomorrow for an evaluation and/ or adjustment. I am a massage therapist retired and did work on his posterior muscle groups but hesitated because I felt if this was related to the kidneys I didn\u2019t want to complicate matters. In your opinion, what\u2019s the most likely cause? Thanks for your consideration, Lou Doctor: He may have a further issue with disc degeneration or arthritis higher than his previous laminectomy surgery.If his symptoms are not improving, I would get MRI imaging to get this evaluated further."
},
{
"id": 87838,
"tgt": "What does abdominal pain with fever and nausea indicate?",
"src": "Patient: im haveing alot of abdominal pain below my belly on both sidim haveing alot of abdominal pain below my belly on both sides its buring blowed stabbing feeling worsens when i move ive had slight fever and ive had nausea for awhile now what could it be and what should i do? Doctor: Hi! Good evening. I am Dr Shareef answering your query.If I were your doctor, I would advise you for a urine routine,microscopic and culture sensitivity test to rule out a UTI. In addition, I would go for some routine blood tests like a CBC, blood sugar, and also an ultrasound of the abdomen if thought necessary after a clinical examination of your abdomen to rule out any kind of other intra abdominal pathology, like inflammation/infection of any of the organ therein. In case of a female patient, would also think of a gynaecologist consultation to rule out a PID. Further management would depend on the clinical findings and reports of the investigations noted. Till then, you could go for an anti spasmodic drug for a symptomatic relief.I hope this information would help you in discussing with your family physician/treating doctor in further management of your problem. Please do not hesitate to ask in case of any further doubts.Thanks for choosing health care magic to clear doubts on your health problems. I wish you an early recovery. Dr Shareef."
},
{
"id": 137771,
"tgt": "What causes pain and swelling in the knuckle joints?",
"src": "Patient: Son has swollen knuckles middle joint, seems to. Ache a lot. Nothing terribly serious but after an activity, he wears out quicker than most and complains of minor joint pain in his ankles a arms. Tonight he said he has some shaking in his hands. He has Aspergers so I know he gets anxious but don t want to miss something. Also he has stretch marks from rapid growth spurts. Any direction I should look into? Thank you Doctor: Dear Patient, Welcome, and thanks for sharing your concern I went through your query, and I feel, the most important thing that you need to rule out, is inflammatory arthritis, and for that you need to get his investigations done, get his CBC, ESR, CRP, ACCP RF done and see if any one is abnormal and indicative of pathology, if not than stay cautious and repeat these after a period of 1 year.I hope my advice would have been useful, in decision making regarding your treatment, still if you have any clarifications or doubts feel free to contact back.Thanks"
},
{
"id": 217295,
"tgt": "What causes cramping pain in the feet?",
"src": "Patient: hi can i ask you about pains i have been getting in the soles of my feet, mainly in my right foot. it is like a cramp when i rest or stand for a long time. i have had it for the last month and it is sore. i am quite young so i feel that i shouldn t be getting these pains. could you possible tell me what could be causing these cramp pains? Doctor: U should wear soft shoe.chek serum fbs ppbs.follow up with the reports.what is ur age.does the pain resolves with rest?"
},
{
"id": 125561,
"tgt": "What causes pain in the side of the palm?",
"src": "Patient: Hi dr samuel..the side of my palm felt pain after playing badminton..which is baby finger side of my palm..any way to prevent it? Or what causes it? Im sure that im holding and playing in the right way..but this happen everytime after i played badminton...it takes 2-3 days to recover everytime. Doctor: Hello, Most probably it will be due to minor contusion or sprain. As of now you can use analgesics/anti-inflammatory combination like aceclofenac/serratiopeptidase for symptomatic relief. If symptoms persists better to consult an orthopaedician and get evaluated. Hope I have answered your query. Let me know if I can assist you further. Regards, Dr. Shinas Hussain, General & Family Physician"
},
{
"id": 31108,
"tgt": "What causes blisters in the mouth?",
"src": "Patient: Clinical FindingsSite and nature of specimen: recurrent ulceration over tongue biopsy for H P EGross appearance: tinny soft tissue bit 0.5cmMicro. Appearance: Revealed hyperplastic and dysplastic stratified .......................... squamous epithelium underlying it there are inflammatory cell collections, endarteritic micro vessels and striated muscle fibers.No evidence of malignancyOpinion Hyperplastc and Dysplastic epithelium with non-specific inflammation Throat swab for culture and sensitivity and Sensitivity Report: SW175/10Microscope Findings:Organism Isolated: Coagulase positive staphylococci Staphylococci grown in culture,ANTIBIOGRAMAntibiotic\u00a0\u00a0\u00a0\u00a0\u00a0\u00a0\u00a0\u00a0\u00a0\u00a0\u00a0\u00a0\u00a0\u00a0\u00a0\u00a0\u00a0\u00a0\u00a0\u00a0\u00a0\u00a0\u00a0\u00a0\u00a0Sensitivity/resistant1. Peniciline\u00a0\u00a0\u00a0\u00a0\u00a0\u00a0\u00a0\u00a0\u00a0\u00a0\u00a0\u00a0\u00a0\u00a0\u00a0\u00a0\u00a0\u00a0\u00a0\u00a0\u00a0\u00a0\u00a0\u00a0\u00a0R2. Streptomycin\u00a0\u00a0\u00a0\u00a0\u00a0\u00a0\u00a0\u00a0\u00a0\u00a0\u00a0\u00a0\u00a0\u00a0\u00a0\u00a0\u00a0\u00a0\u00a0\u00a0\u00a0\u00a0\u00a0\u00a0\u00a0++++3. Tretracyclin\u00a0\u00a0\u00a0\u00a0\u00a0\u00a0\u00a0\u00a0\u00a0\u00a0\u00a0\u00a0\u00a0\u00a0\u00a0\u00a0\u00a0\u00a0\u00a0\u00a0\u00a0\u00a0\u00a0\u00a0\u00a0+++norf4. Erythromucin\u00a0\u00a0\u00a0\u00a0\u00a0\u00a0\u00a0\u00a0\u00a0\u00a0\u00a0\u00a0\u00a0\u00a0\u00a0\u00a0\u00a0\u00a0\u00a0\u00a0\u00a0\u00a0\u00a0\u00a0\u00a0R5.Chloromycetin\u00a0\u00a0\u00a0\u00a0\u00a0\u00a0\u00a0\u00a0\u00a0\u00a0\u00a0\u00a0\u00a0\u00a0\u00a0\u00a0\u00a0\u00a0\u00a0\u00a0++++6. furadantin\u00a0\u00a0\u00a0\u00a0\u00a0\u00a0\u00a0\u00a0\u00a0\u00a0\u00a0\u00a0\u00a0\u00a0\u00a0\u00a0\u00a0\u00a0\u00a0\u00a0\u00a0\u00a0\u00a0\u00a0\u00a0-7. septran\u00a0\u00a0\u00a0\u00a0\u00a0\u00a0\u00a0\u00a0\u00a0\u00a0\u00a0\u00a0\u00a0\u00a0\u00a0\u00a0\u00a0\u00a0\u00a0\u00a0\u00a0\u00a0\u00a0\u00a0\u00a0++8. wintomylon\u00a0\u00a0\u00a0\u00a0\u00a0\u00a0\u00a0\u00a0\u00a0\u00a0\u00a0\u00a0\u00a0\u00a0\u00a0\u00a0\u00a0\u00a0\u00a0\u00a0\u00a0\u00a0\u00a0\u00a0\u00a0-9. ampicillin\u00a0\u00a0\u00a0\u00a0\u00a0\u00a0\u00a0\u00a0\u00a0\u00a0\u00a0\u00a0\u00a0\u00a0\u00a0\u00a0\u00a0\u00a0\u00a0\u00a0\u00a0\u00a0\u00a0\u00a0\u00a0R10. furoxone\u00a0\u00a0\u00a0\u00a0\u00a0\u00a0\u00a0\u00a0\u00a0\u00a0\u00a0\u00a0\u00a0\u00a0\u00a0\u00a0\u00a0\u00a0\u00a0\u00a0\u00a0\u00a0\u00a0\u00a0\u00a0-11. gentamycin\u00a0\u00a0\u00a0\u00a0\u00a0\u00a0\u00a0\u00a0\u00a0\u00a0\u00a0\u00a0\u00a0\u00a0\u00a0\u00a0\u00a0\u00a0\u00a0\u00a0\u00a0\u00a0\u00a0\u00a0\u00a0++++12. kanamycin\u00a0\u00a0\u00a0\u00a0\u00a0\u00a0\u00a0\u00a0\u00a0\u00a0\u00a0\u00a0\u00a0\u00a0\u00a0\u00a0\u00a0\u00a0\u00a0\u00a0\u00a0\u00a0\u00a0\u00a0\u00a0-13. phexin(cephalexin)\u00a0\u00a0\u00a0\u00a0\u00a0\u00a0\u00a0\u00a0\u00a0\u00a0\u00a0\u00a0\u00a0\u00a0\u00a0\u00a0\u00a0\u00a0\u00a0\u00a0R14. norbactin(norfloxacin)\u00a0\u00a0\u00a0\u00a0\u00a0\u00a0\u00a0\u00a0\u00a0\u00a0\u00a0\u00a0\u00a0\u00a0\u00a0R15.ceporan\u00a0\u00a0\u00a0\u00a0\u00a0\u00a0\u00a0\u00a0\u00a0\u00a0\u00a0\u00a0\u00a0\u00a0\u00a0\u00a0\u00a0\u00a0\u00a0\u00a0\u00a0\u00a0\u00a0\u00a0\u00a0-16. ciprofloxacin\u00a0\u00a0\u00a0\u00a0\u00a0\u00a0\u00a0\u00a0\u00a0\u00a0\u00a0\u00a0\u00a0\u00a0\u00a0\u00a0\u00a0\u00a0\u00a0\u00a0++++\u00a0\u00a0\u00a0\u00a0\u00a0\u00a0\u00a0\u00a0\u00a0\u00a0\u00a0\u00a0\u00a0\u00a0\u00a0\u00a0\u00a0\u00a0\u00a0\u00a017.pefloxacin\u00a0\u00a0\u00a0\u00a0\u00a0\u00a0\u00a0\u00a0\u00a0\u00a0\u00a0\u00a0\u00a0\u00a0\u00a0\u00a0\u00a0\u00a0\u00a0\u00a0\u00a0\u00a0\u00a0\u00a0\u00a0++18. amikacin\u00a0\u00a0\u00a0\u00a0\u00a0\u00a0\u00a0\u00a0\u00a0\u00a0\u00a0\u00a0\u00a0\u00a0\u00a0\u00a0\u00a0\u00a0\u00a0\u00a0\u00a0\u00a0\u00a0\u00a0\u00a0++++19.cefotaxime\u00a0\u00a0\u00a0\u00a0\u00a0\u00a0\u00a0\u00a0\u00a0\u00a0\u00a0\u00a0\u00a0\u00a0\u00a0\u00a0\u00a0\u00a0\u00a0\u00a0\u00a0\u00a0\u00a0\u00a0\u00a0++++20.cefatriaxone\u00a0\u00a0\u00a0\u00a0\u00a0\u00a0\u00a0\u00a0\u00a0\u00a0\u00a0\u00a0\u00a0\u00a0\u00a0\u00a0\u00a0\u00a0\u00a0\u00a0\u00a0\u00a0\u00a0\u00a0\u00a0++++21.netilmicin sulphate usp\u00a0\u00a0\u00a0\u00a0\u00a0\u00a0\u00a0\u00a0\u00a0\u00a0\u00a0\u00a0\u00a0\u00a0\u00a0- (netromycin inj :im/iv)(R)resistant\u00a0\u00a0\u00a0\u00a0\u00a0\u00a0\u00a0\u00a0\u00a0\u00a0\u00a0\u00a0\u00a0\u00a0\u00a0(+)sensitive\u00a0\u00a0\u00a0\u00a0\u00a0\u00a0\u00a0\u00a0\u00a0\u00a0(-)drug not used\u00a0\u00a0\u00a0\u00a0\u00a0\u00a0\u00a0\u00a0\u00a0\u00a0\u00a0\u00a0\u00a0\u00a0\u00a0\u00a0\u00a0\u00a0\u00a0MEDICINE BEING USED1.TELMA-40mg-01 DAILY2.AZTOR EZ-01 DAILY3.PREGAMET 75mg 02 (mor-eve)4. LEAZI-GPS \u2013 1.5 DAILY5.GLYCOMET GP \u2013 I.5 DAILY (MOR-1 EVE-HALF)6. TRYPEAN 10mg \u2013 02 (MOR-1 EVE-1) 7.ECOSPIRIN 75mg \u2013 01 daily8.ALPAZ 1mg \u2013 01(NIGHT)9.ORSPASTE10. CIPSLEX \u2013 10mg used for 10 years and discontinued for the past 2 months 11. Using the medicine for Diabetes and High BP form last 4 years \u00a0\u00a0\u00a0\u00a0\u00a0\u00a0\u00a0\u00a0\u00a0\u00a0\u00a0\u00a0\u00a0\u00a0\u00a0\u00a0\u00a0\u00a0\u00a0\u00a0\u00a0\u00a0\u00a0\u00a0\u00a0complaints1.blusters in mouth recurring2.burning of tongue and roughen of tongue Doctor: Hello,The microorganisms found under microscopic study, Staphylococci, are treated best with Cefadroxyl 500 mg (Drxoxyl) twice a day for 5 days. It is safe to be taken with all the medicines you are taking regularly and it will not interact with any of them.You can apply Triamcinolone Acetonide 0.1% (Kenacort) gel topically on the blisters and ulcers 3-5 times a day. Avoid spicy food for some days. Drink lots of fluids. I hope this information is useful to you. Thank you for choosing HealthcareMagic. Best,Dr. Viraj Shah"
},
{
"id": 63740,
"tgt": "What causes lump between vagina and crease of leg?",
"src": "Patient: I have a lump thaat has suddenly appeared between my vagina and the crease of my leg. I felt tremendous pressure last evenin befor I went to bed and this morning found the lump. It is as big as quarter, deep under the skin, I almost thought of a hernia. The only thing different in my life is tha I had shoulder surgery 7 days ago. Any Ideas? Doctor: Hi, dearI have gone through your question. I can understand your concern. You may have inguinal hernia or enlarged inguinal lymphnode. You should go for examination. If needed go for ultrasound study. If it is lymphnode or soft tissue tumor then you should go for fine needle aspiration cytology or biopsy of that lump. If it is hernia then go for surgery. Consult your doctor and take treatment accordingly. Hope I have answered your question, if you have doubt then I will be happy to answer. Thanks for using health care magic. Wish you a very good health."
},
{
"id": 160454,
"tgt": "Suggest remedy to cure pneumonia in infant",
"src": "Patient: hi Doctor my son who is 8 month old he got Phenomonia when he was 1 month old now he has allergie and ashthama when ever he gets such problem his nose become blocked so i consult with child specilist they prescribe Antibiotic and other cough and flue medicine palease suggest me how could i save my son from this disease which would not effect my son life in long run for e.g long run ashthama tnx Doctor: Hi, It is common to have asthma at this age, but the symptoms will gradually disappear as long as you child grows up. Take care. Hope I have answered your question. Let me know if I can assist you further. Regards, Dr. Salah Saad Shoman, Internal Medicine Specialist"
},
{
"id": 202230,
"tgt": "What does big mass in left scrotum indicate?",
"src": "Patient: I have just discovered that I have in my left scrotum a mass that is just as big as my testicle. It now appears that I have 3 testicles in my scrotum. My last memorable check was 27 years ago and i discover a liquid sac and was told that it was not a problem. Doctor: could be fluid collection, could be hernia, could be a tumor.you need to get a scrotal ultrasound asapPlease rate 5 Stars! I strive to provide the best answers to your questions."
},
{
"id": 41668,
"tgt": "Is injection necessary when follicle size is 18mm on 7th day?",
"src": "Patient: Hi I had a follicle of size 18mm on day 7th,. Is it necessary for taking injections for follicle size and is it also necessary for trigger? Without these injection is there any natural way to increase the egg size?My age is27, weight is 75kg,height is 5.3 .I got all my tests(blood tests7ultrasounds) cleared getting nothing wrong & all is fine. Considering my case, when will I be ovulating approx? Kindly reply. Doctor: Hi welcome to healthcaremagic.I have gone through your question.18 mm follicle size is about to rupture within 2/3 days, ad follicles generally rupture at 20/22 mm size, but you had this size on 7th day is slighly contradictory.I would advise to do follicular scan to follow up for follicular rupture.Ideally at this size no injection is required to rupture follicles. But if there is tendency of follicles converting to cyst in past cycles then it is necessary.So consult gynecologist for further treatment.Hope i answered your question.Would be happy to help you further.Take care."
},
{
"id": 106894,
"tgt": "Why are pain medications not prescribed for back pain?",
"src": "Patient: maybe, i have had digerrative disck disease, my wjole like, i fell off a 2 story building twice, plus multipul ectt.. u get the picture. lasyt year in dec i was worling full time had a great dr. and after he left the new guy wouldsnt talk to me, i went from 380mg ms contin and 10/325 3 times a day, why are all of the dr. so scaref to give out pain meds to thosew who need Doctor: hipain medicines cause harmful effects on liver and kidneys.so generally doctors avoid prescribing them for longer periods of time.thanks"
},
{
"id": 97096,
"tgt": "Can we go for resuscitation despite of DNR order?",
"src": "Patient: Looking to bring my 85yr old mom to traditional med ms for dx and poss tx. She has only seen a naturopath in the last 20 yrs she is very paranoid and has hallucinations. Takes multiple supplements for lupus Rx by naturopath. Not wandering as yet. At no awake and belligerent at times. She lives with my sister in an addition attached to the house all one level. Thinks the public comes in and steals her toilet paper so she hoards it in her closets and under the kitchen table. I am concerned because in the event of her death that she will be recusitated despite the fact that she does not want this she does not have a portable DNR order at her house. Any suggestions.? Thanks so much for your time. Doctor: Hello, I hope I can help you with your problem. The rules surrounding your question may vary state by state, but the essential idea behind these laws is the same. That is, if a person has a DNR order, and a hospital is aware of it (they have the document), then the patient cannot be intubated. Now, that is not to say that it does not happen, and usually people will err on the side of caution by intubating everyone.As such, you need to do all you can to make sure that you take care of this situation before it becomes a problem. Usually you can speak with a doctor about the appropriate steps to take. Usually this means that one of the other family members will need to be legally responsible for your mother if she is not able to make her own decisions.The issue of medical competence is not something for a doctor to decide, but a court, and that is where your issue becomes tricky and it really depends on state law. If she has drafted such a document in the past, you need to do all you can to get one. If not, you may need to talk to an attorney as well.I hope this helps."
},
{
"id": 64476,
"tgt": "Suggest treatment for a lump on eye",
"src": "Patient: Hi recently about a month ago a bump showed up above my eye between my brow and lid. I was wondering if it could be from a sinus infection or something? its really ugly i'd like it to go away. also around the same time my left ear become muffled or clogged and is still the same. im not sure if they have anything to do with each other but i thought id mention it anyways Doctor: Hi,Dear,Good evening.Thanks for your query to Health Care Magic.I studied your query in-depth. In my opinion the lump on eye-mostly is due to Chalazion-or meibomian cyst,epidermoid cyst of the lid , or could be orbital dermoid presenting there,or could be Dacryo-cystitis lump/ or could be sebaceous cyst/or could be Pilomatrixoma?Treatment -will depend on the diagnosis.If its a lump with discrete edges-it could be-EXcised with Biopsy, and if its dacryoscystitis-DacryocystoRhinostomy(DCR) could be done after antibiotics/Tb NSAIDs treatment control.Your clogged left ear which started same time a month ago-may be due to the -URTI with sinusitis / or with Dacryo-cystitis with URTI-where the upper respiratory tract infection could play a role in its developmental connection and could be treated by treating URTI infection- by Antibiotics /Nasal drops/NSAIDs.Hope this would clarify the worry you had.Would love to Wellcome your queries to HCM."
},
{
"id": 113003,
"tgt": "Have pain in lower right back. Ultrasound shows ovarian cyst and gallstones. Advised surgery. Recommendations?",
"src": "Patient: Hi,I had my uterus removed almost 2 years ago, my ovaries remain. After severe low right abnormal and low right back pain, I have been diagnosed with ovarian cysts. I had an ultrasound and MRI to confirm. During the MRI, they also detected gallstones. I am 48 years old, physically fit, I'm 5' 1\" and weigh about 105 pounds. After the tests, I had a physical exam with a Gyne and was told the pain was caused from my right ovary attaching itself to my body as well as my colon and I would need surgery. Is there any other possibilities and how probable is this? My regular Doc retired and it's very difficult to find a Doc that will really take the time to hear me, as they are all trying to keep their 15 minute appointment to less than 15 minutes. I really would like to know where to go to find out why I am having such pain in my lower right back. Doctor: It depends on how big are the ovarian cyst - bigger the cysts > 4cm usually - if not treated with surgical removal there is complication of cyst getting twisted / rupture inside - which is considered as acute emergency Site of pain in lower part of abdomen is due to findings of ovarian cyst on MRI as gall stone pain is more so in upper part of tummy. Following any abdominal operation - chances of adhesions will increase & that is what has probably happened having said that adhesions can't be diagnosed 100% by clinical examination as best way to diagnose & treat adhesions ( adhesiolysis ) is by laparoscopy Hope this helps"
},
{
"id": 79160,
"tgt": "Is it safe to travel while on treatment for pneumonia?",
"src": "Patient: My son has always had allergies, sinus infections. He was getting shots in High School and then quit in college and into the future. Seasonal pollen allergies are the worst. Always blowing nose and sniffles. He is 32 and a new dad with a 9 month old. He lives in Manhattan-lower east side. I do not like the air there. He rides the subway to work. We recently spent a whole week together at a rental home in Montauk Long Island. After he returned back to the city, 5 days later he had 3 days of fever spiking. Went to ER chest x-Ray showed pneumonia in one lung. They put him on Levaquin for 6 days. He got dehydrated, had to have 2 bags of IV fluid pumped into him. Had sever head ache and fever would not go down. Meningitis was suspected as he had a little stiff neck too. Is 6 days of meds enough? Should he fly to a bachelor party in 10 days? Concerned mom Doctor: Hello mam, its not about whether 6 days of medicines are enough or not, its about the severity of the disease and whether the infection is responding to the antibiotics what he is getting. If there are new things like meningitis coming up or at least suspected i would advise him against going on any travel in this condition. what he needs right now is total infection free status with appropriate and adequate dosage of the antibiotics with totally being disease free.I hope I answered your question satisfactorily.thank you"
},
{
"id": 67064,
"tgt": "What causes bump on the back of head?",
"src": "Patient: HI, about 10 minutes ago, i hit my head on my kitchen tile floor. I have a small bump on the back of my head, a bit stiffness in my neck and my right eye as a little blurry (but it normal now). I fell from a height of about 5 feet. Do I need to see a doctor? Doctor: HiWelcome to hcmYou and any of you family member should watch for unconciousness, giddiness, black eye,ear or nose bleeding or fits and should go to hospital stat. When these alarming signs develops you needs CT scan to look for bleeding or damage inside your skull/brain. As per your discription of mode of injury you dont require all this but you should be careful. If medical help is not nearby then better to get admitted for observation if you are filling giddy or blurring of vision at present.Regards."
},
{
"id": 171507,
"tgt": "What causes continuous erection and redness in tip of penis?",
"src": "Patient: My 23 month old son has had an erection for more than 8 hours (it seems). I m not certain that the erection has been constant. Each of 2 times that I ve changed his diapers his penis been erect and when I just changed his diaper now, the head of the penis was red. Should I seek medical assistance? Doctor: Hi.... by what you quote I feel that your son might be having a infection of the tip of the penis. This is called balanoposthitis. The other rare causes of constant erection could be an infection of the shaft of the penis or because of Scorpion sting. I suggest that you seek medical attention immediately.Regards - Dr. Sumanth"
},
{
"id": 42536,
"tgt": "Is the sperm count normal?",
"src": "Patient: AOA Respected doctor, I am married since last three years and have a child 2 years now. We are trying to conceive 2nd child since last two years but not getting any success. First child from operation. I have gone through my semen test and here is the report, i dont know up-to what level it is ok and what should i do now. Please help my by giving some kind advise that what should i do....1 - Quantity---------------------------------2.0ml. 2 - Colour ---------------------------------Greyish white 3- Transparency ---------------------------OPAQUE 4- Viscosity ---------------------------------Viscous 5- Time of liquification --------------------15 Min 6- ph -----------------------------------------7.5 7- Fructose ---------------------------------Positive 8- Total Count -----------------------------20 Mill/ml 9- Sperm per ejaculate ------------------90 Mill/ml 10- Percentage motile sperm -----------44% 11 - Rapid linear progression -----------10% 12 - Slow/Non lin progression ----------15% 13 - Non progressive ---------------------75% 14 - Normal form --------------------------60% 15 - Large oval -----------------------------05% 16 - Small Oval -----------------------------05% 17 - Tapering ------------------------------10% 18 - Round ----------------------------------05% 19 - Pin ---------------------------------------05% 20 - Mid piece abnormalities -------------10% 21 - WBC -------------------------------------03 22 - RBC --------------------------------------00 Please doctor guide me what to do, do i need any treatment. What kind of treatment and is this problem curable? Best Regards Ahad Lahore Doctor: hai,As per quote your semen analysis is normal with some notable parameters to discuss.from your semen report rapid progressive (forward progressive sperm)should be more than 50% (your s 10%) otherwise all other parameters are within normal. NO need to worry. your problem will be fixed with the below advise.intially no need for any medication.If your are a smoker and alcoholic kindly stop it completely.follow healthy lifestyle and food habit.add protein nourished foods like Almond,cashew,channa,soyabean,peanut in your daily food intake.Add jaggery,black gram,and dal prepared with ghee.,dates,pomegranate,fig-it will help in producing healthy sperm count. take diary product like milk,yogurt,cheese,butter etc. Add banana and tuber and leafy vegetables.Avoid stress, fried item,junk and caffeinated drinks.Do walk for 3 to 5kilometers.continue this for one month and take agian s semen analysis. if your earlier mentioned problem is solved you can have child. If its not inproving kindly visit a infertility specialist for further treatment.thank youHope i answered your query."
},
{
"id": 29920,
"tgt": "Suggest remedy for persisting cough despite medication",
"src": "Patient: Hello, I can not stop coughing. For the past five years I have always had a cough. Lately, I have been coughing more often and it is only getting worse. I have been to the md's office and even though I am prescribed medicine it is not working.my body aches from coughing so much. I cough up clear phlem and sometimes it is stained with blood. Doctor: hello,welcome to healthcaremagicideally cough should subside with the medicationbut if it is not subsiding, some investigations need to be done which are-chest x rayHRCT lungsbronchoscopy( to look for foreign body)also sputum smear for tuberculosisyou need to undergo an antibiotic course if you have not taken.further management will be planned after findings in the investigations.hope this helpsthank you"
},
{
"id": 171302,
"tgt": "What causes pain in knee and leg?",
"src": "Patient: Hello my 4 yr old daughter wakes up crying and also crys during the day due to knee/leg pain.she has had no injuries or fractures,ithis has been going on for at least 6 months.i thought it might have been growning pains but now its chronic.i need advice?? thankyou Doctor: HiWelcome to the HCMI completely understand your concerns but don't worry. This type of pains can be due to vitamin D and calcium deficiency or due to arthritis such as juvenile rheumatoid arthritis. I would recommend you to go for a proper clinical examination by orthopedic surgeon. He may advice for X Ray of involved joints, complete blood count, ESR, CRP, ANA, RA factor, 25OH D3, and calcium levels. You may upload the reports for my reference as well. This will help me to guide you in the best possible way.Take care"
},
{
"id": 68224,
"tgt": "What causes lump on chest?",
"src": "Patient: Hi, I have a brother who is currently incarcerated. When I went to visit him today, he told me about a lump on his chest. He also told me that the doctor there was giving him medication but was telling him the wrong name of the medicine. I was just wondering, aren't they suppose to do some type of exam on him to determine what it is before they just give him medication for it. The reason I ask this is because he told me that they said it was nothing that they could do. So I was wondering if they were suppose to diagnose him with something before just giving him medication? Doctor: Welcome to health care magic. 1.The history is bit inadequate, as you have not described the lump and its location on the chest, however i will try to explain you the possible causes.2.The most possible cause could be infective aetiology, hair follicular infection with collection within - which needs antibiotic in early un complicated condition, where late and complicated needs excision.3.Other possible cause will be lipomas which needs to monitored unless until symptomatic where excised.4.Finally bone lesions arise on the chest wall, and hernias. Which need to examine and do needful treatment.5.An ultrasund scan will help in the assessment of the nature of the lump, its source and extensions.6.Suggest to get examined and diagnosed and treated appropriately.7.Probably the doctor would have visually able to make out the type of lump he is dealing so started treatment. Good luck.Hope i have answered your query,any thing to ask do not hesitate to ask.http://doctor.healthcaremagic.com/doctors/dr-ganesh/62888"
},
{
"id": 49033,
"tgt": "Is longer dose of cipro required as there is an urge to urinate all the time?",
"src": "Patient: I have been on cipro for 3 days for inflamation in my bladder. The perscription is over today and although there is no burning or bleeding like there was to begin with, I feel the urge to urinate all the time and feel I need to be on medication longer. Doctor: Yes, your concern is very true.In my hospital, I usually prescribe cipro minimum for 5 days.Actually choice of antibiotic & duration of the course depends on severity of infection judged by parameters like presence or absence of fever, diabetic or not, complete blood count, number of pus cells in urine etc.Please give me your details as per above mentioned parameters, so that I can advice u further accordingly.Hope this information is satisfactory for you."
},
{
"id": 36347,
"tgt": "How to diagnose asbestosis?",
"src": "Patient: Hi, I was diagnosed with sarcodosis over a year ago by one doctor but wasn t satisfied with my treatment so i switched doctors and was told i didn t have sarcodosis after getting MRI, PET SCAN, CAT SCAN. What else can it be, i was reading up on other causes (asbestosis or mold) how could i find out. I have worked in restaurants and fuel stop. Doctor: Hello ,I understand your concern. I am Dr. Arun Tank, infectious disease specialist, answering your concern.It is highly unlikely that you are having asbestosis.Asbestosis occurs on the persons who constantly exposes to the asbestos or working in asbestos company.HRCT, is the best diagnostic tool that can diagnose the sarcoidosis and asbestosis.As you are working in the restaurant and fuel shop it is unlikely that you have asbestosis.I will be happy to answer your further concern you can contact me here or you can contact me on bit.ly/DrArunWe wish you a best health at healthcare magic. Thank you,Dr. Arun Tank"
},
{
"id": 70056,
"tgt": "Have a lump like appearance in the lower side of the abdomen",
"src": "Patient: I had my gallbladder removed 2months ago and have had no problems from that side/surgical sites. However, about 3 weeks ago I noticed a bruised feeling on the opposite lower side of the abdomen and when pushed feels like a hard lump. It's still there and its worse if pressed. Any ideas? Doctor: Hi. This may not be related to the previous surgery. The probable reason can be a fecolith, a mass, colitis or a lump worth diagnosis , may be a mass which needs urgent attention. Or simply be a hernia.Please go for a clinical diagnosis by a consultation with a Surgeon, Urgent CT abdomen, colonoscopy..."
},
{
"id": 215934,
"tgt": "What to do for the pain after plasma donation?",
"src": "Patient: i was donating plasma 9 days ago, there was slight pain during the donation. The nurse had to probe the vein to get to it. Ever since then there has been a sharp/burning pain starting at my elbow at the point of puncture, and radiating down my arm. It s very painful, and I can t straighten my car without sharp shooting pain. Now I m starting to feel little bumps along my arm. There is no swelling, redness, or discoloration. But it feels like a huge bruise, even when there isn t a stretch on the arm. Doctor: Hello and Welcome to \u2018Ask A Doctor\u2019 service. I have reviewed your query and here is my advice. You can apply Thrombophob ointments to the site of needle prick. It will provide symptomatic relief. You can also take analgesics like Ibuprofen and Diclofenac for symptomatic relief. Hope I have answered your query. Let me know if I can assist you further. Regards, Dr. Shinas Hussain"
},
{
"id": 201502,
"tgt": "Can the sexual stamina be increased to sustain for longer periods?",
"src": "Patient: Thanks actually i wants to know about my penis size an increase sex power. Doctor my penis is 6inch in long and 2 and half inch in grid. My first question is it sufficient to make my partner happy. My 2nd question is how can i increase my sex duration in bed. Last sometimes i cannt over 2/3 minutes when i fuck my partner. Doctor i ve done a long time muster bed is it for that reason. Im so much tense pls give some advice what can i do now? Doctor: Your penis size is long enough to satisfy your partner. You need not worry about it. By practicing yoga, you can delay your time of ejaculation. There are other simpler techniques also like:1. usage of condoms (Also there are condoms which contains benzocaine, which will delay your ejaculation and you can last long in bed, but regular use is not advisable)2. Giving pressure at the junction between your scrotum and anus with finger (even your partner can do this for you)3. Reduce anxiety and fears4. Do not think about anything else other than sex while doing sex.5. Pause for a while when you are about to ejaculate for 3 sec to 1 min.Even after trying these, if you are having PME, then you may require further consultation and tests.TC"
},
{
"id": 81563,
"tgt": "Suggest remedy for breathing problem",
"src": "Patient: Hi Doc, May concern towards my health are having Shortness of Breathing(SOB), got nervous or feel dizzy sometimes or let may say my heart beat runs so fast everytime I laugh too much, just like my heart is sqeezing..When I was in high school I was exempted during our P.E. because many times I m collapsed at school. During that time I was taking a medicine as what the doctor s advice then if I should not becareful I might have hole in my heart. What I am afraid of is that last June I feel exhausted & hard to breath for an hour.Please can you help me Doc with regards to this concern?Thanks,Virginia T. Niez, 35yrs. old Doctor: Thanks for your question on HCM. In my opinion we need to rule out1. Cardiac cause2. Respiratory causesfirst for your symptoms. So get done ECG and 2D Echo to rule out cardiac cause. Get done chest x ray and PFT ( PULMONARY FUNCTION TEST ) to rule out pulmonary causes. If everything is normal than no need to worry much. Your symptoms may be due to anxiety. So avoid stress and tension. Be relax and calm."
},
{
"id": 178889,
"tgt": "How can eye and eyebrow bruising and soreness be treated?",
"src": "Patient: My four year old daughter fell into a cement block today and before I was even able to pick her up the side of her eye and eyebrow were black and blue and welted. She will not let me put ice on it and I have her some Advil it s very sore to touch. Should I be concerned and seek medical attention? Doctor: Hello dear,welcome to HCM forum.Bruising around eyes following a fall needs evaluation to rule out head injury.If there was no history of loss of consciousness,closely watch for symptoms like drowsiness,severe headache,nausea,vomiting etc.If no such symptoms,no emergency and you can wait for your appointmentMeanwhile,you can continue advil,and ice pack application will help to relieve edema.Hope this answer will help.Wishing for a aster cure,Thank you."
},
{
"id": 105884,
"tgt": "Is it harmful to work in an air-conditioned environment if I have asthma ?",
"src": "Patient: I 21 year old male. Recently having chest pain at left and tightness at chest and throat . Chest x ray done. Doctor told it was asthma . And i have to work in air conditioned areas. Is this dangerous? Doctor: Hello naresh.take it easy.it is not at all dangerous.follow the instructions given by your doctor."
},
{
"id": 200700,
"tgt": "What causes feeling of having something under foreskin of penis?",
"src": "Patient: I have something under the foreskin of my penis. It has been there since I remember, even childhood, and it looks weird and has always caused some concern. Please help, it looks similar to a suction cup on an octopus, which sounds really gross but it does. Doctor: Thanks for asking in healthcaremagic forumIn short: May be fordyce spotsExplanation: Fordyce's spots are the visible white coloured sebaceous glands usually located over there. Can be located sometimes near mouth also. These are harmless unless you meddle with it. If it is of cosmetic concern then meet dermatologist for its removal. Good luck."
},
{
"id": 31418,
"tgt": "Could the wound with pus discharge be due to gangrene?",
"src": "Patient: my father is 77 years of age. he has developed a wound in his leg wherein pus and blood was oozing out. his wound is bandgaed after applying betadin onitment and tablet augumentim 625 is given to him. the area near the wound is black and red in colour. is it gangrine Doctor: Hi thanks for asking question in HCM.Your wound is discharging pus with blood.So it seems to be in infective condition, that is unhealthy wound.Just merely black area around wound is not gangrene.In gangrene whole finger,major portion of finger,major portion of leg is black discoloured and that part look dead.Here continue daily dressing with antibiotic coverage.If blackish area increasing then consult doctor.It is better for you to do your sugar estimation to rule out diabetes.Because diabetic patient are more prone for gangrene.I hope i have solve your query.thanks."
},
{
"id": 123904,
"tgt": "What is the difference between symptoms of knee sprain and a torn ligament?",
"src": "Patient: What is the difference between the symptoms of a knee sprain and a torn ligament(acl) - I fell a week ago and I can walk on it with a brace, it hurts when I turn it, I can t bend it back all the way and i all most can straiten it. I heard a pop when I fell? Doctor: Hello, As there is surely a difference between the ligament sprain and torn ligament. In a sprain, there will be pain and swelling which will subside after some days and the knee function will resume. In torn ligament, there will be no movement in the knee even after some time. After the torn ligament there will be grinding of the knee. Doing MRI scan will help reveal the status of the knee internal structures and help to plan the next course of action. In ligament sprain, there is conservative management and in a torn ligament, there is surgical management. Hope I have answered your query. Let me know if I can assist you further. Regards, Jay Indravadan Patel, Physical Therapist or Physiotherapist"
},
{
"id": 99192,
"tgt": "What causes rashes on face, neck and chest?",
"src": "Patient: My daughter has cerebral p. she has developed severe rashes in the last 2 to 3 weeks,on chest neck and faces with redness. I brought her to dr. He prescrib novomoxin because I explain she not well but she can t talk to me so I try to guest,she has Beene 10 mg 3 times daily for 3 daily and I don t see the rash going but I just research rashes and Catherine is on vitamine 12 orally for 1 month and a injection 1week ago +vit c to absorbe iron because she don t take orange juice she is tube fed.is the novo oxen negative bad if it s a allergies to vitamines,overdose. Doctor: I would simply advise you on \u00c0yurveda's principles. Totally harmless and safe.Get capsules Neem, Haridra ( Himalaya co.). Dissolve in water. Put little dissolved sugarcandy to it ( if pt. doesn't have diabetes). Put in feeding tube 3-4 times a day. Also if you can get Haridra Khand powder from Ayurveda shop. 1 tsp BD to be dissolved in water and feed.No side effects. Rashes will go. Does not interact or counteract with other medicine. Keep gap of 15-30 minutes between this and allopathy med.All the best. Your daughter will not have rashes within a week of cont'd.feeding."
},
{
"id": 3510,
"tgt": "Should i go for a pregnancy test after taking medicines?",
"src": "Patient: I had my iui on 18 th July. Today I went to doctor for pregnancy test but it was negative......I took duphaston10mg,nucell and folvite5mg for 15 days .....my doctor told me to stop medicine now and wait for more 2-3 days ..were these medicines right? Shud I go for pregnancy test again after 2-3 days? Doctor: Hi,Welcome to Healthcare magic.I am Dr Ramadevi Wani. I will be answering your concerns today.Usually after IUI if one conceives, pregnancy test is positive after 15 days.The medicines you were taking are right. One of them is vitamin and the other medicine is to support pregnancy if it occurs.Did you do the blood pregnancy test today? It is a reliable test. There is no need to repeat it. Stop all the medicines. You will start your period in 3-4 days time. I wish you get pregnant next time.If you have done urine pregnancy test, then there is a possibility that sometimes it does not detect pregnancy. In that case I advise you to repeat the pregnancy test after 2-3 days.I hope this is helpful.If you have further concerns, I will be happy to answer.Best wishes,Dr Rama"
},
{
"id": 79005,
"tgt": "What causes chest tightness and light headedness?",
"src": "Patient: I have a tight chest, constant light headed ness, pressure in my temples, nausea and a sore neck. Also frequent urination and excessive wind. I recently had an ECG and bloods because of the chest tightness and pains in my legs. All clear. Worried as I feel very peculiar. Normally healthy 46 year old male. Doctor: If all investigations done are normal, then you could probably have anxiety attacks. Also try antacids as it may be GERD as well."
},
{
"id": 23092,
"tgt": "Suggest treatment for heart palpitation and chest pain",
"src": "Patient: Hi, I am a 27 year old female, 5'7\" and 306 pounds. I have a history of anxiety and panic attacks and heart palpitations. However today I have been having palpitations since about 9 am, they come and go but hav been getting a pain in the middle of my chest(sharp) when the palpitations happen. I am also a smoker. What is going on? Doctor: Hi welcome to HCM. I understand your concern.Well there can be few causes of palpitations. I would like to know have you ever had an ECG done for your palpitations? Any past consult for these palpitations.It can be because of increased consumption of alcohol, caffeine, anxiety itself, deranged thyroid hormone levels, some underlying heart conditions not previously known causing irregular beating of the pump.Usually taking a dose of beta blockers can help provided there are no contraindications to their use e.g asthma, Diabetes etc.I hope my consult proved of some help.Wishing you good health."
},
{
"id": 58646,
"tgt": "Have fatty liver, high SGPT level, high cholesterol. Prescribed Ursofalk and Pravaz. Can Pravaz be stopped for a while?",
"src": "Patient: Hi my name is diomhel my sgpt is 132 and diagnosed with fatty liver. And hve high cholesterol my doctor gve me ursofalk for my sgpt and pravaz for my cholesterol. My ask doctor because im going to hve my repeat sgpt test for my medcal abroad in two weeks. Can i stop my pravaz for a while then contenue after i lower down my sgpt? Because i am worried that it might affect the results. :( Doctor: Welcome to HealthCare Magic Hope you are doing good. Well if you are going through Medical test required by the embassy or home office , you should stop it for awhile because if you studied the literature of pravaz one of its side effect is it give you Abnormal liver tests. you can should continue it after for your high cholesterol level , during discontinuing the medicine , avoid junk food , do regular exercise and take care of your diet and health. I hope have answered your query , feel free to ask if you have any query. wish you good health . Regards"
},
{
"id": 40451,
"tgt": "How can I conceive while suffering from PCOS?",
"src": "Patient: I m 22 years old and diagnosed with pcos i hav twin girls who r now 3 years old from last 1 year i m trying to concieve i m taking neodipar But no positive result last month i took gynaecosid for periods dis month i didnt gt my periods nd its been 13 days above the date i had periods last month pregnancy test is negative Plz help me i want to concieve nd be on normal period cycle thanku Doctor: Hi, But you should take oral contraceptive pills for a couple of months for hormonal regulation, afterwards, you should get an ultrasound pelvis to check whether you were responsive to treatment or not. If not responsive then consider IVF treatment. Hope I have answered your query. Let me know if I can assist you further."
},
{
"id": 199279,
"tgt": "How to resolve the problem of erection while meeting a women?",
"src": "Patient: Good day..My name is Wood. Am having a problem meeting a woman. Any time i want to meet a woman my penis will not stand but when am not with a woman it stand and also any time i think of a woman it stand but when trying to meet with woman it will not stand.Please i want to know the problem and how to resolve it.. Doctor: Hello WoodConsistency and volume of penis erection depends upon number of factors like How frequently you meet a woman?Nutritional statussecretions of prostate and seminal vesiclesHygiene there are medicines available for erectile dysfunction, you need not to worry you can ask your doctor for sildenafil group of drugsIn my opinion you need not to worry,this is not a serious problemIf it gets even thinner or lumpy than it could be urethral discharge You should consult your doctor for seven analysis or urine culture to rule out the possibility of UTIHave plenty of fluids maintain good hygieneMultivitamin and food supplements to boost your heath and vitalityMake your bedroom your bed room more conducive and appealing for sex.Spend more time in foreplay and take your time Hope you will find my response informative and helpful please feel free to ask for follow up questions, I will gladly answer you Best wishes"
},
{
"id": 177075,
"tgt": "What causes smelly and watery stool in an infant?",
"src": "Patient: Hi! I have an 8 month old girl who has been having yellow, mostly watery and some mucusy and very smelly stools for a week now - usually 9 poops a day. Also drinking formula normal but little desire for food ... Thoughts ? No fever She also has no teeth yet but a few little buds Doctor: HIWell come to HCMChances of bacillary dysentery would be very likely, some time this comes around on its own you can wait for this for couple of days during this hydration is very much matter and this need to be corrected if child is dehydrated, if symptoms does not improved the this can be managed with antibiotic, but better to have a pediatric opinion, take care."
},
{
"id": 146495,
"tgt": "What is opsoclonus myoclonus syndrome and how is it treated?",
"src": "Patient: i have a 69 year old aunt who went into hospital with a virual infection and now has been told she has opsoclonus mycolonus syndrome the hospital have admitted they havent a clue as this is so rare, she is now presently on a ventilater, highly sedated & on a lot of treatments. we are also in the dark Doctor: Hi, I had gone through your question and understand your concerns. Opsoclonus/ myoclonus syndrome is a rare neurological disorder of unknown causes which appears to be the result of an autoimmune process involving the nervous system. Most af cases in children are associated with neuroblastoma (tumor), in adults is associated with breast and lung carcinoma and some viral infection. Symptoms are related with abnormal eye movements, twitching of muscles, disequilibrium, disorders of speech and lethargy. There is no definite cure for this syndrome, treating the associated condition may improve the symptoms. There are several medicaments used for this syndrome : corticosteroids, immunoglobulins and chemotherapy if associated tumor. Hope this answers your question. If you have additional questions or follow up questions then please do not hesitate in writing to us. I will be happy to answer your questions."
},
{
"id": 92793,
"tgt": "Have h.pylori infection, ulcer healing, gastritis. Endoscopy done. Prescribed amoxy, levofloxacin. Culture report, resistant to amoxy. Correct medication?",
"src": "Patient: I have a h.pylori infection .a healing ulcer..and gastritis...endoscopy done...doc has prescribed amoxy and levofloxacin for 10 days..although for my UTI which was diagnosed amonth back i have completed a course of Nitrofan 100 mg for 8 days..the culture report shows resistant to amoxy..questiois ..1)will the amoxy work in h. Pylori?..2) ir there a corelation in the senstivity or they r independent of each other?? Thanx Doctor: normally if bacteria are resistent to antibiotic group it is resitant in all placesyou must have sensitivity report as there are resistant and sensitive salt list go for sensitive salts according to reports after consulting doctor"
},
{
"id": 1374,
"tgt": "Will taking Evatone-2mg and ovashield together is safe?",
"src": "Patient: Hello Sir, Since long we are try to conceive baby. I had one miscarriage . Now my doctor recommend to do follicle study. i have done. Now my doctor recommend to take Evatone-2mg for next 5 day daily morning, noon and night. Today is my 12day of cycle. I want to know that can i take Evatone for next five day daily 3. And also i m having ovashield daily. Can i take both medicine togather. Is this any harmfull? Please help me. Waiting for prompt reply. Doctor: Hi How are you doing ?taking Ovashield will increase the number of follicles coming out of ovaries. Evatone is Estrogen hormone to be released from ovary. This hormone is needed to have a good thickness of inner lining of uterus, which is the place embryo/ baby sits or implants. These medicines are not harmful , but they alone will not help in pregnancy. Hope I have clarified your query , do let me know if any more queriesAll the bestDr.Balakrishnan"
},
{
"id": 114233,
"tgt": "Back pain with frequent urination",
"src": "Patient: male 72 yrs old back pain above hip pain comes and goes frequent urination unset of pain usually happens when twisting waist Doctor: hello kaith.. \u00e2\u0080\u0098' the above signs are mostly of two diseases named urinary tract infection or secondary diabetes.. go to ur family physician for further concern.. wish u a very good health"
},
{
"id": 44274,
"tgt": "Does Oosure help start periods? Have PCOS, normal thyroid profile",
"src": "Patient: i am 23years old married..suffering from pcos ..got normal thyroid profile ..for the last 1year we tried to conceive.. sperm analysis was done and it is good..doctor gave me glucophage and i continued it for 1yr..last month i visited another doctor and she prescribed oosure caps and folgel for 30days.i want to know that will oosure helps periods tou come and after 30days should i continue it or wait for periods to come.. Doctor: Hello. Thanks for writing to us. Oosure capsules are for the general good health of the eggs that are formed in case there is ovulation. They do not per se induce the ovulation. If your periods are irregular then hormonal treatment is needed. I hope this information has been both informative and helpful for you. Regards, Dr. Rakhi Tayal drrakhitayal@gmail.com"
},
{
"id": 94540,
"tgt": "Had appendix removal. Why am I getting abdominal pain during urination?",
"src": "Patient: I am 46, I had my appendix burst 4 weeks ago and alas have had it removed. Everytime I need to pass wind I can feel it travelling down my stomach and I have excruciating sharp pain with it, it does not hurt when I pass urine or open my bowels which have all been normal, the pain does not dissipate until the wind has passed, what could be the cause of this. Doctor: Hi. The problems are looking like to be of gastritis or Diverticulitis that involved inflammation of stomach or intestinal walls that can be due to increased gastric acid secretion in the stomach. U should avoid chillies and take medication such as antacids and drugs that can increase gastric motility such as Domperidone and antacid such as Esomeprazole."
},
{
"id": 120772,
"tgt": "Is surgery the best option for avascular necrosis?",
"src": "Patient: I got my MRI and Bone Scan done. It showed me having AVN on left femoral head. It is Stage I. Few days back, pain flared up so much that besides being painful I could not stand or move my left leg. I got admitted in the hospital with the help of pain killers like Tramadol, the pain subsided. However, am advised to be careful and avoid load bearing on left leg. Further my doctor suggested that I should undergo immediate surgergy to open up channels for blood supply to femoral head to retain and protect its natural shape. Pls advise. Doctor: Hi, Avoiding weight bearing would prevent any collapse of dead bone in avascular zone. As suggested by your doctor,surgery in early stages of disease would help preserve your femoral head. In Advanced stages, joint replacement would an option. Hope I have answered your question. Let me know if I can assist you further. Regards, Dr. Rajesh Gayakwad, Orthopedic Surgeon"
},
{
"id": 161644,
"tgt": "What causes unconsciousness and twisting in palms/mouth in a child?",
"src": "Patient: Hi, may I answer your health queries right now ? Please type your query here... Sir, my daughter is 15 years old. She is getting her palms twisted, mouth twisted and vibrating, some time goes unconscious, gets irate this she is facing since past 2 years Doctor: Hi, Please let me know the symptoms given above in more detail. As I understand your 15 year old daughter is getting episodes of twisting of her palm and mouth and at times she also becomes unconscious. What to you mean by vibrating. Are they convulsions ? Convulsion is when we see jerky movements of the arms or legs with twitching of the facial muscles. Sometime they may be more localized . Your daughter may getting epileptic seizures. This can be confirmed by an EEG . It will be helpful you can forward a video of the episode. You should also consult a neurologist. Hope I have answered your query. Let me know if I can assist you further. Take care Regards, Dr Anil Kumar Bhatnagar, Neonatologist"
},
{
"id": 214773,
"tgt": "Suggest me some natural remedy for rectal yeast infection",
"src": "Patient: How do I treat a rectal yeast infection??? Could a boric acid sits bath help...if so, what would the solution be?This seems to be a chronic condition...have been on RX oral drugs, powders, creams, etc. I want to go more for something natural, and not so powerful....causing side effects that I have now, such as allergic reactions and so much burning and rawness in that area. Thank you.... Doctor: **1. since your age and sex is unknown, thus it is wise to mention that it mostly occurs in Men than in women in between 40-60 age group, and since it is a symptom and not diagnosis thus one has to undertake a meticulous examination to find any secondary [hidden] cause of the same.2. since fungal infections are common around anal area, thus moist,warm conditions and/or sweating around Anal area both results in overgrowth of Candida (yeast infection) causing fungal infection resulting in inflammation and subsequent irritation and itching.PS. also one has to look out for any Immunocompromised state [as routine] like diabetes,HIV3. In Ayurveda we hold that most of the itching is caused due to problems associated with \u2018Vata dosha\u2019 and by pacifying the aggravated \u2018vata\u2019i.e. impurities in the Rakta (blood) and Stress we can bring down the signs and symptoms of itching.4. Make sure, after bath you give enough time to the area to get itself DRY, either Sun exposure or drying with Soft Towel,because Fungus grows in warm, and moist area.5. Reduce sugar intake, as it is the main food of yeast, bacteria and fungus.6. You can opt for a variety of Ayurveda medicines like Haridra,Amalaki,Trikatu, Triphala, Neem capsules,Haridrakhand,Agastya Rasayan by Consulting with an Ayurveda Physician for further necessary management.7. since itching is main concern thus WARM sitz bath [kati snan in Ayurveda] is recommended. [Boric acid sometimes may cause irritation or problems from absorption, thus if irritation persists, consult your Doctor]PS. because hot baths cause blood vessels to dilate, on rare occasions some people can feel dizzy or have palpitations (rapid or abnormal heartbeat), do check with a cardiologist regarding any baseline Investigation."
},
{
"id": 179923,
"tgt": "Suggest treatment for fever in child",
"src": "Patient: Hello, my 2 year old has had a low grade fever for 2 days, no appetite and irritable. Will not drink water but have nursed him a bit. he is usually an active child so for him to want to be held and not to play or talk is unlike him. Should we be concerned? Doctor: Thanks for posting your query at HealthCareMagic. I am sorry that your 2 year old son is down with fever. It could well be due to a viral infection that causes cough and cold but the details are not clear from your answer. I would have also needed to know the weight of the child to be able to mention the correct dose of the medicines he requires. He needs to be given paracetamol syrup and cold sponging to relieve the fever. Some anti-allergics may also be given alongwith. Please get in touch with a paediatrician who will be able to find out if there is cough accumulated in his chest.I hope that helps. Feel free to revert back in case of further queries."
},
{
"id": 216141,
"tgt": "How to treat pain in my ankles and shin?",
"src": "Patient: I was hit by a baseball on my shin, off the bat to my shin while playing 3rd base, not once but twice in the same game. Its been about a month and there is bruising that spread from the impact and its continuing to spread down to my ankles its very painful should i be worried? Doctor: Cannot say in your particular case but fractures can heal wrong and disrupt normal functioning. Haven't had all that much problems in long bones that stay straight even with fractures.Then, pain can be from various causes. going outward from an injury may imply a (very small) nerve is irritated. This could be permanent. On the other hand low doses of cheap nerve dampers like anti-epileptics or even topical lidocaine are very very effective."
},
{
"id": 123262,
"tgt": "What causes the burning sensation in knees?",
"src": "Patient: What causes the burning sensation in my knees ? Almost continuous. Tolerable during the day, but condition deteriorates at nighttime causing loss of sleep. No obvious signs of swelling of the knees, nor any discolouration in that area. I am female. Age 72. Doctor: Hello, As you are 72 we can expect some degenerative changes in the knee joint and this may be leading to the burning sensation. I will advise for knee x-ray or MRI will be more good. Also using hot water fermentation and doing simple exercises like static Quadriceps and static hamstring should help regain the knee strength and avoid unusual burning sensation. Hope I have answered your query. Let me know if I can assist you further. Take care Regards, Jay Indravadan Patel, Physical Therapist or Physiotherapist"
},
{
"id": 79255,
"tgt": "Suggest treatment for cough and phlegm production",
"src": "Patient: Bad cough phlem cant sleep Hey I started to get this bad cough with phlem and last night I coughed so hard I gaged and then threw up phlem and then food. Is this normal? My stomach hurts a little too and this is the first time it has ever happened to me. I still have this bad cough. Doctor: Thanks for your question on Health Care Magic. I can understand your concern. By your history and description, possibility of either bronchitis or lung infection is more. So better to consult pulmonologist and get done 1. Clinical examination of respiratory system. 2. Chest x ray 3. PFT (Pulmonary Function Test).Chest x ray is needed to rule out lung infection. PFT is needed for the diagnosis of bronchitis. You may need antibiotics, antihistamines, anti inflammatory drugs. Inhaled bronchodilators and inhaled corticosteroid (ICS) are also useful in this. Hope I have solved your query. Wish you good health. Thanks."
},
{
"id": 111586,
"tgt": "Could tingling and numbness in the feet be due to Back pain?",
"src": "Patient: tingling and numbness round edge of both feet (toes) for at least two yrs. No pain not diabetes. GP says probably related to back problem yrs ago. Does this sound right even though no back pain? Sensation slightly reducing over time in right foot, middle two toes. Does this sound normal Please? No other health issues by the way. Is this service free? Doctor: Hello,I had gone through the case and found that it might be due to pressing of nerve.So I will advise to go for neurological test Electromyogram test of feet muscle and Vitamin B12 test and take treatment by neurologist.Hope my answer will be effective for you.Thanks"
},
{
"id": 95730,
"tgt": "What is the reason for my sister to have a constant tightness in the right abdomen from 6 months who also underwent an colonoscopy , and how can it be diagnosed ?",
"src": "Patient: Hello Doctor , My sister has a constant tightness in the right abdomen area from past 6 mnths. We got the sonography done and its normal,Next we went to an abdomen doctor , he did colonoscopy still nothing showed up. Day by day her tightness is increasing. She had cyst one year back but now she does not have it. Can you please guide how to get it diagnosed? She has become weak and mentally depressed Doctor: Hello. Right abdominal rigidity can be due to various causes,however since sonography and colonoscopy has come out with negative reports, this would rule out major changes. However it would be advisable to get an abdominal CT scan done to look for any hidden changes which could not be revealed through ultrasound. Plain xray of the chest is also advised to rule out any underlying lung infection which can mimic such symptoms. Any other associated symptoms should also be consulted with the concerned physician. Hope this will help. Regards."
},
{
"id": 156223,
"tgt": "What can i do to improve my situation as i am having a long history of bladder cancer?",
"src": "Patient: hello, i have a long history of bladder cancer, since 1999.......every year since i have averaged two out patient surgeries to remove 2 or three small malignant growths. have had the BCG treatment at the onset....doctor has not recommended radiation...just continued close watch with cystoscopes every three to six months........do you have any suggestions as to what i can do to improve my situation????? .... Doctor: since you were recommended bcg therapy it means you had early stage disease. Follow up is for 5years. after 5years investigations are done if clinically warranted only. So be glad that you dont have bad disease and you have chance of cure!"
},
{
"id": 54867,
"tgt": "Suggest treatment for fatigue after undergoing a lap cholecystectomy",
"src": "Patient: Lap Chole on 12-27, have not taken any pain meds, afraid of vomiting, moving around in the house a lot but I am so tired. Have Lupus but not sure which kind yet........ supposed to go back to work on Tuesday......what to do about the fatigue...... Doctor: hi.noted history of fatigue and laparoscopic cholecystectomy. any laparoscopic surgery has a faster recovery rate as compared to open surgery. do not be afraid or anxious to move around and take in any food if the doctor has permitted you to do so already, because this will help your recovery a lot. ambulation/movement will help motility of your intestines as well. low fat diet is recommended. if symptoms are persistent, it is best if you do a follow-up consult with your surgeon, for physical examination and clinical evaluation. diagnostics and management will be directed accordingly.hope this helps.good day!!~dr.kaye"
},
{
"id": 219299,
"tgt": "Can thyroid abnormality trigger the growth of fetus?",
"src": "Patient: I have tested positive in the pregnancy test last Friday and yesterday we saw the Thyroid test report to be showing abnormal value for TSH of 17 where it should be 3.5. From my last menstrual cycle start it is now 46 days ( though I know the conception could only be a maximum of 40 days ago) and in the scan done yesterday the doctor says the foetus seen is only showing 35 day growth. This could be due to the thyroid causing delayed growth or the foetus having stopped growth due to the same or some reason. Also I had mild discharge yesterdy of brown period like strands, though very little. The doctor has adviced me to take some Thyroid medication and the following : Dufaston and Trenaxa 100 mg; and to wait till the next 10 days after which we can scan and see the growth and test TSH. If all is not well we may need to medically cleanse the uterus, as it is known that Thyroid abnormality may cause improper intellectual and other growth in the baby, or the baby may have stopped growing by then. I would like to take a second opinion and confirm we are on the right track, as either way it is a very important decision to make for us. Doctor: Hello and welcome to healthcare magic.First of all let me assure you, the role of thyroid is essential for foetal mental growth but it doesnot affect growth of the embryo so early on in pregnancy.You DO NOT REQUIRE TO ABORT , just because your tsh level is deranged. Wrong date of last menstrual period is the commonest cause of apparent early foetal growth retardation.Repeat the usg as instructed, your thyroid profile will take 4-6 weeks to normalise.Please do write back if you have any other queries."
},
{
"id": 78632,
"tgt": "Suggest remedy for bronchitis",
"src": "Patient: I have bronchitis, allergy(Asthma tendency) since last 2 years , I am facing right side side chest pain along with phelgum since last 2 years as well. I used to take medicine such as callbrid. ,AB Pheline. Telecast L,Montral. i had shortness of breath as well. But now has reduced. I use to do Yoga, Pranayam . so has controlled over it but not fully recovered. I have done ECG, PFT, Chest X-Ray and all sorts of bood test, Endoscopic, ultra sound etc Doctor: Thanks for your question on Health Care Magic. I can understand your situation and problem. For exact diagnosis of bronchitis, PFT (Pulmonary Function Test) is must. This will not only diagnose bronchitis, but it will also tell you about severity of the disease. And treatment of bronchitis is based on severity only. Best and first line treatment of bronchitis is inhaled bronchodilators and inhaled corticosteroid (ICS). So better to consult pulmonologist and get done PFT. You require inhaled treatment for bronchitis. You will definitely improve with inhaled treatment. Hope I have solved your query. I will be happy to help you further. Wish you good health. Thanks."
},
{
"id": 19667,
"tgt": "What causes throbbing sensation in heart?",
"src": "Patient: Good Evening, I am curious about an uncomfortable throbbing sensation in my heart (to the rhythm of my heart) after exercise (not during). It is not a pain but almost. I have had it for years and it is, I think, getting more pronounced but very slowly. A curious feature is that it will disappear for a few seconds when I shift my weight from one foot to another. If I am standing, it can last a few minutes. It is not there if I sit down. I don t think it is angina. In parenthesis, I am one of those persons who feels every heartbeat under normal circumstances. Doctor: Hello there. I read through your question and understand that you are having palpitations. Now palpitations can be due to anxiety or It can be due to cardiac causes. To rule out any cardiac cause I would suggest you to get an EKG done. I hope that answers your question. If you have any further questions please feel free to contact."
},
{
"id": 103453,
"tgt": "Hearing heartbeat in ears, ear canals burning and itchy, have allergy, using ocean nasal spray, swollen lymph node",
"src": "Patient: I have been hearing my heartbeat in both ears for 3 days now. It is so annoying, as it comes and goes throughout the day. My ear canals are burning a slightly and itchy. My sinuses also seem to be really dry, as I have bas allergies . I have been using Ocean Nasal spray for the past 2 days hoping the whooshing sound will go away. I also had a swollen lymph node near my ear a few weeks ago, but it has no almost gone completely away. What do I need to do to get the whooshing sound to go away? Doctor: hi, this looks like blocked eustesain tube due to upper respiratory tract infection. you need antibiotics, analgesics and nasal decongesants which makes the tubes open and you will be symtom free. meet ENT specialist."
},
{
"id": 149579,
"tgt": "Have disc herniations and numbness in spinal cord. Medication? Cure?",
"src": "Patient: I am not addicted to pain meds, but I need to find a pain doctor in my area. It appears, most physicians do not want to prescribe pain meds, because of the individuals that abuse the system. Those of us that have 7 and 8 mm disc herniations and numbness in their spinal cord can't get help. I have recent tests to confirm my diagnosis, but have no health insurance any longer. I can not afford to keep wasting money on doctors that are afraid to treat their patients. Doctor: Hi,You might be suffering from degenerative disc disease & it occurs due to repetitive overload or stress to the disc & it increases the risk of disc herniation & spinal canal stenosis. For this condition you have to do stretching & strengthening exercises in which you have to stretch tight muscle & strengthen weakened ones which will alleviate stress on facet joint and disc. I would suggest getting this evaluated by an orthopedician for an accurate diagnosis and appropriate management. Meanwhile for pain control take anti-inflammatory drugs like ibuprofen."
},
{
"id": 175653,
"tgt": "What does Polychromatic with bile cells red cell changes mean?",
"src": "Patient: Hi, A 5 Y/O MALE with fever and pallor. WCC of over 300x10/L, both LOW HB AND PLTs. Numerous Smudge cells are seen and with about 50% Blasts cells. It is Polychromatic with bite cells red cell changes. What would this presentation be suggestive of? Thank you. Doctor: Hi...this is suggestive of leukaemia or in common terms blood cancer. But you need not be disheartened. Now a days with scientific developments, treatment is possible and there are many kids who have conquered cancer.Regards - Dr. Sumanth"
},
{
"id": 223488,
"tgt": "Is pregnancy possible while having contraceptive implant?",
"src": "Patient: Can I become pregnant on the contraceptive implant? I feel very strange but can t put my finger on why. Have pregnancy symtoms. Taken a home pregnancy test and it was negative eventually. When from slight positive to negative within the 3 minutes. I just don t feel right. Thanks. Doctor: Hello,Thanks for trusting us with your health concern. As for any other mode of contraception, there is a slight failure rate for the implant too. If you are not sure of the home pregnancy test, you can go for the estimation of the serum beta-hCG titres to confirm/exclude pregnancy. Home pregnancy tests should be done 3-7 days after the missed period with a fresh early morning sample of urine. A trans-vaginal sonogram done a week later can give the status of the pelvic organs and clarify the situation. Hope you find this information useful. Take care."
},
{
"id": 14380,
"tgt": "Suggest remedy for warm rash with little swelling and itching on arm",
"src": "Patient: my right arm has a rash and is warm to touch it is also a little swelling I don t know if I have insect bites I thought it was mosquitoes bites because it really itches a lot what can I can I put on nothing seems to help I have tried benadyrl and nepsporin Doctor: Hello dear,The rash in your right arm can be either due to insect bite, dermatitis or fungal infection (Tinea) in that region.Management includes:1. Keeping the area dry & avoid too much exposure to water or sweat formation.2. Antihistamine preparations like cetirizine to reduce the itching sensation & provide symptomatic relief.3. Application of topical antifungal cream like Clotrimazole or Miconazole to tackle the causative organism.4. Topical antibiotics to prevent secondary bacterial infections.If symptoms still persist or aggravate further, then it will be better to consult a Dermatologist & get a proper examination done to find out the actual cause.So that appropriate management steps could be taken.Also avoid wearing tight clothes, as it will cause sweating & predispose to fungal infections.Wishing you a Good Health.Take care."
},
{
"id": 106942,
"tgt": "Does Spironolactone cause lower backache?",
"src": "Patient: I am Mtf i have been taking 100 mg spirolactone and 4 mg of Estrodial for 6 weeks now, i noticed recently for about the last 2 weeks that the Spiro which i take in the morning with food i believe is causing me lower back to hurt? Could this be the Spiro or something else. I drink lots of water, no coffee or soda. I also watch my carb intake as i am also a controlled type 2 diabetic. Any idea what could be going on? Doctor: hi sir/madam,Thanks for your question on Healthcare Magic.there could be few possible reasons for the backache,it could be the exacerbation of previous problem you have.secondly it could be due to the fact that you are on diuretics like rusemide and spironolactone which can lead to backache secondary to eectrolyte disturbance and dehydration.Basically the muscle of the back produce tis backache.i would suggest that you do inform this to your doctor and have an evaluation in order to find out what exactly is causing the backache and then have appropriate treatment.hope this was helpful.have a healthy day."
},
{
"id": 177887,
"tgt": "What causes white stain on hand and neck?",
"src": "Patient: hi,my son is 3 month l saw white stain on her hand around tow cm and a little on neck and i think her face may be have it im not sure about her face ,i dont now what is it?i scared it developed ,what do u think about it ,that time i saw it didnt developed i dont know what is it Doctor: Thank you for the query.Your concern regarding such things is right.But instead of panicking it is advisable to show him to a pediatrician/doctor/skin specialist as it is not possible to draw any conclusion from your description of the stain.If it is normal deviation you can be at ease at least.And if any problem diagnosed one may go ahead with possible treatment."
},
{
"id": 139174,
"tgt": "What causes cramps in hand?",
"src": "Patient: Trying to find out possibly causes for my husband s hand cramping? he also has had trouble with buttocks and pain shooting down his legs a nerve test has been done along with MRI we waiting for results. The hand cramping is something new?Thanks.Sharon Doctor: Hello,I have studied your case. I hope your husband is not a smoker or alcoholic or Diabetic. Because all these three conditions can make him prone for vitamin B 12 deficiency which can cause hand cramping. It can also occur due to less salt or calcium in body. So I would suggest you to get his blood work done. Also start taking Pregbalin M at night time. It will take care of vitamin B-12 deficiency. Also I would recommend you to take carnisure tablet daily along with calcium and vitamin D3I hope this answer will be helpful for you.let me know if there is any other followup questions.thanks"
},
{
"id": 69200,
"tgt": "What causes a pea sized lump below the collarbone?",
"src": "Patient: I have a small, hard, pea sized lump below my collarbone up close to where it meets up with my shoulder. It doesn't hurt or anything. It doesn't really move much either, maybe a very small amount but it seems pretty stationary. Should I be worried? Doctor: Hi,The lump is usually a lymph node or rarely a lipoma. If it is present within a short duration and if it is going to disappear, nothing to worry.You can get a procedure called as fine needle aspiration cytology for confirmation. It helps in identifying the exact cause.The lymph node could be enlarged due to infection in surrounding area like oral cavity. Lipoma is a benign tumor of fat cells.You can ask your doctor to order this test. Take care.If you have any further queries, happy to help again."
},
{
"id": 115735,
"tgt": "What is the treatment for low WBC count?",
"src": "Patient: My father is ill since last six month to year time, he is taking treatment in Defence hospital, delhi. but we did not find any imporvement. It seem that white blood cells proportions are much less then what is normally required. also his palm turns black patches. even a very little spices food, he find difficults to take. now a days health detoriates at much faster rate. could any one suggest me what to do, where to take treatment and what should be done. Doctor: Hi, dearI have gone through your question. I can understand your concern.He has low wbc count. There are many causes of low wbcs, simple viral infection to bone marrow failure can lead to low wbc count. Bone marrow examination is needed in persisitent low wbc count. Just go for bone marrow biopsy to search the cause then take treatment accordingly.Hope I have answered your question, if you have any doubts then contact me at bit.ly/Drsanghvihardik, I will be happy to answer you.Thanks for using health care magic.Wish you a very good health."
},
{
"id": 145516,
"tgt": "Suggest treatment for herniated discs and spinal stenosis",
"src": "Patient: I have been diagnosed with herniated discs and spinal stenosis about 6months ago -- have had physical therapy and cortisone injections-- they work for while and then the pain is back. My neuro surgeon said not to come back until I am ready for surgery. How do I know when the time is right. Doctor: Hello. I have been through your question and understand your concern.It is not a standart feature but basically the neurosurgeons say that if pain does not go away in 6 weeks after using Steroid or Non steroid anti inflammatory drugs and the pain is still unbearable you should consider surgery.Hope this helps.Best regatds"
},
{
"id": 179611,
"tgt": "Why does a child refuse to eat?",
"src": "Patient: hi doc. My son is 21 months old. I am exhausted in trying to feed my son but this bad boy didn t eat any thing. I give him fresh milk but he didn t feed more than one litre in a day. No fruit no potato or chicken. I want to give him nido milk. Suggest which nido is best for him. And any other suggestions. Regards Jasim s mom Doctor: Hi one litre of milk for 21 month baby is more than enough.Do not force feed child.Let hin develop habit of feeding .I will also like to know weigt of your child.thanks"
},
{
"id": 220741,
"tgt": "Is there a possibility that the surgery was unsuccessful?",
"src": "Patient: i just had a surgical termination 2 days ago & i wish i can take it back. it was twins, i didnt want to do it. i still feel very much pregnant minus the extreme nausea. is it possible that the procedure was unsucessful? theres no bleeding, no pain, just alot of crying and an extremely large amount of depression. Is it possible that the surgery was unsuccessful? Doctor: Hello dear,I understand your concern.In my opinion the surgical method ensures clearance of pregnancy mostly.In case of incomplete abortion there will be be irregular spotting or continuous bleeding.If you don't have any of those complaints then you need not worry.The chances of retained products are rare.So relax.In order to confirm you can as well go for ultrasound for confirmation.Nothing to worry.Best regards...."
},
{
"id": 160443,
"tgt": "What causes itching on neck of a child?",
"src": "Patient: Can I Please get help with this questions,that I have ask alot of doctor,What do you call a child neck that is scratch up real bad,the scratch appear by they self,My two children being having scratching on their neck since they was little,I took them to family doctor he donot what to tell me,so i being back and fourth to doctor,Can I please get help with this! Doctor: Hi, You need to do for them some lab works, like liver function, kidney function, complete blood pics and bleeding and clotting time. Take care. Hope I have answered your question. Let me know if I can assist you further. Regards, Dr. Salah Saad Shoman, Internal Medicine Specialist"
},
{
"id": 118733,
"tgt": "Have addisons disease, seizure disorder. Taking topamax and hydrocortizone. Low platelet count. Frequent blood transfusions needed. Why?",
"src": "Patient: im 33 i have addisons diease and a seizure dysorder i take hydrocortizon 20mg twice a day and topamax 200mg twice a day for the last yr i have had low blood volume and low platelet low enough that i have had to have 12 blood transfusions this yr the sunday after thanksgiving i got a stomach bug that made me end up in the hospital and 2 more units of blood i would like to know why this keeps happening and why now i feel so bad but all my levels are in the low normal range can some one help Doctor: Helloas you have said that you have addison taking hydrocortisone and seizure taking topiramate When platelets become low with addison then this could means ITP idiopathic thrombocytopenic purpura meaning low platelets as both share same antibodiesSecondily hydrocortisone also causes low plateletsH.Pylori is a famous bug of stomach that cause low platelets and on/off stomach problems with nausea and vomitingHope this explains your query , get h.pylori test"
},
{
"id": 57655,
"tgt": "What diet should be taken for gallstones?",
"src": "Patient: I have known for perhaps 5 years that I have gallstone(s). I think I have had some discomfort after a meal, right lower rib area. Not too bad. What foods are better than others? I think I know (stay away from fatty foods, butter,cream, etc. Is avocado another bad fatty food for this condition? Perhaps some shell fish, prawns??This discomfort is very new, would like to have more info. Thank you Doctor: HiThank you for asking HCMFor Gall stone disease if you are asymptomatic you can take all kind of fishes.If you are symptomatic then you should take the less fatty fishes like pollock,haddock, orange roughy.Shellfish is naturally low in fat, making it a good choice in your diet for gallstones. A 3-ounce portion of cooked lobster or shrimp contains less than 2 grams of total fat.It is better to avoid them fried or seasoned with butter or margarine.It is better to take steamed food.Yellow of egg and the cakes containing those should be avoided as they are good stimulants for gall bladder.Hope this may help you.Let me know if you have any further query."
},
{
"id": 24287,
"tgt": "How safe is taking Zorcor for cholesterol?",
"src": "Patient: my cholesterol is 241. I am a 57 y.o. female with djd in joints and ileosacral spinal djd for which I recieve steroids injection in my L knee every 4 to 6 months. I have a total knee replacement as of march this past year. My bp is borderline for which I take 1/2 half tab hctz and pain meds along with prilosec qd. My doctor has notified me I had to start Zorcor for my cholesterol. I do not like taking loads of meds. Can my cholesterol elevation be caused bythe steroids injections. I walk and use my wii to exercise daily and water aeorobics 2x a week. I don't want to starttaking more meds especially after reading the side effects. Do I have a valid point? Doctor: Hello!Thank you for asking on HCM!Regarding your concern, I would explain that total levels of cholesterol is not a confident indicator of your cardio-vascular risk. A Total cholesterol/ LDL ratio more than 4.5 could indicate a higher cardio-vascular risk related to dyslipidemia and the necessity of statin intake. From the other hand, it is true that steroids cause elevation of cholesterol levels. So, for the moment, I would recommend some modifications to your diet (reduce saturated fat and increase food rich in omega 3 fatty acids). As soon as you will stop taking steroids, your cholesterol levels will return to normal again. Hope you will find this answer helpful!Best wishes, Dr. Iliri"
},
{
"id": 142840,
"tgt": "What is the cause of tingling and numbness?",
"src": "Patient: my left hand tingles when i cough. There is always a some level of numbness but gets worse when I cough. Also, I took 1 trazadone pill to help me sleep when I fell and cracked some ribs on my left side, could this be a related symtom? I took the pill about 2 weeks ago, and noticed the numbness the next day. Doctor: The act of coughing is somehow activating your brachial plexus and specifically the C8/T1 sensory nerve roots exiting the spinal cord therefore, some degree of compression of the ribs or shoulder girdle are mechanically pressing on those ribs when you forcefully cough. If there's ALWAYS some numbness then, there is baseline compression and this should be checked out by going to a neurologist, getting an examination, and then, proper testing. There may be a solution of physical therapy that can help reduce the compression...or if necessary, surgical decompression. Please rate this a 5 STAR ENCOUNTER and write me at: www.bit.ly/drdariushsaghafi with more questions."
},
{
"id": 17101,
"tgt": "Suggest therapy for a problem in the heart causing abnormal heart beat",
"src": "Patient: I Have A Heart murmur AORTIC STENOSIS. I have a mauld leak of blood going back into my heart as the /\\ closes (sorry i dont know what it is called but it is in the artey) and i have an abnormal heart beat (fast). I wonted to know if i would be able to go on rollercoasers. Doctor: Hello, After going through your medical query I understand your concern and I would like to tell you that if you have aortic stenosis then you can feel rapid heart rate or skipped beats. It is recommended for you to get a HOLTER (ambulatory ECG) for a minimum of 48 hours done and consult with reports. Hope I have answered your query. Let me know if I can assist you further. Regards, Dr. Bhanu Partap, Cardiologist"
},
{
"id": 181273,
"tgt": "Should i still wear the retainer as it hangs in one side?",
"src": "Patient: I had braces for two years and have been wearing a top retainer for about a two years as well. Recently I havent been wearing my retainer... Its probably been around 2 weeks. Now the back of one of the sides will not remain pressed to my teeth (its a clear retainer) and hangs down slightly, compared to the rest of the retainer, which is clinging o my teeth quite tightly. Should I still wear the retainer with hopes that it will help shift my teeth back into place? Or will I need to see my ortho? Doctor: Hi..Welcome to HEALTHCARE MAGIC...I have gone through your query and can understand your concern..As per your complain Retainer is an Orthodontic appliance that is used to maintain the aligned position of the teeth after completion of the orthodontic treatment to align the teeth..If the retainer is not pressing on the teeth and has become loss it may be possibly due to movement of the teeth due to not wearing the retainer for a long time..It is of no use to wear it as untill it is able to apply pressure it will not give any positive results and if you wear it there can be relapse of misalignment of teeth..You should consult your orthodontist and get evaluated for whether you further need retainer or not and if required you need to either get it adjusted or a new retainer may needed to be fabricated for you that applies appropriate forces over the teeth and helps in maintaining its alignment..Hope this information helps..Thanks and regards..Dr.Honey Nandwani Arora.."
},
{
"id": 147084,
"tgt": "What does this MRI report of spine indicate?",
"src": "Patient: helllo doc my MRI scan of spine shows the following 1. partial loss of lumber lordosis 2.early degenerative changes of spinein the form of marginal breaking. 3.irregerularity of adjacent endplates L4/L5 & l5/S1 vertebratewith partial dessiccation & partially reduced heights of L4-5 &L5-S1 disks 4.spinal canal narrowing mid at L4-5l (10 mm) severe at L5-S1 leve level (6.5mm) Doctor: HI THANKS FOR POSTING YOUR HEALTH QUERY ON HEALTHCARE MAGIC.THSE CHANGES ON YOUR MRI SUGGEST \"DEGENERATIVE DISC DISEASE/LUMBAR SPONDYLOSIS/SPINAL CANAL STENOSIS\".IT COMES DUE TO IRREGULAR WEAR AND TEAR OF THE SPINE,WITH AGEING,TRAUMA TO SPINE ETC.DRUGS LIKE PREGABALIN ,AMITRYPTILINE ARE USEFUL TO DECREASE PAIN.AVOID LIFTING HEAVY WEIGHTS..INJECTION METHYLCOBALAMIN -B12,1000ug WEEKLY FOR A MONTH FOLLOWED BY MONTHLY ONCE FOR A YEAR WOULD PREVENT \"SPINAL CORD DEGENERATION\"DEFINITIVE TREATMENT WOULD BE TO PERFORM \"LAMINECTOMY\" WITH COMPRESSION RELEASE.CONSULT EXPERT NEUROSURGEON FOR YOUR HELP.THANK YOU.TAKE CARE."
},
{
"id": 12272,
"tgt": "Can dialantin pills induce psoriasis like skin infections?",
"src": "Patient: Hi how u doing? Does psoriasis get on the inner thighs and does it itch bad.... I was wondering if dilantin can cause it too because i take that for epeilepsy and i know i have had itchy skin from it and been taking it for 30+ yrs!!!! Oh im sry i see it says to put my age on here i am 37 5'11 and 160lbs Doctor: Hi. First of all, its not clear if you are having psoriasis in your groin area. It could be tinea cruris too. And, phenytoin is not known to cause psoriasis in current medical literature. On the contrary, there are some works on phenytoin as a treatment for psoriasis! Here is one such paperhttp://europepmc.org/abstract/MED/21970250Now coming to your condition, i think you should give antifungals a try. Apply Onabet cream twice daily. Update us after a week. If possible, post clinical photograph for confirmation. Hope it answers all your queries"
},
{
"id": 25178,
"tgt": "What are the symptoms of arrhythmia?",
"src": "Patient: Hi, I am in Austin and the temperature is 30F. I was inside my house on the heat and went outside to put the garbage cans inside. I ran a little back and forth since there were 3 large cans. It has been 30 minutes and since then my hear won t stop clicking . It feels like arrhythmia, but the clicks are stronger than I ve ever experienced. I read only that this could be a result of a thermal shock and that we are supposed to make these transitions from hot to cold gradually. My concern is that 30 minutes have passed and the feeling won t go away. Am I at risk? Is this an emergency? Thank you. Doctor: Thanks for your question on Healthcare Magic. I can understand your concern. No, this is not thermal shock. In my opinion, you are mostly having stress and exertion induced tachycardia which is giving you feeling of heart clicks. But since it is not settled after 30 minutes, we should definitely rule out arrhythmia (rhythm disturbances in heart). Because arrhythmia can cause palpitations, clicking feeling of heart, pounding sensations in chest etc. So better to go to emergency room and get done ecg and 2d echo. If these tests are normal then no need to worry for arrhythmia. You are mostly having stress and exertion related tachycardia. Don't worry, with rest, you will be alright.Hope I have solved your query. I will be happy to help you further. Wish you good health. Thanks."
},
{
"id": 126278,
"tgt": "Is physical therapy the right method of treatment for a fracture?",
"src": "Patient: Index finger swollen due to fall. Late call on Doctor to say it is fractured. Finger knuckle is as swollen as first day mm,[=- (2 months). No RX given for anything. Painful and not comfortable. Doctor said next step is physical therapy next month. This seems w ong??? Doctor: Hi, Physiotherapy is not the first step for fracture management. We have to immobilize the affected finger for two weeks and physiotherapy is required only after that. Consult an orthopedician and get evaluated. Hope I have answered your query. Let me know if I can assist you further. Regards, Dr. Shinas Hussain, General & Family Physician"
},
{
"id": 86758,
"tgt": "What causes severe abdominal pain in children?",
"src": "Patient: My son went to the ER the night before last with severe abdominal pain. The conclusion was that he is constipated. He has taken 2 doses of miralax and, today I gave him a children's enema. Still no significant bowel movement. How concerned should I be about this? I really can't afford to miss work, but I don't want to let this go too long. Doctor: Hi ! Good evening. I am Dr Shareef answering your query. In my opinion, you should take him back to the ER of the nearest hospital for a review to rule out any surgical cause for the pain abdomen. For the present avoid repeating any more enema. Please do not hesitate to ask in case of any further doubts.Thanks for choosing health care magic to clear doubts on your health problems. I wish you an early recovery. Dr Shareef."
},
{
"id": 102156,
"tgt": "Does budamate and rotacap have any prolonged effects in asthma patients?",
"src": "Patient: Hi, I am suffering from asthma from the past 40 years. Now, i am 58 years old and suspected that it may be changed into COPD. I am using daily one Budamate 200 rotacap from the past 3 years and one montek lc tablet from the past 1 year. Recently, i am feeling throat pain and viscid phelgm. Can i continue the combination of these two medicines for asthma, is there any danger and other side effects for prolonged use of these medicines? Please give me advice in detail. Thank You. Doctor: Hi,The viscid phlegm may be due to LC component of montek LC. Usually levo cetrizine is not recommended for more than 15 days as this can dry the secretion and make symptoms worse.So you can continue the inhaler and use only montelukast 10 mg without any other combination. If symptoms worsens the budamate dose can be increased to prevent the symptoms.Regards"
},
{
"id": 156084,
"tgt": "Is the painful lump in the breast benign?",
"src": "Patient: hi iam 33 with a pea size lump in right nipple area of my breast. my mother had massive invassive cardonma breast cancer ...iam having shoot pains through my breast were lump is an goes through an burns down inner arm as if i have musle rubb on...i have a referel for cancer center but its taking for ever to get in....what do you think? Doctor: It can also be fibroadenosis which is benign. But try to expedite your referal to cancer center.If the size keeps fluctuating with your periods If the pain keeps increasing during menstruation its benign. My only concern is family history which is a red flag."
},
{
"id": 14038,
"tgt": "What causes marks, bleeding, lumps and bruising on the forearm?",
"src": "Patient: I scrape myself, or like making a bed, if i scrape it between the mattress or headboard tI get reddish purple marks. Later if touched or scraped again, usually bleeds. some bumps, I am 58 almost 59. I have people tell me i have old age bruising. It looks awful and I m embarased. Only happens on front fore arms. Doctor: Hello and Welcome to \u2018Ask A Doctor\u2019 service. I have reviewed your query and here is my advice. It could be because of decrease collagen content in skin. Take Vitamin C supplements daily. Avoid scratching/rubbing the skin. If you have itching, take Levocetirizine/Cetirizine tablets. In case of worsening of rash or appearance of new lesions, do consult your Dermatologist. Hope I have answered your query. Let me know if I can assist you further. Regards, Dr. Siva Subramanian"
},
{
"id": 56604,
"tgt": "What causes fatty liver?",
"src": "Patient: Hello sir, A week back i did a complete Hemogram blood test.Results showed that my platelet count was 35000 wheras all other counts were normal.Hemoglobin was 16.4. Also had a Urine Routine test and USG of Whole obdomen.My urine test were normal wheras Usg test showed fatty liver..Currently i do not have any sypmtoms or and kind of weakness.I am 33 years old..Kindly suggest Doctor: hello dearyou have two condtion simultaneously asymptomatic fatty liver at your age is normal and I treat it conservatively with fat free diet and avoidance of alcohol for three monthsif it does not changes then i start statins and ursochol"
},
{
"id": 196249,
"tgt": "What causes jaw pain and headache after unprotected sex?",
"src": "Patient: hi am a man who had unprotacted sex with a girl after the 3rd time i have gonarria, after that i lost my apatis and i lost 8 kilos plus i have night sweats and body itching diarrea from time to time only that so on the last 3 months i had 4 hiv tests all negetive and i made the girl to do another 3 tests in the last 4 months all return negetive . but right now am having jaw pain and head eake only the left side , am worried should i do another test after all that? Doctor: Hello and thanks for your question.It's my first priority to help you.Well as you describe,chances of HIV are extremely less to nil if you have HIV tests results negative after 4 months.Your pain can be associated with other possible causes like Sinusitis,Neuralgia,Tooth infection.I really assure you to no need to worry about HIV.Still I would recommend to see your dentist or ENT specialist for a clinical exam.Start a multivitamin supplement to boost your immunity.Hope this helps."
},
{
"id": 79959,
"tgt": "Suggest treatment for bacterial infection and cough",
"src": "Patient: i m coughing from more than 4 weeks with green, yellow sputm, i got me tested for sputum,chest x ray and mantaux. chest x ray was fine and sputum test didnt give anything but my mauntaux enduration sized 12x13 mm. so doctor says i m having a bacterial infection,what do yo suggest Doctor: Thanks for your question on Health Care Magic. I can understand your situation and problem. In my opinion, you should get done sputum culture and sensitivity report. Sputum culture will isolate the causative organism and sensitivity report will guide you about effective antibiotic treatment. By this way, you can prevent unnecessary use of antibiotic and antibiotic resistance. Also get done PFT (Pulmonary Function Test) to rule out bronchitis. Because bronchitis can have similar symptoms. You may need inhaled corticosteroid (ICS) and inhaled bronchodilators. So consult doctor and discuss all these. Hope I have solved your query. Wish you good health. Thanks."
},
{
"id": 88632,
"tgt": "What causes stomach pain and cramps?",
"src": "Patient: i have been suffering from severe stomach pains and cramps along with severe boughts of watery stool movements. I have been given imodium, biscopan, nexium and recently Amitriptyline. My doctor doesn't know whats going on or how to manage this, do you? Doctor: HI.Severe stomach pains and cramps associated with severe bouts of watery stool movements may be due to gastroenteritis. Get the stool, blood and urine tested, also blood sugar to rule out diabetes, thyroid functions. Continue the medicines you have. Immodium should be taken only after a bout of loose stool and 6 capsules in 24 hours as a maximum dose. Do not take more than sic capsules. Adding a proper antibiotic against the intestinal bacteria and metronidazole should have helped you. Treatment of the primary medical problem if exists is a must.Amitryptilin has no role in such conditions and you may please stop this in consultation of your Doctor. IF there is no relief, colonoscopy and CT scan of the abdomen may be needed. Consult a Gastroenterologist for proper evaluations and treatment ."
},
{
"id": 143310,
"tgt": "Could manipulation under anaesthesia for frozen shoulder cause pinched nerve?",
"src": "Patient: I had a MUA for a frozen shoulder last Thursday. I m at home doing the passive motion chair exercises 6-8 hrs a day. Don t go back for FOLLOW-up visit till Sept 4th. Started onto ing these sharp, tingling pains in ny neck maybe 2 days ago. They won t stop. Could this MUA have caused a pinched nerve somewhere? Doctor: Hi,I understand your concern and would explain that your symptoms could be related to a pinched nerve or a discal herniation in the cervical spine causing radicular pain. It could be related to MUA, but there are also a lot of other causes to consider. I recommend consulting with your neurologist for a careful neurological exam, a nerve conduction study and a cervical spine CT scan. Meanwhile, I would recommend holding a cervical collar, which will help relieve the pain. Hope to have been helpful. Let me know if I can assist you further. Kind regards, Dr. Aida"
},
{
"id": 157978,
"tgt": "Mess pants with no warning, nausea, pain, lost 19 kilos in 12months. Had colonoscopy, diagnosed with diviticular disease. More tests needed?",
"src": "Patient: Hi my name is Sue and I have had a few colonoscapies and have been diagnosed with diviticular disese and 12 months ago i was given another colonoscopy and had 30 plus polyops on the diviticular and always have infection and mess my pants with no warning, I feel nausious a lot and the smell of food cooking makes me feel sick, I also have lot of pain and in the 12 months have lost 19 kilos, What should I do as the sergeons want to do more tests but the last sergeon I went to didnt want to so another colonoscopy due to being afraid he may perferate my bowel Doctor: Dear Sue. You are not just suffering from diverticular disease but also polyposis of the colon, a condition in which multiple polyps form in the colon and sometimes in the small bowel. Some of these polyps may with time develop cancer. Your having pain and weight loss are not good signs and more in favour of cancer development. I am assuming that the polyps that have been removed previously were sent for biopsy and that none of them showed cancer. I would advise you to first undergo a contrast enhanced CT scan of the abdomen. That will give us a fair idea of what we are dealing with and if there is any evidence of cancer development. Also, a colonoscopy is advisable (before the CT) but only by an experienced gastroenterologist in whose hands it shouldnt be a problem."
},
{
"id": 48180,
"tgt": "What causes huge pain on the lower part of the abdomen area?",
"src": "Patient: Hi my brother was in the hospital yesterday in the emergency as he had huge pain on his abdomen and his lower part and then he was told that he have nonobstructing right renal calculus noted after he did a CT scan. I want to know what medication/diet he should have that may help. Unfortunately, I don t know what is his kidney stone size. Doctor: HelloTreatment of renal calculus depend upon many things like size,composition of calculus etc.Calculus less than 4 mm is expected to pass spontaneously whereas large calculus may need intervention.Patient may need diet alteration depending upon its composition like patient with urate calculus should avoid high purine diet.Your brother may need stone dissolving drugs initially.He should drink lot of water and need regular follow up.Take CareDr.Indu Bhushan"
},
{
"id": 119574,
"tgt": "Suggest treatment for numbness and soreness in knee after ACL replacements",
"src": "Patient: I am 52, about 195 lbs and 6 3.I have had two ACL replacements and a third surgery on left knee, found out I no longer have an ACL. I have been expierencing what seems like a deadness in my knee after prolongedwalking. I work for a major railroad and am constantly walking on uneven surfaces,It seems my knees goes dead,then my hip gets really sore, I feel myself walking differently then my lower back starts aching badly. I guess my question is what can I do? I amnow toldi have a bulging disc in the l5 s1 area. Doctor: Hi, As you mentioned that you have two ACL reconstructions previously but still you don't have ACL in your left knee, so whether the left ACL failed again or it is the right knee for which ACL reconstruction done twice. Whatever the case if you are involved in tasks that involve walking frequently and on uneven surfaces you need to have ACL reconstruction on the left side. Secondly, for disc herniation at L5-S1 level, further line of management depends on the amount of herniation as well as on the severity of symptoms and their response to conservative treatment. Usually, we do surgery for disc herniation in case when conservative management fails over a period of 3 weeks, or their is acute and progressive neurological deficit or their is severe uncontrolled pain. Take care. Hope I have answered your question. Let me know if I can assist you further. Regards, Dr. Rohan Shanker Tiwari, Orthopedic Surgeon"
},
{
"id": 195669,
"tgt": "What could cause testicular pain?",
"src": "Patient: Hi Doctor, Thank you for such service first of all. I am a 23 year old male having this issue or concern for about almost 2 years. It first started during the summer before the past one with pain on my right testicle and constipation. I looked around online and I see that constipation may cause testicular pain. At the beginning the pain was on the right top of testicle and side. It was not a 24/7 pain. it comes goes. but most of the day around 90% of the time I had pain. Since I did not have insurance I did not consult with the doctor. I did take home remedies such as prune juice, dates, and sometime tried dulcolax for the constipation, eventhough it did not work as I needed. Once I got the insurance about a year later I went to the doctor, but testicle pain was not bad as it was before but constipation got worse. Once I did couple of appointments with the doctor he prescribed me to do a colonoscopy and result came back negative. They said it is IBS. Since I had/have gas trouble also they did endoscopy only thing it showed on the endoscopy was some marks were on my beginning of the throat. He took a biopsy of it and result came back negative. Also we did ultrasound of the abdomen and it came back negative as well. Once the testicle pain came back again my doctor prescribed me to a urologist and he did a physical examination and told there is an infection on my epididymis. prescribed me sulfamethadoxale tmp ds for three weeks and it didnt satisfy what my problem. He then did a ultrasound and only thing he found out was there is extra fluid but that is something not worry about and that is not the reason of your pain I am worrying about. About the sexual activity. Due to my backpain, One time I went to spa where they offered masterbation I received it. with my gf I just had oral sex. So since this is something I was worrying about urologist did test for chlamydia and gonorrhea and both result came back negative. I have lost around 15 pounds from the day I saw my physician and I was in so much worried and tensed during these days. As of now my condition is this, sometimes i get narrow bowel movement(amount of total stool- only a little), sometimes thick one in the beginning and small pieces after that, sometimes I have abdominal pain, left side of my stomach feels something blocking there, gas, bloating(not always but quite often). Mucus on my stool(some time)Urination- sometimes too often, when I push hard to go to bathroom I have clear discharge from my penis, 3-5 times I have seen white mucus in my urine. some times I have seen my urine is cloudy. When I go to bathroom in the morning, I have seen that my testicle gets larger and feels that all the fluids filled in there that is really uncomfortable. I feel pain or discomfort on right side of the groin it only happens some times and feel pain and discomfort behind the testicle. I do have gas/bloating. I do have a doubt if I lost appetite or not. Any kind of inputs are appreciated in advance. Thanks. Doctor: Hello and Welcome to \u2018Ask A Doctor\u2019 service. I have reviewed your query and here is my advice. Since the gastroenterologist diagnosed as IBS your all symptoms could be because of this. Regarding testicular pain you need to visit the urologist and take a different antibiotic may be for 3 to 4 weeks till the pain subsides. Hope I have answered your query. Let me know if I can assist you further."
},
{
"id": 42665,
"tgt": "How to improve sperm count and quality through medicines?",
"src": "Patient: I am a healthy person but my sperm count is low 13million , The Active motility of my sperms is 0%. While 30% is sluggish. I dont drink/smoke/take any drugs, Will I be able to father a kid. I am ready to take medicines.Which Multivitamin/medicine would you recommend? Doctor: Hi,Thanks for writing to HCM.With this semen report the chances to become father are less. For my patients I suggest LYCo-Q or paternia tablets which have shown good results. You can discuss with your doctor.Important thing to remember in this is no drug gives 100% results and it takes long duration to show improvement.I would suggest that you rule out varicocele in you by a doctor. If it is there, get it operated. And do some life style modifications like --\u00a0\u00a0\u00a0\u00a0\u00a0Reduce weight, do exercise.-\u00a0\u00a0\u00a0\u00a0\u00a0Avoid caffeine.-\u00a0\u00a0\u00a0\u00a0\u00a0Ware loose under garments.-\u00a0\u00a0\u00a0\u00a0\u00a0Avoid trauma to testis.After taking tablets for 3 months I usually advice to get repeat semen analysis. If same state is there then better to go for IUI. It is intrauterine deposition of sperms. Here healthy sperms are selected and placed inside the uterus. You can try 4 to 5 cycles of IUI before proceeding to IVF.Hope I have been helpful.Regards\u00a0\u00a0\u00a0\u00a0\u00a0Dr. Ashish Verma"
},
{
"id": 219935,
"tgt": "Can a person have sympathy symptoms while living with a pregnant female?",
"src": "Patient: I have a question. My cousin and I are EXTREMELY close, her boyfriend, her, myself, and my boyfriend hang out a lot together. Her and I are 11 yrs apart, but we do everything together. I just moved to Virginia from South Florida about 3 months ago and we have always been close since I was young but now since I am up here and we live together now everything we do is together. She is about 6 weeks pregnant, I have been there since the day we found out she was pregnant. She has symptons such as excessive farting, cramps, is tired a lot her boobs hurt all the time and are getting bigger. She says that she is always hungy and craves vegtables, all the time. Also, she gets her fair share of morning sickness, and crazy dreams. She also trips over her words and forgets things, and her eye twiches. Now, I seem to be having some of these same symptoms, but because I am a girl, I am wondering if I am just having sympathy symptoms or is it a possiblitly that I am indeed pregnant. It is to early to take a pregnancy test, it is to early to test myself becuase I am not supposed to get my period for another week or so. But I know that her boyfriend is having a few of these sympathy syptoms, and he doesn t live with us. But in my case I am wondering if I can have these symptoms even though I am a woman. Doctor: HI, I understand your concern. Sympathy symptoms are called Couvade symptoms in male partner of a pregnent lady due to psychological involvement in pregnancy leading to some hormoanal changes in would be father's body.. causing few similar symptoms like pregnant woman. How ever same hormonal changes are not expected in a near & dear female friend's body & are . not reported . However intense desire to conceive can cause such changes in a female body longing for child birth Here also hormonal changes result due to psychological factor& is called as Pseudosyesis. IN your case the pregnancy can be confirmed / excludedby blood HCG test around expected day of periods /after delay in periods by 8 days ( Home pregnancy test ). Thanks."
},
{
"id": 118496,
"tgt": "Can clot in leg be surgically removed?",
"src": "Patient: I had a blood clot, DVT in 2007. It was discovered I have the Factor5 Lieden chromosome, and is susceptible to DVT. Anyway I am currently on warfarin daily since the incident. My problem is the leg that the clot was in is larger than my other leg. It also has a darker color in the lower extremity, and when I run on the treadmill, it becomes to painful after 30 minutes. I was wondering if it would make sense to surgically remove the clot or try to dissolve it so there is no blood constriction. I am worried it will get worse as I get older. Doctor: HI, thanks for using healthcare magicA DVT is a clot that occurs in the deep veins. In the veins in the legs there are two sets of veins, the superficial veins (closer to the surface) and the deep veins.Normally blood goes in one direction towards the heart and there are valves that prevent blood from going backwards or staying in the legs.When a deep vein thrombosis occurs , the back up of blood can cause damage to these valves and dilatation of the superficial veins.Because blood is no longer efficiently flowing in one direction, there are permanent issues with swelling in the leg.This also contributes to the change in colour.What you are seeing is due to the damage caused by the clot that was there previously and does not indicate that a clot is currently present.There is not likely anything to dissolve.I hope this helps"
},
{
"id": 224828,
"tgt": "Should I-pill to be taken after having periods to be safe?",
"src": "Patient: Hi Doc, Me and my wife had sex without protection and very next day she had her periods..which were normal on the expected date. Now my worry is should we go for an i-pill or is it ok as she got into periods the next day..and hence surely will not get pregnant?? Doctor: Hello and welcome, If the very next day periods have occurred and they were regular then there is no need for an emergency contraception. She will surely not get pregnant. So do not worry at all. Hope this satisfies your query. Thanks for using HCM.\u00a0\u00a0\u00a0\u00a0\u00a0Feel free to ask any more questions that you may have. Dr Madhuri BagdeConsultant Obstetrician and Gynecologist"
},
{
"id": 105531,
"tgt": "Dark red circle on chest. Relief with anti allergic medicines. Stiff muscular strip formed. Cause?",
"src": "Patient: Hi, I had about 6 inches diameter dark red circle that extnded on the front of my chest to back of left hand shoulder calf and covering full armpit. This was about 4-5 weeks ago. I thought it may be due to some insect bite as I work in my yard growing vegetables. It did not itch . I took few anti allergy tablets and today its redness has diminished by about 90%. But today I noticed there is about 4x1 inches long muscular lump in my shoulder calf behind the left arm and close to armpit in an area that was covered by the large red circle described above. It feels like a stiff mucle strip and the skin above this stiff muscle is reddish color 2x2 inches rectangular size. There is little pain when I squeeze this red muscular strip. What could be the reason? Thanaaks in advance for your inputs. Gope. Doctor: Hello Gopu, It may be a form of urticaria since it has had a good response to antihistamines. Insect bite is a strong possibility but as it is coming up in different areas and different times it may be urticaria that has started after an infectious trigger. You can continue to use the antihistamines but have it checked out if you have symptoms of muscle aches, fevers, night sweats etc."
},
{
"id": 151029,
"tgt": "Operated of tumor in spinal cord, relative thinning of upper dorsal cord. What is the meaning?",
"src": "Patient: hello doctor, my father is being operated of tumour in spinal cord in 2010. and recently (nov.18.2012)we got his MRI report which is saying this ..... here is evidence of signal intensity alteration noted extending from D4 to upper lumber region with relative thinning of upper dorsal cord (D1-D4 level). post operative changes are noted in lower dorsal and upper lumber level in form of laminectomy and distorted posterior soft tissue planes. IMPRESSION - post operative case of cauda equina tumour shows. signal intensity alteration extending from D4 to upper lumber region ....? post operative changes ?? myelitis ...So doctor,what this is saying or what should we do now....it is curable or not? Doctor: Dear rolimishra, I compliment you for taking care of your father. I would advise doing MRI with contrast. Post operative changes can be differentiated from recurrence better if contrast is given. If there is no recurrence then you need not worry much and concentrate on rehabilitation."
},
{
"id": 136373,
"tgt": "Suggest treatment for swollen and painful ankle and feet",
"src": "Patient: I have feet and ankle swelling in both. Started a few days ago. At first thought it was just from the heat. But they have never been like this b4. I have diabetes, high blood pressure, and stage 3 kidney disease. Also have had 2 bypasses on my left leg. Doctor: Hi, Thanks for the question. You should seek medical attention as the swelling in your feet and ankles could be a reflection of deterioration in your kidney function. Best wishes"
},
{
"id": 38270,
"tgt": "Could stomach pain with fever be indicative of stomach flu or UTI?",
"src": "Patient: My 17 yr old daughter complained of severe stomach pain. I took her to the Er on Friday morning. Anything acute such as gallbladder, pancreas, appendix problems was ruled out. She did have an UTI and was given antibiotics and fluids along with morphine for pain and sent home. That might she had hard chills, back pain and her head and stomach hurts. She has fever up to 102 for the past 24 hours. I am try to give the antibiotics a chance to help but it doesn t appear to be working. I m leaning toward stomach flu. Doctor: HI, thanks for using healthcare magicIt would be best to give her the antibiotics are prescribed if she was diagnosed with an UTI. Antibiotics are required for effective treatment against urinary tract infections.If the antibiotics are not working for this infection then she may need another type of antibiotic (it is possible that the bacteria may be resistant to the antibiotics given).Urinary tract infections can cause chills, abdominal pain, back pain , fever.Stomach flu would normally be associated with a history of vomiting and diarrhea.I hope this helps"
},
{
"id": 41695,
"tgt": "How to achieve conception (2nd) when having azoospermia and hormonal problems (in wife)?",
"src": "Patient: Hi After having my first baby, ve not been able to conceive We went for a series of test and found out that my husband has low sperm count:5% And my LH is high and my progesteron is low. I took clomid for 2months yet no result Need advice on possible solution Doctor: Hi welcome to healthcaremagic.I have gone through your question.As your husband has low Sperm count i would first advise to do color doppler ultrasound of scrotum to rule out Varicocele and vascularity of testes."
},
{
"id": 9187,
"tgt": "Suggest remedy for dry skin",
"src": "Patient: I, am 49 years old lady. TFT (FT3 normal, FT4 normal in range) but TSH 6.91 uIU/ml Method ECI came out in my Star imaging and Path lab report on 26/4/2014 I have noticed that my skin became puffy and dry skin for last 7-8 days and i went to dermatologist in Kaya Clinic they hv suggested me face wash, Tube MOISTUREX and Diprovate S for my neck( to remove black spot / patch) . (at that time, i did not get my tsh report). But now i am worried as i hv seen my TSH reports on following: Blood sugar fasting : 97 mg/dl Blood Sugar PP : 117 mg/dl (seems to be OK) KFT : Boood Urea, Blood Urea Nitrogen, Serum Creatinine, Uric Acid, Calcium, T. Protien, Albumin) are within Normal Limits but : GLOBULIN 4.18 g/dl and A/G Ratio : 0.89 indicates ? I think some insect bited me on my neck or some other reason, forpuffiness. only due these symptoms, i underwent for blood test. Please suggest and clarify on the above. (Presently, my skin is becoming dry from cheeks, throat little bit thirsty, urine frequency : 6-7 times a day) I would also like to know diet chart to follow seriously as i m not much interested in medicine, but pls suggest if it cannot be ignored. best Doctor: helloAccording to your symptoms of dryness and your blood reports, it is suggestive of a thyroid problem. your TSH is a little on the higher side and indicates hypothyroidism.Your reports are of 2014. You will need to repeat the tests and see the current levels. This condition very commonly causes dryness of skin. Along with that, in long standing conditions it can also cause a mild swelling and puffiness of skin over a period of time. 7-8 days swelling maybe too short to be caused by thyroid disorder and could be due to an allergic/irritant reaction. Or, you may not have noticed it earlier. Have you applied any cosmetic over the area or undergone any procedure/facial etc,?Also going by your age, since it is close to menopause such symptoms are common such as swelling,redness, hot flushes of skin. dryness due to loss of moisture.etcAre you on any regular medication? or new drugs?Keep a check on your blood pressure as well.I suggest that you consult a physician or endocrinologist for the thyroid as you may need medication to bring it to normal. This could directly affect your skin drynessYou can continue the moisturex SOFT cream twice a day for all dry skin. Use a mild moisturizing / glycerine soap. Do not scrub over the areas.Diprovate S can be continued for 2 weeks n stopped slowly.Drink fluids, include iodine in your diet. Reduce consumption of cabbage, cauliflower.etc for the thyroid. Do consult with the physician regarding the best diet for you."
},
{
"id": 168142,
"tgt": "How to get rid of the itching and dry skin?",
"src": "Patient: My 2 and half years old daughter is suffering with severe eczema. She was born at 23rd week of my pregnancy weighing 1 pound only. Her skins is very dry and nothing we are doing seems to help. She seems to be allergic to everything under the sun. We mainly feed her milk and sometimes she is able to tolerate rice foods. Do you suggest any home remedies for the VERY dry skin and itching. She can not sleep in the night due to itching and since yesterday she is jerking/shivering in her sleep and while she is awake. Any advice? Doctor: Hi Thanks for writing to Healthcare Magic.I understand that yours daughter is suffering from Atopic dermatitis. Its important that the skin is not dried completely soon after bathing. Also its recommended that you do not use hot water to bathe. It should be lukewarm. Though this sounds silly , its important information. Also as soon as child is towel dried, its important to apply the cream for hydration all over body. This will help lock the moisture in skin. Use of full cotton cloths, socks. Avoid food items she is allergic to.Hope you find this information useful"
},
{
"id": 151667,
"tgt": "Prominent Venous channel in left frontal region of brain,Venus Angioma",
"src": "Patient: Yesterday the Doctor told that my nephew has got Prominent Venous channel in left frontal region of brain after analyzing the MRI report and told it s a case of Venous Angioma. He is around 22 years and having his final semistar in Engineering on 26th of April2012. he is very much depressed. Plz suggest what to for the treatment? Doctor: Hi welcome to H.C.M.Forum. he is having exam in this month. for angioma you can plan treatment after exams, as it is not an emergency. if dont know the fact hide it for few days. dont worry everything will be fine."
},
{
"id": 49060,
"tgt": "What is the treatment or ways to remove the 11mm stone from kidney?",
"src": "Patient: Hi Doctor, I had 11mm stone in left kidney and 11 mm left ureter , 8mm & 4 mm in right kidney. Met an urologist last week and had laser surgery and stent was placed in both the kidneys after the surgery(Dr. said stent will be removed after three weeks) After one week, i had automated flow of urine and i went to another urologist as the First Dr. who did the surgery was out if station. He told me to take X ray and uv scan and told me that stent in left kidney got misplaced. He removed the stent from left kidney. However the scan report shows still 11mm stone is there in left kidney and 8mm stone is there in right kidney(stent still there). stone which was in urter and 4 mm stone in right kidney is not there now As of now dr. said lets discuss once the Dr.(who did laser treatment) came from vacation. What should i have to do now.?? How to remove the 11mm stone in left kidney. Will 8mm stone on right kidney will not be there after stent removal? Doctor: Hi, You have B/L renal calculii with left ureteric calculus . Probably your ureteric calculus was removed if in mid or lower part or pushed in if around PUJ and followed ESWL ( can't comment as you have not provided the detail of your operation) hence not visualized in xray . In ESWL calculus are broken in small fragments so that it can pass through small passage (5mm) of ureter through urine and spent provide easy n broader passage preventing clogging of calculus . Fragmentation of calculus depends upon consistency , size , shape , hardness ....etc hence sometimes does not fragmented n requires multiple sitting . Now as stent misplaced following (which can happen) surgery or probably ESWL Not given to left side ( as B/L in case ESWL should be given in different sitting) and Fragmentation of Rt renal calculus was complete so they are still there ( and can be commented after detail of sx ) So in my opinion you need to have multiple sitting of ESWL to remove calculii or in your case PCNL would be treatment of choice . Do stone analysis test and maintain diet accordingly , drink lots of fluids .Hope this ll helpThanks n regards"
},
{
"id": 110949,
"tgt": "Suggest remedy for compression fracture",
"src": "Patient: Hi doc, m 28 yrs old and hv suffered compression fracture d11 due the fall on the stairs. This is my 4th day since d fall howevr today i felt more pain in d lower back than i had anytime before, moreover my right foot had a bit of pain too which then resulted in some light numbness and now it seems to b okay, is this all part of recoverin process Doctor: YOUNG AGE D11 WEDGE COMPRESSION FRACTURE WITH ANKLE PAIN FIRST PLEASE RULE OUT CALCANIUM FRACTURE IN THE FOOT VERY COMMON ONE MISSED IN THIS KIND OF INJURY . IF U CAN ABLE TO WALK WITH OUT NEUROLOGICAL DEFICIT YOU ARE LUCKY BECAUSE DUE TO THE SEVERITY OF INJURY SOME TIMES PATIENTS DEVELOP PARAPLEGIA .TREATMENT INCLUDE GOOD AMOUNT OF REST IN FIRST 1 MONTH AND AVOID WEIGHT LIFTING FOR 3 MONTHS ANALGESIC AS REQUIRED AVOID JERKY TRAVEL YOU WILL BE ALRIGHT IN COUPLE OF MONTHS WISH A SPEEDY RECOVERY GOOD DAY"
},
{
"id": 100579,
"tgt": "Can one be ingredient allergic?",
"src": "Patient: I enjoy Mexican food however, no matter which Mexican restaurant I choose, within 2 hours I have dizziness, light headed and, upset stomach symptoms, no vomit but cramps. Feel like diarrhea is coming on but it never does. Weakness and at times, slight case of cold sweats. Again, it makes no difference which restaurant-fast food Mexican restaurant chains or very elegant ones. Apparently there is one ingredient that they all use that, no other restaurant uses. I have a clean bill of health and no known allergies. Doctor: HI, thanks for using healthcare magicIt may be best to avoid the food altogether if you are not sure which ingredient is causing your reaction.The symptoms that you are experiencing indicate that you may be allergic or intolerant of some aspect of the mexican food.It may be possible to speak to an expert in this type of food to determine what special ingredients may be used. This way you may systematically determine which is causing your reaction.I hope this helps"
},
{
"id": 72965,
"tgt": "Suggest treatment for severe chest pain",
"src": "Patient: i have had a constant searing pain around my right rib cage for about six months now for about four days i have been hurting between my shoulder blades and in my right shoulder i had my gall bladder removed two years ago so i know it 's not that i also experiance nausea and pain in upper chest area could this be some type of cancer Doctor: Thanks for your question on Healthcare Magic.I can understand your concern. Possibility of musculoskeletal pain is more likely. Don't think directly about cancer. Get done chest x ray first. If chest x ray is normal then no need to worry for cancer or other lung disease. Avoid movements causing pain. Apply warm water pad on affected areas of chest. Take simple painkiller like paracetamol or ibuprofen. Don't worry, you will be alright with all these. Avoid stress and tension, be relax and calm. Hope I have solved your query. I will be happy to help you further. Wish you good health. Thanks."
},
{
"id": 27126,
"tgt": "Is it alright to take Vertizac for a month to control BP?",
"src": "Patient: My grandfather is taking dutas t for BPH. He was taking CCB for BP.but he has vertigo for last 3 days.BP was checked and it was 100/60.BP medicine was discounted.he has been prescribed vertizac tablet for 1 month twice daily before food.but myself as a medico told him to take it for 15 days.give suggestions. Doctor: Hello, Vertical is for symptomatic relief of vertigo, so as soon as the symptoms resolve your father can surely atop the drug. Two weeks shower more than enough, rather if it is persistent post 2 weeks, again a review visit to the diagnosis should be done to make sure we are not missing anything . Also the cause of giddiness might be the blood pressure and so stopping (though I would prefer gradual down titration) of the meds, should be enough. Lastly do keep a follow up of his blood pressure reading. Hoping I could help For further assistance you may contact me on HCM. @ Http://doctor.healthcaremagic.com/doctors/dr-priyank-mody/70273. Regards"
},
{
"id": 58985,
"tgt": "Detected with gallblader sludge particles. On medicines. How long will they take to dissolve?",
"src": "Patient: Hello Dr,I am 26years old married woman, weigh 66kg. I am detected with gallblader slugde particles. I am on medicine Rebium plus and Golbi 300 for 3months. I want to know will the suldge particles dissolve in 3months?And if i am planning to conceive, will gallblader sludge particles would cause any harm.Can I plan a baby now with these gallblader stones? Doctor: Hi,Welcome to HCM,You are young and having gallbladder sludge these are very small cholesterol particle,you have not mentioned about your cholesterol and Triglyceride level, You are taking OBT Ursodeoxycholic Acid-300 mg, it should help to improve bile flow and clear your gall bladder.Your weight is 66 kg if you are overweight then try to reduce weight it helps in reducing fatty liver.If there is no other health issue planning for pregnancy should not be a problem go ahead and good luck to you.Please take advice from Gynecologist regarding stopping these medicines and taking supplementation like folic acid.Hope this helps Take care.Good Luck.Dr.Akhilesh Dubey M.D."
},
{
"id": 14614,
"tgt": "Can lotrimin clear up all types of rashes?",
"src": "Patient: I practice hot yoga 3 or 4 times a week and just recently broke out with a rash. Small red raised bumps two look circular but no ring around and don't really resemble a ringworm. So I try topical like cortisone and neosporane. After a couple of days my boss suggested I try lotrimin and right away the rash is dissapearing and is not raised. The rash was on my chest and shoulder and one on my back. My question can lotrimin clear up other types of rashes or am I to assume it was in fact a ringworm, since the treatment is working. Doctor: Hello. Thanks for writing to us at healthcaremagicFungal infection can begin as small bumps which may later spread on to the surrounding skin. Ring formation is a later feature and may not be obvious when the infection is just starting and limited to a small area. Itching is a feature of fungal infection.Lotrimin is an antifungal, clotrimazole 1% cream and it would be useful for superficial fungal infections. It won't be effective for rashes of other etiology.Since your rash seems to have responded to topical clotrimazole 1% cream, therefore most likely it was a fungal infection. You must continue it for the required duration i.e 2-4 weeks for a complete cure, so that there is no relapse due to inadequate treatment.Regards"
},
{
"id": 148708,
"tgt": "Suffering from severe back pain due to spinal stenosis. Family history of spinabifida. Treatment?",
"src": "Patient: I have had trouble with back pains since I was a child, as my spine is curved, but as I ve got older the pain has got worse over time and had started to travel down my legs when walking for more than 10 minutes, recently it has got even worse, shorter time of how long I can walk, and in the cold it s even worse. I went to doctors as it was really hurting, and been diagnosed with Spinal stenosis , my dad s sister has spinabifida, my brother had to have a metal rod placed in his back when we was kids. will I be able to claim dla as this is effecting my health? and if so how do I go about doing this? Doctor: Hi there,Thanks for your query.You have not mentioned if you too have spina bifida.Due to spinal stenosis (narrowing of the spinal canal), there is compression over the nerves which supply the buttocks and lower limbs.Initial non-operative treatment with pain-killers, reduction of swelling with use of steroids can be tried. If there is no response, surgery to widen the canal and remove compression over the nerves, is the only option.You have not mentioned age. If the problems, including curved spine) have been since childhood, it may be a congenital condition. I am not aware of your country's laws, whether disability due to congenital conditions is entitled to compensation. You will have to consult the concerned authority to learn about the entitlements for dla.In case of any further queries, revert back to me THROUGH DIRECT PREMIUM QUERY (Paid), addressed to me personally. You will receive- almost immediately- my response in great details.If you find my response helpful and informative, do not forget an \u201cexcellent\u201d (5-star rating) to my answer, to ENCOURAGE all doctors- engaged in social service- to render sound advice to the FREE queries. Fond regards and have a nice dayDr. Rakesh Karanwal"
},
{
"id": 223874,
"tgt": "Does undergoing laparoscopy affect the birth control?",
"src": "Patient: I had a laparoscopy four weeks ago today.(june23).. Im on the depo shot but im due for a new one july3. Does the laparoscopy affect the birth control. Am i still fertile enough to get pregnant on it. I have been bleeding 3days now but after we had intercores the bleeding went away. Doctor: HelloNo, laparoscopy doesn't affect the action of the depo shot.Bleeding while on the shot can occur but so long as you're on time with your injections you should be protected against getting pregnant"
},
{
"id": 62020,
"tgt": "What causes lumps on belly button with leakage ?",
"src": "Patient: My son has 1 1/2 inch lump above hisbelly button that is very tender to the touch. A large are on the oUtside of the belly is red. Now his belly button is leaking fluid. He doesn t have internet so I m asking for him . He went to the doctor s yesterday and she said she thinks it s an infection but if it didn t go away in 48 hours to come back. Ok for belly button to leak? Doctor: Hi,Dear,Welcome with your query to HCM.Studied your query in full depth of its details.Reviewed it in context of your health concerns.Based On the facts,Dear, you need to Consult Surgeon /or paediatric Surgeon,to assess the tender / red leaking belly button.USG study with Contrast Study / or CT abdomen with Contrast Study, would fix the cause of -Omphalo-mesenteric duct cyst or it could be due to the Patent Urachal Cyst/ or it could be due to Umbilical polyp,or Umbilical Granuloma,Vitello-intestinal patent duct leaking due to the intestinal obstruction in the para-umbilical hernia.Tender red tender lump in the para-umbilical area could be from obstruction of the Para-umbilical hernia.Surgical Correction would be needed on urgent basis to deal with this leaking belly lump button with lump.This reply would help you to plan further treatment soon with your treating doctors.Best of Luck and early recovery.Welcome any further query in this regard,which would be replied in next session.Good Day!!Dr.Savaskar M.N.Senior Surgical SpecialistM.S.Genl-CVTS"
},
{
"id": 97915,
"tgt": "Have sever pre-ejaculation problem. What can I take to over come it?",
"src": "Patient: I am 35 yr old and having severe pre ejaculation problem, I have taken many medicine of ayurveda , alopathic & even homeopathic from different source, but not getting any result. Our married life become bitter day by day. I am not even lasting for 1 minutes. Please suggest me any medicine which I can overcome this problem. Doctor: Hi, Research shows that the average man will ejaculate with in 1- 3 minutes of penetration, while the average woman will orgasm after 12- 15 minutes ! With such a large gap it is no surprise that more than 30% of guys suffer from the pain of premature ejaculation. Here are some strategies for help. 1.Wear condam. Condoms reduce stimulation for most men, which should prolong the time before ejaculation. 2.Use analgesic creams and sprays. There are two benefits, [1 ] the man achieves ejaculatory control but maintains sexual sensation, [2 ] and the transference to is partner is greatly reduced. 3.Slow down intercourse. 4.Try stop- and - start - method. 5.Avoid \" Active \" positions and try \" Passive \" posiions. My advice, Most men gain more control over ejaculating as they get older, you are just 35 years young , try not to worry too much. If you routinely ejaculate less than a minute .Take ASWAGANDHA LEHYAM .It is the best sex tonic. Take VIAGRA CAPS to increase the time."
},
{
"id": 180006,
"tgt": "Suggest remedy for fever with foot pain in children",
"src": "Patient: Hi my son is three years old and for about three days he s been having an on and off fever of 101.6 and it s always right after he wakes up from his nap about noon. The fever goes away with children s Tylenol but returns the next day after the nap. And he complains about pain on his feet it seems like it hurts when he steps on them. Plz help Doctor: Hi...your son seems to be going through a viral illness. You are right in giving paracetamol. Itis usual in viral illness to have body pains and especially of the limbs. But only thing which you need to look for is - his activity when there is no fever and if he is active when there is no fever, you need not worry. If the fever doesn't settle in another 48 hours or if his activity is getting depressed or if his limbs are getting a reddish rash kindly consult your pediatrician.Regards - Dr. Sumanth"
},
{
"id": 83078,
"tgt": "Lupus, scheduled for kidney biopsy. Clitoris pain, abdominal pain. Should I go to ER?",
"src": "Patient: I have lupus . I am waiting to be scheduled for a kidney biopsy . In the meanwhile for the past three days or so I have had pain in my clitoris which I never had before. I thought it might be a yeast infections but for the itching and pain/soreness I have no odor or discharge. I do have abdominal pain and a frequently varying temp of 99.1 - 100.4 this has been normal for the past few months. The reason for the kidney biopsy is the lupus. I have protein in my urine for three months now. I also have crohns disease . I feel very fatigued and also am bloated. The pain is very concerning to me and I wanted to know if I should go to the E.R. Doctor: hi even if you dont have any odour or discharge it could be an infection, so you should definitely get it checked out"
},
{
"id": 39896,
"tgt": "What treatment is suggested for umbilical infection?",
"src": "Patient: Hi i have an umbilical infection and havw been put on antibiotics as there was greenish discharfe now white sticky discharfe not a lot but have noticed a little blood when cleaning very gently. While it was checked and diagnosed there was no blood. Shall i just continue on with the antibiotics and hope it clears or should i get it checked again because of the new blood issue? Doctor: If the injury to your umbilical area is superficial (involving only the skin), I suggest that you do the following:1. Complete your antibiotic regimen2. Clean the area with saline, and then with betadine. Apply gauze soaked in betadine and dress the wound. Do the dressing twice daily.3. Continue this for at least 7 days and then re-evaluate.You will be Ok.If the injury is deeper, however, consult a general physician."
},
{
"id": 286,
"tgt": "Can I conceive with low platelets level?",
"src": "Patient: hi. iam 29yrs old.....trying to conceive for at least 5yrs now......my husband and i had full check up but it came out normal. the only issue with me is i have low platelets and low iron level....just curious if platelets level low has anything to do with my problems in conceiving Doctor: Hello and Welcome to \u2018Ask A Doctor\u2019 service.I have reviewed your query and here is my advice.You can conceive. How much is the platelet? Any recent history of fever or joint pain. For iron, you can start taking tablets from 4th month.Hope I have answered your query. Let me know if I can assist you further.Regards,Dr. Sheetal Agarwal"
},
{
"id": 31913,
"tgt": "What causes bleeding whenever you come out of the toilet?",
"src": "Patient: i went to the toilet earlier and it hurt a bit but not as much as it has done before but there was some blood on the paper after(quite a bit really) and its now been 3 hours since and there is still some blood. this does happen but comes and goes but this is more blood than any other time. i have just had a operation and some of the tablets i have been taking for the pain may cause some issues. i also went to the doctors for this in october but its been fine since and i done nothing once i had been to him Doctor: as per your symptoms it points towards internal hemorrhoids and if you are experiencing constipation then definitely it points towards hemorrhoids or to anal fissure but thats bit more painful condition ,after reading your case scenario i would suggest you to use laxatives available in tablet and syrups forms.Lactulose is quite a good treatment for constipation and use more liquid in your routine and avoid meet and oily food along with daily exercise this will help you in improving your bowel movements and last but not the least avoid straining while defecation.Do remember your case scenario is deficient in not mentioning the type of surgery you had so the above mentioned treatment only implies if you are experiencing this thing."
},
{
"id": 20320,
"tgt": "What causes tightness in the chest?",
"src": "Patient: i just had an echocardiogram done few days ago i have a left bundle branch block i am about to be 24 years old. it showed i have a heart murmur and 2 leaky valves and thickening doctor said it is nothing to worry about but my dad had to have surgery for leaky valves. the doctor said the murmur is leaky valves is it possible that the leaky valves means nothing and i can continue to excercise and live life as normal with no check ups?my chest is tight and feels like its is hard to breathe doctor said it is anxiety Doctor: After being told that your heart is not normal, anxiety is completely normal. Many times that something does not appear to be normal, we cannot know what that will mean later. Minor valve leaks are sometimes tolerated for a lifetime. EKG findings also may be just something you should be aware of. A left bundle branch block is one of those things. Any time that someone wants an EKG from you, you should warn them that you already know that it is present. It also is of uncertain importance long term. However, it is a good idea to have someone listen to your heart and review overall importance to you every few years, just in case you change or we learn something in the mean time."
},
{
"id": 51741,
"tgt": "My 9 year daughter is suffering from IgA Nephropathy",
"src": "Patient: My 9 year daughter passes blood stained urine evry time she gets cold. she is suffering from IgA Nephorapathy. she is been put on prednisolone. now recentely doctor told she has developed hypertension. in this young age she is developing so many disease. is there any permenant cure for the disease? Doctor: i feel pity on that, blood in the urine every time if she gets cold is horrible. children are frequently prone to cold and cough, do yu mean every time she has blood in urine. please not to neglect meet a kidney doctor as soon as possible."
},
{
"id": 121057,
"tgt": "Are there exercises that will help the knee back into alignment?",
"src": "Patient: I was wondering if you could help me? I was kneeling down for about 10 minutes and when I went to stand up i couldn t straighten my right knee. I feel like my patellar tendon is stuck and needs to pop. I feel medial and lateral pain on the anterior side. Are there any exercises that will help to get my knee back into alignment or do I need to go see an orthopaedist? Doctor: Hello,Some warm packs on the knee coupled with ibuprofen or diclofenac gel locally will help improve your situation. Some light physical exercise like flexion and extension of the knee initially will help improve your situation. If your situation does not improve, consulting with a physiotherapist will be needed.Hope I have answered your question. Let me know if I can assist you further. Regards, Dr. Ilir Sharka, Cardiologist"
},
{
"id": 219331,
"tgt": "Are Susten 300 pills safe during pregnancy?",
"src": "Patient: Doctor advice me to take Susten 300 sr 1 Tab daily after Dinner, i want know that have any problem to take susten 300. please advice and i am 7weeks pregnant and also doctor said in my blood test they find thyroid 7.78. If it is anything affect my baby. Doctor: Hallow Dear,Susten 300 is a Progestational medicine. Throughout the pregnancy, Progesterone is required to maintain the pregnancy. It prevents the uterine contractions and stabilizes the pregnancy. Probably your doctor might have noticed some factors which might have indicated the possibility of instability of the pregnancy. Therefore he/she must have started Susten 300 to stabilize the pregnancy. So Susten 300 is very safe during pregnancy and necessary in Progesterone deficiency or Pregnancy at risk. One such factor which may affect your pregnancy is your Thyroid levels. In fact in Thyroid function test, T3, T4 and TSH are evaluated. I believe you have mentioned the level of 7.78 as TSH level. Yes, 7.78 TSH is high indicating that your Thyroid is functioning low. This may affect the pregnancy and/or baby adversely. Hence, you need to take Levothyroxin throughout the pregnancy. Please get the prescription from your Obstetrician. I hope this helps you today."
},
{
"id": 52631,
"tgt": "Is laparoscopy advisable for gall bladder removal?",
"src": "Patient: Hello Doctor, I am from Allahabad, Uttar Pradesh. I have been recently detected with chronic cholecystitis and fatty liver (grade -II). My gall bladder is densely packed with calculi forming a dense posterior acoustic shadow (WES complex). Around 1 year back, my gall bladder was having a singly calculus of 16mm size. I want to know if laproscopic surgery is possible for removal of my gall bladder, as the ultrasonologist doubted on it and suggested open surgery in my case. Kindly guide me. Doctor: Hello,You need an operation for gall bladder removal. Kindly attach your sonography report for giving more comment on that.If as per reports and examination laparoscopy operation possible, then better to operate by laparoscopy. Consult surgeon for examination and discuss your case.Hope I have answered your query. Let me know if I can assist you further.Regards,Dr. Parth Goswami"
},
{
"id": 177654,
"tgt": "Suggest medication for persistent cough in the night",
"src": "Patient: Hello Doctor,My son is 7 yrs old. From last Friday that is 31st Jan, we have noticed, he started coughing when he goes to bed at night. He doesn't cough the entire day and it only happens when he lie-down in bed. The same thing is happening for last few day and we are pretty much concerned.Doctor advised, Duolin + Budecort, we understand Budecort is steroid, is it advisable to take these medicine. The other medicine advised by doctor is Budesol, I think that\u2019s also is a steroid. What is the difference between Budecort and Budesol, should we administer this medicine, are they ok for children of 7 yrs.Thanks Doctor: Your child is probably suffering from a viral infection for which your doctor is rightly treating. It is not unusual to have the cough increase at night but remain fine throughout the day. Please continue the present treatment and things should gradually improve.Duolin contains a combination of Salbutamol and Ipratropium Bromide, both of which are bronchodilators and help in reducing the spasm that causes wheezing. Budecort contains budesonide which is a steroid and needs to be given in order to reduce the reaction that causes the bronchospasms. Budesal contains salbutamol along with budesonide and so has both the effects of duolin and budecort. Moreover, budecort and budesal are two different brands marketed by two different pharmaceutical companies.You should administer the medicine as directed by your doctor. It is a very common medicine and a lot of children get this medicine without any complications. So you need not worry."
},
{
"id": 143032,
"tgt": "What causes sudden mental instability?",
"src": "Patient: my wife has a chronic cough, she has been to the ER four times in the last two weeks, The next to last visit the prescribed her three different medications, Acetaminophen-codeine 300,Ventolin HFA 90 inhaler, methylpresdnisolone 4mg. She took these medications one day before I took her back to the hospital. she is now itching everywhere, swollen face and neck area, and has lost all of her mental thoughts. The doctors tell me that this is not a side effect to the medication but because she has pneumonia. she also has kidney failure, and missed treatment twice. They say it is nothing to worry about. She does not know who she is, where, when, or can hold a conversation with you. Doctor: Hello!Thank you for asking on HCM!I read carefully your question and would explain that her symptoms seem related to an allergic reaction. From the other hand kidney dysfunction coupled with pneumonia could mimic her troubles. The memory troubles could be relate to a temporary dysfunction of her brain due to her clinical situation (pneumonia and low oxygen saturation, kidney failure). For this reason, I recommend bringing her to the ER for an advanced treatment of all her problems. Dialysis may be necessary for her kidney function. She may need to stay on Oxygen therapy, besides antibiotics for her pneumonia. You should discuss with her doctor on these issues. Kind regards, Dr. Aida"
},
{
"id": 110883,
"tgt": "What causes lower back pain that radiates to stomach while standing/walking?",
"src": "Patient: have really sharp pains in lower back they penatrate around to lower part of stomach and when standing or walking over 10 to 15 min. fell like my thighs trying to lock so i cant walk the pain only stop when im laying down cant keep on doing this i have gain so much weigh from just working come home get in bed because im in so much pain havent taken no meds because i have low tolerances get sleepy just taken tylenol Doctor: welcome to Health care magic.1.Looks like for there is a nerve compression ( which generally give the above mentioned symptoms).2.Intially i would ask for a anterior - posterior and a lateral x-ray of lumbar spine, depending on the findings i would recommend basic exercises and lie on the flat bed.3.If that doesn't work, i will ask for MRI is best investigation for disc pathologies and chord compression.4.These symptoms could be seen Age related and underlying medical conditions.Anything to ask ? do not hesitate. Thank you."
},
{
"id": 202482,
"tgt": "Has constant masturbating from a young age affect my penis?",
"src": "Patient: Dear Dr im 25 and discovered that i have ED since i was 18. Ive been to several urologists and done all tests and all Urologist say im normal and i should go on daily cialis. I have done that and did not work with me.. cialis viagra and livrta dont work for me and i dont know why! I had an penile injection and had a full erection. I have been using face down masturbation since i was a child and i cannot reach to orgasm only by this way. I think the constant bending of my penis for masturbating has either efected my brain or harmed my penis. Please advise me on what step i should take next? Doctor: HelloLet me explain about sexual orgasm , penis size, erection , premature ejaculation.As you mentioned that you are doing masturbation since childhood .Viagra, livrta, cialis , all these drugs are nor working but injection causes erection .This means there is no defect in brain or hormone or in penis .As you mentioned that different urologist don't find any disease .In my opinion sexual orgasm reaches its climax when there is discharge both partners.Size of penis varies from 10-15 cm in full erection .Premature ejaculation means discharge at once within 2 minute in all sexual activities ( including fore play also ).As far as about you , in my opinion you are perfectly alright .This is a psychological problem , so consult a psychiatrist and get his opinion.Good luck."
},
{
"id": 175436,
"tgt": "How to treat a little itchy bite like spots on the chest of a child?",
"src": "Patient: Hi, my 5 year old daughter has woken up with what seems to be little itchy bite like spots on her chest and up her neck on the front? She has no fever but has recently recovered from tonsillitis with an anti bioticShould I be concerned?RegardKerri reubenson Doctor: Dear Kerri, it seems that your daughter has got these skin lesions in chest and neck either due to plain allergy or due to drug allergy from recent uses of antibiotic.these lesions will go automatically withot any medical intervention and you you dont need to worry much about this.thank you.my best wishes for your little one."
},
{
"id": 16816,
"tgt": "Suggest treatment for heart burn",
"src": "Patient: i am suffering fro heart burn for 2 years.i have done endoscopy.but result shows mild and moderate ulcer with no hp negative.i took sucral for 1 month.but hurt burn is still going on.i am taking ppi.but is there any permenent sollution? ppis have also side effect , so how i manage my heart burn............please give suggestion. Doctor: Hello, After going through your medical query I understand your concern and I would like to tell you that in chronic ulcer it is recommended for you to not to take hot eatables, eat them on room temperature and take mostly cold, avoid spicy, caffeine-containing products and carbonated drinks. Hope I have answered your query. Let me know if I can assist you further. Regards, Dr. Bhanu Partap, Cardiologist"
},
{
"id": 115892,
"tgt": "Suggest treatment for purple lesions on the arms due to subcutaneous bleeding",
"src": "Patient: My forearms began subcutaneous bleeding three years ago. The ugly deep purple lesions about the size of a quarter appear suddenly without any trauma and take about three weeks to go away. I am 75 years old. Have AFIB since 2002. Was on Coumadin until 2012, when I began to see grapefruit size purple lesions on my back. Changed to aspirin 325 mg daily and that worked OK until last January when I started to get nosebleeds about 1 - 2 times a week. In February I changed the dosage to 81 mg daily and the nosebleeds stopped. I take metoprolol 100 mg twice a day and diltiazem 120 mg SA once a day since 2002 for my AFIB. My primary care doctor says my skin is too thin because of my age. My dermatologist says the lesions are not treatable. Can anything be done to stop this? Doctor: Hi,Thanks for asking.Based on your query, my opinion is as follows.1. You are at present having senile vasculitis.2. Avoiding injury or rubbing your skin on rough areas is very important.3. Also, good hydration and applying moisturizers on skin, to prevent skin dehydration is important.4. The purple areas will slowly become darker and disappear. It wont be painful. At joints, cushion and other places, light protection could be helpful. It is not treatable, but further injuries can be controlled.Hope it helps.Any further queries, happy to help again."
},
{
"id": 104448,
"tgt": "Acne, sensitive skin, Applied prescribed cream, allergy on face, swelling, itching, redness, dryness, pain on face, open pore. Cure for scars and open pores?",
"src": "Patient: Hi, I have acne on my skin. I took homeopathic medicines. Acne is still there. Now i have consulted with Allopathic doctor. My skin is very very sensitive. She gave me these medicines: 1. Emodel lotion (as a face wash) 2. UVRID sunscreen lotion for UVA nd UVB rays. 3. Glyco 6 cream (apply for 2 hours only) 4. Clyndamycin cream (apply once at bed time) 5. Dr. Reddy s Antioxidants to eat (daily 1 capsule) I had started this course and after 2 days there were allergy signs on my face. Swelling ,itching,redness,dryness and pain evrything was there on face. Then i took Medrol, Allegra and flutivate(ointment). Right now the doctr is saying that you can continue the medicines now but again after 2 days of applying the medicines there are symptoms of allergy. I dont undertand what should i do for my acne and scars. There are open pores also. After 1.5 months i will get married. Before that i want to get rid of this scras and open pores problem. Please suggest. Please revert in mail if posiible. Regards, Ankita India Email ID: YYYY@YYYY Doctor: Acne is normally in oily skin Sto taking Greesy and oily food no use of creams oils on face and body no musterd coconut walnut groundnut almonds and soya use olive for cooking and application if need Take doxi 100 mg bd for 3 week Apply clindamycin gel on face morning evening apply retinoic acid at night and swich off lights if dry you can add little fluticasone ointment do for 3 months"
},
{
"id": 18666,
"tgt": "What causes an angina despite following a strict diet?",
"src": "Patient: I eat lots of salads everyday (big bowl for lunch) do not eat any fried foods at all. But recently I found breathing hard going up a hill (one that I usually run). I do not eat bread, nor do I eat red meat, yet my doctor said I just suffered angina. How is that possible? Doctor: Hello and Welcome to \u2018Ask A Doctor\u2019 service. I have reviewed your query and here is my advice. I would explain that it is important performing a comprehensive differential diagnosis between different possible causes that may lead to this clinical situation: chronic anemia, a lung disorder, a cardiac disorder, etc. For this reason, I would recommend consulting with your attending physician for a physical examination and some tests to investigate for the possible causes: - a resting ECG and a cardiac ultrasound - cardiac enzymes - a chest X-ray study and pulmonary function tests - blood electrolytes. An exercise cardiac stress test may be needed. You should discuss with your doctor on the above tests. Hope you will find this answer helpful! Kind regards."
},
{
"id": 124601,
"tgt": "How long does carpel tunnel syndrome take to get cured after a surgery?",
"src": "Patient: EVERY NITE MY LEFT HAND WAKES ME UP WITH SO MUCH PAIN YEARS AGO A DR. IN NJ SAID I HAD THE START OF CARPEL TUNNEL IN BOTH HANDS BUT NOW THE LEFT HAND IS FAR WORSE THAN THE RIGHT I WEAR THE BRACE EVERY NITE NOW WHEN I SAW MY PC DR. SHE SAID WOULD I BE INERESET IN SURGERY NOW IM WONDER HOW LONG IT TAKES Doctor: Hello, Surgery is the best possible option for carpal tunnel syndrome. It is a simple procedure which can be done under local anaesthesia. You will improve in a short time following the procedure. Long term physiotherapy is required for best results. Hope I have answered your query. Let me know if I can assist you further. Take care Regards, Dr Shinas Hussain, General & Family Physician"
},
{
"id": 108365,
"tgt": "What causes mild lower back pain with light headedness?",
"src": "Patient: Hello. I've been experiencing extremely mild lower back pain for the past few days, from my several months of sleeping on the couch. (Unfortunately, that's the only option for my sleeping.) While showering today, my back was still bothering me and I tapped it a bit in hopes that the pain would go away. Instead it intensified, and I can't even bend down to touch my toes. Additionally I feel light headed and a bit short of breath. What should I do? Doctor: Hi, I have gone through your query and understand your concerns. You should be worried but not too much.In my opinion you are probably having spinal issues like prolapsed inter vertebral disc / nerve root compression at spinal level, which is caused by your couch.i will suggest you to take some NSAIDS ( Non steroidal anti inflammatory drugs) if you are not allergic to them. and use a flat / hard bed for sleeping. I will suggest you to apply some pain relief gel at bed time.you will be fine soon.wish you a good health.Thanks & Regards Dr. Shiv"
},
{
"id": 50086,
"tgt": "Diabetes, fluid retention, ultrasound showed splenomegaly. What to do?",
"src": "Patient: SIR WE HAVE DONE FOLLOWING TESTS (CASE HISTORY) PATIENT NAME-PREMALATA BISWAL,AGE -76 YRS,(FEMALE)HAVING LONG STANDING DIEBETIC TYPE-2,COMPLAINED ABT ASCITIC FLUID RETENTION.1ST WE DID U S OF ABDOMEN & PELVIS .RESULT OR IMPRESSION:CHRONIC MEDICAL RENAL DISEASE GRADE -III,MILD SPLENOMEGALY ,MILD TO MODERATE ASCITES ,THEN WE DID FLUID EXAMINATION-FLUID EXAMINATION(22.09.2012) : SPECIMEN-ASCITIC FLUID,COLOR-PALW YELLOW,VOLUME-08 ML,CLOT-NIL,SUGAR(GLUCOSE)-1588 MG/DL,PROTEIN-1.2MG/DL, ADENOSINE DEAMINASE ACTIVITY-08 U/L,GRAM STAIN-NO BACTERIA SEEN,Z.N STAIN OF AFB-NEGATIVE,CYTOLOGY TOTAL COUNT-225 CELLS/CUMM,DIFFERENTIAL COUNT:CYTOSMEAR FROM CENTRIFUGED DEPOSITS SHOWS: LYMPHOCYTES-70%,MESOTHELIAL CELL-25%,POLYMORPHS-5%,NO MALIGNANT CELL SEEN.AFTER WE DID CA 125,RESULTS 42.00 U/ML,CA-19.9 RESULTS-126.91 U/ML,AFTER THIS WE PERFORMED PLAIN & CONTRAST(BOTH ORAL & IV)ENHANCED CT SCAN OF WHOLE (29.09.2012)ABDOMEN,IMPRESSION:FEATURES ARE SUGGESTIVE OF 1)MODERATE ASCITES,2)MILD THICKENED ANTERO-PYELORIC WALL OF STOMACH,3)RIGHT RENAL CYST.4)MINIMAL RIGHT SIDE PLEURAL EFFUSION.SUGGESTED :UPPER GI ENDOSCOPY.AFTER THIS WE DID ENDOSCOPY ON 30.09.12 IMPRESSION- FEATURES OF GASTRO-DUODENITIS.KINDLY SUGGEST WHAT TO DO? Doctor: Her ultrasound findings are suggestive of chronic kidney disease (CKD) however there are many tests which are needed to confirm it.1) Hemoglobin and CBC2) Kidney Function Tests3) Urine routine and 24 hours urinary proteins4) HbA1c levels5) ECG6) EchocardilographyAlso the sugar levels in ascitic fluid are very high indicating that her diabetes is uncontrolled.Accumulation of fluid in abdomen can be a part of diabetic kidney disease as well as of associated heart disease.All these reports are required to know that.Meanwhile, she shouls reduce her sugar, salt and water intake and take medicines like Lasix or Dytor if not taking already.Should get in touch of a nephrologist in your city."
},
{
"id": 206946,
"tgt": "What causes abnormal feeling?",
"src": "Patient: I was working out this morning and it felt like the air I was breathing was moving through my ear. I don t think air was actually moving through my ear, but it was very loud. Swallowing seemed to help and it hasn t happened again today. What is causing this? Doctor: this is due to blockage of nose and your canal connecting ear and throat get communicated make you feel in such a way. so do not worry. take carehope my answer helped you.take care."
},
{
"id": 181032,
"tgt": "What does a bump on the right side of the uvula indicate?",
"src": "Patient: I have a small bump on the right side of my uvula. My primary Dr. saw it on a check up. He referred me to an ENT but they cant see me for over 3 weeks. Should I try to find another ENT? I also have a sore throat and a long history of sore throats. Plus going to get ultrasound for abdominal pain. (probably unrelated). Doctor: Hi..Thanks for the query..A bump on the uvula can be due to causes like a mucous cyst or a Mucocele..If it is painful and inflamed it can be an Abscess..You should consult an ENT Physician or an Oral Surgeon and get evaluated so that the exact nature of the lump can be ruled out..If needed an FNAC can be done..For now you can do warm saline gargles.Gargle with numbing mouthwash containing Lidocaine..Avoid hot and spicy foods and drink plenty of warm water..Hope this helps..Regards.."
},
{
"id": 215550,
"tgt": "How can chronic pain be treated?",
"src": "Patient: My father is 86yrs old at a rehabilitation home (physical therapy) forced to be graded as chronic when he is actually acute with serious complications. He is not receiving diagnostic testing & is in severe pain . Woprse then usual. He was put on dilaudid q24, tramadol 1q6prn. NOT WORKING!!I( m at my wit s end , I can t seem t get any help. Doctor: Hello, As a first line management you can take analgesics like paracetamol or aceclofenac for pain relief. If symptoms persist, it is better to consult a physician and get evaluated. Hope I have answered your query. Let me know if I can assist you further. Regards, Dr. Shinas Hussain, General & Family Physician"
},
{
"id": 183917,
"tgt": "Suggest treatment for toothache and facial swelling",
"src": "Patient: i was told about 2 months ago that I needed a root canal and haven't had the money to take care of it. I often have major pain in that tooth, it sometimes feels loose and I can't bite down on food without it hurting. This morning I woke up and my entire cheek, from my eye to my lip is swollen\u2026I'm sure I need to have this tooth pulled but is this a very urgent matter? What is the cause for the swelling? Doctor: Dear sir/Mam, I understand your problem.Every tooth has three layers .The third innermost layer has nerves and blood vessels.when the tooth decay is progressing in first two layers we will not have pain since they dont have nerves.When the decay enters the third layer , it damages the nerve and blood vessels.These decayed tissues will develop pus.This infectious pus will not have enough space inside the root canal.so it spreads up through the root tip by resorbing bone.Thus there will be bone resorption and pus collection around the root tip.Thats why you feel loose and painful when you chew something since you increase the pressure inside the pus collection.when the pressure from pus collection keeps on increasing , it will keep resorbing the surrounding bone and comes out as swelling of cheek.so permanent solution for this condition is to clean the pus filled tooth with root canal treatment.If you are not affordable at this time atleast take some antibiotics like cefixime 200mg twice daily for 5 days to control the pus collection.But this problem should be taken care of before 6 months(maximum) to prevent loss of tooth and bone structure.Hope your query is answered.please dont hesitate to contact back for further queries."
},
{
"id": 41838,
"tgt": "Is pregnancy possible with excess testosterone?",
"src": "Patient: infertility I have been trying to get pregnant for over 10 years and I just started asking for help. I had a cyst when I was 15 and took birth control for that. also I have to much testatorome. I don't know what that is. will I be able to have children? I'm 25 years old. and also have irregular periods. Doctor: yes with medications it is definitely possible.Dr. Prerna Gupta MBBS (AIIMS), MD (AIIMS), (Gold Medalist)DNB, MNAMS, MRCOG (UK)Commonwealth Fellow in Reproductive Medicine (BCRM, UK)consultant Moolchand prernagadodia@gmail.com"
},
{
"id": 201955,
"tgt": "How to eat healthy and have more energy in life?",
"src": "Patient: I'm 35 yrs old man working for an IT company. I feel really tired, sometimes my chest pains, sometimes i get migraines, sometimes severe headaches and giddiness. My diet is rice, wheat and veggies. sometimes i eat irregular. my wife says i eat more. what shld i do to get better energy and live better?? Doctor: HIWell come to HCMIt is very simple just cut down the calorie contained diet, and have more proteins and fiber contained diet, have more vitamins and this can be got from the fresh vegetables and fruits, that's all, hope this information helps,"
},
{
"id": 24492,
"tgt": "Does hypertension cause soreness in rib cage along with dull pain in chest?",
"src": "Patient: I ended up tearing a right shoulder muscle and ripped some cartilage around my elbow. The pain was intense and my regular physician gave me a toradol injection for the inflammation and 6 days of Prednisone. My BP was to almost 190/100. Usually I am in the 100-115/55-70 range despite smoking. The pain got worse the next day and since I felt my BP elevate as well, I went to the ER. My BP was 249/110 and they took me to an ER room. The doctor was ordering MRI s and when I told him I was on workers compensation, he suddenly decided that all I needed was xrays and only for the right shoulder at that, as if soft tissue damage will even show clearly on xrays. He kept saying I worked for WC, you can have them do all those haha. I am going on year 9, and it took 3 years to get one body part accepted while after 9 years, I still litigating over other injuries. Anyway, he left me there for 25 minutes after I had 2 techs get xrays; he speny maybe 90 seconds with me, saying this is an ER, I don t have time to listen to all your problems when I was trying to tell him what had happened. He put down calcific tendonitis (misspelt it as well), torn muscle, and elevated BP on the report. Then a nurse came all grumpy, telling me to take a clear grl yhst eas roughly the shape of a 1,000mg fish oil but had a yellow/orange color. I asked him what he was but all he told me was we give this to everyone with high BP . And they read my vitals again which showed 182/5minutes later. He adked the doctor if they should release me, and when the doc said yes, that was it. The ER was at maybe 20% capacity while the doctor eas sotting at the nurses station and chatting away with a nurse. It has been 5 days and my BP never shot above 140. However I have this dull, almost bruised feeling on my left back, right around the 5th rib which feels like a bruise. It almost feels as if it is under yhe bone and cartilage, and straight on the heart but I can t really tell and nothing makes it go away. I wasn t checked or even asked if I displayed any heart attack symptoms or anythong else other than you can get WC to take care of that . Is it possible that the high BP may have caused something or even bruised my heart somehow, or is it possible that I may have had a minor heart attack? My BP was over 180 for 2 days straight, especially considering it rarely goes above 130. And now that I checked thr online patient portal, my 240+ intake BP has also been removed from the transcripts. I am one step from filing a complaint for alst letting me die. Doctor: Hi,You had elevated blood pressure because you were taking Prednisone. Medications like prednisone, steroids, have a side effect of causing high blood pressure. But you should check your blood pressure also after stopping prednisone, as it is possible that you had slightly elevated blood pressure from time to time and you didn't know about it, and prednisone just made it worse.Your pain is related to your muscle and cartilage problems, don't worry, it is not a heart issue.Take careCome back if you have any further questions"
},
{
"id": 94371,
"tgt": "Pain in the lower abdomen after intercourse. Reason?",
"src": "Patient: Goodnight I had intercourse last night, A hour later i was waken up by a bad stomach pain in my lower abdomen . I when to the bathroom to pass urine and it was pain full as it cam out. It has been about 6days since my boyfriend and i had sex so i dont know if this could have caused it. I still have to discomfort in the abdomen now and if i sit down to hard i feel it. Do you have any idea what it can be? Doctor: Hello! Thank you for the query. Such symptoms are most likely caused by urinary tract infection. Intercourse can trigger such infection. Frequent urinating, burning while urinating and lower abdominal pain sometimes radiating to the back are characteristic for this conditions. I suggest you to have urine analysis, blood work and abdominal ultrasound performed. Antibiotics seems necessary. You should also drink more mineral water. Hot baths should ease the pain. Hope this will help. Regards."
},
{
"id": 104776,
"tgt": "Had flu, no strep, lost voice, tiredness, seal bark, stuffed sinus nose, lump in throat. Bronchitis?",
"src": "Patient: I had flu , Inf A per test, no strep , lost voice for four day, lung xray clear 4 wks ago. Still very very tired, have seal bark, stuffy sinus nose, and difficulty breathing, feeling of lump in throat, clear mucus coughs up three to 8 times a day, very very hoarse voice (this part I worry about). I have asthma, nebulizor w meds makes voice and chest feel worse, antibiotics doing nothing after 1 1/2 weeks on Cephlixin for what Doctor said was bronchitis. Not sure if bronchitis. Throat hurts in front when I touch it. Female 41 Doctor: Hi ! It seems to me like mixed oral infection, some time too much asthma pump make condition worst. Then, if pain on throad it could be due to cold or thyroid gland found in neck. What to do ! Take worm water gargle, inhale vicks. And ask doctor for oral infection antibiotic. And if you have more mucus then normal it could be Tuberculosis. Do consult these things with your doctor. Thanks. Bye."
},
{
"id": 30448,
"tgt": "Suggest treatment for weakness along with kidney pain while suffering from swine flu",
"src": "Patient: my boyfriend was diagnosed with swine flu and a kidney infection (after about 2 months) in February this year. Two weeks after he began getting waves of lethargy and kidney pain again, which is still going. What could it be, as we are both really worried? Doctor: Hi, dearI have gone through your question. I can understand your concern.He had swine flu and kidney problem. Now he has kidney infection. He should go for urine routine and microscopic examination and ultrasound abdomen. It will give you exact diagnosis whether he has urinary tract infection or any other pathology. Then take treatment accordingly. Hope I have answered your question. If you have any doubts then feel free to ask me. I will be happy to answer.Thanks for using health care magic. Wish you a very good health."
},
{
"id": 204510,
"tgt": "What causes mixing up of words in a sentence?",
"src": "Patient: I occasionally use the wrong word in a sentence. For example, today I was talking about my daughters car and referred to it as a celica when I knew full well it was a civic. Usually, the wrong word begins with the same letter...but not always. Any ideas? Doctor: Hello and Welcome to \u2018Ask A Doctor\u2019 service. I have reviewed your query and here is my advice. Excessive stress and lack of attention and concentration often causes these problems. I must acknowledge that almost all of us have some tongue slips and it is not necessary to have a pathology for same. In my opinion stress management can help you immensely. Hope I have answered your query. Let me know if I can assist you further."
},
{
"id": 4824,
"tgt": "Irregular periods, pinkish to darker discharge, chance of getting pregnant. when should i test?",
"src": "Patient: Hi. I don't really see any answers online that help and this is the first time I have had this happen. I had my menstrual cycle 13-16 of last month. Three days ago I started lightly spotting. It started out pinkish and getting darker. I don't know what would be causing this. I'm 28 years old and never had it happen and there is a chance I could be pregnant. When should I take a test? Doctor: Hi and thank you so much for this query.It is best to take a urine pregnancy test at least after having a missed period. This allows sufficient time for the levels of the hormone to rise too levels that can easily be detected. Tests done before this date may give false negative results because of low levels of the hormone in the body. So after 16th of this month, you can conveniently do so.If you long to become pregnant, I wish you luck with the outcome. However, if you don't want to be pregnant now, consider adopting and correctly using a contraceptive method. I hope this helps. Thanks for using our services and feel free to ask for clarifications.Dr. Ditah, MD"
},
{
"id": 219993,
"tgt": "Suggest treatment for cyst in the right ovary",
"src": "Patient: i had a vaginal ultrasound, the result showed that i have a 5 cm. cyst in my right ovary and that my endometrium is enlarged to 20.2 mm. i am 37 yrs. old and i am delayed for my menstruation for 45 days counted from the first day of my last menstruation, i am not pregnant, im suffering from nausea and dizzeness for more than a month already. do i need a biopsy test? Doctor: yes, it will be the correct choice of treatment sp long the pregnancy test is negative. the ultrasound findings are not normal. Endometrial biopsy will be the able to determine any pathology or disease process in the uterus."
},
{
"id": 142035,
"tgt": "Is severe body pain related to Microvascular disease of the brain?",
"src": "Patient: I am a 73 year old female. I have been diagnosed by a Neurologist as having a moderate amount of micro vascular disease of the brain. Some current symptoms include dizziness, stumbling, falling, and very poor memory, especially short term. But, the thing I am having the most trouble with is pain. I have a RIGHT hip total replacement. The PA at my surgeon s office assures me that nothing is wrong with the hip, and has referred me to a back specialist in their practice. The pain is very severe in my lower back, and travels down my right leg into the foot. Then both feet ache severely. Often the pain is in my pelvic area, in the bones of my seat, and almost seems to mimic menstrual cramps. Often the pain is in the groin. The pain seems to migrate in stabs to almost any part of the body, hands, feet, calves. and any joint. My question is, could this ever increasing pain be related to the Micro Vascular disease? I do plan on following through with the Back Doctor. Doctor: Hello!My name is Dr. Aida and I am glad to attend you on Healthcaremagic!Your symptoms could be related to sciatic nerve, which is a possible pinched nerve, originating in the lumbar region. There is no connection between this issue and the microvascular disease of the brain. It could be caused by a bulging intervertebral disk in the lumbar column. For this reason, I recommend performing a lumbar column CT scan and a nerve conduction study. Meanwhile, I would recommend avoiding prolonged sitting or standing up position. Hope you will find this answer helpful!Best wishes!"
},
{
"id": 61868,
"tgt": "What causes lumps in lower stomach?",
"src": "Patient: hi im 26 yearsold, i keep getting bloated a lot , im a skiny person and decided to lie down and feel my stomach. on the inside of my left hip don t towards lower stomach i have a couple of lumps and thought it may be my bowels, but i had bowels open and still can feel it, what could it possibly be, kidney stones, cyst or something serious. Doctor: Hi,From history it seems that there might behaving few enlarged inguinal lymph node giving this picture.Or if lumps are still above can be few hard fecoliths giving feeling of lumps.Consult your doctor and get examined.Ok and take care."
},
{
"id": 88938,
"tgt": "How to cure stomach pain after eating?",
"src": "Patient: I have been having sharp stomach pains after I eat and for the past couple days my legs and arms just start having pains. I can't explain it very good. I thought I might be pregnant but I also haven't had my period since the first time I had sex a few months ago and the tests turned out negative. I am worried about the stomach pains I have been having. What could be wrong with my stomach? Doctor: Hi there,Welcome to healthcaremagic.I read your problem and the first thing which came into mind is gastritis or gastric ulcers.I would ask you to take omeprazole or pantoprazole 40mg tablets for a month every morning empty stomach and see if you get relief.If you do not feel good after this you should go for an upper GI endoscopy.In the meantime,Don't smoke, and avoid coffee and alcohol. These habits increase gastric acid production and weaken the mucosal barrier of the GI tract promoting ulcer formation and slowing ulcer healing.Don't take aspirin or nonsteroidal anti-inflammatory medications. Avoid eating any foods that aggravate symptoms.Hope i could answer you.Feel free to ask anymore queries."
},
{
"id": 147759,
"tgt": "What precautions you recommend for a bulged disc in c7 vertebrae?",
"src": "Patient: I have been diagnosed with a bulged disc in c7 vertebrae. What care would u recommend? I am currently seeing a physical therapists and he has started traction. I have also had an epidural injection. Would you recommend chiropractic care. Thank you. Karen Doctor: Hi,Continue with physiotherapy, cervical traction.Avoid putting pillow under your neck while sleeping.Do neck exercise daily and regularly.Go for short way diathermy.Take calcium. vitamin A and D supplementsOk and take care."
},
{
"id": 134055,
"tgt": "Can tingling in legs occur due to pinched nerve?",
"src": "Patient: About 7 wks ago, I was bending over picking pine cones and gum balls up off the ground. Since then I have had some pain in my right back leg, but now what is bothering me most is the numbness and tingling in that r. leg. I think i have a pinched nerve in my right side below the waist. My Dr. here has did a regular xray that showed a narrowing on the right side of my back. He thinks I need a cortisone shot. I have been taking predisone, but i THINK i HAVE A PINCHED IN THAT BACK SIDE Doctor: Hi,As mentioned by you, i can understand that there is possibility of Lumbar spine disc degenerative disease. Also, undergoing an MRI is the recommendation to understand the level of compression and then you can be helped by a physical therapist. Using the Therapeutic Ultrasound therapy and TENS therapy will help reduce inflammation and pain. Also, post that strengthening the muscles of the spine, core, pelvic floor and lower limbs will help for controlling the symptoms and recovery also. Cortisone and predisone are the secondary lines of action.For any pinched nerve there is a safest way to practice1. Assess the patient and make provisional diagnosis2. confirm the diagnosis using the radiology - like MRI scan3. Rest - sleeping on a hard surface and avoid bending4. Physical therapy - Electrotherapy & Exercises5. Cortisone6. Surgery.Now, taking cortisone will help reduce your pain, but the actual pain will be present, since cortisone will only block the sensory pathway to the brain and possibility of pain to come back are there after cortisone.Also, cortisone is a steroid, so steroid has its own side effects with are unknown, and i will personally do not advise to undergo cortisone. I will advice to undergo physical therapy, though it will be a slow process but there are no side effects.If physical therapy fails in 12 weeks then the cortisone is advised,Also, in my daily practice i have seen patient with lumbar spine disc degenerative disease, and all have been helped with the following protocol1. Rest2. Physical therapyI wish you a good healthy futureRegardsJay Indravadan Patel"
},
{
"id": 74236,
"tgt": "What causes persistent cough and thick sticky mucous?",
"src": "Patient: hi, i have a question with regards to my problem..i have a cough for about 1-2weeks old \u00e0nd i take ciprofloxacin and.bromhexine for that.. i just notice that my pleghm doesn t comes out for like it s too sticky..hope you ve help me..my pleghm colors white.,and it always giving me a hardtime to cough.tnx in advance Doctor: Thanks for your question on Healthcare Magic. I can understand your concern. You are having thick mucus expectoration which is difficult to cough out. N acetyl cystine (NAC) is the drug of choice for this. NAC is mucolytic, expectorant, antioxidant and anti inflammatory.All these actions are helpful in your case.. So you should start NAC. Dose is 600mg twice daily. You will mostly improve with NAC in 10 days. Hope I have solved your query. I will be happy to help you further. Wish you good health. Thanks."
},
{
"id": 82436,
"tgt": "What could cause pain under rib cage and back while breathing deep?",
"src": "Patient: Have a medium pain in my right side, under my rib cage in line with my arm when arm is extended straight down . The pain is also felt in my back and under my right breast bone . The pain increases with each breath I take to the polnt the breath is shortened and the pain is compounded if I lay down . I am a 66 year old male in fair physical condition , l walk do yard work without any muscle discomfort. I am recovering from back surgery last Dec. and a full left hip replacement in April this year , but I m sure the operations have nothing to do with this pain . Thank you for any information you may provide . Doctor: Thanks for your question on HCM.Chest pain on deep inspiration is seen in pleurisy. Pleurisy is inflammation of pleura and can be seen in1. Pneumonia2. Pleural effusion3. Tuberculosis. So ee need to rule out all these first.Get done chest x ray to rule out all these.If chest x ray is normal than no need to worry much. It might be due to musculoskeletal pain only.So try to follow these.1. Avoid strenuous exercise. 2. Avoid heavy weight lifting.3. Take good painkillers with muscle relaxant. 4. Avoid bad postures in sleep.5. Apply warm water pad to the affected part."
},
{
"id": 30945,
"tgt": "What causes pain and itching sensation around anus?",
"src": "Patient: I have a bad itching sensation around my anus along with sharp pains. My lower abdomen also has sparatic sharp pains. My tubes have been tied for 6 years now and the cycle time for my periods are off schedule..... can anyone help ,e please? Thank you. Doctor: Thank you very much for contacting healthcare magic. According to your complain you have problem of anal itching that is most because of pinworm infection around anal region. Pinworm infection is common at anal region. Anal fissure like condition cause anal itching problem with pain.External hemorroid is also cause anal itching problem. Skin infection like psoriasis may cause anal iritation problem. Chronic diarrhea may cause anal itching problem. Some spicy foods cause this type of problems. Anal region traumatised by excessive scratching also.That may cause anal itching problem. Anorectal examination is helpful for further diagnosis. Take care.Thank you very much."
},
{
"id": 217955,
"tgt": "What is the remedy for pain in ring finger ?",
"src": "Patient: So I been recently feeling a pain on my ring finger on my right hand by the vein I took my ring off but if I touch it it really hurts. It is on the side of my finger close top on the side closer to my pinky. Please help ; I had this feeling since yesterday and there seems to be no swealing ! Doctor: just apply some ice to the painful site of your finger to numb the area, 2-3 times a day. strap your finger to the middle finger for a day or two. this shall take away the pain."
},
{
"id": 137275,
"tgt": "Suggest remedy for raised bumps in elbow",
"src": "Patient: I have a red raised bump about the size of a dime on elbow. I can still lift my arm and do daily activities, but if it gets touched or touches the couch or bed it s painful. it s hard, & doesn t seem like it s full of any liquid. any idea on what it could be? Doctor: Hello, I have studied your case.There is possibility of elbow synovitis.Till time you can do physiotherapy.If you can send your x ray and clinical photograph I can help you better for management.Hope this answers your query. If you have additional questions or follow up queries then please do not hesitate in writing to us. I will be happy to answer your queries. Wishing you good health.Take care."
},
{
"id": 95304,
"tgt": "Can any online doctor help me with the abdomen scan reports ?",
"src": "Patient: Hello doctor, I made a US complete Abdomen scan yesterday & got the result with me. Impression: 1. Left renal cortical cyst. 2. A Heteroechoic cyst in the left ovary . 3. Hemorrhagic Cyst. Is there any harm on the above 3 results. Please let me know. Mrs. Jeyanthi Doctor: Hi Jeyanthi,Welcome to HCM.You have not mentioned the reason you had to do Ultra Sound Scan and your age. Your result is perfectly alright and you have nothing to worry about. Renal cortical cysts are incidental findings on US and have no significance, unless they are of rapid growing type.Heteroechoic(inhomogeneous)cyst and haemorrhagic cyst, I think are referring to the same cyst. Haemorrhage in the corpus luteal cyst is occasionally seen after ovulation and it subsides of it's own within 3 months and does not require any treatment.It could of course be Chocolate (endometriotic) cyst, in which case it can cause pain during menstruation, and it needs treatment."
},
{
"id": 222303,
"tgt": "How long does it take for Cytotec to take effect?",
"src": "Patient: i took cytotec last friday, orally 2 tabs and vaginally, 2 tabs with 2 hrs interval. then i took another last sat, 2 tabs orally then on sun, i inserted another tab into my cervix. last night, i inserted again 2 tabs of cytotec, then took 6 tabs of it orally but still nothing happened. no abdominal cramps, no bleeding. im on my 6 eeks of pregnancy. hat does it mean Doctor: Hallow Dear,I sincerely feel you should have taken medicines for termination of pregnancy on an Obstetrician's advice. Cytotec (Misoprostol) works better if taken after Mifepristone. However, only Misoprostol also can terminate the pregnancy. Usually, the termination process is complete within seven days. If this medicine fails, there is a possibility of congenital malformations in the baby and hence continuation of pregnancy is not advised. Please report to the Obstetrician to get this pregnancy terminated by suitable method. I hope I gave you the correct message,Dr. Nishikant Shrotri"
},
{
"id": 97741,
"tgt": "How to cure acne pits ?",
"src": "Patient: Hi sir I am Abhishek Purakayastha from West Bengal kolkata, my age is 19+ and my problem is acne pits. I only just want to know that is here any remedy to get rid of this ance by homeopathic process because since my childhood I use to get treatment by homeopathic doctor and homeopathic medicines are very effective on me. Please sir give some suggestion or medicine or syrup name to cure this problem. Doctor: Dear Abhishek Purkayashtha ,Hello and welcome to Healthcare Magic .I am happy to know that you have always used Homoeopathy with good results.At the age of 19 years , the acne pits are bound to gradually reduce on their own because of the constant restructuring of the skin layers .So unless the scars are very unsightly you need not worry about them .The best thing would be for you to visit your nearest classical homoepath who would be able to assess your constitution and then give the best possible homoeopathic treatment to you .If you are in a hurry then you should visit a qualified dermatologist who would possibly give you skin peeling treatments to lessen the pitting and scarring faster .Feel free to put forward any other queries , I shall be happy to answer them to the best of my capability ."
},
{
"id": 177381,
"tgt": "Suggest treatment for cough and cold in a child",
"src": "Patient: my daughter is 3 years old and for the last one week she is down with cough..she is not able to sleep properly or even eat..today my doctor prescribed her Corex DX syrup for cough and quantity is 3 ml thrice a day.. i want to ask for how many days i have to continue and is there any major side effect. Doctor: Hi...you can continue for a maximum of 5 days and no major side effects. Sometimes rarely it can cause faster heart rates. Regards - Dr. Sumanth"
},
{
"id": 132759,
"tgt": "Could the hemorrhoids cause pain in the tailbone?",
"src": "Patient: I have external hemorrhoids that I ve had since my children were born. I am a 33 year old female, healthy, hypothyroidism. Recently, I started experiencing a painful tailbone area. This is new and alarming. I thought it might be a pilonidal cyst, but doctor said she didn t see or feel one. Could this pain in tailbone be caused by the hemorrhoids? Doctor: hihope this msg finds u in good health. i have gone through your complaints and under stand your concern. u r right. .hemorrhoids is one of d reason for tailbone pain..but it is quite uncommon. i suggest u get an xray done to find out whats wrong. u can take analgesics and anti inflammatory tablets. nothing to worry about. hope your question has been answered. if u have any follow up queries, feel free to consult me anytime. thanks Take care god bless"
},
{
"id": 32245,
"tgt": "What are the symptoms of tinea versicolor?",
"src": "Patient: Hey i think i have tinea versicolor, i went to a doctor and he thought i have a iron deficiency, i have been taking iron , vitamin a tablets, but the red sports dont seem to go,i have 1 dot with a radius of 4mm around my thumb, another one near my middle finger, on my left hand i have a 1 cm radius near my pointer finger, and i got a 3 cm radius of a red spot on my crease of my arm, i also have some itching on my neck, where i am starting to grow hair, **im 15 year old Doctor: Hi,Looking to the history it seems that you are having tinea versicolor infection, a fungal infection.You might require one oral course of anti-fungal medicine like Flucanozole 150 mg once in a week for 4 weeks.Apply anti-fungal lotion on affected parts.Ok and take care."
},
{
"id": 225512,
"tgt": "Can I take Daine after using it six months ago?",
"src": "Patient: Hey, can I take daine 35 I took 6 out of it about a month ago, so should I continue with it are get a new pack??????And can you please tell me what to do, I haven t seen my period for 3 months.. Started spotting to day... I don t believe that s normal, my doctor say I ave cycst r pcos .. An am tired of trying clomid an metformin cause am not getting any result. Took clomid for 3 cycle, my last cycle was on 26 of april... Please help me in any way u can to acheive my dream to have a baby... Doctor: Hi,Thanks for the query. You once check for the pregnancy. If the test comes negative, the spotting you are having could be menstrual flow. If you want to choose daine, you have to start with new pack. But as long as you use daine, the pregnancy is not possible as prevents the pregnancy. If you want regularization of the cycles and to plan for pregnancy, you can go for progesterone pills in the second half of the cycle. Decreasing the weight will help in regularization of the cycles and in treating PCOD. If you used 4 to 5 cycles of clomiphene, take gap for few months. And try naturally by tracking the ovulation. Otherwise go for ovarian drilling surgery with your doctor's advice. For more details you can ask me directly through the premium forum. Take care."
},
{
"id": 137277,
"tgt": "What causes selling in the feet?",
"src": "Patient: I went to a podiatrist today about my foot that I hurt really bad while skateboarding... They sent me to get x-rays done but when screening the x-rays they said my foot is too swollen to read them and they made me an appt. for next to find out for sure when the swelling goes down. I play college soccer and we move in soon. I was wondering would it be best to redshirt for the year? And also I ahve an appointment for tomorrow for my calf which I seen a RN today about but the couldn t get me in today and he thinks that there is a good possibility it may be torn. Do you believe that I won t be hearing good news? Doctor: Hello, There is most likely possibility of early plantar fasciitis.For plantar fasciitis,Shoe/footwear should be with soft sole. You can use silicon pad insoles.Avoid exposure of cold to foot.Start plantar exercises consulting physiotherapist also Physiotherapy like TENS and ultrasound will help. Take calcium supplement with vit D.Do exercises of foot before getting out of bed.Hope this answers your query. If you have additional questions or follow up queries then please do not hesitate in writing to us. I will be happy to answer your queries. Wishing you good health.Take care."
},
{
"id": 130403,
"tgt": "Could constant sore neck, out of line lower jaw after tooth extraction and cracking the neck mean damaged nerves?",
"src": "Patient: Hello. I allowed somebody, untrained to crack my neck a year ago. Since then. I cannot seem to keep my head straight and my neck is always sore. I am wondering if my nerves have been permanently damaged. Also, I had impacted wisdom teeth removed from my lower jaw, and since then my jaw has been out of line, I am needing to have the top two removed, will this balance out my face again? Doctor: Hi i am Dr Ahmed Aly thanks for using healthcaremagic site ,I had gone through your question and understand your concerns .. In my opinion such injuries to spine or to the nerves coming out of it to supply your neck may take a long time to resolve completely as any other nerve injury that requires time to resolve in addition to your tooth extraction that may have exagerrated your pain a little i think the removal of other jaw may improve your facial expression but i suggest CT , MRI to exclude any cervical disc herniation and for proper management of your neck problem completely . Please click THANK YOU and consider a 5 star rating with some positive feedback if the information was helpful. Hope the above information helps you,"
},
{
"id": 173983,
"tgt": "What causes faint red spots on hands and feet?",
"src": "Patient: Hi, My 5 month old baby girl has tiny \"pen dot sized\" faint red spots on her hands and feet. Looks like she may have them on her chest and stomach as well. She is not fussy nor does she appear to be uncomfortable. She does not seem to have a fever either. Any idea what it could be? Doctor: Hi...as of now I can say that as the dots are not bothering her, you need not worry about it. But...Skin conditions are best diagnosed only after seeing directly. I suggest you to upload photographs of the same on this website, so that I can guide you scientifically. Hope my answer was helpful for you. I am happy to help any time. Further clarifications and consultations on Health care magic are welcome. If you do not have any clarifications, you can close the discussion and rate the answer. Wish your kid good health.Dr. Sumanth MBBS., DCH., DNB (Paed).,"
},
{
"id": 75203,
"tgt": "Suggest treatment for pulmonary fibrosis",
"src": "Patient: My wife, Varsha Desai, age 58, 5' 2\" height, 73 kg weight now, i.e. 10 kg less than normal, is having pulmonary fibrosis surfaced after 8 months of breathless trouble,, and also hypothyroidism for 12 years and rheumatism after age 53, i.e. menopause. Her lung function 30% only. Is there any scope of lung lavage to cure such illness? Who is the best doctor in Mumbai. I am from Gujarat, Valsad. If needed, I can send scanned copies of reports and CT scan plates. Doctor: LUNG LAVAGE IS NOT INDICATED IN PULMONARY FIBROSIS AS HERE LUNGS ARE NOT FILLED WITH ANY MATERIAL. THEY ARE SHRINKING. SHE MAY NEED LONG TERM OXYGEN THERAPY AND CPAP. I CANNOT GIVE YOU ANY REFERENCE OF A PULMONOLOGIST AS I AM NOT FROM MUMBAI BUT YOU CAN GO TO ANY TERTIARY CARE HOSPITAL, GOVT OR PRIVATE WHERE YOU CAN MEET A PULMONOLOGIST."
},
{
"id": 188782,
"tgt": "Severe tooth pain, upper lip tender, swollen. Swelling below left eye. Treatment?",
"src": "Patient: friday and saturday I had severe tooth pain from what apparently was a nerve dying and decomposing (ice water is the only thing that helped.) Last night the tooth pain went away but my upper lip became tender, i assumed from such a long exposure to cold water. I woke up this morning with no tooth pain, but my upper lip is swollen and continues up along the left side of my nose, i can see the swelling just below my left eye's vision Doctor: Hello,Thanks for writing to us.Swelling of upper lip along the left eye may be due to spread of infection into sinus spaces .I would suggest you to visit a dentist and get a detailed clinical as well as x-ray evaluation done.Drainage as well as curettage of abscess will help to reduce the swelling.Take complete course of amoxycillin as well as analgesics to relieve the underlying infection.Tooth has to be root canal treated if necessary.Hope this helps."
},
{
"id": 225747,
"tgt": "Is elestra a good medication for birth control?",
"src": "Patient: Hi I started taking diane35 a month before I was getting married, I took the med for three months, not after three my husband was nt wide me so I discontinued it...I took it again last month now nw my doc has given a new media elestra for contraception ...is it a good pill ...I m nt sure plz guide me...I will b wide my husband after one night a half months now want to use sum contraception. Doctor: hello,thank you for using healthcare magic,diane and elestra both contains same pharmacological substance in short both drug are same,both are oral contraceptive pill ,only its brand name are different..continue taking elestra or diane both are same. take each pill daily at fix time . it is effective birth control method. if you forget to take any pil then take it as soon as you remember and continue rest of the pill as usual .plus use additional method like condom.. if you forgets pill and have unprotected intercourse then take i pill within 72 hours of unprotected sex and its other dose after 12 hours..hope this helps you..take care.."
},
{
"id": 29289,
"tgt": "Are black tiny things in the stools parasites?",
"src": "Patient: Hi, recently Iv been noticing that I have black seed like things in my stool. Some bits bigger than others. Over a long period of time I have been eating bread with seeds in them but haven t noticed it up until now. Iv now seen it more than three times even after Iv stopped eating seeded bread. Could it still be digesting through my system? Or should I be worried about parasites Doctor: Hello and thank you for the question,You need to see a doctor. Black spots in the stool could be blood from the upper digestive system. Blood turns black as it moves through the system. See a doctor soon,Regards"
},
{
"id": 149209,
"tgt": "Dizziness, nausea and difficulty in breathing. Diagnosed of Bells palsy. Is it an emergency?",
"src": "Patient: My wife age 72 has been ill for awhile she has gotten worse the last week or so we have to emerg twice and she has been completely out. she has dizziness, nausea, and she has vomiting. also she cant eat because she cant swallow she was diagnosed with bells palsy whereby her left vocal cord is paralized. should take i her back to emerg????? Doctor: Hi! Welcome to HCM! The path of facial nerve is long and complex,so there are many causes that may result in facial nerve paralysis.The most common is Bell's palsy, the exact cause is unknown most probably due to viral infection. Bell's palsy causes sudden weakness in your facial muscles. This makes half of your face appear to droop. Your smile is one-sided, and your eye on that side resists closing. Although for most people, Bell's palsy is temporary. Symptoms usually start to improve within a few weeks, with complete recovery in about six months however as the symptoms are aggravating my advice is that you should get your wife examined by a neurologist. During the physical examination, a distinction will be made between paralysis and paresis (incomplete paralysis). paralysis is far more serious and requires immediate treatment. It must also be determined whether other nerves are involved in the motor defect or not. I sincerely hope this will help!"
},
{
"id": 200832,
"tgt": "Suggest treatment for bleeding frenulum during intercourse",
"src": "Patient: My frenulum broke while I was tryin to insert my penisIt started bleeding heavily I somehow managed at that time bt I m scared abut the futureI don't want this to b known by any of my family memberIt was my first time eitherI might have bleeded a little on her lips Will she get pregnantI m very scaredPlz help please I m very scared Doctor: Thanks for asking in healthcaremagic forumIn short: If the tear is incomplete and small you can wait till it heals by applying local antibioticsExplanation: Bleeding will not cause pregnancy. And frenulum tear if small heals by itself with local antibiotics/oral antibiotics & anti inflammatory drugs. |If large/complete please visit a doctor immediately. Your family member will not be informed by a doctor , so visit a doctor at the earliest to prevent any problem in future."
},
{
"id": 38730,
"tgt": "Is rabies transmission possible after being bitten by a pet dog?",
"src": "Patient: Hi, may I answer your health queries right now ? Pl ease type your query here...hello doctor i get a bite before 6 days from my puppy who has 3 months old and i didn t make any rabies shot and my dog did receive his vaccin in the same day of the bite (before 5 hours of the bite )..do i still have any risk Doctor: Hello, Thnx to contact us. If I am your treating doctor I would like to advice you that receiving shot just 5 hours before the bite will not immunize the dog. There is a little chance of you get rabies, because puppy is unvaccinated before five day. To develop immune effect vaccine shot approximately requires 15 days. Proper wound care immediately after bite is necessary to reduce the chance of getting rabies, if you have taken proper wound care than need to take vaccine is reduced. But mind well there is a little chance still remains if you do not take the vaccine. In other case if you take vaccine, you are safe from the rabies. So in my advice you should start with the vaccine shots. If you have anything else to ask please contact me. Thanx. Dr. Arun Tank"
},
{
"id": 212684,
"tgt": "One pupil larger than the other, have had costochondritis, been stressed. Cause?",
"src": "Patient: Hi. I only noticed about 2 weeks ago that my left pupil is larger than my right pupil. I have had costo chondritis, fibre maliga since 2010, just had a stress test and heart alright. I haven t been able to read signs unless ontop of them or read tv guide on tv, or see peoples features since last April. The pain from these costochondritis attacks almost cause convulsions. Medication changed and pain better now, but do the difference in pupils relate to any of the above or what is the cause? Thank you Doctor: Hi there ~ A difference is the size of pupils is not uncommon. It is also not affecting your vision and so I would advise to stop worrying about the size of your pupils. I am sure it has also not come up in normal conversation. I am glad that your stress test was negative and your costochondritis is also better. If your fibromyalgia is giving you problems, I would advice you to see a qualified psychiatrist. Deep breathing and relaxation exercises are helpful in these situations. Take care and have a nice day."
},
{
"id": 193073,
"tgt": "Suggest treatment for varicocelectomy causing worm like movement in scrotum",
"src": "Patient: I have been feeling some worm like movements in my scrotum for the past 24 years. As a young man, I used to masturbate a lot and it was after one such session that I started feeling disorders in my scrotum till it developed to these movements. When the movements are on I actually see the scrotum sack move as well and if I put my finger under the scrotum I actually feel the movements like one is hold some worms in his hand which are sort of \u201cstruggling to escape\u201d, that kind of feeling.Over the years I have been consulting General Practitioners who would tell me that my condition was normal and there was really nothing to worry about. However some would prescribe me amitriptyline for a couple of months, but this never really helped by situation. Now over the years I have developed a very bad odour coming from my genitals which I suspect could be linked to this problem. I have often thought perhaps the scrotum skin is so won out (though it does not look so) that it now sticks. Now I worry more about the odour than the movements as they are not painful as such. The odour makes it quite difficult for me to freely mix with people and this is worse in my professional circles. Just last year I went to consult a urological surgeon who advised me that I had varicoselectomy that is why I feel the wormlike movements. He said to correct that I needed to undergo a minor operation which I did, but to no avail. I have also consulted a dermatologist about, but so far the prescriptions I got from him have not really helped my situation either. I need your assistance please! Doctor: Hello,You can opt for surgical correction. It is a simple procedure and done on OP basis. It is a simple procedure and complications are rare,Hope I have answered your query. Let me know if I can assist you further. Regards, Dr. Shinas Hussain, General & Family Physician"
},
{
"id": 113764,
"tgt": "Lower back pain, tightness in thighs, pinching pain near tail bone. MRI done, SI joint normal. Muscle sprain or nerve problem?",
"src": "Patient: Hello Doctor, I suddenly had a a severe lower baack pain 2 n a half weeks back.But now the pain has reduced but there is tighness in my thigh and littlte pinching pain near my tailbone(to the right and sometime to the left)...and just around my bum area.I am not able to sit for long though...and if i stand my legs feels little weak. I had taken physiotherapy for 9 days....and had consulted a doctor who said its something in the SI joint .IN MRI SI joint is normal.So is it just a muscle sprain?Nerve problem?I am 28 year old femaile...and i have a sitting job but i am active as i go for joggin walking swimming and occassional horse riding(just trottingt only) Doctor: you have not mentioned what physio treatment you have taken.. as the MRI is normal go with traction and neural mobilization you will get batter.."
},
{
"id": 50764,
"tgt": "Have problem in passing urine. Taking pain killers and anti inflamatories. Is it a kidney disease?",
"src": "Patient: worried abt having kidney disease female 62 88 kilos been on pain killers and anti inflamatories for ages itchy genitals very very tired. dont think I sleep soundly at nite. signs of full bladder,, on getting to toilet, find I only expel about 1 tablespoon of urine. this happens quite often. once or twice up in the nite to pass urine then busting to go in the morning. Morning passing of urine normal. Lose bowels sometimes sticky Doctor: Hello! I think you must be having a urinary tract infection (UTI). plus the history of cunsumption of pain killers may contribute to a chronic renal insufficiency. Kindly get a urine examination and antibiotics for UTI. Antibiotics should be given once the doctors has examined and necessary test are done.Thanks"
},
{
"id": 92987,
"tgt": "Medium bulge, pain on inguinal area, appears during movement. Possibility of failed hernia operation. Solution?",
"src": "Patient: Have possible failed inguinal hernia operation from 9yrs ago. medium bulge on rightside of inguinal area. able to push back in, but movement during day causes the bulge to reappear. some continual pain at site of bulge. Dr. said nothing can be done because of previous operation which was done the suture method. Been dealing with problem for about 2yrs & bulge/pain is slowly getting worse. Thank You! dhp Doctor: Hello,As you may be aware that there is no medical treatment for hernia and it has to be treated surgically.So it may be better if you can go for a second opinion.Now there are minimal invasive procedures available which can help to treat your condition."
},
{
"id": 121549,
"tgt": "Can i change immobilizer to allow bending after undergoing knee surgery?",
"src": "Patient: Question about knee. I had knee surgery (meniscus repair and plica removal) a week ago. I can bend my leg fully as diagnosed by my doctor. I was wondering if I am allowed to change my immobilizer to allow me to bend? If so what degree should I set it to? Doctor: Hello,You should not bend or walk without consulting your doctor.Sequential walking and weight bearing will be better for full recovery. Physiotherapy can help to attain full range of movements.Consult a physiatrist and he will direct you accordingly.Hope I have answered your query.Thanks"
},
{
"id": 102520,
"tgt": "How do I reduce swelling due to allergy to preservatives in food?",
"src": "Patient: i am have an allergy by preservative things like after eating coke or lays chips, my body get small spot swelling, and i need to scratch that spots than for that i am taking levocetirizine. pls suggest me how to get free from this. My name is ravi shekhar Doctor: Hello, Welcome to HCM,If you are confirm that you are having allergy to these products, Prevention is better than cure.Try to avoid these food products.In unavoidabl circumstances , if you are having any itching or rashes you can continue the same medicine which is prescribed by yoir doctor.Hope I am able to answer your query.Than you."
},
{
"id": 169759,
"tgt": "What causes fever in babies?",
"src": "Patient: my baby she s 1 year 6months 2 days back onwards she s walking up in the midnite and crying she has high temprature too why does it happened? she is 8kg now.she is taking food too.is it the symtoms of chicken pox ?she has already taken chicken pox vaccsination Doctor: It seems like normal viral fever., chicken pox doesn't come with skin manifestations such as vesicles and blisters..so kindly check out if she has cold and running nose. Give her paracetamol syrup 125mg per 5 ml syrup at dose of 3 ml 6th hrly and give tepid sponging for fever ..it will get subsided"
},
{
"id": 138632,
"tgt": "What causes weakness in legs after a lower back injury?",
"src": "Patient: Rught after hurting my lower back yesterday, both my legs became extremely weak and shakey feeling as though they couldn t support me anymore. My lower back feels very very weak now too. What s wrong with me? Do i have to pay to continue to consult I m sorry, i just read that I have to pay. I have no money at this time. Thank you anyway for taking the time to read my question. Doctor: Dear patient You have got paraparesis due to back injury. This is serious issue and you need to consult spine surgeon as soon as possible. Spinal cord and nerves travels through spinal canal and back trauma may lead to vertebral fracture or disc prolapse with spinal cord compression. Spinal cord compression leads to weakness in both legs and loss of bowel and bladder control. This if not treated in time may not recover.so the need of urgent consultation. I would advise mri of lumbosacral spine with screening of whole spine to confirm diagnosis. Visit expert spine surgeon with report. All the best."
},
{
"id": 161373,
"tgt": "What is causing a lump behind my child s infected ear?",
"src": "Patient: baby has had bad ear infections - doctor gave ear drops and antibiotics. Drainage started upon use of ear drops. Infection seems to be gone, baby is happ and no fever. She now has a lump behind her left ear. Is this fluid buildup and can this cause damage. How do I get rid of it? Doctor: Hi, By what you say I feel that this could be postauricular lymphadenopathy which is reactive lymphadenopathy which could be because of the ear infection. It will gradually get better by itself as the infection settles down no need to do anything for it. Hope I have answered your query. Let me know if I can assist you further. Take care Regards, Dr Sumanth Amperayani, Pediatrician, Pulmonology"
},
{
"id": 175052,
"tgt": "How to increase my baby's weight?",
"src": "Patient: My baby have 11 months and having moderate milk allergy his wt is 6 kg after month of treating milk allergy there no increase in body wt I have done metabolic screening test and and celiac disease test anf result is negative what can I do for increase body wt ? Doctor: Thank you for your consulting in Health medical care . To begin with,you try to find cause of allergy .The cause is inside.I suggest :1.Consult your pediatrition and allergist2.Check stool analysis for control of digestion.3.She might require stool test for ova and cyst4.And also stool culture for flora and sensitivity to antibitics for detection disbacteriosis of intestine.5.Enzymes can be given during 1-2 months for gain weight6.Probiotic:darolac 1 caps 1 time 1 month7.MultivitaminsBest regardsDr.Svetlana"
},
{
"id": 73132,
"tgt": "Is tuberculosis a hereditary disease?",
"src": "Patient: Hello doctor, I had kept a maid to take care of my daughter who is 1 and a half years old.She was with her for three months taking care of her.One day the maid fell sick and coughed up blood once.After that I discontinued her services immediately.This episode is now two months old.I got the maid s sputum test done for tuberculosis which was negative.However on questioning her she told me that her father had died of tuberculosis 14 years ago.Other members in her family include two sisters,mother and one brother.I also got a digital xray done and a helical CT CHEST,both of which were normal.I want to ask of there is a chance that she may be suffering from active tuberculosis given her symptoms and family history of tuberculosis. Thanks Doctor: Thanks for your question on Healthcare Magic.I can understand your concern. No, tuberculosis is not hereditary disease. It is infectious disease caused by bacteria, mycobacterium tuberculosis. It spread by droplet infection in coughing, sneezing etc. Since your maid had negative sputum test and HRCT thorax, no need to worry for active tuberculosis in her. So your daughter is also not at risk of tuberculosis. Hope I have solved your query. I will be happy to help you further. Wishing good health to your daughter. Thanks."
},
{
"id": 52757,
"tgt": "Does udiliv helps to dilute gallbladder stone?",
"src": "Patient: what is the purpose of taking tablet udilv for glodbladder stone my son who is 9yrs old has stone in galbladder 5.4mm he has been advised to take tablet udilv. i want to know what does this tablet actually does. does it helps to dilute the stone? my e.mail id is YYYY@YYYY . Doctor: Hi there,I'm not familiar with this remedy, but there is no medication that will help dilute gallstones. The only treatment for symptomatic gallstones is surgery, or diet medication in which one avoids all fats, but this will only prevent gallbladder attacks and will not treat the stones.I hope this answers your question. Please feel free to contact me should you have any further questions."
},
{
"id": 63674,
"tgt": "What causes painful cyst under the knee?",
"src": "Patient: about 2 weeks ago i got a non painfull squishy lump just below the neecap on my leg the next day brusing started to appear on my knee and shin it has slowley moved around my shin and started in the left side of my foot it is now on the right side of my foot an on the top of my foot near the toes and on my toes Doctor: Hi,Dear,Thanks for the query to HCM. I studied your problem in depth and I understood your concerns. Causes for the painful cyst undrer the knee-In my opinion your non-painful lump below knee cap-appears to have re-activated infection in it and has caused cellulitic spread till top of the foot of leg.This has developed due to the possible blood infection of the existing cyst or due to re-activation of the chronic died out cystic swelling below the knee.This advise is based on the facts from the history you give and needs further clinical check.So don't build up wrong concepts and create more psychic complications in you which would increase risks and costs to you.Hope this would relieve your problem.Welcome for any more query in this regard to HCM.Write good resume and Click thanks if you feel satisfied with my advise.Have a Good Day.Dr.Savaskar M.N."
},
{
"id": 92811,
"tgt": "Sore groin, no visible bulges, pain during walking",
"src": "Patient: hi my partner has had a sore groin for 2 days. there is no visible bulges round his groin area. He hasn t been doing any exercise but his job requires him to stand for long periods ata time he works in a factory. When he pushes his hand in where the pain is he says it doesn t hurt, but when walking back from work last night the pain started up again and it does intensify the pain at times Doctor: Welcome to Healthcare-magicGreetings of the dayIt would be better if you could be more detailed where exactly is the pan as groin is such a big area. Try keeping the area clean and dry. Consult a General Surgeon.Regards"
},
{
"id": 35511,
"tgt": "What causes swelling in lips with itching, numbness and redness?",
"src": "Patient: My lips started out earlier in the week with a few small bumps. Things gradually worsened and now my upper and lower lips are swollen, red, itchy at times and feel like they are burning along with a numb tingly feeling. The skin around my mouth is also affected and swollen. i have used no new products or eaten anything new. I have never had this happen before and it is painful, as well as embarrassing. Any ideas of what it could be and what I can do to make it go away? I have tried antihistamines, hydrocortisone cream, anti-fungal creams, aloe, ice packs and warm compresses. Doctor: Hello,Salvia can cause lip irritation. Use of vasoline can coat and protect your lips. If you are recovering from any respiratory infection, this can be transferred by your salvia. This is common with strep. If you have a viral reaction, you will need time for the virus to get out of your system. Your choice of products may help lessen symptoms until the virus is gone.Evaluate your sleeping habits. Any dental concerns such as grinding your teeth or mouth breathing? This can dry your lips and throat. Are there any other oral symptoms such as a coated tongue or red tissue that can indicate a fungal infection? This can require a prescription such as Nystatin. Probiotics or Acidophilus can help reduce chances of infection. Other medications can produce side effects and your medical history should be reviewed. Seasonal changes such as dealing with dry heat can cause tissue reaction, chapped lips and dehydration. Drink water and more water to be sure to be well hydrated. Avoud irritants and limit acidic intake. Consider visiting a dermatologist if symptoms persist. Consider a visit to your dentist if you are due for a checkup to look for additional oral factors.Thank you for your inquiry."
},
{
"id": 175740,
"tgt": "What causes pain while urinating in a four year old?",
"src": "Patient: My son is almost four and has urinated four times today and complains that it hurts when he urinates. At first I thought he may have gotten soap in there after his bath. But he had to go while at the store and then again when we got home and it hurt both times. Does he have a UTI? Doctor: please test his urine from pathological laboratory, please check the stream of his urine and so also his prepuceal skin can pulled up so then accordingly i will advise"
},
{
"id": 63155,
"tgt": "What does small skin tag and a lump under the breasts mean?",
"src": "Patient: i have a very small skin tag coming out along my bra line in the front and under the tag i am feeling a small lump size of a small pea or so. is that normal because i had a tag once removed inthe same spot about 5 to 6 years ago but the lump is new. Doctor: Hi,thanks for your query.Read and understood your history of surgery for the skin tag 5 to 6 years ago at the same spot where you have hot a skin tag now. Only difference is there is a small lump under it. Recurrence of the lump at the same spot after so much indicates that there is some irritation, ans it is most probably the bra strap that is rubbing over the same spot over yeas to cause a lump called as papiloma. I would advise you in such a case the following. Consult a General Surgeon to get a confirmed diagnosis and get the swelling excised for two reasons:One, you get rid of hte lump which is not a normal part of the body.Secondly, you get a confirmed diagnosis by histo-pathology examination the nature of the lump.Change the bra in such a way that the new ones do not rub at all on the previous spots."
},
{
"id": 166398,
"tgt": "What causes white patches on the skin along with diaper rash?",
"src": "Patient: My 15 month old daughter has had diarrhea, and therefore has developed a diaper rash, as she tends to do. We have been keeping her clean and slathered in Balmex, but now I noticed several white patches where the rash was most severe. I thought it was leftover diaper cream, but when I tried to wipe them off, they wouldn t move, and hurt her. Could this be thrush? Doctor: Hi.... I agree with you, this could be mild fungal infection or thrush and suggest you to do two things.You can upload an image of the region on this website and get back to us or con sult your pediatrician.I am suggesting this because skin conditions needed to diagnosed and be treated after visualising them.Regards - Dr. Sumanth"
},
{
"id": 13431,
"tgt": "Suggest remedy for skin rashes on the body",
"src": "Patient: I have a rash that started 5 days ago. They are round and raised a little, no discharge. They do not itch, except for new ones that form, as they are forming they itch slightly. They started on my hands and wrists suddenly. After 5 days they are only up to my forearm. They spread very slow. I have looked at many rashes online and I just cannot figure out what they may be. Doctor: Hi, It will be better if you attach a picture along with the query. If symptoms are severe better to consult a dermatologist and get evaluated. Hope I have answered your query. Let me know if I can assist you further. Regards, Dr. Shinas Hussain, General & Family Physician"
},
{
"id": 10298,
"tgt": "How can i control my hair fall?",
"src": "Patient: I have straightened my hair 3month ago ,from last 1month my hair is falling soo much ,I think half of my hair have been reduced ,I hv tried hot oil massage Bt still I have same problem ,day by day my hair is becoming too much thin .How can I control my hairfall ND get back my old hair Doctor: Hello and Welcome to \u2018Ask A Doctor\u2019 service. I have reviewed your query and here is my advice. Any artificially treated hair (permed/straightened) where the natural keratin bonds break needs super duper care. Apart from hair oiling you need to also up the nutrition quotient of your diet. Also start consuming multivitamins which have biotin in them. Hope I have answered your query. Let me know if I can assist you further. Regards, Dr. Smruti Pevekar"
},
{
"id": 127779,
"tgt": "What causes a numb and tingling sensation in the fingers?",
"src": "Patient: had a nerve test done on left elbow but my left hand, the pinky finger and the one next to it is still going numb and tingling. it is worse. please help. also had an injection in my right elbow and the skin is thin and i feel bone and the whole area is very white. Doctor: Hello,You may need a nerve release surgery. Sometimes physical therapy can help too. You will need to see ortho and hand surgeon. Hope I have answered your query. Let me know if I can assist you further. Regards, Dr. Priya Sadanandan"
},
{
"id": 160932,
"tgt": "What causes repeated finger infection with swelling and pus in a 2-year-old?",
"src": "Patient: Hi, My daughter is 2 years old and she has excema from 1-1/2 years. Recently her left hand middle finger(2 months ago) had some infection on the top of the finger(don t know the reason of infection) due to which had swelling and puss. Then she had to go thru minor surgery to take/clean out the puss and has been dressed for a week. After 2-3 weeks the wound got worse again (swelling and puss formation) and had to go for another minor surgery. Now again after 2 weeks the same infected area started getting puss and swell. Can anyone help us find the root cause and save the child. Thanks, XXXX Doctor: Hello, The symptoms are suggestive of recurrent paronychia. Consult a general surgeon and get evaluated. The pus has to be drained effectively. Antibiotics has to be started according to the culture reports. Hope I have answered your query. Let me know if I can assist you further. Take care Regards, Dr Shinas Hussain, General & Family Physician"
},
{
"id": 151629,
"tgt": "Persisting feeling of motion in head, stiffness in neck, numbness in forehead, taking ezeepam, etilaam, valance. Suggestion?",
"src": "Patient: Hello sir I am feeling motion in my head and stiffness in neck ,my doctor earlier gives me vertin for it,i feel numbness in forehead and feel like passaway that moment but after 4-5 weeks he gives me ezeepam-10mg,etilaam-0.5mg,valance-od 500mg.Now arround eight months are passed but still i need to take the same medicine regularly.Please suggest what i have to do. Doctor: Dear Neelgagan My suggestion to you is that you consult an ENT specialist asap as most of the \"feeling motion\" episodes are due to the involvement of the labrynthine system in your ear. vertigo is a very vast symptom so it is exactly hard to tell you exactly what is going on. If you can give more details maybe we can get closer to a diagnosis, but you will need to run a few tests. please do tell me your age, gender and more details about the motion. Do you see objects around you moving or you get the feeling that you are moving? is the movement in circles or verticle? Take good care of your self Thank you Akif"
},
{
"id": 80453,
"tgt": "What causes glassing / crystallizing of chest area?",
"src": "Patient: I went to the doctor today forresults from my ct scan of chest. She said they showed lots of glassing, when asked what this was, she said crystaliging. Do I need to see a specialist for this? I have been sick for 2 months now and he antidbiotics they have given me has not helped yet. Doctor: Yes, the doctor wants to say that there are glound glass opacities...this is one type of terminology in CT chest usually suggests any acute condition..there are many causes for it. & yes, you should consult the chest specialist. As it may not get benefited with antibiotics only...so, just consult chest physician & he will individualise your treatment & will find out the exact cause of your illness"
},
{
"id": 89676,
"tgt": "What can be the reason for the pain in the right abdomen after bypass surgery?",
"src": "Patient: my brother has undergone bypass surgery for mitral valve replacement and clot removal.now he is taking ecosprin75,and acitrom 2 mg. daily.for the past two days he is complaining pain on the right side of the stomach.what it can be?should we consult our doctor or not Doctor: You should consult a Doctor , this can be due to gastritis or so, but make this sure by clinical examination of a Specialist Doctor and get upper GI endoscopy and ultrasonography done. You may have to get your medicines changed to another in consultation with your Cardiologist . Take PPI, antacid get , soft and bland diet and see for the results ."
},
{
"id": 50978,
"tgt": "Born with one kidney. Moved to pelvic area. Attached by bridge. What is wrong?",
"src": "Patient: all through my pregnancy i was told my daughter had one kidney then there was none then there was one etc when she was a week old she had a scan her self which revealed she didnt have one. at 1yr old the specialist sent her for a nuclear ultrasound which showed she had a kidney it was joined by what they called a bridge and was smaller, now she is 2 not long had another scan and it showed her kidney has moved to her pelvic area, its getting a little confusing Doctor: Hi Welcome to HCM. It is most probably HORSESHOE KIDNEY. A CT scan can clinch the diagnosis. Hope this helps, Best regards DR GS"
},
{
"id": 209852,
"tgt": "Suggest remedy for suicidal thoughts and sadness",
"src": "Patient: Ive been suffering from suicidal thoughts sadness and crying spells for many years. I'm only 18. I've attempted suicide many times and last year I've made precise lacerations on my arm. I also feel like killing the people I live with What is wrong with me? Doctor: HiThanks for using healthcare magicI think, you have severe depression and in that case, you need immediate care. Better to consult a psychiatrist as early as possible. You need antidepressant and psychotherapy. Medicine would help to control these suicidal thoughts and psychotherapy would help to control self harming tendency. In case, you need further help, you can ask.Thanks"
},
{
"id": 85526,
"tgt": "What the Ubicar and Emanzon-D tablets are for?",
"src": "Patient: Hi, i went for ivf process, gt negative result. now, my legs are badly swollen and my energy levels are getting down. my dcotors advised me Ubicar (one tablet for two months) and Emanzen-D (twice daily for 5 days). pls advise what these medicines are for. thanks neha Doctor: Hello, Ubicar Tablet is used to treat nutritional deficiences. Emanzen D tablet is a pain relief medication.Hope I have answered your query. Let me know if I can assist you further.Regards,Dr. Dorina GurabardhiGeneral & Family Physician"
},
{
"id": 77884,
"tgt": "What are the implications from various tests post chest pain?",
"src": "Patient: Hi, my mum felt chest pain after the arguing and spontaneous subside afterwards.However, chest pain onset again during hot shower over the sternal region with no radiation for 2 nights. She then attend AED, ECG showed sinus rhythm with no ST elevation; trop I What is the implication of these?And what investigations suggested? Doctor: Hi. I can understand your concern. ECG is normal.GERD (gastroesophageal reflux disease) can cause similar kind of chest pain. GERD is due to laxity of gastroesophageal sphincter. Because of this the acid of the stomach tends to come up in the esophagus and cause central chest pain and nausea. You can take proton pump inhibitors. But along with drugs you need to follow certain below mentioned lifestyle modifications for better symptomatic relief. Avoid stress and tension. Avoid hot and spicy food. Avoid junk food. Avoid large meals, instead take frequent small meals. Quit smoking and alcohol if you have these habits. Go for walk after meals. Keep 2-3 pillows under head in the bed to prevent reflux. Loose weight if you are obeseDon't worry, you will be alright. Hope I have solved your query. Wish you good health. Thanks."
},
{
"id": 169067,
"tgt": "How can fever in a child be treated?",
"src": "Patient: My daughter has high fever of 101-102 from past 1 day. I am thinking of taking her to a hospital in Vijayawada. She is 1 year 11 months old. Could you please suggest a good pediatrician in Vijayawada or suggest what we could be doing to take care of my daughter in this condition. Doctor: Take her to Dr sudhaker at priya children hospital..., patukauva and before going there, give your child 150 mg paracetamol syrup to reduce the temperature."
},
{
"id": 77882,
"tgt": "Suggest remedy for chest pain while coughing",
"src": "Patient: I went to the ER almost 2weeks ago after falling into the wooden arm of my couch while moving it. They did x-Ray and said bruised ribs. Prescribed ultram and muscle relaxer. First few days seemed better but the past 4 days have been worse. Meds don t help, I m having to get help to get up from a sitting or lying position very painful to cough and laugh and bend over. What should I do Doctor: Hi. I can understand your concern. Possibility of musculoskeletal pain is more. Ibuprofen is anti inflammatory. You will definitely improve with combination of painkiller and muscle relaxant drugs. Also apply warm water pad on affected areas. Avoid movements causing pain. Don't worry, you will be alright in 1-2 weeks.Hope I have solved your query. Wish you good health. Thanks."
},
{
"id": 5462,
"tgt": "Irregular periods. Took treatment. Get pregnant?",
"src": "Patient: i got marrid in june 2009,we never intercourse completly bcoz we didn't want baby at that time but we nevr take any sort of external precautions,in dec 2010 my husband came to malaysia and i still remain in pak but juz after my husband went my periods became irregular and after some time it stop i took treatment and finaly periods become regular,now in feb 2013 i came to malaysia my husband and me met after 2 years now we want baby but my periods again become irregular like my last date were as follows 17,jan2013 befor met to husband then after met husb my periods start on 1march2013 nd now i would be on 1 april bt it started on 21 april. Doctor: Hello, There are many causes of irregular periods like stress, hormonal imbalance,thyroid disorders,sudden gain or loss of weight etc. I would advice you to undergo certain tests like CBC with ESR, Thyroid function tests and an ultrasound of pelvis. Meet your doctor with the reports and if needed you can again take the treatment for regularizing your cycles, as this is needed before you plan your pregnancy. Don't worry,every thing would be fine. Take care. Dr. Rishma Lakhani MBBS,DGO"
},
{
"id": 26090,
"tgt": "What causes breathing difficulty and heart fluttering?",
"src": "Patient: all week i have been having trouble breathing because of the allergy levels so ive been stuffed up bad. im also sensing heart fluttering. im also taking beta blockers at the moment but my husband also just left out of the country. could a mix of the stress and his leaving be causing? Doctor: Hello!Welcome and thank you for asking on HCM!I understand your concern and would explain that your symptoms seem to be related to anxiety. Anyway, I would recommend performing a careful physical examination and some tests: -a resting ECG and cardiac ultrasound-blood lab tests (complete blood count, thyroid hormone levels, fasting glucose, blood electrolytes, kidney and liver function tests) to exclude other possible causes of your symptomatology. I recommend you to carefully monitor your blood pressure during the day and refer those values to your attending physician. If all the above tests result normal, the main cause of your symptomatology would be just pure anxiety. Hope to have been helpful!Greetings!Dr. Iliri"
},
{
"id": 206188,
"tgt": "Suggest herbal medication to improve concentration while having ADHD",
"src": "Patient: My son who is 39 years old, has ADHD, he has a prescription for the problem, however he can t see the doctor until the 24 th of October. He will leave for Philadelphia tonight for a week long conference. The question is: is there a herbal medication that will help him concentrate? Doctor: DearWe understand your concernsI went through your details. I suggest you not to worry much. ADHD is a mental disorder, for which psychiatric treatment is a must. Along with psychiatric treatment you should also try psychotherapy, yoga and meditation. Some herbal medicines like brahmi might be useful for memory issues. But for ADHD there are known herbs. Please do not be cheated.Psychotherapy techniques should suit your requirement. If you require more of my help in this aspect, Please post a direct question to me in this URL. http://goo.gl/aYW2pR. Make sure that you include every minute details possible. I shall prescribe the needed psychotherapy techniques.Hope this answers your query. Available for further clarifications.Good luck."
},
{
"id": 30117,
"tgt": "What is the treatment for influenza and red rashes all over the body?",
"src": "Patient: I have been little sick with some flu for a couple of days about 4 days ago, took lot of vitamin c, about 5000mg a day, because ma friend said, that her doctor recomendet to get better easier, together with cold flu medication and amino acid suply pill, Icq 10, Multivitamin, spoon of flax seed oil organic and lot of vegetables and fruit - thought It is healthy, now I have this alergie like looking pink to dark red rash on my whole body, exept my face and armd. Pretty much everywhere where something touches the skin, when I scratch anywhere, the scratch leaves mark on my body and did not left for last two days...I have been taking benadryl, two days to get it to leave, but don t seem to work. l Doctor: HI, thanks for using healthcare magicThe rash could definitely be an allergic reaction to one of the products mentioned especially if you have not used them before or it can be a viral exanthem which is a rash that is associated with viral infections including the flu.You can continue to use the benadryl every 6 hours but if there is no improvement, you may need a short course of oral steroids.This would require a prescription from your doctor.I hope this helps"
},
{
"id": 152609,
"tgt": "What is the dosage of Acyclovir?",
"src": "Patient: I have a question about acyclovir? i had a breakout of genital herpes after having chemo therapy.I have only had very few since. the treatment was twice a day for about 5 days. it made me very sick. i recently had another one and the perscription says 800 mgs every 5 hrs. I am worried about a reaction! i am 69 yrs old and female. Doctor: HiDosage of acyclovir is 400 mg orally 3 times a day for 5 to 10 days.I would recommend you to follow this dose.RegardsDR DE"
},
{
"id": 219536,
"tgt": "How does an injection help when the embryo is not seen?",
"src": "Patient: Hi mam i m 30 years old and i conceive a baby of six weak but my doctor says that i conceive baby but rrsult is lightly positive and in ultrasound no embryo is seen yet.she prefer me that injection so kindly suggest me which type of this injection is helping me. Doctor: Hi.Most women undergo an ultrasound by the 6th week during which the gestational sac is seen, and pregnancy can be confirmed, but some women can take longer, so do not worry about that.A lightly positive result is also positive, ask your doctor to check your hCG levels and recommend an injection accordingly, I recommend taking injection fertigyn 5000 IU, but your doctor would be the best judge.Best wishes."
},
{
"id": 144508,
"tgt": "What does the mri report regarding spinal indicate?",
"src": "Patient: Can you explain this report from an MRI? : L4-5 There is a broad-based subligamentous disc herniation extending 7.4mm beyond the endplate margin. There is 7.5mm of caudal extrusion. There is moderate to severe stenosis of the central spinal canal and lateral recesses impinging upon the the traversing L5 nerve rootlets. There is mild biforaminal narrowing associated with lateral disc bulging. There is no compression of the exiting L4 nerve rootlets. L5-S1 There is transitional morphology at L5 with a pseudarthrosis on the left. There is no disc herniation, spinal stenosis or foraminal compromise Doctor: HelloMRI findings may suggests degenerative changes in lumbar spine.Degenerative changes are generally age related findings.It is due to chronic stress on annulus fibrosis.At L4-5 level, there is intervertebral disc herniation which is causing moderate to severe stenosis of the central spinal canal and it is also impinging the traversing L5 nerve roots. There is also mild narrowing of bilateral lateral neural foramen at this level.AT L5-S1 level,there is pseudoarthrosis(false joint)at L5 level in left side.Majority of degenerative disc disease patients are benefited by conservative treatment in the form of physiotherapy,analgesics and muscle relaxants.Surgery is advised in non responding cases.Physiotherapy helps in strengthening of paraspinal muscles and it prevents further changes.Since you have severe narrowing,I expect you may require surgical intervention.Now a days,surgery is done by minimal invasive endoscopic method.You need proper evaluation by a neurologist.Get well soon.Take CareDr.Indu Bhushan"
},
{
"id": 192879,
"tgt": "What is the treatment for swollen penis and groin pain?",
"src": "Patient: Hi, may I answer your health queries right now ? Please type your query here... My husband has had a problem now for three months he gets swelling on the penis and has had pain in his groin. he has only on lower testical due to being premi. we have been to the emergency room were they have taken ultrasounds . no one understands the problem. he has also had high blood presure for the first time in his life. he is 51 as of today, he is very strong and physically in great shape . he has a very poor diet. eats pizza and pork chops and salad do you have any ideas. Doctor: HI, I can understand your concern for your symptoms, the groin pain can be due to renal calculi and prostate infection. you can take anti-inflammatory drugs for the swelling and pain. I Hope I have answered your query. If you have further doubts , I would be happy to help you. Happy day"
},
{
"id": 67937,
"tgt": "Suggest remedy for lumps on butt hole and scrotum",
"src": "Patient: My husband has been complaining today that he has a lump on the outside if his butt hole. I looked at it tonight and it is the size of a marble, maybe a tiny but bigger. He says it burns, itches, and hurts when he sits. It s red but it is not bleeding and it does not have a head on it for puss to come out. It s just a red lump that burns and itches. It s on the outside of his but hike between his butt hole and scrotum. Please help! Doctor: HI.By the description you have provided, this is a case of a boil, it can be a beginning of a fistula in ano or a perineal abscess. I would advise you the following: Consult a General Surgeon;Investigations of blood and urine particularly for blood sugar;Get a prescription for antibiotics and anti-inflammatory medicines . Hot fomentation;Regular review.If resolves well and good,But if not he may need a small surgery to have a incision and drainage of the area and to investigate for potential fistula."
},
{
"id": 149797,
"tgt": "History of disc surgery. Need neurologist in Huntsville due to permanent nerve damage",
"src": "Patient: I am looking for a good neorologists in Huntsville,Alabama that takes Workmans Comp. I have permanent nerve damage due to 3 prior disc surgery' s from 2003-2004. I am allergic to numerous pain meds. & procedures don't work either. So I am at the end of my rope and I desperately need to go to a good doctor . Can you Please help me Doctor: HI,Thank for choosing HCM,You have not described what exactly you are looking for help to show you some good neurologist, need some medicine,, moreover you have not say anything about your clinical complain,, please be in touch with HCM,, further queries welcome,, good day"
},
{
"id": 65296,
"tgt": "What are the symptoms of lipoma?",
"src": "Patient: I have a soft lump right below the inner crease of my elbow. It feels like a sting if I press on it. On the outer side of the same arm, but on the other side I have a lipoma. Could they be related? If not what would case this soft lump in the crease? Doctor: Hi and welcome to hcmlipomas are soft well circumscribed and usually not painful subctaneous lesions.WIsh you good health. Regards"
},
{
"id": 223610,
"tgt": "How many I pills can be taken in a week?",
"src": "Patient: Its like that my wife had 2 ipills earlier and on last monday night we tried for intercourse , but didnt do that but as we were intimate closer and alsmot near i wish to give her third ipill, morover she has got period on tuesday night she has started period will there be any side effect if she takes ipill or is it not required Doctor: Hallow Dear, You seemed to be in confused state about the use of I-pills, which are post coital emergency contraceptive pills. These pills are to be consumed within 72 hour AFTER the unprotected sexual intercourse; not before the intercourse. Any intercourse performed after the consumption of these pills is not protected by the already taken pills. Each intercourse needs to be protected by I-pills within 72 hours. These pills have following after effects: 1. Few days after the consumption of the pills, there is some amount of withdrawal bleeding which lasts just for few days and does not require any medical attention. 2. The period following these pills may get delayed by few days up to a week or so. More the number of pills consumed during the cycle, more the number of days delay in the period. Such delay should not cause any concern.However, I would like to advise you to opt for some regular routine contraceptive method like birth control pills, condom or intrauterine device. You are a married person and have regular sexual relations. I-pill like post coital pills are emergency contraceptive pills and should be used as an emergency method and not as a regular routine method. I hope you got the information, advice and the message. Dr. Nishikant Shrotri"
},
{
"id": 19970,
"tgt": "Suggest treatment for block in the vein",
"src": "Patient: Dearsir. My father is paralises patient(left side) and he also suffering with diabetic. he got 1 blocked in right side a vain in neck to direct brein.,,can it is possible to cure from your side.please help me.becuse of he has been totally on Bad since 13.5.09. Doctor: Hi ThereAfter going through your query I understand your corn earn for your father I would like to tell you that choice of treatment for blocked arteries is medicine as well as intervention that depends on the blockage of the arteries. You should get a bilateral Carotid artery Angiography done and personally consult a Cardiologist or a vascular surgeon with the reports. If there will any possibilities arteries can be opened with help of stent or medicine .Hopefully this has answered your query.Kind RegardsDr Navneet Mahajan"
},
{
"id": 48687,
"tgt": "What is the cause of vomiting and nausea with kidney stones?",
"src": "Patient: iv been having kidney stones scince i was 17 about 40 of them to be exact i been having different pain though and nausuos to alot of different foods. over the olliday i had primerib and emieatly vomited after been really tired and feverish, what does this sound like Doctor: Kidney stones cause stretching of renal capsule and stimulate vagus nerve, these r the causes of vomiting in stones, the se will subsides only after remove l of stone only"
},
{
"id": 174446,
"tgt": "What causes sneezing followed by swollen eyes and body rashes?",
"src": "Patient: i was in a store in the check out line with my 3 month old an he started to snezze an had a lot of snot cum out then i noticed his eye started to swell an get wattery an his face got blochie spots on it it started to go down an the redness is going away any idea what this could of been ?? Doctor: HiWelcome to the HCMIn my opinion your child seems to have had an allergic reaction to some allergens in the store. It might be an aeroallergen.Since allergic reactions can recur, so it's important to be careful and try to identify and then avoid the things from which your child is allergic.Where possible, breast feed your child for at least 6 months. Breastfeeding provides a nutritious and balanced food source for your baby, reduces the risk of gastrointestinal tract infections and may also reduce the risk of developing allergic disease in early life.Delay the introduction of solid foods until the child is 4-6 months of age. Thereafter, foods can be introduced, with a new food introduced every 2-3 days. Introduce one new food at a time so that any reactions can be readily identified. If he suffers again, try using anti histamines. If it doesn't respond, do go for your clinician advice after a proper examination.In case of further questions, do call back"
},
{
"id": 27043,
"tgt": "Is it possible to have blockage even if ECHO and ECG are normal?",
"src": "Patient: Hi.I am diabetic. My ECG and ECHO reports are normal and I don t have any pain in chest as well. I can walk properly and climb stairs without fatigue. But my TMT was positive and I underwent angiogram. In that it shows around 90\u2105 blockage in LCA. Now docs have advised me for CABG. Is it alright? Is it possible to have so much blockage even if my ECHO and ECG are normal? Doctor: Hello. Thank you for your question and welcome to HCM. I understand your concern. Yes. It is possible. EKG is a test that reveals the ad hoc, at the moment, state of electrical system of the heart. If there is some acute ischemic process going on, it is captured by the EKG, because these problems, in acute setting, are reflected in it. Echocardiogram is a very useful tool to asses the status, movement and function of the heart wall, valvular system and other. It is very sensitive and specific if you had a recent or remote myocardial infarction, but it cannot depict blockages in the coronary arteries. Diabetes does significant changes to arteries of various diameters, in the body. It does not spare the coronary arteries either. I do not have your angiogram report to tell you more, but if coronary artery bypass grafting was recommended to you, then it must have been at least two-vessel disease and/or coronary blockage in a critical site, where stent placement carries a lot of risk. I hope I was thorough with my answer. Good luck with the procedure. Best regards, Dr. Meriton"
},
{
"id": 214928,
"tgt": "Is there any home remedy to cure burn marks caused by applying garlic on pimples ?",
"src": "Patient: i used raw garlic on my face to get rid of pimples which has left burns on my face leading to marks ....please suggest a home remedy to cure these marks i have burns due to applying raw garlic for the pimples on my face ... which have left marks on my face ...please suggest some home remedy for curing these marks and getting my original white skin back Doctor: hi u need to consult dermatologist by using home remedy ,u have already spoil ur face there is myth that home remedy is safe and effective pimple is due to infection and hormone ,so no home product is effective ,so dont try it again. for scar and spot u need to apply some cream . avoid sun exposure . dont rub over it just sponge over it"
},
{
"id": 107500,
"tgt": "What causes severe back pain after drinking water?",
"src": "Patient: I just started drinking water and I notice that after drinking a lot of water my back starts to hurt. This is the only liquid that does me like this. I was told to drink water but if I have to go through pain of drinking it I will pass. Why is the reason my back hurts only after drinking water. Doctor: Dr. Hanif warmly welcomes you with thanks for a consult! I have carefully worked through your case, and can well realize your health worries. Being your physician, I want to assure, I will take care all of your medical concerns according to the latest medical guidelines.Possibilities:Stomach Ulcers.Urolithiasis (stones that may be in the kidneys, ureters or other areas of the urinary system).I preferably suggest you to go for a complete abdominal ultrasound to see the reason for the pain being a stone as mentioned above.If it isn't due to the urinary system I suggest you to make an appointment with a gastroenterologist for evaluation. Until then you may take Pantocid-L and Tylenol for symptomatic relief. Hope I answered all of your questions. Please click on 'I Find This Answer Helpful' and don't forget to add your precious positive feedback to help me able to assist you better in future too.For any disease you have, a timely follow-up regarding the treatment efficacy and to see if there is any need to change the treatment options is very necessary. So keep a follow up to see how you doing. Remember to write down my name in the start of your question next time, & I will be with you right away. I am available 24/7 for providing you expert medical opinion on any health issue. Have a blessed time ahead.Regards!DR. MUHAMMAD HANIFUSAFOR DIRECT QUESTIONS: http://doctor.healthcaremagic.com/Funnel?page=askDoctorDirectly&docId=70199"
},
{
"id": 75080,
"tgt": "What causes chest pain and numbness in arm?",
"src": "Patient: i have been awakening in the middle of the night with chest pains, and numbness in my right arm... it feels like i am having a panic attack or something, but also like a heart attack. it becomes difficult to relax until i get up and walk aroudn a bit, eventually it goes away... is this just stress? Doctor: Thanks for your question on Healthcare Magic. I can understand your concern. Yes, possibility of stress and anxiety related panic attack is more. But better to first rule cardiac cause for this. So get done ecg and 2d echo. If both these are normal then no need to worry for heart diseases. Consult psychiatrist and get done counselling sessions. Try to identify stressor in your life and start working on its solution. You may need anxiolytic drugs too. Don't worry, you will be alright. Avoid stress and tension, be relax and calm. Hope I have solved your query. I will be happy to help you further. Wish you good health. Thanks."
},
{
"id": 185767,
"tgt": "What causes lump on gums?",
"src": "Patient: I developed a lump on the side of my gum the size of a pea the last two days which is hard and painful. Should I be concerned? I tried to squeeze it but have irritated it. Should I be concerned? My grandfather had jaw cancer from smoking. and had part of his jaw removed and has since passed. I am not a smoker. Doctor: Any bump in the the mouth is of concern. Inflammed tissue can require simple treatment such as a cleaning or be an indication of a more severe condition such as a cancer. A visit to the dentist for an exam and cleaning or a limited exam with a possible X-ray of that location is advised.Suggest you rinse area and gentle dental care. Avoid area when chewing with a soft diet. Anti-inflammatory medication may help.Common conditions:Inflamed gums- Swollen, red tissue, sensitive, tender. Action required professional cleaning, plaque and tartar removal, evaluation of periodontal health.Irritation under gums - swelling, pain, broken tooth under gum, something caught between teeth, periodontal abscess. Dental cleanng with numbing under gum, remove object, possible medications include anti-inflammatory, pain and antibiotic, rinsing.Exotosis, tori or excess Fibrous tissue - hard bump, not moveable becomes sensitive if subject to trauma. Thin tissue covering bone is prone irritation. Treat with warm salt water rinsing, avoid area, non irritating diet. Allow time to heal. No action required and may have been present prior just never noticed. Condition is just extra normal tissue. Only concern if distinct, abrupt change or interfers with daily function or dental treatment planned.Infection - swelling localized to a tooth may develop from bottom of the root. pain from pressure and spread of infection is common. Tooth can be very sensitive to pressure. Bubble on gum with tract from tooth may develop and release pus. Action by dentist may include drainage, antibiotic and further treatment options of a root canal or extraction. Anti-inflammatory medications helpful to reduce discomfort.Infection - swelling large with hard feeling tissue may be prone to spreading to face and neck. It can indicate a systemic reaction that can become severe. It may still be a reaction to a single tooth that requires antibiotic treatment, but you need to control the spread of the infection. Seek Emergency Room care if dental office not available. Stronger anti-inflammatory medications may be required.Cancer - bumps, sores, pain, areas that do not heal with irregular borders, history of smoking, chewing tobacco, high alcohol beverage consumer, family illness history all can be signs of concern. Certain locations are of greater concern. Further testing can include a biopsy and referral to a specialist. Ulcerated areas can often be viral and need a short time to heal while patient can rinse, avoid area, and keep to a non-irritating diet. Biting your tongue or cheek can also cause bumps. Healing time and rinsing area again will help this condition to resolve. A dental exam is recommended if questionable areas are noticed in the mouth.In my office, I commonly see teeth with infections causing swelling. The initial complaint is often described as a lump on the gum. If the area is moveable and does not appear to be attached to the bone, it is often the initial stage of an infection starting to spread. Schedule a dental visit to put your mind at ease and prevent your condition from getting worse. Hope you feel better soon."
},
{
"id": 90233,
"tgt": "Suggest treatment for ruptured appendix",
"src": "Patient: my son is in the hospitaland has a ruptured apendix. He is getting antibiatics and lots of fluids. He has pain when he pees,and dark green stool. Last night I left him with my husband and I think he was inseptic shock, How danderous is all of this. Should he go into surgery . He is 13yrs. old, has fevers and is in a lot of pain. At times he seems better and than hes not again. HELP! Doctor: Hi.Thanks for your query.This is a case of appendicitis with a complication. He needs an urgent CT scan , and if the appendicular abscess it present , it has to be drained immediately otherwise this will be life-threatening. Septic Shock, green stools, painful urination are all indicating this. Request the Surgeon to operate him the moment they feel he can be. It takes a few hours for the body to get settled and at least fit enugh for the patient to sustain the anesthesia and surgery. Timely intervention will save him"
},
{
"id": 48232,
"tgt": "Is dialysis the only option in case of chronic kidney problem?",
"src": "Patient: hello sir good morning my mother has chronic kidney problem she has only left side kidney which measurement is not normal or when i concern my local nephrologist . he says the kidney functioning only 7% .he suggest for dialysis bt what i do? sir piz tell me. i am so upset by knowing that. Doctor: HelloThanks for query.Your mother has solitary kidney with only 7% function .The main function of the kidney is to excrete end products of protein metabolism (Creatinine)If her serum creatinine levels are high she needs dialysis .Follow advice of your Nephrologist .Dr.Patil."
},
{
"id": 121256,
"tgt": "Suggest remedy for toe pain",
"src": "Patient: hello sir, one of my toe is in a bad shape it pains a lot. It seems like the end of the nail has pierced into to the skin or flesh causin a lot of pain while walkin . This is been happening from 3 months. I m not able to remove the nail, even though i remove it wit great pain it occurs after some weeks. Wat is the best treatment so that i would not occur in future? Doctor: Hello, The symptoms are suggestive of a condition called ingrown toe nail. Surgical excision of the extra nail will be the treatment of choice. Consult a general surgeon and get evaluated. Nothing much to worry as it is a simple procedure and can be done under local anaesthesia. Hope I have answered your query. Let me know if I can assist you further. Take care Regards, Dr. Shinas Hussain, General & Family physician"
},
{
"id": 99550,
"tgt": "Suggest treatment for sharp pain in the clavicle with asthma",
"src": "Patient: I have a sharp pain right on/underneath the right side of my clavicle. It is like the sharp pain you sometimes get when you breathe in, but a few clicks more dull and constant instead of when I move or breathe. Cold seems to make it worse. I am a 20 year old female, 125lbs with long term asthma (caused by RSV when I was little) and recently diagnosed anemia (which I take iron supplements for). Any ideas on what this is/how I can make it stop? Doctor: thank you for the question.You may have an x ray of chest and for clavicles.Take acetaminophen 500mg twice a day and do hot fomentation"
},
{
"id": 49622,
"tgt": "Fluttering feeling where kidneys are located, frequent urination, feel heaviness on left kidney",
"src": "Patient: I am 17 years old, female, and have been experiencing a fluttering type feeling on the right side of my body near where my kidneys are located. I've also been frequently having to urinate constantly throughout the day. Sometimes I don't have to pee but I feel like I need to. I've also been feeling a heaviness in my kidney on the left when I lay down or tke a bath. Should I see a doctor? Doctor: Hi,from history it seems that you might be having some urinary tract infection giving rise this type of problem.go for urine check up for urinary infection.After report go for one antibiotic medicine course.Take plenty of water.Ok and take care."
},
{
"id": 197737,
"tgt": "Noticed orange pieces which looks like glass pieces in semen",
"src": "Patient: some times whaen i cum ,i have 2 or 3 very small orange in color pieces that come out. they loook like little pieces of glass,and i can tell when its going to happen cause it feels like my load is going to be huge,but its just those pieces coming out first with semen Doctor: HelloThanks for query .Orange like pieces in semn coukd be due to infection of Prostate or Seminal Vesicle .Get your semen examination ans semen culture done to find out the organisms causing infection .Taking Doxicylcine twice daily for couple of weeks will help to resolve the issue .Dr.Patil."
},
{
"id": 103265,
"tgt": "Severe cough, blurred vision, dizziness, broken ribs. Taking medicines for depression and High Bp. Diagnosis?",
"src": "Patient: I have been successfully tx for my depression for many years with Cymbalta 60mg BID & Gabapentin 1200 mg A?M? & P?M? with a NOON dose of 600mg. I take med for high BP (under contro) & chronic pain & insomnia with PRN xanex 1-2mg or zolpoden 10 mg, not both together! Up until mid Dec. 2013 I have been fine. I also have asthma and hay fever since teenager. I am now 55 years old. It too was well controlled with albuteral inhaler PRN use & Advire inhaler BID. But after a vaccine shot fot this years flu & a whooping cough vaccine shot, boyhood given in same arm, I have been sick ever since. At first it was flu like symptoms & diarrhea. Not overly concerned. then I started coughing so hard & repeatedly I could hardly breath. If I did throw up it was hands full of plain clear to white mucus, not stomach contents. Long & short I've had & been treated for a sinus infection, laryngitis that lasted over a month. The antibiotics helped the sinus infection but gave my thrush& did't help my laryngitis. I started getting blurry vision & dizziness so bad it caused me to fall repeatedly. I broke 4 ribs, 2 on each side in 5 places & still kept caughing like crazy. I have seen several DR. & My Pscyatrist. They communicated well together. Then I started getting some sort of unusual seizure type symptom. my body will tighten up, my arms cill start to pull close to my body & I start to shake uncontrollably for several minutes. I can' t talk & have trouble breathing, but I can still hear & don't lose consciousness. Doctor: HIThank for choosing HCM, One possibility is there \"it could be because of the vaccine short given to you\" the vaccine some time gives such symptoms which is temporary phase and will disappears with its own, you can try some symptomatic cough reliever available on counter, but not steroids, have hot drinks, drink Luke worm water frequently, take bed rest, if does not come around then investigation would be needed, have best day."
},
{
"id": 122383,
"tgt": "Should a protruding left rib cage worry us ?",
"src": "Patient: My left rib cage is protruding; saw a breast surgeon who wasn t concerned. Had a chest xray and nothing showed. It is not painful; however, it is uncomfortable. More important is the pain on the right and left side of my rib cage. If I fold my arms across my chest, the pain is at both sides where my hands are. It feels like it is right under the rib cage. I am getting no help from my primary. I have not been diagnosed as having celiac; however I have a very bad gluten allergy and have been off gluten for almost 5 years. I am 5 4 and 128 pounds, with extreme fatigue. My GYN was very concerned and that is why I went to the breast surgeon. I wouldn t even know what type of physician I should see at this point. Doctor: Hello, The protruding rib cage can be related to a muscle weakness. The abdominal muscles play a large role in holding your\u00a0rib cage\u00a0in place. If the muscles on one side of your body are weaker, it may be causing one side of your\u00a0rib cage\u00a0to stick out or sit unevenly. I suggest doing physical therapy and exercise to treat it. No other treatment is required. Hope I have answered your query. Let me know if I can assist you further. Regards, Dr. Dorina Gurabardhi, General & Family Physician"
},
{
"id": 61919,
"tgt": "Suggest treatment for a painful lump on the shoulder",
"src": "Patient: There is one lump now causing pain at the front of my shoulder . . . It developed some time now under where my bra strap rests. All my life I have had very large breasts, so I ve had to pull my straps very tightly for support. Ultrasound results were relayed to me as nothing to worry about yet, my doctor is suggesting an MRI because there is some damage . . . How should I interpret things thus far in this regard? I am confused by the word damage and what that might suggest, yet suspicious that there might be something serious because of the need for the MRI!? Doctor: Hi,Dear,Thanks for your query to HCM.Dear I read your query and reviewed it with context to your query facts.Based on the facts of your query, you seem to suffer from-Bra Strap Abrasion with Normal USG of the Breast.MRI for the damage ? in this scenario,I would suggest you to undergo Mammogram and repeat USG for Second Opinion from another Radiologist.This would reduce your anxiety and worry caused by these ambiguity of reports from the earlier doctors.MRI if suggested ? can't be opined blindly unless physical examination from Surgeon and second opinion on the USG from another Radiologist is taken.Hope this would help you to plan further treatment with your family and specialist doctors there.Welcome for any further query in this regard.Good Day!!Dr.Savaskar M.N.Senior Surgical SpecialistM.S.Genl-CVTS"
},
{
"id": 22555,
"tgt": "What causes palpitations followed by leg pain?",
"src": "Patient: yesterday after lunch i was feeling great. about 1 hour later my heart started beating extremely fast, and it felt like something was pressing down on my chest, and right leg pain. I checked it and it was 132 (resting) it went away after about 5 minutes. this am i woke up with major irregular heartbeat and headache. i just ate lunch and i seem fine but im still worried sick. am on levothyroxine for hypothyroid for 2 weeks now. Doctor: Hello,Thyroxine can cause tachycardia so I need to know how much dose you are taking, also the Tsh level before starting thyroxine. If you are feeling your pulse is irregular then you need an ecg to look for atrial fibrillation. You also need to know your hemoglobin level as it may also aggravate palpitations. You can visit your doctor again and ask for dose reduction if possible.Hope I have answered your query. Let me know if I can assist you further.Regards, Dr. Sagar Makode"
},
{
"id": 34916,
"tgt": "What causes dengue fever?",
"src": "Patient: my daughter is 3 yr 8 months old. she is a case of dengue fever evidence by antibody IgM - positive. IgG-negative.now platelet-2,4600. lowest platelet was-162000. s electrolytes- normal.First 3 days fever was present.then 2 days was afebrile. last 3 days -subnormal temp{95.F-96F}. . pulse-80-90/min. now no medication, first 5 days was on azithromycin.paracetamol on first 3 days.i m worried about her subnormal temp.what can i do. Doctor: Hello dear,Thank you for your contact to health care magic.I read and understand your concern. I am Dr Arun Tank answering your concern.Current platelets was increases as compared to previous one.This is the sign of improvement. With this improvement if the blood pressure is normal than there is no need to worry.But low temperature and fall in blood pressure is adverse sign and should be taken care with volume increasing solutions.Once the volume increases both the blood pressure and the temperature increases.I will be happy to answer your further concern on bit.ly/DrArun.Thank you,Dr Arun TankInfectious diseases specialist,HCM"
},
{
"id": 52527,
"tgt": "What could uniform rash on back be with history of pancreatitis?",
"src": "Patient: My husband has a uniformed rash on his back. No raised welts or pimples. He says there's no itching or burning but I'm afraid it may have something to do with his pancreatitis which he was diagnosed with 4 years ago. His doctor said he has a neucrotic pancreas. Doctor: Hello and Welcome to \u2018Ask A Doctor\u2019 service. I have reviewed your query and here is my advice. The rash has nothing to do with pancreatitis. Pancreatitis never produce rashes and it might be something else. You can apply calamine lotion over the area and if it\u2019s persist, better to consult a dermatologist. Hope I have answered your query. Let me know if I can assist you further."
},
{
"id": 30499,
"tgt": "What causes foul taste in mouth and symptoms of lactobacillus?",
"src": "Patient: Last week I had a ua/cs done as I have frequent uti s ever since a spinal cord injury 30 yrs ago. It came back negative for infection but my urologist recommends cranberry pills plus a probiotic due to lactobacillus in my urine. I have a very unpleasant taste in my mouth that reminds me of how my urine smells...foul. I took in yet another urine sample to the hosp lab today and actually hope it comes back positive so I can get things cleared up prior to starting the probiotic. Any info as to why I have the foul taste and symptoms of lactobacillus. In 30 yrs I ve never heard of this and wonder about the sx and tx. Thanks! Doctor: Hello,I can understand your concern. Bad smell from mouth could be due to many reasons related and unrelated to dental diseases. It could be due to dental infections, gum disease, sinus problem, food habits, inadequate oral hygiene etc. In addition, kidney disease can also cause bad breath from mouth.Thus, I would advise you to wait till the results of your test come. If they indicate presence of kidney disease, then you can start treatment for the disease and the bad breath will eliminate on recovery from the disease. If the test shows negative for any pathology, you should visit a dentist to look for presence of dental infections or gum problems. Treatment of the disease can sure such problem. By the time the underlying cause is detected and treated, you can use mouthwash containing Chlorhexidine such as Colgate Plax or Hexidine. You can take 10 ml of liquid and squish it around mouth for about 30 seconds before spitting it. Take care no to use immediately after brushing teeth and do not eat or drink anything for half an hour after its use. Brush twice a day and use dental floss to clean interdental spaces. I hope this information helps you. Thank you for choosing HealthcareMagic. I wish you feel better soon.Best,Dr. Viraj Shah"
},
{
"id": 82351,
"tgt": "Suggest remedy for persistent sneezing followed by pain below the rib cage",
"src": "Patient: I sneezed about a week ago. I have always been what I am now seeing is called a bracer . I contain my sneezes when they occur. when i did this the other day during said sneezing, I felt a sharp almost popping sensation below my ribcage on the left side of my body. I since have accidentally done the same thing again while lying down and the pain has now shifted around towards the front and below the rib cage. The pain is not persistent but very noticeable. Is this just a pulled muscle? Doctor: Thanks for your question on HCM.In my opinion you are having musculoskeletal pain only due to excessive sneezing. Sneezing causes forceful contraction of expiratory muscles and if repeated than muscles can become sore and gives pain.So better to control sneez first.Start antihistamines twice daily.Avoid exposure to cold.Drink plenty of fluids and keep yourself hydrated.Take good painkiller and muscle relaxant."
},
{
"id": 50730,
"tgt": "One kidney, severe chest, lower back pain after consuming alcohol. Bufferin helpful. Is this normal?",
"src": "Patient: The night before last I had a little too much to drink and I have one kidney, the day after I had a hangover as should be expected however last night I could not sleep due to severe pain all over my chest and lower back and it is still hurting, taking bufferin helps but only for about 20 minutes then the pain comes crashing back. Is this normal and/or will it go away with proper hydration and food or do I need to visit my doctor? Doctor: did this ever happen? i dont think it is related to kidney issue, but would suggest you to go and see a general physician"
},
{
"id": 27497,
"tgt": "Suggest treatment for chest pain while breathing",
"src": "Patient: Today in work I lifted a tote box full of shoes it slipped on the tote under it and hit me in the chest,it really hurt and as soon as I done it i felt sick.it still hurts now,it doesn't hurt to breath but its a constant pain that almost feels like heart burn.should I go to the hospital or should I just rest? Doctor: Hello!Thank you for asking on HCM!I understand your concern, and would like to explain that your pain is originated from traumatic injury of chest wall structures.First of all, it is necessary to exclude important damages of thoracic cage (ribs, sternal fracture, etc as well as soft tissue contusion, like intercostal muscles and more profound structures: lungs, heart, aorta, esophagus, etc).I recommend to seek medical help to your local ER service: a careful physical examination, chest X ray study, pulmonary and cardiac ultrasound, coupled with some lab tests (like AST, ALT, CK-MB, troponin, etc), would be necessary to rule in/out the above mentioned potential injuries ( depending on severity of shock).Meanwhile, a painkiller medication would be helpful.Hope to have been helpful! Greetings! Dr. Iliri"
},
{
"id": 189768,
"tgt": "What causes cavities in the teeth and how are they prevented ?",
"src": "Patient: Sorry Im not really good in english So heres my problem I have a hole in my tooth then the gums started filling the space in the hole Now Its still growing What do I do What is that called? Is it dangerous? Doctor: Hello and thank you for your questions. First cavities are caused by bacterial infections in your teeth. The bacteria produce acids that eat away at the outer surface of your teeth (enamel) and then spread to the inside of your teeth. Cavities are prevented by: -decreasing frequency of sugar intake, especially sticky, tacky snacks -avoiding beverages high in caffeine, sugar and acids: coffee, energy drinks, fruit drinks, sports drinks -following the 3-2-1 rule: brush 3 times a day, floss 2 times and gargle with Listerine 1 time daily -do not sip on soda pops or fruit drinks, drink them and then switch to water, the longer you spend sipping on a soda the more exposure you have to acids that eat away at your teeth. -go to the dentist annually and have a cleaning twice a year. -if you have cavities get them fixed right away before they progress and become worse. The hole in your tooth is called a cavity. The gums growing into the hole is called hyperplastic overgrowth. You are having an irritation in the area, the gums are swelling and growing due to this issue and filling in the void of your tooth. If the cavity is in the top of the tooth, you may actually have hyperplastic pulp also known as a pulp polyp. You need to have the cavity filled, possibly a root canal and crown, or an extraction of the tooth depending on how bad the hole is. This is not necessarily dangerous, but could lead to more discomfort and a more severe infection down the road. Don't sit on this until it becomes painful, have it repaired and addressed as soon as you can. Best wished and good health Dr. Ward"
},
{
"id": 55755,
"tgt": "Are the iron levels and essential Thrombocythemia a matter of concern?",
"src": "Patient: My son(33 yrs) had some blood work done and his iron level was 194. He has a hematologist for essential thrombocythemia, who he saw yesterday, as his liver test showed ALT(94) and AST(51) which was down from the prior blood work done a week before. His hematologist said not to worry about the liver tests but I wasn t sure about the iron. I also was wondering if the hematologist was looking at the liver info re only cancer since that is his specialty too. Should any of this be of a concern? Doctor: Hi, dearYour son has essential thrombocythemia. It is myeloproliferative disease. He needs treatment for that. Don't worry about iron level and slightly high liver enzymes level. It is not a matterof concern. He should take aspirin to prevent any clotting. Chemotherapy is useful. Permenent treatment of choice is bone marrow transplantation. Consult your doctor and plan accordingly. Hope I have answered your question, if you have doubt then I will be happy to answer. Thanks for using health care magic. Wish you a very good health."
},
{
"id": 146824,
"tgt": "Are spasms down the back normal after a lumbar puncture?",
"src": "Patient: I had a lumbar puncture on 10/28 and it was quick and went well. Afterwards I followed procedure and stayed on my back until the next day. My back feels like I have a fist pushing near where the puncture was done. It feels uncomfortable to sit down for more than few minutes. I get a spasms that go up and down my back that feels like a fluttering. Is this normal. Doctor: Good morning (if morning is where you're at). I'm Dr. Saghafi, a neurologist and have had many patients report different things to me following an LP.People sometimes report different symptoms of pain, burning, uncomfortable pressure following an LP...even if the procedure went swimmingly well. You should let your provider know if pain becomes particularly poignant or you develop fever, chills, or a \"lump\" in the area of the puncture which seems to be enlarging or discoloring the skin. Otherwise, follow your post procedural instructions and you should feel better in a few days.If you'd like to address further questions directly to my attention on this network you can look me up at:http://doctor.healthcaremagic.com/doctors/dr-dariush-saghafi/68474"
},
{
"id": 23614,
"tgt": "Can tricuspid valve regurgitation cause atrial fibrillation?",
"src": "Patient: My mother is 66 years old, she had experienced a stoke for the third time last year 2010. She had transient ischemic attack last week and we recently discovered she has paroxysmal atrial fibrillation through ekg. She has a history of hypertension and diabetes but these two are well controlled within this year. She had an echocardiogram last August 2010 with mild tricuspid regurgitation. I have these questions: 1. Is atrial fibrillation the cause of my mother's stroke for the past years which was not detected until recently.? 2. Does tricuspid valve regurgitation caused her atrial fibrillation? Doctor: Hey,Regarding the answers to your question1) AF is a very common cause of stroke. It may remain undetected for years and may present first time as stroke when the ecg shows af. So yes it may be old which has caused stroke2) Mild TR generally does not cause any problem and may be physiological in some patients. IT DOES NOT CAUSE AF untill any chamber of the heart is dilated because of significant TR. As your mother is diabetic and hypertensive, she is at high risk of coronary disease, which may cause AF.So ruling out that is also important after she is stabilised. ThanksDr Sameer Maheshwari"
},
{
"id": 211926,
"tgt": "Have shivering and gaining weight. Taking risdone and olimelt for depression. Can this be the side effect of the medicine?",
"src": "Patient: Hello Doctor,This is Rahul and i would like to share something about my elder brother Rohit.His age is about 28 he is suffering from depression. we consult to doctor and he prescribed some medicineas such RISDONE & OLIMELT. But after taking this his hand start shiver and he start aging weight.Whats your opinion about this ,Please reply back to me on YYYY@YYYY Doctor: Hi Rahul, welcome to health care magic. i would like to know still more about your brother's symptoms to diagnose him correctly.the drugs which are prescribed to your brother belongs to a class psychotropics and i may not be able to conclude whether these drugs are correct to your brother with out knowing all his symptoms.and the drugs which are prescribed to your brother are known to cause these side effects, but nothing to worry as side effects can be controlled . consult your psychiatrist"
},
{
"id": 39091,
"tgt": "Suggest medicines for yeast infection during course of antibiotic medication",
"src": "Patient: Yeast infection, i came off antibiotics this week which resulted in a yeast infection. however i relapsed and i am starting a stronger course of antibiotics today. can i take something to treat my yeast infection during my course of antibiotic medication? Doctor: HiWith you query, it would have been beter where do you have yeast infection its k you can also use some anti fungal drugs to get rid off thus but under medical supervision. Hope this helps you"
},
{
"id": 68692,
"tgt": "How can an arm lump with soreness and itching be treated?",
"src": "Patient: hi, approx 7 years ago i found a lump on the inside of my upper arm and went to the doctors who said ot was a cyst and if it wasnt bothering me could be left and it would hopefully go away by itself. it is now considerably bigger and extremely sore to touch and is also itchy. could this be something serious should i go get it checked out Doctor: welcome to Health care magic.1.Symptoms suggest an infective aetiology - but duration of the lump suggest lipoma ( fat content).2.The most possible chances are Infective in relation to symptoms.3.If you are my patient i would have examined and ask for the ultrasound lesion - to find out the nature, origin and extensions of the lesion.4.Depending upon the size - a course of antibiotic may help, in bigger size a small incision and drainage may be necessary.Hope it helps you. Wish you a good health.Anything to ask ? do not hesitate. Thank you."
},
{
"id": 220860,
"tgt": "Is traveling safe after ultrasound shows low lying placenta at 19 weeks?",
"src": "Patient: Hi Doc....I m 22 weeks pregnant now....My last scan report in 19th week has shown that my placenta is low lying...I am planning to travel to Australia in 25th week....I have to join my husband in Australia...Just wanted your suggestion if travelling is safe and what measures need to be taken during the flight journey Doctor: HI, Thanks for the query. I understand your query. LOw lying placenta & journey at 25 weks of pregnancy is a bit risky. THe risk depends on the distance of placental edge from internal Os. Generally it is best avoided.. for fear of physical stress & bleeding, which is difficult to control in later weeks of pregnancy. You should consult your treating doctor ,being well versed with your case, he would be in a better position to advise you exactly. thanks..."
},
{
"id": 162560,
"tgt": "Is taking Ibuprofen while suffering from chicken pox harmful?",
"src": "Patient: I just gave my 4 year ibuprofen (motrin) for some pain she was having all day so she could have a better nights sleep, and not 30 minutes later did i realize she has the start of chickenpox. I read ibuprofen is harmful given to someone with chickenpox. Now I'm really worried, will she be okay? Do i need to take her in? Doctor: Hello and welcome to \u2018Ask A Doctor\u2019 service. I have reviewed your query and here is my advice. It is true that it is not safe to give Ibuprofen during chickenpox. But if is only chickenpox lesions on her, you need not worry. But if she develops high grade fever or swelling or pain in the limbs or abdomen, then we need to suspect necrotising fascitis and then you need to approach her pediatrician or the nearest emergency room. Hope I have answered your query. Let me know if I can assist you further."
},
{
"id": 17250,
"tgt": "What causes sudden increase in the heart beat?",
"src": "Patient: I am a 23 years old guy who lives in lebanon one day i was working and suddenly my heart start to beat faster arround 100 pulse/min and i were fainting my skin were so white i went to the ER and i slept for one night in the hospital. The day after i went to my doctor and asked me to do several medical exams like blood test,TSH,Urine,X-Ray for my chest,ECG,scanner for my stomach and everything went normal.i ve been like this about 2 months and i barely can moves i always feel sick and i am going to die i am feeling so weak and i always have a severe headaches and i am not balacend,and my vision is not always clear. Some doctors told me that i am having a stress disease but i am not sure about it and others told me could be my stomach.so i went to a gastroloc and told me that i have a kind of infection in my stomach with ebb so he gave me a medicine but i am still feeling that i am not normal there is something wrong. Please if you have any advise for me about my case let me know,what type of medcines i have to see? I ll be waiting your response as soon as possible because it drives me crazy and don t know waht to do.I need to know if my case could cause a death? Thank you, Doctor: Hi, Anxiety can cause an increase in heart rate and palpitations which is a normal phenomenon. The heart rate becomes normal as soon as the stimulus goes away. The exact diagnosis of the palpitation depends on the time of the ECG if the ECG is taken at the time of palpitations then a proper diagnosis can be made. But as the palpitations are short-lived, we generally advise for a 24hr continuous ECG monitoring of the heart which is called Holter monitoring. You don't need any medication for now but further management will depend on the Holter report. Hope I have answered your query. Let me know if I can assist you further. Take care Regards, Dr Sameer Maheshwari, Cardiologist"
},
{
"id": 112097,
"tgt": "Is Ibuprofen sufficient to treat contusion causing sharp pains in lower back?",
"src": "Patient: I fell on stairs a little over a week ago with my lower back on the right side hitting the edge of the step. I had severe pain which is lessening but I sometimes get a sharp pain when sitting down or standing up. It was diagnosed as a contusion but there is no bruise mark. X-rays did not show any breaks. I try to walk around a little bit and take 600 mg ibuprofen every 4 hours. Why no bruise mark if it is a contusion, how much longer until pain goes completely away and should I use any other type of treatment? I use an ice pack every few hours. Thanks. Doctor: Hi there,Thanks for writing in.Initial 24-48 hours after any injury, it is best to use an ice pack intermittently. However , now the injury is almost a week old, in which case hot fomentation would be ideal.Apply a small quantity of pain relief gel like volini(diclofenac gel) to the affected region of the back. Do not massage or rub the gel too much.. Just apply it lightly. After half an hour or so apply a hot water bag/ hot fomentation to the affected area. Do the above at least 3 times a day. Reduce the oral intake of ibuprofen gradually. Too much of this pain relief medication can cause gastritis.Hopefully you should be completely okay in another week or so. If the pain persists or increases in intensity, please visit your physician at the earliest...Hope this helps.Regards,Dr. Divya Kuttikrishnan"
},
{
"id": 102440,
"tgt": "What should be done for allergic reaction to generic Lipitor?",
"src": "Patient: I seemed to have developed an allergic reaction to generic Lipitor no problems before takeing regular Lipitor 10 years generic 1 year is this possible .ten mimutes after I take generic my tongue feels funny peppery overall very legaric sometimes nauseus Doctor: Hello,Welcome to HCM,It may be due to allergy to the drug and the other constituents of the generic lipitor.As you are having to this generic drug I would like to advise1. Stop this drug Immediately.2. Oral Antihistaminics3. If you are having intense itching you can apply cold pack.Drug should be taken on doctor advise.Thank you."
},
{
"id": 39227,
"tgt": "Suggest cause and medications for itching and pain in vulva",
"src": "Patient: 4 yrs old girl has recently developed pain and itching of vulva plsu difficulty controlling urine. Her clitoris looks protruding and longer than normal. What specialist to see? no thanks, i will check it my self , it should say that people have to pay to start with Doctor: Hi,Welcome to HCM!It primarily appears that she is suffering from a fungal infection of the vulva. Fungal infection may occur due to poor personal hygiene or decreased immunity. She must be started on an anti fungal tablet as well as local antifungal cream. She might also be given an anti histaminic like cetrizine to get relief from itching.Another possibility is Lower urinary tract infection. Also, the primary problem might be urinary incontinence. Due to leakage of urine, she might be having inflammation of the vulva and surrounding skin. If possible, visit a dermatologist (skin specialist) to correctly diagnose it after examining. Hope this information helps. Feel free to ask if you have any doubt.With warm regards,Dr. Sridhar Reddy"
},
{
"id": 28752,
"tgt": "How can MRSA be treated?",
"src": "Patient: I was swamped and confirmed to have MRSA. It seems like I have breakouts every now and then on the back of my neck on the side of my face chin and nose area. My new doctor put me on doxycycline antibiotic 100 mg twice a day, and advised me to put Bactroban only on my nose area. I don t think the medicine is doing anything. I am also diabetic. What should I do? Doctor: Hello and Welcome to \u2018Ask A Doctor\u2019 service. I have reviewed your query and here is my advice. I understand your problem. Drug of choice for MRSA is Vancomycin. Doxycyclin is used to treat acne bacteria. You have bumps on the back and neck. May that doctor thought it is back pimples. Consult him again. Hope I have solved your query. Get well soon. Thank you"
},
{
"id": 13097,
"tgt": "Suggest treatment for rashes as chlorox didn t work",
"src": "Patient: I had a rash & went to my dermotologist on 7/2 & he prescribed the following: 4 T of chlorox ib 16 oz, of water , Use gauze twice a day soaked in that solution for 10 min. Then apply Cloderm & rub in, thoroughly rub in Neosallus. I did the chlorox solutation for ten days the creams. Then dropped the chlorox solutation but have continued w/ the creams until today. It looks awful, but seems to be drying up. Shud I continue. Doctor: Hello, Your symptoms seem to be related to a bacterial infection. I suggest continuing using Clorox for antibacterial properties. I also suggest using an antibiotic cream for local application. Hope I have answered your query. Let me know if I can assist you further. Regards, Dr. Dorina Gurabardhi, General & Family Physician"
},
{
"id": 65001,
"tgt": "Suggest treatment for a lump on the shoulder",
"src": "Patient: I have had intense pain for three weeks. It started with a knot in my right shoulder and now radiates down my right arm. I have had two deep tissue massages this past week but relief only lasts for one day and the massages are very painful. Could this be some type of inflammation? I am wondering why it does not get better. Doctor: Hi,dear you lost to fix your question in the heat of writting.In my opinion still,2 massages past week lead to some relief.The story indicates that it-is lowly infected knot on the right shoulder.I would advise to take local-surgeons opinion and take the needful treatment.Thnks for query to HCM.Wellcome again to HCM"
},
{
"id": 92116,
"tgt": "What does symptoms of abdominal and upper back pain mean?",
"src": "Patient: I woke up today afte three hours of sleeping (worked overnight shift) with severe upper abd pain that went to my back. I tried to reposiion myself thinking it was just back pain or sciatica flareup. I then became very nauseated and vomited and abdominal pain was intense. I have not had diarrhea but was belching a lot. The nausea has subsided but abdominal discomfort continues. When I tried to go back to sleep, back pain was theuu biggest issue. I am mostly concerned about the severe abdominal/back pain that woke me up. Now that I am up I still have abdominal and upper back pain. I have to go to work in a couple of hours. I am just wondering what my symptoms might mean and kin Doctor: hi, do you consume alcohol? did you have similar episodes previously? in any case, your symptoms suggest 1) pancreatitis- more probable if you are alcoholic 2) peptic ulcer- if you smoke, or have stress or have recently taken pain killers. Avoid hot and spicy items, oily foods and hot tea. if pain do not subside, consult a physician. hope this helped."
},
{
"id": 40146,
"tgt": "Is TT injection enough after being scratched by a stray dog?",
"src": "Patient: I have been scratched by a stray dog many times by nails with the skin area being red and bit swollen feeling a bit burning sensation; however after some time that redness,swelling and burning sensation disappears, is it fine if I take only TT injection??? Doctor: Hello,Welcome to HCM,As you are bit several time a dog on a nail, if it is a scratch or abrasion or bite on the nail requires proper and adequate treatment. The dog has all the potential to transmit rabies, which is 100% fatal but it is 100% preventable by timely treatment.Tetanus inj will protect you only against development of tetanus not against the rabies, For your condition I would suggest you to follow1.Thorough wound wash with soap and water2.Antirabies vaccine on days 0,3,7,14 and 28. If the cat is healthy after 10 days you can discontinue last two shots of antirabies vaccine.3.Inj Tetanus, 0.5 ml should be taken.4.If there is any open wound you may require a course of antibiotics like tab Augmentin.Don't take any risk with this disease.Consult your doctor for proper and adequate treatment.Thank you."
},
{
"id": 204964,
"tgt": "How to cope with excessive tension and lack of focus?",
"src": "Patient: sir i have the problem that i am all the time in heavy tension ,which i dont have any interest in studying and other activities of my life and even i dont have any confidence for presenting my presentation.i am bore all the time and just thinking ,i dont have the power to get any idea from my study Doctor: HelloI understand your concerns. I think you are having mild depression. I would suggest you to practice relaxation exercises like deep breathing and meditation. Try to eat green vegetables and take plenty of fluids.All the best."
},
{
"id": 128282,
"tgt": "Suggest treatment for restricted movement and pain in the shoulder",
"src": "Patient: My son plays football and recently he was running the football and a linebacker hit his right shoulder with his helmet. He has been having issues raising his arm, in front of him and across his body. It has gotten a little better with anti inflammatory and pain meds but now it s stalled out on the healing. Doctor: dear sir/ madam since your son has a history of injury, most likely he has injured his rotator cuff,i wouldlike to examine him for any local tenderness, give rest to his arm ,give a arm sling pouch, ice pack , for a week, and then after a week do some manuevers to test rotator cuff,and then start physiothearpy to the shoulder"
},
{
"id": 213323,
"tgt": "Cervical pain, head reeling, depression. On Nurokind plus tablets. Treatment?",
"src": "Patient: I have survical pain for the last fifteen days and at times head reeling also.I am not feeling fresh & energetic.Should i take Nurokind plus tablets for ten days? For the last fifteen days i am getting survical pain and at times head reeling.I am also not finding fresh and bit depressed.Should i take Nurokind plus tablets for ten days Doctor: Hi, From your query it is clear that that you are having depression and cervical pain. Sometime depression and low mood itself can lead to neck pain due to muscle spasm. Nurokind plus contain multivitamin and will relieve your symptoms, if they are due to vitamin deficiency. It is safe. So, in my view you can continue it. I hope this information has been both informative and helpful for you. Wish you Good Health. Regards, Dr. Ashish Mittal www.99doctor.com"
},
{
"id": 120666,
"tgt": "What causes pain in the sub mandibular gland?",
"src": "Patient: hello sir, i m 29 yrs old male & i feel slightly pain in my lower neck (submandibular gland) while i eat , my lower neck(submandibular gland left side) swells when i eat & after meal it goes away. what can be the causes of this? only during meal time it happens. Doctor: Hello,I read carefully your query and understand your concern. Your symptoms seem to be related to swollen salivary glands.I suggest:- massaging the affected\u00a0gland. -applying warm compresses to the affected\u00a0gland. -rinsing your mouth with warm salt water. -sucking on sour lemons or sugar-free lemon candy to encourage\u00a0saliva\u00a0flow and reduce\u00a0swelling.If the symptoms continue, I suggest to see an ENT specialist. Hope my answer was helpful.If you have further queries feel free to contact me again.Kind regards! Dr.Dorina Gurabardhi General &Family Physician"
},
{
"id": 188469,
"tgt": "Had root canal treatment and crown replaced. Now have deep periodontal pocket on mesiolingual surface. Suggest?",
"src": "Patient: I recently had a root call retreatment on #14 and had a crown replaced after. The tooth was asymptomatic prior to the retreatment, but had what appeared to be a periapical radiolucency on the mesial root. Several months later, I now have a fairly deep periodontal pocket on the mesiolingual surface despite having an excellent hygiene routine: I floss 1-2X a day, brush 2-3 times a day, and my last prophylactic cleaning was in June. Is there anything I can do to help this pocket outside of seeing a periodontist? It isn't painful, but can be a food trap and it is sometime malodorous when flossing. Doctor: Hi,Thanks for asking the query,According to your clinical symptoms i suppose that you have developed a pocket in the crowned tooth.I would suggest you to go for complete mouth scaling and polishing.Take lukewarm saline and antiseptic mouthwash rinses.Avoid eating from the affected side.Maintain a good oral hygiene.Hope you find this as helpful,Regards.."
},
{
"id": 19480,
"tgt": "What is the treatment for high blood pressure?",
"src": "Patient: Hi, may I answer your health queries right now ? Please type your query here... I'm 41 yrs and weigh 155 at 5'7\" I eat no red meat, or chicken. My blood pressure has been climing higher and higher now the bottom number is over 100, I just check it it's 153/106 what is going on? Doctor: Hello,You should have a healthy lifestyle like avoiding fatty, oily and high calorie diet. Have low salt diet and monitor blood pressure regularly thrice a day for one week then once or twice a week. If bp is persistently more than 140/90 mmhg, then you should be on medicines for it. I would have started tab Amlodipine 2.5 mg once a day and would have gradually increased according to need. Regular exercises like brisk walking, jogging according your capacity atleast 30 min a day and 5 days a week. Lots of green leafy vegetables, fruits. Avoid smoking and alcohol if any. There shouldn't abdominal fat deposition or obesity. Hope this helps you and get back if you have any questions."
},
{
"id": 113995,
"tgt": "Is it safe to take sAZO 500 during pregnancy ?",
"src": "Patient: I am suffering from RA and plannig for second pregnancy my age is 31 year and i have polysistic ovarian disease my doctor told me that it is safe to take sAZO 500 during pregnancy. I want to know how much safe is sazo during pergenancy and what are the side effect for the same Doctor: Hello.Welcome.Side effects of sulfasalazine are nausea,vomiting,rash,oral ulcers,joint pains and decrease in sperm count.SAZO is safe and can be taken in pregnancy.Good luck."
},
{
"id": 162085,
"tgt": "Can a toddler be given double dosage of Emeset after vomiting?",
"src": "Patient: Our daughter is 26 months and we gave her 2.5ml of Emeset 2mg/5ml today afternoon and evening on prescription. She vomited immediately in the evening and we have given her another dose. She had light bland food and ORS. But now at night she is feeling nausea again. Can we give another dose? Thank you. Doctor: Hi, In fact any medication which is vomited within 10 to 15 minutes can be repeated. But this particular medication is not something which need to be used regularly and please give it only if there is vomiting. Hope I have answered your query. Let me know if I can assist you further. Regards, Dr. Sumanth Amperayani, Pediatrician, Pulmonology"
},
{
"id": 175524,
"tgt": "What could cause black, very solid and smalll stools in a child while having slight rash on back of knees?",
"src": "Patient: My two year old has had black very solid small poo for about 5 days now. No diet changes or anything I cam think of. He did have a slight rash on back of knees that looked a bit like ecceema last week - almost cleared up now but he never had that before. Just want to make sure there is nothing underlying. Thanks Doctor: Hi...I don't think anything serious is happening in the kid. Just make sure if you are giving any iron supplements. This can make the stools hard and black. Otherwise if the kid is active and his usual self - you need not worry.regards - Dr. Sumanth"
},
{
"id": 200576,
"tgt": "Suggest treatment for erectile dysfunction",
"src": "Patient: Iam 25 year old bachelor from india now in dubai.for the first time ever i checked my blood sugar cholestrol yesterday. My blood sugar was 163 and cholestrol 231. I took the test 6 hrs after food. And the nurse said its a bit high for my age. My concern is does this level cause impotence. These days i feel my irrections are weak .and iam going to get married in few months. Doctor: Hi, dearI have gone through your question. I can understand your concern.If yo take test after 6 hour of food then your sugar level is high. You should go for fasting and post postprandial blood sugar. If need repeat the test. If your sugar level is persistently high then you should start anti diabetic drugs. Your erectile dysfunction may be due to diabetes. Consult your doctor and take treatment accordingly. Hope I have answered your question, if you have any doubts then contact me at bit.ly/Drsanghvihardik, I will be happy to answer you.Thanks for using health care magic.Wish you a very good health."
},
{
"id": 65151,
"tgt": "Suggest remedy for painful lump at anus",
"src": "Patient: A few days ago, I pulled my lower back quite badly. Since then it seems to be healing with rest, ice and gentle exercise. However, a simultaneous problem has occurred that is causing a great deal of pain. I noticed that my bottom was also very painful, and when I looked there was a red and painful lump at the entrance to my anus. I took this to be a haemorrhoid. However, it is a very swollen and red perineum that is actually causing the pain. Is this related? Are they related to my back? And how can I treat it? At the moment, the combination of injuries means I can barely sit or sleep. Doctor: Thanks for your question on HCM. I can understand your situation and problem. I don't think your back pain and swelling at anal region are associated. By your history and description, your anal region swelling is mostly due to boil (skin infection). It is common in diabetic patients. So first rule out diabetes. Also consult surgeon and get done clinical examination. You need incision and drainage with thorough surgical dressing. You may need antibiotics, painkiller and anti inflammatory drugs. So better to consult surgeon and discuss all these. Avoid too much moisture and keep proper hygiene of anal region."
},
{
"id": 86410,
"tgt": "What could excess fluids in the abdomen suggest?",
"src": "Patient: Hi my son has fluid in his abdomen. He has been sick to his stomach and not been able to eat. He has been getting very shaky and not sleeping. Not sure if he has been throwing up for he is in Florida and I am in Tennessee he has just been admitted to hospital for further testing. Can he have cancer (I pray that is not what it is) he is 23 years old. Doctor: hi,Dear,Thanks for your query to My HCM Clinic and understood your health concerns.Excess fluid in the abdomen is due to many reasons-Commonest-is Ascites / due to the liver cirrhosis from Viral hepatitis / alchoholic hepatitis.-Dont worry till you get definite reports from the treating doctors.Talk to the hospital to get the right information.Relax till then.Wishing early recovery to your son.Wellcome for further queries in this matter.RegardsDr.Savaskar /India"
},
{
"id": 3567,
"tgt": "How to determine the cause for delay in achieving conception?",
"src": "Patient: Hi me and my partner have been trying for a baby for 18 month and had nothing, ive had blood tests which are fine, my partner is going for a sample this week, but he is worried he cant have kids or has low sperm, just wanted to know what help is there for us? Doctor: Hi,Welcome to Healthcare Magic. I am Dr Ramadevi wani. I will be answering your concerns today.Please note that among women less than 40 years trying for pregnancy, 80% conceive within the first year and 90% conceive by the end of second year. It is only since 18 months you are trying for pregnancy. So don't worry.It is good to know that your hormone tests are normal. Why your partner and you are thinking of negative things. As mentioned above it is natural for some to have this delay in getting pregnant.Let your partner have the semen analysis test after two days of abstinence. If it is not normal consult infertility specialist. There is help available.To increase your chances of conception maintain normal BMI, quit smoking ( if you're a smoker) and have intercourse at least three times in a week.If you have any further concerns do contact me through Healthcare Magic."
},
{
"id": 131223,
"tgt": "Suggest remedy for red and swollen area around elbow worsened with stress",
"src": "Patient: I missed while shooting up around my elbow.it is now read its been like this for about a day and a half and kind of swollen but it seems to go away when I m sleeping but it gets irritated very easily if I m strenuousI woke up this morning and it was not as read it doesn t seem to be getting any worse but it doesn t seem to be going away either is there anything that I can do without having to go to emergency room do I need to go to the emergency room I don t know what to do. Doctor: you do not need to go to emergency roomyou need to avoid extending your elbow for 3 days and take anti inflammatory drug 3 times a day for 3 days( take cataflam 50 or Alphinturn )Good Luck"
},
{
"id": 214476,
"tgt": "Suggest natural ways to control bp",
"src": "Patient: yes doctor i'm 31 male 5.7 height & 79 kg weight recently when i gone for a medical checkup they told i have high blood pressure but last year i was 65kg without any issue - so i really wondered about this change - can u please help me out to bring my BP normal naturally? Doctor: Hi I have gone through your query . I am unable to find the description about your BP count and since when you are having it ? what are the syptoms you are facing ? Anyway , before going to get any remedy , you should find out the cause of trouble .In Ayurveda , the 3 factors dominate our metabolism of Vat ,Pitta , Kapha imbalance among these 3 , cause different health problems . BP is normally outcome of disorder equilibirium of Vayu , due to psychic and physical factores like mental strain ,worry, enxiety , lack of exercise , excessive intake of salts and fats , indiscrimate use of alcohol are few to count , persistant constipation IF you try to keep control on them ,half of the battle is won . Correction of your life style is the biggest Home remedies , regularity in taking meals, wALK exercise and rest shuold be maintained .PANAYAM / DEEP BREATHING , ANULOM -VILOM , BHRAMERI SHAV-AASSAN VERY HELPFUL TO BRING DOWN BP Our two Nosril have differnt role to play . The right one is surya naadi --for controlling heat of body ,and left is Chandra naadi for cooling process . In case of HBP generally , right nostril becomes active . In day time our left nosril should work and at night ,right nostri functions , If you are having HBP always check your nostril ,if it is right nostri ,close this with you thumb and start the left one . Lie down straight , calmly .keep your mind calm and quite ,without any tention , Within 10 minues BP will calm downThis a instant remedy .supplement are to be used --Garlic 5-6 cloves cut into pieces , gulp with glass of water in the morning . Take lemon in water twice before meals , Take buttermilk with simple meals containing low fat , fiber , veges fruit vitamins ,minerals , coconut water , garlic , ginger in every meal oranges guava apples are good . Avoid fried ,fast foods tea , coffee , alcohol , mental strain .Almond oil 1 tspoon in cup of hot milk at bed time , help relaxing nerves of brain If you hold magnet south pole in your right hand sitting on a woggen stool feet also on wooden plank ,will bring down your BP within 10 -15 minuets hope this will help to solve your query Take care All the bestHave any doubt mail at drsuchda@gmail.comDon't hesitate to get back if have an further query . Meditation also helps for peace of mind"
},
{
"id": 188829,
"tgt": "Swelling in roof of mouth, tooth pain, eye pain, swollen pallet. How can it be fixed?",
"src": "Patient: Yesterday I noticed the top right side of the roof of my mouth is swollen. My teeth started to hurt my right nostral on the inside hurts and now my right eye is starting to hurt seems like there is pressure building up from my swollen pallet. And the next day the roof of my mouth has swollen even more. If I push on it it hurts. I'm really worried. What might I have and how to fix it r Doctor: Welcome to HCM forum.. Swollen palate which arise suddenly, along with a painful tooth is generally a palatal abscess in relation to the offending tooth. Your hurting tooth is either have deep dental caries or it has a periodontal( gum) problem. There is nothing to worry. Its a very common symptom of a deeply carious tooth or a tooth having gum problem. Paining inside nostril and under eye area is mostly referred pain from your infected tooth.You can start and antibiotic like Augmentin 625 mg or Clindamycine 600 mg X 8 hourly X 5 days and Paracetamol 1gm as an when necessary to relieve pain and swelling.But it is always better to visit a dentist as early as possible, diagnose the reason for swelling and start medicines according to his/her prescription, because this kind of abscess may require drainage of pus from the abscess, and which will relieve your symptoms faster. The dentist also will start the treatment of the tooth responsible for this to avoid any future problem like this. Thank you."
},
{
"id": 190563,
"tgt": "Asphalt on the gum line not wearing off with water after a fall. How can it be cleaned?",
"src": "Patient: My son fell this evening and basically did a face plant onto asphalt. The scarps have been cleaned and the tears are gone. He hit his front teeth they are a little sore and the one is ever so slithly loose. Since he is four that does not concern me what does is the asphault on the enamual which should wear off but it looks like there is some under the gum or at the gum line and water is not removing it. Any recommendations on cleaning this out of the gums? Doctor: there may be more severe damage after a fall than just slight looseness and ashphalt on enamel. the black line you see under the gum may be a fracture or something. dont take things so lightly...you should see a pediatric dentist soon."
},
{
"id": 101484,
"tgt": "Suggest remedy for chronic cough producing yellow and green thick, stringy sputum along with asthma symptoms",
"src": "Patient: I have had asthma like symptoms with a chronic cough that produces both yellow thick globs of sputum and think dark green stringy sputum that almost looks wormlike. I have been treated for infections, take inhalers, and also a prednisone treatment at least once a month with only a few days of relief then right back to episodes Doctor: HIThank for asking to HCNI really appreciate your concern looking to the history given here I would like to say that it could be bronchitis and in my opinion this need to be treated with Tab Ciprofloxacin 500 mg twice in day for 7 days drink plenty off water and no need to worry about this, hope this information helps you, take care and have a nice day."
},
{
"id": 220968,
"tgt": "Suggest possible tests to confirm pregnancy",
"src": "Patient: Hello I'm Joel my girlfriend, after having 1 week period delay, took a pregnancy test, which resulted positive. However, 2 weeks later she blood-checked in a hospital and the resulted NOT pregnant. Though her period didn't come yet, I wasn't given any clear answer from the hospital, please help! thank you Doctor: Hi,Thanks for writing in.1. Home pregnancy test is by estimating the beta subunit of hCG in urine. This is positive after 15 days from sex if pregnancy has happened. However there are some limitations when the result can be false positive and pregnancy has not occurred.2. The blood beta subunit of hCG estimation is done in the hospital and this is accurate. This usually is a quantitative estimation and not wrong. Therefore if they have measured beta subunit of hCG in blood and it is negative then pregnancy is not a probability and the home pregnancy test is false positive.3. If in doubt then you can go for a transvaginal ultrasound after 6 to 7 weeks from the last menstrual periods and this will most likely show a pregnancy. Please discuss with your doctor in detail and request for an ultrasound scan if required. Please do not worry."
},
{
"id": 41670,
"tgt": "Suggest remedy for infertility problem",
"src": "Patient: My height is 1.62, Blood group O+, I am 48 years. Have two abortions. Have no child. Have not seen menstral period since February, 2010. My husband has low sperm problem. Can you help us in place of advice. Doctor: Hello....im afraid,u seems to be approaching MENOPAUSE i.ea condition when a female's monthly menstural cycle stops naturally n ovaries stops producing ovum (eggs with which male sperm fertilizes resulting in pregnancy) but plz dont get disheartened as modern technologies can still help u in having a baby....(1) 1st of all kindly get these tests done from a good lab and tell me the reports of these tests - (a) complete hormonal profile including LH , FSH , ESTRADIOL , THYROID PROFILE (b) TORCH PROFILE (c) ULTRASOUND OF UTERUS AND OVARIES (d) SEMEN ANALYSIS OF UR HUSBAND.....tell me the reports as soon as possible so that i can guide u further....regards.....Dr Rahul Rathee"
},
{
"id": 37960,
"tgt": "What causes bleeding from stich after butt implant?",
"src": "Patient: My daughter got butt implants 3 weeks ago. She got stitches and one of them has been bleeding a ill bit. Last night she was complaining of leg pain and nauseous. Today she got in the car for five minutes and when she got up she was bleeding a lot from that stitch. It was clear fluid and blood. Called surgeon and he said its normal. Is it really? It happens only when seating not when laying on her stomach Doctor: Hello, Thank you for your contact to health care magic. I understand your concern. If I am your doctor I suggest you that slight bleeding and oozing of fluid from the stitch for few days is normal. But if it occurs after more than week it is abnormal and should be consulted by the doctor.I will be happy to answer your further concernYou can contact me. Dr Arun Tank. Infectious disease specialist. Thank you."
},
{
"id": 64353,
"tgt": "What causes painful lump on the groin area?",
"src": "Patient: i have a pea sized lump in my groin are where my abdomen and leg meet ... it hurts when I press on it ... i have a doctors apt on saturday ... what could this be? ... i had sex with my girlfriend maybe a month ago and had a really hard ejaculation ... this is when the pain started and that lump came Doctor: Hi,Good afternoon from India. Thanks for your query to My-HCM Clinic.I studied your query in depth.I understood your concerns.My opinion -You have-Chancroid/Soft sore mostly.-Its caused by the bacterial infection by H.ducreyi and is cotagious disorder.-Your last contact is the cause of this groin painful lump.-Hope this would help you to recover from worry-some query.Wish you a Fast Recovey.Wellcome to HCM again."
},
{
"id": 65024,
"tgt": "What is the large bump on my son's lower back?",
"src": "Patient: Hello,My 4 year old has had a bump about the size of a nickel on his lower back near his spine for months now. It was bigger last month when we took him to the doctor and she stated that it was 98% chance a bug bite. The bite went down but you can still feel a bump. He doesn't show any signs of it bothering him, and the doctor said that it is not on the bone. It has a bruise effect on it as well...any ideas of what this may be? Very concerned. Doctor: HI,Dear thanks for the query.Plz upload the photo and more info-over 2-3 sessions,which would fix the cause of the bump, which is there for months now.Thnks.Wellcome.Awaiting for the updates to my inbox on the HCM."
},
{
"id": 200780,
"tgt": "What causes itchiness in the urethra after masturbation?",
"src": "Patient: I am 14 year old boy and masturbated for first time yesterday. I did it again twice today. There was no semen that came out, except some watery liquid that looked like urine. I then stopped right away when I saw that. Later this evening, I started having itchy urethra. Sometimes it went away for while, but then sometimes it comes back few minutes after I pee. I might have masturbated intense/hard. Will this go away? Is it dangerous? I am uncircumcised. Doctor: Thanks for asking in healthcaremagic forumIn short: Friction/irritation due to lack of hygiene can cause thisExplanation: Friction during masturbation with dryness can cause this feeling. It may also be due to lack of cleanliness at that region. So, make it a habit of cleaning your penis by reatracting foreskin after sex/ejaculation. Hope this helps you.Please let me know."
},
{
"id": 190652,
"tgt": "Recurring canker sores in mouth, red spots on the back of throat, no fever. Trying to get off affrin nasal spray, on saline. What is the cause?",
"src": "Patient: my daughter has had repeated canker sores in her mouth and red spots on the back of her throat for about three weeks now. we thought it was hand , foot and mouth but now think different. she now has them on her tongue and roof of mouth. has no fever or has not felt sick at all., however she s trying to get off affrin nose spray and is on half saline and about half affrin now. Doctor: hello, as per location of sores- it seemed to be some viral infection, but all viral infections mostly heals within 2weeks.. but since your daughter is still facing them, i would suggest you to consult an oral medicine specialist as by clinical examination he can diagnose it whether- infective/allergic origin/side effects origin.. Afrin Side Effects These include but are not limited to: High blood pressure (hypertension),Heart palpitations,Difficulty passing urine,Signs of an allergic reaction, such as:An unexplained rash or hives,Itching,Unexplained swelling, especially of the mouth, lips, or throat,Wheezing or difficulty breathing. I would also suggest you to consult an ENT surgeon for nasal decongestion ..and get proper treatment at earliest.. multivitamins containing Vit-Band C, Lycopene will help in healing of sores in all cases.. if sore are painful then apply local anaesthetic gels.. prior to meals for comfort during chewing.. after consult from ENT surgeon/Oral medicine specialist take medicines/gels- antiviral/antibacterial/antifungal (infection) or antihistaminic/steroids (allergic/side-effect) please try to get her off from Affrin at earliest.. take care.."
},
{
"id": 127085,
"tgt": "What can cause acute pain and swelling in the foot?",
"src": "Patient: ok i have foot pain with a small area the swells and turns blue on my feet it started with one but now its both i was going to a pyhisical terpy and and for the fist foot and the dr was using ultra sound to relax the inner tissue the last time i seen them he touched that spot and it felt like a knife was shoved in to my foot then electric connected to it but for the most part my feet now when i run or walk to much or stand for long times they fell like there being smashed Doctor: Hi, How long have you noticed this swelling ? Does it always remain like that or does it disappear too ? It could be a varicose vein swelling. Kindly tell the exact spot where it is and whether it is bony hard or soft. Hope I have answered your query. Let me know if I can assist you further."
},
{
"id": 197710,
"tgt": "How long will it take for circumcision to heal?",
"src": "Patient: Hi i am age of 31 male i am planing for circumcision including removal of frenulum i mean fully removal of foreskin .how long it will take to heal completely heal and when i can have intercourse?how long it will take the surgery?thank you for your kind advice Doctor: Hello and .As an Urologist, i can understand your anxiety. After circumcision,it usually takes about a week to heal completely.By 3-4 weeks after surgery, you should be able to have intercourse.Duration of surgery is about 10 minutes.It's a routine surgery and you don't have to worry at all.Hope all your doubts are cleared.If you've any doubts,you may send it as a direct question, to me.Dr.Matthew J. Mangat."
},
{
"id": 197373,
"tgt": "Can Neosporin be used for incisions?",
"src": "Patient: I had a vasectomy on the 15th and one side healed really fast and the stitches came out. The other side did not heal so fast but my stitches came out and is open but not ruptured... There is no pus or swelling. I was wondering if I can put neosporin on the incision site? Doctor: Hi thanks for contacting HCM....Here from stitched site no pus is coming or not swollen ....So chance of infection at stiched site is very low ....It will take around approx 10 to 15 days for resolving absorbable sutures....The more comment possible after examining that area....But according to your history here no need for worry....Neosporin antibiotic cream can be applied over stitch although as such not needed just now...Healthy and balanced diet with more fruits taken for fast healing ...You can consult doctor if needed ...Hope your concern solved.Take care"
},
{
"id": 126102,
"tgt": "Is Naproxen safe for gout attacks while on Eliquis?",
"src": "Patient: I recently started taking Eliquis for atrial fibulation and in the middle of the night I developed gout in my knee. I have Naproxen from previous bouts of the gout. I s it safe to take a few Naproxen, it s an emergency. The pain is great and I can t put weight on it. Al B. Doctor: Hello, Yes, you can take Naproxen for pain relief from an acute attack of gout. It has no significant interaction with Eliquis. Hope I have answered your query. Let me know if I can assist you further. Regards, Dr. Jayesh Vaza, Orthopaedic Surgeon"
},
{
"id": 94027,
"tgt": "Abdominal pain, bloating, vomit feeling. Taking vegetable juice. What happened?",
"src": "Patient: Hi, we started vegetable juicing about 4 months ago - myself and 3 kids. We are NOT on a juice only diet we are on our regular food diet but just started incorporating about a cup and a half of vegetable only juice everyday. The first 4 1/2 months were fine but all of a sudden after drinking my juice one evening, I started with severe abdominal pain and bloating. I was belching continuously and kept feeling like i would vomit. None of the vegetables were any different than what we normally juice ( a combination of parsley, spinach, cucumbers, capsicum, carrots, beets etc). Also the kids were fine and continue to be till today but I have gotten this every single day for the last 2 weeks. I kept trying because was not convinced it was the juice since it made no sense that I would suddently develop this intolerance so I started eliminating 1 vegetable everynight till I was down to just cucumbers last night and I still suffered the same symptoms! No symptoms eating the whole vegetables even in huge quantities and nothing else has changed. Kids still fine with all vegetables. I'm wondering what happened. Any ideas? Doctor: Hi welcome to Health care magic forum. Thanks for choosing H.C.M.F. As you describe your self and 2 kids were taking vegetable juice containing, parsley, spinach, cucumbers, capsicum, carrot, and beets since 4.5 months. You are alone facing abdominal pain, bloating, nausea, in spite of omiting one vegetable per day, but no symptoms to children and nosymptoms to you when you eat same vegetables. I am not clear whether you have tried with out cucumber juice. Any how what ever the reason, is omit the juice comletely, if still the problem is continuing after few days, you must be having some problem with your stomach. If that happens i advise you to consult a gastroenterologist for diagnosis and treatment. you may need to have gastroscopy done besides other routine tests for confirmation. wishing for a quick and complete recovery. Best regards."
},
{
"id": 158277,
"tgt": "Fluid around heart discovered after having kidneys checked. Have hydronephrosis. Further?",
"src": "Patient: I went in to have my kidneys checked after delivery and the Ct discovered I have fluid around my heart . Why is this? I am scared to death something is really wrong with me. I have hydronephrosis during pregnancy and kidney stones . This is why I had the ct. I got the call saying that was all fine but now I have the fluid around my heart. I am afraid of having cancer or something serious. What could be causing this? Doctor: Hi and welcome to Healthcare magic.Thank you for the query.This is not cancer but this is not normal too. It is not dangerous if there is minor amount of fluid in pericard but sometimes it can indicate pericarditis and other heart disorders so it should be checked up frequently. Wish you good health. Regards"
},
{
"id": 57488,
"tgt": "What does my liver CT scan mean?",
"src": "Patient: left belt line to grone feels like a tube full and tight swelling and i sit and drive alll day had a ct scan this is what it says diffuse low-attenuation of the liver with focal sparing in the left lobe and ajacent to the gallblatter and a fatty liver and everything else is ok Doctor: HIThank for asking to HCMI can understand your problem your CT report of liver is suggestive of some nonspecific changes in images which need to be correlated with the clinical conditions but most of the times such changes are very much normal findings (diffuse and low attenuation) so nothing to worry about this hope this information helps you take care have nice day."
},
{
"id": 74663,
"tgt": "What causes problem while breathing?",
"src": "Patient: I have had breathing problems for several years now, due to allergies, that is getting progressively worse. and, yes I do take allergy medicatin and have tried inhalers. But, I've noticed for about the last year or so that my calves on both legs feel tight and seem to be somewhat enlarged. Are the two related and should I be concerned? Doctor: Hello,Both causes are different. Prolonged exposure to allergy can cause respiratory problems. You may be suffering from allergic rhinitis and asthma.Hope I have answered your query. Let me know if I can assist you further.Regards,Dr. Siva Kumar Reddy"
},
{
"id": 146297,
"tgt": "What causes pins and needle sensation in foot?",
"src": "Patient: Hi my name is luca and recently I have been having this sensation in my foot where it feels like someone is poking it with a pin it dose not hurt and it does not really affect what I am donig in my every day life but I would just like to know what is causing it thank you Doctor: Pins and needles is a pricking, burning, tingling or numbing sensation that is most commonly felt in the arms, legs, hands or feet. It does not usually cause any pain, but it can cause numbness or itching.'. A common cause is awkward postures that compress the nerves. Tingling sensations can also be symptomatic of nerve damage or nervous system disorders.The feeling of pins and needles is known as \"paresthesia.\" Most people associate it with sleeping incorrectly on a nerve. This causes the nerve to stop sending signals so that it \"falls asleep\" and creates a feeling of numbness. When pressure is taken off of the nerve, the body sends shock waves down the nerve to wake it up. This is what creates a feeling of pins and needles.But paresthesia doesn't just occur when a nerve has pressure. It may also occur during anxiety, especially during panic attacks. That's because during panic attacks, the body becomes more prone to hyperventilation.During hyperventilation, the body has too much oxygen and not enough carbon dioxide. That imbalance causes every vessel inside the body to constrict, while simultaneously reducing the release of calcium into the nerves. Both of these cause the nerves to feel \"tingly.\" Some vitamin deficiencies, such as low levels of magnesium, Vitamin B12, and vitamin B5 can also cause a tingling feeling and contribute to anxiety sensations .Prim rose oil is another very effective home remedy for tingling numbness. Apply the oil in to the affected area and gently massage, 2 to 3 times everyday. It shows quick results.Exercise of overall body and perticularly feet by rotating feet and legs from heels and knees will help blood circulation and allay feeling of tingling . Yoga and pranyam with meditation help attain perfect disease free ,healthy life .and intake of fiber ,& diet of all essential nutrents helps gain over all health .Steer clear of ,fast foods ,processed foods , caffeine and alcohol ,smoking, mental stress , worry , anger ,constipation , they reduce the flow of blood to the extremities.It's still important to see a doctor to ensure it is nothing more serious. Hope this helps you solve your query .Take care ,All the bestDon't hesitate to get back if have further query"
},
{
"id": 84946,
"tgt": "Does allergy to ibuprofin cause discoloration of scrotum?",
"src": "Patient: My 3 yr old s scrotum is bright purple on the top part and darker than normal on the bottom half. He is acting normal, no fever, no swelling. We found out he is allergic to ibuprofen last week, with lip swelling two times we gave it to him, so we are sure he is allergic to ibuprofen. Could this color change in his scrotum be at all related to having given him the ibuprofen from five days ago? We notice the discoloration started yesterday (4 days afterward) and thought he was just cold from being in the ocean. We are seeing our ped later today, but wanted to do some research in the meantime. Thanks if you have any thoughts. Doctor: Hello, In my opinion, the symptoms of scrotal discoloration are not related to Ibuprofen allergy. There are many other conditions that can cause scrotal discoloration. One of them is testicle torsion. So, I suggest doing a scrotal ultrasound to confirm the diagnosis. Hope I have answered your query. Let me know if I can assist you further. Take care Regards, Dr Dorina Gurabardhi, General & Family Physician"
},
{
"id": 108058,
"tgt": "What causes pain in the left side of the lower back?",
"src": "Patient: My girlfriend has been getting back pain for a few days. It suddenly came while she was bathing. She mentions that it's on the left side of her lower back. It didn't get better after seeking treatment from a principal physiotherapist. She went to the doctor to get it checked and was asked to visit a gynae. Any idea what might be the problem? Doctor: HelloLeft sided lower back pain may be due to many reasons like musculoskeletal causes,pelvic pathology,genitourinary calculus etc.History suggests that pain may be due to muscular spasm.She need proper clinical examination and routine investigations.Investigations include routine hemogram,,renal function test,urine RE/ME,ultrasound of abdomen,X-ray L/S spine (AP /lateral).Ultrasound of abdomen is important in diagnosis and treatment.Proper treatment depend upon findings.Get well soon.Take CareDr.Indu Bhushan"
},
{
"id": 42976,
"tgt": "What kind of diet and treatments are required to increase sperm count?",
"src": "Patient: Hello Doctor, I got married 2 years before, i have sperm count 40 million and motility 45 %. Kindly advice to increase the sperm count and also advice what are the food item will help us to increase the sperm count. Please help me to resolving those issues. Thank you, Raj M Doctor: Dear raj,Thanks for writing to healthcare magic.You have not written the complete report but from the minimum information you have given,You do not have significant problem as 40 million count is normal so is motility.Following foods help in improvement of count and conception.Green leafy vegetables.Sprouts. Foods rich in lycopene like tomatoes.Avoid Smoking as it kills the sperms potency.Alcohol.Tight pants/ trousersThanksDr Bhagyashree"
},
{
"id": 147194,
"tgt": "What causes dizziness and balance problems?",
"src": "Patient: Hi, I'm a 21yo female, 5'8, 110kg. I woke up this morning with severe dizziness and unbalance. I was literally leaning towards my left side. It's been an hour and I'm not as unbalanced anymore, but I still feel extremely airy and very slightly nauseous. Doctor: Hi,Thank you for posting your query.I have noted your symptoms and based on that, you seem to be suffering from peripheral vertigo. The commonest possibility in your case could be BPPV- benign paroxysmal positional vertigo, where the symptoms of dizziness get exacerbated on change of head or neck position.For relief of symptoms, you may take betahistine tablets.You should undergo an ENT evaluation if symptoms persist for long.Vestibular rehabilitation exercises would help prevent its recurrence.I hope my answer helps. Please get back if you have any follow up queries or if you require any additional information.Wishing you good health,Dr Sudhir Kumar MD (Internal Medicine), DM (Neurology)Senior Consultant NeurologistApollo Hospitals, Hyderabad, IndiaClick on this link to ask me a DIRECT QUERY: http://bit.ly/Dr-Sudhir-kumarMy BLOG: http://bestneurodoctor.blogspot.in"
},
{
"id": 133949,
"tgt": "What causes tingling and numbness in arm?",
"src": "Patient: I have a problem with my right arm. its a lot bigger than my left. I have throbbing pain through my whole arm. I have let this situation go to long about 2 months. I have no doctor at this time . im a 49 year old female, I also get tingling and numbness. im really scared . my father has had a stroke. right now I needed to talk to someone because im having severe pain in my whole arm. please help what should I do. Doctor: hi,thank you for providing the brief history of you.as mentioned by you the history a thorough neuromuscular assessment is needed.As you mentioned your symptoms post and assessment an MRI is guided . As it appears to be a pinched nerve in the cervical spine. For which I can recommend you a physical therapy. The therapist will help you with therapeutic ultrasound therapy and TENS therapy. Also with exercises on later stages you will recover well.in my clinical practice i see cases with similar problems. and 99% respond well to physical therapy.regardsJay Indravadan Patel"
},
{
"id": 6747,
"tgt": "What causes excessive chin hair growth ? Could it be a result of hormonal imbalance or is it due to vitamins and BCP taken by me ?",
"src": "Patient: I am 25 years female. I have never taken birth control pills . I have started taking prenatal and some other vitamins like E and B12. I have fibrosis so my doctor recommended taking vitamin E. I have started developing chin hair and it is looking very bad. My body weight is 132 but my body fat is very high 33%. I read online that obesity may cause the hair growth. I am wondering am I taking excess amount of vitamins that is causing problem. I am thinking to go to gynecologist to check my hormones level. Please guide me through this. What will be the cause of my chin hair growth. Doctor: Hi, Chin hairs indicate ovulation problem due to harmonal imbalance .Your thinking is correct. PLease get your harmone level&sonography done .Please dont worry The condition is reversible.Only proper treatment by Gynac &endocrinologistis needed.The vitamines you are taking are not related to hairs on chin. For time being, you can think of electrolysis by butitian,this will enhance your confidence.&side by side treatment by specialists would control the problem. Thanks"
},
{
"id": 155012,
"tgt": "What are the symptoms of bowel cancer?",
"src": "Patient: Hi i am 33 years old , had my second delivery by c section two weeks ago. I am now experiencing black stools and when I wipe it's bright red blood. I am taking vitamins, including iron, and my family has a history of bowel cancer-my dad. Should i be worried. Thanks in advance. Doctor: Hi, dearI have gone through your question. I can understand your concern. Symptoms of bowel cancer are blood in stool, weight loss, abdominal pain. Symptoms of obstruction may be there. You have family history of colon cancer. So you should go for colonoscopy and if needed go for colonic biopsy. Early diagnosis is very important. Consult your doctor and go for colonoscopy. Hope I have answered your question, if you have doubt then I will be happy to answer.Thanks for using health care magic. Wish you a very good health."
},
{
"id": 127991,
"tgt": "What causes pain and numbness in the legs?",
"src": "Patient: my mom is 80. about 3 weeks ago her leg swelled up from the knee down. was checked for blood clots. negative, just finished antibiotics because doc said it was an in fection . now she has numbness, sores that are scabbing over, and she says her ankle and foot hurts to touch it, Doctor: It might be resulting due to the blockage of lymph node or may be DVT.proper diagnosis is made on the basis of investigation."
},
{
"id": 221862,
"tgt": "How to treat swollen vagina during pregnancy?",
"src": "Patient: Hi, im 34 weeks pregnant. my vagina feels really sore and achey. im struggling to turn over in bed or even open my legs. i checked in a mirror and it looks really swolen. i went to a midwife and she said the baby is head down. but not in position. is there anything i can do to help? Doctor: if you take my Advice then I would suggest you that get your blood pressure chequed as these can be ominous sighs of eccllampsia also I would suggest you to get a urine for albumin done as early as possible"
},
{
"id": 84050,
"tgt": "Is prednisone causing eye inflammation, itchy stools and bloatedness?",
"src": "Patient: I have been on prednislone for nine months and have stopped - due to inflammation in my right eye - I have now stopped and have paid in my knees and my faceis coming up with spots and itchy. My tummy seems to be very bloated too. Any advice is verymuch appreciated. Many thanks Doctor: Hello,You have been using prednisolone for a long time and it is advisable to reduce it's dosage gradually in order to prevent the side effects of stopping it. Use probiotics to re-establish the intestinal flora.Hope I have answered your question. Let me know if I can assist you further. Regards, Dr. Olgeta Xhufka, General & Family Physician"
},
{
"id": 180579,
"tgt": "Suggest treatment for bad mouth odor post tooth extraction",
"src": "Patient: I had my tooth extracted out last Wednesday but to the disappointment it snapped of and the dentist then had to give up after spend on 1 hour trying to remove it. just the root Is now left , went back the next day for a clove paste appiled as in a lot of pain , have to wait 4 weeks to be referred to hospital but it's there anyway of getting this disgusting smell and taste away in the mean time. Doctor: Hello,As per your query you have symptoms of bad mouth odor post tooth extraction which occurs because of the persistence of tooth infection leading to facial space infection.Need not to worry. I would suggest you visit an oral surgeon once and get a complete examination done. You should take the proper course of antibiotics and get the dental procedure done.Do warm saline gargles and chew on sugar-free candies to stimulate salivary flow. Maintain complete oral hygiene by use of an antimicrobial agent. Keep applying ice packs on jawline on a daily basis.Hope I have answered your query. Let me know if I can assist you further.Regards,Dr. Harry Maheshwari"
},
{
"id": 53427,
"tgt": "Is it proper to avoid treatment for liver cancer,hepatitis b?",
"src": "Patient: Hi Doctor, My Father is suffering from liver cancer, he is 64 years old , we are currently checked in CMC velllor(Tamilnadu ,Indai).Now Doctor said it comes under stage four liver cancer with Hepatitis B. Doctor said no need to take any treatment just live it as it is. Pls advise what should i do . he is normal now but little bit sick. Doctor: Hi,I am sorry but stage 4 of liver cancer is a highly advanced stage where the cancer has metastasised to various other organs and parts of body, on top of that viral infection with Hepatitis B has further complicated the things. Therapeutic approaches for the treatment of HCC can be classified into three categories: potentially curative, palliative, and symptomatic. Potentially curative treatments, including liver resection, transplantation, and local ablation, are associated with promising 5-year survival rates of up to 75%. But is not possible when carcionoma is in a very advanced stage. Due to the presence of excessive metastatic disease, the patient is not a candidate for curative surgical resection. Other forms of local therapy, such as percutaneous ethanol injection, cryosurgery, radio frequency ablation, hepatic intra-arterial chemotherapy, or chemoembolization that typically play a role in the palliative treatment of patients with localized, non-resectable disease, are not appropriate in this patient with metastatic disease. Cryosurgical procedure can be tried but are not very successful. Follow advice of the doctor at CMC Vellore.Hope I have answered your query. Let me know if I can assist you further. Regards, Dr. Ramesh Kumar"
},
{
"id": 173406,
"tgt": "How to control bloody mucus stool in kids?",
"src": "Patient: My five year old has had a bloody mucus stool it also had worms in it. I had treated this but was informed by my child that they spat the medication out. my other two children also have watery stools, with no blood or worms. I'm unsure as to what this is. 2 of my other kids also have chicken pox atm. Any idea what's wrong? Doctor: many thanks for consultation.bloody diarrhea and chicken pox are both highly contagious diseases.Bloody diarrhea is bit serious issue .this child needs admission in the hospital for the following;1. intravenous fluids2.stool culture3.intraveous antibiotics and other supportive medicine.if they have worms then needs few arming of all family after the acute problem is over.other 2 children need rehydration and rehydration with oral electrolytes solution only.chicken pox is contiguous disease that mean they catch from each other.now they can not get in future because they are immune from it now."
},
{
"id": 134584,
"tgt": "What causes popping in the ribs?",
"src": "Patient: im 15 and since i was 9 ive been able to pop/crack the middle of my ribs. it never used to hurt but now it does i cant lay on my belly on side anymore and havent been able for a few months because its uncofterable there and it has been popping/cracking when i went to the doctors when i was 9 with it he said it would go away but i hasnt it went away for a few months and then came back. plase could you help me as it hurts a lot. Doctor: you can meet an orthopedic and a pulmonogist. post their advice you can visit a physical therapist who will teach how to perform breathing exercises to strengthen the respiratory muscles. hope this works."
},
{
"id": 13676,
"tgt": "What caused itching in the palms of my hands and rash around suture?",
"src": "Patient: I had colon surgery not quite two weeks ago. I have suddenly started sleeping all day long and the palms of my hands itch so bad I could scratch forever and not satisfy this itch it is so intense. No rash or bumps however I developed an itchy rash around two of the four incision sites that were pretty itchy as well with a rash??? I have racked my brain but cannot find the common thread. Please Please help me. Doctor: Hi, Itching may be due to either irritation or allergy. Consult the dermatologist for the perfect diagnosis and proper treatment.Antihistaminics like cetirizine would relieve the itching. Apply mild steroid cream on the affected area. Avoid soap contact. If needed,oral steroid may be taken after consulting doctor in tapering dose. Hope I have answered your query. Let me know if I can assist you further."
},
{
"id": 187190,
"tgt": "Could infection in his teeth cause the chest pain if heart tests are normal?",
"src": "Patient: Hi, my name is Debbie. My son had a heart attack 2 years ago and had a stint put in. After he has had some teeth problems lately he has had severe chest pain but they have ruled out his heart multiple times. Even done stess test and imaging to be sure stint and heart is okay. He has no damage to his heart at all but continues to have chest pain lately he has had some teeth get very bad has had to have 3 extracted and they have all been infected. They put him on antibiotics after the fact. Could the infection in his teeth cause the chest pain? Doctor: Hi,Thanks for posting the query, No this is not related to dental causes, i would suggest you to consult to an Physician and get the checkup done.Also get the infected tooth checked and treated by a Dentist.Take care!"
},
{
"id": 161246,
"tgt": "What causes irregular heartbeat in a child?",
"src": "Patient: My 5 year old son wakes up during the night and has an irregular heartbeat when he wakes. The heartbeat has long pauses, but not all the same length in time. He also stops breathing for periods of time during sleep as well. Is this something that needs immediate medical attention? Doctor: Hello, Yes, you need to consult a pediatrician for the above problem. 24 hours ECG monitoring also available nowadays to diagnose exactly what is the problem. Hope I have answered your query. Let me know if I can assist you further. Take care Regards, Dr Rajmohan, Pediatrician"
},
{
"id": 83074,
"tgt": "Red rashes on arms, legs. Using creams, no relief. Lupus symptoms?",
"src": "Patient: Hi,doctor. I would like to ask something for a frind of me who lives in XXXXXX. She has all arms,legs and front body, full of red and iches,fast 2 weeks already. She has used many cremes but doesn``t hele her. The doctor says allergy , but I saw the pictures that she send to me and I think is something like Lup\u00fas, do you knos this disease? Doctor: Hello, Your friend may be having an acute generalised eruption of urticaria that can take long to control if not on optimal medication. 80% of people with urticaria do NOT have allergies but respond to long-acting non-sedating antihistamines like allegra, levocet etc If the rashes leave a scar, bruise then a separate condition called urticarial vasculitis may be the problem for which a skin biopsy with special stains will be required. You should ask your friend to see a Dermatologist right away. Lupus has several menifestations like photosensitive rash on face, arms and legs (exposed to sun), but also with arthraligia, arthritis, serositis, blood changes, immunological abnormalities (like positive ANA test) and several other systems may be affected. So it is very early to tell unless at least 4 of 11 criteria are fulfilled to be labelled as lupus. Thanks."
},
{
"id": 28986,
"tgt": "Is anti rabies vaccine effective in preventing a cat scratch infection?",
"src": "Patient: What percent of effectiveness is the rabies vaccine in humans. My son was bitten by a bat and he s terrified that the vaccine might not work. Some of the websites he saw said that it almost always cures rabies but the word almost has us very scared. Doctor: Hello,Rabies vaccine is very effective with very high preventive rate and works for against rabies whatever the host animal is. The course of vaccination post exposure is four doses at ( 0, 3, 7, 14 ) day. Two doses (0, 3) day are required post-exposure if already vaccinated within two years.Hope I have answered your query. Let me know if I can assist you further.Regards, Dr. Mahmoud Abouibrahim"
},
{
"id": 44226,
"tgt": "Total sperm count/mL 70,000,000.Am I fertile?",
"src": "Patient: i had done a semen analysis and this was the result: volume: 2ml color: greyish white blood: absent reaction: alkaline/PH8.0 viscosity: normal liquefaction time: 30min total sperm count/ml: 70,000,000 sperm count/ejaculate:140,000,000 motility: 1st hour: 50% 2nd hour: 40% 3rd hour: 30% type of motility: rapid progressive 30% non progressive: 20% immotile: 50% abnormal forms: 50% type of abnormality: amorphous pinpoint head & enlarged midpiece agglutination: absent peroxidase test: spermatogenic cells/ml: 100,000 W.B.C.s/ml: 0000 R.B.C./H.P.F: 1-2 Trichomonas: absent others: absent and a comment is written that CASA and alpha glucosidase are suggested these were the result...the question is: depend on these results am i fertile or not......? and what are CASA and alpha glucosidase Doctor: Hi, Thanks for your query. I have read your query & I understand your concerns. You have absolutely normal report. There is no doubt about your fertility. I am sure you will conceive with present semen analysis. You dont need any treatment for fertility. If you are in hurry to conceive please go for assisted reproductive technologies like IUI. I believe your motility will increase when you follow above instructions and repeat the test after 3 months. All the best. I hope I answered your query. I will be available for any followup queries you have. Regards, Dr.Mahesh Koregol IVF & Infertility Specialist."
},
{
"id": 44909,
"tgt": "How to conceive with poor egg quality ?",
"src": "Patient: hello i m 28 years old . i got married last 3years before. we are trying for baby from last 1 year but i m not get pregnent. now i m taking treatement for pregnency. i was do folicle study on 2nd day of my period at tat time my fsh level is 10.8.they gave me letrozal2.5mg for 5 days twice at night.Then again i did folicle study on 11th day of the period ,at tat time my doctor said tat my result is so poor the egg size is 13x11 and 11x11. so plz tell me is it major problem according to get pregnent & when i will be pregnent ,please will you help me doctor? Doctor: Hi, Welcome to HCM. Normally a follicle gradually enlarges and approximately 14th day of the menstrual cycle, it ruptures releasing the ovum which has to get fertilized with the sperm in the Fallopian tube, leading to pregnancy. Usually follicle ruptures when it attains the size of 18 to 25 mms. In your case it appears that the follicle failed to mature. There are medicines available for stimulating the follicle to mature, but these medicines have to be taken strictly under medical supervision."
},
{
"id": 8652,
"tgt": "Can I take melacare forte for getting a fair complexion?",
"src": "Patient: hi doctor. i am thinking to use melacare forte cream so before i go for it i wanna know your suggestion and views of melacare forte cream. i heard from some of my friends that melacare forte cream just take out a layer of skin by which its look fairer n while we stop using it our faces becomes dark then ever before ..... thanks a lot ..... Doctor: hi , Melacare forte cream is a known lightening agent. It is one of the triple combinations that is widely used by dermatologists all over. It is a good and effective agent if used under medical supervision. Using such combinations as OVER THE COUNTER DRUGS is not really recommended. Yes, there is no doubt that it does improve the skin tone when used. It is used by dermatologists for various hyperpigmented skin conditions. But i would recommend you not to use it without prior assessment and recommendation by a skin specialist. One of the known problems with such medications is that it can cause hyperpigmentation , if not used properly and as advised by a dermatologist. Such medications have to be used only for a limited period of time. Excessive use can cause problems. Also these are to used only at night time. Hope i have answered your query Dr Geetika Paul"
},
{
"id": 97529,
"tgt": "Suggest treatment to remove ovarian cyst other than surgery",
"src": "Patient: helo... i just take some test.. one of this transviginal ultrasound they found out that i have the ovary cyst.. as the doctor read the result.. the doctor said i have to take asap operation must do... pls..help me to send some alternative what should i do about my situation..theres any other way to cure this..or to atake a herbal mecicine. thanks Doctor: hai,ovarian cyst can be cured with Ayurveda medicine I advise to follow a healthy lifestyle.Be hydrated.take aloe vera juice 20ml twice a day.beneficial for uterus and ovaries.add turmeric in your daily food intake. it will dissolve all tumor and got anti inflammatory action.avoid fried,junk,caffeinated drinks etc.ayurvedic medicine like Kachnaar Guggulu,Trailokyachintamani Ras , Abhrak Bhasma will be help ful in ovarian cyst.kindly consult a ayurvedic specialist in person to treat your condition as per classical ayurvedic treatment.we treat as a whole not a single condition.thank youhope my answered will help to treat your condition."
},
{
"id": 194301,
"tgt": "Advice required to allay fears about penis size",
"src": "Patient: hello, I am about 27th years of age, I have a question that I didn't ever ask anybody because of shame. My family push me to get married but I am so much concerned about my penis size. Though it may hilarious to all of you but its really painful for me coz my penis length is only 4.5''. Now I decide not to get married and don't want to ruin any girl's life. Sir could you please help me to take a decision in this critical stage of my life. This problem just ruining my life from my youth stage. Doctor: Hi, Many Research says the average length of a flaccid penis is 3.61 inches, while the average length of an erect penis is 5.16 inches. Few studies recommended flaccid penis length of fewer than 1.6 inches or an erect penis of fewer than 3 inches should be considered for treatment. Surgical (liposuction around their pubic bone, skin graft), inflatable penile prosthetics. Hope I have answered your query. Let me know if I can assist you further. Regards, Dr. S.R.Raveendran, Sexologist"
},
{
"id": 139164,
"tgt": "Can Potassium cause ankle swelling with spasms and leg cramps?",
"src": "Patient: i been notice swelling of my ankles i been on over the counter potassium taking 3 tabs daily 550 mcg daily was not on potassium,my heat had a bad spasm,with also leg cramps my dr put on a blood pressure diovan/htz 80.12.5 for 6 years now,am i taking too much potassium? Doctor: Dear patient Potassium excess In blood leads to all symptoms you described. Potassium is an electrolyte and leads to fluid retention which is seen as ankle oedema. It also alters membrane function and leads to muscle irritability and muscle spasm and cramping pain. Excess of potassium is bad for the heart and has life threatening side effects. I would advise to stop potassium stat and check for your serum electrolyte level. If it's raised you need to take urgent visit to expert physician. All the best."
},
{
"id": 113643,
"tgt": "Severe pains in spinal cord. Pricking sensation, numbness. What is the treatment get relief from this pain?",
"src": "Patient: I suffer from Spinal problem with severe pricking, stitching pains running from the top of my spinal cord at the neck down to the tips of my toes. Esepecially on the left side just where the Kidneys are there is severe pains on the left side of spine and when I sit or stand for long thee is a numb but deep pain from the left side of my hip to the knee . There ae pricking pains especially on both big toes. There is also numbness as if cells and muscles are dying. I had lifted a very heavy girl and played with her about 14years ago and the pains started then. I have been to several Physyiotheripist Consultant doctors, all they give are painkillers, tractions, and lights treatment, which has not done any good. In fact the pains are moving upwards to the nape of my neck and the shoulders and below the shoulders at the back. Doctors are advising surgery, but I am afraid of landing in a wheelchair or bed ridden if something goes wrong. Doctors say that screws and other item will be inserted and I am not in favour of any part of my body being screwed and whatever. WHAT IS YOUR DIAGNOSIS ON MY SPINAL PROBLEM AND CAN YOU GIVE ANY ADVISE TO RELIEVE THE PAINS WITH EXERCISES? Doctor: Hello, Thanks for posting your query You might be suffering from PIVD. Due to disc bulge , pressure occurs over nerves & this may cause pain & numbness over big toes. You may require a MRI to confirm the diagnosis & to know how much pressure occurs over nerves due to disc bulge and at which level. You may require surgery if pressure is more , but don,t affraid of surgery. With physiotherpy. spinal extension exercises. avoid forward bending &prolonged sitting ,pressure over nerve can be reduced & symptoms can be relieved. Regards"
},
{
"id": 17325,
"tgt": "What causes abnormal working of lungs after a mitral valve replacement?",
"src": "Patient: hello, i m BRIJESH,son of MANJULABEN...She is hospitalzed last 2 month..she is open haret & chnge mitral valve..opertion is good but she lungs are not working prpoperlly..& she is legs are not working properlly & she nt abale to standing.after opertioan lungs & legs are good.. working..i dont no ? ple do some guide & medicine...ple....ple....am in very attech my mother..regards BRIJESHKUMAR..0000 Doctor: Hi, I understand your concern and would like to review her tests reports in order to be able to give a more professional opinion. Could you please upload those tests for me to review! Hope I have answered your query. Let me know if I can assist you further. Regards, Dr. Ilir Sharka, Cardiologist"
},
{
"id": 23913,
"tgt": "Suggest treatment for heart palpitations",
"src": "Patient: Hi, im ana. I have heart palpitations,but a couple nights ago i started feeling really sick always during the night. Out of nowere i feel extremly dizzy to the point that i feel if i get up i'll fall, my heart starts beating quicker, i get vomiting, a big headach, but what worries me the most is that mt mouth goes numb to were it makes it hard to move. I would like to know if this is due to my heart oalpitations or something else. Im 19 weigh 109 and am 5'7. Doctor: Hello Ana, your palpitations and the rest of your symptoms could be resulting from something underlying them. If your thyroid hormone is abnormal, for example, this could explain your symptoms. Another possible explanation is dehydration, so be sure to drink enough water (at least 64 ounces, or 2 L, per day). Another common reason for these could be pregnancy. I hope this helps."
},
{
"id": 178377,
"tgt": "Are Ambroxol Hydrochloride and Guaiophenesin oral syrups for running nose?",
"src": "Patient: Hi, My daughter is 7 months old; she is having running nose & congestion; Doctor advised to syrup Ambroxol Hydrochloride, Terbutaline sulphate & Guaiophenesin (AERODIL) 2.5 ml 3 times a day!!! I doubt whether syrup should be given or oral drops have to be given. I have already given 2 doses of 2.5 ml. Will this affect my baby??? Pl reply Doctor: HiThanks for writing to healthcare magic.Yes you can give.Dose is fine.This medicine for congestion and will bring out her secretionsNo harm doneWishing your child speedy recoveryRegardsDr Arun"
},
{
"id": 28159,
"tgt": "What could be the reason for having a tightening sensation of my heart?",
"src": "Patient: Every so often my heart will tighten as though I am scared when I am not and my rate will speed up considerably. It often occurs at random intervals and the tightening sensation only lasts for a few seconds when ever it does occur. From what I can tell, nothing specific triggers it, however it often makes me gasp or become short of breath until the sensation subsides. Doctor: Hi,After going through your case I want to know-1) Age & gender.2) Presence of diabetes, blood pressure problem, cholesterol problem.3) Family history of heart disease.4) Habits like smoking, alcohol.5) Any medicines are you on.You should not ignore these symptoms.There might be following causes for it -1) Heart attack.2) Cardiac arrhythmia (Electrical disturbances in conduction system of heart). 3) Anxiety if there are any mental stress.Though it needs to be confirmed by evaluation. I will advise you -1)You should visit emergency care during episode at that time electrocardiogram (ECG) may give important information. 2) Resting ECG, electrophysiological study of heart, 2DEcho of heart.3) Blood levels of cardiac enzymes Troponin I/T, CPK-MB.4) Tests for kidney function.5) Thyroid function. So better to consult cardiologist as early as possible.I think this may helpful to you.If found helpful vote helpful.Feel free to write back your queries if any."
},
{
"id": 85064,
"tgt": "Could itching in finger and foot be caused by using cytotecs?",
"src": "Patient: itching after taking cytotecs my girlfriend and I trying to do a self abortus. after a few minutes after taking 2 pills of cytotecs under the tounge and 1 pill in the vagina, she s starting to feel itch on her finger and foot. she also feel cold. what s happen? please help. thank you Doctor: Hello dear and Welcome to \u2018Ask A Doctor\u2019 service. I have reviewed your query and here is my advice. The symptoms seem to be related to an allergic reaction. I suggest to stop using Cytotec. I also recommend using antihistamines such as Cetirizine 10 mg to relieve the itching. Hope I have answered your query. Let me know if I can assist you further.Kind regards! Dr.Dorina Gurabardhi General &Family Physician"
},
{
"id": 199183,
"tgt": "What could be the reason for not getting proper erection?",
"src": "Patient: I just got married and I am not able to get proper erection, It was all good before and suddenly it started to give issues. I can get erection sometimes in the morning when I get up. On the bed I am not able to maintain the erection don't understand what is going on? Doctor: DearWe understand your concernsI went through your details. The given symptoms are not enough to have a proper diagnosis. Erectile dysfunction could be due to many reasons, but more prominent is psychological reasons like performance anxiety or stress. You should consult a physician to make sure you do not have physical problem like diabetis or erectile dysfunction due to hormonal imbalances. Then approach a psychologist for counseling.If you still need my assistance in this regard, please use this link. http://goo.gl/aYW2pR. Please remember to describe the whole problem with full detail.Hope this answers your query. Available for further clarifications.Good luck."
},
{
"id": 50137,
"tgt": "Pain in side. Done CT scan. Advised Nuclear scan. What is the problem? Treatment?",
"src": "Patient: It is known that Hydroureteronephrosis occurred after post hysterectomy . Pain in my right side of my stomach .. after ct scan they say that contrast is not flown even after 5 hours in my right kidney .. They say that it is little impaired .. It is also known that my right ureter is narrowed (distal collapsed) ... what do you think is the actual problem & the right treatment for it ? Now advised scan is Nuclear scan . Doctor: Nuclear scan is unlikely to give any additional information in your case.You should get a cystoscopy and ureteric stenting by a urologist instead.what needs to be seen also is if the ureter is not accidentally injured or tied during hysterectomy, in which case a ureteric stenting will not be possible.You have not mentioned when and how was the hystetectomy done?"
},
{
"id": 142928,
"tgt": "What causes warm palms and feet with dizziness and headache?",
"src": "Patient: Hi, I'm Ana and I'm 23 years old. My palms and feet are always warm, even if I'm inside an air conditioned room. I always feel tired even if I have 8 hrs more of sleep. Sometimes I have like those stabbing pain like head aches that usually occur in one area of the brain. Why is this? Doctor: HIWell come to HCMI really appreciate your concern, if it is not the febrile condition then it is nothing to worry and to rule out this I would suggest to read the body temperature else the worm feet and palm are not indicative of any disease, headache could be due to stress and other symptoms could be that only, hormone changes could be the reason hope this information helps."
},
{
"id": 136786,
"tgt": "What causes weakness in legs with dizziness?",
"src": "Patient: I started experiencing weakness around my laps and legs after birth. Sometimes, I feel like my legs are locked and cannot be lifted and this sometimes get painful. I feel dizzy sometimes too. All these started three years ago when I had my last born. Please what could be causing or what exactly is the problem? Doctor: Hello, I have studied your case. Leg weakness can be reduced byMassaging the cramped muscle with your hands or oilDrinking plenty of fluids to avoid dehydrationStretching your leg muscles or riding a stationary cycle.Taking diet rich in calcium and potassium or oral supplements of the sameCheck your vit B12 and Vit D 3 levels.Till time, avoid lifting weights, You can consult physiotherapist for help.Physiotherapy like ultrasound and interferential therapy will give quick relief.Hope this answers your query. If you have additional questions or follow up queries then please do not hesitate in writing to us. I will be happy to answer your queries. Wishing you good health.Take care."
},
{
"id": 111374,
"tgt": "What causes back pain and tired always?",
"src": "Patient: Im 14, weighing 65-69 kg and im a female. I have pain on my lower left part of my back 3 days after running during PE. It has been a week now and it still hurts. I also found that i have a random bruise on the back of my elbow which is less reddish and purple-ish. I feel dizzy whenever I try to stand up quickly and i am always tired/sleepy. My skin has loss most of its tan but now it looks a little pale. Not too pale. Should i see a doctor? Doctor: Dizziness, paleness, bruising, and fatigue are a concern, especially for possible anemia. You should see a doctor and get vital signs and blood tests, including a complete blood count."
},
{
"id": 87982,
"tgt": "What could cough with body shakes and abdominal pain suggest?",
"src": "Patient: I have a horrible cough and I lost my voice temporarily, it's back after three days but the cough is still here and it's so violent my whole body shakes. I'm also developing a pain in the lower left side of my abdomen now too when I cough. I've been on antibiotics for 3 days now and have 2 more days left, but the only thing that has improved is my voice came back. What could be wrong with me? Doctor: Hello,Wait for the completion of five days of antibiotic, take codeine plus cpm cough syrup one to two teaspoons three times a day and tablet Meftal Spas.Hope I have answered your query. Let me know if I can assist you further. Regards, Dr. Dilawar"
},
{
"id": 124290,
"tgt": "Need medicine for swollen heels",
"src": "Patient: Hi, I have recently worked in a job that required me to wear flat shoes and was onmy feet most of the day,, iv been left with painful swellings just above both heels, which makes it almost impossible to walk first thing in a morning, Can you help Kind regards Beverley Doctor: Hello, What I feel the swelling is occurred due to lack of venous return. Dipping the feel in water and then keeping it elevated over a pillow in lying position should help reduce one. Also, if you are having too much swelling than using Lasix as a medication should help reduce more. Please let me know if you have any high blood pressure or diabetic symptoms too. Hope I have answered your query. Let me know if I can assist you further. Regards, Jay Indravadan Patel, Physical Therapist or Physiotherapist"
},
{
"id": 299,
"tgt": "How should PCOS with amenorrhea be treated for a successful pregnancy?",
"src": "Patient: I have pcos, and have been on and off metformin for 2 years. all my doctor wants to do is keep putting me on metformin, now i have changed my doctor. ive read about clomid, how can i be put onto this? and how long does it take to fall pregnant on it? i dont have monthly bleeds, although i did have one in january, started on Jan 21st for 5 days, but i havent had one since. me and my bf are sexually active as we want to get pregnant. i done a HPT in feb that said negative, and my doctor did a blood test and told me that \"everythings fine and normal\" but didnt say exactly what, and when i asked he just repeated \"everythings fine and normal\" please adivse. :( Doctor: Hello and Welcome to \u2018Ask A Doctor\u2019 service.I have reviewed your query and here is my advice.Get serum TSH and serum prolactin tests done then take clomiphene from day 2 of the cycle with gonadotropins, if needed, as prescribed by a gynecologist. Go for serial follicular study till follicle rupture.Hope I have answered your query. Let me know if I can assist you further.Regards,Dr. Sheetal Agarwal"
},
{
"id": 85562,
"tgt": "For what sporelac,bidanzen forte,rabicip are used?",
"src": "Patient: i had undergone haemmoroids surgery and fistula surgery n dr had prescribed me sporelac,bidanzen forte,rabicip d,movon and tab ceftum in after surgery medication.what these medicines are for as i m felling lot of pain and bleeding is coming when i m passing stool. Doctor: Hello, Sporlac is lactobacillus which is usually given along with antibiotics to keep gut flora normal. Bidanzen is an anti-inflammatory and it might have been given for the inflammation post hemorrhoid surgery. Rabicip is Rabeprazole, an antacid that was given to you because you were prescribed pain killer and other drugs. Hope I have answered your query. Let me know if I can assist you further. Take care Regards, Dr Prabhash Verma, General & Family Physician"
},
{
"id": 47190,
"tgt": "How to treat kidney infection?",
"src": "Patient: Had a kidney infection and was hospitalized for 3 days, 4 weeks ago. I took all my antibotics. But now I am having pain again on my left lower back but I am not running a fever, last time I had a fever of 104 and chills. Could this be kidney infection again with no fever? Doctor: HelloThanks for query .You were hospitalized for fever due to kidney infection and treated with antibiotics .Now you have pain in lower back which .In view of Pain in lower back with previous history of proved kidney infection in recent past ,possibility of stone either in kidney or ureter has to be ruled out .Please consult qualified Urologist for clinical examination and get following basic tests done to confirm the diagnosis.1) Urine routine and culture.2) Ultrasound scanning of abdomen and pelvis.Further treatment will depend upon result of these tests and final diagnosis.Dr.Patil."
},
{
"id": 222751,
"tgt": "What are the side effects of Naturogest during pregnancy?",
"src": "Patient: , may I answer your health queries right now ? Please type your query here... my wife is taking naturogest 200mg capasule one daily and same strength 1 injection per week. she is in 8th week of pregnancy. Is it it safe or it will have adverse effect on baby? please clarify. Doctor: Hi,Progesterone supplements are given in early pregnancy to prevent miscarriage. It is not certain exactly how helpful they are, but are prescribed by many doctors, sometimes even when there are no risk factors for miscarriage are present.You need not take both tablets and injections, any one would suffice. Does your wife have any symptoms or risk factors of miscarriage?Hope this helps.Regards."
},
{
"id": 28809,
"tgt": "How can UTI be treated?",
"src": "Patient: So I bought some test strips and I have a urinary tract invention \u201cLeukocytes\u201d my sides and back hurt allot and my kidneys do also. Unfortunately I\u2019m a truck driver so I can\u2019t really go to the DR\u2019s ..... luckily I\u2019ll be home on April 4th witch is only 5/6 days away. Y\u2019all think That I can fight it out until I can get home and get anti biodics or is this crap serious ? I don\u2019t have a fever FYI Doctor: Hello and Welcome to \u2018Ask A Doctor\u2019 service.I have reviewed your query and here is my advice.To treat Urinary tract infection (UTI), you can take Levofloxacin drug for 5 to 7 days. Your UTI will be alright after that.Hope I have answered your query. Let me know if I can assist you further.Regards,Dr. Kaushik Talukdar"
},
{
"id": 132308,
"tgt": "Should I consult the doctor for the shooting pain in the calf started after the treatment for Pulmonary embolism?",
"src": "Patient: Hi, I recently had a pulmonary embolism and was prescribed warfarin. My INR levels have been good. My last reading was 2.2. However, suddenly today I started getting a sharp pain in my groin. It wasn t bad at first but now it is quite painful. Also, it started shooting pain down my right calf. I am worried that it may be another clot but I don t know how that would be possible. Also, It seems to go away when I am sitting, but hurts when I am standing or walking. Should I go to the ER Doctor: As you had pulmonary embolism and you are on treatment for it.If you are having any pain in the calf there may be chances of dvt.You should visit your doctor and consult regarding the condition and in such cases you need to be careful."
},
{
"id": 16850,
"tgt": "Does cyanotic chd cause loose stools?",
"src": "Patient: hi doctor, my daughter is 20 months old with cynotic CHD.for the last week she was sufferimg from cough and cold. now she is better ans again started with loose motions..i wanted to know if this is due to her heart problem for other problem...the doctor here have pescribed her with ondem and derolac and zinconia.are these medicine all right...tham nks...waiting for your reply Doctor: Hi, CHD will not cause diarrhea. It may cause breathlessness and decrease oral intake or may cause bluish discoloration.Diarrhoea may be related to GI infection.And yes, the treatment was written is absolutely right. Hope I have answered your query. Let me know if I can assist you further. Regards, Dr. Sameer Maheshwari, Cardiologist"
},
{
"id": 159812,
"tgt": "Is homeopathy treatment good for oral cancer?",
"src": "Patient: Hello, My question is related to oral cancer . I was addicted to tobacco and at the age of 24 I got a pimple on my lower lip which wasn t healing , so 4rth day I decided to go see a doctor. He didn t tell me anything but I told him everything that this pimple isn t healing from last 4 days and also I said I got a lump feeling in my neck right side of my neck.. I also lose weight not much. Dr. only advice me to start the course of homeopathic medicines and from last 2 years(now I am 26) I am continuing his medicines and now I don t cough like before(but sometimes when its cold), pimples are disappear(but comes again when I eat oily and hard food or fruits or drink), loss of hair and no glow on my face.Please I am depressed a lot, What will you suggest me? Doctor: A \"whitehead\" (more commonly known as a pimple or a closed comedo) is a follicle that is filled with sebum, but lacks a small opening to the skin surface. You should go to a dermatologist to double check it. Do not think to oral cancer, because this would not disappear and come again."
},
{
"id": 84263,
"tgt": "Will taking flucloxocillin cause delayed period?",
"src": "Patient: I am currently taking flucloxocillin for a boil that i have i know about the side affects but did not see anything about mmy period being late or missing altogether. I am 6 days late and have done a pregnancy test and it came back negative. could this be because of the antibiotic Doctor: Hi,Flucloxacillin is an antibiotic commonly prescribed to treat infections caused by susceptible bacteria like skin and wound infections. Its common side effects include nausea, vomiting, diarrhea, bloating and indigestion. It is not known to cause delayed menstrual period. Delayed period could be due to infection, stress, change of job or place, or hormonal disorders.Hope I have answered your question. Let me know if I can assist you further. Regards, Dr. Mohammed Taher Ali, General & Family Physician"
},
{
"id": 38774,
"tgt": "Can infection fro handling pigs cause haemophila parainfulenza?",
"src": "Patient: Hi, my husband has just been diagnosed with haemophilas parainfluenzae. He also suffers low white cells of 3.01 being his recent reading. He works in a pig factory and is in contact with pigs blood on a daily basis, is it possible that he got this from the pigs? Doctor: Hello,It is unlikely that the haemophilus parainfluenza infection your husband has is directly related to his job at the pig factory. This bacteria often lives in our bodies without causing infection, but can cause pneumonia, upper respiratory infections and even heart infections. You mentioned that your husband's white blood cell count is low, this bacteria is thought to be especially active when the general immune system is compromised so it could be something else led him to get sick with this. There is also a chance that the haemophilus parainfluenzae was reported on a sample but is not causing his underlying symptoms, since as I mentioned it is a normal component of some parts of our bodies. I hope I answered your query, please let me know if I can be of further assistance,Dr. Patel"
},
{
"id": 218669,
"tgt": "Is TSH level of 3.58 safe during the sixth month of pregnancy?",
"src": "Patient: HI I HAD MY THYROID TEST DURING 6TH MONTH OF PREGENENCY I.E AT 24 WEEKS AND tsh VALUE IS 3.58 I AM USING THYROXINE ALREADY FOM PAST 8 MONTHS AS I AM DETECTED WITH LIL THYROID BEFORE I GOT PREGNENT BUT MY DOC SAID NOT TO WORRY ITSONLY BIT MORE THAN BORDER RANGE.aM I AND MY BABY SAFE WITH 3.58 TSH VALUE DURING TH MONTH OF PREGNENCY? Doctor: Hello,The goal for ultra TSH in pregnancy changes compared otherwise. So at twenty-four weeks, we expect ultra TSH to be less than three. In your case, it is 3.58 which minimally raised. In such cases, I advise patients to do an anti-TPO antibody\u00a0blood test. Accordingly, decide the dose of\u00a0thyronorm. Visit your physician he may increase your thyronorm dosage only for weekend dose.Hope I have answered your query. Let me know if I can assist you further.Regards,\u00a0Dr. Tejashree Bhor"
},
{
"id": 64483,
"tgt": "What causes swollen lumps on crease of elbow?",
"src": "Patient: I was sealing my deck yesterday which also meant getting weeds out of the way. Last night I noticed a red line maybe a line of blood about 1 1/2 inches long in the crease of my elbow. Right now it is red, 2 swollen lumps on each side and as long as the line and about a quarter inch wide and a little painful. Maybe a bug bite? or a scratch from the wooden deck? or something from a plant? I live in Homer, Alaska Doctor: Hi,Dear,Good Morning.Thanks for the query to my HCM online- clinic .I studied your query in-depth.-In my opinion the line with 2 lumps is mostly due to scratch from the Wooden deck. with the start and end lumps- due to high pressure trauma on the elbow crease.b-The possibility of bedbug bite needs to be ruled with the ER doctor, but its chances are remote.-Hope this would resolve your worry-some query.Wellcome again to HCM .Have a good day."
},
{
"id": 83013,
"tgt": "Diagnosed with lupus. Safe to go ahead with tummy tuck surgery?",
"src": "Patient: My daughter in law was planning on a tummy tuck in the next couple of months, but was just diagnosed with Lupus. Would it now be unwise to go ahead with the surgery? She's scheduled for a consultation with the plastic surgeon in two weeks, but will cancel the appointment if this is something that she now needs to forget about. She hasn't had a chance to speak with her primary care doctor yet, because she was notified via email with the results of her blood work. Thank you very much Doctor: Hi ! Well , presently she needs to be evaluated for the disease activity by doing some routine tests like CRP, ESR , Urine R/M , for proteins , RFT etc. . If they are raised that means disease activity is going on . plus there is a concern about the medications she will be on by your doctor . if she is on immunosupressive therapy that needs consideration . so , in nutshell she needs detailed evaluation for lupus before undertaking any kind of procedure because that may lead to disease flare. Thanx. Take care."
},
{
"id": 213533,
"tgt": "ADHD, depression, get dreams about a fight or conflict, have ended up hitting my partner. Reason for this strange behavior?",
"src": "Patient: Hi, Every so often I dream about a fight or a conflict and I end up enraged and I have hit my gf while sleeping. I only started last year and it s happened three times. Last night I elbowed her in her head, as if I m acting out my dream. She was hurt and upset. We went bak to sleep and a little later on I woke up on top of her grabbing her. What the he ll is wrong with me? I suffer from depression and ADHD . Although I have suffered with depression for 3 years and I ve been diagnosed with ADHD for 12 years. Don t know if this is anything to do with it but it is worrying me. Because it is strange behaviour. Doctor: Hi... Thank you for choosing HCM... Kindly visit a psychiatrist... He can guide you with specific behavioural modifications to avoid this behaviour in future... Thank u..."
},
{
"id": 209509,
"tgt": "What causes depression and memory loss?",
"src": "Patient: hy i am zeeshan from pakistan, my brother who is 25 are suffring a brian problems, he is very hopeless, lack of understanding, foreget the past activities very soon, sleepless,disopointment but nothing happend, sorrowfullness, we checkup different doctors but nothing avail , pleaze can u tell what is it? Doctor: Hi Zeeshan,I read your query and do appreciate your concern for your brother. The symptoms that you have mentioned suggest that your brother seems to be suffering from depressive disorder. His hopelessness, sleeplessness, low mood, etc., can all be symptoms of depression.I think you should seek a psychiatric consultation for your brother. Treatment with anti-depressants like escitalopram or sertraline would be effective in his case. These are prescription medications and you would need to follow up regularly for evaluation of the improvement and adjustment of dosage accordingly.Hope this information was helpful. Best wishes."
},
{
"id": 171425,
"tgt": "How do we handle a child who has been living with drug addicted mother?",
"src": "Patient: hi my name is nancy and i just got custody of my great granddaughter who is 4 i am 68. her mother is in prison for drugs when she was a baby she had to go though withdraw of drugs. to make matters worst my granddaughters mother had her but she died last year at age 39 of a drug overdose. my great granddaughter was in her hands till she did this. how do you handle a child who has been though all this. i am at my wits end she has loud out burst and cries all the time. Doctor: DearWelcome to HCMWe understand your concernsI went through your details. This is really great. You just got custody of your great granddaughter who is 4 and you are 68. I envy your luck. Handling 4 year old child is not easy, but I can assure you, the child could never have acquired any of the drug dependency syndrome behavior patterns she had seen. Because she is too young to even notice that. Just give her love, affection and security. Naturally, within a span of one or two months, she will be your child.If you require more of my help in this aspect, please use this URL. http://goo.gl/aYW2pR. Make sure that you include every minute details possible. Hope this answers your query. Available for further clarifications.Good luck. Take care."
},
{
"id": 182989,
"tgt": "Numbness in the teeth.Cause?",
"src": "Patient: When I bite down on my tooth it feels like it's numb, but I also get this tender pain. It kind of feels like how it would feel if a dentist froze it. I'm just wondering what may be wrong with my tooth? This has been going on for a few weeks now, but I do not have the money to have it taken care of. Is my issue serious? Doctor: Thanks for your query, I have gone through your query.The numbness in the teeth can be because of the nerve injury(neuropraxia) secondary to the pressure on the nerve by the impacted wisdom tooth or any cyst or benign tumor. Consult a oral physician and get a radiograph done to rule out these pathologies, later based on the cause we can treat accordingly.If I am your treating doctor, I would have suggested you to take capsule neurobion(multi vitamin supplement) which helps in nerve regeneration. Regarding your financial issue, you can go to a educational institute which is attached with a dental hospital, where you can get the treatment and radiographs for free or at a nominal price. I hope my answer will help you, take care."
},
{
"id": 187288,
"tgt": "What is a large pea sized lump on jaw bone inside mouth?",
"src": "Patient: I have a large pea sized lump on my jaw bone inside my mouth. Swollen, gums and my gum is white. It doesn't hurt unless I press on it. Not sure what to do. My dentist can't see me for a month and I do have a doc apt on Friday, but I figured I ask before hand Doctor: Hello and welcome.Thanks for sharing your concern.Please provide me details about the lesion to answer better in this regard.It appears to be an infection of the gum.Do you have bleeding gums?when did you last noticed it?Are you taking any medications ?Awaiting response.Thanks.Take care."
},
{
"id": 157348,
"tgt": "Am relapsed DLBCL patient. What are the other suitable chemotherapy regimens?",
"src": "Patient: Iam a relapsed dlbcl patient relapsed after autologous transplant done in Aug-2011. I have taken R_CHOP 6 cycles since Aug-2009,6 cycles of R_MINE after 1st relapse in Aug 2010.Now what are the other suitable Chemo regimens available for treatment .If radioimmunotherapy with Zevalin would be suitable for me. I am a humanmale aging 49 yrs. Doctor: there are many chemotherapy regimens for relapsed DLBCL like RICE, GEM OX, DHAP etc. Choice of regimen is decided by the treating doctor. Radioimmunotherapy is not done widely. it is expensive and it has got its own side effects. Since it has relapsed very soon, complete cure cannot be achieved now and whatever regimen if followed, the aim is to prolong survival and offer good quality of life"
},
{
"id": 59846,
"tgt": "Ultrasound shows collapse of gallbladder stone, fatty liver. Curable?",
"src": "Patient: My Dad is 61 year old . He has diabetes but not on insulin . Due to pain in abdomen and fullness , he took ultrasound. The ultrasound report states that the Gallstone bladder has been collapsed and Liver shows mild fatty change. Would you please suggest whether it is curable or how serious it is. And do we need to go for surgery. Doctor: Hi Dear thanks for your question. Your father 61 year old Diabetic is having Cholilithesis that is gall bladder stone and fatty liver.Relax there is nothing serious,both condition are treat able.For mild fatty liver changes,he has to do nothing except to decrease his fat intake and go for exercise .For Cholithesis consult some surgeon and get his gallbladder removed,he will be symptoms free. Hope i answered your question."
},
{
"id": 73894,
"tgt": "How to treat my breathing problem?",
"src": "Patient: Hi, may I answer your health queries right now ? Please type your query here...my absolute esinofils cont is 740.i have problem in my chest . i can't breathj beasily. i am taking medicines from last 5 months but no gain. so please suggest me some nedicines Doctor: Thanks for your question on Healthcare Magic.I can understand your concern.Breathing difficulty, high eosinophil etc are suggestive of asthma more.So better to consult pulmonologist and get done clinical examination of respiratory system and PFT (Pulmonary Function Test).PFT will not only diagnose asthma but it will also tell you about severity of the disease.You are may need inhaled bronchodilators (formoterol or salmeterol) and inhaled corticosteroid (ICS) (budesonide or fluticasone).Oral combination of antihistamine (levocetrizine or fexofenadine) and anti allergic (montelukast) once a day at night is also beneficial.Don't worry, you will be alright.Hope I have solved your query. I will be happy to help you further. Wish you good health. Thanks."
},
{
"id": 209135,
"tgt": "Suggest treatment for sudden behaviour change and gloomy in nature",
"src": "Patient: sir this is for my wife. she is 40yrs old and working as a teacher.so far she was proper in all sense. but of late her behavior has changed totally. last year her mother expired and her father got married again. nowadays she is always gloomy and is not able to concentrate on her profession properly, whereas she was awarded continuously 4 times as best teacher in maths earlier.even at home she is gloomy and ready to cry at any point of time and she seems to be feeling insecured as if.for everything she is looking for help and undecisive. what should I do and what sort of problem is this. Doctor: Hi, thanks for asking. In my opinion your wife seems to be suffering from depression. I would suggest following things for your wife. 1) Get her thyroid levels checked. Hypothyroidism may lead to depression.2) consult a psychologist and take counselling sessions to combat depression.3) you may also consult a psychiatrist for she might also need medications also."
},
{
"id": 152002,
"tgt": "hand tremors",
"src": "Patient: hi i am 33 male i have tremors in my hands appears specially when holding a full cup and cannot hold things properly i want to ask what is the origin of that tremors and if it related to any other disease? thanks Doctor: THanks for the query Looks like u r sffering from involuntary movements. It could come in people who are chronic alcoholics but most often than not it is idipathic and u can take tab betacap tr in the night to treat it. But dont take the pills without going to a neurologist Have a healthy life"
},
{
"id": 76908,
"tgt": "What causes chest pains with black outs and dizziness?",
"src": "Patient: So lately I've been having some chest pains and I've fainted a couple times. I black out and get dizzy usually when I stand up. I can't see and I get all numb and dizzy. I've fallen over a couple times. I've also been having back and joint pains, and my ribs are very weird. One of them juts straight out, but the other ones kind of sideways. I don't know if that has anything to do with anything though. Im a 14 year old girl. Please help. I've already been to a dr., but they hadn't really said anything was wrong. I still think somethings wrong though. Doctor: Thanks for your question on Healthcare Magic. I can understand your concern. At your age of 14 years, heart diseases are very unlikely. But your all symptoms like chest pain, blackouts, dizziness etc are commonly seen with heart diseases like valvular heart diseases. So first get done ecg and 2d echo. If both these are normal then no need to worry for heart diseases. Sometimes, stress and depression can also cause similar symptoms. So better to consult psychiatrist and get done counselling sessions. Try to identify stressor in your life and start working on it's solution. You may need anxiolytic drugs too. Don't worry, you will be alright. Avoid stress and tension, be relax and calm. Hope I have solved your query. I will be happy to help you further. Wish you good health. Thanks."
},
{
"id": 193433,
"tgt": "Is 100% spermatozoa a matter of concern based on the report?",
"src": "Patient: Dear Docs,Please see below my semen report and kindly let us know if it's OK? Let me know if there any issues with it and if possible to cure them?Interval between start of ejaculation and analysis (before liquefaction) : 10Appearance: Creamy WhiteLiquefaction Time(min): 30 minConsistency: ThinVolume 2.0mlpH: 8.0SPERM MOTILITY (%) (100 Spermatozoa)a) Rapid Progression: 30%c) Non-Progressive Motility: 10%b) Slow Progression:10%d) Immotile:50%Agglutination: -Vitality: -MORPHOLOGY (%)Count / ml: 88 MillionNormal: 70%Abnormal: 30%Head Defects: 10%Tail Defects: 10%Neck & Mid Piece Defects: 10%Cytoplasmic Droplets: NilHeadless 'Pinhead': NilPus Cells : 2-3Red Blood Cells : 0-1Epithelial Cells : NilMiscelleneous : Nil Doctor: Hello,Your semen analysis is almost normal according to WHO guidelines except for motility. 50% is non-motile. I can get a direct check-up with an andrologist.Hope I have answered your query. Let me know if I can assist you further.Regards,Dr. B. Radhakrishnan. Nair"
},
{
"id": 163433,
"tgt": "What causes a decrease in the appetite?",
"src": "Patient: hi, im feeding my baby (24 weeks) milk and solids but for the past 3/4 days his appitite has decreased massively, today he has had 4oz s @ 8am, half a jar of apple and cranberry breakfast @ 11am and only 2oz s at 13:30, normally he has between 5/7oz s and a whole jar in the morning and one in the evening, is this normal and should i be worried? thanks for your help Doctor: Hello,This feeding habit for last 3 to 4 days is also normal. Baby has fluctuations on feeding habit during infancy.If it persists for longer time or you noticed decrease in weight, activity, regression of milestone or any illness following it, then needs to consult doctor.Hope I have answered your query. Let me know if I can assist you further.Regards,Dr. Sachin Kumar Agarwal"
},
{
"id": 63399,
"tgt": "What are the two big lumps growing in my abdomen?",
"src": "Patient: I was diagnosed to have had a hiatal hernia about 3 years ago, and now there;s two big lumps growing in my abdomen and are causing me discomfort can a ultrascan detect the hiatal hernia, only that I was told by my GP that a scan can never pick it,is this true Doctor: HHi,Dear thanks for the query to HCM Virtual Clinic.I studied your query in all the details given by you.Two big lumps in your abdomen are very difficult to comment without more information as -What's its location?what's size, tender,reducible,soft or hard and so on.Regarding your query for the USG-Yes USG can detect the hiatal hernia.What your doctor might be referring USG in diagnosis of two big lumps.So I would advise you to consult your doctor,ER Surgeon,who would help you out.Hope this would help you to act fast and plan treatment with your doctor.For the questionable lump at the site,I would advise you to check with Your ER surgeon..Hope this reply would satisfy your query.Write excellent reviews if you like this reply and find it helpful to you,as your comments and reviews would improve my rating for the visitors of the HCM clinic.Welcome for any further query in this regard to Me.I would love you to help out of this intricate problem of lump.Wishing you fast recovery.Have a good day.Dr.Savaskar M.N.Senior Surgical SpecialistM.S.Genl-CVTS"
},
{
"id": 132888,
"tgt": "Need treatment for hip and leg pain after falling down",
"src": "Patient: Hi, While I was playing football I landed heavily on my left hip. I cannot lie on that side (happened yesterday) but have been ok pretty much all day (just pain). This evening I turned awkwardly and it felt like I was being stabbed in the left side of the hip which is and always has been very tender. Now when I walk when my leg reaches a certain position it feels very painful. The muscle is very very tender in a specific point. I work in theatres as a charge nurse and so have had football injuries before and so am reluctant to go in for an x-ray as my colleagues have told me to stop playing ever since a knee injury. What I really want to know I suppose is what is the likelihood that this is muscular? Will It do any harm leaving it a while. I m guessing if it is a fracture it would be acetabular as opposed to femur as the pain has not been severe when walking? Thanks Doctor: Hello, I have studied your case history.Traumatic hit injury can lead ligament or muscle injury.If there is fracture in your leg bone then it is difficult to walk on itYou will need X ray of knee and hip.Clinical examination is important to rule out ligament injuryFor these symptoms analgesic like ibuprofen and chymotrypsin medication can be started. Do ice fomentation and rest to joint.Hope this answers your query. If you have additional questions or follow up queries then please do not hesitate in writing to us. I will be happy to answer your queries. Wishing you good health.Take care."
},
{
"id": 161469,
"tgt": "Suggest treatment for cold urticaria with rashes",
"src": "Patient: Our 8 year old daughter was diagnosed with cold urticaria 2 years ago. It reached -10 today and her rashed appeared again. We gave her Benadryl and within 1/2 hr it improves. Should we let her outside when she is at school. Also, what are the chances of her out growing this? Thank you Doctor: Hello, It may be due to dermatitis. You may apply hydrocortisone ointment. Can give syp cetirizine after pediatrician consultation. Watch for symptoms like breathing difficulty, swelling, fever. In case of fever, you can give syp acetaminophen. Till then maintain proper hygiene and hydration. Avoid allergens. Hope I have answered your query. Let me know if I can assist you further. Regards, Dr. Shyam B. Kale, General & Family Physician"
},
{
"id": 173186,
"tgt": "Is Sporlac advisable for loose motions in children?",
"src": "Patient: My daughter was suffering from loose motions, fever and vomiting since past 2 days. Doctor advised Rinifol z, calpol and ofm. The fever and vomiting have stopped but loose motions are severe. Please advise whether i can give her sporlac that i have in sachets with water Doctor: thanks for asking question ,Your daughter suffered from loose motions,fever and vomiting , in which fever and vomiting stopped, Generally loose motions took longer time to control then fever and vomiting,you asked to add sporlac in above mention treatment.The Rinifol z contains the drug which is sporlac has. so there is no advantage to add sporlac in above mentioned treatment. loose will decrease in one or two days . no thing to worry. give curd if daughter age is more then 6 months . whishes healthy family thanks"
},
{
"id": 146908,
"tgt": "Suggest treatment for numbness in arms and hands",
"src": "Patient: I have anxiety, Im told, but when I told my psych dr that the numbness starts in my fingertips(both hands), goes into my hands and up my arms and is also in my lips and mouth and further during an attack. The meds Im on are not working all the time and the numbness can stay in my fingers and lips for days or longer. My psych told me to pretend this doesnt happen. Its all in my head. It is not. Im not a crazy person and this is all new to me since September. Doctor: Hello dear,The symptoms as mentioned in your post can be attributed to:1. Para-spinal spasm and entrapment of nerve fibers at the Spinal region.2. Inadequate hydration status & electrolyte imbalance of the body3. Dietary deficiencies of certain nutrients like Iron, Calcium, Vitamin D, Vitamin B12, Folic acid, etc. which help in blood formation & nerve conduction.4. Peripheral neuropathy caused due to any fluctuation in blood pressure or blood sugar level.Symptomatic relief can be obtained by intake of nervigenic agents (like Vitamin B6, Vitamin B12, Folic acid) as well as multivitamin & multimineral supplements including iron, calcium & Vitamin D needs to be added in your diet.Physiotherapy & postural exercise will also be helpful.If symptoms still persist, kindly consult a Neurologist & get a complete Neurological examination done.Certain investigations like complete blood count, serum electrolyte estimation, fresh estimation of blood pressure & blood sugar levels & nerve conduction studies will be helpful to rule out any pathological cause for the symptoms.X ray & MRI of spine can also be carried out to detect any nerve entrapment.Wishing you a Good Health.Take care."
},
{
"id": 5534,
"tgt": "Planning for a baby. Irregular period, brown discharge. Taken meftal spas. Help?",
"src": "Patient: Hi doc, we are married for 2 years now and now planning a baby. my periods were due around the 4th of march and did not happen till date jan. but i had been facing been in the lower belly for many times in these days. In fact i checked for the pregnancy test but it was negative. but yesterday evening the periods started which was much lesser than regular. i took meftal spas for relieving myself from pain. it was later followed by a strange material outflow for once. this was a mix of thick brown blood and some white substance...kind of the layering substance which i couldn t understand. kindly suggest. Doctor: Hi, Thanks for query. As you say your period are not regular,chances of pregnancy are also uncertain because your ovulation time is not fixed. Pregnancy test sometime be false negative,but may be you are pregnant. May be meftal spas cause contraction and may be that white material is early sign of pregnancy. but don't worry,whatever happened is happen,go to some gynae,get USG done. Took treatment for your irregular periods Everything will be fine. good luck take care"
},
{
"id": 173557,
"tgt": "What causes swelling on the eyelid of my 10 month baby?",
"src": "Patient: Hi, my 10 month old baby fell last night and he had a slight bump on the front right side of his forehead. He cried for a bit and then calmed down and continued playing and crawling around. This morning when he woke up the bump was gone but now his right eyelid is swollen and he can't open it up all the way because of the swelling. I called my Dr but can't get in to see him...should I be worried? Other then the swollen eye he seems normal, crawling and playing with his older sibling. Doctor: Hi,Thanks and welcome to healthcare magic.Injury to the forehead causing a bump usually leads to swelling of they're lids.Sometimes the flour may also change to bluish black and is known as black eye.It will disappear in due course of time .If the boy is active,no vomiting or fits within 48 hours of injury you need not worry.Hope this answer serves your purpose.Please feel free to ask fruit queries if any.Dr.M.V.Subrahmanyam."
},
{
"id": 159477,
"tgt": "Sore throat, headache and cough. Suffer from anxiety and depression. Do I have cancer?",
"src": "Patient: About 3 weeks ago I came down with the flu sore throat runny nose achy headache once these symptoms cleared I had a cough was coughing greenish phlegm flu cleared was checked by doc but I still have a sore throat it is only on one side and it only hurts to swallow and my ear hurts when I swallow also it doesn t feel like a typical sore throat now. Please do not answer that I may have cancer as I am a very anxious person and suffer from major depression and anxiety and I am very worried that it is throat cancer Doctor: Hello Melembru, I read Your Problem. Do not Be Penic. Believe me You are not Suffering from Throat Cancer. Throw away your worries. Be Relax and calm. You Probably got Upper Respiratory Tract Infection after Common Cold episode. You need to do is to follow few Instructions. 1) Avoid Cold Stuffs like Cold drinks, Ice creams etc. 2) Avoid Oily foods having more cheeze,butter& all. 3) Drink HOT breverage like Hot Coffee,Hot Tea, Hot Soup 3 times a day 4) Do Gargling with salt water 2-3 times a day as local remedy. You can Purchase Throat Gargles by Medical store with Your Doc Guidance. 5) Consult Your Doc again to treat Ur throat Infection & Ask him for Proper Antibiotics. Ask me again if any queries. With regards, DR JIGNESH"
},
{
"id": 178055,
"tgt": "What causes chronic cough in a child?",
"src": "Patient: My grandson is 12 years old and has asthma. Last year he missed 30 days of school because of cough. He was only in school one week and began coughing Friday and is still coughing today and will miss school again tomorrow. Friday the doctor said that it was a virus and gave him an antibiotic, cough syrup and a steroid nose spray. Yesterday, his regular doctor said that he has an upper respiratory infection and gave him prednisone and lidocane for his throat. He says that he has a very sensitive nerve in his throat that triggers the cough. He says another child would probably not even have this cough. There have been many times that this doctor has said that he has a tic. He went all summer out of school and only had a cough onetime and it only lasted 3 days. His regular doctor did send him to a neurologist one time but, nothing ever came of it. We are at our wits end. Please help. Doctor: Probably he is asthamatic and many drugs have been given including antibiotics,steroids bronchodialotrs ,probably this has caused reflux oesophagits which is causing him irratable cough .please give antacids before giving food and medicines"
},
{
"id": 106284,
"tgt": "Can penicillin be transfered to anyone through intercourse ?",
"src": "Patient: I am allergic to pennicillin. My boyfriend is taking it. Can it be transfered to me through is semen causing me to have an allergic reaction? Doctor: NO"
},
{
"id": 204051,
"tgt": "How can I stop the habit of masturbation as the thoughts are disturbing me a lot?",
"src": "Patient: hello sir i am suffering from sexual thought these thought are disturbting me lot i regulary masturbate.i want to do stop these? even i am not interset in praying a god .why these hapennig with me i dont no?please help sir please. Doctor: Mastrubation is very normal unless it affects your regular activities.During teenage most of the boys masturbate as puberty hits them. Its natural to have sexual fantasies and then masturbate. If your masturbation is effecting your daily activities and its hindering your work then you should consider changing your habits. First of all you need to stop watching sexually stimulating videos and pictures.You should find some new hobby, something of your interest as this will take your mind of masturbation. Keep yourself busy. This will help you take your mind of this and sexual fantasies. I hope this works for you"
},
{
"id": 161989,
"tgt": "What causes breathlessness with stuffy nose and red dots on chest?",
"src": "Patient: Hi. My granddaughter is 4 1/2 mos old. She has a very stuffy nose and when she starts to fall asleep she cannot breath. She sounds like she is gasping. After sitting up for a while she seems a little better. We just noticed small tiny red dots on her chest, abdomen, arms and legs. Not too many. Doctor: Hi, By what you say I feel that she might be having a lower respiratory tract infection which could be either viral or bacterial. At this age, it could be acute bronchiolitis and she will require medical attention or oxygen therapy also. I suggest you take her to the nearest Emergency Room as soon as possible. Hope I have answered your query. Let me know if I can assist you further. Regards, Dr. Sumanth Amperayani, Pediatrician, Pulmonology"
},
{
"id": 3322,
"tgt": "What are my possibilities of conceiving through IVF?",
"src": "Patient: Im 30 yrs old married since last 10 yrs,in 2004 bfor marrage i suffer with tuboovarian for which i had an opration. i took AKT-3, for six mnth. Later after 5 year i oprated for ectopic pregnancy, now im trying to conceive and doing treatment for the same, two cycles of follicular shows follicals became too big n hemmorage yet not ruptured.is it any possiblity to coceive throught ivf. Doctor: hi..in IVF the egg formation is stimulated and eggs are collected and placed in media and then fertilised with sperms outside the body and fertilised eggs are implanted in uterus and tubes are bypassed .For this to happen,ovaries have to be properly functioning and the uterine lining forms the bed for embryo to growa normal menstrual cycle partly suggests that ovaries are good, your age seems to be OK for good results,hormone levels: FSH,LH,ESTRADIOL, AMH would give us the ovarian functioning,hence after the work up,your chances of conception could be tentatively explained"
},
{
"id": 15727,
"tgt": "Reddish spots in the pubic area. On antibiotics. Spots turning black. Why?",
"src": "Patient: I have went to the doctor to check about these reds spots that I have in my pubic area. He said he thinks it's just fliclitous which is just an infection. He gave me some antibotics for 20 days twice say. But it seems to me they are not going away. They do not hurt or itch. Nor does it hurt when I pee. But it seems to me I think two of them are turning a darker color. I am not for sure what it is. But my doctor seems to think it is what he told me. But I have a check up next Tuesday to see. Doctor: Hello,Thanks for the query.There are various conditions which can present as red patches in pubic area.Mostly it is fungal in nature,Other conditions like angiokeratomas should also be considered.Please meet a dermatologist for exact diagnosis. Let me know if you have any other doubt.you can ask a direct question to me on this forum, following the below link.https://urldefense.com/v3/__http://www.healthcaremagic.com/doctors/dr-rahul-kumar/64818Thank__;!!Mih3wA!SBzm6_kI6hCZ58EPH6N_05MFfiPbxWXT0a2TJCdFQObRWm5mV5ur7hW2pl4Ymw$ you"
},
{
"id": 185811,
"tgt": "What could extra skin/ tissue between teeth could be?",
"src": "Patient: My daughter is almost 4 and last night I thought she had some food in between her teeth when I tried to remove it I noticed it was like a skin between her teeth which is easy to see. I would like to know what causes something like that. It does not hurt her but very visable. Doctor: Hello:)Welcome to HCM.Your kid seems to be suffering a gingival inflammation. Gingiva is more also more visible during the falling of deciduous teeth.I suggest you for a checkup with your dentist to rule out any infection.Gargle after every meal and brush twice daily.Regards."
},
{
"id": 189013,
"tgt": "Did filling in most of the teeth. Drilled & filled the upper back. Now roof of the mouth hurting, sore. Recommendations?",
"src": "Patient: Had all of my front teeth and upper right back teeth filled. Doc said upper back was in root. He said he was medicating it. And should be fine. So he drilled and filled. Now the roof of my mouth hurts. After I take Motrin it stops hurting all together. But say like when I first wake up. It hurt not throbbing but super sore Doctor: Hi,Thanks for asking the query,According to your clinical symptoms i suspect that you have developed infection in one of your tooth that has drained its abscess in the palatal area leading to pain. A thorough clinical examination will be helpful. Take an x-ray of the affected region.Root canal treatment has to be performed.Antibiotics and analgesic will give temporary relief treatment of the affected tooth is required.Take care!"
},
{
"id": 209808,
"tgt": "Suggest treatment for perception problem and inability to focus eyes",
"src": "Patient: I have a very serious problem about perception. I cannot focus my eyes for long on the face of whoever I am talking to. I always feel very concious of where my eyes focus but i really just can t control it. Years ago I had a public breakdown wherein I just let all my feelings out, I was angry because I am suspected of being a thief. My officemate superior made hints, remarks although not directly so as excuse himself I m sure that it made me uneasy, annoyed and depressed. I just go along usually by engaging on his insensitive remarks. By then, it became constant and he always try to trap me and then observe my reaction. As a result I became paranoid. He cannot target others because they are like him and I am only a civilian employee. Prior to my breakdown I was diagnosed with hormonal imbalance due to goiter. I feel when I talk to another as I try to avoid it so that I will not be suspected, my eyes still wander on material things like cellphone. I now have developed fixation about these things, which is due to overly being sensitive to gossiping and observation by others. My eyes are doing the opposite, my brain says don t look. My other officemates know that I didn t do anything but doubt is always there given the fact of what they have observed about me. Am I suffering my anxiety? With OCD? paranoia? I was no like this before... help me pls.. I just want to die Doctor: DearWe understand your concernsI went through your details. I suggest you not to worry much. I sincerely think you are overly worried about this. A sort of obsession for the time being. But I thin OCD cannot be the problem. You are also not paranoid. Anxiety, of course, is part of obsession. Everyone are obsessed with something somehow. Some people check switches again and again. Some make sure that their looks are perfect. Some sees that their home is in order. etc. Such obsessions are of positive nature. In your case, your obsession can be treated as positive unless it is not interfering with your work or life severely. If it affects your work or normal life Please post a direct question to me in this website. Make sure that you include every minute details possible. I shall prescribe some psychotherapy techniques which should help you cure your condition further.Hope this answers your query. Available for further clarifications.Good luck."
},
{
"id": 145904,
"tgt": "What causes blackout while smoking a cigarette?",
"src": "Patient: A while back, I was smoking a cigarette around 4 AM and all of a sudden my sight started blacking out and I told my boyfriend, at the time, that I felt like I was about to pass out. Right after that, I hit the ground and as soon as I woke up I didn t remember where I was. When I asked my boyfriend what happened he said I passed out and it looked like I was having a seizure? Is this reaction a seizure or? I ve also been but by my dog and it made me pass out on my kitchen floor. My mom told me I was slapping myself in the face and she was taking me to the doctor, while in the car she said I tried unlocking the door and getting out while the car was moving. I have no rememberance of the scene when I was bit by my dog. Why? Doctor: According to me you may be having episodes of seizure as it has occurred 2-3 times and not associated with posture. Your narration is not adequate for making a definite diagnosis but I will suggest for EEG and neuroimaging either CT or Mri brain . wish you good health."
},
{
"id": 14161,
"tgt": "What is the treatment for skin rash?",
"src": "Patient: i have this rash around my knuckles that burns when something touches it. i have had it for three days now. water lotion and touch makes it burn more. now it is starting to itch. what could it be? i may have touched some mold when i was painting in a house. do you think that could cause the rash Doctor: Hi,It may be insect bite dermatitis. Kindly consult the dermatologist for the perfect diagnosis and proper treatment.I would suggest.... - oral antibiotic for one week- oral steroid in tapering dose- antihistaminics - apply mild steroid with antibiotic cream Take rest. Avoid soap bath. It may recover in a week or two.I hope this would help you.Thanks. Dr. Ilyas Patel MD"
},
{
"id": 195405,
"tgt": "How can small bumps on the penis head be treated?",
"src": "Patient: Hey last summer there was bleach in the toilet for cleaning. Then I was peeing didn\u2019t realise the bleach after few hours all itchy lots of pimples now the pimples are very tiny little black some skin shredding off don\u2019t look healthy to me the tip is smooth just tiny spots Doctor: Hello and Welcome to \u2018Ask A Doctor\u2019 service. I have reviewed your query and here is my advice. If you're penis touched bleach then you may developed above symptoms. It may be due allergic reaction of skin to bleach. you may use moisturizer cream or local steroid cream to recover. Do not worry it will subside it's own. If symptoms not improved please consult your doctor he will examine and treat you accordingly. Still queries, please feel free to ask. Take care"
},
{
"id": 8618,
"tgt": "Reddish sore spot on the face. Pus formed. Can I use benozyl peroxide?",
"src": "Patient: Hi, sorry if I am doing this wrong but I am new to this site. I had a large spot on my face which I messed around with, I know I shouldn't have, and now it has become really red, sore and there has been puss coming out of it for a couple of hours. I have covered it with a band aid to avoid messing around with it any further. I have work in the morning- what should I do to not make this situation any worse? I have Benozyl Peroxide cream if this helps. Doctor: Hi there ~I understand your concerns. You are probably in the right place if you are seeking help in the form of advise. I believe that what you had was a follicle which you did not treat correctly. I think that you should also need to see a doctor that is giving you systemic medications to help prevent the reoccurance of any of acneform lesions. I hope this helps. Take care and have a lovely day!"
},
{
"id": 135497,
"tgt": "What causes sore arms and legs with fatigue and light sensitivity?",
"src": "Patient: i ve got sores on my legs arms head. they are red ,round and when heals leaves scarring. i have taken tests to determine autoimmune diseases, but keep coming back negative. i am inflammed in arms legs hands. extremely fatigued, sensitive to light ,achy joints. sores in nose. also the sores when healing are leave a deep whole in the skin. thank you for your help. Doctor: You could be having Porphyria and light sensitivity due to that. Also SLE and Sjogren syndrome need to be ruled out. You may need a course of steroids for your therapy. Kindly consult with a Rheumatologist."
},
{
"id": 149137,
"tgt": "Diagnosed with chiari malformation, syringomyelia. Complaining of hotness in body, getting goosebumps. Also has pot syndrome. Suggestions?",
"src": "Patient: Hi my daughter has chiari malformation and syringomyelia with constant low pressure headaches that her neurosurgeon is trying to figure out now. She just recently had her shunt changed because it was flowing too fast but low pressure headaches are still occurring. For two days now, she is complaining of her body getting the feeling of hot starting in her hands and moving up her arms then neck and the. Down her body. All the while she gets \"goosebumps\" but doesnt feel cold. Any ideas or thoughts on this? They also have told us she has pot syndrome. But these last two days of checking her BP and HR, she has been regular numbers. Doctor: HIThank for asking to HCMI carefully gone through the history of your daughter, the over all clinical symptoms are very likely in such spinal cord disease, which may not be worry some at this stage I think expert neurosurgeon manages such case in very nice way, I would advise you to stick with your neurosurgeon, every thing will be alright your another concern is pot syndrome this also is part of spinal disease and the treating doctor will definitely take care of it, take care of your child, I would be happy to help you for any further query, bye"
},
{
"id": 39359,
"tgt": "What is the cause for diarrhea, vomiting and headaches?",
"src": "Patient: Hi I have been having weird symptoms for the last 3 days shakiness headaches jittery nausea and vomiting. I feel like I have brain fog I feel very lethargic and when I eat I feel ok for 15 mins then back feeling worse I had some sugar then half hour later I'm sick again I'm very worried I also have diarrhoea Doctor: hello welcome to health care magic, you are suffering from gastroenteritis. with diarrhoea body loses water and minerals causing dehydration. dehydration is the cause of your symptoms. start taking oral rehydration solution and plenty of fluids. if the symptoms persist or you develop fever consult a physician for appropriate medication. hope this info was useful"
},
{
"id": 6842,
"tgt": "What are the chances of pregnancy if I have sex on 11th day of my cycle ?",
"src": "Patient: hello doctors i am asking is that what is the chance to have pregnant when have ONE time sex during 11th day of the cycle? i m 28day cycle , but its sometimes delay by 3 or 4 days. thanks :) Doctor: Hi, In a 28 day cycle the fertile period is usually between 12-16 days.It differs with people.If you arent having any clear,jelly like discharge,you should be safe but the very low chance might be there."
},
{
"id": 7616,
"tgt": "Suffering from pimples, oily skin. How to get rid of the acne marks?",
"src": "Patient: hellooo,doctor m sufferring 4m pimples since i was a student of class 8.But after consulting wth doctor when i was in class 12 i was perfectly cured.but unfortunatly again during m last years of graduation i got them again,which left very bad spots,wat should i do nw,,i hv 2 oily skin,,,,now m 22 yrs old,pursuing m masters. plz reply...thnz Doctor: Hai, Welcome to HCM. Pimple at this age is due to excessive activity of your oil glands of your skin due to bubbling hormonal action. Up to the age of 25, it is an ongoing process. After the hormonal tides start reduced your problem will vanish completely. So the treatment cannot totally eradicate the appearance of pimples whereas it will help to reduce the intense to the maximum extent. Frequent face wash, consuming adequate water, vegetables, fruits, greens and avoiding foods containing butter cheese, ghee, and vanaspathy etc will help you to keep this menace well under control along with specific medical treatment for pimples. Medical treatment has lot of choice according to the severity, your skin and type of lesions from topical preparations, oral antibiotics, soaps etc. Your dermatologist will guide you in this regard for a better treatment of choice to suit your present condition. Following all these measures together you can keep your problem well under your control. Wish you a speedy recovery. Dr SM"
},
{
"id": 107839,
"tgt": "What causes bruised feeling in lower back and abdomen?",
"src": "Patient: Hi, I m 21, I m 5 2 , I weigh 135 lbs, and am physically healthy for the most part. Tonight, I noticed my lower back felt bruised and it hurt to simply adjust myself when sitting. Then, went to bed, I decided to lay on my stomach to avoid the pain, but my stomach also felt bruised and felt a hard rod-like thing at the top of my abdomen. When I touched it, it also hurt. The pain feels familiar to when one does exercise, but I haven t done anything strenuous, or done any abdominal exercises that might cause this pain. I m a little scared because I ve been taking a Chinese almond for weight-loss and I don t know if this might have anything to do with the pain, but even then, I ve only taken it for a few nights and I hadn t taken one in two nights until today. I hope this is clear and hopefully I can get some answers... Doctor: HelloNo part of your history is clear. In fact you first need a psychiatric evaluation. Bruise is not a feeling but the external sign after an internal haemorrhage.You cant get answers to illogical questions, they probably would be made up answer. First express your symptoms clearly in simple words."
},
{
"id": 7739,
"tgt": "How can one get rid of big and painful pimples on the face ?",
"src": "Patient: doctor i am a 24 years girl and i have been troubles by the pimples i got on ma face since 2 years now. i tried ointments..tablets.. parlor.. herbal treatments but of no use. I did not have pimple problem in ma teens also but now all of a sudden.. it is big and painful..and mostly around my mouth. what should i do? Doctor: Hi..dear beenu., Thanks for choosing HCM., PIMPLES...since 2 months.., difficult to contrlo.., Only solution is.., 1) COMEDONE EXTRACTION... by RF., it must be done by..Dermatologist.., 2) Tab..Isotretinoin 20 mg daily for 2 months., 3) Tab.Azithrmycin..250 mg daiy for 1 week., 4) Take plenty of green vegetables and fresh fruits., 5) frequent washes with Triclosan soap., 6) Topical Clindamycin 1% 2 time sdaily ., ok..thanQ"
},
{
"id": 83268,
"tgt": "What is the use of susten 200 and emfolic dha?",
"src": "Patient: I am 30 years old, my height is 5feet 1inch and weight is 53 kgs, my BP - 120/70mmhg. My LMP is 5 december 2011 on 10th january I came to know that I am pregnant by pregnancy kit test done at home. Yesterday I consulted Gynaecologist; she prescribed me Emfolic DHA (od/morning) and susten 200 (od/night)for 1 month. My menstrual cycles were very regular. Is the prescription is correct one. Shall I take susten 200? Doctor: Hi,Susten is a Progestin which is a female hormone. It works by correcting the levels of progesterone, thereby stimulating the lining of the uterus to prepare for a pregnancy and protect an existing pregnancy. This helps in the treatment of infertility and prevents spontaneous abortion (miscarriage). Progesterone is especially helpful in suspected deficiency of the hormone.Take care. Hope I have answered your question. Let me know if I can assist you further. Regards, Dr. Saranya Ramadoss, General & Family Physician"
},
{
"id": 191106,
"tgt": "Exposed jaw bone right lower jaw",
"src": "Patient: after my hip surgery I NOTICED MY RIGHT LOWER JAW WAS SORE I WENT TO MY DENTIST THINKING IT WAS AN ABSESS HE SAID IT WAS EXPOSED BONE PROBABLY DUE TO THE DEVICE THAT WAS USED DURING SURGERY. I SAW HIM AGAIN TODAY AND HE SAID IT WAS GETTING BIGGER AND GAVE ME A MOUTH WASH NUMBING AGENT, HOWEVER THE PAIN IS STILL THERE . CAN THE GUM BE STICHED TOGETHER ? Doctor: hi, is that a lip surgery being mistakenly written as hip surgery? can you please give some more details of the surgery as well as the problem that you are facing..... tc."
},
{
"id": 17362,
"tgt": "How to treat patent foramen ovale?",
"src": "Patient: My 11 week old daughter has been diagnosed with PAC. She was on a holter monitor while in the hospital, just after birth, and again approximately 6 weeks later. The findings were that there had been no decrease in the PAC s. Also, at the 6 week follow-up, during her echocardiogram, PFO was found. She is scheduled to receive another holter tomorrow, and to see an electrophysiologist in 2 weeks. I would like to know, what type of questions should I be asking? Doctor: Hi, The foramen ovale typically closes in 75% of people at birth but residual \"probe patency\" is a common normal variant. In about 25% of people, a patent foramen ovale (PFO) persists into adulthood. PFOs may be associated with atrial septal aneurysms (a redundancy of the interatrial septum), eustachian valves (a remnant of the sinus venosus valve), and Chiari networks (filamentous strands in the right atrium). Defects more than 4 mm in infancy have been noted to become larger in natural history studies. A \u201cbubble study\u201d is done on the echocardiogram to detect the presence of a patent foramen ovale. Hope I have answered your query. Let me know if I can assist you further. Regards, Dr. Tushar Kanti Biswas, Internal Medicine Specialist"
},
{
"id": 126988,
"tgt": "How can scoliosis and osteopenia in the thoracic region be treated?",
"src": "Patient: Hello Doctor, My Wife is suffering from severe pain in thoracic region since 18 months. Pain starts between the shoulder blades and radiates to shoulders also to the chest region. The pain triggering area is hyper sensitive, even a feather touch causes extreme pain. We have consulted several orthopedicians, neurologists, physiotherapists and even psychotherapist. None of the treatments helped. Except for electro therapy, it gave her a little relief that too for a short period. She is diagnosed with mild scoliosis and ostropenia in thoracic region. But all the doctors ruled saying out these two cannot cause the pain she is having. Also nothing found in the MRI of spine. Doctor: Hi, Consult an orthopedician and get evaluated. Hope I have answered your query. Let me know if I can assist you further."
},
{
"id": 108136,
"tgt": "What causes lower back pain and pelvic pressure?",
"src": "Patient: I have severe sharpe pains in my lower back and some pelvic pressure. I don t feel like something is right. I went to the hospital but the nurses just stuck a blood pressure thing on my arm and had me drink water. They didn tcheck anything on me or the baby to be sure, let alone even listen to what problems I have. What do you think? Doctor: DEAR FRIEND, I NEED TO EXAMINE YOU THOROUGHLY, NEED X RAY OR MRI TO CONFORM DIAGNOSIS, SO DO THAT AND SEND ME REPORT AND VISIT PHYSIOTHERAPY CLINIC AND ASK DOCTOR TO GIVE YOU LUMBAR TRACTION AND IFT THERAPY FOR INSTANT PAIN RELIEF... LET HIM TALK TO ME... AND I THINK THEY ARE NOT CONCENTRATION TO DIAGNOSE THE CAUSE OF BACK PAIN... SO BETTER YOU GO TO PHYSIO 9033762190 DR HEMANG KUMAR JANI"
},
{
"id": 161423,
"tgt": "Suggest treatment for vitiligo in a child",
"src": "Patient: my daughter of 8yrs is suffering from vitiligo for last 2 & half years. it was a small 2mm spot on the right knee at the beginning. from there it has spreaded on both her legs, hands and face. she is taking homeopathic treatment from Dr. Batra s Clinic since beginning but no improvement. kindly help. Doctor: Hello, There is no treatment of vitiligo so effective so as to effectively make all the spots disappear. There is no cure for vitiligo. Treatment with UVA & UVB light help to slow down the discoloring process & return some color to your skin. You are suggested to get your child physically examined by a dermatologist. Hope I have answered your query. Let me know if I can assist you further. Take care. All the best. Regards, Dr. Nupur K, General & Family Physician"
},
{
"id": 169553,
"tgt": "What is the treatment for tonsillitis in a 3 year old?",
"src": "Patient: My daughter is 3 years and 6 months old. she has tonsils with the following symptoms. one doctor has adviced surgery and another doctor has asked us to wait till she is 5 years. can you advice. 1. Her tonsils are big and they dont touch each other. 2. she can eat without any problem but her weight is very low. (11Kg) 3. She gets frequent throat infections and cold. we are giving antibiotic alomost once in a month. Doctor: if ur child is getting more frequent infections and frequent antibiotic usage then u need to get the surgery done as it improves the overall output of your child."
},
{
"id": 154133,
"tgt": "Does lumps behind ear indicate cancer?",
"src": "Patient: Dear sir,My mom is suffering from damage of saliva gland. There is two lumps behind her ear at present their is no pain but doctor inform us in future it would be painful and there is no treatement in allopathy.I want to know is it cancer. Is there any treatement in Ayurved please suggest so we give her best treatement. please sir. Doctor: Hello,It is important to make a confirmatory diagnosis of the lumps behind the hear which your mother has. It is likely that those are lymph node enlargement involving the retro auricular group of lymph nodes due to a condition of the parotid gland. If pain is there, and it is present from a few days, then acute infection is suspected.If no pain is there, then suspicion of chronic infection like tuberculosis or tumor that is slow-growing needs to be considered. Please get an ultrasound scan done and an FNAC needle test to confirm the diagnosis. Treatment is following confirmation of diagnosis. Please do not worry.Hope I have answered your query. Let me know if I can assist you further.Regards,Dr. Vivek Chail"
},
{
"id": 200042,
"tgt": "Can Tramadol help to treat pain and stone in penis?",
"src": "Patient: I HAVE A BLADDLE STONE AND I AM IN PAIN WHEN I TRY TO PASS WATER. I HAVD THE MICO TREATED BACK IN MARCH AND MY PROSTATE WAS REDUCE. BUT MONTHS LATER I AM STILL HAVING PAIN. MY DOCTOR DID A PROCEDURE WHERE BY HE LOOK IN MY PENIS AND NOTICE A STONE ABOUT THE SIDE OF A QUARTER. THE PILL TRAMADOR WHICH IS A PAIN RELIEVER. I WONDER IF I TAKE THEM WILL IT HELP Doctor: Thanks for asking in healthcaremagic forum The treatment of stones depends upon its size and location. So, please provide details regarding that. Tramadol is an excellent pain killer but will not be helpful in curing your stone problem but only pain because of stone. All the best. Will be waiting for your details."
},
{
"id": 47410,
"tgt": "How to treat small cyst noticed on both kidneys?",
"src": "Patient: Ultrasonogram of kub region shows: Kidney appear mildly echogenic with blurring of their corticomedullary differentiation measuring approx 8.6 cm on right and 9.4 cm on left in craniocaudal dimensiion. small simple cyst are seen both kidneys.And HR Urinary total protein 464 mg /24hrss Cretanine 1.76 mg/dlS uric acid 6.60 mg/ dlswelling and unbearable pain in left foot. Doctor: HelloYour findings may suggest renal parenchymal disease and small simple cysts in both kidneys.Simple renal cysts are generally age related developmental findings and it only require follow up ultrasound.Renal cysts may require treatment if there is pain,increase in size of cysts,infection etc.Increased echo texture of kidneys may indicate renal parenchymal disease.It may be due to many reasons like glomerulonephritis,chronic diabetes mellitus,chronic hypertension etc.This need clinical correlation and further investigations.Investigations include routine hemogram,random blood sugar,urine RE/ME,complete renal function test(include serum urea,creatinine,creatinine clearance,serum sodium,potassium,chloride,bicarbonate,phosphate),GFR estimation,24 hour urinary protein excretion etc.You may need diet restrictions and medicines after investigations.Get well soon.Take CareDr.Indu Bhushan"
},
{
"id": 93145,
"tgt": "Blood test done, liver enzymes elevated, high cholesterol, abdominal pain, metallic test in mouth. Cure?",
"src": "Patient: I am 38 and recently had a blood test as part of a life insurance physical. My liver enzymes were elevated, my cholesterol was at an all-time high of 281 despite me being in the best shape I have been in since my 20 s. A year ago my liver enzymes were elevated during a routine physical, but returned to normal in a follow-up test 3 weeks later. I have had some upper abdominal pain that seems to be aggrevated after meals and this morning I began having a pretty severe metallic taste in my mouth. My doctor can not see me until 5/10, should I wait or should I seek a follow-up to the blood work prior to that? Doctor: hi.. you have not given the history of alcohol intake.. which may cause alcoholic liver disease and raised liver enzymes.. it also alters the lipid profile.. causes gastritis.. you are on the antacids..with proton pump inhibitor.. metallic taste could be due to some drug intake..see your physician, take multivitamins/liver protectives like silymarine and liver enzyme preparations.. all the best"
},
{
"id": 162916,
"tgt": "What does this UFR test rate for UTI indicate?",
"src": "Patient: hi, My Daughter is 1.4 year old. She is having fever only in the night. today UFR report came and shows 10-15 pus cells. think that she is having a urine infection. need to do a culture right? doctor has given a antibiotic till culture report comes. I just want to know is there any complications in UTI to my baby and what are the causes to have UTI? Doctor: Hello and Welcome to \u2018Ask A Doctor\u2019 service. I have reviewed your query and here is my advice. Based on your reports Ur child is suffering from complicated UTI if urinary symptoms are present and requires treatment with antibiotics. Hope I have answered your query. Let me know if I can assist you further."
},
{
"id": 57774,
"tgt": "What specialist to refer for swollen and painful feet & legs along with water blisters 2 months after gall bladder surgery?",
"src": "Patient: Hello. My mother-in-law (86 yrs) had gallbladder surgery 2 months ago. Since then her feet & legs have been swollen & very painful to touch, along with water blisters. We have been battling w/her to see her doctor but she says her doctor doesn t care . We ve told her to get another doctor & she tells us that she doesn t know what kind of doctor to see. She s been putting this off through the holidays, but the holidays are over. Where do we begin??? And what kind of specialist should she see? Sorry...I didn t realize that I had to pay for this answer Doctor: Hi. There is a slight possibility of having deep venous thrombosis of the legs causing this. Till she gets ready to see a Doctor you can elevate the legs above the heart level for the edema to get drained in the heart. Apply locally liquid povidone iodine to avoid secondary infection. Surely she needs to see a Doctor Vascular in particular ."
},
{
"id": 109575,
"tgt": "What causes swollen lymph nodes in armpit and lower back pain?",
"src": "Patient: I have several things going on. The two worst and most obvious symptoms is slightly swollen lymph nodes under arm pits accompanied by pain, and pain down both armpits to about mid torso and lower back pain. The pain is becoming more frequent and intense. There seem to be no lumps in my breast and within the year i have had extensive lab work done, in which i was told that my lab work looked remarkable. What can this be Doctor: Hi Welcome to healhcaremagic After going through your query I concluded that you have enlargement of lymph nodes in your axilla. You are also having backache. You have got much blood work done but you found nothing. In my opinion you can get your lymph node biopsy done to reach at conclusion. You can also MRI of spine done to see cause of backache. Till then you can take analgesic such as ibuprofen for pain relief. Vitamin D3 supplements will be required to strengthen your back if it is low. You can discuss your doctor about it. Hope your query get answered. If you have any clarification then please don't hesitate to write to us. I will be happy to answer your queries. You can also write to me directly on below link:https://www.bit.ly/askdrsudhirorthoWishing you good health. Take care."
},
{
"id": 1957,
"tgt": "Can BV cause trouble in conceiving?",
"src": "Patient: I have had BV for 2 months now and it keeps coming back despite antibiotics. Also, I am trying to get pregnant and are tracking my cervical mucus. However, it seems to be stretchy all the time. Is it because of the BV or is something wrong (hormone imbalance for ex) Doctor: Hi I think it can be due to BV You can take some vaginal prebiotic to restore vaginal flora and ph.Ask regarding placentrex injections from your doctor. Hormonal reasons can also be there. Consult your doctor."
},
{
"id": 132604,
"tgt": "Is leg cramps and muscles tightness due to fracture?",
"src": "Patient: About a week ago i slipped trying to enter the shower area hit my shin over the edge of the step it s a hard step now after a week the injured area is still swollen bruised my leg hurts my ankle hurts i feel my leg is cramping and muscles are very tight and sore i want to know if my leg is ok or maybe i have a fracture? Doctor: Hi Hope this message finds you in good health.I have gone through your complaints and understand your concern.I dont think u have a fracture,u seem to have injured your soft tissues.Take rest,analgesics,crepe bandage and u should be fine.Nothing to worry about.\u00a0\u00a0\u00a0\u00a0\u00a0I hope your question has been answered.If you have any follow-up queries,feel free to consult me anytime.Thanks,Take care,God bless."
},
{
"id": 91587,
"tgt": "What could be done for abdominal pain while coughing and sneezing?",
"src": "Patient: Being treated for flu related pneumonia with steroids and antibiotics. Woke up with severe stabbing pain on rt lwr abdomen just above hip when coughing. No real pain any other time except when sneezing or coughing. Pain now has moved laterally 6\" across abdomen to just below rt rib cage. Abdomen feels pretty hard. Doctor: HI. The most probable reason can be inflammation of appendix, large bowel or even the gall bladder. When these organs are severely swollen even sneezing can increase the pain. Te best way to get a diagnosis os to have a clinical examination by a Doctor ( preferably a Surgeon) and undergo ultrasonography / CT Scan"
},
{
"id": 211771,
"tgt": "Suffering form mental confusion, not having food. MRI, blood tests done. What should be done?",
"src": "Patient: My husband is suffering from extreme mental confusion. He has had an MIR and well as many blood tests. All tests have proven to be negative. He is refusing to leave his bed and just doesn't want to talk or eat. He is very confused and can't seem to voice his concerns or thoughts. What can I do besides committing him, which is not an option? Doctor: DearWe understand your concernsI went through your description. You have not mentioned the age of your husband. He must be suffering from life anxiety. MRI and other blood tests were normal and therefore he is supposed to be alright physically.I should ask you so many questions. When this started? What were the basic reasons for this? Is he suffering from any other lifestyle disease? Is he satisfied with his family and professional life? Any unnatural death in your family or nearby? etc.Usually, when a person who sees actual life, he gets confused about his role and should normally become anxious. Fear and anxiety are after products. Fear keep him in one place without moving and anxiety keeps him thinking.Normally such patients need expert intervention. With the help of an expert Psychologist / psychotherapist your husband should become alright.Consult such a specialist. Or book a telephone appointment through Healthcare Magic for me and talk to me. I shall help your husband in every way I can.Hope I am able to partially clarify your doubt.Good luck."
},
{
"id": 136319,
"tgt": "Can therapy solve fixed flexion deformity problem of finger?",
"src": "Patient: hi, i fractured my right 5th finger, a surgery was performed which lead to fixed flexion deformity. i had to go in for a second surgery 8 weeks ago involving tendonlysis and removal of screw. however, at the moment, my therapist told me that the scar appears to have adhesion over the extention area. my question is can i have my finger back , and also can theraphy solve this problem? Doctor: Now physiotherapy is a good mode of improvement for u rather than unnecessary multiple surgeries So if u want 100 % mobility its not possible but surgery followed by good physiotherapy will give good results in 70% cases"
},
{
"id": 186202,
"tgt": "What causes teeth infection?",
"src": "Patient: I am 56 y old and having constant teeth infection problem the dentist prescribed me droxyl clav and enzyflam i have taken 1 dose in the afternoon but still swelling and pain is not stopped shall i continue the same dose or shall i take doxcip and diclofinac Doctor: Thanks for using Health care magic.Read your query.As you have informed that u have consulted the dentist and taking the prescribed medicine,and taken a single dose.,my advise is to continue with same medication as prescribed for a prescribed period.If after 24-48 hours also you are having the same intensity of pain or the swelling is increasing ,you can report back to the dentist and he will have a look at it.hope this was beneficial and you recover soon.Thanks and regards."
},
{
"id": 160587,
"tgt": "What causes white dots in the feces?",
"src": "Patient: My 3 month old baby, who is formula fed, has white dots in her poo. Her poo is always a mix of firm, runny and watery. I have tried her on lactose fee formula and it made her constipated, the easy to digest formula is what she is currently on. Do you have any thoughts about the right formula for her? Doctor: Hello, The white dots are probably unrelated to formula feeds. Possible cause like baby acne must be ruled out. You can try good quality formula feeds like duphalac must be ruled out. Hope I have answered your question. Let me know if I can assist you further. Regards, Dr. Shinas Hussain, General & Family Physician"
},
{
"id": 70690,
"tgt": "How can wheezing and cough along with headache be treated?",
"src": "Patient: I am suffering from a respiratory infection that I contracted over three weeks ago while traveling in Egypt. I took a Z pack (antibiotics) and other OTC cough expectorants and suppressants but nothing brings any relief. I have a wheezing cough deep in my chest with thick yellow phlegm, muscle and head ache, and I cannot hear out of my left ear. I wanted to get a test to see what allergy is affecting me but all local doctors require a first-time consultation before conducting an allergy panel. It s Friday afternoon and I m miserable. What can you suggest until I get in to see a specialist at the local allergy clinic who is not available until August 9th. Thank you! Debra Doctor: Hello, According to the history, you provide you need an ENT doctor consultation. Meanwhile, you can take painkillers orally. Hope I have answered your query. Let me know if I can assist you further. Take care Regards, Dr Jnikolla, Pulmonologist"
},
{
"id": 114582,
"tgt": "How can severe anemia be treated?",
"src": "Patient: I was severely anemic due to fibroids. Was transfused 2 units of blood. Had a hysterectomy in June. Am currently taking iron once a day. My H&H is normal and my ferritin level is now a low normal I think about 31, iron studies are normal. How long should I continue taking iron for? I feel really good when I take it; energy levels increased almost overnight when I resumed taking it again (I had stopped due to post-op sluggish bowel/constipation). I m afraid now that I m not getting my period, I may be taking too much iron; am afraid of overdosing. Doctor: Dear madam, Duration for which iron supplementation should be given is not clear in any of the recommendations. But it is very clear that, after hemoglobin is normalized iron supplementation should be continued for certain duration, to refill our iron stores. Generally hemoglobin level normalizes after 2 months of starting therapy. I give iron supplementation for a total duration of 6 months for all my patients. I do not monitor ferritin levels or iron profile. Lack of having periods is certainly not due to iron overload, but it is because, your uterus has been removed as you had multiple fibroids. So don't worry of about not having periods. Hope I have answered your question. If you have any further questions I will be happy to help. With regards, Dr. Girish Kamat"
},
{
"id": 106444,
"tgt": "Can allergy be a reason for high BP?",
"src": "Patient: Im 23, and have high blood pressure (160/110 ish). I dont smoke (anymore, quit 1 year ago), dont drink more than 8 pints a week, dont do drugs, low cholestrol, not overweight (im 6 3 and about 90kg) and lead an fairly active life. I have seen 2 Consultant Cardiologists now, and after practically every test possible they have both concluded they dont know why my BP is up. While i don t doubt their proffessional opinion, Im sure there is something causing it. Would an allergy (eg food etc) or something unknown to me cause it? A friend mentioned Diabetes , would this be worth following up? Can anyone think of anything, or point me in a direction of help? Oh by the way, I m not stressed, and dont eat large quantities of salt, I eat very balanced diet , including lots of green stuff! Sorry, all my tests came back ok/clear. Kidneys fine. Doctor: Reduce your salt intake"
},
{
"id": 207888,
"tgt": "What are the symptoms and treatment for bipolar disorders?",
"src": "Patient: do you think i could be bipolar, BPD or hormonal, im a 15 year old girl. this is the stuff ive been experiencing.. -weird thought or dreams. - i talk really fast at times and at times slow. - ive been more interested in sex. i know thats the whole point of hormones bubt its like ALOT like i used to think it was yucky nad gross even when i started my period, which was 2 years ago. -MOOD SWINGS!. like one day ill be laughing and the next ill be crying. my self esteam suffers from this and my moods range from anexity, extreme anger, extreme sadness and doubtfulness, happiness wherer i cant stop laughing for no reason and cant stop smiling and i think im better then anyone even though i know thats wrong. its litterally like gears in my head that i cant feel turning but cant do anything to stop them - constant abandonment or feeling empty and alone. i always feel alone, no matter how many people are with me and i feel like everyone could leave me at any minute. - ive gone through alot in my life and its been REALLY HARD. please my mom doesnt believe me and im underage so i cant go by myself. Im scared of whats happening to me and i just want some sort of answer to go by. ive tried online quizzes and tests but i know that doesnt count as a real diagnosis. Doctor: DearWe understand your concernsI went through your details. I suggest you not to worry much. From the symptoms you present, I don't see the reason to classify this as bi-polar disorder. You could be having stress disorder. Usually due to disappointment in life after a series of stressful events. Prolonged stress and disappointment combines together to create lethargy and mood swings temporarily. I assure you, these are temporary. Concentrate on your career ' studies and be active. Ignore these symptoms. Do consult a psychologist if symptoms did not vanish within 15 days. If you require more of my help in this aspect, Please post a direct question to me in this website. Make sure that you include every minute details possible. I shall prescribe the needed psychotherapy techniques which should help you cure your condition further.Hope this answers your query. Available for further clarifications.Good luck."
},
{
"id": 20190,
"tgt": "What causes high BP despite a healthy diet and exercise?",
"src": "Patient: Hi there, three months ago, I had normal blood pressure (I m 55 years old), for whatever reason, my blood pressure has been sitting at 148/101. I m 5-9 , 140 lbs, I exercise twice a week and the only thing I did new in the last 6 months was go on the isagenix diet (bars and shakes) - and yes I am a smoker Doctor: Hello,Blood pressure is an age related change due to hardening of vessel as one ages. Although, exercise, diet has some protective effect but not cent percent. So you should have a healthy lifestyle like avoiding fatty, oily and high calorie diet. Have low salt diet and monitor blood pressure regularly thrice a day for one week then once or twice a week. If bp is persistently more than 140/90 mmhg, then you should be on medicines for it like Amlodipine 5g once a day. Regular exercises like brisk walking, jogging according your capacity atleast 30 min a day and 5 days a week. Lots of green leafy vegetables, fruits, fish once or twice a week, avoid meat. Avoid smoking and alcohol if any. There shouldn't abdominal fat deposition or obesity. Get your lipid profile and sugars tested once.. Hope this helps you."
},
{
"id": 207391,
"tgt": "Suggest treatment for sluggishness in performing daily activities",
"src": "Patient: my cousin age 28 always takes 1 hour in toilet , 1 hour for shower , 1 hour for eating food and most of time stuborn to finish his things as priority , we have put him under Psychriatic treatment , during medicine course during his stay in medical centre for 4-5 months he was quite silent and same thing went till 3 months at home also , but recently he again started act same . what is treatment for him . Doctor: Hello,Go for his thyroid level I think he has hypothyroidism one tablet daily will help him a lot. So dont worry. Hope I have answered your query. Let me know if I can assist you further.Regards, Dr. Awadhesh P Singh Solanki"
},
{
"id": 197552,
"tgt": "How long will it take to heal the torn frenulum?",
"src": "Patient: My husband torn his frenulum about 3weeks ago...it's a very tiny tear right under the head of his penis...he is uncircumcised...we have abstained from sex but it still hasn't healed...we tried to have sex last night and it started to bleed a little and hurt so we stopped...how long will this take to heal? Doctor: HiGreetings. Frenulum is the most sensitive part of Penis and it has an arterial supply which is the one bleeding during intercourse.Any friction or even the erection can disturbe the normal healing and can bleed. But it depends up on the severity of the tear.So if there is bleeding even after 3 weeks I suggest you to consult a surgeon who will help you with either antimicrobial ointments or if a big tear will suture or use cautery to coagulate. That will speed up the healing.Hope my answer helps you. Regards"
},
{
"id": 92945,
"tgt": "Loose stools, abdominal pains, cramps. WBC high, ESR raised. What could be the reason?",
"src": "Patient: hi I ve had loose stools and abdominal pains and cramps for sometime . My WBC is slightly high and ESR also raised. When I smoke or eat it seems to trigger the pain mainly from the upper right side of abdomen. I thought it was a lactose intolerance but I m now concerned it may be more sinister? Please help I m really worried Doctor: Hi and thanks for the query,These characteristics are compatible with an acute infectious process. This is not compatible with lactose intolerance. You must have had a past history while younger and the association of fever, raised white blood cells and erythrocyte sedimentation rates make this possibility unlikely. It is reasonable to do a stool examination and culture to exclude Typhoid and E coli infections.Collection of further samples and sending to the laboratory should be followed by administration of specific broad spectrum antibiotics like quinolones. You should consult your family care physician for further advice,Kind regards.Bain LE, MD."
},
{
"id": 158347,
"tgt": "Lost huge weight after surgery fro stomach cancer, CKD with kidneys, breast cancer. How to gain weight ?",
"src": "Patient: Hello there, im fr. TORONTO in the year 2007 i was diagnose with stomach cancer n have my surgery done , they removed my stomach n lost more 50 pounds. Im a petite lady 5ft. Use to be 125 pds. Now im only 81 pds. I cannot eat proper food coz I got blooded easily. I need help, coz I have CKD as well both of my kidney r damage n I cannot have kidney transplant yet coz im still on my remission, im looking for a kidney donor too! Im am desperately needed help. Im so depressed already, thats is not d only thing, I have also been diagnose of breast cancer in 2009 n removed my left breast . What r my chances? Living with almost nothing inside my body. I need help who ever can ans. N understand my situation. Is der such thing as a plastic stomach or any.... so I can put back n gain weight. Greatly appreciated hearing fr. Expert Doctor. Doctor: I am putting it very bluntly that it is difficult to make your weight increase by any means. kidney disease disease has to improve to for you to gain weight. there is no plastic stomach or any other kind of things to make you gain weight. As you have multiple cancer- check for any recurrence of disease, so that if treated properly, you may gain weight."
},
{
"id": 305,
"tgt": "Suggest remedy for getting pregnant",
"src": "Patient: i am 24 year old,my weight is 50 kg,i have pcos.i got pregnant on 2008.after2 months got Miscarriage..since 2008 we are trying for baby with no luck..i am taking ovulation madicine from over one n half year with 15oo mg metformin..dr.did leproscopy on nov 2009 for pcos drilling..while leproscopy dr. seen some infection..she gave my akt4 for to months and akt3 for 4 months..now i finised my akt cource..my qu is how do i no that my infection is cure?? is there any test? is it ok to take ovulation medicine after taking one and half year?? plz help to get pregnant..i heard about vitex ,royal jelly and baby asprine...is it working for pcos?? should i try?? how to use these thing..plz tell me Doctor: Hello and Welcome to \u2018Ask A Doctor\u2019 service. I have reviewed your query and here is my advice.You should do serum TSH and serum prolactin. Then on day 2 of periods start with clomiphene or letrozole with gonadotropins as prescribed by your gynecologist. Then do follicular study till your follicles grow till 18 mm and get rupture and have a relationship with husband.Regards,Dr. Sheetal Agarwal"
},
{
"id": 8312,
"tgt": "How can face darkening caused by benzoate based products be treated?",
"src": "Patient: hello Dr samuel, i am having a problem of facial darkening anytime i use products with benzoate, my face has become very sensitive of recent and brightening products dont seem to work anymore on my face. i am presently using fair and white exclusive fade cream and body lotion. Doctor: Hello. Thank you for writing to us at healthcaremagicBenzoate based products can cause contact sensitivity and possibly it has lead to post-inflammatory hyperpigmentation/ PIH.A common example of benzoate based products is PABA or para amino benzoate which used to be a common sunscreen agent in the past but now has been replaced with other safer ingredients.I suggest you to use a broad spectrum sunscreen, covering both UV A as well as UV B part of the electromagnetic spectrum. The sunscreen should be PABA free.Use it throughout the day. Reapply every 2-3 hours, specially outdoors.At night I suggest you to use a moderately potent topical steroid e.g fluticasone propionate 0.05% Or mometasone furoate 0.1%, once or twice daily at night over the dark areas on face.I suggest you to visit a dermatologist in person for exact visual assessment and appropriate prescription.Regards"
},
{
"id": 221471,
"tgt": "Is Ovamit, suggested for inducing pregnancy ?",
"src": "Patient: can iuse ovamit to induce pregnancy iaborted 2mo ago d&c was done twice due to incomplet evacuation this 3 missed abortion after 2 missed idid investigation all was normal my doctor advice to use asprin &heparin for the living child but in this pregnancy in spite of use missed abortion ocur usually i abort3to 4mo. Doctor: HiDr. Purushottam welcomes you to HCM virtual clinic!Thanks for consulting at my virtual clinic. I have carefully gone through your case, and I think I have understood your concern. I will try to address your medical concerns and would suggest you the best of the available treatment options.Please do not worry .I will suggest to get laparoscopy and hysteroscopy.In case of early pregnancy losses, there can be a septum in the uterus, that can be tackled during hysteroscopy.Tablets to induce ovulation are fine, but remember your problem is not of getting pregnancy but is about its continuation.Along with aspirin and LMW heparin use of micronized progesterone will be of help.I hope my answer helps you.Thanks.Wish you great health."
},
{
"id": 210890,
"tgt": "What do you suggest for post traumatic stress?",
"src": "Patient: I have a 35 year old son who has been diagnosed with Post Traumatic Stress, he suffers bad with night terrors and wakes up in a very angry mood, difficult to be around. This man has a partner and 4 young terrors, he realises he has many issues to work through that are causing his anxiety and is prepared to but there needs to be something chemically that will help him through the process. He feels better with Diazepam but this treatment is not easily available in NZ as they say it is addictive, do you have some other suggestions for medication to help with his sleep and mood problem? From a concerned Mum Doctor: Hello,Thanks for choosing health care magic for posting your query.I have gone through your question in detail and I can understand what you are going through.Diazepam is just ofr the temporay relief of the anxiety. He will probably need paroxetine or sertraline or propranolol or clonidine if it is certainly a post traumatic stress disorder. It would be est to see a psychiatrist who can re confirm the diagnosis or you may contact us on our premium queries so that we can get further details from you as well. Lot many psychotherapies are also possible...Hope I am able to answer your concerns.If you have any further query, I would be glad to help you.In future if you wish to contact me directly, you can use the below mentioned link:bit.ly/dr-srikanth-reddy\u00a0\u00a0\u00a0\u00a0\u00a0\u00a0\u00a0\u00a0\u00a0\u00a0\u00a0\u00a0\u00a0\u00a0\u00a0\u00a0\u00a0\u00a0\u00a0\u00a0\u00a0\u00a0\u00a0\u00a0\u00a0\u00a0\u00a0\u00a0\u00a0\u00a0\u00a0\u00a0\u00a0\u00a0\u00a0\u00a0\u00a0\u00a0\u00a0\u00a0"
},
{
"id": 99920,
"tgt": "How to determine food allergies?",
"src": "Patient: My husband recently was diagnosed with having red meat allergy due to a tick bite. His body reacts with hives. He has given up all red meats 4 months now, and still has hive outbreaks. We are trying to figure out if he could have other types of food allergies, like dairy, foods with red dies ...He has an allergist appt this week for more testing, but wanted to try to have more info for the allergist to test for. Doctor: Hi thanks for using Healthcaremagic It is possible that your husband is allergic to many food materials ..The best way is to find out by yourself..But it is not possible always..There are some panel of allergy testIt is available in leading Diagnostic centres They will tell you the severity of allergy to common food materialsit is not 100% reliable,but you will get a clue...thank you....."
},
{
"id": 144385,
"tgt": "What causes shock sensation in the head?",
"src": "Patient: How long does it take the body to return to a normal state after antidepressants with drawl? My Doctor is excellent had we been slowly whining me off prisqic . I get those shocks in the head every few days. when will the shock zaps stop? Doctor is slowly whining me off over a few weeks, Do the zaps in the head cause brain damage Doctor: Thank you for your query.discontinuation syndrome is a group that occur after cessation of antidepressant and/or anxiety medications.These medications increase the availability of 'happy' chemicals in your brain which help to ward of depression and anxiety.when it is gradually weaned off and stopped, the 'happy' chemicals return to its original state. This triggers certain feedback mechanisms and other ' not so happy ' chemicals which may cause symptoms till the balance has been restored- usually about 2-3 weeks. depending on the type (long or short acting) medication, the incidence , time taken for the symptoms to appear and disappear may vary.Symptoms include fatigue, flu- like stuffy nose/myalgia, vomiting, sleep disturbances, nightmares , jerking movements of particular parts of the body (tremors) and electric shock sensations( as you described it). These are basically nerve signals returning to its premeditated state and do not cause brain damage.If you continue to experience these symptoms after 3 weeks or you experience severe mood disturbances enough to prevent normal life and routine, talk to your doctor regarding medications which help you ease through the the process.hope this helps. wish you good health and please contact us if you have any questions"
},
{
"id": 30971,
"tgt": "Can I get infection through open wound?",
"src": "Patient: Hi, i work as a chef and recently cut myself at work, cleaned myself up and put on a sterile glove as usual as i have a first aid qualification, however i then went to the toilet as i was feeling a bit sick and light headed, as often happens after a close cut! The bathroom was in a state and hadn't been cleaned so i decided to change my glove and then wash my hands in the kitchen, i have since been told that i must wash my hands in the bathroom, surely this is risking contamination and infection? Many thanks Joe Doctor: Hi,There is no problem washing your injured part in bathroom as you have not to touch washbasin or other stuff which are likely contaminated.Simply wash your hands with antiseptic lotion or soap.Washing in kitchen is again dangerous as this night contaminate food causing serious problem to your customers.Ok and take care."
},
{
"id": 174484,
"tgt": "What could be the cluster of tiny blisters on chest and abdomen of child?",
"src": "Patient: My 10 year old boy has a cluster of tiny blisters (about 30 over a 3 1/2 inch area) on his chest and several others on his abdominal. He has had them for about two weeks. It seems like a few more have popped up over the last several days. What could this be? Doctor: Hi,Thank you for asking question on health care magic.Looks like staphyllococcal skin infection.antibiotic containing ampicillin and cloxacillin for 5 days may help.Local application of antiseptic lotion or ointment will help in recovery.Hope this answer will serve your purposePlease feel free to ask any more queries if requiredTake careDr.M.V.Subrahmanyam MD;DCHAssociate professor of pediatrics"
},
{
"id": 45859,
"tgt": "What causes parenchymal cyst in kidney?",
"src": "Patient: Hello. I recently had a CT bc I felt something on my liver. It turned out to be a hemangioma. In the report it also mentions a subcentimeter parenchymal cyst in my right kidney. No one even mentioned it. Does that mean it isn\u00b4t important_ I have never had a kidney cyst before. Doctor: Hello and Welcome to \u2018Ask A Doctor\u2019 service. I have reviewed your query and here is my advice. Kidney cysts are absolutely harmless and are incidentally detected during ultrasound or CT scan. They wont cause any harm and doesn't require any treatment. You can go for a followup scan after one year to check whether the cyst has increased in size or not. Hope I have answered your query. Let me know if I can assist you further. Regards,\u00a0\u00a0\u00a0\u00a0\u00a0 Dr. Shinas Hussain"
},
{
"id": 171869,
"tgt": "What causes loose bowels in 6 months old?",
"src": "Patient: HI Doctor, I am Mrs Naik, my son just completed 6months and have recently started him on cerelac (as of now a scoop of rice and a scoop of wheat), which he enjoys eating. However i have noticed he has been passing very loose stool since 2 days. Some say its a sign of teething too (hes already on dentonic)...... is the loose motion a reaction to recently starting cerelac and normal and will eventually pass away? Doctor: First of all, stop using Dentonic. It is useless. Secondly, do not label any loose motion as being due to teething unless it clears in less than 48 hours, and is less than 3-4 per 24 hours.Your contention that they might have occurred due to Cerelac may be correct but you need not stop it. The motions will generally recover within days.Dr. Taher"
},
{
"id": 138523,
"tgt": "Can muscle twitching and fatigue be the cause of difficulty walking?",
"src": "Patient: my brother has had a few episodes of extreme muscle twitching of all four extremities which cause him to have difficulty walking. after the episode is over he is extremely fatigue, wold sleep, when he wakes he is not having the shakes on the outside but says that he is having the shakes on the inside aand he still is having difficulty walking. Could this mean he might have MS Doctor: Hi,Thanks for your query.It seems that you have muscle twitches or fasciculations. These are small, local, involuntary muscle contraction and relaxation visible under the skin arising from the spontaneous discharge of a bundle of skeletal muscle fibers. You might consult your doctor who can order 1. a few blood tests including thyroid function tests, 2. Serum electrolytes including Serum calcium and potassium levels. 3. Serum B12 levels In case the above tests are normal further testing can be done including - nerveconduction studies, Electromyography, etc. These tests might help to diagnose any underlying neurological issues. I do hope that you have found something helpful and I will be glad to answer any further query.Take care"
},
{
"id": 143611,
"tgt": "How can seizures be healed?",
"src": "Patient: My son who is 41 years old started having seizures last October. The doctors have not determined what has caused him to start having them. he has them one the average of 1 - 2 per month He is taking keppra. When he has these seizures he passes out. I live in Texas and he lives in Washington State. He has been seeing an neurologist but nothing seems to be helping. He has had CT and MRI s but nothing is showing up. He is loosing his memory with these seizures. He forgot my parents died 3 years ago when I just spoke with him and he asked me the same questions 3- 4 times. I would like for him to see someone else but living so far away, I am not sure what I can do or who I should turn to in Washington State. He lives in Richland. Do you have any suggestions what to suggest him to do or where to go for more help. Money is a concern as he has not been able to work, nor drive and he does not have any insurance. Thanks Doctor: Hi, 41 yrs old patient with seizure disorder on tab keppra is having recurrent seizures .So antiepileptic should be changed or its dose should be increased .If seizures are controlled by this then it should be continued for 3 yrs .Thanks"
},
{
"id": 87087,
"tgt": "What can cause severe and sharp pain in the abdomen?",
"src": "Patient: hi im jeaneth, i want 2 ask this one:july 1 i have my period then i took chlomid at the 2nd day of my period til 5th day of my period...the doctor also found infection during ive done papsmear wid her..i took also flagyl almost at the same time.i took flagyl for 9 days..now i have check up lately & they found me pregnant..but i have intensely sharp pain on my lower abdomen not consistent its on and off sharp pain.they check me but only fluid they can see on the ultrasound?doctor told me i have to come back after i week to see again my tummy..im afraid to take any pain releiver..pls help me.. Doctor: you are suffering from PID( pelvic inflammatory disease). supported by ultrasound too. you should take medicine for 1 week and should go for follow up."
},
{
"id": 37837,
"tgt": "What causes fever with body chills and muscle aches?",
"src": "Patient: My daughter is 18 and never gets sick. She is very active. Last Friday she woke up not feeling very well but still went running. She went to work and said it felt like she was walking through quick sand. Every muscle hurt. She came home and has had fever, chills. We went to Insta Care and they tested her for strep. It was negative so they sent us on our way. It has been 5 days now and she is no better. She has no appetite, a headache, and says she has a pain in her stomach area. Her tonsils and lymphnodes are swollen in her neck. Not sure what is going on. Doctor: HelloThank You for contacting HCM.Symptoms are suggestive of Acute Tonsilitis. I would suggest you following things:> Take amoxicilin/clavulanic acid 625mg thrice a day for 7 days> Take Diphenhydramine + Dextromethorphan for cough.> Take acetaminophen for fever & pain.> Ask her to take off from job and rest for 2-3 days at least.> Take hot water in a cup, add 1 table spoon salt in it. Perform gargles (mouth wash) with it three times a day for at least 3 days. It will give soothing effect.> Take boiling water in a pot, put your head over the steam and cover whole with a sheet. Take deep breaths in it.(Avoid getting closer else it might cause burn). Do this twice a day.AVOID:> Chilled water> Drinks> Spicy foodHope this answers your question. If you have additional questions or follow up questions then please do not hesitate in writing to us. Wishing her good health"
},
{
"id": 105419,
"tgt": "Sinus infection. Had deviated septum. Why do I still suffer from rhinitis?",
"src": "Patient: Hi doctor, For the last one year i am suffering from sinus infection .I had a deviated septum and was operated by a Yale Surgeon last year .There was a spur on my nose may due to a fall I had year before. was also taken out.Still I suffer from rhinitis as I have many allergies my ENT specialist asked me to take allergy shots and for last seven months I have been on allergy shots every week,and also on singular , astepro , .15 %nasal spray in the morning and fluticasone propionate nasal spray at night with zertex tablet every night. please be good enough to explain to me why I have this condition for so long and what I should avoid eating and what else I should to do to get better.My e-mail address is YYYY@YYYY you will not forget to write to me soon. Thank you. Mrs.Shiranee Fernando Doctor: Hi, Shiranee, Yhanks for query, You have got allergic rhinitis and unless you identify the allergen it is difficult to cure. There are many causes of having allergy and you have to find out the allergen observing one by one by avoiding that thing. These factors are, Dust,weather,food like certain cereals,vegetables like bengan,tomato,cold drinks Cosmetics like soap,duo spray,perfumes,some clothes,synthetics etc. By trial and error you identify the allergen. To know the allergen now allergy tests are also carried out. Ok and bye."
},
{
"id": 155239,
"tgt": "Suggest treatment for nagging cough in cancer patient",
"src": "Patient: my mother has been detected cancer of food pipe last week. She had nagging cough since last three months. i want to know how much is the growth in three months. what medication you advice She is 87 years old. All paremeters are very good except infection of the lung ranjit Doctor: The cough that your mother has had may have developed due to a abnormal communication between the foodpipe and windpipe. That makes it an advanced tumour. If there is increase in cough, you may have to show a gastroenterologist for stenting.As she is already 87 years old, any aggressive oncological management is not advised"
},
{
"id": 225343,
"tgt": "Blood pregnancy test negative but have all pregnancy symptoms. Can removal of Mirena cause this?",
"src": "Patient: I recently removed my mirena on October 23rd and recieved my period on October 25th( very heavy for 4 days) my periods are normally light and on time every month. I've had intercourse every other day ever since the removal. I was supposed to get my period on November 22 and never received it. I took a blood pregnancy test and had a hcg of 1.40. They told me negative, but I feel pregnant: nausea , abdominal pain, cramps, tired/sleepiness, could this mean that I'm pregnant or that my mirena is playing tricks on my body? Doctor: hi,Thanks for writing to us.These symptoms could be due to mirena withdrawal and occur due to hormonal changes that occurs due to discontinuation of mirena. Also your pregnancy test is negative, so pregnancy is ruled out.please meet your gynaecologist to help you out with the same.Regards"
},
{
"id": 223966,
"tgt": "How to prevent unwanted pregnancy?",
"src": "Patient: doctor i have an urgent problem. i had sex yesterday at 7:00 in evening, now it is 10am here in dubai, we did not use condom and i am not on any contraceptives,it was just an impulsive thing to do to have sex like that. i want to know how i can prevent myself from getting pregnant. the cheapest or the natural ways because i am also broke right now. thank you. and if ever there is a best solution that would require me to buy a medicine or something tell me as well coz i really dont want to get pregnant. Doctor: Hello dearUnderstand your concernAccording to your question you have did sex at 7:00 in the evening and it is 10 am in the morning. Means just only 15 hours has been passed after sex.I advice you to take emergency contraceptive pill like morning after pill that contain 1.5 mg levonoprogesterol. Take it as early as possible and it is very effective in preventing pregnancy in 95 percent cases.It prevent the ovulation and implantation.After taking it breakthrough bleeding will occur with in 3 week and your period may be early/delayed or excessive due to hormonal imbalance.In future use proper contraception to prevent this type of situation.Hope this may help youBest regardsDr. Sagar"
},
{
"id": 72320,
"tgt": "What to do for the discomfort in breathing?",
"src": "Patient: I feel problem in taking breadth, When i try to take long breadth I feel Blockage, I feel that i am wearing tight clothes, doctor suggest me for the test ECG, XRAY, PFT which is normal, Now he suggested me to go for gastropinion..I feel blockage in food pipe.. Doctor: Thanks for your question on Healthcare Magic.I can understand your concern. First of all no need to worry for major heart and lung diseases because your all reports are normal.Possibility of stress and anxiety related symptoms is more likely. So better to consult psychiatrist and get done counselling sessions. Try to identify stressor in your life and start working on its solution. You may need anxiolytic drugs (propranolol and flunarizine combination) too.Don't worry, you will be alright with all these. Avoid stress and tension, be relax and calm. Hope I have solved your query. I will be happy to help you further. Wish you good health. Thanks."
},
{
"id": 183161,
"tgt": "Suggest remedy for sore tooth",
"src": "Patient: Hi, may I answer your health queries right now ? Please type your query here...MY TOOTH LOWER RIGHT IS VERY SORE WHEN I CHEW(I HAVE NO WISDOM TEETH)ANYTHING THAT IS HARD. TOOTH IS NOT SENSITIVE TO COLD OR HOT DRINKS. HAS BEEN SORE FOR ABOUT A WEEK. PLEASE HELP! Doctor: Hello, thank you for consulting with healthcaremagic. The cause of this soreness can be an infection of tooth which might have spread to the apex of the tooth, so due to pressure while chewing it is causing pain. Better that once you should visit your dentist and get a root canal treatment of the tooth done. Hope it will help you."
},
{
"id": 3683,
"tgt": "Suggest remedy for getting pregnant",
"src": "Patient: Age:26 yrs,Height-5.6,Weight-70 kgs.medical history:I have experienced 2 missed abortions one in the year 2009 at 5 weeks pre embryonic loss,G sac seen,D & C done, and 2nd in the year June 2010 8.6 weeks fetal heart beat seen,11weeks fetal heart beat absent. D& C done.Anti D was given both times.Have done all hormonal test,infections test,immunology trst,protien C , Protien S ,Homocystien ,HSG xray,Karyotyping also normal of both me and my husband.Husband sperm test also normal.All test are normal.I want to concieve a helathy baby .Please advice doctor. Doctor: Dear Madam,I have gone through your history and do empathize with your feelings. Early pregnancy loss most of the times could be due to chromosomal abnormalities. However, your karyotyping is normal. I would advise you to report to a foeto-maternal specialist to get yourself evaluated thoroughly for any other possible cause. Thyroid dysfunction, particularly low function of thyroid also may cause such pregnancy wastage. I do not see this investigation performed in your list. Please get your blood examined for T3, T4 and TSH. If hypothyroidism is detected, Levothyroxine is the medicine of choice. I hope this will help you."
},
{
"id": 46880,
"tgt": "Are my kidney conditions normal?",
"src": "Patient: Hi, I am Rl,/shra, Sex - M, Age-62.I had to get admitted with with the following symptons:-Fever with chills and rigor, Weakness, pain in feet, vomiting, inc frequency of urine.i am known case for DM type 2 patient since 1981. But after dignosis doc told me that your kidney is 90% damaged and treated me as CKD patient.Some inv report is as follow:-HB on 16/10/11 is 7.3 and TLC is 25000 creatnine is 7.3HB on 25/10/11 is 8.0 and TLC is 9400 creatnine is 6.0On 25/10/11 doc relived me, but told me that you have to go for dialysis within 6 months.i dont understand, when all the reports is improving day by day then why doc told me so. Doctor: HiFrom the information given it seems that you have been admitted with kidney infection which can itself make the kidney function worse either temporarily or permanently. However the creatinine was 7.3 on admission which seems to be improving. Normally creatinine levels are generally less than 1.2 and it does not start to rise until atleast 50% of the kidney has been damaged. Hence your doctor has treated you as CKD patient. It is likely that your creatinine levels will fall further and you may or may not need dialysis in the future but ony continuous monitoring will tell us about the need for dialysis in the future. Let us keep our hopes alive that the creatinine levels will normalise."
},
{
"id": 119928,
"tgt": "What causes pain in ribs with intake of mega red tablets?",
"src": "Patient: I started taking Mega Red tablets to help lower my cholesterol and started having pain in both my right and left side under my ribs? When I stop taking it it goes away why am I having this pain? Could it be a connection from the other drugs I take for High blood pressure? Doctor: Hello,Mega red is a supplement which contains omega-3 fatty acids. It does not cause pain in the side of the ribs as a side effect. I suggest to do a chest X-ray for further evaluation.Hope I have answered your question. Let me know if I can assist you further. Regards, Dr. Dorina Gurabardhi, General & Family Physician"
},
{
"id": 199045,
"tgt": "What causes bleeding in penile discharge without any cuts?",
"src": "Patient: Yesterday about two hours after my shower I noted a patch of blood on the front of my underwear while changing my outer clothing, about three inches long and two wide. The position of the stain and the lack of external evidence of a breach of the skin left me to conclude that the source was a penile discharge. No pain accompanied this discharge and I'm not certain exactly when it occurred. This is a first-time experience. I have a lady friend who has herpes but we've been careful to avoid sexual contact during outbreaks...and those contacts are infrequent as we are both past 65 and usually disinterested. I've never experienced any other STDs, have never used drugs, do drink casually but haven't be \"drunk\" since my military days over 40 years ago. I'm unaware of any cancer in my family but most live well into their nineties with several passing the 100-year mark. Erections are, of course, not what they used to be, but I haven't expected them to be given my age. Surgically, I've had 3 hernia repairs on my right abdomen, the last performed in the early '90s when a mesh was installed. Again, no pain in the region. Not much else to report at this stage...the discharge has not recurred.Dennis Doctor: HelloThanks for query ,Based on what you have described and your age I would state that the bleeding that you have is mostly from the superficial surface of Enlarged Prostate /Since this is first incidence there is nothing to be worried about.Consult qualified Urologist for clinical and digital rectal examination and get following basic tests done to confirm the diagnosis.1) Routine urine test and Urine culture 2) Ultrasound scanning of abdomen and pelvis.3) Serum PSA ,In the mean while take antibiotics for 5 days .Ensure to drink more water.Dr.Patil.Further treatment will depnd upon the results of clinical findings and tests."
},
{
"id": 197940,
"tgt": "Suggest treatment for infertility",
"src": "Patient: I have married for 1 year and 4 mounths. On the 3 rd mounth my wife was pregnant.But after this she miscarriage.\u0130t was tube pregnancy.And i was gived sprem test.My results was very bad sperm is very low and i am oligozoospermia. After this i was started take some drug but it doesnt effected.Also i knowed that i have very low varicose on my left testis.Then i operated on varicose.After operation i wait 3 mounths and again gived test but result was same.I went Turkey.\u0130n Turkey doctor gived me proxeed for take 2 mounths and make again test.\u0130 taked Proxeed for 2 mounth and made again test but result again same.I decited make IVF. So we went to Turkey Ankara to make \u0130VF.My vifes E2 hormon was very high but we decited to make IVF.I gived sperm for IVF and they finded sperms.So we made IVF .But it was also miscarriage.This mounth i was gived sperm test also there is very low rise.On 1 ml the number of sperm is 0.3 mln,ejaculated is 1.2 mln.So i dont know what to do.What is my dianosis?And main question is can i have children in natural way? Doctor: HelloThanks for query .Based on the facts that you have posted Your sperm count is very low (Oligospermia) and have taken various medicines to increase the sperm count but did not get any gratifying results,At the outset I would like to post that there are no medicines scientifically proved to be effective in increasing the sperm count .You have tried IF but it also was unsuccessful .In a given situation I would suggest you to consult qualified Infertility specialist and try Intra Cytoplasmic Sperm Injection (ICSI) with selecting the sperms by sperm concentration technique .With ICSI there are 30% chances of natural conception .Dr.Patil"
},
{
"id": 81405,
"tgt": "What causes cloudiness in lower left lung?",
"src": "Patient: thank you for your help my dad had chest x ray and showed cloudiness lower left lung seen the doctor because had a cough for few days while on a trip to Florida also slept on couch for few hours which found out there was mold on lower part of wall doctor said could be an ammonia but not sure what is your opinion Doctor: Thanks for your question on HCM.In my opinion your father is having left lower zone pneumonia (infection).Travelling history and chest x ray is favouring the diagnosis of pneumonia.So consult pulmonologist and get done sputum culture and sensitivity to guide antibiotics therapy.If sputum is not available than get done bronchoscopy and BAL (Bronchoalveolar lavage) analysis and start antibiotics accordingly.He needs antibiotics, antihistamines and anti inflammatory drugs.So consult pulmonologist and discuss all these."
},
{
"id": 102625,
"tgt": "What are the ide effects of Cyclobenzaprine, Gabapentin USP powder And Bupivacaine hydrochloride present in a cream applied for allergies?",
"src": "Patient: I have been taking ambien for around 6yrs, 1x, Xanax as needed up to 3x as needed, fluticasone nasal and pro-air albterol sulfate during allergies acting up, 2x I have a compound cream that is applied 3x daily it has ketoprofen ultra micronized powder, baclofen usp, powder, cyclobenzaprine, hydrochloride usp powder, gabapentin usp powder, bupivacaine hydrochloride, usp monohydrate powder, diethylene glycol monoethyl either nf liquid, base pcca lipoderm activemmaxand now cyclobenzaprine. What are the side affects Doctor: topical drugs have got minimal side effects. long term use can result in systemic absorption and can impair your concentration. hope this helped."
},
{
"id": 191116,
"tgt": "Hard lump on lower right jaw region",
"src": "Patient: my 1 1/2 yr old just develop a hard lump under her lower right back jaw area. Could it be dental related since her molars haven t come in yet or is it more of an issue with her lymph or salivary gland(infection)? Doctor: hello, continue the given antibiotic upto the date doctor mentioned if there is no progress its better to visit a pedodontist as soon as possible and treat early.. take care."
},
{
"id": 206090,
"tgt": "What causes the feeling of being in flashing lights?",
"src": "Patient: I used to take Zoloft for anxiety then abruptly stoped. I was taking it for about 6 months now and i was taking a dose of an average high amount for my body. About 6 months after i stoped, i have ben getting this weird feeling when i am aroung flashing lights. It is like i am dreaming. i feel like i am not really there and then i come back for a second and then it al feels weird again.i dont know another way to describe it. I am very dizzy almost. My adrenaline starts going and i kind of freak out. It has happend 3 times to me in the span of about 3 months. 2 times were invovled when i was near flashing lights but one happend when i was just at a house party. It scares me in a way a because i have no idea what is happening to me and when i try to describe it to people, they cannot understand.The only way for it to go away is if i lie or sit doen, drink cold water, and closemy eyes. even then sometimes it can take up to 1 1/2 hr for it to go away. Please Help Me. Doctor: DearWe understand your concernsI went through your details. I suggest you not to worry much. I sincerely opine that the problem you are facing now is of your own creation.You know that you are suffering from anxiety disorder. All these given symptoms are related to anxiety and obsession. I think youu had zoloft on a psychiatrist's prescription. You do have anxiety disorder and medicines along with psychotherapy and meditation should help you. Please consult a psychiatrist.If you still need my help, please describe the whole problem in detail and post a direct question to me. I shall definitely help you with psychotherapy techniques to over come your problems.Hope this answers your query. Available for further clarifications.Good luck."
},
{
"id": 203948,
"tgt": "Small white, scaly patch on foreskin, had unprotected encounter. Herpes or STD?",
"src": "Patient: Hello...I have a small white scaly skin patch on my foreskin . About 2 weeks ago a had an unprotected encounter, just hands touching my genitals, a brief moment of oral sex , no intercourse. I m scared to death of herpes or any other std . Several days ago I got the flu like symptoms, sore throat, no fever. I don t think there is any discharge from my penis... Doctor: Hi small white scaly patch over the penis can be candidiasis rather than herpes.because in herpes we get erythematous papules all over the penile skin which will subside by one week.Candidiasis on genital areas can be due to diabetes or white discharge in the partner.it can be treated by flucanazole 150 mg weekly once for three weeks along with application of onabet ointment twice daily can cure the condition"
},
{
"id": 14621,
"tgt": "Suggest treatment for red itchy rash on calves,chest and neck",
"src": "Patient: i have a red flat rash on my upper calfs, chest, neck, both arms and belly, it is itchy, i have not changed my soap or detergent. I have not changed anything in my routine, i have had this rask for a couple of weeks, no new medicines either. Thank-you Concerned Doctor: Hello,Thank you for posting on HCM.It\u2019s very difficult to approach conclusive diagnosis unless for proper physical examination, anyways from your description it seems you are suffering from scabies.Scabies is a contagious disease caused due to mite and it spreads among people who are in close physical contact and by use of common household articles.Treatment is pretty straight foreword. Firstly, i would suggest treatment of your secondary infection (if any) with oral and topical antibiotics.The key to successful treatment of scabies lies in treating everyone simultaneously in the family and others who are in close physical contact. Your doctor would prescribe oral drugs like ivermectin which will kill the parasite. You might also be advised application of permethrin 5% cream and a corticosteroid with fusidic acid cream. Apply the medicated creams exactly as advised by your doctor. Take antihistaminics as advised to relieve itching and irritation. Avoid sharing of clothing and toiletries and launder all you bedding and clothes in warm water.Hope this will take care of your issue.Wish you best of health.Thank youDr Hardik Pitroda"
},
{
"id": 128186,
"tgt": "What causes pain in the hip while walking?",
"src": "Patient: Hi I am having hip pain it s deep like in the socket , I did just have it while walking , cleaning or playing pickle ball, but now its all the time, aches like a toothache. I ve been taking Ibuprofen on a regular basis, it s not helping. I ve been stretching it, seems to make worse. Please advise Thanks Laura Doctor: dear sir/ madami would like to know the age group, and examine your hip for range of motion and tenderness,and have a radiograph of the hip for looking the contour of femoral head and for sclerosis.mostly it could be hip arthritis."
},
{
"id": 93402,
"tgt": "Facial pain syndrome, lymphotic colitis, diarrhea, inflamed colon, selling of belly, fever. Positive for RF. Suggest?",
"src": "Patient: I have my facial pain syndrome and lymphotic colitis and have flares of colitis far too often which are only manageable with steroid treatment.Recently I had a flare and started entocort which controlled the diarheaa but my colon was so inflamed that my belly had swollen immensely with pain. I went to the ER to additionally start on prednisone in which I take a high dose now.I started having a fever and flushing in my face as well as extremely hot. It felt like I would burst into flames.I have tested positive for the RF factor and have pain in all of my joints. This is a highly unusual flare. I have chronic severe occipital neuralgia and myofasial pain syndrome, but there is something else going on. My GP thinks I still just have myofasial pain syndrome, but my latest symptoms and research I have read do not indicate these symptoms are common with flareups. I'm going for a checkup with a fresh eye, and trying to get referred to a rhuemetologist.My lymph nodes are also swollen without tenderness.I feel there is a different issue now.Any advise? Thank you Doctor: Hi, You have facial pain, lymphotic colitis, flares, manageable with steroid, after having steroid colon inflamed and belly swollen, with pain, with high dose of prednisolone had fever and flares, pain in all the joints. It could be a viral fever of the chickan gunya, or dengue. I advise you to consult a physician for diagnosis and treatment. High dose of paracetamol may be prefered. Thank you."
},
{
"id": 119493,
"tgt": "Suggest treatment for low back pain",
"src": "Patient: I am a 24 yr old, healthy and active woman. In July, I was cleaning and took quite a hard fall and hit my back on the side of the bathtub. Ever since, my thoracics and lumbar has been a problem. I have been to the chiropractor repeatedly since and the problem always seems to come back within a week. I went to my doctor and he diagnosed a muscle sprain and put me on somas, but they do not help. Also, a few weeks ago I was putting on a pair of boots and heard a snap in my very lower back...followed by alot of pain. What should I do? Doctor: Hi, It is quiet common to have these types of pain persisting for quiet a lot of time in absence of proper treatment. All you need to do is to give rest to your back, avoid over exertion and lifting heavy weights. Along with warm saline fomentation twice a day, consult a physio also for ultrasonic therapy. If all these doesn't proves to be effective then get an X-ray of the affected part done. Take care. Hope I have answered your question. Let me know if I can assist you further. Regards, Dr. Rohan Shanker Tiwari, Orthopedic Surgeon"
},
{
"id": 67818,
"tgt": "What causes lump under chin?",
"src": "Patient: I have a small lump under my chin. I did not notice it until I a few hours after I screamed as hard as I could a couple of days ago. It doesn t really hurt unless I push on it and then it feels like a bruise. I m wandering if this could be some kind of muscle strain? Doctor: Hi, dearI have gone through your question. I can understand your concern. You may have submandibular lymphnode enlargement or you may have inflammation of submandibular gland. You should take a course of antibiotics. If it doesn't respond to treatment then go for biopsy. Consult your doctor and take treatment accordingly. Hope I have answered your question, if you have doubt then I will be happy to answer. Thanks for using health care magic. Wish you a very good health. Don't forget to click thank you."
},
{
"id": 114551,
"tgt": "What causes leg cramps and night sweats while having low WBC count?",
"src": "Patient: I recently had a complete physical. I am 54 years old... everything checked out great. expect my white blood count was alarming low. I am going in for more blood work in a week. I have not been sick with sore throat or flu like symptoms but I have not felt right for about 3 months. I had thyroid surgery 2 years ago. removed my left lobe but I still have my right side... my levels were great. I am a runner, work out about 4 times a week, play golf, tennis etc. very active . I am a healthy eater . I am having leg crapps, night sweats, chills, still have periods. should I be concerned or is this normal.... thank you. Doctor: Dear Madam,With low WBC counts, symptoms such as chills, may be sign of serious internal infection. Please go to nearby hospital urgently, as you may need antibiotic infusions. You may also need a bone marrow testing, to know the reason for low white blood cell count.With regards,Dr. Girish Kamat"
},
{
"id": 18525,
"tgt": "Is it fine to have salty food once in two weeks in case of hypertension?",
"src": "Patient: Hi, can I answer your health question? Please type your question here...I have normal blood pressure of 120/80 but it only rises about 5-10 mmhg after consuming salty food or when i am under stress. I am a 29 year married woman. My blood pressure reached 140/90 in the last few days before i gave birth to my daughter but then it went back to normal after delivery . It now only rises like i said after consuming food rich in sodium.I do cardio twice per week.Is it safe if i consume fast food every 2 weeks like have a slice of pizza or a MacDonald s sandwich every now and then? I eat healthy the rest of the days. Doctor: Hello and Welcome to \u2018Ask A Doctor\u2019 service. I have reviewed your query and here is my advice. I would like to tell you that you can take salt in your diet but in limit which will help you in managing your normal blood pressure in long run of life. Rise in blood pressure while in stress or during physical exertion is normal which do not require any treatment. It's recommended for you to avoid fried food and continue with your cardio exercise for 40 min 5 days a week. I hope this information will answer your query. Kind Regards Dr. Bhanu Partap"
},
{
"id": 22505,
"tgt": "What could cause electrifying feel on right side of chest?",
"src": "Patient: I had never had a problem with high blood pressure it always ran about 120/80 .About 2 weeks ago ,I got really bad headaches and just felt bad in general so I went to the Dr. it was 188/94 . She put me on a water pill , after about 4 days it started messing with my bladder because of my IC. She did an EKG she said it looked good except that my heart rate was to high.It was 125. She then put me on an ace inhibitor.I quit all caffeine and cut way back on my smoking. After I was on it a few days I started getting a feeling in the right side of my chest that felt like electric shocks. They were so bad that I couldn't sleep.I called her , she put me on a beta blocker, they helped bring the heart rate down and reduced the number of electric shock episodes. should I see a cardiologist to make sure my heart is okay? I feel some pressure in my chest ,mainly at night when I lay down. Doctor: Hi,Do you have chest pain or heaviness on exertion or just chest pain only. It doesn't look like pain and it seems gastritis and reflux disease. Surely BP should be controlled and you can visit to get routine test done like lipid profile, sugar and echo. You should follow healthy lifestyle avoiding fatty, oily and high calorie diet. Have low salt diet and monitor blood pressure regularly thrice a day for one week then once or twice a week. Regular exercises like brisk walking, jogging according your capacity at least 30 minutes a day and 5 days a week. Eat lots of green leafy vegetables, fruits, fish once or twice a week, and avoid meat. Avoid smoking and alcohol if any. There shouldn't abdominal fat deposition or obesity. Also avoid spicy and fatty foods and maintain sleep habits regular and also do regular exercises. Have some walk after meals instead of taking rest immediately. You can take acid suppressant like tab panDsr.Hope I have answered your query. Let me know if I can assist you further.Regards, Dr. Sagar Makode"
},
{
"id": 136766,
"tgt": "Suggest treatment for bone overgrowth in mouth",
"src": "Patient: My daughter has blood clots, is on a HUGE dose of Coumadin, has an extreme bone overgrowth in her mouth, & has a softball size solid mass on her right shoulder that has been growing down into her chest wall for over a year. She is also passing what looks like fat & blood clots in her stool. I know that this all sounds crazy, but she s been getting sicker & sicker & no one seems to know what to do. She s seen multiple doctors in Iowa & is in terrible pain. Do any of these symptoms together suggest anything? She is 36 & has 4 children. I m scared we are going to lose her. Doctor: Thank you for using HCM. I am sorry for your daughter's suffering. By description given it seems like Ewing's sarcoma which needs to be confirmed through Histopathology and immunohistochemistry. Please visit an oncologist at the earliest for confirmation as the survival span decreases with metastatic finding if it is Ewing's sarcoma. Treatment option is chemotherapy."
},
{
"id": 146168,
"tgt": "Suggest remedy for neuropathy with numbness in body and fatigue",
"src": "Patient: I HAVE HAD PROGRESSIVE NEUROPATHY ,ABDOMINAL PAIN, NUMBNESS NOW SPREADING TO ALL AREAS OF BODY INCLUDING ABDOMEN AND FACE-CHIN LIPS, NOSE, BURNING SSENSATION IN LIMBS CANT PUT ANY WEIGHT BACK ONAND FACE. NECK FEELS WOBBLY.ANXIETY, FATIGUE WIGHT LOSS OF 18 TO 22 LBS IN LAST 7 MONTHS . CANT PUT ANY WEIGHT BACK ON. ALSO MUSCLE TWITCHES CONSTANTLY LOSING A LOT OF SENSATION IN FEET ARMS AND LEGS Doctor: tratment for neuropathy is amitryptyline 25 mg at bed time. you have neuropathy symptoms with weight loss. i suggest you investigation as blood sugar, thyroid profile"
},
{
"id": 69887,
"tgt": "What could be the bump behind my left ear?",
"src": "Patient: I get a bump behind left ear..sometimes goes away and sometimes painful..I pop it like acne to relieve pressure and a thick sticky, creamy white substance comes out and it is extremely stinky and potent..what is this? Or where do I go to get it checked out? I dont have a family doctor Doctor: Hi,This is likely to be an infected sebaceous cyst. This in not serious and can be easily treated. Keep the area clean and dry. Apply topical antibiotic cream or ointment available over the counter. If it does not clear up then you may need a minor surgical operation to remove it. Regards,Dr K A Pottinger"
},
{
"id": 213667,
"tgt": "I have loss of consciousness once in a month for 5 min duration and sometimes my eyes roll upwards and I get some frightening picture on mind and forget that picture after gaining consciousness",
"src": "Patient: Respected sir/madam ,my age 45 yrs, height 150 cm, weight 65 Kg, my problem is loss of consciousness once in a month for 5 min duration and sometimes eyes uprolling upwards and gets some frightening picture on mind and forgets that picture after getting conscious. it happened 4 to 5 times in 4 months. Doctor: hi well come to Hcm The way you have described it can not be anxity, it looks like some convulsive dis order better consult ylur Doctor. Anxiety is not serious ailment but convulsive disease does carry risk of its kind. Thanks"
},
{
"id": 209514,
"tgt": "Could taking Ultra hemo can control anger?",
"src": "Patient: I have a young (18 yrs. old) friend who has suddenly become very angry, flying into a rage over the slightest thing. He's always beeen sweet-tempered, and I know 18 is a tough age, but this seems pronounced since he started using a supplement called Ultra Hemo Rage as a preworkout supplement. Could this cause anger? Doctor: HiThanks for using healthcare magicUltra Hemo Rage is a protein supplement and it helps to increase physical strength. In that case, chances of getting anger due to Ultra Hemo Rage is very less. May be due to some reason or stress, he has such anger outburst. Better to consult a psychiatrist for proper mental status evaluation and in case, you need further help, you can ask.Thanks"
},
{
"id": 168723,
"tgt": "Is it normal for an infant to pass stools twice a day?",
"src": "Patient: Hi...my daughter is 9 months old. I started giving her solid foods right from 7th month. She does good. For the past 1 week she passes stools 2 or 3 times a day which is quite not the usual. I gave Econorm sachet 2 times today. Should I consult a peadiatrist or is it normal for the baby. Please advise. -Uma Elamaran Doctor: Hi,Passing stool 2-3 times a day at this age is common by starting solid food.If she is fine, gaining weight and passes urine normally, nothing to worry.you may not require to give any medicine unless frequency increases.Ok and take care."
},
{
"id": 122356,
"tgt": "Suggest medication for swollen neck on the right side, tinnitus and night sweats",
"src": "Patient: I have a swollen neck lymph node on the right side. Its close to my jugular / thyroid. Its been there for more then year. Iv lost a lot of weight and when i swallow most times I can feel it in my throat, but then goes away. I have a constant ringing in my ears. At night my hands turn really red and hot. I have night sweats and can not sleep. What could this be? Doctor: Hi, To make a definite diagnose, you need to do FNAC of this swelling. Lymph node swellings can be due to underlying infection or obstructive reason too. Hope I have answered your query. Let me know if I can assist you further. Take care. Regards, Dr. Tanmoy Roy, General & Family Physician"
},
{
"id": 26009,
"tgt": "Should I give my father Megace?",
"src": "Patient: My father weighs 113, height 5'8', age 89, congestive heart failure, several bouts of pneumonia in the last few years, cannot eat without feeling nauseous - Dr. wants to put him on Megace (have already tried various other things). Is Megace a good option? Doctor: Thank you very much for asking me this question megace in high doses can increase appetite in some cases since your father has been just of heart failure so you should be careful because it can worsen shortness of breath it has some other side effects like thrombophlebitis and pulmonary embolism but usually it is a common practice to give this medication and you should be under constant of care off your physician I hope this was helpful thank you"
},
{
"id": 141836,
"tgt": "Suggest treatment for Fits",
"src": "Patient: dear sir, i am ch pradeep raju m24 since 2002 on wards iam suffering with fits and i am using medicine zeptol 200 upto 2005 later another changed as epilex chrono 300 from these years i am continuing the medicine daily but there is no change if stopped taking medicine or more tired i am getting jerk.now i am taking in only night time but i have absorbed something in brother marriage it is late night 2.50 am but in busy time i used to take in twice a day i don't have any trouble in marriage, but it is my time to have marriage so think what to regarding this can u clarifie regarding this issue as early as possible Doctor: I read your question carefully and I understand your concern.However I am afraid that more information is needed in order to try and help you. To start with it is unclear what types of seizures (fits) you have. There are many seizure types and appropriate medication varies according to that. In addition to that you don't mention any tests. Brain imaging, EEG and some blood tests are needed in all patients with epilepsy in order to characterize seizures and determine best treatment.Furthermore you mention the drugs you have taken but not the dosage (how many tablets per day). It might be that the drug was appropriate but the dosage too low. There are also blood tests which determine drug level in the blood to assess whether to raise the dosage.So as you see more information is needed to try to help you. Please search for medical reports. If no tests were done they should be ordered by your doctor."
},
{
"id": 202016,
"tgt": "Is it ok to masturbate daily?",
"src": "Patient: hii sir...i m 24 yr old and i am unmarried. masturbation is more harmful or not..tell me about this.......i am addicted of this habit and i do 3-4 times daily.... sometimes i feel weak.....bt i take healthy diet....so plz tell me that this will affect me in future very much? tell me about this.. Doctor: Hello dear,Masturbation is a normal phenomena at your age, and does not have any side effect on body or mind, unless you get addicted to it.Since you are feeling that masturbation is interfering with your normal functioning, you can try out the following measures:1. Improve your social life by spending more time with friends and relatives.2. Keep your mind active and busy at all times either through curricular or extra-curricular activities.3. Have a diet rich in fruits, vegetables, fish, nuts & honey.4. Avoid fast foods & lipid containing diet.5. Avoid smoking & alcohol.6. Keep away stress, think positive.7. Exercise regularly & practice meditation.These measures will be helpful in improving your health & build up the confidence level.Take care."
},
{
"id": 64753,
"tgt": "Suggest treatment for keloids",
"src": "Patient: I had small keloids on my both ear lobes in past 16 years i have removed them 4 times by surgery (once by cosmetic-plastic surgery) every time it grew bigger. the last surgery was done 2 years back and i was asked to wear pressure clips now when i stop using them it starts grwoing again, please suggest what am i suppose to do casue i m really fed up of sugerys Doctor: Hi,DEar,thanks for your query to HCM.I studied your query in depth.1-In My opinion the small recurring keloids with 16 yrs history,could be treated by -Inj.kenacort-Intra-lesional , in stages ,to cover the whole of the post-surgical scar.2-Hope this would solve your query.3-Wellcome for any more questions on new or this subject also...Have A Good Day..!!"
},
{
"id": 35240,
"tgt": "What is this bacterial infection called?",
"src": "Patient: theres a bacterial viral infection going around thats hurting people.i had my little girl on july 22nd she got sick in the hospital.i had her in easley they sent her to greenville and then sent her to charleston.they said she got a bacterial viral infection it hurt her organs her heart took up 80% of her chest.she passed away on aug.17th.now theres a 7 year old that has it that i know.and i have a three year old.i was just wondering if you could help me out and tell me what is the name of the bacterial viral infection Doctor: Hello dear,Thank you for your contact to health care magic.I read and understand your concern. I am Dr Arun Tank answering your concern.Sorry to heared about your little one.Diagnosis based on this much short history is not possible.Usually the individual gets only single infection at a time. So if child is infected with either viral or bacterial infection.For bacterial infection we can do culture and sensitivity for blood drawn from the body. Itbeill guide you regarding possible organisms and the drugs effective on that bug. So treatment becomes very easy.For viral infection PCR test is necessary it will guide us regarding possible possible infection and so mode of management can be possible.I will be happy to answer your further concern on bit.ly/DrArun.Thank you,Dr Arun TankInfectious diseases specialist,HCM"
},
{
"id": 139948,
"tgt": "What are the symptoms of recovery from a vegetative state?",
"src": "Patient: Is it a sign when a person in vegetative state holds a phone to ear and tries to speak, eats and drinks through mouth, tries to hold up own head and cries when family members sings happy birthday to them that they understand what is going on around them and that those are actions of a person who is at recovery stage? Doctor: Hello, I would explain that these are good signs of recovery. So, there is still hope!Hope I have answered your query. Let me know if I can assist you further. Regards, Dr. Ilir Sharka, Cardiologist"
},
{
"id": 67657,
"tgt": "Does hard lump in front of ear above ear lobe requires attention?",
"src": "Patient: 22yrs old; 5 4; 174lbs; family history of diabetes, heart disease, and auto immune diseases: MS and lupus. I have a small, hard lump in front of my ear 1 inch above earlobe. Painful when pressed. Didn t notice it until I leaned my face into my hand. Should I go see a doctor? Doesn t hurt unless it s pressed. Doctor: Hi...it may be skin inflammation or infective if pain increase nd throbbing in nature....just try anti inflammatory drugs like brufen 400mg twice a day... if swelling dose not respond we can further plan for ultrasonogram nd treat acordingly...."
},
{
"id": 111433,
"tgt": "What causes sharp pain in the back while playing?",
"src": "Patient: I am a14 year old volleyball player and have been getting a really bad pain in my right side of my back everytime i serve the ball I am lwft handed so you would think if it was to hurt it would be my left side but it's not I'm not sure what it is? but it feels like a sharp stabbing pain everytime i go to serve Doctor: Hi, welcome to our site. I am Dr Saumya Mittal, MD.Read your query. That is a very significant question and i appreciate your problem. I will try my best to answer your queryI think you have a strained back. Ideally I would suggest to you a physiotherapy for sometime and a complete rest from the sport, that keeps aggravating the injury you may have had.In case this does not help you, an MRI of the back would be recommended, and warranted.I hope this helps you. Inform the reports mentioned above/if any other so I can be of help further. I have given you the answer to the maximum considering the information provided. The results of the tests could further enhance my answer to you.Please do understand that some details could be extracted from a detailed history and examination.Please feel free to ask another query. I would be glad to help you. Looking forward to your return query with the details asked so that I can help you further. (If the answer has helped you, please indicate this)Best of luck."
},
{
"id": 19135,
"tgt": "Suggest treatment for rapid heart beat",
"src": "Patient: i have been having these weird spells. my heart beats fast regulary. bout 90 a min, sometimes faster and them i get real hot sick to my stomack and feel like im going to pass out. my doc here said anxeity. my lips also stay numb. i dont believe its anxeity. Doctor: Hello,Your recent clinical symptomatology is highly suspicious of a potential cardiac arrhythmia. Irregular fast heart beating and near fainting support such a conclusion.Coming to this point, it is necessary discussing with your cardiologist on the possibility of performing a careful physical exam and additional medical tests as follows:- Resting ECG- Complete Blood Count- Thyroid hormones level- Blood electrolytes level- Fasting blood glucose- Cardiac ultrasound, liver and renal function testsIn case an intermittent cardiac arrhythmia is suspected, an ambulatory ECG (Holter) monitoring is advisable.Hope I have answered your query. Let me know if I can assist you further.Regards,Dr. Ilir Sharka"
},
{
"id": 183854,
"tgt": "Suggest medicine for lump on the mouth caused due to bony projection of maxilla",
"src": "Patient: I have just had a CT scan for a lump in the roof of my mouth right upper jaw, report goes like this. Bony projection is seen rising from the inner aspect of the maxillary alveolar process adjacent to the right first molar and it is continuous with the cortex. It is keeping with a small osteoma. The maxillary teeth appear normal on this study. Incedentally there is prominent maxillary soft tissue thickening in keeping with sinonasal mucosal disease. I am 44 years of age and have no previous medical history in this area. Doctor: Thanks for your query, i have gone through your query.As such there is no medicines for the osteoma or lump secondary to bony growth. It has to be treated surgically. If you have pain then you can take pain killers like diclofenac. Get the osteoma removed surgically by consulting a oral and maxillofacial surgeon. Regarding the sinus problem, which is reported in CT you have to consult a ENT surgeon.I hope my answer will help you. Take care"
},
{
"id": 215819,
"tgt": "How to cure pain on the lower left side of the body?",
"src": "Patient: I have been having pain on my lower left side. The pain is worst when bending over or sitting at times it fills like a pressure of something getting ready to burst. I have had a ct scan done and the dr. did see some inflammation. I am waiting to see a specialist. Whats going on with this pain? Doctor: Hi, It may be due to muscular pain and nothing much to worry about it. You can take simple analgesics like Ibuprofen or Diclofenac for pain relief. If symptoms are severe consult a neurologist and get evaluated. Hope I have answered your query. Let me know if I can assist you further."
},
{
"id": 83355,
"tgt": "Is diarrhea a side effect of lipotrim?",
"src": "Patient: Hi, I have been on lipotrim for 7 days now and for the last two days have been suffering from diarrhoea. Is this normal? Recently I spoke to someone who was also on lipotrim and has had to have their gallbladder removed-I do not wish to end up in the same situation. Someone else has said that the diarrhoea is just your body getting rid of fat. Should I be worried? Doctor: Hello,Diarrhea can be a side effect of Lipotrim. I suggest using an over the counter option to treat the diarrhea such as Imodium. I also suggest using a plain diet which does not trigger the diarrhea.Hope I have answered your question. Let me know if I can assist you further. Regards, Dr. Dorina Gurabardhi, General & Family Physician"
},
{
"id": 201917,
"tgt": "What causes pain and frequent urination?",
"src": "Patient: I have been suffering from BPH prostate for more than two year. Doctor has examined related tests such as PSA,ultrasound and uroflamatory and obsorved my ailment grade one prostrate. He suggested me to take flotral and later change as alfoo which I am regularly taking. But once or twice in fifteen days I face acute pain and go to urine frequently all overnight due to which I feel restless and uneasiness for a day or two. Kindly advise what I should do at this stage. Doctor: Hi,Thanks for writing in.Your doctor is treating you in the right way. The best way to avoid acute pain once in 15 days is by making it a habit to less drink water at night at least two hours before you sleep and pass urine twice before going to bed. Please make this a regular habit and try your best to follow this strictly. Your grade I prostate enlargement is not big enough to require surgery."
},
{
"id": 187196,
"tgt": "What could cause pain at the bottom of gums?",
"src": "Patient: hiya, since yesterday, my bottom gums at the very back are really itchy as if new teeth are coming up. but am 23 and I have already got my wisdom teeth. any explaination of what it is and how to stop the itchiness. My whole mouth is full of bite marks where i was chewing on my cheeks to reduce the itch in my sleep. Doctor: Hello, Welcome Thanks for consulting HCM, I have gone through your query, as you have painful gums dont worry you consult dentist and go for oral prophylaxis . Painful gums can be due to poor oral hygiene , due to periodontal problem. Do warm saline gargle two - three times a day. Do chlorhexidine mouthwash 2 times a day .Hope this will help you."
},
{
"id": 51061,
"tgt": "Abdominal pain, hiccups on touching. Taking medication for TB. Due to kidney stones?",
"src": "Patient: Hi doctor , I am taking TB medicine from last 4.5 months , currently R cinex every day morning , I have managed without any trouble ,but last 3 days left lower side I am getting pain ,it hurts a lot specially night time , initially I thought gas problem because when I touch I got hiccup , knee pain too,might be a kidney sstone?because when I take more water I feel something is moving,not sure what to do,confused ... Doctor: Hello, Welcome to Healthcare Magic. I appreciate your efforts for medical consultation in so much distress. Well, before I am able tp suggest you anything you need to answer few question \u2026.whether there is any radiation of pain, burning micturition,history of constipation..if female then menstrual complaints Wish you good health. Regards"
},
{
"id": 27822,
"tgt": "What can be the cause of pain on the lower side of my heart?",
"src": "Patient: Hello, so today at work i started getting sever heart palpitations...no caffeine besides a cup of coffee in the morning. Shortly after these started occurring, i felt a severe pain right on the lower side of my heart that came off and on for about 3 hours, and along with all this I was winded just walking around. I went to the doctor and they gave me some blood thinners, checked my heart which came back good and the symptoms have all but gone away. I just got a job as a GM at an automotive shop at 23 and am really run thin stress wise. Could stress be the cause of this? The doctors did not find anything and couldn t give me a very definitive answer about stress causing heart problems. Doctor: Though the symptoms might be due to ischaemia to the heart, as you say the tests have adequately ruled that out. The other possibility of transient arrhythmia should be kept in mind and If symptoms Reccur get 24 hour holter, a stress test if not done would be helpful."
},
{
"id": 47344,
"tgt": "What is the solution for pelvicalyceal system fullness and back pain?",
"src": "Patient: Hello Doctor,from the past 3 months i had leg pain in my left leg , and i had consulted with doctor and he advised to me tako UGC sonography for KUB, and the result is Mild Hydronephrosis in my Left Kidney and he advised to take more water. but Now i have back pain and Stomach pain and Loose motion for a week , and again i went for the Sonography. the Result is Fullness pelvicalyceal system and i have severe back pain in the morning. pls advise me to get out from the problem Doctor: HelloFullness of pelvi-calyceal system may indicate obstruction in drainage part of kidney through which urine is excreted out.It may be calculus,PUJ obstruction etc.Since ultrasound of KUB doesn't mention about any calculus,other causes should be searched.You may need IVP(intravenous pyelography).IVP is important in assessing PUJ obstruction,function of Kidneys etc.You may need some intervention depending upon findings.Empirically you are rightly advised to take plenty of water.CT urography may be needed after evaluation.Get well soon.Take CareDr.Indu Bhushan"
},
{
"id": 103782,
"tgt": "Teenaged, obese, veins in the entire body pop out and get dark, random neck and back pain. Cause?",
"src": "Patient: hi my name is savannah im 17, and wiegh about 170. my veins in my entire hand randomly pop oot and get very very dark. my wrists also hurts and my feet do the same thing. Ive had random neck and back pain but the pain in my wrist and arm are constant. over the years i have abused allergy medicine and was wondering if iam having any affects from it. Whats happening??? Doctor: you are a case of food allergy as per my experiences many patients are allergic to wheat or milk as these causes harmonal imbalance mostly milk asit is animal protein and is not competible with human proteins get blood tests for specific antibodies for milk wheat and common foods you take daily start by wjthdrawing milk and diary from diet i think you may feel good in wk"
},
{
"id": 127652,
"tgt": "How to get rid of shin pain?",
"src": "Patient: I was playing soccer and got my shin kicked pretty hard after the hit my whole leg was hurting I couldn't move it for a brief moment. Then I tried to move my foot towards me and everything was shaking and it hurt. Now I can't really move my ankle because of shin. Why? Doctor: Hello and Welcome to \u2018Ask A Doctor\u2019 service. I have reviewed your query and here is my advice. What I understood you hurt your shin and you are unable to walk. But it would have been great if you provide me is there any swelling in ankle or knee. I hope you didn't sustain a fracture because pain would have been much severe alongside deformity. Still if you are having doubts about fracture then cray with Ortho consult will do the job. If no such problem and no swelling/ instablity of knee. Take rest and apply ice with painkillers. Hope I have answered your query. Let me know if I can assist you further."
},
{
"id": 217466,
"tgt": "What causes pain on back and ribs after injury?",
"src": "Patient: I slipped going down stairs and fell on my back and slid down a few steps. I have pain on the right side just below rib cage and pain on my back right on my ribs. I m 38, 5 9 , 145 lbs and no other health issues. I have no trouble breathing, but it hurts when I cough, laugh or twist, bend to the side. There are no bruises, but the area is tender and hurts when I apply pressure. Other than fracturing a rib or two what other damage should I be worried about? Doctor: hieven though you might have suffered # rib or not any injury to the chest is quite painful due to rich nervous innervation, this pain hardly reduces even if you are on pain killersapplication of local pain relieving gel can be of help.if the pain is not subsiding please go for a chest x-ray,& ultrasound abdomen to rule out for the chest or any soft tissue injury( ultrasound) of the abdomen.you can follow with deep breathing exercises, it really helps in lung expansion & relief of painall the besttake care"
},
{
"id": 12225,
"tgt": "Can taking Gilenya be safe for multiple sclerosis?",
"src": "Patient: I have been on gilenya for ms for a week I have started having extreme upper leg/thigh pain. Worse when standing. Almost like terrible muscle spasms and muscles seem hard Feels like legs are shaky and won t support you. Sitting relieve some if it could gilenya be causing this. Doctor: Hi, Gilenya is fingolimod, it's imported one Main action this drug is to retain the lymphocytes and reduce the inflammation and decrease the further episodes of MS and the main side effect is a disturbance in heart rate. and BP. ALLERGIC reactions, dizziness, and fainting and weakness are the other less side effects. Muscle spasms and shaking are not the side effects of this drug. Don't worry about these. consult your doctor and change the dosage schedule will give better results. Hope I have answered your query. Let me know if I can assist you further. Take care Regards, Dr Puvvula Kishore Kumar, Dermatologist"
},
{
"id": 85348,
"tgt": "Which medicine is effective silverex ointment or silverex nitrate for open wound after skin burns?",
"src": "Patient: hi sir. gd evng. i hv a query!!! for burns treatment ( my son has got his skin burnt with the silencer of the bike ) whether silverex ointment is better or silverex nitrate gel 0.2 %( he has open wound, skin completely removed from tht site ). wt is the difference of these two applicants? Doctor: Hello, Silverex Cream contains Silver Nitrate Topical as an active ingredient. They both have the same effect and can be used to treat skin burns. Hope I have answered your query. Let me know if I can assist you further. Take care Regards, Dr Dorina Gurabardhi, General & Family Physician"
},
{
"id": 102102,
"tgt": "Suffering from headache,cough,throat pain and high BP",
"src": "Patient: I do not feel sick, but I recently acquired a cough. I have to clear my throat, like my sinuses are running. But when I cough I get a sharp pain in my right temple. Sometimes it hurts very much, but does not last for more than a couple of minutes. I take medication for high blood pressure but my BP has been pretty good. I had an angiogram last august and my doctor doubled my BP medication. A few days later my vision went blurry. When I look down, to the side, or try to look at something close up I see double. I have been to two different eye doctors and they said I had temporary paralysis of the nerves that control the muscles that control the movement of my eyes. They said it should improve in time because they said my eyes were healthy. A month after my angiogram my doctor changed my BP medication, which helped stabilize my BP , but vision has not improved in 8 months. I get headaches towards the end of the day, almost everyday. I take aspire or Advil to relieve the pain. I don t know if the new pain in my temples when I cough or sneeze is related to my vision problems. When I get the pain it also radiates into my neck and my vision even gets worse. Doctor: HelloWelcome in H.C.M.Regarding your query for headache, cough and throat pain while you are already taking treatment for high blood pressure.As you said that you are having sinus issue that may be responsible fo your throat problem as post nasal drip is well known for this type of sore throat. Another very important cause of cough in your case may be the role of anti hypertensive medicines. Because the ace inhibitors are very commonly use for controlling blood pressure and they are well known for giving rise to irritation in throat.your next query is for pain over temple may be because of venous congestion due to increase intrathorasic pressure by vigrous coughing, and same may be responsible for blurring of vision too.Probably you got the answer of your queryRegards."
},
{
"id": 33813,
"tgt": "Suggest treatment for body rigors and chills",
"src": "Patient: Hi, may I answer your health queries right now ? Please type your query here... I am a 23year old female. Today i got a very nasty sunburn. It feels like there are needles in the areas of my skin that were burned. Also i am freezing cold and shaking uncontrollably. I realize i may have some sort of sun poisoning or bad burn but i want to know- is it bad to layer on clothes and blankets to deal with how very cold i am? Doctor: The symptoms of body rigors and chills what you have mentioned are most probably indicative of malaria so I will advice you to do blood test for malarial parasite. For treatment you have to take antibiotics as suggested by your doctor along with tab calpol or tab akilos-p 8-12 hourly for fever and body-ache. You have to complete the course of chloroquine as per the advice of your doctor. For sunburn you can use Vaseline or any cream for local application."
},
{
"id": 77483,
"tgt": "What could it be if having pressure in chest area and upper thighs are strained?",
"src": "Patient: my chest cavity area feels as though there is outward pressure and the upper portion of my back, between the shoulder blades, is very sore. Also my legs, upper thighs, feel as though they are over worked, like strained but more like the feeling of blowing through a plugged tube. Doctor: Hi,Dear,Thanks for your query to HCM.Dear I read your query and reviewed it with context to your query facts.I understood your health concerns and feel Concerned about them.Based on the facts of your query, you seem to suffer from-Constipation with Chronic Colitis.The accompanied complaints seem to be due to bloating of abdomen from impacted gas which could be from accompanied enteritis with it.Fibromyalgia with it in thigh, legs could be from accompanied sigmoid bloating and pressure on veins or from accompanied entero-colitis.Other causes of such complaints need to be ruled out before confirming it.For this Second opinion from Surgeon / Physician is needed who would do proper investigation after proper physical assessment to resolve your health issues.Plenty of fluids / Change in dietary habits/ avoiding outside food / less of non-veg, adding vegies to your diet/adding exercises / and yoghurt would resolve your issues.Hope this would help you to treat your health issues in the best way possible. Welcome for any further query in this regard.Good Day!! Dr.Savaskar M.N.Senior Surgical SpecialistM.S.Genl-CVTS"
},
{
"id": 111476,
"tgt": "What may have caused sudden hip pain?",
"src": "Patient: I am in Cross Country and last night we had a race(2.5 miles). As I was warming up, my right hip/side started to hurt every time I took a step with my right leg. I figured it would go away after I rested that night, but it's now the next day and it hasn't. Doctor: HiU seem to have muscle strain around your hip region.U need to take rest, apply ice packs 3-4 times a day and take a combination of anti inflammatory and muscle relaxant( diclofenac+chlorzoxane) twice a day.It will take 3-4 days before your pain vanishes completelyRegards"
},
{
"id": 136326,
"tgt": "What causes pain in knee area?",
"src": "Patient: My problem started with pain from the top of my thigh to my knee. Now the pain is concentrated in my knee area. I can t sleep on my side as the pain really intensifies then. I can sleep in a recliner with little pain. But if I am on my leg too long it starts to pain. I have been taking Aleve for a week and the pain still persists. Have had an x-ray and blood work and nothing has shown up. Any advice or ideas as to what this is? Doctor: Meet a orthopaedician get examination of ur knee done and later furthur mri or other investigations if needed to diagnose your knee problem With just your short history diagnosis is difficult"
},
{
"id": 73355,
"tgt": "Suggest treatment for persistent cough",
"src": "Patient: hi my name is jenny.. ive been having a constant cough and mucus problem for about 2 months now. I went to the ER on jan 17th and had a chest xray and breathng treatment done..the chest xray came back clear...i was Dx with cough, and was sent home with a prescription for zpack.. i did the 5 day treatment and about four weeks later today, my symptoms has yet to change....Prior to going to the ER, i tried eveything over the counter and nothing worked...by the way i do not smoke.. pls help..thx Doctor: Asthma is a common cause of cough that you should be evaluated for. Other considerations are reflux (your symptoms would be worse when you lie flat or at night) and post-nasal drip (mucus going from the back of your nose down your throat."
},
{
"id": 216483,
"tgt": "Will Botox block migraine pain receptors?",
"src": "Patient: does botox actually block migraine pain receptors in the brain? Similiar to triptans blocking whatever they block, (which I cannot take) , besides just the muscular relaxation effect. Probably not asking this just right, just trying to get the information. Would this process be impeded by tylenol, or advil? (while the new botox injections are getting into my system....... Doctor: Hello. Botox only works on the muscles it's injected into--it doesn't block pain receptors in the brain. Advil or Tylenol wouldn't affect the action of the Botox in relaxing the neck muscles"
},
{
"id": 142418,
"tgt": "Suggest medication for Parkinson's disease",
"src": "Patient: RESPECTED DOCTOR, I AM LIYAKATH 63 YEARS OLD.I HAVE PARKINSON . DRS.. GIVEN MEDICINES LIKE SYNDOPA, PARKITIDIN AND PRAMIPEX. I TOOK MEDICINES FOR PAST 3 YEARS. RESULT IS NOT GOOD. I AM EXPECTING A GOOD REMEDY FOR MY PARKINSON. .THANKING YOU,WITH BEST REGARDS,A.LIYAKATH ALI. Doctor: Hello Thank you for trusting HCM Dear here is the some of the reasons for poor response to Parkinson's disease. 1.diagnosis of Parkinson's disease is wrong (presence of \"Parkinson-plus syndrome\", etc)2. drug interaction affecting levodopa like increased clearance, poor absorption, blocking at receptor sites(3) insufficient dose of levodopa(4) poor drug absorption from the GI tract due to concurrent disease(5) increased metabolism due to concurrent disease, resulting in a shortened therapeutic period(6) progression of Parkinson's disease with loss of dopaminergic neurons(7) adverse effects of levodopa, including overdosage(8) development of problems unrelated to dopaminergic status like cognitive impairment or dementia, depression, dysautonomia. Please consult your doctor he revise the dose and drugs and treat you accordingly."
},
{
"id": 217999,
"tgt": "What is the pain in the arm?",
"src": "Patient: For a whole two days now, my husband has been complaining of a continuous pain on the upper right arm, and pins and needles in the thumb, middle and index fingers in his right hand. He is unable to sleep because of it. He said it feels like something deep in the muscles of his upper arm. Doctor: Dear Friend.Hi , I am Dr Anshul Varshney , I have read your query in detail , I understand your concern.It seems he has got some nerve compression causing these symptoms.I would advice you to see a doctor. He would do a clinical examination and would give you a conclusion.If inconclusive he might require MRI / Xray off Cervical spine.At present he might take a pain killer like Acetaminophen in moderated dose.A Hot fomentation over the neck should also help. This is my personal opinion based on details available here. If you still have any query you may please ask me.Stay Healthy. Dr Anshul Varshney , MD"
},
{
"id": 25361,
"tgt": "Are there any diet which can control palpitation of heart?",
"src": "Patient: Are there any foods that calm heart palpitations. I have infected tooth and today got heart palpitations. I ve had them on and off for a few years I am wondering if my halfway done root canal is culprit. But also found out I had anemia and low pott. At one point too so not sure. But now my gum above this tooth is swollen up and I had heart issues first time in a few months. Doctor: Hello and thank you for using HCM.I carefully read your question and I totally understand your concern.Heart palpitations has to do with electrical issues in the heart. There are some electrical impulses that generate in a part of the heart different from normal sinus rhythm that cause a premature heart contraction. During this, you might experience a palpitation, e skipped beat, an anxiety sensation.This electrical issues might happen because of different reasons like stress,coffee or alcohol consummation or pathology's like anemia, hyperthyroidism, active infections or congenital heart diseases.As you explain, you once had anemia, and we don't know if you still have, You have a swollen gum, this meas an infection of your gin give, an active infection. These are some factors that might cause heart palpitations.So, the first thing is to menage your tooth infection, your anemia if it is any. I will also recommend a cardiac echo and a blood analyze.There are not specific foods to calm heart palpitations. There are some thinks that you should avoid like caffeine products, energy drinks because they rise heart frequency. Sometimes regular physical activity improves heart frequencies and this palpitations.So, I will recommend you to resolve your tooth infection, follow this instructions and thinks might improve.Hope I was helpful.Best regards."
},
{
"id": 39805,
"tgt": "Is there any cure for polio?",
"src": "Patient: Hi, I am \"Ranjeet Sinha\", working as a software engineer in one of the MNC in hyderabad.I am 28 years old and suffering from polio since i was 2 years old. I want to know is there any cure of polio of the patient like me.Please reply.. Thanks in advance, Ranjeet Sinha Doctor: Hello,Welcome to HCM,Polio is a disease which affects the nerve and the particular muscle supplied by that nerve and leads to weakness and atrophy of the muscle. Polio is a disease which is irreversible and once the damage is caused it is permanent.The weakness of the affected muscle usually the lower limb will affected which can be strengthend by the exercise but the damage caused by the virus is permanent.The calipers will support you while walking but the damage caused by this entero virus is permanent.Thank you."
},
{
"id": 29183,
"tgt": "What does this rabies antibody titer test report indicate?",
"src": "Patient: Before 1yr in Sep 2016 a street dog bite me on my leg that time I completed 5 Rabipure PEP injection course,before 2 month in Sep 2017 my freind feeding a street cat from my food plate wich I eating.that time I am not taking any vaccine.i cheque my rabies antibody titer test its shows Above 4.0 EU/ML.my question is 1)this leval of antibody protect me from Rabies? 2) I need another vaccine course? Doctor: hi sir...sincerely i would like to tell u that u have adequate level of ab ..u dont have to revaccinate ..no recommendation for that ...rabies vaccine preexposure not recommended except for some cases as those who deal with animals much and vertenaries ..in ur case u have just take the vaccine about 1 year ago ..and u have adequate antibodies ...and i want to tell u simply that even if u were not vaccinated from 1 year u dont need to take the vaccine after that eating from that plate ..rabies risk differ according to the exposure ..biting near the neck more risk then biting in extremetis then crush then contact with saliva ...in ur case u r minimal risk...dont worry u r safe from rabies ..thank u"
},
{
"id": 55714,
"tgt": "Suggest nourishing drinks to control gall stone",
"src": "Patient: I HAVE BEEN DIAGNOSED WITH GALLSTONES AND WAS WONDERING IF ENSURE IS OK TO DRINK? I HAVE LOST ALOT OF WEIGHT AND NEED TO KNOW SOMETHING NOURISHING FOR ME WITH VITAMINS? THANK YOU... OR COULD YOU RECOMMEND SOMETHING THAT IS NOURISHING TO DRINK WITH LOTS OF VITAMINS THAT WILL NOT CAUSE MORE GALLSTONES? THANK YOU Doctor: If I were your treating doctor I would advice you for cholecystectomy first better go for it rather consultation regarding diet.Among diet you can take anything just remember to take low fat and low cholesterol diet and no alcohol, other than these you can use appetizers like nusowin drink, aptivate syrup before meals for weight gain, boss powder and ensure powder is basically protein and vitamin powder it is also a good supplement for weight gain. But as far gall stones are considered gall stones form due to gall bladder non functioning and other factors once gall stones are formed there is no function of gall bladder in general so better remove it before its complications sets in.Hope I have answered your question. If you have any further questions I will be happy to help"
},
{
"id": 3678,
"tgt": "How to get pregnant while having irregular periods?",
"src": "Patient: Hi Doc am 31, had a baby 2yrs and 6 months ago, 8 months after delivary i had one shot of depo once about 2yrs ago. my periods are very irregular. i last had my periods 5 months ago. i was put on nerothistorone to bring back my period. i completed the course 2 days ago but i havent seen my periods yet. please help i want to get pregnant. Doctor: Hello, and I hope I can help you today.Northisterone is a synthetic form of progesterone that, once the course had completed, should make you menstruate in 3-7 days if your estrogen levels are normal.So I would try to be patient and wait another week for your period.If you still have not gotten your period after another week, I would perform a urine pregnancy test to make sure you are not pregnant already, and if not, I recommend consultation with a fertility specialist as they can perform hormone tests and suggest the proper treatment to help you ovulate to have regular cycles and also help to conceive.I hope this answer was helpful, and best of luck with your condition,Dr. Brown"
},
{
"id": 27211,
"tgt": "What is the best position for sleeping to prevent palpitations?",
"src": "Patient: I have had svts and palpitations for 10 years under good control, but just this month I notice having onset of palpitations when I arising from sleeping on the couch then I cannot get back to sleep in bed as the episodes last for hours. I can stop laying on the couch, but what sleep position is best in bed to help prevent heart palpitations? Doctor: As a rule svt is not position related, except a few exceptions. So having them on couch may be a coincidence. However one started, the more heart is away from chest wall the less is the sensation, so to answer you, supine position, ie. You sleeping on your back will be the best position for the same . Regards Dr Priyank Mody"
},
{
"id": 80601,
"tgt": "Are antibiotics and inhaler regimen the correct treatment for pneumonia?",
"src": "Patient: I just inhaled a corn kernel. I was choking and coughing but it didn t come up. The kicker is that I was just diagnosed with pneumonia this morning. So, I m concerned that my lungs are already weak. I feel discomfort in my chest in addition to the pain and discomfort I was already feeling due to the pneumonia. My question is, should I just continue my antibiotics and inhaler regimen or is it necessary for me to be alarmed and visit my doctor again? Doctor: Hello dear, thanks for your question on HCM. I can understand your situation and problem. In my opinion you are having aspiration pneumonia due to choking from corn.And the best treatment of aspiration pneumonia is Bronchoscopic removal of foreign particle (corn).So better to consult pulmonologist and get done Bronchoscopic evaluation. Bronchoscopy also helps in identification of causative organism bu BAL ( Bronchoalveolar lavage ) culture and sensitivity. So appropriate antibiotic can be started.So don't start antibiotics haphazardly, better than first isolate the causative organism and then start appropriate antibiotic according to sensitivity report."
},
{
"id": 51664,
"tgt": "Is inguinal hernia infectious ?",
"src": "Patient: my husband has inguinal hernia . Can his hernia cause urinary tract infection on me? Doctor: you cant get uti from your husband due to hernia .infection may be due to kidney stone or may be the other cause ."
},
{
"id": 9869,
"tgt": "Is hairfall associated with mild neutropenia?",
"src": "Patient: hi i have a 15yr old girl who has been having severe hair loss after an episode of nits.i treated the nits and the hair loss has been ongoing for 1 and half months.she had a blood test that says she has mild neutropenia.is that associated with hair loss? Doctor: Hi, I have gone through your query and hair loss is associated with nits.It will take few more months to settle.Neutropenia is unrelated to hair loss. Hope I have answered your query. Let me know if I can assist you further. Regards, Dr. Asmeet Kaur Sawhney, Dermatologist"
},
{
"id": 49037,
"tgt": "Could swollen puffy face and itching in neck be the symptoms of deteriorating kidney function?",
"src": "Patient: For two weeks I have swollen puffy face in the morning and itchy neck ,I have antihistamine for a week but still keeps coming back , feel fine otherwise.timjed @ WWW.WWWW.WW Femail I also have deteriorating kidney function could this. Be a symptom. I am 67 . Doctor: HIThank for asking to HCMI really appreciate your concern, looking to the history given here to rule out the possibility of real disease there should be some associate clinical sign and symptoms, like low out put of urine generalized edema, increase blood pressure, fatigue, if you see all this symptoms then some investigation needed like Ultrasonography, CT scan, Kidney function test, hope this information helps you, have a nice day."
},
{
"id": 206906,
"tgt": "How to increase concentration in studies?",
"src": "Patient: HELLO im a boy 18 year old i don like givin xams my mind refuses completely when i go 2 give xams and i tell 1 incident i was goin to give an entrance exam on 6 june my mind was refusing a lot 2 give d xams but when i told my mind \"ok dont give d xam\" i started feelin better and fine\" and den i started returning home thinking not 2 give d xams but after that i became again confused and den i decided to give d xams and in d first 1 hour i made question very nicely and solved almost all d ques but den my mind again started refusing and i was unable to think and make question after dat please help me nowdays i have intrest in nothong Doctor: DearWe understand your concernsI went through your details. I suggest you not to worry much. You are suppose to refer to concentration and attention as two. Attention span for any adult is maximum 30 minutes. Sometimes, a person can be attentive to an interesting thing for more than 30 minutes, such as movies or games. Concentration is attention plus understanding. For this too, span os below 30 minutes. If you are able to make your studies interesting and based on variable principle, your concentration level can be increased. Concentration comes only when you are interested in what you do. So please be passionate towards what you are doing or do only those things which you are interested in. It is also possible to improve your concentration level with the help of yoga, meditation and breathing exercises.If you require more of my help in this aspect, Please post a direct question to me in this URL. http://goo.gl/aYW2pR. Make sure that you include every minute details possible. I shall prescribe the needed psychotherapy techniques.Hope this answers your query. Available for further clarifications.Good luck."
},
{
"id": 21388,
"tgt": "What are symptoms of detecting blockage in stent?",
"src": "Patient: Hi, may I answer your health queries right now ? Please type your query here...My husband had an angioplasty done he got a baremetal stent which got blocked very early within 4months but he had no symptom then the doctors did another angioplasty and put an xiense drug eluting stent it's been a year now but I still worry about them getting blocked are there any symptoms which can tell if they are he is only 36 he goes to the gym and after exercising his Bp would be like 120/80 please tell me if there is any medicine that can remove the blockage is angioprim a scam Doctor: Hi thereI understand your concern for your husband health and i would like to tell you that the symptoms that are produces by blockage of a stent are the same that occurs while a heart attack like 1. Central chest pain or heaviness2. Radiation of the pain to the inner aspect of left arm or shoulder or in the neck below the jaw line.3. Breathlessness, sweating, sense of doom4. Vomiting5. GiddinessHis BP is fine. No there are no medication that can be taken by patient himself to take out the block.Just make sure that he follow good lifestyle habits and take his medicines on time.I wish him good health"
},
{
"id": 62514,
"tgt": "What causes red raised lump at the back?",
"src": "Patient: About 6 weeks ago, I felt a small round bump on by back. I asked my husband to see if it was a pimple and if so, could he pop it for me. He looked at it, told me that it was a small raise red dot and he didn't think it was a pimple but did squeeze it a few times to see if anything came out. Nothing. A week later, I noticed that it had gotten larger (again, not really being able to see it), my husband said that maybe it was a pimple after all because it looked like one but didn't have a visible head. He squeezed it a few more times and then pricked it with a sterile needle. blood came oozing out but no puss. he didn't squeeze it again, but had to put pressure on it for at least 20 min and even then, I had to put a bandaid on it because it was still bleeding. Over the next few days, it got larger and larger. What my husband said was once a small red dot the size of pea maybe, and raised above the service, was now at least 1/8 inch high and about the size of a penny. It started to scab over, so I left it alone. Until one day, I was laying on our leather couches, and my skin was stuck to it and when I went to get up, I immediately felt dripping down my back. It took almost 1/2 hour to stop the bleeding that time. When I went to the couch,(sorry for the grossness), stuck to the couch was a big ole' scab. Crusty with edges especially dry. This happened one other time, when I forgot, and scratched by back really hard. I know have what looks like a blood blister, but it starting to get crusty, especially the edges, which are pulling away from my skin and are completely hardened. Where the very center feels kind of mushy, putty like. It feels like If I gently worked on the edges that were already releasing, the rest might release as well. Just don't want to make it worse than I have. Doctor: HI,Welcome to HCM.Based on the facts of your query,you seems to have -Delayed Healing of the Boil on the back,due to Untimely Repeated Squeezing and popping of it by you.Now seems to be healing and is in last phase of the healing.Don't disturb the scab till it falls on its own.Hope this reply would help youWelcome for any further query in this regardWill appreciate writing your feedback review comments,to help the needy patients like you at HCM.Good Day!!\u00a0\u00a0\u00a0\u00a0\u00a0Dr.Savaskar,Senior Surgical SpecialistM.S.Genl-CVTS"
},
{
"id": 34860,
"tgt": "What causes abdominal pain following gastroenteritis?",
"src": "Patient: Hello: I am a 36 year old man. After having a terrible attack of stomach flu in february of this year I began to suffer from pain in my upper left side. the bloodwork showed slightly elevated lypase -but all other tests ngative-what can this be -stomach feels bloated all the time Doctor: Hello there,I am dr.milan an infectious disease specialist answering your question.Hope i have given appropriate guidance to you.I think you have loss of some of your intestinal flora, that is beneficial bacterial organism found in our intestine and you have some damage of intestine in form of small small many ulcers.So til this ulcer got heal you may have some pain in your abdomen. And for bacterial flora you take some probiotic food it will replace your flora to normal one.For some period take care you dnt eat spicy food.if you have any query you can consult me anytime.Give me star rating, helpful vote & thank you according to your satisfaction level.Thanking you."
},
{
"id": 46878,
"tgt": "Suggest treatment for renal stone on right calyx and lower ureter",
"src": "Patient: A 27 YEARS OLD ,MALE PATIENT COMPLAINS OF PAIN IN RIGHT INGUINAL REGION AND IN FLANK REGION.....NO PROBLEM IN MICTURATION...ONLY HEAVYNESS IN SCROTAL SAC...USG SHOWS 5MM. RENAL STONE IN RIGHT CALYX AND SAME SIZE IN LOWER URETER...ADVISE TRT PLAN PLZ...THANKS Doctor: HiYour symptoms are consistent with stone in lower ureter. 5 mm stone in lower ureter can be allowed to pass spontaneously. Some 60-70% of patients pass the stone by themselves. Others might need surgery to remove it.In the first instance I would like you get a CT scan of your kidneys and bladder to confirm the diagnosis as well as blood test for kidney function. Taking urimax might help you pass the stone quicker and with less pain but please visit your doctor for prescription."
},
{
"id": 89138,
"tgt": "How to treat severe abdominal pain and diarrhea?",
"src": "Patient: my stomach, more so right below is in painful acidic knots. I have awful diarrhea i feel sick and queezy there are chunks of undigested food and i have no idea what to do/ what is wrong with me... I took an antacid pill, two, a while ago. I also take anti spasmodics for my bowels but they did nothing. Im in a lot of pain. Doctor: By your narrative u might have some infection in your G.I Tract. You should go for antibiotics like Inj.Metronidazole, Ofloxacin and anti spasmodic.Along with that plenty of liquid orally."
},
{
"id": 16427,
"tgt": "Can my wife take martifur ?",
"src": "Patient: My wife had Harpies. Now she is better. She has to take methotraxate (for R Arthritis ) and thyroxine ( for thyroid). At present methotrexet is at hold. Can she take martifur now? Doctor: Hi, Welcome to HCM. The drug interaction between methotraxate with martifur is not known.If methotraxate is stopped for more than 15hrs then she can take the tablet. Take care."
},
{
"id": 129041,
"tgt": "Suggest shoulder exercises post bicep tendon surgery",
"src": "Patient: Hello! I am recovering from rotator cuff/bicep tendon surgery in my right shoulder. I was in therapy up until 3 weeks ago when my workers comp was denied. I feel that I need to find more excercises than what I have now. I am still very weak, as my injury was very severe, and injured in both shoulders. Where can I find therapy excersizes that will help me continue to strengthen? Thank you. Doctor: Hello,I strongly suggest you to consult your surgeon or physiotherapist to suggest you exercises because shoulder surgery particularly have specific protocols for starting physiotherapist/exercises which only your operated surgeon can suggest"
},
{
"id": 118941,
"tgt": "Asplatic anemia. ATG treatment. Suggestion?",
"src": "Patient: Hi My dad is 62yrs old and he is suffering from aplastic anemia. He had an ATG(horse) treatment now his bone marrow is functioning as expected after 6months of treatment. Initially he responded well. I'm clueless what is to be done now. Initially when he was treated with ATG he was given 30vials of atgam. I'm getting mixed opinion on what is to be done from various specialist. I want a genuine suggestion what is to be done now. shall i go for second ATG(Rabbit) or shall i try out BMT? please help??? Doctor: hithanks for your queryi can feel your concernsif your dad is healthy physically fit ,with no co morbidities and HLA matches donor is available and he is willing then treatment of choice is BMTIf he has any associated illness and his body is not able to tolerate conditioning and further stress of BMT, then go for second ATG.hope it helpsregardsdr.imran"
},
{
"id": 187873,
"tgt": "Should I be concerned about the OPG X-ray showing white shadow in the jaw area and infection and was advised for a CT scan?",
"src": "Patient: I had an OPG xray today and it showed that I have an infection which I suspected however it also showed a white shadow located in the jaw area on the other side of my face. They don't know the cause and am having a CT scan on Monday. Should I be worried? Doctor: Hello, thank you for consulting with HCM.If your OPG is showing that there is some white shadow in it , it means that there is some cyst or tumor in that area, if it is not a normal landmark then there must be some pain or swelling associated with it.But there is no need to worry as there are solutions for each and every problem.Better when the diagnosis is made once again you report.Hope it will help youRegardsDr Dipti Singh"
},
{
"id": 121246,
"tgt": "What could cause hardness in fingers?",
"src": "Patient: Hi Sir, This is basant from Noida, I have been my fingers skin getting harder and breaking after that. I took medicine, In which vitamin E has been given to me, also an ointment which heals it for few days. After few days it again comes back. Please advice on this. Doctor: Hello, Thanks for the query.possible causes like hyperkeratosis must be ruled out. Consult a dermatologist and get evaluated. Topical salicylate will be effective. Hope I have answered your query. Let me know if I can assist you further. Take care Regards, Dr. Shinas Hussain, General & Family physician"
},
{
"id": 155393,
"tgt": "What are the effects of acute pancreritis in long term?",
"src": "Patient: what are the long terms effect of acute pancreritis (sp?) ,if any. Six months ago i had a reaction to a sulfar drug, It fried my pancreas and went to ER. It was not an enjoyable experince. Now I am back to normal but my heart rate tends to be higher (approx 80-100). I do regular cardio vascular exercises and have always eaten healthy. What gives Doctor: Hello Effect of acute pancreatitis depends upon severity of infection.Acute pancreatitis patients recover in majority of the cases.However some may develop abscess,pseudocyst or duodenal obstruction.Acute pancreatitis can be divided into mild and severe pancreatitis.Ranson Criteria is used to determine severity of acute pancreatitis.In severe pancreatitis,amount of necrosis determine the further clinical outcome.You may have repeat ultrasound to assess pancreas.It is good that you are so active physically.Get well soon.Take CareDr.Indu Bhushan"
},
{
"id": 116944,
"tgt": "Why do I have a fluctuating blood pressure?",
"src": "Patient: Why do I have a bouncing blood pressure, prefer not to take any blood pressure medicine since I am very sensitive to certain kind of madicine, and never know when it affect my head, I loose my memory when I take muscles relaxer medicine and certain other medicine Doctor: Hi, dear. I have gone through your question. I can understand your concern. You should take anti hypertensive medicine regularly. Without medicine its common to have fluctuating blood pressure. If you have allergy or hypersensitivity then take go for drug sensitivity test and then go for medicine suitable to you.Hope I have answered your question, if you have doubt then I will be happy to answer. Thanks for using health care magic. Wish you a very good health."
},
{
"id": 151519,
"tgt": "Scan showed straightening of cervical sine, osteo discal bulges touching the spinal cord. What treatment should be taken?",
"src": "Patient: MY WIFE HAD A SCAN AND THE REPORTS SAYS STRAIGHTENING OF THE CERVICAL SPINE WITH OSTEO DISCAL BULGES IN C2/3 TO C6/7. THE MOST SIGNIFICANT LEVEL IS C5/6 WHERE THERE IS QUITE A LARGE OSTEO DISCAL BULGE TOUCHING THE SPINAL CORG AND CAUSING BILATERAL FORAMINAL STENOSIS . OSTEO DISCAL BULGE AT C4/5 CAUSES RIGHT FORMINAL NARROWING. SLIGHT OSTEO DISCAL AT C2/3, C3/4 AND C6/7 ARE ALSO SEEN BUT WITHOUT FORMINAL STENOSIS.....SHOULD I BE REALLY WORRIED ESPECIALLY ABOUT LARGE BULGE TOUCHING SPINAL CORD AND WHAT TREATMENT CAN SHE EXPECT Doctor: The points you mentioned abour your wife is more of imaging findings. The imaging findings should always be correlated with her symptoms and the clinical findings on examination. The mri shows compression of nerve roots coming from neck to upperlimbs and also compression of spinal cord in neck. Intial managenent requires rest to the neck usinf collar and limitations of the daily activities. Pain killers may be required depending on severity. Physiotherapy and rest to the neck are key to improvement. Patients who do not respond to these may require surgeey to neck depending on the patient symptoms and clinical findings."
},
{
"id": 54493,
"tgt": "What is the treatment for acute pancreatitis?",
"src": "Patient: My mother in law (54+) was detected to have liver cirrhosis four years ago when she was being operated gall stones. the operations could not be done. Six months ago she had an attack of acute pancreatitis. Doctors have advised removal of gallstones. Can it be done at CMC Vellore and if yes, whom should we contact. Doctor: It's a case of pancreatitis induced by gall stones, fairly common in females. It has to be operated, that's the only solution.. You can call them directly from their number given on their website"
},
{
"id": 123933,
"tgt": "What causes pain and burning sensation on collarbone and shoulders?",
"src": "Patient: I have been dealing with constant grinding, popping, cracking in my shoulders and collarbone area for the past month...seems like the left is the worse. i do noticed pretty severe pain and burning type sensastion with movement also. I am really frustrated with the pain and looking for answers. Any ideas? Doctor: Hello, As the pain in the collar bone can be due to the muscle tightness of the throat and laxity of the neck and can be associated with the shoulder muscle weakness too. This can be connected to the changes in the normal cervical spine curvature and needs to be addressed by the help of an x-ray or an MRI. As due to changes in the cervical spine curvature there might be pathological changes leading to pain in various forms and adjacent joints. I will advise you to check for the x-ray or MRI of the cervical spine and see if any abnormal changes are seen. If so, then using a neck collar and basic exercises for the Neck and upper limb as a whole will ease the symptoms. Hope I have answered your query. Let me know if I can assist you further. Take care Regards, Jay Indravadan Patel, Physical Therapist or Physiotherapist"
},
{
"id": 140630,
"tgt": "Suggest treatment for pressure in the head after straining neck",
"src": "Patient: Hi, This past sunday I was running near a pool, slipped and hit my head. However, the hit did not leave a bump and I did not have any signs of a concussion. Also later in the night, my friend and I were play wrestling and I think he strained my neck a little. One day later, I started to experience pressure in the back of the head and a slight feeling of being spaced out. Everything else, however, is fine. What do you recommend I do / how long do you think these symptoms will last. Thanks Doctor: Hi, My best advice is that you be checked out by a doctor who is experienced in examining for and diagnosing concussions and traumatic brain injuries. Even though you say you had no signs or symptoms of a concussion you're not a trained expert. Taking a direct fall on solid concrete while running and striking your head almost invariably causes symptoms of a concussion which doesn't mean you have to have lost consciousness or been suffering from major headaches immediately afterwards. You may very well be exhibiting symptoms of what we call postconcussive syndrome and these can last from several days to several months and in rare cases, several years. If you are feeling \"spaced out\" then, at the very least I would recommend getting a CT scan of the brain and if I were your treating or examining neurologist I would likely order an MRI of the brain which can demonstrate contusions of the brain much better than any other testing sequence. Hope I have answered your query. Let me know if I can assist you further. Regards, Dr. Dariush Saghafi, Neurologist"
},
{
"id": 225242,
"tgt": "Can mirena birth control be the source of intense bleeding and abdominal pain during intercourse?",
"src": "Patient: I have the mirena birth control and have had it for a year and a half now. But lately every time I have intercourse I start to bleed. I m also starting to notice when I drink alcoholic beverages I will bleed. My stomach starts to hurt and I bleed. I was just wondering why this would be happening. Doctor: NamastheWelcome to Healthcare-MagicGreetings of the dayBleeding following intercourse could be due to possibly local genital inflammation like Cervicitis.I would suggest you to consult Gynecologist for further assessment.In case you need any further assistance, will be glad to assist you.Wishing you happy and healthy lifeTake CareBest RegardsDr Deepak K KMBBS,MS,MCH"
},
{
"id": 29483,
"tgt": "Is Azithral advisable instead of Monocef shot for typhoid?",
"src": "Patient: I am suffering from typhoid fever last week ,Doctor advised me monocef 2gm injection to me for 5 days one one and tab azithmal 500 one and alaspum one one .May I take tablet instead of injection.May I take multivitamin tablet also for weekness. May I do join my office work also .Which type food may I take How many days should I rest.Please suggest me Doctor: soft food to be taken for 3 weeks and u need rest of 2to 3 weeks.but dont shift to tablet...you can take multi vitamin tablet for weakness"
},
{
"id": 215843,
"tgt": "What causes pain in back,legs and arms?",
"src": "Patient: My husband had an accident and got road rash and there for caused a staph effect ion on his buttocks after giving antibiotics 3 times it is finally gone . But has since then been in so much pain in legs , back and arms is there anyway this is related ? Doctor: Hi, Staph infection and back pain are not related and pain may be due to something else and probably due to some spine related problems that is quite common after a road traffic accident. As a first line management we can go for simple analgesics like Tramadol or Diclofenac for pain relief and if symptoms persist, you can consult a physician and get evaluated. Hope I have answered your query. Let me know if I can assist you further. Regards,\u00a0\u00a0\u00a0\u00a0\u00a0 Dr. Shinas Hussain"
},
{
"id": 198801,
"tgt": "How to cure yeast infection?",
"src": "Patient: Hi, We are trying to conceive but now my husband has what seems to be balatinitis or a yeast infection and I wonder if it is safe to continue trying? The skin on the top of his penis is red, sore and sometimes itchy. He will see the doctor tomorrow, but I would like to know if it is safe to try to conceive while he is being treated for this?Thanks. Doctor: HelloThanks for query ,Based on what you have poste I would state that your husband has what is called as Muco Cutaneous Genital Candidiasis probably due to uncontrolled Diabetes .In presence of infection it is advisable preferably not to have sex to avoid infection being transmitted to you and impending complications to the fetus . Get him checked by Urologist or Physician and plan the conception only after his infection is well controlled .Dr.Patil."
},
{
"id": 148035,
"tgt": "What is the cause of lightheadedness, unbalanced feeling, fatigue, and cold feet?",
"src": "Patient: Hello, I am Ryan Concepcion, 26 years, male. I would like to know what could be the reasons behind my chronic lightheaded-ness (whole day), unbalanced feeling, feeling faint, cold feet, and fatigiue. I am taking Stugeron for the motion sickness, Nexium and gasmotin for my GERD and Multivitamins.. A couple of months back, I was rushed to the hospital a couple of times due to a holiday heart syndrome. I was woken up a couple of times from my sleep due to numbness in both of my hands. Sometimes its just my pinky, at times my whole hands, either or both of my hands. I have been feeling detached from my body, which gives me a sense of blankness in my body. I feel as if I don't have full control of my body. I am very scared that I may have cancer or some sort of terminal disease. I also do have lumps in my neck. One in about 1 cm under my chin, and another one under the right side of my jaw, 2 cm. Both are wiggly, but are fixed. My EENT told me that it is nothing to be worried about. Should I see a neuro for this? or a cancer specialist? cardio? All of my test came back normal. CBC, Holter, Stress test, abdomen ultrasound. I am happy that they're all okay, but none of these test are invasive or aggressive enough to have an in depth look on my body. I just hope that my unbalanced feeling could just be due to some sort of problem in my eyes or ears. Im afraid of having cancer since my mom had one before. Thanks for your time. God bless. Doctor: HIThank for asking to HCMI can understand your problem, what ever happening to you or the symptoms that you have stated here are all because of your emotional and functional disorders, you are thinking in negative ways and that gives you weird kind of ideas, in fact this is your functional condition, and nothing wrong is with you, just take it easy, and no need to worry about anything, hope this information helps you take care and have good day."
},
{
"id": 68488,
"tgt": "How to treat a forehead lump?",
"src": "Patient: My 9 year old daughter has a bump on her right temple. It showed up a week and a half ago. She has been to urgent care who thought she should use a cold compress and its probably bug bite. Week later took her to her pediatrician and she said thought it may have been bug bite that got infected and she is now on Suprax. Pediatrician also said bump could result from overlap where cranium connects together...but that is usually seen in younger kids. Bump is prob size of nickel. Bump is not red and doesnt hurt when you apply pressure. It used to hurt her when we first noticed it. She weighs about 70 lbs and is four foot 8 inches tall. Thinking I need to make dermatologist appt now because it takes at least a month to get to see dr. Is dermatologist correct specialist to see? Doctor: HiIt can be a Dermoid cyst or Sebaceous cyst. Please consult your Surgeon and get examined clinically. It can be removed surgically. Nothing to worry. Wish her good health. Regards"
},
{
"id": 95705,
"tgt": "I have been having stomach hurting. what could it be ?",
"src": "Patient: Hi there I have been having stomach hurting for over a week:( what could it be ? Doctor: Hi, Could you provide further information about your symptoms? Where exactly are you experiencing these symptoms? Is there any association with food? Any sour taste or burning sensation in stomach area? Thanks."
},
{
"id": 215413,
"tgt": "Is Cortizone advisable for shoulder pain?",
"src": "Patient: hi i reijured my shoulder last tuesday on a bicycle and in falling landed on the same area of my right shoulder that previously injured 4 weeks back on the impact of hitting the pavement falling hard on rollerblades during execution of 360 spin had xrays taken thursday so far no word, figuring could have pulled tissue muscle and cartilege,and the rotator cup is omewhat banged up ive got a heating pad and some trammedol,would cortezone help this pain? Doctor: Hello, Cortisone injections often help inflammation. if it is predominantly damage, the cortisone might not do much. If it is mainly JUST inflammation, cortisone at worst works well for 2 to 4 weeks and often it just fixes everything. Hope I have answered your query. Let me know if I can assist you further. Take care Regards, Dr Matt Wachsman, Addiction Medicine Specialist"
},
{
"id": 134106,
"tgt": "What causes pain in the hands?",
"src": "Patient: My thumb, and the top of my left hand from from my lower thumb to my wrist and extending slightly up my wrileft arm-appropimately 5-6 inches hurts. It just started hurting yesterday. I am left handed. I can t recall anything I did to have cause any injury. A vein that runs along that area seems a bit swollen. When I attempt to use my hand for even the simplist thing, it hurts and is also tender to the touch. Any idea what might be causing this? Dona from Va. Doctor: hi,thank-you for providing the brief history of you.As you mentioned the pain is in the left side, from the base of the thumb to 2-3 inches towards the elbow. The area is painful to touch as well.As seeing your symptoms what I can understand is to see two possibilities.1. De Quervain's Tenosynovitis2. cervical spine disc degenerative changes.As you mentioned that you are left handed by nature to perform all activities by left hand the above mentioned two provisional diagnosis comes in mind1. De Quervain's Tenosynovitis - A test called the Finkelstein test can help confirm de Quervain's tenosynovitis. To do this test, you bend your thumb down across the palm of your hand, and then cover your thumb with your fingers. Next, bend your wrist toward your little finger. If this causes pain, you likely have de Quervain's tenosynovitis2. Cervical spine disc degenerative changes - The Spurling's test (also known as Maximal Cervical Compression Test and Framonial Compression Test) is used during a musculoskeletal assessment of the cervical spine when looking for cervical nerve root compression causing Cervical Radiculopathy.Following this two test we can rule out one and diagnosis is close by. Also taking an MRI of the wrist or cervical spine based on the positive test either of the above will help guide confirm the diagnosis.Treatment -1. De Quervain's tenosynovitis - physical therapy can help you with the therapeutic ultrasound therapy, phonophoresis, TENS and exercises.2. Cervical Spine disc degenerative changes - Physical therapy with Ultrasound therapy and TENS therapy initially later performed exercises.I wish you a good healthfor more questions you can ask me and I will be happy to you.Regards.Jay Indravadan Patel"
},
{
"id": 171653,
"tgt": "Suggest treatment for pergo virus infection in infant",
"src": "Patient: sir my wife has delivered a baby in a nineth month that is 21 days before expected date of delivery. When child was born his body was white and there were blood clots on whole body. during delivery he cried and done all the normal acts. Immediately after delivery docter had sent his blood for clinical diagnosis, his hemoglobin was only 3%. his heart was pumping very fast. Imidiately blood was transfused in his body while old blood in his body taken out. In whole blood of the body was changed after it his hemoglobin was 15%. Now doctors after so clinical checkups and pathological investigations they are saying that it was pergo virus infection. if it is right than what i have to do save his life in future Doctor: Hi,I think i can help to get certain facts and terms straight .What your wife had is most likely a PARVO VIRUS B 19 infection while pregnant ,may be from some contact with children with the infection.This is a viral infection in children which is usually mild with red cheeks and lasts for 3 to 5 days and goes away .if your wife works with kids in a nursery she may have picked it up.and if she is not protected with antibodies ,the virus from her blood goes to the baby and stops baby inside the uterus from producing red blood cells .This caused anemia in the baby and needs immediate management with blood transfustion with packed red blood cells-----.in your case the baby had gone to further complication of reduced platlets ,thats why they exchanged ,whole blood .The good news is that it is self limiting disorder and will not cause any problems in the child,his groth of development or chronic anemia,"
},
{
"id": 25318,
"tgt": "Is a heart rate of 97 to 106 at rest serious?",
"src": "Patient: Hey Just wondering. I am a 39 year old female, pretty smallMy heart rate is around 97 to 106 at rest, and my BP is 140/90 and twice I get 175/118I did visit a doctor because I didn't feel well past month or twoThen the doctor gave me a heart medicine to take the pulse down and BP (its still in further examination.)But I wondering what it can be? Do I need the medicine really? Is that bad ? Doctor: Hello and thank you for using HCM.I carefully read your question and I understand your concern.I will try to explain you something and give you my opinion.We talk about hypertension if we have mean value that exceeds 140 / 90 mmHg.A person might have high value during emotional and physical strees so its mandatory to judge on mean values.Usaly hypertension does not give any symtoms but left untreated he slowly modifies the heart. According to heart rhythm, the normal rate is between 50-100 beat for minute.When it exceed 100 we talk about sinus tachycardia. This might have different causes to simple emotional stress, physical activity, coffee consumption or pathologys like anamia, hyperthyroidism.So if we diagnose hypertension and rhythm issue we have to find tha cause and of course treat them.If you treat dhe hypertension than you have nothing to worry. If I was your treating doctor I will recommend some examination like an electrocardiogram, a cardiac echo, a full blood analyze, a holter rhythm and pressure monitoring.This gives a beter view how to treat the problem, medicaly or not.But as you catch values up to 170 I think medical treatment is necessary. Hope I was helpful.Wish you good health. Best regards."
},
{
"id": 191081,
"tgt": "Red patch on roof of mouth and had a cold and sore throat",
"src": "Patient: I have had a cold and sore throat for a few days now. Cold has nearly gone apart from a slight cough. However the sore throat seems to have come back (only lasted a day). I today noticed that I have a few small red patches on the roof of my mouth. They dont hurt and only appeared today. They look like burst blood vessels. I am worried about them. Doctor: viral infection with ulcers in the mouth, it will defenitely take time for to heel though it is npt making any symptoms for the time being it will make some noise in shortly. wash mouth with some mouth washes every day twice or thrice. take a multivitamin tablet once in a day"
},
{
"id": 2449,
"tgt": "What are the chances of successful pregnancy after ectopic miscarriage?",
"src": "Patient: my wife had a ectopic miscarriage resulting in removal of left fallopin tube, later we underwent surgery for removal of fibroid with size in excess of 7 cms. its been 4 months now, what are the chances for a sucessful pregnancy and some tips to decide the important dates for trying Doctor: There are almost fifty percent chance.. As ovulation occurs alternatively every month.. So nothing to worry"
},
{
"id": 194797,
"tgt": "Suggest solution for pre-ejaculation fluid like discharge after bowel movement?",
"src": "Patient: iam 40 years old. For the last one year, iam noticing that a small stain on my under wear which appears to be pre ejaculate fluid - the clear egg white kind of substance. This usually seems to come out some time after I clear my bowels . I have problem with my bowel movement in the sense that I usually have to go to the loo twice or even thrice a day due to some kind of constipation or inability to pass the full stool at one go - I also have lot of gases in my stomach. Perhaps I exert too much to pass motion and maybe because of this , the preejaculate comes out . But this leaves my underwar dirty. How do I prevent this from happening. What medicines can I take Doctor: Hello, Presence of pre-ejaculatory fluid or even semen after urination is a common occurrence and there is nothing to worry about it. You must worry if there is any burning sensation, pain or itching on or around the penis. Hope I have answered your query. Let me know if I can assist you further. Regards, Dr. K. V. Anand, Psychologist"
},
{
"id": 154377,
"tgt": "Does fluid in heart and lung indicate metastases of oesophagus cancer?",
"src": "Patient: Hello, my mother-in-law was treated for cancer of the osephagus a year ago successfully however she has just been re-admitted to hospital with fluid in the heart and lung. Does this mean that the cancer has spread? if so, is it possible for her to have further treatment she is 68 years old and obviously the initial treatment was very invasive? Thank you for your assistance. Doctor: Hi, dearI have gone through your question. I can understand your concern. she has pleural and pericardial effusion. It may be due to cancer or may be due to some other cause like low protein or infection. She should go for pleural tapping and examination of pleural fluid. It will give you exact diagnosis whether it is due to cancer or some other cause. Then you should take treatment accordingly. Hope I have answered your question, if you have doubt then I will be happy to answer. Thanks for using health care magic. Wish you a very good health."
},
{
"id": 114465,
"tgt": "What causes low hemoglobin levels?",
"src": "Patient: My husband has had low hemoglobin for about 6 mos. No one can seem to find where he's bleeding inside.. Also has congestive heart failure.. I think the blood prob is making him so weak.. He 's been in out of hospitals & rehabs for 6-7 mos. Nothing is helping.. Any ideas of what to do I'm really stressed & worried.. Doctor: HiLow haemoglobin levels are caused byA slightly low hemoglobin count isn't always a sign of illness \u2014 it may be normal for some people. Women who are pregnant commonly have low hemoglobin counts.Low hemoglobin counts associated with diseases and conditionsA low hemoglobin count can be associated with a disease or condition that causes your body to have too few red blood cells. This can occur if:Your body produces fewer red blood cells than usualYour body destroys red blood cells faster than they can be producedYou experience blood lossDiseases and conditions that cause your body to produce fewer red blood cells than normal include:Aplastic anemiaCancerCertain medications, such as anti-retroviral drugs for HIV infection and chemotherapy drugs for cancer and other conditionsChronic kidney diseaseCirrhosis (scarring of the liver)Hodgkin's lymphoma (Hodgkin's disease)Hypothyroidism (underactive thyroid)Iron deficiency anemiaLead poisoningLeukemiaMultiple myelomaMyelodysplastic syndromesNon-Hodgkin's lymphomaVitamin deficiency anemiaDiseases and conditions that cause your body to destroy red blood cells faster than they can be made include:Enlarged spleen (splenomegaly)HemolysisPorphyriaSickle cell anemiaThalassemiaVasculitis (blood vessel inflammation)A low hemoglobin count can also be due to blood loss, which can occur because of:Bleeding from a woundBleeding in your digestive tract, such as from ulcers, cancers or hemorrhoidsBleeding in your urinary tractFrequent blood donationMenorrhagia (heavy menstrual bleeding)Causes shown here are commonly associated with this symptom.The treatment of the anemia varies greatly. First, the underlying cause of the anemia needs to be identified and corrected. For example, anemia as a result of blood loss from a stomach ulcer should begin with medications to heal the ulcer. Likewise, surgery is often necessary to remove a colon cancer that is causing chronic blood loss and anemia.Sometimes iron supplements will also be needed to correct iron deficiency. In severe anemia, blood transfusions may be necessary. Vitamin B12 injections will be necessary for patients suffering from pernicious anemia or other causes of B12 deficiency.In certain patients with bone marrow disease (or bone marrow damage from chemotherapy) or patients with kidney failure, epoetin alfa (Procrit, Epogen) may be used to stimulate bone marrow red blood cell production.In certain patients with bone marrow disease (or bone marrow damage from chemotherapy) or patients with kidney failure, epoetin alfa (Procrit, Epogen) may be used to stimulate bone marrow red blood cell production.If a medication is thought to be the culprit, then it should be discontinued under the direction of the prescribing doctor.RegardsDR DE"
},
{
"id": 7172,
"tgt": "Diet to increase my chances of getting pregnant",
"src": "Patient: as u know I am going through folicular study as asked in last question. hat should I do to increase my chances of getting pregnent. I mean any food or drink or more water or any excercise or what else should i try or take care for increasing my chances of getting pregnent Doctor: Thanks for the query There is no specific food item that could increase your chances of fertilization but please do avoid unhealthy practices like smoking drinking and also studies had proved papaya seeds are detrimental Have a healthy living"
},
{
"id": 102402,
"tgt": "Is there any natural remedy for pilonidul sinus?",
"src": "Patient: I am 30yrs male from India.i have pilonidul sinus for morethan 2 year and just before 8 months i have removed the abscess and it worked well for these 8months. Now the pain is high and puss coming over. I dont want to cut that area and suffer for 45 to 50 days by doing gauze over ther. Is ther any natural remedy or any other way to get rid of this disease? Pls suggest Doctor: Hello,Welcome to HCM,The infection of the hair root at the tail bone can lead to this sinus, this area is very sensitive because the skin as lot of sebaceous gland and the skin is very soft.There will be lots of sweat production in this area,prolonged sitting or obesity or poor hygiene of this area will cause all these problems.Only solution is open that area and drain all the pus which is present in that area and sutured that area. If you try the nonsurgical treatment like antibiotics and sitz bath it may not be complete treatment and once again it may recur.Thank you."
},
{
"id": 140599,
"tgt": "Suggest treatment for pain in upper arm and spine",
"src": "Patient: I have pain in my left upper arm. There is some pain in upper spine also. Seen doctors several times. They advised physiotherapy (Hot pack, IFT and exercise ) and muscle relaxant ointments and Etoshine MR tablets. Pain reduces a bit after physiotherapy for 3-4 weeks but again reappears. Need some advice. Doctor: Hello, Since the symptoms are getting recurrent a correct diagnosis of your condition is necessary. This way treatment ways should be clear. Degenerative spine disease, herniated disc disease, spondylolisthesis, etc. are conditions to consider in cases like yours. A neurological examination and cervical spine MRI are necessary. Hope I have answered your query. Let me know if I can assist you further. Take care Regards, Dr Erion Spaho, Neurologist, Surgical"
},
{
"id": 430,
"tgt": "What causes problem in conceiving?",
"src": "Patient: Yes, want to ask a question I've always wanted!! Well, my question is my girl and me have been trying to have a baby. and she has been having some pregnancy symptoms for he past three weeks. And last week we went to see if she was pregnant. But, they told us she was'nt. And we just want to know why is that??? Doctor: Hi and thanks for your query to healthcare magic.I have read your question carefully and understand your concern.If her cycle is regular you should try at least one year on her fertile days .If you will not conceive during this time than you should see a gynecologist.Your gynecologist will suggest to do:a transvaginal scana pelvic exama semen analysis(for you)During the time you are trying she should take folic acid 400 micro gr per day and be calm.Hope I have been helpful"
},
{
"id": 219431,
"tgt": "What does pink discharge in early pregnancy imply?",
"src": "Patient: Hello. Well my husband and I have been trying to conceive for almost 3 months now. I\u00c3\u00a2 ?m usually regular with my menstrual cycle and it usually is light the first day the heave second and third day then light 4 day then gone, well I was supposed to get my period this month on the 5th February 5th which I did, but it was weird . First day was spotting then second day was light and then nothing on the 3rd day. Then it stopped for like 5 days then came back very light pink. I tested for pregnancy on the 18 still with the light pink period and I got 3 positive results, but it\u00c3\u00a2 ??s February 22 and I still have the light [pink discharge. What could this mean? Doctor: Hello,I have gone through your query and understood the concern. Since you have had three positive pregnancy tests, you are pregnant. You can get the status of the pregnancy through a pelvic/trans-vaginal sonogram. Usually, implantation bleed is seen at the time of the expected period and is usually a fresh, light bleed which subsides in a couple of days. Persistent spotting is usually observed when there is a hormone deficiency. If the pregnancy is progressing normally, you are likely to benefit from progesterone supplements, absolute bed rest and abstinence. Hope your query has been answered. Take care."
},
{
"id": 205372,
"tgt": "What causes breathlessness after taking Lexapro?",
"src": "Patient: I have been feeling nauseous and also yesterday I almost passed out. Usually when that happens I lie down and the dizziness goes away, but it took awhile for the feeling to leave. I have also been having these moments throughout the day where I get shortness of breathe and sometimes it is so hard to just breathe. I do have anxiety and depression and about two weeks ago stopped taking my medication which was Zoloft because I felt like it wasn t helping much and was prescribed a new medication(lexapro) and I took it and had a hard time breathing and had this horrible feeling like I was going to die so I decided to wait a few days and I m not sure I should start it because I feel the same symptoms I did when taking it. What should I do and what is wrong with me? Doctor: breathlessness you have described is typical and common symptom with Anxiety. I will suggest you to keep using Lexapro as it take 1-2 weeks before it starts showing its anti-anxiety effects. Also very few cases patients may experience worsening of breathlessness but in my opinion its your anxiety that is causing you difficulty with brathing. You consider visiting your treating psychiatrist to check for paradoxical anxiety with lexapro and can ask if he would like to start you any benzodiazepine for you."
},
{
"id": 92942,
"tgt": "Have pain in the abdomen. Am fatigue. Blood work, scan no problem. Colonoscopy scheduled. Suggestions?",
"src": "Patient: I have been experiencing a dull mild pain in my lower left abdomen for weeks and this past week I have also been experiencing fatigue and feel like I have a fever since I feel hot, but no temp. My bloodwork and ct scan did not show anything.I have a colonoscopy scheduled, but I am very worried. What could I have and what should I do? Doctor: HI Thank for choosing HCMIf your blood test, scanning test are negative then there is strong possibility of ulcerative colitis and the colonoscopy is the only way to confirm it, if turns negative then go for stool examination this will clear the picture, or barium meal study, if every test shows no abnormality then this could be anxiety depression related problem, Have nice day."
},
{
"id": 54761,
"tgt": "Need treatment for liver disease with positive ANA test",
"src": "Patient: received an ANA test that was positive. The results say that it is a speckled pattern seen is nucleur dot and this pattern has been associated with liver disease. My titer is 1:1280 or greater....received an ANA test that was positive. The results say that it is a speckled pattern seen is nucleur dot and this pattern has been associated with liver disease. My titer is 1:1280 or greater.... Doctor: Hi thanks for contacting healthcare magic...Noted you have ANA positivity....So it might be SLE like autoimmune disease.For its confirmation do anti ds DNA study...SLE can affect liver lead autoimmune hepatitis.Liver enzymes study with USG done for it...Immunosuppresent drug can be taken after consultation from doctor....Meanwhile rest liver with low fat diet.Fruits more.Drink distilled water daily...Cooked oatmeal and yoghurt good remedy.Take care.....Dr.Parth MD"
},
{
"id": 11248,
"tgt": "Suggest treatment for hair loss",
"src": "Patient: Hi My Name is Imran Khan. I am 29 Years old. I am suffering with Hair Loss since last 6 Years.Not completely bald but .Hair density is less. I have taken the treatment in Dr Batra's for almost 18 Months. But I didnt find any change in my hair loss. Is there any treatment to to re growth my hair. I am currently Using Follihair Tablets since last 15 Days. Please suggest me. Doctor: Hi,You said you suffer from hair fall for about 6 years. Hair loss may be diffuse. There is no complete baldness. And you took treatment from Dr. Batra Homeoapthic clinic without response. I think there may be family history of androgenetic alopecia in paternal or maternal side. Most probably you may be having initial stage of androgenetic alopecia.please consult nearby dermatologist and have perfect diagnosis and treatment. Still, I would like to suggest as follow..- tab finesteride 1 mg daily in morning- biotin tab 10 mg- vitamin E cap 400 mg- apply minoxidil 10% solution twice a day- rule out internal diseases like thyroid dysfunction,anaemia or otherHave patience for the good result.I hope you got my answer.Thanks.Dr. Ilyas Patel MD"
},
{
"id": 70662,
"tgt": "Is minimal PTB in a chest X-ray a cause for concern?",
"src": "Patient: Hi there! I have a question. I had a previous finding in my chest xray back Feb 2010, it was consider granuloma, right upper lobe. I had treatment for 6 months as my doctor told me that it was consider as PTB. Then recently, on March 16, 2016 I had my chest xray because I was coughing for a week. It says consider minimal PTB vs. non-koch s pneumonia. I was confused because the patch of density he saw is same as before right upper lobe . Is it the scar he saw or posibly it comes back? Doctor: Hello, As you explain the history I don't think it is a the again. Usually after a lung TB remains a nodus and it is a common finding. Hope I have answered your query. Let me know if I can assist you further. Take care Regards, Dr Jnikolla, Pulmonologist"
},
{
"id": 198030,
"tgt": "What is causing chronic testicular pain when reports are normal?",
"src": "Patient: HI doctor Iam 27 year old i had chronic testiculer pain (right side) which come time to time my testiculer ultasound and doppler report was normal I did semenal fluid analysis it was normal apart from it contian few RBC and ++ pus ((best regard)) my email adress( YYYY@YYYY )) Doctor: HelloThanks for query .You have repeated episodes of pain in Rt testicle and your semen analysis has revealed presence of pus cells and Rbc in semen .In view of chronic pain and seem analysis findings the pain could be due to Chronic infection of testes ( Orchitis ).Get semen culture doen to trace out the organisms causing this infection and antibiotics to which they are sensitive to so that you can take appropriate antibiotic as per culture report to eradicate infection .Start taking combination of Trimethoprim and Sulphamethaxozole along with anti inflammatory medicine like Diclofenac twice daily .Later on switch on to appropriate antibiotic as per culture report .It should get cured within a month .Dr.Patil."
},
{
"id": 213102,
"tgt": "Exhausted. Sleep insufficient. Why am I having mood swings?",
"src": "Patient: I m a 21 year old girl and normally a very happy person and ready to go to my classes (and not waste my money) and wake up feeling good. Lately though i have been feeling like no matter how much i sleep it s never enough i feel so exhausted it s causing me to get to where i just want to lay down and try to sleep and miss class. I m also having mood swings. I never get like that normally. I ll be fine and then bam I m rude or upset. it s stressing me out to where i spilled something on my shirt and cried. that is DEFINITELY overreacting and odd for me, for anyone. I have no appetite and when i do eat it s hardly anything before it starts to make me feel really sick and i have to stop. idk what it could be and it s starting to really bother me. Doctor: hi..well, I think you have undergone through some stress of late..due to that you are facing a depressive episode..for that, there is insufficient sleep and loss of appetite..so please consult a psychiatrist..its curable..you will be fine..thanks.."
},
{
"id": 178337,
"tgt": "What causes swelling and blisters around lips of my daughter?",
"src": "Patient: my daughter is experiencing sever swelling of the lips, and parts of her gums for the past 6 days. There is light blistering and redness around the outer edge of the lips. We went to emergency and doctors thought it is allergy related but Benadryl and Reactine only works intermittently. Doctor: hello, thanks for your query, it's unlikely allergy medications ( Benadryl) causing allergy. check out her complete blood picture , viral infections with pediatrician. hope I have answered your query. all the best. take care."
},
{
"id": 217513,
"tgt": "Suggest treatment for shoulder, right thigh and hip pain",
"src": "Patient: i am 36 years male. i am suffering with shoulder pain along with sever pain in heels. also pain in right hip and in thighs and crotch on same side. Blood test shows ESR 106 pus cells in urine upto 30. Chest x ray is normal. I am on pain killers for the last one month. Doctor: hilooks like you are suffering from urinary tract infection with severe body pains.take plenty of oral fluidsplease go ahead with your urine culture to know the organism & drug sensitivityESR :106 & pus cells: 30 in urine indicates you are having the infection since long time.IT IS NOT ADVISABLE TO TAKE PAINKILLERS UNTIL THE CAUSE IS TREATED.all the besttake care"
},
{
"id": 2541,
"tgt": "Suggest remedy for getting pregnant",
"src": "Patient: Hi Dr. i m 24 yr old. i got married in Dec 2014. me and my husband are planning for a baby from last 5 months. We are consulting a Dr. from last 4 months, but still no positive result. he had advised me to take FOL 123,& i m taking it daily after breakfast from last 2 months. i got my period on 12th Aug, 2014 and as per my dr advice i have stared taking Goodova capsules(for 5 days - 13th to 17th Aug). after that on 23rd Aug(12th day aftert my period) he has called my for some treatment. My question is what will be this treatment for? & what are the effects of Goodova?. Will i get pregnant? me & my husband are depressed. kindly help? Thanks & Regards, Doctor: hello..thanks for trusting the healthcare magic doctors for ur health related queries.I guess ur concern is about..remedy for getting pregnant..right.let us discuss ur queries one by one..1.what Wil be this treatment for- u told me that ur doctor prescribe u goodnova for 5 days. this drug is usually given for ovulation induction( that means it helps follicles in the ovary to grow), so that ovulation(during this time follicle rupture n ovum comes out), so that sperm can fertilize the ovum n pregnancy will happen.2. effects of goodnova- as I discribe above, it is given for ovulation induction.3. will you get pregnant- my answer is..OF COURSE, ..YES..U WILL BE GET PREGNANT. because u come under reproductive age group(15-45 years aged women are come under this group), in this group chances of getting pregnant is high.so there is chances.if u have regular menses, then u will have DOUBLE CHANCE OF GETTING PREGNANT. but u didn't give info about the regulatory of ur menses. so it is difficult for me to comment on this.with above discussion u can try out my suggestions-1. DON'T WORRY..RELAX, I WILL GET PREGNANT FOR SURE..JUST KEEP PATIENCE.2. for better results, try to do coitus during fertile period(it's a time from 10th to 16th day of menses) during this period chances of getting pregnant is high.3. I hope u had gone through all investigations of yours and ur husband. if everything is ok, then u have highest chances of getting pregnant.4. if u r still in hurry then visit to nearby infertility specialist. he/she can help u.I hope the above discussion is informative enough, useful and helpful for u.regards- Dr sudha rani panagar( i can understand ur tense-full situation..DON'T WORRY..KEEP PATIENCE..u will be getting pregnant soon. I pray the God that this ovulation induction cycle will be successful n u blessed with a cute n healthy baby. ..hopefully..TWINS)"
},
{
"id": 165606,
"tgt": "What causes headache and vomiting in a 9 year old?",
"src": "Patient: Hi, may I answer your health queries right now ? Please type your query here...My 9 year old child had a bad headache last night and after an hour or so vomited. He eventually went to sleep. He had a school camp to go to starting tomorrow. If it is a vomiting bug I shouldn t send him but if it was caused by a headache/migraine I would. How do I know the difference? Thanks. Doctor: Dear parent,migraine must be episodic,is ur child suffering from such episodes frequently.i mean headache followed by vomiting and after vomiting headache is relieved.if such episodes are frequent then it might be migraine.u can send him to camp giving him some paracetamol tablets for pain and antiemetic for vomiting."
},
{
"id": 130366,
"tgt": "What could be the indent on the top of my arms?",
"src": "Patient: On the top of both of my hands (opposite of the palm side) and near where the hand is connected to my arm there s a little indent, most easily felt when I move my hand down. It s on both sides and has always been there-never a problem and seemed normal. But the one on my right side really hurts. Hurts to move hand-especially downwards, or use any force to move it. It also hurts to apply pressure to it. There is no bruising or swelling at all. And it s most likely not a cyst because I have had two in my foot and one in my wrist and it s not a normal spot and it doesn t feel like a cyst and there s not even a tiny bump. What could this be? Do I need to go to the doctor, or will it go away in a few days? Doctor: you could be suffering from injury to wrist ligament called a tfcc injury. it is usually following a twisting injury or strain of wrist. if the pain is not subsiding even after 2 weeks of rest to the wrist better consult an orthopedician"
},
{
"id": 200766,
"tgt": "What does a circular red and white spot on scrotum indicate?",
"src": "Patient: Hi there. On my scrotum sort of close to the base of the shaft I have a circular red and white dot that I have had for over a year now .. I went to a doctor before and she said it was like a ingrown hair which I think it has a good possibility of being as well. It started as a hair I believe that I seen had a little ball and i squeezed it and pus came out and then it just grew much larger. When I squeeze it hard white puss comes out all stringy, but it is painless and just is a bit red and looks like a white ball above skin. Just wondering if I should go get it checked again as its been a year and its been the same for about 11 months. Doctor: Thanks for asking in healthcaremagic forumIN short: May be a cystExplanation: By your explanation it seems to be a sebacoeus cyst. Sebaceous cysts are harmless unless infected. Please visit a dermatologist for its removal, do not try yourself. Hope you understand ."
},
{
"id": 198687,
"tgt": "What causes clear fluid in the penis with erectile dysfunction?",
"src": "Patient: Dear Sir, I have some sexual problem first every time some little drops of dirty water come out side and also my pennis is not erect its very lose now kimdly help me in this matter howcan i stop dirty drops of water and how i can erect my pennis . help me kindly. Doctor: Hello I share your concernDirty drops you are talking about might be caused by discharge from urogenital tract.At the outset I would like to know about your age and relevant medical historySince how long you have this problem?For dirty drops and discharge I would advise you for a battery of tests including Blood test for hemoglobin, CBC, liver and kidney function test or urine microscopy a swell as urine culture and sensitivity test.You may require a course of antibiotics depending upon the reports.Erectile dysfunction might be related to these dirty drops or discharge we need to identify and address the underlying cause for erectile dysfunctionApart from behavioural therapy and counselling medicines are available for erectile dysfunctionif you can write back to me with these details so that I will be able to tell you more accuratelybest wishes"
},
{
"id": 163882,
"tgt": "Can epilepsy cause over heating of body?",
"src": "Patient: My son gets over heated extremely easy, this happens even when it s cool outside. He s not dehydrated and he is dressed appropriately. Is this a disorder or disease. My son was also just recently diagnosed with epilepsy, could this just be a coincidence? Doctor: Hi... I understand your concern. Overheating and epilepsy are not related. But the symptom of overheating you are telling exactly fits into hyperthyroidism. I suggest you get in touch with a physician or your child's pediatrician and get him evaluated for thyroid issues.Regards - Dr. Sumanth"
},
{
"id": 154279,
"tgt": "Would abdominal CT scans show umbilicus tumor?",
"src": "Patient: My mom had small bowel cancer in August. They did a complete tumor resection and now she has pain in her lower left abdomin. They did bone scan and ct scans multiple and didn't see anything. Yesterday she went to see the surgeon and he didn't like the way her belly button area is swollen and sore. He thinks there may be a lump there and wants to remove it. Would abdominal ct scans show a cancerous tumor by her belly button or is it more likely a surgerical hernia? Is the belly button surgery difficult to recover from? She's 83 years old. Doctor: Hi,Thanks for writing in.An abdominal CT scan is a detailed investigation and will surely show a lump in the umbilical region. CT scan takes detailed images using a concentration of X rays using a computer. In this way the scans make thin sections through the abdomen and then these studied for any abnormality.If there is a lump that appears more with increased abdominal pressure then a surgical hernia is more likely. This happens when the muscles in the anterior abdominal wall are thinned out and separate or in the site of a previous surgery. Since she is having soreness therefore it can be a hernia. A CT scan will tell a hernia from a cancer lump. Please do not worry."
},
{
"id": 191986,
"tgt": "Suggest treatment to control blood sugar",
"src": "Patient: Hi My husband had some blood work done about 2 months ago and the test showed that he was a border line diabetic. He has heal pain in one foot now, that seems to increase while eating sugar. Can this go away with diet alone or should he go see a Dr/ Doctor: thanks for posting your query in HCM,AS per your narration your husband is borderline diabetic. complications of high sugar takes some time to develop depending on patient characteristics. eating sugar getting heal pain at that time may be coincidence. please consult your doctor for heal pain and make sure your husband is doing excercise regularly and reducing carbohydrate intake. get his sugars checked after 3 months thank you"
},
{
"id": 3715,
"tgt": "What are the chances of getting pregnant from unprotected sex?",
"src": "Patient: 1st day of my last menstrual period is june 1..my cycle usually is 30-32 days..then we had unprotected sex on june 13 and 14, using pullout method..on june 15 i took nordette as ecp..i then experienced nausea and abdominal cramps..and as of today june 19, i still don t have withdrawal bleeding..what are my chances of getting pregnant?is nordette still effective by the time i took it?thank you very much!will be happy to hear something from you.. Doctor: Hi, I understand your concern. You had used pull out method ( which is not reliable 0 on 13th & 14th of June. If your cycles are regular, estimated day of your ovulation would be on 17 th of June.this leaves chance of conception if the pull out method was not proper. -Nordette is a contraceptive for regular use & not for emergency use.A very high dose would be required for it's emergency use. Such high dose can cause nausea.. & withdrawal bleeding will be any time before 8-10 days . * If you don't get withdrawal, & found to be pregnant.. you need to terminate the pregnancy( as nordette is known to causefetal malformations.) Thanks."
},
{
"id": 27510,
"tgt": "What could be the severe pain in chest and numbness in arm mean?",
"src": "Patient: For almost a week I've had chest pains in the left side. I went to the er after waking up and my arm being completely numb. They did blood work EKG chest X-rays and everything came back clear. But I still have this very sick feeling I'm overly tired even though it feels like all I do is sleep and my left side keeps getting the tingly numb feeling. I'm getting kind if worried its causing a slight headache. Any guess on what this could be ? Doctor: Hi. Thank you for using HCM.Your age would be helpful in this differential diagnosis. Nevertheless, I would recommend a cardiac stress test. My opinion is tat you should also have a plain posterior-anterior and lateral X-ray of the neck. I hope I was helpful. Greetings."
},
{
"id": 85940,
"tgt": "What does throbbing pain in the lower abdomen radiating towards the lower back indicate?",
"src": "Patient: Since last Friday, everytime I eat a full meal, within 20 minutes after finishing I get a throbbing pain through my lower abdomen, radiating around to both of my kidneys. The pain in my back feels like kidney stones (I have had first hand experience with those) and the abdomen pain feels like gall bladder pain (which can t be because I don t have a gall bladder anymore.) The only thing that makes the pain go away is my heating pad. Any ideas as to what might be causing these episodes? Signed Ruth Peyton YYYY@YYYY Doctor: u should go to hospital as soon as possible to do some investigation , u have a gallstone before and the relation between ur meal and pain make me suspect of acute pancreatitis , we should exclude pancreatitis by ct scan so dont wait go to the hospital ,what about ur stool is there any change?"
},
{
"id": 74372,
"tgt": "Can lung be transplanted if suffering from emphysema and COPD?",
"src": "Patient: I am 54 years old and very healthy. My dad is 75 and has emphysema and COPD. He has been on oxygen for about 4 years now but is getting much worse. I want to give him one of my lungs before he declines too much. Is this possible? Would a doctor do this at his age? Doctor: Thanks for your question on Healthcare Magic. I can understand your concern. Yes lung transplant is an option for terminal stages of COPD and emphysema. But you can not donate your lung to your father. You need to register your father's name in lung transplant registry. There is waiting list and they go according to that list only. So consult transplant specialist and discuss all these. Hope I have solved your query. I will be happy to help you further. Wish you good health. Thanks."
},
{
"id": 118877,
"tgt": "Severe back pain, abdominal pain. Diagnosed with PID. Blood in uterus?",
"src": "Patient: mother of two,had severe back pain ,lower back pain abdomenal pain diagonised as (PID) which led t a antibiotics then a procedural on july 20th 2012.since then i havent seen my period and smtimes i feel lower abdomenal pain like cramp as if the period wants to come bt it hasnt.do i still hav t go in for another procedure?latest scan shows thers collectn of fluid/ blood in d uterus Doctor: hiThanks for your queryi can feel your concernsyour PID seems not cured properly.it can complicate further .i advise you to consult your doctor and take proper treatment.you may need another course of antibiotic after consulting your doctor.you can take painkillers like brufen or diclofenac for your painkeep good hygienewishing you speedy recoveryi am available for your help regardsDr.Amna"
},
{
"id": 1543,
"tgt": "Why is laparoscopic surgery advised if unable to get pregnant?",
"src": "Patient: Hi,i am 28 yrs old, got married 4 yrs back, from 2 yrs we planning for kids but till now no positive response. My doctor took all tests for me and my husband all were normal. Now she is suggesting for Laproscope surgery, So what is laproscope surgery? If i go for it will i conceive? Doctor: hello...thanks for trusting the healthcare magic doctors for ur health related queries.I guess ur concern is about..why laparoscopy is advised if unable to get pregnant..right.well...laparoscopy is advised in most of the patients who are not having baby, now a days.in most of the cases laparoscopic surgery is helpful n fruitful.let us discuss about the laparoscopic surgery. in this surgery, by help of laparoscopy doctor seeing the internal organs of female like uterus,ovary, tubes, whether they are normal or not. in case of patients having ovarian cyst,it will drilled out by help of laparoscopic instruments. this procedure is called laparoscopic ovarian drilling (LOD). along with this doctors usually do tubal latency test. in this test dye is injected into the uterus. when there is spillage of dye from both side of fallopian tubes, that means tubes are patent.in most of the cases women will get pregnant after laparoscopic surgery. so DON'T WORRY. u can go through it. it might be helpful for u in getting pregnancy.with the above discussion u can try out my suggestions-1. RELAX..RELAX. laparoscopic surgery might be helpful for u also in getting pregnancy. so in my advice u can go through it.2.feel free to ask ur treating doctor regarding this matter. He/she can help u.3. u r come under reproductive age group(age from 15-45 years came under this group, in this group chances of getting pregnancy is high. so u have chances of getting pregnancy.4. try to do coitus during fertile period(it is from 10th to 16th day of menses during this period chances of getting pregnant is high). so do coitus as more as possible during this period.I hope the above information is useful, informative and helpful for u.regards-Dr sudha rani panagar( i can understand ur tense-full situation, DON'T WORRY, EVERYTHING WILL BE ALRIGHT. I pray the God that u will be blessed with a cute n healthy baby..hopefully TWINS)"
},
{
"id": 145734,
"tgt": "What is the treatment for pituitary tumor?",
"src": "Patient: I was recently diagnosed with a pituitary tumor, after going to a neurologist his assistant informed me that it is not a pituitary tumor but it next to it by my brain...The reason I even found out about this is through an MRI I had because of vision issues that I am having (double vision) the thing is this is the third incident it seems to come and go with the first time being in 2009, the second in 2011, both with pregnancies, and now this time while I was on an appetite suppressant. What do you think Doctor: Hi, I had gone through your question and understand your concerns. If you had double vision, the tumor infiltrated cavernous sinus, and treatment depends on the kind of tumor.In that area three are the most probable lesions: Pituitary tumor, Meningioma and Chraniopharyngioma.If the size of tumor is small ( below 3 cm) it could be treated by Radisurgery, if it is greater than 3 cm it needs surgery.Don't delay treatment because there is a serious risk of blindness.Consult a Neurosurgeon.Hope this answers your question. If you have additional questions or follow up questions then please do not hesitate in writing to us. I will be happy to answer your questions."
},
{
"id": 104941,
"tgt": "Cold, running nose, laboured breathing, allergy related symptoms. Will inhalers provide relief?",
"src": "Patient: My daughter is having cold for about 3 months..sometimes it subsides..and again running nose ,stuffy nose,fast breathing ,sounds while breathing,this happens now,,,we consult with docter...he gave inhaler(bude cort and levolin...)is it right way...for her to recover from cold if we continues to give for 2 months..her age is 2 years old and..docter is telling it is due to allergies..is she will recover from cold completely..u please tell me the suggestion...what we have to take care for her... Doctor: Dear Concerned., Thanks for writing to us., The condition that you are describing is called \"Acute Exacerbation\" of Chronic Allergic Bronchitis.This condition of acute exacerbation /acute severe attack is due to sudden exposure to allergens such as House Dust Mite(most of the cases), Pollen Grains,Dust,.The exposure releases a cascade of bio-chemical reactions in the body releasing bronchoconstrictor chemicals from mast cells(histamine and like). the 02 years old child's bronchi are naturally small and when it constricts with filled up secretions the air passage gets narrowed leading to laboured breathing.The resultant sounds are medically refd to as Rhonchi and Rales.The Condition as \"Acute Severe Asthma\"(Childhood onset). The best treatment is to find out the cause.Completely keep the house Dust Free.Nebulise with Broncho dilators and Steroids(Budecortil).Prophylactic Antibiotics. As the child crosses into boyhood , the airpassages size will increase and the child will definitely overcome the problem as the Bronchoconstriction will not have marked effect and there will be no precipitation. The inhalers are better than the Tablets./Syrups and Injections because they act locally and do not produce systemic side effects. Wishing a speedy recovery., Best Regards., Dr Lt Cdr ASN Bhushan, Ex-Navy."
},
{
"id": 199160,
"tgt": "What causes erectile dysfunction?",
"src": "Patient: hi i am doing the masturbation from the past 10 years.m doing the masturbation on daily bases. Now m trying to do with my girlfriend but my penis will not get hard to do the sex. please tell me the reason and also let me know is it a serious problem.m 29 years old. Doctor: DearWe understand your concernsI went through your details. You are masturbating for the last 10 years. That is not a problem at all. Masturbation in moderation is healthy and is a stress buster. Don't worry. Regarding the current sexual experience with your girl friend, it could be due to simple performance anxiety because you may be doing sex as a starter. Don't worry. Without getting anxious and worried, carry on. Your problem should be solved. Or you may consult a psychologist for counseling.If you require more of my help in this aspect, please use this URL. http://goo.gl/aYW2pR.Make sure that you include every minute details possible. Hope this answers your query. Available for further clarifications.Good luck."
},
{
"id": 95035,
"tgt": "Pain in upper abdomen. Hurts badly on sneezing. Spot tender, pinkish in color. Uncomfortable and not getting better. Is it muscle pull while doing yard work?",
"src": "Patient: I have a pain on my right upper abdomen (about three inches over from my belly button and two inches up). The pain started about 10 days ago and it hurts badly when I cough or sneeze if I m standing or laying flat, but is okay if I sneeze sitting down. I was doing yard work with some heavy lifting 14 days ago and vaguely remember jabbing by stomach with the handle of a rake (but I m not positive it happened then). The spot is very tender and now has a slight pinkish color in the area (not a black and blue bruise ). It is not getting any better and is uncomfortable when I sit since my rib crushes that area. Any ideas? I thought maybe I pulled a muscle doing yard work but I don t remember any sudden pain doing it. I m 46 yo female. Thanks! Doctor: hello dear pain in upper abdomen after weight lifting could be a result of muscular tear. if you have fever also may be due to gallbladder or liver infection. hence its better to undergo ultrasound examination to rule out these causes. if it is muscular pain , you need to take complete rest and muscle reaxants. since you have tender,pinkish area, you need antibiotics also. hope this answer is helpful to you take care"
},
{
"id": 136229,
"tgt": "What causes lump under the knee after hitting bike pedal?",
"src": "Patient: I was riding my bike and my pedals swung up and hit the bottom of my knee hard and now on the top of my shin under the knee there a hard lump about 5mm in size and bruises get only on the lump is this a bone fracture or will it heal fast on its own and theres pain with movement and touching it when touching it feels rock hard Doctor: HiWelcome to healthcaremagicI have gone through your query and understand your concern.It seems to be hematoma. It will gradually reduce by due course of time. You can take analgesic such as ibuprofen for pain relief. Vitamins B and C help in healing. It doesn't seem me to be fracture but it can be confirmed by x Ray. You can discuss with your doctor about it. Hope your query get answered. If you have any clarification then don't hesitate to write to us. I will be happy to help you.Wishing you a good health.Take care."
},
{
"id": 158182,
"tgt": "Has unresectable pancreatic cancer, tumor encompassing celiac artery. On radiation and chemo for 6 weeks, due for CT scan. Prognosis?",
"src": "Patient: My husband has unresectable pancreatic cancer toward the tail of his pancreas. The tumor has encompasses his celiac artery. He has had 6 weeks of radiation and chemotherapy. He will have his CT scan next week to see if anything has changed. I know the chances of it being resectable are unlikely. His oncologist will probably put him back on chemo using Gemzar. His previous chemo was 5FU. What would you say is his prognosis? Could the cancer have spread during his \"rest\" period? Is his life expectancy measured in months or years and will Gemzar be worth taking rather than living a better quality of life? Thank you. Doctor: Hi and thank you so much for this query.I am so sorry to hear about what your husband is going through right now. It must be a very difficult moment for him and the entire family. Take heart and accompany him as he battles this cancer.His life expectancy is measured in months. This cancer has a very poor prognosis. The decision to treat or not will come from him depending on what he prefers. I understand it is a very difficult decision to make but then, tough times call for tough measures.Treatment should be focused on relieving him of his pains and ensuring good nutrition> Discuss chemotherapy/radiotherapy and the decision to treat with his physician.Hope this helps. Wish him the best of outcomes.If you have more questions, please feel free to ask us.Dr. Ditah, MD"
},
{
"id": 157351,
"tgt": "Blood test done for testosterone, vitamin D, cholesterol,lymphocytes and monocytes. Could it be cancer?",
"src": "Patient: Hi, I just got my blood panel results back the other day and it had the following results: Testosterone , Total: 410 mg/mL (low) Vitamin D: 23 mg/mL (low) Cholesterol: 113 mg/mL (low) WBC: 2.0 k/uL (low) Lymphocytes: 40.7 % (mid-high) Monocytes: 17.5 % (high) The doctor referred me to a hematologist , but the soonest appointment is next week. I ve had the same types of numbers on previous blood panels (over 10 years) and I m hardly ever sick. I m a 41 year old male, I don t smoke and I don t drink. Should I be worried? Could it be cancer? I m freaking out here. Doctor: from what you have written you dont have cancer and there is nothing to worry also. Cancer cells can be made out on peripheral smear itself. Since you said you have the same type of blood picture 10 years back, it all the more indicates that there is nothing to worry. Cancer usually progresses with time. There is one type of cancer of lymphocytes which progresses very slowly but your blood counts dont suggest that type of cancer also"
},
{
"id": 76091,
"tgt": "What is the treatment for tuberculosis?",
"src": "Patient: hi 15 years prior i was effeted with Tuber closes. I had took the full medicine and cured. but the scar is remain on my chest which is visible on my x-ray, which i am unable to travel in Gulf countery for employment. please guide me waht is the permanent treatment for this Doctor: Thanks for your question on Healthcare Magic. I can understand your concern. You are having old healed scarred lesions in current chest x ray.. And these lesions are due to previous tuberculosis. Actually, almost all lung infections heal by fibrosis and calcification. And this fibrosis will remain as it is through out the life. And this will show on chest x ray as scar lesions. Since these are non infectious, inactive lesions, no treatment is required for them. For your visa purpose, you should consult pulmonologist and get done CT thorax and bronchoscopy with BAL (bronchoalveolar lavage) analysis. If both of them are negative fir tuberculosis or other lung infections then you can argue with visa authority for your work visa. Hope I have solved your query. I will be happy to help you further. Wish you good health. Thanks."
},
{
"id": 65709,
"tgt": "Can hypothyroidism in the family causes lump in the throat?",
"src": "Patient: Hello. I have a small lump on the right side of throat. Appears to be located on the thyroid. Blood test suggests to be normal TSH level. It is 1.6. I am not convinced that is normal. Hypothyroidism is prominent in my family. Could you give me some insight to a cause for the lump? Thanks! Doctor: Hi, dearI have gone through your question. I can understand your concern.Your thyroid hormone level is normal so no need to worry about that.You may have colloid nodular, multinodular goiter, adenomatous goiter or some thyroid malignancy like follicular or papillary carcinoma. You should go for Fine needle aspiration cytology and ultrasound examination of neck. It will give you exact diagnosis. Then you take treatment accordingly. Hope I have answered your question, if you have any doubts then contact me at bit.ly/Drsanghvihardik, I will be happy to answer you.Thanks for using health care magic.Wish you a very good health."
},
{
"id": 202976,
"tgt": "What causes weak erection and liquid flowing when penis turns soft?",
"src": "Patient: Hello, I have penis problem. When my penis is getting hard, to erection, some liquid drop from it and it`s just transparent. different from white sperms. Especially it drops when my penis is getting to be soft, back to flaccid state. Is it normal or do I have any problems? Thanks for your attention. Doctor: DearWe understand your concernsI went through your details. I suggest you not to worry much. The clear transpareant liquid which you are mentioning here is just pre semen. That liquid helps in lubrication of the vagina for smooth in and out movement of the penis inside vagina. You do not have to worry about that liquid. Unnecessary worrying about sexual organs may have psychological effect and in future it can create erectile dysfunction. Mind it, please. You can post a direct question to me in this website. Include every detail as much as you can. I shall prescribe some sexology psychotherapy techniques to control your condition. I am sure that the techniques should be a success.Hope this answers your query. Available for further clarifications.Good luck."
},
{
"id": 181383,
"tgt": "What causes a large bump behind the neck while treating a tooth infection?",
"src": "Patient: I was prescribed 500mg amoxicillin two weeks ago for a tooth infection. The infection is not gone and rather there's a large bump on the back of my neck- I think from the lymph node, my lymph nodes behind my ear are swollen, I'm light headed, and my skin feels horribly itchy. What should I do Doctor: Hi.Welcome to HEALTHCARE MAGIC..I have gone through your query and can understand your concerns..As per your complain tooth infection does not commonly completely subside with only medication that is antibiotics and painkillers and need proper treatment of the tooth itself for complete resolution..As far as lump on neck is concerned it can be swollen lymph nodes secondary to infection..Lightheadedness and itchy skin however does not seems to be due to tooth infection, rather they are signs of allergic reaction that can also be due to medication that you are taking..Consult an Oral Physician or an Endodontist and get evaluated and a thorough clinical evaluation and investigations like x ray can help in diagnosis and treatment can be done accordingly..If tooth can be saved root canal treatment can be advised while if there is gross decay tooth might need removal..For itchy skin take levocetrizine and if still not resolved consult a General Physician and get evaluated..Hope this information helps..Thanks and regards.Dr.Honey Nandwani Arora.."
},
{
"id": 154793,
"tgt": "Can lower abdominal pain and uterus thickness be linked to cancer?",
"src": "Patient: Hi, I have been having a lot of pain in my lower abdomen. I was told yesterday that I have a bicornuate uterus. I am 51 years old and had a thermal ablation done 2 years ago. I was also told that there is a lot of thickness within the uterus and that it could be cancer. I will be having my uterus and ovaries removed. Is there a chance that this could be cancer. Doctor: Hi, dearI have gone through your question. I can understand your concern. You may have thickened endometrium. It may be due to many causes like simple adenomatous hyperplasia, complex adenomatous hyperplasia, cystic glandular hyperplasia or endometrial cancer. It will be clear in histopathological examination. You have removed your uterus and ovaries so no need to worry. Hope I have answered your question, if you have doubt then I will be happy to answer. Thanks for using health care magic. Wish you a very good health."
},
{
"id": 115545,
"tgt": "Is blood transfusion advisable with increase white blood cell count?",
"src": "Patient: My mom has been diagnosed with TB previously had treatment 2 years ago , she is now having SOB , cough with a hemoglobin of 77 white count high indication infection she is now on her way to hospital for a blood transfusing should she be having a blood transfusion ? Doctor: Hi, dearI have gone through your question. I can understand your concern. Her hemoglobin level is low. So blood transfusion is advisable for low hemoglobin. Don't worry about her infection and high total wbc count. She can take blood transfusion with high total wbc count. No need to worry about that. Hope I have answered your question, if you have doubt then I will be happy to answer. Thanks for using health care magic. Wish you a very good health."
},
{
"id": 195525,
"tgt": "How to solve erection problem?",
"src": "Patient: I am 32 years old , five feet ten inches in height, 64 kg in weight. I am married. The erection does not sustain for a long time with rigidity. I have five and half inch in length erect penis. My question is can i overcome my erection problem using olive oil intake i mean taking 5 teaspoons of extra virgin olive oil daily. Doctor: Hello and Welcome to \u2018Ask A Doctor\u2019 service. I have reviewed your query and here is my advice. Taking olive oil is good for health, but to know the right cause for erectile dysfunction need to consult your physician. Most common cause is stress, hormonal imbalance or diabetes etc. Do yoga and meditation, do regular exercise. If symptoms not improved please consult your doctor he will examine and treat you accordingly. Hope I have answered your query. Let me know if I can assist you further."
},
{
"id": 217188,
"tgt": "Suggest treatment for chronic pain with history of back surgery and spine fusion in neck",
"src": "Patient: I am 56 years old. in 1979 I had back surgery..L4-5, S1. In 2008 I had shoulder surgery. In 2010 I had a spine fusion in my neck. I have been thru every avenue for pain management...P.T...holistic...etc....I deal with chronic pain and am at wits end with it. I no longer have insurance and my local clinic does not dispense pain medication. I am unable to work and am waiting for a disability claim to run thru the SSI red tape. Is there anywhere I can go to get relief? Doctor: As per your problem i realise you have multiple issues here I suggest to follow basic treatment like have anti inflammatory and pain killer drugs along with that do physio therapy exercises. You can also use hot and cold pack to reduce the inflammation with soft tissue relaxing effect. And if still have too much pain then you should take a visit of one of your doctor who has operated you and check with them what to do after doing needed test and seeing the reports One more thing have more of omega 3 in your diet it is natural anti inflammatory so will help you to recover.Take care."
},
{
"id": 22743,
"tgt": "Can patency of a VP shunt be checked by palpating?",
"src": "Patient: Is it possible to check patentcy of VP shunt by palpating ? I have a client in group home who has had major behavior change and I thought of shunt, I was told mom checked it the other day. Client is blind ,non verbal and has MR so questioning her is not an option. Parent not into office visits and believes it s behavioral. Doctor: Hello and Welcome to \u2018Ask A Doctor\u2019 service. I have reviewed your query and here is my advice. Yes it can be checked. There is sac behind the ear usually right and you have to press it, if you are easily able to press it completely then shunt is patent and if there is resistance to your pressure then it's probably blocked.Hope I have answered your query. Let me know if I can assist you further.Regards,Dr. Sagar Makode"
},
{
"id": 48305,
"tgt": "What could pelvicalyceal dilation in kidney suggest?",
"src": "Patient: The left Kidney demonstrates mild to moderate pelvicalyceal dilatation. The proximal left ureter is also slightly dilated. left renal length = 11.2cm.impression : Left pelvicalyceal and proximal ureteric dilation?Secondary to ureteric stricture or calculus. IVP advise for further evaluation. other wise normal intra-abdominal findings Doctor: Hi,Thanks for writing in.The collecting system of the kidney is made of renal calyces and pelvis. This collects the urine formed in kidney and transports it through the ureter (tubes connecting kidneys and urinary bladder) to the urinary bladder. The pelvicalyceal system is the mouth of the kidney where it connects with the upper end of the ureter.I guess that you have got ultrasound scan done and it shows mild fullness of the pelvicalyceal system in the left kidney, that is a term applied to increase in diameter of the collecting system in your left kidney.It is not a critical situation and please do not worry if you do not have any urinary symptoms. Since it is mentioned as mild fullness and no kidney stones have been found, therefore(i) It can be that your bladder was completely full at the of ultrasound scan(ii) Your kidney pelvicalyceal system is appearing full by itself (extrarenal pelvis) and is an extension of normal appearance.(iii) Rarely an infection of urinary system and kidneys (recent or old) can cause such a finding but this is likely only if you have symptoms like pain in the region of kidney and problems like burning urination or discoloration of urine or any kidney infections in the past.(iv) Stones in kidney or ureter can also cause fullness of pelvicalyceal system and since you do not have any stone therefore this does not apply to you.It is important to know if your scan was repeated after passing urine ans still there was persistent mild fullness of the pelvicalyceal system on left side. This can also be confirmed by doing an IVP which is injecting a contrast and taking 5 to 6 X rays to study the functioning of the kidneys in a better way. Please consult the urologist."
},
{
"id": 112155,
"tgt": "Are the withdrawal symptoms of Tramadol like shortness of breath, chest tightness, backpain fatal?",
"src": "Patient: It s hard to. Breathe right now. My back hurts all over my chest feels tight and my husband won t listen to me bc I just flipped out on him but idk what to do my kid is asleep. Pain is sharp under my left rib bone. My nose is stuffy and I m going through withdrawal from tramadol 600 mg a day. I had none today. No refill till Sunday. Am I going to die? Doctor: Hello. Thanks for writing to us. The withdrawal symptoms of tramadol are mild one and are not fatal. These usually subside on their own in 2-3 weeks. You can take other over the counter pain killers like ibuprofen till then.I hope this information has been both informative and helpful for you. Regards, Dr. Praveen Tayal ,drtayal72@gmail.com"
},
{
"id": 116178,
"tgt": "Suggest diet regime during high ESR",
"src": "Patient: what to eat and what not to eat during high ESR,My current ESR result is 85mm at the end of first hour.please suggest curative measures,my age is 55 yrs.I am suffering from night fever since 10 days,today i got my ESR checked and the result is mentioned above Doctor: Hi,Thanks for asking.Based on your query, my opinion is as follows.1. ESR is not increased due to your diet. It is increased due to your night fever.2. Treatment of night fever, which had led to inflammation has led to raised ESR.3. Get a complete hemogram and a peripheral smear to identify possible cause. Your CRP will also be increased. You can have your usual diet. Further details necessary for complete opinion.Hope it helps.Any further queries, happy to help again."
},
{
"id": 206643,
"tgt": "How do i get rid of all the abuses?",
"src": "Patient: HOW DO I GET OVER ALL THE ABUSE THAT I HAVE BEEN THROUGH ? IT ALL STARTED WHEN I WAS 7 AND THROUGH THE YEARS IT GOT WORSE UNTIL I WAS 45 AND THEN FINALLY I TRYED TO KILL MYSELF IN 2005 WHICH DIDNT WORK ... I HELD EVERYTHING IN UNTIL I WENT TO COUNCELING AND STILL GOING BUT I JUST CANT GET OVER IT AND I WANT TO KNOW HOW .... THANK YOU ... Doctor: DearWe understand your concernsI went through your details. I suggest you not to worry much. I can understand your condition. Past teaches you lessons. Positive incidents teach positive lessons. Negative incidents also teach POSITIVE lessons. It always US who refuse to learn positive lessons and dwell our self in negative thoughts and world. Why do you want to do so? Go and see the poor people and see how they are getting abused?. Go to Africa, Pakisthan, Bangla desh etc. Tell me how many of those children are suffering like you in their old age?Psychotherapy techniques should suit your requirement. If you require more of my help in this aspect, Please post a direct question to me in this URL. http://goo.gl/aYW2pR. Make sure that you include every minute details possible. I shall prescribe the needed psychotherapy techniques.Hope this answers your query. Available for further clarifications.Good luck."
},
{
"id": 223745,
"tgt": "Can plan-B pill be taken while having a period?",
"src": "Patient: I had unprotected sex last Friday (I believe it was on the 27th) and I got the plan b pill only because I wasn t sure if anything came in me. It happened again on the 4th of July. Only difference is that I know that my boyfriend came in me this time.. so I bought another plan b pill. I was going to take it after showering but first I had to use the bathroom. When I was cleaning myself I realized I had blood. I think it s because I might be on my period. Which means I obviously started then, and I didn t know about it until after. DID THE PLAN B PILL WORK EVEN AFTER USING IT ALREADY NOT TO LONG AGO? AND ON TOP OF THAT BEING ON MY PERIOD? I m really nervous. Doctor: Hallow Dear,Plan B pills are post coital emergency contraceptive pills. They are to be taken within 72 hours of unprotected sex when there is risk of pregnancy. The after effects of these pills are that firstly it causes some withdrawal bleeding after few days and secondly the following period may be delayed by few days to a week. Though you may consider of the bleeding after 4 July as withdrawal of the Plan B taken on or around 27 June, it would be too late to be considered as withdrawal bleeding of the Plan B pill. So it has to be your menstrual period. You must be aware that the first cardinal symptom of pregnancy is missing a period. Since you got the period the very same day of the unprotected sex, this intercourse cannot cause pregnancy and hence there is no necessity to cover it by Plan B pill. I hope this will take you out of the nervousness. Dr. Nishikant Shrotri"
},
{
"id": 216451,
"tgt": "Suggest remedy for pain due to trigeminal neuralgia",
"src": "Patient: My mother is suffering from heavy facial pain in the region of eye and nose on the right side of the face since last 3 years. As per the doctor consulted last year, he has diagnosed as Trigeminal Neuralgia and had been prescribed Tegretol-200 1-1-1 and amytrine-10, 0-0-1. During these days there was no issue until last week. Now she is getting the pain worser than ever, pain lasts for hours and unable to speak due to the pain. When consulted the doctor, apart from the medicines prescribed Gabapentin-300mg. But the pain still persists. Can you suggest a remedy ? Doctor: Hi.Welcome to HEALTHCARE MAGIC..I have gone through your query and can understand your concerns..As per your complain as your mother was prescribed with Tegretol which has the active ingredient Carbamazepine which is an anticonvulsant drug and is used as First Line Drug for the treatment of pain due to Trigeminal Neuralgia as well as Gabapentin and amytriptyline which are second line drugs which can relieve the neuralgic pain and in case if nothing works for her you need to consult a Neurophysician and get her evaluated and if required he can advise you to get an MRI or CT SCAN done to rule out the exact cause of trigeminal neuralgia and the most common is an artery compressing over the nerve..Other causes can be any tumour compressing the nerve, multiple myeloma, facial injury causing trauma to the trigeminal nerve etc..In case of compression of the nerve with an artery or tumour the decompression of the nerve by surgical procedure can help..In other conditions where pain is not caused by any compression certain techniques like GAMA KNIFE SURGERY, BALLOON COMPRESSION or RADIO-FREQUENCY THERMAL LESIONING are a few techniques in which the nerve is damaged to reduce pain and in this case if there is a relapse repetition of treatment can be done..Hope this information helps..Thanks and regards..Dr.Honey Nandwani Arora.."
},
{
"id": 143412,
"tgt": "What causes pain in the center of the spine?",
"src": "Patient: I have an uncomfortable pain in the center of my spine. It feels tender to the touch, like a bruise. I thought it would go away but I have had it for several months. Does anyone have any idea what could be causing this and what type of specialist I should be seeing? Thank you!Danielle Doctor: Hello!Welcome on HCM!Your symptoms could be related to a musculo-skeletal pain or inflammation. I recommend consulting with your GP for a careful physical check up and some tests: - a cervico-thoracic spine X ray study- inflammation tests (complete blood count, PCR, sedimentation rate). Physiotherapy may be helpful in case of chronic degeneration of the vertebral column. Hope to have been helpful!Best wishes, Dr. Aida"
},
{
"id": 125573,
"tgt": "What to do for my throbbing leg and numb toes?",
"src": "Patient: I hit my shin on Friday and along with the pain my toes went numb. I also have a swollen knee and pain in my hips. I have no bruising at all but the swelling went down a little yesterday. Now my leg is throbbing and my toes are once again numb. Any suggestions? Thanks Doctor: Hi, Probably there is damage to nerves in your hip or spine that is causing the numbness and throbbing pain. Consult orthopedic surgeon for detailed examination and investigations. Hope I have answered your query. Let me know if I can assist you further. Regards, Dr. Gopal Goel, Orthopedic Surgeon"
},
{
"id": 115715,
"tgt": "Can anemia cause irregular heart rate and breathlessness?",
"src": "Patient: hi there, i have various symptoms, i have irregular heart beats but no real heart diseases, i have very irregular and weak urination symptoms, very weak and tired, and have gained about 4 kg in couple of weeks even thoughi eat less then my 2 year old, lower back pain, breathlessness. I am scared that it may be heart failure or anemia with blood loss. My stomach seems slightly rounder then normal, is there distinctive signs for anemia with blood loss as the cause Doctor: Hi, dearI have gone through your question. I can understand your concern. You may have some anemia due to blood loss or some othet reason. It can leads to breathlessness and congestive heart failure. You should go for ultrasound abdomen and anemia profile to search the cause. Then you should take treatment accordingly. Hope I have answered your question, if you have doubt then I will be happy to answer. Thanks for using health care magic. Wish you a very good health."
},
{
"id": 58457,
"tgt": "Bladder infection. Body resistant to cipro. Reason?",
"src": "Patient: The doctor's office just told me that I am resistant to Cipro and that is why my bladder infection has not improved. This is the second round of medication that they have given to me. My bladder infection started over a month again. Is there any reason for concern that my body is resistant to Cipro and that I have had a bladder infection for this long Doctor: Hi,Thanks for writing in to us.Your doctor intends to inform you that the bacteria causing your bladder infection are resistant to ciprofloxacin. Nowadays due to increased prescribing of antibiotics, the same strain of bacteria which would have been killed by an antibiotic let us say 10 years back, has developed genetic mutation and resistance such that they are able to grow even in the presence of the antibiotic.In other words, the bacteria are becoming more powerful and your antibiotics need to be changed. This is becoming an increasingly common problem in hospitals and you need not worry. The solution to your problem lies in changing over to sensitive antibiotics on your urine culture report. Some of the highly specific antibiotics for urinary infections include nitrofurantoin and newer cephalosoprins. This will be decided by your doctor and you must complete the course for quick recovery.Hope thishelps"
},
{
"id": 6244,
"tgt": "Trying to conceive. How to to boost fertility? Advice?",
"src": "Patient: My husband and I have 2 Children whom were conceived rather easy the first one took one month and the second took only 5 months however we are trying for our third and are on month 10 of trying do you have any over the counter things I could take to boost fertility or any advice on why this time it is taking so much longer? Thanks in advance Doctor: hi, in case of regular ovulatory cycles and normal semen analysis proper planning will help in getting pregnancy, you track your ovulation with ovulatory kits, plan intercourse during that period, normally in case of regular cycles between 12 to 16 days will be ovulatory period, if you plan intercourse in that period at least on alternative days the chances of pregnancy will increase. if even then you are not able to conceive you consult gynecologist once to find out any other underlying problem and get treated. take care."
},
{
"id": 203020,
"tgt": "Should i be concerned over the little bump on my penis due to hard masturbating?",
"src": "Patient: Hello, I have a small bump the shaft of my penis after my girlfriend was masturbating me really hard. immediately after she stopped this little bump was there. It's not an open wound, it just kind of looks like a pimple. It has been there for a month now. I don't know what to do, or if there is any kind of ointment I can get. Doctor: helloIt is due to some injury due to hard masturbation . It should subside by itself. You may use cold compresses and avoid sex for some time.DR SAATIISH JHUNTRRAA"
},
{
"id": 88980,
"tgt": "What causes abdominal pain during cough in children?",
"src": "Patient: 10 year old boy,say's his low abdominal hurts when he coughs and says he need to go to the doctor. He has no fever, his cough is from asthma and it's not bad. He's running jumping and playing just fine. Do you think I should take him or give him something for pain and wait and see? Doctor: Hi welcome to hcmChest diseases lead to pain abdomen.get a chest x ray and ultrasound abdomen.if ultrasound is normal u need not worry supportive treatment is sufficient.use anti tussive syrups ,bronchodilators low dose by consulting your treating doctor.avoid dusty environments and pets in your house.take careThankyou."
},
{
"id": 41957,
"tgt": "Can I become a father as per my semen analysis report?",
"src": "Patient: First Name: sanjay Last Name: kumar I am a: Patient, Family member or Friend Email Address: YYYY@YYYY Subject: sperm analysis Comments: result of my semen analysis. SANJAY KUMAR 28 YEARS MALE 30JUN2011. SEMEN ANALYSIS Time of collection :2.05 pm Time of examination :2.35pm PHYSICAL EXAMINATION Volume :1 ml Colour :Grey opalescene Appearence :Clear Consistency :Hypo Viscous pH :7.5 FRUCTOSE Present MICROSCOPIC EXAMINATION Microscopic examination reveals morphologically normal spermatoza along with pus cells (1-2/hpf) and epithelial cells. Forms with excessively long tails are also noted. Sperm Count :8 Million/ml of fluid motality 05% shows rapid linear progressive motility. 15% shows sluggish linear progressive motility. 20% shows non-progressive motility. 60% are non-motile. SIR, what is my report? could i made a father? plz send me details. Doctor: Hi thanks for writing to health care magic.I have gone through your report.Your semen analysis shows less than normal sperm count 8 million is below normal, 20 million atleast. Decreased motility.You should investigate for this. Ultrasound of scrotum to rule out varicocele.Varicocele is commonest cause for male infertility.But u can become father with this report also.There are so many techniques in this era. Like semen wash out and many more.Consult a urologist and he will advise you further.Have a good health."
},
{
"id": 100469,
"tgt": "What is the alternative to antibiotics in case of allergy to them?",
"src": "Patient: hi my name s anitha.i m allergic to group of medicines like antibiotics,amoxlline group etc i hav fever,headach,low bp,cough from 2weeks i went to doctor but the medicine which he gave again caused alergie,he asked me to take a alergie test and come i don know where in bangalore such tests can u please guide me what to do. Doctor: Hello Anitha,Thank you for asking at HCM.I went through your query and would like to make suggestions for you as follows:1. Allergies to amoxicillin group of antibiotics (penicillins) are very common. Allergy testing for antibiotics is best done by skin testing which is done only at specialized allergy clinics. There are many allergy specialists and allergy clinics in Bangalore.2. Blood testing for allergy to antibiotics is not very reliable except for penicillin group of antibiotics. So even if you do the test for other antibiotics, it is possible that results may not be very reliable. 3. There are many other safe antibiotics apart from penicillin group. In my view, a doctor can prescribe you one of many other antibiotics available other than penicillins. 4. If your doctor really needs allergy testing, please get it done at some special allergy clinics.Hope above suggestions will be helpful to you.Should you have any further query, please feel free to ask at HCM.Wish you the best of the health and a very good recovery.Thank you & regards."
},
{
"id": 61653,
"tgt": "What causes red blistering lumps on face, neck, chest and back?",
"src": "Patient: for the past two weeks i have been developing very red, angry looking blistering lumps on my face, neck, chest and my back. They only seem to come up when I am at work. Every morning I go in and the lumps are still there but only slightly show. As the day goes on they get worse and I can feel my skin burn, my eye lids burn as does the roof or my mouth. On a few occassions my throat has felt tight as though I am getting tonsilitus. I have been to the doctors and have been told to stay off work for at least a week but I am worried that as soon as I go back it will all happen again. I have been given a steroid injection and have to take 12 tablets a day (6 steroids, 5 piriton and 1 cetrizine tablet). I have worked there for over 8 years and love my job very much but we have recently moved into a brand new building.... I am not drinking water at school nor eating anything different so how will I find out the cause of my problem? Doctor: Hi Dear ,Welcome to HCM,Understanding your concern. As per your query you have red blistering lumps on face, neck, chest and back . Well the symptoms you mention in query can occur due to may reasons like Contact dermatitis , allergic reaction due to detergent, soap , fragrance , chemical, metal such as in jewelry, poison oak or ivy, an animal bite, or an insect sting and lyme disease .I would suggest you to consult dermatologist for proper examination . Doctor may prescribe anti histamine , corticosteroid ointmet and may give adrenaline short . For now take benadryl twice a day and avoid any cosmetic product .Doctor may alo order test to find ou particular allergen. Apply ice pack on red blistering lumps to reduce redness and itchiness . Protect your skin from direct sunlight and cover it with cotton cloth while going outside . Do not ru it or touch it again and again.Hope your concern has been resolved.Get Well Soon.Best Wishes,Dr. Harry Maheshwari"
},
{
"id": 200444,
"tgt": "What are the chances of pregnancy with no ejaculation?",
"src": "Patient: Hi I have a question so today my boyfriend and I had sex for the first time and we didn t use any protection, but my boyfriend told me that he didn t ejaculate at all. But honestly I have a doubt is there any chance that u can get pregnant like that? Please HELP Doctor: Thanks for asking in healthcaremagic forum It is possible to get pregnant with precum(fluid from penis before ejaculation), so it is always better to use protection. Check with urine pregnancy test kit with morning sample yourself to rule out pregnancy. All the best. Always insist on protection whatever your partner may say."
},
{
"id": 24061,
"tgt": "Suggest treatment for rheumatic heart disease",
"src": "Patient: I have rheumatic heart disease with complications of an enlarged left atrum and atrial flutter/a fib is it safe for me to be doing heavy lifting and working 12 hour shifts,lately have been breaking out into sweats out of the blue and experienceing brief shortness of breath. Doctor: Hello!Thank you for asking on HCM!Regarding your concern, I would explain that you are not allowed to perform straining physical activity as this may exacerbate your cardiac conditions. I would recommend performing a cardiac ultrasound in order to examine better your cardiac function and structure. You may need to optimize your actual therapy. I would also recommend considering anticoagulation therapy with warfarin as you are at an increased risk for future stroke events. Hope you will find this answer helpful!Best wishes, Dr. Iliri"
},
{
"id": 132149,
"tgt": "How can knots be treated?",
"src": "Patient: I have a fracture on outside middle of foot. Swelling and bruising are gone, but still a knot and tender to the touch. I am 4 wks into the injury and concerned about the knot and if calcium deposits are building. I keep foot wrapped but not sure if it has been tight enough. What else can I do? Doctor: Hi You had a fracture in the middle of foot about 4 weeks ago and are feeling painful knot in the foot. This is a normal healing process when the bones unite , unless there is some mal position of fractures . There is formation of lump( callus) around around fracture which slowly disappears over few months for it to disappear. So dont worry about it now , just reassess after a month or so."
},
{
"id": 56081,
"tgt": "Could the rashes almost like ring worms spreading to the body be due to liver failure?",
"src": "Patient: I have chronic hep c and chirrossis I have been getting a rash that appears when I feel a little sick from hep c.there red ring rash almost like ring worms that start on my lefr side working its way to my breast and tummy ,usually in a 3 to 4 day period.is this symptoms of liver failure or what.please help ,the unknown is the worst part. Doctor: The most characteristic skin signs of liver failure( of any cause, hepatitis C included) are the \"angioma stellatum\" or in english the spider angioma and the \"hepatic palmae\" this last in the hand palmary skin. None of this seems to me as your problem, because both of them are chronic when installed and the location neck and thoracal regions for the first and thenar eminence of palmar hand for te second, but of course hepatitis C can be the cause of many other cutaneous manifestations of importance and not and related more with the viral infection of \"per se\" than with the chronic damage degree. Anywheere I'd suggest a dermatologist consult (of course it's not urgent) to be more clear with your signs. I hope to have answered somehow your question."
},
{
"id": 80821,
"tgt": "Is pantroprazole sufficient to treat chest pain and abdominal pain?",
"src": "Patient: i am mahmud from china. last 3 weeks i felt chest pain,upper abdominal sight pain.. i went to hospital, doctor told to make endoscpoy,,,,i did that and he gave me 2 weeks pantoprazole(2*1),,,,itropride hydrocholaride(3*1),,,,hydrotalcite tablets (3*1).......but now my pentoprazole is finished....have there any rule to take itropride with pentoprazole...............still now i am feeling chest pain...sir Doctor: Thanks for your question on HCM. I can understand your situation and problem. In my opinion we should first rule out cardiac cause for your symptoms. So get done ecg. If ecg is normal then no need to worry much for cardiac cause. Yoh mostly have acute gastritis. In this, along with drugs, , following lifestyle modifications are required for better symptomatic relief. 1. Avoid hot and spicy food2. Avoid stress and tension. 3. Avoid large meals, instead take frequent small meals. 4. Go for walk after meals. 5. Keep 2-3 pillow under head in bed."
},
{
"id": 177273,
"tgt": "What causes foul vaginal odor in a 10 year old?",
"src": "Patient: My 10 yr old virgin daughter has a real strong foul odor coming from her vag and we tried over the counter items like summers eve wash and deep scrubbin. But no change and we fear it will get worse once she gets her monthlys we been fighting this persistent problem for years but every time we seem to have it licked it comes back in full strength pls help Doctor: Hi, I have gone through the medical problems of your daughter carefully and understood your concerns. 1. Urinary tract infection can be a possible cause. Get her urine checked by urine analysis and urine culture and sensitivity testing. 2. Be sure she is still virgin: consult with gynecologist to make sure she did not have forced sexual intercourse.3. Vaginal yeast infections are also to blame. Vaginal swab testing can rule it out. Bare in mind not to go deeper in vagina for vaginal swab testing. 4. Intestinal parasitic infections can be another cause. Examination of stool for ova/parasites is highly recommended. For the moment, I highly advise to:- keep a good hygiene of her genitalia- avoid foods/drinks rich in carbohydrates and caloriesAll the best!Dr.Albana"
},
{
"id": 77491,
"tgt": "Suggest treatment for cough after coming in contact with spray paint",
"src": "Patient: My wife used two cans of spray paint in our basement to paint some outdoor furniture. I know she didn't have proper ventilation and used way too much paint. She has had a very nagging cough for the four days since then. Is there anything that can be done, or does she just need to give it some time? Doctor: Hi thanks for asking question.She had come into contact with paint.so she mostly have Chemical rhinitis or laryngitis.Symptomatic management done for few days.Take semisolid diet for few days with more fruits.Drink more water.Strictly avoid direct and indirect smoking.Steam inhalation done with eucalyptus oil.For congestion relief take antihistaminic drug.Green leafy vegetables taken more.If still your symptoms increasing then chest x ray like further investigation done.I hope my suggestion will be helpful to you.Dr.parth"
},
{
"id": 54329,
"tgt": "What does this scan report of abdomen indicate?",
"src": "Patient: Hello doctor, I am having pain in lower part of abdomen for the last 5 days. I am a female 31 years of age married and my weight is 41 kg.My ultrasound of whole abdomen shows: \"Liver is enlarged 15.5 cm in span with mildly bright echotexture s/o fatty infiltration of liver.IMPRESSION: Mild hepatomegaly with mild fatty infiltration of liver.\"I do not take any fatty food though, nor do I drink, however I daily take thyrox 50 and calcium(corseum) tablets. Please advise me how can my pain be reduced, it is severe at times. Thanks ! Doctor: thank you for posting query at HCM.presentation of fatty liver.try to monitor your thyroid profile (hormone) especially TSH.you maybe needing an increase in the thyroxine dose and you may also be required a medication to lower cholestrol (statin drug)better to also rule out viral causes;further blood tests needed alongwith \"LIPID profile\":1. HBsAg (if NOT vaccinated against HBV) 2. anti HCVmeanwhile:- also test your blood sugar level- LOOSE weight- abstinence from alcohol- FAT FREE diet - NO red meat- vegetables should be ingested daily- Moreover, less activity should be carried out (no sports), REST more. - use lemon juice (lemonade) once in a day- udiliv maybe taken (consult hepatologist/ treating physician)- Repeat LIver function test and lipid profile after 6 to 8 weeks.Health professionals aim to diagnose properly and manage patients according to their limited knowledge. Cure is blessed by the ONE who Created us, whose power and knowledge is unlimited .wish you good health.regards,Dr Tayyab Malik"
},
{
"id": 176460,
"tgt": "What causes vomiting at night while having whole milk?",
"src": "Patient: My daughter is a year and one month she recently like two months ago started drinkijg whole mill but recently at night she tends to wake up whine and throw up cottage cheese looking throw up and smells bad what is it? Should I take her to her doctor? Or switch milks or do organic or how many bottles is a baby her age suppose to consume on a regular basis or day? Doctor: Hi...at this age, the milk consumption should be around 300-400ml per day. By what you quote I feel that she is intolerant to cow's milk. Suggestions -1. Mother should go off the cow\u2019s milk products completely - including biscuits and chocolates and all products related to milk.2. Continue exclusive breast feeding and if it is not sufficient you can opt for Zerolac.3. Cow's milk protein intolerance in transient as it will go off as the kid grows up.4. Avoid all cow's milk products till the kid is 1 year age.5. Mother's milk is the best for good growth of the baby. Rather than stopping mother's milk, it is best for the mother to avoid all milk products and continue breast feeding.If you have any further queries you can approach me at the following link.You can approach me at the following link. Please find the link below.www.healthcaremagic.com/doctors/dr-sumanth-amperayani/67696Regards - Dr. Sumanth"
},
{
"id": 172151,
"tgt": "What could be the red rash on my son s body?",
"src": "Patient: i think my son has chicken pox but its a rash thats red and very close together, its not spots and they dont itch him, he has bin hot and complainning of a headache for a couple of days, he is only 3 so its hard for him to tell me what the problem is? Doctor: Or son may have measles, common in this age group.give him more fluids to drink .give him paracetamol for fever. If your son develops respiratory distress , then u can see doctor."
},
{
"id": 48364,
"tgt": "Is using shockwaves to breakup kidney stones normal procedure to treat them?",
"src": "Patient: My partner is taking pamsvax xl 400mg for kidney stones. He has one stone 12mm. He has been told to take in Evening. He has been on this for 10 days. Before they begin procedure. Which they will zap stone to allow him to pass through unrine. Is this normal? Doctor: Hi welcome to HCM.I understand your concern.Kidney stones in your case,are very small about 1.2 cms.These range of stones can be safely removed through shockwave therapy.But before drugs like pamsvax are to be given to decrease the size of the stone further.This is the normal procedure that is needed to be adopted to prevent the complications during stone retrieval if any.You need not be worried of this protocol as its done for better stone removal.Discuss the above with your doctor.Post your further queries if any.Thank you."
},
{
"id": 37270,
"tgt": "Suggest remedy for persistent cough",
"src": "Patient: I started off about three weeks ago with a flu type illness - hot and cold bad head - this left me with a really bad cough so I got penicillin off the doctor - one week course but still had a nasty cough and went back to doc - she said my lungs were clear but chest infection so another course of antibiotic Clarithromycin I was given week course and now only one day left with them but im still having coughing fits - can you help? Doctor: Hello,Welcome to HCM and thanks for your query.Coughs are caused by either a viral or bacterial infection or due to allergy and/or asthma. So it is important to know the cause. If you have a white blood cell count done, it often helps to differentiate between viral and bacterial infections. If there is infectious mononucleosis, caused by a viral infection, penicillin group of medicines can make it worse.If the cough is really bad and if there is yellow and rusty sputum or fever, it is good to have a chest Xray to see if there is a pneumonitis.If it is a viral or allergic cough only symptomatic treatment with plenty of warm fluids, an expectorant and ketotifen would be helpful.If it is the cough due to asthma, one would require bronchodilators.So the management has to be individualised depending on the cause.There is no shoe that fits all.Take careDr Noble Zachariah"
},
{
"id": 162677,
"tgt": "What causes painful red spots on the scalp?",
"src": "Patient: my 6 year old is complaining of pain from a small red spot on her scalp. It almost looks like a mole. she has not had any injury, and the spot has been painful for a few weeks. it looks to be nearly in the same spot that she had a strawberry hemangioma as a baby. is it possible for that to come back? Doctor: Hello and Welcome to \u2018Ask A Doctor\u2019 service. I have reviewed your query and here is my advice. hemangiomas are not known to recur. This is unusual. I suggest either you upload an image here for me to see and guide you or consult your dermatologist please. Skin conditions are best diagnosed only after seeing directly. I suggest you to upload photographs of the same on this website, so that I can guide you scientifically. Please revert back to me with images so that I can guide you better. Hope I have answered your query. Let me know if I can assist you further."
},
{
"id": 209368,
"tgt": "Suggest treatment for mental disability",
"src": "Patient: m a studend and my room pArtner is suffering from a problem....................he is very confuse..............one day he was going to kill me...........after some time he was behaving normally.....................his all medical report are normal..................tell me wht should i do? Doctor: DearWe understand your concernsI went through your details. I suggest you not to worry much. Your description does not tell the full story. What was the reason for your friend to try to kill you? Was it a simple anger and frustration issue? Was your friend behaving insanely without any apparent reason or the trigger was so small? Was your friend behaving like a maniac?You need to answer these question to reach to a correct diagnosis. For the time being, just one issue like this may not be that serious. But if this repeats do consult a psychologist.If you require more of my help in this aspect, Please post a direct question to me in this website. Make sure that you include every minute details possible. I shall prescribe some psychotherapy techniques which should help you cure your condition further.Hope this answers your query. Available for further clarifications.Good luck."
},
{
"id": 146754,
"tgt": "Suggest remedy to improve memory",
"src": "Patient: I have problem remembering thing whenever i study? It takes a long time for me to absorb the information and then when i remember they slip away the following days.It's difficult for me..I really need to retain what i have study.It's just frustrating! Any help thanks.And could i be suffering from something serious? Doctor: Dear Sir/ Madam,Thank you for posting you query at healthcaremagic.comI advice you to study at relax mode. Do not take your study as a stress. Always study a subject with the fun. It is better to understand the topic well and correlate in practical. Exchange your thoughts and answers with your freind or colligue, this will boost your confidence.Proper sleep is also very important. Avoid continuous study at strech. Swim,talking to friend, stroll, recreational break in between will help. Vitamin E rich diet, protein rich, fish diet are advisable.if you have any further query please revert back to me.Your reply with thank you note will be very much appreciated!!With best wishesTehzib Saiyed(PT)"
},
{
"id": 11328,
"tgt": "Suggest treatment to regain the thickness of hair",
"src": "Patient: Hi, 1 year back first I suffered from jaundice and the very next month typhoid. After that I had heavy hair fall for few months. Now it has stopped hair falling but I lost the volume and it is very thin now. 1 year is over and I am afraid whether I will get back my hair or not. Doctor: Hello,Thank you for posting on HCM.I appreciate your concern regarding hair fall. Hair fall is usually ascribed to multiple factors like diet and nutrition, hormones, stress, acute or chronic medical conditions,drugs,cosmetic products etc.From your description, it seems you might be having Telogen Effluvium, which is hairfall secondary to stress.If i were your dermatologist/trichologist, i would like to take through history including family history, history of recent major trauma/illness/stress/medications etc and recommend some basic investigations like CBC,blood sugar,Thyroid function test.Some other special investigations like trichogram and dermoscopy and further aid in diagnosis and prognosis.I would suggest you a course of oral tablets containing biotin and other essential vitamins and minerals for minimum 3 months. Also, would put you on solution containing 5%/10% minoxidil once a day and a hair serum containing peptides for hair growth at night.(Q sera etc). I would advise use of gentle shampoo and conditioner on regular basis and use of coconut oil twice a week.Also enquire from your dermatologist about upcoming treatment options like mesotherapy and platelet-rich plasma.Avoid combing in wet hair and let them dry by wrapping in towel. Avoid blow dryers and hair-color/dyes. Take plenty of fresh fruits and vegetables in your diet and try to de-stress your routine life.Hope your queries are resolved and wish you best of health.Thank youDr Hardik Pitroda"
},
{
"id": 103400,
"tgt": "Liquids when drank felt in ear too, ear canal swollen sometimes, history of allergies and TMJ",
"src": "Patient: Hello Why is it when I drink water or any other fluid I can feel the water in my ear afterwards? I can put my finger in my ear and feel it is wet?? I also feel like my ear canal is swollen not all the time but it seems like a lot. I do suffer from tmj and have a mouthgard at night but when I place my fingers in my ears I can her the loud noise from my jaw :-( I do also suffer from allrergies I am allergic to dogs and dust mites and take allergie meds and nothing seems to help. I always have a runny nose and swollen air way. I have been to 3 different ent docotors and they see nothing no infection of any kind and say my ears look good. So why the swollen feeling and the water feeling in my ears is it the tmj from my jaw? Doctor: as you have history of allergies in the long term allergy involves the organs like sinuses in your case you can confirm by getting xray pns done and treat accordinglyin sinuses there is postnasal drip which blocks eustachian tubes and causes problems of ear and joint tmj written by youtake doxycycline 100 mg bd for 5 days to clear sinuses fasttab fexofenadine 120 mg bdtab cpm 1 tab nightantaids to prevent gerd dueto pns of sinusesapply neomycin h ointment in nose bdsea water 2 drops at night in each nosewhich will clear ears and jawtake minimum 3 wk and condsult doctor or further management"
},
{
"id": 145862,
"tgt": "Is Zypreza effective for parkinsons and demencia?",
"src": "Patient: my husband has parkinions he is 86 he has some demencia and has some behavioral problems. The have him on 2 sericquell and it makes him so groggy. They want to change him to 1 pill of zyprexa a day to see if he would be more alert. Do you think think zypreza is good? Doctor: I read your query and I understand your concern.As you seem to understand both Seroquel and Zyprexa are used just to control his behavioral symptoms and do not cure parkinson or dementia.The trouble with drugs of that class, antipsychotics, is that they could worsen parkinsonism. For that reason Seroquel was a good choice because it doesn't cause parkinsonism. Olanzapine (Zyprexa) isn't as safe on that regard, but still little effect on Parkinson's compared to some other antipsychotics, so since Seroquel is causing so much sedation it is a good compromise.Another option is Clozapine which like Seroquel doesn't affect Parkinson's, but I guess doctors fear another possibly serious side effect of it, supression of white blood cell production.I hope to have been of help."
},
{
"id": 154489,
"tgt": "Should yellow fever vaccination not be given to patients with tumor?",
"src": "Patient: I am going to Africa and need a Yellow Fever vaccination. I am 64 and in good health other than high blood pressure and cholestoral (on medication for both situations) I had a Pituitary Tumour (benign) removed 5 years ago and do not have any problems other than headaches.I am not on any medication for this. Are there problems as I have been told I should not have the vaccination if I have had a tumour. Doctor: Thanks for your question on HCM. No, you can take vaccine. No harm in it. And there are no such contraindications for yellow fever vaccine administration. The only contraindication is acute infection. If you are not having any symptoms, and you are fit than you can definitely take yellow fever vaccine. No harm in it."
},
{
"id": 47488,
"tgt": "What causes pain in right kidney side?",
"src": "Patient: IAM 26 YEARS OLD GETTING PAIN RIGHT KIDNEY SIDE FROM PAST 3 DAYS ONWARDS WHILE IAM GOING TO BATHROOM AND SITING, SLEEPING AND WAKING TWO DAYS BACK I CONSULTED DOCTOR HE GAVEN ME TWO TABLETS IT IS DIAZOX AND RABIMAC DSR BUT PAIN WAS NOT REDUCED Doctor: HelloThanks for query .The pain in Right side loin can be mostly due to stone in the kidney or ureter .Please consult qualified Urologist for clinical examination and get following basic tests done to confirm the diagnosis.1) Urine routine and culture.2) Ultrasound scanning of abdomen and pelvis.In the mean while take broad spectrum antibiotic like Cefixime along with anti inflammatory drug like Diclofenac twice daily .Get the prescription of medicines from your family Physician.Ensure to drink more water .To keep your urine dilute This will help to control dysuria.Further treatment will depend upon result of these tests and final diagnosis depending upon the size and site of the stone.Dr.Patil."
},
{
"id": 28872,
"tgt": "What causes sinusitis along with body ache and breathing difficulty?",
"src": "Patient: I was diagnosed with sinusitis on Tuesday night, given antibiotics, but I am not feeling any better. I went back to the doctor and he prescribed a new antibiotic. I am still feeling achy all over, very sleepy, difficulty breathing at times (using an inhaler), coughing, congested and head aches. A chest xray was done but showed no pneumonia, it was recommended that I go to the er for blood work and further testing. What else could cause all of these symptoms? Doctor: Hello and Welcome to \u2018Ask A Doctor\u2019 service.I have reviewed your query and here is my advice.If the chest x-ray is clear then the symptoms that you are facing is probably due to sinus infection or sinusitis. You should get a sinus x-ray and blood culture done to find the exact cause.For now, continue taking antibiotics and along with it take anti-inflammatory painkiller like Ibuprofen, decongestants like Pseudoephadrine. Do saline nasal irrigation, steam inhalation and use a humidifier in your room.You can also be advised decongestant nasal sprays containing Xylometazoline or Oxymetazoline. Avoid spicy and chilled foods. Do warm compresses around nostrils.Hope I have answered your query. Let me know if I can assist you further.Regards,Dr. Honey Arora"
},
{
"id": 226012,
"tgt": "Continuous severe bleeding with pain. On implant. Solution?",
"src": "Patient: hi am just wanted to ask, i have been on implant about a year now.and my perids are all over the place. very bad bleeding, like i would be on for about 3. months then of for two days then back on. im also expirenceing harm ake were my implats is, and my pains are realy bad. is there anything that could stop my periods. and ease the paine ? Doctor: You should have the implant removed. Its obviously not working for you. Take ethamsylate/tranemic acid tablets for the heavy bleeding. But there may be another cause for the pain. See your doctor and get investigated."
},
{
"id": 54424,
"tgt": "Can liv 52 effective for fatty liver?",
"src": "Patient: Hello.. I have Non alcoholic fatty liver.I am 48yrs old. I have already started with my menopause. I am 164 cms tall and weigh 125lbs. My SGOT AND SGPT Levels are high. My mother has a srong histroy of diabetes and is on insulin. I exercise most regularly for past 4 years including yoga , simming walk and gym.Kindly advise if I can take liv 52 to help my fatty liver , if yes what should be the dosage Doctor: Hi welcome to HCM....You have fatty liver disease....Yes liv 52 is Himalaya product and it is good Ayurvedic remedy for liver...It can be used in almost every liver disease including fatty liver also....It is available as syrup , tablet both...Two tablet taken daily....with eight hour gap...It detoxify liver....Meanwhile less oil used in cooking and fried food taken very less....Some good food for liver taken like carrot, spinach , onion , turmeric , papaiya , apple , Indian gooseberry etc....Avoid alcohol...Hope your concern solved...Dr.Parth"
},
{
"id": 149143,
"tgt": "Done surgery for scheuermanns disease. Feeling popping in spine, numbness in back. Something to worry about?",
"src": "Patient: Hi I am 19, i recently had a surgery for Scheuermanns Disease fusing from my T-2 to my L-1. it is a little over 4 months since my surgery and I am feeling a poping along the sides of my upper spine where the metal rods and fusion is. I have numbness in the area and had other people feel my back. They say the feel something pushing up. Is this something I should worry about? Doctor: Hi, it appears that the feeling of some thing pushing up, and numbness to touch may be common, for some more time, but it is prefered to consult your operated surgeon, when you have even a small doubt. I advise you to avoid weight lifting, and hard work for some more time.Thank you."
},
{
"id": 200481,
"tgt": "Suggest treatment for premature ejaculation",
"src": "Patient: I am suffering from premature ejaculation. I think i get too excited most of the time and think about sex all the time. I also masturbate too much, almost daily. Kindly suggest how can i reduce this problem. I am still unmarried but I know of this problem during my masturbation as I discharge too quickly, almost in seconds. Doctor: Thanks for asking in healthcaremagic forum kegel exercise can help you. Search it on net you will get it. Masturbation within limits when excited is completely normal and not harmful. All the best."
},
{
"id": 52678,
"tgt": "Suggest treatment for hepatitis B",
"src": "Patient: Query: I am a patient of Hepatities B and my % is 6.33% but at present i have not fever ,yellowish skin,urine problem,any ache,vomiting ,diohirea etc. I am using the medicine Adfovir prescribed by one M.D of my district Mr.jasamudin.so what i have to do now. pls guide and prescribed me. Thanks & regards shatrughan (patient) Doctor: sir, you have to be vigilant in future about your liver status and must undergo monitoring of your viral load and activity status at regular intervals. you have to understand that hepatitis B has no cure at present. one has to cope with it once contracted and prevent its dissemination to other person. consult your physician who started your medications. He will check your hepatitis B profile and according to which he will change or stop the medication."
},
{
"id": 140773,
"tgt": "Will the sleeplessness be due to sporadic fatal insomnia?",
"src": "Patient: Hi there i really think i have SFI sporadic fatal insomnia i have lost ability to sleep altogether and its been 5 weeks now going on 6 weeks i no longer feel tired but my body does im loosing weight pinpoint eye pupils strained eyes all the time 24/7 lost my apatite my emotions are messed up buzzing in my ears I literally cant get one wink of sleep my eyes are all red and puffy my balance is messing up i went into bathroom today and put my shoes on without even putting on my trousers on im loosing my mind Doctor: Hi, Very sorry to hear about the problems you are having. It is apparent you are having difficulty with getting enough sleep but the incidence and prevalence of SFI are so rare that other things should be ruled out first. I recommend that you see a neurologist at first opportunity to do a good history and physical examination. It also seems appropriate that you should undergo a sleep study for your insomnia and that will provide a good amount of information which can then, be correlated to any history that may explain why the sudden onset of these sleep problems. From there additional studies such as MRI imaging of the brain and so forth can be determined although once again, the rarity of this problem would make it highly unlikely that you should have it as opposed to other problems of either an organic or psychological nature that could better explain your symptoms. Hope I have answered your query. Let me know if I can assist you further. Regards, Dr. Dariush Saghafi, Neurologist"
},
{
"id": 47634,
"tgt": "Suggest treatment for increased creatin and urea level",
"src": "Patient: hi i am a diabetic patient for the past 16 years i am taking 4 injections a day my creatinine level is increasing now at persent it is 1.78mg higher than normal same is with blood urea which is 48 now what treatment u suggest me my ID is YYYY@YYYY Doctor: Hi, welcome to HCM. Looking to your history, it seems that you have diabetic kidney disease. Do basic urine report, urine albumin quantification and sonography.check your fundus with ophthalmologist. Control BP. Diet with low salt.Consult nephrologist or you can communicate with me also with above reports. I think this is would be helpful to you. Best wishes. tc. Dr Jay Patel."
},
{
"id": 14130,
"tgt": "Suggest medications for itchy rash",
"src": "Patient: Hi, I have a very itchy rash, mainly on my back, but also chest and abdomen. the first few red dots appeared on my lower inner arms and outer thighs but those areas seem better. the rash. I thought at first maybe bed bugs as we had just been away, but the rash looks more like a heat rash than bites.... help Doctor: Hello and Welcome to \u2018Ask A Doctor\u2019 service. I have reviewed your query and here is my advice. You may be right...most probably it may be miliaria. Consult the dermatologist for the perfect diagnosis and proper treatment. I would suggest you to apply Clotrimazole powder. Take vitamin C tablets twice a day. Take antihistaminics like Levocetirizine at night. Stay in AC room to reduce your body temperature. Take bath in Dettol mixed water. Hope I have answered your query. Let me know if I can assist you further."
},
{
"id": 50305,
"tgt": "Feel bloated after dialysis, fluid accumulates in abdomen. Dialysis more often?",
"src": "Patient: After dialyses I feel so bloated, speciality if I am left over weight. I am 1.47m yesterday after my teatment I have being left at 40kg instead of 39kg. The nurse tell me I should lift my dry wEight by 1/2 kg and I tell them otherwise it is my body I know how I feel. I feel better when left at 39 verse anything over. I undestand this is a common problem with slim patient. All the fluid accumulates in my abdomen. It looks terrible and feels terrible.The only solution I have is to have diealyses more often to keep the dry weight normal without having to take to much fluid everytime. Doctor: Hi, you are on dialysis , and thinking of the weight, it is the doctors who will decide what to do and when. Dialysis should not be done for controlling weight. Any how if you feel so convey your openion to your doctor, and if he is convinced it is well and good. Thank you."
},
{
"id": 81429,
"tgt": "What causes shortness of breath while walking?",
"src": "Patient: I have extreme SOB when I walk,even a few steps.my chest CT,chest X-RAY and heart CT are all negative and look really good. Panic attack is what we hope it is,but can a panic attack happen only when you re up walking and no other time?what can I do to over come this if it is only panic attacks? Doctor: Thanks for your question on HCM.Shortness of breath is seen most commonly in cardiac and pulmonary causes. Since your extensive cardiac and pulmonary workup is normal, no need to worry much for all these.It seems mostly panic attack only.And yes panic attack can cause such symptoms.For treatment, I advice you to consult psychiatrist. Get done counselling sessions. Try to identify stressor in your life and start working on it.You may need some medicines too.So consult psychiatrist and discuss your problems."
},
{
"id": 29027,
"tgt": "Is aggravated psoriasis due to Ocrevus infusion?",
"src": "Patient: Yes thank you, I had my first two doses of ocrevus in September. I have psoriasis and have for years. However they have gotten really bad . On my hands and legs but they are bottom of my feet. I was just wondering if this could be this due to this infusion like a infection Doctor: Hello,You provided incomplete data. In my opinion, psoriasis is not aggravated by taking Ocrevus (Ocrelizumab) infusion. However, you seem to have developed infusion reactions apparently caused by Ocrevus infusion. A dermatologist's opinion is however recommended to find out the cause of flare-up of the disease.Hope I have answered your query. Let me know if I can assist you further.Regards, Dr. Mohammed Taher Ali"
},
{
"id": 6082,
"tgt": "Severe menstrual cramps. Took excedrin. Trying to get pregnant. Pain in the lower abdomen and chest with headache. Pregnant?",
"src": "Patient: Hello, I am 25 years old and been trying to get pregnant since almost 3 years now but no luck. Usually I do have really bad menstrual cramps but this month suddenly it just got so bad (trust me, really really bad) that I just wanted someone to get me out of that hell. I took Excedrin for pain killer and after 5mins I vomited everything I had for lunch. Next day my pains started to disappear but day after that when I woke up my pains re-appeared again at the lower abdomen and sometimes below chest (this is my 4th day of period with decreased bleeding) with slight fever and head-ache. What do you think is happening here? Could I be pregnant or is it something serious that I should worry about? Doctor: Hi, Kaswini, Thanks for query. As this is your fourth day,chance of having pregnancy is less. But as you have fever,your all problems might be due to this. Go for gynaec check up for having any infection in your genital tract or urinary infection. Ok and bye."
},
{
"id": 113326,
"tgt": "Pain in the back on bending down after getting an electric shock in the past. Lumbar strain?",
"src": "Patient: a 1 year and a half ago I fell off a 4 ft ladder after being electrocuted by 277volt 20 amp circuit. I passed out before I hit the ground. I was told by a friend I landed sitting down. I have moderate/severe pain in my right lower back. E.R. doctor told me it was a lunbar strain but have done some research and found out that if it was a lunbar stran it would have went away in about a month. I have pain in my back everytime I do certain activities like digging or just bending constantly. What do you think it is? Doctor: Hello. Thanks for writing to us. The pain that you have might be related to a disc prolapse. An MRI scan of your lower back will help in detecting the extent of injury that you have suffered. Take a mild pain killer till then for relief. I hope this information has been both informative and helpful for you. You can consult me again directly through my profile URL http://bit.ly/Dr-Praveen-Tayal Regards, Dr. Praveen Tayal drtayal72@gmail.com"
},
{
"id": 184210,
"tgt": "What causes gum to turn colour to black?",
"src": "Patient: hello there,Just today I noticed that my gum both on upper and lower row of my teeth are turning almost black. What could be the possible. If this helps: I smoke regularly and drink as well. But I have never ever seen this before. What should I do?? I'm worried Doctor: Hi ... This is called melanin pigmentation ( dark color gingival) due to smoking . this is very comanly seen in chronic smoker. So I will suggest you to quit smoking habit . smoking is injurious to health .it have bad effect on lungs gums and oral cavity. Initially you will find difficulty in avoiding smoking habit but by strong determination you can quit. I will advice you to contact near by dentist. Hope you got your answer ..."
},
{
"id": 70454,
"tgt": "How to treat the lump on the knee which is painful during nights?",
"src": "Patient: i have a small lump on the top outside area of my knee. It's the size of a pea and can be moved about - I first noticed it on Sunday. I saw my GP on Tuesday who prescribed anti-inflammatory drugs - she thinks it could be an abcess. It is painful and wakes me during the night, the drups don't seem to be working. Any ideas? Doctor: Hi ! Good morning. I am Dr Shareef answering your query.Yes, this could be an abscess if it is painful and wakes you during the night. If I were your doctor, apart from prescribing the anti inflammatory drugs to relieve you symptomatically, I would also refer you to a general surgeon to clinically examine you and plan further course of action accordingly depending on the diagnosis and investigations if any.I hope this information would help you in discussing with your family physician/treating doctor in further management of your problem. Please do not hesitate to ask in case of any further doubts.Thanks for choosing health care magic to clear doubts on your health problems. Wishing you an early recovery. Dr Shareef."
},
{
"id": 129951,
"tgt": "How to treat swollen bridge,forehead lump post injury?",
"src": "Patient: Hi my daughter fell and smacked her face on the edge of the swimming pool at a b'day party...she has a swollen bridge and a massive lump on her forehead between her eyebrows....slight difficulty in breathing and is feeling very tired....do u reckon I shud take her to emergency? Doctor: Hi...I recommend you to kindly take your daughter to ER to rule out any damage to eyes and nose...meanwhile I suggest you to ice hetcunjured area to control damage..Hope this is helpful for youkindly revert back in case you need any further help in this regard..."
},
{
"id": 102436,
"tgt": "What are the possible treatments for bronchitis ?",
"src": "Patient: I went to the Florence family clinic about three weeks ago. She told me I had bronchitis. She gave me some cough medicine, inhaler, steroids, and a shot. One week later I went back because I still didn t feel good. She told me I had all the symptoms of walking pneumonia and two ear infections as well. She gave me stronger antibiotics that I m still taking and a shot again. She told me if I didn t feel better by Friday which was four days later I need to come back. I came back bc I still didn t feel good. She took blood to see if it was a vital infection but the results said it wasn t it. I started feeling better until four days later. I m still taking my antibiotics I have two or three days left. Right now I m tired my chest hurts, my throat hurts, my ears do a little, and my headache comes and goes. Doctor: Hello, Thanks for your query.Stop the stimulants and we all know smoking and alcohols are on top. once done good hygiene, bronchodilation, and excercise and physiotherapy with some antibiotic will get you through but it will take time and effort.Continuous medication is needed to keep the disease under control. One has to be regular in his treatment so as to prevent the acute episodes which occur in the presence of triggering factors.I do hope that you have found something helpful and I will be glad to answer any further query.Take care"
},
{
"id": 81668,
"tgt": "Suggest remedies for severe chronic cough",
"src": "Patient: I have been coughing for sometime, but have not been to Doctor. I cough when I am standing, sitting and laying down at night, sometime feel like I am choking, eyes water and nose start to run. It is not all the time, it just hits me and takes a while to stop. Usually using a cough drop will help, may be a while before it hits me again. Could you give me suggestion as to what kind Doctor I need to see? Thanks Lind Doctor: What you have is allergic disorder and hence the chronic refractory cough and runny nose. Start tablet montair LC 10 mg at night with a codeine syrup three times daily.Take good steam inhalations for 10 mins each three times a day Let me know if you aren't better after 3 days still"
},
{
"id": 166883,
"tgt": "What causes mild fever and night sweats in a toddler?",
"src": "Patient: my 5years old was abused at school and had a low grade fever for ten days after iknew i made a blood test for aids which was negative but he still sweats at night and feele tired after little effort and his skin allergy returned shall i do the test again? Doctor: Hello and welcome to \u2018Ask A Doctor\u2019 service. I have reviewed your query and here is my advice. The test for AIDS should be related after a month and 3 months of exposure. Also you need to do tests for Hepatitis B and C, and for other sexually transmitted diseases to exclude any transmitted infection due to the abuse. Hope I have answered your query. Let me know if I can assist you further.Regards,Dr. Salah Saad Shoman"
},
{
"id": 182634,
"tgt": "Can I go to gym after having a tooth removal?",
"src": "Patient: Hello, I ve just had my wisdom teeth removed Monday and now today here is Thursday. The Dentist told me to rest and don t go to the gym for a week, but it s killing me to just eat and lay around all day. A couple days ago I got on the scale and I could be wrong, but I think the scale put me up 2 lbs. So far all I ve eaten since the surgery is: Oatmeal, Soup, Grits, Apple Sauce, Low-fat Yogurt and juice to drink. My question is, will I gain weight this week from no physical activity. Doctor: Thank you for your query,The main purpose of your Dentist advise, is you to prevent any strenuous physical activity. Strenuous physical activity can lead to blood clot coming off from the tooth socket which may hamper the healing process due to wisdom tooth extraction. You can gain some weight but considering the situation, its a better option."
},
{
"id": 149350,
"tgt": "Suffered seizure, viral encephalitis detected. On levopill. Abdominal pain, foot, stomach burning. Hernia noticed in CT scan. Diagnosis?",
"src": "Patient: my wife sufferred viral incephaleties in may 13.she had one seizure before detection of viral encephaleties.she recoved within a day after diagnosis.and on medication foe anti seizure i e levipill 500mg 3 tab per day.but we r getting same indications ie pain abdomen,head each,burning in foot n stomuch.again mri was done ,it was okn again ct scan was done so little hernia was noticed.can pl diagnose the ailment? Doctor: Hi,Thank you for posting your query.Viral encephalitis occurs due to an infection of brain due to a virus, usually herpes simplex. Once a patient recovers from the acute phase, recovery continues over time. Levipil (levetiracetam) is a good anti-seizure medication and it should be continued.Abdominal pain and hernia are unrelated to viral encephalitis. Please consult a general surgeon for opinion and treatment.I hope it helps.Please get back if you require any additional information.Best wishes,Dr Sudhir Kumar MD (Internal Medicine), DM (Neurology)Senior Consultant NeurologistApollo Hospitals, Hyderabad,My personal URL on this website: http://bit.ly/Dr-Sudhir-kumar My blog: http://bestneurodoctor.blogspot.com/"
},
{
"id": 135992,
"tgt": "What causes knuckle puffiness with stinging?",
"src": "Patient: Dominant hand center knuckle is puffy and stings. Is painful when touched. No open wound. Began a year ago. Have taken naproxin for a month with little change. Feels warm to the touch in the center. Any ideas of what to do next? Have been tested for gout and that was ruled out. Also, hand has been x-rayed and arthritis has been ruled out as well. Doctor: hiI think local infilteration with injection hydrocortisone or depomedrol may be tried.this may helpsubside non-specific inflammatory swelling of the knucklebest wishes"
},
{
"id": 63081,
"tgt": "What could red lumps on back and shoulders suggest?",
"src": "Patient: hi i am a 21 year old male and i have not really had any seroise medical conditions for a few years. recently i have begun to develope hard red lumps on my back and shoulders they dont hurt tell you try to squeeze them and wen they pop brown puss comes out followed by blood? is it seriouse? Doctor: hi.\"lumps\" on your back and shoulder might be cysts, sebaceous or keratinous type. these lesions occur anywhere in the body, slowly grow, sometimes get infected, producing an odorous fluid/collection inside, hence the puss you are describing. it is best if you consult with a general surgeon for medical and surgical management of your condition, especially if its bothersome for you.hope this helps.good day!!~dr.kaye"
},
{
"id": 74950,
"tgt": "Elaborate on lung transplantation",
"src": "Patient: I want to know about lung transplantation in your hospital. whether your hospital under take such operations. If yes, can you give me some details such as appx expense and no. of days stay. Patien's age is 45 and lungs are almost failed due to fibrosis. Doctor: Hey there,Well its not done in our hospital.You can attend Sir Gangaram Hospital Delhi for same.About cost and stay of days they will let you know."
},
{
"id": 214435,
"tgt": "Suggest natural products for removing bruise marks",
"src": "Patient: I have problems keeping my skin on my legs clear...on the calf muscle of one leg I have a formation of bruise-like marks and I have also developed a bruise mark on my shin...is there a cream that can keep these things at bay or maybe a natural products that can cut down the brusing marks...have been using propolis gel by SBC...quite a slow process....what can I do? Doctor: Greetings. First of all it's unfortunate that you are suffering from such a rare condition.->As of your problem is concerned follow instructions as below:* A cold compress such as an icepack or a bag of frozen peas should be applied to the affected area for 20-30 minutes in order to speed healing and reduce swelling by constricting the underlying blood vessels in response to the cold. Do not apply ice directly to the skin. Wrap the icepack in a towel.* If the bruise takes up a large area of the leg or foot, the leg should be kept elevated as much as possible during the first 24 hours after the injury.*After about 48 hours, heat in the form of a warm washcloth applied to the bruise for 10 minutes or so two or three times a day may increase blood flow to the bruised area, allowing the skin to reabsorb the blood more quickly. Ultimately, the bruise will fade in color.* If Pain, take medication after consulting your Doctor.Note:Finally it's just a guidance. Not a definitive treatment as we haven't seen you personally. You've to realize that these are temporary & symptomatic home remedies. We don't know what your exact condition is unless we see you personally.If you feel your condition becoming worse than before in spite of above, it's best advise to consult a General Physician who would treat you after ruling out other causes causing you bruise.Hope it helped for you."
},
{
"id": 210781,
"tgt": "What could be done to get rid of anxiety & stress feeling?",
"src": "Patient: I am 17 and suffer from anxiety & stress and I have been prescribed 20mg of propranolol a day, ive been taking them for about a week, I still feel anxious around people even my friends, I start to feel dizzy & sick around people, what do you think I should do? Doctor: HIThanks for using healthcare magicIf you have continuous anxiety, in that case only propranolol would not help you. You need a antidepressant that would help to control the underline anxiety. You should consult a psychiatrist for it. You can also take low dose benzodiazepine on sos basis and can also do relaxation exercise.Thanks"
},
{
"id": 188929,
"tgt": "Pain in knot which was stitched for removing wisdom teeth, feels like cyst. Normal?",
"src": "Patient: Hi there, how are you? I am 3 weeks post op from having my lower impacted wisdom teeth removed. Now that the pain from my dry socket thankfully gone, I've noticed a knot in my cheek where the doc stitched my cheek together to cover the flap of the extraction site. This \"knot\" is very painful and it almost feels like a cyst. Is this normal and part of the recovery process? Doctor: THANKS FOR THE QUERY. AFTER THREE WEEKS OF YOUR TOOTH EXTRACTION IF THE REMAINING SUTURE KNOT IS STILL PRESENT, THERE IS ALWAYS CHANCE OF GETTING INFECTION FROM THAT SUTURE KNOT. WHAT YOU FEELS LIKE CYST THAT CANNOT BE DIAGNOSED WITHOUT SEEING IT, BUT A CYST CANNOT FORM SO EARLY. IT IS A PROBABLY SOME GRANULATION TISSUE, MAY BE COVERED BY EPITHELIUM. YOU SHOULD VISIT YOUR DENTIST AS SOON AS POSSIBLE TO REMOVE THE 'KNOT' AS YOU DESCRIBED."
},
{
"id": 118710,
"tgt": "On Xarelto. If stopped will it increase risk of blood clotting?",
"src": "Patient: XARELTO lowers your chance of having a stroke by helping to prevent clots from forming. If you stop taking XARELTO, you may have increased risk of forming a clot in your blood. If you stop taking it does the risk remain the same as if you never took it at all or is there actually an new risk because you stopped taking it. Doctor: HiThanks for the queryXarelto is Rivaroxaban which is an anti coagulant, prevents formation of clots.If your doctor has prescribed this medicine to you, then obviously you are at some risk of developing clots.If u stop taking this medicine the risk of clot development will be there positively but yes , the risk will be same as before when you were not taking the medicine no newer risk will develop.regards"
},
{
"id": 70942,
"tgt": "What causes shortness of breath and dizziness?",
"src": "Patient: I have shortness of breath, headache, swelling in hands and feet, fatigue, muscle and joint pain, low on vitamin D, some light headedness, memory and concerns/ confusion. Recently the headache seemed to be like a migraine... I saw black flashes in front of me, tremendous pain, couldn\u2019t look at light, and became very fatigued. Doctor: Hello and Welcome to \u2018Ask A Doctor\u2019 service. I have reviewed your query and here is my advice. As you explain the history this might be a migraine headache. This needs painkillers such as Ibuprofen however it would be better for you to be examined by your doctor. Hope I have answered your query. Let me know if I can assist you further."
},
{
"id": 9502,
"tgt": "Toe knuckles have dry tough skin, tans become darker, looks dirty. How can I treat that area?",
"src": "Patient: Both of my big toe knuckles have dry tough skin on them. I can buff it off and moisturize and it will return in a day or two. My feet tan well in the summer and this dry rough skin always tans darker and just looks dirty. I am a 37 yo female. My only medical issues are allergies/ sinuses/ ears. Is this just dry skin or something else and How do I treat this area? Doctor: hi,thank you for your query.tanning of the toe knuckles is a common issue.and it is easily resolvable.Medical cause for it is Anemia or deficiency of heamoglobin in blood.it can be easily treated by using iron tablets for 30 daysin case its just due to tanning,you can start with a basic regimen to be followed daily in summer.1.start by soaking your feet in warm water with salt for 20 mins.2.scrub the knuckles thoroughly with lemon slice esp on the tanned areas.3.wash it off and apply moisturizer these three simple steps will ensure your feet remain soft and tan free..thank you,wishing you good health"
},
{
"id": 91352,
"tgt": "What medicine should I take for stomach pain /cramp?",
"src": "Patient: I m having diarrhea right now with stomach pain/cramps. I m ok with the constant bowel movement and I m willing to wait it out. It s the stomach pain that I want to get rid of. How do I do that without impeding bowel movement? I mean, I would prefer to constant bowel movement so that whatever is causing the diarrhea will be flushed out. Doctor: Hi.Thank you for your question.Let me ensure that you are fine and need not to panic.There can be lot of causes leading to such symptoms.The commonest one being bacterial infection of Intestine.I generally advice following to my patients:1)Antiobotic course preferably with Norfloxacin and Metronidazole for 5 days.2)Adequate diet.3)Oral Rehydation Solution to be taken frequently.Hope i answered your questions.Wish you a healthy life,Regards,Dr.Arun Prasad."
},
{
"id": 182862,
"tgt": "Why is the back of my tongue feeling heavy?",
"src": "Patient: Hi, may I answer your health queries right now ? Please type your query here... my tongue has been feeling heavy for a few hours i ate bar be que chicken then a piece of watermelon and ten minutes later, the back of my tongue was feeling heavy can you say why? Doctor: helloo...read through ur query..accordingly i must say that heaviness of tongue can.be due to burning away of the taste buds at back of ur tongue causing a feeling of heaviness..but nothing to be worried..it will be relieved in few days time...drink lots and lots of water,have vitamin supplements,green leafy vegetables etc..avoid too hot food for few days,follow this for a week within which it will ne healed...be cool and calm...tension can end up in more problems..after a week still persists meet a dentist and get it treated based on clinical presentation...hope ur benefitted something from this reply..have a healthy day!!!"
},
{
"id": 83805,
"tgt": "Is shelcal making me drowsy and sleepy?",
"src": "Patient: Hi, i am 54 year old lady .my Bp is normal 120/80. and my sugar is also normal. as per my doctors advice I am taking shelcal 500 gm tablet last four days . it is prescribed for one month. but today i am feeling head reeling drowsiness and sleepy. can you tell me why i am feeling so. Mrs.Pvk Doctor: Hello,Yes, in few patients it may cause drowsiness and headache. Consult with your doctor he may change the medication.Take care. Hope I have answered your question. Let me know if I can assist you further. Regards, Dr. Penchila Prasad Kandikattu, Internal Medicine Specialist"
},
{
"id": 60363,
"tgt": "If a 63 year old man who drinks everyday, has lower back pain. Is it Cirrhosis ?",
"src": "Patient: If a 63 year old man drinks a quart of voka a day, every day, and has lower back pain could it be cirrhossis if a man 63 year old man drinks more than a quart of alchol a day, everyday, and has lower back pain, could it be cirrohossis Doctor: Hi,Rosemary, Thanks for query, Lower back pain may be due to osteoartheritic changes in the pelvic bone or in vertebrae. As you take vodka for long time,to have any effect in liver you may go for ultra sound to rule out any problem in liver. Low back pain and cirrohosis has no relation. ok and bye."
},
{
"id": 60021,
"tgt": "Elevated ALT, bilrubin, SGPT, abnormal liver function test, stressful job, alcoholism. Treatment?",
"src": "Patient: Hi, i am a 37 male, working with a very stressfull job that requires sometimes staying seated for 12-13 hours. i am a heavy drinker for the last 3 years. the consumption varied with 2 pints a day to 4 pints regularly. all of my tests like Lipid , CBC are within normal range with HDL @62, LDL @86 TGL @164 and VLDL @33. the problem is with my Liver Function test . GammaGT came out @164, ALT @145. Bilurubin @0.2 and the SGPT was also within acceptable range. can i have an opinian what these things indicate. i have mentioned that exercise is one thing which is far from my busy life. age 37, Male, 5 11 , 73kg. could any body help! Doctor: Well your liver parameters are okay. They are not grossly deranged, so nothing active needs to be done. What you need to do is to cut down on your alcohol consumption."
},
{
"id": 116912,
"tgt": "Can prick from the needle from the tailor causes hepatitis c?",
"src": "Patient: I went to get some skirts altered by my seamtress (who is 74 years old). When I was taking one of the skirts off, one of the sewing pins she used to mark where she needed to sew scratched me. It wasnt actively bleeding but did break the skin and was red, which scabbed over within a couple minutes. My question is how likely would it be to transmit hep c say she had jabbed herself with the same needle before I scratched myself? Im thinking it would be low since she had stuck the pin through my skirt, which was pretty thick material. I know that wouldnt kill any virus but probably would make it very low risk? I suffer from anxiety and panic disorder so sometimes its hard for me to separate real risks from situations. Doctor: Hello, Thnx to contact us. I understand your concern. If I am your doctor I advice you that first of all you have to ask for the status of the tailor for Hepatitis C. There is no point of unnecessary worry if the status of the person is Hepatitis C negative. Even if she is positive you got pricked after piercing from the dress, so the chance becomes very less. I will be happy to answer more of your concerns, kindly know me,Wish you a very good health at health care magic. Dr. Arun Tank. Infectious Disease."
},
{
"id": 190172,
"tgt": "Silver cap in the tooth fell off. See a dentist?",
"src": "Patient: My son was laying down in bed and i heard him say My tooth i turned around and it was his silver cap. Just his cap not his tooth. He says it doesnt hurt him it just feels funny. A month or so back he was eating chips and I know he didnt chew it correctly and he poked himself. After that i started to see a redness on the side of his gum (right where the cap fell off) i put baby orajel on it and gave him tylenol the redness went away so did the swelling . I have no clue what to do its already late to go to a dentist is there anything i can do till the morning? Im freaking out! Doctor: Hello there, that's quite usual.....crown getting dislodged. There are various reasons.If the retention is not good it gets dislodged. Check out from your dentist if the same can be retained or requires a change of crown. It's adviced to get it restored."
},
{
"id": 91582,
"tgt": "Suggest treatment for severe stomach pain with dizziness",
"src": "Patient: Hello, my name is Jocelyn and i am 17 years-old. I'm looking for a doctors advice because my mom won't listen to me no matter what i tell her. I haven't slept at all tonight because i've had this god awful stomach pains to the fact that i feel like crying. It started out with my lower back hurting for about 3 days and it still does hurt but not as bad and now i feel like i have to constantly poop and vomit. I also have really bad dizziness and I feel really weak. I recently went to the bathroom, and it still didn't help anything, and when i looked it after i wiped it looked like coffee grounds in diarrhea. Not to be gross but i'm actually starting to get a little nervous. Can you please help? Doctor: Hi. Thanks for an elucidate history. Yes, you have a problem- this is gastroenteritis. The dizziness is suggestive of an active infection with loss of electrolytes into the intestine ( the reverse process due to infection). You need to consult a Doctor for clinical examination , blood pressure and pulse rate. You may need IV antibiotics and supportive medicines as there is vomiting also.. Request your Mother to take you to ER"
},
{
"id": 69447,
"tgt": "What causes small hard lump under the knee?",
"src": "Patient: I have a small hard lump (feels like bone) directly below my right kneecap. It's not under or behind the kneecap, but below. It does not hurt all all when I touch it, however, last night i felt intermittent pain sensations in the area as I laid in bed. Also, I can only feel the hard lump when I bend me knee.Any thoughts would be greatly appreciated! Thanks Doctor: Hi.Thanks for your query and an elucidate history.This can be problem with the tendon. There can be a fibrous band or a calcification of the tendon.This is confirmed by X-ray / Ultrasonography. Take an opinion of the Orthopedic Surgeon and follow his advise."
},
{
"id": 414,
"tgt": "Does pre ejaculating outside vagina cause pregnancy?",
"src": "Patient: Hi, My boyfriend and I were having sex and before he put the condom on he was rubbing his penis around my vagina. Not inside just around the labia. I was wondering could the pre ejaculation fluid still get me pregnant from outside of the actual 'canal' (if so, how?) Also would penetrating me straight after, push the fluids in me? Doctor: Thank you for trusting healthcare magic The chances of you getting pregnant are very very slim from what you just explained but you know it's not impossible. Studies have shown that in some individuals this pre-ejaculatory fluid does actually have some spermatozoa and as u mentioned, during the actual intercourse, what was deposited around the vulva might actually be pushed into the vagina and up to the cervix and hence pregnancy."
},
{
"id": 218488,
"tgt": "What causes lack of fetal movement during pregnancy?",
"src": "Patient: pregnant woman 34 weeks pregnant, she is not feeling any movement from the fetus in approximately 2 hours. She has attempted to re-position the fetus, no response of any kind from the fetus, she has consumed food in an attempt to stimulate movement, still no response. she is not sure whats happening and the next course of action to take Doctor: Hello and Welcome to \u2018Ask A Doctor\u2019 service. I have reviewed your query and here is my advice.The possible causes of decreased or lack of fetal movements are :With increasing gestational age as the baby increases and amount of amniotic fluid decreases sometimes movements may not be appreciated with same intensity as in lesser gestational age.Sometimes because of sleep wake cycle also around for one hour baby movements may not be appreciated.But but really there is possibility for signin fetal distress which should not be overlooked.So, as she is not having any fetal movement for 2 hours even after all attempts better to consult your gynecologist once, get examined if needed undergo ultrasound once to find out the well-being of the baby.Fetal Doppler is a small equipment which helps in finding the fetal heart rate that can be used at home also and helps in monitoring the well-being of the baby. If you want you can purchase one and can use at home. Hope I have answered your query and I will be happy to answer any further follow up queries. Take care."
},
{
"id": 137532,
"tgt": "Suggest treatment for subpublic discomfort",
"src": "Patient: Hello. I went the gyn for annual check up--52 years old. Have had mild subpublic discomfort, and slight aching for past 10 days. I am post menopausal (premature ovarian failure). Had had this feeling once or twice in my years...attubuted to possible UTI. Dr. did dipstick urine and came back positive for blood non-haemolyzed somewhere between just past the negative yelow trace 10 to the yellow 80. Was told that have to wait to culture comes back befoe she will do antibiotics--10 days. I am concerned that a UTI will progress. Your opinion...I understand it would be important to see if it cultures for infection to perscribe the correct meds, but 10 days is an awfully long time to wait, since I already have that UTI feeling... Doctor: Dear Patient,if the pain is more on micturation than the possibility is more towards UTI, but if you feel deep seated pain on the bone, than you could be suffering from what is called Osteitis pubis, this is a non infective inflammation of the pubic symphysis, and a short course of steroids under guidance of your doctor can be prescribed, but if taken in UTI they can be counter productive,Consult your doctor and discuss your concerns with him/her.Hope this helps you,Take care and get well soon."
},
{
"id": 139637,
"tgt": "What causes subdural hematoma?",
"src": "Patient: Hi, I m doing a case study about a 4 month old Amish infant who died of a subdural hematoma. Some suspect Shaken Baby Syndrome; but the case is also leaning towards liver disease and deficiency of vitamin K. Any suggestions on where this is heading? Thanks! Doctor: Hello, Liver diseases can cause increased bleeding tendency and a trivial trauma can lead to uncontrolled bleeding and that might cause subdural hematoma. The liver is the site from which vitamin-K is synthesized and which can be the cause for low vitamin-K as well. In normal kids vigorous shaking can cause concussions and subdural bleed is very unlikely. Hope I have answered your query. Let me know if I can assist you further. Take care Regards, Dr Shinas Hussain, General & Family Physician"
},
{
"id": 102586,
"tgt": "How to heal allergy symptoms like swelling after consuming certain foods?",
"src": "Patient: sometimes after I eat certain foods, the skin around my knuckles in the middle of my fingers (all of them) swell up...not the bones themselves but the skin...it is painful to move them due to the skin being so tight and swollen. Please help as I can't find any answers....thanks. Doctor: Hi,From history it seems that you might be having some food allergy giving rise this problem.Try to find out the food likely to give allergy and avoid taking that food.By trial and error you will find the food cause allergy reaction.Take anti-histamine like Cetrizine or Benadryl as and when required.Ok and take care."
},
{
"id": 158759,
"tgt": "Diagnosed with stomach cancer, had black vomit. On palliative chemotherapy. Is this normal?",
"src": "Patient: My brother has been diagnosed with stomach cancer (stage 4, metastasis) end of December last year and is currently undergoing his second cycle of palliative chemotherapy. He has been in near constant pain for the past 3 months and has yesterday started vomiting black vomit (for about 24 hrs). I've been told by some people that this is a sign that his death is imminent. Is this correct? Doctor: Hi and welcome to HCM. Thank you for your query. I am sorry about your brother and this serious diagnosis. Vomiting black vomit is a sign of upper gastrointestinal bleeding which can be more or less severe but in every case this isnt good sign. It can be bleeding which might be possible to treat with blood transfusions but not for long time.... Surgical or endoscopic treatment is probably impposible to perform. But hope for the best.Wish you good health."
},
{
"id": 19854,
"tgt": "What does incomplete right bundle branch block means?",
"src": "Patient: What does incomplete right bundle branch block and left atrial abnormality mean based on my ECG findings? I am a 45 year old woman, non-smoker and does strenuous 1-1/2 hours of Bikram Yoga almost everyday. I also work as a director of my own firm doing design and construction projects. Doctor: Hi There I understand your concern.I would like to tell you that Right Bundle Branch Block is an electrical conduction defect of the heart and can be seen in normal in some significant no of people. If you don't have symptoms like breathlessness, chest pain or palpitation then you need not worry.Hope this will help.Kind Regards"
},
{
"id": 191015,
"tgt": "Mouth cannot open fully",
"src": "Patient: SIR I 28 YRS OLD FROM ORISSA, I AM MUCH HABITUTED OF TOBACCO LST 5 YR BUT I FORGET TOBACCO FROM LAST 1 YEAR SIR MY PROBLEM IS PRESENTLY MY MOUTH NOT OPEN FULLY OPEND ONLY JUST ONE INCH KINDLY SUGEST ME WHAT I CAN DO FOR MY MOUTH PROBLEM? Doctor: hi, welcome to health magic, yes as the other doctors told you, u have sub mucous fibrosis, now this condition has to treated but treatment depends on the stage of oral submucous fibrosis, in the eraly cases injection theraphy helps but in the later stages surgery has to be done, it is best you see a oral maxillofacial surgeon who specilaises in treating submucous fibrosis you can contact me on 9879834033 Dr Adarsh Desai (Oral Maxillofacial Surgeon & Implantologist) Ahmedabad"
},
{
"id": 176854,
"tgt": "What causes frequent urination after having Nestle Nan Pro 1?",
"src": "Patient: Hi , my baby weight is 3.6 when he is born. After 1 month went for check up still my baby weight is 3.5. Doctor advised to give simlac 1 4 Times a day, simultaneously with BF. After a 2 weeks my child weight increased to 4.2 but simlac make my baby restless unable to burb, constipation, and gastric problem, again with doctor advice I switched over nestle nan pro 1 , it s comfortable for him but have frequent urinatione, will my baby gain weight can u advice. Am giving breast feed too. Doctor: Hello 1 monthold baby can utinate 6 to 7 times a day Which is normalInitially ther is less breast milk which was not enough for ur baby After giving him similac ur baby gain wt That means ur baby is need more milkIf u have enough bf then not to worry and also give nan 1 to him U should gradually decrease nan 1 after ur baby satisfied with bf No worry for urination"
},
{
"id": 155587,
"tgt": "Will topomax cause hair loss?",
"src": "Patient: Hi does topomax cause hair loss (thinning hair)? I was on it five years ago for migraine and most of my hair but unknown to me, I also had stage 3 thyroid cancer. I was treated for my cancer and anemia and have my hair back:) Once again, the doctor wants me on topomax for treatment of severe migraines. Hence, my question- was it the thyroid cancer, or anemia that caused the hair loss(almost baldness) or was it Topomax? Thank you kindly! Doctor: Hi and welcome to HCM. THanks for the query. most probably this was caused by thyroid cancer but topomax may had some small effect on it too.Wish you good health. Regards"
},
{
"id": 219898,
"tgt": "What causes abdominal cramps and blood clots during pregnancy?",
"src": "Patient: Hi,i used mifepristone tablets the day before yesterday,i then used misoprostol tablets (4) for abortion. I was 6 weeks and 2 days pregnant. I experienced cramps and bleeding with cloats,for almost two hours,the problem is am experiencing bloating and slight abdominal pain plus am stil bleeding,,is that ok? Doctor: Hello dear,I understand your concern.In my opinion presence of bloating and abdominal cramps along with bleeding is normal.The bleeding after taking the abortion pills is heavy with passage of clots initially.Then the bleeding decreases but will continue till a week or more.The cramps are due to uterine contractions to expel the products of conception.The bloating can be due to sideeffect of abortion pills.So whatever you are experiencing is normal. I suggest you to undergo medical termination of pregnancy only under medical supervision to avoid complications like heavy bleeding,incomplete abortion etc.Also kindly get an ultrasound after 2 weeks to check for completion of abortion.Hope this helps.Best regards...."
},
{
"id": 44479,
"tgt": "Semen analysis showed low sperm count, prescribed Biophene, Lenova, Exart5. What are the functions of these?",
"src": "Patient: actually i went one of the doctor regarding we need kid purpose, we done all tests. when i given my speram the report showing very small count. but i given my speram 2days without intercourse. the doctor suggested some tabletes. 1. BIOPHENE 2. LENOVA 3.LINEATOR 4. EXART 5. H1Q 300. so i need what purpose we need to use the above tablets? Doctor: Hi, your semen report is abnormal . you have mentioned about count but not about motility. count should be above 20 million /c mm and motility 60%active motility even after 6 hrs . in case of oligo astheno spermia medicines are prescribed hoping improvement of semen quality both in count and motility. semen analysis to be done after 3 months to see the result . it will be good to attend infertility clinic and evaluation of both the partners to be done . Treatment of infertility is a continued process and to be taken till get the result . Please discuss with your doctor and a check up to be done to exclude presence of any condition like hydrocoele,varicocoele etc that may require surgical treatment"
},
{
"id": 113722,
"tgt": "Suffering from back and neck pain. Not responding to over the counter medications. Help?",
"src": "Patient: Hi For the past 1 week i was suffering from the mid back pain and also with neck pain , i dont know the reason of this. I inhaled tablets from the medical shop but it gave me relief for 2 days but suddenly it starts paining again. What should i do now? pls help me with this asap.... Doctor: try muscle relaxant ointment and hot fomentation around the painful areas. if it doesnt help, take oral muscle relaxants with pain killer. only pain killer wont help."
},
{
"id": 185998,
"tgt": "Suggest medications for pain and inflammation post wisdom tooth extraction",
"src": "Patient: Sir I've under went my right upper wisdom tooth extraction last december..i was completely fine till last 4 days;now I;m having pain near the are of extraction..i can see a small inflammed skin..near that area which is touching the below teeth..i'm having pain while swallowing also... Doctor: Your extraction is fine.You did not mention anything about your right lower wisdom tooth. The lower tooth which is poking on the upper last tooth area is traumatizing this area. Check if the lower wisdom tooth is errupting now.You should get the lower tooth extracted or get the sharp tooth rounded.You must be on a course of antibiotics and analgesics for the infection.Regards."
},
{
"id": 102374,
"tgt": "What could cause pain during sneezing ?",
"src": "Patient: hi , i'm 51, male , 6'4\" 285 , twice today I sneezed and pain charged at my back , shoulders , armpits to about my elbows ...numbing with the pain .... subsided about 10 seconds after sneeze ... early am today I coughed hard from spicy breakfast sandwich and got about the same pain...don't think I feel congested.... Doctor: Hello,Welcome to HCM,The sneezing increases the intrathoracic pressure, whenever there is increased pressure it is distributed throughout the thoracic region.Whenever there is a vigorous sneezing it will causes pain in entire thoracic region and this pain will last for few seconds.If the pain is persisting for longer duration the causes should be find out.If the pain is continuing and persisting for longer duration of time you may require physical examination for ruled out any chest pathology.Thank you."
},
{
"id": 186778,
"tgt": "Having problem with bleeding gums",
"src": "Patient: Hi my name is Sai. I have noticed on occasions that I am spitting red saliva. This has happened on three occasions and I am not sure if it is to do with bleeding gums or another cause. I have brushed my teeth regularly however don't seem to see any blood. Can you give me some advise on what it may be? Doctor: Hi! Welcome to Healthcaremagic.I read your query. Do you have any known illness? Or some throat infection? Most probably it could be bleeding from gums, which can be due to gum disease as well as some systemic disease.Do you notice plaque or calculus on your teeth? Get oral prophylaxis done. Dental check can confirm if you have gum bleeding or not. Brush twice daily. Use mouthwash with tannic acid. Take healthy diet, multivitamins and minerals.If bleeding persists, see a physician and get blood test done for clotting factors, Hb, chest xray, etc.At times throat irritation may cause blood on spitting.Hope the answer helps you. Thank you!"
},
{
"id": 196340,
"tgt": "What causes crystal like discharges?",
"src": "Patient: hello dear Dr. my question is that my brother has been passing crystal like discharges pretty regularly, At first they appeared to be clear and now he is telling me it has pinkish color and that it happens often and that it's not painful. is it some thing he can do that is more in a way of natural approach? is there anything he can do to stop or slow down the frequent passing of the stones?? Thank you very much. Doctor: hello,welcome to healthcare magic.I am Dr Nirav.Pinkish crystal like discharge is due to passing of stone plus blood.Blood comes in urine because of adherence of stone to urethra.Take plenty of water and fluid.Take 4 litre of water.Have a better health."
},
{
"id": 208490,
"tgt": "What is the limitation of a person with a 77 IQ?",
"src": "Patient: My 7 year old foster to adopt daughter has an IQ of 77. She has an IEP and is receiving physical therapy and occupational therapy, will she be able to succeed in life when she gets older? Will she be able to live on her own when she gets older? What is the limitation on a person with a 77 IQ regarding jobs? Doctor: I appreciate your concern and query on HCM.Based on the history I would say that:1. IQ from 50-70 is considered Mild disability and such individual can achieve near normal language and social skills. However they need assistance with the home and job and can worsen under stress. 2. Under stressful situations they need more social support and customized help to face the challenges. I hope you find this helpful."
},
{
"id": 169537,
"tgt": "Suggest treatment for mild abdominal pain",
"src": "Patient: Hello Dr my 8 year old had fever 37.1 about 3 days ago with 4 loose motions we gave him ibugesic and sugar salt water mixture. Fever is not there and also the loose motion is OK but he is complaining of mild abdominal pain. He looks OK though very active and playing all sort of his sports. Doctor: Mild abdominal can be due to GI upset due to fever and loose motions. u can give Syp.cyclopam and syp.ranitidine for short duration."
},
{
"id": 156160,
"tgt": "Can lymphoma in neck, jaw, armpit and stomach genetically passed on?",
"src": "Patient: My ex husband has been diagnosed with lymphoma in neck, jaw, armpit and stomach -- he is to see specialcancer dr to discuss treatment ---what treatment is given - we had 2 children - both now adults - what are the chances of this disease being passed on and what should they do - I believe it is genitic Doctor: Hi and welcome to HCM. Thanks for the query. there is genetic component in these diseases but this is no t so significant and they should just do blood tests more frequently. Wish you good health. Regards"
},
{
"id": 174488,
"tgt": "What are the side effect of adderall in child?",
"src": "Patient: Thanks. I had asked my child s doctor to prescribe her a booster dose of her usual medication (Adderall) to help her concentrate in the evenings while she does her homework. Instead he prescribed guanfacine! She s been on guanfacine in the past (it was the first ADHD med we tried) and it didn t do much for her. Why do you believe he did that? Would more Adderall not be approved, or might he have been worried about her ability to get to sleep at night? Doctor: Hi,Thank you for asking question on health care magic.Longterm use of adderal is approved but over dose cases certain untoward symptoms relating several systems.Hope this answer will serve your purposePlease feel free to ask any more queries if requiredTake careDr.M.V.Subrahmanyam MD;DCHAssociate professor of pediatrics"
},
{
"id": 80689,
"tgt": "Suggest remedies for cough,dizziness and nausea",
"src": "Patient: Hi so I have been coughing heaps and and feel very dizzy. When I walk or move I get that dizzy feeling like when you get up to fast, kind off like blanking out for a second. I am tried and feel nauseous and it just seems to get worst. I don t know what s wrong :( also have no energy Doctor: Hello dear, thanks for your question on HCM. Nausea, coughing and low energy can be seen in uncontrolled GERD ( gastroesophageal reflux disease ). It is due to laxity of gastroesophageal sphincter. Because of this the acid of the stomach tends to come up in the esophagus and cause nausea and coughing. So better to follow these steps for better symptomatic relief. 1. Avoid hot and spicy food. 2. Avoid stress and tension. 3. Avoid large meals, instead take frequent small meals. 4. Start Proton pump inhibitors. 5. Go for walk after meals. 6. Keep 2 - 3 pillows under head in bed to prevent reflux. Don't worry, you will be alright."
},
{
"id": 10100,
"tgt": "How can persistent hair fall since childbirth be treated?",
"src": "Patient: Hello doctor, My problem is about my hair fall. It passed10 months of my post delivery. And my hair fall started nearly from 6 months after delivery. It s very horrible day by day loosing so much of hair. Kindly suggest me with good medication. Thank you Doctor: Hello, You seem to have postpartum telogen effluvium which is hormonal due to a withdrawal of hair stimulating effect of estrogen on hair during pregnancy. This is usually self-limiting and recovers in 3-6 months. However, I suggest you take a well balanced nutritious diet along with an oral biotin supplement. Correction of any deficiency like serum iron and vitamin D, vitamin B12 would also help. Hope I have answered your query. Let me know if I can assist you further. Take care Regards, Dr Kakkar S., Dermatologist"
},
{
"id": 2564,
"tgt": "What are the chances of pregnancy as experiencing symptoms after tubal ligation?",
"src": "Patient: I just turned 43, had a tubal (tubes cut and tied) when I was 22yrs old, never missed a period until now (15 days late) my abdominal is growing, a week ago I felt dizzy, faint, nuasea, have been experiencing breast enlargement, weight gain even though I exercise and sharp needle sensations, at time, under my pelvic area, basically feeling like I m pregnant, could this be possible? Doctor: hello, as you have mentioned if the tubes are CUT and tied it is unlikly to recanalize the tubes. there are instances where tubes are recanalized. but that has happened mostly after tubes are only ligated without cutting. but there are instances where healthy pregnancies have happened after tubes are cut ligated and burnt. Best thing would be to do a urine hcg test. If it is negative , repeat the test and a ultrasound scan in another 2 weeks. if it is also negative what i can say is it may be a premature menopause with perimenopausal symptoms or a case of dysfunctional uterine bleeding.hope this helps .thank you"
},
{
"id": 59220,
"tgt": "26 year old with high bilirubin levels. Need to be concerned?",
"src": "Patient: hello, I am 26 y/o/f and healthy to my understanding. I recently had blood work done and what quizzed me was my higher then normal bilirubin count it is 0.29mg/dL. I know from where the bilirubin comes and how it is produced. I am just worry if the level I have should concern me? Ps. I consume a glass of red wine per week or less, not on any prescribed med, I take supplements only Doctor: Hi and welcome to HCM. You dont have to be worried at all. this is almost normal bilirubin. Some doctors even consider normal range till 0.3. If you dont have raised liver enzymes or any symptoms this is really nothing to be concerned. If you repeat tests in few weeks it will probably be normal. anyway, high bilirubin is elevated ueually in liver damage,most common alcoholic,viral or autoimunne or in some hematological disorders, but this elevation would be much higher than you have. SO i suggest just to repeat liver findings n 4 weeks and see is there any progresion. Wish you good health."
},
{
"id": 112907,
"tgt": "Severe back pain after straining. Reason?",
"src": "Patient: I am a home health aide and yesterday I lifted my client that ways the same as me. The first lift I felt bad pain in my lower to middle back and it sounded and felt like tiny bubbles popping or like rice crispies crunching. I ve had horrible back pain all day yesterday, all night and now my lower up to middle back is killing me! Horrible sharp pain. Now I also feel like I have to pee a lot but I go and it s just a very small amount. What could ve happen when I lifted my client and felt crunching in my lower to middle back in the center right on spine??? I m in so much pain!!! Doctor: Hello and Welcome to HCM, Thanks for writing to us. The symptoms you have described are more likely to be symptoms of muscles strain. To get relief from the pain and for quick healing: 1. Do hot fomatation at the site. 2. Apply local anti-inflammatory gel 2-3 times a day. 3. Take anti-inflammatory drugs like Ibuprofen to reduce pain and local swelling or inflammation. 4. Do not lift weight or try to do exercise that involve painful muscles. Give proper time for the injury to heal. Hope this helps you. Wishing you good health... Regards. Dr Saurabh Gupta. Orthopaedic Surgeon. (H1N1)2009"
},
{
"id": 18614,
"tgt": "What causes rapid heart rate and fatigue?",
"src": "Patient: Hi, my heart rate has been incredibly high lately. Movement causes it to spike but it s relatively normal at rest. I m incredibly fatigued lately almost always and have no appetite, and trouble sleeping at night. I m also having headaches and general muscle aches all over my body. I m not sure if it s related but my hands have been shaking and I have muscle twitches from time to time since my symptoms have begun. I m 26 years old, Male, and weigh 200lbs, and 6 2 . I ve been exercising lately and have always been in pretty normal shape, but looking to lose some fat and gain muscle. The heart thing is freaking me out though. I don t have health insurance currently and can t go to my doctor unless it s absolutely urgent, and even then it ll be putting me in debt to go to the ER without insurance so I m really hoping to avoid it altogether. Doctor: Hello and Welcome to \u2018Ask A Doctor\u2019 service. I have reviewed your query and here is my advice. Your symptoms could be related to a metabolic disorder: chronic anemia, vitamin D deficiency, thyroid gland dysfunction, etc. For this reason, I would recommend consulting with your attending physician for a physical exam and some tests: - A resting ECG - A chest X-ray study - Complete blood count for anemia - Thyroid hormone levels for thyroid gland dysfunction - Blood electrolytes - Vitamin D plasma levels for possible deficiency. In the meantime, I would recommend you to closely monitor your blood pressure values. An ambulatory 24 to 48 hours ECG monitoring would help examine your heart rhythm trends and investigate for possible cardiac arrhythmia, if all the above tests result normal. You should discuss with your doctor on the above tests. Hope I have answered your query. Let me know if I can assist you further. Regards, Dr. Iliri"
},
{
"id": 125111,
"tgt": "What causes soft tissue swelling in the legs and tightness from the hips to the knees?",
"src": "Patient: my wife is 59 years old and has soft tissue swelling in her legs from the hip to the knee skin is tight and painful has been checked for clots and blood flow but no problem she has fibromyalgia but nothing else family doctor does not offer any solutions please help Thanks Phil Doctor: Hello, Consult a physician and get evaluated, we have to rule out conditions like renal dysfunction and low protein which can cause similar symptoms. Hope I have answered your query. Let me know if I can assist you further. Take care Regards, Dr Shinas Hussain, General & Family Physician"
},
{
"id": 101340,
"tgt": "What could cause itching (face and limbs) on high sugar intake?",
"src": "Patient: I'm 25 yrs old, African American female with no past medical history. I notice every time I eat or drink something very sweet (in large amount)I start to itch on my face, neck and arms. What could be the reason for that? Is it some kind of sugar intolerance? Doctor: HI, thanks for using healthcare magicIt is more in keeping with an allergic reaction to some component of the meal.Allergic responses tend to worsen as a person is continually exposed to the stimulus.If you wish to be sure exactly what you are allergic to you can ask your doctor about allergy testing. This can be in the form of blood tests or skin tests.It would be best to avoid the triggers.I hope this helps"
},
{
"id": 187555,
"tgt": "Reason for sore on roof of mouth that is painful?",
"src": "Patient: For about a week I've had a sore on the roof of my mouth, but I've looked at the area with a flashlight and seen nothing. There's no redness, whiteness, swelling or anything else I can see. It doesn't hurt as long as my tongue doesn't move or put pressure on it. Its on the roof of my mouth, near the gums of my very back molars. It's a sharp pain. It feels kind of like like how you'd imagine rubbing sand on a rug-burn would hurt. It gets worse throughout the day, probably because of the friction of my tongue on it. Any ideas? Doctor: Hello, Thanks for your query.The roof of the mouth is divided into two major parts. The hard palate is a bony area in the anterior region of the mouth. Behind it, towards the back of the mouth, is the fleshy soft palate, which lacks a bony plate. Pain in this area may be due to a number of issues. A burning sensation on the palate along with the tongue and cheeks may be burning mouth syndrome, a chronic condition that does not have a clear cause.Discomfort accompanied by open sores may be canker sores or herpes.Effective treatment depends on the cause of palatal pain. Non-viral infections can be treated with antibiotics or, in the case of thrush, antifungal medication. The cause of burning mouth syndrome is unknown, so the best course of action is determined on a case by case basis. Canker sores, herpes and mild inflammation due to irritation usually resolve themselves on their own, but antiseptic mouth rinses can help with symptoms. Whatever the issue may be, it is best assessed by a doctor who can give a diagnosis and a plan of action for treatment.I do hope that you have found something helpful and I will be glad to answer any further query.Take care"
},
{
"id": 121904,
"tgt": "How to treat polio leg?",
"src": "Patient: Docter , i am Nandini ,49 yrs .i have a polio leg , was given a muscle transplant 30 yrs ago , as my upper thigh muscles were dead , ten yrs ago i atarted having immense pain in my knee , was again operated and a leg supporting plate was fixed in my lower leg ..i have less pain but ..as my spine is a little thick because of a bad posture ..and as muscles have gone weak after surgeries ..is ther any suppport system which can help me walk ..any machine or the like..as i wud like to walk ..even if little ..so that i may live self dependent .. regards and thanx Doctor: Hi, This is a very individualized case and NERVE CONDUCTION STUDIES would show if the nerves are working. They often shut down or change years to decades after the polio. If it is just weak muscles and the nerve conduction tests will show that if they are done by a somewhat smart doctor, then plain gradual exercise will help it in fairly short order. Hope I have answered your query. Let me know if I can assist you further. Regards, Dr. Matt Wachsman, Addiction Medicine Specialist"
},
{
"id": 142519,
"tgt": "Could eye balls turn puffy and yellow as a result of brain surgery?",
"src": "Patient: My dad just had to have brain surgery because he had cancer on his brain. Now his right eye from the side closest to the ear to the eyeball looks puffy and yellow. It kind of looks like something is growing on his eye. His brain doctor told him it was nothing to worry about that it was just a water pocket. Doctor: Hello!Welcome on Healthcaremagic!I understand your concern and would explain that this edema is related to the accumulation of liquid under the skin in this area. This situation is not indicative of any complication. But it is temporary and it will recover spontaneously. Hope you will find this answer helpful!Kind regards, Dr. Aida"
},
{
"id": 182927,
"tgt": "How to get rid of pain in my teeth?",
"src": "Patient: Hi, may I answer your health queries right now ? Please type your query here...My friend stopped using herion about a week ago.She is feelung better but says her teeth hurt really bad. She has pretty teeth and I was an addict once an remembering this happened to me. WHat can she do to help the pain in her teeth? Doctor: Thanks for using health Care Magic.Read your query.Heroin is very injurious to your health and its good that your friend has stopped taking it. She is not to be taking it again.For the pain if severe, ibuprofen tab ( if not allergic to any medicine ) can help. I would advice her to visit her local dentist to identify the cause of the pain and have the needful treatment done. Use chlorhexidine mouth wash.Have her teeth cleaned by the dentist and maintain a good oral health.Hope this was useful.Thanks and regards."
},
{
"id": 211017,
"tgt": "How to treat extreme bipolar disorder,anxiety and occasional night terrors?",
"src": "Patient: my mother, age 78 has hearing loss. should I schedule an appointment with an ear noae an throat doctor, or should i schedule an appointment with an audiologistnever mind...not going to pay 15 $ for that...i will just ask one of the ER docs I work with Doctor: Hello,Thanks for choosing health care magic for posting your query.I have gone through your question in detail and I can understand what you are going through.You should get an appoint ment with an ENT specialist and if required and advised by ENT specialist, audiologist can be asked to do the audio testing. Audiologist will not treat her. Hope I am able to answer your concerns.If you have any further query, I would be glad to help you.In future if you wish to contact me directly, you can use the below mentioned link:bit.ly/dr-srikanth-reddy"
},
{
"id": 179921,
"tgt": "Suggest treatment for stomach pain and burping",
"src": "Patient: my 8 yr old is lactose intolerant. Today, at a friends she had lots of mac n cheese and some pizza. Has been complaini g of stomach pain since about 3 pm, been to toilet several times but bm is just soft not actual diarrhea, but I a lot of pain, having trouble sleeping tonite. Difficulty burping to get the gas out and threw up twice. I gave tums this afternoon and warm ginger tea tonight. 2 slices toasr with little butter for dinner. Eventually gave her some vhildrens tylenol. How to stop the pain, help the burp vome out? Doctor: Thanks for posting your query at HealthCareMagic. Since you have mentioned, let me mention that your son has lactose intolerance and not milk allergy which are two different ailments. As of now, Lactase enzyme supplements will be able to provide him relief. You may also give him some drotin (drotaverine) tablets to relieve the cramps.The degree of intolerance differs with each person. The best way to assess your tolerance is first to get all lactose out of your system by avoiding all lactose-containing foods for three to four weeks. Then start with very small quantities of milk or cheese. Monitor your symptoms to see how much or how little dairy food you can handle without experiencing discomfort. Once you know your limits, management becomes a little easier.Cocoa powder. Studies indicate that cocoa powder and sugar, or chocolate powders, may help the body digest lactose by slowing the rate at which the stomach empties. The slower the emptying process, the less lactose that enters your system at once. That means fewer symptoms. The calcium in chocolate milk is just as well absorbed as that in regular milk, and you may tolerate flavored milk better than plain.Also note, nondairy foods that contain calcium, include:* Broccoli, okra, kale, collards, and turnip greens.* Canned sardines, tuna, and salmon.* Calcium-fortified juices and cereals.* Calcium-fortified soy products such as soy milk, tofu, and soybeans.* Almonds.I hope that helps. Feel free to revert back in case of further queries."
},
{
"id": 63400,
"tgt": "What causes lump on neck?",
"src": "Patient: Hi, I am a 23 year old female, fit, active, eat healthy, when i was little i did have a lump on my brain, but no further problems. Recently i have noticed a small hard lump about the size of a pea on my neck, its up towards my earlobe and jaw. It has been there for a few weeks now. I also have a few small ones on the back of my neck that have been there for about a year and tend to come and go... the one on my neck hurts a bit sometimes... any ideas? Doctor: HHi,Dear thanks for the query to HCM Virtual Clinic.I studied your query in all the details given by you.The lump you have seems to be chronic Lymph adenitis.Cause of it could be scalp infection,or infestation with lice.Other causes of lymph node enlargement as TB Adenitis need to be ruled out by consultation with your doctor.The one which hurts you needs attention,hence I would advise you to consult Er Surgeon.Hope this would help you to act fast and plan treatment with your doctor.Hope this reply would satisfy your query.Write excellent reviews if you like this reply and find it helpful to you,as your comments and reviews would improve my rating for the visitors of the HCM clinic.Welcome for any further query in this regard to Me.I would love you to help out of this intricate problem of lump.Wishing you fast recovery.Have a good day.Dr.Savaskar M.N.Senior Surgical SpecialistM.S.Genl-CVTS"
},
{
"id": 210674,
"tgt": "Does molestation at 7 affect an adult at 50?",
"src": "Patient: so does a molestaion at 7 affect an adult at 50 usually I had a cousin molest me and I told my father HE told my brother that he knew something had happened but he couldnt do anything because it was his brothers son..my mother wrote a leetr to the molester cousin and didnt tell me until afterwards tSO then the other brother died twwo yrs later and he also tried to moleats me at 13. thats when it all cmae out and I felel like Im in a dark hole The funeral facilitated my talking to the other one and I even wnet out to dinner to talk about it to him but he didnt say much and I didnt say much I said I would never do anything to him but I wnated a settlemt for counseling and damages he wanted to know who my counslelor was THAt infuriated me hed taken all of my power awya already and then I told the police and my family has alienated my daughter form me using other things as an excuse for the wya I am .. or was kind of timid and not tolerant to arguing always letting my father intimidate em and she was creul and told me no one would ever want me my mother NIce after all I went through Doctor: HiThanks for using healthcare magicYes, sometime such kind of sexual abuse at childhood, effect on personality development and could later lead to psychological problem in form of depression, anxiety disorder or schizophrenia. Better to take help of a psychologist to keep mental balance maintained. Try to share your feeling would someone, whom you can trust. That would help you.Thanks"
},
{
"id": 126061,
"tgt": "Should Azulfidine be taken for edema in the knees?",
"src": "Patient: I ve been diagnosed with RA due to a pretty high RA factor but have no pain and, as yet, no joint deformity other than some osteo in my hands. I was originally diagnosed with CPPD, have recurring swelling in knees, a high nucleated cell count but now, after two years, no intracellular calcium crystals. Rheumatologist, a rather doddery one, wants me to start on Azulfadine after 5 months on meloxicam haven t helped keep fluid off knees. I m reluctant to take a heavy duty drug when I have no pain, no symmetrical joint derformity, no RA in xrays, and no pain. Do you think I ought to at least try Azulfadine? Doctor: Hi, It is used to treat rheumatoid arthritis. Also, used to treat ulcerative colitis etc. Many drugs are there for rheumatoid arthritis. These drugs are useful to treat the disease under control. Treatment depends upon the serum RA factor, anti-CCP levels. Please consult your rheumatologist he may advice right dosage of the drug. Hope I have answered your query. Let me know if I can assist you further. Regards, Dr. Penchila Prasad Kandikattu, Internal Medicine Specialist"
},
{
"id": 199706,
"tgt": "What is the itchiness and redness on my penis?",
"src": "Patient: Hi I am not having any burning sensation as I pass urine I am a 62 year old male who was circumcised aged 6 weeks old, I have type 2 diabetes, as I have aged my penis has shrunk to look as if I have a foreskin, my problem is that I have a redness and itch at the head of the penis almost looks like a scald, dam annoying Doctor: Hello I can understand your concernLooking at your history this could be related to STI or UTI which is quite common in diabeticsin my opinion your should consult your GP doctor for examination and a battery of tests like blood for hemoglobin CBC liver and thyroid function test , urine for culture and sensitivity test you may require a course of antibiotics depending upon the clinical findings and lab reportshave plenty of fluids maintain good hygienemultivitamins supplementsHope.this help youPlease fell free to ask for.more clarification I will happily answer you best wishes"
},
{
"id": 117518,
"tgt": "What causes elevated WBC count?",
"src": "Patient: I have had a WBC of 14,000 to 18,000 for the last 2 years taken every 3 months as my doctor says he's \"watching it\". 2 weeks ago my blood work came back with a slightly elevated (again, as usual) WBC of 14,000 , low granulocyte# (8.6 with the normal being 35-80 according to the blood work) elevated monocyte# of 1.20. What does that mean? Should I see a specialist? I take 12.5 attenolol for SVT and ,625 Premarin and a multi vitamin. That's it, Other than general fatigue and hot flashes- I don't feel sick really. Just tired. Doctor: Hi,Thanks for asking.Based on your query, my opinion is as follows.1. Neutropenia with monocytosis and an elevated WBC count requires further evaluation. 2. Chronic infection is a strong possibility and correlation with ESR and CRP levels necessary.3. Further history necessary to identify possible cause. You should meet an internal medicine specialist.Hope it helps.Any further queries, happy to help again."
},
{
"id": 173811,
"tgt": "What causes a red dot in the white of an eye in a child?",
"src": "Patient: My 18 month has a red dot in the white of her eye - doesn't seem to be bothered by it and no other symptons but I don't know if it could be something to worry about it? Do you know what causes this and if she should see a doctor? Will the red dot go away on its own? Doctor: I am pleasure to answer your question. The red spot you see is most likely a small blood vessel. In medical terms, it is called a subconjuntival hemmorhage. Small blood vessels lie underneath the conjunctiva, the clear coating that covers the sclera, the white part of the eye. But most often feel no pain, irritation or light sensitivity.A subconjunctival hemmorhage can occur if a person has too much pressure on the upper part of the body or head. Pressure can be created by straining too hard while lifting something very heavy or coughing or sneezing. Straining during constipation can also cause enough pressure to burst a blood vessel in the eye. Other causes are high blood pressure, diabetes, trauma or a high dosage of blood thinners, such as aspirin.If a red spot in your eye lasts for longer than a few days, you should schedule an eye exam. Your eye doctor will try to determine the cause of the subconjunctival hemmorhage.Hope it is helpful."
},
{
"id": 130511,
"tgt": "Suggest treatment for wrist pain inspite of having Sjogren s Syndrome, hypothyroidism and osteoarthritis in my fingers",
"src": "Patient: I am a 57 year old female with Sjogren s Syndrome, hypothyroidism, osteoarthritis in my fingers. My wrist on the thumb side of my dominant right hand started bothering me about 8-12 months ago. I work with a computer and phone and write a lot. I use a wrist foam pad in front of my keyboard and tried using a device with the mouse for keeping the hand in the right position, but that didn t seem to make a difference. So, I trained myself to use the mouse with my left hand and write with my right hand. It seemed to help for awhile. But the pain gradually returned and worsened. Writing and using the keyboard with my right hand was causing some pain, swelling and partial numbness in the right wrist an inch or so below the thumb. My forearm would ache. Then I noticed the right shoulder aching. I could not move my elbow out away from my body without a lot of pain. I could not sleep on it at night. Then my right arm began aching also. I went to my primary care physician and she put me on Mobic. Later, while driving, I started noticing tingling in my LEFT hand on and off. Both shoulders were bothering me and tightness in my neck. Then I worked late for a couple of times. In the middle of the night, my left hand was burning ON FIRE. It was tight and somewhat swollen. It started with the little and ring finger and gradually moved to the middle finger, then index finder. I held it in cold water and the pain was tolerable. I slept a couple of hours and woke up with it burning again. The left hand has improved in the same order, finishing with the index finger., but it is a little weaker, but getting better. Both hands tire easily and sometimes tingle, like if I text for awhile, though the pain is better. Doctor: Hi,Do hot and cold therapy (do take a bowl of lukewarm water and another of cold water, dip your hand in hot water for three mins then dip in cold water for one mins. Do it 7-10 times). Do some active and light resistance exercises for your upper extremities, along with do finger, wrist stretching exercises.Hope you will find the answer useful. Let me know if I can assist you further.Regards,Dr. Harsh Swarup"
},
{
"id": 131330,
"tgt": "What can i do to ease out muscle spasms and pain?",
"src": "Patient: I have c5-6 Foraminal stenosis diagnosed 6 years ago. I was put on soma and that along with accupuncture, massage and specialized manual physical therapy worked well ( I chose to not take any pain medication for those years). Unfortunately I was rear ended in September of this year and now the muscle spasms and pain are about to end my life. Soma and Vicodin (I have always taken small amounts with never any recreational drugs or alcohol used. I was referred to a pain doctor with his own agenda who decided I was a 'druggie' despite multiple sites of injury in my body due to many years in the healthcare industry lifting heavy patients, walking and neck/shoulder injuries) I hold dual medical degrees so I am knowledgeable and careful with any pain medication and realize the risk/benefit balance. I have taken soma on and off for 12 years. I actually was originally given 4 a day and for many years only took 1. I did not ever experience withdrawal despite discontinuing 2 times due to a period of relative 'ease'. Now I am older and my body is no longer able to compensate for the structural defects. I have been told my condition is not a good one for surgery. What am I going to do? I only want to work, quilt and occasionally garden. Any suggestions? I am sorry. I read your advertisement/website and I don't think you are an ethical company that will actually do anything. so I am not going any farther. Doctor: HiNowadays,minimal invasive spine surgery is possible,consult a spine or neurosurgeon.it seems foraminotomy,laminectomy and or fusion of vertebra are apparent remedies.Lyrica,methycobalamin, tramadol may be alternative medicines.If disc issues are also co existing,epidurals or ozone therapy may be considered alternatively if for technical conditions you can't undergo surgery.opioids are best taken for acute conditions as a short course.Use of cervical collar and isometric neck exercises may be done if condition permits.Also cervical traction if this can be done in your position"
},
{
"id": 80931,
"tgt": "Suggest treatment for severe chest pain",
"src": "Patient: I had a picc placed august 17 I had 14 days of outpatient IV infusion of invanz afterwards. Due to no primary care no one wanted to rove it until I demanded it b taken out due to sever pain n my chest close to my right breast. The pain has not clears up and seems to be getting worse. Can you tell me what it could be?? Doctor: Thanks for your question on HCM.I can understand your situation and problem.PICC is cardiac device and use in monitoring of cardiac diseases.So you are having cardiac disease for sure.I advice you to consult cardiologist and get done1. ECG2. 2d echo3. Stress test (trade mill test)To rule out cardiac cause.Because your chest pain is looks like due to cardiac cause more.If all of the above are normal then no need to worry much for cardiac cause.Take simple painkiller for your chest pain.Don't worry, you will be alright."
},
{
"id": 127138,
"tgt": "How can difficulty in walking due to a knee injury be treated?",
"src": "Patient: Feb/2018 I fell and hurt my knee x-ray and Doctor told me I dislocated my knee cap. For 2 months my physiotherapist has been working and we have been doing exercises. At about 2 1/2 months my doctor says I should be better than I am and sends me for MRI and that came back as nothing wrong with my knee cap that it is fine and that I have a incomplete non-displaced fracture of the lateral tibial plateau posteriorly. Been in a brace since 1 week after accident. Can t walk without the brace and my whole knee area feels very foggy Doctor: Hello, It will take sometimes for complete recovery. As the fracture is undisplaced surgical correction may not be required. You can continue physiotherapy and gradually you will be able to walk with out support. Hope I have answered your query. Let me know if I can assist you further."
},
{
"id": 5224,
"tgt": "Cannot conceive. Did sonography. Have PCOS. On Oosure. Have a cyst also. Chances of pregnancy?",
"src": "Patient: Hello, I have been trying to conceive but since it was unsucessful for about a year went to a doctor and had done sonography tests in the month of january 2013 and my gynaecologist said i am having PCOS and prescribed to me Oosure tablets for a course of 6 months which I am still taking and then to take Clomipure for five days between day 3 - day 7 for four months. If these medicines do not help me conceive within the next four months what is the next step? I also have occasional pain on my left lower pelvis area which sonography suggested a small cyst but it keeps coming and going every cycle but yet it is very painful. Doctor: Hello, I would be happy to help you with your question. You are on the right course with clomid. I agree that after 4 months, re-evaluation should occur. Given your cyclic pelvic pain, this might include a laparoscopy to evaluate the pelvis for endometriosis and patency of the tubes. If this workup was normal, then the next step would be a fertility clinic and possible IUI. I hope that this helps. If you have more questions, please click on \"ASK ME A QUESTION\" link above to directly ask me a question!"
},
{
"id": 65396,
"tgt": "What could a puss-filled lump after mole removal indicate?",
"src": "Patient: Hello! I had a mole under my right ear, which was removed with the electro cautering device 2 months ago, biopsy went out ok, not malign. and in the scar place in the lower part i had like a small lump with a little bit of puss. The area was hurting, so i squeeze gently and a little bit of puss came out and some blood, it doesn`t feel at pain anymore. I applied a little bit of betadine on it. I hope it`s nothing bad , can you answer me please, thanks so much. Doctor: Hi. Thanks for the query. As per your question, I believe you contacted an infection of that region following the procedure. As the pain has come down , it must have healed. If there is persistent pain and pus, better take a course of antibiotics and painkillers. If not, just relax. You ll be fine."
},
{
"id": 114795,
"tgt": "Should i be concerned about the platelet count of 397?",
"src": "Patient: I just had a lab done and my platelet was 409. Then my doctor had me do a follow up in a month and told me to drink a lot of water before testing. I only had one bottle of water the morning prior to my test. My results came back and my count was 397. Should I be concerned? And is there anything I should be doing? Doctor: Hi, dearI have gone through your question. I can understand your concern. Your platelet count is absolutely normal. No need to worry about platelet count. No any drug treatment or follow is required. Just be relaxed. Hope I have answered your question, if you have doubt then I will be happy to answer. Thanks for using health care magic. Wish you a very good health."
},
{
"id": 42263,
"tgt": "What does it mean to have oligozoospermia or azoospermia?",
"src": "Patient: i am married since five years but my wife has not conceived so far. I did many tests both of my wife and myself. Tests pertaining to my wife are completely ok. For me the tests are either showing oligosospermia and sometimes azoospermia. I am totally confused.Please help me out Doctor: Oligospermia means low sperm count and Azoospermia means zero sperm count.First you have to get a hormone assessment [FSH,LH,TESTOSTERONE] to check whether its a sperm production problem or sperm transport/blockage issue."
},
{
"id": 626,
"tgt": "Is it possible to have children at age 44?",
"src": "Patient: Hello Dr chetna, I am from Mumbai , and my age is 44, i got married since three years and now planning to have a child. i know its too late but i wish to have a baby , but unable to concieve since six months , I dont have any other child. I would like to know whether it is possible ever to get a child. can i personally meet an expert doctor like you? Doctor: it's Dr deepti here. yes one can get pregnant at this age provided there are no health or reproductive issues. practice regular sex during fertile period and take folic acid tablets. and make sure you concern and have regular visits to an obstetrician once you conceive as pregnancy at this age is considered to be as high risk pregnancy"
},
{
"id": 22600,
"tgt": "Why can't bypass surgery be performed if having low platelet count?",
"src": "Patient: My mom is a renal failure patient she has three coronary arteries that are blocked and the one that is working is leaking they first wanted to do a bypass however her platelet count dropped and now they want to put three stents in why have they changed the plan Doctor: Hello and welcome to \u2018Ask A Doctor\u2019 service.I have reviewed your query and here is my advice.Low platelet count is a contraindication for any surgery especially a major surgery like bypass. Low platelet count can cause massive bleeding and thus a threat to life. This is why they have advised you against bypass. But still angioplasty also needs giving blood thinner and in low platelet count it can also cause significant bleeding. We generally perform it when platelet count is more than 1 lakh. Hope I have answered your query. Let me know if I can assist you further.Regards,Dr. Sameer Maheshwari"
},
{
"id": 77881,
"tgt": "What causes bruise on the legs with shortness of breath?",
"src": "Patient: I ve been having unexplainable, frequent bruising on my legs and arms as soon as some start to fade new ones show up and some shortness of breath to where I can t finish a lullaby for my kids standing up without gasping for breath. Could this be a medication side effect or something more serious? Doctor: Hi. I can understand your concern. You will need a detailed evaluation and blood tests to assess the cause for the same. Deranged liver function and sometimes connective tissues can cause these symptoms. Consult a pulmonologist for the same. Don't worry, you will be alright. Hope I have solved your query. Wish you good health. Thanks."
},
{
"id": 111487,
"tgt": "What could it be if having stomach bloating, cramps, twinging pain in lower left side of back?",
"src": "Patient: My stomach bloats when i eat anything and any amount, even small amounts. I have food intertolerences but I manage them carefully and this does not feel the same as prior episodes of my interolerences. I am having stomach cramps on and off and also have a sharp twinge pain on the lower left back. Doctor: Hello, I had gone through the case and found that it might be intestinal obstruction or coeliac disease.For left lower back pain might be renal stone, large intestion infection or constipation.So go for ultrasound of whole abdomen, liver fuction test and colonoscopy and coeliac and lactose intoletance test.After getting the proper diagnosis take treatment by gastro enterologist.Hope my answer will be effective for you .Thanks"
},
{
"id": 177734,
"tgt": "Suggest creams to lighten the skin tone",
"src": "Patient: Hi, I got 21 months boy baby. he born in India but, now we are in U.S. myself dark complexion, my husband is wheatish color. my boy face looks little fair but, body is in dark. what to use for his skin complexion? can u please suggest me some soaps or lotions and creams for his fairness? Doctor: Skin lightening is practically a myth and do not fall for it. You cannot ultimately make it lighter. It has a genetic component and you cannot change that. No soaps, lotions or creams are likely to be effective. At the most you can use sunscreen lotions when you take the baby out. Just maintain a healthy lifestyle and a proper diet and things would be at its best."
},
{
"id": 33730,
"tgt": "Suggest treatment for rhinitis infection",
"src": "Patient: Hi there I ve got a real irritable virus up my nose called rhinitis I ve been prescribed otrivine but have no way of getting it tonight is there any replacements that I can use just for tonigh. I have olbas oil, beconase and a variety of allergie tablets and flu tablet?? Doctor: Hello,otrivin is a nasal decongestant. It's not a cure for your rhinitis but it will alleviate your symptoms. If the \"allergy\" and \"flu\" tablets contain a decongestant then you can use them. The \"allergy\" tablets usually contain an antihistamine drug. The \"flu\" tablets may contain acetaminophen and perhaps also a decongestant.If you'd like a more detailed answer, please inform me about the active substances of the tablets that you have.Kind regards!"
},
{
"id": 140334,
"tgt": "What could weakening of right limbs indicate, taking Riscalm, Amantrel & Parkin?",
"src": "Patient: sir i am taking riscalm ls at night daily 1 tablet for fast 12 years now i am using left hand more than right hand so i consulted doctor now he priscribed amantrel 100mg and parkin 2mg after taking this after 4 months now my right hand and right leg is become weak i cant move as left side so now what can i do sir Doctor: Hi, I have noted the clinical details. Based on this, you could be suffering from Parkinson's disease. The diagnosis can be confirmed by clinical examination in most cases. In some cases, 18F DOPA PET CT scan may be needed. After this, the correct treatment can be decided. Hope I have answered your query. Let me know if I can assist you further. Regards, Dr. Sudhir Kumar, Neurologist"
},
{
"id": 197641,
"tgt": "How to get rid of the cyst on my penis?",
"src": "Patient: i think i have a cyst on my penis. i have had it for quaite a while it doe snot hurt. i think it is a cysts as it can move slightly and like i said is not painfull to touch and gives me no irritation. it is at the higher end of my penis Help!!Thanks Doctor: HiWelcome.I have gone through your query.It is difficult to comment on it without examination, so I would either suggest you to see doctor for examination and for exact diagnosis or upload the pictures here, I'll go through them, and provide you with specific advice.Hope this helps.Take care"
},
{
"id": 30364,
"tgt": "Suggest treatment for staphylococcus epidermidis",
"src": "Patient: I have staphylocoque epidermidis, vaginal picotement resist, my mammogram result is abnormal. Please doctor, help me to show me syntome breast , vaginal and epiderm, by the pictures,and how can I do please. Help me please doctor with traitement,advice. I have 50 years old woman. my name is Louisette.what's your name doctor? Doctor: hi! i would advised that you need to subject yourself for blood culture and sensitivity because the antibiotics to be used will be on the basis of antibiotic susceptibility test that infection. generally, the drug of choice is vancomycin, other antibiotics are gentamycin, daptomycin. however, there are bacteria's who is very resistant to this antibiotic."
},
{
"id": 157314,
"tgt": "Can cyst and uterine line thickening get converted into endometrium hyperplasia and cancer?",
"src": "Patient: Hi I'm a 27 200 pd female with diabetes and when I went to get an ultrasound with my obgyn she found I had a thick uterine lining of 16 mm and a cyst of 2 mm .. I have been spotting in between periods and irregular periods .. I read about endometrium hyperplasia and read it can be cancer too .. Should I be concerned? Doctor: Hi, yes endometrial hyperplasia can be one of the signs of endometrial cancer. But since you are in ur twenties, and having irreular periods, get your pregnancy scan done and if need be there, get dilatation and curettage done to check for histopathology."
},
{
"id": 6207,
"tgt": "Doing follicular study and having PCOD. Had HSG: Clear tubes. Taken fertigyn 5000 injection. Had sonography, showed reduced follicle, no free fluid. Can you please guide me?",
"src": "Patient: I am going through the follicular study and have PCOD. After HSG , its been confirmed that my tubes are clear.However, on 16th day of my sono scan, on my left ovary the follicle was measuring 15*14 mm and right had msf.ET was 8.1 mm. on the 17th day, the doctor had asked me to take fertigyn 5000 injection for the follicle to develop. i went in for another sono scan on the 19th day, ET was 8.4 mm, Left ovary the follicle that develped was reduced[regressed]. right ovary MSF and no free fluid was found. i would like to know how the follicle must have reduced its size in 2 days, because the doctor stated that there was no evidence of egg rapture, if so fluid must be found under the ovary which was also missing. Can youplease guide me on this and what must i do. i have been having irregular periods often, for past 7 months, i have been having it regularly on the right date. I am weighing 90kgs Doctor: Hi Piriyadarshini, The follicle reduced in its size because of the rupture. There is no other way that it reduces in size. You need to try again in the next menstrual cycle for a successful conception. Meanwhile, you need to decrease your weight. This is the first goal in infertility treatment. Once you reduce your weight, your hormones will come to balance, you will have regular menstrual cycles, and it will be easy for you to conceive. Keep in regular touch with your treating doctor. All the best. Regards,"
},
{
"id": 57631,
"tgt": "What caused swollen liver/ spleen in a child with history of lead poisoning upto 15?",
"src": "Patient: My daughter had an ultrasound showing an echogenic liver and slightly elevated ALT. her follow-up ultrasound showed a slightly enlarged liver and spleen. All bloodwork came back normal. She is an overall health child with a history of lead poisoning. Highest level of 15. What could be the cause of her liver and spleen being swollen? Doctor: The diagnosis of liver diseases is a very important part of treatment. Correct diagnosis will lead to swift treatment and better recovery.The Global Hospitals Liver Institute is an end to end liver diseases diagnosis, treatment and liver transplant centre in India. The doctors of liver disease use these diagnostic and treatment tools to determine the type of liver disease and extent of liver damage.-Colonoscopy-Paracentisis-BiopsyThese are just a few of the diagnostic and treatment tools used at the best hospital for liver transplant in India \u2013 Global Hospitals."
},
{
"id": 152937,
"tgt": "How can erectile dysfunction due to radiotherapy effects be treated?",
"src": "Patient: i had prostate cancer took radiation treatment and it is gone 2013 8 weeks of it. now cant get a erection and sometimes i can get a soft erection just cant cumming and when i do it is thick and i have to push it out what can i do i just been marrie for a year.not her she makes me feel good in ever way. please help.....60 year old man never had this before.... Doctor: Hi deaErectile dysfunction is a common problem after radiation treatment for prostate.I suggest following.Tab. Tadanfil(cialis) 20 mg one hour before intercourse. But needs precautions in angina patients who are on nitrates.I hope this will help.other options are penile prosthesis which are done by urologisist.last start practice yoga/meditation as therapy will help to calm you down.regards best wishesget well"
},
{
"id": 115223,
"tgt": "What causes clotting of blood?",
"src": "Patient: Hi. Iam a healthy 39 year old woman and i just had my tubes tied on 2nd Feb. Ever since coming out of the operation I had pain on my right side and from last Friday a blood clot has formed approx 7cm long and 1cm wide. I was sent home from hospital saying that they are not too concerned and the clot will dispurse into my body. The doctors also said that it wasn't connected to the surgery and that it is just blood vessels broken. I am concerned. Should I be? Where would this bleeding be coming from? Any opinion would be great. Thank you. Sam Doctor: Hi, dearI have gone through your question. I can understand your concern.Your clot must be situated outside the vascular system. It is called as hematoma. There are many reason for that. any kind ob extravasation of blood can leads to that. No need to worry. It will dissolve by itself.Hope I have answered your question, if you have any doubts then contact me at bit.ly/Drsanghvihardik, I will be happy to answer you.Thanks for using health care magic.Wish you a very good health."
},
{
"id": 1873,
"tgt": "Suggest remedy for getting pregnant",
"src": "Patient: Hello Doctor, I am married for 10 years and trying to concieve from last 6 years. This is my second try. I have a baby girl 7 years old. now my age is 39.My periods are regular . I am taking cervifert capsules and of course siphine 100 mg . My husbands report shows most of the times low sperm count its about 8-9 million . my question is - am i conceive a baby normally or trying to using some other treatment. My reports are normal. I tried 2 IUI. both failed. Doctor: Hi, I think you should go for IVF now, because your age is on a higher side and you have already tried 2 cycle of IUI. In IUI, chance of pregnancy is 10 to 15 percent only. In IVF, it is 40 to 50 percent. So, it definitely has an edge over IUI. Hope I have answered your question. If you have any other query, I will be happy to help. Regards Dr khushboo"
},
{
"id": 104159,
"tgt": "Have athletics induced asthma. Chest pain, exhaustion",
"src": "Patient: Okay. I have athletics induced asthma and almost for the past week I have been having chest pain . I ve been taking my inhaler but it only helps with the pain somewhat. It seems to get even worse if I m laying down in bed on my back. I ve been exhausted the past few days due to this and haven t been able to concentrate on school. Doctor: INHALERS ARE SOMETIMES NOT ABLE TO CONTROL ASRGMA AND AS TIME OF DISEASE INCREASES THE DISEASE BECOME MORE DANGEROUS AND EFFECTS MORE IRGANS YOU CAN ADD ANTI ALLERGIC TAB SINGULAR I TAB BD SP VENTROLIN 1 TSF BD ADD LIWUID ANTACID YOU ARE YOUNG YOU CAN GET ALLERGY TESTS AND GO FOR SPECIFIC IMMUNOTHERAPY FOR PERMANENT CARE PF DISEASE"
},
{
"id": 84635,
"tgt": "What causes lump in armpit and groin after taking sodium valporate?",
"src": "Patient: I am taking sodium valporate for the last two years & I am noticing a lump in my armpit, groin,swelling of tongue on the edges . I got CBC checked a month ago and platelet count is low , wbc is abnormal ? I had fever,headache,chills at that time ? could it be side effect of the drug more serious infection like HIV ? Doctor: Hello,I will suggest you to repeat blood counts again and see if its low. Then most probably its because of side effects of valproate. You need to change the valproate to some other drug according to the cause for which you are taking vaplroate regarding HIV, a simple test in GUM clinic for screening purpose will be enough to tell you about your HIV status.Hope I have answered your query. Let me know if I can assist you further. Regards, Dr. Muhammad Faisal Bacha, Internal Medicine Specialist"
},
{
"id": 208122,
"tgt": "What is the treatment for extreme depression?",
"src": "Patient: What is the maximum allowed dosage of fluvoxamine for healthy 30 year old man? Is it safe to have 200 mg fluvoxamine per day ?Hi, at the age of 23 I was having Fluoxamine per day 175mg to 200mg ( morning 1.5 (75mg), afternoon - 0 and night 2 ( 100mg) ) for Depression, OCD, ADD and social anxiety. In this dosage i was feeling very well and happy. Along with that i was also having Alzolam 0.25 mg per day. I was continuing this for 5 years. In between i sometimes stop medication completely and whenever i feel discomfort i restart the medication. Later on i switched to Venlafaxine and then to Sertraline thinking that it could give me much better benefits however it didnt' work that well with me. I don't know whether it is because of lower dosage of what i am having. Currently i am having sertraline 25mg per day however it gives me side effects like on interest in sex, feeling sleepy, etc. Around 9 months back i was having Crestor 10mg per day for cholesterol but now i have stopped it.Now i want to switch again to Fluvoxamine as i think this was working well with me than any other medicine but in a higher dosage. Is it safe to have 200 mg or 175 mg per day ? I am a 30 yr old health male. Doctor: Hi, thanks for the query. First & foremost I would strongly urge you to NOT to change/switch any medication on your own & what ever are your issues with current set of medications should be discussed & dealt with only under supervision of a qualified medical doctor preferably a Psychiatrist. I am using strong words since all these aforementioned medications come under category of SSRI/SNRI & taking 2 SSRI together can cause a potentially dangerous condition named serotonin syndrome; while abruptly stopping any or all of these medications cause a flu like discontinuation syndrome; not to mention flaring up the symptoms of original illness (OCD; Depression in your case). Coming to your query; Fluvoxamine is a very good choice for controlling symptoms of both OCD & Depression & maximum permissible dose of Fluvoxamine is 300mg/day in a healthy person; so depending upon your symptom response; 175-200mg/day is perfectly fine. I would like to know your symptom profile of both OCD & depression so that I can advice whether we can plan for additional therapy in addition to medications for quicker response. Good Luck"
},
{
"id": 152090,
"tgt": "Diffuse disc bulge with herniation causing moderate thecal sac bilateral transversing compression",
"src": "Patient: mornig sir my husbund 29 years old 5 years back one time he did long jumb practice that timel he fell down with his back he had been back pain and also had radiating pain on leg then problem solved he back to normal from that time he did mild back exercise . 4to5 times that disc problem occur with in this 5 years doctors prescribe some medicines like pain killers 6 months before scaning report is l4-l5 diffuse disc bulge with herniation causing moderate thecal sac bilateral transversing compression (l>r)and bilateral neural foramin narrowing l5-s1 disc bulge anterior thecal sac and bilateral neural foramen narrowing , l5-s1 diffuse disc bulge causing anterior thecal sac identation and bilateral neural foramen narrowing without significant nerve root compression , l5-s1 facetal arthrosis what v have to do sir Doctor: Hello. Thanks for writing to us. Your husband is suffering from disc prolapse which is causing a nerve root compression. In addition to the pain killers that he is already taking, he needs to have a good muscle relaxant like thiocolchicoside and methylcobalamin supplements for improvement of symptoms related to the nerves. I hope this information has been both informative and helpful for you. Regards, Dr. Rakhi Tayal drtayalrakhi@gmail.com"
},
{
"id": 66934,
"tgt": "Suggest treatment for yellow bumps at the back of throat",
"src": "Patient: MY DAUGHTER IS 21, SHE IS A SINGER WORKING HER VOICE 4 TO 6 HOURS PER DAY. SHE HAS BEEN FIGHTING A STRAINED AND SORE THROAT FOR MONTHS. SHE SAYS SHE HAS YELLOW BUMPS AT THE BACK OF THE THROAT, FEELS SORE AFTER SINGING, LIKE SOMETHING IS STUCK IN HER THROAT. NO FEVER, NOT SORE WHEN NOT SINGING. GARGLES WITH WARM SALT WATER, USING A HUMIDIFIER, NOTHING IS HELPING. Doctor: these could be Singer's nodules or benign laryngeal polyps possibly due to voice abuse; other possibility is papilloma or fungal/actinomycetes colonies!consult an ENT specialist.all the best!"
},
{
"id": 196182,
"tgt": "What causes very less facial hair at the age of 23?",
"src": "Patient: Hello Sir,I am 23 now but still my beard has not grown well except i have some hairs on my mustache and some on my chin .My elder brother as well as my father have full beards ,that means there is no genetic problem .sir i am worried ,please suggest me some natural remedies for curing this disease.Thanking you CKM Doctor: hii.kindly test your serum testosterone levels.take b complex tablets twice a day as biotin deficiency cause also this problem.apply eucalyptus oil over the beard area."
},
{
"id": 42455,
"tgt": "Is it safe to take Flovite for infertility?",
"src": "Patient: Good morning. This is jesmin from Bangladesh. Doctor Suggest me to take medicine folvite for infertilty when I viisit her in INdia. but in bangladesh I am not getting this medicine can u please sugggest the generic name of the folvite and siphene Doctor: Hello Jesmin As these two drugs mentioned for the infertility are folvite is a tablet of \"FOLIC ACID \" popularly known as iron tablet and prescribe in infertility as well in pregnant ladies and also in patient with low hemglobin level. This drug you can take from any chemist shop in Bangla desh as folic acid tablet .While sipene is \"CLOMIFENE \" and used in \"anovulatory infertility\" 50 mg form 5th day of menses ( I prescribe in this way 5th , 6th, 7th , 8th, 9th days). While other doctor also prescribe from 6th to 10 th days . Few also advise as first day of menses. So take this drug as prescribed your doctor.Salt or generic name is CLOMIFENE or CLOMIPHENE.Hope this information is good for you."
},
{
"id": 114388,
"tgt": "What does elevated calcium level in the blood indicate?",
"src": "Patient: Hi, I had a blood test, and for years, my calcium has been high for years. At first, I was told it is really nom problem...it was as high as 10.7...now, it is 10.3. My blood test also says I have Normal Parathyroid, Hypoparathyroidism, blood test says LOW, Hyperparathyroidism, primary, high, Secondary, normal to low, Tertiary, high, Non-Parathyroid Hypercalcemia, High....So, should I be concerned? Doctor: Hello and Welcome to \u2018Ask A Doctor\u2019 service. I have reviewed your query and here is my advice. Yes, elevated calcium can cause problems like abdominal pain, kidney stones and drowsiness. Visit your doctor immediately. Hope I have answered your query. Let me know if I can assist you further. Thanks"
},
{
"id": 203686,
"tgt": "Does lyco-Q tablet have any side effects and how long should it be used to get my wife pregnant?",
"src": "Patient: Sir, I am using Lyco -Q tablet from last 4 months by one in daily basis. This is I used under instruction of a gynecologist for the problem of low motility of 10 %. my sperm count is 35 million. I want to know that any side effects for this tab and how long it can be use for pregnancy for my wife ? Doctor: HelloThanks for your query,based on the facts that you have posted it appears that you have Oligo-asthanozoospermia meaning thereby that your sperm count is less and percentage of active motile sperm is less than 10 %. You have been prescribed medication Lyco-Q tablets to help to increase motility of sperms.There are many drugs in the pharmaceutical markets who claim to be effective in improving sperm count but to be truth not a single drug has been scientifically proved effective to increase the motility of sperm.Following measure will help you to increase sperm count and sperm motility to some extent.1) Practice regular exercise for 45 minutes followed by meditation for 1/2 an hour in the morning.2) Take high protein diet rich in vegetables and fruits and Vitamin A,C,D,E.and Zinc3)Take anti oxidants like Almonds 5-6 everyday..4) Avoid alcohol and smoking..Dr.Patil."
},
{
"id": 166824,
"tgt": "What causes headache, shaking and dizziness in a 8 year old?",
"src": "Patient: My daughter is 8 year old and plays soccer.2 days ago, she was tripped during the game and landed on her face, causing a bloody nose. After getting the bleeding to stop she was ready to play again, and did fine the rest of the day. The next day she complained of headache, feeling shakey, dizzy at times,and lack of appetite, plus she had a low grade fever. With motrin, the fever reduces, and then she is able to eat a little something. Are her symptoms and the accident at soccer related? Doctor: Hi,Very good question, an the answer is could be. She needs a check up because she could get some silent traumas in her face. There is a sinusitis that can be a result of some accidents and the symptoms are like you describe. Hope this will be helpful."
},
{
"id": 178696,
"tgt": "What causes headache, fever and diarrhoea in a 3 years old?",
"src": "Patient: Hi my son 3yrs old was presenting with fevers headaches nod tummy aches, after 3 days he started with diarrhea still with fevers and tummy aches. He now is complaining of hand pain every time he goes down to sleep. Not sure if it is puns and needles? He has also started with a few red spots on his lower back and buttocks? Doctor: Thank you for the question.As far as hand pain is concerned it seems to me that he must be keeping hand in awkward position while sleeping which can be corrected easily.For headache, tummy ache and diarrhea he needs medical attention as soon as possible. He will look after the rash too."
},
{
"id": 217367,
"tgt": "What treatment is suggested for pain on the right side of the body?",
"src": "Patient: Hi I'm 18 years old I have pain in the whole right side of my body. It comes and goes. Done every test I could think of except of my brain. I recently had a baby. I have been in pain ever since with some burning in my stomach, head aches and right side of the body pain. Thanks Doctor: Hi, I have read your complain. You said that you are 18 yreas and have a baby . Inspite of so many complains all the investigations are normal. I think these are not related to any physical illness. It might due to some mental tention or stress. I would like to advise you to try some yoga and meditation. Also I would like to suggest you to meet a counseler you will help you .Pl. do not hesitate to meet them and get well soon. Good luck"
},
{
"id": 205965,
"tgt": "What are the symtpoms of OCD?",
"src": "Patient: I think that i have OCD I have taken alot of OCD quizzes and they all say that i might have OCD. I have also read alot of stuff about OCD. I afraid to tell my MOM or dad because they will think that its all just in my head. Doctor: Hello,!!You might have OCD but this needs to be confirmed by a psychiatrist. There is no quiz that can confirm with surety that you have an OCD. And do not feel scared to tell your parents as it is a treatable problem and no need to worry about it. God Bless!!Dr Hitesh SharmaMBBS, MD Psychiatry"
},
{
"id": 204998,
"tgt": "Are Clonazepam and Escitalopram safe to be taken for anxiety?",
"src": "Patient: I am Bhagirath Bhandari. AGE 41.Dr prescribe me ccq 25 .because my semen is low.i want to be a father. I take this medicine 80 days.but now my pressure is 100/70.I am very week. I not sleeping in night and suffer anxiety. Please help....now Dr. Prescribe me escitalopram10mg,clonazepam 1mg.can I take this medicine for cure please tell me this email address - YYYY@YYYY Doctor: Dear userWe understand your concernsI went through your details. Clonazepam and Escitalopram both are anti anxiety and anti depression medicines. You need to consult a psychiatrist in person for a prescription.If you require more of my help in this aspect, please use this URL. http://goo.gl/aYW2pR. Make sure that you include every minute details possible. Hope this answers your query. Available for further clarifications.Good luck."
},
{
"id": 205354,
"tgt": "What causes lethargy, , frequent headaches on the right and ringing in the ears?",
"src": "Patient: Hi there, I m a 34yr old female mother of 2. I weigh about 175lb. I just got off my period a couple days ago. Usually my periods will last a full 7 days. No chance at all that I would be pregnant. I have had tubes tide 8yrs ago. Anyway, I also have been diagnosed with depression a couple years ago. Lately, I feel really tired, not motivated to do anything, wanting to eat more than usual like a couple more times a day, getting frequent headaches lately on the right side, high pitched ringing in my ears, and then after the noise in ears I feel faint. Diabetes does run in the family, as well as high blood pressure. Any info or suggestions is greatly appreciated thanks!! Doctor: Hi, i think u shall consult psychiatrist and medicine both, because u have some stroke like features because of high blood pressure. so better to consult them."
},
{
"id": 192137,
"tgt": "What causes burning sensation around testicles after having protected sex?",
"src": "Patient: Hi, may I answer your health queries right now ? Please type your query here...i had protected sex with my girlfriend few day ago, the very next day this happen, i felt this buring sensation around my testicles, am that sensation in still on till this very moment.it being two day now, and its still burning. Doctor: Hello,Burning sensation around the testicle is commonly due to epididymitis. Epididymitis is caused by bacterial infection. Since you are sexually active, it could be due to STI's, such as gonorrhea or chlamydia. Sometimes, a testicle also becomes inflamed called epididymo-orchitis. You are suggested to get clinically examined by a Urologist or GP. Get assessed and evaluate. You may need surgery apart from medicines. Get STD tests done to detect STI. If reports are positive you may need antibiotics and antivirals. However, other medical conditions that may cause burning sensation are kidney stones, orchitis, UTI, testiculatr torsion or tumor, inguinal hernia, etc. Get blood tests, urinalysis (R/E & C/S), CT, Radionuclide imaging, etc done. Your treatment will depend on the pathology detected. In the meantime you can take pain medications & anti inflammatory for relieve.Take care. Hope I have answered your question. Let me know if I can assist you further. Regards, Dr. Nupur K., General & Family Physician"
},
{
"id": 41116,
"tgt": "Is IUI safe and successful in achieving a pregnancy?",
"src": "Patient: Hi Sir My name is Harish, iam married for 2.7yrs now and we are planned for a baby and thats not successful we are undergoing a lot of treatment i dont have any issues but its seems my wife has now we have finally come to a decision of undergoing IUI procedure. IF you could let me know how safe and how successful is this and if you could also let me know the cost of it. I have got one of my friend who is successful with this after 2yrs of no kid, I have also chcked internet that shows 80% a good procedure. We are not ready for any further treatment we have to go for IUI procedure as its high time. Please help us with this. Doctor: Hello, usual cost is around 1-1.5 lacs but changes with city and centre.IUI is very safe and enhancesw the chances of conception many foldIn case you have any questions in future you can contact me directly on http://bit.ly/drmanishajain"
},
{
"id": 69681,
"tgt": "Suggest solution for lump in inner thigh",
"src": "Patient: I have a lump that I found on my inner thigh while I was in the shower today which I'm really worried about.It's average size,extremely hard and inside the skin I think.I'm 14 and female and really worried as to what this could lead too.Thankyou so much for any advicee Doctor: Hi.Thanks for your query and an elucidate history.Considering your age and findings this looks to be just a boil. May settle on its own. Do not handle/ fiddle with it otherwise it will grow and need antibiotic and anti-inflammatory medicines. IF it is steady and silent - wait and watch. If gives any symptoms of pain , redness, fever or increase in size - please consult a Doctor and insist for a visual and clinical/ physical examination."
},
{
"id": 210852,
"tgt": "What is the remedy for depression when on statin?",
"src": "Patient: Have numbness and tingling in both hands also bladder cramps. I am taking Atorvastating 80mg. Have had mild heart attack resulting in stent also have 3 other arteries 40 per cent blocked. Had my heart attack end of august.Have lost weight, eating very healthy, exercise daily up to 2 hours. I am 75 year old female.Had Guillain Barre syndrome, peripheral neuritis and diagnosed with fibromyalgia, but not incapacitated by anything .Concern: getting depressed and worried about high dose of Statin. I am willing to do what it takes to improve my cholesterol situation. I am constantly being told how well and low it is, but I still see no end to my worry of dropping dead because of too strong dose or more side effects worse than what I am deal with right now.Thank youYolanda Doctor: Dear Madam,You need not worry about the dose of statin , it is going to benefit your body rather than causing any harm.Dr. Shruti"
},
{
"id": 101770,
"tgt": "What causes shortness of breath?",
"src": "Patient: I have allergy from changing weather , due to which I get short of breath sometimes and i need to take seroflo 100 mg rotahaler capsules.I have recently shifted to pune around 2 months ago and since then I have to use it daily. Previously i needed to take this once in a while.Please suggest what might be the reason what I should do Doctor: Hi and thanks for the query,It is important identifying the main environmental factor that triggers shortness of breath with you. It might be useful to use some steroids or anti allergic drugs prior to such exposures or to be armed with some drugs like Ventoline or bronchodilators to prevent any respiratory emergencies. A proper review with an allergist for an immune desensitization sequence could be helpful in some persons. Kind regards"
},
{
"id": 90940,
"tgt": "What causes on and off stomach cramping?",
"src": "Patient: I have a hx of acid reflux. Recently had some chest pain, radiating to back that was relieved when I sat forward so I thought it may be spasms of my esog. Then last Sunday I got sever cramping in my stomach. Very noisey and active. Stayed in mid stomach and did not move lower. I thought I had severe constipation. Took laxitive and enemas the next day and blockage was releived however I am still having cramping off and on. No fever, nausea, and had three bowel movements in the last 6 days but cramping still occuring but lighter. Shall I wait a bit longer to see if it goes away? Doctor: HiThanks for writing in HCMI need further information regarding 1.)your age 2.)how long you are suffering with this problem 3.) any difficulty in passing stool/loose stools 4.) factors aggravating your symptoms 5.)any chronic analgesic intake history/alcoholic 6.)are u under any cardiac or anti hypertensive medications 7).any sudden weight loss 8.)about your appetite my advice is to dont take enema/laxatives without physician advice.my advise is to drink plenty of water,regularly timed food with every 3 hour interval but all in small quantity for eg 2-4 biscuits with adequate water and to avoid spicy and oily food,no alcohol,no coffee,no junk foods . my advise to take diet rich in fibre content,green vegetables ,fruits., I would be glad to provide answers to any further queries.Thanks for using HMC Have a good day!Regards, Dr. V.KUMARAGURU general surgeon"
},
{
"id": 225694,
"tgt": "On Implanton, frequent urination, tiredness, moody. HPT cam back with crooked positive line. What does it mean ?",
"src": "Patient: I am on the birthcontrol implanton (sp) the rod in the arm for 6 months. An I have been having to pee alot, tired, moody just not feeling well all around. So I took a pregnancy test yesterday and it came back with a smear and a crooked positive line all in one. What is going on with me? An what does the smear crooked positive mean?! Doctor: Good afternoonDon not worry those are symptoms of side effects of implanon.The result you got is most likely false positive, I would advise you to repeat the test in near by health facility.Its also necessary for you to see doctor so that other tests like blood pressure, random blood sugar are done to rule out other diseases like pressure.Hope I have answered your question, if you have any more questions I will be happy to help you."
},
{
"id": 195030,
"tgt": "Are grade-2 left hydronephrosis with a visible hydroureter and an enlarged prostate gland treatable?",
"src": "Patient: Ultra sound says : Grade 2 left hydronephrosis with hydroureter seen. No renel Calculus seen. Prostate Gland is enlarged( 42.5 X 44.5 X 38.2 mm) with volume of 37.5 m. Texture is normal. No SOL seen. Impression: -- 1. Left hydronephrosis with hydroureter 2. Grade 1 B. P. H. Urine test shows: Pus Cells: 8-10 H.P.F Age: 51yrs Is there a need to worry about prostate cancer is there is pain in abdomen? Doctor: ...yes the prostate has to ne taken care of ..as the prostate is giving the back pressure effects on kidney and ureter ..the enlarged kidney is damaged with time it shrinks ..secondly enlarged prostate is causing stagnation of urine and causing urine infection treatment of enlarged prostate is medical and surgical medical drug as finasteride and tamulosolin etc are given for surgery various methods are there ..please consult your primary physician he will discuss all aspects of therapy so do not ignore with regards dr varinder joshi"
},
{
"id": 73338,
"tgt": "Can leaking aorta cause shortness of breath?",
"src": "Patient: HI. IVE HAD A DOUBLE BYPASS IN2009 AND NOW THEY SAY I HAVE A MILD LEAKING AORTA ,IM VERY TIRED ALL THE TIME IM HOLDING FLUID ALSO .THEY PUT ME ON LASIX AND AND ALSO SHORT OF BREATH.IVE BEEN TO MANY DOCTORS THEY TOLD ME ITS OK .BUT I DONT FEEL OK .TRY TO WORK OUT CANT BREATH RIGHT.STARTING TO EAT RIGHT .SO WHAT IM ASKING U IS CAN THAT LEAKING AORTA HAVE SOMETHING TO DO WITH ALL THIS ......THANKS SUE Doctor: If by leaking aorta you are referring to aortic regurgitation (leaking of the valve of the aorta), then it can certainly explain your symptoms of fatigue, shortness of breath, and retaining fluid. Medications can help treat your symptoms, but unfortunately definitive treatment requires either open surgery or a minimally invasive procedure (called TAVR) to fix the valve. You should ask your doctor if the severity of your aortic regurgitation requires a procedure."
},
{
"id": 133512,
"tgt": "What could cause huge lump on big toe and elbow after getting hurt?",
"src": "Patient: My Partner hurt his big toe 2 months ago and it has a huge lump on it. He hurt his elbow 10 days a go and his elbow has a bit long lump on it. I told him he should see the doctor but says its fine as he has acid between the joints and has caused this?? Doctor: Hi, I had gone through your question and understand your concerns. You haven't mentioned your partner's age and proper medical history.As per your information, the deposition of sharp acid crystals (uric acid) in joints, often in the big toe is the possibility of the condition called gout,deposits of uric acid (called tophi) which look like lumps under the skin.It affects many joints including elbow.It is one of the most painful forms of arthritis. people who are more likely to have gout if :Have family members with the disease,are a man,are overweight,drink too much alcohol,eat too many foods rich in purines,have had an organ transplant etc.I would recommend you to see a doctor immediately to rule out the condition.Take the medicines your doctor prescribes as directed, tell him about all the medicines and vitamins you take,also plan followup visits with your doctor,maintain a healthy, balanced diet,avoid foods that are high in purines, and drink plenty of water, exercise regularly and maintain a healthy body weight. Ask your doctor about how to lose weight safely.Fast or extreme weight loss can increase uric acid levels in the blood.Hope this answers your question. If you have additional questions or follow up questions then please do not hesitate in writing to us. I will be happy to answer your questions."
},
{
"id": 26843,
"tgt": "Should i be concerned about high pulse rate and BP?",
"src": "Patient: My pulse has been running the last two weeks between 100-115 BPM. My BP has been running higher too. I feel out of breath and at times I feel the rapid beating. Is this anything to be concerned about? I am on Lisinopril now 10mg. But haven't seen much difference in the BP readings or the pulse reading Doctor: Hello. Thank you for your question and welcome to HCM. I understand your concern. There are a lot of physiological fluctuations of blood pressure and heart rate during a day. This comes from various physical, psychological and hormonal changes that happen in our organism in a day. Lisinopril is a drug from the angiotensinogen converting enzyme inhibitor (ACE-I) family, effective and safe in lowering the blood pressure, but does not exert any effect on the heart rate. If the blood pressure figures remain high, the it can be discussed to switch lisinopril with another drug or keep it and add a second blood pressure-lowering drug to your regimen. I would recommend a drug from the beta-blocker or calcium channel blocker (verapamil and diltiazem) family, because, besides having effects on lowering the blood pressure, they also have the feature of slowing the heart rate down and reduce the heart workload. You can discuss with your curing cardiologist, about the precise drug, dosage and frequency of usage. I hope I was helpful with my answer. Kind regards. Dr. Meriton"
},
{
"id": 81640,
"tgt": "What causes sharp pain in chest during bowel movement and arm pain?",
"src": "Patient: HI. I want to know what s going on with me. I get Sharp pains in my chest mostly when I have to poop. My shoulder or arm hurts, I ve been to a clinic they said it was nothing ran EKG and said I was fine. It really scarys me because I have heart problems that run in the family. Doctor: It is something i have not heard of commonly however as your symptoms are genuine, I would suggest you to go ahead and do a stress test so that we know whether there is any ischemia to the heart"
},
{
"id": 125009,
"tgt": "Could severe pain in wrist after a fall while having brittle bone disease be serious?",
"src": "Patient: i have brittle bone disease.i fell last night and hurt my hand and wrist.a and e x rayed it.no fracture showed up.but they said a fracture along the wrist doesn,t show up for 10 days when it,s starting to heal.they give me a splint to wear.the pain is alot worse today,the pain is moving up my arm and into my shoulder Doctor: Hello, The most common complication with a fracture in a person having the brittle bone disease is non-union i.e. fracture does not unite so easily. As in your case, only a few fractures like scaphoid bone fractures of the wrist may not show at the time of injury but maximum fractures are easy to diagnose. As you have a brittle bone disease it is better to go for CT SCAN to diagnose the fracture without wasting time. And of course splintage is a must and for a longer duration in your case than a person without brittle bone disease. Take anti-inflammatory analgesics, calcium, vitamin D supplements and get the CT SCAN done. and for arm and shoulder use ice packs. Hope I have answered your query. Let me know if I can assist you further. Take care Regards, Dr Jaideep Gaver, Orthopaedic Surgeon"
},
{
"id": 94573,
"tgt": "Had intestinal issues, have constant diarrhea. Celiacs disease? Take cuturelle?",
"src": "Patient: My husband has had stomach/ intestinal issues most of his life and it is reaching a point where it is inhibiting his quality of life. He has constant diarrhea , no blood or other visual issues spotted, he feels like vomiting a lot of the time and there is no quelling the pain or discomfort. Symptoms don t present themselves with one type of food over another they are random and can flare up at any time which makes me believe its not something like celiacs disease since grain doesn t effect it negatively but I m not a doctor. Discomfort tends to always be present just a dull pain and other days it can be severe where he wont be able to leave the house. He has been taking culturelle which seems to be helping but it seems to becoming less and less effective. Any suggestions? Doctor: thank you. you not mention the characters of stool.there are some possibilities like irritable bowel syndrome,malabsorption syndrome or chronic pancreatitis.to evaluate the chronic diarrhoea with vague abdominal pain, thorough check up is needed.you could not ignore coeliac disease.some inv. as upper GIT endoscopy with duodenal biopsy,s.TTG and antiendomyseal antibody(coeliac disease),stool exam(fat malabsorption),plain abd.x-ray,usg and other imaging of abd.(ch.pancreatitis) is needed.so you should contact with a gastroenterologist."
},
{
"id": 177805,
"tgt": "Suggest treatment for cold and cough in a child",
"src": "Patient: hi doctor my baby is 11 months old she has runny nose sneezin cold and cough with a slight blocked chest what med shud i gv her and wat food shud i give? YYYY@YYYY i currently hv maxtra , ashthalin, ondem syrups at home alongwith crocin thanks vanessa Doctor: Hi dearwelcome to the HCM,He needs to be given antihistaminic decongestant medicine along with some bronchodiltor with proper doses.Steam inhalation will be help ful,Nebulization with bronchodiltors helps a lot in reliving the congestion.If there is lot of cough and there is some respiratory difficulty then one should add some antibiotic with the consultation of your pediatrician.Hope the query is answered.Thanks"
},
{
"id": 36326,
"tgt": "How to treat the cyst of my son's thigh?",
"src": "Patient: My soon has a cyst on his inner thigh. That is changing locations. He also has chills. This lump came after moving an excessive amount of furniture over the past three of three hottest day. Loss of appetite, & excessive sleep. Please advice, age 28 yrs old. Doctor: Hello,Thanks for the consult...Brief answer,Surgery is needed for that cyst.Detailed Explanation,1. I read your query and i am happy to help you. You already mentioned that your son has cyst on his inner thigh.2. This is similar to cyst called Lipoma because it is soft and change locations but chills in lipoma is unusual 3. Chills represents the infection is systemic4. Kindly advice ultrasound at the site of cyst5. Excessive sleep is due to hard work for the past 3 days and loss of appetite is due to systemic infection6. Surgery is the final treatment of choice in this caseHope this information helps"
},
{
"id": 191796,
"tgt": "Suggest treatment for pain of a diabetic who consumes plenty of candies",
"src": "Patient: hi, my diabetic consumes large quantities of candy and ice cream. He is 70 years old and has a lot of pain. He seeing a pain doctor in 2 days,and I'm sure he won't tell the doctor about all the bad foods he eats.What should I do, if anything. Doctor: you have to encourage him to eat healthy food and decrease candies because it is a durect cause of uncontrolled blood sugar and about pain the most proper pain releif for diabetic patients is gamapentin cap starting dose 100 ml per day at night and can increase the dose gradually"
},
{
"id": 117208,
"tgt": "What does high MCH in blood test mean?",
"src": "Patient: I was in ER last week for vertigo and lab tests were done. I also discovered I had a urinary tract infection. The CBC revealed a high MCH 31.1, immature granulocyte was o.oo and the imm gran% auto was 0.0, my TSH was 7 The ER doc was not concerned but I m wondering what this means Doctor: HI Thanks for posting your query . High MCH means your hemoglobin is more when compared to the size of RBCs. It is very commonly seen in Vitamin B12 or Folate deficiency which is very easily treatable . I advise you to get Serum Vitamin B12 and Folate levels done . Also your TSH is slightly elevated than normal . ( Normal is < 5) . Kindly do a Thyroid profile test in fasting condition and revert back ."
},
{
"id": 142942,
"tgt": "What causes hot hands?",
"src": "Patient: I suffer from fibromyalgia, and have been shown the 8 points of sensitivity on my body by a consultant , however my hands are unbearably hot 24/7, i even wake during the night to put them on a cold surface. Can you confirm if it is associated with my ailment or is it something else? Doctor: It seems like your symptoms are due to fibromyalgia. In fibromyalgia it is found that there is increase in nerve supply of blood vessles to palm. This increases the sensitivity of the affected part to certain sensations like heat.That is the reason for increase in the heat you feel on your pal.Hope this answers your question, thank you."
},
{
"id": 163839,
"tgt": "What causes recurring diarrhea and vomiting in a child?",
"src": "Patient: Every month or so my 13 year old son gets major diarrhea and vomits once. He does not run a fever during this time. He also complains of having a rotten egg taste in his mouth or of burping rotten egg taste. What is this and why does it keep recurring? Doctor: dear userabout your question i would like to tell you that recurrent diarrhea has some causes in this age of your son.......allergy to food cause the intestinal movement to increase rapidly so causing diarrhea and vomiting.......so you should notice if there is food that if he eat symptoms increase to avoid that food second if your son had intestinal infection of the past or had severe gastroenterities that may cause damage of intestinal epithelium and so maldigestion and recurrent diarrhe occur and it will need some time to improve gradually as it is self limitedif your son eat unhealthy food or fatty food or spicy food that may cause these symptomsalso you should do stool culture for him and widal test to detect if there is salmonnela infection causing typhoid or another intestinal infectionas the intestinal infection viral or bacterial should be managed to end of this problemhope that helps youi wish you and your son happy life"
},
{
"id": 196963,
"tgt": "What causes pain in penis and yellow penile discharge?",
"src": "Patient: Hi.. I am 25 years old and married for about 8 months.. From 2 months i am away from my wife and only had sex once 2 weeks ago with a girl friend... From 2 days i feel pain in my penis and a continuous flow of yellow liquid from my penis... It is sticky like sperms but not thick... I feel a lot pain when i go to the wash room...Kindly advise what could be the cause... Doctor: HelloThanks for query .Based on the facts that you have posted passing yellow coloured discharge form penis and burning urination after sex is mostly due to infection of urethra (Urethritis) and needs to be confirmed by doing following basic tests 1) Routine urine test and urine culture 2) Swab test and culture of a discharge from the urethra.Start taking antibiotic like Cefotaxime and Nitrofurantoin twice daily along with urine alkalizer like Citralka thrice daily later on switch on to appropriate antibiotic as per culture report .Ensure to wash your genital with waarm water twice daily Ensure to drink more water to keep your urine dilute Ensure to avoid sexual encounters with female other than your wife .Dr.Patil.."
},
{
"id": 202952,
"tgt": "What is the bump on nut sack that squeezed out white stuff and blood?",
"src": "Patient: I have a bump on my nut sack started out small got to the size of a pea.It was very hard and under the skin but not on my balls I poked it with a needle and clear stuff came out off it. After squeezing it very hard white stuff and blood shot out of it. I cleaned it very well with soap water and peroxide. Then put some antibiotic ointment on it. Is there anything i should worry about? What do u think it is? Thank you for your time. Doctor: Hello,I have gone through your query and understand your concern.I suspect you have Sebaceous cyst also called epidermal cyst.This is mostly seen is hair rich areas like scrotum, scalp etc.It contains the white cheesy substance as the one you had on prick of the swelling.It is not advisable to prick or cut open the cyst.Because it get infected.As you have applied antibiotics nothing to be worried now.It may develop later and it should be excised completely by surgeon.Hope this helps.Please write back for further queries.Wishing you good health."
},
{
"id": 119943,
"tgt": "Suggest remedy for bumps in foot",
"src": "Patient: My 3 yr old son has 4 bumps on the top of his foot around the base of his toes that look like bug bites, but all of them have a faint red circle around them like a bullseye. He says they don t itch, but he won t let me touch his foot. Could this be spider bite? Doctor: Hello,The symptoms seem to be related to an insect bite or also an allergy. You should monitor them. Meanwhile, I suggest using a mild steroid cream for local application such as Betametasone cream.Hope I have answered your question. Let me know if I can assist you further. Regards, Dr. Dorina Gurabardhi, General & Family Physician"
},
{
"id": 185393,
"tgt": "What causes severe pain in the jaw after teeth extraction?",
"src": "Patient: my husband had all his teeth extracted, after a week he is still in severe pain. they did an x-ray of his whole mouth today(a week later. He said he had bone spurs on his jaw bones and had to clip all of those off. He said this splits in his jaw bone were where the tooth grew into his jaw bone deep and it was normal. does this sound right. He's had several complications with this procedure. did they fracture his jaw bone while extracting his teeth, or how can i find out. I need help, he is several pain. They had to cut his gums open again and re stitch his gums. Thanks much Doctor: Hello,This infection which they are mentioning it looks that your husband is havind a disease known as oesteomyelitis.Better that once you should visit an oral surgeon as he will manage the condition more properly. Hope it will help you."
},
{
"id": 24225,
"tgt": "What causes the heart beat rate to lower?",
"src": "Patient: I am 50 yr I am taking tenolol 50 plus amtas 5 for the last 7 years . I walk briskly for 45 minutes daily. For the last about 6 months my heart beat is generally 60 p/m . My bp is absolutely ok . Why heart beat is down . Should I think of some alternative medicine Doctor: dont worry, as you are having good exercise capacity , your heart rate around sixty is acceptable. your heart rate is slightly low due to tenolol. but you can continue same drugs as your bp is normal , having good exercise capacity with slightly low but accptable heart rate."
},
{
"id": 191033,
"tgt": "An adult tooth has sprouted with the baby tooth still in, kindly suggest",
"src": "Patient: My teeth are being strange, an adult tooth has sprouted with the baby tooth still in. The adult tooth won t grow, and the baby is on it s side, immovable as a house. Can someone tell me what s going on ? Doctor: Hi, It is possible that even when the milk teeth is present the permanent teeth erupts around it. The corresponding milk teeth should be extracted. The eruption period for different milk teeth are different. They should be extracted at the proper time. A dentist on examination can tell which teeth has to be extracted at what age."
},
{
"id": 205466,
"tgt": "Suggest remedy for short temper",
"src": "Patient: hi, one of my friend s wife got short temper and so daily quarrel is taking place in their home . She is about 48 or so and little fatty. She does not go out of their home for any mingling with neibours or like that. As a friend am requesting you to advise if anything can be done for this. Thanks Doctor: DearWe understand your concernsI went through your details. Usually anger is due to irritation. Irritation can be due to frustration or disturbance from others. Stress is another factor with which people do become short tempered. Over and above, a sort of low self esteem and low self confidence also triggers anger and aggression. Before advising the method to deal with the client, we would like to know more information about the client. We need to assess the dept of the client's anger. Is it just anger as an emotion is anger disorder? proper assessment of the client would provide the necessary insight.Emotional anger and aggression can be approached with the help of anger management therapy and slight life style changes with awareness. Anger disorder, most probably, could require treatment with the help of a psychiatrist and psychotherapy. I strongly suggest you to advise your friend to consult a psychologist personaly or online. The problem can be cured, for sure.If you still need my assistance in this regard, please use this link. http://goo.gl/aYW2pR. Please remember to describe the whole problem with full detail.Hope this answers your query. Please feel free to post follow up queries. Available for further clarifications.Good luck. Take care."
},
{
"id": 108960,
"tgt": "What causes pain in lower back to front?",
"src": "Patient: I have a pain starting in my lower back going around front - belt line. I feel like a buldge is on my right hand side. No gaul bladder and I do have a fatty liver. I stopped drinking twelve weeks ago and sent off of all fats and on a high fiber diet which helps a little bit. Back pain also. Doctor: Hello, I have studied your case. Pain on right side along with back can be due liver disease in your case.Edema over liver which may lead to pattern of pain in right hypochondrium region.Ultrasonography abdomen and pelvis along with blood LFT will help in diagnosis.Another possibility of pain arising from thoracic spine can be kept which leads to radicular pain from back to anterior area.Infection in thoracic spine can be common or can be due to nerve compression.I will advise to do HRCT or MRI scan consulting your treating doctor.Here are my other useful suggestions to you: Avoid lifting weights, take rest, and eat easily digestible food.Hope this answers your query. If you have additional questions or follow up queries then please do not hesitate in writing to us. I will be happy to answer your queries. Wishing you good health. Take care"
},
{
"id": 179111,
"tgt": "Suggest treatment for mucosal irregularity in bladder wall in a child",
"src": "Patient: hello doc,my name is vivek and i ve a kid name athrv, age 1 yr 3 months, sex male.the patient is sick since over last two months having continuous gains/drops in fever. initially the doctor started the treatment of having infection in stomach. but that was of no use then suddenly the patient started losing his activeness and his temperature also started raising. one day his temperature got raised upto 104 deg celcius. the patient was rushed to some other specialist and he declared the disease to be pnemoniya, the doctor then started his treatment of having the symptoms of pnemoniya. after 2-3 days the doctor again said that he might have a doubt of the patient having some other issue also. he then conducted a ultrasound of the patient, the reprt of which stated that urinary bladder reveals mucosal irregularity seen involving the bladder wall. the doctor then said look i knew that there is some other problem then he stopped the treatment of pnemoniya and started paying attention to the infection in urinary instentine stating that this is the real problem and pnemoniya has got over.please advice Doctor: Hello Vivek, thank you for posting your question.The clinical diagnosis of any fever is based on a few principles. Firsly we enquire about the associated symptoms the child had with fever. Apart from that the extent and pattern of fever, whether it was associated with chills or came at any particular time of the day etc. These details would help us know what would probably have caused the fever and based on that investigations are usually ordered. In case of Athrv, as you informed he was initially treated for a stomach infection, then for pneumonia and finally for urinary tract infection. In such a situation you have two possibilities. Either he had 3 different kinds of infection which presented with fever over 2 months, in which case his fever should have subsided each time with the medication, he should have felt better and then again fallen sick with a new infection. If that is not the case then his fever could probably be attributed to a single cause , with the other two being misdiagnosis. In such cases investigations could give support and clue as to what the exact cause of fever is. Mucosal irregularities in urinary bladder is a very non specific finding and the diagnosis of a urinary tract infection should never solely be based on that. Associated symptoms of pain or difficulty in urination along with a positive urine culture is required for diagnosis. If you could provide any more information regarding what his urine culture report was, or what his associated symptoms were, then it would be much easier for me to provide you any further guidance."
},
{
"id": 167945,
"tgt": "Is thick brown vomiting in an infant post breast feeding serious?",
"src": "Patient: My five day old baby is vomiting thick brown stuff. He is exclusively breastfed. I suspect a stomach virus since others in the family have had it too. He vomited a lot after his last feeding, which was off the breast that is not at all sore. The other is a bit sore and could possibly be bleeding, but it seems unlikely to show considering he hadn t nursed from that side for that feeding. Should I be concerned about the brown? Doctor: probably your baby has swallowed maternal blood , either from the breast or during delivery a viral illness is very less likely make sure that your baby received vitamin k after delivery"
},
{
"id": 102213,
"tgt": "What causes diarrhea, cough, sinus and sneezing inspite of using acetaminophen for headache?",
"src": "Patient: I was sick with a fever anf head ache and body aches 3 or 4 days ago and I took some acetaminophen and I felt better and it was either the next day I had diarrhea and I have had a cough yesterday my sinuses starting draining and ive been sneezeing I never sneeze like this and im a little concerned? Doctor: Hello,Welcome to HCM,The symptoms suggest me that you are having some infection might be of viral origin.As yor symptoms started with headache and fever it may be a secondary bacterial infection which has lead to diarrhea, cough and sinusitis. I would suggest you to followOral antibiotics like Bactoclav, 625 mg , twice daily for five days.Steam inhalation with VapocapsCough syrup like codeneOral antihistamines. These measures will improve your condition. Thank you."
},
{
"id": 123135,
"tgt": "What causes heel pain and feeling of something torn?",
"src": "Patient: Hi, I was running and my heel started hurt immediatly. I rested later and it completely felt fine to walk, just like normal. Then i got a little bit for active(basketball) and it started to hurt terribly again like something was torn in the heel again. I dont what know happened. Doctor: Hello, It could be an attack of plantar fasciitis.As of now you can use analgesics/anti-inflammatory combination like aceclofenac/serratiopeptidase for symptomatic relief. If symptoms persist, it is better to consult a physician and get evaluated. In severe cases you can go for an MRI scan also. Hope I have answered your query. Let me know if I can assist you further. Regards, Dr. Shinas Hussain, General & Family Physician"
},
{
"id": 164645,
"tgt": "What would a large vein in right temple/shoulder in a diabetic child be?",
"src": "Patient: my 6 yr old daughter is type 1 diabetic and we recently stopped pump therapy and returned to injections so her sugar has been out of control for 2 days now. it s getting better as we do as her doctors instruct us but today while at my mother s house, a large blue vein appeared on her right temple by her eye and started extending down her face. also noticed one on her right shoulder. could there be vein problems related to her diabetes or is this unlikely? or maybe something cosmetic Doctor: Hello Welcome to health care magicit is of no significance, dont worry it is normal, it is not due to change in insulin therpy"
},
{
"id": 110147,
"tgt": "What causes bowel incontinence and pain in the back?",
"src": "Patient: Hi. Since having a radical hysterectomy I have had bowel incontinence. Also a searing pain in my groin radiating underneath which goes numb as if I have had anaesthetic & chronic pain in lower back. I cannot move when this happens. I was given a Piresteen kit to clean out my bowel , but after using am in awful pain. Had an MRI this week , no herniated disk. They said I could have nerve damage due to hysterectomy. I have been diagnosed with nerve damage by bowel clinic. Do you think this could be down to my op ?. Thank you. Doctor: Hi, welcome to health care magic after reading your problem my view is that radical hysterectomy is a major surgical procedure which involves gross handling of pelvic structures. and the indication for which this hysterectomy is performed is also very important as it affects the out come of surgery , even in the best surgeons hand."
},
{
"id": 145094,
"tgt": "What causes dementia?",
"src": "Patient: MY 77 YEAR OLD BOYFRIEND HAS DEMENTIA WHICH I BELIEVE IS NOT AGE-RELATED BUT RATHER DUE TO SOMETHING EITHER LIMITING BLOOD FLOW TO BRAIN, A VITAMIN DEFICIENCY, OR SOME OTHER CONDITION THAT CAN BE CORRECTED. WHAT TEST(S) WOULD DETERMINE WHAT IS CAUSING HIS PROBLEM? IT STARTED WHEN HE HIT HIS HEAD WHEN HE FELL DUE TO LOW ELECTROLYTES, HE WAS INA-FIB WHEN THE MEDICS ARRIVED. THE HOSPITAL SAID HE DID NOT HAVE A STROKE, BUT I WITNESSED A SLIGHT SEIZURE. EVER SINCE HE HAS DEMENTIA. I DON'T WANT TO IGNORE DOING SOMETHING THAT MIGHT RESTORE HIS COGNITION AND MEMORY. PERHAPS BLOCKED CAROTID ARTERIES. WHERE SHOULD I GO AND WHAT SHOULD I DO? HE WAS ALSO EXAMINED BY A HEART SPECIALIST AND EVERYTHING SEEMED FINE. I GIVE HIM COCONUT OIL AND ASTAXANTHIN, VIT. B 12 AND D 3 PLUS OMEGA 3 FISH OIL. PLEASE HELP ME. I AM DESPERATE. Doctor: Hello thanks for asking from HCM for your health related queriesI can understand your concern. He has developed dementia. There are a lot of causes for dementia at this age. They are Age related dementia, Alzheimer's disease, Deficiency of Vitamin B12, Niacin deficiency, Vascular dementia or Multi-infarct dementia, Dementia due to some infections etc. In my opinion if you don't think he has age related dementia he should be investigated accordingly to the causes I have mentioned. His brain scan would rule out any evidence of vascular damage to brain and vascular dementia can be ruled out. He should undergo MMSE test (Mini Mental Status Examination) to find out extent of dementia. This would help to judge improvement also. You are already giving him supplements. He should get proper benefit with drugs like Donepezil, Memantin etc. These drugs would help to reduce the progression of dementia.Brain stimulation games like Sudoku, Crossword, Chess etc also helps to reduce the damage.Thanks, I hope this helps you. Take care."
},
{
"id": 170572,
"tgt": "What is the treatment for child who totally depends on others for everything?",
"src": "Patient: I want to consult Dr. Devika Nag about my Six years daughter. She is totally depandent on others for her careness. She either could talk, stand, sit, eating food and so on. She lies on bed only. But she understand our talks. Give me suggestions for her treatment. Lila Pandey Nepal Doctor: It appears that your daughter has a motor deficit with normal mental development ,more appropriate can be motor deficit and a behavioural disorder .There are many causes for this and treatment will depend on the cause .Your child needs a specialist help --start from general pediatrician ,pediatric neurologist and proceed.from there ---more specific assessment can be made"
},
{
"id": 147696,
"tgt": "Positive Babinski reflex, joint pain, muscle weakness, fatigue, fainting. Neurological condition?",
"src": "Patient: Is a positive Babinski reflex a strong indicator of a neurological condition? I am experiencing this, plus extreme joint pain (in almost every joint), muscle weakness , fatigue , dizzy spells/ fainting accompanied immediately by a numb/tingling tounge. My head feels strangly full from the nose up....I don t know how to describe it. It s a sort of heavy feeling but not painful. Every day for the past week, I seem to get a new symptom. Today, I am finding it more difficult to move my arms. Any opinons? Doctor: Positive babinski reflex signified upper motor neuron disorder.You required a good neurologist your problem could be comeing from cranial nerve dysfunction.You may required mri brain.Thanks."
},
{
"id": 129565,
"tgt": "What does muscle ache and fatigue with red rash on body indicate?",
"src": "Patient: I had muscle tiredness and aches then I noticed a few white sports on my skin. I though these may have been mosquito bites but the next day there were more of them and now I have a red rash all over my body. Flu symptoms have gone but my muscles now feel strangely tight. Or maybe it's my skin. Doctor: Hi welcome to the health care magic You are having rash and flu symptoms in past Rashes are present whole over body.. Hence these seems to be viral infection induced rashes... If you post photo then better comment can be given For rashes you can prescribed antihistaminic drug like levocetrizine.. Topical calamine lotion is also beneficial Your CBC report should be done Take care Consult dermatologist for examination of rashes (along with CBC report)"
},
{
"id": 164018,
"tgt": "What causes high level of monocyte in children?",
"src": "Patient: my 11 year old son has a monocyte level of .8 on the form it says that the normal range is .0 - .8 it is denoted as high. everything else seems to be with in normal range. he has no symptoms of flu, fever or cold. What could be the problem. Also we live in a tropical environment. Doctor: Thanks for asking on Healthcaremagic. From the reference range provided it seems that the absolute monocyte count of your son is within the normal range though on the upper limit of normal range. It is not a cause of concern in the absence of any symptoms. Kindly be reassured that everything is ok. Hope it helped."
},
{
"id": 35386,
"tgt": "What diet should be followed when suffering from recurrent chicken pox?",
"src": "Patient: Hello, I got a chickenpox on 14/11/14, now I got black coat on all blisters. I know that I should avoid non-veg for at least 2 months but can I take milk products like butter, paneer, ghee etc.... Please suggest some what should I eat or avoid. Thanks, Chandan Singh Doctor: Hello dear,Thank you for your contact to health care magic.I read and understand your concern. I am Dr Arun Tank answering your concern.No, you are having false belief of diet restriction in case of chicken pox.You can eat whatever you want to eat. You can eat chicken also. Only thing you have to avoid the spicy foods.Chicken pox is caused by the virus. Its infection progression and multiplication has not influenced by your diet.You have to check for the status of your vaccine and according to you have to consult your doctor.Please take bath with medicated shop which can be available in nearby medical shop.Please wear cotton cloths as it can prevent secondary infection.I will be happy to answer your further concern on bit.ly/DrArun.Thank you,Dr Arun TankInfectious diseases specialist,HCM."
},
{
"id": 75069,
"tgt": "What causes constant chest pain in child with high fever?",
"src": "Patient: A long story that I will try to make short. Six year old child with six days of high fever 103-104 in November. a week later she started to complained of chest pain. She complained a few times over two weeks. took to doctor and saw cardiologist. xray normal, echo normal and holter monitor for 24 hours was normal. end of December no more chest pain for about two weeks. Chest pains returns and ped. says may be gastro try tums. First week on Tums still frequent chest pains lasting from 10 sec. to a minute. Then three weeks chest pain free. saty on tums for another week and stop. Week later chest pain returns but more intense and often (15 times per day). Make an appt. with ped. gastro but have to wait 1.5 months (appt. tomorrow). Chest pain is persitent during this time. Then for the past 6 days she has once again a high fever (103-104) with fatigue. Ironically the chest pains seems to subside but still there occasionally. Ped. said possible respitarory infection as she heard slight whizzing. Started antibiotic today. Are these fevers and chest pain related? Doctor: Thanks for your question on Healthcare Magic. I can understand your concern. Yes, chest pain and fever are connected. In child common causes for fever and chest pain are pneumonia and bronchitis. So better to first get done chest x ray to rule out pneumonia. If chest x ray is normal then get done PFT (Pulmonary Function Test) for the diagnosis of bronchitis. She might need antibiotic, inhaled bronchodilators and inhaled corticosteroid (ICS) on the basis of reports. Don't worry, she will be alright. First rule of bronchitis and pneumonia. Hope I have solved your query. I will be happy to help you further. Wishing good health to your child. Thanks."
},
{
"id": 215115,
"tgt": "I lost my vision in one eye after an operation of glaucoma. Now what can i do ?",
"src": "Patient: Dear sir,I have been suffering from glaucoma for the past 10 years.I had a operation in my right eye in2001 in which the operation failed and my sight was gon Later I visited to Shankaran Netralaya,where they prescribed me Lumigan eye drop .However with the request of my wellwishers I visited an ayurvedic centre in Kerela, but there was not any improvement. 4 days back the allopath doctor operated my left eye and once again the operation failed and now i am almost totally blind. Now with ahope I want to know that , is there any hope left for me. I am 83 and alone ,so for the sustainence of my left life I need at least some sight so that I should be able to do my essential works. Thank you.Your s sincerely Md. Samiul Haque. Doctor: Take Tab.Saptamrit Lauh 1 tab thrice a day Trphala Ghrit 10ml twice a day for 3mths"
},
{
"id": 147852,
"tgt": "Could i be getting plursy with pain in my spine and shoulder blades?",
"src": "Patient: Hi, I had Bronco Pneumonia and Sinus Infection about 10-14 days ago then kidney Infection 7 days ago. I am still barely able to function. When will I get my energy back? I can't even go to the grocery store without having to sit down several times. Believe me my list gets a lot shorter at that point! I end up with this gasping breathing thing happening by the time I get to the car. My husband keeps telling me to slow down on my breathing. He is worried I am going to hyperventilate. My heart is pounding just walking upstairs. And the weird breathing thing happens. I don't particularly want to go back to the Dr unless I need to. But I have mytrol valve prolapse and high blood pressure. Another thing is I am having achy pains going down my spin and kinda under my shoulder blades. Could I be getting plursy? Motrin and the heating pad helps. And Vicodin. But want and need to get better. Quick. I have a 6 year old . And my house is looking awful. So please any ideas? Doctor: HI, thanks for using healthcare magicIt can take 2 to 3 weeks for a person to regain their normal level of energy after an infection, so you still have a little while to go.It would be best to take rest breaks to allow your body to continue healing and do not push too hard.The pains are not indicative of pleurisy- this is inflammation of the lining of the lung and it normally presents with chest pain that is worse on taking a deep breath, coughing or laughing.The pains are likely related to muscle strain.I hope this helps"
},
{
"id": 136476,
"tgt": "Suggest treatment for severe leg pain",
"src": "Patient: I am a 47 year old female. Non smoker, drink rarely. I am very active and have had great health. About a week ago I began experiencing pain in my entire right leg. Its a deep, aching pain that feels as if it is in my leg bones. Nothing seems to relieve it. I ve tried ibuprofin, heat, resting it. It is becoming difficult to even sleep. Any suggestions on where I should start with this? Thank you. Doctor: HiHistory suggests you got problem in spinal disc which is compressing the neural element. It can start due to injury or lifting weight. Get an mri done to confirm the diagnosis. Start physiotherapy and avoid heavy exertion. Thanks"
},
{
"id": 103052,
"tgt": "Dry cough, sudden ache that run down the back of head. Allergy?",
"src": "Patient: I have a dry cough for the past 2 months which my doc says could be an allergy (hay fever). I have taken antibiotics for 5 days, the cough has not gone. I now take Zertec every day some times twice a day and it seems to give me relief. However what I am concerned about is that since the cough I keep getting a sudden ache run down the back of my head which occurs atleast twice a day. Is there reason to be concerned. Doctor: Hi, the cough for over 2 months means there may be grouth of uvula, a projectin at the posterior end of the soft palate. When it grows due to cold, sinusitis, pharyngitis it touch the pharynx, and produce cough.Other causes are eosinophelia,(can be confirmed by D.C, and A,E.C)and tuberculosis.the posterior head ache may be present in sinusitis. For local relief of cough there are preparations containing codeine. For total treatment i to my patient with such symptoms prescribe cifixime, and syrup containing codeine. Thank you."
},
{
"id": 114811,
"tgt": "What does a score of 0.3 immature granulocytes indicate?",
"src": "Patient: Should I be concerned with a score of 0.3 immature granulocytes? I am a 75 year old woman with a BMI of 22 and take no perscriptions. I do take naproxen sodium 220 mg twice daily for arthritis and have for over 5 years. My father lived to the age of 94 and his sister is 95 and still works part time. My Mother died at the age of 58 from ovarian cancer. I don t seem to have any of her health issues so far. Could you please inform me about this concern. Doctor: Hi, dearI have gone through your question. I can understand your concern. You have 0.3 immature granulocytes. It has no clinical significance. High number of immature granulocytes seen in infection. But 0.3 has no significance. So don't worry about that. Just be relaxed. No any treatment is required. Hope I have answered your question, if you have doubt then I will be happy to answer. Thanks for using health care magic. Wish you a very good health."
},
{
"id": 21428,
"tgt": "What causes arms, legs and face numbness?",
"src": "Patient: i am 55 yr old female. i do have high blood pressure. i was sitting in recliner this evening kind of 1/2 napping and had what seemed to be a very real dream falling in the bathroom and not being able to speak or move. my husband said that i was thrashing and somewhat screamed. after that I got up and my arms, legs and face feel really numb. that was about 3 hrs ago and still very numb. Doctor: Hi,This could be a brain stroke (Transient Ischaemic Attack) & you should immediately consult a neurologist for further evaluation & management.Thanks"
},
{
"id": 44841,
"tgt": "Why I am not able to conceive though both our reports are normal ?",
"src": "Patient: Hi Doc, I am 27 and trying to conceive from past 6 months, FSH, LH and prolactin levels are normal. My husband is also not having any issues. Periods are normal and my follicle size will be 23 mm before ovulation . I have ovulation kit with the help of that and bbt we are planning. My doc is telling that I do not have any issues. Can you pls tell me why it is delaying? do i need to go for any other tests? will Travelling cause trouble? Thanks. Doctor: Hi Welcome to HCM Read your prob, you should get thyroid profile, as hypothyroidism is also cause of infertility. you have proper ovulation then you should check patency of tube. as its blockage or any disorder at the leval of tube lead to infertility. discuss it with your doc n start proper treatment. travelling cant affact fertility."
},
{
"id": 157572,
"tgt": "Chronic low WBC count. On IVIG treatment. Have history of breast cancer. Advised bone marrow biopsy. Cancer?",
"src": "Patient: I had breast cancer in 2003. I have been cancer free for 10 years. after my cancer my white blood cell count became extremely low and I have been taking IVIG treatments for 8 years. Until 3 months ago it was at a good level but then I started having all kinds of infections an my Dr. increased my dosage of my IVIG meds. and instead of every 28 days moved me to every 21 days. I had some labs run the last wk. and was told that my white cells had dropped to a chronic low and I would probably have to have a bone marrow biospy within the next 6 months. The nurse said she was sorry to have to give me such bad news. I don't understand exactly what is happening. Can you help me to understand? Doctor: HIThank for asking to HCMI can understand your concern, the low WBC counts indicates so many etiologies from infection to hematological condition and that need to be differentiate from each other the first step would be hematology test, and the bone marrow is the only way for this to find out the exact cause, let us hope for the good you can ask any further question without hesitation have great day."
},
{
"id": 98794,
"tgt": "Suggest treatment for blisters on the inside of lower lip",
"src": "Patient: I have a blister on the inside of my lower lip. It almost looks like my lip is swollen where the blister is, although it doesn't look like herpes because it isn't on the outside of my mouth. I thought it might be a canker sore; it is not irritated when I drink orange juice or put salt on it. I have had cold symptoms I don't know if this is related. The blister is painless and looks a lot like the color of the rest of my mouth. Doctor: even though blister is not hurting u should consult ent Doctor and if required get cytological or histopathological examination done of d blister then only definitive diagnosis can be done."
},
{
"id": 96476,
"tgt": "What should be the proper diet for gastro-esophageal reflux disease patients?",
"src": "Patient: I have a gastric problem. I had been admitted last month for severe pain in my abdomen. Doctors says that it is because of high acidity. An endoscopy test shows patches in the oesophagus. Also, I have been advised to have less spicy and oily foods. My question is can I have chicken eggs with my lunch? Also, can I eat non-spicy non-vegetarian food? I have to take Ocid 20 mg twice in a day for three weeks. Please help me. Doctor: I have ACCIDENTLY\u00a0\u00a0 happened\u00a0 a puncher at\u00a0 my esophagas\u00a0 gastric\u00a0 junction due to a siver phsical action of my hand for pulling long rubber hose ,at that time\u00a0 i feeled siver pain during\u00a0 eating for several days. the pain reduced day by day\u00a0 by applying local medecineas per the advice of a doctor. But after 3months my chest pain reached the maximum ,cough,unable to even stand\u00a0 or\u00a0 walk\u00a0 then it was cured\u00a0\u00a0 after\u00a0 taking\u00a0 a local injuction\u00a0 from the near by Medical\u00a0 college\u00a0 at back side of my spinal code (\u00a0 middle of shoulder)this is about 30 years ago- now i am having hart burn--& taking treatment.now i am having geographic\u00a0 mark on my toung . is it\u00a0 asymptom of any other dises.(sadanandan_p2003@yahoo.co.in)"
},
{
"id": 173093,
"tgt": "Suggest remedy for fever and yellowish discharge from eyes",
"src": "Patient: My one year old has a fever, yellowish,sticky discharge from eyes and is twitching in his sleep which makes him wake up. Does he have pink eye? His lids are slightly swollen, but eyes are somewhat clear. He also has a cold. Is there anything I can do at home to make him feel better? Doctor: Hi, I had gone through your question and understand your concerns. Conjunctivitis and fever are commonly observed in adenoviral infection. If such a patient comes to me in my clinic I would prescribe:-wash eyes with tea water;- Ciprofloxacin drops 1 drop 6 times every 2 hours for 3 days,then 1 drop 4 times for 3 days;- Paracetamol 10-15 mg/kg/dose along with tepid sponging can be used for fever up to 100F;-Zincovit syrup 5 ml daily for boosting immunity can be given for 1 month;-Keep baby hydrated with plenty of water,fresh juices.Since this a prescription-based medicine I would suggest you to meet the local doctor to confirm the diagnosis if fever will more than 3 days."
},
{
"id": 116198,
"tgt": "What causes high MCV and MCH count and low PLT count?",
"src": "Patient: high MCV count and MCH count and low PLT count. Also the hematologist ordered I get blood extracted cause it is too thick. I went a couple of days ago and even with drinking plenty of water there blood could not be extracted. I am going back tomorrow and have been in low dose aspirin during. Please what are the causes and what could it be? Doctor: Welcome to H.C.M. I Am Dr krishna Dubey.My pleasure to help you.(MCV) is the average size of the red blood cells.When the MCV is elevated (as in your case) the RBCs will be larger than normal and are termed macrocytic.The MCHC is normal or increased in macrocytic anemias as megaloblastic anemia.Low platelet are also common in megaloblastic anemia.Thick blood means there are chances of clot formation so your doctor is put you on Aspirin.I advise you to repeat your blood count and peripheral blood film.This will help us to find out the cause and further management.Thanx for query."
},
{
"id": 205619,
"tgt": "Suggest treatment for pituitary disorder and depression",
"src": "Patient: Hello, I am trying to figure out if my husband could have a pituitary disorder. For the last two years he has had consistently low levels of testosterone with normal levels of LH and FSH. A year ago blood work showed he had low total t4 but normal TSH. This week his free t3 and free t4 were as low as they could get withing normal range. His t4 was .8 and that was the lowest in the range. However, his TSH was on the low end of normal. Despite undergoing testosterone replacement therapy for a year his levels never increased. He has developed severe mood problems and was diagnosed Bipolar type 2 a year ago after a hospitalization. He has very severe depression and absolutely no energy. He never has euphoric mania. He only experiences periods of terrible rage consistent with IED, racing thoughts, and the need for activity. Even in his high states he has no energy. Growing up he was overweight but lost 60 pounds. Later he rapidly gained a hundred pounds and lost a patch of hair out of his beard. He has always felt bad about his chest as well and felt that even at his thinnest he too much fat there. He has been rapid cycling and experiencing mixed states. Also, he has not had any break or normal period since suddenly developing his mood problems two years ago. I do not understand why his pituitary sending hormones do not increase when he has low levels of hormones such as thyroid and testosterone. Is it possible he has an endocrine problem worsening bipolar? Or could the bipolar be a misdiagnosis? Doctor: Hi! I'm Dr. Neel Kudchadkar and I will help you with this problem. I would like to politely request you to give me some information about your husband, such as 1) Did he have any thyroid problems BEFORE a bipolar II affective disorder (Hypomania with dysphoria and depression: BPAD Type 2.) Did the bipolar issues surface first and then, a diagnosis of thyroid/ pituatory / HPT was made? 3) Regardless of the hormone levels currently in his blood serum, I would state with confidence that Lithium is the culprit behind the thyroid problem. Lithium Carbonate is the most common drug prescribed for Bipolar II, and causes thyroid problems. I would ask you to stop Lithium if it is being taken and to try another medication called as Lamotrigine at a dose of 100mg at night after food. This drug (Lamotrigine) was designed exactly for the state of mind your husband is in. I would state with confidence that the Bipolar diagnosis is 80% probably right. I'd like to have a conversation with you as regards the details of this case, and being a neutral observer, you could provide me with these details. Kindly provide me with the details, including what current medications your husband is taking, because these are important, for his cure. You might wish to contact me 24x7 at this link http://www.bit.ly/askdrneel Please send an email to customercare@healthcaremagic.com with the subject \"ATTENTION: Dr. Neel Kudchadkar\" A mutually beneficial dialogue could begin. In confidence,Dr. Neel Kudchadkar (Psychological Medicine)HealthCareMagic"
},
{
"id": 161835,
"tgt": "What causes excessive breathing in the baby?",
"src": "Patient: Hello, I have a 4 1/2 month old baby who is breathing at 72 breaths per minute resting. She was 38weeks at delivery and, while small (2.75 kg) was pretty good for a twin. Her growth is fine and development normal. I am just concerned about breath rate as our 3 year old has bronchiolitis obliterans and is oxygen dependent. I think we did not diagnose her problems early enough. Any advice? Doctor: Hello, By what you say this seems to be a medical emergency to me. The baby's respiratory rate requires hospitalization and evaluation. It could be acute bronchiolitis, please take her to the nearest Emergency Room as soon as possible. Hope I have answered your query. Let me know if I can assist you further. Regards, Dr. Sumanth Amperayani, Pediatrician, Pulmonology"
},
{
"id": 139041,
"tgt": "What causes swollen,itchy red lump after an injury in my arm?",
"src": "Patient: i knocked my arm half way up between my wrist and elbow on the radius side about 2 years ago at the time it was bruised and swollen but know and again it goes itchy red hot and a swollen and a lump appers which gets bigger in size its happening now why is this ? Doctor: Hi, I value your concern regarding the symptoms. I have gone through your symptoms, and in my opinion these are signs of infection, and you will need an X ray and some blood investigations, to make that clear, your treatment will depend on the findings in your examination and the report of your tests. Hope this answers your question. If you have additional questions or follow up questions then please do not hesitate in writing to us. I will be happy to answer your questions. Wishing you good health.Special note- Any medication prescribed needs to be taken after consultation with your personal doctor only."
},
{
"id": 59135,
"tgt": "Had jaundice. Have high level of SGPT and SGOT. Is hepatitis F without liver transplantation chronic?",
"src": "Patient: doctor i was being affected by jaundice(hepatitis E) before 1 and half years,due to water contamination and never undergone liver transplantation andall,and had alcohol after 6 months...when i went for LFT...my SGPT and SGOT level went high and Total billirubin too and caused jaundice again...and now it is normal doctor...can i consume alcohol again else after how many days can i have it????kindly dont mistake.....is hepatitis E without liver transplantation chronic????doctor? Doctor: Hello You Know that if any liver disease alcohol is strictly contraindicated. then why you want to drink it . you know very well last time you report also up going so keep away from alcohol if you live normal life and take proper food then no any problem It is particularly dangerous for pregnant women not you one more think hepatitis E vaccine developed by Chinese scientists only and approve by china only. you can take help of ayurveda also Dr Vinod"
},
{
"id": 157581,
"tgt": "Has prostate cancer, PSA increased. On hormone therapy and radiation. Done blood test. What is wrong?",
"src": "Patient: Thank you for being there. We are beside ourselves. My son-in law has prostrate cancer; 2009 he had a prostectomy, 2011 radiation, and now his PSA has doubled (1.3 > 2), Gleason of 9. Before he has any more surgery, radiation, or hormone therapy - we'd like to rule out any fungal issues. Is there a doctor any where in West Michigan that understands the antifungal issues that impact prostate cancer? My son-in-law is active, healthy, wife and 3 beautiful children (13, 15, 17). Could the last blood test have been \"wrong\"? Doctor: If the PSA is 2.there is nothing to worry., because the upper limit of normal(ULN) 4ng/ml, but you should repeat the PSA., then consult your oncologist."
},
{
"id": 194786,
"tgt": "What causes pain in penis and testicles with watery semen?",
"src": "Patient: hello. i wish i can get full information about what i am gona ask. i have sexual problem and my penis have pain and the egg i dont know what its called but also pain in my right egg of the penis and when i cum its dont cum that much and its like water its not strong as before. i wish i can have some information or medical that i can use Doctor: Hello, Without having a physical examination of your testicles and without having an investigation result of your sperm nobody will be able to provide you treatment. Please consult a doctor in person. Hope I have answered your query. Let me know if I can assist you further. Regards, Dr. K. V. Anand, Psychologist"
},
{
"id": 35134,
"tgt": "Suggest medications for red bump like swelling in eyelid",
"src": "Patient: I woke up thinking that my eye was swollen. It feels like a bruise every time I blink. I looked where the pain is and the area is a little red and there was a little bump on the rim of my eyelid. I was wondering if it is a stye or pimple and how to treat it. Doctor: hi,From history it seems that there might be having formation of stye giving this sensation.Put antibiotic eye drops 2-3 hourly for 3-4 days.If swelling and pain increases, go for one antibiotic medicine course for 3-5 days.Ok and take care."
},
{
"id": 90502,
"tgt": "Suggest treatment for pain in lower abdomen till groin",
"src": "Patient: I have been having issues with my lower abdomen for the last 18 month or more. My symptoms are varied but key features are pain on the left side from the belly button wraps round to the side bad down into the left groin . There is a subtle swelling of the abdomen on the left and also left top of thigh. There is a feeling that there is something heavy in the abdomen pushing out the pelvic bone and ischium, the bone I am sitting on and causing pain. I have suffer from night sweats and find it difficult to sleep as there is always pain.. I had tests and scans but it didn t turn up anything. It was suggested that I had IBS but the pain and pressure is very localised to left side of body only. I have been round the loop a few times to try and get to a diagnosis but was advised to take IBS medication. This doesn t make any of the symptoms go away and I do despair of the pain. Doctor: Hi.Thanks for your query.Read the whole history carefully. To recapitulate: 18 months-pain from belly button to --- as described - to groin- swelling- feeling of pushing- night sweats- This can not be due to IBS.There has to be a reason for this. \\I would advise you to go for a PET scan also along with an MRI of the abdomen and upper leg. This will definitely help to find out or rule out the cause. Also important is a clinical examination , I mean real physical examination to confirm by a Doctor the severity of the swelling - to confirm it is really present and its relation to the other findings. Also go for the blood and urine and stool tests - rule out diabetes, typhoid.Blood culture and sensitivity does help sometimes.Occasionally the findings of osteomyelitis does take some time to be picked by the CT or MRI. So the need to repeat,.."
},
{
"id": 64654,
"tgt": "What causes tender lumps in legs?",
"src": "Patient: I am 55 year old man i got painless multiple (>3)small lumps (about 1 cm in diameter ) on the lower anterior part of my both legs it looks like skin elevation, and on pressure it feels like click, they are colorless and not tender, i have well controlled HT, otherwise i have no other medical problem, what might be the cause of these lumps? Doctor: Hi,Dear,Good afternoon.Thanks for your query to my HCM-virtual clinic.AFter indepth study of your query,In my opinion-these multiple-lumps are -of -Venous varcosity,indicating non- competant veinous valves due to deep venous thrombophlebits.Hope this would relieve your worrysome query.Wellcome to HCM for more queries"
},
{
"id": 106084,
"tgt": "What is the difference between odimont and odimont lc ?",
"src": "Patient: Hi doctor whats the difference between odimont and odimont lc and is it reccomended to take it before or after a meal? Male 27, 5 7 Doctor: Hi Gaurav Rakheja Welcome to HealthcareMagic. Odimont is Monteleukast .Odimont LC is Monteleukast plus Levocetirizine.Odimont LC has higher efficiency than Odimont as it has an additional drug. Both the combinations are used in treatment of allergy , allergic rhinitis. It can be taken after a meal. If you need more information feel free to mail me. Take care"
},
{
"id": 106292,
"tgt": "Having bump under my tongue without pain",
"src": "Patient: hi i have a bump under my tongue no pain i went to the doctor he said i had the flu i feel like im sleepy sore throat feels like its going away sneezing all the time and it looks like there was more then one Doctor: take a course of roxythromycin 150mg twice a day before food for 5 days and feelthe difference"
},
{
"id": 5024,
"tgt": "Delayed periods along with severe cramps, aching legs, mild headaches and pain in abdomen. Not on contraception. Possible pregnancy?",
"src": "Patient: Hi there,The start of my last period was 19th June, u haven't had a period since.... However I have been having period crams like I'm going to start bleeding anytime but nothing Iv had aching legs, mild headaches and mild pains from lower left to right abdominal Iv been TTC for 9 months now and have never been on any contraception Could I be pregnant ? Doctor: HI and thanks for this query.If you had an unprotected sex and you are not on any contraception, then the chances of being pregnant are real.From the information you have provided, the best thing to do at this time is to do a rapid pregnancy test. this will tell whether you are pregnant or not. This will greatly influence what further options should be pursued.Please, if you don't want to become pregnant now, you may consider adopting and using a contraceptive method. Also, condoms protect against STDs. Please, talk to your doctor if it is something you would consider.Hope you find this helpful and informative. Ask us more questions if they do arise. We shall gladly answer them.I wish you the best of outcomes with this.Dr. Ditah, MD"
},
{
"id": 148362,
"tgt": "Will hydrocephalus go away on its own?",
"src": "Patient: A couple days ago I had a cat scan. It said on the results some Encephalomalacia in the Occipital region Bilateral. What is that could it be bad or get worse and how can it affect me. I think I could of got it from the hydrocephalus I was born with don't know if it was there when I was younger. Also can Hydrocephalus go away on its own? It said the shunt was still intact without fracture but its also been 13 years without problem from that. Doctor: Hi,Thank you for posting your query.First of all, I would like to reassure you that your CT findings are not something serious and there is no need to worry about them.Encephalomalacia represents an are of damage to brain, which has happened a long time back. Possibly due to your old surgery of shunt placement.Yes, your hydrocephalus has got cured, as per the latest CT scan report.I hope my answer helps. Please get back if you have any follow up queries or if you require any additional information.Wishing you good health,Dr Sudhir Kumar MD (Internal Medicine), DM (Neurology)Senior Consultant NeurologistApollo Hospitals, Hyderabad, IndiaClick on this link to ask me a DIRECT QUERY: http://bit.ly/Dr-Sudhir-kumarMy BLOG: http://bestneurodoctor.blogspot.in"
},
{
"id": 172524,
"tgt": "How to treat diarrhea in a 1 1/2 years old child?",
"src": "Patient: My daughter is 1 1/2 year old and had been having diarrhea for 5 days now, mucoid, greenish stool and based on the stool exam, there is the presence of WBC 2+. She had been taking Metronidazole for 5 days now and Zinc supplement. What might be the problem? Doctor: Bacterial dysentry is the common cause of infective diarrhoea not amoebiasis. WBC of 2+ does not denote an infection and does not require treatment. Change the diet to a reduced sugar and milk diet and continue the zinc supplement.Yogurt and banana and rice is good for tummy and should be given,ORS can be added if there is dehydration and child not feeding well"
},
{
"id": 93409,
"tgt": "Pain in the right side of the abdomen, dribbling after urination, headaches. Reason?",
"src": "Patient: Hi, im a 21 year old female and I have been experiencing a sharp pain in my right side,feels like its coming from my ovary, not all the time but when I poo and if I sit in a funny position.also I have been having bad stomach pains (like period pains, in the same area) but I haven't had a period this month? Also when I urinate, after I've finished and left the bathroom I constantly dribble like I've stopped urinating before I've actually finished? Then when I go back to the toilet and go irinate again, it's like I haven't urinated for a while and I still get loads that comes out? Also I have been having really bad headaches and feeling quite sleepy throughout the day, and feeling abit sick every so often? I hope this makes sence to you, and hope you can help me, thanks Katy Doctor: urinary tract infection which is very common and causes theseit might be associated with gastritis and gerd causing headache nausea vomitingstake norflox 400 mg bd for 2 wkcoverage for uti and digestive system will be theretake citralka 1 tsf bd it will cover as local antiseptic for urinary tracttake gelusil 2 tsf tdsmeftal spas sos for paintake lot of warm water keep candid vaginal passery at night in vagina for 5 nights as fungal usually develops in these cases"
},
{
"id": 155908,
"tgt": "What causes adenoma cancer?",
"src": "Patient: sir, my sigmoid colon operation was made before 7 year ago dignose was adenoma cancer and doctor was make a colonstomy at my stomache and say it is temperory colonstomy still contain at my stomache doctor say i was 1984 seminoma was and rediation for testical operated. please help me what can i do i am 65 years old Doctor: there is no matter if oyou had seminoma cancer. if there is no sign of cancer in bowel then colostomy should be occluded. ask your doctor about it. it should have been done much earlier. but you need to repeat colonoscopy again to verify any other malignant diseases in bowel.wish you good health. Regards"
},
{
"id": 90663,
"tgt": "What causes sharp pain in lower right abdomen?",
"src": "Patient: The two mornings, i haven woken up at four o'clock in the morning and cannot move. My lower right abdomen feels like it is being squeezed. It hurts more when I lean forward to sit or get up. And hurts even more when i get up from lying down. Three nights ago, I had a slight fever and believed i was catching something, but have not gotten sick. The fever went away, but i still feel nauseous. The pain occurs about an inch to an inch and a half from my hip on my right side. It feels slightly bloated, but hurts if i press on it. Doctor: Hi welcome to HCMYour symptoms are suggestive of ureteric colic.A stone or an infection in urinary tract causes thisTake plenty of water and get a urine routine microscopy and an ultrasound abdomen with pelvis done Thank you"
},
{
"id": 105100,
"tgt": "Shortness of breath, chest and body pain, purple marks on arms after prolonged bathroom cleaning using cleaning products. Suggestions?",
"src": "Patient: Hi, last night I was cleaning my bathroom and I exaggerated on the cleaning products. I mixed them too much. I inhaled the cleaning products fumes which I almost fainted. I. Finish cleaning the. Bathroom. Now I have shortness of breath. My chest hurts and my whole body. The stomach also hurts. Sorry besides that I also have purple marks on my arms it looks like someone punch me in the arms too many times. I touch them and it hurts. Doctor: hi o.k do,n,t worry now you left that place.respiratory r become normal after leave out of effect means 48 hrs.you have patechial heamarhage.m/b its due to overuse of limb.if it faded after 4 days than o.k.other wise you will take aopinion of physician .after seeing he can tells what happanned. bye dr neeraj gupta"
},
{
"id": 185539,
"tgt": "What is the small red sore on the gum?",
"src": "Patient: I have had a small red sore on my gum in the middle going toward the back of my mouth on the right side on and off now since July. I have used the Orajel antiseptic healing rinse. It went away. I stopped using the rinse and it came back. I began brushing my teeth more and flossing regularly and it went away again. I did fall short of my oral hygeine practices and it came back. I went to the dentist. He did X-rays for other pain I was having on the other side of my mouth (upper). I showed him the sore and he said it did not look serious and prescribed me some amoxicillin for possible infection. He told me to brush the area really good and floss. I started to brush really well again and floss and it went away. I did not take the antibiotics right away. Well now it has come back and I am taking the antibiotics, although I took half the bottle and stopped and now I am taking the rest, so it may not be effective. Ultimately, I am seeking professional advice. Does this sound like something serious? If not what is your conclusion? Desperate to heal Doctor: Hello, Read your query, as you have small sore dont worry it is not as such serious condition this red sore can be due to inflammation in gums due to deposition of food debris this condition is known as gingivitis or inflammation of periodontal ligaments that is periodontitis which leads to Periodontal Abscess . For this you can apply ointment Gum paint on sore and do betadine mouthrinse , take proper nutritious diet , you can consult with local doctor and take Vitamin C supplements with folic acid. Do luke warm saline gargle two - three times a day. Consult dentist and go for oral prophylaxis and currettage for healthy gums .Hope it helps you .If you have further query I will be happy to help. Regards, Dr. Priyanka tiwari"
},
{
"id": 206379,
"tgt": "Suggest therapy for anger and violent behavior",
"src": "Patient: im suffering with problem that i cant control my angry ,im behaving like animal,i dnt knw what im doing at tht time.......my brain want to take revenge on some if anybody insult or any cheat immediately i behave like anything i cry like anything if anybdy insult me. plz wt is my disease ....i would like to commit suicide if i ll be like dis.........wt shall i do ? Doctor: Hello It appears that you have symptoms like anger outbursts, violent behaviour, extreme sensitivity to rejection and insult, revengeful attitude etc. Even on minor insult you want to take revenge or cry excessively. You want to kill yourself. Most probably your complaints are due to some personality problem. Other possibility is Impulse control disorder. Severe anger spells are seen in both disorders. Antisocial personality, Narcissistic personality etc disorders may present with revengeful attitude. Consult a good psychiatrist for treatment, don't feel bad about yourself. Mood stabilisers like Sodium Valproate, Carbamazepine etc drugs can be used to control anger outbursts and to stabilise the mood of patients. Try anger management sessions from some good psychologist to decrease anger.Thanks, hope this helps you"
},
{
"id": 110738,
"tgt": "Suggest remedy for back pain in smokers",
"src": "Patient: hi, Recently, for about 1 or 2 weeks, i feel a pain in the upper mid section of my back. I am a smoker and while i inhale cigarettes, i feel the pain gets deeper. I do not feel it in my chest, It does not feel like its the lungs, I do not have problems breathing, or any other side effect either. No sweating or numbness either. I have been experiencing pain through all my back lateley and it feels like if I had done a wrong movement. Should I be worried?, I am afraid of something serious. Doctor: Hello, I have studied your case. Due to compression of nerve root there can be pain in your back and chest.Due to occlusion of vessels there can be burning sensation.This can be related to smoking also.I will advise you to do MRI thoracic spine, HRCT [CHEST] and EMG- NCV [nerve conduction study]For these symptoms analgesic and neurotropic medication can be started.Till time, avoid lifting weights, Sit with support to back. You can consult physiotherapist for help.Physiotherapy like ultrasound and interferential therapy will give quick relief.I will advise to check your vit B12 and vit D3 level.Hope this answers your query. If you have additional questions or follow up queries then please do not hesitate in writing to us. I will be happy to answer your queries. Wishing you good health.Take care."
},
{
"id": 74526,
"tgt": "Suggest treatment for chronic constipation and chest pain",
"src": "Patient: my daughter is 12 suffering from chronic constipation chest pains facial swelling and occasionally spits blood and her doctor has no idea what the problem is oh and i cant forget the varices in her throat that was discovered when she was scoped three weeks ago Doctor: Respected user , HiThanks for using Healthcaremagic.comI have evaluated your query thoroughly .* Management of both are dependent upon the cause of the same .* Diagnosis is the prime concern for that .* General measures for the same to get better recovery - Allow her plenty of liquids .- High fiber diet .- Soft , light diet .- Avoidance of oily , spicy , non veg stuffs , beverages .- Remain ambulatory with regular walking , exercises , YOGA .Hope to clarify your query Welcome for further assistance Thanks for using Healthcaremagic.com & giving opportunity to assistWishing fine recovery to your Angel .Regards ."
},
{
"id": 23880,
"tgt": "Suggest remedy and tests to check and control high b.p?",
"src": "Patient: I am hypertensive 140/90,160/100,160/110, 136/82 ,150/100 are the readings within which my B.P will be always. I have also got tested echo,ECG,Renal artery doppler all are normal. Sugar & cholestrol are within normal ranges. I am taking daily Losar-A(morning & night- 1 tablet each) Nebistar -H( 1 in afternoon) & Minipress-XL 2.5mg ( 1 tab in night along with Losar-A, as stated already). I am 38 yrs old. My height is 5.5\" and weight is 98 kgs. Please advice me further tests or change inmedication to control B.P Doctor: Hi,You need treatment correction for a better control of blood pressure.I would advise you to increase the dose of Minipress-XL from 2.5 mg to 5 mg.If blood pressure is still high in 10 days, then further corrections are needed.Take careCome back if you have any further questions"
},
{
"id": 168488,
"tgt": "Does recurring vaginal discharge in infants need medical attention?",
"src": "Patient: This morning when I changed my 9 month old baby girl, I noticed that she had a thick, white, vaginal discharge in her diaper. There was a lot of it but there was not rash or signs of irritation. This has happened once before, 2 months ago. Is it anything I should worry about? Doctor: dear madam, it is so nice of you to take good care of your baby.vaginal discharge in infants can occur if the mother is taking harmones and she is breastfeeding.just make sure baby food or supplements dont contain oestrogenic components.have a good day"
},
{
"id": 94154,
"tgt": "Pain in the chest and stomach while breathing. Need to be checked?",
"src": "Patient: hello there, i was just wondering weather you could help me. I have pains in my chest and stomach when im breathing , i have pains in my right&left ribs and pains between my shoulder blades, whilst i press between my ribs i have a really sharp pains and where my appendix is, its just those two places that when i press or knock it really hurts, i was just wondering weather this is something i should have to go to my doctors about? could you give me a clue to whats wrong with me many thanks anna Doctor: Hi welcome to Health care magic forum. Thanks for calling H.C.M.F. You can have full faith upon me regarding the answer. You have pain in rt and lt ribs, Between the chest and stomach.There is pain between the shoulder blades, You have sharp pain between the ribs when you press, that is called tenderness ,and it suggests the location of the problem at that place. Appendex is an organ present at right lower place of the stomach, so don't worry about appendicitis. Your problem appears to be peptic ulcer, gastritis , gall bladder pain, liver problem , or pancriatitis. I advise you to consult a gastro enterologist, for diagnosis and treatment. You may need to have GASTROSCOPY, C.T.scan of whole abdomen besides other routine tests for confermation. Wishing for a quick recovery. You can have your doubts cleared if necessory. Best regards."
},
{
"id": 143351,
"tgt": "How to get rid of static sensations in head?",
"src": "Patient: Hello I am having static sensations in my head and is very annoying hearing these sounds. What can I do to get rid of this? Last week on my job was hard for me to use headsets. Every time I put the headsets on, the static got worse. The bad part about it my job requires me to use them. Doctor: Hello!Welcome on HCM!Regarding your concern, I would explain that your symptoms could be related to an inner ear disorder. Other possible causes would be: - a brain lesion like a tumor, infection, etc.- thyroid dysfunction - chronic anemia. For this reason, I would recommend consulting with an ENT specialist to exclude an inner ear disorder. Further tests may be needed if an inner ear disorder is excluded: - a brain MRI- thyroid hormone levels- complete blood count- PCR, sedimentation rate. Hope to have been helpful!Kind regards, Dr. Aida"
},
{
"id": 87784,
"tgt": "What causes rapid breathing & stomach pain?",
"src": "Patient: What do I do I might have OD on vyvanse,my usual dosage is 30 mgs and I took 3 capsules equalling 90 mgs. I m a 15 year old female,I m about 132.9 lbs,I m 5 5. I looked up symptoms and the only ones I ve had a little rapid breathing(calmed myself that s gone) and still currently a little stomach pain,nothing major. I m just worried and would like your opionion. Doctor: Hi! Good evening. I am Dr Shareef answering your query. At the outset I would suggest you to be careful on the way you handle your drugs. What ever little side effect a drug has, might compound itself when you take a over dosage of this. Even though you did not have any serious side effect till now, I would advise you to consult your family physician or the doctor in the ER of your nearest hospital for a proper physical examination with related investigations if need be of your respiratory and cardiovascular systems as a precaution.I hope this information would help you in discussing with your family physician/treating doctor in further management of your problem. Please do not hesitate to ask in case of any further doubts.Thanks for choosing health care magic to clear doubts on your health problems. I wish you an early recovery. Dr Shareef."
},
{
"id": 123606,
"tgt": "Suggest treatment to reduce pain in my shin muscle",
"src": "Patient: my shin muscles are paining n getting tightened if i do regular walking, im doing stretches n walking though its not helping giving icemassage evernight to releie pain its not working any special stretches or treatment for my shin muscle to walk longer distance without discomfort? my age is 30 male Doctor: Hello, I think you these are related to your shoe or footwear and the medium u walk. Correction of gait and also having flexible footwear should solve your problem. Your endurance levels look good by history only thing is troubling which I see is the footwear. Try walking barefoot and see if the pain comes. If pain doesn't come then your footwear has issues. And if comes then the foot muscles need some exercises. Hope I have answered your query. Let me know if I can assist you further. Regards, Jay Indravadan Patel, Physical Therapist or Physiotherapist"
},
{
"id": 134954,
"tgt": "Suggest remedy for pain in foot",
"src": "Patient: Hi Doctor, My job requires a lot of walking. While I am walking I don t get any pain. I start getting a sharp pain un top of my foot is like a sharp pain and I only getting it when I am relaxing. My question is why am getting this pain on my right foot (un top) very sharp pain when am relaxing? Doctor: HIWell come to HCMI really appreciate your concern, it is the muscular condition may be deposition of lactic acid and second it could be due to improper gate, drink soda water, have some rest, for the pain \"Tab Nimesulide 100\" mg can be tried else this is nothing to worry hope this information helps."
},
{
"id": 19751,
"tgt": "Suggest treatment for pain around carotid artery, under jaw radiating to ear/shoulders",
"src": "Patient: hi, I have always had pain around my right side carotid artery, under my jaw, radiate to my ear, and my shoulders. Sometime i feel it is on the side of my throat. It is wearing me out, my memory is geting weaker and weaker, sometime i feel as if everything blocked as it affect my proper breathing and i dont feel i am relaxed...i am 49 by the way and i had this for few years now ... but it is geting worse and causing me to be rigid and stiff... can u tell me plz what i should do and what could be the diagnosis and what kind of doctors i should visit... thx.. MK Doctor: Hello MK!I passed carefully through your question and would explain that your symptoms could be related to a temporo-mandibulary joint dysfunction or a bulging disc in the cervical vertebral column, causing radicular nerve pain in this region. For this reason, I would recommend performing a cervical spine MRI study. If this test results normal, I would recommend consulting with the ENT specialist for a physical exam of the temporo-mandibulary junction. Anxiety could also play an important role on all this clinical scenario, too. Hope you will find this answer helpful!Kind regards, Dr. Iliri"
},
{
"id": 225838,
"tgt": "Missed taking loette pills. Bleeding started. Can I take primolut?",
"src": "Patient: I accidentally missed taking a Loette pill a few nights ago, then started bleeding, so am taking Primolut to make it stop. Is it ok to keep taking my Loette pills at the same time as taking Primolut 5mg? Or should I cease taking the Loette until the end of this cycle? I still have 10 active Loette pills left in the tray. Thanks in advance. Doctor: hi.. thanks for using HCM..you are taking oral contraceptive pill.. missed taking the tablets..you have not mentioned the reason for taking the tablet?if it is for contraceptive?... wait for 5 days for bleeding to occur, then start with the new pack of pills from 5th day onwards..primolut will not help to stop bleeding..all the very best.."
},
{
"id": 198215,
"tgt": "What could cause swollen gland between leg and groin with yeast infection on penis?",
"src": "Patient: help I have a swollen gland its between my right leg and groin, also I have a yeast infection on my penis that I have been fighting for a year, I have been to many docs, I have been tested for every STD every couple of months, I'm really worried because the swollen glad just happened a couple days ago help Doctor: Hi,It seems that due to itching because of fungal infection there might be having some bacterial infection producing tender, enlarged inguinal lymph nodes.Go for one course of antibiotic medicine for 3-5 days.Aply anti-fungal cream locally.Take Benadryl or Cetrizine for itching.Keep local hygiene clean, dry and airy.Ok and take care."
},
{
"id": 53476,
"tgt": "Suggest treatment for liver tumor",
"src": "Patient: Doctor my mother 72 years old she have sugar & Bp 6 months before one stone in lever we removed from expert Dr.Now she was not sleeping last five days, i shown one Dr he checked he told something growth in right side liver to food wein size is 8.23 cm what is the rectification for this problem Doctor: Hi and welcome to Healthcaremagic. Thank you for your query. I am Dr. Rommstein, I understand your concerns and I will try to help you as much as I can.First, you need to see if this is benign or malignant tumor and you should do mri scan and ercp for this then further therapy should be planned which include chemotherapy, surgery or other ablative solutions.I hope I have answered you query. If you have any further questions you can contact us in every time.Kindly regards. Wish you a good health."
},
{
"id": 36003,
"tgt": "Suggest treatment for infection in intestine",
"src": "Patient: my mom had a colonoscopy and the findings were melanoma coli he removed several polyps but some of his writing i cant read. he said come back in two years but my mom has gotten 3 infections in two months. he didnt tell her anything else, about the bad part of her intestine he said that wasnt his job. she is going back to the doctor with the paper he gave her. i dont understand why he didnt tell he about the bad part now what? Doctor: DO NOT WORRY.THIS IS NOT AN INFECTION.IT IS CAUSED BY EXCESSIVE USE OF LAXATIVES E.g; SENNA FOR A LONG PERIOD.IT IS JUST BLACK COLORATION OF INTESTINES.IT IS A BENIGN FORM,NOT TRUE MELNOMA.DO NOT TAKE LAXATIVE WITHOUT DISCUSSING YOUR DOCTOR.IF YOU HAVE ANY QUERY ASK AGAIN.WE ARE HAPPY TO HELP YOU.FEEDBACK TO HCM."
},
{
"id": 48478,
"tgt": "Suggest medicine for cortical calcification in kidney",
"src": "Patient: my ultrasound showed The right kidney measures 10.9x4.8x4.6 cm with a cortical Thickness of 1.4cm while the left kidney measures 11.4 x 5.0 x 6.1 cm with cortical thickness of 1.4 cm. Both kidneys are normal in size and corticomedullary echopattern. A 0.4 cm cortical calcification is now noted in the lower left kidney. There is no evidence of calyceal lithiasis or hydronephrosis. No focal masses noted. The perinephric areas are clear. What is cortical calcification? what is the medicine for this...? Doctor: Hellonothing to worry,it means there is slight deposition of calcium inside your kidney tubules but that can occur normakly in many people with age so u should not be worried"
},
{
"id": 85946,
"tgt": "What does terrible pain in the lower right abdomen indicate?",
"src": "Patient: yes i was diagnosed with diverticulosis 7 yrs ago ? i am 6o one yrs old and have had terrible trouble with my stomach the past 2 wks ? when i go i go when i don t i don t alternates ? i have been having terrible pain in rt side and lowere stomach could this br an attack ? Doctor: Hello and Welcome to \u2018Ask A Doctor\u2019 service. I have reviewed your query and here is my advice. You can take Ofloxacin and Ornidazole tab twice a day for 5 days. You can take Tramadol and Paracetamol tab twice a day for 5 days. You can take Omeprazole and Domperidone tab twice a day for 5 days. Don't eat outside food, commercial products, stale food, eat home made rice and khichhdi, drink more amount of water, buttermilk, juiceetc. Do ultrasound if not resolved. Consult the gastroenterologist. Hope I have answered your query. Let me know if I can assist you further."
},
{
"id": 55516,
"tgt": "Suggest remedy for itchiness due to high bilirubin levels",
"src": "Patient: Hello,I was on INH therapy to prevent TB, three weeks ago I had upset stomach, skin itchy, acid reflux, nausea. Almost two weeks ago I was diagnosed with jaundice. My liver enzymes initially were between 2,000-3,000. Now they are less than 1000. My Bilirubin level stayed at 12.5 for the first week and now it is at 12. My doctor ruled out Hepatitis A,B,C, autoimmune disorders and my INR has been normal. My question is how long will it take for the bilirubin levels to decrease more, this itchiness thing is driving me nuts! I can live with yellow eyes and skin but 50mg of Benadryl at night is not working. Doctor: Hello! Thanks for putting your query in HCM. I am a Gastroenterologist (DM). presence of itching with jaundice is either obstructive jaundice or sometimes in cholestatic jaundice. So I will suggest you to get an ultrasound abdomen also to see for any obstruction of the billiary tract. In present scenario it may be due to isoniazid induced. So stop it. It may take around 4 weeks for bilirubin to be normalised. For itching I will suggest you you take tablet ursocol 300mg thric a day and tablet avil twice a dayI hope I have answered your query and this will help you. If you have any further query I will be happy to answer that too. Remain in touch and get-well soon."
},
{
"id": 13886,
"tgt": "How to treat rash with slight pain and itching on leg/shin?",
"src": "Patient: hi im 46 i have a rash on my shin bone started on my l leg now on my r its only on my shin bone went away on l leg i used cortizone cream and benadryl i have lil bruising no pain.slight itching rash went away in 3 days any suggestions Doctor: Hi, Itchy rash on leg could be due to variety of causes. But in my opinion, the first condition to be ruled out is an infection. So, I advise you to consult your Dermatologist to rule out the causes and to come to a diagnosis. Hope I have answered your query. Let me know if I can assist you further."
},
{
"id": 72092,
"tgt": "Suggest remedy for chest pain",
"src": "Patient: Hi. I am a 31 y.o. female, 50 lbs overweight. I have hypothyroidism and high cholesterol. I have been havign mild chest pain on and off for a month or so, but didn t think anything of it until a week ago (on a Monday). It was severe enough that I couldn t concentrate and accompanied by nausea. It starts at my breast bone and curves upward towards my right shoulder. It feels like a tube of pain and also radiates into my back between my spine and rt shoulder blade. It also feels as if something is pushing on my windpipe at the bottom of my neck. I went to the Dr. same day and he was sure it was a gallstone attack. I went in on friday for an Upper GI and Ultrasound. The Upper GI was normal and the ultrasound showed no gallstones, though they did see a few benign spots on my liver that they are recommending that I have looked at via MRI. I am assuming this means that it is not my gallbladder causing the problem then. However from Thursday through Saturday I had yellow diarrhea (it s now Monday again) and thought the chest pain has lessened it is still there. It doesn t get worse or better with position, exercise or eating, though it is a little more uncomfortable with deep breaths. I am feeling increasingly tired. My Dr s appt is tomorrow for follow-up, but the fatigue has me a little worried. I also belch excessively but have read that this may be a side-effect of the Lovaza I am on for my cholesterol. What do you think could be causing the chest pain? Doctor: Thanks for your question on Healthcare Magic.I can understand your concern. Since you are overweight, we should first rule out cardiac chest pain. So get done ecg, 2d echo and stress test (trade mill test)If all these are normal then no need to worry for heart diseases. Sometimes, undiagnosed stress and anxiety can also cause similar symptoms. So better to consult psychiatrist and get done counselling sessions. Try to identify stressor in your life and start working on its solution. You may need anxiolytic drugs too. Don't worry, you will be alright with all these.Avoid stress and tension, be relax and calm. Hope I have solved your query. I will be happy to help you further. Wish you good health. Thanks."
},
{
"id": 145036,
"tgt": "What causes dizziness,tingling in fingers with burning sensation on feet sole?",
"src": "Patient: I have been experiencing dizziness, tingling in my fingers and toes and tingling with a burning sensation on the bottom of my feet and tingling of my tongue. I was placed on meclazine which makes me feel groggy and out of myself. What can I do? Thank you. Doctor: Hello dear, As per your narration it seems that you are having peripheral neuropathy like symptoms. It can occur due to many reasons like diabetes, vitamin deficiency, vasculitic, toxin, drugs, immune mediated or idiopathic. You need a proper evaluation and investigation. You need to consult a neurologist. Some investigation like nerve conduction study, blood investigation and if needed nerve biopsy may be done. As for treatment, in many cases symptoms improve and recovery occurs but prognosis depends upon cause. Hope you found the answer helpful. Wishing you good health. Dr Neeraj Kumar Neurologist"
},
{
"id": 39845,
"tgt": "How to treat yeast infection?",
"src": "Patient: Okay so I thought I may be getting a UTI and so they prescribed me septra and had some sort of a recaction so now right as you would insert A tampon there is a sore and I have a serious yeast infection and there is swelling by my butt crack and it itches my email address is YYYY@YYYY Doctor: Hello,Welcome to HCM,The symptoms and the history are suggesting me that you are having fungal infection. the fungus are the normal comensals of the genitals which are kept under control by helpful organisms when ever there is a imbalance in the environment these fungus will activate and produces the symptoms of fungal infection.As you are having the symptoms of the fungal infection you have to follow1.Good hygiene2.Keeping that area clean and dry3.Topical Antifungal cream like Ketaconazole 4.Oral Antihistamine.Thank you."
},
{
"id": 97699,
"tgt": "What home treatments can a patient taking chiropractic treatment in case of tremors do?",
"src": "Patient: I am having full body tremors and have been going to a chiropractor and it has been helping a lot but I am currently on vacation out of state and having issues with my tremors again and was wondering if you had any home treatments to help calm them down Doctor: **1. for treatment purpose one has to look CLINICALLY for the presence of any static tremor [tremors at rest] OR kinetic tremor [action tremors] which will give an insight of the cause of tremor [since no thyroid report, brain disease, or addiction is mentioned in the history]2. commonest cause of tremors in clinical practice are physiological tremors due to anxiety, fear or nervousness and alternative treatment will be directed to calm them down.. Avoid: excess physical \u2013mental exertion, sun exposure, corrylium, smoking, errhine therapy, betel leaves, gruel, soup, day time sleeping, garlic, ginger, onion, spicy-heavy- junk food, mustard oil, pickles.. Take: mint, coriander, tomato, cumin, plenty of water, vitamins and mineral supplements, apple, banana,grapes,dates. Prescription drugs available are: Shrikhand Assav, mahakalyanak avleha, ashtang lavan, kharjooradi manth,smritisagar ras,trifla churan,brihat vata chintamani ras,ghritkumari,ashwagandha,brahmi,jatamansi, stresscom.. You can practice: bhujangasana,halasana, paschimottasana, sarvangasana, shalabhasana, shavasana and vakrasana and pranayamas like bhastrika and kapalbhati.. Lemon juice mixed with sugar in a glass of water helps release stress removing hormones from the body and bring mind at peace/rest.\u00a0\u00a0\u00a0\u00a0\u00a0i.\u00a0\u00a0\u00a0\u00a0\u00a0Take 1 spoon black pepper powder and 5 spoons jaggery in 4 cup water and boil till 2 cups remain, filter it and drink for 1 month.ii.\u00a0\u00a0\u00a0\u00a0\u00a0Take 3 teaspoons of bitter gourd with a glass of water every morning for a month.iii.\u00a0\u00a0\u00a0\u00a0\u00a0Mix 4-5 dates in half glass of water and drink 2 times daily for a month.iv.\u00a0\u00a0\u00a0\u00a0\u00a0Regular intake of Apple.v.\u00a0\u00a0\u00a0\u00a0\u00a0Apple, almond, banana, broccoli, cantaloupe, melon, carrot, cashew nut, corn, cauliflower, grapes, guava, lemon, mango, orange, pineapple, soybean, spinach have anti oxidant and detoxifying properties so take them in plenty."
},
{
"id": 53699,
"tgt": "Suggest treatment for elevated liver enzymes and edema of legs",
"src": "Patient: Hi Docs my dads is a non alcoholic and is positive for hep-b. His SGOT is 90, SGPT is 75, Total bilirubin is 5.7, direct bilirubin 3.4; he is passing dark colored urine once in a day and othertimes it is normal colour. Has odema of legs on and off. Can u please suggest what we treatment he needs. Thank u very much in anticipation. Doctor: Hello,1) Dark color of urine is due to high bilirubin (Jaundice)2) Edema of legs suggest less amount of protein in body because of less production by liver. This also cause collection of fluid in stomach (ascites)3) Status of hepatits-B positive + jaundice + less protein suggest possibility of cirrhosis of liver. This condition is irreversible and only option for possible cure is liver transplantation4) Till liver transplantation he need some medicines to support liver function and to slow the progression of disease.5) Also he need some investigations for evaluation of extent of disease (CT scan abdomen, upper GI endoscopy)6) I suggest to consult medical gastroenterologist or hepatologist for proper evaluation and staging of disease so that appropriate treatment will be started."
},
{
"id": 17059,
"tgt": "What causes heart palpitations and chest pain?",
"src": "Patient: Hi, may I answer your health queries right now ? Please type your query here... \u00a0\u00a0\u00a0\u00a0\u00a0\u00a0\u00a0\u00a0\u00a0\u00a0\u00a0\u00a0\u00a0\u00a0\u00a0\u00a0\u00a0\u00a0\u00a0\u00a0\u00a0\u00a0\u00a0\u00a0\u00a0\u00a0\u00a0\u00a0\u00a0\u00a0\u00a0\u00a0\u00a0\u00a0\u00a0\u00a0\u00a0\u00a0\u00a0\u00a0\u00a0\u00a0\u00a0\u00a0\u00a0\u00a0\u00a0\u00a0\u00a0\u00a0hi i keep getting pains in my chest and sometime it feels like my heart beats an extra beat and some time it feels like it misses a beat and i feel fainty when it happen what can this be? thanks Doctor: Hi, Any type of chest pain should never be underestimated and should be evaluated in detail.Missed heartbeat could be because of ectopics (extra beats from a part other than the normal conducting system of the heart).You should soon see a cardiologist and get an ECG and 2D echo. Hope I have answered your query. Let me know if I can assist you further. Regards, Dr. Sameer Maheshwari, Cardiologist"
},
{
"id": 152016,
"tgt": "Head Throbbing No Pain,should I go to the E.R ?",
"src": "Patient: Hello, I am having a throbbing vein in right side of my tempal in my head but no pain, this is scary, being doing this the last couple of days. Could this be stress or should I go to the er? Doctor: Thanks for the query It is normal for the vessel on the temporal region to throb. If u r not having any symptoms then ignore it. U dont have to worry abut it Have a healthy living"
},
{
"id": 160201,
"tgt": "Painless lump for years",
"src": "Patient: painless, mobile lump on shoulder for years Doctor: Hi, You might have lipoma on shoulder which is benign i.e. non cancerous. If it is in connection with shoulder joint you consult orthopedic surgeon for diagnosis and treatment."
},
{
"id": 82406,
"tgt": "Suggest treatment for shortness of breath and burning chest",
"src": "Patient: I cleaned two days ago with bleach in all the rooms of my house, last night and today i have had burning in my chest, i cant smell anything and shortness of breath, sometimes it makes me feel as if im going to pass out. Is this something i need to go to the er for to get checked out? Doctor: HelloAll these symptoms , chocking ( shortness of breath ) and burning sensation are due to irritant nature of ammonia produced by bleaching .Since two days ago you cleaned your house and still symptoms present so take some steroid inhalation 2 puffs 2-3 times and stop . This will relieve congestion caused by bleach.Burning of chest will also be relieved by this but if you feel burning in chest then take antacid ( containing oxetacaine ) and this will provide complete relief.In my opinion this is not case of E R just annoying ."
},
{
"id": 76023,
"tgt": "Is fuzzy spot on chest x ray dangerous?",
"src": "Patient: i took a medical exam which included a chest xray and i noticed a small blur on the screen. The technician said if its something the doctor will let you know. Well now i have to redo the medical test for three consecutive days. Can you help ease my worries and tell me a reason why i would have to go for three consecutive days? Doctor: Thanks for your question on Healthcare Magic. I can understand your concern. Fussy spot on chest x ray is commonly seen with 1. Lung infection like tuberculosis or pneumonia 2. Old fibrotic lesions 3. Lung cyst 4. Malignant nodule. To rule out tuberculosis and other lung infection, sputum sample is needed. And specifically for tuberculosis, sputum should be given for consecutive three days. This is standard protocol for the diagnosis of tuberculosis. So get done three sputum sample reports. Also consult pulmonologist for further evaluation of this spot. Hope I have solved your query. I will be happy to help you further. Wish you good health. Thanks."
},
{
"id": 6522,
"tgt": "Can tablet prescribed to get regular periods ensure regularity as I trying for pregnancy ?",
"src": "Patient: I had irregular periods till feb 2011 i used tablets to get my periods as per the doc advice Doc advised me not to take any more tablets due to my health From then i got the periods almost properly 38days 35days 30days 33days July 3rd was my LMP Due you think my periods are regular now I am trying to get pregnant Nothing good has happened till now Any advice I had irregular periods, till feb 2011. I used tablets to get my periods as per the doc advice. Doc advised me not to take any more tablets due to my health. From then i got the periods almost properly. 38days, 35days, 30days, 33days. July 3rd was my LMP. Due you think my periods are regular now. I am trying to get pregnant. Nothing good has happened till now. Any advice? Doctor: Welcome to Healthcare Magic Your periods still appear to be irregular. Once they regularise by the birth control pills you can think of getting pregnant. Have you been tested for PCOD and other hormonal disturbances like thyroid problem, high prolactin hormone as these can also cause irregular periods. Get a thorough checkup by your Doctor for the causes and get appropriately treated."
},
{
"id": 215113,
"tgt": "Problem of delayed menses",
"src": "Patient: i have a problem of delayed menses . i want 2 know any home treatment . Doctor: Delayed periods can be due to stress, tension. You can take more of papaya fruit to get back the periods. It is occuring for many times than see a gynecologist"
},
{
"id": 170656,
"tgt": "Could splotchy raised rash on cheeks, chin and palm be due to antibiotics for ear infection?",
"src": "Patient: My 19 month old has a splotchy raised rash on his cheeks, chin, palms, arms, belly, and feet. He has been on an antibiotic for an ear infection for the last ten days (sulfamethaxone?). Could this be antibiotic related or is it something else? Thank you. Doctor: Hi, this appears to be a allergic reaction due to the drug that you are giving. I think you should stop the drug immediately and should give hydroxizine syrup 5 ml PO three times a day for 3 days. Take care."
},
{
"id": 193782,
"tgt": "Are there risks involved in masturbation?",
"src": "Patient: Hi..Iam Vinu..age 20..Iam here to ask about risks of masturbation..i have this habit for about 1 and half years..at the initial times it feels good..but recently iam feeling a little pain in my right testicle..however it disappears after 2 days...is it normal??...pls send me measures or steps to do...waiting for your kind advice.... Doctor: Hello,The excess masturbation that is frequency more than 2-3 times in week can cause groin discomfort.So you can have pain because of that. However if testis pain present than investigate with ultrasonography of testis for primary work up. Yoga, meditation, less porn can help in reducing frequency of masturbation. Make short term and long term study goals which will help in reducing frequency. Hope I have answered your question. Let me know if I can assist you further. Regards, Dr. Parth Goswami, General & Family Physician"
},
{
"id": 87098,
"tgt": "What causes continuous vomiting?",
"src": "Patient: My daughter came home from school and we went to music lessons, she was behaving normally. As we got in the car to come home, she suddenly felt ill. She thought maybe her stomach was just hungry, but no luck. Half way home she started vomiting, and continued to do so every 20 min. until about 3:30am. What's up? Doctor: Welcome to health care magic.continues vomiting may be due to gastritis. maintane dehydration by giving him some plenty of water.watch for signs of lethargy, dry tongue, urine output less , dehydration severe, might be going to hospital for admission.hope u have satisfy with this answered."
},
{
"id": 219629,
"tgt": "What does putting a bag on baby when it is born mean?",
"src": "Patient: im 33 weeks pregnant and i just lost my mom i smoked pot during that time i havent used it in about two weeks i got kicked out of my clinic and had to transfer to another . they will still monitor my urire and they said they were going to put a bag on the baby when she is born what does that mean and do you think ill be clean i was really stressed and thats the safest thing to use wile pregnant i know its not and axcuse but im just needing some help i dont want to loose my first baby Doctor: Putting a bag on the baby means they are going to do a urine drug screen on the baby. It could potentially affect your custody status if it is positive. The good news is you are only 33 weeks. Hopefully, you won't deliver in the next four weeks. Marijuana takes about 4 weeks to get out of your system. I implore you to stay clean for the remainder of your pregnancy. Good luck and I'm sorry about your mom."
},
{
"id": 140268,
"tgt": "Suggest treatment for tingling sensation in fingers",
"src": "Patient: I was recently told i have a retracted eardrum. I spend much of my day clenching and grinding due to ocd and anxiety. I initally went to the doctor because it felt like a bug in my ear and was dizzy. He found the retracted ear then. However, it has been a few days and now my finger tips are having shock like or prickly feelings in them. They are tingling as i type. Doctor: Hi, Retraction of the eardrum is usually a sign of some type of space-occupying lesion that may be causing the eardrum to pull inward a little more than normal. It can also be as as result of increased NEGATIVE PRESSURE within the middle ear since there is a connection between the place behind your ear drum and your EUSTACHIAN tube which opens up into your NASOpharynx which is literally the uppermost portion of the BACK of your mouth (high up). The ENTRANCE of that tube you see in the picture anatomically connects to the middle ear and here is a GREAT picture of that. See where that AUDITORY TUBE is in that picture? It connects with the opening into the NASOPHARYNX....that is the EUSTACHIAN TUBE which equalizes the pressure between the middle ear and the outside world (your mouth). Look at the 2nd picture and you'll see what looks like a big WHITE LAMP SHADE called the TYMPANUM or TYPMPANIC MEMBRANE. That's the eardrum. See where it's connected? And see how it naturally bows INWARD toward the space in that middle ear. Can you see where something in that space could conceivably cause that TYMPANUM to draw in a little more.....OR...what I believe is also a possibility is that the AUDITORY TUBE (Eustachian Tube) may not be working properly which could cause NEGATIVE pressure within the cavity. Things that cause Eustachian Tube dysfunction in that regard could be a mild infection with swelling of the middle ear lining, allergy reactions to things in the air during allergy season, or even pharyngitis which can cause the Eustachian to be blocked or swollen on the OTHER end where it opens into the nasopharynx. Generally speaking, these processes are all easily treatable either conservatively with good hydration, maybe some nebulizations with steam to open up the nasal passages a bit, etc. and the problem will resolve. Or if there is evidence for an actual infectious process, antibiotics, etc. However, the symptoms of the ear as you've described and especially if there is tympanic retraction does not really explain prickly feelings or shocklike symptoms in the fingers. Those may be due to a bit of a flare of your OCD and anxiety. You may be applying more force typing than you realize and should consider doing some deep breathing exercises and think in terms of letting the muscles in the hands and arms DECONTRACT or RELAX....That is going to go much farther for those sensations than anything else at this point. You may also NOT have eardrum retraction since that's not always an easy thing to see unless the doctor was an ENT specialist. Sometimes if the light source used is not powerful enough or if for some reason the patient has a slight anatomic variation of the external auditory canal where there could be an extra little curvature or different placement of the ear apparatus it could appear as if the tympanum had traction on it when in fact, it's really NOT retracted....best way to verify that would be to have a specialist look and see what's up. Hope I have answered your query. Let me know if I can assist you further. Regards, Dr. Dariush Saghafi, Neurologist"
},
{
"id": 179289,
"tgt": "What causes flea bites on the scrotum of a 4 year old?",
"src": "Patient: My four year old has two flea bites on his scrotum and says a playmate kicked him there today as well. I checked at 3:30PM, and there was no bruising. He said it hurt. At 9:PM my wife checked and found his scrotum to be heavily bruised and swollen, particularly on the left side. Also, what troubles me most, is that the scrotum skin is quite hard to the touch on that side now... to the point the testicle cannot clearly be felt. Should we head to the ER? We ve applied ice to some good effect. Doctor: Scrotal injury as described by you seems to be severe. This area is most sensitive part of the body, so the child must be in severe pain. Injury to testicles also need to be evaluated at the earliest. Do not delay. kindly take the child to ER immediately."
},
{
"id": 132868,
"tgt": "Suggest treatment for swollen rib cage",
"src": "Patient: Ig OK t into a physical fight with someone about five days ago and we went down to the ground and fell on a bicycle and the handle bar went into my rib cage right where my heart is the pain is worse at night and its pretty swollen on the area but there s no bruise that can be seen also when I run my hand across the swollen area it like feels crackly like liquid moving and maybe bubbles I feel the pain is really bad at times and instead of it getting better its staying painful I m sorry I thought this was free Doctor: Thanks for your question on Healthcare Magic. I can understand your concern. You had blunt chest trauma due to fighting and the cracking sound is mostly due to rib fracture. So better to get done chest x ray for the diagnosis of rib fracture. Follow these steps for better symptomatic relief in such cases. 1. Avoid movements causing pain. 2. Avoid bad postures in sleep. 3. Avoid heavyweight lifting and strenuous exercise. 4. Apply warm water pad and ice pack application on affected areas. 5. Take painkiller and anti inflammatory drugs. 6. Sit with proper back and chest support. Don't worry, you will be alright with all these in 1-2 weeks. Hope I have solved your query. I will be happy to help you further. Wish you good health. Thanks."
},
{
"id": 184528,
"tgt": "What causes a white spot inside the nostrils?",
"src": "Patient: I am a 58 year old woman who just visited the dentist. In his office he is using a CT scan to get 3D pictures of the jaw, neck, face, etc. On mine, he noticed a white spot just to the side of my right nostril. What could this possibly be? He said I needed a more specific scan to that area to get a better picture. would a deviated septum show up like that? Doctor: Hello, Read your query, as you have white spot inside nostril it can be due to folliculitis , nasal polyp, sinusitis, or deviated nasal septum for this I will suggest you to consult ENT specialist and go for investigations like PNS view for deviated septum and its treatment. Hope it will help you. Wishing you good health.Regards, Dr. Priyanka tiwari."
},
{
"id": 171458,
"tgt": "Suggest treatment for insomnia in a child",
"src": "Patient: My 8 year old step daughter has insomnia any suggestions? She is a perfect angel all day but when night time comes around she fights it says she isnt going to sleep comes in every hour and says she cant sleep. she is on adhd meds and also a sleeping pill at night. I hoestly dont think she is adhd at all i think she cant focus because she is so tired. I am at my wits end Doctor: hi.good day to you.sorry to hear about your step daughter.i think she needs a child psychiatry counselling.insomnia may be due to unknown deep fears or doubts she herself dont realise.try to avoid or minimise television or internet usage by her to less than 2 hours per day.encourage physical activity and outdoor activity.avoid junk food in diet.encourage plenty of fruits and vegetables.dont let her go empty stomach or too heavy meals at night.try regular bedtime activities.hope u got your answer.take care"
},
{
"id": 134427,
"tgt": "Suggest remedy for pain in lower legs and ankles",
"src": "Patient: I m trying to find out why my lower legs and ankles ache all the time. I have been to a podiatrist (sp) and have had orthotics made for both feet and I my legs and ankles still ache. I was told to take Aleve, but I don t want to keep taking pills all the time. Should I go to a orthopedic doctor and see if it is my bones. Please give me some ideas as to what I should do. Doctor: Hiis there history of trauma to your ankles or feet? are you always on your feet at work or at home?there are multiple conditions that can bring about your symptoms. If you have no history of trauma to your lower legs I would suggest you start with seeing a general practitioner(GP), he can definitely order plain radiographs and might be able to help you. he will decide whether you need to see a specialist orthopaedic surgeon or not. I hope this helps you"
},
{
"id": 141916,
"tgt": "What is the treatment of dizziness ?",
"src": "Patient: I am a 33 year old female that for the past year I have a unset of hot sweats, fatigue, lightheaded, weak and sometimes whole body pain about 3-4 times a month that normally last for 20-30 mins after the sweating stops and im not dizzy I still fell fatigue for about a hour or longer. It seems to take a lot out of me. I have ended up in the ER because of it and they said I was just dehydrated. I have had blood work taken and even taken while I was having the symptoms and it came back fine. I have had a MRI of the head and nothing. I have seen a cardiologist and had my heart check and had a monitor for two months and it came back normal. I do get a headache sometimes before the episodes start but not all the time I have seen a neurologist for my headaches and he cant seem to figure why I get them. I am at the point I just want to know why this is happening. Doctor: Hello!My name is Dr. Aida and I am glad to attend you on Healthcaremagic!It is necessary to perform differential diagnosis between different possible causes that may lead to this clinical situation: a thyroid gland dysfunction, hypoglycemia, orthostatic hypotension, or seizures. For this reason, I would recommend consulting with your doctor and performing some tests: - thyroid hormone levels- complete blood count for anemia- a Head Up Tilt test for orthostatic hypotension- blood electrolytes- an EEG for seizures. Consulting with an ENT specialist would help investigate for an inner ear disorder, which may mimic this situation. You should discuss with your doctor on the above tests. Hope you will find this answer helpful!Kind regards!"
},
{
"id": 126019,
"tgt": "Does a torn Achilles tendon heal on its own?",
"src": "Patient: I ve had achilles tendon pain for 2 years and was diagnosed with a partially torn AT. can it mend on it s own? Dr. Positano said it can with a brace and icing. I ve had open heart surgery recently and haven t been able to take care of it. I also have lower back pain. Can these two problems be related? Doctor: Hi, In my opinion, both these problems can be related in the form of a slipped disc with sciatica pain. Partially torn tendon achilles can heal with fibrous tissue and may function normally also. Please get your check up done by Orthopedic Surgeon for correlation of back and ankle pain. Hope I have answered your query. Let me know if I can assist you further. Regards, Dr. Gopal Goel, Orthopedic Surgeon"
},
{
"id": 2869,
"tgt": "What are the chances of my pregnancy after being injected with hcg 10000?",
"src": "Patient: Hi doc, my doc have out of fociluar study and yest I was given hcg 10000 injection in evening at 7pm. What are my chances of pregnancy this time. When should I have relationship with my husband. Please help. I m 32 yrs married since 3 yrs eager to conceive. Doctor: Hi,The Hcg injection will make your ovary ovulate after 36 hours from taking it. You should have intercourse after 36 hours of the injection. This is the time where you have the highest chance to concieve.Hope I have answered your query. Let me know if I can assist you further. Regards,Dr. Salah Saad Shoman"
},
{
"id": 34411,
"tgt": "Could red spots with sore throat and congestion indicate flu?",
"src": "Patient: I have red spots about the size of pencil erasers on my left leg & foot. Noticed yesterday and have more today. Checked for fleas none found, my dog also were treated 2 weeks ago. Also woke with sore throat and some congestion. Could it be part of a flu? Doctor: Thanks for posting you query to health care magic.your symptoms and history sggestive of flu like infection .It could be due to some virusus or bacteria.you should undergone blood investigation : complete blood count that will help in detection of platelet level along withother blood cells ..for throat pain I advised you to do warm saline gurgle 3 times a day .Tablet Paracetamol 750mg (adult dose ) for throat pain and congestion .review me after blood report to receive further treatment ..Hope you would be satisfied with my answer . Feel free to communicate if any query ..regards,Dr.Manish PurohitInfectious disease specialist"
},
{
"id": 164794,
"tgt": "What causes dislocation of knee joint in a child?",
"src": "Patient: hi, my daughter is 1 in 3 weeks and for about 10 months when shes get angry she clicks her knee and ankle out of joint and then back in, i took her to the doctors twice and they sent me away saying it was normal, ni went back a third time and she did it while he was holding her knee, he seemed quite shocked and has referred her to a specialist at a hospital, im just wanting more info on this and maybe what it could be and varioius treatments? Doctor: This is not normal. There are some diseases in which dislocation of knee occurs. You should get the child examined by a Orthopaedic doctor."
},
{
"id": 150495,
"tgt": "Synovial cyst compressing the nerve root in the lumbar MRI, foraminal narrowing, disc bulge. Had lumbar discectomy. Severe pain",
"src": "Patient: I received my lumbar mri results and this is what it stared...there is a right sided synovial cyst compressing the descendjng right l5 nerve root..l4-5 there is a right disctomy with decreased size of disc herniation but still mild to moderate right foramial narrowing.there is a right sided synovial cyst compressing the descending right l5 nerve rootl5-s1 there is a diffuse disc bulgjng central/left paracentral disc herniation compressing the descending left s-1 nerve root.I received a lumbar discectomy on october 2012 and a cervical neck fusion on june 2012..and my symptons and pain remain the same! I really do not want another surgery..we have tried injections. Medications and theraphy but nothing works. Also the synovial cyst was not present prior to my surgeries..this is a new co dition that was just discovered Doctor: There are joints between vertebral bodies to facilitate the smooth movements. The synovial cysts are formed in these joints and act as pressure causing points around the spinal canal. weather in your case surgery is mandatory or can be avoided ? you need to consult your doctor- preferably neurosurgeon"
},
{
"id": 41144,
"tgt": "Suggest cost effective treatments for infertility",
"src": "Patient: My husband and I are having trouble conceiving. My hormone test came back fine. I have PCOS and I think my husband has a coagulation problem. I do not have normal periods, and I am also a diabetic (type1) our insurance will not pay for any type of infertility treatments. Is there anything we can do that will be somewhat inexpensive to get answers. I am 27 and my husband is 30. We have been trying for over 5 years. Doctor: Hello, in my opinion opt for ovulation induction and inutero insemination at the earliest after controlling your blood sugar levelsIn case you have any questions in future you can contact me directly on http://bit.ly/drmanishajain"
},
{
"id": 128702,
"tgt": "What causes swelling on eyebrows and cheeks?",
"src": "Patient: Hey, I m Stephanie, 25. I have lots of health problems. Got up today and above my left eye, eyebrow area, it s swollen and super tender to touch. I have another on my right cheek. I do take the Imaitrex STAT dose injections when need but I have not had a headache. Doctor: Dear patient sudden onset of swelling and pain above left eye may be due to infection of upper eyelid may be stye. you need to consult opthalmic surgeon as son as possible. Usual treatment is short course of antibiotics and with painkillers for symptoms relief.Do ice fomenting to reduce pain and swelling. Tab diclofenac 50.mg should be taken for pain relief.Hope this will be helpful."
},
{
"id": 28442,
"tgt": "Suggest treatment for fluctuating BP readings",
"src": "Patient: i have been told I have atrial fibrillation. i have pulse136 heart rate and my blood presure this time 104/60.. i am 77. they have me on metoprolol 50 mg and diltiazem hcl 300mg. Also ramipril 5mg simvastatin 20mg , 1 aspirin 325 , terazosin. It seem the medication isn t working. What are suggestions This condiion only happened with in the last two months. I was allways had blood pressure 130 /80. And normal ekg reading. thank you appricate your imput. Doctor: Hi welcome to HCM.I understand your query and concern.Atrial fibrillaton is known to cause fluctuations in blood pressure.You are currently placed on right kind of drugs by your physician.I advise you to have a baseline 2 dimensional echocardiography,ECG and lipid profile to assess the basic cardiac reserve of your heart.Restrict the intake of salt to less than 6g/day.Regular physical exercise in the form of brisk walk for 20 min a day for 5 days a week is pretty useful.Drugs like antihypertensives and antiarrhythmics will help.Reduce the intake of fatty and fried food.One pomegranate a day will help to keep your heart at good pace without clot formation. Consult a Cardiologist for further expert management.Post your further queries if any.Thank you."
},
{
"id": 75990,
"tgt": "What does a mantoux test being positive mean?",
"src": "Patient: My friend fell down from a flight of stairs in mid 2010. She has a spine injury and now since is pain in her back is getting severe, she went to the doctor. Her mantoux test is positive. What does that mean?? Does it necessarily means that she has TB?? Whatever the case, is it cureable?? It has been a year since she had the accident and her checkup was on July 2011. Please help me out. Doctor: Hi thanks for contacting HCM...Tuberculosis is curable through AKT course of antibiotics....So don't worry about that....But positive mantoux test not always means TB positive ....It could be latent TB infection....If BCG vaccine given then also test can be positive ....Atypical mycobactetia also gives positive reaction....Tuberculin skin test also affected by few factors like....../age/immunity/chronic illness etc...So for tuberculosis confirmation other tests done like chest x ray, sputum examination etc....Consult physician for examination ...Take care"
},
{
"id": 145754,
"tgt": "Suggest treatment for ankylosing spondylitis",
"src": "Patient: I am taking tests to see if I have ankylosing spondylitis. The doctor seems to think that I have it, as do I. Once he sees the results of my x-rays and its confirmed that I have ankylosing spondylitis, will I need to take off work for an extended period because my job is very physically demanding. I am pretty much on my feet all day and do a lot of bending and heavy lifting. Jason Doctor: Hi,Thanks for writing in.Ankylosing spondylitis is a condition causing pain and stiffness in the spine. This can cause spinal deformity and complications arising out of it. Non steroidal anti inflammatory drugs are the first line of treatment for this condition. If the pain does not respond to this group of medications then stronger pain relievers are like naproxen are given.There is a new category of medications called tumor necrosis factor (TNF) blockers and these have been known to help patients of ankylosing spondylitis. These include medications like Adalimumab, Etanercept, Golimumab and Infliximab.Other than the above, improvement in general living conditions can also be seen after physical therapy and surgery to release involved joints."
},
{
"id": 2364,
"tgt": "Can pregnancy happen by rubbing of the ejaculate on the vagina?",
"src": "Patient: I ve read many, many replies (on many websites) from doctors regarding the chances of getting pregnant by a small amount of ejaculant on one s hands being rubbed onto the vaginal opening. Some simply say yes, possible b/c it only takes one sperm , while some say It s highly unlikely , etc. What I d like to know is: 1) Is there any medical professional here who actually knows of a case of pregnancy happening from such a situation? and 2) are there any more scientific studies on this? Thank you. Doctor: hello,thnks fr trusting healthcare magic docttors fr ur health related queries.i guess ur concern is-probability of getting pregnant in tis kind of scenarios n any scientific studies on these???lets discussion part by part...well lets have short discussion about semen n sperm...semen is tat white lotion thing what guys ejaculate..in tis lotion small small sperms will be there,which meet ovum n women becms pregnant....semen is sticky in nature...imagine u have had a very oily food,,even if u wash with soap water still u can feel that sticky nature fr sometime..same way semen is...it sticky in nature...with tis sticky nature of semen,it wil be sticking to the part of vagina where ever its been rubbed,,thereby sperm too.....so tis sperm may meet the ovum present inside vagina and women bcomes pregnant...usually sucess rate of pregnancy increases when men ejaculate inside the vagina...tis is the usual norm...however medical field is know for many miracles....tis small amount of sticking semen can also cause pregnancy....but chances are very very minimal.....whatever replies u have come across online it seems to be correct...no one can pin point n say pregnancy is possible or not possible...1.is there any medical profession:as such separate departments are not availble to judge in these special scenarios as u hv told above..generally its obstetric n gynecology dept tat deals with pregnancy n infertility..2.are there studies:there may be present....fr tat u need to access scientific journals of medical fields.......i hope this was informative enough,n useful fr ur query..regards:dr.sudha rani panagar..(usually a women can pregnant after 12-13yrs when her menses start...n a women cant become pregnant after 50yrs,as her menses stop.....but there its been reported oldest women to to pregnant is approx abv 60yrs n youngest women approx 10yrs....wanted to give example to say medical world is know fr miracles!!!!!!!)-"
},
{
"id": 109905,
"tgt": "Suggest treatment for back pain",
"src": "Patient: Hi I am 25 years old and suffering from a back pain from last 3years. It was not a constant pain but i used to get it once in 4-5 months . But from last 8-9 months my condition got very worse. Now my pain is constant 24X7. I cant walk or stand for more than 5-8 minutes. So finally i went for a MRI scan. My report says : A disc bulge at L3-L4 and disc herniation (protrusion) at L4-L5 and disc herniation (extrusion) at L5-S1 level causing significant compressive effect over the traversing and exiting nerve roots bilaterally. I consulted with few Orthopedic Doctors , they all r saying to go for a surgery. But one said to go for a physiotherapy. i consulted with one M.P.T (Neuro) he is saying , i can get rid of all this pain in just 3-4 months of regular therapy. I am completely blank what to do or not. Can any one please suggest me how i should proceed. Thanks Doctor: Most back pains get better with physiotherapy. Spine surgery is usually required only if you have significant pain going down the legs or symptoms like numbness or weakness in legs. If physiotherapy doesnt help, there are other options like injections. But you need ti be evaluated properly by a specialist spine surgeon to make the proper decision."
},
{
"id": 128930,
"tgt": "What causes pain in the right foot?",
"src": "Patient: Hi for about a month now I ve been experiencing some pain in my right foot which travels from my small toe to my ankle. It hasnt stopped me from doing anything. But today the pain increases dramatically and has shifted to the sole of my foot and ankle, I cannot bear weight on foot and can only walk by using the side of my foot Doctor: Hello,Thank you for using healthcaremagic.I read your question and understood your concern.The most probable disgnosis is plantar fascitis.The treatment includes nonsteroidal antiinflamatory medication , physiotherapy for streching the achilles tendon and heel cord, sleeping with night splints and using shoe inserts. Sometimes cortison inhection in the heel is needed.I wish you quick recovery.Dr. Selmani"
},
{
"id": 104547,
"tgt": "Suffering from excessive sneezing, gastrointestinal problems. Related symptoms?",
"src": "Patient: Bizarrely, I seem to be suffering from sneezing (no real congestion, just a lot of sneezing) and gastrointestinal upset concurrently. I would have thought the two systems affected by these things were completely discrete, but as I m not a physician, what do I know? I m not seriously ill, just feeling a bit off, and rather confused. Have you an opinion on this? Doctor: Hello, Thanks for your post. Sneezing and gastrointestinal upset can be due to Allergies or Viral infections. Sneezing indicates something irritating upper airway passage and body wants to get rid of it by sneezing force. Some times Gastroesophageal Reflux Disease can also irritate upper airway passage causing sore throat or sneezing. You should take plenty of water. You may take over the counter ant allergic drugs. Sort out what type of GIT upset you are having? if symptoms persists or worsen then visit your doctor. Hope it helps. Take care!"
},
{
"id": 157926,
"tgt": "Cancer survivor. Taken tamoxifen. Had sex. Am I likely to get pregnant?",
"src": "Patient: I am a 52 year oIld female, a cancer survivor. I took tamoxifen for five years doing this time my period came on off and on. In December 2012 I completed my 5 years of tamoxifen. My period came on June 24. I was so heavy I could only stay inside. During this time I was out of state on vacation. Upon my return I went to see my oncologist to ensure everything was find. I had not been sexually active for over 5 years. I had sex in July about 2 weeks after my period came on. Is it possible I could be pregnant? I am having symptoms of being .pregnant Doctor: Dear Ma'am,Sorry to hear about your problem. Periods do tend to re-start after completion of tamoxifen though initially they are irregular. There is a chance that you can get pregnant if you had your periods once. You should get a urine pregnancy test done to rule out pregnancy.Hoping for the best."
},
{
"id": 218530,
"tgt": "Is pregnancy possible despite negative pregnancy test results?",
"src": "Patient: I have been on the pill for about a year now and have always had a regular period I don't miss the pill and take it at roughly the same time everyday. This past month I missed my period it's been two weeks so I took a test today and it's negative. Could I still be pregnant? Also, this past month my whole routine has been changed I am now up at 5:30 am and have a one year old could lack of sleep cause my period skip? Doctor: Hello and Welcome to \u2018Ask A Doctor\u2019 service. I have reviewed your query and here is my advice. As you took the pills as per the schedule the possibility of pregnancy is less. Sometimes with prolong intake of pills amenorrhea can be seen. And also as you said alterations in daily schedule like dietary changes, exercise changes, psychological stress etc, can also affect menstrual cycle by affecting the hormone levels. As your Urine Pregnancy Test gave negative result the possibility of pregnancy is less, but if you want to confirm better to go for blood test for pregnancy, that is estimation of Beta HCG levels. If the levels are less than 5mIU/ml pregnancy is almost ruled out. Hope I have answered your query. Let me know if I can assist you further."
},
{
"id": 116181,
"tgt": "What causes iron saturation even after taking iron supplements?",
"src": "Patient: Hi I had a test done in mid- July that showed my Hgb 101, Iron saturation at 0.05 so I was recommended to take an iron supplement. I have been taking it since mid-July. Recently I went for new blood work. Hgb 118 but Iron saturation is still 0.05 Where should I go from here? Doctor: HiBased on your query and findings my opinion is1. Your Hemoglobin concentration has improved with iron supplements. 2. It takes some time for saturation to come to normal as the iron stores need to be restored with the supplements you are taking. 3. So dont bother much about iron saturation as it is corrected only over time.4. However if you are still concerned you can get a reticulocyte count done to check how you are bone marrow I'd responding to supplements.5. Meanwhile be regular with supplements, have iron rich foods.Hope this helps"
},
{
"id": 12864,
"tgt": "What causes an itchy rash on the back?",
"src": "Patient: six months ago, I had a terrible sore throat. Went to urgent care, put on Zpack. Went to beach and got sunburn. The burn bubbled when I exercised, and I popped the bubbles :( Few days later, I had a terrible itching sensation on my upper abdomen and scratched only to awake with a red, itchy rash.. was bumpy all over (not blistered). When I would exercise, it would get worse and more irritated so I stopped and let it dry out. It lasted a few weeks before it dried up and flaked off, leaving white spots all over my stomach. (possibly b/c i was tan?) there are still a few scars. Last week, I got a small (50 cent piece sized) rash on my rib cage that looked similar. This one bubbled. Dermatologist visually diagnosed it as HSV (i dont agree) and put me on Anti-viral. Patch dried up and new patch appeared on back rib cage. Lasted two days, is dried up now. Not terribly itchy, no pain, no yellow crust, no genital or facial blisters... Ideas?? Doctor: Hi,You may be right. It may not be herpes simplex (HSV). I feel that most probably it may be impetigo. It relapses off and on. I think immunity may be poor due to some reason. You should consult other dermatologist and should take second opinion.Hope I have answered your query. Let me know if I can assist you further. Regards, Dr. Ilyas Patel, Dermatologist"
},
{
"id": 53682,
"tgt": "What causes frequent migraines and vomiting of bile and mucus?",
"src": "Patient: Hi, my problem is related to my liver. So every few weeks or so, I get migraine and i start throwing up acid and mucous, which is called bile and I throw up for a whole day, even at night like I stay up all night throwing up and sometimes I even throw up for maximum 2-3 days. And i've tryed TONS of medicines from everywhere, and nothing helped! What should i do? What medicine would cure this for good? Doctor: firstly i tell you that your liver is perfectly ok this is not related to liver , liver normal secretes bile to bowel , why you having vomiting with migraine is due to a reflex mechanism which is stimulated due to intense pain result into pylorospasm called reflex pylorospasm that is contraction of pyloric part of stomach which result in content of stomach out orally.avoid drinking liquid at the time of pain drink coffee (caffeine) to relief mild pain consult your physician for severe pain thank you"
},
{
"id": 143135,
"tgt": "Suggest treatment for pain after undergoing microdiscectomy",
"src": "Patient: I am a 35 yr old man who had l4 l5 microdiscectomy on March 3rd. After the surgery the pain that once haunted me daily had been greatly reduced. Still I had pain and was on pain meds til about mid April. I have been doing pt since mid March and it s very painful. I still do pt and it still hurts a lot. I also find myself in great pain when not doing physical therapy, some days I can t even do my home exercises. When I told my orthopedic dr bout my pain he said I should be ok without the pain meds and I wish it were so. I just want to get better and if I can t always do the exercises that will help me how can I? What do I do? Doctor: Hello!Thank you for asking on HCM!Regarding your concern, I would explain that physiotherapy is really helpful in such cases. But sometimes chronic back pain is persistent despite surgery and physiotherapy. That is why I would recommend starting low doses of amytriptiline or gabapentine for the pain. These drugs can help against chronic back and nerve pain. They should be taken for at least 3-6 months, in order to have the desired effects. Hope to have been helpful!Kind regards, Dr. Aida"
},
{
"id": 210293,
"tgt": "How to confirm whether a person is suffering from autism?",
"src": "Patient: Hi, I m a 17 year old girl from New Zealand. Since a few years ago, I have been quite aware of the fact that I have a high probability of being a high-functioning autistic. I did some research at my local autism centre and everything seems clear, but my parents - especially my mother - continue to deny the allegations, saying that I m just making excuses and that you re not autistic or whatever, you re just stupid . Here are the things I have discovered. - I stayed in kindergarten 6 months longer than I should have been due to my development delay problems (late speaking, wearing diapers until I was 4, etc). Even before then, I apparently had a special needs carer/tutor lady come into my house. - I am socially anxious, and have trouble socializing or interacting with other people unless it is completely necessary. Up until I entered high school (13 years old), I was not very social and only talked to a select group of people. - I did not speak until I was 5, and according to my mother, when I did speak it was just simple phrases like Yes or No , and even then it was only to certain people such as member of my family (Mother, father, older brother by 2 years). - Every school I have been in has placed me in their GATE society (the Gifted and Talented programme), and they have placed me in nearly every annual Creative Writing programme held at school, and after reading all the Gifted information that the school provided there s a fine line between Gifted and Autistic. - I don t know if you ve already observed it from here, but I have this extended vocabulary which I sometimes wish I didn t have. Sometimes I think I speak like I have Asperger s Syndrome. - There s this online autism test created by Simon Baron Cohen that was featured in (I think) Embarrassing Bodies. I took it and got a score of 35. I d really like to get a diagnosis so at least I m aware of whether I m autistic or not. I know this isn t enough information but it s the most I can do via the Internet. Thank you. Doctor: Hello and welcome to Healthcare Magic. Thanks for your query.I understand that you are wondering whether you may be having a diagnosis of Autism / Autistic Spectrum Disorder. However, a detailed history (including a childhood history from your parents) and a detailed psychological assessment is necessary to make a definitive diagnosis of an Autistic Spectrum Disorder. Usually, Autistic Spectrum Disoders get diagnosed early in childhood itself, with one criteria being that the onset of symptoms should have been before the age of 3 years. If you would like to clarify your doubt, I would suggest that you consult a psychiatrist in person for a detailed evaluation. Online tests, especially the self-rated ones are usually unreliable.Wish you all the best.Regards,Dr. Jonas SundarakumarConsultant Psychiatrist"
},
{
"id": 23473,
"tgt": "What causes pain and burning sensation in chest?",
"src": "Patient: Hi. I am experiencing pain in the middle of my chest, this started this morning when i was drinking a cup of tea, it was a strong buring sensation but it still hurts now. I was being sick on Wednesday night (due to heavy drinking). I have never had this happen before though. can anyone offer any advice? Thanks Doctor: see this is termed medically as gastritisin lay man language we call it heart burnreto sternal( behind mid bone of chest) pain and burning sensation is thereas you say its after drinking hot cofee and boozing its a typical presentation of gastritis dont worry its not a cardiac origin paintake non spicy food for few daysavoid excess oildonot drink and smokeavoid tea and cofeetake tab neksium 40mg twice daily for 21 daystab digene 1 tab twice for 7 daysyou would be fine"
},
{
"id": 219801,
"tgt": "Is it advisable to travel when I am pregnant?",
"src": "Patient: I live in reno Nv.. which is about 4200 feet above sea level. I wanted to visit my mother in-law tomorrow for mothers day. she lives in lake tahoe nv. which is about 6300 feet. the drive is only 1hr15mins. but i am 35weeks and 4 days pregnant. is it ok to go, or not? Doctor: hi I understand your concern. as per your history I can suggest you can travel but with great caution.because at your stage chances of preterm delivers are high.I could not detect from your history that it's a high risk pregnancy or low risk one ...by saying this I mean hope you are not having placenta previa etc.if that is not there then you can travel but try and take progesterone support 2 days before going and continue it for one week.ang be gentle with the driving.hope this solves your query."
},
{
"id": 10177,
"tgt": "Suggest remedy to avoid hair loss",
"src": "Patient: i am having hair loss .so i had consulted a doctor,and i have gone through all the required tests.but the reports are normal.and suggested to use kera xl serum and miniscalp plus,protifit miki protien,spexdime shampoo and iofer sf and bio ext tablets for three months. to use there products is it safe are or not and while using those sparys and drops have to wash hair daily?and when to apply hair oil then??please answer my question sir. Doctor: Hello and Welcome to \u2018Ask A Doctor\u2019 service. I have reviewed your query and here is my advice. I have gone through your complaints and these products are safe to use. You don't have to wash your hair daily. Go for twice weekly wash. You can apply hair oil 2 hours before head wash and wash it off. Hope I have answered your query. Let me know if I can assist you further."
},
{
"id": 223637,
"tgt": "Does taking morning after pill after 72 hours of sex prevent pregnancy?",
"src": "Patient: I got my period on the 2nd of october and finished around the 5th. I had unprotected sex with my partner on the 9th and took the morning after pill straight after. We had protected sex the next day and noticed leakage around the base of the condom. Did not really worry about it because it could have been from me, but on the 12th I started to worry that it may have been his cum. So the next day I went to go get the morning after pill again but it was well after 72 hours of having sex. Should I be worried? Doctor: Hello dearI understand your concernDo not worryChance of the pregnancy is nil as you had sex in the safe period of the cycle.During safe period, no ovum is released so no fertilization will take place.But chance of the hormonal imbalance and side effects are high as you used morning after pill two times with in short time.It also cause delay in period by 8-10 days.Avoid stress, take healthy diet, drink plenty of water and do regular exercise.Do not use morning pill so frequently as cause menstrual irregularity, hormonal imbalance and fertility issue.Hope this may help youContact HCM for further health queryBest regardsDr. Sagar"
},
{
"id": 35759,
"tgt": "What causes sore throat,headache and nausea?",
"src": "Patient: My papillae are extremely swollen right now. I ve also got a sore throat and congestion (when I woke up this morning, my lymph nodes were horribly swollen). I had a fever for part of the night, I believe, but seem to be normal temperature now. I ve got a headache and nausea that goes on and off (it was really bad while I was trying to get to sleep last night, but hasn t really bothered me sitting up so I think it s just related to congestion). Should I get checked for strep? Doctor: HI, thanks for using healthcare magicSore throats are more commonly viral. They are due to viral infection in 80 to 90 % of cases and most therefore do not require antibiotics.IF the sore throat persists beyond 3 to 4 days, is extremely severe or is associated with a fever lasting a few days, then bacterial infection can be considered.The use of oral pain killers as well as throat sprays or gargles may be helpful to you.Examples of gargles or sprays are chloroseptic, covonia, diflamI hope this helps"
},
{
"id": 148062,
"tgt": "Can brain angio and lumbar puncture been done?",
"src": "Patient: My husband has been since last June, they did a work up and found 2 spots on his brain possible demylenation, he is now having symtoms again which are numbeness of the face and lips, headaches and his leg feels hallow all on the Left side. Today we went back he had a second MRI and it show significant increase in spots from his June MRI, they are now asking him to have a brain angio and a lumbar puncture.. Differental dx of MS or a vasculitis of the vessels in his brain? Thoughts or suggestion. When he first became sick he had speech problems, member issue, fatique and severe pain all on the left side at times he could not walk. He is on Neurontin and a few other medications. They also thought possible seizures, he is a smoker but, the MD today said, he had enough increase in spots in the brain that she is not comfortable with just saying they are ischemic changes from smoking. Doctor: HIThank for asking to HCMI really appreciate your concern for your husband looking to history given here I would advise to go for the investigations suggested (Angio brain, and lumber puncture) these are the important tests by which diagnosis can be cleared, I hope this information helps you, have nice day."
},
{
"id": 219838,
"tgt": "Did Ovofar delay my periods?",
"src": "Patient: hi I m trying to conceive and my gynec priscribed me ovofar 50 mg 1st cycle I took it from 3rd to 7th day and I got my period in 22 days, then I took ovofar again from 3rd to 7th day and this time its 37 days and cant see any signs of periods and home upt tests are negative.(took 3 tests in last 5 days) latest one was few hrs back. I am having crams since last 1 week (which I usually get 1 day before my periods or during 1st and 2nd day ) not constant or everyday but randomly. can taking ovofar delayed period? or affect upt? please guide I m confused Doctor: Hi there,,The symptoms irregular periods sometime short sometimes long is more due to an-ovulation, due to hormonal imbalances.The ovofar is to correct that. You probably have PCOS, which is causing the irregular cycles and which is why the doctor has prescribed you the ovafor.You need to first get completely evaluated for PCOS before proceeding with the treatment.I would advise you to get the blood tests such as FSH,LH, TSH and prolactin done. These will help determine if the cause of irregular cycle and infertility is them.You also need to do ultrasound pelvis to check for problem is in ovary and womb.Your partners semen analysis is also necessary to get a complete picture.Kindly do the above tests and discuss with your gyanecologist.If PCOS is diagnosed by blood tests and ultrasound, medications such as metformin need to be started.Hope this helps.Regards."
},
{
"id": 91851,
"tgt": "Suggest medication for abdominal pain",
"src": "Patient: I have abdominal pain under the rib cage from both sides , I have done ultrasound and nothing showed up,I removed the appendix at age 12 and I removed gallbladder 2 yrs ago , age 49, this pain has been there for about a month , blood test showed low hemoglobin count and EXTRA fritanin LEVEL ( IRON IN THE BLOOD) , BLOOD PRESSURE AND COLOSTROL ARE VERY NORMAL. Doctor: Hi and thank you so much for this query.I am so sorry to hear about this pain. Because various tests have failed to show any major abnormality, I will suggest that you get good pain medications to suppress this pain. What medications you get would depend on your doctor's evaluation and appreciation of the severity of this pain. Some drugs can only be gotten with prescription while others like Tylenol, Ibuprofen, aspirin, etc can be gotten over the counter. Get consulted so that you can b prescribed what is best indicated for you.I hope this addresses your query fully and provides enough information for you to act on. Feel free to ask for any clarification or more information if needed and I will gladly respond to.Thanks for using our services and I wish you well.Dr. Ditah, MD."
},
{
"id": 80860,
"tgt": "Experiencing tightness in chest that goes to my throat and jaw line and feels lethargic",
"src": "Patient: i am a lady of 66yr went to my doctors 1 week ago had all blood test came back ok still feel so unwell the ti it started when on the Friday i had tightening in chest going up to my throat and jaw line still feeling so lethargic all time do house then need to rest Doctor: The symptoms you are telling all point towards pain related to your heart.You better get a electrocardiogram done."
},
{
"id": 80747,
"tgt": "Suggest remedy for chronic cough with all tests being normal and no help from any medications",
"src": "Patient: I have had a cronic cough for over a year. I have had chest xrays, breathing test ,cat scans, and a bronscopy done. I have been on pozac, acid reflux meds, inhalers, allergy meds. Now I m on Nuronton and zerlax. I have been to two pulmonologist and still the cough persist! please help! Doctor: Hello dear, thanks for your question on HCM. I can understand your situation and problem. Since your extensive pulmonary work up is normal, no need to worry for pulmonary causes. Uncontrolled acid reflux disease can cause chronic cough.And for proper control of acid rrflux, you need to follow these lifestyle modifications along with drugs.1. Avoid hot and spicy food. 2. Avoid stress and tension. 3. Avoid large meals, instead take frequent small meals. 4. Go for walk after meals. 5. Keep 2 - 3 pillows under head in bed. 6. Avoid alcohol and smoking, if you have these habits. 7. Loose weight if you are having obesity. Don't worry, you will be alright."
},
{
"id": 125523,
"tgt": "Does leg pain after felt pop in calf muscle indicates charlie horse?",
"src": "Patient: I was sleeping and felt a pop in my calf muscle followed by a severe cramp. I got out of bed and straightened my leg by standing. It was painful to walk on the leg afterward and now some hours later is still painful altough better than this morning. I have also put an elastic brace aroung my calf for support. Was this just a charlie horse? I have never felt a pop like this. Doctor: Hello, It could be a muscular pain as in Charlie horse. As a first line management you can take analgesics like paracetamol or aceclofenac for pain relief. Drink plenty of water as dehydration worsen the pain. If symptoms persist, it is better to consult a physician and get evaluated. Hope I have answered your query. Let me know if I can assist you further. Regards, Dr. Shinas Hussain, General & Family Physician"
},
{
"id": 167496,
"tgt": "Why is my toddler having seizures, more weight gain and trouble breathing?",
"src": "Patient: Have a friend with a son who is three. At about 2 1/2 years, he began to have a severe and rapid weight gain. Other symptoms followed in the next few months - seizures, more weight gain, trouble breathing, and other random symptoms. This past week, his symptoms caused a visit to the ER, where they found he also has an enlarged heart. He was moved to the PICU and has been undergoing endless tests, including ekg, eeg, CAT scans, MRIs... everything. And they can t pinpoint the problem. Anyone with any ideas? Doctor: 2.5 YEARS RAPID WEIGHT GAIN POINT OUT ENDOCRINOLOGICAL PROBLEM.IS HE HAVING HYPERTENSION?{IF SO,MAY BE CUSHINGS}OR WT GAIN MAY BE DUE TO FLUID ACCUMULATION {NEPHROTIC SYNDROME}QUITE COMMON IN THIS AGE GROUP.IN NEPHROTIC SYNDROME BLOOD WILL CLOT MAY BE CAUSING SEIZURES.CHECK OUT HIS PROTEIN LEVEL,LIPID LEVEL{YEAH LIPIDS WILL BE IMBALANCED IN NEPHROTIC}DO HIS BLOOD COAGULATION PROFILEHOPE HE GETS BETTER....."
},
{
"id": 180069,
"tgt": "How to increase appetite in child?",
"src": "Patient: My grandson, age 3, will not eat. Only french fries, potato chips, graham crackers, and a few cookies. He refuses to eat all other foods. No vegetables, no meats of any kind and no sea food. He will drink apple juice, milk and yogurt, but the yogurt must not have any fruit in it. I know he s hungry at times, because he says his tummy is growling and he cries. What can I do? Where can I get help? Doctor: Clearly it is not a medical issue but habit issue. And to change habits one need to discipline, not only for eating but in all other aspects of daily habits.Be firm, all Carb rich foods have killed his appetite. You need to be firm in not providing what you are giving now & no alternative to healthy food should be available even if he remains hungry."
},
{
"id": 209388,
"tgt": "What medication is suggested for lack of interest and concentration?",
"src": "Patient: sir,My child have 13 complete and 14th running . he is eighth now.he was in sixth onwards he had very much lezzy. minimum home work also didn't done. defaults are coming. teachers warns me. he could not able to study.we thought it is adulteration period . he must change . but it happens reverse. in the school he got only 40% only. so they told retain class. his behavi our is also not good . he is escapes every thing. in house he hated me and beets me. but his nature is soft how he change.why is like that?rama Doctor: Hello and welcome to Healthcare Magic. Thanks for your query.I understand that you are concerned about your son's declining academic performance and behavioural problems. Now, these problems could be due to a variety of causes like learning disability, conduct disorder or other behavioural problems. From the limited information that you have provided, it is difficult to arrive at a definite diagnosis. He will need a dettailed evaluation and psychological assessment in order to pin point the diagnosis. So, I would suggest that you take him to a child psychiatrist for a detailed evaluation and further management.Wish you all the best.Regards,Dr. Jonas SundarakumarConsultant Psychiatrist"
},
{
"id": 113950,
"tgt": "Every time I eat I have severe upper back pain. Can an online doctor help me ?",
"src": "Patient: iIm 19 and bulimic everytime i eat i have server upper back pain can you helpme please Doctor: hi sabrina, welcome to hcm thanks for the query your back pain may be due to strain or poor posture or i may b due to any injury if you are obese try to put down your weight if possible try to take physiotherapy for that try to maintain good posture during working hours take small breaks after long duration of sitting or standing use thin pillow during sleeping do yoga and back strengthening exercises avoid stress. take good sleep if the pain is severe consult the specialist take care"
},
{
"id": 94665,
"tgt": "Had unprotected sex, stomach pain and vibration in limbs after normal period",
"src": "Patient: hello sir. i got physical relation with my girlfriend on 29july 2012 without any precaution and her menstrual cycle have came properly on 2 august then on 30 august then 27 september .cyccle of 2 august was now as usual but after that cycle was good as usual .she said to me that she is not feeling well , she has very much pain in her stomach .and vibration in her hands and legs ... what should i do sir Doctor: hi vimal, Pain in the abdomen with vibrations can be attributed to menstruation.However weakness can be due to anemia and the cause for all this needs to be evaluated,best by a GYN.So get her evaluated All the best"
},
{
"id": 160120,
"tgt": "I got a tumor at my right neck. Would you suggest the best treatment to fix it ?",
"src": "Patient: Hi! I got a tumor at my right neck and it was there since last one month. I have consulted one cancer specialist in Vizag and he said like it is vallecula epiglottis. He also said this is in stage 4 and this kind of cancer won t spread across other body parts. As part of the treatment, he said he will apply biotherapy, chemotherapy and radiotherapy. Please suggest the best treatment for this and how much it would cost to cure this permanently. Doctor: welcome to healthcaremagic it will be good to attend a special center dealing with cancers lime tata cancer center or as you find suitable in private and paying hospitals you will get packages and have to choose as per your choice"
},
{
"id": 71547,
"tgt": "What are the chances of having TB again?",
"src": "Patient: Hello doctors.. my name is John Trice i wanted to ask regarding the fear that i am having.. i had a check up way back 2005 ... i was diagnosed that i have TB.. the mild one... years past.. i did not have follow up on that... now i have a job requirement.. that is the medical exam.. im afraid if i will have that disease again.. could i have the disease back? Doctor: Hello,For tuberculosis once confirmed, the anti-tuberculosis drug should prescribe for six months. You had not taken any medicine so it might be still present or you might recover. For that, your chest x-ray and sputum acid-fast bacilli (AFB) test done primarily to rule out tuberculosis. Consult pulmonologoist for examination. Hope I have answered your query. Let me know if I can assist you further.Regards,Dr. Parth Goswami"
},
{
"id": 48223,
"tgt": "What causes pain below back side of the ribs?",
"src": "Patient: 44 male with extreme fatigue for last two( never tired and always stays busy). Woke up yesterday with extreme pain toward bottom of back rear ribs. Do not believe its his ribs, no s/s of any issues with the ribs. Was told 2 years ago after an ultrasound that he had no left kidney. Concerned there is something more going on here. Hasn t lost any weight, eating is normal. Doctor: HIWell come to HCMIt is advisable to get done X-Ray of chest to rule out the underlying cause of rib pain because this could be due to some lung pathology, chances of infection need to be ruled out, if this turns unremarkable then this is nothing to worry because this could be then due to muscular spasm, try to maintain good posture, try the medicines given bellow1) Tab Acetaminophen 500 mg once in day2) Tab Diclofenac 50 mg once in day3) Tab Chlorzoxazone once in dayalong with this better to come out of stress if you ever have this, take care."
},
{
"id": 124076,
"tgt": "What causes numbness in left arm at night?",
"src": "Patient: Hi. I have a question about my arm. My left arm always goes numb during the night meaning I can t feel it and when I go to lift it in the air it s feels super super heavy and jus falls right back down n stays like that for awhile afte I wake up. What does that mean? And should I be worried? Doctor: Hello, We need a physical examination and assessment of the upper limbs. As you mentioned about having weakness in the arm and unable to lift, I feel this should be related to the nerve impingement. As nerve has a function of motor and sensory any disturbance in the nerve functioning can lead to such symptom. As it is occurring only in the lying position I think this should be due to pressure falling over the nerve in the cervical region leading to dysfunction of the nerve and leading to symptoms. Please do some neck stabilizing exercises for which you can find videos online about cervical spondylitis. Most cases in my 12 years of experience have recovered well with simple exercises. Hope I have answered your query. Let me know if I can assist you further. Regards, Jay Indravadan Patel, Physical Therapist or Physiotherapist"
},
{
"id": 136467,
"tgt": "Suggest treatment options for facial paralysis",
"src": "Patient: my friend has contracted shingles and ended up getting facial paralysis (Ramsay Hunt) and wonders how long it will take to regain her facial muscle or is there something she can do. She is now on Prednesone, and the rash on her face is gone as she had anti viral drugs immediately. She is so worried about regaining muscle on the side of her face. Is there something she can do in the meantime? Doctor: hi,i appreciate your concern.as soon as acute stage is over,and u feel comfortable with medications say after 2 wks,you need to see ortho/,PMR, expert who will decide about exercise programme,if neede electrical stimulation,.you may also need to see opthalmologist for eye care technique .thanks.any further query is most welcome."
},
{
"id": 5478,
"tgt": "Discoloured discharge from vagina, problem in getting pregnant for second time. Help",
"src": "Patient: hi i have been trying too get pregnant for 2 and a half years now after my 1st son i fell pregnant quick with him but now im struggling too get pregnant again but iv started having alot of discharge now and its getting really bad my boobs are always sore and im bloating alot but i have just researched discharge and it says it can bloke tubes i am very worried not as i have had discharge for nearly 2 years but got worse as times gone on and changed in couler alot please help .. Doctor: Hello, I would advice you to get a check up done from your gynecologist to rule out local infection which can be a cause for discharge,if it is so then you need to take a course of antibiotics to clear the infection. Second advice would be to go for a thyroid function test since you say you are bloating up,as abnormal thyroid levels can also lead to infertility. Meet your doctor with the reports,and follow her advice on further management . It is not a big problem so don't worry it will get solved soon. Take care. Dr. Rishma lakhani"
},
{
"id": 133138,
"tgt": "Does bulging disc cause swelling on feet?",
"src": "Patient: Thank you, my feet have been swelling up for a couple of days, very slowly. Only issue I have right now is an injury to lower back that makes it hard to walk, sit and lay down...Old injury that flared up 3 weeks ago, I had injured (bulged disk between L4-L5) that was repaired by Chiro. in 92. Would this cause swelling in feet and around ankles? Oops...as I do not have a job right now I see I have to pay sorry...checking elsewhere...Thanks Doctor: hi,thank you for providing the brief history.as you had a past injury of spine this swelling cannot be related to the back injury.This is more of a systemic pathology and needs a thorough clinical examination and diagnostic test post that.Due to weakness of the cardiac system or kidney issues this kind of swelling is assumed and without any Thorough clinical examination and diagnostic tests it will be tough to mention. Also this is not a direct injury to the joint so mostly can be systemic. Keeping the leg elevated and improving your physical activities should help reduce the swelling.Regards Jay Indravadan Patel"
},
{
"id": 43641,
"tgt": "Have primary infertility. Back ache, vaginal itching. Swab shows colonization of staphylococcus",
"src": "Patient: I have primary infertility for 3y, i think it was because of reduced freq. of I.C. so all our investigations were normal that i did them 5mo. after mariage... recently i develop backache with slight vag. iching, endocervical swad shows colonization of Staphylococcus saprophyticus & alot of lactobacilli, does this finding affect my fertility, ensure that i`m has freq. I.C. NOWADAYS........thanqs alot. Doctor: Hi,Thanks for your query , Infertility can be caused by numerous different factor - Reason - Inability of the egg to attach to the uterus lining . Inability of the egg to travel to the uterus. Inability to produce egg. Autoimmune disorder. Hormonal imblances. wishing your good health. Regards, Dr.Sharmila"
},
{
"id": 220543,
"tgt": "Can breast tenderness and thick vaginal discharge be symptoms of pregnancy?",
"src": "Patient: I take my birth control months at a time and then stop and my period comes immediately. I had unprotected sex a month ago and stopped my pill and my period is still not here. I have had some breast tenderness and thicker than normal discharge but two at home tests say negative. Am I pregnant? If not, why is it so late? Doctor: HiDr. Purushottam welcomes you to HCM virtual clinic!Thanks for consulting at my virtual clinic. I have carefully gone through your case, and I think I have understood your concern. I will try to address your medical concerns and would suggest you the best of the available treatment options.In a woman with regular cycles, day 10 to 20 of the cycle is the most fertile period of the cycle. If you had unprotected sex in this period , then only there is chance of getting pregnancy. Now, if your urine tests are negative; then you can safely wait for menses to come. If you want to induce menses, Tab Regestrone twice a day for 5 days.Please get proper information about using contraception method, so as to avoid any such situation in future.I hope my answer helps you.Thanks.Wish you great health.Dr Purushottam"
},
{
"id": 66545,
"tgt": "What is the lump on my vaginal lip?",
"src": "Patient: Hi Im 18 and during the summer I have had a large lump on the lip of my vaginal area and the doctor treated it for both a cyst and or chlamydia but then it went away. I just now got it again and I just want to know if it could be an std or something reallly serious Doctor: Hi, dearI have gone through your question. I can understand your concern. Many types of benign cysts are common in that area. You may have either some cyst or some sexualy transmitted disease. You need examination. Consult your doctor and take treatment accordingly. Hope I have answered your question, if you have doubt then I will be happy to answer."
},
{
"id": 8196,
"tgt": "How to get rid of pimples. I am 16",
"src": "Patient: hi... i am 16 years old and i have lots of pimples on my face so i am in need of your suggestion doctor please help me out!! Doctor: acne can be treated with local applications of cremes, some antibiotics, retinoids and LASERs. LASER though expensive ( costs upto 5,000/- ), can clear the face off active acne and acne scars. The results are quite dramatic and patients simply love their skin after LASERs"
},
{
"id": 95445,
"tgt": "My boyfriend is having sever abdominal pains. he is vomitting up blood",
"src": "Patient: hi my boyfriend is having sever abdominal pains. he is vomitting up blood , cant keep anything down. its been going on for several days now. i dont know what i can do for him or what we should do about it. we have went to the hospital twice now and no luck. Doctor: What do you mean by no luck? Its a medical emergency no point in coughing blood for so many days. There is no home remedies for this. If you want try antacid ( ACID AR) it might help you out."
},
{
"id": 49006,
"tgt": "Is kidney spotting controllable through medication?",
"src": "Patient: My husband has two spots on his kidney, the sizes are 1.7 cm and 2.1 cm. The expected growth in 12 months on both is 1 cm. The Dr is suggesting watching the growth and when they reach 3 cm remove the kidney. Is there anything else that can be done before they reach 3 cm? Doctor: Your concern is true but it is very difficult to adress it and given a very limited information , the only advice that can be given confidently is to visit a doctor"
},
{
"id": 23745,
"tgt": "What causes chest pain with headaches?",
"src": "Patient: Have a teenage son the age of sixteen whose pulse rate is 188 during excerise on a treadmill. He has MVP and reguiritation and has been having headaches and sharp chest pains which are mainly at night. Thickness of the wall of his heart is 1.7 what could be the problem ? Doctor: Hello , The pulse rate of 188 for intensive exercise on treadmill for a 16 year old is normal , if it's sinus . Mvp can explain all the symptoms you describe like chest pain , headache . So Mvp is completely being and is not going to cause a major problem is 95 % people ever , it does cause myriad of symptoms. Reassurance might help him . Do consult your doctor for consideration on starting a beta blocker in small dose which would help for symptom relief and also to prevent any progression of mitral regurgitation . Regards"
},
{
"id": 16412,
"tgt": "What could be cause and treatment for the recurring skin rash post a treatment for yeast infection a year ago ?",
"src": "Patient: Hello i am a 26yr old male and I had a problem last summer with a yeast infection and I had a full body skin rash from it.I got treatment and it went away and now I think it is coming back.I have red circle patches that are maybe a little bit smaller than a half inch on my side a few spots on my shoulder and back.Some of the patches are a little bit bigger and blotched together. I also have what appears to be acne on my inner thigh and groin area its a bump here and a bump there that itches.And on on my left inner leg I have a big pimple looking bump. My testicals also itch and the base of my penis is itchy sometimes to. But I dont have any sores or breaks in the skin on my penis or testicals.Also my testicals are very red. Thank you for your help and answers Doctor: Hi, You must have fungal infection on all the parts you described,this infection is known for recurrence. you apply anti fungal cream and take oral anti fungal tablet orally for one month. Ok and bye."
},
{
"id": 23883,
"tgt": "What causes headache while on medication for high bp?",
"src": "Patient: my dad is 70 yrs old.he has got a BP of 170/100.last month it was 160/90.recently complained of a mild shiver feeling.since then he has got a mild headache alternately on the two sides.at present he is taking (i) cilacar 10 mg(ii) zurig 80 mg (iii) thyroid 12.5 mg(iv) ketosteril 2 tab twice daily. for treatment of high creatine and uric acid.is the headche a side effect of the medicine he is taking? Doctor: Hi,The headache is rather caused by high blood pressure than by medications.He should take his medications and if he still has high blood pressure, he should visit his physician to revise the treatment to have a better controlled blood pressure, and mist probably headaches will disappear gradually when blood pressure is well controlled.Take care"
},
{
"id": 179505,
"tgt": "What does rattle not clearing through coughing after being treated for tonsilitis mean?",
"src": "Patient: My son has just noticed a rattle that won t clear through coughing. He has had no cough or illness sytoms, with fever . He has also just finished a course of penicillin for tonsillitis last week. He is 10? About 3 yr ago he had inhalers for a persistent cough during winter and had been told he could be borderline asthmatic but it was the one and only time he has ever needed them in his life ??? Thanks Doctor: Rattling sound perhaps is asthma. Asthma may get precipitated again, take proper treatment and don't link every thing to tonsillitis."
},
{
"id": 133290,
"tgt": "Can inflammation cause feelings of jaw displacement?",
"src": "Patient: hI,3 DAYS ago i had swelling/inflamation in my ankles,knees,shoulders as well as muscle fatigue and weakness.the symptoms have gotten better but now my jaw feels displaced and it hurts when I swallow.I initially thought it was a food reaction but now? For the record,I had my yearly bloodwork done 2 weeks ago and came back clean. Doctor: Hello,I can understand your concern. As you are having generalized weakness of muscles all over your body, it might be responsible for the jaw displacement feelings or pain while swallowing as muscles responsible for jaw movements and swallowing might be affected by the weakness. If you have been to the physician and have done the investigations, do you what is the cause of these symptoms? In addition, the problems of pain while jaw movements and swallowing must get better with improvement in other areas of the body. If that does not happen, then you should visit a dentist as these symptoms might be a part of some joint or oral cavity related problem which can be diagnosed by examination by the dentist.I hope this information helps you. Thank you for choosing HCM. Let me know if you have follow up questions.Best,Dr. Viraj Shah"
},
{
"id": 132769,
"tgt": "Suggest treatment for pain due to rib injury",
"src": "Patient: I slipped and fell on a ceramic tile floor 4 weeks ago. the majority of my body came down on my right rib cage. Now it hurts to go from a sitting position to standing, sneeze and sleep on my right side. The area on my side is tender to the touch. I was was diagnosed with a fatty liver in 09/2014. Shouldn t I be getting better than worse? Doctor: hiHope this msg finds u in good health. i have gone through your complaints and under stand your concern. u seem to have injured ur ribs and d soft tissues around. u should get an xray done to find out whats wrong. use rib belt.analgesics and anti inflammatory tablets.. local hot fomentation. avoid sleeping on injured part. nothing to worry about. hope your question has been answered. if u have any follow up queries, feel free to consult me anytime .thanks Take care god bless"
},
{
"id": 39478,
"tgt": "Is foot and mouth disease contagious?",
"src": "Patient: Good evening. My daughters had hand foot and mouth beginning last week, so about 12 days now for the earliest. My question is not about them really. My husband nor I have gotten the virus, at least we have shown no symptoms. However I work with a woman who is 8 months pregnant and is afraid to be near me or touch anything I have touched, for fear of her baby getting it. Should I keep my distance from her if I have no symptoms? I keep reading different lengths that the virus can be alive and that Lysol does not kill it. My kids seem fine, that have gone back to daycare. Can I give it to this girl? If I go to my primary doctor can they test me to see if I have it? I need to do something to be sure I can be at work. In need of advice! Sincerely, stressed out! Doctor: Hi and thank you so much for this query.I am so sorry to hear about this fifth disease that your daughter has. It is a contagious disease caused by a virus. It would have been advisable to avoid too much contact with her during this time. There is no need to go and get tested for this condition a there is no drug that kills the virus. The system is what masterminds and quarantine the virus. If you have no manifestations, you have nothing to worry about.I hope this helps. I wish you well. Feel free to ask for more information and clarification if need be and I will gladly respond to."
},
{
"id": 142548,
"tgt": "Could non functioning pituitary tumour be causing throbbing sensation in the head?",
"src": "Patient: I have been diagnosed with a non functioning pituitary tumor. Last night my head started hurting on the left side but it wasnt a constant pain. It will throb every minute or so...sometimes every thirty seconds. It is very painful and nothing I take is helping it go away. Could it be my tumor? Doctor: Hello!Welcome on Healthcaremagic!Headaches are common symptoms of pituitary gland tumors. But your headache characteristics is not typical of this disorder. The throbbing headaches, could be related to migraine or high blood pressure. Did you measure your blood pressure values?Anyway, if the problem persists, I would recommend performing a new brain MRI to evaluate if there is progression of the tumor and a visual field evaluation (because tumors in this region can cause visual field defects). A close monitoring of your blood pressure values is still necessary, to exclude blood pressure fluctuations, which are common causes of throbbing headaches. Hope you will find this answer helpful!Kind regards, Dr. Aida"
},
{
"id": 203430,
"tgt": "Can bacterial vaginosis cause small blister like bumps on the penis bumps and should medical treatment be sought?",
"src": "Patient: My boyfriend has discovered several small blister like bumps on his penis by the base...I went to the doctor immediately and found out that I have a bacteria infection..bacterial vaginosis could this have cause the blistering on his penis or should he seek medical treatment for other diagnoses Doctor: Hi,From history it seems that he might be having this infection due to ingrown hair follicle infection or folliculitis.This is also possible that he might got this infection due to your problem as well.He is advised to go for one antibiotic medicine course.Apply antibiotic cream locally.Keep local part clean and hygienic.Ok and take care."
},
{
"id": 87324,
"tgt": "Suggest treatment for severe headache and lower abdominal pain",
"src": "Patient: I'm having alot of symptoms all over the place. not sure if all have anything to do with one another but here it is.. I get CRONIC headaches at least every other day. I have a sharp pain in my lower abdomin, back pain, nausea, bloating and it almost feels like a baby is moving in my belly but I took a home pregnancy test and it came back negative. I also have the Mirena IUD Doctor: Hi! Good evening. I am Dr Shareef answering your query.All the precautions including birth control pills and IUD are not 100% fool proof in protection against a pregnancy. If I were your doctor, I would go for a serum HCG determination to rule this out and ultrasound of abdomen if need be. At the same time I would also go for a urine routine/microscopic test along with a culture sensitivity test to rule out an unsuspected aymptomatic UTI and treat it if positive. The head ache might not be related to the IUD, and could be a separate entity. I would do all the routine investigations including a blood pressure measurement and serum lipid profile and treat if any abnormality detected.I hope this information would help you in discussing with your family physician/treating doctor in further management of your problem. Please do not hesitate to ask in case of any further doubts.Thanks for choosing health care magic to clear doubts on your health problems. I wish you an early recovery. Dr Shareef."
},
{
"id": 215049,
"tgt": "How to make teeth yellowish to white ?",
"src": "Patient: my teeths r yellowish how to change them normalmy freins are advising me to use salt while brushing is tat good Doctor: hi prabhu rub your teeth by a cotton piece socked in mouth glycerin"
},
{
"id": 24962,
"tgt": "What causes the ups and downs in the blood pressure readings?",
"src": "Patient: i am 52 year old woman, overweight and trying to lose the weight but in the mean time i am racked with up and down readings from my brand new bp machine....i took a reading before exercising and it read 122/57 then after a 10 minute walk (a bunch of stair outside) and it was 152/75. that is quite a jump, isn't it. granted i did take it the second time right after the exercise...after a full 15 minutes later it is now 119/60. is this all normal? is there a place or slot i should fit into for women in their 50's ? Doctor: HiActually a reading of 152/75 after being active like walking isn't that much of a jump and really is fairly predictable.On the whole, your optimal blood pressure should average in the 110-120/70's range"
},
{
"id": 195825,
"tgt": "Will frequent masturbation reduce sperm count?",
"src": "Patient: Hello Sir, m 17 years Old and i have been masturbating since 3-4 years for ONCE in a Day. I want to ask IF this Regular activity of mine can affect my health or Sperm Frequency... OR can it cause any Memory problems cause i m AN Engineering student.. i Eat a lot. and LUCKILY my Weight and height are OK... but i m concerned about ma Memory loss due to masturbation if any????? Doctor: Hi,Masterbation is a safe way of satisfying your sexual desires and it has no side effects.Semen is produced daily in our body and masterbation will not affect memory or fertility. Hope I have answered your query. Let me know if I can assist you further.Regards,Dr. B. Radhakrishnan. Nair"
},
{
"id": 178887,
"tgt": "How to make infant take formula milk?",
"src": "Patient: Hi. My baby is 6 month old, she is breast feed from the beginning.i tried to introduce the bottle and formula for her but she won t take it..I tried different brands like naan and similac but she didn t seem to like any .i have introduced solids and she is taking it. But I am worried about her milk intake as I have less milk supply.. Please advice Doctor: Hi, you can give her milk based foods like curd , chacha, lassi, etc, try dairy milk if she takes it .Hope this answers your question. If you have additional questions then please do not hesitate in writing to us."
},
{
"id": 102226,
"tgt": "Any suggestion for recurring red, itchy, raised spots when suffering from allergies?",
"src": "Patient: I have red itchy, raised spots that keep appearing. They disappear and reappears in different places. I've had the chicken pox when I was young. I'm taking anti fungal pills and using creams but it keeps coming back. I do suffer from allergies from outside but it seems like maybe a food allergy. Any suggestions? Doctor: Thank you for your question!It seems your symptoms are due to allergy. Antifungals and topical creams will not help your condition.There are many triggering factors for allergy like stress, specific foods, exposure to dust etc. Take Cetrizine tablets twice daily for 10 days. If symptoms persists, consult the physician. Blood reports may be required to know the cause of your allergy."
},
{
"id": 74538,
"tgt": "Is vomiting bile,excess saliva and cold flashes normal during recovery from Gastritis?",
"src": "Patient: Hey Doctor, I was sent home 2 days ago from gastritis. However, I am still suffering from vomiting bile after meals. Recently, I started experiencing hot face and cold face flashes. Additionally, I started salivating a lot. I have been passing black stools for the past 2 days which I assume was the build up from the 4 days in and out of the hospital. Also, I have pain in the center of stomach form the sternum down? Is this normal during recovery? or should i head back? Doctor: Respected user , Hi Thanks for using Healthcaremagic.comI have evaluated your query thoroughly .* This is not normal recovery part of the gastritis , immediately consult your doctor again .Hope to clarify your query .Welcome for further questions .Thanks for using Healthcaremagic.com & giving opportunity to assistWishing fine recovery .Regards dear take care ."
},
{
"id": 213939,
"tgt": "I am feeling depressed about my health",
"src": "Patient: Sir, I am 30 years old I have married, I always feels that I will die but I went to the master check up but they advised that your health is fine but I could not change my mind it always struck in mind that i will die or i will become mental so please give suggestion for changing the minds and live happily. Doctor: Hello. It appears from your post that you get intrusive thoughts about dying or getting psychologically ill. What you need to do at this stage is to talk to someone about this. Also, you should consult a mental health professional (psychiatrist, psychologist) for a much detailed history and mental status evaluation. There might be underlying fears and anxieties which you need to explore and work upon. You might benefit from psychotherapy. Take care."
},
{
"id": 216332,
"tgt": "How to treat soreness and pain in the palm of the hand?",
"src": "Patient: I have bruised the palm of my hand on more than one occasion in the same general area. The bruising occurs at the base of my ring finger on the fleshy part of my palm and is very painful. Both times I only used my hand to grasp something which resulted in a visible bruise appearing immediately and soreness which lasts for days. Is this just bad luck or could it mean something is not quite right ? Thanks Doctor: Could be either. With ONLY one spot having bruises, then all the serious conditions involving blood clotting are just very unlikely. THEN, it's what causes irritation one spot. There can be just a mechanical reason. Usually is. The flesh is thin there, the area is exposed. Frequently there is also a bony part (bone spur)"
},
{
"id": 51694,
"tgt": "Why am I having pain on the right kidney as stone was detected on the left kidney ?",
"src": "Patient: detcted kedney stone in left kedney two mm size but my pain was on right side. doctor suggest me neeri tabs for two months. Is it any relation on right side pain? right kedney have no stone. Doctor: Welcome to Healthcare Magic Good Day If there is not stone in the right side there must be some problem with the liver or gall bladder. Any jaundice. Indigestion, gas, loose motion. It could also be musculoskeletal. Try applying Volini gel at that area. If pain reduces then it is not trouble."
},
{
"id": 63555,
"tgt": "What causes painful lump between anus and vagina?",
"src": "Patient: Hi I have a painful lump the size of a gumball between my anus and vagina. It started about a week ago as a pea size. Now it is getting bigger and it hurts when I touch it. I also have a small rash around that area.(not sure if related) What could this be? Doctor: Hi, dearI have gone through your question. I can understand your concern. You may have hemorrhoid ( piles ). You should go for examination. You should take plenty of water and high fiber diet. Avoid constipation. Local anesthetic gel can help for pain relief. Permenent treatment of choice is surgery. Consult your doctor and take treatment accordingly. Hope I have answered your question, if you have doubt then I will be happy to answer. Thanks for using health care magic. Wish you a very good health."
},
{
"id": 207470,
"tgt": "What causes boaderline personality disorder?",
"src": "Patient: I am studying BDP (Boaderline Personality Disorder) and am starting to believe that I also suffer from the syptoms. I am aware that it is common for hypochondria to form in the health industry so I am being pesamistic but would like a second opinion non the less. Doctor: When one is reading medical literature or psychiatric conditions sometimes reader feels that he or she is also suffering from that disorder.Borderline Personality Disorder is conceptualized in various ways by different theorists. ICD and DSM do have specific criteria. In my opinion, if you are not feeling considerable mental distress or problems in work, family or social sphere, there is no need to worry.If you are facing difficulties or have significant mental distress, then you should consult a psychiatrist. You should feel free to tell him what you have read and how you feel it applies to you. The psychiatrist will evaluate your condition and suggest interventions.I hope this answers your question."
},
{
"id": 209100,
"tgt": "Suggest methods to combat stress",
"src": "Patient: One month ago I started feeling light headed and blurry vision that would come and go and also my blood pressure would go up. I went to the doctors to find out what could be causing these problems. The doctor had me do some blood work. Everything came out normal except for my bun/creatinine ratio was a little high. He then told me that my problems are possibly caused by stress. I am the kind of person that stresses about everything so I decided to take it easy but a few days later I started to get pressure on both sides of my neck that goes up to my jaw and the light headed and blurry vision seems to be getting worst. Were the pressure in my neck is I have found to small movable balls. One is right under my jaw and the other one is on the side of my neck. Doctor: HiThanks for using healthcare magicI think, you have anxiety disorder. In that case, you need antidepressant with low dose benzodiazepine. That would help you to control anxiety symptoms. Better to consult a psychiatrist with proper diagnosis and treatment. You can also try some relaxation exercise that would keep you relax and calm. In case, you need further help, you can ask.Thanks"
},
{
"id": 34118,
"tgt": "Suggest treatment for sinus infection",
"src": "Patient: Been on antibiotics because I felt exhausted, just couldn t think, yawned a lot, lots of sinus drainage, runny nose and eyes once in awhile, some wheezing and coughed up fluid once. Now feel better after being on antibiotics for 5 days (one dose left), have a bit more energy until I stand up, have an intermittent dry cough, feel the need to yawn, wheezed a lot today so used an inhaler today, chest feels tight. Just coughed and felt really dizzy...was sitting down fortunately. Thanks...I don t have $15.00... Doctor: For sinus infection you have to take some antibiotic such as Cap Ampoxin 500 mg or Tab Augmentin 625 mg three times daily after meals with Tab Wikoryl or Tab Diominic-dca three times daily after meals for running nose with Tab Rantac or Tab Famotid two times daily empty stomach. If running nose is very severe than you can use Otrivin or Nasoclear nasal drops. Another thing what you have to do is to take steam inhalation at least three to four times daily. It is very important in case of sinus infection. You have to be careful enough because if you are not following the instructions properly it will progress to chronic sinusitis which is somewhat more troublesome and irritative."
},
{
"id": 224950,
"tgt": "Is yamini the correct birth control pill and will there be any side effects?",
"src": "Patient: Hi, I am 24 years old, I have been prescribed Yamini as a birth control measure. I have acne, unwanted hairs but my periods have been regular. I am 5.4ft and weigh 73.3 kg. Is yamini correct for me or should I go for some other oral-contraceptive. Am I sure to gain weight and loose hair if I take yamini? Doctor: Hi,Welcome to HCM.Yamini is correct pill for you. It will reduce acne & unwanted hair. But along with it you will have to try some weight reduction program to increase its efficacy. It will take at least 3-6 months for its good effects to be visible, so be patient.Regards."
},
{
"id": 11531,
"tgt": "Suggest treatment for pigmentation and scars on face",
"src": "Patient: hi doctor,im having acne comendones,its moderate.i have already tried retia a and clindamycin but not gettin relief.i am 21 yrs old,male.im also having acne scars and some tiny black pigments like mole.what can i do to get relief from these pigmentation and scars Doctor: Hello,Welcome to healthcare magic.I understand that you are having comedones and acne scars, along with black mole like lesions.If the comedones are not responding to topical gels, you may benefit from getting comedone extraction procedure done by a dermatologist. This will hasten the clearance of the lesions. You could continue the gels you are currently using while you do this procedure.Acne scars can be treated by procedures like microneedling and lasers. However, this can be started only after the active acne has cleared.The black, mole like spots on the face could be lesions called as \" Dermatosis papillosa nigra\". This can be removed by electrocautery procedure done by a dermatologist.Hope this helps you.Take care."
},
{
"id": 82463,
"tgt": "What causes internal pain on the right side of the chest?",
"src": "Patient: Dear respected senior Doctr, one of my patient got internal pain on the right side of the chest. Even there is no any reason of fracture ? Sign and Symptom of the patient. *lost of apetide *headaches. *billy pain *breathing problem. Pliz do a helpline for me Doctor: Thanks for your question on HCM.In my opinion you should get done chest x ray first. As all the symptoms you are mentioning are favours lung infection.So chest x ray is needed to diagnose pneumonia.If this is normal than get done PFT (Pulmonary Function Test) to rule out bronchitis.So better to consult pulmonoligist and get done chest x ray and PFT to rule out pneumonia and bronchitis."
},
{
"id": 78589,
"tgt": "What causes pressure and tightness in the chest region?",
"src": "Patient: Hello Doctor, Thanks for your time. I have been having some pressure / tightness in my chest region lately. I do not smoke or consume alcohol etc. I however sit for extended periods of time, as I am in IT industry. Could my occupation be causing this pain? Doctor: Thanks for your question on Health Care Magic. I can understand your concern. Your occupation has no direct connection with your symptoms. You are having sedentary lifestyle (prolonged sitting). And sedentary lifestyle can cause heart diseases, Heart diseases can cause chest pain, pressure and tightness. So better to get done ecg and 2d echo. If both are normal then no need to worry for heart diseases. Sometimes gastritis and GERD (gastroesophageal reflux disease) can cause similar symptoms. So avoid prolonged sitting. Walk or change your position after some time (1-2 hours). Start proton pump inhibitors. Avoid stress and tension. Avoid hot and spicy food. Don't worry, you will be alright.But first rule out heart diseases. Hope I have solved your query. Wish you good health. Thanks."
},
{
"id": 3087,
"tgt": "Is there any way i can help releasing the egg for an implantation?",
"src": "Patient: hi how are you.i had just ovulate on sunday me and my husband has been very intimate everyday since dec,22,2013,and my question is there any way that i can help the relases of my egg been implant health into the uterine wall to increase pregancy.please help me with a answer .if possible thank you Doctor: there is no such thing madam..it's all God's grace for all that to happen so just wait and hope for the best"
},
{
"id": 114105,
"tgt": "I was suffering from slipped disc. what should I do now ?",
"src": "Patient: I was suffering from slipped disc 15 days back which was painfull and also had experienced numbness in ma left leg.And then i went to the hospital and was on traction for 5 days and was under complete bed rest.Later the doctor advised me to go for physiotherapy which i did.And now i m doing ma excersises regularly and i m feeling much better right now.But i m still experiencing some kind of numbness in ma left toe and ma right thigh.So what should i do now ? And as i am a sports person,a badminton player,when can i get back to ma play ? please suggest some remedies doctor. Doctor: \u2018\u2019 Numbness can be definitely covered by homeopathic medicine. You should take Colocynth 30 ,6 pills TDS. you should ask your doctor or physiotherapist regarding your play & exercises. Wish you good health"
},
{
"id": 136361,
"tgt": "Suggest treatment for severe pelvic pain",
"src": "Patient: I ve a really bad pain above my bikini line, it feels like it splitting in two. When lying on my back trying to get up or even turning in the bed stops me in my tracks. I ve also had pain coming and going on my right side, like when I cough! I also feel contisaped and I suffer from UC Doctor: You are suffering from a systemic inflammatory diorder of ankylosing spondylitis and bilateral sacroilitus is the cause of u r pain.Get hlab27 done. X ray of pelvis with both Si joint is required. And anti inflammatory medications with etrococcib and indometahcin are helpful."
},
{
"id": 32530,
"tgt": "What causes small bumps all over the body?",
"src": "Patient: I'm 21 years old and have had chicken pox. I feel like I have them again, I first thought that I was allergic to something but the bumps arent going away. They are on my stomach, legs and feet now and have been spreading/I've been getting more over the past 2 days. I also have a fever... I do not have insurance and was wondering if there was anything I could do at home to get healthy as fast as possible. Doctor: HiThanks for your query at HCM.I understand your concern and situation.It seems like you have acquired viral infection. But usually the infection are self limiting and resolve within a week. You must visit the physician and get haemogram and serological and molecular tests to rule out certain common viral infection. Take plenty of rest, drink plenty of water, acetaminophen if fever. Eat protein rich balanced diet with multivitamins and calcium supplements.Hope I answered your queryDr Sheetal Verma"
},
{
"id": 196527,
"tgt": "What causes a rising PSA with arm and shoulder pain?",
"src": "Patient: My PSA has risen in less than a year from 0.9 to 3.88.\u00a0\u00a0It has never been higher than 0.9.\u00a0\u00a0Urologist has put me on an antibiotic, and wants to recheck in six weeks.\u00a0\u00a0I have also been having arm and shoulder pain, muscle twitches all over, some shakiness.\u00a0\u00a0Is it possible that I could have cancer, considering the short timeframe, or do you think it is just a bad infection?\u00a0\u00a0I am 45 years old.\u00a0\u00a0Could a prostate infection affect nerves in the body elsewhere?\u00a0\u00a0My Neurologist thinks I have a Post Virus Syndrome attacking my body but this was before the PSA test..\u00a0\u00a0He sent me to Urologist when I started having symptoms- hurting in that region and leakage after urination. Member Comments (1) by RC03, Jun 24, 2011 04:59PM More information: digital exam was normal.\u00a0\u00a0Neurological symptoms came on suddenly six months ago. Clear MRI brain and Cervical spine.\u00a0\u00a0clear EMG as well.\u00a0\u00a0I am on Cembalta for nerve pain.\u00a0\u00a0My PSA was the first I have had since Aug. 10.\u00a0\u00a0Dr. was looking for MS type illness. Doctor: HelloThanks for query .The pain in shoulder and arms can not be due to prostate infection .Your PSA levels which has raised from 0.9 to 3.88 can be considered as a normal variation as it has not raised above 4 which is considered to be a land mark to suspect cancer of prostate .The raised PSA is likely to be due to infection of prostate (Prostatitis) and can be controlled with long term course (6 Weeks) of appropriate antibiotic .As regards your doubt about MS it is most unlikely as your MRI brain is normal ..The possibility of generalized Viral Neuritis can be a considered and taking Methylcynocobalamine and other vitamin supplements should help to get relief from pain in arms and shoulders ."
},
{
"id": 109697,
"tgt": "What is the pain in the mid back after lifting weights?",
"src": "Patient: This problem happened to my friend years ago after doing heavy yard work. On Sat. last week he lifted weights...something he normally does. For a couple of days now it's been hurting him in the middle of his back whenever he eats. Did he pull a muscle or what? Doctor: Hi,Thanks for your query. After going through your query I came to know that your friend got pain in middle of back , it hurt only when he eats. It does not not seems to be related to muscle or bone. As it is related to food, it is gastrointestinal pain likely radiating to back. It is likely to be due to acidity. Pantaprazole before meals is likely to help. He should not eat foods and fried food. He can be further investigated by doing ultrasound of abdomen.You can discuss with your treating Doctor about it. I do hope that you have found something helpful. If you have additional questions or follow up queries then please do not hesitate in writing to us. I will be happy to answer your queries. Wishing you good health. Take care."
},
{
"id": 108000,
"tgt": "Suggest treatment for acute lower back pain",
"src": "Patient: Hi, I seem to have acute pain on my low back, positioned towards the right hip bone. the pain initially started on my left side and changing postion, walking and exercizing does help. However, it tends to become stiff/frozen making it very difficult to flexibily move. It hurts more when I am seated. Kindly help. Thank you. Doctor: Hi,Thanks for writing in to us at hcm.Yhere are various causes to acute lower back pain. The following medication can be useful: Tab diclofenac 50mg twice a day after food for three days.Did you have any history of fall ?If the pain doesnt subside with the above medication kindly visit an orthopaedician for examination and Xray.Regards,Dr.Riyanka"
},
{
"id": 185122,
"tgt": "What causes bubble formation in between teeth while on braces?",
"src": "Patient: Hi, well I have noticed a bubble forming in between my teeth. I also have braces so I'm worried that it will affect that. I'm 11 going on 12 in 5 days. I showed my parents this and they said its nothing. My next dentist appointment isn't until a month. Should I be worried. P.s I brush my teeth twice a day and sometimes floss (About once a week) Doctor: Thanks for your query.Read your query.The bubble which you have mentioned is not clear whether you are speaking about your gums or any decay in between the teeth.If you mean the bubble in gums , I would suggest that is inflammation of the gums usually due to deposits and calculus. Visit your dentist and get it cleaned and later on maintain it well.If it is decayed tooth then get it filled as soon as possible.You can call your dentist office and take the opinion of your dentist and if necessary visit him and get it done earlier than your appointment date.Thanks and regards."
},
{
"id": 12993,
"tgt": "Cause for the rash?",
"src": "Patient: Hi, I had a rash that is slowly going away for about 3 weeks, not sure if it is related or not. Now I have a a very severe cough to the point of puking, I was very light headed for a few days, I have felt very weak and tired, I'm have a harder time breathing, have a bad taste in the mouth that doesn't go away. Any idea what it may be? Doctor: Hi, Both these symptoms could be related. It seems to be related to a viral infection which can cause both the rash as well as other symptoms. Hope I have answered your query. Let me know if I can assist you further. Regards, Dr. Asmeet Kaur Sawhney, Dermatologist"
},
{
"id": 173671,
"tgt": "Will pupil size change after wearing spectacles?",
"src": "Patient: my daughter is 4 yrs old and she has cylindrical power -3.5 in both eyes. she is wearing glasses from 10 days. but it was observed that her pupil gets changing in size i have been observing this from one week ,i.e after wearing glass.... is this normal, or should i immediately go to doctor? kindly advice Doctor: Hi,Thanks and welcome to healthcare magic.whether the spectacles are used or not pupil size will be changing constantly depending on the external light.If the light is more the pupil contracts or if the light is less or dark the pupil will dilate .Hope this answer serves your purpose .Please feel free to ask further queries if any.Dr.M.V.Subrahmanyam."
},
{
"id": 126536,
"tgt": "How can a painful lipoma in the upper arm be treated?",
"src": "Patient: I have a lipoma on the top of my left wrist. There is a bruise at the same spot. the bruise is small and has been in the same spot for several months. I also has a scan that showed a sac in my right upper arm. Mid arm on the outer side. It inhibits the movement in my arm. I have arthritis (Psoriatic arthritis ) also. Now a lipoma had come up on my right fore arm. The 2 lipoma do not hurt. But the sac that showed up on my scan is painful. advise or concern ? Doctor: Hi, You can consult an orthopedician and get it removed. Lipoma is a harmless condition and does not cause much harm. Hope I have answered your query. Let me know if I can assist you further. Regards, Dr. Shinas Hussain, General & Family Physician"
},
{
"id": 168379,
"tgt": "What causes redness and swelling in scrotum?",
"src": "Patient: Hello, I noticed yesterday that my six year old child s scrotum has turned red and swollen. He feels slight discomfort on touching it. As well he is walking with his legs spread out to avoid it being rubbed again his thighs. He is otherwise well. I applied some polysporin cream on the scrotum. Since last night, the scrotum appears less red and swollen. Could you please let me know what is wrong with him and how to treat the illness? Doctor: Hello. I just read through your question.It is possible there is a mild infection not the area I recommend consulting with your doctor so the correct diagnosis can be made and the appropriate treatment plan implemented."
},
{
"id": 160679,
"tgt": "Is peeling skin around the tip of penis of an infant normal?",
"src": "Patient: I have a two month old son, recently his skin around the tip of his penis is peeling away, the best way to explain it is that it look like it has been ate around it! Is this normal? I had him at the doctors already because it looked red raw and he said he will more than likely need to be circumcised when he is 2 or 3 as he said it looked very tight. Doctor: Hi,If it is not hurting, and there is no active discharge or bleeding, you need not worry now. Some babies develops excoriation / superficial ulceration there from frequent wetting. Just keep the area clean and dry. I used to give local antibiotics like fusidic acid for local application twice daily in such cases- this will prevent secondary infection and promote healing.Regarding tightness, it is very common at this age- known as physiological phimosis, and usually will disappear its own by 2-3 years of age. So if the urine stream is normal(not abnormally narrow/broken), nothing to worry. Avoid forceful retraction , as this causes scarring and adhesions.Hope I have answered your question. Let me know if I can assist you further. Regards, Dr. Muhammed Aslam TK, Pediatrician"
},
{
"id": 208099,
"tgt": "What causes horrible nightmares?",
"src": "Patient: In March of 2014 I fell on the ice. In May of 2014 I had rotor cuff surgery. I had one tendon completely torn loose, two others ripped and one bi-cep shattered. The tendons were repaired and the bi-cep was cut loose. Since the accident I have experienced horrible nightmares. Especially on nights when my pain level is high. Can these nightmares be because of the pain? Doctor: DearWe understand your concernsI went through your details. I suggest you not to worry much. Yes. You are right. The night mares were due to the agony and pain you were in. The nightmares should be discontinued around three to six months time. Just try to ignore them.If you require more of my help in this aspect, Please post a direct question to me in this website. Make sure that you include every minute details possible. I shall prescribe the needed psychotherapy techniques which should help you cure your condition further.Hope this answers your query. Available for further clarifications.Good luck."
},
{
"id": 29707,
"tgt": "What causes discomfort during urination while on Amoxi-clav?",
"src": "Patient: I am on amox-clav 875mg for pneumonia. I have been on this for a week. However, I am developing discomfort similar to what would be a UTI. My urine is the color of tea. Is this normal. My urine is not cloudy - just occasional discomfort and the dark urine. Sometimes I feel as if the amount of urine is less than what it should be. I am staying hydrated due to pneumonia but something doesn t seem right. Thank you for your help. Doctor: Dark colored urine is a known serious side effect of using Amoxclav.I recommend stopping th drug immediately and visiting your doctor ASAP. A renal function test may be required to check if there is some abnormality in the functioning of your kidneys.Hope this answers your question, thank you."
},
{
"id": 118153,
"tgt": "Any suggestion for glucose level of 113 with family history of diabetes?",
"src": "Patient: All my blood work panel is normal range except Glucose reading is 113. I am 60 years old about 220lbs. 5 9 . I have diabetes on both sides of family, all grandparents, parents that have had diabetes have lived into their 90 s with long term diabetes, so I pay attention to this higher then normal reading. Is this to be alarmed about or pay attention thru diet and weight loss? Doctor: Dear, I hope this is a fasting level. It is mild increase. At present you do not need any medication. You need to reduce your carbohydrate intake and at least a daily good walk. This will burn your extra calories. Since you have bilateral history it is good that you keep a periodical ceck on it. Dr. Ticku"
},
{
"id": 49520,
"tgt": "Kidney test reveals bilateral renal calculi and small vesical calculus. Is it normal?",
"src": "Patient: SirKidneys both are normal in size ane echotexture.Rt kidney 10.6X5.4cms. 4mm calculus noted in middle calyx, no hydronephrosis.Lt kidney 10.3X5.5cms. Same to rt kindey.Bladder 5mm calculus noted .Impression;- BILAYERAL SMALL RENAL CALCULI.SMALL VESICAL CALCULUS. Doctor: Hipresence of stone in kidney and bladder can not be normsl and that answers your queryBladder stone may pasd out without interventionThe other stone may require surgical intervention bTake care Dr Lal Psychiatrist"
},
{
"id": 116985,
"tgt": "Should I be concerned about a high RDW value?",
"src": "Patient: I had a CBC & Diff blood test done. Everything came back within normal ranges except RDW which was 15.3 My doctor didn t even address this because my tsh level was high and that is what she discussed with me. Should I be concerned about the RDW result? Doctor: Hi,Thanks for asking.Based on your query, my opinion is as follows.1. RDW indicates red cell distribution width. It indicates variation in size of the cells.2. If the hemoglobin is normal, not to worry.3. RDW varies with iron deficiency or thalassemia. If hemoglobin is low, it indicates possible etiology. 15.3 is only mild variation. Not to worry.Hope it helps.Any further queries, happy to help again."
},
{
"id": 78814,
"tgt": "What is causing chest pain at irregular intervals?",
"src": "Patient: I walked into the corner of a car door about 3 weeks ago I am female. Ever since then the right side of my chest is painful when I move certain ways and sometimes I don't feel it at all. Its not a sharp pain its a dull pain that is present sporadically. Doctor: Thanks for your question on Health Care Magic. I can understand your concern. Intermittent, dull aching chest pain should be first evaluated for cardiac diseases. So better to get done ecg and 2d echo. If both are normal than no need to worry much. Your chest pain is associated with movements. So musculoskeletal pain is likely. So avoid movements causing pain. Avoid heavyweight lifting and strenuous exercise. Avoid bad postures in sleep. Take painkiller and muscle relaxant drugs. Apply warm water pad on affected areas. Don't worry, you will be alright. But first rule out heart diseases. Hope I have solved your query. Wish you good health. Thanks."
},
{
"id": 47481,
"tgt": "What causes sever pain and blood in urine after kidney stone removal?",
"src": "Patient: If I have severe pain after having a kidney stone removed on Monday and a stent placed, the stent was removed yesterday and I am doubled over in pain off and on, now it has been going on for the last 3 hours. What would you advise? I also have blood in my urine. Doctor: Hello and welcome to HCM.As an Urologist, i assure you,many experience this after stone removal.The pain is mainly due to spasms of the ureteric muscle, and bleeding from the stone fragments, rubbing against the congested walls of ureter.The pain can be relieved with analgesic- antispasmodics, like Dicycloverine -Paracetamol tablets, twice or thrice daily, with 12-15 glasses fluids daily.If you've severe pain and bleeding, report to the hospital, for an injection.If you want my expert opinion, on any doubts, send it as a direct question.Dr.Matthew J. Mangat."
},
{
"id": 58244,
"tgt": "Have fevers off and on, stomach issues, loss of appetite, night sweats, extreme fatigue. Have ultrasound result. Can any online doctor help?",
"src": "Patient: My 22 year old son has lost over 65lbs in past six months. Has had fevers off and on, stomach issues, loss of appetite , night sweats , extreme fatigue, feet neuropathy . He s had blood tests and ultrasound done shows fatty liver, gallstone (dr says from losing weight it s not symptomatic), anemia , enlarged liver & spleen , elevated liver enzymes . GI dr said its all from alcohol abuse . He has drank alcohol in excess at times, but nothing for months now. I can t believe that he did all that damage in few years of drinking. He s maintained a job and doesn t drink daily. His labs were neg for all Hep, HIV. Chest xray normal. White cells normal. Red low = anemia. I just feel like once the GI dr heard he drank that was all he considered. We ve moved him back home, he s on medical leave from work and just not improving. I ve been giving him Ensure and as many calories as possible. Why is he still having fevers? Last night it was 101.4. What other tests should he have done? Any ideas what s the problem? Why are his leg pains so bad. Should we take him to hospital?? GI gave him 20mg of Elavil, which is no help. He s also taking Nexxium to tolerate food better. On top if this the poor guy had 2 wisdom teeth out last month, one didn t heal so there s hole to sinus which needs repair. DR doesn t want that done until he s healthier. He has had a week round of antibiotics for that due to small infection. Yet the fevers persist. Doctor: Hello!Thank you for the query.Symptoms described by you are most likely caused by some chronic disease. Leukemia and Hodkins Lymphoma should be considered at first. Chronic pancreatitis is a next possibility.I suggest you to take him to the doctor once again. Blood work should be repeated and abdominal CT done (it will tell if there is any chronic pancreatitis or not). He should also consult hematologist.Hope this will help.Regards."
},
{
"id": 188327,
"tgt": "White, raised bump near teeth. No infection, soreness, discomfort. Which type of doctor to see?",
"src": "Patient: I have a small round white raised \"like\" bump located inside the mouth on the inside area of the gum next to my lower teeth. It is located near a newly prepared implant. The dentist has made x-rays and has taken the crown off of the new implant to make sure there is no infection in the implant area. None was found. The white bump is not sore or uncomfortable at all. If the dentist doesn't know what it is - what other doctor should I see about this? I'm concern since my dentist does not know what this is that it may be cancer. Doctor: Hello welcome.Please do not worry,if the lesion was cancer,your dentist would have definitely informed you.Sometimes there could be unexplained tissue reactions which might or might not progress.you must keep a closer look on the lesion.Further you can meet your implantologist for help in this regard.Hope it helps.Thanks.Take care."
},
{
"id": 41831,
"tgt": "When will my follicle rupture and how soon can I undergo IUI?",
"src": "Patient: age 39/5 feet 4 inches/67 Kgs I am undergoing treatment for primary infertility Currently undergoing A follicular study ,,today being the 13th day of my cycle .I have my Endo as 7.8mm and ro has follicle measuring 13x10mm and 15x10mm and lo has follicle 16x10mm and 18x10mm..all I am concerned is when would the follicle rupture and how soon can I undergo IUI as adviced .....and are the readings fine enough to undergo the procedure ??? Doctor: Hi welcome to healthcaremagic.I have gone through your question.Your follicular study is fine Absoultely normal. Follicles rupture generally when they get upto 19/20 mm, on 14/15 th day.So you can go for iui intrauterine insemination.Hope i answered your question.Would be happy to help you further.Take care."
},
{
"id": 50698,
"tgt": "Blood in urine due to prostate infection. Perforated kidney. Chances of bladder cancer?",
"src": "Patient: I have just seen a uroligist for blood in my urine. I have been under a doctors care who said I had a prostate infection. I have been on antibiotics for 6 mo. I had kidney stones about 5 or 6 yrs. ago, I passed one but the doc then said my kidneys looked like they have been shot with no. 8 bird shot. could this be my kidney stones acting up again or possible bladder cancer which kinda matches up with my symptons? Doctor: Hi and thanks so much for the query.I feel humbled being offered the opportunity to counsel on your health worries.I am so sorry to hear about the blood in your urine.Urine is produced in the kidneys and it flows through the ureters, get stored in the bladder and finally through the different parts of the urethral including the prostatic portion.Blood in urine can come from any of these organs. Kidneys/ureter/bladder/prostate/urethra etc. It is very non specific in localising the exact problem but at good telling us to dig further and get answers.We need to characterise this blood in urine...Quantity...all urine? Look for other urinary symptoms, abdominal pains etc before narrowing the list of options. Also, your age is going to be of great help.For now, it can be any of the above but until you get evaluated by your urologist, we shall continue to list even more. Please, you need a proper consultation and investigations to give you the exact answer.Hope this helps.Wish you good health."
},
{
"id": 104825,
"tgt": "Shortness of breath, high fever, chills, difficulty swallowing. Asthma symptoms?",
"src": "Patient: Hi.. I woke up with an extreme shortness of breath and a very high fever. My eyes feel hot when I close them and I am constantly getting chills. I am always cold even when in multiple blankets. I have asthma and I think it made it worse. My throat I very swollen and its hard to swallow . In having chest pains as well.. What could be wrong? Doctor: Hello, Welcome and thank you for posting in HCM Per your description you are suffering from an respiratory tract infection and an associated pharyngitis (inflammation of the pharynx). This affections are typically caused by viruses (but with very moderate fever 37.5-38.5 degree Celsius) but when fever is very high with swollen throat (tonsils, uvula and pharynx ) it is generally bacterial infection ( strep). Having respiratory signs like shortness of breath, chest pains etc typically indicate a respiratory affection which could be an extension of the germs in the pharynx. For diagnosis, a rapid streptococcal test or a throat swap + culture have to be done to rule out possible bacterial infection. Chest x-ray is also important to rule out pneumonia. Your asthma symptoms are present as a result of the affection which has provoked the crisis since you are a known asthmatic and microbes affecting your respiratory tract might induce asthma symptoms and even exacerbate them. You will need to see your doctor soonest for prompt management before it gets any worst. Steroidal/non-steroidal antiinflammatory, antipyretics and antibiotics would have to be administered either orally or intravenously depending on the severity of your condition. Bronchodilators would also need to be administered. I believe you will require emergency medical attention. Hope this helps"
},
{
"id": 175311,
"tgt": "Should RBS be constantly monitored in premature baby with hypoglycaemia?",
"src": "Patient: I had a premature baby with Hypoglymecia detected at birth and he was under doctor observation wherein they applied some glucose and his RBS levels raised to normal in next 5 hrs. I wanted to know, if i have to care of monitoring his RBS regularly or it is cured completely Doctor: Thanks for consulting in Healthcare Magic. I congratulate you with a baby. Actually, premature babies are often have anaemia, because they didn't receive enough iron from mother. You should control hemoglobin, iron,calcium at your baby at least 1 time in month, pay attention to color of her skin , is it yellow or pale. Breastfeeding is golden standard for baby. Mother sshould also eat food, which reach of calcium- dates,Apple's, popomegranate, beets,carrot and proteinsBest regardsDr.Svetlana Shrivasta"
},
{
"id": 124799,
"tgt": "Does ovoid plaques cause unexplained dizziness ?",
"src": "Patient: I ve suffered from unexplained dizziness episodes for 7 yrs now and just discovered if I turn my head slightly to the right, it makes it go away. Have had many tests done including MRI & MRA of brain and a spinal tap. The MRI showed ovoid plaques in/on left and right frontal lobes but spinal tap did not show anything. Was just wondering if the dizziness is related to these ovoid plaques or possibly my torticollis? Doctor: Hello, Those plaques might partially obstruct the blood flow to the brain and leads to the symptoms. Consult a physician to rule out other causes also. Hope I have answered your query. Let me know if I can assist you further. Regards, Dr. Shinas Hussain, General & Family Physician"
},
{
"id": 134732,
"tgt": "Suggest remedy for pain and swelling in wrist",
"src": "Patient: I had my trapezium removed 8 weeks ago, i have had swelling the entire 8 weeks. I has an ultrasound to check for a blood clot that was negative. My wrist is very ridged and moves very little. I have had 2 therpy sessions, I m still in a splint. Are these post op complications normal? Doctor: you need good amount of physiotherapy and ultrasound with intermittent splint support. ice application will be benifiscial"
},
{
"id": 85435,
"tgt": "Is coughing a side effect of trigestrel?",
"src": "Patient: I was changed over from Thriphasil to this Trgestrel a bit more than a month ago by our clinic sister. SInce then I also had the bloody discharge and also never stopped coughing. I have been to the the doctor twice for antibiotics. I just want to know can the coughing be a side effect or not? Doctor: Hi, Usually not. Side Effects are nausea, vomiting, headache, bloating, breast tenderness, swelling of the ankles/feet (fluid retention) or weight change may occur. Vaginal bleeding between periods (spotting) or missed/irregular periods may occur, especially during the first few months of use. Hope I have answered your query. Let me know if I can assist you further. Regards, Dr. Ajeet Singh, General & Family Physician"
},
{
"id": 94167,
"tgt": "Has vomiting, diarrhea, tiredness, arms and leg pain. Serious?",
"src": "Patient: I have a 5 year old who woke up Monday throwing up. She only threw up once but has had diarrhea for the last couple of days and has been very tired. Today she woke up complaining of arm and leg pain and we carried her down the stairs. She ate a good breakfast this morning which was the first solid meal she has had in a few days. She did walk from a little after that but is very tired. Is this a simple stomach bug or something more serious with the arm and leg complaints? Doctor: Hi, Thanks for posting your query. Do you have fever/ spasmodic abdominal pain/ nausea or vomiting/ acidic belching? With the available described symptoms, there could be possibility of acute viral gastroenteritis. Myalgia is an integral prodromal symptom of viral infection. You should consult with internal medicine specialist/ pediatrician and should go for thorough check up. You should also go for complete blood count, ultrasound imaging, and serum electrolytes. Viral infection has also some effect on liver and that's why loss of appetite and fatigue is very common with viral infections.You should take complete antibiotic therapy, antispasmodics, probiotics, antipyretics. You should maintain your hydration by taking potassium rich liquids. Avoid milk and all milk related products. Take care, Dr. Mayank Bhargava"
},
{
"id": 197851,
"tgt": "What does the sperm culture report indicate?",
"src": "Patient: H, may I answer your health queries? Please type your query here...Hi doctors..my semen culture reports says\"Normal Microbial flora grows on culture.i am trying for baby for last 1 year ,doctor recomed tests.sperm count is 35 million/ml .plz suugest wat to do???are the reports are normal.... Regards lalit sharma Doctor: Hello Lalit and .As an Urologist, i can fully understand your concern.1.A semen culture reported normal, means, there's no infection in it.2.A minimum sperm density of 20 million/ml,is enough for pregnancy.3.Other parameters of semen test, like motility,morphology, aren't available.4.So, repeat the semen test, at a reputed lab, and send the report.5 An ultrasound scan (doppler) of scrotum, is advisable.6.FSH,LH,testosterone,and sugar also needed.Get examined by an Urologist.You may send the reports to me,as a direct question,for an expert opinion.Dr.Matthew J. Mangat."
},
{
"id": 160941,
"tgt": "What causes vomiting in a baby while suffering from cough?",
"src": "Patient: Hi my 11 months old baby vomits whenever he has cough. Excactly two weeks ago he had a cold and was prescribed penicline. He has been vomiting eversince at least once a day, his cough is not so bad right now so why is he still vomiting? should I take him to the GP? thanks Doctor: Hello, A severe cough might induce spasm and lead to vomiting. Generally, it will settle once the cough improves. If symptoms persist better to consult a pediatrician and get evaluated. Hope I have answered your query. Let me know if I can assist you further. Take care Regards, Dr Shinas Hussain, General & Family Physician"
},
{
"id": 149689,
"tgt": "Bout of severe constipation followed by pain in upper right quadrant, numbness in right side of face, given Morphine, Reglin. Any suggestions ?",
"src": "Patient: My issues first started in December with a bout of severe constipation followed by pain in upper right quadrant. I went to my Dr who ordered an X-ray of my abdomen and found that my Spleen was enlarged (16 mm), blood work showed only slightly elevated Bilirubin and RBW. Dr ordered a CT scan with contrast of abdomen and chest which was completed 2/1/13 (follow-up scheduled for 2/4/13). Soon after CT scan I developed a headache that progressively worsened throughout the day and kept me from sleeping that night. At one point it was so intense I was certain I was seconds from having a stroke . The next day I had numbness on the right side of my face and down my right arm but the headache, although still there had lessened in intensity briefly but returned as the day went on. My wife insisted on taking me to the ER. My BP was 154/101 on arrival, I was given Toradol, Morphine and Reglin and had a CT of my head and sent on my way as they found no abnormalities. The next day the headache was gone. Everything was fine for awhile until two nights ago when I started feeling pressure behind (?) my ear. My wife massaged the area (said it was to drain lymphs) and by doing so I could feel the draining, which brought about significant relief initially but the draining (?) has not stopped and is excessive. I can feel the draining almost constantly and it is visible under my skin on the side of my face, down my neck and even on my arm near my elbow. Its not so much painful as it is uncomfortable and is becoming worrisome. I m not sure if all or any of these symptoms are related. Any suggestions or guidance would be greatly appreciated. Doctor: Hello. Thanks for writing to us. The numbness on the right side of the face is not related to the constipation and pain in the upper quadrant. A sudden exposure to cold or an injury can cause such symptoms. If the symptoms persist, do consult a neurologist.I hope this information has been both informative and helpful for you. You can consult me again directly through my profile URL http://bit.ly/Dr-Praveen-Tayal Regards, Dr. Praveen Tayaldrtayal72@gmail.com"
},
{
"id": 138594,
"tgt": "Suggest treatment for severe hip joint pain",
"src": "Patient: I m feeling pain in my Rt femur to hip joint for about five month. But it increased now. It is difficult to sit& get up. After doctor advice. X ray shows small radioopaccities noted in pelvic cavity s/o phleboliths . Is this normal disease which can be cured through medicine?. Doctor: Hello,Welcome to the magical world of health care, I went through your query, and what you are talking of would be arthritis, and the phleboliths are osteophytes, medicines and physiotherapy can give you relief, but the final cure is normally done by joint replacement, the decision, will be an consensual one between you and your consultantI hope my advice would have been useful, in decision making regarding your treatment, still if you have any clarifications or doubts feel free to contact back.I hope a 5 star rating if you feel guided in your treatment,Thanks"
},
{
"id": 30329,
"tgt": "What are the side effects of Depo provera injection?",
"src": "Patient: I had a baby 6months ago and in may of this year I got the depo shot (I have had it many times before with no problems) and now my period has been lasting for 3 weeks or longer and still coming every month. I don't know what's wrong. I have not had the depo shot since may Doctor: HI, thanks for using healthcare magicThere are some persons who have bleeding issues in response to depo provera.These issues can last till weeks or months after the shot has been discontinued.There are medications that your doctor can give you to help decrease the bleeding eg primolut.Your doctor may also suggest imaging of uterus to see if there are any other causes of prolonged bleeding such as fibroids or endometriosis.I hope this helps"
},
{
"id": 24237,
"tgt": "How do we best treat patients with Ischemic Heart Disease?",
"src": "Patient: i am 58 year old female. i have examined as a mild MR and inducible ischaemia patient. i am under treatment for 6 months under a heart specialist. but till now i have no effect of medicines. there is swelling in my legs every time. my b.p. remains on slightly higher side 150/90 mmhg.what to do? Doctor: Dear Mrs,You should seriously talk with your doctor to revise your treatment.There is a need to increase blood pressure medications doses or change the treatment. Usually when a patient has inducible ischemia we perform coronary angiography to see how many heart arteries are affected and how is the percentage of the blockage, and the further tactic of treatment of inducible ischemia depends on the results of coronary angiography. So you should also discuss the possibility of coronary angiography with your doctor.About the swelling legs, it can be caused either by heart problems or by medications or leg vessels problems. Your doctor should examin you thoroughly to find out the cause, as the treatment depends on the cause.Take careCome back if you have any further questions"
},
{
"id": 203439,
"tgt": "What is the little button between penis and belly called?",
"src": "Patient: this is kind of awkward but here i go, i am 17 years old and i have little button on the skin of the... lower abdomen (between the penis and the belly, i don't know what its called). i did my recherche i think it might just be acne (given my age) or fordyce spots, what do you think, should i be worried? (aditional info : i am still a virgin so i know its probably not an STD) Doctor: Hello,I have gone through your query and understand your concern.Your query is ambiguous and incomplete.You said that it looks like acne, I suspect it to be a infected hair follicle.As this the hair rich area, probably, it might be a infected hair follicle.This is usually caused by staphylococcal bacteria.It is not a sexually transmitted disease.You need the examination by your primary care physician to confirm the diagnosis.You need antibiotics like amoxycillin to get rid of this infection.Wash the area with the warm water.Sitz bath will be helpful.Hope this helps.Please write back for further queries.Wishing you good health."
},
{
"id": 43036,
"tgt": "How can the chances of pregnancy be increased?",
"src": "Patient: hi Doctor are you really there?I have been trying to conceive for two years. I have regular cycle- 28 days , and i have 3 to 4 days of period. i have some fibroids which my doctor told me are still small and not a cause for infertility... my husband tests were ok. I have done 3 cycles with chlomid .. and one failed IUI. twice I have felt all the signs of pregnancy only to get my period after being a day or two late. This month my period was one day late but before it came , I had spotting for like 5 days, nausea, restlessness and tender breasts... but the period came it was very dark in colour for the first few hours.... and after it became the normal red. Could I have conceived and failed to implant? what can i do to increase my chances of pregnancy Doctor: Hi,Welcome to healthcaremagic.I read your query and I understand your concerns.Following is my reply:1) We exist and help you with all issues.2) Please let me know size of fibroids which can give an idea for me to advice you further.3) Please let me know semen analysis of your husband.Let me know if you have any more queries.Regards,Dr. Mahesh Koregol"
},
{
"id": 104213,
"tgt": "Swollen lip, sore throat, hives, bruises. Taking benadryl. Allergic reaction to minocyclin?",
"src": "Patient: Swollen bottom lip, swollen throat (difficulty swallowing ), and hives . Bruised where previous hives were the other day. I have been given benadryl and steroid pills to help, but they aren t helping my throat/lip.... its an allergic reaction to Minocyclin. how long do I have to put up with this and what should i do in the meantime? When i was given the prescriptions it was just for hives, at the time my mouth and throat weren t swollen. Doctor: 3- 4 week take lot of warm water to recover take antiallergic fexofenadine 120 mg once or twice according to your need apply kenacort mouth gel on lips throat tds syp gelusil 2 tsf tds continue 2-3 week"
},
{
"id": 127171,
"tgt": "Is a painful bruise on the calf a matter of concern?",
"src": "Patient: I have a mysterious large bruise covering my calf that I hadn\u2019t noticed until I went for a run today (I\u2019m assuming it\u2019s running related since I haven\u2019t done anything that would have caused it) it wasn\u2019t painful until now, it\u2019s just a mild throbbing. Should I be concerned? Doctor: Hello and welcome to \u2018Ask A Doctor\u2019 service. I have reviewed your query and here is my advice. It could be due to over stretching of your ligaments and muscles. It can happen after a prolong physical exertion as in marathon running and jumping. In most cases it will resolve by it own even with out treatment.As of now you can apply ice packs for symptomatic relief and faster recovery.If symptoms persist, you have to consult an orthopaedic and get an MRI scan done. Hope I have answered your query. Let me know if I can assist you further."
},
{
"id": 95177,
"tgt": "What could be the cause of abdominal pain as I am on hcg diet drops ?",
"src": "Patient: I have left side stomach pain and ive been on the hcg diet drops for a few days. Im wondering what is causing this pain? Doctor: Hi welcome to H.C.M.Forum. left sided stomach pain is harm less. if it is at the top due to indigestion, in the middle due to worms, or renal stones, lower means due to urine infection. any how anti spasmodics and ant biotic is to be used with the advise of a doctor. thank you."
},
{
"id": 211026,
"tgt": "What are the physical signs of bipolar disorder?",
"src": "Patient: what are the physical signs of bipolar disorder? My brother has both mood issues so they say he has bipolar disorder but can his physical symptoms be from bipolar disorder. He has headaches, trouble walking his leg barely hold his weight, tires easily, headaches, and always dizzy. Doctor: HiThanks for using healthcare magicBipolar disorder is a psychological illness and its never cause physical problems. Sometime due to poor insight, patients harm themselves, which lead to physical abnormality other wise physically patient remains sound. In your brother case, there may be some brain dysfunction or nerve compression in leg. Consult a neurologist to find out the cause.Thanks"
},
{
"id": 12540,
"tgt": "How to prevent psoriasis? Medication?",
"src": "Patient: how can I prevent my psoriasis? I have it on my skin and scalp . I m a 24 year old male and I have this since I was about 13 years old. I m using an effective ointment, anti-inflammatory and it s very effective but it s too expensive. Is there any methods or medications that I can use so that I can prevent this skin problem? Thanks. Doctor: Hi, There is no permanent cure for psoriasis. But it can be checked nicely. Some small plaque psoriasis responds well & recurs rarely. Aggravation of psoriasis occur due to 1. Mental stress 2. Physical stress 3. Respiratory infection 4. Excess dryness Dandruff may be a part of it or it may be be a seborrhic one. So to reduce psoriasis you should 1. Reduce stress by yoga, meditation 2. In cough, cold consult doctor early 3. Apply moisturiser after bath when body is moist 4. Over scalp apply ketoconazole/coal tar/mild steroid shampoo Good luck."
},
{
"id": 27047,
"tgt": "What causes frequent giddiness?",
"src": "Patient: I am getting frequent guidiness. because of this I feel afraid to travel anywhere through bus and train, avoided watching movies and running. what may be the problem kindly help me Thangaraju Doctor: Hello and welcome to HCM. I carefully read your query. Thank you.Giddiness or a sudden feeling of falling, without it ever happening, is more a symptom or certain types of anxiety, agoraphobia and generalized anxiety disorder/panic attacks. I am assuming that you do not sleep well.. I would recommend you to follow a strict regimen, where you sleep well and be physically active for at least 60 minutes, since it does well to you, but not only running, also walking, biking, swimming etc. If you feel like panicking from your state, then consult a psychiatrist to further address your problem, and decide whether to start treatment or not. Remember, anxiety is not a mental illnes. It is merely a disorder that affects your quality of life.I would also recommend some cardiac area tests, just to exclude the organic origin of this phenomenon: - EKG- 24-hour blood pressure monitor- 24-hour Holter rhythm monitorAfter, the results are back, feel free to contact me or consult your cardiologist. I hope I was of help. Take care. Kind regards, Dr. Meriton"
},
{
"id": 201392,
"tgt": "Will night fall cause any problem in future?",
"src": "Patient: Hello sir, i m 14yrs old.when i thinks something about sex before sleeping A white sticky and thick liquidoften falls from my penis during sleeping.i am very worried about it that whether it may led a problem in future such as in reproduction.please tell me something to stop it Doctor: HelloThanks for your query,based on the facts that you have posted it appears that you have problem of night emission .Night emission is common at the age of adolescence and there is no abnormal in it .Do not worry It will not have any ill effect on your sexual potency in future.Dr.Patil."
},
{
"id": 31886,
"tgt": "Should i lance the bitten site to relieve pressure?",
"src": "Patient: i believe my son was bitten by a widow while irrigating a few days ago, he only mentioned it to me an hour ago because it hurts so bad, the site is swollen and red with the each puncture sight covered in puss, his stomach hurts (but did before) but the pain is different now, followed with lightheadedness, should i lance the site to relieve pressure Doctor: HI, thanks for using healthcare magicBased on the description , he may have an abscess which is a localized collection of pus.It would be best if he goes to a doctor for the area to because it needs to be done under sterile conditions and the area needs to be probed to get rid of any pus pockets that may be present.He may also need a course of antibiotics.I hope this helps"
},
{
"id": 54760,
"tgt": "Suggest treatment options for late stage cirrhosis of liver",
"src": "Patient: What is the wait on obtaining a liver for transplant purposes? My husband may have Late-Stage cirrhosis of the liver. What are some of the treatment options will he have? Currently, Kurt is living on Zofran and Promothazine. Are there other options (maybe a little stronger) that will help with my husband's nausea? Doctor: Hi I can understand your concern....Noted he is having late stage cirrhosis.So ultimate treatment is liver transplantation ....Other new option with less documentation is stem cell therapy.....Stem cell can regenerate liver tissue as having pluripotency ....Meanwhile for ascites prevention take low salt diet....Diuretic if needed can given....If you have splenomegaly and features suggestive portal hypertension beta blocker can be given.....Take spinach and carrot juice...Fruits more....Less invasive procedure like TIPSS like shut placement can done to reduce pressure...Take care.....Dr.Parth Goswami"
},
{
"id": 149824,
"tgt": "Diagnosed with trigeminal neuralgia. Having constant pain, increases while swallowing. Suggestions?",
"src": "Patient: I have seen the GP today who has diagnosed trigeminal neuralgia ..however the pain is constant and throbbing..feels like a vice across my face..every time I swallow the pain increases..searing..excruatiating...Is this TN as that seems to be talking about waves of pain..this is pain constantly only increases are every time I swallow..pain radiates then across right side of face. Doctor: Hi,Thank you for posting your query.The pain could be trigeminal neuralgia (TN). However, I agree with you that typically, the pain is episodic in TN. So, in your case, it could be called as atypical TN. I would recommend doing an MRI scan of the brain, to find out any underlying cause for the TN.Please get back if you require any additional information.Best wishes,Dr Sudhir Kumar MD (Internal Medicine), DM (Neurology)Senior Consultant NeurologistApollo Hospitals, Hyderabad,My personal URL on this website: http://bit.ly/Dr-Sudhir-kumar My email: drsudhirkumar@yahoo.comMy blog: http://bestneurodoctor.blogspot.com/"
},
{
"id": 14178,
"tgt": "Suggest remedy for a strange rash",
"src": "Patient: I have a strange rash that has come and gone in the last few months, but now it seems to be staying, spreading and evolving. It has turned some spots on my shoulder white and is mildly itchy. I went to see my nurse practitioner and she told me not to worry about it, but it has gotten worse since I last saw her, last Friday. Please help me in finding out what this thing is! Doctor: Hi.As per your case history you are having allergic dermatitis.My treatment advice is \u2013 1. Avoid using any new products like soap or perfume.2. Apply a mild antibiotic plus steroid cream like fusiderm-B cream twice daily on it.3. Take an antihistamine like levocetirizine for 7-10days .4. Other treatment options are oral steroid and hydroxyzine given only after consulting a dermatologist.Thanks.Dr.Harshit Bhachech.MBBS, DDVL."
},
{
"id": 56469,
"tgt": "What does high level of enzymes suggest?",
"src": "Patient: About ten days ago I saw my doctor. He did blood work and found that my liver enzymes were high. This is the second time this has happened. When the nurse called with my results she said the doctor wanted me to redo the blood work in 2 to 4 weeks unless I feel nauseated. But she did not say what to do if that happened. Today, I have been nauseous all day long. Have felt like vomiting at times. Is this something that can wait until tomorrow or should I have it looked at today? Doctor: HelloIt is important to know which liver enzymes are raised.Different conditions are associated with differently raised enzymes.Like,Increased GGT is associated with alcohol drinking.Alkaline phosphatase is increased in obstructive jaundice.It is also important to know other parameters of liver function test like bilirubin level.You may need few more investigations like viral markers and ultrasound of abdomen.Proper treatment depend upon findings.Get well soon.Take CareDr.Indu Bhushan"
},
{
"id": 187974,
"tgt": "Does the pain in the canine teeth lead to migraine?",
"src": "Patient: I have been having tooth pain in the back of my mouth the tooth broke and I still feel a piece there stuck In my gum and now for the past week my front canine tooth has been looking like it is slowly deteriorating and the pain is unbearable I can barely eat anything and it is making me cry over the torture and is giving me an extreme migraine, I have no insurance, and need a dentist or help immediately should I just go to the er? Doctor: Hi,Thanks for asking the query,Pain can be related to dental reasons.I would suggest you to consult the Dentist.Get the broken tooth removed immediately.Take a course of antibiotics and analgesics.Take care!"
},
{
"id": 163929,
"tgt": "Suggest treatment for child suffering from blocked chest and infected throat",
"src": "Patient: am steve from kenya and my daughter of 2 yrs have been having blocked chest and infected throat where by wheneve she gets sick and after been examined she is prescribed antibiotic drugs of which she has taken for long hw can u assist she stop completely with infections n taking of antibiotic steve +0000 or YYYY@YYYY Doctor: Hi...Thank you for consulting in Health Care magic. Greetings from Chennai.By what you quote I feel what your kid could be having viral associated wheeze or multi triggered wheeze. I have a few questions for you -Questions:1. How many days per month does your kid cough or feel breathless?2. How many nights per month does your kid's sleep get disturbed due to above symptoms?3. Does your kid feel breathless when running around or playing with other kids?4. Are the symptoms when there are seasonal changes?5. Is there any family history of asthma or any other sort of allergies like skin allergy etc.?6. Is the cough always associated with fever?Please revert back to me with answers so that I can guide you better.You can approach me at the following link.Once the page opens there will be an option below my image as \u2013 ASK ME A QUESTION \u2013 click on it.Please find the link below -www.healthcaremagic.com/doctors/dr-sumanth-amperayani/67696Regards - Dr. Sumanth"
},
{
"id": 75533,
"tgt": "What causes pounding headache and tightness in chest?",
"src": "Patient: It's summer time, so I sleep with my fan on and my door closed. This morning I woke up with a pounding headache and it feels as if my chest is very tight, right down the center. It's not focused on my left side, or my right side but it's directly in the middle and something as simple as making a quick movement causes it to flare up some more. It has happened once before and a co-worker of mine said it was probably a muscle pull, I work out every day and I do proper stretches, and it just happened over night. What do you think it could be? Doctor: the type of history you are giving, it seems as if it is some muscular type of pain only. another possibility is of dyspeptic pain , but one sincere advice is not to ignore this pain,but to consult a doctor for its proper assessment, as you never know it can be any one type of pain from a long list of causes of chest pain"
},
{
"id": 182091,
"tgt": "Suggest treatment for blisters in mouth after teeth removal",
"src": "Patient: I have had four teeth pulled recently due to a horrible taste in my mouth since Christmas of last year. after getting the teeth pulled nothing has subsided my gum just seeps and leaks making me sick to my stomach and causing blisters in my mouth. I have been sent around to three different dentists and no one has been able to figure it out. I recently have had a CAT and a ENT for my sinuses scan and nothing came back. I really need your help. Could I be allergenic to my fillings in my other teeth? Doctor: HelloI am not able to understand did you pull out four teeth just to heal the bad taste. 1. You should check your blood sugar levels, fasting and random.2. Then try using betadine gargle for two weeks.3. Kindly give information like your age, male or female, since when you are having this problem, do you have any other systemic conditions? 4. If possible send 2-3 intra-oral photos so that i can give you more accurate cause of the problem.5. Did the problem start since you had those fillings?Thank you"
},
{
"id": 168275,
"tgt": "Suggest treatment for breathing difficulty and nasal congestion",
"src": "Patient: Hello Doctor My child is of 2 months he has the problem in taking breathe and has some sound in the nose. we are giving nose drops otrivin but also it seems that it has not cured and also we are giving sporidex drops readymix from the past three days. Please suggest me to overcome this problem. Doctor: Hi....I will not suggest using either otrivin or sporidex ( there is no rationale of using antibiotic here). Try normal saline nasal drops or spray 4 times daily in both nares. If it is severe try to change posture during sleep; like side ways or prone. If these dosen't work seek an ENT consultation."
},
{
"id": 44670,
"tgt": "How to check whether the embryo transferred through ivf is of good quality or not ?",
"src": "Patient: Dear Doctor I had IVF, 10 embryos total and 3 transferred on day 3 with 8 cells. By day 6 all remaining 7 were bad quality to freeze. Does it mean that the average quality is bad and the ones transferred to me also? I am 42 years old. Thank you Doctor: Hello. Thanks for writing to us. Sometimes the preservation is not optimum and the embryos do not survive as well outside as they survive inside the uterus. It necessarily does not mean that the embryo implanted in you are also of a bad quality. I hope this information has been both informative and helpful for you. Regards, Dr. Rakhi Tayal drtayalrakhi@gmail.com"
},
{
"id": 10803,
"tgt": "Suggest treatment to stop hair loss",
"src": "Patient: hello Dr i am 20years old studying girl i had hair fall from last 3 months but i ignored it but now from last 15days my hair fall has increased like anything n i have almost lost my all hairs even aft i apply oil (pls kindly requesting u to suggest som tips to stop hair los n get fast hair growth Doctor: Hi... You seem to have telogen effluvium ..... There may be some cause....like anaemia,vitamin and nutritional deficiencies,thyroid dysfunction, stress, anxiety,worries,travelling,any other internal disease,certain drugs.... ..etc. Poor hygiene and poor nutrition may also be responsible for telogen effluvium. You consult dermatologist for firm diagnosis. Investigations may be done to find out the causes.To controll hair fall and to improve hair growth you may take biotin 10 tab daily for few months and apply mild steroid lotion on scalp at night daily.. If you have dandruff use anti dandruff shampoo containing ketoconazole. For hair oiling and shampoo use herbal preparations. Take treatment for few years.Have patience..for the good result.I hope this would help you.Thanks..Dr. Ilyas Patel MD"
},
{
"id": 96995,
"tgt": "Suggest remedy for accidental consumption of kerosene",
"src": "Patient: MY FRIEND ACCIDENTALLY TOOK TWO BIG SWALLOWS OF KEROSENE THINKING IT WAS WATER. hER SON HAD FILLED HER WATER BOTTLE WITH THE STUFF. WHAT IS THE ANTIDOTE? WOULD OFFERING HER A GLASS OF MILK HELP PREVENT STOMACH ULCERS? SHE IS ON THE WAY TO MY HOUSE. EVERONE IN THE E.R. WAS TOO SLOW TO RESPOND ACCORDING TO HER, THEYDID NOT HAVE AN IMMEDIATE ANSWER. SHE WAS AFRAID TO DIE THERE. ERIKA Doctor: The poison is corossive in nature so gastric lavage is not done. U can try charcoal which is an universal antidote. But its done at ER or ICU only"
},
{
"id": 37267,
"tgt": "Suggest remedy for cough,nausea and lightheadedness",
"src": "Patient: About 6 weeks ago, I was treated with walking pneumonia. Took Biaxin - started to feel better 3 weeks ago. I have 2 children, so resting has not been easy. Last week, I had a massage and have felt much worse since. Am back to coughing up phlegm in the morning and have been feeling a bit nauseas and lightheaded if I ve taken on too much. Am a teacher -going back to work in 2 days and am worried. Is this normal? Do I just need to keep taking is as easy as possible or should I see my doctor again? Doctor: Thankyou for the question.First of all I want to know ,have you completed your course of antibiotics,as incomplete treatment can be the reason of your symptoms too.Have you checked your temprature recently?Ofcourse you should not take it easy and should visit your doctor and discuss further dear.For home remedy for coughing,nausea and lightheadedness i am going to tell you a remedy surly it can help you.Garlic \u2013 Inhalation.If the infection is not completely blown, inhalation should be on the first place.\u2022\u00a0\u00a0\u00a0\u00a0\u00a0Boil water in a pot and add fresh ground garlic (3-4 cloves).\u2022\u00a0\u00a0\u00a0\u00a0\u00a0Lean your head over the bowl and cover with a cotton towel.\u2022\u00a0\u00a0\u00a0\u00a0\u00a0Start the inhalation. Initially you may be feeling uncomfortable, but the steam is amazing decongestant to help clear the airways.Also you need to modify your life style ,e.g ..pay attention to your diet,regular exercise and plenty of water,fruits vegetables.Get your haemoglobin checked too as these symptoms can be because of anemia too and you can discuss further.hope my answer will be helpfull.will be happy to hear from you again.Dr.Maheshwari"
},
{
"id": 192340,
"tgt": "Suggest treatment for sperm leakage with nerve weakness",
"src": "Patient: hi doctor..i am 20 years old..i had a habit of masterbating ..i was doing masterbating for 2 years..but before 1year itself i stoped it completely..and am not doing it not even think about it..but weekly once i my sperms comes out while sleeping without ma knowledge...and i thoroughly feel that i got impotence..my pennis becomes small and weak..am geting sperms within a min..and i feel that i have got nerves weakness..my fingers are shivering... can i overcome this porblem doctor??there is any treatment for this??can i full ma life partner aftter ma marriege..?pls tel me doctor..am so worried about this pls Doctor: Hello, There is absolutely no harmful effect. There are only myths about it. One can do it whenever he/ she wants. But remember, anything in excess is not useful. If you are addicted to it, it can divert your attention and may cause loss of concentration in studies, works and family life. Hope I have answered your query. Let me know if I can assist you further. Take care Regards, Dr. Sujoy Dasgupta"
},
{
"id": 200321,
"tgt": "Could male enhancement pill be taken with other medications?",
"src": "Patient: My husband has has prostrate cancer and surgery. He is unable to perform sexually. He has tried Viagra but ran out of them. We visited an adult store and purchased one pill named Miracle zen for male sexual performance . The problem is he is taking allopurinol, lisinopril, and lovastatin and is little concern about using this male enhancement pill. Doctor: Good Day and thank you for being with Healthcare Magic! I am just concerned that most OTC male enhancement vitamin or supplements contains testosterone and its derivatives and may have a very bad effect on his prostate cancer. I would caution you with the use of such products. Better stick with sildenafil. I hope I have succeeded in providing the information you were looking for. Please feel free to write back to me for any further clarifications at: http://www.HealthcareMagic.com/doctors/dr-manuel-c-see-iv/66014 I would gladly help you. Best wishes"
},
{
"id": 166769,
"tgt": "What causes cracking and pain in shoulder?",
"src": "Patient: My 4.5 month old keeps cracking in left shoulder or neck area (?) followed by a painful cry....? It s increasing in frequency, and the same after administering advil (tried for pain and inflammation). the crying only last about 1 minute. Dr said he was teething, Chiro felt some tightness on left side of C2/C3. I know popping/cracking is normal, but why is he screaming in pain? Worried Mom. Doctor: Hi,By what you say I feel that it could be a physiological phenomena. It could be a very initial stage of tics also. I am telling that it could be tics because of the immediate cry that baby is doing after the particular cracking. I suggest that you keep the baby under follow up with your pediatrician with the clinical tips.Hope I have answered your query. Let me know if I can assist you further. Regards,Dr. Sumanth"
},
{
"id": 208761,
"tgt": "Suggest treatment for psychiatric problems",
"src": "Patient: sir my mother is 50, 5.4\",50kgs...and she is undergoing a psychiatric problem(as we have assumed it to be)..she is a patient of hypertension...very short tempered and that worsen our problem...she has a different world going round her mind...can you suggest me what steps i can take to cure her..any treatment Doctor: If you feel she had had some psychiatric problem, just take her to psychiatrist with current medicines going on for hypertension and other medical records..As there are more than 300 diagnosis in psychiatry, from such small comments I cant tell you specifically about treatment.But as you mentioned, she has different world going around she might have psychotic features and schizophrenia. however she is hypertensive , we also need to rule out any brain damage through MRI. So without waiting go to doctor soon because earlier the treatment started better the result."
},
{
"id": 214850,
"tgt": "Pain in the center of chest, gastric problem, uneasiness, burping. Any simple natural remedies?",
"src": "Patient: Hello sir,,i feel very uncomfortable and i could not breathe properly and i feel little pain in center of my chest! I also have Gastric problems for couple of years! Will this lead to Heart Attack? What should i do to avoid this uneasiness due to gastric problem other than taking medicines. I feel very tired when i get this kind of pain in center of my chest! Suggest me some ways to get out of gastric trouble other than using tablets.And also i have burps every time which makes me even more uncomfortable. I am 21 years old.Thank You. Doctor: Hello,Thanks for the query to H.C.M. Forum.Gastric producing reasons are following as,Stress (Tension),Spirit,(alcohol)Strain ,Smoking,Spiced food,Sedentary life style,Hectic hurried life,Over eating (feeding),Untimely eating,Strong coffee & tea,No proper rest after meal.Your query is without tablets, treatment of gastric trouble.As I have mentioned the causes of gastric problem.If you want to get relief ,please avoid these reasons. Hope, you have received your answer.Is further any question I will be happy to answer. Wish you good luck.Dr. HET"
},
{
"id": 105795,
"tgt": "Does montair 10 cause difficulty in conceiving ?",
"src": "Patient: i m trying to conceive. And i m taking montair 10 tablet for my asthma . Is it problem for me to conceive. Doctor: Hi welcome to H.C.M.Forum. you can use any upto confarmation of pregnancy. but for the 1st three months of pregnancy it is better not use any drugs except pregnancy categery drugs.rest of the drugs the attending doctors will decide . thank you."
},
{
"id": 5227,
"tgt": "Trying to conceive. Came off the pill. Had miscarriage. Hormone tested normal. On norethisterone. Will this helps in conceiving?",
"src": "Patient: Hi my husband and i are trying to concieve and since i came off the pill in aug i got pregnant in oct 2011 and had a delayed miscarriage in december. In jan 2012 i had an AF although it seemed unusually light. Since then i have missed 2 AF and on CD 61. My doctor done hormone test and results came back showing normal levels. My doctor has precribed me 5 days of norethisterone however i am told this will induce an unovulatory cycle and wanted to know if it will affect me trying to concieve. I am trying not to stress as i am told this is why my AF is delayed and thus this pill is to help me try to relax by seeing an AF. Please advise. Doctor: Hello. Thanks for writing to us. Taking norethisterone will only help in regularizing your periods. After you stop taking the hormonal tablets, the chances of conception will be much better. You can try for conception from the next cycle.I hope this information has been both informative and helpful for you. Regards, Dr. Rakhi Tayal ,drrakhitayal@gmail.com"
},
{
"id": 202463,
"tgt": "What to do if skin is peeling off on the sides of the scrotum?",
"src": "Patient: Hai , doctor .It's this peeling off dry skin on my scrotum both sides , as well as the sides , exactly where the scrotum touched the inner sides of the tigh .it's just peeling off literally every day and then this red lines that shows tge tearing of skin . That's really painfull . Doctor: HIThank for asking to the HCMI really appreciate your concern, looking to the history given here I could say that this could be dry type of allergic condition and no need to worry about this it can be well treated with \"Diphenhydramine lotion\" apply this lotion three times in day keep the area clean, take good care of hygiene it would come around soon hope this information helps you, have a nice day."
},
{
"id": 89992,
"tgt": "Can D&C cause abdominal pain?",
"src": "Patient: I had 2 episodes of sever sharp pain on left side of lower abdomen that lasted for less then a minute. Its been almost a month since I had D & C. I am still waiting for my cycle to come. I was 6 weeks pregnant when I had D & C. Kindly advise what may be the problem? Doctor: HI.The pains which lasted just for few minutes can be due to intestinal colic. The D&C may not cause such a problem after one month. Do you have any other problem like loose motions , nausea or vomiting ?"
},
{
"id": 79922,
"tgt": "What causes shortness of breath while running?",
"src": "Patient: I AM QUITE FIT AND HAVE BEEN TRAINING FOR THE LAST 4 MONTHS WITH INCREASING INTENSITY FOR AN ULTRA MARATHON. OVER THE LAST 4 WEEKS I HAVE COMPLETED A 50KM AND A 57KM EVENT. SINCE THEN I HAVE BEEN EXPERIENCING SHORTNESS OF BREATH AND I KNOW ITS NOT DUE TO MY FITNESS LEVEL. I EXPERIENCED THIS IS THE POOL TODAY AND ON MY 27KM RUN YESTERDAY. MY BREATHING WAS QUITE ERRATIC AND DIFFICULT TO CONTROL. SHOULKD I GET THIS SEEN TO OR IS IT JUST FATIGUEA? Doctor: Thanks for your question on Health Care Magic. I can understand your concern. No, this is unlikely due to fatigue. Possibility of cardiac or Pulmonary disease is more in your case. So better to consult doctor and get done 1. Ecg and 2d echo to rule out cardiac diseases. 2. Chest x ray and PFT (Pulmonary Function Test) to rule out Pulmonary causes. Better to avoid further running. First diagnose yourself, start appropriate treatment and than you can continue your running and other exercise. Hope I have solved your query. Wish you good health. Thanks."
},
{
"id": 127622,
"tgt": "Are symptoms of digital neuroma and calcaneal spurs similar?",
"src": "Patient: Deaf (Male, 55), suffering from round the clock tinnitus, hyperacusis, etc. For last 5 years or more on Atorin F to control cholesterol although non-smoker, non-drinker. Now I have calcaneal spurs under left foot and on arcoxia 90 mg. Is there a possibility of digital neuroma what is being treated as calcaneal spurs? Thanks! Doctor: Hello and Welcome to \u2018Ask A Doctor\u2019 service. I have reviewed your query and here is my advice. No. Calcaneal spur can be detected on an X-Ray. I hope this information has been helpful for you. Regards, Dr. Praveen Tayal"
},
{
"id": 42898,
"tgt": "Is ectopic pregnancy related to ovarian cyst?",
"src": "Patient: hi im 34year old woman and last year Dec 2012 I had an ectopic preganancy of unknown location at11 weeks. I have been trying to conceive since April 2013 and with no luck. my doctor thinks I may have an ovarian cyst on my left ovary and I am waiting to do for a scan. I just would like to know if these 2 incidents are related to each other or has the treatment I received interfered with me trying to conceive (methotrexate injection) or is this possible my bodies way of telling me that a pregnancy is just not meant to be. I also have a 12 year old son and I am in good health. I work with children with disabilities and I am terrified that if I continue to try and get pregnant I may have a child with a disability Doctor: New born child to have disability can not be linked to previous tubal pregnancy Reason of tubal pregnancy can be investigated.I do not think ovarian cyst can be relatedtoit."
},
{
"id": 190562,
"tgt": "Painful holes in gums, discomfort in moving mouth, talking or eating. Remedy?",
"src": "Patient: Hi I am seventeen and had my wisdom teeth removed around four years ago due to a crowded mouth. I have recently developed three small holes in my gums and they are very painful. They hurt and sting whenever I Move my mouth or talk and it feels horrible if any food or drink touches them. I have one on the left side of my mouth towards the front (bottom), one on the right near the back (bottom), and another on the right (top) near the front. They are all near where my gums connect to my cheek and hurt really bad, please help? Doctor: hello and welcome to HCM forum, according to the history presented by you i would like to inform you that i won't be able to provide you with the diagnosis , as clinical check up needs to be done first. you have mentioned that you have sensitivity ,this is due to recession of gums and dentine exposure. These holes seem to be periodontal pockets, and need a dentist's attention. as you said your teeth are crowded, therefore in such cases, periodontitis is very common. i would recommend you to visit your dentist, get your teeth cleaned, as well as maintain your oral hygiene. i wosh you good health, take care."
},
{
"id": 174577,
"tgt": "Suggest treatment for enlarged blood vessel of heart",
"src": "Patient: My son is 4years old and we just found out that he has an inlarged blood visel that goes to his hart. The docter he seen said he just wants to keep an eye on it to make shore it dont get any bigger. I just want to know if it does get bigger what happens from thare is surgery needed Doctor: HiThe information provided by you is insufficient to suggest further course of your child's health.Don't worry. Most of the heart diseases in children have cure now a days."
},
{
"id": 29758,
"tgt": "Suggest remedy for persistent high fever post blood transfusion",
"src": "Patient: My mom is 89 and had a blood transfusion to receive 3 units of blood after doctors discovered she had a bleeding ulcer. The bleeding stopped and she has been out of the hospital for 6 days recovering. This evening she developed a fever of 101.5. I just gave her 325 mg of Tylenol and after one hour the fever is still 101.3, so I gave her another 325 mg Tylenol.My e-mail address is: YYYY@YYYY Doctor: Hello,If the fever is just a mild reaction to the transfusion it will surely subside after a few hours.If she had a bleeding ulcer before, there are chances she could have a perforation which can be serious. If she has severe pain abdomen, please take her to the hospital."
},
{
"id": 86172,
"tgt": "What causes pelvic pain and abdominal bloating after an endometrial biopsy?",
"src": "Patient: I had my endometrial biopsy done 3 days ago. It was an awful experience as I was not given any painkillers or local anaesthesia. Till now I have lower abdominal and pelvic pain. I had fever for 2 days. No fever today. My abdomen is really bloated. Any tips for better recovery? Doctor: Respected user , HiThanks for using Healthcaremagic.comI have evaluated your query thoroughly .* Pelvic pain is in relation with basic pathology of uterine problem along with post procedure discomfort .* Abdominal bloating is in relation with stress induced gastritis or other medications given post procedure to be taken orally .* Recommendations for better recovery- Drink plenty of liquids .- Soft , light diet .- Avoid oily , spicy , junk foods .- Cap.omeprazole (20)mg morning and evening on empty stomach Tab. drotaverine (40)mg morning and evening after meals for 2 days Consult later doctor if required .Hope this clears your doubt .Welcome for further guidance .Regards ."
},
{
"id": 187228,
"tgt": "What is the treatment for relieving pain in tooth?",
"src": "Patient: My wisdom tooth is comming in and im in sever pain.. my stomach hurts like im going to vomit and my neck and head is killing me.. im also a little dizzy and I dont see the dentist untill tomorrow .. what should I do and 800mg ibuprofen is not touching the pain and Orajel isnt helping either Doctor: HiThanks for writing in.Wisdom tooth eruption causes pain in most of cases.In such cases you cantake diclo-paracetamol combination along with amoxi-clav 625 twive a day for 3 days.Do warm saline rinses 3-4 times a day.Visit dentist if problem still persists.RegardsDr. Neha Sumra"
},
{
"id": 221460,
"tgt": "How can the pregnancy stage be correctly confirmed?",
"src": "Patient: My Girlfriend is Pregnant, and we went to planned parent to get the confermation, she said we were about 5 weeks, then last night she was not feeling well so I took her to the ER and her BETA level was 36 and they wanted to re-check her so we decided tonight to go and it went up to 64.11 They said she was only a few days pregnant? How is this possiable Doctor: Hallow Dear,You can locate the duration of pregnancy by following ways:1. The egg release takes place 14 days before the next menses. It can roughly tell you the probable date of fertilization.2. Usually the period of pregnancy is counted from the first day of last menstrual period.3. Ultrasonography and hCG levels tell you the gestational period from the day of conception. Hence there is a difference of about 2 weeks in the clinical gestational period and gestational period estimation by these investigations. When you are informed 5 weeks pregnancy, it refers to 5 weeks from last menstrual period; which actually is about 3 weeks from day of conception. This should clarify your dilemma. Dr. Nishikant Shrotri"
},
{
"id": 173618,
"tgt": "Is nebulizer or Flovent good for a child unable to sleep due to excessive cough and wheezing ?",
"src": "Patient: My son (4 years old January 23, 2014) did not sleep much last night due to excessive wheezing and coughing. He has had difficulty breathing since he was a baby and likely has asthma. We do not treat him preventatively. We have been giving him ventolin every 4 hours since yesterday evening. We have pulmicort (.25 mg/ml) that we can give him with a nebulizer or flovent (50mcg). Is one better than the other? Doctor: Flovent is better then pulmicort for quick relief from wheezing and cough. but nebulization with salbtamol is many time more effective then both. If your child has frequent cough (3-4 nights in a months ) . he should be treated with preventive therapy like daily inhalers . if child is not treated properly some child may develop disease which is less responsive to treatment."
},
{
"id": 140222,
"tgt": "What could cause brain fluttering with dizziness in patients with concussion?",
"src": "Patient: Hello. I am a healthy, active person. A little over a year ago I got a concussion, with no litigation concerns. The one symptom that I am most curious about is brain fluttering. I notice it more when I am waking up. I d also like to return to a strenuous workout, but fatigue, dizziness, and head buzzing symptoms worsen when I push it. Any answers or education to be offered? I m really wanting to get back to life. Doctor: Hello, I would explain that your symptoms could be related to post-concussion syndrome. I recommend practising Yoga and regular sports. An antidepressant (sertraline, paroxetine, etc.) can help improve your situation. Hope I have answered your query. Let me know if I can assist you further. Regards, Dr. Ilir Sharka, Cardiologist"
},
{
"id": 30083,
"tgt": "Suggest treatment for Tuberculous meningitis",
"src": "Patient: a young patient diagnosed TB meningitis and on ATT, referred to ophthal op for c/o diplopia.. on examination her vision is 6/9 in both eyes, restriction of abduction, dextroelevation ,dexrodepression mimicking lateral rectus palsy in right eye other movements are normal. diplopia is present in all gazes.. no ocular pain. MRI shows lenticular infarcts and lesions in cerebellum. no lesions related to orbit.. what could be the reason? Doctor: Hi, Thanks for your question. I do understand your pain and discomfort. I have seen many cases with similar complaints. Please find my advice below-:1)Start treatment for tuberculous meningitis INH+RIF+PZA 2)Check ICP and add steroid3)management of Lentricular infarct should be with-Activan,Cerebyx,Cardene & Mannitol4)Control BP and ICP in ICU which is the key factor of patient management & recovery. I hope I was able to address your query. If you have any further questions, please do not hesitate to write to me. Wishing you all the best. Thanks,"
},
{
"id": 72870,
"tgt": "What causes painful penis, chest and throat pain with fatigue?",
"src": "Patient: Hi, I a having alot of symtoms that similar to HIV/aids and the main ones are my penis hurts, chest and back pain, ear and throat pain, fatigue. I have got tested for HIV and all the stds and they came back negative.Can u please tell me what can be the cause of these symptoms? Doctor: Hello dear , hiWelcome to Healthcaremagic.comI have evaluated your query thoroughly .* There are different causes responsible for the same as - underlying pyrexia of unknown origin - deficiency of nutrients , vitamins , trace elements , minerals - low hemoglobin levels - others .Hope this may help you .Regards ."
},
{
"id": 75476,
"tgt": "What causes chest pain after coughing?",
"src": "Patient: Hello I'm 18, male, 260lbs, 6'0\"Last night I was making myself cough to get flem out of my bronchus tubes (just getting over an Upper Respiratory Infection) and I seemed to have hurt myself. Its been about 24 hours and now I feel accute pain from right under my right shoulder blade straight through to the front of my chest, also on the right side. Its hurts to breathe or lifting things even slightly moving my arm. Can someone tell me what is wrong? Doctor: Hi welcome to health care magic... Your work up done to rule out respiratory cause which seems most likely here like... -brobchitis or -lung parenchyma infection or pleurisyYour auscultation should be done first and than chest x ray done... If needed than spirometry done to assess severity of infection.... You might given levofloxacin like suitable antibiotic course... If bronchoconstriction present than you might given bronchodilator drug by nebulization.... For congestion relief take benadryl drug for few days Take one tsp ginger juice with honey daily So according to the cause further treatment can be guided... Take careAdvise : Pulmonologist consultation for examination..."
},
{
"id": 113803,
"tgt": "Recurring lower back pain, foot numbness, X-ray showed no sign of slip disc, taken Efetran, ultracet, V12. Cause?",
"src": "Patient: I am aged 48 suffering from lower back pain . I cloud not get up from where I was sitting on the floor. Nerve catch and I was taken to Ortho. Dr.Ortho gave pain killer injection & V12. Took x-ray and no sign seen on slip disc but said not to rule out. Advised for MRI in case of SOS. Medicine taken Efetran, Ultracet, Benzaday15, PAN D for 10 day. I was quite comfort and do my regular work/travel etc. After a month the pain developed. The pain from lower back to foot , numbness at the foot etc. Pl advise. Doctor: Hello.Thanks for the question.This is most probably because of a pinched nerve or lower limb radiculopathy (Sciatica ). Common cause is bulging intervertebral discs in the vertebral column that compress on the nerves as they come out of the spinal cord. Symptoms are experienced along the area of distribution/path of that particular nerve. Xrays cannot be ideal to evaluate this so go ahead with MRI. Conservative treatment include anti-inflammatory medications, physical therapy or chiropractic treatment, and avoiding activity that strains the neck or back. The majority of radiculopathy patients respond well to this conservative treatment, and symptoms often improve within six weeks to three months. I guess conservative treatment are not proving helpful in your case. If patients do not improve with the treatments listed above they may benefit from an epidural steroid injection. With the help of an X-ray machine, a physician injects steroid medication between the bones of the spine adjacent to the involved nerves. This can help to rapidly reduce the inflammation and irritation of the nerve and help reduce the symptoms of radiculopathy. In some cases the symptoms continue despite all of the above treatment options. If this occurs and the symptoms are severe, surgery may be an option. The goal of the surgery is to remove the compression from the affected nerve. Depending on the cause of the radiculopathy, this can be done by a laminectomy or a discectomy. A laminectomy removes a small portion of the bone covering the nerve to allow it to have additional space. A discectomy removes the portion of the disk that has herniated out and is compressing a nerve. I would suggest you to see a spine specialist who will take into account severity of your symptoms and MRI findings to guide about the treatment. Thank you."
},
{
"id": 144448,
"tgt": "What causes seizures?",
"src": "Patient: my son had a stroke when he was before he was born, he has spastic hemiplegia, cerebral palsy on his right side of his body, he also has chiari 1 malformation in his brain, if he was struck in his head could or would it cause seizures, he has been hit in his head at school and I m worried it might cause seizures to start, I was told by his dr that it could and that strokes and seizures go hand and hand, just no age of when Doctor: Hello!Welcome and thank you for asking on HCM!I understand your concern and would explain that your son's brain has been damaged since birth (probably perinatal hypoxia).This damages to his brain, are not only the cause of motor deficits and a delayed psycho-motor development, but also can cause seizures, as a result of the dysfunction of those brain regions. You should know that seizures in this case are more probable to occur during the first years of life, but can also occur later. A head trauma can induce them, as it can cause a new imbalance in the brain cell function and inflammation. If he starts having seizures, the treatment with antiepileptic drugs should be started, to prevent further recurrent seizures. Hope to have been helpul!Best wishes, Dr. Aida"
},
{
"id": 165850,
"tgt": "Suggest treatment for itchy sore bump",
"src": "Patient: .My son is 5 and for the last week he has been complaining about having a itchy sore bum in the inside he been getting up in the middle off the nite crying and a dont know what to do. Doctor: I understand your concern.Worm infestation is very common in this age. You should get stool examination for more confirmation. However I would like to start oral Mebandazole for 3 days. May your child be blessed with health."
},
{
"id": 71094,
"tgt": "Are severe chills while suffering from COPD a matter of concern?",
"src": "Patient: I don t know, I have COPD, and just after retiring with my oxygen and CPAP, I started shivering, I covered up but that didn t seem to help, I finally got up and now have a heating pad and blanket on me and am still shaking...I had an excessive amount of coughing last night, but seemed to be doing better today until this hit.My muscles seem very tight thru my shoulders. I also sent to feel movement in my head..Should I be concerned? Doctor: Hello and Welcome to \u2018Ask A Doctor\u2019 service. I have reviewed your query and here is my advice. * The severe chills with cough at night suggests underlying lower respiratory tract infection. * I recommend to immediately consult your PCP and get proper management guidelines after X-ray chest and lab tests. Hope I have answered your query. Let me know if I can assist you further."
},
{
"id": 108525,
"tgt": "Suggest treatment for back pain and puss cells in urine",
"src": "Patient: Hello Doc, my name is Anne, I m having a back pain, and I have pus cell in my urine, my doctor prescribe me antibiotic, but the pain still there... And my arms and shoulders get numbed. My sister was having an operation of kidney stone last 2000. And she felt same symptoms, could I possibly have kidney stone too? Doctor: HiThank you for asking HCM. I have gone through your query. Kidney stones also can give same symptoms along with coexisting urinary infection also. In such case an ultrasound examination will help to rule out that. Urine routine as well as culture and sensitivity should be done and change antibiotic if necessary. Take more water. Cranberry juice or barley water will be helpful. In case of small kidney stones taking potassium citrate in water also will be helpful. Hope this may help you. Let me know if anything not clear. Thanks."
},
{
"id": 117666,
"tgt": "What does blood contamination with organophosphate pesticide mean?",
"src": "Patient: Gregor who works at a pesticide factory comes to the clinic complaining of muscle spasms that interfere with his movement and breathing. A blood test shows that he has been contaminated with organophosphate pesticide, which is an acetylocholinesterase inhibitor. How would you explain to Gregor what this means? Doctor: hi, you have organo phosphate poisoning. OP poisoning is the most common poisoning. your muscle spasm is just due to that. you have acetyl choline esterase inhibitors which is responsible for your symptoms. you should take antidotes of that. Dont worry you will be ok. just be relaxed. consult your physician and take antidote according to your need.thanks for using health care magic."
},
{
"id": 74277,
"tgt": "What could constant breathlessness with cough suggest?",
"src": "Patient: Hi My mother age 51 is suffering from heavy breathlessness, cough and fewer, we have consulted GP, after taking medicine, she doesnot have fewer but breathlessness is there, once she stops taking medicine againg she get fewer, GP has also recommended DOXYCYCLINE AND LACTIC ACIDE BACILLIUS CAPSULES, DOSY-1 LDR FORTE. above are symptoms, please advise what she is suffering from Doctor: Thanks for your question on Healthcare Magic. I can understand your concern. Breathlessness in patient aged 51 years can be due to heart (heart failure) and lung (bronchitis or pneumonia) related diseases. So better to consult internal medicine doctor and get done ecg, 2d echo, stress test, chest x ray and PFT (Pulmonary Function Test). Ecg, 2d echo and stress test are needed for heart diseases. Chest x ray and PFT are needed for lung diseases. Treatment is different and depends on cause. So first diagnose and then start appropriate treatment. Hope I have solved your query. I will be happy to help you further. Wishing good health to your mother. Thanks."
},
{
"id": 192759,
"tgt": "What causes puss cells in semen?",
"src": "Patient: hello doctor i hv a 10-15 pus cells in semen i take lot of a hello doctor i hv a 10-15 pus cells in semen i take lot of antibiotics bt no results appear & spern counting is 30mill/ml and activity is 20% may i able to give a baby to my wife wht is the reason of pus cells Doctor: Hello, It can be due to infection somewhere in the urinary system like prostatitis or even UTI. Consult a general practitioner and start a short course of antibiotics empirically. You can repeat the test after two weeks, if there is still signs of infection, a culture study is required. Hope I have answered your query. Let me know if I can assist you further. Take care Regards, Dr Shinas Hussain, General & Family Physician"
},
{
"id": 40049,
"tgt": "What is the cause of fever and vomiting after sex?",
"src": "Patient: i've had sexual intimacy with my partner who was using cocaine an hour before and i've been getting fevers and throwing up ever since could it have been any possibility that it would have cause me getting sick, i am on anti-depressants could it have interacted.? Doctor: Hello,Welcome to HCM,The symptoms which you are having is maybe due to some Iinfection in the body. The source of infection is may be on any part of the body.As you had sexual intercourse previous day it may be one of the reason for your symptoms. But there are many reasons for the symptoms. I would suggest you to undergo battery of test for your symptoms and take proper and adequate treatment. Thank you."
},
{
"id": 73741,
"tgt": "How to cure prolonged cough in small kids?",
"src": "Patient: my daughter is 2and half year old.she is suffering from very severe cold &cough since 3months.we have met children specialist and he gave her cefidinir and cough syrup.but there is no change,and now bad smell comes out from her nose &mouth.what could we do now?please suggest any medicine for her. Doctor: Thanks for your question on Healthcare Magic.I can understand your concern.Better not to start any drug directly.She is having chronic cough and cold not improving with routine antibiotics. So we should definitely find out cause for this.So consult pediatrician and get done clinical examination of respiratory system and chest x ray.Possibility of lung infection or allergic asthma is more likely in her case.She may need higher antibiotics and anti allergic (montelukast) drug on the basis of reports.So don't start anymore drugs without proper diagnosis.Hope I have solved your query. I will be happy to help you further. Wishing good health to your daughter. Thanks."
},
{
"id": 205865,
"tgt": "What causes learning disability and hearing abnormal voices?",
"src": "Patient: i hav been told my daughter has bpd but mental health doc says she is not go n to label her she is 19 and doesnt take care of herself or cant is ceoliac and doesnt follow diet has lost a lot of her 2nd teeth and for years we wer told mild learning disabilty now this was put on dbt course which has now been removed from because of her learing issues were do i stand also thers the voices som tell her not to talk to and others tell her to talk to them Doctor: Hello thanks for asking from HCMShe has been suspected to have bipolar disorder but the diagnosis was not confirmed. She has coeliac disease and is refusing to follow dietary regimen. Her complaints that she is hearing abnormal voices that are conversing to her. Such voices can occur due to some perceptual disturbances like Hallucination mainly auditory hallucinations. Such symptoms should not be ignored and I would advise you to report to her doctor about such symptoms. These symptoms can occur due to some psychotic episode or may be schizophrenia. Medicines like antipsychotics as olanzapine, risperidone etc are commonly used in such symptoms. Discuss with her psychiatrist for prescription of drugs.Thanks, hope this helps you"
},
{
"id": 34388,
"tgt": "What causes fever with rash on hand and arms?",
"src": "Patient: hi I'm 40 yrs old female & going through alot of stress at the moment I feel like i have a fever but my hands and feet are cold ? I have also lost 6 pounds recently curently weigh 124 pounds. but I also have a hand and arm rash due to exposure of the sun in 2 min wicth itces like crazy and turns red ! Doctor: Hi,Itchy rashes over skin of hands and arms could be due to eczema,allergic hives,atopic dermatitis, tinea corporis etc.The important thing is to diagnose first rather than treatment.You can go to skin specialist for detailed history and inspection of rashes to have a clear diagnosis.You might require topical sunscreen lotion or corticosteroid like beclomethasone, clobetasol or mometasome with antifungal cream like miconazole or itraconazole with antihistamines orally to reduce itching and inflammation.Some time oral antifungal once dosage with prednisolone are also effective.So,you must consult to dermatologist.Thanks"
},
{
"id": 113713,
"tgt": "Pain in the right side of back. Diagnosed Cervical Spondilitis. Prescribed Aceclofenac & Paracetamol. Any side effects ?",
"src": "Patient: Hi Doctor, It is awesome to have your noble advise to mankind. I appreciate from my heart. I am 50 years now and had some pain in the right side of my back. Consulted Orthopaedic surgeon explained my problem. It was diagnosed as Cervical Spondilitis. Prescribed with Aceclofenac & Paracetamol tab 1 a day after food. Feeling much much better this is due 10 tabs I have used a week back. Do I need to use if I get pain. Please advise and will there be any side effects. Doctor: Hello and welcome to HCM If you take some precautions than you may not need any medication for a long time: 1. Avoid jerky movements of the neck. 2. NO pillow for sleeping 3. Use hard bed. 4. Don't keep the neck in single posture for a long time. 5. Do regular cervical exercise as advised by your physiotherapist. Now about the side effects of the medication: long term continuous use of this medication can show it effects like hepatic and renal dysfunction, nausea, depression, stomach discomfort etc. But occasional use are allow for a long term also. If you feel any of the side effect in you than immediately discontinue the medicine. Best Wishes"
},
{
"id": 44711,
"tgt": "What could be the cause of infertility if I have a history of low abdominal ,neck and back pain ?",
"src": "Patient: my age is 31yrs,weight is 70. medical history unable to get pregnant since 3yrs of marriage. i use to have low abdominal pain and back pain,& neck pain if am sitting now. Doctor: Hello Do medical investigations so as to find out for any gynaecological cause may be creating lower abd.pain. You should try to lose your wt. If the tests are normal,go for ayurvedic treatment which will make help u to conceive. Panchakarma & oral medications together get you result. there are many effective medicines in Ayu. which will accurately work on your problem."
},
{
"id": 89435,
"tgt": "What causes pain in my upper abdomen?",
"src": "Patient: I am having pain in my upper stomach area under the ribcage which comes and goes and sometimes more painful than other times. I had symptoms of what I thought were gallstones several months ago but an ultrasound was done three days after I had symptoms and everything looked good. Not sure if it is gallstones this time. I've have pain for 9 hours. Doctor: Hello, thank you for writing in to HCM.You seem to be suffering from Acid peptic disorder. Take small frequent meals instead of three main ones. Include a lot of fiber in your diet along with plenty of oral fluids. Try to get into a proper routine of going to sleep & waking up. Also include some form of excercise in your day to day routine.Take pentataprazole tablets with domperidone on empty stomach fast acting tablets . If pain is unbearable you can take Tab acuvin after that .Also, if you're constipated you can take a mild laxative. Even if get relieved You must see a doctor for investigations like ultra sound abdomen or even gastroscopy so that your future course of medicine is decided if at all required. Hope this was helpful."
},
{
"id": 16711,
"tgt": "What causes vibrating feeling in heart?",
"src": "Patient: I am 62 years old and have had hypertension for over 20 years. I am in reasonably good physical health otherwise. At rest, and at times when in motion, I feel a definite vibration in my heart with sometimes a twinge of slight pain jab in the heart area (to left of center of chest). Doctor: Hello, I would explain that your symptoms are not typical of any cardiac disorders. Anyway, considering your age, I would recommend consulting with your attending physician for a physical exam and a routine check-up including a resting ECG and cardiac ultrasound, a chest X-ray study, complete blood count, blood electrolytes. Hope I have answered your query. Let me know if I can assist you further. Regards, Dr. Ilir Sharka, Cardiologist"
},
{
"id": 76280,
"tgt": "Suggest treatment for chest pain",
"src": "Patient: I snorted bath salt, have done it other times before, and am now having a mild pain in my chest. I've had it for about 3 days now, but seems it is getting better. I have read online the chest pain can sometimes last up to 10 days. The last thing I want to do is go to a doctor and say I did this, so should I try riding out for a view more days. I dont feel im in any danger, it's not extremely painful just uncomfortable, and is not getting worse, but better day by day. Doctor: HiWhile this could be gastric reflux or a chemical bronchitis from the salt, I can't say for sure it isn't cardiac so you should have a doctor check it out and there'd be no need to mention snorting the salt"
},
{
"id": 18587,
"tgt": "What causes abnormal heart beats?",
"src": "Patient: My heart does this thing. It will be beating regularly, then suddenly it will beat a real hard pound, then it feels like a drum roll, then two more real hard pounds, then it feels like it skips a couple beats, then back to normal. This leaves me feeling like I m out of breath, but I m not and it exhausts me for about 2-3 minutes. What is it doing? And sometimes my left arm will hurt, and it has gone numb a couple of times. Am I headed for a heart attack? I m 58. Doctor: Hello and Welcome to \u2018Ask A Doctor\u2019 service. I have reviewed your query and here is my advice. I would explain that your symptoms could be related to a cardiac arrhythmia. For this reason, I would recommend you consulting with your attending physician for a physical exam and some tests: - a resting ECG and a cardiac ultrasound - an ambulatory 24-48 hours ECG monitoring to investigate for cardiac arrhythmia - blood electrolytes - thyroid hormone levels for thyroid gland dyfunction - complete blood count for anemia. The pain in your left arm could be related to possible local inflammation or a pinched nerve. Anyway, an exercise cardiac stress test would help investigate for coronary artery disease. Hope I have answered your query. Let me know if I can assist you further."
},
{
"id": 222488,
"tgt": "What causes stoppage in menstruation?",
"src": "Patient: hello im a 24 year old female and i am really woried and a bit scared i dont know what to do i have a 4 year old girl and dont remember my pregnancy much but my question is i havent had my period since october and have taken (hpt) and even a blood test and they have all been negitive but im showing all the signs of having a bun in the oven i have gotten sick my breast have filled out some and are tinder my nipples are sore i have mood swings i have been eating more i have backachs im an insoniac and have been sleeping alot but my question is could i still be pregnant? and my hcg levels are just to low to pick up yet or could something be wrong with me can you please help me and tell me something that could help i am really woried and if i am pregnant i dont want to lose the baby i am rh-neg and need to catch it quick so if i need to i can get a Rhogam shot and be safe for me and the baby please help thank you very much Doctor: If Hpt is negative then chances are very rare that you are pregnant. you might have some harmonal imbalance that causes missed periods. Consult your gynaecologist for checkup and usg and blood tests"
},
{
"id": 189219,
"tgt": "Pericoronitis. Swollen, painful little white pus pockets around wisdom tooth. History of tooth extraction. Suggest",
"src": "Patient: I got pericornitis around my lower left wisdom tooth 10 days ago and i got the tooth removed 7 days ago and I have just finished my course of antibiotics 2 days ago and I have healed from the wisdom tooth extraction but the pericornitis is still there the gum where the tooth used to be is still red and swollen and it still has little white puss pockets around there and it is still very painful and tender. should i give it another week for the infection to clear up on it's own or do you think I may need to get stronger antibiotics or maybe get antibiotics through an IV directly in the area or do you think i shoulg go to a periodontis an get the gum in that area cut off??? Doctor: Hi, Thanks for writing, Puss pockets state that the infection has not subsided even after a antibiotic course. As you did not mention which antibiotics are you taking, i cannot determine its effectiveness. I recommend you to visit your dentist once in case he wants to increase the dose or days. Also tell him about the pain so that he also prescribes an anti inflammatory analgesic. Meanwhile you can rinse your mouth with warm saline, which will give you symptomatic relief. Also use a antiseptic mouth wash 2-3 times a day. Don't let food accumulate in that area and flush your mouth after every meal. Do visit a periodontist in case the newer dose of antibiotics do not solve the issue. Stay healthy, Thank you, Dr. Tarush Thakur."
},
{
"id": 83863,
"tgt": "What are the uses and side effects of HCQS?",
"src": "Patient: Dear doctor, I have been prescribed HCQS by my doctor as I have developed serious rashes on my feet which has been diagnosed as urticaria vasculits. I am a lady who is 49 years old and currently on depakote as mood stabilizer . Is it safe to take this drug and what are the watch outs. Doctor: Hi, HCQS is a disease-modifying anti-rheumatic drug commonly prescribed to treat rheumatoid arthritis, lupus and other autoimmune diseases. Generally it is very well tolerated. However it may cause nausea and diarrhea, which often improve with time. Less common side effects include rash, changes in skin pigment, hair changes, muscle weakness and anemia in individuals with G6PD deficiency or porphyria. In rare cases, it can cause visual changes or loss of vision. Such vision problems are more likely to occur in individuals taking high doses for many years, individuals 60 years or older, or those with significant kidney disease. It is recommended that you have an eye exam within the first year of use, then repeat every one to five years. Hope I have answered your query. Let me know if I can assist you further. Take care Regards, Dr. Mohammed Taher Ali"
},
{
"id": 90773,
"tgt": "What causes abdominal pain?",
"src": "Patient: my name is ml.,lda i had a ovary removed by july 2011 my doctor advice me to removed because i have a breast cancer. My problem now i getting pain in sudden my stomach i don't have idea what is causes. i feel weak and cold. please if you can give me some advice what to do thank you. Doctor: hi my dearyou need to do upper endoscopy . it may be related with gastritis or stomach ulcer. wishing you all the bestdr. klerida"
},
{
"id": 6658,
"tgt": "What treatment should I take to conceive ?",
"src": "Patient: Hi ..I am 30 . We are trying for babyfrom past 5 months.I have taken Fertyl tab in the month of March.But i got me period regular,then next month i took letrozole still no success,please advice what to do. Does my age is reason for not getting pregnant Doctor: hi,Ask try to do contact in ovulation period which is on 12 to 18 days from first day of menses.still it is not happening then go for ovulatory study first it will know to you the exact time for ovulation then do occordingly."
},
{
"id": 91985,
"tgt": "Reason for abdominal pain after childbirth?",
"src": "Patient: Hi two years ago I had my daughter the birth was normal except for a manual removal of the placenta, since having my daughter I have been getting server abdominal pains that comes and goes, at first the doctor told me it's because everything was settling back after the birth, but I kept having them, I did take co codamol for the pain which I think was making them worse, making the pain spread all over my tummy, I couldn't really pin point where they came from I have since stopped taking co codamol and just take paracetamol the pain still returns and lately I can locate the pain to my lower left abdomen, when I get the pain I have diarrhea and I urine frequently these symptoms are only when I have the pain as I'm typing this I don't feel pain and don't have the other symptoms but I know it will return, the pain seems to start off mild then goes to server before disappearing I have had a urine test and blood test twice and they have come back normal, my previous medical history is as follow 3 children 2004 having a c section with my middle child please can u help :( I do have a weak bladder from the 3 children so I'm not sure if it's a symptom on Monday I did wake to a one off lower abdomen pain which was server but went shortly after I emptied my bowels this don't happen often tho Doctor: Hi. 2 years is a long time to suffer. You should have undergone a CT scan of the abdomen by this time to rule out any organic problem, as per your history. This may be due to a focus of inflammation on the left side of abdomen irritating the bowel and bladder. ."
},
{
"id": 43220,
"tgt": "Infertility history, had miscarriages, fallopian tube opacified, normal uterine cavity. Had HSG. What can bo done?",
"src": "Patient: Hi,This is senbagavalli, which is 28 years old and 2 years of infertily history.Miscarried about a year ago. Had HSG on 2 weeks before. And the result shows the below:FINDINGS:There is normal uterine cavity contour.There is no uterine cavity filling defect identified.The left fallopian tube opacifies initially.Additional Pressure was required for contrast opacification of the right fallopian tube. Initial left peritoneal contrast spillage is observed.There is delayed right fallopian tube peritoneal contrast spillage.IMPRESSION:1.Patent fallopian tubes bilaterally2.Delayed right fallopian tube contrast opacification and peritoneal spillage.3.Normal Uterine cavity contour with no filling defect identified.I have few queries:1.Whethere HSG had cleared the delayed right fallopian tube or still is delayed right fallopian tube and cant able to rectify.Please clarify the above. Doctor: Hi, right fallopian tube spillage wad delayed but definitely spillage was seen . Moreover you have conceived once but miscarriage happened. It shows your tubes are fine and nothing to worry about. Let me know if you have any more queries . Regards, Dr. Mahesh Koregol"
},
{
"id": 147956,
"tgt": "What does progressive thoracic endplate discogenic and facet degenetation with small disc herniations mean?",
"src": "Patient: Hi there I had a Mri done the other day because of burning back pain and right side rib pain. My report reads as follows sounds like a lot of things are going on. Made an Appt to see my Doc on the 24th of this month. here is my report a mild upper thoracic levoconvex curvature. There is a mild kyphosis at the T11-12 level. The cord demonstrates normal size and signal characteristics. Mild acute, Modic type I endplate degeneration is seen about the anterosuperior endplate of T12. No suspicious marrow infiltration is seen. There is mild lower thoracic endplate degeneration including small Schmorl's nodes. Multilevel disc desiccation is seen within the thoracic spine with mild loss of height.At T2-3, left greater than right facet arthropathy is seen without spinal canal narrowing. Mild left- sided foraminal stenosis is seen without nerve root impingement.At T6-7, a tiny right central disc protrusion is seen which flattens the right paramedian thecal sac without cord compression or foraminal stenosis.At T7-8, a tiny right central disc protrusion is seen which indents the thecal sac without cord compression. Facet arthropathy is seen without foraminal stenosis.At T9-10, a central disc protrusion is seen with an annular disc fissure. This indents the thecal sac and gently contacts the ventral thoracic cord without compression. Facet arthropathy is seen without foraminal stenosis.T11-12, a tiny disc bulge is seen with flattening of the thecal sac but no lower thoracic cord compression or foraminal stenosis.Review of the remaining levels reveals less prominent discogenic and facet degeneration without significant spinal stenosis, cord compression or nerve root impingement. In comparison to the 2009 study, degeneration has progressed.Evaluation of the paraspinal soft tissues is unremarkable.IMPRESSION:Progressive thoracic endplate, discogenic and facet degeneration including small disc herniations, notable at T9-10 where a central disc herniation and annular disc fissure contacts the ventral cord without compression or nerve root impingement. Doctor: Hi,Thank you for posting your query.I agree that the MRI spine report is long and reports a lot of abnormalities. However, they are minor in nature and there is no need to worry about them.These abnormalities are due to degeneration of vertebral bodies and discs (located in between the bones over back). As a result, there is mild compression or pinching of nerves in the back. This is giving rise to the pain.Treatment would require medications such as pregabalin capsules for pain relief. In addition, physiotherapy with back extension exercises would also help.I hope my answer helps. Please get back if you have any follow up queries or if you require any additional information.Wishing you good health,Dr Sudhir Kumar MD (Internal Medicine), DM (Neurology)Senior Consultant NeurologistApollo Hospitals, Hyderabad, IndiaClick on this link to ask me a DIRECT QUERY: http://bit.ly/Dr-Sudhir-kumarMy BLOG: http://bestneurodoctor.blogspot.in"
},
{
"id": 212669,
"tgt": "Fully depressed with life. Any online doctor can help?",
"src": "Patient: Hi i have been looking online for a solution or an answer to my questions i never really been gd at going to doctors so dont go there when im ill just stay at home. Well im 22 years old now i had a preaty horrible childhood until the age of 7 with an abusive father and i was fine then when i got to 13 years old everyone in my family would put me down say im gonna be just like him then i started acting out in school and got in trouble alot mixed in with worng crowd then got bullied at school and got abuse at home evverywhere i turned people would put me down and hey i dont mind thats its life but they put me down to the extent where when i was 15 i wanted to just die and did try to kill myself and ever since i was 15 right upto now i have times when i will be happy one minute and then all of a sudden will cry for no reason at all cause inside im hurting and then i try to lsten to music but that makes it worse sometimes the only think that takes my bad thoughts away is to close eyes lie down and for a second as stupid as it sounds i see myself saving someones life the hero honestly its affecting my life and my working life cause one second i up the next im down again i dont want to get up in the morning i just dont see the point i dont see my friends i have no social life and thats not cause i sit down all day its because i feel to low to go out sometime i ask myself do i really have a ppurpose to be here am i being punished for something i did i dont know maybe im being stupid with these feelings but i just want a bit of professional advice cause i just cant go on like this i thought it would go away as i got older but it just gets worse and harder every day thanks in advance sorry about my grammar i was never really big on english Doctor: Hi, Welcome to HealthcareMagic Forum From history following are possibilities in your case: -Personality issues -Anxiety disorder -Depression -Combination of above problems Considering severity and long duration of problem, in person history, mental status examination and general physical examination are needed to diagnose and treat such cases. Apart from above advice following simple measures will help you: -Decrease worrying by keeping yourself busy and distracted from worrisome thoughts by talking to family members or watching T.V. or counting back (100 to 0). -Healthy diet: Milk and fruits in diet. -Daily morning walk -Breathing exercise, yoga and meditation (which you can learn from some expert in person) -Online treatment with cognitive behaviour therapy (form of psychotherapy) . Wish you Good Health and all the best. Regards, Dr. Ashish Mittal www.99doctor.com"
},
{
"id": 68736,
"tgt": "What is the treatment for recurrence of abscess under the armpits?",
"src": "Patient: I am suffering from repeated absses in both my under arms since last 15 years. It pains and swells when pus forms and oozes out. After much treatment atlast doctor told it is TB and started treating me with Efficin injection and Rifa I-6. But there is no improvement. Doctor suggested to go for surgery now. What should I do? Plz help. Doctor: Send the pus for C/S + AFBShave hairs regularlyWash armpits with antiseptic Start antibiotics according to the culture report."
},
{
"id": 116389,
"tgt": "What are the effects of IGa deficiency?",
"src": "Patient: I am IGa deficient. 49 years ago my daughter was born with IGa deficiency. My daughter 53 and son 45 are both normal IGa. About 40 years ago (I was not aware of IGa) I donated blood. Since I worked in the lab I was aware that the recipient had a reaction to the blood. - The lab went crazy rechecking all availble factors. - A few years later the same thing happened. I have not donated blood since but am curious if the IGa could be the cause of these reactions. I am told to have washed cells only as a recipient but I couldn't have developed anti-bodies. What is your opinion. My daughter and one grandaughter would like to know also. Phyllis Dubey Doctor: Hi, dearI have gone through your question. I can understand your concern. You have iga deficiency. So you may have anti iga antibodies in your blood. It hives reaction with iga. When you take washed rbc there is no reaction, because washed rbc doesn't contain iga. Hope I have answered your question, if you have doubt then I will be happy to answer. Thanks for using health care magic. Wish you a very good health."
},
{
"id": 155391,
"tgt": "Why is my throat sore and tonsils red after chemotherapy?",
"src": "Patient: My husband has Cll/lymphoma for 3 years. He went thru chemo first year and has been doing ok, He had a cold for about a week and the severe sore throat has lingered. His tonsils look very red. Is this a reason for concern and should his oncologist be contacted? He is 62 yrs. Doctor: Thanks for your question on HCM.Chemotherapy causes immunosupression. So if a person is on chemotherapy, he is at higher risk of infection. And Upper Respiratory Tract Infection ( URTI ) is very common as it is the vulnerable and easily infected site.So he is having URTI , post chemotherapy. Better to follow these steps. 1. Drink plenty of fluids. 2. Avoid oily and spicy food. 3. Warm water gargles 5 - 6 times a day. 4. Macrolide group of antibiotic. 5. Antihistamine and anti inflammatory drugs. He will be alright in 5-6 days.After this episode ask for influenza vaccination for him. As it will prevent future URTI."
},
{
"id": 68971,
"tgt": "Suggest means for removal of lump in nose",
"src": "Patient: My 12 year old daughter has a small lump at the side of her nose, it is very small and skin coloured, she has had it a number of years, but she is getting concious of it now and wants it removing, what is the normal procedure to remove it and is it likely to lead a scar. It is approx 2mm diameter Doctor: Hello!Thank you for the query.Depending of the type of the lesion (mole like or not), it might be removed with laser or classic incision. Lesions can be removed with laser (especially that small) if there is no need to diagnose it by histopathologist (moles needs to be verified by a histopathologist). SO if its a mole, it needs to be removed in the classic way and stitches left.In both cases, the scar should not be visible. Face is the area where the healing is very fast especially if its about younger persons.Hope this will help.Regards."
},
{
"id": 85855,
"tgt": "What is substitute for Perdium to treat lack of peristalsis in Canada?",
"src": "Patient: I have issues with lack of peristalsis..65 yrs old with a varity of ailments. I saw posted to try Perdium, 2 tablets a day. I can not locate what it is..on line or the net. Could you tell me what the medical name of it is to see if I can purchase it in Canada.. Doctor: Hello, Perdium is a herbal medicine that you can find in name psyllium or senna. You can also find it as tea. Hope I have answered your query. Let me know if I can assist you further. Regards, Dr. Dorina Gurabardhi, General & Family Physician"
},
{
"id": 197762,
"tgt": "What causes blood from urethra when suffering from enlarged prostate?",
"src": "Patient: i'm 55ys old with daily sexual intercourse. i have experienced few drops of blood in my breif coming from urethra.it was painless and infrequent,just 3 times over 40 days. it is not related to intercourse and i have no urinary symptoms.my ultrasound study showed enlarged prostate (vol. 32ml.) Doctor: Good day and thank you for being with healthcare magic! Blood in the urethra that is not related to sexual intercourse is worrisome for us urologist and you need to have a complete urological workup. The reason for this is blood In the urine and in the genitourinary system is a red flag for us we need to rule out cancer or malignancy. The more common condition that could cause this would be infection and stones in the urinary system but we have to rule out the most dreaded condition which is cancer. You need to have a urinalysis, and a ct urogram (CT SCAN with IV contrast of the abdomen) to check where the blood is coming from. IF those two comes out normal then a cystoscopy should be performed. Cystoscopy is inserting a scope in the bladder and looking inside to check the urethra, prostate and inside the bladder to rule out mass lesions inside and to determine where the blood is coming from. I hope I have succeeded in providing the information you were looking for. Please feel free to write back to me for any further clarifications at: http://www.HealthcareMagic.com/doctors/dr-manuel-c-see-iv/66014 I would gladly help you. Best wishes."
},
{
"id": 126559,
"tgt": "What does numbness in the arms and fingers indicate?",
"src": "Patient: Hi! Over the past few weeks, I have experienced recurring tingling in my fingers which lead to temporary numbness...bi laterally. In the last 2 days, the numbness had grown up both arms. Like I mentioned before, the numbness is not permanent, but lasts anywhere from a few minutes to half hour. I have Lumbar arthritis.L2-L5 with multiple bone spurs, but this is not related to arm issues, is it? I have thought for years I might have MS due to extreme tiredness and other weird symptoms. What can this numbness be? Sandy Doctor: Hello. The numbness in the arms and fingers can be related to a mild nerve root compression or Methylcobalamin deficiency. Taking proper supplements after a blood test will help. Hope I have answered your query. Let me know if I can assist you further. Regards, Dr. Praveen Tayal, Orthopaedic Surgeon"
},
{
"id": 219761,
"tgt": "What causes throbbing hip pain in the mid of night during pregnancy?",
"src": "Patient: I am 8 weeks pregnant and i just went for a 30 minute bike ride last night. when i went to bed i felt fine but i woke up in the middle of the night with throbbing hip pain. it was really bad on my right side but also hurt on my left. Should I be worried about this? Doctor: Hello dear,I understand your concern.In my opinion the pain might be due to muscular origin.Any type of physical strain should be avoided in 1 st trimester ie.,upto 12 weeks.Due to bike ride there might be muscular cramps or spasm leading to pain.During pregnancy even a small physical strain might lead to muscle pain.Nothing to worry.Avoid physical stress and intercourse.Take Panadol if pain is severe.And take good rest.Observe for 1-2 days and if the pain is increasing or not decreasing then kindly get an ultrasound done to know the well being of fetus.Avoid stress.Relax.Hope this helps.Best regards...."
},
{
"id": 171726,
"tgt": "What causes stomach pain and pink color drops during urination of a 4 year old?",
"src": "Patient: my little boy 4 y/o just went to the restroom his urine color was normal light yellow but I notice that the last two - three drops were kind of pink. he told me my tummy hurts before he went and just after he finish he said now it doesn t hurt anymore. what should I do? Doctor: HiI understand your concerns about the pink colour you noticed. Pink urine can be caused by:Beets, blackberries, or certain food coloringsHemolytic anemiaInjury to the kidneys or urinary tractMedicationPorphyriaUrinary tract disorders that cause bleedingSo, I would recommend you to go for a proper urine routine and microscopic examination to rule out any blood or RBC. If it's normal then there is nothing to worry. Just give plenty of water every day to keep him well hydrated and avoid dehydration.HOPEFULLY THIS WILL BE HELPFUL.Take care"
},
{
"id": 18788,
"tgt": "Suggest remedy for high BP",
"src": "Patient: Hi, just having probs with blood pressure, it was low, so the drs took me off Cardizem CD and put me on fludrocortisone, up to 0.3mg daily. Was stable about 6 months, then my BP jumped to 160/90, was in hospital overnight (about 3 monts ago) and it came back down to normal. Fludrocortisone changed down to 0.2mg and BP stabilised. I have been off the florinef for about a week now,as a test with drs ok, and currently (in bed at almost 4am!) my BP is sitting at 159/116 with a heart rate of 75. I am 18 1/2 years post heart transplant, current ag is 32. Doctor: Hello and Welcome to \u2018Ask A Doctor\u2019 service. I have reviewed your query and here is my advice. I think your Fludrocortisone dosage has to be further reduced or stopped altogether. Please contact your cardiologist or local doctor, he will adjust the dose and keep a watch over blood pressure. For present scenario, either visit emergency or take any blood pressure medication, if you have with you at home. Hope I have answered your query. Let me know if I can assist you further."
},
{
"id": 82847,
"tgt": "Does positive ANA indicate lupus?",
"src": "Patient: Skin problems Ever since I was a kid I have had a problem with this. It started on my arms and eventually spread to the majority of my body. The majority of it looks like whiteheads and a few look like blackheads. My mom was told that they would go away by the time I was 30 by my pediatric doctor. I am now fixing to be 32 and it's worse than ever. I was told by my kids doctor that it was hyperkeratosis(not sure about the spelling). From what I have been able to find out on the Internet, it does not really fit what I am experiencing. I am currently 31yrs old, 5'5\" tall, 240lbs, and none of my medications have skin side effects(already did my research). If you have any questions please feel free to contact me. I greatly appreciate your help. Thank You, B. Musselman P.S. On a different subject; I have also have had a positive ANA. Does this mean that it is possible that I have Lupus like my mom does? I do have a lot of the symptoms of it, but I can't afford to see a rheumatologist. Been denied health insurance two times within the last year and a half. Hope you can help ease my worries a little bit. Thanks again for your time. Doctor: Dear Madam ,There may be a possibility that you may be having lupus , if your ANA testing done is positive.Dr. Shruti"
},
{
"id": 226245,
"tgt": "Taking tri sprintec. No period. Can I take a urinary pregnancy test to ease my mind?",
"src": "Patient: I m new to taking tri sprintec. I take it religiously and the first pack I had my period 3 days before placebo week. This month however I m on my second placebo pill and no period. Can I take a urinary pregnant test to ease my mind. And will it give me a true result or will the birth control force it to record positive because of the hormones in the tri sprintec? Doctor: Hi, Thanks for the query. It will take sometime for menstrual rhythm to get adjusted while taking the pills. So you need not worry, possibly you will get periods soon. If you still having doubt, better to go for blood test for pregnancy. Compared with urine pregnancy test blood test will give more accurate results in ealr pregnancy also. For more details:http://srsree.blogspot.in/2012/05/pregnancy-tests.html Take care."
},
{
"id": 21573,
"tgt": "Suggest remedy for redness and swelling in arm post BP check up",
"src": "Patient: A day after having my blood pressure taken with a blood pressure cuff on my upper right arm, there is a swollen, red, hot area half way around my bicep which is exactly the shape and position of the cuff. Never had this happen before. Also had a flu shot same day after blood pressure taken - shot was 3-4 inches above the red swollen area. What s up? Doctor: Hi,This is Dr Sameer.Some peoples have allergy to the bp cuff. Your symptoms looks similar. It happens because of some rubber cuffs.Don't worry, it will go in a day or two.We prescribe Tab Levocetrizine once daily for such patients. Apply some moisturizer over the area & it will go away.Remember not to again use that type of cuff again.Thanks"
},
{
"id": 35909,
"tgt": "What causes bumps in my face, back, arms and some in mouth?",
"src": "Patient: my childs school has bumps that first start out looking like ant bites then blisters and sometimes just small pimple looking bumps...They itch sometimes...The bumps are on our face,trunch,feet,arms,back and some have them in the mouth. they come and go...Does this sound like scabies? Doctor: HIGreetings from Dr.Divakara.PThanks for posting your query. Yes it could be scabies , chickenpox , herpes or some bacterial infection. I need to have a look at it so that I can be sure of what it is. So kindly post a picture of these bumps or otherwise consult a dermatologist. You can use Calamine lotion for preventing Itching. Kindly revert back with the picture. Hope this information is useful to you. Any clarification feel free to ask."
},
{
"id": 48146,
"tgt": "What causes bleeding in the urine?",
"src": "Patient: Good evening. I have had polycystic kidney disease for many years and under a consultant nephrologist. Having had a recent test I was found to have slight furring of a coronary artery so as a precaution he placed me on a daily dose of 75 mg aspirin and avorstatin once a day! I am otherwise fit and get plenty of exercise whilst on a healthy vegetarian diet! After 2 days of taking the aspirin I have developed blood in my urine. I feel well in myself but would like to know if I can stop taking the aspirin until my hospital appointment at the end of the week? I have read that this can cause blood in the urine! Advice wether the abrupt cessation of the aspirin until I see the consultant is advisable please? Doctor: Hi,Aspirin has ante-platelet action which help in prevention of abnormal blood clot formation but it may lead to bleeding tendency. In your case presence of polycystic kidney and aspirin treatment both are risk factor for blood in urine.I will advise you to consult your doctor immediately or if it is not possible then it is better to stop aspirin till your consultation with your doctor.Because many of the time intra-cystic bleeding may not be associated with blood in urine and we cannot estimate actual blood loss externally without doing investigations like complete blood count, ultrasound of kidneys."
},
{
"id": 206000,
"tgt": "What is causing my bouts of sleep walking?",
"src": "Patient: hi i had fits 2 years backs and i am admitted to a hospital i was there for a month as i forgot my my past after i was cured i was suggested to take levepil 1gram oxetol 600 topomac 100 mg and eptoin 100mg and from two day days i am feeling a little dizzy and suddenly yesterday my wife told me that i wolk at the night and pissed in the hall and i dint remember that Doctor: dear,from your complain it appears to be a problem of sleep walking.I would like to know1) do you get bad dreams, if yes how often?2) have someone ever noticed you waking up screaming in midnight?3) do you feel sleepy during day? if it persist for more days a course of SSRI may help you.I would also advice you to get your sleep hygiene right.to have clear diagnosis and yo above que might help me.thanks"
},
{
"id": 219471,
"tgt": "Suggest remedy for blisters on nipple with black discharge during pregnancy",
"src": "Patient: I have a blister on my nipple. I am 19 weeks pregnant. Before pregnancy I had black discharge coming out of this nipple as well. The doctor did an ultrasound on my breast (before I was pregnant) and said he had no concerns at the time. Now that I m pregnant I am still getting black discharge and I now have what seems like a blister right beside my nipple. What should I do? Doctor: Hi.Blisters on the nipples or around them (in the areolar region), most commonly occur due to 2 reasons, either due to friction (from clothing, sexual activity, etc; or what we call a milk blister which occurs while breastfeeding.As for the black nipple discharge, this could be associated with a haematoma in the breast, an abscess, etc; and will require more concrete investigations into finding the cause as you will soon have to breastfeed.Best wishes."
},
{
"id": 77122,
"tgt": "Suggest remedy for sore neck with cold with congestion",
"src": "Patient: Hello. I've had a 'cold' with a lot of congestion and, having asthma, I've had a LOT of coughing. Now the back of my neck hurts on both the left and right side when I turn my head. I'm assuming this is from the cough and congestion but as the cold is getting better, the neck soreness is not going away. Hot shower helps, as does massage, but I'd like a bit of reassurance :) THANKS! Doctor: Hi,Dear,Thanks for your query to HCM.Dear I read facts of your query and reviewed it in context to your health issues submitted in your query.I understood your health concerns and feel Concerned about them.Based on the facts of your query the reply of your query is-Your current health issues seem to be due to -the Cold aggravating your asthma with accompanied bronchitis.This would create more problems like the one -straining of neck musles and chest muscles from repeated coughing and asthma causing dyspnea.The neck soreness is thus from strained neck and with accompanied? viral Myalgia, which would recover with time and treatment of Asthma by -Antibiotics/ Brohchodialators/ Plenty of fluids and proper bronchial drainage to avoid pneumonia or brohchiectasis.This is therefore more alarming condition and situation needs to be dealt by urgent Consultation with Chest Physician for ruling out TB / and other causes for this.The fact that hot showers and massage helps in relieving is more rewarding as this Myositis from Strain would resolve with Pain killers and NSAIDS supplements for few days timeTherefore Cause of these complaints in your case needs to be investigated by Chest X-ray and by blood tests and if need be by CT Chest.Other causes need to be verified,as discussed above.For this you need to consult Chest Physician .Do's in your case-Check With Doctors as suggested above-Rest and Fluids in plenty with Brohchial Care.You need to Urgently plan Corrective Antibotics and other care,as your case is bit complicated and seems to be neglected.Get physical check with Chest Physician who would do needful Chest X-ray and other investigations.There are many causes which need to be verified amongst the causes discussed above before planning specific investigations and remedies.High protein / high Calories diet to built up body.Vit B12 and Folic Acid- to stabilize the body and bronchial stress.Thus You should consult Chest Physician.This would resolve the complaints of your's.Constant follow-up with local family or specialist Chest Physician for long term would give you long remissions of such complaints in time to come,which would depend on meticulous follow-up treatment of your health concerns.Just don't worry and be patient and co-operate with your doctors,till you verify with your attending doctors.For early recovery-please follow above do's which would resolve your complex health issues.Hope this would help you to treat your health issues in the best way possible. Welcome for any further query in this regard to following link-http://doctor.healthcaremagic.com/Funnel?page=askDoctorDirectly&docId=70229Wishing Good Healthy Life!!Dr.Savaskar M.N.Senior Surgical SpecialistM.S.Genl-CVTS"
},
{
"id": 147257,
"tgt": "Tingling in feet and fingers, urinary incontinence, frequent urination, urination urgency. Family history of MS. Do I have MS?",
"src": "Patient: Hi, my name is Jennifer, I am 37 yrs old and my mother has MS. I have been experiencing some symptoms related to this disease. I was just looking up some early warning signs and there were quite a few on there that I definitely do have. Now whether it s MS or not I can t of course make that diagnosis on myself. There are 21 of them however that I do have. The ones that are more noticeable than others are the tingling in the toes, feet and fingers, urinary incontinence , frequency in urination , urination urgency, short term memory loss and unsteady gait/loss of balance are just a few of them. I am of course going to see my physician over this, but I was wanting to get an opinion from someone on what they think all this could be meaning. Also, I don t have regular insurance, I am on a medicaid card so I don t know how serious my physician will take me about all this. I know I m not imagining these symptoms is the thing. It s been going on for quite some time now. I had a regular Dr but he moved out of town so I am in the process of finding a new one....which I will very soon. Thank you for your time and I will wait patiently for an answer. Doctor: Hi Jennifer, thanks for the query. As you might have already searched & know; multiple sclerosis (MS) has plethora of signs & symptoms. But it can present in various ways including tingling numbness, urinary incontinence or even frank Psychiatric problems like depressive symptoms, manic states etc. Usually the common ones like tingling numbness, gait imbalance, urinary incontinence are screened for by doctors. But you DO need to know that MS is NOT strictly a hereditary disease. Hence you are at a slightly higher risk but it do sent mean you are having MS for sure based on having some symptoms like tingling numbness, incontinence (you mentioned). Hence try to relax first. Then visit your Neurologist who with help of your history, clinical examination & MRI Brain (if needed) will diagnose or RULE OUT MS! Lastly, MS is no longer a disease with grave prognosis as it used to be once upon a time & with advent of new drugs; patients do lead a near normal disability free life. Hope this helps..good luck"
},
{
"id": 29001,
"tgt": "How can UTI and painful urination be treated?",
"src": "Patient: I know I have a uti my urine is all cloudy smells hurts in my urethra when I pee and hurts for some time after. My Dr. got my results in today however the nurse aide didn t call me back and tell me what to take. My GP Dr. said he can t give me a script because my urine dr. sent the pee to be checked.I am peeing blood also and my gp dr. is waiting for my poop tests to come back in a couple days. Now what can I do for the pain I am in also I m very anemic and they are trying to find out where my blood loss is going. And I am having awful pain from my low back through my legs and feet terrible cramps like muscle spasms. Doctor: Hello,I think you might be having kidney stones alongside urinary tract infection (UTI). Until you get urine test reports, I advise to:- Increase fluids intake- Take cranberry supplement- Take Ponstan or ibuprofen or Spasmex for the pain- Take iron supplements for anemiaHope I have answered your query. Let me know if I can assist you further.Regards, Dr. Albana Sejdini"
},
{
"id": 84501,
"tgt": "What are the side effects, prevention and benefits of Ecosprin AV 75 ?",
"src": "Patient: I have beeen taking Ecosprin AV 75 for the last 5 years. I am 48 years old. Last year cholestrol was 137. I stopped taking ecosprin for the last one and half months and now the cholestrol is 217. should I start taking ecosprin av 75?what are the side effects and prevention to be taken? and benefits of ecosprin av75. Doctor: Hi,Ecosprin AV 75 contains ASPIRIN 75 mg and Atorvastatin 10 mg. I noticed that you are having dyslipidemia as your cholesterol 217. Indication of aspirin mainly secondary prevention of cardiovascular risk (means you had angina, coronary artery disease, minor stroke). Three is a limited role for primary prevention (for those who had risk factors for CVD such as smoking, dyslipidemia, diabetes, hypertension, etc.).if you have started for secondary prevention then you should continue taking aspirin and statin. For primary prevention, you can reassess your risk factors and then decide whether or not to take aspirin. The side effect of low dose aspirin is very few. Peptic ulcer gastritis is one of them. To avoid that you can take aspirin after a meal. You make consider taking PPI such as Pantoprozole 40 mg once a day if you have symptoms.Hope I have answered your query. Let me know if I can assist you further. Regards, Dr. Mahendra Shivram Thakre, Internal Medicine Specialist"
},
{
"id": 31311,
"tgt": "Suggest cure for diarrhea and bleeding in the rectum",
"src": "Patient: I had a bout of vomiting and diarrhea in the middle of the night and now my rectum is bleeding. NOT just spotting on the toilet paper. I don't bleed unless Iam sitting on the toilet and when I cough the blood spurts out. I first noticed it this morning before I hopped in the shower I coughed a lot( I have a bad cold right now ) and 3 big spots of blood had dripped out onto my floor. Now everytime I'm on the toilet today blood spurts out..should I go see a doctor or wait it out? Doctor: Thank you for your query Yes, any rectal bleeding should be checked out immediately, especially in the quantity you mentioned and that spurts. Do you have any history of hemorrhoids, liver disease or intestinal/rectal surgery? Did you have any recent diarrhoeal disease? Sometimes increased intra abdominal pressure (such as when you cough) could rupture tiny blood vessels in the anal canal that can bleed If this continues, I recommend you go to the ER. If it is minimal or stops, see your physician ASAP to rule out serious disorders and prevent further blood loss. Expect to be hospitalized, with blood work. Depending on where the bleeding point is and how big it is, how much blood loss has occurred, you may need cautery/pressure dressings, blood products. These are just a few things. It may just be a small vessel bleed and you may be allowed to go home after a few hours Hope this helps. Wish you good health. Please get back to us if you need any clarification"
},
{
"id": 116400,
"tgt": "Suggest medication for treatment of high ESR levels",
"src": "Patient: i am a Female, age 26 years,i received my blood test report in which the ESR 1st hour 52mm , 2 i am running cold,fever, backpain,since 21 days, i am a diabetic , i feel twisting & indigestion problem since 6 months, please suggest treatment and diet in regular course, thank u, XXXXX XXXXXXX kindly prefer a right medicines for treatment of high ESR. Doctor: Hi, dearI have gone through your question. I can understand your concern. There are many causes for high ESR. Tuberculosis, anemia , inflammation, autoimmune disorder all can leads to high esr. You should search for the cause of your high esr. Consult your doctor and investigate for that. then you should take treatment according to the the cause.Thanks for using health care magic.Wish you a very good health."
},
{
"id": 205179,
"tgt": "What causes depression despite taking Celexa everyday?",
"src": "Patient: I was taking Celexa 60 mg a day and I was starting to feel depressed more than I usually do. She put me on Cymbalta and so I have weened off of the one and I am taking the cymbalta 60 mg a day. I feel very agitated and I get upset quickly. I have also been feeling more anxiety episodes with new Med. I meet with my doctor tomorrow, any Recommendations? Doctor: please i form you doctor sbout fresh symptoms.the fresh symptons may be due to the unique action of cymbalta.u should be fine soon."
},
{
"id": 149233,
"tgt": "Taking epilim for epilepsy. What are the risk associated in taking anabolic steroids?",
"src": "Patient: rubHaloI am currently on Epilim 500mg twice daily (one in the morning and one in the evening) for frontal lobe epilepsy and on CynGen 60mg in the mornings. Both are for depression. I am currently gymming a lot and want to start using anabolic steroids (Dianabol). Are there ANY risks whatsoever for me if I use these steroids with my Epilim and CymGen? Doctor: Anabolic steroids are usually given to increase the weight,usually with some side effects.Drug interactions may not be significant,but should be used,if advised by a NEURO-PHYSICIAN."
},
{
"id": 116229,
"tgt": "What should be the normal value of creatinine level?",
"src": "Patient: my father is a kidney patient. his creatine level was maintaining consistently around 2.7. But from last two month, level increased to 3.5. he is taking nephrosave as medicine including medicines like amtas 50,moxovas. From one week his health condition has been detoriated drastically, he is not able to walk at his own. please suggest for improvement if possible. Age is 80 years approx. Doctor: Welcome to H.C.M. I am Dr Krishna Dubey.My pleasure to help you. Norma Cretanine value is 0.7 to 1.3 mg/dL.Your father creatinine level is high.Your treating doctor is giving correct medicine.But even after proper treatment Creatinine value can increases.You have to give him strictly renal diet. Strictly avoid fruits.Repeat creatinine level twice in a week and if not increases then once in a week.Because if persistently increases and condition deteriorated then dialysis wll be required.Thanx. Keep in touch."
},
{
"id": 135894,
"tgt": "What causes numbness in the face and legs?",
"src": "Patient: My left face and arm and legs go numb when I sit just for two minutes. Then then the other side feels the same after a while both sides are numb even my stomach. I have occasional chest pain and arm pain that comes and goes. Could this be related to my anxiety disorder? Or more serious problem like clogged artery? My heart beats fast when I eat sometimes feel loss of breath I Just turned 24 I also sometimes have abdominal pain. I also have a hyperventilation syndrome.when to ear did many x rays and EKG standard blood work all came back Normal. Doctor: your sex please? Anxiety disorders can cause symptoms like these.usually all invesigative parameters are within normal limits.Kindly don't worry.RELAX.You may take Diazapam 5mg at night.If i does not settle down in due course (4-5days) you may need to consult a psychiatrist."
},
{
"id": 108747,
"tgt": "Suggest remedy for lower back tenderness",
"src": "Patient: lower back tenderness when pushed in the specific area. however no pain otherwise at all even lifting weights there is no pain at all. should i continue to lift weights or stop immediately. if so, should i implement some stationary biking cardio for maximum blood flow to the area ? Doctor: Thank you for the question. If some area is tender then it will be safer to wait for relief and avoid weight lifting till then, however light massage and back strenghtening exercises in lying posture should be initiated and gradually shiftover to more vigrous one once tenderness disappears"
},
{
"id": 124565,
"tgt": "What causes pain in neck down the clavicles?",
"src": "Patient: I have been having an electric shock-like pain in my neck down the lateral aspect towards my clavicles for a few days. I ve expereinced it before, a few times a year previously. I don t know if it is related to my work- I m a bedside nurse (30 years old) and do a lot of lifting, moving of heavy patients. Doctor: Hello, It could be due to conditions like cervical spondylitis. As a first line management you can take analgesics like paracetamol or aceclofenac for pain relief. If symptoms persists better to consult a neurologist and plan for an MRI scan. Hope I have answered your query. Let me know if I can assist you further. Regards, Dr. Shinas Hussain, General & Family Physician"
},
{
"id": 157146,
"tgt": "Are cystic lesions cancerous?",
"src": "Patient: I had a colondoscopy done yesterday....a couple spots were noted as having cystic lesions {50 cm}....normally a biopsy/sample would have been taken except I am normally on blood thinners. {aortic titanium valve implant} so there would have been the risk of bleeding. So now I am scheduled for a cat scan this week......what are they looking for...what will scan show .....if anything ....is there a cancer concern? If this is cacerous...what is next possible step procedures....58 year old male....very good health...Thankyou! Doctor: Hello.The cystic lesions are benign and only become malignant exceptionally. why do you a tc-scan is because you are taking anticoagulants and biopsy is not possible.If tc-scan is normal are not necessary more additional tests. I wish you good health.(If the answer has helped you, please indicate this)"
},
{
"id": 155511,
"tgt": "What is the life expectancy in a large B cell lymphoma suffering person?",
"src": "Patient: My father is 83 and has had previous large b cell lymphoma that he recovered from. the other day during a scan by his heart m.d. a large mass showed up on his liver. he has had nausea and vomiting the last several weeks. he was told he has lymphoma again and a biopsy of his liver is to be done next week. What is the possible prognosis and survival rate. Thanks so much!!! Doctor: Thanks for your question on HCM. Lymphomas have two phases in all patients. 1. Remission2. Crisis. Your father is having crisis phase right now. Appearance of new lesion in liver indicates worsening of disease. And biopsy is must.Usual five year survival with B cell lymphoma is 60%. And life expectancy is atleast 4-5 years with regular chemotherapy sessions.So start chemotherapy again and continue with maintenance therapy for better life expectancy."
},
{
"id": 188544,
"tgt": "Pain in lower jaw upto ear. No help from prescribed ibuprofen and Tylenol. History of tooth extraction. Help",
"src": "Patient: I am a 37 year old female ..I had a lower tooth pulled 5 days ago and I am in so much pain I am thinking of going to the er .. my lower jaw hurts up to my ear.. I have been taking the ibuprofen 800 mg that the dentist gave me I have also tried Tylenol but nothing seems to help I really don't want to go to the doctor because I have 3 small kids please help Doctor: Hello,Thanks for writing to us.You have to administer amoxycillin along with analgesics.Gargle frequently with lukewarm saline.Avoid spicy foods.Anxiety and stress has to be avoided.Take adequate rest.Irrigate the socket with betadine and apply eugenol as well as lignocaine into socket.Hope this helps."
},
{
"id": 58503,
"tgt": "Took antibiotics for typhoid fever. Had chest pain. Test report shows enlarged liver. Reason?",
"src": "Patient: Reasontly 3 weeks befor i was affected by typhoid fever negetive and taken anti biotic for tablet for one per da 18 day 18 tablets (6 x3) - one week before i have chest pain, now i went for body check up i came to undersatand that my lever get enlarged -why lever gets enlarged? please give me your valuable suggestion Doctor: HelloThanks for writing to HCMLiver may be enlarged due to variety of reasons.It may be due to infective etiology like hepatitis, Fat deposition, drug toxicity, tumour, prior infection, metabolic disorders etc.You need proper clinical examination and few investigations like routine hemogram,RBS,LFT,Lipid profile.Treatment may be given if there is any problem in investigation reports.Get well soon.Take CareDr.Indu Bhushan"
},
{
"id": 181629,
"tgt": "What causes severe pain in the neck and ear after wisdom tooth removal?",
"src": "Patient: I got my 2 lower wisdom teeth pulled 9 days ago from today. I am currently taking antibiotics because the left duct after extraction got infected. The infection seems to be gone but I now have locked jaw, pain in my neck, ears, and it hurts to swallow. Doctor: Hello,Thanks for the query,The symptoms you mention are typical symptoms of dry socket also known as alveolar osteitis that form after tooth extraction due dislodgement of blood clot from socket . Loss of blood clot occur due to any forceful movement of mouth like rinsing , swishing and sucking.I would suggest you to :-1 Do not rinse your mouth vigorously2 Do not smoke(if you have the habit)3 Do not touch that area with tongue or finger.4 Stick on to cold and soft diet, avoid spicy and hard food stuff.5 Do warm saline rinses gently several times a day .6 Take ibuprofen (if not allergic ) or acetaminophen for pain and swelling.Visit dentist for proper examination . Dentist will clean the socket with betadine regularly for 3-4 days .Wishing you a good health.Thank youDr Harry Maheshwari"
},
{
"id": 3357,
"tgt": "What are chances of successful conception with the given sperm analysis report?",
"src": "Patient: Hi Doc, my hus has below sperm factors.pls advise if we can try naturally. sperm count... 68 million/ml total count per ejaculate... 168 million motility fast proggressive..10% slow progressive.. 30% sluggishly progressive...15% non motile.. 45 % morphologhy normal forms..55% pls advise i have pco and my periods go missing once in 3 months Doctor: Hello dearI understand your concernIn sperm analysis: count is normal.Total sperm motility should be >=40%.So semen analysis is within normal limit and conception is possible.But PCOS can cause problem in conception.To regularize the period consult the gynecologist and undergo reproductive hormone analysis, USG scan and ovarian follicle study for better management.Drugs like krimson 35, metformin, clomiphene citrate, duphaston will help youDo regualr exercise and lose weight (10-12%) if you are obese.Take healthy diet, avoid stress and drink plenty of waterHope this may help youContact HCM for further health queryBest regardsDr.Sagar"
},
{
"id": 106392,
"tgt": "Do I have asthma or a heart related issue ?",
"src": "Patient: I have high normal blood pressure . My pressure in my lh arm was 135/74 and in my rh arm 153/85 yesterday morning. By noon I had shortness of breath and my lungs were tight I wheezed inhaling and exhaling. Do I have asthma or a heart related issue ? Doctor: you will be alright, i do not thing these are related to anything, i suppose you have to relax. saying within yourself, 'All is well\""
},
{
"id": 60290,
"tgt": "Does an SGPT level of 130 indicate that I need to quit smoking immediately ?",
"src": "Patient: i have a sgpt problem of 130 level. do i need to stop smoking male 24 yrs of age.. weight: 83kg, height: 5 7 , no medical history . i have a sgpt problem of 130 level. do i need to stop smoking? Doctor: Welcome to Healthcare Magic Good Day Yes, SGPT level is significantly high. You need to stop smoking and you need to stop drinking alcohol if you do drink. There is liver damage. It can be due to any cause. Hepatitis is a common cause due to alcohol, Hepatitis virus infection also. You may get tested for Hepatitis B infection as well if you haven't yet."
},
{
"id": 26649,
"tgt": "What causes heart palpitations?",
"src": "Patient: I have lost 55 pounds since March of this year. I weigh 228 pounds now. For the past 2 weeks, I have had heart palpitations. I started having Employee health record my BP. I have always had a normal bp of 120/80. Recently, it's been low for me. For example, today it was 110/69 and my pulse was 58. I have no other symptoms/problems other than the palpitations/hesitation. My physical is scheduled for a week from today. Should I try to get in earlier to see a doctor or should I just continue monitoring my bp/pulse until next Friday? I'm scared to do the wrong thing, but I don't want to panic either. Thank you for your help. Doctor: Hello and thank you for using HCM.I carefully read your question and understand your concern.But you should not worry,its nothing serious.As you describe, my opinion is that we are dealing with a rhythm issue.Someone can feel strong heart beats for different reasons. It may be a simple sinus tachycardia. This means that the heart is beating faster than 100 beats for minute. Its normal range in a resting condition is between 60-100 beats for minute. When it goes up you may feel a strong heart beat in your chest and a sensation like your head and body is beating too.Another reason for this sensation might be what we call an ekstrasystol.This are electrical heart impulses that generates in some parts of heart different from normal sinus rhythm.This generate that escaped beat that you feel.You have this sensation like a beat is skipping,you may experience palpitations or an anxiety sensation.This extrasistoles may happen from different reasons from simple emotional stress to other pathologies like anemia,hyperthyroidism, extreme weight changes ect.So, my opinion as a cardiologist is that you have nothing to worry and you don't have reasons to precipitate your doctor appointment.If I was your treating doctor and If these symptoms are interfering your daily live I will recommend some examinations like a blood analysis to exclude anemia,the thyroid hormones,an electrocardiogram, echo cardiography and of course a holter monitoring.This will help us to document and see what happens with heart rhythm. At meantime I will recommend you to avoid coffee,caffeine products because they rise extrasistolic beats. I encourage you to have physical training because sometimes it helps in reducing extrasistoles,tachycardia, even making them disappear.Wish I was helpful.Take good care.Best regards"
},
{
"id": 167458,
"tgt": "Suggest treatment for snake bite",
"src": "Patient: June 18th to 27th Scouts are 3500 to 4500 feet up on the Appliachia Trail. All are 11 nto 14. Already observe Copperhead and Rattlesnake. I have a suction kkit and Bendryl liquid. I am a EMT. I plan to suck the poison wth my suction unit. Wash bite area. I am 2-3 hours from hospital of physician, what is tghe treatment? Doctor: Treatment is antivenom. Reach hospital immediately. Till then tie up the part. Don't give any medicine."
},
{
"id": 184872,
"tgt": "What causes the feeling of glands under the jaw?",
"src": "Patient: last night i was feeling my own glands under my jaw as i was feeling a bit crook, as i was doing so i think a squeezed in a little too hard and broke my hyoid bone... feels like a litte wish bone that hugs around your throat, i heard a snap and my fingers fell in further... Doctor: Hello,I have read your inquiry and am concerned that you feel you may have fractured the hyoid bone. I suggest you consult an oral surgeon or ear, nose and throat specialist. It is rarely fractured unless there is violent squeezing. Muscles and ligaments hold it in place offering the protection of flexibility. Cartilage is also in the throat. This bone is also well protected by the mandible. There are blood vessels near by. Torn tissue is a possibility.Are you experiencing any changes associated with your tongue and speech? Do you feel gagging or cough with pain? Swelling or discomfort associated with swallowing and chewing indicate more reason to feel concern. Monitor for signs of tissue change with bruising indicating bleeding. If you have swollen lymph nodes this could indicate an infection due to mouth, sinus or respiratory conditions. You may have discomfort due to these conditions.Take anti-inflammatory medication such as Tylenol or Motrin. Benadryl antihistamine may be helpful. Do not stress the throat. Keep well hydrated with liquids and a soft diet. Avoid all irritants including smoking, alcohol, acidic and spicy foods and liquids, hard or crusty foods. Do apply any pressure to the throat or stress the jaw with excessive movements.Diagnosis can be difficult . X-rays, CT scan and a laryngoscopes may be recommended if a break is suspected.Thank you for your inquiry. I hope you feel better soon and continue with an evaluation if you continue to suspect a fracture and especially if symptoms increase."
},
{
"id": 12342,
"tgt": "Suggest treatment for plaque psoriasis on skin",
"src": "Patient: Hello Doctor,I am suffering from Plaque Psoraisis skin disease from nearly 20 years now.I have been on following medicine since then:DIPSALIC F (Salycilic Acid Ointment)But along the way now these medicines are not giving results. Please suggest some other medicines which I can replace these for.Or some other treatement that can be more effective . I have patches on my scalp,beardline, on my back. Doctor: Hello. Thanks for writing to us at healthcaremagicPsoriasis has a chronic course, the course being fluctuated by remissions and relapses, however there is no permanent cure for psoriasis.Treatment depends on the severity of the disease. Topical treatment is the mainstay for limited disease; widespread or extensive disease would require systemic treatment in addition to topical treatment.One reason for lack of efficacy of topical treatment is when the disease is extensive Or is spreadingTopical treatment options for psoriatic plaques includes mainly potent topical steroids ointments/lotions, either plain or in combination with salicylic acid.The ointment which you have been using contains clobetasol propionate and salicylic acid combination.Other options include Topical Vitamin D analogues like calcipotriol, calcipotriene etc in combination with or without steroids, are also effective for plaque psoriasis.However, topical therapy alone may not be sufficient for extensive disease Or spreading psoriasis. This might be a reason for lack of efficacy in your caseIn thses situations, Systemic treatment with Oral agents like methotrexate, Oral retinoids(Acitretin), Oral cyclosporin can be started in addition to topical treatmentI would suggest that you seek an appointment with a dermatologist in your region.Your dermatologist would be able to assess the severity and extent of your disease and advice accordingly.Hope this helpsRegards"
},
{
"id": 52588,
"tgt": "What causes elevated SGPT levels post kidney transplantation?",
"src": "Patient: Hi I have gone through kidney transplant on sep 2016... At that time my sgpt and sgot was normal but now it s increasing sgpt 235 and sgot 88.. Doc has given two med heptral 400 and ursocol 450 all test a b c are neg and ultrasound was ok normal size what shoud I do plz help plz :( Doctor: Hello and Welcome to \u2018Ask A Doctor\u2019 service. I have reviewed your query and here is my advice. According to mentioned history your SGPT level is high and for that you have been prescribed or so Deoxycholic acid. Both is drug protect the liver and can help in dissolving the cholesterol in bile. So you can continue with this drug. Your viral marker study and ultrasonography examination is normal and so no need to worry. You have to repeat your SGPT after few days and then if still it is elevated than we will do further work up. Sometime viral infection other than hepatitis virus can lead temporary elevation of SGPT. Hope I have answered your query. Let me know if I can assist you further."
},
{
"id": 51098,
"tgt": "High creatinine. Had pneumonia. Is dialysis needed?",
"src": "Patient: Hello sir, my mother is 92 years old ... she s been in the hospital almost a month now..because of her pneumonia absena .. now her lungs is cure..and doctor found that her a creatinine is high at 800.. and needs to dialysis . doees she need to go dialysis? is that critical? her heart is good and the neurologist said her brain also okay.. Doctor: Hello Thank you for getting in touch with us. As I see, your mother hasa creatinine level which is really high. Normal creatinine levels in the human body are less than 1.2 mg/dl and a level of 800 is really high. It would have been better for us if you had told us the Blood Urea Nitrogen levels as that would have helped us in recommending some suitable treatment for you. If you have those values, please feel free to share with us. As she is also suffering from pneumonia, it might be a case of multi organ failure. She needs to be kept under observation. Hope this helped Regards Dr. Gagandeep Dhillon"
},
{
"id": 216511,
"tgt": "Suggest treatment for knee pain",
"src": "Patient: I have knee pain beneath, and around knee cap, mostly on the interior side of knee cap. Knee feels stiff, but no swelling. I have had this issue for some time with no relief. Shooting pain comes with stress on joint, especially when trying to lift with resistance. What type of injury is this likely? What stepa should I take moving forward? Doctor: Thanks for contacting healthcare magic. You have problem of stifness of knee and pain problem that may be because of arthritis problem. Take one x ray for further diagnosis. Consent a physician for further diagnosis. Any injury may cause bursitis means inflammation of bursa.Avoid excessive straining of knee.Proper physical examination and history give exact idea. I hope my guidance is helpful to you. Take care. Thanks."
},
{
"id": 115177,
"tgt": "What is the difference between blood poisoning and poison ivy?",
"src": "Patient: what are some signs of blood poisoning? I have either poison oak or ivy. It spread to my arms and neck but my legs are TERRIBLE! Especially my right foot which is almost purple from the rashes and swelling. Also, my right foot is almost 1 and a half times bigger than my left! The whole right leg is swelling and poofy. Do i have blood poisoning or is this just a symptom from poison ivy? Doctor: Hello and welcome to HCM,A history of exposure to poison oak or ivy is the probable cause of allergic symptoms.The rashes and swelling developing on legs is an allergic reaction to the poison oak or ivy.This allergic reaction is different from blood poisoning.The allergic reaction is mainly tissue reaction and not blood reaction.You need to consult your primary healthcare provider for clinical assessment and treatment of the allergic reaction.Anti-histaminic is the group of drug used for treatment of alllergic reactions.Consult your primary healthcare provider for further work up.Thanks and take careDr Shailja Puri"
},
{
"id": 127291,
"tgt": "How can burning sensation in the fingers be treated?",
"src": "Patient: I have had a severe burning sensation in all of my fingers for a few days now, it went away but came back the next day , I also had a bad burning sensation on just one finger a few weeks ago, but it\u2019s getting worse and I just want to know how to make this burning feeling go away. Doctor: Hello and Welcome to \u2018Ask A Doctor\u2019 service. I have reviewed your query and here is my advice. Burning sensation in the fingers may be due to neuropathy pain. Common causes like carpal tunnel syndrome or diabetic mono neuropathy or disc herniation or prolase with compression of nerve root or infection or drugs or vitamin B12 deficiency etc. Until examination is done it is difficult to say what it is. You can Take tablet Pregabalin once daily for five days. consult your orthopedician he will do nerve conduction study and treat you accordingly. Hope I have answered your query. Let me know if I can assist you further."
},
{
"id": 75489,
"tgt": "Does tendonitis in pectoral muscle hurt during breathing?",
"src": "Patient: hi im a 48 yr. old female smoker, hx of respiratory issues such as copd, astma, i went to the pulmonary clinic they did a thoraic ct, thats when they seen the slight copd, and a nodule on my right lung 5mm, just watching it for, now but i have tendonitist in my pectoral muscle should this hurt when i breathe Doctor: Yes tendonitis can hurt while breathing or with any movement. The nodule in the lung needs to be kept under observation."
},
{
"id": 47078,
"tgt": "What cause headache and fever after diagnosed with nephrotic syndrome?",
"src": "Patient: hello doctor,my friend's 4yr old son has been diagnosed with nephrotic syndrome n is on steroids,.he's been complaining of severe frontal region headache with constant low grade fever..what could be the reason?doctors say it can range from sinusitis to CVST...what do we do?can you pls suggest ur opinion? Doctor: Hi thanks for asking question.Noted your friends child diagnosed with nephrotic syndrome..So if it is associated with inflammation of glomerulus that is glomerulonephritis then also such symptoms do occur.And secondly yes if cough , cold , sore throat present and headache and fever might be from acute viral infection..Treat it symptomatically.If abdominal symptoms present then abdominal diseases like hepatitis can lead headache.The steroid also can lead to headache ...I hope my answer will helpful to you...Take care..."
},
{
"id": 81934,
"tgt": "Suggest remedy for pain in chest with cough and sneezing",
"src": "Patient: thank you. a week ago this monday I fell forward and the front left chest area hit a vaccum clean and fell over with me as I landed on it. The next day I began to have pain in that area. And now the pain is intolerable. Ir hurts to breath, cough, sneeze and so on. There is no bruising. But the pain is horrible. What could I have done? Doctor: Thanks for your question on HCM. You are having blunt chest trauma. And this can lead to following complications. 1. Rib fractures2. Pneumothorax3. Pulmonary contusions4. Muscular injury. And in all of the above pain on breathing and sneezing, coughing present. So get done chest x ray to rule out all these.If chest x ray is normal than it is mostly muscular pain. Try to follow these steps for better symptomatic relief in muscular pain. 1. Avoid heavy weight lifting and strenuous exercise. 2. Avoid bad postures in sleep. 3. Take good painkiller and muscle relaxant. 4. Apply warm water pad on affected site. 5. Wear chest belt to avoid mobilisation of affected area."
},
{
"id": 80066,
"tgt": "Suggest treatment for epigastric hernia",
"src": "Patient: I am a 39 year old female, I had epigastic hernia op, with local anaesthetic some 8 years ago. immediateldy following op I found it difficult to breathe in my right side, and I seemed to, at that time have to push something in, to breathe, although this was some way to the right and lower of where the small op took place. the plastic surgeon who did the op, mentioned that I did not have a particular kind of scan from my gp, to show him, so he was unsure of to what extent there would be a hernia ,when he opened me up. I am wondering if only part of the hernia was seen and fixed,as the little pea sized bump which was the hernia is seemed to have become fine. I get electric shocks in my mid section, often, it seems my whole right side has become affected, with sensations in my skull on the right side of a kind of seizure. also my right side becomes stiff and sort of stuck, and my lower body seems to have contorted/twisted , with my toe rotated,the big toe,so my previous centre of balance is altered, when I manage to meditate and relax back to an equilibrium , right/left side, I can feel something in the centre of my body,which to me feels like the sixe of a broad bean, that really does not feel right. note, all of these and other symptoms have been discussed with different doctors, who mostly have put it to stress. also it really feels like there is a real physical thing wrong, which is debilitating on a daily basis, which I think, it somehow affecting me neurologically, rather than psychologically, as some have said. please help Doctor: Hi,Dear,Thanks for the query to HCM .I studied your elucidate-query in depth.I feel concerned about the worry and fear expressed by YOu for the wrong surgery done.In my opinion you require re-exploration of the repaired hernia to relieve post-operative local- electric shock like pains ...and others.I would diagnose this -\"as-post-operative-obsessive neurosis\",which needs to be investigated-with-Surgeon as follows-i-Please get the USG /CAT scan study,which would fix the reason creating electric shocks-which mostly is-due to the strangled nerve while repairing your epigastric hernia.ii-I would have done re-exploration -of the repaired part and would have exiced the fibroneurotic area -after clinical and operative localisation of the problem area as expressed by.iii-All your post-operative psychological problems are organic defect related and not merely due to psychologic behavioural prolbem-as felt by most of doctors. I do agree with you on this issue that there is a physical post-operative defect which is causing all the trouble you have.iv-So please act on the lines suggested by me herein above- and request your surgeon-for -a re-exploration of the repaired hernia-to remove the neuro-fibrotic area- involving the nerve in the repaired tissue.v-Hope you would recover with these health tips. I would love to attend your queries soon.Have a good time."
},
{
"id": 10786,
"tgt": "Could taking lexapro cause hairfall?",
"src": "Patient: Hi im an 18 year old male who has been on lexapro for about 6 months i had very thick hair before and now am thinning greatly all over and it does not follow mpb. I have read hundreds of accounts describing the same symptoms. Obviously this is awful and i am wondering if i get off lexapro the right way if my hair will be restored. Doctor: Hello. Yes, lexapro can indeed cause hair loss, although it is one of the less common side effects. Naturally, if you quit it, your hair will go back to normal."
},
{
"id": 223625,
"tgt": "Is it safe to use mtp cipla kit ?",
"src": "Patient: Hi, i am 23yr old female, wt 55kg, last yr i got married, after my marriage i took an mtp cipla kit, and then after 3 months an ipill and then again after 3 months another ipill.. After taking mtp kit, i started loosing hair and acne on my face, but now the problem is that i am loosing weight, the last ipill that i took was 4 months back,, can it be any side effect of these pills.?? Doctor: Hallow Dear, I believe, you had taken MPT kit for performing medical termination of pregnancy. Then twice after that you have taken I-pill pills at the interval of three months. I hope you took these pills within 72 hours after the unprotected sex each time as emergency contraceptive measures. These pills or MTP kit are not responsible for your weight loss and loss of hair; that too 4 months after the last pill had been consumed. Please get your general health check up done; in particular get your X-ray chest done for any evidence of tuberculosis and also get complete blood examination done. These symptoms usually are seen in general debilitating diseases like tuberculosis, anaemia and also in protein deficiency. Please increase the amount of proteins in your diet, consume good amount of green leafy vegetables and fresh fruits.Please do not ignore the weight loss. Report to your Physician ASAP>Dr. Nishikant Shrotri"
},
{
"id": 99993,
"tgt": "What causes dryness of mouth?",
"src": "Patient: I'm a healthy, fit 64 yr. old female and my voice has been \"croaky\" for two years. It's worse in the morning and about the same time I started noticing my voice problems, I noticed my mouth being dry when I wake up at night or in the morning. So, it appears that I'm mouth breathing at night. I don't have acid reflux, and my vocal cords have been scoped twice, but the last time was a year ago. I take a probiotic with a broad spectrum of helpful bacteria and a total of 60 billion organisms, and this has helped somewhat. My voice seems to clear up more in the summer and when I go to the coast sea kayaking each Sept.- probably due to more moisture. I do sleep with a humidifier. I don't know if I have post nasal drip or allergies, other possible factors, as I'm not symptomatic for these things. I also tried nasal irrigation using a Neti pot last spring, but didn't notice much difference. I used to have a powerful voice and did some public speaking, but now my voice is embarassing and is hurting my career in sales. I'm very fit and active and live in NW MT and ski/hike/ workout 5-6 days/week, and don't look my age, but sometimes sound like I'm 90. HELP! Doctor: HI, thanks for using healthcare magicSome persons may have all or only some of the symptoms associated with a medical condition.The fact that the voice change is worse in the morning may indicate that it may be related to post nasal drip which is normally worse at this time or in the evening period.Some persons may be unaware of the drip itself.You may want to consider allergy testing to see if a particular trigger can be identified and then eliminated.In addition the use of a topical steroid nasal spray and oral antihistamine may also help.I hope this helps"
},
{
"id": 204307,
"tgt": "How can panic attack along with social anxiety be treated?",
"src": "Patient: Hello I think I m having mental health issues that keep getting worse every year.... everyday I wake up feeling like I m doomed, lump in throat, no energy, no emotion other than just miserable, I get really overwhelmed and just on edge to socialize or do work etc... I m not suicidal but every night I go to bed I just wish I wouldn t wake up... there s more but I just done know how to explain it all, basically I m always having panic attacks before doing anything and no energy motivation to do anything that needs to be done... I m always on edge and can t focus on anything just dreading responsibility and overall miserable Doctor: Hello and Welcome to \u2018Ask A Doctor\u2019 service. I have reviewed your query and here is my advice. From the given details it is clear that you are anxious and have anxiety disorder. As you have mentioned it could be social anxiety disorder. The panic problems are stemmed out of the social anxiety. You need to treat the anxiety to get rid of panic attacks. I suggest counseling and CBT. Hope I have answered your query. Let me know if I can assist you further. Regards, Dr. K. V. Anand"
},
{
"id": 139261,
"tgt": "Suggest treatment muscle tightness in legs and arms",
"src": "Patient: I have muscle tightness and restlessness in my legs and arms (no pain) on a daily basis and at night also. I have a problem getting to sleep or continue to wake up and am sometimes up until 4 a.m. I keep wanting to rub them. I m 77 and I do walk 45 minutes to 1 hour a day. I am already on a prescription of Tramadol 50 mg (one to two pills at night) to help relieve the tension and help in sleeping. It s really getting to me. My current physician did not prescribe anything. Doctor: Dear Sir/Madam,I thank you for having faith in Health Care magic and for seeking help on your issue; I have gone through your symptoms, and in my opinion, if you do not have pain then discontinue tramadol it does not relax your muscles, I would recommend you to take bath before going to bedtake a mild sedative drug after prescription get your Vit D and Cal level done and take supplements if lowtake Vit E 500mg daily for 2 week and then stop.discuss this with your GP before following and learn to Relax.I hope I have been able to provide you with a satisfactory solution, if you want to clear any doubts about my opinion, feel free to contact.Special note- Any medication prescribed needs to be taken after consultation with your personal doctor only.ThanksDr. Narender Saini"
},
{
"id": 10364,
"tgt": "How to prevent hair loss?",
"src": "Patient: I am 27 years old, I weight 162 pounds and I'm 5'2. Lately my hair has been falling out. I've consulted this with my regular doctor, he says that I'm probably loosing my hair due to stress. He offered to prescribe me anxiety medication without doing any type of test, blood test etc. I honestly do not believe this, since I know I am not stressing more than the usual. As far as I'm concerned I am healthy, I've never had any serious health problems. Please help! At this pace I feel like I'm going bald, you can actually see a lot of my sculp now. How can I prevent loosing my hair, what can I do to promote hair growth? What kind of testings do I need to find out why I'm loosing so much hair? Doctor: Hello and Welcome to \u2018Ask A Doctor\u2019 service. I have reviewed your query and here is my advice. You might be suffering from androgenetic alopecia.you can try topical minoxidil and PRP treatments. Get a blood work up done prior to that. you can even consider hair transplant as you are young and the results will be rewarding. Hope I have answered your query. Let me know if I can assist you further."
},
{
"id": 194386,
"tgt": "What causes itching and redness of the penis?",
"src": "Patient: Hi just recently my bell end has been itchy and red its cracked and bleeding aswell but today I have noticed my foreskin has gone really tight when flacid and I cant pull it bback im 25 and have recently been using ciprofloxacin for a urinary tract infection Doctor: Hello, You are having balanitis as per mentioned history. It is inflammation of glans penis foreskin. You can be prescribed a topical steroid for your phimosis problem. Phimosis means you cannot able to retract foreskin back over glans. If no improvement than minor surgery can be done to remove foreskin called circumcision. I suggest you consult Urologist for examination and discuss all these. Hope I have answered your query. Let me know if I can assist you further. Take care Regards, Dr Parth Goswami, General & Family Physician"
},
{
"id": 218312,
"tgt": "The scan report says right lateral vertical is mildly prominent and within the normal limits. What is to be done next?",
"src": "Patient: Hi myself sundar and my wife is 22nd week pregnant and our doctor advised her to take livogen z & triple a cal tablets and scan report says right lateral vertical is mildly prominent m s 7mm within the normal limits & normal limits is 9mm. So what to be done for this? Should she take any extra tablets for this? Doctor: Hello and Welcome to \u2018Ask A Doctor\u2019 service. I have reviewed your query and here is my advice. As per this scan report, one of the right ventricles (cavities in the brain) is slightly enlarged. Nothing much to worry about it now. She can take folic acid tablets for brain maturation. Frequent follow up scan would be better and do a neuro sonogram after birth. As of nothing much to worry and be relaxed. Hope I have answered your query. Let me know if I can assist you further."
},
{
"id": 49266,
"tgt": "Dull ache around the right kidney area with foamy urine and negative dip test in a high BP and cholesterol patient",
"src": "Patient: Hi, I have a dull ache/tenderness around the right kidney area. I don t have diabetes, but I am on high blood pressure medication and lipitor for cholesterol, I did dip stick test on urine and it came negative on protein, although urine apears to be foamy recent blood tests are normal Doctor: HelloThanks for your query,based on the facts that you have posted it appears that you have UTI which is likely to be associated with stone either in kidney or ureter.Please get your routine urine test and urine culture done to find out the organisms causing this infection and antibiotics to which they are sensitive to.Ultrasound scanning of abdomen will help to rule out presenceof urinary stone.Please take broad spectrum antibiotics like Cefexine along with urinary antiseptic like Nitrofurantoin twice daily and urine alkaliser thrice daily.Later on switch on to appropriate antibiotics as per culture report.Get the prescription of medicines from your family Physician.Ensure to drink more water.To keep your urine dilute This will help to control dysuria."
},
{
"id": 3539,
"tgt": "How can I conceive?",
"src": "Patient: Last month I had tried to conceive but had an early period which lasted for two day. usually my period lasts for 6 days. my reports say that my uterus is anteverted normal in szie and echotexture, plus same goes for the ovaries. just want to know how logn will it take to conceive as i ahve a busy schedule Doctor: Hi,Welcome to Healthcare magic. I am Dr Ramadevi wani. I will be answering your concerns.Since your period usually lasts for 6 days, there is a small possibility that the 2 day period you had is implantation bleeding. Just get a pregnancy test done.It would have been nice if you had provided me some more information like your age, BMI, duration of marriage, regularity of menstrual cycles etc.The ultrasound report is perfectly normal.Among couples trying for pregnancy, 80% of them conceive in the first year and 90% by the end of second year. So don't be worried. You will conceive.If it is more than two years of trying, then consult doctor and get baseline hormone tests and HSG for tubal patency for you and semen analysis for your husband done. The tests results will help in planning the treatment.To increase your chances of conception maintain normal BMI, quit smoking ( if you're a smoker) and have intercourse at least three times in a week.Take a tablet of folic acid daily.I hope this is helpful.If you have any further concerns do contact me through Healthcare Magic.Best wishes,Dr Rama"
},
{
"id": 135547,
"tgt": "Suggest remedy for pain in ribs",
"src": "Patient: I ve had a pain in right side around rib area going on 3 weeks. No injury very painful to touch hurts to laugh cough when I lay on that side I can hardly get up. I ve had chest X-ray ultrasound nothing. Supposed to get ct Next week. Pain meds do not help. When I lay down I almost feel like there s a knot Or something?? I m so frustrated & need to figure out what this is. Any suggestions? I ve had gallbladder Out 2 yrs ago. Doctor: Hello,I have studied your case and I think that it can be due to hair line fracture in ribs which are not seen on x rays but it can be see on CT scan and even if there is no fracture present on CT symptoms are suggestive of bony injury. You need to take NSAIDS and also need to avoid weight lifting or any jerky activities. This pain will take some time in improvement."
},
{
"id": 93776,
"tgt": "Abdominal pain after intercourse. Taken Advil and Gas-X. Hurts to walk, cough, sneeze. Painful bowel movements. Cause?",
"src": "Patient: Last Friday, beginning about 10 minutes after intercourse with my husband, I experienced severe lower abdominal pain for approximately 2 hours before pain began to subside with help of Advil and Gas-X. Today is Wednesday and I am still experiencing pain. It hurts when I walk, cough, sneeze, stand up from a sitting position, et cetera. The sharp pains near my ovaries are gone today, but I am also suffering from trapped gas in my abdomen and pressure in my rectum. I have the urge to defecate, but I can't. I had a bowel movement this morning but it was painful.What is causing all of this, and why has sex triggered it suddenly? Doctor: Hi welcome to Health care magic forum. Thanks for calling H.C.M.Forum. you have got Lower abdominal pain started 10 minutes after intercourse. After 6 days also you are feeling the pain, while walking, cough, sneez, stand, etc. It is paining during defecation. It appears to be the infection of the vagina, anus, rectum. Other causes could be pelvic infection, urinary tract infection, stones in the urinary tract, etc. i advise you to consult a surgeon for diagnosis and treatment. You may need to have M.R.I.besides other routine tests for confirmation. Wishing for a quick and complete recovery. Best regards"
},
{
"id": 141809,
"tgt": "What causes a sting sensation in a cyst on the back of the neck?",
"src": "Patient: I have a cyst on the back of my neck that was drained a few months ago. Started getting a sting sensation yesterday. What does it mean? I am supposed to have it surgically removed but can't afford it right now (it swelled up to the size of a golf ball before the drainage and took over a month to stop draining!). Doctor: Hello,The sting sensation could be related to an infection. For this reason, if it hurts when touching or the stinging persists, I would recommend consulting with your general physician for a physical exam and some tests (complete blood count, PCR) for inflammation. You may need to drain it again if its dimensions increase. Hope I have answered your query. Let me know if I can assist you further.Regards, Dr. Aida Quka"
},
{
"id": 14326,
"tgt": "What causes red bumps on back and inner thigh?",
"src": "Patient: My almost 3 year old son has red bumps almost like a rash beginning on his lower back to butt and inner thighs. They are in clusters and also by themselves. In the last few days they have spread. He also says they hurt. I know it is not diaper rash but not sure what it is Doctor: Hi, it may be lichen urticatus..also called popular urticaria. It is considered as hypersensitive reaction to insects.Kindly consult the dermatologist. I would suggest Antihistaminics, steroids in tappering dose, mild steroid cream..etc. And avoid contacts with insects...okThanks.Dr.Ilyas Patel MD"
},
{
"id": 170367,
"tgt": "What causes brown patches on the skin?",
"src": "Patient: Hi. I have a 9 month old daughter , she has got brown pathches and we consulted the skin specialist , they said it is just an molted spots and gave skin cream to apply on it and will get fade as she grows . Am afraid if its NF. Is it possible to DNA test and find out the same. please could you give me some help Doctor: HiCafe au lait spots also can give brownish coffee coloured appearance on skin. But it's very unlikely to be missed by any trained doctors. Six or more caf\u00e9-au-lait spots or hyperpigmented macules =5 mm in diameter in prepubertal children and 15 mm postpubertal if present than a thorough examination and laboratory evaluation to rule out neurofibromatosis will be useful.Hopefully it will be of help. I would be happy to help you in any further questions. Take care"
},
{
"id": 182715,
"tgt": "How to treat pain in right jaw after retrieval of wisdom tooth?",
"src": "Patient: my right side of my jaw hurts when i open it. i put ice on it but i feel like its getting worse. i feel like its swollen ever since i put ice on it. i had all four wisdom teeth pulled out about a year or year and a half ago and i havent had any problem... whats going on? Doctor: Thank you for your query,The chances of your pain and infection due to extraction of your wisdom teeth which were extracted year before are rare.As I can understand from the symptoms written, this pain may be due to infection in your jaw muscles.I would advise you to take muscle relaxant and consultant your Dentist to rule out any other pathology."
},
{
"id": 143724,
"tgt": "What causes the motor skills to become abnormal and severe ulcers on both the legs?",
"src": "Patient: Hi, today I witnessed my elderly father(80 years old) suddenly find it difficult to walk. He could not seem to move his legs . My mother had to help him sit in his wheelchair and put him to bed. His hands were twitching constantly and he was unable to hold anything. He has been unable to chew or swallow his food however he did drink when a cup was held to his lips. My father has experienced bouts of this over the last 3 months,the last one being 9 days ago. My Mother has told me that usually after a few hours my Fathers motor skills and speech become normal. However todays attack was longer than normal. He has suffered with severe ulcers on both his legs over the last 4 years and has been hospitabilised a number of times with bad infections. My Mother has consulted the GP and they have refered my Father to a neurologist however we are still waiting to see the consultant. Can you give us some understanding of what could be wrong with my Father please Doctor: Hi,I am Dr Mittal.I have read your message.I think I can help you.The symptoms of the query are quite suggestive of a stroke. The sudden onset of the symptoms, the motor weakness, the swallowing and speaking difficulty, the shaking of hands. All symptoms are suggestive of the diagnosis of stroke. The fact that he had been having similar episodes over the last 3 months is suggestive of mini strokes that we call TIA medically. These are usually warning signs of an impending stroke.I am unaware from the query about the duration of the symptoms. Within 4 to 8 hours onset should be treated urgently and should be visiting an EMR as soon as possible. He will need urgent treatment.If it is more than that, he needs to meet a neurologist who will probably perform a CAT scan of the brain with or without the angiogram (depending on the protocol of the hospital).As per the report of the CAT scan, medicines may be initiated. I think a low dose statin and antiplatelets should help. I would suggest a scan prior to the initiation of the medicines because the closest differential is a possible cerebral hemorrhage (less likely in view of the mini strokes) where these medicines are not to be given. Meanwhile, i think you should start him on physiotherapy for the motor weakness.Another possibility I want to consider here is possible diabetes and hypoglycemia attacks. If the GP has thought about this as a possibility and tested sugars, there is probably nothing to worry about. Otherwise hypoglycemia in a diabetic patient would explain the brief episodes. And a hypoglycemia can also explain the prolonged symptoms. In such a case, sugars need to be well monitored.The third possibility I need to consider is a vasculitis. This is a common cause of stroke, but usually is thought of in young patients. The last 2 causes I have considered in the view of preexisting foot ulcers which cannot be explained by TIA or current stroke as the ulcers predate the stroke. I think you need to get the scan and meet up with a neurologist. Unfortunately, as explained, I cannot suggest medications without the scan.I have tried to make it as simple as possible. Please feel free to contact me for any further clarification. Best of luck, Dr Mittal"
},
{
"id": 137144,
"tgt": "Suggest treatment for pain in hand when suffering from cortikobasal degeneration",
"src": "Patient: My husband have cortikobasal degeneration. He is now in hospital suffering very bad cramps in his hand. Hand wants to take itself to wrong possision on to his chest and it makes horrible pain. He is given medication for the pain but nothing helps to this pain given movement. Tramal baklofen lyrica and rivatril are what they give to him now. I want to know if you have some ideas for us to help him. Doctor: Hi there.You are having muscle spasms. Apply hot water bottle and warm soaks to reduce this pain. Also consult your Doctor regarding the use of an effective anti-spasmodic medication called Baclofen which will offer good relief."
},
{
"id": 166752,
"tgt": "What causes snoring in a 6 month baby?",
"src": "Patient: hi Girish here from Bangalore. My 6month old baby has snooring problem always (awake or during sleep)...local paediatric doctor said it can be due to wind pipe & sound box stick together & assured us that it will gradually become OK over a period of time..but I am not fully convinced...could you please advice on further steps? Doctor: Hi,Snoring has different cause and at this age group your doctor has advise correctly. With the developing age, I hope the problem will resolve. At times it can be due to over weight so you have to check the weight of your baby. Otherwise it is quiet normal, and nothing to worry about much. Wishing your baby a healthy life.Hope I have answered your query. Let me know if I can assist you further. Best Regards,Dr Zeeshan"
},
{
"id": 120912,
"tgt": "What causes hole in skin bone?",
"src": "Patient: I am 20 and three years ago doctors found a hole in my left skin bone , he did a x-ray and said the whole was ok as it was smooth around it , but just recently I have noticed Iv developed two more holes on my right skin bone . What should I do and do you know why this is occurring Doctor: Hello,It is important consulting again with your attending physician for a physical exam. I would also recommend checking vitamin D and electrolyte plasma levels for possible deficiency.Hope I have answered your question. Let me know if I can assist you further. Regards, Dr. Ilir Sharka, Cardiologist"
},
{
"id": 18744,
"tgt": "What causes numbness in arms and blood and pressure in nose?",
"src": "Patient: last tuesday (9/21/10) i went to a concert and i was crowd surfing i fel on my head my arms went numb for a few seconds left a goose gedd with minor blood felt some pressure in my nose not really sure what i felt exactly and right now i dont remember much from the concert. i remember what songs they played i jest dont remember if they sounded good or not. and i had some head akes a few days after the concert(i expected that) im jest asking this cause my grandma is worried about me as for me i dont think its a big deal Doctor: Hello and Welcome to \u2018Ask A Doctor\u2019 service. I have reviewed your query and here is my advice. If you are feeling no severe headache or vomiting or convulsions and 72 hours have passed then no problem .You should take painkiller as tab Aceclofenac two times. If you are having above mentioned symptoms then you should undergo NCCT ct scan head as soon as possible. Hope I have answered your query. Let me know if I can assist you further."
},
{
"id": 113880,
"tgt": "How to overcome recurring backache ?",
"src": "Patient: Hello, I m Yogesh H Pathak, age 36 yrs, weight 90 kg. suffering back problem since last 20 days. The pain is exactly at below left shoulder side and due to continuous pain now my left arm also paining. I had visited Rehabilitation center and took traction for 10 min daily and hot water therapy for 4 days. After that my pain was reduce but not 100%. Still sometime its paining. Kindly suggest for what reason its happening and how it will sort out. Thanks Doctor: Welcome to Healthcare MagicThis kind of pain probably indicates spinal nerve compression. Have you taken X-ray/MRI. Are you having loss of weight recently. Any cough. You need to get some tests like complete blood counts, Haemoglobin, if you have cough you can get sputum test to rule out TB. Are you a smoker. It would be best if you consult with Neurosurgeon first. Avoid doing any heavy activity like lifting weights. Eat high protein diet like pulses, cereals, eggs, fresh fruits and vegetables. Drink plenty of water. Exercise daily. Kindly reply on the above issues."
},
{
"id": 224620,
"tgt": "Suggest best ways to prevent pregnancy",
"src": "Patient: Hi, am 23 boy and in a relationship for the past 4 years. I wana know what are the best ways to avoid pregnancy for my gf and have safe sex. Under no circumstance i want her to get pregnant. Tell me the best contraceptive plzzz. I am waiting for your answer Doctor: HelloThanks for writing to us with your health concern.Condoms are the best choice as they protect against sexually transmitted diseases also.Again, if not willing, birth control pills can be used by her eg. Femilon.Take care."
},
{
"id": 146416,
"tgt": "What does focal region of cystic encephalomalacia/gliosis in cerebral hemisphere mean?",
"src": "Patient: My husbands MRI came back stating there is a focal region of cystic encephalomalacia and gliosis in the left cerebral hemisphere primarily within the portion of the head and body of the left caudate nucleus. I have been trying to find a explanation online with no luck. Please explain what that means and symptoms. Doctor: Hello Cystic encephalomalacia/gliosis refers to loss of normal brain tissue and it is filled by glial cells.It may be due to many reasons like post trauma,infection,ischemia/stroke etc.Gliosis is a reparative mechanism in brain tissue.There is small area of gliosis in your husband's case. This condition doesn't need any specific treatment.It is an healed stage and patient only need symptomatic treatment.Get well soon. Take CareDr.Indu Bhushan"
},
{
"id": 30333,
"tgt": "Suggest care and precaution needed for malaria",
"src": "Patient: Hi, may I answer your health queries right now ? Please type your query here...hi i want 2 enquire abt a friend of mine.he is 20, he is a poor eaterand has recently develpoed maleria in the past 4 days and he went today he has been very cold and developing high fever. and painful lungs. a doc came home and said its pneumonia.i wanted to know how long would it take for him 2 recover overall. what would be best for him to do and avoide, he has also stoped eating, he complains he cant eat, what is best to eat????hes become very week. please help me!!! Doctor: Hi thanks for asking question.here is some advise for you.Take enough rest.Drink plenty of fluid to prevent dehydration.Avoid smoking and excess spicy food.Take fruit juice and fruits.If solid diet cant taken take semisolid food or fruit juice.herbal remedies are helpful.pleuricy root helpful.Garlic and ginger are well known remedy for respiratory condition.Carrot juice and vitamin c is helpful.Green leafy vegetables helpful.Complete course of antimalarial drug and antibiotic course has to be taken.Overall recovery will mostly take around 15 days.If still no benifit then further work up should done.I hope my suggestion will help you and you will get well soon."
},
{
"id": 90268,
"tgt": "What causes abdominal pain, testicular pain?",
"src": "Patient: My name is Ivan ortega, I was diagnosis with varicocele Left testicular constution. But, the insurance denied the surgery in 2004 and now I was diagnosis with Hydrocele, Varicocele Bilateral Constution. Abdominal gastrointestinal pain, and many doctor say that I am toatl 100% total diabled. The cause of this was: I was working as usually full time in a Arabis Company and a heavy box hit in my testicles. Please help to know if the disability is permanent? and if the surgery does not guaraanteed the prostate enlarge, testicular pain, abdominal pain go away or disapear? a Doctor told me that there is not garanteed, that I would lost my both testicles? Text me., Ivan ortega 0000/ 0000...86-12 126th, Street,. Richmond Hill, New York 11418... I can hire you for a Supreme or Fexderal Court deposition ! Doctor: HIThank for asking to HCMI really appreciate your concern if this varicocele then in my opinion this need to be repaired as early as possible to prevent further damage and any thing worse happening chronic state of this disease could be having bad clinical out come, it is all depends upon the type and status of defects and that can only be seen after opening the scrotum, take care and have a nice day."
},
{
"id": 66601,
"tgt": "What causes a lump in the left forearm?",
"src": "Patient: hi sir, i have an lump in my forearm left hand since i was age of 13. its not paining.. i have done the sonography and report comes shows that there is lump bi-lobiluted, mass. pls advice the reason why its occured also if i go under surgery to remove the lump will it come again?? what precaution should i take?? Doctor: Hi, dearI have gone through your question. I can understand your concern. You may have some ganglion cyst or other soft tissue tumor like lipoma or neurofibroma. You should go for excision biopsy and histopathological examination. If ganglion cyst is there then there is a chance of recurrence. Other tumors are not known for recurrence. Consult your doctor and take treatment accordingly. Hope I have answered your question, if you have doubt then I will be happy to answer. Thanks for using health care magic. Wish you a very good health."
},
{
"id": 220210,
"tgt": "How to check out for possibility of pregnancy without tests?",
"src": "Patient: i think i could be pregnant. the first day of my last period was march 19 2010. i havent had my period this month and i m having all symptoms of it except a positive test and severe pain. i ve taken tests online and they all say i may be. how can i really find out without getting a home test? Doctor: in delayed cycles sometimes urine test is delayed positive. to confirm it go for serum beta Hcg a blood test.if Hcg test negative then you are not pregnant.symptoms can be due to hormonal imbalance."
},
{
"id": 207162,
"tgt": "How to overcome habit of masturbation?",
"src": "Patient: I used to masterburation at least 1 times in a week.I feel heavy nervous and i feel very sinful .pl tell me if i am doing any wrong think for my health. i used to go gym everyday. I want to give up this bad habit.pl sugest me..i am waiting for your favourable reply Doctor: Hi dear,Thank you for your query.Masturbation is not a bad habit and it is not affect your health any way.it is a normal phenomena and 99% of male is doing it .if you do not masturbate your semen comes out as night fall.So don't take it as a bad habit.Thank you"
},
{
"id": 83019,
"tgt": "Suspicion of temporal arterius. Blood work negative for lupus. Is flying and sun tanning safe?",
"src": "Patient: I am not sure I have Temporal Arterius but I will be having a CT Scan and MRI completed next week.. My Neurologist took a lot of blood work last week which all was negative to diseases such as Lupus but would Temporal Arterius have been diagnoised with the blood work I too? Also is flying and sun tanning harmful with temporal aretrius or any other blood vessel problem ? Doctor: hi temporal arteritis is more of a clinical diagnosis, with a high esr and the final is actually a temporal artery biopsy or nowadays a pet scan."
},
{
"id": 15547,
"tgt": "Have Skin lumps. When squeezed have pain. Seems to be a sebaceous cyst. Treatment?",
"src": "Patient: All of a sudden Ive has random under the skin lumps appearing. One on my inner Labia I have tried to pop it but nothing happens. Another on my groin area under the skin which nothing happens whenI squeeze just alot of pain I had recently went to the doctors who took a quick glance and in a minute said the one I showed her was a sebaceous cyst.A few days later another appeared under my skin on my butt. I didn't think sebaceous cysts could just appear like this out of nowhere. Do you know what this couldBe Doctor: Hi,Lump you have in inner labia might be due to sebaceous cyst now becomes infected and giving rise to enlarge lymph nodes in groin.Another lump on butt might be due to ingrown hair follicle infection.Go for one course of antibiotic after consulting your doctor.If there is infection in sebaceous cyst, get it removed with sac.Ok and take care."
},
{
"id": 96773,
"tgt": "Suggest treatment for dislocated shoulder",
"src": "Patient: hi doctor i am 21 years old man 2years back i fall down from the bike my right shoulder was dis located my hand turned back that time some unknown person came and he turned my hand correct position and next day i went to hospital so doctor just given some medicine and he gave one belt to my hand after one month i removed that n doing my work some day when i throwing the boll it was again dislocated and with in seconds again set again so from that day when throwing something it getting dis located now yesterday i went to swimming pool when i swimming my hand was to much dislocated again so from yesterday to much paining i cant lift my hand also.....so plz tell me sir Wat i have to do i have X ray also.... Doctor: Hello,It is habitual shoulder luxation, and you need orthopedic surgery to stabilize the shoulder joint.Hope I have answered your query. Let me know if I can assist you further.Regards,Dr. Iven Romic Rommstein"
},
{
"id": 213330,
"tgt": "Getting dad arrested for having sex with me since teenage, facing difficulty to cope up with it. Suggestions?",
"src": "Patient: Im having a hard time coping with the fact that Im going through court to have my biological father arrested for having sex with me from age fourteen to six months ago. I am only turning eighteen in five months. I feel like I am doing something wrong by having him arrested. I cant even talk to my boyfriend about it. Whats wrong with me... Doctor: Hi, This question involve lots of moral, ethical and legal complication. Following steps will help you: -Discuss matter with your mother or close family members regarding your role in case. -Do not be judgmental and tell only truth and your feeling in court of law. -Maintain your routine, study, interaction with friends and sleep like your previous self. -Also remember: time is biggest healer and things will solve soon. I hope this information has been both informative and helpful for you. Wish you Good Health. Regards, Dr. Ashish Mittal www.99doctor.com"
},
{
"id": 85322,
"tgt": "What are the possible side effects of foracort 200?",
"src": "Patient: Morning! I have been taking forecort 200 for almost 2 yes now. It has been prescribed my doctor. What are the possible side effects of foracort 200? Currently I have been suffering with a constant weight loss and fall sick every third day. Also recently I vomit out after I eat. Please advice Doctor: Hi, Cannot say in your particular case, but generally, this isn't a direct action of this drug. It can cause yeast infection, mainly at the back of the throat. It can speed up the heart, maybe. Certainly, all asthma inhalers are sprayed into the lung and can trigger asthma and lung irritation in a few sensitive people. Hope I have answered your query. Let me know if I can assist you further. Regards, Dr. Sameen Bin Naeem, General & Family Physician"
},
{
"id": 135329,
"tgt": "Suggest treatment for swollen and painful hand",
"src": "Patient: Dear Dr, i had donated blood around a week ago. After 5 hrs, area where needle was inserted swelled and there was so much pain that i was not able to bend my hand. After 1 day pain and swelling reduced but there is still redness at some distance from that spot. i observe that redness is increasing and spreading to other parts with greenish effect. So is there any major injury due to needle? Pratik Doctor: Hi Dear,Welcome to HCM.Understanding your concern. As per your query you have swollen and painful hand. Well there can be many reasons for symptoms you mention in query like hand injury , hypothyroidism , lymphedemaedema or allergy . I would suggest you to keep your hands clean and dry , apply petroleum jelly , take benadryl twice a day and take ibuprofen or acetaminophen for pain . If condition doesn't get well then consult dermatologist for proper examination . Doctor may order anti inflammatory along with immunosuppressants along with antibiotics . Doctor may also refer you to endocrinologist for thyroid problem . Hope your concern has been resolved.Get Well Soon.Best Wishes,Dr. Harry Maheshwari"
},
{
"id": 100612,
"tgt": "What causes flushing in face and lips?",
"src": "Patient: Hello. I wonder if you could tell me about a symptom I've been having with increasing frequency over the last month or so, now almost constant over the last 2 or 3 days. I'm a 45 yr old male and my lips often get flushed, sometimes bright red, and today my face feels flush as well. More symptoms/suspicions if you need them. Thanks. Doctor: Hello.Thank you for asking at HCM.I went through your history and would like to make suggestions for you as follows:1. I would suggest you to take an antihistamine like hydroxyzine for 2-3 days. It will help.2. I would also suggest you to avoid hot/spicy foods for a week if it is possible.Hope above suggestions will be helpful to you.Should you have any further query, please feel free to ask at HCM.Wish you the best of the health.Thank you & Regards."
},
{
"id": 42370,
"tgt": "Is there still a chance for a successful pregnancy?",
"src": "Patient: sir i am a married man for 20 years now my age is 47 and my wife is 37.we are still awaiting our child.in the 2nd year of our marriage my wife conceived but due to hypertension as doctors stated she got abortion in the 8 month.after that she developed a gaanth and 1 fallopian tube was removed.since then means from 1997 we are trying hard for child we attempted many IUI and recently we went for IVF for 3 cycles but with negative results.do you think we still stand a chance to get our child.?she started getting irregular periods irregular ovulations her eggs were not hatching on time etc.my semen showed 20% 1 grade 40% II grade with count of 10 millions kindly suggest Doctor: HelloAs far as known to me \"gaanth\" means tumor in fallopian tube and one removed . So make clear one thing in your brain that ovulation occur once in the time of two months because one tube is no more . Your semen analysis is not complete in this query , so I will write down about fertility.Normal fertility is associated with sperm counts of > 48 milliom /mL with a motility of >63 % with 12 % exhibiting normal morphology .SUBFERTILITY is seen with sperm counts of You only mentioned that there is 10 million sperm ( a total or in one mL???).First of all get in follicular study of your wife , ovulation time , size of dominant follicle , maturation date , burst date , endometrium thickness . Still you can have baby but it depends upon many circumstances , so either upload all reports or consult one more infertility specialist and get his opinion.Hope this information will be helpful for you."
},
{
"id": 155301,
"tgt": "What causes bleeding in eyes while on treatment for multiple myeloma?",
"src": "Patient: My father is being treated for relapsed multiple myeloma. He is taking ecosprin 75, lenalidomide 15 mg daily, bortezomib 1.8 mg weekly through IV and des methadone 10 mg weekly. He has had several side effects including fever, cold, cough. Today he also developed a sub conjunctival haemorrhage I. His right eye. Should we review the dosage of medicines he is being given because his present weight is only 45 kg. what has caused the eye bleed? Pls advise. Doctor: The only reason I can think of for the hemorrhage is ecosprin since it is an antiplatelet drug. It should be stopped in consultation with your oncologist. Also, get a complete blood count done and see if the platelets are adequate. multiple myeloma can cause bone marrow suppression and fall in platelet counts and bleeding."
},
{
"id": 54150,
"tgt": "Suggest treatment for collapsed gall bladder",
"src": "Patient: i developed gall stone pain during pregancy i am 36 and since my daughters birth have had many painful episodes. i have had 3 scans wich show my gall bladder has collapsed but they cannot see any stones however the pain i get is in my upper right side under my chest this spreads around to my back & into my right shoulder and is excrutiating. the last episode was so painful i admited myself to hospital hoping to have it sorted however after been kept in over night i was discharged yesterday although they are aware of the gall bladder i am still terribly uncomfortable and have had very dark brown urine today could this be connected? Doctor: eg:lsee cholysistectomy must done until do antispasmodic and fluid diet restrict lipidand fat dark urine during to complication of gall blader"
},
{
"id": 209568,
"tgt": "What makes my hands sweaty in social situations?",
"src": "Patient: in mostly social situations, i start to get freezing sweaty hands and feet, and my whole body shakes. with this, i would assume its anxiety, but mentally i do nothing of the normal symptoms. i dont feel abnormal, or self conscience about anything. im completely calm. i have no trouble talking to people, but these physical attributes i cant control. and its not every time in a social setting, only maybe 65% of the time. please help, what should i do? Doctor: According your history if you don't feel any anxiety mentally and only sweating and shaking , it can be due to your hyper sympathetic nervous system.And if it is so it can be controlled by propranolol or other medicines.However you didn't mention your age and sex, I advise to consult physician if required for evaluation of thyroid disorder and if your doctor feels, he can refer you to a psychiatrist for social anxiety disorder."
},
{
"id": 87354,
"tgt": "What causes abdominal pain after laproscopic procedure to correct the cyst?",
"src": "Patient: A month after laparoscopy procedure, my wife is suffering from pains on her lower right abdomen side, right underneath her rib cage. She describes it as a pressure pain, that coincidentally, she claims before the surgery this very same pressure pain was located lower, almost to where her appendix would be. What could be causing this pains? To give you more info: The procedure was to correct a cyst, that was missed by her gynecologist, but spotted by her gastroenterologyst, due to the fact that it was located outside her uterus. Doctor: Hi.Thanks for your query and an elucidate history.The typical pains in abdomen as pressure pains, could have been due to a problem with colon or so. This is so as the pain has persisted even after removal of the cyst.I would advise you the following. CT scan of the abdomen and colonoscopy.to get to the root of the problem. Enteroclysis if required.The final diagnosis for the pressure pains will lead to a proper treatment ."
},
{
"id": 116410,
"tgt": "Suggest remedies for prolonged pain in my knuckle after an injury",
"src": "Patient: Hey so I was hanging out with my friend and he picked me up and my pinky caught the door and he went up and bent it really far (toward the back of the hand)....usually when it bends to far the pain lasts for about an hour but this happens about 5 hours ago and it still hurts.... it hurts when I flex my pinky, when I try to crack my pinky nuckle and when I bend it toward the palm after a certain amount I can still clinch and stuff...oh and its the nuckle closest to the palm the biggest nuckle.... thanks please just let me know if I need to go to a doctor or is it just severely sore Thanks, Mike Doctor: Hi,Thanks for asking.Based on your query, my opinion is as follows.1. An X-ray is necessary to rule out any fracture.2. It does not appear to be a fracture, but it needs to be stabilized to avoid any further tear if it has already happened.3. Take NSAID pain killers and also you can apply local painkiller spray. If the pain does not disappear in a day, meet your doctor to get an X-ray done.Hope it helps.Any further queries, happy to help again."
},
{
"id": 121106,
"tgt": "What causes numbness and tingling in hands/feet during night?",
"src": "Patient: I get numbness tingling in hands and feet at night.i have lot of digestion problems along with bloating and gas This has started since two months.i am 39 female.no diabetes sugar 90 fasting.B12 is 190.no one is able to understand my problem.please help me as I cant sleep at night. Doctor: Hello,I read carefully your query and understand your concern. Your symptoms seem to be related peripheral neuropathy. I suggest using anti inflammatory medications such as Acetaminophen to relieve the symptoms. If the symptoms continue, I suggest to see a neurologist and get evaluated. You might need prescription painkillers. Hope my answer was helpful.If you have further queries feel free to contact me again.Kind regards! Dr.Dorina Gurabardhi General &Family Physician"
},
{
"id": 176127,
"tgt": "What causes bad body odor in children with rheumatic fever?",
"src": "Patient: hi my son is 4 years old and he has JIA and Rheumatic fever and about 2 months ago he started having a really bad odor to him we tried deodorant thinking it was him sweating and nothing helped we spoke to 2 different doctors and they think its a metallic smell but don t know what could be causing it Doctor: Hi,Welcome to the HCMBrief answer... it could by hyperthyroidism or metabolic disorder..Detailed answer...Since sweating can occur normaly when there is increased temprature outside and body responses by increasing sweating to balance the body temprature..But if there is particular odor ,it can be disbalance of metabolism which can cause metallic odor.As thyroid hormones have crucial roles in controlling metabolism ,and if there is hyperthyroidism it can cause sweating and anxiety. Other cause can be amino acid metaboilc disorder.I would suggest that get thyroid function test ,blood electrolytes , urine test to detect amino acid metabolites , and follow accordinhly.Regards,Dr.Maheshwari"
},
{
"id": 42766,
"tgt": "What causes infertility?",
"src": "Patient: I am28years married 13 months back. i want child for which i made semin analysis which shows semin motality 40%. then i take COQ 10 300mg for 45days but my wife is not still conceiving. the reports of my wife are normal. Muneer Ahamd Khan Anantnag Jammu and Kashmir Doctor: Hi,Thanks for writing to HCM. Sadly medicine has not come up with such treatment which has dramatic effect on sperm motility. Only multivitamin and zinc supplements are prescribed which improve sperm quality.I would suggest you some life style modifications-stop smoking and alcohol and caffeine .- regular excercise, ware loose under garment.- healthy food intake and avoid stress.-maintain ideal weight.I would suggest you to discuss with your doctor regarding use of ovulation inducing drugs like clomephine for your wife . This will help multiple follicles to mature and rupture .Thus increasing chances of pregnancy .Hope I have been helpful . RegardsDr.Deepika Patil"
},
{
"id": 172682,
"tgt": "Suggest treatment for brown color vomiting",
"src": "Patient: My 4 year old girl started vomiting 24 hours ago. Since she did not eat anything. I was giving her boiled water only, and she was vomiting irreguraly. In the last 8 hours she strated vimiting frequently with vomit being breen but turned brown in the last few hours.Should I panic or wait for few more hours? no diarrhea at all in the period, mild stomack pain in the begining but none since. Responsive, going in and out of sleep, wakes to vomit... Doctor: Hi dear, It seems she has acute gastritisWith such presentation in my clinic I suggest:1.Ondansetron 2 ml orally to stop vomiting.2.Cefixime 4mg/kg/dose twice a day for 5 days3.Vizylac 1 capsule daily for 1 month4.ORSWishing good health and speedy recovery"
},
{
"id": 133921,
"tgt": "What causes severe pain and soreness in neck and shoulder after EGD?",
"src": "Patient: Had and upper EGD this morning. Everything went well. Small gastric polyp removed. However, my neck and upper shoulders are sore and even once in a while I have a shooting pain. I think this is probably due to positioning during the procedure. Any thoughts? Doctor: hi,thank-you for providing the brief description of your history.A thorough neuromuscular assessment of you is advised.As the pain is in the shoulder and neck you can aspect this due to the position. As it is postural pain you should be fine in a day or two.Also, if the pain goes longer, taking the help of a physical therapist you will have relief of pain and also strength in the muscles of the neck and shoulder.RegardsJay Indravadan Patel"
},
{
"id": 103482,
"tgt": "Suffering from infant asthma caused due to dust allergy. Using medications, nebuliser and nasal spray. So many medicines required?",
"src": "Patient: Good afternoon, I am a single mom with a little boy of 2 and a half. He was diagonsed with Infant asthma when he was 10 months old. However I believe it is caused mainly by Allergy - especially dust and dust mite as he does not generally wheeze or cough. But when there is a change of season he gets a cough that sounds very similar to croup . he is currently on Nebuliser - Duolin at night before bed, he has one puff seraflo, 1 tablet of singlair, Avamis nasal spray , 5ml Antihistamine and 1ml Valegan. then in the morning nebulise with Ventolin as I find his heart races after the ventolin. He has 4ml Aspelone, 4ml cough mixture, Alergex antihistamine and multivitamines. the Poor child seems to live on medications. Please help me to make sense of this and if it is necessary? Doctor: THE MILK ALLERGY SOMETIMES PERSISTS BEYOND 2 YEARS OF AGE AND SPOIL YOUR WHOLE LIFEAS MILK THEORY SUGGESTS CHILD NEEDS MILK ONLY BREAST FOR 2 YEARS AND AFTER THAT AGE NO NEED OF MILKWHEN WE ADD MILK AND DIARY PROTEINS THESE ARE ANIMAL PROTEINS AND ANIMAL PROTEINS ARE NOT COMPETIBLE WITH HUMAN PROTEINS CAUSING ALLERGY ASTHMA AND OTHER DISEASES WHICH PERSIST FOR LIFEIF YOU CAN GET BLOOD SERUM TEST FOR SPECIFIC ANTIBODIES FOR MILK WHEAT POTATO CHANA RICE NUTS EGG AND ELIMINATE THEM ACCORDING TO RESULTS YOU WILL BE FINE IN FEW WEEKSMEAN WHILE START WITH COMPLETE ELIMINATION OF DIARY AND MILK PROTEINS"
},
{
"id": 92708,
"tgt": "Started experiencing abdominal pain, loose stools, painful, was ovulating. What is wrong?",
"src": "Patient: hey i ate breakfast this morning after i started experiencing abdominal pain mostly on the left side i thought i had to go number two i tried and got a little loose stool out but nothing major i have been hurting all day i took a shower and also noticed i was ovulating as well please tell me what u think it is and what i should do. Doctor: Hi,It appears you might be having abdominal discomfort due to ovulation.Nothing to worry.Take one antispasmodic tablet like Dicyclomine and you will be alright.Take light diet to day.Ok and take care."
},
{
"id": 78797,
"tgt": "What causes pain in swollen cervical lymph node?",
"src": "Patient: Sir,I have a Cervical Lymph Node on the left side. Montoux test showed 28mm and ESR was 27 per hour. Physician has started my TB Medicines and I gained 3kgs in last 15 days (earlier I had lost 12 kgs in around a year). Now for last 4 days this swelling sometimes pain and gives shrinking feeling. Is everything alright ?Thanks for your kind reply :) Doctor: Thanks for your question on Health Care Magic. I can understand your concern. No need to worry much for this symptoms. You are improving with gaining weight, absence of fever and shrinking in size of lymphnodes. So all these are favourable and suggest that treatment is working. Pain is mostly due to healing and fibrosis in the lymphnode. Anti tb drugs cause healing in the lymphnode. Healing takes place in the form of fibrosis. So this process is painful to some patients. Shrinking in size is suggestive of healing also. So no need to worry at present, continue anti tb drugs for atleast 6 months. Don't worry, you will be alright. Hope I have solved your query. Wish you good health. Thanks."
},
{
"id": 173915,
"tgt": "What do cluster of bumps on face and above lips indicate?",
"src": "Patient: My 6 year old son has a small cluster of opaque to skin colored bumps on his face, just above his lip. They have been there for a few months. My dermatologist thought that it was just a pimple, but the bumps have been increasing in number. When scratched or picked they do not have any liquid in them. I don't believe that it is a cold sore. I have these from time to time and know how they dry up, scab, etc. Doctor: Hi,Thank you for asking question on health care magic.It may be a pimple as your dermotologist has told or it may Herpes simplex infection.Any way it is not possible to diagnose without actually seeing the lesion.Follow the advice of the dermotologist.Hope this answer will serve your purposePlease feel free to ask any more queries if requiredTake careDr.M.V.Subrahmanyam MD;DCHAssociate professor of pediatrics"
},
{
"id": 177629,
"tgt": "Suggest treatment for vomiting and stomach pain",
"src": "Patient: My 2 year old son has been up all night crying about stomach pains. He started with them around midnight. At about 5 am he began vomiting a milky white watery fluid on and off. I have given him some child s paracetamol, but it s doesn t seems to have helped him. He also has a temperature and is lethargic + moaning in pain due to his stomach ache. Doctor: Hi...this could be stomach flu which is starting and he might start to have diarrhea too. If there is vomiting you can use Syrup Ondansetron as prescribed by your pediatrician (It is a prescription drug). Unless the kid's having low urine output or very dull or excessively sleepy or blood in motion or green bilious vomiting...you need not worry. If there is diarrhea - I suggest you use zinc supplements (Z&D drops 1ml once daily for 14 days) & ORS (Each small packet mixed in 200ml of potable water and keep giving sip by sip) as hydration is very important and crucial part of treatment. Regards - Dr. Sumanth"
},
{
"id": 92547,
"tgt": "Have pain in right side of abdomen after playing pickle ball, large blue mark. Give me suggestion",
"src": "Patient: I have been playing Pickle Ball, twice a week, 2 hours. After I went after a ball, I experience a pain in my right side of the abdomen. This morning I woke up and noticed a large black and blue mark in the area of my pain. I'm concerned that I may have pulled a muscle. What should I do about this? YYYY@YYYY Doctor: Hello, Thanks for the query to H.C.M. Forum.This is not a case of pulled muscle but is a case of hematoma with contusion .This contusion and bruise developed bluish discoloration on the injured part.Ice massage with serratiopeptidase + anti inflammatory tablet only for 2 days will provide complete relief .In my opinion there is not a cause of any concern.Rest will also help in rapid recovery.Good luck. Dr. HET"
},
{
"id": 179490,
"tgt": "What does pus cells-2-3 and protein-negative in urine test suggest?",
"src": "Patient: ROUTINE URINE ANALYSI Straw Yelow /- Urine Automated -Reflctance photmetry Clear COLUR(URINE) PH (URINE) 7.0 - SPECIFC GRAVITY(URINE) 1.02 - GLUCOSE (URINE) NEGATIVE - PROTEIN (URINE) NEGATIVE - BILRUBIN (URINE) NEGATIVE - KETONE (URINE) NEGATIVE - UROBILNOGEN (URINE) 0.2 0.2-1.0 E.U /dl - Manual-microscopy SEDIMENT (URINE) PUS CELS 2-3 / HPF RBC NIL / HPF EPITHELIAL CELS 1-2 / HPF CAST NIL / HPF CRYSTALS NIL / HPF OTHERS (URINE) - - * End OfReport * Director-Central Laboratory Services Dr.Uma sekar M.D,CP Dr.Feb Renjitha Suman M.D ADITONAL PROFESOR Al investigations have their limtaions whic are imposed by the limts of senitvy and specifty of indvidual asy procedures as wel as the quality of the specimen recived by laboratory.Isolated laboratory investi this is my son 5 yrs old his report kindly inform that he s having any infection or not Doctor: Hello dear,welcome to HCM.Apart from 2-3 pus cells,the report is normal.But,whether this is significant or not,depends on the clinical condition of the child.If symptoms like fever, frequency,urgency,burning on urination etc are present,this needs to be treated with antibiotics.Your doctor will guide you.You may be asked for a urine culture test.Hope this answer might help.Thank you."
},
{
"id": 44389,
"tgt": "Ovary removed. Infertility confirmed. Irregular periods. What to do?",
"src": "Patient: Hi I had an overie removed last jan.2011..Things went wrong and they said i would not beable to have kids..But this past month my last 2 periods have been more odd them they have been..I just got mine thusday night and it just stoped last night..No spotting or cramping with both or anything just stoped..But iv been feeling kinda sick on and off all month and eating kinda odd things..Iv been wanting to cry on and off ...My bf has been saying im pregnant but i just got over my period? what should i do? Doctor: hi terra thanks for writing to us u need not worry for that sometimes it happened like this i dont think u r pregnant dont get so much tensed consult a gynecologist thank u"
},
{
"id": 195765,
"tgt": "Is intercourse generally painful to the penis or foreskin?",
"src": "Patient: Hi, I might seen pretty dumb asking this, but. I am a virgin, uncircumcised and I am pretty afraid if it might hurt me when I have sexual intercourse with a female. Like it might cut my uncircumcised area or the part below like, when the foreskin goes down and my penis isn't used to it, that area perhaps. I don't know, I am really confused on this kind of matter and scared a bit. Doctor: Hello and Welcome to \u2018Ask A Doctor\u2019 service. I have reviewed your query and here is my advice. It will not cut your foreskin during intercourse. Can you retract the skin backwards completely. If it hurt you then might cause some pain during intercourse. Hope I have answered your query. Let me know if I can assist you further."
},
{
"id": 46406,
"tgt": "Suggest remedy for water retention in body while on chemo treatment",
"src": "Patient: My father has been taking chemotherapy for cancer. He has cancer in his thyroid. Two weeks ago he was hospitalized for congestive heart failure. He is still in the hospital. Problem is he is retaining mass amounts of water. He has only one kidney. They have been restricting his water intake and administering lasiks to remove the water. But it isn't working. Any ideas as to what may be wrong? Doctor: Hello and welcome to HCM. As an Urologist, i can understand your anxiety.Lasix is given to flush out excess fluids from the body.But if the kidney function is less,it can't be given,due to side-effects.As you've not written the kidney function reports,it's difficult to comment.If he's retaining fluids,he should get the following tests done :1. urine routine,culture and ACR.2. blood routine,creatinine,RBS, LFT,proteins and platelets.3. ultrasound scan of abdomen.He should restrict fluids,salt and protein intake.Dr.Matthew J. Mangat."
},
{
"id": 23000,
"tgt": "Can a diabetic on blood pressure medication take nitroglycerin?",
"src": "Patient: 59 year old male. Diabetic on blood pressure medication, high A1C, also taking glipzide and another medicatin. always complaining of being tired. last year had a episode where he was given nitroglycerin tablets and admitted to hospital. no damage to heart was found. Doctor: DEAR USER,THANKS FOR CONSULTING WITH HCMI UNEDERSTAND YOUR CONCERN..AS SUCH TAKING NITROGLYCERIN IN PATIENT WITH DIABETIC CAUSES NO HARM..ITS USUALLY GIVEN WHEN PATIENT IS SUSPECTED TO HAVE EITHER ANGINA OR HEART ATTACK OR SOMETIMES FOR HIGH BP EVEN ON CLINICAL SUSPICIONREGARDING YOUR PATIENT BEING TIRED IN DIABETES THERE ARE MANY CAUSES OF TIREDNESS.. A DETAILED EVALUATION OF THE PATIENT BY A DIABETOLOGIST CAN HELP TO FIND THE CAUSEIF HIS SUGAR ARE NOT GETTING CONTROLLED AS YOU SAID HE HAS HIGH HBA1C. HE MIGHT BE STARTED ON INSULIN THERAPY AFTER CONSULTING A DIABETOLOGISTHOPE I ANSWERED YOUR QUERY. YOU CAN MESSAGE ME FOR ANY FURTHER CONCERNS"
},
{
"id": 143640,
"tgt": "Could left posterior parafalcine meningioma in MRI mean the presence of tumor ?",
"src": "Patient: Left posterior parafalcine meningioma measuring 1.3 cm x 1.4 cm x1.3 cm. MRI report shows; focal infiltration of the sagittal sinus demostrated with stenosis versus occlusion of sagittal sinus .. Should I be concerned about stenosis? , or is this a normal behaivor or these tumors? Also remainder of brain demostrates mild to moderate degree of white matter chronic small vessel ischemic changes. Could this be caused by the presence of this tumor? Doctor: Hi, Welcome to HealthCareMagic.com I am Dr.J.Mariano Anto Bruno Mascarenhas. I have gone through your query with diligence and would like you to know that I am here to help you.Stenosis means blood flow is interfered. So you have to worry. The other chronic small vessel changes can be due to this. Please consult a Neurosurgeon at the earliestHope you found the answer helpful.If you need any clarification / have doubts / have additional questions / have follow up questions, then please do not hesitate in asking again. I will be happy to answer your questions.Best Wishes for Speedy Recovery Let me know if I can assist you further.Take care.-oOo-PS 1 : After all your doubts have been cleared, kindly (a) HealthCareMagic at http://bit.ly/askdrbruno this Question (b) Rate my Replies and (c) Give your Feedback. PS 2 : In the future, for continuity of care, I encourage you to contact me directly in HealthCareMagic at http://bit.ly/askdrbruno"
},
{
"id": 216618,
"tgt": "Suggest remedy for pain in leg",
"src": "Patient: Hi, I m a 22 year old female and have increasingly found it painful to get up after squatting or sitting down. It is mostly only painful in my right leg - unsure of whether it s the joints or muscles. Am unsure if it is just a sign of general unhealthiness, but if so, shouldn t the pain be in both legs? I also have quite poor blood circulation - often get chilblains in winter but not sure if this is related in any way. Doctor: hi,thank you for providing the brief history of you.As you have this symptoms I feel this is more of the muscle weakness leading to inflammation in the knee joint. For which we need a thorough clinical examination.Also we may need an x-ray or an MRI depending upon the examination that it is more of a bony issue or soft tissue issue.Most of the pain are due to weakness of the muscles and with simple medication and physical therapy they respond well.In my clinical practice most patients with knee injuries are referred to physical therapy and they respond well with a combination of drug therapy.Regards Jay Indravadan Patel"
},
{
"id": 145409,
"tgt": "What does my MRI results mean?",
"src": "Patient: Mom just did an MRI. She has at least seven supratentorial and on infratentorial compartment parenchymal metastasis. Supratentorial compartment white confluent FLAIR hyperintensity be combination of posttreatment effect and /or chronic microangiopathy. Major intracranial flow voids appear patent. Evidence of global parenchymal volume loss. No orbital masses. Mastoids are underpneumatized an dcompletely opacified. What does all the mean. Doctor: I'm sorry of these findings. Your mother have to see an oncologist to find out the primary tumor and than a neurosurgeon to decide the treating strategy of the cerebral metastasis. I hope everything goes ok."
},
{
"id": 8730,
"tgt": "I am 22 year old female. Looking for sun protected cream which not cause damage to skin",
"src": "Patient: hi..i d like to ask about sun block creams.and what s the better one which not cause damage to skin?? female 22years...about sun block creams is it dangerous?i mean using it on long run..as i use it daily twice and i fear if there is any problem...thanks.. Doctor: Hi, Do not worry most of the sunscreen do not harm..but you have sensitive skin..I would suggest Spectraban Sensitive 30 plus SPF..it good..apply 7:30Am and reapply at 12 Noon.If problem persist can consult nearest Dermatologist"
},
{
"id": 225846,
"tgt": "Bad cramps without periods. On Beyaz birth control pills. Concerning?",
"src": "Patient: I am on Beyaz birth control tablets. My \"sugar\" pills began on Wednesday and during the middle of the day today, I started getting really bad cramps like my period had started. I put a tampon in, and when I went to take it out, there was barely any blood. This happened again later on.. Should I be concerned? I usually only get cramps if I have a heavy flow. I'm confused Doctor: Hi thanks for the query.You should take some antispasmodics like mefenemic acid tablets. This may be an indication of start of your period and may be relieved with start of your period.Hope this information is helpful and informative to you.If you have further queries you can directly reach out to me on the following URL:http://doctor.healthcaremagic.com/doctors/dr-deepti-goyal/65111"
},
{
"id": 20177,
"tgt": "Is hypertension related to shoulder and neck pain?",
"src": "Patient: My BP is right now at 160/110,I had checked long back and my pressure is always at 120/100 or 120/95,my lower line is always at above 95. 2yrs back my pressure was at 180/115.Im not in any medication but right now since 2 days im taking amlong 2.5mg.I have a backpain at my soldure as well as neck upper neck. Doctor: Welcome and thank you for asking on HCM! I have gone through your question. You are taking the right treatment (amlong). If your back pain persists get an ECG and Echocardiography test done at the earliest.Hope to have been helpful!Wishing good health, Dr D Sunil Reddy"
},
{
"id": 37668,
"tgt": "Suggest cure for a skin infection near the anus",
"src": "Patient: I just noticed some raised what feels like and looks like a keloid or rough skin lesion (skin colored) right next to my anus. I have been tested, no STDs but I do have HPV (no symptoms). Could this simply be scarring from rougher sex or something else? When should I be concerned? Doctor: Hello,Welcome to HCM,Wart which is painless and itching are the classical of Wart which is caused by Human Papillomavirus (HPV).Human Papillomavirus (HPV) infection can be transmitted to the partner as this is a sexually transmitted disease.This should be diagnosed by physical examination and confirmed by Polymerase Chain Reaction (PCR).The treatment for this condition includesCryotherapy- which should be done in the hospital.orLocal application of 0.5% solution of podophyllin, which should be applied twice daily for 3 days and nothing should be applied for next 4 days (7 days cycle). This cycle should be continued for 4 weeks.orLocal application of 5% Imiquimod cream for 3 times in a week for 16 weeks (4 months).Thank you."
},
{
"id": 2885,
"tgt": "Can the positive pregnancy test result be due to capsules?",
"src": "Patient: Hello. I been taking the hcg activator capsules for 11 days. (2 in the morning and 2 around lunch). My period has been late this month for 7 days. I just took a pregnancy test and it came out positive.... Can it be due to the capsules? And also at this point if I were pregnant could those capsules have harmed the baby? Thank you. Doctor: Hi,HCG activator capsule dose not cause positive pregnancy test. Your delayed period by 7 days and positive pregnancy test suggest pregnancy. These tablet does not affect the baby. I would suggest go for blood HCG test to confirm pregnancy as it is more sensitive and specific than urine pregnancy test. Avoid stress, take healthy diet, drink plenty of water and do regular exercise.Hope this may help you. Contact further if follow up needed.Best regards,Dr. Sagar"
},
{
"id": 214508,
"tgt": "Suggest treatment for cysts in the left ovary of 8x8 cm",
"src": "Patient: Hi, I'm diagnosed for an ovarian cyst in my left ovary in the ultrascan report. it was found to be 8cm*8cm with calciferous lining. my right ovary was normal. PCOS was not mentioned in the report & the CA-125 test came out to be 41 which was slightly more. the doctor has given 1 month tablets & asked to wait for 1 month before going to next step. i had regular periods & no sharp pain. can i get my cysts cured by taking natural therapy of beets, aloe & molasses? is laparoscopy the only way to treat my cysts? pls help me out Doctor: welcome to Health care magic.1.According to me, I would suggest to get operated.2.As the cyst size is quite big, the natural remedies will take quite along time to take action.3.And suggest to followup with CA -125.4.To mention PCOS there are criteria (Both ovaries should be bulky with multiple follicles -more than 8 and so...)5. In your case its cyst. So please act accordingly.Anything to ask ? do not hesitate. Thank you."
},
{
"id": 198360,
"tgt": "Suggest medications for improving sperm count",
"src": "Patient: i am shanmugam sir affected from azoospermia. my speam report speam nill i already some doctor get my report no improve i take ayeverthic medicin no improvement my question is improve count and which doctor consult the treatement sir please give me a solution Doctor: HelloThanks for query .Your semen analysis has revealed Azoospermia (Absence of sperms in semen ).The absence of sperms can be either of the due to two reasons1) Testicles are not producing sperms at all2) There is blockage in the passage of transmission of sperms from epididymis to Urethra .Consult qualified Urologist for clinical examination .He would ask for a test called Testicular biopsy to confirm whether your testicles are producing or not,If there is a spermatogensis he will do a radiological test Vaso Vasogram to rule out /confirm the blockage in tube (Vas Difference )If the azoospermia is detected to be due to blockage in Vas Difference it can be corrected by surgery .There is no medicine that can cure azoospermia due to absence of spermatigenetic activities by Testiticles..Dr.Patil."
},
{
"id": 163720,
"tgt": "Suggest remedy for vomiting with diarrhea with runny nose and cough",
"src": "Patient: My son is almost 7. Very high functioning autism which is only important in this situation because of his non-compliance with continuing with tylenol and his refusal to have blood draw and the labs quick reply that I should bring him back when he is ready to comply. He s been sick now for about 3 weeks. Started with 48 hours vomitting...then about a week later diareha, pooed himself while asleep a couple of times...then just seemed to be brewing a cold that never broke until last week he was pretty much lethagic, curled up in a fetal position with head on my chest...sleeping, much decreased apetitie and fluid intake. Had been able to get him to take tylenol intermittentl because he seemed to be feverish. He s had a chest x and its clear. green fluid stool is at the lab pending results for O and P (I think). But has refused blood draw, despite very heavy incentives. Still runny nose and cough. What do you think could be going on? Doctor: Hello! Welcome to HealthcareMagic! One should be concerned in such situations. The condition seems to be due to transient low immunity. But in this case, things have extended pretty long. It will be better if you stay in touch with pediatrician rather than keep giving him Tylanol. The doctor may prescribe other symptomatic treatment for the relief. Once reports are received medicine can be prescribed accordingly. Take care Regards, Dr. Ratna Mulay, Pediatrician"
},
{
"id": 122786,
"tgt": "How can I reduce pain on the right side of my left knee?",
"src": "Patient: I have pain on the right side of my left knee. When I first get up it hurts to walk but will get better as I walk, although the pain is still there but not to bad. Sometimes the top of the knee will hurt until I bend it or straighten it just the right way. I am 58 and like to run but this is stopping me. It came on after I worked on my deck and had to do a lot of up and down bending and kneeling. Doctor: Hello, As per your history, it may be due to knee sprain. For that you can take tablet acetaminophen for pain. Apply diclofenac gel or ointment locally. Heating pads or icing will also help. Along with that avoid up and down stairs. In case of not getting relief, kindly do MRI knee after orthopedician consultation. Hope I have answered your query. Let me know if I can assist you further. Regards, Dr. Shyam B. Kale, General & Family Physician"
},
{
"id": 196995,
"tgt": "What causes pus cells in semen?",
"src": "Patient: my semen analysis showed pus cells and whenever I ejaculate there is inflamation over my penis. Sir, what could be the reason. My age is forty five years, weigh is eighty kgs. and I was treated by antibiotics a couple of times and also took homeopathy treatments but still it the problems remains. Doctor: HelloThanks for query .Presence of pus cells in semen is strongly suggestive of infection either in Seminal Vesicle or in majority of cases in Prostate .Taking antibiotics either Doxicycline or combination of Sulphamethoxazole and Trimethoprim twice daily helps to control the infection .However since you had taken antibiotics for many years I would suggest you to do Semen culture to detect organisms causing this infection so that you can take appropriate antibiotic as per culture report to help in eradicating infection .Dr.Patil."
},
{
"id": 174509,
"tgt": "Can shots for pneumonia causes diarrhea and black colour in the prick area for 18 months old?",
"src": "Patient: My child is 18 month he had pneumonia two weeks ago Nadal had three shot two days later he had diarrhea first It has been 9 days know the color is black his but Bottom does not look good what should I do he was seen by the Dr they did take stool culture should I call the Dr now Doctor: Hi,Thank you for asking question on health care magic.the Pneumonia shots have no relation to diarrhea.The black color in the prick area may be due to subcutaneous hemorrhage.Local application of rashfree ointment may help.if diarrhea does not subside within 2-3 days you may go for stool culture.Hope this answer will serve your purposePlease feel free to ask any more queries if requiredTake careDr.M.V.Subrahmanyam MD;DCHAssociate professor of pediatrics"
},
{
"id": 141815,
"tgt": "Suggest treatment for twitching in toe,weakness in arm and back pain",
"src": "Patient: I have low thyroid along with back, hip and pain all over. I also twitching in my middle toe. I am on levothyroxine .025 mg daily. I am nervous because I am having pain and also so weakness in my left arm. Back pain is all over with buttocks and hip pain. I know this is a lot, but can someone help? Doctor: Hello,I understand your concern and would recommend going to the ER for a physical exam and some tests:- A resting ECG- Blood electrolytes- Complete blood electrolytes- Thyroid hormone levelsA brain CT scan may be needed if the left arm weakness persists.Hope I have answered your query. Let me know if I can assist you further.Regards,Dr. Aida Quka"
},
{
"id": 146463,
"tgt": "What causes dizziness and pain on left shoulder till back of ear?",
"src": "Patient: Last month I was feeling giddy if i turned my head down wards and left side.Dr has given me Stutgen 75 and vertin 16Now afer a month I have atendency to fall on my left side while walking and after getting down from the moving car when stopped, , I have a tendency to fall back.What may be the cause of it . I feel the heaviness on left sde of my face and feel a pain also radiating in the left shoulder till back of the ear and sometimes reaching on the let eyebrow.Will hysiotherepy help me. Doctor: Hi, I had gone through your question and understand your concerns. The symptoms you describe probably have a neurological underlying cause and you need correct diagnosis before thinking of treatment or physiotherapy, so I suggest you to consult a local Neurologist and to discus with him the possibility of imaging study of your brain and cervical spine. Hope this answers your question. If you have additional questions or follow up questions then please do not hesitate in writing to us. I will be happy to answer your questions."
},
{
"id": 96480,
"tgt": "Stomach infection in my 5 1/2 month old",
"src": "Patient: my 5 and a half month was suffering from cough and congession when my doc put him on tixilix and vent pd syrup. then the very next day he got loose stomach wherein whenever he drinks milk he paases a loose stool.its been one week.he was then advised vizylac,,gramogyl and was side by side put on nebulizer at home only.latest we were giving him tixilix,cepodum,deryphylin,entrogermanium,nasal drops,vizylac and gramogyl.doc gave him isomil as welll.but still he is not improved.and ya amikasin injection are also given for 5 days.what to do?? Doctor: Hi... Passing loose stool following milk can happen\u00a0 when weaning is started.. This could be a normal\u00a0\u00a0 phenomena or also sign of a GI infectioin which can be judged\u00a0\u00a0\u00a0\u00a0 by the\u00a0 treating doctor as he has examined the child.. You can\u00a0 change to a\u00a0 different quality of milk. Follow with the medical advise of your pediatrician... If the diarrhoea persists inspite of all measures then restriction of milk has to\u00a0 be considered."
},
{
"id": 116465,
"tgt": "What does the following lab result indicate?",
"src": "Patient: So my lab came back and this is what it said.Ketonese have trace. Occult blood 3+. Protein trace. Rbc 10-20. Squamous epithelial cells 10-20. And bacteria few. My calcium was also 10.3. No one is giving me answers just making me do more testing. Im scared. I just want the prognosis. Do I have cancer ? Doctor: Hi,Thanks for asking.Based on your query, my opinion is as follows.1. Presence of blood along with squamous epithelial cells and few bacteria indicates a possible urinary tract infection. However, if you are above 50 years, a diagnosis of cancer needs to be ruled out.2. Get an ultrasound to identify any pathology in the kidney. Urine culture will help in identifying any infection. Based on sensitivity, appropriate treatment can be taken.3. Until a renal biopsy is done to identify the cause, cancer or other cause cannot be confirmed. Get an ultrasound and if a pathology is present, a renal biopsy may be done for confirmation.Hope it helps.Any further queries, happy to help again."
},
{
"id": 226190,
"tgt": "Have heavy bleeding after slimline copper IUD fitted. Normal or remove?",
"src": "Patient: Hi, I had a TT380 Slimline copper iud fitted three weeks ago and have been bleeding quite heavily ever since. Is this normal? Will it ever stop? I am 37, height 5ft 5in, weight 10st 3pounds (not normal weight had first baby almost 5mths ago I am in the process of losing weight). I am in no pain just bleeding lots and not passing any clots. Any advice? Is it not suitable for me? Should I have it removed? Also due to have period next week. Doctor: Dear helbridle, The intrauterine device is by far the most suitable method for your age. Excessive bleeding is a side effect, but there is medical treatment. First of all you should exclude organic causes, the most important of which is displacement. This could be ruled out by a pelvic ultrasound, preferably transvaginal. If the IUD is in place, you may start medical treatment and there are many options, some of which are the following: - mefenamic acid 500 mg tid for 5 days and resume with the beginning of menses. - ibuprofen 600 mg once daily throughout menses. - tranexamic acid 1 gm qid for 5 days and resume with menses. Another option is to change this type by the progesterone medicated type. I hope these words were usefull and wish you the best of luck. Dr. Ahmed Bahaa."
},
{
"id": 41982,
"tgt": "Suggest treatment for infertility",
"src": "Patient: My husband and i have been trying to have a baby for the past 2 yrs, but unsuccessful. I have been to the doctors but nothing happen, i also did a HSG test but still nothing. My husband refuses to see a pathologist to test is sperm. Doctor tell me what else can i do? Doctor: Hi, Welcome to HealthcareMagic. Before you start any treatment it is important to know your husband sperm count is normal or not. If that is not normal then he needs treatment. Whatever you take treatment it will do in vain. I suggest you to convince him and also you may go for ovulation inducing drugs like clomephine. This will help multiple follicles to mature and rupture. Thus increasing chances of pregnancy. Hope I have answered your query. RegardsDr.Deepika Patil"
},
{
"id": 34371,
"tgt": "Suggest treatment for severe chickenpox",
"src": "Patient: I m turning 19 this year and having severe chickenpox. I have blisters all over my body, including in the mouth and throat, also on my eyelids. I don t know what foods are essential in this condition. I also wonder if I can have rice. If I can t, can you give me examples of foods I can consume that won t aggravate the itchiness or cause irritation and inflammation? Thank you. Doctor: Hello,Welcome to HCM,Chickenpox is a viral disease which is transmitted by droplet infection and droplet nuclei. You can acquire this disease if you are exposed to a person having active infection.As you mentioned in your history that you are having a wide spreading white pencil-eraser sized rashes in your genital area including the penile head.The chickenpox can affect all parts of the body from head to toe including genitals.It is highly communicable 1-2 days before the appearance of rash and 4-5 days thereafter. After which virus tends to die out and looses the capacity to transmit disease to normal individuals.Chicken pox is a self limiting disease which will subside by itself. Antiviral drugs like valacyclovir can be taken, it will not cure the condition but it will halt the progression of disease and prevent further complication associated with the disease.Topical application of calamine lotion will dry up the rashes. NSAIDs like Ibuprofen can be taken to control the pain associated with the disease.Thank you."
},
{
"id": 143516,
"tgt": "What causes popping sensation in head?",
"src": "Patient: Years ago I experienced a popping in my head. Before it happened I had a dull headache. I bent over and I heard and felt a popping sensation in the back of my head. I was unable to stand upright without a rushing feeling in my head. I was hospitalized and for several nights I would have a strange rushing sensation when I would sit upright. I know this sounds strange, but it has always concerned me but I don t really speak about it because of how crazy it sounds. I was never diagnosed. I basically diagnosed myself with TIA. Doctor: HiPopping inside the Brian is not a usual phenomenon. it does happen to few people. it doesn't happen inside the brain it is related to the muscles attached to your head, neck, cervical spine or just the jaw. don't have to worry about it , doesn't cause any harm to you. unless you have a stiff muscle in the above said regions. it can also be a release of the wax in the ear which causes the same effect. hope I helped you to get some idea. best is to ignore it."
},
{
"id": 78919,
"tgt": "Is left arm pain normal in costochondritis patient?",
"src": "Patient: I have been diagnosed with costochronditis-pain in breast bone, that has gone into ribs, left arm, back shoulder blades over about three weeks. Have felt like im having a heart attack with it. Feel very anxious about it, blood pressure and pulse all have been checked, again today by chemist and are within normal range. Left arm is sore from Shoulder down to hand, more so when I raise it. Feel quite anxious about it and feel my heart racing although I don t think it is according to chemist and doctor last week. Do you think I should get it rechecked or is this normal symptoms for this condition? Doctor: Thanks for your question on Health Care Magic. I can understand your concern. In my opinion, you should get done ecg and 2d echo to rule out heart diseases because these symptoms are not normal after costochondritis. So better to get done ecg and 2d echo. If both are normal than no need to worry for cardiac diseases. Sometimes musculoskeletal pain can also cause similar symptoms. So avoid movements causing pain. Avoid bad postures in sleep. Avoid heavyweight lifting and strenuous exercise. Start painkiller and muscle relaxant drugs. Apply warm water pad on affected areas. Don't worry, you will be alright, but first rule out heart diseases. Hope I have solved your query. Wish you good health. Thanks."
},
{
"id": 177477,
"tgt": "What is the treatment procedure for JRA?",
"src": "Patient: My grandson just got diagnosed with JRA and we got all kinds of answer regarding treatment, what do you suggest, is there a hot line that I can call to get more information, what kind of research are being done to treat this, I have lots of questions. Thank you for your time. Gabriella Doctor: Juvenile rheumatoid arthritis (JRA) or Juvenile idiopathic arthritis (JIA) also known as juvenile chronic arthritis (JCA), is an autoimmune condition that affects children and adolescents and could lead to serious complications like eye inflammation which could even lead to blindness. Treatment is usually done with non-steroidal anti-inflammatory drugs and intra-articular corticosteroid injections. Sometimes methotrexate or TNF alpha blockers like etanercept, are used as disease modifying anti-rheumatic drug (DMARD) to suppress joint inflammation. There is no hotline available to help you procure information on the condition but a simple google search with the terms that I have mentioned, would give you enough information that would be both relevant and reasonable to satisfy the answers to your questions. You can also remain under the follow up of a rheumatologist who will be able to give you appropriate information and also would be able to treat the condition."
},
{
"id": 122844,
"tgt": "What causes swelling and heaviness in knee/ankle?",
"src": "Patient: I have a swollen knee, ankle and knee feel heavy, almost like they are a sleep. I did not injur it that I am aware of. I do walk and run. I weigh 138 lbs at 5ft 4 in. I have taken anti inflamatorys for a week and have iced it. any idea what could be wrong? Doctor: Hello, It could be arthritis or joint effusion. As of now, you can use analgesics/anti-inflammatory combination like aceclofenac/serratiopeptidase for symptomatic relief. If symptoms persist better to consult an orthopedic and plan for an MRI scan. Hope I have answered your query. Let me know if I can assist you further. Regards, Dr. Shinas Hussain, General & Family Physician"
},
{
"id": 111239,
"tgt": "What causes back pain, insomnia, and no appetite?",
"src": "Patient: Hi. Hope you could help me. Im 23 and i missed 2days of my period. My LPM was april 19.. Had contact with my boyfriend may 4.. And now im feeling dizzy,having lower back pain, insomnia, no appettite, and had negative preganancy test awhile ago. Hope to hear from you thanks Doctor: Hello,I had gone through the case and found that you must again go for pregnancy test.If negative then consult to gynecologist because it might be hormonal imbalance.Hope my answer will be effective for you.Thanks"
},
{
"id": 18013,
"tgt": "How can shortness of breath, swelling in the legs and headaches while suffering from atrial fibrillation be treated?",
"src": "Patient: 60 yr old female, I have 2 ruptured discs in my neck, had an ekg 2 was ago & it showed I have A-fib. I\u2019m feeling short of breathe, legs are swelling even with water pills Furosemide 40 mg & Spironolact 25 mg, blood pressure medication Losartan/Hct 25-6.25 & a new pill for the A-fib Verapamil 80 mg one twice a day. I also have a headache which is normal for me with the ruptured discs but this headache feels different. I can\u2019t get to my blood pressure cuff but I can feel my heart rate is rapid which is also normal for me & usually controlled with Metoprolol 50 mg. I have an appt with a cardiologist the middle of June & if he clears me surgery on my neck 7/16. Any suggestions? Doctor: Hi, If the cervical spinal cord lesion does not have any symptoms then it may not need any surgery, other cardiac elements can be managed with the help of medicine, that you are already on. For the surgery clinical symptoms are the criteria else it all depends upon the patient surgeon's consultation, lower limb swelling could be due to cardiopulmonary disease, dose of the diuretic needs to be increased. Hope I have answered your query. Let me know if I can assist you further. Regards,\u00a0\u00a0\u00a0\u00a0\u00a0 Dr. Akhtar Husain"
},
{
"id": 7124,
"tgt": "When should a pregnancy test be done ?",
"src": "Patient: If I wasn't supposed to start my period until 10/1/10 and I took a home pregnancy test on 9/30 that came back positive when is the latest I could have conceived? Doctor: Thanks for te query You can expect the child by 10 th ooctober Congratulations Have a healthy living"
},
{
"id": 177347,
"tgt": "Suggest treatment for pain in head,neck and back",
"src": "Patient: Hello my daughter has been having a lot of different symptoms. Her head hurt so bad it feels like she s being Scalped. Her neck and back hurts. she gets random sharp stabbing pains all throughout her entire body. she is 20 years old and otherwise healthy.Any thoughts Doctor: she has probably chronic tension type headache. please provide duration of headache. is there photophobia or phonophobia , any history of stress, altered sleep patternshe should take tab amitryptiline 25 mg at bedtime daily"
},
{
"id": 66425,
"tgt": "What causes lumps in arm pit and in breast?",
"src": "Patient: I HAD BREAST CANCER 5 YEARS AGO IN MY EFT BREAST LUMPECTOMY LATLEY I BEEN TELLING HER I GET PAIN SOMETIMES SHE SAID IT WAS SCAR TISSUE AND LEFT IT AT THAT BUT IT CONTINUED A FEW WEEKS AGO I FELT AT EAST 5 LUMPS UNDER MY ARM PIT AND SOME IN MY RIGHT WHAT CAN IT BE THANK YOU VERY MUCH Doctor: Hi, dearI have gone through your question.I can understand your concern.You have history of breast cancer. You there is high chance that you have some metastatic breast cancer in your axillary lymphnode. You should go for examination. If needed go for biopsy of axillary lymphnodes. Then take treatment accordingly. Hope I have answered your question. If you have any doubts then feel free to ask me. Thanks for using health care magic.Wish you a very good health."
},
{
"id": 199923,
"tgt": "What causes sensitivity in testicles?",
"src": "Patient: Hi, I haven t worked out in a long time and just started to again in the past 5 days, so I am very sore. But I am having a sensitivity issue with my left testicle, it is just more sensitive than usual and seems to be closer to my body. It probably started 2-3 days ago. I don t know if they are related, but its the only thing I can think of. What should I d Doctor: HelloThanks for query.Discomfort in left Testicle following exercise is mostly due to Varicocele ..Please consult qualified Urologist for clinical examination and get following basic test done to confirm the diagnosis.1) Ultrasound Scanning of Scrotum.2) Colour Doppler study of scrotum.Further treatment will depend upon the findings on clinical examination and results of these tests.Dr.Patil."
},
{
"id": 157206,
"tgt": "What does the development of a lymph node on the back of left neck and in the arm pit of a three year old indicate?",
"src": "Patient: My three year old has one large lymph node on the back of his left neck which has been enlarged for two years. Last week it enlarged substantially. The pediatrician was quite concerned with its size. CBC normal now we go to a pediatric surgeon to see what they think. Since Friday the lymph Node is still enlarged but not quite as large as it was Friday. The pediatrician said all his neck lymph nodes were enlarged and one under his jaw bone. He has shotty palpable lymph nodes in the Arm pit and groin areas. Does this sound cancerous? Doctor: Thanks for query.Most lymph node enlargement in children is due to benign self-limited disease such as viral infections, and adenopathy is secondary to an increase in normal lymphocytes and macrophages in response to an antigen. Other less common mechanisms responsible for adenopathy include nodal accumulation of inflammatory cells in response to an infection in the node (lymphadenitis), neoplastic lymphocytes or macrophages (lymphoma), or metabolite-laden macrophages in storage diseases (Gaucher disease).In young children,front of neck lymph nodes as large as 2 cm, armpit nodes as large as 1 cm, and groin nodes as large as 1.5 cm in diameter are normal, and further evaluation is usually not indicated. malignancy is usually associated with nodes larger than 3 cm in diameter.However, the presence of even shotty (< 0.5 cm) supraclavicular or epitrochlear nodes may be associated with malignancy and warrants further evaluation.as your child blood counts are normal I hope its not a serious issue.biopsy of the node will make things more clear.regards"
},
{
"id": 117406,
"tgt": "Cause for fluctuation in blood sugar level?",
"src": "Patient: I was working around the house and suddenly felt really dizzy, shaky, light headed. My dad has an old blood sugar level reader and I checked it. it read 57. I ate these maple sugar pecans and it went up to 90. i still feel shaky but a little better. am i ok? Doctor: Hi,Thanks for asking.Based on your clinical history and query, my opinion is as follows:1. Lightheadedness and dizziness is due to hypoglycemia meaning low glucose levels.2. Glucose levels above 80 and below 120 are normal. So a level of 90 is not of concern.Hope it helps.Any further queries, happy to help again."
},
{
"id": 49863,
"tgt": "67yrs oldhad Prostatitus. pain in testicles and lower abdomen. Does acid reflux affect stomach pain?",
"src": "Patient: I am 67 years old. I have had Prostatitus for about 35 years. When it flares it is extremely painful in the right testicle and the pelvic area. Feels like severe arthritus.Recently the pain is higher up in the lower abdomen below the belly button. Had a complete physical . Everything was good including P.S.A. except kidney which showed creatinine of 103. Went to a specialist per my primary and he said it was not a big deal but ordered an ultra sound on kidney and bladder today.I have had bouts with acid reflux also but just in the chest and throat. Could acid reflux affect stomach pain also? it is a burning / arthritic pain across the lower stomach that hurts w hen I walk or in a car that goes over a bounce. Not to load you up but this is usually accompanied by a fluttering of gas that seems to originate on my top right leg where it joins my body. Thanks Doctor: see acute prostatitis is very painful.need per rectal examination good ultrasound of Kidney bladder prostate.if present will need 6 week of antibiotic .acidity can cause generalized pain.lower abd pain less common"
},
{
"id": 196483,
"tgt": "How long my body takes to produce testosterone after steroid injections?",
"src": "Patient: I m 20 years old height 6.2 feet weight 70 kg I masturbate after every 5 days... n I hav just completd the steorid cycle n I hav not done pct so my question is dat in how many week my body will produce testerones Will my muscles grow naturally after cycle ?? I almost exercise daily on high weights . Doctor: Hi, I had gone through your question and understand your concerns.First, it is definitely not advisable to have steroids in such young age and for this purpose. irt may have serious side effects and negative effects for your health. Second, it may not be efficient for muscle grow at all and its levels may need several weeks to eleavted. I suggest to use natural methods for muscle building and avoid such procedures and steroids since this is not necessaryHope this answers your question. If you have additional questions or follow up questions then please do not hesitate in writing to us. I will be happy to answer your questions. Wishing you good health."
},
{
"id": 158198,
"tgt": "Suffering from pain and swelling in the ankles. Diagnosed with gout and CLL. Elevated uric acid levels. How to get relief from the pain?",
"src": "Patient: My boyfriend has CLL and has been having a lot of difficulty with pain and swelling in the ankles. So much pain that we had to go to the emergency room because walking/bearing weight was impossible. The ER doc diagnosed it as gout & gave him a prescription without even doing labs. Follow up blood tests with his private Dr have shown normal uric acid levels and still no answers as to why the pain and swelling. He describes the pain as walking on shards of glass Doctor: Dear Sir,Sorry to hear about your friend's problems. His symptoms could either be due to the uric acid crystals deposited in his feet, which will take time to go away or as a side effect of the chemotherapy which he is receiving for CLL. He should meet his oncologist and seek the appropriate pain killer.Hoping for the best."
},
{
"id": 103731,
"tgt": "Excessive sleep in eyes and redness. Why are they shut with mucus in mornings?",
"src": "Patient: the past two nights I have had an excess amount of sleep in my eyes, more in the right than left eye. My eyes are also red in the inner corners and veiny. I don t think it is pink eye because they aren t completely red and stuck shut with mucus in the mornings. Also, they aren t tearing a lot, but they do burn a little from time to time (especially the right eye). Is this possible allergies? even though I am not experiencing any other signs of allergies? or is it maybe some kind of infection I need to see an eye doctor for? Thanks. Doctor: it may be local eye allergy or snus infectionsinus acute infections also have same symptompstake metrogyl 200 mg bd for 10 daysallegra 120 mg odneomycin h eye ointment in nose and eyes bdchromal forte eye drops in eyes tdsit may taje 2 wk to subsidetake med for 2 wk will be ok"
},
{
"id": 123242,
"tgt": "What causes pain in leg?",
"src": "Patient: I m a 27 year old female and was showing off last 4th of July playing basketball. I slipped when I jumped up after the ball and fell directly on my foot while it was basically sideways. I heard a loud pop not on my ankle but the side of the calf above the ankle. My whole ankle was swollen and a little black and blue. Also the back of the calf was swollen. It took a couple months to heal, and I used an air cast, but never went in due to lack of insurance and funds. I haven t had any problems until lately, if I just sit down and then get up, my whole leg hurts. Mainly in the chin bone and just below the knee. I have no idea if it s related, but it feels like it is, it s been going on for a while and it also hurts if I turn my leg in. Just wondering what it could be if it is from my ankle. Should I wait to see if it goes away or have this checked right away? It kind of feels like if you were holding a stick and trying to break it. Lots of pressure and pain, but once I walk a little it seems to go away. Doctor: Hello, As you had a fall on the side of the ankle I will advise taking an x-ray of the ankle straight away as any ignored injury of the past may land up into chronic and later leads to change in the biomechanics of the gait and leads to other pain symptoms at the adjacent joints. Hope I have answered your query. Let me know if I can assist you further. Regards, Jay Indravadan Patel, Physical Therapist or Physiotherapist"
},
{
"id": 160232,
"tgt": "I have a spot on liver, had cancer",
"src": "Patient: I went to the dr yesterday and the dr told me that when they did a cat scan of my chest it revealed a spot on my liver That has been over a year ago and they never told me of the findings. I have had cancer before is it possible it could be that? Doctor: Hi! with your previous cancer status the spot in the liver could be a metastasis or a deposit, however if it has been there for a year it is unlikely since in such a situation it would have grown or spread elsewhere.Also please elaborate what cancer you had before what treatment etc."
},
{
"id": 92758,
"tgt": "Have had nausea, abdominal pain and discomfort, diarrhea, back pain, shoulder pain. Done MRI and ultrasound. What is wrong?",
"src": "Patient: I have had nausea, abdominal pain and discomfort, and diarrhea off and on for over a month now. At first I couldn\u2019t really eat anything I had zero appetite, but always felt hungry. Then I started eating just fruit; worked my way up to chicken nuggets (from home not fast food). I was feeling better until I ate a fast food cheese burger, I threw it up an hour after eating it and was back a square one. Finally I discovered that grapes helped me feel better and be able to start eating a little again. I did good for about 5 days and then started feeling abdominal pain and nausea again. I was able to keep homemade yogurt fruit smoothies down and fruit. The feeling of hunger and no appetite to eat was overwhelming. I took to taking walks or pacing my hallway and that helped a little with the pain. I went to the chiropractor and after he pushed in and down my stomach the pain and nausea was immensely better and I was able to start eating again. Then I started getting extreme back pain (like I was stabbed between the shoulder blades and couldn\u2019t get the knife out). I had a MRI, ultrasound, and blood tests done. Although I am grateful that I have been feeling a lot better I still want to know what is wrong with me. I know the DR.\u2019s are trying, but a month and a half of this is really wearing on me. I have no energy and any of the slightest exercise besides my walking makes me feel weak. Thought I would see what another doctor\u2019s view point might be. Thank you for your time. Doctor: You are suffering from peptic dyspepsia,hence take LANSAPRAZOLE 30 mg one tablet at 6 pm on empty stomach daily for 1 month along with CYPROHEPTADINE tablet 1 per a day at 7 pm half-an-hour before dinner.Try to consult a GASTRO-ENTEROLOGIST& HEPATOLOGIST,for further treatment."
},
{
"id": 105765,
"tgt": "Breathing problem while walking,climbing,talking",
"src": "Patient: Dear Sir, I am of 28 years old. I have a problem of brathing while inhaling at some times, not continuosly specially when to have a continous discussion & communication with frends and othe people and feel suffocation . Even when to going upstairs of 4th floor apartment. it roots from back 5 years, started, when during teaching, (when i was a teacher), my throat was struk up, so after continous teaching & speaking with students, at the end of night, while going to sleep , feeling to tired off, or feeling imbalecment during breathing , but not much problem. Now i am doing a job in a private industry. i quit teahing since back three years. but have a problem of breathing while, walking, upstainring and to talks and chats with my friends, not continuous but at some times. while talking with my friends, i stop and take long & suffocated breath. Doctor: Hi welcome to H.C.M.Forum, you are having mild gastric complaint, and anaemia. for 2 days you eat only curd and rice. then you stop taking junk foods, spicy food, ristrict chillies, and sour foods and juices. if not relieved you can use PPI, antacid and iron preparations. thank you."
},
{
"id": 72412,
"tgt": "Suggest treatment for pain in sternum after an injury",
"src": "Patient: Hi, I fell down stairs last wednesday and I hit my back. But now I am experiencing sharp pain and pressure in my anterior sternum and it hurts a lot when I bend my chest to my stomach and when I raise up out of bed. I got several x-rays taken and the doctors said nothing is wrong. But I am in pain and I don't know what to do. Doctor: Hello dearWarm welcome to Healthcaremagic.comI have evaluated your query in details .* This is in relation with possible sprain / strain involving ligament around sternum , issue with neurovascular bundle or others .* Guidelines for better outcome - Heating pad application 5 minutes each 3 times a day .- Local application of analgesic spray or ointment .- Intake of proper analgesic as tramadol with paracetamol after getting prescription with your doctor .- Deep breathing exercises , YOGA will fasten the recovery .Wishing you fine recovery .Feel free to ask any further queries .Regards ."
},
{
"id": 187980,
"tgt": "Can numbness in lip and chin and gum area after having wisdom tooth extracted be considered normal?",
"src": "Patient: I had a bottom wisdom tooth that was impacted removed this morning at 9:00 am. I still seem to have numbness in the lip and chin on the right side along with the gum area? When the proceedure was finished the Dr told my husband that everything went fine? How long with this take to go away? Doctor: HiThanks for writing in.Its normal to have effect of anaesthesia after extraction.It will subside on its own with in 24 hrs.Do not panic.Complete your course of medicines as prescibed by your dentist.Take proper rest.RegardsDr. Neha"
},
{
"id": 144037,
"tgt": "What causes fever and high BP after a stroke?",
"src": "Patient: hi there, my 88 years mom had a stroke recently, she also has diabetes. she left hospital for a few days and had to go back due to high blood pressure and now she is developing fever can you tell me what would cause that high fewer please and thank you Doctor: Hi, Welcome to HealthCareMagic.com I am Dr.J.Mariano Anto Bruno Mascarenhas, and I am here to help you. I have read your question with care and understand your concerns. 1. Fever can be due to a. Autonomic Disturbancesb. Infection from bed Soresc. Lung InfectionsHigh BP can be due to a. Increased Pressure inside the skull due to strokeHope you found the answer helpful.If you need any clarification / have doubts / have additional questions / have follow up questions, then please do not hesitate in asking again. I will be happy to answer your questions.Best Wishes for Speedy Recovery Let me know if I can assist you further.Take care."
},
{
"id": 122716,
"tgt": "What causes sharp pain on the knee and calf?",
"src": "Patient: I get a sahrp pain that runs from the bottom of the knee through the calf down to my second from last toe. It does not do it all the time, might be a couple weeks between. But when it does it is very painful. The muscles do not get stiff but i can not walk and usually lasts from 5 seconds to 20 seconds. Doctor: Hi, As first-line management, you can take analgesics like paracetamol or aceclofenac for pain relief. If symptoms persist, it is better to consult a physician and get evaluated. Hope I have answered your query. Let me know if I can assist you further. Regards, Dr. Shinas Hussain, General & Family Physician"
},
{
"id": 64025,
"tgt": "Suggest medication for blackish lump noticed near the rib area",
"src": "Patient: My son has got a red lump suddenly near his rib area (we noticed after returning from shop) and it changed to greenish color after 2 hrs or so. He is not feeling any pain but that lump is somewhat blackish now. We thought it may be due to injury while playing in shop but others feel it may be due to someother reason also. My son is 3&1/2 years with good medical history. Doctor: Hi,Thanks for the query to HCM.-I studied your query in depth and understood your concerns.-Cause of the lump in rib are--In your case it is due to the -insect bite with bruise.Treatment--Diclofenac gel locally x 5 days-Plz keep a watch for 1 wk more.It would go on its own.Hope this would resolve your query.Wellcome further for any query in this regard.Wishing fast recovery.Write a good review ASAP.Have a Good Day."
},
{
"id": 11920,
"tgt": "Dark pigmentation on face. Using kligman with hydroquinone, switched to demelan. Demelan safe to use?",
"src": "Patient: HI i AM 53 YEARS OLD AND HAVE BEEN USING 4% KLIGMAN with hydroquinone for almost a year and half. I was advised by my dermatologist to stop and start using Demelan. My skin was very lovely while using kligmans but once i stop using for a few days the skin turns black again. Now that i am using Demelan I AM AFRAID THAT IT IS NOT GOING TO REALLY show results. I have very dark pigmentation on my face. please help. Doctor: Dear dhana..pigmentation problems need to be 'visualised' & 'assessed' by a dermatologist or cosmetolgist before prescribing medication as each patient's problem is unique. One man's food is another man's poison is an adage that springs to mind. I am happy the age old tried & tested Kligman's formula worked for you. I understand you are not happy with demelan. However, you may want to consider using a sunscreen in the morning depending on your dermatologist's recommendation. All the good work of the kligman's maybe being undone by lack of regular & proper use of suscreen that incidentally has to be used even on cloudy days & when not in contact with direct sunlight & rea-pplied each 3-4 hrs to be beneficial. I do hope this advice is useful. Do feel free to send in your photographs & contact me on this portal even in a private capacity if you prefer anonymity. Regds & good luck. Dr Praveen Rodrigues MD Dermatologist & Cosmetologist, Bangalore, India"
},
{
"id": 47017,
"tgt": "Is soreness in kidney due to pre work of hemo rage?",
"src": "Patient: im taking a pre work out by the name of hemo rage ive been taking the regular dosage maybe a tiny bit over once. i feel a sore ness on my kidney area. it could be from doing my work out but i just want to make sure this pre work out isnt causing problems for my kidneys Doctor: HiMost likely this isn't a problem with your kidneys since most kidney problems except for kidney stones don't cause pain.That said, if this doesn't resolve in a few days you should have a doctor check it out"
},
{
"id": 157980,
"tgt": "Swollen tongue, vomiting, lymphoid hyperplasia, difficulty swallowing. Done biopsy. Is it cancer?",
"src": "Patient: I have had a swollen tongue on the rt side only for 8 months and ct showed enlargement on the rt side so I had a biopsy of the basevof my tongue. It showed reactive lymphoid hyperplasia. Two months later my tongue feels larger and im having real difficulties getting food caught in my throat to the point of coughing for hours and even vomiting while trying to get food dislodged. My Dr is not concerned about a dx as long as its not cancer. After getting the path report today I googled this hyperplasia and saw several instances of cases similar to mine. Why didnt my Dr know about this pseudolymphoma and explain to me thats what I had instead of saying he had no idea what was wrong? He was ready to refer me to Mayo Clinic! Anyway, could I need possible surgery to remove the tissue thats catching food? Internet says sometimes its necessary for symptom management but now I dont really trust my Dr so I dont want to \"bother\" him with more concerns. I just know I cant live like this. Im now avoiding solid food. Who should I consult and is it possible that I may need surgery? Doctor: Hi! welcome to HCM! reactive lymphoid hyperplasia of tongue is an uncommon entity.lesion behaves in benign manner .The term pseudolymphoma is unfortunate because it has word lymphoma in it though lesion is not malignant. since your biopsy has been done and it has shown no malignant cells and just reactive lymphoid cells so you can be assured its not lymphoma just benign proliferation of lymphoid cells. you can consult an ENT/surgeon and let him examine and if required treat it surgically. hope this will help you. take care!"
},
{
"id": 159329,
"tgt": "Had Mirena replaced. Having period, not heavy. Pap smear shows low grade abnormalities. Cancer?",
"src": "Patient: I have been using a Mirena now for almost 10 years... yes I replaced 5 years ago.... since i had the new one in 3 1/2 years ago.... I had the occasional period that last about 4 days this continued for about 3 months I was put on medication then my period stopped, stomach cramps , but the period it is not heavy.... is this normal....I have again started with a period again and no it is not heavy.....I had a recent pap smear (4 weeks ago)and they found Ihad low grade abnormalities ..... does this mean anything serious.... (like cancer ) Doctor: Hi, low grade abnormality does not signify anything, proper pathology report you have to mention. there may be high grade squamous intraepithelial lesion or low grade intraepithelial lesion. management will differ. consult your gynaecologist. take care."
},
{
"id": 24524,
"tgt": "What dosage of acitrom is needed for normal prothrombin time/international normalized ratio?",
"src": "Patient: I am a 45 years old male with coronary artery disease.I was on acitrom dose of 1/1/2 mg but that didnot work out as now my PT/INR values are 111.5/ 8.69.Now the doctor has told me to stop taking acitrom.According to pharmacogenetic testing of acitrom for VKORC1 &CYP2C9 gene,I am responder for VKORC1 gene, normal metabolizer for CYP2C9*2 and poor metabolizer for CYP2C9 *3.So now what dosage will be approprate for me to maintain normal range of PT/INR? Doctor: Hallo... acitrom one dose can not be told ...we usually titrate dose .. every weekly ...once stable inr is there ...once we do every 4-6 wk inr ...some pt on 1mg dose achieve targeted inr but some pt .. require higher doses. Just monitoring frequently to keep inr in range and titrate dose"
},
{
"id": 81216,
"tgt": "Suggest remedy to cure constant coughing",
"src": "Patient: Sir, I am constantly coughing for the past the 2 months now. I have been taking anbiotics which give temporary relief for 2-3 days. In these days too I do cough but it is reduced, but never stops. The coughing increases to levels where I ultimately land up puking the entire food. Such coughs are accompanied by headaches and cold. I did my AEC tests and awating the results. Doctor: Thanks for your question on HCM.I can understand your situation and problem.You are having chronic cough (cough more than 2 weeks). And chronic cough should be evaluated for1. Lung infection like tuberculosis and pneumonia2. Bronchitis and asthma.So consult pulmonologist and get done1. Chest x ray to rule out infection.2. CT thorax. If chest x ray is inconclusive to diagnose lung infection.3. PFT (pulmonary function test) to rule out bronchitis and asthma.Don't take haphazard treatment. Better to consult pulmonologist and discuss all these possible cause and start treatment accordingly."
},
{
"id": 66279,
"tgt": "What does a painful lump in elbows indicate?",
"src": "Patient: I have a painful, moveable lump in both of my elbows that I have no idea what are. I have had one lump in my right elbow for quite some time, but it has just now started to hurt. The lump in my left elbow just recently happened after I fell on my left elbow playing basketball. Doctor: Hi, dearI have gone through your question. I can understand your concern. You may have some benign cyst like sebaceous cyst or ganglion cyst. Or you may have some soft tissue tumor like lipoma or neurofibroma or something else. You should go for fine needle aspiration cytology. It will give you exact diagnosis. Then you should take treatment accordingly. Excision is the treatment of choice. Consult your doctor and plan accordingly. Hope I have answered your question, if you have doubt then I will be happy to answer. Thanks for using health care magic. Wish you a very good health."
},
{
"id": 106198,
"tgt": "Can i give loratadine and piriton to my 16 month old baby",
"src": "Patient: can i give a 16 month old loratadine and piriton for two weeks? my 16 month old has been coughing and sneezing and having a runny nose since her first birthday, now the ENT doctor prescribed loratidine and piriton for two weeks,is that okay? Doctor: Did you wonder why you child has been coughing and sneezing and having a runny nose for 4 months? Not normal. Candida overgrowth is to blame research this subject and a light is going to go off in your head. Probiotics will help cure the problem and yes it is safe to treat the symptoms with loratidine 2.5 ml or 1/2 tsp once a day that is all that is needed until you fix your baby from the yeast making him/her sick. I use BioGaia when my kids were neonates and threelac after age 1."
},
{
"id": 47072,
"tgt": "Is it possible to have kidney transplant after pulmonary edema and heart failure?",
"src": "Patient: My grandfather is 75 yeras old with ESRD. he is undergoing dialysis for the last 3 years. recently he had 3 episodes of pulmonery edema with heart failure. he has haemodialysis 4 days a week now. is there a possibility for kidney transplantation. he has been a chronia diabetes typeII patiend for over 30 years. Doctor: Hello and welcome to HCM.As an Urologist and transplant surgeon, i can understand your concern.Kidney transplants are usually not done at 75 years age.The rate of rejection is high,as the risk rates are very high.Recurrent pulmonary edema and heart failure,with long standing diabetes, are major medical conditions. Surgery also is a high risk major procedure.All these are to be considered,in decision making.It's best for him to continue with maintainence dialysis.Dr.Matthew J. Mangat."
},
{
"id": 176135,
"tgt": "Suggest treatment for fever and cough",
"src": "Patient: my daughter is 11th month old she is sufering fever and cough from last three days.after blood test,maleria came.last two days she is taking medicine but cough is not getting down.she is not kaing any food only salaine is giving.pls advice any symptomp of swaine flu. Doctor: Fever and cough are very common ailments in children of such a tender age. Most of the times it is caused by certain viruses but it does not necessarily have to be swine flu. Separate varieties of influenza affect humans and pigs. When the variety that affects pigs, affects humans, it is called swine flu. The presentation is simply cough and cold and nothing more, the symptoms that we commonly refer to as 'flu'. Let me assure you that the hype about swine flu is more because of technical reasons like genetic recombination in the influenza virus giving rise to new strains which could cause a pandemic. But such developments are extremely rare and you need not worry. In case you are apprehensive, you can get flu vaccine for your child.The treatment of fever and cough is use of plain paracetamol to reduce temperature and use of some antiallergic like cetirizine. Antibiotics are generally not required as they are ineffective against viruses. It should recover on its own in 5-7 days."
},
{
"id": 176317,
"tgt": "Why is an eye examination prescribed for a 18 month old child?",
"src": "Patient: My doctor has ordered an eye examine for my son (age 18 months) because he is not developing and recently for 1week has been throwing up and has lost weight. He is only 17 pounds after this past week, and has been slow to gain up to now. Roy e slayton @icloud.com Doctor: Hi...some of the inborn errors of metabolism have eye and fundus and retina problems. So when a kid is failing to thrive and developmentally delayed it is usual for the pediatrician to ask for an ophthalmological evaluation and consult. It is indicated very much as it will give clue tot he diagnosis or diagnosis itself.Regards - Dr. Sumanth"
},
{
"id": 101586,
"tgt": "Suggest treatment for severe allergy",
"src": "Patient: Dear Sir, Iam35 years of age weight approx 70kg. i have severe allergy-first thing in the morning i fee wetness in my nose and then by 30 minutes i have severe sneezing and nose discharge and heaviness. this is the daily routine. I tried taking homeopathic medicine . There was some releif then the same continued. Please advise Doctor: Hi,I had gone through your query and understand your concern.Your symptoms are suggestive of allergic rhinitis.Before giving my opinion ,I need to know few information like >Nasal irritation,nasal blockage,triggers like dust,cold etc in favor of rhinitis>cough,wheeze ,breathlessness to exclude asthmaThen I would advise you to :>Wash nasal cavity with normal saline or water>Steroid nasal spray>Oral antihistamineSince this is prescription medicine ,so my suggestion is :please meet your local doctor and get treatment accordingly.Hope this helps you.If you have additional queries,please do not hesitate in writing us.Wish you good health."
},
{
"id": 160019,
"tgt": "Black spots on liver. Had cancer surgery before 6 months",
"src": "Patient: My wife that is 31yrs old had a major operation 6mos ago. She had thymic cancer with 3 large malignant thymomas pressed against her right lung her heart and her liver ..they removed her right lung and removed then reconstructed part of her pericardium (lining of her heart) and the tumor on her liver was completely encapsulated so they didn t take a slice of her liver like he though he would have to . Dr. Calhoun of uc davis who is a straight up stud removed 100% of the tumors it looks like so far. Her first 3mos checkup was fine last week for her second checkup they said it was still OK. They called us today and said she needs to come back for another ct scan because they just noticed dark spots on her liver..what would you think this could be?...they think her tumors were very slow growing so it wouldn t make sense that already it would be showing up..but this is a very very rare case. So they don t have a lot of other cases they can refer to. She spent a month in ICU because of every complication in the book..a lot of infections (cdif etc..) I am hoping it s just something like that. what do you think the odds are for all the different possibilities it could be ? Doctor: thanks for the query from the history it is quite evident your wife hads had a very tough time but from the diagnosis of malignant thymomas i have to tell you that it means the liver has got metastasis from the thymomas that is what they mean by the dark spots in the liver and mostly they want to resect it now i would suggest you talk to the oncologist who's incharge of the case for a clear cut explanation take care."
},
{
"id": 157616,
"tgt": "Diagnosed with stage four ovarian cancer. Took colostomy. Seeds were left on the liver and diaphragm. What could it be?",
"src": "Patient: Hi, I appreciate any assistance you could give me. WIthout going into my aunts whole history, the brief version is that she was diagnosed with stage4 ovarian ca. A massive debulking of the abdomen was done as well as a T.A.H. During SX, the Dr found that the cancer was wrapped around her colon as well. AT that time, she was also given a colostomy. Unfortunately seeds were left on the liver and diaphram as well as a small mass. She is undergoing Chem at this time. I lost my 38yr old sister to P.C, and my mother a few months ago from Lung Ca. I am my aunts only caregiver and was wondering what your educated guess would be with regard to prognosis? I haven't asked her Oncologist to date because she is always in the room with me and doesn't want to know. My sister didn't want to know her prognosis either but as her primary caregiver as well, I managed to catch her doc alone, so I was prepared as best as I could be to lose her in 8 short weeks. Doctor: Hello, I have gone through your query and understood your concern. According to above mentioned details, stage IV ovarian cancer overall median survival rate ~36 months and progression free survival ~17 months. As you have said debulking had done and residue has left, 5 year survival after surgical debulking with residual disease >2 cms is ~20%. 2 year survival rate for stage III+IV disease was 60%. Hope this helped you. Thanks."
},
{
"id": 225888,
"tgt": "Taking birth control pills. Had periods for 2 weeks. Had unprotected sex. Feeling sick in mornings. Pregnant?",
"src": "Patient: Hello,I have just started the pill after not been on it for quite some years....within 3-4 days of taking the pill I bled for about 2 weeks none stop,I then had finished a pack and had to take my 7 day break....so I took my 7 day break and was having unprotected sex,within that 7 day break I never bled once and still arnt bleeding now....could I be pregnant,I have been feeling sick in the mornings but I put that down to me been anemic ?? Doctor: Hi,Thanks for posting your question here, I will try to answer it to the best of my abilities.60% of women have an egg ready to go on the last day of the break, which means that if you take the pill even a few hours late on that day, there is a chance of getting pregnant.As such question yourself did you take the pill at the right time? If not, well take a home pregnancy test in a couple of days. That should let you know what to expect.I hope this answered your question."
},
{
"id": 166353,
"tgt": "What does itchy, spreading small bumps on the body of a child mean?",
"src": "Patient: Hi, may I answer your health queries right now ? Please type your query here... My son is 6 years old, earlier today he had some itchy small bumps on both of his forearms. I put some lotion on them and he did not say anything else. We finished our evening routine and he just woke up, covered in sweat with the small bumps now spread to his chest, upper arms, face and some on his legs. I tried to take his temperature but the reading twice only went up to 95. He is scratching and crying. Doctor: hi,Welcome to this forum.Can understand your concerns.this type of Rash can be due to insect bite, measles, are chicken pox.The Rash due to insect bite usually occurs on exposed parts of body like hands and legs, there is itching on the rash.In measles, the Rash appears all over the body and there is associated fever and itching.In measles, there are fluid filled rash which increase overtime and there is itching.You should attach a picture of the skin lesion for proper advice.I hope this will help you. Wishing your childhood health . take care."
},
{
"id": 107508,
"tgt": "What causes lower back and pelvic pain?",
"src": "Patient: Hi doctor, iam 32 yrs old. Iam having left pelvic as well back pain,only left lower side ( both). Pain Is not stable...it s very mild, stays for a sec and the goes...but I feel even during movements in left side...have this problem for past 6 months...iam very much worried. FYI I had my second delivery inJan 2015 (VBAC) Doctor: Hi welcome to HCMI have gone thru your query and can understand your concern . Pelvic pain can be a sign that there is a problem with one of the organs in your pelvic area, such as the uterus, ovaries, fallopian tubes, cervix or vagina. It could also be a symptom of infection, or a problem with the urinary tract, lower intestines, rectum, muscle or bone .So there can be many causes concerned with your problem .You might have to undergo a lot of medical tests to find the cause of the pain. The treatment will depend on the cause, how bad the pain is and how often it occurs.Another prominant cause of your pain also can be imbalance of (Vayu Tatv gas , in your body , which is responsible your pains and inflammation .Proper regular physical activity , walk ,meditation ,pranyam - deep breathing ,Yoga - back bending exerises to relieve your back pain , to boost your blood circulation and detox your system under guidance of trained guide and balanced diet, including ,all essential nutrients as fiber ,vitamins,minrals and antioxidants Ginger , Garlic , Amla , Aloe vera , nuts , Turmeric powder a spoon in a cup of hot milk at bed time ,Cocconut water ,all in natural form help, eradicate Vayu Tatv ( gas) alley pains and inflammation and give resistance from diseases . Avoid food that is high in saturated or trans-fats like fried , fast foods , refined sugars and fine floor prducts , tea , coffee , alcohol , sleeplessness , dehyderation , constipation .worry ,stress all induce toxins in your body and cause you sufferings .It is advisale to consult your doctor before following any regimeHope this helps solves your query .Take care , All the best .Don't hesitate to get back if have any further queryv"
},
{
"id": 31823,
"tgt": "Does runny nose, watery eyes and cough indicate swine flu?",
"src": "Patient: my best friend came over a few days ago and called me yesterday and said he had an upper respiratory virus. every since he left my house, my nose is stuffy, i have a SEVERE cough that will not stop, and watery eyes. i do not get sick often, maybe once a year. i am scared. do u think i have the swine flu? Doctor: Hi.Welcome to HEALTHCARE MAGIC.I have gone through your query and can understand your concerns..As per your complain in case if your friend had upper respiratory tract infection and he visited you during the active course of infection there is a possibility that you have got exposed to the virus particles leading to upper respiratory tract infection and the stuffy nose or congestion and cough can be due to it..Although these symptoms can be seen in case of swine flu also but chances are more towards upper respiratory tract infection and not swine flu..I would suggest you to consult an Otolaryngologist and get evaluated and a clinical evaluation and blood investigation can be diagnostic..As of now you can take decongestants like Phenylephrine orally and nasal decongestant sprays containing Xylometazoline..You can also do warm saline gargles and steam inhalations..Avoid chilled foods..Take Tylenol for generalized body aches..Hope this information helps..Thanks and regards.Dr.Honey Nandwani Arora."
},
{
"id": 142148,
"tgt": "What causes persistent voice hoarseness after removal of acoustic neuroma?",
"src": "Patient: I had an acoustic neuroma removed Feb 2015. in the last 2-months I have constant hoarseness in my voice. I also have had increased difficulty with swallowing and have omitted certain foods. About the same time i started having hoarseness, i was having problems with vomiting. My PCP Rx Bentyl and it has helped.I return to Mayo in Rochester in a couple of weeks. Any suggestions Doctor: Hi, I understand your concern.Hoarseness in your voice could be due to an acute irritation of the larynx and trachea and vocal cords. It could be due to acute viral infection, or irritation of acidic liquid of stomach when you had the episodes of vomiting. It is better to get checked by ENT specialist to determine the right diagnosis. Keep good oral and throat hygiene. Gargle with warm water and salt. Take antihistamine orally until you see ENT specialist. Wish fast recovery."
},
{
"id": 50969,
"tgt": "Lump above belly button, pain on touching. CAT scan shows kidney failure, blockage in bladder. Treatment?",
"src": "Patient: i have a lump above my belly button and it hurts to touch or when i lean against something. I just recently found out that my left side kidney is not working and has not for a while the way i understand from the two CAT scans that i have had. I was told that there is a blockage on the top of my baller, so is this why i am hurting and have the knot above my belly? I am tired of it hurting for I can not get any rest with it hurting. Doctor: hello, welcome to healthcaremagic, from the description of your symptoms, the swelling you might be having at the belly button could be -abscess -hernia - mass would suggest you need to get an ultrasound done as you have other abnormalities present, to know what is exactly wrong. would suggest get reviewed by a specialist for appropriate investigation and treatment. wish you a quick recovery and all the best. take care."
},
{
"id": 150921,
"tgt": "Nausea, blurred vision, limited hearing, pressure in head after fall. Need to worry?",
"src": "Patient: Hi I m 20 an I just fell on stairs really bad. I ]hit with my butt almost a dozen steps covered in carpet. When I got up and I got so nauseous, my vision got blurred and limited and I was hearing as if my ears were blocked and I felt pressure in my head. This went for like 5 minutes, now I m sitting on one side and I can t feel my butt but I m sure that if I try to lay on my back it will hurt like hell...should I get worried about my temporarily reduced vision and hearing or about breaking my tailbone? I must mention that I did fractured my tailbone when I was around 12 years by falling on ice. I m waiting for your opinion at YYYY@YYYY OR YYYY@YYYY Doctor: hi there, am dr.surendran nice to meet you here in HCMagic Unfortunately, once you\u2019ve had one spinal fracture, chances are high that you\u2019ll experience another one within a year.Because spinal fracture symptoms are often dismissed as normal back pain that comes with age, many people don\u2019t receive proper treatment soon enough. This increases their risk for developing long-term health complications from spinal fracture, such as spine deformity. Minimally Invasive Procedures for Treating Spinal Fracture . other non surgical modality, Vertebroplasty is a pain treatment for vertebral compression fractures that fail to respond to conventional medical therapy, such as minimal or no pain relief with analgesics or narcotic doses that are intolerable. Vertebroplasty, a nonsurgical treatment performed by interventional radiologists using imaging guidance, stabilizes the collapsed vertebra with the injection of medical-grade bone cement into the spine. This reduces pain, and can prevent further collapse of the vertebra, thereby preventing the height loss and spine curvature hope helped your query regards dr.surendiran hcmagic"
},
{
"id": 18110,
"tgt": "What causes tingling sensation on the face while suffering from idiopathic intracranial hypertension?",
"src": "Patient: Hi. I had an MRI done Wednesday. The results showed: No acute intracranial abnormality. Few punctuate remote microhemorrahges in the left parietal lobe. Also Empty sella. Bilateral petrous apex cephaloceles associated with the Meckels caves. Bilateral dilation of dural caves of the oculomotor nerves and of cranial nerves IX-XI in the bilateral jugular foramina. The diagnosis-----Idiopathic intracranial hypertension. I looked up iih. Symptoms are headaches, and vision loss. I have neither one. I originally went to the doctor because I was having tingling in my lips and face. Any suggestions? Doctor: Hello and Welcome to \u2018Ask A Doctor\u2019 service. I have reviewed your query and here is my advice. I would explain that this is a neurological disorder, related to the increased pressure inside the brain space. I would recommend consulting with a neurologist and discussing about the possibility of starting Acetazolamide. Steroids may help improve your situation too. Hope to have been helpful!"
},
{
"id": 133550,
"tgt": "How to treat ankle sprain?",
"src": "Patient: ankle sprain a week ago (due to sports, have sprained many times). was a typical stepped in a hole and rolled it type of injury (on golf course). this swelled a bit and then went down quickly, was painful for a day or so and then no pain. BUT rash on outside ankle bone (the joint bone area) and itchiness persists. haven t seen doctor, resumed physical activity...but not sure what s going on. Doctor: 1)If it is very recent injury (2)do crepe bandaging around ankle jt.3)at night while taking rest elevate ankle so that swelling can be avoided4)if swelling is presented around ankle jt orthopaedic concern is needed.5)take rest and do not apply pressure on affected limb6) take concern of Physiotherapist."
},
{
"id": 13451,
"tgt": "How long does it take for bug bites rash to go away?",
"src": "Patient: I have had a severe allergic reaction to bug bites. The rash is bright pink and is covered on my arm back legs and feet. How long will it take for the rash to go away? I have been prescribed hydrocortisone 2.5% and hydroxyzine 25mg tablets and also prednisolone. I m very distressed please respond to this. Thanks Doctor: Hi, The rash usually resolves in around 2 weeks. But new rashes occur, if you have not made your environment bug free. You can continue the treatment advised by your doctor and follow him/her. Hope I have answered your query. Let me know if I can assist you further. Regards, Dr. Siva Subramanian, Dermatologist"
},
{
"id": 82780,
"tgt": "Is olmark suggestible for proteinuria due to SLE of kidney?",
"src": "Patient: Hello sir, My sister ( 37 yrs) was detected with SLE two yrs back March 2010,at that time she wasnt passing urine,she had abdominal pain and was vomiting. Her creatnine level was 6 at that time.She had rashes on her face and arms.ANA test was positive and kidney biopsy confirmed SLE.She was put on dialysis ,at the same time they detected the blockage in the intestine also.Ileostomy was done and stoma bag was fixed,she was given the treatment of immuno suppresents and ENDOXIN,her condition improved and the creatnine level came down to 1.5, but she continued with the problem of proteinurea and high blood pressure, her albumin level was still 1 ,so it took 6 months to again join the intestine .Protein leakage was +++ in the routine urine test.Her creatnine level was stable between 2 to 2.5 for two yrs ,problem of proteinurea continued ,now it is at the level of ++.and her BP also remain at the higher level.Now the problem is that her creatnine level has reached 4 ,had a problem of Ascites also 250cc of water is removed last week, test of water is normal.As her TLC level came down to 3.9 dr has stopped MYFORTIC ,but he has prescribed OLMARK 40mg .Do you think that is ok? Doctor: yes, It can help to prevent protein loss from urine,but this should be avoided if creatinine is more than 4"
},
{
"id": 218287,
"tgt": "What causes brown vaginal bleeding and thick endometrial plate with no RPOC?",
"src": "Patient: Please i missed my period for 2 months and 2 weeks and had all the symptoms of pregnancy and some days back i spotted brown discharge and 2 days later i stared bleeding with no pains. I went to see the doctor and this was the text result giving 2 me That no myometria mass seen and that the endometrial plate measured 8.6mm thick. There is no obvious cyesis or retained products of conception( rpoc ) Doctor: ..mam your ultrasound report saus that there is no pregnancy ( cyeisis )..there is no retained products ..as in some abortions some products remain in uterus .no myometrial mass means there is no fibroidso just relax cycle sometimes get delayed wait for next month if the cycle is not regular by then please get ascessed for hb ..Tlc ..Dlc ..ESR...Thyroid status as your ovaries and uterus are normalif you have any further query then please come back to hcm site ..i will be happy to help you with regards dr varinder joshi"
},
{
"id": 74908,
"tgt": "What causes chest pain and tooth pain?",
"src": "Patient: Hi, i had an angioplasty in 2003 and had another angiogram a few months ago everything looked ok after having chest pain, i have been having a major toothace and am suppose to have it extracted tommorrow but now woke up with chest pain going down arm should i ignore that as they told me all was ok last time? Doctor: Respected user, hi I evaluated your query thoroughly.*Even though work-up before few months was ok , one should never ignore chest pain anytime with pain going down arm.*I strongly recommend to seek medical advise by consulting Physician doctor immediately without delay.Hope this clears your doubts.Thanks for using Health care magic & keeping trust in our medical services.Welcome for any further assistance.Wishing you a happy healthy life ahead.Regards dear take care."
},
{
"id": 196837,
"tgt": "Does leg cramps occur frequently during intercourse?",
"src": "Patient: My guy had a leg cramp the other night while we were having intercourse. We enjoying a particularly good session with lots of foreplay. It has never happened before. He said it didn't hurt, but rather felt good. Huh? Is this something that happens frequently with guys? Doctor: no, this might be an occasional cramp but if it is repeated then he should check his potassium level in blood"
},
{
"id": 87769,
"tgt": "How to treat mild burning in the abdomen area after having laparotomy?",
"src": "Patient: I had a laparotomy 20 days ago. I have been walking daily, usually 3-4, 30 minute walks, but I am experiencing a mild \"burning\" in the area of the incision along with the regular inflamation that goes along with the procedure. Should I be icing the area, massage? Any suggestions, and is this normal. Doctor: Hi.Thanks for your query and an elucidate history.It is nice to know that you are walking after 20 days of Laparotomy.To have a mild burning pain is usually due to the nerve endings being cut during the procedure and caught during stitching and getting entrapped in the fibrosis which is a normal process of healing. This is normal in many a patients an dwell, the response of the patient depends also on the pain-bearing capacity, that is tolerance too. This will get solved over the time over the next 3 to 4 weeks. Icing : try and continue if it helps. Plain paracetamol in an appropriate dose may help on the basis to be taken only if necessary. Massage should never be done as it may help temporarily but can alter the body's natural healing responses. Do not worry since you have only mild form of burning. You do not need any active treatment. Continue walking."
},
{
"id": 169554,
"tgt": "Is enlarged heart and brain in fetus sign of digeorge syndrome?",
"src": "Patient: Hi doctor,today I had my 20week anomaly scan and unfortunately all wasn t well n we discovered that our unborn baby is sfd and also has an enlarged heart(which takes uo to a 3rd of the abdominal) and enlarged posterior ventricle in the brain. By partners first son (with another woman) was born digeorge syndrome could this be connected? Doctor: digeorge syndrome consists of heart defects ,clefr palate.,,cleft lip,loe set ears,widely spaced eyes.and per se those two signs not enough to say it as digeorge syndrome."
},
{
"id": 70515,
"tgt": "Could lump below bikini line which is sore be a blister?",
"src": "Patient: Hi! For the last two months I have had right side pain below rib cage, had an ultra sound done and no gull stones so my doc wants me to get an endoscopy. Yesterday I found a lump (cyst like) below my bikini line, inner thigh on the left side. It's sore to touch but not painful. Could this be a hernia? Doctor: HI. A lump below the bikini-line is more probably a boil or an impending abscess.This is the most unlikely site for an hernia.Hernia are very rare in females /"
},
{
"id": 62235,
"tgt": "Suggest treatment for a lump on the wrist",
"src": "Patient: I have a slow growing lump on my inner wrist. It seems to be fluid filled, is not painful, and when I shine a bright led light through it, the resultant color is a yellow hue; it is not opaque. It seem to be unrelated to any acne, as there seem to be no loci of a potential skin opening. It has grown significantly over the last year, though doesn't seem to interfere with the use of the hand. What should I do about it? Doctor: Hi, dearI have gone through your question. I can understand your concern. You have lump in inner side of wrist and it is fluid filled. No opening on skin. Your all description favour either ganglion cyst or some other benign cyst. You should go for ultrasound study once to know exsct site and location. Complete excision is the treatment of choice. Consult your doctor and take treatment accordingly. Hope I have answered your question, if you have doubt then I will be happy to answer. Thanks for using health care magic. Wish you a very good health."
},
{
"id": 26881,
"tgt": "How to treat accelerated pulse rate?",
"src": "Patient: I was sitting in class when my pulse suddenly started to accelerate and i got light headed my nose started to run I became extremely uncomfortable and I wasn't able to sit still. My hands are quaky and I feel like I just got punched in the stomach. What could this mean? Doctor: Hello!Thank you for asking on HCM!Your symptoms seem to be caused by a panic attack. I would like to know your age. Have you experienced other similar episodes before? I recommend consulting with the GP for a careful check up (physical examination, a resting ECG, a routine blood test, blood pressure monitoring, thyroid hormone levels, blood electrolytes, blood sugar, kidney and liver function tests). If all the above mentioned tests result normal, the most probable cause would be a panic attack.Hope to have been helpful!Greetings! Dr. Iliri"
},
{
"id": 161826,
"tgt": "Why does the child reject the bottle?",
"src": "Patient: my baby is almost 10 months and still rejecting bottle. Currently, I have to stop nursing due to illness, and my baby have been vomiting and still refuse any bottle. the only time i can get a little fluid come to her mouth is when she is sleeping, but these causing her waking up often. should i continue bothering her in her sleep, or forcing her take the fluid when she s awake? Doctor: Hi, Mother's illness unless it is hepatitis B or HIV should not refrain the mother from feeding the baby. I suggest that you continue breastfeeding and stop bottle feeding if your baby is refusing the bottle. Hope I have answered your query. Let me know if I can assist you further. Thank you. Regards, Dr. Sumanth Amperayani Pediatrician, Pulmonology"
},
{
"id": 20700,
"tgt": "Can heart rate have increased due to high Blood pressure?",
"src": "Patient: My Blood pressure is recording 166 over 86 and my pulse is 69. I had 24 hour cuff which showed an average of 149 over 70 but that was last year. I do not have medication because my heart rate goes so low 40 that I get dizzy and extremely tired and get short of breath on the medication. Am I severely at risk if I continue with my blood pressure at this level and occasionally the systoloc reading can br higher but yhe distoloc reading is rarely above 85? Doctor: hello Welcome to HCM. Your BP is definitely higher as documented by the 24 HR cuff and this reading of 166/86 and you must be started on medications. High blood pressure over a long time can cause target organ damage which includes heart disease, enlargement of chambers of the heart, brain stroke, kidney damage and damage to retina of the eye among others. Since you have been suffering from High BP for more than one year, it may have started gradually affecting your body . There are many other medications which lower the BP and do not cause a decrease in the pulse rate. You can be safely started on those medications. I recommend tab telmisartan 40 mg once daily in the morning. Please consult your primary doctor about this. Wishing you good health.Regards"
},
{
"id": 201877,
"tgt": "Could low testosterone be the reason of having premature ejaculation?",
"src": "Patient: Yes I currently experience premature ejaculation and was wondering if could be a sign low T? I am currently a 52 year old male. I am in good health 6 foot and weigh 205 lbs. And lately I always seem to be tired with no energy and sometimes feel low for no reason. Doctor: HIWell come to HCMThe symptoms that you have must be because of the age and this is nothing to do with the hormone level, and what ever the hormone level you are having this would also be due to the age and this is nothing to worry and no need to treat this condition, hope this helps."
},
{
"id": 58875,
"tgt": "Taken Uniferon, Ribazol, Liv 52 for hepatitis C. Weakness, dizziness. Suggestion?",
"src": "Patient: I am from Pakistan having problem of Hepatitis C, First it got Uniferon + ribazol for about 9 months but result finally was detected. In the meantime, I was using Liv 52 one tablet daily and feeling good. But doctor suggested to go for PEGASYS + RIBAZOLE 400MG, but after 13 injections result obtained was not 2 log, so again doctor ask me to stop treat and go on Liv 52 1+1. But If I take 2 tables daily I feel further weakness and dizzy. Because of this reason I unable to do my office work properly.Whereas doctor and Live52 advisor ask this is a very good supplement but your said complain probably because of any other reason. Now suggest me what to do, my age is 47 Doctor: Hello, Thanks for the query to H.C.M. Forum.Since , you have full treatment of hepatitis C ,but ,what about the report at present ? Also mention the reports of liver enzymes such as S G O T, S G P T. bilirubin, c p k , Amylase ( pancreas). Liv 52 are commonly used drugs in Hepatitis . In my opinion it never causes any weakness. So your age is at present 47 years and this age is prone to many diseases. Consult a physician and get in blood for C B C, E S R , sugar , lipid profiles. Routine blood pressure is also important. Hope I have answered your question. If further any question I will help. Get well soon. Dr. HET"
},
{
"id": 306,
"tgt": "Is pregnancy possible despite thyroid problem and tumor in the womb?",
"src": "Patient: My sister in law is now 41 yrs and is having thyroid and tumor in her womb. Moreover her menstrual period has stopped from last month only. Previously due to some family reasons she had 3 times abortion. Can she have a child now? How much expenses is involved to do so? Please inform me kindly. Doctor: Hello and Welcome to \u2018Ask A Doctor\u2019 service. I have reviewed your query and here is my advice.Get thyroid function, serum TSH, free T3, and the serum prolactin done. Send USG report as we need to know the exact location and size of tumor, after that we can decide.Regards,Dr. Sheetal Agarwal"
},
{
"id": 200446,
"tgt": "What could small bump in scrotum & anus be?",
"src": "Patient: HI, Recently I noticed a Small BB sized (or smaller) bump Inbetween my scrotum and anus closer to my scrotum, and it is deep under the skin, cannot see it. and when I push hard on it I feel pressure, It also moves around, I am going to get a physical but before I do I would like some suggestions, i am very worried and do not want it to be cancerous. Please help. Doctor: Thanks for asking in healthcaremagic forum A bump may be just a cyst/boil/neurofibroma. If may not be cancerous. Please do not worry instead visit a doctor/surgeon for exaination and further management. All the best."
},
{
"id": 160885,
"tgt": "Can a child contract hepatitis-B infection from a carrier?",
"src": "Patient: A member of our family has been diagnosed as hepatitis b carrier. She ( the hapatities b carrier ) has been looking after my little daughter ( 24 months old) for the last 3 months. My daughter has taken 4 series of vaccination against hepatitis b ( 1 directly after birth , and the other 3 vaccinations over the last 2 years). My question is that is there any chance that my daughter could be affected by the disease? Doctor: HiSince your daughter is vaccinated and the caretaker is in a carrier state,the chance of infection may be less.Serological test can be done to confirm the presence of antibodies due to vaccination.Hope that was helpful.Let me know if i can assist you further.RegardsDr.Saranya RamadossGeneral and Family Health Physician"
},
{
"id": 131446,
"tgt": "Noticed a big bruise on calf with a knot in center",
"src": "Patient: I have a big bruise on my calf with a knot in the center. The bruise is fading after a few weeks. It s still got a few little circles around the knot. But I noticed that I had some swelling in that ankle the last few days. And before that I had quite a bit of discomfort towards the bottom of my knee, and on the top of my foot into my ankle. Are these possibly all connected? Or should I not worry about it? My other leg is fine. Doctor: In my opinion you should consider deep vein thrombosis , try a simple test by streching your calf for 1 min then sudden release , if pain increase then you probably have DVT in which case i recommed visiting ER for doplarGood Luck"
},
{
"id": 120576,
"tgt": "What causes pain and cramp under the cast for fracture in ankle?",
"src": "Patient: hi, i have a spiral fracture in my ankle and my leg and ankle will be in cast for 6 weeks, im 3 weeks in an i am in agonising pain with cramp under my cast. I,ve read several things such as lack of potassium and salt can cause this and have heard the most severre symptom can be a blood clot. Is this common and can it be avoided?? Doctor: Hello,Burning, Stinging, or skin Irritation can be caused by too much pressure on the skin or by a wet\u00a0cast. To ensure that your bone continues healing properly, call your orthopedist to schedule a splint or\u00a0cast\u00a0change. Meanwhile,I suggest using painkillers such as Ibuprofen to relieve the pain.Hope I have answered your question. Let me know if I can assist you further. Regards, Dr. Dorina Gurabardhi, General & Family Physician"
},
{
"id": 192940,
"tgt": "Could masturbation cause infertility?",
"src": "Patient: Hi Sir,this is rumaan{Name changed) i m 26years old staying in dubai got married last year this is my second visit to india we have lot of sex everyday but the resulT my wife didnt become pregnant so i would like to consult myself with a specialist in Dubai what the reason of itlet me tell you few more things about mei was doing mastubaration before married is it the reason .i ejacuate early within 15minutes the second thing is sometime after ejection my penny take too many hours for the next erection i feel ashamed in front of my wife i want to have a good sexual life i want to become father could you please help me on this matter. thanks in advance,RUMAAN Doctor: Hi,Masturbation for thrice a week is actually good to health. If you're doing more than 3 times, it may cause vitamin deficiency, but does not affect fertility.Hope I have answered your query. Let me know if I can assist you further. Regards, Dr. S. R. Raveendran, Sexologist"
},
{
"id": 217297,
"tgt": "How to treat swollen ankles?",
"src": "Patient: I just returned from a two week visit to Turkey and Greece. My ankles were very swollen after the ten hour flight each way. Also , my legs and knees were stiff and sore during the entire stay. from the constant stair climbing and walking. I am 65 and thought I was in pretty good shape physically. Please offe advice for the next trip. Thank you, Darlene Doctor: Do exercise.wear soft shoe.check ur bp serum urea creatinine fbs ppbs.give me ur drug list.whether u take amlodipine or not.wear compressive stockings if other reports are normal"
},
{
"id": 22738,
"tgt": "Suggest treatment for high cholesterol and tsh levels while having neurofibromatosis1",
"src": "Patient: My son is 14 years old and has been having very high pulse rates and high blood pressure. For that reason he just had a series of blood test where his moncytes count was 1296 and 1+ was the reading for occult blood in the urinalysis. His choloesterol and tsh levels were also high. He has been diagnosed with NF1. I m obviosly concerned. Is there anything we would do before the long wait to get back to the doctor? Doctor: Hello and welcome to ' Ask a Doctor' service.I have reviewed your query and here is my advice. High cholesterol and tsh is certainly not an emergency so you can wait or you get back with reports so that I can advise the dose of thyroxine which he ll need to take. For high cholesterol he should first adopt dietary changes like low oily and low fat, sugary diet. Also regular exercises will help. If it doesn't help he may need tab Atorvastatin depending upon the cholesterol levels.Hope I have answered your query. Let me know if I can assist you further.Regards,Dr. Sagar Makode"
},
{
"id": 176257,
"tgt": "Suggest treatment for fever, forehead pain and pain in both eyes",
"src": "Patient: Hi my 6 year old son has had high temperature today an tonight complaining of forehead pain and pain in both his eyes when moving his eye balls up an down and side to side. I have given him calpol during the day which has helped with bringing his temperature down. He is asleep now. Doctor: Hello. I just read through your question.What you describe happens often when a sinus infection is developing. I recommend fluids and rest. If the symptoms worsen or persist for more than 2-3 days, I recommend consulting with your doctor."
},
{
"id": 112911,
"tgt": "Lower abdominal pain, vomiting sensation. Prescribed pantaprosole, vitamin tablets and some pain killers. Suggest",
"src": "Patient: Hi,Recently i had lifted a gas cylinder which is of 16.9 kgs and walked till my kitchen from store room, which will be around 15 meters distance.One the next day i had slightly felt lower nerve or we can say as lower abdominal pain. I had neglected that one as i dont see much problem with that to me. Next day morning i had done my regular yoga went to office on my bike, and while returning from office to home, i felt that my legs are feeling numbness and started vomiting sesation. Some times i feel like my heart beats gets variation.But stomach is normal, there is no swelling anywhere in my body.I had consulted doctor on 19th april, when i felt very numbness and weakness and pain in lower abdomen. Docter has given me pantaprosole, vitamin tablets and some pain killers. Its has been since 3 days i am taking rest for 3 days.After this 3 days, i still feel like vomiting sensation sometimes and weakness.Just want to know is it okay, or do i need to do any additional checkup Doctor: you should add antiemetic to pantoprazol[ eg PAN -D ] IF IT DOESNT get relived you need USG abdomen you can add muscle relaxant to ge t sprain relif any symptoms of infection like fever , redness , swelling , loose motion , constipation? any pain in back,groin , thoracic region is pain agrivating on exertion , movement , driving vehicle?"
},
{
"id": 205155,
"tgt": "How to handle hostile and confrontational people?",
"src": "Patient: I need some help to understand the problem with one of my family relatives.She is not listening to anyone, do what she wants. Keep sitting calmly and do not speak anything.Ready to fight if anyone is talking about her to anyone else. Looks like a mental conditions or issue in brain. please advise at your earliest convenience Doctor: If u really want to help then you should get her consualtataion done with a psychiatrist viz a viz psychologist"
},
{
"id": 60418,
"tgt": "How much time it will take to recover from fatty liver ?",
"src": "Patient: i am sreedhar male 37 years grade 3 fatty liver ,weight 54 kgs what can be the treatment and how much time it will take to recover Doctor: hi sreedhar, welcome to hcm thanks for the query you can come out of it by regular exercise and strict diet restriction take more amount of fruits and vegetables in the diet. Avoid fried food, processed food, all junk food.,red meat Eat whole grains avoid smoking and alcohol.Keep your cholesterol levels in check take care"
},
{
"id": 129218,
"tgt": "What causes swelling in lower leg?",
"src": "Patient: My lower right leg is swollen,with fluid like feelin.when I press on it the fluid like feelin moves down and you can actually see it and then my skin has my finger indention in it for several minutes. But I can move whatever it is. Went to Er they checked for blood clots and found nothing.can u please help. Doctor: What you describe is pitting edema, and can be caused excessive fluid in your body which you can get with heart failure,or kidney failure, although I would expect it to be on both legs.My advise is to get a proper examination by a physician for general complete health assessment.I hope that answers your question."
},
{
"id": 201088,
"tgt": "What causes soreness on tip of penis glans and foreskin?",
"src": "Patient: I am a 32 year uncircumcised married male. Oflate I have been masturbating about 2- 3 times a week and having safe sex with my wife once a week. But my fore skin and the tip of my glans has been feeling a little sore. I normally have been masturbating 3-5 times a week since I was 14 but never faced this issue. What could this be due to? Doctor: Hi,From history it seems that you might be having fungal infection on the part.This infection might be due to sex through your wife.Apply anti-fungal cream locally.Go for one oral anti-fungal medicine course like Flucanozole 150 mg once in a week for 4 weeks.take the treatment both partners.keep local hygiene clean, dry and airy.Ok and take care."
},
{
"id": 92057,
"tgt": "Is there a relation between left side abdominal pain and blurred vision ?",
"src": "Patient: I think i strained myself lifting something heavy.I now have lower left side abdo pain when i move or cough, have also just started to get blurred vision when i am stationary it looks like im moving and things are getting further away. this also make me feel queasy. Doctor: Hi. Are you suffering from hernia? See if there is a bulge at the area of pain in abdomen. This may not be directly related to the vision problem. Better check with the Ophthalmologist. If Eye exam is normal , you have to check for the medical problems which can cause weakness and the feeling you have."
},
{
"id": 127909,
"tgt": "What causes pain and numbness in the legs?",
"src": "Patient: I have numbness in my legs when I sit or lie down. Have had back checked, EMG of lets, and vascular check. All negative. The doctor doing the EMG said thyroid could do this. I do take armour thyroid and have for about 6 yrs. Blood tests are ok. Could I be taking too much thyroid. Never really had a lot of symptoms except being cold. Doctor: Hello,Leg numbness can be due to loss of potassium. You can follow the below treatment guidelines:- Drinking plenty of fluids to avoid dehydration- Stretching your leg muscles or riding a stationary cycle- Taking diet rich in calcium and potassium or oral supplements of the same- Check your vitamin B12 and Vitamin D3 levels.Till time, avoid lifting weights, you can consult physiotherapist for help. Physiotherapy like ultrasound and interferential therapy will give quick relief.Hope I have answered your query. Let me know if I can assist you further.Regards,Dr. Vaibhav Gandhi"
},
{
"id": 39648,
"tgt": "What causes sore throat after a high fever?",
"src": "Patient: I have many puss pockets in the back of my throat. It is very difficult to swallow and eat, yesterday I had a high fever and the chills but now it is only the sore throat that remains. I chewed tabacco for 5 months then I quit. Do you think it is anything to worry about? Doctor: Hi,Thank you for asking in HCM,You are most likely suffering from a throat infection, responsible for your sore throat and fever both. Fever occurs due to your body's protective mechanism against the infection. You should start with antibiotics immediately and also ward saline gargles. Once you start antibiotics, the infection should subside in 5-6 days. If it doesn't , you may have to undergo furthur investigations. As far as tobacco chewing is concerned, tabacco is a cancer causing substance and the as such depends on a number of factors like the amount you consumed, the duration, the type. But the problem you are describing appears to me most likely as bacterial infection, which can be cured by antibiotics.Hope this solves your query. Feel free to ask for any doubts.Regards,Dr. Sridhar Reddy"
},
{
"id": 95517,
"tgt": "Lower right abdominal pain - what is causing it ?",
"src": "Patient: lrq is a lump pelvic/ abdomen ct came back \"normal\" radiologist said it's in the \"skin\" of the abdomen due to \"trauma\" MRI scheduled but cancelled to rule out bruise, cyst or \"fatty tissue\" cuz insurance won't pay due to ct performed. It's painful, like an internal bug bite. It itches and throbs! Ever hear of this? Thought is was having 2 c-sections so thought nothing of it till pain became worse! Doctor: Hi, Welcome to Healthcaremagic Thanks for your query A lump which has been screened to be in abdominal wall is more likely to be a benign problem. Depending on the etiology it may or may not resolve spontaneously. If it is increasing in size or causing pain it merits a treatment, surgery or medical management. It requires a careful assessment by a surgeon and decide about the management. It may be excised and sent for histopathology. Take care"
},
{
"id": 103663,
"tgt": "Air with mold, foul smell of urine and excreta, rotten food, maggots. Removing smell?",
"src": "Patient: I am a property manager and was in a tenant s house. The tenant has servere mental health and had hosed the inside of her unit. The unit was very moldy, very smelly, with urine and poo, cat litter, rotten food, maggots etc. I was in the property for an hour removing personal papers for the tenant as they have been hospitalized and will be moving into supported accommodation. However I can get the smell out of my nose! Everytime I inhale I can feel the smell Doctor: NEED TO ADD ANTIBITICS LIKE DOXYCYCLINE 100 MG BD FOR 3 WKADD ANTIFUNGAL FORCAN 150 MG ONCE AWK FOR 4 WEEKAPPLY NEOMYCIN H EYE OINTMENT IN NOSENEED T USE MAGGOT KILLING OILIT IS AVAIABLE IN MARKET TO KILL IN NOSE NED TO PUT THAT OIL IN NOSE AS MANY TIMES S YOU CAN"
},
{
"id": 11828,
"tgt": "Hyperpigmentation on the face, turned red and scaly, itchy on using melaglow. Help?",
"src": "Patient: What medication should you suggest for hyper-pigmentation on the face, on a specific area in a female patient of 20 years....(Its not the dark circles , it is on the left upper part of cheek....a dermatologist prescribed melaglow to apply it.....it got very fade after applying it for 2 months but later it got worsened.....it turned red...and scaly....and started itching....so the cream was stopped....now there is no itching but the pigmentation is still present)....please help Doctor: Hi, Welcome to HCM. 'Hyper-pigmentation on upper part of left cheek'-it needs a proper clinical diagnosis before starting treatment. Common cause of this type of spot is melasma. For management of melasma you could apply a sunscreen cream /lotion (e.g Spectraban) at morning and a 4% Hydroquinone preparation (Eldopaque/ Eldoquinn forte) at every night on affected area. You might required laser surgery if your diagnosis would nevus of Ota or other conditions. Therefore i suggest you to visit a dermatologist before starting any treatment. With regards."
},
{
"id": 136,
"tgt": "Are ovulation sticks reliable?",
"src": "Patient: Hi there Just a little advice please. My husband and i have decided to try for another baby. I took my last contraceptive pill on 13th October, bled and had my usual withdrawal bleed. I decided to use Ovulation sticks to work out when i am ovulating as would like to wait for the pill to be out of my system before trying. We had sex on 22nd and i used the ovulation stick that afternoon, it showed positive. Could i be pregnant? Never used the sticks before, are they reliable? i started to bleed lastnight and have had terrible cramps? Sorry little confused, cant remember what happened when i came of the pill last time and conceived our little boy. Many thanks Jan Doctor: Hello,You should stop taking contraceptive pill if you want to get pregnant and ovulation test kits are reliable up to 99 % for detecting ovulation but it detect LH surge. It will give you an idea that ovulation is going to occur in a day or two. You can go for TVS and see whether rupture occurred or not.Hope I have answered your query. Let me know if I can assist you further.Regards,Dr. Sheetal Agarwal"
},
{
"id": 28610,
"tgt": "What causes severe fatigue while suffering from mononucleosis?",
"src": "Patient: My daughter had mono at age 8. She has always been tired. As of late, she has been extremely exhausted. She has been taking antidepressants since she was around 12 for a chemical imbalance. No matter how much sleep she gets she is tired. She was ruled out for sleep apnea. She struggles just to get through the work day. Her dr is sending her to a specialist. Otherwise, she feels fine...any thoughts? Doctor: Hello and Welcome to \u2018Ask A Doctor\u2019 service. I have reviewed your query and here is my advice. Fatigue can be caused of different conditions. I assume your daughter has already had blood tests to rule out anemia (CBC and ERS), thyroid problems (TSH), and basic metabolic profile (ALT, AST, creatinine and electrolytes, lipids fasting blood sugar, etc).If all these tests have been normal than chronic fatigue syndrome can be considered as the cause of the fatigue she is having. Mononucleosis is accused as a possible cause of chronic fatigue syndrome. It is not very clear why it does so but studies have shown that patients with chronic fatigue syndrome test positive for mononucleosis showing that they have had it in the past.Depression can also cause extreme fatigue so an optimal treatment of depression with tablets and phsychotherapy might be needed.Trying to have a sleep hygiene with regular times of going to bed and getting out of bed is advisable. Exposure to daylight is important to help be awake during the day and regulate the sleeping pattern.Being active as much as she can, regular exercise, at least 30 minutes brisk wall three times a day might be helpful.Eating well and regularly is important. Getting involved in activities she enjoys as well.Hope I have answered your query. Let me know if I can assist you further. Kind regards, Antoneta Zotaj, MD"
},
{
"id": 88471,
"tgt": "What causes abdominal pain?",
"src": "Patient: I'm 21 I weigh 120 and Ive been having pains in my upper abdominal just under mg right ride where my stomach is and it hurt this morning to stand up bad to lay back down for a minute it hurts to push where the pain is just started happening over the past 3 days? Doctor: Hello...yoy r suffering from ACUTE GASTRITIS(1) first of all get a ULTRASOUND OF ABDOMAN(2) start this treatment n take it for 1 week\u25cf Tablet Ranitidine twice daily\u25cfCapsule omeprazole once daily before breakfast\u25cf Tablet Dicyclomine three times a day"
},
{
"id": 170710,
"tgt": "What causes fever,respiratory infections and swelling in neck of a child?",
"src": "Patient: my 3yrs old baby is having fever respiratory infections and swelling in neck area doctor said it is an infection in the lymph node and started antibiotics now she is having mild oedema over the face and her urine test shows albumine positive what may be the reason? Doctor: Hi! the swelling in the neck can be caused by swollen lymph nodes. During infection such as that of the respiratory tract, the lymph nodes get activated to fight these infections.They are part of the body's immune response. Bacterial infection of the lymph nodes need antibiotics so you need to continue the medicine prescribed by your doctor.Transient Protein in the urine on the other hand can be caused by fever, dehydration and stress. transient because when you repeat the urinalysis, protein should be negative already. How high is the protein in her urine? However, theres also a disease called nephrotic syndrome which presents with edema noted around the eyes and legs. For this, further work up should be done. I suggest that you go back to your doctor. Hope I was able to help. Thank you for choosing healthcare magicSincerely, Hannalae Dulay-See, M.D.Pediatrician"
},
{
"id": 152056,
"tgt": "What can i use to get rid of shingles ?",
"src": "Patient: I have shingles in my right arm, hand, 3 fingers. Should I continue to try and use or not. Hurts either way - just do not want to do more damage. Should I ice or use heat? Looking for relief!! Doctor: THanks for the query How can u have shingles just on the right hand and some fingers, shingles is a generalised illness affecting the trunk, arms, face, etc. Are u sure about the diagnosis? Apply calamine lotion for relief. Applying heat might leave u with permanent scars Havea healthy living"
},
{
"id": 209870,
"tgt": "Having noticed difficulty in concentrating & low memory power",
"src": "Patient: hi Deshmukh Ji . . Am a Student and am unable to concentrate on words ..whenever in read ..my brain often skips hard-words . It won't accept those .. with all force , wen i read a tuff article, threz a huge pressure , and my memory power is low on things with i dont concentrate much . How to overcome this ..how to link my eyes to brain . am trying with several books ..problem arouses only wen its tuff to understand .. thank q ... siddardha Doctor: DearWe understand your concernsI went through your details. I suggest you not to worry much. You must understand that memory power never goes down for a healthy young individual. But anxiety, low interest level, low self confidence etc can reduce concentration and create memory problem. Active reading is your solution. Effective learning is the key. Reading two hours continuously without understanding anything or understanding 10% of what you read is waste of time and energy. Studying for 30 minutes and understanding 20 minute stuff is real reading.How to achieve this? There are several techniques. Search internet and you shall get many. 1. Be serious and passionate about your studies. Don;t just study to satisfy somebody and to pass exams. Learn for your future.2. Change subjects every half an hour.This is a beginning. More studying (effective learning) techniques are available. You can post a direct question to me in this website. Include every detail as much as you can. I am sure that the techniques should be a success.Hope this answers your query. Available for further clarifications.Good luck."
},
{
"id": 79886,
"tgt": "What is the cause of chest discomfort?",
"src": "Patient: Normal heart beat. Just feeling of pressure being released and going from chest down to fingertips. It is like something is backing up and then suddenly releases. This is happening on a regular basis. Like every 5 minutes or so. Have had problems with myocardial valve in the past. Doctor: Thanks for your question on Health Care Magic. I can understand your concern. Since you are having problem with heart valves in the past, better to rule out cardiac cause first for your symptoms. So get done ecg and 2d echo to rule out heart diseases. If both are normal then no need to worry much for cardiac disease. Vitamin b12 deficiency can cause similar symptoms. So get done vitamin b12 level. And if deficiency then you need supplements. Hope I have solved your query. Wish you good health. Thanks."
},
{
"id": 65682,
"tgt": "What causes swelling on the neck?",
"src": "Patient: Hi, yes, I just notice that im having a hard swelling on the left side of my neck! First I was just sensing a like a pressure in my head, and then I just check now again and feel this hard thing on my neck! How serious can that be? Im feeling like dificulty with breathing as well. Doctor: Hi! thanks for writing to and sharing your health problem in brief!In such cases of hard lump beside the neck with some breathing difficulties, if I were your family physician, I would first examine it carefully regarding its origin, mobility, any more lump in the surroundings and the status of thyroid as well!As per my experience of biopsy from neck lumps, I could suggest you few possibilities as listed below:1. enlarged lymph node: could be just reactive or physiological, due to viral infection, tuberculosis, cancer or even lymphoma etc.2. benign cystic lesions like branchial cyst, thyroid colloid cyst, lymphatic cyst, sebaceous cyst etc.3. inflammatory or tumor-like conditions like organizing abscess, dermatofibroma, neurofibroma or even lipoma.Therefore, you should not neglect this and you must get it to a physician's notice for a definitive diagnosis and further management...Regards,"
},
{
"id": 190648,
"tgt": "Having pain and pus discharge from nasal cavity into oral cavity. Test shows normal. Is it chronic sinusitis?",
"src": "Patient: hello,Iam a dentist .my patient comes complain of pain and bus discharg from nasal cavity into oral cavity ,he said its chronic and recurent ,on examination nothing abnormal just mobile suppernumary teeth opposing to upper premolar and 1st molar.after extraction their is bus ,I wash with salin and give him amoxicilline and mitronidazol .he has no other systemic broblem and he is 22 years old.is it enough and what is diagnosis ,is it may be ca or just from chronic sinsitis Doctor: hi doctor if extraction was a tough one or take a x-ray to ruleout the oroantral fistula any fracture in the sinus wall a simple test is to find when u give a cup of water to hold in the patient mouth u may find some droplets come thro nose and also u get an ent opinion too take the discharge and give for examination to ruleout any lesion thanks dr.karthikeyan virukshaa dental care coimbatore"
},
{
"id": 136150,
"tgt": "Suggest treatment for severe muscle weakness in both legs",
"src": "Patient: I am female, 59 yrs old, have type2 diabetes & high blood pressure. I am having servere muscle weakness in both legs. It start after standing or walking for a minute, I have to sit to keep from falling. My calves also get weak and ihave the ground with my toes to keep from falling. Doctor: I am sorry to hear about your problem. you are suffering from intermittent claudication secondarily to the uncontrolled diabetes. All what you need to do is go for the complete control pf diabetes for which you need advice from diabetologist and the dietician. you should also get a ultrasound doppler of both lower limbs including arterial and venous system to rule out any deep cein thrombosis."
},
{
"id": 171614,
"tgt": "Should I worry fro Alkaline Phosphatase level being 344?",
"src": "Patient: Should I worry about my 4 month old having Alkaline Phosphatase level being 344 u/l, ALT being 112 u/l, AST being 131 u/l, creatinine being .1 mg/dl, BUN/creatinine ratio being 70.0, and Total Protein at 5.9 g/dl.? Baby is having plenty of wet diapers (he is exclusively breastfed). Thanks Doctor: Hi,Thanks and welcome to HCM .Your investigation report suggests that the baby may be having some liver problem.Needs investigation to establish the diagnosis .Consult pediatrician as soon as possible.Hope this reply serves your purpose.Further queries invited .Dr.M.V.Subrahmanyam."
},
{
"id": 66878,
"tgt": "What causes lump in anus and painful excretion?",
"src": "Patient: Hi, I m 21. I ve sensed one tiny-grape-like movable lump internally in the anus some months ago. It caused no problem back then, I took some pills at the advice of my doctor but it worked only temporarily. And now I m having pain down there, esp. at the time of defecation and occasionally with slight bleeding. I m not constipated. I didn t go through any kind of pathological test. What should I do now? Doctor: not to worry much as this is benign condition like an infected and persistent sebaceous cyst or skin tags / fibroepithelial polypif really worries you can go for an FNAC test for confirmationall the best!"
},
{
"id": 10090,
"tgt": "Suggest remedy for hair loss",
"src": "Patient: I was on Trinessa (birth control pill) to stop my profuse bleeding (I was told it was due to hormones) since last year (4/26). I ve recently stopped taking it (last month) because I thought the bleeding would stop (and it did--thank GOD!). However, since taking this pill I ve had major hair loss every time I wash, condition, and comb my hair (I always use my fingers). It usually is about two 50 cent pieces worth of hair. I usually get a shower 2 -3 times a week. I also take 5,000 mg of Biotin and a multi-vitamin every day. Will this help with the hair loss? How long will it take for me to stop seeing lots of hair fall out? Doctor: Hi, I have gone through your complaints and you seem to be having hair loss related to hormonal changes. You can continue with Biotin and multivitamins. Also, apply Minoxidil lotion to the affected areas of the scalp. It will take around 4-5 months for hair loss to settle. Hope I have answered your query. Let me know if I can assist you further. Regards, Dr. Asmeet Kaur Sawhney, Dermatologist"
},
{
"id": 207120,
"tgt": "What is the treatment for depression and chronic tension?",
"src": "Patient: i am sumit khaitan(25years old) the patient of depression and chronic tension type headache since 6 years i was taking amtripytiline 1oo mg daily at night for 6 years last year i had severe nausea so i went for a lft test and it shows the higher sgpt and sgot so doctors stooped the medicine from last year still now have withdrawl effects and i have severe giddiness and weakness i need your opinion as soon as possible plz mention your fees and details if possible i need some consideration. i cn t able to work because of this problems. I am the patient of chronic hepatitis(elevated liver enzymes) since last one year. Before that I was taking amytripytiline 100mgdaily for 6year as per treatment of MHC&H of Bagyam for chronic headache and depression. But last year I had severe nasea so I go for a LFT test it shows that the higher liver enzymes. So as per doctor advise I had stooped the Amytripityline and taking lonzeep md 0.25mg as per need and monitor the sgpt and sgot level. Last(Nov-2010) time you saw me and you had advised to avoid medication and do physical exercise i am trying to do but (severe cough and headache, severe lower back pain and urine pain, sleep disorder, giddiness, nausea, mild pain over liver area )forced me for medication. Now i am taking only cough syrup here doctor fear to treat me after seeing the report. so please suggest some antibiotics and anxiety relief medicines which can I take as per need .Now I am taking C-Stone 2tab at morning but it is also not working. I have done usg 1 months ago it shows now stone size is 4.2mm in left kidney in lower poll . I have lower back pain and lower abdomen pain all the time and problem in urination and frequently urine infection is happened I can not take antibiotic\u2019s frequently for liver . Fever is still continuing. Dr shab I am in very big problem what I can do please suggest me as soon as possible. Doctor: For the treatment of anxiety, you can take escitalopram and clonazepam as and when required for sleep. Kiney stone will pass away through urine with heavy fluid intake. For problem in urination, you can take antibiotic azitgromycin which is liver safe."
},
{
"id": 104059,
"tgt": "Having cough and clogged ear. Had cold. Allergy? What could this be?",
"src": "Patient: Hi, I caught a cold, or I thought it was only a cold, exactly two weeks ago. It knocked me out for two days then i started feeling better. I have been continuing my normal activites and exercise since last Saturday but my throat never stopped hurting although it is not as bad as it was the first few days. I would compare it to allergies. My concern in that I still have a cough and now I have had a clogged ear for two day? What could this be? Doctor: you must hae took medicines for cold decongestants paracetamol and antibiotics cause this or if you neglected again this can happen the mucus fills in sinuses leading to post nasal drip and hence the symptomps writen by you develop pnd iritates the thrat leading to swelling and pain and pnd falling in trachea leads to cough take anti allergic singular 10 mg bd tab fexofenadine 120 mg od syp tossex 1.2 tsf night syp gelusil 1 tsf bd put sea water drops in nose 1 drop bd to clear throat take for 3 week to complete recovery and to prevent recurences"
},
{
"id": 1276,
"tgt": "Can Polycysitic ovaries affect pregnancy?",
"src": "Patient: hi i have politistic overies and do not really overlate that often i had a period at the end of july im worried this past week or so had loose stools felt vey nausea and i have gone off coffee which i love i have done two pregnancy tests which are negative could i still be pregnant Doctor: hello...thanks for trusting the healthcare magic doctors for ur health related queries.I guess ur concern is about...can polycystic ovaries effects pregnancy.well..YES..it effects in getting pregnancy.let's discuss about the polycystic ovarian syndrome( PCOS). in this case patients usually have Polycystic ovary diagnosed with the help of ultrasound.along with this there is usually having problem of irregular menses. some patients having excessive hair in unusual places in the body.few of them having insulin resistance n or thyroid problems etc. this all because of hormonal imbalance which occurs in case of polycystic ovarian syndrome.all this creates problems in getting pregnant.but DON'T WORRY..EVERY PROBLEM HAVE SOLUTION. with the above discussion u can try out my suggestions-1. DON'T WORRY..it can be treated with the help of medications.2. try to do sex during the fertile period(it is from 10th to 16th day of menses during this period chances of getting pregnancy is high).3. if u r in hurry then visit nearby infertility specialist. He/she can help u.I hope the above information is useful, informative and helpful for u.regards-Dr sudha rani panagar( DON'T WORRY..RELAX. I can understand ur tense-full situation. I pray the God that u will be blessed with a cute n healthy baby..hopefully TWINS)"
},
{
"id": 105053,
"tgt": "Developed foot swelling and bruising after treatment for throat infection. On allergen immunotherapy for asthma. Cause?",
"src": "Patient: I have allergic bronchial asthma and am currently undergoing allergen immunotherapy . Recently i contracted a throat infection that caused fever and took a painkiller. The fever subsided but my foot swelled up and i had a bruise . I m 26 years old, 5 6 and weigh 57 kgs. Should i be worried about my random bruised and swelled foot? Doctor: It does`nt appear that the swelling and bruising in your foot is related to your immunotherapy. You may take a course of antibiotics and chymoral forte and keep the foot raised(not hanging down). The swelling will pass off soon. However if swelling does`nt pass off you may have to consult a physician to rule out any DVT."
},
{
"id": 166084,
"tgt": "Suggest treatment for cough & wheezing",
"src": "Patient: Hi, My 4 months old son has had a cough cold and is wheezing since his 2nd lot of injections 2 months ago. I have been to see several doctors who have said he is fine . However his hands and feet turn a dusky purple colour when I lay him flat on his back and he also gets very restless and struggles to breath, this is causing him to have broken sleep at night please help! Doctor: helloi would like to know the detailed history. Do you have family history of asthma or allergy? Another important question is that was he born through C-section? because they are more prone to allergies and recurrent wheezing.i would recommend nebulization with salbutamol in episodes of wheezing (0.2 ml solution with 2ml 0.9% saline) 8 hourly for 2 weeks and give monteleukast sachet 4mg mixed in 1 ounce of luke warm water at night for 3 months . i hope this will help."
},
{
"id": 3820,
"tgt": "What are the chances of pregnancy after having protected sex and delayed periods?",
"src": "Patient: Ma'm plz help me i need some inquiries i had safe sex but my mensus is about 15 days late. Is it pregnancy?i want to check it and i bought the pregnancy test too but i m nt able to check it. what happens if i m pregnant. how can i get an abortion? i m just 19 .i cant bear this. I will never have sex again but this time i need to knw something plz tell me. Doctor: Hi,Don't be stressed. Are your mensus regular?What time in your mensus cycle you had sex? If you had sex in the first and last week of your mensus cycle there is no need to worry. Pregnancy does not happen during this time.What did you use for protective sex? I presume the partner must have used Condom. If it is been used correctly your chances of pregnancy are very less.Do get a pregnancy test done. Postponing things is not a solution. If you are not pregnant ( of which there is a significant possibility) you will be happy. If you are pregnant consult your doctor at the earliest and get medications for medical abortion.i hope I have answerd your questions."
},
{
"id": 165065,
"tgt": "What causes drawing of mouth towards one side in a 10 month old?",
"src": "Patient: my daughter has taken here 10 month old daughter to the hospital emergency room because it appears that her mouth was drawing on one side and she could not keep the pacifier in her mouth like she normally does. she called the peditrician and he said to take her to childrens hospital. i have not heard back yet from them. she everything else seemed normal and this started this afternoon but she is also teething. what might it be Doctor: Dear concerned grandma,your concern is appropriate. But in such cases one cannot say anything without examining the child. Have patience. They will tell you every thing eventually.At times it takes longer to work out the diagnosis. So they must be busy."
},
{
"id": 74772,
"tgt": "Suggest treatment for left sided chest pain and productive cough",
"src": "Patient: Okay. I am 68, Type 2 diabetes, overweight. Have been having a lot of phlegm in my throat in the mornings for a couple of years. Over the last couple of months, I have also developed a productive caugh with upper left chest pain that is noticeable, but not severe and usually is relieved whenever I cough up sufficient phlegm. What gives? Doctor: Hi welcome to the health care magic Your main problem is excess cough... I need to know following details... 1.Do you have fever? 2.are you having chest pain and dyspnea complaint?? 3.smoming history?? It might be allergic cough, chronic infection induce cough, bronchitis, bronchiectasis etc... As far as approach is considered work up done as follow... -Above history asked first -Auscultation -Chest x ray -CBC -if needed spirometry and sputum cytology examination According to cause I can give further comment on treatment Take care Hope your concern solved Consult pulmonologoist for examination and discuss all these with him"
},
{
"id": 145659,
"tgt": "Suggest treatment for cervical spondylosis in neck and migraines",
"src": "Patient: Im 34 yrs old with cervical spondylosis in neck, a disc bulge in neck, carpal tunnel, migraines, and ive had 3 nerve blocks that didn t work .....im scared now because of the pain I have and especially when the baby gets bigger .....tylenol does not help so what do I do ? Doctor: Hello!I understand your concern!If I were your doctor I would recommend you an antidepressant like amytriptiline . It will reduce your migraine frequency and will also help with the neck pain and carpal tunnel problems. This drug must be started after a cardiologist consulting because it can cause heart problems and hypotension. Another drug I would recommend in your situation is Topiramate, an antiepeleptic drug, good for migraine and also for neck pain and carpal tunnel. These drugs are given under medical prescription and should be taken for at least three months.Physiotherapy and neck massage are also helpful for the neck pain, causing reduce of migraine frequency too.Hope to have been helpful!Thank you for using HCM!Best wishes Dr.Abaz"
},
{
"id": 49354,
"tgt": "Suffering from diabetes, developed Pylonephritis and pedal edema. Can I start Cilar10?",
"src": "Patient: My wife is 60yrs old suffering from type2 Diabtes 10yrs.and HTN 5yrs.Recently she developed Pylonephritis Rt.Kidney and with inj Meropenem and Oflox.S.Creatinine reduced to 1.5 from3.6; Her B.P is170/80; with Telma H40 , B.D.She developes pedal edema to CCB. Shall I start Cilar10 instead of Telma 40.H? Doctor: HIThanks for your queryTelma H is not known to cause pedal edema. Cilacar on the other hand is a variant of CCB and has a small chance of producing edema.So I would probably stick to the current prescription.However if there are any problems in the future its best to change medications under the supervision of your treating physician.Hope this helpsGood luck"
},
{
"id": 13855,
"tgt": "Suggest treatment for winter rashes",
"src": "Patient: what i thought was winter rash for the past 6 or 7 winters has lingered these past two years, leaving one seriously itchy scar to remain over the summer with quite a nasty looking scar which itches intensely under physical (exercising) and emotional stress. Doctor: Hi, Your skin might be dry, that is why you may be having itching during winter. Xerosis might be the reason for the itching in winter. You might have developed keloid which is itchy while exercising and emotional stress. Consult the dermatologist for the perfect diagnosis and proper treatment. Hope I have answered your query. Let me know if I can assist you further."
},
{
"id": 156621,
"tgt": "Suggest treatment for low grade asocytoma",
"src": "Patient: hello my father is a patient of low grade asocytoma (tumor) and he is taking tegral and rivotril for 10 years now.he also has been infected by hypitytices c but in the pcr test the virus is not found.he also is anemic with hemoglobin of about 9.7.so in that scenerio what you people suggest him Doctor: A negative PCR means that your father does not have active hepatitis C virus infection.Treatment of choice for Low grade astrocytoma is safe surgical resection. If tumor is located in such eloquent area of brain that it cannot be removed safely then radiation therapy is the treatment.I hope it helps. Please discuss it with your treating oncologist."
},
{
"id": 33241,
"tgt": "Suggest treatment for a bad cold & cough",
"src": "Patient: 5 days ago I came down with what seemed to be a really bad head cold. Started to get more congested in throat chest and nose. I now have a very bad chesty cough, nausea, loss of appetite, headache, fatigue, sore body all over and am tired but cant sleep. Do I need to see my Doctor? Or is it just a common cold? Doctor: HelloCold , chest congestion, sore throat , running nose , all are an indication , may be due to these factors:1 Flu or viral infection .2 ALLERGIC BRONCHITIS, diagnosis can be confirmed by whistling sound , short of breath , mild degree fever ( you are having all these symptoms .3 Asthmatic allergic bronchitis .When such patient visit my clinic I prescribe them to take LEVOFLOXACIN 10 mg / kg body weight once in day ( orally ).Tylenol tablet according to necessity .But these are prescription drugs , so please consult a doctor and get his help."
},
{
"id": 221780,
"tgt": "Can recurring UTI affect the fetus during pregnancy?",
"src": "Patient: hi..im maebil and on my 38th weeks aog,during my 37th weeks my ob realize that i had uti,so i was given antibiotic but recently wen i had my urinalysis again i found out that i still had this uti?will this affect my baby?i am really worried with my condition..am really hoping for ur response..thnk you so much Doctor: HiDr. Purushottam welcomes you to HCM virtual clinic!Thanks for consulting at my virtual clinic. I have carefully gone through your case, and I think I have understood your concern. I will try to address your medical concerns and would suggest you the best of the available treatment options.1]Firstly do not panic.2] UTI recurring during pregnancy in early gestation weeks has its effects on pregnancy like risk of preterm delivery , at 38 weeks now you need not worry about it. If your symptoms have subsided then please do not worry.3] I will suggest to have urine culture and sensitivity test to find out UTI causing bacteria and effective antibiotic for the same.4] Please have plenty of water ,8 to 10 glasses per day.5] Please be sure that UTI is not going to harm your baby.6] May GOD bless you with bundle of JOY.I hope my answer helps you.Thanks.Wish you great health."
},
{
"id": 199283,
"tgt": "Does masturbatipon for years stops growth?",
"src": "Patient: sir i m in bad habit of masturbation .its for years .i belive tat its has cased wrse effect on me.its has stoped me frm growing .i started at age of 12 nw i am 19.pleae help me i am in shadows///////////whenever i think of anything inhave automatic release of sperm.... plse help me Doctor: Hello my dear friendI share your concern, you need not to worry, We are there to help you outExcessive masturbation at an early age is a matter of concern It affects your psychological development and may cause weakness and poor health after excess masturbation.In my opinion you should consult a psychologist for counselling and behavioural therapy if you can't control it on your ownSo concentrate on your studiesPlease don't waste your time thinking and doing about these thingsYou should take good nutritious food to restore your healthExercise regularlyStay cheerful and happyThanks for your questionplease feel free to ask for more assistance,I will be happy to help youBest wishes"
},
{
"id": 90547,
"tgt": "What causes abdominal pain after appendix surgery?",
"src": "Patient: My daughter 15 years of age has undergone an appendix surgery on 31/09/2011, Till date (12/09/2011) she is complaining of pain in right lower abdomen which goes right lower back and then spreads to right leg. Fever of 100 to 101. Vomiting at the time of eating. I contacted the concerned surgeon but he does not find any unusual. Her stool and urine are normal. Can u guess what may be the cause ? Doctor: Hi ! Good evening. I am Dr Shareef answering your query.If I were your doctor, I would have advised her routine/microscopic and culture sensitivity test and an ultrasound abdomen for your daughter.The fever could be due to a silent asymptomatic UTI (urinary tract infection). As she is not married, we possibly could rule out a PID (Pelvic inflammatory disease). Further management would depend on the investigation reports.I hope this information would help you in discussing with your family physician/treating doctor in further management of your problem. Please do not hesitate to ask in case of any further doubts.Thanks for choosing health care magic to clear doubts on your health problems. Wishing you an early recovery. Dr Shareef."
},
{
"id": 83735,
"tgt": "Could ringing in ears be due to side effect of taking oxyelite pro?",
"src": "Patient: I have been taking oxyelite pro for 2 days now, just 1 pill in the morning, and I have been getting a ringing in my ear. Is that normal? I am 27 years old, healthy as a horse, 220 lbs and I eat a very healthy diet and exercise (in the process of losing weight) Doctor: Hello,Ringing in the ears (tinnitus) may be possible with oxylite pro in which recently FDA found fluoxetine drug. Which may cause this side effects but same time you have to check other possibilities like ear infection or allergy or other drug side effects etc. Please stop it and see if no improvement please consult with your ENT specialist he will examine and treat you accordingly.Take care. Hope I have answered your question. Let me know if I can assist you further. Regards, Dr. Penchila Prasad Kandikattu, Internal Medicine Specialist"
},
{
"id": 94679,
"tgt": "Hit by metal rod, lower abdominal injury. Chances of internal bleeding?",
"src": "Patient: My son hit his side, lower abdomen on his camping chair with the medal rod while runing with it. He cried but didnt complain of pain said it hurt a little. I rubbed the area his right lower abdomen on his side. I was wondering if i should take him to the e.r for check up of internal bleeding? I asked google and now Im worried. please help Doctor: Hi thanks for your question your son was hit by a metallic rod on right lower abdomen and didn't complain of any pain. I would suggest to get him examined by a physician and get ultra sound of his abdomen done to rule out internal bleeding.clinical examination and ultra sound will rule out internal bleeding.If internal bleeding is ruled out by these 2 means ,then there is no need to go to E R. Hope this answer your question"
},
{
"id": 141262,
"tgt": "What causes discomfort in the back of the head?",
"src": "Patient: I am 72 and in good health but about two weeks ago I began to get these sensations in back of the head. they don t hurt just a bother and I wonder is this PARESTHESIA and should I worry other wise I feel fine. Everyday I feel as though it will go away but I will have my check up on Monday Nov. 20th and I will mention this to my Dr. thanks Doctor: Hello and thanks for your question here at HCM. I have read your question and understand your concerns. Paresthesia points to symptoms, the probable condition that may cause such issues is occipital neuralgia that is linked to the upper cervical nerves who may be pinched or compressed. Yes, you should mention these issues to your Doctor since you are going for check up. Your Doctor may undertake other steps to diagnose you correctly. Hope I have answered your query. Let me know if I can assist you further."
},
{
"id": 215503,
"tgt": "Suggest treatment for severe persistent pain in legs",
"src": "Patient: My husband has severe pain in his feet and going up his legs. His neurologist at first thought he had neuropathy. He does not have diabetes, which he knows that a lot of people who have neuropathy, and it seems his is different from others pain. He takes neurontin, and another pain killers but really does not help that much. Can you tell me what we can do for him ? Does fibromyalgia share same problems as neuropathy? Thank you. Doctor: Hello, As a first line management you can take analgesics like paracetamol or aceclofenac for pain relief. If symptoms persist, it is better to consult a physician and get evaluated. Hope I have answered your query. Let me know if I can assist you further. Regards, Dr. Shinas Hussain, General & Family Physician"
},
{
"id": 198282,
"tgt": "What medication is suggested for mass on both adrenal glands & white cells in urine?",
"src": "Patient: I recently had a CT Scan and prior blood work. My doc. informed me I have a mass on both of my adrenal glands and I also had white cells in my urine. What could the mass's be and could they be related to the white cells in my urine as a sign of something? Doctor: HelloYour CT findings suggests bilateral adrenal mass.Adrenal mass may be due to many causes like benign or malignant causes.You may need hormonal assay and CT guided biopsy.Commonly adrenal mass is due to adenoma,this is a benign cause.WBC in urine may indicate urinary tract infection.You may need clinical correlation and investigations like routine hemogram,random blood sugar,Urine culture and sensitivity(C/S).You may need antibiotics depending upon C/S report.You should drink plenty of water.Get well soon.Take CareDr.Indu Bhushan"
},
{
"id": 154951,
"tgt": "Does removal of thyroid cancer improve fertility?",
"src": "Patient: Hi. I am twenty five. I have been trying to conceive for almost a year now with no luck. I have just recently been diagnosed with papillary thyroid cancer. They have removed the right side of my thyroid where the cancer was and I am returning for surgery again next week when they will remove the left side of my thyroid. They are then going to start me on 100mg of eltroxcin. I am wondering if my fertitilty issues had anything to do with having thyroid cancer or will my fertility chances improve once I am on eltoxcin? Doctor: Hi,Thanks for writing in.If you are taking chemotherapy and radiation then there is risk of temporary reduced fertility depending on the medications and radiation. However there are methods to take care of your fertility before starting chemotherapy and radiation. This can be done by taking the services of a fertility clinic and getting your eggs removed and stored at cold temperatures in the cryogenic laboratory. When you wish to have babies, the egg can be fertilized with sperm from your partner and then the embryo can be implanted in to your uterus. This technique is offered by many fertility clinics and will work best for you. Please do not worry."
},
{
"id": 55174,
"tgt": "Can cholestasis be diagnosed without taking bile salt levels?",
"src": "Patient: had some blood tests done for my liver as have some red blotches on my chest appear since ive been pregnant. At 32 weeks my ALT levels came back at 55, i had some more blood tests on saturday at they are not 111. I have had a liver scan today, a consultants appointment tomorrow and more bloods on wednesday. They have diagnosed me with Cholestasis. Is this something that can be diagnosed without having the bile salt levels done? I am now 35 weeks and want to push for an induction around 37 - 38 weeks. Doctor: hi.to answer your questions, yes, cholestasis can be diagnosed even without bile salt levels examined. ultrasound is a diagnostic tool with a high sensitivity for diagnosing hepaticopancreaticobiliary tree concerns.. additional diagnostics (such as ct-scan, blood exams) will be recommended as needed. if you're not experiencing any symptom related to it, an elective cholecystectomy (surgical removal of gallbladder) may be done after you give birth. you will be advised accordingly if you consult with a doctor, preferably a general surgeon, re your cholestasis. low fat diet is also advised for now.hope this helps.good day!!~dr.kaye"
},
{
"id": 107470,
"tgt": "Suggest treatment for neck, shoulder and arm pain while having cervical herniated disc",
"src": "Patient: I have just suffered bulging discs in my cervical spine at the c5/6 and c6/7. Having lots of nerve pain through my neck, right shoulder and arm. Cannot sleep. Do you have any suggestions to help. I have just started taking neurotin in the last 24 hours. Not helping so far. Movement feels the best, but they have suggested not using my arm much. I use a manual wheelchair due to a 27 yr old spinal injury at the T12 area. So, need to use it some. Laying or sittjng still causes more shooting pain. I am looking to see if there is something I can do naturally to help me sleep and to lessen the pain. Thank you. Doctor: Hi,I had gone through your question and understand your concerns. In my opinion you may add some muscle relaxants and steroid intramuscular injections and some anticonvulsants that helps you sleep easier with lesser pain attacks. Neck exercises like extension, retraction, shoulder retraction, and isometric hold are highly suggestive in such cases. You may ask your neurosurgeon for further evaluation to recommend you the required dosing according to your condition.Sometimes a surgical solution is better especially after 6 months of treatment by physiotherapy and medications. Spinal surgeons can remove just the protruding portion of the disk. Rarely, however, the entire disk must be removed. In these cases, the vertebrae may need to be fused together with metal hardware to provide spinal stability. Rarely, your surgeon may suggest the implantation of an artificial disk. Please click on THANK YOU and consider a 5 star rating with some positive feedback if the information was helpful. Hope the above information helps you. Any further clarifications feel free to ask.Regards,Dr. Ahmed Aly Hassan"
},
{
"id": 197181,
"tgt": "Is a wart considered to be a symptom of HIV?",
"src": "Patient: i have a small slightly raised bump on the top of my foreskin. It does not hurt or itch. It is not a blister it is not filled with pus. It's more like a swollen pimple but nothing can be squeezed out. Is been there for about a year now. I picked at it when i first noticed it and it did bleed. It formed a scab after that and now it sort of looks like a scar or scar tissue. I was somewhat promiscous before seeing some sex workers and although I tried to be very safe therr were a few times where intercourse did happen without protection. Ive been to a sexual health clinic 4 times now and the diagnose the first 3 times was that it was probably nothing. The last and 4 time the doctor froze off the bump thinking it could be a wart but she wasnt 100% sure. Now after the treatment the scab from the freezing has fallen off but the original bump is still there looking like it first did when i first noticed it. Is it a normal skin bump or hpv Doctor: HelloThanks for query .Based on the facts that you have described the small lump that you have on foreskin since one year is most likely to be a Sebaceous Cyst,However the the dianosis can be confirmed only on clinical examination in person .You need to consult qualified General surgeon for clinical examination .he would be the best judge to decide need for it to get it excised along with sac (in Toto ).This is a simple surgery done as a out patient procedure and offers permanent cure .Please do not worry this is definitely not HIV.Dr.Patil."
},
{
"id": 182348,
"tgt": "Suggest treatment for oral thrush",
"src": "Patient: My son has tonsils and adenoids out on Wens. Yesterday I noticed oral thrush. He's in pain. Called in diflucan but he's got 1 day left on the abx. My concern is that the thrush will increase risk of bleeding when the scabs fall off. I'm a nervous wreck. Doctor: The treatment depends on the underlying cause, age and the severity of the symptoms. If the oral thrush has been caused by certain habits (smoking, pills, wearing dentures), then your doctor will suggest specific changes that would heal the lesions automatically. If the infection persists then antifungal medications are prescribed, which could be in the form of pills, drops, mouth wash or lozenges."
},
{
"id": 158165,
"tgt": "Diagnosed uterine cancer. Prescribed Provera BID. What is this medicine for?",
"src": "Patient: yes i,m a 53 yr. old woman who wad diagnosed in early july of this yr. with uterine cancer. I have had many test done, went to princess margaret hopspital today in toronto ont. they perscribed me 20 mg of Provera....BID Thers was so many diffrent drs. and assistance talking to me i dont even remeber what this medication is for. Doctor: Hello, Thanks for the query to H.C.M. Forum. Medroxyprogesterone is the ingredient of PROVERA . In your case as you were a patient of uterine cancer in July 9 early). This is uses as a palliative treatment in endometrial cancer. Dose is 200- 600 mg. depending upon the nature, type and stage of cancer. So your doctors are going on right path. Take is regularly and remains in contacts with your treating doctors. Good luck. 'Dr. HET"
},
{
"id": 170959,
"tgt": "What causes a freckle above vagina and clitoral swelling in a child?",
"src": "Patient: 4 year old daughter has constant itchy vagina and anus, for almost a year. It s not pinworms or yeast, now clitoris is swollen a lot, has been for some weeks. she also has a freckle just to one side above vaginal opening. what is causing swelling? should we worry about freckle? Doctor: HiConstant itching can be due to local eczema, psoriasis or acrodermatitis. I would recommend you to visit a dermatologist for proper clinical examination and definitive treatment.Take care"
},
{
"id": 85411,
"tgt": "What are the side effects of a flu shot?",
"src": "Patient: I recieved a flu shot about 37 hours ago and started having pain in my arm after about 5 hours. It got progressively worse very quickly and ibuprofen and Aleve did not take care of it at all. I have never been in such pain and I delivered 10 babies! I lost the use of that arm in a couple of hours and the pain is affecting my whole shoulder, arm, neck, head, & collar bone. Is this what I must look forward to when I choose to get a flu shot? Will this pain ever go away completely? Doctor: Hello, The symptoms seem to be related to side effects of the Flu shot. I suggest using anti inflammatory medications such as Acetaminophen to relieve the pain. I also suggest ti apply cold compresses at the area of the injection. The symptoms will go away gradually. Hope my answer was helpful. If you have further queries feel free to contact me again. Regards, Dr. Dorina Gurabardhi, General & Family Physician"
},
{
"id": 220457,
"tgt": "History of ectopic pregnancy. Taking folic acid. Taking Oosure. Can I conceive?",
"src": "Patient: My husband and myself started ttc from Feb 12 and conceived in April 12. Unfortunately it was an ectopic pregnancy and done a laproscopy on June 12. After that my gynac suggested me a rest for 3 months and prescribed me folic acid . But I m yet to conceive. On Jan 12th I got my periods and my gynac suggested me to take Oosure for 30 days, Cervifert for 10 days(started yesterday), and Rizorich for 30 days. Can i expect conception dis month...... Doctor: hello,These are all supportive medicines for healthy pregnancy. Basically pregnancy depends on proper timing of ovulation & unprotected sex. Ovulation occurs approx 14 days before due date.It is better to use ovulation prediction kit (OPK) to predict ovulation time and prepare for that day for unprotected sex for high chance of pregnancy.Avoid stress,take healthy diet with Vit-E & Folic acid supplements,control body wt and proper sleep.Be well."
},
{
"id": 178227,
"tgt": "Can nebulization be given to a child?",
"src": "Patient: I have a 4.7 year old boy. He has wet cough n chest congestion. How safe is it to give him nebulization with these combinations? 1. Asthaline 1 respule + 0.2 ml budecort + 2ml normal saline 2. Duolin 1 respule + 2ml normal saline. Would there be side effects for nebulization? Doctor: Hi, I had gone through your question and understand your concerns.Nebulisation is a form of drug delivery used for direct delivery of medicines in lungs .As drug used is in very small amount and acts directly in lungs ,it has very less side effects as compared to oral medications . Some sensitive persons can have tremors with asthalin and Duolin nebulisation . Both asthalin and Duolin with or without budecort are very much safe in prescribed doses . Contrary to popular belief nebulisation does not lead to habituation .Hope this answers your question. If you have additional questions then please do not hesitate in writing to us.Take care.\u2022 DISCLAIMER: - All the information provided here is for information purpose only , it is not a substitute for the advice of a physician after physical examination , it is not intended to replace your relationship with your doctor. This information in no way establishes a doctor-patient relationship. Before acting anything based on this information do consult your doctor. I recommend that online users seek the advice of a physician who can perform an in-person physical examination"
},
{
"id": 69483,
"tgt": "How to cure a lump on the jaw line?",
"src": "Patient: Hi, today i realised i have a small limp on my jawline! It is not close to my ear and it gets painfull when i press it , its the size of a pea and its under my skin and it just feels like that its become part of the bone! I am currently sick and on antibiotics if that helps ! Would happen to know what that is!!? Thanks Doctor: Hi.Thanks for your query and an elucidate history.This is most probably a lymph node as per your description and location of the lump.You are already under the treatment with antibiotics. May help."
},
{
"id": 20012,
"tgt": "Are Amlodipine and Cozaar a good combination for high BP?",
"src": "Patient: Hi I have high blood pressure I was on inhibace doctor changed it to amiodipine then to amiodipne and accupril and now again to amiodipine 5mg and cozaar 50mg is this a good combination I am have trouble sleeping and these two medications seem to make it worse. Doctor: hello and welcome health care magic forum. Yes , this is a good combination for HTN. it is a combination of calcium channel blocker and ACE INHIBITOR .Regards"
},
{
"id": 57299,
"tgt": "How can hernia be treated?",
"src": "Patient: fatty infiltration of the liver i have been diagnosed via letter after ultrasound with the above disease. i rarely drink but have been three stone overweight. have recently lost a stone and two pounds. i am on the following medication- ventolin seretide and pariet for a hiatus hernia. the letter also said that i had one galstone. i have had 2 minor pain management procedures on nerves in upper back 5months ago twice. and was on painkillers volterol and mobic due to upper back pain caused by kyphosis . hence the pain management procedures. while my diet is not perfect it is definitely not way off in the fat consumption area. i did take lamisil tablets for fungus toe for 8 months last year. and finished them three months before the ultrasound. thanks Doctor: Hi,You need to get treated for the fatty liver at the earliest. A trial of lifestyle modification, dietary modifications and supportive medications for hiatus hernia will suffice.regards"
},
{
"id": 158921,
"tgt": "Stage four bowel cancer, chemo no help. Swollen feet, is it due to tumor or circulation?",
"src": "Patient: Hi my mum has stage four bowel cancer which has spread to the liver and to the peretoneum. Operations weren t an option as the cancer has spread too far, she tried chemo back in November but was no point in continuing as the pain and after effects were too horrific. She is doing really good seeing as her oncologist said she should be dead by now, mum feels it is the supplements that are helping to boost her immune system . Her feet have become quite swollen the last few days, is this from the tumours and part of the body breaking down or is it more circulation and she needs to try and move her feet more? thank you Doctor: Hi and welcoe to HCM. I am sorry that you re confronting with such serious disease,but I am glad that your mum feels good. It is always hard to predict every malignant disease. Swollen legs can be caused by tumor pressure on pelvic veins,and this isnt uncommon. But also you should look for heart and kidney disease because both can cause swollen feet. Whatever it is she should lie with elevated leg,can put some pillow undernetah and do feet exercise. For the beginning blood pressure and echocardiography and basic labs would be good to evaluate kidny and heart function,although those swelling arent gonna hurt her very much if this isnt exreme. Wish you good health and good luck"
},
{
"id": 190742,
"tgt": "Sore gums and mouth, bleeds while brushing, white coat on tongue, swollen lips, fever, head and stomach sore",
"src": "Patient: Hello Doctor My 4 year old son has sore gums which bleed when brushed, and a white coating on his tongue . His lower lip is slightly swollen with a few blisters on the inside. His mouth is so sore that he has eaten or drunk very little today. He had a fever from Saturday night until yesterday, but his temperature is now back to normal and his energy was better today. He mentioned a few times that his head and stomach had been sore, but not today. Thanks Doctor: Hello, as per your description ur son might be suffering from soft tissue infection of mucosa(most common-primary herpetic gingivostomatitis), its common in 3-5yr age group children.. .. give him proper rest.. and regulate the body temperature by giving NSAID- like paracetamol..This disease usually go in 7 days by body immune process only symptomatic treatment is given.. If it persists later then that investigations are performed.. our body defense mechanism is sufficient to cope up with disease if we are in healthy stage.. The pain in lesions might had been causing him difficulty in eating and swallowing.. so use a anaesthetic oral gel 15 mins prior to his eating or drinking schedule let him use antiseptic mouthwashes after eating to prevent secondary bacterial infection in the erosive lesions meanwhile.."
},
{
"id": 26938,
"tgt": "Why is my heart sore and has shooting pains?",
"src": "Patient: Hi im 13 years old and more recently i feel like my heart can be quite sore i don't know what it is and what could be the cause of it , it is like shooting pains is going through it and it is like it is attached to something and it pulls away and it gets quite sore do you know what it could be? Doctor: HelloWelcome to HCMIt is very difficult to understand your case. You have not mentioned site of pain? Right sided pain or left sided pain or central pain? since how many days or hours or minutes your suffering from these symptoms? is there any association with breathlessness, fever, cough? Is there any history of trauma ?Sorry to say but you haven't given proper history.But at the age 13 it is rare possibility of heart related pain. Most common cause of chest pain in pediatric age group is costochondritis ( Inflammation of junction between rib cage and sternum or breastbone) which can be treated with simple anti inflammatory medications.Please take an appointment with your pediatrician or physician for detailed examination with history.Regards"
},
{
"id": 81591,
"tgt": "Having noticed pain in the bottom portion of chest & difficulty in breathing due to an accident",
"src": "Patient: i was in a fourwheler accident two weeks ago and i still have heavy pain in my bottom portion of my chest. i can bend laydown on both right and left sides, but when i lay down on my back i feel like my ribs are pushing down on my lungs making it more difficult to breath, should i get an x-ray to make sure i dont have anything broken? Doctor: HIWell come to HCMI really appreciate your concern, looking to the history given here chances of rib fracture soft tissue inflammation is very likely and to confirm this better to get done the x-ray, if this detects the fracture then nothing much can be done, but of course pain can be managed with any NSAID, hope this information helps, have a nice day."
},
{
"id": 145520,
"tgt": "What causes severe pain in buttocks and numbness in leg?",
"src": "Patient: Hi doc, I have severe pain in my right buttocks and numbness in my left leg right down to my toes. When I walk it feels like a band is tightening around my upper leg and prevents me from walking normally. I had two incidents where my legs just caved in. Tenderness and back pain. Doctor: Hello. I have been through your question and understand your concern.The symptoms you are reporting seem related to a lumbar herniated disc.I recommend a lumabr MRI to rule out this eventulity. Hope this helpseish you the best health"
},
{
"id": 111718,
"tgt": "What is the remedy for piles with back and abdomen pain?",
"src": "Patient: hello doctor, i'm 23 years old. i have one 11 month old baby. now my problem is legs pain when i sleep, white discharge, late period, little bit back and abdomen pain, felling thirsty so much in winter also, most painful problem PILES... i want to know exactly what is my problem? Doctor: Most of your problems like piles is due to pregnancy so this will improved soon. In pregnancy all blood vessels are enlarge so piles becomes more problematic. try to avoid constipation. Take more fibres in diet and syrup cremmafin will help. Also drink plenty of water. For black and late period and white discharge meet your gynecologist , she will give you some drugs to keep in vagina for white discharge.Pain in leg due to lack of iron ,calcium and vitamin B 12 . So supplement all these substance in your diet."
},
{
"id": 61007,
"tgt": "What does a lump on the lower back indicate?",
"src": "Patient: I have a large lump on the lower right side of my back, ive had it for at least a year, I also have severe pain on my right side from the top part of my buttocks that goes all the way down my leg most of the pain is in my lower right side of my back and down and really intense in the middle of my right buttocks, any thoughts? ive had it for a couple years now and its just getting worse. Doctor: Hello,It can be a lipoma, neurofibroma, or else. You need clinical confirmation by a consultant surgeon.Hope I have answered your query. Let me know if I can assist you further.Regards,Dr. Bhagyesh V. Patel"
},
{
"id": 130009,
"tgt": "What causes sharp pains from shoulder blade to armpits, numbness in fingers?",
"src": "Patient: I am 29yrs old. weighing 75kgs. i am experiencing sharp pains in my left shoulder blades to the armpits, and somewhat numbness of the fingers. I have 3 EKG test and each one seemed better than the previous one. kindly advise on what this health problem could be. thank you Doctor: Hello,It sounds like you may have throacic outlet syndrome. In this syndrome the nerve in the brachial plexus in the shoulder get inflamed and can cause shoulder pain, and some arm and hand pain and numbness. See a physical therapist or naturopath that is skilled in treating this condition.Regards"
},
{
"id": 33985,
"tgt": "What causes enlarged spleen while having hereditary spherocytosis?",
"src": "Patient: I have hereditary spherocytosis and I had my gallbladder taken out when I was 11, I am 14 right now. I weigh about 160 and It is 3:00 am in the morning right now and My spleen is enlarged, right now it is hurting, I can not jump or stand because it hurts, I just don't know what is wrong and I am scared, please help? Doctor: DearI really appreciate your concern for the enlarged spleen and its discomfort.Hereditary spherocytosis is due to genetic defect in erythrocyte ( red blood cell) membrane proteins result in erythrocyte instability resulting in production of spherical red blood cells (spherocytes), which are removed rapidly from the circulation by the spleen.Hemolysis (break down of spherocytes) primarily is confined to the spleen resulting in enlarged spleen.If the enlarged spleen is hurting you a lot then visit your treating doctor for possible treatment involving removal of spleen (splenectomy).Hope this advice will help you in seeking the relief from the discomfort caused by the enlarged spleen.Wishing you best of your health."
},
{
"id": 78049,
"tgt": "What causes heat in chest & wooping cough?",
"src": "Patient: hi i am dxt can u pls.help us about desease i am sufferi from last twenty year when ieat medicine it is controled but in summer it is much painful l have experiencedtwo much heat in my chest and wooping cough my face became black during cough. Doctor: Thanks for your question on Health Care Magic. I can understand your concern. Heating and burning in chest with coughing is suggestive of bronchitis (inflammation of airways). So better to consult pulmonologist and get done clinical examination of respiratory system and PFT (Pulmonary Function Test). PFT is must for the diagnosis of bronchitis. And it will also tell you about severity of the disease. And treatment of bronchitis is based on severity only. You may need inhaled bronchodilators and inhaled corticosteroid (ICS). So consult pulmonologist, get done PFT. First diagnose yourself and then start appropriate treatment. Hope I have solved your query. I will be happy to help you further. Wish you good health. Thanks."
},
{
"id": 48545,
"tgt": "Is surgery essential for kidney stones?",
"src": "Patient: i have an 11mm kidney stone in my left kidney . i have no signs of having this stone and didn t even know i had it until my doctor called for a scan recently. it appears to be in the bottom of the kidney and its not moving. my concern is should i have surgery or not .help Doctor: Helloif its silent n not causing any problem then no you dont need to go for surgery moreover 11 mm is a small sized stone n can be get rid off by medicines only"
},
{
"id": 31558,
"tgt": "Suggest treatment for spots on foot and fever",
"src": "Patient: im 52 years old about 2 weeks ago some spots came up ontop my left foot no ich,1 week ago spots moved to right foot and still on left,,i have out ringworm ,creams foot creams hydracodone creams on them,no help still no itch,,just small spots some look like scabing,,now its on my legs and i have been running fever and sick at stomach,went to doc they said not flu,dont know what spots are ,can you help Doctor: Hi thanks for asking question in HCM.Here infection mostly seems to be tinea like fungal infection.Wear clean cotton cloths.Clean affected area with warm water and soap and dry it thoroughly.Especially clean the area between skin folds and between toes.Change slacks daily.Clean towel used.apply clotrimazole containing cream over affected area.Itraconazole tablet can be used.I hope this will mostly help you.Don't worry..."
},
{
"id": 94648,
"tgt": "Having severe abdominal pain, complete loss of feeling in left leg. Had hysterectomy previously. Suggestions?",
"src": "Patient: My mother is in the hospital with severe abdominal pain and complete loss of feeling in her left leg, she s had a hysterectomy but still has her ovaries .... please help .... the Drs in Boise don t have any idea, I d just like to know what they should be testing her for at the nueroligist YYYY@YYYY thank you Dee Dee Doctor: Hi, Unlikely that her pain and loss of feeling are related. However rarely an spine problem can cause abdominal pain also. You should get an opinion of neurologist get test like NCV, EMG done and an MRI of spine done if required. Rarely in bed ridden people with spine problem can have severe constipation causing abdominal pain. Worth while getting an sonography done of abdomen. Hope this answers your query. Please do reply in case of any further questions. Wish her a speedy recovery."
},
{
"id": 115371,
"tgt": "What should be the normal value for TSH levels?",
"src": "Patient: My 4 year old has had his bloods done to day at the hospital doctors have rung this afternoon concerned his TSH levela are very high they have asked me to increase his daily medication to 100 mg then go back to hospital in three weeks are there any liquid forms of Thyroxine to make it easier to give to my child Doctor: Hi, dearI have gone through your question. I can understand your concern.Normal value of TSH is 1 to 5.25. High TSH suggest hypothyroidism. Your son may hvae hypothyroid. He should take external thyroxin according to your doctor's advice. Check his thyroid function test regularly.Hope I have answered your question, if you have any doubts then contact me at bit.ly/Drsanghvihardik, I will be happy to answer you.Thanks for using health care magic.Wish you a very good health."
},
{
"id": 144377,
"tgt": "Suggest remedy for pain caused by obstructive hydrocephalus",
"src": "Patient: My friend is suffering from obstructive hydrocephalus age 21 female Her recent CSF test showed 15.8 !! Headaches are unbearable !! The doctor said to avoid stress and the following medicines are prescribed :- Morning : reblast vozet vertin 16 Brainet Night : vortex vertin16 Brainet Neufil Please suggest me something how can she minimize her pain ? Also she is a working professional.Her doctor said its fine if she is working but to avoid stress is wat he told her. Doctor: hi,it's looks more of understanding how do we balance out human system. certain times medicines only help to slow down the blood pressure or may be to slow down the brain signalling. why don't we try performing relaxation exercise and allow body to grow on its own and give it a time to act as a healer. body is a self medicine in 99% of the cases. we need to boost the immune system and metabolic system by regular exercises and as the body adapts to the process it will heal the later aspect as well.I hope this will help your friend."
},
{
"id": 197403,
"tgt": "What causes semen discharge after passing urine?",
"src": "Patient: Hi,Doctor while passing urine at last some few drops of sperms comes out and its not all the time can i know why is this and how can i stop this . secondly i masturbated after 2 months and i notices that the sperm color was light yellow and why is that so is this okay if not that how can i make it right ? please Advice My age is 26 Sex - Male Doctor: Hi thanks for contacting HCM...You have history of semen discharge in urine ....So you have retrograde ejaculation problem.In this case semen instead of urethra passed into bladder retrogradely ....This is as result of weak spnicter of oddi function.....Following can be cause for that .....-tamsulosin if taking for BPH-few antihypertensive drugs -damage to autonomic nerve supply supplying to bladder -diabetic neuropathy -prostatic opetation...etc...Certain drug like imipramin and chlorphenerimine can tight sphincter but not as effective as surgery....For yellow senen examined it to lab ...If pus cell present infection can be there .Consult surgeon with keeping my answer in mind....Take care"
},
{
"id": 189894,
"tgt": "Have weak adult teeth and pain in the gum. Suggestions?",
"src": "Patient: Hello, I am currently a student applying for dental insurance. It will not be effective for about 3 weeks which is when I plan on making an appointment to see a dentist but one of my adult teeth is falling out now. It must have became weak quite a while ago and I didn t notice until now that when I take bites my tooth has been being chipped away. There s only about half of it left now and as I said it s extremely weak. There s a dull throbbing pain in the gum area surrounding the root of the tooth and in the same exact place on the other side (those teeth are fine as far as I know though...) I m wondering if I should pull it out tonight or go to the hospital or what my other options are....thank you. Doctor: Hi, Thanks for posting your query. Periodontitis or pyorrhea is a set of inflammatory diseases affecting the periodontium , i.e., the tissues that surround and support the teeth . Periodontitis involves progressive loss of the alveolar bone around the teeth, and if left untreated, can lead to the loosening and subsequent loss of teeth. Ideally, a periodontist ie: a gum surgeon must examine you and the treatment may consist of the following: -deep scaling -non surgical placement of drugs (antibiotics) in pockets to prevent further infection and bone loss -flap surgery (in very advanced cases where non surgical intervention does not work) -splint loose teeth (If present) However, until you visit your dentist you may use a good chlorhexidine mouthwash and warm saline gargles twice daily to flush out the stuck food from the pockets thus preventing further infection. Regards."
},
{
"id": 191046,
"tgt": "Can chocolate lead to diabetes ?",
"src": "Patient: As i ve heard that the more sugary things you eat, the more chance you have of getting diabetes and i have a sweet tooth! Doctor: hello, according to your diet if you excercise well and burnout calories you can lead a normal life without getting into diabetes. daily practice of walking and yoga even before getting diabetes can keep you and your chocolate relation together! happy life"
},
{
"id": 139893,
"tgt": "What causes vomiting and concussion?",
"src": "Patient: My 5 year old son has now hit his head for the second time in 8 months. The first time resulted in sleepiness, vomiting and a concussion. We now have some of the same symptoms. Already having trouble staying awake within 15min of hitting head. Should we return to er? Doctor: Hello, Most likely there is no serious cause. If symptoms remain mild and there is no clinical deterioration over the next few hours, there is no need to go to the ER. On the other hand, if there is severe headache, or repeated vomiting, it would be better to go to the ER. Hope I have answered your query. Let me know if I can assist you further. Take care Regards, Dr Sudhir Kumar, Neurologist"
},
{
"id": 18626,
"tgt": "How can hypertension be managed while on Lisinopril and Propranolol?",
"src": "Patient: Hi, I am on 20 mg of lisenopril and 10 mg of propranolol to treat hypertension, but lately my blood pressure readings have been very high.This morning at 8am it was 144 over 90 and now at 7pm it was 144 over 100.Should I take an extra propranolol or lisenopril Doctor: Hello and Welcome to \u2018Ask A Doctor\u2019 service. I have reviewed your query and here is my advice. You should continue with Lisinopril. Propranolol is not a drug of choice for hypertension, although it may lower it. My suggestion is to stop Propranolol and start on Tablet Chlorthalidone once a day. However, if you want to continue Propranolol then change the dose consulting your doctor . Hope I have answered your query. Let me know if I can assist you further."
},
{
"id": 26852,
"tgt": "What causes BP alternating between high and low?",
"src": "Patient: My blood pressure has varied from 167/111 to 80/30 for weeks now Up then down continuously as of late. I keep having chest pains, shortness of breath, dizziness, blurry vision, nausea, extreme fatigue, headaches, muscle cramps, body aches, my urine has changed frequency and smells strongly. My back hurts frequently and I am confused often. Doctor: Hi,It can be due to a stress or an infection. But it depends on how your blood pressure changes and at what hours. I would advise you to keep a blood pressure dairy for a week and show your doctor. In this case your doctor can help you to control blood pressure.I would advise you to have an urinalyses and blood simple to find out whether there is any infection.Wishing you a good healthIn case of further questions don't hesitate to askRegards,"
},
{
"id": 55271,
"tgt": "Suggest treatment for enlarged fatty liver compressing gallbladder, bloating and constipation",
"src": "Patient: My symptoms are : I am suffering from enlarged fatty liver. It is compressing the gall bladder. Stomatch always feels bloated. I have developed constipation. I feel feverish sometimes. It pains on the right side of the ribs in the stomatch. Is there any treatment for such treatment? Can I get cured? Doctor: Hello! Thanks for putting your query in HCM. I am a Gastroenterologist. Fatty liver has many cause like Hepatitis B, C, Metabolic disorders, NAFLD, alcohol intake etc. So I will suggest you to visit a Hepatologist for evaluation who will advise tests and then manage accordingly. Meanwhile there are few medicines which I found very useful in my patients are ursodeoxycholic acid and vitamin E. Do regular exercise and reduce your weight if you are overweight, avoid alcohol if you drink.I hope I have answered your query and this will help you . Wish you a good health"
},
{
"id": 223189,
"tgt": "What are the chances to get pregnant after taking i-pill?",
"src": "Patient: Sir, i was doing sex on 7 december with my gf and she take i pill on 8 december. but on 15 december we again doing sex and again my gf take i pill within hour after sex. means she take i pill two times in amonth then what you say that she will be pragnant or not?? Doctor: Hello,In the current scenario, the chances of conception are very less since she took the emergency contraceptive pill on both the occasions within the stipulated time interval.However, there is an expected delay in the next natural period by a week or two since the hormone milieu is disrupted due to the pill intake. Please see your health care provider for further management if she becomes overdue by ten days.Prefer regular contraception. Avoid using more than two emergency pills in a year and definitely not in a single cycle. Hope I have answered your query. Let me know if I can assist you further.Regards,Dr. Shanti Vennam"
},
{
"id": 39946,
"tgt": "What are the causes and treatment for puss filled bump",
"src": "Patient: I got the bottom of my belly button pierced in June. It was healed but just recently I noticed a hard bump under my skin right next to it. Yellow puss and some blood come out, is this an abscess and is there something I should do while I wait for my next doctors appointment? Doctor: Hi,welcome to healthcare magic forum.Your description points towards an abscess.It needs incision and drainage.Then only,antibiotics can act.If you have a delay for your appointment,you apply osmotic agents like magnesium sulfate paste,and start antibiotics and analgesics(i usually gives amoxycillin+cloxacillin to my patients),gently express out any pus,and keep the area clean.Hope you will find this information useful.Thank you."
},
{
"id": 110864,
"tgt": "Should i be worried about the sore lower back with no bruising?",
"src": "Patient: I fell and hit my lower right back right above my waistline -- there is no bruising (external) but it is very sore. Should I be concerned about internal bleeding? A bruise would show if there was, right? Could I have bruised a rib and see no external sign of it? Doctor: Hello, Thanks for your query.From description , it seems you might developed muscular or ligamentous strain over back .I would suggest getting this evaluated by an orthopedician for an accurate diagnosis and appropriate management. You may get the MRI of the area affected under his/her guidance.Meanwhile following measures will help with the pain\u2022 Lie on a hard bed.\u2022 Anti -inflammatory drugs like Tablet Motrin 1 tablet as and when required \u2022 Avoid forward bending.\u2022 Avoid strenuous activity & lifting heavy weights.\u2022 Apply diclofenac gel on the affected area. Warm compresses will also help.I do hope that you have found something helpful and I will be glad to answer any further query.Take care"
},
{
"id": 131254,
"tgt": "What are the precautions for recovery of broken fibula?",
"src": "Patient: I suffered a broken fibula on Aug. 12,2013. No surgery required and healed well in about eight weeks in an aircast. However since the removal of the air cast I have had continual pain and discomfort on the inside of the ankle. Started physical therapy 1 week ago. Is there more I can do to correct the problem? Is it a type of tendonitis and will it eventually correct it self? I am 70 yrs. young!! Doctor: In my opinion inner ankle pain is not related to your fibula fracture, there is no tendon on the inside of the ankle for tendinitis muscle weekness from long time disuse can cause arthritis you can confirm by preforming ankle pump exercise for 4 min , if pain go away then i am correct , if not then you can suspect L3/4 spodilitisGood LuckGood Luck"
},
{
"id": 57603,
"tgt": "Can sclerosis of liver be caused even after not drinking ?",
"src": "Patient: Hi my name is marian and my mom was diagnosed with sclerosis of the liver the doctor shes going to had told her that her liver is badly scared and dont know how that is my mom is not a drinker or a smoker never did drugs my mom is a diebetic and was just recentally told that her kidneys are bad as well pls id like to know about sclerosis of the liver in how can it be scared we live on the big island of hilo hawaii and here dont have any good doctors pls help me in ant information there is that can be helpful to me in anyway ..thank you Doctor: HIThank for asking to HCMI really appreciate your concern for your mother, the sclerosis is kind of changes of tissues from inflammatory to hardening, and the underlying cause of this tissues changes could be infection (Bacterial , viral , fungus) or genetic, injury, etc. hope this information helps you have nice day."
},
{
"id": 83077,
"tgt": "ITP, dry and brittle nails, excessive hair loss, itchy scalp, lump behind ear. Autoimmune disease?",
"src": "Patient: I ve recently been diagnosed with ITP. I also have very dry brittle nails and have had excessive hair loss for several years now. Lately i have had extremely itchy scalp and Yesterday I discovered a painful lump on the base of my skull behind my ear. Could this be some sort of autoimmune problem like Lupus or even worse a lymphoma? I m concerned. Doctor: Hello, Autoimmune conditions can co-exist with ITP so your symptoms of brittle nails, alopecia would fit in provided at least 2 other criteria were fulfilled to diagnose SLE. There are several others such as discoid rash, malar rash (butterfly rash), serositis, arthritis, blood abnormalities, CNS abnormalities, ANA positive, ACL abs etc so a thorough clinical & lab reports are required before such a diagnosis is done. Swollen lymph nodes (if that is what is the painful lump behind the ear) can be a presenting feature in some SLE patients as well. Thanks."
},
{
"id": 111855,
"tgt": "Should undergoing hip replacement for chronic pain invoke fear?",
"src": "Patient: I am about to undergo total left hip replacement in a couple of weeks. Am 53 year old female and have been having chronic pain for the past 17 years and of course have gotten worse the past two years. I am just a little afraid at this point as it sounds like such an invasive surgery and complicated recovery time. I am worried about any complications Also is there anything I should be worried about? Doctor: Hi,I have read your problem.You should not be worried about complications after total hip replacement. Medical science has been so much advanced that, even if you have systemic disease like hypertension , diabetes etc. no complication occurs after surgery.You should without any hesitation consult your doctor for total hip replacement.I hope you have got your answer.Thanks.Dr.Singh."
},
{
"id": 88368,
"tgt": "What causes cramps in the center of the stomach?",
"src": "Patient: my son has stomach cramps - center of stomach - no fever. He was nauseated for 2 days vomited the first day - held food the second but still complains of stomach cramps. Wants to sleep and lay flat - does not want to get up and walk around because of the pain. Doctor: HI.The symptoms of vomiting , pain in the abdomen and pain on walking is definitely an indication for investigations:I would advise you the following:Take him to the Pediatric Surgeon to get a physical examination done to get a clinical diagnosis , to be supported by ultrasonography, blood and urine tests and stool tests. It may be due to : -Tummy tonsils- meaning enlargement of the lymph nodes of the abdomen.-Appendicitis- by default this has to be ruled out in every case of pain in -abdomen of a child.-Worms.The best diagnosis is a high resolution ultrasonography. some blood, urine and stool tests.A therapeutic trial with a course of an antibiotic, metronidazole and probiotics under the guidance of a pediatrician / Surgeon always helps to solve the problem and to get a diagnosis.Also given is a dose of medicines active against worms in intestine.I hope a consultation with a Pediatric Surgeon, investigations and a course of an antibiotic and metronidazole should solve the problem.If this is indicative of appendicitis , please go for the surgery of appendectomy."
},
{
"id": 214312,
"tgt": "Suggest natural remedy for the removal of gall stone",
"src": "Patient: During an ultrasound scan, it was found that there is a 8mm gallstone. I remember having stomach pain once an year back, do not know if it is because of this. In this case, What could be done? are there are any natural ways to bring this out, would there be any complication if i leave it for now. Doctor: HelloNow diagnosis is established , so you can concentrate solely on this point.Gall stone is usually multiple but in your case it is one 8 mm size , very small . I want to tell you that gall bladder stone is very common , exact reason is not known.Now a days for dissolution of cholesterol-rich-gallstone is used \"URSODEOXYCHOLIC ACID \" orally .Dosage :6-12 mg daily as a single dose at bed time or in the 2-3 divided doses contined for 3-4 months after radiological disappearance of stones.Doses may be divided unequally w/a higher dose given before bed time to counteract increase in biliary cholesterol situation which occurs in early morning .Maximum doses 15 mg /kg.This is a commonly used drugs for gallbladder stone , so you can try .Mild side effects are there but benefits are more , so you can use .Nausea , diarrhea , pruritus , gall stone precipitation are common side effects.Hope this will help you.Other option is surgical removal of gallbladder .But in my opinion take this treatment regularly and also get in ultrasound after every one month to observe the size of gall stone , even size increases after 3 months treatment then get a surgery.Hope this will help you."
},
{
"id": 17166,
"tgt": "Suggest remedy for high BP and headache",
"src": "Patient: I am on high blood pressure medication, I do exercise everyday, I am watching what I eat, I quit smoking for the last 3 months/ which I use to smoke a pack a day / but now I get back to smoke may be 4 cigarettes a day, now my blood pressure goes high and I am having head ace for the last 3 days and holding me neck and head in the back..what should I do..? Doctor: Hi, You first should stop smoking, and monitor your blood pressure for sometimes a day and if the values are high, you should see your doctor to add any other medications. Hope I have answered your query. Let me know if I can assist you further. \u00a0\u00a0\u00a0\u00a0\u00a0 Regards, Dr. Anila Skenderi, General & Family Physician"
},
{
"id": 37362,
"tgt": "Recommend treatment for sinus infections",
"src": "Patient: Hello, I ve experiencing a Sinus infection, with swollen tonsils, earache, and a sore jaw. I also notice that it seems my wisdom teeth are slightly cutting thru. I do not feel any pain in my teeth. My throat is really sore and since I ve been gargling salt water for days, the swelling in my tonsils have gone down a little. Do u think it s just a bad sinus infection or it s all caused because my wisdom teeth might be coming in? Doctor: ello,Welcome to HCM,The symptoms of sinusitis are related to inflammation and congestion of the sinuses with or without an infection.There are many organisms that are causing sinus infection. For your symptoms I would suggest you to follow1. Warm facial compresses applied for 5 to 10 minutes would encourage drainage and reduce pain2.Oral anti inflammatory pain killers such as ibuprofen would reduce the inflammation and help with any facial pain3.Oral anti histamine combined with decongestants like zyrtec d, allegra d will help.Topical decongestants are normal saline or otrivine, both are available over the counter,you can try the otrivine first if it is available.4. Steam inhalation would also help with drainage.Thank you."
},
{
"id": 66578,
"tgt": "What causes lump on wrist with a bruise?",
"src": "Patient: I had a small lump form on my wrist with very sharp pain. After maybe 15 minutes, the pain started to get better slowly. I then noticed a bruise had formed around that spot. As far as I know, I did not hit it on anything. Is this something to be concerned about? Doctor: Hi, thanks for sharing your health concerns with HCM!If I were your treating Doctor for this case of wrist nodule with bruise, I would come up with three possibilities, these include: 1.\u00a0\u00a0\u00a0\u00a0\u00a0some insect bite or small hematoma due to unnoticed trauma2.\u00a0\u00a0\u00a0\u00a0\u00a0a neurofibroma or ganglion cyst with pressure effect \u00a0\u00a0\u00a0\u00a0\u00a03.\u00a0\u00a0\u00a0\u00a0\u00a0folliculitis or some microabscess I suggest you take some ice/cold compress and wait for a while; if not receding then go for an FNAC of the nodule for confirmation and to relieve your concerns!Hope this answers your question. If you have additional questions or follow up questions then please do not hesitate in writing to us. I will be happy to answer your questions. Wishing you good health."
},
{
"id": 136506,
"tgt": "What causes numbness in shin?",
"src": "Patient: I hit my shin on a chair at school and it hurt for a while then went numb. Now 32 and a half hours later my shin is still numb but when I press the area it leaves a dent for a few minutes where I pressed it. On the other shin that s fine I pressed the same area and there s no dent so it obviously shouldn t be there. What s wrong with my shin and what do i do? Help please! Doctor: Hello, I have studied your case. As per your history there can be some differential diagnosis,There can be involvement of lateral fibular [peroneal] nerve which may lead to leg numbness and burning sensation.EMG-NCV [NERVE CONDUCTION STUDY] will help in diagnosis.Another possibility of muscle spasm leading to sudden onset of symptoms.Clinical examination and muscle relaxant will help in early recovery.Physiotherapy like ultrasound and TENS will help.Hope this answers your query. If you have additional questions or follow up queries then please do not hesitate in writing to us. I will be happy to answer your queries. Wishing you good health.Take care"
},
{
"id": 119011,
"tgt": "Anemic, breathing problem, fluttering heart. On iron pills. Advise?",
"src": "Patient: Just wondering. I have been diagnosed a few years ago with being anemic. I haven't taken iron pills for a few years now but all of a sudden feel like my breathing gets wierd. Is this a sign my iron may be low? I think about my breathing too much and I think this interferes with breathing normally. I get heart flutters too. Doctor: Blood hemoglobin levels can give a clue of anemia. With prolonged untreated anemia there can be breathing as well as heart failure symptoms and so it should be treated adequately.If your hemoglobin in in normal range then consult doctor for other reasons for breathlessness"
},
{
"id": 195389,
"tgt": "What causes pain just above the genital area?",
"src": "Patient: For the last 4 months I have had a pain just above the genital area... It started after a football game so didnt think too much of it at the time (I'm 42 so get a few pains after games) .. This pain is quite intense when I cough so thought it might be a hernia but do but see any inflamation Doctor: Hello and Welcome to \u2018Ask A Doctor\u2019 service. I have reviewed your query and here is my advice. Pain above genital area may be due to rectus abdominus muscle pain or spasm or it may be perineal pain syndrome or it may be urethritis or UTI etc. Until examination is done it is difficult to say what it is. Please get ultrasound abdomen and urine examination is done and get treated accordingly. Hope I have answered your query. Let me know if I can assist you further."
},
{
"id": 33157,
"tgt": "What causes large bump on the inner thighs?",
"src": "Patient: Hi, I have a question about a large bump on the inside on my thigh...I'm 19 years old and have had these reoccurring bumps on the inside on my thigh, near my groin area. They always seem to go away. But within the last 6 months, the one I have now has just gotten bigger and more red. It is very tender and feels squishy, almost like it has liquid inside it. It never has a head either. Within the past few days it's gotten very swollen and tender. Last night it popped and there was no puss, only blood. It's smaller now, but still large enough to bother me when I walk. I was looking at it earlier and there still isn't a head, just a large hole where it popped. Also, if I squeeze it slightly, I can feel a harder bump behind it, more so inside in my thigh a little bit, not on the surface. I'm going to the doctor tomorrow morning, I was just wanting an idea of what it could be and what they might do about it so I can prepare myself. Doctor: Hi, The causes for the bump are many.The cause is from infection which is the most common cause. I need more information about your physical cleanliness, you habits (sexual) and the undergarments you wear as all these can influence the bump in your thigh.I suggest you see a doc at the earliest and get treated as the infection can spread further if not appropriately treated."
},
{
"id": 130610,
"tgt": "What causes a diffused large B cell lymphoma?",
"src": "Patient: Shortly after I had massage therapy, where the therapist Squeezed my neck muscles really hard. Perhaps squeezing a lymph-node I was diagnosed with diffuse large B cell lymphoma shortly after Is it possible he ruptured a lymphnode and sent cancerous particles to the rest of my body? Doctor: Hi,In my opinion I don't think the massage therapist ruptured the node them they infected the body but he just touched an inflamed node that caused pain that caused you to go for the check up to find that you had a diagnosis of it is also called Non Hodgkin Lymphoma (NHL). It occurs when the body makes too many abnormal lymphocytes. These abnormal cells don\u2019t die. They continue to grow and divide. This enlarges the lymph nodes. Its main cause is unknown till now researchers are looking for a main cause but it may have some factors that may increase NHL risk include: old age, since most people are aged 60 or older when diagnosed, the use of immunosuppressant drugs, an infection, particularly with HIV, Epstein-Barr virus, or Helicobacter pylori rarely. Exposure to certain chemicals, such as weed and insect killers.Hope the above information helps you. Hope I have answered your query. Let me know if I can assist you further.Regards,Dr. Ahmed Aly Hassan"
},
{
"id": 134663,
"tgt": "What causes swelling in knee and will Ace Proxtvon help?",
"src": "Patient: Thanks you so much Doctor. I am Mrs. Pillai and 57 years old, the problem is my left leg at the Knee are some swelling and while changing direction some pain is there. Accordingly I met Ortho surgen and taken Xray and Blood Test(CCBC, ESR, RBS, RA (QUANTITATIVE). All the reports was o.k. in the range. Xray report is with doctor only, but he said there is no fracture, but in side little scratch there. following medicines prescribed: (spelling error might be there) ACE PROXYVON gastracid liquid -opain -knee cap When I have seen the side effects of the Ace Proxyvon, I am very much tensed eventhough our doctor prescribed, for a 2nd opinion I was searchin on the internet, this is what I want to know... Doctor: hello Mrs pillai.The pain medicines are best taken under cover of an antacid or anti-acidity medicines like pantaprazole.Your medicine proxyvon is indicated for severer pain and may be stopped in 2-3 days or taken sos, it can be replaced with simpler medicines like voveran (Diclofenac), there is no need to change doctor unless there is mistrust, I see no reason at all, rather confide your fears in this doctor -he will explain what to dobest wishes"
},
{
"id": 39595,
"tgt": "Suggest remedy for red, long, infected abdominal scar",
"src": "Patient: I have an old, long, abdominal scar that has recently become red and infected in part of it. When I realized it was infected I began to apply peroxide and neosporin twice daily keeping it covered. The infection has improved but is still there, it is still red, the wound is still open and after six weeks is now not improving. Any suggestions? Doctor: Hi,Welcome to HCM!Sorry to hear about your problem.Since your scar is infected, merely applying the lotion/powder on it won't suffice. You will have to take an oral antibiotic course. The wound may require closure again and hence, I'd suggest you to visit your treating doctor and get it sutured again. Also, it will need proper dressing.Also, I'd like you to get your blood sugar investigation done. A common cause of non-healing wounds and recurrent infection is Diabetes. Hence, it's bettr to rule it out.Hope this information helps. Feel free to ask if you have any doubt.Wishing you a speedy recovery.Warm regards,Dr. Sridhar"
},
{
"id": 51762,
"tgt": "TREATMENT FOR GALLSTONE",
"src": "Patient: PLEASE ADVICE ME IF THERE ANY TREATMENT FOR GALLSTONE EXCEPT SURGERY AS IN THE CASE OF KIDNEY STONES Doctor: Hello.welcome.Medicine is given but it works very slow.Also the stone must be cholesterol stone of less than 20 mm.Dissolution rate of stone is 25 % to 50% after 2 years.Medicine is Ursodeoxycholic acid.Consult your treating doctor for the dose and requirement of the medicine.good luck."
},
{
"id": 22133,
"tgt": "Suggest treatment for a clot in the nerves near the heart?",
"src": "Patient: hi there,i have a question regarding my dad well hes around 50 years old and he has a small clot in his nerves near the heart which the doctor said that no need for any stunt and he has a stomach problem as well.when ever he changes his medicine he doesnt feel good and this time he didnt feel good at all and when he left the medicine he felt better.what is the reason behind this Doctor: hello, Clot or blockages in heart arteries are basically a fat deposits which occurs due to high cholesterol, blood pressure and as a age related change. He might not be having critical blockage and thats why deferred Stenting. He should have a healthy lifestyle like avoiding fatty, oily and high calorie diet. Have low salt diet and monitor blood pressure regularly. Regular exercises like brisk walking, jogging according your capacity atleast 30 min a day and 5 days a week. Lots of green leafy vegetables, fruits, fish once or twice a week, avoid meat. Avoid smoking and alcohol if any. There shouldn't abdominal fat deposition or obesity. Get your lipid profile and sugars tested once.. He should not leave the medicine otherwise blockages may increase and lead to heart attack. These medicine can cause stomach and uncomfortable feeling due to acidity and reflux disease. Does he have upper abdominal pain, nausea, bloating, burping, increase in pain on food, sour water feeling in throat or chest burning, if yes it further supports the diagnosis. You should avoid fatty, oily and high calorie diet. Have some walk after having food instead of taking rest. Have multiple small meals instead of heavy meals. Have regular sleep habits and avoid stress. Lots of green leafy vegetables, fruits. Avoid smoking and alcohol if any. You can get prescribed tab Pantoprazole 40 mg beforebreakfast once a day for 2 weeks."
},
{
"id": 225315,
"tgt": "Cramping, nausea and spotting. Taking loestrin. Pregnancy tests negative. Other reasons?",
"src": "Patient: I am currently on Loestrin 24 FE and a week before my period was to start I started cramping and spotting, first light pink and then brown. I spotted and cramped for 5 days. Now that I am off of my active pills I have been nauseous and experiencing a tightening/ letdown sensation in both breasted but I have taken two at home pregnancy tests and both were negative. I am supposed to come on my periosld today and haven t. Could I be pregnant or am I just reacting negatively to my birth control . Doctor: Hello. Thanks for writing to us. The mild spotting and nausea that you are having is likely to be due to a side effect of the pills. Pregnancy is not likely if you have taken all your pills regularly. I hope this information has been both informative and helpful for you. Regards, Dr. Rakhi Tayal ,drrakhitayal@gmail.com"
},
{
"id": 26762,
"tgt": "What causes high cholesterol?",
"src": "Patient: I was just told I have high cholesterol it is 6 I am not over weight, I do excersice I eat fairly healthy hardly eats sweets or junk food I do have 90% piece of chocolate every day is there other reason that cause this I am 65 and never had problems efore. Doctor: Hello!Thank you for asking on HCM!I understand your concern, and would explain that chocolate intake can contribute in high values of cholesterol.Abnormal cholesterol levels may also be secondary to the following:1-Diabetes2-Liver or kidney disease3-Polycystic ovary syndrome4-Thyroid gland dysfunction5-Drugs that increase LDL cholesterol and decrease HDL cholesterol (progestins, anabolic steroids and corticosteroids).I recommend consulting with the GP and performing some lab tests: (a complete blood count, kidney and liver function tests, thyroid hormone levels, fasting glucose) to exclude these possible causes. I would explain that the total cholesterol plasma level is a weak indicator of cardio-vascular risk. It is more important to perform Total cholesterol/ HDL cholesterol ratio in order to calculate your cardio-vascular risk. I recommend consulting with your GP on the above issues and consider the possibility of starting a statine or just continuing with a diet modification : - avoid chocolate - take a lot of soya food and nuts- take a lot of omega 3 and olive oil, including salmon fish- use a lot of carrots in your diet- Green tea and matcha tea- Flaxseed and Hemp Seeds etc. Hope to have been of help!Best wishes, Dr. Iliri"
},
{
"id": 190986,
"tgt": "I had a tooth pulled and it was complicated. What can i do at home for the remedy ?",
"src": "Patient: On monday I had a tooth pulled and it was complicated. I knew the pain was going to last longer then normal but towards the end of the week it has got worse. I called the office in the middle of the week with no call back yet. I did research and had spoke to some people believe to be a dry socket. I dont smoke and I did everything the way I was told to but like I said the tooth was complicated. Im in extreme pain and evertime air enters my mouth it is even worse. Is there anything I can do for the pain until I get back to the dentist ? He had wrote me a prescription that lasted just for a few days but it made me very sick. Thank you so very much. Doctor: Hi, A dry socket is a complication of extraction of a tooth. It can be managed by irrigating the socket with saline or chlorhexidine and placing a eugenol pack which has to be replaced in successive visits."
},
{
"id": 137039,
"tgt": "Suggest remedy for twitching and pain in wrist",
"src": "Patient: Severe twitching and pain at the back of the wrist. This has been going on for more than a year. I have a operation on my elbow 3weeks ago for numbness in the last 2 fingers. The last finger is still numb and the upper arm 2 inches at the back of the elbow is still painful for more than 2 years. Please do whatever you can. Thank you. Doctor: Hi there.It appears your radial nerve may be compressed or getting irritated in the region of your elbow surgery. You can apply hot water bottle to the painful area around the elbow. Take pain killer and muscle relaxant like Diclofenac with Thiocolchicoside for relief from pain and muscle spasm. Consult your Doctor and he may advise you some arm exercises and other modes of treatment if you do not find relief in this."
},
{
"id": 223941,
"tgt": "Is pregnancy possible despite taking I-Pill?",
"src": "Patient: I had my LMP on 25 Jul4 2014. I had Intercouse on 28 July n took I pill on next day. After 7 days of I pill I had bleeding per vaginally for 3-4 days. But till today means about 2 months completed & I haveno menstral periods. . Please guide me... Doctor: Hello dearUnderstand your concernYes in some case around 5 %, there may be chance of pregnancy as it has some failure rate. Bleeding after 7 days after taking pill is breakthrough bleeding due to high progesterone as a side effects of I pillBut I think chance of pregnancy is less as you had sex just after complete the period and it is safe period.Since 2 months you did not get period, you have to consult the gynecologist to find the cause of delayed period.First do home pregnancy test and or blood HCG and USG to rule out the pregnancy. In absence of pregnancy other causes are hormonal imbalance, ovarian cyst/PCOS, adenomyosis and thyroid disorder should be rule out.In absence of pregnancy, period can be induced by progesterone injections in case of hormonal imbalance.Take healthy diet and fresh fruits, avoid stress and do regular exercise.Hope this may help youContact HCM for additional queryBest regardsDr. Sagar"
},
{
"id": 50436,
"tgt": "CKD stage 5, under dialysis. Espogen 4000 recommended. Advise?",
"src": "Patient: my father was diagnosed with CKD stage V and is under dialysis thrice a week since last 1 month. Doctor recommended Espogen 4000 thrice week. Per dialysis appox 1 ltr water is drained. I have 2 queries in this regard:\\1) What it means to drain 1 ltr water, is it good or bad.2) Do we have to give the injection Espogen whole life???? Doctor: HiThanks for the query.1) Patients on dialysis do not pass adequate volume of urine. Therefore the excess fluids are removed or as you call \"drained\" on dialysis. Thus it is an essential part of dialysis; anywhere from 500ml to 4L of fluid may be removed as per the nephrologists' discretion.2) Espogen is used to increase hemoglobin. Once Hgb is >10gm/dl the dose may be reduced or stopped altogether.Hope this helps.Good luck."
},
{
"id": 12651,
"tgt": "Is it a Psoriasis ?",
"src": "Patient: Hello Sir, My son is having a dried skin and getting cut often and he is having this since three months and since 10 days we are applying t-bact and we felt it got reduce but again it got cut and getting pain. Is it psoriasis? Doctor: Hi! Sriniva M., Welocme to HealthcareMagic forum, In psoriasis one gets silver white scales with skin peeling off associated with itching.Common sites on involvement are elbows ,knees,scalp ,extensor surface of lower leg,lower back.Cuts are rare.It would be good to show to a nearby dermatologist . Take care, Dr.Chawda"
},
{
"id": 1045,
"tgt": "Is pregnancy possible by ejaculation in the mouth?",
"src": "Patient: 1. Can i be pregnant if my boyfriend cumed in my mouth but i spitted it out in a tissue then immediately i kissed him and i transfer some left over sperm in my mouth to him. Then he immediately lick my vagina and spitted in my vagina. Would there be a chance that i would be pregnant? Will the sperm die if the sperm comes in contact with the saliva because it is slightly acidic?2. Can i be pregnant if I blow job my boyfriend and he cleans up the sperm then wash his hands with a sanitizer after 5 minutes he fingered me again. I was wet. Will I be pregnant?3. Can i be pregnant if my boyfriend and I was dry humping and my boyfriend was naked and he was rubing his penis on my shorts i had a pantie but i learned after there was a hole in the shorts and i was so wet is there a chance that i can be pregnant? Doctor: Hello Welcome to HCM. I read all your 3 questions. 1st thing I want to tell you is, you are safe. Do not worry . 1. Regarding your 1st query, after kiss there will be minimal chance of semen or sperms entering into his mouth and then into vagina. A big NO. 2. Since he's used sanitizer, all sperms would be dead and again rare occasion to get pregnant 3. Even third instance, if you are not sure of ejaculation entering vagina then you need not worry All in all, do not worry and wait for your regular periods. Hope this helps you."
},
{
"id": 70345,
"tgt": "Suggest treatment for hard lump on the right side of the penile shaft",
"src": "Patient: Hi, I had a circumcision on the 16th and everything seems to be healing well until I noticed a hard lump start to develop three days ago on the right hand side of the penile shaft. I wash the area daily and have used a cotton wool bud under the head of the glans to prevent smegma build up and to keep the area clean but the smell is quite pungent after about twelve hours, please advise. Doctor: Hi. You need to show this to the operating surgeon just in case of a problem he can diagnose the best and treat accordingly.Anyway, This looks to be a just granuloma due to stitch or a small collection .May need antibiotic. Always use locally liquid povidone iodine ( betadine)."
},
{
"id": 189819,
"tgt": "Bad taste in the mouth, swollen tongue. Taking acyclovir. Need further medication?",
"src": "Patient: My tongue is raw on the side and back. It has been bothering me for at least 2 months. I have been having a bad taste in my mouth. Sometimes feels like it is swolen. Dentist lasered it about a month or more ago, but it is just as bad or worse. He put me on acyclovir but told me to stop it after he put an oral medecine on there once. Doctor: Hi, Thank you for the query. Acyclovir is used to treat viral infections caused by herpes zoster (shingles). The soreness what you are feeling is because of the herpes infection. Viral infection reoccurence rate is very much high . If you were asked to stop the acyclovir it might be because the infection is subsided and topical application of which medicine is not specified .Take vitamin supplements and betadine gargle . I advise you to see your dentist and discuss about the condition . Hope i have answered your query."
},
{
"id": 152332,
"tgt": "I have Nerve Problem like a wave of pinpoint needles moving through out my body",
"src": "Patient: Hello, I have been having a real hard time figuring this out. When ever I go to do any type of physical work, this painful sensation occurs. The only way I know how to describe it, is a wave of pinpoint needles moving through out my body. It is extremely painful and has definitely been getting increasingly worse to the point I am having to stop whatever I am doing. Do you have any idea what this could be? Doctor: It could have Nerve involvement. Have you thought about a evaluation and opinion by a Neurologist!"
},
{
"id": 11337,
"tgt": "Suggest treatment for hair fall in males",
"src": "Patient: Hi my name is Peter. I have suffering from hair fall . around 20 hair strands fall everyday in the morning. I used to use hair gel in till mid of last year and then i stopped. now i apply almond oil every morning after the bath. Apart from the I use to keep my hair wet does it contribute to hair fall..?? My dad is also bald. So i would also have the same hereditary problem but what could be done so as to reduce the hair fall rate.?? Doctor: Hi Peter..It seeme that probably you may be having male pattern baldness like your father. In your case, it may be starting phase..stage I. It is also known as androgenetic alopecia.You consult dermatologist for firm diagnosis. Reduce stress and anxiety related hair loss. If you wish, you may start treatment,otherwise you may wait.Finasteride 1mg daily with biotin containing tab may be suggested by your dermatologist. He might prescribe minoxidil 5% soln to apply on the scalp.You may cleanse the scalp with ketoconazole containing shampoo.Have patience for the good result.Thanks.Dr. Ilyas Patel MD"
},
{
"id": 27803,
"tgt": "Does Betavert and Telma has any side effects on the body?",
"src": "Patient: Hello, I was detected with High BP two years before and I was taking Cardace 5 mg for some time and then took homeopathy medicines. I did not find a relief and stopped taking all the medicines for two months. Now I am having high blood pressure (145 / 100) constantly with mild dizziness. Doctor recommended Telma 20 for BP and Betavert 8 mg for mild dizziness. I do not recollect any signs of vertigo except for very mild dizziness especially after i take my bath. Does these medicines have any side effects. Can I continue these - please advise. Thanks, Venkat R. Doctor: I think the dizziness is due to the blood pressure being on higher side, taking telma should take care of it. Betavert is just for symptomatic relief and you can avoid if no giddiness/dizziness Telmisartan is similar group like care, take same care that you used to take with cardace, very well tolerated drug with minimum side effect However don't stop anti hypertensive drug without yours doctor s advice Regards Dr priyank mody"
},
{
"id": 128959,
"tgt": "How to treat bruise and pain around the knuckles and fingers after an injury?",
"src": "Patient: Hi, I recently hit a wall and I heard a pop, however I am able to move all of my finger with a little pain and some discomfort. My knuckles are tender and of course hurt but the pain has spead to my metacarpals. I have always had a high pain tollerance, yet the bruise has spread past my knuckles. When I bend my finger I also feel a creaking feeling Doctor: Hello,As you are able to move all your fingers without much problem ,there is unlikely to be a fracturesuggest you ice application,analgesics and antiinflammatory drugs and rest for 3 daysHope this helps"
},
{
"id": 66132,
"tgt": "What could be the reason for having a lump below his left ear?",
"src": "Patient: my 5 year old has a lump below his left ear at the end of his hair line - when you touch it lightly it does not hurt - but if you apply a little pressure he says it hurts - we also noticed he has been scratching his head on the center he has a dry scabby spot what does this mean Doctor: Hi,From history it seems that there might be having some infection on scalp giving rise enlarged lymph node in the neck.There might be having infection on scalp like fungal infection, dermatitis or dandruff.Consult your doctor and get examined and then go for treatment accordingly.Give him regular head bath.Ok and take care."
},
{
"id": 163045,
"tgt": "What causes abdomen pain and uncontrollable bowels in a child?",
"src": "Patient: Hello, My granddaughter is almost 8 years old, her sister 7 and her brother 4 years old. She complains of stomach aches during the day school and in the nights, does not feel she has to poop, and her panties are getting dirty. This is about 2 and half months after the birth of her little baby-sister which is now 3 months old. She does not feel the bowel movment. The kids are getting full attention of their parents and even are very spoiled. Can you advise me how to solve this problem. Thank you Doctor: Hi...what you are quoting is exactly called encopresis. This occurs when the kid has hard stools, almost similar to constipation. But what happens here is the liquid stools will seep out on the sides of the hard stools and leak out. But, the main treatment of this is relieving of constipation.Suggestions:1. Natural methods are the best to relieve constipation.2. Constipation is a risk factor for UTI3. Maximum milk consumption per day should not exceed 300-400ml4. Minimum 3-4 cups of fruits and vegetables to be consumed per day5. Toilet training - that is - sitting in Indian type of lavatory daily at the same time will help a lot.Hope my answer was helpful for you. I am happy to help any time. Further clarifications and consultations on Health care magic are welcome. If you do not have any clarifications, you can close the discussion and rate the answer. Wish your kid good health.Dr. Sumanth MBBS., DCH., DNB (Paed).,"
},
{
"id": 153036,
"tgt": "What causes pressure in head in neck while undergoing chemotherapy?",
"src": "Patient: I m going through chemo right now, about a month I felt like the veins in my neck were bulging. Had a scan done to make sure there was not a blood clot from my port causing a blockage, which there was none. This week-end the same thing has happened again what could be causing this pressure in my head and neck. Doctor: hello dear. You have not provided the details of type of illness (cancer) you are having. Also it is very important to know that some chemo drugs can cause such type of sensations. So please let us know. However these symptoms are not serious one and you should not worry at all.regards"
},
{
"id": 76278,
"tgt": "Suggest treatment for chest and arm pain",
"src": "Patient: hi, i am a 18 y/o male about 5 6 in height and way aprox 70 kg i have been getting chest pains all around my chest often radiating to both my arms, chest pain usually around bottom ribs. also when i drink cold water i get a cold sensation in my chest usually lasting aprox 20 min. also have heart palpatations ever 3rd or 4th beat for about 20 to 30 min.. i have no medical problem that i know about. have been to doctor and had blood taken as well as chest x-ray and still waiting results, doctor believes it to be a problem with my cartilage.. however i am concerned and wish to have multipule opinions on the matter. Doctor: Hi thanks for contacting HCM...The pain mainly felt over lower ribs .So it could be inflammation of costal cartilage that is costochondritis ....For that ibuprofen antinflammatory drug taken ..Rest taken. ...Hot pad can be applied....Avoid strenous work...Proper sleep posture.Second possibility could be bronchitis like respi condition for which chest x ray useful ....Third less possibility is cardiac problem.If you have constricting type pain then angina like ischemic cardiac problem ruled out ....Tread mill and ECG useful for it .Take care .Hope your concern solved."
},
{
"id": 12588,
"tgt": "Is salisa kt shampoo effective in curing psoriasis ?",
"src": "Patient: i have psoriasis sir(especially skin peels of like the dadruff in the head regoin and eye brow region)..so i have been adviced to use salisa kt shamppo..is it worth using that? Doctor: Hi...dear sudheendra.., Thanks for choosing HCM.., PSORIASIS..it is autoimmune and papulo- squamous inflammatory disorder...with., genetic predisposition...ok., there is no cure..only control the disorder., first commonly seen in scalp area.., called .....SCALP PSORIASIS., 1) Apply NO SKURF...on scalp throughout ., night ,next morning wash scalp., 2) Apply Salicylic acid 6% and betamethasone.., creamafter 2hrs after bath.., 3) Tab...neotrex 2.5 mg 3 times 12 hrly .., weekly once.., 4) Avoid fatty and nonveg items..ok..thaQ."
},
{
"id": 79019,
"tgt": "What causes pain in the breasts?",
"src": "Patient: hi i had a hysterectomy 9 years ago i am now 56 for the last few months i have been having stabbing pains in both my boobs but only at the sides not all over i had a mammagram and mentioned this and was told nothing was showing up but the pain is getting worse Doctor: It could be related to hormonal imbalance at this age. Try evening primrose oil for local application."
},
{
"id": 51897,
"tgt": "What is the use of folic acid in urinary tract infection ?",
"src": "Patient: what is the use of folic acid in urinary tract infection folic acid in UTI Doctor: Even my doctor had advised my uncle the same. It does prevent urinary tract infections and that is the reason it is advised. You should take it. Also take citrus fruits like orange. lemon which are very helpfu."
},
{
"id": 164921,
"tgt": "What causes diarrhea with rectal bleeding and mucus?",
"src": "Patient: five month old has diarrhea with few specks of blood and it looks like white specks? She had been on elecare due to acid reflux but she was doing so good we tried mixing in some breastmilk so not sure if this has anything to do with it. like a mspi reaction to milk, a virus, she is also teething, or if I may have possibly contaminated the milk in some way with well water? Not sure if that is even a possiblity. She sometimes has mucus too. Doctor: Hi, thanks for your question. from your history it looks like your child has dysentry which can be treated with a course of antibiotics like cefixime. it should improve with 2-3 days."
},
{
"id": 90302,
"tgt": "How to heal sore and red navel leaking fluid with bad smell?",
"src": "Patient: My navel became sore and a little red. Then I found some fluid leaking out with a foul smell. Should I just usw somw salt water solution and clean it and see if gets better? I have never had this happen before. I have been swimming in the ocean last week and in a pool. Could this cause it? Doctor: Hi there,I had gone through your query and understand your concern.Swimmer's Itch is usually common after swimming in lakes/oceans/swimming in contaminated water.Nothing to worry.Kindly consult a Dermatologist at the earliest who can confirm the diagnosis.The treatment is usualy Topical Steroids/Antihistamines /clean the area with antiseptic solutions and Avoid swimming for few day and keep the area dry & clean"
},
{
"id": 3191,
"tgt": "Does earlier abortion, taking birth control pills lower chances of getting pregnant?",
"src": "Patient: i had abortion earlier, & i had birth contril pill for some days. now trying to get conceive, but itz not happening, we partner had intercourse regularly, missed in between some days. I was expecting this month but my bad luck dint happen. Dint know what is the reason. I wash myself immediately after the sex is that fine. or this is also one of the reason. I need someone to suggest me. My hubby may not accompany with me to go With Dr. immediately. Plz help. Doctor: Hello, reason for earlier abortion if treated then it will not affect next pregnancy. Some medical conditions cause recurrent abortion like antiphospholipid syndrome. In these conditions pregnancy can be planned with proper prophylactic medications. So the reason for abortion is important. If it was habitual or accidental then take proper care during pregnancy. OCPs are hormone based contraceptives which when taken affects fertility and body hormone levels. Fertility returns in 3-12months depending upon the duration of usage of ocps."
},
{
"id": 46323,
"tgt": "Is CT scan needed to see if there is cyst on kidney?",
"src": "Patient: I did a Lumbar CT scan due to a herniated L4/L5 disc. The CT scan showed a possible cyst on my left kidney. I then did an ultrasound and they were unable to detect anything, now they are recommending a CT scan specifically for the kidney, is this necessary or was the ultrasound confirmation enough that no problem exists? Doctor: Good day and thank you for being with health care magic!!! If the cyst is small and it is characterized as a simple cyst then I don't think further CT SCAN is needed for a simple cyst. I hope I have answered your question properly."
},
{
"id": 23788,
"tgt": "Can high BP be the cause of nausea?",
"src": "Patient: I am 57, 5'2\", 138 lbs. recently had very high blood pressure ,at peak 170/102, found what appears to be a lump in middle of upper stomach, below chest bone, painful when pressed. Never had BP that high, and BP went down in few hours. Already on BP meds for BP and was controlled, little nauseated some days. Doctor: Hello!Thank you for asking on HCM!Regarding your concern, I would recommend performing further tests to investigate for secondary causes of high blood pressure: - complete blood count- inflammation tests (PCR, sedimentation rate)- thyroid hormone levels for possible thyroid dysfunction- kidney and liver function tests - blood electrolytes- fasting glucose and HbA1C for diabetes- a chest X ray study for your lungs- an abdominal ultrasound to examine also the lumg in the middle of upper stomach. A CT scan may be needed. The charachteristics of the lump that you describe seem to be related to inflammation. If all the above tests result normal, you should consider making some changes in your antihypertensive therapy (as your actual blood pressure values are really high) in order to have a better control of your blood pressure. You should discuss with your doctor on the above issues. Hope to have been helpful!Kind regards, Dr. Iliri"
},
{
"id": 160296,
"tgt": "Shooting pain in left breast and under left arm pit",
"src": "Patient: I have a history of female problems within my family. I am always scared to go to the Doctor. I noticed a lump in my left breast at the top of my breast. I ignored it because it only showed up when i was premenstral and went away after my period. Now it is there all of the time and it is painful. The lump itself hurts to the touch and sometimes it sends shooting pains through my breast and under my left arm. i am 31 and have had a history of female problems. I know i have waited to long,My Doctor appt. is in 2 days but i first noticed the lump 2yrs ago. I'm in trouble aren't I? Doctor: Visit MEDANTA\"THE MEDICITY\" A super Speciality Hospital In sec-38 gurgaon where U can get Consultation with Dr ashok vaid(world renowned oncologist And Dr tejinder kataria 9world Renowned breast cancer surgeon )........or visit medanta.org or call for assistance 09419191439"
},
{
"id": 201620,
"tgt": "Suggest treatment for lower abdominal and testicle pain",
"src": "Patient: Hi I m a 22 year old male, I ve had testicle pain for the past 2 weeks. It s a dull ache on my right testicle and sometimes I alternates to the left testicle. I ve also had lower abdominal pain. My doctor checked for lumps or abnormalities but there was nothing, the pain is often worse when touching my testicle. The pain seems to be coming from the top and the bottom of my testicle. My doctor told me that my testicles were tender and that it may be an infection, he put me on antibiotics, clavam 625. After a week or antibiotics the pain is still there and now the tip of my penis burns too, no burning while urinating, just general burning sensation at random times. Any thoughts? Doctor: Is it a pain or a dragging sensation / heaviness ? at your age this pain could be due to Infection of testis(orghitis), varicocoele ... dilated veins above the testis which u can feel as multiple chords, injury .. due to tight underwears, a small injury leading to hematoma and pain.. It can all be settled with a ultrasound scan of the testis .... so get back with the reports ? It is not a serious problem, but surely needs appropriate treatment. Did u have any fever or a swelling around your ears ? sometimes mumps can lead to a problem like this !! ("
},
{
"id": 175129,
"tgt": "How to treat worms in kids?",
"src": "Patient: my sons dog has worms we just found out and treated the dog. she is about 8 months old and still messes sometimes in the house we clean it and steam clean once a week with bleach but i am scared my 10 month old is gonna get worms now how can i treat my baby Doctor: Hello dear,Thank you for your contact in Helthcare. I had and understand your concern.Dog has lots worms, child crawls everywhere ,so probably baby have been infected, that's why i propose to give him Zentel(albendazole)-syrup-an antiprotozoal and anthelmintic agent. He acts on intestinal and tissue forms of the parasite , the active to eggs,larvae and adult worms.We used zentel for helminthiasis :enterobioza, ascariasis(dog's worms),giadiasis in children and etc. How to use:It is assumed Zentel suspension with food. Children up to 2 years once 5 ml orally in bed time. If clinical improvement after 3 weeks dosn't occur , a second course of treatment. In addition, cut nails of baby shortly, don't allow to dog go inside child's room, wash hands and toys of baby with soup. Grow quicklier Dr.Svetlana Shrivastva"
},
{
"id": 44203,
"tgt": "ICSI procedure, embryo transfer, spotting, period like pain. Urine test done, shows hcg levels. Pregnant?",
"src": "Patient: Hi we have undergone icsi procedure for the first time. 3 embryoes were transferred. on day 13 after transfer our doctors ask my wife to go for urine test . the hcg level was 115. on the same day my wife had only 2-3 spots of blood and after two days she had little more drops but pain as in periods. can u please guide whether she is pregnant or not Doctor: Hi, Your wife is pregnant, considering B-HCG being 115. But i would suggest you to meet your doctor for repeat B-HCG test and to consider stepping up her medications so that maximum support for pregnancy can be given.Spotting is common problem in early pregnancy, especially after IVF. Close follow up is required in your case to see the increasing titre of B-HCG for healthy pregnancy. If B-HCG is falling then it can be a biochemical pregnancy. Regards."
},
{
"id": 145751,
"tgt": "What causes cerebrospinal fluid leak after treatment of pituitary adenoma?",
"src": "Patient: Hi, I am a sufferer of giant pituitary adenoma. After the treatment, I am suffering from CSF leak problems. In six months, I ve done CSF leak repairing surgery twice, but have yet to heal. Here is my situation: Oct.2011-- Water broken during pregnancy. After about 3 weeks, right eyelid ptosis and double vision. Nov.2011\u2014The doctor has done the surgery on me in trillium hospital Mississauga. The surgery was successfully, a part of the tumor was removed. My eyelid and vision returned to normal. Mar.2013 --MRI found the tumor grew again. I were taking radiation treatment 25 times at Princess Margaret hospital during April to May. After treatment, tumor has shrunk small. Mar.2014\u2014I suffered from CSF leak. The Doctor has done the craniotomy surgery on me. But unfortunately it was not successful. On July, the doctor has done another surgery on me through right nose hole. After the surgery, I was feeling good. But now I found it is leaking again. I really do not know who could help me. I would really appreciate if you could give me some advices or recommend someone who is good at CSF leak repairing in Ontario. Thank you! Jenny Song Doctor: Hi,Thanks for writing in.The pituitary gland is located in the piuitary fossa region or sella which is located in the midline above and behind the eyes. Sugical approach to tumors of pituitary can involve through the skull as in craniotomy or through the nose. The chances of CSF leak are higher in the surgical approach through nose. During surgery for pituitary adenoma, the tumor is approached though the roof of the nose to reach the tumor. This is called trans sphenoidal surgerydone through the nose. Unavoidably, the surgery involves making a small aperture which connects the brain to the roof of nose. This is then sealed and should heal by itself in most patients. In a small group of people, this communication does not heal completely and might persist as a route through which CSF leak occurs.Treatment of a CSF leak is done by surgical repair or in some cases through use of temporary diversion of CSF through a lumbar drain or a ventriculostomy (a catheter placed in the ventricle of the brain). Please discuss these options with your neurosurgeon."
},
{
"id": 33864,
"tgt": "How to treat the infected area on the side of the feet?",
"src": "Patient: My husband has an infection on the side of his foot x's 1 week that has increased in size from the original size of approximately 1\" (blisters, redness and red dots around the initial site). Over the course of 5 days the blisters have broken and the infection site has enlarged and now the foot is swelling along with redness spreading away from the site. His foot is quite painful when walking and must use a cane since yesterday to avoid putting full weight on the foot. I've been cleaning the site with peroxide and antibiotic creams and sometimes with Aveno anti-itch cream. We first thought this was poison ivy as we do have this plant around our home, but this looks different and I now believe he may have a staph infection and needs to be on antibiotics. Please give us your opinion Doctor: Hi,from history it seems that there might be having acute skin infection with cellulitis producing oozzing, swelling and severe pain.He might require one course of antibiotic medicine like Amoxicilin, Clauvinic acid combination course for 5-7 days.It is advisable to do dressing by your doctor only.Rule out diabetes as well.Advice him to raise his affected leg.Ok and take care."
},
{
"id": 139877,
"tgt": "Should antidepressants be stopped while treating mesial temporal sclerosis?",
"src": "Patient: I ve been having insomnia and consequest migraine headaches for the past 8 years. The very first neurologist i met had prescribed me an MRI which concluded that everything was normal. For the past 8 years i ve been on and off neuromedicines, without any permanent relief. i ve been visiting another neurologist for the past 2 years who had prescribed me almost all kinds of Benzodiazepine family medicines over different periods of time. he had to keep rotating the meds due to me developing immunity from the particular BZD medicine. a week ago he asked me to undergo MRI scan once more, which concluded that i have MESIAL TEMPORAL SCLEROSIS on both sides (R L). thus my doctor asked me to spend 3 days at the hospital during which he had given me intra-venous medicines. 4 days ago, he asked me to stop all anti-depressants (BZD) and has prescribed me Rovastat TG (Rosuvastatin and Fenofibrate), Citistar (Citicoline 500mg) to be taken 3 times a day, as well as Dayo OD (Divalproex 250mg) taken once every night. Since i am offthe anti-depressants, i have been feeling extremely depressed for the past 3 days. I would like to know if the line of treatment is correct, and what are my medical scopes in future that i should be aware of? Doctor: Hello, Usually, patients with mesial temporal sclerosis usually have seizures as their main symptom. as advised by your doctor it's better to stop bzd its role in depression in very minimal or none. The next thing is Divalproex can helpful for your migraine headache and also it has mood-stabilizing property, it's another effect is it can cause sleepiness and it has anti-seizure activity also. Some of migraine prophylactic drugs can cause worsening of headaches like Propranolol. It's better to wait for two to three weeks for your new medication to act if depression is more bothering you newer antidepressant medication is available which a psychiatrist will prescribe. If you have abnormal lipids rosuvastatin and fenofibrate are useful or else it can be stopped. The rest can be continued. Hope I have answered your query. Let me know if I can assist you further. Take care Regards, Dr Sateesh Gudla, General & Family Physician"
},
{
"id": 70215,
"tgt": "What causes lumps between the eye and nose and swelling in eyes and face?",
"src": "Patient: I have a several bumps come up between my eye and nose and several more popped up towards my eye brow. My eye is swollen and woke up this morning my left side of my face swollen. This started Thursday and seems to be getting worse. Ive put antibiotic cream on the bumps that have opened up and warm compress. What is this? Doctor: Hi ! Good morning. I am Dr Shareef answering your query.From your history, it seems you have contacted some kind of infection of the skin which could either be a viral infection (herpes) super added by bacterial infection. For a definitive diagnosis, and proper treatment I would suggest you to consult a Dermatologist/general surgeon in your area soon so that further spread and complication could be avoided.I hope this information would help you in discussing with your family physician/treating doctor in further management of your problem. Please do not hesitate to ask in case of any further doubts.Thanks for choosing health care magic to clear doubts on your health problems. Wishing you an early recovery. Dr Shareef."
},
{
"id": 60686,
"tgt": "Am i having mild anemia & if so what is the treatment for it ?",
"src": "Patient: I am a 29 years old mother. I am having the following health concerns for the past one month. Muscle ache, mild itching, and sore tongue and i feel too tired. I do not have any other problem in my body. I feel that am i having mild anemia? I took swine flu vaccine two months back. The tiredness persists for two month and got increased for the past two week. I think it may be the reason of vaccine side effect. Because my mother had swine flu vaccine and due to its side effect, she is in treatment now at Apollo hospital with ana positive + jaundice history. Can you pl advise is it because of H1N1 flu vaccine side effect, if so, what is the treatment. Doctor: Muscle ache and weakness are known side-effects of the vaccine. Anemia can be due to various reasons though. Usually such sideeffects go away with time, but its been 2 months now. I would suggest you see your family doctor and get yourself examined and tested first. Alternative therapy , like Homoeopathy can help you with these symptoms."
},
{
"id": 35336,
"tgt": "How to get rid of bumps below chin while suffering from tuberculosis?",
"src": "Patient: 30, 177 cm, 68 kgs, i am suffering from TB told by my doctor. There is two bumps (with swelling) just below chin. Done biopsy needle last 10 days and given tablets like R CIN, RANIDOM, CIBA, COMBUNEX. I have been using for last ten days but there is no cure in it what i have to do next. Please help on this issue. Doctor: hi. Welcome to HCM.If those swellings are because of TB then these will subside over time. You will have to take anti-tuberculosis treatment for 6 month if you had not taken previously. New cases are initially treated with four drugs: isoniazid, rifampin, pyrazinamide, and either ethambutol or streptomycin. After 2 months, they are then treated with a continuation phase of 4 months with isoniazid and rifampin.Generally tuberculosis infection takes time to respond.Take care."
},
{
"id": 202994,
"tgt": "How can i stop my sperm leakage?",
"src": "Patient: Please tell me that how can I stop my sperm leakage?When I am asleep my sperm comes out automatically, when I see porn movies nd my penis erects automatically my sperm comes out.It has become a big problem for me.Please help me out with this.I cannot tell this to anyone not even my parents.Please suggest me some medicine or any way to get rid of this. Doctor: HIThank for asking to HCMI really appreciate your concern for this, but this is very common condition among the youngsters and this is not the disease this is physiological phenomenon which does not need any medicine, just try to control your thoughts, and stop worrying about this, hope this information helps you, take care and have a good day."
},
{
"id": 213677,
"tgt": "What can be done for my dad's behaviour ?",
"src": "Patient: hello doctor i want to know about my father, he is 65, 4 yrs bck my mother expired on the 2nd day he wanted to marry again , he would ask each and every woman widow or divorcee to marry him or have a livein relation for this he threw me out of the house and married a wrong woman she stayed with him for a year and left meanwhile he called me back, now again he is up for the same thing wants me out of the house he is asking every woman without knowing her detales for a livein relation no matter of what age for this he wants me out again this guy s mentality is as such that no good woman can stay for long with him , he is ready to marry a thousand times. he would ask the maidservant to marry him ,her daughter also, as a matter of fact he even forced me to call my mother in law ,a widow. i want to know is this normal or he is having some psychiatric problem please help Doctor: Dear This does not seem to have normal behavior. Need to check wherethere it is a hyper- sexual behavior? Check for othe symptoms, i.e. sleep distubances, spending excessive, talks more, increase religiosity or jocularity, But best way is to meet psychiatrist for his detailed evaluation."
},
{
"id": 51348,
"tgt": "Need kidney transplant operation. Abnormal creatinine level. What can we do?",
"src": "Patient: Hello, My name is Rahul, am from Hyderabad, INDIA. My sister (29 years) is suffering from kidney problem. Her serem creatinine level is 9.9mg/dl for now and doctors here say that kidney transplant operation has to be done as soon as possible. But getting the source for kidney is never easy. My entire family is worried about this. I request you to suggest some thing that can help us to save her. I hope you will definitely come up with some positive way to get out of this problem. Please Help! Regards, Rahul Doctor: hi, as i dont know what kidney problem is your sister suffering from it is difficult to say whether a kidney transplant is going to solve her problem.kidney or renal transplant has its own problems and its not easy to say that is the best of solutions as it itself involves a large amount of expense plus its own known side effects which are very common llike a person has to stay on immuno-suppressive drugs, highly toxic drugs whole of his life.he/she has to stay a restricted life preventing hiself/herself from even a small infection.prerequisites for renal transplant are too many.not every time a transplant is successful and how much does it affect ones quality of life is again a question mark.my sincere advice would be take a good nephrologist's opinion, investigate thouroughly for the cause of renal damage as not few diseases relapse even post transplantation. all the best.hope i have helped you in some way."
},
{
"id": 225605,
"tgt": "Taken I-pill. What are the side effects? Can we take it after a gap?",
"src": "Patient: Hi, Dr. i am 24 year old girl, i have taken i-pill last month but my periods had nt came up yet. My basic query is what are the measure side effects of using i-pill. i have taken around 5-6 i-pill after six month gaps is it harmful to me. tell me that will i get pragnant in future or not. no problems should occur in future kindly ensure. Doctor: Hello. Thanks for writing to us. After taking an I-pill, it is common to have a delay in the periods due to the hormonal effects of the tablet. It is best not to take this medicine repeatedly and try a regular contraceptive instead.I hope this information has been both informative and helpful for you. Regards, Dr. Rakhi Tayal ,drrakhitayal@gmail.com"
},
{
"id": 177634,
"tgt": "How long should Asthalin expectorant and Mox 250 be continued?",
"src": "Patient: Sir, my 5 and a half year old son had cold, fever cough and congestion last week, now his fever and cold is almost finished but congestion remains. We consulted our local pediatrician that he recommended us to give him Asthaline Expectorant and Mox 250, so as a second opinion I would like to know how much more time I can continue with these two medicines, Plus he seems a little weak and less-hunger after these. Thank you. Doctor: Hi...greetings from Chennai. if there is no wheeze and if the kid is not coughing severely and his sleep is not getting disturbed, I do not suggest any medicines usually in my practice. That too when the fever and cough are settling, there is no need to use an antibiotic like Mox250. I think he is recovering from a viral illness and suggest you to stop all drugs and he will get better by himself.Regards - Dr. Sumanth"
},
{
"id": 110365,
"tgt": "What causes sharp pain on the mid section of back?",
"src": "Patient: I have a sharp pain on the mid section of back on the right side. I have also noticed that I have been unirating a lot more than usual. The back pain has been coming and going for a couple months now. Any ideas on what it could be/ if anything at all? And how it could be treated Doctor: Hi Welcome to healthcare magic.By reading your query it seems to be simple backache. Treatment of simple backache is analgesics(diclofenac 50mg three times a day after meals), after pain relief regular back exercises so that pain don't reoccur. You should also your VITAMIN D3 done. If there is deficiency than it should be supplemented."
},
{
"id": 64076,
"tgt": "Suggest remedy for recurring lump with pus on knee",
"src": "Patient: I had a injury on my knee some 12 years when I fell and I had 12 stitches. The wound healed up fine, but since then I had sore knee over the years. Just few months ago I had this boil kinda lump show up which had pus in it, the wound healed up but then it keeps coming back the lump with the pus in it, it just would not heal. The infection dries up but comes back on the same spot on the old stitch mark Doctor: Hi,Dear,Thanks for the query to My HCM Clinic.I studied your query in details and understood your health concerns.-Treatment and Cause of your lump on knee-a-Mostly it is-a retained/burried- stitch thread-which is causing Chronic Abscess with soreness and pus discharge.b-Other Cause of ?Osteomyelitis of the knee bones below the infected but healed stitched scar wound,needs to be ruled out by Xry KNEE with nearby wound area.c-I would advise-to consult ER Surgeon-who would remove the -burried stitch thread.ER orthopaedic surgeon would treat it if there is Xry knee lead to treat Osteomyelitis of knee bones.This advise would help you to plan treatment with your doctor.Hope this resolves your query.Wellcome for further query in this regard. Have a Good Day...!!With Regards"
},
{
"id": 86365,
"tgt": "Suggest treatment for pain and tenderness in the lower abdomen",
"src": "Patient: My lower abdomen has been tender for around 2 maybe 3 weeks now. I haven t felt myself. I ve been feeling sick but not been sick, I get hot and cold flushes along with aches and pains. I have recently been waking up with a stuffy nose sometimes with blood, and the same at night. My period is due today and I went to the toilet before bed and noticed I have one blob of like brown jelly discharge. What could this be? Doctor: A kind of mild indigestion due to various reasons but tenderness is the term you have to understand. If you press or touch it, it would be paining. It could be infection in intestines, mesentery, uterine infection etc. Its better to be evaluated clinically for error less comments.Brown jelly is some kind of not so normal blood of m.c. With some simple herbal preparations such as Sy Evecare or Ashokarishta Liquid for 4 weeks, it can be addressed."
},
{
"id": 120262,
"tgt": "Are ganglion cyst on foot and pins and needles on legs related?",
"src": "Patient: I have a ganglion cyst on my foot, which they have tried to remove twice with no luck. I am getting pin and needles sensations in my right leg. My cyst in on my right foot. Are the two related? I have not been diagnosed with sugar as of yet. But I feel I have neurolopathy. How do they diagnose that? Doctor: Hi, A ganglion cyst rarely causes pins and needles sensation. Usually, it is due to nerve involvement not by ganglion cysts, ganglionic cysts are usually asymptomatic. Thank you. Hope I have answered your question. Let me know if I can assist you further. Regards, Dr. Jaideep Gaver, Orthopedic Surgeon"
},
{
"id": 75036,
"tgt": "Suggest treatment for chest pain and difficulty in breathing",
"src": "Patient: I sometimes get pains in my chest that feel like gas. The pain travels to my head. Most times going from a dull pain to a shape pain. The pain gets so bad, I feel a \"tremble\". I have to grab hold of something and concentrate on my breathing. After a few deep-concentrated breaths, I am able to make the pain bearable and slowly it subsides. It happened this morning, I had only had water to drink and ate an early modest meal last night. Doctor: Respected Sir, hi I evaluated your query.Answer is lying in your question only.It`s body`s reaction to stess manifasting as clinical parameters.* Have adequate rest and sound sleep of 8 hours without pills.* Do Deep Breathing exercises and YOGA for 30 minutes daily on regular basis as Gentleman promise which is the only remedy in your case, no medicines is going to work.* Avoid smoking / alcohol consumption if doing.* Try to control your anger in any minor / major issues.* Keep more laughing from inside to be happy for yourself and surrounding world. Hope I tried my level best to justify your query and concern. Thanks for using Health Care magic. Regards. Wishing you a very happy healthier life ahead. Bye Dear Take care."
},
{
"id": 103355,
"tgt": "Mild sore throat followed by wheezing attack causing choking, cough and breathlessness. Suggest",
"src": "Patient: Hi my name is david and i am from chennai. I have had a history of wheezing and yesterday i had a mild sore throat, followed by wheezing even became so bad that i began to choke and could breathe only in certain positions since i was choking my cough kept increasing making my throat even more sore.. Does this mean my wheezing is worsening or is it a one off episode Doctor: Hello dear,Wheezing is caused due to broncho-constriction (obstruction of smaller airway passages) which is indicative of Hyper-responsiveness of air passages (seasonal allergy or asthma)And whenever an individual with history of asthma is exposed to Respiratory tract infection (sore throat in your case), the symptoms aggravate.Management consists of:1. Treatment of the upper respiratory tract infection by warm water gargles, anti histamines, and anti-biotics (if needed)2. Asthalin inhaler- provide symptomatic relief by causing broncho-dilation (dilating the smaller airway passages, relieving the obstruction & increasing airflow to lungs)So, you can use it whenever you are having an acute attack.3. Montelukast preparations- used as a maintenance therapy to relieve symptoms of seasonal allergies & asthma.4. Anti-tussives can also be used to provide symptomatic relief from cough.5. Protect yourself from exposure to cold, dust or other allergens.So, kindly consult your Physician & start treatment accordingly.There is no need to worry, you will be fine.Wishing you a good health.Take care."
},
{
"id": 196065,
"tgt": "What causes the curvature of the penis?",
"src": "Patient: Hi i am 14 years old and i have a curved and bent penis. My penis is curved upwards to the left and i am very worried that this is affect my sex life in the future. I masturbate with my right hand often and i hope this is not the cause. Please help me out Doctor. Doctor: Hello Thank you for trusting HCM It\u2019s common for the penis to curve slightly to the left or right when it\u2019s erect. However, if you have a more significant bend in your penis, which may cause you pain or difficulty having sex, then there will be a problem. It may be a symptom of peyronies disease. Please consult your doctor he will examine and treat you accordingly."
},
{
"id": 28805,
"tgt": "How can UTI along with severe fatigue and dizziness be treated?",
"src": "Patient: hi...ive been ill for 10 days now. started with vomiting for 8 hours, went to ER,diagnosed with UTI. No better after 2 days and a shot of Rocephin from my family Doctor. Went back to ER. Sodium and potassium low, Hct low. Otherwise WBC \"on the low side of normal\"Still fevered now, today is the 10th day. Very weak, fatigued, dizzy. seeking answers please... :( Doctor: Hello and Welcome to \u2018Ask A Doctor\u2019 service.I have reviewed your query and here is my advice.The severe fatigue and dizziness narrate underlying systemic effects of the Urinary infection.It is recommended to get correction of the vital parameters.If you do not get an adequate response by oral medicines, you may require intravenous therapy for faster recovery after discussion with your doctor.Hope I have answered your query. Let me know if I can assist you further.Regards,Dr. Purva Patel"
},
{
"id": 134821,
"tgt": "Suggest remedy for red spots on knees and arms",
"src": "Patient: I have a pimple problem Dr. prescribe me Sotret Tablet & Picon Now my face is ok but I am facing some other problem on my body.. My back side of Knees,arms & Thysis appearing some reddish spots ..Can you please suggest why spot is appearing??Any tablet side effect or some other disease?? Doctor: Hi Dear,Welcome to HCM.Understanding your concern. As per your query you have symptoms of red spots on knees and arms which seems to be due to allergic reaction, sudden reaction from allergens, due to poor immunity of body and instant response of body to certain stimulus. Need not to worry. I would suggest you to start with intake of antihistamine medication such as benadryl. Drink plenty of water. You can take Allegra as well and start application of clindamycin gel on these areas. Visit dermatologist once and get it examined and start treatment after complete examination. You can apply aloe vera gel on them as well. Apply vitamin E oil at night. Avoid touching or pricking skin with any sharp object as it may get infected. Also apply cold compresses , which will help in reducing inflammation and redness . Hope your concern has been resolved.Get Well Soon.Best Wishes,Dr. Harry Maheshwari"
},
{
"id": 203718,
"tgt": "What is the remedy for reddish bump on glans, upwards enlarging meatus?",
"src": "Patient: reddish bump on top of glans. actually have had it for 25 years but more obvious now. noticed it after having sex with a girl that seemed to have a bump on uppermost part of vagina ...maybe friction caused it. Also have another issue, meatus has been enlarging upwards, its now twice as long as 40 years, dermatologist advised me it was stretching.... Doctor: Hi, welcome to Health care magic forum. Previously it might be the grouth, and now probably it got infected, and there is possibility of growing more. I advise you to consult a dermatologist for diagnosis and treatment. The meatus, you have observed now only, it would have stretched long before slowly to this size, and you need not worry about it. Take more of green leafy vegetables, pulses, sprouts, and protein rich foods to maintain a good health. Wishing for a quick and complete recovery. Thank you."
},
{
"id": 203248,
"tgt": "What causes red spots on head of penis?",
"src": "Patient: Hello, For the past few months I ve had a small reddish / brown (very light) spot on the head on my penis. It s in the centre on the front (not near the urethra). STD tests were negative, so I m not concerned about that. It doesn t appear to have changed size, nor is it rigid. It is completely flat. Any thoughts? Doctor: HelloThanks for your query,based on the facts that you have posted it appears that you have infection of the glans penis (Ballanitis).You need to take broad spectrum antibiotic like Cefexine along with anti inflammatory drug like Diclofenac twice daily.along with topical antibiotic ointment like Neosporin twice daily.Ensure to wash your genitals with warm water twice dailyEnsure to avoid sexual encounters till it heals up completely.Dr.Patil."
},
{
"id": 194566,
"tgt": "What causes bulging and twisted vein under foreskin?",
"src": "Patient: I have what appears to be a bulging and twisted vein under the foreskin, directly on top of the penis. It's there when flaccid or erect. I sit for very long periods of time on the computer, could this be a varicose vein or lymphangiosclerosis?. Also, are there any at home remedies for this?, I'm embarrassed to see a urologist.. Doctor: Hi, Sitting long periods in front of computer do not have any effect on your penis. Unless you have pain or numbness on your penis, no need to worry about the vein. If you want a proper diagnosis, it needs to be inspected personally. So consult any doctor. Hope I have answered your query. Let me know if I can assist you further. Regards, Dr. K. V. Anand, Psychologist"
},
{
"id": 132379,
"tgt": "Could tingling sensation in arms be caused by slip disc or shoulder surgery?",
"src": "Patient: I had shoulder surgery 6 months ago for a torn rotator cuff. I am now having symptoms of numbness, burning and tingling down my arm and into my hand, forefinger and thumb in the same arm I had shoulder surgery. It does not stay like this though. It comes and goes. Also, its not always in my hand and sometimes it just tingles in my arm. My shoulder still hurts from my surgery . I do not have this tingling and burning sensation while sleeping. Could this be a nerve from a slipped disc or from my shoulder surgery? Why would this come after six months. Doctor: hiAn x ray spine may be done to r/o neck spondylitis, the most common cause for pain radiation to arm with or without disc implication...you may rotate and move your neck in all directions to check for yourself if it radiates or accentuated pain down arm or cause any neck pain...Shoulder exercises are necessary to keep power of shoulder muscles and a weakened shoulder can itself lead to this sort of symptom, so check up with an orthopedics doctorphysical therapy too is requiredthanks,best wishes,Dr sunil srivastava"
},
{
"id": 202163,
"tgt": "What makes me feel dripping urine even after urinating?",
"src": "Patient: It feels like my penis is dripping urine when I don t have to use the bathroom, and when I do urinate it starts out as a slow stream and then a full stream. When I get done urinating it feels like I have a little bit of urine left in my uretha, but I can t get it out Doctor: Hello and .This kind of problem are generally seen in the patient with prostate problem, previously infected STDs, cystitis and in urethral inflammation/infection.I would ask for an ultrasound scan initially and find out the residual quantity of the urine after you urinate and prostate size.Do not Hesitate to ask if any quires. Hope it helps. Thank you."
},
{
"id": 145027,
"tgt": "Suggest treatment for burning pain in wrist area",
"src": "Patient: I was getting a blood draw the other day, and the needle hit a nerve going down my right arm. Immediatly I had burning and pain in my arm. Now every time I try to use my right arm I get burning pain in my right wrist area. what can I do to make this better? Doctor: Hi I did review your concernYour pain can be due to carpel tunnel syndrome or any bony pathology irritating your wrist nerves. I would advice you to consult a neurologist and he can diagnose the condition and then probably splinting the hand in a dorsiflexed position or local steroid injection will help you relieve of your pain.I hope this helps.Let me know if you have any more questions or concerns.Wish you healthy and happy life ahead."
},
{
"id": 98852,
"tgt": "What is the substitute for asthma pax depot?",
"src": "Patient: My mom is 66 years old and she is having Asthma since last 20 years, she was taking Asthma pax depot for last so many years but now we have stopped due to high pulse rate ( because of this medicine as suggested by doctor,) but her condition is becoming bad, can you please suggest Doctor: Hi.I appreciate your concern for your beloved Mom.I went through the history given in your query it seems to me that Asmapax Depot (Ephedrine, Phenobarbitone, Theophylline) is not an ideal medication to relieve her asthmatic attack of difficulty in breathing or wheezing, etc.The fact that the drugs like Ephedrine and Theophylline which are present in Asmapax Depot can cause increased or irregular heart beats, palpitations, agitation, tremors, irritability so, they should not be used.I would suggest her a Salbutamol inhaler which is more effective in relieving the symptoms of difficulty in breathing or wheezing but having a less tendency to increase the heart beat.So, Salbutamol can be an alternate but safer medication for her.If her symptoms aggravate she must be examined by a doctor for the possibility of giving steroid injection to reduce the symptoms of asthma.Hope I could answer your query."
},
{
"id": 187342,
"tgt": "Should rinsing with chlorhexanate gluconate be followed by washing mouth?",
"src": "Patient: I just rinsed with chlorhexanate gluconate prior to oral surgery tomorrow. Now I must swallow a pill. Should I rinse out the dregs of the chlorhexanate or may I swallow the coating and rinse it down with water and the pill? That is, should the germicidal rinse remain in my mouth a while and not be rinsed away? Is it dangerous to swallow? Ione Doctor: Hello, Thanks for your query.Chlorhexidine gluconate oral rinse provides antimicrobial activity during oral rinsing. Patients should be instructed to not rinse with water, or other mouthwashes, brush teeth, or eat immediately after using Chlorhexidine gluconate oral rinse. Chlorhexidine gluconate oral rinse is not intended for ingestion and should be expectorated after rinsing. Chlorhexidine gluconate oral rinse is indicated for use between dental visits as part of professional program for the treatment of gingivitis as characterized by redness and swelling of the gingivae, including gingival bleeding upon probing. Some patients may experience an alteration in taste perception while undergoing treatment with Chlorhexidine gluconate oral rinse.I do hope that you have found something helpful and I will be glad to answer any further query.Take care"
},
{
"id": 615,
"tgt": "How will i conceive after stopping Depo shots?",
"src": "Patient: Hi, I had the depo injection to prevent me from getting pregnant for around 1year. I stopped my depo when it ran out in Septemeber and i havnt had an injection since. I have no started regular periods and I am wanting a baby. after reading reviews online I know it can be difficult to get pregnant after the depo but I have now been off the depo for 7-8month. what is the best possible way to spead up me getting pregnant. me and my partner desperatley want a baby and it seems to be taking ages? any tips or suggestions on how to speed things up? Doctor: Hi, Thsnks for the query .I understand your concern. Depo contraceptive shots take 6-10 months to get regular menses. Please wait for 2 more months... you may resume normal periods automatically. Side by side you can get investigation like usg/ thyroid function test to exclude any other possible cause for menstrual problem (which may be coincidence) Thanks."
},
{
"id": 88946,
"tgt": "Cause for pain in right abdomen and weight loss?",
"src": "Patient: pain in right side of abdomenhigh fever (101,102)weight lossno proper digestion & ultrasound of abdomen& x-ray of chest are although normal .in blood test bilirubin 2.1mg%,sgot(ast) 52i.u. ,sgpt(alt) 69i.u. has increased .can you plese suggest something Doctor: hi welcome to hcmelevated enzymes indicate some pathology in the liver.it could be hepatitis.complete liver function tests should be done.repeat your ultrasound after one week.drugs like pentoxyphylline will help.take syrups like zymax for better digestion.take 3-4litres of water.avoid fatty foods.consult your doctor for expert management.thankyou"
},
{
"id": 21825,
"tgt": "What causes high BP, dizziness and fatigue?",
"src": "Patient: I have really high blood pressure lately, dizziness and fatigue. Most recently I have a sensation in my leg that is painful to the skin touch.. also , once in a while I will get a pulsating pain that lasts for just a minute. Yesterday as I was sitting at my desk, I had a heat sensation in my back and then i broke out in a sweat. What might be going on? Doctor: Hello , High blood pressure per say may cause the symptoms of headache , dizziness and fatigue . Treatment of hypertension with lifestyle change or medication such that the blood pressure reading is below 140 mg / 90 mg should be the priority here . Also if you are more than 45 years old , obese ,smoke or have other risk factors , an cardiac evaluation by doing a and ECG and If needed and 2d Echo and TMT should be considered. However again the sweating episode can still be due to fluctuation in BP . Hopefully I could help Regards Dr. Mody"
},
{
"id": 27481,
"tgt": "Suggest treatment for cardiac stress test abnormal without operation",
"src": "Patient: My husband is scheduled for a cardiac cath tomorrow morning because his nuclear stress test came back abnormal. He does not have angina. He has had a bit of loss of energy and endurance. Those were his only symptoms. We were told there is an 85% chance there is a 70% blockage in his RCA. I know once you have the stent, there is no turning back so I am hoping for clean arteries. We would like to have a conversation with the doctor before the procedure to make sure he doesn t decide to put in the stent if my husband could manage without it. What questions should we ask or how could we present it so he knows we trust his judgement, but really want to be conservative here? When is the Fractional Flow Reserve (FFR) indicated? Doctor: Hello. Thank you for your question and welcome to HCM. I read your query and understood your concerns.Allow me to start my answer with an analogy. Stent placement in a coronary artery is a remedy to the artery, as it is taking antibiotics when you have sore throat. If there is a blockage greater than 60%, then placing a stent is reasonable. There are no drugs or therapeutic remedies that would cause regression (\"growing down\") of the intracoronary atherosclerotic plaque. With the maximum of usage and dosage, they could stop it from building up, at best. Percutaneous coronary intervention (PCI: balloon + stent) is a safe and effective treatment of intracoronary plaques. It is accompanied with a great deal of symptom relief. With the right and proper care after the stent implantation, it should be successful. As about the fractional flow reserve, it is used by the operator when he has doubts whether to deploy a stent or not. In this term, I would only speculate if I gave you a precise answer, because I would had to see it myself. Summing up, my opinion and recommendation is that, if that plaque results over 70% in coronary angiogram, a stent should be implanted. At the end, the choice is always yours.I hope I was helpful. Take care and good luck!"
},
{
"id": 181461,
"tgt": "Can tooth enamel erosion be caused due to consumption of Marijuana?",
"src": "Patient: my son, has not attended his teeth as he should for years now. He has enamal erosion which is black virtually all all the line between his teeth and gums. He also smokes weed, but says this has no impact. Is there anything which can be don to save the teeth? Doctor: Hi..Welcome to HEALTHCARE MAGIC..I have gone through your query and can understand your concerns..As per your complain black lines between the teeth can be either stains over the teeth due to improper maintenance of oral hygiene..Enamel erosion can occur in Marijuana smokers or any other recreating drug smokers as after smoking the individual tend to clench his teeth tight and it can lead to erosion of teeth..I would suggest you to consult a Dentist and get him evaluated and a thorough clinical evaluation can help in diagnosis and treatment can be done accordingly..He can be advised to get scaling and polishing done if there is staining..Fillings can be done if there are decays in teeth..Enamel erosion can be further prevented by ruling out the cause of erosion and preventing the same..Hope this information helps..Thanks and regards.Dr.Honey Nandwani Arora."
},
{
"id": 34053,
"tgt": "Suggest treatment for sinus infection",
"src": "Patient: I am 66yrs old female, I had taken antibotics for sinus infection for a week, but I am still feeling crappy. My stomach is quezzy , I have no appetite, in the evening, I usually get a low grade fever and am freezing. I take a Prilosec already, and no matter what I drink, or eat, my stomach gets upset. This has been going on for over a week now. Sorry, I did not know I would have to pay for some advice. Sorry to of bothered you. Doctor: Hi welcome to hcmI understand your query and concern.Your symptoms are suggestive of Acute Sinusitis.I advise you to get an X ray PNS and CT PNS to confirm the diagnosis.Meanwhile I advise you to take Tab.Amoxiclav 625 mg twice daily for 1 week,Tab.levocet twice.Avoid allergic foods like tomato,brinjal etc.Drink hot soups and hot liquids.Avoid cold baths.Consult your physician for expert management and follow up.Hope I have answered your query at the moment.Post your further queries if any,Thank you"
},
{
"id": 53872,
"tgt": "What causes high liver enzyme and low vitamin d levels?",
"src": "Patient: i got results from my blood work saying that i had high liver enzymes and low vitamin D levels. i went in because i had a loss of appetite and was getting headaches that would last awhile. i do not drink and have never drank. i am young and healthy so what could it be? Doctor: hi.thanks foro posting query.high liver enzymes should be thoroughly investigated.causes1. virus ( HepA, B or C)2. alcohol3. obesity4. autoimmunetherefore, visit a gastroenterologist/ hepatologist and have the blood tests done to rule out the viral causes in particular.further course of action will be advised accordingly.wish you good and sound healthregardsDr Tayyab Malik"
},
{
"id": 174559,
"tgt": "Is swelling under left eyebrow after getting bumped harmful?",
"src": "Patient: My son and daughter bumped heads, my daughter immediately had a huge swelling just under her left eyebrow, that has partially closed over her eye. It's comes out approx 4-5cm I think and looks like it's filled with fluid. Should I be worried or just continue putting a cold compress on it. She seems fine but in pain. Doctor: Hi,It seems that there might be having hematoma formation following hard hit.Apply ice pack 2-3 times a day for 2-3 days.Consult your doctor and get examined to rule out any injury in her eye.Ok and take care."
},
{
"id": 91522,
"tgt": "What causes painful lump under left testicle and abdominal pain?",
"src": "Patient: hi i have a med sized lump under my left testicle that is very painful i found it 24 hours ago . in the last couple of hours my testicle has became very swolen and my lower adomen hurts and im feeling aches and pains as if i am getting a cold, and a little bit of pain on both sides of neck Doctor: Hi ! Good eveining. I am Dr Shareef answering your query.Possibly you have got epididymo-orchitis (inflammation of the testicles) which could be a viral or bacterial cause if there is no history of trauma. You might be developing mumps which is giving rise to the pain on the sides of neck, and feverish feeling. If I were your doctor, I would advise you for an anti inflammatory medicine along with a proton pump inhibitor, an antibiotic if need be, and a proper scrotal support. A routine blood tests would be helpful as a baseline. You would be needing close observation.I hope this information would help you in discussing with your family physician/treating doctor in further management of your problem. Please do not hesitate to ask in case of any further doubts.Thanks for choosing health care magic to clear doubts on your health problems. Wishing you an early recovery. Dr Shareef."
},
{
"id": 13069,
"tgt": "What are the welt like rashes on my body?",
"src": "Patient: I had a welt-like rash on my upper arms, shoulders and back last night after dinner. It spread rapidly and moved around those parts of my body. My wrists and hands were also itchy with bumps. This morning I woke up with a sore throat and runny nose, like I had a cold. What could it be? Doctor: Hi, It may be allergic urticaria and allergic rhinitis. Consult the dermatologist for the perfect diagnosis and proper treatment. I would recommend you to take antihistaminics like bepotastine under supervision of your doctor. Avoid contact with suspected allergens and also avoid dietary stuff which may precipitate the disease. Hope I have answered your query. Let me know if I can assist you further. Regards, Dr. Ilyas Patel, Dermatologist"
},
{
"id": 28522,
"tgt": "How can I get rid of weakness, abdominal pain, proteus infection and salmonella?",
"src": "Patient: Hi. I m a caucasian 61 y old female diagnose with Salmonella 1:40 and a mild proteus infect. After 7 days on Flagyl I am worse today - can t keep even organic coconut water down for dehydration. Stols are no longer hello but almost constipated and then very loose - stools come about every 5 hours. Stomach cramps intermittent (associated with antecedent to bowel movement). I m feeling very weak. Any thoughts? Thanks Oh - I m living in Mexico and never eat off the street. Doctor: Hello, It is good to know that you are not getting loose stools at present. 1. Weakness is because of dehydration & sodium, potassium depletion in your body electrolyte balance. Take ORS, soups & juices. Stay well hydrated. 2. Take semi-solid food( porridge/ oats) with boiled Raw banana (one at a time). You can boil sago (Tapioca sago) in milk & take it at intervals, it effectively corrects bowel movement. 3. Take Spasmol (Spasmoproxyvon), one sos when abdominal pain, antacids & PPI's, Pantoprazole - 40, once daily for 5 days. In the meantime, stay away from stress, stay relaxed. Take adequate sleep. Avoid addictions if any. Take medications always under your treating doctor. Hope I have answered your query. Let me know if I can assist you further. Take care Regards, Dr Nupur K, General & Family Physician"
},
{
"id": 222999,
"tgt": "How can throat pain with dysphagia during a pregnancy be treated?",
"src": "Patient: hello, I've just found out that i'm pregnant and my throat is in agony - i think from all the sickness. When I swallow or cough it feels like razor blades. I can't sleep and eating really hurts. Is there something that I can take without hurting the baby? Any help would be most appreciated, thanks Doctor: Hi dear, I have gone through your question and understand your concerns.Pain in the throat with dysphagia in pregnancy can be due to oral thrush or pharyngitis.I would suggest you to consult an ENT specialist to get properly examined, investigated and treated accordingly.Hope you found the answer helpful.Wishing you good health.Dr Deepti Verma"
},
{
"id": 212622,
"tgt": "Diffuse axonal injury after accident. Currently suffering from short term memory loss, rude behaviour. Treatment?",
"src": "Patient: Hi Doctor, my brother wilson met with an accident on 10 Oct 2012 and he suffered from diffuse axonal injury , 7 days he was in unconsciousness , after 10 days they shifted to general ward on that time his left side parts was not woking proprely like(strock) after doing oil massage and physio treatment now he can able stand and walk. now his condition is his suffering from short term memory loss . his unable to recognize what is going on surrounding him and nowdays his behaviour also changing rudely talking & beating every one. is there any treatment is there to recover completely how earlier he was, please help me on this sir am having one and only one brother in my life. Doctor: Hello, welcome to Healthcare Magic. I is sad to hear about your brother health. Neuronal injuries often are permanent, so complete recovery is rare in such cases. Over a period of time some improvement in memory and behavior is possible. After brain injuries some personality changes are possible which can lead to aggressive behavior. In such cases low dose anti psychotic medicines are helpful. For that you can consult some nearby psychiatrist. Wish you all the best. Regards, Dr Ashish Kumar Mittal www.99doctor.com"
},
{
"id": 149383,
"tgt": "Dent on top of forehead possibly on skull. No history of any medical problem. Help",
"src": "Patient: HI, I recently noticed a on the top (right corner) of my forehead, a dent that was not there yesterday. I slid my fingers across the dent, and it feels as if, it's in the scull. I do not have any type of medical condition that I am aware of.I am very nervous and confused about how this happened. I don\u2019t know if stress might be the culprit but that the only thing I can think of.Hopefully you can help me figure this out.Thank You Doctor: Hi,Thank you for posting your query.Dents in the skull are often asymptomatic (they do not produce any symptoms) and can be left alone. There is no need to worry about them. They can occur spontaneously (without any reason) or in relation to an injury to head.If you develop headache or any other neuro symptoms, please get a CT scan of th head done.Best wishes,Dr Sudhir Kumar MD DM (Neurology)Senior Consultant Neurologist"
},
{
"id": 121900,
"tgt": "Can HCG diet cause side effects like numbness in fingers?",
"src": "Patient: I started the HCG diet about two months ago. I have had idiopathic peripheral neuropathy for 30 years. It has bothered my feet and ankles. Since starting the diet, I have developed numbness in my fingers. That could either be a natural progression of the PN. Could it also be a result of the HCG diet? Thank you. Doctor: Hello, HCG diet is not approved for weight loss unless it was recommended by your doctor. As it contains restrictions in foods, you might suffer a lack of nutrients, plus changes in pH of body fluids. All these changes might affect the already damaged nerves, precipitating neuropathy pain flare-up. So you should stop this diet, and if it was recommended by your doctor, report this to him/her. Hope I have answered your query. Let me know if I can assist you further. Take care Regards, Dr Albana Sejdini, General & Family Physician"
},
{
"id": 157192,
"tgt": "Should I be concerned for the ovarian cancer having the symptoms of abdominal bloating, early fullness and upper abdomen contractions?",
"src": "Patient: Hi, For about 6 weeks I ve been having various GI issues. Upper abdominal bloating, early fullness after eating, mid back discomfort, upper abdominal contractions. HIDA scan, gallbladder ultrasound came back normal. I feel all the discomfort is pretty high. Should I have concerns of ovarian cancer? Doctor: Hi Thanks for your query. Ovarian cancer is often insiduous and can oresent with these kind of symptoms to start with. You have said gall bladder ultrasound was normal. Did it include the whole of abdomen? if so, then possibilty of ovarian cancer is low. However, more detailed evaluation, if possible by a gyn onc is required to rule out ovarian cancer. Hope this helps. Regards."
},
{
"id": 134045,
"tgt": "What causes pain and heaviness in legs after total knee replacement surgery?",
"src": "Patient: I had a total right knee replacement 2 years ago. After one year I I expected full recovery, but I am still experiencing pain and my leg feels heavy ? I apply heat, ice, go for swim and physical therapy treatments. How long do I have to endure this? What could be the problem? Doctor: hi,thank you for providing the brief history of you.As you mentioned that post 1 year of TKR the pain is still persisting regardless of undergoing all protocols. It is sad to hear this.Well, let me help you further, the first thing you should is to visit the one who performed TKR and get the thorough orthopedic assessment. As the one who operated the knee has seen inside of your body well so he is the one to comment.Now to look further, total knee replacement, is it a bicompartment or tricompartment or unicompartment? this makes a lot of sense in understanding the Surgical procedure.To look ahead, post TKR how many weeks the physcial therapy was performed, I personally recommend my patients atleast 12 weeks of protocol and post that as per the assessment the next number of weeks will be designed.Also, to understand that prior to TKR you had a lot of weakness in the muscles of the thigh, hip and core. Now if this three area are not Strengthened to Muscle Grade 5, then you need some more input of the physcial therapy.Usually, my cases post TKR can perform 40 squats in a 60 seconds. also, they can perform, sprinting, running, jogging, jumping etc. I get 100% knee stability.Also, in TKR your internal structures of the knee are removed to provide a new implant. As the role of the implant is to just hide the knee joint sensory system to sensation less. The degenerative changes of the bone is irreversible. Also, as the internal structures of knee are removed, the joint is super immobile and only the strength in the muscles can hold the joint intact and get you rid of pain. There is alot to understand about the TKR, as what I mentioned is just a few points. With my patients I initially spend alot of time making them understand the Arthitis of knee, how to avoid TKR, What is the TKR procedure, what are the plus and minus of TKR, how to get the best joint stability and how the whole process will be followed with strenous exercises. By this, my 50% of Job is done as they understand their joint better and perfrom the best possible exercises advised under supervision. Within 12 weeks of time we get 80% stability and the last 20% we go slow but painful procedures and again in another 8 weeks all is settled. I wish you get your doubts clear by sitting down with a Surgeon and the physcial therapist as well to get the best results.RegardsJay Indravadan Patel"
},
{
"id": 153475,
"tgt": "Suggest treatment for throat cancer",
"src": "Patient: Hi, my brother is a patient at the ENT section of the Johannesburg General hospital. he has been diagnosed with throat cancer. They have inserted a tube into his throat for oxygen. He cannot eat and has been there for a while. He is in terrible pain and seldom receives medication. He basically is left to himself. What is the possible solution to his disease. Doctor: Hi, dearI have gone through your question. I can understand your concern.Your brother has throat cancer. Treatment depends on stage of cancer.In early stage surgery is helpful. In late stage surgery is not helpful and chemotherapy and radiotherapy remains the treatment options.Consult your doctor and start treatment according to stage and spread off cancer. Hope I have answered your question, if you have any doubts then contact me at bit.ly/Drsanghvihardik, I will be happy to answer you.Thanks for using health care magic.Wish you a very good health."
},
{
"id": 210640,
"tgt": "How does your endocrine system assist your reaction in the below mentioned situation?",
"src": "Patient: Hi, I am taking psychology and I've stumbled upon a question. Let's say you are driving down the highway when your tire blows, immediately you swerve several times and end up on the side of the road. Explain how the endocrine system assisted your reaction. Doctor: Hello,Thanks for choosing health care magic for posting your query.There is a boost of norepinephrine. It produces an alarm reaction and improves your reaction time.Hope I am able to answer your concerns.If you have any further query, I would be glad to help you.In future if you wish to contact me directly, you can use the below mentioned link:bit.ly/dr-srikanth-reddy\u00a0\u00a0\u00a0\u00a0\u00a0\u00a0\u00a0\u00a0\u00a0\u00a0\u00a0\u00a0\u00a0\u00a0\u00a0\u00a0\u00a0\u00a0\u00a0\u00a0\u00a0\u00a0\u00a0\u00a0\u00a0\u00a0\u00a0\u00a0\u00a0\u00a0\u00a0\u00a0\u00a0\u00a0\u00a0\u00a0\u00a0\u00a0\u00a0\u00a0"
},
{
"id": 106407,
"tgt": "My eyes still come red and srutch, what can I do ?",
"src": "Patient: my eyes has scrutch 8 months doctor told me grass allergy but now in mongolia is winter still come red and srutch what can i do Doctor: your symptoms are too wage, elaborate it a bit. this sounds soo usual. what do you mean by srutch?? do have any kind of discharge from the eye, since when do you have this? are you under any medication for the same?"
},
{
"id": 221413,
"tgt": "When can a confirmatory pregnancy test be taken?",
"src": "Patient: Okay I have. Had a period sept 27 to the 29 and one day after it ended completely,I went to use the restroom wiped and I seen light pink dots. And two brownish almost redish. I have taken 3 home pregnancy test and they came back negative. Buht now I am feeling pain and stiffness in my legs when I walk and stand for a certain amount of time. Even when I lay down they still hurt.and I ve been shaking a lot lately due to sensitivity of the cold weather. What do you think this could be? Do you think I took the pregnancy test to early to tell? Doctor: Hi I understand your concern. Earliest detection of pregnancy can be by blood HCG testing around expected date of menses. Home pregnancy test can give reliable results after 8 days dealy in menses. Usg abdomen by vaginal probe can diagnose pregnancy of 5 weeks & with abdominal probe by 6 weeks. thanks."
},
{
"id": 199851,
"tgt": "Am I having side effects from Cortisone injections?",
"src": "Patient: Dear sir Recently I discovered that I have ependymoma,and my neurological doctor described me 5 cortisone injections but after the third one I have a sever left kedny pain just when I lay down and in the same time I had herpes over my penis since 10 years ago but It was treated but now I saw it again with great amount over the penis again. So please inform me if it is because the cortisne injection. Doctor: Hello dear,Thank you for your contact to health care magic.I read and understand your concern. I am Dr Arun Tank answering your concern.Recurrence of herpes over the penis shaft is because of cortisone injection.Pain in abdomen may be because of the cortisone injection. But it is required to be investigated.Cortisone is the steroid which causes immunosuppression. Initially it cures the infections like herpes but later grows with much higher intensity.So in my advice if the cortisone injection is not necessary to take than you can avoid it or you can take it for short period of time and than you can avoid taking it. Alternatively you can keep some time gap inbetween the drug taking.I will be happy to answer your further concern on bit.ly/DrArun.Thank you,Dr Arun TankInfectious diseases specialist,HCM."
},
{
"id": 203281,
"tgt": "How to control masturbate addiction?",
"src": "Patient: iam 19 years old ,iam shaking my penis two times a day.i have to control it ,after finishing the shaking of penis ,sperm will come out ,i go and wash it,by daily doing this center of my chest is getting like a hole. pls doctor you have to say a reason for that ,and also controling measures Doctor: Hi and . Masturbation is very common phenomenon in adolescnets and you no need to feel guilty and anxious about it. Here are few tios to control urge for masturbation. 1.Find another outlet for your time and energy. Fill your life with engaging activities.2. Enjoy some out door sports3. develop new hobby4. If you have problems before going to bed or in the shower, refrain from any temptation to masturbate. For instance, if it's a problem late at night, drop to the floor and do push-ups until you're too exhausted to do anything but fall asleep. If you find shower time too tempting, start using ice cold water only5.If you masturbate frequently because you feel lonely, find ways to be as socially engaged as possible, means avoid loneliness6. Stop watching porn on your computer or TV. Throw away all porn material from your room.7. Their are many myths involved about masturbation, your chest will not be affected due to masturbation If nothing helps you, consult a local doctor for help. All the bestRegardsDr Santosh"
},
{
"id": 135977,
"tgt": "What causes itching and swollen toes?",
"src": "Patient: A few days ago, one of my toes started itching, then swelled up and turned red at the tip. Then, 3 days ago, I developed a hives like rash covering pretty much my whole body. Today, I have blisters on the affected toe and also smaller ones in the area around it. Went to the doctor and she had me get a blood test for uric acid (testing for gout, I guess). The uric acid test is in normal range - 6.1. The toe is not painful, but itches like there is no tomorrow. I am a 60-yo male, and active - I do water aerobics and walk 2+ miles. Also have been hiking recently. Any ideas? Doctor: HiWelcome to healthcaremagicI have gone through your query and understand your concern.You can discuss with your doctor about it. I think you should exclude any cold exposure, diabetes and circulation disturbance. It can be something like cold allergy and circulation disturbances. You can get it clinical examination by general surgeon for better understanding. Hope your query get answered. If you have any clarification then don't hesitate to write to us. I will be happy to help you.Wishing you a good health.Take care."
},
{
"id": 73559,
"tgt": "Suggest treatments for gynocomestia due to asthma",
"src": "Patient: sir i am mubashir, 26 years old, married, male, i am an asthma patient and have bee using various salbutamol inhalors like ventoline etc since my childhood i remember. i developed a bulky chest normally refered as man boobs/ gynocomestia. i need your expert opinion that how can i get rid of it, i searched net about it and have come to know about asthma as one of the cause for it. i cant afford surgery, is there any way out !!!! Doctor: Hello dearWarm welcome to Healthcaremagic.comI have evaluated your query thoroughly .* The gynecomastia per so has no other treatment than surgery .Hope this will help you .Welcome for any further guidance .Regards ."
},
{
"id": 125250,
"tgt": "What causes restless legs and pain in elbows?",
"src": "Patient: Hi I am a 34year old female, never sick really, PMH of Pneumothorax, appendectomy, Irregular heartbeat, I take cardicor 1.25mg - short pr interval with PAC s + PVC s, High biliruben I notice in my urine sometimes but mostly my eyes are yellow, Anyway my concern is regarding my legs my upper legs get so restless and heavy I fell like its an effort to drag them sometimes expecially after exercise and I have a constant ache/pain in my elbows? Any Ideas? Doctor: Hi, As a first line management you can take analgesics like Paracetamol or Aceclofenac for pain relief. If symptoms persists better to consult a neurologist and get evaluated. You might require drugs like Ropinirole. Hope I have answered your query. Let me know if I can assist you further. Regards, Dr. Shinas Hussain, General & Family Physician"
},
{
"id": 112325,
"tgt": "Back pain, prescribed naproxen, tramadol,muscle relaxers. Feeling numb in stomach. Treatment?",
"src": "Patient: I have been having sever back pain in my middle back. Been to the doctor twice and been told they do not see anything wrong with me and once gave me 3 days of muscle relaxers and Naproxin and the second time I was I was given 2 days of Tramadol. I am now having numbness in my stomach area in addition to the back pain. Doctor: these can be due to acidiy and gadtritis caused by medicines as all of the pain kilers cause hypercidity and gastritisneed to take antacids to clear the stomach no oily non fatty no greesy food low protein diets with high fiber and plnty of water intake will help"
},
{
"id": 140220,
"tgt": "What causes dry eyes with dizziness?",
"src": "Patient: About once a week while at work I have a suddon onset of very dry eyes and then become very dizzy and feel like I could almost pass out. After 1-4 hours it seems to go away. I have had labs done and an MRI done and they said everything looks fine. This has been happening for over a month now. Any ideas on what might be going on. Doctor: Hello, I would explain that your symptoms could be related to Sjorgen disease or any other eye disorders. I recommend consulting with an ophthalmologist and discussing these possible disorders. Hope I have answered your query. Let me know if I can assist you further. Regards, Dr. Ilir Sharka, Cardiologist"
},
{
"id": 176780,
"tgt": "What could a bump on forehead of a child be?",
"src": "Patient: my daughter is 5 and weighs approximately 50 pounds. she woke up this morning with a goose egg on her forehead and didnt complain of hitting her head the day before. tonight before she went to bed i noticed it went to her left eye and and her nose bridge looks swollen and a little bruising around her eye. we thought maybe it was a reaction so we gave her some childrens benadryl and the swelling went down a bit so we gave her some before bed time. what do you think is causing this? Doctor: Hello. I just read through your question. It is possible that even a small, unnoticed tap to the head, perhaps even while asleep, caused the bump. It is not unusual for the swelling to move downward with time as a result of gravity. It will resolve on it's own. There is no need for worry."
},
{
"id": 107940,
"tgt": "What causes severe back pain and terrible headache?",
"src": "Patient: My 13 year old daughter has been experiencing lower back pain along with terrible headaches. Her stomach has been upset also. This has been going on for about two weeks. On Saturday, we noticed that she had red lined going across her back from the middle to lower region. Today her discs in her lower back look swollen and the red line are now on both hips. She has an appointment tomorrow with our family doctor and I wanted some information or questions I should ask the doctor. Doctor: Whether there is reduction in intervertebral disc spaces? If so which area in lumbo sacral spine. What causes are guessed e.g. degeneration of bones due to TB ( chances are lower at this age), arthritis or spondylosis ( less chances).Treatment may be decompression of nerves by lumbar support or traction. Pain killers will be given.Hope it helps you."
},
{
"id": 64313,
"tgt": "Suggest remedy for lumps on the knee cap",
"src": "Patient: I fell hard on both knees on a tile floor 2 months ago....yesterday I was sitting down and all of the sudden a lump developed in my knee cap, my knee swelled & my food & hip felt tingly like the circulation was interrupted. 30 year old female. No fever, redness etc. Doctor: Hi,Good Evening.This is Dr.Savaskar from India,attending your query to my HCM virtual Clinic.-I studied your query in-depth and Understood your concerns.-You suffer from mostly-Pre-patellar bursa-post-traumatic.Its also known as Housemaids knee.Treatment-a-Tb NSAIDs to reduce the bursitisb-Antibiotics-to control any mild infection causing bursitis.c-Xry knee and orthopaedic surgeons opinion -if it does not subside in 1 wks time.d-Non-regressing bursal cyst-is Surgically Excised and treated permanently.-Hope this would help you to recover fast.Wellcome to HCM if you need more help.Wishing you fast recovery.With Best of Regards Dr.Savaskar M.N. M.B.B.S;M.S.(Genl-Thorasic-CVTS) i)Senior Surgical Consultant /Thorasic Super-specialist-CVTS, ii)Specialist in-Non-Curable-Diseases-like Cancer,Asthma,Psoriasis and Rejuvenation and Tissuefailure therapies."
},
{
"id": 394,
"tgt": "What causes difficulty in conceiving?",
"src": "Patient: Hi Doctor, I m 27 years old and weight 57 kg 5 6 height. No medical issues.for the last 4 years we are trying to get pregnant. but this doesnt happened yet. Husband got low sperm count around 5 million. Took some injections and count came to 18 million. Once we did IUI but didn t get conceived . I would like to know what all precautions should I take after intercourse on Ovulation. I m working and my husband drops me to office everyday in two wheeler. Is travelling can be an issue or anyother food routines that I have to follow inorder to conceive? I m a keralite. Doctor: Hi, first of all I want to know regarding your reports if everything is fine. Then what was the motility of sperms and morphology also. You can try 3 to 4 cycles of IUI. Travelling is not an issue. Have a healthy diet. You should be in contact with your husband for next 2 to 3 days of ovulation."
},
{
"id": 34822,
"tgt": "What causes anal itching while on medication for staphylococcus?",
"src": "Patient: Hi, may I answer your health queries right now ? Please type your query here. Hi am 34yrs 5ft 68kg.my result raed staflocucus,i was gvn 10 days AVECF INCTION WHC I KOMPLET LS SUN,ALSO DACIN-F 300MG.BUT AS I WAS TAKEN THE INJECTN I STARTED PURGEN ON D 7TH DAY.doc aftar stulen my annal(anus) itches me alot.wht do i do?thanks Doctor: Hello,Thank you for posting your query to HCM.From what you are describing it might be a thrush infection. It came after the use of an antibiotic (Dacin-F), and the anus is a place where fungal infections tent to grow.If I was your caring doctor, I would suggest to take Canesten 2% cream, to use twice/day for 5 days. If it is used with a thick layer, spread to cover about 2 cm over and outside the borders of the itchy place, it will be enough to clean the infection. Try to keep the area clean and dry. If it doesn't get better, or is getting worse, consider to have it checked by your local doctor. Its very low possibility to be something else, but sometimes is better to have a look.Let me know if you have any further question.Regards,Dr. Klarida Papaqako."
},
{
"id": 64188,
"tgt": "Suggest treatment for a lump in the tailbone area",
"src": "Patient: I went to the Hostpial about six months ago and they found a lump on the left side of my tail bone and it only hurts every once in awhile they said they think it s a fatty tumor but not sure and wouldn t give me an mri.today it s hurting again and I live in a very small town now.inwas just wondering if it could be s.a.?and should I be worried about the placement? Doctor: I welcome you to healthcare magic for your health concernsI am Dr Fahim and I will looking into your problem I have gone through your question and understood your concerns. There is lump near your tail bone and it is painless most of the time. It appears to be pilonidal cyst with differential diagnosis of lipoma as well, at present nothing is required except observation. There is chances of it to get infected, then it will results in pilonidal abscess. Timely drainage of this abscess in important, otherwise it will result in pilonidal sinus which will require a surgical excision of the sinus tract under spinal anesthesia. At the moment observe this lump and keep the area dry and clean, if it get painful consult general surgeon.I hope my answer will help. Do rate if you like it.Regards"
},
{
"id": 128,
"tgt": "Is pregnancy possible with 25 MN sperm count and 15 % mortality?",
"src": "Patient: Dear sir, I am 42 yrs old male.MY height is 5 feet, 9 inch and my weight is 58 kg. i am diabitic since 3 yrs. i am taking glynase (5 mg) two tablets in morning before breakfast and piomed 30 one tablet after lunch.My gluse level is well under control. I drink two pegs of whiskey two times in a week.I want to know, can whiskey may create problems in future? Doctor: Hello,Yes drinking, smoking, and tobacco chewing affects sperm count so quit it. Do serum TSH, LH, FSH, and testosterone. Then do testicular Doppler to find the exact cause of low count and start taking Siphene to increase sperm count.Hope I have answered your query. Let me know if I can assist you further.Regards,Dr. Sheetal Agarwal"
},
{
"id": 86831,
"tgt": "What causes a gripping pain in my stomach?",
"src": "Patient: Hi, I have a pain in my stomach it feels like a fist gripping me inside or like the feeling after you've been punched in the stomach I just want to know if it could be something serious that I may need to have the light run down my throat and have checked out what do you think it could be? Doctor: Hi.Thanks for your query.You have a history of fist griping in the stomach or as if punched in the stomach.I do not think that passing the light down the throat (endoscopy you mean it seems) may be helpful if this is stomach volvulus or duodenal problems. I would advise you to visit ER ASAP, get a simple X-ray of the abdomen in the standing position, ultrasonography to get an idea about the diagnosis and get further investigated like : Enteroclysis and so on and get treated accordingly."
},
{
"id": 113855,
"tgt": "Does chronic lower back pain indicate kidney problems and what tests should be conducted to confirm it ?",
"src": "Patient: Hello, my mum has got lower back pain at the right side. she had a urine test that showed no problems, she has no nausea or vomiting and no urinary disorders . could she have a kidney problem? by the way, she s always had pain all over her back that comes and goes but this right side pain has been for months so far and more frequently. what doctor should she consult and what analyses or imaging should she have? Thanks in advance. Doctor: Hello. Welcome to HCM forum. Locallised pain in the right lower back can be from kidney, but is not necessarily so. The causes can range from simply musculoskelatal to visceral. You need to tell us in detail, the character of pain. priminarily she can consult any experienced nephrologist, who will rule in or rule out kidney problems. The investigations required by her will be: 1. USG -KUB region 2. Complete blood picture 3. If required - Abdominal CT 4. lastly MRI-spine if no visceral cause is decided. Take care."
},
{
"id": 148802,
"tgt": "MRI reveals Flair signal intensity in brain, vascular malformation and Glioma. What do the findings imply and treatment?",
"src": "Patient: Hi, I have a health concern for quite a while.I have been diagnosed with hyperprolactinemia (80) 6 months ago. I did not have any period for 3 months and before it was very irregular. I got sick two months ago (dizzy, hot flashes, headache, lightheaded, falling asleep) the MRI scan did not show a prolactimona, but there is a focal area of increased FLAIR signal intensity seen along the posterior lateral aspect of the right side of the midbrain with no associated contrast enhancement or mass effect. Differential diagnosis would include a small vascular malformation, low grade glioma or possibly a demyelinating process. I started Cabergolin two weeks ago. I also have pressure on both ears since last week. My physician has no idea what can be wrong. The neurologist wants to do another MRI next month.What could be wrong and what could I do to get better? Doctor: HIThank for asking to HCM I think the report of MRI is nothing much in specific, it does not show any abnormality as such, I personally concentrate on clinical complains of the patients rather than the investigative reports unless when it needs to be correlate with the clinical conditions, if you do not have any symptoms then why to worry, I think nothing is wrong with you every thing seems okay because of the report of MRI you became conscious have nice day"
},
{
"id": 113151,
"tgt": "Upper left back pain. Have hiatus hernia, weight in mid abdomen. No help with Omeprazole. Suggestions?",
"src": "Patient: Hi - I have been getting upper left back pain for quitea while now - I can t sleep on my back as I feel as if there is something there. I have a 2cm hiatus hernia (diagnosed about a year ago) am about 15 stone , with most weight in mid abdominal region. Was given omeprazole but no benefit to back pain. This now constant. Sometimes helps to eat, sometimes worse on eating - pleass help !! and thanks Doctor: Hello. Thanks for writing to us. The pain in your back can be a referred pain from the hiatus hernia. Some life style modifications like avoiding oily and spicy food, sleeping with head end of the bed raised and taking omeprazole can help in relieving the symptoms. I hope this information has been both informative and helpful for you. You can consult me again directly through my profile URL http://bit.ly/Dr-Praveen-Tayal Regards, Dr. Praveen Tayal drtayal72@gmail.com"
},
{
"id": 175532,
"tgt": "Is CBC advisable for a child diagnosed with Coxsackie virus?",
"src": "Patient: Alright How can a child diagnosed with Coxsackie virus get a cbc before the virus is gone? Isn t the pediatrician suppose to wait for the virus to be gone before ordering a complete blood count? Because the levels he in fact knows will seem to be alarming but in truth false alarming. Then send the child to a hemo without any treatment for the virus now the hemo doctor has only the present information without being told of the virus diagnosis jmml. How can the hemo diagnose a small disease and treat it with 6mp a know chemo that causes leukemia for long periods of time. Doctor: hi dear,your problem is two fold. first, doctor has not discussed with you in detail and second is the disease JMML you mentioned. coxsachie virus is less of problem and mostly the infection is self subciding. i would be more concern of JMML told to you. its a type of blood cancer and needs full battery of test for diagnosis.is you hematologist is just suspecting it? otherwise discuss with him again. giving 6 MP means he has some strong basis.if it turns out to be JMML,proved then you should discuss the treatment option with your hematologist."
},
{
"id": 133838,
"tgt": "Could bump above right eyebrow due to accident indicate broken bone?",
"src": "Patient: Is it possible I have broken a bone right above my left eyebrow. Three evenings ago I apparently walked in my sleep and walked straight into the mantle of ou fireplace. Right above my left eyebrow is a bump that hurts horribly and my upper part od nose was hit too. I keep getting headaches. Doctor: hi,thank you for providing the brief history of you.A thorough neuromuscular assessment is advised.As mentioned in your history about the pain over the eyebrow and nose, I feel you may have hit the bone and irritated the sinuses which has lead to sinusitis.Applying ice with gentle massage post that in that area will help you ease of pain. The headache you are getting is related to the sinusitis so it will subside as the inflammation from the sinuses goes down.RegardsJay Indravadan Patel"
},
{
"id": 100073,
"tgt": "Suggest treatment for allergic reaction to pine nuts",
"src": "Patient: Yesterday I had an anaphylactic reaction to pine nuts. I realized as a young adult that I was allergic to them. Yesterday was by far, the worse reaction ever. Swollen face, tongue, and a restricted airway, as well as severe itching all over. It was also the first time I had to have a shot of epinephrine. I also had 2 steroid shots, as well as dyphenhydramine. I was told to continue Benadryl for 3 days. My question...what should I expect over the next few days? Of course, I m tired, but have a terrible headache today. My face is also flushed and hot. Thanks so my much! Doctor: Hello,Thank you for asking at HCM.I went through your history and would like to make suggestions for you as follows:1. The best management of known allergies causing anaphylactic reactions is avoidance of the food you are allergic to. Unfortunately there is no 100% \"curative\" treatment for such food allergies.Hence my humble suggestion to you would be avoidance of pine nuts in all forms. Please also avoid those food items which contain pine nuts as ingredients.2. As you have a serious anaphylactic reaction requiring epinephrine, future management plan would be to keep a epinephrine pen along with you for unanticipated anaphylactic reactions. It can be life saving in such accidental unanticipated reaction. Please consult an Allergist who will prescribe you such a pen for self-use and also teach you how to use it and when to use it.Hope above suggestions and information will be helpful to you.Should you have any further query, please feel free to ask at HCM.Wish you the best of the health ahead.Thank you & Regards."
},
{
"id": 120056,
"tgt": "What could be the cause pain and tightness in arm?",
"src": "Patient: i have left humerus pain radiating down to forearm and a tightness to hand.i have had an mri of neck and i had some pathology but was told it was mild. i am now going to have a shoulder mri to r/o rotator cuff tear or other pathology. now i have spasms to flank hips and calf? what could be wrong with me? i work in the operating room and i am getting very frustrated.i am 55yrs old and have been employed for 33yrs. Doctor: Hello, Your symptoms may be due to cariogenic, First consult a cardiologist or Physician for proper investigations like Blood pressure checking, ECG, 2D Echo. Hope I have answered your query. Let me know if I can assist you further. Take care Regards, Dr. Krishnakanth. K"
},
{
"id": 214569,
"tgt": "Suggest home remedy to remove fluids in ears",
"src": "Patient: I have A LOT of fluid on ears. Iam taking veramyst, singular, zyrtec, astepro, teklar, and prednesone by doctors orders. Is there any home remedies I can d to speed my recovery...like perxiode, alcohol, or holding my nose while blowing to pop my ears? Any info will help..going to disney wold this week and need relief. Doctor: HiThe only home remedy i can think of is using a ear bud to clean the fluid off, which is not exactly a solution to your problem.When you say fluid in ear, do you mean to say you are prone to infection or common cold or ear wax? In any case i would recommend you two things -1. Septlin Syrup or Tab (Himalaya Pharmaceuticals) 2tsp/ 1 tab - thrice a day - after food.2. Sarivadi Vati (Patanjali Pharmacy)) - 1 tab - thrice a day - after food.Note : This is not a replacement to your prescription from doctor but a natural supplementation, which can be consumed alongside.You should be able to see good results in a matter of ten days or so. I hope this has been helpful.With warm regardsDr Rashmi Kamath"
},
{
"id": 104838,
"tgt": "Severe rash, itching inside arms, around arm pits, torso sides. Is it allergic reaction?",
"src": "Patient: Hello, I am wondering if I should go and see a doctor. I am staying with my inlaws right now and as soon as we arrived 6 days ago I got some sort of rash on the inside of my arms. It appeared after I had a nap on their fold out bed. Although I dont seem to have any allergies I do often have itchy eyes, a runny nose and sneezing when I sleep there. Now I dont have any of those symptoms but I do have the rash which itches intensely and it seems to be getting worse. It now looks like I have mosquito bites all up the inside of my arms, around my armpits and up the sides of my torso. My back is also itchy as well as my tummy and under my breasts but there are not as many visible spots. Does this sound like an allergic reaction or could it be something contagagious? Should I go to the doctor since I have had this for 6 days? Besides the itching I am not really affected. Thank you Doctor: Hello, Thank you for writing to us. House dust mite allergy is a likely possibility given the itchy eyes, runny nose and sneezing. Urticaria can happen (itchy hives) sometimes due to the dust allergy but usually unrelated. You will need skin prick tests or specific IgE blood test to find out what your allergies are (aeroallergen panel). Long acting antihistamines like cetirizine 10mg or fexofenadine 180mg plus flixonase nasal spray 2 spras/nostril twice daily is recommended for severe house dust mite allergy. You will need to take both for at least 2 months to have a good effect. The antihistamines will also work for the urticaria. I hope that was useful. Best Wishes."
},
{
"id": 127914,
"tgt": "What does burning aches between the shoulder blades indicate?",
"src": "Patient: Hello my wife has hypothyroidism and the last 3 months has a developed an almost constant pain that kind of burns and aches a little above center between her shoulder blades ... any thoughts! She has been on thyroid meds since she was 26. She is 61 now . Thank you for a response in advancr ... Doctor: Hello,Do a serum TSH and give her tablet Gabapentin at night once daily for 10 days. There may be dizziness after first dose.Hope I have answered your query. Let me know if I can assist you further.Regards,Dr. Saptarshi Bhattacharya"
},
{
"id": 118130,
"tgt": "What is the remedy for blood clot in my lung caused by c section?",
"src": "Patient: i went to the dr for originally what i thought i still had a blood clot in my lung due to my c section when i went they decided to do a chest radiograph and they suggested for me to go to a cardiologist and do a cardio echo and cardio mri are there cardiologist that do both or do i need to go to a specific dr for one then go to another i have no clue what i am doing Doctor: Hello,Your Cesarean section was probably done under spinal anesthesia and any blood clot formed should have been around the sectioned area. If you are talking about a thrombus (blood clot) formation inside the blood vessel which has moved to the lungs (embolism), then the chances are very rare. Usually thrombus occurs in surgeries in long post operative immobilization or prolonged bed rest. Even if it has occurred it usually dissolves over days. If the thrombus were to have been big, it would have caused obstruction symptoms resulting in difficulty in breathing.Your complaint of blood clot to the cardiologist must have made them to suggest and echo and MRI. However, if you want to get it done, it is done by a radiologist in most of developed scan centers. You can make an appointment and meet. I would usually suggest to wait and watch as the possibility of blood clot in lungs is low, if you do not have any breathlessness. Hope I have answered your query. If you want to ask any further questions, will be happy to help again."
},
{
"id": 28349,
"tgt": "Is it alright to exercise inspite of my heart ablation?",
"src": "Patient: I have. Few electrical issues with my heart . Have been a ablated and things are a little better . My efraction is between 35 and 40. With that said, when I exercise I have a sustained heart rate of between 210 and 230. Am I doing more harm than good? Doctor: Hello! Thank you for asking on HCM! You need to be very careful and to avoid excessive physical exertion, facing your reduced LV ejection fraction. I would recommend just waking and continuing to be active that way, but no exaggerated strain. If you experience such high heart rates again, that you need to see your attending cardiologist for a comprehensive evaluation, and define a possibility of ICD implantation (if life threatening arrhythmia results).Hope to have been helpful to you. Greetings! Dr. Iliri"
},
{
"id": 64700,
"tgt": "What causes a lump above eyebrows?",
"src": "Patient: I have a lump above my eyebrows, in the corner, when I smile. I am sorry if this isn t a typical medicar question, but I haven t seen it on someone else and I am sure it s not a disease, far from it. Just wanted to make sure it was all normal. I feel ashamed to even explain why I am asking this, but I need to make sure. Is a bulge, a lump (not sure those two are the same, english is not my first language, i just heard those terms) normal to appear when you smile? Does everyone have that (because I didn t really notice it on anyone else)? Doctor: Hi,Dear ,thanks for the query to HCM.Your query doesnt specify if its the inner or outer corner of the eyebrows? / whether one or two eyebrows?That it is a bit in-complete and confused query.Still,the possibilities could be-skin crease coming together could form a lumpy skin at the eyebrow corners / or it could be-sebaceous cyst even.For the exact cause hence I would advise you to consult-a ER-Surgeon.Hope This would solve your query.Wellcome again for any more query."
},
{
"id": 80466,
"tgt": "What is the treatment for chest pain?",
"src": "Patient: Hi I've had chest pains for about 3 hours. I don't know whether its a muscle or heart attack. The only other symptom I have is nausea. The pain is constant but worsens with torso movement. I don't think it's a heart attack but I'm not sure. What should do? Doctor: Hello dear, thanks for your question on HCM. In my opinion you are having mostly GERD ( gastroesophageal reflux disease ). But better to rule out cardiac cause first. So get done ecg. If ecg is normal then no need to worry. Since you have nausea, possibility of GERD is more in your case. It is due to laxity of gastroesophageal sphincter. Because of this the acid of the stomach tends to come up in the esophagus and cause the symptoms like chest pain and nausea. So better to follow these steps. 1. Avoid hot and spicy food. 2. Avoid stress and tension. 3. Avoid large meals, instead take frequent small meals. 4. Start Proton pump inhibitors. 5. Go for walk after meals. 6. Keep 2 - 3 pillows under head in bed to prevent reflux. Don't worry, you will be alright."
},
{
"id": 86942,
"tgt": "Suggest treatment for severe abdominal pain",
"src": "Patient: I am a 67 year old male and ever since yesterday have been experiencing pain from front ta back of my adomenal area, it does not hurt to touch but is a persistent on the inside, when slight pressure is applied it seems to give it a little relief. On a scale of 1 through 10 I would rate pain level as 10, did not sleep well due to pain Doctor: Hi,From history it seems that there might be having abdominal colic pain mainly of renal colic.If there is no renal symptoms like frequency of urination or burning micturition then it might be having intestinal colic.Take antispasmodic medicine like Meftal spas or Cyclospam to give symptomatic relief.Take plenty of water.Take light diet.Ok and take care."
},
{
"id": 186953,
"tgt": "How to treat pin holes and old scars in the teeth?",
"src": "Patient: I have pin holes and old scars from my bad teeth that 10 dentist would not take care of for 20 years my bottom big tooth caped in gold kept absessing for over 20 years. I became very sick I also have tmj now.any way life long story.I know a lot and learned a lot and I am still alive I am 54 the gold tooth is gone now. Only because the dentist was getting ready to retire.After I returned for the 3rd. time I did not take any antibiotics my choice.I am getting help with my teeth finally by wonerful doctor dentist.Is there any thing I can do to heal old uneven skin tissue and pin hole scars? Doctor: Hello, thank you for consulting with healthcaremagic. as you are mentioning , it looks that your tooth was infected and an abscess was formed below the tooth. And when an abscess becomes chronic, it forms a sinus opening, which drains through an opening.In your case, as now the tooth is removed, the abscess will heal as well as sinus opening will get closed. If not then you should visit an oral surgeon to get diagnosed with the condition and even get stitches done.Hope it will help you."
},
{
"id": 148360,
"tgt": "Sciatic pain due to pinched nerve in back leading to lot of inflammation. Medicines not helping",
"src": "Patient: I am having a sciatic pain for the las 3 Months i went to a chiropractic and i was told a disc in my back pinched a nervs and i had a lot inflamtion i had taken inflamation medicine is got better but it does not go awy and i can not live with this pain anymore i have not had a good nite sleep in 3 months please help do not what to do Doctor: Dear Sir , Well looking into your problem it is advisable to you to consider surgical option for decompressing the nerve. . Dr. Shruti"
},
{
"id": 205803,
"tgt": "Suggest treatment for depression and anxiety",
"src": "Patient: I DONT GET HUNGRY I CAN GO DAYS WITHOUT FOOD, I TAKE A COUPLE OF BITES AND I FEEL LIKE A ATE WHOLE MEAL,I DONT FEEL HUNGER SO I FORGET, WHAT REMINDS ME TO EAT IS A REALLY BAD HEADACHE AND DIZZINESS THEN I FAINT, NOW IAM GETTING DEPRESSION AND AXIETY MAKING IT WORSE Doctor: Hello thanks for asking from HCMYou have symptoms of poor appetite associated with headache, dizziness and these symptoms has resulted in depressive symptoms and anxiety. Some times due to underlying depression individuals develop reduce appetite and which further aggravates depression and anxiety. Consult a psychiatrist for treatment and evaluation of your symptoms.For anxiety and depression medicines of SSRI class like Paroxetine or other antidepressants like Mirtazapine etc can be used. These medicines will help in improving appetite also and would reduce your symptoms of depression and anxiety. Visit a psychiatrist for prescription of the drugs.Thanks, hope this helps you."
},
{
"id": 209991,
"tgt": "Is mood change frequently a symptom of bipolar depression",
"src": "Patient: My husband easily nervous and agressive lost control become violented by words i react another way to make calm his mood keep changing im thinking maybe his brain is psycholigac disturbed cannot concentrate.I explain him peacefully but his mood keep changing always .I like to know if he is in bipolar depression he lost a job and feel insecurity cos i do all my work.Please give me advice what will i do to him to help shall i bring him psychologist doctor to make him calm. Doctor: Hello,Thanks for choosing health care magic for posting your query.I have gone through your question in detail and I can understand what you are going through.Yes it could be a symptom of bipolar depression but this will require a detailed assessment by a psychiatrist to confirm the same. This illness will require him to take valproate or carbamazepine to make his mood stable and this will start showing improvement in 2-3 weeks. Hope I am able to answer your concerns.If you have any further query, I would be glad to help you.In future if you wish to contact me directly, you can use the below mentioned link:bit.ly/dr-srikanth-reddy\u00a0\u00a0\u00a0\u00a0\u00a0\u00a0\u00a0\u00a0\u00a0\u00a0\u00a0\u00a0\u00a0\u00a0\u00a0\u00a0\u00a0\u00a0\u00a0\u00a0\u00a0\u00a0\u00a0\u00a0\u00a0\u00a0\u00a0\u00a0\u00a0\u00a0\u00a0\u00a0\u00a0\u00a0\u00a0\u00a0\u00a0\u00a0\u00a0\u00a0"
},
{
"id": 211565,
"tgt": "Severe back problem since teens. Disturbed. Consult psychologist?",
"src": "Patient: I believe i was raped as a very young child and have no memory of it. i am now 63. should i seek psychological he i have had major lower back problems since a teen. i never bled during my first sexual experience. all i remember is always hiding in a closet and my parents told me i would never go to sleep and walk around the house at night a lot my parents have never talked of anything personal with any of their 5 children and whenever doctors have asked meif i was hurt when i was a child, i asked my mother and she said i was very sick at one time and thinks i had scarlet fever Doctor: Hi,Thanks for writing in.It is possible that you are remembering memories of younger childhood days and this is causing distress right now. Some of it is normal and acceptable but if this is causing significant distress and anxiety in your daily life, then you need some help. Help can be in the form of keeping yourself preoccupied and making friends. You may share your thoughts with old friends or join support group for the elderly.If there are facilities near you home, you can seek help form a mental health professional and take counseling sessions from a psychologist. You can also start new hobbies and avoid bad memories and thoughts into your mind.Hope this helps"
},
{
"id": 34341,
"tgt": "Should i get a tetanus shot after a nail prick on foot?",
"src": "Patient: we recently tore down an old shed in our backyard. while moving pieces of wood across the yard i stepped on a nail in one of the boards. some of the nails are rusty and some are not, i do not know which one i stepped on. there was barely a prick on the bottom of my foot. hardly any bleeding i almost didnt realize there was blood. i cleaned my foot and doused it in peroxide, and put a band-aid on it. i am 21, female, approx 250 lbs and have no clue if i have recieved a tetnus shot in the past 10 years. what should i do? Doctor: Hi,Injury by a nail, rusted or not, injection of Tetanus toxoid is required.As you have no idea about this shot in the past, you have to go for Tetanus shot.Ok and take care."
},
{
"id": 106487,
"tgt": "What can I do for severe back pain?",
"src": "Patient: I m having severe back pain in kidney area, went to family doctor today, they ran labs, said they should be back in about 2 hours. They sent me home saying that they would call me with the results. I never heard anything back. I have done as instructed, drank plenty of water and used heat on back. My back continues to hurt very bad and now seems to want to radiate over onto my right side to front of stomach. Could it be serious at this length of time? Doctor: Hi, Pain over the loin area might be due to renal calculi (Calculus/stones in kidney/ureter) mostly. An ultrasound scan is mandatory for this situation. Don't worry much but be careful. Early diagnosis will help you to get better fast. Hope I have answered your query. Let me know if I can assist you further. Regards, Dr. Ajaygupta009, General & Family Physician"
},
{
"id": 189987,
"tgt": "Pain in toe, groin, leg, lower back, teeth keep chattering. What is going on?",
"src": "Patient: Hi, yesterday I woke up feeling fine went to work as normal then at mid day I got a sharp pain in my top of my big toe on my right leg the pain went up my right leg and into my groin then traveled round to my lower back the pain was so intense I went straight to the doctor he told me I just had a pinched nerve in my lower back and gave me some strong anti inflaming drugs and told me to rest I ve been In the most intensive pain all night my teeth won t stop chattering and my body is dripping wet please help Doctor: Grinding of teeth at night is known as bruxism. It can occur because of stress or anxiety or can occur because of traumatic bite present. It causes soreness of jaw, headaches, sensitivity of teeth etc. Teeth grinding can cause wearing of the tooth structure which may involve dental pulp and cause soreness of tooth. If dental pulp is also involved due to tooth wearing, then you will require root canal treatment. You have to visit your dentist to get your teeth evaluated. Your dentist may take an x-ray to see that whether dental pulp is involved in sore teeth or not. Its better if you get a custom made mouth guard from a dentist than using the mouth guard which you got from pharmacy. Mouth guard should be made according to patient\u2019s teeth and alignment of his teeth. Mouth guard absorbs the chewing forces and prevent damage to teeth and surrounding tissues. Main causes of teeth grinding are stress or traumatic bite. You need to reduce your stress levels. If there is traumatic bite present which is causing teeth grinding, then dentist can correct it. By the time, you visit your dentist you can take OTC pain meds."
},
{
"id": 50272,
"tgt": "Have kidney cyst, flank pain, history of renal artery stenosis, family history of renal disease. Second opinion?",
"src": "Patient: I have a 3 cm cyst in my left kidney mid pole with frequent bouts of flank pain (negative for stone ) in addition for the last 3 years there is always blood in my urine ranging from trace to gross amounts, while most kidney cyst are benign there is a history of renal disease in my family and my sister died at 3 months from renal failure , I have been treated for renal artery stenosis and Pyle nephritis, the cyst was 2.5 cm 7 to 8 months ago and my urologist says it s probably a benign cyst come back in 18 months should I get a second opinion ? My dad is in renal failure and my cousin died of liver failure but had renal disease. Doctor: In view of strong family history you can not take the findings to be benign in nature.I suggest you to go for second opinion so that timely evaluation be done"
},
{
"id": 34299,
"tgt": "Suggest remedy for virus in blood",
"src": "Patient: hi Dr., My self Praveen /26 ,having HbsAg+ , consulted Gastrologist and Hapetogologist and as per Dr's view , i am suffering form Virus in inactive stage , where Virus is present in blood but not active by HbsAg. , it may or may not target the liver until unless virus gets activated. so please suggest me that is there any Aruvedic or english medicine for making Virus from + ve to -ve stage in blood ? if so, where can i get good treatment ? Doctor: HiThanks for posting your query. Well definitely there is medication in English medicine which can eradicate the Virus from the body. But in our practise we doctors won't start patient on the medicine unless the virus is active and affecting the liver . In your case the virus is inactive so no need for treatment as of now . Keep checking your Liver function test and also do HBV DNA levels every 6 months . If any of these 2 tests shows high value then treatment can be initiated else its not needed. Note : Treatment for Hepatitis B is for 1-2 years . Its a daily medication. So hence treatment is not indicated in Inactive stage. Hope this information was useful to you. Any clarifications feel free to ask."
},
{
"id": 170195,
"tgt": "What causes light green colored stools in a 6 month old?",
"src": "Patient: hello doctor! my baby is 6 months 5days,she has different color stools......at times her stool is of light green colour......then suddenly after few hours her poos are blackish seeds in yellowish colour stool...m worried about my baby ...can u suggest what is wrong with my child.my baby weight is 6kg 400gm,her height is 65 cm Doctor: Hi, Welcome to HCM. I know that you are very concerned about your child but don't worry. A baby who is preferably on milk diet may sometimes pass green, yellow, golden and brown coloured stools. This is a normal thing. The most important point to remember is that the child should not be in distress and should be feeding normally. I hope this will help you. Wishing your child good health. Take care."
},
{
"id": 78530,
"tgt": "Suggest treatment for dementia and COPD",
"src": "Patient: My mother is 95 1/2 and has dementia. She is frail and has copd. The pa now put her on .25 of morphine for pain which we, her children, haven t noticed her having pain. PA says she is having pain so needs the morphine rather than Ativan and Tylenol. She seems to be sleeping more and responding less since on morphine. We are very concerned. thank you Doctor: Thanks for your question on Health Care Magic. I can understand your concern. In my opinion, her current symptoms are mostly due to morphine. Morphine is opioid analgesic. It acts directly on brain. Along with analgesic effect, it causes sedation and respiratory depression. So her more sleeping and responding less is mostly due to sedation of morphine. Another reason is high carbon dioxide (hypercarbia). COPD patients are known to develop hypercarbia if respiratory center in brain is suppressed. High carbon dioxide can cause lethargy, sleepiness etc. So better to stop morphine and consult pulmonologist to rule out hypercarbia. Hope I have solved your query. I will be happy to help you further. Wishing good health to your mother. Thanks."
},
{
"id": 152770,
"tgt": "How to treat swelling of prostate and pain during urination ?",
"src": "Patient: I had Postate Cancer treatment, Brackytherapy in July 2009 all went well untill 2 months ago, prostate swollen painfull to sit, painfull during and after urination, had blood work, stool test, nothing found no infection etc, had urethra scope looked into extremly painfull I cryed out had to lay down to allow nausia to pass. I take Lotrel for hi-blood pressure, viagra and vacuum system for intercourse. We tryed food alegery (acidic foods) Avodart, Uraxatrol, Proposed, Pentoxfylline with no results. Currently on Elmiron, Rapaflo, Phenazopy. Doctor: Thank you for your question. Yes painful urination following prostate cancer treatment is not uncommon however should be investigated well.1. urinalysis confirm if there is infection.2. CT scan and or ultra sound of pelvis to check if there is regrowth of prostate cancer.3.PSA level 4. Full haemogram to check for amount of blood and bacterial infection.Treatment Will depend on the results. Consulting urologist and oncologist will be helpful."
},
{
"id": 122027,
"tgt": "How to determine the bruise on the rib bone after falling?",
"src": "Patient: How do you know if you have a bruised rib, i fell on the driveway on my left side and have pain under armpit towards breast I have no bruising or swelling. Pain is about a 7 and is still there after taking 600mg of ibuprofen,pain is a 4 after the medication. Doctor: Hello, The symptoms seem to be related to the injury. I suggest to do a chest X-ray to check for a fracture. Meanwhile, I suggest using anti inflammatory medications such as Ibuprofen to relieve the pain. I also suggest to maintain the area immobilized to prevent further damage. Hope I have answered your query. Let me know if I can assist you further. Regards, Dr. Dorina Gurabardhi, General & Family Physician"
},
{
"id": 44905,
"tgt": "What treatment should I take to conceive as I am having PCO and thyroid problem ?",
"src": "Patient: hi iam 29 years old i had miscarry at 15 weeks of pregnancy in 2010 april and now iam suffering with PCO on both ovaries nd had thyroid prablm also now iam trying to consive past 1 year please advise me what treatment hav to take Doctor: you will have to take treatment for PCOD & thyroid problem,when they both are control, you will be able to con concieve."
},
{
"id": 132452,
"tgt": "What causes acute rib pain?",
"src": "Patient: I am suffering from pain all over especially my ribs. I have had abdominal ultra sounds tons of blood work all came back perfect. My doctor wants to do a hide a scan she is convinced it is my gall bladder. I have tremors with knife stabbing pain everywhere. Please if you can give me any advice. It is destroying my quality of life. Doctor: Hello thank you for writing us here. Such pains on \"ribs\" are felt but to inflammation of gall bladder or due to inflammation of cartilage of bones, commonly called costochondritis. The treatment of both is different. You should get yourself checked and examined properly with Proper investigations. Ultrasound abdomen and x ray with CBC.Best regards,Dr Gunjan"
},
{
"id": 108182,
"tgt": "What to do for severe back pain?",
"src": "Patient: I have a severe back ache problem, started with a neck pain 2 years back. I got an MRI done and it did not show anything much, slight compression in C4-C5, which my doc says wasnt a worrisome problem. I went for physio and then managed with the problem somehow. 8 months back, I had a sudden pain in my lower back also, when I saw the doc, he advised bed rest and asked me to take an X ray. The Xray was normal, infact he told me the curvature showed a healthy spine. he then advised blood tests, and that showed my Vitamin D was almost zero. He then put me on Vit D medicine and shots and I did get a lot of relief. The pain was more or less under control with some intermittent spurts once in 2-3 months. Now, the pain has come back and both upper and lower back. My vitamin d is normal now and Myoril has stopped acting on my system now. Also, there is pain in hands and legs sometime. pls advice Doctor: HelloAs per description of your clinical symptoms it seems that you have Radiculopathy due to nerve compression ( Presently at the cervical region ,c4-c5). symptoms for this are same as described by you, but may worsen if not taken care of.If I were your treating doctor I would have suggested you the following:1. please take necessary precautions to maintain a good posture for your spine. you may even use cervical coller.2. Get a MRI screening of your whole spine with brief descriptions of cervical and lumbo-sacral spine, to rule out any disc compression or worsening of the cervical compression.3. Also get a report of vitamin D levels in your blood.Depending on the investigation reports it could be decided whether you need a conservative management with exercises and supportive medications or a definitive management by surgery ( decompression/discectomy/root block etc).For now, please avoid bending forward, lifting heavy weight, sitting or sleeping on the floor, squatting. Instead I would suggest you to use a firm bed and flat pillow to sleep, and always sit with back rest.Thank you."
},
{
"id": 198601,
"tgt": "What causes leg pain when suffering from epididymal cyst?",
"src": "Patient: i have a epididymal cyst , have recently met with a surgeon who confirmed this is not harmful and was advised to have a urine microscopy for testing infection , which turned to be not infected. Was advised with painkiller sumo course as a pain killer after which issue with cyst pain was gone . it still pains once in a while , during wrong positioning of legs in sleep which may put some pressure on it . Is it a matter of concern ?? is this a normal behavior with cysts on epididymis ?? do we have any non surgical cure of it ?? Doctor: HelloThanks for query .You have been diagnosed to have Epididymal Cyst (Spermatocele) and having pain in leg occasionally .The epididymal cysts are benign in nature and does not any treatment unless it increases in size or gets infected.The pain in leg that you have is not due to Cyst per se but may be due to other cause which needs to be traced out .There is no medicine that can cure or resolve the Spermatocele .Dr.Patil."
},
{
"id": 70103,
"tgt": "Should I be concerned for swollen lumps in both sides of groin?",
"src": "Patient: I have swollen (non visible) lumps in both sides of groin, larger on right side and painful when pressure applied, GP said swollen lymp nodes and not to be concerned but I am as I have no infection except the common cold, dr said they are mobile...any help appreciated, thank you Yvonne Doctor: I. Consult a Surgeon , go for a course of an antibiotic and anti-inflammatory medicines as advised and go for FNAC = fine needle aspiration cytology reporting to rule out a bigger problem an a small nodes. This site nodes are grown only on a reason ."
},
{
"id": 69958,
"tgt": "What causes lump in thigh after an accident?",
"src": "Patient: hello, i had a fall about 3 weeks ago off my motor bike. i went to the hospital and had several xrays they have said no broken bones, the external bruising was very bad but faded away now but the lump on my right thigh is big and painful, should i ask for a second opinion or leave it a while longer??? Doctor: Hello!Thank you for the query.This lump is most likely a blood collection in the soft tissues (hematoma). Due to some veins rupture and bleeding blood has appeared outside the blood vessels. Large hematoma wont go away by itself. Sooner or later it will become infected and turn into an abscess.That is why I suggest you to do not wait with it and consult a surgeon. Blood should be drained.Hope this will help.Regards."
},
{
"id": 137791,
"tgt": "Suggest remedy for muscle spams with twitching in arms",
"src": "Patient: Hi I ve been experiencing daily head and ear pressure , lots of mucus in throat and my throat feels like it s closing. Also body aches, twitching in arms and legs and muscle spasms. Fatigue. I ve been tested for allergies and had a catscan for sinuses and all clear. Seen an ent multiple and he s saying anxiety but that s not the case Doctor: Dear Patient,your history looks like sinus involvement, but you say your ent doctor has ruled out any pathology. there, so there are two options, either you see some other ent doctor or you can take an opinion from a neurologist. I hope that helps you ,Thanks"
},
{
"id": 72289,
"tgt": "What causes breathlessness after eating or during stress?",
"src": "Patient: hi i have high cholesterol levels for which i am taking fibator 10 and aztor 10 .also i have fatty liver for which i am on udiliv 300 . due to stress i am having nexito 5 mg . for sometimes now i have having light breathlessness specially when my stomuch is full and also at the time or over stressing .i need to know your views sir.i am taking fibator for 2 years but aztor for 3 months as my trigyceride was nearly 350 .please guide Doctor: Thanks for your question on Healthcare Magic.I can understand your concern. Stress causes GERD (gastroesophageal reflux disease). And GERD is worsening by heavy meals.So your breathing difficulty is mostly due to stress and GERD.For stress, consult psychiatrist and get done counselling sessions. Counselling sessions along with anxiolytic drug (nexito) is beneficial for stress management.For GERD, avoid hot and spicy food, avoid junk food, avoid large meals. Take pantoprazole tablet on empty stomach twice daily. Quit smoking and alcohol if you have these habits. Don't worry, you will be alright with all these. Avoid stress and tension, be relax and calm. Hope I have solved your query. I will be happy to help you further. Wish you good health. Thanks."
},
{
"id": 156381,
"tgt": "Suggest remedy for stage 4 oral cancer",
"src": "Patient: hello my father has oral cancer and its of 4 stage he has recieved 3 chemotherapy sittings 2 operations and 1 month radiation treatment .he has also recieved target treatment and now he is undergoing ozone therapy but still has 5 tumors on his face age43 help him Doctor: Your father has received almost all possible treatment modalities available. At the moment only palliative treatment looks possible. If the tumors on his face are resectable then surgery should be done. Otherwise palliative chemotherapy should be given. For details regarding the chemotherapy options previous chemotherapy given, current performance status of patient, age of patient and baseline lab investigations would be required.Please discuss it with the treating oncologist and if you have any further query I would be more than happy to answer."
},
{
"id": 194374,
"tgt": "Treatment for gynecomastia",
"src": "Patient: Hello, i have a problem. I am 14 and i have gynecomstia.. i think. I workout and i am not over weight i have very strong chest muscle and i do cardio and exercise and of course i eat healthy. I was told before that my hormonal levels are alright by a doctor, what should i do nothing i do works i heard surgical removal is an option if so how much does one cost Doctor: Hello, If you are already doing your exercise. Keeping all your hormones at a normal level. It can be a genetic and surgical option is the way. Hope I have answered your query. Let me know if I can assist you further. Take care Regards, Dr S.R.Raveendran, Sexologist"
},
{
"id": 33908,
"tgt": "What are the symptoms of UTI and what medicines should i take?",
"src": "Patient: im 23 y/o, 4\"11, 43 kgs, i had UTI when i was pregnant 1 year ago.do i have UTI if my urinalysis results are these color: yellow, transparency: hazy, reaction: acidic, ph: 6.0, spec gravity: 1.005, rbc 0-1/hpf, pus cells: 0-3/hpf, squamous: few, bacteria: few, mucus threads: few, others are negative. what medicine do i need to take? Doctor: Hi,Welcome to healthcare magicThanks for writing. I am Dr. Saddiq ul abidin. I have your question completely, I understand your concern and i will try to help you in best way possible.For UTI to be labelled on a urinalysis, the most significant finding is the presence of pus cells or WBC, in more than 5-8 quantity/HPF, though a clear cut demarcation has not been drawn, because associated history of burning micturation and urinary symptoms like urgency, frequency and fever are also given due importance. Other associated findings are leukocyte esterase to be positive, while in certain nitrogen fixing bacteria, presence of nitrite may also be a clue. In severe UTI's or the ones caused by stones or obstructive uropathies, RBC may also be significantly raised. The presence of bacteria alone, indicate contamination of the sample mostly.The detailed analysis report which you have shared is suggesting however the presence of an acidic urine. For which i would have suggested you, the use of some alkylating agent like Citrosoda sachet twice daily for at least five days, if you were my patient.If you are experiencing the symptoms like the ones i have mentioned above, you can however have a urine culture to be sure and then treat the infection if the cultures turn out to be positive, and you can let your treating physician chose by the sensitivity spectrum according to the culture if positive.I hope this answered your question, If you have more queries, i am happy to answer, otherwise rate before closing the discussion. RegardsDr. Saddiq ul abidin M.B.B.S ( Licensed Family Physician)Resident Medicine"
},
{
"id": 82115,
"tgt": "Suggest medication for severe cough and itchiness in throat",
"src": "Patient: I am 26 weeks pregnant .I am having severe cough(this is been troubling me for almost 3months) with phlegm and itchy throat at night.What is the solution?I have tried lemon & honey, inhalers, lozenges , herbal cough powders, hot water etc etc nothing helping me how can I stop this cough? Doctor: Thanks for your question on HCM.In my opinion you are having upper respiratory tract infection (URTI).Try to follow these for symptomatic relief.1. Avoid spicy and oily food.2. Warm saline (water) gargles 5-6 times a day.3. Start cephalosporin group of antibiotic. it is safe in pregnancy. 4. Start antihistamines twice daily.5. Paracetamol twice daily.6. Avoid cough suppressant containing codein.7. Drink plenty of fluids."
},
{
"id": 158154,
"tgt": "Suffering from liver cancer, hapetits c. Tumor removed. What will be the cost for the second surgery?",
"src": "Patient: Hi doctor.....actuallu my father is suffering from liver cancer n their liver is also80% damage ......he also have hapetits c and he is having canada PR but no medical insurance.....me want to know how much it expensive their treatment in canada n he has already one surgery of removing tumor in india.......if secind surgery is to be done how much its cost......he is no medical insurance....thanks doc Doctor: Hi welcome to health care magic forum.Ther cost is very high and it is not advisable to have liver resection without insurance. this is complex surgical procedure and the outcome is hardly predictable considering his liver condition. In most countries malignant diseases and treatment should not be charged but check it with doctors in your place.Hope I have answered your query. Wish you good health. regards"
},
{
"id": 77567,
"tgt": "What causes pain under shoulder blades while breathing?",
"src": "Patient: Hi yes I have a concern, I keep having a sharp pain just under right shoulder blade when I inhale, just started this morning. I am 35 5ft tall and weight 110lbs. I currently have cold symptoms runny nose headace lack of energy? Any ideas? Thanks! Doctor: Hi,Dear,Thanks for your query to HCM.Dear I read your query and reviewed it with context to your query facts.I understood your health concerns and feel Concerned about them.Based on the facts of your query, you seem to suffer from-FibromMyalgia with Inspiratory Catch from Flue and Exertion.Tab Motrin x for 5 days would resolve this issue.Hope this would help you to solve your health issues in the best way possible.Welcome for any further query in this regard.Good Day!!Dr.Savaskar M.N.Senior Surgical SpecialistM.S.Genl-CVTS"
},
{
"id": 184200,
"tgt": "What causes excessive salivation?",
"src": "Patient: iWHAT IS THE REASON FOR EXCESSIVE SALIVATION IN HUMANS i AM 82 YEARS OLD, 6FT TALL, 16STONE, iT HAPPENED AFTER MODERATE EXERCISE IN THE GYM WHICH I ATTEND ON A REGULAR BASIS. B EING DIABETIC I RECENTLY STARTED ON 2 METFORMIN PER DAY. your advice will be very welcime Best regards Fane MURRAY Doctor: HIThank for asking to HCMI really appreciate your concern and let me tell you that hyperacidity, sick feel, nausea, and some anti-psychiatric drug causes the excessive salivation, and these needs to be ruled out separately, proper treatment of underlying cause would subside this symptom, take care and have a nice day."
},
{
"id": 63539,
"tgt": "What is the treatment for a lump in the face and headaches?",
"src": "Patient: About six months ago i got a ball to the face, the doctor said it was a concussion, i have been getting headaches ever since. I woke up 2 days ago with a very bad headache and two very bloodshot eyes,I'm having a very hard time focusing on anything. I know the redness is not pink eye. Could this be related to my concussion? I'm 16 Doctor: HI,Dear,Thanks for the query regarding your daughter.I studied your query and understood your health concerns.Remedy-You seem to have raised Intra Cranial Tensions with ? SOL-Space Occupying Lesion,which may be post-trauma by ball on face 6 mths back.Consult your ER Surgeon,who would check and do the needful treatment after investigations with Neurosurgeon.MRI of the brain would fix your prolbem.So Don't worry and act fast as suggested.Welcome for any query to HCM and me in this regard.Have a Good Day.Dr.Savaskar M.N."
},
{
"id": 188956,
"tgt": "Taking Previ Dent for loose bridge. Have rash on inner thigh. Side effects?",
"src": "Patient: I have been using Previ Dent 5000 for over a month for a loose bridge that will soon be fixed. I broke out with alarge very warm pink patch on my inner calf six days ago. I was painful and sore and I went to the doctors and wastreated for inflammation using Ibuprofen twice a day. Today it moved to my inner thigh in a larger blotch. I didn'tthink at the time it could be related to the Previ Dent but could it?I noticed while reading about the side effects that you should only use this for a period of 4 weeks. My dentist said itwas o.k. to continue using it until the bridge is fixed. What do you say? Doctor: hello and welcome to HCM forum,I would like to inform you that Prevident 5000 does not cause skin rashes, I do not feel that the red patch is from Prevident, but I will recommend you to see a dermatologist first.It is okay to use prevident 5000 for a month and then wait for a week, after which you can use it again. I will suggest you to give a break for a week and start using it again.I hope this answer was useful to some extent,Thank you for writing to us,i wish you good health,take care."
},
{
"id": 198238,
"tgt": "What could cause inflammation of Meatus, have recently had unprotected oral intercourse?",
"src": "Patient: Hi,I just recently noticed this morning that my meatus is a little inflamed. I went to a local urologist and he prescribed antibiotic ointment as a precaution and I requested (he did not require) a urine test as I had received unprotected oral sex a week prior. I am circumcised, masturbate quite frequently and am in asia currently. As I do not exactly have the highest confidence in the local doctors, I was wondering what diseases, if any, I am at risk of having and if the urine test should pick them up. Thanks. Doctor: HelloThanks for query .You have noticed your external urinary meatus to be inflamed after having a oral sex .The oral cavity of human being is habitat for many bacterial flora hence it is very common to get infection of genitals after receiving oral sex .You need to take broad spectrum antibiotic like Cefixime along with anti inflammatory drug like Diclofenac twice daily.along with topical antibiotic ointment like Neosporin twice daily.Ensure to wash your genitals with warm water twice daily.Ensure to avoid sexual encounters till it heals up completely.Dr.Patil."
},
{
"id": 48882,
"tgt": "Should we try to save an amputated, infectious and failing kidney patient?",
"src": "Patient: patient with toes amputated, infection returned, vascular disease, diabetic, kidneys failing, frail condition, family chose not to have more surgery, withdrawn all helping medications, only giving pain medications, withdrew insulin pump, no feeding tube as patient can no longer swallow-Should more been attempted to preserve this life, or slowly killing patient as it appears \"merciful\" Doctor: Hello!While it is hard for me to comment without knowing the specifics in this case it is a classical medical dilemma care givers face the world over. If the patient is not very old ( above 80 yrs) my view is to give it everything you've got- the full gamut of medical, surgical & supportive therapy and then let the 'body' decide what it wants to do - stay on or go.Best of luck!"
},
{
"id": 19028,
"tgt": "Suggest suitable remedy for a heart disease",
"src": "Patient: Had mechanical valve 25 yers ago at age 37...having it replaced in a week and aortic aneurysm repaired. Trying to decide on tissue or another mechanical valve....my body does not tolerate coumadin..very frequent protimes...two or three times monthly for 25 years. I am now 62...any advice? Surgery to be done at Duke. Doctor: Hello,A tissue valve lasts for some years and needs to be replaced again, probably 10-15 years. Mechanical valve lasts much more, but need to take coumadin. You are 62 (young), so if I were you, I would choose mechanically.Hope I have answered your query. Let me know if I can assist you further.Regards, Dr. Anila Skenderi"
},
{
"id": 68839,
"tgt": "What is the hard lump on the collar bone?",
"src": "Patient: hi! im 52 years old male and a smoker for 30 years, it seems like my collar bone has some sort of hard lump, like the bone poked out but no pain, is this might be related to lung cancer or c.a.d (coronary artery disease). please help, thank you so much Doctor: welcome to Health care magic.1.Lung cancer is one of the cause for the mass at the collar bone.2.Or any bony lesion arising from the collar bone itself.3.An x-ray chest / apicogram initially to evaluate the lesion.4.depending on the involvement CT could be suggested.5.Yes even arteries cause lump but they are not so hard. ( an ultrasound will help in case of soft masses)6. However get an appointment and see your doctor and do needful investigations.Anything to ask ? do not hesitate. Thank you."
},
{
"id": 1137,
"tgt": "Will i get pregnant due to PCOD problem?",
"src": "Patient: Hi this is akshaya 26 years old. Got married before 3 Years. I have PCOD Problem taking OBIMET 500 MG. getting Regular periods for more than a year. this month discontinued OBIMET and Taken OVACARE AND FOLIC ACID TABLET ovulation not happened and not ruptured till 20th. Doctor adviced me to continue with OBIMET and folic acid Tablet. After 2 Months she will give some Tablets to STimulate. Actually want to know whether will i get preganant Doctor: Hi, I think you can get pregnant. You have to take medicines like clomiphene for the growth of your follicles and track your follicles growth by repeated ultrasound and when your follicles is more than 17 to 18 mm, take injection for rupturing the follicles. Be in contact with your husband for 2 to 3 days after injection. Take progesterone for next 2 weeks. Do a urine pregnancy test at home after that. You can try like that for 3 to 6 months. Hope I have answered your question. Regards Dr khushboo"
},
{
"id": 75940,
"tgt": "What causes chest pain with esophagus burning?",
"src": "Patient: I woke up to crushing chest pain and went to the doctors. They ruled out that it was my heart. I am supposed to take Prevacid when I feel like I need it. I have been taking it for 10 days know 30mg 2x/day. My esophagus hurts/burns continually now. Is it the medicine? What could it be? Doctor: Thanks for your question on Healthcare Magic. I can understand your concern. By your history and description, possibility of GERD (gastroesophageal reflux disease) is more. GERD is due to laxity of gastroesophageal sphincter. Because of this the acid of the stomach tends to come up in the esophagus and cause symptoms of burning esophageal pain, chest tightness etc. You are taking antacid (pantoprazole). Only drugs are not important in this. You need to follow below mentioned lifestyle modifications also for better symptomatic relief. 1. Avoid hot and spicy food. Avoid junk food. 2. Avoid stress and tension, be relax and calm. 3. Avoid large meals, instead take frequent small meals. 4. Keep 2-3 pillows under head in bed to prevent reflux. 5. Quit smoking and Alcohol if you have these habits. 6. Go for walking after meals. Don't worry, you will be alright with all these. Hope I have solved your query. I will be happy to help you further. Wish you good health. Thanks."
},
{
"id": 65440,
"tgt": "How to treat a lump on the ankle?",
"src": "Patient: Hello, I have a lump at the front of my ankle and raised arteries around the area. There is a dull ache around the area which I cannot describe as painful. I am 45, I run and play tennis frequently. Average height and weight and generally of good health. Doctor: hi dear thanks for the query on HCMTHESE SWELLING ARE SUGGESTIVE OF VARICOSE VEIN ON YOUR lower limbs.you need a venous doppler of the lower limb and a consultation with a surgeon/ vascular surgeonthank you"
},
{
"id": 110941,
"tgt": "What causes pain in upper right side of back on movements?",
"src": "Patient: If I have had two chest xrays in the past four months that were normal, and two complete blood counts, including metabolic panel...and my organs ultrasounded...but still have a pain when I move a certain way on the back upper right side of my back....where my liver would be....that I have had for two years...doctors keep saying it is muscular...but it never goes away and I am a ballet dancer...I know my body well...should I see a doctor? Doctor: Hello, Thanks for your query.The fact that your pain is occuring with movements indicates a muscle strain. Any strained muscle when stretched gives severe pain.Now after 4 months if pain is severe ,I would suggest getting this evaluated by an orthopedician for an accurate diagnosis and appropriate management. You may get the MRI of the area affected under his/her guidanceI do hope that you have found something helpful and I will be glad to answer any further query.Take care"
},
{
"id": 48144,
"tgt": "What are the symptoms of kidney stones?",
"src": "Patient: Hi,I am a 34 y/o female with a personal history of kidney stones. I am having lower back pain in waves, but no where near as severe as it was when I had kidney stones the past two times; both required surgery to remove them. I am also urinating more frequently than normal and am nauseous. I typically consume 90 oz of water a day. I got a urine test to check for leukocytes and nitrates. Nitrates have come back negative, leukocytes positive (not trace, but a pretty dark color in the positive range). My question is should I go see my doctor? Urgent care? ER? Are these early symptoms of kidney stones? Doctor: Dear patient,Classical symptoms of kidney stones are: severe flank and groin pain, blood in urine, nausea/vomiting and restlessness. Sometimes it can be without symptoms, because depends on the size of the stone.Presence of leukocytes in urine, your mild lower back pain, frequent urinating and nausea , together they indicate you have urinary tract infection. For this I would recommend to drink cranberry juice and 3-5 days course with Bactrim twice daily to help fight bladder infections.Hope my answer was helpful for you."
},
{
"id": 107145,
"tgt": "Suggest treatment for lower back pain due to severe scoliosis",
"src": "Patient: Hi I have been having pain in my lower back, Buttock area right side for several weeks now. I had a massage and that helped but it didn t go away. Went to Chiropractor and he thought it was between my tailbone and Pelvic joint. I have severe Scoliosis and I m 60 years old. I know I have arthritis and have had Scoliosis surgery at the age of 31 and had a fusion of the lumbar vertebrae s. Just never had this pain. It doesn t go down my back like Sciatica. Just in my rear near tailbone. Suggestions d Doctor: Hi, I had gone through your question and understand your concernsScoliosis in your age needs no further surgical treatment but really it may cause symptoms like pain .You should do new x ray for your lumbosacral spine or better to do MRI.FOr my patients in just condition I will advise them to wear a supportive brace,Avoidance of any heavy activities,receiving muscle relaxant and strong analgesia when needed."
},
{
"id": 39514,
"tgt": "Will zimig tablets help in treating ring worm infection?",
"src": "Patient: Hello Doctor, Myself Karthik............ I am suffering with ringworms in my thigh area since 3 weeks and it causes severe itching sensation in the night, I have been using emoderm cream and it doesn t show any improvement, and from today I have been advised to use zimig tablets for 1 week........ does it work in curing my problem sir. Doctor: Definitely it will work. You have received the proper treatment in the form of tab ziming which contains terbinafine 250 mg. And it is the drug of first choice for the ringworm/tinea infection now a days. It should be taken on once a day dosage at least for seven days can be extended for total of 14 days as per the requirement.Along with that it is better to use topical anti fungal such as oxyconazole or irtaconazole which will hasten the recovery. For itching you can have tab levocetrizine 5 mg at night time for 5 days.There is no use of the emoderm in this infection. Caution: Tab levocetrizine may cause drowsiness ij few patients .it is advisable not to drive after having tablet."
},
{
"id": 133906,
"tgt": "What causes leg pain and numbing in hand?",
"src": "Patient: i have had an xray done on my knee with nothing abnormal coming back. I have horrible pain when trying to straighten out my leg from a bent position to the point of tears. i also have had numbing in my hands for about two weeks off and on. please help! Doctor: hi,Thank you for providing the brief history of you.A detailed clinical musculoskeletal assessment is advised.As mentioned in your history about the normal x-ray and pain still persists while bending the knee, than an ortho clinical assessment is needed, as your pain sounds more of a soft tissue injury than a bony one. So x-ray cannot show anything except the bones which appeared normal. Your ortho may also advice you for MRI of knee, which will help in addition to see the soft tissue status. As pain during straightening the knee is related to the soft tissue injury and should get assessed.Based on the diagnosis, you will be recommended for simple pain killers and physical therapy.With physical therapy, your pain will reduce by use of therapeutic ultrasound therapy and TENS therapy. Also, exercises will be performed to regain the ROM of the knee joint and also the strength of the muscles.In my clinical experience, I have seen cases with such injuries, and they respond well to the medication and physical therapy. By 2-3 weeks of time, they have optimum functionality without pain.RegardsJay Indravadan Patel"
},
{
"id": 172482,
"tgt": "Does shoulder dystocia in a child at birth cause future complication?",
"src": "Patient: my son was born with 3.800 kg and he had left shoulder dsylocatia in the time of delivery through vaccum and he had jitterness twice but he did not get that jitterness further and now he is good and active and he is normal whether he ll get further complications later Doctor: Thank you for sharing your query on Healthcaremagic, I have carefully worked through your case, the babies with shoulder dystocia can develop temporary or permanent complications, the most common of these injuries are damage to the brachial plexus nerves, the potential long-term consequences depends on the nature and extent of the injury.Fortunately most brachial plexus birth injuries are transient, the majority of such injuries resolve by 3 months, with a range of 2 weeks-12 months, only a 4 to 15 % result in some degree of permanent injury such as hand muscle imbalance, deformities of the shoulder and elbow and the compromised arm can be 10 cm shorter than the nonaffected arm. Please kindly rate if my advise has been helpful Wishing you a good health Dr Tania Portelles-Driggs"
},
{
"id": 188041,
"tgt": "What could cause rashes on hard palate and sore throat?",
"src": "Patient: two weeks ago I had sore throat and a small rash on the hard palate of my mouth that had itched and swollen glands, then it all went away and I was fine, I was tested for strep and mono and both negative now two weeks later I have a worse case of sore throat and rash on top of mouth and it burns and very painful what could it be? could it be mono and it just hadnt showed up the first time ? help!?? Doctor: hello there! Sore that of that extent and rash needs to be evaluated. As it my indicate a scarlet fever a complication of simple strept infection. ANy ways nothing to worry about simple one visit to otorhinolaryngology and do as directed would help. and you will be good as newRegardsS Khan"
},
{
"id": 34534,
"tgt": "Suggest treatment for infection in eyebrow area",
"src": "Patient: I have an infection in my eyebrow area, I am guessing it is an ingrown hair, the infection has involved my eyelid and the lump is about the size of a large pea. Hot packs? Antibiotics? I am 6ft 2 \", 280 lbs, adult onset diabetes. Had chest boil approx. 3 months ago. Doctor: HI, thanks for using healthcare magicThe use of hot packs and also topical antibiotics would help. Topical antibiotics should be available over the counter.The hot packs should be applied 2 to 3 times daily for 10 to 15 minutes.If the use of the above is not sufficient for resolution then oral antibiotics may be needed.I hope this helps"
},
{
"id": 7289,
"tgt": "May I have chance to get pregnant this month ?",
"src": "Patient: Hi..Am 25 years old..married 1.5 years completed..am trying to get pregnant since 1 year but no luck..last month i went to doctor and she is suggested me to take folic acid morning and night timings and ovacare at afternoon timings. my cycle is 30 days. last month period was 22. so am taking tablets from my 2nd day of period..and i did tests for thyroid is normal ..my husband semen is 60million/ml and morphology normal 80%..so i think there is no problem at all..i did follicular scanings from 12 day 16th day i skipped on 15th day..16th day doctor said you have ovulated..so we had love 12th to 16th(daily once).so is there any chance to get pregnant...Please tell me..if i may not get pregnant then is there might be problem internally... Doctor: Hi Welcome to Healthcare Magic Forum Accordingly everything is fine, medically speaking u have to become pregnant, all the best. If not then u have to undergo investigations to find out the reason. Hope I have answered ur question Regards"
},
{
"id": 113734,
"tgt": "Lower back pain. On injections and physical therapy. MRI showing Schmorl node type superior endplate defect, disc protrusion",
"src": "Patient: I have had low back pain for a couple of years now. I have had injections , physical therapy and a tens unit at home. Nothing has worked. Sometimes the pain will go into my hips and legs. It has gotten to the point that I can stand at my sink for more than five minutes to wash dishes. I am only 39. My MRI reads: The alignment is normal. Vertebral body marrow signal is benign throughout. There is a small Schmorl node type superior endplate defect at L2 centrally and a similar slightly smaller lesion at L3. The lesion at L2 has signal which suggests it may be relatively recent. T12-L1: tiny left paracentral disc protrusion minimally indenting the thecal sac without significant canal or foraminal stenosis . L5-S1: small central disc protrusion just abutting the ventral thecal sac without significant associated canal or foraminal stenosis. Can someone please explain some of this and what other options are there? Doctor: Hai, Thanks for writing in. Schmorl's node are nothing but disc protrusions inside your vertebral bodies. They are benign and usually don't cause any problems. After reading your MRI it seems there doesn't seems to be any significant findings. Small / tiny disc protrusions might be present for all individuals. Since there is no canal or foraminal stenosis, the pain is not a neurogenic / discogenic pain. Your MRI also didnt comment anything abnormal in your sacro-iliac joints. Sometimes spondylolysis at L5/S1 or any other levels can be missed by MRI. After a thorough clinical examination, you might need to undergo stress views of your spine +/- CT at the levels to rule-out any instability of segment or spondylolysis. Till that time you can wear lumbo-sacral belt, back strengthening exercises, lying on hard bed etc.. Avoid leaning forward, sitting /standing in same posture for longer periods, lifting heavy weights. Regards, Dr.Sathyadharan"
},
{
"id": 152386,
"tgt": "What physical therpy exercise streches the good limb to bad limb",
"src": "Patient: what physical therpy exercise streches the good limb to get the bad limb to imitate? this exercise is called something and i need to know what. Doctor: yes physiotherapy will improve the blood supply to the muscles. so its helpful to convert a bad limb to good limb. if you take pain killers, it cant be taken for a long time as it cause ulcers and acidity. so physiotherapy is good."
},
{
"id": 116239,
"tgt": "Does deficiency of iron cause chronic fatigue?",
"src": "Patient: Even though I am sleeping well each night, I am still feeling like I have had no sleep for days, I have no energy at all and am finding it hard to concentrate could I be low in iron - this seem to have come on me all of a sudden over the last two days. Doctor: Hi,Thanks for asking.Based on your query, my opinion is as follows.1. Yes, iron deficiency leads on to anemia and this will cause fatigue, tiredness, breathing difficulty, bone pains, palpitations etc.2. You need to get your hemogram done to identify the anemia and type of anemia. Hope you have no bleeding in urine or stools. Get your iron studies done for confirmation of iron deficiency.3. Improve iron in diet through fish, egg, ragi. Also take iron supplements along with Vit C to improve absorption. Check reticulocyte count after 15 days to see for improvement. Hope it helps.Any further queries, happy to help again."
},
{
"id": 206112,
"tgt": "Could diabetes and sleep apnea be due to PTSD?",
"src": "Patient: I am a retired veteran suffering from PTSD. I have been in therapy for 2 years now, slowly but surely, my outlook on life is improving. I am taking prozac and bupropion, for my depression, as prescribed by my Mental Heath Dr. Both My Mental heath provider and my PCM take the time to listen and help with any problems that have occured, they are in the same clinic and talk to each other about my care. During my cognitive therapy, I was having problems sleeping, mainly because of nightmares. I am unable to take sleep aids because, they make me to drowsy to get to work the next day. So i can only take them on weekends. because of this, I found myself binge eating. I was also drinking around a Gallon of soda a day, had to stay awake at work. this has resulted in me becoming Type II Diabetic. I also have been recently diagnosed with mild sleep apnea. from research I have seen many studies on how PTSD is a Major factor in both of these problems. My problem is my Medical team is not will ing to put it in writing that the 2 conditions could have been caused ( not the right word I know). by the PTSD. How do I find someone that I can get a second opinion from, that would be willing to put it in writing. Doctor: HiThanks for using healthcare magicIn that case, better to consult a psychiatrist. You need written medical certificate for your problem. Only psychiatrist could help you in this matter. In case, you need further help, you can ask.Thanks"
},
{
"id": 144891,
"tgt": "Suggest treatment for neuropathic pain in ankles and calf",
"src": "Patient: I m a 48 year old lady having problem from last 8month neuropathic pain in my Calf & surrounding ankle.from last 2 month I m suffering urine problem I have to go toilet 3-4times in night.I m a patient of o/A having Glucsamin calcium multivitamin omega From last 5years.My pain in toe fingers & Ankle from last 8 month kindly suggest me the right way of treatment to get result.I will be highly thankful to you Doctor: Hello,Welcome to health care magic. I am Dr Alok Sinha, MD Psychiatry from India. I can understand your concern. I will try my best to answer your query.I have carefully read your question and could understand that you are suffering from osteo-arthritis and neuropathic pain.Firstly let me educate you about various available medicines for pain.Pain is a noxious stimulus which should be managed promptly. Now a days pain management is done through a multidisciplinary approach which includes physician, surgeon, anesthetist and psychiatrist. If pain is recurring and simple pain killers like NSAIDS are ineffective then some alternatives should be tried. There are few medicines which act locally like pain relief gels and sprays. Some newer oral medicines act at spinal cord and brain level and reduce the perception of pain. Some medicines contain combination of a analgesic and an opioid. Some refractory cases required nerve block.Now coming to your case, since you are having chronic pain of osteo-arthritis and neuropathy, you need regular medicine. Common pain killer NSAIDs may be dangerous as they may cause peptic ulcer and renal complications. Opioid medication may be addicting. Hence you need something with little side effect.If a lady like you comes to my clinic, I suggest them to relax and try some De-stressing measures. Because stress and anxiety increases the perception of pain. These destressing activities are:-- healthy balance diet- regular exercise, yoga, meditation & relaxation exercise- adequate rest and sleep- listening soothing slow relaxing music etc. If needed I prescribe them some medicines like Pregabalin and Nortriptyline in combination along with methylcobalamin and folic acid. These medicines act on spinal cord and brain pain pathways and are relatively safer for long term use. Results in clinical settings are rewarding. You may discuss these issues with your doctor. Hope I have given you a way. I wish you all the best."
},
{
"id": 53893,
"tgt": "Could inflammation in the liver be a matter of concern?",
"src": "Patient: I recently had my gallbladder removed due to gall stones that were discovered during an ER visit for severe upper right quadrant pain. My liver enzymes were slightly elevated at the time of my surgery so my Dr. conducted a liver biopsy during my surgery. It later showed that I have some inflamation, I am wondering how concerned I should be...he repeated the labs and I haven't heard anything...? Doctor: 1) You might have inflammation in liver which may occur due to many reasons2) To quantify severity of inflammation, liver function test is must3) Generally treatment for hepatitis is medication only and in most cases it resolve with medical support.4) In your case, liver function test might come elevated also due to episode of cholecystis (inflammation in gall bladder and surrounding area)."
},
{
"id": 2597,
"tgt": "Does pregnancy occur after stopping depo provera?",
"src": "Patient: Hi,i was due for my depo provera shot on the 26th of June and decided not to get it as we wanted to come off it and try for a baby i had been on depo for 18 months roughly. I still have no sign of a menstral cycle and have been having pregnancy symptoms but taken 5 pregnancy tests which are all negative? Doctor: Hi,Take primolut n 0.5 mg tablets 1 twice daily for 4 says then stop it you will have menses after gap of 5 to 6 days.Hope I have answered your query. Let me know if I can assist you further. Regards,Dr. Sujata Kute"
},
{
"id": 63721,
"tgt": "What causes a lump on the jaw bone?",
"src": "Patient: Hi, I have a pea size lump on my jaw bone, just by the corner. It was at first bigger and painful, inflamed, and as I am pregnant I assumed it was hormonal. Now it has shrunk in size and is no longer painful but will not go. It does not move around but is not attached to the jaw bone. I am 31, 5 months pregnant. 5ft 1 normal weight for height and pregnancy Doctor: Hi, dearI have gone through your question. I can understand your concern. You may have either enlarged lymphnode due to reactive hyperplasia or submandibular salivary gland enlargement. You should go for fine needle aspiration cytology. It will give you exact diagnosis. Then you should take treatment accordingly. Hope I have answered your question, if you have doubt then I will be happy to answer. Thanks for using health care magic. Wish you a very good health."
},
{
"id": 87827,
"tgt": "What causes abdominal pain during menstruation?",
"src": "Patient: Hi, may I answer your health queries right now ? Please type your query here... hiya, i had a c section 9 months ago and in the second month of having my every month i get belly ache one time a was sick and the rest is toilet, and also will it affect it to try 4 another baby as ive been trying 4 3 months and now im worried Doctor: Well you should hormonal tests,ultrasound and see gynecologist to rule out some serious disorders.but in most cases this is normal and isn't indicating pathology and won't effect on fertility.Wish you good health. Regards"
},
{
"id": 113076,
"tgt": "Pain in the lower back after drinking alcohol. Have persistent low WBC and low vitamin D levels. Severe diarrhea. Concerned",
"src": "Patient: I am experiencing some pain in my lower back ( mostly left side) after drinking a glass of red wine. I never drank much alcohol until lately ( the past few months) having a glass of red wine with dinner. I am a 33 year old female. I did suffer for a number of years with a low white blood cell count (and low vitamin d levels), but it seemed to get better as the stress of school was removed and my hemotologist thought i wouldnt need to come back. this was about two years ago. In addition I became vegan about two years ago. About a half a year in I started having severe diarrhea . I was put through a bunch of test (e.g. Full endoscopy , X-ray, ct, nucleotide scan) but nothing was ever found. The problem has subsided to where I can function but, I have never fully recovered to having normal digestion . I do know I was on the lower end as far as my b12 levels, but have been taking supplements. I am worried about this pain and am not sure if it could be serious. Doctor: Hello. Thanks for writing to us. The pain in the back specially after drinking a glass of wine is likely to be related to gastric acidity. It is not likely to be related to any serious pathology. Avoid having any alcoholic drink on an empty stomach. I hope this information has been both informative and helpful for you. Regards, Dr. Praveen Tayal drtayal72@gmail.com"
},
{
"id": 135593,
"tgt": "Does a leg cut need further medical attention?",
"src": "Patient: Hi I bumped my lower left leleg on a tv yable last Thursday and I have a cut on my left lower leg and ive been cleaning it daily with bactine and applying Neosporin a couple times daily however it is pinkish red around my cut do I need to contact my dr for an antibiotic? Doctor: Hi Thanks for your query. I have gone through your query and understand your concern. I think your wound is healthy. You need to visit doctor if you you have fever or pain is more. Otherwise you need to worry it will heel itself. You can discuss with your doctor about it.Hope your query get answered.Wishing you a good health.Take care."
},
{
"id": 170693,
"tgt": "What causes pimples on the inner thigh?",
"src": "Patient: my four year old daughter keeps getting pimples on her inner thigh and because they are very tender she won t let me pop them. She has a history of molluscum bumps but these are what I would consider a pimple. What causes these and what can I do to prevent them. Doctor: Hi, this appears to be pyoderma, you should give a proper antibiotic course of 5 to 7 days like amoxyclav and apply mupirocin cream at the lesions. Once they are completely healed then chances of recurrence are rare. In children, all drugs are given according to weight so i advice you to revert back with exact weight of child. Take care."
},
{
"id": 161788,
"tgt": "Suggest treatment for baby suffering from productive cough",
"src": "Patient: Dear Dr. My son Rohan(Age 8 months) has been suffering with productive cough since 27/12/2010. Our pediatrician prescribed Topex BR 10drops x 3 times, Clavam 15 drops x 3 times. But no improvement and the weezing sound worsen day by day. He suffering too much. Please give us a solution. Doctor: Hi, By what you say I feel that your son could be having a viral lower respiratory infection or wheezing episode. I feel that he might get better with oral levosalbutamol given. But this is a prescription drug and you can discuss this with your pediatrician. Hope I have answered your query. Let me know if I can assist you further. Regards, Dr. Sumanth Amperayani, Pediatrician, Pulmonology"
},
{
"id": 203856,
"tgt": "Arousal, erection and itchiness in scrotum with pain, occasional ejaculation on holding girl's hand. How can I overcome this?",
"src": "Patient: Hi doctor.. I am getting excited even when my girl friend just holds my hand and walks with me. i have no intention of having sex with her now. But when ever she is with me or even when she holds my hand and walks, my penis gets hardened and i have this itching sensation in my balls and an hour or so a pain in my balls and penis. Ijust have to leave from ther giving some excuse. I am truely in love with her. Initially i thought since this was the first time i touch a girl i feel so. BUt this happenes everytime i am with her. My penis leaks at times then. i browsed net is this called premature ejaculation?? I used to mastrubate before (say a year back )but now i dont do that. Has it some thing to do with it.?? I need ur help doc. Please help me overcome this. Is this curable.?? i just want to spend some time with her with our all these pain and problems.!! Doctor: Hello,Thanks for choosing health care magic for posting your query.I have gone through your question in detail and I can understand what you are going through.No this may not be a premature ejaculation. Its normal for a boy to get stimulated leading to the some pain and seminal leakage. There is no need to worry. masturbation is also safe and there is no problem. This thing will settle down with time. You may even masturbate before meeting the girl which will reduce the stimulation and you will be comfortable. Team is the best healer in this case....Hope I am able to answer your concerns.If you have any further query, I would be glad to help you.In future if you wish to contact me directly, or want to further details about your query, you can use the below mentioned link:bit.ly/dr-srikanth-reddy"
},
{
"id": 220798,
"tgt": "How to abort pregnancy?",
"src": "Patient: hi, I have a query regarding pregnancy. Got married 2 months back. i didnot get my periods on 10 th of this month. went to doctor and doctor confirmed that i am pregnant. me and my husband dont want to have child now becuase of family problems. and would like to terminate pregnancy. can u pls suggest method for this Doctor: Hello dear,I understand your concern.In my opinion the number of weeks of pregnancy should be ascertained before going for method of termination.Usually the medical termination of pregnancy is possible upto 6-7 weeks of pregnancy.Anything after that needs surgical procedure for termination.So ultrasound might be needed to know the number of weeks of pregnancy .And the abortion pills need to be taken only under doctor supervision as they are associated with complications like incomplete abortion,heavy bleeding etc.So I suggest you to consult doctor for further management.Best regards..."
},
{
"id": 20847,
"tgt": "Suggest treatment for high blood pressure",
"src": "Patient: hi i am 34 years old female having high blood pressure , last checked my pressure was 186/135, the doctor at doha clinic gave me iv , my pressure came down to 140/90, but i have constant pain in my left hand and left back. is the pain due to high pressure. wat is the treatment to high blood pressure. please advice. thank you Doctor: I hope you are fine now. you asked about the treatment of high blood pressure, in my opinion I would suggest my patients to maintain a chart of almost one week by checking their blood pressure twice in a day and come to me for the adjustment of their medicines(urgent management is required when bp is very high). you can change your lifestyle which also has a good effect on overall health. moreover I would suggest you to go to your doctor and get your ecg done for the pain in chest or arm. I hope you got your answer."
},
{
"id": 50502,
"tgt": "Has cellulitis, failed kidney. On steroids, flucacillin. Sufficient?",
"src": "Patient: where to start... crack on heel caused infection diagnosed as cellulitis requested steroids due to kidney failure twice already - 1 + vasculitis - 2 + septacemia given antibiotc flucacillin only... did not help - much less cure cos I need more help than ordinary patient but am ignored !!! told by kidney consultant my immune sytem is compromised... completely clapped out more like... it fails to protect/defend me... hence my request for streroids which I said would be necessary - but meets only with doubt/denial... consultant no help as he tells me I am fine - every time I say I am in big trouble... so I have no-one to protect/defend me either... now I hurt like hell burning from toe to knee on both legs sitnging like post herpatiic pain which followed chickenpox at 40 and shingles at 66.... at my wits end - for way to get help here so appreciate advice/assistance/improvment... having read next step - concerned about cost ... I am much into co-operation re reseach and rescue - for we unfortunate oddities so that we can help profs and they in turn can help us i.e. co-operation for the benefit of both parties... would appreciate warnings re cost if reqiuired here Doctor: hello,Thank you for your query.To treat cellulitis you need to take antibiotics which acts against gram positive organisms as those are the most common cause but in your case you was already on flucacillin so you need to take cephelexin 250mg twice a day and tab.chymoral forte three times a day.You can also apply glycerin and epsom salts on the site so that it helps you to decrease the swelling.Using steroids may makeyou dependent due to you pain and they must be discontinued my tappering doses may be thats the reason why your doctors are not prescribing you steroids.Before going to steroids you need to corrrect the situation with common drugs as they treat the infection easily.Intravenos antibiotics are needed if there is no response with oralantibiotics.Dont be panic and dont loose the hope, everything will be fine with correct treatment.Hope my answer may satisfy you.Have a good health and you can contact me anytime if you have any other doubts.Thank you."
},
{
"id": 175652,
"tgt": "Can Ascoril be given for child s cough and runny nose?",
"src": "Patient: my son will be three next month.he came down with throat infection and developed fever accompanied by cough and runny nose . he has been administered cephalexin syrup 125mg/5ml three times a day. it is day 3 of fever(temp around 100-102 F) with 8 hourly interval. now his cough is making him very uncomfortable because it is very mucousy. Can I give him ascoril ls expectorant for his cough. For his fever I have been giving him Ibugesic plus...so please could you guide me about the cough syrup to be given or should I take him to the GP today? thanks Doctor: Hi....Cough and cold are viral 95% of the times in children. For cold you can use anti-allergics like cetirizine and for nose block, saline nasal decongestants will do. Paracetamol can be given in the dose of 15mg/kg/dose (max ceiling dose 500mg) every 4-6th hourly, that too only if fever is more than 100F. I suggest not using combination medicines for fever, especially with Paracetamol.You can give Ascoril. But I suggest you take him to GP...to check if he has got any wheeze.Regards - Dr. Sumanth"
},
{
"id": 221472,
"tgt": "Is it safe to take Primoult n while trying to conceive?",
"src": "Patient: i have some family function on dec 10. but my mensus may fall on dec 10.. but i m the in charge of that function.. i am trying for pregnancy for past 10 days. can i take primoult n? does it affect my pregnancy chance? pl help me. i m confused abt ths / Doctor: HiDr. Purushottam welcomes you to HCM virtual clinic!Thanks for consulting at my virtual clinic. I have carefully gone through your case, and I think I have understood your concern. I will try to address your medical concerns and would suggest you the best of the available treatment options.I will suggest that if getting pregnant is your priority then instead of using Primolut N, you can use Cap SUSTEN sr 200mg 2times a day to postpone the menses.It will remove the fear and risk , even if you get pregnant.And Susten will not have effect on pregnancy, instead it is a helpful support for pregnancy.I hope my answer helps you.Thanks.Wish you great health."
},
{
"id": 49290,
"tgt": "How to treat non pitted edema on legs with elevated creatinine and glucose level while having IV medications for lyme,bartonella, and babesiosis?",
"src": "Patient: Slightly elevated creatinine levels, high glucose levels. On many IV medications right now because I have Lyme, Bartonella, and Babesiosis. Meropenum and Cipro in the a.m., Meropenum and Tigecycline in the evening. Now developing non pitted edema on my tops of feet and on my legs. I also have factor V Leiden if that makes any difference. Should I be worried about the edema? Is it likely fluid retention? Doctor: HIYou have not mentioned your age and gender this was extremely necessary here without this I may not give you right opinion or advise, any way I could say here that you have to stop un-necessary medicine, taking medicine for no good or valid medical reason always brings something bad, elevate the limb, cut down the salt in diet, take care and next time give complete information bye."
},
{
"id": 215534,
"tgt": "Suggest an effective alternative for Methadone",
"src": "Patient: I have been taking a low dose of methadone for pain from my doctor for over two decades. I know I am dependant on it, however my perscription is taken how it is supposed to be taken and I,ve never misused it I have never taken any illicit drugs, taken alcohol or smoked marijuana and have no desire to do so.my main question is there any other pain medication that would work for me that does not have such a negative reputation. Doctor: Hello, You can consider alternatives like hydrocodone. Gabapentin can also be tried. Hope I have answered your query. Let me know if I can assist you further. Regards, Dr. Shinas Hussain, General & Family Physician"
},
{
"id": 16259,
"tgt": "Applied savlon on face overnight, red burn patches on face, washed with ice cold water, redness still there. Medical advice?",
"src": "Patient: I have used raw savlon on my face and kept it overnight. I n the morning I got burning sensation. I saw red burn patches where I have applied savlon. I have wahsed my face for nearly 20 mins with ice cold water. Now the pain has gone but the area is very red. Will it leave scars? How long will it take to heal? Do I need to take any medical advise? Doctor: the prolonged contact with savlon has, caused this condition, avoid exposure to sunlight, and apply desowen lotion or desonide lotion for a couple of days."
},
{
"id": 27563,
"tgt": "Suggest treatment for enlarged heart and tremor",
"src": "Patient: 44 Male, very athletic, physically fit and active. Diagnosed with enlarged heart (idiopathic cardiomyopathy) 3 years ago. EF ratio of 50. Diagnosed 5 years ago with essential tremor as well. I understand essential tremor worsens over time and my symptoms are very strange. Fatigue, aggression, mood swings, tremors in hands arms and can be any body part. Muscle cramping all over body (was bad for about a week and has subsided) happens a couple times during the week. Tremors are worse at rest. My cognitive test today was normal/average however I know that my performance was sub-par based on cognitive deterioration over the past year. I cannot sit still, feel like I tense up and need to wiggle around (only good way to describe). Running blood test and trying to schedule MRI. Thoughts and or suggestions? Doctor: Hello,I have gone through your query.Thanks for using HCM.first of let me tell you EF of 50% is not bad at all.Are you on medicines for cardiomyopathy?I suggest you to get your thyroid functions tested .You may start beta blockers In consultation with your doctor which help both tremors and heart.My best wishesDr.Rajesh Teli,MD."
},
{
"id": 162217,
"tgt": "Is it safe to nebulize a 4-month-old while sleeping?",
"src": "Patient: My baby boy is 4 month old. My family Dr. Suggest me to nebulize with clenil compositum 3 time a day with gap of 8 hour and use singulair. He is suffering from cough and asthma. As well as chest infection. can i nebulize when he sleep ? And is it right medication for him ? Doctor: Hi, It is safe to nebulize even when the baby is sleeping. But it should be done by oxygen and not from a nebulizer from room air. It is not safe. Nebulization should always be done with an oxygen source, please. Hope I have answered your query. Let me know if I can assist you further. Take care Regards, Dr Sumanth Amperayani, Pediatrician, Pulmonology"
},
{
"id": 223178,
"tgt": "What causes prolonged menstrual bleeding for almost two weeks?",
"src": "Patient: I startedmy (current) period on the 28th of May, it is now the 10th of June and I m still bleeding. When they had happened before my doctor would put me on birth control temporarily to regulate it but I am currently out of state, away from my primary for another couple of weeks and have my healthcare through the state. Is there another way I can make this stop? Doctor: Hello and Welcome to \u2018Ask A Doctor\u2019 service. I have reviewed your query and here is my advice. Yeah, you can use Antifirinolytics like Tranexamic acid like tablet Trapic for 3 days to stop or reduce the amount of bleeding temporarily but the reason for your menorrghic cycles should be evaluated first. Hope I have answered your query. Let me know if I can assist you further."
},
{
"id": 26675,
"tgt": "How can i control my blood pressure?",
"src": "Patient: i am wondering if I may obtain better control with my hypertension if I take lisiniprol twice a day instead of once. the drug works very well for me and I am contraindicated for diuretics. but during the night my BP can rise abruptly. it is controlled beautifully during the day. I take 20 mg every morning. keep my BP around 115/70. it cn jump to 150/95 at night. Doctor: hi, recommended dose of lisinopril is once a day, firstly you can try taking it at night time and if during day your bp shoots up you can add another drug like amlodepine 5mg or metoprolol sr 50 mg."
},
{
"id": 204011,
"tgt": "Having sperm leakage, night falls, premature graying of hair. Treatment to cure sperm leakage?",
"src": "Patient: i am 21 years old. I have this problem of sperm leakage. It happened due to night falls. I had night falls very often. Say twice thrice a week. Now my night fall occurs once in a month but the problem is when I go to washroom the sperm automatically comes in my urine . Due to these night falls and sperm leakages my entire structure of body is changed and my hair is becoming premature grey. My reproductive organs have also become very weak. Please help me stop this sperm leakage. I am facing grave problems in my daily life Doctor: DearWe understand your concernsI went through your description. I suggest you not to worry much. Body produces sperm cells whenever it is sexually aroused. Produced semen should be thrown out of the body. Sexual intercourse, masturbation helps here. If you do not indulge in such activities, body finds its ways to throw out the semen and one method is night fall of drips while urination. These procedures are normal and cannot cause physical weakness. Body weakness, premature gray hair etc are not at all related with night fall and dripping of semen. But anxiety is the cause in your case.You might need psychotherapy and other cognitive behavioral therapies. I can help you through telephone. Please do contact me through customer care of www.healthcaremagic.com for a telephonic consultation session. Hope this answers your query. Available for further clarifications.Good luck."
},
{
"id": 56806,
"tgt": "How to treat the chronic intermittent abdominal pain?",
"src": "Patient: my 15 year old daughter has chronic intermittent abdominal pain with abnormal liver function and abnormal clotting profile. previous ultrasound showed coarse liver texture. the liver appears to be enlarged and exhibits a diffusely coarse echotexture with no apparent focal leisons identified.the spleen appears enlarged for her age. Doctor: HI Well come to HCM Abdominal pain may not be due to the liver findings detected in ultrasonogragy and not even due to spleen enlargment, such pain is common in female of this age this could be vague type of abdominal pain, nonspecif, such pain can be treaed with following medicine, dysmanorrhea need to rule out. 1) Tab Clinidium with Chlordaizepoxide three times in day this is nothhing to worry would come around soon, take care."
},
{
"id": 51167,
"tgt": "Abdominal pain, numbness in leg and vagina. Cannot hold urine. Recovered from UTI. Have ovarian cysts",
"src": "Patient: I am having pain in my left abdomen, feels like kidney and ovary , i have numbness in my left leg and side of my vagina, left side, feels like its falling asleep, just numbness and circulatory pain, no tingling . Recently cannot hold urination . Immediate urge, no waiting time. Just was treated for uti, finished meds, also have ovaeian cysts but only on eight side, to my knowledge Doctor: Hello and welcome to HCM You have mentioned that you have alump on left side of the abdomen. You need to get an ultrasound abdomen done to know the site and origin of this lump. It can arise from various organs present beneath left side of the abdomen. It could be an ovarian lump, kidney lump, lump arising from intestine, bony limp, a soft tissue mass. An ultrasound and/or CT scan is indicated. If the lump is palpable then fine needle aspiration cytology needs to be done. The type of lump can be made out be seeing the character of the cells aspirated on FNAC. Regarding your symptoms of numbness on left side of the body, pain and inability to hold urine; it seems that there is some kind of compression of the nerves. This compression could be due to the mass peresent on the left side of the abdomen. Consult your doctor at the earleat for investigation and correct management. Thanks and take care Dr Shailja P Wahal"
},
{
"id": 129592,
"tgt": "Why am I loosing control on my legs?",
"src": "Patient: I had a heart attack in 2007 and have felt well since and on the usual medicines. Last week I lost the control in my right leg and it recovered . Two days later I lost control in both legs and fainted. I am awaiting blood tests and have had my blood pressure tested. The latter was excellent. Please comment. Doctor: Dear patient the history that you give here points to a classic embolic stroke known as TIA.YOU should go to the ER immediately and tell them to get you an MRI/CT head. Mri will be better but if it takes time a ct will also point to a tia.Excellent Blood pressure may rule out a haemorrhagic stroke but it does not rule out the possibility of an embolic stroke which is common in patients with a history of a previous heart attack.I suggest you review the medications with your doctor and consider adding clopidpgrel 300mg with the aspirin you probably are taking."
},
{
"id": 134229,
"tgt": "What causes twitching and pulsating sensation in legs post herniated disc surgery?",
"src": "Patient: For over a week now I have had a pulsating sensation in both legs along with twitching and pulsating in my feet. I don t have a doctor right now and I am very concerned as to what this could be. I am a 76 year old female with compressed discs in my back and bone on bone in ankle. When this pulsing first started it was very strong in my left outer leg. Doctor: hi,thank you for providing the brief history of you.as you mentioned you have compressed disc in your lumbar spine the symptoms you are facing is because of the disc irritating the nerves. This is my opinion.What you should understand here is that the spinal muscles and the core muscles are weak so you need to focus on strengthening the same which will help you on a longer run. Since your age is 76 I will always be worried because at this age little pain is like hell out of cry.Let's take a talk about the symptoms you have and how to.reduce it. There is McKenzie extension position which is taken will centralize the pain in your back. post that you can use hot water bag to over the Lower back region to reduce the pain.Later on the exercises for spine, core, should be initiated. You can take a help of a physical therapist. strengthening the body muscles is a natural way of healing the pathologies related to muscular injuries. I wish you get helped by this.with the grace of God I wish you a speedy recoveryregards"
},
{
"id": 92612,
"tgt": "Abdominal pain, nausea, loss of appetite. Taken Dimetapp for cold. What is this?",
"src": "Patient: Hello. My friend is experiencing some abdominal pain, and I wanted to get some help to figure out what it might be. He is a 15 year old male. He said that he is having pain, in his mid abdomen, near his navel, and pain to the lower right of his navel. He gets more pain when he walks around, or moves suddenly. He has not many other symptoms opposed to that. The pain has been continuing since Saturday morning (it is Sunday afternoon right now.), and it has been progressively getting worse over that time. He pressed down on the area where he is experiencing pain, and it is a bit firmer in that area. He was recently taking Dimetapp, for a cold, from the Monday to Friday before he began to experience pain. He takes 10mg of melatonin each night to help him sleep. He went to the washroom, and there was no blood in his urine, or stool. He has no allergies to anything that he is aware of. This has never happened before to him. He ate a bit of granola earlier this morning, but had a loss of appetite, so he did not consume much. He also feels a bit nauseous. If you have any more questions, or any idea of what this might be. You can email me back at YYYY@YYYY and I will reply back as soon as I can. Thank you for your time. Doctor: Hi, Please send him to the ER as most probably is having apendicitis. Doctors in the ER will evaluate his condition better. Do not lose more time. Hope it helped!"
},
{
"id": 31781,
"tgt": "What is the treatment for severe cold?",
"src": "Patient: hello sir, My name is Arjun. I am moved to bangalore 8 months back. I started having cold regularly in these 8 months. Now for the the past 10 days I am having severe cold and it is not going away. I am taking tablets like cold flu and cold act. Also having benadryl. Can you plz suggest me some doctor whom I can visit for this problem. Doctor: Hi Dear,Welcome to HCM.Understanding your concern. As per your query you have severe cold which could be due to upper respiratory track infection specially viral infection and it could be sinusitis. It is due to impaired immunity of body and increased susceptibility of getting infected. Need not to worry. I would suggest you to do warm saline gargles and betadine gargles alternatively , avoid eating spicy and heavy food and consult ENT specialist for proper examination . Doctor may order throat swab test , vial culture test and blood test . Doctor may prescribe anti viral along with anti inflammatory Levocetrizine. Doctor may also refer you to gastroenterologist if he/she find problem to be related with acid reflux . Hope your concern has been resolved.Get Well Soon.Best Wishes,Dr. Harry Maheshwari"
},
{
"id": 199928,
"tgt": "How to overcome masturbation addiction?",
"src": "Patient: Hello sir I am 23 years old. I had a habit of masterbation after 2-3 days, but one day I felt nurvis and I thought I got weakness in my penis..so it increase my blood pressure due to is problem so tell me how I increse my penis size and stronge sex.... Doctor: HelloThanks for query.You had been indulged in masturbation since many years and now facing problems related to sexual activities and general weakness..First of all there is myth in a mind of common man that excessive masturbation is the reason of all sexual problems But I would like to state that it has been discussed in scientific forums all over the world and proved scientifically that masturbation does not have any negative effect on any organ or system in the body.All the problems that you are facing now are mind related and due to anxiety.Following measure will help you to boost up your confidence and improve your health and sexual potency. 1) Practice regular exercise for 45 minutes followed by meditation for 1/2 an hour in the morning.2) Take high protein diet rich in vegetables and fruits and Vitamin A,C,D,E.and Zinc3)Take anti oxidants like Almonds 5-6 everyday..4) Avoid alcohol and smoking..Though it is harmless, excessive and frequent habitual masturbation does affect quality of erection and delays ejaculation.Please try to reduce the frequency of masturbation by keeping yourself busy in activities like sports,reading ,social work etc so that you will not get free time to masturbate.There are no medicines that can increase size of the penis.Dr.Patil."
},
{
"id": 3582,
"tgt": "What could delay in pregnancy indicate?Suggest remedy",
"src": "Patient: hello dr,we are trying to get pregnant from last 1yr,still we dit get.we have taken all test,me doesnt have any preblem but dr gave ovacare tablet for 1 month,my husband have sperm 4millions/ml,he is taking oligocare and addyzoa for i month.could we have chance to get pregnant naturaly or will we go for IUI/IVF.Please tell madam,i dont know what to do Doctor: I healthcare magic user, yes you can conceive naturally but less chances about 5%. if I would have been your doctor I would have advised your husband to check for fasting blood sugar and post lunch sugar. Let me in detail clinical history ,Age, weight, and blood reports such as AMH, TSH, prolactin, blood sugar so that I can guide you whether to go for IUI or IVF."
},
{
"id": 169568,
"tgt": "What causes stomach pain in a child?",
"src": "Patient: Hi, my daughter is 3 yrs old. I was called by her day-care center that she had an episode of pain in tummy and right after that she was very pale, weak and she closed her eyes for a couple of seconds. They gave her a biscuit and when I arrived 5 minutes later she was looking good. I saw that she didn't poo since yesterday and when we got home, she did. Then after that, she was active, played outside and full of energy. I don't think I need to take her to a doctor as I guess it's from the constipation she had pain then I don't know if her blood pressure went low or her sugar. Can you please advice? Doctor: Hi Dear,Welcome to HCM.Understanding your concern. As per your query your child have symptoms of stomach pain. Well there can be many reasons for symptoms you mention in query like food allergy, gastroenteritis, electrolyte imbalance, acid reflux or systemic problems. Need not to worry. For now avoid eating spicy food and also avoid seafood and nuts. I would suggest you to consult gastroenterologist for proper examination. Doctor may order endoscopy of esophagus , X-ray of upper digestive tract and ambulatory acidity test. Doctor may prescribe H2 blockers , proton pump inhibitor along with antacid. Doctor may also refer you to allergy specialist or cardiologist if he/she find problem to be related with food allergy like seafood.Hope your concern has been resolved.Get Well Soon.Best Wishes,Dr. Harry Maheshwari"
},
{
"id": 176660,
"tgt": "What causes watery stools while being on dumex dupro?",
"src": "Patient: Hi, my son drinking dumex dupro step 2 now. But recently he s stools are watery & sometimes after each feed he will pass motion & sometimes when he farted, the stools will comes out watery. Is he allergic to the milk? Or is it normal as he s started his solid food? Doctor: Hi...in this age the kids keep whatever they find in the mouth and it is normal phenomenon to get diarrhea like this and it is most probably viral in origin. Once it starts it will take 5-7 days to completely get better. Unless the kid's having low urine output or very dull or excessively sleepy or blood in motion or green bilious vomiting...you need not worry. Regards - Dr. Sumanth"
},
{
"id": 14485,
"tgt": "What is the remedy for rash on penis,thigh and butt?",
"src": "Patient: a rash on penis, thigh, and butt for three months that has not gone away. Rash acquired 2 weeks after remicaide treatment. Received creams and ointments that have not worked. Recently started taking suppression medication, because the doctor thinks that it may be herpes, that has not worked. The rash continues to spread every day. I am very frustrated and don\"t know what to do. Have another appointment with doctor tomorrow. Doctor: Hello,Thank you for posting on HCM.It seems you are suffering from Tinea cruris and corporis, a kind of fungal infection.I would suggest you to consult your dermatologist for proper management of the condition.I usually recommend proper course of oral anti-fungal drugs like Fluconazole or Itraconazole along with anti-fungal cream like luliconazole for local application at bedtime. You can additionally use antifungal dusting powder containing sertaconazole during day time and a soap containing ketoconazole for rinsing of affected areas. Take oral antihistaminics for itching as required.Maintain hygiene over those areas and avoid wearing tight undergarments.Hope your queries are resolved and wish you best of health.Kindly spare some time to rate my answer and drop your valuable review at the following link:https://urldefense.com/v3/__http://doctor.healthcaremagic.com/doctors/dr-hardik-pitroda/67169If__;!!Mih3wA!SBzm6_kI6hCZ58EPH6N_05MFfiPbxWXT0a2TJCdFQObRWm5mV5ur7hW9rW0j8A$ you require any further assistance in future, you can reach me directly through the above mentioned link.Thank youDr Hardik Pitroda"
},
{
"id": 29025,
"tgt": "Suggest treatment for body pain while suffering from flu",
"src": "Patient: I was diagnosed with the flu yesterday. Started Tamiflu. I also have had pain in between shoulder blades, yesterday pain in the upper left side of my abdomen, under breast area, that radiates into back. Comes in waves. I figured it\u2019s all due to inflammation but am getting worried now.... was wondering if I should go back to doctor. Is this all part of the flu or is it unrelated? Doctor: Probably it is cause by the flu or as a side effect of the Tamiflu. Try to assume some antiinflammatory drug, such as ibuprofene: if it makes you feel better, your pain is due to the flu. Otherwise, if you don't have any improvement from the antiinflammatory drug, you should do back to the doctor."
},
{
"id": 159267,
"tgt": "Advanced liver cancer, blisters on feet and hand, painful abdomen. Taking morphine. Suggest?",
"src": "Patient: My brother has inoperable advanced liver cancer and has been on Sorafenib . The dose had to be reduced due to severe side affects such as blistered feet & hands, he is also on oral Morphine and is suffereing pains and hardening accross the stomach. Is this the cancer or the meds he is on? What are your feelings on Marijuana as pain relief at this stage? He was diagnosed in July last year and the tumour then measured 13.8 cm Doctor: HI, i DONT think marijuana is a great idea. He has a delicate health status and should comply with his physician s prescriptions. Remember that most drugs are metabolized by the liver, and in his case his physician certainly does a keen selection of drugs before he prescribes. I suggest you respect his normal drugs and report any unwanted effects to his primary care physician. Hope this helps, thanks and best regards, Dr Luchuo."
},
{
"id": 20599,
"tgt": "Suggest ideal dosage of Tazloc 40 mg to treat hypertension",
"src": "Patient: my mom diagnosed with systemic hypertension which is very mild. after taking tazloc 40mg once in the morning she feels very fatigue and tired. she was afraid bcoz of that. is tazloc 40 is needed for her. 40 mg is too much dose for her r it is correct Doctor: HelloMy be 40 mg is a bit more for her that's why she is feeling fatigued. What you can do is to reduce the dose of TAZLOC to 20 mg and keep an eye on her blood pressure regularly.If her bp stays below 140/90 mm Hg then she can continue with 20 mg and hopefully she will also not feel fatigued.Kind Regards"
},
{
"id": 26684,
"tgt": "Should I continue enam 5mg when having BP 140/80?",
"src": "Patient: hai, i am satheesh, my mother had bp for past 5yrs she took Tablet enam 5mg for that and now recently for 10 months she having normal bp 110/70 then we stoped that tablet, but now i checked her bp is 140/80 is it normal or we want to continue the tablet.... Doctor: Hello!Thank you for asking on HCM!I understand your concern and would explain that the actual blood pressure values that you refer are above the normal limits. In such a case, I would recommend to consult with the GP for a careful physical examination and a routine check up (chest x ray, resting ECG, blood lab tests such as : complete blood count, fasting glucose, blood electrolytes, kidney and liver function tests, thyroid hormone levels, blood lipid profile, etc.) to exclude other causes of high blood pressure. It is important to make some modifications to her lifestyle: - diet modifications (reduce salt intake and caffeine)- physical exercise (aerobics, walking)- lose weight if she is overweight. If these modifications are not effective in reducing blood pressure, I recommend discussing with her attending physician the possibility of starting an antihypertensive therapy such as enam (and find the proper dosage with slow tapering). Hope to have been helpful!Best wishes, Dr. Iliri"
},
{
"id": 174548,
"tgt": "What should i do as my baby is suffering from breathing problem?",
"src": "Patient: Hi Doctor,I have a 3mths old baby. Our family doctor gave Muolite Drops for him as he has cold. I gave this 4 drops 3 times a day with Noscoclear (3times a day). but still my baby is suffering from breathing problem. He is not having milk. pls suggest. Doctor: hi Breathing problem with decreased intake of feeds in a small baby may be due to Respiratory tract infection involving lower part(Bronchiolitis) which needs close observation and treatment with Nebulisation+ oxygen.Please take him to a near by child specialist for immediate treatment"
},
{
"id": 21259,
"tgt": "How to control fluctuating BP levels after having 6 stents on heart?",
"src": "Patient: I am a 46 year old female. I have 6 stents in my heart. I have taken Benicar 20 mg for about 8 months. Last Thursday evening I got ahead ache that would not go away. Friday morning when I got up on the right side of my head felt like someone was drilling in it. Later that day I checked my blood pressure and it was 160 / 95. Dr. was closed. I went on Monday and she switched my meds to 40 mg and told me if it didn t go down or started having chest pain to call heart Dr. On Thursday it was still high I went to heart dr office and it was 166/110. They just added HCT to the meds. My blood pressure bottomed out Friday night. It wouldn t read on the machine. Then when it finally read it was 85/52 and my pluse was 118. I got real hot and sweaty and almost passed out everything was like it was far away. I got sick to my stomach. I called the heart dr. and the nurse that was on call just said that it was probly because that my blood pressure was so high and it was trying to regulate is what happened. It was 110 / 85 P was 98 when I called. I not so sure about that. This morning my head still hurts and I feel weak and tired. What should I do? Seams like no one is worried about it except me. I don t have alot of money so please answer for cheap. Doctor: Hi ThereAfter reading all the details provided by you i would like to tell you that your bp medicines needs to be adjusted while keeping your blood pressure under check.Its advisable that you should monitor your bp daily at same time for a period of one month and then visit your cardiologist to get your bp meds adjusted.Till then continue taking Benicar 20 mg and HCT as advised by your doctor.I wish you good health"
},
{
"id": 894,
"tgt": "Can pregnancy happen while having irregular periods and on Glycomet?",
"src": "Patient: Hello Doctor, Iam,22years I got married past 8 months.Iam not able to get pregnent as my periods are irreregular.I come in periods two months.My gynocolagist has given me the tablet Glycomet.Can i still get pregnant while iam on this medication? Doctor: Hello,Thank You for the query at HealthcareMagicMetformin(Glycomet) help to lower blood sugar levels, thereby eliminating excess Insulin in the body. The reduction in Insulin levels have shown to improve fertility and help getting pregnant, especially in women with Polycystic Ovarian Syndrome (PCOS). Continue Metformin as prescribed by your doctor. It is an effective treatment that have shown to help conceiving.Hope I answered your questionPlease address any more concernsRegards,Dr.Lekshmi"
},
{
"id": 50708,
"tgt": "Have lower back pain. Had kidney stones, surgically removed. Am diabetic. Recurrence of kidney stones?",
"src": "Patient: I am experiencing lower back pain with groin pain that seems to get worse in the late afternoon. I had kidney stones surgically removed about 25 years ago as the whole urinary tract was completely blocked. I am also diabetic. Is this a reoccurrence of kidney stones with different symptoms? At what point should I see my doctor? Doctor: How bad is your pain? is it severe? The chances of you having another kidney stone after 25 years is really high. I suggest see your doctor and get a urinalysis and an ultrasound to see the size of the stone and it will guide you and your doctor on what to do with it. Hope this helps."
},
{
"id": 23908,
"tgt": "What causes high pulse rate while on medication for blood pressure?",
"src": "Patient: I am on blood pressure meds. Last couple days my pressure has been 109/88 and 102/89. my pulse has been high. Such as today easily been going between 118 and 121. It not the first time its happened by my doctor doesn't seem worried about it. when my pulse gets faster I feel it. Doctor: Hi,Thanks for writing to Health Care Magic, I am Dr Asad Riaz, I have closely read your question and I understand your concerns, I will hereby guide you regarding your health related problem.The only way any blood pressure medicines may cause palpitations or an increased heart rate is if the blood pressure drops too low or if you get postural hypotension which is drop in blood pressure on standing upI would advise you to get an EKG done preferably during those episodes and if thats not possible for you as they dont last long enough for you to get to hospital, al 24 hour EKG monitoring that is called a Holter is the way to goI hope this answered your question, if you have more feel free to ask.Dr.Asad Riaz.General and Family Physician.."
},
{
"id": 101569,
"tgt": "What causes swelling on face?",
"src": "Patient: Hi, I've been getting swelling (seem to be insect bites) especially on the face (both right and left side), once i developed allergy and my complete face got swollen. Looked for the insect traces in the bed and couldn't find any. First attempt: left side of my face, right hand ring finger, right side of my face. Moreover my husband, baby and myself sleep on the same bed and only me getting this problem. Recently I've been given Ferrous Fumarate as my Heamoglobin content was low. Will this have any relation to my problem? Doctor: HelloAs you are having swelling over the face right and left side with finger also . Sharing same bed with baby and husband . The new drug is ferrous fumarate i.e. iron preparation for elevation of hemoglobin.I want to tell you that this swelling is the side effect of ferrous fumarate .As I have noted in my so many patient that iron toxicity also causes pain in all over body with joints pain and swelling ( as in your case). Two options are there ,1 Either change the medicine or 2 take some anti allergy medicine .In my opinion consult your treating doctor and get his opinion .Good luck."
},
{
"id": 158344,
"tgt": "Diagnosed with breast cancer. It is invasive. Now itching in various parts. Blister appears. Numbness in fingers. Suggestions?",
"src": "Patient: Hello, I was diagnosed with Breast Cancer a year ago. It ended up being invasive, as well, being in the other breast (after a bilateral mysectomy). it appears it did not go anywhere however as of late, I start itching in various parts of my body (primarily hands, feet and upper back) and can't stop. I itch until I blister my skin and it becomes open. On top of this, for the last couple of weeks, I have numbness in the tips of my finger. Do you feel that the \"Green Goblin\" might be back? Doctor: Hi there, Usually itching is not directly a sign of cancer , but along with jaundice in late stages of liver metastases it can be symptom, exclusively itching may indicate a simple skin condition and is best addressed by dermatologist. Numbness of fingers are also non specific and donot directly indicate cancer coming back. See a skin specialist and follow their advise. Regards"
},
{
"id": 162284,
"tgt": "What is the treatment for the G+ve infection with FSF tolerance in a 1 week old?",
"src": "Patient: hi college a full term baby aged one week old and triglycerid were checked twice 4.5 ,the baby on full feeds ,didn t received any TPN,pending the result of lipid profiles of his parents,the baby developed G+ve infection and has been controlled by antibiotics,the baby is ok and tolerating FSF Can you help Doctor: Hello, And must receive additional information from you so that I can learn what it is you want to know. 1. Why does such a young infant need to have his serum lipids (triglycerides are one kind of lipids in the blood) checked? Why twice? What does \"4.5\" at the end of the first line of your question indicate? 2. What part(s) of his body was infected by gram-positive bacteria? Antibiotics are the correct treatment for a gram-positive infection (such as Strep and Staph) of spinal fluid, blood, lungs, skin, joints and bladder. 3. Does FSF mean full-strength formula? What is its full name, please? 4. Why did you mention TPN? Was the infant ill? Did he have a GI problem? 5. What do you need help with? I would continue working with you to answer your questions., if you like. When you return to our website, please ask if you can return to a specific doctor because he wants to continue to help you. If you select the Elite Panel of questions when you return, then we can have a back and forth \"talk\" by computer. I will be at this website Nov 25, 26, 27, 28 from 6 PM to 9 PM USA, EST. Hope I have answered your query. Let me know if I can assist you further. Take care Regards, Dr Arnold Zedd, Pediatrician"
},
{
"id": 15136,
"tgt": "Can wearing calf high socks at work cause red blotchy rashes ?",
"src": "Patient: I work in a spice mill, and I wear long pants with calf-high socks while I am working. Now that it is getting hotter, when I take my socks off at the end of the day, after sweating all day, both of my legs where the socks were, are very red and blotchy. They stay that way for a few hours to sometimes until the next day. Am I allergic to my own sweat? This is something which never used to happen. Is there anything I can do? maybe corn startch? Doctor: HiThanks for writing to us.These red rashes could be due to contact dermatitis which is occuring due to your socks. Right now to treat the same, use a good moisturiser over affected area along with steroid cream like mometasone cream. Also change your socks and wear cotton socks. Make it a point to wash your socks in a mild detergent. The socks that you wear shouldnt be too tight as that can also irritate the skin.Hope this helpedTake care"
},
{
"id": 97133,
"tgt": "Suggest remedy for bump in elbow after stitches",
"src": "Patient: Hi. i have met with an accident before 8 months ago. i got 5 stitches above my right eyebrow and there is a bump on area now . at first the nurse didn t removes all the sutures. i have consulted other doctor and he removed all the stitches. . i have consulted doctor 8 hours after the injury. how can i get rid the bump. . Doctor: Hi and welcome to HCM. Thanks for the query. well,sometimes more than 6 weeks is neccesary till swelling and oedema after injury subside. if it lasts more than this then there could be posttraumatic fibreoma and in this case it may be permanent and only surgery can remove it. so consult your doctor about it.Wish you good health. Regards"
},
{
"id": 7448,
"tgt": "Suffering from recurring acne. Not cured by medications. Safe to use retin-a or clindamycin?",
"src": "Patient: I have a suffered from acne all my life .. 21 years old and im still not able to get a clear face.. my face has a few pimples here and there and mostly its red or pink scars all concentrated on my cheecks and chin .. i have tried many medications but it would only clear while im on medication and as soon as im finished acne would come back.. my face is oily but with some creams like isotrexin my face gets really dry red and itchy.. i have just started taking clarithromycin 250 mg twice daily for a week but i was woundering if i should be using any other medications like retin-a or clindamycin gel and if i can use any kind of bleaching creams .. my face has become darker in the past while.. thank you Doctor: Hi, Hope you are doing great. Meet a dermatologist and discuss the possibility of rosacea along with acne. The two conditions may coexist. Use a broad spectrum sunscreen with a SPF of 30. Apply every 3 hours. Wash your face with lukewarm water. Do not use scrubs. Always pat dry. Use a mild cleanser like cetaphil. Use a cream containing adapalene/clindamycin/ azelaic acid/glycolic acid. Any of these creams can cause irritation, use a moisturiser like cetaphil to prevent it. Hope this helps you. For more personal advice I need to see your pictures. Dr. Rahul Nagar MBBS, MD Consultant dermatologist New Delhi, India"
},
{
"id": 74402,
"tgt": "What is a chest sinus infection?",
"src": "Patient: what is a chest sinus infection? I thought sinus related to head only.Query about my father aged 77. Had bypass surgery Sept 2010. Had pnuemonia in left lung after surgery,also partially collapsed lung. Chest wound still not healed properly.Still occaisonally weeping.Cardiologist diagnosis:chest sinus infection. Doctor: Thanks for your question on Healthcare Magic. I can understand your concern. You are talking about paranasal sinuses (head sinuses). But your father is having Sinus in the chest wound. In normal wound healing, wound close from beneath in 14-20 days time. But when wound is failed to close it remain open and forms track like structure which is open on chest wall and going deep in chest cavity. Most common cause for Sinus formation is wound infection. Once wound infection is treated, Sinus will be healed and closed. Hope I have solved your query. I will be happy to help you further. Wishing good health to your father. Thanks."
},
{
"id": 123941,
"tgt": "What causes pain in wrists, ankles and shoulders?",
"src": "Patient: My joints such as my wrists and ankles and shoulders, mostly the wrists. It is a sharp constant pain that never goes away. No medicine the doctors prescribed has helped. I am 16 years old and I play golf but this has been going on since 13 and I ve only played golf 1 year. How do I fix this? Doctor: Hello, Improve your Bones and muscle strength. Always do stretching & warm-up Exercises before starting your Sports activities. Avoid Sports until you experience Pain. Take Milk and Protein rich diet. If the symptoms persist then go for Vitamin D, B12 tests, to rule out the Deficiency & X-Ray of Wrist & Elbows, to rule out Wrist/Elbow Inflammatory disease (Wrist Tenosynovitis/Carpal Tunnel Syndrome/Tennis Elbow/Golfer's Elbow etc). Hope I have answered your query. Let me know if I can assist you further. Take care Regards, Dr Saurabh Singh Rajan, Sports Medicine Specialist"
},
{
"id": 37161,
"tgt": "What is the treatment for a leaking umbilical hernia?",
"src": "Patient: I have an umbilical hernia that seems like it may be leaking a tiny tiny bit. It's more red than usual today and I have stomach cramping. I'm concerned with the tiny area that skin appears shiny like it may be leaking a little bit. Should I go to the emergency room? Doctor: Hello,Welcome to HCM,Your umbilical hernia may be getting worse and the pain is being caused by fatty tissues getting trapped in the hernia. This can reduce the blood flow to the tissue and cause the pain. Your symptoms may be relieved by lying down.If the hernia is the cause of your discomfort, there is a risk that tissue may get stuck and become irreducible. This would result in the need for an emergency operation to relieve the trapped tissue.Therefore, I suggest you see your doctor who will be able to examine the umbilical hernia and assess whether it is the cause of the problem.The only definitive treatment for umbilical hernia is surgery. An uncomplicated umbilical hernia is harmless. Only when it develops complications like obstruction or strangulation, it becomes an surgical emergency.Thank you."
},
{
"id": 17125,
"tgt": "What causes hiccups and low output of urine after an ischemic stroke?",
"src": "Patient: Hi. My father is the patient. he s 80 yrs old and had a stroke 5 months ago-he was diagnosed with an ischaemic stroke. Comorbidities are: hypertension, diabetes mellitus and prostate cancer. he s currently discharged from the hospital. he s been having very high blood pressures at night like 194/90 and during the day, especially when we sit him down, it falls so low like 90/70 for about a month now. he also started having seizures some weeks ago, about once a week and now, for the past two days, he s been vomitting. his urine is also highly concentrated and sometimes in a day, urine output is less than 100mls. his appetite has also declined and he s lethargic with lots of hiccups. can you please tell me what is happening? Doctor: Hi, Low urine output and hiccups, vomitings suggest uremia/renal failure; need to know BUN, serum creatinine, serum electrolytes and urinalysis report apart from full blood count. He is also having postural hypotension from lying to sitting position (lying to standing -more sensitive but lying to sitting is sometimes all that is possible for a less mobile patient). Prolonged bed rest and chronic hypertension (decreased baroreflex sensitivity and LV compliance) and? drug and diabetes (autonomic neuropathy) could be the cause. He may require renal replacement therapy. Hope I have answered your query. Let me know if I can assist you further. Regards, Dr. Tushar Kanti Biswas, Internal Medicine Specialist"
},
{
"id": 5667,
"tgt": "Taking thyronorm. Trying for pregnancy. Need suggestions",
"src": "Patient: Hi Doctor,I am looking for Gynacologist advice I am 35 yr old and have a 8yr old daughter,my husband is 40yrs now. We are planning for 2nd child now from the past 1 yr but in vain. This october 2012 I was diagonised this THyroid on border line. Doc advice to take Thyronorm 50mg . Now the throid levels are normal After sex the semen spills out of the vagina even after keeping a below under the hips and spill is around 2-3 times after intercourse. Kindly suggest what is the best measure or method to ensure pregnancy occurs. Doctor: Hello there, Thanks for writing to us You need to continue thyroid medication. You need to consult a gynaecologist and need to get tests done to find out any other factor for infertility, like semen analysis, tubal patency test by HSG, baseline USG. Depending on the results of the tests, you would need so e treatment for conceiving. Hope this is helpful to you. Dr. Mrs. Sumeet Baheti"
},
{
"id": 47633,
"tgt": "Are there chances of kidney swelling and high creatinine in a new born?",
"src": "Patient: I would like to tell you that my baby is 16 days old and at the time of birth he had braething problem according to the dr. But dr. also found that there was bleeding from the way of urine and also blood in urine it was continuing for two days after that it stoped. As dr. said that the creatinine level is more than 2 and it come down near to 1.5 but it has raised up to 2.4. After that dr found the reason of creatinine high is that there was a obstracle in the way of urinine. They cleared that way of urine. But the creatinine level came down from 2.5 to 2.06 and again it raise up to 2.4. Now dr is saying that the 75% of kidney has been damaged and it was fault from bybirth. That s why the creatinine level is not coming down. I am very much worried about it if the creatinine will not come down and it may cause that I may lost my child. Dr are going to discharge my baby on friday. One thing that dr say there is sweeling in kidney but I have t saw swelling on his body Doctor: Hello and welcome to HCM.As an Urologist, i can fully understand your concern.Kidney obstruction is diagnosed, on ante-natal scans, done before birth.All obstructions(blocks), are re-confirmed by repeating the scan,after birth. If treatable by surgery, kidney function is checked by doing an isotope renal scan, along with the blood tests like Hb, and creatinine. You've not mentioned, how was the obstruction cleared by doctor. I can give an expert opinion, if you send me all the reports,as a direct question, in my name.Dr.Matthew J. Mangat."
},
{
"id": 192209,
"tgt": "What causes rashes and itching on the tip of penis?",
"src": "Patient: hello sir ..i m aryan from chandigarh.... i m in a relation with a girl from last 4months.we made physical relation many times.....two times in her periods..... three or four time i dont use condom.. and one time in periods.... now i came toknow that she has also physicalrelations with four boys... now my problem is that frm some days i feel itching inside the tip of my penis...now i feelit in the middle of the penis.....in starting also some raches are on the tip of the penis bunt now there is no raches..... plz tellme what todo.. is any thing serious.... Doctor: Hello, If the rashes continue and there is still pain and tingling sensation in the area then i suggest you get tested for herpes and syphilis. Hope I have answered your query. Let me know if I can assist you further. Take care Regards, Dr. Manuel C See IV"
},
{
"id": 216429,
"tgt": "Suggest treatment for chest and back pain",
"src": "Patient: Yesterday I was having really bad chest pains and back pains I thought I was having a heart attack went to emergency room and they took my ekg , the pain was so bad it hurt standing straight up ,burp, cough or drink something or eat it really hurts , today the pain is still here but not as bad as yesterday today it's more in my right upper back , and something hurts when I cough , burp , drink , eat , chest hurts alil , bending down hurts moving my right arm hurts what can this be ekg came back normal Doctor: Hi, sometimes severe acidity can cause an inflamed food pipe (esophagitis), that can be severe. Please take a liquid antacid with local anaesthetic like lidocaine in it for 2-3 days. It should improve. Regards"
},
{
"id": 99507,
"tgt": "Suggest medication for dust allergy",
"src": "Patient: my daughter is suffering by dust ,smale and unknown eliments from a long time according to doctor.and she getting breathing problem at those times.she is now 8+yrs old .reading in class three.doctor has given her inheller and a tab montare.in a deep suffering condition we have to give her asthalin and budecord by nebuliser mechine.grouth rate of her health is very poor .can not want to eat food.less energetic .so what i have to do now.pls give me your valuable suggetion to keep her normal .we both are very much orrid about that. atanu bandyopadhyay. jalpaiguri.0000 mail - YYYY@YYYY Doctor: Hello Mr Atanu Bandopadhyay,I went through your daughter's history and would like to know more about her as follows:a. What are her nose symptoms - running nose, sneezing, nose congestion - and how frequent?b. What are her chest symptoms - wheezing, breathing difficulty - and how frequent?c. Does she get symptoms while physical activities like exercise, outdoor games, etc?d. Does she have frequent headaches?e. Does she have eye symptoms - red eyes, itchy eyes, excessive watering?f. Does she have any digestive complaints - frequent acidity, vomiting, constipation, frequent stomach pain, etc?g. Does/did she have any skin allergies like dermatitis or urticaria?h. Day symptoms or night symptoms?i. Seasonal variations?Above details would help me to know more about her health and therefore to make more specific suggestions for her.At present, from your provided details, I would like to make suggestions for her as follows:1. For identification of allergies, I would suggest her skin testing for common air-borne allergens like house dust mites, pollens, molds, insect proteins, animal dander (if you have pet), etc. This will help you identify the substances causing allergies to her as well as to know the measures to avoid them.2. Based on allergy testing, an Allergist-Immunologist may also prescribe her allergen specific immunotherapy which works on immune system to gradually improve her allergy symptoms over a long period.3. For symptom relief and prevention, I would agree with your current treatment. I usually suggest my such patients regular levocetirizine + montelukast.4. Allergies and asthma are a bit worrisome but not very much. With proper understanding of the disease, care and use of medicines children can live very normal lives like most other normal children.5. Please avoid exposure to dusts, smokes and air pollution as much as possible.6. A healthy diet rich in vitamins & minerals as well as regular physical exercise including Yoga & Pranayama will also help her in a long run by improving her immunity and lung capacity respectively.Hope above suggestions will be helpful to her.Should you have any further query, please feel free to ask at HCM.Wish your daughter the best of the health ahead.Thank you & Regards."
},
{
"id": 22568,
"tgt": "What causes high blood pressure inspite of taking Cozaar?",
"src": "Patient: I've been in a high stress situation for 2 days and my b.p. Has been as high as 180/116 and won't go below 155/98. I'm on 100 mg Cozaar and it normally doesn't exceed 135/85. Planning to call dr in the morning. Anything else I can do in the interim? Doctor: Hi,Cozaar will take some time to act, if there is no chest pain, headache or shortness of breath, then there is not urgency of getting down bp. You monitor it thrice a day and if still remains high then need to add some more medicine usually Amlodipine. You can get this prescribed from local doctor. Also use some relaxation techniques like yoga and regular exercises to reduce stress. Also have low salt diet, healthy food. Get your lipid and sugars tested. Avoid smoking and alcohol if any.Hope I have answered your query. Let me know if I can assist you further.Regards,Dr. Sagar Makode"
},
{
"id": 50366,
"tgt": "Did nephroectomy, urethroceal. Pain during passing urine, intercourse. On monilia, visicare. Pain reduction possible?",
"src": "Patient: Hi doctori am a woman 48 yrs old before 12 years i did nephroectomy for the right kidney ( it was atrophy and 11 5 function problem start with first pregnancy and repeated in the second ) last year( 14/April) i did another operation in the bladder to remove urethroceal which cause to me chronic UTI , still i am suffering of pain during passing urine , pain in intercourse ,can not control urine, swelling in right foot ( like elephant foot ) my doctor who did the operation keep given me Antibiotic which cause me alot of problem like monilia , diarrhea some times bloody mycoid stool, stomachs pain , also he keep given me visicare 10 mg ., and recently gave modurtic 50 mg with all previous medicine i mention ...my question is i am fine ,is it normal , and when i can releive all pain and swelling ??? Doctor: HI Thank for choosing HCM, sorry for hear that , now you do one thing go for urine culture sensitivity test, drink lot of fluid, do not hold urine go for urination as soon as it is possible,, do not worry it will come around,,, have nice day."
},
{
"id": 55065,
"tgt": "What causes nausea after gall bladder stone surgery?",
"src": "Patient: Why am I so unwell after Gall bladder removal? I did not have stones but a gall bladder that was not pumping. I am now suffering from nausea ,and if I don t have that I have the runs. It feels sore and tender inside like it was just operated on. Feel like crap most of the time. I have also had a stool test done which showed very low in digestive enzymes. My operation was 14/8/2014. Many Thanks Doctor: hi.noted recent gallbladder surgery and post-operative nausea. recovery after surgery is different from one case to another, depending on the pre-operative condition/co-morbidities. are you eating well? even those who has been operated on several years back, could occasionally experience pain/discomfort on the operative site, but tolerable in character and resolving spontaneously. it is best if you do a follow-up consult with your surgeon for physical examination and clinical evaluation. diagnostics (such as scout film of the abdomen, ultrasound, electrolytes, etc.) and management will be directed accordingly.hope this helps.good day!! ~dr.kaye"
},
{
"id": 74795,
"tgt": "What causes breathlessness and numbness in the hands and fingers?",
"src": "Patient: Hi! my girlfriend had a really bad experience last night around 2 a.m. First of all she got cold and she was feeling bad for the past few days but last night she was so upset, she was having big trouble breathing because of the mucus and she start crying. After about an hour her both hands (mostly the right arm through the hand) go numb and she wasn t able to open her fingers without my help. Also her feet got cold during this attack. I did some massage to her arms, hands, neck and feet. She was finally better and we could sleep and now she feel fainted, no strength to get up. Do we need any kind of medicine or it was just a normal attack when you cry so much? I m trying to find an answer and i will appreciate your help! Thank you. Doctor: Respected user, hi I evaluated your query thoroughly.* Relates to profound hypothermia effect on the body .* Follow certain steps at home for faster recovery- turn on room heater maintain room temperature properly till she is in comfortable zone not suffering tremors .- cover her body with enough warm clothing .- give more liquids by mouth preferably hot coffee, green tea , home made juices .- avoid stress / anxiety / panic .- refrain from smoking / alcohol at present .- convey her to relax , deep breathing exercises help a lot- tab. acetaminophen (325) mg as stat dose .- if possible get her Blood pressure checked by digital machine .- consult in ER if required .Hope this helps you.Welcome for any further assistance .Thanks for using Health care magics & reviewing my answer.Wishing her fine recovery.Regards dear take care."
},
{
"id": 39873,
"tgt": "What are the treatments for cytomeglavirus and sarcoidosis?",
"src": "Patient: Hello there...I have the Cytomeglavirus and it has been active since June last year..I have only one Kidney left and also have Sarcoidosis..What can I do to alleviate the symptoms of the Virus please as its making my life miserable..Many thanks in advance..and to be honest I have no money to pay for your advice..Im sorry I will have to leave this question answered..as I cant pay..Sorry to have bothered you Doctor: hi, Cytomegalovirus is highly treatable with drugs like Gancyclovir and foscarnet. I guess you have probably been kept on them. Sarcoidosis is treated with systemic steroids and it also shows good response. There is little to worry. all you need to do is to consult a doctor and get treatment. Hope this helped."
},
{
"id": 91877,
"tgt": "Why do I have pain and swelling along the incisions of abdominal surgery?",
"src": "Patient: 12 weeks ago had emergency ab. surgery for adhesions, which was successful. was gently exercising (stretching)and now have pain and, I believe, some swelling along incision, with some focal soreness. No obvious hernia, butwondering what is realistic time frame for my pre-surg. flat, toned stomach to return. still looks like I'm a bigpregnant. Will this take a year for internal swelling to go down? I am only 108 lbs! never mind.am not paying for answer as ex-H is internist. Thanks anyway. Doctor: Hi. Pain and swelling at incision site after exercising can be due to hernia. It may be easily noticeable in the beginning.IF you have no attendant complications , it may take 3 to 6 months to return the shape.There should not be any intestinal swelling as it will cause you pain/ vomiting/ constipation .Go for high resolution sonography and a X-ray of abdomen in a standing position."
},
{
"id": 66928,
"tgt": "Suggest treatment for a lump under the jaw",
"src": "Patient: About 10 days ago I noticed a lump Right under my jaw at the bottom of my earlobe it was sore and tender and occurred the day after I had my teeth cleaned at my dentist. The doctor gave me an antibiotic and it went away but now about a week later I have a larger piece of skin looks like skin tag on the back of my tongue. It wasn t there yesterday and seems to have popped up overnight Should I be worried Could this be related ? Doctor: it is a benign cystic tumor-like condition that is prone to infections repeatedly!not to worry much; just do an FNAC test for confirmation and consult an ENT specialist.all the best!"
},
{
"id": 214275,
"tgt": "Suggest remedy to get the toe color back",
"src": "Patient: Hi I used vingear & bread & wrapped up my toes with corns for 2 days overnight....after a shower I was able to quickly remove the dead skin which was white but is now pink and I cannot do that treatment anymore cause they burn if I do try this again... what home other home remedies can I use to get the color back on my toes Doctor: You have not done the right thing. The vinegar is an acid that has eaten away the skin of the area and you have simply peeled off the skin from the area. As a result, the area is now red and you are feeling the burning sensation. Please do not do it again. Be patient, apply some antibiotic ointment and some ice and in a week or so, the portion will heal as best as it can."
},
{
"id": 220170,
"tgt": "Is skin irritation after taking Novaclox for increased TLC during pregnancy normal?",
"src": "Patient: Name: Pratibha Sharma Age: 31 (F) I am pregnent from last six month. My LMP is 14-09-10. Last week I got fever all test were normal except increase in TLC (11500). My Gyne doctor advised me to take Novaclox, but it causes skin irritation alongwith I don t like anything to eat. There is pain observed from Coller Bone to right abdomen. Please advice me. Doctor: Hello dear,I understand your concern.In my opinion the elevated leucocyte might not be considered as abnormal.Because there is elevation of leucocyte count upto 15000 in pregnancy.So you need worry regarding the elevated counts.Relax.hThe pain from collar bone to abdomen might be due to muscular pain.Some or the other type of non specific pain appear in pregnancy which subside on their own.Don't worry.Take Panadol if pain is severe.The skin irritation might be due to sensitivity to antibiotics.Take adequate fluids upto 3 litres per day to prevent dehydration and UTI.Hope this helps.Best regards..."
},
{
"id": 144444,
"tgt": "What causes dizziness and tingling on tip of tongue?",
"src": "Patient: I have tip of tongue tingling and have been dizzy for two days now...havent changed any medication nor do I have any allergies...diet has been the same..please advise not sure what fo make of it....oh I dohave high blood preszure and am taking medicine for that Doctor: Hi Dear,Welcome to HCM.Understanding your concern. As per your query you have dizziness and tingling on tip of tongue. Well there can be many reasons for symptoms you mention in query like nerve entrapment , neuropathy, diabetic neuropathy, multiple sclerosis, diabetes multiple sclerosis , dry mouth and TMJ problem . I would suggest you to consult neurologist for proper examination . Doctor may order nerve conduction test and take history to confirm the diagnosis . Doctor may prescribe nerve supplement or may consider surgery in severe nerve damage . Doctor may also refer you to endocrinologist for diabetes or oral surgeon for TMJ problem . For now keep yourself hydrated , do warm saline rinses and maintain proper oral hygiene . Hope your concern has been resolved.Get Well Soon.Best Wishes,Dr. Harry Maheshwari"
},
{
"id": 193275,
"tgt": "Provide details of effect, side effects and rest needed after gynaecomastia",
"src": "Patient: hi...i am 20 years old & weight 65kgs,suffering from gynecomastia.i have avg boobs size.i have consult the doctor & he adviced the surgery.i need some information abt surgery.any side effets are there?how many days it will take to get skin fit in chest area.can i get perfect chest shape.is there any chance for again coming of man boobs?how many days i have to take rest after surgery? Doctor: Hello,After surgery it takes around 2 weeks to heal the area on average. You need to rest after surgery and wear compression garments. Surgery is generally considered safe however, there are associated risks of infection, bleeding and anesthesia complications. Yes one can have gynecomastia after surgery but its very uncommon. 90% of the patients after surgery get complete cure.Hope I have answered your query. Let me know if I can assist you further. Regards, Dr. Sameen Bin Naeem, General & Family Physician"
},
{
"id": 102128,
"tgt": "What are the reasons for dryness and swelling inside lips?",
"src": "Patient: I have been treated for actinic keritosis for the past 11/2 years. However, the inside of my lips are still swollen and my lips are very dry. Could something else be going on. I have been tested for food allergies, lupus, Vitamine B 12 defiiency, shrogrins. Thanks for your help. Rose Chamblee Doctor: I think the medicines u are using can cause this in my opinion change the salts of medicines you are taking for other problem"
},
{
"id": 90358,
"tgt": "What is causing pain in abdominal wall close to pelvis?",
"src": "Patient: I have crampy, achy pain in my abdomen, but it is not affected by eating. I think it is in my abdominal wall - worse down closer to my pelvis. I have had a bad head cold - could I have injured abdominal muscles from blowing my nose? Seems to remit after taking NSAIDs. Woke up with is yesterday. Doctor: Hi,Their are many reason,It may be simple muscular pain,Apendicities,amebiasis,renal colic,it may be hernia,Your detailed history is essential,is it associated with nausea vomying, does it radiate to back,If its simple muscular pain it will be look k on it own,in day or two,If problem persist,You can ask direct question over here or consult your physician."
},
{
"id": 170662,
"tgt": "Suggest remedy for abdominal pain during cough",
"src": "Patient: I wasn t sure which specialty to pick, so I just picked this one. A few minutes ago I coughed and it felt like something in my lower left abdomen (around the ovary/uterus region) split open and the next few times I coughed that pain was there again. Could this mean anything really bad? Doctor: Hi, this is just spasm of muscles which has occurred due to severe coughing. I think that you should not worry about this. This will heal by itself. Take care."
},
{
"id": 84789,
"tgt": "Does trigestrel pill posses any impact on menstrual cycle?",
"src": "Patient: hi Doctor i realised about 2 weeks later that i was taking the Trigestrel pill,the other way around. can it affect my periods? because i havent received my period this month,and i havent been sexual active since end of June,but i got my period in July. Doctor: Hello, Trigestrel is mainly used as an oral contraceptive pill although it is used for other causes like menstrual irregularity. It shouldn't affect the menstruation negatively. But, sometimes due to hormonal changes or stress changes in the menstrual cycle are observed. Do not worry, there are other factors like stress which cause delayed menstruation. I would suggest you wait for a few days and if this persists, do visit a gynecologist regarding this, depending on your history the doctor will ask you to get some tests done if needed. Depending on this, the doctor will be able to guide you with appropriate treatment. Hope I have answered your query. Let me know if I can assist you further. Take care Regards, Dr Manisha Auradkar, General & Family Physician"
},
{
"id": 66268,
"tgt": "What causes painful lump on top of butt crack?",
"src": "Patient: At the top of my butt crack it feels as if there is a lump and it hurts a lot whenever any pressure is put on it, its hard to even sit down. It feels hard at first but you can press it, I choose not to cause it is very painful. I'm not sure where it came from or what it is so any help would be fantastic. Doctor: Hi, thanks for writing to HCM! If I were your treating Doctor for this case of painful lump on top of butt crack, I would come up with three possibilities, these include: 1.\u00a0\u00a0\u00a0\u00a0\u00a0skin or hair follicle infection due to some friction injury or pilonidal sinus2.\u00a0\u00a0\u00a0\u00a0\u00a0The second possibility is of an ischiorectal abscess or some papilloma/neurofibroma getting infected\u00a0\u00a0\u00a0\u00a0\u00a03.\u00a0\u00a0\u00a0\u00a0\u00a0The last possibility is of some infected dermoid cyst or benign tumor / tumor like condition \u00a0\u00a0\u00a0\u00a0\u00a0Overall, it is inflammatory or benign and not worry about this!I suggest you to see a surgeon for examination and few tests for confirmation and to determine the line of treatment!Hope this answers your question. If you have additional questions or follow up questions then please do not hesitate in writing to us. I will be happy to answer your questions. Wishing you good health."
},
{
"id": 184500,
"tgt": "What causes bad odor in mouth?",
"src": "Patient: I have a bad odor in my mouth. I have been to a dentist nothing is wrong with my teeth are gums. I am not diabetic. I use all kinds Of expensive mouth wash. Nothing is working! The upper part of my stomach hurts when I cough. I can not get rid of this bad breath . What could be causing this? Doctor: Hello, Thanks for consulting HCM, Read your query, as you have bad breath this can be due to deposition of food debris , plaque , calculus in subgingival area of gums , dental caries , yeast infection in mouth , dry mouth , decreased salivation, habit of chewing tobacco , drug induced or upper respiratory infection like pneumonia , bronchitis , liver or kidney related disorder . For this you should consult dentist and go for oral prophylaxis , Regular use of mouthwash , warm saline rinses , use of waxed dental floss to clen interdendal areas of teeth , if your bad breath continues then consult physician and go for examination and treatment. Hope it will help you. Wishing you good health.Regards , Dr. Priyanka tiwari"
},
{
"id": 181996,
"tgt": "Suggest treatment for tooth pain",
"src": "Patient: Hello doctor,I was advised a root canal couple of montha back..but to my work schedule i was not able to get the treatment done. I am having severe toothache since yeaterday and my appointment with doc is at 8pm today..yesterday i had oxalgin painkiller.. i had diclogem at 9am..then oxalgin at 11am. But there is no relief at all.. Doctor: whenever nerve or pulp of the tooth gets involved, the only definitive treatment would be root canal, medications can only give you transient relief. Get your RCT done at the earliest."
},
{
"id": 6103,
"tgt": "Trying to concieve, transvaginal ultrasound shows ovulation, egg released spot bleeding, urine test negative, scheduled for laparoscopy. Reason?",
"src": "Patient: I ve been trying to conceive for almost 2 years, I had a transvaginal us yesterday and my dr showed that I had ovulated on my left and the spot where the egg was released filled with blood. He moved the wand to check the right then asked me if I was sure I wasn t pregnant and when I told him I had just tested negative he immediately removed the wand and did a urine test (not blood). It came back negative then scheduled me for a laproscopy for the following week. My question is what did he see that made him think I could be pregnant and should I push for a blood test before the lap in case I m pregnant. My period has yet to show but I ve been spotting on and off brown blood for about a week and a half. Comes and goes Doctor: Hello Welcome to HCM forum , Lap procedure is done to confirm the patency of tubes as well as to see any ovarian and uterine pathology It can be seen with HSG also but diag lap gives a more clear picture and one can do ovarian drilling if needed Sperm count and motility should be normal Best wishes"
},
{
"id": 55195,
"tgt": "What does ALT-47,Bilirubin-0.6 and Alkaline phosphate-59 suggest?",
"src": "Patient: I am 51 yrs old, 5'4\" and 138 lbs female. My AST (SGOT) is 53. I had a skin cancer removed 3 months with reconstructive surgery and have been on oral antibiotics twice since the surgery. The most recent was about 2-3 weeks prior to my lab draw. My ALT is 47 and my bilirubin 0.6, Alkaline phosphate 59. Should I be worried? In April 2011 my AST was 24 & ALT was 21. Doctor: Hi thanks for asking question.Noted you have very mild elevation of liver enzymes.You also have history of cancer.So here your enzymes elevation could be secondary to cancer.Do first USG of abdomen to rule out fatty liver like pathology.If liver pathology present then take care regarding avoiding trans fat , taking more fruits , omega 3 fatty acid oil good , regular exercise.You have not mentioned which cancer was present.If melanoma was the cause although chances here less but liver metastasis can present that should be ruled out by USG and FNAC if needed.If you have muscle trauma or muscle pathology then also enzymes can be elevated.Also if heart problem history present in nearby past then it can also lead to elevated enzymes.Wish you good health.Take care. ."
},
{
"id": 84237,
"tgt": "What the Placentrex injections are for?",
"src": "Patient: I have undergone Laparoscopy surgery 15 days back and now my doctor prescribed me Placentrex injection for 10 days daily. what for these injection used and is there any side effect. I report does not show any tube blocks but she told that tube is swelled it does not allow for follicles to grow. please suggest Doctor: Hello,Placentrex injections are used to treat inflammation, pelvic inflammatory disease, to reduce the risk of infertility and adnexal mass, any type of tubal damage, fornix tenderness and uterine restricted mobility. You can take the injections it is helpful in conceiving. But side effects are associated with it, like weight gain, headache, dizziness, change of appetite, acne, nervousness, etc. If you have such symptoms you should report urgently to your treating doctor so that the doses can be monitored.Hope I have answered your question. Let me know if I can assist you further. Regards, Dr. Nupur K, General & Family Physician"
},
{
"id": 143983,
"tgt": "Suggest treatment for lower back pain in a scoliosis patient",
"src": "Patient: I am having lower back pain... during the day it is a minor inconvenience just an dull ache but when I sleep if I lay on my right side I will wake up with a sharp stabbing pain in my lower right back to the point I have trouble getting up or standing for a few minutes then it subsides. I have been to the chiropractor and have an issue with scoliosis (sorry about the spelling) Doctor: Hi, thanks for asking from HCM.I can understand your concern. If you are having severe backspin on lying down on right side, it might be due to paraspinal muscle injury on right side. Such muscle injury usually manifests as dull pain on same side on standing, pain becomes severe on stretching like bending/lateral bending towards left side or pressure on muscle while lying down/direct pressure. Tenderness can be felt at pain site. There is one close differential diagnosis of it - Lumbar disc disease. It can present as pain on either side, pain is sharp/current like/radiates upto same side thigh-calf-foot. Pain becomes severe on forward bending/standing/sitting and some relief on lying down. Your backpain is more like of muscular type. It is not getting relieved due to continuous exertion. You can follow few tips : Avoid prolonged physical exertion: Avoid weight lifting: Ask some family member to massage your back. Apply hot fomentation over back thrice a day.: Apply painkiller gel like Diclofenac thrice-four times a day after proper prescription.: You can take Tab Thiocolchicoside-Diclofenac combination after proper prescription. : Avoid pressure at pain site.: Continue Chiropractor assistance.If still no relief, consult doctor for examination of back. If there will be doubt of disc disease, he will advise MRI of lumbar spine.Hope it will help you. Thanks. Take care."
},
{
"id": 127411,
"tgt": "What causes tension across the shoulders and neck accompanied by dizziness?",
"src": "Patient: During the past week I have had 6 or 7 episodes of severe tension across my shoulders, up my neck, and into the back of my head accompanied with dizziness and tunnel vision. They have only lasted a couple of minutes and resemble past experiences from working overhead for a long period, only now I have not been doing so. They subside after sitting and resting for a few minutes. Doctor: Hello and Welcome to \u2018Ask A Doctor\u2019 service. I have reviewed your query and here is my advice. Tension across the shoulder with dizziness can be a result of severe cervical spondylosis, which is a degenerative condition. But other conditions like vertigo should also be ruled out. I suggest you take an X-ray cervical spine and review with it and also to consult with an ENT specialist. Hope I have answered your query. Let me know if I can assist you further."
},
{
"id": 214676,
"tgt": "Suggest home remedy for dark spots on face",
"src": "Patient: Doctor,my name is sharath from Bangalore,karnataka.. Due to bursting of the pimples, my face is filled with dark blackspots.. Can u plzzz tel me a home remedy to get rid of these black spots?? From past two days,i am applying lemon juice to the black spots.. Is it ok..??? Doctor: **1. since there is post inflammatory hyper pigmentation, in best Interest it will be wise to see a Dermatologist for proper evaluation and management.2. stay away from the sun during peak hours [11 AM to 4 PM] or ensure a good sunscreen applied on the area to limit damages.. wash your face with Besan and luke warm water after coming from outside (best natural bleaching agent), removing excess sebum and dead skin cells, and acts as a deep cleanser.3. Potato acts as a very good bleaching agent, this would lighten the marks.4. Take two spoonful of gram flour, add a pinch of turmeric, few drops of lemon juice and make a paste with a spoonful of curd. Apply this pack regularly and you will notice drastic changes.5. Make a mixture of tomato juice and lemon juice, apply this mixture daily with a cotton ball. It lightens the skin tone and marks.6. i. Use fresh vegetables and fruits, stay hydrated (drink 10-15 glasses of water/ day)ii. Avoid: Sugar (in any form), caffeine, cheese, oily-spicy-dry-fried food, chocolates, cakes, iodized salt, soft drinks.iii. Avoid exposure of the skin to severe climatic conditions.PS. . To get proper result, you are suggested to use any cream for 3 to 4 months. You may ask for Prescription Drugs, maily: aloe vera cream, kumkumadi oil, khadira arishta,saariva assav."
},
{
"id": 63059,
"tgt": "What causes a lump in the lower back?",
"src": "Patient: Hi I have a lump about the size of a half golf ball in the lower right of my back ( just where the dimple is ) It hurts me most when I try to get out of bed in a morning, or when I get up after relaxing, a massuer confirmed that its not a muscle knot, I am waiting to go for a scan, what could it be Doctor: hi.it is best if you consult with a doctor, preferably a general surgeon, for medical and physical examination. based from your description, it could be a cyst (a sebaceous or a keratinous type), a fibroma or a lipoma. these lesions can occur anywhere in the body and have the tendency to recur. management (medical and surgical) will be directed accordingly.hope this helps.good day!!~dr.kaye"
},
{
"id": 130695,
"tgt": "Should i be concerned about painful sprained toe after an injury?",
"src": "Patient: hello, about three months ago i believe i dislocated/ sprained my big toe really bad. I was skateboarding when the incident happened. i landed on my skateboard side ways and my toe was jammed to the front of my shoe. it hurts really bad when i push it inwardly towards my other toes\u2026i still cannot run and i can only walk on it for a short period time before it starts to get really sore. the only way i can go about my day is by doing a turf toe tape job on it, without the tape it gets really sore. do you think i need surgery or maybe therapy? Doctor: Hi,Do hot and cold therapy (have one bucket of hot water and another of cold water, dip leg in hot water for three minutes and then dip in cold water and remain there for one minute). Do it 7 to 10 times. Its good you did tapping done. Just continue doing it, do active movements of your toes when ever possible in a day and early morning before getting out of bed do stretching of sole of foot. Continue doing it for one month I feel this will help you and save you from undergoing any surgical procedure.Hope I have answered your query. Let me know if I can assist you further.Regards, Dr. Harsh Swarup"
},
{
"id": 118013,
"tgt": "What are the symptoms of anemia?",
"src": "Patient: Hi, my maternal grandmother had Talisma blood disorder and was diagnosed in the U.S. many years ago, no one in Canada could figure this out. I am having similar problems with my health but wanted to know what are the exact symptoms of the type of anemia. My parents are both Italian Doctor: Features of anemia are generalised weaknesses, easy fatigue pallor, high pulse, in severe cases vertigo dizziness may occur. These are general symptoms. And also many more are there. You should go for complete blood count and peripheral smear examination."
},
{
"id": 166200,
"tgt": "Is fever in the evening with congested chest be a symptom of typhoid?",
"src": "Patient: My daughter is 3 yrs of late she she getting fever in the evenings and in the morning her fever is little less or normal.some time she says he has a pain in her stomach.her chest is congested.is it typhoid.doctor geve her corocine and asked us to moniter after every 6 hrs Doctor: hi, chest congestion with fever points towards upper respiratory tract infection. In typhoid there is history of fever without any focus, and the fever continues to increase. Without treatment fever increases in typhoid and can cause complications like pain abdomen, pneumonia, hepatitis. Kindly do complete blood count, typhidot IGM hand x ray chest. It will help in ruling out typhoid and chest infection. temperature chart it should be done so that we may know that whether temperature is increasing or not. Review with reports. Take care."
},
{
"id": 219013,
"tgt": "How to detox from smoking during pregnancy?",
"src": "Patient: I am 33 weeks pregnant and was smoking up until two days ago and haven't took a hit since.for the past 2 months all I drink is water and eat ice no Soda or juice.i think I might be in labor will it show up in the baby's system and if so what can I do to clean the baby's system and how long does it stay in the baby system Doctor: Hi, Thanks for the query. I understand your concern. Smoking has it's dizards in firm of allergy/respiratory disfunction in form of bronchitis providing less oxygen & less provision for energy needed for fetal rearing. Baby is likely to have less birth weight &less resistance. But as you have left smoking now...baby will get better energy for rest of weeks (about 5-6 weeks) & will recover to some extent from the damage. -It will also help you in giving birth .. the delivery . To make up the loss you should have nnourishing diet with proteins (milk,pulses,non veg food except for liver)/vitmines&minerals (plenty of salads,fruits . Take B complex,vitamin Esuppliments along with routine calcium &iron . Do not ever try yo smoke again... not while the baby i\u00df by your side,for the indirect inhalation of dmoke would dsmage babies respiration . Take care thanks."
},
{
"id": 144624,
"tgt": "What causing stomach bloating,headache and diagnosed with erythromalagia?",
"src": "Patient: i was recently diagnosed with advanced microvascular ischemic disease secondary to an MPN (JAK2 positive either ET or PV ... diagnosis seems to go back and forth. BM Biopsy pathologist indicated ET with differential diagnosis of pre-myelofibrosis). MRs of Brain revealed multiple white matter hyperintesities both hemispheres and a larger lacunar infarct in an area affecting executive function. i was also diagnosed with Erythromalagia. i am concerned about possible Cognitive Vascular Disease as i have noticed significant cognitive decline over the last year or so. I am also wondering if i should have diagnostic procedures to see if the small vessel disease has affected other areas of my body or organs or if there are blood clots anywhere in my body. I have persistent headaches, stomach bloating and pain and leg pain (could be bone pain). i am currently being treated for high cholesterol (just after the MRI findings) and also Hydrea. The RBCs have dropped to within range (also had a phlebotomy) but the platelets have gone up since treatment started 3 weeks ago. What other diagnostic procedures would you recommend to identify damage or decreased function in other organs or areas of the body? Thank you so much in advance. Doctor: You have not mentioned your age and other associated medical problems. The risk factors for ischemic disease need to be evaluated (other than cholesterol level)-1. serum homocysteine level2. serum vitamin B12 level3. ANA, C-ANCA, P-ANCA (to look for vasculitis)4. hs CRP5. Apo-A"
},
{
"id": 180167,
"tgt": "Suggest suitable medication for high fever",
"src": "Patient: hello sir, my daughter is 4.5 years old, she was having mild fever for the last two days, but since afternoon she is getting fever in every 4-5 hrs, in the afternoon (12:30 pm) fever was around 102 F, WE GAVE HER meftal P, temperature gone down. in the evening (6:30pm) fever was very high ( 104 F) with shivering, AGAIN meftal P (3.75 ml) was given and temperature gone down significantly ( 100 F) within one hour, in the late evening ( 10:30 pm) tem. again went up ( 103 F) with shivering, BUT THIS TIME we gave her T-98 and did sponging using ice, it took 2 hrs for temperatute to come down, out of meftalP and T-98 WHICH IS MORE suitable? Doctor: thank you for asking querry.As per your information your child must be having some viral fever or urinary tract infection.if fever is of long duration get complete blood count and urine examination done.for fever paracetamol is safest drug but you can give meftal temporary.best option is remove clothes of baby give paracetamol and do sponging.hope this is helpful to you."
},
{
"id": 88842,
"tgt": "What causes persistent pain in left lower abdomen?",
"src": "Patient: I have persistent pain on my left lower abdomen from 2004. It is sort of on and off pain which radiates to my back as well. I had a laparoscopy surgery done in 2004 for endometriosis. I have shown many doctors and did many investigations in last so many years but nothing is coming out. Ultrasound is done at least twice in a year, CT scan done last year but doctor is not able to detect the cause of the pain. There is no cyst anymore as per the reports. Could you please help me what should I do to detect the reason behind this irritating pain. Now I am scared if it is something bad like cancer ?? Doctor: Hi.Thanks for your query. Since all the reports are normal now and the pain persists, we have to look for odd reasons , uncommon causes for pain in abdomen.The possible causes are:Prolapsed intervertebral disc at the level of lower thoracic spinal cord. This can be diagnosed by an MRI of the thoracic spine.Post-herpetic neuralgia- history and trial of Gabapentin under your Doctor's guidance can help.IBS- all reports are normal- irritable bowel syndrome- treat the anxiety and stress, and the IBS.Few more investigations would help to understand te cause and treat accordingly:Color doppler of the abdomen to rule out vascular insufficiency to the intestines due to occlusive arterial disease of the intestines.Enteroclysis to see for the dynamics and mechanical obstruction.If all the investigations are normal- you may please consult a Psychiatrist."
},
{
"id": 131036,
"tgt": "What is the remedy for the nausea and vomiting caused due to popping in shoulder?",
"src": "Patient: My husband started feeling air in his left ear when he is breathing. It started early this morning when he turned over in bed onto his left shoulder. Something popped in his shoulder causing pain and the pain went up the left side of his neck to his ear. He immediately became nauseous and vomited green bile (horrible smelling). He is also experiencing more pain in his upper back. Doctor: First green vomit is normal , as you said bile second he does not need any treatment as this is due to sever painso he can take cataflam I.M if nausea persist then he can take 2 pills of primpranfinally poping in shoulder with sever pain requires an MRI scan or physical examination , i recommend he see an orthopedic doctor by the way air in his ear while breathing can be due to common cold ( nothing to worry about )"
},
{
"id": 204242,
"tgt": "What causes nightmares while on Paxil?",
"src": "Patient: Quesstion: Have been having violent nightmares almost every night. Gotta get to what cause is. Used to take Paxil for depression - but not clinically depressed at all. Don t like a lot of pills with side effects - any suggestions gratefully considered. M. O Brien, Apache Jct. AZ Doctor: Hello and Welcome to \u2018Ask A Doctor\u2019 service. I have reviewed your query and here is my advice. If on Paxil the nightmares are persisting then this means that the depressive symptoms are persisting and REM sleep duration is more. This result in non refreshing sleep and nightmares. You should consult your psychiatrist again for treatment. The dose of the drug should be increased which will improve the sleep and will help in reducing the nightmares. Hope I have answered your query. Let me know if I can assist you further."
},
{
"id": 185045,
"tgt": "Is persistent pain and swelling after hematoma on gums normal?",
"src": "Patient: my daughter got a hematoma on her gum above her upper front teeth as a result of bridgework. it has been several weeks and the pain has come and gone. However yesterday herllips and chheks swelled up again and she is in a great deal of pain. the pain medicine isn't helping her. is this normal and what can she do? Doctor: HiWith your quwry, dont know how did you confirmed it was a hematoma however such lesions must be clinically examined and treated for good prognosis otherwise it may act as nidus for infection.Hope your daughter is nit having any other systemic complications like bleeding disorders my sincere suggestion is visi your doctor and know the reason for sudden increase in swelling which is not a normal clinical sign.Hope this helps you"
},
{
"id": 118165,
"tgt": "What is the cause of spitting up blood?",
"src": "Patient: i have been spitting up blood every morning for the past month, even when i wake up from naps i spit blood.... but during the day my spit is clear..... i have a pain in my chest and feels like my right lung as well. can u tell me what it could be from Doctor: Cause of spitting up blood can be tuberculosis, bronchitis, pneumonia, bronchiectasis, and bronchogenic carcinoma. I will suggest u do a CT scan of chest with contrast.For temporary basis u can take tab Pause 500 mg thrice daily after food for 3 days.RegardsDR De"
},
{
"id": 212870,
"tgt": "Feeling down and unable to concentrate during morning times after changing tablets from Pexep to Mirtax prescribed for panic attacks. Reason?",
"src": "Patient: Hi I am Clint 40 yrs old I use to have panic attacks around 8 years ago until i went to a a doctor. he prescribed me pexep tabs which i started from 20 mg and went up to 40 mg. the medicines were of great and i felt good. but recently i went back to him and asked him to give me something cheaper as pexep 30 mg really got expensive and with age i found that less effect. now he has prescribed me mirtax 7.5 with maxgalan tbs before dinner ever since i started it i am feeling totally druged and and down unable to cosentrate on anything escpecially in the morning. Can you tell me the reason for feeling this ways or what should be done????? Regards Cllint Doctor: Hello...... Thanks for your query. The symptom of excessive sleepiness can be attributed to T.Mirtax which is a sedative anti-depressant/anxiolytic and inability to concentrate might be due to anxiety symptoms which have increased following shift in medication. I would suggest you to consult your psychiatrist again and report of the above symptoms since you require a cheaper alternative to T.Paroxetine which at the same is equally effective and free of side-effects. Wish you early recovery. Regards Dr Sundar Gnanavel Psychiatrist"
},
{
"id": 29325,
"tgt": "What causes an itchy rash along the waistline?",
"src": "Patient: I develop a rash only at night and not every night, that is on my front midsection from the waist up to under the breasts and on the lower part of the breasts. It is prevalent along the waist between my breasts but below the breasts. It is very itchy. Doctor: Hello and thank you for your question.It sounds like you have some sort of dermatitis.Sometimes we can pinpoint a cause for dermatitis, but 80% of the time we dont know why it occurs.What I would recommend for you it to see a dermatologist and possibly get a biopsy done to see what this condition might be.Some advices that I can give to you are:-avoid scrubbing the areas of the rash with soap- try to take quick showers with not very hot water.-you can also use a lotion which is available over-the-counter, called calmine lotionIt is important to see a dermatologist as soon as possible to get better.Hope my answer was helpful,if you have any other question I will be glad to answer,thanks!"
},
{
"id": 66675,
"tgt": "Suggest medications for fleshy lump in foreskin shaft & scrotum",
"src": "Patient: Hello, since the start of puberty, I have gotten fleshy lumps on my foreskin, shaft and scrotum. I am not confident enough to tell anyone and I don t like the idea of going to the pharmacy. I m not sexually active and would like to know how to get rid of it, cost effective and without telling anyone. Thank you for your time Doctor: hi, this just could be some normal variation or familial condition; or could be some sebaceous cysts or papilloma like benign condition; not to worry at all but get examined by some surgeon for physical examination.all the best!"
},
{
"id": 222698,
"tgt": "What causes spotting, bleeding and right abdominal cramping during a pregnancy?",
"src": "Patient: I am 4 weeks pregnant. Yesterday I started bleeding bright red. Not a lot just a bit of spotting. I had no pain or discomfort. Now I am passing dark brown blood. I have a small cramp in my lower right side. I am 35 years old, I have had 2 healthy pregnancies in the past and never miscarried before. Could I be miscarrying and what should I do? Doctor: Hello, and I hope I can help you today.Unfortunately your symptoms are consistent with miscarriage, and because you are having pain on one side- there is also a possibility of a tubal pregnancy (unless you already had an ultrasound that saw the pregnancy in your uterus)Either way, you need an urgent medical evaluation. The most efficient way to do this- and get all the tests you need right away; including a blood test for the hormone level of your pregnancy and a pelvic ultrasound- is to go to a local emergency room. I am sorry that the prognosis of this pregnancy looks bleak, but nothing can be done to prevent miscarriage once the body has started the process. At the age of 35, one in 7 pregnancies will miscarry. So I hope you get a proper evaluation of your condition soon, and if this pregnancy is not viable, still, the greatest chance is that everything will be fine if you try to conceive again afterward.I hope I was able to answer your question and that this information was helpful.Best wishes, Dr. Brown"
},
{
"id": 221574,
"tgt": "Suggest treatment for vaginal spotting during pregnancy",
"src": "Patient: ive recently done a pregenc test and turnet out positive and for the last week ive had what seems period pains and notced today a spot of blood i think i looked on the internet to find this normal but am cncerned as ive had a miscarriage before and realise there is a higher risk of a miscarriage this time round im so nervous as i dont want to go through the same thing i did before is this another sign of a miscarriage? please help me Doctor: Do one ultrasound. whether everything is ok with the baby and position or not...den start on tab progesterone 20mg tablets and folic acid. n avoid sternous work"
},
{
"id": 35810,
"tgt": "Suggest remedy for urosepsis",
"src": "Patient: I have dy purchased from Shop Rite. I received a phone call this morning telling me I had purchased contaminated water. I took a gallon container in the house that I put in the recycling bucket and the information on the container coincides with the info given by phone. I have been feeling unwell since last Saturday morning, 6/7/14. That morning I had terrible cramps and diarrhea enough to keep me from attending a wedding ceremony. I recently used that water to make lemonade that is in the refrig. that I will through out. In 2008 I was hospitalized with UTI and found to have E-Coli infection that was urosepsis.I was in criticaI condition in a Florida hospital. I am a heart patient, still not feeling well with malaise all week. I am scared. Please advise. Thank you doctor. Josephine Crivelli, 76 years old. Doctor: Hello dear,Thank you for your contact to health care magic.I read and understand your concern. I am Dr Arun Tank answering your concern.I suggest you to do urine culture and sensitivity. Take the treatment according to the report.Taking treatment in this way cures you not only completely but also rapidly.I also suggest you to take plenty of water, as much as you can because water can flush of your residual bacteria from the system.Please maintain good hygienic habit as it is equally important as treatment. Washing hand frequently is also a good tool against all this.I will be happy to answer your further concern on bit.ly/DrArun.Thank you,Dr Arun TankInfectious diseases specialist,HCM."
},
{
"id": 88454,
"tgt": "What is the treatment for severe abdominal pain?",
"src": "Patient: him im 24 male i have been sick for 3 days now, constant pain in my stomach, started with throwing up than moved to diaherria and its liquid that is really dark and looks kinda bloody, i havent been able to eat or even really drink without having to go to the bathroom. please help Doctor: Hi,It seems that you might be having acute gastro-enteritis, bacterial or amebic.Go for one course of ofloxacilin, tinidazole combination medicine course for 3 days.Take antispasmodic medicine like Dicyclomine for abdominal pain.Take light diet like curd, rice, butter milk.Take plenty of water and ORS liquids.Ok and take care."
},
{
"id": 61867,
"tgt": "Suggest remedy for lump on forehead",
"src": "Patient: So I have this bump on my forehead...I thought it was a pimple, tried popping it. It got badly infected to the point where my eye is swelling up and any touch hurts it. I have pictured if you wish to see but km just wondering if it should go away or if I should go in Doctor: Hi,From history it seems that there might be having ingrown hair follicle infection or folliculitis and by trying popping it precipitated infection giving rise induration, swelling and cellulitis.You might require one course of antibiotic and anti-inflammatory medicine course.Clean local wound properly and apply antibiotic cream.Ok and take care."
},
{
"id": 186031,
"tgt": "Suggest medication for infection and inflammation of the molar tooth",
"src": "Patient: last molar tooth in lower jaw dont have place to come out & it is inside gum. It is to be removed after some days. But now I am having infection at that place & pain also. I am taking combiflam after every 8 hours for last 3 days. Is it safe? Any other tablet to reduce infection & inflamation? Doctor: Hello! Welcome to HCM.Infection and pain due to a third molar impaction is common among the age group 18-21. combiflam is a pain killer and it is okay to take it t.i.d. You can also take a course of antibiotics to reduce the infection. Regards."
},
{
"id": 221766,
"tgt": "What causes watery discharge and bleeding in first trimister of pregnancy?",
"src": "Patient: hi, i am 7 weeks pregnant through ivf. we saw a heartbeat 5 days ago. today i had a sudden gush of watery discharge that was running down my leg. after that, i am now having slight bleeding. the blood is pinkish red. do you know what could have happened? Doctor: HI, I understand your concern. Make sure that you are taking hormone supplements strictly as advised. Your symptoms suggest threatend abortion.. you should be rushed to your treating doctor the earliest.Timely treatment can prevent any mishap. - you should take bed rest & shifted to hospital while lying position. Thanks."
},
{
"id": 128303,
"tgt": "Suggest effective treatment for bells palsy",
"src": "Patient: My wife is suffering from Bells palsy right side of face.dr.prescribed Tab Benfomet plus,Tab Evion LC and Ecosprin 75 mg..she developed vertigo after one single dose of all above tab.we stopped medicine after consulting Dr. What is further action.please inform Doctor: Dear patient bells palsy is mostly idiopathic means without any reason so treatment is physiotherapy and multivitamins. It recovers in most cases fully in 6 to 8 weeks. Till physiotherapy should be continued. Evionlc is vitamin e and should not cause vertigo.ecosprin is given for thinning of blood and does not cause vertigo. other reasons for vertigo needs to be ruled out and you need to consult your treating physician as soon as possible."
},
{
"id": 162696,
"tgt": "Should the ER be visited for confusion and behavioural changes after taking Augmentin?",
"src": "Patient: My son Is 3 1/2 and was prescribed Augmentin for reoccurring strep. Took one dose 5 hrs ago and has been crying out in his sleep he talks to us but never fully wakes up. Acts as if something is after him. agitation, confusion, unusual thoughts or behavior is a side effect of the antibiotic. Should I take him to the e.r? Doctor: Hello and Welcome to \u2018Ask A Doctor\u2019 service. I have reviewed your query and here is my advice. Yes please take him to the er as this could be due to dehydration or a rare side effect of the antibiotic. Hope I have answered your query. Let me know if I can assist you further."
},
{
"id": 201763,
"tgt": "How to improve penile size?",
"src": "Patient: Hi I'm 20 yrs old and I'm an indian guy my penis is abt 3cm when not hard and 13cm when hard.. I will like to know will it grow... And another aspect how will I be able to detect that its mature and reached its final stage.. Pls do reply with some suggestions thanks.. Doctor: Thanks for contacting HCM I am sorry to hear that are displeased with the size of your penis. Since you are now 20 years old you have reached full maturity. The size of your penis will not change at this time. There are no known treatments medication or supplements that help improve the size of your penis. Hope this answers your medical question. Please contact us again with your health care concerns and questions"
},
{
"id": 41732,
"tgt": "Suggest treatment to increase sperm count",
"src": "Patient: i am 35yrs i have been taking treatment to increase the sperm count for 2yrs.. i had done all the test req. took medicine for year.. tried all english medicine, tried with ayurvedic medicine still the same no luck Doctor: Hi welcome to healthcaremagic.I have gone through your question.If not done then i would advise to ultrasound color doppler scan to rule out varicocele, vascularity of testes.There are not any sure shot medicine that increase sperm counts directly. All these medicines are only supportive and nutritional.Hope i answered your question.Would be happy to help you further.Take care."
},
{
"id": 32240,
"tgt": "What causes cut and pimples near vulva while having yeast infection?",
"src": "Patient: I had a yeast infection I went to the doctor and got medication it itched alot and I scratch alot I also havelike a paper cut down there on the side of my vulva and I feel some really tiny like pimples thay dont hurt and they have no color what do you think it is Doctor: HI, I understand your concern. The cut & pimples on vulva during yeast infection- due to severe itching & body reaction to the infection along with hair root infections. Thanks."
},
{
"id": 182080,
"tgt": "Suggest remedy for bad breath",
"src": "Patient: I am at the end of my rope. I have tried everything in an effort to cure my bad breath and nothing, I mean NOTHING works. My girlfriend just left me because of this problem. I have tried everything on the market. Every remedy offered, from the inexpensive to the very costly. I also tried natural remedies such as grapefruit seed extract, including the drops, nasal spray, mouth spray and the tablets. I have tried the colostrum, capsules as well as the powdered form, I also tried the liquid chlorophyll which did nothing but turn my tongue and teeth green. It has been four months and there has been no change. People can smell my breath from several feet away. I Get regular dental cleanings and I brush five times a day and my teeth appear white and clean my I even brush my tongue and it appears pink but my breath still stinks. furthermore, what makes it even worst is the fact that I can not smell it. I feel like CRAWLIN INTO A BOX and hiding everyday; how does one live the rest of their life this way. I FEEL LIKE A leper that has been exiled from society because of my breath. Someone please help me!!!! I will do anything. I am successful, educated, I own my home but, at the end of the day I have absolutely no friends. After surviving eight hours of workday {which by the way is how I feel from day to day} I have to muster the strength to get through the day.The the whispers, laughs and sly comments are all most unbearable. On top of all of this I have to come home to an empty house. My breath controls my life. To make matters even worst I work in the educational field so I have to speak regularly. What do I do?? Doctor: Thanks for your query, I have gone through your query.It is very sad to listen to your depressing condition, nothing to worry, be bold.It can be solved, no need to get depressed for halitosis.The bad breath could be because of the deposits over the teeth causing gum infection. Or it can be because of the pus discharge secondary to gum or tooth infection or any respiratory tract infection like sinusitis or gastrointestinal disorders. Consult a oral physician and get yourself examined to rule out these conditions.if it is gum infection get the teeth cleaned once and maintain oral hygiene after that. You can use mouth wash.if it is tooth infection get the tooth restored.If it is respiratory tract infection consult a pulmonologost and gastro enterologist if it is a gastrointestinal disorder.I hope my answer will help you, take care."
},
{
"id": 224902,
"tgt": "On birth control. Missed taking pills. Feels sick, vomiting. Is it food poisoning or pregnancy?",
"src": "Patient: Food poison or pregnant? I am on birth control took last pill of the pack last Saturday, this Sunday I should have started my next pack but I forgot. had sex that day, on the next day (Monday morning) I took my Sunday night pill. and then my Monday night pill at the regular Monday night time. Tuesday (Today) I woke feeling really up sick, threw up and then felt completely normal after. Please tell me this is food poisoning or something Doctor: Hello and welcome to HCM,This has most likely happened as you have taken two pills in a day. They are known to cause nausea and vomiting. This is not food poisoning so do not worry. If you are feeling all right now, there is no need to worry. Continue your pills as directed and try not to miss any more pills as it will decrease the contraceptive efficacy. Hope this satisfies your query. Thanks for using HCM.\u00a0\u00a0\u00a0\u00a0\u00a0Feel free to ask any more questions that you may have. Dr Madhuri BagdeConsultant Obstetrician and Gynecologist"
},
{
"id": 31442,
"tgt": "Suggest treatment for acute bacterial epididymitis",
"src": "Patient: I have had an attack of acute bacterial epididymitis due to an E-Coli infection about 3 weeks ago. The testicular swelling has reduced 75% post antibiotics and NSAID's. There is also associated funiculitis which worsened a week ago - but has improved in the last 4 days with bed rest. Am continuing on Doxycyline 100 mg BD. My question is - when can I I resume sexual activity with a partner? Doctor: Hi thanks for asking question.As far as acute epidydymis treatment is concerned in your case treatment has to be started with single i.m injection of ceftriaxone 250 mg.Then doxycycline 200 mg or levofloxacine 500 mg has to be taken for 10 days.Reduce your physical activity.Use scrotal support and elevation.Ice pack can be helpful.According to some study sitz bath useful.Continue using antiinflammatory drugs for few days.Drink more water with fruits and green leafy vegetables.Avoid sex until swelling completely relieved and you fill better.I hope i have solve your query."
},
{
"id": 184667,
"tgt": "Can i smoke weed prior to wisdom tooth removal surgery?",
"src": "Patient: I'm having my impacted wisdom teeth removed tomorrow morning. Is it okay to smoke weed tonight? I won't be put under, just nitrogen. I'm 29, female, average health and weight. I'm not a regular smoker, just recreational. this will be my first surgery and I have no health problems. Doctor: Thanks for your query, I have gone through your query.The smoking before extraction will not affect you. but you should not smoke after extraction because it will dislodge the blood clot and leads to dry socket. so do not smoke after extraction for minimum 3 days. I hope my answer will help you, take care."
},
{
"id": 81402,
"tgt": "Cause for mild ache in the chest area?",
"src": "Patient: I have a dull achy spot on the left side of my chest, sortof where boob meets armpit, extending inward a bit over the top of the boob. I somehow can t really tell how deep it is, like if it s breast tissue or lung tissue or what. Sometimes my left arm gets tingly or feels, like, weak. I notice it more when I m in bed and it never lasts very long but today, right now, the boob-armpit spot has been bothering me for like two hours. Doctor: Thanks for your question on HCM.As a rule,left sided chest discomfort should be evaluated for cardiac cause first.So get done ECG to rule out this.If ECG is normal than get done chest x ray to rule out pulmonary cause if present.If chest x ray is also normal, than it seems muscular pain mostly.So try to follow these.1. Avoid heavyweight lifting and strenuous exercise.2. Avoid movements causing pain.3. Take good painkiller and muscle relaxant.4. Apply warm water pad on affected site.5. Avoid stress and anxiety."
},
{
"id": 47435,
"tgt": "What does kidney ultrasound suggest?",
"src": "Patient: 2 weeks ago i have difficulty in passing urine so i concern doctor who advised me for my kidneys ultrasound, my report is:BOTH KIDNEYS:normal in size,shape and texture corticomedullary distinction intact 2-3 small calculi seen in calyces measuring 0.3 cm right kidney show fullness of pelvicalyces left kidney show mild fullness of calyces no perinephric fluid collection seen URINARY BLADDER:mild thick walled no calculus or growth seen in lumen at scan time.please explain me my disease and also please tell me the good doctor in karachi for kidney problems. Doctor: Hi, dearI have gone through your question. I can understand your concern. Your ultrasound abdomen reports suggest that you have 2-3 very small kidney stones. No any other problems in kidney, ureter or bladder. You should drink plenty of water. If you don't have any symptoms then no treatment is required right now. Because stones are very small in size. If you have pain then diclofenac sodium is used for pain relief. Follow up ultrasound is required. Then you should plan accordingly. Hope I have answered your question, if you have doubt then I will be happy to answer. Thanks for using health care magic. Wish you a very good health."
},
{
"id": 59726,
"tgt": "Severe abdominal pain in the night. Is it due to acidity or gall bladder stones?",
"src": "Patient: Getting severe abdominal pain at the center of the abdomen during night at around 2 oclck, persists till 4 o clock,,,continuous,,,horrible pain, (especially by eating masala items, maida etc) may i know what the cause may be,,,is it acidity or gallstone problem,,hv consulted many doctors, but nothing has helped. kindly help me out doctor. Doctor: hi there, what you have described looks like a case of peptic ulcers and as you have rightly noticed spicy food and refined food item like maida make it worse. It would help you to get started on antacids after taking a prescription from your doctor. Also you could make minor changes in your food habit like taking small frequent nongreasy, non spicy food .Also it would help if you took a short stroll after your evening meal instead of going to bed immediately. Have your meal at least 1hr before retiring for the day. If you are overweight then some sort of exercise and weight reduction may also help to relieve you of these symptoms.Doesn't look like a gall stone problem to me but you could get an ultrasound abdomen done to rule out any such problem. Take care"
},
{
"id": 186093,
"tgt": "What causes small bumps on the upper side of mouth?",
"src": "Patient: I have a patch of small bumps on the upper side of my mouth. It s between the last tooth and cheek on my gums. It doesn t hurt. Just annoying since I can feel it with my tongue. I take care of my teeth and seem to always have cavities and sensitive teeth. But these bumps are annoying! What could they be?? Doctor: Thanku for using HCM.You did not mention if they are hard or soft and when did it appear. This might be an abscess with drainage due to infected tooth. consult your dentist.hope i answered your query"
},
{
"id": 34379,
"tgt": "Suggest treatment for parasite as flagyl and sepro didn't work",
"src": "Patient: I have been taking flagyl and sepro for parasites. Once I was done within 2 days the symptoms came back again. Can't eat anything withpit explosive dio I'm 56 yr old female. My husband was intensive are for 10 days so the Dr thought i picked up a parasite since it started within 2 days of being released. Doctor: Hello and thank you for your question.You really need to know what parasite you have in order to treat it properly. Ask your doctor to do a stool culture and parasite panel to figure out what you have then you can get appropriate treatment.Regards"
},
{
"id": 4012,
"tgt": "Suggest treatment to become pregnant",
"src": "Patient: my periods are always irregular after my marriage...3.5o t yrs passed after my marriage but i am unable to conceive...7 months ago i was expecting but it resulted with a miscarriage after 2and half months of pregnancy....after follicular study and taking letoval....now i again want to conceive....please help. Doctor: HIWell come to HCMIn my opinion just keep trying forget about the regular or irregular period because this is natural phenomenon and does not need any treatment, it is depends upon the compatibility, hope this information helps."
},
{
"id": 150907,
"tgt": "Diffuse bulge and protrusion, root compression. Descication. Treatment?",
"src": "Patient: 1.hello sir i am kavita, my problem is the dessication,diffuse bulge & posterocentral protrusion in L5-S1 intervartebral disc causing mild bilateral descending root compression. 2.dessication, diffuse bulge and posterior left paracentral protrusion of L4-L5 intervertebral disc causing left descending nerve root compression. Doctor: Hello,Thanks for posting this question in HCMI don't know if there are clinical signs of any nerve roor compression or not. I also don't know your age . If you have pain related to nerve root compression may improve with medicines like Gabapentine, pregabaline, Duloxitine etc.You basically need a detailed clinical evaluation by a neurologist who will determine if you have any deficits or not. If there are deficits and symptoms are still persisting, surgical treatment will be another option. If there are no complaints and MRI was done on routine basis, these findings are not of serious concern.Please get examined by a local neurologist and in the mean time you can discuss with him regarding the medicines that i have suggestedHope this helps"
},
{
"id": 54348,
"tgt": "Would fatty liver due to high Alt level come down after stopping the shot of vodka?",
"src": "Patient: my alt is 62 and I have been drinking one shot of vodka every day. I have a fatty liver according to the sonogram. I take no medication. The Alt level was in the teens before I started taking voda. I stopped all alcohol. Should I expect the alt to go down? Doctor: Hi thanks for contacting HCM...The alcohol might be contributory factor in fatty liver ....Your weight should be estimated ....If obese then also fatty liver can occur...In some cases there might be idiopathic cause....So yes first step is better to stop alcohol...And if going to take then not over safe limit...Regular exercise done...Use less oil in cooking....Sunflower , olive like good oil can be taken .Fried food less.Trans fat food like non veg , baked pestry , meat , butter , cheese etc taken very less.....Refined food avoided.Udiliv tablet can be taken....With keeping this in mind consult gastroenterologist..Take care.Dr.Parth"
},
{
"id": 197353,
"tgt": "Suggest treatment to increase the duration of erection",
"src": "Patient: Im 39 years ol and i have so my erction is not long time in the same i have made love just one time for 2 monthes and i have made masterbation for 4 years so i feel afraid as i am going to get a girl freind and we will marrry after 5 mo nthes so what is the longest erction it should contiue Doctor: Hi ...According to history you are talking about premature ejaculation...But as it was first time mostly performance anxiety will be the cause. ....If after marriage you have same continue problem like ejaculation before reaching orgasm then you have to consult psychiatrist...Following are some suggestion for such case....Double condom can be used....Stop start techniques and squeezing technique useful....Do masturbatiin before sex found useful...Do sex in comfortable position...Kegel exercise for making pelvic floor muscle strong ...Local topical anesthetic application in form of gel found useful....Regular exercise and balanced diet...Avoid stress is the main measure ....Take care"
},
{
"id": 189756,
"tgt": "Teethes pulled out, top left hand side of mouth, flap of skin coming out of gap. Solution?",
"src": "Patient: I got three teeth pulled out in about Novemberish, its on the top left hand side of my mouth and there was still a gap, slit thing after the stiches and everything. Now recently there was a gooey substance coming out my tooth , and now i have this flap of skin coming out of the gap, and it does bug me when i talk. Do you know why or how it got there and how i can get rid of it? Doctor: Hi, Thanks for asking the query, From the description this can be healing socket or interdental papilla that came out after extraction .. I would advice you to use warm saline gargles Maintain a good oral hygiene . use 0.2%chlorhexidine mouthwash solution twice daily . As the extra mass remained is creating a problem you can visit the dentist and get it removed. Regards....."
},
{
"id": 220538,
"tgt": "Should I be worried for pregnancy while being on BC pills?",
"src": "Patient: I am 31 year old mother of two. I have been on a combination birth control pill for exactly one month. I took my pill every night. I am on my second day of \"inactive\" pills and I have not started my period. I had sex with a condom once. Another time my partner just barely penetrated and I made him stop in fear of pregnancy. Should I be worried? Doctor: Hallow Dear, If you are so regular with your birth control pills, the chances of pregnancy are very rare. Moreover you had only one intercourse, that too with condom on. If you do not start your menses while on inactive pills (2nd pill), you may continue with the next pack as per the instructions - this is what WHO recommends. You may get the menses by the time all inactive pills are consumed. Some times it is noted that women may not get menses after inactive pill. To sustain the effect of birth control pills, it is advised that you start with the next pack if you are very regular with the previous pack. Even after the second pack inactive pills also if you do not get periods, then you may have to report to the Gynaecologist for further investigations of pregnancy or nay other cause. I hope this helps you. Dr. Nishikant"
},
{
"id": 113807,
"tgt": "Lower back pain, kidney infection test negative, taken antibiotic and naproxen, smoke liquid nicotine",
"src": "Patient: I feel sick to my stomach, and lower back pain on both sides of my back and around my rib cage in the back, I was tested for kidney infection a week ago and it was neg.but I had all the symptoms so was treated for 3 days with antibiotic and naproxen , I smoke liquid nicotine in an electric cigerette and am wondering if this may be causing problems, other than e- cig I have no other habits, my yearly blood tst also came back good( CBC) Doctor: Hello. Thanks for writing to us. The habit of smoking is not likely to cause the back pain you are having. It is more likely to be due to a myofascial syndrome which subsides with a combination of pain killer and a muscle relaxant. Doing a hot fomentation will also help you temporarily. I hope this information has been both informative and helpful for you. Regards, Dr. Praveen Tayal drtayal72@gmail.com"
},
{
"id": 170608,
"tgt": "What causes pimple like sores on abdomen?",
"src": "Patient: A child in my care has pimple like sores all over the left side of his abdomen. His parents squeeze them to remove the puss and treat them with antibiotic ointment. The poor kid is itchy and sore. What could be the underlying cause of this rash? How can it be prevented? Doctor: Plz provide the clinical details... Does child has fever cough cold??? Blood investigation will be needed... A clinical examination and councelling of parents regarding hygiene is utmost important..."
},
{
"id": 126338,
"tgt": "Can seizures cause one-sided body weakness?",
"src": "Patient: Good evening..My mother is experiencing muscle weakness, disoriented, we re having to repeat commands to her several times,left side weakness..the hospital has done an mri,egg, and CT but the only thing that showed up was possible seizure activity..Is it possible she may be having TIA S? Are there any other test that may be done to possibly find out what s causing these symptoms? Doctor: Hi, After going through the reports, it is clear that TIA cannot be ruled out, though my first thought would be in the form of seizure and medicine thereof. Seizure medicines can cause the feeling of muscle weakness, disorientation etc. There is at present no test which can confirm TIA. Hope I have answered your query. Let me know if I can assist you further. Regards, Dr. Gopal Goel, Orthopedic Surgeon"
},
{
"id": 68592,
"tgt": "What causes feeling of lump in tailbone after sitting down longer?",
"src": "Patient: hi. a few days ago i was sitting down for a few hours at course and the next day i felt pain in the tail bone, just at the top of my bum. when i sit i can feel a lump but when i stand i cant feel anything. sitting n laying down really hurts. any ideas?? Doctor: Hi ! Good evening. I am Dr Shareef answering your query. This could be because of inflammation due to sitting on your butt for a long duration. If I were your doctor, after a clinical assessment of yours, I would advise you for a broad spectrum antibiotic, an anti inflammatory drug, and a proton pump inhibitor for symptomatic relief, and possibly a cure. If it persists or increases, I would refer you to a general surgeon.I hope this information would help you in discussing with your family physician/treating doctor in further management of your problem. Please do not hesitate to ask in case of any further doubts.Thanks for choosing health care magic to clear doubts on your health problems. I wish you an early recovery. Dr Shareef."
},
{
"id": 157989,
"tgt": "Have pleomorphic adenoma below ear. Can homeopathy shrink the tumor?",
"src": "Patient: Hi. Dr. Bhatti, I had pleomorphic adenoma below the right ear, I am going for consultation for it, I think he wants to have it remove, but I am afraid because of the so many facial nerve , do you think any homeopathy treament or other can shrink the tumor , the tumor sometimes feel like a marble, sometimes it is small, it is not painful at all, but by evening, i can feel that my saliva is incresing. Any suggestion? Doctor: Pleomorphic adenoma is a tumor of the salivary glands and going by your description, it is in your right parotid gland which is a major salivary gland in the human body. Problems with this tumor if left untreated for long are dual, one it will keep increasing in size and cause pain and disfigurement. Two, it can get converted in to a malignancy (cancer that spreads). For both these reasons, it is advisable for you to get it removed at the earliest. The surgery that is done is called superficial parotidectomy. The only problem with this surgery is the 3-5% risk of injury to the facial nerve. However , this injury is much less likely to occur in the hands of an experienced surgeon who hass been doing it for long. In my practice for example, i have never encountered a permanent postoperative facial nerve injury. So i would suggest you get it done from a cancer surgeon rather than an ENT or general surgeon. Also, there is no other treatment for this tumor in any science. Please dont be misled by people who claim that they can treat you without surgery. They cannot. They will only waste your time and money."
},
{
"id": 126138,
"tgt": "What does soreness in the rib-cage indicate?",
"src": "Patient: I am experiencing soreness in my right rib area. Seems to be at the end of the ribcage and/or behind the right ribs. It s sore to press on the area at the bottom of the right rib. What are the possible reasons that ia having these symptoms. Have been noticing it for about three weeks. My check-up w/ my PCP dr was good ion June 28. CBC & lipid panel all good. Vit D was low- 21. Doctor: Hi, This is because muscles weakness. You need to walk regularly and do regular exercises. If pain is more then you can take local infiltration at trigger point. Hope I have answered your query. Let me know if I can assist you further. Regards, Dr. Anuj Gupta, Spine Surgeon"
},
{
"id": 105677,
"tgt": "Allergic to dust, strong odor, running nose, sneezing, cough with phlegm. Cure?",
"src": "Patient: hi, i am lakshmi from chennai, tambaram. i am suffering with dust allergy since 8 to 10 yrs. if i expose to the dust and if i inhale any strong smells like perfume.. and dhoob sticks.. immedaite reaction occurs like running nose and sneezing continuiosly for half-an-hour. Now a days even i am getting cough with flom. Really i am suffering alot. please suggest me. Doctor: You are suffering from allergy which can present in many forms and in your case it is in the form of Allergic Rhinitis. First of all you should avoid dusty, smoky atmosphere and strong smells as much as you can. Take antihistaminics like cetrizine or allegra or loratadine. Also take montair 10 mgm once a day, fluticasone nasal spray 1 squirt per nostril twice a day. Avoid junk food and foods and drinks with strong flavours and artificial colouring agents and preservatives. Go out in the fresh early morning air and do some deep breathing exercises."
},
{
"id": 56611,
"tgt": "Suggest remedy for high SGOT and SGPT levels",
"src": "Patient: Recently I have done few tests ............... bilirubin is within normal range .8 but SGOT and SGPT are 72 and 84 My eyes are normal and so is my bowel and urine.Is there a cause to worry?What are the suggested medication and measures that needs to be taken to bring the levels SGPT/SGOT to normal?2 years back the SGOT/SGPT were 98/106 but was back to normalancy within 2 months?Pls advice as my doc is out of town..............All other test pertaining to Blood Sugar/Hb/Counts/Urea/Creatinine/Occult Blood are within normal limits...... Doctor: HelloIncreased SGPT indicate liver injury.It may be due to many reasons like hepatitis,alcohol intake,altered lipid profile,medicines,auto immune causes etc.SGOT is non specific and it increases in many conditions.You may need few more investigations like lipid profile,viral markers and ultrasound of abdomen.I suggest tablet ursodeoxycholic acid 300 mg twice daily for three months to my patients.It helps in regeneration of liver cells.Get well soon.Take CareDr.Indu Bhushan"
},
{
"id": 95377,
"tgt": "What treatment should be taken for stomach pain ?",
"src": "Patient: hello doctor ..my friend suffer.. he havin pain in stomache from last two three years..n he consult to doctor he tell dem \"pathri hai\"..please tell me what shud he do... hello doctor..my friend suffer.. he havin pain in stomache from last two three years..n he consult to doctor he tell dem \\ Doctor: Hi Miss preet, Welcome to Healthcare magic forum, Pain abdomen since 2-3 years is very long to neglect, get ultrasound scan done to know the exact cause of the pain, it may be kidney stone or gall bladder stone, after scanning consult ur doctor for further treatment. Hope I have answered ur question Regards"
},
{
"id": 46947,
"tgt": "Suggest medication for hypertension in a person suffering from kidney stone",
"src": "Patient: My blood pressure is 176/97 and my pulse rate is 56 . I have kidney stones I ve been trying to pass for the past week . My family doctor is not available right now . Yesterday he prescribed a blood pressure pill to work with my blood pressure medicine I am currently taking and I just took one dose yesterday . When should I see any results and at what point should I be concerned with the high blood pressure Doctor: HelloYou should see a lowering of your blood pressure within three days of taking the new medication and I'd like to see your pressure in the 120's/80's range"
},
{
"id": 171052,
"tgt": "Is asthalin nebulisation or saline recommended for nose block of a 6 month baby?",
"src": "Patient: hello Doctor, my girl was preterm by 2 months, now she is 6 months old and has weight of 3.6 kgs. She is having nose block because of which she is not sleeping at all in the nights (earlier she used to wake up only when she is hungry) and dry cough sometimes... shall i give asthalin nebulisation or saline would do? Doctor: Hi, welcome to HCM. Your baby has nose block due to which he is not able to sleep at night. Sometimes, nose block can interfere with sleep in children. You can give saline nasal drops like nasoclear for this. There is no need to give nebulisation with asthalin unless your child has wheezing in chest. I suggest you to give only nasal drops and if you feel that she is still having problem then you should get her chest examined by a doctor. I hope this has helped you. Take care. Regards - Dr Deepak Patel, MD Pediatrics"
},
{
"id": 122661,
"tgt": "How to treat tight,shiny skin on legs?",
"src": "Patient: My father is 71 years old, post colon cancer pt, (6 yrs clear), had knee surgery a few years ago; skin on both legs is shiny , mostly hairless (below knee) and appears tight . He does not complain of pain, itching or any insensitivity but looks not right . He does take cumidin, lasix and I believe another med, but not recalling what it is; causes? Doctor: Hello, The tight and shiny on the legs can happen due to swelling or edema. He needs a few investigations to find the cause of swelling and then get it treated accordingly. Hope I have answered your query. Let me know if I can assist you further. Take care Regards, Dr Praveen Tayal, Orthopaedic Surgeon"
},
{
"id": 215399,
"tgt": "What causes pain behind my leg?",
"src": "Patient: I feel like I have stressed or tore a ligament in the back of my leg. Behind my knee and down my calf is very painful. I can t think of a way I could have done this. My real question is, which is better,heat or ice on leg to help with the soreness. Doctor: Hello, Cannot say in your particular case, but generally, ice immediately to help the pain but after a day, while ice will still deaden pain, it won't do anything to the injury while heat will lower inflammation after a day. So will naproxen. Hope I have answered your query. Let me know if I can assist you further. Take care Regards, Dr Matt Wachsman, Addiction Medicine Specialist"
},
{
"id": 71492,
"tgt": "Chest pain in rib cage radiating to collar bone along with regular headaches. Stress test and heart fine",
"src": "Patient: Chest pain center of upper rib cage, radiates to both sides of my chest to upper sides of collar bone, been like this for three months, also have had headaches everyday for almost two months temples twitching pain in front and rear of head. Have had stress test and heart is fine...stressful job. Please help. Doctor: Hello,The possible issue seems to be inflammation involving rib cage with radiating referred neuronal pain to the sides of chest and collar bone. I would prefer to take an MRI of the chest for further line of action. Primary relief with analgesic, anti-stress medicines can be obtained.Hope I have answered your query. Let me know if I can assist you further.Regards, Dr. Bhagyesh V. Patel"
},
{
"id": 126521,
"tgt": "Does vitamin E help in treating severe leg pain?",
"src": "Patient: I have lots of leg pains from surgeries i have had on my leg, I need relief so I can get some sleep and walk around house without being in pain, I take a muscle relaxer which helps some, but at night is when i need the relief, will vitamin E help with muscle soreness Doctor: Hi, Vitamin E does not help to relieve pain. As the first line of management, you can try analgesics like Acetaminophen or Tramadol for pain relief. If symptoms are severe consult a physician and get evaluated. Hope I have answered your query. Let me know if I can assist you further. Regards, Dr. Shinas Hussain, General & Family Physician"
},
{
"id": 216523,
"tgt": "Suggest remedy for sharp pains in temples",
"src": "Patient: hello! i get this sharp pains in my temples. they would, i don t know, 2-3 times in one day. it s not an everyday thing though. it would occur at random times and would last 2-3 seconds. is this something serious? oh and if it helps, i wear glasses. 150/125 vision Doctor: Thank you very much for contacting healthcare magic. You have problem of sharp pain at temple region that may be because of migraine problem. Some tension cause tension headach.Any sinus infection or sinus inflammation may cause temple region pain problem. Cluster headach may ruled out. In cluster headach one side of pain at temple occurs.Check your refractive error.Consent a physician for diagnose. Proper history and physical examination give exact idea. I hope my guidance is helpful to you. Take care.Thank you."
},
{
"id": 127874,
"tgt": "What causes twitching in the shoulder when diagnosed with severe back pain?",
"src": "Patient: Hello, my husband has been experiencing back pain for the past week, right below the neck, in the center. It has progressively gotten worse, and now his left shoulder has an uncontrollable twitch. He now has an elevated heart rate and feels that something is wrong. What do you think could be the problem? Doctor: I will suggest you ayurvedic medicinetab yogaraj gugglu 2 tab bdcap shallaki 1tds. it will cure your problem"
},
{
"id": 80724,
"tgt": "What is the treatment for chest pain?",
"src": "Patient: I am experiencing chest pains - left side specific, for the last 18 hours. No nausea, no sweats, no sense of doom. They began shortly after Unboarding after a 4 hour plane flight. The pain goes away when I lie on my back or right side. It is worse when vertical or lying on my right. Doctor: Hello dear, thanks for your question on HCM. I can understand your situation and problem. In my opinion we should first rule out cardiac cause for your left sided chest pain. So get done ecg first. If ecg is normal then no need to worry much for cardiac cause. Since you have this pain afyrr plane journey, pulmonary embolism can be the cause. So get done1. D-Dimer2. CT pulmonary angiographyIf both are normal then no need to worry for pulmonary embolism. This might be simple muscular pain. So consult doctor and discuss all these."
},
{
"id": 82247,
"tgt": "Suggest treatment for congestion and breathing trouble",
"src": "Patient: Hi, I am 18 years old. I ve been experiencing congestion and difficulty breathing. My throat tightens up everytime I eat or drink and then it loosens up after a while. The pain is gone, all that is left is the tightness and slight itchiness. Its pretty scary, what sickness do I have? Doctor: Thanks for your question on HCM.It looks like upper respiratory tract infection (URTI). But we need to rule out lung infection. So get done chest x ray first.And if it is normal than mostly you are having URTI.Try to follow these.1. Avoid hot, oily and spicy food.2. Drink plenty of fluids.3. Warm water gargles 5-6 times a day.4. Start macrolide antibiotic. 5. Start antihistamines and anti inflammatory drugs."
},
{
"id": 57644,
"tgt": "What is the cause for passing more blood than wee?",
"src": "Patient: Hi I currently have Gaul stones and am,having to have an op to remove my bladder but this past week I've been,feeling as if I constantly need a wee but when I go its only a tiny amount it also is now hurting when I wee and I am passing more blood than wee ... Could this be to do with the Gaul stones or is it something else .. Please help what with the pain from the Gaul stones and now this is unbearable Doctor: Hi, thanks for using healthcare magicThe burning on urination , frequency and blood in the urine are likely due to a urinary tract infection.You should consider visiting your doctor who would likely examine your abdomen and also perform an in office urine test. Urine may also be sent to the lab for further examination.A prescription for antibiotics would be given if the infection is confirmed. Increasing fluid intake is also necessary.I hope this helps"
},
{
"id": 21310,
"tgt": "What causes air bubble under the rib along with increased heart rate?",
"src": "Patient: Feels like an air bubble under my rib located a few inches below nipple. When I start to feel this happening my heart rate increases until I manage to burp it out. Been to hospital and heart is fine, I believe it is also. Stomach feels alittle upset. Sometimes I get a fever right before heart rate increases. When i burp it's always coming from the right side of my throat, same side pressure comers from. After burping air builds right back up in same spot. Doctor: Hello, It appears to a acidity and gastritis. Do you also have upper abdominal pain, nausea, bloating, burping, increase in pain on food, sour water feeling in throat or chest burning, these are the other symptoms and may be associated. You should avoid fatty, oily and spicy diet. Have some walk after having food instead of taking rest. Have multiple small meals instead of heavy meals. Have regular sleep habits and avoid stress. Lots of green leafy vegetables, fruits. Avoid smoking and alcohol if any. You should get prescribed tab Pan DSR 40 mg or esomiprazole domperidone combination beforebreakfast once a day for 2 weeks."
},
{
"id": 165631,
"tgt": "What causes underweight and slow development in child?",
"src": "Patient: This is about my daughter who is 1-1/2 years old. For the past 3 months and even now she is suffering from constipation and was prescribed laxopeg outcome was too much diarrhea after tat she had tonsillitis and was prescribed taxim-o and enterogermina and for some time I am seeing changes in my daughter behavior she cries too loudly banging her head to me or arching her head backward and forward keeps on asking for biscuits, she neither burps after eating her food nor does gas and she is very slow in all her developments she is still not walking she is doing crawling and sitting and when she was born she was underweight. I want to know are these behaviors normal to a child or needs some special care. Her temperment is very extreme, which is very difficult to control. I am worried and would be grateful if you clarify my query Doctor: by 1 and half year a child should be able to walk alone or crawls upstairs or should be able to ask objects by pointing and should follow simple commands so with your history it can be global developmental delay(GDD) but before i label it GDD baby should have delay in 2 or more skills that includes language motor or social development. if your baby is truly GDD a proper diet is required with regular physical therapy, speech therapy and pediatric psychologist opinion with these i would like you to do routine investigation just to rule out any underlying disease"
},
{
"id": 183299,
"tgt": "Suggest remedy for injury in tooth",
"src": "Patient: My 3 yr old son fell at the playground. He pushed one of his front teeth up into the gum. I would approximate about 1/3 to 1/2 of the tooth is up in the gum. He bit all the way thru his bottom lip in multiple spots. He has been to ER and said there is no need to see a dentist. Is this true? Doctor: Thanks for using Health Care Magic.Read your query.Have the tooth being pulled down to its position or it is staying pushed up in the gums?I would advice you to visit his pedodontist and have a radiograph done to evaluated if the underlying tooth has been affected or not.Any infections developing has to be ruled out.Hope this was useful.Thanks and regards."
},
{
"id": 68475,
"tgt": "How can a pelvic region lump with redness be treated?",
"src": "Patient: Hi doctor, i have a very small lum on my pelvic region and it showed up recently during the beginning of my menstrual cycle. It's slightly red but not reall detectable until u touch it. It's been here for two days and is slightly smaller. Any advice? ? Doctor: Welcome to Health care magic.1.It could be localised inflammatory change in the subcutaneous plane.2.Mean while do not scratch or press the lesion - as you will complicate.3.Sinse its asymptomatic - would recommend to wait and monitor.4.Maintain local hygiene - thinking in terms of infective process.5.If no change advised an ultrasound to evaluate if there is any collection or swelling and where it is arising from and its contents. Good luck.Hope it helps you. Wish you a good health.Anything to ask ? do not hesitate. Thank you."
},
{
"id": 188700,
"tgt": "How many Valium tabs should I take before root canal?",
"src": "Patient: Last time I had a crown at my dentist and took 2 5mg tabs of valium . Did not even feel relaxed at all. Have a root canal tomorrow. How may can I take and hoe long does it take to work I am 250 lb female. I usually do nitrous but they have none. I want to feel like I dont care but not totally messed up. If I try 15mg 1 hr before and I get there and I need another one how long will it take to work? Doctor: Hi, Thanks for posting the query, Valium is used to treat anxiety disorders, musle spasms , siezures. Take valium exactly as prescribed by your doctor do not take larger or smaller amounts for longer then recommended. Call the doctor at once if you feel that the medicine is not working or if you think you need to use more dose than usual. Generally for relief of symptoms of anxiety the recommended dose is 2-10mg , 2-4 times daily. I would suggest you to concern a doctor before taking the drug. Hope this helps out. Regards....."
},
{
"id": 108008,
"tgt": "Suggest remedy for back pain",
"src": "Patient: Hi Sir, I have started using Femilon recently according to my doctor's advice. I am just two days into using the pill. But, I did not take them in the order that was listed on the back of the pill i.e Sun,Mon and so on. Is there a problem with that? I am also having back pain for the past 2 days. Please advise? Doctor: DEAR FRIEND, I NEED TO EXAMINE YOU THOROUGHLY, NEED X RAY OR MRI TO CONFORM DIAGNOSIS, SO DO THAT AND SEND ME REPORT AND VISIT PHYSIOTHERAPY CLINIC AND ASK DOCTOR TO GIVE YOU LUMBAR TRACTION AND IFT THERAPY FOR INSTANT PAIN RELIEF... LET HIM TALK TO ME... 9033762190 DR HEMANG KUMAR JANI"
},
{
"id": 46904,
"tgt": "Is it safe to increase protein intake while having pancreas deficiency and on kidney dialysis?",
"src": "Patient: I have pancreas deficiency. I am on kidney dialysis & my dr wants me to increase my protein to 100 g a day. Is this safe? Since I started trying to do this I have been vomiting. So I know I am not getting the protein out of what I eat. Oh by the way I am on Creon to help with the digestion. Doctor: As an Urologist and kidney transplant surgeon,i understand your anxiety.In people on dialysis,protein intake is restricted to 40gm/day.If your body weight is more,depending on B.P.and electrolytes,it may go up.It's best decided,in consultation with your nephrologist.The vomitting may be, due to increase in creatinine,electrolyte imbalance or even insulin deficiency.The insulin dosage will need to be adjusted.Creon is pancreatic enzymes supplement,given with caution in renal failure.Your nephrologist may ask to stop it.You may send doubts to me directly.Dr.Matthew J. Mangat."
},
{
"id": 60467,
"tgt": "Loose motion , slight abdiminal pain, gas and burning in the stomach due to Hepatic Hemangioma",
"src": "Patient: Hi Doctor , I am a male, 48 years old, I am diagnosed by CT Scan with 2 Hepatic Hemangioma , measuring about 2x2.1x2. Since last one and half month I am suffering from loose motion , slight abdiminal pain, gas and burning in the stomach and during motion. My question is about the treatment option and if there is any risk in the long run. Is this stomach disorder is due to this hemangioma? please advise. My height is 5 fit 5 inch. Doctor: Hello and welcome to healthcare forum. Your hepatic hemangioma never produces such type of symptoms so dont think that it has any relation with your present complains and your present problem is mostly due to some gastro intestinal infection and will be cured by antibiotics under medical supervision. You does not require any further treatment for this hemangiomas at present or in future as it will not cause any problem. Best wishes for your health."
},
{
"id": 38716,
"tgt": "How to cure ring worms on arm and neck?",
"src": "Patient: Hi, ive had two ring worms on my arm since may 2011. Both of my arms had one but they went away. A few months back i started getting it on my neck, and then on my right arm it came back. Then i started getting it on my face. The docters gave me ketoconazole 2% cream. It did not work so i went back for a refill. It still didnt work so they gave me pills. The pills did not work either. About a month ago my aunt brought me the same cream but from the Dominican Republic. It was working really well, untill i came back to my school on campus and took a shower it started getting red. My friend told me about useing clorox bleach. Ive been useing it for a week now and its just very red on my neck and arm. What should i doooooo! Doctor: Thank you for writing us,Hope you'll get satisfied answer here.Brief Answer - Ringworm, also called tinea, is a skin infection caused by fungi, microscopic organisms that are similar to yeast and molds. It does not have any relation to worms, but is called \"ringworm\" because the infection can produce ring-shaped patches on the skin that have red, wormlike edgesDetailed Answer - If you have jock itch, athlete's foot or ringworm of the body, you usually can be treated with a nonprescription antifungal cream, lotion or powder. Your doctor can recommend the appropriate treatment for your condition. Some of the nonprescription medications available include terbinafine (Lamisil), tolnaftate (Tinactin), miconazole (Micatin and other brand names), clotrimazole (Lotrimin, Mycelex) and undecylenic acid (Desenex and other brand names). Be sure to follow the directions carefully when you apply the medication. If your symptoms do not improve significantly after you have used a nonprescription medicine for about two weeks, call your doctor. Your doctor may suggest a prescription antifungal medication that will cure the problem. For extremely stubborn infections, the antifungal medication may be taken by mouth rather than being applied to the skin.If you have a fungal infection of the scalp, beard, fingernails, or toenails, your doctor may treat you with an oral antifungal medication, such as oral terbinafine (Lamisil), itraconazole (Sporanox) or griseofulvin (Grisactin and other brand names). You also will have to apply antifungal creams or shampoos to your hair and scalp. Complete treatment can take two or three months.When To Call A Professional"
},
{
"id": 19448,
"tgt": "What causes feeling of heartbeat in throat/chest while having first degree heart block?",
"src": "Patient: I was diagnosed by EKG with first degree heart block, but when I look at the symptoms, I think that I have many symptoms of second degree mobitz II heart block. I have been having unexplained tiredness and episodes of unexplained light-headedness. Also I get episodes of feeling my heart beating funn in my throat and chest. It beats consistantly and then skips and then continues and skips again. Doctor: sir ...please see this ..in heart block when the compensatiory beat comes after blocked beat it is forceful and felt as heart beat or pulsation in throat ..donot assume by symptoms ..diagnosis whether it is first degree or second degree is confirmed by ECG"
},
{
"id": 126372,
"tgt": "What causes pain in the lower leg post workout?",
"src": "Patient: For the past couple of days I have had moderate pain on the inside left leg below my knee including my calf muscle. 2 weeks ago I did move and did some heavy lifting and about 5 days ago I awoke with a Charlie Horse . The pain from the Charlie Horse went away within a few minutes after massaging my leg. I didn t notice any lower leg pain after my moving until now and my other leg feels fine. Is this something to worry about? I am not taking any medication for the pain and only notice the pain when I touch that part of my leg. Thank you. My email address is YYYY@YYYY Doctor: Hi, That's the natural phenomenon, if you do sudden heavy exercise which body is not used to then definitely body will pain. It will definitely improve with time. the area which is tender to touch is due to muscle soreness. it will improve. Next time start with a gradual increase of exercises and not on the first time. Hope I have answered your query. Let me know if I can assist you further. Take care Regards, Dr Anuj Gupta, Spine Surgeon"
},
{
"id": 2976,
"tgt": "Can pregnancy happen after taking HMG injection?",
"src": "Patient: i am 38 y/o lady with w=67kg va 157cm in height. i had HMG injection in this cycle .i had 17.8 and 16 mm follicle in lt ovary and 13.3 in lt ovary in 11th day .my hysterosalpingogram and my husbund semen analysis was normal.pregnancy is possible in this periode????best day of intercourse?best day of HCG injection? Doctor: Hello and welcome to \u2018Ask A Doctor\u2019 service. I have reviewed your query and here is my advice. There are chances for your conception. You need to check whether the ovulation is occuring or not. This can be known by a kit available. If ovulation comes positive have intercourse from the day ovulation shows positive and take the injection once the follicle is ruptured and egg released which can be known by an USG. Hope I have answered your query. Let me know if I can assist you further.Regards, Dr. Sushma Dubbaka"
},
{
"id": 91840,
"tgt": "Why stomach is enlarged with abdominal pain and tenderness?",
"src": "Patient: Hello Doctor. My mother s stomach has enlarged over night to the point where she looks like she s pregnant. She has lower abdominal pain on the right side which is very tender right below her rib cage. No constipation or any of those common issues. Do you have an idea as to what it may be? Thank you(: Doctor: It is most commonly enlarged Gall bladder. Do an ULTRASONOGRAPHY and a CT SCAN of whole abdomen.Then report me."
},
{
"id": 208405,
"tgt": "What causes depression with cold flashes?",
"src": "Patient: hy, i am 26 years old,y height is 5.3,weight is may be 51kg,and i am a patient of depression from last 8 years...sir from last 3 days i am feeling that my boody suddenly get very cold,i am very sleepy today i went to doctor he checked my fever which is normal and also check my blood pressure which is also normal,if every thing is normal then what is happening with me,sir i am also a patient of depression and using tofranil150mg daily..i am really worried ...can you please guide me properly Doctor: hi dear,as you are suffering from depression since last 8 years and also you are on treatment so you are known that in depression there are chances of anxiety features in between you feel like body suddenly get cold and sleepy and anxiety due to that.so not to worry it may be due to any upcoming anxiety or it may be due to some underlying thoughts.consult your treating psychiatrist and ask about such complain.and if needed change medication.not to worry much.Thank you"
},
{
"id": 24838,
"tgt": "What causes tiredness, nausea, headaches and sweats?",
"src": "Patient: My husband was at his gp complaining of tiredness , nausea, headaches , sweats and lack of energy. He is otherwise very fit and healthy 49 yrs. no ph , urinalysis nad awaiting results for thyroid, LFTS CRP fasting blood glucose NAD . We are going on holiday next week abroad which deeply worries me as he is so miserable with the above complaints . Suggestions would be most appreciated Thankyou Sharon Jones Doctor: Hello!Thank you for asking on HCM!I carefully read your question and would explain that his symptoms could be related to a metabolic disorder (adrenal gland dysfunction, thyroid dysfunction) or inflammation (including a chronic infection). I would recommend performing further tests to investigate for the possible cause: - complete blood count and sedimentation rate- blood electrolytes- cortisol plasma levels- liver and kidney function tests I recommend you to closely monitor his body temperature during the day and refer the values to his doctor. Further tests may be needed if suspicions of a chronic infection are raised. Hope you will find this answer helpful!Best wishes, Dr. Iliri"
},
{
"id": 118636,
"tgt": "High WBC, thrush from antibiotics given for the same, fatigue, low grade fever. No diagnosis from any tests. Any ideas?",
"src": "Patient: I have had a high wbc for well over a month, i have been on 3 different antibotics , admitted into the hosptial and was given iv antibiotics, now i have a higher wbc and thrush from the antibiotics, i am always tired, they have ran all the tests they say and cannot find anything actually wrong except for my wbc, they are sending me to a infectious disease doctor, i am always tired, and low grade fevers alot, , i have lots of people trying to tell me what is wrong, i am just wondering if there is something my family doctor is missing Doctor: Hi, Most probably you are having systemic fungal infection due to the use of antibiotics, for which anti-fungal treatment (systemic) is needed. Talk to your doctor about this option."
},
{
"id": 41923,
"tgt": "Does Oosure plus help in getting pregnant?",
"src": "Patient: sir i am married for 3yrs plus ....not conceived yet ....Have PCOS !! age is 27yrs ht is 5 ft 5inches wt 60 kg .....dr has given me oosure plus bd !! semen analysis is normal !! will it help thanks in anticipation sir mail id is YYYY@YYYY Doctor: Hi wel come to Health care magic.I have gone through your question.You are suffering from pcod ( polycystic ovarian syndrome). In that follicles dont develop upto the mark. Androgen level increased. Insulin resistance develops. Weight reduction. Metformin is frontline of management. You should consult gynecologist for the same.Oosure prescribed for increase follicles size.Take care."
},
{
"id": 23744,
"tgt": "What is the life expectancy for triple vessel coronary disease with diabetics?",
"src": "Patient: Wondering the life expectancy of an 81 year old male with the following - thank you - triple vessel coronary disease. Because of his age, diabetes and renal function bypass surgery is not an option. Dr feels as thou the Rt Coronary artery is the culprit vessel. It has two blockages in the Rt Coronary Artery. The artery is very large. He fixed both of those blockages with balloons and a stent. Excellent results. Because of his kidney function we can\u2019t give him any more contrast (dye) today. The other two arteries with blockages are very small diabetic arteries, significant disease but difficult to get to the blockages with balloons and stents not worth chasing right now. He wants to keep him over night. See how he does for the next few weeks and possibly stage the other two arteries down the road. Doctor: Hello ,Your doctor is quite right in all the decisions he has taken . opening up the culprit large size artery will help us tide over the crisis . Also at 81 years , This might be enough along with medication to help him live a normal life free of any symptoms. If at a later date in spite of optimisation of medicines he still has symptoms which impairs his quality of life , one may go for an elective angioplasty for left side vessels . There is no way that one can predict the life . Considering that acute crisis has been tied over and if there is not much damage to the heart muscle he should do good for at least short of some years . also we have to understand that angioplasty is not going to improve his lifespan , but would give him a better quality of life .Regards Dr. Priyank Mody"
},
{
"id": 173980,
"tgt": "What causes white creamy stool in kids?",
"src": "Patient: The 2 year old just had a white creamy stool (sort of pale white).. does not show any symptoms of diarrhea nor sick. He did had a mild fever two days ago but seems to be ok now. Yesterday, he did not wanted to eat but today he seems to eat normally. Any suggestions. is this something I should be concern about. Doctor: Hi...this can sometimes happen due to the food content. Nothing to worry about it. Unless the kid's having low urine output or very dull or excessively sleepy or blood in motion or green bilious vomiting...you need not worry. Moreover you say that the kid is eating normally today. An active child is a well child.Regards - Dr. Sumanth"
},
{
"id": 23517,
"tgt": "Can lisinopril taken for high BP lead to cough?",
"src": "Patient: Lisinopril was prescribed to lower my blood pressure. It helped a lot or about a week-- but then I got a terrible hacking cough that took 8 days to go away. The doctor switched me over to losartan. It worked pretty well-- but after 2 weeks I got a bad, hacking cough again. I cut the dosage in 1/4 (one tablet, 25 mg. is 4 pieces). That worked for a few days then I got horrible flu symptoms and the cough came back.I don't relish taking any pills ever-- but, my blood pressure is out of control without it. Because of a history of allergies and autoimmune things-- my theory about this sudden High BP is an allergy to cottonwood which is blanketing the entire town.I do have heart problems-- AI, AR, MVP, MR and a pulmonary vale & triscupid valve problem. I'm 60-- but in pretty good shape.Maybe my heart is just worn outor maybe I have allergy myocarditisI know see a doctor-- but none have openings and they are way, way too expensive anyway Doctor: your medicine comes under the class of drugs called ace inhibitorsone of the main side effect of this drug is dry coughso dont worry it the commenest side effect encountered by many patientsjust switch to another group of drugs called AT1 antagonist which has almost the same mechanism of action but dry cough is not a side effectswitch off current medicationand start tablet telma h once dailythank you!!"
},
{
"id": 163228,
"tgt": "Suggest treatment for eye twitching and mouth pulling in children",
"src": "Patient: Hello; My 9 year old daughter has this eye twitching and pulling of the mouth thing that she has been doing for some time. I am not sure how too help her. She often cries cause the children are starting to tease her. Please help. Must i take her to a doctor? Doctor: Hello and Welcome to \u2018Ask A Doctor\u2019 service.I have reviewed your query and here is my advice.These are involuntary muscles around her eyes and mouth contracting. There will not be a problem once the child is assessed by a doctor.You remain assured as there will not be a problem apart from the little embarrassment it causes.Rarely these can be a trouble in terms of getting more complicated. So, you must take her to a doctor.Hope I have answered your query. Let me know if I can assist you further.Regards,Dr. Prafulla Srinivas"
},
{
"id": 83880,
"tgt": "What causes loss in sense of smell and taste when on antibiotics?",
"src": "Patient: I have lost my sense of smell and taste. I am taking Symbicort, spyriva, zyrtek, singulair daily. I take flonase and albuterol when needed. I have been taking most of the meds since December. Flonase and spyriva since mid april. If I stop any of these, will my sense of smell come back? Doctor: HiAntibiotics and many drugs can cause loss of taste and smell.Many antibiotics affect zinc absorption and zinc deficiency can cause a metallic taste or loss of taste.Antibiotics can also kill good bacteria in the oral cavity thereby increasing the risk of oral infections.These infections can also affect taste and smell.Steroid inhalers can cause sore throat and increased risk of fungal infections such as candidiasis in the mouth and pharynx.Fungal infections can also affect taste and smell.Mouth gargling with salt water or oral antiseptics like betadine along with regular brushing and tongue cleaning,the symptoms can be controlled.Hope I have answered your query. Let me know if I can assist you further. RegardsDr.Saranya Ramadoss, General and Family Physician"
},
{
"id": 3879,
"tgt": "Can I get pregnant when suffering from PCOD and hypothyroidism?",
"src": "Patient: hello doctor, Myself shruti, i got married recently three months ago, and two months ago i got diaognised for hypothyroidism and PCOD condition. I am not getting my periods regularly. Is there any chance of conceiving? I got my TSH levels back in normal condition. Doctor: Hi Good evening PCOD and Hypothyroidism are easily treatable conditions. Follow your gynaecologist advice regularly to get your periods regularly. Then there should not be a problem in your conception. Check your TSH also regularly. Start taking Folic acid under your Gynaecologist supervision. Good luck."
},
{
"id": 53510,
"tgt": "What causes high bilirubin levels in LFT?",
"src": "Patient: i am going through test for lupus and i know i have a gallstone and was told awhile back my liver function was a little high but they just took my blood for like i said testing and found my bili to be high at 1.13 mg/dl is this high really ? i go to my gastro dr this week to see about getting my gallbladder out just need advice Doctor: Hi and welcome to Healthcaremagic. Thank you for your query. I am Dr. Rommstein, I understand your concerns and I will try to help you as much as I can.AST (SGOT) and ALT (SGPT) are reasonably sensitive indicators of liver damage or injury from different types of diseases or conditions, and collectively they are termed liver tests or liver blood tests. So the next step is to look for certain liver damage and most common casue is fatty liver disease. It should be confirmed by ultrasound. Other causes are viral hepatitis,cirrhosis, medications, alcohol intake or autoimmune diseases. To verify exact cause, US, bilirubin levels and tumor markers should be additionaly done Then appropriate treatment can be started.I hope I have answered you query. If you have any further questions you can contact us in every time.Kindly regards. Wish you a good health."
},
{
"id": 55982,
"tgt": "Is it normal to have abnormal levels of SGOT and bilirubin after gallbladder removal?",
"src": "Patient: My SGOT is 59, bilirubin is 1.7. Six years ago I had my gallbladder removed because the doctor thought it was to blame for my enzyme elevation. Before the surgery I was tested for everything possible and nothing was found now with no gallbladder the same problem is back, what could be the cause? Doctor: hi,There is slight elevation of SGOT and Bilirubin ,but i dont think its because of your gall bladder,we need to think of few other causes which will lead to such changes.I would have helped you better with complete liver function test,with this anyway you can get an ultrasound abdomen to look for any abnormality in the liver.If you are an alcholic there will be mild elevation of enzymes,and if you are having indirect hyperbilirubinemia then need to rule out causes for the same.Hope this information help you Thank you."
},
{
"id": 19660,
"tgt": "What are the complication due to getting stabbed in the heart?",
"src": "Patient: Hi, may I answer your health queries right now ? Pl My friend was stabbed in the heart and he went from ICU to the life trauma unit in less than two days.He seems to be doing better but I want to know if it is still a possibility he could die or have future complications? Doctor: Hello!Welcome and thank you for asking on HCM!Regarding your concern, I would explain that the risk of a cardiac arrhythmia may be present, due to the trauma. Anyway, it depends on the possible cardiac injury and other commorbidities. For this reason, I would like to directly review his cardiac ultrasound and ECG, coupled with blood lab tests (and cardiac enzymes), if you can upload them for a more professional opinion. Hope you will find this answer helpful!Kind regards, Dr. Iliri"
},
{
"id": 99159,
"tgt": "Could the black bruising on leg be an allergic reaction?",
"src": "Patient: Hello. I'm a college student who recently stated shooting up. I don't remember if this is a bruise or if it's a spot where I shot up the other day. I was hospitalized 2 days ago for a few hours due to a severe allergic reaction I had to something unknown. I was going to sit down and play on the Internet when I noticed on my leg a black bruise looking thing. About the size of a golf ball. It's pink and black much like a bruise but has darker purple spots in the center of it. It doesn't hurt, but looks odd. On my legs there are some little like sores on my feet like a picked zit, there is no pus and they seem to be going away. Around my crotch area and upper leg there's scratches, anfew kind of look like smeered blood, from where I itched so hard from the reaction the other day. My sinus mucus is a dark yellow, been gettIng a cold it seems. I'm curious, if i was in the hospital 2 days ago from an allergic reaction and they took my blood wouldn't they have seen if i was septic or not?? Or had blood poisoning? I'm done using needles for good, I just hope I don't lose my life or a limb. I don't have a fever right now, don't think I have had one recently. Doctor: Hi,Welcome to health care magic,I have read your query and understand the concern,This could be some sort of allergic reaction due to which there is blood extravasation has occurred at your upper leg area.You must need to consult general surgeon first to rule out any surgical possiblites like thrombosis or thrombophlebitis.You may require some blood works, ESR,PT with INR and ultrasound of local part.Treatment will be based on the underlying cause.However, you can take symptommatic treatment like antihistamines and topical steroid application.Regards,"
},
{
"id": 188284,
"tgt": "Have bad tooth, soreness in gum, swollen face. Suggestions?",
"src": "Patient: I have a bad tooth that has behaved itself mostly for some time. it has been sore to press on the area under my nose above the gum and to bit hard on it . .or if food gets into the decayed area . .but otherwise no biggie. This morning I woke up and my face is swelling in that area and sore . . . . the tooth no longer hurts however even to bite hard on it. Suggestions Doctor: Hi,Thanks for asking the query,According to your symptoms i suppose that you have developed infection in your tooth this had spread to the periapical tissue leading to pain and swelling.I would suggest you to visit to an Dentist.Get an x-ray done.Start with root canal treatment of the tooth.Take complete course of antibiotics and analgesics.At home take lukewarm saline and antispetic mouthwash rinses.Hope this helps out,Regards..."
},
{
"id": 117104,
"tgt": "How to reduce IgE level in blood?",
"src": "Patient: Hello. I have an Ig E level of 1200 wheras the normal level should be around 120. I am highly allergic to a lot of things. Every situation in which normal individuals sweat, like heat, tension, workout etc, i intead become red and develop a rash. Any solution? or a good immunologist in bangalore? Doctor: Hi, dear. I have gone through your question. I can understand your concern. You have high ig E. Its seen in allergy. You should take immunosuppressive drugs like steroids to control that. Consult your doctor and take treatment accordingly. Hope I have answered your question, if you have doubt then I will be happy to answer. Thanks for using health care magic. Wish you a very good health."
},
{
"id": 187307,
"tgt": "Is there any home remedy by which i can stop widening of gaps between my upper front tooth ?",
"src": "Patient: Hi!I have bad plaque on my bottom front tooth but i have little plaque on my upper front tooth. Still the gaps between my upper front tooth is widening day by day. Is there any home remedy by which i can stop it. Please also tell me foods for healthy gum & strong teeth. Doctor: Hello, Welcome Thanks for consulting HCM, I have gone through your query, as you have mentioned that you have deposition of plaque in between upper and lower gums and widening of tooth is also there dont worry for this you should consult dentist and go for Scaling and root planning . For widening of tooth you should go for orthodontic treatment go for braces . For healthy gums take proper healthy diet , green leafy vegetables , fruit juice. Avoid oily food , junk food . Do proper brushing , use chlorhexidine mouthwash 2 times a day. Do warm saline gargle two - three times a day.Hope this will help you."
},
{
"id": 45901,
"tgt": "What causes right sided groin pain?",
"src": "Patient: Sharp intermittent right side groin pain. Lasts approx 2 secs. Started for first time 2 months ago. History kidney infection, and because of this scar tissue in right kidney with small calcifications. Also had urinary tract infection a month ago. Doesn't seem to be related to movement. Seems to becoming more regular and sharper. Can't feel any signs of hernia. Doctor thought it might be nerve related but I'm not sure, thinking it might beva kidney stone? Doctor: Hello and Welcome to \u2018Ask A Doctor\u2019 service. I have reviewed your query and here is my advice. It is possibly a kidney stones. Previous infection might have caused predisposition to develop a stone. Get an ultrasound scan to look for any stones anywhere in your urinary tract. You can take antispasmodic drugs like cyclopam for pain relief. Drink plenty of water as minor stones will flushed away with hydration. If ultrasound shows stones better to consult a urologist and he will direct you accordingly. Hope I have answered your query. Let me know if I can assist you further."
},
{
"id": 81870,
"tgt": "How long should Lecope-M and Mahaflox be taken for cough?",
"src": "Patient: hello, I was suffering from cough and cold for 2-3 day, so my doctor adviced me to take lecope-m tablet for 5 days once a day along with mahaflox. After 5 days i was feeling better but not totally. So i went to consult the same doctor and he advised me to take Lecope-M MEDICINE for 1 month daily and once a day.. 8-9 days have already passed. Should i continue my medicine for the Month or i can stop taking this medicine. And is this medicine (lecope-m) harmful for my body if i took this medicine for a month. Doctor: See ideally mahaflox should have not been given foe a normal viral cold and cough. Such a higher antibiotic for no justified reason is foolish. Lecope again is a drug for allergic cough and cold due to asthma or allergic rhinitis. There is no harm if u continue lecope-M for a month, however it is not required.Consult another better doctor"
},
{
"id": 121410,
"tgt": "What causes severe sharp pain in the neck?",
"src": "Patient: I have an odd neck pain, it doesnt actually hurt until I turn my neck wrong way or move too suddenly then it feels like I got electorcuted , I end up not moving for that moment as the pain is bad, its like I disloacted my neck temporaily, pain goes to front to ears sometimes, can bulimia cause this, i cant seem to do normal movements with my neck without getting that sharp pain anymore, this started about a week ago when I thought i slept the wrong way and it would go away Doctor: Hello,You seem to have sprained your neck. This commonly occurs due to bad posture or improper sleeping positions. I would recommend you to use pain relieving gel (local application) along with hot fomentation. Please consult a physiotherapist for correcting your posture and also to strengthen your neck muscles,Hope I have answered your query. Let me know if I can assist you further. Regards, Dr. Santosh S Jeevannavar, Orthopedic Surgeon"
},
{
"id": 186783,
"tgt": "What causes swollen behind and between upper teeth?",
"src": "Patient: This morning when I was brushing my teeth there was one spot that I felt just a tiny bit of discomfort. Now, it is a little swollen behind and between one of my teeth, upper. It almost feels like I burned it with something hot, but that did not happen. I was just at the dentist for xrays, cleaning etc a couple weeks ago and all was fine. Is this something that will most likely go away? Doctor: Hello, Welcome Thanks for consulting HCM, I have gone through your query, as you have mentioned that you have swollen gum between upper teeth it can be due to periodontal problem or due to gingival Abscess. Do warm saline gargle two - three times a day. You can apply ointment Gum paint on painful gums twice daily . Consult dentist and go for Currettage , if it is carious then go for Root canal treatment. Hope this will help you."
},
{
"id": 120994,
"tgt": "Suggest treatment swollen ankle",
"src": "Patient: I dance over 30 hours per week. Last year in December, I injured my left ankle. Before the injury, it never popped or anything. It hurt bad and was swollen often as well as showed a little bruising. I saw several doctors. One described to me part of the problem by folding his hands together so that his fingers were interlaced and so that his hands made a parallel line to the floor. He said that the ligaments should be like that. Then he pulled his hands away from each other so that i could see holes through his interlaced fingers. He said that that was what ligaments were like now. Another doctor said that it could have been a sprain but that I should rest it and that it could heal on its own. Another (my dance doctor) said that the ligaments were strained. I had an MRI done and nothing showed up. So, another dance doctor said that the pain was probably from the swelling fluids. The injury is on the outside of my left ankle. It goes around the joint, starting below the joint and curving around to the right and up about an inch or two above and two the right of the joint. It is significantly more swollen than my right ankle. I ice it after dance. It has been going on for a full year now. I am almost 15 and 116-118 lbs now. What could this be? Doctor: Hello, You need clinical examination of your ankle where few special tests need to be performed.These findings have to correlated with MRI findings.If clinical findings are positive and MRI findings negative,You should undergo extensive supervised physiotherapy for 6 weeks. Hope I have answered your query. Let me know if I can assist you further. Take care Regards, Dr. Rajesh Gayakwad"
},
{
"id": 221577,
"tgt": "Can cough and cold cause harm during pregnancy?",
"src": "Patient: Hello, I am currently 25 weeks pregnant and have been suffering from dry cough for the past 1 month. I initially took Robitussin cough syrup and zyrtec for a couple of days and then I stopped taking them since I was too reluctant to take any kind of medicines during pregnancy. I do have stuffy nose as well. The cough is mainly dry but I do feel a bit of mucus in my lungs when I cough. During the daytime, the cough is intermittent but it really gets worse in the evenings/nights. I have been trying to resort to most home remedies like steam inhalation, gargles as well as honey and lemon drink. My question to you is if I continue to not take any medications and cope with this cough on my own, will my cough and cold harm the growth of my baby? I don't know for how long with this cough last but is it important for me to get this cough fixed by taking medicines or is it fine if I continue with my home remedies? I am just worried about its effects on my baby. Please advise. Doctor: HiDr. Purushottam welcomes you to HCM virtual clinic.I have gone through your query. I think I have understood your concern, I will try to suggest you the best possible treatment options.1] If you are not having cough with yellow or green sputum, then non medical remedies can be taken.2] I will suggest to take DUOCORT inhaler 2 puffs 3 to 4 times a day for 5 days.3] Only problem with dry cough is that it causes pressure over lower part of tummy, and sometimes involuntary passage of urine. Otherwise cough, cold treatment as such should not pose harm to baby or pregnancy.I hope my answer helps you.Thanks.Wish you good health."
},
{
"id": 80131,
"tgt": "What precautions should be taken after having chest pain?",
"src": "Patient: hello.. my dad has chest pain at times and this has become commom especially during the winters. he has got his tmt done and it has come out to be mild..please advice how serious is the situation and what precautions he should be following.Please help. Doctor: HI,Dear, Thanks for the query from you for your dad.1-Though you didnt update the age of your dad and didnt update how many winters he is being having these chest pains-1-a-From the facts told by You, and In my opinion -your father , who is having frequent chest pains during winters specially,is really serious case of chest pain and was lucky to enjoy the bygone winters by the grace of GOD,1-b-Though your tmt came out to be mild, you should take it seriously,as the test give just a guide line of the internal changes, but the duration for which your father had chest paint, indicate that his heart has a seriously compromised blood supply.2-So my caution and advise to you is that you should take your fathers chest seriously, and go to ER physician or cardio-specialist and get his -Angiography- done to judge the severity and extent of the compromised heart blood supply, 2-a-and if the your ER cardio-advises andy corrective stent and angioplasty, plz get it done ASAP, to add more healthy life and to add active yrs to his old-age llife. -3-And as it is hearting him in every winter , you should take my CAUTIOUS-ADVISE very seriously-to save his life URGENTLY, though his tmt report is mild.3-Still in the meanwhile-get some anti-anginal drugs with the advise of ER physician-but dont depend on it to delay further CAutious-Treatments any more.4-So- I would advise you to URGENTLY check up with the local ER Cardio- doctor and to get it treated ASAP-urgtly.4-Hope this would solve your query.5-Wellcome to HCM for any more query."
},
{
"id": 138697,
"tgt": "What could lump formation after a jaw injury indicate?",
"src": "Patient: I was head butted by my horse. She caught me just under the cheekbone. I had bruising that I assume gravity caused to show up at the jaw. The bruising is long gone but now I have a fairly large sore lump in my cheek spanning between the cheekbone and jaw. What is it and what s caused it? Doctor: Dear Sir/MadamI have gone through your query and read your symptoms.In my opinion, this could be due to infection or hematoma collection, and in both the cases you will need to see your doctor, and ask him to get yourself investigated, as normally staff do not follow the recommendations of the concerned doctor, and prefer to follow whom they faith more.I hope that answers your query. If you want any more clarification, contact me back."
},
{
"id": 50510,
"tgt": "Pain around kidneys, ultrasound shows multiple renal calculi, fullness of pelvicalycial system. What medicines should be taken?",
"src": "Patient: recently i have done ultrasound of kidneys due to some pain in kidney..ultrasound report shows that ..multiple calculi are seen in both kidneys, largest one measure 0.8 cm is seen at mid pole in right kidney and another largest one in left kidney measure 0.8 cm at upper pole. fullness of pelvicalycial system is seen in right kidney.no renal parenchymal disease is seen. no cycst seen in both kidneys. normal seen of gall bladder,common bile duct , pancreas , spleen and urinary bladder . conclusion: multiple bilateral renal calculi. fullness of pelvicalycial system is seen in right kidney possibility of lower down obstruction should be ruled out. i wana ask about this and about the mediciations of it. thank you pls help me online or email me at YYYY@YYYY thank u Doctor: Hi, many thanks for the query!You need to do- Sr. creat., Sr. uric acid, X-ray KUB, Urine (R&M),IVU (Intra-Venous Urography).If stone can be seen causing obstruction in lower ureter, surgical removal should be done, ureteroscopic removal (URS) is preferred these days.Take antispasmodics, pain killers, diuretics, tamsulosin with your doctor's opinion.Drink plenty of water so that at least 2 litres of urine is voided in 24 hrs.Wish you a good health.Take care.Regards."
},
{
"id": 169346,
"tgt": "What causes yellow eyes and mouth in new born?",
"src": "Patient: HELLO MY GRANDBABY HAS BEEN HAVING YELLOW EYES, EVERY SINCE SHE WAS BORN. BUT THE DOCTOR SENT HER HOME. NOW HERE EYES HAS GOTTEN MORE YELLOW, AND HER STOMACH IS REAL TIGHT. NOW SHE IS IN THE HOSPITAL SHE COULDN T BREATH. THIS DOCTOR SAID THAT HER MOUTH WAS YELLOW, SO HE TEST HER BLOOD AND SAID THAT IT WAS TO HIGH, SO WHAT DOES THAT MEAN. THEY NOW SAYING THAT THEY MAY HAVE TO DO SURGEY ON HER SHE IS 4 MONTHS OLD, IS THIS A GOOD IDEA. SHE MIGHT HAVE TO HAVE A LIVER TRANSPLANT. Doctor: could be neonatal jaundice which only needs photo therapy.but jaundice due to biliary atresia need to be operated as it cannot be managed medically."
},
{
"id": 31445,
"tgt": "Could recurring perirectal abscess be due to fistula?",
"src": "Patient: Hi, may I answer your health queries right now ? I have a perirectal abscess and it ruptured two days ago and now is filling up again, I have an appointment in 2 days with a colorectal surgeon. Does this mean I have a fistula, this is the third one in 2 months. Doctor: Hi thanks for asking question.You have perirectal abscess which burst so ya chances of fistula is there.For that rectal examination has to be done by proctoscopy.And you mentioned that this is for the third time.So it is better to get examined proctoscopy.I hope you have no further query regarding approach for your case.thanks."
},
{
"id": 206792,
"tgt": "Suggest remedy to overcome OCD",
"src": "Patient: i think i might have ocd..every nite before i go to bed for about the last 4 years i have to lock all the doors of the house (which isnt so bad) but in the last year i have to look in my cupboreds and under my bed..its realy stupid i`ve asked myself what i thinks going to be there and i dont know..im woried that its going to get worse..and the worst part is if i leave my room i have to do it ALL again Doctor: Hello,As you have described your problem, it appears that you are right in diagnosing your problem yourself. Yes it is OCD. Before it worsens further, I think you should start treatment with anti-obsessional drugs like tablet sertraline 50 mg once at bed-time daily. Dose might need to be increased depending on the response.Hope your problems resolves at the earliest.Thanks."
},
{
"id": 166823,
"tgt": "What causes breathlessness in children?",
"src": "Patient: My son, 11+ recently seems to have breathlessness concerns. Not all the time and whenever he breath hard, I noticed his lips went pale as if lack of blood. Sometimes nausea. He is of the small built, not obese, no soft drinks , nor indulge in junk food. What can be wrong? Jo Doctor: Hi,You are right to be concern breathlessness could be a result of cardiac, pulmonary or otorrino problems. You must make a check up with him with a doctor and ask for exams to check this organs, ok?"
},
{
"id": 125108,
"tgt": "What causes swelling, blistering and discoloration from the knees below along with cramps in the calves?",
"src": "Patient: For the past 30 days my legs has been experiencing swelling blistering changing color from the knee down and cramps in my calf that last a minute and it goes on for about a half hour Doctor: Hello and Welcome to \u2018Ask A Doctor\u2019 service. I have reviewed your query and here is my advice. Swelling may be due to cellulitis or edema due to cardiac or renal failure liver failure or Infections or myxedema or filariasis or varicose veins etc. Until examination is done it is difficult to say what it is. Keep on elevation of your limbs , use crepe bandage if no blisters , use antibiotics and pain killer medication. If symptoms not improved please consult your physician he will examine and treat you accordingly. Hope I have answered your query. Let me know if I can assist you further."
},
{
"id": 113798,
"tgt": "Lower back pain due to pinched nerve, treatment",
"src": "Patient: I have pain on my lower right side going into my back my doctor says its a pinched nerve but will not give me anything for the pain or find out where exactly it is and always wants to pass the buck on to someone else i have told the referral line i have no faith in this doctor he does not want to listen to me go for a nerve block shot i told him i had those and they dont work Doctor: Hi,Rrgagnonrite, Thanks for query, It is not clear that you get pain on lower right side,whether pain is on right lower abdomen going back. If this is picture then possibilities are, 1,Appendicular pain, 2,Renal colic, 3,lumbo sacral spondilitis. Go for X ray lumbo sacral spins,routine urine check up. Mean while you may take some anti spasmodic,analgesic medicine. Ok and bye."
},
{
"id": 28461,
"tgt": "Suggest treatment for rib cage pain and heart palpitations",
"src": "Patient: Hi, I suddenly have a horrible feeling under my ribcage on the left near my heart but just under, it s similar to a heart palpitation but it goes on for about 40seconds at a time and happens every few minutes or so and feels like something is beating very hard against my ribs or skin. I can feel it with my hand if I touch my ribs when it s happening. Any idea what this is as I ve never experienced this feeling before. Thank u. Kirsty. Doctor: Hi welcome to HCM.I understand your query and concern.This kind of rib pain should be ruled out for pericarditis as in your case.This needs following set of investigations.I advise you to have a baseline 2 dimensional echocardiography,ECG and lipid profile to assess the basic cardiac reserve of your heart.Restrict the intake of salt to less than 6g/day.Regular physical exercise in the form of brisk walk for 20 min a day for 5 days a week is pretty useful.Drugs like antihypertensives and antiarrhythmics will help.Reduce the intake of fatty and fried food.One pomegranate a day will help to keep your heart at good pace without clot formation. Steroids will help to control the symptoms effectively.Consult a Cardiologist for further expert management.Post your further queries if any.Thank you."
},
{
"id": 58842,
"tgt": "Has high Bilirubin count and sweats. Had knee replacements on both knees. Eyes are yellow. Suggestions?",
"src": "Patient: My husband has been told he has a high Bilirubin count, He will not get to see a Dr for another 10 days. We are worried because he has been having terrible sweats for 2 years now. He has had his heart check several times with no complication. He had knee replacements on both knees and has never fully healed. I think his eyes are not completely yellow but some yellow. Last time he was at his pcp I asked and was told it was allergies. He is not a drinker or a smoker. Doctor: Hi,Did your husband recieve blood during the surgery? If he did, it is possible that he may have gotten hepatitis. It is also possible to get hepatitis due to halothane which is an anesthetic in surgery. These are the 2 possibilities that seem most likely. Yellowish discoloration of skin is a clearcut sign of liver disease. What he needs to do is consult the surgeon who did the procedure and tell him the symptoms. Then the doctor can suggest the appropriate treatment. I hope this helps. Wish you the best.Dr Vineet"
},
{
"id": 211302,
"tgt": "What is the treatment for sleeplessness and mental problems with hyperactivity?",
"src": "Patient: HELLO DOCTOR,MY NAME IS LAKSHMI(18 YEARS). I\u2019M ASKING THIS QUESTION ON BEHALF OF MY GRANDMOTHER SAROJINI NARAYANAN NAIR. ME AND MY PARENTS LIVE IN UAE, AND MY GRANDMOTHER LIVES WITH MY UNCLE AND HIS FAMILY IN KANNUR, VALAPATTANAM, MANNA. MY GRANDMOTHER IS 72 YEARS OLD, DURING THE MONTHS OF NOVEMBER AND DECEMBER SHE SUFFERS A PROBLEM OF LACK OF SLEEP AND MENTAL TROUBLES WHICH MAKES HER MORE ACTIVE AND NON-STOP TALKING.WOULD YOU PLEASE SUGGEST SOME MEASURES TO CURE HER. Doctor: HIThank for asking to HCMThis is geriatric problem I personally do not advise any medicine for this because this is age related problem, people are generally asking for medicine because they get disturb due to the behaviour of old person and not to treat the old person, which is not fair, tell every body to spend more time with her share the talk with her these would be best measure for her, have nice day."
},
{
"id": 25778,
"tgt": "Suggest solution to high bp",
"src": "Patient: Hi! I am a 49 year old female. I have always had \"normal\" blood pressure readings. On Friday had went to my GYN for my annual pap, and my blood pressure reading was \"high\". They took 3 readings: 146/96, 142/96, 148/92. I checked it again that night and it was 144/97. Saturday it was 140/103. I did not check it Sunday. Today it is 152/105. Should I be concerned? My GYN said if it continued to be \"high\" to call my Internist. Doctor: Thanks for your question on Health Care Magic. I can understand your concern. Yes, you should concerned for these blood pressure readings. Upper limit of Normal blood pressure is 140/90mmhg.Your all blood pressure readings are more than this, so you are definitely having hypertension. You should consult your internal medicine doctor for this. You will need antihypertensive drug, strict salt restriction in diet and exercise. Antihypertensive drugs include amodipine, telmisartan, Losartan, ramipril etc. So don't worry, you will be alright. Avoid stress and tension, be relax and calm. Hope I have solved your query. I will be happy to help you further. Wish you good health. Thanks."
},
{
"id": 213132,
"tgt": "Early eighties, have anxiety, get unconscious with sleeping pills, crave for tea. How can I overcome this?",
"src": "Patient: My wife s grandpa aged 80+ thin health was active until 75...now complaining for anxiety / sometime its aggravate. He can t take slipping pills even 1/4 of tablet...if its given 1/4, he gots unconscious for long hours 12-18hrs. What to do? Sometimes he continues hue & cry all the day asking for food especially tea. (don t take much food) Doctor: This condition is known as Senile Dementia but needs proper evaluation,can be controled with mild dose of suitable antipsychotic along with Donepezile,rivastigmine,etc."
},
{
"id": 212510,
"tgt": "Obsessive-compulsive disorder, heridetary schitzophrinia. Symptoms treatment?",
"src": "Patient: I have been experiancing SEVERE BUTTERFLYS AND very scary feeling for about a month I am unable to perform normal task due to the overwhelming difficulty it puts my mind to, I was unable to eat and woke up every morning with this feeling that last all day and night the Mental health put me on Prozac and wellbutrin cause I have bipolar 2 and obsessive compulsive disorder and add, but it did not take it away then a regular dr. put me on hydroxazine which only helped about 30 percent but now i went back and he put me on zanax and lexapro which since then it has helped since 2 days ago but my modivation level is not back I cannot seem to ever clean my house it is severly overwhelming to me is this some form of phobia there are many more symtoms I live with daily to long to go into but have got to figure out how to get my motivation back and become social again also since lexapro works on the dopamine in the brain and prozac works on the seratonin balance can i take some of each I have mother and brothers and grandfather with schitzophrinia and have at least 20 phobias thanks lucy Doctor: Hello and welcome to Healthcare Magic. Thanks for your query. I understand that you are going through a difficult time due to your multiple psychiatric problems. The medication which you have been started on recently is a combination of an SSRI anti-depressant (Lexapro) and anti-anxiety medication (Xanax). It is important to take the medication as advised by your doctor striclty and not make your own treatment decisions. (By the way, Lexapro does not work on dopamine - now, this is why it is important to follow the advise of qualified specialists, rather than make such wrong assumptions and try to make your own decisions) SSRI anti-depressants like Lexapro may take about 2 weeks to bring about their beneficial effect and so, you will have to wait for some more time till you see significant improvement. Wish you all the best. Regards, Dr. Jonas Sundarakumar Consultant Psychiatrist"
},
{
"id": 55098,
"tgt": "What causes yellowish sclera with normal bilirubin levels?",
"src": "Patient: Hi.I have noticed that my sclera is a bit yellowish compared to others.Though it s not the entire sclera only the exposed portion.I have normal bilirubin level and liver enzymes.But the thing is I have a low hemoglobin level.I m just wondering what might be causing the yellowish discoloration.Thank you. Doctor: hi.noted history of yellowish discoloration over the exposed sclera. this is what you call a dirty sclerae, secondary to environmental factors. if you want a clinical evaluation, a consultation with your doctor is still best, for complete physical examination and confirmation. good hygiene is also advised and avoidance of polluted areas.hope this helps.good day!!~dr.kaye"
},
{
"id": 58735,
"tgt": "Pain in upper abdomen after liver biopsy, lower back pain. Having hypothyroid, lupus. Should i call hospital where biopsy was done?",
"src": "Patient: I am 58 yr old female with diagnosis of Hypothyroid, MS, Lupus, Arthritis, Sj\u00f6gren's syndrome, I have had elevated liver enzymes since 9/2012 ... meds have been ruled out ... ultrasound showed nothing ... liver biopsy on 7/9/2013 and waiting on results ... immediately after the biopsy I felt some pain/pressure in my upper abdomen just under my rib cage ... having a dull constant pain in the area of my left kidney and also some low back pain ... could any of this be a result of the liver biopsy or something else that has to do with my elevated liver enzymes ... should I call the doctor or hospital where the biopsy was done? sick of being sick! Doctor: Hi and welcome to Healthcare magic.Thank you for the query.yes,you should. There are few serious compications of liver biopsy including bleeding. So you need to do ultrasounds to rule this out. But it can also be pain frombiopsy because liver capsule is roken and this is commonly very painful in some people. YOu need to take strong painkillers and patient for several more days. Also,you should rest laying on your stomach.Wish you good health. Regards"
},
{
"id": 210436,
"tgt": "What causes periodic anxiety, claustrophobia and quite sudden onset?",
"src": "Patient: I suffer physically from periodic anxiety, clausterphobia , quite sudden onset but not often and have never had this occur before this past year. What's causing it, is my diet related to this? I can't see any obvious difference in my routines, habits etc.... I am physically active, 40 year old mom of two (if that helps??) Doctor: DearWe understand your concernsI went through your details. I suggest you not to worry much. Claustrophobia is the fear of having no escape and being in closed or small spaces or rooms. Are you getting it right? For every occurrence, there has to be a trigger. Diet has very little to do about it. It has to be your lifestyle. Prepare a log sheet of every activity you do in a day. also mark the occurrences of periodic anxiety and claustrophobia. Over a week's time, you should get a pattern. Or continue the log till you get a pattern. Or, the given details are not enough to diagnose your problem.Please post a direct question to me in this website. Make sure that you include every minute details possible. I shall prescribe some psychotherapy techniques which should help you cure your condition.Hope this answers your query. Available for further clarifications.Good luck."
},
{
"id": 80476,
"tgt": "Reason for the pain and tightness in the chest?",
"src": "Patient: I have been having pain and tightness in my chest for approx 20-30 mins feels like a huge cramp going from honey area between breast and straight through to back helps a tiny bit if hold my breath but not even have no indigestion to my knowledge burping just fine Doctor: Hello dear, thanks for your question on HCM. I can understand your situation and problem. Your symptoms like central chest pain and burping are suggestive of GERD ( gastroesophageal reflux disease ) more. But better to first rule out cardiac cause for your chest pain. So get done ecg. If ecg is normal then no need to worry for cardiac cause. GERD appears more.It is due to laxity of gastroesophageal sphincter. Because of this the acid of the stomach tends to come up in the esophagus and cause the symptoms. Try to follow these steps for better symptomatic relief. 1. Avoid hot and spicy food. 2. Avoid stress and tension. 3. Avoid large meals, instead take frequent small meals. 4. Start Proton pump inhibitors. 5. Go for walk after meals. 6. Keep 2 - 3 pillows under head in bed to prevent reflux. Don't worry, you will be alright."
},
{
"id": 49159,
"tgt": "Any advice for a 55 yr old having difficulty passing urine which is more like a watery stool with no pain in kidneys?",
"src": "Patient: MY HUSBAND (55YR) HAS BEEN HAVING TROUBLE PASSING URINE FOR A FEW DAYS - FULL BLADDER, SMALL AMT OF URINE AND IN AGONY WHEN TRYING TO GO. LATER TONIGHT HE NOTICED HIS URINE WAS A REALLY DARK COLOUR WITH WHAT LOOKED LIKE PARTICALS OF DUST OR MINUTE BITS OF SOMETHING (MORE LIKE A WATERY POO) AND HAD A COUPLE OF TINY LITTLE BLOOD CLOTS. THIS IS NOT THE FIRST TIME HE S EXPERIENCED TROUBLE GOING BUT NEVER WITH BLOOD CLOTS OR BITS OR THIS DARK. I SUGGESTED IT MIGHT BE HIS PROSTRATE INCREASING. NO PAIN IN KIDNEYS. NEARLY IN TEARS TRYING TO PASS URINE THOUGH. CAN YOU HELP OR ADVISE PLEASE? Doctor: hi, your husband seems to have 2 issues1. prostrate enlargement resulting in straining to void urine2. severe urine infection. you need to get an ultrasound abdomen for prostrate and cystitis or any renal stones. urine microscopy and culture will identify the infection. before that, you may try OTC antibiotics like norfloxacin. Hope this helped."
},
{
"id": 17657,
"tgt": "Can heart be bruised or enlarged leading to chest pain?",
"src": "Patient: i was in a car accident like 2 months ago, broke a few bones, had surgery(hip and femur)a month later i began feeling left chest pains, left arm pains...typical heart attack symptoms in other words. last night was the 5th time i went to the e.r. thanks to this pains and again they found nothing wrong with me or my heart so i guess the ekg and mri were all good. the pain is still there on a daily basis my parents refuse to take me to the hospital they think i m insane. my questions are can my heart be bruised or enlarged?or that would have been ruled out with the ekg and mri?they said its not my lungs but the pain is on left back also, could it be only muscle pain?i don t have insurance and all this medical assistance is pretty expensive. if anyone could please get me on the right direction i would greatly appreciate it, this pain is so stressing it feels like a tickle on my lung or heart and i m scared if overlooked it will kill me. by the way i m a 19 yr old grl and nobody in my family has any kind of medical problems.... Doctor: Hello, I passed carefully through your question and would explain that it is less likely that your symptoms are related to the heart. In my opinion, your symptoms could be related to a musculoskeletal pain or anxiety. The resting ECG and MRI can help exclude a serious medical disorder. I would like to review your tests reports if you can upload them for a second opinion. Another option would be to perform a cardiac exercise stress test in order to completely exclude any cardiac issues. Hope I have answered your query. Let me know if I can assist you further. Regards, Dr. Ilir Sharka, Cardiologist"
},
{
"id": 93175,
"tgt": "Having sharp pain in lower abdomen, bloated stomach. Found ovarian cyst. Should I go for medical check up?",
"src": "Patient: Hi i have had sharp pains in my lower abdomen since saturday and my stomach is very bloated it seems to go away and come back ( been in hospital 2 months ago and had an overian cyst size 3x4 cm i have check ups with a gynacologist and it seems to have shrunk down to 2,5 x 2 cm about 2 weeks ago they have told me that if i got sharp pains again i should go back into hospital. What would you recommend? Many thanks Cindy Doctor: hi,thank you for your queryAn ovarian cyst is any collection of fluid, surrounded by a very thin wall, within an ovary.most of the cysts are harmless under the size of 5cm.most patients experience cyclical pain due to ovulation that occurs every cycle.if the pain is interfering with your daily life you should consult a doctor.since you'vs already been diagnosed i would advise you to 1.keep track of your symptoms. Mark on your calendar when symptoms seem to flare up, like headaches, bloating, or pain.it will show you how close to ovulation some symptoms happen, which in turn will help decide your treatment.2.Pain relievers, including acetaminophen/paracetamol, nonsteroidal anti-inflammatory drugs such as ibuprofen canbe taken.3.A warm bath, or heating pad, or hot water bottle applied to the lower abdomen near the ovaries can relax tense muscles and relieve cramping, lessen discomfort, and stimulate circulation and healing in the ovariesbut as i said if the pain is extreme or sudden pain, associated with heavy bleeding, or other concerns, don't wait and see if it will go away.cosnult your OB/GYN immediatelythank you,wishing you good health"
},
{
"id": 75932,
"tgt": "What can cause pain in ribs?",
"src": "Patient: i have a pain on the left part of my ribs and i don't know the curse some times it makes me feel so cold and i cant stay in a room where an airconditon is put on is and it make's a lot of noise as if thire ar too many gases in my stomach some times i have to press my ribes so hard just to relive the pain what culd this possiblly mean ? Doctor: Thanks for your question on Healthcare Magic. I can understand your concern. By your history and description, possibility of GERD (gastroesophageal reflux disease) is more. GERD is due to laxity of gastroesophageal sphincter. Because of this the acid of the stomach tends to come up in the esophagus and cause symptoms of gaseous distention of stomach, chest pain, chest tightness, pressure etc. So follow these steps for better symptomatic relief. 1. Avoid hot and spicy food. Avoid junk food. 2. Avoid stress and tension, be relax and calm. 3. Take pantoprazole and levosulperide combination on empty stomach. 4. Avoid large meals, instead take frequent small meals. 5. Loose weight if you are obese. 6. Quit smoking and Alcohol if you are having these habits. Don't worry, you will be alright with all these. Hope I have solved your query. I will be happy to help you further. Wish you good health. Thanks."
},
{
"id": 117459,
"tgt": "Is rbc of 4-10 level a cause for concern?",
"src": "Patient: I just came from the dr's office and rcved my blood work. I am a 52 yr old female and everything was withing range except my RBC. It says 4-10. My doctor is not concerned. Should I be? I did have my mentral period when I had my blood drawn if this has a barrier on it.. Thank you Doctor: Nothing abnormal. RBC count if it falls <4, then there is some cause of anemia due to decrease RBC mass. If your hemoglobin is normal and RBC count 4.10 million/cumm, then do not worry."
},
{
"id": 111369,
"tgt": "How to treat the pain due to disc prolapse?",
"src": "Patient: yes sir this is s.m karim i have disc prolapse before 20 years and had surgery in dxb but after 2 years again i had pain started before 2 years i took mri it was shown that i had l3-l4-l5and s1 problem dr advised me to do surgery what is your advise Doctor: It depends on the severity of the disc injuries. But the rule in general regarding the spine is to keep surgery only as a last resort. Try everything else first: physical therapy, massage, acupuncture, medications, etc. Then only if things remain unbearable then consider surgery, because the outcomes of spine surgery simply aren't as reliable as we would like (or as reliable as the surgeons would admit!)."
},
{
"id": 80783,
"tgt": "How can one be shifted between the hospitals with ventilation machine?",
"src": "Patient: hi, please tell me is it good to travel a patient with ventilated machine? This patient now in attached to the ventilated machine and we want to take her to another hospital? how we can do this? how long we can keep her after remove the machine till go to next hospital? please let me know what to do? Doctor: Hello dear, thanks for your question on HCM. I can understand your situation and problem. It is extremely risky to shift patient with ventilatory support. So just make sure that patient is having genuine reason for transfer because during transfer patient can be critical and may expire.Now a days, ventilators with battery back up are available. In these ventilators, we can continue machine for 1 hour during travel.Another way is to start ambu bag ventilation in patient. So discuss all these in detail with your doctor and then take decision about transfer."
},
{
"id": 94509,
"tgt": "Pain below the ribs. Pain while laughing and coughing. Do I have hernia?",
"src": "Patient: Yes, since Thursday I have had a small quarter size pain in my stomach below my ribs but not deep inside my stomach but rather closer to the outside of my stomach. It s a short sharp pain that has been occurring only when I laugh hard, sneeze or cough . Just about an hour ago I bent down to pick up a toy off the floor and felt the sharpest pain ever, like a ripping feeling. I m thinking I have a hernia but not sure. I can actually touch the spot that hurts but there is no bulging like described on the mayo clinic sight. I m not trying to self diagnosis myself but rather get myself prepared for what my dr might tell me it is. Also wondering if next Friday is ok to wait for my appt. Doctor: Hello, HeatherRudd07, From your description and the location , it does sound like you have an \"Epigastric Hernia \". If you also see a lump in that spot when you strain, or cough , that confirms it. It could be a very small defect and abdominal contents like small portion of the intestine can protrude through the defect. If the prtrusion stays and if you can't push it back, then it becomes an emergency situation. That is called as \"Strangulated Hernia\" and requires emergency surgery for correction. If you test yourself as above, you can wait till you see your doctor. In the mean time avoid heavy lifting etc. You can wear a velcro abdominal binder for protection. I wish you well."
},
{
"id": 49053,
"tgt": "Does long term nephrology treatment result in yellowing of tooth?",
"src": "Patient: AS my son 6 yrs old ,he getting treatment in nephology ,he used to take omnacotil according Dr.dosage,any side effect while he take this medicine ,bc's now my son permanent front tooth got yellow colour,if i check with dental Dr. there is calcium defenency effect ,pls advice ,this treatment start from 2 yrs old. Doctor: hello thanks for consulting at hcm..plz consult ur dentist for acheck up and review with ur nephrologist,,since corticosteroid does cause calcium deficiencies,,u have treatment for this like veneers,,hope it helps,,tc"
},
{
"id": 203781,
"tgt": "Loose scrotum, hanging lower than testicles, loose skin. Treatment?",
"src": "Patient: Hi, My scrotum has become very loose and hangs lower than my testicles. Imagine a bag with a couple of oranges at the bottom of the bag. -This is normal. Imagine the oranges are held half way up the bag. So there is excess empty bag below the oranges. -This is my situation. I can t find any information regarding this. Please help. Doctor: DearWe understand your concernsI went through your description. I suggest you not to worry much. As you are aware about the normalcy of your testicle sacks. You know that they can be loose and hang down along with the testicles. You should also learn that the testicles are not actually two free oranges. They are attached to the scortum with the help of blood veins and other tube like structures. Means even when the sack hangs, the testicles can be anywhere in the sack. There is nothing wrong in your case.You might need psychotherapy and other cognitive behavioral therapies. I can help you through telephone. Please do contact me through customer care of www.healthcaremagic.com for a telephonic consultation session. Hope this answers your query. Available for further clarifications.Good luck."
},
{
"id": 52877,
"tgt": "What does the elevated liver enzymes and low white cell count indicate?",
"src": "Patient: I have a horseshoe kidney located right below the belly button. I have all the symptoms that go along with gallstones and or pancreatitis. I get stabbing pain right below the breast bone towards the naval area that then radiates to my lover back. Had CT of belvic and abdomen and shows no inflamation but many abnormalities with organs. Blood work shows elevated liver enzymes and low white cell count. What would you think may be causing these symptoms? Doctor: Hi and welcome to Healthcaremagic. Thank you for your query. I am Dr. Rommstein, I understand your concerns and I will try to help you as much as I can.SGOT and SGPT are sensitive indicators of liver damage or injury from different types of diseases or conditions, and collectively they are termed liver tests or liver blood tests. So the next step is to look for certain liver damage and most common casue is fatty liver disease. It should be confirmed by ultrasound. Other causes are viral hepatitis,cirrhosis, medications, alcohol intake or autoimmune diseases. To verify exact cause, US, bilirubin levels and tumor markers should be additionaly done Then appropriate treatment can be started.I hope I have answered you query. If you have any further questions you can contact us in every time.Kindly regards. Wish you a good health."
},
{
"id": 114928,
"tgt": "What causes blood clot in arm?",
"src": "Patient: My husband possibly has a blood clot in his upper arm. Painful but no redness nor swelling detected, he was hospitalized about 10 days ago and they could not get an IV started. They made about 8 attempts and finally he told them no more. This area bothering him now is painful and that is where they tried the IV a couple of times. He is on plavix. I have put an ice pack on the area , is this the correct thing to do. He has dr appt tomorrow late afternoon. Doctor: Hi, I am sorry for the situation your husband is going through. It is normal to get blood clot after trying to put on IV. Sometimes, they got infected at the site of injection. Apart applying cold compresses, I advise to look for signs of infection. If no infection, then, advised to apply heparin cream locally if there is bruise at the site of injection. Taking plavix will make it longer for the blood clot to get healed. All the best!"
},
{
"id": 140999,
"tgt": "Suggest remedy for sleeplessness in a person suffering with cerebral palsy",
"src": "Patient: I have a 19 year old son with cerebral palsy and he has a struggle staying asleep. I give him 10mls chorale hydrate at 8.30,put him to bed at 9 as he has fallen asleep on my lap. But he wakes and continues to scream and won t settle. I have to wait till about 11.30 to give him another 10mls chorale hydrate. Sometimes I have to give him diazepam but don t want to give him that really. He seems to have a lot of trapped wind as he is double up with it and when I get him out of bed and back in the living room he burps. I just need something to get him asleep and him to have a full nights sleep as he is p.e.g feed at night so if he is asleep he will really benefit. Doctor: Hi, You should consider giving your son medication for sleep such as Valerian, Melatonin, etc. Also, chamomile tea in the evening may help promote a better sleep. Diazepam may be used and should be avoided prolonged use to avoid dependence. Hope I have answered your query. Let me know if I can assist you further. Regards, Dr. Erion Spaho, Neurologist, Surgical"
},
{
"id": 55276,
"tgt": "Will fatty liver cause dizziness and stomach problems?",
"src": "Patient: Hi I just got some blood work done and my gyno said I might have fatty liver. My blood said I was at 168. Would this cause me to have dizziness and stomach problem and unable to lose weight. I've also had lingering heaches for about 6 months now. I'm taking armour thyroid and Glumetza. Doctor: Hello! Thanks for putting your query in HCM. I am a Gastroenterologist. Fatty liver usually dont cause dizziness, however abdominal discomfort or mild abdominal pain can be due to fatty liver.Also fatty liver has many cause like Hepatitis B, C, Metabolic disorders, NAFLD, alcohol intake etc. So I will suggest you to visit a Hepatologist for evaluation who will advise tests and then manage accordingly. Meanwhile there are few medicines which I found very useful in my patients are ursodeoxycholic acid and vitamin E. Do regular exercise and reduce your weight if you are overweight, avoid alcohol if you drink.I hope I have answered your query and this will help you . Wish you a good health"
},
{
"id": 218099,
"tgt": "What should be done to treat pain after hiking?",
"src": "Patient: Hi dr my friend was in a accident 4 days ago and was discharged from hospital his ears are still bleeding and he complains that I his head feels heavy... with that said yesterday afternoon he felt beter and went hiking all these pains came back last night... Doctor: Dear Friend.Hi , I am Dr Anshul Varshney , I have read your query in detail , I understand your concern.Pain in head can be due to Post traumatic Concussion syndrome.But bleeding ears is not acceptable.I would advise you to get treatment from your ENT specialist.Bleeding ears also require a CT Scan head to be done.For pain in head , he can take after discussion from your doctor Tab Escitalopram 5mg twice a day for 7 days.This is my personal opinion based on details available here. If you still have any query you may please ask me.Stay Healthy. Dr Anshul Varshney , MD"
},
{
"id": 48547,
"tgt": "Suggest treatment for kidney disease",
"src": "Patient: Am a diabetic, type 2 not on meds, now have start of kidney disease,mostly from meds for B/P like Hydrochlorot, just dropped it, also on Amlod-Benazepril 5-20, 12 yrs. What B/P is safe for me, & can I flush my kidney s or somehow reverse problem? Doctor: Hrlloyou r taking a big risk by not taking treatment,moreover u r a diabetic which impose further risk on your kidney functions"
},
{
"id": 179445,
"tgt": "What causes grey color and thick stool discharge in a child?",
"src": "Patient: My almost two month old son just had a bm and it was a grey color and thick. He filled his diaper up and another one soon after I changed him. He does has gas problems and I have noticed that he has went from having multiple bm s a day to only going 1 or 2 times a day or he will skip a day. Should I be concerned and take him in? Thanks in advance Doctor: Hiwelcome to HCMI understand your concern but 4-6 formed stools or a stool in 2-3 days (unless it is very hard) is normal for a two months baby.Colour of stool does not matter unless it is blood stained or contains blood.If your baby is active , accepting feed well, passing urine every 3-4 hourly , not febrile, you not worry.Passing stool with gas is normal.Hope I answered your queryFeel free to ask any other querythanks"
},
{
"id": 43175,
"tgt": "Went through IVF. Had embryo transfer 2 days ago. Bed resting, pineapple core is good for implantation?",
"src": "Patient: Hi Dr. Samuel,My name is Linda. My husband and I went thru an IVF (ICSI) procedure. I had just had my embryo transfer 2 days ago. We had only 1 embryo with 4 cells (after 2days of insemination). My question is are:1) is bed resting helps the implantation and if yes, how long?2) i heard about pineapple core is which good for the implantation. I did not knw about this till today. Today is my 3rd day of transfer, can I start still take the pineapple core?Thank you in advance. Doctor: HI,1) Bed rest wont help. Keep moderate activity on.2) You can eat pineapple core but not associated with implantation.Regards"
},
{
"id": 83934,
"tgt": "Is blackish or reddish patch due to clonate clobetasol BP?",
"src": "Patient: skin specialist docter has given me an oinment of clonate,clobetasol BP.But after apliying this on targeting area wchish he caused due to sun..a blackish or reddish patch appeared now..so is dis permanant scar created by dis tube..or it ll fed within time Doctor: HiThe spots will reduce in few weeks.Do not worry.Avoid using the steroid cream for more than 5 days.Always use sunscreen before stepping out.Sunscreen should be applied atleast half an hour before going out and reapplied every 3 to 4 hours.Cover face and hands as much as possible.Hope I have answered your query. Let me know if I can assist you further. RegardsDr.Saranya Ramadoss, General and Family Physician"
},
{
"id": 84907,
"tgt": "What are the side effects of trenaxa mf tablets and primosa 1000 oil?",
"src": "Patient: Hi, i have been prescribed trenaxa mf tablets and primosa 1000 oil softgels for painful periods and citing chances of endometriosis... I would like to know side effects of the prescribed medication if there are any and precaution for the situation to not worsen.. Thank You :) Doctor: Hi,Trenaxa MF is a combination medicine used in the treatment of heavy menstrual bleeding and acute pain that occurs with menstrual disorders. Bloating, Headache, Dizziness, Ringing or buzzing in the ears. Weight gain,Shortness of breath Swelling of face, lips, eyelids, tongue, hands and feet are some of the adverse effects.Primosa contains Primrose oil which promotes healthy skin and hormonal balance. It contains linoleic acid that forms a protective coating around nerve fibers. Primrose oil is a rich source of omega-6 essential fatty acids.It is a safe drug.Side effects are less. Kindly take the drug only for the prescribed period to avoid side effects.Hope I have answered your query. Let me know if I can assist you further. Regards, Dr. Saranya Ramadoss, General & Family Physician"
},
{
"id": 180834,
"tgt": "What does swelling in the cheek after taking Amoxicillin indicate?",
"src": "Patient: yes please. my wife went to dentist with a tooth infection on sat was precribed amoxicillin 500 mg 1 pill three times a day for 7 days. although the pain in her tooth has subsided her check has swollen up since yesterday and it is getting larger we are concerened that this may be a reaction to the amoxicillin she ie currently in remission from laynex cancer (three years) and takes alot of meds for that. Doctor: Hi..I can understand your concern..Swelling on face when there is tooth Infection is commonly seen when there is spread of Infection to the soft tissues leading to cellulitis..If there is pus formation below the root tips If the tooth leading to abscess, it can also be a cause..So my suggestion is to get her clinically checked along with x ray of the tooth..If there is Infection causing swelling then higher antibiotics can be advised followed by treatment of the tooth for permanent resolution..If it is due to reaction to antibiotics then antibiotics can be changed..Hope this helps..Regards."
},
{
"id": 172938,
"tgt": "What causes cold with redness around the vulva?",
"src": "Patient: Dear Dr. my two year old has a number of symptoms that I m not sure are related. She has had a cold which presents as a night cough and generally runny nose for the past two weeks. Today she developed soreness/redness around her vulva (we changed nappy brands so it could be that, she has very sensitive skin). I checked her mouth for white spots in case it was thrush and didn t see any but did notice that her tongue is covered in little red dots. Any advice? The soreness is really bothering her, the night cough too, for that we are using a humidifyer. Regards, Eimear. . Doctor: Hi...I feel by what you quote he should be having a - Hand Foot Mouth disease. This is one viral illness among all other exanthemas which can cause fever followed by rash over palms and soles. It is a self-limiting disorder and itching can be really worrisome. I suggest you use any over the counter antihistamine if you have one with you now. You can use Hydroxyzine at 1-2mg/kg/dose (Maximum 10mg) every 6th to 8th hourly for 7 days. This can even cause some peeling of skin in the next 4-6 weeks and do not worry about it.Regards - Dr. Sumanth"
},
{
"id": 71046,
"tgt": "What causes soreness in the chest accompanied by dry cough?",
"src": "Patient: I have a very sore area in my chest in the size of a softball , just slightly left of center. Accompanied by a deep dry cough that makes the pain worse.started with the chest pain13 days which started in sinuses for about 10 days then settled lower. Symptoms much worse reclining. No fever,very tired Doctor: Hello and Welcome to \u2018Ask A Doctor\u2019 service. I have reviewed your query and here is my advice. The soreness in chest with deep dry cough indicate underlying lower respiratory tract infection. Looking towards the symptomatic severity I recommend to get an X-ray chest for further management with your family doctor. Though primary relief can be obtained with inhalation of vapors of boiled water with added tincture Benzoin solution. Regards."
},
{
"id": 183511,
"tgt": "Is the pain at side of face due to tooth infection or ear infection?",
"src": "Patient: Hi I have a pain at the side of my face which I think is a toothache. I have been to a dentist and she took xrays and I need a crown - the tooth is cracked but she felt the nerve was still ok and that I may have an ear infection. I went to my doctor and he said my ear cannal is a little red and I may have a tooth infection. I'm still getting pains - mostly at night and still don't know what the problem is. Should I go back to the dentist or the doctor? Doctor: Thanks for your query, I have gone through your query.The features what you are telling looks like a tooth infection. This kind of pain comes in case of pulpitis. Consult a good oral physician and get it evaluated and diagnosed. If it is infected then get the tooth treated with RCT and get the crown done after that. Mean while you can take a course of antibiotics like amoxicillin 500mg and metronidazole 400mg tid for 5 days (if you are not allergic). I hope my answer will help you, take care."
},
{
"id": 63214,
"tgt": "Suggest treatment for a painful lump on the armpit",
"src": "Patient: I have a lump under my armpit that has been there for years. As of recently it started hurting when I touch it. The lumps have been there for years but are getting bigger within the past couple of days and started hurting a couple days ago. I am about to be on my period if that means anything. I have also felt these same bumps before under my other armpit, on the crest of my arms, and on my Bikini line that are al noticeably there. I know that is where a lot of lynphnodes are but for some reason these ones under my armpit are irritated all of a sudden. What could it be? I m about to schedule an appointment with my doctor. Doctor: Hi,Dear,Thanks for your query to HCM virtual Clinic.I studied your query in all the details put forth by you.I understood your health concerns.But the query needs to be updated with more details,like-What is size/ hard or soft feel of the lumps under the armpit??is it your first period when lumps have got sore?Any associated lump in the breast?What about the lumps along the bikini line in the groin (in the lympnode area)Did you had any recent viral infection?(Infectious mono viral )As the information about these lumps is not sufficient and as I don't have any first hand information of these lumps as told above,I would advise you to consult ER Primary care doctor(PCP) or a Surgeon, who would check it up and would rule out many other causes responsible for such lumps, and would treat accordingly.Cause pf these lumps on the armpit-Still from the data given from you,following causes need to be ruled out-- Infectious mononucleosis viral infection or any viral fever / or Acne with boils may cause such lumps . -IF your period is to be related to these underarm lumps,then whether similar sore lumps are there in the breast(like Fibroadenosis with axillary tail of the breast being affected with such a lump-a benign tumour of breast).Moslty your lumps are of infective nature and need to be confirmed by second opinion from your doctors.To confirm other causes I would hence suggest you to consult ER Surgeon or PCP doctor.Treatment-This would depend on the cause of these lumps,which would be decided by your treating doctor.If medicinal treatment is not useful from PCP, then ER Surgeon would do FNAC biopsy with HPR,/X-ray Chest/CT scan study and would fix the reasons and treat it according to its benign or malignant nature.So till the time you consult ER PCP and or ER Surgeon and get his definitive opinion ,DON'T WORRY AT ALL about it being tumor,but you need to be worried as it would complicate into other problem associated with it.And act fast on the above suggestions,which helps you to plan treatment with your ER Surgeon. Hope this would resolve your query with full satisfaction.Welcome for any further query in this regard to ME by direct question at HCM.Write excellent review and hit thanks, to narrate your experience ,which would improve my rating for many other visitor's to HCM.Wishing you fast recovery.Have a good day.Dr.Savaskar M.N.Senior Surgical Consultant.M.S.Genl-CVTS"
},
{
"id": 73504,
"tgt": "What causes stabbing pain in the chest?",
"src": "Patient: On my right side I have sharp stabbing pains and I am very uncomfortable the pain stays with me alot, I was told I have cholesterol polyps on my gallbladder wall. What should I do it seemed like my dr. isn't concerned. I am just uncomfortable and I get stabbing pains on my r side arae. Doctor: Hello dearWarm welcome to Healthcaremagic.comI have evaluated your query thoroughly .* This seems in relation with inflammation of the gall bladder giving referred pain over lower chest .* Strongly recommended to visit ER .Hope this clears your query .Regards ."
},
{
"id": 168644,
"tgt": "Suggest remedy for fever with headache in children",
"src": "Patient: My two year old started with a fever of 101 in the morning and then started complaining of a severe headache on the right side of her forehead. No other symptoms, but not herself asking for naps, but headache keeping her up. Crying in pain about the headache. Should I take her to the ER or is this premature. Doctor: HiWhile this could just be the flu, with the headache and her not being herself she needs immediate medical evaluation to make sure she doesn't have meningitis"
},
{
"id": 132875,
"tgt": "Suggest treatment for cyst in adrenal gland",
"src": "Patient: Hi I ve been having lower back pain and pain in my abdominal muscles for about three weeks I went to my physician and the CT scan showed that I had a cyst on my adrenal gland the CT said that it was 3 cm I m still in a lot of pain even with the muscle relaxers and pain medicine I m wondering what the likelihood would be the cyst rupturing and what the symptoms of that would be. Are there any treatments available? Doctor: HelloIn general adrenal cysts don't cause any pain and they don't rupture.Usually these cysts are just followed but they can be surgically removed if necessary"
},
{
"id": 196042,
"tgt": "What does this semen analysis report indicate?",
"src": "Patient: I HAVE DONE SEMEN ANALYSIS DETAILS AS BELOW : VOL 2.5 PH 8.0 FRUCTOSE DETECTED , COLOUR -GRAY-WHITE, LIQUIFICATION - 40, MOTILITY - 70, MOTILITY - GRADE 0-NON MOTILE - 30, GRADE1&2 - SLUGGISHLY MOTILE - 30 , GRADE 3&4 - ACTIVELY MOTILE - 40 , VIABILITY L 00 , SPERM COUNT- 80 , ABNORMAL FORMS-15, HEAD DEFECTS-06, GLOBULAR HEAD-03, MICRO HEAD-00, PIN HEAD-00, GIANT HEADS-03, DOUBLE HEADS-00, TAIL-09, SHORT TAIL-02, CURLED-05, BITAIL-02, AGGULTINATES - NOT DETECTED, RED CELL - ND, PUS CELL-ND, EPITHELIAL CELLS-ND, CRYSTALS-ND, AMORPHOUS DEPOIST-ND, BACTERAI-ND.....PLS GUIDE AS DOC SAYS THERE IS SOME MAJOR PROBLEM WHICH LEADS TO INFERTILITY AS MY WIFE HAD MISCARRIAGE 2 TIMES. THNX Doctor: hi dear.welcome to healthcare magic.i have analysed your query.i understand your problem.on seeing your reports,i found that liquefaction time is prolonged in your case.normal value should be 20 minutes.but in your case,it was 40 minutes.it may be one of the causes for your problem.consult with your treating doctor for this problem.thank you.hope i have helped you."
},
{
"id": 11816,
"tgt": "No improvement in pigmentation on using momate s, meleiro cream. What should be done?",
"src": "Patient: I am using momate s cream with meleiro cream with a proportion of 3x1 but my pigmentation again looking same position as before. Last month pigmentation were 80% reduced but this time again in same position. What should I do should I stop the medicine because when I repeat the medicine my skin become more sensitive & redness. Plz tell me. Doctor: Hi , Thanks for writing in. Tanning and pigmentation over face are your problems. Momate S and melrio cream are very strong creams with potent steroid in them.You should have not used these creams in first place.They thin out the skin and may cause permanent damage. Sun protection is most important for any cosmetic treatment. Use good quality sunscreen. Avoid sun exposure. Use brevoxyl face wash. Apply Kojivit cream at night. Visit a dermatologist for some clinic based procedure like chemical peeling and laser. Hope this helps. Regards DrSudarshan MD Dermatology."
},
{
"id": 223796,
"tgt": "Can I pill cause any problem in future?",
"src": "Patient: Top of the noon to you Sir, I am 22 years old. I had intercourse with my girlfriend and i din used protection and i am very much sure that sperms does not came out during intercourse, but still is there any chances that she might get pregnant? And if she take Ipill, is is safe for future for her? Your kind reply will be highly Appreciated. Thanks And Regards Doctor: hello dear.understand your concern.look every contraceptive methods has side effects,some has more some has less.i pill have also some side effect,but you can take it once ,not do such harm.but frequently & more numbers of use can do infertility & pregnancy failure.so i advised to use condoms,diaphragm ,patches,BC pills....or do sex in safe period of cycle like 10 days after cycle & 10 days before next cycle.hope this will help you.best regards.Dr.sagar"
},
{
"id": 54797,
"tgt": "What is the life span with bile duct cancer and hypertension?",
"src": "Patient: hi! my father in law was diagnosed with a bile duct cancer, no metastasis found.he s 71 yrs.old this oct.4, he had hypertension before but now that he is suffering from the so called bile duct cancer he s blood pressure now is 90 - 70, or 110 - 60, what perhaps is his life span... i am worried coz my husband, his son is just 1 month away Doctor: hi.noted history of bile duct cancer and hypertension. when was the last work-ups done? with regards to his life span, it is dependent on the cancer staging and/or the advancement of his disease. it is best if he consult with a doctor, preferably a general surgeon, for physical examination and clinical evaluation. complete metastatic workup must also be done. diagnostics and management (medical and/or surgical if indications are found) will be directed accordingly.hope this helps.good day!!~dr.kaye"
},
{
"id": 29642,
"tgt": "What causes severe itching on the hands?",
"src": "Patient: I occasionally have a break out of severe itching on my hands, so bad that I end up rubbing them so hard that I get popeye swelled hands. I know it s a severe allergic reaction but I don t know from what. I do suffer from hay fever, grass, ragweed and pollen. Can you help me? Trex... Doctor: Hello,You need to get some blood tests done. Kindly meet your doctor and discuss this. You need to get complete blood count to see if there is eosinophilia in blood. Generally, people with hay fever have this. In that case, you will need to take tablet Banocide Forte or Hetrazan or DEC 3 times a day for 15 days. Severe itching can be reduced by taking tablet CPM or Avil or Cetrizine.You need to find out which substances cause allergy to you and thus avoid it.Hope I have answered your query. Let me know if I can assist you further.Regards,Dr. Sanjay Kini"
},
{
"id": 205387,
"tgt": "Suggest treatment for side effects of Zoloft",
"src": "Patient: Hello Dr. I have been having depression and intrusive thoughts for five years now. Tried Zoloft for two weeks and was very ill with it. Stomach cramps etc. I am afraid of side effects of meds like tolerance after a whole and other issues. I am now quite tired of dealing with this condition naturally. What help can you suggest. Doctor: Hi,I can understand your concern about side effects after using Zoloft (Sertraline). Loose motions, stomach cramps, headache, abdominal discomfort, sleep disturbance are common side effects in initial weeks of treatment with Sertraline. Other better alternatives are reducing the dose of Sertraline or swithcing to Tablet Mirtazapine or Tablet Venlafaxine with Clonazepam. Kindly discuss with your treating psychiatrist.Hope I have answered your query. Let me know if I can assist you further. Wish you rapid recovery.Regards,Dr. Chintan Raval"
},
{
"id": 61754,
"tgt": "What causes lump on right side of abdomen?",
"src": "Patient: I have a dime sized lump on the right side of my abdomen. It is about a hands width below my rib cage. It is not painful to the touch or anything like that. It is round and not irregular in shape. It does not stick out from my body. I am a 43 year old female in good health. What could this be? I have a yearly doctor s appointment in 2 months and did not know if it would be okay to wait until then to have it checked. Doctor: hi.based from your description alone, it could be a lipoma (fatty deposit), fibroma (fibrous tissue formation) or a cystic formation (sebaceous or keratinous cyst). these are all benign lesions, but other lesion types must also be ruled-out. it's not an emergency case, but if the lesion is increasing in size, causes pain or discomfort, and is bothersome for you, a consult with your doctor, preferably a general surgeon, is best for clinical evaluation. diagnostics (such as xray or ultrasound) and management (medical and/or surgical, excision biopsy of the lesion) will be directed accordingly.hope this helps.good day!!~dr.kaye"
},
{
"id": 41786,
"tgt": "What is the treatment to increase the sperm count?",
"src": "Patient: Hi i had a sperm analysis and it was found that i had only 5% of normal heads of my sperms and 95% as abnormal. Also the the volume and thickness of the sprems were normal. The sperm content was 120 million. motility was normal . What do i do i have no child to date and what do i do to normalise the heads? Doctor: Hi welcome to healthcaremagic.I have gone through your question.As your semen analysis has normal motility and count, but morphology is not normal.I would advise to do ultrasound scan color Doppler to rule Out varicocele.Also do reports for infections like IgG of chicken pox , mumps as these might damage cells of testes.Consult a urologist he will advise you further.Hope i answered your question.Would be happy to help you further.Take care."
},
{
"id": 77553,
"tgt": "Should I be worried about cough with green phlegm and increased eosinophils?",
"src": "Patient: i am coughing up green rough phlem and my x-ray shows increased vascular brochi lines, my eosinophils is also increased slightly. my Dr told me the was nothing to stress about and he gave me some medication, symptons passed after the medication but now it's back. what could this mean? Doctor: Hi thanks for asking question.Here first possibilies of allergic ashthma is to be rule out if associated breathlessness present with cough.Second it can be allergic acute bonchitis.Thirdly eosinophil get increased in parasitic infection also.filaria or ascaris like worm can lead to hypersensitivity lung reaction.albendazole use for ascaris worm.Hetrazan also useful drug for filaria and allergic condition.For ruling out whether it is obstructive or restrictive lung dz pulmonary function test done.So becoz yr condition has reccured it should be investigated to rule out cause.Finally green sputum can be becoz of pseudomonas infection that also ruled out if no other cause found by sputum culture.I hope my suggestion will you"
},
{
"id": 183254,
"tgt": "Is it safe to fill dental cavities under anesthesia?",
"src": "Patient: My daughter has minor cavities and they want to do work on while she is unconsious/out with the gas,is it safe for such a young girl?...what are the risks?and what questions should i ask to insure my daughter is SAFE? I was reading,and i read that oxygen mixed with another gas is the most recomended,over the one which is not mixed because i believe from the mercury, Doctor: Hello, Read your query as your daughter has minor cavities in teeth this can be done with anaesthesia also you can discuss with your dentist if it is possible to go and if your dentist advised you to do under anaesthesia then it is safe and harmless procedure with no risk dont worry go for treatment .Hope this will help you."
},
{
"id": 188524,
"tgt": "Swelling under ear after extraction of tooth, sinus pressure,feeling of liquid building up with sting under scalp. Help",
"src": "Patient: hi ..sinus preasure..feels like a liquid building fast ..head preasure .and..lightheaded ..then i feel preasure and a sting ..and can feel cold liquid leaking out under the scalp with an odor ..also feel something running down the back of the throat ..back lower tooth removed ..abscessed ..swelling under ear down since tooth pulled ...also preasure down back of neck when tooth came out . Doctor: Hello There, Thanks for your query.As you provided the detail, it seems that there is infection in the space under the area,from where the tooth pulled out.So i would like to suggest you visit an oral surgeon and get the thorough clinical examination done.Mean while you can perform betadine mouthwash gargles and maintain good oral hygiene.Nothing to get worried.Take care.Dr. Gunjan Gupta"
},
{
"id": 109810,
"tgt": "Suggest treatment for lower back pain",
"src": "Patient: hi its been 9 months now and still am suffering from lower back pain. My docotor said its sciatica. Had ones epidural injection after mri scan and my specialist ruled out I have a nerve entrapment .L5/ s1 Have been fallowing exercises as adviced by my physio therapist but till now Pain on my back and left leg is worse when I stand or sit longer than 1ominutes. I try to do lighter work not to stress my back but still the pain is just there and am taking painkillers. I dont feel the pain when I am lying flat on my back. What could be the best exercise so that I will recover and not to go for another epidural.? Doctor: HiThank you for asking HCMI have gone through your query.Your back pain is most likely sciatica due to nerve compression of entrapment.For this i advise my patients with NSAIDs or opiods for pain relief, muscle relaxants like cyclobenzaprine and neurotropics.I send them along with this for physiotherapy and muscle strengthening exercises.Lummbar supporting belt will be helpful for you also.It may take 1 to 6 months for improvement.Some will get early improvement.Even after this you have severe pain and disability then you can think about surgical options like Laminectomy or microdiscectomy or chemonucleation depending upon which suits you.Hope this may help you.Let me know if you have any further queries."
},
{
"id": 28916,
"tgt": "How can an infected surgical wound be treated?",
"src": "Patient: Nearly 2 months ago I had melanoma surgery and the removal/testing of 2 groin glands. Have an infection that lingers and a 3 inch red, swollen gland surgery site. Have used ice faithfully. Would hot paks give better results? (Am finishing a 10-day course of penicillin... an iffy treatment deal because of multiple chemical sensitive to most big drugs. In the past , several pen... Rxs have been refilled with the same nothing effect on infections. Sulfa has been substituted afterwards, but the last 3-day Sulfa drug resulted in infection healing but also the development of a rash. Thanks for your wisdom. Doctor: Hello,Continue with the ice pack. Do not switch to a hot pack. Complete your Penicillin course. If it doesn't resolve, you could get a swab taken from the wound to see what bacteria are growing. A lot of people are allergic to sulfa drugs. You could ask your clinician to prescribe an alternate antibiotic. In the meantime, take a Cetirizine for the rash and you can use calamine lotion as well. Hope I have answered your query. Let me know if I can assist you further.Regards, Dr. Pranav Balakrishnan"
},
{
"id": 45619,
"tgt": "What causes orange urine, blood in saliva and lower abdominal pain?",
"src": "Patient: I went to urinate and was shocked that it was a reddish orange color. when I left the house about four hours ago my urine was very clear. The day before that I was spitting and out came out of my throat bright red. Everyday since then I have had headaches and pain in my lower abdomen. One week before that I tried to do a master cleanse with salt water but later realized that it was regular salt. I Did this for only one day. I drank plenty of water and ate lots of fiber to offset it. I also have small stones which come from not drinking enough water. I am pretty much a health eater. Doctor: Hello, Orange colored urine due to your urine being concentrated which is due to less water intake .If you have no urinary symptoms like burning, increased frequency, etc it can be cured by drinking more water (4-5 liters per day). This will help to keep your urine dilute and faint in color. Hope I have answered your query. Let me know if I can assist you further. Take care Regards, Dr Ivan R. Rommstein, General Surgeon"
},
{
"id": 130840,
"tgt": "Diagnosed with Scoliosis which is causing numbness and pain",
"src": "Patient: Hello, I am 25 and I was diagnosed with scoliosis 10 years ago. It mild and the pain has always been minimal. Lately though the pain seems to be more severe and my right hand seems to go numb and shakes as well. The curve leans towards my right though. Doctor: Hi I have read your details. Normally in Idiopathic scoliosis , since it develops over a period of time , you don't come across neurological involvement either in legs or in arms. If you feel numbness in arms and pain then I suggest you get MRI of neck and back to be able to assess the cause of pain and numbness. You might also have to undergo nerve conduction studies. Take care. Dr Gopal Goel"
},
{
"id": 219606,
"tgt": "Which vitamin should one take during pregnancy?",
"src": "Patient: Hi, may I answer your health queries right now ? Please type your query here...I am 34 years old. I am pregnant for 2 months.slight bleeding had happend, When I went to doctor for check up, he gave me medicine : 1) for folic adic(Folinext) 2) endogest - 200. I have taken endogest for 15 days 2 pill orally. Now again when I went up he continued those two pills.I want to know: - What is the need of the endogest? - How long it should be taken? - is there any side effect of these pills on child me? The doctor made my ultrasound 1 time,and she told for bed rest because my baby bag mouth has opend - how often it should be made in pregnancy ? - is there any side effect of ultrasound on developing child? The doctor didn\u2019t give me any kind of vitamin tablets. -On which months the vitamin should be taken? Which type of vitamin I should take at each month? Doctor: HI.1. Folic acid will have an effect, not a side effect. It will prevent major birth defects falling under the category of neural tube defects, so please do not even think of discontinuing this supplement2. Endogest is a form of progesterone, and it is a pregnancy supporting drug which has been prescribed to you because of the bleeding you had early on. The bleeding means yours could be a threatened pregnancy, and the progesterone in endogest will help prevent that miscarriage from taking place3. No side effect of any ultrasound what so ever on the developing fetus4. Ultrasound is usually conducted at least once every month during pregnancy, but has to be conducted more often during the last trimester5. You can take the folic acid (which is the vitamin/pre-natal supplement) throughout your pregnancyBest wishes."
},
{
"id": 211121,
"tgt": "Does taking ashwagandha brahmi combo cause intense anxiety symptoms during periods?",
"src": "Patient: I've tried ashwaganda brahmi combo and the first week I felt so much better,had more energy ...etc,then when I got my period had very intense anxiety simptoms.Do you think this might be do to those supplements.I started those to help with stress!Thank you Doctor: hithanks for using healthcare magicAshwaganda brahmi is a herbal product. It hardly cause any side effect. In ur case these anxiety symptoms may be due to premenstrual syndrome which happens due to hormonal imbalance. If again have these symptoms, then consult a psychiatrist for further treatment.Thanks"
},
{
"id": 126197,
"tgt": "What causes prolonged right-sided hip pain post cervical fusion surgery?",
"src": "Patient: I am 50 years old. Female. He of 3 neck surgeries. Fusions from c2 thru t 1. All my symptoms have returned and have MRI and Ct scheduled tomorrow. My question now is severe enough pain in my right hip. Has been at least 1 month. Pain runs down the outside of my leg to the outer area of my calf. Thinking arthritis but have not yet had X-ray. Taking hydrocodone prescribed by the neurosurgeon but it does not touch the hip pain Doctor: Hi, The symptoms are more likely to suggest the possibility of spine-related problems. It is better to consult a neurosurgeon and get an MRI scan done for further assessment. As of now, you can try analgesics like Gabapentin which is more effective for similar pain conditions. Hope I have answered your query. Let me know if I can assist you further. Regards, Dr. Shinas Hussain, General & Family Physician"
},
{
"id": 2958,
"tgt": "Does Clofart increase chance of conceiving twins or triplets?",
"src": "Patient: I am trying to conceive form past couple of months. My doctor suggested me to take Clofert 100gm for 5 days starting from the 2nd day of my periods. I have read few post which say that intake of Clofert increase chances of twins / triplets. I not keen in having twins/ triplets. Need your help on this. Thanks! Doctor: Hello and welcome to \u2018Ask A Doctor\u2019 service. I have reviewed your query and here is my advice. Clofart are tablets of Clomiphene citrate. Clomiphene citrate is used for induction of ovulation. These tablets do sometimes cause release of multiple eggs from the ovary which may result in multiple pregnancy, that is twins of triplets. Occasionally, there are many eggs released, which is known as hyperstimulation of the ovary. However, if you need induction of ovulation due to failure of natural ovulation, you should not deny the treatment of Clomiphene citrate (Clofart) for the possibility of multiple pregnancy. You need the eggs to get released. Hope I have answered your query. Let me know if I can assist you further.Regards, Dr. Nishikant Shrotri"
},
{
"id": 77595,
"tgt": "What causes pain in my chest in the heart area?",
"src": "Patient: Im 20 years old and 2 years ago i had my little girl.. i also had severe pre-eclampsia, since then i have sufferd with pain in my chest in the heart area.. every now and then (sevral times a day) i have had an ecg, all came back clear and also tryed tablets to see if it was to do with indegestion, they didnt do anything but mabie made it abit worse. The docters have given up on me abit but im realy worried. Dont know what the next step is. Doctor: Thanks for your question on Health Care Magic. I can understand your concern. No need to worry for heart diseases because your frequent ECG s are normal. In my opinion, your allergy symptoms are related to stress and anxiety. So better to consult psychiatrist and get done counselling sessions. Try to identify stressor in your life and start working on its solution. You may need anxiolytic drugs too. Don't worry, you will be alright. Avoid stress and tension, be relax and calm. Hope I have solved your query. I will be happy to help you further. Wish you good health. Thanks."
},
{
"id": 133487,
"tgt": "What causes sharp twinges near pelvic bone?",
"src": "Patient: Hello, I seem to have a sharp twinges (not really painful) on left side between top of thigh and pelvic bone. It feels like it is right in the artery or vessel - whatever it is that is there. It kind of pulsates when I touch the artery. It is not painful but a twinge sensation. I do a lot of running and I am 65 years old.Don t feel like it is anything but muscular but not sure.Any ideas? Doctor: It might be a piriformis muscle spasm which caused twinges.or might be some vascular prb I am not so sure bcs need to be examined. But if you feel it is muscular prb you search for periformiss muscle streatching and do stretching.apply hot pack."
},
{
"id": 159049,
"tgt": "Black mole on buttock. Is it cancerous?",
"src": "Patient: A small very black mole appeared on my 6 month old s buttocks one day all of a sudden, a few weeks ago. I can t tell if it is getting larger or not. I can t get in to see a local dermatologist for about a month, so I am just searching on the internet to see what kind of mole this might be, and if it is possible it could be cancerous.. Doctor: Dear hovedona, It is simply a normal mole . breathe easy. These pigmented spots do come in life time. keep it under obsevation.I f there is any form of sudden change in its size colour or texture consult your family physician. Truly, Dr. Ticku Jawahar, Oncologist"
},
{
"id": 126746,
"tgt": "What is the first line of treatment for rheumatic arthritis and osteoarthritis?",
"src": "Patient: What is the first line of treatment for rheumatic arthritis and osteoarthritis? I was diagnosis 5 years ago at age 47, positive rheumatoid factor of 20, sed rates have been 15-20. Problem is in the last 8 months joints in fingers , shoulders and knees are flaring more frequent. So is it time for a biologic? Doctor: Hello, You can consult a rheumatologist and get evaluated. As a first line management you can take analgesics like Tramadol for pain relief. In severe cases you have to start on drugs like Methotrexate. Hope I have answered your query. Let me know if I can assist you further. Regards, Dr. Shinas Hussain, General & Family Physician"
},
{
"id": 142794,
"tgt": "What is the cause of memory problems?",
"src": "Patient: problems with train of thoughts hi I'm 44 yrs old I'm weeks from turning 45 , lately I've had some problems with my memory, and my train of thoughts I forget everything, I'm doing things that are just weird example. I took the milk out of the refrigerator and I always shake the milk jug.well today was no different except that I took cap off first then shook it . you can imagine the mess I made, am I going crazy? Doctor: hithanks for using HCM.memory problem can be caused by variety of cause so please get investigation done like routine CBC, blood sugar, serum B12, serum D3, and MRI brain. These symptom also can be due to Depression. So consult a psychiatrist . he can diagnose your problem and can manage accordingly. thanks"
},
{
"id": 105500,
"tgt": "Suffering from sneezing and mucus coming out of it. Noticed blood. What could be the reason?",
"src": "Patient: I have been sneezing for a month or more now. It happens a couple of times a day, a strong sneezing with mucus and liquid coming out of it. I have noticed blood in few cases. I have an ache in my face, yes, all the parts of my forehead ache if I touched it. And these days, I am finding it hard to walk for a few hundred meters. What could be the reason? Thanks in advance. Doctor: Hi, Thanks for using the HCM. You are suffering from sinusitis. Its an inflammatory condition. So kindly consult your physician to get proper antibiotic, anti inflammatory drugs. You can start doing salt water gargle three times a day and steam inhalation twice daily. Hope I have answered your query. If you have any further questions I will be happy to help. Thanks get well soon...."
},
{
"id": 105003,
"tgt": "Coughing due to allergy leading to frequent urination. How to control?",
"src": "Patient: I have allergies lately since working a new job in a concrete commercial building where there is a karate academy and alot of children. Since working there I have had a tickle in throat everyday for about 6 weeks. I also notice after 6 weeks that the coughing is causing me to urinate its constant now and totally embarassing? What can I do? Doctor: Hi, According to your symptoms, you may have allergy to concrete dust in form of allergic rhinitis or asthma. For confirmation , you need to stay away from dust for a period of two-three weeks If you feel better then , diagnosis of occupational allergy is confirmed. Regarding your question regarding urination, cough may lead to stress incontinence. You did not mention about your age. Males above age of 50 years may have benign hyperplasia of prostate which has stress incontinence as one of it's symptoms. You require to consult an urolgist and have USG of KUB region. You may also use Tablet montelukast-levocetrizine tablet once daily at bed time for 2 weeks followed by tab levocetrizine once daily. Hope, I satisfied your queries."
},
{
"id": 94138,
"tgt": "Feeling discomfort and pain in the lower abdomen, back pain. Not appendicitis. What else?",
"src": "Patient: Good day. I have been feeling some sort of discomfort or pain at my lower abdomen , around the belt line for a while now. The pain is on my left which is your right. About 5 years ago, the pain was severe and I went to see a doctor thinking it was appendicitis but it was not. The pain subsided for a while but sometimes comes and goes. The pain is not sharp like appendicitis but just gives me serious discomfort that sitting down sometimes in one place is a little difficult and uncomfortable. My palms would be sweaty sometimes at that point Also I have been having this nagging back pain for a couple of months now. I dont notice in particular how i walk whether i bend a lot or not. I crack my back a lot like am cracking my fingers. Its quite disturbing. Would appreciate your feedback. Doctor: There are several reason for abdominal pain. It is also possible that sometimes this pain can spread to the back. However it is also possible that your pain could also be originating from the back. It is important that a doctor can take a detailed history and examine you to find out. Also investigations such as an abdominal ultrasound and spine x ray may be able to provide more information about the cause. It is advisable that you stop cracking your back as this may put unnecessary strain on the spine and if there is pain may effect it."
},
{
"id": 87931,
"tgt": "What causes severe abdominal pain?",
"src": "Patient: sir, I am facing severe pain near my left abdomen. The pain is like insorting pin from inside belly and it lasts for 10-15 minutes but it is continuing for the last 7 years. it is more painful during winter than any other day.I have made all tests on blood nothing wrong found except vitamin d deficiency. Now my doctor wanted cect upper abdomen. Can you please suggest the cause of pain. Doctor: Hi.Thanks for your query and an elucidate history. Your classical history of '' pain is like inserting pin from inside the belly'' and for the last 7 years is most indicative of a probable slipped disc at the thoracic vertebrae.I would advise you the following:First of all to get a Neurologist's opinion as the pain looks to be more of a Neurological reasons.Go for MRI of the spine;Secondly take an opinion of the Gastroenterologist, this will help in confirming or ruling any local intra-abdominal problem and get a probable diagnosis and plan for a treatment. He shall also ask for some investigations if required."
},
{
"id": 49606,
"tgt": "Underwent Testosterone Therapy, PSA level risen. Would testosterone cause the elevation in PSA level?",
"src": "Patient: My husband's PSA level was l.5 in June 2013. He also passed 2 kidney stones this past summer. The CT showed 4 on the scan. He began taking testosterone replacement therapy 3 months ago because his testosterone level at that time was 157. He was tested again Monday of this week and his testosterone was 190 but his PSA level had risen to 4.1. His doctor has referred him back to his urologist for further testing. Would the testosterone cause the elevation? Doctor: HI thereThanks for your query.Administration of testosterone is known to cause an elevation in PSA. It might be the cause in your husband too.Nevertheless we should do an Ultrasound of the prostate to see if that is contributing to the rise.So I think its appropriate that you consult a urologist.Hope this helpsGood luck."
},
{
"id": 126089,
"tgt": "How to treat severe pain in the left large toe caused due to a bunion?",
"src": "Patient: Thank you: I did not know this was available. I have a bunion on the outside at the base of my left large toe. In the last 24 hours it has become quite swollen which of course generates extreme pain. I plan to see a doctor today, however, any assistance you can give would be greatly appreciated. I have taken anti-inflams with no success. I took, much later a low dose tramadol which has not alleviated the pain at all. I have used ice with not reduction in swelling. Thank you. Doctor: Hi, You can take analgesics like Tramadol or Aceclofenac for pain relief. If symptoms persist, to consult an orthopedician and get evaluated. An MRI scan will be helpful for further assessment. Hope I have answered your query. Let me know if I can assist you further. Regards, Dr. Shinas Hussain, General & Family Physician"
},
{
"id": 172271,
"tgt": "Is Omnicortil Steroid too strong for a 1.4 month baby?",
"src": "Patient: I'm mother of 1.4 months old boy. He has got severe cough, it echoes when he coughs.So doctor had prescribed Omnicortil Steroid. Was this strong medicine required to be given for a little kid? This caused him shivering. Please guide me on what medication to be given and care to be taken during such instances. Doctor: Hi...as you have rightly pointed out Omnacortil is a steroid (Prednisolone). This is not a routinely indicated drug in a nay group as such. For a 1.4 months old baby there should be good reason to give oral steroids like this. Unless the kid is severely breathless requiring hospitalization, then for emergency sake it might be give. But I would not just give it for cough, even though severe as cough is basically a protective reflex and need not be treated as such. Rather we need to ascertain the cause for cough.Regards - Dr. Sumanth"
},
{
"id": 80535,
"tgt": "Can I continue taking Crocin, Erithromycin and Cetrizine for congestion, dry cough, light fever and body pains?",
"src": "Patient: congestion in lungs with dry cough.with light temparature and body pains, it must be due to taking watermelon four days back in this winter season. I am taking crocin, erythromycin and citrizen for the past three days. whether it is ok and to continue. Pl. help me. Doctor: If you hav no symptoms at present then its ok to complete the total treatment for 5-7days.. However if you dont feel any improvement then its better to get done Xray chest and you need to visit your family physician."
},
{
"id": 72997,
"tgt": "Suggest treatment for lung disease",
"src": "Patient: my uncle, whose age is around 45 is suffering from serious lungs disease. As per the specialist doctors from Kolkata he can't be cured since his lungs is drying continuously. He is also suffering from artheritis. We have all the necessary reports and prescriptions but we don't know from whom to have a second opinion..kindly help us out Doctor: you can send me your reports and all medication list, other wise i am not able to understand your problem by you this small complain yyyyyyyyyyyyyy05@yyyyy.com"
},
{
"id": 58096,
"tgt": "Fatigue and occasional diarrhea. Liver function test shows marginally high Bilirubin. Is this serious?",
"src": "Patient: i have been feeling for teh last month weakness but some time i feel better .I checked the CB Blood test that was ok and also chcked Liver function test which wa also ok but slighly high in Biluribin. my eyes now becomes reddish yellow some time specially at night and some tim i have lose motion like once in a week may be becuase of stress pls advise is there any serious issue Doctor: Hello!Thank you for the query.Slightly elevated bilirubin with no other symptoms can be caused by Gilberts syndrome. This issue is caused by some enzymes reduced activity. This condition does not need any treatment. However before such diagnose, some other reasons should be ruled out. Your symptoms may be caused by viral liver infection and gallstones.I suggest you to have checked conjugated and unconjugated bilirubin levels, viral liver tests and abdominal ultrasound.Diarrhea can be also a symptoms of chronic intestinal infection. Please have stool test and stool culture test done.Hope this will help.Regards."
},
{
"id": 63757,
"tgt": "What causes painful lump at back of jawline?",
"src": "Patient: I have been getting my bottom wisdom teeth in for a while now but I have this REALLY painful lump (or it's just swollen back there) on the right side at the very back. It hurts for me to swallow because of it and I notice that right under my chin close to my neck, it's also sore when I touch there. At first I thought it was a canker sore (I think thats what you call them) but now I'm not so certain. Could it be my wisdom tooth coming in wrong or something like that? oh and by the way I'm only 18. Doctor: Hi dear,Welcome to HCM.Understanding your concern.The symptoms you mention you mention in query could be due to few reasons like :-- Due to blocked duct of salivary gland . The duct of salivary gland become blocked due to any foreign body or stone formation , saliva get accumulated and get infected . This is the reason for pain. - Due to infected lymph node . Lymph nodes get infected due to bacterial or viral infection.- Due to dental abscess . When infection or pus get accumulated ate the root tip .It don't seems to happen because of wisdom tooth or canker sore .I would suggest you to consult dentist for proper examination . Doctor may order X-ray and blood test to confirm the diagnosis .Doctor may prescribe antibiotics (amoxicillin) for at least 10 days or perform minor surgery to remove salivary gland .If your wisdom tooth is impacted then you should also get it extracted , so that it will not cause problem in future . Hope your concern has been resolved.Get Well Soon.Best Wishes,Dr. Harry Maheshwari"
},
{
"id": 50696,
"tgt": "Hypodense fluid attenuating cystic. Needs a surgery?",
"src": "Patient: hi i have a hypodense fluid attenuating cystic mass measuring 8.9-6.8cms in the hepatorenal rescessthis cyst has a hu of 9 and is cause indentation in the midpole of the right kidney what should i do ,can it be rectified by medicine or it should be operated immidetaly, and what kind of food should i eat , iam 34 years old and iam a chef Doctor: is blood work done? cyst measures big. but i would suggest you to get it removed as it might push on your kidney and cause more problems"
},
{
"id": 193830,
"tgt": "What causes sever headaches, painful cramps and with blood during ejaculation?",
"src": "Patient: Please help me i dont have a credit card... knew this would happen I will never find whats wrong with me.... pleaase find it in your heart to answer this for me without having to be paid.. Please help i have had this problem for awhile but doctors have not giving me a explanation...Any time I try to have a bm. or Climax I get a horrible painful cramping tensing feeling in my head and i cant see when it does it... when it first begins it feels like a balloon, well this morning i was going to the bathroom and wasn t pushing hard.. my head did the cramping and tensing up well i blacked out and ended up on the floor. and when i checked the toilet there was blood again... when this happens I have to hold my head and apply pressure for it to ease up.. it also makes my nose bleed and side of my face num.... most time when this happens while im having a bm i will think i went and look and its nothing but blood and I was still leaking I went to the doctor for that and still no explanation.. everytime i have a bm its horrible pains i breakout in sweats I shake i go num, and throw up from the pain... I am begging you to please help me find whats wrong with me, i know my body and somethings wrong,, I have 3 great children i wanna be with them for a long time i know people say theres a age when things start falling a part, but please help me.... Doctor: Hello, These symptoms indicate in most cases prostate issues such as prostatitis or hypertrophy and rarely it may be sign of STD infection. You should avoid intercourse for till symptoms persist and drink plenty of fluids. If symptoms persist it would be good to do a prostate ultrasound and check your PSA levels to rule out a prostate tumor or hypertrophy and further treatment depends on these findings. In most cases, it can be controlled with medications but sometimes, in case of a tumor, open surgery is required. And severe benign hypertrophy may be managed endoscopically so you should consult urologist about appropriate options. Hope I have answered your query. Let me know if I can assist you further. Take care Regards, Dr Iven Romic Rommstein, General Surgeon"
},
{
"id": 223859,
"tgt": "Are there any side effects for i pill?",
"src": "Patient: Hi , I have sex with my wife with protection but during sex the condoms got broken . Would it lead to pregnancy . I also want to know she can have I-pill for the same but we are not sure if sperm ejeculates inside her vagina or not.Is having I-pill is dangerous for health or not. Please sir kindly answer these Questions. Thanks Doctor: Hi dear there are high chances for getting her pregnant as condom has teared..if you don't want child then u can give I pill but under consultation of gynaecologist..if she misses her periods by 10 days do the urinary pregnancy test and ultrasound to confirm the pregnancy then it can be treated through medicine also..common side effects are bleeding, pain in abdomen, headache, nausea etc. it can be treated through painkillers such as acetaminophen..hope this will help youthanks and regardsDr.Sohil Takodara"
},
{
"id": 64031,
"tgt": "Suggest treatment for cyst in ovary",
"src": "Patient: I am 22 yrs old before a yr i had acyst in my right ovary after taking a course of medicine it was cleared but now i am having some tiny little lumps inside vagina which sumtimes has itching i daily have a dettol bath and use dettol soap previously these lumps came and after mensuration it dissappears but now a dayz it bcums less in numbers but they dont go permanently my doctor told me to take M2 tone fr 6 mnths so dat i can hav clear mensuration but now if i take take dat tonic i hav lot of pain while periods is dis very serious problem?plz tell me dr. Doctor: Hi,Thanks for the query to HCM.-I studied your query in depth and understood your concerns.-Cause of the cyst in is--In your case it is due mostly because of -the acne of the labia majora of vagina.and would go with time.-Remedy--Treat by ocp to regulate your menstrual cycle.-Gynaec opinion and antibiotics for x 5 day time would resolve these lumps.This would help you to plan treatment with your doctor.Don't worry and things will settleHope this would resolve your query.Welcome further for any query in this regard.Wishing you a fast recovery.Don't forget to Write a good review ASAP.Have a Good Day."
},
{
"id": 7339,
"tgt": "Can i eat fertyl 50 mg on my 4th day ?",
"src": "Patient: my doctor advised me to take fertyl 50mg on my 3rd day but i wasn t able to take it..can i take it today on my 4th day and for how long?? is it still 5 days???thanks a lot Doctor: Hi, Welcome to HealthcareMagic Forum. Yes, if you have missed it, you can take it from 4th day and continue for 5 days, as directed by your Doctor and go for regular follow up. You should be seeing your Doctor Immedately if you experience abnormal vaginal bleeding. Wish you Good Health, Take Care."
},
{
"id": 53285,
"tgt": "What does hepatomegaly on an ultrasound mean?",
"src": "Patient: hello my daughter just got an ultrasound and ithello my daughter just got an ultrasound and it came back with.... Heterogeneous nodular appearance and coarse in echotexture.larger hyperechoic nodule in left lobe of liver measuring 1.9 cm and right lobe measuring 2.1cm. pls refer to hepatomegaly asap. what does this mean? Doctor: Hi and welcome to Healthcaremagic. Thank you for your query. I am Dr. Rommstein, I understand your concerns and I will try to help you as much as I can.It means that there is a liver enlargement and treatment depends on exact cause. Also, suspicious focal liver lesions are seen and these may indicate benign cyst but also malignant tumors. So you should do CT scan as soon as possible. Other causes are various. if this is caused by diet, obesity and diabetes then dietary and lifestyle changes will be beneficial in every case. Alcohol and viral hepatitis are most common causes and in this case medical evaluation is required.Also if there is infectious disease or biliary duct disorders then you need to do some tests to rule out more serious issues which require specific medical treatment.I hope I have answered you query. If you have any further questions you can contact us in every time.Kindly regards. Wish you a good health."
},
{
"id": 187894,
"tgt": "What is the cause and treatment for tooth ache?",
"src": "Patient: Hello,i have a bloc sport worth whole in my top right side f(3rd)tooth,and it makes so much pain the whole night.(most of the time i have pain at night).but my question is i have a pain in my next 2 tooth at the top of the right and bottom left worth red marks arounded at the1 tooth.why i have like this how can i get relief on it? Doctor: Thanks for sharing your concern with Healthcaremagic..!Please note as per your details, we assume that one tooth of your has been grossly decayed as result of the decay the infection will generally reach the dental pulp and causes pain during night. Please consult any dentist and you would be generally referred for RCT(Root Canal Treatment) of the infected tooth / it would be advised for extraction, if the decay has significantly damaged your tooth structure.kind regardsDr Raju"
},
{
"id": 186230,
"tgt": "Is lock jaw fatal?",
"src": "Patient: Hi I have a wisdom come through its grown funny but dentist said it should be ok when it grows more I'm just in a lot of pain with lock jaw I'm 5 months pregnant I'm on penicillin I'm very very worried Becuase I keep reading that lock jaw can cause death is this true do I have anything to worry about ???? Thank you Doctor: Thanks for using Health care magic.Read your query.Yes the tooth will take a little time to come out in correct position. since u are in your fifth month of pregnancy,u can only take pencillin and paracetamol for pain .Don't have to worry about lockjaw causing death,just relax and take the medicines your dentist have prescribed .Once the tooth is in position it will be alright.Continue doing warm saline gargles and keep your oral hygiene good so that the area does not get infected.Also get a X-ray done to know the exact position of the tooth.Hope this was beneficial.Thanks and warm regards."
},
{
"id": 8867,
"tgt": "Is there any medicines to treat dark circles under the eyes ?",
"src": "Patient: hello doctor i m divya i m 24 now i m sufferring from dark circles under my eyes ple help me Doctor: hi well come to HCM Dark circle under eye is of no medical significace.however for cossmetic purpose you may consult dermatologist"
},
{
"id": 189967,
"tgt": "Taking paracetmol and nurophen for painful cracked tooth. Getting pain on side of face. Cure?",
"src": "Patient: hi there i have a cracked tooth at the back of my left hand side in my mouth and its wobbly also a wisdom tooth at the back at the top i have been taking paracetmol and nurophen i have been in pain since friday with it also it has been effecting my ear and the side of my face the tooth that is woobly is rotten. what would you recommend? many thanks stacey Doctor: Infected broken tooth can lead to pain and soreness of cheeks and jaws. Impacted wisdom tooth eruption can also cause pain in the jaws and teeth. Constant cheek irritation by broken tooth can also cause pain and tenderness. If tooth is broken, then you need treatment for your broken tooth. If nerve of the tooth is not exposed, then tooth can be restored with dental filling or crown but if nerve of the tooth is exposed or necrosed, then root canal treatment followed by dental crown is required for the tooth. If tooth is badly mutilated, then post and core procedure is required after root canal therapy. If broken tooth is unrestorable, then tooth extraction followed by replacement of missing tooth with dental implants or dental bridge is required. Only pain meds will be of little help. Hot compresses are not indicated as they will aggravate the swelling. You can do warm saline rinses to reduce the swelling. You need to get treatment done for infected tooth. If you have cost constrains, then you can visit a dental school where treatment is done by dental students under supervision of experienced dentists. You can also look for free dental clinics near you or can talk to your local dentist if you can pay in installments for treatment. By the time you visit your dentist, you can avoid eating from side of broken tooth. You can also apply orajel for some relief. Maintain good oral hygiene to avoid any secondary infections."
},
{
"id": 221082,
"tgt": "What are the early signs and symptoms of pregnancy?",
"src": "Patient: doctor i am 26 years. Married before 1 and Half year back. now me and my husband staying together. last month i have menses on 27 to 31. this month no menses till now. but I have little pain like menses pain from 10- 12 days...it will come 6 to 7 times per day... what is the problem...is I am pregnant Doctor: HiDr. Purushottam welcomes you to HCM virtual clinic!Thanks for consulting at my virtual clinic. I have carefully gone through your case, and I think I have understood your concern. I will try to address your medical concerns and would suggest you the best of the available treatment options.1] If you have missed your periods, get morning's first sample of urine tested.2] In case of any doubts, blood beta HCG and USG will be of help.3]Most common symptoms of pregnancy are- 1.\u00a0\u00a0\u00a0\u00a0\u00a0Missed periods2.\u00a0\u00a0\u00a0\u00a0\u00a0Nausea3.\u00a0\u00a0\u00a0\u00a0\u00a0Vomiting4.\u00a0\u00a0\u00a0\u00a0\u00a0Dry vomits5.\u00a0\u00a0\u00a0\u00a0\u00a0Breast pain6.\u00a0\u00a0\u00a0\u00a0\u00a0Some discharge from nipples7.\u00a0\u00a0\u00a0\u00a0\u00a0Giddiness8.\u00a0\u00a0\u00a0\u00a0\u00a0Loss of appetite9.\u00a0\u00a0\u00a0\u00a0\u00a0Food cravings10.\u00a0\u00a0\u00a0\u00a0\u00a0Frequent urination11.\u00a0\u00a0\u00a0\u00a0\u00a0Increased vaginal discharge12.\u00a0\u00a0\u00a0\u00a0\u00a0Sleepiness You may get any of these symptoms.I hope my answer helps you.Thanks.Wish you great health.Dr Purushottam"
},
{
"id": 190857,
"tgt": "i have a mouth sores with blood clots",
"src": "Patient: i have a mouth sore with blood clots. can i be told whata these are and advis me what to do. Doctor: Usually mouth sores are due to Vit B and C deficiency. It is advisable to load yourself with Vit B complex supplements and Vit C. Application of local ointments like Mucopain would lead to pain relief and healing would be possible with ointments like Hexigel. Try applying these before and after meals"
},
{
"id": 141829,
"tgt": "Will a person with no brain activity recover?",
"src": "Patient: I have a friend that is in the hospital. She is diabetic, she did not have a stroke or anything, but they say there is no brain activity. Could she recover from that? They say it looks like she is trying to wake up, her eyes open every now and then, but basically she is sleep. Doctor: Hello,The chances of meaningful brain recovery are less. Also, it would take a long time. If possible, share her\u00a0MRI\u00a0brain and EEG reports, so that we can give you a more detailed answer.Hope I have answered your query. Let me know if I can assist you further.Regards,\u00a0\u00a0Dr. Sudhir Kumar"
},
{
"id": 180923,
"tgt": "What does swelling in the tongue indicate?",
"src": "Patient: My friend experienced a swollen tongue as she was in bed, she felt like she was being choked. It went away when she sat up. She has blood clot in right thigh and has been on warfarin for as least 6 months. She stresses out easily and has her mother on hospice in her home and she is the caretaker. Nurses and other helpers do come in home occasionally. Doctor: Hi .Thanks for the query..Swelling of tongue that happens suddenly is commonly due to Allergic reaction..Allergy can be due to common causes like any foods or medication, any big bite, environmental pollutants, animal danders etc..So my suggestion is that if the swelling is severe and causing breathing difficulty she should immediately consult an Emergency room and get evaluated..Anti-allergic medication like Levocetrizine, Allegra etc can be advised along with a steroid shot..A blood test can be done to confirm the exact cause of the swelling..Hope this helps..Regards.."
},
{
"id": 198278,
"tgt": "Is continuous masturbation bad for health?",
"src": "Patient: i am vijay,25 varied myself about my habit masturbation. Please tell me is it bad for health? if so what is the side effects? how can i control,i tried a lot,but minimum per 2 time a day i have to do it.please guide me how to stop this,up to what age it may happen?any medicine please... Doctor: DearWe understand your concernsI went through your details. Don't worry. Masturbation is natural, normal and is not harmful for your health. Masturbation in moderation cannot harm your health directly. There are no health related side effects. But if you masturbate more than essential, it may cause tiredness and related stress and anxiety. I suggest masturbation thrice a week.You can engage in physical exercises, outdoor games, education, career, entertainment with friends etc to keep yourself away from masturbation. Become so tired that you cannot masturbate. If you require more of my help in this aspect, please use this URL. http://goo.gl/aYW2pR. Make sure that you include every minute detail possible. Hope this answers your query. Available for further clarifications.Good luck. Take care."
},
{
"id": 38605,
"tgt": "What is the treatment for a cat bite?",
"src": "Patient: Daughter, 10 yrs, got a cat bite. It was not deep but just two superficial marks. It blessed a little and the wound was cleaned with antiseptic liquid. The bleeind stopped immediately. We went to ped and gave first dose on arm. Since she got a little reaction(eyes and lips were white, BP dropped for max a minute), the next four doses were given on thighs as per schedule. Now we got to know that immunoglobin is also given along with first dose but it was not given in her case. Though she has taken the TT booster dose as per the pediatrics schedule. Now the fact of not giving immunoglobin is worrying me especially because the cat is not traceable. Can you advise please? Doctor: HIWell come to HCMIf the patient being vaccinated then just forget about any thing that worries you immunoglobin single short would be enough when there is not indication of any zoonotic infection and condition is on precaution level only, hope this helps."
},
{
"id": 43152,
"tgt": "Diagnosed with PCOS and taking fertyle to induce period. Is it safe to take meprate tablets?",
"src": "Patient: hi.. I have been diagonized with pcos.. doctor advised me to take fertyl tablets for first 10 days of menstrual cycle to induce ovulation. on 13th day my dominant follicle size was 25mm, 25mm.. Doctor told me i would ovulation next day.. on 15th day there were no follicles.. so I was told that I ovulated on 14th day.. then she prescribed to take meprate tablet for 10 days from 18th day.. is it safe to take meprate tablets for 10days? is there any chance to have periods within this days while i m taking meprate? Doctor: HelloWelcome to HCMThank you for writing to us.As mentioned you have developed a follicle and has ruptured on day 14 of your cycle.It would be advised your used miconised progesterone instead of medroxy progesterone (meperate) for better support of the endometrium and if you get pregnant that cycleWish you all the bestHope to have answered your queryDr Nandita Thakkar"
},
{
"id": 130687,
"tgt": "Is surgery needed for knee pain?",
"src": "Patient: This is Negar Aryanpour from Iran,(31 years old, 170 cm, 60 kg).Following my x-ray alignment picture, the left foot has the degree 4.9 and D=433 and for the right foot degree 3.9 and D=430. I should mention that one of my knees has a little pain and after walking for a long time, and sense a pain on my right Pan. Let me know if the surgical correction is necessary now or not? is my age appropriate for doing surgery? Please I'm wondering do the instrument placed in the knee get removed latter on? Or do the patience stay with it for life? Too deep for me...I want to know what if the edge of the bone left space? Or will the bone regenerate in that space . Doctor: Hi,As per your history and age surgery is not the only option. Physiotherapy can help you out in releiving pain with machines (ift and tens) for pain control and graded exercises protocol for strengthening the muscles do visit any physiotherapy clinic they will help you out. Hope you find the answer useful. Let me know if I can assist you further.Regards, Dr. Harsh Swarup"
},
{
"id": 132285,
"tgt": "Why is my right clavicle not fully formed?",
"src": "Patient: I have a right clavicle that never completely formed according to an xray my doctor took. Two pieces there are not connected. There is no scare tissue according to my doctor shown on the x ray. I can move my shoulder and make one of the pieces noticeable under the skin at a point. He suggest that it may have occurred before birth. Can you tell me if this may be caused by vitamin deficiency during a pregnancy? Can you tell me more information about why one of my clavicles may not have fully formed? Doctor: Hi Hope this message finds you in good health.I have gone through your query in detail and understand your concern.its not due to vitamin deficiency but its due to genetic reasons that have occured during embryonic stages of pregnancy.it will cause no harm though..Hope your query has been answeredNothing to worry about, You should eventually get back to normal.Get back to me for any FOLLOW UP QUERIES anytime.Kind Regards,Dr Mahaveer Patil...(MBBS,MS,Mch)"
},
{
"id": 218518,
"tgt": "Is pregnancy possible despite negative HCG report?",
"src": "Patient: My periods are 14 days late. Hcg blood came negative on 28th jan. Dr asked to take primolute n for 5 days. I have completed the course. Earlier i had so many pregnancy symptoms but now nothing except for fatigue, backpain and headaches. I have a very regular 28 day cycle and never have missed a period earlier otherthan my first pregnancy in 2013. Could i still b pregnant? If so will it b dangerous for the fetus? Doctor: Hello and Welcome to \u2018Ask A Doctor\u2019 service. I have reviewed your query and here is my advice. Serum BHCG test is highly specific of pregnancy. If that test turned out to be negative then chances of pregnancy are quite unlikely. Other common cause of delayed periods is hormonal imbalance for which hormonal therapy (Primolut N) is given after confirming negative pregnancy test. Your doctor started you on Primolut N after confirming negative BHCG test.Hope this answers your question. If you have additional questions or follow up questions then please do not hesitate in writing to us. Wishing you good health."
},
{
"id": 19912,
"tgt": "How safe is Sorbitrate and Nikoran for chest pain?",
"src": "Patient: Sir, I am diabetic type II patient since 2000. I am 61 yrs old now. During March 2010 i underwent angio for proximal LCX 100% occlusion. Adhoc PTCA+ stent to LCX. I was put under the following medication :-disprin, clopitab,pantocid,tonact and aten . Again i felt pain/burning /heaviness in my heart since last one month. So this time i consulted another cardiologist and he has added the following tabs ;-sorbitrate and nikoran. iwas taking sorbitrate but its side effect .....severe headach and neck pain started . Again i consulted doctor . he said discontinue sorbitrate and suggested for stent test .Kindly advise .I am unable to bear the pain in my chest . Doctor: Hi ThereAfter going through your query I understand your concern.I would like to tell you that sometimes after angioplasty there can be development of new lesion or blockage that can cause you Angina or chest pain. You have been prescribed with the proper treatment by If sorbitrate and Nikoran are not helping in your chest pain then you should get a Angiography done to evaluate the cause of chest pain. Angiography is the only test that can give you the proper idea of the disease and will help you in treatment .Hopefully this has answered your query.Kind RegardsDr Navneet Bansal"
},
{
"id": 118492,
"tgt": "normocytic normochromic neutrophilia in blood picture, vitamin d :4.98 ng/ml",
"src": "Patient: I have seen normocytic normochromic neutropilia in my blood picture whats this? and I do also have vitamin D deficiency, my vitamin d report shows 4.98 ng/ml. Kindly suggest. Doctor: Hi, I think your neutrophilia might be due to any infection in your body.For your low Vitamin D , you need Vitamin D supplementation either in injectable form or weekly tablets or sachet.Along with Vitamin D take calcium rich diet like dairy products such as milk, cheese and yogurt.Hope it helps."
},
{
"id": 40899,
"tgt": "What causes difficulty in getting pregnant?",
"src": "Patient: Me and my husband have been tring to get pregnant. Its been 24+months now. The doctor said everything is okay but we still havent gotten pregnant. We have all the stuff for at home insemnation including a speculum what do you think we should do on this matter.Also is it normal for Sperm to turn in to clear snot looking balls when it hits air?I know in the research ihave done it sayes that its not enough water but thats all he drinks is water and green tea no soda at all. plz help us thanks Doctor: we need to know the details of semen report. if all the factors are normal, pregnancy is just a matter if time. we xall it unexplained infertility where chance if success is very high when we start actual treatment. pls read- https://YYYYYYYYYY.YYYYYYYYYY.com/post/YYYYYYYYY-YYYYYYY/0000"
},
{
"id": 211654,
"tgt": "Palpitations, anxiety, loss of confidence, excessive sleep, gastric problems. On rabeprazole sodium. Worry about side effects?",
"src": "Patient: I am a 42 year old female. I used to be very courageous and confident but unfortunately this past two months i am getting palpitations and anxiety. Also I fall asleep as soon as I go bed but only for 3 to 4 hours than I wake up at about 2am than thats all I cant go back to sleep no matter what I do .This is affecting my health and interrupting my daily routine . I consuted my family Doctor he prescribed me Clonotril 0.5 mg 1/2 - 1/2 - 1 per day. He also prescribed some sleeping dose but I didnt take it due to fear of addictionj to it and also the side effects. But i took the clonotril as prescribed . I still have sleep problems and some palpitations in the morning. I am also having severe gastric problems . Can you help me and advise me on what medication to follow. I am now taking Rabeprazole sodium for gastric morning and night and clonotril night dose. Kindly advise me I am very worried about side effects. I also want to become my normal self and regain my courage. Doctor: HelloClonotril being a benzodiazepine is a very effective medicine for controlling anxiety symptoms and can be taken for 4-12 weeks but not more than that to avoid developing dependence on it.SSRI antidepressant like Sertraline or Escitalopram are currently the first line medicines for controlling anxiety symptoms however they can cause gastric side effects like nausea, constipation, diarrhea initially as a side effects therefore in your case, you can discuss with your doctor about trying Mirtazapine which is a sedating antidepressant and a good antianxiety medicine too. It can help in sleeping and you won't be needing any other sleeping pill once you start taking Mirtazapine. It does not cause gastrointestinal side effects and can be a good option in your cases in my view. Clonotril can be taken on as and when needed basis if you are much worried about developing dependence on it.Kind regardsVikas"
},
{
"id": 148503,
"tgt": "Electric shock in arm, tiredness",
"src": "Patient: Hi, am 32yrs old abt 130lbs and 5.5ft tall. For the past few weeks i ve bn having this electric shock on my left arm esp. my fingers whenever i exert pressure like carrying things with them.it really feels as if electric current is passing through my hand .i also feel some sort of tiredness on the arm. My email is YYYY@YYYY . Doctor: Dear I am sorry for the health problem you are experiencing.I would recommend you to take contact to a neurologist to undertake two examination, one to exclude a periferic problem with ENG and neck exploration probably with MRI.There is nothing to be worry because any of them can be treated but first of all you have to be right diagnosed.Wish you all the best"
},
{
"id": 216328,
"tgt": "Suggest treatment for chronic pain in wrist and swelling in veins",
"src": "Patient: I HAVE ALOT OF PAIN IN MY WRIST, ALSO MY VEINS AND FLESH ON MY RIGHT ARM HURTS BADLY. MY VEINS ARE PURE BLUE AND A BIT SWOLLEN. YESTERDAY MY PAIN WAS WITH MY THUMB. TODAY I HAVE PAIN IN THE TOP OF MY LEFT HAND WHERE MY VEINS ARE. I BEEN HAVING THIS PAIN FOR ALMOST TWO MONTH.. EVERY MORNING ARM (RIGHT) IS NUMB. FOR ABOUT 3-4 WEEKS STRIGHT HELP Doctor: Ok, there are limits from not having an examination. Veins often look funny. If there is a difference between the arms then it would be more likely to mean something. Venous clots in the arms are kind of rare. The whole arm swells up like a balloon and is quite alarming in these rare situations and there is usually something else going on to cause it (trauma, clotting issues, etc.).Then, lying on some structures irritates them. This is common. Most people have a little of this sometime. Neck problems or wrist problems make it likely to happen to someone all the time. A wrist brace often helps."
},
{
"id": 141619,
"tgt": "Suggest treatment for burning sensation in the lower leg",
"src": "Patient: I have a couple bulging disc s in my back, causes pain around my waistline and the pain radiates to my left leg at the ankle area. the pain in the lower leg is a aching burning sensation causing me to lie down and put my leg on a pillow to get relief which takes at least 30 min. to help. would back surgery also help the leg pain. I cannot get my housework done or walk for any length of time Doctor: Hello and Welcome to \u2018Ask A Doctor\u2019 service. I have reviewed your query and here is my advice. You are suffering from pain due to compression of some nerve between your bulging (prolapsed) disc. As you have been told you may require operation of back (lumbar spine). But that would be decided as per your MRI scan of lumbosacral spine. So you need to undergo an MRI. Meanwhile you should take analgesics like Tramado and Tab. Gabapentin. I hope I have answered you query. If you have any further questions you can contact us in every time."
},
{
"id": 128999,
"tgt": "What causes tingling sensation in the leg?",
"src": "Patient: My question is I have had pain off and on for three years, at times quite severe. I still do my part job and house and yard work at age 81, BUT I m sick of the pain. It is in buttock and into hip area. At times runs into groin muscle. Often I have pain on outer thigh area and sometimes in shin area. Sometimes it goes into tingling in front of leg and arch of foot. One side only. I wonder if it might be periformis syndrome? I am seeing a chiropractor which keeps me going, but not a cure. PT here is hard to get into as over busy. I had acupuncture which did take care of it for a number of months, BUT can t afford that out of pocket expense anymore. What can I do?? Doctor: Hello,Thank you for using Healthcaremagic.I read your question and understood your concern.I suggets you get a MRI of lower lumbar spine as this seems a problem of that region ( discal hernia or degeneration) and see a neurologist.Dr. Selmani"
},
{
"id": 13561,
"tgt": "What causes itchy rashes and hives?",
"src": "Patient: Every 3-4 days, a raised, itchy rash will appear somewhere on my 12 year-old s body. Never in the same place. First it was on buttocks, then hands, forearms another day, shins a few days later. Yesterday it appeared on his wrists, knees and cheeks. It is alleviated in a matter of minutes after administering separate dosages of prednisolone, Zyrtec and Zantac. The hives first appeared May 10 after a day spent in the woods with friends. He may have rubbed a Bounce dryer sheet on his body to deter insects. But the rash will not completely go away and we are almost out of the prescription. Any thoughts? Doctor: Hi, The ideal management of hives is to find the cause for the hives. Hives (urticaria) is usually caused by food, drug or environmental changes. Antihistamine and steroid can give a temporary relief. The ideal treatment is to avoid yourself from the causative agent. I request you to consult your Dermatologist who will help you in managing the condition. Hope I have answered your query. Let me know if I can assist you further. Regards, Dr. Siva Subramanian, Dermatologist"
},
{
"id": 126451,
"tgt": "What causes pain below the thumb?",
"src": "Patient: I am a 70 year old male and am having pain in the area just below my thumbs on either hand. Is this arthritis, and if so, what type of arthritis might it be? My doctor gave me an Rx for Voltaren cream but it really doesn t take the pain away. The pain is constant and it is getting harder to use my thumbs for gripping something. Are there any treatments that I might be able to try to correct the problem or to relieve the pain? Doctor: Hello, It could be an arthritis possibly rheumatic arthritis affecting small joints. We have to rule out other conditions like gout and contusion also. Better to consult an orthopaedician and get evaluated. As of now you can continue your pain medicines as usual. Hope I have answered your query. Let me know if I can assist you further. Regards, Dr. Shinas Hussain, General & Family Physician"
},
{
"id": 129389,
"tgt": "Should i be concerned about the sore knee after a fall?",
"src": "Patient: Hi I was painting my bathroom and did not realize that i was not balanced correctly on the small ladder and it tipped to the right and i fell off landing on my feet the wall broke my fall. now my leg is real sore below my knee. should i be concerned? Doctor: Hello,I think you should be concerned as you may have many kinds of problems starting from meniscus tear, ligament tears or any fracture. I suggest you to go and see an orthopedic doctor and make x-rays and/or MRI.Hope I have answered your query. Let me know if I can assist you further.Regards,Dr. Edvin Selmani"
},
{
"id": 165693,
"tgt": "What causes diarrhea and vomiting without fever?",
"src": "Patient: My daughter has had diarrhea and forceful vomiting (10-15 times with each episode) 4 times in the last 3 weeks. The symptoms last a couple of hours first thing early in the morning then she is fine the rest of the day and several days after. There is no fever or blood in the stool/vomit that accompanies this. Doctor: Good afternoon..!!Sudden diarrhea and vomitings lasting only in the mornings for few hours with symptom free periods inbetween might indicate a behavioural problem / eating disorder in your child.So it's better to visit your doctor for further evaluation and management.Thank you, have a nice day..!!!"
},
{
"id": 134119,
"tgt": "What could cause numbness in tailbone and butt cheeks?",
"src": "Patient: HI I went to the hospital on 30 th of may siting up blood ILL try to make it short the next morning I had 486; respiratory failure they put a tube down my throat for 3 days and I was in there until the 9th of may after I got released I came home my tailbone is completely numb butt cheeks numb toes are tingeling I don t know how long I was found by a nurse that happen to look in on me what causeing my numbness thank you can you help me my name is joe Doctor: hi,as you mentioned you are having tingling and numbness from the tail bone to the toe, I will advice to take physical therapy as the use of therapeutic ultrasound therapy and TENS therapy for the symptoms. with the grace of God I wish you a speedy recovery."
},
{
"id": 51791,
"tgt": "Is my urine result OK?",
"src": "Patient: Hi, may urine result RBC-O-2 and pus cells 0-1, I already taken cifrofloxaxin. last friday my RBC- is 2-4 and my pus cells 10-12...is this alarming tnx.. Doctor: U have two urine test results here !0 -12 pus cells signifies infection of the urinary tract and u have received the correct antbiotics for it, hence ur pus cells have reduced in number. 2-4 rbc during UTI is common Have a healthy living"
},
{
"id": 32800,
"tgt": "Suggest treatment for fever and chest congestion",
"src": "Patient: I have hadwhat i believe are flu-like symptoms for the past few days which include fever 100-102 which lasted for bout day and a half then non stop congestion for bout 3 days now and most importantly altho ive felt overall better i still feel weak and tired. my boyfriend caught something called upper respiratory infection and i stayed over his place to help. we did kiss b/c he was feeling better. do i have mono? Doctor: Hello. Thanks for asking Healthcare Magic. Infectious mononucleosis (Mono) is an infectious, widespread viral disease most commonly caused by the Epstein\u2013Barr virus (EBV), one type of herpes virus. There are less chances that you have mono because symptoms are thought to appear around 4\u20136 weeks after initial infection. It can be a bacterial infecction. i would suggest to start with antibiotics and paracetamol for fever. Take rest and have fruits and plenty of fluids. Hope this helps. Thanks and Regards"
},
{
"id": 127720,
"tgt": "Having noticed pain at the back part of the pelvis towards back bone",
"src": "Patient: HelloI started getting pain in the back part of my pelvis - and my skin right above it was red. Then the pain and the redness moved over to my backbone. The redness and pain are rising upward on my backbone. I've had this for approximately one week. Can you tell me what the cause may be. Thank you. Doctor: Hello,The causes of pain over back part of pelvis are:1. Sprain2. Overuse3. Injury4. Blunt trauma5. Pneumonia6. Cholecystitis7. Other lung infectionsFor that, you can take tablet Brufen. Apply diclofenac gel or cream. Also, apply ice locally and do x- ray chest.Treatment mainly depends upon underlying conditions. Kindly follow with detailed history or consult with orthopedician.Hope I have answered your query. Let me know if I can assist you further.Regards,Dr. Shyam B. Kale"
},
{
"id": 186037,
"tgt": "Experiencing pain & trismus on the left jaw",
"src": "Patient: i had a toothache on my 2nd and 3rd molar on the right.when i visited my dentist and she gave me antibiotics, but 2days after taking the antibiotics- the original pain went away but now im experiencing trismus and the pain is on the left jaw, i cant even insert a thin cracker in my mouth. what should i do? Doctor: Thanks for using Health care Magic.Read your query.I would advise you to continue the medication,also do warm saline gargles.Revisit the dentist and get a checkup done (maybe on the left side your third molar will be having a problem which is causing the trismus and pain).Get the needful treatment done .Thanks and regards."
},
{
"id": 22227,
"tgt": "What causes chest pains with dizziness?",
"src": "Patient: i am an 18 year old male, i have been getting chest pains for a while now. they come and go but when they do i have trouble breathing, my heart hurts and i get really dizzy and light headed and feel distant. i get cramps in my arms and my fingers tingle when this happens and occasionaly i also get a sharp pain under my ribs when i breath in and out. Doctor: Hi,This is Dr Sameer, cardiologist.Pain in chest can have a lot of causes. Chest pain due to heart disease is non localized, squeezing type & increase on exertion. Still you need to do a ECG & 2D Echo for cardiac evaluation.Chest pain at your age can be simple muscle pain or costochondritis(inflammation of joint between bone & cartilage of chest wall). Both are treated with pain killer. Tab Ultraset will be safe & good for you. Still get a X Ray done to rule out any chest abnormalities.Thanks"
},
{
"id": 52358,
"tgt": "What causes abdominal pain while breathing?",
"src": "Patient: I am 52 years old ; I have gallstones I had a attacker\u2019s 3days ago , the next day after the Attack I was back to normal , starting yesterday my gall bladder feels sore , when I move it feels as if it is being squished and hurts when I take a deep breath Thanks David Doctor: Hi, You need to investigate with ultrasonography abdomen to rule out cholecystitis. The symptomatic gall bladder needs to be removed. You can complete the course of antibiotic. If repeated attach occurs then you need cholecystectomy operation. Hope I have answered your query. Let me know if I can assist you further. Regards, Dr. Parth Goswami, General & Family Physician"
},
{
"id": 165350,
"tgt": "What is the black oval spot under the right foot of a baby?",
"src": "Patient: My 22 months girl has a black oval spot under her right foot. Just notice it 2 weeks ago. I am sure It is not a mole. I assumed it was a dry blood under skin from a pinch or something like that. After 2 weeks it haven t change at all, I am worry now. Can this be someting else? Doctor: Hi...Thank you for consulting in Health Care magic. Skin conditions are best diagnosed only after seeing directly. I suggest you to upload photographs of the same on this website, so that I can guide you scientifically. Hope my answer was helpful for you. I am happy to help any time. Further clarifications and consultations on Health care magic are welcome. If you do not have any clarifications, you can close the discussion and rate the answer. Wish your kid good health.Dr. Sumanth MBBS., DCH., DNB (Paed).,"
},
{
"id": 14936,
"tgt": "Should I continue HPV vaccination 2nd dose after getting a rash on the torso and being doagnosed with pityriasis rosea after the 1st dose?",
"src": "Patient: HPV vaccine and Pityriasis rosea 8 years ago after giving the birth I was diagnosed with cervical abnormalities and tested positive for HPV. I have undergone a laser treatment. However, recently I have had again a positive test for HPV and my gynecologist insisted that I should have HPV4 vaccine. He thinks that recurrence of HPV it is due to my low immunity and that having a vaccine is the only way how I can get some HPV antibodies. About 4 weeks ago I had my first shot, thought I initially felt ok, now I have developed a rash over my torso as well as cheeks!! and was diagnosed with Pityriasis rosea. I don t know what to do about the second shot. Should I wait until this rash is cleared (it could be up to 8-12 weeks)? Or should I go ahead and risk more problems or just not to continue with this vaccine? Doctor: Hello,Thanks for the query,P roseae is a response to any stress.Its not a severe condition,You can continue with the vaccinePlease meet a dermatologist for exact diagnosis.Let me know if you have any other doubt.you can ask a direct question to me on this forum, following the below link.https://urldefense.com/v3/__http://www.healthcaremagic.com/doctors/dr-rahul-kumar/64818Wishing__;!!Mih3wA!SBzm6_kI6hCZ58EPH6N_05MFfiPbxWXT0a2TJCdFQObRWm5mV5ur7hUOMa8clQ$ you a good health.Thank you"
},
{
"id": 129260,
"tgt": "What is the reason for achy joints and muscles on right side?",
"src": "Patient: My joints are achy on the right side of my boddy, from my ankle to knee to hip, to back to shoulder. I have been overly tired and my muscles ache, again this is only on my right side, the left side feels completly normally. I have recently had an xray on my right knee this is all clear. Doctor: Hi,For exact diagnosis to be made get serum uric acid levels done and send me the report.Till then have cap.indomethacin25mgs thrice a day by prescription of your family doctor.Thanks,Dr.CHANDER MOHAN SINGH."
},
{
"id": 31423,
"tgt": "What could cyst on face with oozing puss suggest?",
"src": "Patient: i have had a cyst on the left hand side of my face for years now which has never caused me any pain but every now and then it starts excreting pus and the smell is so vile and dirty it makes ;me sick then all of a sudden it stops and never bothers me for months again.. what is this? Doctor: Hi thanks for asking question.Here it seems to be bacterial infection mostly whenever you get affected like from that of staph.aureus.When this boil or furuncle like lesion appear you can apply local antibiotic cream over it.Oral antibiotic like amoxiclav can be useful if severe case with fever.After healing of that lesion take care not for recurrence.Check your sugar level, because diabetes can lead to recurrent bacterial infection.Drink more water.Avoid excess fatty and oily food.Green leafy vegetables and fruits are helpful.If still no improvement then we will think further.I hope you will understand my concern.thanks"
},
{
"id": 67441,
"tgt": "What causes bluish lump on the arm with pain?",
"src": "Patient: Okay I have a lump it s not very big.. I looked at it an it s like bluish black! And I was squeezing it before I actually looked at it and it sticks out a bit.. And I kept like squeezing it but no puss or anything like that comes out just blood.. And it hurts when pressure is applied. It s squishy to and it s in my skin. Like when you like grab it in the skin. And you squeeze it sometimes it feels like when you pop a zit. But in my arm in the lump. Doctor: hi,dear friend,thanks for sharing your problemas you detailed here, you may have infected sebaceous cyst or developing abcess but it need more detail from you. abcess at developing stage causes imflammation and pain and after few days pus may form.so first you consult your family physician and start primary treatment.thank you for sharing queries.please share for any other queries.get well soon dear friend."
},
{
"id": 134926,
"tgt": "What does Tear with small right posterolateral disc prolapse at L2-3 level suggest ?",
"src": "Patient: i make the MRI on my back the report conclusion ; - spondylodegenerative changes in term of loss of normal T2 - annular tear with small right posterolateral disc prolapse at L2-3 level indenting the thecal sac and the related nerve root what is mean ? Doctor: hiit means your leg nerve is compressed in the spine and you will get pain in back and legu need to undergo nerve decompression thanks Take care"
},
{
"id": 67540,
"tgt": "What causes painful lump near hairline left of spine?",
"src": "Patient: My almost 6 yr old daughter has a lump near her hairline just to the left of her spine. It s very noticeable to the touch. Over the past few years she will get shooting pains in that exact location. We noticed this today because she said she felt pain when tilting her head to one side when she was in class. But when I touch it she feels no pain. She also has a bumpy, itchy rash on the back of her head. Not sure if this is related but thought I would mention it. Doctor: Welcome to health care magic. 1.History of painless lump at the hairline, most possible cause could be congenital cause, Cyst. 2.The next possible cause could be lipoma - fat content. Lastly hair follicular infection - but generally seen with pain.3.In this case i would recommend ultrasound examination of the lump - that will help to evaluate the nature of the lesion - cystic, solid, mixed. Its source and extensions. Vascularity status.4.After that if there is any involvement of spine an MRI might be necessary - as it will help to evaluate if there is any spinal involvement.5.I would recommend to o see your GP, let it get examined and you will be sent to scan and further evaluation.Good luck.Hope i have answered your query,any thing to ask ? do not hesitate to ask.http://doctor.healthcaremagic.com/doctors/dr-ganesh/62888"
},
{
"id": 119289,
"tgt": "Joint pain, all tests normal except hematology test. What kind of blood disorder is it?",
"src": "Patient: My mom suffers from an unknown terrible pain in joints and bones. She is 52 and 150Cms high.The pain increases noticeably during sleep. Sometimes she feels the pain in specific spots, her back, neck, backbone, shoulder ,... . She has done every kind of blood tests by now. The results are normal except in Hematology test. ESR 1st hr: 25 ESR 2nd Hr: 45 I.K (Katz index): 24 Is there any serious problem? I want to know what kind of blood disorder she may have. Doctor: Hi sara_naznazi_9 A raised ESR is only indicative of an inflammation in the body, even a mild infection in the body in the body can give rise to raised ESR , however for your mother at the age of 52 it is well within the normal range . I would like to know what other tests were done for your mother . Does she experience any burning or tingling sensations in her arms , forearms or fingers . does she experience weakness in her hands did she do any strenuous work in the last few days Is she a diabetic I suggest you to get an X-ray of her spine done to determine any problem with the inter vertebral disc (spinal cord) in which case she will have to avoid lifting heavy weights . you will have to consult a neurologist for further management."
},
{
"id": 1688,
"tgt": "Can i get pregnant after being treated for uterine infection?",
"src": "Patient: I gave birth to my second child 22 months ago, shortly after I had a staph aureus infection which grew a heavy culture in the cervix. I took two courses of antibiotics which cleared up the infection. Can I try for a baby again - would the infection have caused any damage to the cervix? Could the infection come back? Doctor: Hi, I think the infection is not permanent. It goes away with the antibiotics. So, you can plan a second baby. There will not be any difficulties in that. Hope I have answered your question. Regards Dr khushboo"
},
{
"id": 65996,
"tgt": "What causes lumps on the eyebrow with bruises?",
"src": "Patient: Woke up yesterday mornin with a hard swollen lump right above my eyebrow. It narrows as it goes up my fore head. It feels bruised. It does not move if u try to push it. It is visibly swollen. No disocoloration. Nothing resembling a pimple or bite mark. Should I worry or c if it goes away on its own? What could be the cause? Doctor: Hi, dearI have gone through your question. I can understand your concern. You may have some inflammatory swelling or dermoid cyst. You should wait for few days. If it does not go away then you should go for fine needle aspiration cytology or biopsy to search the cause. Then you should take treatment accordingly. Hope I have answered your question, if you have doubt then I will be happy to answer. Thanks for using health care magic. Wish you a very good health."
},
{
"id": 187484,
"tgt": "Suffering from severe sinusitis after dental treatment. Having numbness and jaw pain after having augmentin",
"src": "Patient: I had and still suffer from severe sinusitis after dental treatment. I had upper teeth pain that was really bad. after a week now I finished a course of augmentin 1g every 12 hours. Now I am suffering lower teeth/jaw pain and numbness. is this due my sinus problem. Doctor: Hello, Welcome Thanks for consulting HCM, I have gone through your query, as you have mentioned that you have toothache and painful jaw , pain can be due to Carious tooth or Periodontal problem , dont worry it is not due to sinus problem , it can be due to toothache for this you should consult dentist Go to dentist and go for investigations IOPA x-xay if carious tooth go for Restoration or Root canal treatment , if pain is due to Periodontal problem go for Scaling and root planning Do warm saline gargle two - three times a day Hope this will help you. Wishing you good healthRegards Dr. Priyanka tiwari"
},
{
"id": 164798,
"tgt": "What causes presence of pus cells in stool in infant?",
"src": "Patient: Hi ! My 5 month girl has pus cell in stool 6to9/hpf is there any thing to worry..she had fever 102 F one weeek back and was passing soft stool 2to 3 times a day...now fever is gone..but stool is soft...septran charged for 5 days ...folcovit kid is given once a day.... ! Doctor: This is suggestive of stool infection. If the symptoms have subsided then there is no need to worry. Otherwise antibiotic needs to be changed based on stool culture report."
},
{
"id": 109260,
"tgt": "What causes pain in left side of back?",
"src": "Patient: i have a problem. i get pains in the left side of my back but i am having to be very careful what i eat, if i eat a normal meal sometimes i get really bad pain in the left lest side of my chest and the pain in my back moves up higher. it lasts about 12 hours. sometimes i am really hungry but afraid to eat .whats wrong with me I'm loosing weight. Doctor: HiThank you for asking HCM. I have gone through your query. Problems like peptic ulcers can give radiating pain towards back like you mentioned. Even colitis can give such symptoms. A physical examination is necessary and in case of gastric signs a consultation with a gastroenterologist is well appreciated. Fibrogastroduodenoscopy might be needed to rule out peptic ulcer in case of signs of that. Proton pump inhibitors like rabeperazole , mucosal protectors like sucralfate and diet will be helpful for you in such case. Hope this may help you. Let me know if you have any further queries. Thanks."
},
{
"id": 37815,
"tgt": "How to prevent infection in Gallbladder?",
"src": "Patient: I JUST HAD MY GALLBLADDER REMOVED YESTERDAY. I HAD GALLBLADDER STONES AND IT WAS VERY INFECTED. I AM DIABETIC, SO I DONT WANT TO DO ANYTHING THAT WOULD CAUSE PANCEATITIS OR ANY PROBLEM TO OTHER ORGANS, SUCH AS MY KIDNEYS OR LIVER. WHAT CAN I DO TO PREVENT THAT? AND WHY WOULD IT CAUSE OTHER ORGANS TO BECOME DAMAGED? Doctor: Hello,Welcome to HCMThanks for posting your query in HCM.I understand your concerns.Stones in the gall bladder is formed due to various reasons like stasis of bile in the gall bladder for long lime or any hemolytic disorders.As you have already got your gall bladder removed,there is no need of worrying about damage to other organs.It wont have any effect on kidneys or liver or pancreas.But as you are diabetic,you need to control your blood sugar levels within normal limits.Diabetes itself can cause damage to various organs including kidneys.So, i would advice you to strictly stick on to the diabetic diet and follow.Hope you are happy with the answer.Thank you"
},
{
"id": 82376,
"tgt": "Suggest medication for persistent cough",
"src": "Patient: My 5 years old son has been coughing on and off for a about 2 days now. Im at work and my husband just called to inform me that my son keeps coughing. Yesterday I started giving my son cough medicine that the dr prescript last time he got sick which was about 2-3 months. I am not sure if I should continue with the medicine or take him to urgent care since his dr office is closed til Monday. Doctor: Thanks for your question on HCM.In my opinion you should not worry much as your son is having dry cough only.So continue the same cough medicines and consult doctor on tuesday.But during this time if your son develops 1 fever2 expectoration3 high respiratory rate4 not taking well orally, crying alot etcThan take him to the ER and get done chest x ray.Otherwise upper respiratory tract infection seems more. So give him plenty of fluids anf cough medicines."
},
{
"id": 177119,
"tgt": "What causes runny stools, tiredness and high fever in an infant?",
"src": "Patient: my daughter is 7 months old and shes been the last 3 days not eating much runny stool hardly super tired but doesnt sleep long she cries all threw the night and prior to this she slept all night shes been crying when she eats shes been running fevers but not super high until tonight its 104. something she was born with 3 vsds only one closed should i be concerned its her heart? Doctor: the child appears to be having acute gastroenteritis. she is running high grade fever, so a sepsis screen should be done to rule out any systemic infection. excessive crying in AGE can be due to rashes in the peri-anal region due to loose stools. apply some local application like Siloderm cream for the same. give her a course of oral antibiotic like syp Cefixime or Cefalexin(sporidex)."
},
{
"id": 84382,
"tgt": "Will taking penicillin causes pain and weakness in the arm?",
"src": "Patient: Hello, I am taking penicillin VK 500MG Tabl and I am noticing that my right arm is very weak and it it painful to move the arm. Should I stop taking this prescription? I was prescribed this because I had a small abcess under one of my teeth. Thank you in advance. Doctor: Hi,Yes, although it is rare but penicillin can cause pain and muscle weakness. Based on the history you seem to have hyperkalemia (increased potassium level) apparently caused by using penicillin VK-500 tablets. I advise you to stop taking this drug and consult your dental surgeon for an alternate effective but a safer antibiotic apart from undergoing drainage of the abscess.Hope I have answered your question. Let me know if I can assist you further. Regards, Dr. Mohammed Taher Ali, General & Family Physician"
},
{
"id": 224350,
"tgt": "Can break in the contraceptive pill cause pregnancy?",
"src": "Patient: have been changed from Microgynon 30 to Loestrin 20, i was given the new pill during my 7day break and i was told that i could carry on taking my new pill after 7 days as normal ... Will I be protected against pregnancy straight away? On the leaflet it said I should have carried on taking my new pill without a break but doesn't say whether I need a second contraceptive if i did not do this ... any information would be great full Doctor: Hi,The instruction is meant for the continuation of the medication while you are on it after each cycle. Since you have started afresh, your doctor may have suggested the standard method. Either way, during the first couple of cycles of the pill, you would need additional protection as ovulation is not effectively inhibited sometimes. You can rely on the oral contraceptive pill alone later on, but please make sure you take the pill regularly at the same time everyday. Hope this helps."
},
{
"id": 39920,
"tgt": "Will the readministration of the same medicine for treating UTI be effective?",
"src": "Patient: i had may medication to treat uti infection with ciprofloxacin,500mg 2xaday for 7 days...repeat urinalysis but still blood+1leuco+2pus cell5-10..diagnoze again for untreated uti..and bilateral caliectasis..i wonder they gave me again the same med for the same infection ciprofloxacin..would it work if i take again the same meds for the second time Doctor: Hi,welcome to healthcaremagic forum.It depends on the interval between the two attacks.If the current UTI is just the continuation of the previous one(ie.persistent symptoms with treatment),it is better to do urine culture and sensitivity test before continuing the same medicine.If your previous UTI responded to ciprofloxacin,and now you got another attack after few months,there is nothing wrong in trying the same drug.I hope you will find this information useful.Thank you."
},
{
"id": 217712,
"tgt": "Suggest remedy for neck pain",
"src": "Patient: I've been working in front of PC since more than 11 years .. that's why i have a neck pain especially when i had to forward my neck towards the screen.I made an MRI photo and discovered that in addition to the infection and stress in the Muscles and nerves of my neck there is a disc problem in my C3, C4 and a little bit C5 i would like to ask if it's safe for me to make a treatment session using mesotherapy that injects some substances into my neck muscles ..Best regards,Naim Bazzi Doctor: Dear Sir, In physiotherapy IFT and ICT is best treatment for your conditions.which is reduce your pain and relief from stiffness. Kindly go with ergonomics training for your working related problems. Exercises are permanent solution for your conditions. Kindly start with free neck exercises and go with neck strengthening exercises."
},
{
"id": 53884,
"tgt": "What does relative increase in renal parenchymal echogenicity mean?",
"src": "Patient: Relative increase in renal parenchymal echogenicity which may relate to normal variance (40%) or early, non specific medical renal disease (60%). Please correlate. Low density (uric acid, oxalate, xanthine or matrix) calculi both kidneys, non obstructing at present. What does this mean? Doctor: Hi and welcome to Healthcaremagic. Thank you for your query. I am Dr. Rommstein, I understand your concerns and I will try to help you as much as I can.This may indicate some interstitial kidney disorder and if there are symptoms and signs of kidneys failure, you should od biopsy to confirm possible diagnosis. ALso, CT scan would be advisable for better visualization of kidney parenchyma.I hope I have answered you query. If you have any further questions you can contact us in every time.Kindly regards. Wish you a good health.DR. Ivan Rommstein"
},
{
"id": 194714,
"tgt": "What causes a red rash on the sternum?",
"src": "Patient: I am 59 yrs. old, weigh 142lbs and 5'4. I had my left kidney removed3 yrs. ago because of infections. I had apartial hysterectomy in \"89\".I had Basil Cell Carcinoma removed from my face 4 yrs. ago. Myon-going 6 month problem is a small round red rash on my sternumchest. I was told before it my be Lymphoma but now after seeinghow medical cremes effected it I have been told it isn't cancer anda biopsy has been refused. The round red rash changes in sizeand color daily if not hourly. I have no fever, lumps or weight loss,or itching.When I have applied some cremes such as \" Protopic\" it has caused swelling. The amount of time 6 -months I have had thisrash has me concerned and of course even the slightest possibilitythat it my be a blood cancer such as Lymphoma is frightening. Doctor: Hello, You have to go for a biopsy to confirm the diagnosis. We can not make a diagnosis without biopsy. Consult a general surgeon and he will direct you accordingly. Hope I have answered your query. Let me know if I can assist you further. Regards, Dr. Shinas Hussain, General & Family Physician"
},
{
"id": 99364,
"tgt": "Suggest treatment for itching in neck",
"src": "Patient: I am 26.My name is Sudha.I have ring worm like skin itching in my neck only on one side for the past 1 month.some times it troubles me by itching.yesterday night i found my face having some allergy results small pimble like in my face but there is no itching...Please give me some suggestions...Thanks Doctor: As you have mentioned in your history it is clear that you are suffering from some sort of fungal infection. For this first of all you have to take some antibiotic preferably cap doxycycline 100 mg twice daily after meals for at least 5 days along with tab levocetrizine or tab fexofenadine twice daily after meals for itching with tab rantac or cap omez twice daily empty stomach. ?You can also take some anti-fungal medicine like tab fole 150 mg once daily in night for at-least 5 days. In addition to all you have to use clobit-gm or valnerate-c cream for local application. Last but not the least, avoid scratching of local part, apply vaselline or coconut oil as it will be quite helpful in reducing the severity of itching."
},
{
"id": 47268,
"tgt": "Suggest diet to decrease oxalates level in body",
"src": "Patient: which foods can i eat that contains Oxalobacter formigenes? I have oxalate kidney stone and need some help controlling my oxalate intake but if i decide to cheat on my low oxalate diet i need to know what i can eat to help decrease some of the oxalates. Doctor: hai,Dietary oxalates are usually restricted to 50 milligrams per day Since intake of dietary oxalate accounts for only 10-15% of the oxalate that is found in the urine of individuals which form calcium oxalate stones, many researchers believe that dietary restriction cannot significantly reduce risk of stone formation. However i will list the Oxalate containing substances.avoid or reduce the intake of the followings which contain oxalates in dense level.Fruits blackberries, blueberries, raspberries, strawberries, currants, kiwifruit, concord (purple) grapes, figs, tangerines, and plums Vegetables spinach, beets (root part), beet greens (leaf part), collards, okra, parsley, leeks and quinoa,. celery, green beans, rutabagas, and summer squash Nuts and seeds-almonds, cashews, and peanuts Legumes- soybeans, tofu and other soy products cocoa, chocolate, and black tearegardsDr.S.Senthilnathan"
},
{
"id": 11958,
"tgt": "Why there is no relief from pigmentation which is very itchy even after having medications post pregnancy ?",
"src": "Patient: Hi, I have had 5 years of fertility and Ivf treatments . I eventually fell pregnant with twins who are now 1. In the last 12 months of fertility, I found a pigmentation change on both sides of my face on the jaw area. Then while pregnant I was very itchy and frequently drew blood through scratching on the arms. Now, the pigmentation is on my forehead as well and my upper chest, arms and top of my back are very itchy again as well as blotchy and motled. I am Taking the pills again now which is the only. Change in the last 6 months since it has intensified. It is driving me crazy! Doctor: Hi...dear Megiohns.., Thanks for choosing HCM., Pigmentation changes are quite common in Pregnancy..., Bcoz it is due to hyperactivity of hormones..., So.., after delivery only U will get relief..,ok Meanwhile.... 1) Use SUNSCREEN LOTION morning times..., 2) Banatan (curatio) Night times..., after delivery your problem will solve...ok.., good luck"
},
{
"id": 150820,
"tgt": "Brain MRI without contrast shows few nonspecific punctate foci, recommended CSF. Second opinion required ?",
"src": "Patient: Brain MRI without contrast came back w few nonspecific punctate foci of T2 prolongation and left temporal encephalomalacia along the lateral cortex. Radiologist recommended CSF . Saw Neuro for the 1st time and he walked in and within 2 minutes stated that he didn t think i needed the CSF. Ordered EEG thinking i may be having mini seizures- tech stated that it appears normal. Have not seen Neuro yet. Tech closed the door before EEG and said off the records, i think you should get a second opinion ?? She stated that i should see Neuro that specializes in MS ?? Doctor: Hello Thanks for the query, First of all i would like to tell you that the interpretation of MRI should be done only in a given clinical context., has to be correlated with the clinical data. In the absence of clinical problem of a patient it is difficult to say if you need CSF or not. I am sorry that i don't know your age and if you have high BP, diabetes and cholesterol problem. You need not see a consultant who specializes MS but you need c,clinical evaluation by a neurologist. If you are not happy, better go for a second opinion. The clinical interview will clarify so many things and we will be able to guide you better. Best wishes"
},
{
"id": 151930,
"tgt": "How to know you are getting recovery from anti tuberculosis ?",
"src": "Patient: Dear Sir , My mother age 55 yrs, is suffering from TB of spine and has abscess from L5 to s1 she ie undergoing treatment of antituberculosis from last 2 mths but she has still pain in her lower limbs how can we know that she is recovering as somties she has severe pain somtimes no pain Doctor: Hi, Thanks for query, Your mother has cold abscess. After complete treatment she will be alright. Do follow up and blood report and Xray frequently will guide her recovery. ok and bye."
},
{
"id": 53127,
"tgt": "Suggest risk with water retention in the liver in elderly",
"src": "Patient: my grandmother 75 has suffering from liver problem she has large amount of water in liver. The problem has been from last few 5 months. Doctors of bgh has told that her situation is critical and have no good scope of treatment for this problem. Plz guide me its quite urgent. Doctor: Hi dear,The problem your mother is having is called ascitis,Ascites, is uasually a late manifestation of cirrhosis of the liver,pathophysiologic mechanisms of ascites include the role of inadequate renal prostaglandin production in the development of the hepatorenal syndrome and the possible role of nitric oxide in the pathogenesis of the renal complications of cirrhosis. The aim of medical therapy is to achieve a negative sodium balance and, consequently, fluid loss. Large-volume paracentesis is safe and effective in the management of tense ascites, but use of diuretic agents should be continued to prevent reaccumulation of ascites. Liver transplantation, transjugular intrahepatic portosystemic shunts, or LeVeen shunts should be considered in selected patients with persistent ascites.Please share her detailed reports and investigations for further opinion."
},
{
"id": 187088,
"tgt": "Is there any other option to save my tooth as filling fell off and gum swelling?",
"src": "Patient: I have no idea what's going on with my tooth. About 5 years ago I had a full in and barley my gums under it started swelling and a few years later my fill in fell off. I've gone to the dentist they said they can't do anything about it but just to pull it off. Is there anything to do to save it? I really don't want an implant :( please help me. Doctor: Hi. Welcome to Healthcaremagic.I read your query. Since your dentist checked that tooth personally, tooth must have past the point of restoration. Ask your dentist if they can restore it with post and core. Or i suggest it would be good to get it removed to avoid further infection.If you dont wanna go for an implant, you can go for a bridge, also called as Fixed partial denture.So go to your dentist, get your tooth radiographed and decide.Hope the answer helps you. Thank you!"
},
{
"id": 128523,
"tgt": "Suggest treatment for muscle cramps",
"src": "Patient: I have muscle cramps all over my body. From my jaw line to my toes. The worst are in my feet and legs at night. I m now getting really bad ones in my hands. Also, body cramps below my breasts and around into my back. And both hips. When sitting too long I m getting unusual cramps that seem to start in the back of my hips and travel down the who leg. Doctor: hi,this is most probably potassium and magnesium deficiency , eat dome banana and drink plenty of water.. also get calcium potassium,mg levels tested"
},
{
"id": 3707,
"tgt": "How to increase chances of getting pregnant?",
"src": "Patient: Dear Doctor, I have been married for 3 yrs but did not conceive yet. I am undergoing treatment. My doctor suggested to take injection (Ovigin 5000 ) on my 18 day after my cycle. And today is 28th day where again i got date. Please let me know do i get pregnecy very soon .. i am very much worried. Doctor: HIWell come to HCMI really appreciate your concern, pregnancy is more natural phenomenon than physiological one and it depends upon some compatibilities like presence of ovum and sperm at the time of coitus and right proportion of hormones if these are absents then conception may not be possible, along with the medicine try have the coitus on the day of ovulation, hope this is information helps, take care and have a nice day."
},
{
"id": 225753,
"tgt": "Taking depo, have pregnancy symptoms. Am I pregnant?",
"src": "Patient: Ok so I've been on birth control 4 4years n 1year I'm been off it... I started again on the 3month depo n 2months on it my fiance came in me twice.. Its been 3weeks and I feel the same way I felt with my first pregnancy but somehow I'm experiencing these symptoms way early,am I pregnant? I have all the symptoms... Please help Doctor: hello,thank you for using healthcare magic,well with depo its unlikely that you are pregnant, sometimes with depo such symptoms might occur so no need to worry,if you want you can take home pregnancy test or more confirmatory test like beta hcg level of blood or ultrasound it will give you absolute result..hope this helps you..DR.NEHAL.."
},
{
"id": 164963,
"tgt": "What causes white spots on the back and arm?",
"src": "Patient: My son of five years old has white spots on the skin. Especially on his back and arm. What s the cause of this spots? I noticed them this morning. He had a very difficult period this two months. He used antibiotics twice on March 2017 once for mycoplasma and once for throat infection.Thanks Doctor: Hello and welcome to healthcare magic.Your history points towards two possible conditions.1.is condition from family of eczema (allergic condition and reactions of skin) known as pitriyasis alba.This may be due to antibiotisc or any other triggering factor.Treated by topical steroids (that suppress immune reactions) moisturising and avoidance of trigger2.is a condition called tinea versicolor.It's caused by fungal infection of skin.It is treated by antifungals.The conditions are differentiated by dermatologist.They will examine your son's skin scraping under microscope if fungi are visible then this is tinea alba if not then its pitriyasis albaHope you find this answer satisfactory.Good luck"
},
{
"id": 90975,
"tgt": "What causes stomach pain and vomiting?",
"src": "Patient: my son three days ago had his tensills removed .igave him amoxicilin and pain medicine . he has terrible stomack pain and i stopped his antibiotic also he started vomitting very hard, also he doesnt drink fluids. he is 11 years old , 175 cm tall and weight 80 kg. help me Doctor: Hi.Thanks for your query and an elucidate history.As it is just 3 days post-tonsillectomy, he should be admitted for observation, intravenous fluids and antibiotics, observation , preparation for unseen emergency and all . The stomach pain and vomiting can be due to irritation, unable to accept something which you might have given orally such as a particular thing to drink or eat/ viral infection and so on. Vomiting in this recovery phase is very bad as the stomach contents spoil the healing tonsillar fossae. Can cause infection, bleeding and so on.Insist on admission to the hospital."
},
{
"id": 51629,
"tgt": "What causes lumps and bruising around the fistula after dialysis ? How can they be treated ?",
"src": "Patient: after dialysis i have a large lump and bruise around my fistula should I worry after dialysis i have a large hard sore lump and bruise around my fistula should I worry? Doctor: hi u should not worry. its due to irritation . apply good antibiotic cream with mild steroid like hydrocortisone. it will go slowly. if itching is there take tab cetrizine at night"
},
{
"id": 110048,
"tgt": "What causes lower back pain, left leg pain and excessive sweating?",
"src": "Patient: I am a 38 yr old femake, 5'5\" about 160 lb. I have been suffering with horrible lower back pain in the L4, L5 region. I have had multiple steriodial injections that have not worked. I also suffer from left leg pain down to my foot and sharp pains that radiate up my back. As strange as it sounds, since all this began I have suffered with horrible sweating. Not just annoying sweating but change my clothing, sheets and pillows at night sweating. I am at a lose of what to do. I can't seem to find any relief and the doctors can't seem to help me either. What are your thoughts? Doctor: Hi,Welcome to healthcare magic.After going through your query I think You are suffering from chronic backache. Reasons of back pain can be weak muscles and bone and disc prolapse and.Treatment of back pain is exercises, neurotropics such as mecobalamine once DAILY and analgesics (DICLOFENAC SLOW RELEASE TABLETS) . Sometimes vitamin D deficiency is the cause so get your vitamin D checked .If it is low then vitamin D supplementation( weekly with milk) can be taken.Eat milk, fruits and green leafy vegetables daily.To further investigate MRI of the region is advised.You can discuss with your treating surgeon. I think your query answer."
},
{
"id": 73213,
"tgt": "Suggest treatment for pulmonary tuberculosis",
"src": "Patient: i am RMO in a reputed hospital. AKT (combutol,r-cinex,pyzina regimen) started for a 78 yrs male patient who was found to have extensive PT.Chest physician advised combutol-400, r-cinex 300+450, mgs at night and pyzina in after noon. Is this regimen correct? Doctor: Thanks for your question on Healthcare Magic.I can understand your concern. No, this is not appropriate regimen.This patient is receiving under dosage of ethambutol. Also the timing of drugs are not proper. Ideally, all anti tubercular drugs should be taken together in the morning on empty stomach. If patient is feeling nausea and vomiting then he can take drugs after breakfast. But all drugs should be taken together. Dose of ethambutol should be 600-800mg.Dose of pyrazinamide should be 750-1000 mgs.Hope I have solved your query. I will be happy to help you further. Wishing good health to your patient. Thanks."
},
{
"id": 100526,
"tgt": "Suggest treatment for constant dry cough",
"src": "Patient: Hello Sir, I have dry cough problem since 1.5 months i consult many doctors but not diagnose very well and still suffer from this problem.I got cough after eating any meal and it continous whole day when i talk or take walk outside also so is there any serious problem Doctor: Hello.Thank you for asking at HCM.I went through your history and would like to know more about you like - What is your age? Do you have nay complaint other than cough? Do you have regurgitation of foods/liquid after a heavy/spicy meal? Do you have breathing difficulties? etc.However, from your current history, I would like to make suggestions for you as follows:1. Were I treating you, I would prescribe you montelukast, levocetirizine and an antacid like omeprazole regularly for 2 weeks. If there is improvement, I would continue the antacid for 4-6 weeks.2. I would suggest you regular warm saline gargles.3. I would also suggest you to avoid exposure to dusts, smokes and air pollution as much as possible.4. A light, nutritious diet and especially a light evening meal will also help you in an early recovery.If above medicines do not improve your symptoms, I would think of some other more serious causes and would suggest you investigations like complete blood counts, ESR and chest X-ray.Hope above suggestions will be helpful to you.Should you have any further query, please feel free to ask at HCM.Wish you the best of the health.Thank you & Regards."
},
{
"id": 9841,
"tgt": "Suggest treatment to control hair fall",
"src": "Patient: i am suffering from hair loss problem since i have moved to bangalore from delhi.. i am maintaining proper and balanced food diet and does also not have any additional mental tension.. i have realized that my hair roots have become thinner.. any suggestions?? Doctor: Hi, I would recommend you to apply Minoxidil 5% lotion on the affected areas of the scalp twice daily and take hair supplements like tablet Follihair A once daily. You need to continue these for 4 to 5 months to see significant results. Hope I have answered your query. Let me know if I can assist you further. Regards, Dr. Asmeet Kaur Sawhney, Dermatologist"
},
{
"id": 189380,
"tgt": "Have tooth decay, turned black, outer layer decayed visible inner layer, have yellow teeth",
"src": "Patient: Hi Doctor, i am 32 year old female and facing problem of tooth decaying. its happening since last 15 year and now my four teeths decayed alot and outer layer of tooth is completly decayed and inner line is visible. i dont have any pain in these teeths and one of the teeth start becoming black. my teeths color is yellow, please advice on aall this issues Doctor: Hi, Hello and welcome to HCM, Since it is clinically visible that the teeth are already decayed visit the dentist once. Since you are not having pain based on Xray findings, if the decay is not deep enough restorations ie., fillings of the teeth are enough. But if the decay is extending deep close to or involving pulp ,they might require root canal treatment. The yellowish colour of teeth might be due to calculus ie.,deposits on tooth surface or due to fluorosis. The treatment for this is get a Scaling done once ,and if the yellow colour still persists better get a bleaching done or veneering. Hope so this will help you, Thank you."
},
{
"id": 213782,
"tgt": "How can mania be treated ?",
"src": "Patient: 6 years ago we thought my uncle has menia so we consult in sir ganga ram hospital but now his condition become worst his maind as well as body stops working condition become worst day by day pls help us Doctor: dear rini going thrugh ur short note,the only thing which is understandable is that ur uncle is having a major psychiatric disorder.being a psychiatrist i wud suggest that u immediately take ur uncle to a psychiatrist for the correct diagnosis and starting of immediate treatment before its too late.for further queries,visit-www.bestpsychiatristindelhi.blogspot.com"
},
{
"id": 203160,
"tgt": "What is the remedy for the difficult or no ejaculation?",
"src": "Patient: My wife and I having been trying to conceive for the last 7 years without success. I am currently taking Staminogro. In 2012 I had a varicousectomy, and my wife has had a few bouts of endeometreosis. We have also had a few IUI treatments. Sometimes it happens that I find it difficult or even impossible to ejaculate. What can I do to overcome this condition. RegardsAnthony Harvey Doctor: Thank you for asking anthony harvey! Provided history indicates both partners compromised and it needs an expert fertility consultant with advanced recent fertility techniques to impregnate your partner.So seek the help of a fertility consultant and let them help you. Seek a sex therapist for erectile dysfunctions and ejaculatory disorders.Take care"
},
{
"id": 129183,
"tgt": "Suggest medication for pain due to herniated disc",
"src": "Patient: Hi, I feel off a garage ladder while working for Verizon I lost my case due to an incompentant lawyer supplied by my union, I also lost my appeal an my job, which I then hired a private lawyer to sue verizon . Mean while I need an operation on my knee plus I have a hernated disc which has become orhtritic also I have pain in my other knee due to a boating accident an neck an shoulder pain, since I am broke an I am on medicare which only cover 80% I can t offord the operation now, also I live in Rockland County an most pain mangement doctors don t except medicare an when I do find a dotor they prescribe tramadal or oxicodone 5mm. the only medication that works is oxicodone 30mm.. do you have any suggestions Doctor: a) Rest: Rest and Anti-inflammatory and analgesics.Physical Therapy Management in PIVDb) Reduction: Continue bed rest and traction for 2 weeks may reduce the herniation in over 90% cases.c) Chemonucleolysis: dissolution of the Nucleus Pulposus by percutaneous injection of a proteolytic enzyme (chymopapain)Operative Management-Disc removal 1.open2.microdisectomy"
},
{
"id": 127544,
"tgt": "Can Fintor cause vomiting?",
"src": "Patient: im on a fintor had to cut back to evry second day feet and ankles swelled with a red rash bl isters on full strength with the cut back to every second day rash stil there and swelling gone down a lot Ihave beenon this drug for a month but the last few days cant keep any thing down even ginger ale comes up after a hour or two Doctor: Hello and Welcome to \u2018Ask A Doctor\u2019 service. I have reviewed your query and here is my advice. What is fintor is it a drug or device etc???It maybe reactionary rash please consult your physician immediately he will examine and treat you accordingly.Take care"
},
{
"id": 80410,
"tgt": "What causes persistent fever in child with pneumonia?",
"src": "Patient: My daughter, age 7, has had a fever over 101 since Sunday 2/22. She was diagonsed with pnumonia on 2/24 and started antibiotics right away. Her fever and cough have persisted. I would think 7 days into the RX she would be back to OK...Thoughts on what we should do? Doctor: Hello dear, thanks for your question on HCM. I can understand your situation and problem. Treatment of pneumonia should be guided according to the causative organism. Since she is still having cough and fever even after 2 days of antibiotics, better to get done sputum culture and sensitivity. Sputum culture will find the causative organism and sensitivity will guide you for antibiotics. So by this we can prevent haphazard use of antibiotics. Because haphazard use can delay the recovery and increase chances of drug resistance. So better to consult pulmonologist and get done sputum culture and sensitivity to start appropriate antibiotics."
},
{
"id": 204784,
"tgt": "How can severe stress and depression be treated?",
"src": "Patient: I am on 20 mg of escitalopram. I have had an extremely stressful 2017 with many deaths in my family. My daughter is now engaged to an abusive, controlling, drug user and dealer and has our 6 year old son in this situation. We raised him for the first 6 years of his life and now that she feels she has a man to help her raise him, she has taken him and the fianc\u00e9 refuses to let us see or talk with him. I am devastated an do not know how I can ever be happy again. I worry about him constantly, I cannot eat or sleep. My husband and I are both in counseling and I know from that, I have no control over the situation, but it does not make my emotions any easier. I try to not go in public, mainly because it is so hard for me to have people ask me about my Grandson. I go to my Doctor tomorrow and would like to be able to help with the decision on what to do with my medicine. Do you have any suggestions. Doctor: hi madampresently, your still in depression. your sleep disturbances, loss of appetite, feeling low, wanted to be alone all indicate that your still continuing in depression and your not responding to tab escitalopram even with 20mg. but, you didn't mention from how many days your using that.if your using above dose for a trial of three months, it indicates that the medicine is not working for you and should shift to another antidepressant class along with cognitive behavior therapy. I also recommend you to investigate for thyroid profile. Also advise you to go for regular physical exercise."
},
{
"id": 43981,
"tgt": "Unable to conceive, irregular periods, weight gain. Prescribed susten, sefstart, ovacare. Cure?",
"src": "Patient: i am 24 yrs old. 2 yrs back i have been married . till now i have not concieved . we consulted a doctor and the test reports says that i have small fists in my womb. so my doctor recommended me some medicines like Susten vt 100, Sefstart, duphaston 10mg and ovacare tablets.previously i use to be 75 to 80kgs of weight now my weight is 103kg after marriage increased suddenly. also i had irregularity of perids. Doctor: Hello madam, Well the principal problem might be a disorder in ovulation due to obesity as a result of which also you are getting irregular periods. Please do a thyroid profile and a fasting plasma insulin and please loose weight . Well susten or duphaston are pregnancy supportive hormones that might be taken in the second half of menses if you expect timely ovulation only ( in case of normal monthly cycles). If your cycles are delayed, ALONE it might not help on a pregnancy a though you may get normal cycles due to withdrawal effect of progesterone.The rest two are OK. good luck"
},
{
"id": 132461,
"tgt": "Can smacking the middle and ring finger knuckles be related to internal injury?",
"src": "Patient: I smacked the middle and ring finger knuckles of my hand yesterday when my other hand dropped my curling iron. I didn t get burned but those knuckles and down to the middle of the back of my hand is very swollen. I get a snapping sensation that brings tears to my eyes when I move that hand certain ways. Is it just a tendon moving in a sore area or could I have broken a bone in there? Doctor: Hi Hope this message finds u in good health.I have gone through ur msg and understand your concern.it may be a soft tissue inflammation or a trigger finger/ get an xray done Get back to me for any FOLLOW UP QUERIES anytime.Take care,God bless.Please click \u201cTHANKS\u201d"
},
{
"id": 179021,
"tgt": "What causes bleeding of nose quite often?",
"src": "Patient: Hi Doctor, Myself Sophia, My son is 5.5 yrs and he gets bleeding from nose often like every 15 to 20 days, and when it starts bleeding it takes a lot of time to stop. I consulted a paediatrician she said to consult an ENT specialist. He has got fever and feeling very tired too. Whereas he was never like that before as he is very active boy and never sits in a place but from 3 - 4 days he says that he s tired and will take rest. Due to this I wanted to know whether it is a serious problem or it is common now a days. Actually i m really very scared. Doctor: Nose bleeding in this age group is mainly due to nose pricking. once a vessel gets injured while pricking the vessel becomes fragile and prone for rebleeding. I think at present your child is suffering from viral infection which is causing the fever and tiredness. It also causes stuffy nose which makes the child involuntarily to put his finger in nose causing repeated bleed."
},
{
"id": 213478,
"tgt": "Hallucinations, incoherent speech, not aware of surroundings. Taking zolfresh, arip medication. Treatment options?",
"src": "Patient: Hi.. Need answer to my question very very urgently...My father in law (appox age, 67 yrs) was put under medication 10 days back by a psychiatrist in XXXXX. He is having Arip (20 mg) and Zolfresh (10 mg) daily. for the past couple of days, he has been having hallucinations and talks incoherently and I feel that he is not aware of his surroundings. Today while sitting on the sofa, he urinated and passed stool . He is talking in a very abrupt manner. If asked a particular question, he answers something completely different. Please advice as soon as possible. I am really worried... Please help.. Doctor: HI neha2000. THANKS FOR YOUR QUESTION. FORTUNATELY I AM DOING MD IN PSYCHIATRY NOWADAYS. FROM THE MEDICATION IT SEEMS THAT YOUR FATHER IS A PSYCHOTIC PATIENT. FOR LAT FEW DAYS HE BECOME VIOLENT,AGITATED,INCOHERENT TALK CONFUSED,INCREASED HALLUCINATION THIS TYPE OF CLINICAL FEATURE IS SEEN IN DELIRIUM AND AGGRAVATION OF HIS PSYCHOTIC SYMPTOMS.AS DELIRIUM ARE MORE COMMON ,LIFE KILLING CONDITION SO WE TAKE THIS CASE AS DELIRIUM UNTIL PROVE OTHERWISE.SO MY OPINION EITHER STOP OR DECREASE THE DOSE OF ARIP TO 2.5-5 MG,STOP ZOLFRESH AS IT HAVE SIDE EFFECT OF DELIRIUM AND ADD SMALL AMOUNT OF LORAZEPAM IM/SOS. BUT FIRST GO TO YOUR PSYCHIATRIST TELL ALL THE THINGS AND ACCORDING TO HIS SUGGESTION YOU PROCEED. THANK YOU DR SOURAV GANGULY"
},
{
"id": 140925,
"tgt": "Need treatment for difficulty in reading and memory problems with a history of Hep C",
"src": "Patient: Have HCV 30+ yrs. Female/57 vision-problems reading and deciphering what I am looking at - Having difficulty reading and typing simple tasks at work. Memory key issue for tasks that are repetitive. Have BS degree= Administrative /Secreterial work for many yrs. Become very nervous, depressed. No family , financial or obvious issues or problems- Just HEP C (from hospital transfusion1980). No smoking, drugs or drinking Is this the Hep C. Need help , but what kind? I cant function at my job anymore. Doctor: Hi, You will benefit for a formal Neuropsychological consultation which could also be preceded by a visit to a behavioral neurologist in order to see if you may be exhibiting signs and symptoms of cognitive dysfunction secondary to your history of HCV or any of its treatments (if you've received such). There may be other rather simple metabolic abnormalities that should be checked as well as serum Vit. B12, D, B1, and a good thyroid profile. Hope I have answered your query. Let me know if I can assist you further. Regards, Dr. Dariush Saghafi, Neurologist"
},
{
"id": 73168,
"tgt": "What causes left sided chest pain?",
"src": "Patient: Hi i am a male 5'10 185 lbs. and exercise regularly. I smoked weed for fifteen years but pretty muched stopped since sept. of 2010. I have smoked since maybe once a week but will not anymore. Anyway i have been having these chest pains that lasts for seconds and come and go only when i am relaxing like watching t.v. oron the computer for five months now..When i go to the gym and run and lift weights i feel great, like really great. But when i am at home or say at the grocery store i get these chest pains that hurt real bad. It feels like a horse kicks me in the chest or sometimes like a real bad burning feeling. It is located on my left side on the center and sometimes under my left arm on my chest. It does not spread through my whole chest. Had numerous Ekg's done, blood work, chest x-rays and a treadmill stress test. All came back (-) except an rbbb during my stress test. Dr. said not to worry that i did excellent on my stress test. What can this be? It only lasts for seconds and comes and goes. Some days i do not get it at all. Other days i feel like it is waiting to happen during the day and it will happen a few times. I go back for a holter moniter in a few days and then an echo the next day. Is this something serious ? Why do i feel so good when i work out and exercise? I did have a lot of stress last year but this happens when i do not even stress about anything. Please Help? I only stress when i get these pains also. Thanks. Doctor: Hello dear , hiWelcome to Healthcaremagic.comI have evaluated your query thoroughly .* This is more in relation with subcouncious mind theoray of perception of different sensations in various states , usually it is not related to heart and other chest organs .* Sounds very difficult to read , but it is true and documented in medical sciences .* You need relaxation techniques , deep breathing exercises , concentration techniques , YOGA to recover fully without medicines .Hope this will help you for sure .Regards ."
},
{
"id": 145602,
"tgt": "What causes numbness in forehead and both forearms with elevated sugar levels?",
"src": "Patient: I started having numbness in my forehead and both forearms about 9-10 weeks ago. I now have some numbness in both feet around my toes ( feels like walking on cotton) Now I have slight numbness on my tongue. Had bloodwork done. Blood sugar was a little high. 6.7 but everything else was good. I went to a chiropractor for about 6 visits. Nothing has helped. Just saw a neurologist and she doesn t think its neuropathy because it didn t start in my feet and I don t have any pain. I just had an MRI and haven t had the results as yet. I ve completely changed my diet. I m following the Paleo plan. Any idea what this might be? I m thinking of going to an acupuncturist next. Thank-you Doctor: Maybe you have to consult an endocrinologist too regarding your high blood sugar. Polyneuropathy can in some cases start from the upper extremities. But the numbness in the forehead and tongue doesn't comply with neuropathy diagnosis. Did the neurologist recommend the MRI and what region? Do you have any other symptoms or did the doctor found any other sign during the physical examination? I will be interested to know the MRI result or any other test that you've taken."
},
{
"id": 25331,
"tgt": "What causes weird feeling on the chest after a long walk?",
"src": "Patient: I recently noticed this weird feeling in the middle of my chest after long walks or when going up stairs. Maybe I'm experiencing heart flutters. My blood pressure is perfect although and for the most part pretty good shape for a 40 yr old male. Could this be caused by stress or something else? Doctor: Hello and thank you for using HCM.I carefully read your question and I understand your concern.I will try to explain you something and give you my opinion. You should know that typical heart pains have some characteristics. They are retro-sternal pains like squizzing, tightness or heaviness. They last from 5 to 20 minutes and may be irradiated to arms, jaw. This are typical ischemic heart pains.Heart rhythm issues like tachycardia, fibrillation, extra sistolic beats might give symptoms like chest discomfort or this weird feeling in the meadle of the chest.But we can not be sure until we do some examinations.If I was your treating doctor I will recommend some examination like an electrocardiogram, a cardiac echo, a full blood analyze, a holter rhythm monitoring and a strees test.This helps us to differentiate between real ischemic heart pains or just a random extra sistolic beat caused by streess.Hope I was helpful.Wish you good health. Best regards."
},
{
"id": 116905,
"tgt": "Is hemolytic anemia curable?",
"src": "Patient: i am struggling with Auto immune hemolytic anemia from 3 year. i want to know whether this disease is curable or not? now i am using wysolone tablets. and my blood count fluctuates and goes down when i stop the medication. because of using this steroids from long time, i am started getting seviour joint pains.how long i need to suffer with this pain? is there any permenent solution for my sickness? Doctor: Hi, dear. I have gone through your question. I can understand your concern. You have autoimmune hemolytic anemia. Treatment is immunosuppressive agent to prevent destruction of rbcs. So steroid is the drug of choice. One other solution is splenectomy. It will give some benifit. Consult your doctor and plan for surgery accordingly. Hope I have answered your question, if you have doubt then I will be happy to answer. Thanks for using health care magic. Wish you a very good health."
},
{
"id": 70142,
"tgt": "Should i be concerned as i am having a lump behind my ear?",
"src": "Patient: I woke up this morning with a lump behind my right ear that is tender to the touch. I also have a red rash on my forhead along the hairline that is sensitive to touch. I have had some night sweats in the past week. Anything to be really concerned about? Doctor: Hi,Thaks for writing to HCM.That can be a boil or furuncle or even an infected lymph node following the rash on the forehead.Take a course of antibiotic in consultation with your local doctor for the prescription, and it should resolve.RegardsDr. Ashish Verma"
},
{
"id": 177899,
"tgt": "Is Zifi cv and Dolo right medication for typhoid in a child?",
"src": "Patient: my son aged 3.5years weighing 14 kg was diagnosed with typhoid fever[ranging upto 104.5F] 10 days back and is under zifi cv 100 bd & DOLO for last 10 days.Sometimes he still have fever upto 101F.SHOULD SAME TREATMENT BE CONTINUED & PROGNOSIS CONSIDERED POSITIVE? Doctor: Zify & dolo are the best medicines for typhoid fever but fever must settle down in five days.typhoid fever more than 5days has to be investigated further like blood test,Xray chest and urine test Followed by approite medicines"
},
{
"id": 130103,
"tgt": "Why am I having pain in my right arm?",
"src": "Patient: Hi SirMy right is paining from last one week and moreover today it is gaining much pain and ticking and tingling in palm, fingers especially when I rest my hand on chair bar. What would be the reason for this sir. very little numbness of three fingers also started three months back in the same hand. Doctor: Hi..The reason is neural irritation from your neck..this could happen due to various reasons.. overuse..postural issue of neck...facet joint irritation...etc...I would suggest you to initially ice up your neck frequently...15 mins possibly once in every 2 hours..keep your neck supported properly...while sleeping. Keep your neck supported with a cervical collar-- soft collar...Condition will improve..once the pain and tingling sensation comes down then I should start with neck rehabilitation exercises like.. Neck retraction .. shoulder shrugs...resisted neck extension...like keep your hands clasped behind the neck..press your hand towards head and head should be pressed against the hands..the position is maintained for 30 secs..same can be done sideways while hands kept head above your ear...Hope this is helpful..Please revert back in case you need any clarification"
},
{
"id": 69358,
"tgt": "Suggest treatment for a large fluid lump on the head",
"src": "Patient: Hello, My husband is having an ultrasound this pm on his head. Couple weeks ago he had a large fluid filled lump on his head and no hair was growing on the lump. The lump has since gone down in size though not completely. Still bald. No pain. What could it be Doctor: FNAC - fine needle aspiration cytology of the contents should be done to know the diagnosis.Does he have fever?"
},
{
"id": 59744,
"tgt": "Inflammation and tingling in leg. Pain near kidney and liver. Done LFT and KFT. Cause?",
"src": "Patient: Sir/Madam, I m 33yrs old. Height 5 6 , weight 65kg. About six months ago, i had an inflammation and tingling in my leg and foot portion. thereafter i had a pain on my right kidney and liver portion. So i went for test and the result of LFT was SGOT 55, SGPT-82 and Gamma GT-85. KFT result was sodium 134 and chloride 92. i dont take alcohol bt take lot of bettlenut. please tell me what may be the causes and measures to be taken by me? thanks Doctor: Hello and welcome to HCM Regarding to liver function test and electrolytes, the interpretation is: 1. SGOT is marginally raised, normal level is less than $) IU/L 2. SGPT is also increased, normal level is less than 40 IU/L 3. GGT is also marginally increased, normal level is less than 70 IU/L These values indicate a slight liver cell damage and mild obstruction of the biliary tract. Get few other tests related to liver- bilirubin (direct, indirect and total), alkaline phosphatase and serum preteins. These investigations will provide some more information. An ultrasound abdomen should be done to know the status of liver, biliary tract and kidney. The electrolyte level is normal. Sodium and chloride are within normal limits. Consuming a lot of betelnut is deleterious to health. It is known to cause heart attack, stroke, hypertension, COPD and cancer. Tobacco is known to cause of various organs and organ systems like oral cavity, larynx, oropharanyx, esophagus, lungs, kidney, bladder, stomach, pancreas etc. So, consumption of tobacco in any form is harmful. If you develop any lesion in the oral cavity, report it promptly to your doctor. It is better that you stop consuming betel nut or go for tobacco less betel nut. Thanks and take care Dr Shailja P Wahal"
},
{
"id": 54687,
"tgt": "What causes stomach pain,dizziness and fainting after gall bladder removal?",
"src": "Patient: Two and a half years ago I had my gall bladder removed. Evert couple of months I still stomach pain on my right side and get dizzy, sweayy, fainty feeling. Sometimes I do pass out. This past week I've fainted twice. Why and is this normal? Im only 23! Doctor: Thank you for your query.Firstly, for the stomach pain you have, similar to the gall stones pain, its better to repeat an \" Ultrasound examination of abdomen\" to rule out gall stones in Common Bile Duct.secondly, as you passed out twice last week, do following:1. complete blood cell count test2. ECG. 3. Monitor Blood pressure then visit a cardiologist or internal medicine physician.if you have any further queries, feel free to ask.regards,Dr Tayyab Malik"
},
{
"id": 143118,
"tgt": "What causes unstability while walking and body jerks while sleeping?",
"src": "Patient: i feel unstability on my head and body during walking and sometimes during sitting on chair and working on laptop and sometimes during movement of head is xet cr 12.5 is ok for me i feel jerk in ,my body while sleeping Doctor: Hello!Welcome on HCM!I carefully read your question and would like to exaplain that your symptoms could be related to an inner ear disorer. Xet cr 12.5 can be helpful in such cases. But, if you do not feel relieved after a month on this medication, I would recommend consulting with an ENT specialist for a physical exam and labyrinthine tests, to exclude an inner ear disorder. Dimenhydrinate can also help in such cases. Hope to have been helpful!Wishing all the best, Dr. Aida"
},
{
"id": 167784,
"tgt": "Are chills and high grade fever symptoms of flu?",
"src": "Patient: My 19 year old son has the following symptoms and we are wondering about the possibility of meningitis: 102 temperature chills stiff neck He does not feel like throwing up and has been sleeping on and off all day, but is having trouble falling asleep right now. Could it also be the flu? Doctor: having a high grade fever and a stiff neck is a red flag , it could be a simple flu but it can be meningitis , so in order to clear that up physical exam can reveal meaningful irritation signs if it's there and if not can pretty much exclude that . so I highly recommend taking him to his doctor you perform the exam ."
},
{
"id": 58236,
"tgt": "Diagnosed Hepatitis C by using needles. Undergone methadone treatment. Feeling weak, appetite loss, WBC high. Reason?",
"src": "Patient: hi i was diagnosed with hepititus c a few years ago from using needles. i have been tested a couple times sense and it was undetectable. i relapsed a year ago but didnt use needles. i got on a methadone treatmetent program 7 months ago. it helped with my addiction but i started to get server headachesand it eventaully went away. i started a new job 4 months ago and its night shift. resently i have been feeling weak,confused,dissy,no appitite,wait loss and dehidration. i have no insurance so i cant see a normal doctor. i went to the er and they did blood work and nothing came back wrong,all they said is to drink more water and get off the methadone. i do drink plenty of water. i read that you can feel if your liver is enlarged by pressing down under your ribs. i did this and i cant really feel anything but when i do this there is a slight pain. my question is do you think its the hep c or the methadone. and also when i started the methadone program they did blood work and my liver was fine but my white blood cells are up. i also had a tooth infected at the time of the test. thank you Doctor: HiI am sorry for your situation.If the levels of liver enzymes are not increased, then, you should be in peace regarding hepatitis C.Most probably, the methadone you are using is the culprit of the headache and feeling weak and loss of appetite. With regards to your high WBC, as you are saying to have an infected tooth, most probably that is the cause.Drink plenty of water, have your tooth well-treated, try to be physically active to overcome all your problems. All the bestDr.klerida"
},
{
"id": 109808,
"tgt": "Suggest remedy for stomach ache and back pain after fever",
"src": "Patient: Hi Iam Magesh 35 Year old male, Two weeks back I had a sever throat infection with fever, later intermittent fever with sever body pain, last 4 days I have sever stomach ache, doctors checked kidney, lever , by urine, blood, xray and ultra sound scan and said nothing shows any wrong alarm, and they said it might be virul infection, after three days of medication in the hospital still i am having stomach ache and back pain Doctor: hiWell it could be viral,Hope they have evaluated your complete blood picture,viral infection can usually last more than three days,so maybe u have to continue that treatment and in case it doesnt subside after a week,maybe look at higher investigations."
},
{
"id": 193697,
"tgt": "What causes nausea with pain in sternum?",
"src": "Patient: Hi. I just had some roast beef on bread with mayonnaise-based horseradish sauce. About 30 minutes after, I started having nausea and some pain. The pain was about a 1 to 1.5 and located directly below (as in floor direction) my sternum. I also had a little pain in my back like a slight burning. It's starting to go away now, but I still have a slight bit of nausea and it feels like there's a slight \"lump in my throat.\" I took some omeprozole to help and it seems to have had some effect. Could this be my gall bladder? I have been having this same thing happen on and off for about 3 or four months now. And last time I went to the ER (about a month ago), they did an ultrasound on my gall bladder and had said I do have a stone. Doctor: Hi, Yes, there are a high chance it can be due to gall bladder. Doing an ultrasound again will help you to know increase in stone size/ number or inflammation. Hope I have answered your query. Let me know if I can assist you further. Take care Regards, Dr S.R.Raveendran, Sexologist"
},
{
"id": 43279,
"tgt": "Going through fertility treatment. Having long periods, using aygestin. Have bloating, pain and night sweats. Advise?",
"src": "Patient: Hi i am going through fertility treatment. I was having a 2 week period so dr put me on aygestin to stop it. I quit using aygestin last wednesday. Got a period Sunday and in the past 3 days I have been in hell. So much pain bloating night sweats....how long will this last. I swear I look 4 months pregnant?!! I can't even eat much because I feel like I'm going to burst and get dizzy. Supposed to start taking clomid tomorrow. Any help or ideas? I called dr today spoke to asst and they always blow me off. Said its normal but I'm miserable! Doctor: Hi, it could be due to the drugs you are on, so don't worry, you can continue to have the treatment. you explain your doctor it is miserable and unbearable so you want some thing to stop this pain before starting the clomed . the thing depends upon the explanation you give to him. Thank you."
},
{
"id": 85259,
"tgt": "Could a sinus infection causes difficulty in breathing?",
"src": "Patient: Since Friday, I have been experiencing bloating to the point I look pregnant, I m also having difficulty breathing from the pressure-like there isn t enough room-pressure on my diaphragm. My bowels have been loose, and flatulence. I also have some discomfort on my right side along the base of my rib cage in the back. I ate whole grain bread, and an apple yesterday, today I took some probiotics. I normally do not have this problem, last month I was on antibiotics & steroids for 3 weeks due to a sinus infection....could this cause this? Is ther anything I can do to get relief? Thanks! Doctor: Hello,It is unlikely be related to sinusitis. Severe acid reflex may be the cause. If symptoms persist, it is better to consult a physician and get evaluated. As of now antacids like omeprazole or pantoprazole for symptomatic relief.Hope I have answered your query. Let me know if I can assist you further. Regards, Dr. Shinas Hussain, General & Family Physician"
},
{
"id": 132464,
"tgt": "What are the symptoms of a groin muscle pull?",
"src": "Patient: I m wondering if I have pulled a groin muscle. I have pain along each side of my abdoman from my testicles up and to my hip joints. this started in December and has eased off but is still there. This started as a result of bending and twisting while putting wood into an outdoor furnace. Doctor: Hi Hope this message finds you in good health.I have gone through your complaints and understand your concern.I generally see many cases like this at my clinic.pulled groin muscle causes pain once you try to lift the leg or move it in either directions. also causes pain while walking,runningYou should eventually get back to normal in coming few days.Nothing to worry about, as long as i am there to help you.\u00a0\u00a0\u00a0\u00a0\u00a0I hope your question has been answered.If you have any follow-up queries,feel free to consult me anytime. Take care,God bless."
},
{
"id": 149890,
"tgt": "Fainting before vomiting, nausea after wisdom teeth removal, breathlessness, rapid eye movement. Scared",
"src": "Patient: My 17 year old daughter hs started fainting right before she vomits......two separate times earlier this year she had the stomach bug and told us she woke up in the middle of the night on the bathroom floor right as she was about to vomit. Then yesterday, she had her wisdom teeth out and was very nauseaus. She fainted in bed and i witnessed it for the first time. It almost looked like a mini seizure to me, she stopped breathing for a few seconds, she had rapid eye movement, and her arms tightened across her chest . She is perfectly healthy and has no allergies. Very scary. Doctor: Hello,Thanks for writing to us.Extraction of wisdom tooth usually leads into tiredness,nausea etc.Plenty of fluids as well as nutritious diet has to be given.IV glucose solution has to be administered.Antigastric tablets has to be administered prior to intake of antibiotics.Please do not take medicines in empty stomach.Take adequate rest.Anxiety and stress has to be avoided.Take care."
},
{
"id": 213536,
"tgt": "Prescribed feliz-s 5mg and 10 mg for paranoid, nervousness, behavior change. Any side effect? Don't have any depression",
"src": "Patient: Doctor asked me for feliz-s 5mg and feliz-s 10 mg for total of one month and then asked to meet him. But i have not started taking it as i m hesitant of side-effects after withdrawing it after 30 days. I found on internet that it is a depression cure medicine but I didn t consult doctor for depression. I went to doctor for paranoid, nervousness , behaaviour change problems and he prescribed me these feliz-s tablets. Can you please guide me whether this feliz-s is anyway related to sexual activities. Also he told me that i will loose some weight as this medicine lowers one s hunger. Is that also right? Should i take these medicines or not as I m only 24? I think i don t have any depression problem. May be I think a lot. Also my BP came 140/90 one time and 122/80 other time. Is this High BP? Please try to answer all my questions. Doctor: Hi You seem to be quite worked up so please relax a bit. Let us try to understand your problem. first, did you consult a psychiatrist or a GP?. Feliz-s is Escitalopram, an SSRI prescribed for Depressive disorder,Anxiety disorder and OCD as well. So I assume your doctor must have prescribed you after carefully considering all pros and cons. it is not right to think that it will lower weight. It can cause reduced appetite but this side effect is temporary . this is often associated with nausea so nobody should think of losing weight with Feliz-s S/E has to be tackled by psychiatrist, when present. They have sexual side effects as well like reduced libido and delayed ejaculation. All SSRI have similar Side Effect Profile. Decision to take medicines has to be taken by considering risk-reward ratio -that means whether the disease is more trouble some or the Side effects. For Example-Pain Killers can cause stomach ulcers that may bleed you to death- do you stop taking pain killers!!! make an informed decision in consultation with your doctor best of luck"
},
{
"id": 82806,
"tgt": "Can lupus causes protein in the urine or UTI or SLE?",
"src": "Patient: Hello. My daughter has SLE/mixed connective tissue disorder. Several times over the last 2 years she has had urinalysis with 1+ protein. She has had several normal urinalysis in between her abnormal ones. Her most recent one shows 1+ protein, trace ketones, no glucose, trace blood and trace leukocyte esterase. Due to her lupus, she also has pulmonary fibrosis which she took 9 monthly cytoxin infusions for and fatty liver disease (non autoimmute) Could her occasionally throwing protein in her urine and ketones be the starting of the SLE attacking her kidneys? Or could it just be a UTI? Doctor: it could be a UTI, but the only way to confirm it is with a urine culture and sensitivity.however if the proteinuria continues she will need to get it checked for lupus.also you need to know the exact amount of protein she is losing by getting a urine protein creatinine ratio done."
},
{
"id": 122597,
"tgt": "What can I do to get rid of stiffness in the knee after knee replacement surgery?",
"src": "Patient: I had a knee replacement done in June 2009. I am 85 years old now and wonder if there is any way to lubricate the parts like we do with WD40. My knee is getting very stiff? I do not wish to be billed for this question or your answer; also I do not wish to receive a bunch of emails. Thank you Doctor: Hello, The stiff knee is quite common after knee replacement surgery. You have to start on physiotherapy to get back on usual. Consult a physiatrist and he will direct you accordingly. Hope I have answered your query. Let me know if I can assist you further. Take care Regards, Dr Shinas Hussain, General & Family Physician"
},
{
"id": 194355,
"tgt": "What causes bumps on the shaft?",
"src": "Patient: I have a couple of 1mm sized bumps spread apart by atleast a cm to 2 cm's apart on my shaft. It started with one or two after s handjob and sex then I master aged a day later and it seemed to have spread. It doesn't itch burn or secret anything. The occasional one has a white dot usually with a hair. Some not. Been two weeks now. I think are slowly going away. Doctor: Hello, Since it is related to post-sex. 1. Herpes - Genital warts (HPV) can be diagnosed by the presence of very painful blisters, sallow ulcers or red base. There are treatment options for genital herpes, but there is no cure. As to the treatment, it is based on relieving basic symptoms, early healing as well as the decrease of symptoms duration. 2. If it\u2019s not painful- Syphilis, diagnosed by Blisters appear as the first symptoms of primary syphilis. They usually disappear on their own with time, but remains in the body. first blisters on penis appear, important to seek medical help. 3. molluscum contagiosum - small, painless bumps. Need to do HIV, VDRL, urine complete. Hope I have answered your query. Let me know if I can assist you further. Take care Regards, Dr S.R.Raveendran, Sexologist"
},
{
"id": 29784,
"tgt": "Suggest treatment for throat infection",
"src": "Patient: My father had a throat infection and during that period and after that he wasn't able to speak clearly.Looks like he is bit stammering and not able to speak clearly ,just wandering is it is reaction of antibiotic or something.he has high blood pressure from last few months and taking medication as wellPlease advise.. Doctor: HI..your father is a known hypertensive having difficulty in speaking clearly.Considering his age and hypertension history rather than thinking of allergy i consider it to be dysarthria or neuroligal difficulty in speech.The problem can be from cerebrum,cerebellam, small part of brain called basal ganglia or the spinal cord.Kindly get him investigated and take up neurologist opinion and proceed.Take care."
},
{
"id": 192690,
"tgt": "What causes white head pimples at the base of penis after intercourse?",
"src": "Patient: I got 3 tiny whitehead pimples at the base of my penis about 5 days after protected sex with a new person. I had also shaved that same area about 2 days before then encounter for the first time. I popped them quickly and they seems to be mildy infected but responed to alcohol swabs and OTC antibiotics. The were very small, like a pin head. Does this sound like herpes or more like folicullitis or something similar. The girl had no symtoms and tested STD clean in Agusut of 2010. Thanks. Doctor: Hello,It may be due to smegma. There is no need for any active management. In case of bleeding you may require laser surgery after surgeon consultation. Maintain proper hygiene. Avoid harsh cosmetic products.Hope I have answered your query. Let me know if I can assist you further. Regards, Dr. Shyam B. Kale, General & Family Physician"
},
{
"id": 213562,
"tgt": "Fear of failure, tensed, rapid heart beat, obsessive. Treatment?",
"src": "Patient: Hello doctor,am 27 years old.I am suffering from a series of psychiatric problems.I am feeling feared always,my heart beats very fastly throughout the day and feeling of fear and some secretion is there in my stomach always.I am not relaxed for even a minute in my life for the past 10 years.I am feeling obssessed to do certain things,in case of failure i sense a terrible fear.I consult a psychiatrist but he treats me just for depression .Please help me. Doctor: Hello. Thanks for writing to us. Your symptoms are suggestive of anxiety related depression. Avoiding stress and practicing relaxation exercises like deep breathing and yoga will be helpful. You can also ask for cognitive behavioral therapy from your psychiatrist. I hope this information has been both informative and helpful for you. Regards, Dr. Praveen Tayal drtayal72@gmail.com"
},
{
"id": 64712,
"tgt": "Could itchy, soft lump on shin with grey centre be a bruise?",
"src": "Patient: My husband has a soft lump on shin area. It is the size of a half dollar and soft to the touch. The center of it seems to have a greyish color to it.....possible a bruise? But he said he doesn t remember hitting his leg on anything. And just to the right of it, he said it is itchy.....it could be just a coincidence. Any ideas??? Doctor: Hi,Dear ,thanks for the query to HCM. I studied your query in-depth.In my opinion -this lump on the shin may be Erythema Nodosum-due to sulpha reaction-?sarcoidosis?crohns disease?some of the EN may be itchy.?Or it could be-a -spider bite?bed-bug bite?Consult your ER doctor who would take care of it.Hope This would resolve your query.Well come again for any more query."
},
{
"id": 103836,
"tgt": "Yellow spots on hands, cold, allergic to pollen. Advise?",
"src": "Patient: so on my right hand i have 2 discolored skin spots which are yellow, they are not huge, one the size of the top of a qtip and the other even smaller, on my left hand there are around 8 - 10 really tiny yellow spots around the size of a ball point tip. They do not hurt and are not rashes. I also have allergies to pollen which have been acting up and i believe i have a cold. I have noticed them yesturday Doctor: Hello,Just discolouration of the skin. These are only macular rashes. With this size and no itching it is most unlikely to be due to any allergic reactions to any allergens.Common cold is significant here. This macular rash may be due to viral infection.If it is a common cold they are self limiting and subside on their own."
},
{
"id": 143635,
"tgt": "Need knowledge on brain atrophy and disorder of brain circulation?",
"src": "Patient: Nov. 2010 fell and hit head on concrete steps. CT scan confirmed bleeding in the brain. Followup MRI in April 2011 indicate diffuse brain atrophy and disorder of brain circulation. Feeling fine now but can you explain brain atrophy and disorder of brain circulation. thank you. Doctor: cerebral atrophy means brain has shrunk and problem in brain circulation meams there.could be a.block in one or two of Cerebral arteries only angio can reveal the exact site of block"
},
{
"id": 90846,
"tgt": "What causes pain when taking a deep breath or stretching slightly?",
"src": "Patient: Hi, I keep getting a sharp pain in the bottom half of my left side every time I breath deep or stretch slightly, this has been there since last night and I have woke up today and it is still the same. I have also noticed that i am going to the toilet a lot more than usual, could somebody tell me what this may be, my mum said the pain is around my kidney area Doctor: Hi ! Good morning. I am Dr Shareef answering your query.If I were youir doctor, I would advise you for a stool test and a urine test followed by a culture sensitivity of the urine to decide on the appropriate antibiotic for a possible urinary tract infection. Apart from this, an ultrasound abdomen could be a helping tool in ruling out any other urological or pelvic pathology. Your abdomen also needs a physical examination by the doctor. Therefore I would advise you to consult your family physician and go as per his advice.I hope this information would help you in discussing with your family physician/treating doctor in further management of your problem. Please do not hesitate to ask in case of any further doubts.Thanks for choosing health care magic to clear doubts on your health problems. Wishing you an early recovery. Dr Shareef."
},
{
"id": 15675,
"tgt": "Rashes on stomach and face which appeared after black eye. No medication taken. what it could be?",
"src": "Patient: Hi i am a 23 year old male. I have a small round rash on my stomach and to smaller round rashes below and about it. I also have 3 what looks like burst blood vessels on my face. This first blood vessel appeard after a black eye and the other 2 have recently appeard. I have had the rash on my stomach for over 12 months now and it is sometimes itchy. I don't take medication for anything. Do u have any ideas what it could be? Doctor: Hi, It seems that you are having erythema annulare centrifugum.Mostly there may not be any cause.But, some time there may be some cause. Certain food like tomato, contact dermatitis, infections, drugs like chloroquin, penicillin, finasteride..etc,stress,internal diseases..etcVitamin D, steroids , immunomoduators may be helpful.Local application of calcipotriol cream may improve the lesion.Still, you consult dermatologist.I hope you got my answer.Thanks .Dr. Ilyas Patel MD"
},
{
"id": 192405,
"tgt": "How to treat tightness around foreskin?",
"src": "Patient: First of all, i need to know which doctor sholud i consuld, i have a problem related with the foreskin. it is very tight and it has developed a ring like structure on the inside which actually makes it tight. it is impossible to retract it over the head. who will solve my problem - skin specialist, general physician or someone related to sexual health? Detail description : age 28, never engaged in sexual activity except masturbation,the skin used to pull back(still it was tight) when i was teenager, then sometime during college i developed this condition. the while white skin looks like dead skin. now there is some part of glans where the foreskin connects to the glans which also has become while in colour and appears dead. major part of foreskin looks healthy and the problem lies with little area only Doctor: Hello, PHIMOSIS is a congenital narrowing of the opening of the foreskin so that it cannot be retracted. Phimosis is a very common problem. In most of the cases you can clear the problem yourself. Phimosis could cause moderate to severe pain while having vaginal intercourse. Apply oil on your foreskin and on the glans. Move the foreskin forward and back slowly without hurting you much. Because the oil is lubricant, it will be easier than without oil. Do this forward and back movement 20 times several times in a day. Repeat the same every day for around one month. You will get the elasticity and you will be able to retract the foreskin completely. In case if there is no result from the above massage, you can always go for a minor surgical procedure called circumcision. Therefore I suggest consulting a skin specialist for physical examination, diagnosis and treatment. Hope I have answered your query. Let me know if I can assist you further. Take care Regards, Dr. K. V. Anand"
},
{
"id": 115804,
"tgt": "What does the following blood test result indicate?",
"src": "Patient: hi. my blood test are as follows, anything seem not correct? have been having test for a year but obviously do not understand them, esr 20 crp 17.6 Hb 13.7 mcv 90.1 wbc 5.8 neut 2.96 plats 243 alt 18 alp189 serum creatine 67 ana negative Doctor: Hi, dearI have gone through your question. I can understand your concern. Your esr and crp is high suggestive of some inflammatory disease. Your rest of yhe reports are almost normal. Your alp is high. You should search for inflammatory cause in your body and start anti inflammatory drugs accordingly. Consult your doctor and take treatment accordingly. Hope I have answered your question, if you have doubt then I will be happy to answer. Thanks for using health care magic. Wish you a very good health."
},
{
"id": 24669,
"tgt": "What causes vomiting and elevated bp while having gerd?",
"src": "Patient: I am 32 yr old. 350lb, 6.1ft tall.. I am vomiting constantly with pains in my stomach...i have been to the GI, ER and no diagnosis besides GERD...my head is hurts and my bp rising high then falls back to normal. It feels like something is in my throat, please help Doctor: Hello!Welcome and thank you for asking on HCM!I carefully passed through your question and would explain that your symptoms seem to be related to anxiety. GERD could also mimic this clinical situation. I would recommend performing some further tests to determine the possible cause: - a fibrogastroscopy to examine your digestive tract- some blood lab tests (complete blood count, thyroid hormone levels, cortisol and aldosterone plasma levels for adrenal gland dysfunction, blood electrolytes). - urinary metanephrines to exclude possible pheochromocytomaA head CT scan may be necessary if abnormal physical examination. If all the above tests result normal, you should consider anxiety as the main cause of your problems. In such case, you should consult with a specialist of the field and discuss the best treatment options. Hope to have been helpful!Best wishes, Dr. Iliri"
},
{
"id": 40455,
"tgt": "What does the following follicular study report suggest?",
"src": "Patient: Hello I have done follicular study on 2nd of my period ET was 2.9mm then I have done on 9th day ET was 5.4mm then I have done on 11th day left ovary MSF was 13x12mm and the ET was 5.5mm. kindly help me whether I will be able to get pregnant in this cycle. I have taken Letroz tablet from 5th day of my cycle for 5days. Doctor: Hi, Endometrial thickness is very thin which will not augment your chances of implantation even if conception occurred. I would recommend that you refer to your doctor for progesterone supplements to increase your endometrial thickness. Hope I have answered your query. Let me know if I can assist you further."
},
{
"id": 222386,
"tgt": "How dangerous is extra renal pelvis in kidney of fetus?",
"src": "Patient: Hi, my wife is expected and after having a ultrasound in the 28th week of pregnancy, there were extrarenal pelvis - right kidney 5.8 and left kidney 4.4. I want to know how safe this after birth and is there any harm of this extrarenal pelvis. Please give me detailed information regarding extrarenal pelvis in kidney of fetus. Doctor: Hello, and I hope I can help you today. A extrarenal pelvis is a variation of the anatomy of the kidneys which is actually found in about 10% of the population. It is absolutely no long-term medical consequences but it can be confused with other types of urinary obstruction which do not have as good a prognosis therefore it is always important to monitor it if it is seen in a fetus to make sure it is not another more dangerous diagnosis like urinary obstruction.So there are no consequences to the baby's health by having an extrarenal pelvis. Many people who have this condition are not diagnosed until they have a test such as an ultrasound of the kidney or CAT scan for another reason and have never had any symptoms over the course of their lifetime.So there's no reason to worry further about the health of your baby because of the extrarenal pelvis.I hope I was able to adequately answer your question today, and that my advice was reassuring.Best wishes to you and your wife for the rest of the pregnancy,Dr. Brown"
},
{
"id": 58804,
"tgt": "Have nonalcoholic fatty liver disease, on TRICOR, gone off this medicine. How long will it be in body ?",
"src": "Patient: Hi I have been on TRICOR 145 mgs for the last 7 years and now I have nonalcoholic fatty liver disease. I have taken myself off the meds a week ago now . my legs are feeling much better as the burned all the time since I was on the meds. I told my doctor I went off the meds and she said she would have done the something . I wanted to know how long will it take to get the TRICOR out of my body ? I have gone on coq10 and fish oils and milk thistle . plus I have been juicing beets and carrots to help my liver. and walking 2 mils a day . can you please help ... thank you. Doctor: Hi, thanks for using healthcare magicNon alcoholic fatty liver disease is one of the most commonly seen liver diseases of this present age.It is related to lifestyle issues such as diet and activity.There is actually a correlation between the high use of carbohydrates and fatty liver. This relates to the storage of excess carbohydrates.The tricor would not induce fatty liver,it is actually used to treat fatty liver and associated conditions in some persons.To prevent any further progression and to possibly improve the liver you may want to consider reducing the use of carbohydrates . This is actually the recommended dietary change.The exercise would also be helpful.I hope this helps"
},
{
"id": 145478,
"tgt": "What causes recurring sudden passing out with stopped breath?",
"src": "Patient: No aparent reason otherwise healty male. Non smoker non drinker passes out stops breathing. Heart attack and stroke have been ruled out. Keeping body on 34 degrees giving nitro. Medical induced a coma medical induced parlysis. Full life support? Your best medically idea would be? Doctor: Hi,Thanks for writing in.There are causes other than heart attack and stroke that might cause a sudden passing out. Please find the conditions as follows.1. Vasovagal syncope. This is a condition when due to the stimulation of the vagal nerve and due to over activity, there can arise a shock like state when the patient becomes suddenly unresponsive.2. Hypoglycemia. This happens due to subnormal amount of glucose circulating in the blood. This causes insufficient nutrition to the organs including the brain and can cause sudden collapse.3. A sudden generalized seizure. This happens when there is sudden outburst of electrical activity in the brain and this leads to a severe epileptic seizure from which the patient might take time to make a recovery.4. Electrolyte imbalances like low sodium or potassium states which give rise to sudden cessation of alertness and awareness.5. Bleeding from any organ inside the body that is not visible from outside. The patient goes in to a state of hemorrhagic shock. Some bleeding cases can be self contained but the rapid loss of blood causes sudden loss of consciousness.6. Sudden attacks of apnea due to sleep or even during the day can cause a confusion state and the inability to keep awake due to reduced supply of oxygen to the brain. Please discuss the above possibilities and any reason that might have been pre existent and was not known earlier."
},
{
"id": 25897,
"tgt": "Should heart stutters be cause of worry?",
"src": "Patient: For the past hour I have been having what I can best describe as heart stutters--it feels like my heart is trying to skip a beat. It's occurring every 5-10 minutes. Should I be worried? I am in my early 30s and drink energy drinks on a daily basis. Could the caffeine be the cause of this? Doctor: Hello,I have gone through your query.Thanks for using HCM.Basically your complaint points towards possibility of extra systoles.Yes you must give up coffee.If your symptoms persist you should get ECG done and get further work up if needed,My best wishesDr.Rajesh Teli,MD."
},
{
"id": 108393,
"tgt": "Suggest treatment for severe back pain",
"src": "Patient: I jumped off a chair lift at my local ski hill about 10 feet from the ground. It hurt my back and now when I jump or walk with a spring in my step I get a sharp pain in the middle of my back but slighty to the left. I was wondering if it s serious and I should go see a doctor or just let it heal on its own Doctor: Hello, I have studied your case. Fall from height may lead to vertebral compression fracture.You may need to see doctor and do further investigation.You need decompression if fracture is compressing spinal cord.If there is no neurological involvement then conservative brace or rest is sufficientLater on follow rehabilitation and physiotherapy.Hope this answers your query. If you have additional questions or follow up queries then please do not hesitate in writing to us. I will be happy to answer your queries. Wishing you good health.Take care."
},
{
"id": 218927,
"tgt": "What causes nausea during pregnancy?",
"src": "Patient: Hi, I m 14 1/4 weeks pregnant with my second baby. This pregnancy I ve got off pretty lightly with nausea compared to last time (this time I just felt sick for around 1/2 hour a day from weeks 7-9, but then felt fine since. I do quite often still feel faint in the morning, but this usually goes away after having breakfast. I haven t vomited last pregnancy despite feeling much more sick. This pregnancy I hadn t either until today! I felt more faint than usual this morning. It seemed to improve after a couple of small chocolate squares. At 12:15 I started feeling a bit sick, and by 2 I vomited once. I ve felt really weak since but not sick. I ve eaten some dinner but just don t have an appetite as big as normal. I feel faint if I stand for too long. I m worried I have got something that could affect my unborn baby? Could it be listeriosis? I ve got no flu like symptoms though, just those described above, and I m super careful with food and hygiene at the moment. Does this sound like an illness or late morning sickness? Hopefully someone could help as I m worried and searching online is not helping. Doctor: Pregnancy first trimester is characterised by morning sickness.It may vary in duration and severity depending from person to person. It is advisable to not take too much liquids after breakfast and take dry goods in morning. Also, tablet Doxylamine and pyridoxine supplementation also helps. severe cases are called hyperemesis gravidarum which need hospitalisation."
},
{
"id": 79616,
"tgt": "What is the pain am I having in my chest below my breast?",
"src": "Patient: I have a pain that comes and goes from the middle of my chest just below my breast bone and radiates around to the middle of my back. At some points it feels like something flipped in there and the pain is really bad for about 3 seconds then it's gone. Doctor: thanks for asking your questioni completely understand your problemin my opinion at that young age peptic ulcer/gastritis is a possibility.cardiac diseases are less likely at that age.you need to take a proton pump inhibitor like pantoprazole once daily after consulting a general physician.actually physician can do an ecg and rule out any other causethanks /regardsfeel free to ask more questionsmay god bless you with good health"
},
{
"id": 180881,
"tgt": "What causes upper molar toothache?",
"src": "Patient: I have tooth aches that vary in intensity when I change positions. The tooth is an upper molar on my left side. Pain increases when I lay down and decreases when I sit upright. I went to my Dentist about 2 weeks and he filled a cavity I had in the wisdom tooth next to the molar giving me pain. The pain still exists. What do you think is going on? Doctor: Hi..Thanks for the query..Your symptoms are pointing towards dental abscess formation due to deep Infection in tooth..When infection reaches below root tips of the tooth there is pus formation and on lying down the pressure increases causing increased intensity of pain..So my suggestion is to consult an Oral Physician and get evaluated..An x ray should be done so that exact condition of tooth can be ruled out..If the decay is such that tooth can be saved then Root canal treatment can be done under antibiotic cover..If the decay is deep extraction is advisable..For now start taking antibiotics like Augmentin and anti inflammatory painkillers like Ibuprofen and do cool compresses over the cheek..Hope this helps..Regards ."
},
{
"id": 16105,
"tgt": "Rashes appeared on back and shoulder. Could it be scabies?",
"src": "Patient: My son has a rash that runs from bottom back of his hair line to his shoulders. He has blisters. I was told that it is an allergic reaction to something but I m afraid it may be scabies . How can I tell for sure. They are not running in a striped pattern and I m told that scabies are very painful. He is not complaining of pain except if he touches it. Doctor: This is unlikely scabies..coz sacbies generally occur on interdigital space..itching in night. how old is your son ?"
},
{
"id": 98533,
"tgt": "How can itchy rashes caused by Alendronate Sodium be treated?",
"src": "Patient: I have had reactions to a pill I have been taking once a week for nine weeks..it is Alendronate Sodium 70mg. I have had Hives, swollen legs, ankles. very itchy skin, and now today a swollen upper lip... My Dr. told the Nurse to tell me to take Benadryl every 6 hrs till medication is out of my system... does this sound ok??? Doctor: hi sir/madam,Alendronate Sodium is also called Fosamax.Fosamax can cause irritation, inflammation, or ulcers of the esophagus which may sometimes bleed.Taking Fosamax exactly as your doctor prescribes can help lower your chance of having esophagus problems.If you get chest pain, new or worsening heartburn, or have trouble or pain when you swallow, immediately stop taking Fosamax.direct consultation is required with your symptoms with all detail reports.hope it was help ful"
},
{
"id": 102616,
"tgt": "How to ease the breathing problems?",
"src": "Patient: I've taken advair since it was available. I'm a lawyer, there are no jobs for lawyers, so I started my own firm, but I still can't make enough to afford my old health insurance (which went up to 800 a month) I'm now on the subsidized plan, which will only cover 200 of the 479 that advair is. What can I do? I can't breathe, they also won't cover flovent, that's 270-something out of pocket. I've already been lectured by my doctor that I'm overusing my rescue (xopenex) inhaler, and as a result my heart rate is 115 resting. Doctor: hi, you can try oral drugs like salbutamol or deriphylline or terbutaline (tablets) with along monteleukast which are very cheap compared to inhalers. they reduce the frequency of episodes and also the requirement of inhalers. next time you visit your GP, tell your problem and ask for them. hope this helped."
},
{
"id": 48016,
"tgt": "Do i have kidney dysfunction?",
"src": "Patient: I am female, I had fever for 4 days due to inflammation in tonsil, I took Panadol Extra ,then I had pain in my kidney , so I stopped to take panadol and I made analysis for kidney function and urine analysis I find that creatinine is 0.8 and urea 22 but albumin is + in urine and pus cells 6-8My question is Have I kidney dysfunction Doctor: Your kidney function test are within normal.Albumin in urine can be positive in fever.More than 10 pus cells in urine are significant. So don't worry you don't have any kidney dysfunction."
},
{
"id": 186614,
"tgt": "White smelly stuff and swollen glad behind ear related to impacted wisdom tooth?",
"src": "Patient: Hi i have been having trouble with my mouth. I think i have an impacted wisdom tooth, as i have white smelly stuff coming out it. I also woke up with a swollen gland behind my ear. (Not badly swollen, more tender) im just scared. Hoping its all related to the wisdom tooth. Doctor: thanks for your query, the the white thing that is coming out from the tooth could be pus discharge and the wisdom tooth might have infected resulting in pericoronal abscess formation. the swollen structure could be the regional lymphnode secondary to tooth infection. consult a oral physician and take radiographs and a course of antibiotic and get the tooth removed. i hope my answer will help you. take care"
},
{
"id": 161071,
"tgt": "Provide details about Ehler Danlos Syndrome in children",
"src": "Patient: yes my daughter had a repaired muscle capsule this summer with a orthopedic specialist ( Dr. Bradley..team Dr. for the Pittsburgh Steelers ) he mention my daughter has Ehler Danlos Syndrome. Can you give me some details or recommended Dr. listing for treating this Syndrome in Pittsburgh, PA? Doctor: Hello,It is a congenital condition affecting collagen synthesis which might cause different problems during the growth and development. The collagen based structures like ligament and bones will be brittle and the child might be prone to develop fractures and injuries more frequently. There are various types and prognosis may vary between them.Hope I have answered your query. Let me know if I can assist you further. Regards, Dr. Shinas Hussain, General & Family Physician"
},
{
"id": 151401,
"tgt": "MRI done to check VP shunt functioning. What does the report indicate?",
"src": "Patient: Recently had an MRI done that was to check that my vp shunt is functioning correctly and not over shunting. My question is it says slit like lateral and third ventricle and I ve had others who have Chiari and hydrocephalus like myself say that is a sign of syphoning, is this true. Other question is it made a side note that it noticed on my thoracic MRI the my liver and spleen are prominent. What the he k does that mean? Does that mean like enlarged? Doctor: REGARDS FRIEND SLIT LIKE VENTRICLES ARE NOT SIGNIFICANT IF THERE ARE NO SYMPTOMS . MRI DORSAL SPINE SEEMS TO BE INSIGNIFICANT AGAIN Dr. Vineet Saggar (MCh) Neuro Surgeon / Spinal Surgeon"
},
{
"id": 124239,
"tgt": "Can weakness and spasticity of ankles be treated?",
"src": "Patient: A few doctors have told me that I needed to have my left ankle s ligaments repaired because of the continuous rolling. A few doctors have told me I have sustained clonus in the left leg. However, all scans and tests show normal. Symptoms include: severe weakness, spasticity, rolling, hyperreflex, etc Doctor: Hello, As mentioned you been facing issues with weakness and spasticity of the ankle I think it's not the case you mentioned in the history. Clonus - it's a higher motor function controlling and just by the ligament of the ankle this can not happen. Spasticity - this is also a higher motor function controlling and not the ligament connection. Hyper reflex - I think you meant by this is that the plantar reflex is having hyper tone. So this is also the Higher motor function and nothing to do with the ligament. Did you mention about scans in history? Were you talking about the MRI? Have you taken the MRI of ankle or the brain? If of brain than what did it reveal in the higher centres of brain? and if ankle than what did it reveal about the internal structures of the ankle? you even mentioned about the test? what tests have been done? blood? urine? I don't think surgery of ligament makes any sense over here with respect to the symptoms you mentioned. The symptoms you mentioned are of higher motor functions in the higher centres of the brain. I will recommend you to meet up a neurologist rather an orthopedist. A neurologist will be the right person to guide you. Coming back to clonus, spasticity and weakness. There is a therapy plan which you can try out and that is using a balance board rehabilitation program under a guided neurophysical therapist. Hope you will feel better. But I will still insist that you meet up a neurologist and have a word. this will reduce your worries. Hope I have answered your query. Let me know if I can assist you further. Regards, Jay Indravadan Patel, Physical Therapist or Physiotherapist"
},
{
"id": 14808,
"tgt": "Suggest treatment for severe itching all over the body?",
"src": "Patient: For the past month I have severe itching at night, all over my body and head, (no rash). It gets so bad that i have taken benedryl almost every night. It used to just happen only when I was in our den for more than 1/2 hour or when I ate anything with soy lethicin (commonly found in most chocolates). Also itch if I used soap products like Tide. So I stick with the same skin, hair, makeup and cloths washing products. I have recently started taking Solostar Lantus(15u) at night. I have known allergies to dust mites, cinnamon and soy lecithin. I have changed from a feather to a foam pillow. I itch whether I sleep in my own bed or on the sofa. Doctor: Dear Madam,From what you described, you have allergies often which cause itches. However it would be good to rule out causes of itches especially given the fact that this particular itches as you said have been a problem since a month now and at night.Accumulation of certain substances in the body due to their not being able to be excreted can also cause such itches. This is common when the gall bladder which uses bile to excrete toxins from the body has some issues.So kindly see a physician for lab test especially for the gall bladder to rule out other causes other than allergies. Check to see if something ahs changed too in your diet or bed or what you use to ensure its not simply some allergy. However it would be good to see a physician to rule out some other cause such as gall bladder malfuntion.Hope this helps. Quick recovery and regards,Dr Mbong"
},
{
"id": 138677,
"tgt": "What is the treatment for pain in the arm?",
"src": "Patient: I have had this pain in my left arm down to almost my wrist. It first started where i would have a sharp pain between my armpit and boob. It doesnt hurt all the time but has become more often lately. Its an aching throbbing feeling but also an numbing weak feeling to where i loose my strength in my arm. After a while it will pass but dont no if its something serious. Need help Doctor: Hi,take this seriously, it could be a lump compressing on your nerves, or any infection, you need an examination by a doctor to exactly judge what your symptoms mean, and what is there cause,I recommend you to consult your doctor at the earliest."
},
{
"id": 92849,
"tgt": "Pain in lower stomach. Undergone endoscopy, colonoscopy. Has constipation, acid reflex. Kidney stones?",
"src": "Patient: My mom has a pain on her lower left stomach, she can't stand the smell of food and is not eating well at all. She had a endoscopy and a colonoscopy but they turned out all right. She also has constipation, acid reflex. She has always suffered from constipation, but what i think is happening is she might have kidney s stones. What do you think DR. Doctor: Hi, the cause of the left lower abdominal pain, might be due to the gastric irritation , stones in the ureter at the lower end, constipation, ovarian cyst, fibroids or worms in the intestines etc. So i advise you to consult a surgeon for diagnosis and treatment. thank you."
},
{
"id": 168713,
"tgt": "Suggest treatment for mouth ulcers and itchy blisters on skin in a child",
"src": "Patient: both my daughters age 1 year old and 4 year old have a inflammation in throat with ulcers inside the mouth and on tongue.they have also had fever for over 2 days with itchy blisters on palm of hands, arms, ankles and aoles of feet. The younger daughter also has it in her nappy area. Please advise? Doctor: Hi...I feel by what you quote he should be having a - Hand Foot Mouth disease. This is one viral illness among all other exanthemas which can cause fever followed by rash over palms and soles. It is a self-limiting disorder and itching can be really worrisome. I suggest you use any over the counter antihistamine if you have one with you now. You can use Hydroxyzine at 1-2mg/kg/dose (Maximum 10mg) every 6th to 8th hourly for 7 days. This can even cause some peeling of skin in the next 4-6 weeks and do not worry about it.Regards - Dr. Sumanth"
},
{
"id": 48923,
"tgt": "Is yellow urine a sign of kidney infection?",
"src": "Patient: I am currently being evaluated for a possible lupus diagnosis. My anti dsDna is positive, as well as my anti-histones, and a high positivie ANA. I've been having a slew of symptoms, but I'm most concerned about kidney damage. I noticed I have bright yellow urine almost all the time. It's bright enough that the toilet paper looks neon. I'm not currently taking any vitamins or medications. Is this a sign of kidney damage? Doctor: hai,usually the color of urine is depends on the type of food intake (includes medication).ususally urine color is yellow.unavoided urine for long time will cause variation in concetration as well as the color.since you are in evaluating the Lupus and ANA positive,lupus will cause damage to kidney at certain level. urine routine and renal function test will rule out the kidney condition.thank youhope i answered your query"
},
{
"id": 134291,
"tgt": "Suggest remedy for bumps due to blood clots in the shin",
"src": "Patient: I recently been the hospital for a xray on my shin as I fell down the stairs and hurt my leg they told me I had blood clotted in my leg that had caused a large bump ...now am worried about it because blood clotts are dangerous will this go away on its own ? Doctor: Thanks for your query, I have gone through your query.As far as the blood clot in the soft tissue (hematoma) is common following any trauma. If the clot occurs within the vessels then it is dangerous. Nothing to worry this will subside by its own. You can consult a general physician and get it evaluated once to rule out the above said conditions. Mean while you can give alternative hot and cold fomentation.I hope I have answered your query, take care."
},
{
"id": 96034,
"tgt": "Suffering from abdominal pain with burning sensation in the lower part of belly and cramps",
"src": "Patient: Hi, for the past 4 days i have had a LOT of pain in my stomach, i get a burning sensation in the lower part of my belly and cramps just below my ribs. i have been taking ibuprofen every 6 hours but it only eases the pain slightly, and today it has started becoming painful a few mins after i eat. My stomach is gurgleing constantly. I have been passing wind, and have has regular bowel movements please can you help me?? Doctor: Hi You could be having acid reflux and possible peptic ulcer disease.First stop eating oily,spicy food. Drink plenty of water. Avoid smoking/alcohol. You should never be on an empty stomach for long periods. Start antacids before meals like omeprazole. Dont take ibuprofen before you take the antacids.Always after food. If the pain increases,you need an ultrasound and prescription antacids so please see a doctor."
},
{
"id": 13075,
"tgt": "Need remedy for itchy red rash all over the face",
"src": "Patient: my 8 year old daughter has a red rash all over her face (almost like tiny red bumps all close together). She doesn't have this any where else but she has a cold sore on her mouth and what looks like one on her nose too. She says the rash is very itchy... Doctor: Hello,I read carefully your query and understand your concern. The symptoms seem to be related to a viral rash.I suggest using Cetirizine 10 mg daily to prevent the itching. I also suggest using Calamine lotion for local application. Hope my answer was helpful.If you have further queries feel free to contact me again.Kind regards! Dr.Dorina Gurabardhi General &Family Physician"
},
{
"id": 95778,
"tgt": "Painful jabbing on lower right side of the body",
"src": "Patient: On Sunday night I noticed this really sharp localized pain that would occur when I coughed on the lower right side of my abdomen right south of my navel. The pain itself doesn't feel gastric, instead it feels like the muscle is strained or like there is a bruise in the area. At first the pain only occurred when would cough, but now sudden movements hurt too, and the pain is from my lower abdomen up to the the bottom rib on my rib cage and feels like I have a big bruise in the area. The pain has been lurching to the right and the area is tender to the touch, but it is also fairly mild. I don't have any more symptoms; no fever, no loss of appetite, no nausea, and I really haven't engaged in any activities to bring this on. I was just wondering if I should be concerned about this due to it's sudden nature and the area of pain. Doctor: Hey this looks more like a muscular one rather than something internal. Have you done any strenous exercise and thereby got this pain, then it is confirm. And if it comes suddenly then it could be a muscle spasm but due to intestinal problems as well."
},
{
"id": 134809,
"tgt": "Suggest remedy for severe pain in neck",
"src": "Patient: I have a pain on the left side of my neck. I have trouble turning my head to the left and tilting it back. What might this be? I had an Ultrasound on my Caratoid artery earlier this hear and my doctor said I had a 50% buildup of Hollenhorst plaque. Is this pain related to my Caratoid or something else? Doctor: hi, welcome to healthcare magic.well , i don't think this pain is related to your carotid.you seem to have spasm of your trapezius muscle( stiffness of your neck muscle)please do the following:1. support your neck by using a soft cervical collar2. do hot fomentation by a hot water bag or electric heating pad 3-4 times a day3. take a mild antiinflammatory drug like ibuprofen or acetaaminophen4. avoid driving, reading, using too much computers or video games5. you can also use a cervical pillow for sleepingit will take 5-7 days for complete recovery.please take care.thank you"
},
{
"id": 57482,
"tgt": "Multiple calculi, alternative to surgery?",
"src": "Patient: I had whole abdominal ultrasound last week and the result says multiple calculi are found within, the largest measures 1.9cm. My doctor said I need to undergo surgery , but I am thinking of seeking second opinion. Is there any alternative to break or dissolve these gallstones without going through surgical procedure? Thank you Doctor: gall stones disease is a very common disease,treatment of choice is laparoscopic cholecystectomy.previously they recommend secondary bile acid may help but now srgry is well estsbilished entitity"
},
{
"id": 192920,
"tgt": "What causes soreness and discharge from male breast?",
"src": "Patient: Hi, may I answer your health queries right now ? Please type your query here...im a 15 year ol male my right nipple is bigger than my left and is a little sore when i touch it. when i sqeeze it a little bit of clear liquid comes out. can you explain? i think i have breast cancer and im scared. Doctor: Hi, I can understand your concern about your symptoms. It can be due to hormonal imbalance and increased estrogen levels. Rule out for cancer also. Hope I have answered your query. Let me know if I can assist you further. Take care Regards, Dr S.R.Raveendran, Sexologist"
},
{
"id": 193049,
"tgt": "How to cure frequent urination and pain in scrotum region after excess drinking?",
"src": "Patient: Hi doc.I have a male friend who recently been on a drinking bender for nearly 3 weeks straight. A couple of days ago he feels like he needs to urinate every 5 minutes and when he does it's like peeing razor blades . His balls are also very painfull like they have been rubbed raw but they aren't. He thinks he has an STD and did a test. They also gave him at the time antibiotics for urinary tract infection. Do guys get that? Doctor: Hi, It can be due to alcohol. Rule out for STD panel of test. If it is negative, you can take urine culture and start respective antibiotics.Hope I have answered your query. Let me know if I can assist you further. Regards, Dr. S. R. Raveendran, Sexologist"
},
{
"id": 97569,
"tgt": "What is the maximum dosage of hydroxyzine hcl?",
"src": "Patient: How much hydroxyzine hcl can i take at a time and how often? I suffer from Major panic and anxiety disorder. I was taking 1mg klonapin three times a day for 4 years. They cold terkied me off of it. Suffering from high blood pressure and high panic and anxiety because of this bright move by my mental health provider. My medical doc gave hydroxyzine. Doctor: high panic and anxiety levels have to be tackled first here blood pressure will be taken care of.the maximum calm mind the maximum normal blood pressure.here a combination of chandraprabha along with sarpagandha,kurasani owa.jatamaunshi,brahmi combination should be according to me.shirodhara nad nasya will be of additional benefit.yoga therapy of anulome vilome and kapal bhati will be enhancing the the medicine effect."
},
{
"id": 125107,
"tgt": "What causes chronic swelling from the hips and throughout the legs?",
"src": "Patient: my sister has problem with her leg. it swells from the high down to the toes without pain. she has been in this situation since five years ago. swelling used to occur when she stands or sits for a long time. but now it swells without prolonged sitting or standing. completely she doesnt have pain . it feels heavy thats all. please help! Doctor: Hello and Welcome to \u2018Ask A Doctor\u2019 service.I have reviewed your query and here is my advice.Swelling in the legs after prolonged standing is caused by the mechanism that liquid part of the blood comes out the vessels (veins) and accumulated in the surrounding tissues. It is called peripheral edema.Variety of conditions may lead to peripheral edema. It could be heart problems, venous deficiency, liver or kidney problems or the side effect of certain medications. I would like to ask the following questions which may aid to clarify the cause of her peripheral edema.Does she have visible purple veins under the skin of her legs? Does she have any other health problems (heart, liver, kidney diseases, and high blood pressure)? Does she take any medications? Has she had any trauma or surgery (especially in the lower extremities) in the past?She may get the benefit of wearing elastic compression stockings, elevating leg with the pillow while on a bed, not sitting/standing long time which helps to prevent fluid buildup in the legs.Hope I have answered your query. Let me know if I can assist you further.Regards,Dr. Malik Amonov"
},
{
"id": 217505,
"tgt": "Suggest remedy for pain and fever",
"src": "Patient: am 34 yr old female,wt.45,ht. 130 cm,suffering from pain right hand finger pain at metatarsal jt.,with swelling at rt. index finger,pain in left elbow jt.,pain in both ankle ji.,pain in both sole.,since 3 days no fever,mother having history of seronegative RA,taking pain killers, having difficulty while walkig , kindly suggest treatment Doctor: you can use hot pack to relief pain .moist heat can also used active movement for hand arm elbow and strengthening exercises. take physiotherapy treatment it will help full in stopping conditions to be worsened up ,for swelling elevate with the help of pillow will help to reduce swelling and ankle to movement and static quads that will be helpful for her wish here good health."
},
{
"id": 69075,
"tgt": "What does painful lump at base of wrist indicate?",
"src": "Patient: I have a lump (swollen muscle) at the base of my wrist on the pinky side (flexor digiti minimi brevis?) and it is painful, although I have not lost any use of my hand or fingers. I get sharp pains when using my hand for typing or lifting or even moving it certain ways. I cannot put any pressure on the wrist as in trying to push myself up. Is this something I should go see my doctor about? Doctor: Hi,From your description of what brings on the pain, I think this is a ganglion. This is a swelling in the tendon at the wrist. I suggest you see a hand surgeon for assessment. Surgical removal is the commonest treatment.Regards,Dr K A Pottinger"
},
{
"id": 77480,
"tgt": "What is the cause of cough and breathlessness?",
"src": "Patient: I have been coughing off and on the last 5 weeks and in the past week i have lost my breath choking while eating something soft..hard to catch my breath...i do have and have had allergies all my life..never took anything for them...my doctor gave my an antibiotic and cough medicine in a pill 4 days ago...i dont have trouble swallowing at all,it seems like i get strangled. Doctor: Thanks for your question on Health Care Magic. I can understand your concern. Since you are having allergies, possibility of allergic bronchitis is more in your case. You are having chronic (5 weeks) cough and breathing difficulty which is not improving with routine cough medicines, so possibility of allergic bronchitis is more. So better to get done PFT (Pulmonary Function Test). PFT is must for the diagnosis of bronchitis. It will also tell you about severity of the disease. And treatment of bronchitis is based on severity only. You will improve with combination of antihistamine and anti allergic drugs. Inhaled bronchodilators and inhaled corticosteroid (ICS) are also useful. So consult your doctor and discuss all these. Hope I have solved your query. I will be happy to help you further. Wish you good health. Thanks."
},
{
"id": 83237,
"tgt": "Can facial flush be related to lisinopril taken for hypertension?",
"src": "Patient: I currently take Lisinopril for hypertension. I feel i suffer from something that causes my face to get flushed easily, like I am easily embarrassed. I have noticed when i skip my Lisinopril, I don t have the flushing issue. Can they be related? If not how can I fix the embarrassed look, I feel its taking control over aspects of my life. Doctor: Hi, Based on the history facial flushing (cutaneous vasodilation) seems to be due to an adverse effect of angiotensin converting enzyme inhibitor lisinopril. I suggest you to stop lisinopril and take an alternate effective but a safer medicine for hypertension after consulting your doctor. Hope I have answered your query. Let me know if I can assist you further. Take care Regards, Dr. Mohammed Taher Ali"
},
{
"id": 175955,
"tgt": "What is the difference between cow s milk and Dexolac powder?",
"src": "Patient: Hi Doctor, We are blessed with TWIN GIRLS, they were born in the 30st week (in 7th Month of Pregnancy)..with a weight of about 1.6 Kgs & 1.8Kgs . Doctors advices us to use Dexolac special care milk powder..as mother milk was not sufficient. Now..after 1.5 months after they are born(they are nor more than 1.75 Kg 2.5Kgs), Doctors adviced continue the Dexolac special care. What is the exact difference between cow Milk and Dexolac powders? Can I feed cow milk to both babies. How much quantity should we feed and how many times a day ? We were feeding Dexolac Powder every 2hrs 50ML to 70 ML sometimes babies throw away some milk from their mouth(kind of vomit ) like after 1 hour of feeding..Please answer...Thanks Doctor: YES POST FEEDING VOMITING IN TOP FED BABIES IS KNOWN CONDITION ONLY BURPING CAN HELP THESE BABIES BUT IF BABIES ARE PUTTING ON WEIGHT THEN NOT TO WORRY.BOTH THE MILK ARE GOOD PROVIDED PROPER DILUTION IS MAINTAINED AND SO ALSO AS LONG AS WEIGHT OF BABIES IS SHOWING GAIN NO PROBLEM FEED THEM WITH DEXOLAC OK"
},
{
"id": 72272,
"tgt": "What is the treatment for wheezing problems?",
"src": "Patient: Hello Doctor I am suffering from a wheezing sound that is coming from my windpipe and when I cough a small amount of blood comes up. This only occurs when I get this wheezing sort of gurgling sound in my chest. And most time I feel there is something stuck in my chest. Recently I was in the hospital and they performed a bronchoscopy and gastroscopy however the results came up clean. They also did a tests for TB and bronchitis and these also came up clean. Can you please help me I am really worried. Doctor: Thanks for your question on Healthcare Magic.I can understand your concern. You are having hemoptysis (blood in sputum) and wheezing (whistling sound). Both these are commonly seen bronchospasm. And most common cause for bronchospasm is bronchitis. So I insist you to get done PFT (Pulmonary Function Test) despite of normal chest x ray and bronchoscopy.PFT will not only diagnose bronchitis but it will also tell you about severity of the disease and treatment is based on severity only. You will mostly improve with inhaled bronchodilators (formoterol or salmeterol) and inhaled corticosteroid (ICS) (budesonide or fluticasone).Don't worry, you will be alright with all these. Hope I have solved your query. I will be happy to help you further. Wish you good health. Thanks."
},
{
"id": 137721,
"tgt": "Suggest medication for swelling of the shin area after an injury",
"src": "Patient: 67 year old female, 5 4 175#. 3 days ago, Right shin took a hard hit from cast iron lawn sprinkler a bit higher than midway. Second hit to same area on grocery cart yesterday. Shin swelling mostly down with ice and elevAtion, however, blood pooling both sides of ankle. Swelling getting worse, feeling the pressure more. Is this something that needs to be seen, or just more ice and elevation? Doctor: Hi,Thanks for your query.From description it seems to be soft tissue injury . Swelling might be due to haematomaformation(collection of blood).You need an x-ray to rule out any bony injury. I suggest you to consult an orthopaedician nearby for a detailed examination. You may get the X-ray of the area affected under his/her guidance. Beside icing and elevating it, you can take anti-inflammatory drug like motrin to reduce pain and inflammation.I do hope that you have found something helpful and I will be glad to answer any further query.Take care"
},
{
"id": 199327,
"tgt": "Suggest remedy for severe itching in testicles",
"src": "Patient: I had applied savlon undiluted liquid under my testicles as I had itcjing problem. After applying I am having severe burning sensation. The underwear toucing the skin is aggravating the probelm more. Please suggest corrective action to overcome this problem. Thanks Doctor: HelloThanks for query.You have developed allergic dermatitis of scrotal skin after applying concentrated Savlon.Please wash your scrotum with warm water and use topical Steroidal Cream twice daily .This will help to control the itching and the irritation and heal up the lesion.Dr.Patil."
},
{
"id": 179642,
"tgt": "Why is the color of the stools green and appear smelly?",
"src": "Patient: My daughter is 3.5 old. She is adopted girl since the day 2 she is on lactogen 1. She is passing green and smelly stool twice a day since last 10-12 daya. We have did her stool routin test. Her report say mucus present (+) reducing substance positive (trace). She is on simyl mct since yesterday. But she is not enjoying it. Pls suggest. Mrs. Huda Doctor: Hello Is she having loose stools , if stools are formed , she is passing urine every 3-4 hourly , active, accepting feed , you need not worry for colour of stools, if there is no presence of blood.Thanks"
},
{
"id": 113381,
"tgt": "Slipped disc in S2-S3. No relief with yoga, massage, painkillers. Hurts when coughed, sneezed. Solution?",
"src": "Patient: Hi there. I am only eighteen years old and i sliped a disc in my s2-s3 area of my back OVER A YEAR AGO! I have stopped taking pain killers after throwing up my stomach lining and to this day have not had a full night of sleep. I do a lot of yoga , stretching and general back/ pain exercises but nothing seems to help. I have lost faith in physio and chiropractors also and cannot afford massage so my muscles are tight with little direction. When.i cough and sneeze my back tenses on the nerve and i feel extreme pain down my left thigh. Any help you can suggest would be apprechiated. Thankyou! Doctor: Hello, nothing to worry we can work your case by homoeopathy send us the reports of x ray, also let us know if you have pain any of lower limbs with numbness , can you easily bear weight on both limbs easily, do not yoga or exercise which are not recommended by physio, the problem completely solved with proper case taking and letting us know your reports"
},
{
"id": 130672,
"tgt": "What does an indentation over the sacral area indicate?",
"src": "Patient: Hi..my sister noticed a small indentation over the sacral area on the gluteal cleft 4 days ago. There is no mass of hairs or cyst over the indentation. It is a well circumscribed non inflammed indentation. Should we go and seek for medical attention to see if it is related to any bony deformities/spina bifida/ renal problems etc?Thank you in advance Doctor: Hi,If this dimple is painless and doesn't increase in mass I wouldn't worry. Usually they don't need any medical treatment. Rarely, sacral dimples are associated with a serious underlying abnormality of the spine or spinal cord. Examples include: Spina bifida. A very mild form of this condition, called spina bifida occulta, occurs when the spine doesn't close properly around the spinal cord but the cord remains within the spinal canal. Hope the above information helps you. Let me know if I can assist you further.Regards,Dr. Ahmed Aly Hassan"
},
{
"id": 134512,
"tgt": "What causes swelling and inflammation in thumb?",
"src": "Patient: Hi I had what I thought was a bit midway between my elbow and wrist. I suffered severe swelling in the thumb and inflamation in thumb and the veins were distended. Treated with two lots of keflex subsided but not completely about a week ago pain is back again although not as severe as is swelling also not as severe Doctor: I think if you have tried a self medication it is never advisable. kindly meet a physician close by and get it chekced. Mean time you can try contrast bath starting with hot water and ending with hot water. Try performing some small finger strengthening exercises by using a tennis ball for strengthening the thumb muscles. you should be fine if done properly."
},
{
"id": 84384,
"tgt": "How effective is antibiotics to destroy enterobacter?",
"src": "Patient: i had a complication from gallbladder surgery and was told i had the start of an abscess and was rushed to surgery the day after my gallbladder was removed. they said the culture grew enterobacter. i m on heavy doses of antibiotics. my question is do the antibiotics work and will this ever go away? Doctor: Hi,It depends upon which antibiotics you are taking. Since you did not mention the name of antibiotic prescribed by your doctor it is very difficult to answer your question. However, if your doctor has prescribed the antibiotic based on the culture and sensitive report they usually work and thus help in eradication of the infection.Hope I have answered your question. Let me know if I can assist you further. Regards, Dr. Mohammed Taher Ali, General & Family Physician"
},
{
"id": 11394,
"tgt": "Suggest remedy for rapid hair fall",
"src": "Patient: hi doc, i am a boy, age 25, i have been suffering the excessive hair fall and Dandruff problems. i have been using different types of shampoos but it couldn t stop the hair fall & Dandruff completely. For this excessive hair fall I am getting Bald. For this i look like 7-8 years older than what i am. So please give me a effective solution. Doctor: Hello dear,Taking into consideration the case history as provided in your post, the complaints of hair fall is related to dandruff.So, the problem of dandruff needs to be addressed before initiation of treatment for hair fall.Dandruff is basically Seborrheic dermatitis & can be treated with shampoos or lotion containing salicylic acid, coal tar, ketoconazole, or selenium.The causes of hair loss/thinning of hair are:1. Environmental factors2. Stress3. Deficiency of certain minerals, vitamins & amino acids in diet. e.g. zinc, Vit B complex, lysine.4. Any incidence of surgery, shock, etc.Management includes:1. Treating the underlying cause2. Avoidance of stress3. Dietary modifications4. Minoxidil solution to be applied locally to stimulate the hair follicles5. Multivitamin & mineral supplementationSo, it will be better to consult a Dermatologist to find out the underlying cause & start treatment accordingly.There is no need to worry. Your hairs will grow back again.Wishing you a Good HealthTake care."
},
{
"id": 188208,
"tgt": "Have pain and bump in lower lip. Using braces. How to avoid this?",
"src": "Patient: Hello I was a the the orthodontist and I was getting fitted for braces and the orthodontist stretched my lip very far and then all of a sudden I felt this pain in my lower lip. After I was done I looked I'm the mirror and I had this big red bump on my lip. It's not a pimple though because it's on my lip. What should I do to get rid of it? Doctor: HI, thanks for using healthcare magicThe trauma that you experience to the lip would have caused an inflammatory reaction that would cause the swelling or bump that you are seeing.It will resolve on its own. The use of anti inflammatory pain killers such as ibuprofen, cataflam, aspirin, naproxen, celebrex, may speed up the process .I hope this helps"
},
{
"id": 160736,
"tgt": "What causes chest pain in children?",
"src": "Patient: Hello, im 14 yrs old, and since i came back from holiday i was like very ill and after being ill all these days my vision annoys me like it starts to blurr in times alot,and also i have asthma so i think my asthma has worsened kinda iswell but i use my pumps and today i feel like vomiting so in school after lunch i felt sick like feel like throwing up and in class i drank water so i started getting a headache iswell, had paracetamol and it kinda went off, but i felt sick even more an after school, while i was walking home my vision blurred alot and i had a headache again and still do now but when i got home my chest started paining so i took one inhaler (green one) and 10 mins later its started paining again so i took my other one (brown inhaler) and its still pains and like my chest is kinda tight feeling iswell, my dad says he will take me to the doctors, but i want my chest pain to go..please tell me..what can i do. Doctor: Hi,You are feeling ill with headache, vomiting, dizziness and chest pain- This can indicate prodrome of a viral fever or gastroenteritis as you are coming back from holiday. Chest pain may be a manifestation of worsening asthma, although it usually causes dry cough and breathlessness too, and will get relieved with your inhaler.You can take paracetamol to relieve chest pain. Ibuprofen will give more relief but it may cause worsening of asthma and hence better avoided. Meanwhile take bed rest, have frequent fluids and a soft diet.Hope I have answered your question. Let me know if I can assist you further. Regards, Dr. Muhammed Aslam TK, Pediatrician"
},
{
"id": 192184,
"tgt": "Suggest treatment for hanging testicles",
"src": "Patient: hiii sir my name is manjeet i\"m suffering from very serious problem from around last one year my both testis were always hanging and my left testis is hanging more than the left one. please help me sir ,tell me what should i do and its tretment is possible or not . Doctor: Hello, Hanging testicles are not a serious problem unless you consult a doctor in person. Do consult a doctor. Therefore, I suggest consulting an urologist for physical examination, diagnosis and treatment. Take care. Hope I have answered your question. Let me know if I can assist you further. Regards, Dr. K. V. Anand, Psychologist"
},
{
"id": 115757,
"tgt": "How long does Diltiazem take to improve Raynaud's disease symptoms?",
"src": "Patient: I have Raynaud's which is not primary it would be secondard because of schlederma. I recently started on diltiazem 30 mg. 3 times a day. My question to you is how long do you have to take the dose before you will see improvements with the raynaud's Doctor: Hello,I understand your concern.I am Dr. Arun Tank, infectious diseases specialist, answering your query.Dilitiazem is vasodialotor drug.It causes direct improvement in its vaso dilatation.It shows it's effect until you continue with the drug. Once you stop the drug its effect will gone.I advice you to take the drug when you need it most. So not continue the drug because its effect will gradually diminished over the time.As you have secondary disease, if scleoderma improves Raynaud's to will improve.I will be happy to answer your further concern, you can ask me on bit.ly/DrArun. Thank you.Dr Arun TankInfectious diseases specialist."
},
{
"id": 219666,
"tgt": "what causes palpitation and dizziness during pregnancy?",
"src": "Patient: Hi, may I answer your health queries right now ? Please type yo \u00a0\u00a0\u00a0\u00a0\u00a0\u00a0\u00a0\u00a0\u00a0\u00a0\u00a0\u00a0\u00a0\u00a0\u00a0\u00a0\u00a0\u00a0\u00a0\u00a0\u00a0\u00a0\u00a0\u00a0\u00a0\u00a0\u00a0\u00a0\u00a0\u00a0\u00a0\u00a0\u00a0\u00a0\u00a0\u00a0\u00a0\u00a0\u00a0\u00a0\u00a0\u00a0\u00a0\u00a0\u00a0\u00a0\u00a0\u00a0\u00a0\u00a0\u00a0\u00a0\u00a0\u00a0\u00a0\u00a0\u00a0\u00a0\u00a0\u00a0Hi Im 32 weeks pregnant. Its 4 in the morning and I am very dizzy my heart is racing my head is pounding and i feel like throwing up! I have Gestational Diabetes but my Blood sugar is 91. It could be high blood pressure. should I go to the ER or wait and call my dr in the morning? Doctor: You should go to your delivery hospital immediately. With your history of gestational diabetes, you are at increased risk for preeclampsia and eclampsia, which can be deadly for you and baby. Preeclampsia is a disease unique to pregnancy- you typically see elevated blood pressure with protein in the urine and a bad headache. Eclampsia is preeclampsia plus a seizure. Seizures in pregnancy can significantly damage your blood supply to your brain and your baby. Please go get evaluated immediately. Good luck!"
},
{
"id": 45929,
"tgt": "Suggest treatment for sinus and kidney infection for a C-diff patient",
"src": "Patient: My Mother has C-Diff and can no longer take most antibotic so the question is how do we treat small infections like a sinus infection or a kidney infection. I know the one antibotic she can take is Flagaral but if she takes it to much will it help her when the c-diff acts up again. Also how contagious is it when she isnt sick with it. Doctor: Hello and Welcome to \u2018Ask A Doctor\u2019 service. I have reviewed your query and here is my advice. Clostridium difficle infection occurs when the normal good bacteria in our gut is killed by too many antibiotics and bad bacteria grow too much. To replace the normal gut bacteria she must eat lots of CURD and yogurt. Daily stop all antibiotics and use probiotics like sporolac etc. Only overuse of antibiotics will lead to this condition. For sinus infections do use steam inhalation, nasal decongestents and for urine infections ask her to take plenty of water, avoid postponing urination and personal hygiene. She can be given antibiotics like tab nitrofurantoin for urinary infections which usually doesn't cause c difficle diarrhea. That too take upon doctors prescription and after urine culture test. Hope I have answered your query. Let me know if I can assist you further."
},
{
"id": 47705,
"tgt": "Still having noticed stone in kidney & blood in urine",
"src": "Patient: Hi I had surgery to remove stone in kidney but instead after I found out stone is still there and doctor out a stint in I have been in a lot of pain my husband took me to his office only to find out nothing can be done right now stint can t be removed yet. I still have blood in urine and I couldn t use the bathroom I even tried suppository nothing then I started throwing up and swelling the dictir said it was normal sometimes I m scared something isn t right today I have been in pain especially in my back and I can t eat because my mouth and throat is so messed up from vomiting so much one more thing my mouth is so dry water doesn t help the more I drink the more dry my mouth gets Is this all normal Doctor: HelloThanks for query .Your husband has stone in the kidney and urologist has planned enoscopic surgery to remove the stone but could not remove the stone may be because of some technical issues and has placed stent in his renal unit .Now he has psvere pain and blood in urine and vomiting .In a given situation iI would suggest you to get him admitted in hospital and treat him on following lines 1) I V fluids to maintain fluid and electrolyte balance 2) I/v antibiotics to prevent infection 3) Pain killers 4)Drugs to control vomiting The further surgery for removal of stone can be planned after he fully recovers from this episode .Do not worry these things do happen once in a while and here is nothing to be worried about it .Dr.Patil."
},
{
"id": 81035,
"tgt": "Having dry cough, chest pain and on and off fever. What does chest examination suggest?",
"src": "Patient: hello sir,a 70y/f housewife ,compaints of dry cough 2 months and chest pain(back of th chest) 2 months, history of fever on and from last 2 months and also has a history of weight loss.pet exposure is there. chest examination: B/L decreased air intensity and creapts on the lower part of the chest. x ray chest:a homogenous opacity with air bronchogram on rt middle and lower zones. please tell the dd.thanks Doctor: Thanks for your question on HCM.I can understand your situation and problem.By your symptoms and x ray finding, following are the possible causes.1. Bacterial pneumonia2. Tuberculosis3. Pleural effusion.So better to consult pulmonologist and get done1. Sputum examination for AFB to rule out tuberculosis2. Sputum culture and sensitivity to find out causative organism for pneumonia.3. Ultrasound examination of chest to rule out pleural effusion.4. Bronchoscopy with BAL (Bronchoalveolar lavage) analysis if sputum production is not there.So infective etiology appears more in your case.Better to consult pulmonologist and first diagnose yourself and start appropriate treatment."
},
{
"id": 186592,
"tgt": "Suggest cause and medications for tooth infections post smoking",
"src": "Patient: allergic reaction after tooth pulledi got a tooth pulled a few weeks ago that was really infected bad. ive been on antibiotics for the last 2 months so the infection is pretty much gone out my body but now when i smoke i get an allergic reaction and hace to take benadryl and rinse my mouth and anything else to stop it. ive never been allergic to anything besides pollen and stuff like that. why would this be happening? Doctor: when the tooth was pulled out was any minor surgical procedure done.it seems that the extraction socket has not healed completly. your smoking habit is aggravating the problem. i suggest you to immediatly stop smoking and visit a dentist to get an x ray of full mouth (Orthopentamograph) to reach to a conclusion."
},
{
"id": 57132,
"tgt": "What could cause positive hepatitis B after taking vaccination?",
"src": "Patient: sir, my family (my self wife son ) taken 3 doses of hepatitis b vaccination. After some time when i was sick , had blood test and my self tested +ve for hepatits b hbs ag but my wife and son are -ve and my hbe ag is -ve and hcv -ve what is my condition . if i had sex with a lady is there any chance of transmitting the virus Doctor: Did you check for hepatitis B before vaccination? In all probability you were already positive at the time you took the vaccine. You need to check Hepatitis B DNA viral load. If it is high you might transmit the infection during sexual contact. If your sexual partner is vaccinated then the has negligible chance of getting the disease."
},
{
"id": 201359,
"tgt": "Will pubic hair cause pimple on penis?",
"src": "Patient: Sir I am married and was having some pimple kind over my penis. It was thee for a month and now it is giving me some white fluid from that. I visted a doctor and they did some test related to VDRL etc and result was fine. They gave me a fuscidic acid oinment and i applied it. They told that it was due to hair growth in that place. Doctor: Hi,It seems that there might be having some ingrown hair follicle infection or yeast infection.Apply fucidine cream or anti-fungal cream.Keep local hygiene clean, dry and airy.Keep part well shaved.Ok and take care."
},
{
"id": 219349,
"tgt": "Does frequent blood tests harm the fetus during pregnancy?",
"src": "Patient: Hi, I am 12 weeks pregnant and I had my first scan last friday - they also took blood again as the first lot of blood came back saying I had antibodies in it - I am now really worried as the midwife called me to ask me to come in tomorrow to give more blood - is my baby at risk? This is 3 Times I have given blood in the last 2 weeks and I don t think it is normal - the midwife said on the phone last night that it was nothing to worry about but Im getting slightly paranoid now Doctor: Hello and welcome to healthcare magic.Let me assure you repeated blood tests WILL NOT HURT YOUR BABY.If I were you I'd be more concerned about what are the blood tests that are being done.I would have liked to have few more details like your age, history of previous pregnancies (if any), your blood group etc to give you a more detailed answer.Please do get back if you have any more questions."
},
{
"id": 129943,
"tgt": "Why do I have a crawling sensations in legs and triceps?",
"src": "Patient: having crawling sensations in legs and triceps, mostly when I lay down at night or try to relax on couch..taking a muscle relaxer called tizanidine 4mg. only at bedtime to sleep better..Tizanidine was prescribed by a pain spec. for my lower back .. when crawling sensations happen have to really stretch good to slow them down...if I lft anything fairly heavy, I know within a day or so I will have stiffness and crawling sensation.. I drink plenty of water and try to stay away from caffeine ...driving me nuts Doctor: Hi..Your issue is due to tired muscles as well...I recommend you to go and work for improving endurance if your muscles..You need some salts in your body which could get replaced with ors...Stay away from heat...and keep your core temperature Nirmal as possible...Take warm shower at night before sleep as this will help to relax your muscles...Drink plenty of water..Take good care...protein diet..Hope this is helpful for you..Kindly revert back in case you need any further help in this regards..."
},
{
"id": 92545,
"tgt": "Abdominal pain, aggravated after meals, tiredness, bloating, bruised feeling inside stomach. Blood test shows high bile level",
"src": "Patient: For the last 4 weeks I have been suffering with abdominal pain. It is aggravated by eating pretty much all food so I am not eating a lot which is very draining and making me very tired. I feel very bloated and the inside of my stomach feels bruised. The pain is generally in my upper stomach but spreads to the sides and up my back. I have had blood tests and my bile level came back a little high. I also had an ultrasound to check my gallbladder but it turned out to be fine. Doctor: Hello, I have gone through your query and understood your concern. In my opinion, the symptoms you mentioned above indicative of peptic ulcer disease and area could be near lower one third of your stomach. If you were my patient, I would suggest you to take Tablet Nexpro 40 mg for 2 weeks and it should be effective. But also confirm the diagnosis by consulting a gastro enterologist. Hope this helped you. Thanks."
},
{
"id": 61609,
"tgt": "Suggest treatment for painful lump in the breast",
"src": "Patient: Hello,Our patient has lumpiness in both her breast with pain for last two months. Now we've gone through some test in lab and waiting for the reports.Her right sided breast lump becomes more stiff than the left one. She is afraid whether it's a tumorous or not!!How can we get an expert advise to take the right decision to the next step??Thanks for your quick response on this.regards,Nazmul/Bangladesh Doctor: Please go for mammogram followed by fine needle aspiration cytology ( fnac ). An expert surgeon may make a diagnosis by clinical examination to some extent."
},
{
"id": 162398,
"tgt": "What causes enlarged heart in a child?",
"src": "Patient: My 10 year old boy was having fever 3 days back and the dr gave him a course of antibiotic. today is the 4th day and his fever was still 39 and is having a very bad cough. I took him again to the Dr. today who sent him for an X-ray to check the lungs. The Dr. then said that she found that the size of the heart is big for his age and said will send the x-ray to the cardiologist to view it. I am very worried now and would like to know what might be the resons for having enlarged heart. Doctor: Hi, From the scenario you describe I feel that this could be a viral myocarditis which is acute in onset or the other possibility is that of a longstanding dilated cardiomyopathy which got aggravated because of the viral illness. But whatever be the cause your child will require immediate cardiologist consultation, echocardiogram and if the ejection fraction is less then he might even require hospitalization. I am sorry to say this. Hope I have answered your query. Let me know if I can assist you further. Regards, Dr. Sumanth Amperayani, Pediatrician, Pulmonology"
},
{
"id": 6002,
"tgt": "Trying to conceive, off birth control, having unprotected sex, got periods. Could i have ovulated during my period?",
"src": "Patient: I am TTC. I got off birth control in the middle of the pack on April 23rd. I spotted for a few days and then got my period April 26th and it ended May 1st. I have been taking ovulation tests since April 30th and there all comming up with a 2nd line but its fainter than the control line . The pst 2 days i have gotten no lines at all now. Did i miss ovulation maybe? Could i have ovulated during my period? I have been having unprotected sex. Doctor: Hello. Thanks for writing to us. Since you left taking the oral pills in the middle of the cycle, the chances of ovulation are very low. The faint line that you are getting on the test is likely to be false positive test. You need to try again from 12th day onwards. I hope this information has been both informative and helpful for you. Regards, Dr. Rakhi Tayal drrakhitayal@gmail.com"
},
{
"id": 198387,
"tgt": "How can I treat Oligospermia with medication?",
"src": "Patient: Hello sir, I am 30 years old mail, married.. i have conserned doctor .. my lab test reports are shown a case of 1) oligospermia Doctor has said that i have to take tablet for 2 months compulsary has to done .. I dont want ... I want to control with tablets if possible..Please advise me what should be done to control this... Doctor: HelloThanks for query .Based on your statement Your semen analysis has revealed low sperm count Oloigospermia .Following measure will help increase sperm count 1) Practice regular exercise for 45 minutes followed by meditation for 1/2 an hour in the morning.2) Take high protein diet rich in vegetables and fruits and Vitamin A,C,D,E.and Zinc3)Take anti oxidants like Almonds 5-6 everyday..4) Avoid alcohol and smoking..Taking Clomiphene on alternate day for 4 months helps to increase sperm count .Dr.Patil."
},
{
"id": 97054,
"tgt": "What causes pain after butt cheek injury?",
"src": "Patient: Hi, several years ago I had a bicycle accident and fell on my right side with my right butt cheek taking the brunt\u2026 my entire cheek was black for weeks. it has since healed with no pain until recently. since then my right cheek was somewhat de formed due to the accident. Now when i sit for long periods my but on that side starts to hurt and a slight tingling in my right leg happens. should i be concerned Doctor: Hi thanks a lot for your query....i would like to clarify to you that this injury is very common in falls.this can cause pain in leg below few years later as in your case.this is generally referred to as \"SCIATICA\".this is due to undue stretching of the long sciatic nerve in the buttocks...THIS CONDITION CAN BE DIAGNOSED CLINICALLY.NERVE PROTECTING AGENTS AND NERVE ANALGESICS LIKE PREGABALIN.FOOT AND LEG EXCERCISES Can be helpfull.its better if you undergo NCS-NERVE CONDUCTION STUDIES..TO ASESS THE DEGREE OF NERVE INJURY AND ITS CHANCES OF RECOVERY PRETTY ACCURATELY.IT CAN ALSO GUIDE THE SURGEON TO DETETMINE THE DEGREE OF NERVE OPERABILITY.IT SUGGEST YOU MEET AN EXPERT ORTHOSURGEON FOR FURTHER QUERIES.THANK YOU"
},
{
"id": 2805,
"tgt": "What is the purpose of this medication?",
"src": "Patient: My doc prescribed the following tablet... i have a SIMPLE cyst in my left ovary... will upload the reports... For the time being kindly tell me the purpose of below mentioned tablet... 1)Folfit-1-0-0 ( once a day) 2)fertisure F-1-0-1 (twice a day) can i concieve this time Doctor: Hi,Both folvit and fertisure F will provide vitamin and micro nutrient required in pregnancy. It will help to become pregnant, form quality egg, maintain reproductive health and weight, decrease insulin resistance. If your period is not regular then following drugs will be useful after complete evaluation like reproductive hormone analysis, USg\\G scan and ovarian follicle study. Clomiphene for maturation of follicle, progesterone in second half of period to support implantation and pregnancy. Metformin for induction of ovulation and insulin resistance. Avoid stress, Do sex in fertile phase of period, drink plenty of water, maintain proper pelvic hygiene and do regular exercise. All the best.Hope this may help you. Contact further if follow up needed.Best regards,Dr. Sagar"
},
{
"id": 119987,
"tgt": "Suggest treatment for swollen lymph in groin",
"src": "Patient: I am getting an ultrasound for swollen lymph in groin. My leg has been aching with shooting pain on and off. I thought it was a hyrnia, but the Dr. thinks it is a swollen lymph node less than a centimeter. Can this cause so much pain in my leg where i have to limp sometimes? Doctor: Hello, Most of the time less than one centimeter swollen lymphnode in groin does not cause pain in leg.Shooting pain on and off if more on bending it should be differentiated from sciatica. Hope I have answered your query. Let me know if I can assist you further. Take care Regards, Dr. Krishnakanth k."
},
{
"id": 141973,
"tgt": "What does this brain MRI test result indicate?",
"src": "Patient: Can you translate this diagnosis in layman terms please? Brain MRI: Mild brain volume loss and associated mildly diffuse bilateral supratentorial chronic small vessel ischemic changes without superimposed acute infarct, mass, or intracranial hemorrhage. Doctor: Hello!My name is Dr. Aida and I am glad to attend you on Healthcaremagic!These brain MRI findings are suggestive of chronic changes to the brain small vessels and brain aging. They are not indicative of any serious medical condition. These changes are usually related to aging and uncontrolled hypertension, diabetes or dyslipidemia. I would recommend treatment with daily aspirin 80mg if no contraindications. If the patient has memory problems, it would be necessary consulting with a neurologist and performing some cognitive tests to investigate for possible dementia. Hope you will find this information helpful!Kind regards!"
},
{
"id": 221066,
"tgt": "What are my chances of getting pregnant?",
"src": "Patient: ok hi am 26years old.had a 5yrs old baby boy.anyway i had a DNC done in this june 2010 for missed miscarriage.i really want to have another baby.so my first cycle after DNC was in 22 july.so we had intercourse on 28th and 30th?my question is what are the chances of getting pregnant??or if we have intercourse on 5th or 6th august..is it possible to get pregnant??i have 28 days cycle..thanx Doctor: Hi there,Usually the first cycle after the miscarriage is not ovulatory. So we cant really predict that you will get pregnant right away or not.It is possible, as some women do get pregnant right way, but I would advise you to wait at least for one period before you try to get pregnant. The body will recover from the miscarriage and you will be better prepared for the pregnancy.Take folic acid and eat healthy.Hope this helps.Regards."
},
{
"id": 65745,
"tgt": "Suggest treatment for swollen glands behind ear",
"src": "Patient: hi i have swollen glands on the back of my head behind my ears and on one side of my neck i just noticed one growing under my arm i am very worried. they dont hurt the one on the back of my head and by my ears are hard as rocks i feel fine what could it be Doctor: Hi! Good morning. I am Dr Shareef answering your query.If I were your doctor, after a general physical examination of yours, I would advise you for a FNAC (fine needle aspiration cytology), and an excision biopsy if need be for confirmation of the diagnosis of the FNAC. One could plan out a treatment only after getting the cytology/ biopsy report along with the reports of some routine blood tests.I hope this information would help you in discussing with your family physician/treating doctor in further management of your problem. Please do not hesitate to ask in case of any further doubts.Thanks for choosing health care magic to clear doubts on your health problems. I wish you an early recovery. Dr Shareef."
},
{
"id": 68368,
"tgt": "What could a distinctive lump below the ear (anti-helix) indicate?",
"src": "Patient: Hello, I have a hard lump in my ear, just under the skin/cartilage of the concha or anti-helix. It seems to be a lump either on the cartilage or on the bone underneath it since I can hear them rubbing against each other when I move my ear. The sound is distinctively different from that which I hear when moving the other ear and if I push down on the ear at all hard, there is slight sensitivity in the place where I am guessing the bump is, or whatever it is that causes this sensation. I noticed it some months ago, perhaps over a year ago, I can't remember when. I didn't think much of it at the time but since it is still there and seems to be more noticeable (i.e. painful) now, I thought I would find out about it. Doctor: Hi, it could very well be a bony lump. But you should see your Doctor and take his help to rule out a Parotid gland swelling. This can be done by doing an Ultrasound along with an FNAC if it does not come to be a bony swelling on Ultrasound. Do get it done. Take care and feel free to ask any questions.Dr Rishi, New Delhi, India."
},
{
"id": 9465,
"tgt": "What is the treatment for moniliasis?",
"src": "Patient: Hai Doc., skin on my penile shaft is dry, rough and wrinkled. tip of the penis is itchy sometime. i had unprotected sex once and its been while a six months ago. small red sores like on the penile head are appearing once in a month. pimple like appearing on my face and body some time and more importantly my skin is getting dry and shiny on face, legs and hands. i went to dermatologist, tested negative for HIV, he diagnosed it as Moniliasis (yeast infection) and recommended ammoxycilin and dicloxicin. but i didn't find any best result. i would like to have your suggestions or advice. please its very urgency. Doctor: Hi,You seem to have candidosis. It may be called candidal balanoposthitis. It is caused by Candida albicans. You consult dermatologist for firm diagnosis. DM should be ruled out by testing the blood.You may take antifungal like itroconazole cap once a day for two weeks or ketoconazole tab once a day for two weeks. If there is itching, you take levocetirizine. Apply antifungal cream like clotrimazole or miconazole. Take a Sitz bath in dettol water. For other problems, you discuss with your dermatologist and take treatment accordingly. I hope you got my answer.Thanks.Dr. Ilyas Patel MD"
},
{
"id": 216496,
"tgt": "What causes pain and soreness in left side of the stomach?",
"src": "Patient: I got in a car accident today and i don t really want to go to the doctors because i feel just fine. I went to softball practice and everything. My only problems is my left side of my stomach kind of hurts if i bend over or push on it. Also, when i went to the bathroom my stool was red-looking. There was no blood or anything. I really don t want to go to the hospital but i also don t want to have a bigger problem and not do something about it. Any easy suggestions as to what might be wrong? Maybe its just from the impact or airbag or something? I hope its not a bigger issue Doctor: Thank you very much for contacting healthcare magic.. You have problem of left side abdominal pain and blood in stool that may be because of intestinal infection like ulcerative colitis. Diverticulitis may ruled out. Stool examination test is helpful for diagnosis. Colonoscopy can done for further diagnosis. Peptic ulcer may cause blood in stool and abdominal pain. Proper physical examination and history give exact idea. Consent a physician for further diagnosis. I hope my guidance is helpful to you. Take care. Thank you."
},
{
"id": 73167,
"tgt": "Suggest cure for a respiratory problem",
"src": "Patient: Hello, I have a respiratory problem, I have a \"cold\" since 3 days now, no temperature, just dry cough and lack of energy, my nose is flowing.I took paracetamol one for 6 hours than since today i started to take 2 for 6 hours but not a change. I also took vitamin C 1000mg effervescent but I stopped since it was giving me headache . I am also Hiv positive but I am in a country where I cannot get specialized help so I was wondering if it is ok to take antibiotics and which ? thank you Doctor: Thanks for your question on Healthcare Magic.I can understand your concern. Possibility of viral upper respiratory tract infection (URTI) is more likely.So take following drugs. 1. Azithromycin 500mg once a day after breakfast for 5 days.2. Levocetrizine 5mg twice daily after food for 5 days.Along with drugs, also drink plenty of water, do steam inhalation and warm water gargles 5-6 times a day. You will mostly improve with all these in 5 days.If not improving then consult your doctor. Hope I have solved your query. I will be happy to help you further. Wish you good health. Thanks."
},
{
"id": 56178,
"tgt": "When should i go to retest as my sgpt and sgot are 90 and 77 respectively?",
"src": "Patient: hii,my sgpt is 90 and sgot is 77 i have been drinking alcohol for the last 10 years but 2 years before they were normal ,my gall bladder is also removed now have stopped drinking and taking udiliv and liv 52 daily from a week when should i go for retest Doctor: Hello, I would like to tell you that because you were chronic alcoholic and you had gall bladder problem, your liver could be damaged somewhat and that reflects as high SGPT and SGOT. But now you have stopped drinking and removed your gall bladder, also taking udiliv and liv, so liver tissue will start to recover. In initial 3 months or so, you can have somewhat high liver enzyme levels, so i would recommend you to repeat the liver enzyme tests after 3 months of treatment and if it remains still on the higher side, then i would suggest you to go for ultrasound to find out the extent of injury and discuss with your physician. So don't worry, take enough precautions like stop drinking alcohol, avoid fatty foods and take healthy diet, take medications, then your liver enzymes will be normal after 3 months or so. Hope this answers your question and feel free to ask any further questions."
},
{
"id": 144283,
"tgt": "What causes tingling sensation in arms,legs and face?",
"src": "Patient: I am having tingly arms and legs which sometimes become aching. Also get tingly feeling in my face. Alse my tongue feels funny and sometimes my throat feels like it has pressure on it and makes hard to breath.any ideas? Does not always happen but did happen for about a monthnon stop Doctor: well though your symptoms seems to be vague it is always good to rule out some things. we must know your age and diabetic status before we investigate into your symptoms. if you are young and non diabetic then nothing to worry about much , just have to focus on your nutrition rich in vitamins, regular exercises and exposing yourself to sunlight in the morning. and cut on your junk foods! if you are a middle aged person with or without diabetes and if these symptoms are troubling you very often you must visit a neurophysician or a neurosurgeon and get investigated accordingly.might have to get your nerve conduction studies and other relevant tests ."
},
{
"id": 134067,
"tgt": "What do I do for butt cheek pain after falling?",
"src": "Patient: I slipped on some wet cement tiles today and landed on my left butt cheek, not my tailbone, and now I am in a lot of pain. I know you can t break your butt cheek, but what would cause so much pain. It is not bruised, it just hurts a lot. It is hard for me to walk and I can t sit on that side of my butt. My question is what could be causing so much pain? Doctor: Hi,Thank you for providing the brief history of you. As you mentioned the mechanism of injury we should understand that when ever there is a fall, there might be two things, either muscular injury or either a bony injury.Undergoing an MRI of the Lumbar spine will help provide some input. Sometimes due to fall there are chances that you may damage the soft tissue of the lumbar spine and also the bony alignments are disturbed. The reason for this is the whole load is borne by the lumbar spine as there is a tremendous thrust on the hip region due to fall. Most cases landing on the hip region during the fall, land into the injury of the lumbar spine and get misdiagnosed due to improper assessment plan. I will recommend to undergo an MRI of the lumbar spine to get assess further into the depth of the issue.RegardsJay Indravadan Patel"
},
{
"id": 104071,
"tgt": "Had inquinal hernia surgery, penis and scrotum got red, itchy, swollen. Allergy?",
"src": "Patient: i Just had inquinal hernia surgery 4 days ago. On day 2 my Penis and scrotum got red and itchy and now my penis is extremely swollen and red. And my penis and scrotum itch. The urgent care doctor said it was an allergy of some kind, but allergy medicines are not helping and the penis is still very swollen and uncomfortable. Doctor: the medicines used during surgery antibiotics painkillers paracetamol brufen vitamins dietry suppliments or antibiotic any medicine can cause this it will take 3 - 4 week after stoping all the medicines used after and during surgery which caused allergy reaction even if the antiallergics are taken they can take 3 wk drink lot of warm water to flush out the effect of drugs causing allergies take fexofenadine 120 mg morning and tab atarax 10 mg at night apply quardiderm ointment over the affected area 3 - 4 times in a day\\ keep area dry and airy"
},
{
"id": 31681,
"tgt": "Suggest remedy for water infection",
"src": "Patient: I started off with what seemed like a water infection, weeing all the time with blood in it, I went to the docs and he gave me antibiotics, but the day after this I started getting cramping/pains in the bottom right of my stomach and it is really uncomfortable and I have now had diahrea, the doctor has given me some more antibiotics higher strength and there is still no difference, please help Doctor: HI, thanks for using healthcare magicIt is possible that the diarrhea and lower abdominal pain are side effects of the antibiotics. Antibiotics can commonly cause diarrhea.Using probiotics with the antibiotics can help with this side effect.The pain can also be a consequence of the urinary tract infection as well. You should consider continuing the course for a few days to see if improvement occurs.I hope this helps"
},
{
"id": 166384,
"tgt": "Why does a child vomit after consuming milk?",
"src": "Patient: My daug ht er will be 12 months old tomorrow and we switches her to whole milk abput a month ago when I found out I was pregnant again. She was a breast fed baby. She seemed to be doing ok with the whole milk until 2 days ago. Now after.she drinks it she throws up and it looks like cottage cheese and smells really bad. Whats causing this? Doctor: better start some formula milk as whole milk is not suiting your baby most formula milk with multinational brand are better"
},
{
"id": 205668,
"tgt": "What is the best medicine for anxiety disorder?",
"src": "Patient: I ve been prescribed and take between 1mg-3mg per day of xanax, depending on how bad my anxiety is or if I have a serious attack, which is the reason I was prescribed it. I m currently in a pre-trial program due to some silly trouble-mistake I made over a year ago and my pti officer is forcing me to stop my prescription and I get a urine test weekly... I went to a detox program for a week, but want to make sure I know to inform her if I may still test positive even though I haven t taken it since I heard that xanax takes a lot longer to be out of your system compared to other medications, esp. if I ve been taking it regularly for about a year. Doctor: Hello,Thanks for choosing health care magic for posting your query.I have gone through your question in detail and I can understand what you are going through.Medicines are the best options to take care of these symptoms. Paroxetine is the the most popular medicine and my drug of choice. I generally prescribe my patients 12.5 mg per day at night which takes care of anxiety symptoms. Also certain medicines like propranolol and clonazepam can be give as required when there is a panic episode or expected situation where panic attack can appear.Hope I am able to answer your concerns.If you have any further query, I would be glad to help you.In future if you wish to contact me directly, you can use the below mentioned link:bit.ly/dr-srikanth-reddy\u00a0\u00a0\u00a0\u00a0\u00a0\u00a0\u00a0\u00a0\u00a0\u00a0\u00a0\u00a0\u00a0\u00a0\u00a0\u00a0\u00a0\u00a0\u00a0\u00a0\u00a0\u00a0\u00a0\u00a0\u00a0\u00a0\u00a0\u00a0\u00a0\u00a0\u00a0\u00a0\u00a0\u00a0\u00a0\u00a0\u00a0\u00a0\u00a0\u00a0"
},
{
"id": 222178,
"tgt": "What are the chances of pregnancy after having birth control pill?",
"src": "Patient: I just had sexual relations with my girl friend and I ejaculated inside of her, as soon as we were done went and bought BCP. The thing is my girl friend has been on the pill 7 times in different occasions. My question is, Can she be inmune to BCPs? and how efficient are BCPs? Doctor: Hi dear, I have gone through your question and understand your concerns.Birth control pills are effective in preventing pregnancy if they are taken regularly at a consistent time.These pills are not emergency contraceptive pill and are not effective, if taken after the sexual intercourse.Hope you found the answer helpful.Wishing you good health.Dr Deepti Verma"
},
{
"id": 141361,
"tgt": "Suggest treatment for restricted movement in the hand and finger",
"src": "Patient: Patient name : Swasta Bag,Student,B,Tec,4th year He was accident by police car with right arm injury heavily,and admitted Belle view nursing home,Kolkata, three month earlier. Diagnosed by Dr. : fracture mid shaft right humerus with avulsion of right radial nurve. Treatment : Hole clocked plate and repair of injured nerve done by five-o-vicryl by orthopedic. Feedback : unable to movement finger,wrist,even hand up and down freely. Awaiting your suggestion and advise . My E mail : YYYY@YYYY Doctor: Hello and Welcome to \u2018Ask A Doctor\u2019 service. I have reviewed your query and here is my advice. Class c fracture of humerus may associated with radial nerve injury may takes time to recover. This type of fracture need tendon transfer more useful than nerve grafting. Only 25% if cases recover after nerve grafting as compared to tendon transfer which is 75%. Please consult your neurophysician he will examine and treat you accordingly. Hope I have answered your query. Let me know if I can assist you further."
},
{
"id": 20252,
"tgt": "Can high BP lead to problems with erection?",
"src": "Patient: I am a 26 year old male with a blood pressure reading of 150-80 but around last year march i was 130-80 and since my current reading every time i go to the hospital my blood pressure is high for some time and goes down later but some times it doesnt go down. i am an anxious person too. i got to know this from my sex life. i lose my erections when about having sex and when i am relaxed i get it back. i will be going for a pre employment medical exams for an oil rig engineering position and i want to know if it will affect my chances of having the job Doctor: Blood pressure at your doctor's office or a hospital is often much higher than your average. That's not to say that you shouldn't address the things that cause high blood pressure, just that medicine may not be needed.To get a real idea, get your own cuff. Record your blood pressure on a schedule every day for two weeks. It should be taken seated after a 10-15 min period of relaxation. When you meet with your doctor again, bring the record. That trumps what the nurse gets in the office.Look to your diet. Limit sodium intake to Exercise should be anything that is maintained for longer than 20 minutes. Exercise every day.After lifestyle measures are instituted, reassess the impact on your blood pressure. If you've been successful, stop measuring regularly but stick with the lifestyle changes."
},
{
"id": 49023,
"tgt": "Can white things seen in urine be the kidney stone passing?",
"src": "Patient: I just had a kidney stone surgery but was left an stent inside until today, am having lots of pain and now am seeing white things in my urine, could that be particles of the stone? I know i had a 7 mm stone and was blasted, how do i know am passing stone? Doctor: Yes, your concern is very true.In my hospital, after lithotrity i.e. stone blasting , I always ask my patients to collect urine in a container & send it for stone analysis.Also, to all my patients who want advise to prevent stone formation in future, I advise to do metabolic work-up & 24 hours urine analysis.You can discuss this with your doctor.Hope this information is useful to you.Thank you.Regards."
},
{
"id": 138117,
"tgt": "What could ribs pain with difficult breathing suggest?",
"src": "Patient: my husband is 44 yrs old and has pain on the left side under the rib cage, very tender to touch and hurts to breathe and move, he s not one for complaining and had a high tolerance for pain, i am concerned. should we head to the emergency room, it started yesterday and is worse today......what should we do? could this be muscle pain and give it a day or two?? Doctor: Hi,Thanks for your query.Strained muscle is the most probable reason. Strained muscle usually appears only when the chest is moving, and should be completely gone when lying down and not breathing. If the pain is sharp and well localized, more intense when breathing, rib nerve issue is the most probable reason.In both cases, rest and anti-inflammatory medicines like Diclophenac should help.I do hope that you have found something helpful and I will be glad to answer any further query.Take care"
},
{
"id": 26197,
"tgt": "What causes right bundle branch block?",
"src": "Patient: hello dr... im 28 years old and i have a rbbb... found it out a few years ago... then when i was examined by a cardiologist he said there was nothing and i also had a normal ecg and 2decho... today i was playing basketball and felt weak... i went to the e.r. and the internal medicine consultant told me i had rbbb... i have no other symptoms, just anxiety and felt a bit weak... i just wanted to ask if this could be caused by high cholesterol(ldl) levels? or is it something to worry about? Doctor: Hi Thank you for expressing your concern here.If I were treating you, I would not be too much concerned about Right bundle branch block if you did not have any associated disease in the heart. Right bundle branch block is associated with several structural heart diseases. But it appears that they are very less likely to be present in your case, from the history and the ECG & Echo report. Isolated right bundle branch block is known to occur in several healthy individuals and there are so many studies done to prove that when they exist alone without any other heart disease they do not affect the long term health of the person.Weakness could be because of many reasons. Did you check your hemoglobin? Please consider checking it, as iron deficiency anemia is one of the common causes of such weakness. High cholesterol will not cause weakness usually. What were the levels of Cholesterol. If it is too high, treatment may be warranted for it. Hope I answered your question!"
},
{
"id": 152077,
"tgt": "Nerve block for CRPS, please help",
"src": "Patient: My husband has CRPS in both feet. He had a nerve block on the left in November which went fine and it helped with the pain. Two weeks ago, he had an attempted nerve block on the left. The doctor said he could not get past some bone in his spine to get to the sympathetic nerves . Three separate times my husband felt a sharp pain from his thigh down to his knee while the doc attempted to get past the bone. The doc finally gave up without completing the procedure. Since then my husband has been in horrible pain and has swelling in his leg and especially his knee. He says this is not the same pain as the CRPS. Pain killers are really not helping. He had an MRI of the spine this morning. Could the doc have hit the sciatic nerve or some other nerve? It s bad enough he has the pain of CRPS but now this is making his pain 100 times worse. Thanks.. Doctor: Hello Thanks for your query.\u00a0\u00a0\u00a0\u00a0\u00a0 Nerve blocks success depends upon so many factors ,some times even well experienced cannot hit the proper area of nerve block. Failures in nerve blocks are well known . Sometimes instead of relief one gets symptoms worsened . don`t gets dis heartened from failures ,as failures are pillars of success. there is no harm in trying again Complex regional pain syndrome (CRPS) is a chronic progressive disease characterized by severe pain, swelling and changes in the skin. . \u2018Hope I have answered your query, I will be available to answer your follow up queries, \u201cWish you Good Health and trouble free speedy recovery\u201d"
},
{
"id": 84926,
"tgt": "What causes swelling around ankles/feet when on bi-est 80/20 prog 200 troche?",
"src": "Patient: i take biest 2mg (80/20) prog 200 troche lately I ve noticed really bad swelling around my ankles and feet not to mentioned my legs feel so tight and to me swollen I have had my gall bladder out about six months ago so of course my weight has practically doubled so i don t know if all these symtons are from the gall bladder being out or the estrogen Doctor: Hello,I think the weight gain is secondary to estrogen and progesterone intake. Removing a gall bladder has nothing to do with weight gain.Hope I have answered your query. Let me know if I can assist you further. Regards, Dr. Muhammad Faisal Bacha, Internal Medicine Specialist"
},
{
"id": 34114,
"tgt": "Suggest permanent cure for jock hitch",
"src": "Patient: Dear Doc, Is there any permanent cure for jock itch (me & wife including my children (10 year old girls & 4 year old girl) have this dark colouration around groin and itchy some times. The fungal infection creams & powders seems to be of no use Kindly advise Doctor: Hello dear,Thank you for your contact to health care magic.I read and understand your concern. I am Dr Arun Tank answering your concern.The pigmentation that arises after the jock itch is because of the after reaction of the melanin.As such there is no such cure for this condition. Only wait and watch can cure such a problem. Our immune system can gradually digest the extra pigments. But this process is very slow.If you want fast result than you can apply vitamin A, poly unsaturated fatty acid zinc etc. Various combination of the above product is available in the market and can be taken under your doctors guidance.Please do not expose the part to the direct sun light, avoid extra sweats to the parts and wear the cotton cloth that can absorbs the sweat.This much care can bring the change.I will be happy to answer your further concern on bit.ly/DrArun.Thank you,Dr Arun TankInfectious diseases specialist,HCM."
},
{
"id": 96559,
"tgt": "Is an ER visit advisable for a child with a cut on the eyelid?",
"src": "Patient: HEllo my 6 year old son hit his eye on a table. He got a cut on his outer part of the eyelid. It\u2019s righ at the crease of the eyelid. Should I take him in to ER to get it looked at or will it be okay. It\u2019s not bleeding a lot. It\u2019s not very deep but did open up top thin layer of the skin. He says it\u2019s not to painful. Doctor: Well come to HCMThank for asking Your concern being appreciated, such injury of the face particularly near eye should not be taken as granted or taken lightly but it has to be taken seriously I would suggest to see the surgeon, at least for cosmetic reason wound may need stiches again it need to be ruled for bony injury, hope this information helps."
},
{
"id": 35567,
"tgt": "What causes pimples on butt, pain in testicles and constipation?",
"src": "Patient: hi. my 9-yr old son has 4 pimples on his butt, two that popped, two that haven't popped. he also said his testicle kind of hurt him today. but thenhe said it might have been his pants too tight. he's had constipation problems and fissures and a hemmorhoid problem lately. could this b a bacterial imbalance in him? I had one but was treated for it. mayb he could have gotten it off toilet seat? i do't know Doctor: not to worry, this is boils mean infection of hair follicle and consult to ur doctor take small antibiotic course along with hot fomentation. secondly constipation and boils different thing, yes chronic constipation can create problem of fissures,hemorrhoids but u can avoid by using plenty of water and fiber diet."
},
{
"id": 82470,
"tgt": "Suggest treatment for chest pain",
"src": "Patient: I have this pain in chest for the last couple of days. i have had gasses and acidity issue but today morning at times when i breathe i can feel this pain. I had got my ECG done on 15th OCtober and it was normal. Do i have anything to worry about? My doctor has prescribed MOZA MPS to be taken now Doctor: HelloYou didn't mention your age and the side , where you are having pain but ECG is normal means at present we can rule out heart disease.Your doctor prescribed mosapride with methypolysioloxasane as this act as antiflatulants and used in hyper acidity cases.In my opinion all these symptoms may be due to these reasons , these include;1 Hyper acidity or chronic gastritis , diagnosis can be confirmed by upper G I endoscopy.Try raising the head of your bed about 4 inches with blocks . To control stomach acid avoid eating or drinking for 2 hours before you lie down. It also might help yo avoid drinking alcohol or drinks with caffeine in them or eat chocolates or greasy or oily foods .Also take some antacid ( but if it contains oxetacaine will be better ) but if pain is severe you may need drugs like pantaprazole-D once or twice in a day on empty stomach.Over eating is the another causes of such symptoms , so avoid it .Tension or stress are the most common reason of such symptoms ,so be careful about that also.Hope this will help you."
},
{
"id": 63897,
"tgt": "Suggest treatment for a painful lump on the hip",
"src": "Patient: i hit my hip on a railing nearly two weeks ago.. it was the middle of the night so i went back to sleep. the next day i felt sore, saw a bruise but no lump/knot within 2-3 days later i started to get a lump.. now it s nearly 2 weeks later... i don t have the bruise any more.. just the knot... i even tried to massage out the lump.. it s only gotten bigger should i go to the ER... Doctor: Hi,Dear,Good Morning and Thanks for your query to my virtual Clinic At HCM.I studied your query in depth and understood your concerns .Causes for painful lump on hip as follows-Mostly they are - could be Abscess in post-trauma Hematoma on the hip with infection in it,due to which it has increased in size.Treatment-Tab-NSAIDs -like VoltarenTab-Amoxycycline 250 mg x three times a days Dressing and Surgical I and D for Acute on Chronic Abscess.Check with ER Surgeon and check your fbs and pps,and plan for-Scraping of the Chronic abscess wall and adequate drainage. Don't worry of them,as they could be relieved with proper treatment.Hope above reply would help you to relieve of the issue you have.Welcome for any further query to ME and HCM.Wishing you a Fast Recovery.Write Good review for the benefit of my new patients.Click thanks ASAP after this.Have A Good Day.With RegardsDr.Savaskar"
},
{
"id": 178959,
"tgt": "Suggest treatment for persistent cough",
"src": "Patient: i have three yr old son, of late he is having frequent cough problem. During the day his coughing is not much but moment he sleeps in the night, he gets up coughing leading to removal of sticky and slimy cough, it is only after that he is able to sleep properly. can u pl help me with probable reason for this ? When he was taken to local peadatrician for recurrent cough we were advised to give him ascoril as and when such cough episodes occur. Is is safe/proper to provide medication as suggested by peadatrician Doctor: Hi, I had gone through your question and understand your concerns.You have not mentioned which ascoril you are using , is it ascoril + expectorant , Ascoril-D or Ascoril-C syrup . It is safe to use Ascoril in prescribed doses.Based on little history provided by you , your child may be suffering allergic cough ( nighttime ) a trial of Montelukast and Bambuterol or inhalation bronco-dilator with nhalation steroid can be tried after discussing with your doctor to see the response .Hope this answers your question. If you have additional questions then please do not hesitate in writing to us.Wishing your son good health.\u2022\u00a0\u00a0\u00a0\u00a0\u00a0DISCLAIMER: - All the information provided here is for information purpose only , it is not a substitute for the advice of a physician after physical examination , it is not intended to replace your relationship with your doctor. This information in no way establishes a doctor-patient relationship. Before acting anything based on this information do consult your doctor. I recommend that online users seek the advice of a physician who can perform an in-person physical examination"
},
{
"id": 2356,
"tgt": "Can dry humping cause pregnancy?",
"src": "Patient: Hello Dr, I have a very simple question. Can dry humping with my boyfriend with our clothes on get me pregnant? I am seriously worried about it. I started to be conscious of everything I do; how long I sleep, how much I eat, pee, how my body looks. I am scared. Thank you. Doctor: Dear Ms, no this can not get you pregnant. to get pregnant a complete intercourse is required with the ejaculate inside the vagina"
},
{
"id": 24111,
"tgt": "Suggest treatment for controlling high blood pressure and dizziness",
"src": "Patient: I am a healthy person 48 years old male. Married, happy and I have a normal nice life,and family. I never faced any serious medical problems. I work hard and my mind is always busy. last August I faced a medical problem feeling that I am go fall down with bad feeling that my heart is going to shut down followed with tachyacardia. This was repeated once or twice per week and each time it became longer in duration and worst. I was transfered to hospital several times with no clear diagnose I visited several doctors and did all possible tests, ECG. blood tests, brain and cervical MRI and MRA and etc and all are clear. Then one doctor diagnosed my case as panic attack and accordingly I stared to take some medication (Dogmatil,unirox, and regulators for BP ) assuring that during the attacks my BP raised to 170/100. In normal conditions my BP is normal even low 110/70. Since last August and up to now I start feeling better but Istill have moments when it is about to start again then I start feeling with discomfort of missing beets above and around my stomach. Also this bad feeling comes after meal and when I laydown immediatly after lunch. Also I have to to have rests from time to time and to laydown otherwise I will feel dizzy again Can you please advise me what to do? (Sorry for my language) Doctor: Hi,When the stomach is full it can press the diaphragm and some nerves and cause such complaints. I would advise you to eat frequently with small portions, don't drink any liquid with the meal and don't lie down after you eat. This will releive your symptoms.Take careCome back if you have any further questions"
},
{
"id": 55708,
"tgt": "How to control fatty liver, sgot and sgpt?",
"src": "Patient: i Am Harsh Thakur suffering from fatty liver taking ursokem from the last two months along with sylimarin my sgot and sgpt has increased but not shown much improvement i have quit drinking from the last 6 months so what should i do to control sgot and sgpt and fatty liver Doctor: Hi thank you for posting your query in HCM. i had gone through your question. I understand your concerns. My opinion is that you should not much worried about this as the fatty liver is a reversible condition. This can be achieved by simple lifestyle modifications for weight loss.I would advise you to follow these instructions given below. Try losing 1 or 2 pounds per week not more than that, as it would lead to health problems. Go for cardiovascular exercises like brisk walking, running, swimming for 45min a day, 6 times a week. Strength training 4times a week.Avoid alcohol , quit from smoking.Avoid fatty foods, junk foods, deep fried foods, simple sugars, pizzas, chocolates, ice creams, soft drinks. Have a balanced and healthy diet that should include complex carbohydrates like brown rice, oats, brown bread. High protein foods like egg white, pulses, beans, sprouts, soya beans, milk, cottage cheese. Low fat foods. Use olive oil. Have 4 to 5 servings of green leafy vegetables along with all mixed vegetables. 5 to 6 servings of all fruits except banana, mango, chickoo.Drinky 3 litres of water per day.If you have diabetes keep your blood sugars under control.Get regular check ups for cholesterol and triglycerides from your doctor. Hope i have answered your question, if you have further questions i will be happy to help."
},
{
"id": 220978,
"tgt": "What is the best treatment to get pregnant?",
"src": "Patient: hello this is tabish here i am 28 male i got married last year. firstly i would like to know that when we make love it took lond time in errection secondly after ejculation i feel so tired i want to know why i feel like that. i also want to know that we dont want to have baby yet does it create problem in our health if we delay and which is safest method for delay. regards tabish... Doctor: Hi, Thanks for the query under HCM forum. I understand your concern. - Your problem is both physical & psychological. The thought that the sex may result in pregnancy ( where as you want to delay it ).. has a nigetive approach towards sex. So please rest assured, that there are multiple options for avoiding pregnancy ( to be adopted by male & female both) . You can use condom/ your wife can use intrauterine contraceptives or oral cyclical contraceptives - according to your doctor's advise . - THe cause of your problem may be physical problem too(though, both physical & psychological factors go hand in hand ) You have to get your semen analysis done after 3 days abstinence & consult a urologist for needful medication. - Please don't worry, you can safely postpone child bearing for 2-3 years with use of suitable contraceptives.. it will not harm your health. Thanks."
},
{
"id": 221133,
"tgt": "What causes abdominal discomfort during pregnancy?",
"src": "Patient: i am 24 i had a miscarrige in may10. i am 4 weeks pregnant and am experiencing slight tingles in my right lower belly. i had these last week for 2 days on my left side. they last about 40 seconds and are frequent through out the day they started yesterday afternoon. i have no bleeding or spotting. is this normal? thanks haley Doctor: Hello dear,I understand your concern.In my opinion the tingles in abdomen might not be a problem.If you have any pain abdomen,backache,spotting,,bleeding etc then it needs to be evaluated.Dont worry.Take healthy nutritional diet and adequate fluids upto 3 litres per day.Avoid physical stress and intercourse.Antenatal yoga or exercise are helpful.Best regards..."
},
{
"id": 60110,
"tgt": "Pulsating pain around the rib cage that is sensitive to movement, relieves for short time on eating post gall bladder surgery. Is it related?",
"src": "Patient: i had my gall bladder surgery in 2005. yet periodically i get episodes of what feels like the same attack symptoms prior to the gall bladder surgery.. i have had the latest one for 3 days now.. it is what i would describe as a rolling pulsing achy pain right where my rib cage meets. it is sensitive to movement and position, and can feel quite sharp at times... and for some reason eating something, even when not hungry seems to make it feel diminished for a short time... could this be related to the gall bladder even though i had surgery in 2005? or could it be completely different cause? Doctor: hello welcome to health care magic The answer is you should avoid eating oily and spicy stuff. Avoid cola drinks, excessive hot drinks, tea and coffee, and even alcohol. you should not lie down immediately after food. Get a check Upper GI endoscopy to rule our hiatus hernia. If you are obese, has to reduce weight. Such symptoms are usually present in obese. when you feel this immediately drink a cup of coffee and let me know at payalrawatchauhan16@gmail.com thanks"
},
{
"id": 56289,
"tgt": "Is liver transplant effective for person with HBV affecting half of liver?",
"src": "Patient: hi Dr.my brother who lives in Ghana has been diagnosed with HBV which seems to have affected his liver. Im told that more than half of the liver is affected according to the ultrasound images. In ghana they dont do transplants. so im trying to get him to India where they profess to have extensive expertise. But ghaians Drs says he may not benefit from the op Doctor: Hi, I had gone through your question and understand your concerns.Yes , in India liver transplants are not that uncommon . But mere affection with Hepatitis B virus does not need a liver transplant. Indications are 1. end -stage cirrhosis2. Chronic viral hepatitis3. primary hepatocellular malignanciesand othersWhat I think is that your brother is suffering from Chronic hepatitis due to Hepatitis B virus.Now, in patients with chronic hepatitis B, in the absence of measures to prevent recurrent hepatitis B( like putting him on antiviral drugs), survival after transplant is reduced to some extent , however prophylactic use of hepatitis B immune globulin during and after transplantation increases the survival to around 35-40%.Do not panic.I would suggest you to bring him to India. There are premium institutes like All India Institute of Medical Sciences (AIIMS) which does liver transplants daily.Hope this answers your question. If you have additional questions or follow up questions then please do not hesitate in writing to us. I will be happy to answer your questions. Regards."
},
{
"id": 185583,
"tgt": "What causes painful bump on gum?",
"src": "Patient: Hello, I'm writing because a year ago I had all 4 of my wisdom teeth taken out because they were impacted and causing me pain. A couple months after the fact, a small solid but painless bump appeared on the outside of my gums by the upper left extraction site. As recently as a month ago, the bump seemed to be more sensitive and a tiny pointed bone like protrusion seemed to work it's way through the soft tissue. That has since healed over and isn't nearly as swollen, however now the upper right side seems to be starting to swell. Anything you can tell me to help would be appreciated. Doctor: Hello:)Welcome to HCM.This can be a bony protrusion from the extraction socket.Nothing to worry. See your dentist once.He will remove it.Do not worry, it is as easy as you remove debris from your teeth.Its not a painful procedure.The sharp bone can project out some times during healing and irritate the surrounding cheek and soft tissue.Take a OTC pain killer if necessary.Apply quadragel ,a soothing ointment on the area. Regards."
},
{
"id": 16157,
"tgt": "Itchy rash on stomach, spreading around the sides, tiredness. Using cortisone cream. Cure?",
"src": "Patient: For 4 days now I have an itchy, light bumpy rash in my belly which spreads around the sides up to the bra line,. put cortisone 10 creme on for the last two days twine a day, no improvement. What should i do. I m also very tired..just mentioning it in case its related Please help! There are no new soaps ect that on touch that area Help please :) What coud ld it be and how do I treat it??? Doctor: Please stop to use cortison cream, since it is a case of fungal infection. Apply oxiconazole cream twice daily and take orally Fluconazole(50mg) 1 cap daily for 1 months and anti histamin Fexofenadin 120mg daily for 1 month"
},
{
"id": 143081,
"tgt": "Suggest remedy for poor memory",
"src": "Patient: extremely poor memory, has serious hearing problems that got progressively worse in a very short time...poor comprehension...and never can direclty answer a question, but goes around the question and if he does not understand (definately not intentional) Doctor: Hello.Thanks for the query.Ideally, I would have liked to know more details.Who are we talking about?What is the gender of the person?What is the age of the person?Are there any issues other than these?How long have the symptoms been present?How are they progressing?These questions and the related information is vital to the solution.So please let me know the details as soon as possible.Best of luck, Dr Mittal"
},
{
"id": 144086,
"tgt": "What causes nausea, headache and numbness in limbs after a head injury?",
"src": "Patient: Hit my head on the wall the other night in the middle of the night. Went back to sleep but in the morning(yesterday) had headache and then throughout the day became very weak. I felt weird, like this high feeling all day I had nausea, numbness in legs and arms and very weak to move off the couch. Troubles sleeping last night because of tension in my head/face, and like my body wouldnt let me fall asleep when I was getting close. Today i feel fine, just a bit off but not all those symptoms. Should I go to doctors to get checked out even though i feel okay at the moment? Doctor: Hi, I am Dr.Bruno. I have read your question with care and understand your concerns. Let me try to help you The following symptoms following Head injury require evaluation by a Neurosurgeon1. Headache 2. Vomiting 3. Dizziness4. Giddiness5. Double Vision 6. Blurring of Vision 7. Numbness8. Weakness 9. Fits10. Bleeding from Nose 11. Bleeding from Ears12. Bleeding from throatOf these, you have a) Headacheb) Numbnessc) WeaknessSo You should go to doctors to get checked even though you feel okay at the moment Hope you found the answer helpful.If you need any clarification / have doubts / have additional questions / have follow up questions, then please do not hesitate in asking again. I will be happy to answer your questions.Let me know if I can assist you further.Take care."
},
{
"id": 73045,
"tgt": "What causes breathing difficulty with dizziness and headaches?",
"src": "Patient: hi i'm 15 years old and for a about a week now or so i've been haveing trouble breathing at times like a tight air passage, dizzynessheadaches, and occasional white dots flying around where ever i look. i'm wondering what it might by but i think it may be high blood pressure? Doctor: Thanks for your question on Healthcare Magic.I can understand your concern. Possibility of stress and anxiety related symptoms is more likely. But better to first rule out heart diseases for your symptoms. So get done blood pressure monitoring, ecg and 2d echo. If all these are normal then no need to worry fo hypertension or other heart diseases. Consult psychiatrist and get done counselling sessions. Try to identify stressor in your life and start working on its solution. You may need anxiolytic drugs too. Don't worry, you will be alright with all these. Avoid stress and tension, be relax and calm. Hope I have solved your query. I will be happy to help you further. Wish you good health. Thanks."
},
{
"id": 10139,
"tgt": "How to overcome the hair fall and split end problems?",
"src": "Patient: hii iam faceing hair fall n spilit end and many more hair problems. please sir will u help me. what i have to do for my hairs growth and also how to increasse my skin complection.i wanna visit ur clinic once will give me ur number address.thank you. Doctor: Hello and Welcome to \u2018Ask A Doctor\u2019 service. I have reviewed your query and here is my advice. I can understand your concern for hair problems of hair fall and split hairs. Consult the dermatologist for the perfect diagnosis and proper treatment. There may be some cause like anemia,vitamin deficiencies,mineral deficiencies,stress,worries, travelling,internal diseases like thyroid dysfunction, etc. Tablet Biotin, iron tablets, vitamin E capsule, minerals tablets, high protein diet, exercise, meditation, etc. may be considered. Oil the hairs with castor oil twice a week after shampooing with herbal shampoo. Relax and have positive attitude. Hope I have answered your query. Let me know if I can assist you further. Regards, Dr. Ilyas Patel"
},
{
"id": 169799,
"tgt": "What causes strep throat and sinusitis in a 5 year old?",
"src": "Patient: My daughter is 5 years old..she just got over her strept throat about a week and a half ago..she has a low fever again today and has been very thirsty the last few days..she just doesnt seem to be healthy lately..in fact all three of my children dont seem to be very healthy..my 8 year old boy just had pnumonia the same time my daughter had strept and my 7 year old girl has sinusitis..what is going on? is it the time of year or are my kids just not healthy..worried about my 5 year old though..thank you Doctor: Hello, Welcome to forum. From the history stated it seems to be a viral infection circulating in your house Its very common among children, and usually occurs in many family members at a time. Please give paracetamol in dose as mentioned on the bottle and also sponge the child from top to bottom with normal tap water with every fever spike. Give the child plenty of fluids and rest. Disclaimer:This provisional advice provided by me stands subject to the actual examination of reports/image and is based entirely on inputs provided to me. It should be correlated with clinical findings"
},
{
"id": 168686,
"tgt": "What causes cyst on the shaft of the infant s penis?",
"src": "Patient: Hi, may I answer your health queries right now ? Please type your query here...hi, i brought my 5 month old son to the doctor because i noticed two pocket like things on the shaft of his penis. there was white stuff inside, almost like a pimple. she said they were cysts. and prescribed Nystatin. It doesnt seem to be doing much. Is this a serious problem? what is it caused from?-thank you Doctor: HiNot a serious problem and frankly nystatin wouldn't do anything for cystsThese are caused by blocked oil glands and for now they need no treatment"
},
{
"id": 162485,
"tgt": "What can cause fever along with swollen bumps in the throat?",
"src": "Patient: My 7yr old and 5yr old sons started having a fever 3days ago, there throats became swollen yesterday and today they have bumps all over some are filled with puss what could this be? Doctor: Hi, I have gone through your question and I understand your concerns but please send a photo to be able to help you with a better diagnosis.. however, this could be due to a viral infection such as herpes or a bacterial infection such as strep. Hope I have answered your query. Let me know if I can assist you further."
},
{
"id": 130754,
"tgt": "Causes of pain on right shoulder on the top and around clavicle bone",
"src": "Patient: I am having paid on my right shoulder on the top and around my clavicle bone. Also I am having pain on the right side of my back from about mid way down to my shoulder blade. I have had two MRI s done, one for my shoulder and one for my neck and it doesn t show anything that is wrong. What could be causing this pain? Doctor: Hello and welcome to \u2018Ask A Doctor\u2019 service. I have reviewed your query and here is my advice. It seems to be a muscular pain which is caused by shortening of muscles around shoulder and neck (trapezius muscle). Apply heating pad for 5 to 10 mins and then do some stretching of neck and shoulder muscles and sleep supine. Don't lie on right side (side lying position). If you know the stretches then well and good otherwise contact any physiotherapist. Your therapist will guide you with stretches. Hope I have answered your query. Let me know if I can assist you further.Regards, Dr. Harsh Swarup"
},
{
"id": 166813,
"tgt": "What causes high fever with headache, tired, bloated and less appetite?",
"src": "Patient: I have a 14 year old daughter who has been having high fever for one week, with headaches, feeling tired, and feeling bloated after eating about 1/2 portion of what she usually eats. X-ray normal. Doctors say to wait two more days to start doing blood tests as they believe it is a virus. Doctor: Hi,Since the child is having fever for one week duration it is better not to delay blood tests any longer. In your description you haven't mentioned the other symptoms child is having like, cough sore throat, runny nose. Does the child have diarrhea? Did the symptoms severity reduce with time or did it worsen? With viral fever usually symptoms reduces after day 5, it is most likely a viral fever but it is better always to do your blood test and exclude bacterial infections or other serious infections. Hope I have answered your query. Let me know if I can assist you further.Regards,Dr. K.r.w.m.manori Karunathilaka"
},
{
"id": 206077,
"tgt": "Suggest treatment to overcome depression",
"src": "Patient: Hi, I have been suffering from internal struggles for close to five years now. I haven t told my parents anything, so I am not sure if I am suffering from depression or anything else. Five years ago I was sexually harassed by my older brother, and I was so confused I kept it a secret and began to watch adult videos. Since then I had a harder time trusting in people, and I also had one friend at that time use me. Four years later I finally broke down from hiding my feelings and secrets from people, so I ended up telling my friend. She accepted me and I told my other friends, but later on she had problems too and took her anger out on me. She said things like she wanted me dead and cussed me 8ut a lot, so I cried and broke under the pressure. I ve had a hard time believing anything people say, even close friends, and I constantly feel I am a disgusting, horrible, and trashy scum of a person. I have felt that if I were to die no one would be sad and they d be happy, and after I lost a loved one I wished I could switch places with them. I have had suicidal thoughts and whemever I had a lot of anger built up I silenced it by hurting myself, for example digging my nails into my skin. Ontop of all this my mind is now kind of split in two and at some points I am a normal person but then I ll switch to being extremely mad, hateful, suicidal, and depressed. When I became ike this I usually have to hae a friend calm me down. Also, I feel tired all the time, and I never really slept right, I d stayup all night and sleep for maybe a couple hours the next night. My eating habits weren t good either, and sometimes I would throw all of my food away. I feel a lot of the times that I have no happy future and happiness isn t real. I hate myself, and can t trust people. Many nights I had to either shut down my mind or cry until I was tpo emotionally and physically tired. Also, my brother got a girlfriend, and I have thoughts a lot about how I hate that he is happy while I m suffering. I haven t told my parents yet because I don t want to hurt them but I m also scared. I have had dreams of them disowning me and hating me. I ve dealt with all of this since five years ago, and I am a teenager currently. Am I depressed? Do I need to seek help? Doctor: DearWe understand your concernsI went through your details. I suggest you not to worry much. I sincerely suggest that you are victim of your own negative thinking. You cannot term your current state of mind as clinical depression. Rather, this is just disappointment. Disappointment which arises out of unforeseen life events do bring such symptoms like sadness, irritation, anger, lethargy, lack of interest, lack of appetite etc. Please understand this fact and make sure to introspect. Talking to your friends, out door activities and being creative should give you relief. Find the reason for your disappointment and consult a psychologist if need be. Psychotherapy techniques should suit your requirement. If you require more of my help in this aspect, Please post a direct question to me in this URL. http://goo.gl/aYW2pR. Make sure that you include every minute details possible. I shall prescribe the needed psychotherapy techniques.Hope this answers your query. Available for further clarifications.Good luck."
},
{
"id": 102479,
"tgt": "How to treat a chronic cough accompanied with a headache and pain in abdomen?",
"src": "Patient: I have ben very achy the last few days, and as of last night my stomach seems hard and distended and it painful when I breathe deeply. I have also had a chronic cough for over a week, along with a headache, the coughing also causes pain in my abdomen and pain in my temples. Doctor: Hello,Welcome to HCM,Chronic cough will increase the intraabdominal and intracranial pressure, which is leading to your symptoms.Pain in the abdomen and the temple is evident that you are having vigorous chronic cough.You may require sputum examination,chest X ray and blood examination, this will guide us towards the possible cause for the cough.After identifying the cause treatment becomes easy.I would suggest you take1.Oral Antibiotics2.Oral Antihistaminics3.Cough syrup4.Analgesics.Consult your doctor do all the investigation and you can take this medicine by taking prescription from him.Thank you."
},
{
"id": 138203,
"tgt": "Suggest treatment for upper thigh pain after using an elliptical",
"src": "Patient: upper thigh pain after using an elliptical....can be relieved by meds, but it is always there. Sometimes it is hard to raise from a sitting position...I tend to limp and it almost feels like it is going to lock on me. This happened in March and it will not go away. It started as an intense inner thigh pain, but now is mostly on top and outside of thigh. I take a blood thinner...warfarin 7.5 per day. Heart rhythm problem. Doctor: Dear patient Your pain is attributed to hip pathology. This can be in form of early arthritis or chondral injury. I would like to advise Xray of pelvis with both hips anteroposterior view to confirm diagnosis. Visit radiology center nearby you and get it done. Meanwhile start tab ultracet p twice a day for pain relief. You need to consult expert orthopaedic surgeon with report. All the best."
},
{
"id": 98293,
"tgt": "How can severe allergic reaction to a wasp sting on the face be treated?",
"src": "Patient: I just got some kind of wasp or stinging bee sting me on my lip when weeding. The strangest thing, though, was my nose immediately started to bleed. I watched for whatever it was to come back out but it did not. The nest was in or close to the ground. My husband looked right away for a stinger, but did not see one. It immediately started to swell up. I took 1 Bennadrel (sp?), then soon after a second, and put an anti-sting fluid on. It s less than an hour later, and I look like a freak! Now will put an ice pack on it. About all I can do?? Doctor: Hi Go immediately to emergency room for epinephrine and prednisone injections. Hope I have answered your query. Let me know if I can assist you further. Regards, Dr. Olgeta Xhufka"
},
{
"id": 193467,
"tgt": "What causes mild blisters around the head of penis?",
"src": "Patient: Hi, may I answer your health queries right now ? Please type your query here...hi, i have herpes phase 1 syptom, today by chance i observed a mild blister has reoccurred on sides of pink area of my penis after 6-12 months.i am in habit of masterbating every week and 2 weeks back i visited a massage parlour where a handshake was given by a female on my penis.I have a very busy worklife and will not be able to consult a doctor before weekend.please suggest an ointment eg silverex,triben,nadifloxacin available with chemist in mumbai Doctor: Hi, If you are sure you have herpes then you should know it is not curable. You will be always herpes positive and there will be episode of rashes on and off depending on immunity status. If it is not related to herpes and due to dry sex activity local moisturizing cream with antibiotic can help you. Hope I have answered your query. Let me know if I can assist you further. Regards, Dr. S. R. Raveendran, Sexologist"
},
{
"id": 72568,
"tgt": "What causes unconsciousness, breathlessness and lack of oxygen?",
"src": "Patient: Hi, My mom is around 45 and she has been healthy so far. Since some months she feels heat in backside of her head and then gets unconscious for sometimes (Some Minutes), and then when she gets conscious she complains for breathing and lack of oxygen. Need help to diagnose what the issue can be? Doctor: Hello dearWarm welcome to Healthcaremagic.comI have evaluated your query for your mom thoroughly .* There are different reasons for this presentation as - psychosomatic manifestation of underlying stress , anxiety . - cerebro vascular insufficiency - others .* Needs precise clinical evaluation with necessary work up as MRI brain and others .Hope this clears your doubt .Wishing her fine recovery .Welcome for any further assistance .Regards dear take care ."
},
{
"id": 4943,
"tgt": "Trying to conceive for 2 years. Does having blood clots during periods be a reason for not getting pregnant?",
"src": "Patient: Hello I have been trying to get pregnant for the past 2 years but I haven't been successful ( I already have 2 daughters) but i been told that due to me having blood clots while in my period every month is the reason i can't get pregnant. Is that true if so what can i do? I really want another baby. By the way I'm 38 years old.Thank you Doctor: Hi,Thanks for writing in to us.At age 38 years, the number of viable eggs in a woman significantly decreases with increasing age. The reason of not getting pregnant due to blood clots in your periods is not right.If you have two daughters already, you do not have any structural abnormality in yourself. Please try to have intercourse with your partner around the date of ovulation, one day before ovulation and for four dates after that regularly.Keeping your age in mind, I would suggest you to consult a fetal medicine clinic and discuss the information on issues with children born to mothers around your age. Some screening tests may become necessary to be done once you get pregnant to confirm that the unborn baby is normal in all respects.Hope this helps"
},
{
"id": 225861,
"tgt": "Had continuous intercourse after safe periods. Bought mifeprin kit to avoid pregnancy. Advise?",
"src": "Patient: Hi doctorI had intercouse with my parter after my safe periods continously for 7 days.i am sure I wil get pregnent and wil skip my periods.have 10 more days for my regular cycle date.to avoid this pregnency I have bought mifeprin kit and was told to use the first tablet so it ll continue the regular days.is this safe.i need remedy before the cycle as it shld continue as usual.pls suggestRegardsRia Doctor: Hi,Thank you for choosing Healthcaremagic. It is not advisable to take this kit before pregnancy, because this is abortion pills, which are intended to be used after a confirmed intra uterine pregnancy of 7 weeks. You would have gone for emergency pill like levonoargestral, rather than taking MTP kit.I hope this information has been both informative and helpful for you. In case of any doubt, I will be available for follow-ups. If you like my answers kindly rate it, and write a review as well. Please do not forget to accept it.Thank you,Wish you good health.Regards,Dr ArifYou can consult me again directly through my profilehttp://www.healthcaremagic.com/doctors/dr-arif-n-khan/65133"
},
{
"id": 213044,
"tgt": "Spells of lightheadedness, sweats, cheat pain radiating to shoulder without rapid heartbeats. History of anxiety attacks. What could this be?",
"src": "Patient: Since last night, I have getting light headed, I got the sweats while doing dishes and a pain in my left chest going back into my shoulder blade area. I have had anxiety attacks before. It almost felt like that but with out the anxiety or rapid heart rate . Doctor: hi..this may be serious as it resembles pain of ischemic heart disease like myocardial infarction..so please visit a cardiologist..after his clearance, you can visit a psychiatrist for valuable advice..thanks..and remember, although you may have experienced past attacks, the causes may not be the same..and we should err on the wrong side.."
},
{
"id": 179781,
"tgt": "What causes high fever,vomiting and diarrhea?",
"src": "Patient: Sir, My one year baby girl has got high fever, vomiting & diarrhea.she had given medicine for all a day back.but still she is sick.her blood result, haemoglobin is 9.9 gm/dl and ESR 90 mm/hr. Two months back also she had same problem. Doctor: Common cause of fever, vomiting , and diarrhoea is gastroenteritis , but she is anaemic, ESR is elevated so it is advised to rule out any chronic infection that may be reason for recurrent infections"
},
{
"id": 134062,
"tgt": "Suggest remedy for pain in bicep area",
"src": "Patient: I was told I have tendonitis and an arthritis spur. My pain is in the bicep area, (and back of upper arm from time to time), Is this possible? I have extreme pain in the bicep and I can not lift my arm or put it behind my back, I can not lift hardly anything without the stabbing drop-what your-holding type of pain. The doctor suggest ibuprofen and possibly prednisone pack and possibly injection, with physical therapy. Can I please get your opinion if this sounds right. I know it is only your opinion. Thank You.Joi Doctor: hi,as mentioned by you, have you taken an MRI of the shoulder to confirm the diagnosis made?well if not, then the one is advised. Also, physical therapy sounds good. As it will help you reduce the pain by Therapeutic ultrasound therapy and TENS therapy. Post which exercises will be implemented to improve the strength. It will be slow process and longer as well. But in personally I have seen cases with bad shoulder injuries who even were recommended surgeries but till date no surgery and full shoulder functionality.I wish you continue the physical therapy for a little longer as you will have a lot of benefits.regards.Jay Indravadan Patel"
},
{
"id": 177893,
"tgt": "How to treat fluid in ears?",
"src": "Patient: my 24 month old grandson recently had an ear infection Friday he went for his follow-up and the Dr. said infection gone but he have water in his ear how do I remove the water. If the water remains in his ear will it eventually cause another infection? Doctor: Hello. I just read through your question. This is not uncommon. Sometimes fluid can remain in the ear after the infection is gone, and sometimes this can lead to another infection. In most cases, though, fluid in the ears resolves on it's own. You should follow up with your doctor in about one month to confirm this. If the fluid is still there, your doctor may want your grandson to be seen by an ENT. Until then, however, there is nothing to do."
},
{
"id": 197638,
"tgt": "Which disease can cause dark red hard bump on penis?",
"src": "Patient: i had a dark red hard bump on my penis head three weeks ago. i removed it one day later. i am afraid of syphilis infection. what diseases can cause such dard red hard bump? it was painless, but felt uncomfortably (can felt something there) and had diarrhia at that time. Doctor: HiWelcome.I have gone through your query.It is difficult to comment on it without examination, so I would either suggest you to see doctor for examination and for exact diagnosis or upload the pictures here, I'll go through them, and provide you with specific advice.Hope this helps.Take care"
},
{
"id": 88529,
"tgt": "What causes fever, sore throat and runny nose along with abdominal pain?",
"src": "Patient: I am 18,female.I just became sick on Sunday. I had fever,chills,sore throat,stuffy/runny nose & starting last night I began to get sharp pains in my left abdominal area. I have to pee often & nothing really comes out. It also hurts a little when I move. Doctor: Hi! Good evening. I am Dr Shareef answering your query.From your symptoms it seems that you have got a urinary tract infection (UTI) with a possible pelvic inflammatory disease specially if you are sexually active. If I were your doctor, I would advise you for a urine test (routine, microscopic and culture sensitivity), a blood test for CBC, and treat accordingly. I might go for an ultrasound of the abdomen in case urine test did not show anything. Further management would depend on the reports of investigations. Till then, I would advise you for an alkaliser, plenty of liquids, and an anti inflammatory drug, along with a decongestant for a stuffy/runny nose.I hope this information would help you in discussing with your family physician/treating doctor in further management of your problem. Please do not hesitate to ask in case of any further doubts.Thanks for choosing health care magic to clear doubts on your health problems. I wish you an early recovery. Dr Shareef."
},
{
"id": 120911,
"tgt": "What causes pain in joints and bones and change in facial features?",
"src": "Patient: My ESR rate is 33mm/hr and I m a 29yr old female, and I ve been having joint and bone pains every day from my shoulders down to my fingers and toes. I ve also been developing puffy bags under my eyes that don t go away and dry scratchy eyes that especially bother me in sunlight or glare. My lips also are gradually getting larger, and my facial features are changing in general. What could be the reason? Doctor: Hello,Your symptoms could be related to an inflammatory disorder. For this reason, I recommend checking ANA and ANCA titer antibodies, FR levels and GH plasma levels. You should discuss with your doctor on the above tests.Hope I have answered your question. Let me know if I can assist you further. Regards, Dr. Ilir Sharka, Cardiologist"
},
{
"id": 95562,
"tgt": "Medline endometrial thickness and dominant follicle is seen in right ovary",
"src": "Patient: ei m 30 yrs old last month i had gone for sonography of abdomen and pelvic in which endometrial echo is seen in midline and measures 5mm in thickness & a 14mm dominent follicle is seen in right ovary what does it means . & also my periods are very light only spotting occurs i want to know is there anything serious Doctor: Welcome to Healthcare Magic Please provide details about your periods. How many days is it usually for. Since when did it change. You have probably gotten this scan a few days after your periods have stopped. The thickness of the uterus inner lining is quite normal and there is a follicle which will ovulate soon when it becomes around 20mm. You need to get your Thyroid hormones checked if your periods are irregular. Also get LH/FSH ratio done. From the scan nothing is confirmatory of any problem as yet. It would be better to get further investigated ti rule out PCOD and Thyroid problems."
},
{
"id": 89990,
"tgt": "Suggest remedy for abdominal pain",
"src": "Patient: I had GI Bleed 2 years ago. Slow with anemia. Endoscopy and colonoscopy cleared everything with in 3 months. For about a year now diffuse lower abd pain intermittent, stabbing white to clay colored stools. 40lb weight loss. I don t take NSAIDS or other pain meds. Most stools are loose. I tolerate very little food. Mostly yoguot, non milk. I eat almond butter, fruit and vegetables in small frequent meals. Doctor: HIThank for asking to HCMI really appreciate your concern and looking to the history given here I could say that this is the chronic condition of diarrhea and now the stool examination need to be done, till then you can try Tan clinidium with Chlordiazepoxide three times in day. take care and have a nice day."
},
{
"id": 145304,
"tgt": "What causes dizziness,hand tremors and disorientation?",
"src": "Patient: I m 19 years old, aside from asthma, a soft systolic heart murmur and dizziness I m normally fit and well. The other night I was on first aid duty at the open air theatre for a status quo concert and just over an hour before the end I was sat in a chair in the first aid post and my head went *funny* I felt likeiI wasn t really with it and I felt lightheaded. I gradually got worse over the next hour as my hands started shaking which got worse then just stopped, I was confused and disorientated. I had barely any energy and it was difficult trying to move and I was having problems talking and at one point couldn t speak at all. I kept zoning out or there would be times when I knew people were talking hut didn t know if they were talking to me or even what they were saying. I was taking to the hospital by our ambulance crew where I had an ECG done which was normal and my blood sugars were normally but I m not sure about anything else. I don t remember a lot of what happened, some of this is what I have been told by my colleagues. I currently work in a theme park for which I do a 40 mile round trip on a moped as well as having several other first aid duties to cover this season so I would be very grateful for an opinion on this and what I could do to stop it happening again. Doctor: Hello!I read your question and understand your concern!In my opinion these troubles may be caused by epilepsy. But epilepsy is not commonly lasting for hours,but minutes and it can leave disorientation and weakness for hours after a seizure. Although repeated partial seizures can occur. Another cause would be a panic attack.I would recommend a EEG to check for epilepsy, and a brain MRI to rule out possible cerebral lesions.Hope to have been helpful!Best wishes Dr. Abaz Quka"
},
{
"id": 192421,
"tgt": "What causes discharge of whitish liquid from penis?",
"src": "Patient: I'm a male of age 19 and my private has just in the past few days started secreting a white-ish to yellow-ish liquid sometimes.There isn't any severe itching, and it only slightly burns when urinating. But everywhere I go online looking at these symptoms it says STD, I was checked for STD 5 months ago and have not been sexual active for 8 months, even then, I have not actually ever had Intercourse. I'm started to get worried and don't know what to do. I'm scared that I may have to get blood taken, and that's an extreme phobia of mind which induces panick attacks.The liquid is similar to that of when I got my ears pierced and they were infected. Doctor: Hello, If you are masturbating and you are able to produce semen and sperm, is there any resemblance? Therefore I suggest consulting a skin specialist for physical examination, diagnosis and treatment. Hope I have answered your query. Let me know if I can assist you further. Take care Regards, Dr. K. V. Anand"
},
{
"id": 128872,
"tgt": "What causes pain under the armpit and on shoulder?",
"src": "Patient: I am having a dull pain under my right armpit and right shoulder blade. My chest feels heavy pretty much all day. I am constipated and nauseated all day everyday. When I push on my stomach and intestines it hurts.A month ago I had my gallbladder removed and thought all these symptoms would go away but they haven t. So what else is wrong with me? I need answers so please any help from you would be great. Doctor: Hi. I understanded your question and I will help you. If I were your treating doctor.,I would firstly ask you to do abdominal ultrasound to rule out any serious abdominal problem. also you should know that shoulder pain and dyspepsia may persist for a period after cholecystectomy which is called post cholecystectomy syndrome.. I advise you to regulate your meals and avoid fatty meals. Also I need to exclude any problem in the breast as breast masses and mastitis lead to enlarged axillary lymph nodes and heaviness in the arm pit."
},
{
"id": 224846,
"tgt": "While on birth control pills, can one engage in sex without condom?",
"src": "Patient: Hello, I just started birth control about 6 months, Lo lestrine fe, then switched to Cryselle last month. I am recently sexually active. I use condoms because I tend to forget pills sometimes, however I haven t at all this month. I am wondering how safe it is to not use a condom since I m on the birth control? Would it be best to still continue to use a condom? Thank you. Doctor: Hi, I had gone through your query and understand your concerns.If you take regularly birth control pills, you can definitely participate in sex with out condoms. but it is always best to continue with condoms if you can, if you are not regular with pills.Hope this helps. Thank you."
},
{
"id": 21750,
"tgt": "What causes trouble in breathing after a valve replacement?",
"src": "Patient: Hello from Margaret My Mom is 80 years old. She had a valve replacement on December 29, 2010. She has been experience chest pain that comes and goes. She says when she experiences the pain it feels like she can t breathe. She takes oxygen at home and does help the situation Do you know what can be causing this? I was with her at her last Dr. s Appt and the cardiologist said something like it is more difficult for the heart to pump blood after valve replacement? My Mom weighs about 190 and is 5 feet6 inches tall. She has lost over 100 pounds over the last two years. She does not have a good appetite. I am concerned she does not adhere to a healthy diet when she does eat. What do you think? Thank you in advance for your help. Doctor: Hello , Though healthy diet is always a priority and should be adhered too at all times . The symptoms of breathlessness , pain are not explained by diet alone . Considering its 7 years since the valve replacement, it's strongly suggested to check it's status by doing a comprehensive 2d echo cardiogram and look for valve function whether there is any reason stenosis , degeneration, any leak which could very well explain the symptoms. Losing weight rather helps , as now the heart has to supply to less body surface area of body so again this is not explained by weight loss alone . Regards Dr. Mody"
},
{
"id": 155670,
"tgt": "Effective treatment for cancer and severe malnutrition",
"src": "Patient: My husband was diagnosed with cancer in October 2013. He had chemo and radiation for 7 weeks. He was recently hospitalized for severe malnutrition for 3 weeks. He will try to eat but does not swallow his food. All he is getting is tube feedings through the night. I am worried that the same thing is going to happen again from not eating. What can be done. Doctor: Thanks for your question on HCM.If your husband is not able to swallow than Total Parenteral Nutrition (TPN) is the only treatment we can give him.In TPN all the nutrients are given through larger veins of the body. Mostly through external jugular vein. So hospitalisation and central venous catheter is must . In TPN infusion contains1. Protiens2. Carbohydrates3. Lipids4. Multivitamins5. Minerals and trace elements. You can also give fruit juices, water, mashed food through naso gastric tube."
},
{
"id": 5070,
"tgt": "Irregular periods, severe hair fall. Trying to conceive. USG shows bulky ovary. Treatment to get pregnant?",
"src": "Patient: hi doc.. i am 33 years old female my height is 4''5' and weight is 58.....my problem is irregular periods and sever hair fall....for which i have been suggested usg lower abdomen...........and it shows my overy is bulky ..right overy measures 16cc and left overy 17cc.......i am not under any medication till now....this problem has started long back12 years ago................i am married for 4 years ...now i want to become pregnant...i want to know what r my chances and what medications should i take to get pregnant soon....my last period was 12days back...but it was scanty. Doctor: Hi,Thank you for choosing Healthcaremagic. Your irregular periods needs to be evaluated, you need some basic investigations like thyroid function, Follicular Stimulating Hormone and Leutinizing hormone FSH: LH ratio, prolactin level and random blood sugar levels. Husband semen analysis is very important. As your married life is 4 years, you need urgent evaluation. I hope this information has been both informative and helpful for you. In case of any doubt, I will be available for follow-ups. If you like my answers kindly rate it, and write a review as well. Please do not forget to accept it.Thank you,Wish you good health.Regards,Dr ArifYou can consult me again directly through my profilehttp://www.healthcaremagic.com/doctors/dr-arif-n-khan/65133"
},
{
"id": 152021,
"tgt": "My inner tounge is not functioning properly. I am 56. What could be the reason for this?",
"src": "Patient: sir, i am aged 56 i am unable to speak properly i feel my inner tounge is not functioning proper i feel cough is blocking in my throat Doctor: hello arun, unable to speak can be due to various reasons like paresis of tongue to vocal cord problems. i would advise you to meet a neurologist and get a through examination done. rule out hypertension also, as of your cough , adequate medicine will help it. if your inability to speak is due to an infection then with adequate treatment you will be fine soon. i hope i have answered your query. take care"
},
{
"id": 144730,
"tgt": "What could cause abrupt paralysis of the entire right side of the body?",
"src": "Patient: I have had a stroke. Within 24 hours i had recovered from the entire right side of my body being paralized. My gen. doctor thinks I had a tai, my neurologist thinks I had a stroke and a hospital in California where my son took all my records thinks I had a stroke, buy of a different type. How do I find out who is correct? Doctor: You are a lucky person to recover from th paralysis you suffered for 1 day. Medically , you have had a TIA which in colloquial terms means a stroke which recovered within 24 hours. Affection of brain function due to loss of blood supply means a stroke which could be transient or full. I think it is more important the treatment is better understood as the TIA can progress to a full stroke if not careful. TIA is a warning and keeping healthy would be a good idea to concentrate on now. Wish you all the best"
},
{
"id": 148110,
"tgt": "What is the medicine for nerve pain in hand after shoulder surgery?",
"src": "Patient: What type of medicine can be prescribed for nerve pain in my hand after shoulder surgery and a nerve blocker? I had that pain pump nerve blocker. Surgery was just December 26. Whole arm slightly burns and a lot of burning and random sharp pain in my palm and thumb area. I cannot sleep (worse at night) and the pain med (norco) does not relieve pain Doctor: hi, nerve pains are treated by drugs like gabapentin or pregabalin. They should only be prescribed by the doctor, as they have certain side effects, and are specially reserved for some conditions only. Hope this helped."
},
{
"id": 15490,
"tgt": "Circular, red rash on right side of rib cage, itching, gets bigger on scratching. Causes?",
"src": "Patient: Hello, few months back i had a small circular red what looked like a rash on my right side on my ribcage..it would itch ..the more i scratched it it became bigger ...then it went away ....and then i noticed a dime size dark rash under my armpit ....that also itches ..and the more i scratch the more it gets bigger ....what is this? Doctor: hi,the skin rash that you have is tinea corporis. armpits and groins are the commonest areas for this fungal infection. you need to apply topical antifungal cream twice a day for around 10-15 days. you can also take antiallergic tablets if there is lot of itching.the armpit area should be kept dry. wear loose cotton clothing to prevent moisture retention in the body folds. feel free to contact for further queries."
},
{
"id": 160557,
"tgt": "Suggest treatment for measles in a child",
"src": "Patient: My daughter Rouchelle is 14 months old and seems to have baby measels(Lexi her older sister had it at 16months). I just cant remeber what to do and not to do. She had a fever fit on friday night and the local dr gave her antibiotic and pain meds, what else should I do. Thank Philippa Doctor: Hello, Measles is a self limiting condition. Paracetamol and vitaminA are the treatment. Hope I have answered your query. Let me know if I can assist you further. Take care Regards, Dr. Shinas Hussain"
},
{
"id": 73096,
"tgt": "What causes interstitial Lung disease?",
"src": "Patient: sir, MY uncle has ILD-Interstitial Lung disease.from my research over google i found that the cause is important to be known. the cause wat i feel is due to breakage of acid bottle in bathroom which caused lot of smoke and that time he wasnot able to be normal for 15-20 minutes. then he was normal .may be that would have been the reason for this ILD. Please suggest wat we should do Doctor: Thanks for your question on Healthcare Magic.I can understand your concern. ILD (interstitial lung disease) can not be caused by single exposure of any chemical.Chronic exposure (for years) of chemicals can cause ILD.It is not possible to find out cause for each and every ILD patient. Some patients can develop ILD just because of aging (old age).So your uncle is not having ILD due to breakage of acid bottle.He might be having age related ILD. Better to start Perfinodone and N acetyl cystine (NAC) for ILD rather than searching for cause. Hope I have solved your query. I will be happy to help you further. Wishing good health to your uncle. Thanks."
},
{
"id": 14573,
"tgt": "Suggest treatment for itchy rash on the thighs",
"src": "Patient: Hi I have a rash going down my thighs it is very itchy and red , I have been treated for an advanced case of thrush this week due to being in a remote location with insufficient medical supplies and also have managed to have a small amount of tines on 2toes. Doctor today gave me Doric-ds ointment IAm traveling again in 3 days does this sound right Doctor: Hello,Thank you for posting on HCM.It seems you are suffering from Tinea cruris, a kind of fungal infection.I would suggest you to consult your dermatologist for proper management of the condition.I usually recommend proper course of oral anti-fungal drugs like Fluconazole or Itraconazole along with anti-fungal cream like luliconazole for local application at bedtime. You can additionally use antifungal dusting powder containing sertaconazole during day time and a soap containing ketoconazole for rinsing of affected areas. Take oral antihistaminics for itching as required.Maintain hygiene over those areas and avoid wearing tight undergarments.Hope your queries are resolved and wish you best of health.Kindly spare some time to rate my answer and drop your valuable review at the following link:https://urldefense.com/v3/__http://doctor.healthcaremagic.com/doctors/dr-hardik-pitroda/67169If__;!!Mih3wA!SBzm6_kI6hCZ58EPH6N_05MFfiPbxWXT0a2TJCdFQObRWm5mV5ur7hW9rW0j8A$ you require any further assistance in future, you can reach me directly through the above mentioned link.Thank youDr Hardik Pitroda"
},
{
"id": 83052,
"tgt": "Done back surgery for inactive lupus. Now have leg, hand, hip and back problems. Cause?",
"src": "Patient: the doctor told me I have inactive lupus . I have had back surgery, the pain has not went away. Actually it is alot worse. Now I have leg, hand , hip, and back problems. Could lupus be the cause of this. They told me the only thing that could be done is to manage the pain. I had to much scare tissue . It is very difficult to sit and stand for periods of time. I have gone to many doctors and they all said the same. It is very difficult to not know what is going on. I don t want this type of pain for the rest of my life, I am only 46. Doctor: This could be lupus causing pain die to arthritis."
},
{
"id": 194171,
"tgt": "Suggest treatment for lack of libido and erectile dysfunction",
"src": "Patient: i am 30yrs, height 175 cm, weight 85 kg, male. i never had any medical problems except the occasional cold or fever. My problem is that i am not able to show interest in sex with my wife. during sex also the penis take long time to get erect and then loose the erection very soon. due to this i and my wife are not able to enjoy sex. i used to masturbate a lot when i was unmarried. kindly help me. Doctor: Hi, Masturbation won't be the cause for your issue. Low interest in sex can be due to hormonal imbalance or anxiety. Going for a laboratory investigation will help you. Hope I have answered your query. Let me know if I can assist you further. Regards, Dr. S.R.Raveendran, Sexologist"
},
{
"id": 174419,
"tgt": "Can cetrizine be used to treat cough and cold for my child?",
"src": "Patient: my 5 months old daughter developed cold & cough ,i gave her syp cetrizine 1/4 tea spoon &three times steam she got relieved with cold & throughout night did not even cough but again in morning she started coughing not much but yes she was coughing what medicines or syp . is to be given to her Doctor: Hi...Thank you for consulting in Health Care magic.Cough and cold are viral 95% of the times in children. For cold you can use anti-allergics like cetirizine and for nose block, saline nasal decongestants will do. Paracetamol can be given in the dose of 15mg/kg/dose (max ceiling dose 500mg) every 4-6th hourly, that too only if fever is more than 100F. I suggest not using combination medicines for fever, especially with Paracetamol.For cold you can use Cetrizine at 0.25mg/kg/dose every 12 hourly for 3 days.For nasal block, plain saline nasal drops will do, every 4-6th hourly to relive nasal congestion.Hope my answer was helpful for you. I am happy to help any time. Further clarifications and consultations on Health care magic are welcome. If you do not have any clarifications, you can close the discussion and rate the answer. Wish your kid good health.Dr. Sumanth MBBS., DCH., DNB (Paed).,"
},
{
"id": 196733,
"tgt": "Suggest treatment to increase the size of the penis",
"src": "Patient: hello doctors how r u i hope u r all is well. my questio is that i am 23 year old and my penis is 6 inc but i like to grow my penis longer then natural position plz give me advice which type of oil or medicin i can use. for growthing penis fast please answer my mail question on my personal mail id my id is ( YYYY@YYYY ) i am waiting for ur reply thanks for all of u Doctor: Dear service userThank you for writing to Healthcaremagic.I have gone through your question and understood your query regarding size of your penis.Dear friend, the concern regarding penile size is very common especially among young men. The width and length of erect penis vary greatly from one individual to another. Your concern to increase the size of penis is seems unrealistic. Large penis gives better sexual satisfaction to partner is a very common myth.The fact is that the size of penis has no relationship with sexual satisfaction of the partner. The lack of knowledge causes fear and apprehension. Another fact is that only outer one third and the front wall of vagina have nerve fibers and hence sensitive to stimulation and contact by penis. Rest of inner two third vagina is largely insensitive. So if you want to arouse your partner during sex, you should focus on the areas of maximum nerve endings hence clitoris and inner lips or labia minora, the outer wall and outer two third of vagina. Now you can understand that female sexual gratification, the size of erect penis could be anything from two inches or more! (Your is six)You remember that what important is strength and not the length! Entry and to and fro continuous movements are adequate. It is men's technique, not his penis size that characterizes the lover in him.Avoid these unrealistic expectations and be happy.I hope this will help you.If you have any further questions I will be happy to help. Wish you good health."
},
{
"id": 100723,
"tgt": "What could cause continuous coughing and breathing problem?",
"src": "Patient: Yes I have been to 2pulmonary doctors, a eye, ear and nose doctor, and an asthma/allergie doctor. I also go to an oncologist. None of them have helped me. I cough all the time. Sometimes I cough till I choke and can t breath. After a really bad spell I have trouble breathing for awhile. I also have trouble breathing when I do normal activities. I m tired most of the time. Do you have any suggestions as to what might be wrong with me? Doctor: breathlessness on doing normal activities may in fact go in favor of cardiac issues. there is a condition called CCF, congestive cardiac failure. Basically the blood is pumped out less from the heart to the body, and also some may be getting filled in the lungs.The best way is to get an echocardiography done to confirm the diagnosis. If proven accurate, the diagnosis will warrant initiation of diuretics that may help youBest of luckDr Mittal"
},
{
"id": 7213,
"tgt": "Will antisperm antibody help me to conceive ?",
"src": "Patient: I AM 25 YRS OLD AND GOT MARRIED IN 2009. BUT DO HAVE A BABY. I CONSULTED THE GYNECOLOGIST ANS SHE SAID MYSELF AND MY HUSBAND HAVE ANTISPERM ANTIBODY. MY COUNT IS 28 U/ML MY HUSBAND COUNT IS 55 U/ML. WAT IS THE SOLUTION FOR THIS PROBELM. PLS HELP ME TO OVER COME THIS BY USING SIMPLE DRUGS. IM VERY SACRED ABT THIS. Doctor: Hi Welcome to HealthcareMagic Anti sperm antibodies are cause of infertility. Simple measure to treat this is to have sexual contact with condoms only for 2-3 months ( Condom therapy ). Prevention of entry of sperms in your body for certain time will lead to disappearance of antibodies from your body. After 2-3 months you can plan pregnancy. If it doesn't work then there is medical treatment with limited success. IUI can be tried too. consult an infertility specialist. Take care."
},
{
"id": 10220,
"tgt": "What should I do to treat hair fall?",
"src": "Patient: HelloI have been facing problem of hair fall since last couple of months. I never had this problem except in some seasons. These days I am applying Figaro Olive Oil...and I have changed my Shampoo to L'Oreal Paris. Could you please advise If I'm opting right way or should I consult to related doctor? Doctor: Hello and Welcome to \u2018Ask A Doctor\u2019 service. I have reviewed your query and here is my advice.Significant hair loss is loss of more than 100 hairs per day. If you have significant hair loss, please do consult your family physician/dermatologist.Hope I have answered your query. Let me know if I can assist you further."
},
{
"id": 85374,
"tgt": "Is itching the side effect of triptomer 25mg?",
"src": "Patient: warm welcome , i have take triptomer 25mg for past 6 months daily night once. time being i having an itching in my hole body. i think it would be leads to itches i have an tingiing effect in my head ie right side only .doctors advice to took 25mg triptomer for day on night only Doctor: Hello,Yes, you are right although rare but tryptomer can cause itching over the body. For whatever purpose you are taking thus medicine i suggest you to consult your physician to run some tests for liver dysfunction and change your prescription accordingly.Hope I have answered your query. Let me know if I can assist you further. Regards, Dr. Sameen Bin Naeem, General & Family Physician"
},
{
"id": 65298,
"tgt": "What causes painful cyst under the armpit?",
"src": "Patient: Hello, i am 36, female, I am 5'8\" 270lbs. I am a Type 1 Juv. Onset Diabetic, insulin dependent. Last sugar was 146 and slowly coming down. I have been seen already for an abbess (cyst) in my right armpit.. but my symptoms have changed. The cyst became extremely painful tonight and it even deformed from its original shape, went from a smooth ball to a lumps all over the general area. It popped and drained starting from green, then to the brown/grey and then to pure dark blood. I have become extremely nauseous and have even vomited. and have a general feeling of being not well plus a headache and stomach ache. what should I do? Doctor: Hi I am Dr Fahim, General Surgeon. I will help with your problem . . . Its sad to know that you are suffering from this condition. But encouraging thing is that it is fully curable and you will be fine in few days time.You are having an abscess in the right armpit, which is more common in diabetics. Since the abscess ruptured so the pain will reduce, but you mention that some more area is also involved. Its seem there is localized cellulitis. If such patient walk in my clinic, I plan for local treatment of the abscess under local anaesthesia. All the pus is drained and any necrotic tissue removed, so that focus of infection is eliminated. this process is repeated on daily basis for approximately 5 days. Second systemic antibiotic cover eg (amoxillin&clavulanate) + analgesic eg Paracetamol and good control of diabetes.I hope will help you with this problem. Do rate my answer if you like it.Regards"
},
{
"id": 147384,
"tgt": "What does lacunar infarcts in posterior aspects of corona radiate bilaterally indicate?",
"src": "Patient: my husbands ct scan report says that there are old, lacunar infarcts in the posterior aspects of the corona radiata bilaterally. and that there is a moderate amount of patchy hypodensity in the periventricular and deep white matter of both cerebral hemispheres. what does this mean? Doctor: HIThank for asking to HCMI really appreciate your concern, you have not mentioned your husband's age and the clinical complain for what he has been gone for CT scan here the given report need to be correlate with the clinical complaints, just on the basis of given report it can be said that such findings are seen in old age patients and with the patient whom had history of TIA (Transient ischemic attack) due to hypertension, hope this information helps you, have nice day."
},
{
"id": 211417,
"tgt": "Depression with varied mood swings. Hypersensitivity to smell and sound. How do I solve this?",
"src": "Patient: My son has never been in trouble he turned 18 last Christmas and decided to try weed. He now smokes and recently got in trouble. Problem is he was depressed prior to all this. Had ups and downs in his mood. During argument about using my car he said some very dark things against my husband and I he repeatedly kept saying he will kill himself. Things he said were very graphic. He is refusing help from a doctor, I feel lost like there's nothing we can do. What are my options? He is destroying himself and we can't do a thing. He has bouts of anger. He in hypersensitive to smell and sound. Doctor: I am sorry for all the trouble you are suffering. Your story is heartbreaking. The first priority is to determine if he poses a physical threat to himself or others. Physical safety is at risk.It is imperative that you discuss this with a mental health professional, even without your son. An experienced clinician will recognize these symptoms and know exactly what to do. i wouldn't waste any time as I am concerned for all of you. Good luck."
},
{
"id": 135900,
"tgt": "Suggest remedy for swelling,pain and redness in right leg and ankle?",
"src": "Patient: my right leg and ankle (fused from an old accident) has been red, hot, very swollen and painful. one doctor did an MrI said it was vascular no infection. another treated me with antibiotics for 10 days. No improvement.I am seeing a vascular surgeon this Wed. I feel sick, weak, tired. Who is best suited to diagnose and treat me.I have had 5 surgeries on the ankle to help the pain over 20 yrs. I am 80 and just had colon cancer surgery. Doctor: It needs to be investigated. With a history of surgery for colon cancer,possibility of secondary deposit should be ruled out. A biopsy from the hot spot may have to be done. PET examination should be helpful."
},
{
"id": 152925,
"tgt": "Is radiation treatment standard routine for stage 3 NHL?",
"src": "Patient: Hello, I am a 49yrold female with stage 3b NHL(DLBCL) diagnosed by tru-cut biopsy in nov last year & DUE FOR MY 5th chemo regime(R-CHOP) on 25-01-2011. The hematologist treating me says after the 6th cycle a PET SCAN is due & he says i will definitely need to undergo 2/3 wks of radiation treatment. Is this the normal standard routine? Pls help & advice! sincerely yours, r.jayanthi Doctor: Hi, Radiaiton for lymphoma dpends upon the disease and histopathological type of lymphoma . and if you have been advised for radiaiton ,plz take radiation treatment otherwise it will recur."
},
{
"id": 224739,
"tgt": "Is emergency contraceptive pill safe while breast feeding?",
"src": "Patient: HI Dr I am 26 yrs old, I have 8 months kid, I am still feeding her. My husband is coming down this friday i would like to know is it safe to use emergency contraceptive pills. Will it also delay my periods. He will be in town for 15 days. So please do suggest me with the name of the pill. Doctor: Hi,If you are feeding your baby, it is better not to use emergency pill contraception; you may choose one of the barrier methods or go for insertion of an IUD as an emergency contraception. Emergency pills do contain excess hormone and this can cause breakthrough bleed as well as delayed periods and hence as such is not advisable on a regular basis. Emergency pills should be used only once or twice a year. Hope your query has been clarified. Wish you good health."
},
{
"id": 220464,
"tgt": "Why did i feel air passing through my vagina in pregnancy?",
"src": "Patient: Hello, Im a 25 year old woman and Im rolling up on my 9th week of pregnancy. During the night I woke up to air being passed through my virgina. Yes it felt wired and I was questioning and a little worried with this being my first pregnancy. So what I want to know is was that normal , why does that happen ,and of course what makes it happen too? Doctor: Hi, Dr Purushottam Neurgaonkar here. I welcome you to HCM VIRTUAL CLINIC. I have gone through your question, and I think I have understood your concern. I will suggest you the best possible treatment options. Please do not panic. It is not a thing to worry about. It is due to passing out of air from the vaginal passage. In some women there is laxity of the muscles around the vaginal opening , and it gets more so during pregnancy due to hormonal changes. This leads air to enter the vagina and with change in position or increase in abdominal pressure while laughing or coughing it comes out with some noise . So do not worry. I will suggest you to get used to pelvic muscle exercise called as Kegelle' s exercise, this involves tightening and relaxing the muscles around the anus and vagina. This will help to reduce the problem .I hope this answer helps you. Thanks Dr Purushottam Neurgaonkar."
},
{
"id": 126815,
"tgt": "What can cause pain in the right arm?",
"src": "Patient: I have a pain in my right arm, high right above my bicep. I end up Geelong it in my shoulder abs sometimes in my wrist very lightly. It s very painful to the touch. Butt sometimes it just aches dully. I m familiar with regular muscle soreness. I can t exactly tell HOW this feels different, but it does. Thank you. Doctor: Hi, It might be a simple musculoskeletal pain. As a first line of management you can take analgesics like Acetaminophen or Tramadol for pain relief. If symptoms persist, you can consult an orthopedics and get evaluated. Hope I have answered your query. Let me know if I can assist you further."
},
{
"id": 164780,
"tgt": "What causes lower abdominal pain in teens?",
"src": "Patient: my neice is 12 yrs old and keeps experiencing intermittent lower abdominal pain. Went to the ER they ran all sort of tests, mri, sonogram, uti test, gallbladder test - and everything was good, howeveer, she keeps getting this bad pain in the abdominal area only. Doctor: hi, if all test are normal like ultrasound abdomen and MRI 10 chances of colicky pain are very high. Colicky pain can occur in children more frequently. Colicky pain tends to go away in short time. There is no need of any antibiotic for this. Take care."
},
{
"id": 137793,
"tgt": "What causes swelling in foot and difficulty in walking?",
"src": "Patient: Hi , I have a swollen foot , and it hurts when I walk on it , under the skin I can see clear little spots lik bubbles under , it feels like a bubble around three toes , between my big toe and pinky toe, I didn t have any accidents , it just started two days ago out of where , I don t know what to do ? Doctor: Dear Patient.Are these hard or soft. If hard possibility of corn is there but if it is soft then these could be infective or water filled blisters. I feel u should see your doctor and get this examined. I hope that helps.Thanks"
},
{
"id": 188439,
"tgt": "Removed wisdom teeth. Now painful on one side. Hurting jaw. Extracted tooth spot is deep, canker sores. Treatment?",
"src": "Patient: i had my wisdom teeth taken out 6 days ago. around the 3rd day after extraction, one side of my mouth started becoming extremely painful. the pain extends all the way up half of my jaw. why does my jaw hurt so badly? also, the gum on one of my back molars completely pulls away. this is painful also. the site of where the tooth was taken is extremely deep and has canker sores. Doctor: hiii, I have gone through your query, and I can understand through the painful situation you are going from because post extraction pain as per in your case is call DRY SOCKET which is a very painful condition,occurs because of secondary infection to the extraction site.in this condition you must visit your dentist for the cleaning of the extraction wound,because this can't be handle at only home care.your dentist would clean the area thoroughly and place the medicated pack to the wound.you have to take antibiotics and pain killer medication prescribed by doctor to reduce the inflammation and pain. some tips that you have to do at home are : 1) Add 1 tsp of salt to the luke warm ( not so hot )water and do the gargle with it, 2-3 times a day.2) if you smokes,then you strictly have to stop smoking for some days until the wound heals.3)try to keep mouth as clean as possible.Don't worry the pain goes away as soon as the dressing is given and proper medications are taken,the wound will heals by itself."
},
{
"id": 200709,
"tgt": "How to remove itching cracks on foreskin near tip of penis?",
"src": "Patient: My foreskin near tip of penis has some cracks like impression with itching sense while pulling back of foreskin there is burning sensation and the cracks are became widened. I am cleaning reguraly with normal water ,applying Boroline so that skin become soft atleast but it is not healing prmanently. Kindly suggest the suitable cream and tablets . My age is 49 . Kindly help me to get out of this problem. Regards, Sabyasachi Das Kolkata Doctor: Thanks for asking in healthcaremagic forumIn short: Dryness can cause thisExplanation: Usually balanoposthitis(infection of foreskin and glans) can cause inflammation and scarring of foreskin which contracts afte some time causing cracks when tried to retract. SO, please visit a doctor for ruling out infection. For time being you can apply oint soframycin locally for relief. Hope this helps you. Please let me know."
},
{
"id": 113925,
"tgt": "How to cure back pain ?",
"src": "Patient: null Doctor: welcome to healthcaremagic from the description it seems that you are having root pressure and causing the pain and other symptoms the pain will not go away unless the pressure is removed consultation with orthopaedic /neuro surgeon will help hope you get relief soon"
},
{
"id": 124425,
"tgt": "Are weakness in legs and hands due to masturbation?",
"src": "Patient: i am 32 years old and my question is, doing masturbation my legs and hands gone weaker i feel weakness in my hand my hand and legs ,sometimes i feel pain in my leg veins. So it due to masturbation or something else, how do i overcome it to my original form Doctor: Hi, Yes, you are right. This particular act of what you do leads to lot of consumption of energy and isometric contractions of muscle leading to setting in of fatigue. Kindly avoid doing more often to conserve energy. Hope I have answered your query. Let me know if I can assist you further. Regards, Jay Indravadan Patel, Physical Therapist or Physiotherapist"
},
{
"id": 102285,
"tgt": "Why am I suffering from dust allergy, stuffy nose, sneezing and breathing problems post sinusitis surgery?",
"src": "Patient: Hi, I am knklI Under gone Sinusitis surgery last year, but post surgery got relief for only 3 months. Again I am suffering from dust allergy, stuffy nose, frequent sneezing & breathing problem.Please suggest a way for permanent relief.Thank,khkk Doctor: Allergy whatever you have cannot be treated permanently, but you can take precautions like using cotton clothes, avoid carpets at home. try to use masks during dusting process. sinusitis surgery will not be of help, drink warm water during the attack, steam inhalation with plain steam, do not add vicks."
},
{
"id": 123776,
"tgt": "Suggest treatment for numbness and tingling sensation in the leg and foot",
"src": "Patient: I am having a problem with numbness,tingling in leg and foot , mostly in back of leg, pain seems to be centered in back of knees. Yesterday my leg went completely numb and it felt like I had a ball, the size of a baseball on the top part of bottom of my foot just above the arch. I have had disc surgery before for a splasher disc ( L5S1). But it wasn t this painful. Can you help? Thank you in advance for any information you can offer. XXXX Doctor: Hello, With the history, I understand that there is muscle weakness in the lumbar spine region post surgery and needs exercises to be performed like the core stability, spinal muscle strengthening, lower limb strengthening as well. In my clinical practice, this is the least common I have seen with. But after a thorough clinical examination and assessment, they respond well to the spine rehabilitation program. Hope I have answered your query. Let me know if I can assist you further. Regards, Jay Indravadan Patel, Physical Therapist or Physiotherapist"
},
{
"id": 196308,
"tgt": "What causes swelling near anus?",
"src": "Patient: Sir, i'm 33 years old male. Ht-178cm. Wt-82Kg. No major problems before. I'm a healthy person. For the past one month i found swelling near anus. It pains once i press it. No major blood stools. Two weeks back once blood found on stool, but, not very much. Please guide me what to do. Thank you. Doctor: Hi most likely it is a small abscess--pus filled. You will need an antibiotic course. Please see your doctor about this--regards"
},
{
"id": 222013,
"tgt": "What are the chances of pregnancy with delayed periods?",
"src": "Patient: I take birth control and have a monthly period. Admittedly, I have missed some pills recently and my boyfriend and I do not use protection. I took my last active pill on Tuesday, November 2nd. I start my active pills again tomorrow Tuesday, November 9th. I have not had my period all week. I am wondering what the likelihood is that I am pregnant and what I should do at this point. Doctor: Hello dear,I.understand your concern.In my opinion as you have missed the pills and also had unprotected sex there is risk of pregnancy.As you have missed the period you need to do a urine pregnancy test.If the test is negative then there is no chance of pregnancy.Whatever may be the result consult your doctor for examination and for furthur management.Nothing to worry.Best regards..."
},
{
"id": 137591,
"tgt": "Suggest treatment for broken tailbone and ruptured discs",
"src": "Patient: I had a terrible fall landing straight down on my tailbone. I broke my tailbone and ruptured discs. I am having a lot of pain down legs and in neck and head. Should I go to a neurologist? Or should I start with a DO as a primary care physn? I got chiro, and then stopped everything to just try to be stable and it s worse. Doctor: Dear patient You have got trauma to your tailbone and there is possibility of fracture. But how you were diagnosed with tailbone fracture and ruptured discs? Is it self diagnosis or based on investigation? If no investigation is yet done I would advise Xray of sacrum anteroposterior and lateral views to confirm diagnosis. Ruptured disc can be diagnosis after doing mri examination of spine. If your Xray is showing fracture of tailbone treatment is 1. Use of tyre tube filled with air to avoid pressure on tailbone for one month and soft pillow 2. Tab diclofenac 50mg twice a day for pain relief 3. If it's significantly displaced manual reduction under image intensifier can be done. If mri is showing ruptured discs you need to consult expert spine surgeon with report. All the best."
},
{
"id": 41701,
"tgt": "What causes the inability to conceive even after a follicle study?",
"src": "Patient: Hi, doctor i`m shalini 24 Fe 5.5 h, 74 kg, i have under gone 3 cycle of follicle study 1st cycle was not clear it got ruptured in 13 mm now my follicle size is 20 mm and got ruptured my gynecologist advised to have sex and susten tab from 18th day,we kept relations as advised by doctor , but on 30th day of my cycle i got periods, i m worried whether i will get conceive or not.. Also my husbands semen count is 110, . please help me out. Doctor: Hi welcome to healthcaremagic.I have gone through your question.As your follicles are developing but it not fertilized by sperms so i would advise to go for histosalpingography HSG to rule out tubal blockage.If report of hsg is clear then try naturally in fertile period for atleast 4/5 cycles.Then consult gynecologist for further treatment IUI Intra uterine insemination or IVF invitro fertilization.Hope i answered your question.Would be happy to help you further.Take care."
},
{
"id": 198755,
"tgt": "What causes a sudden loss of erection from penis?",
"src": "Patient: Good Evening Doctor....Suddenly i have lost erection in my penis...some 10 -12 days back i had sex but from last 5-6 days this problem suddenly appear ? Please suggest what to do and also suggest the medicine for penis erection ? My age is 30 yrs and my email address is YYYY@YYYY Doctor: DearWe understand your concernsI went through your details. You are just 30. Please stop being panic. There could be nothing wrong. Recall what happened when you first lost your erection. There could be some stress, physical fatigue or anxiety related incident. Please do not generalize. That stress, physical fatigue or anxiety related incident was an one time affair. If you obsess with it, the erection problem can happen again and again.I suggest you to keep away from sex for another ten days. Don't vehemently try again and again. You may not succeed because of the underlying anxiety condition. You can try relaxed after strictly 10 days. Don't worry. You will be a success.If you require more of my help in this aspect, please use this URL. http://goo.gl/aYW2pR. Make sure that you include every minute details possible. Hope this answers your query. Available for further clarifications.Good luck. Take care."
},
{
"id": 208319,
"tgt": "How to control anger due to stress and fatigue?",
"src": "Patient: Dear Sir, My life is quite fast means i wake up 5 o'clock in morning and travelling every day 150 km for job and returning back to home arround by 7 pm. My sleeping time is 11 pm to 5 am only. Now i m always feeling lazy sleepy and most important thing is i m geeting anry fast and whenever i m getting angry i always throwing so many things some of them are break also. Now please help m what to do to control my short temper. I m always shouting and speaking without thinking at the time whenever i got angry. Please suggest m what i have to do and to whom i have to consult personally Psychiatrist or mental doctor.?? Lalitkumar Doctor: Dear Mr. Lalit,I went through your query in detail and can understand your concern regarding your symptoms. Feeling lethargic, and irritable can be symptoms of underlying depression. It is more plausible especially since you are feeling stressed with your job currently.You should consult a psychiatrist for complete evaluation and initiating treatment. Ant-depressant medications like escitalopram would be helpful in controlling your symptoms. In addition you can consider relaxation techniques like deep breathing and progressive muscular relaxation which will help your relax and feel better.I do hope that I was able to answer your query. Best wishes for a speedy recovery."
},
{
"id": 204178,
"tgt": "Why is Zocor recommended while on Prozac for depression?",
"src": "Patient: I want to know about Prozac and Zocor? Which is an inhibitor and which is an inducer and why would I stop the Prozac that a client is taking for depression that is taking Zocor? I am so glad that you are responding. What role does CYP450 and 3A4 play? I will not pay for this. I will continue to search. Doctor: Hello and Welcome to \u2018Ask A Doctor\u2019 service. I have reviewed your query and here is my advice. Prozac is an enzyme inducer, and it causes metabolism of Zocor. That's why it should be avoided. Hope you got your answer."
},
{
"id": 178471,
"tgt": "What causes cramping, urination pain with frequency and rectal bleeding?",
"src": "Patient: My daughter is almost 12 years old. Has not started her menses. Has been complaining of cramping most all day. Now she s saying that it hurts just above her pubic bone when she is urinating. She has felt the need to urinate all day, even right after she has just emptied her bladder. There is a little blood on the paper when she wipes and a few drops have dropped in the toilet. Is this just the start of her menses, or should I get her to her PCP? Doctor: It could be the start of her menses, and teens usually do get more cramping pain than older women because some of the cycles may skip ovulation. But, if the symptoms persist for more than a week, it would be better if she got a physical examination done by a gynecologist."
},
{
"id": 218018,
"tgt": "What is the cause of buttock pain with facet disease?",
"src": "Patient: My leg or knee gave out on me after that happen my whole leg as slowly turned outward, especially my foot, very noticeable...pain in opposite buttock...nothing shows on Mri of the hip, the spine shows facet disease, an old fracture. Nothing on the pelvis...What could cause this? Doctor: Hi, after going through what you have written, it is possible that you have an arthritis (inflammation of the joints) or misalignment (called listhesis) of the spine. This can cause pressure on the nerves that supply the legs in turn causing weakness and pain. If I was seeing you in clinic, I would recommend an MRI of the spine. Treatment initially involves pain control with Acetaminophen (Tylenol) :25-650 mg every 4-6 hours as needed. Do not take more than 3,000 mg of Acetaminophen over a 24 hour period. You can also try Ibuprofen (400-800 mg three times a day with food)- it has an anti inflammatory effect. Do not take Ibuprofen if you have a history of chronic kidney disease, bleeding from your gut or severe reflux. If this does not help other pain medicines for nerve related pain (like gabapentin, pregabalin and duloxetine) can be tried. I would recommend physical therapy and based on what the MRI shows refer you to a neurosurgeon. Since MRI's and pain medications need to be ordered by a doctor (except Acetaminophen and Ibuprofen which are over the counter), I would recommend seeing a general physician or an orthopedic surgeon as a start.Please let me know if that was helpful. If it was, please take a moment to provide a rating. Thanks!"
},
{
"id": 74821,
"tgt": "Does non-veg diet help recover lymph node TB?",
"src": "Patient: Dear Dr...., My son, 25 years old, non-vegetarian by choice has been suffering poor liver health, always susceptible to pancreatitis etc. Of late he has been diagnosed with lymph node tuberculosis. Should he turn vegetarian or does a non-veg diet help him in his recovery with medicines for tuberculosis? Please advise...Thanks... Doctor: respected user, hi I evaluated your query thoroughly.* Certain ingredients of non-vegetarian food are known to damage liver, pancreas, stomach , intestines & general body systems in immunocompromised persons .* I will recommend vegetarian diet with high protein intake ( if Liver function are under control ) for better recovery with TB.- also adequate content of iron, vitamins, minerals, trace elements, anti- oxidants, carbohydrates boost up recovery.- total abstinence from alcohol / smoking / any abuse substances are cornerstone of the management along with any diet & medicines.Hope this helps you.Welcome for any further queries.Thanks for using Health care magic & giving me an opportunity to assist you.Wishing him fine recovery.Regards dear take care."
},
{
"id": 200824,
"tgt": "What causes blood in semen while having diabetes and enlarged prostate?",
"src": "Patient: I m a 67 yr old male with coronary disease, diabetes, high blood pressure & an enlarged prostate. All of which I on medication. I m not sexually active. Recently I noticed blood in my semen. I will be making an appointment to see my urologist but would like to know what could be the cause & how is this condition treated? Doctor: Thanks for asking in healthcaremagic forumIn short: Infection or injury might have caused thisExplanation: An infection of urinary tract or an injury over there can cause bleeding like this. So, visit a doctor to get yourself checked for the problem. Hope this helps you."
},
{
"id": 23371,
"tgt": "Can 'arkamin' and 'minipress-xl' be taken after renal transplant?",
"src": "Patient: i am 38 yrs (female) & undergone renal transplant in 2003. i was on hypertensive drugs for last 8 years. i was on drugs aten50 BD, Dilzem 60 BD, Losar 50 BD for hypertension. my regular levels of blood pressure was maintained at 140 to 150/ 80 to 90. for last 4 months my BP levels have shot up to 180 to 185/ 80 to 90. Doctors have asked me to stop Dilzem 60 & Losar 50 & added new drugs arkamin 2 tablets thrice, minipress xl 5mg BD & also added Asomex 5 mg once a day in morning. i want to know if this conbination & drugs are OK for post renal transplant patients like me. Please advise. Doctor: hi noted all your details dearthe combination given to you by your doctor is perfect one for renal patientsyour target bp should be 130/85mm of hgcalcium channell blocker amlodipine is highly safe in renal patient try to avoid a prescription of losar by any cardiologist in future as often in renal patient with deranged lidney parameters at 1 antagonists are not indicated you current nmedication is a good onetake your bp every third day and maintain its recordstry doing regular exercise 45 minutes a dayavoid too much fat and oily foodtxhave a good recovery"
},
{
"id": 65805,
"tgt": "What causes a discharge filled lump on top of the arm?",
"src": "Patient: Hi Dr, I have a lump on the top off my arm when squeezed a white discharge comes out ( there is a smell) has been there for yrs, each time it's squeezed ... It seems no more comes out till it builds up and it becomes a prominent lump once again. Now it's bigger then before hasn't been touched in a while, it isn't painful until squeezed until it goes from white discharge to clear then a bit of blood. What is it? And how do I get rid of it? Regards Nikki Doctor: Hi, this is a Sebaceous cyst or a Dermoid cyst.It is a common swelling in all parts of the body. The are cysts collected with Sebum or secretions of the Sebaceous glands in the skin. The only definitive treatment is removal by Surgery. Pinching it and squeezing it are not advisable as this can lead to infections. It can cause redness and pain. I suggest you get it removed by a small Surgery and can be done by any Surgeon under local anaesthesia without getting admitted to the hospital. So, I suggest you get it removed and get rid of it.Take care, Dr Rishi, New Delhi, India."
},
{
"id": 195821,
"tgt": "What causes breaking of frenulum with bleeding?",
"src": "Patient: hay, me and my girlfriend were just about to have sex and she gave me a hand job but in the process my frenulum broke and there was alot of blood! we haven't had any intercourse for about a week now is there any chance it will break again when if we were to have sex again? Doctor: Hello and Welcome to \u2018Ask A Doctor\u2019 service.I have reviewed your query and here is my advice.For some individuals, frenulum is very thin and when it breaks, leads to bleeding. Next time also, you may face the same problem.Effective treatment is removal of foreskin (circumcision). My suggestion is please consult general surgeon for more suggestions.Hope I have answered your query. Let me know if I can assist you further.Regards,Dr. Siddartha"
},
{
"id": 58645,
"tgt": "Have hernia. Scan shows mild fatty infiltration liver, no free fluid",
"src": "Patient: Good morning doctor, my name is Martin, I am suffering from hernia.Yesterday I have scaned it , from the result:- hepatobiliary system says that Echotexture is increased diffusely(mild) what it mean, and the impression was:-1) Mild fatty infiltration Liver, 2)Normal billiary system, pancreas&spleen, 3) Normal kub areas, 4)No free fluid. Thanking you Martin. Doctor: Hi Martin ! Most often the ultrasound result shows fatty liver, which is not to be worried about. However, one has to go for a liver function test, and abstain from alcoholic beverages, apart from avoiding fast fried food. You should go for regular exercises and lot of roughage diet with enough liquids. Hernia is not related to fatty liver, and its treatment is surgical for which you will have to consult a surgeon. Good luck."
},
{
"id": 26724,
"tgt": "What cause dizziness, headaches and difficulty to breath?",
"src": "Patient: Hello, I am a female, 31, and 3 weeks ago I travelled to Mexico City, the first 5 days I was ok but then started feeling very dizzy with a heavy headache and difficulty to breath. (This was on June 26th) I went to the doctor and she said it was low BP (80/60) and gave me some drops to feel better, I felt slightly better but kept getting very dizzy to the point I could not even walk more than 5 min. I went to the cardiologist and the ear doctor and they both said I was ok, and gave me some Medicibe for the dizziness, which helped a little but u was still feeling I could not breath and I got insomnia as well. Then went to the ER and my blood results were ok and they sent me home. I had to take a plane back to the U.S. And even though I was very dizzy I did ok in the flight.( July 5) Doctors in Mexico though it was related to the high altitude in Mexico (8,000ft) and once I got to lower altitude it would get better. Before my trip to Mexico I participated in a triathlon (June 14) and it was really rainy and muddy, I reached Mexico on June 20th and the next day I participate in a half marathon but I could only run 8 miles and felt aweful (June 21), the next day I was ok. I have been a runner for 10 tears and t was not my first long distance run and a triathlete for 4 years. In June 5th I started getting some fever (99.2 F) and it goes away when I take paracetamol or ibuprofen but then it comes back ( usually in the evenings, around 5-6 pm), I went to the doctor and I took an urine test and a thyroid one, still waiting or results. I have had a heachae ( in the frontal part) for 2 weeks now, it occasionally goes away but comes back with the fever and have been feeling really tired, also the lights bother me a lot and can t stand the computer for more than 4 hours. I am still not sleeping 100%, keep waking up every 2-3 hours) What can it be? We had a tick problem at work and I m worried it could be related. Thanks! Monica Doctor: Hello Monica!Thank you for asking on HCM!Regarding your concern, your symptoms seem to be related to a chronic infection. The episodes of fever are very concerning about an infection.I would like to know if you have noticed any strange lesions in your skin, that may lead to a tick bite. Usually the first signs are skin lesions and than the other symptoms occur. The other symptoms (dizziness, fatigue, headache, difficulty breathing) could be related to a tick bite or any other chronic infection, affecting multiple organs. Anxiety could also be related to these episodes, but it can't cause fever. I recommend consulting with an infection disease specialist and performing some general blood tests (complete blood count, inflammation tests, blood electrolytes) and some specific tests for chronic infections (like Lyme disease). A brain MRI may be necessary to exclude possible intracranial lesions. Hope to have been of help!Best wishes,Dr. Iliri"
},
{
"id": 37786,
"tgt": "Suggest treatment for vomiting/sour mouth while on medication for gland tuberculosis",
"src": "Patient: Hello sir, my wife has been sufferring from gland tb for one year. She has been taking akt3 from then. But often she feels a feeling of vomiting and her mouth becomes very sour. Beside she takes rabifast dsr regular. Please advise me in this regards. Thank you. Doctor: Hello,Thank you for your contact to healthcare magic.I understand your health concern, if I am your doctor I suggest you that all your current drug regimens are correct. You can take a rabeprazole and domperidone drug half an hour before the meal to start its effect in good motion. Do not discontinue treatment because of this condition. Your body will be adapted soon with this drug.I will be happy to answer all your future concern. Thank you,Dr Arun TankInfectious disease specialist.Wish you a best health at health care magic."
},
{
"id": 189115,
"tgt": "Gums hurting, little red, white bubble, pus came out through the top of gums on pressing. What is it?",
"src": "Patient: my gums in front of my 2 front lower teeth were hurting for about 3 and were a little red and the pain went away so I checked it out and there was a white bubble so I popped it throughout the day and today I checked it out and the bump is pretty much gone but when I pressed on it pus came out through the top of my gums why is that? Doctor: Hello and welcome to hcm forum,Periodontal abscess has developed beneath the gums of lower front teeth.Deep scaling and root planning is adviced.Abscess has to be drained and curetted.Take complete course of antibiotics.Maintain oral hygiene well.Use soft interdental toothbrush and brush your teeth invoving all aspects of teeth.Also get your blood sugar level monitored.Scaling has to be done whenever plaque gets accumulated.Use chlorhexidine mouthrinse to gargle.Take care."
},
{
"id": 112599,
"tgt": "Have back pain. Feels like muscle pain. Cure?",
"src": "Patient: I have lower back pain, it has been aching a while and thought it was my matress but since Monday it has really started to aceh all the time. I do go to the gym so not sure if its muscle pain or something else. It feels like I constantly have to stretch it over to pull it but doesn;t quite feel like in my muscles and seems to now also be going round the sides a little bit Doctor: Hi and thanks for the query,It might be necessary to precise for how long you have been having this pain and if there is an identifiable trigger of the pain. A clinical evaluation might be necessary. An R ray of the lower spine and a spinal CT scan might be useful. These shall depend on the clinical appreciation of your doctor. It might be necessary to precise if the cramps you experience always correspond to the pain or not. Any changes in the sensitivity or at times weakness in the lower limbs.Doing serum electrolyte tests to exclude any electrolyte abnormality like Low Calcium and Magnesium levels should be done and identified. Levels of Vitamin B12 and Vitamin D could also be useful.Erythrocyte sedimentation rate and C reactive protein tests to measure the degree of inflammation could be useful to ascertain a detailed insight.I suggest you consult your physician for a proper clinical evaluation and paraclinical testing.Best regards,Luchuo, MD."
},
{
"id": 50151,
"tgt": "Rib pain, triggers with holding urine, sweet smelling urine. Kidney issue?",
"src": "Patient: For two almost two weeks now on each side of my back by the bottom of my ribs hurt. Or if I have to pee and I don t go they hurt. Also some time when I m doing going pee I get sharp pains back there. And my urine has a very smelly almost sweet smell. But I have no pain were I pee out of. Could it be a mother kidney in fiction. I had one four years ago that almost killed me I thought it was the flu and my whole body shut down my temp was 107.8 couldn t eat drink walk or even thing. But I never have pain when I pee Doctor: It looks like your kidney infection is happening again. The sweet urine can be ketones in urine. Consult your doctor before your symptoms gets worse. Kidney infections can occurs over weeks before you seek medical help."
},
{
"id": 1920,
"tgt": "Can taking bubble bath when trying to get pregnant cause birth defects?",
"src": "Patient: My wife and I will be trying to conceive next month. If we make love in a hot tub filled with bubble-bath, is there any danger that the sperm will be damaged in such a way as to cause birth defects? We are patient and it wouldn't be a huge blow if bubble bath prevented our lovemaking from resulting in a conception on that cycle, but we would be very sad if the use of bubble bath led to birth defects either due to contamination of the womb environment or due to damage to the sperm. Doctor: Hi I think there is no effect of such things on sperms. But one thing that can happen is that it will decrease the chance of pregnancy because of washing out of sperms. They will not be able to enter vagina due to the water flow."
},
{
"id": 26658,
"tgt": "What does the PPBS level in my cholesterol test indicate?",
"src": "Patient: I am 56 years old male and working as an executive in a company and having physique of medium built. My fasting blood sugar level is 148 mg/dL and PPBS level is 207ml/dL. Cholesterol is 245mg/dL. The above lab result is based on the test carried out on 15.07.2012.Please advise the treatment required for the above Doctor: Hello!Thank you for asking on HCM!I read your question carefully and understand your concern. Your blood glucose test (post postprandial and fasting) indicate a low tolerance towards glucose and probable diabetes. I recommend consulting with the GP for a careful physical examination, a resting ECG and some lab tests : - HbA1c (which indicates the mean blood glucose values during the last 3 months)- kidney and liver function tests- thyroid hormone levels- blood electrolytes,- urine analysis, etc. An endocrinologist consult is needed to start the proper therapy, besides diet modifications (diabetic diet) and physical exercise (including loss of weight). Regarding your cholesterol levels, I would recommend performing also HDL, LDL and triglycerides levels. You should know that total cholesterol is not a pure indicator of cardio vascular riks. The most important indicator of cardio vascular risk would be Total cholesterol/ HDL ratio (and decide if you need any therapy).The increased cholesterol levels may be related also to the diabetes, but some lifestyle modifications (diet, physical activity and weight loss) are helpful, besides possible need of statins. Hope to have been helpful!Best wishes, Dr. Iliri"
},
{
"id": 193763,
"tgt": "What causes the semen to turn transparent after masturbating over 4-6 times?",
"src": "Patient: I'm 20 years old and i masturbate 4-6 times a day. My sperm turns transparent if i masturbate over 4 times and again turns white if i don't do it for a day or two. Is that normal. Should i stop masturbation? And how many times can a person masturbate a day? Doctor: Hello. I have reviewed your query and here is my advice. Semen will become transparent after 4 to 6 times a day that is because the time is not enough for your testicles to produce enough sperm. So only semen comes out with few sperm cells.Hope I have answered your query. You can contact me for treatment options. Let me know if I can assist you further. Regards, Dr. K. V. Anand"
},
{
"id": 145145,
"tgt": "Does this MRI report of spine suggest the need for surgery?",
"src": "Patient: I am 56 yr old female and have been diagnosed with moderate discogenic and spondylotic changes in the lower 2 lumbar levels greatest at L4-5 where there is a grade 1 degenerative listhesis with rather severe central canal and lateral recess narrowing bilaterally. there is broad-based posterior disc protrusion at this level. There is moderate left and mild right foraminal narrowing. Also at L5-S1 there is facet arthropathy w/a broad based right paracentral and foraminal disc protrusion w/moderate right lateral recess and mild right foraminal narrowing. Left neural foramen and left lateral recess and mild narrowing. There is overall mild facet overgrowth at L3-4 with mild central canal and lateral recess narrowing. Question: Will I need surgery? Doctor: I read your question carefully and I understand your concern.The decision to undergo surgery though is not taken solely on your imaging results but also on what symptoms do they cause and their intensity. Also usually conservative treatment and physiotherapy is always tried for some months first.If you have intense pain which hasn't responded to physical therapy then I believe surgery for the changes in L4-L5 with severe canal and lateral recess narrowing. At time if the symptoms do not correspond to compression at this level nerve conduction studies are done beforehand to make sure which level needs to be addressed.I hope to have been of help."
},
{
"id": 166072,
"tgt": "What is the treatment for high fever in a child?",
"src": "Patient: My daughter is 2.10 yrs old. She had 102.7 fever last thursday when she had gone to day care. Given Meftal to her which brought down the fever immediately. Then she had fever on friday morning. She is taking Meftal, Tamim O and relent. Now she is having lower end fever. Is it Viral fever? how long it would be? Note: She is active and taking food/drinks regularly Doctor: Hi, if fever is persisting then investigations like complete blood count, malarial antigen test and widal test should be done. Review with report. A re-examination by doctor should be done. Take care."
},
{
"id": 62779,
"tgt": "Suggest treatment for itchy large lump on knee",
"src": "Patient: Hi, I have a large lump on my knee that has been there for half a week. it started off as an itchy scratch and grew into this this lump that has a hard surrounding and is very warm to the touch, also it emits pus and as of today blood. Could you help me in finding out what it is and a remedy for it? Doctor: Hi,Welcome to HCM.Based on the facts of your query,the itchy large lump on the knee after a scratch,seems to be Furuncle after the itchy scratch,which got burst and is now bleeding.The bleeding is not to be worried,as the pus is drained and collected blood in the lump is draining now.Its not cancerous lump.So don't worry it.Consult your Surgeon who would treat it with all the care it needs.This reply would resolve your current issues and would reduce your anxiety with it.Hope this reply would help you to plan treatment with your doctor.Will appreciate your Hitting thanks and writing excellent review comments to help needy patients like you. Welcome with any other further query in this regard.Good Day!!Dr.Savaskar,Senior Surgical SpecialistM.S.Genl-CVTS"
},
{
"id": 53697,
"tgt": "What causes a gallbladder polyp?",
"src": "Patient: I am 49 years old, I had a very bad pain on the right side of my body under my breast, with radiating pain to lower stomach, and chest area. I went to the doctor after a weekend of doubled over pain and very sore to touch. the dr. ordered a gallbladder sonogram. I have a gallbladder polyp 7mm. I have an appointment with a surgeon tomorrow. Should I be concerned, the polyp has increased in size. Doctor: Hi and welcome to HCM. Thank you for your query.There is no specific cause of gallbladder polyp and it may grow in any man. In most cases this is benign condtition and sometimes it may cause pain similar to gallbladder stones pain. Rarely ,poylps may have malignant potential and in such cases cholecystectomy is recommended as soon and they are discovered.Wish you good health. Regards"
},
{
"id": 58466,
"tgt": "Stomach ache, fatty liver, piles problem. Bilirubin .4, sgpt and sgot 156 and 112. On liv 52 and other medicines. Suggest diet plan",
"src": "Patient: i had 1.5 bilirubin 2 weeks back and 108 sgot and 125 sgpt, now bilirubin is .4 but sgpt and sgot are 156 and 112 respectively. i get stomach aches. i have fatty lever from one year. having Liv 52 and other medications.i have piles problem too. please help me the precautions i should take and diet i should follow. I am already avoiding oily food and i dnt drink. Doctor: Hello,thanks for your query,you are having consistantly high serum bilirubin level and high sgot and sgpt.,you also mentioned of stomach aches for all this you need to do some tests like sonogragphy of abdomen and consult a general surgeon asap.for time being billary secretogauge like ursodiol can help and for piles laxative like lactifibre along with lots of oral fluid and avoiding spicy/oily food will help. hope this will answer your query. dr.akhilesh chamediya"
},
{
"id": 141572,
"tgt": "How to treat for the pain in chest and spine caused by a hit?",
"src": "Patient: I was playing football Saturday and was hit in the ribs and upper body Sunday I played football without equipment on and a player hit me dead square and center in my chest now my chest and spine have been having pain and I can't sleep at night because it hurts so much Doctor: Hello and Welcome to \u2018Ask A Doctor\u2019 service. I have reviewed your query and here is my advice. There is a possibility of rib fracture. You need to get examined by a doctor. X-ray of chest and thoracic spine is required. Hope I have answered your query. Let me know if I can assist you further."
},
{
"id": 199452,
"tgt": "Can a nerve damage to the bladder cause pain in penis?",
"src": "Patient: Hi, I have a painful pins a needles sensation down the length of my penis. I'm 26 years old and have two disc injuries in my spine after a car accident four years ago, which has also caused nerve damage to my bladder. I also have high blood pressure. What could this sensation be? Doctor: Hello dear,Thank you for your contact to health care magic.I read and understand your concern. I am Dr Arun Tank answering your concern.No nerve damage to bladder won't cause pain after a so much time. If you receive a pain immediately after injury than we can count it because of injury.This type of sensations are common in nerve regeneration, so if your damage nerve are getting repaired than you can have such a feeling.In my advice you should take tab pregabalin, tab vitamin B complex. This will help in recovery from such a problem.Please do perineal muscle exercise in other word stop and release urinary sphincters as much time in the day as possible. This will help recover fast from the problem you are having.Some physiotherapy and electrotherapy can also cure such a problem. You can visit nearby physiotherapist they can guide you regarding exercise.I will be happy to answer your further concern on bit.ly/DrArun.Thank you,Dr Arun TankInfectious diseases specialist,HCM."
},
{
"id": 8643,
"tgt": "Use depiwhite cream for face brown spots, now no improvement. Can work on computer applying cream in night?",
"src": "Patient: Hi sir. I am using depiwhite cream to remove brown spots from my face and i am using this cream for many months. I saw a little improvement at the start when I started to use it. But not seeing any improvement now. I do night shift and work in front of computer. I want to know can I apply this cream on affected areas of my face for whole night and can work with computer. Is computer produces UV rays which can harm my skin or should I discontinue to use it as not seeing improvement. Please suggest me. Regards, Dinesh Doctor: Dear Dinesh Yes you can apply the cream for the whole night. Today's computer with lcd screen do not emit uv rays so u need not worry. If your skin is photo sensitive then we may consider offering some photoprotective measures. If the current treatment is not helping much then a change of treatment regimen may help. Chemicals peeling may be added to the current regimen. The pigmentation you mentioned, has it been diagnosed as freckles /lentigenes or melasma? The prognosis of both are different. Regards Dr Amit"
},
{
"id": 121746,
"tgt": "What is the further treatment for my knee injury?",
"src": "Patient: I am male,56 years,5 8 ,80 Kgs.Due to a sudden twist while getting down from the jeep, my left knee got injured with complete tear of acl and fragment tear of meniscus on 8th Nov2011.Some Drs advise Arthroscope surgery and some Drs advise tablets and Exercises.I am taking FreeFlex Forte Tabs & Thrize Tabs two times daily and continuing the Knee exercises as per the advise of a Dr.I feel now that my pain is reduced considerably and i am able to walk with a little normal speed.Please advise me about my further treatment. Mohanaraj T Doctor: Hello, Your symptoms seem to be related to a tear of meniscus. I see that you have already taken proper treatment. I suggest continuing the medications as prescribed by your doctor. I also recommend to do regular check-ups. If the pain continues arthroscope surgery may be needed. Hope I have answered your query. Let me know if I can assist you further. Take care Regards, Dr Dorina Gurabardhi, General & Family Physician"
},
{
"id": 91874,
"tgt": "Cause for severe abdomen pain and chest pain?",
"src": "Patient: I am 50 yrs old about a week ago I had severe pains in my abdomen and chest and struggled to breath ( I suffer from asthma) Ive since been sick a couple of times and the pain has never gone but reduced. The pain is now building back up again in my chest, but I have noticed a hard lump like area with a pea sized lump in the same place. This lump/harder area is just to the left of my sternum and the pain is getting very slowly worse. I recently had an x-ray and ecg for a slightly less pain about 6 weeks ago but everything was ok. Bit scared but don t want to waste time for Drs any ideas. Doctor: HI. Noted the history carefully. This lump can be coming from the rib and needs investigations like FNAC ( fine needle aspiration cytology) and may be a CT scan to rule out any internal problem showing on the outside. X-ray may just give an ides and do not confirm or rule out many diseases."
},
{
"id": 222454,
"tgt": "Is a a D-dimer test related to pregnancy?",
"src": "Patient: i am 17/40 wks pregnant and am having breathing difficulty so a D-dimer test and an echocardiograph were performed. The D dimer came back positive 536 ng/ml and my echograph shows more blood pumping through one of the heart chambers, I have been told thiese could just be related to the pregnancy. what do u think Doctor: D-dimer test has limited role in pregnancy as it normally increases in all pregnant women. SO, if one suspects that you are suffering from thromboembolism, you need other investigations and before results of investigations come, you need to start heparin in urgent basis."
},
{
"id": 142977,
"tgt": "Suggest remedy for multiple sclerosis?",
"src": "Patient: Hello my son was diagnose with multiple sclerosis last year in may. He is having a hard time dealing with it and refuse to take the injections that the doctor recommend and I do not know what to do. He has these mood swings that he could be very mean and very angry. What can I do to help him? Doctor: Hi, Welcome to HealthCareMagic.com I am Dr.J.Mariano Anto Bruno Mascarenhas. I have gone through your query with diligence and would like you to know that I am here to help you.We need to evaluate whether mood swings are 1. Due to MS2. Due to Some Other problemFor this Clinical Examination and Mental State Examination are needed. This cannot be managed over internet. You need to consult your neurologist in personHope you found the answer helpful.If you need any clarification / have doubts / have additional questions / have follow up questions, then please do not hesitate in asking again. I will be happy to answer your questions. In the future, for continuity of care, I encourage you to contact me directly in HealthCareMagic at http://bit.ly/askdrbruno Best Wishes for Speedy Recovery Let me know if I can assist you further.Take care."
},
{
"id": 205898,
"tgt": "What causes the feeling to hug and needy as a baby at the age of 23?",
"src": "Patient: Hi, may I answer your health queries right now ? Dear Doctors, I m a 23 year old female,and I ve a serious problem since I was very young though I m trying to cope with a and forget about the whole matter but it still comes back and increase at certain times,My problem is some females that I met for e.g if at school a teacher,if at club then a certain coach if at family visits,one attracts me not from first time but after many visits and long stays I feel with her that I wanna be a baby,I pretend to fall asleep to feel my mouth over her chest as I wish to breastfeed I imagine this,and this idea doesn t come to me to anyone but certain people,I try to hug her to feel that she is giving me that what a child needs,in our society many made comments what is this?why do you love this.... like this?but I swear I only feel relief when I do this,I feel relaxed but when I see people s look or afraid of anyone s words so I stop but this desire increases,all what I want is to feel like a baby breastfeeding from her mother and sleeping and she relaxes me. my mum when I was very young she used to hit me and always shout me,studying always and my school was stressful,my brother when young I saw how mum deals with him and looks how he attaches to her chest but then she shouts to go and usually stay at my room trying to immitate with a pillow Also my parents had fights and infront of people they act perfect,my mum was betraying my dad and I heard this and carried it inside my heart,this was from long time,I don t want a love from mum,though I don t love her but never hate her,This makes me so bit confused,please I need your help. Thanks type your query here.. Doctor: DearWe understand your concernsI went through your details. I suggest you not to worry much. I don't see any mental disorder with the given description. It could just be a sort of obsessive thinking. Not OCD. You are creating opportunities to think obsessively. Best method to overcome the situation is ignorance. Ignore those thoughts and be busy with your life. You should practice psychotherapy techniques to streamline your life style and meditation and yoga techniques to calm your mind, body, streamline your metabolism and thinking style. I shall provide you with the required counseling and psychotherapy techniques to overcome these problems. Please describe the whole problem in detail and post a direct question to me.Hope this answers your query. Available for further clarifications.Good luck."
},
{
"id": 24641,
"tgt": "Why my husband's muscle enzymes are high?",
"src": "Patient: My husband, a fireman, suffered a dissection of the aorta after inhaling high concentrations of exhaust fumes from a vent saw while at a fire. A blood clot developed over the tear, causing a 93% blockage. He was treated with a stent, spent 5 days in critical care, before spending a breif period of time on the cardiology ward. We were told there were no signs of plaque, medical irregularities (aside from the dissection), he has no medical history. Hes very fit (215, 6ft2), young (29), we've always eatten impeccably (only whole grains breads and pastas, lots of vegetables, lean meats, fish ect). Its been almost 3 months since the heart attack, and his muscle enzymes are still high. Why? Doctor: I need to know of he has some kidney problem... and which enzyme and how much is value.. borderline raised value ...not much to worry...."
},
{
"id": 46615,
"tgt": "Noticing abdominal pain and sore back",
"src": "Patient: hello, my sister recently had some abdominal pain and sore back, so thinking it may be a kidney stone or infection, (she had that happen after her first child) she had an ulltasound performed. they found her liver to be enlarged. last night she went to emergency with a racing heart and poss an anxiety attack. I believe the doctors said she had a heart arrhythmia. I do think she is under a lot of stress. My nephew is 3 and my niece is 6 months so she has her hands full and is thinking of selling her house. Is it possible that this is just an anxiety attack, or are these two symptoms possibly something more dangerous. Doctor: abdominal pain with back sore may be a sign of pancreatitis. so please look at the pancreas. It may be necessary to do a CT abdomen to diagnose that. regarding racing heart, anxiety is a possibility but we should rule out arrhythmia by doing further tests like holter or Electrophysiological study."
},
{
"id": 201403,
"tgt": "What causes small white particles at the edge of the penis glans?",
"src": "Patient: Hi, about a month ago I observed some very small white grains (like needle heads) just at the edge of my penis glans under the prepuce. They are just a few in number and present like a semi ring. Sometimes these grains become a little itchy as if some needles are being pushed there. Also, the smega deposited underside the glans has increased in quantity and is very foul smelling. Earlier it used to be almost negligible and without any foul smell. None of these was present before a month. Can you please help me out. Doctor: Hi,It seems that this is due to collection of smegma on glans.Due to non cleaning of glans there is deposition of smegma giving rise to thick collection and foul odor.Remove collection of smegma by averting fore skin of glance gently and clean glans penis.If there is soreness on the part, apply antibiotic cream.Make a habit of cleaning glans by averting fore skin daily while taking shower.Ok and take care."
},
{
"id": 78058,
"tgt": "Suggest medication for pain noticed under the sternum while breathing",
"src": "Patient: So yesterday every time I would breathe in or move it would hurt right under my sternum,and today it was not so bad, but today I was just sitting and if felt like someone hit me under the collarbone and it made my whole body jruked and it only hurt for a second. So what is wrong and do I need to go see a doctor? Doctor: Hi,Dear,Thanks for your query to HCM.Dear I read your query and reviewed it with context to your query facts.I understood your health concerns and feel Concerned about them.Based on the facts of your query, you seem to suffer from-Acute MI(Myocardial infarct)?Hepatitis ?Pancreatitis?Acute Duodenitis.These possibilities need to be kept in mind besides other which cause similar complaints under the sternum.Pre-cordial Catch Syndrome (PCS or Texidor's Twinge) with sharp pain due to Costo-chondritis with ? Parietal Pleuritis of the lower chest ribs,also needs to be ruled out while assessing this complaint complex.Remedy--Investigate by X-ray Chest with standing diaphragm/ CT Abdomen and as needed by treating specialist based on the physical findings.-Would depend on the cause assessed and investigated from Physician/Surgeon/Chest Physician.-Tab Advil 400 mgm would reduce the pleuritic pain over time.-Hepatitis pain would reduce with Conservative and supportive treatment with Iv Fluids +Vit Bplex +Vit C--PPI -Tab Veloz D -20 /30 mg x 2 times a day-Inj Fortwin 30 mg/ Iv if need be.This would be supplemental with supportive treatment as per the causes in your case.Hope this would lead to the exact diagnosis and proper treatment with your doctor there.Hope this reply would help you to plan further treatment with your doctors there.Welcome for any further query in this regard.Good Day!!Dr.Savaskar M.N.Senior Surgical SpecialistM.S.Genl-CVTS"
},
{
"id": 204448,
"tgt": "How can ADHD be diagnosed and treated?",
"src": "Patient: I know you can not give a true diagnosis. However, I think I may have ADHD and or ADD, what is the process in which I would take to get an actual diagnosis. If it is proved that I have either or, then what happens next because I have also in the past been diagnosis by a licensed therapist with Generalized panic disorder and social anxiety disorder along with depression. I do not wish to be on synthetic man made drugs, I do wish to do something more natural based. Here are some of my symptoms. Before what I consider a big event, like a job interview, or going to the DMV, starting a new school, or something like that, the night before I will not sleep. Along with that when I am feeling excited, stressed, sad, in a state of urgency, I become very scatter brained. In my daily life I do struggle to manage my personal finances and loath completing household chores. Along with these bullet points \u2022 Lack of focus and trouble listening and paying attention \u2022 Being easily distracted, disorganized, and frequently forgetting and losing things \u2022 Failing to follow through \u2022 Making mistakes that seem careless. I do make lists, I make every good intention to write them down then forget my lists... Help Please. Another thing, I try to make plans with people, but I end up canceling on them at the last minute. Perhaps this goes back to my social anxiety thing.... I want to go out and socialize I love people but I am scared to socialize, anything outside of work and grocery shopping, i m pretty much a hermit. Doctor: Hello and Welcome to \u2018Ask A Doctor\u2019 service. I have reviewed your query and here is my advice. To diagnose ADD or ADHD you need to visit a psychologist or psychiatrist. There are standard psychometric tests to diagnose these psychological disorders. Hope I have answered your query. Let me know if I can assist you further. Regards, Dr. K. V. Anand"
},
{
"id": 44626,
"tgt": "What is the duration for treatment of infertility ?",
"src": "Patient: hi this is juher looking for answer the treatment duration for me Doctor: Hello juher; welcome to HealthcareMagic Please see that your question is not proper or complete and does not describe why and for what you want treatment for.So please write in detail what your problem is.If you have a report of semen analysis then you write the full report so that we can advise you about it. Thanks"
},
{
"id": 142922,
"tgt": "Can acidity effect brain?",
"src": "Patient: Can acidity effect brain??? as yesterday I faced a huge problem with my head....it was spinning and to heavy weird sensation...I could not move....one of my brother told me it is because of acidity and took some antacid and sleeping pill(he also advised) then after 3 hours sleep I felt better... Doctor: Hello!Welcome on HCM!Your symptoms do not seem to be related to stomach acidity. An inner ear disorder could be the cause of your troubles. For this reason, if the problem persists, I would recommend consulting with an ENT specialist for a careful physical check up and labyrinthine tests. I would also advise you to monitor your blood pressure, because blood pressure fluctuations can lead to these troubles. Hope to have been helpful to you!Take care, Dr. Aida"
},
{
"id": 190991,
"tgt": "How can I permanently get rid of temporary cavity ?",
"src": "Patient: i had cavity on 8 teeth 4 years back. Then i consulted a dentist and filled my teeth with cement and white color filling material ( i dont know that may be amalgum ). At that time, i had deep and spread black color cavities on the upper side of my teeth. Temporarily they filled with material. But i dont know about this. What is cavity ?? all my teeth get afftected by this. how to permanently remove from my teeth ?? is it correct what i have filled with my teeth ?? is it a temporary solution ?? I wish to bring back my teeth without cavity. what to do ?? Doctor: Hi, Once the cavity is formed, it is not possible to bring back the teeth without cavity as you expect.all you can do is prevent formation of caries in remaining teeth by- -Maintaining good oral hygine-proper brushing (early morning&before going to bed ),(after sticky food like chocolates, sweets ,) - Diet with calcium & vitamines.(MIlk,salads,fruits etc). Treatment of should start earliest. -Temporary filling,cement filling,silver filling are the steps of filling.(the silver/amalgum filling is permanent. -Deep caries can be treated by Root canal treatment. After the treatment you have to be careful by proper brushing. Also visit your dentist every 6 month &have regular check up. . - Mind it -Badly Infected caries teeth may need extraction. Thankyou"
},
{
"id": 118150,
"tgt": "What are the symptoms of anaemia?",
"src": "Patient: I had a big loss of blood last week due to a post tonsillectomy bleed. I'm now on ferrous sulphate. I feel a lot better but, a week and a half on, I'm still feeling heart beats strongly in my chest some times and am feeling v anxious a lot of the time ESP when my heart is beating lie this . Is this just the anaemia ? Would re starting my citalopram help with this too? I was on it before my tonsilectomy on the 16th jan . Thans Doctor: Symptoms of anaemia areEasy fatigue and loss of energyUnusually rapid heart beat, particularly with exerciseShortness of breath and headache, particularly with exerciseDifficulty concentratingDizzinessPale skinLeg crampsInsomniaRegardsDR De"
},
{
"id": 3724,
"tgt": "Could i be pregnant as i had bleeding only while wiping?",
"src": "Patient: Ok I had my period last week it s been over for about 4 days no bleeding at all but I went to the bathroom and pee is normal but when I wiped its like a pinkish color...does that mean I could be pregnant? If so would I be able to take one and it tell me Doctor: HI, I understand your concern. Yes ,( provided there is normal bleeding routinely),,scanty bleeding during menses can be implantation bleeding & there are chances of conception. You can confirm it by home pregnancy test after 4-5 days. Thanks."
},
{
"id": 97912,
"tgt": "Diagnosed herniation with severe nerve root compression, poor ankle reflexes, pain. Can it heal by itself without surgery?",
"src": "Patient: I was diagnosed 2 months ago with large l5s1 herniation with severe nerve root compression and significant DDD at that disc. Treated for 2 wks with acupuncture but a twist renewed the agony. Symptons were poor ankle reflexes unable to toe rise and painful sciatica .. Surgery was arranged for 4 days later however the feeling started to return and could do tow rise on morn of surgery so surgeon postponed ... This was 5 wks ago. I still have skin numbness at back of leg and last two toes and am very limited in what I can do. I take 100 mg anti inflamm a day stopped other meds 3 days ago. Can a large herniation heal by itself. I am usually a fit healthy perso. (yoga 2 a week cardio and weights 3 timesaver week) there has been improvements in mobility etc. anyway 36 female 126 lb 5'3 .. I should add that I didn't get the MRI for 2 wks and only had acupuncture and paracetamol for pain relief during that time the onslaught of sciatica was horrendous and fast but improved !! Doctor: 1. L5,S1 herniation is either due to Trauma or bad posture (postural dysfunction), please specify your's? 2. because DDD (degenerative disc disease) is there, probably weak inter-vertebral disc due to poor posture can be the the cause (only proved otherwise), and L5,S1 is the commonest site for herniation. 3. Compression of sciatic nerve has lead to Sciatica, L5 nerve impingement has lead to numbness and causes weakness in extending the big toe (foot drop),whereas S1 nerve impingement results in loss of ankle reflex (patient cannot do toe rises). 4. Any bladder/bowel dysfunction? 5. have you consulted for Osteopathic/chiropractic manipulation? 6. Symptomatic disc herniation rarely heal on their own, but before opting for surgery, please consult a physiotherapist (PT)for the same, but before that make sure you get an MRI done of the spine to rule out any STENOSIS, which could result in edema and MASSAGE given by PT will further aggravate the symptoms. 7. If you are insured,check with the Insurance company about the same. 8. If pain and symptoms continue, consider microdisectomy or lumbar herniated disc surgery."
},
{
"id": 54879,
"tgt": "What causes elevated SGOT and SGPT levels?",
"src": "Patient: HiMy Age 43 y maleNo medical historySome problem in Blood Pressure since last 1 year but controlled through diet, walk and of course medicine...Tellusartan 40tellisartan'went through Health check up yesterdayeverything within normal limitsbut SGOT was 108 and SGPT was 128ESR was 20Kindly let me know its implications Doctor: Hi thanks for asking question.Noted you have high blood pressure with elevated liver enzyme and taking telmisartan for blood pressure.Your liver enzyme could be elevated by fatty liver.USG done for it...with serum cholesterol estimation...Drug induce liver enzyme can be elevated...If need complete liver profile test can be done with viral marker....Meanwhile rest liver with low fatty diet.Through diet control and regular exercise control blood pressure and lipid level.If still enzyme elevated muscular disorder and cardiac history needed.Take care.Dr.Parth"
},
{
"id": 113982,
"tgt": "Why i am having back pain after the treatment of TB ?",
"src": "Patient: Hi! I was a former TB patient 4 years ago and was treated for 6 months. At that time, I only had night sweats , back pain and decreased appetite. I went to the doctor with a chest x-ray which revealed an infiltrate. The doctor diagnosed me to have TB so I was treated. I have been experiencing back pain that comes and goes for 2 years. It usually occurs when I am stressed and during cold weather. I had a chest x-ray last year that revealed an old scar of TB and the doctor said that I am completely healed from TB. Still, my back pain occurs. What do you think seems to be a problem? Could this be TB again but the doctor failed to recognize it? Doctor: Hi welcome to healthcareMagic Since you are an old TB patient, do not neglect the pain, you need to Do CT or MRI to rule out the possibilities of Potts spine ( TB of spinal cord), which is more chances in patients with previously infected with TB.. Hence with Xray , it may not be visible to see it, in early stage of disease, you need to do CT or MRI... consult an orthopedician they will guide you....Hope i have answered your question Takecare"
},
{
"id": 32207,
"tgt": "What could bumps on tongue during a viral infection suggest?",
"src": "Patient: Last week I had an Upper Respiratory Infection. The doctor told me that since it was viral he couldn't give me any prescription meds so I started taking Sudafed. I noticed a few days ago that when I looked in the mirror at my throat I noticed some bumps on my tongue toward my throat. I checked again this morning and they were still there. These bumps don't hurt and they are the same color as my tongue. What are these bumps? Should I be concerned? Doctor: Hi Dear,Welcome to HCM.Understanding your concern. As per your query you have bumps on tongue during a viral infection and upper respiratory tract infection which seems to be due to transient Lingual Papillitis . These occur when papillae become irritated and slightly swollen. They occur due to various reasons like stress , hormonal changes or acidic and spicy food . It generally doesn\u2019t require treatment and clears up within two weeks. Do warm saline rinses 2-3 times a day . If condition does not improve you should visit oral physician once and get it examined. Doctor may prescribe antibiotics like amoxicillin for at least for 10 days along with vitamin supplements to boost the immunity. Maintain oral hygiene. Apply orabase gel on these bumps.Hope your concern has been resolved.Get Well Soon.Best Wishes,Dr. Harry Maheshwari"
},
{
"id": 24274,
"tgt": "Could stopping Spironolactone medicines temporarily lead to any complications?",
"src": "Patient: my mom takes spironolactone med due to her current heart problem. This med was priscriber by MD in Ethiopia and she is hear in USA. The Dr. here is doing a lot of test but in the mean time she is running out of this med. So if she stop this med what is the risk? Doctor: Brief Answer:There are no risks from the interruption of Spironolactone, while her blood pressure values are normal.Detailed Answer:Hello,Thank you for using HCM.There are no risks from the interruption of Spironolactone, while her blood pressure is not higher than the normal. Spironolactone can be stooped immediately without any problem. Mean time your mother should monitor strictly her blood pressure values at least three times a day.If the values are higher than 140/90 mmHg, than she should refer this to the treating doctor, in way to prescribe any other anti hypertensive drug which does not interfere with the exams she is going through.Hope this is helpful.Let me know if you need more clarifications. Otherwise please close and rate the answer.Kind Regards, Dr. Juarda"
},
{
"id": 3772,
"tgt": "Is medication required while trying to conceive?",
"src": "Patient: Hi I am 35 years old my weight is 65.5 kgs and height is 5'3.We Got married in October 2009. we are planningFor family now but we want to know if we really need to see the Gynacologist before we plan.is there any medicine required to Ready for pregnancy? My periods are perfect. Doctor: Hi,Were you using contraception all these years to avoid pregnancy?Since your periods are perfect, in all probability you are ovulating regularly.You are slightly overweight. A normal BMI increases the chances of conception. It will be excellent if you can reduce around 5 Kg weight. IF you and your husband smoke, you must quit smoking. It is important to have intercourse at least three times in a week to improve your chances of conception.Have one tablet of folic acid daily.If you don't conceive within 6 months of trying for pregnancy, you must consult Gynecologist for baseline subfertility tests ( hormone assay and fallopian tube patency test for you and semen analysis for your husband).Based on the results of investigations treatment can be planned.I hope I have answered to your satisfaction."
},
{
"id": 87511,
"tgt": "Can history of diverticulitis be the reason behind abdominal pain?",
"src": "Patient: HI, I ve been having issues with abdominal pain for the past 3 1/2 days. I have a history of diverticulitis and when this kicked in on Friday afternoon and got a lot worse by Friday evening, I went to the ER to be proactive. They did bw and a CT scan. All was normal but the ER dr did say he felt it was caused by constipation due the amount of stool in my upper section of my large colon and they sent home and told me to take some magnesium citrate. It was 3Am by the time I got home so I took the recommended pain meds (800mg of ibu and 1000mg of tylenolPM) and went to sleep. When I woke up the pain was gone. We had a party so we went to that. After we got home at 4:30 I took the mag cit. It didn t kick in and so at 8PM I took a dose of miralax and within 30 mins, I had very loose stool. I thought that was the end of it. Then this morning I felt mild pain and it built until at by 1Pm it was pretty painful again. I ve had nothing but loose stool for the past 3 days so I don t think it is constipation. I have no fever, no nausea and no vomiting but the pain can get pretty severe, up to a 6 or 7, not debilitating but enough that I cannot concentrate and just want it to go away. Oh, I also just had a colonscopy due to the diverticulitis (had a flare-up in early June) and all came back normal, with the exception of the mild diverticulosis. Doctor: Kindly mention your age and sex also.As per the history one important thing has been missed. Stool examination especially for occult blood. How many diverticulae are there? there might be many and sometime numerous.However, even if there is only single of them it is always prone to recurrent inflammation causing pain, bloody diarrhoea etc...Kindly have a stool examination and physical check-up to rule out other possibilities like amoebiasis, drug reaction or appendicitis.regards,"
},
{
"id": 178334,
"tgt": "Is taking BCG injection twice harmful?",
"src": "Patient: Hi doctor,My baby boy is 1 month old. He as born on 27 th of January, monday. We were told that the bcg injection as given on the same day he was born. Later the doctor asked us to come back on next Monday to take bcg. They assured that it was not taken on that day. So we made my kid take the vaccine the next Monday. But now we can see 2 scars one has popped out and the other one is about to. Please let me know if there's any side effects of taking the vaccine twice.Shyama. Doctor: Hi, I had gone through your question and understand your concerns.No , there are no side effects , it is not a reason for concern, but there will be two scars which is normal BCG reaction . As two are closely spaced scar formation is also not a problem, in case of interval of more than 4 weeks due to immunity development scar may be a bit larger but it is not going to harm baby.Hope this answers your question. If you have additional questions then please do not hesitate in writing to us.Take care.\u2022\u00a0\u00a0\u00a0\u00a0\u00a0DISCLAIMER: - All the information provided here is for information purpose only , it is not a substitute for the advice of a physician after physical examination , it is not intended to replace your relationship with your doctor. This information in no way establishes a doctor-patient relationship. Before acting anything based on this information do consult your doctor. I recommend that online users seek the advice of a physician who can perform an in-person physical examination"
},
{
"id": 39553,
"tgt": "What causes yellow skin and yellow colored cornea?",
"src": "Patient: my 18 year old daughter is doing a juice fast (she is drinking the Bolthouse Farms 100% fruit juice smoothie), which started Monday morning and today she started getting yellow skin and the whites of her eyes are turning yellow? What should we do to make the yellow go away? Doctor: Dear FriendWelcome to Health care magic. I am Dr Anshul Varshney , I will shortly try to help you with my opinion.Yellow discoloration of Eyes and skin can occur due to Jaundice.There is no point in waiting.Get her investigated.I would advise you to get her following investigations:1. Liver Function Test2. Viral Markers.Ask her to stop the juice fast and to take high carbohydrate diet, as in jaundice there are chances that she might get into low blood sugar status.Based on the available details, this is my best advice for you. You should discuss with your doctor and share my opinion, I am sure he/she will agree to my opinion. If you wish to discuss the problem further , you may please ask.Stay Healthy."
},
{
"id": 15430,
"tgt": "Itchy reddish rashes on body. Is it food allergy?",
"src": "Patient: Hi Doc, Im 35 year old, I have an itchy skin with a bit of redness for the past two days. Whenever I give in to the itchiness it turns into a bit outer beveled or rather slightly raised patches more like itchy maps/reddish rashes around the neck , chest, abdominal area, shoulders, forearms, back and thighs. Occured mostly during the second part of the day. Tried change of clothes etc...Dont really think I have any food allergy ...Any advice please? Doctor: Hello.itchy red rashes all over the body can be due to acute urticaria.it is due to food allergy.insect stings and dyes used as preservatives.it can be cured by antihistamine.antibiotic and use of calamine lotion over the lesions"
},
{
"id": 24252,
"tgt": "Is the cardiac catheterization procedure recovery period long?",
"src": "Patient: 59 yo female, 5'5\", 206 lbs, diabetes type 2 for past 4 yrs. Having cardiac cateritization this morning outpatient cardiac center due to mildly abnormal nuclear stress test, may have partial blockage. Will I be able to go work on Friday; I sit all day at a desk. I forgot to ask my dr. Doctor: Hello , Angiography is done through hand you would be discharged on the same day . You can go to routine work from the same very day . If it's done through leg I would advice 1 day rest for the wound to heal . Regards Dr. Priyank Mody"
},
{
"id": 78820,
"tgt": "Suggest treatment for pain in chest and neck",
"src": "Patient: I occasionally have pain in the middle of my chest that radiates up the right side of my neck. It's a kind of pressure pain. It usually goes away in a couple of minutes. I also have fibromyalgia. I'm a 50 yr old female and I'm also a smoker. No high blood pressure or cholesterol problems. Doctor: Thanks for your question on Health Care Magic. I can understand your concern. Since you are smoker, possibility of GERD (gastroesophageal reflux disease) is more. GERD is due to laxity of gastroesophageal sphincter. Because of this the acid of the stomach tends to come up in the esophagus and cause the symptoms of Central chest pain, chest pressure, tightness etc. You are also having fibromyalgia. And for this you must be taking painkillers. These painkillers can also cause GERD. So avoid painkiller drugs. Quit smoking as soon as possible. Start proton pump inhibitors. Avoid stress and tension. Avoid hot and spicy food. Avoid large meals, instead take frequent small meals. Don't worry, you will be alright. Hope I have solved your query. Wish you good health. Thanks."
},
{
"id": 50999,
"tgt": "Diagnosed with kidney stones. Severe pain. Is there any medication stronger than narcos?",
"src": "Patient: Ive been diagnosed with kidney stones and they r very very painful my doc had gave me 40 narcos about 3 weeks ago . They sumwhat helped with the pain but now theyre gone and he wont perscribed anything..i called this morning to make a same day appt bkuz ive been missing skool/nterviews due to this pain. And my appt to c an urologist isnt till march....do u think my doc will perscribe more pain meds today? & is there something more effective then narcos? Doctor: Hi thanks for your question . Norco is combination of acetaminophen and hydrocodone. Acetaminophen is nosteroidal anti inflammatory drug wher as hydrocodone is opoid analgesic.Combination of two is avery strong pain killer. Stronger pain killer than hydrocodone is morphine and pethidne both are opoids.If you have pain which is not relived by the drug you are already taking,you can request your physicion to increase the dose or prescribe you stronger pain killer like pethidine and morphine. Hope this answers your question."
},
{
"id": 152976,
"tgt": "Should Lupron depot be stopped altogether?",
"src": "Patient: i have pain in my left lower abdomen. on a scale of 1-10 it is a eight.and at times a 9/10. I feel and look bloutted amost all the time. I tire very easily..I had prostate surgery in 2010 for an aggressive type of cancer i had 28 tx of radiation. and did fairly well...I only got tired toward the end. Now I really don t want to eat the smell of food makes me sick. it causes back pain in my left side. I also take Lupron I have seen my doctor I have had blood in my stool and urine since I finished radiation therapy. I know that bone cancer is a possibility I did delay a shot by 2 months i thought about stopping jt all together. What do you think?\\ Doctor: Dear GreetingsIt seems that there is a progression of the disease bony pain and bleediing manifestation are could be part of it.One should get the bone scan done to find out the cause of bone pain.Mri pelvis also recommendable to rule out the local recurrence.Se.PSALupron depot should not be stopped immediately as other therapies can be add on to the Lupron.Mri pelvis will also help t"
},
{
"id": 70900,
"tgt": "What could be the reason behind sharp chest pain after masturbation?",
"src": "Patient: Hi Doc, I have not done sex in a while now, I don't have a partner. I started masturbating three weeks ago and I have been enjoying myself. Now the problem started about a week ago, every time when I reach orgasm I experience a sharp chest pain. what could be the possible reason behind this? Doctor: Hello and Welcome to \u2018Ask A Doctor\u2019 service. I have reviewed your query and here is my advice. * As per my clinical experience, the chest pain after masturbation is in concern with underlying stress, anxiety or fear of something not obvious to happen after masturbation. * In a nut shell, it is psychological component, not organic. Hope I have answered your query. Let me know if I can assist you further."
},
{
"id": 102549,
"tgt": "What do you suggest for drug allergy with fever and cold when on microdox?",
"src": "Patient: Hello Dr. I am suffering from fever and Cold. 6 months i had fever and went to Dr.He gave me microdox tablet. After i took them, I got black spots in my skin . I met Skin specialist.He told that this was drug allergy, due to intake of tablets. Day before yesterday i took half paracetomol , yesterday i found one more black area in my shoulder, What to do Sir? Doctor: Hello,Welcome to HCM,Before using any medicine inform your doctor if you are allergic to any medications.Some side effects of doxycycline may occur that usually do not need medical attention. These side effects may go away during treatment as your body adjusts to the medicine. Appearance of rashes to this drug is very rare.If the rashes are increasing stop the medicine and inform your treating doctor regarding this. He will help you and tell whether it is allergic to drug or something else.Thank you."
},
{
"id": 21111,
"tgt": "Suggest remedy for shortness of breath and rotating pain in front of chest",
"src": "Patient: went to er on 2-17 with light chest pains and shortness of breath they did bloodwork and one was elevated so admitted and did a heart cath the next am which was neg no stents placed at all I am 39 been diabetic for 33 years blood pressure never high dr said it was a stress heart attack however still having light shortness of breath and a rotating pain in front of chest and rotates around to the back on left side dr wants to put a monitor on next wed is that too late? Doctor: hello. thanks for posting here.Since you have a normal coronary angiogram you need not worry about a heart attack. As your doctor told you, you may be experiencing a stress attack or a panic attack. Also please get a chest x ray done to see if there are any lung related cause of chest pain and shortness of breath like infection( pneumonia) or asthma.your doctor wants to connect you with a rhythm monitor which can also help here. as far as your concern about whether it's late , you can wait for the appointment, there is no immediate hurry. thank you"
},
{
"id": 137381,
"tgt": "What causes cramp in the toes?",
"src": "Patient: I hurt my foot while walking the west highland way. 11th may Thought it might be ligament. Pain on side of foot. Now constant pain over top of foot. Toes cramp. Constant swollen. Been x-rayed not broken. Getting worse instead of better. Getting fed up. Doctor: Could be sprained. Dip in hot water often and rub muscle relaxants.Still to add simle things for rapid recovery. Put little curcumin ( turmeric) in warm milk or warm water. Drink twice. Also apy some turmeric in clarified butter while warm. Cover with bandage/ cloth. Do for few days. But turmeric stains cloths. Wear accordingly.It will help you. My wishes."
},
{
"id": 201366,
"tgt": "Why it is unable to pull the foreskin even on erection?",
"src": "Patient: I need some assistance on tight foreskin. Currently i was not facing any issues with it but i am unable to pull my foreskin completely and even on erection i can see pre-cum on it when my foreskin was unable to pull over erection ..... Could you please assist me where can i get better information and preferably in bangalore location --India. Doctor: DearWe understand your concernsI went through your details. I suggest you not to worry much. Foreskin retraction is not that necessary for now. But for cleanliness and to have painless sexual act, retraction is considered good. A surgical procedure called circumcision can be performed to have full retraction. But before going into that, practice retracting the foreskin several times a day by using some oils like coconut oil. Naturally, on practicing, it should be done. If it is not done even after a months practice, consult a surgeon for circumcision. Psychotherapy techniques should suit your requirement. If you require more of my help in this aspect, Please post a direct question to me in this URL. http://goo.gl/aYW2pR. Make sure that you include every minute details possible. I shall prescribe the needed psychotherapy techniques.Hope this answers your query. Available for further clarifications.Good luck."
},
{
"id": 152686,
"tgt": "Is oral chemotherapy safe for stage 3 rectal cancer?",
"src": "Patient: My sister has stage 3 rectal cancer her plan of treatment is chemo orally and ration so many days and so many days off. Also she is a dialysis patient 3 days a week my questions is oral medication ok because there no place to place the port due to moving it from place to place during dialysis. YYYY@YYYY Doctor: Hi there,I'm sorry to hear about your sister. She is lucky to have someone concerned about her to ask questions on her behalf. First of all, stage III rectal cancer is typically treated with both chemotherapy and radiation. Short-course radiation therapy is commonly used in Europe, but longer duration combination therapy is still preferred in the United States. Initial studies demonstrating its effect were undertaken in the adjuvant setting, in which 5-FU\u2013based chemotherapy with concomitant irradiation proved better than irradiation or surgery alone in preventing both local and distant recurrences. Prolonged infusional 5-FU and twice-daily oral capecitabine are considered equivalent in terms of radiosensitization, although no trial data are available to compare the two. The appropriate sequencing of therapy was assessed in a large German study, in which patients were randomly assigned to receive standard, infusional 5-FU plus irradiation either before or after definitive TME. Preoperative combined modality therapy resulted in a lower rate of local recurrence (6% vs 13% at 5 years), a lower toxicity rate (acute and chronic), and a higher rate of sphincter preservation than did postoperative therapy. Preoperative chemoradiotherapy has thus become the standard of care for patients with stages II and III rectal cancer.(Source - Andr\u00e9 T, Boni C, Navarro M, et al.: Improved overall survival with oxaliplatin, fluorouracil, and leucovorin as adjuvant treatment in stage II or III colon cancer in the MOSAIC trial. J Clin Oncol. 27 (19):3109-3116 2009 19451431)So basically, to answer your question, oral chemotherapy can be used, but IV chemo should also be used in order to maximize her chance of a cure. She will still need surgical excision, but this will be done after her chemotherapy and radiation is complete. If her dialysis access is a catheter, it is possible that they can use the catheter for chemotherapy infusion. I hope this is helpful. Please let me know if you have any further questions."
},
{
"id": 79116,
"tgt": "What causes vibration in the chest area?",
"src": "Patient: Dear Doctor, I have a strange buzzing feeling in my chest... like there s a cell phone inside my chest buzzing. I ve looked into this online and it seems to be quite common. Or at least I see a lot of people asking about what this is. I m not in any pain. I do have Hashimoto Thyroid. Can you tell me what this might be? Doctor: Thanks for your question on Health Care Magic. I can understand your concern. Vibrations in the chest are commonly seen in arrhythmia and palpitation. And both are common in thyroid diseases. Since you are having hashimoto 's thyroiditis, possibility of thyroid hormone imbalance as a cause for your Vibrations is more. So first get done thyroid function test. If this is normal than get done ecg, 2d echo and Holter monitoring (24 hour continuous monitoring of ecg) to rule out arrhythmia. If all these are normal than no need to worry for heart and thyroid problems. Sometimes, stress and anxiety can cause similar symptoms. So avoid stress and tension, be relax and calm. Consult psychiatrist and discuss all these. You may need anxiolytic drugs too. Don't worry, you will be alright, but first rule out thyroid and heart related diseases. Hope I have solved your query. Wish you good health. Thanks."
},
{
"id": 138327,
"tgt": "Suggest treatment for a broken humerus",
"src": "Patient: I have had four surgeries on a broken humerus, the head was snapped off the bone. First, a plate was put in to hold together, then removed because a screw broke and caught on plate. It was fused with cadaver bone. Three months later, it broke again from an auto accident. The third surgery put another plate back in and my own bone was used for hip, but prior to surgery a muscle test was done and at the time, I had lost over 30% of my muscle. Infection set in and after a month of iv antibiotics of Vancomiacin, the plate was put back in. This last surgery was done a week ago. There was always hardness, like a rock was in my arm, after the second surgery. I was wondering if it s possible that I have myopathies and if so, how do I know? Four of the five tests for infection were positive prior to the third surgery when the plate was put back in. Doctor: Dear Sir/MadamI have gone through your query and read your symptoms.In my opinion, yours is a tragedy story where some doctors feel only that only operation is the answer to fracture, had it been kept in cast, things would not have been that bad, but to be frank, its you also who has to be blamed, as you have had repeated insults to the bone. the hardness you talk of if its only in the limb where you were operated so many times could be due to fibrosis of repeated surgery and infection, but if this is throughout the body then other causes has to be sought out.I hope that answers your query. If you want any more clarification, contact me back."
},
{
"id": 160841,
"tgt": "Suggest treatment for loose stool while on ear infection medication",
"src": "Patient: Hi, I have a 6 month baby boy weighed 4 weeks ago at 21lbs. for the last 4 weeks he has been having 7oz formala feeds every 4 hrs during the day, 7am,11am,3pm,7pm, plus i have been weeaning him with baby cearal,fruit and veg purees 3 times a day too. All was going fine and he loved his solids and milk and has slept through the night for the last few months. Last friday he started to refuse his solid and milk, and vomited after a solid feed. I took hime to a doctor who said he had an ear infection and gave him pencillen to take 3 times a day. We are also giving him calpol. I took him back to the doctors on tuesday as he is still refusing all solids he won t let anything in his mouth and most bottles. They checked his throat which was fine and his ears were still red. Yesterday he had 5 oz at 9am in the morning and would not have anything in his mouth till 5pm when he had another 5oz. This is very inlike him to go such a long period without any food. He is still on the pencillen and has been for 6 days now. My question is, should i be worried he is not eating any solids or refusing some of his bottles. He is very pale and feels much lighter, he is waking through the night crying too. He is not vomiting and is not having many poo s but i guess he wouldnt as he has not had as much food. He does have bad smelly wind and when he does poo it is loose and has lots of mucus in it. When should i expect him to have regular bottles again? should i go back to my gp? Doctor: Hi,It seems he was having acute otitis media- infection of the middle ear. Your doctor has given him the right medications- if his pain and fussiness is coming down and no more fever, he is responding to the treatment. It make take a couple of days for his appetite to return to normalNow, regarding his loose stools, its quite common to have frequent loose stools after taking penicillins. This is because of change in normal flora of our bowel. Usually this will get back to normal after few days of completing antibiotic course. In such cases i used to prescribe probiotics for 3-5 days and rarely replacing the antibiotic if its disturbing much. If he not febrile, pain and fussiness has come down and is playful- nothing to worry, just wait 2-3 days- he will be alright. Hope I have answered your question. Let me know if I can assist you further. Regards, Dr. Muhammed Aslam T. K, Pediatrician"
},
{
"id": 172336,
"tgt": "Why is cough not curing even after taking ambrolex and cefalexin medoxine?",
"src": "Patient: my son is 3 months and 19 days. it's been week since he has a cough and plegm (colds). i already send him into a doctor and prescribed an ambroxol ambrolex (mucolytic) and cefalexin medoxine as antibiotic. but then for ten days now my baby is still sufferring from his cough, nothings change, his mucos is yellowish..please help, what is the best medicine can i use? or maybe it is need to change his antibacterial? ohhh... please help me asap now. thank you. Doctor: Hi Read ur question Ur baby had only complain of cough and cold then No need to take antibiotic . ambroxol is anti mucolytic. U have to show ur baby to paediatrician as 3 month baby not have that much longer cough . just give him burping after feeding for atleast 20min . it may also relived from mild cough For cold u have to us atarax drop 10 drops three times a day"
},
{
"id": 219809,
"tgt": "What causes lack of gestational sac on pregnancy scan?",
"src": "Patient: I got my first ultrasound yesterday when I turned exactly 7 weeks. They did both the vaginal and abdomonel ultrasound but all they could see was the gestational sac, no heartbeat or anything. Is everything OK? Did they just tell me wrong about how far along I was? Is there still a chance I could have an ectopical pregnancy? Doctor: if u r having regular periods and your gestational age is 7 wks,there should be development of fetal pole and cardiac activity..but still there may be chance of delayed ovulation if your periods are prolonged ..so best advice is to repeat scan after 10 days and see if there is development of fetal pole and heart beat.."
},
{
"id": 124760,
"tgt": "Suggest treatment for legs,feet and arm swelling",
"src": "Patient: My teenage daughter has been having trouble with light swelling of her legs, feet and arms. They turn purplish blue and also have white spots all over them. She also has several spider moles. She has had this for years, but seems to be getting worse. She also has food allergies. Doctor: Hello, We have to rule out possible causes like Reynaulds phenomenon and other vascular problems. Consult a general surgeon and plan for a vascular Doppler. Hope I have answered your query. Let me know if I can assist you further. Take care Regards, Dr Shinas Hussain, General & Family Physician"
},
{
"id": 196237,
"tgt": "Does the low size of penis cause sexual problems?",
"src": "Patient: Dear Doc,I am a guy of 25 years and the issue that really made me concerned is my sexual organ size since i have never had sex experience and living in a society where we have a different culture since my family is asking me to get married and I don't exactly know that am I capable of breaking the virginity or what is called hymen cause i have a 4 inch size but i don't have any other sexual problem please let me know or send me a study link many thanks in advance Doctor: penis size doesnt conerns with sex.you can actively involve in sexual life like other people.dont be afraid .dont try to use some local products assuming that they increse penis size.they will be having more side effects .dont fear.you can have a better sex life like others.all the best ."
},
{
"id": 150219,
"tgt": "Suffer from cervical spondylosis. No relief from pain medication. What to do?",
"src": "Patient: Hi, I suffer from cervical spondylosis of c4, c5, c6 n c 7. Medication has stopped having an effect on the pain. Even tramadol 500 mg or heavy muscle relaxants like Procyvon etc do not give me relief. Have stopped taking medication n started yoga. The severity n frequency has gone down now but I still get pain twice a week instead of 4 -5 times a week earlier and the pain always starts in d morning on waking up.Pls advise Doctor: 1. because diagnosis was made prior to treatment/medication it might be presumed that factors like subclavian stenosis/ cervical facet syndrome were ruled out?2. is morning pain associated with any numbness in arms,vertigo,or restriction of neck movements?3..try avoiding curd, jaggery,heavy and constipating food at night.. make sure to avoid late nights and afternoon naps..avoid stress,to back or spine,prefer hot water to drink. treatment may/may not RELIABLY stops progression of the disease,treatment aims to delay further deformity through good posture,stretching and deep breathing exercise.. You can consult an Ayurveda Physician there for ama pachana (toxin absorption).ama nirhana (toxin elimination) and shothahar (anti inflammatory) treatment..in between you can take natural calcium supplements like guggulu, muktashukti, milk,guava,sitaphal,ashwagandha"
},
{
"id": 54923,
"tgt": "What is the treatment for an enlarged liver?",
"src": "Patient: My father is suffering form chronic liver problem due to the intake of alcohol for past 30 years but in april month of 2012 because of an accident he suffered form multiple hemorrhage and hematomas due to this problem he lost his memory but now he is better but has not fully recovered he is undergoing neuro treatment and recently his liver ultrasound was done in which it was reported that the size of the liver has increased and i have attached the reports of the ultrasound. Doctor has advised only udilive 300 mg, Rifagut twice a day. Is this the only medicine available if not pls suggest. Doctor: Hi thanks for asking question...30 year alcohol drink is a very long history and your father might have cirrhosis...But here in history you have written enlarge liver so it can probably hepatitis here...Anyhow udiliv good drug that help in dissolving cholesterol in bile..Give him less fat diet and avoid refined food.Fruits juices given more.Give him daily green leafy vegetables...Take one tsp cumin seed powder and mix it with butter milk one glass and given orally daily.....Papaiya fruit useful.No more drug need.If edema occurs then diuretic drug might need.Don't self medicate any new drug.Low salt diet taken....I hope your concern solved,....."
},
{
"id": 81795,
"tgt": "Suggest remedy for fibrohazed infiltrates in the right upper lobe",
"src": "Patient: hi doc, my friend got her chest apicolordotic result and it said she had a fibrohazed infiltrates seen in the right upper lobe compatible with PTB of undetermined activity? does she needs to take the isoniaziz, rifampicin, ethambuthanol. she fears she might relieve from work. hope to here from you. thanks Doctor: Thanks for your question on HCM. In my opinion she should undergo CT THORAX and Bronchoscopy and BAL ( Broncho Alveolar Lavage ) analysis for TB, to determine activity of the lesion.No need to give prophylactic anti Tb drug before confirming activity of the lesion.As fibrotic lesion is seen mostly after healing from old infection. So may be she is having only scar lesion and not active lesion.So no need to give anti Tb drugs. Better to consult pulmonologist first and get done CT THORAX and Bronchoscopy to determine activity. And start treatment accordingly."
},
{
"id": 2836,
"tgt": "Do hot flashes and diarrhea indicate pregnancy?",
"src": "Patient: Hi..i had sex with my husband a week(3 times different days)before my period.he cum inside me..now ive got my period..its the third day today,i am feeling very nauseated,sick,getting hot and cold flushes and diarrhoea..is there any chance that I can be pregnant? Doctor: Hi,If you have got your periods normally, there is no chance of getting pregnant. If periods are less as compared to previous menses, you should contact your doctor. Symptoms which you are telling can be menstrual symptoms also.Hope I have answered your query. Let me know if I can assist you further. Regards,Dr. Khushboo Priya"
},
{
"id": 72065,
"tgt": "What causes chest pain with acid reflux?",
"src": "Patient: My daughter 14 and has been having really painful chest pains for a while now. she used to have acid reflux, we went to the doctor and they gave her medicine for it and it went. but a couple months on from then she's been getting a different pain on the left side of her chest, by her heart. it feels really tight and it's hard to breath. but she takes gavisgon and it goes. she gets woken up, mostly every night now, at about 2/3 in the morning. and sometimes get it in the evenings when lying down, or even just slouching in a chair. also, our family has history of twisted organs (don't know the medical term), my father had twisted lungs. is there something wrong with her, or nothing to worry about? thanks Doctor: Thanks for your question on Healthcare Magic.I can understand your concern. Possibility of GERD (gastroesophageal reflux disease) is more likely for her symptoms.GERD is due to laxity of gastroesophageal sphincter. Because of this, the acid of the stomach tends to come up in the esophagus and cause chest pain, chest tightness, breathing difficulty etc.So better to consult gastroenterologist and get done upper GI (gastrointestinal) scopy.Hiatus hernia should also be rule out. Treatment of GERD is lifestyle modifications as following. Avoid oily and spicy food. Avoid large meals, instead take frequent small meals. Avoid stress and tension, be relax and calm. Take pantoprazole tablet on empty stomach.Keep 2-3 pillows under head in bed to prevent reflux. Go for walking after meals. Don't, she will be alright with these lifestyle modifications.Hope I have solved your query. I will be happy to help you further. Wishing good health to your daughter. Thanks."
},
{
"id": 128256,
"tgt": "What causes painful cramps in the toes with tingling sensation in the calves?",
"src": "Patient: I m having painful cramping in my left toes and it feels somewhat like a charlie horse. It is difficult to get them to straighten back out. They are staying more bent than normal when not experiencing one of these cramps and I m unable to fully make a fist or bend them. With this my left leg is experiencing tingling in the back right above the knee down into the calf. What could this be and should I make an appointment that may takes weeks or go to an emergency room? Doctor: A neurologist consultation is a must.Id advise neuro-physiological testing of leg and foot nerve distribution.hot packs,massage with vitamin E oil,calcium,vitamin B complex and minerals like mangnesium, mayhelp.toe raise exercise I.e.standing on toes n ball of foot sole with heel high in air is good exercise walk bare feet on sand.another exercise is try to pick small objects by toes. Blood sugar testing to r/o diabetes"
},
{
"id": 71956,
"tgt": "What is pulmonary langerhans histiocytosis?",
"src": "Patient: Hi, may I answer your health queries right now ? Please type your query here...have been diagnosed with Pulmonary langerhans Histiocytosis X. 63 y/o white male and have been told my only hope for the future is a lung transplant. My PFT shows 26% lung capacity and my carbon dioxide/oxygen exchange last year was 51%, this year 41%. My doctor expects a continuing degradation and under pressure admitted that I have probably 4 years left. Should I obtain a second opinion? What are my chances of obtaining a lung transplant? Should I even bother to pursue this option? Doctor: HelloRegarding the history i fully agree that unfortunately only lung transplant is helpful in such conditions.RegardsDr.Jolanda"
},
{
"id": 84889,
"tgt": "What causes headache and weakness after taking Zinetac tablets?",
"src": "Patient: sir iam having acitidy problem since 10days iam went to doctor he had given me zinetac tab,pantodac tab,and digene syrup but at the i started taking it iam getting iticing,headache,stomach getting tighted,weakness.and till now its like that only not a single change Doctor: Hi.I can understand your concern.Suggestive of side effects of Zinetac.Headache, dizziness, nausea or vomiting, stomach upset, and allergic rash are some of important side effects of Zinetac.Consult your treating doctor for an effective alternative but a safer medication or a dosage reduction if the headaches are very debilitating.Meanwhile take rest in a quiet and dimly lit room, heat or massage therapy may help if the headaches recur. Drink plenty of water and keep yourself well hydrated.Hope I have answered your query.A feedback is appreciated."
},
{
"id": 174861,
"tgt": "Is it safe to continue the injection for loose stools?",
"src": "Patient: Hi, My 12month old having liquid stool and when stool is cultured report says it contains E.colic bacteria.Doctor prescribed netromycin 10mg injection 2 times daily for 3 days.After first injection was given he got high fever Is it safe to continue the injection. Doctor: Netromycin is frequently used in young children and is generally tolerated well. Since it has been started, the course needs to be completed. The side effects could be effects on the ears and the kidneys. But these are infrequently seen on short term course like that of 3 days.E.coli is a part of our normal gastrointestinal bacterial flora and if stool of any individual is cultured, E.coli or certain organisms are definite to be found. It does not mean infection."
},
{
"id": 214236,
"tgt": "Suggest home remedy for loose motion while on medication for constipation",
"src": "Patient: I m 27 years old and suffering from loose motion as i have been doing gym for last 4 months and lost 10 kg weight and reach to 62 kg now to get 6 pack abs taking supplementa and for last 5 days low carbs diet and now 2 days back i was in concipation condition. I went to doctor and took some medicine and since last night after taking one blue pill with hot milk as per Dr instruction it cauased me like vomiting and now suffering from loose motion. Just i took one Nflox TZ for relief after Psyllium Husk Poweder with curd and waiting for result. Could you suggest is the right way to get relief by home treatment. Doctor: Hi, I understand your concern. First thing to do is to withdraw the medicine which caused the side effect. Maintain fluid balance by taking ORT. Take light nourishing liquid diet with salt & sugar, boiled rice with curds & salt, light Sago pudding. The loose motions should be controlled in 1-2 days. To avoid constipation in future, have high fiber diet with plenty of water(10-12 glasses in a day), hot milk with clear butter1 spoon at bed time helps to soothen the stools. Thanks."
},
{
"id": 140526,
"tgt": "How can a painful lump above the collar bone while suffering from neck stiffness be treated?",
"src": "Patient: my neck is very stiff. it appears a knot above collar bone, back side. its painful,have trouble sleeping. when i try to massage it, a small sharp shock occurs and slightly clicks years ago i was t boned, and had trigger point injections. 2.that was over 12/ 14 years ago. im trying ice. can you help me please. Doctor: Hello, If you are suffering from an acute bout of cervical pain that was not really there before recently (even though you do have the remote history of the accident) and you believe there is a lump of some sort above the collar bone then. I would recommend you see a doctor and have that looked at to make sure it is not a mass that is impinging upon one of the cervical nerves causing the pain and electric shock-like sensation. Ice is an acceptable intermediate therapy but this should be looked at by a doctor with possible imaging studies to define the knot a bit more precisely. Hope I have answered your query. Let me know if I can assist you further. Take care Regards, Dr Dariush Saghafi, Neurologist"
},
{
"id": 176330,
"tgt": "Found blood in my baby s urine. Is it normal?",
"src": "Patient: My baby has the flu yesterday and I took her to our dr. He wanted to try gravol before he admitted her. ... she stopped puking last night around 8 and that was pure bile... I gave get 2oz of pedialite then she slept I woke her up at 3 have her agravol and tylonal for a fever and laid her down again. .. she had not peed yet. .. when I changed her diaper2 hrs later there was blood in her urine not a lot but it was def blood was red and In the front of her diaper notthe back. .. she just spit up a bit more pediatite on me and I went to go change her diaper and all and there was blood again. .. that s not normal is it? ? She s 5 months old Doctor: Hi..this is not normal. In view of throwing up, I feel that she might be having a urinary tract infection or a renal colic too. Other option (though remote) is that, blood could have been from the stools. In such case the possibility of bacterial dysentery has too be considered. Unless the kid's having low urine output or very dull or excessively sleepy or green bilious vomiting...you need not worry. But I suggest you take her to her paediatrician.Regards - Dr. Sumanth"
},
{
"id": 114631,
"tgt": "What causes heavy menstrual bleeding while having anemia?",
"src": "Patient: About two weeks ago i was feeling nauseous, and had swelling in my ankles, and slightly dizzy, i thought that i may have been pregnant but not sure, but now i have had my period which is very heavy, it has always been heavy. but i had heavy cramps and i have not had cramps in years, i have heavy periods but no cramps, could this have been an ecoptic pregnancy or just the fact that i have anemia and my hemoglobin levels were low Doctor: heavy periods and anemia are related both ways. Prolonged or heavy bleeding causing fall in hemoglobin is indication for blood transfusion most of times. So visit to hospital may be needed if symptoms are present or ongoing bleeding. Gynecologist review check needed. get basic Hemograme with smear."
},
{
"id": 222155,
"tgt": "What are the early signs and symptoms of pregnancy?",
"src": "Patient: my wife missed her peeriods for 10 days now. She is always tired now, mostly sick in the morning. She has vomiting sensation after taking food but not vomiting. gets headache often, frequent urination are other symptoms. Is she pregnant ? We were sexually active for more than a month continuously.. Doctor: Hello dear,I understand your concern.In my opinion all your said symptoms suggest pregnancy.But it needs to be confirmed by doing a urine pregnancy test a week after missed period.The test will be mostly positive by now if there is pregnancy.So consider doing urine pregnancy test.And if positive consult gynaecologist as you need prenatals like folic acid .And also take medicines to decrease sickness.BBest regards...."
},
{
"id": 69283,
"tgt": "What causes lumps in my neck?",
"src": "Patient: After my initial discovery of \u00e0 small lump in my neck I had an ultrasound (result: infiltration of subcutaneous fat), a fine needle biopsie (result: inconclusive) after which the hospital removed the whole lump (result: you are ok). After the operation I soon felt \u00e0 big lump forming which is still in my neck. It is really big. Scar tissue? I am deeply worried. You can actually see the lump. I was so happy for being ok. Do I have cancer after all? Doctor: Hi,Welcome to HCM.If the lump has formed soon after the surgery, then it could be a hematoma (blood clot) due to a minor bleeding following the surgery.It will resolve in few weeks, depending upon the size.Keep a watch over it, if it gets painful or red, it means it has got infected and then it needs evacuation and antibiotics.Its not a cancer.RegardsDr. Ashish Verma"
},
{
"id": 42039,
"tgt": "How to get pregnant while suffering form PCOS?",
"src": "Patient: ive been ttc since the past year, ive had two late miscarriages and had a d&c done. the doctors said i have pcos im just really worried because my mentrual cycle is irregualr i have my menstruation for 14 days and they are light in the beginning. please can you give me some advice or Dua to help me. thank you Doctor: Hi, Welcome to HealthcareMagic . First you need to regularise your cycles by taking oral contraceptive pills for 6 cycles and reducing weight, metformin tablet and multivitamin tablets after consulting your doctor. With this your pcos will be treated and also cycles get regularised. After that you may start taking ovulation inducing drugs like clomephine . This will help multiple follicles to mature and rupture . Thus increasing chances of pregnancy . Hope I have answered your query. RegardsDr.Deepika Patil"
},
{
"id": 119642,
"tgt": "Why my leg shakes when keeping the leg on leaned surface?",
"src": "Patient: hello. I wish I could find Answer why my leg shakes if I put my feet on a leaning surface or on a stretcher of a chair and keep the other on the ground. I don t exert any force to keep balance, and i don t feel any pain. But if somebody tells me about it and I see it, it shakes fast and I have to take it off the stretcher even if It feels comfortable to sit like that. Doctor: Hi, Hyperthyroidism or anxiety can cause such shaking and tremors. So better to check your thyroid profile to rule out that. In case of anxiety breathing exercises and beta blockers like metaprolol will be helpful. Hope I have answered your query. Let me know if I can assist you further. Take care Regards, Dr. Nishad BN"
},
{
"id": 159107,
"tgt": "Over weight, breast cancer, invasive tumor. Taking lisinopril, generic prozac, multivitamins. Help?",
"src": "Patient: I take lisinopril , generic prozac , multivitamis for adults over 50, glucosamine chondroitin and claritin. Do any of these drugs cause high calcium. I am 60, 80 to 100 pounds over weight and had breast cancer when I was 35. It was an invasive tumor 2.5 cm and I had a modified right breast mastectomy, No chem or radiation tx. My Dr. did blood tests and told me my calcium was 10, something 5 days ago. Doctor: Hi, Your Ca++ is within normal limit, those drugs has little chance of causing hypercalcemia, anyway overweight is not a good for breast cancer survivor, You have to keep your BMI 25-30 for better control of disease, Consult your oncologist."
},
{
"id": 35681,
"tgt": "What causes fluctuation in body temperature after IVF treatment?",
"src": "Patient: I am a IVF patient. Embryo transfer happened on 22nd Nov. For the past 3 days plus today, I am having fluctuation in the body temparature. Today afternoon, it is 102. What is the reason. Having Dolo 650 - half from today - morning, afternoon based on my Dr's advice. Doctor: Hello,Thank you for your contact to health care magic.I read and understand your concern. I am Dr Arun Tank answering your concern.No, body temperature never rises after IVF, but it can be monitored during the IVF procedure. Once the IVF is success there won't be any fluctuation in temperature.I advice you should look for the other causes of the temperature.Some other infection can cause rise in temperature. You can take paracetamol for the fever under your doctors guidance. But please consult once to doctor for this fluctuation in temperature.I will be happy to answer your further concern on bit.ly/DrArun.Thank you,Dr Arun TankInfectious diseases specialist,HCM."
},
{
"id": 220481,
"tgt": "No periods yet, sore breast, headache, back pain, thick white discharge with odor, hpt negative. Pregnant?",
"src": "Patient: I'm 10 days late. no sign of my menstral cycle coming. and I'm having symptoms of being pregnant. my boobs are sore but not too sore I'm getting headaches, my lower back hurts and I have thick white discharge. with no smell or odor. is there a possibility that I'm pregnant and still getting negative pregnancy test results? Doctor: Hello,First of all, you need to undergo one blood test for beta-HCG and TVS scan to rule out pregnancy at the earliest. Early pregnancy test may give negative result.In exclusion of pregnancy, other reasons are hormonal imbalance, thyroid disturbance, vaginal or pelvic infection, ovarian cyst, stress/ anxiety, over wt or loss, irregular diet habit etc.Please consult with gynecologist following test reports to proceed other relevant investigations to pin point the diagnosis. Take suggestive measures accordingly.Try to avoid stress/ anxiety, take healthy diet with iron supplements, drink plenty of water, maintain genital hygiene and take sound sleep. Good luck."
},
{
"id": 196184,
"tgt": "Is watery and transparant sperms instead of being creamy normal?",
"src": "Patient: Hello,Just want to know about the difference in \" outcome\" of my sperm.I am 70 yo and still enjoy masturbating since I have no partner to play with .Since several month now my sperm does not look the same : whitish and creamy, as usual, but more waterly and transparant like if it is more about precum . ?Waht can cause this change. ?Thanks for your answer. Doctor: as the age advances semen production also decreases.more semen production makes it more thick.in old age,as its production decreases makes it more watery.also over masturbation (more than once a day)can cause watery semen."
},
{
"id": 79347,
"tgt": "What causes breathing difficulty with sneezing and wheezing?",
"src": "Patient: For the past 2 years I got breathing difficulty in the months of Febraury and March alone.It usually starts with continuous sneezes for 2-3 days then vezzing.doctors prescribed me Deriphylin to control that.Also they gave Clavam for antibiotics.During the course of the medicine i was alright.Now i used to get only sneeze and runny nose always.I find it very difficult.Also i am afraid that whether i will get vzing next Feb.march too. Please give the way to overcome this difficulty completly. Thanking you. Doctor: Thanks for your question on Health Care Magic. I can understand your situation and problem. By your history and description, possibility of seasonal rhinitis and asthma is more. Your symptoms typically seen in February and March. This is characteristic of seasonal disease. Your symptoms like sneezing, running nose, wheezing etc are suggestive of rhinitis and asthma. So you need inhaled bronchodilators and inhaled corticosteroid (ICS), combination of antihistamine and anti allergic drugs and intranasal spray containing steroid. Start these drugs in mid January and continue till March ends. Since all these drugs are prescribed medicines, you need to consult pulmonologist for them. So consult pulmonologist and discuss all these. Hope I have solved your query. Wish you good health. Thanks."
},
{
"id": 188734,
"tgt": "Inflamed gums, headaches, dizziness. Family history of HHT. What should be done?",
"src": "Patient: My sister and uncleboth died of pulmonary hypertension. Now family members tell me the correct diagnosis was HHT. My sister was 40 when she died. Although no one seems to have symptoms should my family and my sisters also be tested? I have gingivitis and have my teeth cleaned every 3 months. These past few weeks i think my gums are inflammed. i have had some headaches, been dizzy and not feeling well in general. What should I do? Doctor: Hello,Thanks for writing to us.Since your gums are inflammed,you must get full mouth scaling done.Maintain oral hygiene well.Brush and floss your teeth as well as tongue daily.Use chlorhexidine mouthrinses to gargle.Regarding your physical concern,please do visit a GP ..Hope this helps."
},
{
"id": 175967,
"tgt": "How long does marijuana stay in an unborn baby s system?",
"src": "Patient: Hello I am 35 weeks pregnant and I recently stopped smoking marijuana. I was only smoking to control my morning sickness and my doctor recently told me that my baby will still have traces of the marijuana in his system when he is born. Is this true? I have a very high metabolism so my real question is, how long does marijuana stay in an unborn baby s system? Doctor: yes it can stay. and moreover smoking causes low birth weight babies. there are other safe treatment for morning sickness. you should not have smoked weed"
},
{
"id": 193693,
"tgt": "What are the chances of pregnancy having low sperm count?",
"src": "Patient: hello doctor i am 31 years old and married since 5 years but still not pregnent have all test done every things normal my hubby count is also normal now i am on letoval2.5 for 5 days from day 5 and lynoral from day 5 upto 21 days... i am having 22 days period and with watey discharge throught period...pls suggest me the current drugs are how much effective? and i also have hcg10000 iu shot on day 14 but no result Doctor: Hi, Your drugs are effective. Still, it depends on person to person. There are multiple probabilities for the delay in conception. Hope I have answered your query. Let me know if I can assist you further. Take care Regards, Dr S.R.Raveendran, Sexologist"
},
{
"id": 200252,
"tgt": "What causes small spreading spots on my penis?",
"src": "Patient: For the past few months I have had spots on my penis emerging. Started on shaft mostly. Little bumps you can barely feel or notice when not erect. When erect they are more visible. Tested for std s and came back fine. Tried fungal cream etc. nothing working. Started slowly spreading and now some on head of penis. You can see example in photo. Any feedback on what it could be would be appreciated. Thanks. Doctor: Thanks for asking in healthcaremagic forum Sorry, I do no have access to your pic. As it is not causing you any problem, it is most likely to be fordyce's spots, which are visible sebaceous glands over there and are harmless. Please visit a dermatologist if it is causing you cosmetic problem. All the best."
},
{
"id": 49524,
"tgt": "Have IVP test result. Can any online doctor help?",
"src": "Patient: Good evening doctor, i am sagunthala ,i felt back pain in left kidney,Doctor suggested me IVP test and IVP report showed that partial left pelviureteric junction obstruction causing grade 3 hydroureteronephrosis and prompt contrast excretion from both kidneys.Doctor reported me go for surgery.What is the problems with that, if surgery is necessary or any other test is required or due to prenatal PVJ or due to pregnancy.please suggested your answer. Doctor: Hi,as per your reports there is obstruction at pelviureteric juction leading swelling of kidney which now level 3. Surgical intervention is needed asap. Otherwise kidney may become non functional due to back pressure of urine. Obstruction needs to removed. Medical treatment is of no use in this case.If there is subsequent pregnancy then both nephro surgeon and obs/gyn need to be consulted.regards"
},
{
"id": 113556,
"tgt": "Fall in lower back, soreness, pain above hips and between ribs. Any thoughts?",
"src": "Patient: Thanks for taking my inquiry! My wife fell on the stairs earlier today. She slipped decending the stairs and hit a stair frame in her lower back. I was at work when she fell. She went through her busy schedule with kids events and though she said her back was very sore, she didnt stop. Now, it is evening and she is really complaining that her back is hurting. She can not lay down and sitting hurts. I have put hand pressure on her lower ribs in back and she says that there is no pain. She says the pain is located above her hips and between her ribs, in soft tissue . Any thoughts? And yes, we will go to the doctor in the AM. Doctor: Hi Welcome to HCM. Since she is not in excruciating pain and being ambulant , it does not look like a fracture. It should only a soft tissue injury and should heal with rest, ice and pan killers. Still, she needs to be examined physically--- Regards"
},
{
"id": 61059,
"tgt": "What does a lump on the abdomen indicate?",
"src": "Patient: Step-Daughter has a bulge/lump on left abdomen about belly button height. Quite large. Has to hold stomach when she coughs, etc. her mother had a hernia when she was around the same age that ruptured and she had emergency surgery and was very ill. Don\u2019t know if it\u2019s the same, but also worry about lymphoma. Doctor: Hello dearWarm welcome to Healthcaremagic.comI have evaluated your query for your step daughter in details .* This seems more likely to be hernia , rather than lymphoma .* Kindly get an ultrasound examination of the abdomen & pelvis for further management guidelines .Hope this clears your doubt .Wishing her fine recovery .Welcome for any further assistance .Regards take care .Dr. Bhagyesh ( MS , FMAS - consultant surgeon )"
},
{
"id": 142137,
"tgt": "What is the treatment for dizziness and tremors?",
"src": "Patient: Hi doc so lately i have been getting dizzy while i am standing my vision goes black i loose my hearing and all i hear is like a ringing/buzzing noise i also get very weak and have to sit down because i feel like i am going to be ill i also get very pail and shake even more than i normally do can you please help me? Doctor: Hellothank you for trusting HCMDear you may be developing postural hypotension. caused by decreased blood volume in dehydration,of blood loss or decresed oxygen carrying capacity in anemia, thalassemia etc.. or decreased blood pressure in septicaemia, hypovolemia, cardiac problem. vestibular apparatus problem like BPPV, vestibular neuronitis, labyrinthitis etc...please get done hemoglobin level, ECG, chest x-ray,bp monitoring in lying down and erect posture,CT scan, vestibular apparatus examination needed. use tab.diligon 25 twice a day for five days. please consult your neurophysician/cardiologist they will examine and treat you accordingly.take care"
},
{
"id": 108114,
"tgt": "What causes stomach pain with very bad back ache?",
"src": "Patient: Dear Doctor, i m 33 years old i have a very bad pain in my back and stomach wich get worse during the night for more than 3 months even i can t sleep after 2 o clock i wake up tell morining when my stomach start to some noise i can sleep ( stomch is geting sowllen and full of gas). kindly assist me. thanks Doctor: Hi,your symptoms might be because of parasitic infection giardiasis better to use a course of flagyl medications oral 500 mg twice daily for five days after asking your physician It may be because of pancreatitis and gallstones it will be better to avoid fatty food, smoking alchol and other modifiable risk factors(smoking, alchol wieght reduction)It will be better to undergo ultrasonography and stool examination,serum lipase lavels once amd start the antiprotozoal for giardiasis after Consulting your physicianLast but not least avoid fatty foods,smoking reduce alchol intake,wieght loss will help."
},
{
"id": 124777,
"tgt": "What causes numbness and tingling on toes after a bunion surgery?",
"src": "Patient: Hi....I had bunion surgery 3 weeks ago this Friday. I went to my doctor this past Tuesday and he put a hard cast on my foot. My foot and my toes are numb almost constantly and they tingle. Is this normal? I ve never had a cast before, but it s driving me crazy! If it s normal, then I will just deal with the annoyance for the next 4 weeks, however, if it s not normal, then I will call and schedule an appointment to go in and have them fix it. Thank you in advance for your advise. Doctor: Hello, Abnormal feelings like pain or tingling after plaster cast is applied is not normal. you must visit the ER. It needs to be changed and wound if any be inspected. Hope I have answered your query. Let me know if I can assist you further. Take care Regards, Dr Nirmal Chander Gupta, Orthopaedic Surgeon"
},
{
"id": 110867,
"tgt": "How to treat lower back pain?",
"src": "Patient: ive been having lower back pain for several months continuously, ive been to my GP for 3 times already but all he does is give me pain releiver which is not helping. i work as a care assistant and im afraid i might have hurt my back,, is it possible that i may have had a slip disc? Doctor: Hello, Thanks for your query.You seem to have a prolapsed inter vertebral disc. This disc may be compressing upon a nerve root that either exits or traverses through that level. Having known this basic anatomy, you must avoid any activity that aggravates the condition, like bending forward and lifting heavy objects, sitting long hours on computer, inappropriate sitting posture, etc.The medications will relieve pain and help muscles to relax. should Physiotherapy shall also help a lot, especially exercises that strengthen your back. Kindly note that exercises only be done when it doesn't hurt. Adequate rest and proper nutrition is equally important. Drink lots of fluids since dehydration affects the disc. May be its time you need an MRI done to have a diagnosis established. With that one can formulate a definitive treatment plan.I do hope that you have found something helpful and I will be glad to answer any further query.Take care"
},
{
"id": 66904,
"tgt": "What causes lump in throat, fever and runny nose?",
"src": "Patient: Hi, I had a fever starting about two weeks ago. Just the general runny nose, sore throat , etc. It subsided about a week ago but now i have a lump in my throat which is seriously sore and continually getting worse. What is it? I am on a university campus and health service closed at 5 and had classes all day so wasnt able to go. Doctor: it is due to enlarged tonsils or the lymph nodes and these are all body's immunity to infective agents and not to worry too much!take hot saline gargles thrice twenty times daily for 1/2 days and if not resolving take antibiotics!all the best!"
},
{
"id": 46015,
"tgt": "Could cough be due to kidney stone reformation?",
"src": "Patient: I have had a really bad cough for the past week with no signs of infection -- no congestion, no fever, no sinus pain. It's a post-nasal drip type of cough. I had this same cough last June and when it didn't go away, tests showed I had a UTI (with no symptoms). Took an antibiotic and a month later still had the UTI (again with no symptoms). Was sent to a urologist and, after a KUB CT scan, it was found that I had a very large kidney stone. Urine culture showed the presence of proteus bacteria generated by this stone. I had surgery to remove this stone (actually three procedures) and, though I am not totally 100% stone-free, the urologist was happy with the outcome and a CT showed only fragments remaining. Could this cough once again be a sign of stone formation that is producing the Proteus bacteria? A course of Z-pack since Friday hasn't even touched the cough. Doctor: good day and thank you for being with healthcare magic!I don't think the cough is related to your stones urinary tract infection and Proteus."
},
{
"id": 92422,
"tgt": "Developed abdominal external bleed, severe abdominal pain. What might be the cause?",
"src": "Patient: I have developed a slight abdominal external bleed. 4 or 5 days ago I have severe abdominal pain. Across the upper part of my stomach and continuing down my right side into the groin area. The pain has subsided with only mild tenderness, now the slight bleed has occurred. Doctor: Hi, Thanks for your query.According to me the most likely cause of your right abdominal pain would be a renal colic because of some kidney calculi(stone).I could not understand the bleed you are trying to explain. Is it bleeding in urine or with stools. Bleeding in urine can occur with renal stones.Meanwhile take plenty of fluids, painkiller in form of diclofenac 50mg twice daily and tab buscopan SOS.in case pain recurs kindly consult your GP and get your ultasound abdomen.Hope it helps."
},
{
"id": 164926,
"tgt": "What causes red pimple like bumps with whiteheads on thighs?",
"src": "Patient: My nine year-old daughter has pimple-like bumps on her upper inner thighs and bottom. They begin as colorless small bumps and progress to a red pimple with a whitehead that pops. I thought perhaps it was due to not wiping well enough after she goes to the bathroom but it is not getting better. I am going to call her pediatrician tomorrow am. Originally the doctor said it was irritation from underwear. Any ideas in the meantime? Doctor: hi, thanks for the question. from your history it looks your child has folliculitis, a form of bacterial skin infection which can be treated by application of local antibiotics like mupirocin."
},
{
"id": 178791,
"tgt": "Is there any problem for taking over dose of synthroid 88?",
"src": "Patient: Do I need to worry if my 3 yr old soon to b 4 in aug, 38lbs.....could have possibly taken 9 pills of synthroid 88. Called poison control and they said not to worry that he will b very hyperactive and moody for nxt 4 days. But my wife is still concerned even though we were told it will b ok after it works out of his system. What symptoms should we look for for the worst case scenario? Not sure if he took that many or not....that s the impression we are getting sence he threw away one of the packages and we can t find it now. That was earlier this morning not sure exactly what time.....but Definately over 9 hrs ago. Doctor: Hi thank you for choosing healthcare magic. Regarding your son don't worry that much and just wait and watch as he must not have taken that many tablets as he is fine, you should expect hyperactivity, increased heart rate, not able to sleep, irritability and tremors of his hands etc, as he doesn't have these after 9 hrs, I don't think you should worry that much. Just monitor him for another few days and he should be fine.Hope this has been helpful to you, kindly feel free to contact again for any queries. Kindly rate the answer if so. Take care. Dr J Sravanthi"
},
{
"id": 139842,
"tgt": "Suggest recovery time for dysphasia?",
"src": "Patient: my father has been injured in RTA having head injury with multiple contusion, mri brain shows subdural sub acute hematoma formation in right occipito-parieto-temporal lobe causing mild mass effects with pacthy bleed in left centrum semiovale. he is disoriented nd dysphasia now. how much time it will take to be in fully recovery... doctor prescribed tab epsolin 1 tds, epilive 500 1 od, nexfolin 1 od. Doctor: Hello, The recovery from this situation is slow and it may take up to 3-4 months. During this period of time, repeated CT scans and close monitoring of his blood pressure values is necessary. Hope I have answered your query. Let me know if I can assist you further. Take care Regards, Dr Ilir Sharka, Cardiologist"
},
{
"id": 54585,
"tgt": "What causes pain in gall bladder area?",
"src": "Patient: hi , i have done gallbladder surgery exactly 1 yr. back. In whole this 1 yr i never get a problem from removal of gallbladder i.e. no pain , no nausea .but from last 2 days m having slightly pain on gallbladder area with back pain on my right side of shoulder . which come on and off Now i want to conceive baby , want to plan for second child but i m scared of this pain ? I don't know what it is and what to do ? please help ! Doctor: thank you for posting query.need further evaluation.advice:- get an ultrasound (abdomen) to rule out any stone remnants in common bile duct- also get a complete blood cell count(CBC) test- avoid oily food altogether- you may need to take nexium/pantocid 40mg once daily before breakfast- avoid eating stomach full and mouth full- avoid spicy food wish you good health.further queries are welcomed.Health professionals aim to diagnose properly and manage patients according to their limited knowledge. Cure is blessed by the ONE who Created us, whose power and knowledge is unlimited .wish you good health.regards,Dr Tayyab Malik"
},
{
"id": 6994,
"tgt": "I am Ab +ve and about to marry a O",
"src": "Patient: I am Ab +ve and about to marry a O - ve Guy. How does it effect to my body after marriage. was there any issue for conceiving? Doctor: Hi, Welcome to HCM. In your case there is no problem. If the female is Rh Negative and the Male is Rh Positive, there is a possibility that their child may be Rh Positive. In such cases there may be problem in the subsequent pregnancy, for which preventive measures are available. If the child is negative then there is no problem."
},
{
"id": 213125,
"tgt": "Pregnant. Suffering from anxiety, bipolar disorder and depression. Is it safe to take anxiety medicine?",
"src": "Patient: i am 9 weeks pregnant and i have not seen a doctor scince i got my proof of prgnancy because i have to wait till the 20th of this month. i have ptsd, anxiety , bipolar disorder , and depression . i have not been taking my medications and my anxiety has been really bothering me for the last 3 days, and its really upsetting me and making me worry. i have tried alot of other things to help me calm down but its just getting worse. can i take my anxiety medication? Doctor: hi..first trimester is vital for the organ-development of foetus..but apart from this, judicious use of medication is recommended as continuing with your bipolar depression may hamper the birth weight of the baby..for anxiety, a minimum dosage of anxiolytic may well be chosen..and as you progress further with pregnancy, dose adjustment is required in a proper manner..so visit your psychiatrist regarding all these..have a healthy life"
},
{
"id": 51553,
"tgt": "I am a diabetic patient, albumin in urine",
"src": "Patient: Hi, i am a diabetic patient since 18 years, now the albumin is present in my urine sample since 1 year, but the blood urea and creatinine is normal and scanning report of kidney also normal, the doctor says here, i am admitted for test for kidney biopsy, please advise me sir. and also i have no retinopathy and any heart diseases. Doctor: Hi Chandru,.....albuminuria is an early sign of kidney disease...now it could be because of diabetes or any other cause also in your case....hence, consult a nephrologist, get quantification of your proteinuria and if necessary get a kidney biopsy done...if it turns out to be diabetic nephropathy then you need rigorous control of your DM and BP and if anything else, then get treated for that disease specifically...bye."
},
{
"id": 101639,
"tgt": "What is the best treatment for wheezing in children?",
"src": "Patient: Hi my 2 & 1/2 year old boy kid have showing the symptoms of wheezing one week back and it was the first time since his birth, I have consulted with the doctor and advised for antibiotic for wheezing & allergy, I am sure that, this kind of routine treatment not enough for curing this ailment Doctor: Hello.As your child already has wheezing for more than one week, I would consider nebulization with salbutamol & oral montelukast.I would definitely advise you to consult a pediatrician who will examine the child & suggest a suitable schedule of nebulization.I would also advise light, warm diet & fluids to the child.Hope my answer would be helpful to you.Wish your child a faster recovery & the best health.Regards,Dr Parin"
},
{
"id": 67601,
"tgt": "Suggest remedy for lump under skin near elbow",
"src": "Patient: My daughter has what feels like a knot under the skin near her elbow. It is about the size of a pea and you can only see it when she bends her elbow. She has not complained about any pain in that area or her arm in general. Is this something I should seek immediate care for? Doctor: Welcome to health care magic. 1.The most possible cause could be ganglion - usually exist from the nerve, tendon and joint space.2.They are painless, asymptomatic, does not need any acute treatment.3.Usually seen in the sports persons involving active limb movement.4.If you feel any growth in the lump - an ultrasound is investigation of choice to evaluate the nature of the lesion, its source and extensions.5.Suggest to monitor the lesion for any changes, and no active intervention is needed.Good luck.Hope i have answered your query,any thing to ask ? do not hesitate to ask.http://doctor.healthcaremagic.com/doctors/dr-ganesh/62888"
},
{
"id": 76558,
"tgt": "Suggest treatment for lungs tuberculosis",
"src": "Patient: I hav been detected with lung tb coz i used to smoke ,i am on medication now n will complete my 6 month course as reccommended by my doctor on aug 19th ,my age 25 and im a female n a flight attendant by profession , but i hav been told that my tb mark wont go for 2 years atleast and this would be an obstacle for my career, plz advice Doctor: Hello, thanks for using Healthcaremagic. i had gone through the dtails you have posted. tuberculosis commonly infects lungs. it produces pneumonia if not treated at this stage it prodeuces cavitation and then heals by fibrosis. if fibrocavity or fibrosis is establisted it wont go off totally. so whether you have this fibrosis or not we need to have a look at your chest x ray. though fibrosis is seen on chest x ray you will be cured for tuberculosis...Hope this helps.."
},
{
"id": 202632,
"tgt": "What causes dryness on the foreskin with little red tears?",
"src": "Patient: Hi there. I recently (around 6 weeks ago) started seeing this girl, who is now my girlfriend. We have had lots of unprotected sex (she is on the pill), but in the last week I have noticed that the end of my foreskin keeps drying out, and consequently flaking. It is irritating if I pull it back, as the skin is so tight from being dry. If I do pull it back, the skin develops little read tears.I just want to know really if my new sexual partner has given me an STI? As I have never had this problem with any other girl I have had sexual relations with.Any help would be much appreciated :) Thanks Doctor: Both of you can under go std test. If negative use vesline ointment over fore skin and use condom, which should be lubric ated."
},
{
"id": 157638,
"tgt": "Having lump sensation in throat after eating, no pain. Is this cancer or some form of indigestion?",
"src": "Patient: I have had a lump sensation in my throat for a few months now. The sensation comes and goes and there is no pain associated with it. The lump sensation is comes mostly after I eat and is accompanied by a lot of burping. It is especially irritating around my menstrual cycle. What could this be, is this cancer or some form of indigestion? Doctor: Hi, Relax, your problem does not fit in a description of cancer. It seems most likely to be GERD and please take proton pump inhibitors with prokinetics for few days. Use mucain gel before meals and avoid spicy and oily foods. Regards"
},
{
"id": 52949,
"tgt": "Suggest treatment for fatigue and shortness of breath post gall bladder removal",
"src": "Patient: My name is Rose 56 years old. 12 years ago had gall bladder removed, since then have alway had digestive problems. Lately have been experiencing extreme fatigue always feeling tired. Always out of breathe I know my weight has a lot to with it (am over weight). This last week and a half have been experiencing mild nausea everyday I have an appetite but once I start to eat I get nauseated or grossed out have a low grade fever off and on. Getting concerned since my mother passed from pancreatic cancer. Doctor: Hi, I had gone through your question and understand your concerns.This looks to me like so called postcholecystectomy syndrome which has all these symptoms and this occurs in about 5 to 40 percent of patients who undergo cholecystectomy, usually can be transient, and rarely persistent or lifelong. Symptoms include fatigue, dyspepsia, nausea, and vomiting, flatulence, bloating, and diarrhea, persistent pain in the upper right abdomen. Some individuals may benefit from diet modification, such as a reduced fat diet, following cholecystectomy since it may be more difficult for digestion of fatty foods. Postcholecystectomy syndrome treatment depends on the identified violations that led to it. Typically, the patient is recommended dietary restriction table with fatty foods, enzyme preparations, antispasmodics, sometimes oral ursodeoxycholic acid can alleviate the condition.Hope this answers your question. If you have additional questions or follow up questions then please do not hesitate in writing to us. I will be happy to answer your questions. Wishing you good health."
},
{
"id": 107651,
"tgt": "Suggest treatment for back and hip pain",
"src": "Patient: Iv been getting a lot of back and hip pain after I had MRI scans to show a slip disk I was also allowed a nurve root block on l5 this has help also the MRI scan piled up 2 cyst and said to send me to see a gynaecologist. My gp also done a internal and seen 2 white cyst. So when I went to the hospital 5 days later to have a internal scan there was nothing there. So I m confused. Iv also started bleeding 1 week after my period finished. Seen my gp who said maybe the cyst has just gone!. What with in 5 days. Please advise Doctor: Slip disc should be treated by lumbar or sacral support or traction of vertebral column. Later when pain subsides, some physio or yoga can be done that will sustain the relief. After nerve block there may not be much pain because of same nerve. However it may come at other place.If there were uterine cysts, may be these were only accumulation of blood in some kind of bulgings which gave way to bleeding. Hard cysts do not go so soon.Being Ayurveda doctor, I suggest to my patients to take extract / decoction of leaves of Coral Jasmine plant (Night glowering Jasmine - Harsingar). It's common I'm India and South Asia. I do not know if its at your place. However you may try to get online. If needed we may help you too.Intervertebral disc problems, Sciatica, endometrial bleeding etc may be controlled effectively by it. Alternatively pain killers and hormonal styptics are available in allopathy.Hope it helps"
},
{
"id": 93044,
"tgt": "Have abdominal pain. Have bloating, pressure, swelling. CT scan normal. Shows predominantly fat attenuation mass. Take out the mass?",
"src": "Patient: I am 42 with abdominal pain. My symptoms are bloating,pressure, swelling , pain on my lower right sideI look like I'm 8 months pregnant! My stomach usually gets bloated the week of my menstrual then it goesBack to normal, but this time it has been swollen and painful for 2 days. I had a pelvic abdominal CT Scan done last month and the results were normal ( liver spleen and pancreas). It did show i have a heterogeneous but predominantly fat attenuation mass in my right adrenal gland approximately 3.3x2.9x3.8 cm My questions are Can this be one of the causes of my swollen abdomial ?Should i request this mass to be taken out? My Dr said its nothing to worry about because of the size and follow up in 4 months have another CT SCAN done to see if it has grown. Should I go to the Hospital emergency regarding the symptoms i am feeling now?? Doctor: Hi, 1) Based from the information you are having abdominal pain in lower rt side and having bloating, pressure and swelling. I would need you to check for pregnancy test you may first try urine test. If urine test comes positive the have an ultrasound to check if the pregnancy is not inside the tube and only in the uterus. Their are chances of ectopic pregnancy or pelvic inflammatory diseases that needs to be ruled out. 2) Now from the mass ,based from the information provided you may have Benign tumors that arise from the adrenal gland can be either non-functioning or hyperfunctioning (whereby normal hormones of the adrenal gland are produced in excess). In some cases, these masses produce no complications, while other masses may cause life-threatening symptoms due to the excess production of certain hormones. If possible to get it resected if your doctor thinks you do not have any risk from operative surgery just to be on safe side. Hope it helps.Dr. JAY PATEL."
},
{
"id": 112577,
"tgt": "Severe upper back pain. CT scan shows nerve damage in arm, neck with bone spurs, arthritis. Why is pain so severe?",
"src": "Patient: I have been having severe upper back pain, I went to an after hours clinic because it had gotten so bad, they referred me to a Neurologist he had a CT done and a nerve test done on my hand because it was also giving me alot of issues. He told me I have nerve damage in my arm and my neck has bone spurs and arthritis, but back back hurts worse in my shoulder blade area. He gave me shots in my back which helped for 3 weeks. He has reffered me to an Ortho Surgeon, I do not have heath insurance and they want $ 500.00 up front. I wonder why my pain is so severe? Doctor: Hi, thanks for writing to HCM.Propably, you could be having a herniated disc in the cervical spine causing compression over the nerve roots that leads to pain in the scapular region and in the arms.The general treatment guidelines for this problem is as follows -Rest: A soft cervical collar is advisable. It should not be used for more than a week or two as it may worsen the condition with prolonged usage.Medicines: Analgesic are needed in the acute phase. Muscle relaxants are essential for relieving muscle spasms once the acute pain subsides. Neurotropic vitamins like METHYLCOBALAMINE (activated Vit B12) or PREGABALIN will help in alleviating the neuropathic pain.Cervical traction: may enlarge the disc space, permitting the prolapse to subside. Intermittent cervical traction for not more than 30 minutes at a time. Weight upto 8kg can be added.Physiotherapy: Once the acute phase of pain has subsided, Isometric strengthening exercises of the paravertebral muscles are started. Moist heat can be usefulHope this information is helpful. Good day"
},
{
"id": 124628,
"tgt": "What causes the body to feel hot, dizzy and shaking?",
"src": "Patient: I was fine earlier today and then out of no where my whole head felt really hot. But my body cold. My whole body was physically shaking and I felt dizzy and wanted to throw up. My blood sugar was 108 so I know my sugar wasn t low or high. This happens every once in a very long time. Doctor: Hello, It could be due to conditions like UTI. Other causes are also be ruled out. If symptoms persist, it is better to consult a physician and get evaluated. Hope I have answered your query. Let me know if I can assist you further. Take care Regards, Dr Shinas Hussain, General & Family Physician"
},
{
"id": 131980,
"tgt": "Can Subchondral sclerosis cause tingling and pain in legs?",
"src": "Patient: I have pain in my leg bones, joints and muscle , With tingling in the calf muscle Region .Ive had ultra sound ,X-rays and Doppler done . The X-ray and ultra sound result say Subchondral sclerosis but after googling this it doesn t explain or mention the tingling or weakness at all , I don t get to see my GP until next week Doctor: hi You have pain in leg muscles bones and joints with tingling in calf muscle region.YOur X Rays and Doppler show Subchondral Sclerosis . You are right as these are not likely to cause any tingling. In my opinion you have pressure on the nerves starting from your spine or leg region. I suggest that you start pregabalin for the time being and also get S Vit B12 levels."
},
{
"id": 54230,
"tgt": "What causes high liver readings?",
"src": "Patient: HI Doc i'm male 26 years old weight is 84kgs and i recently had a high liver reading and hav since had more test to fing out why..I'm going crazy wondering wat is wrong!!..wen i had the high liver and kidney test i was sick with pnemonia but had not started medication so the meds couldnt be the problem...i hav never done drugs ive been with the same partner for 4 years and had a baby in the last 2 years and nothing was wrong with my partner...i was also sick before i got pnemonia and am constintly tired...i get 3 or four day hangover wenever i drink even if its a small amount im sick the next day!! pls any help would be much appreciated!! Doctor: Thank you for posting your query at HCM.history of increased liver enzymes. further evaluation needed.increase in the level of liver enzymes(AST,ALT) reflect an INJURY to the Liver. medically, the condition is know as \"Hepatitis\". it maybe caused by:1. Virus (A, B, C, D, E) 2. Alcohol 3. drug induced (medication)4. Gall bladder or common bile duct pathology 5.\"Fat excess\" further evaluation neededdo following test:1. Viral serology ( hepA, B and C) anti HAV, HBsAg, anti HCV2. lipid profile3. ultrasound abdomenHepA virus is a common cause and self limited disease.You may visit hepatologist and results of above tests as soon as possible.advice:abstinence from \"Alcohol and drugs\" - LOW fat diet should be followed- NO red meat- vegetables should be ingested daily- Moreover, less activity should be carried out (no sports). - use lemon juice (lemonade) once in a day- walk 30 to 40 minutes everyday.- or swimming is alternative to walking- NO sports and NO jogging-\"recheck liver enzymes after 6 to 8 weeks\".if any further questions, feel free to ask.Health professionals aim to diagnose properly and manage patients according to their limited knowledge. Cure is blessed by the ONE who Created us, whose power and knowledge is unlimited .wish you good health.regards,Dr Tayyab Malik"
},
{
"id": 98206,
"tgt": "I feel hot in my legs after having beef. IS there any special reason ?",
"src": "Patient: im 23 years old 5 6 bout 170lbs after eating beef n broccoli with some rice suddenly after i felt hot. and i can t lay down and sleep cuz i feel hot in my legs at nite it usually isn t ab din da morn but it always acts up at night Doctor: hi you have flushing reaction . it is common after ingestion of some food and drugs , you must identify the agent and try to avoid it better to consult doctor oily and spicy food ,beef etc are common agent i hope i answered your question ."
},
{
"id": 154181,
"tgt": "Can 1st stage cancer be successfully treated?",
"src": "Patient: sir...a very good friend of mine has cancer...,and the cancer was caught on 1st stage..but now her doctors are saying that it is not cureable in india..,and now shes going to new york on 4th for treatment.., all i want to know is dat is there any possible treatment to save her life..?? Doctor: Hi,Thanks for writing in.It is important to know the details of the cancer and its extent. There are many types of cancer that can occur. They can involve any structure in the body and they are classified based upon the cell that is present in the cancer. The cells are further classified as well differentiated, ,moderately differentiated and undifferentiated.When a cancer is stage one it refers to the size and that it has not spread locally and there is no distant spread. However such cancers can have poorly differentiated cancer and if not treated properly, they can go from stage 1 to stage 4 quickly.Most cancers detected at stage 1 are treatable however there are some cancers which require longer treatment so that they are under control and do not recur after treatment is complete. Please do not worry."
},
{
"id": 167864,
"tgt": "Can tuna and corn cause allergic reactions in a child?",
"src": "Patient: i just noticed blister looking bumps on my sons neck, (front and back of the neck), and a small one on both eyes, and one on his mouh. I just found out he was given tuna, at his 1 year bday party, three days ago, and baby corn caserole, he has never tried any of these things before. Could it be an allergic reaction to the tuna, and corn? Doctor: Hello,I can understand your concern. Yes, there is a chance that the child is allergic to either corn or tuna. I would advise you to visit a pediatrician for the prescription of the antihistamine to control the allergic reaction. If the ulcer in the mouth is painful, you can apply local Benzocaine gel over that such as Mucopain, 3-4 times in a day.If he develops recurrent episodes of such allergic reactions, you should go for a blood test that evaluates what substances the child is allergic to so that the exposure can be avoided in the future.I hope this information helps you. Thank you for choosing HealthcareMagic. I wish the child feels better soon.Best,Dr. Viraj Shah"
},
{
"id": 224150,
"tgt": "Long period, had unprotected sex after first depo-provera injection, back pain, cramps. Normal?",
"src": "Patient: Hello, I am close to my third depo-provera injection . I am full aware of how effective it is and the consequences. However, I am still worried. My current period has lasted way over a week (nearer to two) and I m still bleeding . It s not specifically light or heavy, it varies. I have had unprotected sex about four/five times since my first injection on the 31st May, this year. I have what feels like period pains almost all of the time and I m starting to feel sharp pains in my back and also on and around my right boob. My stomach will often have more pains if I lean down onto it, more than usual. I am unsure as to whether this is normal for this specific contraception . Thank you for your help. Doctor: hii have understood your concern.Please do not worry.I will suggest you to opt for a healthy diet and regular exercise regimeFor bleeding to stop use of Tab PAUSE NF 3 times a day will be of help.Also, use Capsule of Vitamin D 3 60,000 IU once a week for 8 weeks and daily calcium supplement will be of help.Your stomach pains and breast pains seem to be PMS - premenstrual syndrome. For that Vitamin E, B LONG F and PRIMOSA one each at night for 3 months will be of help.I hope my answer helps you.ThanksDr Purushottam"
},
{
"id": 11717,
"tgt": "Hyper pigmentation. Taking skin peel treatment, sunscreen, aziderm 20%, limcee not helpful. Cure?",
"src": "Patient: dear sir, my name is prasanna . male 28 .light brown skintone iam having hyperpigmentation problem. from the child hood. this problem lowers my confidence level when iam going to the public fuctions etc.now iam regularly taking skin peel treatment once in a month. and also doctor recommending me to use home care products such as sunscreen, aziderm20% tab.limcee . but pigmentation is frequently appearing in my face. please help me with this problem. iwant a complete cure for this problem. Doctor: Hi Prassana,To treat hyperpigmentation, it is essential to know the underlying cause for the same. Results obtained in cases of hyperpigmentation also depends upon the depth of pigmentation. Superficial light brown pigmentation responds better as compared deep slate gray pigmentation. If the above prescription is not providing desirable results, you may try stronger pigmentation reducer creams like tretinoin+steroid+hydroquinone combination creams ( skinlite, melaliteXL) or hydroquinone 4 percent creams. Use of sunscreen is mandatory. You may also take sessions of peels for better results.Hope that helped.Take care"
},
{
"id": 137480,
"tgt": "What causes sharp pain between shoulder blades?",
"src": "Patient: Hi. I woke up this morning with sharp pain I between my shoulder blades a little to the left of the center of my upper back. It only hurts when I breath in deeply. I m a 15 year old female and have had a bad back my whole life. I ve gone to the chiropractor on and off since I was 6. I ve never had this sort of back pain, only in my lower back but now it s my upper back. It feels like someone is stabbing me. Thanks Emma Doctor: Just do upper trunk rotation stretch along with pectoralis stretching.you had a coatovertebral joint irritation .if can find a physio its only matter of a single visit with ribcage mobilization."
},
{
"id": 205404,
"tgt": "What do the words ncu bug ble mean in appointment card?",
"src": "Patient: Hi, I have a friend with an mri that shows 2 sacks of fluid on top of her brain and another cyst (Ibelieve on the left side. She has frequent headaches that last for about five minutes. She was told by a very respected nuerologist to stay away from stress and was given depacote , bupap among other things. Her son then went missing for about a week and then was found drowned in a river. So much for staying away from stress. On top of that it has been over two months and the medical examiner is still holding his body. Anyway she goes back to her neurologist with a complaint about not being able to feel the top of her left foot. He then checks the pulse in her neck and cannot find one, or a very weak one. And sends her for tests. The test was something where he stuck something up to her neck and checked her pulse and even then he had trouble getting one on the left side. She said it was painful because he pud to push so hard. Now she goes back in a month for more test. The doctor wrote on her appointment card the letters NCU BUG BLE. Just wanting some thoughts on her condition and something about what she moght expect to happen from here, Thanks in advance Doctor: Hi. NCU i think might be NCV nerve conduction velocity, that is the kind of exams the doctor has made. As for more information it would be better ask the doctor who wrote that to be more specific.I hope have been helpful"
},
{
"id": 177238,
"tgt": "What causes orange spots in urine?",
"src": "Patient: I have a five month old baby. Approximately 6 x s in the last two months he has had some orange spots in his urine. My doc took a sample and said it s just crystallization and usually only happens with newborns. She s not worried, but I am still concerned b/c it keeps showing up intermittently and I do not understand why. Doctor: HI...This is a physiological which occurs in some kids. Nothing to worry if this is not bothering her in any way. This happens sometimes with crystals in the urine. But if there is a family history of renal stones, then she needs to be evaluated for that.Regards - Dr. Sumanth"
},
{
"id": 93303,
"tgt": "Abdominal pain, unable to sit for long. Done scans. What is wrong?",
"src": "Patient: My son has lower right abdoominal pain and is unable to sit up for long periods of time and has problems walking because the pressure seems to hurt. The hospital sent him home because they cannot figure out what is wrong. He had 3 ultra sounds and 2 scans and they have ruled out appendix and other common problems but he still is in pain Doctor: Hi,I assume your son has been treated at a good hospital.There are many clinical reasons for pain.I would suggest that you take services of a pediatric surgeon and get a CT scan of abdomen done if still in diagnostic dilemma.Hope this helps."
},
{
"id": 115494,
"tgt": "What causes lower level of IgA and IgM with history of fibromyalgia and breast caner?",
"src": "Patient: I have low IgA and IgM. I have fibromyalgia, osteoarthritis, and had breast cancer 7 years ago. Would any of these diagnoses have anything to the low IgA and IgM test results? Igm was less than 28.0 with normal range 60-263 and IgA 56 with normal range 68-378. Doctor: Dear friend, Greetings from HCM.... i appreciate your concern ... But i dont see any relationship between the Immunoglobulin levels and the conditions mentioned above ... i think you need not relate to them .. infact low immunoglobulin levels indicate your poor nutritional status...you take good amount of proteins like egg, fish, sprouts, take more of green leafy vegetables, citrus fruits and dry fruits, also nuts and milk ... all these gives you micro/macro-nutrients required for the production of proteins ... zinc and vitamin c tablets could be taken if necessary ... take care and do contact us for more queries .... THANK YOU"
},
{
"id": 118049,
"tgt": "What does \"presence of giant platelets and a few atypical lymphocytes\" indicates?",
"src": "Patient: I am 70 years old. I had a CBC 4 months ago with a low platelet count. It has varied up and down from a low of 113K to 139K. I have had no symptoms of bleeding problems. My last CBC, two days ago, showed a platelet count of 139 K (almost the lower limit of normal, 140 K). However, they reported the presence of Giant Platelets and a \"few\" atypical lymphocytes. My Hematologist is not available until next week. Any thoughts on what this means in an asymptomatic patient like me? Doctor: Few giant platelets and other parameters ars normal is not the worrisome. but presence of few atypical lymphocytes is definitely worrisome feature. Many it may be due to eithr reavtive lymphocytes looks like atypical. But it may be some blast like cells and not clear cut blast at that time we write it as atypical ymphocytes.so best is repeat cbc after few days.if still persist then consult hematologist.flowcytometry may help you to clear the doubt."
},
{
"id": 49649,
"tgt": "Having burning pain in kidneys, abdominal and back pain. Cure for symptoms?",
"src": "Patient: my sister is having burning pain in her kidneys, no signs of kidney stones, she would be in much more pain. she has had for about four hours. has had abdominal pain off and on for some time now. not sure of fibro. not all smptoms.has had back pain off and on, can you come up with some thing? or just a little help, please? Doctor: HIThank for asking to HCMYour sister's symptoms are very much vague of course she might be having some kind of trouble but that might not be related with kidney, we need to think something else beside the kidney, first of all the antispasmodic(Dicyclomine ) should be tried, I think the problem will go away with this even if does not go away then she need to be examined, but I think this will come around have nice day."
},
{
"id": 135799,
"tgt": "What causes painful spasms with a bulge feeling in rib cage area?",
"src": "Patient: Hi, I have been getting fairly frequent painful muscle spasms on my rib cage area. I can actually feel a bulge when this happens. The spasms seem to come up in various areas, front, back sides. Also I have a fairly large lump (golf ball size or more)on the lower left back. It is fairly hard but not bone like and is not painful.What could be causing this? Doctor: Hello,I have studied your case and I think that spasm in the body is most probably due to low salt and calcium in body. I would suggest you to check your vitamin D 3 and vitamin B12 level and see if there is any deficiency.Also whenever you are having spasm then you need to take salt full glass of water. Also take calcium daily with once a week vitamin D 3. Methylcobal injection can be taken one daily for ten days.I hope this answer will be useful for you. Let me know if there is any other followup questions.thanks"
},
{
"id": 153449,
"tgt": "Suggest treatment for cancer of ovary and lungs",
"src": "Patient: HI MY AUNT IS DIAGNOSED WITH PLEURAL EFFUSION .SHE IS 69 YRS OLD... ON EXAMINATION OF FLUID IT REVEALED PRESENCE OF METASTATIC MALIGNANT CELLS.. CHANCES ARE CANCER STARTED FROM OVARY AND TRAVELLED UPTO LEVEL OF LUNGS.. MY QUESTION IS WHAT CAN BE DONE AND HOW MUCH IS THE PROBABILITY OF SURVIVAL...WHAT TREATMENT WILL WORK BEST. Doctor: Hi,Thanks for writing in.It is important to know the findings of cancer in ovary that she is having. The CA125 level is also important to know. In patients of cancer ovary with pleural fluid showing metastatic cells indicates that the cancer has spread to other parts of the body and probably she is having stage 4 cancer.Treatment is surgical debulking any tumor in the pelvis and then giving chemotherapy in cycles. Surgery will remove any tumor that is operable and chemotherapy will stabilize the disease. Survival in stage 4 cancer is up to 2 years with good care and treatment. Please do not worry."
},
{
"id": 117058,
"tgt": "Can reduced platelet count result in acute weakness?",
"src": "Patient: I am a 33 years old woman. I suffer from acute fatigue and weakness. I got a blood test done which showed my platelet count as 1.25 lakhs. Can reduced platelet count result in acute weakness. I have no fever. Although body immunity seems to have gone down as I catch cold very frequently. Doctor: Hi, dear. I have gone through your question. I can understand your concern. You may have acuye viral infection. Platelets reduction is very common in viral infection. So no need to worry. It will become normal within few days. If you have any symptoms of bleeding then consult your doctor urgently and check your platelets count. Otherwise no treatment is required. Just drink plenty of fluid orally. Hope I have answered your question, if you have doubt then I will be happy to answer. Thanks for using health care magic. Wish you a very good health."
},
{
"id": 207127,
"tgt": "Suggest treatment for extreme behaviour due to bipolar",
"src": "Patient: hi, i suffer from bipolar and am currently feeling very low with violent outbursts and self harming. I got diazepam from my doctor but they are not keeping me calm or helping with my anxiety. I have got a meeting with my psychiatrist on thursday, but have just cut my arm again and dont know what to do. It is not deep and does not require treatment, but I am scared of myself Doctor: Start taking mood stabilisers like divalproex sodium. Please discuss with your psychiatrist about your symptoms that whether or not you have symptoms of borderline personality disorder."
},
{
"id": 69182,
"tgt": "Suggest remedy for painful lump in pubic area",
"src": "Patient: Hi, I am 32 weeks pregnant and have had significant pubic pain for the last couple of weeks that I attributed to pubic symphysis diastasis. In the past 2 days the pain has gotten significantly worse and is now excruciatingly tender to palpation on the left side. Tonight I noticed a hard lump on the left side of my mons pubis that is incredibly painful. I have not had any other systemic symptoms, vomiting, fever etc. Any ideas? Doctor: Hi.Thanks for your query.The pain in this area with a swelling on one side can be due to an abscess or symphysis diastasis as you rightly thought about.The only way to get a proper diagnosis is by clinical examination by an Orthopedic Surgeon, and/ or MRI ( MRI is safe in pregnancy).The treatment will depend upon the diagnosis.IF an abscess - needs a nice course of pregnancy-safe antibiotics and anti-inflammatory medicines.If diastsis- anti-inflammatory may be sufficient.Gynecologist's opinion about an early caeserian section can also be sought if the problem is severe."
},
{
"id": 37194,
"tgt": "Suggest treatment for Bell s palsy",
"src": "Patient: Bell s Palsy: I had bell s palsy that started about 2 months ago and took 4 weeks to recover. The right side of my face was affected. I was relieved that recovery didn t take too long. However, just last weekend the left side of my face now feels a bit tight and numb. I feel like it s a very very slight case of bell s palsy. I am able to blink my left eye and move the left side of my face (but slightly less movement now compared to the right side). Is this normal? For the other side of the face to get affect only a few weeks after recovery for one side? Doctor: BEFORE SUGGESTING ANYTHING I WOULD LIKE TO KNOW YOUR AGE AND SEX. REGARDING CONCERN OF YOUR LEFT FACE WEAKNESS ASSESSMENT CAN BE DONE ONLY BY A DOCTOR, IF IT SO,THEN YOU HAVE TO GET THROUGHLY INVESTIGATED FOR CONNECTIVE TISSUE DISODERS LIKE SARCOIDOSIS, BECAUSE RECURRENT BELLS IS NOT A NORMAL PHENOMENA"
},
{
"id": 143846,
"tgt": "Is it normal to have swelling after removing spinal stitches?",
"src": "Patient: My uncle had a spine surgery and the stitches where removed 15 days after the surgery. It s been 20 days from the day of surgery but he feels there is some swelling at the stitch place. However there is no warmness or pus coming out and no pain. Does he need any immediate attention or it is normal to have such swelling? Doctor: Hi, it is expected to have some swelling post-surgery for some time. However, if this swelling persists for more than a month it is advised you refer to your doctor. One common complication of spine surgery is the leak of CSF fluid and forming a cyst than look like a swelling."
},
{
"id": 46797,
"tgt": "Suggest treatment for kidney shrinking",
"src": "Patient: I am 50 year old and high blood pressure patient and now doctor told me that my both kidneys are shrinking and my serum creatinine level is 3.9 please suggest me is there any treatment exclude transplantion of kidney, any diet preacaution and any other medicine. Doctor: HelloThanks for choosing HCM. I can understand your concern but the final treatment is the transplantation for any CKD patient. But in your case i think as the creatinine not that much high and if you dont have any symptoms you can be on medications and diet modifications like low potassium diet and high protein diet you can maintain like this without going for dialysis or transplantation.ThanksDr.Alekhya"
},
{
"id": 40797,
"tgt": "Suggest treatment for infertility",
"src": "Patient: Hi I am just turning 33. I have an amh of 5, 10 dominant follicles, no blocked tubes or any apparent reasons for having had 2 miscarriages and a complete molar pregnancy all in 2 years: I have a 13 year old from a previous relationship. My husband has been checked and he s fine. My doctor had me on letrozole, progesterone for the end of my cycle and just prescribed prognova because lining is 7mm. We have been trying to conceive for 7 months. She has recommended ivf but we can not afford it. Is there anything else we can do or do I have any hope of conceiving naturally? Doctor: Hello and Welcome to \u2018Ask A Doctor\u2019 service.I have reviewed your query and here is my advice.I think you can conceive naturally also. Get a thyroid profile and prolactin level done. You can track your ovulation by repeated ultrasound and when follicle is more than 17 to 18 mm, take an injection for the rupture of follicles. Be in contact with your husband every 2 to 3 days after your periods stop. Take progesterone for next 2 weeks after rupture is confirmed on ultrasound. Do a urine pregnancy test after that. You can try like that for 3 to 6 cycles. If it doesn't work, you can go for IUI.Hope I have answered your query. Let me know if I can assist you further.Regards,Dr. Khushboo Priya"
},
{
"id": 60714,
"tgt": "What should be an effective diet during jaundice?",
"src": "Patient: I am a 22 years old boy suffering from jaundice for the last 6 days. I am taking complete rest. My diet includes oranges, apples, sugarcane juices, tender coconut, barley water, rice, skimmed milk and buttermilk. How long will it take to switch over to a high protein diet? When can I start my training for football? Please advise. Doctor: Your hepatitis is a virus infection and most people recover in about ten days and gradually get over it in about 4 weeks. Initially liver inflammation prevents protein metabolism and people lose their appetite. If your appetite improves you should be able to eat normally and gradually be able to get to training."
},
{
"id": 184539,
"tgt": "Suggest suitable medication for an infection in the gums",
"src": "Patient: My daughter had a loose tooth fall out and she now has noticed that she has a loose piece of gum flesh hanging from her upper gums. I originally thought that she had some food stuck between her teeth. Apparently it seems to hurt her because she s constantly playing with it with her tongue. What could this be? Would it have to snipped off by a dentist. Doctor: HiThank you for your query with HCM, as you mentioned she had a Tooth fall what you see is nothing but gingiva or gum margin which was intact with or to hols the tooth in place. Ask your daughter not to continously disturb the gum tissue tag which eventually heals by itself when left undisturbed. If problem persists or causes inconvenience consult you oral surgeon for best treatment. Till then she can do warm saline gargling and if pain is present she can also some analgesicsHope my answer would be helpful to your query."
},
{
"id": 136177,
"tgt": "Suggest remedy for painful legs and knee with swelling",
"src": "Patient: I fell on May 29 th.and landed on my knees. I am sure I did not break anything. The left knee swelled and I put ice on it and the swelling went down. it has been 2 weeks and I am now experiencing throbbing pain in both legs from the knee down. I am 75 and have not seen the doctor. Do you think this is serious. Doctor: Hello, I have studied your case.As per your age and history there is possibility of soft tissue injury in knee.Most commonly meniscus injury occurs may be associated collateral ligament injury. You may need to do MRI knee to see for any functional defect.As per your symptoms you need to do physiotherapy and knee exercises to improve range of motion.Rest and ice fomentation along with supportive brace will help to reduce locking episode and pain.Small meniscus tear can be managed conservatively with physiotherapy with USG and TENS. Large meniscus injury can be managed by arthroscopic repair.Another possibility of patellar ligament tightness which can be evaluated clinically.Hope this answers your query. If you have additional questions or follow up queries then please do not hesitate in writing to us. I will be happy to answer your queries. Wishing you good health.Take care."
},
{
"id": 188163,
"tgt": "I am a 73 year old female. My pulse rate",
"src": "Patient: I am a 73 year old female. My pulse rate usually is around 75-80. The last week it has been in the 90 range. I had extensive dental work the last two weeks and a sinus infection. I took Benedryl at night for the sinus congestion. Could that be the cause for the rise in the pulse. Doctor: Thank you for consulting with healthcaremagic. Actually which you are expecting the rise in pulse rate it is actually between the normal value only. Yes there can be rise in pulse rate because of infection. But yours comes under normal value only.So please do not take tension.Take care"
},
{
"id": 157388,
"tgt": "Will zinc help in healing after a spiradenoma cancer surgery?",
"src": "Patient: I am 70 yrs old and have a spiradenoma cancer on my head which was removed twice . The second operation is not healing as well as the first operation. I have a home care nurse who comes in every day to change the dressing. What I would like to know if taking zinc would help me heal faster. I am on warfarin. Doctor: Hi and welcome to HCM,thank you for your query.I dont htink that zinc can help in these conditioon and there is no proof that it can faciliatet healing. however it cant harm you and any proved supplements can only be beneficial. But dont expect some miracle. The most important thing for wound sis to prevent infection and have frequent dressings changes.Wish you good health. Regards."
},
{
"id": 148272,
"tgt": "What treatment do you suggest for a painless knot on the back of neck and spine ?",
"src": "Patient: My dad showed me what I would describe as a knot on the back of his neck right next to his spine. He said it is not painful but becoming annoying. It seems firm to the touch. It is also under the skin, but im not sure as to how deep. Is it possible for it to be a Limpoma even tho it is firm? Thank you Doctor: Hi, It might be lipoma. A further physical examination by the surgeon is needed to determine whether to get it surgically removed. Usually, lipoma is benign and would cause pressure to the surrounding organs (spine). A differential diagnosis from arthritis should be made too.Until you see a surgeon, I suggest to let it stay as it is without applying pressure. All the best!Dr.Alba"
},
{
"id": 32700,
"tgt": "Suggest treatment for TB in uterus",
"src": "Patient: hi, i am 37, 5 ft tall, weight is 58kg. live in mumbai. been diagnosed with TB in the uterus on basis of the test mentioned done in 2008. need to re-check. can someone tell me where in mumbai is the following test done: Identification of Mycobacterium Tuberculosis Pathogen Associated Molecular Pattern(PAMP) Doctor: Hello,In Our country for TB one programme is running called RNTCP. under this diagnosis ,treatment andmonitoring of all TB cases is done freely.You visit tertiary care govt. Hospital , get enrolled and you will get free treatment. Also test you have asked is available there known as GENEXPERT.Thank you."
},
{
"id": 114453,
"tgt": "How long does bleeding on the thumb take to cease?",
"src": "Patient: I just cut a piece of skin off the tip of my thumb. I\u2019m holding it above my heart and putting pressure on it with paper towels. What should I do when it stops bleeding? And how long should I wait before it stops bleeding? It\u2019s been a good 10 min already and still bleeding Doctor: Hi welcome to HCM. I can understand your concern.Dear,if you cut your skin, bleeding is a natural phenomenon and it is nature's blessing that bleeding stops by itself after a while and if the wound is small it gets healed on its own within few hours.If wound is little bad it needs some remedy to stop bleeding and healing the wound.If we get wounded just press your wound with cotton balls dipped in calendula tincture the bleeding will stop within minutes and heal the wound very fast,depending upon the density of the wound. Applying Turmeric powder is also very effective in healing wounds.It is an anteceptic,antioxident and antinflammatory. If the wound is serious,Turmeric Powder half spoon in a cup of hot milk helps healing fast and helps alley pain and distress.Hope this helps you solve your query.Take care.Get well soon.Don't hesitate to get back if you have any other query. Dr Suchda Consultant Homeopathy +Alternative therapies"
},
{
"id": 142723,
"tgt": "What causes pain in the mid part of the brain?",
"src": "Patient: hi i have a severe pain in mid part of my brain from last 2 years i took lot of treatments and didi cityscan also but no result now i have got fed up now i want to die otherwise plz help me to cure my headache i feel even my blood floow through this part like pulse Doctor: Thanks for your question dear your headache according to history is migraine type so you better sleep well avoid coffee or tea and avoid watch TV in dark room and you tab naproxen 250mg BD"
},
{
"id": 212110,
"tgt": "OCD, burping, obsessions of heavy trachea, suprasternal notch, breathing. On medicines. Causes?",
"src": "Patient: does burping causes suprasternal notch heaviness ?? i m 38 years old patient of obsessive compulsive disorder having obsessions of heaviness of trachea supratsrnal nothc breathimng etc on medicne prescribed by a psychiatrist. i was feeling very good since last 4 month . but 3 days back i started relapsing my symptoms of obsessions of suprasternal notch heaviness etc .i m having burping too much i woonder is there any relation of burping and suprasternal notch heaviness ?? Doctor: DearI Understand your concern.Your doubt may be true with the available description you gave.But best judge would be your ENT specialist Please consult him.For OCD, medications for OCD may not be effective without lifestyle change and rellaxation technique.Also get advice from a yoga therapist.Wish you good future"
},
{
"id": 186027,
"tgt": "What is the cause of lesion inside mouth, fever and chills?",
"src": "Patient: I have a lesion on the inside of my mouth, on the inside of my jaw at the very base of my gums, where my gums and the bottom of my mouth meet (under my tongue). It started a couple of days ago, and now my ear, cheek, jaw and neck on that side are hurting, as well as the side of my tongue. I've also started getting a low grade fever/chills. Do I wait it out? Go to the doctor? Are there any measures I can take to help this heal faster? I am a 34 y.o. female with lupus SLE that is well-controlled with Plaquenil. Doctor: helloas per ur query.direct involvement of the oral cavity by lesions of lupus in the mouth may indicate active disease. The only way to reliably distinguish whether or not an oral lesion is associated with active disease is by means of a biopsy.Oral lesions associated with active disease are usually red ulcers surrounded by a white halo and white radiating lines.Oral lesions may respond to treatment with topical or intralesional steroids. The control of active systemic disease will usually aid in the control of oral lesions, as well.hope this ans helpsregardsDr. Shesh"
},
{
"id": 6888,
"tgt": "What medicines should I take for pcod,ovulation and pregnancy and what are the best possible days to conceive ?",
"src": "Patient: pcod , overweight , irregular period duphaston taken for 15 days daily 1 tablet period come on after16 day i.e 11 july my periods were totally upset now. doctor gave atropine and voveran injection my thyriods were normal. she tested hpe and pcr ,tvs . 1 ut a/v -64x32 2 et------6mm 3lt overy cyst--37x31. no med were given for regulising my periods and for proper ovulation . pls suggest when we intercourse ? is this med suffeicent for my pcod+in ovulation+preg? Doctor: Hi Welcome to HealthCareMagic The size of the uterus, ovary, and edometrial thickness is within the normal range... The ovulation normally occurs at 14th day of normal menstrual cycle.. since today is first day, 12 to 16 days from today is good for coitus.... yes, this above medication is sufficiant....Hope i have answered your question.. TakeCare.."
},
{
"id": 182522,
"tgt": "How to treat canker like cyst and tooth pain after extraction?",
"src": "Patient: I recently had a tooth pulled ( 7 days ) and there is continuous pain around that area, i went to the dentist and he said my body was rejecting the fact that i got my tooth pulled and said that canker like cysts were forming. He told me that they would go away on thier own but i woke up this morning and they are spreading from the back to the front of my upper gum line....any advice? is this more serious than he is making it out to be? Doctor: After tooth extraction there is some chances of developing complication like Dry socket actually it is a unhealed bony socket of your extracted tooth which occurs due to dislodgement of the blood clot from the site of extraction.you can go to another dentist and get zinc oxide eugenol dressing of the unhealed socket and change it after every 24 hrs for 5-6 days you can see healing starts after 2 days and pain also get relieved in the mean time you can take analgecis tablets containing diclofenac potassium 50mg with serratopaptidase."
},
{
"id": 143573,
"tgt": "Suggest treatment for bulged discs and degeneration of the spine",
"src": "Patient: i have got 3 bluged discs and also degenaration of the spine cant seem to get a doctor to understand my pain and i have call pain clinits and they tell me must be refered by a doctor and also drs i have been to keep telling me to have phyisical therpy but i did once and it didnt help at all just made it worst Doctor: Hi, bulging discs and degeneration of spine causing severe pain requires treatment. For this ,I would like to see your MRI films to see cord and nerve compression.If it is causing severe compression it requires surgery otherwise can be managed conservatively.Medical treatment includes medications and physiotherapy and alternative pain modalities. Thanks"
},
{
"id": 39922,
"tgt": "Pale and numb white lips a symptom of shingles?",
"src": "Patient: above upper lip is pale and white and upper lip feels numb and tingly. I have had some red swollen patches with lots of burning on the upper lip goes through stages of burning, swelling, peeling, painful and then clears up for a little while and then starts over again. This has been going on since March of this year. One dermatologist says it is shingles, another says not and they don't know what it is. Had the area frozen but still is going through all of this same stuff. Doctor: Hello dear,To have clear cut idea,we have to see the lesions directly.But you can decide whether it is shingles or not,from the following.1)Shingles appears years after you had a chickenpox.2)Tingling,numbness,and pain precedes development of the lesion.3)The lesion appear as group of fluid filled vesicles.4)It will be present on one side only(right/left).It will not cross the mid line of our body.5)recurrent attacks are rare,unless we have immune compromising conditions like uncontrolled diabetes/HIV/connective tissue disorders/other chronic illnesses.It can usually be treated with antivirals like acyclovir,analgesics,and vitamin supplements.If this is not the case,you pleas follow your dermatologist.I hope you will find this information useful.Thank you."
},
{
"id": 99110,
"tgt": "Could nausea be due to laryngitis?",
"src": "Patient: My girlfriend and I are worried that she is pregnant. About 2 weeks ago and i was only in her for a second. I know i didnt cum in her. But recently she has been feeling really nauseous. She went to the doctor and he diagnosed her with laryngitis. But the doctor did not know about this instance. So i'm curious if she could be pregnant or if it is just a symptom for the Laryngitis. Doctor: Your question is very specific...Nausea is a non specific symptom of many underlying physiological(like pregnancy) or pathological condition.Usually laryngitis only does not cause nausea .Laryngitis may coexist with other condition causing nausea.Thanking you."
},
{
"id": 97074,
"tgt": "Is recovery possible following difficulty in brain function after drowning?",
"src": "Patient: my husband drown in a cold river in oregon a year ago. he was under water for two hours, which was documented by his watch and the emergency room doctors. he survived but is now having issues with brain function, eye movement, and memory. he is on oxygen because of later pneumonia. does he have a chance of recovery? the doctors say they have no way of knowing if any of this is permanent and are amazed that he is alive. Doctor: Hi and welcome to HCM Thanks for the query. it is always hard to rpedict recovery. brain is still mystery but if there was certain period of hipoxy then brain damage may be too significant for normal functioning. Wish you good health. Regards"
},
{
"id": 120610,
"tgt": "Suggest remedy for swollen & sore legs",
"src": "Patient: My son went to the dr a few days ago and had a xray because his leg was swollen and sore to the touch. They couldn t find anything wrong. It has gotten better, but now the left side of his head is sore to the touch and he won t let me touch it. Even putting his shirt on hurt it? What could this be. He will be 4 at the end of December. Doctor: Hello, The migration soreness may be due to following reasons- -Presence of infection -Presence of connective tissue inflammation due to tissue disorders. -Deficiency of vitamin D. You may ask from your doctor for a complete blood examination including CBC and vitamin D profile. At present you may ask for prescription of good anlgesic and anti inflammatory preparation for relief. Hope I have answered your query. Let me know if I can assist you further. Take care Regards, Dr. Mukesh Tiwari"
},
{
"id": 160887,
"tgt": "How long will eczema in the whole body take to heal completely?",
"src": "Patient: hi, my child is 3yrs old and she is suffering from eczema for the past 2 months.first it started at the back of the knees as thick scales and gradually spreads over her arms,elbows,stomach,back,upper& lower legs as small coin like patches with itching esp at the backof the knees.undergoing homeo treatment.treated with calc sulph,lyco.seems like itz better in some parts and again spreads in some other parts. now is bieng treated with sulphur for the past two days.how long will it take for my child to cure completely?kindly advice.thanx Doctor: HiIt could take several weeks for skin to return to normal since the eczema is more widely present all over the body.Skin disease takes more time to recover since the epidermal turnover or shedding time is 30 to 40 days.I suggest you to wait for 3 weeks for the skin to clear eczema.Hope that was helpful.Let me know if i can assist you further.RegardsDr.Saranya RamadossGeneral and Family Health Physician"
},
{
"id": 81618,
"tgt": "Chest X-ray: Infiltrations, small nodular lesions and minimal fibrosis",
"src": "Patient: My niece chest x ray report says------ Infiltrations, small nodular lesions and minimal fibrosis seen at lt.upper and mid zones and rt. upper zone. findings are suggestive of active kochs lesions. Sir can you tell me what does this report indicate and whether the disease is contagious. The patient is 34 yrs old and has a 3 year old son. Doctor: Hello dear, thanks for your question on HCM. The x rray description, you have provided, is suggestive of active, infectious tuberculosis.Tuberculosis commonly affect upper lobes.Lesions like infiltration, nodules and less fibrosis favours active lesion.So get done sputum for AFB as soon as possible and start the treatment according to weight. Pulmonary tuberculosis is infectious disease. I spreads by droplet infection. Due to coughing, sneezing and deep breathing these droplets are generated and discharged in atmosphere. From her they can Infect normal healthy individual. So ask her to wear mask and avoid coughing in public places. Her son will need Isoniazid prophylaxis to prevent infection."
},
{
"id": 131465,
"tgt": "What does the dislocation of the subcortical fracture mean?",
"src": "Patient: Over 3 months ago I dislocated my kneecap. I got an MRI and it showed the dislocation gave me a subcortical fracture, my groove is almost straight, and my cartlige isn t damaged. My knee has made cracking sounds and some of them, it hurt. What could this mean and what is causing it to hurt? Doctor: subcortical fracture is fracture of tibial plateau , it is not serious but it can cause complications like O.A or meniscal tear I recommend rest ( need to see x ray to determine how long ) followed by Physical therapyGood Luck"
},
{
"id": 41956,
"tgt": "What is the prognosis for the follicular study attached?",
"src": "Patient: I have undergone follicular study, i got the report. i want to know my report is normal. on 15th day end.thickness is 10 and right ovary is 16.60 * 9.85cm and in left 1) 17.8*15.8cm,2) 17*13.2cm, 3) 18.8*9.65cm. pls tell me is it everything normal.on 16th day it got ruptured and given comments as Fluid in POD+ Doctor: Hi thanks for writing in health care magic.I have gone through your question.As your reports showing follicles size should be in mm (millimeter).!!Endometrial thickness 10 mm is veey good for implantation. Free fluid in pod (pouch of douglas) suggest follicles has ruptured.On 16th day follicles can rupture.Your reports are absolutely normal.Have a good health..."
},
{
"id": 117181,
"tgt": "Is it to be concerned about the cbc blood test results?",
"src": "Patient: Just got the results of my cbc blood. My Eosinophil was 13. I have lupus and Disconnective Tissue disease and am taking 400MGS of Plaquinel for it. I see a rhymatoligist regularly. He has no told me about all the blood work i recently have had. He runs a full metobolic panel on me every 3 months. My family Dr. gave me the results. I also have recently found out that i have fully torn rotory cuff and need surgery. I am a Male 57 yrs old. Thank you Doctor: Hi, dear. I have gone through your question. I can understand your concern. Your have connective tissue disease and lupus so its common to have slightly high eosinophil count. Its no need to worry. For your rotator cuff injury you should go for surgery according to your doctor's advice. Your cbc has no any relation with that. You can go with surgery with eosinophil count 13. No need to worry about that. Hope I have answered your question, if you have doubt then I will be happy to answer. Thanks for using health care magic. Wish you a very good health."
},
{
"id": 107553,
"tgt": "Suggest treatment for back pain and heavy menstrual flow",
"src": "Patient: Hi doctor , I m 52 yrs old and having backpain and on the sides of pelvis i still get my periods and quiet heavy but tolerable, only for 3 days due to which i am anaemic 10.3 haemoglobin and . now i have fever 99 degrees c for past 4 days accompanied withspasmodic backpain and pain in sides of pelvis.This pain is on and off from august 13.USG was done 4 times since then and it revealed bulky uterus with size 98x43 x62 and a small anterior wall fibroid 24x20 x16mm endometrial thickness 9.3mm ovaries are normal in size and echotecture. my menses are due today but i did not get still, but i have back pain sides of pelvis , 99degrees fever and slight nausea. i took 3 tablets of meftal spas since yesterday and is getting temporary relief.i also take rabeprazol 20 in the morning and levogastrol for my acidity and tenormin 50 mg for BP and escitolopram for anxiety . please advice Doctor: hi,i can understand your feelings,at age of 52,probably u are in perimenopausal stage.back pain with heavy bleeding made u anaemic.i have a suspician of pelvic inflamatory desease.due to fiboids in uterus wall,uterus is bulky it seems and also leading to spasmodic pain.i have seen patients getting benefitted by taking antispasmodic medicines during this stage.along with medications,my pts used warm pads on abdomen andback 2-3 times in a day.i hope and wish you will also be benefitted.thanks.u are wellcome for any further quaries."
},
{
"id": 149952,
"tgt": "Had vagus nerve schwanomma removed. Having paralyzed vocal cord. Developed Frey's syndrome. Sweating in the side of head. Serious?",
"src": "Patient: Hi! Long story short...I had a vagus nerve schwanomma removed in June. Now I have a paralyzed vocal cord, I still have trouble swallowing but my voice is stronger. This past week I developed Frey s Syndrome and the side of my head sweats. The past two days my left side of my forehead , ear, above the eye, temple area is twitching. It feels foggy and like crawly skin. Is this nerves coming back? It just doesn t feel right and I want to make sure it s not a sign of something serious? Doctor: Hi,A clear description of your query; thanks!Yes, the twitching on the forehead, ear, above the eyes, temple area is a sign of recovery and improvement.Be rest assured that it is not a sign of something serious. Wait for a couple of days more. You would definitely improve.Regards,"
},
{
"id": 140266,
"tgt": "What causes a warm body with no actual fever?",
"src": "Patient: The last two nights my son has woken up with his body being very warm but when i have taken his temperature it has only been 36 degrees , still worried i gave him some panadal , and within 1/2 an hr to an hr his body has cooled down . He still drinking and eating and has been great when he wake up later in the morning . Not sure if i should be worried or he is one of these people who have warm bodies ? Doctor: Hi, Is there any chance that the thermometer could be miscalibrated or broken if it's electronic or something similar? You could try another thermometer. But if the temperature is accurate and you are sure you are taking his temperature correctly I wouldn't give him any medication just because he feels \"warm to the touch.\" There are variables at play that can mislead you into thinking somewhere is febrile or hyperthermic when in fact, they are normal. There's nothing wrong with encouraging lots of fluids (especially water) but I wouldn't give medications to lower temperature if it were my son. Using another example which I believe everyone is even more familiar with would be the person who feels cold or whose hands are clearly colder than anybody around them....yet they do not have lower body temperature on this basis. The only thing you'd typically recommend to someone like that is if they feel cold, put a sweater or blanket on, right? There's no medicine that's going to correct that situation, right? Unless of course, there is something hormonal going on but then, again there'd likely be other symptoms and clinical signs of a problem. Hope I have answered your query. Let me know if I can assist you further. Regards, Dr. Saddiq Ulabidin, General & Family Physician"
},
{
"id": 188823,
"tgt": "What is the remedy for soreness on the palate ?",
"src": "Patient: I have been treated by my dentist for 4 weeks now for an extremely sore palate . She tells me taht she sees my palatal rugae are inflamed but that there is no sign of specific ulceration/ broken skin but has treated me unsuccessfully for bacterial/ then fungal and finally steroid lozenges to no avail!! Nothing is working. She has also cured my gingival bleeding condition and instilled me with better oral hygiene measures - using interdental brushes and corsodyl but my palate is still agony. She thinks it might eb an allergy but what do you think. Its soooo painful!!! Doctor: Hello,Thanks for writing to us.Soreness on palate can be due to-1. denture stomatitis caused due to dentures or appliances inserted.2.abscess developed due to spread of infection into periapical area through deep caries or periodontal infection.I would suggest you to get a thorough clinical evaluation done.Discontinue use of appliances or dentures causing allergy.Pus if present has to be drained and curetted.Take care."
},
{
"id": 34251,
"tgt": "Suggest treatment for mycobacterium avium-intracellulare infection",
"src": "Patient: I was diagnosed with mycobacterium avium-Intracellulare a couple of weeks ago. My treatment involves three medications Clarithromycin, Ethambutol and Rifabutin every day for a year. Whai are my changes of beating this disease? I am 67 year old adult and I have started to lose weight, however will I continue to lose weight as I get treatment7? Doctor: Hi, I can understand your concern about loosing weight and the infection. MAI is treated in most cases by the above mentioned combination of drugs you have been taking. It will take some time. Eventually when you start feeling better you will eat more and will start gaining weight. I would meanwhile advice you to eat healthy food every day including lots of fruits including bananas for complex carbohydrates and green leafy vegetables and 2-3 eggs per day to increase protein uptake. since you need lot of protein to build up bodies' immunity.I hope this helps. Wish you a healthy recoveryPlease let me know if you have any more questions for me"
},
{
"id": 63689,
"tgt": "How can a shin lump with pain on palpation be treated?",
"src": "Patient: i have a lump just below my shin where i got tackled in football its been there for 4 weeks now and is the same size and shape there is no real pain there unless i press on it ,. 2 years ago me and my mate smashed shins together playing football and im wondering if it has anything to do with that as when that happened my leg swelled up and was black with a big bruise Doctor: Hi,Dear,Thanks for the query to HCM. I studied your problem in depth and I understood your concerns.Treatment of shin bump-I dont find any reason to associate current painful lump to 2 yrs back trauma,as it would healed by now.For the lump below shin 4 wks ago-I would advise you to Use anti-inflammatory drugs / cold compresses to resolve the lump /and take ER Ortho- Surgeon's consultation with CAT scan Local Voltaren gel(diclofen gel) would resolve the boney bumpThrombophobe ointment locally to resolve the fibrosing hematoma and to reduce any hematoma on left side.This advise is based on the facts from the history you give and needs further clinical check.So don't build up wrong concepts and create more psychic complications in you which would increase risks and costs to you, but just ask a query to HCM and be comfortable to resolve your health issues.Welcome for any more query in this regard to HCM.Write good resume and Click thanks if you feel satisfied with my advise.Have a Good Day.Dr.Savaskar M.N."
},
{
"id": 59779,
"tgt": "Done liver function test. Under strict diet control. Matter of concern?",
"src": "Patient: Hi Doctor, I did my liver function tests on 14th Oct and my SGOT was 87 IU/l and SGPT was 120 IU/L and I repeated the test again on 27th Oct 2012 and the report showed SGOT is 95 IU/L and SGPT was 131 IU/L. During this time I am under strict diet control and avoided all kinds of oily food. Is it a matter of concern and what is that I am required to do to reduce it. Regards Kishore Doctor: Hi Kishore, I hope that you have followed up with a gastroenterologist / hepatologist for further evaluation. If you are overweight then you should lose weight to get rid of any fat in the liver. Avoid alcohol completely. Make sure you have been tested for the most common (and uncommon) liver diseases such as viral hepatitis, autoimmune liver disease, wilson's disease, hemochromatosis, alpha 1 antytripsin deficiency, etc. If your doctor is unsure about the reason for your abnormal liver enzymes you may require a liver biopsy. Also make sure you are not taking any medications that could damage your liver. Check with your doctor. Hope that helps. Let me know if you have any further comments or questions."
},
{
"id": 183360,
"tgt": "Can taking Metronidazole and Dicloxacillin for tooth pain cause hives?",
"src": "Patient: Hi, I have dental problem like Wisdom teeth. i am getting pain. I met doc today she given me antiboitics Metrinidazole and Dicloxocillin.. one year back i used some antibiotics for cough and pain..After using them i got Hives(chronic allergy). Now I am afraid to use Metrinidazole and Dicloxocillin and in doubt of getting Hives again after using these antibiotics..Can you please advise me.. Doctor: hi. we understand your problem. some people may have increased sensitivity or adverse reaction to a particular drug. If its the same drug which you consumed during your cough episode then yes it might again give you the same allergic reaction. the best thing is to carry the prescription card which you were given earlier and show it to your present dentist so that he/she can make necessary modification in your medications"
},
{
"id": 15959,
"tgt": "Suffering from raised red rashes all over body, diagnosed as pityriasis rosea. Causes and treatment?",
"src": "Patient: I have a raised, very red rash all over my body. It started on my chest , and on the area here, you can no longer see the individual spots. My GP initially thought it might be pityriasis rosea , but changed her mind when I went back; she seems baffled as to what it could be. I therefore went to a walk in clinic which prescribed antibiotics , but I have seen no results. What do you think this could be and how long do antibiotics take to work? Doctor: Hello dear. Thanks for the query. pityriasis rosea is thought to ba a viral infection. it usually follows a viral sore throat or viral fever. It heals on its own , For faster relief topical mild steroids like desonude can be used. Wishing you good health. Thank you"
},
{
"id": 144343,
"tgt": "Suggest treatment for mild diffuse slowing, broad field generalized epileptiform",
"src": "Patient: my son who is 4and half yrs old had an abnormal eeg 1.mild diffuse slowing 2.broad-field generalized epileptiform During sleep and drowsiness there is high voltage as much as 400-500 microvolts with triphasic morphology with anterior and posterior phase gradient durations up to 1 sec. he has been asked to take keppra 3ml twice a day Doctor: hi,thanks for the brief history. since you have mentioned all things and a neurologist opinion and medication will be of help. also along with it if the physical therapy is advised he will of help. as physical therapy will help improve the metabolism and coordination in the body.with the same of God I wish you a good Health"
},
{
"id": 4238,
"tgt": "Will althea prevent from getting pregnant?",
"src": "Patient: Hi, my bf and i had sex on the 3rd of oct, im supposed to have my period on oct 17th. And now its the 24th, 7 days late and i still don t have my period yet. We used protection and withdrawal. Is it possible im pregnant? I started taking althea 3days ago. Doctor: hello,As per your due date, you did sex close to your ovulation period and withdrawal method has high failure rate (15 to 27 %).As your period is 7 days late, you should undergo one home pregnancy test to rule out pregnancy. Althea pill is not helpful at this stage. Kindly, consult with your local doctor."
},
{
"id": 135071,
"tgt": "Suggest remedy for persistent pain in knees",
"src": "Patient: Hi, My Son has pain right below his knees. We have taken him to the doctors and they have said it is growing pain and he should only stretch. The pain become sever as he plays basketball and just recently after he jumped up during the game the pain became unbearable to the point that he had to come out of the game. He has not been able to play ever since. Is there anything we can do to either reduce or mask the pain st least during the games. Doctor: Hello there,I have read your question, The site of pain you are mentioning and doctors diagnosis is totally different. Dr has told you Groin pain, which means Adductor muscles have been injured but according to your complaints (pain below his knee) my diagnosis is totally different.He might be suffering from Jumpers Knee.Jumpers knee or patellar tendonitis is pain in the tendon which attaches the kneecap or patella to the top of the shin bone or tibia. It is usually an overuse injury caused by repetitive strain.Patellar tendonitis can be a tricky condition to treat and requires a substantial period of rest and a thorough treatment and rehabilitation program. In the most severe or persistent cases, surgery may be required. Treatment for patella tendonitis consists of reducing the initial pain and inflammation through rest and ice followed by a rehabilitation program which will include eccentric strengthening exercises.Home treatment- Apply cold therapy on a regular basis, especially during the acute stage which is usually the first 24 to 48 hours and after any form of exercise.Professional treatment-anti-inflammatory medication e.g. ibuprofen. Do not take ibuprofen if you have asthma and check with a doctor before taking any medication. A patella tendon taping technique can be used to relieve pain and take the pressure off the tendon in the same way a patella strap works.A professional therapist may use electrotherapy in the form of ultrasound or laser treatment to reduce pain and inflammation and aid the healing process.Hope this answers your question. If you have additional questions or follow up questions then please do not hesitate in writing to us. I will be happy to answer your questions.Wishing you Speedy recovery."
},
{
"id": 20920,
"tgt": "Could the chest pains be the sighn of heart attack?",
"src": "Patient: My boyfriend of 10 months is havin throbbin chest pains. Before I met him he was 200 n something lbs.he lost the weight but now he's gainin it all back.also,he was born with a heart murmur and worst he's a smoker.my question is could the chest pains be signs of a heart attack? Doctor: Hello, thanks for posting a question. I understand your concern towards your boyfriend's health. Chest pain alone is not a symptom of heart attack. Heart attack comes along with other symptoms like shortness of breath, palpitations, nausea etc. I advise he sees a physician for check up because of his history of heart murmur, been overweight and smoking habit. Stay healthy"
},
{
"id": 200011,
"tgt": "Suggest treatment for testicle pain and jelly like semen",
"src": "Patient: hello dr , i got my semen analysis report today..can you please check and let me know if there are any infertility issues with that report?. I have undergone bilateral varicocele surgery 4 years before. I am facing some pain in and around testis and also in the joint connecting abdomen and legs and i am facing this pain especially after every ejaculation.. also my semen is becoming into more jelly one containing more liquid than usual. so i did semen analysis today and got my report as in in attachment. please let me know Doctor: Thanks for asking in healthcaremagic forum I do understand your concern, but I cannot access your report with your question here. Varicocele can cause dragging pain in the testicles and varicocele is know to have recurrence. Regarding your semen, the consistency and colour may vary with infection. So, please send your detailed report of semen analysis for further suggestions. All the best."
},
{
"id": 110369,
"tgt": "What is the remedy for the back pain due to an injury?",
"src": "Patient: I got kicked in my upper back accidentally by a kid with special needs and it really hurt. There wasn t any bruising but it was red in the spot where I got kicked. It hurt a bit when ever I bent over but it went away. It s been a year but my back still aches and I don t know what to do about it. What should I do? Doctor: Hi.Welcome to healthcare magic. BY GOING through your query it seems that it is not related to old injury. It is simple backache. treatment of simple backache is back exercises and analgesics (diclofenac) . you need vit D3 supplement if it is deficient."
},
{
"id": 89872,
"tgt": "What causes sharp abdominal pain?",
"src": "Patient: I am having really sharp pain in my lower abdomen on both side. I used to have ovarian cysts when I was younger but was put on birth control. I have been off of birth control for almost a year now? The pain is so sharp that it is even making me sweat. What could it be? Doctor: HI,thanks for using healthcare magicIt is possible that the pain is related to the ovarian cysts. Ovarian cysts can cause pain as a result of different possible causes : (1)inflammation (2)infection of the cyst (3)bleedingSince the pain is so severe, it would be best to see your doctor for an evaluation to determine the cause. This would involve physical examination and an ultrasound.I hope this helps"
},
{
"id": 149120,
"tgt": "MRI with T2 white matter hyper-intensities cerebral hemispheres. Episode of vomiting with confusion, blurred vision, fatigue. Stroke or MS?",
"src": "Patient: Thank you for your help ! My husband had an MRI that states he has scattered T2 white matter hyperintensities in both cerebral hemispheres. He has been treated since 5-26-12 for physical & heat exhaustion due to working 18 us hrs. A day three days in that wk. we were in a heat wave also. He got so sick and vomiting with confusion, blurred vision , severe headache. Fatigue , memory loss and pain and weakness more on the left side. An employee had to drive him back to the main office two evenings in a row who h is totally out of character for him. He was the foreman and never let anyone drive his work truck. He has had pain blocks in neck and knees thinking his stumbling was due to other issues. Two different drs. Told him it sounded like he had a stroke so we go to a neurologist . This is the diagnosis of the MRI with contrast. For several months the symptoms have continued now he has to use a cane to help him be more stable. Please help us understand could be have had a stroke or is it more probable to have MS ? He seems to have all the symptoms of both & he is only 46 yrs. old. The drs. Tell him his bisy appears to be much older than he is. His hair is falling out badly & he has had knee replacement 4 knee surgeries & 2 lower back surgeries . We are so confused at thus point !!! Doctor: Hi,Thank you for posting your query.Based on the information provided, the most likely diagnosis is brain stroke. Neurological deficits in stroke do take time to recover, and it is not unusual to find weakness persisting for a few months. Regarding a possibility of multiple sclerosis (MS), it is unlikely, as there seems to be a single episode of neurological illness. In MS, there would be multiple episodes of neurological disturbances.I hope it helps. Please get back if you require any additional information.Wishing you good health,Dr Sudhir Kumar MD (Internal Medicine), DM (Neurology)Senior Consultant NeurologistApollo Hospitals, Hyderabad, IndiaClick on this link to ask me a DIRECT QUERY: http://bit.ly/Dr-Sudhir-kumarMy BLOG: http://bestneurodoctor.blogspot.in"
},
{
"id": 4060,
"tgt": "Will i get pregnant naturally after having a D&C?",
"src": "Patient: am merried on2010 sep.still now i dint get a child.before 4 months i got pregnant.during 45th day scanning it was blighted sac.before 3 month it was terminated through D&C.after D&C we tried for two months to conceive.but still now i dint get any good news.whether i have to wait for natural pregnancy or have to begin treatment as infertility.please send any solution for my problem, Doctor: Hi, thanks for using HCM..I understand your concern.. Since you had natural pregnancy once,u can try for a normal method for 6 months before taking infertility treatment.. what's your age? If its less than 35 you can wait for 6 months otherwise You can start infertility treatment. Did u get periods after D and C? Is it regular? Sometimes excessice curretage can cause asherman's syndrome which can cause amennorrhoea.. Normally a routine D and C doenot cause infertility. Is ur husband's semen analysis report normal? If you are too anxious for getting preg u can take ovulation induction drug after consulting your gynaecologist and follow up with follicular study to know the exact time of ovulation for a timely intercourse.. Hope i have answered your queries. Good day."
},
{
"id": 220431,
"tgt": "Will Venlafaxine cause harm to the baby in the womb?",
"src": "Patient: hi, i'm female 28 years old... i'm on venlafaxine its been almost a year knnow.. and i just found out i'm pregnant i must be one month along and i'm so scared that this pills will hurt my baby i don't know what to do pls help me can i stay on venlafaxine i have bad anxiety or do i have to stop pls help me tks Doctor: Hello,I have gone through your query and understood the concern. Venlafaxine has been shown to be associated with lung complications in the new born when used in pregnancy. However, stoppage of this medication also can lead to relapse of depression. Therefore, you should discuss with your neurophysician as to the appropriate management to be undertaken. Do not start or stop this medication on your own. Please use folate supplements and take adequate rest. Hope you find this information useful. Take care."
},
{
"id": 155139,
"tgt": "Suggest treatment for testicular cancer",
"src": "Patient: (Testicular Cancer Problem) Background Male Asian(Chinese and Cambodian)-American age 19~20 Problem: I just recently self examined myself for testicular cancer, but I can not currently tell exactly what I am feeling. Both my left and right testicles have no problems as in: no bumps on them, and no other weird feeling;however there is a distinct difference of the epididymis(i think ) between the left and right one. My left testicle seems to be covered/overlapped(closer towards the body side) by a rather large amount of material(the material is squishy and the amount is approximately the same volume as the testicle). My right testicle does not have such large material, but I do feel it having the epididymis. I also do not have/feel any of the symptoms from testicular cancer. Question: Should I immediately see my doctor? or Should I ask my doctor whenever I see him/her again? Doctor: You probably have what is called a varicocele. This is nothing but a dilation of the veins of the testicle. This is not cancerous and does not lead to cancer.Lie down on the bed and hold your left testicle slightly raised. After a while, the swelling will disappear. This is diagnostic for varicocele. It is not urgent but when you meet your doctor, get an ultrasound of the scrotal area and of the whole abdomen done. Sometimes, varicocele is due to a kidney tumor which one must rule out."
},
{
"id": 14511,
"tgt": "What causes itchy rashes when body over heats?",
"src": "Patient: Concerned our teenage daughter started this summer with when getting overheated she develope a rash. Now instead of a rash it is itchy welts the size of a breath mint. It only occurs when she does a physical activity or from taking hot shower. Our doctor prescribed singulair this summer which didn't work so we discontinued, now she is trying zafirlukast 20mg. She has only been taking it for one week, but I see no difference. To be quite honest, the only thing that actually works is products like benedryl (which sometimes make her drowsy). I did give her a generic form of antihistamine yesterday which worked well, without the drowsiness. I would really like to know what this is and how long it will last as I really do not want her on meds. Doctor: Hello. Thank you for writing to us at healthcaremagicThis condition is known as cholinergic urticaria.Cholinergic Urticaria classically presents with itchy, welts which are small and uniform in size, distributed all over the body. The reaction is intense and sudden but short lasting.The reaction commonly develops in response to certain stimuli which raise core body temperature e.g exercise/ physical activity, hot shower, spicy food Or in response to strong emotional reactions like anger etc.It is a type of physical urticaria and is usually chronic and recurrent.Apart from avoiding activities that trigger this type of urticaria, treatment is symptomatic with Oral antihistamines e.g cetrizine or loratadine. A single dose once daily is usually enough.Topically she may use a soothing lotion e.g calamine, during an episode of cholinergic urticaria.Regards"
},
{
"id": 131333,
"tgt": "What causes swollen ankle , numbness , cramping , vertigo and livedo reticularis?",
"src": "Patient: I ve had symptoms off and on for last 2 years , I passed out twice and the Dr never went into much cause I was pregnant but recently I have livedo reticularus , Raynaud s , fatique, swollen ankle and trapazoid , crazy tingling sensations, numbness, cramping feeling, vertigo, petichia on abdomen and flu like feeling. My Dr took some blood work and as far as I know the only positive were ra titer was high, aso was high,. I m 35 And am very active. This is really worrying me. My Dr is sending me to a rheumatology but they can t see me for close to 2 months. Is this k to leave your body in this state that long before treatment? Doctor: hiif rheumatoid factor is positive,your treating doctor may ask for serum anti ccp test to confirm rheumatoid arthritis as it is more specific. if it turns positively high you could be put on DAMRD anti rheumatoid drugs like hydroxychloroquin,methotrexate and or on cortisone too for relief.you could follow up rheumtologist later.Treating doctor has to screen liver and kidney function tests before begining anti rheumatoid or disease modifying drugs.etanercept injections can also be considered"
},
{
"id": 83223,
"tgt": "Is it OK to take Myteka tablets during pregnancy?",
"src": "Patient: hello Doctor im 35 year old pregnant for 11 weeks, im having allergy problem since i was a child n just before my pregnancy my doctor suggested Myteka tablets (montelukast sodium) i want to consult is it ok if i take Myteka tablets during my pregnancy not on regular basis only when i feel irritation n restless .please reply me asa possible im really misrable now a days .thanks Doctor: Hi,Montelukast belongs to FDA category B and relatively safe to be used in pregnancy. I would suggest you to take the drug only when you have symptoms and avoid using the drug continuously for more than 10 days. Avoid the allergen which could be food or dust or fabric if possible. Mind relaxation techniques and breathing exercises can improve your lung performance.Take care. Hope I have answered your question. Let me know if I can assist you further. Regards, Dr. Saranya Ramadoss, General & Family Physician"
},
{
"id": 45739,
"tgt": "What to do for the shivering of head,hand and other body parts?",
"src": "Patient: My mother has undergone RIRS for kidney stone on 24th march,then ESWL on 28th march, is taking one antibiotic,one pain killer and one b complex supplement pill since then, urine reports are normal but since 31 st march she has developed shivering of head,hand and other body parts,.. what to do now? Doctor: Hi, Shivering may not be related to stones, it could be due to infection somewhere in the body, such as urinary tract infection. You can continue antibiotics for symptomatic relief and in case of persistent symptoms, better to consult a physician and get evaluated. Hope I have answered your query. Let me know if I can assist you further."
},
{
"id": 80313,
"tgt": "Does coughing with mucus & milky white tissue like substance a sign of having bronchitis?",
"src": "Patient: I had been having a very high body temperature, felt exhausted, felt pain in my chest and been tired for a four to five days. Now my temperature is almost normal, I feel no pain, but I've lost my voice (I can only whisper and hardly speak) and my bronchi are making some weird sounds. I have started to cough a lot (still nothing hurts) and when I cough, three times I cought out some white milky body tissue. Can those be parts of my vocal chords? And is this influenza that turned into bronchitis? Please answer, I'm so afraid that I won't be able to speak normally anymore. Doctor: Thanks for your question on HCM. I can understand your situation and problem. No, whitish thing is not your vocal cord. So no need to worry about it. This thing is sputum (expectoration). And your history and description suggest possibility of post viral bronchitis. So better to consult pulmonologist and get done 1. Clinical examination of respiratory system. 2. PFT (Pulmonary Function Test). PFT is needed to rule out bronchitis. It will also tell you about severity of the disease. And treatment of bronchitis is based on severity only. So get done PFT. You may need inhaled bronchodilators and antihistamines drugs. Don't worry, you will be alright."
},
{
"id": 154468,
"tgt": "Suggest naturopathy treatment for rectum cancer",
"src": "Patient: Good Mornins Sir, This is ranga rao calling from tenali, a.p. india, sir, my wife is suffering with rectum cancer since 2009, intially we used homeo medicine but there is no resutly after that we went in alopathi inthe month of march, 2011 thedoctar did radition along with chemo tablets, tumor was diluted 90% now,she got fisher the tool is going on throug vegina since 8th july, 2011. the doctor has told it is bad. sir, is there any treatment in naturopathy. pl. Doctor: Hi,Thanks for writing in.Your wife seems to be having a surgical complication arising out of radiation to the rectal cancer. The rectal tissue might have become friable due to radiation and chemotherapy and this caused a false passage between the rectum and vagina.Unfortunately there is no cure in naturopathy for the above treatment. It is strongly advised that you consult an oncosurgeon preferably specialized in gynaecology and surgical oncology for surgical correction of the problem. This problem cannot be treated with medicine and further waiting can cause more complications and discomfort.Infections can also happen due to the communication between vagina and the rectum and this can cause significant problems in future."
},
{
"id": 218810,
"tgt": "Are headaches and stomachache signs of an unwanted pregnancy post protected intercourse?",
"src": "Patient: I had sex for the first time today with a condom, it did not break and he did not come inside me or near my vagina, I have a bad headache and stomach ache now. Since it was the first time I m freaking out about pregnancy, is there a chance I could be pregnant Doctor: chances of conceiving are negligible as intercourse was not proper. psychological u are worried that may b causing headache."
},
{
"id": 96390,
"tgt": "My brother getting pain in abdomen at hypochondriac region, please help",
"src": "Patient: Dear Sir, My brother getting pain in abdomen at hypochondriac region ann it is decresed by bending forward and pain radiating to back. by this symptoms I concluded it as pancreatitis and now he is taking Creon tablets tid, and esmeprozole bd, now appetitie is increased, pain also releaved but for how many days I need to continue these medication and what is the prognostic examintions. Plese let me know. Thanking you sir Yours truly, Dr. R. Naresh Kumar Doctor: Hi.. Depending on the chronicity of the condition the type of treatment and the duration of the treatment varies. Ingeneral an acute pancreatitis help within a week unlike chronic pancreatitis who need long therapies.. Hence depending on the tests and scans doctor will be able to decide the further course of action..."
},
{
"id": 103563,
"tgt": "Have breathing difficulties. Suggested mild asthma. Could this be a reason for breathlessness?",
"src": "Patient: HI.I've been have breathing difficulties for about 2 to 3 days already.When I have breathing difficulties, I wasn't doing anything intense at the moment.What might have caused this?I have been to the doctor few months ago and he did say I have mild asthma, which I think might also be known as Exercise Induced Asthma.Could this have contributed to the breathing difficulties I have been experiencing this few days? Doctor: YES YOUR MILD ASTHMA MUST HAVE DEVELOPED IN BREATHING DIFFICULITIESYOU MUST TEAT YOUR ASTHMA ACTUALLY THESE ARE ALLERGIES AND THEY INCREASE GET ALLERGY TESTS THROUGH BLOOD SERUM OR PRICK TEST TO FIND THE CAUSE AND TREAT PERMANENTLYAT PRESENT YOU NEED TO TAKE ONTELUCAST 10 MG BDSYP ASTHALIN WHENEVER YOU GET TAKE MED FOR 2-3 DAYS"
},
{
"id": 184150,
"tgt": "Could taking avlicor be safe for brown patch on corner of lips?",
"src": "Patient: I have a brownpatch on the corner of my lips, the local med clinic thought it was cold sores, prescrbd me 4 pills, avlicor and sentme on my way. Well its been there for 3months now, i went back two and ahlof months aftermy first vivst and am being told it might be fungus now, they told me to put lotramin on it. what is it what can i do?? I am a 26year old male, 5ft8in at165 pounds Doctor: hello...reading through your query i may say that it can be fungal or it can be due to vitamin deficiency...so better takemultivitamin tablets for 1-2 weeks...and apply topical antifungal agents....keep ur oral hygiene at good level too...and more over reduce stress or tension...be calm...and happy...have a nice day!!!"
},
{
"id": 66211,
"tgt": "What could cause the lump behind the right ear?",
"src": "Patient: I am 33years old and female. I have a lump behind my right ear that I discovered yesterday. It is flesh colored and feels hard and bony. It is the size of a dime right on my mastoid bone. I have not been feeling I'll. Am wondering if my glasses could be rubbing on the bone causing inflammation or if it is more serious. Doctor: Hi! welcome to HCM!Well, if I were your family physician for this complaint of a flesh colored and feels hard and bony lump behind the right ear, I would just think of some benign conditions and convince you not to worry at all!However, possibilities are : 1. neurofibroma/dermatofibroma2. enlarged lymph node due to chronic infection in the scalp, dandruff, ear, nose, sinus infections etc3. benign osteomaOnly a microscopic examination and clinical assessment can confirm what is it therefore please don't worry and see your doctor for relief or revert back to us if necessary!wishing you good health!w"
},
{
"id": 190388,
"tgt": "Severe tooth ache, eases with cold water in mouth, difficulty in sleeping. Remedy to alleviate the pain?",
"src": "Patient: i have a very bad tooth ache of which only soothes and pain goes away while i swish cold water in my mouth. I have bad teeth and the tooth that i bothering my does have a hole on the top very near the gum line. I have tried everything and water seems to be the only way to sooth the pain... however i need to sleep and i cant swish water while i sleep and i am a mouth breather so when i open my mouth to breath the air it starts hurting again. anything i can do so get me to sleep without the pain? Male 41 years old overwieght smoker and take heart meds to slow rate down (propafenone, metoprolol , bayer) Doctor: Hello and thank you for your question. Your tooth has a large cavity which has infected the nerve, the nerve is inflamed and swollen. The cold relieves the pain because it causes the pressure to decrease for a minute. The damage to the nerve is irreversible and the tooth if restorable needs to have the decay removed, a root canal performed and a crown or large restoration placed in the tooth. Or if the tooth is non-restorable, the tooth needs to be pulled before you get a large infect in your jaw. Best regards Dr. Ward"
},
{
"id": 178845,
"tgt": "What causes froth coming from mouth?",
"src": "Patient: Hi My four years old girl suffers allergies occasionally including skin and nasal allergies . Nowadays she has allergy on her face and is on Zyrtec and Avil . I have just noticed that while sleeping she had froth coming out from her lips.. I saw it earlier as well about six months ago but then I thought it is bacause of possibility of pin worms and I got her de wormed. Does this foam / saliva thing has to do anything with allergy or it is something else?? Doctor: Yes its related to allergy. She is suffering from allergic rhinitis which is causing her nasal obstruction particularly at night. As her nose is getting blocked she is compelled to breath through mouth cauing the frothing. Use Otrivin mini nasal drop before sleep at night. These will relieve the symptoms. You can also try Duonase nasal spray as second option."
},
{
"id": 198908,
"tgt": "Need treatment for night fall",
"src": "Patient: hello sir, i have night fall problem from 10 years....i have used medicines like himalaya confido,charak neo,baidyanath chandrapraba vati but no benefits.......lease suggest me best medicine for frequent night fall....my body is very weak ....soetime night fall occur without dreams..lease suggest medicine. Doctor: DearWe understand your concernsI went through your details. Please understand, night fall (nocturnal emission) cannot make your body weak. Your body produces semen whenever you are sexually excited. Produced semen is stored within your body and is a waste product. That waste product is ejaculated when you indulge in sexual activities or when you masturbate. If you do not do this, body finds its own way to throw the waste product out. Body does it through nocturnal emission or through urine. This cannot affect your health. To prevent nocturnal emissions, please start masturbating or indulge in sex twice a week. I also suggest you to consult a psychologist for counseling.If you require more of my help in this aspect, please use this URL. http://goo.gl/aYW2pR. Make sure that you include every minute details possible. Hope this answers your query. Available for further clarifications.Good luck."
},
{
"id": 71429,
"tgt": "What causes tiredness and body pains after taking treatment for chest infection?",
"src": "Patient: hi i suffer with aritas and the medication im on is omeprozole motilium and have codeine and tylex for pain relief 7 mths ago i got a servere chest infection had 3 lots antibiotics and steriods since then i have been ill i have pain thoughout my body and have no energy i no longer leave the house 2 weeks ago the doctor gave me prozac im still in lots of pain and feel so ill i just dont know how to gt help anymore from doctor as it seems all they give me is pain killers can you help please Doctor: Hello,It is not that the treatment causes these symptoms it is the infection itself causing it don't worry.Hope I have answered your query. Let me know if I can assist you further.Regards, Dr. Jnikolla"
},
{
"id": 210704,
"tgt": "Could the extreme fatigue, depression and lethargy caused due to the overtaking of Methylprednisolone?",
"src": "Patient: I think my wife is accidentally overtaking Methylprednisolone (medrol). Her prescript is for 1 \u00bd 4 mg. tabs per day. I think she s been taking 6 tabs per day. She is extremely fatigued, depressed, and just has zero energy and is lethargic. Could this be from overtaking the methylprednisolone ????? Doctor: HiThanks for using healthcare magicCorticosteroids also lead to depression. In her case, if she has been taking them for last many years, depression could be due to this. Better to consult a physician for decrease the dose of it. Decreasing corticosteroid would help to decrease the underline depression.Thanks"
},
{
"id": 32091,
"tgt": "What precautions should be taken after wound from rusted metal?",
"src": "Patient: earlier this afternoon i accidently cut myself on a rusty piece of metal. i went to the bathroom and cleaned it up with soap and water and then got a bandaid for it. it is a small puncture wound and from the outside it looks clean. i am up to date on my tetanus shot and i am just wondering what i should look out for. thanks for the input. Doctor: Hi Dear,Welcome to HCM.Understanding your concern. As per your query you got a cut from piece of rusty metal wound which could cause puncture wound and rusted metal injury, as your immunization is up to date you need not to worry about. Tetanus is usually taken once in 5 years. You should go for booster dose of immunization if injury is there. You should visit general physician / surgeon once and get it examined. Surgeon will examine your wound completely and start treatment as per requirement. Maintain area clean and dry. Cover area to prevent any debris exposure. Hope your concern has been resolved.Get Well Soon.Best Wishes,Dr. Harry Maheshwari"
},
{
"id": 220009,
"tgt": "How to terminate pregnancy?",
"src": "Patient: i was having two cesarian delivery . one was eight year back and another two year back . But , second time my uterus got reupture during pain and doctor had done immediate surgery to save the life . Now i am pregnant (5 weeks) for the third time and dont want to continue this pregnancy. Kindly suggest what to do and any risk attached to my life Doctor: you should consult and visit a gynecologist who is expert in terminating the pregnancy. the doctor can help you by doing a transvaginal sonogram and then decide what method of termiation will be safer."
},
{
"id": 6430,
"tgt": "What treatment should I take to conceive ?",
"src": "Patient: I have irregular period.. It ranges from 60 to 80 days... Today my period came..I have to become pregnant ... What treatment I have to take... I done ultra sonic scan follicles seen. No pcod.. What to do doctor .. Pls reply me and tell the way I have to become pregnant.... Doctor: Hi, Welcome to HCM. Since you have irregular periods its difficult to manage on your own if you are wanting to conceive within few months. it is important for you to follow a obstetrician so that she can follow you closely with your dates and guide you and help you to conceive soon. Have a good health. Take care."
},
{
"id": 71191,
"tgt": "What does this CT scan report indicate?",
"src": "Patient: My wife underwent tracheostomy in August due to Respiratory failure. Currently she is on 1 litre O2 support. A CT scan and virtual bronchoscopy has been done recently which states s/o infection. Doctor has advised Wysolone. The tracheostomy site has developed GRANULOMA causing 70 % airway blockage as per her Discharge Card issued in the month of Augusf. Please advice further treatment. Doctor: Hello and Welcome to 'Ask A Doctor' service. I have reviewed your query and here is my advice. Chronic tracheostomy is often lead to infection and granulation at the site of tube. So she needs to remove all those granulation tissue through surgery.She will need new tracheostomy tube insertion. Proper tracheostomy care is must to prevent granulation tissue formation and infection. Hope I have answered your query. Let me know if I can assist you further."
},
{
"id": 15137,
"tgt": "Uneven skin tone. Taken treatment no improvement instead face burn. Is there any treatment without skin bleaching?",
"src": "Patient: Hi I am having trouble with uneven skin tone . I have try over the counter products and nothing working I went to a doctor who help with African American skin tone . Everything they give me make my face burn or sting really BAD. with no improvement at all please help or recommend something that might help without skin bleaching.' Doctor: Hi,It seems that you are not satisfied with your facial skin condition.It may be dry, pigmented ans wrinkled. The glow of skin may be poor.You may not be happy for your skin.Improve diet,have healthy food,take more green leery vegetables,take fruits, drink sufficient water. Avoid junk food.Take proper sleep,avoid worries and tension,have positive attitude.Avoid excessive sun exposure.Apply alovera and vitamin E containing cream in morning and vitamin C with kojic acid containg cream at night daily.Take orally vitamin C,nicotinic acid and glutathion.Cleanse the face with glycolic acid and kojic acid containing face wash.Your skin may be improved.I hope you got my answer.Thanks.Dr. Ilyas Patel Mzd"
},
{
"id": 58610,
"tgt": "Diagnosed by primary complex by TB specialist. Prescribed R-Cine. Side effects?",
"src": "Patient: My 7 years old son diagnosed by primary complex by TB specialist He prescribed R-cine with some other medicine and 5mg Wysolone, two tablet per day, he is taking Last 5 months, doctor said 6th moth it will be tapered and treatment will be end. Every month he is testing his blood test and liver test. I afraid of whysolone shall I continue or I can request to taper the tablet , what will be side effect of whysolone, I read about side effect in the web side I really worried please advice me . Thanks Sugumar Doctor: if your son is not showing any side effects of steroids that is gain in weight, facial puffiness, increased respiratory infections then better to wait but in case steroids stopped also, if he develops any kind of infections u need to mention with doctor that he has taken steroids for such a problem."
},
{
"id": 198712,
"tgt": "How to treat epidermal cyst and e.coil in urine?",
"src": "Patient: epidymal cyst and e.coil in urinehi doctor,i have e.coil in my urine culture report and also epidymal cyst in my left testicle..my symptoms are pain in penis tip(suddenly occurs),vomiting sensation ,clear sticky discharge but no colour it comes when i squeez my penis tip,sudden back pain and elbow pain is this is any std syptoms if which one is it or it just an uti plz help me i am confused with doctor Doctor: HelloThanks for query.Based on the facts that you have posted I would state that you have UTI due to E-Coli organisms and Epididymal Cyst on left side .Please take antibiotics as per culture and sensitivity report along with urine alkalizer twice daily .Ensure to drink more water to keep urine dilute .This will help to reduce dysuria.As regards Epididymal cyst it is a benign cyst and does not need any treatment .You may take pain killers like Brufen as and when necessary.Dr.Patil."
},
{
"id": 155910,
"tgt": "Suggest solution for health conditions in cancer",
"src": "Patient: My aunt has signed a DNR her blood pressure is 90/50 and her pulse ox is 85. She has cancer. What would be your best educated guess of time we have left with her. she is 60 and is bleeding from somewhere, but the drs cant seem to tell us from where. they were giving her units of blood every few days. Doctor: Hi, dear. I have gone through your question. I can understand your concern. She is suffering from cancer and bleed like a open tap. Every day a unit of blood is required. Her pulse and bp is low. So overall she has very poor prognosis. No one can sqy exact time she left. But she don't have much time that we can say from the report. Hope I have answered your question, if you have doubt then I will be happy to answer. Thanks for using health care magic. Wish you a very good health."
},
{
"id": 197527,
"tgt": "Suggest treatment for itching in the scrotum",
"src": "Patient: Hi, hope you are well. I am experiencing an itchy sensation on scrotum, and since I itched a bit too much I can ripped off a bit of the skin, so can I apply Canesten?Age 22, height 5'7, weight 68 kgs, no previous records as such nor haivng diabetes or any other sickness or cancer Doctor: HICanestan will help but it sounds like you have a fungal skin infection and you need prescription ketoconazole cream to resolve the problem"
},
{
"id": 155070,
"tgt": "Could a blister on the bottom of foot be a symptom of cancer or tumor?",
"src": "Patient: Hello. I am a 14 year old boy who knows nothing about cancer but I believe I might have it. A couple of months ago I noticed a lump on the bottom of my right foot. It looked a lot like a blister so I leaved it alone to see if it would maybe go away. But after a few weeks I didn t go away so I poked a whole in it with a small clean needle to find out it I fact was a blister. So a month or 2 later it came back, but it was bigger and had a scab or something on the top, it got bigger in size and looked like a blister again. I took a Little bit of the scab off but it felt like I was taking off my real skin and started to bleed. It hasn t got much bigger yet but still is growing a little. it is about as big as a dime right now. So is it cancer or tumor or something. I haven t noticed anything strange happen yet! Please help me. Doctor: HiI would advise you to do a biopsy test from your blister.Mri of foot also needs to be done.You can apply soframycin ointment locally over it.RegardsDr de"
},
{
"id": 151037,
"tgt": "70 year old, has loss of memory, ends up bed wetting. Sign of dementia?",
"src": "Patient: My father in law, 70 yr old, very active and fit person residing in India. since a week of so he has been losing memory of day to day activities. just realised that he is bed wetting . we have admitted to the hospital. his blood pressure is normal. Still waiting for neurologist to give us diagnosis. Medical conditions: Cholesterol & diabetes is under control with medication from the last health check that was done few months ago. I hope this is not a sign of dementia . Could you kindly advise what other conditions could cause the same. Doctor: Thanks lot for posting your questions in this web site. The problem in elderly individual is generally difficult to handle and a given medical problem can have mutiple causes. Regarding your father, if you are very sure that the problem has started very recently, a medical condition called as delirium needs to be ruled out. This a medical condition which may present similar to your father, provided that the onset of illness is of short duration. As the causes of delirium are so many, your father would require blood tests to rule out any infections, altered liver funtions, kidney functions, CT scan or MRI scan of brain to rule of any tumor/stroke which may explain his problem. The second part of the story is that this illness might have started earlier in a less severe form and might have worsened now. If this is true, a possibility of dementia is high on the card. My advise will be to get the blood tests, liver and kidney function tests, serum sodium, thyroid function including MRI Brain. If all these tests are inconlusive, he may be subjecetd to a detailed neuropsycological evaluation to see if he has dementia or not and we can manage accordingly after establishing the diagnosis. The third part of the story in these types of cases is that a single patient can have both and if delirium is treated properly, he gets back to the previous state. Hope this will help you If your father is in India, they can contact me Best wishes"
},
{
"id": 143876,
"tgt": "What causes atrophy of temporal lobes of the brain?",
"src": "Patient: My husband has atrophy of temporal lobes of his brain. He has in the past been diagnosed with Alzheimer s. Now the are not sure of diagnosis. They are also talking about hydrocephalus. Wonder if you can give info on diagnosis and how it relates to Alzheimer s Doctor: alzhimers disease is a type of degenerative dementia. Predominantly medial temporal lobe and parietal lobes are affected. This may explain MRI findings in this case.. however if not Alzhimers....then....other possibilities to be ruled out ...EG....Mesial Temporal lobr sclerosis."
},
{
"id": 203487,
"tgt": "How can i get rid from masturbation?",
"src": "Patient: dear sir i continuously masturbated from last 15 years, so my semen have thin and now when i masturbated i feel pain in my lest testis with pain feel on left side heap(thigh). kindly solve my problem. how i increased sperm, how i get rid from masturbation. thanking you Doctor: Hello,,Masturbation is normal after puberty. It is one type of sexual function. But if person has to masturbate frequently and/or causing any troubles then it is a matter of concern. Frequent masturbation is reflection of deep seated tension, stress, or high sexual desire. Homeopathy is very much helpful in this type of cases.Homeopathic treatment is based on total understanding of patient on physical and mental level. And that's why detail history of patient is needed for proper homeopathic treatment. Your problem can be cured permanently with homeopathy.So consult to good classical homeopath nearby you or contact us.Have a better health."
},
{
"id": 208602,
"tgt": "Suggest treatment for stammering while speaking to unknown person",
"src": "Patient: when situations get extremely formal,i start stammerring...when i fear i start stammering...i always set my mind that i wont stammer,ill be confident when i speak to an unknown person or a person holding a lot of importance...but most of the times i stay fumbling over words unable to make a fluent speech. can you suggest me something by which i can get rid of this problem? Doctor: Hi..Thanks for the query. Do you fear the possible scrutiny by people in authority in formal situations? Do you become too much cautious about how you look, dress, talk etc. in these formal situations? If answers to these questions are yes then you are suffering from social anxiety disorder or social phobia. That is your real problem is NOT stammering but excessive fears of unwanted scrutiny by strangers, superiors etc. in formal situations. The treatment for social anxiety consist of behaviour modifications, cognitive behaviour therapy, role play, guided imagery techniques which are taught by psychologist & anxiolytic drugs like etizolam, clonazepam in combination with SSRI group of drugs. You can consult a Psychiatrist for both set of treatments & I am sure you will do very well in these formal situations! Good luck"
},
{
"id": 15889,
"tgt": "Itchy dark rashes on the leg spreading. reason?",
"src": "Patient: I have a itchy rash on the outside of my upper right leg. It came last summer - went away after summer. And is now back after I have been in the sun and is spreading! They are faint dark patches with some streaking. It looks like a laid a curling iron down on a piece of paper really fast. Please help! Cortisone cream seems to help a tad bit but not much. Doctor: Hello, Welcome to health care magic It looks like you have a fungal infection from your description. We call it Tinea. Apply an antifungal cream like ketoconazole 2% twice a day over the patches. Even after the rash subsides continue applying for another week so that it won't recur again. Do not apply cortisone which is a steroid. Even though it temporarily decreases itching, it allows infection to become extensive. Hope this helped Take care"
},
{
"id": 224501,
"tgt": "Does copper T cause weight gain?",
"src": "Patient: Hello Doctor,My wife having age of 27, saying that he has gain weight due to copper t. And felling trash and wants to remove it. Is it true because of copper t there are chances of weight gain ? Her weight increases from 52 to 60 even through no changes in a lifestyle within a period of 6 months and of course after wearing of copper t. We have 1 child and need no more.What next step that we can do ? Doctor: hi thanks for writing to uscopper t responsible for weight gain is a pure misconceptionits action is purely local and not systemicweight gain can be due to various factors such as physical inactivity,depression,thyroid disorders and manykindly get evaluatedhope this information was useful"
},
{
"id": 170776,
"tgt": "Suggest diet plans for a 5 month baby who doesn t drink milk",
"src": "Patient: HI, I HAVE A FIVE MONTH OLD BABY SHE STARTED REFUSING BOTH BREST MILK N FORMULA SINCE A MONTH.... I CONDULTED DOC BU NO HELP. SHE DOESN CRY FOR MILK BU KEEPS ON SUCKING HER THUMB.. SHE TAKES MILK WHEN SHE IZ ASLEEP N THEN ALSO IN DEEP SLEEP SHE STOPS.. I FEEL SHE DOESN GET PROPER NUTRITION PLEASE SUGGEST????? Doctor: Hi.....At this age according to WHO only exclusive breast feeding is indicated. But as you say the baby is completely refusing milk of any sort, you can start giving Cerelac stage one rice or Nestum rice stage 1 etc. All are same in calories and energy. Only thing is palatability which is subjective for each kid. Other home-made food options will be - 1. Daal and rice well cooked and made into a porridge along with ghee2. Boiled apple or banana and mashed into a paste.3. Any made cereal porridge preparation with additional ghee added.Remember certain principles like -1. Add one food material / cereal or fruit per week. This will be useful because if he develops vomiting or diarrhoea, we will know what to avoid.2. Add vegetables and fruits made into a paste surely to avoid constipation.3. Do not add too much sugar as it may cause diarrhoea.Hope my answer was helpful for you. I am happy to help any time. Further clarifications and consultations on Health care magic are welcome. If you do not have any clarifications, you can close the discussion and rate the answer. Wish your kid good health.Dr. Sumanth MBBS., DCH., DNB (Paed).,"
},
{
"id": 219731,
"tgt": "Can foreplay cause pregnancy?",
"src": "Patient: Hi, may I answer your health queries right now ? Please type your quemry here...my girlfriend was giving me a hand job and I finished it and may of got some semen on my hand. About 10mins later I touched the outside of her vagina just at the entrance of the hole. I m worried of her getting pregnant and I m really stressing over this ! Should I be worried Doctor: Hello,Thanks for writing to us.In above situation, her chance of pregnancy is RARE or NIL if you just touched outside the vagina and NOT inserted inside vagina.You should know that HIGH chance of Pregnancy depends on unprotected sex with ejaculation inside vagina during fertile period or ovulation day.Practice safe sex in future. Maintain genital hygiene. Good luck"
},
{
"id": 25428,
"tgt": "How to treat breathing problem due to side effect of medicine?",
"src": "Patient: sir my mom is heart pasint.she her angioplasty in mey 2011. afer angioplasty his health is well but 2 month leter his problam in heart beating bcoz she got a shock in his sister death. i have been check up in doctor s. mohenty DM . he got the medenine but . but heart medenine got a side effect. she have been problam Breathing. so what i do sir Doctor: HIWell come to HCMI really appreciate your concern, if this is cardiac condition and angioplasty being done then patient has to take the medicine, cardiac medicine may not have any serious side effects it has got the negligible side effect that can be tolerated without any trouble, what ever the symptoms your mother having that could be due to some stress, depression, so no need to worry about this, hope this information helps."
},
{
"id": 126442,
"tgt": "Can Hyaluronic acid be taken when diagnosed with stomach ulcers?",
"src": "Patient: I had a bleeding stomach ulcer last year that was cauterised. A second endocopsy showed scarring but no further ulcers. I used to take liquid Hyaluronic Acid for joint pain and stopped whilst waiting for ulcer to heal. Now take 10mg Omezropole daily. Can I start total the oral Hyaluronic Acid again or would it have a negative effect on my stomach? Doctor: Hi, You can safely take Hyaluronic Acid, it won\u2019t worsen your stomach related problems. Hope I have answered your query. Let me know if I can assist you further. Regards, Dr. Shinas Hussain, General & Family Physician"
},
{
"id": 188088,
"tgt": "How to heal swollen and sore inferior labial frenulum?",
"src": "Patient: I cut my inferior labial frenum. This happend about 3 days ago i was wondering if i needed to go to the dentist or what i should do to help it heal it not a very bigg cut but the inferiir labial frenum is sore and swollen also a little red around the cut area Doctor: Hello,I would advice you to visit a dentist and get a thorough clinical evaluation done.Apply topical steroid on the affected area.Vitamin becosules as well as iron supplements has to be administered.Avoid spicy,acidic foods.Tobacco or pan chewing has to be discontinued.Maintain oral hygiene well.Take care."
},
{
"id": 33039,
"tgt": "When can alcohol be consumed after recovering from enteric fever?",
"src": "Patient: Hi doctor..i was suffering from enteric fever few days back..it has been 2 weeks since my 15 day antibiotic treatment is over and 2 and 1/2 weeks since i had fever..i wanted to know from when can i start consuming alcohol...i am not a heavy drinker..i drink it over weekends moderately Doctor: Dear , it is strongly suggested that you refrain from alcohol consumption. Take to yoga in it's place during your free time"
},
{
"id": 189268,
"tgt": "Severe pain on right side of face, numb, pain in right ear, had nerve removed from tooth. Cause of pain?",
"src": "Patient: Hi iv had a nerve removed from a tooth and since then iv been in extrem pain on the right side of my face it's numb and I get shooting pain in my right ear and it hurts down my right side of the throat I find it hard to swallow.. I'm on anti biotics for four days and it's worse at night. What is going on and what do I do.. No one can seem to tell me what's going on and I can not get rid of the pain ??? Doctor: Dear friend. Thanks for sharing your concern. your history suggests that you have got root canal treatment done.which is still incomplete. please give me complete history,regarding your root canal. your present pain is due to infected canal and infected pulp tissues there, causing pain. It can be relived by pain killers temporarily, but you need to see your dentist soon ,so that infected pulp tissues can be thoroughly removed ,canal can be prepared and then obturated (filled) aseptically. Meanwhile continue with your antibiotics and pain killers. Do antiseptic mouth rinsing. Hope it helps. Thanks. Take care."
},
{
"id": 38709,
"tgt": "Suggest treatment for uvulitis",
"src": "Patient: hi,i have Uvulitis what can i do for it.....this is the second time in a little more then a year. i have severe allergies to just about everything from trees and grass to foods. i have been talking allergy shots. do you think this could have something to do with it . also can predisone help with this..... Doctor: Hello,Welcome to HCM,As you are having some allergic manifestations for more than a year, you should be very careful with the symptoms as it may be dangerous to the life. I would suggest her to undergo absolute eosinophil count (AEC) which will be raised in the allergic condition.For the present symptoms you can give her Tab Montek LC which helps to control the symptoms, but it is temporary. She need to avoid the allergens to protect herself from getting these allergy symptoms.I would suggest her to undergo skin prick test (SPT) to identify the type of allergen causing these symptoms and can prevent herself from exposing to these allergens. Otherwise she can be given immunotherapy either with subcutaneous immunotherapy (SCIT) or sublingual immunotherapy (SLIT).Thank you."
},
{
"id": 37361,
"tgt": "Can vaccinated dog bite cause any infection or rabies?",
"src": "Patient: my son got bitten by our dog who is already fully vaccinated against rabies. i took my son to hospital and they gave him a tetanus as well an anti rabies vaccine but after that i decided not to continue with the course. will that have any side effects on my son? Doctor: Hello,Welcome to HCM,As your son was bit by a vaccinated pet dog which has led to wound, dog is known to transmit rabies,which is 100 % fatal but it is 100% preventable by proper and adequate treatment.As there was no bleeding from the site, according WHO categorization it is categorized into Cat II, Which requires following treatment.1. Wash the wound thoroughly with soap and water to washout the rabies virus if it is present in and around the wound.2.Active immunization with anti rabies vaccine on days 0,3,7,14 and 28.I would suggest you to give atleast three shots of anti rabies vaccine on days 0,3 and 7.3.Inj TT was given4.Topical application of T Bact ointment.The aforementioned measures will help to prevent development of rabies. Thank you."
},
{
"id": 18642,
"tgt": "What makes a pacemaker ineffective?",
"src": "Patient: Why would my dads pace maker go off after he just 3 months ago had a triple bypass and was in hospital till Feb 14 th had his surgery on December 22 just this week went back to work but is only working half days and just got off and undressed then it hit him on way to sit down Doctor: Hello and Welcome to \u2018Ask A Doctor\u2019 service. I have reviewed your query and here is my advice. I would like to tell you that pacemaker doesn't work in high pacing threshold setting which is very uncommon. Pacing threshold can increase on its own if there is an electrolyte imbalance, hypoglycemia, hypothyroidism or acid-base disorder has developed in the body. To avoid these complications your father needs to be very regular with meals, diet, medicines, blood test and personal physician consultations. He will also require pacemaker calibrations on very six months or as per his physician decides. Hopefully this information will answer your query."
},
{
"id": 187539,
"tgt": "What is the cause for intense pain in my lower gum?",
"src": "Patient: About 5 or 6 years ago when I was in middle school, I had to have a root canal on one of my bottom teeth. since then I have gotten braces put on and taken off my teeth. I still have a wire on the back of my bottom teeth. I have never had an actual problem with my root canal. Recently i'm starting to feel soreness and at sometimes intense pain starting from where my lower lip and lower gum meet to the starting of the top of my chin. I hurts to put pressure on it. When i put pressure on it , it sends a pain sensation through, I guess, My nerves to one of my teeth. I was thinking it might be because the wire on the back of my lower teeth had came off but i'm starting to think otherwise. Also i thought about this new found pain to be related to my root canal i had 5 or 6 years ago, but the strange thing id that the pain sensation is going to another tooth instead of the one I had a root canal on. It also hurts when I make facial expression that involve my muscles uncle my lower lip. (example: when going to kiss). To my knowledge I have not hit this area that is sore to make it hurt. Doctor: Hello, Welcome Thanks for consulting HCM, I have gone through your query, as you have said that you have undergone root canal treatment 5 years ago and Orthodontic treatment also now you are feeling pain dont worry it can be due to Orthodontic treatment may be due to treatment oral hygiene is not maintained properly thats why pocket formation is there any you feel pain and senstivity , You should go to dentist and go for investigations IOPA done to get proper diagnosis and treatment , Hope this will help you. Wish you good health."
},
{
"id": 109317,
"tgt": "What is the back pain after standing on the injured ankle?",
"src": "Patient: i twisted my ankle two nights ago. it is slightly swollen and not very painful, but today was my first long day back at work since the accident-standing all day. i have horrific lower back pain on my right side, the same side as my ankle injury. are they related? any advice on relieving pain Doctor: Hello, Thanks for writing to us, I have studied your case with diligence.Yes ankle injury and back pain can be related.Instability may increase due to tear and laxity of ligament.I will advise to repeat MRI ankle for soft tissue involvement.Till time you can continue use supportive splint or brace.Physiotherapy like ultrasound and TENS will help in early healing.If age is more you can manage it conservatively with brace and physio.Complication can be ankle arthritis ,profound instability ,recurrent fall ,inability to walk.Hope this answers your query. If you have additional questions or follow up queries then please do not hesitate in writing to us. I will be happy to answer your queries. Wishing you good health.Take care."
},
{
"id": 20206,
"tgt": "Suggest treatment to control blood pressure",
"src": "Patient: hi,my blood pressure on three separate occasions on two different days is around (i.e +/- 2mmHg) 140/110.I'm 27 yrs old and obese,height 6.2,Family history of B.p on paternal side,I;ve recently started jogging(no fast running) for 45 minutes daily in order to lose weight,diet is maintained healthy(balanced carbs proteins fats),also recently i've reduced smoking cigarettes to 1/day,previously i used to smoke 10/day.ineed to know what medications i should use?please answer. Doctor: Hi, Thanks for posting in HCM. I understand your concern. we should never start the BP medication right after first reading your diastolic BP 110 its abnormal as per your age. still we can wait for 1 week if you don't have symptoms like headache ..all day heavy head feeling. we should start medication..like telma 20 mg for this we have to see your creatinine levels.if you are not an asthama person we can start on stamlo 5 mg and these are prescribed drugs ...with out the test done we cant start these medication other wise the diet plan which you had started and your regular exercise might decrease the B.P with out any medicationHope the information provided would be helpful. All the best. Regards,Dr.sameena"
},
{
"id": 174859,
"tgt": "How safe is atrax drops for cough in infants?",
"src": "Patient: hi m from mohali. my 2 week old baby boy is colic n had cough. a doctor prescribed atrax drops along with colic aid. after giving him 4 drops of atrax as suggested by the doctor he is sleeping constantly n not taking feeds properly. is it safe to give him atrax 4 drops three times a day? Doctor: Atarax (hydroxyzine) is a good antiallergic and helps in relieving cough in children. It is safe but the main problem it can cause is excessive sleepiness. Although it is safe to give atarax thrice daily, in such a case, you might hold the dose and get the child examined by a pediatrician so that he would be able to assess if the child is otherwise fine. Colicaid is safe too and can be used."
},
{
"id": 175848,
"tgt": "Suggest treatment for persistent loose motion",
"src": "Patient: My grandson is suffering from toothing and has loose motion one doctor has suggested ofm suspension n there is different opinion that it is not to be given my grand is 1year7 month old loose motion is not in control kindly pls advice that the above medicine can be given to him Doctor: Hi...loose motions form toothing come because the kids of this age keep everything in the mouth and this is cause for viral diarrhea. Stop OFM - it is an antibiotic.It seems your kid is having viral diarrhoea. Once it starts it will take 5-7 days to completely get better. Unless the kid's having low urine output or very dull or excessively sleepy or blood in motion or green bilious vomiting...you need not worry. There is no need to use antibiotics like OFM unless there is blood in the motion. Antibiotics might worsen if unnecessarily used causing antibiotic associated diarrhoea.I suggest you use zinc supplements (Z&D drops 1ml once daily for 14 days) & ORS (Each small packet mixed in 200ml of potable water and keep giving sip by sip) as hydration is very important and crucial part of treatment."
},
{
"id": 216014,
"tgt": "What causes severe foot pain without injury?",
"src": "Patient: Hurt my foot playing baseball. Had an xray, it didn t show any break but it hurts to walk on. It hurts more in the morning and when I rest it it hurts when I go to walk. If I take advice and keep moving on it it seems to hurt less. It seems to hurt more when I rest it. Doctor: Hello and Welcome to \u2018Ask A Doctor\u2019 service. I have reviewed your query and here is my advice. If it's paining too much at rest may be a hairline fracture would have happened on the foot so kindly use the limb for betterment. I would recommend light exercises for the foot for 30 min each day to be in healthy state. Take ibuprofen preparation to reduce inflammation. If pain is too much then Diclofenac or Tramadol can be taken only with prescription for pain relief. Pain killers can give relief and healing occurs easily inside. Hope I have answered your query. Let me know if I can assist you further."
},
{
"id": 130867,
"tgt": "Could the severe cramps in the thighs be caused due to Bystolic having varicose veins removed?",
"src": "Patient: I have been experiencing severe cramps in my thighs at night. Could these be caused by Bystolic? I take 5mg at night and 2.5 in the morning. I have had ultra sounds and varicose vein removal prior to taking Bystolic. I never experienced the cramps before taking this medication. All lab work returns perfect such as potassium and magnesium levels. I stay well hydrated, also. These cramps are unlike charley horses. They are extremely painful. I am unable to move much less walk when I have these. Any ideas and/or suggestions?And yes, I have tried putting a bar of soap under my mattress and drinking pickle juice and tonic water. I dread going to bed at night but am becoming exhausted. Please help. Thank- you Doctor: In my opinion bystolic could have provoked and old problem like hip or back arthritis , i recommend you do back and hip x ray to confirmGood Luck"
},
{
"id": 161095,
"tgt": "What is the treatment for stomach ache in a child?",
"src": "Patient: Hi, may I answer your health queries right now ? Please type your query here... Hello my 5 yr old daughter has been complaining of tummy ache and foul smelling flatus for over a year - all the docs hv only asked us to give her de-worming medicines and Dulphalac but she is still losing weight and still complaining of flatus and tummy ache. Is this amoebiasis? HOw do we detect the same? How do we treat her? Doctor: Hi,Whatever you are describing is called Chronic Pain Abdomen in medical terms. The differential diagnosis of abdominal pain in children varies with age, gender, genetic predisposition, nutritional exposure and many environmental factors. The causes are many including - constipation / acid peptic disorders / inflammatory bowel disorders / irritable bowel syndrome / worm infestation etc. I suggest you see your pediatrician or a pediatric gastroenterologist for this.Hope I have answered your query. Let me know if I can assist you further.Regards,Dr. Sumanth Amperayani, Pediatrician, Pulmonology"
},
{
"id": 9275,
"tgt": "How can skin dryness and itching be treated?",
"src": "Patient: i used dermovate NN-oinement for 4 weeks but not continuasly in some small parts of my skin and after that i stopped using it from 6 months my skin got dry and itchy now it is better because iam taking cetrizine what i sholud do to stop itchy and the dry still there Doctor: hi, welcome to HCM,you are suffering from xerosis.few general measures can help you in keeping skin oily.avoid use of hot water and excessive bath.use oatmeal bar for bath.avoid sun exposure.wear cotton cloths first.apply emollient containing urea and lactic acid, two to three times a day.thanks"
},
{
"id": 158911,
"tgt": "Had a thyroidectomy and papillary cancer. Taking symthroid pills. High thyroid level. What to do?",
"src": "Patient: I had a thyroidectomy because of being hyperthyroid also had papillary cancer. I am on symthroid pills, ,15mg had blood work done and my thyroid levels are very high. I feel very week, cannot sleep properly, feell light headed and out of sorts. Also my left ankle calf is swollen and have pain in my legs. What should i do? Doctor: The pain in left ankle calf is not related to thyroid. Try alternative medicines. Homoeopathy has great result in Hyper as well Hypothroidism as most of the increased level is due to ongoing stress. Since homoeopathy treats individual it will definetely help you."
},
{
"id": 166643,
"tgt": "What causes frequent urination and bowel movements in children?",
"src": "Patient: Hey, I have a three year old son who, in the past 4 days (i am writing it all down, has had to urinate more frequently than normal. He is going to the potty at least 10 times a day and each time, he is peeing in volumes. He is not drinking any more than normal and he says he has no pain when peeing. What should I do Doctor: Dear parent , I understand your concerns but these symptoms could be due to a urinary tract infection or due to diabetes. I recommend that you get a urine analysis done to detect any urinary tract infection present and a blood test for his blood sugar level to exclude the possibility of diabetes"
},
{
"id": 140904,
"tgt": "What causes seizures in a seven year old having low appetite and giddiness?",
"src": "Patient: Hi Doctor, My 7 year old daughter is suffering from seizures since she was 4 yrs old. EEG report has some issues, I can send you the report if you want so that yoou can have a look. As adviced she is under medication. Valparin 200 syrup-10ml-Twice daily Lamosyn 25 tab- 1 tab-Twice daily Cloba 5 tab- 1 tab-Twice daily for couple of months we didnt notice any head drops happening however we have noticed that for the past one week she is again getting head drops. Please advice. Also her apetite has gone down drastically. Yesterday night she was also complaining og giddiness. Doctor: Hi, If you would like to upload the last EEG report for further review then, please do so at: www.bit.ly/drdariushsaghafi and I will happy to review it in the context of additional questions but it certainly seems that an adjustment to her medications would be helpful in this case. However, from a distance such as this and without being able to examine your daughter it is not possible to make valid recommendations. She is likely to be followed by a pediatric epileptologist and this is a highly specialized provider who has the best vantage point over your daughter compared to myself or anyone else online. Has there been consideration to either doing home monitoring with video recording or in-hospital 24/7 monitoring (if frequent enough) since this would be the best way of making on the fly adjustments to her meds which can then, be watched in real time for changes in the brain's electrical rhythms. Reduced appetite is a clear function of increasing seizure episodes as is the symptom of giddiness which could be a form of gelastic seizure that may be part of her seizure syndrome vs. side effect of medications. Hope I have answered your query. Let me know if I can assist you further. Regards, Dr. Dariush Saghafi, Neurologist"
},
{
"id": 142241,
"tgt": "Does ice tea cause vertigo?",
"src": "Patient: I have been drinking a lot of ice tea and recently had an episode of Veritgo. Went to an ENT, test normal. Went to a Balance/Dizziness Center for a vestibular test - normal. Wondering if tea causes levels of iron to drop, affecting the blood to make me dizzy? Doctor: No. Ice tea can't cause dizziness but low haemoglobin definitely can cause dizziness. many study shows that even ice tea related with good cardiac health and vascular health... still you are getting giddiness even consulting many doctor please stop and look that it stops or not ? if no then u can continue with ice tea and will require evaluation for dizziness like tilt table test and autonomic neuropathy evaluation."
},
{
"id": 187377,
"tgt": "Is it safe and useful to put braces at the age of 25?",
"src": "Patient: Hi Doctor,I'm 25 year old ,female. I have a question my 2 front teeth is long and jutt out a bit as mother has it,I'm thinking put up braces ,Is it safe at this age? and will it be useful ,I mean can braces make my teeth straight ast this age,If so how long it takes? Doctor: Hi, Thanks for consulting HCM, Here is Dr. Priyanka tiwari answering your query, I have gone through your query , yes , at this age it is safe , you can go through Orthodontic treatment . And as you are asking about time period of treatment , this treatment take time of atleast one year. You Should consult orthodontist for treatment. Hope I answered your query."
},
{
"id": 189101,
"tgt": "Lower front protruded teeth hits upper teeth which are root canalled and crowned. Not interested in bracing. How to avoid this?",
"src": "Patient: hello doctor,i am 45 an employee.my lower front teeth (single one)protruded from others and make hitting on my upper front teeth.my upper teeths are root canalled and crowned with porcelain caps.my dentist says bracing takes too much time and i dont interested in that .did there any other ways?like crowns implants etc.did implants look protruded when i replace the protruded teeth with an implant.did bite correction works?plese answer me kindly.\\\" Doctor: Hi,Thanks for asking the query,You need to visit an Orthodontist he will take x-rays and photographs of face, impressions to determine what type of treatment is needed.Removable retainer can be used to stabilize the position of the teeth.Small type of rotations can be treated with selective grinding of teeth.Ceramic braces with cosmetic tooth coloured braces can be used in patients afraid of using dental braces.I would suggest you to discuss the treatment options with your Dentist.Hope this helps out.Regards..."
},
{
"id": 216964,
"tgt": "Suggest treatment for pain in the bottom of heal due to blisters",
"src": "Patient: My son is crying from pain on the bottom of his heel saying it hurts it hurts he keeps grabbing his heel and I looked and saw little red bumps almost like small blisters under the skin of his heel.. I just gave him some Benadryl and I m not sure if its an allergic reaction to something but also he s had a fever all day and last night. Not sure if this correlates. If you have any suggestions id be grateful. Doctor: Unlikely to be allergic. Foreign body like splinters could be there and would need removal. Mild skin reaction to fever goes away with the fever and not by itself bad BUT. If fever high or with other symptoms can be serious. DO NOT GIVE ASPIRIN OR ASPIRIN LIKE DRUGS due to Reye's syndrome; benadryl and tylenol quite safe in recommended pediatric doses."
},
{
"id": 54681,
"tgt": "What does bubble om liver duct indicate?",
"src": "Patient: I had my gall bladder removed about 6 years ago. At that time the Dr. told me I had a bubble in a liver duct that may have to be taken care of in the future. I will I know if and when this happens? What will I feel? I have been having bad constant pain in my left upper side. It feels like it is right against my rib cage or pressing on it. Could this have something to do with the bubble thing? Or what could it be? Doctor: Hello! Thanks for putting your query in HCM. I am a Gastroenterologist. Bubble in liver duct may be presence of air which may have entered during cholecystectomy or due to passage of any stone from CBD into duodenum. So there is nothing to worry and it will disappear after sometimeI hope I have answered your query and this will help you . Wish you a good health"
},
{
"id": 213701,
"tgt": "How can I solve my husband's psychological problem that he can't have sex ?",
"src": "Patient: hi..my husband is of 48 years and he is having psychlogical problem that he has made a pshycological thinking thah he can not do sex...and most of the time when he tries to do sex he fails to perform...and due to this many suicidal ideas comes in his mind.he is suffering from this problem for last 25 years..he consulted a psychiatrist but no relief was there.... Doctor: As a life partner of him you can rule out what is exactly fear in his mind. try this skillfully by discussing on d subjects he likes.you can make d best counseling as a wife.never put any pressure or make negative thinking. Vajikaran Therapy in Ayurved is quite useful."
},
{
"id": 85699,
"tgt": "Can avil 25 be taken for curing rashes over body due to eating fish with history of having open heart surgery?",
"src": "Patient: I had an open heart by pass surgery an year ago.Yesterday night I had fish . Soon after eating eat I developed rashes all over the body that itched. I want to take avil 25 an anti-histamine drug. I want to know wheteher it has any side effect on heart Doctor: Hello, You can safely take Avil or other antihistamines. Previous heart surgery or cardiac issues are not a contraindication for Avil. Hope I have answered your query. Let me know if I can assist you further. Take care Regards, Dr Shinas Hussain, General & Family Physician"
},
{
"id": 167029,
"tgt": "Suggest remedy for boil on the back of left thigh",
"src": "Patient: My son is 6yrs. old. Last Tuesday he had what looked like a boil appear on the back of his left thigh. My husband and I put Neosporin and a gauze square on it Saturday. Sunday morning we looked at the gauze and there was some fluid on it. We just thought the boil burst and it was healing. Today is Wednesday, April 13 and the area in the middle of the spot has a red ring. It looks almost like a bulls-eye. The area is a little painful to him. Any answers would help me! Thank You! Doctor: Hello. I just read through your question.It is likely the area will not heal with oral antibiotics. Additionally, it may need to be drained by a doctor.I recommend consulting with your doctor about this."
},
{
"id": 192771,
"tgt": "What is the treatment for erectile dysfunction?",
"src": "Patient: Hi, may I answer your health queries right now ? Please type your query here...doc I have problem in erection and i had problem of stone as well,from last few days i have pain at the side of my abdominal and testorone.but after medication the pain has gone but still erection problem persists. Doctor: Hi, Okay, I am glad the pain is gone. I'm not sure how that would relate to the ED. Without an exam, I can only give general information and cannot give a specific treatment/diagnosis/medication to someone I am not actually seeing. So, ED in general often is associated with hypertension, or cigarettes but lack of exercise is the main issue. It is more of a problem if someone is diabetic but otherwise the medications work in about 90%. Hope I have answered your query. Let me know if I can assist you further. Regards, Dr. Matt Wachsman, Addiction Medicine Specialist"
},
{
"id": 131925,
"tgt": "Suggest treatment for neuropathy",
"src": "Patient: I WAS DIAGONONISED CANCER OF THE CERVIX IN 2004 BUT UNFORTUNATELY AFTER GOING THROUGH MAJOR OPERATION OF TOTAL HISTRETOMEY AND THE RADIO AND CHEMO THERAPIES I LATER DEVELOPED NUMBNESS IN THE LOWER LIMBS WEAK NUEROPATHY. mY LEGS NOW SEEMS AS IF THEY ARE STICKS I STRUGGLE TO BEND MY KNEES AND IF I ITS DIFFICULT FOR ME TO STAND UP. I NOW DONT KNOW WHAT TO DO, HOW BEST CAN YOU ASSIST ME? Doctor: hi This kind of weakness is very common after radiotherapy and chemotherapy . I suggest that you start good diet rich in vitamins proteins and carbohydrate ( though you will be inclined not to eat) so that your body recovers fast . Wait for 2-3 months and then if you feel that numbness is not improving then go for MRI and NCV to assess damage to nerves if any."
},
{
"id": 68755,
"tgt": "What causes sore red lump on the back of the thigh?",
"src": "Patient: I've woken up with a red lump on the back of my thigh, it has a dark centre (almost looking as if blood is trapped under the skin) and is sore to touch...I was wondering if anyone could tell me what this could be and whether I need to go to the doctors or if it will disappear on its own. Doctor: Hi,This could have been caused by an insect bite. As you describe it as painful and red, I think it may be infected and there is a risk you may develop cellulitis. Therefore, I suggest you go to the ED for assessment and possible antibiotics. Regards,Dr K A Pottinger"
},
{
"id": 209183,
"tgt": "Are there any side effects of taking Fortesta on behavior?",
"src": "Patient: MY husband is 48 and has been taking Fortesta for Low T for a about a year and half. In increasingly larger doses. After starting this he began to change, researched and carried out having an affair and moved out 8 months ago. He seems very disconnected,depressed and sad has memory problems, not at all like himself. We have been together for 25 years. He was also, diagnosed with type 2 diabetes about 7 months ago(which he is in denial about). He stopped taking his Cymbalta as he says he does not need this. Could the Fortesta be a factor in such out of character personality changes in him? He does not seem like the man I know Doctor: HiThanks for using healthcare magicFortesta contains testosterone and it has lot of side effects. It also effects on behavioral in form of increase aggression, impulsiveness, change sexual drive, mood swings etc. In that case, he needs to stop it and such side effect would improve. Better to consult a psychiatrist or physician for proper management. In case, you need further help, you can ask.Thanks"
},
{
"id": 143058,
"tgt": "Is advance volume loss and age related neuronal attrition is complicated for recovery?",
"src": "Patient: Hi Dr. Keerthi. My husband had a traumatic head injury 20 years ago and was in a coma for 20 days. The injury left him with a limp in his walk and memory problems. Four years ago he had a stroke which complicated his recovery. He is now unable to walk and has severe aphasia. The doctors are saying he has advance volumn loss in his brain. Could it be that this advance volumn loss and age related neuronal attrition is what is complicating his recovery. Thank you for any answer you may have. Doctor: Hi, Welcome to HealthCareMagic.com I am Dr.J.Mariano Anto Bruno Mascarenhas. I have gone through your query with diligence and would like you to know that I am here to help you.Question : Could it be that this advance volumn loss and age related neuronal attrition is what is complicating his recovery. Answer : Yes. The Volume Loss and Age Related Neuronal Attrition are the cause of slow and delayed recovery Hope you found the answer helpful.If you need any clarification / have doubts / have additional questions / have follow up questions, then please do not hesitate in asking again. I will be happy to answer your questions. In the future, for continuity of care, I encourage you to contact me directly in HealthCareMagic at http://bit.ly/askdrbruno Best Wishes for Speedy Recovery Let me know if I can assist you further.Take care."
},
{
"id": 131437,
"tgt": "Does Belladonna works effectively for the hip pain having Osteoarthritis?",
"src": "Patient: Hello, I have osteo arthritis and have severe hip pain. I have had a cortizone shot for my hip some time ago. Now the pain is unbearable, this is 24/7. I have been taking vicoprofen and flexeril they do not touch it. My sister in law had told me that she was given a prescription for Belladonna for her hip and it took the pain away. I would like to ask you your thoughts on this and does this work for pain? Doctor: HiI may suggest an MRI for diagnosis of hip disease.you may try accelofenac 100 mg twice a day and chloraxazone 500 mg twice daily.Do hot fomentation and apply voveran TPS gel of novartis around the joint.use walking stick in opposite hand to relieve weight bearing pressure on hip.consult hip surgeon for evaluation.Belladonna or alternative systems of medications are often good for symptomatic relief.zBut my concern would be to firstly know exact cause and removal or treatment of underlying disease process to try finish problem at the rootBest wishes"
},
{
"id": 181679,
"tgt": "Suggest treatment for severe tooth infection",
"src": "Patient: Hi. I have a tooth that is hurting very badly and I can't get in with my dentist until Monday. Can Cipro be used as a preventative med over the weekend? I feel like I have some infection going on. The Cipro is still good, and was prescribed for something else about 3 months ago, but it's the only antibiotic I have on hand. Doctor: Thanks for your query, I have gone through your query.The tooth infection can be secondary to a decayed tooth. Nothing to be panic, you can take ciprofloxacin till you consult your dentist. Later you can get the tooth treated with RCT. You can also take diclofenac for the pain.I hope my answer will help you, take care."
},
{
"id": 19566,
"tgt": "What causes rapid heart rate with pulsations in throat?",
"src": "Patient: Hello.I am 13. Completly healthy. A while back i had something wrong with my heart.They had me take 2 EKG's, i wore a halter for 24 hours, and i had to run on a tredmill and they did a aultra sound on my heart. I have 3 'cuplets' they called it. Apperently i have PVC. i havnt been back to the hospital for over 7 months. My moms scedualed a check up appt. but i would like to know what this is:I will just be sitting doing nothing and i can feel my heart speed up and i can feel it in my throat. and it gets harder to breathe.what is this? Doctor: you are having pvc ...and this is due to many disaeses as high bp or ischemic heart disease ..as you have already underwent all cardio tests ..i assume these are normal ..now please go for thyroid profile and sugar level ..if everything is ok .then avoid tea coffee alcohal and smoke ...avoid fast foods and herbal products .."
},
{
"id": 127006,
"tgt": "What causes leg ache after a herniated disc surgery?",
"src": "Patient: i had back surgery for a herniated disc and 2 compressed nerves about 2 months ago and i started having leg pain 2 days ago. It is from the right knee down on the back of my leg about 4-6 inches. It is only when i lay down and try to move or pick my leg up. Doctor: Hi, It is due to nerves impingement caused by herniated disc. You can consult an orthopaedician and get an MRI scan done. Hope I have answered your query. Let me know if I can assist you further."
},
{
"id": 178755,
"tgt": "What causes redness around the mouth of an infant after starting cerelac?",
"src": "Patient: hi doctor, my baby is now 6 months. i started giving cerelac and aptamil for him. at once we gave these things there is redness around the mouth ( places where it spills). after 10 minutes it vanishes. is he allergic to that, shall i discontinue these things. need your advice. thank you Doctor: Dear Sir/ Madam,Thank you for posting your query at healthcaremagic.comSome important points to be noted while feeding the cerelac. 1. Freshly made cerelac to be fed2. Let the child take its own time to swollow, do not worry of dribbling around the lips and chicks3. Do not wipe in between4. It should not be very hot, mild temperature.5. Once the full feed is over, then gently wipe it with little warm water and dap it.Generally with all precaution it should not cause any allergy or redness.Please feel free to consult me if you have any further query.Your thank you note is very much appreciated!With best wishes,Dr. Vishwanath Patil"
},
{
"id": 225190,
"tgt": "What medicine is suggested for unprotected sex to avoid pregnancy ?",
"src": "Patient: Hi,I have had unprotected sex and was wondering if I could use Falmina as an emergency contraceptive. I took 3 pills immediately after intercourse ( I'm sure that is wrong, but I have no idea). Basically what is the proper dosage of Falmina if I am trying to use it as an emergency contraceptive? (how many and when should I take it???) Doctor: Thanks for query on HCM.Emergency contraception is effective only in the first few days following intercourse It should be taken within 72 hours after unprotected intercourse.Falmina is combined pill and can work as a emergency pill but you have to repeat it after 12 hours to prevent pregnancy.Now wait and watch for next cycle or withdrawal bleeding.Otherwise consult gynecologist for further guidance."
},
{
"id": 122975,
"tgt": "Suggest treatment for bruised knee",
"src": "Patient: I fell on my knee a few months ago and of course my knee was really bruised and black and blue for about 2wks and my other knee felted bruised but no black and blue but still to this day its hurts if i gobto my knees or if sonething slightly hits its sometimes even to the touch what should i do Doctor: Hello, It is quite common after the fall. It is nothing but a collection of blood underneath the skin. Nothing much to worry and it will settle by itself. As first-line management, you can apply icepacks for faster recovery. If symptoms persist better to consult an orthopedic and get evaluated. Hope I have answered your query. Let me know if I can assist you further. Take care Regards, Dr Shinas Hussain, General & Family Physician"
},
{
"id": 30545,
"tgt": "Suggest treatment for wound after cyst removal from underarm",
"src": "Patient: i had a surgery to remove a cycst in my underarm now the bandaid has come of and something is projecting out from the wound what m i supposed to do there are no doctors in vasai as it is sunday where m i supposed to go Doctor: hiwelcome to HCMI am here to assist you, and i can understand your concern.Donot worryyou had underwent cyst removal surgery and they have closed it after the removal.if the bandied had opened , put a new one if have or use clean or sterile cotton to close the operated site. avoid touching with your fingers and water to that area.it may lead to infection.continue your antibiotics and pain killer tablets if your surgeon prescribed after surgery.tomorrow you can go and visit your doctor for dressing and follow up.I Hope i have cleared your queryThank youTake care"
},
{
"id": 163629,
"tgt": "What causes child to be unable to breath sometimes?",
"src": "Patient: I have a 7 month old who at times gets strangled easily and at other times will suddenly be unable to breath. He acts as if he were holding his breath; however, he isnt he just cant get air in. He turns blue nearly passing out before finally regains ability. Can the flap that prevents foods etc from going into your lungs be sticking and causing inability to allow air passage also? Doctor: Hello,Many infants go through what is called as breath holding spells. There is no known cause for it, and it seen more often than one would think, these usually last for about 1 minute. Breath holding spells occur when baby is hungry and cries or is seeking attention and does not know how to express.Infant breath-holding or turning blue is more common than you think in babies. As a parent you will be scared to death but please understand that the infant does not have it in him to hold his breath long enough to cause damage. The body\u2019s natural mechanism of breathing will kick in and override the forced breath-holding.You should be concerned if they happen too often or while feeding or happen because of high grade fever or seizures.Hope I have answered your query. Let me know if I can assist you further.Regards,Dr. Vignan Rachabattuni"
},
{
"id": 176176,
"tgt": "What causes micro penis in child?",
"src": "Patient: My son is 8yrs old. He is 58 inches tall and weights 94 lbs. I am concerned his penis isn t keeping up with the growth of his body. should I have his testosterone level checked? My 6 yr old is 52 inches tall and weighs 63 lbs, His penis is larger than my 8 year old. Doctor: Hi,Welcome to the HCMMicropenis with tall height can be caused by chromosomal (genetic )abnormality.In boys most common abnormality with these signs is XXY syndrome.To be on safe side get chromosomal study test donefirst and discuss with the doctor.Regards,Dr.Maheshwari"
},
{
"id": 32593,
"tgt": "Suggest treatment for painful mouth ulcers",
"src": "Patient: I am suffering from mouth ulcer for last 2 years. Earlier it used to appear as a white circle inside the chin and tounge. Now-a-days the size has become very big and appears mainly on inside chin. It's painful. Tried several medicines, but no use. Pls help Doctor: Hi..Welcome to HEALTHCARE MAGIC..I have gone through your query and can understand your concern...As per your complain it seems that you have Major Apthous Ulcers as you told that they are large in size and major apthous ulcers has a size more than 10 mm..Apthous ulcers that do not respond to any other treatments are finally treated by Steroids..You should take a short course of steroids under supervision of your physician..Along with it take a multivitamin tablet daily..For pain and inflammation Ibuprofen can be taken..Do warm saline gargles..Soda bicarb gargle will ay help..For localized relief from pain and discomfort apply a numbing gel containing Lignocaine over the ulcers for relief especially during meals..Hope this information helps..Thanks and regards..Dr.Honey Nandwani Arora.."
},
{
"id": 140170,
"tgt": "What could be the cause of my cold sweats, lightheadedness and muffled hearing after falling?",
"src": "Patient: 26 yo female - 5 6 145lbs. I recently missed the bottom 2 stairs and fractured my ankle. I have been to the DR recently for an annual physical. All bloodwork/labs show normal including thyroid and hormones. The night I fell and fractured my ankle, about 5 min after the fall I got extremely hot, started sweating, and got light headed. I felt like I was going to pass out. I lost hearing except for a high pitch ringing noise. This lasted a minute or two. This was by far the most extreme version of this happening, but I have had this happen several time before. What could be the cause of my cold sweats, light headedness, and muffled hearing??? Doctor: Hello, I would explain that your symptoms seem to be related to low blood pressure, probably caused by a physiological reaction to the pain and trauma. In my opinion, there is nothing concerning in this situation, considering the trauma you have gone through. Hope I have answered your query. Let me know if I can assist you further. Wishing you good health. Regards, Dr. Ilir Sharka, Cardiologist"
},
{
"id": 40005,
"tgt": "Could severe chills followed by fatigue, headache and low fever after very hot bath indicate flu?",
"src": "Patient: Last night, I got severe chills, I took a hot bath, (maybe too hot) and today I am very tired, and sore, with a headach very low fever. I have a rash on my stomach, but I think it may be because I had the bath water too hot. Do you think it is just a virius or some type of flu? Doctor: HelloWelcome to HCM, The symptoms are suggestive of some flu like condition probably of viral origin.The symptoms of high grade fever with chills along with some rashes on the abdomen are in favor of viral infection. For this condition I would suggest you to follow1. Oral Antipyretics2. Oral Antibiotics like Zifi3. Oral antihistamines. These medications helps to reduce the symptoms, but these can be taken only after getting prescription from your doctor. Thank you."
},
{
"id": 78499,
"tgt": "What causes persistent cough with CXR showing \"blunt costophrenic angle\"?",
"src": "Patient: hi,i'm a medical student.i want to know the etioligy of persistant cough and CXR with this findings: CXR of blunt right heart boarder+right costophrenic angle blunt+midline shift to right+elevated right hemidiaphragm.chest sono doesn't seen effusion.tnx Doctor: It may be a thick effusion which is missed on ultrasonography...Or a Mass...A lateral view would help,in arriving at a diagnosis..."
},
{
"id": 96939,
"tgt": "How to get rid of soreness on backside near ribs after i fall back on ground?",
"src": "Patient: Male. Fell back onto ground one week ago and became sore on left side of back near ribs. Pain has migrated around the front and side lower ribs and tissue below lower left rib cage. Now abdomen is swollen on left side below rib cage and very painful to touch, bend or lay. No bruising has occurred. Doctor: You need immediate surgical attention. It can be a migratory infection. May get healed by high potency antibiotic, or may need local surgical care. You should not ignore it. It can progress to high morbid condition."
},
{
"id": 20077,
"tgt": "Is it possible to donate arteries for a person with arteriostenosis?",
"src": "Patient: Hello, my name is brittany and i have a very serious question for you. My aunt has a very weak heart and very small arteries because she was born with arteriostenosis, the problem is it has nothing to do with a plaque formation, just very narrow arteries. I am will willing to donate some of my arteries to help her, is it possible for me to donate? Doctor: HelloI can Understand your concern for her, its not practically impossible but it will significantly increase the risk of other complications to her. As putting your vessels in her will be just like an organ transplant and then in future she has to be on very strong immunosupresants which can make her other wise ill.So that why its not done.God Bless"
},
{
"id": 12954,
"tgt": "What could it be if having cluster of small pimples which leaves dark blotches when scratched?",
"src": "Patient: Last winter I developed an itchy rash on my lower back. I treated it with hydrocortozone (.5%). The itching is gone but I am left with a large dark blotch which isn't going away even after using Neostrata lightening cream. This winter I have that same rash on upper arm and it has spread to my back. It looks like a cluster of small dry pimples moving onto my back in straight line. When I scratch it, it leaves dark blotches that just won't go away. Doctor: I would recommend you to apply benzoyl peroxide 2.5 percent gel on the bumps twice daily and a depigmenting cream containing kojic acid on the dark spots.For itching take antihistaminic like tablet chlorpheniramine maleate once daily."
},
{
"id": 114640,
"tgt": "How can hyponatremia and leukocytosis be treated?",
"src": "Patient: hi, my dad has been getting really bad fevers 102-103 degrees that last a couple days where he feels freezing cold and he is sweating. He went to the doctor last month and got bloodwork and they gave him a diagnosis of leukocytosis and hyponatremia and told him to get another blood test, well he felt fine for a month so he didn t get it done, and now all the symptoms are back. What could cause this? Does he have an infection? He has type 2 diabetes and hypertension and he drinks alcohol most evenings. He also hit his head pretty bad about 2 months ago and knocked himself out cold. I appreciate your help. Doctor: Hi, I had gone through your question and understand your concerns.It is likely that he is having an infection as he is diabetic also and diabetics can have occult infections specially urinary tract infections.if i were his treating physician , i would like to take complete history and do complete physical examination to look for any infectious cause and would like to do urine r/e, urine c/s , blood c/s ,MP smear and CXR. As he is having high grade fever ,sweating, leukocytosis and hyponatremia all these in old age patient can be due to Legionella pneumonia and would also like to work up for that.Hope this answers your question. If you have additional questions or follow up questions then please do not hesitate in writing to us. I will be happy to answer your questions."
},
{
"id": 143993,
"tgt": "What causes stiffness and numbness post slipped disc surgery?",
"src": "Patient: history of disc herniation at L4,L5. Have been experiencing stiffness and achy in right posterior hip. Feels like a knot there. Starting yesterday I have numbness at that location, going down into my leg and I have numbness and weakness in my foot , which I can t flex up towards my leg. Doctor: hi,thank-you for providing the brief history of you.As you have undergone surgery for the L4-L5 disc herniation there is a possibility of the fragment from the disc to dislodge again or may be a muscle spasm in the lumbar paraspinal muscles which takes the incisional point.Taking an MRI should make understand if the disc fragment is dislodged again. If MRI report comes fine then it's just muscular and with muscle relaxant it should be fine.In my clinical practice we make approach in a way to diagnose correctly and make sure the pain do not returnRegards Jay Indravadan Patel"
},
{
"id": 86368,
"tgt": "Suggest treatment for severe upper abdominal pain",
"src": "Patient: I have had an annoying little pain in my upper abdomen almost under my rib cage for some time now. It almost feels crampy?? It sometimes feels as though it s connected to my female parts or it s bowel/gas related. It s just annoying not actually painful?? I mentioned it to a doctor once and she gave me antibiotics for ibs.....didn t seem to help but I didn t take them as instructed either. Doctor: For severe upper abdominal pain you can take some antibiotic preferably cefadroxyl or oflox-oz after an expert opinion along with cyclopam or meftal-spaz for severe pain and cramps in abdomen. You can take omez-d or rekool-d empty stomach at-least one hour before meal for severe gastric upset."
},
{
"id": 77464,
"tgt": "What causes coughing, yellow greenish phlegm for a prolonged time?",
"src": "Patient: hi there. its about 4 month that I'm coughing, its getting worst. phlegm, sometimes yellow,sometimes green ,sometimes white with thin black traces,it looks more like dust,nothing like blood. it makes me go short of breath, sometimes its so bad i cant even finish a sentence, I've done chest ray and they said its fine. ct scan says chronic sinosite plus other medical terms that i dont understand. i have stopped using perfume, but no changes.In three different countries and climate same caugh,getting worst by day.any clue? Doctor: Thanks for your question on Health Care Magic. I can understand your concern. In my opinion, you are mostly having bronchitis. It is inflammation of airways which causes cough, mucus expectoration, breathing difficulty etc. It is not diagnosed by chest x ray or CT scan. PFT (Pulmonary Function Test) is must for the diagnosis of bronchitis. It will also tell you about severity of the disease and treatment of bronchitis is based on severity only. You will improve with inhaled bronchodilators and inhaled corticosteroid (ICS). Oral combination of antihistamine and anti inflammatory drugs is also beneficial. Don't worry, you will be alright.First diagnose yourself and then start appropriate treatment Hope I have solved your query. I will be happy to help you further. Wish you good health. Thanks."
},
{
"id": 213299,
"tgt": "Taking resperdonex. Feeling lethargic after started the medication. Can the dosage be lowered?",
"src": "Patient: I have a 90lb son who has been on resperdonex1mo. He was lethargic and flat so i stopped dose for 36hr and restarted .5 am and .5 pm...(which was half the 1mg bid dose he was on)he is currently lethargic and has a flat affect on this dose...could he just be sensitive to this med??is there a therapeutic dose lower thant this??his agressive/violent behavior is well controlled on this med but I feel I ve lost him as a person. Thank you. Doctor: Therapeutic dose of risperidone varies from diagnosis to diagnosis For schizophrenia or psychosis it is 2-12 mg/day Blunting of affect can be because of risperidone Such low dose is unlikely to cause blunting There can be alternative reason related to illness for \"loosing him as a person\""
},
{
"id": 114317,
"tgt": "How else can anemia be treated apart from a blood transfusion?",
"src": "Patient: I started with a surface vein blood clot in my leg. that has since cleared up. the pain is finally gone. During this process My blood was tested and am found out to have Anemia. This far I have had a colonoscopy/Endoscopy/CT scan each one of these tests has come back clear. I should tell you I use natural medications such as plant green iron and such to get my numbers back up. 6.1 is my hemo. count and they tell me that is low. Now they want to do a blood transfusion and I am really not on board with this. What our some other options? Doctor: Hello and Welcome to \u2018Ask A Doctor\u2019 service. I have reviewed your query and here is my advice. I think you need transfusion because your hemoglobin is very low and after that you can continue to take iron. Hope I have answered your query. Let me know if I can assist you further."
},
{
"id": 84086,
"tgt": "What are the side effects of plan b pill?",
"src": "Patient: I took a plan b one step on Sunday morning, today Wenesday, I have a horrible yeast infection. Is that normal? Is anything else I could to help my ph balanced. I have never taken a Plan B before, I didn t expect the side effects to be this uncomfortable. Doctor: HiHormonal pills does not cause yeast infections.But certain drugs can alter the vaginal ph and cause overgrowth of harmful bacteria in the vagina.This could result in vaginal infections.Drugs in the form of tablets or cream containing clotrimazole can be used. Ph balancing washes can be used once a day.Hope I have answered your query. Let me know if I can assist you further. RegardsDr.Saranya Ramadoss, General and Family PhysicianHope I have answered your query. Let me know if I can assist you further. RegardsDr.Saranya Ramadoss, General and Family Physician"
},
{
"id": 206533,
"tgt": "Does affinity to spell words after a mental trauma need medical attention?",
"src": "Patient: Since my mother s health declined and recently passed away I have started spelling all the time. Random. Words from conversation, thoughts, tv, whatever. Should I be worried? I have an appt set up for later in the week for other health concerns and wonder if this is something I should address. Doctor: Hello You have not mentioned the details of the symptoms you are facing but from the summary it appears that after your mother passed you developed the symptoms. You have started spelling random words from conversion, thoughts etc. I would like to know if you don't spell words then do you feel distressed or anxiety?Some times in obsessions individuals tend to have idea or thought and he can't control the thought. These thoughts produce stress and in order to relieve from stress of thoughts individuals are compelled to do some activities like counting, spelling words etc. Such symptoms can be the part of Obsessive Compulsive Disorder. In severe anxiety following your mothers death you have developed such symptoms.Consult a Psychiatrist for detailed treatment.Thanks"
},
{
"id": 90850,
"tgt": "Does pain under belly button indicate hernia or strain from workout?",
"src": "Patient: Hi so I'm 5'10 weigh 73 kg. I go to the gym 6 times a week and lift pretty heavy weight. 3 days ago I went out to a sushi place to eat. A day after that I had watery diarrhea, small amount pretty frequent in the morning but got better as the day progressed. That same night I went ton the gym and lifted really heavy weights, working out my back, i also worked Out my abs. It's been 2 days and I have this pain under my belly button. Its not the pain I usually feel after an abdominal workout. I don't see a bulge and I palpated the area and I dint feel a lump or anything irregular. I also started taking protein shakes 3 days ago which also exacerbated my diarrhea that day. I was wondering if the pain I am experiencing under my belly button is a hernia? It doesn't huts per se but when I flex my abs and lean forward it is tender right under the belly. I just want ti know if it's a hernia, if it's related to the diarrhea or strain from working out 6 days a week. Thanks Doctor: Umbilical hernia usually has a bulge,defect can be palpated, on coughing with hand placed over umbilicus (belly button) an impulse is felt.Only rarely an umbilical hernia becomes irreducible, i.e. bulge doesnt disappear on lying down, then obstructed & strangulated causing pain there.Seems that pain is due to exercise.Diarrhoeal pain is usually generalised all over abdomen."
},
{
"id": 169952,
"tgt": "Suggest Febrex Plus dosage for a child",
"src": "Patient: hi my baby 3 yrs old is having cold for the past 2 days due to cold she had vomitting sensation i took her to a general physician yesterday he gave me vomikind 4mg she had one dose today i feel after that she is having cold and fever can i give febrex plus syrup?pl tell me the dose Doctor: Hello and Welcome to \u2018Ask A Doctor\u2019 service. I have reviewed your query and here is my advice.Can you tell me the weight of the baby? If so only I can suggest the dose details. If it is 125 mg and her weight is more than 10 kgs you can give 4mL, 4 times daily and tepid sponging is also advisable.Hope I have answered your query. Let me know if I can assist you further.Regards,Dr. Sai Vamsi Krishna.s"
},
{
"id": 1112,
"tgt": "Does having gastric problems affect chances of conception?",
"src": "Patient: Hi,We are trying to conceive a baby from past 1 year but didn\u2019t have success. I have following to queries:1.\u00a0\u00a0\u00a0\u00a0\u00a0We both are suffering from gastric problems does it have relation with pregnancy.2.\u00a0\u00a0\u00a0\u00a0\u00a0I have 30 day MC so after how many days we should have intercourse to have the positive result. Doctor: Hi, I think gastric problem has no effect on your chance to conceive. Be in contact with your husband every 2 to 3 days after your periods stop. It will increase chances of your pregnancy. Try naturally for 6 months and if it doesn't work, then you can go for evaluation. Hope I have answered your question. Regards Dr khushboo"
},
{
"id": 121077,
"tgt": "How to cure plantar fasciitis?",
"src": "Patient: I m suffering Plantar facities for the last 3/4 days (with acute pain on left foot), Dr prescribed Etura 500mg & tab Razo 20 mg..taking the same for the last 2 days.... releif not yet seen..how many days..it require rest..is there any way out for immediate releif? Doctor: Hello,I read carefully your query and understand your concern. Your symptoms seem to be related to plantar fasciitis. I recommend to continue the treatment as prescribed by your doctor. Plantar fasciitis\u00a0usually resolves within 6 to 18 months without treatment. With 6 months of consistent treatment, people with\u00a0plantar fasciitis\u00a0will\u00a0recover\u00a097 percent of the time.Hope my answer was helpful.If you have further queries feel free to contact me again.Kind regards! Dr.Dorina Gurabardhi General &Family Physician"
},
{
"id": 175712,
"tgt": "Suggest treatment for frequent stool and nappy rash",
"src": "Patient: Hi, my 3 month baby still do frequent small amount of poo and causing nappy rash. It s like every hour or every nappy change something is there.I put her off nappy for plenty time. The nappy rash is always there. I went to the doctor, DD was digonosed by lactose intolorance. Doctor recommend to feed lactose free formula but I did berestfeed and formula together. Now I want to go with brestfeed only but when I stop formula for 12 hr she start frequent poo. I am wandering the cause is lactose intolorence or something else. When can I stop feeding formula? Doctor: Hi...Greetings from Chennai. After going through your question in detail - this is my opinion and this is what I would have done if I were your treating pediatrician.Possibility -1. Cow's milk protein allergy is a strong possibility as there as blood in the stool in exclusively breast fed baby.Suggestions -1. Mother should go off the cow\u2019s milk products completely - including biscuits and chocolates and all products related to milk.2. Continue exclusive breast feeding and if it is not sufficient you can opt for Zerolac.3. Cow's milk protein intolerance in transient as it will go off as the kid grows up.4. Avoid all cow's milk products till the kid is 1 year age.5. Mother's milk is the best for good growth of the baby. Rather than stopping mother's milk, it is best for the mother to avoid all milk products and continue breast feeding.If you have any further queries you can approach me at the following link.You can approach me at the following link. Please find the link below.www.healthcaremagic.com/doctors/dr-sumanth-amperayani/67696Regards - Dr. Sumanth"
},
{
"id": 67060,
"tgt": "What is the large lump on the end of my tail bone?",
"src": "Patient: The end of my tail bone has a large lump (a little smaller than a golf ball). It veers to the right and it aches all the time. I cannot sit on any hard surfaces and always have to use a pillow. My husband thinks it had gotten bigger since we first noticed about a year ago. I haven t injured it at all and am having no bathroom issues. What could explain this? Doctor: Hello and welcome to HCM,A lump at end of tail bone required imaging studies of the backbone.Imaging studies will show the origin of the lump whether the lump is connected to the back bone or not.In case the lump is not arising and is a soft tissue swelling, an aspiration cytology of the lump is required.Aspiration cytology the cells are aspirated with a needle, cells are spread on a glass slide and stained with appropriate stain.The stains are studied microscopically to know the nature and character of the lesion.The management of the lesion can be planned after clinical assessment and relevant investigations.Thanks and take careDr Shailja Puri"
},
{
"id": 203973,
"tgt": "What is the reason for not getting erection for long during intercourse?",
"src": "Patient: Hi every time I have sex with my wife. I only last a few seconds. I try to stop for a few seconds but it doesn't help. After I go I can't get it hard again. I'm afraid my wife will leaveme if this keeps up. If I drink and get drunk I will last longer more than if I wasn't drunk. Please help me. YYYY@YYYY Doctor: hi i think u are suffering from erectile dysfunction.it can be due to diabetes,nervous dysfunction or anxiety.first rule out diabetes by getting blood sugar done.if it is not raised treat it with neurokind plus tablet in the morning andthrilpil tablet in the night.sometimes erectile dysfunction is due to low testosterone levels,. in this case it is treated with injection of testosterone undeconate"
},
{
"id": 225423,
"tgt": "Feeling nauseous after birth control shot, tired, brown tissue seen in urine. Can antibiotics interfere with birth control ?",
"src": "Patient: I had surgery about 6 weeks ago, and around that time I got updated on my birth control shot. I am on the depo shot but was also on antibiotics for an infection. A day after my antibiotics ran out I had sex with my boyfriend and we didn't use protection. A few weeks later I started feeling nauseous and tired all the time. Lately I have been craving weird foods and eating more than normal... Also when i got to the bathroom i urinate little brown pieces of tissue... Could my birth control have gotten messed up from my antibiotics and could i be pregnant? Doctor: you are thinking right direction get consultation from your gynaec doctor whio can tell you after physical examination and may be ultrasound to get wexact posion and treat accordingly"
},
{
"id": 64689,
"tgt": "What causes a lump on the forehead?",
"src": "Patient: My partner has had a lump on the front of his head for about 13 years and over the last few weeks it has been changing slightly and the other day when he felt it he realised he had something crusty in his hair around it... When he looked in the mirror it burst. Lots of cream/brown stuff came out along with some blood and a few small (aprox 2mm) hard lumps came out. It ended up looking like nothing more than a big spot by the end but has since got raised again and is painful. I don't know if there is a link but he has been in a bad mood and going to bed early too. Do you know what this could be and if we should worry about it!? Doctor: This is an infected sebaceous cyst that has burst and released the pus and muck. It is a benign condition. Go to a general surgeon and he will clean it and prescribe antibiotics and once the infection clears up, he shall remove it."
},
{
"id": 144683,
"tgt": "What causes body tremors?",
"src": "Patient: Hi ..My father(age 53) is suffering with tremors in the whole body (hands, legs, head) and some times not able to walk also. We have consulted 1) Neurologist and took a MRI scan as he advised but he said nothing problem from his side and advised to use medication ZAPIZ and TOPAMAC. 2) After that consulted Endocrinologist as my father Thyroid profile results were bit high and He advised us to use CIPLA 40 after observing results of Thyroid T3, T4 ,TSH , Free T3,T4 , Thyroid scan. 3) As we had doubt on leg bones and it would be reason for tremor in legs, we have consulted Orthologist, he took some x-ray to both legs and gave some medicine to strength legs. After consultation of all these specialists and as they advised, My father is using...ZAPIZ, CIPLA LA 40 for tremors. After using these medicine , tremors in the body is not reduced and sometimes it is increasing because of this medicine as per my father s observation daily. Please advice how to proceed. Doctor: I would like to ask you some questions:1. Are these tremors happen at rest or while doing some activity?2. Is there any slowing in daily activity along with tremor?Tremogram can be done to look for frquency of tremors as different types of tremor have different frequencies. Also need clinical evaluation about any feature of parkinsoism."
},
{
"id": 9610,
"tgt": "What causes dry skin near the vagina between the thighs ? How can it be treated ?",
"src": "Patient: kind of blisters and itchy sensation at both the sides near the vaginawhere my thigh joint connects to the pelvis when I scratch itit leaves dry skin around,what do I do? I am 29 years old with regular sex,I started noticing a kind of blisters and itchy sensation at both the sides near the vagina where my thigh joint connects to the pelvis when I scratch itit leaves dry skin and as well dark scars,what do I do?It gives a lot of discomfort during intercourse.Please help me. Doctor: kindly show it to the doctor and then start the medicines.It seems you need antifungal treatment, tablets and local ointments.Do not forget change your innerwares twice in a day."
},
{
"id": 71670,
"tgt": "What does this chest X-ray indicate?",
"src": "Patient: I have a copy of chest x-ray that I had to check for Pneumonia The doctor said no Pneumonia but rather asthmatic bronchitis. I had asthma when I was very young but had outgrown it. The finding said no acute infiltrations slight coarsening of the bronchovascular markings. The costophrenic angles are sharp heart not enlarged aorta is uncoiled. Can you tell me what all this means Doctor: Thanks for your question on Healthcare Magic.I can understand your concern. I have gone through the x ray report you have mentioned. This x ray report is normal. No need to worry for pneumonia or tuberculosis. Asthmatic bronchitis is diagnosed by PFT (Pulmonary Function Test). Chest x ray is almost always normal in asthmatic bronchitis.So better to consult pulmonologist and get done clinical examination of respiratory system and PFT (Pulmonary Function Test).If PFT is showing obstructive defect then asthmatic bronchitis is likely. Hope I have solved your query. I will be happy to help you further. Wish you good health. Thanks."
},
{
"id": 106119,
"tgt": "Suffering from asthma and feel weak and dizzy when i stand",
"src": "Patient: Hi, I am 18 years old and I expierenced a asthma attack today that sent me to the emergency room. I was having pain in both my calfs during the attack and they told me it was from low oxygen levels in the blood due to the quick heavy breathing and that it should go away in a couple of hours. I was told this at 9:30 in the morning, and it is currently 6:30 at night. I am still expierencing pain in my calfs, a tingling feeling in both knees, and some new pain in my thighs. There is also a discomfort feeling, painful from time to time, where I have been diagnosed with tendinitis in my left elbow. I haven t had problems with asthma before like this, and have had no past medical issues. Is this normal for the leg pains and the pain in my elbow to still be there 10 hours after the initial start of the attack? Also, when I stand, I start to feel weak and a little dizzy but am fine when I am sitting down. Doctor: this can be due to electrolye imbalance. get eletrolye level checked. & take tretment accordingly."
},
{
"id": 226354,
"tgt": "Medical abortion done, have slight bleeding now, was prescribed Nuva ring. Can I insert it now?",
"src": "Patient: I had a medication abortion three days ago. It was successful and I m awaiting my follow up appointment in about two weeks. I was prescribed a Nuva Ring for birth control . I read Nuva Ring can be started during your period within the first few days. Counting day one of bleeding as one and after seven days the Nuva Ring would be effective. After having the abortion is the same true? Id like to insert it now. I m bleeding still but very lightly. Doctor: Hello. Thanks for writing to us. After an abortion, you can get a check ultrasound scan done to make it sure that the uterine cavity is empty. Then you can get it inserted for contraceptive purposes. You can get it inserted now with the light spotting. I hope this information has been both informative and helpful for you. Regards, Dr. Rakhi Tayal drrakhitayal@gmail.com"
},
{
"id": 204689,
"tgt": "How can aggressive behaviour and severe anxiety be treated?",
"src": "Patient: Hello, My daughter is 25 with behavior challenges. She is aggressive moderately. She pushes, scratches, and is uncooperative at times. She is mentally retarded autistic, and has anxiety issues. She takes medicine prescribed by her psychiatrist. Nothing is working. She also has cerebral palsy and hydrocephalus. Is there a medicine for this. This week she was on the ground outside the store, she did not want to get in the car, maybe because I did not get her a cake. Doctor: Dear caregiverpatients of mental retardation with behavioural issues can be treated with medications . if patient is behaviorally stable , patient cm atleast self manage herself and also makes life of caregivers more easier Regards"
},
{
"id": 135162,
"tgt": "Does arthritis cause hip pain?",
"src": "Patient: Hi, I ve had an x-ray of my hip and am told no fractures, but possible arthritis. But I experience great pain in the hip and about to see orthopedic dr. for closer examination as feels like something unaligned. Figure to get diagnosis and start physical therapy. Sound about right course. Could it be something else, though? Doctor: Hi.Arthritis can cause inflammation of the bone that results in reduced space at the joint, and so possible friction between the bones in the hip (which is a ball and socket joint). So treatment of arthritis should be the primarily approach, physiotherapy is secondary.Best wishes."
},
{
"id": 39881,
"tgt": "Is oral LP dangerous and what is the cause?",
"src": "Patient: hi doctor, my cheek was swollen inside with a white color path and a small white color lump. i consulted a doctor and he said its Oral LP. can you please tell me if it is dangerous and is the cause due to a bacteria or viruse?my email id is YYYY@YYYY Doctor: Hello,You are suffering from oral lichen planus.Its cause is known and is thought to be due to autoimmune proces.Very frankly speaking it has no known cure but various treatments are offered which kep it under control.As compared to cutaneous lichen planus,oral cutaneous planus is difficult to treat sometime and oral ulcers and lesions may persist for many years.Thanks"
},
{
"id": 66168,
"tgt": "What is the cause of red painful lump on the side of my breast?",
"src": "Patient: Hi, I had a red lump on the side of my breast, it was quite painfull when I pressed it. So I squeezed it and it turned to a purple colour, a bit of what I thought was pus came out and a lot of blood afterwards, everytime I squeezed it more blood came out. I have no idea what it is , but it's still there. Doctor: Hi! don't worry this is related to skin only and the breast tissue proper is not affected at all!This is very common and people get very concerned about such bloody nodules; I consider some possibilities in such cases as I have seen so many biopsies of such pathology; possibilities are:1. a small infected inclusion cyst or folliculitis2. a small hemangioma getting infected3. a small / thin sinus on the skinAnd the discharge, though bloody, is nothing but pus!Therefore you have to see your doctor for a physical examination and some microbiological tests and to plan a treatment protocol!regards,"
},
{
"id": 37096,
"tgt": "Does ESR is ~32. Dengue Serology test was indeterminate confirm a dengue fever?",
"src": "Patient: My father is suffering from a) mild fever, body ache - first fever 10 days back but had disappeared and have again started last 4 days, b) blood count was all normal 10 days back when fever started, but platelet counts has declined to 85 (last 10 days) from 125 but has inched up to 95 last 24 hrs, c) ESR is ~32. Dengue Serology test was indeterminate. Doc suspected dengue but now starting to think it could be some thing else. Any suggestions/ possibility? Doctor: HelloYour father is having fever from 15 days with a gap of 4 days .Serology test for dengue is negative ( indeterminate ) . The counts of platelets is not a parameter for dengue .However, you didn't mention the other symptoms and type of fever , but may be due to these possibilities , these include:1 Typhoid fever , diagnosis can be confirmed by Widal test , serology and stool test .2 Tuberculosis , is another most common cause of such fever ( long standing fever ,low grade ) . ESR is further an indication of tuberculosis or any other chronic infection.3 Jaundice , this is most common cause beside above mentioned reasons for fever . Get in blood for HBASG for hepatitis .However , there are so many other reasons of such fever , like UTI , bronchitis , sore throat .But in my opinion consult a physician and get his opinion.Good luck"
},
{
"id": 203714,
"tgt": "What is the permanent cure for cracked foreskin?",
"src": "Patient: hi, i have a red inflammed area on my foreskin that cracks and itches. it appeared overnight when i was not sexually active, i have had it for months and months now, i have applied various types of oils and ointments such as antiinflammetery cream and hemp, and paw paw ointment, help please Doctor: Hi,From history it seems that your problem might be due to infection resulting from collection of smegma under fore skin.Due to local poor hygiene and deposition of smegma giving rise to crack or infection.Clean the part and remove smegma gently and apply antibiotic cream locally.Make a habit to clean the glans by averting foreskin while taking bath.Ok and take care."
},
{
"id": 80753,
"tgt": "What is the treatment for chronic cough?",
"src": "Patient: I have a chronic cough. a I have a chronic cough as doctors haveprescribed a full regimin of omnaprozole which I took and the result was my cough did not go away but instead tore up my stomack and instestines, gave me severe diarrhea and very painfu hemorhoidsl I never have hearburn. I use the nettie pot twice dAILY AND IT SEEMS TO HELP, BUT STICKY THING, CANDY, SOFT DRINKS, MILK GIVE ME THIS MUCUS WHICH IN TURN MAKES ME COUGH iF A PIECE OF LETTECE LAND ON MY THROAT AND STICKS i HAVE PARASYMS OF COUGHING aI have a chronic c Doctor: Hello dear, thanks for your question on HCM. I can understand your situation and problem. In my opinion we should rule out pulmonary causes for your chronic cough.Chronic lung infection or bronchitis can cause chronic cough. So better to consult pulmonologist and get done1. Clinical examination of respiratory system. 2. Chest x ray. 3. PFT ( pulmonary function test ). Chest x ray is needed to rule out lung infection. PFT is needed to rule out bronchitis. You may need inhaled bronchodilators and antihistamine drug. First diagnose yourself and then start appropriate treatment. Don't worry, you will be alright."
},
{
"id": 101360,
"tgt": "Can bruises in skin cause symptoms like stomach issues & allergy?",
"src": "Patient: My 10 year old daughter keeps getting bruises on her arm. They look like purple tiger stripes. She is currently having some stomach issues that are being treated. She is also allergic to pork and seafood. She also has asthma. Can any of this be related to the bruises Doctor: Hello.Thank you for asking at HCM.The most important thing that I would like to ask you is whether they itch.If they itch, I guess, they are urticaria, which is common with other allergic disorders lie food allergy, asthma, etc.If they do not itch, I would suggest you to show them to a pediatrician or dermatologist. Hope this will help you.Wish your daughter the best of the health.Regards."
},
{
"id": 67658,
"tgt": "What causes a painful lump on my buttocks?",
"src": "Patient: I ve recently notice it would hurt to sit down, I thought I bruised something but when I knew after the next day it was a more serious pain I felt a ball like a marble size under my skin on my left bum cheek it felt like a bruise until I found that. I hope you can answer my question. Thanks Doctor: Hi,It seems that there might be having ingrown hair follicle infection and now forming abscess on the part.Go for one oral antibiotic medicine course for 5 days.within a day or two lump will be subside.Keep local hygiene clean and dry.Ok and take care."
},
{
"id": 218874,
"tgt": "Is greenish discharge from the breasts during pregnancy a concern?",
"src": "Patient: I am 37.6 weeks pregnant and have been pumping my breasts with my OB doctor s instruction. Tonight when I pumped the colostrum that I expressed had a greenish/gray color. I am a Postpartum nurse and have never seen this color of colostrum in any of my patients. Is it something I should be concerned about or is it due to something I have ate? It was this color from both breasts. Doctor: hello...thanks for trusting the health care magic doctors for your health related issues.greenish discharge from the breast is not normal during postpartum period. there is something goes wrong inside ur body, so u have to concerned about it.in my suggestion visit nearby gynecologist,he/she can examine u directly and advice accordingly."
},
{
"id": 178843,
"tgt": "What type of food can be given for the baby?",
"src": "Patient: Dear Doctor I would like to know about my son. He is not eating properly only breast feeding. Also he is not walking himself now he is 14 months. Can i give him mutton soups etc., what are the major food i can give to him. Looking forward for your valuable reply. Doctor: Thank you for posting your question.The diet of a 14 month old child should comprise of everything that is included in a basic family meal, in short the baby should ideally eat whatever other family members are eating, as long as its all homemade.He may be given all vegetables, fruits. You may boil and mash the vegetables and then add some salt to make it palatable.You may add rice, pulses. You may slowly introduce strong proteins like egg white or chicken/meat.The consistency should initially be kept curd like and gradually thickened. Alone breast feeding would not be sufficient for the growing needs of your child at this age thus complementing it with other foods is very necessary at this point.You need to use the method of trial and error to know and understand what kind of foods suit your baby. Any more delay in establishing proper complementary diet would have adverse consequences on his nutritional status.If you need any more information, do write back."
},
{
"id": 158641,
"tgt": "Diagnosed with esophageal cancer. Spread to lungs, kidneys. His condition? Suggestions?",
"src": "Patient: My dad just received a call from his doctor and he didn't ask any questions nor does he quite understand. He found out on Friday from a biopsy that he has esophageal cancer. He had a CT scan this morning and the doctor called and told him it's spread to his lungs and kidneys, but he doesn't know how much, what stage, or anything. In fact, he thinks it's only a little on his lungs. This is bad, isn't it? Doctor: Hi, Spread of oesophageal cancer to kidney is a rare possibility. Anyway as the disease has already spread to lungs the prognosis is not so good. This is at stage IV . Chemotherapy is the option right now. In case there is oesophageal stricture palliative radiation by brachytherapy can be given. Consult your treating oncologist."
},
{
"id": 63976,
"tgt": "Lump under armpit which looks like part of vein, but no history of varicose vein. Do you suggest to meet doctor?",
"src": "Patient: HI. My partner has a small lump under his right armpit which is green and looks like a part of a vein. He does a lot of physical work in his job as a gardener but has no other varicose veins and there is no accompanying discomfort. is this something he should get checked out by his GP? Doctor: Hi, dearI have gone through your question. I can understand your concern.He may have enlarged axillary lymphnode. It can be due to reactive hyperplasia, tuberculosis, lymphoma or metastatic carcinoma. He should go for examination and if needed go for fine needle aspiration cytology or biopsy of that lump. It will give you exact diagnosis. Then you should take treatment accordingly.Hope I have answered your question, if you have any doubts then contact me at bit.ly/Drsanghvihardik, I will be happy to answer you.Thanks for using health care magic.Wish you a very good health."
},
{
"id": 14582,
"tgt": "What does red skin rash after body hair removal indicate?",
"src": "Patient: Hi, I'm a 22 year old male. I have alot of body hair and I recently buzzed all the hair off my chest. It has grown back over a cm in length and now I find that I am getting some sort of rash made up of red spots no more than a cm in diameter. I have no other symptoms that I can place. Doctor: Hello. Thanks for writing to us at healthcaremagicI will keep a possibility of seborrheic dermatitis. It usually presents as dry, scaly, itchy and red patches.If I was the treating doctor I would have asked you to apply a moderately potent topical steroid e.g fluticasone propionate 0.05% cream, twice daily for the patches.An oral antihistamine e.g cetrizine 10 mg once daily would provide you symptomatic relief from itching.Topical steroids are prescription medicines. I would suggest that you visit a dermatologist for a confirmatory diagnosis and appropriate treatmentRegards"
},
{
"id": 6071,
"tgt": "32 year old, planning for baby. Have hypertension, hiatus hernia, had an abortion, ovulation stopped. What to do ?",
"src": "Patient: Hi,I am 32 year old lady. Right now i m suffering some fertility problems. 2 years back, i was first time pregnant ,unfortunately i had to terminate pregnancy at 6 months as it was IUGR baby. now i want to plan for baby....but i have hypertension and hiatus hernia so i need to take medicines for the same. problem is now my ovulation stopped from last 6 months....now what to do? Doctor: Hello. Thanks for writing to us. If your uterus and ovaries are normal with patent tubes, then ovulation can be induced with the help of ovulation inducing drugs like clomiphene. You can get these prescribed from your gynecologist. I hope this information has been both informative and helpful for you. Regards, Dr. Rakhi Tayal drrakhitayal@gmail.com"
},
{
"id": 97172,
"tgt": "How to treat the back pain caused by an accident?",
"src": "Patient: Hi I ve been going to my doctor for back pain since I was 16 after I had a ATV crash and got hit buy a truck on my left side they did ex rays and could not see nothing wrong my pain when away for a few years I was 18 and got a Job iron scraping I fell while carrying heavy weight on loose footing and hurt my back the same part I didn t go to see the doc that time I had 2 other injeries since all to my left side.for the past 4 years I ve been in bad pain I ve try therapy and injection s and it has done nothing for me but east my money and time. I got a MRI and they said I got birth defects putricals and dehydrated disck es but my pain should be mild. Now I m 26 I ve ate so much ib profin it fucked up my stomick no help for so long I turn to the street doctor stared buying pain killer s I got my life back for a while as fare as working and walking but my tolerens got to high so I was working to be pain free .now I m not taking pain killers I had to take methadone to get off the now I m not working I m losing every thing in cluding my wife and kid. I m not on drugs 4 the past 4 month I m still in pain in my back and mently and my left ball has Ben swelt up but anyway I feel like just being done with life now that all is lost and was wondering if it is common for people in cripiling pain to shoot them selves Doctor: Pain killer will provide only a symptomatic relief, it will not treat the condition. I have seen a lot of patients with low backache of long long duration.You may be having a disc prolapse or something like that.I would advise you to do back stretching exercises and physiotherapy. I have seen a lot of patients who are benefited by physiotherapy.Sometimes when pain is very high, you can take analgesic like Tab Dicloran but never forget to take Tab Rantac or tab Pan 40 with it, as all the pain killers causes gastritis. I hope physiotherapy and stretching exercises will help you."
},
{
"id": 6024,
"tgt": "Trying to concieve, folic acid tablets, follicle ruptured, small periods, pregnancy symptoms. Pregnant?",
"src": "Patient: Hi, I am 26 yrs married and trying to get pregnant, I had my last periods on 5th April, then I have gone through follicular study and then I had hCG 5000 iu on my 15 th day (on 19th May) in consultation with my gynac then after it was found out the follicle (19mm) has been ruptured...so i was hoping to be pregnant for sure... before 3 days of my due date the home pregnancy test found out negative and on 5th June night i found some little blood and then next day a quite heavy flow and after that very small (menses lasted for only one and a half day) i taking folic acid tablet everyday..with good diet...i am wondering whether pregnant or not coz still have the symptoms like sore breast, bad lower back pain , headache.. it really alarming thing for me...please help me for my queries Thanks Doctor: Hello. Thanks for writing to us. Since you had a heavy bleeding even if it was for only one day, the chances of pregnancy are minimal. You need to try again in the next cycle as the chances of conception in a single cycle are only around 20% in spite of ovulation. I hope this information has been both informative and helpful for you. Regards, Dr. Rakhi Tayal drrakhitayal@gmail.com"
},
{
"id": 155727,
"tgt": "Should we fear malignant melanoma from the mole on areola?",
"src": "Patient: My daughter has a new mole on her areola. She has tanned against my wishes (she is 19). Her young cousin had a malignant melanoma on her groin at age 15. she is going to see a dermatologist. Is a mole in this location more risky for cancer than other moles on the body. Doctor: Thanks for your question on HCM.Moles are usually benign skin pigmentation. But at certain places like face, forehead, arm, feet, hands etc they have more chances of turning out in malignant melanoma. The reason for this is exposure to ultraviolet rays.So in my opinion no need to worry for mole on areola as it is not exposed part to the ultraviolet rays.But if you hsve family history of malignant melanoma it is advisable to remove it surgically."
},
{
"id": 117878,
"tgt": "Should I continue taking hydroxyurea for Essential Thrombocytosis?",
"src": "Patient: Next mo. I'll turn 65. I suffer from ET (Essential Thrombocytosis). I was made aware of the possibility of this condition in 1995 during a routine office visit by my cardiologist. The cardiologist couldn't get a definitive blood count after a routine CBC was taken and referred me to a hematologist. Several tests, including bone marrow aspiration, were taken to determine what was going on with my blood. Turned out to have a blood disorder called ET rendering my platelet count in the low one-millions. I was immediately put on Anagrelide which I stayed on for approx. 4 years. But because of severe side affects, specifically palpitations which this hematologist didn't seem to consider serious, given my condition, I sought the services of yet another specialist. I was introduced to hydroxyurea in high doses which brought about a whole different set of problems. Where the palpitations were alayed, I began noticing a darkening of my nails, feet and hands, and severe bouts with dizziness, as well as lower back pain, infrequent fatigue, and a host of other side affects caused by this medication. I was scheduled for another bone marrow aspiration and it was determined that my bone marrow was hardening. I discussed this with the doctor and advised him that I would no longer take such a high dosage of drugs, thereby having my dosage reduced. I was not very pleased with this doctor and consequently did some extensive research and ended up with the specialist I now have. Unlike with the previous specialists, this specialist informed me of other alternatives, none of which of course were holistic, which were explained to me and gave me an opportunity to choose which method of treatment I would rather have. I opted to stay w/the Hydrea. I have now been on the Hydrea for 11 yrs. As with all previous specialists, and with medication, as I am monitored every 3-4 months, my platelets have continued to be in the mid-500s to 700s. I currently take 2 500mg hydrea twice a day. I have read the comments from others who have this condition and we all have similar concerns. . .lukemia, stroke (if med stopped), etc. What can I do, who can I turn to? I want to get off this drug, but am fearful of the outcome. Doctor: Hi,Welcome to health care magic and thanks for asking.After going through the clinical and drug history, I seem to agree with you.Hydroxyurea, if taken for a long time does lead to increased risk of leukemia. However, if stopped, increased platelet count could lead to increased blood viscosity and chances of stroke are high.Options include regular plateletpheresis or changing over to anagrelide for short periods in between.Other options includes reducing dose and close follow up for development of leukemia and having an option to treat at an early stage.You can also add aspirin to reduce the risk of thrombosis.Any further queries, happy to help again."
},
{
"id": 84538,
"tgt": "Are the metrogyl, zerodol, becosule right medications for ulcer?",
"src": "Patient: Hi Dr Chakraborty, I am suffering from ulcers in my mouth since last couple of days. A friend asked me to take metrogyl 400, zerodol sp, rablet 20, becosule and use glycoseptol mouthwash, candid mouth paint and clenora gel. Is this the right medication? Doctor: HiYes,these are the right medications for mouth ulcers. Zerodol sp itself can cause mouth ulcers, gastritis and can be avoided.Candid mouth paint can also be avoided since aphthous ulcers are not caused by fungi. Overmedication should be avoided at all costs to avoid unwanted side effects.Hope I have answered your query. Let me know if I can assist you further. RegardsDr.Saranya Ramadoss, General and Family Physician"
},
{
"id": 224925,
"tgt": "Can the medicine Primolut handle unplanned pregnancy if taken within 72 hours?",
"src": "Patient: Hello I am on primolut n to regulate my periods....... 2 days back I finished my peiods and my husband yesterday evacuated in side me I took primolut n 2 tabs as an emergency pill will it work .....or can I be pregnant or use anyother contriceptive within 72 hours plzz plzz advice we dnt want the baby now ........ Doctor: Hi, You had your last mentruation which ended 2 days back and unprotected sex occurred 1day back so you were in safe period hence chance of pregnancy is least then again you took progestin only pill as emergency contraception. So don't worry chance of pregnancy is negligible and for your irregular menstruation continue medication as prescribed by your doc.Thanks n regards"
},
{
"id": 110440,
"tgt": "What causes pressure in chest and back relieved by burping?",
"src": "Patient: I keep having preasure in my chest and back I get some relief when I burp. Don't want to be stupid and just think gut stuff is going on if my heart is trying to get my attention. I slept well last night but it is coming back now. I even did 30 minutes on my excercise bike this morning. I am a woman 50 years old. No insurance What do you think? Doctor: Hi, thanks for posting your concern in the HCM.I think the pressure in your chest and back are symptoms of bloating. This may be due to peptic ulcer disease or non ulcer dyspepsia, or some intestinal or gall bladder or liver pathology. History regarding your bowel habits, is very important in this regard. 1. Do you have irregular bowel habits? If yes, is it predominantly constipation or diarrhea?2. Have you ever had blood in your stool or colitis?3. Are you having nausea and/or vomiting?4. Do you have abdominal pain?5. Do you have any known comorbittYou also need to have the following tests done-Complete blood count, urea, creatinine, electrolytes, fasting TSH, LFT, amylase and lipase.USG of whole abdomenFor the time being I would suggest you to have low fat diet, avoid spices, alcohol and carbonated beverages.I also recommend my patients to have Simethicone 60 mg tablet 3-4 times daily after meals and at bedtime for 7 days,and Enterogermina oral suspension twice daily for two weeks.Please also consult your local gastroenterologist for further evaluation.For any further questions,please write back to me.Regards,Dr. Kaushik"
},
{
"id": 201103,
"tgt": "What causes painful foreskin of penis?",
"src": "Patient: Hlo sir .....I am navdeep Singh from India Punjab ...I HV penis pain ...I meant to say ...that my penis upper skin is paining....when I do intercourse or master bat ..in DAT time when my penis stimulate full ....my penis upper skin wasn.t going to back full so plzzz tell me the solution bcz ...I want to enjoy my sex life..! Doctor: Hi,From history it seems that you might be having tight foreskin giving this problem.This is called Phimosis and it is advisable to go for Circumcision operation.If foreskin is very tight then problem of Para-phimosis is likely after sex or during masturbation will lead to emergency problem.Consult surgeon and get examined.Ok and take care."
},
{
"id": 94973,
"tgt": "Pregnant. Bright green discharge refers to? Why did I have heart murmurs?",
"src": "Patient: Hi i am 26+4 weeks pregnant with my 2nd child and have had problems from the start of pregnancy ie alot of cramping which has been bearabe and not to bad and bleeding, i am A Rhesus D negative and have already had 2 lots of Anti D in this pregnancy so far. I have been told my cervix is only 2.6mm and could poossibly have a premature labour . 2 weeks ago i went to gp with alot of discharge that was a very bright green. I had a swab and was diagnosed with Group B Strep , i was told to inform my midwife which i did and nothing come of it. My results show: Pus cells ++ Epithelial cells +++ B haemolytic Strept Group B +++ Yeasts and anaerobes not isolated A few days later i went to see my consultant regarding my heart murmur for a check up and he saw the results and sent a urine sample to be cultured and was also told i would need IV antibiotics when in labour, but no one can tell me what will happen if my labour is as quick as my first, i had no pain and then had a long intense cramp which lasted 10 mins i went to the hosp straight away and 13 mins later my first child was born, i received a call from midwife when the results were back saying i needed to go on antibiotics and was told to go on a 7 day course of amoxycillin which i have completed and my oddly green coloured discharge has just got worse, i called gp this morn and he said to see the practice nurse and speak to her. Since this morning i have had terrible pains on the right side of my bump quite low down and i really painful when sitting standing and also laying down. I cannot seem to get in touch with my midwife all day and am worrying as to go to the hospital not. I dont want to waste anyones time. What would you suggest as i am in alot of pain. Doctor: Hello. Thanks for writing to us. The infection you are having needs to be treated as it can further precipitate a preterm labor. The antibiotics will help you but these should be taken according to the culture sensitivity report. For pain, you can do a hot fomentation in the area and attend an ER for proper management. I hope this information has been both informative and helpful for you. Regards, Dr. Praveen Tayal drtayal72@gmail.com"
},
{
"id": 60216,
"tgt": "What permanent solution is available other than surgery for removal of gallbladder stones ?",
"src": "Patient: wanted to discuss with someone regarding Gallstone problem. My husband is 45 years old. He has been diagonised with gallbladder stones (multiple). What are the options available for a permanent solution other than surgery as there have been instances where people have undergone gallbladder operation but the problem still persists. Doctor: Hello. welcome to HCM. Treatment of already formed gall stones is nothing but surgery. You are taking note of instances where people are not treated by surgery but there are umpteenth number of cases where cholecystectomy (gall stone surgery) has healed people. Laproscopic or open surgery is a safe option. Long standing gall stones may complicate issues by causing superimposed infection, perforation and even malignancy. So please be wise and judgemental for your benefit. take care"
},
{
"id": 117241,
"tgt": "Suggest treatment to reduce esr level",
"src": "Patient: Hi doc! I had all my blood tests done. Sugar fasting, pp, cholesterol. Everything is just fine. My blood pressure is good by my ESR count was 46. I don t have any problem. I have hernia which is not giving me problem. My feet get swollen up by the evening as I keep sitting on a chair with my feet hanging for about four hours. What should I do to reduce my ESR level. Thanks Doctor: Hi, dear. I have gone through your question. I can understand your concern. you have esr 46. Which is higher than normal. There are many causes of high esr. Inflammation, tuberculosis, anemia, autoimmune disease, paraprotienemia, malignancies and many more can leads to high esr. You should consult your physician and investigate for your high esr level. Then take treatment accordingly. Hope I have answered your question, if you have doubt then I will be happy to answer. Thanks for using health care magic. Wish you a very good health."
},
{
"id": 45376,
"tgt": "I am on 500mg of apo-clarithromycin for one more day. I am scheduled for an IUI tomorrow. Would there be a problem?",
"src": "Patient: I am on 500mg of apo-clarithromycin for one more day. I am scheduled for an IUI tomorrow. Would there be a problem? Doctor: Hi, In my opinion one dose will not create any problem, plus clarithromycin is considered safe in pregnancy. All the best,"
},
{
"id": 61714,
"tgt": "What causes lump on forearm?",
"src": "Patient: hello, i am a 27 year old female, healthy, with a history of no major medical problems. twice in the past couple months, a large lump has appeared on my forearm (once on the outside of the right arm, and then on the inside of my left arm) which is then surrounded by a huge bruise within several hours. both times, this occured without any trauma to the affected area. any ideas why this is happening? thanks! Doctor: Hello hope the answer helps you.As you mention that without prior trauma noticed bruised lumps in arm twice.I think a detailed histroy as for any bleeding diathesis as maleena,hematochazia needs to be sought but otherwise you are healthy fit person and any bleeding disease in family and not on any anticooagulants then better you sit and wait to see it resolved out as not associated with fever to clue anywhere.If stll happens again in future then needs consultation"
},
{
"id": 61701,
"tgt": "Suggest treatment for hard painful lump on the arm",
"src": "Patient: Shot up MS Contin yesterday and almost immediately got a large golf ball sized lump. Is not growing in size nor is it red, black or blue. Just very tender. It hurts to extend or flex my arm. Today (the next day) I have a dull ache shooting down my arm into my pinky fingers (feels like it s running down the bone). What could this be ? Doctor: Hello hope the answer helps you.As you have a large sized nontender lump which hurts on flexion and extension.As sir you donont have local inflammatory signs and not running fever so it is unlikely to be abscess,boil or furuncle as having pus.It could be any lipomatous swelling as lipoma,or containing nerve fibre as neurofibroma till proven or ganglion like cystic lesion along synovial tendon sheath.Dull ache I guess is due extrinsic compression over nerves going in arm and foream bcoz of size.You dont worry if not associated with any systemic upset as Gastrointestinal tract or respiratory better consult general surgeon sir hope the answer lessen your anxiety"
},
{
"id": 151991,
"tgt": "I have acidity and head spinning problem",
"src": "Patient: I am aged 59. Height 5 2. Weight 117lbs. I have acidity and head spinning problem. Doctor: Hello.welcome.Head spinning or vertigo as it is called can be due to cervical spondylitis,ENT prolem,hypertension etc .Best would be to consult physician and get yourself properly diagnosed and treated.Good luck."
},
{
"id": 137273,
"tgt": "Suggest remedy for painful bruise on the thigh",
"src": "Patient: I fell off my mountain bike and landed quite hard on a rock that bruised my upper thigh. I also have small bruise just blow my pelvic bone. The thigh bruise is especially painful when I bend my leg. I can t pull my bruised leg toward my chest. I iced it and took some ibuprofen. I feels a bit unstable going down stairs. Doctor: Hello,I have studied your case, Due to blunt injury there can be hematoma collection .The bleeding that occurs result in hematoma collection. Over a period of time, as hematoma resolves it cause local skin symptoms around the hematoma. Continue ice application, and thrombhophobe ointment may help and take rest.You can do x ray to see for fracture, giving supportive splint will give early recovery.If required you may need to do blood investigation for prognosis and recovery.Take precaution to avoid infection around hematoma.Hope this answers your query. If you have additional questions or follow up queries then please do not hesitate in writing to us. I will be happy to answer your queries. Wishing you good health.Take care."
},
{
"id": 33488,
"tgt": "What causes sore throat, fever and palpitations?",
"src": "Patient: Hi, may I answer your health queries right now ? Please type your query here...Hi. for the last couple of days i have suffered from a sore throat, slight cough, aching neck, fatigue, breathlessness, dizziness, temperature, fever and fast pulse. What could this be? Doctor: Hello dear user!Thank you for using HCM for sharing your concerns!I've gone through your query and I understand your concerns.All these symptoms you are describing are indicative for an upper respiratory infection. It may be viral or bacterial and to determine this we have to do a complete blood count to see WBC.Fever, sore throat, cough are exclusively symptoms of an infection. Palpiations and fast heart rate is normal since you have fevers. Neck ache and hardening, dizziness, light intolerance, and red spots in chest or body are rarer symptoms that indicate a more severe condition and which must be treated immediately, and this is meningitis.I don't intend to worry you, and you should not do that, but some immediate things you should do for now is:- Go to your ER and do a complete blood count and leukocytes. If your WBC are high you have to take antibiotics and better injections of through your veins for a faster healing. If nothing suspicious for bacterial infection is found then you can stay home and take there your medications as described from your family doctor.I hope you will be better soon.I wish you a good health!DR ERIOL."
},
{
"id": 172120,
"tgt": "How much Bacigyl-N suspension should be taken for diarrhea?",
"src": "Patient: I am in India with my 4 childrens and 3 of them with diarrea. The 9 year old yesterday was going very often to toilet. The other 2 are pretty ok, playing and eating but still with diarrea for a long time, now like 10 days. Sometimes it looks that they get better but is a long time now. I've just bought Bacigyl-N (norfloxacin and Metronidazole) suspension. Is it ok to give them this medicine? and how much it will be the dose? Kids 9 years old, 6 years and 2 years and a half.I would really appreciate your time and your answer. Thank you!Candelaria Doctor: Hi...Thank you for consulting in Health Care magic.It seems your kid is having viral diarrhoea. Once it starts it will take 5-7 days to completely get better. Unless the kid's having low urine output or very dull or excessively sleepy or blood in motion or green bilious vomiting...you need not worry. There is no need to use antibiotics (LIKE NORFLOXACIN AND METRONIDAZOLE) unless there is blood in the motion. Antibiotics might worsen if unnecessarily used causing antibiotic associated diarrhoea.I suggest you use zinc supplements (Z&D drops 1ml once daily for 14 days) & ORS (Each small packet mixed in 200ml of potable water and keep giving sip by sip) as hydration is very important and crucial part of treatment. If there is vomiting you can use Syrup Ondansetron (as prescribed by your paediatrician).Regarding diet - Avoid fruit juices as they might aggravate diarrhea. You can give zinc supplements & ORS apart from normal vegetarian porridges & soups.Hope my answer was helpful for you. I am happy to help any time. Further clarifications and consultations on Health care magic are welcome. If you do not have any clarifications, you can close the discussion and rate the answer. Wish your kid good health.Dr. Sumanth MBBS., DCH., DNB (Paed).,"
},
{
"id": 172633,
"tgt": "Suggest treatment for seizures and mood changes in a child",
"src": "Patient: my 12 year old son is a victim of 'right front lobe..' epilepsy' since 3 year and using 'Encorate leq.' three times daily but it is not controlling 'seizure',and day by day he is going angry 'frequent mood change' how do i do something better to get rid of that uncivilized disorder?is it curable permanently,because my son has a dream to be an 'Indian Army Officer' in future.may i give him assurance for it? Doctor: Frontal lobe epilepsy is difficult to treat...associated with un social behavior... Very minimal chance of spontaneous cure...he has to be put on continued anti seizure drugs"
},
{
"id": 110699,
"tgt": "What is the cause for lower back pain?",
"src": "Patient: hello,my name is janis,i have had lower back pain for the past 4 days and now it has gotten into my left hip and also when i lay down it is hard for me to lift up my left leg,i have to use my hands to do so,and also when i get into a car i got to do the same thing,i dont know what is wrong,i went to the hospital and all they done was give me a shot for pain and sent me home and that did not even work,what can i do.never mind i have no credit card,sorry Doctor: Hi, thank you for posting.I have gone through your query and I understand your concerns.The greatest probability is that you suffer from arthritis. Arthritis is a disease that causes painful inflammation and stiffness of the joints.To treat this condition you can take anti anti inflammatory medicines such as Advil.Massage, chiropratic procedures and physical exercises can also help.To confirm the diagnosis you need a spinal and pelvic X-ray, a fibrinogen test and a complete blood count.Spinal Ct-scan is necessary in order to rule out herniated disc.You should contact your neurologist to discuss about the diagnosis and the right treatment.Regards.Dr. Behar."
},
{
"id": 118030,
"tgt": "What causes leg/back pain when having high RBC/RDW and low MCV/MCH?",
"src": "Patient: Hi, my 7 year old daughter has high RBC and RDW but low MCV and MCH. Her hemoglobin is good at 11.5. We are of African decent. She does complain of leg and back pain often. Should I be worried about sickle cell disease or thalasemia? or even a blood cancer? the pediatrician said she has hemoglobinopathy but is doing further testing to find out what exactly. Doctor: High rbc with low mcv is seen in beta thallasemia minor commonly. So just go for hb chromatography. It will give exact dignosis of hemoglobinopathies. If ur hbA2 level is above 3.5 then its thallasemia minor. so go for hb chromatography."
},
{
"id": 128434,
"tgt": "Suggest medical treatment for sprained and swollen ankle",
"src": "Patient: 55YO male seems to be sprained ankle with swelling from twist injury, but feels much better with walking and activity. After sitting an hour or so much worse pain when getting up and almost can t bear weight for the first steps 3 days - using ice and Advil and elevate when can. Doctor: Dear patient Ankle sprain is less likely since pain increases on walking and activity in sprained ankle. where exactly is the pain? if it is within ankle and improves with activity inflammatory condition is more likely like early arthritis. If pain is in the bottom of heel bone likely disease is plantar fascitis. you need to be examined by orthopaedic surgeon to conform diagnosis. Meanwhile start tab zerodol sp twice a day for pain relief."
},
{
"id": 33370,
"tgt": "Suggest remedy for cankor sores in mouth",
"src": "Patient: Hi Doc,I hope this works. I have a question about my mom. She is in her mid 50 so she's in menopause. She's had this problem before when she was younger but hardly this frequent and it last for days too. She has those cankor sores in her mouth ie lips, gums, even underneath or side of her tongue. I know for certain they are not cold sores, she does not have herpes. She claims that she gets them whenever she eats fried foods, but I still feel that there may be more to it. The good thing is that it goes away, but it takes days to weeks and it really worries me because it can be so painful that she finds it difficult to eat or talk. I want to know whether there are other underlying problems indicated from her constant cankor sores. About 2 years ago, she had H.pylori infection in her stomach which is now taken care of, she doesn't take other medications except antihypertensive meds. She did an endoscopy about 8 months ago and she's healthy. Do you know anyone or of any similar situation as my mom for having frequent cankor sores for reasons other than \"bad food habits\" because I am find it out of ordinary to be getting them because of the occasional fried foods. She eats relatively healthy and she tries to stay away from fried foods so it's not a frequent diet. Please let me know. Thank you!! Doctor: Hi..Can understand your concern..As per your complain Canker sores also known as Apthous ulcers occurs due to a number of reasons although the exact reason for its occurrence is unknon still..Although a few factors a considered as causative agents for canker sores, which are:-Acidic, sour and spicy food..-Nutritional deficiencies like Vitamin B12,cfolic acid and iron.-H.Pylori infection.-Allergic reactions.-Hormonal changes like during mensuration and menopause.-Stress.-Poor Oral Hygiene.-Immunocompromised conditions like HIV, uncontrolled Diabetes.-Radiotherapy and Chemotherapy for cancer.-Denture wearing.-Allergy to certain toothpastes etc.Although oily food has not been found to cause canker sores but it taken very hot and with spices can be a factor..H.Pylori infection which causes peptic ulcers in stomach has been found to cause canker sores and as your mouth has been exposed to it you should get a checkup done to find out that whether it is still becoming active at times and causing canker sores.She can takes nutritional suppliments like a multivitamin tablet daily for a month to fulfil for any nutritional requirements by the body..Avoid hot and spicy and sour foods..Maintain a good oral hygiene..Use alcohol free mouthwashes..Do warm salt water rinses on daily basis once a day.In case if any underlying disease is ruled out you should get it treated and it can be helpful in reducing the frequency of canker sores..Hope this information helps..If you find the answer helpful kindly write a positive review and click on found this answer helpful as a token of appreciation..Thanks and regards..Dr.Honey Nandwani Arora."
},
{
"id": 44431,
"tgt": "Pus cells and infection found in semen analysis. Reason and chances of relapse of infection? Pregnancy chances?",
"src": "Patient: We r married since 8 years, but i am trying to conceive last 4 years , its nt clicking. i have undergone laproscopy , doctors first detected tb in the follopian tube , but now the tubes are open , i had done the course of Akt 4, Akt3 for six months. my present doctor is happy with my follicular study report. Now my questions is : my husband has done semen analysis 4 years back and have got it done recently both times pus cells were found in his semen. doctor told some infection,and have given medicines for the same. 1)what could be the reasons for such infection. 2)what r the chances of relapse of such infection? 3)what is my chances of my getting pregnent? 4)If u hold your urge to have sex or release elseway can there be , chances of relapse? 5)what should i do to to help? Doctor: if your treating doctor satisfied with your follicular study report then there are good chances of getting pregnant.The infection in your husband can be treated with specific antibiotics (get culture/sensitivity done).After taking 6months of ATT the relapse chances are very low. To prevent relapse in your husband both of you should maintain strict personnel hygiene."
},
{
"id": 78115,
"tgt": "What causes upper chest and lower throat pain while taking breathes?",
"src": "Patient: Hi, I thought I was having a heart attack last night. Pain in upper chest and lower throat, it has eased this morning but painful if I breathe in or lean over, I have a headache as well, I have been constipated past 2 weeks but no other symptoms. It s very painful and feels like somethings stuck in my lower throat.I can drink and swallow too. Doctor: Hi. I can understand your concern. Ideally ecg and 2d echo should be done first in your case. If both are normal than no need to worry for heart diseases. If the chest pain is disturbing your routine activity then get a chest x ray done to rule out any lung infection or it could be just a muscle pull. Do not lift heavy weightsDon't worry, you will be alright. Hope I have solved your query. Wish you good health. Thanks."
},
{
"id": 182747,
"tgt": "What causes redness and soreness on tongue?",
"src": "Patient: Hello, I have been having some issues with my tongue and gums lately. When I awake in the morning the tip of my tongue is sore then goes away then the sides of the tongue are red and sore now today all of a sudden the upper left gum and gums behind bottom teeth became red, when I rub my tongue across I can feel something and it s red, fine and rashy looking, doesn t hurt but naggy. What could this be? Doctor: Thanks for your query, I have gone through your query.The soreness in the tongue and the gums can be because of herpetic gingivostoamatitis. Nothing to be panic, consult a oral physician and get it evaluated. If i am your treating doctor, I would have suggested you to take topical anesthetic and analgesics like 2%lignocaine and choline salicylate gel (anabel gel) apply 3-4 times daily before food. Consume lot of liquid.I hope my answer will help you, take care."
},
{
"id": 200530,
"tgt": "Suggest treatment for white heads on penis shaft?",
"src": "Patient: Hi, I have a small cluster of what appears to be white heads (6-8) on the shaft of my penis. Two months ago I had a blood test and urinalysis for STDs, all results came back negative. I ve been sleeping with one partner, she recently had a bladder infection and just finished her antibiotics which may have given her a yeast infection. We had sex Monday morning (1a.m) and I noticed the cluster Tuesday morning (1a.m). After we had sex Monday, I didn t practice good hygiene. I didn t take a shower until 2 o clock in the afternoon. I also work out a lot in compression shorts and wear boxer briefs. I wanted to know what this could possibly be. Doctor: Thanks for asking in healthcaremagic forum White spot like lesion may be due to fordyce spots/pearly penile papules which are harmless. So, visit a dermatologist for examination and further management. All the best."
},
{
"id": 131029,
"tgt": "How long does the pain last after a sprain on knee?",
"src": "Patient: About 2 weeks ago I sprained my knee, while skiing. X Ray at the doctor's office was negative for a fracture but now I have feeling that I have damaged soft tissues. How long does the pain and clicking last? Should I go back to see the orthopedic doctor? They suggested MRI as a next step. Doctor: HelloI understand you had a skiing injury and the xray did not show bony fractures. The possibility remains of a tendon or cartilage injury. Do you have a lot of pain and swelling? Do you have any bruising and are you able to bear weight on your ankle joint? If you have any of these symptoms you should go ahead and get the MRI to rule out other injuries. The pain from a simple sprain can last 3 to 4 weeks, but a tendon or cartilage injury could last longer or even require surgery.I hope you feel better soon"
},
{
"id": 201860,
"tgt": "Suggest treatment for severe itching in the groin",
"src": "Patient: hi doctor, last 2 week i went to doctor due to itchiness at my right groin & d doctor prescribed me calamine cream and piriton said that i might had napkin rash or something like that. however, the itchiness became more intense and had spread to other groin! it is so severe that sometimes i suddenly wake up at night and cannot stop scratching until i put ice around d area. somehow the itchiness gone. is there anything i can do to stop d itchiness? Doctor: Hello dear,The symptoms as mentioned in your post suggest that you might be having fungal infection (Tinea) in the groin region.Management includes:1. Keeping the area dry & avoid too much exposure to water or sweat formation.2. Antihistamine preparations like cetirizine to reduce the itching sensation & provide symptomatic relief.3. Application of topical antifungal cream like Clotrimazole to tackle the causative organism.If symptoms still persist or aggravate further, then it will be better to consult a Dermatologist & get a proper examination done to find out the actual cause.So that appropriate management steps could be taken.Also avoid wearing tight undergarments & trousers...as it will cause sweating & predispose to fungal infections.Wishing you a Good Health.Take care."
},
{
"id": 127228,
"tgt": "What can cause pain and stiffness in the shoulder?",
"src": "Patient: For almost a year I have been dealing with shoulder pain. The pain is often a dull achey feeling that causes my shoulder to be uncomfortable and it often feels stiff. Recently the pain has been more jabby at the top at the bottom of my traps. I ve noticed that my shoulder often makes cracking noises when it gets stretched which aren t painful but are new. I also noticed recently that it is incredibly hard for me to raise it straight up despite it being my dominant arm. It gets stuck and hurts too much to raise it more and needs outside help, but my other arm has no problems with being raised straight up and down in the air Doctor: Hello and Welcome to \u2018Ask A Doctor\u2019 service. I have reviewed your query and here is my advice. You may have a situation called shoulder bursitis. To relieve pain use ice compresses, rest, anti inflammatory and pain medication such as Ibuprofen and Codein prescribed by your doctor. All the best."
},
{
"id": 165123,
"tgt": "How to cure hearing loss in baby?",
"src": "Patient: Hi, I am Khalil ,my son age is 1 1/2 month. We are observed that my son is not listens properly. So we consulted to children\u2019s doctor, they told us to done the OAE/ BERA test in KEM pune.We had done this test and report said that his Right Ear: Moderate severe hear loss and Left Ear: Severe hearing loss. They told us to repeat the BERA test after 1months. So my question is there is chances of improving in listening? What is solution for this and causes? Regards, Khalil Doctor: The ability of hearing is very important in development of speech in kids. Results of BERA performed by an audiologist not only help in identifying the cause of hearing loss but also help in deciding type of hearing aid that might be required.Early identification of the cause of hearing problem along with early treatment (like hearing aid, cochlear implant etc.) has been associated with improved speech outcomes in infants.Your child may require hearing aid at least in left ear."
},
{
"id": 223391,
"tgt": "What causes bleeding after stopping the contraceptive pill?",
"src": "Patient: Hi i have the implanon in my arm and due to me bleeding non stop i was put on the pill. Thats been fine for a year now. But 3 weeks ago, as i do every month i stopped my pill, bled for 7 days and took my pill again to start a new month. Its been 2 weeks now an im still bleeding. Why? Doctor: implanon side effects is irregular bleeding...this may last from 6 months to 1 year...u r bleeding still because of imbalance of hormones...u can consult ur gynec for further tx"
},
{
"id": 153651,
"tgt": "What is hairy cell leukemia?",
"src": "Patient: My husgand has hairy cell leukemia. Each time he goes tot he doctor and the doc asks how he feels he says \"great\". I am going next time because he is always tired, sweats alot, bruises, is very pale, and breaths heavy. If he doesn't tell the doctor this, the doctor doesn't know. Also, his counts have dropped considerably. Doctor: Hi,Thanks for writing in.Hairy cell leukemia is a type of abnormality in lymphocytes which are white blood cells. If his counts are going down then you should consult his doctor and acknowledge them about the bruises, feeling tired and sweating. It is possible that he requires aggressive treatment for hairy cell leukemia and his general health also should be cared for.Giving him a healthy diet and adequate amount of fluids is required. Many times counts might drop during recovery also but if it is persistently reducing by large numbers then it requires to be discussed in detail keeping in mind his health condition and development of weakness and fatigue. The main features of hairy cell leukemia includes hairy cells are found in the blood and bone marrow. The number of red blood cells, white blood cells, or platelets may be lower than normal. The spleen may be larger than normal. Patients are followed up for response to treatment with chemotherapy medicines and most of them are treated. Please do not worry."
},
{
"id": 157789,
"tgt": "Pus and blood oozing after thyroid surgery. Taking antibiotics. Suggestions?",
"src": "Patient: I had thyroid surgery on august 12, 2013 I have been having problems with my cut oozing blood and puss. went and saw my surgeon 3 weeks ago she said I could be having a allergic reaction to the stitches inside she gave me some antobiotics and here it is 3 weeks later and its still oozing. i'm afraid that the cancer is not all gone Doctor: Hi you need more evaluation. The wound should heal in maximum of 10 days. Now pus from wound suggest that the wound is infected and this should be drained completely. The wound will not heal unless drained. We can't tell whether the cancer has spread or not with your explanation. Get done USG neck and culture sensitivity of the secretion for correct management and consult your doctor for draining the pus. Continue antibiotic and anti inflammatory drugs, maintain your sugar level and keep the area dry and clean.Hope this helps you.Regards"
},
{
"id": 101424,
"tgt": "Suggest remedy for breathing issues with sneezing",
"src": "Patient: I had a sneezing fit last night and am now having trouble breathing normally. I noticed it starting before I went to bed, and thought I could sleep it off, but it made it hard for me to sleep and very restless. Now I am having a hard time breathing all together, almost like my lungs can t take in enough air or that my bronchus is swollen. It makes it a chore for me to walk up my gravel hill from my pastures where my horses are, which is a trek I do every day at least five times. Doctor: HI, thanks for using healthcare magicIt is possible that you are coming down with a viral upper respiratory tract infection.This would cause sneezing and fatigue, along with the other respiratory tract symptoms. Treatment would involve rest, fluids and cough/cold medications. These medications would be available over the counter.I hope this helps"
},
{
"id": 37052,
"tgt": "What is white worm like on toilet paper after having flagyl for positive c diff?",
"src": "Patient: I was tested for c diff and it came back positive. I am now on my 2nd round of flagyl, and I have a low grade temp of 99.9. I noticed that both times I've taken this medication, my stool looks suspicious. I notice small 1/2 inch white looking worms on the toilet paper. At first I thought it was just the t.p. balling up when I wiped but then I noticed a couple on my stool in the water. Is this just from the meds or is there a parasite concern. Doctor: welcome to hcm, Hopefully the worms are coming out. send a sample for testing if you find next time. if it is confirmed that that is worm only you may need benzimidazole drugs for parasitic worm manifestations.hope this explains with regardsDr.Amarnath"
},
{
"id": 119998,
"tgt": "Suggest remedy for sore and throbbing hip, elbow and jaw after falling",
"src": "Patient: I tripped and fell down 2 steps landing on my right side - landing on my hip, right elbow and my jaw. I immediately put ice on it, but developed a goose egg size bump on the right side of my neck. It is sore and throbs. I have taken ibeprofen and have been sitting in a chair resting. Doctor: Hello,The symptoms seem to be related to the fall. I suggest using anti-inflammatory medications such as Acetaminophen to relieve the pain. I also recommend to ice the painful area and rest. If the pain continues,you should check with X-ray for fractures.Hope I have answered your question. Let me know if I can assist you further. Regards, Dr. Dorina Gurabardhi, General & Family Physician"
},
{
"id": 155824,
"tgt": "Suggest treatment for precancerous cells on colon wall",
"src": "Patient: Good evening. During my colonoscopy this past Monday, a cluster of cells was noticed on the colon wall. A biopsy was taken and the result, I received a call this a.m., precancerous. Repeat the colonoscopy in three years with a more extensive prep. This one was two days. Seems to be the usual recommendation? Doctor: hi.... it may be a dysplastic polyp... interventions depends on grade of dysplasia and involvement of base of that lesion.. if type 3 dysplasia and involving base of excised tissue you need intervention now itself.. if not can followup at close intervals..taking aspirin, multivitamins, avoiding red meat, taking high fiber diet helps in reducing ur cancer risk.. fecal occult blood testing, serum CEA blood tests can be done now.. get an appointment with an oncologist and review your reports please..."
},
{
"id": 208969,
"tgt": "Suggest treatment for post traumatic stress disorder",
"src": "Patient: HI ,DOCTOR INFACT I WAS RAPED SEXTUALLY ABUSE WHEN I WAS 10YEARS OLD UNFORTUNATLY THAT TIME I HIDE THAT FROM PARENTS BUT LAST FEW YEARS I WANT TTRARTMENT MY PARENTS WANT ME GET MARRIED CAN I MARRY OR IS THERE TRATMENT AVAILBLE FOR THAT KID OFF PATIENT IAM 28 YERS OLD MAN Doctor: Hello,Thanks for choosing health care magic for posting your query.I have gone through your question in detail and I can understand what you are going through.Paroxetine,clonidine, and clonazepam are the drugs used for PTSD.Hope I am able to answer your concerns.If you have any further query, I would be glad to help you.In future if you wish to contact me directly, you can use the below mentioned link:bit.ly/dr-srikanth-reddy\u00a0\u00a0\u00a0\u00a0\u00a0\u00a0\u00a0\u00a0\u00a0\u00a0\u00a0\u00a0\u00a0\u00a0\u00a0\u00a0\u00a0\u00a0\u00a0\u00a0\u00a0\u00a0\u00a0\u00a0\u00a0\u00a0\u00a0\u00a0\u00a0\u00a0\u00a0\u00a0\u00a0\u00a0\u00a0\u00a0\u00a0\u00a0\u00a0\u00a0Wish you good health,Kind regardsDr. Srikanth Reddy M.D"
},
{
"id": 222139,
"tgt": "What causes nausea and metallic taste in mouth during pregnancy?",
"src": "Patient: hi...i m 18 years old and four weeks ago i found out i was pregnant two weeks after i found out that i was pregnant the doctor told me that my baby has no heartbeat and it wasnt moving and at 8 weeks it should be very active she said so i had a D&C done after multiple ultra sounds but i had sex one day before i found out the baby was dead now am wondering could i be pregnant again? is that possible? i really hope it is because i really wanted my baby...i have a metalic taste in my mouth and am nauseous and vomiting and my breasts are hurting me what should i make of that?? Doctor: Hello dear,I understand your concern.Iam sorry for your condition.In my opinion there is no chance of pregnancy due to intercourse done the day before the fetus was dead.So the sore breasts,,metallic taste might be due to some other cause.As the pregnancy was abnormal it is better to give a gap of 3 months before planning next conception.Start taking folic acid tablets one month before planning the pregnancy.So relax.Avoid stress.Best regards...Dr Srilatha"
},
{
"id": 5185,
"tgt": "On birth control pills for PCOs. Light bleeding in periods, vomiting feeling. Had unprotected sex. Possible pregnancy ?",
"src": "Patient: Hi Me and gf had unprotected sex 12 days after her periods (May 2013). She was diagnosed with PCOS earlier and so has been on birth control pills since then (since Feb 2013). However, her periods this time are abnormal. She is not bleeding enough. She also is feeling a bit puke-ish. Do you think she can be pregnant? If so is there a way we can find out except the urine test? Doctor: Hi, Thanks for writing to us. The chances of pregnancy have always been there with unprotected sex. You had unprotected sex but chances of pregnancy in your case are rare because she is on birth control pills but no method of contraception is 100% effective so you can consult a gynaecologist for blood test and ultrasonography to rule out pregnancy. Good luck. Take care."
},
{
"id": 63556,
"tgt": "What is the hard and painful lump on the back of my neck?",
"src": "Patient: Hi i have a bump on the center back of my neck about the top of my spine. It is hard red and painful. I can move it a little side to side but not up and down. Ive had it for about 3 days now and has stayed about the same size, maybe the size of a quarter. It first looked like a boil and had a white head until the next day there was no head at all. what might it be? Doctor: Hi, dearI have gone through your question. I can understand your concern. You may have some enlarged lymphnode or benign cyst like sebaceous cyst or it can be soft tissue tumor like lipoma. You should go for fine needle aspiration cytology or biopsy of that lump. It will give you exact diagnosis. Then you should take treatment accordingly. Hope I have answered your question, if you have doubt then I will be happy to answer. Thanks for using health care magic. Wish you a very good health."
},
{
"id": 223886,
"tgt": "Can birth control pills cause nausea?",
"src": "Patient: Hi, I ve have intermittent nausea since I gave birth 5 months ago (major complications and on a antibiotics 3 months following). I started birth control 2 months ago and am wondering if there may be a link to the nausea. I also seem to get an increase in nausea when I m late taking a pill. Do you think there s a link between the pills and the nausea? Thank you. Doctor: hello dear.understand your concern.firstly you have intermittent nausea may be due to pregnancy but now its may be due to side effects of birth control pills(BCP).BCP has some side effects like nausea,spotting,breast tenderness,headache,weight gain,mood changes,missed period,decrease of libido(affect sex drive),vaginal discharge,visual changes,etc.... you can take tab.ondet or tab.reglan & tab.digene for nausea.or you can change your BCP,take another hormonal pills which suited you more.stay calm & dont be much anxious because it aggravate the symptoms.hope this will guide you.best regards.dr.sagar"
},
{
"id": 31573,
"tgt": "Suggest treatment for severe fungal infection in the groin area",
"src": "Patient: I have severe fungal infection in the groin area.I have this problem for over 2 years. I am not diabetic.About a month back on the advice of a skin specialist i took a week's course of Sporanox(itraconazole). the infection vanished only to reappear after 15 days. I repeated the course this time with canditral (itraconazole) The problem stopped. again I have the infection back last 3 days.Red patches.sporadic small small pus blisters.The infection area is widening and spreading to the penis.Any help pls Doctor: Hi i understand your concern very well.You have tinea fungal infection of groin.We will start treatment and then to prevent recurrance ,we will take preventive measures.continue Itraconazole tablet along with use clotrimazole containing cream.Apply cream over affected area along with cover sourrounding normal 3 to 4 cm area for cream application.When you get relief for preventing repeated fungal infection take following advise.The Groin area should not remain wet after bathing.Clean it properly and allow to dry.Use clean underwear and change it daily.Use your own towel.Do not share towel with others.Use warm water for bath.This will surely help you.With best regard.HCM."
},
{
"id": 69587,
"tgt": "What is the medication for lump on the banged my skin?",
"src": "Patient: Hello there, Just over 3 weeks ago, I banged my leg (shin) into a table. There was bruising directly at the site of the bang and also bruising on the inside of my ankle. All I am left with now is a lump where I actually banged my shin. What could this be? Doctor: Hi ! Good evening. I am Dr Shareef answering your query.From your history, it seems that you have got an organised hematoma on your shin at the site of impact. There is nothing to get worried. It might take take some more time to get absorbed. In case it persists, you may have to get it clinically examined by a general surgeon of your area, who might consider of evacuating it by a small incision if need be.I hope this information would help you in discussing with your family physician/treating doctor in further management of your problem. Please do not hesitate to ask in case of any further doubts.Thanks for choosing health care magic to clear doubts on your health problems. Wishing you an early recovery. Dr Shareef."
},
{
"id": 205392,
"tgt": "Suggest treatment for unemotional behavior while having anxiety",
"src": "Patient: My 19 year old son was originally taking Prozac for anxiety which were causing terrible headaches. He did very well on it but had terrible drowsiness and couldn t stay on it in college. He has moved to Citalopram because he found he was very short tempered and wanted to explode on people (more a phrase than anything physical, very stressed with college life) when he came off the Prozac. The citalopram has helped with that short temperedness but he feels completely emotionless now. Of the serotonin reuptake inhibitors, is there one that would be better to consider. We have an apt with the doctor in another week but I would like to go in prepared with a game plan. Doctor: Hi,This is one of the known side effects with serotonin re-uptake inhibitors. It can be helped by adding bupropion (dopamine reuptake inhibitors) or switch to SNRI like des-venlafaxine.Hope I have answered your query. Let me know if I can assist you further.Regards,Dr. Walid Hassan"
},
{
"id": 141042,
"tgt": "How to differentiate between cerebral amyloid angiopathy and Parkinson s disease?",
"src": "Patient: I had Polio when I was a young child; I am now 78 years old and had a hemorrhagic stroke about 15 months ago. The Neurologist at the local hospital said they had no idea what caused the brain bleed ; so I consulted a Neurological Team (consisting of a Neurologist, a Vascular Specialist & a Neurological Surgeon) at Shands Hospital in Gainesville, Florida. This is the highly respected teaching hospital for the University of Florida. The neurologist diagnosed my problem as CAA. I have fallen 8 times and injured my head 3 times (one time resulting in 7 staples to close the wound in the back of my head & subdural hematoma (sp), and another time 5 stitches in my forehead). Each of the 8 times I fell I required assistance in getting to an upright position, something that was unknown to me before my stroke of 15 months ago. Before the stroke I was described by wife as disgustingly healthy , a term with which my personal physician also agreed. I would appreciate your answer to the similarities & differences between CAA, Post Polio Syndrome & Parkinson Disease. With Kind Regards Robert Moger Doctor: Hi, The conditions you mention do not have much in common apart from being neurological conditions, there rarely is the needed to make a differential diagnosis between these conditions. Post-polio syndrome is characterized mainly by fatigue and worsening of prior neurological sequelae of polio, it has none of the manifestations you mentioned. Parkinson's disease is a chronic disease progressing over many years characterized mainly by slowing of the movement, rigidity and tremor. It may include falls but later on in the course of the disease, not in the first few years. There are no vascular lesions like hemorrhage on imaging. CAA, on the other hand, is characterized by the presence of multiple hemorrhages on MRI imaging. Manifestations may vary depending on the manifestation of major bleeds and their location may manifest with motor deficits and seizures which might be causing your falls. So it is a diagnosis which is easily differentiated from the other two by imaging features, there shouldn't be an issue distinguishing these conditions from each other. Hope I have answered your query. Let me know if I can assist you further. Regards, Dr. Olsi Taka, Neurologist"
},
{
"id": 12239,
"tgt": "How can pustular psoriasis be cured?",
"src": "Patient: I have pustular psorasis (not too severe on feet and palms) and taking medication for that (cortizone based cream). I have recently noticed a dark circular patch on my right foot. It is itchy sometimes. Could you please tell me what that is and how can it be cured? Doctor: Pustular psoriasis is rather difficult to treat and till date no medicine is available to cure the conditions,,,but there are treatment which may improve and give you long time remission...like topical steroids, vitamin-D analogs, PUVA, ultraviolet light B (UVB), acitretin, methotrexate or cyclosporine depending on the severity...maximum benefit with minimum of medicine should be the goal...talking about your lesion over foot this could be psoriasis but psoriatic patch is more greyish white scaly with underlying erythema rather than dark as you mentioned...You should better show it to a dermatologist to rule out any other fungal infection...thanks"
},
{
"id": 107948,
"tgt": "Suggest treatment for severe lower abdominal and back pain",
"src": "Patient: Hi. My lower abdomen and back alone with my left leg have been hurting for most of the day. The back pain and leg pain are more of an ache and my abdominal pain is more like really back cramps. I have a mirana so I don t have periods. I have had it over a year. I m not sure if this is serious enough to go to the ER or if I should wait it out? Doctor: Avoid bending. Take warm water bath. Try to drink turmeric 1/2 tsp in warm milk or water daily. Avoid cold drinks. Avoid stress.Wait and watch."
},
{
"id": 135180,
"tgt": "Suggest treatment for hard callus bump after stitches",
"src": "Patient: My husband had stiches on the side of his middle finger about a year ago. When it healed there was a small bump....we thought scar tissue. Over the last couple of months it got bigger. It was very hard, like a callous. Today he pressed on it and puss came out. A large rubbery thing came out of it, it was solid white. No pain, some blood....what the heck was it? He is 38, 5'9, 155lbs....no medical issues. Doctor: Hi..Welcome to HEALTHCARE MAGIC..I have gone through your query and can understand your concerns..As per your complain it seems that there is formation of Keloid at the site of stitches that is an Overgrowth of scar tissue..At times the scar tissue grows more than normal and leads to formation of a hard growth..It seems that there is infection at the Keloid site leading to pus discharge..I would suggest you to consult a General Physician and get evaluated and a clinical examination can help in diagnosis and treatment can be done..In case of infection first of all you will be advised to clean the area with antiseptic solution..Application of Triple Antibiotic Ointment can be advised..Once the infection resolve the Physician can advise you Silicone patches, pressure dressing etc to shrink the lesion..Steroid injection can be given in the Keloid in an attempt to shrink it..You can also be advised Cryotherapy, lazer and radiation to shrink the Keloid..Hope this information helps..Thanks and regards.Dr.Honey Nandwani Arora."
},
{
"id": 217780,
"tgt": "What causes pain in the heap?",
"src": "Patient: Hi, my name is Sangita. I'm from Malaysia. I'm just 23yrs old. Currently i'm having a heap pain on my left. But i could identify the exact place as my main is also at infront of my heap. I also feel that my whole left leg is pain when i press my heap at back. Could you advise me for further regarding the pain.. thanks.. Doctor: Hi sangeeta. This type of pain usually occurs due to a disc problembat the lower back causing pressure on the nerves that go down to the foot. Another reason for such pain is the arthritis of the sacroiliac joint. To confirm the diagnosis one needs an MRI Of the lower back or LS Spine.presently you could take a combination of diclofenac sodium 75 mg and thiocolchicoside 4 mg twice daily, after food. Show to a near by doc ( orthopaedic or pain specialist) and start your further treatment"
},
{
"id": 31494,
"tgt": "Does throat pain, fever and breathlessness indicate sign of swine flu?",
"src": "Patient: sir i am a 37 year old female ,i have asthmatic problem for past 10 years, past three days i have throat pain and respiratory problem,i cant breathe freely. from yesterday night it has increased and i had severe head ache. from morning i feel feverish and body ach too. is this seem to be a symptom for swin flu please answer me immediately becos i have a 10 year old son. Doctor: Hi thanks for asking question.These symptoms can be because of any viral infection especially any influenza flu.Do your CBC examination.Mostly it will reveal lymphocytosis if viral infection.It may be swine flu also.For it you have to contact doctor and your swab can be send for confirmation of it.Until that if strong doubt then temi flu can be taken.Drink more water and eat green leafy vegetables with more fruits.If severe bronchoconstriction present then steroid can be taken for 5 days to relieve bronchial edeama and congestion.Thanks."
},
{
"id": 127016,
"tgt": "What is the need for two surgeries for a total knee replacement?",
"src": "Patient: hello my name is Ross, I am 51 year old male, and I m to have TKR surgery next week, they told me that now I may have to take up to two different surgeries, because I have two screws put in when I had ACL surgery back in 1993. I m not really understanding why they need to have 2 separate surgeries now? Doctor: Hello, We have to wait few weeks after screw removal and only after we can go for knee replacement. So two surgeries are required but screw removal is a simple procedure compared to TKR. Hope I have answered your query. Let me know if I can assist you further."
},
{
"id": 85996,
"tgt": "Is discomfort in the lower abdomen after pressing it hard a symptom of an internal bruise?",
"src": "Patient: I've been having pain in my lower right stomach for about 5-6 months, have been to the hospital they said it was gas. Went again last night and they said it was colitis. The doctor pressed really hard on the area I said was painful, and now the area is uncomfortable. Like a 2/10 pain wise. There's no blood in my stool. Could he have ruptured or bruised something? Doctor: Hi, Pain in right lower abdomen for 5 - 6 months can indeed be due to a chronic problems and needs the following: -Clinical evaluation and examination by a Gastroenterologist. -Colonoscopy and biopsy. -CT scan of abdomen. -Stool tests for three consecutive days -Tests of blood and urine. -Any other tests as needed. This will give the proper diagnosis and a plan for a proper diagnosis. Colitis is one of the commonest reasons but needs the cause that caused it to have a specific treatment. Hope I have answered your query. Let me know if I can assist you further."
},
{
"id": 15826,
"tgt": "Rash around body. Diagnosed as chronic urticaria. Taking medication, reappearing. Advice?",
"src": "Patient: Suffer from a rash around my body and was diagnosed as chronic urticariaHe has conducted many tests including examinationCBCAnd rheumatism and examinations of the liver tests did not show any abnormal resultsBut the problem persists and treated with anti-histamine, but my body did not respondThe Kortizon addressed for 45 days on signing 60 mg per day, 30 mg am and 30 mg pmAnd softened by 5 mg dose every 5 daysAnd now stopped taking Kortizon on 05/03/2013But the symptoms reappeared, but slightly on the arms and abdomenPlease advise me because of such symptomsThank you Doctor: Hi, thanks for using healthcare magicIf possible you should have allergy testing done to determine what you are specifically allergic to. Allergy testing can be done as skin tests or as a blood test, they can check for allergens in your environment, foods, medications , chemicals.This would allow you to eliminate what it causing the allergic response.It is important to avoid mental stress, over tiredness , alcohol, tight clothing as all of these are thought to cause or worsen the rash.You may require long term treatment with a non drowsy anti histamine.I hope this helps"
},
{
"id": 129105,
"tgt": "Suggest treatment for leg injury",
"src": "Patient: I recently had knee surgery, I just fell down about 8 stairs after my knee dislocated. I landed hard on my back/ bum. I now have a very bad headache and can barely move my leg. It is possible I may have a concussion? Should I go to the er or can it wait until tomorrow to see a clinic dr? Doctor: Hello Thank you for using healthcaremagic I read your question and understood your concern I think you should go to the ER immediately to chech both the head and the knee. You should not wait until tomorrow Dr. Selmani"
},
{
"id": 12226,
"tgt": "Suggest treatment for psoriasis",
"src": "Patient: Hi I am suffering from Psoriasis (upper body and scalp is majorly effected) and want to know if there's any photo-therapy clinic available in North-West Delhi? I am based out of Rohini and looking for a clinic around this place. Thanks in advance for your suggestions.Warm regards,Anupam Doctor: Hello, Psoriasis presents as well defined, thick scaly plaques mainly involving the scalp, ans extensor surfaces like elbow, knee, lower back etc. Unlike extensive psoriatic disease for which both oral and topical treatment is required, for limited disease topical treatment alone is effective. However, for extensive disease systemic treatment with oral immunosuppresives like methotrexate, cyclosporin, Phototherapy (PUVA), Biologics etc may be required for disease control. Specific topical treatment options for psoriatic plaques are topical steroid with or without salicylic acid, topical Vitamin D analogs like calcipotriol, calcipotriene etc. These are available in ointment form for plaques involving the body and in lotion form for scalp psoriasis. Topical therapy may be combined with oral/systemic therapy for widespread/extensive disease. Besides specific remedies for psoriasis, moisturizers form an important part in the management of psoriasis because of the thick scaly nature of the plaques. Moisturizers enhance the penetration of topical active ingredients as well as prevents or delays side effects like skin atrophy etc. An oral antihistamine e.g cetrizine 10 mg may be required for symptomatic relief from itching. Treatment may take 2-4 weeks to show noticeable improvement. Therefore, I would suggest him to continue with the prescribed medicines and follow up with his doctor every 2-4 weeks. Hope I have answered your query. Let me know if I can assist you further. Take care Regards, Dr Ivan R. Rommstein, General Surgeon"
},
{
"id": 126148,
"tgt": "Is swelling around a fractured spot on the foot a serious concern?",
"src": "Patient: Hi! About 16 weeks ago I fractured my fifth metatarsal on my left foot. This was confirmed by X-ray by my orthopedic surgeon and I was prescribed a stiff-soled-boot that I wore for about 12 weeks. Follow-up x-rays confirmed that fracture was aligned and bone was reforming slowly but surely. At my last doctor\u2019s appointment (12 weeks) Dr. said I could stop wearing the boot and resume normal activity per my tolerance, which I have, avoiding jumping activities. I swim and walk daily for exercise. Very little pain since around 12 weeks, but my foot often felt/feels tight or cramped, especially on the top around my 3-5 metatarsals. In the last few days though, my foot has felt sore, appears a bit swollen in the same spot the bruise had formed when I initially broke it, and in that spot, I feel an odd \u201ccreaking\u201d sensation when a flex my toes and a mild/moderate and dull pain when I walk and swim. The feeling is quite bothersome! Notes: 1. I am female, 36 and vegan. I take Vit D and Calcium supplements daily. 2. I am asking here because my doctor moved after my last appointment and I didn\u2019t want to go through the trouble of seeing someone else in his practice if it isn\u2019t serious. Doctor: Hi, It may be due to hematoma or contusion following the trauma. You can apply ice packs and take anti-inflammatory drugs like Serratiopeptidase for symptomatic relief. If symptoms persist, better to consult an orthopedician and get an MRI scan done. Hope I have answered your query. Let me know if I can assist you further. Regards, Dr. Shinas Hussain, General & Family Physician"
},
{
"id": 61981,
"tgt": "What causes lump on upper thigh with bruised texture?",
"src": "Patient: Hi I have an oval shape lump in upper thighs.. When I run them it feels brused but no colour.. I have had this over 2 months, they have got smaller but still there.. This started when I started my new job which is sitting all day 12 to 14 hrs shift.. I am quite active but since I started this job have no time to excersize like I used to. I am a bit worried about theses lumps. Doctor: Hi,Dear,Welcome with your query to HCM.Studied your query in full depth of its details.Reviewed it in context of your health concerns.Based On the facts, You mostly seem to suffer from-Ring worm bumps is the possibility.Without Much information from your query I would suggest to take Second opinion from Surgeon,who would examine n decide on its cause and would treat it accordingly as per the cause of it.Hope that ,This reply would help you to plan further treatment soon with your treating doctors.Best of Luck and early recovery.Welcome any further query in this regard,which would be replied in next session.Good Day!!Dr.Savaskar M.N.Senior Surgical Specialist"
},
{
"id": 194492,
"tgt": "Suggest treatment for night fall",
"src": "Patient: hello this is khalil ullah from afghanistan age 18. dear sir actually i have sex problem since 2005 still now.when ever i sleep during night night so then due to bad dream ,my semen come out and when i see to cloths it became wet.so some time due to bad dream it happens and some time with out bad dreams in which i don,t know about myself during sleep although my semen come out.and day by day i become lazy and losing weight so please help me doctor what should i do.thanks Doctor: Hello, There is no need to worry about wet dreams because that is a natural process and usually cannot be controlled. Hope I have answered your query. Let me know if I can assist you further. Regards, Dr. K. V. Anand, Psychologist"
},
{
"id": 27249,
"tgt": "What causes heart pounding?",
"src": "Patient: ok, i think i was in about 7th grade, i was walking you know slow and easy down the hall and up the stairs, i don't know why but as soon as i sit down my heart started pounding and i could feel it like it was going to come out of my chest what could this be? Doctor: Hello! Thank you for asking on HCM! Regarding your concern, I would like to explain that your pounding heart feeling (that is palpitations), is among the most common heart related complains. It may be caused by just by an increased anxious state, is also a common finding during and after any physical exertions. These above options are almost physiological conditions, and you shouldn't worry about.But, we have to be sure no any other pathological reasons is responsible, first of all any arrhythmias.So, a careful physical exam, coupled with a resting ECG, and if suspicions are raised for a possible arrhythmia, a cardiac ultrasound and an ambulatory 24 to 48 hours ECG monitoring are necessary.Several blood tests (complete blood count to exclude a possible anemia; thyroid hormone levels to exclude a possible high abnormal thyroid function; PCR to esclude any inflammations, etc).Hope to have been helpful! Greetings!"
},
{
"id": 190281,
"tgt": "Painful lumps on roof of mouth, swollen, cough. Can you help?",
"src": "Patient: Hi Dr, I ve developed two painful lumps on the roof of my mouth- it s swollen on either side near the top but not the middle, and when pressed the lumps feel like bruises. When I don t press them, i can feel an aching in the top of my mouth and also a bit in my jaw. I m normally very healthy but at the moment have fresher s flu- just a cough though, no other symptoms and then the aching in the top of my mouth. I m a bit worried about it so wanted to see what I should do. I havent had it very long I d just like to know a possible cause. Could you help? Caitlin Doctor: Hello, Thanks for posting your query. Painful lumps on the roof of mouth might be due to Torus Palatinus, Growths in Maxillary Sinus, Tooth Abscess. A bump on the palate, though necessarily not a very alarming condition, may be an important indicator of a serious disorder like maxillary sinus cancer of minor salivary gland cancer. Hence, it is best to visit a dentist nearby and get the condition diagnosed and treated at the earliest, so as to prevent any problems in the future. Regards."
},
{
"id": 12119,
"tgt": "Is homoeopathic treatment help to rid from chicken pox spot",
"src": "Patient: Hello Doctor ! I m 19 years old. I am having so many chicken pox scares on my face which looks like a hole on the skin . To reduce this I m taking laser treatment ( taken 2 times till now). But problem is that after laser treatment the spots became darker then previous. I am so worried about this. Is there any other way to get rid from these spots. What about homeopathic treatment ? Please suggest me which treatment is best & what should i do ?? Thank You Sagar Jaiswal Doctor: hi it is not good sign in laser treatment which laser is being used? there are other effective ways -dermaroller -peeling -dermaabration first one is give good result ur doctor must have explain u that no treatment give more then 60% result in chickenpox scar. fractional co2 is good one and only effective in scar"
},
{
"id": 205285,
"tgt": "Suggest ways to wean off Zyven",
"src": "Patient: Hi iam 26 years of male .I am suffered depression and panic attacks last august .then I consulted the doctor he prescribed me zyven of plus 50 mg .after 3 months of taking this medicine I recovered from those problems one day I missed dose I feeling head ache very severely .then go the doctor he told me that do not stop it it should be taken at least one year .my problem was that there are side effects if taking it long term use .Hw can I get rid of those medication Thanking you sir Doctor: Hi Sir/MadamWelcome to Health Care MagicI can certainly understand your concern and. Worry about the side effects of desvenlafexine side effects , as it is SNRI is serotonin norepinephrine reuptake inhibitor , due to which serotonin and norepinephrine increases which is very useful in depression. The main pathology in depression is decrease in serotonin , which causes sad mood, fatigue , lack of interest in work, sleep disturbances , decreased appetite, hopelessness, worthlessness and all, panic attacks mainly due to Norepinephrine imbalance, you have mentioned there are no further attacks ...But it is very important to continue these drugs for atleast 6-9 months even after full recovery to prevent relapse and for maintenance . There is nothing to worry much about its side effects like nausea, vomiting, gastritis ...But these are present with any drugs ...It may have diastolic hypertension but which is very rare in your age group as you are still young, so no need to worry about its side effects as it is required for the brain to balance the neurotransmitters...being a psychiatrist , I can definitely advise you to continue the above Medication for atleast 1 year for good results.Consult your psychiatrist for further follow up Post your further questions if any, I will available to take up your follow up questionsThank you"
},
{
"id": 43638,
"tgt": "Trying to get pregnant. Have polycystic ovaries. Normal TSH and T4 ranges. Any treatment?",
"src": "Patient: Hello, I recently got tested for microsomal ab and the result came.back portraying high levels of the previously mentioned test. I also was tested for T4 and TSH but came back in normal ranges. I am a healthy (slightly overweigh 22 years old woman and trying to get pregnant for about a year with my husband. I am very frustrated and angry because I havent been able to concieve. Other thing I think you should know i have also polycistic ovaries. How can I control this and hiw can I achieve becoming a mother. Is there any treatment or medicine I can use in order to overcome this? Doctor: Hello, \u00a0 PCOD is definitely a big problem with most of the females but a patient approach definitely bears fruit to the patient. \u00a0 For PCOD,The most important treatment apart from ovulation induction medicine and hormonal treatment \u00a0is,shedding your weight and being in your normal BMI range and other treatment is to take care of your insulin resistance.\u00a0 The most important aspect of treatment is use of insulin sensitising medicine called as METFORMIN. \u00a0 Since patients of PCOS have incresed insulin resistance ,you need METFORMIN to reduce the insulin resistance. \u00a0 Loosing weight is very important to regain fertility in a patient of PCOS. \u00a0 It is said that even if you are able to shed 10% of your weight,you hall be able to get your periods back.\u00a0 You must consult a Gynecologist and get all the required investigations and proper treatment done.\u00a0 You shall definitely get pregnant,although timing can't be predicted.\u00a0 Best of luck \u00a0 Thanks"
},
{
"id": 103977,
"tgt": "Pain under the rib cage. Prescribed with tramadol, codine injection for pain. Suggestions?",
"src": "Patient: since Sunday I have had a pain under my rib cage on the left side . I have been prescribed tramadol by my gp. the pain didn't ease so went to A &E my urine showed trace of blood with protein my bloods however was clear. I had a codine injection for pain, but even though the pain is not as bad as soon as I eat the pain brings tears to my eyes and hurts also when I cough, sneeze and even yawn Doctor: IF THE PAIN IS NOT SUBSIDING WITH THESE PAIN KILLERS YOU HAVE TO UNDERGO TESTS TO DIAGNOSING THE CONDITION IT CAN BE KIDNEY STONES GALL BLADDER STONES APPENDICITIS OR CAN BE CARDIAC SYMPTOMPS GET ULTRA SOUND ABDOMEN FOR KUB AND APPENDIX ALL BLOOD TESTS ECG ECHO TO DIAGNOSE THE PROBLEM AND TREAT ACCORDINGLY"
},
{
"id": 93837,
"tgt": "Abdominal discomfort, gas, pressure under diaphragm, ascaris lumbricoides unfertilized egg. Done stool test, X ray. Help?",
"src": "Patient: Hi From about one and a half months I have been facing abdominal discomfort like gas/pressure is accumulating just under neath my diaphragm. I got a positive ascaris lumbricoides unfertilized egg in my stool test. My chest xray, abdomen erect xray, abdomen ultrasound, cbc blood test were all normal. I took albendazole 4 tablets for the Worms. In addition I was prescribed aristogym syrup, bifilac capsule, sompraz capsules for ten days. Prior to stool test I took parit tablets and ganaton tablets for two weeks. Right now I still have abdominal discomfort and pressure and it increases a bit after eating, I also feel a little burning after food especially spicy food. Can you suggest me next course of action, whether endoscopy is a must and if a barium meal xray would be of any help. Thanks and regards Pranay Gupta Doctor: Hi welcome to Health care magic forum. Thanks for calling H.C.M.Forum. You had upper abdominal pain, Not responding to medicines. It appears to be due to gastric irritation due to peptic ulcer, or irritant diet. It also may be due to gall bladder problem.It may also be due to lactose intolarance. I advise you to consult a gastroenterologist for diagnosis and treatment. You may need to go for M.R.I., gastroscopy, besides other routine tests for confirmation. I advise you to avoid spices, junk foods, or oily foods, to avoid gastric irritation. take only junk foods, like bread, curd, etc. Wishing for a quick and complete recovery. Best regards."
},
{
"id": 81864,
"tgt": "Can TB reoccur?",
"src": "Patient: My dad had undergone 6 months medication for tuberculosis 4 years ago and few weeks before he got severe cough and fever. I got worried so I send him to the hospital. He did chest x ray twice . The result says that there are fibrohazed densities in the right upper lobe and hazy densities in the right lower lobe. Impressions: minimal PTB right upper lobe and pneumonia right lower lobe. His doctor prescribed a medicine for 6 month medication same as what he prescribed before but what he said is it seems that the TB occurred again. To get a free medicines we went to a local health unit but when they test his saliva, he said that he was negative on TB? What was result means? Does his disease re occur? Doctor: Yes TB can reoccur.However both the doctor who prescribed tuberculosis treatment without doing a sputum test and the health unit who ended up doing test of saliva are wrong.What your dad needs to do is to give a proper sputum sample and not saliva for the test. If he is bringing out some amount of sputum, then a better test to do will be GENEXPERT/RIF test for tuberculosis.If he is not bringing any sputum then u need a HRCT to confirm the lesions seen on Xray chest are new or old"
},
{
"id": 7486,
"tgt": "Suffering with acne, red breakouts on both arms. No improvement with treatment. Effective remedy?",
"src": "Patient: Hi,I am 18 years old,Indian male(Wheatish complexion & Oily skin)...i have been suffering from acne for many years (mild-light mod)...and i just can t get them off my face.....used different face washes,skin toners,lotions....NOTHING ACTUALLY WORKS!! ;_-(.....plz plz plz recommenf me some medication(SAFE)....also i have a lot of red breakouts in both of my arms....HELP! Doctor: Hi, please be patient and understand that acne being an age dependent disease is going to come back. I suggest you to start Cap Minoz Od daily along with oral dapsone for around 3 months and topically Clindac A gel twice a day and Faceclin A gel at bed time, keep your face as dry as possible. Take care"
},
{
"id": 35293,
"tgt": "Does fever weight loss with rashes suggest typhoid?",
"src": "Patient: hi my blood test shows typhi \"o\"1:640 and typhi \"h\" 1:40..sometimes fever occurs 99.5..i lost 12 kg weight ,suffering from high pulse rate low bp,nausia ,and red spots ,rashes on my chest,spots are very small,i even cant go outside from my home because of low bp.......these sympotms are from last 1year ,but since my childhood i got infected by typhoid 4 to 5 times now my age is 26 years ...please suggest me is it typhoid or not Doctor: Thanks for posting your query to hcm. Your widal titer is suggestive of ttyphoid fever. you shoud take complete course of antibiotic treatment as per your consulting physician as titer of your widal is very high .repeat the widal after 15days a decrese in titre with clinical improvement will sugfest recovery .hope I solve your query.Best wishes for your good health.regards,dr manish puroit"
},
{
"id": 13182,
"tgt": "How to treat the rashes and weakness of my right arm?",
"src": "Patient: I have a couple of rashes scattered on my right arm. I couldn't write in class, because my arm was weak. I can't open a door with my right arm, or do any \"heavy\" stuff. My arm is still weak. If i go to the doctor, what will they give me for the rash and the weak arm? Doctor: Hi, I would recommend you to apply topical steroid like cortisone cream on the affected areas along with a frequent and regular application of moisturizing lotion on the entire arm. This will take care of your rash. Hope I have answered your query. Let me know if I can assist you further. Regards, Dr. Asmeet Kaur Sawhney, Dermatologist"
},
{
"id": 182651,
"tgt": "Can Peroxyl mouth rinse be used after tooth extraction?",
"src": "Patient: I ve recently had my wisdom teeth extracted on the bottom . The one side I believe is infected . I I have peroxyl mouth rinse . I have heard no peroxide and then some say use it . Wondering if it s ok to use . I have got clove oil, antibiotics ,salt water , and pain meds . It s been 4 days since and it is rediculious still ok the one side.. Is it safe to use ? Doctor: Thanks for your query, I have gone through your query.If i am your treating doctor, i would strictly advise not to use hydrogen peroxide solution as a rinse. The hydrogen peroxide burns the tissues so do not use, it has to be used cautiously by the dentists not by the patients. I advise you to do saline rinse, take little warm water put salt in that and gargle your mouth. Complete the course of antibiotics prescribed by your dentist. I hope my answer will help you, take care."
},
{
"id": 63571,
"tgt": "How to treat a sore bruise and lump on buttocks?",
"src": "Patient: My daughter has a very sore bruise on her butt crack. She's had it for about a week. It seems to be red, purple, and swollen and feels as if there is a lump there. She doesn't know what she could have done to get this, and it doesn't seem to be getting better. Any ideas? Her age is 24, a little overweight, but in general good health. Doctor: Hi,Dear,Thanks for the query to HCM. I studied your problem in depth and I understood your concerns. Cause and plan of Treatment-In my opinion on the given data,you seem to have bruise with cellulitis around it due to infection of the bruise.Its not getting better due to the friction at the butt crack between 2 butt cheeks.Treatment-Consult ER Surgeon / and take anti-inflammatory / and antibiotics for 5 days time, and this would resolve the issue of the lump on buttocks.Plz check out with your ER Surgeon who would treat it accordingly.So don't build up wrong concepts and self medicate your self which would increase risks and costs to you.Instead just ask a query to HCM and be comfortable to resolve your health issues.Welcome for any more query in this regard to HCM.Write good reviews and Click thanks if you feel satisfied with my advise.Have a Good Day.Dr.Savaskar M.N."
},
{
"id": 607,
"tgt": "Can non-penetrative intercourse cause pregnancy?",
"src": "Patient: me and my girlfriend were fooling yesterday,we kissed and for the first time my penis touched her vagina but there was no penetration,when i felt like coming i moved my penis quickly to a laps and thats were i released my semen,now she has started her period today,is there a risky of her being pregnant and does the menstrual fluid wash away the semen in case there was a bit of spillage?please help Doctor: if she has got her period then there are no chances of getting pregnant from your last event of sex."
},
{
"id": 24523,
"tgt": "Does CABG require so many follow up medications?",
"src": "Patient: is it very necessary to take asprine ,plagrine & astorvastine after CAGB, age 62years , 172cms, wt 80 kgs, undergone cabg in 2005 , My stress thallium report is negative but TMT is always Showing +ve .I am taking medicines for HYper tension Amlong -A, Olemasartan Medoxomil 20mg And Plagrine 75mg+Asprine 75mg & Astorvas20 + asprine 75 mg ,pl tell me is it very necessary to take asprine and other medicines for heart health Doctor: Hi,You should know that CABG solves only the problem with blocked arteries at the moment of the surgery. But atherosclerosis, which causes the blockage, is mot cured with CABG. So you should continue to take medications to prevent further narrowing or blocking of the arteries.Hope this was helpful for youWishing you good healthIn case of further questions don't hesitate to askRegards,"
},
{
"id": 153226,
"tgt": "Is reconstructive micro vascular surgery advisable after tongue and jaw cancer surgery?",
"src": "Patient: i undergone tongue & jaw cancer surgery before 9 years .now i am totally disease free .because of radiation i am not able to wide open mouth .doctors suggesting reconstructive micro vascular surgery ,i want to know that who is the best doctor in india ? Doctor: hi,Thanks for writing in.If you have got treated for tongue and jaw cancer by surgery 9 years back and now are disease free then you are cured from the cancer.Radiation has side effects like causing muscle contracture in the mouth. You might consult a plastic surgeon specialized in flap surgery and release of contractures. They might be able to help you out with surgically reconstructing the region and allowing you to open the mouth. Please note that these are highly specialized surgeries which require skill and the outcomes might differ after surgery. You might like to consult doctors in plastic surgery at a tertiary hospital in your city and then know the consequences and if a surgery might be possible or not. This can be told after clinical examination of the opening of your mouth. Please know all the results of such a surgery before getting it done and discuss in detail."
},
{
"id": 3407,
"tgt": "Is it safe for IUI procedure at 14 day of period, primolut was given for 16 to 25 days?",
"src": "Patient: I am a 29 year old, diagnosed with PCOS, and have irregular menses.Been trying to conceive and carried out an IUI procedure recently after a basl temperature drop- sign of Ovulation.Usually when i take Primolut,it induces bleeding.But my Doc prescribed it for Day 16-25 of period.Is it safe even after IUI procedure @ day 14? Doctor: Hallow Dear, If you are a case of PCOS, you should not go for IUI empirically. Just do not go by temperature changes in the Basal Body Temperature Chart; some other relevant information is important and required before attempting any Artificial Reproductory Technique like IUI. In patients with regular menstrual cycle, day of ovulation is 14 days before the expected menstrual cycle. However, since you are a PCOS case, your menses must be irregular. Hence, better not rely on day 14 of the cycle. I would advise you to go for ultrasonography ovulation monitoring. It will help following way: 1. It will precisely detect the day of ovulation when you can go for Intrauterine Insemination (IUI). 2. It will inform us about the size of the follicle when it ruptured to release the egg. Follicles of size between 18 mm to 21 mm release the healthiest eggs having good chances of successful pregnancy. 3. If the eggs are not getting released naturally (which is characteristic of PCOS), your Gynaecologist can plan induction of ovulation by some medicines like Clomiphene citrate or HMG/HCG combination and then plan IUI accordingly. May be you might require Metformin or Myo Inositol as an adjuvant therapy. 4. Ultrasonography will inform us about the development of the uterine cavity lining (endometrium). Endometrium of thickness between 9 mm to 14 mm is most conducive for implantation of fertilized egg. If the endometrium is thin, your IUI may go waste. Hence, please submit yourself for ovulation monitoring from by Ultrasonography day 9 of the cycle and then plan for IUI.Please do not take Primolut pills for regularizing the menses; they may interfere with the egg release. Ovulation induction medicines will regularise your menses along with the induction of ovulation. Please report to some Infertility Consultant. I hope you got the message. Dr. Nishikant Shrotri"
},
{
"id": 173740,
"tgt": "What is the reason for the child to pass thick grey stools after being formula fed?",
"src": "Patient: Since my daughter went exclusively to formula and we switched kinds she has been having small thick grey poops and going only about every 2 days. She was put to formula due to slow weight gain, and is getting polycose added. Is this from iron in the formula or should I worry? Doctor: Hi...Thank you for consulting in Health Care magic.What your kid is having is not a diarrhea and is only a gastro-colic reflux. It is quite common for babies of this age group t pass small amount of diarrhea or loose stools soon after feeds. This need not be treated as diarrhea and especially antibiotics are not indicated.Unless the kid's having low urine output or very dull or excessively sleepy or blood in motion or green bilious vomiting...you need not worry.Hope my answer was helpful for you. I am happy to help any time. Further clarifications and consultations on Health care magic are welcome. If you do not have any clarifications, you can close the discussion and rate the answer. Wish your kid good health.Dr. Sumanth MBBS., DCH., DNB (Paed).,"
},
{
"id": 3712,
"tgt": "What happens when she becomes pregnant again as she is Rh negative?",
"src": "Patient: My daughter just had a miscarriage . Come to find out she is rh negative. She did receive a shot of rogam not sure of that spelling. This was her first pregnancy. What happens when she becomes pregnant again? And should she have another shot of this when she becomes pregnant? Doctor: HI, I understand your concern. If a Rh - ve mother carries( conceives ) with a Rh +ve child , there are rsk of her being sensitized to Rh +ve blood . To clear this effect Anti D injection (Rhogam )is given within 24 hours of delivery & abortion .This helps in- - Avoiding complications in next delivery, - Avoiding serious reaction after blood transfusion in mother .. in case she needs it through out her life. * So, in case of your daughter- It's good that she has received anti D after her abortion. That has made her coming pregnancy / blood transfusion ( if she needs it ) safe. She will have to get Anti D injection after her next delivery too.. If her new born baby is Rh +ve. Rh - ve mothers need this precaustion every time the give birth/ abort. Thanks."
},
{
"id": 36167,
"tgt": "Suggest treatment for dog bite",
"src": "Patient: Hi, one dog has bite my child (3 yrs old) on 10.12.2014, we continue Rabipur vaccine as 0, 3, 7, 14 & 28 days schedule . I want to know this is correct or not. also I want to know in future any test need to conduct to be confirm on 100% cured of that patient after complete this schedule or any extra care need to maintain. Regard, Alal Doctor: Hello,The schedule of vaccination is perfectly all right.There is no test required to be done,once the immunisation is complete.Once you are immunised completely,you are cured 100%.I hope it satisfies you.Thanks"
},
{
"id": 26106,
"tgt": "What causes heart palpitations and increased heart rate at night?",
"src": "Patient: I have heart palipitations and an increased heart rate at night once I lay down to go to sleep. They happen throughout the night and even wake me from sleep. Do you any idea what could be causing this? Heart desease does not run in my family and I've never really had heart issues. Doctor: Hello!Welcome and thank you for asking on HCM!I understand your concern and would explain that your symptoms may be related to anxiety or autonomic dysfunction related to nightmares. Coming to this point, I would recommend performing a careful physical examination and some tests:-resting ECG and cardiac ultrasound-chest X ray study-blood lab tests (complete blood count, thyroid hormone levels, blood electrolytes, kidney and liver function tests, etc.) to exclude other causes of increased heart rate. An ambulatory 24-48 hours ECG monitoring would be necessary to examine your heart rhythm trends during day and night and exclude possible cardiac arrhythmia. You should discuss on the above issues with your attending physician. Hope you will find this answer helpful!Best regards!Dr. Iliri"
},
{
"id": 162204,
"tgt": "Can Mebeverine be given to a child for biliary colic?",
"src": "Patient: Hi, My 10 year old son is in agony in all likelyhood with gallstones, confrimed by two GPs, ultrasound pending.. myself and my mother had gallstones removed in our childhood years, which is why we are fairly certain about him having gallstones. He currently takes codeinphosphate and ibopofen and paracetamol as painkillers (which all do not appear to work). We now suggested him also being described mebeverine which he takes. Is this commonly used for dealing with spasms? can it be recommended for kids... Doctor: Hi, Yes, this is a very good antispasmodic and can be used in kids. Also, make some dietary changes for your child as gallstone pain is initiated by fatty meals, thereby fried fatty meals are to be decreased. Hope I have answered your query. Let me know if I can assist you further. Regards, Dr. Varinder Joshi, General & Family Physician"
},
{
"id": 14452,
"tgt": "What causes rash on the armpit and the stomach?",
"src": "Patient: Hi. 26, 5'4\" 135 pounds. I sometimes get a rash on the outside of my armpits that is not where I put on deodorant. I also sometimes get a rash on my hip bone and/or stomach at the same time. But nowhere else. This will come and go from time to time, but goes away. Doctor: Hello,Thank you for posting on HCM.The condition you have referred to is called Urticaria or hives. Its an allergic manifestation of skin, where an allergen leads to release of certain substances from your blood, leading to itchy skin rash and swelling over soft tissues. Its proper management requires thorough history, clinical and laboratory work-up.You may have to go for specific tests like patch test, food prick test, IgE antibody levels etc. As for treatment part, best would be the avoidance of allergen as far as possible. Try to eliminate possible triggering foods from diet. I would also advise you various anti-histaminics for long duration( atleast 3 months) with or without oral corticosteroids. For non- responding cases there are many other drugs like dapsone, cyclosporine, montelukast, omalizumab etc which can be used in certain selected cases. Hope this will help you in resolving your query.Thank you Dr Hardik Pitroda"
},
{
"id": 166882,
"tgt": "Suggest treatment for fever in baby",
"src": "Patient: Hi, may I answer your health queries right now ? Please type your query here...hello sir.. my son 1 year and 9 months, is having fever of 99-100F for the past 3 days.he doesn't have cold ,playing well..sleeping well. his urine and motion within normal.. what could be the reason for his fever? Doctor: Hello and welcome to \u2018Ask A Doctor\u2019 service. I have reviewed your query and here is my advice. Fever is a sign of inflammation that could be anywhere in the body, since he doesn't have any other symptoms then this may only be due to teething and inflammation of gums due to teeth eruption. For fever give him acetaminophen or ibuprofen syrup every 8 hours and if he develops other symptoms then see a doctor.Hope I have answered your query. Let me know if I can assist you further.Regards,Dr. Salah Saad Shoman"
},
{
"id": 70147,
"tgt": "What causes painful lump just above coxis?",
"src": "Patient: I had a painful lump just above my coxics. It was hard to sit down and very painfull at times and that has now passed then I had some discharge of puss from that area. Now it feels hard like gristle and I can move this lump around just wondered what it is Doctor: Hi,THanks for writing to HCM.That lump is indurated tissue, it should go in a few days. It it dosent or reoccurs then take a course of antibiotic.RegardsDr. Ashish Verma"
},
{
"id": 89144,
"tgt": "What causes pain in upper stomach?",
"src": "Patient: Good Afternoon, I had a total hysterectomy in December and am now expierencing episodes of upper stomach pain. The pain will last a few hours to a few days. People have commented that I may be expierenceing gallstones. Is this common ar completely unrelated? Doctor: HI. Hysterectomy done is a pelvic organ hence to have a pain in the upper abdomen after some months looks to be unrelated. I would advise you to get the upper GI endoscopy, an opinion of the gastroenterologist and Ultrasonography to start with to get to the root of the problem. This can then be treated accordingly. Till then you can have PPI, soft bland diet, and avoid foods and beverages which cause this problem of upper abdominal pain ."
},
{
"id": 186874,
"tgt": "What treatment to be taken for the purple spot on teeth?",
"src": "Patient: my son is 5 years old and he went to the dentist today for a routine dental exam. During the exam the dentist found that my son has a spot of purpleon his top pallet and im worried what it could be . Do you have any insight on what it could be caused from or what it could be ?? Doctor: Thanks for you query i have gone through your query, the puplish spot can secondary to trauma or injury...or it can be ecchymosis or a nevi..consult a oral physician for complete examination and needful..i hope my answer will help you..take care.."
},
{
"id": 197597,
"tgt": "What causes penis pain and blood coming form penis after sex?",
"src": "Patient: My period will start in a week & I just had sex, my boyfriend is complaining of pain in his penis and noted that there is blood or clots that is coming out of it this is the second time it's happened. Can left over menstral blood be the cause & what can he due to ease the pain Doctor: Thanks for asking.It's not possible for a menstrual blood to come out from penis. that's not a possiblity.We need to consider a trauma to penis while sex.I would suggest to consult a local surgeon and get an ultrasound examination and appropriate physical examination."
},
{
"id": 29960,
"tgt": "Does sore throat with fever and diarrhea need immediate medical attention?",
"src": "Patient: My 15 yr old daughter starting complaining of a sore throat on Thursday. It is still bothering her. She started a fever about 24 hours ago. She is having diaherra and not eating very much. She is drinking a lot of water to stay hydrated. She is unable to talk. I was just thinking it was a virus, but now I wonder if she has strep throat. None of my children have had it in the past. How long should I wait to take her to the doctor to possibly get tested? Doctor: Hello, eventhough your daughter is in the age group who commonly get sore throat your symptoms are more towards viral infection. Usually sorethroats associated with diarrhoea are viral flus. So what you need is to1. take paracetamol, loratidine and an antacid like famotidine.2. A plenty of fluids3. Bed rest.4 Steam inhalation But if your child is ill looking, nausea and vomitting you child need blood tests.Other features you may have to be alarmed are high fever with chills, skin rash, child not st all taking orally. if child is not taking orally at all, child needs hospital admission. If your child didnt develop latter symptoms you dont have to investigate her for strep.Given management is enough. Hope this helps. thank you"
},
{
"id": 141934,
"tgt": "What does \"patchy areas in white matter\" mean?",
"src": "Patient: multiple patchy areas of increased t2 flair signal in the periventricular and subcortical white matter. many of which appeared are oriented perpendicular to the corpus callosum several which appears slightly smaller when compared to previous examcan you tell me what this means please YYYY@YYYY Doctor: Hello! Welcome on Healthcaremagic! My name is Dr. Aida and I am glad to attend you! These MRI findings are suggestive of Multiple sclerosis. I would like to know your past medical history and symptoms. It is important consulting with a neurologist. A lumbar puncture and other tests may be necessary to confirm the diagnosis. Hope you fill find this answer helpful! Best wishes! Dr. Aida Quka, Neurologist"
},
{
"id": 89477,
"tgt": "What causes lower abdominal pain in the right side?",
"src": "Patient: hi.. well i have been having stomach ache in my lower abdomen since almost two years now.. earlier i thought there might be some gynaecological problem, so ai got ultra sound done twice and got checked by two difeerent doctors, but both of them said its not because of this.. might be due to the fact that i m fat.. but noe this pain in lower abdomen is like occuring after every half an hor.. if i walk fast or anything.. then it feels like paining.. after 7 minutes it becomes normal, again after some time it pains and then it becomes normal. earlier i felt pain in the entire stomach but now from 2-3 days i feel its more towards the right side... what should i do?? Doctor: Hi.When the ultrasonography is normal , the best way is to get a CT scan of the abdomen and colonoscopy because the pain which has centered around the right side can be sue to Typhlitis , coloitis , appendicitis, diverticulitis. Another option is diagnostic laparoscopy and sos procedure. A prior trial with suitable antibiotics and metronidazole should have helped you."
},
{
"id": 41437,
"tgt": "Is AKT4 safe and effective in IVF treatment?",
"src": "Patient: Hello Doctor,I had 3 abortions in last 3 years and one ectopic pregnancy..I am going for IVF now but 2 time failure implantation so doctor has advised to AKT4 for two months will I be able to concive after 2 months is this treatment safe in pregnancy?are there any side effects of this treatment? Doctor: Hey thanks for your faith in the health care magic.i have gone through your question and understood your problem.Yes, it is sometimes possible to have tuberculosis, and yet test negative on the conventional laboratory test.Evidence of tubercles directly seen on DL and hysteroscopy is the best proof of genital tuberculosis.SHe is right to recommend anti tubercular treatment for you, based on the directly visible evidence.It is definitely possible to conceive a baby while on AKT.Just wait for the initial two months of AKT 4 to get over.Then you can start trying for pregnancy.If the tubes are open, there should be no hitch there.There is no complication to the baby while you are on AKT 3.Hope I have answered your query. If you like my answers and wish me to answer in future, bring it to my attention"
},
{
"id": 89689,
"tgt": "Can mirena iud cause stomach ache?",
"src": "Patient: I got a Mirena IUD last Wednesday and no problem with it until about two hours ago. Ever since then I am having really, really sharp pains on the left side of my stomach, made worse by movement. I checked for my strings and either my fingers are too short to reach them or they aren't there. I know I need to contact my doctor, but is this something that can wait until the office opens tomorrow, or should I go to the hospital? Doctor: Hello, thank you for posting your query to HCM.IUD's usually do cause some pain, but the pain due to them is caused right after one gets them. If the pain dint come immediately, and is coming after a few days, there could be one of the two reasons for it:1. Either your IUD has been displaced2. Or the pain is due to some other factor and not your IUD.Did you ever feel your thread? If you did and cant feel it now, then there is a possibility of displacement of your device. Also is there any bleeding PV? If yes then dont waste time, and immediately see your doctor. If no, then you could take antacids and antispasmodics and wait for the pain to subside for a couple of hours. If that doesnt happen then you must go and see your doctor.Hope this was helpful."
},
{
"id": 149398,
"tgt": "Painful sciatica, difficult to walk. How to reduce pain? Acupuncture can help?",
"src": "Patient: I have very painfull sciatica I can hardly. Walk with it it stops me in my tracks as my legs get heavy and feel weak I've had it for about 3-4 wks but it seems more painfull then it has ever been what can I take for the pain also I have an exercise bike would it be okay to ride it or would it make matters worse i like to go walking to keep fit but Now it seems impossible I have been told accapuncture is an option but is it a cure do you need many sessions to improve the condition. YYYY@YYYY Doctor: Hello, In India there many systems of Medicine practiced. The patient is free to choose the system of Medicine he desires. Since I am not trained in acupuncture I am not able to answer your query. Try Physiotherapy, it will definitely help if done scientifically."
},
{
"id": 141562,
"tgt": "What is the treatment of dizziness ?",
"src": "Patient: My 14 year old son has slight dizziness every other day. His bloodworm is fine, ears are clear, EKG fine, chest X-ray great....on Kapvay for over a year and trileptor since September and no problems there..could this be a cause of puberty because he seems to be changing quickly? Doctor: Hello and Welcome to \u2018Ask A Doctor\u2019 service. I have reviewed your query and here is my advice. If this is the case of grandmal seizure is there, the given medicines are suggestive of it, and you have not mentioned it that for what the medicine Trileptor being give and other drug, if this is given for epileptic seizure so giddiness is likely, of course it may be less common but can be ruled out, the symptom of giddiness is not due to age changes or puberty. Hope I have answered your query. Let me know if I can assist you further."
},
{
"id": 78376,
"tgt": "Suggest remedy for difficulty in breathing",
"src": "Patient: I have been having a hard time breathing for 2 weeks now I have been using albuteral treatments and my inhaler it feel like a grown man is standing on my chest. I have a stuffy nose and a horrible cough but no sore throat. What can I do to help my breathing? Doctor: Thanks for your question on Health Care Magic. I can understand your concern. By your history and description, possibility of bronchitis or lung infection (pneumonia) is more. So better to consult pulmonologist and get done 1. Clinical examination of respiratory system 2. Chest x ray 3. PFT (Pulmonary Function Test). Chest x ray is needed to rule out lung infection. PFT is must for the diagnosis of bronchitis. It will also tell you about severity of the disease and treatment of bronchitis is based on severity only. You may need additional drugs like antibiotics, inhaled bronchodilators and inhaled corticosteroid (ICS). Chest physiotherapy and deep breathing exercises are also useful in your case. Don't worry, you will be alright. Hope I have solved your query. Wish you good health. Thanks."
},
{
"id": 65559,
"tgt": "Suggest treatment for retroparitoneal cystic lymphangiom",
"src": "Patient: My brother had this when he was young and required a blood transfusion. He has now at age 33 been diagnosed with a rare disease (Mesinteric/Retroparitoneal Cystic Lymphangioma) please help! He's very ill and we're trying to find answers...are these linked? Why would he have had to have a transfusion? Doctor: Hi, dearI have gone through your question. I can understand your concern.No it has no relation with this. For retroperitoneal cystic lymphangioma he should go for surgery. Complete removal is the treatment of choice. Blood transfusion may be due to some other reasons. Consult your doctor and plan accordingly.Hope I have answered your question, if you have any doubts then contact me at bit.ly/Drsanghvihardik, I will be happy to answer you.Thanks for using health care magic.Wish you a very good health"
},
{
"id": 50619,
"tgt": "On Neoral after kidney transplant, diverticulitis, anemia, advised Vit C, iron. Does rose hips have negative effect on Neoral ?",
"src": "Patient: My wife is 19 years out from kidney trnaplant and taking Neoral. Recent diverticulitis and anemia resulted in RX for vitamin C, iron and antibiotics. Some Vitamin C oferings contain rose hips, can rose hips have negative effect on Neoral?Can she also take Geritol in addition to the iron (niferex) with no negative side effects? Doctor: Hi, Your wife aged 19 years had kidney transplant, now diverticulitis and anemia, treated with vit.C, iron and antibiotics. some vitamin .C contains rose hip did it have negative effect on neoral. She has crossed kidney transplant and is under the supervision of the doctors, If it has any negative effect also it is not always, and you enquire with your doctor and try to have another alternative with out rosehips. Dont try to give her the one with rosehips, as the doubt play a role in creating negative effects. Thank you."
},
{
"id": 32505,
"tgt": "What causes severe infection under the tongue?",
"src": "Patient: My boyfriend had major surgery on his throat. He had a large infection under his tongue in the throat that the surgeon had to cut into his throat area and drain the infected area. He has now has to deal with increased tightening In the throat area from the scar tissue that has formed. When he bends over to tie his shoes he feels like he is being strangled. Is there any sort of procedure or laser that would help decrease the scar tissue. Thank you Doctor: Hi Dear,Welcome to HCM.Understanding your concern. As per your query you have symptoms severe infection under the tongue post surgery which is due to healing and formation of scar tissue. Well need not to worry about it. You can go for micro-ablation or surgical procedure to remove this scar tissue. I would suggest you to wait for some time to get this scar area healed naturally. Maintain proper oral hygiene by brushing and flossing. Use tongue scrapper very gently to remove any micro-debris attached to tongue. Avoid accumulation of any food particles there as it will leads to fungal infection. Drink plenty of fluids. Visit oral surgeon once and get it examined properly and go for chemical/ electro cauterization procedure if prescribed.Hope your concern has been resolved.Get Well soon.Best Wishes,Dr. Harry Maheshwari"
},
{
"id": 193977,
"tgt": "What causes blood from penis during intercourse?",
"src": "Patient: First off, I'm a 30 y/o white male. I was having sex last night and noticed a little blood coming from my penis. I was worried but thought maybe it would be fine by morning. This morning I urinated and alittle blood came out first and then the urine. There is a stinging pain when urinating now. The sex was alittle more rough than usual but nothing crazy. What maybe wrong? Doctor: Hello, Occasional blood through semen or through urine is common and there may not be anything to worry. Still, you may talk to a physician for physical investigation. Hope I have answered your query. Let me know if I can assist you further. Regards, Dr. K. V. Anand, Psychologist"
},
{
"id": 223525,
"tgt": "Do birth control pills cause brown discharge through the vagina?",
"src": "Patient: I am on the birth control pill ortho tri cyclen lo, I ve been on it for nearly three months, I have the last hormone pill tonight. Today i started experiencing a light brown discharge that has a bad odor. I am sexually active but always wear a condom. I haven t experienced this before and I m not sure what to think. help. Doctor: Hi, I think you should consult a gynecologist. It can be due to some infections. You may need some antibiotics. Hope I have answered your question. Regards Dr khushboo"
},
{
"id": 37068,
"tgt": "How to relieve cough due to flu?",
"src": "Patient: I am just getting over the flu and have occurred a cough. Every time I breath I want to cough. My stomach muscles are really sore as well as my ribs and I have to sleep sitting it. Is this still part of the flu symptoms and can you recommend anything? Thanks Doctor: Hello,I understand your concern.I am Dr. Arun Tank, infectious diseases specialist, answering your query.In my opinion cough is protective reflex. In case of any infections it is not advised to stop the cough.I suggest you take the full drug for infections. Once the infection is cured your cough will also be cured.I will be happy to answer your further concern, you can ask me on bit.ly/DrArun. Thank you.Dr Arun TankInfectious diseases specialist."
},
{
"id": 4223,
"tgt": "When will I conceive? When should I take pregnancy test?",
"src": "Patient: hii my name is prema. i m try to conceive but i not get pergnancy. my last period date is 26 jan 2014. my docter advice me to do follicula monterion. my report are very goodmy docter gave me susten 300 for 5 days. know i want to know that i will conceive or not and which date i want to do pregnany test. plz help me Doctor: Hello and welcome to HCM,I am Dr Nilajkumar a consultant gynecologist and I will be helping you in your queries. I understand the query. If you have regular cycles, the fertile period is from day 10-20. Keeping alternate day sexual contact on these days has the maximum chances of pregnancy. You can also use ovulation detecting kits and try to keep regular sexual contact during this time for maximizing the chances. It may take a few months for the contact to occur during ovulation so keep trying. Regarding conceiving in this cycle: it is difficult to say as it is unpredictable. But you have a good chance.Pregnancy test can be done 7 days after missing the periods using the first morning urine sample. Also start folic acid tablets to prevent brain defects in the baby.Best of luck.Hope this was helpful. Feel free to ask any further queries and I will be happy to help.Thanks for using HCM.Have a good day.Dr Nilajkumar Bagde\u00a0\u00a0\u00a0\u00a0\u00a0\u00a0\u00a0\u00a0\u00a0\u00a0Consultant Obstetrics and Gynecology"
},
{
"id": 14309,
"tgt": "What causes skin rashes on the upper body?",
"src": "Patient: My husband, son, and I are all getting over an ailment that involved a fever, a cough, aches, and nasal congestion. My husband and son have recovered nicely with no treatment but after my fever broke I realized I had a sunburn type rash all over my arms, shoulders and around my ears. It seems to be going away. Is this common with a usual viral 'bug'? Doctor: Thank you for your question.The symptoms that you describe are very common after viral infections. It is well known that many viruses cause rash in addition to other symptoms, such as the ones you have cough, aches, fever and nasal congestion.Theses viral rashes usually are not serious. Try do drink plenty of water and have some rest.If you notice some other symptoms please feel free to contact me again.Wish you good health!Dr. Migena GEGA"
},
{
"id": 179576,
"tgt": "Is Cetzine effective for cough and how long should it be taken?",
"src": "Patient: Hello Doctor, My kid is 4 years old. Past 2 days, she was sneezed. Yesterday onwards, she suffered with sneezing, cold and cough. I ve checked with my doctor he advised me to give Zifi 50mg tablet 3 times per day and continue 3 days. Other than that, Cetzine syrup (2.5ml). How long i ll give Cetzine. Will it cure cough? She is taking medicine for Primary Complex too for past 4 months. Thanks for your immediate response. Doctor: Helloyes you can give those medicines prescribed by your doctor.mostly cold will last upto 7-10 days. cetzine can be given for one week.cetzine is for cold and sneezing. zifi contains cefexime that is an antibiotic which will help in reducing cough.Hope this answer helpsRegardsDr. Shesh"
},
{
"id": 104760,
"tgt": "Found small tiny tag in throat. Its itchy and have sinus problem with breathing difficulty. Cure?",
"src": "Patient: Hi my name is Tamara about a month ago I found that there was a small tiny skin tag, flap of skin dangly mole like thing on the soft pilate on the roof of my mouth toward my throat at first I had some allergy symptoms like itchy throat so I just figured my throat was irritated from sinuses. Since then it s a month later and this soft flesh dangly mole like thing is still there. It doesn t hurt or anything every once in a while it feels agitated and my throat gets a itch and it makes me cough here and there but not all day. Also I noticed that my throat and chest gets tight somethimes and it doesnt feel so easy to inhale air. What does this sound like I m so scared. Doctor: hello , nothing to get scares you get the problem checked by some ent specialist and do the treatment as advised , or other wise we can help you with homoeopathy for this problem , do let us know if the problem persist at dr2uonline at the rate yahoo.co.in"
},
{
"id": 61493,
"tgt": "What causes a lump under the chin?",
"src": "Patient: I have a small (2 - 3 cm) lump under front part of my chin. Firm. Not movable. Can be felt inside mouth as well. Not painful. It s been there approx. 3 weeks. No obvious enlargement. Today (3/2/2017) woke with jaw pain and headache. Taken 600mg Ibuprofen. No relief. Take 40mg Prozac, 300mg Welbutrin, vit. D and B3. Non smoker, non drinker. 5 8 200 lbs. Doctor: Hi welcome to HMCYou could be having an obstrusted Salivary Gland. Esp. Bcause you are on Bupropion 300mg, which causes dry mouth by decreasing secretion of glands.See a local doctor to confirm it, as there are other posobilities as well."
},
{
"id": 43487,
"tgt": "Semen analysis of husband shows motility rate 20%. Advised Siphene for me. Reason?",
"src": "Patient: Hi doctor, I am 28 year old married female. My periods are regular. we are trying to conceive since 1 year without any luck . So we went to an infertility specialist to ensure that things are normal. Doctor did my husbands semen analysis and the mobility rate is only 20%.I am undergone trans vaginal sonography, and i am normal.But for me doctor prescribed siphene 100 mg for 5 days(day 3rd to day 7 of cycle) and advised for scan on 9th or 10th day. My question is siphene 100mg needed for me,when my scan result came as normal. Doctor: Hi there ~I understand your concerns. Since the semen analysis of your husband shows that the motility rate of your husband's sperm is 20% you might need to go for an IVF or an IUI(though less of a chance than IVF). I wonder if you have consulted another gynecologist for a second opinion. I think you should and make sure you do not leave any stone unturned at this time.I hope this helped and I wish you the very best. Take care!"
},
{
"id": 194369,
"tgt": "How to get an erection during sex?",
"src": "Patient: Hey doc. I'm 21, and in love with a girl I'm just friends with. I recently tried sleeping with a different girl, and found that I couldn't keep an erection during the sex. It was my third time having sex, and it's happened each time. I do masturbate regularly, so I believe I have E.D. I hear that vitamins can help treat it, but I'm unsure. Is there a type of medication I should be on to help with this? Doctor: Hello, Simple reason can be due to a guilty feeling since you are already in love with a girl. If you can masturbate normally and ejaculate then physically you are fine. If you have this issue with the girl you are in love it then we can worry or if its a 1st-time sex there is this possibility. Other causes for Erectile Dysfunction are : 1.\u00a0\u00a0\u00a0\u00a0\u00a0Decrease sensation of penis- can be due to infection, diabetes, spinal injury , recent masturbation 2.\u00a0\u00a0\u00a0\u00a0\u00a0Stress due to work or family related, night duties 3.\u00a0\u00a0\u00a0\u00a0\u00a0Depression 4.\u00a0\u00a0\u00a0\u00a0\u00a0Varicocele 5.\u00a0\u00a0\u00a0\u00a0\u00a0Fear of failure - having one episode of ED once may make you feel you will end up with ED 6.\u00a0\u00a0\u00a0\u00a0\u00a0Low self-esteem (Chronic Masturbation ) 7.\u00a0\u00a0\u00a0\u00a0\u00a0Homosexuality 8.\u00a0\u00a0\u00a0\u00a0\u00a0Religious constraints 9.\u00a0\u00a0\u00a0\u00a0\u00a0Negative attitude towards Your partner 10.\u00a0\u00a0\u00a0\u00a0\u00a0Hormonal issue \u2013 thyroid issue, low testosterone 11.\u00a0\u00a0\u00a0\u00a0\u00a0Long term drug usage Hope I have answered your query. Let me know if I can assist you further. Take care Regards, Dr S.R.Raveendran, Sexologist"
},
{
"id": 23332,
"tgt": "What causes low BP, lumps on neck and loss of weight?",
"src": "Patient: My blood pressure has been low for the last month. 88/58. Now I just noticed 2 marble size lumps on the side of my neck. I had a fever earlier in the week and problems with allergies. Lost 55 pounds recently and lowered salt intake. Nor sure if all is related. Doctor: HILUMPS IN THE NECK CAN BE A INDICATOR OF INFECTION OR IT CAN BE ANYTHING SERIOUS LIKE CARCINOMA(cancer)sides of the neck has many nodes called as lymph node , its function is to trap any infectious agent in the body, swelling in lymph node indicates that there are infectious agents in body,as your lymph nodes are trapping those infectitous agents ,the size is increasing however in some case swollen nodes are a indicator of carcinomaits very possible that these enlargged nodes are presssing the vagus nerve which passes in the viscinity ,vagus plays a imp role in maintaining blood pressure,increased pressure on vagus can lower the bp due to overstimulationbut a fever is indicative of ongoing infectioni would suggest youa total leucocyte countdiffrential leucocyte countcomplete blood count test to rule out any ongoing infection if its normal then a Fine needle aspiration cytology of enlarged nodes should be done to rule out carcinoma"
},
{
"id": 111872,
"tgt": "Is MRI required for a back pain?",
"src": "Patient: I found out today from my orthopedic dr that my L5 is fused to my sacrum on one side and he also diagnosed me with DDD. I am having severe lower back pain with pain going down the back of my left leg. I have not had any numbness but my reflexes were weak on my left leg. He prescribed anti inflammatory meds. I have only had xrays done. Do i need to have an mri? He said that would be the next step if i didnt get relief with meds. I have limited coverage insurance and dont want to run out of that if im going to have to have surgery. Doctor: your symptoms does suggest need for MRI. Remember MRI is the best investigation for vertebral column problems. If budget is a constrain may be you can opt for CT scan."
},
{
"id": 66787,
"tgt": "What causes lump on the buttock just below the tailbone?",
"src": "Patient: I have a painful lump in the top of my left buttcheck about an inch under my tailbone. At first i thought it could be a Pilondial cyst but it has no opening or anything. Its very when im trying to sleep and i only got it today. Any idea what this could be? Doctor: thanks for sharing your health concern with us!well, it is just a sebaceous cyst or a neurofibroma and as well could be a pilonidal cyst/sinus as you mentioned! never poke it as this may invite infection!an FNAC test could confirm it. all the best!"
},
{
"id": 5202,
"tgt": "Planning to conceive. Irregular periods. Taking fertyl. Any advice?",
"src": "Patient: hi mam im 29 years married women,now we are planning for a baby,but my menstual cycle is irregular,according to my doctors order im taking tab. fertyl from my second day of cycle to 5 th day.usually my menstrual cycle duration is 35 to 40 day, so hw many days we need to do intercourse?from 10th day to 18 th day or 10 th day to 22 day(because my menstrual cycle duration is 35 to 40 days ) thank you. Doctor: Hi, Thanks for writing to us. You can try from 10th to 22day because your periods are irregular so your ovulation is also irregular. Take folic acid. Wait till your period become regular with medication for pregnancy. Good luck. Take care."
},
{
"id": 46051,
"tgt": "What causes clogged ears while on dialysis?",
"src": "Patient: Hi - my dad is a dialysis patient for almost 3 yrs now. We were just wondering why his ears get clogged but more importantly what can he do about this? It eventually goes away but of course very bothersome and with dialysis 3 times a week it seems to be all the time - thank you Doctor: this occurs because of rapid removal of urea during dialysis which leads on to swelling of minute structures inside his ears. this revert back with time and hence his clogging becomes clear after a while. this can be minimized by decreasing the blood pump speed and by other techniques during the dialysis procedure. I think for this, it is good to discuss with the concerned nephrologist who is responsible for writing the dialysis prescription"
},
{
"id": 225378,
"tgt": "Painful arm after getting an Implanon. Is this a side effect?",
"src": "Patient: hi, i am here to ask a question about the implanon, i got the implanon placed in my left arm last thursday. i have had no problems at all untill now, all day i have had slight pins and needles from my elbow down in my left arm, the pins and needles get a lot worst when i strech my arm out. could this have somthing to do with the implanon? what should i do? please help. thank you Doctor: Hello,I have gone through your queries and understood your concern. In my opinion that must be a side effect of the implanon and mostly they will resolve by time but if it persists and too severe then you need to consult your treating doctor to check the place of the implanon and they will decide whther to keep or remove it. Thank you."
},
{
"id": 189695,
"tgt": "Been experiencing throbbing pain behind ear. Dental reasons?",
"src": "Patient: My father (83) has been experiencing a throbbing pain behind his right ear. He thought is was from a tooth ache so we went to the dentist . Everything was good, no dental Issues causing the pain. Warm compress makes it feel better. Ok during the day very painful when sleeping and In the morning. Also sensitive to touch. Doctor: hello,m according to the history u gave there are no dental problems well might be a case of TMJ dislocation well i would advice you to go for physiotherepy of TMJ. soft diet is recommended to avoid unnecessary pressure on the joint. also please see to it that the patient should not open the mouth too wide,even while yawning chin should be supported by your palm. i would also prescribe you amoxicillin 500mg twice daily with diclogesic twice daily after meal. muscle relaxants, which will take care of pain and swelling. i would also reccomend you to consult an ENT also. take care regards"
},
{
"id": 193609,
"tgt": "What could cause shorter penis, narrow at the base of shaft during erection?",
"src": "Patient: Recently when I began having erections, I noticed that my penis is much shorter when fully erect and is also much more narrow at the base of the shaft. This has never happened before. I am 40 years old. It is as if something is holding it back and preventing it from maintaining full size and girth at the base. Doctor: Hi, Phimosis. Contracted foreskin can give a picture as you say. Share a picture for confirmation. Check for blood sugar and foreskin infection. Hope I have answered your query. Let me know if I can assist you further. Regards, Dr. S. R. Raveendran, Sexologist"
},
{
"id": 68686,
"tgt": "What causes lump on the face?",
"src": "Patient: Good morning Sir,I have noticed a small lump next to a mole on my face, it has been there for a month, however yesterday some white stuff came out of it, could I assume this is a cyst or spot rather than a tumor, I have an appointment at the hospital next week, should I be concerned?Many thanks. Doctor: Its most likely to be a Sebaceous cyst which would have got burst and the stuff would have come out. The only worry now that it has burst is of infection. If it becomes red or is painful, then you might need a course of antibiotics and anti-inflammatory medicines. You can get it removed by a small operation once the infection subsides. Take care,Dr Rishi, India."
},
{
"id": 49015,
"tgt": "Should I go to the hospital if have UTI, fever 99-100 and not too bad pain as the doctor is suspecting kidney infection?",
"src": "Patient: I have a uti and I'm on antibiotics. My doctor said I possibly have a kidney infection. And that if I still had a fever to go to the hospital. My temperature has been between 99 and 100. But the pain isn't too bad if I stay in one place. Should I go to the hospital? Doctor: U need to be sure that infection has not damaged the kidney which can be seen on ultrasound scan of kidneys. If symptoms improving then wait else better to visit a doctor."
},
{
"id": 76128,
"tgt": "What are the early symptoms of pneumonia?",
"src": "Patient: I have real bad allergies and for the past week I been trying to take of the allergies part, but now I cant shake whatever it is. Im coughing up dark yellow phelm with a little blood in it and my chest is a little tight. The darinage in my sinus cavity is continuous. What should I do? Do you think I might have walking pneumonia? Please help I am very uncomfortable. Doctor: Hi welcome to HCM...According to history here you can have bacterial infection in form of bronchitis or pneumonia like infection....Consult pulmonologist without delay...Your auscultation and blood pressure checking also needs to be done ...Chest x ray then done to rule out cause..If bronchitis there spirometry also done to detect severity...Then treatment needed with cefpodoxime 200 mg tablet or levofloxacin like antibiotic for 7 days...Healthy and balanced diet taken..Avoid smoking if habit.Then if needed for further work up sputum cytology can be done .Take care Hope your concern solved"
},
{
"id": 179360,
"tgt": "What causes sore bones/stomach in a child?",
"src": "Patient: Hi my 3 year old boy complains of having sore bones deep inside he had a blood test and his platelet level was 501 he also complains of having a sore stomach after eating any ideas please, he has been out of sorts since his two year old vaccines and a usually placid boy now becomes hysterical easily and often wakes crying from pain. Doctor: Watch for gluten allergy (wheat and related products) this may be the cause of pain in stomach after eating.For other pains he needs further evaluation by a doctor."
},
{
"id": 140641,
"tgt": "What causes dizziness in mornings?",
"src": "Patient: i wake up in the morning feeling dizzy, its like i am going to have a fever and keep on farting, and had a pain on my upper stomach its like there is a heavy feeling inside of it. I checked my blood pressure and it is 90/60. i need your advice on what should i do. thank you. Doctor: Hello, Since your blood pressure is low 90/60 this could be the cause for you feeling dizzy in the morning. Getting a cardiac assessment done should be of choice. Because 90/60 is considered as low blood Pressure and it needs cardiologist attention. Now for gastric, you need to improve your eating habits and also metabolism so gastric issues don't prevail longer. You should consider meeting up a cardiologist and not ignore the low blood pressure. Hope I have answered your query. Let me know if I can assist you further. Take care Regards, Jay Indravadan Patel, Physical Therapist or Physiotherapist"
},
{
"id": 64845,
"tgt": "What causes ridge on the head that hurts while running?",
"src": "Patient: I have recently noticed a ridge on the top of my 9 year old daughters head that wasn't there before, she complains that it hurts when she runs ( can feeling it bouncing and hurting). Wondering what it could be.She is otherwise healthy, no colds, sore throat, or fever. Doctor: HI,Dear,Thanks for the inquisitive query to HCM.1-I studied your query in depth.2-In my opinion , the ridge on top of your daughters head -indicates -Cranio-syn-ostosis , 3-Please show it to the paediatric doctor and keep a watch on the -Skull Circumferrence-and mental development and school progress of the child under doctors observation.4-Hope this would solve your query and worry for your 9 yr old daughter.5-Wellcome for more queries to me On HCM.6-Thnks once again for your inquisitive query.Have a Good Day..!!"
},
{
"id": 14807,
"tgt": "Suggest treatment for rash on buttocks and thigh area",
"src": "Patient: dear doctor, i have been having rash on my buttocks and tight area and also dark, someone advice me to mist Tenovate GN and M into my cream, that it will clear d rash and brighten my colour, l have use it 3weeks now, but still rashes on my buttoch, wht do i do now? Doctor: HIThank for asking to HCMI really appreciate your concern looking to the history given here this could be hypersensitivity reaction and such reaction is always idiopathic, in my opinion such condition should be best treated with the \" Tab Levocetrizine 10 mg three times in day and Tab Hydroxyzine 10mg twice in day\" no need to apply any thing, hope this information helps you, take care and have a nice day."
},
{
"id": 15623,
"tgt": "Itching, scaling, burning like red patches spreading all over left foot, toes and legs. Is it curable ?",
"src": "Patient: WHEN I WAS IN 1ST STANDARD MEANS 6 YRS OLD I WAS FALLAN DOWN I GOT SUM PAIN AT MY RIGHT LEG NAIL SIDE. I TRIED MANY DOCTORS BUT NOT GOT CURED AND IT\u2019S SPREAD MY TOW LEGS. SUDDENLY IT\u2019S DISAPPEARED WHEN I WAS IN 22 YRS OLD. LAST 10 YEARS IT WAS NOT THERE. SUDDENLY IT\u2019S APPEAR ON MY FOOT LEFT SIDE. I TOOK TREATMENT IN HOMEOPATHIC BUT INSTEAD OF CURING ITS SPREADING ALL OVER MY BODY. RIGHT NOW MY AGE 30. ACTUALLY BEFORE I DON\u2019T KNOW WHAT KIND OF DECEASE IT IS. ACTUALLY IT IS PSORIOSIS. ALWAYS IT ITCHING, SCALING, BURNING ITS LOOKING LIKE RED PATCHES . I WANT KNOW ITS CURABLE OR NOT. Doctor: Hello,Thanks for the query.Yes it is psoriasis.Although there is no definitive cure but psoriasis can be controlled.Once remission is attained drugs can be tapered and you will require periodical follow up with your treating doctor.It will take its own time but effective treatment options are availableLet me know if you have any other doubt.you can ask a direct question to me on this forum, following the below link.https://urldefense.com/v3/__http://www.healthcaremagic.com/doctors/dr-rahul-kumar/64818Wishing__;!!Mih3wA!SBzm6_kI6hCZ58EPH6N_05MFfiPbxWXT0a2TJCdFQObRWm5mV5ur7hUOMa8clQ$ you a good health.Thank you"
},
{
"id": 16692,
"tgt": "Can Amiodarone be replaced with Sotalol post a cardioversion procedure?",
"src": "Patient: I m going back into the hospital tomorrow to have another cardioversion. (My 3rd try at this) The first time to 3 shocks but my heart stayed in AF. I was put on Defedilide for 3 days and then had my 2nd cardioversion and my heart stayed for about 4 days. I was taken off the Defedilide and put on Amiodarone for 7 days for my next cardioversion tomorrow. First I m not comfortable with the be side effects of this drug and would like to get off of it as soon as I can and would like to go on Sotalol if possible. I only have one kidney. I lost my left kidney to cancer in 2010. Doctor: Hello, After going through your medical query I understand your concern and I would like to tell you that amiodarone is anti-arrhythmic which can help in cardioversions but sotalol will not help much but control heart rate, so preferably stay on Cordarone. Hope I have answered your query. Let me know if I can assist you further. Regards, Dr. Bhanu Partap, Cardiologist"
},
{
"id": 162667,
"tgt": "Should formula milk be changed if an infant suffers from stomach cramps?",
"src": "Patient: My grandson is a preemie. He is now almost 8 weeks old. He spent almost 7 weeks in hospital. At first he had to be fed mostlt thru nose tube. He gradually was dismissed because he ate all formula thru bottle. He has been home 1 week now. He came home on. Similac neosure preemie formula. Gold can. Seems his fussiness is getting much worse. It is worse after he eats and he cries as if in pain from a cramping. He just doesn t seem to be happy at all. Do you think another formula might be the answer.. Thank you Cindy Renquist YYYY@YYYY Doctor: Hello and Welcome to 'Ask A Doctor\" service. I have reviewed your query and here is my advice. I would like to know how many weeks was the pregnancy and what his birth weight and current weight are. Has he been on Similac Neosure Preemie Formula since birth? When did he start to become fussy? Does he cry all the time? In what position is he when fed? Does he spit up or vomit? If you hold him upright while feeding and for 20-30 minutes afterwards and he doesn't cry, then it is possible that he could have reflux of formula out of stomach and back up into esophagus. Because premature infants weigh less than full-term babies, formulas for premature infants contain more calories per ounce than formulas given to full-term babies. I do not recommend changing formula. Continue the same formula that was given in the hospital. I do strongly recommend your grandchild be seen by his pediatrician as soon as possible for evaluation. Hope I have answered your query. Let me know if I can assist you further. Arnold Zedd, MD, FAAP"
},
{
"id": 200921,
"tgt": "Suggest treatment for bleeding penis post masturbation",
"src": "Patient: Hi I know this is awkward but I was just masturbating when I felt a weird feeling close to ejaculation. I thought it was normal and then shot but it was blood that came out. Im pretty scared, I have been masturbating a lot and want to know what I need to do. Im frightened. Doctor: Thanks for asking in healthcareamgic forumIn short: Is it from urethra or from outside structures?Explanation: Hope you did not injure yourself while masturbating. Was it from urethra inside or from outside structures? If its from urethra stones/Urinary tract infection can cause this. If not cut/bruise to surrounding structures can cause bleeding. Any ways visit a doctor if you cannot make out."
},
{
"id": 137915,
"tgt": "Suggest remedy for burning sensation in legs",
"src": "Patient: What is causing a burning sensation on top of my Lt foot 3 inches from the beginning of My toes in a circular shape. I noticed that I got a pedicure for the first time 2 years ago and I noticed a puffy darker at the cuticle of MT big toe and second toe. My gglucose has been in the normal limits. So please don t tell me I am becoming a diabetic. Doctor: Dear Sir/MadamI have gone through your query and read your symptoms.In my opinion, the symptoms you describe are suggestive of nerve involvement, and mainly peripheral neuritis, or compression neuritis can cause such symptoms, now since you say you are not diabetic, lets concentrate on other causes, and the next major reason for peripheral neuritis is Chronic Alcoholism and long term use of certain drugs. so make sure if you fall in one of those categories. also make sure that you do not have any spinal problem or pain, as nerve compression can also give such symptoms, moreover local compression can also cause such symptoms. you need to consult a neurologist, and work with him to find the cause. I hope that answers your query. If you want any more clarification, contact me back."
},
{
"id": 1072,
"tgt": "Will i get pregnant after taking Ovofar and Duphaston?",
"src": "Patient: Hi... My name is Vidya. 24 yr old married last 1n half yr. Our doctor prescribed me to take Ovofar from my 3rd day of period for 5 days. and Duphaston 10mg from 17 th day for 10 days. so my Q1) Is there is any side effect taking this tablet. Q2) What is the use of taking these tab. Q3) By taking these tab can get pregenent on the same month. Plz answer to my Q. Thank You Doctor: Hi, these medicines are given to increase the chance of pregnancy. Ovofar is for increasing the growth of your follicles. Duphaston is given to support the implantation. Side effects are minimal. So don't worry about it. You can get pregnant the same month. Be in contact with your husband every 2 to 3 days after your periods stop. It will increase chances of your pregnancy. Hope I have answered your question. Regards Dr khushboo"
},
{
"id": 111264,
"tgt": "Suggest permanent cure for back pain",
"src": "Patient: Hi, i am 41 years old , 6 ft H and I have back pain since more than 10 years at lower and higher part but not going to my leg and i can walk and run without problem but this pain is continous and sometimes is severe. please advise if this series as i am worried. Doctor: Beat thing for you would be good regular back strengthening exercises. These specific exercises and hot fomentation for relieve stiffness before exercises will help you in long run."
},
{
"id": 30731,
"tgt": "What causes spreading, itchy rashes on skin below the wrists?",
"src": "Patient: Hi, I came to Chennai in July. Last week I developer rashes on skin below wrists on both hands. I sweat heavily and there is itching on the spots. There are small patches turning red now and today are a little higher and looks like they are spreading. Doctor: Hi,From history it seems that there might be having some allergic rashes might be due to some systemic allergen or local allergies due to handling some chemical or using some thing giving rise this allergic reaction.Take Benadryl or levo-cetrizine medicine for 2-3 days.Apply Caladryl lotion locally.Ok and take care."
},
{
"id": 126216,
"tgt": "Suggest treatment for pain and swelling in the inner knee",
"src": "Patient: I have had pain behind my knee and in my calf for several weeks. I did have some pooling of blood in one area of the inner side of my calf a couple of weeks ago, and also swelling in my ankle. I am feeling confused because I have had pain in the knee area since I broke my kneecap 9/15. Any ideas of the possible cause? Doctor: Hello, Consult an orthopedician and get evaluated. An MRI scan will be helpful to look for possible causes like effusion. Hope I have answered your query. Let me know if I can assist you further. Regards, Dr. Shinas Hussain, General & Family Physician"
},
{
"id": 218450,
"tgt": "What does an ultrasound scan of 20 weeks of pregnancy looks like?",
"src": "Patient: I m 35 years old pregnant with my second baby.According to the ultrasound..its single live intrauterine pregnancy in frank breech presentation with good cardiac and somatic activities.20 weeks and 5days by composit aging. Placenta right posterolateral grade 1. Lowest placenta edge 6.2 cm. away from the internal cervical OS. Normohydramnios 4.3 cm sVP. Fetal weight is 383 grams. Is the result is normal for my baby in 5mo. Doctor.? Do I need to get worried about being Caesarian. My 1st baby boy is normal delivery.. Thank u if u can answer my what bothering me now.. Doctor: Hello and Welcome to \u2018Ask A Doctor\u2019 service. I have reviewed your query and here is my advice. In my opinion, your 20 weeks pregnancy USG scan is within the normal limit. Your baby is growing well. You need not to worry at all. You should remain relaxed and happy during your whole pregnancy period. Don't think too much. Go for routine your check -up as advised by your gynecologist. Hope I have answered your query. Let me know if I can assist you further."
},
{
"id": 8069,
"tgt": "I would like to know what can be done for pimples?",
"src": "Patient: would like to know what can be done for pimples and pimple marks I would like to know what can be done for pimples and pimple marks I am 22 unmarried and working as HR. from past 2 months i have black heads slightly improved but marks r still there will chemical peeling help reduce marks? Does it have any side effect Doctor: Hi!Richa, Welcome To HealthcareMagic forum, Before telling you details about pimples ,let me tell you first about chemical peel.Its a very safe procedure if done by a dermatologist and it can be done even if you have pimples .It works both for pimples and dark spots.The best peel would be salicylic acid peel . Now, pimples are multifactorial ,and therefore one has to find the exact cause of it. It can be because of wrong use of soaps to moisturizers or any fairness creams which one is using , to hormonal disturbances if anybody has(esp in females).So,a complete work up is necessary in all patients.Also one should remember that its not a short course treatment,but a matter of say 6 months or so if pimples are sever . Firstly I would advise you to stop all cosmetic creams if you are using on face. Also look for other features of hormonal imbalance in body like weight gain,irregular periods,facial hair & scalp hair loss.If you have any of these then get a hormonal profile done.In such cases conventional treatment(antibiotics) for acne would not work,and one needs hormonal tablets to cure acne. In case you don't have any hormonal imbalance & you don't respond to conventional treatment, then one can take retinoids .Retinoids is a wonderful medicine for acne ,provided one gets all the relevant investigations and takes care of all the precautions associated with it.Along with chemical peel ,this would give you a flawless skin. I advise you to consult a dermatologist and take proper treatment . After few days my web site would be launched (www.skinhairclinic.com),you can get details and pictures about pimples and their treatment. If you have any further questions you can write back to us. regards, Dr.chawda"
},
{
"id": 36135,
"tgt": "Can rings cause any infection?",
"src": "Patient: I have a belly peircing and i had a dangle ring in it. My boyfriend and i have tons of crazy sex all the time. last week we were in the hot tub, had some more sex and now he has small red bumps on his stomach around the belly button area. Is this just irritation from my belly ring? Doctor: MAY BE ,CONTACT DERMATITIS DUE TO REPEATED IRRITATION. APPLY QUADRIDERM CREAM FOR A WEEK.CONSULT YOUR DOCTOR FOR PHYSICAL EXAMINATION,TO RULE OUT DISEASES DUE TO WATER."
},
{
"id": 101291,
"tgt": "What can i do to reduce puffiness of my face?",
"src": "Patient: hi i have been taking cleocin since 1230 on wednesday prescribed to take 2 initially, followed by another 2 after 6 hours now i am taking 1 every six hours the problem side of my face has steadily swollen to the point where the puffiness has reached my eye can i do anything to reduce my problems Doctor: HI, thanks for using healthcare magicIf the swelling has occurred as a reaction to the use of the antibiotics then you may need to contact your doctor about an alternative.The use of an antihistamine would help to reduce the swelling if related to an allergic response. Anti inflammatory pain killers such as ibuprofen/naproxen/cataflam may also be helpful for you.You may need to see your doctor for a reassessment.I hope this helps"
},
{
"id": 154418,
"tgt": "Can Dexa scan detect cancer of the bones?",
"src": "Patient: Can a dexa scan show if cancer is in the bones? My husband has advanced prostate cancer,did radi for 9 weeks,the Dr still says not sure if that worked?? He has really bad bone pain,is on Morphine x2 daily with little relief,had a bone scan that one Dr says showed alittle uptake in hips,but the only thing they want to do right now is this dexa,I just need to know if cancer will show up in this! Thank you,from a worried wife!! He is 6 3 238lbs Doctor: Hi, dearI have gone through your question. I can understand your concern. You have history of prostate cancer. If is known for bone metastases. Dexa bone scan is useful in detection of any none involvement. Then you should take treatment accordingly. Hope I have answered your question, if you have doubt then I will be happy to answer. Thanks for using health care magic. Wish you a very good health."
},
{
"id": 93371,
"tgt": "Pain from rectum to stomach and stomach cramps. What is it?",
"src": "Patient: I am having horrible pain shoot up my rectom into my stomach and sometimes back Down to my vagina. I have had stomach craps all day but, have had stools today. And yesterday. Pain was so bad thought I was in labor. Wouldn't hardly walk let alone sit. I laid down and felt a little relief. Went to the bathroom again to pee and the pain came back just like that. What could it be? Do I need to go to the hospital? Help. Doctor: Hi ! I would like to have a few more information from you: 1) Whether you are having any kind of anxiety !! 2) If you are being ever diagnosed to have hemorrhoids ! 3) How are your food and bowel habits ?The pain of this kind could be without any organic disease process which is seen in some anxiety states. This is called Proctalgia fugax. This is a recurrent pain in the rectal area which may also radiate to other perineal part. The pain could be so excruciating that the person can faint at times. It is advisable to lie down during the attack, and to massage the area in front of the rectum at the initial period of pain. You can also take some anti anxiety drugs if advised by your doctor. Sitz bath ( siting on a tub with warm water) also helps sometimes.However, if the symptoms increase and persist, you should get reviewed by your doctor to some organic causes of such pain, such as, prolapsed hemorrhoids, irritable bowel syndrome, or any infiltrating growth in the rectal area.Wishing you an early recovery."
},
{
"id": 191684,
"tgt": "Suggest treatment for hypoglycemia",
"src": "Patient: My normal blood sugar level is between 60 and 70 when I am not fasting and when I am. It spikes to up to 200 when I eat and than drops rapidly back to 62 within an hour or two. My Grandfather on my Dad's side was diagnosed with hypoglycemia and my Grandfather on my Mom's side was diabetic. Should I be concerned about the spikes or where it regulates itself?? Doctor: Hello, Thanks for the query.I have seen the details. My suggestion is get a oral glucose tolerance test (OGTT) done. That will give a clear indication about whether you have any problem with blood glucose. Secondly if persistently low glucose levels are seen and are causing any problem it is better to get your C-peptide level to see if there is any excess insulin. Another important point in a normal person even when one eats any sanck blood glucose level does not go above 140 mg. So if it going beyond that (200 mg) then it appears to be presence of diabetes. Therefore, please get an OGTT done. Thanks."
},
{
"id": 19335,
"tgt": "What would be a 12 year old's predicted heart rate?",
"src": "Patient: Hello Doctor! I would like to ask what would be a 12 year olds predicted heart rate? Could you help me please? Could somebody help because it is very important for me and my child could be suffering if i do not find out any sooner! Even if you are not a doctor of this kind of things please can you help me however much you know? Doctor: welcome to hcm. ..predicted heart rate of 12 year old boy is between 60 to 80 ...but what is problem with your child ..please visit emergency as there are many causes of rapid and slow heart rate ..some are not so dangerous but some are dangerous ..so your child need evaluation"
},
{
"id": 200753,
"tgt": "How does HCG injection dosage work?",
"src": "Patient: Hello Dr. Rynne, I am Ibrahim 28 years old man and it is two years that I got married, but no kid yet. Finally my physician told me that your Y chromosome is negative, therefore I have to use HCG injection to get sperm. so I want to ask you how dose HCG injection work? Doctor: Thanks for asking in healthcaremagic forumIN short: If Y chromosome is negative then you are not a male.Explanation: Please visit a andrologist/gynaecologist for opinion. If Y chromosome is not there thenthe individual is nat a male and it is not possible to get sperms without Y chromosome. PLease send your reports for any suggestion. I think you are confused."
},
{
"id": 189027,
"tgt": "Bad taste in mouth, cold, severe pain in jaw, gums and teeth. Detected slightly inflamed gums. On ibuprofen. Help",
"src": "Patient: my problem started last week. i was exhausted and had a horrible taste in my mouth. i have a slight cold too. on monday i had a terrible headache and stomach ache but the real problem is my jaw gums and teeth, im in agony. the pain is intense i dont know what to do,ive been to the doctors she gave me strefen to help and nystan just in case and the dentist he said my gums were slightly inflamed.he told me to floss. im using corsadyl mouth wash and am having to take 600mg ibruprofen every 3 hours to try keep the pain under control.the pain wakes me up and is stopping me from getting on with my day to day life. Doctor: Hello,Thanks for the query.your teeth pain is due to gum infection.poor oral hygiene aggravates this condition.Its nice that you are using pain killers and mouth washes.you can also do frequent warm saline rinses till you visit your dentist.In this case through examination along with professional cleaning of gums is must.careful examination might also lead to know if there is deep pockets in your gums and if there is any bone loss too.Also get an x ray done.you can start antibiotics upon consultation.You can ask me a direct query on this forum following the below link.http://www.healthcaremagic.com/doctors/dr-anshumala-singh/65146Thank you"
},
{
"id": 186686,
"tgt": "Suggest treatment for swelling and puss formation",
"src": "Patient: Hi, I have swelling around a crown and puss is coming out of it. Foul smell and taste, however, very little pain, slight tenderness to touch. I went to the dentist a few days ago and he irrigated it and said to come back in a week or so. It seems to be getting worse and I wanted a second opinion. Thank you :D Doctor: thanks for your query, the pus discharge from the gums around the crown can be treated with a course of antibiotic followed by curretage (deep cleaning of the gums) if there is gum infection. other wise if tooth is infected it has to root canal treated(if not treated or not properly treated) and crown has to be replaced..these conditions can be confirmed by radiographs... consult your oral physician for needful...i hope my answer will help you..take care.."
},
{
"id": 218046,
"tgt": "What is the cause of heel pain after walking on the treadmill?",
"src": "Patient: I walk on the treadmill for 45 minutes daily except Sundays but when I am finished my heel hurts on the left foot. I am diabetic but I do not eat before exercising so what is causing the pain? It does not last all day but the pain remains on the left foot only. The right foot has no problems. Doctor: HIThank for asking to HCMI really appreciate your problem, the hill pain while walking on treadmill could be due to the uneven gate while walking you have to take maintain the good gate while walking, have rest for few days this will be alright, hope this information helps you have nice day."
},
{
"id": 86267,
"tgt": "Suggest remedy for intense abdominal pain",
"src": "Patient: Hi! I m Lea and I m 16. This evening I had really intense pain in my lower left abdomen. It was the worst pain I had ever experienced. It got better with pressure but now it s getting worse again. The pain is now all over my stomach but its origin in my lower left side. I also feel cold and shakey and a little achy. I also have been urinating more than usual and there has been some yellow/green discharge in it. Doctor: Hi.Thanks for your query.Noted your history of more frequent urination with some yellow/green discharge in it. Intense pain in lower left abdomen, getting better with pressure. now the pain is all over abdomen. I would advise you the following in such a situation. First of all send urine for tests of routine, microscopy, culture and sensitivity before antibiotic is started to get a proper reports which help in further treatment. Further tests of blood, stool ultrasonography of the abdomen and so on.Clinical evaluation and examination by a General Surgeon to have a proper diagnosis and further proper treatment.I hope this answer helps you to get an early and proper diagnosis and treatment."
},
{
"id": 167432,
"tgt": "What causes painless dark black patch below tongue in children?",
"src": "Patient: My 3-year old granddaughter saw a dentist yesterday - found a large dark/black spot on back of tongue - were concerned, made appointment for EENT -- mother is my adopted-from-India daughter. This has never been noticed before, no pain, no known injury We re really concerned - should we be?? Doctor: Hello there,Thank you for posting your query with healthcare magic,I would like to inform you that unless the lesion(black spot) is increasing in size over years, it is not much of a concern.Also, these kinds of spots are sometimes hereditary and do not cause in problems in future.But only because it was noticed by your dentist and ethically it is advisable to get it checked by a specialist, therefore it is always best to be safe.Kindly see the specialist at your best time, and stay calm.i hope i was able to answer your query,if you have any further questions, kindly feel free to write back.thank you"
},
{
"id": 94559,
"tgt": "Stomach pain due to inflamed bladder, high count of white blood cells. How to normalize?",
"src": "Patient: i am having stomach pain and i can not stand up straight without it hurting. i went to the er room on monday 12/24/12 and i was diagnose with immflamate bladder and they also said i have count of white blood cells, this has been going on for about a month. and also my stomach is big as well but i am not pregnant nor do i have tumor .. can you please help me out Doctor: Dear madam, uroinfection can cause symptoms u re describing but if it is prolonged and antibiotica arent helpful then you should try to find another causes. You need to do US or CT scan of abdomen and pelvis to rule out some intraabdominal disorder. Also you should do urinoculture and repeat blood tests. Inflamed bladder usually doesnt go with such kind of pain. After u do tests i ve mentioned everything should be clear. Wish you good health"
},
{
"id": 180301,
"tgt": "Suggest medication for autism",
"src": "Patient: Hi, this is sumita 4m Kolkata, India. I have a kid of 2 yes old and a local homeopathy doctor said that my child is autistic. During my pregnant I had few complications like I couldn t eat any proper food and medicine due to vomiting tendency. And my baby was in footing breech poison and her weight was very low. So my doctor gave me gro9 to increase babies weight. Can it effect my child brains? l Doctor: Hello Sumita, Firstly a local homeopathy doctor cannot diagnose autism. There are special questionnaires and development tests which help in making the diagnosis. I advice you to consult a paediatrician for a detailed evaluation. Secondly, eating less or vomiting during pregnancy does not cause autism. You should stop worrying and you should definitely not blame yourself. But please first get the diagnosis confirmed. Wishing you good luck.Regards,Preeti"
},
{
"id": 180589,
"tgt": "How can dry mouth be treated while on Doxepin?",
"src": "Patient: I have been taking Doxepin since 2003 for my fibromyalgia to help me stay asleep. I have developed dry mouth, burning tongue in the last 2 months. What medication can I use, to replace Doxepin, that will not interfere with my arthritis medications. Arava and Plaquenil Doctor: Hello and Welcome to \u2018Ask A Doctor\u2019 service.I have reviewed your query and here is my advice.Dry mouth and burning sensation in mouth is due to the side effect of Doxepin. You should consult your treating Physician and get evaluated. If needed, the dose can be titrated or the drug can be substituted. For dry mouth, chew sugar free chewing gums. Gargle with specialised mouthwash like Magic mouthwash. Suck ice pops to reduce burning sensation. You can use oral lubricant like Oralgel. Drink plenty of water.Hope I have answered your query. Let me know if I can assist you further.Regards,Dr. Honey Arora"
},
{
"id": 56335,
"tgt": "What does GGT level of 260 indicate?",
"src": "Patient: Does a ggt iu/l of 260 really matter? my alt is 60, AST is 31, Alkaline Phosphatase is 104, BILIRUBIN IS .16 MG/DL WITH TOTAL BILIRUBIN MG/DL AT .5, ALBUMIN IS 4.2, total bilirubin is 4.2, and liver fraction is% 64 should I do any thing or just forget it? Doctor: HelloYour findings suggests raised GGT,ALT and total bilirubin level.Increased GGT and ALT may indicate liver injury.It may be due to many reasons like hepatitis,alcohol intake etc.You may need few more investigations like routine hemogram,viral markers,ultrasound of abdomen.I suggest tablet ursodeoxycholic acid 300 mg twice daily for three months to my patients.It helps in regeneration of liver cells.I also suggest to take vitamin B complex capsules.Vitamin K can be prescribed in needed.I suggest follow up test after one month.Avoid alcohol if you take it.Get well soon.Take CareDr.Indu Bhushan"
},
{
"id": 83314,
"tgt": "Could the pain in right pelvic area be due to side effects of rifafour pill?",
"src": "Patient: I have pain in my right pelvic area. I had a cyst before, and the Doctors performed a procedure where they drained water / pulse to make it go away, that was done in December 2011. I had previously (in April 2011) had my apendicitis removed and absess drained in my stomach. Later they disovered that I have TB in my abdomen. I started taking Rifafour on the 2nd of December 2011, could the pain in the right pelvic area be side effects from taking RIFAFOUR pills Doctor: HiAbdominal pain can occur as a side effect of TB medicines.Abdominal pain may also be related to ovarian causes.TB drugs can raise liver enzymes which can also cause abdominal pain,jaundice,clay colored stools denote liver inflammation or damage.I would suggest to take abdominal , pelvic ultrasound and liver function test which can identify the cause of pain.Hope I have answered your query. Let me know if I can assist you further. RegardsDr.Saranya Ramadoss, General and Family Physician"
},
{
"id": 59520,
"tgt": "Alcoholic with severe knee pain. Gall bladder distended, UTI. Referred to gastro enterologist. Why?",
"src": "Patient: My mother-in-law has been weak, sitting in her recliner 24/7 (due to knee pain ) and not even going to the bathroom. She has been using the bedsite portable potty for about a month. She has a history of alcoholism . We just got results of blood work--no sign of hepatitis . Abdominal scan showed gallbladder was distended. She also has a silent UTI. Still has dark urine. Today we were told we were being referred to a gastroenterologist . What are they looking for? Doctor: Hi thanks for your question. Alcoholic,,Knee pain, silient UTI and finally distended gall bladder. The treating physician has to fix a priority,which condition is more serious and needs urgent attentation of a specialist.So I think you should have asked this question from treating physicion who may be able to explain you better.In my openion reason for referal to the gasteroenenterologist could be a very much distened gallbladder,which may rupture leading to biliary peritonitis with high motality.The cause of distension of gallbladder must be the one which could be managed by non surgical means . Hope this answers your question."
},
{
"id": 123803,
"tgt": "Suggest treatment for dull aching pain in hip radiating to the leg",
"src": "Patient: I am experiencing a dull aching pain in my right hip that travels down the side of my buttocks, the side of my leg just past the knee. It has been present for the past three days. It keeps me awake until I find a psosition that reduces the pain. Usually laying on my back with pillows under my right knee. Doctor: Hello, As you have this radiating pain it sounds to me the pathology of the lumbar spine and the nerve root entrapment. Due to which the symptoms are followed down the line of the waist in the lower limb. Using the lumbar brace, hot water fermentation will help reduce the muscle spasm at the lumbosacral region. Taking an MRI will be of good choice to understand the soft tissue abnormalities. Kindly do McKenzie maneuver as it will centralise the pain and use hot water fermentation after that. You can find about McKenzie maneuver online anywhere. Post the centralising of the pain do core stability and lumbar spinal muscle strengthening exercises. Also focus on lower limb strengthening this will help correct the posture. In my clinical practice of over 12 years, most cases with lumbar Radiculopathy recover with conservative therapy in 3-4 weeks of time. Hope I have answered your query. Let me know if I can assist you further. Regards, Jay Indravadan Patel, Physical Therapist or Physiotherapist"
},
{
"id": 81641,
"tgt": "Why does breathing stop in sleep?",
"src": "Patient: hello doctor,i am female 25 years,i have problem,when i am sleeping,about to be in deep sleep,suddenly my breathing stop,and due to this my heart beat increases and i wake up,it happens daily,evening time acidity is the problem, so please guide me,as i am not getting proper treatment, Doctor: Hello,Thank you for asking at HCM.I went through your history and would like to make suggestions for you as follows:1. I would like to know more about your symptoms like whether you have recurring nose allergies, nose obstruction, sinus symptoms, stress, etc.From your given history, I would think of 3 possibilities:a. It may be obstructive sleep apnea, usually due to nose obstruction due to allergic rhinitis, sinusitis, etc or some other obstruction in upper respiratory airway. In that case, I would suggest you to get examined by a ENT specialist who will examine your nose from inside and may identify the cause.b. It may be gastroesophageal reflux causing choking at night. As you have acidity, this is also a possible cause. In that case, I would suggest you to take an antacid like ranitidine daily before meals for at least 4 weeks.c. It may be night terrors, especially if stress is a problem for you. In that case, relaxation and consulting a psychologist may help you.Hope above suggestions will be helpful to you.Should you have any further query, please feel free to ask at HCM.Wish you the best of the health ahead.Thank you & Regards."
},
{
"id": 28754,
"tgt": "Suggest treatment for headache, sore throat and body pain",
"src": "Patient: Symptoms of an 18 year old male seem to be flu like: headache, sore throat, aching everywhere, feels light headed like he will pass out but doesn t, but the weird symptom is that he woke with a rash and redness on the first (largest) knuckles of each hand. He has had a fever ranging from 99.5 to 102.4 and it comes and goes. Any idea if this is something to be concerned about. Also his neck and back are aching, but so are his hips and most joints. Doctor: Hello and Welcome to \u2018Ask A Doctor\u2019 service. I have reviewed your query and here is my advice. Does he has recurrent sore throat? or Chest palpitations? His joints are painful and also has rash and all above symptoms goes towards a streptococcal infection. Or rheumatic fever/heart disease. Though you have not mentioned about his heart condition. I suggest you to go to a medicine specialist. Do not ignore. Hope this will help you Get well soon Thank you"
},
{
"id": 2686,
"tgt": "How long will it take to get pregnant after stopping provera?",
"src": "Patient: hi I am trying to get pregnant and I was prescribed provera for ten days my cycle came on and the ten days are up. Me and my husband have not used a condem since then. My question is how long would it take to get pregnant after getting off of provera ? Doctor: Hi,Provera contain progesterone and use in person who have luteal phase abnormality. Treatment response is variable. You can use it for 4-6 months, but make sure that ovulation and ovarian follicle study is normal.It will make uterine environment favorable for implantation and support pregnancy. You can start folic acid tablet also. Avoid stress, take healthy diet, drink plenty of water, maintain proper pelvic hygiene.Hope this may help you. Contact further if follow up needed. Best regards,Dr. Sagar"
},
{
"id": 107614,
"tgt": "Need medication for soreness in back & bottom right side of the body",
"src": "Patient: I have a really sore back, bottomr right hand side and its giving me difficulty sleeping at noght, walking and even when I laugh. I ve also been really light headed, lostt my balance yesterday and fell into a wall, today I ve woke up and the pain has stayed at my back but also travelled to my neck. My friends have said my eyes look funny and black, and I have been experiencing numbness in my thumb. Also I have lost my appetite and do not feel like eating, when I do my very mouth hurts. Help please! Doctor: hello welcome to Health Care Magic I am your physician Dr Prakash I think you did not hit your head but if you did it your head or your spine then I should suggest you should have MRI of head and spine as I think you should be suffering from neurological problem and should consult a Neurologist"
},
{
"id": 58089,
"tgt": "Blood tests showed high gamma gt level, taking diclofenac for possible GOUT. Am I likely to damage my liver?",
"src": "Patient: HelloI recently had a blood test done and had gamma gt levels of 1600 !!! The first question I was asked is if I drink. And i do 5 or 6 pints ecery Friday maybe 1 or 2 on Saturday but not a drop from Sunday - ThursdayI am trying to find nthe reason behind the raised Gamma GT levels, and I have noticed a trend. I have been using Declofinac for the last 6 or 7 years and noticed that ecery blood test ( 3) that I have had over the last 3 years was done whe nI was taking Dyclofinac for possible gout. The blood tests were done to determine if it was gout or not. Could the Dyclofinac be the reason behind teh high Gamma GT levels ? Is there another pain reliver that can help with joint pain (it has turned out no to be gout) ? without damaging my liver. Thank You Doctor: Hi and welcome to HCM. Thanks for the query.Dyclofenac may be cause of elevated ggt but not so significankty so you should rule out other possible causes of ggt such as alcohol intake,hepatitis, muscle damage or some metabolic disorders. also it can be elevated in smokers too. You should repeat LFTs in several weeks and see is there any progression or improvement. if this is still high you should do liver biopsy and other tests for further evaluation.Wish you good health. Regards"
},
{
"id": 219736,
"tgt": "Elaborate on non-calcium channel beta blockers",
"src": "Patient: My husband and i have been trying to concieve for almost 2 years now. The last time I got pregnant and miscarried (2nd time) was in November. We went to see a fertility specialist and he asked about medications my husband takes. He thinks his amlodipine is the problem and recommended my husband go on a non-calcium channel beta blocker. Is there such a thing as a non-calcium channel beta blocker? Doctor: Hi there,Beta blockers are available for hypertension which can be used, they are different from calcium channel blocker group, but they too have side effects and discuss them completely with the physician before using them.Since you have been trying to conceive for 2 years now , you need to get completely evaluated, for the same.Get blood tests for hormonal levels, ultrasound to check womb and ovaries and dye test to check for tubes.Your partner's semen analysis is also essential. If he is having further issues you could consult a andrologist and /or consider intrauterine insemination.Take folic acid while trying to conceive.Hope this helps.Regards."
},
{
"id": 85594,
"tgt": "Is early periods a side effect of taking Microlite?",
"src": "Patient: Hi there, I have been on Microlite pill for nearly 2 years. Last month my period came one week early ( still had a week left to take it and I did) . Whilst this month it has arrived with 9 more pill days to take. Is this normal, do I need to change pill? Doctor: Hello, When there is some hormonal imbalance then you may have early or delayed periods even if you are on Microlite. Yes, you can get clinically assessed & examined by a gynae. Other medical conditions like thyroid issues, PCOS, anemia, cervical cyst, fibroid, polyps, etc can be detected & treated at the earliest. You can change your BC pills if you so desire. Hope I have answered your query. Let me know if I can assist you further. Take care Regards, Dr Nupur K, General & Family Physician"
},
{
"id": 128757,
"tgt": "How to treat lump under incision post acl reconstructive surgery?",
"src": "Patient: Hi Dr. Siegel,two and a half weeks ago, I had an acl reconstructive surgery done on my left knee. My surgeon used an allograft and the surgery went well. However, I have a lump under the incision site and it's semi hard. I wanted to know if this is normal? Doctor: hiI understand your concernthis semi hard lump most probably is the screw which is used to fix the ACL graft to your shin bone it may be somewhat long or not sunken in the boneif its bio absorbable it quite simple don't need anything it will resolve, but if its titanium you should consult your doctor."
},
{
"id": 74303,
"tgt": "What can be the cause the chest pain, bowel problem and urinary issues?",
"src": "Patient: I have scoliosis. I was told in my 20 s. They acted as if it was no big deal. I also have degeneritive disc diease and have had 6 surgeries. Now my back legs every where I hurt. The only that helps is wearing a back brace and I see a doctor that aline my bones and my tail bone will not say alined. i cant breath and have chest pain and vowel issues and urniary issues as well. my hips have been out of line. I have been tested for everything and they say I am health but I don t feel that way and no one has said anything about this. Doctor: Respected user , HiWelcome to Healthcaremagic.comI have evaluated your query thoroughly .* Chest pain has vivid etiological factors ranging from cardiac , pulmonary , rib cage or systemic problems related need detailed evaluations and analysis .* Bowel , bladder issue are more of spinal nerve compressions from the degenerative disc problems .Hope this clears your query .Always welcome for any further assistance .Regards dear take care ."
},
{
"id": 197387,
"tgt": "Suggest ways to discontinue masturbation",
"src": "Patient: hi my age 22. year... i .masturbate from age 15th to now..i masturbate in excessviesly so much...3 times in a week....during this time i lose my confidence ..my sex power ... nd also addicat of masturbation..i cant live without it..m suffering form ds...i mean..there r no pleasure of sex..its only addication ...i mucles is weak..all body of me is decresing ...i . m very thin ..my penis is now also small....my diet is not healthy... please suggest me to avoid masturbation Doctor: Hi thanks for contacting ...Overmasturbation that is greater then three per week lead some problems .....esp.lack of concentration , groin discomfort , problem in study , social adaptation problem etc....It is pleasure giving practise but not done excessively....Yoga and regular exercise helpful ...Take healthy and balanced diet with more fruits....Green leafy vegetables more ....Fruits more ...Excess porn watching not done ..If need consult psychiatrist for behavioural therapy...Take care"
},
{
"id": 118020,
"tgt": "What does it mean by elevated calcium level in blood and urine?",
"src": "Patient: I am 46 years old, 5'1 and 149 lbs. I have hyperthroidism. I saw my md yesterday due to severe neck and backache. My md told me that I have myofascial pain. He ordered a series of blood and urine tests. His nurse called me and told me that my calcium level was elevated in blood and urine and to go to lab for more urine and blood tests. What does this mean? Doctor: Parathyroid is the gland situated in the neck along with thyroid. One of its function is to maintain calcium level in the body. Normal level is 9 to 11mg/dl. Hyperparthrodism is increase parathormon level in the blood which in tufn increase the cslcium level in the blood and urine. It will leads to pain snf your other feature. PTH level snd calcium level is advised."
},
{
"id": 58568,
"tgt": "History of hysterectomy. Have pancreatitis, low WBC, high liver enzymes. Have morphine pump. Nausea, pain. What to do?",
"src": "Patient: in 2005 I had a hystroactomy that is when I got this pain that was so painful . I was still in hospital and no one believed me. I got out of hospital. well I could write a book. no doctors believed me went to many doctors and two hospitals they all said it was in my head. to me all I knew was I was in so much pain I was dying and well I went to rio grand hospital and they believe I was in pain and nothing was working. well it does not show up in my blood but they did a mri and they said I had cancer my pancrase was very very large all I knew I was in pain and thank god they believed me. they said I had 6 or so months to live. well I have been to so my doctors I am still alive and in so much pain. about almost two yrs ago I had a morfin pain pump put in and I hate it cause very so many weeks or so I get the pain so high again and I get so sick nusia and throw up all I have go in cause I am dihidered need iv and doctor up the morphln . I don't have a live. it harts to talk. and I cant do much with this pain I cant think. I don't know what to do any more. sorry about the spelling if you can help me. I think doctor did something wrong when he did the hystroacomy cause he was so unkind when I said I had this pain he got mad u don't know how ugly he was to me a nurse said he throw away my medical records and made an other one so I don't know . at the time I just wanted my pain to go away.i never went back to him. I am in so much pain its been eight yrs what do I do???????? oh they say what I have is cronic pancreatitis , low white count cells. high liver emz, my feet and legs swell up bad. bad gland and eight things in my tyroids that I cant get bioci need gland out first, and other things I didn't have. now I have every doctor and no money and It harts to eat. Doctor: Hi, sorry to know about your hystory, any how there wont be any relation between the hysterectomy, and pancriatitis. And you have so many problems, probably they are diagnised in finding the cause of pain, but not actually felt of the problem. I to my patient with such pain prescribe omeprazole, domperidone, dycyclomine hydrochloride, for pain relief and epitite. Practice diversion procedures like yoga, meditation, reading books and hearing music. Thank you."
},
{
"id": 41370,
"tgt": "Suggest treatment for infertility",
"src": "Patient: anshu (26YEARS,F) YYYY@YYYY rohtak(haryana)India i am suffering from primary fertility.there is no issue from last five years. i am having bicournate uterus with one cervix. one side hytometra collection n other side proper functioning.i m looking 4 prper advice. Doctor: Hello dear,Thank you for your contact to health care magic.I read and understand your concern. I am Dr Arun Tank answering your concern.Yes, having a bicornuate uterus is the reason for infertility.You have to go for surgery.1. Do surgical reconstruction of bicornuate uterus to one.I suggest you should first reconstruct your uterus in the manner that it becomes capable of gaining fertility.Once you are done with your surgery then you can try for natural pregnancy.You will be succeeded surely if not than you can step for second part that is artificial insemination.Please do not worry much, because worrying can cause reduction in fertility.I will be happy to answer your further concern on bit.ly/DrArun.Thank you,Dr Arun TankInfectious diseases specialist,HCM."
},
{
"id": 41491,
"tgt": "How does an incipient varicocele on left testicle affect fertility?",
"src": "Patient: Hello Doctor, we have been trying to conceive for the last 3 years and still looking for the solution to our problem, my wife is consulting a specialist, and on my side I run some spermogram studies to see what the numbers where, so this is what I got so far, Volume: 4.5 ml; Count: 124.500.000 per ml; Motility: 15% (Fast Linear Progression), 25% (Slow Linear Progression), 30% (without linear motion), 30% (dead); Morphology: 70% (Ref: 30%).I am 37 my wife 36. Besides an ultrasound says that I have an incipient varicocele on my left testicle. What's your opinion of this, is it ok ? Doctor: HIWell come to HCMI really appreciate your concern, even if the amount of ejaculated semen is little less but the sperm count is okay, and this this is nothing to worry, there are enough chance of conception even with the history of varicocele, hope this information helps, have a nice day."
},
{
"id": 101995,
"tgt": "What could cause difficulty in breathing and speaking after having surgery?",
"src": "Patient: I have had 3 prior surgeries and with each procedure I experienced the same frightening problem in recovery.I awoke unable to breath. I could not speak and my breathing my extremely shallow. I would describe the sensation as one I would think an extreme asthmatic would experience. The air slowly returned each time. I am having surgery again next week and I am terrified. Can you suggest to me what the cause might be. Thank you Doctor: Hi,Since this is happening only after surgery, you may be having more influence by anesthesia or due to muscle relaxant succinylcholine. Do you have any other problem. If not check for pseudocholine esterase level in blood which is responsible for clearing the succinylcholine. If defect then discuss with your doctor for using other drugs for anesthesia. Hope this helps you. Regards"
},
{
"id": 201541,
"tgt": "What is the treatment for pressure and irritation in the scrotum?",
"src": "Patient: Hi. I am a 30 year old male. I am having a dragging sensation in my right scrotum for about 7-10 days. There is no pain, but when I palpate, the spermatic cord feels thickened compared to the left. I compared the size of both of my testes but they are same, no visible mass. I m worried. Doctor: Hi,There can be an infection in the scrotum which is causing the pain. This can mean an infection in the testis and epididymis. The surest sigh of such an infection is pain relief and feeling comfortable on lying down and lifting the scrotum with your hands. To confirm an infection requires consultation with a surgeon. If a single diagnosis cannot be found after clinical examination then an ultrasound scan of scrotum has to be done. This will show and features of infection, abnormal blood flow or collection of fluids as found in hydrocele. If it happened few days back then it is most likely and infection. Such infections can be treated by medications. A complete course of antibiotics and pain killers might help. Please avoid doing heavy physical exercises and going to the gym for a week."
},
{
"id": 117879,
"tgt": "How long will it take to get back the normal Hb level after hysterectomy?",
"src": "Patient: Hello I had a hysterectomy on Aug 7th the day after they found my hemoglobion was 7.8 then by thursday evening it was a 5.9 so the radioligist went in thru a artery in my leg and found a place that was oozing some blood and foamed it up and i had 2 bags of blood gave to me thru IV the next morning it was back up to a 8 and i came home which was Friday. I went to ER on Monday for a bad constipation pain but was relieved they checked my blood again and it was a 10 . My question is how long does it take to get it back to a normal level like 12 or so ? And when can I have a glass or 2 of wine Doctor: Hi,welcome to health care magic and thanks for asking.The hemoglobin level should come back to normal within a month, if you are taking supplements.Wine is not contraindicated and is not a negative factor for low hemoglobin. If there are no other risk factors, you can take a glass of wine. Alcoholics are known to suffer from malnutrition. So try to keep up your nutrition and if required, take supplements.Any further queries, happy to help again."
},
{
"id": 207714,
"tgt": "How safe is XR for ADD?",
"src": "Patient: I am wondering if there are any techniques to increasing or prolonging the effects of adderall. I am currently prescribed 20mg XR, once a day. I have been taking the drug for ADD a little over 6 months now and have noticed my ability to stay focused has greatly decreased. Doctor: Adderall is known to cause dependence and tolerance. Here are some tips that will help:A. Take Adderall only as prescribed and not for recreational purposes.B. Discuss with your doctor whether 1. you may increase the dose, since you now need a higher dose. However, remember that tolerance will develop to higher dose as well. 2. you can consider a drug holiday in which you slowly decrease Adderall and stop it. You then restart it later on. However, in the period that you are not taking Adderall, your ADD symptoms will return.C. You may discuss with your doctor switching to another medicine such as Atomoxetine which rarely causes dependence and tolerance.D. You may also see if there are any other medications you are taking that interact with Adderall.E. Don't ignore behavioral management for your ADD.I hope this answer is helpful."
},
{
"id": 31893,
"tgt": "What could itchy groin and penis suggest?",
"src": "Patient: for the past 6 months i have heavy itchiness around groin and penis first i got two dots like pimple size then when i use to scratch now it getting spread over all and become more. Nowadays in the time of taking bath i use to rub in that place then the skin will break and blook is coming.. more than 2 months i used itch guard but still it is not cured. please tell me which specialist do i need to contact. Doctor: Hi,From history it seems that you might be having fungal infection on the part.Apply anti-fungal cream locally.Go for one oral antifungal medicine course.You can consult Dermatologist for this condition.Ok and take care."
},
{
"id": 27297,
"tgt": "Should i be concerned about the high BP of 186 over 94?",
"src": "Patient: My blood pressure is 186 over 94 and I am 84 years old. Never had any heart problems and I take 1 Atacand a day. I don't feel any different now, no headache, no confusion. Is this dangerous? Should I go to see a doctor immediately or can I wait until tomorrow? Doctor: Hello dear user!I have gone through your query and understood your concerns!Thank you for sharing them on healthcaremagic.Blood pressure of 186/94 is considered high (third stage of hypertension), and it must be treated even though it doesn't give you concerns. Such level of blood pressure may lead to heart, kidneys, blood vessels, brain damage etc. The pill you are taking every day is an anti hypertensive drug, that helps normalizing blood pressure but it seems to be not enough. You have to add other drugs in your therapy, like diuretics or other class. Please consult with your doctor as soon as possible to give you the proper medication. Have an active lifestyle, with physical activity, diminish the amount of salt in your diet, be careful with fatty foods. You may have to go through an analysis to check your lipids and sugar in the blood because generally abnormal levels of them goes together with high blood pressure.I hope this answer was helpful to you!Please kindly rate it as helpful and write a short review about your experience with me! I would appreciate that a lot.Thank you and best regards!DR ERIOL."
},
{
"id": 37232,
"tgt": "Would it be required to have yellow fever vaccination ?",
"src": "Patient: I a 72 year old female, going to the Napo River Region of Caco Equador. I her ethave had conflicting information as to whether or not I should take the yellow fever vaccination. Because of my age I was told to us deet greater than 25% on exposed areas and Primetheon on my clothing. Is that sufficiant protection. Doctor: Hello,I understand your concern.I am Dr. Arun Tank, infectious diseases specialist, answering your query.In my opinion you should take the vaccination.Addition of protection and repellent will help you further in protection.Visiting a risky area without protection should not be done at your age.The area you have mentioned is having high prevalence of yellow fever.I will be happy to answer your further concern, you can ask me on bit.ly/DrArun. Thank you.Dr Arun TankInfectious diseases specialist."
},
{
"id": 201476,
"tgt": "What causes soft bump like on the tip of the penis?",
"src": "Patient: Hi, firstly i have never had any sexual contact and for as long as i can remember i have had this soft bump on the tip of my penis/foreskin area and it doesnt cause any discomfort at all. i dont think it is genital wart because there is no way i could of contracted it. Thanks. Doctor: Hi,thank you for using health care magic.I can understand your concern.Soft bump over penis / foreskin area should get examination by doctor even though it is not producing any discomfort. It can be a collection of fatty tissue most probably but It is possible that the bump can possibly affect your sexual life in future if it is large enough.If you also think so then get help of surgeon. Surgeon would be in best position after examination of the bump , to give you advise about what should be done.Hope this will help you.Regards."
},
{
"id": 31177,
"tgt": "Suggest treatment for persistent fever and cough",
"src": "Patient: i was having fever with slight cough (103oF) from 7/4/10, took medicine (nimuselide+paracetamol+ antibiotics and cough syrup (corico) but did not get relief. again revisit the Dr he changed antibiotics and advised to continue the syrup and nimuselide+paracetamo. no relief by these drugs also. after taking antipyretics it subsidized for 10-12 hrs only kindly suggest Doctor: Hi,From history it seems that you require some investigations to come proper diagnosis of having fever.I suggest,X-ray chest,Complete blood checking with TC, DC, ESR, Platelet count,MP etc.Routine urine check up.S. Widal test for typhoid,If require blood culture.Consult your treating doctor and discuss about this.Ok and take care."
},
{
"id": 186849,
"tgt": "What is the treatment for pain in the gum?",
"src": "Patient: FOR WEEKS NOW MY JAW AND MOLARS ARE SORE ,, HAD A PANORAX X-RAY AND THEY SEE WHITE SPOT ON LOWER GUM WHERE THERE WAS A WISDOM TOOTH THAT ONE DENTIST TRIED TO ROOT CANAL UNTIL IT GOT SO BAD IT GOT PULLED OUT,, HOWEVER THE PAIN WHERE THERE IS NO TOOTH BUT A WHITE SPOT IS STILL PAINING ME???????? Doctor: Hello, Welcome Thanks for consulting HCM, I have gone through your query, as you have pain in gums after extraction dont worry pain remains after extraction atleast 15 days . As you have white spot on extraction socket area it can be due to sinus opening is there or it can be bacterial growth , or white lesion present. For this you should consult dentist and go for Oral examination if there is bacterial infection then go for betadine irrigation , or if it is white lesion then go for treatment . Hope this will help you."
},
{
"id": 151203,
"tgt": "Loss of consciousness, memory loss. MRI shows white focal calcification. Serious condition?",
"src": "Patient: My father lost conciousness 2o days ago suddenly and for last 2-3 months he used to forget things. He is 56 yrs nw and he had epilepsy in early ages. In MRI, it was found - Left medial frontal lobar calcific focus with minimal perifocal edema/gliosis- may be calcific granuloma and left parietal lobar white focal microbleed/ calcification . what does all these mean? is there anything serious? Doctor: Hi, First of all, let me reassure you that they MRI findings do not represent any serious neurological problem. These findings indicate that he had suffered from neurocysticercosis, a parasitic infection of brain, the most common cause of secondary epilepsy in India, in the past. This is the cause of epilepsy in him. The current MRI indicates that the infection has healed, as it has become calcified. He may require to continue anti-epileptic medications. Please show him to a neurologist for further advice. Best wishes, Dr Sudhir Kumar MD DM Consultant Neurologist Apollo Hospitals, Hyderabad"
},
{
"id": 135361,
"tgt": "Suggest treatment for osteoarthritis",
"src": "Patient: hello,i am 45 years old into active sports, playing badminton twice a week for the last 10 years. had pain in my knees and diagnosed as OA. Dr prescribed orcerin GM as well as cartigen which I have been taking regularly last 3 months. Still feeling my knees are weak and cannot climb stairs. No pain around the joint but can feel some hard swelling behind the knee supposed to be due to tight hamstrings/ sciatica. please advise. does orcerin gm cause weight gain? Doctor: Hi Dear,Welcome to HCM.Understanding your concern. As per your query you have treatment for osteoarthritis. Well there can be many reasons for symptoms you mention in query like nerve compression , osteoarthritis , injury or baker's cyst . I would suggest you to consult orthopedic surgeon again Doctor may order certain test like CT scan ,nerve conduction test or physical examination . Doctor may recommend physical therapy in addition to medicine already prescribed . Doctor may also recommend cortisone injection or surgery in case of severe damage . For now give your knee proper rest , stop playing for some days , do gentle massage with sesame oil and take ibuprofen or acetaminophen for pain . Hope your concern has been resolved.Get Well Soon.Best Wishes,Dr. Harry Maheshwari"
},
{
"id": 103953,
"tgt": "Chest pain, hot flushes after taking caffeine. Am I allergic to caffeine?",
"src": "Patient: Hi,I took 1 100mg caffeine pill today, and about 5 minutes later I was experiencing slight chest pain, not constant but it was there for a good half hour, then I experienced hot flushes an hour after the chest pain. I've drank coffee before, so it's not a allergic reaction to caffeine, I was wondering why I experienced chest pain.Thank you. Doctor: you CANT DETERMINE WHEATHER YOU ARE ALLERGIC TO CAFFENE TAKING COFFE IS DIFFERENT THAN PILL AS PILL CONTAIN BASE COLOR AND OTHER SALTS WHICH CAN BE ALLERGIC FOR YOU YOU START WITH ANTIALALERGIC LIKE FEXOFENADINE 120 MG BD SYP GELUSIL 2 TSF TDS TAB ATARAX AT NIGHT ANDD LOCAL APPLICATION OF HISTOCALAMINE DO FOR 10 DAYS MUST GET ECG DONE TO RULE OUT HEART INVOLVEMENT"
},
{
"id": 199943,
"tgt": "How to cure balanitis on the head of the penis?",
"src": "Patient: Dear Sir, Actually I had an unprotected ANAL sex for the first time in my life one week ago with a gay and just for a very few mins. I was in receptive position. Though I remember quite well there was no any exchange of fluids; semen. May be precum (but not sure). I have a balanitis on my head of penis now. I checked with the partner he confidently says he doesn t have any HIV but he doesn t wants to trouble to go to HIV tests and it is not possible even for me to force him. The worst thing is I am dying every minutes thinking that I may be infected. I realize my mistake now. Though I have done tests today and waiting for results. I heard even a test after 30 days is not fully sure. Any words from your side, i appreciate? Doctor: HelloThanks for query.You had unprotected anal sex with a male partner and whose HIV status is not known to you so worried about acquiring HIV.You have already done basic screening test for HIV today and result s awaited.Result of single test is not conformative as incubation period to acquire HIV is 2-3 months ..Repeat the blood test after 3 months to be rest assured about whether you have acquired HIV or not.Dr.Patil."
},
{
"id": 768,
"tgt": "Does hormone imbalance and irregular periods cause difficulty in conceiving?",
"src": "Patient: Hi Dr.Chetna Jain, iam sudheer i have a query for my wife.she has Harmone Imbalance problem and her periods also doesn't come regularly..if the period comes it comes only for a couple of days some times for 3 days..she is telling that it takes time for her getting pregency late is it so? and also when we do sex she immediately clean my sperum..if she cleans my sperum she will not get pregency right?she has to put my sperum for a while i think so.can u suggest me in this regard.. Doctor: Hi,I understand your concerns.Following is my reply:1)\u00a0\u00a0\u00a0\u00a0\u00a0Some sperms do remain inside even though she cleans.2) There is possibility of pregnancyYou can contact me anytime directly to ask question by pasting following link in your browser:http://bit.ly/askdrsoumya"
},
{
"id": 157823,
"tgt": "Stage 2 lung cancer, had stroke, smokes and drinks. Chances of having a re-occurrence of Cancer?",
"src": "Patient: Hi, my husband was told he had Stage 2 lung Cancer, about year ago, but after they saw that it wasn't Lung Cancer, but had to remove the Mass, in his left lung, and shortly, after four months exactly he had stroke on his left side, after all this, he continue to smoke and drink, Question, what are his chances of having a reoccurrence of a stroke or lung Cancer? Doctor: HIThank for asking to HCMOnce your husband knows about his health problem then must be away from such habits, but even after knowing these he continues on it then the consequences would too bad, no need to ask for that, the chances of bad diseases is very very likely, tell him to stay away from all these things, take care of him, give him a moral support, have good luck."
},
{
"id": 1913,
"tgt": "Should i go for hysteroscopy if unable to conceive?",
"src": "Patient: Hello Dr,I am female of age 27 and i am suffering from pcos problem taken metaforin for 1 month,clomide and hcg 500 injections for 3months but still cant able to concive its been almost 1 year since am trying to concive.My gynec suggested me to go for laproscopic any hystroscpy surgry shall i go for it what should i do now ?can i ever concive..Is ther any hope with this surgery that i can concive Doctor: Dear Ms , you have 2 options either to go for a laparoscope and hysteroscope or you go directly for an IVF treatment"
},
{
"id": 149223,
"tgt": "Coughed and split blood, I am ALS patient, on medicine, oxygen, warfarin 5.5mg, INR done. Recommendations?",
"src": "Patient: This morning I coughed up a small amount of blood,i'm an A L S patient , on a number of medicines,also on oxygen 24 hours a day at 4.5 liters. i'm on warafin 5.5 mg a day,checked my INR today which is 1.9. informed my family doctor and waiting for a return call. yesterday morning and the afternoon I coughed and spit up blood. I'm not able to walk.Could I possible have a blood clot forming the warafin is breaking it up? Doctor: Hi,Thank you for posting your query.Yes, the most likely cause of coughing small amounts of blood in your cases, could be warfarin-related. Warfarin makes the blood thin, which results in bleeding at times. The reason for your warfarin use in not clear from your description. The dose of warfarin depends on the indication for using warfarin.I feel the dose may have to be reduced for a few days, after which you should recheck the PT, INR; and then adjust the dose of warfarin appropriately.I hope it helps.Please get back if you require any additional information.Best wishes,Dr Sudhir Kumar MD (Internal Medicine), DM (Neurology)Senior Consultant NeurologistApollo Hospitals, Hyderabad,My personal URL on this website: http://bit.ly/Dr-Sudhir-kumar My blog: http://bestneurodoctor.blogspot.com/"
},
{
"id": 189131,
"tgt": "Had abscess tooth. Another broken tooth painful. Ears clogged. Does the abscess spreads?",
"src": "Patient: Hello,I have had an abcess tooth before that was pulled. I have another broken tooth that has been giving me trouble for quite some time, but i dont have insurance, so i have not been to the dentist. There is no swelling around the tooth, but it is painful a lot of the time. Lately my head has felt pressure. My ears are clogged and the left side of my nose is runny all the time, which is the side that the tooth is on. could this be a spreading abcess? Doctor: Hi and thanks for the query,Its very appropriate for you get consulted by a dentist. It very possible for a poorly treated tooth infection to spread, especially in case of an abscess. Nearby structures like the sinuses and the bones around can easily be attained. Osteomyelitis of nearby bones is a common complication of poorly treated tooth infections. Consultatinga dentist at this point in time is of utmost importance for an appropriate evaluation and treatment. Adequate drainage and prescription of antibiotics might be needed.thanks for the query and hope this helps,Best regards,Luchuo, MD."
},
{
"id": 74093,
"tgt": "What is the treatment for sharp pain in the right side of the chest?",
"src": "Patient: Yesterday, I felt a pin prick pain that radiated from the right chest vertically down to about the same height as my navel. Today, I felt pin prick pain in the right side of my abdomen same as as my navel. I would feel this pain about 15 minutes or so apart in the last hour. Any thoughts on what this might be and what my next steps should be? Doctor: Hello The next step is to do a chest X Ray and an abdominal ultrasound too.Take care Regards Dr.Jolanda"
},
{
"id": 148426,
"tgt": "Shoulder pain, cracking and popping in shoulder. Have impingement syndrome",
"src": "Patient: Ive previously been diagnosed with impingement syndrome , bursitis n cervical radiculopathy . However diagnosis were all from different drs. I have a lot of shoulder pain that goes into my arm n hand. Cracking n popping in front of shoulder. But I ve told drs that I experience most severe pain between spine n shoulder n usually when its bothering me. sometimes I breathe in n it sounds like my vertebrae crack several times- almost like in upward sequece. They don t seem sure why this happens. Also I ve noticed lately that I have spot on spine that hurts n it is close 2 affected area- little concerned. Over the weekend I was having shoulder pain n medication was not helping. My dr So Doctor: Dear it seems you have both problems, bursitis and cervical radiculopathy.It is time you do radiologic examination to determinate the diagnose.My advice is cervikal and scapulo-humeral MR.Starting wearing a cervical collar and use medications for pain reliaf.Wish you all the best"
},
{
"id": 95204,
"tgt": "What causes thick white discharge with abdominal pain ? How is it prevented ?",
"src": "Patient: I have been having a white lumpy discharge in Abdominal pain what can that mean Doctor: Welcome to Healthcare Magic You could be having pelvic inflammatory disease. What is your age. Are you sexually active. You need to get examined by your doctor to confirm and be on required antibiotics."
},
{
"id": 226289,
"tgt": "Taking 200 micrograms of thyroxine and contraception pill. Got positive HPT. Possible?",
"src": "Patient: Hi, I am currently taking 200 micrograms of thyroxine and a contraception pill daily. Don t know what the contraceptive pill is called but I must take it at the same time every day. I should be taking 175 micrograms of thyroxine but I ran out of the 25 micrograms and the 50 micrograms about 2 weeks ago and due to working full time I just didn t get round to asking for more tablets until yesterday so i now have some more on order which I will get on Monday. But the issue is that I am approx 10 days over due my period and I have done a pregnancy test and it says I am pregnant 2-3 weeks. Could I be pregnant? Doctor: Hello, When taken correctly, birth control pills can be up to 99% effective in preventing pregnancy. However, women who are taking birth control pills do on rare occasions get pregnant. When this happens, it is most often because of something the women did (or, more likely, did not do), not because the pill itself somehow failed to work. One may miss some dose and forget it or you might vomit it out or if you are taking some other medicine like anti seizure medicine or some herbal medicine or some antibiotic,rendering it less effective. You can definitely be pregnant however you may confirm it again by doing pregnancy test by both the ways,urine as well as blood estimation of beta HCG. The accidental increased dose of ELTROXIN has no relation with your pregnancy. Thanks"
},
{
"id": 54550,
"tgt": "Suggest treatment for pain due to gall stones",
"src": "Patient: hello dr. I'm mother of 7 yrs old & 4 month old (both boys).i have this heavy feeling of pain in my full stomach as well as in lower abdomen from last 3 months. my menstural cycle hasn't started yet. my doc has done tests as well as ultrasound which diagnose the gallblader stones.But dr said this is not gall stones pain.I'm really fed up of this 24 hour pain.unable to understand what to do.I'm not getting any relief for this pain dont know what hes looking for and i want to know what to expect for possible diagnosis .whatever i eat i feel pain.i have already discarded most of the thing from my meals.i have gastric problem also(farting). Doctor: HIYou need to be screened for an H pylori stomach infection which can be eradicated with antibioticsYour symptoms are very typical for this"
},
{
"id": 60621,
"tgt": "Fatty liver with normal ESR",
"src": "Patient: my husband is 29 years with liver function test result (ALT) 78 and (AST) 43 cholesterol 211 but normal blood picture and normal esr what is wrong with him. our doctor suggested fatty liver and excluded hepatitis and fibrosis is it true? thxx alot Doctor: Hi; welcome to HealthcareMagic If the ALT/AST is increased then it should be seen if the bilirubin is increased .If there is increase in bilirubin then it is hepatitis or else it is a fatty liver with a high cholesterol.Your husband needs to take a healthy diet and keep away from fatty ;fried food .Please consult your doctor for a detailed diet plan and exercise to reduce the cholesterol. Thanks"
},
{
"id": 42468,
"tgt": "Are pains in ovaries and uterus early signs of conception?",
"src": "Patient: hi i am pooja my age is 27 year's i had my follicular study starting on the 10th day after the periods. today is 14th day i have 20.8mm egg on left ovary and ET is 11.2mm and my doctor gave me an hucog 10,000 injection. next day when my egg ruptured my doctor did my iui on 2nd september. Now i m having little pains in my right and left ovary and in uterus also. any chances od conceiving plz tell. Doctor: Hi,Welcome to HealthcareMagic .Pain in ovary or uterine region is not due to conception. These are mainly due to side effects of progesterone which will be given after IUI. Chances of conception are good as iui is done. Problem of ascend and reaction towards sperm is solved. But it is also true pregnancy happens by luck and chance. There is no 100% guarantee for any treatment. Hope I have been helpful .RegardsDr.Deepika Patil"
},
{
"id": 92836,
"tgt": "Have mild abdominal pain, white discharge and vaginal swelling. Prescribed Diflucan. No change. Had intercourse. Cure?",
"src": "Patient: I have been having mild abdominal pains, also I've noticed that I have a white fishy discharge and vaginal swelling. I went to the doctor and she checked me for chlamydia and also did a blood test. My results were all normal. She thought I may have a yeast infection so she prescribed me diflucan which did nor get rid of any of my symptoms. I had unprotected sex with my boyfriend for the first time and the symptoms started about 4 day's afterwards. First it was like diarrhea and hot flashes and then it turned into abdominal pains and vaginal swelling and discharge. I don't know what it can be its scaring me. Doctor: hi you should do gonococcal microexamination of discharge. it seems to be sexually transmitted.you can use Ciprofloxacin 500mg twice daily for 7 days.Regards"
},
{
"id": 217692,
"tgt": "What causes chest pain after hurt while fighting?",
"src": "Patient: Hi I was play fighting with a firend and the hit me pretty hard in the chest. Now I am experiencing chest painwhen I turn or move, I can't lift anything and its a little hard to breathe. The pain is in my chest and shoots into my upper back. Can you tell me what mightbe wrong Doctor: This type of pain usually occur due to fracture rib or a bruise on the rib and usually takes a while before going away even if you take regular analgesics. It may take even upto 6 months. If too much of trouble get a nerve block from a pain specialist after you get an X Ray of the chest."
},
{
"id": 49070,
"tgt": "Several stints to abdominal organs. Mysenteric artery & kidneys. Concerned about adrenal gland tumor. Help?",
"src": "Patient: My mother is 88 and has several stints to abdominal organs-- mysenteric artery & kidneys, femoral bypass along with other problems such as occlusion of the celiac artery . Two or three heart valves leak- aortic & m and She takes 5mcg synthroid . There is a benign tumor on one of the adrenal glands the size of a small orange. Recently she had a pace maker installed because she was passing out at regular intervals. But now her blood pressure has dropped dramatically and her pulse has increased dramatically. Blood pressure averages about 110/75 and pulse has jumped from a good 69 to 85-110. The mysenteric stint was recently ballooned successfully and her blood pressure seemed very erratic afterward and the progressed to these lows. Normally for her blood pressure was high at an average of 160 /65 for years (with regular erratic swings both up and down but not staying either way for very long). I m concerned that the adrenal gland tumor may be the cause of the recent drop in pressure and increase in heart rate. This is the 4th day that her heart rate has been elevated constantly. Doctor: She should have her urine and blood tested for adrenal hormones (cortisol, metanephrines, acth) that way we can check to see if the tumor is causing these swings. Please rate 5 stars! I strive to provide the best answer I can to your quetsions!"
},
{
"id": 65724,
"tgt": "Suggest remedy for itchy lump by the corner of lower lip",
"src": "Patient: Hi, I have this small bump by the corner of my lower lip. At first I thought maybe it was a cold sore even though I have never had one before. But it itches and doesn t hurt. What could it be and what could could I for it? I m in college and don t want a weird bump by my lip. Doctor: Hi, dearI have gone through your question. I can understand your concern. You may have infection or some benign cyst like lesion on ypur lip. You should go for biopsy of that lump. It will give you exact diagnosis. Then you should take treatment accordingly. Hope I have answered your question, if you have doubt then I will be happy to answer. Thanks for using health care magic. Wish you a very good health."
},
{
"id": 153183,
"tgt": "Is colonoscopy advisable without anesthesia?",
"src": "Patient: my father was to get a colonoscopy done yesterday for which he followed all the pre-requisistes.. however during the procedure due to acute pain he didn't allow the doctor to continue... now he is adamant on getting it done with anasthesia... is this recommended ? Doctor: hi.if your father cannot tolerate it the first time, it is best if he undergo colonoscopy on anesthesia. if he is old, a cardio-pulmonary clearance will be done prior to the procedure.hope this helps.good day!!~dr.kaye"
},
{
"id": 141090,
"tgt": "What causes numbness and tingling sensation in the legs?",
"src": "Patient: What could be the cause of a numbness and tingling as though the area had fallen asleep to last 12hrs now? Started out only in my left shin under the knee cap all down to my ankel. At onset I was grocery shopping, felt an urge of nausea and light headed with an instant severe headache like between my eyes/forehead (different than how my migraines usually begin) I also noticed red hue splotch in my vision which kinda went from transparent to solid over the 3 or 4 different times i seen it. Doctor: Hi, You may be having what is sometimes referred to as complicated migraine or \"stroke equivalent\". This translates usually into a TIA or other forms of temporary of blood flow to the brain due to either a clot that quickly breaks up or some change in hemodynamics such that pressure is lost to an area of the brain subserving the sensory function of the left leg. Hope I have answered your query. Let me know if I can assist you further."
},
{
"id": 189119,
"tgt": "Suffering from fever, blisters, canker sore, sensitive gums and not able to eat the food. Guide",
"src": "Patient: I am 26, and have had the following in order:Night 1 : slight feverNight 2: heavy fever, fever blisters Night 3: heavy fever, canker sore, whole mouth becomes extremely sensitive around gum lineI can barely eat, it is so painful to eat anything that isn't as soft as a fried egg. I have dipped snuff for 5 years but I don't have any \"lumps or lesions\" in my mouth. I am hungry but loose all appetite after a few painful bites Doctor: Hi user , welcome to health care magic. With your histroy it more looks like herptifrom of ulcer.in that case yor sore or ulcer will come down in 7 to 14days . Visit your near by dentist for further treatment.you can take some painkiller medcine likeacetaminophen and ibuprofen. And local application gel like dentogelor mucopain gel before food and after. To start with antibiotic .,need a clear examination and your health status.hope this answer of mine will helpful .warm regards Dr.leeladevi"
},
{
"id": 142160,
"tgt": "What causes extreme exhaustion, photosensitivity and headaches?",
"src": "Patient: I have been having issues for years with exhaustion, can sleep for 12 hrs and feel like i never slept at all, photosensitivity, headaches and migraines twice a year tat end up with me in the hospital because i cant speak open my eyes and tears run down my cheeks for extended periods of time, jaw pain, extreme joint pain, i get heart palpitations, anemia, i am always freezing( it could be 75 degrees and i am just ok not sweating like everyone else), when i am in the sun i get a very itchy rash everywhere the sun touched me and it doesn t go away for days, no matter what i use or take my acne never really goes away, i have trouble focusing, and i get depressed for no reason at all. do you have any ideas as to what this could possibly be? I have been to doctors around where i live but i live in a very rural area and they are baffled. Doctor: Too much sleep is a known risk factor for severe headaches. Your symptoms are very similar to what a person with migraine headaches may experience. You should be seen by a headache specialist and have a full metabolic workup implemented to check issues such as your thyroid gland function, adrenal function, and other metabolic issues that could cause imbalance and eventual migraines. Please write me at: www.bit.ly/drdariushsaghafi if you would like to further discuss your concerns. You may be suffering from additional problems involving DYSAUTONOMIA and migraine headaches may be just one manifestation of your problem. Please rate this as a 5 STAR encounter."
},
{
"id": 10355,
"tgt": "Need treatment for hair loss",
"src": "Patient: I had folliculitis then my hair started to fall out in the front of my head, then I got sores white ringworm. The dermatologist just gave me cream, I still have sores that ooze. I went to another Doctor She gave me nystatin cream and gris ofulvin. I still have the sores and alot of oozing, I lost my hair in the front , I have to wear a wig when I go out. Please help me. I had this since Jan 2014. I can t handle this anymore. It itches alot too Doctor: Hello and Welcome to \u2018Ask A Doctor\u2019 service. I have reviewed your query and here is my advice. As per your case history of folliculitis and hairfall, my treatment advice is - 1. Avoid hair oil application for 1 month. 2. Apply an antifungal lotion like Clotrimazole lotion twice daily on affected part. 3. Take an iron supplement and vitamin B12 supplement once daily for 3 months. Hope I have answered your query. Let me know if I can assist you further."
},
{
"id": 76156,
"tgt": "What is the treatment for chest pain and weakness?",
"src": "Patient: i am male 24 years from india and facing problem of stomach for two years.some time pain in my muscles some time pain in chest bone and some time heart beat high some time difficult in getting breath.much weakness and mentally weak and nervous and fear all the time.many times visited doctors some gave me emeprazole treatment and some mantally treatment.when stand i feel pressure on my chest.plz tell me what to do.all the liberay tests are ok Doctor: Thanks for your question on Healthcare Magic. I can understand your concern. No need to worry for major heart or lung related diseases because your all reports are normal. By your history and description, possibility of stress and anxiety related GERD (gastroesophageal reflux disease) is more. So we should first treat your stress and anxiety. So consult psychiatrist and get done counselling sessions. Try to identify stressor in your life and start working on it's solution. You may need anxiolytic drugs too. Avoid stress and tension, be relax and calm. For chest pain, do following things. 1. Avoid hot and spicy food. Avoid junk food. 2. Avoid large meals, instead take frequent small meals. 3. Take pantoprazole tablet on empty stomach. Don't worry, you will be alright with all these. Hope I have solved your query. I will be happy to help you further. Wish you good health. Thanks."
},
{
"id": 24350,
"tgt": "Suggest remedy to control high bp during 9th month of pregnancy",
"src": "Patient: hi doci m in my ninth month of pregnancy all during 8 mths of my pregnancy i had normal BP but all of sudden in my ninth mth my BP has shoot up to 140/100 as we dont have family history of high BP nor i m over weight nor do i have any swollen on my legs or hands anywhere on my body still .....kindly help me and pls suggest me some remedies to control high B. i m on alphadopa medicine Doctor: Hello ...usually in pregnancy alpha methyl dopa ... labetolol and occasionally amlodepin is prescribed...it need to be monitored very carefully...so please visit Ur physician or cardiologist for medication as need to monitor closely take care"
},
{
"id": 159989,
"tgt": "My mother in law with colon cancer. Could this be caused by liver failure ?",
"src": "Patient: My mother in law has colon cance. It has spread to the kidneys and liver. She went to the cleveland clinic and they are trying to figure out chemos for her. My question is, her lower poart of her legs, feet and ankles are swelling. Could this be caused by liver failure ? Doctor: hello, sweiing is called oedema. It is due to fluid accumulation. Can be due to kidney, liver or heart disease. I would advise you to get a renal function test and a ultrasound of the abdomen. It wil help to know if the mets have migrated elsewhere. Also get a liverfunction test to know the status of her liver. I hope i have answered your query. Take care"
},
{
"id": 54713,
"tgt": "What causes increase in SGPT and SGOT levels?",
"src": "Patient: Hello sir, my dad is suffering from T.B diseases from last 3 months even he is also kidney patient , he goes for dialysis twice in week, but from last month his S.G.P.T and S.G.O.T are increasing day by day . S.G.P.T is reached upto 276 level and S.G..O.T is reached upto218 level and one week I have stopped T.B tablets according to doctor concern ,may be cause, due to T.B tablets but after stopping tablet , even S.G.P.T is level are increasing not stopping to control it has reached upto 586 and S.G.O.T level upto 372 , sir can u please suggest me what will I do to reduce the S.G.P.T and S.G.O.T level Doctor: Hi thanks for contacting HCM....Noted you have elevated liver enzymes..You are taking treatment fie TB.So drugs esp.isoniazide can elevate liver enzyme by drug induce liver affection...So just now rest liver by taking low fat diet.Junk food and non veg avoided.In your lunch and dinner take salad made of green leafy veg like carrot , spinach and tomato...Fruits taken more....Avoid alcohol...Take one tsp cumin seed with butter milk early in morning...If still enzymes high and if suspected tuberculosis of liver also ex.military TB then biopsy needs to be done....Hope your concern solved...Dr.Parth"
},
{
"id": 147034,
"tgt": "What could cause severe dizziness ?",
"src": "Patient: This afternoon i was over come with severe dizziness and lack of balance. i do not feel ill, really. no headache, no ringing in the ears, no blurry vision, a slight nauseated feeling and i just can t seem to get my balance. everything is learching to the right...my head feels like i have a heavy weight inside. Doctor: Hi,Thank you for posting your query.I have noted your symptoms. The underlying cause of your dizziness could be either peripheral or central.Peripheral causes are mostly related to ear disease and in this case, symptoms of dizziness get aggravated with change of posture of head or neck.In central causes, there could be a lack of blood flow to brain, leading to a stroke, which can cause similar symptoms.A clinical examination would help in differentiating the two.I hope my answer helps. Please get back if you have any follow up queries or if you require any additional information.Wishing you good health,Dr Sudhir Kumar MD (Internal Medicine), DM (Neurology)Senior Consultant NeurologistApollo Hospitals, Hyderabad, IndiaClick on this link to ask me a DIRECT QUERY: http://bit.ly/Dr-Sudhir-kumarMy BLOG: http://bestneurodoctor.blogspot.in"
},
{
"id": 145120,
"tgt": "Suggest treatment for vestibular neuritis",
"src": "Patient: Hello: My husband has been suffering with vertigo for 8 months. He was diagnosed with vestibular neuritis. He has been to vestibular rehab, which helped his balance but he is still dizzy. They said there is nothing left he can do. So, my question, could this be something else? If so, what other diagnoses would you explore? He is excessively tired during the day everday, which is very much not like him. thank you! Doctor: in allopathy medicine roral steroids and anti emetics.but in Homoeopathy science have some good result for seen in vestibular neuritis."
},
{
"id": 15638,
"tgt": "Have rash all over legs. Using triamcinolone and RetinaA gel. Do I need laser treatment?",
"src": "Patient: I have a rash on both lower legs. It began 4 years ago with a solitary red mark in the middle of my lower leg. I thought it was the result of \"dry shaving\" my legs. I have been to two dermatologists, after having 3 biopsys all showing eczema (disputed x2) I have been being treated continuously over this 4-year period. Presently I am using triamcinolone and RetinA gel. My legs today are a mess. Today, it has been very warm, I have a fine red rash over both legs as well as the other marks and red areas from previously failed treatments. I also have two holes where the biopsies were done. Both doctors are board certified dermatologists. I need new thoughts on this. Would I be a candidate for laser treatment. I am now very self conscious of the appearance of my legs. they are a real mess. Doctor: Hi, thiscould be a secondary rash to some thing else in the body(positive V.D.R.L.)or anemia. or defeciency of the vitamins or minerals in the body. I advise you to consult a physician for diagnosis and treatment. Take more of green leafy vegetables, pulses, sprouts, and proteine rich foods. Thank you."
},
{
"id": 51794,
"tgt": "What could be the jelly like thing in my bowel movement ?",
"src": "Patient: Hello, Today morning When I woke up I was fine. After 15 min or so I felt very dizzy. I thought I may be hungry and then I ate something. But even then I had little bit of dizziness. After that when I went to for my bowel , my stools had some solid jelly like thing of the size of a Almond, light red in color and is floating.. What could be the jelly like thing in my bowel movement. After few min after I ate something I felt normal. But still I wonder what is e red jelly like thing in my stool? Doctor: hello, what you are talking about might be a fresh blood clot. i would advise you to consult a physician and get a complte health check done. i would advise you a colonoscopy done to rule out large bowel bleeding. i hope i have answered your query. take care"
},
{
"id": 210376,
"tgt": "Suggest remedy for constant depression, lack of energy and intrest in life?",
"src": "Patient: Hi my name is kenny guzman an ive bin battling with deppression for over 5years now im 28 years old with one child an I just feel like I have a lack of enerygy a lack of intrist an its to the pount ere its affecting my life an house hold as well as me working last time I went for a check up was about 4years ago on a deppression scale I was a 6 its 2014 an I feel like I have alot of pressur on me an I just wanto to know if u could try to apply for disability benefits... Doctor: DearWe understand your concernsI went through your details. I suggest you not to worry much. Depression will not go away if you sit idle and always think about it and always scold your fate. You have to decide, get up and GO. Work i the only thing which can take you out of this juncture. For which you have to be a bit persuasive. Self persuasive. Make sure you work, whatever may your physical condition. What ever may your mental condition. Ignore tiredness, laziness. Let me tell you, that is the only way out. Do you want to be out of your current situation? Then follow me strictly.Please post a direct question to me in this website. Make sure that you include every minute details possible. I shall prescribe some psychotherapy techniques which should help you cure your condition.Hope this answers your query. Available for further clarifications.Good luck."
},
{
"id": 104521,
"tgt": "Asthma, blood infection, black rashes on skin. On telukast, symbion. What may be wrong?",
"src": "Patient: I am from bangladesh ctg. My son has asthma when his age 6 month know he is 3 years 6 month. Every 1/2 month he admitted in hospital for his asthma he has some blood infection also. one kind of black rash seen his skin. he take daily TELUKAST 5,SYMBION 4.5/160 INHALER ,ARTICA SYRUP,when he admitted in hospital HIS NEBULISER MEDICINE IS WINDEL PLUS 2.5ML, trixon 1gm, dexo 5mg three time in a day. when doctor test his blood test sometime report was good and sometime report was bad example his WBC was high, his IGE was 120,and his ESR is always 20. he has some blood infection but our doctor don;t find what type of infection it is. I want test his allergy test and blood test also. . . pls suggest me which doctor will better for my child. i can;t see his suffering Doctor: Hi and thanks for the query, Your child has a predisposition to develop allergies. The results of thr IGE levels extremely high and association of asthma and rashes are very characteristic. The raised WBC sometime simply means then there was an inflammatory process that was going on. On ESR of 20 is really no worry. Management of the child shall require a reevaluation of the pulmonary and allergic aspects of his health condition. I suggest you an allergologist/allergsit and also probably a pulmonologist later for the management and control of the asthma. thanks and best regards, Luchuo, MD."
},
{
"id": 202655,
"tgt": "Semen analysis test done for physical, volume, high viscosity microscopic motile non motile. Looking for advice",
"src": "Patient: Hello, I d been semen analysis test, and This s my result: Physical: Volume: 3ml pH: alkaline Total sperm account: 18 M/ml High Viscosity Microscopic Motile Non motile after 30 mint 40 % 60 % after 60 mint 35 % 65 % after 90 % 30 % 70 % Also Semen Culture: Their s no any growth after 2 days incubation So please, give me your advice, and what s that means? Doctor: HelloThanks for your query,based on the facts that you have posted it appears that you have what is called as Oligoasthanozoospermia meaning there by that your sperm count and motility of the sperm is low (almost 0) as per WHO standards.Truly speaking there are no medicines that can increase sperm motility however following measure will help you to improve sperm count and motility of sperms.1) Practice regular exercise for 45 minutes followed by meditation for 1/2 an hour in the morning.2) Take high protein diet rich in vegetables and fruits and Vitamin A,C,D,E.and Zinc3)Take anti oxidants like Almonds 5-6 everyday..4) Avoid alcohol and smoking..Dr.Patil."
},
{
"id": 217800,
"tgt": "Suggest treatment for bruise and swelling on wrist",
"src": "Patient: Hi, about 2 weeks ago i fell and hurt my wrist. It was bruised and a wee bit swollen but not sore to move. The bone however on the inside of my wrist is very tender to touch. If I straighten and roll my hand and arm inward - thumb toward the ground it is painful but general use is not really inhibited. I am starting to feel a bit on pain down into my thumb. Am thinking whatever it is will heal on its own. Can you bruise a bone? Doctor: Hi, as per details provided by you, you feel pain and difficulty in moving the limb. So there could be just bruise, muscular tenderness or some hairline farcture too. So kindly consult orhtopedician to rule out the fracture. If there is no fracture you can take some pain killers with serratinopeptidase, like aceclofenac sp. But first you need ortho consultation.thank you."
},
{
"id": 62775,
"tgt": "What causes bruise on forearm with bump in middle while on Cymbalta?",
"src": "Patient: Hi, may I answer your health queries right now ? Please type your query here... I have a gold ball size bruise on my fore arm and i dont remeber hitting myself at all and it has a white bump in the middle. I am on cymbalta is this something to worry about I have been bruising easily Doctor: Hi,Welcome to HCM.Based on the facts of your query,the bruise on the forearm with white bump in the middle,seems to be due to Boil with cellulitis with pointing white head/ or it could be a insect bite with bruise/cellulitis with poining head.As you don't remember any bite or trauma and worried,Second Opinion from Dermatologist /or GP,would help you to rule out any other causes of such a bruise with a lump.I doesn't see Any relation of this bruise with bump to Cymbalta.You should not worry as it is not Cancerous and is treatable bruise with bump.This reply would resolve your current issues and would reduce your anxiety with it.Hope this reply would help you to plan treatment with your doctor.Will appreciate your Hitting thanks and writing excellent review comments to help needy patients like you. Welcome with any other further query in this regard.Good Day!!Dr.Savaskar,Senior Surgical SpecialistM.S.Genl-CVTS"
},
{
"id": 22032,
"tgt": "Why the heart rate is low during the morning?",
"src": "Patient: Age 78, heart rate sometimes as low as 39 when arising in the morning,no noticeable symptoms. Also while hard exercising the pulse will suddenly drop from130-140 to 70-80 with no outward symptoms during the workout. After a few moments of continuing jogging, the heart will increase to 140 and slowly increase to a rate of approx. 155. Normal perspiration and uniform decrease in heart rate during cool down. My age is 77,good body mass index, cholesterol 175 with a good ratio, EKG and Holter and stress test show no problem, Please evaluate and advise. Doctor: hello, Normally, heart rate drops during rest and more during and in elder. Also, your maximum heart rate which is achieved during exercise is also fine. So you don't have any symptoms and holter is normal, so i think no intervention is needed at this stage. Also, you should get one thyroid test done, which can also cause low pulse rate."
},
{
"id": 191337,
"tgt": "What complications can arise when one has diabetes and hypertension?",
"src": "Patient: if you have diabetes and hypertention with a herniated disc, what complication can arise. I need a reference. I ve been searching the internet and can t find an answer. Diabetes and Hyperlipidemia is fairly controlled. My references have to be no older then 5 years Doctor: Hello and Welcome to \u2018Ask A Doctor\u2019 service.I have reviewed your query and here is my advice.If you are suffering from herniated disc along with diabetes and hypertension. Then you must be more cautious about your diabetes and hypertension control. i.e your fasting and postprandial, RBC and hba1c should be normal always. Also, your blood pressure should not exceed 140/90. Also, don't lift weights, don't sit down cross-legged, use a western toilet. Indulge a lot of calcium in your diet with milk, eggs etcetera. Drink a lot of water, so that you don't have constipation. Direct complications from diabetes and hypertension in the herniated disc are increased in Neuropathic pain and a decrease in sensations (I. e tingling and numbness in limbs) if diabetes and hypertension are fairly out of control. Herniated disc mostly depend on the extent of herniation and pressure over the cord that in turn depends on your lifestyle, food, weight etcetera. Hence follow a healthy lifestyle with proper and right food and back strengthening exercises so that you may not have any major problem due to a herniated disc. Hope I have answered your query. Let me know if I can assist you further.Regards, \u00a0\u00a0\u00a0\u00a0\u00a0Dr. NR Kumar"
},
{
"id": 220937,
"tgt": "What causes severe back pain during pregnancy?",
"src": "Patient: Dear Doctor,I am writing it for my sister who is pregnant and her due date is January 2011. I am enclosing her details below for your kind reference;Name: Archana ThomasAge: 27Gender: FemaleSince the first month of pregnancy onwards she is suffering from severe back pain and still it\u2019s the same. She consulted the doctor who advised her to take rest for few months and also as per the doctor\u2019s instruction she is taking the below medications: 1 (N P-100) progesterone softgelatin capsuls 100mg 2 Rolant-dsr 3 ecospirn 75 4 (duphaston) dydrogesterone tablet B PBut we are worried about taking lot of medicines in this early stage of pregnancy. What the doctor telling is that the back pain can lead to abortion and that\u2019s why she suggested her to take rest.Can you please assist us on this? Thanks,Ambili Thomas.Email ID: YYYY@YYYY Doctor: Hi, Thanks for the query under HCM forum. I understand your concern. Back ache in pregnancy can be due to- - Gravitational changes in pregnancy leading to over strain on back muscles. ( strengthening of the back muscles with pregnancy Yoga is the answer. ) - weakness of bones- due to calcium, vitamin D deficiency.( needs supplements ) - Pain in joints of pelvis ( pain management ) -Due to decent of baby in pelvis - Vaginal / pelvic infection ( Deep vaginal swab test & specific antibiotics) - Urinary infection( needs investigations & treatment. - Due to hormone deficiency with possibility of abortion.. this is more possible in early months of pregnancy, when placenta is yet to form/ after development of placenta when it's weak. NOw, as your sister is approaching her delivery, no point in hurrying now. Please investigate for it, if the pain persists after delivery. Thanks."
},
{
"id": 142257,
"tgt": "What is the cause and treatment for dizziness and pressure on frontal lobe?",
"src": "Patient: I just finished going on a walk this morning and I suddenly started seeing stars and got dizzy in one eye. I had to cover my eye and try to focus buy was barely able to drive home. When I got home 15 mins later I started noticing pressure build in my frontal lobe and progressively get worse. It has lasted and gotten worse over the last hour. I'm at ER now. What can it be? Doctor: Hello!Welcome on Healthcaremagic!Your symptoms could be related to optic nerve inflammation (neuritis optica). A stroke can not be excluded either. For this reason, I recommend performing a brain MRI with gadolinium enhancement to exclude this possibility. Hope you will find this answer helpful!Kind regards, Dr. Aida"
},
{
"id": 97661,
"tgt": "What is the medication for increased eosinophills of 11?",
"src": "Patient: which medicine should we prefer for increased eosinophills homeopathic or allopathic??is there any side effects from allopathic medicines?? i don't want to take medicines often for this..!my eusinophill has increased to 11 and i have been having dry cough for 3 continuous months every winter since 3 yrs Doctor: **1. cough for continuous 3 months every winter since 3 years is highly suggestive of 'Bronchitis' and consultation with Chest Specialist is highly recommended for necessary Examination and Investigations.2. If there is EFFECT of some -pathy, it will have its side effects, thus prior to starting medicine, do consult for possible adverse effects and/or any disease which might get aggravated due to drug intake.PS. Steam Inhalations:2-3 times a day.. warm salt water gargles. Avoid tobacco in any form. cover the mouth while coughing to prevent spread of infectioni. Avoid: deep fried food, sour fruits, cold water, rice and milk at night.ii.Take: lukewarm water with honey for gargles, old brown rice, chapatti with no oil, papaya, black grapes, warm/hot drinks often, take boiled vegetables, soft diet, sweet fruits."
},
{
"id": 177258,
"tgt": "Suggest remedy for loss of appetite and vomiting after intake of Azithral",
"src": "Patient: hi, my daughter is almost 4 years old .we wear in india for 2 months. we got back from india day before yesterday. she had abad caugh the before we get on the plane. so the doctor gave her azithral 200 twice daily for 6 days. today is her last dose. it seems doesnot work at all. now she lost her appetite and seems throwing lot. at this point what to do? Doctor: The common side effect of azithral is nausea , loss of appetite and diarrhea that is gastric upset. Now that the course is over her appetite should come to normalcy in few days. Regards"
},
{
"id": 195077,
"tgt": "What does a tiny blood blister on the testicle indicate?",
"src": "Patient: I found small little blood blister on my left testical, I was doing yard work and thought it was dirt until it didn t come off, looking closer I could see it was a tiny blood blister, is it possible that I might have pinched the skin to have caused this? As I am not sexually active haven t been with anyone in 8 years. any advice? Doctor: Hi, It can be a superficial infection or eczema. STD is very unlikely. As the first line of management, you can apply topical antibiotics like mupirocin after cleaning with soap and water. Hope I have answered your query. Let me know if I can assist you further."
},
{
"id": 21518,
"tgt": "How to overcome Guillain-Barre syndrome?",
"src": "Patient: I am an artist - musician, age 56, male, married, residing in Goa at present with my family. I was a victim of GB Syndrome at the age of 36 in the year 1991. Doctors who treated : Dr Dubhashi at GMC, Dr Nathan at Shushrusha, Dr Arun Shah, Dr Mahale(cardiologist), Dr Prajakta Kanitkar in Pune, Dr S Y Kalelkar in Mumbai and Dr Shyam Bhandarey in Goa. Due to cardiac problem I was advised not to do vigorous physical exercises. With the help of Prescribed Medicine, meditation and active performance of classical music I could recover upto 90%; that too after 19 years. Disability has remained in the legs. Can I come out of that? Doctor: Hello SirI completely understand your concern and I would like to tell you that Approximately 80% with GBS become independent after 6months of treatment, about 60 % achieve full recovery of the bodies motor strength by 1 year. And in about 5-10% of patients full recovery is very much delayed or incomplete recovery.Now in your case as it was quite a delayed recovery (19years) so probably the weakness in legs may persist. So it's advisable that you should continue with physiotherapist for leg strengthening exercises as probably it would be helpful.I wish you good health"
},
{
"id": 206531,
"tgt": "Suggest methods to correct an aggressive behavior",
"src": "Patient: I have an 11 yr old adopted son with XYY.Due to his aggressive and difficult behaviour he is in residential care.In the u.k.doctors are reluctant to treat with meds.but I am a nurse and I am reading about the use of drugs such as olazapine,valporate and oxacarbamaze in the u.s.Do you think I should ask for him to be trialed on these drugs?I am desperate to give him some quality of life..so far that is very compromised.. Doctor: Hello Aggressive behaviour is common in this age and boys tend to have sense of independence which cause them to defy orders of parents. Most of the times aggression is positive in form of verbal aggression and also physical aggression. Passive aggression is also common. Most of individuals improve with time but if the aggression is not tolerable and cause significant dysfunction then there is need to consult a Psychiatrist and Counsellor. Medicines like Sodium Valproate can be used to decrease aggression but only under expert supervision.Thanks, hope this helps you"
},
{
"id": 37130,
"tgt": "Will taking Zinc & B6 be preferred for chronic fatigue & lyme disease?",
"src": "Patient: Hello. I've had chronic fatigue/Lyme disease for 8 years. Got sick at 13 and I'm 21 now. I was on anti biotics for 8 months with moderate relief. However when I got on the paleo diet and started supplementing high doses of zinc and b6 as recommended by a naturopath I felt 90% better within weeks. I'm not sure why though. Is there any connection between the two supplements and disease? Doctor: Dear friend , Greetings from HCM.. i read your question .. you were diagnosed to have lyme disease ..It is due to rare bacteria borrelliia ... it is transmitted through tics ... it could affect mainly joints (mimics rhumatoid arthritis), nerves and heart too ... treatment : antibiotics are primary doxycyclin is prefered...in addition, since it involves nerves it may cause nerve damage to prevent that we usually supplement B12, folic acid, B6, and minerals... please take care of your diet take more of green leafy vegetables, fruits, and protein rich diet you will be alright soon ... THANK YOU"
},
{
"id": 51576,
"tgt": "What are the consequences of presence of E.coli and albumen in urine ?",
"src": "Patient: my urine examination has showed up albumen value 3 plus and its culture has shown growth of e.coli in it after 48 hrs.what does this indicate and what are its consequences?how can this be treated?please help me to know about it.. Doctor: Hi,Welcome to HCM.The urine report suggest urinary tract infection,you need to take antibacterial drug to reduce the infection and have plenty of fluids.If untreated you may develop fever associated with chills etc.For albumin in urine further evaluation of kidney function has to be done like serum creatinine to rule out kidney disorder.Take care."
},
{
"id": 42789,
"tgt": "Will i get pregnant if had sex immediately after removal of implant?",
"src": "Patient: had implant removed today after 6 month due to wanting a baby, been ovulating and having unprotected sex on the correct days with my partner, clinic woman told me today how id be unprotected as soon as i got it removed, how long does it normally take to fall, and could i possibly fall now? Doctor: Hi,Thanks for writing to HCM .It will take one month atleast to become pregnant I.e from next cycle. If some damage has occured while removing of implant that will shed off by next cycle. There are less chances of getting pregnant this cycle . Since you are ovulating normally there shouldn't be any problem for you to conceive. Hope I have been helpful .Regards Dr.Deepika Patil"
},
{
"id": 34437,
"tgt": "What causes painful red bump after spider bite?",
"src": "Patient: help i think i got bit by a spider and i have 2 bites and they are the size of golf balls and very hard , hot , red and extremely painful . what should i do ? im scared to go to the er and dont know if this warrants taking away from real emergencies . Please help . Thank you Doctor: Dear,symptoms mentioned by u are suggestive u about possible spider toxin leading to local inflammation bu as mentioned by u, it they are golf ball sized than its really big enough and u must visit a physician or ER.U must start chlor pheniramine along with antibiotic like amoxyclav. asap.ice application will reduce swelling and tenderness.chymotrypsin along with serratio peptidase can be used in tds dose.thanksrevert back for follow up."
},
{
"id": 179973,
"tgt": "Will lysol help to treat rashes on feet/hands of a baby?",
"src": "Patient: My 18 min old grandbaby has hand foot and mouth...... She has the rash on her feet, hands, a couple places on her mouth nose area and rash on her bottom. I have her for the night, will Lysol work to spry everything down with to help keep it from spreading to my other rand child and daughter? Doctor: thanx for posting query .hand , foot and mouth disase is a viral disease and it is a mild disease and it will be cured by itself in due course of time . lysol does not have any role in treating it or preventing it. you can give PCM for fever, and antihistaminic in case the baby has itiching . rest assured your baby will be fine"
},
{
"id": 170230,
"tgt": "What is causing my 3 years old s penis to itch?",
"src": "Patient: My son is 3 yrs and 5 mnths old, he seems to be having some sort of irritation on his penis, I think it itches because he becomes relieved when he scratches it. This has been going on for about 2 days now what could be the problem? He also developed a sudden spot about 3 weeks ago on his head where now his hair does not grow, my research shows that it could be an infection/fungus. He has been using antibiotic alot because he is repeatedly develops a flu. Please explain, could the two problems (spot on head and itchy penis be related), what is the solution? Doctor: HiWelcome to the HCMThe most likely reason for your child's symptoms is fungal infections which are common after prolonged use of antibiotics.Do not worry. You may use clotrimazole lotion at both the places thrice daily. Hopefully it will improve in next 2-3 days. Take care"
},
{
"id": 49168,
"tgt": "Is injection of tordol and norflox right medication to treat kidney stones?",
"src": "Patient: Just got back from ER...kidney stone. Second one in two days. CT scan says it is 5mm in size. This will be number 8 for me. I am a 70 yr. old female and I do have a high tolerance for pain. I was gievn an injection of tordol and one of norflex and something for nausea. I feel much better right now. I was told that the stone is half way down the ureter. I am hoping it will pass quickly. Doctor: Hello,Thank you for the query.The injections you have had consist of an analgesic (toradol) to relieve the pain, and an antibiotic(norflox) to prevent infection of the urinary tract. The stone is relatively small and goes away by itself. I think it will pass quickly the remaining half of the ureter."
},
{
"id": 29200,
"tgt": "Do fever, cough, diarrhea and weakness need medical attention?",
"src": "Patient: I have had a fever, Fabry s pain crises, coughing, wheezing, queasiness, diarrhea and extreme weakness for 5 days. My fever is finally starting to go down (from a high of 102.6 to 100.5). Is there any point in seeing a doctor, or should I just let it run its course. Doctor: the best time to start treatment when infection is growing and high fever..now u r in recovery phase ..that means that infection decrease now ..if symptoms improved ..dont warry..but u need fluids ..fluids plse"
},
{
"id": 71166,
"tgt": "What causes pain in back in the lung area?",
"src": "Patient: my wife is on wafrin for over one year now she has had pain on both sides of her back in the lung area and i have been massaging phorpain gel, after a scan last year it was discoverd that she had three clots in the lung i am concerned that it may be another clot, she is sweating a lot and her skin is cold and very moist i may add that she is also on prednisolone Doctor: Hello and Welcome to \u2018Ask A Doctor\u2019 service. I have reviewed your query and here is my advice. Pulmonary embolism (PE) can be recurrent. So better to consult pulmonologist and get done clinical examination of respiratory system, blood pressure monitoring and CT pulmonary angiography to rule out PE. If CT scan is normal then no need to worry for PE. Hope I have answered your query. Let me know if I can assist you further."
},
{
"id": 151725,
"tgt": "There is no improvement after taking treatment as the mouth automatically shakes and does not have any control on facial muscles",
"src": "Patient: hi my father is shaking his mouth automatically. he canot control his mouth ....we feel so sad.......he is taking treatement but no improve .....please help dr Doctor: Hi, welcome to H.C.M.Forum. he must be having some problem with his tongue. he should use neurotropics in high dose, and anxiolytics. ct scan is to be taken and treated if some thing is wrong accordingly, under the guidance of a neurologist . he is moving his mouth to compensate the defecit of the tongue. thank you."
},
{
"id": 133986,
"tgt": "Suggest medication for bad pain on inner right leg",
"src": "Patient: I have bad pain when I walk on my inner right leg it feels like in between my hip and leg for about 5 days and its not getting better I have to lie down on my right side or it will hurt and i can walk for no more than 30 sec. I have to walk with both knees bent if my right leg straighten out when i walk the pain will come very fast. I m taking inflammatory and T 3s and they don t work very well Doctor: hi,thank you for providing the brief history of you.a thorough neuromuscular assessment is neededas mentioned by your symptoms, it appears to me that you need a thorough neuromuscular assessment. As post which is the symptoms are connected to nerve related issues then an MRI of the lumbar spine will be performed to see the soft tissue status. As even you mentioned that the position you need to take for relieving the pain and your knees stays bend while walking. With medication and physical therapy combination there is a massive improvement in the health status. Also in my clinical practice dealing with such symptoms and post in detail neuromuscular assessment and MRI the precision of therapy is planned to get the optimum 99% results.RegardsJay Indravadan Patel"
},
{
"id": 1113,
"tgt": "How to get pregnant with adenomyosis?",
"src": "Patient: hi any ob gyn on line? just want to know had endometrosis did the surgery last february and they remove my right ovary and the doctor told me that i still getting the pain cos i have adinomyosis as while just want to find out i really want to have a baby im 35 years old cod tell me what to do? Doctor: Hi, I think you need some medical support for conceiving. Take some medicines like clomiphene for the growth of your follicles and track your follicles growth by repeated ultrasound and when your follicles is more than 17 to 18 mm, take injection for rupturing the follicles. Be in contact with your husband for 2 to 3 days after injection. Take progesterone for next 2 weeks. Do a urine pregnancy test at home after that. You can try like that for 3 to 6 cycles. Hope I have answered your question. Regards Dr khushboo"
},
{
"id": 19384,
"tgt": "What causes weakness post Amlodipine intake?",
"src": "Patient: Since it is the weekend, of course, I cannot speak with my internist who yesterday recommended I take Amlopdipine besylate 2.5 tabs, not because of high blood pressure, but perhaps related to his recommendation I take more salt in my diet. (I have not seen the results of blood work that is the reason for this, nor have I spoken to him--only to his assistant.) It s either a high or low reading of potassium? In any event, I took one tab last night at 7:30 but this morning felt weak, etc. so took my b.p. and it was 75/57/84; after breakfast and several hours later it was 84/70/72. At 12:35 this afternoon it was 101/71/73. I do not intend to continue taking this medicine until I get more information on Monday morning. (Normally I do not have any high blood pressure except last week, and this was corrected with different medication.) I realize it is difficult to comment without more information, but it helps to contact somebody. Pauline Cosyns, age 92 Doctor: Hello, Your mentioned bp are readings are low, which is the cause for your weaknesses. We don't wish to initiate Amlodipine or other bp medicines unless bp is above 140. So it think you should discontinue the medicine and have some salt sugar water if bp falls below 100 or you continue to have weakness. Monitor your bp thrice a day using electronic home bp apparatus. Hope this helps you and get back if you have any doubts."
},
{
"id": 179621,
"tgt": "Can Azee, timnic and crocin be given to a child for throat pain and fever?",
"src": "Patient: hi, docter my daughter is 6.5 yrs old & she is having cold since tom. it stated with throat pain in morning till night she was having high fever, throat pain , cold & nose blocked, so I gaved her timnic & crocine ds yest. night, but she is having fever in morning also ,so can I give her AZEE 200 along with timnic & crocin ds pls advice. Thanks. Doctor: Hi you can give t-minic and crocin(paracetamol) but for a prescription drug specially antibiotics you need to consult your doctor for type , doses and duration of antibiotics if required.thanks"
},
{
"id": 148794,
"tgt": "Numbness in arms which had sustained nerve and arterial damage from an accident. How to treat this?",
"src": "Patient: I WAS ABOUT 13 YRS OLD , WHEN MY HAND WENT THRU A SHATTERED WINDOW,,, I HAD A OPERATION AT THE TIME... MY DOCTOR THEN TOLD ME I HAD CUT 2 NERVES AND 2 ARTERIES... WHAT ARE MY CHANCES WITH PAIN, IM NOW 41YRS OLD.. I HAVE NUMBNESS AT TIMES... IT SOMETIMES SHOOTS ALL THE WAY TO MY RIGHT ARM.... WHAT SHOULD I DO? I DONT KNOW IF IM GOING TO NEED ANOTHER OPERATION OR SHOULD I JUST EXCEPT IT..... Doctor: Welcome to Healthcare-MagicGreetings of the dayIts common to see pain in the distribution of nerve following nerve injury- it occurs usually due to nerve Regeneration. There is no cause of concern if there is no weakness in the region. Occasional tingling and pain is very common, its highly likely that you will require any further Surgery.Take CareRegardsDr T ShobhaMBBS MD"
},
{
"id": 215482,
"tgt": "What causes persistent pain in lower right abdomen?",
"src": "Patient: I had a tremendous pain in my lower right abdomen like something inside dropped. I went to the ER and through cscan or imaging they ruled out appendicitis and kidney stones. Nothing abnormal showed up in the scan or the blood work. They did specify I had a hiatal hernia. The pain is still there and feels as if somthing moves when I bend a certain way, it also hurts to lay flat. I am 53 5 ft. 5 female weight around 132. The only things I have had done is a colonoscopy in January and 7 years ago a ablation (not sure of the spelling). What could be causing the pain to continue. Would the scan have showed a drop in the cervix etc? Doctor: Hello, It could be a renal stone probably. As a first line management, you can take analgesics like paracetamol or aceclofenac for pain relief. If symptoms persist, it is better to consult a physician and get evaluated. An ultrasound scan is required for further assessment. Hope I have answered your query. Let me know if I can assist you further. Regards, Dr. Shinas Hussain, General & Family Physician"
},
{
"id": 21686,
"tgt": "Suggest treatment for chest pain and elevated blood pressure",
"src": "Patient: Good Evening, 2 years ago the doctor performed an EKG as a baseline for new patients. Last month another EKG was performed due to slight chest pain and blood pressure being elevated. The EKG reflect a changed so the doctor ordered an Echo. The results from an Echo last week showed a leaky valve. The doctor stated that the leaky valve was more than likely a contributing factor to the migraines that I have had for years. The doctor suggested returning in a year for another Echo. Received blood pressure and anxiety meds. My blood pressure has dropped. Do you recommend following up with a Cardiologist? Thank you, Cecilia Doctor: Hi CeciliaAfter going through your medical details I would like to ask what sort of leaky valve and would also like to know the reason for that leakage as a cardiology practitioner and also your Blood pressure has dropped. I would like to advice you to consult a cardiologist personally for evaluation of your ECG, Echo and titration of your BP medicines.Wish you Good Health"
},
{
"id": 124159,
"tgt": "Why does elbow hurt even after using ice, tylenol and motrin?",
"src": "Patient: Dear doc, I have not been doing much this summer and just got back into vollyball. I have been at a camp for the last week and my arm is Killing me! It starts where your arm folds and both ways. Please help me because I can barely move it and it aches really bad! PS I have tried ice,Tylenol, and Motrin. Doctor: Hi, As you played a volley Ball this could be related to ligament sprain of the elbow joint. Doing hot water fermentation should be of use to release the muscle tightness which has occurred due to ligament sprain. Having stabilized the elbow into crepe bandage should be good for healing to occur. Hope I have answered your query. Let me know if I can assist you further. Regards, Jay Indravadan Patel, Physical Therapist or Physiotherapist"
},
{
"id": 50096,
"tgt": "Suffering from chronic renal problem, under dialysis. Had low Hb levels. Had convulsions during blood transfusion. Reason?",
"src": "Patient: Yes thanks my mom is suffering from diabetes and hypertension since many years and now she has chronic renal problem and under dialysis since one year.she got infected with hcv virus last month .she was under the hcv treatment with Pegasus injection 135ml dose and copegus 200mg once a day .last week she got drop in her hemoglobin 6 so we plan for blood transfusion . We went for dialysis and than when last hr left they started blood transfusion and she got the convulsion . Please tell me why it happen Doctor: causes of seizures during dialysis can be abnormally high or low blood pressure, low or high sodium or calcium levels, presence of infection, high fever and some drugs specially antibiotics.rapid removal of toxins during a fast dialysis can also cause seizures.if she has it again then some tests of blood and a CT or MRI of brain may need to be done and some antiepileptic drug may need to be given regularly.it is advisable that you consult your nephrologist and follow his advise."
},
{
"id": 116285,
"tgt": "What causes rise in WBC levels?",
"src": "Patient: For several months my WBC has steadily risen, from 13.0 to 18.4 , with neutrophils at 13.9 and my iron is at 41 and ferritin at 16 , also showing large platlets. Doctors have been dismissive until now I have been refered to a hematologist at a cancer center. Could it really be cancer?? What could some possibilities be?? Doctor: Hello and welcome to HCM,A rise in white blood cell count (WBCs) and associated rise in neutrophils is suggestive of infection especially bacterial infection.Thus, a clinical assessment to look for focus of infection is required.If infection is ruled out, then neoplastic conditions have to be looked for.However, neoplastic condition is associated with very high blood counts so possibility of neoplastic condition is low.Iron level of 41 and ferritin level of 16 is low and thus investigations have to be done to look for the cause of low iron and ferritin.Thus, consult your hematologist for further management.Thanks and take careDr Shailja P Wahal"
},
{
"id": 20565,
"tgt": "What causes high blood pressure?",
"src": "Patient: 15year old male,6 tall 170 give or take 5 pounds, went for pyshical said blood pressure was high works out 3 to 4 times a week . mostly strengh training for football. drinks a protien shake after workouts, was wondering if this could cause high blood pressure Doctor: Hi,High bp in young has certain causes. Protein shake is definitely not one of them. You need to be evaluated for the causes so that definitive treatment could be done.Causes include1) Hyperthyroidism - test to be done - Thyroid function test2) Kidney & renal artery abnormalities - tests to be done - KFT, renal Doppler3) Pheochromocytoma - test - 24hr urinary VMA &\u00a0catecholamineHope you get well soon.Thanks"
},
{
"id": 194063,
"tgt": "What are the side effects of following masturbation?",
"src": "Patient: doc am from india can u plz tell me i use to masturbate since 7 years and now my age is 20 and am getting worry that it will effect on my real sex life after marriage that i will never give a birth to my child is this ture and every time i try to stop masturbation it wont why doc plz help me really am worry about this Doctor: Hi, Masturbation is normal, it\u2019s just another way of having sex without partner. It\u2019s a good practice , helps to calm your mind and release happy and stress bursting enzymes. Having 2 to 3 times a week is not a issue , more than that is also not a issue , but needs adequate nutrition and exercise which will be difficult to fulfil. Ways to reduce Masturbation 1.\u00a0\u00a0\u00a0\u00a0\u00a0Sex education 2.\u00a0\u00a0\u00a0\u00a0\u00a0Physical exercises 3.\u00a0\u00a0\u00a0\u00a0\u00a0Sleep adjustment 4.\u00a0\u00a0\u00a0\u00a0\u00a0Quality of sleep 5.\u00a0\u00a0\u00a0\u00a0\u00a0Healthy and balanced diet 6.\u00a0\u00a0\u00a0\u00a0\u00a0Programming for leisure time 7.\u00a0\u00a0\u00a0\u00a0\u00a0Development of spirituality 8.\u00a0\u00a0\u00a0\u00a0\u00a0Taking a cold bath CAUSES OF MASTURBATING 1.\u00a0\u00a0\u00a0\u00a0\u00a0Curiosity 2.\u00a0\u00a0\u00a0\u00a0\u00a0Unfamiliarity with sexual issues 3.\u00a0\u00a0\u00a0\u00a0\u00a0Seclusion 4.\u00a0\u00a0\u00a0\u00a0\u00a0family problems 5.\u00a0\u00a0\u00a0\u00a0\u00a0suspicious and unfriendly friendship 6.\u00a0\u00a0\u00a0\u00a0\u00a0lasting diseases 7.\u00a0\u00a0\u00a0\u00a0\u00a0Sense of inferiority 8.\u00a0\u00a0\u00a0\u00a0\u00a0Imitation 9.\u00a0\u00a0\u00a0\u00a0\u00a0premature puberty 10.\u00a0\u00a0\u00a0\u00a0\u00a0cultural factors Negative Effects of Masturbation 1.\u00a0\u00a0\u00a0\u00a0\u00a0The weakness of physical power 2.\u00a0\u00a0\u00a0\u00a0\u00a0Poor vision 3.\u00a0\u00a0\u00a0\u00a0\u00a0Damages to the sexual organs and impotence 4.\u00a0\u00a0\u00a0\u00a0\u00a0Lack of memory and forgetfulness 5.\u00a0\u00a0\u00a0\u00a0\u00a0Aggression and anger 6.\u00a0\u00a0\u00a0\u00a0\u00a0prefers loneliness Hope I have answered your query. Let me know if I can assist you further. Take care Regards, Dr S.R.Raveendran, Sexologist"
},
{
"id": 67174,
"tgt": "What does a lump right on hip flexor indicate?",
"src": "Patient: Hello I have a lump right on my hip flexor and I was wondering if it is cancerous there is no pain, I noticed it about a week ago and I am in competitive soccer so I don t really know what it is, I have like been keeping my eye on it and it s has not grown at all but it has like gotten smaller!? Doctor: HiWelcome to hcmYou have not told your age, size of lump and it increases on coughing. If increasing on coughing and decreasing on lying down means it is hernia which needs surgery. If its size not changing it can be a lymph node/ kind of gland which is enlarged either due to infection or tuberculosis/TB or very rarely some kind of lymphoma/type of cancer. But exact cause can be diagnosed only after you provide the above mentioned information.Regards"
},
{
"id": 133217,
"tgt": "Reason for pin and needles in the neck and head along with burning in the legs?",
"src": "Patient: Hi I have pins and needles in neck and head, also burning in left leg and tingling in my feet. It also feels like needles are pricking my finger tips and worst of all extreme fatigue....used to be competitive swimmer, run and do triathlons, do not have the energy for any of the above anymore . Regards Denise Doctor: HiThis is known as neuropathy and it could be from diabetes, vitamin B12 deficiency, or a kidney or liver conditionYou need to see a doctor for blood work to see what exactly is the problem"
},
{
"id": 150654,
"tgt": "Hit at a dead spot, extreme burning in upper spine and neck. Unable to tilt head and intermittent nerve pains. Is it healing process?",
"src": "Patient: Hi there, I recent was involved in a rear end collision, I was hit at a dead stop on the freeway at 50mph. I experienced extreme burning like someone lit a firecracker in my upper spine and neck after impact. It s been three weeks since and I am still unable to tilt my head either direction, I also am experiencing intermittent nerve pains in my extremities, spasms, weakness and fatigue In my upper body and constant pain in my neck and shoulders. There is also an awful crunching noise in my neck now that hasnt gone away. I had X-rays and they said everything was normal, but it definately doesn t feel like it! I m worried I might be permanently damaged! Im looking for the next step to take in this long healing process? Doctor: Hi, Thank you for posting your query. Based on your symptoms, we should exclude a diagnosis of central cord syndrome. In this condition, due to whip lash injury, the central parts of cervical spinal cord in injured, resulting in symptoms similar to what you have. However, it is reassuring to note that symptoms improve with medical treatment and surgery is not required. MRI of cervical spine may be done to confirm the diagnosis. Pregabalin capsules may be taken for symptom relief. Please get back if you require any additional information. Best wishes, Dr Sudhir Kumar MD (Internal Medicine), DM (Neurology) Senior Consultant Neurologist Apollo Hospitals, Hyderabad, My personal URL on this website: http://bit.ly/Dr-Sudhir-kumar My email: drsudhirkumar@yahoo.com"
},
{
"id": 2365,
"tgt": "Is there chance for pregnancy with low level beta?",
"src": "Patient: Hi dr. I am 36 yrs old. I did my Fet on June 20th. 3rd day 8 cell good quality embryo. My beta was on 1st July and it came out as 15.6. So my dr told to repeat beta after 48 hrs and it came out as 23.6. So she advised me to do a beta again on 9th July. I am worried. Is there a chance of pregnancy with such low level beta. Please reply. Doctor: Hi,The rise in Beta HCG level is not very satisfactory. In 48 hrs it has not doubled. It doesn't seem to be a healthy pregnancy. But surprises can come any day, next beta HCG may be high & rise later. There is a pregnancy for sure, hence the Beta HCG, but healthy or not id doubtfulAll the bestDr.Balakrishnan"
},
{
"id": 22819,
"tgt": "Can I take aspirin regularly to prevent heart attack?",
"src": "Patient: i am male, age 33, 180cm, 83kg, my LDL is 190(4.88) and HDL is 1.05. I was on Lipitor for two years 10mg. it kept my LDL at 100, but make my legs weak. Now i am off lipitor for the last three months and my legs feel better. in additional to healthy food, is taking aspirin regularly instead can be another solution to prevent heart attack? what is the recommended dosage? Doctor: hi, if you have discontinued it then you should monitor your lipid profile 3 monthly, do regular exercises, low fat diet.Try other medicine like rosulip 5 mg bedtime and e week after starting you can do cpk test, it will indicate if any sideeffect is there or not. this medicine has got cardio protective effect besides it's cholesterol lowering effect. Advisable to continue life long. At this age Aspirin is not indicated and even if you want to take you take 75 mg bedtime."
},
{
"id": 10517,
"tgt": "Suggest treatment for hair fall due to hypothyroidism",
"src": "Patient: Hello sir, My name is Amruta, I am 26years old. From past one year I am suffering from hypothyroidism and because of that I am suffering from severe hair fall and premature graying of hair. Recently I have noticed receding hairline and most of the gray hairs are present in the right hand side of the scalp. As per my doctor she suggested me medicines for hypothyroidism and told me that it will address the hair problems also but day by day my hair loss and gray hair problems are increasing. So it will be helpful if you can suggest me whether I should visit a dermatologist or I should take some medicines. Thank YOU. Doctor: Hello and Welcome to \u2018Ask A Doctor\u2019 service. I have reviewed your query and here is my advice. As per your case history of hair fall due to hypothyroidism, my treatment advice is - 1. Use a good herbal hair oil and shampoo for routine use. 2. Take an iron supplement and vitamin b12 supplement. 3. Other treatment options are topical Minoxidil, mesotherapy done by a dermato-surgeon. Hope I have answered your query. Let me know if I can assist you further."
},
{
"id": 7760,
"tgt": "Which medicine and care regimen should one follow to get rid of acne ?",
"src": "Patient: I am suffering lot Pemples on My Face ..Its Looking So ugly..please give some suggestions and instruct some medicines Doctor: Hi,Kiran, Welcome to HCM, Wash your face with face wash 3-4 times a day, Apply branded pimple cream, Go for antibiotic medicine avoid applying face cream or powder. Avoid stress, Avoid fried and junk food. Ok and bye."
},
{
"id": 67763,
"tgt": "Could fluid filled knot on the head be a matter of concern?",
"src": "Patient: I fell off a step stool at my house last night. We have marble floors and it was a pretty nasty fall, it slid out from under me and I fell flat, my head bounced. The knot on my head was baseball size after the fall. After showering tonight it now feels soft, like there s fluid. Should I be concerned? Doctor: HiIt looks like Haematoma, collection of blood or Edema. The swelling gets subsided in course of time. Nothing to worry. Regards"
},
{
"id": 210831,
"tgt": "Is memory, speech and concentration problems related to stress?",
"src": "Patient: Hi, I am doing some research as I have recently been experiencing memory and concentration issues along with speech. I'm wondering if something is effecting my central nervous system or if it is related to stress. What type of doctor should I set up an appointment with to have this checked out? Doctor: DearWe understand your concernsI went through your details. I suggest you not to worry much. You are partially right and partially wrong. You are doing some research. Due to stress it is possible that you experience problems in speech, concentration and memory. But it is not to be directed towards central nervous system. Nothing could be wrong there. This is just stress induced.You have to just take rest for some time. If it is not possible, then learn to relax. Practice relaxation techniques. You can post a direct question to me in this website mentioning all your details and I shall prescribe apt relaxation techniques which is scientific (Psychotherapy methods). Results are guaranteed. Need to learn to live happy with stress.Hope this answers your query. Available for further clarifications.Good luck."
},
{
"id": 151358,
"tgt": "Suffering from parkinsons disease. DBS treatment?",
"src": "Patient: hello i am dr ashish a dentist from bangalore. my uncle is suffering from parkinsons disease from past 10 years(he is under medication ever since). i wanted to know more about DBS treatment in bangalore. is there any hope if he under goes DBS? is there any other treatment which cures this disease? who is the best doctor here in bangalore to consult?... kindly help... Doctor: Hi Dr Ashish, Thank you for contacting Healthcare Magic. Deep brain stimulation (DBS) is a helpful treatment for Parkinson's disease (PD). It is indicated in certain specific situations such as tremor-predominant PD and drug-induced dyskinesia. In Bangalore, you can meet either Dr Murali Srinivas (Neurologist at Manipal Hospital) or Dr Guruprasad (Neurologist at Columbia Asia hospital) to know more details about the procedure. Both the doctors are good neurologists with experience in DBS and I know them personally and academically. Best wishes, Dr Sudhir Kumar MD DM (Neurology) Senior Consultant Neurologist Apollo Hospitals, Hyderabad"
},
{
"id": 28579,
"tgt": "Are frequent sinus infections normal in a TB patient?",
"src": "Patient: Hi, I suffered a mild TBI several years ago. I was hit with a golf ball line drive in the forehead. CT scan showed sinusitis immediately after injury but no fractures. The radiologist originally thought it may be a CSF leak at first but ruled that out after a second opinion. Since then I have had chronic migraines, chronic sinus infections, in both the maxillary and sphenoid sinuses, chronic pain and insomnia. in Feb I was hospitalized in ICU due to a severe sphenoid infection and cellutis. Doctor on call said he had only seen my condition twice in 32 years and I would need emergency surgery. CTscan showed severely infected sphenoid but MRI did not show the same result so they opted not to do surgery and give me multiple rounds of IV antibiotics which tamped down the infection. The infection continues to come back so I have a sphenoid sinus surgery scheduled with septoplasty as well in a couple of weeks. Is it common for TBI patients to have this extent of sinus issues? Should I be worried the infection may be in the bone, which is why the CTscans (most current a few weeks ago) show the chronic infection but the MRI results do not.? Thanks Doctor: Hello and Welcome to \u2018Ask A Doctor\u2019 service. I have reviewed your query and here is my advice. Tuberculosis may result in an immunocompromised state of the body. Your frequent relapses of sinus infection is chronic, continue to take the medications prescribed. No worries, after you have had your Septoplasty surgery & surgery for sphenoid sinus done, your situation will surely get better. You will be more comfortable after recovery. Hope I have answered your query. Let me know if I can assist you further. Regards, Dr. Nupur K."
},
{
"id": 3312,
"tgt": "Could i be pregnant as i have short periods?",
"src": "Patient: hi i am recently married in may 2015 and trying to conceive, last month i had very short and light periods of one and a half day , this month on 18 of August 2015 i got bleeding of three days it was not heavy , although my periods was due on 24 of this month but i got it earlier , i havnt had any pregtest yet, suggest me can i be pregnant? I am 5 2 and weight is 50 Kg,always have regular periods of 4 -5 days , i am fit and healthy otherwise. Doctor: Hi there,After going through your question, I am of the opinion that:1) Some women get little bleeding every month for 3 to 4 months of their pregnancy.2) However, that bleeding is very light and does not even fill one pad.3) Sometimes, when you start trying for pregnancy, there are hormonal changes which can cause a change in normal menstrual pattern.4) You can get a urine pregnancy test at home to just check whether you are pregnant. If that is negative, you can just sit back and relax. In case you miss your periods, you can repeat the urine pregnancy test or a blood test called serum beta HCGI hope this was helpful.Regards,Dr. Shikha Shah"
},
{
"id": 206561,
"tgt": "Suggest treatment for restlessness,suicidal thoughts,irritation, mood swing,loss of energy",
"src": "Patient: hello doc, 24 female here, m suffering from chronic depression from last 20 months due to differences between me n my parent. as they are having conservative values they used to ban my social activities n repeatedly blaming me for my so called mistakes due to which, from last 20 months, i even niether met a single frnd nor talked. no one understands me n there is no one to whom i can share my feelings even my mom use to blame me 2.. thus i feel too depressed n lonely.. i cant sleep till 4 am thus wake up in noon.. this became my routine.. even my eating, bathing, sanitary habits became worse, sometimes i use to hold my hunger maximum i can, upto 8-9 hrs n sometimes i use to eat at intervels of half to one hrs. now a days m fearing of getting rejected by every one, dats why became hopeless n worthless.. other symptoms include restlessness, suicidal thoughts, irritation, mood swing, loss of energy, loss of interest in everything.. so i request you to help me to get rid of this worst era of my life.. thank you Doctor: Hello,Chronic depression needs to be treated with antidepressant medications.I advice you to start with tablet Paxidep 25mg every night. This will help you in depression and would relieve all your symptoms. But i must tell you that antidepressants take 10-15days to show their effect. So please take medicines regularly.Thanks"
},
{
"id": 43926,
"tgt": "Failed IVF procedures. On AKT-4 and R-cinex. Damaged endometrium lining. Will I be able to conceive?",
"src": "Patient: hello doc, two three months back i did ivf but failed after that i did a tb test which comes as a positive. so doctor prescribed me a akt-4 for 2 months and r-cinex for another 4 months, now i m following as doc advice me. before my ivf my endometrium was 26mm and now my endometrium is only 2mm and doc said that my one endometrium lining is damaged out of three lining. so can i able to conceave in a future and is there any treatment for this type of problems... please help me i m very concerned Doctor: Hello Thanks for your query.You had undergone IVF 2 months back which failed.You have been diagnosed to have Tubercular Endometritis and you have been taking anti Tubercular drugs regularly.The thickness of your endometrium has reduced to minimal only 2 mm. So It is very difficult to predict about your chances of conceiving in future Let us hope that your endometrium regenerates after AKT treatment."
},
{
"id": 145457,
"tgt": "What causes lipoma underneath the right scapula?",
"src": "Patient: I have decks syndrome. I believe I have a lipoma underneath my right scapula and I have had pain for 2 years now. I have complained to my surgeon, my primary, my pain management. Any ideas? I am in serious pain daily. I have had 28 surgeries in 3 years and I have had some painful lipoma but I believe this one is my second most painful lump. Doctor: Hello, Welcome to HCM. First of all your swelling should be confirmed as a benign condition like lipoma. You may be able to get the pain causing nerves ablated. After confirmation of the diagnosis, you should consult a doctor specialist in pain management. Best wishes"
},
{
"id": 41496,
"tgt": "What causes a slimy discharge with breast pain?",
"src": "Patient: hi, I am 32 yrs, i am married for 3.5yrs and want to concieve. I get irregular periods and have digonised with PCO. Of late i am getting white slimy discharge and little pain in breast. My periods are due in 5 days. Are these normal symptoms or anything else. Doctor: HIWell come to HCMof course this is case of irregular menses, and ovarian cyst, but this does not causes any breast discharge, if you have this, better to get this clinically examined, just for the safety, this could be normal one, but no need to take any chance, take care have a nice day."
},
{
"id": 221834,
"tgt": "We didn t use protection can I still get pregnant?",
"src": "Patient: Hi, may I answer your health queries right now ? Please type your query here... Hi, my question is that I alway have my period rite but this month When I got my period for 3days I was spotting ant then for 5days I was bleeding and then I had sex with my husband but We didn t use protection can I still get pregnant? Doctor: Hallow Dear,The 3 days spotting you had was premenstrual spotting. The menstruation is to be counted from the day you has normal bleeding for 5 days. If immediately after that you had intercourse with your husband, there is most remote possibility (practically no possibility) of pregnancy. The egg gets released 14 days prior to the next expected menstruation. The egg has life of 24 hours while the sperms are active for 72 hours. Therefore a period of about a week around the day of egg release is the Fertile Window. Your intercourse was far away from this period. So the chances of pregnancy are almost nil. Hope this helps you,Dr. Nishikant Shrotri"
},
{
"id": 80239,
"tgt": "Suggest treatment for granulomatous lesions seen in CXR",
"src": "Patient: Hi, 2 months ago my husband was having persistent cough and coughed out blood once. I am not sure if it was blood streaked sputum or all blood because he did not tell me right there and then, so I did not see the color and everything. So we went to see a doctor and she ordered chest xray, CBC. The xray was abnormal so she ordered sputum AFB and culture which turned out negative and CBC was also normal. And as hard headed as my husband is he did not go back for follow up. A week ago he said he cough and went to a Pulmonologist and did another CXR which showed granulomatous lessions suggesting Koch's. The CXR from 2 months ago hasn't changed. Do we have to do another sputum culture? By the way, he is a smoker of 10 sticks a day since he was a teenager. Doctor: Hi, welcome to Healthcaremagic. Koch's or tuberculosis is a disease which may affect any of the organ of body but most commonly, it affects lungs. The patient may present with productive cough with sputum and low grade fever in evening and lethargy with weight loss. Sometimes, sputum culture is negative because tuberculosis bacteria is fastidious organsism and requires time for positivity on culture media. I would suggest that please start the antituberculosis drugs and do a gene xpert TB PCR test for confirmation.Ask him to quit or reduce smoking."
},
{
"id": 202875,
"tgt": "Pain in penis tip, abdominal pain, frequent urination, nausea",
"src": "Patient: I feel uncomfortable at my penis tip and feels like paining in the abdominal area. Also i am frequently urinating. Today i drink around 3 liters or water an hour before i vomited. Doctor: Hi and .You are probably suffering from urinary tract infection. Follow these instructions1. Take Tab Zenflox UTI 1-0-1 after food for 5 days2. Drink Cranberry juice3. Drink lot of fluids4. Take cap Omez D 20 mg one before breakfast for 10 days5. Avoid spicy foodIf above steps did not help then please contact a local doctor for help.All the bestDr Santosh"
},
{
"id": 81380,
"tgt": "What causes heartburn and chest pains?",
"src": "Patient: i have been having heartburn frequently sharp chest pains and like someone is sitting on my chest and i have a lump under my chin it hurts but not like having a sore throat or strep its a burning sensation in my throat kindda do i have reason to be worried? Doctor: Thanks for your question on HCM.In my opinion you are having GERD (gastroesophageal reflux disease) mostly.But better to rule out cardiac cause first by ECG.If ECG is normal than you are having GERD mostly.It is due to laxity of lower esophageal sphincter. Because of this the acid of the stomach tends to come up in the esophagus and cause heart burn.Try to follow these steps for better symptomatic relief.1. Avoid hot and spicy food 2. Avoid large meals, instead take frequent small meals.3. Avoid stress and anxiety.4. Take proton pump inhibitor.5. Go for walk after meals .6. Keep 2-3 pillows under head in bed to prevent reflux."
},
{
"id": 43621,
"tgt": "Done sperm analysis. Will I be a father?",
"src": "Patient: hello sir dr. i m 26 year old and i got married 4 month ago and still i cant be a hopefull for a father my volume is 4.0 ml, reaction is alkaline, liquification time is 30 min sperm count is 65 million/ml sperm motility is excelent 35, good is 10, poor is 05, non-motile 50, morphology head piece defect 30%, mid piece defect 05%, tail defect 20% , pus cells 2-3 /hpf, red blood cells nill, epithelial cells nill, Doctor: Hello, Welcome to HCM, I am Dr. Das Look, your semen analysis report indicates normal sperm count (normal is > 15 ml as per WHO 2010). The motility is within normal limit (as per WHO 2010, grade a should be >25% or all total >50%). The Normal sperm count is also within normal limit (as per WHO 2010, normal sperm should be > 04%). The pus cells are present (as per WHO 2010, normal semen should contain no pus cells). So, overall, your report indicates possibility of genitourinary infection. For which you need a broad spectrum antibiotic. Regards."
},
{
"id": 7373,
"tgt": "48 hs after taking Levonelle 1500 pill am I still infertile ?",
"src": "Patient: Took Levonelle 1500 on friday morning and have had unprotected sex on sunday morning (48 hs after the pill ) . 48 hs after taking the morning after pill am I still infertile? Doctor: Pill should be taken within 72 hrs of unprotected intercourse not vice versa."
},
{
"id": 156385,
"tgt": "Will taking \" Imitinef Mercilet\" be safe for cancer?",
"src": "Patient: Hello Doctor, One of my friend's son is having blood cancer, but unfortunately was not able to identify which type of it. I would like to know whether the drug named, \"Imitinef Mercilet\" will be useful in curing blood cancer. I am asking this because, I got a mail which says that the above said medicine will cure blood cancer and it is available free of cost in \"Adayar Cancer Institute\", Chennai, India.Thank youNinzo Thomas, Oman Doctor: Very sad to hear about your friend's son. You have not mentioned the type of blood cancer which the patient suffers from. Imitinef Mercilet is Imatinib Mesylate and is used in the treatment of the following blood cancers:1. Chronic Myeloid Leukemia CML with philidelphia chromosome positive2. Myelodysplastic / Myeloproliferative with PDGFR gene re-arrangement3. Relapsed or refractory Acute Lymphoblastic Leukemia with philadelphia chromosome positiveAmongst the above mentioned blood cancers, best response is observed in CML. Its sends the disease in remission in CML but does not cure it. Only option for cure in CML is bone marrow transplant. I hope your query has been answered. If you repeat your question mentioning the blood cancer type, I would be able to advise you better."
},
{
"id": 169016,
"tgt": "Suggest treatment for sepsis in an infant",
"src": "Patient: Hi my 11 days baby suffering from sepsis from 1st day at present tlc is under control I.e. 8000 but crp level still fluctuating and at present it is 79. Dr using vanco n one more antibiotic from 3rd day as he told using in combination. Can u please suggest anything he may miss? Doctor: Sepsis at this age is treated with pen/genta and not vanco or combination of 3rd gen cefo....I suggest you take the baby to a best peds center for proper management."
},
{
"id": 29401,
"tgt": "Can Meningococcal vaccine of 1ml lead to an over-dosage in a teenager?",
"src": "Patient: My son is 14 and half year old and I m not sure about his vaccine meningococcal. Can we give now, if it is extra dose - any harm?. I also want to go for HAV but doctor advised 1ml dose where I checked on site the recommended dose up to 18yrs is 0.5ml? Pl advice both my queries. Doctor: Hi, I understand your concern. Meningococcal vaccine is administered to children of 11 to 12 year old. A booster dose is recommended at the age 16 year old. Your son is 14 year old. Although you do not remember if administered the first dose, I advise to wait until he gets 16 year old to get the booster. Another booster is recommended in youngsters of 16 to 23 year old. Hope this answers your query."
},
{
"id": 27674,
"tgt": "Are BP being 159/73, heartbeat 86 with irregular heartbeat while being on lisinipril concerning?",
"src": "Patient: I took my blood pressure and it was 159/73 with 86 heart beat. It also showed that I had an irregular heart beat. Should be doing something right away? What would be causing this? I take 5 mg Lisinipril but it has been giving me pain in my left leg, especially when I am in bed and change positions, so I have not been taking it as I never had the pain until I started taking the lisinipri. I do have a Dr. appt. on the 3rd of April. Should I contact him sooner? Thank you Doctor: Hello, A late answer I suppose...Your blood pressure seems to be uncontrolled. The best way to help your doctor in similar situations is to measure your blood pressure every morning and afternoon for a week or so and record the measurements. Your doctor will find your records very helpful. The irregular rhythm needs immediate clarification though. It could be just a few innocent systolic but I could also be atrial fibrillation or other types of significant arrhythmias. I mentioned atrial fibrillation because it is fairly common in individuals with uncontrolled blood pressure.You should discuss your problems with your doctor regarding lisinopril side effects and perhaps choose an alternative regimen.I hope I've helped! You can contact me again if you'd like any clarification or further information. Kind regards!"
},
{
"id": 196946,
"tgt": "What causes scrotum infection?",
"src": "Patient: My husband found out a node inside his scrotum, it brings little pain and is separated from the scrotum wall. He is convinced that it is only a little infection caused by the hairs on the scrotum area. I am afraid is something else. Could you please clarify my ideaBest regards Anna Doctor: HelloThanks for query .A lump (Node) with little pin in scrotal sac could be either a a staphylococcal infection of a hair follicle (Boil) or a Sebaceous cyst .However the precise diagnosis can be made only on clinical examination by a qualified General surgeon . The scrotal skin is rich in sebaceous glands and hence prone to get sebaceous cyst due to accumulation of sebum beneath the skin. Normally it fades away without treatment however need to be treated if increase in size or get infected. Take antibiotic like Doxicycline and Diclofenac twice daily for a week .If persist even after taking antibiotics consult General Surgeon for evaluation .It may need to get excised under local anesthesia.Dr.Patil."
},
{
"id": 54222,
"tgt": "Can i continue with the regular diet or follow food restrictions for mild jaundice?",
"src": "Patient: I had a mild jaundice when i was infected by malaria... my bile count was 2.2 mg/dl... after a month i did a blood test now and found bile segments to be absent.. Now can i continue with my regular diet ?.. or am i supposed to follow food restrictions for a month more ? Doctor: Hi thanks for contacting HCM....According to history you had jaundice ...In jaundice rest given to liver ....So excess fatty and fried food avoided....Now you have normal report ...So you can assume full normal diet...But assume it gradually....Junk food and fried non veg still kept limited for few days....With your normal routine diet also take fruits daily for maintaining healthy life .Hope your concern solved.Take care"
},
{
"id": 127529,
"tgt": "Is Gemfos safe for the treatment of osteoporosis?",
"src": "Patient: My question is that, My mother is an osteoporosis patient. As per doctor s advice she takes orthotrue capsules twice a daily and also take Alensol D every sunday. She is now taken Gemfoss 35 (advice one of my relative). is it safe for her? Rupesh Chatterjee Kolkata Doctor: Hello and Welcome to \u2018Ask A Doctor\u2019 service. I have reviewed your query and here is my advice. Dear Gemfos may cause below side effects. Gastrointestinal disturbances Electrolyte disturbances Musculoskeletal pain Headache Hypersensitivity reactions Blood disorders Joint or muscle pain Hope I have answered your query. Let me know if I can assist you further."
},
{
"id": 56371,
"tgt": "Suggest remedy for liver ailments",
"src": "Patient: Hi, sir good evening, 3 days back US Abdomen Whole reports came, In reports all are normal but Liver: 16 CM-Enlarged in size echogenecity NO IHBD/SOL, Report end given Impression: Hepatomegaly with fatty changes, what about the liver position healthy or not? please give me answer. Doctor: HelloYour findings suggests mildly enlarged liver with fatty changes.It may be due to many reasons like hepatitis,alcohol intake,obesity,altered lipid profile,medicines,auto immune causes etc.Fatty liver is reversible condition.Obesity and sedentary life style are its common causes,so these factors should be ruled out.You may need to change your diet and life style.You should a go for brisk walk regularly and avoid fatty food.You may need investigations like routine hemogram,random blood sugar,liver function test,lipid profile.Treatment depend upon findings.Get well soon.Take CareDr.Indu Bhushan"
},
{
"id": 151314,
"tgt": "Spinal bruises, seizures, excessive sleepiness. Normal ECGs. Treatment?",
"src": "Patient: My granbaby has bruises on top of his spine . he is also sleeping more than he should. He is 9 months old. When he was 7 months old, he started to have seizures. He has had 4 since that time. No one is able to tell us what it is. The seizures came first and they told us in the hospital that he will grow out of them- eventually. They have taken 2 ECGs but nothing has shown up. Now they are doing blood work- but we have no results yet. Doctor: Hi welcome to Health care magic forum. The sizures started in the 1st week it self means that there might be head injury or there might be congenital problem of brain. I advise you to consult a neurologist for diagnosis and treat ment. other possibility is febrile convulsions. for this he must have fever every time he gets the sizures. You may have to take the MMR EEG along with other tests for confermation of the problem. Wishing for a quick recovery and thanks for calling."
},
{
"id": 8647,
"tgt": "Original skin tone is fair due to sunlight suffering from skin tan. Suggest best cosmetic product?",
"src": "Patient: Hi, i am a teenager boy with dark complexion. I applied lot of natural things and cream for fairness but there is no effect. My original colour is fair but in childhood at the age of 10 to 12 year i don't care of my skin and play in sunlight and not use any cream. Today everyone love fair skin and i also want fair skin . Any one qualified doctor please suggest me some cream at affortable price and some step for fair skin , please suggest cream for teenager level.... Doctor: If your natural skin type is fairer and you have got darker due to tanning then you can follow the following instuctions for getting your natural skin colour. 1.Try to avoid going out in direct sunlight from 11:00am to 3:00pm.Use umberella,large brim hat and full sleeved cotton shirts when possible. 2.Invest in Neutrogena sunscreen with spf 50 and always apply it 1/2 hr before going to sunlight.reapply every 2 hrs for complete benefits. 3.Use Ban a tan cream or Niltan cream at night before going to sleep. Apply the cream on the face at night after washing the face with cold water and mild facewash. 4.you can also consider getting 3 sittings of glycolic acid peel from dermatologist after your skin is primed. Hope it helps"
},
{
"id": 211740,
"tgt": "Diagnosed acute psychosis, drowsiness. On chlopramzine, parkin. Can I switch medication?",
"src": "Patient: I have a 3 year old daughter who has just woke up soaked in sweat, her hair and clothes were both saturated. I do not feel that her room is overly warm. She was sleepy and a bit out of it but she did sit up and cooperate with me as I changed her wet clothing and I did coax her to drink some water. Her temperature in bothears was 97.8 degrees. Should I be of concern? This is unusual for her. She was itching I noticed also (could just have been a bug bite.) She woke up crying by the way, this is what brought us to her room. Any suggestions would be helpful. Doctor: Hi, the reason for such ahuge sweating might be due to vomiting sensation, stomach pain, utricaria, or insect bite. The reason for the cause is to be traced out. So i advise you to consult a pediatritian for diagnosis and treatment. Thank you."
},
{
"id": 100774,
"tgt": "How to cure chronic asthma?",
"src": "Patient: I am a 55 year old female with chronic asthma..I've had asthma all my life but the last 6 years its gotten worse..My question is I've been on prednisone since 2005 and I can't seem to get off it..I am on 5 to 10 mg. per day..is there any advice to help me? Doctor: researchers are working on genetical therapy in biotechnolgy and breathrough for asthma may be coming soon, just take adequate care to avoid known environment precipitating your attacks, and other precautions like strenuous physicalefforts,dust,pollens etc.then it may be possible to reduce dose of medications"
},
{
"id": 2499,
"tgt": "When should i plan for a baby if i have irregular periods?",
"src": "Patient: hello doctor,I am 28 years old, i got marrried 5 months back. Recently i have been diagnosed with tb and also my periods are not regular, it come only for 1 day with very less bleeding. Can you pls guide me how fertile am I and when should we plan a baby?Regards - Parminder Doctor: Hello Parminder,Since you have been diagnosed with TB , first job is to complete the Anti-TB drugs ( ATT). If you have been diagnosed with TB in lungs, damage to uterus is not very significant. But can't always predict. Extrapulmonary TB - can damage Uterus. Your reduced flow may be due to the stress of heavy infection & may be normal once cleared. TB in uterus usually affects inner lining, reducing the thickness, blood supply. Ovaries are spared.As soon as ATT is over, meet an infertility specialist & plan a pregnancy.If the inner lining of uterus is damaged, it is irreversible. You can get pregnant only by IVF- SurrogateAll the best"
},
{
"id": 168752,
"tgt": "What causes bumps with white heads on abdomen of a child?",
"src": "Patient: Dear Doctor, my daughter with 34 monthes and her weight is 14.5 kg and height is 95 cm ten days ago we found pimply with white heads on her tummy around 7 nos I put Antipruritic cream for her and next day we went to the doctor and he said it might be chicken box however she had got her vaccination and after 2 days disapeared without any medecine today new one apear on her tummy again and there is no fever please advice what is this? is it stomach proplem ? Doctor: Hi,It seems that she might be having chicken pox infection.This can be mild form as she has taken vaccination.This can be better judged seeing clinical examination by the doctor.Give Benadryl syrup if itching is there.Give her paracetamol syrup for pain.Withing few days she will be alright.Ok and take care."
},
{
"id": 109188,
"tgt": "How to treat chronic pain in the ulnar nerve post accident?",
"src": "Patient: Hi..I have chronic pain .. back, shoulder, ulnar nerve, right leg, left knee occasionally..I was a pedestrian and was hit by a car..I have been on pain medication which helps me to keep moving..my doctor has now accused me of being more addicted to the pills and treats me very shabbily when I see him..to the point that I get terrible anxiety when I have to go and see him..he won t send me to any specialists because he said they can t help..I am so frustrated and then I find that the pain elevates when I worry or become agitated..I really need to see someone as a lot of the pain has been determined to be coming from my spine but I know that my elbow really hit the ground hard when I was thrown and have read about ulnar release surgery..my doctor is uninterested and it has been 2 1/2 years..my life is getting smaller and smaller and I have become withdrawn..not sure where to turn for help..thank you Doctor: dear friend don't worry,you should take multivitamines with combination of methylcobalamine once daily. take your pain medicine only when needed or severe pain.in cases of nerve injury from accidents some homeopathic medicines have powerful action without side effects and it will help you the nerve to heal fast.if you interested in homeopathy,please take following homeopathic medicines for one month and then imform me about your pain.-arnica 6, 5-7 drops in a cup of water twice/thrice per day.-hypericum 30, 5-7 pills twice/thrice per day.hope you get your answer.for more queries plz don't hassitate to ask.get well soon dear frienddr.manu khimani"
},
{
"id": 26080,
"tgt": "What causes shortness of breath and suggest treatment?",
"src": "Patient: I feel short of breath sometimes. Even if im just laying down. it goes and comnes. When I do heavy lifting is when I feel it the most. I am over weight and i do have sleep apnea. I was thinking i might have a fatty heart. If so Is there a cure for it? Doctor: HelloShortness of breath and exertional breathlessness may be due to cardiopulmonary causes.You may need proper clinical examination and investigations.Investigations include routine hemogram,random blood sugar,lipid profile,liver and renal function test,thyroid profile,ultrasound of abdomen,chest X-ray(PA view),ECG in all leads,ECHO,TMT,PFT etc.Further treatment requirement depend upon these basic investigations.Treatment depend upon findings.Get well soon.Take CareDr.Indu Bhushan"
},
{
"id": 222246,
"tgt": "Suggest remedy for low progesterone levels",
"src": "Patient: Hi, we are trying for a baby for about 3 years and 6 months, my periods used to be varying between 30 and 35 days normally, only once (2 years before) it had been 40 day cycle ,..last cycle I had spotting on day 22, 23 and 25. I'm visiting my gynoe and have very recently started investiation, they took blood test on day 25 (as my cycle is varying) for progesterone and the results say very very low progesteron level (I think the results said 3 , I didn't note the measurement units and also it said the normal should be somewhere around 30). I haven't got my periods, I'm on day 43. Negative pregnancy tests on day 35 and 38. Is there anything very worst happening?? Doctor: Hello dear,I understand your concern.In my opinion the most common cause of low progesterone might be polycystic ovarian disease.The causes of irregular cycles like thyroid abnormalities ,overweight,obesity,stressetc.So all the above things should be ruled out by doing thyroid profile,ultrasound.If overweight or obesity consider weight reduction.Even 10% of weight reduction itself make cycles regular.Avoid stress as it causes hormonal imbalance and irregular cycles.So relax.Wait for spontaneous onset of period if still no period consult gynaecologist for periods or withdrawal bleeding.Nothing to worry.It will become regular if cause is treated.As the pregnancy test is negative pregnancy is unlikely.Best regards..."
},
{
"id": 75448,
"tgt": "Suggest treatment for bronchitis",
"src": "Patient: Hi,My daughter has had a cough for a few weeks now. She has been to a dr a couple of times and given antibiotics. She woke up yesterday with red spots (some raised, some not) covering her entire body. I took her to the ER and they said she was misdiagnosed and has bronchitis and an infected bronchile tube which had brought out the bronchitis causing the red spotsThey prescribed her medicine and today when she woke up, the spots were worse and are now itchy whereas yesterday they were not itchy. Is this normal? Doctor: Hello dear, Thank you for your question. In my opinion, the nature of the red spots may suggest an allergic reaction from the antibiotics. In order to create the right idea, I would like to ask you a few questions:-What type of antibiotic has your daughter taken? -Did the ER doctors prescribe a different antibiotic or the same one?-Can you describe the shape, dimensions and location of the red spots? Do they disappear if you press them shortly?For the moment I would suggest to consider discontinuing the antibiotics and consult a doctor.I hope this helped.Best wishes,Dr. Alma"
},
{
"id": 40436,
"tgt": "What is the treatment for rashes caused due to shingles?",
"src": "Patient: I am 56 years old. I have shingles for the second time in 6 months. This time the rash is on my arm near my elbow, about the size of an egg in size. The rash has lasted now for almost 3 months. It got better after about one month, but now has just stayed the same with no further improvement. Should I be concerned that it is not going away? Doctor: Hello,Welcome to HCM.Varicella zooster or shingles is a viral infection caused by human herpes virus III. The lesions can present in any part of the body and distributed along the affected nerve dermatome.The virus remains as dormant in the nerve endings for very long period, when ever the body immunity comes down it will flare up produces the rash on the particular nerve distribution.This is a self limiting disease and it will reduce by itself.I would suggest you to Apply cold packs on the lesion which will reduces the hives and pain,You can apply antiviral and steroid cream on the lesion.Oral antiviral drugsoral antihistamines.Thank you."
},
{
"id": 128914,
"tgt": "Suggest treatment for pain in the feet",
"src": "Patient: i am 53 years old. I am overweight. I have been dieting for months. I have diabetes. I know I should exercise, but my feet kill me. I wear good shoes. over the past several years, i have become very weak. i have difficulty walking any distance, or upstairs. My feet hurt all the time. They ache so bad and will have sharp pains in them. I do not think it has to do with the diabetes. Sometimes they will even feel like a bone is broke in them. I feel bad most of the time and can not get anyone to listen to me. They give me b 12 vitamin. I have sleep apenea, and have been on a cpap machine for years, I have never felt like it helps at all, other than controls my snoring. I supposedly have narcolepsy also, and am on medication for that too. I think if it was not for that I could sleep 24 hours a day. I am so tired all the time. But my feet are my main problem seems like. I know i need to exercise, but My feet hurt so bad I don t want to walk. i still work and by the end of the day, i just want to get off of my feet. Doctor: Hello, Thank you for using healthcaremagic.I read your question and understood your concern.You may have plantar fascitis so I recomend you to use custom made shoe inserts with arch supports. Also check your osteoporosis as you may need calcium and vitamin D medication. Diabetes affects also the nerves of the legs so you take medication for neuropathic pain such as LYRICA.Non weight bearing exercise and sport such as swimming , water aeroby and stationary bike is good for you to loose weight but not walking nor jogging.Dr. Selmani"
},
{
"id": 210159,
"tgt": "will one month s treatment be enough for my dad s temper flares?",
"src": "Patient: My dad (82) showed sudden temper flares, mood swings and aggressive behaviour. Psychiatrist prescribed oleanz rapitab 2.5 initially once a day. That did not change his behaviour. After 4 days he was taken to the doctor again. Dosage was increased to 5mg twice daily. Now after 5 more days of increased dosage, he is his usual calm and quiet self. How long should he take the medication? Will one month s treatment be enough? He doesn t have diabetes or hypertension. Will taking this drug increase the risk of diabetes or hypertension? Doctor: Hi,I read your query. Before I go on to answer, I would like to mention that I would have wanted to know if your father is suffering from any forgetfulness too. That would be important because in that case it would be important to treat him accordingly with some anti-dementia medication. Currently, it would be difficult to say how long he would need olanzapine. It would be required to review him periodically and take a decision accordingly. Olanzapine doesn't increase the risk of hypertension although long term intake can lead to dyslipidemia and increased risk of diabetes. However, regular monitoring will help in taking care of that. Hope this information was helpful. Best wishes."
},
{
"id": 144017,
"tgt": "Suggest treatment for disc bulge, thecal sac and neural foramina",
"src": "Patient: Dear doctor. Presently I am suffering from Lumber-Spine related problems causing difficulty in seating/walking for more than15 minutes. MRI reports indicated disc bulge, thecal sac,neural formina causing mild to moderate compression over L4&L5 existing nerve root. The concerned spine surgeon has advised medication only I.e Shelcal OS & Neuroday(methyl cobalt mine)daily with Physiotherapy/exercise.What are the other Effective Remedial measures in my case.? Doctor: Hi, I am Dr.Bruno. I have read your question with care and understand your concerns. Let me try to help you Question : What are the other Effective Remedial measures in my case.?Answer : For your condition, i.e. \"disc bulge, thecal sac,neural formina causing mild to moderate compression over L4&L5 existing nerve root\" the initial treatment is Nutrients + Physiotherapy / exercise. You have to start with this. Most of the patients get relief with these measures Only if there is no relief with the above measures, you have to take tablets to mediate pain. If the pain still persists, or if you develop and numbness or weakness, you may require surgery . . .But don't worry . . . Only a minuscule percentage of people with Spine Problem actually require surgery. Most of them get relief with tablets and physiotherapy itself Hope you found the answer helpful.If you need any clarification / have doubts / have additional questions / have follow up questions, then please do not hesitate in asking again. I will be happy to answer your questions.Let me know if I can assist you further.Take care."
},
{
"id": 173037,
"tgt": "What cause itchy rashes all over the body to year old baby?",
"src": "Patient: my daughter is a year and half has a rash all over her body some of the rash have white heads on them also there are blisters between her toes and on her feet and pimple like bumps on her body in different places, The rash makes her itch alot its even in her hair any help to ease her pain will help Doctor: Hi dear,I understand your concern. You should be worried,but not too much. It seems your baby has child's infection-Chicken pox.With such presentation in my clinic, I usually prescribe antihistamine medicine-cetrizine syrup, you should apply ciprit to affected area to kill infection and decrease itching. Don't take tub during 5 days,only wash intimate areas.Wash her mouth with soda water. Rash will stop after 5 days.Hope I answered to your question. If you have more questions then don't hesitate to write to us. I always happy to help.Wishing your baby good health"
},
{
"id": 81536,
"tgt": "What does reticular densities in upper lung in chest x-ray suggest?",
"src": "Patient: Chest x ray PA View: reticular densities are seen in the right upper lung. The heart is not enlarged. The Thoracic aorta is not remarkable. The trachea is at the midline. Both hemidiaphragms and costopheric sulci are intact. The bony thorax is unremarkable. Conclusion: consider an inflammatory process as in koch s infection in the right upper lung. please correlate with other parameters. Doctor: Thanks for your question on HCM.Reticular densities in upper lobe suggest infective process mostly pneumonia.And upper lobe lesion is most commonly seen in tuberculosis.Tuberculosis is caused by mycobacterium tuberculosis bacteria. And it has great effinity for oxygen. Upper lobes being more oxygenated, favours more infection with TB bacilli. So upper lobe lesion should be rule out for tuberculosis first.So get done sputum examination for AFB to rule out tuberculosis."
},
{
"id": 55946,
"tgt": "Suffering from Hep C. Can a doctor prescribe me a low dose Adderall?",
"src": "Patient: Hi, I have Hep C for 10 yrs. Now I am starting to feel symptom of the hep C. The symptoms include chronic weakness extreme tiredness. Some days I feel I cant even get out of bed. my question is, do you think a Dr. can prescribe me a low dose of Adderall? Doctor: Welcome at HCM I have gone through your query and being your doctor I completely understand your health concerns.... What is your age? What is your serum albuminl levels and bilirubin? For how long you have been suffering from this ? Have you done any treatment?? I'd there any associated co morbidity??? In my opinion you should not take adderal and it is also not recommended in chronic liver disease as it increases the risk of development of hepatic encephalopathy ( hepatic coma ). You should consult hepatoligist and start treatment for hepatitis c. Once it is cured everything would be settled Get well soon Regards Dr Saad Sultan"
},
{
"id": 97089,
"tgt": "Is it safe to fly 5 hours after head injury?",
"src": "Patient: My son hit his head yesterday (Monday) and knocked him self out for a second. He is experiencing some dizzyness, headache and fatigue today. We are waiting on his Doctor to respond if he needs to be seen. My question is: is it safe to fly 5 hours by plane to Hawaii if in fact he has a concussion? We had planned to leave on Friday or Sunday. Doctor: Hello dear,.Dizziness and head ache following head injury,especially with history of loss of consciousness needs evaluation for internal bleed or other pathologies before flying.Kindly see your doctor without delay.You may be asked for CT/MRI scanning of head.Hope you will find this answer useful.Wishing for good health,Thank you."
},
{
"id": 219930,
"tgt": "Can darker nipples,soreness and abdominal pain be symptom of pregnancy?",
"src": "Patient: had my period in the begining of this month, had sex on 6th day, and now my nipples are darker in color and a bit sore and sensitive, i have pains in my lower abdonamol i think thats what it is called, sort of like cramps, and my mood swings have been very out of wack latley, im very hot at times, very sleepy. Could i be pregnant this month? last october i got DNC due to unhealth baby. Doctor: Hello dear,I understand your concern.The 6 th day of the period comes under safe period.Safe period is the one where in there are no chances of pregnancy even with unprotected intercourse.So the sore breasts,cramps ,mood variations might not be due to pregnancy.But if the period was abnormal from your usual period like spotting or scanty bleeding then there might be suspicion of pregnancy.In that case I suggest you to do a urine pregnancy test to confirm.But if the bleeding was normal then there is no suspicion or chance for pregnancy.Hope this helps.Best regards...."
},
{
"id": 98010,
"tgt": "How to get rid of balgam?",
"src": "Patient: How to get rid of balgam collected in nervse near ear? thank u Doctor: 1. this is probably excess wax/phlegm/kapha in the external ear which if in excess may cause ear congestion, reduced hearing and stuffiness. 2. Increase your fluid intake (warm water with lemon juice and honey) and avoid cold and constipating food (dairy products,sugar,wheat 3. take diet rich in sulphur (garlic,onion) which helps in relieving congestion and increase intake of herbal tea (Lime,ginger,Fenugreek) 4. Ayurveda 'Jal neti' is effective in such conditions. 5. Vitamin C rich diet is beneficial to increase the immunity."
},
{
"id": 192958,
"tgt": "What could cause on and off pain in left side of groin?",
"src": "Patient: Hi, may I answer your health queries right now ? Please type your query here.. my left groin is in side sowhere but some times its feel come up & go down that time I feel pain its happen like 40 years but now more painfuu offen so it is better to takeit out?. Doctor: Hello, You need to see your doctor to get checked and to rule out any testicular infection as well. Pain could also be due to torsion of testis as well. Hope I have answered your query. Let me know if I can assist you further. Take care Regards, Dr SAMEEN BIN NAEEM, General & Family Physician"
},
{
"id": 39634,
"tgt": "Suggest the cause and medications for bumps around the eyes",
"src": "Patient: I have a small yellow bump where my bottom eyelashes are. It's not on my eye, it's on the top of my skin and it hurts when I blink; it kind of looks like a small ball of puss. I don't want to pop it because I'm pretty sure it isn't a pimple. What do you think it is; what should I do? Doctor: Hi,Welcome to HCM!I believe you are suffering from infection of eye lids or related glands, commonly called a 'stye'. You'll need to take antibiotic eye drops in order for it to heal. If too much pus accumulates, it might even need drainage by incising. Kindly consult an ophthalmologist and start with antibiotic eye drops.I hope your query is solved. Feel free to ask if you have any doubts. Regards,Dr. Sridhar"
},
{
"id": 167440,
"tgt": "Can i use clotrimazole cream for my 4 year old child?",
"src": "Patient: Hi,my daughter ha swoken up crying saying that her private bits are hurting her,shes scratching and seems in discomfort.Shes currently lying down witha cold flanel between her legs but its not soothing her.I have got clotrimazole cream here that I got prescribed,is it ok to apply a little of that?shes nearly 4? Doctor: Hello. I just read through your question.It is perfectly safe to use the clotromazole. If you do not notice any improvement, I recommend consulting with your doctor."
},
{
"id": 55837,
"tgt": "How to treat cirrhosis of liver?",
"src": "Patient: My mother has cerosis of the liver, but when blood work was done, ca 125 , showed up in her blood results. My mother has had a partial hystro, leaving her ovaries, some 39 years ago. They have checked the ovaries and they show to be fine. Besides ovarian ca, what else is this enzyme associated with? Doctor: Hi,Raised CA-125 levels suggest ovarian malignancy but if ovaries are fine & partial hysterectomy is done, then the other possibility of raised CA-125 could be cirrhosis of liver, although its rare & non-specific for liver.I hope, my answer will help you.Regards,Dr. Maria Daud"
},
{
"id": 179480,
"tgt": "What is the treatment for persistent cough and fever in a child?",
"src": "Patient: Hi my son is 3 yrs and 2 mnths old...weight is 14.6kg and height is 100 cm...he was having cough since the last 2 days and the dr prescribed him levolin nebulalisation .063 mg ....after giving it twice he has started having fever...I hope that this is not a side effect and also how many time can I neubulise him in a day and at wht intervals Doctor: Hi...this fever is not a side effect of Levolin Nebulization. But it is not safe to give nebulization with a home nebulizer without an oxygen source like wall mount oxygen supply or oxygen cylinder. I suggest you shift over to metered dose inhaler rather than a nebulizer. If you still prefer to use a b=nebulizer you can give the same dose you mentioned every 4th hourly or 6th hourly based on the kid's wheezing.Regards - Dr. Sumanth"
},
{
"id": 53027,
"tgt": "What is the prognosis in a person with extremely elevated liver enzymes?",
"src": "Patient: Hi. My friend's daughter was just released from ICU. My friend said that the doctor said her liver function was @ 30% only. She is only 30 and according to my friend she did not have a chance to live and no chance of liver transplant. Her liver enzymes were extremely elevated, although I don\"t know the exact figures. The patient is extremely thin and she is out of ICU but according to my friend the doctor has warned them of irreversible neuronal damage and other problems. What are some of the scenarios in which a young otherwise healthy female would suddenly end up in ICU with these symptoms and the possibility of death? Doctor: It could be following Hepatitis infection--most likely, since your friend does not seem to have swelling which could be a sign of cirrhosis."
},
{
"id": 87066,
"tgt": "What could pinching abdomen, with a history of abdominoplasty suggest?",
"src": "Patient: I had abdominoplasty in 2011 and every since i ve always felt a pinching in my abdomin near my belly button. Now its progressed to extreme pain in which i feel every day. Sometimes it last for a few seconds and sometimes it last for minutes. I ve seen my plastic surgeon on several occasion. He gave me an injection yesterday and the pain is still coming. What can this be? Doctor: Hi.Thanks for your query. Abdominoplasty is an extensive surgery and it is but natural to cut many nerves. Some of the nerves on regeneration can give you any sort of weird symptoms, pains and pins sensation. You have got a Sensory Nerve problem and the nest way is to try the following medicines under the care of the Doctor as they are prescription medicines. Amitryptilin, Gabapentin or tegritol. these will definitely help you,; if not then it will be wise to have re-exploration and cut off the nerves causing such a severe problems."
},
{
"id": 176339,
"tgt": "Suggest treatment for fever in a child",
"src": "Patient: Hello doctor, Previously when my 2 year old son gets fits during fever at 104 F doctor treated with parasitamole alongwith fricium .now he is 4 years and getting fever of 104 F but not gets fits .should i give him fricium with parasitamole ? Should it safe to give previously ? Doctor: Hello,Welcome to the HCMBrief...You do not need to give him frisium (clobazam)Detailed answer...Since febrile fits are common in children ,and there is difference in epilepsy disease and fits because of only fever.Incidence of recurrent febrile fits is 20 to 30 percent ,and if he does not gets fits after that episode,then that means neither he is having epilepsy nor febrile fits.For fever you may give him paracetamol,and if fever persist,get checked by a pedeatrition ,he may need antibiotic course.Regards,Dr.Maheshwari"
},
{
"id": 111400,
"tgt": "How to treat back pain?",
"src": "Patient: i had an L3-4 disectomy and decompression in july and have been ok, but recently i have had pain in my upper back just below my shoulder blades and it is making it difficult to breathe properly, was just wondering if you could give me some advice because its really getting me down again Doctor: This sounds like muscle spasms of the upper back muscles. Topical ice, anti-inflammatory medications (advil/alleve), or massage could help greatly. Sometimes muscle spasms from one region (low back) can communicate with the muscles of a neighboring region (upper back), so this is common and not likely worrisome. If these measures don't work, see your doctor and they can prescribe physical therapy/chiropractic."
},
{
"id": 62194,
"tgt": "How to cure lumps on groin area?",
"src": "Patient: i Keep getting large lumps under my skin in the groin and inner thigh area, they are about the size of a 10 pence piece in diameter, sometimes larger. They tend to go very hard and round, they are sensative to touch, they will wither disperse in my skin or work up tot he surface and burst - they have blood and puss in them, these are very painful. My doctor said when i went on the pill this would halp and it did for the 1st year but they have come back and i still have no idea what is causing this or how to treat? Doctor: Hi, dearI have gone through your question. I can understand your concern. You may have enlarged inguinal lymphnode. It can be due to reactive hyperplasia, tuberculosis or lymphoma. You should go for fine needle aspiration cytology or biopsy of that lump. It will give you exact diagnosis. Then you should take treatment accordingly. Hope I have answered your question, if you have doubt then I will be happy to answer. Thanks for using health care magic. Wish you a very good health."
},
{
"id": 159984,
"tgt": "Liver Cancer with Hepatoma",
"src": "Patient: My mom aged 64yrs. Doctor said she got liver cancer due to large liver tumor in the right lobe of liver associated with small porta hepatis lymph node and 2 small lung nodules. Hepatoma with early lung metasis is suspected. But for her full blood test showed that Alpha fetoprotein is 24.5, AST is 88 and ALT is 88. For Hepatitis A , Hepatitis B and Hepatits C is negative. Currently my mom referring to liver specialist for further investigation. But I need your comment please. thanks . Doctor: dear nini, it is possible to have liver cancer with the values mentioned, the final diagnosis will be based on tissue examination pathologically and IHC."
},
{
"id": 195157,
"tgt": "How can bumps on the penis and testicles be treated?",
"src": "Patient: Hello. I have pimples on the skin of penis ,two on ball sack and one on the head.they are not painful...I never had sex, I masturbate sometimes.sometimes itching which won t last than a minute .after a week it looks like a small worm on skin...please can u tell any solution? Doctor: HHi, Bumps on penis may be due to rash or folliculitis or acne or sebecious cysts or eczema, etc. Until examination is done it is difficult to say what it is. Avoid tight clothing, use moisturizer cream, avoid soaps and deodorant that cause allergy. Use tablet Cetrizine twice a day for five days. If symptoms not improved please consult your dermatologist he will examine and treat you accordingly. Hope I have answered your query. Let me know if I can assist you further. Regards,\u00a0\u00a0\u00a0\u00a0\u00a0 Dr. Penchila Prasad Kandikattu"
},
{
"id": 89074,
"tgt": "What causes severe cramping and vomiting?",
"src": "Patient: how is it that at being 24 years old now, have been experiencing vicious cramping, vomitting from cramping, and have had to leave work and school because i cannot move with out pain. MY obgyn changed up my birth control but as far as i can tell i feel worse. HELP Doctor: Hi! Good evening. I am Dr Shareef answering your query.All the precautions including birth control pills are not 100% fool proof in protection against a pregnancy. If I were your doctor, I would go for a serum HCG determination followed by an ultrasound abdomen to rule this out. Only after ruling this out, I would further investigate you to rule out any other pathological process in the pelvic adnexa or in the appendix. Till that, you could go for an antispasmodic drug for a symptomatic relief.I hope this information would help you in discussing with your family physician/treating doctor in further management of your problem. Please do not hesitate to ask in case of any further doubts.Thanks for choosing health care magic to clear doubts on your health problems. I wish you an early recovery. Dr Shareef."
},
{
"id": 203647,
"tgt": "Can mastrubation cause cut on penis shaft and can shaving the pubic area lead to small bumps?",
"src": "Patient: Hi doc this is so embarrassing but I have a cut on the shaft of my penis and i think it's because of too much masturbation and seems like its not healing quickly should i stop masturbating until it heals? And I've been seeing some small bumps too is it because I have shave that area too is the cause of small bumps? Doctor: DearWe understand your concernsI went throug your details and I uggest you not to worry much. Masturbation in moderation cannot cause any damage to either your penis or to your health. Masturbation is normal and natural. You can masturbate one per day at the most. Because above that brings you a lot of anxiety feeling and a bit of extra tiredness. The cut on your penis could have been due to your own nails, dryness and masturbation or rashes. Stop maturbation until it heals and on't worry about it. Shaving with contaminated blade can cause bumps. Blades used elsewhere if used in pubic area can also cause bumps. Keep the area neat and clean and see for a couple of days. If no relief, please consult a physician.Hope this answers your query. Available for further clarifications.Good luck."
},
{
"id": 204698,
"tgt": "What is the prognosis for a person with chemically induced psychotic disorder?",
"src": "Patient: My son was diagnosted with a cemically induced sycotic disorder 2 yrs ago and received a prolixin shot now the symptoms are back n he s not willing to go to the doctor because he don t want to stay. he s in and out, will he get wrst or is he fighting it Doctor: Dear drug induced psychosis and other psychotic disorders are different entities detailed history is neededdetailed examination by a psychiatrist is needed Regards"
},
{
"id": 223429,
"tgt": "Am I still pregnant after taking cyotec?",
"src": "Patient: i got my periods in 7th dec and had unprotected sex on the 6th day last day on my periodsi took emkit 0.75 together 12 hours later. then 11 days later i took cyotec am i still pregnant my urine test show negative i have taken 8 tablets of cyotec till now Doctor: Hello,Thanks for sharing your health concern with us. I have gone through your query. Firstly, I would like to know why you took the medical abortion kit without confirming pregnancy clinically. Secondly, you had the intercourse during the safe period of your cycle and in addition, you took the emergency contraception soon after which rendered you safe. In the current scenario, since your hormone status would be disturbed, you may experience menstrual irregularities. I would ask you to see a specialist, get a proper clinical evaluation followed by a trans-vaginal sonogram to know the status of the pelvic organs and plan further management. Hope this helps."
},
{
"id": 32100,
"tgt": "Suggest treatment for heavy cough during pregnancy",
"src": "Patient: Hello doctor,i am 3 rd month pregnant and having through t infecting from last 5 days.because of this i am getting heavy caugh and ear pain too.i was said having antibiotics is not recommended for a pregnant lady as the baby lose the resistance power because of this.so,i am taking just cough syrup now.plz suggest. Doctor: Hi Dear,Welcome to HCM.Understanding your concern. As per your you have heavy cough during pregnancy which could be due to respiratory tract infection. I would suggest you not to take any self prescribed medication during pregnancy for cough as almost every type of medication is contraindicated. I would suggest you to take home remedies such as do gargle with saline warm water . Drink green tea in morning everyday. Avoid intake of oily, fried food and cold carbonated liquids. You should visit ENT specialist and get it examined thoroughly and start treatment after proper prescription. You should apply mist humidifiers in room and take cough expectorants.Hope your concern has been resolved.Get Well Soon.Best Wishes,Dr. Harry Maheshwari"
},
{
"id": 225324,
"tgt": "What could be done to prevent pregnancy after 12 hours of intercourse?",
"src": "Patient: Sir last night i had sex with my girlfriend without condom, and 2-3times i ejaculated outside vagina on bed, now i am really scared if a little of my sperm has entered her vagina while intercourse,. Please help me its been only 12hours. Thankyou soo much Doctor: You can take post coital pills available in pharmacy within 72 hours of unprotected intercourse...that will eliminate chances of pregnancy"
},
{
"id": 16518,
"tgt": "Suggest treatment for postural tachycardia Syndrome and ischemia",
"src": "Patient: Hi I am 58 years of age, female and I have been living with P.O.T.S. since 1997. Symptoms became more life altering / depilating in 2004 and have progressed until now. I was diagnosed with POTS & IST at the Mayo Clinic in July of 2010. I am in several studies at Mayo and Ciders Sinai Hospital in Los Angeles. Ciders diagnosed me with Coronary slow flow and ischemia was evident in my heart during a subsequent cardiac MRI with adenosine. They unblinded a study because I lost 9% of my heart function in one month taking Ranolizine in the study they were conducting. Currently I am having many arrhythmias and becoming more compromised in my ability to work and accomplish daily life\u2019s tasks. I have seen MANY doctors in multiple cities and until now few knew much about POTS. My question is what new information is available. I am accustom to ignoring extreme changes in BP and HR and through CATH procedures have shown no blockage so I am assuming the arrhythmias are just something I have to live with. (I have had them they are just worse.) Drugs have always made me worse. I am under the care of a local cardiologist. Have had many tests I am extremely healthy with he exception of all of this. Most published studies are older but finally there are a few newer ones. Because my heart has been beating so fast for so long I believe it is prematurely aging it thus motivating me to do what ever I can to slow that process? (My resting HR averages over 100.) Any suggestions? Any studies you recommend or new published studies? I would like to differentiate what subtype I have if that is possible. I realize my heart is just the symptom of some other etiology that is causing my ST and T depressions. Thanks you! Kathy process Doctor: Hi,Based on the information you provided, I would look toward diagnosing an occult arrhythmia which could be detected by Zio monitoring. Also this diagnosis could be coronary vasospasms which are difficult to detect, have you consulted an electrophysiologist? I have seen POTS usually as a result of another underlying disorder. You should shots triggers for these conditions such as dehydration, caffeine, alcohol, decongestants, tobacco, etc.What symptoms do you experience during these episodes? Do you get headaches or profuse sweating? This would prompt a work up for a rare condition called a pheochromocytoma. There are many possibilities but I would recommend electrophysiology and internal medicine consults.Hope I have answered your query. Let me know if I can assist you further. Regards, Dr. David Hoch, Internal Medicine Specialist"
},
{
"id": 31233,
"tgt": "Suggest remedy for infection and soreness in navel area",
"src": "Patient: i had an infection in my navel recently and i sprayed it out with my shower head. afterwords i went to swab it out and it appeared(felt) like it had opened. I even stuck my finger in and it felt like it went very deep , much more than originaly. could my navel have opened somehow? Doctor: Hi, Thanks for posting in HCM. I understand your concern. Kindly do the following to overcome the infection in the naval area. 1. Wash the area with warm water and mild antiseptic solution like Savlon twice daily. 2. Apply Neomycin cream on the lesions after drying. 3. Take anti-inflammatory medication like Ibuprofen after food. Keep the area under good hygiene. Hope the information provided would be helpful. All the best."
},
{
"id": 77035,
"tgt": "Suggest treatment for lung pain and gastric problem",
"src": "Patient: I am getting pain in back side of lungs when breathing or moving my right hand to up and down. I have severe gastic problem also. I am using Pantaprazole for gastic problem. Still I am having pain continuously from last 3days. Which medicine I need to use for quick relief. Doctor: Hi,Dear,Thanks for your query to HCM.Dear I read facts of your query and reviewed it in context to your health issues submitted in your query.I understood your health concerns and feel Concerned about them.Based on the facts of your query the reply of your query is-Your current health issues seem to be due to -Duodenitis /or hepatitis/ or pleurisy / or pneumonia/ or pleuritis/ Colitis in hepatic flexure/liver abscess/ Cholecystitis / Gall Stones .In view of the complaints, you seem to have duodenitis / or cholecystitis or Gall Stones / or pneumonia or pleurisy / or hepatitis / or hepatic flexure colitis.All these causes could give complaints like back pain while breathing or moving right hand up and down.Therefore Cause of these complaints -like above needs to be ruled out.Other causes-like discussed above , need to be verified.For this you need to consult attending GI Surgeon and Physician.Do's Suggested-Consult and physically examine from GI Surgeon / and Physician.They would Do needful Investigations-like USG abdomen / Liver/Gall BladderX-ray / CT Chest and Abdomen and would fix the exact cause.Stool and Blood tests would fix the Colonic and other causes.As the pain is continuing for last 3 days-Keep on bland non spicy DietBe on Liquids and semisolids x 1 weekReduce on Vegies and fats.Tab Mebeverine 1 x 3 times a dayTab Combiflam 1 x 2 times a day Tab Ciplox oZ- 1 x 2 times a day x all for 10 days timeTab Zincovit 1 x 2 weeks.Thus You should consult attending GI Surgeon and Physician.This would resolve the complaints of your's.Constant follow-up with local family or specialist GI Surgeon and Physician. for long term would give you long remissions of such complaints in time to come,which would depend on meticulous follow-up treatment of your health concerns.Just don't worry and be patient and co-operate with your doctors,till you verify with your attending doctors.For early recovery-please follow above do's which would resolve your complex health issues.Hope this would help you to treat your health issues in the best way possible. Welcome for any further query in this regard to following link-http://doctor.healthcaremagic.com/Funnel?page=askDoctorDirectly&docId=70229Wishing Good Healthy Life!!Dr.Savaskar M.N.Senior Surgical SpecialistM.S.Genl-CVTS"
},
{
"id": 74121,
"tgt": "Suggest treatment for pleurisy",
"src": "Patient: Hi, I have had bronchitis that led to pneumonia which led to pleurisy. Yet I have hhad the past week with my pleurisy a pounding heart and resting heart rate around 105. I have had pain in my left neck and wrist and hand. Should I be worried or is this something to expect with pleurisy? I am 27 and have otherwise been very healthy at a healthy weight. Doctor: Hello! Thanks for posting your query . Pleurisy is a common complication of pneumonia . The good news is , if detected in early stage, it can be completely cured with the right choice of antibiotic .I would prefer to start you on antibiotics like amoxyclav and azithromycin to start with , which generally work well for community acquired pneumonia. Before the first dose of antibiotic, I would suggest you send a sample of your early morning phlegm for gram stain and culture sensitivity (the test that detects which organism has caused your phlegm to get infected and also tells which antibiotic will work to kill that organism ). As far as your pain is concerned , it seems to be a musculoskeletal pain . The pounding heart is also mostly likely due to the pain. So taking analgesics, hot compress and physiotherapy should help. Do consult a physician in person for a complete check up and evaluation . Hope my reply was of help. Do drop your valuable feedback. Thanks."
},
{
"id": 28964,
"tgt": "How can skin infection on the penis be treated?",
"src": "Patient: I ve had this thing on my penis for like 7 months now. Its not an std or anything, its just a thing that i got while masturbating and now the skin on my penis has turned into something near a dead skin cause it has a pinkish color. At first I didn t thought about it much but now I think i have to do something or it will remain like this, so I started putting 3% hydrogen peroxide on it three days ago. Is it safe putting 3% hydrogen peroxide on the skin and will it fix the problem? Doctor: Hello, It could be an infection because you don't know what it is. So, my advice, please go to a dermatologist. He will look into this and prescribe you proper medicine. And about the dead skin thing well dead skin in dark and painless. If it's pink, it's healthy. Pouring 3% peroxide can make it worse so, make a visit to a dermatologist as soon as possible.Hope I have answered your query. Let me know if I can assist you further.Regards, Dr. Prashant"
},
{
"id": 136123,
"tgt": "Suggest treatment for swollen and painful ankle",
"src": "Patient: my ankle is swollen following a severe calf cramp a week ago. could it be sprained? I feel no pain in the ankle and very minor pain in the calf. my calf feels very good compared to when I got the cramp. The cramp came after a 2 mile moderate walk in which i experienced tightness. I walked thru the tightness and it loosened up. The cramp came later in the day while swimming. I jumped into the pool and felt the cramp in mid air. Doctor: HelloI have read your query. Muscle cramp are common. cramp can be reduced by drinking water and juices. Take calcium supplements and vitamin D supplements and vitamin E. Physiotherapy like ultrasound and Tens may give you relief. I hope I have answered your questions. If you have further questions please feel free to contact us. I will be happy to answer. Take care."
},
{
"id": 203280,
"tgt": "What do you suggest for a red and swollen penis?",
"src": "Patient: Hi Doc! My penis and scrotum and the skin around my anus is very red and swollen: In fact all the skin on my penis dried and peeled off but the redness has come back. I have had no sexual contact with anyone besides my girlfriend of 3 years. First symptoms were after I masterbated using saliva but the lubrication was not enough and my penis was a little red after. from there is just got itchy then spread to other areas through scrating. I have had the problem for about 5 weeks now. thanks for your help Doctor: Hi,from history it seems that you might be having fungal infection on the part.due to itching there might be having some bacterial infection causing redness and swelling.Apply anti-fungal lotion or cream locally.If infection is more go for one course of antibiotic medicine course to control bacterial infection.Go for oral anti fungal medicine course like Flucanozole.Keep local hygiene proper.Ok and take care."
},
{
"id": 216591,
"tgt": "Suggest remedy for pain and soreness in chest after injury",
"src": "Patient: I was jumping on a trampoline and did a backflip.and landed on the back of my neck and my chin hit my chest and my neck popped.Now my chest hurts to cough and cross my arms.Can I please get an explanation as to what may be the cause of my chest pain.i have looked at other websites for explanations and they seem to all have similarities with a hurt arm also.my arm is not in pain just my chest. Doctor: There is a high possibility of spine trauma, clavicle fracture,etc.Suitable investigation and imaging yet should help to find out the cause.I strongly advise to consult a local orthopedist.Regards"
},
{
"id": 32911,
"tgt": "How long does tissue damage take to cure?",
"src": "Patient: I walked into an Urgent Care for injection for allergy to ant bites. About 30 seconds after injection I started screaming in pain. They said they injected two inches to high, I was taken to car in wheelchair. Now five months later I am still in teribble pain and can barely walk! I was told I have Tissue damage? How long does this take to heal, what else could be wrong? Thank you, Bonnie Age 69, 5', 90 pounds Doctor: Hello, welcome to HCM. Ther could be injury of nerve branches, hence you had sever pain and difficulty in walking. It will come back to normal but it will take time. mobilise that limb oftenly. It will come back to normal."
},
{
"id": 46851,
"tgt": "Suggest treatment to remove kidney stone",
"src": "Patient: my son age 22years is having 5mm stone in lower pole of ieft kidney. he has gone through 3 days hydro therapy in a hospital where 15 glucose bottles were injected to him.but the stone did not came out. he is suffering badly from pain. doctor has advised him to take plenty of water and take pain killer? can you suggest suitable medicines by which this can be cured? Doctor: Hi5 mm stone in lower part of the kidney usually does not cause pain unless it drops into the ureter- the tube draining urine from kidney to bladder. I presume the diagnosis is based on a CT scan of the kidneys and bladder and not based on ultrasound alone. If not then I would advice you to get a CT scan of kidneys and bladder. Ultrasound is not good for diagnosing stones in the ureter.If the stone in ureter is confirmed on CT scan and if your son is still in pain then the only treatment is do to surgery to remove the stone. If the stone in ureter is small then he can be treated by a combination of pain killers called NSAID(diclofenac, ibuprofen, Naproxen etc) and tamsulosin."
},
{
"id": 139430,
"tgt": "Can toxoplasmosis cause neurological problems?",
"src": "Patient: Can Toxoplasmosis cause neurological problems such as random pains, weakness, numbness and so on? I am a 50 yr old, straight white male, 5 10.5 height and abt. 200 lbs. No HIV. Diagnosed with Toxoplasmosis at age 20 probably from pregnancy by mother (contact with a cat). Went blind in one eye. Surgery and medicine treated the eyes. Diagnosed with migraine (not the headache but neurolgical). Flashes of light, double-vision a year after diagnosis of Toxoplasmosis. Had the highest Toxoplasmosis count on record in 1982. Diagnosed with cubital tunnel and other arm conditions at age 36. More symptoms after that. Tested for Multiple Sclerosis with a lumbar puncture at age 48. Negative. MRI showed some minor scarring. Also showed mini stroke had previously occurred. Another mini stroke at age 48 causing double vision for over 2 months. Also diagnosed Type 2 diabetic in 2009. Can this be Toxoplasmosis causing all of the random pains and neurological trouble with arms? Doctor: Hi, Toxoplasmosis at age 20 will not cause any symptoms at age 50. You need detailed evaluation to establish the cause of your symptoms. It is possible that anti-migraine treatment can make you feel better. Hope I have answered your question. Let me know if I can assist you further. Regards, Dr. Sudhir Kumar, Neurologist"
},
{
"id": 101772,
"tgt": "Is it needed to use budocort 200 inhaler if wheezing happens only twice?",
"src": "Patient: my 2 years old son had wheezing, only for 2 times, as a result of sever cold and cough. he has been prescribed with budocort 200 inhaler to use two times a day for one and half years, as a precaution. i'm tensed to use it as my son is only 2 years old, and has to use for the next one and half years. its a steroid right? it may cause side effect also? as he had wheezing only 2 times, is it advisable to use this inhaler? Doctor: Hi and thanks for the query,I am afraid I am not of the opinion this inhaler should be used by the child at this stage. Wheezing occurring only twice does not really compel usage of this steroid. I would suggest some watchful waiting to see if the situation repeats itself once more. It s too early in my opinion. Kind regards"
},
{
"id": 62258,
"tgt": "Suggest treatment for bump on foot",
"src": "Patient: There is a bump on my right foot that I found out I had after looking in the bushes for my Bakugan. I tried digging for it with my right foot. I guess that was pretty stupid, but I don t know what the bump is. Could you help me? (I forgot to mention my veins are purple and they are showing. Doctor: I studied your query in full details updated from you.I understood your health concernsBased on your query data, In my opinion ,Dear You seem to suffer from Insect bit with a bump,with bruise,which has caused your veins purple.Treatment- Rest / ElevationTab-Anti-histamin-Like Zyrtec-or cetrizineTab-Motrin.Hot fomentationIf no relief, consult a Surgeon for further treatment.Hope this would resolve your query and worry and Anxiety accompanied with it.Welcome for any further query in this regard to ME.I would love to help you out.Awaiting for any further query.Wish you fast recovery from this intriguing health problem.Have a Good Day.Dr.Savaskar M.N.M.S.Genl-CVTS -Senior Surgical Consultant."
},
{
"id": 107121,
"tgt": "Suggest treatment for back pain",
"src": "Patient: I have pain on the right side of the back. Sometimes it seems to be around the ribs, other times along the mid spine. It comes and goes in the course of the day. There is a sense of tightness. It has gone on for a few months. The right hip has ached off and on for years. The shoulder are often tense. The shoulder tension and hips have been an issue for years, but the back is new. Thank you Doctor: Hi, I had gone through your question and understand your concernsWith such presentation in my clinic ,I would like to exclude radicular pain from the back to the hip by doing MRI for the back .I advise you also to do x-ray pelvis to exclude any hip pathology .I advise you to receive muscle relaxant ,neurotonics and NSAID and I prefer wearing lumbosacral belt."
},
{
"id": 192275,
"tgt": "Is non obstructive azoospermia curable?",
"src": "Patient: Hello DoctorThis is Mohit here. I am suffering from Non-obstructive Azoospermia (Sertoli Cell only Syndrome), Germ Cell Hypoplasia. I wish if it would be curable with your advice and guidance. I have undergone FNAC twice. First, few years back, and the result were \"Maturation Arrest\" and second has been done now and the result is \"SCO, Germ Cell Hypoplasia\". Please suggest if it is curable. Currently my attending doctor suggested that it is not curable. Other details are:Here is the outcome of reportsLH: 9.8FSH: 16.7Testosterone: 192FNAC:-Specimen: Bilateral Testis-Gross: Thread Like-Microscopic Exam: FNA smears from both testis predominantly show clusters as well as single scattered sertoli cells. In addition, right side shows few germ cells with occasional spermatidThe features are of germ cell hypoplasia.Please suggest if it is curable or not. If yes, kindly suggest how should we proceed and where can we consult to get the desired result.-RegardsMohit YYYY@YYYY Doctor: Hi, Unfortunately it's NOT curable. However, there is 30-40% chance of having sperms from the testicles if we do TESE testicular sperm extraction) by which we make small cut in the testes under anaesthesia and see under the microscope. These sperms can be used for ICSI (intracytoplasmic sperm injection) during which the sperms are injected inside the wife's eggs (collected after stimulating the ovaries by injections). The embryo is then transferred inside her womb. In this way, you can become the biological father of your baby. Hope I have answered your question. Let me know if I can assist you further. Regards, Dr. Sujoy Dasgupta, OB & GYN Specialist"
},
{
"id": 170991,
"tgt": "Suggest treatment for congenital scalp problem of a 10 month baby",
"src": "Patient: i am a mother of a 10mos old baby boy. he was born with a congenital scalp problem, his brain was really seen during birth..just a thin like a plastic was covered, a day after it was fully covered. i think the scalp granulated.however,there is no hair grows.is there a possibility or other means to have a hair.. Doctor: HiIt seems that your child had encephalocele which is well operated. The poor hair growth at the site of repair is usually permanent due to lack of hair follicles in the scalp scar. Don't worry as it can be easily treated later by hair transplant later once he grows up.Take care"
},
{
"id": 52229,
"tgt": "What can cause a painful pulling sensation on the right side of the abdomen after removal of a gallbladder cyst?",
"src": "Patient: I had a liver cyst adjacent to my gallbladder. The cyst was deroofed and the gallbladder removed. I have 5 small incisions. My surgery was 9/15/2017. I am having a pinching and painful pulling when I move in certain ways and a pain in my right side that is getting worse with time Doctor: Hi, This looks to me like so-called postcholecystectomy syndrome which has all these symptoms and this occurs in about 10 percent of patients who undergo cholecystectomy, usually can be transient, and rarely persistent or lifelong. Symptoms include systemic symptoms such as sweating and heat sense but also dyspepsia, nausea, and vomiting, flatulence, bloating, and diarrhea, Persistent pain in the upper right abdomen. Some individuals may benefit from diet modification, such as a reduced-fat diet, following cholecystectomy since it may be more difficult for digestion of fatty foods. Postcholecystectomy syndrome treatment depends on the identified violations that led to it. Typically, the patient is recommended dietary restriction table with fatty foods, enzyme preparations, antispasmodics, sometimes oral ursodeoxycholic acid can alleviate the condition. Hope I have answered your query. Let me know if I can assist you further. Take care Regards, Dr Iven Romic Rommstein, General Surgeon"
},
{
"id": 224618,
"tgt": "Pain in legs, bright blue veins on chest, shoulders. Started taking Yasmin birth control. Should I continue this medication?",
"src": "Patient: Hello, I am 27 years old mother of two young children. I started the Yasmin birth control pill 5 weeks ago. I have had constant pain in my leg which has lasted 2 weeks and iv now noticed bright blue veins showing up on my chest and sholders. Veins dont usually show on my skin as I am olive colour. should I continue with the pill and ride it out or should I stop untill investigated? Apreciate any advice. Doctor: Hello. Thanks for writing to us. The pain in the legs with the prominent veins are likely to be due to the hormonal pill that you are taking. In case the pain in legs increases on walking then vascular disease needs to be ruled out after stopping the pills.I hope this information has been both informative and helpful for you. Regards, Dr. Rakhi Tayal ,drrakhitayal@gmail.com"
},
{
"id": 198823,
"tgt": "What causes dizziness and nausea after an erection?",
"src": "Patient: hi im a 39 year old male ..i want 2 know why do i feel dizzness and nauseas during an errection ..i took my blood pressure and its normal.. i do not have no medical problems but high blood pressure ...im taking librium 4 like a month....it feel like im about 2 have a heart attack what could that be Doctor: DearWe understand your concernsI went through your details. You have to answer few questions. Were you stressed or physically tired during the current erection? Were you anxious? Dehydration? If you were stressed, physically tired and anxious, you could feel dizziness. Nausea could follow when you are anxious. Dehydration can also show the same symptoms. Still, I suggest you to consult a physician for thorough investigation. Take care.If you require more of my help in this aspect, please use this URL. http://goo.gl/aYW2pR. Make sure that you include every minute details possible. Hope this answers your query. Available for further clarifications.Good luck."
},
{
"id": 151621,
"tgt": "MRI showed disc protrusion causing compression of nerve root and thecal sac.
Chances of regenerating the discs?",
"src": "Patient: hi my name is wadah al omary i am 51 years old ,85 kg , 178 cm hight. My MRI report is following: L4 - L5 paracentral disc protrusion compressing the ? nerve root exit canal w deavy disc space. L5-S1:small central disc protusion compressing the thecal sac . My question: IS it possible that my discs can be regenerated by any chance? Can they regain their fluid (jelly like status)? What precautions should I take ? Will it ever get better or cured? or it will remain the same or get worse in my future life? Can I play , do some sports in this condition? Many thanks for providing answers. best regards, wadah omary Doctor: DONT LIFT HEAVY WEIGHTS, BEND UNNATURALLY, OVERTURN, OVERSTRETCH, OVERWORK YOURSELF. Take Hypericum 30 4 pills x 4 times daily x 15 days tell after 15 days.."
},
{
"id": 172147,
"tgt": "What causes stomach pain, cramps and pink color stool to a 3 year old?",
"src": "Patient: My 3 yr old had a tummy ache last night, i assume she was cramping, and when going to the bathroom, she all of a sudden wanted to go in a diaper and there was a pinkish color in her mushy stool. is that pointing to an irritable bowel? She is acting normal and seems to have slept really good. Doctor: She might have acute bacillary dysentery.she has to be given cefixime or other antibiotic for that. Give her lot of fluids. If her stomach pain is severe and continuing , u have to do ultrzsound abdomen for intessusceptipn..."
},
{
"id": 153189,
"tgt": "What causes whole body cramping after thyroid CA treatment?",
"src": "Patient: in my case total thyroductmy done for thyoried CA & after that i had taken taken nuclear medicine treatment (I -131) for thyroid CA before 9 months & As per recommendation of Jaslok hopital From last month i had stopped thyroxine to check reocurrance of CA, now the results had come no reoccurance of ca seen in all test but i am facing severe cramps in my whole body , what is the reason behind those cramps ??????? Doctor: Hello! Good Day! I've read your concers and will try to breakdown the anwers for you. Whole body cramping can be from several factors- electrolyte imbalance, dehydration, lack of oxygen in the body. However, in your case, it might be caused of lack of thyroid hormones since you stopped your thyroxine. If I was your treating physician, I will request for the following: thyroid function tests, electrolyte panel (Na, K, iCa), CBC. Treatment depends on your results."
},
{
"id": 206930,
"tgt": "Suggest a remedy for lack of concentration",
"src": "Patient: hi dr. There is a boy child in family. Who failed in 10th class with 5 subjects out of 6. But he is normal child as other childs. He has a good physical, mental health. But he cant sit quit for amoment. I think he has a problem of extra energy. I need help for him. So what can i do. Plz help. Doctor: DearWe understand your concernsI went through your details. I suggest you not to worry much. The description given here is not enough to have a proper diagnosis. Still from the given information, it seems that your son may have troubled with Hyper activity disorder, comes under ADHD. To have proper evaluation and diagnosis, you need to consult a psychologist.If you require more of my help in this aspect, Please post a direct question to me in this URL. http://goo.gl/aYW2pR. Make sure that you include every minute details possible. I shall prescribe the needed psychotherapy techniques.Hope this answers your query. Available for further clarifications.Good luck."
},
{
"id": 175881,
"tgt": "Suggest treatment for persistent nausea in a child",
"src": "Patient: hi my 4 year old daughter went to her babysitter this morning feeling perfectly fine and after breakfast started vomiting and has been non stop vomiting all day and still vomiting up liquids this evening. We have given her pedialyte, about 1/2 cup over the last hour and has not had food since 12pm. Is this type of vomiting normal for a toddler? She has had viruses before but nothing ever to this extent. At what point is too much? Doctor: Hi...by what you quote I feel that the kid is having prodrome of watery diarrhea (Starting stage). If there is vomiting you can use Syrup Ondansetron as prescribed by your pediatrician.If diarrhea starts - it will take 5-7 days to completely get better. Unless the kid's having low urine output or very dull or excessively sleepy or blood in motion or green bilious vomiting...you need not worry. I suggest you use zinc supplements (Z&D drops 1ml once daily for 14 days) & ORS (Each small packet mixed in 200ml of potable water and keep giving sip by sip) as hydration is very important and crucial part of treatment. Regards - Dr. Sumanth"
},
{
"id": 170237,
"tgt": "What causes leg pain and loss of appetite?",
"src": "Patient: Hello my I have a 4 year old who keeps complaining her legs are hurting , she has got loss of appetite also , I just read in a magazine that a woman took her son to the doctors because he was complaining of his legs hurting and it turnt out that the little boy had hodgkins lymphoma I'm really worried Doctor: Hi, welcome to HCM. I have read your question and I know that you are very concerned about your child. Most common cause of pain in legs in 4 year old child is hypocalcemia. In my opinion, you should do calcium and phosphate levels and X ray wrist ap view to look for rickets. I don't think that it's a hodgkin lymphoma, however if you want to rule out it, then you should do a complete blood count with peripheral smear. I hope this will help you. Take care."
},
{
"id": 220287,
"tgt": "Can pregnancy still happen without an intercourse?",
"src": "Patient: Hi, may I answer your health queries right now ? Please type your query here... i and my boyfriend were messing up...he was wearing boxers and pants and me was wearing underwear and pants.....and i was on my ovulation day.... what s the possibility of getting pregnant ??? i m really worried Doctor: Hello dear,I understand your concern.In my opinion there is no chance of pregnancy in your case.Usually there is scope for pregnancy when semen is ejaculated inside or around the vagina during the fertile period.As nothing such happened you need not worry.Just messing up with clothes on doesn't result in pregnancy.Avoid stress and wait and check for the period.Stress regarding the thought of pregnancy also might delay the periods confusing the situation.So avoid stress as there is no chance of pregnancy.Hope this helps.Best regards..."
},
{
"id": 197740,
"tgt": "Suggest treatment for low sperm count and high staph aureus growth",
"src": "Patient: 29, Eezy. Good day. I'm married and have been expectant but after conducting sperm test, the result shows that my sperm count has dropped to a minimal and that I have high growth of Staph aureus. Pls, which of the antibiotics do i take and for how long can I treat this and also boost my sperm count? Doctor: HelloThanks for query .Your semen analysis ahs revealed Oligospermia (Low Sperm Count ) with presence of infection .Take Doxicycline twice dailky for 4-6 weeks and repeat the smen nalysis after 5- 6 weeks .This should help to eradicate infection and increase sperm count .Dr.Patil."
},
{
"id": 212521,
"tgt": "Anxiety attacks. What is the name of panic attack that physically kills an individual?",
"src": "Patient: my mom has had anxiety attacks since she was a child.. she has been leagally dead because of these.. and needed a defibulator to come back her heart and blood pressure stops.. and she faints right away.. then is in a comma state without reachtion.. then shakes whole body jumping uncontrollably .. she has shakes to any mental stress as low as feeling guilty of telling someone she would do something and then find that she cant.. she has had a stress test .. they diagnosed her with a severe panic attack but i forgot the name to sign her up for disability .. she is currently living with me. and has been told she cannot work. she has been tested for parkensins and other illnesses. please tell me what the name of the panic attack that physically kills a individual. thank you for your time Doctor: Hello and welcome to Healthcare Magic. Thanks for your query. To answer your question directly, no panic attack \"physically kills\" an individual. No anxiety disorder, including panic attacks, requires a \"defibrillator\" to bring back the patient to life. The facts which you have mentioned that \"her heart stops\" and that she becomes \"legally dead\" are totally inconsistent with a panic attack. However, you mention that she has had tests and has been told by the doctors to have \"severe panic attacks\". It is not possible for me to give you an exact diagnosis without a detailed history and a proper evaluation / psychological assessment of the patient. So, I would suggest that you discuss and clarify with her doctors regading her exact diagnosis. Wish you all the best. Regards, Dr. Jonas Sundarakumar Consultant Psychiatrist"
},
{
"id": 172294,
"tgt": "What causes sleeplessness in infants?",
"src": "Patient: My newborn son is a week old today. He usually doesn t sleep much but tonight all of a sudden he seems to be really out of it. He seems to be breathing fine and pulse is normal but we can t wake him up. He will open up his eyes occasionally but that s about it. Doctor: Hi,Thanks and welcome to healthcare magic.Most of the babies suffer from colicky pain during first three months of life.This physiological and clears after 3 months of age.This is because the peristaltic movement of the gut is felt by the baby.Colimex-DF drops 5 drops 3-4 times daily will give relief.Hope this reply is OK for you.Please feel free to ask further queries if any.Dr.M.V.Subrahmanyam."
},
{
"id": 19147,
"tgt": "Is pain in middle of chest serious?",
"src": "Patient: Hey, my 17 year old boyfriend has a really bad concave chest and for a while he has been complaining of chest pain and in may it was so painful that he could not take deep breaths, walk or lay down and now again today its that bad. he said it feels like sharp shooting pains in the middle of his chest... is this serious? and does it have to do with his concave chest? Doctor: Hello!Welcome and thank you for asking on HCM!I understand your concern and would like to explain that your boyfriend's complaints do not seem to be related to any cardiac disorders. A musculo-skeletal pain or inflammation could be the cause of all this clinical situation. Coming to this point, I would recommend consulting with his family physician for a physical exam and some tests: -a chest X ray study and pulmonary function tests- a resting ECG- complete blood count, PCR, ESR for inflammation. Meanwhile, I would recommend taking ibuprofen three times daily for some days. Hope to have been helpful!Kind regards, Dr. Iliri"
},
{
"id": 107793,
"tgt": "Suggest treatment for Scoliosis",
"src": "Patient: I was diagnosed with scoliosis when i was in middle school. I am now 23. I have days when my back hurts all day. I work as a nursing assistant. I have a back brace that i bought. Its not a medical brace just a ordinary brace. Do i need to follow up and have it rechecked since its been a while since a doctor last looked at my back. Doctor: Dear- thanks for using our service and will give you my medical advise. It is important to have a follow up of your scoliosis with an orthopedic. This is a condition that can get worse and give you more symptoms as you get older. You nee a medical brace, one that has been made for your specific measurements and support. This is something that will avoid to get worse and will relieve the pain.I hope that my advise has been helpfulDrSara"
},
{
"id": 105636,
"tgt": "Child has cough, running nose, Allergy Rhinitis, taken allegra and omnacortil, used asthalin inhaler and budanase nasal spray. Side effect?",
"src": "Patient: Hi Doctor , My 2 year old son suffers from Allergy Rhinitis . The most common symptoms are chronic cough leading to throwing up and running nose . The first time we realised he had this was when he was 19 months . He was given medications like allegra , omnacortil . After 2 months of prolonged illness he was also given asthalin inhaler and budanase nasal spray since he was not getting any better . He has been experiecing the same symptoms since 6 weeks now . Allegra helps but temporarily . Can i start with the inhaler and nasalspray again ? Also what coudl be the side effects of these medications if continued for 2-3 months at a stretch ? Doctor: Hello, Your son has typical symptoms of allergic asthma and combination of bronchodilators (such as budesonide inhaler with spacer device), montelukast 4mg, antihistamines and steroid nasal sprays (used for 6 weeks and stopped before restart after 3 months) is the standard management plan. As with all steroid usage, growth retardation is a recognised side effect but much lower than oral steroids as inhaled steroids are generally used to control airway inflammation along with montelukast. Skin prick testing is recommended to ensure common household allergens like cats and dog dander are not making the asthma worse when avoidance will be key. House dust mite allergy is also important to know as changing bedding or adding a layer of protection with dust-proof covers for mattress, pillows, duvets are important. Important to get rid of old soft toys as they accumulate dust very quickly. I hope that was useful."
},
{
"id": 111529,
"tgt": "How long will steroid shot take to heal tailbone injury?",
"src": "Patient: I injured my tailbone 7 months ago and still have a hard time sitting and the pain is constant. Not always debilitating but always there like a bad headache. I have had pt for months with ultrasound and a steroid shot but still in pain. Is time the only thing to heal this? Doctor: I think,you are right.Perhaps time is the only relieving factor.In the meantime,you should adopt a suitable posture like using of cushion while sitting,avoiding the pressure contact of tail bone area while sitting etc."
},
{
"id": 95990,
"tgt": "Bloated stomach and swollen abdomen",
"src": "Patient: Hello, i hope you can help me with what this is. For the last 4 days my stomach has been SO bloated! And the sides of my stomach, below my ribs feel achy. So does my back. Its not painful but im quite slim and this uncomfortable bloating isnt normal for me. Ive recently had ECG s and found abnormal ECG. But said it might be normal for me. The reason for that was i was gettin chest pain . They told me its costochondritis & anxiety . Im not anxious! Still get chest pain and shoulder blade pain. But this bloating is new. Im only 23/ female and quite healthy. With a good diet. BUt i do smoke. This bloating is very uncomfortable, also (Ahem..) Ive been a little gassy lately and a bit constipated. Is there anything i can do to relieve it. Scared it might be hernia, ulcer or worse. Please Help :( Doctor: Hi At present these symptoms are likely due to acid peptic disease and gastritis. But I wonder as your ECG is abnormal and you are 23 years and even then they say you are normal. Even it is costo chondritis and pain is due to gastritis your pain should be re evaluated. Consult physician and discuss about above points."
},
{
"id": 38534,
"tgt": "Who should be consulted for sweat gland infection?",
"src": "Patient: Hi. I've had a sweat gland for the last 3 years that constantly gets infected. It began when I was pregnant. My Dr. gave me antibiotics several times, but it kept coming back. It's located between my vagina and thigh, in the crease where the two connect. Should I go to a GYN for this or a dermatologist? Doctor: Hi, You should get consulted with dermatologist for best treatment for infected sweat gland. Apart oral antibiotics, you might need surgical treatment. Consult with dermatologist first. If he/she will ask for surgical treatment, then, should schedule appointment with general surgeon. Wish fast recovery!Dr.Albana"
},
{
"id": 195004,
"tgt": "Is a mildly in-homogeneous prostrate posterially after passing a kidney stone a concern?",
"src": "Patient: Just received the results from a renal ultrasound following what appears to be the passing of a kidney stone. No more problems found except an issue with the prostrate (which was not the focus of the scan). Said it was mildly inhomogeneous posterially but otherwise unremarkable . Six months ago had digital exam which was fine and a PSA under 1. Any comments on this? Could the funding be related to the kidney stone trauma? Was in the emergency room for 8 hours on morphine for excruciating pain. Doctor: Hi, You might have a slight enlargement of prostate, which is common among men above fifty years of age. The pain may be due to kidney stone and not related to prostate. You can consult a urologist for further evaluation. Hope I have answered your query. Let me know if I can assist you further."
},
{
"id": 11060,
"tgt": "What causes bald spot on head with dark center?",
"src": "Patient: my husband has a hair loss on back of his head with a dark center about the size of a tennis ball it doesnt seem to bother him. he has been in the cleveland clinic for 2 months and just got out survived septic shock what do you think the bald spot with dark center is from Doctor: HIWell come to HCMI really appreciate your concern, this could be Male Patron Baldness, this can be managed with \"Minoxidil 5% lotion\" apply this on hairless area, else nothing can be done, hope this information helps, take care and have a nice day."
},
{
"id": 88362,
"tgt": "What causes recurring abdominal and back pain?",
"src": "Patient: The last few months I have had lower back pain and abdominal right side pain. Thought I had a UTI, tested for micrcoscopic blood in urine. CT scan showed no kidney stones. Continued to have mircroscopic blood in urine..Was sent to a urologist, did another CT scan with contrast, no urological issues, right ovarian cyst and small amt of pelvic ascites..I want to follow up with my GYN, but what should I ask them to test for at this point.? I had a pelvic exam about a month ago, but many of the symptoms I have concern me about ovarian cancer. What should I ask for test wise to further identify the recurring abdominal and back pain and bloating Doctor: Hi.Thanks for your query.Read the history and understood it. The tests you should ask for are : A course of an antibiotic and metronidazole to be taken by you and your partner. Diagnostic Laparoscopy and sos procedure, send the fluid for analysis to see that you are not suffering from tuberculosis of abdomen.Biopsy if required, To get the offending tissues out ."
},
{
"id": 146788,
"tgt": "What causes dizziness while standing or lying own?",
"src": "Patient: when lying down for a while, and then standing up suddenly, or vice versa... i get dizzy for a bit.. a feel strange in my chest, like there is a small pressure inside.. should i be concerned? \u00a0\u00a0\u00a0\u00a0\u00a0\u00a0\u00a0\u00a0\u00a0\u00a0\u00a0\u00a0\u00a0\u00a0\u00a0\u00a0\u00a0\u00a0\u00a0\u00a0\u00a0\u00a0\u00a0\u00a0\u00a0\u00a0\u00a0\u00a0\u00a0\u00a0\u00a0\u00a0\u00a0\u00a0\u00a0\u00a0\u00a0\u00a0\u00a0\u00a0\u00a0\u00a0\u00a0\u00a0\u00a0\u00a0\u00a0\u00a0\u00a0\u00a0\u00a0\u00a0\u00a0\u00a0\u00a0\u00a0\u00a0\u00a0\u00a0\u00a0\u00a0\u00a0\u00a0\u00a0\u00a0 \u00a0\u00a0\u00a0\u00a0\u00a0\u00a0\u00a0\u00a0\u00a0\u00a0\u00a0\u00a0\u00a0\u00a0\u00a0\u00a0\u00a0\u00a0\u00a0\u00a0\u00a0\u00a0\u00a0\u00a0\u00a0\u00a0\u00a0\u00a0\u00a0\u00a0\u00a0\u00a0\u00a0\u00a0\u00a0\u00a0\u00a0\u00a0\u00a0\u00a0\u00a0\u00a0\u00a0\u00a0\u00a0\u00a0\u00a0\u00a0\u00a0\u00a0\u00a0\u00a0\u00a0\u00a0\u00a0\u00a0\u00a0\u00a0\u00a0\u00a0\u00a0\u00a0\u00a0\u00a0\u00a0 and another thing.. I found out /saw that I was breathing very shallow .. and somewhere i've read it can cause high blood presure and all kind off bad side effect to the human body... i wonder does that have anything to do with it? Doctor: Hello, Thank you for trusting HCM.you might have postural hypotention, which will give symptoms like the one you have, its due to fast changes in your blood pressure and your heart is trying to compensate for those fast changes. its not dangerous per se but you have to be careful not to change posture so quickly you might lose your balance and fall down hit something that would be dangerous. So once you wake up take time setting at the edge of your bed before you stand up same for laying down. for more details and proper diagnosis please refer to a cardiologist. Dr Nazzal"
},
{
"id": 215542,
"tgt": "Can Valium be taken while on Lyrica?",
"src": "Patient: I had three surgeries on my ankle in June. A few days ago I finished taking my Oxycontin and lyrica after 47 days on them in the hospital. I was not told about withdrawals and suffered for two days before my doctor put me back on lyrica to taper off. Now I m extremely restless and can t sleep. I m so exhausted. Can I take a Valium or some other sleep aid with 75mg of lyrica? Doctor: Hello, You can safely take valium while on lyrica. It will not interact with each other or affect the efficacy of one another. Hope I have answered your query. Let me know if I can assist you further. Regards, Dr. Shinas Hussain, General & Family Physician"
},
{
"id": 144102,
"tgt": "Suggest treatment for intracranial pressure",
"src": "Patient: Hello doctor,my name is reva from malaysia...my friend did mri scan and found out he is suffering from intracranial pressure..he has severe headache,hypertension and vision problem..he is 25 years old.recently he is complaining bleeding from nose..pls give me some advice and treatment...or any ayurvedic treatment Doctor: Hi, I am Dr.Bruno. I have read your question with care and understand your concerns. Let me try to help you Question : Pls give me some advice and treatment...Answer : Increased Intracranial Pressure can be due to many reasons. It can be due to a condition called Aqueductal Stenosis which can be treated with Endoscopic Third Ventriculostomy. Or it can be due to Post Traumatic or Post Meningitic Hydrocephalous in which case the treatment will be Ventriculo Peritoneal Shunts. Or it can be also due to Blockage in Blood Flow in Veins, which we have to give tablets and if needed do a procedure called Lumbo Peritoneal Shunt. In Short, Ask your friend to consult a Neurosurgeon who will evaluate and do the best for her Hope you found the answer helpful.If you need any clarification / have doubts / have additional questions / have follow up questions, then please do not hesitate in asking again. I will be happy to answer your questions.Let me know if I can assist you further.Take care."
},
{
"id": 80068,
"tgt": "What is the treatment for chest and throat pain?",
"src": "Patient: I am having pain while breathing, throat and chest feel inflamed. It started two days ago with a pain on the left side of my chest ( come and go ) yesterday pain started in left shoulder and left side of neck, couldn t sleep, took two Advil and able to sleep. This morning throat and chest feel inflamed, both in and out breathing. No fever and blood pressure is 123/78. Email YYYY@YYYY Doctor: Thanks for your question on Health Care Magic. I can understand your concern. Chest pain, throat pain and inflammatory feeling in chest is mostly seen in lower respiratory tract infection (LRTI). So better to get done clinical examination of respiratory system and chest x ray. If both are normal then no need to worry for LRTI. Left sided chest pain with shoulder pain can be seen in cardiac diseases also. So also get done ecg and 2d echo. If these are also normal then no need to worry for cardiac disease. You are mostly having upper respiratory tract infection (URTI). You may need antibiotics, antihistamines and anti inflammatory drugs. Since all these drugs are prescribed medicines, you need doctor 's prescription for them. So consult doctor and discuss all these. First diagnose yourself and then start appropriate treatment. Hope I have solved your query. Wish you good health. Thanks."
},
{
"id": 185397,
"tgt": "Can painful gums / ulcers be normal after complete lower jaw teeth extraction?",
"src": "Patient: Dear Doctor, I got extracted all teeth of my lower jaw, a week ago, due to deep infection, as advised by my family dentist. Still I am having pain in the gums & some ulcers. Now my dentist has prescribed Tab. Rebagen to be taken thrice a day for 5 days. Is it right treatment? Will it have any side-effects? Kindly guide. Doctor: Thanks for using Health Care Magic.Read your query.After extraction ,pain in the gums may or may not be present .Tab Rebagen is usually prescribed as a long term treatment for the apthous ulcers.You can continue with it as your dentist may have prescribed it to you after a thorough examination.I would advice you to follow a few instructions:* Try to avoid spicy and oily food stuff. * Use some astringent with anaesthetic mouth paste like mucopain gel to apply it locally in the oral cavity at least thrice a day before meals.* Take vitamin and mineral supplementation.* Drink plenty of water to maintain hydration of the body.* Betadine gargles diluted in water at least thrice a day after meals to maintain oral hygiene.Try these measures for a week and it will help you.Hope this was beneficial.Thanks and regards."
},
{
"id": 105700,
"tgt": "Allergic bronchitis, using asthalin inhaler and salsol nebulization, rapid heartbeat, depressed",
"src": "Patient: hi...i suffer from allergic bronchitis ...its really bad in april and may. i use salsol solution for nebulization. i mostly use asthalin inhaler but when it gets really bad, then i use salsol. of late i have noticed high increase in my heart rate . as a result i can t sleep properly or do anything. this has left me depressed, scared and worried. please help. thanks in anticipation. Doctor: Hi Udit, You are suffering from Allergic Bronchitis. The correct terminology is Bronchial Asthma. Asthalin or Salsol(Same salt) gives only temporary relief. What you need is inhaled steroids which is the mainstay of treatment of allergic conditions. You may take Seroflo or Formetrol 2 puffs 2 times a day. Rinse your mouth nicely after taking these puffs to avoid any fungal infection in the mouth. You should also take Montair LC 1 tab per day and a Pantocid before breakffast. The fast heart rate may be due to disease itself or due to Asthalin/Salsol. Tiova inhaler will also help. You must avoid known or unknown allergens as far as possible. Deep breathing exercises and healthy lifestyle do help ."
},
{
"id": 189482,
"tgt": "Have gum, jaw pain. Had tooth pulled, taking antibiotics, advil, celebrex for pain. Lead to brain infection?",
"src": "Patient: had a tooth pulled five months ago, two days ago started having horrible gum, jaw pain in the area. i'm on antibiotic 500mg amox tid....no fever, take advil and Celebrex for pain without relief. it throbs to my pulse, entire left side of face hurts...I have very high tolerance to pain, I refuse to crowd a er with tooth pain...my only concern is it being a serius infection that goes to my brain Doctor: Hello and welcome to hcm, The JAWache may be due to- GINgivitis. bone loss. periodontal infection. root pieces remaining within the tooth. Get a thorough clinical as well as x-ray examination. Root pieces has to be extracted if present. Massage your gums with gum tone gel. get your teeth cleaned. Take vitamin c rich fruits and supplements. Take care."
},
{
"id": 145332,
"tgt": "How to treat sudden attack of vertigo?",
"src": "Patient: 1. recent sudden attack vertigo ( medical dx, ( room rotate, vomit, decreased balance)) 2. ct scan without contrast showed: gray matter white matter well preserved, an air fluid level noted within right maxillary sinus, right maxillary sinus air fluid level correlate acute sinusitis, mild mucosal thickening ethmoid air cells, opacified left maxillary sinus likely consistent with chronic sinusitis 3. hearing test showed right hearing loss ( below what is considered border line for hearing, ex can hear man voice, child level and higher nope) left ear ok 4. vertigo has subsided, balance better, tinnitus and decreased hearing left ear remain 5. question is don t know what could be Labyrinthitis, maxillary inflamm disease, Meniere s ., how to manage it Doctor: Hello. I have been through your question and understand your concern.It seems to be Menieres syndrome. The CT scan is within normal features and the sinusitis is definitely not the cause. Labyrinthitis could be the cause but you need an MRI to diagnose it. I think it is Menieres disease because it has all the features. You should go directly to the ENT to manage this situation.Hope this helps.Please feel free using HCM again"
},
{
"id": 86968,
"tgt": "What causes upper abdominal pain?",
"src": "Patient: I am having pain in my stomach, mostly from the bottom of my rib cage down to the belly button, on both sides. It feels as though my stomach is bruised on the inside and someone keeps pressing on the bruise. I am almost four months post-partum (3rd child) and this is the first time I'm feeling like this. I have been working out for almost two months now, and got a back massage on Friday for the first time. I started feeling like this the next day (Saturday), and this is now the third day with pain. I have been going to the bathroom regularly and don't feel nauseous at all, I just have this pain in my stomach that feels worse when I walk and lay on either of my sides. Any idea what this could be ? Thank you Doctor: Hi.Thanks for your query.You have a very specific history of pain in the upper abdomen which increases on walking or laying on side, without any nausea or bathroom related problems. This in fact can be a sore peritoneum due to inflamed intestine or lymph nodes in the said area or colitis of the transverse colon. Get a course of an antibiotic and anti-inflammatory medicines for 5 days, see the results; take strict bed rest and be on liquid diet. If there is no response please follow this:Consult a Surgical Gastroenterologist., get a clinical examination done and the investigations as suggested by Him likeRoutine tests of the blood, urine and stool, Ultrasonography and CT scan if needed of the abdomen.Colonoscopy if warranted.-This will give the proper diagnosis which is the most important part of any treatment."
},
{
"id": 11794,
"tgt": "Dark spot near right eye, looks like a bruise. Due to allergic reaction, sun exposure or thyroid?",
"src": "Patient: Hi there I have a dark spot near my right eye for about 4 months now. It looks like a bruise but doesn t fade or go away. I am not sure if it has something to do with my thyroid or sun damage or I got allergic to a skin product. In any case is there a solution for it to go away. Please let me know. My family doctor had checked it too twice and said it will go away with time but its been 4 months and the mark is still the same. Thanks Doctor: hi dear, first thing bruise wont last for 4 month, so its not that. sun allergy should be on entire face, not on one spot, same goes for cosmetics. also if it is any type of allergy there must be some itch. so what i feel is that it is a kind of nevus , i mean a kind of mole which appear late in life. so i suggest you to meet a dermatologist and get checked what it is. thank you"
},
{
"id": 139350,
"tgt": "What could squishy, swollen part on foot after being stamped mean?",
"src": "Patient: About a month ago, I was stepped on very hard in soccer practice with a girls cleat. The bruising lasted weeks, but they have finally faded. There is, however, a bump still on the top of my foot. The squishy part of it had moved beneath my ankle bone. There is no severe pain, but it is squishy and appears swollen. can you please tell me what is wrong with it? Doctor: Nothing wrong sir the blood that collected is getting dispersed and will vanish in few days, since no more pain nothing to worry. Enjoy playing, but prevent injury as much as you can, for rest we are there."
},
{
"id": 223865,
"tgt": "What causes lower abdominal pain and frequent urination post consumption of contraceptive pills?",
"src": "Patient: Need answers pls.. im 28 y/o, my partner and i have been trying to make me pregnant but it always turned out negative for almost 1 yr now. 2013 i take oral contraceptive for 14 months and i stop taking it on nov. 2014. for the next 5 months i have my regular period for 30-35 days cycle but for the next months my period became irregular. my next period came after 91 days, then after 74 days, then 54 days, then after 43 days. and now im 63 days late. i already take a hpt and the result is negative. whats happening to me? lately in experiencing a lower abdominal cramp specialy in the left side, a frequent urination ( almost every 15 -30 mins), and sometimes clear discharge. i gained weight and i always stay late at night. i sleep around 3 to 7 am for 3 months now.. thank you in advance for the help infos.. Doctor: HelloHealth care magic user I understood your concern and My opinion is that you must be having urinary/vaginal infection at present thats why you are having such symptoms .start following treatment 1- clingen vaginal pessary2 -T NItrofurantoin 100 mg bd Regarding your delayed period and infertility problem After going through your history i feels that you are case of PCODplease do regular exercise for 30 min and try to reduce your 10% body weight that will definately help in getting pregnancyI hope my answers help you Thanks"
},
{
"id": 55574,
"tgt": "How to check liver condition after Fatty liver diagnosis?",
"src": "Patient: Dear Doctor, I was diagnosed with FATTY LIVER a couple years back during a ultra sound and camera checkup between my ulcer check up. I want know what should i to have check up my liver condition is their and specific test or ultra sound can be done to exactly the size in reference to regular size of lever. every night i take one 500mg FISH OIL, one NIACIN and one SIMVASTATIN for my high TRYGLYCRIDE. My TRIGLYCRIDE approxi. 230. Doctor: Hi, dearI have gone through your question. I can understand your concern. You have fatty liver. You can monitor that by your sgpt level and ultrasound abdomen. Nothing elese is needed. Continue your fish oil and niacin. Go for regular exercise. Avoid alcohol and smoking. Yoir triglycerides level is also high continue your simvastatin as per advise. Hope I have answered your question, if you have doubt then I will be happy to answer. Thanks for using health care magic. Wish you a very good health."
},
{
"id": 186929,
"tgt": "What causes frequent sneezing, pain and blood clots after tooth extraction?",
"src": "Patient: I had a rear upper molar extracted three days ago, during the procedure the dentist told me my root to my tooth was embedded into my sinuses, they put some type of patch in as a fill after they removed my root. Since then I have had several sneezing incidents and on top of the pain I have had lots of drainage and some blood clots . I have a follow up in 3 days but is there anything else I can be doing in the meantime? Doctor: thanks for your query, i have gone through your question, the complication during extraction that is formation of communication between oral cavity and maxillary sinus could have resulted in some inflammation of the sinus results in sneezing which in turn creates negative pressure and induces bleeding...nothing to worry it will heal in one week...consult your dentist for suture removal and review...continue antibiotics...take soft diet..avoid vigorous gargling...i hope my answer will help you..take care..."
},
{
"id": 83559,
"tgt": "Does Trimethoprim cause hives?",
"src": "Patient: Hi I have a 4 month old boy who got diagnosed with bilateral reflux at 7weeks old He has been on trimethoprim since 7 weks old and this week seemed to develop hives how come? Has he become allergic or immune to this antibiotic? We have gone to emerge due to his pedi being away on vacation and they recommend stop and try this new anti called cephalexin for 14 days until I see pedi again what u think I feel my baby is becoming a Ginny pig Doctor: Hello,Trimethoprim can cause hive as a side effect. I agree with your doctor to start another antibiotic. Meanwhile, I suggest using an antihistamines to relieve the hives such as Cetirizine 10 mg.Hope I have answered your question. Let me know if I can assist you further. Regards, Dr. Dorina Gurabardhi, General & Family Physician"
},
{
"id": 192936,
"tgt": "Does daily dosage of Coumadin lowers the sexual drive?",
"src": "Patient: Hi, may I answer your health queries right now ? Please type your query here... hi, just wondering if a daily dose of cumadin can lower the libido in a 49 yr. old male?.....he has a protein C definciency. He has been on cumadin for over two years now. Thank you Doctor: Hi, If the protein powder contains steroid, it may lower the libido. You can check the description in the label.Hope I have answered your query. Let me know if I can assist you further. Regards, Dr. S. R. Raveendran, Sexologist"
},
{
"id": 129305,
"tgt": "What causes a swollen hand?",
"src": "Patient: My husband had a stroke in May and his left hand has swollen in the past couple weeks -- he thought it was arthritis causing it due to playing golf, but he hasn't played golf in over a week and it is still swollen. His hand also shakes -- could the stroke in May and the swelling be connected?? Doctor: Hello,I think it is related somehow as the stroke is related with the limitation and range of motions of the arm. This limitation of motion can increase the risk for poor blood circulation due to any deep vein thrombosis which gives swelling of the hand. I would recommend you to do a Doppler ultrasound and respective blood tests for deep vein thrombosis to rule out this complication.Hope I have answered your query. Let me know if I can assist you further.Regards,Dr. Edvin Selmani"
},
{
"id": 74493,
"tgt": "Can bening nodules cause breathing issues?",
"src": "Patient: im 54 im having breathing problems im always trying to catch my breath. I can be breathing normal but then i need to catch my breath. My doctor found a Benigne Nodule. Been having cat scans in the last year.... but cat scans keep coming back normal. I also found out that anxiety also causes it. what can i do to help my breathing. Doctor: Respected user , HiThanks for using Healthcaremagic.comI have evaluated your query thoroughly .* Benign nodules location , size is of importance to comment about the query. Kindly consult with the scanned copy of the report of CT scan .* Suggestions for better breathing - Deep breathing exercises - Regular walking in fresh air - YOGA - Avoid stress , anxiety - Have regular sound sleep of 8 hrs - Avoid noxious substances , house dust , pollen etc .Hope this clears your query .Welcome for further guidance anytime .Regards ."
},
{
"id": 204036,
"tgt": "What is causing right testicle to get bigger and hurt while walking? Also having pain in stomach and back",
"src": "Patient: It's happened a few days ago.My right testicle is getting bigger and it's hurt to walk sometime. I also encounter lower back pain and and lower right stomache pain sometime it goes away . Most of the time when i lay down all the pain goes away but my right testicle is still big . I also got a fever before that and also i have musturbate for few times without release . What could it be? Doctor: Hi. Your findings are more suggestive of a hernia. Does the coughing or straining at stool / urination or lifting-up something increase either the size and / pain? If not it can be due to infection in that part as also suggested by fever. You must visit the Surgeon at once ."
},
{
"id": 12522,
"tgt": "Severe psoriasis, liquid is oozing out of the scales, burning sensation. Any treatment?",
"src": "Patient: Hello.. I have severe psoriasis and have been taking ayurvedic treatment for the last one year. It has worked on most infected parts other than my leg. In the last 3-4 days, the scales have dramatically thickened and now some liquid is oozing out all the time which is really worrying me. Its burning a fair bit. The infected area looks really red and black. Skin looks very flaky. Have stopped my medicines for a couple of days now. Dont know what to do. Doctor: Hi friend, Welcome to Health Care Magic See a Dermatologist.. The treatment is fairly advanced nowadays / there is a choice of drugs / and you have a good chance of improvement. Local applications and systemic drugs like immuno suppressants are very helpful - in reducing the flare up and delaying complications... Take care Wishing speedy recovery God bless Good luck"
},
{
"id": 152747,
"tgt": "What are the signs and symptoms of cancer in the neck?",
"src": "Patient: I have large bumps that several doctors call fatty tissue on either side of my neck, sort of on my shoulders that seem to be getting larger. They are not hard or sore but look unseemly. They are growing larger and I am concerned about the possibility of cancer.n Doctor: Hi welcome to the health care magic I can understand your concern Your doctors are telling you about Lipoma like lesions... It can be multiple and it is Benign tumour For rule out nature of lesion whether Benign or malignant I would suggest one investigation that is FNAC .... If FNAC doubtful then confirmatory biopsy taken for histopathology examination So for getting relaxed from cancer stress I suggest you to investigate with FNAC or biopsy Take care Hope I have solved your query"
},
{
"id": 153615,
"tgt": "What are the symptoms of pheochromocytoma?",
"src": "Patient: Hi,I suffer from severe headaches. I have been to see my GP recently as i know started to feel funny when i get these headaches. I start to feel shaky, i sweat, feel like i am going to pass out. After the attack i feel very drained and have a very bad headache. the GP has tested my sugar and thyriod, all is normal.Lately my severe headaches wake me up at night, and no medicane i takes helps. I am a female and 29 years old.Should i be worried? could it be possible that i could have phaeochromocytoma? Doctor: Hi,Thanks for writing in.Pheochromocytoma is a cancer of the adrenal gland. The adrenal gland secretes steroid hormones and therefore a pheochromocytoma patient might have symptoms because of increased adrenal hormone circulating in blood.Some of the symptoms of pheochromocytoma includes the followingepisodic and sudden onset of severe headachessweatingabdominal painhigh blood pressure not getting controlled with medicationsrapid heart rateirritability and anxietyPatients also might have increased plasma free metanephrines. Please consult the endocrinologist and get clinically evaluated. Please do not worry."
},
{
"id": 62089,
"tgt": "Suggest treatment for lumps",
"src": "Patient: i have two lumps on the right outer skin of the scrotum and one below on the left side that just appeared a couple days ago. before they showed up i had a lump on the shaft of my penis. a couple days after that showed up in looked as if one of my veins was blocked it felt as if there was a worm in it. and it was painful. a couple more days went by and i popped the lump and what looked like puss came out and it looked like a small shallow hole. after about 6 days it started to heal and the vein went down. and a couple of days ago it started healing and thats when the lump on my scrotum appeared Doctor: Hi,Dear,Thanks for your query to HCM.Studied your query in full depth of its details.Reviewed it in context of your health concerns.Based On the facts,You seem to suffer from-Boil on the base of the penis from Furuculitis of the pubic hairs/ which late on was popped by you with pus discharge.By the time the penile boil healed with the supplying engorged veins died out, you had scrotal boil on right and left side of scrotum.Treatment-Would be-Blood tests for Diabetes to rule out the multiple crop up of the Boils in private parts.Antibiotics +Combinflam +scrotal hygiene by dettol sponging to reduce the spread at other scrotal sites. x 5 days.Shaving of the part should be dealyed til complete healing of the private area.This reply would help you to plan further treatment soon with your treating doctors.Best of Luck and early recovery.Welcome any further query in this regard,which would be replied in next session.Good Day!!Dr.Savaskar M.N.Senior Surgical SpecialistM.S.Genl-CVTS"
},
{
"id": 184606,
"tgt": "What causes pain in mouth but not teeth?",
"src": "Patient: Recently I have been having pain in my mouth, like aching teeth, but it is not my teeth. My mouth feels hot and my tongue seems quite blistered. I have taken antibiotics for the last few days, and warm salt walter rinses, but in the night I wake up with a feeling of a hot mouth and inflamed tongue. could this be shingles of the mouth? Doctor: Hello,The most likely diagnosis is oral thrush. This requires an anti-fungal medication such as Nystatin. Comfort combination rinses can combine anti-fungal treatment, pain relief and anti-inflammatory medication. Sometimes, corticosteroids are prescribed. Try warm salt water rinses, stay well hydrated, avoid irritants such as smoking, acidic foods and alcohol. Maintain good nutrition, balanced vitamins and good daily dental care. I recommend taking Benadryl for relief. Other anti-inflammatory medications to consider are Tylenol and Motrin. Vitamin B12 is recommended.Are you suffering from Shingles or have you had prior episodes? Have you noticed any mouth blisters? Please provide additional details concerning your medical history and any medications taken. This virus travels nerve pathways. Shingles can affect the mouth. A generalized, overall discomfort is common for thrush, an allergic reaction, a deficiency or dry mouth.Thank you for your inquiry. Consider a clinical exam with your dentist if symptoms persist. I am available to assist you further and will gladly answer additional questions."
},
{
"id": 126596,
"tgt": "What causes severe pain in the buttocks?",
"src": "Patient: For a month, I have had severe pain in my lefts buttocks, hip, leg and foot. I had a laminectomy L1,3,5 in 2014. The current (lower spine/thorastic) MRI shows no major changes, just scar tissue from previous surgery, spinal narrowing, degenerative disc disease, bulging disc and spondylosis. The neurosurgeon says this will go away--that was three weeks ago. Pain Management team has me in aquatics, celebrex, ibuprofen and Tylenol. Yet, the pain continues. Further, the left leg goes to sleep from the buttocks to the foot when I m walking, and I threaten to fall. Could something else be going on other than back issues? Does this warrant a call to my PcP? Should I just give up and live with the severity? I need answers, or help. Doctor: Hi, The severe pain in the buttocks that you have can be related to degenerative changes in the spine causing a mild nerve root compression. Hope I have answered your query. Let me know if I can assist you further. Regards, Dr. Praveen Tayal, Orthopedic Surgeon"
},
{
"id": 156541,
"tgt": "What is the treatment for CA gall bladder?",
"src": "Patient: my friend is suffering from C.A .gall bladder and that too of 4th stage .Suffering from acute jaundice .Billirubin value is 16 and value of alkaline phosphate is 800 .Doctor recommended him ursodeoxycholic acid & Silymarin tablets .What is your opinion ? Doctor: You are suffering from stage 4 gall bladder cancer.Since your bilirubin level is high that is 16 no chemotherapy can be given now.First u have to reduce your bilirubin level my doing metallic stenting as there may be common bile duct obstruction by the tumour.If this fails then a procedure called percutaneous transhepatic biliary drainage can be done.When bilirubin becomes less than 3 then palliative chemotherapy in the form of cisplatin and gemcitabine can be tried to control the tumour.Intent of treatment is palliation of symptoms and not cure.Regards DR De"
},
{
"id": 147635,
"tgt": "Any suggestion for suffering from sciatica?",
"src": "Patient: I have sciatica. I am ready for hydrocodone/ibuprof 7.5-200MG TB. Not help to much. So, i bay today motrin IB. I don t no how many pils used for may problem. Next week go back to doctor for visit. Thank you and sorry may English no to good! Maria Deak YYYY@YYYY Doctor: Hi Maria,Thank you for posting your query.I have noted that you have sciatica. I assume you meant you have back pain and radiating pain to thigh/legs. This is often caused by compression of nerves in the lower back.For symptom relief, you can take pregabalin or gabapentin capsules. The usual dose for pregabalin is 75-150 mg daily and gabapentin is started at a dose of 300 mg twice daily.I hope my answer helps. Please get back if you have any follow up queries or if you require any additional information.Wishing you good health,Dr Sudhir Kumar MD (Internal Medicine), DM (Neurology)Senior Consultant NeurologistApollo Hospitals, Hyderabad, IndiaClick on this link to ask me a DIRECT QUERY: http://bit.ly/Dr-Sudhir-kumarMy BLOG: http://bestneurodoctor.blogspot.in"
},
{
"id": 86212,
"tgt": "What causes severe abdominal pain and tremors after having BP medication?",
"src": "Patient: Head heaviness and upper stomach tremors start in the morning approx 2 hours after a thyroid medication and an hour after blood pressure medication. Having eaten with blood pressure medication. This eases up at night. Approx 12 hours later. What is causing this?? Doctor: Hi.Thanks for your query, albeit short and specific. Since you have not mentioned the names of the medicines that you are taking for thyroid problem and the high blood pressure that is hypertension, it is difficult to pin pint the cause and effect. Yet your history is classical that the pains caused after a specific time after the thyroid and blood pressure medicines indicate That these are causing the problems and that is lasting for 12 hours. I would advise you the following:Consult the treating Doctor immediately so that the new prescription for change medications that may not cause the same problems. I hope this answer helps you."
},
{
"id": 20458,
"tgt": "What causes sense of leaking heart after implantation of stent?",
"src": "Patient: My dad had a stint put in in October. He's felt chest compressions and tightness in his chest the last few days. Last night he said it felt like his heart was leaking.... Does this make sense? He won't go to the dr. because he went Monday and the dr set up a stress test, but it isn't until March. Doctor: Hello,Brief answer: This is a false sensation but anginal pain should be checked.Explanation: My opinion is that shortly after percutaneous coronary intervention many people experience some chest discomfort or a sensation of leaking heart but these all are false senses. If your dad is experiencing some chest pain like the ones he experienced before that would be an issue. I suggest my patients the following: to be adherent to the anti-ischemic medications like Aspirin, Plavix, etc., and avoid any stress or over exertion.Conclusion: I suggest you not to be alarmed by such simple complaint. Stick to medications. Only chest pain, sweating, and nausea are the red alarm complaint that you should go the ER.Hope I have answered your query. Let me know if I can assist you further.Regards,Dr. Mahmmad Gamal"
},
{
"id": 76709,
"tgt": "Is a fluid aspiration from a lump painful?",
"src": "Patient: hi i ave tb i on medication but now i found a lump in the back of my neckjust at the top of my spine will they take fuilod from it because i dont want them to i had it done on my lung and it was very painful thank u my age is 54 5 foot 8 ins and iam a diabetic Doctor: Thanks for your question on Healthcare Magic. I can understand your concern. Yes, fluid aspiration from neck abscess is must for the diagnosis of type of infection. Since you are diabetic, this kind of purulent infection can be seen with mixed infection like aerobic, anaerobic bacteria with tuberculosis. So abscess fluid (pus) culture and sensitivity report is needed.Culture will isolate the causative organism and sensitivity report will guide about effective antibiotic treatment. For this we need abscess material and it is available only after aspiration. It is not much painful procedure. Intensity of pain will be similar to simple needle prick. If you really afraid of pain then ask your use local anesthetic agent (lignocaine) before the procedure. Don't worry, this won't cause pain. Hope I have solved your query. I will be happy to help you further. Wish you good health. Thanks."
},
{
"id": 211031,
"tgt": "Should i ask the doctor for rx of xanax for depression and panic attacks?",
"src": "Patient: Hello! I have an intake tomorrow with a psychiatry. I'm on amitrytyline for my depression ,also before I went to prison in Philly,pa I was on Xanax for my panic attacks and real bad anxiaety should I ask the dr for a rx of Xanax? Which is strongest 1 mg or 2 mg? Doctor: Greetings !Well yes you may ask your doctor for prescription of xanax at dose of 1 mg to be used as a PRN that is for as and when required basis at the time of a panic attack.Although 2 mg dose is stronger than 1 mg dose but in combination with amitriptyline 1 mg dose PRN shall make for a more plausible solution for your panic attacks.I hope this helps.Regards"
},
{
"id": 53307,
"tgt": "What causes the liver enzymes to go high?",
"src": "Patient: My son had recently had a blood test, he is 13, and his liver enzymes were high. He is on two type of mood medications, the doctor said that these medications are not responsible. We were told it is possibly \"Gilbert syndrome\". He also has dry patches on his back, toes and above his elbows. Doctor: liver enzymes are elevated because of gilbert syndrome. consult a gastroenterologist for the treatment of gilbert syndrome"
},
{
"id": 222047,
"tgt": "What are the symptoms of pregnancy?",
"src": "Patient: My period is 4 days late and I usually am regular. My lower back has been hurting, I have been exhausted and tired more than usual, my boobs are sore, and the foods I used to love I don t want to bother with anymore. My boobs are not as sore as they were but still. Do you think I could be pregnant or should I just wait it out? Doctor: Dearyou have missed your periods by 4 days.Symptomps you are having can be due to other reasons also like some deficiency of vitamins and iron and calcium.why not go for urine for pregnancy test and become tension free dear?all the bestDr.Mira Butani"
},
{
"id": 203051,
"tgt": "What are the treatments for erection problems?",
"src": "Patient: hi. I recently got married. my age is 34. with no illness. no smoking. I am in sexual relationship everyday for max 1 hr. but that does not involve intercourse. my penis erection last for 2-3 min.after that it loses it hardness. when I am about to enter vagina it already flaccid. no ejaculation happen. this makes no entry in vagina. if erect ejaculation happens within 2-3 thrust but not strong erection to enter inside. Doctor: Hello,,Erection problem involves the brain, hormones, muscles and blood vessels,problem with any of these can affect the normal functioning of the penis.Most common causes are fatigue, stress, relationship issues and alcohol consumption. The morbidity like heart disease, diabetes, obesity and multiple sclerosis can cause erection problem.I would suggest you to follow some remedies like lifestyle changes like exercising, losing weight and curbing alcohol intake if you are a alcholoic. Medicines like Viagra, Cialis and Levitra can be used which increase blood flow to the penis and sustains the erection of penis.But these medicines should be taken only after consulting your doctor.Thank you."
},
{
"id": 140560,
"tgt": "What causes episodes of pass outs, nausea and weakness?",
"src": "Patient: I have been feeling extremely exhausted and having episodes where I feel like I m gonna black out usually when I m walking. I also have been feeling really weak and have had a headache for three or four days now. I find it hard to find words at times. And feel nauseous but haven t vomited. I m taking my medications regularly. Just feel horrible. Doctor: Hello, You need a detailed neurological examination. This would be followed up by a brain scan to determine the cause of headache and word finding difficulty. Hope I have answered your query. Let me know if I can assist you further. Take care Regards, Dr Sudhir Kumar, Neurologist"
},
{
"id": 193847,
"tgt": "What is the treatment for phimosis?",
"src": "Patient: I'm am a 21 year old boy suffering from phimosis. i started noticing the condition this condition around 4 years back before which i had a normal functioning penis with full retraction of the foreskin. The foreskin started to get tighter as i grew up and now i'm unable to retract my foreskin. The tightening has stopped now. Last year i consulted a local general physician who confirmed that it was phimosis indeed. Is it possible to cure this condition without any surgical treatments like circumcision? Doctor: Hi, Need for circumcision depends on the severity of the phimosis. If it started for 1 year you need to get a direct check-up with a urologist and rule out diabetes or infection. Hope I have answered your query. Let me know if I can assist you further. Take care Regards, Dr B. Radhakrishnan. Nair, OBGYN"
},
{
"id": 57224,
"tgt": "Any suggestion for developing extended spleen and liver with vomiting and diarrhea after eating?",
"src": "Patient: I have a child one years old . after start eating he develop extended spleen & liver with vomitting & diahrea. he wad diagnosed as PKU Phe ( 500) . we stopped protein in his food but still have diahrea & vomitting frequently we teat fo Gucher disease & it is negative. We hope we can have your opinion Doctor: HIThank for asking to HCMI really appreciate your concern and as long as the diarrhea is concern I would say that 4 to 5 frequency of passing stool in day would be normal only hydration is matter, the live and spleen may be normal one, at the most stool examination may be the option to rule out some disease of infection, take care and have a nice day."
},
{
"id": 179532,
"tgt": "Is sweating normal in a child while being on Clavam 625?",
"src": "Patient: Doctor has prescribed Clavam 625 for my son aged 10 yrs, weighing 41 kgs, twice a day, for his throat infection. He is out of fever and seemingly normal but temperature is below 98 and he is sweating a lot. Is this due to the medicine? Is it ok to be so? Doctor: Hi...it is not due to the antibiotic clavam. Its probably his fever which is coming down now. Do nor worry if he is active and talking well even though he's sweating a lot. I hope a throat swab culture or a rapid antigen testing has been done before starting an antibiotic.Regards - Dr. Sumanth"
},
{
"id": 156776,
"tgt": "Had cervical cancer. Is it possible for painful nodes to be a recurrence?",
"src": "Patient: I had cervical cancer stage 1b1, adenosquamous carcinoma in 2010. I now have back pain for one year and swollen painful lymph nodes in groin for 2 days. I have heard that painful nodes are usually not related to cancer. Is it possible for painful nodes to be a recurrence? Doctor: ya it is possible to have recurrence of cancer. but biopsy is required to confirm the cancer, as in almost 50% of cases infections are also cause of enlargement of enalarged groin lymphnodes"
},
{
"id": 158318,
"tgt": "Had robotic surgery for endometrial cancer, getting a second opinion on treatment. How long is safe to wait to start treatment?",
"src": "Patient: 4 weeks post op robotic surgery for endometrial cancer ..everything removed..13 nodes removed and negative, pelvic wash negative, all stuff negative except cervix , contained in uterous but vascular invasion. Am getting a second opinion on treatment. How long is safe to wait to start treatment? Still have bit of swollen belly and some sharp pain in lower left pelvic area a d moving slow but better..I want this second opinion..should have within next 10 days..but long can I wait to start treatment and be safe!! Do not wa t to wait too lo g! Tnanks Doctor: Hi For second opinion I need your histopathalogy reports,, chemotherapy depends upon the gross look of lesion during intra-op. & also gross look of dissected organ or a part of organ having lesion, more over hystopatholgy report also extremely needed because from this it can be decided the stage and nature of carcinoma, and prognosis, in my opinion you better discus this with your onco-surgeon or chemotherapy physician they are the good judge, Get well soon be in touch with HCM."
},
{
"id": 171270,
"tgt": "What are the black crusts on the head of a month old baby?",
"src": "Patient: My 4wk old baby boy has tiny black specs on his head you can barely see them. If you scratch on it sometimes it will come off but they are usually really hard to get off. I use a brush when I wash his head and sometimes the more rough side of the baby wash clothes to make sure it s clean but they are still there. Sometimes I find them on his neck and today there are two or three on his cheek. Doctor: Hi.... by what you quote I think that your kid is having scalp seborrhea. This could be because of a mild fungal infection. But usually skin conditions are best diagnosed after directly seeing them. So I suggest that you upload the images of the skull of your kid with the black heads. In this way we will be in a better position to help your kid with accurate diagnosis. The other suggestion is to consult a paediatric Dermatologist regarding this.Regards - Dr. Sumanth"
},
{
"id": 182860,
"tgt": "Suggest treatment for tiny little bumps on tongue",
"src": "Patient: I have tiny bumps all over my tongue. After using the tweezers and pulling one off, it looked like a white head. This has been coming and going for over a year. My Dermatolagist said it looked like \"Geographic Tongue\", but that's before I pulled one off and it looked like a white head. What do you think? Doctor: Thanks for using health Care Magic.Read your query.The white patches described suggests a geographic tongue which is because of depappillation if the tongue. It may sometimes cause tongue discomfort and sensitivity to certain foods.It can occur sometimes due to nutritional deficiency ,anemia or stress.It doesn't usually require any treatment as it reduces on it own .Avoid using hot, spicy and salty foods for a while.If still very persistent ,consult your oral surgeon to rule out erosive lichenplanus.Hope this was helpful.Thanks and regards."
},
{
"id": 226050,
"tgt": "Taking birth control pills. Had unprotected sex. Safe? Should I go for second day pill?",
"src": "Patient: Hello.I have a question about plan B pill. My first day of my period was 18 April.I was taking birth control pills but i stopped them on 9th of March.Last night i had unprotected sex and i would like to know if i am safe or i should get the second day pill. And is it safe to start taking birth control pills again after such a short time? Thank you for your help. Doctor: As you have stooped taking your pills long ago, you are totally unsafe. you should get an emergency contraceptive. Yes you can again start taking birth control pills."
},
{
"id": 12988,
"tgt": "What causes itchy, recurring rashes below belly button?",
"src": "Patient: rashi have a rash right below my belly button thats been there for two years itches all the time it wontt ggo away tried lotions an creams . it goes awaay then it just comes back the itch is unbearable now its got this weird smell like peanut oil any ideas Doctor: Hi,It may be some allergy. ...contact allergic dermatitis. Consult the dermatologist for the perfect diagnosis and proper treatment. I would recommend you to take a course of steroid in tapering dose. Take antihistaminics like cetirizine 10 mg daily.Apply steroid cream. Avoid contact with suspected allergens.Hope this helps.Dr.Ilyas Patel,Dermatologist"
},
{
"id": 52550,
"tgt": "What does this blood report indicate?",
"src": "Patient: Dear Doctor, my Blood report has following value for Liver Function. ASPARTATE AMINOTRANSFERASE (SGOT ) PHOTOMETRY 39.1 U/l ALANINE TRANSAMINASE (SGPT) PHOTOMETRY 60.6 U/l GAMMA GLUTAMYL TRANSFERASE (GGT) PHOTOMETRY 97.4 U/l Please advice as what precaution needs to be taken. Mehul Doctor: Hello and Welcome to \u2018Ask A Doctor\u2019 service. I have reviewed your query and here is my advice. The given liver function test values are almost normal. GGT is slightly on higher side. Nothing much to worry and no treatment is required as of now. You can repeat the tests after a week to reassess. As of now drink plenty of water and take rest. Hope I have answered your query. Let me know if I can assist you further."
},
{
"id": 51645,
"tgt": "Why is my creatine level high post kidney transplant ?",
"src": "Patient: Hello Dr. I m post Kidney transplanet(mar-2009), from past 4 months, creatine leave has been increaed from 1.3 to 1.7, i m on pangraph 0.5 mg on night,720mg & 540 mg of myfortic, wyslone 10 mg, with BP tables, Can u please help y these creatine may go up..? and chance to bring it down to 1.3.? Doctor: Hello. Thanks for writing to us. The cause of rise of creatinine needs to be seen. Please visit your physician to undergo physical check up and also to undergo relevant investigations required in your case. If the medicines you are taking already are not effective, increase in dosage may be required after consulting your physician. Try to avoid having pulses. I hope this information has been both informative and helpful for you. Regards, Dr. Rakhi Tayal drtayalrakhi@gmail.com"
},
{
"id": 117626,
"tgt": "Is flying advisable with repeated haematoma?",
"src": "Patient: I have recently been in hospital after bursting a blood vessel and causing a hematoma. I had a severe bout of coughing which caused the burst blood vessl. The doctor said the hematoma would eventually be absorbed into my body and no further treatment was necessary. I am now being treated for the chest infection which caused the cough. What i would like to know is, will it be okay to fly in september , this is the second time this has happened to me . Doctor: Hi,Thanks for asking.Based on your clinical history and query, my opinion is as follows:1. Hematoma is due to bursting of blood vessel and is minor.2. As there is no other risk factor other than coughing and you are being treated for it, there is no risk due to flying.3. Nothing to worry, you hematoma should be small one and it will get absorbed in a month.Hope it helps.Any further queries, happy to help again."
},
{
"id": 10638,
"tgt": "Can taking Jack3d workout supplement cause hair loss?",
"src": "Patient: i am taking a pre workout supplement called Jack3d. I have recently noticed a slight receding hairline and i am only 19 years old. I have taken the supplement for approx 1 month and have only recently begun noticing the loss. could this be caused by taking jack3d? Doctor: Hi.As per your case history of hairfall.Using such supplement will mostly not cause hairfall. So you can continue it.My treatment advice is \u2013 1. Take good nutritious diet full of green leafy vegetables and milk.2. Use a good herbal shampoo and coconut hair oil for regular use.3. Take an iron supplement once daily and vitamin b12 supplement once daily for 3 months.4. Other treatment options are topical minoxidil, oral finasteride and mesotherpy done by a dermatologist.Thanks.Dr.Harshit Bhachech.MBBS, DDVL."
},
{
"id": 215549,
"tgt": "What causes intermittent pain in the groin area?",
"src": "Patient: I am having intermittent pain from the top of my groin down to my knee. I will be walking and all of a sudden it begins to burn and I have pain and it gets very weak from the top of my thigh to my knee and I have to make it to somewhere to sit for a minute or 2 and then it usually goes away and I can walk ok although it does feel weak for a little while. Doctor: Hello, As a first line management you can take analgesics like paracetamol or aceclofenac for pain relief. If symptoms persist, it is better to consult a physician and get evaluated. Hope I have answered your query. Let me know if I can assist you further. Regards, Dr. Shinas Hussain, General & Family Physician"
},
{
"id": 69149,
"tgt": "What could be the lump on thigh which is painful when swells?",
"src": "Patient: i have a lump on my inner thigh. It swells up big time.. every couple of days if i choose i can pop it like a pimple. A LOT of nasty stuff will come out. I only have lots of pain when it swells. Its been around for at least 6 monthes. What could it be?> Doctor: Hello!Thank you for the query.This lump is most likely a sebaceous cyst which has gotten infected. As long as it wont be removed with surrounding capsule, the problem will be recurring.That is why I suggest you to consult a surgeon with this issue. Please do not pop it.Hope this will help.Regards."
},
{
"id": 134775,
"tgt": "What is the treatment for a spinal fusion for scoliosis correction?",
"src": "Patient: What type of limitations can be expected for a teenager who has had a spinal fusion for scoliosis correction? My nephew has also 2 steel rods screwed in place on either side of most of his spinal column. His fusion goes from just below neck to almost his waist. Also he has a goretex patch instead of a diaphragm on his left side since just after birth. It was repaired a few years ago. It was and still is wired to his rib cage. He was one of the first to survive this newer technique. I am concerned if trampoline use and sparring are safe activities for him. My brother seems to think anything is fine- I and his grandmother are concerned Doctor: helloGenerally after fusion surgery and harringtons instrumentation, precautions are taken by avoiding jerks and unusual bouncing as the metal struts as well as spine are subject to stress break on unusual sudden vibration and therefore, it may be inadvisable for trampoline or sparrng activities.You may also consult your spine surgeon for activities that are permissible depending on patients general physique and amount of movement his physical shape can permit without danger.Usually muscle strengthening exercisesare advised both active and passive under a trained physiotherapistbest wishes"
},
{
"id": 137419,
"tgt": "Suggest treatment for carotidynia",
"src": "Patient: My husband has chronic carotidynia for 2 months now. Treated with valium, prednisone, pain killers, and neurontin. Also mouth wash to help kill throat pain all to no avail. He is depressed and having difficulity with stability cordination. What is next? He is misserable. Doctor: Hello,I have studied your case and carotidynia is a difficult issue to treat. It is very frustrating for patient some time. I would recommend you to take Gabapin nT one night time to take care of this pain. If this is not showing any effect then it is better that you should plan for surgery. You need to see ENT specialist for this. As sometime it can be migraine due to vascular aneurysm. In that case surgeyr may be needed."
},
{
"id": 136785,
"tgt": "Suggest treatment for shoulder pain radiating to neck and head",
"src": "Patient: Can you help me pls. I am experiencing pain originally from my right shoulder that radiates on my neck and head, Im really bother about this pain, beacause i never get any trauma on either any side of my body particularly my right side. i really have pain on my head, basically same like a migraine.thank you Doctor: Thanks for your query, I have gone through your query.The shoulder pain radiating to neck and head can be secondary to spasm of the neck muscles(torticolis), which might result in pain in the neck, shoulder and head. This can occur because of the abnormal sleeping posture, excessive load on the neck or shoulder muscles. Nothing to be panic, you can take an analgesic like diclofenac with muscle relaxants like chlorzoxazone(if you are not allergic) after consulting your physician. You can give hot fomentation over the muscles.I hope I have answered your query, take care."
},
{
"id": 44526,
"tgt": "Blocked fallopian tubes, RHE-FD, missed periods for 3 days, chances of pregnancy ?",
"src": "Patient: I am 29yrs old married woman i m suffering from infertility 2 minths ago my doctor took my many test like hormones and PCR but all test are normal then my doctor advised me andoscopy then she found that my both fallopian tubes are blocked(b/l corneed blocked)she didnt get any resion because my all test are normal then se advised me 3 months course of RHE-FD ( rifampicin ,isonised\u00f0ambutol hydrochloride tablets along with zeed antioxident, multivitamin and minerals capsule and medoliv liver syrup.for any infection occurs in tube.let me know can these drugs helpful for me ?one more thing i missed my period for 3 days is there any possibility for me for conceiving because i m taking these medicines fornear about 2 months please answer me. i will be very thankful . Doctor: Hello. Thanks for writing to us. With bilateral tubal blockage, the chances of conception are very low. The treatment you are recieving is based on the fact that the commonest cause of tubal blockage is infection. This treatment might help you. You can continue with the treatment advised to you by your gynecolgist. I hope this information has been both informative and helpful for you. Regards, Dr. Rakhi Tayal drrakhitayal@gmail.com"
},
{
"id": 140995,
"tgt": "What causes occssional pain in spina iliaca anterior superior?",
"src": "Patient: Hi, im 22, 1.65m and 62kg. I had two surgeries, first of astrocytoma GII Th7-Th11 in 1997 and second of scoliosis in 2003 but anterior approach. The scar after the second suregery stretches from 1 cm medial from spina iliaca anterior superior diagonally to 1 cm from my spine. Occasionally I have a pain that I would describe as a stabbing 5 cm from spina iliaca anterior superior on linea axilaris anterior, usually its 5-6 cramps in half an hour and then go and it happens once in hmm 2 months. But last was today and i have pain every 5 sec for hours. What that could be??? Doctor: Hi, Seems like a pinched nerve is causing your symptoms. Since you had scoliosis surgery, in rare cases, a hardware failure may result in such symptoms. I think that there is a need for an evaluation by your surgeon and by image studies of your spine in order to understand better your condition. Painkillers may help in reducing symptoms. Hope I have answered your query. Let me know if I can assist you further. Regards, Dr. Erion Spaho, Neurologist, Surgical"
},
{
"id": 5161,
"tgt": "History of irregular periods. All test seems normal. Want to conceive. Will B long F help to have regular periods?",
"src": "Patient: Hello doctor, i m 28 yrs married woman from India, I had a history of irregular periods. although my hormones test & ultrasound reports come normal. As we are start trying to have baby and my periods history is from march is 27th march to 31st march in april no periods & negative pregnancy test so I go visit a doctor she advised me to have Rablet20 for five days& folic acid tablets for one month after having this medicine I had periods on 18th may apart from that she told me to go to different test like TSH,USG,,Insulin, Procatin & LH all the test are normal still I have irregular periods. After this I go to visit the doctor she advise me to have B Long F for three months .Is this tablet help me to have regular periods. Please advise me I would like to conceive & Doctor told me to track the menstrual cycle to get pregnant . Doctor: Hi,The cause of irregular periods while the hormonal assays and the sonogram findings are normal, could have a few possibilities. It could be due to irregular ovulation, it could be due to weight disturbances or stress and it could be constitutional. You should try to have some regular daily exercise, maintain a healthy body weight, eat plenty of fresh fruits and vegetables and follow your doctor's instructions and maintain a menstrual calender.The medication is a nutritional supplement and can be used as a prenatal vitamin. Hope your query is answered. Good luck."
},
{
"id": 100455,
"tgt": "What is the reason to get sick and dizzy only after eating pork ribs and not other pork items?",
"src": "Patient: I am a nurse for over 30 years and recently noticed that every time I eat \"pork ribs\" I feel sick and dizzy the next day. Hence, when I eat any other pork items, I don't experience the same effect. Looking for information or literature concerning this issue. Thanks. Doctor: Dear Sir/ Madam,Thank you for posting your query on healthcaremagic.comNormally ribs contains more protein and blood extract. This could be the reason for dizzy spells.I advice you to avoid eating ribs of pork.You may consult dietician to get better advice and dietician can help with other alternative.Hope I have answered your query. Please revert back to me if you have any further question. I will be happy to assist you.With best wishesTehzib Saiyed (PT)"
},
{
"id": 208874,
"tgt": "Can paranoid personality disorder be associated with lax ligaments?",
"src": "Patient: how the hell is paranoid personality disorder or almost any other mental health disorder associated with lax ligaments and hypermobile joints? (you need to update your site s search results). Also, aside from Marfan syndrome and Ehlers-Danlos syndrome, are there any other genetic conditions where the main symptom is joint laxity to where turning an ankle 90 degrees the wrong way doesn t result in a sprain. Doctor: Hello,Thanks for choosing health care magic for posting your query.I have gone through your question in detail and I can understand what you are going through.No such relationship exists..Hope I am able to answer your concerns.If you have any further query, I would be glad to help you.In future if you wish to contact me directly, you can use the below mentioned link:bit.ly/dr-srikanth-reddy\u00a0\u00a0\u00a0\u00a0\u00a0\u00a0\u00a0\u00a0\u00a0\u00a0\u00a0\u00a0\u00a0\u00a0\u00a0\u00a0\u00a0\u00a0\u00a0\u00a0\u00a0\u00a0\u00a0\u00a0\u00a0\u00a0\u00a0\u00a0\u00a0\u00a0\u00a0\u00a0\u00a0\u00a0\u00a0\u00a0\u00a0\u00a0\u00a0\u00a0"
},
{
"id": 42770,
"tgt": "Does irregular menstruation cause infertility?",
"src": "Patient: hi,i am 28 yrs old single girl, i have problem with irregular menstruation and i am worried about is there will be any problem to conceive?also,i am having breast tenderness and pain before my periods. and i am always embarrassed about my small breast as i am 28yrs.so please provide me good suggestion accordingly. Doctor: Hi,Thanks for writing to HCM .Yes irregular cycles cause infertility. So I suggest you following investigations to know the cause of irregular cycles and take treatment accordingly.-- thyroid profile, serum prolactin, ultrasound abdomen to rule out polycystic ovary disease (pcos).The pain in breast during cycles is seen in some people. Don't worry for that. You can take primrose tablet after consulting your doctor and pain killer.Normally breast develop in proportionate to body . If you still feel your breasts are small compared to your body then for external appearance you can ware thick padded bra.For your sexual life, then you can go for breast enlargement surgery by a plastic surgeon. Hope I have been helpful .Regards Dr.Deepika Patil"
},
{
"id": 223585,
"tgt": "Can taking Yasmin tablet effect the menstrual cycle?",
"src": "Patient: hi I took yasmin tablets on Thursday 21st after having intercourse and still continuing but wasn t sure if it would have worked so I took postinor 2 sd on Friday 22nd and my periods is supposed to come on the 30th what can I do to make sure that it comes on time and should I stop taking the yasmin? Doctor: Hello,I have gone through your query and understood the concern. As you have taken the emergency contraceptive pill soon after the intercourse, you are amply protected. Also, if you are in the safe period of your menstrual cycle, you are naturally protected. The use of the regular hormone contraception can be continued if you wish in future, but, there is no need in this cycle if you are using it only as prevention from that act. Your next natural period can get delayed by a week or two since the emergency pill contains excess hormone. Please see a doctor if you become overdue by 10 days. Hope this helps. Let me know if I can assist you further. Regards,Dr. Shanti Vennam"
},
{
"id": 150435,
"tgt": "85 years old. Diagnosed encephalomalacia. Histroy of brain damage. Recommendations?",
"src": "Patient: My sister suffered severe brain damage at age 26 ( major car wreck, head crushed, lost one eye and has a steel plate in her head and in a coma for 3 and 1/2 months and was not supposed to live.) and now is 85 yrs old. Her Dr's report says there is a very large area of encephalomalacia occupying most of the left hemisphere in the distributions of the left ACA AND MCA. A smaller area of encephalomalacia is present within the superior right frontal lobe.Healed right frontal craniotomy defect is present. I am and have been caring for her for years, she is very high maintenance. What does this all mean as for as brain damage?Her memory is fair; however, her logic is very poor to non existent. Doctor: Hi, Encephalomalacia refers to loss of normal brain tissue which is replaced by scar tissue which does not do the function of brain cells. Brain cells once dead do not normally regenerate and usually when one area of the brain is damaged, over a period of time the adjacent parts of the brain take over the functions lost. However, in your sister's case, the loss of brain tissue is very severe and one half of the brain has already nearly lost complete function. The other half is also affected. Overall, it is unlikely that her mental functions would improve a great deal at her age. Sincerely, Vamshi"
},
{
"id": 50304,
"tgt": "Cysts in both kidneys. Stomach bulging. Suggestions?",
"src": "Patient: my douther has a cyst in kidney i was worried a lot here is the findings: There are two cysts in right kidneys;2.5x2.5x3.2 cm cyst at interpolar region with thin internal septation...0.6x0.8x0.9 cm at lower pole;simple cyst. there are three cyst in left kidneys. 2.5x4.9x3.3 cm cyst at lower pole with thin internal septation.- 0.8x1.1x0.7 cm cyst at lower pole;simple cyst.- 1.2x1.1 cm cyst at interpolar region;simple cyst...that is the findings of ospital. and she havent got her monthly period for almost 6 moths.. and all she khows she is fregnant but when she goes for untrasound they found out that she got all this cyst. and now she got big tommy that all she khow she is fregnant. when they do the blood test she is not pregnant. im so worry for my douther.we are nowliving here in thailand ..but we are from philippines and the problem here they cannot explain well to us the situation they told us to wait for 1 year. here is my e-mail add. YYYY@YYYY im hoping that you can help me with this .. thank you very much. Doctor: Hiyour daughter seems to be suffering from polycystic kidney disease.you have not mentioned about her age,any High BP, any family history of kidney disease, exact dimensions of kidney on sonography, renal function tests if done.She needs to be tested for renal function tests,urine analysis, and for presence of cysts in other organs.This may be a herediatery condition or sporadic one.Her abdominal distension and irregular menses can be related to the kidney problem.You need to consult a Nephrologist.Take care"
},
{
"id": 90035,
"tgt": "What causes pain in abdomen under ribs?",
"src": "Patient: Hello. I feel extreme pain in my abdomen just under the rib towards the left of my body. i could describe this pain as A heavy amount of pressure In abdomen a twisting wrenching feeling in the gut. As well my food seems to take much longer to digest, My stool is always in a mucousy state, i go the bathroom 30-35 times per day and i dont seem to be so interested in food. What could this be? This all began 3 years ago after a bout of anorexia and have gottwn nothing but worse. Doctor: welcome to Health care magic.1.It seems like you need a medical care as soon as possible.2.30-35 times passing stools is not at all normal.3.you are giving a history of 3 years ( hope you are on some medication)4.If you are my patin t i would ask for a stool sample ( to rule out any parasite or presence of any bleed ).5.Your pain/feeling on your left side is due to the underlying bowel symptoms i think.6.Get an abdomen pelvis ultrasound get done to rule out few causes.Anything to ask ? do not hesitate. Thank you."
},
{
"id": 41109,
"tgt": "What are chances of healthy conception with morphology issues and polycystic ovaries?",
"src": "Patient: my husband has a morphology issue and i have polycystic ovaries...i had 1 iui and it wasnt sucessful,i was going to start on a second one but i got a weird mid cycle period.that lasted 5 day ...took a pregancy test with digital and blood test in which both were neg..what the chances of my husband and i having a baby? and because of the morpholoy issue will our baby be born with birth defect....my husband is 63 and im 32 Doctor: Hello, no dear there can be a sucessful pregnancy and the morphology issues result in implantation failures and not birth defectsIn case you have any questions in future you can contact me directly on http://bit.ly/drmanishajain"
},
{
"id": 146090,
"tgt": "What causes headache along with blurred vision?",
"src": "Patient: My husband I'd 55 years old, 6'3, 254 lbs. he is normally in fairly good health.Today he is complaining of a severe head ache, blurry vision, no nauseous, BP 138/77, HR 97 HR 85I am to 55, but have poor health. ,I also woke up with a stiff neck, fingers numb and severe pain in my right lower lung. No fever. I am an asthmatic and migraines. I am concern with the sever neck n stiff neck and the Lower kidney/ lung area. Past a stone 5 was ago. Please give me some suggestions and your opinion would be so much appreciated.Thank you Doctor: Hello thanks for asking from HCMHe is 55 years old and have normal BP, HR but having severe headache with blurred vision. I would advise you to get his vision test done for any refractive errors as poor vision may present with such symptoms. Such headache could occur without any definitive cause and a dose of OTC analgesic helps to reduce headache. But if it was a repetitive headache with blurred vision then it needs evaluation. It could occur due to Migraine or Cluster Headache. Both type of headaches present with severe headache and blurred vision. I would advise you to take him to a neuropsychiatrist for help. Medicines like Amitryptiline, Sodium Valproate etc have excellent results in containing such headache. Drugs like Propranolol can be used for prophylaxis of such headache.You have complaints of stiffness of neck, numb fingers and pain in lungs. Stiffness of neck and numbness can occur due to some spondylitic changes in your age, though proper evaluation and examination is needed to make exact diagnosis. I would advise you to consult a physician for pain in lungs and your other complaints.Thanks, hope this helps you."
},
{
"id": 195620,
"tgt": "What causes tiredness and drowsiness?",
"src": "Patient: I am 62. Male. Weight 13 stone. Height 6ft 2 ins. No medical conditions. I feel tired from 16.00hrs each day. I fall asleep in front of the TV. I wake every morning at 4.30. I am drinking twice as much fluid per day as 3 months ago. I do not smoke. I drink 10-14 units of alcohol per week.I take regular exercise. I feel blood tests might help. What do you think? Doctor: Hello and Welcome to \u2018Ask A Doctor\u2019 service. I have reviewed your query and here is my advice. It is good t know that you have no medical conditions at present. But if you feel some changes like tiredness or sleepiness then surely you should go for a thorough medical check up to rule out any developing problem. Some medical conditions like thyroid disease, heart disease, diabetes, sleep disorders may cause fatigue or tiredness. These diseases are easily treatable with a few medications. Now, I don't say that you may have some of these but at your age it is good to go for at least thyroid tests & Cardiac check up. Some developing problem can be ruled out at an early stage & you can will be feeling fine very soon. Stay away from addictions & eat healthy. Hope I have answered your query. Let me know if I can assist you further."
},
{
"id": 205760,
"tgt": "Suggest treatment for thanatophobia",
"src": "Patient: Hello Doctor Hello .....My Name is Naresh ....I am 32 Yr old i jhave death phobia and taking Feliz-S-20 sinece last 8 month i just want to know what is side effect of this dose and how long it is ok to contunue my doctor said it will continue for 2 years Doctor: DearWe understand your concernsI went through your details. Death phobia is not morbid and it just categorized under anxiety disorders. I think you also should be having health anxiety disorder. Your doctor is almost correct. It takes time for a cure provided you are doing what he advises. You also should be trying to get yourself right. Psychotherapy techniques, meditation and exercise are of immense help. A good psychologist should be able to provide you the necessary treatment. You should combine these three treatment methods with the medicine. Talk to your psychiatrist about this option.If you still need my assistance in this regard, please use this link. http://goo.gl/aYW2pR. Please remember to describe the whole problem with full detail.Hope this answers your query. Available for further clarifications.Good luck."
},
{
"id": 206046,
"tgt": "How long does it take for Loritab to be out of the system?",
"src": "Patient: if you been on tramadol 50 mg 2 pills twice a day for about 4 years and loritab 7.5 1 pill a day for 30 days presribed by your doctor. And don t have refills and going off cold turkey. How long does it take to get out of your system. Just very nervous, shaky, lil depressed, some nausea and diarrehea. Doctor: Hello,Thanks for choosing health care magic for posting your query.I have gone through your question in detail and I can understand what you are going through.It may take 4-5 days for the withdrawal to settle down. Hope I am able to answer your concerns.If you have any further query, I would be glad to help you.In future if you wish to contact me directly, you can use the below mentioned link:bit.ly/dr-srikanth-reddy\u00a0\u00a0\u00a0\u00a0\u00a0\u00a0\u00a0\u00a0\u00a0\u00a0\u00a0\u00a0\u00a0\u00a0\u00a0\u00a0\u00a0\u00a0\u00a0\u00a0\u00a0\u00a0\u00a0\u00a0\u00a0\u00a0\u00a0\u00a0\u00a0\u00a0\u00a0\u00a0\u00a0\u00a0\u00a0\u00a0\u00a0\u00a0\u00a0\u00a0"
},
{
"id": 202554,
"tgt": "How to have a smooth insertion while intercourse?",
"src": "Patient: i am 35 male divorced.i went for a second marrage.she is 30.i am slightly over weight.during sex i could not insert fully.she was very tight. i tried lubricants.but i was not able to insert into vagina.please help.i want a happy life.my penis is big and fat.please help me.my wife feels a lot Doctor: HelloThanks for your query,based on the facts that you have posted it appears that you have normal hard erection but unable to penetrate vagina during intercourse .Your wife has severe pain in her vagina.This is most likely to be due to what is called as Vaginismus.Please consult and get her examined by a qualified Gynaecologist.She may need to dilate her vagina under short general anaesthesia..Dr.Patil"
},
{
"id": 1421,
"tgt": "Are there pregnancy chances with non penetrative sex?",
"src": "Patient: me and my girlfriend had intense foreplay yesterday and we were without clothes we want to if sperms not ejacuated inside vagina may be splill on hands and bedsheet could make her pregnant? also she got her periods on 13th last month till this time she did not get til now,so is there chance of preganancy? Doctor: Hi, there can be a chance of pregnancy if by any chance sperms came in contact with vagina. If her periods have delayed, do a urine pregnancy test at home. If positive consult a doctor if you don't want pregnancy. If negative wait for your periods. Hope I have answered your question. Regards Dr khushboo"
},
{
"id": 102874,
"tgt": "Have blistery rash on hands and feet. Performed allergy skin and food allergy testing. What are the findings?",
"src": "Patient: I have has a blistery rash on my hands and feet that comes and goes in a 3week cycle. I have had allergy skin testing and food allergy testing which came out no allergy. The allergist sent me to a derm...the derm took two biopsy samples...results today showed neutrophil cells were present so she wanted to take another biopsy to determine what it is. My question is what all is she looking for? She mentioned ruling out lupus with a blood test in the future. Doctor: neutrophils are found in acute infections and the cause can be any like bacterial viral or fungus and they are trying to find the disease related to treat in my opinion"
},
{
"id": 124542,
"tgt": "Feeling excessively tired & having back pain,headache & breathing issues",
"src": "Patient: I have been feeling excesivly tired and have been havig alot of neck and upper back pain accompanied with headachs and my breathing seems a little harder i am only 21 and i am a smoker, i am also wonderig if this could be caused by mould in my apartment? I have not seen any signs but it is an older building. Doctor: Hello, It is nothing to do with an old house or mould. You need to stop smoking as it is causing weakness, also take good nutritional diet and also check for your hemoglobin, vitamin D, and B12 level. If there is a low level of vitamins then it can cause weakness also. Hope I have answered your query. Let me know if I can assist you further. Regards, Dr. Naveen Kumar Sharma, Orthopaedic Surgeon, Joint Replacement"
},
{
"id": 88693,
"tgt": "What causes discomfort in the stomach during mornings?",
"src": "Patient: Most mornings I wake up and have a sick feeling in the pit of my stomach. I sushi feel quite tired these mornings, more than usual. Sometimes I will make myself sick to feel better but this is not often. I do not have breakfast in the mornings as I do not feel hungry until about 10am. I am not pregnant either. Any thoughts? Doctor: Hi,It seems that you night be having hyper acidity type of pain as in the morning stomach is empty and acid secretion continues.Immediately after getting up take some thing like milk, biscuits etc.This will dilute acid secretion will give you relief.Many a time due to hypoglycemia developed as there are 8-10 hours passed taking dinner.Take some sugar, tea, fruit juices.Avoid fried and junk food. Ok and take care."
},
{
"id": 57693,
"tgt": "What is the remedy for digestion and liver problems with weakness after sex?",
"src": "Patient: Helloo sir..:-) sir i have a week liver prblm..when i take strongs diet like milk etc nd other heavy diets..my liver cant consme diets. ..i have a loose motion after these diets...nd some time vomting..my internaly body is very week..nd i have a problem un sex..means weekness ...so plz give me some good tips to cover my liver prblm nd sex weekness..tell me some good diets to cover sex weekness.. Doctor: Dear Friend.Hi , I am Dr Anshul Varshney , I have read your query in detail , I understand your concern.I feel you have diagnosed yourself about weak liver.If you really feel so , get a Liver Function Test done. The symptoms you are describing can be because of any of the following problems :1. Acid Peptic Disease2. Gastro Esophageal Reflux Disease3. Lactose Intolerance4. Celiac Disease.You should get yourself evaluated.Weakness feeling regarding sex is just Psycogenic.Stop worrying about that.This is my personal opinion based on details available here. If you want to discuss your issues further, you may please ask usStay Healthy. Dr Anshul Varshney , MD"
},
{
"id": 219205,
"tgt": "Can pregnancy cause lower back pain and abdominal pain?",
"src": "Patient: Hello I have a headaches which I have been having for weeks, lower back pain, lower abdominal pain that comes and goes now butn it was constant a few weeks ago, feeling of nausea, sleepy all time, my menstrual just started the 26th and it s gone now I too pregnancy tests but they all came out negative blood and all. I had the same problem with my son didn t get a ppositive test until I was three months pregnant could this be the case here? Doctor: Hello, and I hope I can help you today.Pregnancy symptoms can vary widely between women, and many of the symptoms that are associated with pregnancy, like fatigue, headaches, bloating and spotting can also occur with an irregular period.The most common sign of pregnancy is a missed menstrual period.So if you just had your period, and your pregnancy test is negative, then you are not pregnant.It is not possible to have a pregnancy test only become positive at three months, because the developing pregnancy is what produces the hormone in the first place. So you cannot have a growing fetus without a positive pregnancy test.I hope I was able to answer your question today and that this information clarifies things for you.Best wishes,Dr. Brown"
},
{
"id": 25014,
"tgt": "Suggest treatment for high BP",
"src": "Patient: i have had high blood presure for 7 years, sometimes spiking 230 over 115. Sometimes under control. Previiously bilateral strokes found in cerebellum. Yesterday am about 6a Bp 168 / 98 p,76. I couldn use my fingers hardly rt hand and went to er. dr said it was axiety and tetany, gave me a referral and ativan. Today I cant type this without typing i over and over, and i cant pay the piano. dr took cat scan said was no stroke plus old strokes didnt show up, can this still be tetany? Will it go away? Doctor: Hello ,Whenever you are anxious , you tend to hyperventilate causing transient decrease in calcium resulting in tetany like symptoms. During such spells just relax and try taking deep and normal breathes or else rebreathe in a socks . This would help you control the spell . However the blood pressure range you have mentioned is surely on the higher side and treatment for the same is required, on consultation with your doctor you may titrate the dose of your blood pressure medicine or consider starting one .Lastly always remember that anxiety per say will push your blood pressure up and calming techniques are best in such situations. Hoping i could help . Regards Dr Priyank Mody"
},
{
"id": 57239,
"tgt": "Why am I having high SGPT and SGOT values?",
"src": "Patient: goodevening.. for the last 6 months i have high levels of sgpt (400) and sgot (200). i have been tested for hepatitis but i don't have. they tell me to do a liver biopsy so as diagnose if i have an autoimmune hepatitis. do u think this is a good advice to follow? am i in danger having these high values? i am 35 yrs old, 53 kgrs, don't drink, don't take medicine of any kind. thank u for your time..mary Doctor: Hello Thanks for writing to HCMIncreased SGPT indicates liver injury.Increase in SGPT may be due to many reasons like hepatitis,alcohol intake,altered lipid profile,medicines,auto immune causes etc.SGOT is non specific and it increases in many conditions.You need proper evaluation.You need few more investigations like random blood sugar(RBS),renal function test(RFT),lipid profile,GGT level,urine RE/ME.You are rightly advised for biopsy to know the exact etiology.You should avoid red meat and high fat milk products.You should avoid fried and junk food.Medicines should be prescribed after complete investigations.Get well soon.Hope I have answered your question.Take CareDr.Indu Bhushan"
},
{
"id": 151829,
"tgt": "Why do I feel contraction and expansion of the brain while breathing and twitching of eyes ?",
"src": "Patient: With every heart beat it feels like my brain is being squeezed inward, with every breath it feels like my brain is being pushed outwards, it s horribly painful, worse than any headache or migraine i ve had before, my eyes won t stop twitching when I close them and the pain gets worse when I do. I can t sleep, what s wrong with me???? Doctor: Hi Ambert Farrell Welcome to HealthcareMagic. From the description you have given , it appears that you may be having Glaucoma.A condition where the pressure in eyes is increased.I advise you to visit your Optholmologist initially and get evaluated.If needed further investigations can be done to rule out any CNS problem. Wish you speedy recovery. Take care."
},
{
"id": 188109,
"tgt": "What is the reason behind cheek pain, tooth and nose pain with salt like taste persistent in mouth?",
"src": "Patient: I have been having pain on and off for 6 months lasting over a month at a time. Pain changes places but usually starts in the ear. Then teeth start to hurt. Its now moved to the cheek and the side of my nose. Its only occuring on the left hand side. Been to the docs and they have loomed in my ear and theres no infection and my throat doesnt hurt. My neck is tender but nothing swollen. Suddenly had strong salty taste in my mouth for the last 3 days.Particularly bad in the night and I wake up with painful teeth, thought it was bruxism so bouggt a mouth guard but it doesnt help. The pain is driving me nuts. When I get rid of it it just comes back a month later. Doctor: Hi,Thanks for asking the query,This could be due to infection in the tooth.Get an x-ray done of the tooth.A root canal treatment has to be performed.Take a course of antibiotics and analgesics.Take care!"
},
{
"id": 112880,
"tgt": "Have severe back pain. Had two surgeries for spine conjunction. Advised for surgery now. What to do?",
"src": "Patient: My father got two operation at CMC vellore 5yrs back...due to some spine conjuction .One is on cervical spine and after 7 month one in Lumbar spine..After relieving some pain for some yr now he got severe pain in back and now doctor in Odisha suggest to go for an operation which need two plate at LS-4.I need to know whether it is advisable or we should consult other doctor.Thanks..Debadarsini Doctor: Hello. Thanks for writing to us. From the description provided it seems that your father is having a severe degenerative disc disease. The gradual progression of the problem has caused the further compression and the surgeon has suggested a surgical treatment for him. I hope this information has been both informative and helpful for you. You can consult me again directly through my profile URL http://bit.ly/Dr-Praveen-Tayal Regards, Dr. Praveen Tayal drtayal72@gmail.com"
},
{
"id": 76377,
"tgt": "Can depression cause stabbing pain in chest?",
"src": "Patient: Hi, I have sharp stabbing pains originating in my lower abdomin sometimes pelvic region, traveling up to just under my chest. Unsure of what it is, I am 19, 5'4, weigh about 137 lbs, and have depression/anxiety history and am taking sertraline 100 mg. Doctor: Thanks for your question on Healthcare Magic. I can understand your concern. Yes, uncontrolled stress, anxiety and depression can cause similar kind of stabbing abdominal pain. But better to first rule out abdominal pathology for your symptoms. So get done ultrasound examination of abdomen. If this is normal then no need to worry for abdominal pathology. Uncontrolled stress, anxiety and depression are likely cause for this. Sometimes, only drugs are not important. Counselling plays very important role in management of these psychiatric illnesses. So better to consult psychiatrist and get done counselling sessions. Try to identify stressor in your life and start working on it's solution. Don't worry, you will be alright. Avoid stress and tension, be relax and calm. Hope I have solved your query. I will be happy to help you further. Wish you good health. Thanks."
},
{
"id": 166427,
"tgt": "How can diarrhea be treated in a child?",
"src": "Patient: Hello doctor, my daughter is 7 months old. her stool is semisolid and occasionally watery since birth. she s having 3 meals a day along with milk 3 times. she passes stool 3-4 times a day, which ranges from solid to watery. i got her stool tested and its greenish grey, pH is 6 and semigested. i gave her antibiotic for 5 days, but there was no change. i ve stopped wheat products. she had IUGR, now she s not gaining weight properly. what should i do? Sara Doctor: Good morning..!!Babies upto 1 year can pass stools even upyo 15 times a day.Consistency can range from solid, semi-solid to watery.Colour can be anything (worry only if she passes fresh blood).You need to worry only when she becomes dull, not taking feeds, not passing enough urine per day (minimum 6 times) etc. Consult your doctor if any of these red flags are seen.Thank you, have a nice day..!!!"
},
{
"id": 2325,
"tgt": "Any chances of getting pregnant after condom broke off and taking escapelle?",
"src": "Patient: I have a problem. I had sex with my boyfriend yesterday and the condom broke. We realized that immediately, he even says that the condom broke while he was not inside anymore., but I''m scared. The condom broke before he ejaculated but I know that also in this case there is a chance to get pregnant. It was the 12th day of my menstrual cycle. I don''t have a regular 28 days cycle, the last time the distance between the cycles was of 33 days and before that it was of 31 days. Anyway, I took a Escapelle pill 15 or 16 hours after I had sex. I read that within 24 hours the efficiency of the pill is 95% but I still worry. How big are the chances to get pregnant? Doctor: Hello dearI understand your concernsThere is very less chance of pregnancy around 0-5% as you took Escapelle pill with in 15-16 hours.Condom was broken before ejaculation so chance of pregnancy depend on how much amount of precum entered into vagina.But pill can cause earlier period, delayed in period by 1-2 week and excessive bleeding can occur.So do not worry if period will delay by 1-2 week.If period will delay by more than 2week then under urine pregnancy test and or Blood HCG test to confirm pregnancy by consulting gynecologist.Meanwhile avoid stress as stress itself cause delays in period.Drink plenty of water, do regular exercise , eat healthy diet and maintain proper pelvic hygiene.Hope this helps youbest regardsDr. Sagar"
},
{
"id": 186301,
"tgt": "How to get rid of large bumps at the back of tongue?",
"src": "Patient: I have had large bumps at the top back portion of my tongue since March. I felt like I had something on my throat and I looked in the mirror to see. My tongue was brown and I scrubbed it off with a toothbrush. I then noticed what I believe are enlarged circumvallate papillae at the back of my tongue. I did some research and I quit using the Listerine whitening mouthwash. I am not sure how or why I have them or how to get rid of them. In January I went to the dentist and had dental crown procedure and received my temporary crown. When I went back the dentist was not satisfied with the fit and took another mold and put my temporary back on. The same thing occurred on my 2nd visit. It seems the irritation started between my 2nd and 3rd attempt to get my permanent crown put on. I saw my doctor and he have me some lozenges for thrush mid March. It is now May and they are starting to make my throat feel more irritated and dry. It doesn't hurt to swallow. I have also been a smoker for several years. Any ideas on what may have cause this and how do I get rid of it? Doctor: hello thanks for the consult at hcm..yes it is enlarged circumvallate papillae which is called papillitis, u need mutivitamin therapy,,and u also have thrush -- i think u may be need topical antifungal and systemic also,,do salt water gargle, then clean ur tongue with a soft brush..keep ur tongue clean,, do visit ur oral physician and general physician too,, for a check up and treatment,,since it is not coming u may need culture and some blood test too.hope it helps,,tc"
},
{
"id": 219314,
"tgt": "How to determine the date of conception?",
"src": "Patient: i m pregnant but i don t have regular periods i know my last one was on november the 21 oh the 28 and it was not normal it was brown color for 2 days only. i have sex with my husband everyday from november to december the 6 or 7 then he was gone i had sex with this other guy on december the 15, 16,and 17....the doctors say they are going to go with my baby measurements so i m 19 weeks going to 20 weeks on saturday i really want to know when is the day i got pregnant Doctor: Hallow Dear,It is not clear from your history whether you had your last menstrual period (LMP) on 21 or 28 November or from 21st through 28th of November. I will presume your LMP to be 21 November for the current calculations. Also I am presuming your menses to be regular coming after every 30 days. If there are any changes in this, you may then make the modification in the calculations. Egg is released (ovulation) from the ovaries 14 days prior to the next menses. Your expected menstruation would be on 21 December; hence the expected date of ovulation would be 7 December. The egg has life of 24 hours. Hence any sexual intercourse around 6-7 December would be responsible for the pregnancy. Since, it is not clear on which date the baby was 19weeks, it would not be possible to make any calculations based on that. With the available history, it seems you are carrying the child of your husband. If you still have any doubt, the final confirmation can be done by DNA test after the delivery of the baby. I hope this helps you."
},
{
"id": 54857,
"tgt": "What causes jaundiced skin to disappear ?",
"src": "Patient: I have experienced extreme fatigue on a fairly short-lived but regular basis for my entire life, and have often coped through impromptu napping, after which I would typically feel fine. Many years ago, low blood sugar was thought to be the culprit. Today, for the first time ever, I woke to find my skin jaundiced. I was too tired to get up and look at the whites of my eyes in the mirror, and the jaundice has disappeared. Any ideas? Doctor: Hi thanks for asking question...Noted you are complaining yellow skin and jaundice under suspicion...First for jaundice confirmation, do your serum bilirubin estimation with LFT profiles...If no jaundice then no need of worry.For your fatigue do your peripheral smear examination , serum ferritin and vit.b12 Level.Correct anemia with iron tablet and multivitamin tablet....If raised bilirubin then further work up done with consultation of doctor...Take care.Dr.parth"
},
{
"id": 58766,
"tgt": "Leg and arm cramps. Have liver disease, diabetes. Any suggestions?",
"src": "Patient: my husband who is 56 yrs young is suffering most nights with leg cramps and arm cramps they are so bad that sometimes he cant even walk. He legs are also purple. He is in end stage liver disease and is a diabetic. But his sugars have been good. They only happen at night. We have tried everything from magsisum, tonic water, posseuim more calium ( dont know the spelling of the vitamins ) exercise and so on. He just suffers so bad from these cramps. Any suggestion Doctor: Hello! Thanks for putting your query in HCM. I am a Gastroenterologist (DM).Cramps in cirrhosis may be due to Magnesium, Calcium, Zinc, Vitamin E and Potassium deficiency. Get these levels in serum and see if there is any deficiency then supplement it. Especially zincIf these levels are normal then the other cause is reduction of effective circulating volume and transfusion of injection albumin once in a week may also helpI hope he is not smoker, if he is please ask him to avoid smoking.Also see for the peripheral pulses or may be get colour doppler to see arterial and venous flow in blood vessels of legs as they are purpleI hope I have answered your query and this will help you. Remain in touch and get-well soon."
},
{
"id": 184649,
"tgt": "What causes dry cracked lips and sore tongue?",
"src": "Patient: i hav been having dry cracked lips,a sore tongue,difficulty eating or swolling,extreme fatigue,feels as well as a sore throat,either stuffy nose or runny nose,ive juss recently recovered from an absess tooth as well,i take 6 different medications and on the evra patch. Doctor: Thanks for your query, I have gone through your query.The cracked lips and sore tongue could be because of the anemic stomatitis. or it can be because of the herpes infection or recurrrent aphthous ulcers over the tongue.Consult a oral physician and get yourself examined to rule out the above mentioned conditions. You need to get a blood investigation done to rule out anemia.You have to take nutritional supplements for anemia. consume lot of green leafy vegetatbles and fruits. You can also take topical anesthetic and analgesics like anabel gel for the pain. Apply lip balm or Vaseline over the lip.I hope my answer will help you, take care."
},
{
"id": 193375,
"tgt": "What could cause soft, blue bumps on the head of penis?",
"src": "Patient: Hi, may I answer your health queries right now ? Please type your query here...I have three blue bumps on the head of my penis. They are soft and have been there for as long as i can remember. They never hurt and are the colour of a vain in my arm. When i get an erection they get larger. Doctor: Hello, Genital warts are small fleshy growths or bumps that can appear on the shaft, and sometimes head, of the penis or under the foreskin. They're caused by the human papillomavirus (HPV), which is a sexually transmitted infection (STI). As you have no other symptoms like pain, itching or burning, you should not worry too much. You can confirm and have a sigh of relief from a physician or skin specialist in person. Hope I have answered your query. Let me know if I can assist you further. Take care Regards, Dr K. V. Anand, Psychologist"
},
{
"id": 146538,
"tgt": "What causes pain in arms after taking cervical epidural?",
"src": "Patient: I had a cervical epideral today. I felt immediate relief in my neck but before I could even leave the office, I began to have aching in my arms and shoulder blades. They said numbness is normal and some pain but if persists longer than 12 hours to call the office. It s been 4 hours and I m still in pain. My arms throb and ache. The said they would call me to check up but they are now closed and do not open until Monday. Should I be concerned? Do you feel this is a normal post procedure reaction? If so, how long do you think it will last? I m just concerned that it could be permanent:( No one wants to go in for pain relief and come out with more pain:( Doctor: It can be cervical radical a pain secondary to thrum to root while ritual anaesthesia. Try tab pregaline for symptomatic benefit."
},
{
"id": 216074,
"tgt": "How to get rid of the cramps in my legs at night?",
"src": "Patient: I ve been having cramps in my legs at night while sleeping for quite some time now, perhaps 2 or 3 times a night. Sometimes the pain is in my calves, or in my feet, either in the arch of my right foot, or starting on the top side of my left foot. It s been happening now for about 3 months; nothing in particular before that with the exception of the odd time, which is probably quite normal. It seems to be worsening; is there anything I should or could be doing about it? Doctor: Hello and Welcome to \u2018Ask A Doctor\u2019 service. I have reviewed your query and here is my advice. This is incredibly common. This can be due to an electrolyte problem and supplements of magnesium and potassium MIGHT be helpful. Mostly, it is an age related problem beginning about age 40 and needs some medication to lower the symptoms. Hope I have answered your query. Let me know if I can assist you further."
},
{
"id": 179717,
"tgt": "What is the treatment for geographic tongue?",
"src": "Patient: Hi , I m a mother to a recently turned 4 year old boy . He s very active ,very advanced for his age everything . When he was about 1 1/2 he was diagnosed with Geographic tongue. His pediatrician advised me to stay away from citrus foods and drinks and we have . Later I ve notice that he has been telling me his mouth tends to get hot I could not pin point why checked to see if he had a fever each time no fever puzzling he also has a really hard time speaking, so I started to do more research because his pediatrician basically told me he had geographic tongue and left it at that with no more information on it. I m very concerned because my son isn t talking well at all and he complains of hot mouth and his tongue doesn t ever really seem to clear up. Is this something I should be worried about ? Also my son has a very selected amount of food he will eat and he will not try anything new. Does this have anything to do with his condition? In desperate need of answers. Also should I find him a new pediatrician that would provide me with more information Doctor: geographical tongue is normal variant. it is not a cause of concern although there may be some problem with spicy foods."
},
{
"id": 201538,
"tgt": "Suggest treatment for night fall",
"src": "Patient: hai respected doctor.. i am 23+ now.. figure just oky.. not slim, height lkay az normal people.. weight about 63.. when i sleep sperm comes out.. .. it happens to me 3, times a week, or some times.. through which physically i feel that i am not too healthy.. and some times i feel healthdowm .. Doctor: HiI had gone through your query.You need to gain sex education and need to learn normal function of our reproductive system.Sperm production occur in scrotum 24 hourly and storage occur in scrotum.But it has limited capisity to store it so if you are not sexually active means if you are not doing regular sex or masturbation then it will automatically comes out in dream while sleeping.It is normal and do not lead to any problem.Just relax and read from various sites.No need to worry.Feel free to ask.Happy to help you further.Thank you.Take care."
},
{
"id": 150496,
"tgt": "Had CT scan done. Can any online doctor help me to read it?",
"src": "Patient: We are looking into adopting a little boy from China. His \"issue\" is Congenital incomplete brain development. His CT scan states: Indicated in CT image:Flake-like low density shadow was observed in right side ventricle with clear borders, obviouslywidened surrounding cerebral sulcus, obviously widened right side ventricle; midline was rightshifted.Imaging:1. Right side ventricle encephalomalacia lesion2. Maldevelopment of right brainThoughts? Doctor: with findings seen on CT scan there appears to some form of developmental lag in development of right right cerebral hemisphere ( right sided brain) there could be some damage to brain due to difficult child birth- causing damage- which will be fixed( non progressive by now) with some loss of abilities other possibility is some syndrome congenital causing this- possibility it being progressive increases U need to get MRI for further characterization of pattern also child needs to be assessed by doctor for his/ her abilities and any other abnormal features/ conditions in body."
},
{
"id": 171344,
"tgt": "Suggest remedy for high cholesterol",
"src": "Patient: i am a generally healthy 16 year old girl. Not overweight, but I do have some kidney problems. Anyway, I have had fever for 2 weeks, and the doctor couldn t find anything that was wrong. She said my throat was red, but it doesn t hurt. For an hour I have had heart pain. Oh, last time I checked I have high cholesterol. Doctor: Hi,Welcome to Hcm,Your problems seem very non specific. The fever may be just a simple viral illness which will resolve soon. High cholesterol is not a common thing in this age. I would however like to see the complete fasting lipid profile to comment on it. But let me assure you that what ever it may be, its not something which is difficult to set right. So be happy and enjoy yourself. You can revert back with complete report. Take care."
},
{
"id": 98644,
"tgt": "Are breathlessness, swollen eyes and tightness of throat signs of anaphylactic shock?",
"src": "Patient: my daughter, aged 28 drank a cup of organic green tea with natural flavors. It was labeled green/blueberry tea and the contents as green tea with natural flavors. Within five minutes, she was not able to breathe through her nose, her eyes swelled up and her throat became very tight. Gave her benadryl, which she was just able to swollow. It stopped the reaction and ten minutes later, she could still not breathe through her nose so I gave her a face mister with eucalyptus essential oils and the membranes started to recede to allow her to breathe through her nose. Is this anaphylactic shock? Doctor: Hi and welcome in HCMNo these are not signs of anaphylactic shock but of an allergic reaction.Continue with the treatment.Best wishesDr.Jolanda"
},
{
"id": 166686,
"tgt": "Can intake of coke cause nosebleed in a child?",
"src": "Patient: Hi, may I answer your health queries right now ? Please type your query here...my son had a drink of coke a cola, yesterday at tea time, went to bed 2hrs later & when he got up this morning i noticed he d had a nosebleed in the night, could the cola have caused it? he is 4yrs old, this is only the 4th or 5th time he has had it in his life. Doctor: Hi,Coke doesn't cause nose bleeds. As he had it previously too, I feel that it could be due to the Kisselbach's plexus of blood vessels in his nasal septum which might have ruptured due to nosed picking and this is the most common reason for the nose bleeds.Hope I have answered your query. Let me know if I can assist you further. Regards,Dr. Sumanth"
},
{
"id": 17617,
"tgt": "What causes high blood pressure with high pulse rate?",
"src": "Patient: My 46 year old male friend has been feeling unwell, his blood pressure is quite high and is heart beat is very high, some readings have been 91 and above. I would be very grateful for some advice about how normal this is, he is asthmatic and takes medication, his breathing is very hard, but not like an astma attack, the slightest thing makes him breathless, he takes other medication like co codomol and amitriptyline, he has had pain in his stomach right by the rib cage that it is thought might be an ulcer. but we are quite concerned about the heart beat, sometimes it has been very fast and it is pssible to hear a ticking fluttering type noise, very clearly. please any advice would be a help. Thanks Doctor: Hello, It is necessary for performing some tests in order to investigate for the possible causes underlying tachycardia and high blood pressure values: - a resting ECG and cardiac ultrasound - complete blood count for anaemia - thyroid hormone levels for thyroid gland dysfunction - kidney and liver function tests - PCR and ESR for inflammation - blood electrolytes for possible imbalance. Increasing the doses of anti-hypertensive drugs may be needed, in case all the above tests result normal. You should discuss with his doctor on the above tests. Hope I have answered your query. Let me know if I can assist you further. Take care Regards, Dr Ilir Sharka, Cardiologist"
},
{
"id": 104783,
"tgt": "Cough with mucus, cold, wheezing, asthma. Taking winolap-5. Safe to continue medication?",
"src": "Patient: well 3&1/2 months back i was prescribed WINOLAP-5 for my wheezing . i had that for about 10 days. i was fine afterwards. Lately i cough out mucus during night time & now due to cold i am having bad stuffy nose , wheezing as well light asthmatic feeling can i continue with winolap-5? my e-mail address is YYYY@YYYY Doctor: Dear Mr. galaxyfunworld, Good morning, By your history it is apparent that you have allergic tendency with off & on episodes of breathlessness. Cold or flu like illness aggravates the symptoms. I suggest you to have your chest examined by your doctor for assessing the degree of bronchospasm. Meanwhile you may please continue T. Winolap for 5-7 days. Plus Steam inhalation/ nebulization 3-4 times a day and a short course of bronchodilator + steroid therapy would also help. I hope you find my response to be helpful and informative. I hope this answers your query, I will be glad to reply any follow up query that you have. Wishing you best of health. Regards,"
},
{
"id": 188280,
"tgt": "Irritating sensation in front teeth. No relation to cold food, no cavity. Low vitamin D. Help",
"src": "Patient: I have an annoying sensation with my teeth mainly the 2 big front teeth, what could that be, they don't appear to have any decay or cavities, they just have that annoying discomfort sensation With no relation to eating or drinking anything cold, now I do have low vit d because of barring children, pls help me with this concern. Doctor: Hi,Thanks for asking the query,This could be due vitamin deficiencies.Go for complete mouth scaling and polishing.Take multivitamin suplements.Get complete checkup done also take the x-ray of the tooth.Hope this helps out,Regards..."
},
{
"id": 6186,
"tgt": "Trying to conceive. Having painful irregular periods. Had unprotected intercourse. On Solaray Fertility Blend and prenatals",
"src": "Patient: Im ting to get pregnant but I have painful irregular periods but they have been coming every month.my fist period came on april 12-19 and my most recent was may 23-29 I had unprotected sex the 24th,27th and the 31st. I really want a baby im 27 years old. Also my periods are only really painful the 1st and 3rd day other days are tollerable. Please help I need to knowhow to better my chances I am on solarrays fertility blend sp-1 and taking prenatals . Doctor: If your period is a 45 day cycle, the probable day of ovulation and consequently the day when chances for conception are the highest is 2 weeks before the day of onset of period. If intercourse happens to be on the same day or a day or two before of after, the chance of getting pregnant is highest. n your case, the possible of conception in this cycle are far fetched. Timing of sex is most important for conception to occur."
},
{
"id": 142415,
"tgt": "What causes indentation in the forehead?",
"src": "Patient: I woke up this morning with a small indent on my forehead 1\" below the hairline, directly up from the middle of the brows. It looks as though someone has pushed soft dough with their fingertip and has left a slight impression. It is not tender and it has been there all day so far. Any idea? Doctor: Hello Thank you for trusting HCM Dear single day bony indentation is less common so it is evaluated weather it is bony or soft tissue defect then only we come to know the best way to treat. Please consult your doctor he will examine and treat you accordingly."
},
{
"id": 94629,
"tgt": "Abdominal pain extending to back, difficulty in urination, nausea. Normal ultrasound, blood work. Appendicitis recently done. Help?",
"src": "Patient: Hi my daughter has been in hospital for two weeks with abdominal pain which goes through to her back and is painful when her tummy is pressed and her back. She had her appendix removed 7 weeks ago in Turkey as they had ruptured and the poison affected her blood. She is having trouble urinating she can go but it takes her a long time, has been feeling nauseous and today started vomiting . The doctors yesterday started treating for pid with oral antibiotics but they are not sure that is the cause. She has had an ultrasound abdominal and a ct of her tummy and all looks fine. The pain is getting more severe and she has gone from feeling sick to vomiting and the pain is no better even though she is taking paracetamol , codeine , ibuprofen and oramoph and also anti sickness tablets. Her bloods are showing up clear, although one was slightly, but only slightly they say, for infection which they think could be left over from the appendix op. A small amount of fluid was seen on the ultrasound that shouldn t be there but ultrasound was carried out last Monday and CT last Wednesday. She has had an internal examination and that was extremely painful. Doctor: Dear Tina, I dont know the most important thing; how old is you daughter? Symptoms u re describing can be related to appendectomy complication especially if it was perforated and pelvic type of appendicitis. It seems like ureter was irritated which is a common complication and urinary infection should be ruled out definitely. Abscessus collection after appendectomy which US is describing could cause such symptoms, and it must be treated by antibiotics. Percutaneous puncture of collection can be also done if neccesary but i suggest just to wait for the effect of antibiotics. If febrility and pain persist then u should repeat CT scan. Also i think that you shouldnt give analgetics to child because it can hide symptoms, be care about it...you must know the intensity of pain no matter how it hurts because it guides doctor to the right diagnosis. Wish you good health, dr.Ivan Romich (Rominho)"
},
{
"id": 80992,
"tgt": "Suggest treatment for viral bronchitis and severe coughing with green yellow sputum",
"src": "Patient: I have two different opionions. my doctor said i have bronchitis viral. I was on steroids which i got off. im taking levoflaxacin. one a day. the i went to comm south to make sure i dont have pneumonia. chest xrays are normal. having coughing fits. finally am coughing up sputm. yellow green last three days. Ive been told i have viral infection. it sure sounds like bacterial bronchitis. if i have bacterial bronchits. am i taking the right antiobiotics. email adress YYYY@YYYY Doctor: Hello dear, thanks for your question on HCM.I can understand your situation and problem.In my opinion you should consult pulmonologist and get done1. PFT (pulmonary function test)2. Sputum culture and sensitivity to guide antibiotic therapy.For the diagnosis of bronchitis , PFT is must. It will also tell you about severity of the disease. And treatment is based on severity only. So get done PFT. You may need, inhaled bronchodilators too.Since, you have started yellowish expectations (mucus), possibility of bacterial infection is high.So we need to first isolate the causative organism an dthen start appropriate antibiotics.For this sputum culture and sensitivity is needed.Consult pulmonologist and discuss all these.Better to avoid haphazard use of antibiotics."
},
{
"id": 204263,
"tgt": "What are the symptoms of depression?",
"src": "Patient: Mood swings! lack of ambition! Fault finding/blames others, withdrawn, non communitive, aurguementive, remembers only parts of prior experiences, angers when told there was more to events, acts like he doesn t believe facts, strong sense of entitlement lacks any interest in socializing, or intimate relationship Is this a normal depression or indicating something more? Doctor: Hello and Welcome to \u2018Ask A Doctor\u2019 service. I have reviewed your query and here is my advice. It can be a part of personality disorder. A detailed assessment can help in knowing the exact cause. These issues are handled one by one depending on the severity. As these symptoms affect person's social and occupational life even the person himself wants to work on it and improve in those areas. Medicine can help in mood swings and controlling aggression. Hope I have answered your query. Let me know if I can assist you further."
},
{
"id": 60246,
"tgt": "Why is the Hernia operation done and what is the cause of it ?",
"src": "Patient: Hi may i know why Heranya Operation is done? Whats the main cause of it? What they ll do? Doctor: Hello. I suppose you wanted to ask about hernia operation. Hernia operations are done to repair the defect in the frontal wall of the abdomen as the hernia forms due to the weakness in the muscle layer wall of the abdomen. This weakness can be caused due to persistent inguinal duct in the inguinal canal, any previous surgeries, other underlying pathology within the abdomen which increases the intra abdominal pressure, defect in the correct formation of the muscle layers or due to underlying defect in collagen production due to genetic defect. This procedure involves placing a mesh which will form and strengthen the abdominal wall. Hope this will clarify. Regards."
},
{
"id": 1253,
"tgt": "Can i get conceived with multiple developing follicles in right ovary?",
"src": "Patient: I got my follicle report just now on the 9th day of my cycle and it shows left 14mm*12mm,17Mm*14mm, the right ovary shows multiple developing follicles avearge 10mm-MFD and the endometrial thickness is 5.3 mm.I know there is still time for growth but do I have chances of conceving this cycle Doctor: average follicles size should be 18 mm you should get your preg at that size n still u know u hv time to grow cz its only 9 days you should wait for 28 mm minimum n yes in this cycle you can conceive"
},
{
"id": 11185,
"tgt": "Suggest remedy to control hair loss",
"src": "Patient: i hav a very thin hair bt density was fine earlier .bt since last 3 years its density is decreasing ...i m applyin coconut oil with methi and neem once a week and dove shampoo..bt aftr applyin oil n when i comb my hair i get a lot of hair loss..please suggest me some remedies n how to control my hair loss and maintain my hair density gud enough Doctor: Hello,Thank you for posting on HCM.I appreciate your concern regarding hair fall. Hair fall is usually ascribed to multiple factors like diet and nutrition, hormones, stress, acute or chronic medical conditions,drugs,cosmetic products etc.Since you have not mentioned relevant history and description like sex, pattern of hair loss(patchy/diffuse), family history etc, its difficult to put a definitive diagnosis. Still, it seems you are suffering from either Androgenetic alopecia (due to hormones and genetically determined) or Telogen effluvium (Due to any recent stressful event or any other medical condition)If i were your dermatologist/trichologist, i would like to take through history including family history, history of recent major trauma /illness/ stress/ medications etc, your menstrual history and recommend some basic investigations like CBC,blood sugar,Thyroid function test.Also some other special tests like trichogram and dermoscopy can aid in diagnosis and prognosis.I would suggest you a course of oral tablets containing biotin and other essential vitamins and minerals (Tab. Follihair new) for minimum 6 months. Also, would put you on solution containing 5%/10% minoxidil once a day and a hair serum/gel containing peptides for hair growth at night.(Q sera, Grocapix etc). I would advise use of gentle shampoo and conditioner on regular basis and use of coconut oil twice a week.Also enquire from your dermatologist about upcoming treatment options like mesotherapy and platelet-rich plasma.Avoid combing in wet hair and let them dry by wrapping in towel. Avoid blow dryers and hair-color/dyes. Take plenty of fresh fruits and vegetables in your diet and try to de-stress your routine life.Hope your queries are resolved and wish you best of health.Thank youDr Hardik Pitroda"
},
{
"id": 82483,
"tgt": "Suggest treatment for pain in rib cage and cough",
"src": "Patient: I have bad coughing and wheezing for 4 weeks now. I ve been to the dr. He gave me a zpak and cough med. The 2nd time I went to his office he gave me steroids meds because my white blood cells are extremely low so he thinks my symptoms are due to an allergic reaction. This is week 4 and I m still the same. Now I have a sharp stabbing pain on my left rib under my breast. I don t know what else to do. Doctor: Thanks for your question on HCM. In my opinion you should get done chest x ray and Pulmonary Function Test(PFT).Chest x ray is needed to rule out lung infection as this can cause similar symptoms like chest pain and wheezing.PFT is needed to rule out bronchitis as this will also cause wheezing and chest pain.So better to consult pulmonologist first and get done chest x ray and PFT.Steroids should be given in appropriate indication. And low count is not at all an indication. So avoid these drugs and consult pulmonologist first get done proper diagnosis and start treatment accordingly."
},
{
"id": 168887,
"tgt": "Suggest treatment for itching in genital area in a child",
"src": "Patient: My daughter is 6 and her private area has been really itchy for a very long time. I ve taken her to the doctor, who said that she was masturbating. She is not doing that she is full on scratching. I ve checked for pin worms. I did not see anything. I feel so bad for her can you please help. Thanks erica Doctor: Hi,History of having local itching since long might be due to,1, yeast infection,2, having worm infestation,3, having poor local hygiene.Go for stool test for ova and cyst.apply antifungal cream.Keep her local hygiene clean, dry and airy.Do not allow her to play in clay play ground.Ok and take care."
},
{
"id": 203028,
"tgt": "What should be the normal level of sperm count?",
"src": "Patient: volume- 2.5 mlPH-7.5liquefaction 20 minssperm concentration- 42 millions\\mlTotal SP. constraction- 105 millionsP3-31 %P2-46 %P1-3%P0-20%viability-85%Aggultination -NILMorphologyNormal- 56 %Head defects-19%Neck defects-2 %Tail defects 5%Cyloplasmic defects-18 %I would like to know my sperm count is normal or not. could please update the status Doctor: HiThank you for asking HCMI have gone through your query.Your sperm count is normal.You have normal values for concentration,volume,viability,normal forms,PH,Liquifaction time.Hope this may help you.Let me know if you have any further query."
},
{
"id": 68378,
"tgt": "What causes lump on leg and vagina?",
"src": "Patient: I have a lump on the inside of the top of my leg, right by my vagina. It doesn't hurt, its about the same size as a marble but you can feel it and when you put two fingers round it you can feel the actual lump. I have had it for a few weeks now, you cant see it just feel it. What could this be? Doctor: Welcome to health care magic. 1.History suggest that the lumps you have noticed are lymph nodal enlargement.2.Lymph nodal enlargement is seen in any infection / inflammation with in the body also seen in the systemic diseases. And malignancies.3.In this case treat any underlying medical conditions causing the lumps.4.Other possible cause could be vascular cause.5.If they are large and causing any discomfort - get an FNAC - fine needle aspiration cytology for confirmation if cells. Good luck.Hope i have answered your query.Any thing to ask do not hesitate. Thank you."
},
{
"id": 125011,
"tgt": "How to treat abdominal injury?",
"src": "Patient: i had a fall where i hit the area around my left kidney, but not on my kidney, on the edge of a large boulder. i get a deep burning pain when standing but can move normally and it is not very painful to the touch. is this most likely just a deep bruise? Doctor: Hello, You need to visit 'ER of a hospital and be admitted for observation. These blunt injuries to the chest, abdomen or skull may at the time give symptoms delayed. Be careful. Hope I have answered your query. Let me know if I can assist you further. Take care Regards, Dr Nirmal Chander Gupta, Orthopaedic Surgeon"
},
{
"id": 44824,
"tgt": "Why am I passing black motion after an embryo transfer ?",
"src": "Patient: Hello doc. I am getting black motion after embryo transfer will it effect my embryo and getting pregnancy . Pls let me know Doctor: Hi Welcome to heath care magic black motion will not affect embryo how ever blakmotion is need to be clinically evaluated.. suggest to consult a physician"
},
{
"id": 66447,
"tgt": "What is the cause of a lump on the forearm?",
"src": "Patient: I have suddenly developed a lump on the top of my forearm about 2 inches by 1 inch. I have been carrying heavy bookbags on a new job that always seem to fall on that area. Could there be any connection. It hurt to the touch more the other day but now not as much. Any idea? Doctor: Hi, thanks for sharing your health concerns with HCM! If I were your treating Doctor for this case of sudden forearm lump, I would come up with three possibilities, these include: 1.\u00a0\u00a0\u00a0\u00a0\u00a0a bursa or ganglion due to repetitive exercise /pressure2.\u00a0\u00a0\u00a0\u00a0\u00a0The second possibility is of a benign cyst or hemangioma/hematoma\u00a0\u00a0\u00a0\u00a0\u00a03.\u00a0\u00a0\u00a0\u00a0\u00a0The last possibility is of some lipoma / neurofibroma like benign condition or some lymph node due to infection in surroundings !\u00a0\u00a0\u00a0\u00a0\u00a0Overall, it is benign and not to worry about this but you could go for FNAC test for confirmation if still worried!Hope this answers your question. If you have additional questions or follow up questions then please do not hesitate in writing to us. I will be happy to answer your questions. Wishing you good health."
},
{
"id": 177804,
"tgt": "What causes foul smelling stool in a child?",
"src": "Patient: My 6 year old son has very stinky poop and sometimes he has to go a few times a day. He usually is in a hurry to go (like diarreah) but he spends a lot of time in there ( like he is constipated) Its been happening for a few months now. Can you give me any ideas of suggestions to try Doctor: Hi dear Welcome to the HCM,Baby has contacted some stomach infection.Stool culture will tell about the exact bug causing the problem and also the exact drug to be used .Hand to mouth hygiene to be strictly maintained.Take care of the hydration.Hope the query is answered.Thanks"
},
{
"id": 119498,
"tgt": "Suggest remedy for knee problem",
"src": "Patient: bp hi im a 23 year old male 5 8 10st 10 a day smoker and recently my blood pressure has been high and abnormal i was taking dicloflex for a knee problem but stopped and im having the same problems so ive decided to monotor more regular can u help??? Doctor: Hi There is no relation between dicloflex and blood pressure,moreover dicloflex is not a good medicine to be used as a long term painkiller. You need to do blood pressure charting thrice a day in the morning,evening and night. For your knee problem you need to get a X-ray done to know about the exact cause for the pain. Take care. Hope I have answered your question. Let me know if I can assist you further. Regards, Dr. Rohan Shanker Tiwari, Orthopedic Surgeon"
},
{
"id": 131815,
"tgt": "Suggest treatment for severe body pain",
"src": "Patient: hi sir plz solve my problem is Lt sided body pain &swelling of back &shoulder joint from many year i have try allopath medicin 2to3 year than after homoeopath medicine in 1year but my symptoms is aggrivat &in this mounth X-ray report show c3-c6 vretebral gap &over growth of bone sir i suffring from sever body pain i again started allopath medicine plz give u r suggatio what i am doing for parmanent cur Doctor: HiDepending on X-rays, MRI findings, if large osteophyte and nerve pain radiation exists, removal of osteophyte, and surgical fusion can be a permanent solutionfor this, seek opinion of a neurosurgeon or a spine surgeon who may co relate clinical symptoms severity, and radiological findings,and offer permanent solutionIf not very severe symptoms, conservative measured may suffice, like use of collar, physiotherapy and cervical traction intermittentlyThanks"
},
{
"id": 96153,
"tgt": "H. pylori count is 180+, how high it is ?",
"src": "Patient: I have had a blood test showing a 180+ H pylori count, how high is this? Doctor: Hi, Welcome to Healthcare Magic Forum, H. pylori infection occurs when a bacterium called Helicobacter pylori infects your stomach or the first part of your small intestine. You are having this Infection since it is confirmed by your Blood Test. In many people H. pylori infection causes no signs or symptoms and doesn't lead to any complications. But for others, H. pylori can lead to serious complications, such as ulcers and stomach cancer. Yo need to take antibiotics for a minimum if 14 days along with antacids."
},
{
"id": 79013,
"tgt": "What causes pain in my torso while I am breathing?",
"src": "Patient: I was skateboarding and was hit by a car about a 1 1/2 weeks ago. My right upper torso hurts. it aches every time I breathe and the pain has been getting worse over the last few days. Did i bruise my lung? there was/is no swelling or noticeable bruising on my skin. what do you suggest or what's your prognosis? Doctor: It would be advisable to get an evaluation done with a doctor as you might have injured your ribs. Generally nothing to worry about but pain may take time if ribs are fractured."
},
{
"id": 196344,
"tgt": "Can we plan for child while having controlled non ulcer dyspepsia?",
"src": "Patient: My wife want to conceive. She is not on any medication. But I had non alcoholic fatty liver problem and non ulcer dyspepsia for lat 6 months.It is under control. I just stopped medication (\"URSECTOR 300MG - URSODEOXYCHOLIC ACID TABLETS\" AND Morease SR for irritable bowl syndrome) Just want to know can we plan for our child now? Doctor: Hello,Fatty liver and dyspepsia is not at all affecting the future child. Yes, you both can plan a baby. As per my advice before conception check your both thalassemia minor report and your wife thyroid and diabetes profile. For conceiving sexual intercourse done around 14 days of ovulation time of menstrual cycle means time between 12 to 18th day of menstruation period.Hope I have answered your query. Let me know if I can assist you further.Regards,Dr. Parth Goswami"
},
{
"id": 67703,
"tgt": "Suggest non surgical treatment for ganglion cyst",
"src": "Patient: is there a treatment for ganglion other than surgery (even with surgery what are the chances it won t reoccur). I have one that is growing in the inner side of my right wrist. I have been taking ayurveda medicines for two months but no help. I am also insulin dependent diabetic. Does Ayurveda cause sugar levels to go up (I find that since I started the ayurveda treatment my sugar levels have also gone up). I am also keloidal so if I have to go for surgery what surgery is possible (pin hole?) I live in New Delhi Doctor: Hi, dearI have gone through your question. I can understand your concern. No any medical treatment is there for ganglion cyst. You may try aspiration to reduce swelling but its temporary. Permanent treatment is surgical excision. You should go for complete excision of ganglion cyst. Hope I have answered your question, if you have doubt then I will be happy to answer. Thanks for using health care magic. Wish you a very good health."
},
{
"id": 196277,
"tgt": "What causes pain in testicles while having a bowel movement?",
"src": "Patient: When I poop my testicles hurt prior to going and while going? I have cyclic vommiting syndrome and bad bile reflux and am taking miralax twice a day for constipation I et bad stomach pain and pain in anus when I poop also and a few days ago had a little blood I stool my doc said it was probaly a Fischer Doctor: Hi and welcome to Healthcaremagic. Thank you for your query. I am Dr. Rommstein, I understand your concerns and I will try to help you as much as I can.This type of pain and location is suggestive of epididimitis which is inflammation of part of testicle and this not uncommon in males especially in young age. This is usually treated with 10 days of antibiotics, usually ciprofloxacin and there are no permanent consequances. MAsturbation and intercourse should be avoided till pain persist, if possible since I may prolong inflammation course. Some more serious conditions such as tumors or torsion are rare but in doubtful cases you should do Doppler ultrasound or scintigraphy. In this particular case, I don\u2019t think this is required and this should improve on antibiotic therapy.I hope I have answered you query. If you have any further questions you can contact us in every time.Kindly regards. Wish you a good health."
},
{
"id": 97102,
"tgt": "Are Jugular veins cut when doing an emergency tracheotomy?",
"src": "Patient: HOW FREQUENTLY ARE JUGULAR VEINS CUT WHEN DOING AN EMERGENCY TRACHEOTOMY? MY FRIEND WAS INVOLVED IN A TRAFFIC ACCIDENT COULDN'T GET HER INTUBATED;TRIED TO DO A TRACH;THEN AFTER HER 02 SATS DROPPED TO 15 THEY FINALLY GOT THE TUBE IN...A LITTLE LATE I'M THINKING Doctor: Hi,From you description the tracheotomy was performed under extreme emergency conditions. In these circumstances there would be a significant risk of damaging tissues and structures in the area as the operating conditions would not be ideal. Regards,Dr K A PottingerMBChB. FRCA"
},
{
"id": 39849,
"tgt": "Is scrotum itchiness means yeast infection?",
"src": "Patient: Hi. I am having white spots on my back. I sweat a lot too. And the skin under my scrotum itches a lot but not spreading. Is this a yeast infection altogether? I live in south east asia and very hot and humid. I try to be topless if possible. Can you help? Doctor: Hello,Welcome to HCM,The season and the temperature has lead to excess of sweating which has lead to your problems.The fungus can grow easily where ever there is a moist and wet area.The genital are most common site for fungal infection, because it can grow easily in wet and moist area and the skin under the scrotum area more prone for growth of fungus and produces the symptoms of itching.I would suggest you to follow1.Keep the area dry and clean2.Apply antifungal cream over the lesions3.Oral Antihistamines.Thank you."
},
{
"id": 148887,
"tgt": "Chest tremors, quivering lips. MRI shows white matter changes, cyst in sinus cavities. History of anxiety",
"src": "Patient: I have begun experiencing internal body tremors. They began as slight tremors in my upper left chest area and have since spread to my head and legs. The only visible symptoms are quivering lips that are only noticeable when I smile. MRI showed slight white matter changes and a cyst in both left and right sinus cavities occuping over 50% of the cavity itself. The tremors appear to be more noticable when I change body positions. Laying flat on my back produces none, while laying on either side seems to initiate these tremors. I am a 51 year old female with a past history of anxiety, although not an isue for some time. The only abnormal medical tests I have had in the past year is a positive D-dimer and a slightly elevated RA factor (18). Also very heavy benign PVC s (don t stress them-heart is good). Thank you. Doctor: Your MRI is not significant. The tremors could be related to anxiety. sometimes we feel we are not stressed but our body is. Get a check up. Mild medicine like Propranolol may help. If there is anxiety an SSRI like Sertraline or Escitalopram may help under a Doctors advice."
},
{
"id": 216568,
"tgt": "What causes chest pain in a patient with normal pulse rate and pressure?",
"src": "Patient: Hello, I do the 5:2 fasting diet. I had my blood pressure taken the other week and that was fine I had a resting pulse rate of 100 which was high but I had just lost my dog. (Found him) I checked it agin a week later and was 85 I have starting getting a slight pain across my chest. Not enough to cause pain but it s worrying me slightly. I feel absolutely fine apart from this but wondering if I should get it checked out. Thanks Doctor: Hello there I may be able to help you hereI feel you are having some sort of anxiety depression perhaps fear of losing your dog again!Please do some introspection. There is no harm in getting a check up scheduled though..hope that helps"
},
{
"id": 48002,
"tgt": "What causes pressure to urinate?",
"src": "Patient: Latey I feel like I have presure to urinate, right after I Urinate. I have no problems sleeping at night, but this accures more during a work day when I'm more active. I have been to the doctors and I have no prostate problems or UTI. When I sit I feel fine, Its mostly when I'm working. I do construction.This is becoming a problem and affecting my days. Doctor: Hello and welcome to HCM.During the daytime,if you happen to sweat more, increase your liquids intake.To check if your urinary flow rate is normal, there is a test called \"flowrate\".You should do that to ensure you're not having a low flow rate, suggestive of some block in the urinary passage.In males,flow rate is reduced due to a prostate gland enlargement most commonly.Other than a rectal examination to check the prostate gland enlargement, you should also get an Ultrasound examination.This will guide us regarding treatment."
},
{
"id": 190965,
"tgt": "My mouth not open properly because my tobacco habit",
"src": "Patient: my mouth not open properly because my tobacco habit. what i do? my mouth not open properly because my tobacco habit. what i do? Doctor: Hi, Your condition could be oral submucous fibrosis. The treatment includes intralesional injection of steroids, excision of fibrous bands and exercises for TMJ. Above all the habit of tobacco chewing should be stopped at the earliest. consult an oral surgeon."
},
{
"id": 224351,
"tgt": "Will the morning after pill work in consecutive months?",
"src": "Patient: Hello I took the morning after pill last month and had my period (it was early) and I took it again this month. Will it still work? I actually didn't have sex on both ocassions but his penis (and cum) was in contact with my vagina although he didn't enter me. Doctor: HiYes the morning after pill works whenever taken. It does work when taken in consecutive months.Even the cum contains sperms. Therefore you have done the right thing by taking morning after pill to avoid pregnancy.If you have regular cycles of 26-30days then the first week and the last ten days of your cycle are safe period. Having sex during those days does not make you pregnant. I wrote this as i felt this information is useful for you.I have a advise for you. If you wish to avoid pregnancy it is better for you to discuss with your doctor and use birth control pills or have copper intrauterine devise fitted. They are more effective than morning after pill in preventing pregnancy.The morning after pill if used within 24 hours of unprotected sex is 95% effective. Since there is a small chance of failure of the method, if your period is delayed for more than a week consult your doctor.I hope I have answered to your satisfaction."
},
{
"id": 213868,
"tgt": "I have been on ARIP MT 15 MG for the last 6-7 months and I feel fine. Should I still be on medications ?",
"src": "Patient: Hello I am 25 years old married woman for 9 months but i do not live in with my husband and my inlaws i am taking ARIP MT 15 MG for the last 6-7 months before that i was taking 10 mg of the same medicine but now from around 1-2 months i am not takng mediine properly and regularly as i do not feel any need to take the medicine i feel myself fine and ok is this thought right or wrong overall i am taking medicines for the last 2 years besides this i was taking flouxetine 20 mg but doctor stopped my medicine 8-9 months before also i am very addictive to listening to music and remain indoor for the whole day i got angry very fast what shoiuld i do i am unable to sustain a marrige Doctor: Hello Thanks for your query.\u00a0\u00a0\u00a0\u00a0\u00a0 you & your family including in laws needs good counseling,if all of you sit together & discuss the real situation ,solution can be there. \u2018Hope I have answered your query, I will be available to answer your follow up queries, \u201cWish you Good Health and trouble free speedy recovery\u201d"
},
{
"id": 104969,
"tgt": "Cough, cold, mild wheezing, excessive sneezing, throat congestion, occurs in cold climate. Cure?",
"src": "Patient: Hi Dr., I generally suffer from cold and cough during rainy/winter season accompanied by mild wheezing and extreme sneezing . I however try to keep my self warm and also avoid cold beverages. But the wheezing seems to come back every winter and causes a lot of suffocation especially during the night and early morning. Can you please help me? Doctor: Hi, Welcome to HCM Your problem is quite common one and seasonal allergic wheezing indicates a Bronchial asthma due to naso bronchial allergy. Beside keeping warm and avoiding cold beverages you have to see a Chest Physician and a detail examination with proper history . Then a Pulmonary Function test with effect of bronchodialator will confirm the diagnosis and quick relief medicine may be given in inhalation form. Allergic status can be checked with serum IgE level an allergy testing. If you take anti allergic and bronchodialator and if required steroid in inhalation or oral form will help you a lot antibiotic if required. If particular allergy is there immunotherapy may give you long lasting relief. Once you improve you can take up yoga and Pranayam. Take care Best of luck."
},
{
"id": 71190,
"tgt": "What prognosis is recommended for this chest CT scan result for signs of TB?",
"src": "Patient: I have had a year long plague of pneumonia with reason not found . An opacity in chest right upper lobe was found . Two bronchoscopies and one needle biopsy yielded no results .. although my symptoms were fever , weight loss , shortness of breath and cough that doesn t stop .. TB tests were negative ..No specific bacteria or fungi found during this time. They diagnosed this to be lung abcess and finally docs suggested me augmentin for several months which did kill all the symptoms but the mass in my chest would not go away completely .. surgery was advised and they were going to do a lobectomy but ended up doing a wedge resection .. now after a month of healing from this I m still having same symptoms and a CT still shows a mass or opacity in the upper lobe (right) . Doctors are suggesting a full lobe removal but my point is I have been through this once with no results .. what are the chances that this is a misdiagnosis and I have TB .. should I take a second opinion and treat for TB ? Doctor: Hello and Welcome to 'Ask A Doctor' service. I have reviewed your query and here is my advice. Yes, you should definitely go for second opinion. Recurrent mass like lesion on upper lobe is mostly due to 1. Tuberculosis. 2. Lung cancer. So empirical anti tubercular drugs should be started first. No harm in taking TB treatment. If mass is improving then no need to worry for cancer. But if mass is persistent then possibility of cancer is more likely. Hope I have answered your query. Let me know if I can assist you further."
},
{
"id": 176504,
"tgt": "What causes green stool in infant?",
"src": "Patient: HI DR I HAVE 6 MONTHS OLD BABY ,SHE IS PASSI;;;;;;NG GREEN COLOURED STOOL ,IAM FEEDING HER FORMULA MILK AP;TAMIL NO BREAST MILK ,NW I HAVE STARTED WITH RAGI AND BAN;ANA;;;;;;;;;;;;;;;;;;;;;;;;;;;;;;;;;;;;;;;;;;;;;;;;;;;;;;;;;;;;;;;;;;;;;;;;;;;;;;;;;;;;;;;;;;;;;;;;;;;;;;;;;;;;;;;;;;;;;;;;;;;;;;;;;;;;;;;;;;;;;;;;;;;;;;;;;;;;;; Doctor: Thanks for asking but your question is not complete. Let me reassure that greenish stools are normal in children of this age. It occurs due to rapid transit of fecal matter through the large intestine and the bile does not get time to change to yellow colour pigment of stool. 6 months is the right time to start semi solids. You have done the right thing by starting ragi and banana. You can give any type of adult food in mashed form and low on spices. Introduce items one at a time.One possible reason for green stools is that your baby is getting too much foremilk and not enough hindmilk which often occurs when breast milk production comes down. The hindmilk provides most of the calories your baby needs, and makes him feel full. If breast milk is inadequate, no need to give formula milk; you can directly switch to semi solids.You can introduce:CEREALS & GRAINS: Rice-Barely-OatFRUITS: Apricots,Apples,Mangoes,Bananas that you have mentioned Peas,Peaches,Pumpkin,Plums,Prunes.VEGETABLES: Sweet Potatoes,Acron/ButternutPeas,Green beans,Yellow squash.DAIRY: Plain whole milk yogurt.Hope that helps."
},
{
"id": 218195,
"tgt": "What does superior aspect of the uterus on an ultrasound report mean?",
"src": "Patient: Hi, my name is pooja and in my 18week+1 day scan, an amniotic band has been noted in superior aspect of the uterus. All the fetal measurements as per the ultrasound were normal and the band was not attached to the baby. At present, I m 20weeks + 2 days. Do the baby still has danger from amniotic band at this stage? What does superior aspect mean and does it tell the band position in uterus? Which body part is it near to? Can I get it removed? Doctor: Hi, Superior part of uterus refers to the upper part of uterus. Presence of amniotic band usually does not cause any problem but regular growth monitoring of the fetus is needed. Hope I have answered your query. Let me know if I can assist you further. Regards, Dr. Rakhi Tayal, OBGYN"
},
{
"id": 81788,
"tgt": "Suggest treatment for pneumonia",
"src": "Patient: I moved from Ga to Ct and have Pulmanary Fibrosis, I just had a cat scan for a pain in my groin, I guess they did a whole scan , they found a nodule on my left lung ,, I was in the hospital Jan, Feb, March, with pneumonia , I was seeing a dr at middlex, when he was told about the nodule, the office said he was going on vacation, they couldn t see me !! Didn t recommend I see another Dr, I will not go back !! I live in Westbrook, ct need something close ? Doctor: Thanks for your question on HCM. In my opinion you should definitely get done CT THORAX with contrast. As pulmonary nodule in pulmonary Fibrosis patient can be due to malignancy. So characteristics of nodule on CT is important. Characteristics like1. Margins (irregular or not)2. Calcification present or not.3. Associated with lymphadenopathy or not.Etc are needed to rule out cancer.So get done CT THORAX to confirm the cause of nodule"
},
{
"id": 73684,
"tgt": "Suggest treatment for sore throat and cough",
"src": "Patient: About a month ago now I started coughing and having a stuffy nose, coughing up mucus and my tonsils hurt to swallow. THIS WILL NOT GO AWAY, Im not sure what all I was given but NOTHING has worked. What is this, will I need surgery to get rid of it? I wake up every morning with a horrible sore throat and cough I feel like I always have mucus in my chest that will never come up. Doctor: Thanks for your question on Healthcare Magic.I can understand your concern.In my opinion, you should definitely consult ENT doctor and get done endoscopic examination of nose and throat.Possibility of allergic rhinitis or chronic tonsillitis is a more likely.You may need tonsillectomy if tonsils are enlarged and infected.You may need intranasal spray containing azelastin and fluticasone.Oral combination of antihistamine (levocetrizine or fexofenadine) and anti allergic (montelukast) is also good.Steam inhalation and warm water gargles 5-6 times a day are also beneficial.Don't worry, you will be alright but first consult ENT doctor and diagnose yourself.Hope I have solved your query. I will be happy to help you further. Wish you good health. Thanks."
},
{
"id": 41290,
"tgt": "Is the ultrasound report showing mature follicles with no free fluid normal?",
"src": "Patient: I am 23 years old and i had transvaginal ultrasound for follicle maturation(dated 8 march 2011). my report is :para 0+0, The uterus is anteverted & normal size, measuring 6.2x2.4x3.6cm, The endometral thickness is approx 1.1cm, Right Ovary measuring: 3.3x2.4cm(Showing multiple follicles). Largest follicle measuring 1.2x1.0cm. Left Ovary: Left ovary measuring: 3.0x2.0cm(Showing multiple follicles). Largest follicle measuring 1.7x1.7cm. No adnexal mass & no free fluid seen in cul-de-sac. IMPRESSION: Mature follicles seen in left ovary. please tell me my report is ok or not.one year ago i married but no babby why i have any problem in my report please advice me if my report is ok tell me any advice for my husbant. if any english mistake in above mention detail please correct.i am waiting for your kind and soon reply. Doctor: So far the report seems to be normal as far as the development and maturation of the follicles is concerned. Now you have to keep a watch on when the follicle ruptures and the ovum is released. On that day, plan your intercourse to increase the chances of fertilization. If the follicle does not rupture, then probably your Gynaecologist will give you an injection for the ovulation. Have intercourse the very next or same day of the injection.What about the rest of investigations like patency of the tubes, thickness of uterine lining, husbands sperm count, etc. Please keep me informed so that I can guide you further. You can contact me directly also, or through elite question."
},
{
"id": 2417,
"tgt": "Can pregnancy happen after taking Nordette pills?",
"src": "Patient: Hi, me and my gf had sex yesterday and finished around 1pm. Accidentally when I pulled out, the condom was stuck inside her vagina as in literally inside. After 10seconds of realizing it was stuck inside I pulled it off, I am not really that sure if there was a leakage or if I ejaculated in the condom or inside her. At 2:28pm the same day she took Nordette (4pills at once) and at the next day 2:28am (after 12hrs) she took another 4 pills. Is nordette really effective? What are her chances getting pregnant? Thank you Doctor: Hello dearI understand your concernDo not worryChance of pregnancy is less.Nordette sed in correct dose and it is highly effective in preventing pregnancy.It isupto 80% effective in preventing pregnancy.It may cause delayed in petiod by1-2 weeks.If nex period will delay by more than 2 weeks then consult gynecologist.Avoid stress, take healthy diey, drink plenty of fluid and do regular exerciseBest regardsDr. Sagar"
},
{
"id": 64256,
"tgt": "What is the treatment for sebaceous cyst in the chest?",
"src": "Patient: I have a cebacious cyst on my chest it was about the size of a pea, but has become infected and is now the size of a grape, i am on antibiotics and am having it removed in 4 weeks, the cyst is in the centre of my chest slightly to the left, i am a female and am worried about the size of the scar, and will i be able to go back to work afterwards, i work in an operating theatre Doctor: Hi,Good Evening.Thanks for the query to HCM.I studied it in depth and I understood your health concerns.-Treatment -for sebaceous cyst in center of Chest-1- I would advise you to let the infection subside and cool down first.2-So I would treat it with proper antibiotics with Metronidazole and NSAIDs tbs.3-I would Excise the sebaceous Cyst -in a planned way-so as to avoid more scarring .I would be able to put \"Sub-cuticular Cosmetic Stitches\" as to avoid any disfiguring Scar- in the central chest area between the breasts.3-Dont get scared and uneasy,but be cautious and act fast.Hope this would help you a lot to relieve you.Wish you fast recovery-and healthy life.Wellcome to HCM with more queries till you are satisfied.Have a Good Day...!!Dr.SAVASKAR M.N.M.S.GENL-CVTS,Super specialist and Senior Consultant-and Expert in Non-Curable-Disease therapy for Cancer,Asthma,etc,Rejuvenation therapy and Tissue failure -reversal therapies."
},
{
"id": 8577,
"tgt": "What fairness cream can lighten the complexion of oily skin in a 16 y old sri lankan?",
"src": "Patient: Hi Doctor, Nice you have given us a chance to clarify doubts via online! Ah im Ahamed from Sri Lanka, so hope you know what our skin colour is. Actually I wanted to become fairer because I wanted to attract my girlfriend as well as to have a change in life... So the only person can help me is YOU!!! My skin type: oilyAge: 16Ah so please let me know a good fairness cream which will turn me fair within a month, please DoctorThank you... Doctor: Hello and welcome to healthcaremagic.Skin color i genetically determined. Asian skin is type IV skin which contains fair amount of melanin and is prone to tan quickly as compared to caucasians/ people in the west who have very less melanin.Skin type cannot be changed but regular use of a broad spectrum sunscreen gel, which offers protection against both UVB and UVA, can be used to prevent sun induced tanning.A cream containing one or more of the following ingredients e.g kojic acid, glycolic acid, arbutn, magnesium acsorbyl phosphate etc can be use once daily at night to lighten up the tan and impart even tone to the complexion.Hope this helps youregards"
},
{
"id": 27866,
"tgt": "Are the high Systolic, Low diastolic and low pulse a matter of concern?",
"src": "Patient: Hi! My son always has cold hands,which somewhat concerns me. Today ( as part of our homeschooling in science) we measured their pulse rate; I then used my blood pressure monitor to see how close they were to being correct. My sons was rather strange . His systolic was 131 ( which seems rather high to me) and his dystolic was 59( which seemed rather low), and his pulse was only 50 ( which seemed really low for a teen) Should I be worried!? Doctor: Dear- It is important to monitor your son's blood pressure and heart rate. The difference between systolic and dyastolic is borderline high, it could be due to an isolated situation for which I would recommend to get records of a daily measurement at the same time for 2 weeks. If still like that , it might be beneficial to have a complete laboratory examination and cardiovascular evaluation with an electrocardiogram and echocardiogram. However, most of the time it is because the kid does a lot of exercise and the HR gets slow.I hope my answer will be helpful"
},
{
"id": 156840,
"tgt": "What does AVM and AVMS mean in relation to colonoscopy?",
"src": "Patient: Yes in regards to colonoscopy the ascending colon had small punctate red spots and on the recto sigmoid colon. They were not bleeding but had heme on the surface. These spots did not hae the typical appearance of anAVM but could certainly still be AVMS. What does AVM and AVMS mean Doctor: I think it is arteriovenous malformation. That is developmental disorder of the bloodvessels.nothing to worry if its asymptomatic."
},
{
"id": 108247,
"tgt": "What causes pain in upper left chest and back increased upon movement?",
"src": "Patient: I have major pain in my upper left chest and back that increase with movement under the breast that go straight through to the back skin sensitivity to touch in th left breast a burning and or numbing sensation that extends to the nipple. The ER said it is non cardiac, it s been two weeks am taking relation 500mg and cyclobenzoprine 5mg twice a day it s not helping please any suggestions ? Doctor: HiWelcome to healthcaremagicI have gone through your query and understand your concern.You are probably having muscular pain because it is mostly on movements. You can take analgesic such as ibuprofen for pain relief. You are advised to take muscle relaxant such as thiocholcoside for pain relief. Vitamin B and C is helpful in recovery. You can discuss with your doctor about it. Hope your query get answered. If you have any clarification then don't hesitate to write to us. I will be happy to help you.Wishing you a good health.Take care."
},
{
"id": 4511,
"tgt": "Have Deposit provera, sore breast and nipple, frequent urination, white discharge, cramps in abdomen. Pregnant?",
"src": "Patient: I have been off the deposit provera. Shot since June 2013. I have a serious boyfriend. About the last 2 weeks I have had extremely sore br\u00e9asts, super sore protruding nipples (not common) 24/7. Frequent urination even if I don't drink anything. Extreme exhaustion, clear/white discharge, slight cramping. In my lower abdomen. Could I be pregnant.I took a test and it came back negativen Doctor: Hello,Above symptoms are mostly associated with PMS or early pregnancy. You need to repeat pregnancy test after few days or undergo one blood test to exclude pregnancy.In the mean time, you must take balanced diet, avoid mental stress, take sound sleep and maintain genital hygiene. Good luck"
},
{
"id": 220644,
"tgt": "What causes brown discharge during pregnancy after taking Prontogest injection?",
"src": "Patient: Hi. i am pregnant early stage, i have experienced dark brown discharge last sunday when i took a prontogest injection. then had a hcg test which resulted to 4411 after a hcg test that resulted 533 six days earlier. i am on utrogestan twice daily, and on materna and calcitron. yet i have discharged dark brown again tonight. i am not able to reach any hospital now for a prontogest injection again. i am extremely worried . am i miscarriying. urgent anser is really appreciated. Doctor: Hello dear,I understand your concern.In my opinion the brown discharge might not be associated with prontogest injection.The various causes for brown discharge like thyroid abnormalities,progesterone or HCG insufficiency,bleeding from cervix,abnormal pregnancy,unknown causes etc.Infact the prontogest injection are given to stop spotting or bleeding caused due to hormonal abnormalities.The raise of HCG is also apt.The above causes should be ruled out by physical examination,thyroid profile and ultrasound.Dont worry.As the brown discharge does not always suggests miscarriage.I suggest you to consult doctor and rule out the above causes by examination and necessary investigations.It will subside with apt treatment.Avoid stress and physical strain.Take good rest.Avoid intercourse.Continue with medication as prescribed by your doctor.Hope this helps.Best regards..."
},
{
"id": 47595,
"tgt": "Are swollen feet, cellulitis and rash all over body prelude to dialysis?",
"src": "Patient: my husband is diabetic, he has not been well lately. His feet has been very swollen, and he got celulittis on one of his legs. Today when he woke up he had a rash all over his body. He went again to the doctor and the doctor said that he believes it is from his kidney. he put him on predisone 20 mg. 2 times a day. What I want to know is this a prelude to diaylsis. thank you. Doctor: Hi, welcome to HCM.Looking to your husband's history, I as a nephrologist can understand your concern.In your husband's case, it is necessary to treat cellulitis with antibiotic first.Check basic urine report, S.creatinine and sonography to rule out kidney damage.It is not advisable to take prednisolone when there is active infection (cellulitis) in body.Consult nearby nephrologist with above reports or you can get back to me also with reports.Swollen feet does not necessarily indicate need for dialysis.So, don't get panic.I think this would be helpful to you.Best wishes. TC.Dr Jay Patel."
},
{
"id": 100077,
"tgt": "How the fever and episodes of cough can be cured?",
"src": "Patient: hi,my father has episodic asthma. Two days back he had high grade fever, severe cough,wheezing and productive cough. He went to hospital where he was nebulized twice, received iv steroids.After stabilising him doc prescribed him inhaler Salmicort and Salbo,half theophyline tab twice a day and leflox 500mg od. Now he is feeling better but still has fever and episodes of productive cough.When he takes antipyretic fever goes down. He has also has BPH for which he is taking 5mg hytrin. what shld we do. should we wait and continue the same antibiotcs or consult doctor plz guide us about this. thanks Doctor: If he is feeling better at this time this shows infection is resolving. But if he still running Fever and have cough with sputum at least chest x ray and sputum examination is must.he may have developed pneumonitis or pneumonia that we can only find out by x ray. And if he do not respond to levoflox then it's sputum examination and culture that will guide us in further treatment plan . So in my opinion you keep close watch on her symptoms for 2 to 3 days .If you notice any deterioration or no improvement plz consult to your physician for further management . Hope this will help."
},
{
"id": 168221,
"tgt": "Suggest cause and remedy for vomiting and stomach pain",
"src": "Patient: My daughter who is 6 has been getting sick with vomiting and stomach pain where she s doubling over but this only happens once every so often, first it began 6-7 months ago and would be every 7-9 days and would happen about the same time of day, she would then vomit 3 times then fall asleep for the rest of the day and wake up fine the next day. Then it would happen about once every two weeks with the same thing, she would vomit 2-3 times fall asleep and be fine the next day, now it s happening once every 10 days. She s had urine and blood test and have all been clear and i ve done a food diary and nothing she s eating has been out of the ordinary. Doctors dont seam to know what s happening and i m at a loss. Doctor: Hi...by what you quote I feel that your kid is having a cyclical vomiting syndrome. Please ask your doctor to consider this possibility and work up on these lines.Cyclic vomiting syndrome, is a disorder with sudden, repeated attacks\u2014also called episodes\u2014of severe nausea, vomiting, and physical exhaustion that occur without any obvious cause.The episodes can last from a few hours to several days. Episodes can be so severe that a person has to stay in bed for days, unable to go to school or work. A person may need treatment at an emergency room or a hospital during episodes. After an episode, a person usually experiences symptom-free periods lasting a few weeks to several months.Regards - Dr. Sumanth"
},
{
"id": 70407,
"tgt": "What is the cure for the lump at the top of thigh in the groin area?",
"src": "Patient: Hi I have a smallish lump at the top go my left thigh in my groin area, it hurts only when pressed but will also sting after being pressed. I though at first it might be an ingrown hair from shaving but I can t see it. It won t burst. I am a female, could this go on its own?? Thanks Doctor: Hi. This is most probably an enlarge lymph node , could have been due to a minor un-noticed cut or so.Do not worry , as it usually goes on its own. Consult a Doctor if there is increase in swelling , redness and pain as you may need an antibiotic cover and anti-inflammatory medicines. Please do not fiddle or press it more , may activate infection in dormancy"
},
{
"id": 9581,
"tgt": "Having dry skin bubbles on the hands and feet. Using udder cream. Any medication?",
"src": "Patient: I have dry skin air pockets on my hands and feet . They don t itch , they don t hurt. It came on 4 years ago when my washing machine broke and I hand washed my clothes for about a year. the soap was very drying and contained a bleach product in it. Now I have been using udder cream and my hands have become soft but the dry skin bubbles are still there and the whenever I wash dishes for a long period of time they become more pronounced. My feet don t have much of these bubbles and I try to scrub the dry skin pocket off whenever get a pedicure , but they keep coming back in the same place. Doctor: Hello Welcome! I would be pleased to answer your question . From the description of the lesions they seem to be either a hand eczema or contact dermatitis. Which has become chronic at the moment . The Allergy can be due to external or internal allergic agent. The response to the agent results in an 'itch scratch cycle'. As in the more one scratches the more the area itches and so on. The first step in treatment is to break this cycle by giving relief from itching and instructing the patient to avoid scratching as much as possible. It can occur localised as in your case. Here is what will help: 1. Clobetasol propionate cream + Mupirocin cream: Mix and apply over the affected area on the right shoulder twice a day for 14 days then switch to Fluticasone propionate cream once at night for 2-4 weeks. In the groin/upper inner thigh use Fluticasone in place of Clobetasol from the beginning. The specialist to see will be a dermatologist and he or she can prescribe these for you. Nothing over the counter will help much, the only product with some benefit is 1% Cortizone cream. 2. Tab Claritin 10 mg twice a day for 14 days, then once a day for 6-8 weeks. Initially the dose can be increased to 2 tablets taken twice a day if there is insufficient relief in itching. 3. Sarna sensitive lotion can be used 2-3 times a day to give further relief from itching. This can be continued even when the above creams have been stopped. 4. Avoid scratching the affected areas as much as possible. This is most important. Whenever it is not bearable soak a towel in ice cold water and keep it on the affected areas or apply the Sarna sensitive lotion. Lastly keep the affected areas as well moisturized as possible with a moisturizer of your choice. NOTE: 1% Cortizone cream, Claritin and Sarna lotion can be bought without a prescription. Please feel free to ask if you have any queries."
},
{
"id": 87497,
"tgt": "What causes abdominal pain and spotting?",
"src": "Patient: Hi I am 30 yrs old. I felt a very small abdominal pain in the morning. Later that day I felt I had to go. Only a small amount of mucus and brownish red blood came out. Have gone to the bathroom a couple more times and the same thing is happening. Any ideas? I am 5' 4\" 122 lbs. In good health have had 2 C-sec. Oldest child 8 months. Doctor: Hi! Good evening. I am Dr Shareef answering your query.If I were your doctor, I would advise you for a serum HCG test rule out an unsuspected pregnancy if you are sexually active all these days, an ultrasound of abdomen to rule out any pelvic pathology, and a gm staining and culture sensitivity of the vaginal discharge to rule out a PID(Pelvic inflammatory disease). Till then, I would advise you for an anti spasmodic along with a proton pump inhibitor drug for a symptomatic relief.I hope this information would help you in discussing with your family physician/treating doctor in further management of your problem. Please do not hesitate to ask in case of any further doubts.Thanks for choosing health care magic to clear doubts on your health problems. I wish you an early recovery. Dr Shareef."
},
{
"id": 193255,
"tgt": "What causes fever with stomach discomfort and sensitive skin?",
"src": "Patient: Hi, my boyfriend is experiencing some tummy problems with it rumbling and feeling generally uncomfortable. He also has quite a high fever and his skin is pretty sensitive. He had breakfast and lunch but is feeling too weak to eat anything now. He tried to vomit, but to no avail. Is this just because his system is run down due to stress and exhaustion or is it something more serious? He rrecently started taking vitamin b supplements, but i'm not sure whether or not he is still taking them. Any suggestions? Doctor: Hello, You are talking about a weakness, upset stomach, discomfort, high fever and sensitive skin. Vitamin B supplements may not have anything to do with his present conditions. Because he has high fever, it should not be taken lightly. Therefore, my advice would be to consult a doctor earlier. Hope I have answered your query. Let me know if I can assist you further. Take care Regards, Dr K. V. Anand, Psychologist"
},
{
"id": 160758,
"tgt": "Suggest treatment to increase concentration",
"src": "Patient: Hi, this is Anees Sir i had a child of age 12 years, he has less interest in study, his logic is very weak, he is unable to add some numbers what a 4-5 years child can do very easily, he has no concentration on study, please tell me what should I do, what type of doctor should i contact or what are the best possible way to cure Doctor: Hi,Is he having features of hyperactivity? how is interaction with peers, including interest in group play? How was his overall development so far, like any delay attaining milestones? With details provided, I will suggest you get his IQ (intelligence quotient) tested and evaluate for learning disabilities- best done at a child development centre (a team of pediatrician, child neurologist, and child psychologist).Hope I have answered your question. Let me know if I can assist you further. Regards, Dr. Muhammed Aslam TK, Pediatrician"
},
{
"id": 86557,
"tgt": "Suggest treatment for gastroparesis",
"src": "Patient: I have a constant abdominal pain in the upper left quadrant right under the breast bone. It never goes away. I have multiple stomach issues but there is still a problem that is undiagnosed and I believe this pain is the source. When I eat and swallow I instantly feel a sensation in that area and will experience a bad odor from my rectum. If I am on an empty stomach that same area with the pain will sound like it is weezing at times like a bronchial breach but in the abdomen. Also it gurgles exclusively in that area of the pain when I am having stomach issues. I have been diagnosed with gastristis, esophagitis and a hiatal hernia but none of these conditions have been associated with that specific abdominal pain or the odor I experience when I eat and after meals. I started a fast to see if I can give the digestive system a break and reduce symptoms. I think I may be suffering from gastroparesis after research. Other than bentyl I am not on any other medication for my stomach but I am sick all the time from it. Odor issue is one of the main concerns I am trying to rid of. It all seems to be associated with the pain in the abdomen I can apply pressure in that area and increase the odor. Is that associated with gastroparesis haven t seen anything about odor? Doctor: consult gastroenterologist and get lower GI endoscopy done.Exact cause can be detected by such investigations.You might be having some infection in your GI tract because increased bacterial flora is usually the reason for bad odour which require treatment with ofloxacin 400mg BD 5 days."
},
{
"id": 149216,
"tgt": "Feeling of nerve endings discharging, started in a flash. Any ideas?",
"src": "Patient: I came across the following question and response on your site: WWW.WWWW.WW The question is basically my question.... \"I am expercing a condition where i feel my nerve endings are discharging. . This started in a flash where I felt like a switch was turn on in my head....\"Can you please point me in some direction where I can do more research on this? Doctor: Hi,Thank you for posting your query.I need some more information to better understand your problem and give my advice.1. Since how long are you having these symptoms?2. Has it happened only once or does it recur?3. Have you lost consciousness during these episodes?Possibilities include a seizure phenomena and vitamin deficiency.I hope it helps.Please get back if you require any additional information.Best wishes,Dr Sudhir Kumar MD (Internal Medicine), DM (Neurology)Senior Consultant NeurologistApollo Hospitals, Hyderabad,My personal URL on this website: http://bit.ly/Dr-Sudhir-kumar My blog: http://bestneurodoctor.blogspot.com/"
},
{
"id": 113140,
"tgt": "Lower back pain, MRI shows sacralization of L5 vertebra, mild diffuse bulges of L1-2, L3-4 & L4-5. What to do now ?",
"src": "Patient: I have been suffering from lower back pain since 3 month.Recently i have done MRI scan which shows sacralization of L5 vertebra and L4-5 and L5-S1 disc exhibit signal changes-S/O partial Desiccation , Mild difuse bulges of L1-2, L3-4, & L4-5 are noted,indenting the thecal sac . No significant nerve root compression or displacement.The neural foramina are normal. No significant facetal degenerative changes or ligamentum flavum thickening. I have also done blood test.i came to know that Vitamin B12 is 187.everythingelse is normal. what exercise should i do and what to do now Doctor: dear onewelcome to HCMyours MRI report is not bad.please do spinal extention exersises,heat therapy.donot bend forword direction,donot pickup weights have analgesics,muscle relaxant.local application of massage oil will help.have b12 injections,see ortho surgeon there.dr s k guptadrskg1@gmail.com"
},
{
"id": 12712,
"tgt": "Suggest treatment for rashes and strep of child",
"src": "Patient: My 4 year old has a rash, it started on Thursday on his forearms, pediatrician said it looked like contact dermatitis, he also was diagnosed with strep, but the rash doesn't look like a strep rash. On amoxicillin for strep, on Friday, rash grew hot to touch and became very red and itchy, so I started benedryl. Today he looks like he has a sunburn, only places unaffected are groin and ankles and feet. Fever still come back, rash still warm to touch, seems the brightest on his cheeks. Oh, he's non-verbal autistic as well so he can't describe any other symptoms to me. There have been no changes to soap, detergent, environment, or foods in the last 3 weeks, and he is a fair skinned red head as well. Doctor: Hello,I read carefully your query and understand your concern. The symptoms seem to be related to scarlet fever.Scarlet fever is an illness that can happen in kids who also have\u00a0strep throat\u00a0or\u00a0strep\u00a0skin infections. The\u00a0strep\u00a0bacteria make a toxin that causes a bright red, bumpy\u00a0rash.I suggest to keep the area\u00a0dry\u00a0and clean, using unscented, gentle soap and warm water. Applying calamine lotion or hydrocortisone cream may help reduce and soothe the\u00a0rash. Gargling with warm salt water can soothe a\u00a0sore throat.Hope my answer was helpful.If you have further queries feel free to contact me again.Kind regards! Dr.Dorina Gurabardhi General &Family Physician"
},
{
"id": 29761,
"tgt": "Suggest remedy for severe cough with breathlessness",
"src": "Patient: Hello ,My 3 month old son has a really bad cough which it sounds like he has mucus and is having trouble coughing it up . Last night his dad said he seemed to have a hard time breathing so he had to pick him up and help him . Also you can hear him basically wheezing when he breaths.He doesn't have asthma & he's had his TDAP shot . Thank you , Tyler-Marie Bowie-Powell Doctor: Hello,He needs to see a pediatrician to determine what is wrong and if he needs antibiotics. He should see a doctor today so take him to emergency if his pediatrician is not available.Regards"
},
{
"id": 144799,
"tgt": "Suggest treatment for chronic coccyx pain along with neuropathy in both legs",
"src": "Patient: I am a 68 year old male, not diabetic and no other underlying conditions except controlled high blood pressure and depression. For the past 4 years I have had chronic coccyx pain accompanied by neuropathy in both legs and pubic groin pain, and pain when I urinate. The pain is very severe at times I have had a multitude of tests after seeing all the major experts (neurology, gastroenterology, orthopedic surgeon, urology, rheumatology, pain management, etc.). All the major blood tests, 2 MRIs, CT Scan, colonoscopy, sigmoidoscopy, nerve conduction, physical therapy and steroid injections have not resulted in a diagnosis. The only finding is a slight nerve conduction deficit in my right thigh. So no definitive diagnosis. I now take Cymbalta, Narcos & Lorazapam. I took Gabapentin and Lyrica which did not help and the Narco s are starting to not help much. The Cymbalta, however, has improved my depression. I have been offered surgery to remove my coccyx but am told that results are 50\\50 and there is a very long recovery period and high risk of infection. I cannot relate this condition to a specific event but I have had a multitude of falls while doing sports such as skiing, biking, etc. and other types of falls during my lifetime. Any suggestions? Thanks. Doctor: Hi there, thanks for your question. Coccyx pain is one of the few conditions in medicine, where treatment can be frustrating to both the patient and doctor. One of the most important things needed is patience with the condition. You should try using a coocyx cushion which is a inflatable balloon kind of thing. Use this cushion in all situations , whenever u have to sit. while sleeping it is better to sleep on sides , rather on your back. Since you have tried all options anasthetic injections or surgery done by an expert might help. I would suggest you to meet a well trianied spine surgeon and consider injections or surgery after his or her consultation. Since its an area of sensitive skin , definitely there is chance of infection and akin breakdown. so your intensity of pain and doctors assessment will give best chance..All the best. regards."
},
{
"id": 99421,
"tgt": "What causes an itchy smelly rash on the belly button after having sweets?",
"src": "Patient: Every single time I eat anything sweet even fruit but really bad if I eat actual sugar such as cake or a cookie, I get a red itchy, smelly rash in my belly button or fold of skin within an hour. I have been doing really good losing weight, haven't had anything sweet but apples and strawberries for a month so today I allowed myself some treats and I am immediately itchy. Annoying, what should I do? Doctor: Hi,This seems to be food allergy which causing your rashes over abdomen.Food allergy can be diagnosed only on the basis of detailed history taking and thoroughly clinical examination by an expert.Although there are laboratory allergic testing of food is being done nowadays but none of them is reliable clinically and also adds extra cost to the patient.The best way to cure allergy is to avoid contact of that particular allergen. However there are very good medicines like epinephrine,steroids, antihistamines, mast cell stabilizers are available in market to combat allergic symptoms.Consult dermatologist or allergic specialist for further advise.Thanks."
},
{
"id": 78523,
"tgt": "What causes chest pain similar to precordial catch syndrome?",
"src": "Patient: I am 36 and I have a pain that is exactly like Precordial Catch Syndrome (PCS) except I've had it over 3 hours & I can't seem to make it pop. What could be causing this? Currently I am suffering from a herpes outbreak as well as a lower back injury in case that helps with potential causes. Thank you! Doctor: Thanks for your question on Health Care Magic. I can understand your situation and problem. Possibility of post herpetic neuralgia is more. It is common after herpes infection. It is due to hypersensitivity of nerves in the affected area. Best treatment of this kind of neuralgia is pregabalin and vitamin b 12 combination. Application of cold ice bag is also useful. Avoid stress and tension, be relax and calm. Don't worry, you will be alright with above mentioned drugs. Hope I have solved your query. I will be happy to help you further. Wish you good health. Thanks."
},
{
"id": 152683,
"tgt": "Suggest treatment for painful cramps in the legs while suffering from chronic leukemia",
"src": "Patient: Hi, my father is 64 years old, he is suffering the leg vasoconstriction which will easily get muscle cramps while sleeping and easily tired while walking, feel quite week. And he is also suffering chronic leukemia for 2 years, but under good control by medicine. One year ago, he starts feel his legs week, and easily get muscle cramps, quite painful, probably due to the medicine s side effect, so would you please advise how to deal with it? is there any medicine to treat it? Please let me know, thanks very much for your help in advance. Doctor: Hi there,I'm sorry to hear about the medical problems your father is having. When I hear the symptoms that your father is having, the first thing that comes to my mind is claudication. Claudication is pain while walking due to arterial insufficiency- basically, the muscles in the legs don't get enough blood flow when they are being used, and they start to ache. This is caused by peripheral vascular disease and is common in smokers, diabetics, and patients with high cholesterol levels. These patients will often have a history of coronary artery disease and sometimes renal problems as well. The first thing I would recommend is for your father to start a waking regiment and walk consistently every day. He should aim to walk for about 30 minutes a day. If he can't do 30 minutes, he should do as much as he can, and then try to do a little more each day. The most important thing is daily consistency - he has to do it every day, not just one or two times a week. I'd also recommend he check his cholesterol levels, and if they are high a statin such as lipitor (atorvastatin) can help lower his levels and prevent further damage. Finally, if he continues to have problems with walking after 6 months of a dedicated walking problem, then he should see a vascular surgeon and have his arterial vessels looked at in his leg with ultrasound. I hope this is helpful. Please contact me with any questions."
},
{
"id": 222613,
"tgt": "What causes spotting during 7th week of pregnancy?",
"src": "Patient: hi i am in my 6th week of pregnancy and i woke up around 2 oclock this morning to use the bathroom and when i wiped there was light blod on the tissue as if it had been mixed with discharge or something... I actually am going into my 7th week today? Any ideas on what this could be? Doctor: Hello dear,I understand your concern.In my opinion the spotting might be due to various reasons like hormonal abnormalities like thyroid abnormalities,progesterone insufficiency,any miscarriage,any excess physical strain,cervical bleeding etc.All the above causes should be ruled out by doing ultrasound,thyroid profile etc.Contact your doctor and discuss regarding these said things.Avoid physical strain and intercourse.Iam just trying to give you information and not diagnosing any condition ad it needs physical examination and necessary investigations.So relax.Best regards...."
},
{
"id": 75154,
"tgt": "Suggest opinion and food diet for pneumonia patient",
"src": "Patient: My father faced a heart attack few months back then doctors installed a metal stun in his heart. Few months passed. One day he was having fever and we found phemonia captured my father. His one lung was collapsed and lots of fuild was taken out of his lung. Now my father is under medication at home. My father is having swelling in his feets and He cant able to eat food. Doctors says there is a very little problem in his liver. please give us your opinion and what kind of food diet he should go for. Doctor: Swelling in the feet can be due to many causes. His heart, kidneys, lungs and liver should be checked. Even his state of nourishment should be checked. Sitting for long with legs haning down can also cause swelling feet. So get him tested thoroughly. Diet will depend on the test results. If all is well then he can have high protein diet with plenty of fruits and vegetables and slight restriction on salt."
},
{
"id": 37566,
"tgt": "Suggest treatment for mucus,cough and drainage",
"src": "Patient: My husband has had a lot of mucus, cough, drainage for a week now. Coughing with slight exerstion. Now coughing up globs of gray or brownish mucus. He IS NOT a smoker. 5 days ago he packed a moving truck in Florida in 90 degree weather & then drove it for 2 days to Md. I believe he got heat exhaustion that day as well as the other. He looks really tired & worn out. Doctor: Hello,This is Dr. Klarida Papaqako answering your question.The symptoms you describe are those of a possible Upper respiratory tract infection. Cough with mucus is normal, because its a symptom that clears the lung airways. It may continue for up to 3 weeks, until the lung airways regenerate their mucous. Drinking lots of fluids is recommended, and if that does not help mucolitic cough syrups may be used as an alternative.If your husband has fever, shortness of breath, chest pain, and is not getting better, then I would recommend doing a X-ray and consider treatment according to examining doctors evaluation. Pneumonia may come as a complication in these case, and it needs treatment with an antibiotic.Hope this helps,Regards."
},
{
"id": 95258,
"tgt": "Can a gastric bypass and anal rectal prolapse cause blood in stools ? How can this condition be treated ?",
"src": "Patient: Hi. I have had blood in my stool...about a tablespoon everytime and pain and irritaion. I have constant loose stool due to a gastric bypass and have had a anal rectal prolapse. Should I be worried? what should I do? my lower back has also been hurting. I am 32 years old Doctor: all your symptoms r due to r prolapse ... u need to look a good surgeon for rectal correction surgery ....he can plan asurgery after some hemostatics & antibiotics if needed"
},
{
"id": 222241,
"tgt": "What are the chances of getting pregnant without penetration?",
"src": "Patient: I was fooling around with my boyfriend 2 days ago. Although he did not put his penis in my vagina or near it at anytime, he did ejaculate on my butt. A tiny bit of semen dribbled down to my vaginal area, so I wiped all of it up immediately. I did not feel anything go inside at all, just around. My period ended the day before this occured. Could I be pregnant now? This question has been torturing me and giving me terrible anxiety and I need to know what my chances are because I can't live like this until my next period. Please please help Doctor: dearyour boy friend has ejaculated near your butt, you wiped it immediately & you are sure nothing has entered in vagina.chances of you getting pregnant is very very negligible .but why invite anxiety, have safety measures.Dr.Mira Butani"
},
{
"id": 28634,
"tgt": "What causes nail infection after recovering from hand, foot and mouth disease?",
"src": "Patient: I am 61 yrs old female. I got hand.foot mouth about a month ago it was very bad I had doctors in New Hampshire look at it I ended up in Boston at one of the major hospital. Now my finger nails are (Iwould call it dying) they are changing colors and coming away from the nail bed. Do you think it is related. Doctor: Hello and Welcome to \u2018Ask A Doctor\u2019 service. I have reviewed your query and here is my advice.Your symptoms seem to be related to a fibger infection.Some suggestion that I can give to you are:-Apply warm compresses or soak the finger in warm, soapy water for 10 to 20 minutes, at least twice a day.-Apply antibiotic or cream.-Keep the infected area covered with a sterile bandage.Hope I have answered your query. Let me know if I can assist you further."
},
{
"id": 5933,
"tgt": "Taking IUI treatment, prescribed PUREGEST tablet, feeling dizziness. Pregnant or side effects?",
"src": "Patient: Hello doctor, I have been married for 2 years nows, I was detedted with Ovarian cyst and got it removed earlier this year, post which my iui treatment started since my husband sperm count was low... this was my second IUI and his sperm count was only 5Million afyer wash, though he is on mediciaton for last three month and I am prescribed PUREGEST 200mg tablet for 15 days,, Everytime I take this tablet I feel Dizzines , Also with this low sperm count is it p ossible for me to have conceived already/... Thanks in advance for your reply Doctor: Hello. Thanks for writing to us. The dizziness you are having is a common side effect of Puregest that you are taking. Even if you are pregnant, this natural progesterone preparation is not likely to harm you. I hope this information has been both informative and helpful for you. Regards, Dr. Rakhi Tayal drrakhitayal@gmail.com"
},
{
"id": 142873,
"tgt": "What could a person unable to talk and hear much after stroke and being restless might be experiencing?",
"src": "Patient: Sir, my father developed stroke, and he has been taken to the hospital! And now, on bed, he is restless and always feeling pain. He can't talk, but he can speak silently and hear not very much. I want to know what i can do and what is the cause of this RESTLESSNESS that he is experiencing????? Doctor: Hi, Welcome to HealthCareMagic.com I am Dr.J.Mariano Anto Bruno Mascarenhas. I have gone through your query with diligence and would like you to know that I am here to help you.The Restlessness is because of loss of Blood Flow to a part of brain There is nothing active you can do other than waiting for the drugs to work and his functions improveHope you found the answer helpful.If you need any clarification / have doubts / have additional questions / have follow up questions, then please do not hesitate in asking again. I will be happy to answer your questions. In the future, for continuity of care, I encourage you to contact me directly in HealthCareMagic at http://bit.ly/askdrbruno Best Wishes for Speedy Recovery Let me know if I can assist you further.Take care."
},
{
"id": 106051,
"tgt": "Breathing problem with soreness in belly",
"src": "Patient: My son, 12, has had a cbc, ultrasound , CT scan , X-Ray, and an allergy test and everything checks out okay. He is having a hard time breathing and soreness from his belly button to his solar plexus . This has been going on for over a month. He s been prescribed prednisone and prilosec, but nothing seems to be working. He doesn t have a problem eating and says that a cool drink eases the pain. He s a very active kid, please help. Doctor: hi well come to hcm. since he is on steroids which at times leads ulcer or acute gastritis.usg and ct may not reflect it so if needed better submit himfor endoscopy or barium meal exam. which may help you if treating Doctor agrees. Thanks"
},
{
"id": 89463,
"tgt": "What causes cloudy urine with abdominal pain?",
"src": "Patient: A couple days ago my urine was cloudy, then it burned when I peed. Now I am expiriencing frequent lower abdominal pain. I have also noticed tiny specs that come out with my urine. When I went to the doctor they took a sample of my urine and it tested negative for a UTI. Any ideas of what it could be??? Doctor: Hi,I have gone through yoir query in detail,Usually with such complaints i suspect, could be a renal stone,You will have to undergo ultrasound abdomen and pelvis and KFTget your reports analysed by the urologist as they are expert in treating and diagnosing such condition."
},
{
"id": 95871,
"tgt": "My baby has stomach problem, please help her",
"src": "Patient: Hi dr My baby is 8 months old now. Today when we touched her stomach, we felt like a pimple inside her left side of stomach. We are really worried and want to know wht it actually is. So Pls help us Doctor: Hi Welcome to Healthcare Magic Forum Is baby crying of pain? how is the feeding habit? how is bowel habits? baby is playing well? Is baby is having fever? If everything is fine then observe, to know exactly the cause of it kindly consult ur paediatrician. Hope I have answered ur question Regards"
},
{
"id": 78114,
"tgt": "Could the productive cough and non acid reflux be related?",
"src": "Patient: Last year, after testing, I was diagnosed with non acid reflux disease. I am coughing up a lot of phlegm. My doctor said this was a symptom, but it seems to be getting worse. There is no blood or discoloring and I don t have chest congestion. Could this be related. Doctor: Hi. I can understand your concern. Chronic cough is commonly seen in bronchitis and lung infection. Get a chest x ray done if it is normal, no need to worry for lung infection. Possibility of bronchitis is more in your case. So better to consult pulmonologist and get done clinical examination of respiratory system and PFT (Pulmonary Function Test). PFT is needed for the diagnosis of bronchitis. It will also tell you about severity of the disease and treatment of bronchitis is based on severity only. You may need inhaled bronchodilators and inhaled corticosteroid (ICS)Also avoid stress and tension. Avoid hot and spicy food. Avoid junk food. Avoid large meals, instead take frequent small meals. Quit smoking and alcohol if you have these habits. Go for walk after meals. Keep 2-3 pillows under head in the bed to prevent reflux. Loose weight if you are obeseDon't worry, you will be alright. Hope I have solved your query. Wish you good health. Thanks."
},
{
"id": 211314,
"tgt": "Feeling confused and nervous , suggest treatments ?",
"src": "Patient: Yes, Is there an support group to go to when you feel like you are in this fight by yourself because the person that gave me this infection says that it is nothing wrong with him so he don t know what I am talking about. It feels like I am so confused and nervous... Doctor: HIThank for asking to HCMYou should have a open dialogue with the person without any hesitation, unless you let him know your pain he/she will not put forward something what ever there in mind, maintaining a good relation is best weapon and this could develop intimacy, in such situation this works very nicely hope this could help you have nice day."
},
{
"id": 167422,
"tgt": "Suggest treatment for cold in a toddler",
"src": "Patient: My son has a constant symptom of cold.Either a cough or a runny nose.Sometimes the mucos is thick and it also smells bad when I go near him. He does not take any medicine regularly.He is a normal eater and we are vegetarians. He goes to a day care.He is 2.5 years old. Can you pls advice. Doctor: symptoms are suggestive of allergic rhinitis.levocetrizine syrup avoidance of allergens.multi vitamin ENT evaluation"
},
{
"id": 29207,
"tgt": "Suggest complications with salivary gland infection on tongue",
"src": "Patient: Hi, may I answer your health queries right now ? Please type your query here... I have a sharp white bit of skin under my tongue on the middle of my glands that hurts when i touch it, ive got an infection in my saliva glands so i dont know if its normal when you have a gland infection to get this? Doctor: Hi..Thanks for the query..A sharp tiny bit under the tongue on the middle of gland seems to be a salivary gland stone or a sialolith that might have formed in the duct of the salivary gland and it can even more common to occur after salivary gland infection..I would suggest you to consult an Oral Physician and get evaluated and a thorough clinical evaluation and investigations like cross sectional x ray or MRI of the salivary gland can help in diagnosis and treatment can be planned accordingly...As of now you can apply numbing gel over it in case of pain..Once the diagnosis is confirmed and in case if it is a stone the physician can try to remove it by pushing it manually towards the duct orifice..If it is not removed manually a small incision/ cut can be given under local anaesthesia over the stone and removed..Hope this helps..Regards."
},
{
"id": 169969,
"tgt": "What do red spots above the lips indicate?",
"src": "Patient: Hi, my 9 yr old girl has red spots on her nose, above lips and mouth area. These spots have white heads on them. She also has very riasen taste buds at the back of her tongue and one blood shot eye which was a little weepy this morning. Are these symtoms of a cold virus? Doctor: This is called molluscum contagious. .it will spontaneously disappear mam.if the child has fever or lethargy then we have to evaluate mam"
},
{
"id": 101536,
"tgt": "Should i take foracort and rotacap inhaler if no cough is there?",
"src": "Patient: Hi,I am a 34 yr female with no previous history of asthma or breathing issues,neither does anybody in my family suffer the same.i had severe cough 3 weeks ago,which has now subsided.my first doctor recommended a course of erythromycin,which I took and got some relief.he told me tht the cough will gradually go away.on seeking a 2nd opinion,I was told to take fortacort 200mg rotacap inhaler morning evng,I continued for a week and then stopped.shud I continue to use it as recommended by the doc .i currently have no cough or any othr symptoms of asthma.if yes,wht r the possible side effects Doctor: Hi and thanks for the query,These drugs need specific indications to be taken. First , because they have specific side effects that need to be monitored carefully and secondly, because the effectiveness is dose dependent and patient dependent.I suggest you meet an internist and possibly a pulmonologist first. Regards"
},
{
"id": 130590,
"tgt": "Experiencing chronic pain around left shoulder and neck region",
"src": "Patient: I have chronic pain around my left shoulder and neck region. I have had several scans and the latest was a nerve test. The neurologist referred me to a vascular specialist. I am waiting to see the specialist. This weekend I have experienced stabbing pain in my left buttock region and left leg pain. In the past my heart rate dropped to the 40 s and nobody could figure out why. Also experienced dizziness. They ruled out heart problems but stated I had a hole in my heart but because only three bubbles passed during the test I was told to quit looking any further. This was three to four years ago. It is affecting my life. Pain everyday. Doctor: Hi,The chronic pain of the neck and shoulder could be:- Spondeliarthorsis with mean the body of the vertebra with their articulation are damaged- an hernieted vertebral disc- thoracic outlet syndrome/scalene syndrome.- boursitis of the shoulder (probably something from the shoulder. Only an orthopedics or neurosurgeon can do a differential diagnosisHope I have answered your query. Let me know if I can assist you further. Regards,Dr. Rezart Cipi"
},
{
"id": 45101,
"tgt": "How to increase rapid linear progression ?",
"src": "Patient: my rapid linear progression is 0% how to increase ? Doctor: Hi zaroor \"rapid linear progression is 0%\" It means that you are \"shooting blanks\", meaning that your wife won't be getting pregnant by having sex with you. You'll need to consider artificial insemination with donor sperm to get conceived. take care"
},
{
"id": 6146,
"tgt": "Swollen painful lymph node on the groin area during ovulation and menstruation. CT scan normal. Trying for conception after miscarriages",
"src": "Patient: Hello, I have a swollen lymph node on my left groin side. I have noticed that for 3 months the lymph node shows up and gets swollen and painful with ovulation and with my period. I have no idea why ovulation and my period would make my lymph node show up! I have gone to the doctor and gotten a CT scan , I heard back that everything looked fine except they found lymph nodes...I have an appointment on the 18th, but my husband and I want to try to conceive this month after 2 miscarriages. I would be trying to conceive before my next appointment. We have waited 6 months to try again and I really hate to put it off if it is something we are just going to watch. Any advice would be appreciated! Doctor: Hello. Thanks for writing to us. A lymph node swelling in the groin with no other associated systemic symptoms could be due to an allergic cause or viral infection. You can try for conception while waiting for the proper investigations. The conception is not likely to be affected. I hope this information has been both informative and helpful for you. Regards, Dr. Rakhi Tayal drrakhitayal@gmail.com"
},
{
"id": 96569,
"tgt": "Suggest methods to drain out tea tree oil injected in the leg",
"src": "Patient: My wife injected an infected area on my leg with tea tree oil. Now my foot/ankle and leg is swollen. How can I get this tea tree oil out of my leg? I elevate my foot at nighttime and it returns to the original size. During the day while walking around its swells up. .My foot itches. Will activated charcoal bind to it and take it out of my leg and body or am I stuck with this problem for the rest of my life? Is there something that I can take to counteract this tea tree oil? Doctor: HIWell come to HCMI really appreciate your concern, if the tea oil is injected in the system (Human system) so it can not be taken out but it may not be remains in the system for long time and soon will be anabolized, but of course the side effects of this would be matter of concern and that needed to be taken care, it may causes the tissue destruction, infection, abscess formation, and other hope this information helps."
},
{
"id": 123042,
"tgt": "How long does swelling last in case of fracture of metatarsals?",
"src": "Patient: My daughter fractured her 2nd and 4th metatarsals 3 weeks ago and is in a boot cast. How long should the swelling last and when can she start putting weight on the foot? She has a follow up appt. with ortho dr. in a week which will be 4 weeks out since the injury. Doctor: Hi, For fracture 2nd and 4th MT foot, generally 3 weeks cast immobilization is enough. In my opinion, the cast should be removed and physiotherapy should be instituted. Hope I have answered your query. Let me know if I can assist you further. Thank you. Regards, Dr. Gopal Goel Orthopaedic Surgeon"
},
{
"id": 106767,
"tgt": "What causes sharp pain in the shoulders, back, neck and thighs?",
"src": "Patient: Hi, I have had shoulder pain for 3 weeks now and it is going up to the back of the neck. I had a cortizone shot, but that didn t help, no ice, heat, ibuprophene or Advil helped. Going to an orthopedic doctor in 3 weeks. That s the soonest I can get in. Also, my right leg goes numb and tingles during the night while I lay on my back. Then I get a sharp shooting pain in my thigh. Can you please tell me what s going on? Doctor: Hello and Welcome to \u2018Ask A Doctor\u2019 service.I have reviewed your query and here is my advice.In my opinion, the symptoms point towards disc prolapse which needs to be evaluated with X-ray and MRI. Hope I have answered your query. Let me know if I can assist you further.Regards, Dr. Fahim Sheik"
},
{
"id": 116437,
"tgt": "What causes low blood pressure?",
"src": "Patient: hi my name is viviek my mother in law was addmitted in Moolchand for Dialysis where she had low blood pressure and was addmitted to ICU dOCUTORS ARE GIVING MEDICINE AND SHE IS ON VENTILATOR AND CONCIOUS AND RESPONDING I wanted a second openion on that Doctor: Hello,I understand your concern.I am Dr Arun Tank, Infectious diseases specialists, responding to you.Her low blood pressure is because of her unconsciousness. In this case vasopressor drugs should be given. It causes heart to press deeply, this causes resultant increases in blood pressure. I will be glad to answer your further query. Thank you for your contact to health care magic.Wish you a best health.Thank you,Dr Arun Tank."
},
{
"id": 99126,
"tgt": "Suggest remedy for severe allergy problems",
"src": "Patient: i m facing severe severe allergy prbs since five months wehn i came to KSA for work,whole bogy red and swollen mostly, means almost every daynow, and smtimes eye or lip swelling smtimes its lower lip and now its upper lip since last night, 24 hours have been passed, i have taken sm chlorophenairmaine maleate and now taking antihistamines but nothing worked, so tense, cant figure out what actualy is causing it as smtimes i have it wehn empy stomach, last nite had chocolate, but ican have it almost aafter any food, so how to figure out? Doctor: HIWell come to HCMI really appreciate your concern, underlying cause of allergy need to be found out so in future you could stay away from this and for that you have to get done the patch test, till then \"Tab Hydroxyzine 10 mg three times in day\" can be tried, hope this information helps, take care and have a nice day"
},
{
"id": 175594,
"tgt": "Suggest treatment for swollen painful ear",
"src": "Patient: my 3 month old son's ear is swollen from outside and he cries with pain... the doctor i went for gave him otocin-o ear drops, taxim-o syrup, nasoclear for nose and crocin for pain. he thinks that milk went in his ear while feeding..please let me know if the medicine priscribed and diognose done is correct Doctor: HelloYour doctor is right.The most common cause of ear infection in this age is improper positioning of baby during feeding.Feed himin slight upright posture.The doctor has prescribed right medication as taxim o has good coverage for the organism causing ear infection.There is also one antibiotic ear drop for local application. He has taken good care of pain and nasal blockade too.If there is no improvement in next 72 hours revisit your doctor for reevaluation.Regards"
},
{
"id": 185603,
"tgt": "How to prevent bacterial effect on teeth?",
"src": "Patient: my teeth is geting problem by bacteria there is some black cavity in my teeth and even i m loosing dentins on teeth so can u tell me plz that what shoud i have to do for getting white and healthy teeth and how to prevent bacterial effect on my teeth? Doctor: Hi Thanks for your query with healthcare magic, cavities of teeth can be prevented with regular brushing with proper brushing technique in the morning and before going to sleep, rinsing the mouth after every meal, consumption of less sticky foods like sweets, taking sweets in between the meals,use of floss and regular use of mouthwashes (prolonged use is not recommended especially if it is chlorhexidine it may result in staining of teeth).By these methods you can reduce/prevent cavities.Hope this is helpful."
},
{
"id": 130971,
"tgt": "Suggest treatment for leg pain",
"src": "Patient: My son-in-law 34 ears of age his wife (my daughter) who is 7 months pregnant (32 years) and now my son (27 years) and my wife have all come down with severe gastro (vomitting, diarreah, aching muscles, headaches). Son in Law basically over it as is my daughter hopefully. My son wants to know is there anything he can take for aching legs in particular. He cant keep food down.Just taking hydralite ice poles. Doctor: one pill of each streptokine and primran can stop diarrhoea and vomiting , should take 2 pills of zentak and 2 flagil as well for leg ache he can use thiotacid 500 Good Luck"
},
{
"id": 112307,
"tgt": "Chronic back pain. MRI shows spinal stenosis, degenerated disc disease, arthritis. No relief with betablocks, epidurals. Surgery required?",
"src": "Patient: I've been suffering chronic back pain since I was 16. i still did and lived a busy life raising 2 girls alone. i'm 46 now and for the last 15yrs or so, my pain has worsen. My first mri 4 yrs ago stated as follows; spinal stenosis, degenerated disc decease, annulis tear, bulging disc, artheritis. I aslo have bouts with sciatica! In the last year I had about 10 sessions off betablocks in my lower back area weekly, For 2 months in a row I had 2 epidurals. I found that it didnt help my pain at all! The third month, I told the Dr., didnt work so they tried the Lower lumber Braanch block, i ended upwith Nervitis!? now im getting IV Lidicain. had 4 in 9wk intervals. I FFEL WORSE!! Finally my question, 'Should I get surgery!? I CAN'T TAKE IT ANYMORE!!!!! I see my pain surgeon Wednesday. Doctor: Hi, Very sorry to know about your health condition. As it is, it is a chronic condition. I advise you to consult a spine specialist. I to my patient with such symptoms prescribe neurotropic injections, for 10 consecutuive days, ofloxacin, and antiinflamatories. Avoid taking potato, other tubers, tomato, egg, chicken, and sea foods for some time. Thank you."
},
{
"id": 176368,
"tgt": "Suggest treatment for fever and cold hands and feet",
"src": "Patient: Hi i have a question my daughter is 4yr with a fever of 101-102 with ice cold hands and feet, she looks very pale and with some purple color to her her finger nails are almost white the dr said she has and ear infection and gave some antibiotics. Should i be worry or take her to the er? Doctor: High fever with cold extremities indicates a condition called shock which may be serious and caused due to certain bacterial infections. The purple color of fingernails indicates cyanosis and is equally concerning. It is a concerning situation and you should definitely rush her to the ER and get her assessed and treated urgently."
},
{
"id": 157039,
"tgt": "Painful sex after rectal seal repair, blisters in the roof of the mouth, sore nipples in a 63 year old",
"src": "Patient: Thank you for being there. I had a rectal seal repair a year after my last birth in 1990. Sex has become painful. Now I have pain in that area in the middle of the night that wakes me. The roof of my mouth has blisters now and I am feeling very tired even after 8 hours of sleep. I am 63 year old. Both of my breasts have sore nipples and the left one is dimpled. I had 3 pollyps removed during 2009 colonoscopy. I also had a clear mammogram in 2009. My mother died of breast cancer and my grand mother of colon cancer. Doctor: Hi, Both breast cancer n polyp are inherited disorder .You have mentioned that one of your breast has dimple so you should meet a onchologist . After rectal seal repair pain occurs but not after 23years. probably there must be some other pathology.so in my opinion meet a surgeon for proctoscopic examination n breast examination as well and remain in follow up ever 6 months in all reports normal. Thanks n regards"
},
{
"id": 223428,
"tgt": "Does depo shot cause headache and large spots?",
"src": "Patient: I have had the depo injection 2 weeks ago and ever since I ve been on my period and breaking out in large spots which I have never had before and having frequent headaches, I d like to know if there is anything to counteract the depo-provera injection. Doctor: sideeffects of depo shot are headache ,nausea,break through bleeeding,irregular cycles...long term use can cause osteoporosis...ya,ur symptoms r due to depo shots...u can switch to alternative method of contraception after visiting ur gynec..."
},
{
"id": 31168,
"tgt": "Suggest remedy and diet for TB infection in lungs",
"src": "Patient: hello sir my name himanshu ,my sister is infected by TB,she is 24 year;s old,her left lungs is filled by fluied,doctors pull out 500 ml and remaining are there .dr told me to after one week check the fluid via xray,he gave us one week medicine and told me to ate all kinds of food,her wait is 50kg,and she's are troubling lots of,his cough is starting to much,so sir can you give me diet and treatment suggestion?i m wating for your response Doctor: Hi,She might be having pulmonary tuberculosis with pleural effusion.As anti TB medicine started, gradually recovery will start.Continue this medicine regularly for 6-9 months.Give her high nourishing protien food with multivitamin supplements.Give her dry fruits, fresh fruits, ghee, cheese, panier, eggs.Keep chart of her weight and see that her weight should be increasing.Ask her to go for deep breathing exercises.Ok and take care."
},
{
"id": 145808,
"tgt": "What causes bed wetting in a patient with chronic back and neck pain?",
"src": "Patient: My husband has chronic back and neck issues. He will be going in for surgery in the next 14 days for a lower lumbar fusion. He has begun to wet the bed (4 times in the last 3 weeks) should I let his surgeon know, is this emergent or will he be ok until the surgery? Doctor: Dear caring wifeBed wetting is a serious problem and may be due to cut a equina syndrome in which due to large disc prolapse signicant compression of multiple lumbar nerve roots leads to weakness of both limbs and bowel and bowel and non bladder involvement. So please call urgently your surgeon for this issue to get permanent and long term relief. All the best."
},
{
"id": 192397,
"tgt": "Suggest treatment to increase the hardness of the penis",
"src": "Patient: Hi my name is Suman, 30 years old unmarried. I have sexual problem. for few month my penis won't stand hard during sex. but during masturbation my penis become hard. I have no girl friend. so I go to brothels and there during doing sex my penis won't hard and don't get any feeling. please solve my problem. I m in great tension. Doctor: Hello,First of all, you need to follow non-pharmacological measures to improve your sexual life which includes eating healthy, regular daily exercise of your pelvic muscles and wearing loose fitted garments. If all these measures fail to improve your sexual life then get a formal consultation from your doctor to see if there is any infection or hormonal imbalance and get treatment accordingly.Hope I have answered your question. Let me know if I can assist you further. Regards, Dr. Sameen Bin Naeem, General & Family Physician"
},
{
"id": 154848,
"tgt": "How can the inoperable aorta tumor be removed or treated?",
"src": "Patient: I had a tumor wrapped around my fallopian tubes. A hysterectomy was performed. However, part of the tumor was wrapped around the aorta and unable to remove it. I had drastic chemo and now just watching it. It has not increased. My doctor says this is inoperable. I even went to see about the Da Vinci method. They agreed. I would not be a candidate. Do you know of any other treatment or removal of this remaining tumor? Doctor: Hi, dearI have gone through your question. I can understand your concern. If you have tumor around the arota then it can not be removed by surgery. Treatment depends on type of tumor. In general chemotherapy and radiotherapy is the treatment of choice of this unresectable tumor. Consult your doctor and take treatment accordingly. Hope I have answered your question, if you have doubt then I will be happy to answer. Thanks for using health care magic. Wish you a very good health."
},
{
"id": 216787,
"tgt": "What causes pain on both sides of my breasts?",
"src": "Patient: I have had alot of pain on my upper left back and side right about breast level and where my bra hits. It hurts all the time but when I cough it s very very painful and hurts when I sneeze, I have done nothing to break a rib and it hurts to wear my bra and hurts when I breathe in deep. Doctor: Hello,It seems that the bra you are wearing is too tight and pressing muscles in the area where its straps are. Wear a bigger bra or the correct sized bra for you. The helping person at the store from where you do the shopping can help you finding your correct size.In addition, do not wear bra 24 hours. Remove it every night so that the muscles in that area can relax. For relief, you can apply heat compresses in the area. Apply Diclogesic gel or spray (Volini) over the area. Investing in a professional massage can also help.I hope this information helps you. Thank you for choosing HealthcareMagic.Best,Dr. Viraj Shah"
},
{
"id": 226,
"tgt": "How to get pregnant while suffering from irregular periods?",
"src": "Patient: hai madami am 25 years old and married 1year ago now we are planning to conceive and to mention that i have irregular periods so what should i do in order to conceive and i consume more of panner in diet and lot of carbohydrates and i am not obese so what should i do pls help me madam Doctor: Hello,Do serum TSH and serum prolactin and follicular study after taking Clomiphene from day 2 with gonadotropins after visiting a gynecologist. Till follicle size increase 18 mm then ruptures then IUI for early results.Hope I have answered your query. Let me know if I can assist you further.Regards,Dr. Sheetal Agarwal"
},
{
"id": 60486,
"tgt": "My ALT being 78. cholesterol is 213, glucose 107. Is that nornal range ?",
"src": "Patient: I am a 36 year old male, just had my first health screening, I m a little concerned about my ALT being 78. cholesterol is 213, glucose 107, and BMI 30.4. does this sound like fatty liver or something worse?? to follow up on this, the ALT test was repeated and came back at 68 (alt) two months later. Is that nornal range ? Doctor: well seems to me as fatty liver and at present into the state of fatty hepatitis. i would ask you to consult a good gastroenterologist and do consider an evaluation bout your fatty liver. are you an alcoholic?. if not, we might need to evaluate you for NASH also. i guess you need to start with atrovastin and urodeoxycholic acid meanwhile. but the best would be getting an immediate response from a gastroenterologist."
},
{
"id": 111149,
"tgt": "How to treat \" Protruded disc at L5-S1 level wothout nerve compression\" ?",
"src": "Patient: I have been dealing with back spasms for months. After the spasms calmed down, I bent and twisted to pick up some boxes and felt a Sharp pain go down the back of my leg. I have experienced leg and foot pain, tingling and occasional numbness since. After the episode, I had an MRI done and It showed a protruded disc at L5-S1 level without nerve compression. After conservative treatment failed to relieve symptoms after several months, I have been referred to a surgeon. Why am I having leg pain and tingling without nerve compression? Doctor: Hi,It seems that you might be having Sciatic pain giving this problem.If there is no compression on nerves but might be having some irritation giving this problem.As there is no nerve compression, conservative line of treatment will give you relief.Go for physiotherapy like back extension exercise, short way diathermy and Lumbar traction.Avoid weight lifting and more bending movements.Take Vitamin B1, B6 B12 shots or capsules.Ok and take care."
},
{
"id": 63266,
"tgt": "What causes painful lump on the side of the spine?",
"src": "Patient: I have a painful lump on the left side of my spine near the T12 L1. It feels like the size of a dime. It is sensitive to touch, and hurts most when I turn my body to the right, or when I bend over. I also have pain radiating down my left side, and on my left shoulder. Doctor: Hi, dearI have gone through your question. I can understand your concern. You may have some soft tissue tumor like lipoma, neurofibroma, schwanoma or dermatofibroma. You should go for fine needle aspiration cytology or biopsy of that lump. It will give you exact diagnosis. Then you should take treatment accordingly. Hope I have answered your question, if you have doubt then I will be happy to answer. Thanks for using health care magic. Wish you a very good health."
},
{
"id": 59527,
"tgt": "Diagnosed with fatty liver and cholelithiasis. On medication. Recovery period?",
"src": "Patient: Hi im a 23 old female im a nurse by profession .... I m 5ft6in tall and weight 85 kgs i have put on 20kg weight recently and have been diagnosed with fatty liver grade 2 and cholelithiasis . I m trying to loose weight but nothing helps.i m not a foody person i have put on weight just due to hormonal imbalance.i am on medication for that but let me knw how can i correct fatty liver ?????? Doctor: Hi welcome to HCM Some people with excess fat in the liver simply have what's called a fatty liver Although this is not normal, it is not serious if it doesn't lead to inflammation Two types of fatty liver 1) alcoholic 2).non alcoholic Cause include-- high cholesterol or triglycerides diabetes ,Medications ,Viral hepatitis, Autoimmune liver disease, Rapid weight loss ,Malnutrition Overgrowth of bacteria in the small intestine may be associated with nonalcoholic fatty liver disease Fatty liver disease is often silent, producing no symptoms, especially in the beginning. If the disease advances -- which is usually over a period of years, or even decades -- it can cause v problems such as: Fatigue ,Weight loss or loss of appetite ,Weakness, Confusion, impaired judgment, etc As you are overweight, do what you can to gradually lose weight -- no more than 1 or 2 pounds a week Eat a balanced and healthy diet and increase your physical activity. Avoiding unnecessary medications can also help slow or reverse the disease take care"
},
{
"id": 23114,
"tgt": "Suggest treatment for 'white coat hypertension'",
"src": "Patient: Hi doc. im 45, 5'6\", 135 lbs, and had a slight over the average for normal blood sugar result for about 5 yrs. now. My mother is with genetic diabetis. My father died at 72 with an aquired diabetis. My problem does not regard my being partially diabetic, because as far as I know eversince I was a child I easily get nervous on just anything. When I was in my primary years they usually asked me in school to sing a song everyday and during school programs. i feel nervous of those times. when I get older and needs to get my lab tests done, I feel a strong heart throb and the worst during getting my blood pressure result. I dont believe it is the usual \"white coat syndrome\" doc. I need help. I am embarrassed to our company medical team whenever I undergo blood pressure tests. Is there treatment for this doc? please please help. thank you Doctor: Hello!Welcome on HCM!I carefully passed through your question and would explain that your symptoms seem to be related to anxiety. White Coat syndrome causes a raise in blood pressure with the same mechanism (the anxiety and fear when consulting with the doctor). Anyway, you should know that those episodes of high blood pressure are not harmful to your body and do not need to be treated. I would only recommend you to closely monitor your blood pressure frequently during the day (sitting position after 10minutes of total relax) and write those values down. If your blood pressure values are persistently high during the day (above 140/90mmHg), you should consult with your doctor and perform further tests to investigate for the possible causes. Meanwhile, I would recommend some lifestyle modifications to help maintain a normal blood pressure: - avoid salt and caffeine intake- perform a lot of physical activity (walking, aerobics, yoga, etc.)- try to lose some pounds if you are overweight. Hope you will find this answer helpful!Wishing all the best, Dr. Iliri"
},
{
"id": 106472,
"tgt": "What could be the cause for chronic back pain after three surgeries?",
"src": "Patient: I am an 85 yr,old female with chronic back pain after 3 surgeries, why do I have so much trouble getting 5mg hydrocodone 3.25 acetaminaphine 3 pills a day amonth at a time. I used to be able to get 3 months and then a visit with my Dr. before renewing the 3 months. What could have changed? Doctor: Hi, Along with the present medication, you also do some hot fomentation on the affected area like use palms to make warm by rubbing and place over the back area. Also, take some laxative medicines that will defenitly helps you to get relief. Hope I have answered your query. Let me know if I can assist you further. Regards, Dr. Christy J Thundiparambil, Ayurveda Specialist"
},
{
"id": 73672,
"tgt": "What causes persistent sharp pain in the mid chest?",
"src": "Patient: I have been working out and running for several weeks now and this is the first time I have been running since middle school. I ve notice the past two days a slight sharp pain when I inhale all the way. The pain is in the center of my chest. Is this like a pulled muscle surrounding my lung? Doctor: Thanks for your question on Healthcare Magic.I can understand your concern.Yes, possibility of pulled muscle is more likely.So apply warm water pad on affected areas of chest.Take simple painkiller and muscle relaxant drugs. Avoid heavyweight lifting and strenuous exercise. Avoid sudden jerky movements of chest. Don't worry, you will be alright with all these.Hope I have solved your query. I will be happy to help you further. Wish you good health. Thanks."
},
{
"id": 103615,
"tgt": "Throat congestion, shooting pain in nostrils. On medication for allergies. Treatment?",
"src": "Patient: hi i have been having problems with my throat for three weeks now and i was wondering what it is my throat keeps closing up and when i sneeze its like the right side is being blown up has pain shoot up from my right tonsil to my whole throat to my nose i am very worried i thought it was allergy s and took allergy medicine and its not helping what is it and how can it be treated and what can i do? Doctor: THESE ARE SINUS ALLERGIES AS POST NASAL DRIPOF SINUSES CAUSES SOAR THROATAND INFECION CAUSES PAIN I NOSE AND THROATGET XRAY PNS WATERS VIEW AND GET TREATMENT OF SINUSES AFTER CONSULTATION WITH DOCTORNO NEED OF ANTIBIOTICS IN ALLERGIESTO COUNTER INFECTION UNEED DOXYCYCLIN 100 MG BD FOR 10 DAYSWHICH CLEARS SINUSES FASTTAKE ANTIALLERGIC FEXOFENADINE 120 MG BDFOR SHOOTING PAINS ADD FLUNARAZINE 5 MG AT NIGHTAPPLY NEOMYCIN H EYE OINTMENT IN NOSE BDSEA WATER 2 DROPS AT NIGHT EACH NOSE ADD ANTACID GELFOR PAIN ADD DICLO SOSNEED 3 WKBUT BECAUSE OF RECURENCE GET DIAGNOSED AS ADVISED ABOVE AND TREAT"
},
{
"id": 211394,
"tgt": "Have Fibromyalgia and was shaking like convulsion. Told to have anxiety disorder. Is it related to fibromyalgia?",
"src": "Patient: I have Fibromyalgia and today I was in the ER due to uncontrollable shaking, like having a convulsion. This is the fourth time it has happened. But the first time I have been seen for it. Was checked out and told I have an Anxiety Disorder. Is this related to the Fibromyalgia? Doctor: HiFibromyalgia can cause Anxiety symptoms in two ways .one of causes of this conditin is psychological so there is Anxiety anxiety can occour as a coexisting condition independent of itTake careDr Lal Psychiatrist"
},
{
"id": 171412,
"tgt": "What is the treatment for stomach ache and headache?",
"src": "Patient: hi my name is julie. my six year old girl was having stomache ,headache and whitish substance around her inner lips. went to see a doc and was told it was ameoba after a lab test. she was given cyclopam suspension and zagole 200mg/5ml. mouth wash for the mouth due it was fungal. its been five days but all the systems are intact. im worried i dont know what to do any more. i do boil drinking water. any suggestion please Doctor: Hi, since your child has amoeba infection the treatment that your doctor gave is correct. There have been 5 days and there is no improvement at lip edges so you should apply an antibacterial lotion like Zytee lotion for 5 days. Also, if symptoms of pain abdomen present then antibiotic should be given. I hope this has helped you. If you have any more questions, i will be happy to help you."
},
{
"id": 113258,
"tgt": "Suffering from severe back and neck pain. MRI shows Disc bulge at L5. Permanent solution?",
"src": "Patient: I am pharmacist own a medical store and works for more than 13hrs a day. I have done done discactomy in late jully 2007. This ear I am in sever back & neck pain . I have done an MRI scan whitch shows disc buldge at L5 S1 level in month of jan 2013. I culd not do my regular work. Pleas advise me an effectiv & PERMONANT treatment . Doctor: Hi friend, Welcome to Health Care Magic Surgery and fixation is the only permanent solution. You should see an orthopaedician, specialised in Spine Surgery. Take care Wishing speedy recovery God bless Good luck"
},
{
"id": 42626,
"tgt": "What is the endometrial thickness needed for implantation of donor embryo?",
"src": "Patient: hi doctorMy name is Im,mu and I am 33 yrs old and infertile for last 10 years. We are planning to do an IVF with donor embryo next month. May I know the endometrial thickness need for implantation and success of IVF with donor embryo. Is there any way we can assure that IVF is performing with fresh embryo not a FET.Thanks Indu Doctor: Haiwelcome to hcm the ideal endometrial thickness for implementation of embryo will be 8-9 mm.usually the donor embryo will give good success .embryo selection should be done inside lab.no methods will identify the fresh or frozen embryo. Consult your infertility specialist to clear your doubt s.RegardsDr. Vanitha devi."
},
{
"id": 94115,
"tgt": "Recovering from umbilical hernia surgery with mesh repair and abdominoplasty. Abdominal pain wit bulge in the stomach. How long should I wear the belt?",
"src": "Patient: I had undergone Umbillical Hernia surgery with a mesh repair and abdominoplasty on 20.04.2012. I am still wearing abdomin belt. Untill last month I was not bending and doing any sort of work which involves bending. But in case something falls down i just take it by sitting in a chair and then taking those item. Very often there is a mild pain in my upper abdomin. And moreover my upper abdomin is bulged than my lower abdomin where the surgery is done. How long should i still wear abdomin belt ? Why is my upper abdomin paining when i do some work bending. THere is also slight pain in the lower abdomin towards right side. What all precautions i need to take. Pl. advise. Doctor: Hello! Generally it takes 3 months for the scar creation which is strong enough to hold your abdomen. However complete scar creation process and tissues remodeling is over after a year from the surgery. After that time, if there is no hernia, abdominal belt should be useless. However if it is still necessary in your case, it is possible that you still have a hernia present. That is why I suggest you to visit general surgeon, have physical examination and ultrasound if any doubts. You should also know that when you do wear abdominal belt, back muscles (so important for spine stabilization) are getting weaker and weaker. So such belt should be used only temporarily (when lifting or bending). And after 3 months of surgery it should be not used at all. Hope this will help. Regards."
},
{
"id": 206797,
"tgt": "For what am I prescribed Fluanxol 100 mg?",
"src": "Patient: Im due for 100mg fluanxol today. i want to stop it because of major side-effect that last about2-3 days. Do the math it around 78 days in a year.I'd love to come off it. When i ask what is does not one dr answered me. I need guidance. Would i have to be put in detox to do this? Doctor: Hello,Fluanxol is an antipsychotic medicine. It usually come in 0.5mg and 3mg tablet preparations. You can stop it directly. If an antipsychotic is to be taken, take a safer one like olanzapine or risperidone.Thanks."
},
{
"id": 82,
"tgt": "Is pregnancy possible through non penetrative sex?",
"src": "Patient: seven days after my period finished i make sex with my boy friend .. i was naked and he with jeans pants and underware ... he sat on chair and i sat on him .. when we stopped i noticed a dry spots on his pants . could the sperm pass from his pants to my vagina? could i be pregnant? Doctor: Hi, Thanks for the query. I understand your concern. Non penetrative sex can not result in conception. Even in case like yourself.. though vagina was open & exposed.. semen passing through clothes worn by the boy... would hardly have any active sperm & no sufficient seminal fluid to allow sperm to move. So don't worry. & don't forget to use at least barrier devise before to enter such risky moment.. if you are not prepared for a baby. Thanks"
},
{
"id": 204050,
"tgt": "What might the problem be unable to withstand the erection for a long time which started after the damage to lower lumbar in an accident?",
"src": "Patient: I am a 19 year old male, recently involved in a car crash, no serious damage just minor back (lower lumbar) injury.. my sex life has not been the same since the accident, my mom says it is a pinched nerve because i cant seem to keep anerection long enough to finish, i can have an erection but it isnt full and if it goes away it is impossible to get another one.. i need some help with what it might be.. i know it could be ED but before the wreck i didnt have this problem Doctor: Hello,After injury, there may be a damage to your nerves in spinal cord.That's why you have erectile dysfunction.Spinal cord injury (SCI) has a significant impact on erectile dysfunction.Treatment modalities for individuals with SCI are similar to treatments for those without Spinal cord injuries.Stress and anxiety may add the fuel in the fire and thus consult a psychologist.Usual treatment options are1. Sildenafil (Viagra), Tadalafil (Cialis), Vardenafil (Levitra, Staxyn) These drugs enhance the effects of nitric oxide, a natural chemical your body produces that relaxes muscles in the penis. This increases blood flow and allows you to get an erection in response to sexual stimulation.2. Alprostadil self-injection into the base or side of your penis. Each injection generally produces an erection that lasts about an hour. 3. Penis pumps, surgery and implants- if medication did not works.further1. If you smoke, quit.2. Lose weight-Being overweight can cause or worsen erectile dysfunction.3. Exercise regularly - this educing stress, helping you lose weight and increasing blood flow.4. Avoid alcohol- Drinking too much or taking certain illegal drugs can worsen erectile dysfunction directly or by causing long-term health problems.5. Improve communication with your partner and consider couples or marriage counseling if you're having trouble working through problems on your own.Hope this helps.Please write back for further queries.Wishing you good health."
},
{
"id": 187368,
"tgt": "Need medication for chronic bad breath",
"src": "Patient: I have chronic bad breath. I take high blood pressure medicine, but I brush my teeth daily and sometimes twice a day. I scrap my tongue every day. I use the nettie pot three times a week. I drink water all day. I visit the dentist for cleaning and exams twice a year. I use acid reflux medicines once a day. I don't have sinus infections, but I have been diagnosed with a deviate septum, on a mild case because I can still breath. I don't know what else to do. Doctor: Thank you for asking! Bad breath is also called halitosis and many a reasons from acid reflux to dental hygiene can be the reason. Unless there is an etiology connected here with this symptom all i can advise is a good oral hygiene most of which you are already practicing. Flossing, brushing, tongue brushing, tongue scraping, and chewing gums will help.The tongue should be brushed in a gentle but thorough manner, in a posterior to anterior direction, keeping in mind that the least accessible posterior portion smells the worst. Keep hydration status checked as it worsens when the mouth dries out (eg, at night, while fasting).Mouth washes, garlic water mouth rinses would help. Stay in touch with your dental care provider for further help and keep searching for the cause as nipping the evil from bud would be the best advice.Take care"
},
{
"id": 214068,
"tgt": "I see moss growing on my arms, can someone help me out ?",
"src": "Patient: but no one else dose they say my name and say shit about blood on walls I see things too like i see moss growing on my arms i know its not there...but its freeky as hell I can feel it too. I am really scared. can someone help me out ? Doctor: If you are still watching this place, my advice to you is to please go and see a psychiatrist immediately, if the experiences are still persisting. There are many reasons why you could be experiencing these things and as the medical student above has answered schizophrenia is one but only one of the many reasons. The most common reason, if this is a experience which is not repeating again and again,and if this is occuring just before or after sleep, is something which occurs in many \"normal\" people...called hypnogogic or hypnopompic hallucinations...our jargon for these experiences. Other reasons could be doing illicit drugs like cannabis cocaine etc, going through a severe bereavement (where very near or dear ones have died--though in your case it does not look like it) or having depression or mania or severe stress etc. Again, the advice is GO SEE A PSYCHIATRIST"
},
{
"id": 48235,
"tgt": "What causes sharp pain in kidney area?",
"src": "Patient: Earlier today I developed a very sudden sharp pain in my right kidney area. It quickly became unbearable, to the point of having difficulty to breathe and feeling nauseaus. I drank a large glass of water and laid down and eventually the pain subsided, though it remains dull and bearable. I have no history of kidney disease and my temperature has been normal throughout. My urine after the attack looked yellow but not unusual, after drinking plenty of fluids it s now clear. Doctor: HelloThanks for query.Based on the facts that you have posted it appears that you have pain in right flank due to presence of stone either in kidney or ureter.However this can be confirmed only on Ultrasound Scanning of abdomen and pelvis.Consult qualified Urologist for proper evaluation and further treatment.Dr.Patil."
},
{
"id": 127700,
"tgt": "What causes throbbing in left temple and pressure in neck/shin?",
"src": "Patient: i have throbbing in my left temple, no pain. Yesterday we had a health fair at work and my blood pressure as well as non-fasting sugar were pretty low. I have been feeling this for the past 3 days. I also feel a little bit like a pressure in the neck area and sometimes one or both my shins. Doctor: Hello,For the Patient with such Complaints, I suggest a few Lab Investigations done such as:1) FBS & PLBS2) Hba1C3) LIPID PROFILE4) THYROID PROFILEMonitor your Blood Pressure, if it is low. After getting these Investigations done, you can get back to me or at least consult a Physician.Hope I have answered your query. Let me know if I can assist you further.Regards,Dr. Mohammed Abdullah"
},
{
"id": 219991,
"tgt": "Is it safe to have a smear test during pregnancy?",
"src": "Patient: Hi my sisiter was given a smear test when she went to the hospital with stomach pain she was 23 +4 weeks a few hours later she went into labour and gave birth to a baby girl that died 47 mins after birth my question is can a smear test bring on labour and it is normaal to have a smear test while pregnant Doctor: Hi there,,Usually smear test is advised after 25 years of age in women who are sexually active. It is not advised in pregnancy unless a previous smear test was abnormal and she was asked to repeat it.Smear test are non urgent unless a previous one has been abnormal and can be postponed to be done after delivery.But, it is not possible for a smear test to cause labour to start.The smear test just takes cells from the outer part of the cervix and does not affect the pregnancy.The smear test is not done in pregnancy as the pregnancy hormones could cause an incorrect reading of the test not because it is harmful to the pregnancy.It is really unfortunate that your sister lost her baby, but preterm labour labour and delivery could be due to some other cause, which needs to be evaluated as it could be helpful for her next pregnancy.Hope this helps.Regards."
},
{
"id": 115435,
"tgt": "What causes body not to accept blood by transfusion",
"src": "Patient: Hi,I am quite disturbed for my sister. She has delivered a child after surgery on 6th oct and since then she is bleeding continuously. doctors transfused 4 bottles of blood but her body is not accepting the blood, not sure where the blood is going. Her condition is very critial. what could be the reason doctor why her body is not accepting the blood...pls help Doctor: This is a very rare and serious problem. A few more points u need to write about is she bleeding from the injection sites and the vagina? It is a condition which is called DIC(Disseminated Intravascular Coagulation). The treatment for this is Fresh blood and some medicines, along with critical support. The cause of this is not very clear, so treatment is complex, the essentials have been explained.It could be a complication of severe hypertension in pregnancy, or an infection or severe bleeding pv before or after delivery."
},
{
"id": 34691,
"tgt": "Suggest treatment for fever and headache",
"src": "Patient: Dear DoctorI am 25 years old living in Bangladesh. From the past three days I have been suffering from fever between 100 and 102 and painful headaches. The headache comes first and then the fever subsequently. Once during the afternoon and once at night. Doctor: Welcome to Health Care Magic. Thank you for posting your query.I can understand your concern.Fever with headache can be due to many reasons like viral fever , malaria to name a few.I would suggest you take Tab Paracetamol 650 mg - twice daily X 5 days.This will relieve you of fever and headache.I would also suggest you to go for blood tests for dengue , malaria and a total blood count including total platelet count.Do not worry much.Post any other query. We will be happy to help.Have a nice day. Regards."
},
{
"id": 177086,
"tgt": "Should the Pentaxim injection be given to a 4 month old?",
"src": "Patient: Hello Doctor, My baby is 4 months old and my pediatrition suggested to get PENTAXIM injection but I have searched for the injection in almost many pharmacists and they say that there is no supply of PENTAXIM injection still now and not sure when it will be supplied. When I crosschecked with my pediatrition they suggested the alternative for pentaxim is ( infanrix+hibirex) but I could nt get infanrix also in any of the pharmacists due to lack of supply. Please can you guide me where I can get the PENTAXIM injection in Bangalore Doctor: hello... reviewed your question... it is difficult to get pentaxim now a days as it is i. short supply from company... instead you can give him PENTAVAC injection... there are two minute differences...1.. it does not contain injectable polio which pentaxim comntains... so you hv to get it seperately2.. it is associated with pain and fever for two days... but not a major issue as u can hv medicine for that..."
},
{
"id": 156918,
"tgt": "What causes cold chills and high fever while undergoing chemotherapy for colon cancer?",
"src": "Patient: My best friend is 38 and she was recently diagnosed with stage 4 colon cancer. She has taken the first 2 treatments of chemo and she is on a steroid but she just recently started having major cold chills and high fevers. What could be causing these chills to be sooooo bad and the fevers to spike sooooo high? Doctor: your friend immediately needs medical attention. She must be in sepsis, that means infection. while on chemotherapy, immunity is low due to low blood counts and susceptibility to infections is high. She needs immediate medical attention and antibiotic coverage"
},
{
"id": 212717,
"tgt": "Feeling dizzy, stomach pains, phobia of germs. Could this be vertigo or mental illness?",
"src": "Patient: Hi, My mum has been off work for over two years feeling dizzy and sick, the doctors first said it was vertigo but now believe it s down to depression , she also has stomach pains and is very erratic and will kick off at the slightest thing, its as if she likes to argue and she over exaggerates things, she also has a phobia of germs, toilets and being sick, she constantly asked us if her hands are clean and if the washing is clean, when she does the washing if it goes near the toilet or if she walks past the toilet with it and the doors open she thinks it s not clean. Could this all be down to her depression or does she have a mental illness? Doctor: Hello and welcome to Healthcare Magic. Thanks for your query.Depression is a form of mental illness. It can present with various psychological as well as somatic or bodily symptoms. In addition, from the description of your mother's symptoms such as phobia of germs, obsession with cleanliness, etc. it appears that she may also be having an obsessive-compulsive disorder (OCD).Both depression and OCD are treatable conditions and usually there is a good response to anti-depressant medication. There are also effective psychological therapies and counselling techniques which can help her overcome these problems. So, please take her to a psychiatrist for a detailed evaluation and further treatment.Wish you all the best.Regards,Dr. Jonas SundarakumarConsultant Psychiatrist"
},
{
"id": 97456,
"tgt": "Does ayurvedic medicines help in treating gastritis?",
"src": "Patient: Good afternoon, My wife is suffring from Gastric. She has given Mukta shakti vhasma, Kam dudha ras, giloy sat, mukta tishti, avipatikar churn, udaramrit vati. She has pregnent now. can we continue this medicine ? she has also suffring from worm now, can we give her medicine of worm ? Doctor: Your description is incomplete as word gastric is not indicating exact problem, whether there is indigestion or flatulence or hyperacidity or pain or gastritis is there or is there formation of any gastric ulcer etc. The medicines you are using are in majority are antacids and immune boosters. Worms are main reason for abdominal upset. Yes you must treat them and you can use medicines for worm eradication."
},
{
"id": 161707,
"tgt": "Can BVP treat moderate pulmonary valve stenosis?",
"src": "Patient: My son is 5 months old. His Echo report is : Moderate pulmonary valve stenosis noticed. PPG : 50 mmHg. Pulmonary valve thickened and domed. pulmonary valve annulus 8.5 mm. mindly dilated RA & RV. Mild TR noticed with PPG 17 mmHg. Mild PR noticed. Mild RVH. Intact IAS and IVS. No PDA or coarctation seen. No intracardiac mass or vegetation seen. All other valves and tracts are normal. No pericardial effusion is seen. Good bi ventricular function. He advise me to BVP. my Question, is he totally cure after BVP? or any complication arise in his future life? Doctor: Hi, BPV is the treatment of choice for in such cases of pulmonary stenosis. The success rate is very high and as such long term complications are very less if the procedure goes uneventful. There are chances of restenosis or residual stenosis in 20-3- % cases. But as RA/RV is dilated and RVH is there BPV is recommended. Hope I have answered your query. Let me know if I can assist you further. Take care Regards, Dr Pradeep Kumar, Pediatrician, Cardiology"
},
{
"id": 214135,
"tgt": "Dont want baby now, please tell us what to do ?",
"src": "Patient: I am father of 2 months and 3 weeks baby girl yesterday night we both done intercourse without precaution, please tell us what to do and we dont want another baby now? Doctor: Hi.. If your partner is breast feeding, then she would be naturally protected against getting pregnant.. This is called as\u00a0 lactational amenorrhea in medical terms.. This natural protection is confirmed to be active present,\u00a0 if she has not started her periods. But if\u00a0 she has started having her periods, which would indicate absence of this protective mechanism,\u00a0 then you may have to use the emergency pills to prevent pregnancy.."
},
{
"id": 89200,
"tgt": "What causes frequent urination , abdominal pain and fatigue?",
"src": "Patient: Hi I am a 39 year old female. I am 2 days into my menstrual cycle. After several hours of cramping and a hot bath I passed a lot of blood, a huge clot and a bean sized encapsulated mass. The mass is soft. The tissue is slightly transparent with what like like little blood vessels in it. It literally looks like a bean. Any thoughts on what it might be? I had my tubes tied over 12 years ago. I have also had symptoms that I thought were gastrointestinal, but tests at the ER ruled out colon problems and gall bladder. Symptoms are: frequent urination, Abdominal pain and bloating, difficulty breathing, hot flashes, fatigue. I also fill up quickly after eating. I've recently cut out gluten thinking the bloating and pain were related to a gluten sensitivity but the pain and bloating persist. Any suggestions?Thank you Doctor: Hi. The symptoms of requent urination, Abdominal pain and bloating, difficulty breathing, hot flashes, fatigue. I also fill up quickly after eating are suggestive of a genearlized disease like enteric fever or so. You have to get all the relevant blood, urine and stool tests as per the clinical findings of a Doctor on clinical examination. You will need an antibiotic course and supportive treatment. The encapsulated mass line a bean can be in fact be a polyp of the uterus. Since your tubes are tied for such a long period , it is not likely to be fetus. So nothing to worry about. Get an ultrasonography done as this will help for the diagnosis of the genito-urinary system, PID or any other findings."
},
{
"id": 94517,
"tgt": "Extreme upper abdominal pain. No relief with TRamadol, Trazadone. Solution?",
"src": "Patient: I ve been having extreme upper abdominal pain for the past two weeks, with very little relief. I have gone to my doctor who performed an endoscopy , and tried a colonoscopy twice. I did the prep twice and to no avial. The second time, I did go to the bathroom, thought I was cleaned out, but he said I was completely full when he went in. The pain is unreal, and I can t seem to get relief. I ve been taking Tramadol 50 mg, and Trazadone, which no longer work. I m starting to think I m crazy, no one will listen to how much pain I m in. Doctor: Hello! Thank you for the query. Upper abdominal pain has rather nothing to do with large intestine disease (if no bowels obstruction). Such pain can be caused by gallstones or pancreas disease. That is why, what you should have performed at first before any endoscopy is abdominal ultrasound, full blood work, liver tests (AST,ALT,GGP), amylase, bilirubin, urine analysis, abdominal X-ray. If gallstones are causing your pain, hot bath should ease it as well as soft muscle relaxing medicines. Hope this will help Regards."
},
{
"id": 109065,
"tgt": "Suggest treatment for lower back pain",
"src": "Patient: I have been having lower back pain for a few years off and on. I'm in the marine corps so it's not uncommon. Over the last 6-8 months it's been getting progressively worse. It's a dull pain throughout the day, but sometimes when I lay down and relax my back it's the pain is very sharp and intense. I get that same result when I lean on to a counter and let my lower body relax and let the weight of my legs strech my lower back out. Any ideas? Doctor: HelloI think this is most probably due to disc prolapse or spasm of back muscles. You should go for MRI lumbar spine to delineate disc condition.Try to consult a physiotgerapist for back strenghtening exercises.Take muscle relaxant and a pain killer medicine when needed.Try to maintain a correct posture while you work.Thanks"
},
{
"id": 64796,
"tgt": "Why do lumps form on legs?",
"src": "Patient: I have small lumps pimple-like on my legs that have little white granules in them and no fluid. Once I get the little white grains out they go away. I will get several then once break them open and scratch out the little bumps inside i wont get them again for some time. What are they? Doctor: Hi,Dear,thanks for query to HCM, from you for your lumps on legs.I studied your query in depth.1-In my opinion ,these lumps on the leg are White head boils,died down boils/or folliculitis, your leg appears to be hairy and thats the reason you have multiple bumps due to the dried down boils on the legs.2-Hope you got the answer you needed.3-Wellcome for more query to HCM."
},
{
"id": 86260,
"tgt": "Suggest remedy for pain in lower abdomen",
"src": "Patient: I am 17 days late, I m having rib back and hip soreness, I m having a discharge Chang. My lower left abdomen pain below belly button, I have taken two dollar pregnancy text both negative, the Er took blood about three weeks ago and negative, I m moody and now sleeping in I m craving weird things and spicy food and milk, should I see a doctor. I must inform u I tried for a child so could I be pregnant or something else? Doctor: Hi,Dear,Thanks for your query to HCM.Studied your query in full depth of its details.Reviewed it in context of your health concerns.Based On the facts,You seem to suffer from pregnancy obsession.Check with your Gynec and pshyatrist doctor for its treat.This reply would help you to plan further treatment soon with your treating doctors.Best of Luck and early recovery.Welcome any further query in this regard,which would be replied in next session.Good Day!!Dr.Savaskar M.N.Senior Surgical SpecialistM.S.Genl-CVTS"
},
{
"id": 138111,
"tgt": "How to cure sharp stabbing pain on the right calf?",
"src": "Patient: Hello I have been getting a sharp stabbing pain in my right calf.... it only last a few seconds but can go on all day and night then it goes away for weeks or months. There is no swelling and no pain to tough it.... but the pain when it happens is very intense. I spent all of last night away with this happening every few minutes. Doctor: Hi,Thanks for your query.It seems that you are having pain in your calves specially after walking. It can happen due to-1. Calf muscle strain and tear - Usually due to overwork and overstretching of the muscles. Recent increase in leg exercises may cause these. Gradual increase in exercise and pain killers with mild anti inflammatory drugs can help +in reducing the symptoms.2. Muscle ischaemia- associated with diabetes usually. There is acute pain associated with it.3. Deep vein thrombosis- comes last among the list of probable diagnosis. You may get the initial physical examination, relevant tests and treatment done under care of 'Orthopaedician'.I do hope that you have found something helpful and I will be glad to answer any further query.Take care"
},
{
"id": 73314,
"tgt": "What could chest pain with hard breathing indicate?",
"src": "Patient: I have not had a drink in 24 hours but have a horrible pain in the middle of my chest that goes thru to my back and under my arms. it is hard to breathe. I slept most of the day to get rid of my hangover and now have this pain. it's pretty bad. I drank some mylanta thinking it was heartburn it did not help. any other ideas? Doctor: You should seek medical attention immediately. Pain like this can be serious and causes can include tear of the esophagus due to damage by alcohol and vomiting or an ulcer in the stomach. It can also be a tear in a blood vessel or a heart related problem. You should go to the emergency room to have these symptoms evaluated and exclude serious causes. Best of luck."
},
{
"id": 99681,
"tgt": "Suggest treatment for allergic bronchitis",
"src": "Patient: I AM A RENAL TRANSPLAT PATIENT SINCE 2003 NOW I HAVING ALLERGIC BRONCHITIESALREADY HAD TWO COURSES OF ANTI BIOTIC ( TENOVIC 250) AND NOW ON CEFTUM 250IT IS TAKING LONG TIME CURE ( NOW 3 WEEKS)WHT TO DOX RAY SHOWS CLEAR CHEST Doctor: Hello,Thank you for asking at HCM.I went through your history and would like to make suggestions for you as follows:1. I would like to know more about you like your age, reason for renal transplant, any medications, any other medical conditions, allergic bronchitis since what age, how was diagnosed, whether a smoker, your environment, any nose symptoms, any eye symptoms, any hospitalization, etc etc.Above details would help me to understand your complaints better and therefore to make suggestions for you more specifically.2. However, at present, I would suggest you allergy testing for common air-borne allergens like house dust mites, molds, pollens, insect proteins, pet dander, etc. This will help you identify the substances causing allergies to you as well as to know the measures to avoid them.3. For symptom relief, I usually suggest my such patients to use salbutamol/levosalbutamol inhaler to be used on as-and-when-needed basis.4. Regular breathing exercises and steam inhalation may also help in a long run.5. In general, please avoid exposure to dusts, smokes and air pollution as much as possible.Hope above suggestions will be helpful to you.Should you have any further query, please feel free to ask at HCM.Wish you the best of the health ahead.Thank you & Regards."
},
{
"id": 211187,
"tgt": "What to do if am paranoid and isolating myself and have trust issues with people?",
"src": "Patient: Hi. I just did a test and it claims I am paronoid. i got 22. Not sure what this is out of. but it says I should ask a doctor. I feel my paronoia is ruining my life. I hav isolated myself from certain people an dI run away from people I feel are out to get me. My gp told me to join socities in college and all I have done is just isolated myself from the entire year. I have lots of friends that are loyal but I have trust issues with people outside of that. PLease help. how can I improve the way I behave? Doctor: hithanks for using healthcare magicI think u got personality test and base on that personality test ur personality may be borderline personality. In that case this paranoia is part of ur personality. U should consult a psychologist who would help u improve ur personality. It would take 6 month to one year or may be more so dont be in hurry. Thanks"
},
{
"id": 148751,
"tgt": "Narcolepsy, masturbating in sleep. Naming stranger during episodes. Help",
"src": "Patient: I masturbate in my sleep and have been diagnosed with narcolepsy . My doctor has prescribe me something to take at night for the movement issues including the masterbation, twitching, etc. I also have another concern, I say another mans name during these episodes and it really hurts my husband. The name I say I do not know this person. And I feel my doctor is not taking the issue seriously as it is greatly affecting mt marriage. Please help before my husband leaves me over this. Doctor: HiEvery thing which you are doing in sleep is in REM phase better known as dreaming sleep It is well known that physical activites some time comlex one are performed in this phase of sleepIn my experience no drug works as most of the drug are known to increase REM phase .Some time i try SSRI anti depressant REm sleep physical activitiesAs you perform masturbation and take some male name as per Great Freud Sleep and dreams theory you want sex at that time or want sex begore sleep or dreaming about sexual activity at that time which you may not temeber afterwards Nothing wrong or abnormal The husband is best help He should have sex before you go to sleep and if he notices you doing such an act during sleep he should wake you up and induge into sexual or mastutbating activity with you thus helping you out of your unconscious feelings He should immediately make you to cry for his name lastly remove your guilt feeling talk to your husband he will unfestand and you will happy married life If required. Please do take help of psychiatrist where both of you discuss with psychistrist the situation and solutionDr Lal Psychiatrist"
},
{
"id": 117026,
"tgt": "How long should i take medication for high BP and vertigo?",
"src": "Patient: i AM A PATIENT OF HIGH BP AND VERTIG0 ( CERVICAL SPONDI). PRESCRIBED MEDICINE IS A) PROLOMET 25/5 AND VERTIN16 TWICE A DAY AND STUGERON FORTE HALF TAB PER DAY. IS PROLOMET 25/5 IS A COMBINED MEDICINE FOR CHOLESTEROL. HOW LONG SHOULD I TAKE THE MEDICINE Doctor: Hi, dear. I have gone through your question. I can understand your concern. Your medication is for blood pressure only. You have to take it for life time. Vertin is for your vertigo. Yiy should go for physiotherapy and vit b12. Once your vertigo is controlled you can stop taking vertin.Hope I have answered your question, if you have doubt then I will be happy to answer. Thanks for using health care magic. Wish you a very good health."
},
{
"id": 163300,
"tgt": "How can a skin rash during teething in infants be treated?",
"src": "Patient: My grand baby is 3 months old and she has rash around her neck and under her legs and around her neck because she is teething and sweats a lot what can I do about this now it seems like her skin is peel around her neck YYYY@YYYY Doctor: Hello,In my opinion, all you can do is to use warm naps to clean his neck and skin because his skin is very delicate. You may also a menthol powder to keep his skin dry.Hope I have answered your query. Let me know if I can assist you further.Regards,Dr. Elona Dashi"
},
{
"id": 133537,
"tgt": "Suggest remedy for severe fatigue and weakness in leg",
"src": "Patient: I ache all over and have a weakness in my legs when I walk. It feels like they are going to buckle on me sometimes when I walk. This is a new condition over the last couple of weeks. Advil helps, but the conditions comes back when I don t take it. I am on no other medicine Doctor: hi,thank you for providing the brief history of you.As you feel that there is no motor control that it could be a nerve trapping in its course. As the nerve has the dual functions of motor control and sensory . So any entrapment of the nerve will lead to such Buckling of the knee while gait cycle.Also, performing exercises under physical therapy should provide you better control of legs. Working out behind the core and the lower limbs should provide maximum control.Also, in my clinical practice such patients are common and with physical therapy they respond well.RegardsJay Indravadan Patel"
},
{
"id": 1716,
"tgt": "Is the weight gain a sign of bloating?",
"src": "Patient: My husband and I have been trying to conceive after suffering a miscarriage in May. I am 9DPO I have mild cramping, a back ache and my breast are sore, but not as much as I feel they should be. I've also gained weight. I haven't changed my eating habits at all but I've gained 5 lbs--I swear, over night! Is the weight gain a sign of bloating. I've also have a lot of trouble sleeping at night. I just toss and turn...could this be promising of being pregnant? Doctor: hello..thanks for trusting the healthcare magic doctors for ur health related queries..I guess ur concern is about..is the weight gain a sign of bloating..right.my answer is...NO. weight gain is not at all the sign of bloating.let us discuss about the sign n symptoms u have. as u told that u had gain weight n having difficulty in sleeping. so u might be have PCOS (polycystic ovarian syndrome). in this patients usually have irregular menses, weight gain n difficulty in conceiving. few patients having difficulty in sleeping, excessive hair growth in unusual area of the body etc. but didn't give info regarding ur menses whether it is regular or not. so it is difficult for me to comment on this.lastly i want to say that weight gain is not the sign of bloating.with the above discussion u can try out my suggestions-1. first u should reduce ur weight because it may pose difficulty in conceiving because it causes hormonal imbalance which causes irregular menses n patients having difficulty in conceiving.so try to change ur food habbit in a way to reduce ur weight.2. for more details u can visit ur nearby gynecologist n feel free to ask about ur concern. he/she can help u.3. try to do coitus during fertile period(it is from 10th to 16th day of menses during this period chances of getting pregnant is high. so do more coitus during this period.u may get better results.4. DON'T WORRY about the miscarriage u had in past. it doesn't effect ur future chances of getting pregnant.5. if u r still in hurry then visit nearby infertility specialist. he/she can help u.I hope the above information is useful, informative and helpful for u.regards-Dr sudha rani panagar( DON'T WORRY...be happy.every problem have a SOLUTION. so KEEP PATIENCE..be positive.I pray the God that u will be blessed with a cute n healthy baby...hopefully TWINS)"
},
{
"id": 175755,
"tgt": "Can Nido baby formula be given to an infant?",
"src": "Patient: My daughter is going to be 6 months in a week and I recentley started giving her a formula called NIDO.. I ve been giving her half of her regular formula called similac and half NIDO so that she can get used to it and not just jump right into it.. Is it okay to be giving her this formula??.. Because my mother told me that it has honey and when she worked as a nurse she remembered the doctors would say you shouldn t give babies honey because they could have an allergic reaction.. Thanks Doctor: as your daughter is 6 months she started with weaning foods to begin with after completion of five months begin with liquids like top feed milk,soup,juice then gradually start semi-solid by six months about formula please continue only one formula and gradually start as advised above"
},
{
"id": 127365,
"tgt": "What causes numbness on the upper lip?",
"src": "Patient: For the past two mornings, I ve had numbness of the right, upper lip...it comes on slowly...soon after awakening. It also affects my right nostril nare & a small area of my face around these areas. I m a 54 year-old female who is in excellent health (according to my doctor). I exercise & eat a proper diet. I am hypothyroid & have been taking medication for it for several years. I get some blood work done twice a year (per my doctor) & have a healthy lifestyle. The symptoms do not last long...roughly 10 minutes from onset to conclusion. I have no other health issues. Doctor: Hello and Welcome to \u2018Ask A Doctor\u2019 service. I have reviewed your query and here is my advice. It can be due to the use of some bug bite or use of chemical cosmetic, allergic reaction, dental infection(gum inflammation), Shingles or it can be a symptom of developing Bells Palsy or neurological problem. Take antihistamines to rule out any allergic reaction. Visit ER, get physically examined. If it is Shingles or any other infection you will require medication. If it is developing neurological problem or Bell's palsy you need immediate medical attention, don't delay, get evaluated. Hope I have answered your query. Let me know if I can assist you further. Regards, Dr. Nupur K."
},
{
"id": 165803,
"tgt": "What causes fever and red spots on whole body in a child?",
"src": "Patient: My 1 1/2 year old had a fever on 3 4 and 5 with a temp of 102 &103 I gave her ib and tylenol, fever went down but she dif throw up fri and sat. Then on 3/6 no more fever, on 3/7 she had red spots all over her body, I called urgent care spoke to nurse and she said it could be.a viruse thats leaving her body or chicken pox, she said.if chickems.pox I will know the next day. Well tuesday she had even more red satpots everywhere, thr said to bring her in, the drs and ped,.saw white spots in her mouth anf said she might have a small case of measels they took blood and urine, and health dept.will call me with results, I got a call no measels, then got a call from drs officr they said maybe she haf a allgergiv reation from something, they told mr to givr her benefraly and that did not work, they really dont.know what.it.is, red spots are coming and going? What do yoh think it could ne? Doctor: Dear parent,If there is no more fever and only rash is present which appeared after fever subsided then it can be some viral exanthem like roseola infantum.IT IS A SELFLIMITING INFECTION.It will subside on its own .proper hydration and supportive treatment is enough.But u have mentioned that the rash is coming and going,in that case it can be an allergic reaction too.In this case identify and avoid the allergen would be first and then anti allergy medications like cetrizine,rantac and steroids can helpso she needs clinical assessment before starting the treatment.But one thing can be confirmed that it's neither chicken pox nor measles as there is no fever when rash appeared"
},
{
"id": 131401,
"tgt": "Suggestions of advice for having crooked hips",
"src": "Patient: i was working under the house in a crawl space and was sore that night. The next morning i woke up with crooked hips and look like the leaning tower... It fells like my right leg got shorter about a 1/2 inch. i went to The Joint which is a fast food type Chiropractor service... They straightened me out pretty good and was fine that day. The same thing came back that same night. I went to them again and again they fixed me temporarily but it is back... They said my spine was fine and straight... I have been taking muscle relaxers and doing stretches but nothing seems to help.... Any suggestions? Doctor: As I understood it seems like you had some kind of muscle spasm I think you should go and do some analysys for the calcium in blood and an endocrinologist exam. Thus may help"
},
{
"id": 78781,
"tgt": "Suggest treatment for tenderness in the chest and shortness of breath",
"src": "Patient: My wife has trouble breathing. Also has tenderness on her chest. She had a stress test and CT scan....both normal. She has a lung function test today and a breathing treatment didn t help. She is really frustrated. Do you have any ideas on what could be the problem? Thank you!!!! Doctor: Shortness of breath can be due lung problems and heart problems. Evaluation of cardiac cause is required. I would suggest you to get an ECG AND 2D ECHOCARDIOGRAM done. Hope i have answered your questions. If you have any further questions i will be happy to help."
},
{
"id": 156753,
"tgt": "What to do for bloating in the lower abdomen area and mild pain? Could it be cancer?",
"src": "Patient: I have bloating in the lower abdomen area, mild pain, and I am 61 years old. I still have all my female organs, and I am worried that this might be cancer. I have had pap smears for the last 10 years and nothing has ever showed up but I am still worried. What kind of testing do I need? Doctor: you get ca 125 levels and USG abdomen for further characterisation. if it is normal, then nothing to worry."
},
{
"id": 175153,
"tgt": "What causes bruising in spine in a child with mild autism?",
"src": "Patient: My 12 year old daughter has mild autism and epilepsy. She has been complaining of abdominal discomfort and headaches. This evening I noticed she has bruising along her spine starting at the base and goes about six inches up is this something I should be concerned about? Doctor: Thank you for following up.Bruising is usually result of hit due her epilepsy attack or other causes. She has lots of bruisings thats why i suggest her blood vessels are weak and easy injured.In addition we can check blood test and platelates with clotting for excluding of some haematological diseases. For speedy disappearing of bruisings you can apply small amounts heparin ointment, for example- teombophob, if she complains a pain then apply diclophanak gel for reliefAll the bestBe healthy!Dr.Svetlana"
},
{
"id": 218310,
"tgt": "What would a normal USG report at 35 weeks of pregnancy show?",
"src": "Patient: Hi I m 28yrs old and m 35 weeks 1 day pregnant .I did scan and results is single viable intrauterine pregnancy is seen with cephalic presentation and longitudinal lie. Composite gestational age is around 35 weeks 1day.placenta anterior and show grade 2 maturity what it means Doctor: Hello and Welcome to \u2018Ask A Doctor\u2019 service. I have reviewed your query and here is my advice. This report looks very normal. This is pretty assuring that things are going very well. However, if you want to understand the technical terms exactly, I would be glad to explain you. Hope I have answered your query. Let me know if I can assist you further."
},
{
"id": 145597,
"tgt": "What causes back pain after undergoing fusion of L5-S1?",
"src": "Patient: 2 years and 3 months ago I had back surgery. fusion of my LS S1. 4 screws and 2 rods. I m still in so much pain. any thoughts? I ve done injections, burned nerve endings, physical therapy, acupuncture...... I quit taking pain meds bc its no way to live long term. im only 37. Doctor: I think you need to contact the surgeon who operated you and get MRI spine done to see for any dislocation or compression."
},
{
"id": 82876,
"tgt": "Is the long term delay in the diagnosis of Encephalitis and Lupus a matter of concern?",
"src": "Patient: a friends daughter (25 years) was admitted to a pshych ward, with atypical signs, and symptions of a ? psychosis i.e no hallucinations, delusional beliefs , formal thought disorder, nor self harm ideation were apparent other concerns was that this presentation was of a : fast onset i.e virtually one day to the next : We fought to have her moved from a psych ward to a general facility in order to have her appropriately assessed and treated finally teats were carried out and during the lumbar puncture, elevated white blood cells, were discovered there is also a family history of lupus ( autoimune disorder) this person is aware of where she is; o However over time ( the last month) she has noticeably deteriorated sadly it has taken this long for a diagnosis of encephalitis to be provided. the concern here is the delay in appropriate diagnosis, thus delaying the much needed treatment for this person there are grave concerns given that it has been a month till now that this diagnosis has been provided the long term ramifications related to the delay in diagnosis and treatment are seriously concerning and alarming ; your view would be appreciated Doctor: Dear Madam,It is advisable to get the tests done early so that the diagnosis can be made early . and the appropriate treatement needs to be started at the earliest to avoid brain damage .Dr. Shruti"
},
{
"id": 151170,
"tgt": "Numb and tingling nose, fluttering above upper lip. Should I be concerned?",
"src": "Patient: My nose has a strange sensation for the past hour or 2, kind of numbish and tingling . Occasionally I have a light fluttering above my upper lip almost like a feather is tickling me. I seem to feel that more often when I m a little stressed. Are these syptoms anything to be concerned about? Also, there is a dime sized spot on my foot that burns sometimes, it comes and goes. I know that these symptoms could be related to MS but if it was, would the symptoms be continuous or would they come and go? Should I be concerned? Doctor: If you are in regular follow up for your MS then don't worry this will be minor symptoms.Just continue regular follow up and continue your drugs as adviced by your doc."
},
{
"id": 124114,
"tgt": "Can ganglion cysts on the wrist sign for rheumatoid arthritis?",
"src": "Patient: My daughter was just told she had an elevated ANA , her rheumatologist was unavailable for consultation. She showed the test results to an orthopedist who she was seeing for a ganglion cyst on her wrist,He stated it meant she had rheumatoid arthritis . Can you advise? . Doctor: Hello, An elevated ANA alone doesn't definitely mean that she has rheumatoid arthritis. To diagnose that you need multiple lab tests, the ANA is just one of them. There are many auto immune disease that have an elevated ANA, and if she is having a lot of pain she could have one of them. The good news is that there are many drugs to treat autoimmune diseases. I would try and see your PCP and see if they can help get you in to see a rheumatologist. Hope I have answered your query. Let me know if I can assist you further. Take care Regards, Dr Daniel Richard Paull, Orthopaedic Surgeon"
},
{
"id": 64278,
"tgt": "What could painful lumps around the ear & eye suggest?",
"src": "Patient: Hi Im 20 Years Old And Have A Lump On My Neck Just Below My Jawline Near My Ear It Feels Bruised Like Iv Banged It But Theres No Marks I Have Had A Bit Or Earache And Head Ache Aswell!! And I Also Have A Few Little Bumps Just Under My Eye Both On The Left Side Of My Face!? Danielle Doctor: Hi,Good Evening.Thanks for the query to HCM.I studied it in depth and I understood your health concerns.-Treatment-for-tender lumps in the armpits and nipples-1- Mostly it appears to be -Acne vulgaris/ Pimples of the jaw line below ear and below your eyes.infected boil below the nipple with lymphadenitis of the armpit.2-As your are worried I would advise you to check with your family physician,who would fix its diagnosis and would treat it accordingly.Hope this would help you a lot to relieve from your painful lumps around ear and eye.Wish you fast recovery-and healthy life.Wellcome to HCM with more queries till you are satisfied.Have a Good Day...!!Dr.SAVASKAR M.N.M.S.GENL-CVTS,Super specialist and Senior Consultant-and Expert in Non-Curable-Disease therapy for Cancer,Asthma,etc,Rejuvenation therapy and Tissue failure -reversal therapies."
},
{
"id": 29045,
"tgt": "What causes itchy rashes on the upper torso after treating the viral fever?",
"src": "Patient: Hi I ve had a virus for 9 days, got a stiff neck and had a rash on my upper torso that itches ? What is going on? Today the rash is still present, the stiff neck has subsided but it took 5 days. The rash seems to be scabbing but im not sure. Im 61 years old. Doctor: Hello,As per your clinical history is concerned, please follow like this-1) Do a clinical examination by your doctor and to confirm do [CBC with differential and mono spot test]2) Discuss with your treating doctor for a course of antibiotics to prevent secondary bacterial infection, and it should not be [Amoxicillin and other penicillin derivatives]3) Discuss with your doctor about a short course of [prednisone]Do follow lifestyle modifications like this-1) Do not share your personal items with other family members.2) Do not touch anybody by your mouth Hope I have answered your query. Let me know if I can assist you further.Regards, Dr. Uday Nath Sahoo"
},
{
"id": 97296,
"tgt": "Is morphine safe to use?",
"src": "Patient: i have excruciating pain right now due to a fall ....i have been proscribed Vicodin.... but it is not enough ......i find myself taking dangerous amounts just to get relief ........my buddy gave me a 60 morphine and i was just wondering if it would be safe to use as a 135 lbs woman and how my body may react Doctor: Hello!Thank you for the query.Every person has different tolerance to opioids (which is Vicodin and Morphine). Morphine is much more dangerous than Vicodin is. it is very easy to overdose it. Due to overdose it may cause coma and death due to breathing problems.60 mg of Morphine is quite large dosage. You definitely should not take it at once. If you have to use this medicine, please start from the 5mg dosage and if it wont help you may repeat it. 10 mg is usually enough to ease the pain.Hope this will help.Regards."
},
{
"id": 119730,
"tgt": "What causes loss of strength in hands?",
"src": "Patient: I thnk I have a recurrence of CFS (my body is drained after minor exertion), but am a little worried by a loss of power in my hands. I can t do delicate things, nor can I do things that need strength, such as opening a jar. This has been going on for a fortnight. Doctor: Hi, Sudden loss of strength occurs because of derangements of nerve supply to the muscles. It may occur at any level from brain to very junction of the muscles with the nerves. Sometimes, myopathy (pathology of muscles) itself may present as sudden weakness, but muscle coordination remains intact in case of myopathy. You need to get muscle charting done along with blood investigations focusing on markers of inflammation and myopathy. Only after this, we can reach a diagnosis. Hope I have answered your question. Let me know if I can assist you further. Regards, Dr. Rohan Shanker Tiwari, Orthopedic Surgeon"
},
{
"id": 31711,
"tgt": "Does elevated ALT levels in blood cause persistent cough and fever?",
"src": "Patient: My mother is having cough for the last 3 months.She constantly coughs all day.Its hardly when I find her not coughing.ANd every afternoon she has fever.Can you kindly suggest the probable causes? Just now in her blood report I saw that her ALT level in blood is also very high than the normal range.Her's is 127.So is it anyhow linked with her cough and fever? please reply soon Doctor: Hi, Thanks for posting in HCM. I understand your concern for your mother. Having persistent cough with fever along with elevated levels of ALT enzyme could be due to enlarged liver due to infection and irritation of lining of lungs (pleura) through diaphragm, most likely because of pleurisy or collection of fluid by means of pleural effusion. To confirm this, you need to get a Chest X-ray and Ultrasound abdomen done. Kindly consult Physician in this regard with the report for appropriate diagnosis and management. Hope the information provided would be helpful. All the best."
},
{
"id": 67981,
"tgt": "Could the hard lump on the inner thighs be due to swollen lymph node or cyst?",
"src": "Patient: I developed overnight, what I found to feel like a tender semi-hardened bump on my left inner/upper thigh. It felt tender, like a bruise. IThe feeling woke me up a couple of times in the night (but, never at all in the day before).But, in the morning, I can tell that it was tender and didn t go down. It seemed to get bigger. It was on the night of the 11th/morning of the 12th. Now, it s a few days later and the feeling is still there, just slightly bigger, but mostly, harder. I just don t see how something can just develope overnight....confused. Don t know it it s a cyst, or a lymph node...or dare I say...anything else Doctor: Hi, dear. I have gone through your question. I can understand your concern. Its mostly non serious lesion. You may have either some benign cyst or enlarged lymphnode due to infection. You should go for examination and if needed go for fine needle aspiration cytology. Then take treatment accordingly. Don't worry about malignant lesion because malignant mass can not develop overnight. So don't worry. Just be relaxed. Hope I have answered your question, if you have doubt then I will be happy to answer. Thanks for using health care magic. Wish you a very good health."
},
{
"id": 221254,
"tgt": "How to abort pregnancy?",
"src": "Patient: Dear Dr. Chetna, I am happily married since Jan 2009 and following a family planning since then but suddenly this year I missed my periods even after taking intense protection. After a self pregnancy test I got confirm about it. Now both of us don t want to move forward. Could you please tell me if it s possible to take any oral medication for pregnancy as its only 5 weeks as of now? Doctor: HiDr. Purushottam welcomes you to HCM virtual clinic!Thanks for consulting at my virtual clinic. I have carefully gone through your case, and I think I have understood your concern. I will try to address your medical concerns and would suggest you the best of the available treatment options.Yes surely, you can very well terminate the pregnancy with medications till 9 weeks of pregnancy.Please get USG done to confirm location of pregnancy.As per laws of the land where you stay, please get it done under expert medical supervision.It involves use of Tab MIFEPRISTONE 200 mg followed by Tab MISOPROSTOL 600 mcg after 48 hours. You will need to take appropriate antibiotics course for 5 days.I hope my answer helps you.Thanks.Wish you great health."
},
{
"id": 106441,
"tgt": "I can not breathe well and my heart is acting strange. why it is happening so ?",
"src": "Patient: I can't breathe well and my heart is acting strange. I want to know whats the problem.? It's not the first time. I breathe in and out as if I breathed more or less air when I breathe and I feel that I can't breathe normally. My heart pumps really fast sometimes though I'm not tired. What can this be? It's as if I breathed in and I feel as though extra air goes into my lungs, or if I breathe out I let out extra air after. I have had asthma and I haven't been sleeping at regular times but I'm not sure if it relates to my problem. But I said those just in case if it does relate. Doctor: Go to the Doctor....you know you got to."
},
{
"id": 4555,
"tgt": "How to identify pregnancy with irregular period cycle?",
"src": "Patient: ok so i recently had sex on the 2nd and my period was supposed to come on the 17th I'm late by 6 days. i do not have a regular 28 day cycle some times its more some times its less. some periods last 4 days others last 7. there are months were i skip a period and i have been late and sometimes early after having sex. so what does this mean? Doctor: Hi, thank you for asking Healthcare Magic.It is important to remember that pregnancy can occur any before menopause even if your periods are irregular. Irregular periods are caused by an imbalance in hormones that control the menstrual cycle. For me, I think the only thing you can do to know whether you are pregnant or not is to do a home pregnancy test each time you miss your period.I hope that answers your query. If you have additional questions or follow up queries then please do not hesitate in writing to us. We will be delighted to answer more questions from you.I wish you good health"
},
{
"id": 67954,
"tgt": "Suggest treatment for a lump on the cheek",
"src": "Patient: I have a small capsule sized lump in my cheek near my nose, above it I have a mentholated feeling. I have been to three ENT s. No one knows what it is. Someone told me they think it is a vein that died in my face. It seems to shrink at times and then get larger at times. I am keeping a close eye on it and it doesn t appear to be getting any bigger. Doctor: Hi...thanks for sharing your problem...lump which is soft movable and as you said it get emptied when you press it,this shows like it to be venous dilatation(venous malformation)...I will advise you to see a plastic surgeon..he will evaluate it and will give you proper diagnosis...if any question you can ask again....thanksDr ihsan."
},
{
"id": 112588,
"tgt": "Over weight, tired, lower back pain. Problem? Recommendations?",
"src": "Patient: hi I am 43 yrs old and 155 lbs I am so tired everyday I get home from work I have to lay down. I have gained 35 lbs in the last year and my metabolism has seemed to stop working, I have essesive pain in my lower back and have a hard time working my ten hours a day. on the days im off I still feel as tired and it seems to be getting worse. any ideas of what this may be?ntinua Doctor: Hello,History of tiredness, Fatigue and weight gain in the last in year.If you're living a sedentary lifestyle you're that the risk of gaining weight.The body tunes itself according to the activity of the person.If your active your metabolism will increase and if you're otherwise the metabolism shutdown.But there are some medical cases also which can be attributed to it.Hypothyroidism is one condition which causes weight gain, fatigue.Diabetes may also cause a similar picture."
},
{
"id": 107545,
"tgt": "Suggest treatment for severe middle lower back pain",
"src": "Patient: Hello doctor I am 20 years old and I am In pain in my mid lower left side of my back. I was playing basketball 2 or 3 years ago at a family reunion and I extended my left arm for a dunk and it felt like I over stretched my back muscle it hurt so bad I thought it was locked in gas so I had to sit down at my grandmother house, the pain lightened up over the years and made it bearable but never went away i have been to the hospital for xrays and they said nothing was wrong(2 times or more). Im really concerned because I cant enjoy my activities knowing that it hasn't went away Doctor: Hello, I have studied your case. Usually nerve compression cant be seen on x ray you may need MRI for this.If there is any nerve compression then that may need specific treatment.I will advise you to do MRI spineFor these symptoms analgesic and neurotropic[pregabalin] medication can be started consulting your doctor..Till time, avoid lifting weights, Sit with support to back. You can consult physiotherapist for help.Physiotherapy like ultrasound and interferential therapy will give quick relief.Hope this answers your query. If you have additional questions or follow up queries then please do not hesitate in writing to us. I will be happy to answer your queries. Wishing you good health.Take care."
},
{
"id": 116638,
"tgt": "How is Acquired Hemolytic Anemia in an elderly person treated?",
"src": "Patient: My husband has Aquire Hemalytic Anemia since 2012.He had is spleen removed Dcember 2013 ,prior to that he was given Rituxin. None of those 2 treatments di not help him.He is 73 years old & he has being a diabetic & high blood pressure for over 16 years.I am very concern about his health .He was on predisone but was taken off about a month ago.I am very worried & frustrated. Doctor: Hi, dear. I have gone through your question. I can understand your concern. Acquired hemolytic anemia needs some investigation to search the cause. You may have cold antibodies or warm antibodies. Treatment is immunosuppressive drugs like steroids. You should continue steroids. Consult your doctor and take treatment accordingly. Hope I have answered your question, if you have doubt then I will be happy to answer. Thanks for using health care magic. Wish you a very good health."
},
{
"id": 174907,
"tgt": "Why child is having mottled and purplish red arms and leg with fever?",
"src": "Patient: My 2 year old was seen in the ER last night for high fever lasting 3+ days. Fever free today but temp going back up. When he was getting in the tub I noticed his skin , especially arms and legs are mottled looking and purplish red. Should I call the dr. Again? Doctor: Hi....this sort biphasic fever you are describing can occur in viral hemorrhagic fevers like dengue. The discoloration on palms also signifies the same. Sometimes it can lead to drop in blood pressure or bleeding manifestations. I suggest you get back toythe doctor immediately.Regards - Dr. Sumanthhe"
},
{
"id": 177146,
"tgt": "Suggest treatment for a bump to the left of my head",
"src": "Patient: Good evening, My grandson has a bump to the left side of his head it hurts him when you touch it. there is redness around it which seems to be spreading alittle. I thought it might have been a bug bite. He was seen today with his Doctor, but she didn t know what it was Doctor: Hi...it could be a dermoid cyst or a sebaceous cyst or galnd which is getting secondarily infected and that is the reason for the spreading redness and pain. i suggest you see a surgeon and get it examined, it might be forming pus too. She might require antibiotic therapy.Regards - Dr. Sumanth"
},
{
"id": 169723,
"tgt": "Suggest treatment for blocked nose and dry throat in a child",
"src": "Patient: 2 1/2 year old son, 12 kg, 89cm. Allergic to dairy. Hello dr, hope you are well. I am worried about my son. He has had a blocked nose at night ever since I can remember. He snores loudly and breathes through his mouth. He wakes often crying, I suspect because his throat is dry, and wants something to drink. Lately he has also taken to covering his mouth with his hand when getting sleepy and also while sleeping at times. Could this be another allergy, or something else. He doesn t get colds or flu that often, but the nose stays blocked at night (not during the day). He was born 6 weeks prematurely.I would really appreciate your answer.Kind regards. Doctor: Hi Dear,Welcome to HCM.Understanding your concern. As per your query you have blocked nose and dry throat in a child . The symptoms you mention in query can occur due to may reasons like nasal polyp , post nasal drip , strep throat or sleep apnea . I would suggest you to consult pediatrician . Doctor may order throat swab test , viral culture test along with nasal endoscopy or CT scan to confirm the diagnosis . Doctor may prescribe antibacterial or antiviral depending upon the diagnosis along with decongestants and expectorant . Doctor may also recommend surgical removal of nasal polyp . For now give your child steam at least twice a day and maintain propr oral hygiene . Hope your concern has been resolved.Get Well Soon.Best Wishes,Dr. Harry Maheshwari"
},
{
"id": 128468,
"tgt": "What causes back pain while breathing?",
"src": "Patient: I pulled my back 3 days ago and it s in spasm for all these days.it hurts to breathe. I have been taking Tylenol every 4-5 hours with no relief. I m allergic to nsaids and aspirin. I ve tried ice packs, bengay and resting. Nothing stops the spasms. I m uninsured and wondering should I go to er? I d say on a scale if 1-10 the pain is an 8. I m 27 years old. Doctor: Dear patient for pulled muscle you need rest , hot water fomentation. You need to start tab myoril 4 mg twice a day and tab ultracet once at night. this will help you in relaxing your muscle."
},
{
"id": 9205,
"tgt": "What causes dryness and tightness in the lips?",
"src": "Patient: I woke up with really dry lips this morning and my top lip feel pretty tight.. When I look in the mirror I don't see anything but like a purplish mark.. And when I feel it with my tongue it feels like a small rash but I don't see anything. What could this be Doctor: HIWell come to HCMI really appreciate your concern, such symptoms could be due to some allergic condition or fungal infection, in my opinion this can be well manages with \"Miconazole cream\" drink lot of water, have more vegetable diet, hope this information helps, take care and have a nice day."
},
{
"id": 56983,
"tgt": "How to treat fatty liver and diet disorder mediated elevated liver enzymes and bilirubin?",
"src": "Patient: For over than 10 years now my liver enzymes and bilirubin levels been ellavated and after several doctors visits and test results and ultra sound, I have been told it was due to fatty liver and diet disorder.... after this length of time is there a reason I should be concerned ? Doctor: Hello, If you have fatty liver there are chances you have altered liver function tests, but persistence since 10 years with elevated bilirubin levels, may be you should be evaluated further to know if it's just fatty liver , tour doctor would be telling you what are the ideal tests to rule out anything else ( like alcoholism, thyroid levels, cholesterol levels, sugar levels, viral markers such as Hepatitis B or Hepatitis C, in some cases serum copper levels, serum ceruloplasmin levels (only if suspected), ophthalmologist examination, serum Autoimmune markers and finally liver biopsy if everything else is negative), according to your test reports treatment can be initiated by your consulting doctor, incase you have fatty liver with no further changes in the liver than changing your diet with avoiding fatty food and meat and strict life style and exercising will help. Hope I could help you.Thank you."
},
{
"id": 77350,
"tgt": "What causes phlegm in the chest?",
"src": "Patient: My husband wakes each mornin with a huge amount of phlegm on his chest thismakes him want to vomit - what can be the cause? I write from South Africa and don't have a credit card to if this means I have to pay for this information I'm afraid it will have to do without it. regards Dana Doctor: if he is an older smoker,the cause of phlegm may be related to smoke and in this case the phlegm is especially during the morning.Nicotine paralize the cylia of bronchial mucosis and during the night they start to function again and the person have a lot of phlegm when he wake up.Other causes may be a chronic rhinosinusitis or chronic gastroesophageal reflux"
},
{
"id": 26240,
"tgt": "What causes changes in BP, flutters in heart and headache?",
"src": "Patient: My blood pressure now runs in the 90/60 ranges when previously ran at 120/80s. This afternoon it was 92/58 and I'm currently experiences heart flutters (feeling in neck and chest) and I have a headache. Should I be concerned? Why would my blood pressure so much lower for the last year or so after 20 plus years at 120/80? I am currently the same weight as 20 years previously. Doctor: Hi,Make sure you take adequate quantity of water.If you take enough water, but still you have low blood pressure I would advise you to see your doctor and perform an ECG. It will help to reveal the cause of your flutters and may be helpful in detecting the cause of your low blood pressure.Wishing you good healthIn case of further questions don't hesitate to askRegards,"
},
{
"id": 21948,
"tgt": "What could heart palpitation with dizziness, neck pain and weakness indicate?",
"src": "Patient: I have had heart palpitations for about an hour now. I have been mostly laying down from the dizziness and weakness. I have had pain in the left side of my neck that comes and goes. This started after I bent down to pick something up off the floor. I am female, 34 yrs. 5 ft. 0\" 112 # Doctor: Hello!Thank you for asking on HCM!Regarding your concern, I would explain that the palpitations could be related to the pain. While, the pain could be related to a muscle spasm, due to a wrong position during bending down. I would recommend taking ibuprofen for the pain and having some rest. If the problem persists, I recommend going to the ER for a physical exam and some tests: - a cervical spine X ray study- a Doppler ultrasound of the cervical arteries (to exclude possible dissection)- a resting ECG. Hope to have been helpful!Wishing all the best, Dr. Iliri"
},
{
"id": 71282,
"tgt": "What causes persistent cough, vomiting and dizziness?",
"src": "Patient: Hello, I m a 19 year old female. To explain my problem I have to go a few years back. And please ignore the grammar mistakes, my English is not so good. So when I was 11 years old, I had a lung infection and the treatment went for a long time. After the treatment I was totally fine, but cough and cold would appear every winter since then. It would only stay during the winter and then it would go away. But from the last two years I m experiencing it during all the months. The cough didn t go even during the summer season. The cough thing is always staying with me. I ignored the problem, as it was only cough and cold. Now since the last 2-3 weeks, I m feeling nauseous and dizzy. I even vomited several times. When I feel too dizzy that I feel like I would fell, the vomiting sensation starts. I feel that it has something to do with my cough, which has continously stayed for a long time now. And I also stay a bit depressed. I think it is not bronchitis, coz I ve taken the medicine for that and it s not working. I m not sure what is it, please help! Doctor: Hello and Welcome to \u2018Ask A Doctor\u2019 service.I have reviewed your query and here is my advice.According to the history, this might be an allergic cough due to several allergens. Please discuss with your doctor. I hope I have answered your query. Let me know if you have any further questions. Take care.Regards,Dr. Jnikolla"
},
{
"id": 120301,
"tgt": "Suggest treatment for numbness in hand and feet with high heart rate",
"src": "Patient: as I was getting ready to sleep I began feeling symptoms of low glucose ... I immediately checked my blood. It read 68 ... I noticed a faint numbness in both feet and left arm and hand ... my heart rate is also a bit elevated ... I have eaten a little bit and feel better but my level has not improved ... now I m nervous to sleep Doctor: Hello, Low blood glucose can be due to decrease intake of food with excess hard work or exercise.You should keep balance between hard work and intake of food,do not fast frequently.But if this happens without reason and frequently then consult to your doctor and get yourself evaluated. Hope I have answered your query. Let me know if I can assist you further. Take care Regards, Dr. Mukesh Tiwari"
},
{
"id": 186843,
"tgt": "What causes black gum line?",
"src": "Patient: I am very concerned about my teeth turning black at the gum line. It also looks like they are eroding in the same area. I have recently been diagnosed with sugar diabetes. I also suffer from major depressive disorder and have Fibromyalgia. Why is this happening to my teeth? Doctor: Hello, Welcome Thanks for consulting HCM, I have gone through your query, as you have painful gums and teeth are black at cervical region dont worry it can be due to cervical abrasion , or deposition of stains and calculus at cervical region . For this you should consult dentist and go for Restoration of tooth or if it is stains and calculus then go for Scaling andmroot planning. Hope this will help you."
},
{
"id": 90469,
"tgt": "What causes pain in lower abdomen?",
"src": "Patient: IVE BEEN HAVING WHITE DISCHARGE NO SMELL THOUGH, ALMOST A WEEK NOW AND BREASTS ARE VERY TENDER ALSO A PAIN IN MY LOWER RIGHT SIDE OF MY STOMACH. EVERY TIME I SIT UP FROM LAYING DOWN ITS A DULL PAIN AND I AM NOT SURE WHAT IT COULD BE. I FEEL LIKE IVE BEEN DOING SIT UPS FOR WEEKS BUT ONLY FEEL IT IN MY LOWER STOMACH. AND SOME LIGHT BACK PAIN. Doctor: Hi ! Good afternoon. I am Dr Shareef answering your query.From the history, it seems that you have got a PID (pelvic inflammatory disease) giving rise to the discharge, and pain in the area mentioned. This is more so if you are sexually active. If I were your doctor, I would have referred you to a gynaecologist in your area, who after clinical assessment, might perform a culture sensitivity test of the discharge and a urine routine, microscopic, and culture sensitivity test, and an ultrasound of abdomen if need be to institute appropriate treatment.I hope this information would help you in discussing with your family physician/treating doctor in further management of your problem. Please do not hesitate to ask in case of any further doubts.Thanks for choosing health care magic to clear doubts on your health problems. Wishing you an early recovery. Dr Shareef."
},
{
"id": 189708,
"tgt": "Dull pain on right side of face and teeth, pain behind right eye, popping in ears. Due to dental problems?",
"src": "Patient: morning I have had for a week now a dull ache down rhe right side of my face. I wesr contact.lenses and have left them out cos.of the pain. IT aches behind the right eye and my ear is hurting and popping alot. Alot my bottom.right hand side of my.face hurts of touch? my teeth right side also hurting and been dentist and he said teeth were all on but i had a small filling. Doctor: HI, Thanks for asking the query, Pain in the right side of the face can be because of different ailments . It can be because of temporomandibular joint disorders which occurs due to jaw injuries , bruxism. Get complete clinical evaluation of the TMJ done by a dentist. Take a panoromic x-ray. Eat soft diet . Neurological causes such as Trigemminal neuralgia , Glossopharyngeal neuralgia , Bell's Palsy, can lead to facial pain , visit an oral surgeon to rule out the causes . Pain can also be because of infection in the sinuses , Sinusitis , infection in the ear otitis media . Visit to an ENT specialist and get complete check done . Infection of the tooth can also be the reason . get an x-ray of the tooth done . for pain relief you can take tab combiflam 500mg twice daily for 3 days. Hope this helps out . Regards.."
},
{
"id": 222796,
"tgt": "Is Fertomid 50g the right medicine to get pregnant?",
"src": "Patient: hello Dr. my name is Cynthia.I have been u family planning for more 10 yrs after the birth of my son.bt I stop now I want other baby I went to the doctor she give me fertomid 50g .I want to know that its gonna take for how long to be pregnant. after week I went for pregnancy test bt its negative Doctor: Hi,Just taking Fertomid will not help you get pregnant.You need to try for 6 months without any medications and you should be able to get pregnant.May I know how old you are and also what method of family planning were you using?Fertomid is only given to women who have ovulation problems. Please do not take them with out a proper evaluation of cause of infertility.I would not even think that you have a problem, unless you have tried to get pregnant for at least 6 months to 1 year.Hope this helps.Regards."
},
{
"id": 7056,
"tgt": "Is ovulation induction drug safe for a PCOD patient to get pregnant?",
"src": "Patient: which is good for me. i want preg as soon as possible 29yrs,pcod,irregular periods, taken three month course for pcod after that two months past doctor take USG but not found any major follicle in this. now she recommened another three month course with duloute -L and ebexid. what would you suggest? another doctor recommend ovulation induction drug i.e some clomipine type.?IS this safe for me i am confused .which doctor i had to follow? i want preg soon. if i choose induction drug method Is it solve my problem .otherwise one more month would be wasted.and then i have to follow three month course. Doctor: Hello jyoti; welcome to HealthcareMagic Duolute-L is a hormonal pill and is given to you to make your periods regular and treat PCOD.PCOD is one of the cause of infertility because most of the times the ovulation is absent or does not take place and in such cases the doctor advises clomiphene to induce ovulation.So if your PCOD is treated and your periods have become regular then you can take clomiphene for induction of ovulation.So finish your course of Duolute and discuss with your doctor regarding the use of clomiphene and take it under the guidance of your doctor. Thanks"
},
{
"id": 90169,
"tgt": "What causes abdominal cramping with blood?",
"src": "Patient: Last niight I started to have severe lower abdominal cramping. I went to the bathroom and pooped out a great deal. Then I started to poop out blood mixed with tissue preceded with horrible cramping. For the past 85% of the time, it has been blood and tissue only. I have been doing this off on all all night and all day. What could this be? Still hasn't stopped. Doctor: Hi.This occurs in the severe forms of colitis.First of all you have to get a proper antibiotic, metronidazole and anti-spasmodic medicines. for a course of minimum of 5 days. If there is no relief we have to think about other possibilities about ulcer/ Cancer of the large bowel. The best way is to get a colonoscoy and biopsy done. CT scan helps for the confirmation of the diagnosis , staging if there is a cancer. Helps to plan the case and obviously rule out other problems of the intestines and abdomen."
},
{
"id": 45948,
"tgt": "What causes palpitations, sickness, diarrhea with exhaustion and back pain?",
"src": "Patient: I have a duplex kidney on the right..I've been ill for 3 months, having palpations, sickness, diarhhea, exhaustion, lower back pain for a few years but worse for last 9mths.. hardly eating, lost a stone. my doc put it down to anxiety.but I ended up feeling very ill one night went to A&E, showing every urine test positive, blood, white cells etc..now waiting to see specialist...do you think it's more likely to be my kidneys ot anxiety? Doctor: Hello and Welcome to \u2018Ask A Doctor\u2019 service. I have reviewed your query and here is my advice. It's definitely not just anxiety. We need to rule out urinary infection as you have duplex kidney and more prone to get infected. Have you got your thyroid and blood sugar checked? If there are wbcs in urine get your urine culture and next to take antibiotics. Hope I have answered your query. Let me know if I can assist you further."
},
{
"id": 160778,
"tgt": "Suggest treatment for vomiting in an infant",
"src": "Patient: 22 month old baby, has thrown up several times over a ten minute period both tonight and last night around the same time however is showing no other symptoms, no temperature, is allergic to dairy however hasnt had any dairy, what could be wrong with him Doctor: Hi,I guess there is no history of fall from a height in the last 24 hours (if yes, we need to take him to the ER). Repeated vomiting alone is caused usually by acute gastritis or gastro-enteritis (in later condition, there will be loose stools also). Has he taken any drugs (especially analgesic or antipyretic) recently or any change from usual diet? These are the usual precipitating factors for gastritis. If no history of fall from height, fever or loose stools, for such cases I used to give acid suppressants like pantoprazol and antiemetics like ondnsetron for 3-5 days. You can try giving ondansetron suspension 5ml twice daily for 2 days and report to your doctor if not relieved or any above mentioned features are present.Meanwhile give frequent sips of ORS solution or salted rice water to avoid dehydration and give small frequent feeds. Avoid spicy and oily food for 1 week.Hope I have answered your question. Let me know if I can assist you further. Regards, Dr. Muhammed Aslam TK, Pediatrician"
},
{
"id": 197803,
"tgt": "Is increased pus cells in the semen report abnormal?",
"src": "Patient: Dear sir, I have done semen analysis and come up with acitivity grade 3- and motility is 65%, Pus cells are increased and hpf 8-10. this seems to abnormal, is it treatable as acitivity grade 3- means circular movement of semens.Kind regardsAli abne johare Doctor: Hello Ali and .As an Urologist, i do share your concern.You should've done the semen analysis, after 2-3 days of abstinence.Pus cells of 8-10/hpf, is suggestive of infection. You'll need to take a course of antibiotics, for at least 7 days. Ciprofloxacin /Septran/Nitrofurantoin are routinely advised.If you've fever,semen culture, should also be done.In case, you're referring to motility(activity),then grade I- 65%, is considered as normal.For that clarification,refer to the semen analysis again.To confirm semen is free of infection,repeat analysis,after 3 weeks.For an expert opinion, on the report, send it to me,as a direct question.Dr.Matthew J. Mangat."
},
{
"id": 79668,
"tgt": "What causes chest pain and breathing difficulty?",
"src": "Patient: Hi this is pretty complex In march I was eventually diagnosed with a dvt (after seeing 5 doctors) this then progressed into a PE in which two clots were found on my lungs. I spent 5 days in hospital on high dose of deltaparin and discharged on the lower dose of 15,000 units because I self harm by blood letting.I was again later diagnosed having suffered another PE where three new clots were found and a shodow on my lower left lung.They placed an ivc filter in. I went to have this taken out and they had found that it had dislodged and was filled with clots as well as a large clot sat on the filter estimated to be around 5 cm. Again I was Put on 15,000 units of deltaparin.Last night I was admitted for a transfusion and every professional I saw I explained my concern about having had another PE (same symptoms as before) no one took me seriously because I am on deltaparin. They gave me an X-ray which showed nothing but it never has. Clots have only shown up on a ct scan.They basically said because I'm on deltaparin there is nothing more that they would do.My chest pain is worsening (I have been discharged today) I'm finding breathing more difficult. What do I do? Do I have to be dead to get help? Doctor: Thanks for your question on Health Care Magic. I can understand your situation and problem. You are having recurrent Pulmonary embolism despite of optimal anticoagulant treatment. So we need to rule out hypercoaguable states like homocystienemia, protein C and protein S deficiency etc. So better to consult hematologist and distal these. For your present symptoms like chest pain and breathlessness, we need to rule out Pulmonary infarction and recurrence of embolism. So you need to get done CT Pulmonary Angiography to rule out these possibilities. Possibility of hypercoaguable state is more in your case. Hope I have solved your query. Wish you good health. Thanks."
},
{
"id": 72734,
"tgt": "How to cure constant pain in right chest under the shoulder blade?",
"src": "Patient: I am a 30 year old female and I have had constant pain in my chest on the right side under my shoulder blade for 1 month. The pain is also in might right shoulder blade on my back. If I press on my chest it also hurts the spot in my back. What can this be? Doctor: Thanks for your question on Healthcare Magic.I can understand your concern. In my opinion, you are mostly having musculoskeletal pain in right shoulder. So better to consult orthopedic doctor and get done clinical examination, x ray and MRI of right shoulder joint.If it is ligamental or muscular injury then you will need strapping and immobilization of shoulder joint. If MRI is normal then no need to worry much just take painkiller and anti inflammatory drugs like ibuprofen or acetaminophen. Hope I have solved your query. I will be happy to help you further. Wish you good health. Thanks."
},
{
"id": 110534,
"tgt": "Can back pain be caused due to a horseshoe kidney?",
"src": "Patient: I am 30 yrs. old and have a horseshoe kidney, I also recently found out that I have a heart that beats too fast and high blood pressure. I have been somewhat healthy most of my life. 2 c-secs, 2 breast surgeries to drain and remove cycts and artificial ear drum as well as my tonsils being taken out, but other than all that pretty healthy. I was wondering I have had pretty bad back pain for a while now and it seems that the dr. can't find anything wrong from xray. I also had orthopedic surgeon check it out and they found nothing. I was wondering could this be from a complication of my horseshoe kidney. Doctor: Hi, I think renal complications may cause back pain. Renal complications are more particularly when you have a horse shoe kidney. Chances of urinary trat infections are much more with horse shoe kidney. This may cause a lower back pain. Any form of glomerular disease should also be ruled out, particularly because you have back pain, horseshoe kidney and hypertension. As you have a normal Xray and your orthopaedic consultant has excluded any bony abnormalities, musculoskeletal reasons seem less likely. However, mechanical back pain, particularly due to faulty posture during sitting or standing or prolonged sitting or lax abdominal muscles may not display any clinical sign or imaging abnormality. Thirdly, a colonic pain may present similarly. Irregular bowel habit may or may not be associated with this.A fourth common consideration is pelvic inflammatory disease or endometriosis or any uterine abnormality. For the time being, my recommendations are-1. Urine for routine examination2. USG lower abdomen including KUB3. Plenty of oral fluid, preferably 3 lit per day.4. Analgesia with tab paracetamol 500 mg twice daily.5. Abdominal muscle strengthening exercise6. Continue with antihypertensive medication and cardiac medications.Few more informations are needed regarding fever, bladder and bowel habit, pain abdomen for a more detailed answer.Regards,Dr. Kaushik"
},
{
"id": 168058,
"tgt": "What causes high fever and hematuria in children?",
"src": "Patient: My child 8 years old, female, having high grade continuous fever for the last days is having haematuia, which is painless. Analysis shows many RBCs and 5-8 Pus cells. Taking antibiotic for the last 2 days. No other symptom, other routine investigations normal. What it can be ? Doctor: Hello. I just read through your question.The most likely diagnosis is a urinary tract infection. The symptoms and the urinalysis support that. The antibiotics will help. A repeat urinalysis should be done in a few days."
},
{
"id": 147439,
"tgt": "What is the cause of pain in arm and dizziness?",
"src": "Patient: I noticed that more often now I have an off and on dull ache-pain in my left arm that moves between wrist and biscep area. Also, I noticed that I see pinpoint of light with slight dizzy feeling...is this something thats tied together and should I be worried? Doctor: HIThank for asking to HCMI really appreciate your concern looking to the history given here suggestive of some muscular spasm and may be peripheral nerve compression more than enough pain can cause dizzy feel, this condition can be treated with the non steroid anti-inflammatory drugs, and the best drug is \"Tab Diclofenac 50 mg sustain release once in day\" hope this information helps you, have nice day."
},
{
"id": 89492,
"tgt": "Suggest medication for pain on right abdominal area post inguinal hernia repair",
"src": "Patient: I had a double inguinal hernia repair with mesh in 2012. I have had ongoing pain on the right side ever sense. In Jan 2014 I had a CTScan done of the Abdomen and Pelvic area. The person reading the results, commented and said I have Bilateral fat containing hernias on the right side. What should be done if anything. The pain especially gets bad if I m on my feet more than a couple hours. I am 63 years of age. I m sorry I didn t see where there was a charge before asking the question. I have no monies or card at this time. Doctor: HI.The CT scan report is obvious that you have a recurrence of a hernia at the said site and only answer is to get re-exploration repair of the incisional Hernia. At the same time the operating surgeon to look for any entrapment of any nerve in the same area of pain."
},
{
"id": 30061,
"tgt": "Suggest treatment for herpes simplex virus",
"src": "Patient: Greetings to the general public, Am Simran Mendy i want to inform the public how i was cured of HERPES Simplex Virus by a Doctor called Harrison.i visited different hospital but they gave me list of drugs like Famvir, Zovirax, and Valtrex which is very expensive to treat the symptoms and never cured me. I was browsing through the Internet searching for remedy on HERPES and i saw comment of people talking about how Doctor Harrison cured them. I Was scared because i never believed in the Internet but i was convince to give him a try because i was having no hope of been cured of HERPES so i decided to contact him on his email that was listed on the comment ( YYYY@YYYY ) i searched his email on net and i saw a lot of people testifying about his goodness. when i contacted him he gave me hope and send a Herbal medicine to me that i took and it seriously worked for me, am a free person now without problem, my HERPES result came out negative. I pray for you Dr Harrison God will give you everlasting life, you shall not die before your time for being a sincere and great men. Am so happy, you can also contact him if you have any problem Email: YYYY@YYYY add him on whatapp +0000. you can contact me for more information +1989-541-7345 or email me on YYYY@YYYY Doctor: I am not sure what you were trying to ask in this forum.however will like to inform you that according to evidence based medicine and clinical trials, valciclovir/ famciclovir/acyclovir have proven benefits in case of HSV.It might be possible in your case that some associated condition or alternate diagnosis was required.I wish you best of health ahead and be free to ask for any assistance required."
},
{
"id": 212129,
"tgt": "I wake up every day trembling and sweating. Taking Sertraline and mirtazapine. Suggest",
"src": "Patient: I wake up everyday very hot, trembling I feel as though I am sweating, but there doesn't seem to be any sweat.Then I go lightheaded, my body feel weird also, but can not describe how it feels. I feel ill but nothing physically wrong. my head and limbs feel heavy, foggy, sluggish. If I forget myself and get up quickly and start doing morning routine. I then feel these symptoms and have to lie down to recover. At work, I race around with children, but if I push it then, I feel faint, blacking out, pretty lights floating, can hear my blood pounding. yesterday, I raced to the market, although I was being mindful of not to create stress, anxiety in myself, as it can bring on symptoms. I fainted when I finally arrived, about 4-5 minutes after I arrived. Am on Sertraline 50mg. been on anti-depression since 2005. used to be on 200 mg of Sertraline, with 15 mg mirtazapine. So this level of dosage I would had thought I wouldn't be affected by side-effects. so it could be something completely different. I do smoke (rollups), but have really cut down these past weeks. smoked two yesterday. and some days in past week nothing.This has been going on awhile (years), getting worst with time, am 39 years old.happening more often, to start be bothersome. I have to move more slowly in doing things, not get too excited, I can't push my body as I used to. I have to rest physically after activities. Am 5ft 10 1/2 inches. weight is just under 15 stone. Doctor: Dear I understand your concern. You have been diagnosed to hve depression. And you are under medication. Time and again it has been proven that medicines alone cannot cure depression. Change your life style using psychotherapy. Change your body metabolism and way of thinking through Yoga. Decrease your pace of life through meditation. Consult a good psychologist and yoga master soon. Wish you speedy recovery"
},
{
"id": 171005,
"tgt": "What causes red spots with white head all over body?",
"src": "Patient: I was informed last week that mt daughter may get chicken pox as my friends little boy has got them A few days ago my daughter has about 10-15 red spots that looked like they had whiteheads on them so i pur camamoil lotion on her an the next day they had gone could it be a mild form of chicken pox? Doctor: Hi.... I do not think that this is chickenpox. This could be any other simple viral exanthematous illness. If she had fever before this Rash we can be almost certain that it is a viral illness. But if it is subsiding by application of topical lotion then it could also be a simple allergy or Eczema. From next time I suggest that you upload the images on this website so that we will be in a better position to guide you. I am telling this because skin conditions are best diagnosed by directly seeing them.Regards - Dr. Sumanth"
},
{
"id": 48299,
"tgt": "What does the ultrasound report indicate about kidney conditions?",
"src": "Patient: Hello sir,My report of ultrasound of abdomen is as follows. Right Kidney shows 5-6mm size calculus in lower calyx without hydronephrosis. Left Kidney shows multiple (3-4) calculi, largest is 7mm in lower calyx with mild hydronephrois and hydrourerter. Left vesicouretric junction shows a calcalus of size 9mm.IMPRESSION: left vuj calculus (9mm)& bilateral renal calculi. Doctor: Hi,You have been diagnosed to have a single small stone in right kidney and a few small stones in left kidney. One of the stones from left kidney has dropped into the ureter (the tube connecting kidney to urinary bladder). Vesicoureteric junction denotes the site where ureter joins with bladder. This has caused mild swelling in the left kidney due to obstruction. Otherwise your kidneys are apparently normal.The stone in ureter usually causes pain in left flank with or without vomiting and burning urination. Usual treatment is an alpha blocker (eg. Tab. Tamsulosin) and pain relieving medications. If the pain does not subside or kidney function deteriorates or infection sets in, then it needs to be removed immediately. Otherwise, the stone can be monitored and if does not pass in 4 weeks then it can be removed using endoscopy (URS, ureterorenoscopy).After treatment for the ureteric stone, you need to be evaluated to find out the cause of stone formation. Kidney stones can be treated later, either medically or with flexible ureteroscopy and laser.I hope I have answered your query.If you need any clarification, please get back to me.Wish you a speedy recovery,Best regards,Dr. Raguram G MCH Uro, F EndouroConsultant Endourologist"
},
{
"id": 32501,
"tgt": "What causes stomach pain and runny nose?",
"src": "Patient: im a 40 year femal 5'1 and 110 lbs i have divertitus diagnose 3 years ago,my symptoms are severe left stomach pain with some fever,runny black oily stool with blood in it time to time.i eat maybe 4 times a week due to pain.i have no insurance and been to 2 health dept they dont deal with this,i cant seem to get any help or releif,if you might know a program that could help me or any advice on releaving pain and to help me eat, thank you lynnette Doctor: Hi,Welcome to health care magic,Severe left side stomach pain with fever and black stool could have many causes which are acute gastritis or diverticulitis or food allergy or food poisoning.In your case it is likely because of gastritis or diverticulitis.Your should undergo routine blood investigations and serum electrolytes ,liver function tests,urinalysis and stool for blood and abdominal ultrasound to detect the underlying cause.Treatment is mainly symptomatic if there is no infection.Although antibiotics like norfloxacin with tinnidazole should be given for at least 3 days small course with anti-emetics like tab.ondasetron mg twice daily.Antacids like h2 blockers (ranitidine) or proton pump inhibitors (pantoprazole or rabeprazole) will help to reduce excess acid production in stomach.One can also take digene gel or liquid or ulcero-protective agents like sucralfate or bismuth subsallicylate to reduce the burning sensation.Apart from medications he should eat plain foods like boiled rice and green leafy vegetables without adding spices or oil.avoid oily,spicy and fast food.Consult physician or gastroenterologist for detailed medical history and clinical examination to diagnose the cause.Thanks and Regards,"
},
{
"id": 154970,
"tgt": "What are the affects of being on heroin and going through chemo?",
"src": "Patient: My brother is going through chemo for the last 3 weeks because he just got diagnosed with hotchkins lymphoma stage 4 cancer. He was a heroin addict for 5 years and got clean just 2 months ago. He started using again over a week ago. What are the affects of being on heroin and going through chemo? Doctor: hello..heroin is a kind of morphine which is a pain killer with lot of other side effects on uncontrolled use..at low and controlled doses it actually helps a patient but your brother will be on chemotherapy drugs and other supportive medication..it is good if he can stop it because there can be interactions between the drugs..along with direct side effects of heroine..take the help of a psychiatrist he can help for de addiction..if you have anymore doubts i would be happy to answer.."
},
{
"id": 12351,
"tgt": "Suggest treatment for psoriasis on face, scalp and arms",
"src": "Patient: I have psoriasis really bad all up my arms and all over my legs even on my face/scalp/ears its making me really depressed as summers coming up and ill have to be covered in clothes. I want to use sunbeds is there a certain amount of time they will take to work? and can i get any drugs/tablets to help it go away on subscription? Doctor: Hi,You said you suffer from psoriasis. It is a n autoimmune skin disease of unknown cause. There may be genetic tendency. Stress,dry wheather,poor immunity..etc might exacerbate the disease. You consult dermatologist for perfect treatment.You may take methotraxate 15 mg per week 5 mg at 12 hour interval till good result. You may also take acitretin cap 10 mg daily. Itching may be reduced by antihistaminics. Blood tests may be done to moniter the treatment.Application of steroid combined salicylic acid oint or vitamin D analogue cream may be applied on the lesions. Scalp lesions may be improved by shampoo containing tar.Have patience for the result. It is a controllable disease. Discontinue treatment after having disease free period. Repeat the treatment if there is relapse.I hope you got my answer.Thanks.Dr. Ilyas Patel MD"
},
{
"id": 101013,
"tgt": "What does asthma,high sugar level and e-coli indicate?",
"src": "Patient: i AM HAVING WHEEZING(MAY BE ASTHMA) AND USING AROCART INHALER. 70 YEARS OLD. jUST SIX MONTHS BACK, TEST SHOWED MY SUGAR LEVEL AFTER MEALS BETWEEN 139 TO 179. hENCE I TAKE ONE 500 MG GLYCOMET WITH ZINCOVIT PER DAY. NOW i HAVE URINARY INFECTION AND CULTURE SHOWS GROWTH OF ORGANISM E.COLI yESTERDAY DOCTOR ADVISED AMIKASIN INJECTION WITH NITROFURANTOIN AND PAN 40 TAB. fURTHER ADVISED TO TAKE SCAN OF KIDNEY,BLADDER ETC. hAVING SOME CHEST PAIN ALSO (ecg NORMAL). hAVING A NUMBER OF PROBLEMS, ANY RISK TO MY LIFE? T.R.V.RAMANI. Doctor: Mr Ramani,plz do not panic as your condition is not fatal. You need to get a fasting sugar and HBA1C to ascertain the diabetic status. I think you are not diabetic. Meanwhile, Start taking your inhaler with a spacer. Continue with pan40. Your condition might not be as bad as you think!!"
},
{
"id": 135306,
"tgt": "Suggest treatment for severe numbness and pain in the foot",
"src": "Patient: I just had stents put in my left leg and he decide to go back into right leg now I have horrible pain in right foot and a numbness in foot and leg, feels like Im walking on rocks bare footed, he precib a blood thinner ,but I have a bleeding problems very low platlets my whole life,. what can I do ? Doctor: Hi Dear,Welcome to HCM.Understanding your concern. As per your query you have severe numbness and pain in the foot. Well there can be many reasons for symptoms you mention in query like fibromyalgia , multiple sclerosis , herniated disc or vitamin B12. I would suggest you to take ibuprofen or acetaminophen for pain . keep your legs in hot water tub with epsom salt for 15 minutes daily and take proper rest . If condition doesn't get better then consult orthopedic surgeon for proper examination . Doctor may order CT scan , MRI of leg and spin along with physical examination . Doctor may prescribe muscle relaxant along with anti inflammatory , vitamin supplement and physical therapy . Doctor may also recommend surgery in case of severe disc problem . Hope your concern has been resolved.Get Well Soon.Best Wishes,Dr. Harry Maheshwari"
},
{
"id": 103256,
"tgt": "Have allergic contact dermatitis reaction to makeup. Is zinc causing reaction?",
"src": "Patient: I have been having an allergic contact dermatitis reaction to my makeup (at the same time my skin turns red, I experience heart palpitations and a headache. I threw out all traditional makeup and began using mineral makeup from Whole Foods...now it appears to be causing the same, although not as bad, reaction. I also began taking a multi-vitamin called Vitamin Code, a raw whole food multi vitamin for women and now I get the same allergic reaction when I take the vitamins. I'm afraid, it may be zinc..it's the only thing that's in BOTH the vitamin & the makeup. Does this make sense? Doctor: Hello and welcome to HCM,It is advisable to discontinue the use of make up for a some time.Consult your dermatologist for clinical examination of the reaction site.If the lesions are due to allergic reaction, drugs to suppress the allergic reactions are required.What are the constituents of the multivitamin preparation that you are taking?Consult your physician for to know the constituents.Any drug whether supplements, or any other preparation should not be taken without consulting your physician.In order to confirm whether the reaction is due to zinc, allergy test needs to be done.Thanks and take careDr Shailja P Wahal"
},
{
"id": 78612,
"tgt": "Suggest treatment for chest congestion , chronic cough and swollen ankle",
"src": "Patient: I am experiencing SOA, more noticeable congestion from the R lung vs the L, some ankle swelling, a persistent cough. I recently had bronchitis and was on antibiotics. I am 54, a former smoker. I am more fatigued than usual. I have HTN and am on 3 medications. I have noticed an irregular heart beat where it feels like I am skipping beats. I do get up at least 3 times during the night to go to the bathroom. I was wondering about CHF. Doctor: Thanks for your question on Health Care Magic. I can understand your situation and problem. Yes, possibility of congestive heart failure (CHF) is more in your case. CHF is known complication of hypertension. Due to CHF pumping of heart is decreased and fluid over load occurs. And this excess fluid accumulate in lower limbs, lungs etc. So swollen feet, chest congestion, cough, irregular heart beats etc are favouring CHF more. So consult cardiologist and get done 1. Ecg 2. 2d echo 3. Coronary Angiography if required. You need diuretics and cardiac inotropic drugs to relieve fluid over load and improve heart pumping. Hope I have solved your query. I will be happy to help you further. Wish you good health. Thanks."
},
{
"id": 114675,
"tgt": "Suggest an alternative treatment for iron absorption apart from Intravenous Iron Infusions",
"src": "Patient: hemoglobin 66; ferritin 1 ... blood transfusion was given about 2 days ago, but was told this is only temporary to raise hemoglobin levels, but doesn t help the ferritin itself. For some reason, my body does not absorb iron at all. Severe allergic reactions to IV iron already tried. Basically, what have I missed? What else can I try? Doctor: hi there thanks for you query there are alternative methods to take ironyou can take ferrous sulphate tablets which is the most common form others include ferrous fumerate n gluconate"
},
{
"id": 224715,
"tgt": "Will irregular birth control intake affect periods?",
"src": "Patient: Hi, my period is late. I missed my first pill of the placebo week, so I took two the next day. My period hasn't necessarily come. It's just a dark discharge. I did have sex with my husband a few weeks previously. Does irregular birth control intake affect my period? Doctor: I regular birth control pil will effect the period. preg Nancy test can con firm the result. In future be regular in taking pills."
},
{
"id": 63627,
"tgt": "What is the painful lump on the inner part of my nostril?",
"src": "Patient: When i feel into the inner left side of my nose it's completely smooth, how the inside of a nose should feel. When i feel into the inner right side of my nose it's smooth until i reach the bottom left corner of that right nostril. I feel a hard bump, it's part of the bone, if not THE bone. It does not give me any complications when it comes to breathing, it is just painful. It randomly showed up about a year ago and recently started becoming painful. Doctor: Hi,Dear,Thanks for the query to HCM. I studied your problem in depth and I understood your concerns.Treatment-In my opinion your painful right nasal lump- is mostly due to the bruise and sore from the boil near the nasal septum at the base of the right nostril.Sore in nostril is due to the repeated handling of crusty nasal discharge.Emollient or simple coconut application locally,Tab NSAIDS / for 3x 4 days time would resolve this bruise.IF not Antibiotics from the ER Ent-Surgeon would resolve this lump.If it grows in size consult ER Ent-Surgeon.So consult your doctor and don't worry.So don't build up wrong concepts and create more psychic complications in you which would increase risks and costs to you, but just ask a query to HCM and be comfortable to resolve your health issues.Welcome for any more query in this regard to HCM.Write good resume and Click thanks if you feel satisfied with my advise.Have a Good Day.Dr.Savaskar M.N."
},
{
"id": 104733,
"tgt": "Suffering cough and cold, taken Azithromycin, Respira, Benocide forte, no help. Any cure ?",
"src": "Patient: for since 1 month i am suffering cough and cold allergy ,with small small reasons i am suffering from cold..also i am doing gargling with warm water and i have taken Azithromycin tablets 7,Respira syrupy and also Benocide forte 100mg tablet still i am suffering from problem.i dont have smoking habit.can u please suggest some way,i am feedup with this. Doctor: Hi, This is due to upper and lower respiratory tract infection if you are not feeling good with Azithromycin go for more potent Antibiotic such as Tab. Avelox 400 mg once daily for 05 days with an oral decongestant such as Loratidine twice a day for 07 days and an antitussive agent for cough such as Acefyl cough 02 TSF thrice a day for 07 days. Avoid yogurt and other cold thing have gurgles with salt water. I hope I am sucessful in solving your query If you have more you can ask me. Take care Regards, Dr. Azhar Sattar"
},
{
"id": 114836,
"tgt": "What could cause positive result for Benzos in drug test?",
"src": "Patient: I just took a drug test today and it came back positive for Benzos. I do take Celexa and Trazodone every day. Could one of these drugs have caused a false positive for Benzos? I have been tested several times before today while I was still taking these same medications and nothing has shown up before but today for some reason that did. I am very concerned over this because I know in my heart that I did not do any kind of drug and really need some help looking into this!!! Thank you Doctor: Hi I did review your concern.Celexa is a selective serotonin reuptake inhibitor and it can very rarely show a false positive benzodiazipine urine test. If you tell the testing authority that you are taking this medication then it is more likely that they will understand and help you out in this regard.I hope this helps .wish you all the best.Thank you for choosing health care magic."
},
{
"id": 29018,
"tgt": "How can molluscum contagiosum in a child be treated?",
"src": "Patient: My 5 year old granddaughter has Molluscum contagiosum. Her Dr. said there is nothing that can be done about it. It is very bad behind both knees, red and yellow looking. It does not seem to bother her but it definitely bothers me. It is awful looking. She does have Escema and attends a daycare. What can we do??? Doctor: Hello,The key issue here is that she receives the correct diagnosis. Generally, Molluscum contagiosum will just appear as small bumps on the skin. It should not change color from yellow to red or look awful as you describe. It sounds like she might have a different type of infection that needs treatment.However, if she does have molluscum contagiosum as your doctor says, there are no real proven treatments for it. Topical cryotherapy, imiquimod, salicylic acid, etcetera have worked for some patients. Ultimately, I do not believe she has a molluscum contagiosum infection if it appears as you describe. It sounds like she needs to see another doctor for a second opinion.Hope I have answered your query. Let me know if I can assist you further.Regards, Dr. Alexander Underwood"
},
{
"id": 152595,
"tgt": "What questions one should ask an oncologist about cancer diagnosis?",
"src": "Patient: My husband has been diagnosed with cancer in 3 places on his bone scan left collar bone right hip socket & 2 places on his lower spine possibly #17 & 16 vertebra, he will have an ultrasound & then see an onocologist. What are some of the questions he should ask about treatment? Doctor: HIYour husband should ask about the treatment options such as radiation therapy, chemotherapy, surgery and hormonal treatment.RegardsDR DE"
},
{
"id": 112023,
"tgt": "How to cure severe lower back pain?",
"src": "Patient: i have lower back pain from past 4-5 year but earlier it happen 2 or 3 weeks in a year but from last yr it stay regularly and whenever i sitdown for more than 2 minute and suddenly standup it pain very much and i can t stand very quickly my age is 26 yr Doctor: Hello, I have studied your case.Most probable reason for your symptoms could be postural problem as you may be sitting for long time and working on computer for long time.When such patient comes to my hospital we usually take x ray spine or if required MRI to see for any nerve compression.Some exercises which can be done after pain has reduced include- Spine extension exercises, Lying on your stomach flat lift leg 6 inches from ground, do it for other leg.Now lift both hand and leg simultaneously, 6 inch off the ground and stayPosition for around 10 breathes. Core stabilizing spine exercises will help.You may consult physiotherapist for further guidance. He may start TENS, or ultrasound which is helpful in your case.Hope this answers your query. If you have additional questions or follow up queries then please do not hesitate in writing to us. I will be happy to answer your queries. Take care."
},
{
"id": 167154,
"tgt": "Suggest remedy for the burnt mouth and tongue in a breast fed child",
"src": "Patient: my son has burnt his mouth on a hot stone oil burner. He has a white burn mark on the tip of his tongue and just on the front of his tongue. Unfortunatly he can not breastfeed as it is obviously too painful. Have given him some painkiller. Is there anything i should be looking for or need to do? Doctor: Hi..Welcome to HCM..As per your complain it seems that your son has got ulcer on the tongue tip of tongue due to burning and due to it there is inflammation and burning sensation of the tip leading to pain..Along with giving him a painkiller you can apply numbing gel containing Lignocaine over the ulcer to relieve pain as it will numb the area..Along with it do cool compresses over the tongue tip to reduce inflammation and pain..You can also give anti inflammatory painkiller like Ibuprofen as it will reduce inflammation and also relieve pain..In case if you do not see any improvement consult a Paediatric dentist and get evaluated..Hope this helps..Regards."
},
{
"id": 27443,
"tgt": "What causes palpitations and high pulse rate?",
"src": "Patient: Hi, I have been sick since oct.. But the last 7 days I've had pounding heart, pulse is high sometimes, this is laying and standing.. I know normal, this is not it.. Severe fatigue.. I can only stay up like 10 minutes...I had an EKG .. Normal.. Thyroid .. Normal.. She thinks its anxiety/depression..I'mTaking Wellbutrin/ Ativan .(4days only)....nothing is changing..please help Doctor: Hi,Usually it takes 1-2 weeks, before medication starts to act.I would advise you to have regular regimen, not to use computers much, and not to watch much TV, have healthy diet, be more in fresh air, and you will feel better in a week.Hope I could help youWishing you good healthIn case of further questions don't hesitate to askRegards,"
},
{
"id": 119946,
"tgt": "What causes cramps during yawning?",
"src": "Patient: Occasionally when I yawn (lately--last few weeks), I get what feels like a cramp on the left underside of my jaw/chin area. I have to relax my jaw and shake it out to stop it. It is pretty scary. What could that be. I am 44, 5 1 and about 127 pounds in very good physical health with no medical problems or negative medical history. Doctor: Hello,The symptoms seem to be related to temporomandibular joint disorder. I suggest using anti-inflammatory medications such as Acetaminophen to relieve the pain. If the symptoms continue, you may visit your doctor for a physical examination.Hope I have answered your question. Let me know if I can assist you further. Regards, Dr. Dorina Gurabardhi, General & Family Physician"
},
{
"id": 163342,
"tgt": "What are the ill effects of swallowing soft vinyl?",
"src": "Patient: Hi there, just came back from the er with my 9 month old, earlier this evening he swallowed a quarter size thin peice of what feels and looks like soft vinyl, his breathing was great and his throat clear, but i m still worried about this damaging him. It came off of a soft toy ball which was made in 2002, I m also a little worried about what kinda chemicals could be in the peice he swallowed. This happened almost 8 hours ago now, if there was anything toxoc would he have developed symptoms by now? Doctor: Hello,No need to any worries if eight hours are passed and no symptoms appeared. Check for a cough and vomiting and monitor for pulse and temperature.Hope I have answered your query. Let me know if I can assist you further. Regards,Dr. Hina Javed"
},
{
"id": 83516,
"tgt": "What are the side effects of zavolis 50 mg?",
"src": "Patient: sir, I am using diavista30mg but when i consulted a doctor for review he changed the medicine as it is causing burning sensation of urinary tract, and prescribed zavolis 50 mg it is costlier and whether it has any side effects or it is safe to use please let me know Doctor: Hello Zavolis-50 is commonly prescribed alone or along with metformin to treat diabetes. As compared to diavista-30 it is quite safe however it may cause delayed stomach emptying, nausea or flu like symptoms. Hope I have answered your query. Let me know if I can assist you further. Take care Regards, Dr. Mohammed Taher Ali"
},
{
"id": 33168,
"tgt": "Suggest treatment for influenza A & B",
"src": "Patient: Hi , im currently 9 weeks pregnant and just got discharged from the hospital from getting influenza A & B .. They prescribed me robitossin with codeine and ambien and night.. Eith many other medications ( albuterol, azithromycin,ponk magic mouthwas, zofran and tamiflu.....Will it hurt my baby? Im concerned... Doctor: Hello!I really appreciate your concern. Every drug has certain side effects on human body and on developing fetus. But as you are having Influenza infection it is more harmful to baby in comparison to these drugs. So taking risk-benefit ration in mind your treatment is selected.Hope my answer has solved your query. Take Care.Thank you. Best Regards.Dr. Manan"
},
{
"id": 195027,
"tgt": "Suggest remedy for painful testicles with fluid discharge during ejaculation in men",
"src": "Patient: I am 48 years, last few days i am feeling fever with pain in left testis side, i fund brown fluids coming out from balder. I go for urine examined and found RBC cell 10-12/HPF & PUS cell 10-15/HPF.I am completed 5 days of antibiotic, now there is Pain & fever, bur during ejection of sperm it is brown in colour.Please suggest me what can i do.Regards Suraj Patnaik Doctor: Hi, You can have seminal vesiculitis as per mentioned symptoms. Sometimes prostate inflammation also can lead to such a problem. I suggest you investigate with ultrasonography pelvis (preferably transrectal ultrasonography) to check seminal vesicle and prostate. It's advisable to investigate with semen culture study as well to find the underlying cause. Until report comes Levofloxacin like broad-spectrum antibiotic should be started which is having a good effect on seminal vesicle and prostate infection. I suggest you consult Urologist for examination and discuss all these. Hope I have answered your query. Let me know if I can assist you further. Regards, Dr Parth Goswami, General & Family Physician"
},
{
"id": 55761,
"tgt": "Suggest prognosis for health condition with end stage of liver disease",
"src": "Patient: Hi, I am wondering if someone has major edema in legs, feet and belly and diagnosed with end stage liver disease...is there any hope? Asking for a family member. She has just been prescribed two diuretics. Wondering what is in store for her, as well as us the family. Your information is greatly appreciated. Thank you. Doctor: Welcome at HCM i have gone through your query and being your physician I completely understand your health concerns... Life expectancy depends upon how bad the liver functions are. You need to to get her LFTs along with abdominal ultrasound scan to look for ascites . . Your doctor will calculate the overall score called child Pugh score ...it includes bilirubin levels along with serum albumin and PT, degree of ascites and encephalopathy.... Five year survival rate is calculated on this score which us 70% for child A( mild disease).. Meet hepatologist for proper work upRegardsDr saad sultan"
},
{
"id": 129042,
"tgt": "How to treat muscle spasms around the waist after an injury?",
"src": "Patient: I took one pill tonight because I fell down and have pain in my bone in the waist and all around like the muscles and I was feeling better and after the pill I feel a muscle spasm sensation in my waist and I don t know if its normal.. I think I was feeling better before the pill. Doctor: After an injury ,usually the muscles will undergo spasm surrounding the injured region and you need to take muscle relaxants along with analgesics for relief and ofocurse have to take rest for 2-3 daysHope this helps"
},
{
"id": 173821,
"tgt": "What causes mental disorder in a 6 year old?",
"src": "Patient: Raising my grandson for the last Six (6) yrs. Has PTSD from a series of tragic events. Most tragic was finding his 18 yr old brother seven yrs ago deceased from an OD; has drug addicted mother. Now taking 10 mg Paxil for anxiety. Recently, aggression has returned to point of some fear on my part. Grandson more agitated beginning this morning. He explained this morning he accidentally took his regular 10mg of Paxil, forgot, (He says) & took a second 10 mg dose this morning. Also, prescribed Ativan ,which he did take this afternoon; does not seem to alleviate aggression symptoms. Has left for the evening w/friend to stay overnight. Is it possible the add. 10 mg s has made his behavior more unmanageable? Thank You for any assistance to help me. Doctor: Hi,Thank you for asking question on health care magic.Really you have undergone lot of stress in your family.Addition of 10 mg extra of Paxil will not result in agitatation that is unmanageable.You better consult psychiatrist for reassessing the problem and alteration of treatment if needed.Hope this answer will serve your purposePlease feel free to ask any more queries if requiredTake careDr.M.V.Subrahmanyam MD;DCHAssociate professor of pediatrics"
},
{
"id": 40911,
"tgt": "How effective is Ovigyn tablet in treating secondary infertility?",
"src": "Patient: hi! I m 42 yrs old. I have had 5 failed ivf cycles. I am suffeing from secondary infertility. I had an an ectopic pregnancy resultinf in cutting of my right tube in 1999. I had other health issues due to which i could not try conceiving. Tjen i had a chocolate cyst on my right ovary. Sunce past 8 yrs i have been trying ivf. The second attempt resulted in pregnancy but ended as blighted ovum. In my previous ivf cycle my dr. Prescribed ovigyn d 25 mg thrice. My response was fairly good.. Now i am just teying to conceive naturally,though difficult,(i ve never tried the having regular sex route previouslt due to certain circumstamces) i m not seeing any dr. At the moment. Do you suggest that i should take ovigyn tab considering my age? Does taking ovygyn has any advwrse effect on the cyst? Kindly reply & help ,as time is crucial for me Doctor: Hello,Thanks for submitting your question here.You are trying to conceive naturally and have a query regarding the Ovigyn tablet.Ovigyn tablet contains a compound termed dehydroepiandrosterone which helps in improving fertility.It causes ovulation induction if your eggs are not being released.But mind it that it can cause nausea,headache in some woman.This tablet has neither any adverse effect of the cyst nor it is contraindicated as per your age.I suggest you to take this tablet without any confusion while trying to conceive.SincerelyDr Sanjoy"
},
{
"id": 117768,
"tgt": "How to improve hemoglobin levels?",
"src": "Patient: I am 25 years old female and 21 weeks gestational period of my first pregnancy. My hemoglobin level is 8.0 only. Ultra sound scan report is as follws: BPD 52 mm, FL 36 mm, HC 188 mm and AC 156 mm with 156 bpm heart beat. Is there any harm to growing baby with this low hemoglobin levels. Could you please suggest me how to improve my hemoglobin levels instantly. Doctor: HI THANKS FOR POSTING YOUR QUERY ON HEALTH CARE MAGIC.GOING BY YOUR LAB REPORTS ,IT SEEMS OBVIOUS THAT YOU HAVE \"MILD IRON DEFICIENCY ANEMIA\".THIS CAN HAVE IMPACT ON THE GROWING FOETUS.COMPLICATIONS LIKE IUGR-INTRAUTERINE GROWTH RETARDATION IS VERY COMMON IN THE BABY.ALSO THERE IS HIGH TATE PREGNANCY RELATED COMPLICATIONS EXCESS BLEED DURING DELIVERY.ALL YOU NEED CURRENTLY IS THERAPY IN THE FORM OF ORAL IRON-TAB FERROUS SULPHATE 300mg thrice a day. THIS REPLENISHES IRON OF 60mg PER DOSE.THIS THERAPY HAS TO BE CONTINUED FOR 3 MONTHS .THEN THE BLOOD HEMOGLOBIN LEVELS HAVE TO BE RETESTED.SIMULTANEOUS FOLIC ACID 4mg HAS TO BE SUPPLEMENTED TO PREVENT NEURAL TUBE DEFECTS IN THE BABY.CONSULT YOUR GYNAECOLOGIST IN CASE OF FURTHER ASSISTANCE.THANK YOU.TAKE CARE."
},
{
"id": 117097,
"tgt": "Could enlarged blood cells and protein showing 185 be due to intake of a bottle of wine everyday?",
"src": "Patient: I drink a bottle of wine a day and the doctor has told me that my blood cells are enlarged, the results were showing 185 - i think that was for the protein. is this due to alcohol?? i also have asian skin and spider naevi is not that visible but i do have a few marks Doctor: Hi, dear. I have gone through your question. I can understand your concern. Alcohol is one of the cause of macrocytic rbcs and also for your protein level. It can causes fatty liver and long time consumption of alcohol can leads to cirrhosis of liver.So you should avoid alcohol. Hope I have answered your question, if you have doubt then I will be happy to answer. Thanks for using health care magic. Wish you a very good health."
},
{
"id": 182014,
"tgt": "Suggest treatment for infection on tooth",
"src": "Patient: Hi my name is hope. In April 2014 I had a infected tooth which I got pulled. They put me on antibiotics to take. Everything seemed fine. Starting in May I didn't feel the same everything went downhill. I have bin getting sore arms sometimes even get a werid sensation in my body. I still get every now and again. I went to my doctor got a lot of blood work did a X-ray on my sinus and even a mri on my head. It all came back clear. Nothing was wrong. But now starting in October I have bin getting a werid feeling in my head almost like there is fullness I get confused I get mood swings I'm not always in the mood to respond to people when they talk to me. I could be fine one minute then the next my personality will change. It's very frustration and I don't know what to do. I don't know why I feel like this. I just want to get back to my old self and feel good again. Doctor: Tooth extraction and all investigations being normal point to one direction only. To be specific it is depression your going through, could be work related or personnel. You need psychiatric evaluation and appropriate medication and councelling sessions will definetly help."
},
{
"id": 14497,
"tgt": "What do scabs with itchiness and white bumps indicate?",
"src": "Patient: For about 6 months I have been receiving these scabs on my legs and arms and they keep spreading. I have gone to a doctor and she seemed really worried. She suggested it might be allergies but does not know. I can't get into the dermatologist until next month but it's getting worse. The itchiness won't subside at all even with itching cream. And also when it's cold my skin gets even more itchy and white bumps appear on the surface of where I scratch. What could it be? Doctor: Hello,Thank you for posting on HCM.It\u2019s very difficult to approach conclusive diagnosis unless for proper physical examination, anyways from your description it seems you are suffering from scabies.Scabies is a contagious disease caused due to mite and it spreads among people who are in close physical contact and by use of common household articles.Treatment is pretty straight foreword. Firstly, i would suggest treatment of your secondary infection (if any) with oral and topical antibiotics.The key to successful treatment of scabies lies in treating everyone simultaneously in the family and others who are in close physical contact. Your doctor would prescribe oral drugs like ivermectin which will kill the parasite. You might also be advised application of permethrin 5% cream and a corticosteroid with fusidic acid cream. Apply the medicated creams exactly as advised by your doctor. Take antihistaminics as advised to relieve itching and irritation. Avoid sharing of clothing and toiletries and launder all you bedding and clothes in warm water.Hope this will take care of your issue.Wish you best of health.Thank youDr Hardik Pitroda"
},
{
"id": 169365,
"tgt": "How long will the effect of focalin be in kids?",
"src": "Patient: Hello, I m a clinical psychologist and I conducted an eval with a 9 year old client who is diagnosed with ADHD, and currently taking Focalin - I don t have the dosage. He was not on his medication at the time of the evaluation, and I noticed no symptoms of hyperactivity or inattention over a two hour timeframe in the middle of the day. How long with the medication stay in his system? I believed it to be short lasting. His guardian indicates his dosage is effective for remediating symptoms at home and at school. Global question: Does Focalin help kids, mediate symptoms, who do not have ADHD? Doctor: Hello there,Thank you for your question and welcome to HCM. to answer your question about Focalin, the half life is about 8 hours. It also depends on how long the child has been on the medication. If the child has been on it for several years, it will take a longer amount of time to be fully out of his system. If he was on it for 6 months months or less, he should be clean after about 16 hours. You are correct about it being short acting.Hopefully this answers your question. Have a good day!"
},
{
"id": 137261,
"tgt": "What causes sudden swelling and redness on foot?",
"src": "Patient: What does it mean if the top of my left foot is swollen and red? I do not remember hurting it or twisting my ankle, nor did I fall down. I gradually started feeling discomfort while doing daily tasks like laundry, etc., then increasing into pain before I finally sat down and used an ice pack. I have had 2 aortic valve replacements so far, 1993 and 2006. Might this pain be a warning? Please respond. Thank you for your time and patience. Namaste. katie 55yr old 5 2 , 158 lbs, bp 116/66 p 60. Doctor: redness and swelling on feet without trauma may be mainly due to rheumatological or infectious disorders. you can try with an ice bag: if pain worsens, consider suffering from gout. if pain goes better, there is an inflamation, that may be caused by lots of conditions. if you have fever, you likely have a cellulitis (infection). I suggest to you to take an xray and do blood tests (uric acid, crp, esr, blood count, white cells count) to understand as soon as possible the underlying cause. NSAIDs will help you with pain"
},
{
"id": 28310,
"tgt": "What causes rise in BP?",
"src": "Patient: I am taking generic for zebeta. dose is 2.5 mg q am. for atrial fib. lately my bld. pressure has been going up in the late afternoon in the range of 140/80 and just now was 150/90. my heart rate is 53. I have been taking the other half of zebeta around 7:00 pm. The bld pressure comes down after about 2 hrs. and in the morning it is up again. Any recommendations? Doctor: HIWell come to HCMI really appreciate your concern, if you feel that your blood pressure is not controlling well then you need to consult physician for further investigation and it is most advisable to read the blood pressure by physician because if you are reading it on your own then chances of error is very likely, EKG need to be get done on and often if this is atrial fibrillation, you have not given your basic information here and that is your age and gender this was extremely needed here for better advise, cardiac disease always need to be managed by cardiologist, hope this information helps, have a nice day."
},
{
"id": 24834,
"tgt": "How safe is taking rancad 500,korandil 5 and modlip asg 75 for mild chest pain?",
"src": "Patient: My mother is 68 years old, she is a diabetic patient with high blood pressure. She suffers from heart failure and short of breathing, earlier medicine was given to her are glycinorm 40 - twice a day, tazloc h 80 - once a day, cardivas 3.125 - twice a day, nitrocontin 2.6 - twice a day, modlip 5 - once at bed time. After taking the mentioned medicines, she was doing fine and all the reports like blood pressure, blood sugar, serum creatinine, lipid profile etc. are found within its normal range. Since last 3 weeks she is suffering from mid chest pain, which lasts for 15/20 minutes. We consulted another physician and he altered modlip 5 with modlip asg 75. at the same time he added two other medicines, i.e. rancad 500 - twice a day and korandil 5 - twice a day. My mother is afraid of so many medicines. my question is why rancad 500 and korandil 5 was given and modlip 5 was altered with modlip asg 75? while keeping other previous mediciines also? Doctor: thank you for your question on hcm your mother might be suffering from angina cardiac pain.that's why he added aspirin in modilip.but this is not the solution. we want to know exact character of pain.if this is angina then you should consult cardiologist for early coronary angiography"
},
{
"id": 18830,
"tgt": "What causes dizziness and high blood pressure?",
"src": "Patient: Hello. On Monday morning I became very dizzy and disoriented to the point where I could not walk straight. My hands swelled up, as well, and I felt very \"heavy\". This lasted 4 hours, my blood pressure was high (normally not). I had no fever and my pulse was normal. After that went away, I started getting ringing in my ears, seeing black spots in my left eye, a headache (more prominent in the back of my head) and feeling light headed. I went to the doctor at that time, they took blood to test my levels and she told me to have someone pick me up and take me home, to rest and if it got worse then go to the emergency room. I am going back to the doctors on Friday - but would you have any idea of what might be happening? They mailed my lab results yesterday so I'm assuming that everything is normal since they didn't call me. Doctor: Hello,All the symptoms you have described may be due to high blood pressure . Keep a track on your blood pressure and it should be below 140/90 mm Hg. Start morning or evening walk. Cut down on smoking and fried salty foods . If your blood pressure is above 160 /100 mm Hg then start tablet telmisartan once a day.Hope I have answered your query. Let me know if I can assist you further.Regards, \u00a0\u00a0\u00a0\u00a0\u00a0 Dr. Varinder Joshi"
},
{
"id": 4360,
"tgt": "Could delay in period despite taking Ipill be suggestive of pregnancy?",
"src": "Patient: I had I pill on 28th December then I got my periods on 7jan then again I had ipill on 15jan n on 21st got my periods n on 4th feb had intercourse didn't take I pill now today is 19feb still my periods have not started n again I took ipill now when my periods r expected Doctor: Hello,Firstly, let me tell you that you have misused the emergency pill and put it into too much use. An emergency pill is meant to be used only a couple of times a year; else, it can lead to catastrophic patterns of menstruation. Since you have attained menstruation which was indeed a withdrawal bleed and this was followed by sexual contact, you should have excluded pregnancy before taking the next pill. Emergency contraception can be of use only when used soon after an intercourse and not at any part of the cycle. In the current scenario, I would suggest seeing a specialist if you do not see a period in the next two weeks; get a pregnancy test and plan further management. In future, please go for some form of regular contraception and do not use emergency pills so frequently. Hope this helps."
},
{
"id": 33449,
"tgt": "What causes persisting fever?",
"src": "Patient: I am suffering from fever since last whole month.Its between 99 and 100.I dont have any other problem like flu,cough etc but just have headache and weakness now.I had series of tests like typhoid,LFT,UDR etc but they all are normal. Whenever i took panadol,fever goes down but after sometime it reaches between 99 & 100 again. At night it becomes normal but with the rising day it rises. I got married 4 months back but i am not expecting at the moment. Doctor: HIGreetings from Dr.Divakara.PThanks for posting your query. Well if fever is persisting then it means that there is still persisting infection. You have not been completely evaluated with tests. You need to be done certain more test which shall detect the cause of fever. In that regard I would like to advise certain more tests , kindly get these tests done and revert back to me . ( You can skip the tests if these were done prior ). Monteux test, Blood culture sensitivity , Urine Culture sensitivity , IgM Leptospira Abs, Brucella Antigen test, Chest X ray, X ray PNS. Hope this information was useful to you. Any clarification feel free to ask."
},
{
"id": 161171,
"tgt": "What causes stomach pain and vomiting in a child?",
"src": "Patient: hi my 2 1/2 yr old son woke up w/ stomach pains then threw up.been lathargic ever since.w/ his bdy feel HOT 2 touch[sister went to buy thermometer]so not sure of bdy temp.then 2 hrs later throws up yellow bile.has not ate since yesterday.then his body gets freezing cold nd hes shaking bc so cold then w\\ in 15min body is HOT again nd still saying stomach hurts him.HELP Doctor: Hello, It seems your kid is having viral diarrhea. Once it starts it will take 5-7 days to completely get better. Unless the kid's having low urine output or very dull or excessively sleepy or blood in motion or green bilious vomiting. You need not worry. There is no need to use antibiotics unless there is blood in the motion. Antibiotics might worsen if unnecessarily used causing antibiotic-associated diarrhea. I suggest you use zinc supplements (Z&D drops 1ml once daily for 14 days) & ORS (Each small packet mixed in 200ml of potable water and keep giving sip by sip) as hydration is a very important and crucial part of treatment. If there is vomiting you can use Syrup Ondansetron (as prescribed by your pediatrician). Hope I have answered your query. Let me know if I can assist you further. Take care Regards, Dr Sumanth Amperayani, Pediatrician, Pulmonology"
},
{
"id": 33006,
"tgt": "What causes recurring fever with cold?",
"src": "Patient: Since December I have had the flu 4 times already (so practically every month), lasting anywhere from 5 days to 2 weeks. Every time I have had a fever and backaches. The doctors says this is normal and I should not be worried, but last month and this time there is a heightened sense of smell that makes me extremely nauseous. I get my periods regularly and don't believe I can be pregnant. Why am I getting the cold so often? Doctor: Hello,One of the resion for reccurrent infection is our immunity. If immunity is stronger then this problem not occur. Your airway might be more sensitive to the minor changes. So try to improve your immunity by proper diet , doing exercise and try Yogas and Pranayam .Thank you. Take care."
},
{
"id": 101102,
"tgt": "What causes sneezing, watery eyes, throat pain and coughing?",
"src": "Patient: Sir, My eosinophil count is 25. I have been suffering from cold, cough, frequent sneezing, watering of nose and eyes, aching of throat, etc. The moment I get up in the morning I start sneezing and watering of nose and eyes continuously. Anytime I rub my nose I start sneezing frequently. Even in late I would get up and get sneezing frequently with water coming from nose and watering of eyes. I have been taking anti histamine medicines like Le Zyncet, cetzine etc. I have also tried with a full course of eofiles. But these are of no use. Can you kindly help me. Can you advise me some ways of naturopathy. Will change of food habits help in this regard. Kindly help. Regards Kishore Chandra Satapathy Bhubaneswar, India e-mail: YYYY@YYYY Doctor: Hello Kishore Chandra Satapathy,Thank you for asking at HCM.I went through your history and would like to make following suggestions for you:1. Were I treating you, I would suggest you intranasal corticosteroids (mometasone or fluticasone propionate) for at least 2 weeks. I would also suggest you montelukast and levocetirizine/cetirizine for at least 4 weeks.2. Personally I would also suggest you allergy testing which will help you identify the substances causing troubles to you as well as know the measures to avoid them.3. Daily nasal irrigation with nasal saline sprays and regular nasal steam inhalation (please be cautious to avoid burns) will help you and will hasten recovery.4. Please avoid exposure to dusts, smokes and air pollution as much as possible. 5. Regular breathing exercises, plenty of warm fluids and nutritive diet rich in vitamins and minerals (adequate amounts of green leafy vegetables, fruits, sprouts, etc) will also help you in long run.6. As you have asked for naturopathy treatments, apart from breathing exercises and healthy food, regular Yoga & Pranayama also will help you.7. In naturopathy, fasting is considered an important measure for treating allergies but you should perform it only under a naturopathy practitioner's guidance.8. Instilling one-two drops of refined sesame oils into each nostril may be useful as a simple remedy. However, please consult a naturopathy/ayurvedic practitioner to avoid any adverse consequences.Hope above measures will be helpful to you.Should you have any further query, please feel free to ask at HCM.Wish you the best of the health.Thank you & Regards."
},
{
"id": 44990,
"tgt": "I have a blockage in my vas deference, Is it possible to clear the block ?",
"src": "Patient: hai, I am 32 yrs old male. Now I found to have a blockage in my vas deferens. Is it possible to clear the block ? Doctor: HI well come to H.C.M. You should consult Uro-surgen and looking to report he will explain you kind of surgery if required for that. Thanks"
},
{
"id": 121641,
"tgt": "What is healing time for axillary abscess?",
"src": "Patient: my daughter just had an axillary abscess I &D. The doctor packed it and she has been on Clindamyacin 150mg three pills, tid since 12-27. What is the usual healing time? How about showering, I am sure that shampoo will get in it inadvertantly. Please advise.Thanks,A concerned mom Doctor: Hello, I suggest you to care for abscess which include wound repacking, soaking, washing, or bandaging for about 7 to 10 days. This usually depends on the size and severity of the abscess. After the first 2 days, drainage from the abscess should be minimal to none. All sores should heal in 10-14 days. Hope I have answered your query. Let me know if I can assist you further. Take care Regards, Dr. Blerina Pasho, General & Family Physician"
},
{
"id": 71165,
"tgt": "Is wheezing sound that stops after Dulera Inhalation a cause for concern?",
"src": "Patient: I am 64 years old have my physicals yearly. I had a physical on 12/21/16. My doctor listened to my lungs and on the right side heard some wheezing using a stethoscope. He had me breath take 2 breaths of Dulera Inhalation aerosol and it was gone. He had me take a chest x-ray. Was just looking up wheezing Doctor: Hello and Welcome to \u2018Ask A Doctor\u2019 service. I have reviewed your query and here is my advice. I understand your worries. Do you have any breathing difficulties? Any mucus secretions? Dulera inhaler is used to prevent and control asthma so no need to worry about it. If you have any symptoms like breathing difficulty or mucus secretion then, I suggest you to take Salbutamol inhaler with the advise of your physician. Hope your doubts are cleared. Take care"
},
{
"id": 81792,
"tgt": "What causes burning in chest and left arm/neck?",
"src": "Patient: Hello. I have a burning in my chest, up my left side of neck and my left arm feels funny..my hand tingling. On Tuesday I went to the ER for severe chest pain in heart (that is what it felt like). It wasn t crushing but it was in heart and down arm, I was nauseated and very anxious. EKG was normal don t know what blood results were. I m 43 with Celiac as well. I m scared now. Nobody seems to take this seriously. thank you. Doctor: Thanks for your question on HCM. Since you have normal ECG, in my opinion you may have either muscular pain or GERD,as a cause for your chest pain.So try to follow these steps for better symptomatic relief in muscular pain. 1. Avoid heavy weight lifting and strenuous exercise. 2. Avoid bad postures in sleep. 3. Take good painkiller and muscle relaxant. 4. Apply warm water pad on affected site. For GERD ( Gastro Esophageal Reflux Disease ) avoid hot and spicy food. Take proton pump inhibitor. Avoid stress and anxiety."
},
{
"id": 62384,
"tgt": "What causes bump on the back of leg?",
"src": "Patient: It started with 1 bump on the back of my leg and then 1 day later it started bleeding i put a bandaid over it and another day later i found other bumps those too started bleeding soon after some stopped bleeding but about 3 still are bleeding. what is it and what should i do? Doctor: Hi,Welcome to HCM. Based on the facts of your query,You seems to have Bleeding Venus Varices of the leg.Don't worry as it is not cancer.Still if worried, I would suggest, Check with Surgeon and get Second opinion on this and get Color Doppler of the leg veins done if need be.It could be from boil,with scratching,which needs to be ruled out by Surgeons opinion.Contact with a Followup Premium question to ME.Will appreciate your Hitting thanks and writing excellent review comments to help needy patients like you. Good Day!Dr.Savaskar,Senior Surgical Specialist-M.S.Genl-CVTS"
},
{
"id": 59507,
"tgt": "Diagnosed with hepatitis B. Is there a permanent cure? Can I take Liv 52 HB?",
"src": "Patient: i am recently being diagnosed with Hep B as my wife and a child does not have this desease. I visited doctors in many of the hospitals and was advised that, i don t need any treatment further as it s virus is under control. If i would start having this Liv.52 HB, is there a possibility that, i could be cured completely & if yes, for how long do i need to take it. I shall, indeed thank you for your valuable advise in this regard please. Doctor: Hello,\u00a0 Individuals who remain HBsAg positive for at least six months are considered to be hepatitis B carriers.Carriers who have seroconverted to HBeAg negative status, in particular those who acquired the infection as adults, have very little viral multiplication and hence may be at little risk of long-term complications or of transmitting infection to others.\u00a0 However you should keep on monitoring your ALT levels and if they are ever found above 60,then you would require the treatment with interferons and immune modulators.\u00a0 40% of individuals with hepatitis B having normal ALT have hepatic fibrosis. Treatment with LIV -52 has no role. One can never get rid of HEP B,if he is having this infection for more than 6 months. Keep in regular touch with your gastroenterologist and start the treatment immediately,the moment you find that there is reactivation of the disease. Get your wife and child vaccinated with HEP B vaccine. Thanks"
},
{
"id": 151677,
"tgt": "Rectal cancer, on radiation therapy, unable to move legs and arms, motor neuron disease, cure",
"src": "Patient: my mother had rectal cancer and radiation 18 months ago. 2 months after rdiation her legs would buckle. in summary, the past 6 moths she cannot walk and now she cannot move her arms to feed herself,she keeps her arms and hands under her breastbone and they are stiff. She now has spasms in her lower back and legs. She was recently inpatient at XXXXXXXX fl to get a diagnosis and to rule out lou gehrigs disease and paraneoplastic syndrome . MD(Neurologist) said test came back negative other than severe low motor neuron results from the EMG . Therefore he said he cannot give a diagnosis. In the meantime she is not improving and is extremely depressed saying she wants to die. Any thoughts or suggestions? Doctor: what is my suggestion that you atleast have MRI OF Cranium and Spine to rule out any metastatic disease to either brain or Spine with spinal cord compression. and this being as side effect of radiotherapy is not a matter related with."
},
{
"id": 6121,
"tgt": "Fasting sugar, TSH, prolactin normal. Follicular study and ultrasound done. Taking siphene. Can pregnancy occur?",
"src": "Patient: In Oct 2011 we started our treatment. I did my insulin , fasting - 7.27, Thyroid Stimulating Hormone - 0.890 and prolactin - 12.86. Report says normal. And then from jan 2012 we are trying for baby. In April cycle (this month) my gynecologist ask me do Follicular Study on 9th day of my period. And said to call her on 9th after getting report so that she can put me on Inj. FERTI.GYN 5000 ( hcg ). After hearing to report she said no need of Inj and do another test on 12day of cycle. Can anyone tell me why i dont need Inj. Report says:- Uterus is anteverted, normal in size and contour. Endometrial and myometrial echoes are normal. No focal or diffuse lesions noted. Uterus measurement 63X31X34mm. ET measurement 6mm Both ovaries are normal in size and echotexture. Dominant follicale is seen in the right ovary measuring 10x10mm. Right ovary measurement 19x14mm Left ovary measurement 34x20mm Both adnexal area are normal. Again i did scan on 12th day of my period. ET 6mm Right Ovary (Follicles) 11x11mm Left Ovary (Follicles) 10x8mm From last 2 cycle i am having siphene tablet 100mg. Can anyone tell me what is the problem ? What treatment i need? How many month it will take to solve? At least next month i want to get pregnant. Doctor: Hello. Thanks for writing to us. It takes time to conceive in spite of best of treatments. In your case, injection Hcg was not given as your right ovary already shows a developing follicle. It is given when the follicle will mature- more than 18mm in size. You are getting the right treatment. Follow your gynecologists advise. I hope this information has been both informative and helpful for you. Regards, Dr. Praveen Tayal drtayal72@gmail.com"
},
{
"id": 179041,
"tgt": "How to treat constipation in a 7 months old child?",
"src": "Patient: my 7 month old, who is fixing to be 8 months old strains so hard to have a bowel movement, she turns red and purple trying to poop, when she finally does it is little pebbles, usuall one or two, we have tried prunes, pear juice water, warm water in the rectum, suppositories and everything nothing is working what can i do for her Doctor: Thanks for posting on HealthCareMagic. Your daughter is having constipation. You have not indicated whether she is breast fed or formula fed. Constipation is often seen as formula milk is hard to digest.By 7 month of age weaning should have already been done and solids introduced.You may give her lactulose syrup along with foods with high fibre content. Include green leafy vegetables in the mashed and boiled food to be given to him.Vegetables are a major source of fibre. Lettuce, Swiss chard, raw carrots, and spinach are good. You may also give:Tender cooked vegetables, such as asparagus, beets, mushrooms, turnips, and pumpkin.Broccoli, artichokes, squashes, sweet potatoes, and string beans.Vegetable juices.Legumes, such as lentils, black beans, split peas, kidney beans, lima beans, and chickpeasSunflower seeds, almonds, pistachios nuts, and pecansFruits are another good source of fibre. You can give:Apples and bananasPeaches and pearsTangerines, prunes, and berriesFigs and other dried fruitsGrains are another important source of dietary fibre. You can include:Hot cereals, such as oatmeal, farina, and Cream of WheatWhole-grain breads (whole wheat or whole rye)Brown riceHigh-fiber cereals (such as bran, shredded wheat, Grape Nuts, Ry Krisp, and puffed wheat)Whole-wheat pastasBran muffinsSuppositories and P-enemas should be able to make her pass the stool. Consult your doctor if those are not working.Hope that helps. Feel free to revert back in case of further queries if any."
},
{
"id": 181091,
"tgt": "Will an extra skin tag fall off after removing a wisdom tooth?",
"src": "Patient: Hello, I got my wisdom teeth removed four weeks ago and they seem to be healing well, but I m convened cause in my right wisdom tooth socket there seems to be a piece of skin hanging down. Will this fall off? Is it something I should be concerned about Doctor: Hi..Thanks for the query..It seems that there is a piece of gum mucosa that seems to be hanging and has not taken part in healing..When the tooth is extracted the gum mucosa gets hanged as it is not supported by tooth and in case if the flaps are not compressed properly they can get hanged and does not take part in healing..It will not fall off on its own and you should consult an Oral Physician and get evaluated and he can either remove it by a small incision or can stitch it to the remaining portion of gums..Hope this helps..Regards.."
},
{
"id": 11662,
"tgt": "Skin rough after using bentovit. Can I help my pigmentation and stop darkening?",
"src": "Patient: hi doctor, i am 31, female my problem is after 2 years uses of the mixture of archi, fair and lovely, stillman and bentovit n cream my skin became rough and acnebreakout occur. I went to the doc he prescribed the treatment of glaycolic peel, after taking the 2 treatments my acne breakout increased and also my skin became dark and pigmentation occur which i didnt see before. He stoped the treatment and suggest me to take dine 35 pill for 6months, now i have no acne but my skin grdually became more darker and pigmentation increase day by day. In between i went to another dermatologist she also prescribed me to continue the dine 35 and add some cream which is hydronin2 bleaching cream after using it 2-3 days my skin became more darker. Again i went to her she change the cream and suggest to use neutrodarm lightening cream. I stoped the previous one and use it 21-30 days but i didnt see any result. Again and again i went there and she changed the treatment and 1st she had done the treatment by pushing selain with glutathion+vit c and oxylight facial than after 15 days apert IPL laser again after 15 days apert dual yellow light laser . But i didnt see any visible result. After taking laser treatment my skin became dry, tan skin and my eyes became puffy. My pigmentation also increased. Now she prescribed to use neodespigmental cream and cap lusida ds. If i use this cream in my sensitive skin it itching in my skin. Above mentioned theatment doesnt work in my skin. Now plz give me a suggestion what treatment will i take for my pigmentation and how can i make my skin fair? Doctor: Hello,Thanks for writing here.It seems you are having post acne hyperpigmentation along with sensitive skin.Sensitive skin is both oily and dry at the same time.Use a good cleansing lotion like cetaphil for face.Avoid sun as far as possible.Drink plenty of water and eat fresh fruits.Get your hemoglobin level and thyroid checked.Depigmenting creams have slow onset of action .Keep patience and continue with treatment.Hope this will help you.Take care."
},
{
"id": 6718,
"tgt": "What vaccinations should one avoid before pregnancy to ensure baby's safety ?",
"src": "Patient: What r the vaccine to avoid bfor pregranancy?my girl friend had some injection like 25 days ago,am really worried that it ll affect the baby,pls help Doctor: welcome to healthcaremagic any live or attenuated viral vaccine preparatios are not to be taken in pregnancy. for vaccination of close contacts with pregnancy the pros and cons should be considered. beneifits should be weighed against risks and decission taken"
},
{
"id": 188254,
"tgt": "Child's teeth x-ray shows cavities, root canals to be done. On good diet. Second opinion required?",
"src": "Patient: my 4 year old son just had x-rays and after going going to the same dentists every 6 months since he was 1 i was just told he has 7 cavities and has to have 4 root canals. I am beyond upset and feel they missed something months back and feel like this is beyond crazy. no soda, and little juice are given to my kids, good diets are met so i am so in shock!! do i need to get a second opinion or do the x-rays just tell it all? Doctor: Hello,If your dentist has given exact clinical as well as x-ray evaluation,you can trust on him.Root canal treatment is usually done in cases of cavities involving pulp tissue.In this procedure,entire infected pulp has to be removed.You can also go for a second opinion.Brush your child's teeth regularly.Rinse mouth after every meals.Take care."
},
{
"id": 26517,
"tgt": "How to treat elevated heart rate?",
"src": "Patient: Hello doctor. i am 23 years old. when i smoke cigaret. my heart begins to beat so fast. my legs begins to shake. and my eye start tuning me. and a serious headache. the last time i check my weight it was 100kg. i have stop smoke for over one week. and i stil exepreince this symthoms Doctor: Hi,It can be the result of nicotine. Try to avoid coffee and stress, do not smoke and it will disappear gradually.Take care"
},
{
"id": 45232,
"tgt": "Need advice on Semen Analysis report",
"src": "Patient: My husband has done semen analysis . Reports says Colour - Greyish Opaque; Volume - 2ml sperm; Reaction - Alkaline; Fructose Test - Positive; Abnormal Forms 6%; Immature Forms - 3%; Agglutination - Nil; Clumping - Present; Pus Cells - 10-12; RBC S is 6-10 Total Sperm Count - 55Million/ml; Active Motile - 15%; Sluggish Motile - 35%; Non Motile - 50%; His liquefaction time is more than 2 hours. Absentce is 4 Days. Please advice his report. Also if he can help me conceive? Waiting for reply. Doctor: Hello and welcome to healthcare forum. In the report of semen analysis you mentioned liquefaction time of 2 hrs and actively motile sperm % both are abnormal. Your husband requires consultition with urologist but before that 2 more semen analysis report would be advisable. Best of luck."
},
{
"id": 125633,
"tgt": "Are the symptoms of arsenic poisoning same as spinal meningitis?",
"src": "Patient: I m curiou , can arsenic poison be mistaken for spinal meningitis? And a 7 year old is it safe to give them a full cup ( little plastic ones comes with benadryl ) mixed with benadryl , delsum ,musinex and triminic safe? I think my family s inbdangee plz I can t afford to pat pkz I need help 423-790-9267 Doctor: Hello, Arsenic poisoning is very rare and using plastic cup does not cause arsenic poisoning. Hope I have answered your query. Let me know if I can assist you further. Take care Regards, Dr Shinas Hussain, General & Family Physician"
},
{
"id": 30392,
"tgt": "Why am I so prone to infections in my neck, ear and knee?",
"src": "Patient: ikeep taking infetions in my body every 3 months my body crashes itake months to recover doctor keep puting me on antibiotics infection seems to be in my neck area and knees have ear infection at the moment reduced hearig in right ear pain aways on right side of body Doctor: Infection is invasion on body by disease causing agent. Your proneness to infection depends upon your body defense system (immune system) which fight against any such invading agent such as bacteria, virus and fungus. There are a number of factors which jeopardize your immune system resulting in increased susceptibility to infection. Those factors include nutritional deficiency, diabetes, HIV infection, compromised skin integrity such as after surgery or trauma, excessive sweating, compromised blood circulation, congenital immunoglobulin deficiency and chronic diseases etc. Look at these aspects. You might find the cause of repeated infection in yourself. Advise 1.\u00a0\u00a0\u00a0\u00a0\u00a0Investigations \u2013 CBC, blood sugar (fasting & postprandial), HIV screening2.\u00a0\u00a0\u00a0\u00a0\u00a0Try to keep yourself clean 3.\u00a0\u00a0\u00a0\u00a0\u00a0Adopt proper hygienic practices such as bathing and cleaning with good quality soap4.\u00a0\u00a0\u00a0\u00a0\u00a0Wash your hand before taking food and after coming from toilet5.\u00a0\u00a0\u00a0\u00a0\u00a0Take high protein diet such as eggs, meat, cheese, vegetable and fruits6.\u00a0\u00a0\u00a0\u00a0\u00a0Take proper antibiotic after culture & sensitivity report7.\u00a0\u00a0\u00a0\u00a0\u00a0Do not use any antibiotic indiscriminately8.\u00a0\u00a0\u00a0\u00a0\u00a0Start workout in gymGood luck!"
},
{
"id": 226210,
"tgt": "Taken Unwanted 72 after unprotected sex. Had withdrawal bleeding. When will next period happen?",
"src": "Patient: Hi sir/madam, We had intercourse on 11th feb, 2013. and she had taken unwanted - 72 within 4 hours of intercourse. later get withdrawal bleeding on 17th feb, 2013 for 4 days just like normal period. her periods are irregular in general. now when she can get next period. we dont want baby now. is there any chance of conceiving after taking pill within time and also got withdrawal bleeding??? Doctor: HI, thanks for using healthcare magic The risk can vary according to where the woman is in her menstrual cycle at the time of intercourse. If she was close to the time when she would have ovulated or had already ovulated then it does not work as well. Ovulation generally occurs 10 to 17 days since the onset of the last period. It varies from woman to woman. In general , the effectiveness is high if taken within 12 hrs which was done. The effective is approximately 75% which means that it reduces the number of women who would have conceived by 75 %. If the periods are delayed you would have to do a pregnancy test to determine if comception took place. If this is not the case, then the two of you may want to consider using regular contraception such as oral contraceptive pill, injection, condoms, the patch. There are different formulations that you can consider. I hope this helps"
},
{
"id": 27296,
"tgt": "Why did I have sharp pains in my heart and chest?",
"src": "Patient: Hi Dr. and thank you. I had some sharp pains centered in my chest and on sides of heart earlier today. They lasted a few seconds, but seemed to come and go over a 3-5 minute period I was wondering if it was angina or it was something worse, or not even to worry about? Doctor: HI, thanks for using healthcare magicAngina pain tends to occur during activity when the heart is working more, if the pain occurred at that time and stopped during rest , then angina can be a possibility.If you have a history of heart disease, or risk factors such as high blood pressure, diabetes, smoking, obesity, high cholesterol or family history of heart disease, then you should be assessed.Chest pain can come from any structure in the chest from the skin inwards (Eg shingles, nerve related)- muscle, bone, lungs, esophagusI hope this helps"
},
{
"id": 60204,
"tgt": "What is the remedy for severe spasms below the sternum in one post gall bladder removal and subsequent choledocojejunostomy ?",
"src": "Patient: I m suffering from severe epigastric/biliary spasms. I have a lengthy history as a result of a complicated gallbladder removal and subsequent choledocojejunostomy. Since then I get these gallbladder - like attacks just under my sternum, radiating to my right shoulder and arm . Last week I experienced the worst one I ve ever had. Normally I take buscopan as soon as they start, so they never get that severe, with the exception of a few times over the last 14 years. My CT Scan , and XRays are normal, but my LFT s are elevated. I m having another spasm now, but the buscopan is not working, I ve also taken my prescribed pain medication, but no relief. What can I do to stop the spasm?? Please help me Doctor: Hi...you should first get yourself tested for Hepatitis B/C serology given the chronicity and elevated LFTs ..you can try dicyclomine tablets instead of buscopan combined with an anti acid like omeprazole...this woul give relief to the lower esophageal sphincter and biliary stump as well.try modifying your diet with more carbs more protein n less fat"
},
{
"id": 109217,
"tgt": "What causes painful urination, fever, chills and pain in lower back?",
"src": "Patient: My husband and i are in a monogamous relationship and have been for years, so we are not concerned with an std but a few weeks ago he had a small fever and chills with pain in the lower right side of his back that radiated to his groin area. Now he has painful urination. What could be the cause? Doctor: Hi,From history it seems that he might be having chronic urinary tract infection giving rise all symptoms typical of UTI.Go for routine urine check up.After report your doctor will go for specific antibiotic medicine course.Meanwhile he can take antispasmodic medicine as needed.Take plenty of water.Ok and take care."
},
{
"id": 72994,
"tgt": "What is the difference between IPF and hypersensitivity pneumonitis",
"src": "Patient: Hello...My mom is suffering from Hypersensitivity pneumonitis, she was diagnosed with this disease 5 years back and got complete recovery however recently she started getting same symptoms and the CT is more severe than before...her pulmonologist has put her on prednisone and asked us to wait for 2-3 months to see how much she recovers. He hasn't asked for any other test except WBC count check.I am very much worried and wants to know the severity this time, Since her lower lungs show some honey combing (slight) + some amount of intralobular nodules and septal thickening, i am more worried. Though after studying many cases i have found that such CT images are present in HP patient. I just want to know what is the difference between Idiopathic pulmonary fibrosis and hypersensitivity pneumonitis. What kind of pathological tests determines the difference? Doctor: thank u writing me, HP is other variety of ILDs and IPF is other. both are progressive disease, IPF is has no cause HP may has cause , IPF prognosis 3-5 yrs survival but HP has longer survival, CT chest is very good investigation for diagnose them but LUNG BIOPSY is main stay of diagnosing actual disease and can make difference"
},
{
"id": 209114,
"tgt": "Suggest treatment for mind disorder",
"src": "Patient: my mom is patient of mind disorder..she wont have sleep in her eyes till she take sleeping pills,and its just uncontrollabe whn she get hyper.she got anger very quickly..she is imagining thigs and quarreled on the things which she imagine...she beat her children on nonsense things...what to do please suggest . Doctor: HiThanks for using healthcare magicI think, she has psychosis. In that case, she needs antipsychotic. These drugs are not available over the counter. You need written prescription for them. Better to consult a psychiatrist for proper diagnosis and management. With medicine, her anger and odd behavior would decrease. Rest, depends upon the severity of illness. In case, you need further help, you can ask.Thanks"
},
{
"id": 93908,
"tgt": "Had mild discomfort in lower right abdomen, experiencing minor pinching sensation. What might be the cause?",
"src": "Patient: I have had mild discomfort in my lower right abdomen and am experiencing a minor pinching sensation in the same area. The discomfort has been along the majority of the lower right quadrant. The pinching sensation occurs below and to the right of the naval. The pinching is very mild. I am curious what might be the cause of this. Doctor: Hi and welcome to HCM. There are various possible causes but common are diverticulitis, IBD or pelvic organs disorder if you are female. But it can be caused by any lesion in abdominal wall like muscle pulling, abscess or hernia. YOu should do ultrasounds to rule this out. Also try to find is pain triggered by certain food and then avoid it. If nothing helps then endoscopy should be performed. Wish you good health."
},
{
"id": 174787,
"tgt": "What does a lump behind the ear indicate?",
"src": "Patient: Hi, my three year old daughter had a pea sized softish lump behind her ear a while ago and I have recently noticed a similar lump on the back of her neck this weekend. She also has little nose bleeds from time to time too...I have a apt at docs in two weeks but am concerned...should I seek earlier apt?! Many thanks Doctor: This can be lymph node enlargement associated with ear and nose infection. Does the child have recurrent rhinitis? It may lead to epistaxis.If epistaxis is significant then you should visit a pediatrician earlier."
},
{
"id": 33251,
"tgt": "Suggest treatment for lesion in the lower back area",
"src": "Patient: where my ball sack meets the butt (left side) there seems to be a lesion which is weeping... noticed it after shaving in the area. at first it was itching now its sore and weeping... have been using canesten with no improvement? Any idea what it could be? Doctor: Hi & Welcome to HCM.I can understand your problem.The features described in your query are suggestive of a fungal infection of the skin.Use creams containing antifungal drugs like Terbinafine or Clotrimazole on the affected area twice daily. Keep the area dry and cleanUse loose fitting cotton undergarmentsHope to have answered your query."
},
{
"id": 50564,
"tgt": "Ultrasound done on kidney. Suggestions?",
"src": "Patient: Please assess my ultrasound report and give your valuable suggestions & remedial action Right Kidney : Measures 10X4.8 cms. Normal in size, shape and position. corticomedullary differentiation preserved. Pelvicalyceal system not dilated. No evidence of any calculus or mass lesion. Right ureter not dilated. Left kidney: Measures 10.1X4.3 cms. Normal in size, shape and position. Corticomedullary differentiation preserved. Pelvicalyceal system not dilated. Left kidney inter-pole shows 1.2 X1.0cm cystic lesion with echogenic focus within it? Calyceal diverticulum with tiny calculus? Cortical cyst with calcification. Left ureter not diated. Urinary bladder : Normal in size, shape & distension. No evidence of any calculus or mass lesion. Prostate : Measures 3.2 X2.6X2.6 cms corresponding to 11.6 cm3 Normal in size shape & echogenicity. For clinical correlation. Please advise. Thanks Doctor: Good Day !Summary: 1. Right Kidney - Normal2. Left Kidney - a 1.2 cm renal cyst with calcification - I would suggest to get a CT scan with an IV contrast if your serum creatinine is normal to further investigate the nature of the cyst. Majority of the time it is just a simple cysts with calcification but a CT scan will be a better imaging modality to classify and characterize the cyst.3. Prostate - NormalBest of luck and Thank you.Sincerely yours, Manuel C. See IV, MD DPBU"
},
{
"id": 104893,
"tgt": "Experiencing cough and shortness of breath. Why am I feeling this?",
"src": "Patient: Hello, my name is cindy I am a 21 year old women, and well I have a question that ii started to nvolves in my health. Today at 10pm I was feeling short out of breath and also experienced coughing and weezing. At least that s what my partner told me, because I asked him to hear my breathing . My godparents told me that when I was born the put me on some kind of machine for me to ba able to . Though it has been over 20 years and I have never experienced this before in my life, also my father smokes and has smoked around me since I was 5 years old, and well I was wondering what I have. Also when I exercise I feel out of breath and it s hard to breath. Thank you Doctor: you may be suffering from allergic bronchitis in which your respiratory tube from throat to lungs is irritated due to chronic exposure to smoke.it may be environmental or change of place also. if your hemoglobin is lesser also you get shortness of breath. consult a GP"
},
{
"id": 77026,
"tgt": "Can cough suppressant be taken to cure bronchitits?",
"src": "Patient: I have had bronchitis for 5 days. Yesterday my cough was completely gone while the next morning I woke up with a horrible wheeze and had a strong cough all day even after taking gallons of water and only eating soup. Could I take a cough suppressant. Benzonatate is very effective for me. Doctor: Hi thanks for contacting HCM...Noted you have diagnosed with bronchitis ..It might be viral or allergic...If cough is troublesome and dry then dextromethorphan like cough suppressant can be taken....For wet cough ambroxol like mucolytic drug needed....If allergy is due to allergy take antihistaminic like benadryl...If nasal blockage present then nasal spray can be taken of oxymetazoline to relieve congestion....If you are living in cold climate humidifier can be used....Take care.Dr.Parth"
},
{
"id": 74975,
"tgt": "Can stenting cause weak immune system to contract TB?",
"src": "Patient: I am a psychology trainee and was required to complete a PPD skin test to start working at my neuropsychology practicum site. I tested positive and I am worried that I will not be able to work at my practicum site. I have had an active respiratory infection, but the symptoms do not seem to be typical of TB from what I have read: (a) runny nose; (b) sneezing; (c) fatigue; (d) headache and other cold like symptoms. I recently had a stent at the hospital for occipital neuralgia and received 1,000 mg of IV steroids and I am worried that my immune system is compromised. I also have a spinal cord injury and CRPS II, so my body is taxed by pain and pharmacotherapies. Should I be worried? Doctor: Hello thanks for posting here. Stenting itself wont cause immune deficit, but high dose steroids can decrease immunity and you may contract infections. PPD test is positive so there is a suspicion of TB and not a confirmatory diagnosis. It can come false positive also in many cases. So you have to get evaluated. If you have no symptoms like cough, fever, weight loss the probability of TB is less likely. Since you have got symptoms of actice respiratory infection, you must get yourselves checked up. Anyway please do a chest x ray and give your sputum (if you are producing any) for testing. Wishing you good healthRegards."
},
{
"id": 58420,
"tgt": "Suffering from fatty liver, highs triglycerides. Taking Rosav and Liv tablets. Suggest diet?",
"src": "Patient: I am sufferring from fatty liver,highs triglycerides. .tell me d rite healthy diet so I can cure these things in short period of time..my s triglyceride level is 288..even i am taking rosave f 10 after dinner..an\u010f taking liv 52 tablet....I m taking green tea for atleat 4 cups a day..and drinking water around 12 to 14 glasses Doctor: Hi DearWelcome to the platform of Health Care Magic. Negative Balance to be maintained.Output should be more than input.Only then you will be able to reduce your fatty liver and triglyceride. MEANSTry to reduce the oral intake by avoiding extra calories (avoid rich calorie dense diet like fatty meals, avoid sweets etc, And fruits, vegetable intake to be increased)Try to increase output like (increasing physical activity by any means).In this way extra deposited fat in the liver will start depleting along with extra fat circulating fat in the blood.Liv 52 has no proven role, almost work as placebo.Rosave F 10 helps in reducing the blood triglyceride. Lipophage capsule daily once after heavy meals reduces the absorption of fat from the intestine.Vogliphage helps in reducing the absorption of carbohydrates from intestines.Hope the query is answered.Thanks"
},
{
"id": 172181,
"tgt": "Is it normal for a kid to shake head while sleeping?",
"src": "Patient: Is it normal for an infant to shake his head when he feel sleepy? I had a son and he is already 6 months now, since when he was 3 month old, I noticed that he usually shake his head from left to right when he feel sleepy. He still doing that until now. I was worried about it. Doctor: Hi dear, I had gone through your question and understand your concern. You should be worried, but not too much. Babies can do these kinds of movements to relax their bodies before sleepingHope I answered your question.If you have any further questions please don't hesitate to contact me, I am always happy to help"
},
{
"id": 15417,
"tgt": "Child with bruise coloured spot on feet. Applied vinegar water. Fungal infection?",
"src": "Patient: My 6yr old son took off his socks after playing in back yard and there are brown spots all over the bottom of his feet. It almost looks like he got some dirt in his sock or something but the sock is clean. I tried soaking his feet in vinegar water and then vigorously scrubbing the bottom of his feet but no change. Coloration is similar to a bruise but the random occurrences on both feet make it look like fungal growth... (gross). He showers regularly, puts on fresh clothes daily. Any idea what this could be? Doctor: Hello,Welcome and thanks for posting your query to HCM.After reading your query it appears that your son may have either bruises itself or there is some allergic phenomenon to to anything in contact during playing. For the betterment I request you to not wash with vinegar water as can irritate the skin more and also not to rub the area as can lead to further aggravation.You can apply calamine lotion or fluticasone cream over the affected areas and give oral antihistamine if there is itching. Most of the times these kind of rashes disappears itself in a period of 48-72 hours but if these are increasing in number or are deepening in color then you can immediately meet a dermatologist or a pediatrician.With good health wishes.Dr. Sanjay Kumar Kanodia"
},
{
"id": 83543,
"tgt": "Can dosage of Wysolone be reduced as having side effects like osteoporosis and gallbladder stone?",
"src": "Patient: My mother had been operated for pitutory arenoma in1998,after 6months as per Doctor s advice ray was taken,2008 she had Gall Bladder stone as side effects of Wysolone5,and she has become a patient of osteoporosis,due to long use of Wysolone5.is there any way to reduce intake of Wysolone5? Doctor: Hello,It is not advisable to abruptly discontinue or reduce the dose of wysolone. So discuss with her treating physician if the dose can be reduced and if so t needs to be done gradually.Hope this answers your queryPlease address further questions hereRegardsDr.Lekshmi Rita Venugopal"
},
{
"id": 159122,
"tgt": "Suffering from sun burn, pain. Will it lead to skin cancer? Treament, prevention",
"src": "Patient: My girlfriends feels burning and pain on her neck , upper back, and arms after being exposed to the sun, why is she in pain? Is there a symptom that she may be having? She has told me that since being young she had been told to not be out in the sun for an extended amount of time because it causes her pain. Why is that? She is afraid that it will lead to skin cancer , will it lead to skin cancer? Does she have a symptom? Are there any cures, treatments and/or prevention s that she may be able to take? Thank you Doctors for your help. Doctor: Hello, thanks for using HCM. A burn is a damage of the skin by an aggressive agent which could be fire, hot water, steam, chemicals like acid, electricity, sun etc. Sun produces a characateristic burn which is term first degree burn which affects just the first layer of the skin (epidermis). First degree burn is painful due to presence of free nerve fibres in the epidermis and as such sun burns can be severely painful. Long term exposure to sunlight has been implicated to be a cause of most of the common skin cancers, with sun burn increasing the risk of skin cancers even more. A skin cancer like basal cell carcinoma particularly occurs on bear body parts like the neck, face, hands etc and thus strongly supporting the role of sun light on skin caners. Common skin cancers include squamous cell carcinoma (scc), basal cell carcinoma (bcc) and a malignant melanoma (mm). Each cancer type presents with its own characteristic type; bcc present with translucent or flesh colored scar tissue especially in sun exposed areas, scc presents as a red crusted fast growing lesion and mm as a growing lesion with irregular borders, and variation in colour within the lesion. When skin cancers are diagnosed early enough, complete cure is possible unlike when t is diagnosed later along its evolution. What she needs is to avoid frequent sun exposures, once in a while check the skin for any abnormal lesions ec. Best regards and hope this helps"
},
{
"id": 209625,
"tgt": "Suggest prescription for panic disorder",
"src": "Patient: Hi, I have panic disorder and i have been taking anti depressants since two years, Earlier i was taking flupentixol 0.5 mg and melitracen 10 mg at morning and nexito 10 mg at night. Recently, the doctor changed the medicine to Paxidep CR 25 mg at night and Nexito 5 mg at night. I am worried about taking two anti depressants at the same time. Can you advice on the prescription? Doctor: Hello,Thanks for choosing health care magic for posting your query.I have gone through your question in detail and I can understand what you are going through.Its ok to take two antidepressants at the same time. If only you have excess sedation then it is a matter of worry. Hope I am able to answer your concerns.If you have any further query, I would be glad to help you.In future if you wish to contact me directly, you can use the below mentioned link:bit.ly/dr-srikanth-reddy\u00a0\u00a0\u00a0\u00a0\u00a0\u00a0\u00a0\u00a0\u00a0\u00a0\u00a0\u00a0\u00a0\u00a0\u00a0\u00a0\u00a0\u00a0\u00a0\u00a0\u00a0\u00a0\u00a0\u00a0\u00a0\u00a0\u00a0\u00a0\u00a0\u00a0\u00a0\u00a0\u00a0\u00a0\u00a0\u00a0\u00a0\u00a0\u00a0\u00a0"
},
{
"id": 133707,
"tgt": "What causes chronic pelvic pain with tender spine?",
"src": "Patient: I have had chronic pelvic pain for 28 years I am a 45 year old female, I have Fibro, Endometriosis, Adhesions, abdominal scar tissue, Interstitial Cystitis, bradycardia, and have recently noticed the more weight lose(approx 120 lbs over a year, not intentional, but no complaints about it either) my back, spine to be specific is very tender to the touch from mid back slightly above belly button level clear down to my cocyxx. Any ideas, I d like to have my life back, was declared disabled in 2002. Doctor: hi,thank you for providing the brief history of you.A thorough neuromuscular assessment and musculoskeletal assessment is advised.As you have a certain conditions diagnosed and also certain physical symptoms which are restricting your functional ability I will recommend to undergo physical therapy. As with physical therapy the pain and aches can be controlled with non invasive electrotherapy. Post which exercises will be implemented where the muscles will become stronger. It will be a slow and gradual process to undergo. As the pelvic area is painful and spine is tender, physical therapy like - Therapeutic ultrasound therapy and TENS therapy will help you with decrease in the pain. Also, exercises are the key for long term recovery.I deal with similar cases and many more, but with the combination of drug therapy and physical therapy the results are good.RegardsJay Indravadan Patel"
},
{
"id": 191004,
"tgt": "Whether Abscessed tooth be causing muscle pain and cramping, please suggest",
"src": "Patient: Could an abscessed tooth be causing muscle pain and cramping in other areas of the body than the mouth? Doctor: Hi, An abscess can result in muscle pain only in the Head and Neck region. An abscessed tooth can also give rise to fever. A dentist should be consulted and antibiotics and treatment for the tooth should be obtained."
},
{
"id": 46425,
"tgt": "Suggest treatment for kidney cyst",
"src": "Patient: I went to my doctor about a month ago had alot of back pain thought I had a UTI. The doctor ran some tests and later called to say I had blood in my urine and that I did not have a UTI. Sent me for an ultrasound. Found a cyst on my kidney. Does not know where blood is coming from so sending me to a urolgist. My appointment is now in Oct. The pain in the back is getting worse and I have alot of discomfort in my belly. Should I be concerned and call my doctor back or just wait till Oct Doctor: Hello and welcome to HCM. As an Urologist,i can understand your anxiety.After reading your symptoms,you should consult an Urologist urgently. Blood in urine isn't normal and details of your cyst isn't clear on the scan. You should write your age. Do the following tests immediately :1. urine culture,if not done.2. blood routine,RBS,creatinine,LFT,and platelets.3. CT-IVU.According to these reports,expert treatment can be advised.Dr.Matthew J. Mangat."
},
{
"id": 100061,
"tgt": "What causes allergic skin rashes after having peanuts and chocolate?",
"src": "Patient: Well After eating chocolate with peanuts I usually get rashes but then today after not having chocolate with peanuts for days I started to have rashes for no reason. I drank a iced coffee today and I started to get very itchy. The rashes are around my neck and chest and sometimes around hands and other places. I have no clue what's going on. Do you think its serious and do you know why this is happening? Doctor: HelloIt was just co incidence , that you develop rashes after you ate peanuts or chocolates or drank coffee ,because as you mentioned that this time , these rashes developed even without these items , so such type of rashes may be due to these reasons, these include>1 Seasonal allergic reactions , e.g. sudden increase or decrease in temperature or due to allergy to any allergen like , food , mold, pollen , pests , dust , smoking ( active or passive ) .For exact diagnosis , consult an Immunologist and get in allergy tests.2 Worm infestation is the another possibility of such symptoms . Diagnosis can be confirmed by stool examination as well blood examination.3 Tropical eosinophilia , some times this may be the reason of such rashes .Diagnosis can be confirmed by blood examination for eosinophils count.However there are so many other reasons of such symptoms like wheat allergy , lactose intolerance etc .This is not serious but treatment is important so get in done allergy test.Consultation with an Immunologist is important.Good luck."
},
{
"id": 98240,
"tgt": "Treatment of fibriadenoma beside operation",
"src": "Patient: Hi, I am 13 yrs old, 10 days back i find fibroadenoma in my left breast with 7.6 cm size. and doctors advise me to go for a surgery . Is there any alternative methods with out surgery. If Cryoablation preferred option for treatment of breast fibroadenoma, Please suggest the hospitals in India Doctor: Hi,welcome to health care magic forum,yes fibroadenoma is treatable by Homeopathic mode of treatment, there will be no need of surgery, even if you will go for surgery there will be chances of re occurrence, so you should choose Homeopathy for this problem as it will dissolve your fibroadenoma and there will be no chances of re occurrence.Many females are benefited by Homeopathic treatment for fibroadenoma, and the most beautiful thing is that no side effects no knifes and no re occurrence"
},
{
"id": 93571,
"tgt": "Abdominal pain below ribcage, nausea, pain, gas, vomiting. Why is this happening?",
"src": "Patient: Hello, I have had pain in the center of stomach - right below sternum/rib cage. Ate 1/2 of a sandwich and an hour later got very nauseated, pain and very bad gas which all lasted 3 days. This previously happened a month ago also with me - the same symptoms but I threw up 3 times in one day last time. What do you think it is. It's been 7 days now and still have the pain under rib cage. Thank You. Doctor: Hi, Thanks for posting your query.Do you have acidic belching/ retrosternal burning pain/ difficulty in swallowing?Do you have constipation/ watery diarrhea and spasmodic abdominal pain?With the available described details, there appears to be possibility of gastritis.Presence of pain in defined region suggests the diagnosis.You should consult with internal medicine specialist/ gastroenterologist and should go for thorough check up. You should go for complete blood count, serum amylase, liver function test, renal function test, ultrasound imaging and endoscopy for better clarification.You should also go for chest x ray.You should also go for urea breathe test for H. Pylori. Treatment depends on exact diagnosis.Meanwhile, you should take proton pump inhibitors along with prokientics and antispasmodics.Take care,Dr. Mayank Bhargava"
},
{
"id": 129505,
"tgt": "What causes bruising on the top of the hips and tenderness?",
"src": "Patient: I feel like the top of my hips are bruised. Generally it only hurts after I have been driving about 30 minutes, but today it continues to feel tender after being up and moving around. I have a desk job, but am up and about often - I generally walk about 2 miles on a treadmill 3 times week. Doctor: Hi,Get xray of both hips in AP view and send me the report.Avoid any type of exercise till then.Have tab. aceclofenac+thiocolchicoside by prescription of family doctor.Thanks."
},
{
"id": 215760,
"tgt": "Suggest treatment for pain in collarbone",
"src": "Patient: Good afternoon, This morning at 5:00 a.m., I awoke with pain in my left collarbone, also the left side of my head was aching. I got up and started my day. The pain eventually in my collarbone went away. Perhaps I was sleeping with my neck bent, idk. But still having a headache, I laid back down at 7:00 and slept until 9:45. I am 56, female, only medical issues I have are thyroid, which I take 112 mg of Synthroid daily. I am 5 3 and weigh 160. Yes, I am overweight, however I am active. Normally, I never sit to just sit, but actual exercise is optional. Which I know is bad, but to be honest I have no energy, especially today. Thank you for your time and assistance. Jill Doctor: Hi, It may be due to minor muscle pulls or contusion that have developed due to abnormal positioning during sleep. Nothing much to worry and you can take simple analgesics like acetaminophen or Diclofenac for pain relief.Generally the symptoms will settle by itself in couple of days. Hope I have answered your query. Let me know if I can assist you further."
},
{
"id": 11420,
"tgt": "How can hair fall, dry scalp and dandruff be treated?",
"src": "Patient: Hi Doctor, I have sever hair fall complaint. Dry scalp and drandruff. My hair becomes so dry and splitens are more. Not at all healthy. Even if i touch the hair it comes with hand. i want to stop the hair fall and grow my hair. My hair should be healthy and dense. Please advice. Thank you. Doctor: Hello dear,Taking into consideration the case history as provided in your post, the complaints of hair fall is related to dry scalp and dandruff.So, the problem of dandruff needs to be addressed before initiation of treatment for hair fall.Dandruff is basically Seborrheic dermatitis & can be treated with shampoos or lotion containing salicylic acid, coal tar, ketoconazole, or selenium.The causes of hair loss/thinning of hair are:1. Environmental factors2. Stress3. Deficiency of certain minerals, vitamins & amino acids in diet. e.g. zinc, Vit B complex, lysine.4. Any incidence of surgery, shock, etc.Management includes:1. Treating the underlying cause2. Avoidance of stress3. Dietary modifications4. Minoxidil solution to be applied locally to stimulate the hair follicles5. Multivitamin & mineral supplementationSo, it will be better to consult a Dermatologist to find out the underlying cause & start treatment accordingly.There is no need to worry. Your hairs will grow back again.Wishing you a Good HealthTake care."
},
{
"id": 53188,
"tgt": "Suggest treatment for high level of SGPT as Livopal didn't work",
"src": "Patient: Hi myself mithun 34years old(m). My SGPT last check up two months back is lavel is 59 from 88 was on 11months back.My doctor prescribed me LIVOPAL DS twice daily for 100days at the first check up and also prescribed same on 2nd check up. I am not feeling fit even i avoid the fatty foods most of the time.pls suggest me what is going on and what should i do is there anything worry.ThanksMithunage-34yearsheight-5'7''weight-73.5kg Doctor: Hi.Thanks for posting query at HCM.Usually ALT or AST values higher than \"two times the upper normal limit\", is considered abnormal ( in some countries, ALT or AST values of more than 100 are considered abnormal). Value of AST or ALT greater than 85 or above maybe investigated further.Alcohol ingestion and obesity are common causes of fatty liver disease.advice :- Rule OUT Hepatitis B and C- abstinence from \"Alcohol\" - LOW fat diet should be followed, AVOID junk food and beverages- decreased oil consumption (oily food)- NO red meat- green vegetables should be ingested daily- use lemon juice (lemonade) once in a day- reduce weight if overweight/obese-\"recheck liver enzymes after 6 to 8 weeks\" and/or ultrasound.any further questions are welcomed.hope to answer your concern.wish you good health.regards,Dr Tayyab Malik"
},
{
"id": 193718,
"tgt": "What causes difficulty in retracting the penis foreskin?",
"src": "Patient: i am senior citizen and few months back suddenly i noticed that my fore skin is not rerracting later on i found that i am diebetic i controled my diebetc and i found that ido not itche do i required circumcison iam covered by united india insurance please help me Doctor: Hello, If you are able to retract penis foreskin back over glans than it is called phimosis.Phimosis is due to chronic inflammation and so it is irreversible condition. In mild case topical steroid cream might be helpful. But ultimate treatment would be day care minor surgery named circumcision. The foreskin is removed in that. Hope I have answered your question. Let me know if I can assist you further. Regards, Dr. Parth Goswami, General & Family Physician"
},
{
"id": 215080,
"tgt": "Should i stop clexane of my patient before operation ?",
"src": "Patient: excuse me im a nurse and i want to know, shoud i stop clexane of my patient befor operation? Doctor: Hi,you know clexane is anti platelet drug.Normally before any surgery this kind of drugs are not given.In some cardiac procedure this is not withheld.It is better to you please ask the in charge physician about this.thanks and bye."
},
{
"id": 130825,
"tgt": "Can height be increased at 27 years of age?",
"src": "Patient: I am 27 years old, my height is 5.6 or 167 cm. and i want to increase my height, after seeing advertisement of step up herbal body growth formula, i would like to buy but before that i want make sure that it is true at this age i can increase my height, pls. advice Doctor: Height growth stop at age of 20 years. There is now herbal drugs increase the height. Only hormonal traetment has role in increase the height. It is treated by endocrinologist. I hope you are well."
},
{
"id": 222537,
"tgt": "Suggest remedies to reduce blood sugar during pregnancy",
"src": "Patient: This is my 8th month (34 weeks, 4 days), when i tested my blood it is found that I am having diabetics of 160mg, doctor advised to take insulin. Is there any other way to reduce the blood sugar without taking the insulin at this stage. Please advice me. Doctor: HI, I understand your concern. Restriction of carbohydrates/ sugar in diet, exercising are routine personal efforts in control of sugar. But taking medications along with it becomes essential for pregnancy diabetes is hazardous to child & mother too. Take the treatment adised by doctor . It will help your baby to grow normally./ avoid on=ver weight & following complications in both as well as it will prevent baby's health problems later in life. thanks."
},
{
"id": 218549,
"tgt": "What are the chances of pregnancy after skipping birth control pills?",
"src": "Patient: Hi i would like to know what the chances of pregnancy is if i missed the last yellow tablet before the placebo tablets and only took it 3 days after before this Ive never missed any other pill and used them correctly. Im using the trigestrel contraceptive pill Doctor: Hello and Welcome to \u2018Ask A Doctor\u2019 service. I have reviewed your query and here is my advice. If the missed pill is in the 3rd week, the incidence of pregnancy is very low. Especially you mentioned that you were regular and correct in the last 2 weeks and please to be sure you have to get additional contraception for extra 7 days. Hope I have answered your query. Let me know if I can assist you further."
},
{
"id": 119795,
"tgt": "Suggest medications for bone TB",
"src": "Patient: having desisese of t.b . our medicines are not sutting us we are taking regular medice the is in the back boneand because of that the two are damaged n besause not suiting of medicine the problem arises for the liver the liver report is coming too bad plese segest the best medicine which is sutiable for and the best doctor too get apponintment Doctor: Hi, As such, there is no best medicine for tb. TB is treated according to guidelines set up by tuberculosis control committee of nation or WHO. Basically, there are six categories of anti-tubercular medication which are given titrated to the pts category and suitability. Usually, rifampcin a first line drug is known to be highly hepatotoxic and could be substituted by rifabutin in case if there is still need to incorporate the medicine in the treatment. Hope I have answered your question. Let me know if I can assist you further. Regards, Dr. Rohan Shanker Tiwari, Orthopedic Surgeon"
},
{
"id": 179590,
"tgt": "What causes stomach ache, vomiting, loose motion and high temperature?",
"src": "Patient: My 14 yr old son got up yesterday morning complaining of stomach ache,after which he vomited and also passed stool which was watery. He also had a very high temp of 103. We have been giving him 1 advanced crocin every 4-5hrs but his fever is persistent, and keeps fluctuating. And he has gone for motions quite a few times and it has been watery. He is not eating much due to loss of appetite. We keep giving him electoral after each motion to curtail weakness. A friend of mine suggested that I give him O2 tablet as his son had gone thru similar sickness. Please advise... Doctor: HiThanks for writing to health care magic.Diarrhea is usually viral and may last for 5 days.Give ors and zinc.Ensure minimum 6 times urine output in a day.O2 is antibiotic which is unnecessary if there is no blood in stoolsWishing your child good healthRegardsDr Arun"
},
{
"id": 211065,
"tgt": "What are the possible treatments for weak memory and sleeplessness?",
"src": "Patient: I have short attention span, I forget things, I can't sleep, I feel like crying for no reason, I often think of suicide, I can't get up in the morning. But what bothers me the most is that I can't read a book anymore, because it doesn't hold my attention, I do too many things at once and can't finish any of them. I'm always late because I make myself late by doing other things which are not related to my priority. I can't finish my work... What's wrong with me? Doctor: .It might be due to severe stress with anxiety and development of depression.It usually occurs due to some unfavorable circumstances,lack of self confidence.Inferiority complex is also one of the cause for it.Increase your self confidence,try to avoid stress.Make your mind busy with some positive activity.Flupenthixole with other combination drug will be helpful.Do regular exercise,yoga and meditation.Otherwise consult psychiatrist for proper medical management."
},
{
"id": 187611,
"tgt": "Cause of pain, redness, and swelling in gum?",
"src": "Patient: Hi, I have had an ache in my gum for several days. It's in the front of my mouth on the bottom jaw. The gum is swollen and red but just around one tooth, and it is actually below the tooth, not directly around the tooth. The tooth does not hurt it's just the gum. It's painful when I eat or smile, and slightly to the touch. There is also a small white spot in the same area that is troubling to me. Any ideas what's going on here? Doctor: Hello, Thank you for consulting with HCM.Actually when there is infection in the tooth which has spread to the apical region of tooth causes an abscess and swelling in the gum region of the tooth. And when this infection becomes chronic in nature this pus tries to come out and causes a pin point white area on the red swelling of gums.This condition will be well diagnosed in an X-ray. so better you should visit your dentist nearby and get a root canal treatment done of the tooth.Hope it will help you"
},
{
"id": 140142,
"tgt": "Suggest treatment for disc protrusion at T9-T10 level with narrowing of neural foramen",
"src": "Patient: hi, I had a MRI done this is the results. at the t9-t10 level there is a small paracentral disc protrusion seen just medial to the left t10 pedicle resulting in MOD. narrowing of the t9-t10 neural foramen. what does this mean? would this be causing me pain in my back? would me falling and landing on my back have caused any of this, since i had no pain before the fall? and what can be done for it? Doctor: Hello, I would explain that these MRI findings are indicative of a slipped intervertebral disk, which is compressing the nerve root. This finding is compatible with the back pain. It could be related to the trauma or to weight lifting. Anyway, I would just recommend avoiding weight lifting, trying to lose some weight if you are overweight. Consulting with a spinal surgeon may be needed if the pain persists. Hope I have answered your query. Let me know if I can assist you further. Take care. All the best. Regards, Dr. Ilir Sharka, Cardiologist"
},
{
"id": 117685,
"tgt": "Can one get cardiac ischemia at the age of 19 ?",
"src": "Patient: Hi, I keep getting pins and needles up my left arm and sometimes, though rarely in my jaw. My left chest wall hurts as well during these instances. I'm a medical student and know that this is a typical sign of cardiac ischemia... I must add that I'm obese ... but I'm only 19. Is it is probably ishemia? Doctor: yes. its definitly psooble that a person can get cardiac ischemia at age of 19. chances are low as compare to old age, but many cases are noted in past that a person facing cardiac ischemia at very early age. you should go urgently to the emergency department. go for ECG. and then take treatment according to your all reports. but you need urgent intervention.thanks for using health care magic."
},
{
"id": 175678,
"tgt": "How to treat diarrhea in a 7 months old child?",
"src": "Patient: Hi my baby is 7 months old. We changed his milk and He had diherria for the past 5 days, the dr asked us to give him free lactouse milk with low dose. This works fine for two days then we return back to full dose of milk and he he a v sever diherria for the last 3 days. Ex today 12 times dipper with bad smell diherriaHe just vomite once an hour ago and it wad really really big I think from the startuce we added to his milkNow he has cold sweat and finally he is asleep Note: he refused to drink the ORSIs cold sweat seriousMany thanks Doctor: Hi...I went through the history points you have provided - I feel that your baby is having cow's milk protein allergy. Usually babies grow out of this sort of allergy by 1 year of age. My suggestions for you - 1. Mother should go off cow's milk protein completely. This means that you should avoid consumption of anything and everything related to cow's milk - like - milk/ curds/ ghee/ butter milk/ chocolates/ biscuits/ ice creams etc. Even while buying commercial food products, you need to see the ingredients and if they contain milk - do not consume them.2. Feed you baby only exclusive breast feeds till 6 months of age and then start rice based feeds. As your baby is more than 6 months now - it is better to start rice based feeds. 3. If this is followed scrupulously - the baby will stop having diarrhoea in another 3-4 days and then start gaining weight too.4. If you feel your feeds alone are inadequate for the baby, then Zerolac is the only option. Keep me posted about the recovery of the baby and follow of the case.You can approach me at the following link. Please find the link below - www.healthcaremagic.com/doctors/dr-sumanth-amperayani/67696Regards \u2013 Dr. Sumanth"
},
{
"id": 7499,
"tgt": "Done skin peel treatment for acne. Now recurring acne. Suggestions?",
"src": "Patient: Hi, I m having oily skin and have lot of acne . 7 months back, I did skin peel treatment for 5 weeks and it was very quite successful. But recently i started getting acne back. I was asked to do shanas vegetable peel by my beautician. So, I did it 3 times but it of no use. I use clean and clear daily wash and a uv spf 50. Please suggest. Age:27 Gender: Female Doctor: hello thanks for posting your query. i can understand your concern. i recommend that you use a salicylic acid face wash twice a day. also its important to use creams to control the acne as maintenance after the peels, or else there is a recurrence of acne. Treatment of acne would be oral medication, creams and daily skin care routine which further helps in acne. Oral medication will be tablet doxycycline 100 mg once a day for 4 weeks. Creams would be application of clindamycin gel twice a day only on the Pimples. Application of benzoyl peroxide 2.5% gel once a day on the Pimples. Application of Retin A cream 0.025% on full face at night for 2 hrs. All the applications will have to be done for 6 weeks. regards"
},
{
"id": 183258,
"tgt": "What causes chronic sinus issues after having root canal treatment?",
"src": "Patient: I am suffering from chronic sinus issues have coincided with having a root canal treatment to my upper left 7 tooth. I've never suffered bad sinus issues like this before and wondering if perhaps I was allergic to some part of the filling or other, I googled 'congestion after dental work' and found many posts of similar sinus issues after root canals on upper molars.I haven't spoken with the dentist as yet as he wasn't in today, but what should I do? Doctor: hello..read thru ur query...and i must say that nothing to worried in this..its just a usual thing that sinus problem starts after undergoing upper tooth root canal..it is because the upper molar tooth roots are in close proximity with the sinus floor...this might have resulted in filling material going excess into the sinus(in some cases) or just a reaction due to new material coming near the sinus...u can continue with the antibiotic medication by dentist and do steam inhalation(with vicks) for a week..and wait..usually it subsides..if after a week still problem persists meet the dentist and take xray and get it treated based on the clinical presentation...be cool and dont panic...hope ur benefitted something from this reply...have a healthy day!!!"
},
{
"id": 76533,
"tgt": "How to get rid of sinus problem?",
"src": "Patient: I'm suffering from sinus problem. my nose always remains blocked.I can't breathe and even don't get proper sleep.It is also affecting my studies.my symptoms are tiredness, bodyache ,pain in my forehead.now i 've also started feeling depressed.sir, please help me. Doctor: Thanks for your question on Healthcare Magic. I can understand your concern. No need to worry or get depressed by these symptoms. By your history and description, possibility of chronic allergic sinusitis is more. In my opinion, you should consult ENT doctor and get done clinical and endoscopic examination of sinuses. You will be improved with following things. 1. Do regular steam inhalation 4-5 times a day. 2. Drink plenty of fluids orally and keep your self hydrated. 3. Avoid dust and pollution. 4. Take intranasal spray containing steroids (fluticasone) twice daily. 5. Take oral combination of antihistamine (levocetrizine or fexofenadine) and anti allergic (montelukast). Don't worry, you will be alright. Consult ENT doctor and discuss all these. Hope I have solved your query. I will be happy to help you further. Wish you good health. Thanks."
},
{
"id": 176540,
"tgt": "Suggest treatment for headache and stomach ache in child",
"src": "Patient: My 6 and 1/2 yr old daughter is complaining of headache and stomach ache everyday for the past week. I see her playing and laughing one moment then the next she tells me her stomach and head hurt. I don t know if I should take her to see the pediatrician. Doctor: Hi dear,This may indicate conditions like UTI.If not febrile,Let him take plenty of oral fluids. Also give simple analgesics like acetaminophen for 3 days.If not getting better by 2days,kindly see your doctor for evaluation.Take careThank you."
},
{
"id": 177835,
"tgt": "What causes pain while passing a motion?",
"src": "Patient: My baby is one year old now.since I gave birth i used S-26HAGOLD milk then I decided to switched her milk to nido junior . after switching I noticed every time she shit over or poopoo shes crying.is that the effect of her milk?what should I do in order my daughter will not suffer? Doctor: Hi dearWelcome to the HCMInjury at the anal verge to be ruled out.it could be due to the hard stool passage.(fissure).Apply anaesthetic jelly before and after passing the stools. Hope the query is answered. Thanks"
},
{
"id": 103959,
"tgt": "Have seasonal allergies. Taking nasal spray, Zyrtec and decongestants. Guide",
"src": "Patient: I have seasonal allergies and take nasal spray, zyrtec, and a preventative inhaler. I have been having problems for the past few weeks. Pressure in ears, having body aches (on and off), feel very tired... Have been taking 12 hr decongestants every 12 hours and still popping in ears etc. I do not have a fever - usually don't have fevers.Not sure if I need to see a specialist or look into allergy testing.. have avoided that due to having to be off med's for 5 days prior to testing.. Even when I have a sinus infection (earlier this winter) no real sinus pressure to speak of. I did have tubes in my ears twice as a child. Doctor: HI, thanks for using healthcare magic It may be best to do the allergy testing to identify what specific allergen is causing your symptoms. If this can be identified and eliminated to some extent then your symptoms would improve and hopefully the amount of medication needed. Allergy testing can be done by a blood test or skin testing. The allergist would discuss the choice with you. I hope this helps"
},
{
"id": 163841,
"tgt": "How can fever, pharyngitis with nausea and vomiting be treated?",
"src": "Patient: My grandson age 6 has flu and pherengitus. He was running fever of 105. I gave a 2 teaspoon dose of Ibuprofin which he immediately threw up. After the nasua was done I gave him a 2nd 2teaspoon dose. His temp is now running 95.7. He is sleeping normally and breathing normally but I am still concerned. Doctor: dear userthese symptoms you mentioned refer to having bacterial infection or viral infection of your grandsonyou need not to worry as the most common cause is viral infection and so we dont need to give him antibioticsonly symptomatic treatment of antipyretics and give hime multiple fluids will helpthen improvement will occur gradually as the viral infection take about four days and symptoms will improveif symptoms persist or get worse we suggest that there is bacterial infection and antibiotic indicate nowthe doctor that examined the baby can give hime antibiotic such as amoxicillin by clinical examinationwe prefer to do blood test and nasopharyngeal swab for confirming diagnosisantibiotic is prescribed according to these testsjust now you need not to worry if you feel gradual improvement of himand give him more fluids hope that helps you"
},
{
"id": 138907,
"tgt": "What causes sensitivity and pain in shoulder?",
"src": "Patient: I have a very sensitive right shoulder, right where the arm connects to the shoulder. I thought maybe it was a bruised bone, as I spent many hours sleeping on it while traveling on plane. However, this is not a bone. The sensitivity as at the surface or just under. Any sort of touch to the affected area results in serious burning pain, enough to make me tear eyed. I have a strong threshold for pain, so I fear that I might have torn a muscle or something of that nature. Doctor: Hi, take some rest, do not check if its paining by putting pressure on it, and take some analgesics like diclofenac 100mg SR tab daily for 3 weeks."
},
{
"id": 183628,
"tgt": "What causes eating and breathing trouble due to dentures?",
"src": "Patient: had to have all upper teeth removed. Had temporary denture made and am waiting for healing to get 3 more implants to snap into new denture without the palate part. I already have one implant but unfortunately while waiting for everything to heal is now falling out.When I put the temporary dentures in - I feel like my breathing is blocked and I have extreme nasal drip down the back of my throat. Why is this happening? I have had the dentures since last November - they still are horrible and I cannot eat. What is causing all this? Is there a problem with my sinus? Doctor: Hello I think your temporary dentures are over extended just get them rechecked with your dentist this a common problem faced by patients when the denture extension is beyond specially at the palatal region extension. So don\u2019t worry get them corrected u will feel better\u2026regards Dr Rakesh Mishra"
},
{
"id": 115467,
"tgt": "Is nose piercing safe for one with ITP and eye shingles?",
"src": "Patient: hi i got itp and i was wanting to get my nose pierced for awhile(my chemo dr said just get my blod work done befor i do it)and i just got over shingles in my eye but the eye dr says its in the eye lid n come back in 2 month.is it safe to get my nose periced now or wait .what kind of infections risk do i have compared to a normal person.ty Doctor: Hello and welcome to HCM,Low platelet counts associated with ITP can lead to excessive bleeding during nose piersing.If the ITP is under control and platelet count has improved, you can get the piercing done.However, the piercing should be safe and carried out clean needle to avoid transmission of blood borne infections.Shingles is not a contraindication because it has not involved the deratome which supplies the nose.Thanks and take careDr Shailja Puri"
},
{
"id": 166023,
"tgt": "Suggest treatment for anemia in children",
"src": "Patient: hi i recentley had my 7 years old sons lymph node checkd on neck and he said do you kbow he has anemia? i didnt but there special doctor i see didnt say what i should do and my son is very tired at times an his eyes hurt when tired? please could you advise many thanks Doctor: Hi.... I understand your concern. Anaemia is a very simple problem, but at the same time, if it is only isolated anaemia the most common reason is Iron deficiency.As a physician has recently checked your son and told that it is only an isolated anaemia and he is also symptomatic - like easy fatigability, I suggest you use iron supplements at 3 mg per kg per day in 1 of 2 divided doses. But this is a prescription drug and this is the reason why I am unable to give you a prescription here. I suggest that you get back to your son's pediatrician regarding this. He will be requiring iron supplements for 3 months consistently.Regards - Dr. Sumanth"
},
{
"id": 160560,
"tgt": "Could the rash on my baby be related to something I ate?",
"src": "Patient: My daughter is almost 3 months old and I am breastfeeding. Yesterday afternoon she developed an itchy rash on her head, neck, chest and back. Several hours later I noticed it had spread to the inside of her arms. The rash resembled hives as the red areas were raised and irregularly shaped. She did not have a fever. Could the rash be an allergic reaction to something that I ate? I have not changed any detergents/soaps. The only new thing was that I ate some strawberries the about 6 hours before her rash developed. Thank you. Doctor: Hi,Sudden onset of raised, red and itchy rash over body indicates an acute urticaria. This can be a manifestation of allergy to some insult (but unlikely to your diet), a viral illness or to an infection somewhere in the body.But, since she do not have fever or other symptoms now, we need not worry. Just apply calamine lotion over the lesions 2 times a day for 3-5 days. If it is not coming down by then, or if she develops fever, kindly take to her to pediatrician, as this will need antihistamines and steroids for few days along with treatment of any infection.Hope I have answered your question. Let me know if I can assist you further. Regards, Dr. Muhammed Aslam T. K., Pediatrician"
},
{
"id": 32780,
"tgt": "Suggest medication for tape worm infection",
"src": "Patient: My name is Aby Abraham aged 25 yrs young boy.. my I am suffering from tapeworm infestation problem..Can you please Prescribe a good medicine for tapeworm infestation.. Is there any complete cure for this tapeworm.. I have been infected this problem since 2007.. Two doses of praziquantel taken and Albandazol medicine into three ..But This time when i checked my stool i found tineasolium(tapeworm eggs). Suggest some good medicine for this cureness... Please respond to these doctors.. I am weighing 68KG And my height is about 168cm. Doctor: Hello This is a very interesting case and you will get complete cure .No doubt praziquantel is the drug of choice but in few case as I have noted it is not effective at all ,but why I don't know , it is a matter of discussion.Any how , the drug of choice is NICLOSAMIDE for your case .This is a safe drug and you may feel only mild degree nausea .Do as :Low residue die , whole day before and complete fasting OVERNIGHT .Now early in the morning 2 tablets of 1 gm each are given in the morningon empty stomach.The tablets should be chewed in order to ensure a through mixing of the drugs with intestinal contents .Now after ONE hour another 1 gm is given .A saline ENEMA after 2 hour of 2nd dose . If you don't like ENEMA , on previous day take laxative tablet .Sit on the sheath of because it will take at least 2-3 hours for complete evacuation of tapeworms as well as its eggs . When tapeworms start passing ,don't BREAK in between .All these things I have tried on many patients , so will get benefit.If you don't understand any thing I will explain in detail."
},
{
"id": 54958,
"tgt": "Having noticed difficulty in breathing after being diagnosed with pancreatitis",
"src": "Patient: hi lm elaine l have pancreatitis l got from ecrp and had gallbladder removal and lately l have difficulty breathing so l told my consulant at the hospital and he blood tests and an ct scan and it was only two weeks and now he wants to see me on eighth of january you dont think any thing showed up Doctor: hi.noted history of pancreatitis and difficulty of breathing. is your difficulty of breathing persistent and causing you distress? if it's episodic, it may just be a radiation of pain from your pancreatitis and will resolve spontaneously. if not, it is best if you consult with a doctor for physical examination and clinical evaluation. diagnostics (such as chest xray) and management will be directed accordingly.hope this helps.good day!!~dr.kaye"
},
{
"id": 213625,
"tgt": "Will there be any effect on my babies due to my mental weakness during 28 week pregnancy and how can I come out of it ?",
"src": "Patient: I m 28 week pregnant with twins i have become emtionally very weak i will get disturbed and pissed off with family matters and i dont get proper sleep in the night..will these affect my babies health and pls help me suggest to overcome this mental weakness Doctor: hello dear patient any tension or any thing which affects your health will surely affect your babies normal growth. stay calm you can try meditation practice also. avoid any such thing because it will lead to ill health and that will lead to disease than finally tablets and medication which will definetly affect your babies so to avoid all these problem stay calm for few more months. take care regards dr"
},
{
"id": 216187,
"tgt": "What causes severe pain in knees?",
"src": "Patient: I have developed severe pain in my knees when getting up or down from a seated position over the last weeks. I have tried physical therapy for patella femoral syndrome and they have only gotten worse. I am being seen by a pain specialist whom is doing facet joint injections on my back. I see an orthopedic doctor for my knees. I have excruciating pain at night when I straighten my legs out and when I get up. I am scheduled for an mri on both knees. I was wondering if the back injections could ve caused this or what else can it be? Doctor: Dear patient you may be having early inflammatory arthritis of both knees. what us your age ? if you are past 50 yrs osteoarthritis may be likely. I would like to advise xray of both knees anteroposterior and lateral and skyline views. Mri will be more sensitive and specific then xrays. Please take tab ultracet twice a day for pain relief. You need to consult orthopaedic surgeon with report."
},
{
"id": 155762,
"tgt": "How long does end stage of uterine cancer last?",
"src": "Patient: Hello, and thank you for taking my query. Currently, my boyfriend's mother is at the end stage of uterine cancer. She has much edema and is in a nursing home. She had refused treatment and we are currently being supportive of her. My question is how long does the end stage last? Doctor: Thanks for your question on HCM.I can understand your problem and situation. But we can not do anything except to decrease her suffering. Since edema feet are starting the prognosis seems worsening only.Usual life expectancy in end stage uterine cancer is less than 6 months but in your case I think it is less than 3-4 months.Try to discuss end of life issues with her."
},
{
"id": 195944,
"tgt": "What causes brown discoloration in semen?",
"src": "Patient: I just recently noticed that my semen was like a tannish brown color and there were dark brown little chunks in it. I don't know what it is, but I was sick recently and I'm still getting over what I'm sure is just a sore throat. I felt some weakness and nausea for one day, but the rest feels mild. I was wondering what this semen problem is and if it's related to my recent sickness. Doctor: Dear userWe understand your concernsI went through your details. Don\u2019t panic. According to Harvard Medical School, most cases of BLOOD IN THE SEMEN are not serious and can be attributed to six causes: inflammation/infection, obstruction, tumors, vascular abnormalities, systemic factors, or trauma/medical procedures. Therefore please consult a doctor in person at the earliest for physical investigation. If you require more of my help in this aspect, please use this URL. http://yyyy.gl/yyyyyy. Make sure that you include every minute details possible. Hope this answers your query. Available for further clarifications.Good luck."
},
{
"id": 33960,
"tgt": "Suggest treatment for fever , loss of appetite and cough",
"src": "Patient: For the last two and a half weeks I have had no energy, extremely diminished appitite, coughing, constantly running a feaver of 99-102. I have the chills and sweat at night. I have been to the dr twice. Had chest ex rays, blood work ect and everything had come back fine. The dr gave me an inhaler and some antibiotics but nothing has worjed. I still feel horrible. Doctor: Thanks for your query.I think you have a chronic infection likely to be tuberculosis. You can get sputum examination for ZN STAINING and culture sensitivity test for mycobacterium done. If all test are negative. You can get tested by high resolution CT Scan to look for lymph nodes. Some times treatment for tuberculosis must be started on high clinical suspicion. Take adequate rest and eat protein rich balanced diet with multivitamins and calcium supplements.Take care"
},
{
"id": 96118,
"tgt": "Best treatment details and advice for PID",
"src": "Patient: Hi Doctor, My name is Remya, 24 years and married. recently I was suffering from abdominal pain and consult doctor and after test they found that I have PID ( pelvic Inflamatry diseas) and gave some medicines. Is that curable diseas? and may I know the best treatment details and advice ? Doctor: Pelvic Inflammatory disease is curable (But that depends upon the etiology). Yes if properly treated with adequate antibiotics it can be completely cured. But there are chances having residual fibrosis."
},
{
"id": 91237,
"tgt": "Could taking dayquil and nyquil cause stomachaches?",
"src": "Patient: I had a cold for a week and took dayquil/nightquil. I stopped Sunday and went out to eat. Since I have stomach aches and have seen some light colored stool with regular colored stool. At times I feel like I'm constipated. Should I go to my primary care?? Doctor: Hello and Welcome to \u2018Ask A Doctor\u2019 service.I have reviewed your query and here is my advice. Your symptoms may be related with gastritis, food poisoning. You should go to your primary care.Hope I have answered your query. Let me know if I can assist you further.Regards,Dr. Klerida Shehu"
},
{
"id": 18321,
"tgt": "Is taking 5mg of Bystolic with BP medications advisable?",
"src": "Patient: Hello, My name is Carol.I have just been put on 5mg of Bystolic....I find this drug strong for my system. I have taken seven tabs so far one a day. My question : I cannot reach my Dr. until the AM would it be OK to cut my dosage down to 1/2 this evening ( I would be taking 2.5 mg which I am used to taking when taking Blood pressure meds.....I do not feel good taking the 5mg....Thank you. Doctor: Hello Welcome to Ask a Doctor service I have reviewed your query and here is my advice.You have mentioned any symptoms so it can be decided that it is the drug causing the symptoms, moreover you can not change, stop or adjust the dose of drug on your own you need to have opinion form doctor or discuss this, let me inform you that what ever the feeling or symptoms you are having may not be due to the drugs but it could be just functional elements playing the role so need to worry about this.Hope I have answered your query, let me know for further assistance."
},
{
"id": 179945,
"tgt": "What causes abdomen pain in a 8 year old?",
"src": "Patient: hi, my eight year old son has stomach ache. he says that he feels like he has been punched in his stomach. he vomited, and his food looks like it hasn t been digested. It happens regularly as soon as he eats ice cream or any kind of fast food. I keep him on soups but once in a while he gets something different that causes this condition. Doctor: Thanks for putting up your query at HealthCareMagic. Your 8 year old son might be having gastritis. If I had a patient like him, I would give him Lanzol Junior 1 tablet to be taken in the morning in empty stomach for one and a half months. I would also ask him to exercise and play regularly as that increases the movement of our bowels which would ease the situation. I would have also asked him to take Isabgol husk regularly at night at fibre rich food improves gut motility and thereby eases digestion. Avoid fast foods and fat rich food. No need to keep him on soups. You can give him normal food containing less spices, oil or chilli.I hope that helps. Feel free to revert back with further queries if any."
},
{
"id": 139094,
"tgt": "Suggest treatment for pain in the upper arm",
"src": "Patient: I am having pain in my upper arm. It feels like its coming from about half way between elbow and shoulder. sometimes quite sharp. Mainly at night it is worse. If I chang position and lay on my back it feels better. However still feel uncomfortable. feel it during day but not as bad. It does radiate somtimes, but seems centered in middle of upper arm. I have not had any recent injuries Doctor: Sir, Such pains usually originate from spine or local muscles. Since your pain also radiates it means that it is of spinal origin. You need screening of your cervical spine, a good examination and MRI are must."
},
{
"id": 168570,
"tgt": "What is the treatment for vomiting in a baby?",
"src": "Patient: Our 10 month old, just woke up from vomiting. We have a call into the triage nurse to check but no call back yet. She has vomited 3-5x now since we got her out of bed and is understandably tired and fussy. What should we be doing/watching for to make sure it s nothing serious? Doctor: thank u for the query.... basically u have to see that the baby is accepting feeds or nor not...... make sure to give him/her burping for 10 to 20 min at least..... if there are no any abnormal body movements in form of uprolling of eyes and and tighting of body its no emergency.... do make sure to not to give any things apart from his /her daily feeding habits and do consult a doctor.......regards"
},
{
"id": 68353,
"tgt": "What is the lump in my upper neck?",
"src": "Patient: Dear Ask The Doctor: Hi ive been to my docotor about lump in my neck ( swelling.) i already knew i had a tyroid in 2001 but it was said to be normal nothing was detected. how ever ive had problems breathing and finding it hard to swallow and been gettin ear ackes. doctor send off for ultrasound scan but i have not yet recived my letter( can take up to six weeks.) but recently i have noticed a small but very hard lump in my upper neck under the jaw close to my ear. should i go back and inform my doctor of this.. its causing me pain and havin bad ear ake. please help!!!! Doctor: Welcome to health care magic. 1.The history suggest the possible cause could be lymph nodal enlargement, next possible cause could be thyroid mass ( keeping previous thyroid problem in mind)2.The recent lump under the jaw close to ear possibly enlarged lymph nodes.3.You don't have to inform the doctor again, as the neck scan has already indicated.4.The scan will include the needful lumps on the neck.5.Ultrasound will detect the nature of lump, its source and extensions. An FNAC can be taken to know the type of cells. Depending on that treatment will be depending on.6.Lymph nodal enlargement will cause due to infection / inflammation localised or systemic, and presence of any malignancy - all these need to be assessed. Good luck.Hope i have answered your query.Any thing to ask do not hesitate. Thank you."
},
{
"id": 176200,
"tgt": "Suggest treatment for loose motions with high temperature",
"src": "Patient: Hello Doctor, My daughter is having loose motion from morning(8-10 times) today. I have been advised to gove Nutrolin-B and electoral water. In the evening she started with high temperature(99.6F),too much foul-smell stool and she started refusing food..... Can i proceed with suggested medicine as i cannot consult a doctor right now? Thank you veru much ......... Doctor: THANKS FOR YOUR QUERY.Symptoms related to any diarrheal illness are often associated with any infection to the gastrointestinal tract, such as high grade fever, loose motion and abdominal pain.Nutrolin B and Electoral are right to be given. Electral contains electrolyte solution and you can give the medicines. She should be well hydrated with electrolytes, along with symptomatic treatment like NSAID (acetaminophen) for fever, anti secretary drug like racecodotril to reduce the frequency of motion and also to prevent dehydration will help her. HOPE THAT HELPS."
},
{
"id": 32611,
"tgt": "Does persisting fever due to gingivostomatitis need medical attention?",
"src": "Patient: Hi, may I answer your health queries right now ? Please type your query here...My 3.5 year old son has gingivostomatitis. He has had a fever for 5 days now ranging from 38\u00b0-40\u00b0. I have heard that the virus can run up to two weeks. Could a fever accompany the full two weeks? or should I be taking him back to the doctor with a fever now at 5 days? I give him about 3-4 doses of (non-aspirin)pain/fever medicine daily when the fever is at its peak.My 3.5 year old son has gingivostomatitis. He has had a fever for 5 days now ranging from 38\u00b0-40\u00b0. I have heard that the virus can run up to two weeks. Could a fever accompany the full two weeks? or should I be taking him back to the doctor with a fever now at 5 days? I give him about 3-4 doses of (non-aspirin)pain/fever medicine daily when the fever is at its peak. Doctor: HI, thanks for using healthcare magicThe fever can persist for as long as the infection is present. If his symptoms are other wise improving but the fever is persisting then you may need to have him reassessed.This would determine if there is any other cause for his fever.I hope this helps"
},
{
"id": 92314,
"tgt": "Should i consult a doctor as i am having pain and swelling in stomach after the surgery?",
"src": "Patient: I am about 4 weeks post op. I am experiencing pain on a level of about a 6 or 7 in my abdomen area. I am already on an antibiotic for a bladder infection and possible infection in my abdomen. I was also diagnosed with having a hematoma last week. The pain has not gone away, my stomach is swollen and tender to the touch. Do I need to call my doctor? Doctor: Hi,Welcome to HCM.You definitely need to get in touch with your doctor as you are experiencing signs of pain and tenderness. Patients suffering from bladder infection may also present with abdominal pain. I suggest you get in touch with your doctor and get the prescription of antibiotics changed as you do not seem to be responding to them. Thanks."
},
{
"id": 163468,
"tgt": "What causes urine odor, fever with lack of appetite?",
"src": "Patient: My 8 1/2 month old baby girl s pee has a very strong odour. It just started last night. At 9pm she was running a temp of 101.3. Gave some advil, and she drank 6ozs of formula. Her temp dropped to normal. Then at 3:30am, temp was 100.2. Gave more advil, and another 7 ozs. Her wet diapers started to smell last night. Today she has no temp, and isn t as fussy, but she isn t eating or drinking alot. Her pee stills smells really strong. What could be causing this? Doctor: Hello,Baby is suffering from fever due to some infection probably and poorly accepting orally. So she is passing concentrated urine which is smelly also baby may have a urine infection, which is causing fever.Hope I have answered your query. Let me know if I can assist you further.Regards, Dr. Sachin Kumar Agarwal"
},
{
"id": 81610,
"tgt": "Suggest medication for bilateral pneumonia",
"src": "Patient: I have just come home from my 6 th hospitalization this year. 4 have been pneumonia related, 2 were from new A-Fib which I developed from my first bout with pneumonia on 25th of Jan. in April, I checked into ER with A-Fib again. 3 days later I had bilateral pneumonia. When I went to ER, I didn t even have tickle in my throat. My question centers around my oxygen. After 3 weeks in April in Chandler Mercy Hospital, they sent me to a therapy Hospital. When I went I was at a flow rate of 15. After 3 weeks I was back down to 2 during the day and 4 in pm. My oxygenation is 96/97 with oxygen and heart rate is 65/75. If I go off the oxygen, I drop to low 80 s and even 70 s with heart rate in 90 s. Doctor am I able to look forward to getting off oxygen? Prior to this year I be not been in hospital since 2001 for appendix and 2004 for pneumonia but no oxygen! Doctor: Thanks for your question on HCM. In my opinion you should consult pulmonologist and get done1. CT THORAX2. Bronchoscopy and BAL ( Broncho Alveolar Lavage ) analysis3. PFT ( Pulmonary Function Test ). Pneumonia can happen in both lungs but it is not common.I think we need to rule out interstitial lung disease ( ILD ) and chronic congestion first. As possibility of these are high.So CT THORAX and PFT is needed to rule out this. If in CT pneumonia is seen than get done Bronchoscopy and BAL analysis for identification of causative organism and start treatment accordingly. So better to consult pulmonologist and discuss all these."
},
{
"id": 206446,
"tgt": "How to overcome depression?",
"src": "Patient: Hi, may I answer. I think im depressed or having a breakdown, been in my house for over a week and starting to feel really anxious about going outside. I have been crying for the past few hours, i feel like i cant breath or think, i dont know what to do Doctor: Hi , thank you for using health care magic.I can understand your concern.First let your near and dear ones about your feelings. This may help you in getting back to normal.Take help of doctor preferably psychiatrist , mental status examination by psychiatrist is needed in your case for proper evaluation of your mind' condition.Once diagnosed your condition will be helped by medicine like antidepressent /antianxiety as well as psychotherapy like cognitive behavioral therapy.Hope this will help you. Regards."
},
{
"id": 33087,
"tgt": "Suggest treatment for fungal nail infection",
"src": "Patient: I have ms and a fungal nail infection on both my big toes am using a solution for over a year now and it has not inproved toes very swollen and painful. Nail is growing not straight and too thick to cut i also have a cut under toe and have been prescribed a fungal cream this is not helping to heal it. I would appreciate your help. Doctor: For fungal nail infection you can take some antibiotic preferably cefadroxyl or doxycycline after an expert opinion along with combiflam or ibugesic-plus for pain, swelling and inflammation. You can take fluconazole 150mg once daily for at-least 5 days or use candid-b powder for local application. Do hot water fomentation over injured site to reduce the pain and inflammation."
},
{
"id": 205661,
"tgt": "Suggest treatment for anxiety and seizure disorder",
"src": "Patient: I be been tampering off the Fentyl patch for a week now. I was taking it 1 every 3 days (75). Due to the expense I decicied to jurt get good F off right away. I was going 75 every 3 days, then it was every 5 days. Recently I stiopped using it at all & that was about 4 days ago. Now I ve got the shakes indie & out,& major anxiety attacks . Can anyone give me any support to things that did so I can trey to get thie over & done with & get back to life! Thaks AJ Doctor: I sugest to see a General practitioner. In my opinion it will help a therapy with antidepressants with low doses.I hope I've been helpful."
},
{
"id": 108724,
"tgt": "Suggest treatment for chronic back pain",
"src": "Patient: I have chronic back pain, S/P slip and fall x 2 occurances. (Pinched nerve, DDD, and second fall: hernation with tear of disk). Multiple facet injections, 13 epidural injections and 2 years ago last procedure of radio frequency. (All have worked for a time frame of 8 months to 1.5 years). Currently have chronic lumbar pain into bilateral gluteus muscles and right side radiates into thigh down to toes (on the worst case senerio, usually stops in mid thigh). Last 5-6 months numbness noted in left lower abdomen started as half dollar area now grapefruit size area. (Feels like lidocaine feeling after teeth filling). Last back procedure was Jan 2013 radio frequency. What could this be? Edema? Work as floor RN. Thanks! Doctor: HiWelcome to healthcaremagicI have gone through your query and understand your concern.You can discuss with your doctor about it. You are having digenerative disc disease. Your abdominal numbness can be related to radio frequency ablation or compression by pressure by degenerative disc. You need to do MRI lumbar spine to see compression. Analgesic such as ibuprofen is helpful in neuralgic pains. Mecobalamine is helpful in recovery of damaged nerves.Hope your query get answered. If you have any clarification then don't hesitate to write to us. I will be happy to help you.Wishing you a good health.Take care."
},
{
"id": 168277,
"tgt": "Can I give ragi to child while recovering from fever?",
"src": "Patient: my son is 3 yr old,we live in denmark,from yesterday afternoon he has very high fever..docor said it is because of ear infection.and suggested primcilin.and paracetamal for fever...fever is coming downnow....want to ask if i can give him solid like ragi porridge.. Doctor: Hi, yes you can give ragi while recovering from fever. Ragi doesn't interfere with any medicine. Take care."
},
{
"id": 192386,
"tgt": "What causes pain from anus to testicles?",
"src": "Patient: Hi,my fiance has a problem with his testicles.Last night he showed me that his one ball looks normal but the other one is bigger.It almost looks swollen.He says it feels like all the veins from the one testicle has moved over.(Is that really possible?). He says its not sore its just irritating. He doesnt know how long its been there as he has never felt this before. Sometimes when he ejaculates,it comes out like small lumps(blockages) and sometimes he gets this pain running from the back to his penis. I am worried as I would like to have children one day.What causes the lumps(blockages)? What causes the occasional shooting pain from the anus to the balls?And why does it feel like the veins or pipes have moved to the other side? I am really worried about cancer!! Please help. Thanks, Linzi.I have attached my email address. YYYY@YYYY Doctor: Hello,I suggest you to visit your doctor as this could be due to cysts in the testis along with the vericocele and hydrocele as well. You need to get ultrasound of your testis to see what really is the problem. All these need to be addressed by a urologist in timely manner.Hope I have answered your question. Let me know if I can assist you further. Regards, Dr. Sameen Bin Naeem, General & Family Physician"
},
{
"id": 180771,
"tgt": "How can a tooth infection in a toddler be treated?",
"src": "Patient: Hello Doctor!My son is just 2.5 years old. He has an infection at the root of his tooth (upper, front right tooth). Dentist has advised Clamp Kid antibiotic to him for three days (1 tablet per day dissolving in water and serve). I'm not sure if it's safe, giving antibiotics to a kid at this age concerns me. What do you recommend. Doctor: Hello,As there is an infection in right upper front tooth which could be due to tooth abscess, infection of overlying gum, cyst around affected tooth and due to food lodgement over there sometimes build pressure and causes pain.As your dentist prescribed with antibiotic course, you should start with proper course of antibiotics as infection will be subsided only after that. You should do warm saline rinses.Once infection will be subsided get child evaluated and a thorough clinical evaluation and investigation like x ray can help in diagnosis and treatment can be done accordingly.Hope I have answered your query. Let me know if I can assist you further.Regards,Dr. Harry Maheshwari"
},
{
"id": 152213,
"tgt": "Treatment for CerebralPalsy",
"src": "Patient: Hello All, I am too much concerned for my Uncle( Dad's Brother) who is suffering from Cerebral Palsy. This is worsening every day presently he finds it very hard to stand by himself.He is 80% disable now. Can you suggest me any hospital in and around Bangalore that could give some possible treatment for that. Doctor: Cerebral palsy, though not curable many of the deformities and disabilities can be treated with planned surgeries and vigorous physiotherapy. looking at your uncle age it seems we can improve his function lot by doing surgery. you can contact us Dr Manohar Babu KV Consultant Deformity Corrective surgeon Apollo Hospitals Bangalore +91 9164072363"
},
{
"id": 89125,
"tgt": "What causes severe pain in abdomen with belching and pain during breathing?",
"src": "Patient: Sure my name is Teresa and after a bout with severe pain in my right upper side,finally went to er it was so bad,I have been dealing with pain for 8 months now it occures usually after eating today I developed belching after eating and it feeling lke something is in my throat,I get to go to consult with a surgen to have it removed,there is a polyup in the gallbladder and it is enlarged and folded over on its self I am in constant pain especially when sitting up and breathing in deep Doctor: Hi ! Good morning. I am Dr Shareef answering your query.If I were your doctor, I would advise you not to eat from outside if you do due to which you might be getting intestinal infection. But from your history of a polyp in the gall bladder, and pain in the right upper side, it could be an attack of infection in the gall bladder itself. I think, you have taken the right decision to get yourself reviewed by a surgeon, who could plan out further management after related investigations.I hope this information would help you in discussing with your family physician/treating doctor in further management of your problem. Please do not hesitate to ask in case of any further doubts.Thanks for choosing health care magic to clear doubts on your health problems. I wish you an early recovery. Dr Shareef."
},
{
"id": 160315,
"tgt": "What is hormone treatment for breast cancer, Is it effective ?",
"src": "Patient: What is hormone treatment for breast cancer .Is it effective ? How much will it cost in India and how long it can be taken without any hazrds at the age of 85Yrs. Doctor: Breast cancer does develop due to hormonal variations and hence hormonal treatment shall help in treaing the breast cancers."
},
{
"id": 30697,
"tgt": "Suggest treatment for weakness and sickness while suffering from tuberculosis",
"src": "Patient: my husband is suffering from TB, got to know from St. Martha's hospital in month of oct '09. They took 2 bottles of water from his back bone. And they said they want to do some operation. But we didn't had money for operation, so we discharged him. My husband drinks alcohol too much, and he is not taking the tablets which is suggested from the dots doctors. Now he is become very weak an sick, could not have his food or anything. So please tell me any suggestions for this. Doctor: hello thereI understand your concernHe needs to take milk thistle every morning and evening for his ailing liver and somehow finish his anti tubercular regimeHope that helps"
},
{
"id": 164166,
"tgt": "Could vomiting and fever be due to ear infection in a toddler?",
"src": "Patient: I have a five-year-old daughter who woke up this morning with an earache and vomited twice this morning. She only complained of an earache one time today but now has a fever. Can the vomiting and fever be caused by an ear infection although her ears don't seem to be hurting much? Doctor: Hi... vomiting and fever could as well be the presenting symptoms of an acute ear infection which could be very painful. Usually this might require antibiotic therapy after checking the ears with an otoscope. I suggest you take her to the nearest Emergency Room or her pediatrician as soon as possible.Regards - Dr. Sumanth"
},
{
"id": 124010,
"tgt": "Suggest treatment for deformity on thumb & index fingers",
"src": "Patient: i am a 50 year old thalidomide beneficiary , deformity to both thumbs and index fingers. x rays have shown i have under developed shoulder. what does this mean? Is this what is causing me the pain in my neck and shoulder? does this mean that that shoulder is weak? Dont really understand what underdeveloped means. Doctor: Hello, Well, I don't think there is something called as an underdeveloped shoulder as terminology in medicine if it does exist I really don't know. First time listening something like this. Also, maybe the attending physician has meant, that there is a weakness of the muscles of one shoulder when compared to another, apart from that I do not understand what is an underdeveloped shoulder. Not when you mention the pain in the neck and shoulder, I think this should be related to the cervical radiculopathy. If you can take an MRI of the cervical spine we can understand some of the things better. Because the pain in the neck and shoulder can be due to two reasons, one it might be inflammation of the trapezius muscle or might be the nerve entrapment. I will advise you to take a neck collar and use it for a week or two, during this use hot water fermentation for the neck, upper back and shoulder - front and back region. See if you find any changes if any muscle inflammation is present than you might get relief of it and later you can do some slow strengthening exercises under a guided physiotherapist for both the neck and the shoulder muscles. Coming to the deformity of the thumb and index finger, if you put more details in front of me that I can understand and comment over it as, as of now it is little difficult with the so small statement about the thumb and index finger deformity. Hope I have answered your query. Let me know if I can assist you further. Regards, Jay Indravadan Patel, Physical Therapist or Physiotherapist"
},
{
"id": 199145,
"tgt": "Suggest treatment for erectile dysfunction",
"src": "Patient: HI,IM A 53 YR OLD MAN, IM experiencing erectile dysfunction problems,i tried various erection drugs, like Cialis,viagra,levitra, spedra, each were of no use,im a non smoker,light drinker,i take rosuva anti cholesterol drug plus coversyl arginine 5mg tablet-daily, I wonder can you help me solve my problem? im married 6 months and im so embarrassed with my non erections and soft erections,what is wrong? I get prostatitis maybe once yearly-is this part of my problem? or related in any way? so looking forward to some advice? my new doctor has recommended I go to a urologist very soon? yours sincerely- RAY Doctor: DearWe understand your concernsI went through your details. I too suggest you to consult urologist. You got married very late. That could spell out psychological erectile dysfunction. First meet a urologist and confirm there are no organic or physical causes. They you could visit a psychologist for sex therapy and counseling. If you still need my assistance in this regard, please use this link. http://goo.gl/aYW2pR. Please remember to describe the whole problem with full detail.Hope this answers your query. Available for further clarifications.Good luck."
},
{
"id": 7549,
"tgt": "Suffering with acute acne that seems to have substantially decreased with leaving dettol overnight on the face. Is this method safe?",
"src": "Patient: Hello Doctor, I am 28 yrs, female suffering from acute acne since 4 years. Of late I started applying dettol (non diluted) on face and leaving it over night and washing it in the morning. It has worked for me as the acne has decreased substantially. I wonder how safe this might be for the skin? Please advise...Thanks & Regards. Doctor: Hello, Thanks for writing to us. As you had mentioned that after dettol application your acne has decreased & that has being working for you. Its not safe practise. Its true that poor hygiene aggrevates acne so try to use a cetaphil cleansing lotion for cleaning your face. There are many topical antiacne creams & lotions that work well on acne. Try to consult your dermatologist, looking at your skin condition your doctor will prescibe you the suitable treatment. Take care."
},
{
"id": 190290,
"tgt": "Can I have wine or beer as I am taking metrogyl tablet for gum problem ?",
"src": "Patient: i am taking metrogyl for gum problem, can i have a glass of wine or beer Doctor: i am taking metrogyl for gum problem, can i have a glass of wine or beer"
},
{
"id": 120955,
"tgt": "Suggest treatment for strong neck pain and left arm pain",
"src": "Patient: Hi, Recently, I ve been having strong neck pains. Now that pain is slowly going to my left arm, very achy. The past few days I ve noticed that my left upper chest is tightening up. The tightening/numbing sensation goes all the way up to the left side of my neck. I was recently diagnosed with anxiety. Am I just having a panic attack or is this really something I should be worried about? Theres no pain, just tightening. Especially if I lay down, the tightening gets worse. Please do tell me whats going on! Doctor: Hello,Your symptoms seem to be related to cervical\u00a0radiculopathy, commonly called a \"pinched nerve\". It occurs when a nerve in the\u00a0neck\u00a0is compressed or irritated where it branches away from the spinal cord. I suggest using anti inflammatory medications such as Acetaminophen to relieve the pain. I also suggest to maintain the area immobilized to prevent further damage.Hope I have answered your question. Let me know if I can assist you further. Regards, Dr. Dorina Gurabardhi, General & Family Physician"
},
{
"id": 212658,
"tgt": "Getting anxiety and stress, lack of concentration , recurring thoughts. What am I suffering from?",
"src": "Patient: Hi, I am 26 years male ..I have been suffering from lack of concentration , recurring thoughts since last 10-11 years . My mind never gets relaxed and settled down even while sleeping I am continuously having dreams and I can remember then all . From last 3-4 years I have started feeling very anxious and stressed , most of the times I have worries which doesn t have base , my head feels dizzy ,I dont feel comfortable talking to people as i lose my concentration and feel irritated and restless.I get angry easily, feel vulnerable and confused . Some time i feel that I will lose control on myself . Please suggest the possible problem and solution . Doctor: Hello and welcome to Healthcare Magic. Thanks for your query.From the description of your symptoms, you are likely to be suffering from an anxiety disorder, probably a generalised anxiety disorder. However a detailed history and psychological assessment is necessary to pin-point the diagnosis.There are effective treatment options - in the form of medication and psychotherapy, which will help you overcome your problems. So, please consult a psychiatrist for further evaluation and treatment.Wish you all the best.Regards,Dr. Jonas SundarakumarConsultant Psychiatrist"
},
{
"id": 107061,
"tgt": "How can severe pain in the lower back be treated?",
"src": "Patient: Unbearable lower back pain. I have compression fracture in L12. Was diagnosed on Monday with an x-ray. The pain is getting much worse. Lying on ice packs. Have meds for pain, are not working at all. Any suggestions appreciated. Thank You Barbara Horton YYYY Doctor: dear patient treatment for l12 compression fracture is strict bed rest for one month. Rest should be on hard bed. Analgesics like tab diclofenac sodium 50 mg twice a day with tab ultracet twice a day should be taken. In bed physiotherapy and log rolling 2 hourly is advised to prevent bed sore. if pain is still not relieved consultation with spine surgeon is recommended. Vertebroplasty is good option for immediate pain relief."
},
{
"id": 187813,
"tgt": "What is remedy for swallowing issues and acid reflux caused due to hernia when on synthroid?",
"src": "Patient: I just noticed that two of my teeth are turning black. I have had sensitivity issues for years and use the appropriate toothpaste which has helped as my dentist stated that was the issue and after X-rays no other issues. I have been a \"tooth grinder\" for years. I tried a fitted device but it never stayed in as I snore pretty loudly as well. My dentist actually said I grinded my teeth so badly that they matched.... My dentist was a friend as we'll and he stopped practicing in my state as he got married and moved so I have to confess that I haven't seen a dentist in 3 years shamefully ;(. Over the last year I have had some swallowing issues and addressed those and after tests found out that I have a pretty bad case of acid reflux due to a hernia. Could this have contributed to the above mentioned problems. I do also have RA , Graves' disease. The only medication I take is synthroid and ompmrezole and short stint of reglan. Now that I wrote such a book...maybe some thoughts Doctor: Hello, thank you for consulting with HCM.As you are mentioning that you have a habit of tooth grinding for years, it can be cause of tooth blacking.Actually the habit of tooth grinding leads to wearing of tooth and tooth infection, that is why they are turning back.Better you visit a dentist and get root canal treatment done of the tooth, and afterwards capping of it.Hope it will help youRegards"
},
{
"id": 129744,
"tgt": "Is gout and loss of bladder control related?",
"src": "Patient: recently i suffered the worst case of gout ever. I couldn't walk for a week and am only just recovering. I had the same problem 2 month previous. Now I have cut out all beer and am taking concentrated cherry drink.Thing is I have also lost some power when I go to the toilet for a pee. Is this connected? Doctor: Hello, Those problems are probably not connected. You need to see a urologist about the lost power to urinate. In addition your doctor may need to put you on a medication to lower your uric acid so that you do not continue getting gout flares.Not drinking beer is a great idea also.Regards"
},
{
"id": 83501,
"tgt": "Can snorting of the energy pills cause any side effects?",
"src": "Patient: Hi. My friend bought these new energy pills, OxyElitePro. They contain 100mg of caffeine along with Proprietary Blend Bauhinia Purpurea L (leaf and Pod) (Standarized for Bauhiniastatins 1-4), Bacopa Monniera Leaf), 1,3-Dimethylamylamine (Geranium [stem], Cirsium Oligophyllum (Whole Plant Extract), Rauwolscine (Rauvolfia Canescens L [leaf and Root]. Recently he s started cutting open the capsules and snorting them. When i asked why he said the effects would onset faster and he just liked the feeling. Other then the obvious destruction snorting anything does to your nose, what are the other negative side effects, if any. Doctor: Hi, Yes, it may cause some serious side effects. Oxyelite-Pro is a dietary supplement promoted for weight loss. However its safety and efficacy has not been established on any scientific studies. Apart from causing necrosis of the nasal mucosa if snorted, its use is also linked to liver injury resulting in hepatitis. It also contains fluoxetine which is an antidepressant which may cause sleep disturbances, sexual dysfunction and seizures. In view of the above the Food and Drug Administration (FDA) has already advised its consumers not use this supplement. Hope I have answered your query. Let me know if I can assist you further. Take care Regards, Dr. Mohammed Taher Ali"
},
{
"id": 217903,
"tgt": "Suggest treatment for sciatica pain",
"src": "Patient: hi sir, I am abdulla 37yrs old i am suffering form Sciatica pain. As per my MRI i have problem in L1-L2 and L5-S1 I am driver by Profession i have gone through Physiotherapy, Acupuncture feeling better but not comfortable please advice me how to over come this problem Doctor: Assalamwalaikum. Sciatica mis really vrry pain full condition, u hav to take good amount of pain relief drugs, but still be careful with ur liver and kidneys.u csn go for laser treatment fo about 4 to 5 sittings.it will really help u Inshallah."
},
{
"id": 181474,
"tgt": "Suggest treatment for loose teeth in a child",
"src": "Patient: My 18 month old son has no history of trauma to his teeth. But his top central incisors and lower central incisors seem a bit more mobile then the rest of his teeth.Is this normal or should I be concerned? He has all teeth but his canines all just poked through. Could this just be his teeth finding their place? Doctor: Hi..Welcome to HEALTHCARE MAGIC..I have gone through your query and can understand your concerns..As per your complain during the initial period of tooth eruption the and along with erupting teeth the jaw also keep growing leading to movement of teeth and occupying their appropriate position..There are certain grades of mobility and the teeth should lie under it to be normal and healthy..A very slight mobillty of front teeth can be seen as compared to back teeth as the front teeth has a single root and less Anchorage than back teeth with multiple roots..I would therefore suggest you to consult a Pedodontist and get evaluated and he can check whether the teetj are normal as slight mobility is not abnormal as teeth are surrounded in the root area by a cushion like ligament and not tightly bound to bone..Any abnormality needs to be further evaluated by x rays and treated accordingly..As of now maintenance of good Oral hygiene and a healthy diet should be given..Hope this information helps..Thanks and regards.Dr.Honey Nandwani Arora.."
},
{
"id": 117413,
"tgt": "How to reduce the level of sgpt and lymphocyte?",
"src": "Patient: Dear Doctor, i am 31 year old male and have recently undergone through pre employment medical check up and it was found that i have sgpt- 48 and lymphocyte-41 except this all test are absolutely normal test includes LFt, Lipid profile serum, urine rotine and microscopy, CBC and PT/PT-INR. So please suggest what to do(precaution and prevention) to get this reduced. Doctor: HIThanks for posting your query to Healthcaremagic . I think there is nothing to be worried or concerned of these reports . These are minor elevation and can happen due to simple causes like a Viral or bacterial infection . Even one booze of alcohol can elevate your liver enzymes ( but not Lymphocyte ). At present no precaution or prevention is required . Only one thing if you are consuming alcohol kindly stop it for few months . Repeat SGPT and Differential Count after 3 months and revert back ."
},
{
"id": 70245,
"tgt": "What is the cure for the abscess on the jaw and swelling in the face?",
"src": "Patient: Hi, I have an abscess on my jaw & my face has become very swollen. I have just started a course of antibiotics(clonamox 500mg) but my face is continuing to swell. I have also been taking ibuprofen for the pain & inflammation. Is there anything else I can try, to bring down the sweling tonight as I have to go to work tomorrow? Thanks Jennifer Doctor: HI Jennifer.Thanks for your elucidate history. This means there is an abscess and you have to get it drained by a Dentist or a Surgeon. This is the sure-shot treatment. You have to continue medicines you are already under.A timely treatment can help you fast and complete recovery. You also have to get the offending tooth out."
},
{
"id": 102002,
"tgt": "Can nebulisation be done and will there be any side effects?",
"src": "Patient: Hi my son is 7 yr old.As told by Dr he has allergy problem and due to this he is hving treatment for frequent redness and itching in eyes and now he has also started hving cold and cough attacks very frequently. In october he has given tab CEFADROX DT, Syp Levolin and Soventus D along with Dualin through Nebulisation. Now again he is hving the same problem. Can i nebulise him thrice a day for 3-5 days as he feels better with this. He is hving dry cough continuously. Is there any side effect of nebulizer. Doctor: HI, it is good to nebulise but before that levosalbutamol inhaler should be tried as it is more convenient, to manage the condition he should also be given some antibiotics, expectorant cough syrup containing ambroxol and some anti histaminic groups of anti allergic is advisable.if symptom persist then consult a physician for further examination and treatment. thanks and with regards."
},
{
"id": 176579,
"tgt": "What causes burning sensation on nasal passages along with fever?",
"src": "Patient: My 11 yr old son has had a 103 fever since yesterday, aches and I just noticed his red nasal passages. He said that they burn. The fever most times has responded to ibuprofen. None of my boys have complained of nasal passages that burn. Is that an unusual symptom? I am just allowing him to sleep and work through whatever illness this is. Doctor: Hi,It seems that he might be having acute rhinitis producing soreness and congestion leading to redness in the nasal cavity.Nothing to worry.Go for one antibiotic medicine course for 3-5 days.Give him decongestant medicine as well.Apply antibiotic cream in nasal cavity.Ok and take care."
},
{
"id": 216795,
"tgt": "What causes severe pain in lower abdomen while breathing from mouth?",
"src": "Patient: For a few days I have had severe pain in my lower left abdomen when i breath through my mouth. Every single breath produces a sharp, stabbing pain that resonates for a second or two as it fades away - until I breathe in again. It also hurts a little bit if i cough and it hurts a lot when i laugh. Breathing in shallowly or through my nose does not hurt.I am 24 and overweight. I do not smoke. I also do not have the money to go to the doctors and get tests done. Please Help! Doctor: have you started a new exercise regime lately?? or lifted any heavy weight / exerted in a manner different from normal routine?"
},
{
"id": 15055,
"tgt": "Inflamed, itchy rashes on legs after coming in contact with an elephant. What could this be?",
"src": "Patient: Hi, I am in Thailand and have a rash develop on my legs. I first noticed it after riding an elephant in Chiang Mai 9 days ago. The rash has not gone and may be spreading. The rash looks like tiny bites (smaller than mosquito bites), are itchy, inflamed and also some develop pimple white heads. Any idea what it could be? Doctor: hi i think u are suffering from irritant dermatitis due to udt present on elephant skin.it can be cured by atarax 25 mg at night ,antibiotic tice daily and application of moisteriser all over the leg followed by application of diprovate g ointment on the lesions"
},
{
"id": 187843,
"tgt": "How early should a abscessed tooth be treated because despite penicillin shots, pain has increased and have developed fever?",
"src": "Patient: When should you go to a hospital over an abscessed tooth? I began taking penicillin today for infection as prescribed by a dentist. I have taken 3 doses. Recently, I developed more pain along with a substantial increase in swelling along with a fever which i did not previously have. Doctor: Hello, thank you for sharing your problem with HCM. As you are mentioning that you are having abscess in a particular tooth, then there is no need to wait, you should start with antibiotics as well as root canal. As the pain will not reduce till root canal start. So better you start with the treatment as soon as possible. Hope it will help you. Regards"
},
{
"id": 66481,
"tgt": "Suggest treatment for a lump on the head post head injury",
"src": "Patient: I was hit on the right side of the head just off the eyebrow a month ago I had a huge lump and black eye it was really sore after a few days the lump went down but I was bruised all down the side of my face the bruising has only gone but there is still a small lump and is still very tender Doctor: Hi,This lump is normal. It is collected blood or a hematoma.If there is pain, redness or fever then it could be mean that it is getting infected. In that case, you should see your Doctor who can examine you and advice you the right course of medicines most likely antibiotics and anti inflammatory medications. Also, put warm compresses over the swelling it will soothe the pain but done massage it.Take care,Dr Rishi, New Delhi, India."
},
{
"id": 52423,
"tgt": "Is stem cell therapy the right method of treatment for stage IV liver disease?",
"src": "Patient: How well does and are there big benefits to doing stem cell therapy for stage IV liver disease. I suffer from NASH with two small children and would like to be able to prolong either the inevitable or live my life out to a ripe old age with some optimism. Thank you, Debbie Doctor: Hello, Stem cell therapy is useful in cirrhosis of liver. Yes, injection of stem-cell into the liver may lead to regeneration of liver. But don't know exact success rate after stem cell therapy. Liver transplantation may useful in cirrhosis patients. Hope I have answered your query. Let me know if I can assist you further. Regards, Dr. Penchila Prasad Kandikattu, Internal Medicine Specialist"
},
{
"id": 163824,
"tgt": "How can a baby rash with dryness, itching and crusting be treated?",
"src": "Patient: My 4 month old baby has had a rash since 1 week after birth it flares up often. I have tried prescribed hydrocortizone cream, baby aveno products, eucerin lotion, and the doc just changed her milk. Why isn t it going away? Right now it seems to itch her and its dry and crusty looking Doctor: Hi...Thank you for consulting in Health Care magic.By what you say I feel that your baby is having Eczema which is a simple skin allergy which is also called atopic dermatitis.Skin conditions are best diagnosed only after seeing directly. I suggest you to upload photographs of the same on this website, so that I can guide you scientifically.Please revert back to me with images so that I can guide you better.You can approach me at the following link.Once the page opens there will be an option below my image as \u2013 ASK ME A QUESTION \u2013 click on it.Please find the link below -www.healthcaremagic.com/doctors/dr-sumanth-amperayani/67696Regards - Dr. Sumanth"
},
{
"id": 11617,
"tgt": "What is the remedy for pigmentation, white spots on face?",
"src": "Patient: hi.. i am suffereing from skin pigmentation problem.i visited to doctor also, she recomended sunstop 30..but i didn't get any satisfactory improvement on my face, along with this pigmentation problem i am also suffereing from white patches problem on my face , some time it appeares and some time automatic dissappear completly..please recommed somthing.. Doctor: Hello.If you have white spots on face ,a possible diagnosis is fungal infection.Clotrimazole cream twice a day is widely used in the treatment of fungal infection.Please go to your dermatologist to confirm the diagnosis and for prescription.I wish you good health.(If the answer has helped you, please indicate this)"
},
{
"id": 77892,
"tgt": "Suggest treatment for bronchitis and chest congestion",
"src": "Patient: this past sunday I have developed bronchitis or some sort of bad chest cold. I went to urgent care and they put me on steroids and gave me a breathing treatment. It has been 24 hours and I am still finding moments when I wheeze. I have to take a 6 hour flight to california for work tomorrow. Is it safe to fly with this and sinus congestion? I have a inhaler i will bring along with me on the plane. Doctor: Thanks for your question on Health Care Magic. I can understand your concern. In my opinion, you are mostly having acute bronchitis. You are not completely cured from it so better not to travel with these symptoms. You should first treat this and be free of symptoms then you can do air travel. So better to consult pulmonologist and get done clinical examination of respiratory system and PFT (Pulmonary Function Test). PFT will also tell you about severity of the disease and treatment of bronchitis is based on severity only. You will need inhaled bronchodilators and inhaled corticosteroid (ICS) because Inhaled treatment is best for bronchitis. Don't worry, you will be alright with this and able to fly. Hope I have solved your query. I will be happy to help you further. Wish you good health. Thanks."
},
{
"id": 52467,
"tgt": "Is it normal when the liver appears slightly hypoechoic and the intrahepatic biliary ducts are not dilated?",
"src": "Patient: Dear sir. I want to know the meaning of this in upper abdomin ultrasound findings. Its my friend s report. Kindly tell me its meaning. Also tell me what should he eat to overcome from it. Liver appears slightly hypoechoic .. intrahepatic biliary ducts are not dilated. Doctor: it is a very non specific report. hyopecheoic liver maybe seen in hepatitis.i suggest you tell the patient to get his LFT's done."
},
{
"id": 168634,
"tgt": "Suggest medication for high fever and swelling of throat after swallowing fish bone",
"src": "Patient: Hi 4 days back my 14 month baby ate fish bone but he was quite well on that day.after a day he develope high fever, and i went to the gp he found inflamed throat, and gave antibiotics. but without giving tha antibiotics fever interval was increase.today is the fourth day he is having fever 38.5 the same spike as earlier but he is eatint and drinking as earlier. but some time he put his finger inside his mouth and trying to nausiate. he vomited twice after his meal. what can i do for this Doctor: have a laryngoscopy done by ENT surgeon to find out any fish bone left in the throat which constantly causing irritation and foriegn body reaction"
},
{
"id": 100655,
"tgt": "What causes red itchy patches all over the body after eating egg?",
"src": "Patient: hii doctor..when i am excited or go out in the sun..i get red patches all over my body..and it itches badly..and i have noticed that this happens more when i consume egg or skip breakfast...please help me out...it feels really bad when my body itches.. Doctor: Hello.Thank you for asking at HCM.I went through your history and would like to make suggestions to you as follows:1. Were I treating you, I would suggest you to take an antihistamine like cetirizine or levocetirizine when you develop itchy red patches (hives).2. I would also suggest you to apply a good moisturizing cream with SPF 15 or more.3. I would also suggest you to consume foods rich in vitamin A like ripe mango, carrots, spinach, etc.4. Personally I would suggest you to avoid hot/spicy foods/beverages.5. I would also suggest you allergy testing for eggs, if it is positive, I would suggest you to avoid eating eggs for at least 3-6 months.Hope above suggestions will be helpful to you.Should you have any further query, please feel free to ask at HCM.Wish you the best of the health.Thank you & Regards."
},
{
"id": 183443,
"tgt": "How to treat soreness and swelling of jaw after undergoing root canal?",
"src": "Patient: I had a root canal almost 4 weeks ago. And I was pain free till just a week ago I started feeling really sore on my jaw/side of the treatment. It almost feels swollen and there's a little lump feels like comin from under . It's sensitive to touch and bite . Is this normal? Doctor: hellooo...read thru ur query...since it has a lump under and pain and sore...itmight have got infected beneath the root.....dont worry...u have to go to an endodontist take an xray and see the condition..sometimes u might have to undergo a small surgical procedure(apicoectomy)nothing to worry..its just removal of root tip(which causes the infection).and take antibiotics...it will be fine...it happens in some cases(good indication that ur immunity is good)..so be cool get an appointment of endodontist and get it treated according to clinical condition...hope ur benefitted something from this reply..have a healthy day!!!"
},
{
"id": 74734,
"tgt": "What can be the reason for chest pain?",
"src": "Patient: kinda long history but when i was thirty i had cc hyster and radiation then at 32 had lung surgery for masses they thought were cancer turned out good now at 33 started having chest pains again like before pnemonia? so went to er told i have bronchitis and bladder infection and white cells are low to follow up with onco. three days later got ring worm on leg and a few lymph nodes are swollen what is going on with me?? Doctor: Respected user, hi I evaluated your query thoroughly.* Differential diagnosis include bacterial or viral infection of respiratory tract with low immunity leading to super added infections like bladder infection, ring worms on legs . Possibility of micro metastasis must be ruled out in form of relapse.* Thorough clinical evaluations along with necessary data in form of Laboratory , Imaging like bronchoscopy , CT / MRI ,Pulmonary lavage for histo pathology according to the clinician judgement .Hope this helps you .Welcome for any further assistance .Thanks for using Healthcaremagic.com & reviewing my answer thoroughly.Wishing you fine recovery from the same.Regards dear take care."
},
{
"id": 201809,
"tgt": "Is it normal to have 20% rate of sperm count?",
"src": "Patient: hi sir. gud morning im male 29 years married in September 2013... we hardly trying fr childers... in april month i went hospital done sperm analysis test result shows 20% motality , in auguest it increases 42%.... nw currently taking lineator capasules frm 25 days... im not alcohalic, non smoker ... im energetic healthy body..... please give me a better suggestion to product childers....... pls answer naresh.hyderanad Doctor: hello naresh,thonks a lot for your query on HCM,it is very less a value of motality than normal for the sperms.usually more than 90% is considered as normal.nice to see you already being taking tablets for your condition,do continue taking it as advised by your physicianthings will definately improve.but it will not be a drastic process, it will be a gradual process taking time.do be patient.things will improve slowly"
},
{
"id": 105782,
"tgt": "How is nasobronchial allergy treated ?",
"src": "Patient: I am very confused regarding to my health because my doctor never tells me about the permanent treatment of my disease that is nasobronchial allergy . I am continuous taking the medicines (moteklucast & rotacap 200) from last six month. But from last 20 days I am not taking the medicines ,therefore again the condition is same.I am now suffering form sneezing , running nose and choking nose at night this creates problem in breathing .I am 20 years old girl and i am feeling bad due to these problem in front of relatives and friends.please help me to get rid from my disease permanently. Doctor: Hi, I have gone through your query. I would like to tell you one thing about the treatment of allergy. The only effective treatment of Allergy is avoidance of allergen ( Triggering factor). Any substance could be an allergen. Most common allergens for Naso bronchial allergy are dust and pollen. As far as the monteluKast is concerned - this is not useful in treating acute attack of allergy. It will help in reducing the frequency of allergic episodes. For it to be effective you might have to use it for years ( 2-3 Years). In very rare cases may have to be used for very long periods. Rotacaps will give you instant relief but will not effect the recurrence of allergic episodes. The only definitive treatment as i have mentioned earlier is avoidance of allergen. Just list out all the possible allergens with which you experience allergy. Try to avoid those allergens. Try using a cloth mask while going out. Using newer ntihistaminics like Fexofenadine 120 mg ( Allegra) will help you to counter acute attacks. Get your Blood Eosinophil count done. Based on the result. You can use some medicines like DEC ( Di Ethyl carbamazine) which reduces the eosinophils and will prevent recurrent attacks of nasobronchial allergy. But please discuss with your doctor before you start using DEC . Hope you got some solution. Wish you a healthy day Take care"
},
{
"id": 222019,
"tgt": "What are the chances of getting pregnant without penetration?",
"src": "Patient: hi me and my gf didnot have sexual intercourse but yet we play around with each other tools. 3days back while we are together she gave me a handjob and i ejaulated over her hands and some part of my body, we cleaned up nad wiped the sperm with tissue paper and after 15-20mints i put my fingers in her vagina. can it cause pregnancy? pls rply soon as we both are really scared now Doctor: No chanceshe are very very rare. Usually sperms are fragile and can't survive outside body for so long. so don't worry."
},
{
"id": 125952,
"tgt": "What causes right-sided head pain radiating towards the neck?",
"src": "Patient: Last night the right side of my head was hurting so bad I thought about going to thr er. It was very intense and radiated down my neck. This morning when I woke up, the right side now feels sore and tingley. My husband said it was probably because the pain was so intense. Was it just a bad headache? Doctor: Hi, Consult a neurologist and get evaluated. An MRI scan is required for further assessment and evaluation. As of now analgesics like Aceclofenac or Tramadol can be taken for pain relief. Hope I have answered your query. Let me know if I can assist you further. Regards, Dr. Shinas Hussain, General & Family Physician"
},
{
"id": 185964,
"tgt": "Suggest remedy for sore gum",
"src": "Patient: i have developed a very sore gum,on my left lower jaw on the place where my wisdom tooth is to come.I m 20 and never had this much swelling before.There have been days when i developed swelling there but it was not too painful.But this time it is too much and i cant eat anything,even swallowing pains and the left side o my throat has started to pain now.I smoked weed the previous night an when i woke up i had this sore gum.Are these two things related?How can i get well asap? Doctor: Thanks for your query, i have gone through your query. the soreness in the gums could be because of the pericoronitis or pericoronal abscess secondary to infection of the gums around the wisdom tooth. consult your oral physician and get radiograph done. if the space is sufficient for the tooth to erupt then we can remove the flap around the tooth. if space is not there for the tooth to erupt then you have to get it removed. you can take a course of antibiotics. do saline gargling. i hope my answer will help you. take care."
},
{
"id": 96399,
"tgt": "I cannot digest normal food from last 7 months",
"src": "Patient: I am 35 yrs old. I cannot digest normal food. This is happening for the past 7 months. As a result, I feel very weak. My skin is also shrinking and became very dry, full of wrinkles . What to do and what to eat Doctor: It appears to be a chronic problem. Get yourself examinaed ASAP! Gastroenterologist would be fine."
},
{
"id": 99734,
"tgt": "What causes rash and bumps in arm after donating blood?",
"src": "Patient: I h ave never had problems giving blood in the past I am allergic to some adhesive Andmetals but never once have I had any issues well this past. Saturday, 3 days ago I donated and now my arm in the area where I donated, is broken out in some sort of rash or tiny bumps is that normal? Doctor: Hi, as u said u had a allergic to some metal,it might bcuz if this large bore needle which is used to take blood during donation, kindly consult your doctor and take appropriate treatment. But don't worry this is not serious."
},
{
"id": 216391,
"tgt": "What causes pain in lower abdomen,hip,thigh and groin area?",
"src": "Patient: Yes I am a 50 yr old man and I waited to use the bathroom last eveninguntil I got home.I dont like to use public restrooms for #2 so I held it 20min. In th middle of the night i woke up in pain.My lower abdomen, right side of hip, thigh and growing seem to be a little better today but I am concerned.Could I have strained muscles or could it be more serious? Doctor: Dear Patient,I do not think that the pain in the middle of the night is related to the event of the last night. Keeping the urine for 20 minutes normally should not produce extreme pain big enough to wake you up during sleeping. Of course that having strained the muscle may give you pain, and maybe you strained the muscle during holding the urine. What I suggest is to wait for 2 more days. If you experience again the pain, or other symptoms please go and make the urine and blood analysis, to check for prostatitis, uretritis or any infection of the lower urinary tract. Kidney stones may give you pain also, but they are after localized in the back, lumbar region.Meanwhile I would suggest You drink 2-3 liter of water a day, so if there is any infection, a lot of water will alleviate it.Please feel free to write me anytime, and thank you for choosing HCM.All the best,Dr. Eni"
},
{
"id": 184498,
"tgt": "What should be done in the case of fake tooth?",
"src": "Patient: I had a fake tooth come out with the post and lost it. It is a front tooth on the left. I had a root canal and the post receiver is in place. How do I explain what I need done?There is also a sliver of the old tooth there. Is this a crown with a post?Implant? Doctor: Hello, Thanks for consulting HCM, Read your query, as you have mentioned in your query this is known as post and core treatment , in this treatment Root canal treatment is done in tooth after post is placed and core is given as crown of tooth . For this you should consult your dentist and go for oral examination of tooth , if your post is lost then you should go for new post and core by discussing with your dentist. Hope it will help you. Wishing you good health.Regards , Dr. Priyanka tiwari"
},
{
"id": 84950,
"tgt": "Will taking Tylenol cause cold?",
"src": "Patient: Hi, I had a bad cold after I took Tylenol cold medicine & bacterium Imy lips have been swollen and sore. And I noticed red round patches on my left hand and groin are. They are warm to the touch . And my lips have been very hot too.why this happened & what should I do? Doctor: Hi,Cannot say in your particular case, especially without seeing the dermatologic changes. AND, while it is rare, someone can have an allergic reaction to tylenol with irritation especially where the drug has contact (the mouth).Hope I have answered your query. Let me know if I can assist you further. Regards, Dr. Matt Wachsman, Addiction Medicine Specialist"
},
{
"id": 99972,
"tgt": "What causes nonstop coughing even after taking cortisone injection?",
"src": "Patient: I had a cortisone injection for my asthma 10 days ago but I am still coughing non stop. Should I go back to my pulmonologist? I am 68, 5feet, 122 pounds. He did a chest xray which showed possibly previous pneumonia which he said was resolving. Sinus xrays were negative. Doctor: HI, thanks for using healthcare magicIt would be best to revisit your pulmonologist or GP.The repeated coughing indicates that the acute asthmatic attack has still not resolved and you would need additional treatment.This should be done as soon as possible.I hope this helps"
},
{
"id": 186949,
"tgt": "What is the treatment for a flap inside the cheek?",
"src": "Patient: I have what I can only describe as a flap on the inside of my cheek. It is a little sore when I play with it, but otherwise fine. Most concerning is that it seems to have appeared over night (I did not bite or scrape it). What could this be and will it heal on its own or should I seek dental care? Doctor: Hello, thank you for consulting with healthcaremagic. As you are mentioning that you have a flap inside your cheek, it looks that it might have occurred because of trauma only.But if it is painless, then there is nothing to worry, as it will heal on its own. Just observe it , if it starts creating any problems, then visit a good dentist. Hope it will help you."
},
{
"id": 54423,
"tgt": "Suggest treatment for burst blood vessel in liver with extended abdomen",
"src": "Patient: Hi , ive had ultrasound ,ct scan, nuclear mri , blood tests done to check my liver .My symptoms are extended upper abdomen ,nausea. and dizziness. my g.p has reported a burst blood vessel in the liver but with no medication just a refferal to a specialist .I am consulting with a herbalist ,which has stoppped the nausea and dizziness ,but not helped the abdomen!!! which is very large ...ive gone from dress size 10 to 14. Can you please shine some light on the situation! Doctor: Hi I have gone through your query...Here you have enlarge abdomen with dizziness ....If variceal bleed there then through blood loss dizziness can occur....Hypoglycemia , heart failure can lead dizziness....For that sugar estimation and ECG done...If variceal bleed occured as you have mentioned in history then endoscopic band ligation or sclerotherapy useful....Here by sodium and water retention edema can be there....For that diuretic like furosemide tablet can be taken...Take care.Consult gastroentetologist for further guidence and therapy....Dr.Parth"
},
{
"id": 26537,
"tgt": "What causes facial numbness on one side?",
"src": "Patient: My husband is 63 and he s had a heart attack and has two stents. He was recently at the doctors and he is again 100% blocked on the left side and they sent him on his way and said we will see you in 3 mths. I thought that was risky, so I made an appt with a different cadiologist for Monday. But we were shopping and just got home and he said half of his face feels numb, and I m worried about a stroke. He claims it s his tooth ache. Should I be worried? Doctor: HiThanks for posting your query. Well the possibility of stroke is high as a 100% blockage is heart vessels usually be associated with certain degree of blockage of blood vessels to brain. I would recommend you get an Carotid artery Doppler which checks for blockage in blood vessels of brain. Or else he better consult a physician who will evaluate and advise accordingly. But kindly do consult.Hope this information was useful to you.Any further clarification feel free to ask.Regards."
},
{
"id": 102061,
"tgt": "Suggest treatment for skin rash due to food allergy",
"src": "Patient: Hello. I have a thyroid problem and since I started with this problem I have nothing but problems with my health. I now have an allergy to nuts (not the case before this thyroid problem) and other foods and I have developed red, itchy and scaly skin at the forefront hairline, on the temples and on my eye lids and it hurts. Before Christmas I changed the dose of the thyroxine to 125 mg, because 100 mg was not enough and I was not feeling well and gaining weight. I just do not feel well. I will go for a blood test next week to assess the situation, but it does not answer the problem of the red and blotchy skin. Thank you for any advice you can give me in advance. Maria Doctor: your allergy can be from any protein any time anything can start reacting with body proteins to cause allergyit can be oils soaps shampoo creams cooking oils hena dyes. medicines used for other ailmentsit can be food clothes pollens dust grassesalong with controlling thyroid with correct dosages I will advise you to get allergy tests to find out the proteins you are allergic to avoid the allergens which can be avoidedand treat others with sublingual immunotherapy which is gaining favour internationally now"
},
{
"id": 53115,
"tgt": "Suggest medication for a liver disorder",
"src": "Patient: i am on udiliv 300mg for faulty liver disorders - any better mediciene to protect the liver. i had lost 7kgs of weight in 4 months but last 3 months my weight is constant. i have suddenly started looking 10 yrs older than my age 43 and doctor says less of oxygen in body. Doctor: Hi,Liver rejenuvators and antioxidants a re a good option.Take one capsule of sylimarin daily.It is 16 times more potent in liver problems as compared to other anti oxidants.Alternative ayurvedic therapy with LIV52 capsules twice daily can be taken along with it(herbal supplement).Take a multi vitamin capsule daily,Ginko biloba 50mg should be taken once daily,Start following a regular exercise schedule and eat a good and healthy low fat diet,Take 2 cups of fruits daily(cheap and locally available seasonal fruits).Eat green leafy vegetablesavoid too much dairy and meat,Walk 30 minutes a day,Involve in activities like swimming or cycling,Things will start getting better in 2-3 months.Medicines is not a substotute to healthy life style."
},
{
"id": 111540,
"tgt": "Cause for pain in the lower back and thigh?",
"src": "Patient: I have a pain in my lower back and cramps like on my left thigh some times when i sit i feel better but when i walks or stand for more than one hrs. then again pain in my lower back starts few months ago my MRI ( LUMBOSACRAL SPINE) showed as: Mild diffuse disc bulge with protrusion at L4-L5,Annular bulge at L5-S1,Predominatly Para-Posterio-Central(RT LT) resuling compression over the epidural SAC,Thecal SAC,Exiting nerve roots with maintained spinal canal dimension & neural formation stenosis at L4-S1 suggested mild prolapsed intervertebral disc with polyradiculopathy.Early Lumbar spondylosis changes. As per suggestion of Doctor(MCH.ORTHO) i am taking Zevanuron Capsule once a day since one month but there is little improvement in my lower back pain .I am 33 years old and my weight is 74 Kg and height is 5 2 . Should i continue ZEVANURON CAPSULE only or any other medicine ? Kindly suggest me about root cause of pain in lower back ,medicine ?,precausions etc . Doctor: Hello,I had gone through the case and report and found that this is good medicine for strengthening of nerve. But you need to take calcium also.You must go for one blood test Vitamin D3 and Vitamin B12. If it is low then take Vitamin D3 .If pain persist then take some session of physiotherapy and learn some particular yoga for pain relieving.Follow all the instruction and feel better in pain.Hope my answer will be effective for you.Thanks"
},
{
"id": 48785,
"tgt": "What precautions should be taken for a dialysis patient with severe headache?",
"src": "Patient: My roommate is a new dialysis patient...He has AIDs AIDs dementia and now total kidney failure and has started dialysis last week....Today his blood pressure is 162/90, vision is blurry and a bad headache. What should I look for if there is a major problem going on? Doctor: Hi, Visual symptoms and Headchace do not occur at 160/90 BP in Dialysis patients ,as they tolerate higher BP very well.As he has AIDS i suggest you see infectitous disease specialist ,your friend might require CT/MRI and CSF analysis to evaluate for infections to brain."
},
{
"id": 126657,
"tgt": "Suggest treatment for weakness in arms and tightening of shoulder muscles",
"src": "Patient: my arms from the elbow to my hands get really weak in the last week, and i have tightness of my back shoulder muscles just before this happens. It goes away after about 10 minutes but they are really jello like. It doesn t matter what I m doing when this happens. Doctor: Hello, Weakness on your arms from the elbow to your hands associated with tightness in your back shoulder muscles could be due to any neurological problem with the spinal cord. I highly recommend to : - consult with a neurologist and run a good neurological examination - run a good cardiac checkup (EKG, echocardiogram) to rule out heart problems. Hope I have answered your query. Let me know if I can assist you further. Take care Regards, Dr Albana Sejdini, General & Family Physician"
},
{
"id": 216234,
"tgt": "What causes a sensation of glass pieces in skin?",
"src": "Patient: I have the sensation that I have pieces of glass in my skin. Nothing comes out and the sensation is there for a while (weeks) and then it goes away. It use to be on the bottom of my feet and at the top of my head. Now it is on the top of my foot, along the underneath side of my left arm between my wrist and elbow, and all over my head. When I lay down to sleep or rest my head against a piece of furniture it really hurts. The same is true when I put on socks or clothing over the affected areas. I have been checked for MS in the past but the doctors didn t see enough symptoms. These symptoms have developed since that time. When I was checked, they did not do a spinal tap, we just answered questions and I had an MRI, which showed a non-specific lesion in the brain. Other symptoms were pain on the right side of my body, memory issues, speech issues, overall pain, fatigue, etc. Doctor: Quite a lot of possiblities. First, there is the possibility of having glass in there. But generally something would be visible .Sick nerves in localized areas can occur from nerve poisons. It is more typically at the ends of nerves and on both sides evenly, BUT B12 deficiency, thyroid level low, or lyme disease or syphillis or some metal poisonings can do this. Cannot say which is in your case and sometimes a biopsy is needed.MS typically goes for years prior to diagnosis."
},
{
"id": 116046,
"tgt": "Suggest remedy for an internal bleed",
"src": "Patient: My friend is in the Hospital and says Blood tests state she now has an Internal Bleed. She was diagnosed with CHF the other day and she is obese and has sores on her back calves. They ve put her on antibiotics etc, but she is on different medications for various reasons. Gravol, anti-depressants, Tylenol and seizure meds etc. Wondering if these meds have anything to do with her Internal Bleed. She seems very ill, will she pull through this, can they do anything for an Internal Bleed? Doctor: Hello and welcome to HCM,Congestive heart failure and medications prescribed to you are probably not the cause of internal bleed.Internal bleed can occur due to internal injury.Spontaneous internal bleeding can occur due to reduced platelet count or deficiency of coagulation factors.Thus, you need to consult your primary healthcare provider for clinical assessment and prescribe these investigations- complete hemogram, platelet counts, bleeding time, clotting time and other coagulation tests- prothrombin time (PT) and activated partial thromboplastin time (aPTT).In case any of these investigations are abnormal, you need to consult your hematologist for further management.Thanks and take careDr shailja P Wahal"
},
{
"id": 85743,
"tgt": "Is anxiety a side effect of taking Trazodone with Buspirone?",
"src": "Patient: I was recently prescribed Trazadone 50mg for insomnia and two daily doses of Buspirone 5mg for anxiety. In looking at the side effects, I read that there is the possibility of an adverse drug interaction. This, of course, has increased my anxiety. I hope you can help. Thank you. Doctor: Hello, It should not. Using busPIRone together with traZODone can increase the risk of a rare but serious condition called the serotonin syndrome, which may include symptoms such as confusion, hallucination, seizure, extreme changes in blood pressure, increased heart rate, fever, excessive sweating, shivering or shaking, blurred visible. Hope I have answered your query. Let me know if I can assist you further. Take care Regards, Dr AJEET SINGH, General & Family Physician"
},
{
"id": 180345,
"tgt": "Does antibiotic cause metallic taste in the tongue post tooth extraction?",
"src": "Patient: I had a tooth extracted and five weeks later it hasn t healed properly. I went to the dentist who put me on antibiotics for 5 days and I have one day left of this but there s still a fizzing sensation and nasty metallic taste in my mouth. The pain is reduced but it still feels like somethings wrong. Doctor: Hi,Understanding your concern. As per your query you have symptoms of metallic taste in the tongue post tooth extraction. Need not to worry about it. Visit ENT specialist once and get it examined as dysguesia could be another possible cause for your condition. Go for citrus food to stimulate your taste buds. Drink plenty of water and take diet rich in multivitamins. Keep brushing twice daily and use Listerine mouthwash. Do flossing twice a week at least.Hope your concern has been resolved.Best Wishes,Dr. Harry Maheshwari"
},
{
"id": 112329,
"tgt": "On narcotics, chronic back pain, took surgery, meds, nothing helped. Permanent solution?",
"src": "Patient: Yes you may. I have been on narcotis or many years now or chronic back pain after two failed surgeries, with the last on putting in an artifiial disc at L4-5 which only caused things to get worse, consiquently I have been taking pain meds to give some kind of relief. What I have found is that it takes more and more pain meds to be effective. I m finding that I don t feel anything when I take these meds.....not much relief, no buzz which concerns me as I believe there may be something wrong with my liver processing the meds. I m not chasing the high , I ve just always contributed some kind of mental change to knowing the medicine was working. I m afaid that my liver is messed up after years of processing drugs. Can you help me?. Doctor: Hi, as you describe you on treatment for liver problems, and pain medicines to the maximum extent. It is not advisible to have more pain medications with liver problem. When ever you are prescribed pain medicines you should inform your doctor that you are on liver medicines. I advise my patients with such symptoms neurotropic injections for 10 consecutive days, to enhance the resistance to pain along with other medicines, and slowly minimise the dose of pain drugs. Thank you."
},
{
"id": 204438,
"tgt": "How to deal with a person with schizoid personality?",
"src": "Patient: I am in love with a complicated man. I think he is too but I gave him a Luscher color test and with that and another book , he seems to be a Schizoid personality. If I show him a predictable behavior, will it help, when he is afraid he ll be hurt by those that loved him in the past. So, I can t show any power over him, and he seems very easily hurt. What advise have you. . . Doctor: Hello and Welcome to \u2018Ask A Doctor\u2019 service. I have reviewed your query and here is my advice. People with schizoid personality disorder don't desire or enjoy close relationships, even with family, and are often seen as loners. They may be emotionally cold and detached. Psychotherapy and medication, such as antidepressants or mood stabilizers, can help. You must consult a psychiatrist. Hope I have answered your query. Let me know if I can assist you further. Regards, Dr. K. V. Anand"
},
{
"id": 143487,
"tgt": "Could the symptoms of stiff neck and dragging of hands be the symptoms of stroke?",
"src": "Patient: Three weeks ago my boyfriend who is 33 had a stiff neck and we thought he pulled some muscles. We provided him with a better pillow and after a week symptoms went away. Today he went to reach into the cupboard for something and his hand kept pulling to the right instead of where his eyes and brain anticipated by about three inches. After discussion he admitted that he had felt pulling to the right. My fear is that this is related and neurological but his new job does not cover him for insurance for one more week. If these are stroke symptoms we will need to act quickly. Thank you for your time. Doctor: Could be but doesnt seem so this.could be a problem in the cervical spine please get a Mri of the cervical spine done and let s see after the results are in"
},
{
"id": 182375,
"tgt": "What causes gritty feeling in teeth and redness in gums?",
"src": "Patient: Hi, two days ago I started having strange gritty feeling in my mouth, along my inner lips and front teeth. My gums are also suddenly bright red. I don't have any mouth sores or other issues. Any ideas what would trigger that? Even sipping water makes my teeth feel gritty. Doctor: Thanks for your query. I have gone through your query.The gritty feeling in the teeth and the redness over the gums can be because of the gum infection like gingivitis or periodontitis. When there is recession of the gums the dentin of the teeth will get exposed and this results in sensitivity or gritty feeling when you sip fluid or hot and cold things. Nothing to be panic. Consult a oral physician and get your teeth cleaned and use desensitising tooth paste like sensoform or sensodint(strontium chloride and potasium flouride). I hope my answer will help you. Take care."
},
{
"id": 22748,
"tgt": "What does this EKG report indicate?",
"src": "Patient: I would appreciate your interpretation of the following in layman s terms. I am a female in my early 50 s, 5 3 , 121 lbs, healthy, until my EKG results show the following: PR:152 Sinus Rhythm, QT/QTc: 356/381 msec [P:QRS -1:1, Normal P axis, H rate 74], QRSD: 92 msec [-Left atrial enlargement], P axis: 47 [-Negative precordial T -waves, Low voltage - possible pulmonary disease], QRS Axis: 34, T Axis: 40 ABNORMAL . Thank you Doctor: Hello and Welcome to \u2018Ask A Doctor\u2019 service. I have reviewed your query and here is my advice. It's difficult to interpret this report as this interpretation of the ECG machine is also not right. Although if you have symptoms of chest pain, then it's seems abnormal and you will need further evaluation. I would suggest if could upload the copy of ECG, that will be much better.Hope I have answered your query. Let me know if I can assist you further.Regards,Dr. Sagar Makode"
},
{
"id": 198513,
"tgt": "What is the treatment of vas deferens pain ?",
"src": "Patient: good day doctor,I'm suffering from vas deferens pain since some months so I recently got my seminal vesicles injected and my urologist stated that this way also vas deferens, by liquid pressure, would receive the antibiotic. Is that true? because my symptoms didn't change and I recently saw on the net that vas deferens isn't directly connected to the seminal vesicles but both converge into the seminal excretory duct. Am I wrong? Thanks for your time Doctor: HelloThanks for query .It is very difficult to diagnose pain due to infection exclusively of Vas Difference .It is always due to infection of spermatic cord (Funiculitis).Giving injection into seminal vesicle will not give relief from pain due to Funicullitis .You need to take broad spectrum antibiotic like Cefixime along with anti inflammatory drug like Diclofenac along with Diethylcarbamazine (Hetrazan )twice daily for 2 weeks.Dr.Patil."
},
{
"id": 110102,
"tgt": "What causes pain in back and buttocks?",
"src": "Patient: hello, i am 51 yrs old, 5 feet 2 inches in height, 51 kilos and undergone tahbiso surgery last July 25, 2011, i am experiencing joint pains especially back pain on the middle portion of my back up to my buttocks everyday. Is this an after effect of the fall of my estrogen level due to the removal of my ovaries? What can i do about it? Thank you very much. Doctor: Back pain radiating to buttock is one sign of Lumbar disc degeneration and protusion,If the symptom persist more than 6 months. I think you should ask your doctor to do MRI to exclude disc disease."
},
{
"id": 120302,
"tgt": "Suggest treatment for severe pain in tailbone",
"src": "Patient: This morning I woke up with a pain on the right side of my tailbone. The pain is pretty severe. I tried using a heating pad, and even a TENS unit. I also tried stretching exercises. It is making it hard to walk. I am already taking pain pills because of Lupus and Fibromyalgia. The only thing that gives me relief is to lay still. What do you suggest? Doctor: Hello, It looks from your details that you may be suffering from coccydynia.You should do following to have relief in this- -Do warm water sitz bath.For this you have to sit in a tub filled with warm water.Do it at least for three times a day. -Avoid running,hiking or lifting heavy weights.Avoid sitting for long time. -Use coccydynia cushion .You may buy it from market. -Start doing heel stretch exercises & manipulation of heel from your physiotherapist. -Take some like acetaminophen for relief from pain. Hope I have answered your query. Let me know if I can assist you further. Take care Regards, Dr. Mukesh Tiwari"
},
{
"id": 161528,
"tgt": "Suggest medication for fever and throat pain",
"src": "Patient: My toddler has a low grade fever, won t eat, and wakes up crying/screaming multiple times a night. She s a little congested and I think she has a scratchy throat (because I m sick too and have that and she keeps holding her throat when she cries). Does this sound like something to worry over and take her to the doctor? Doctor: Hello, Cough and cold are viral 95% of the times in children. For cold, you can use anti-allergic like cetirizine and for nose block, saline nasal decongestants will do. Paracetamol can be given. I suggest not using combination medicines for fever, especially with Paracetamol.For cold, you can use Cetrizine.For nasal block, plain saline nasal drops will do, every 4-6th hourly to relieve nasal congestion. Hope I have answered your query. Let me know if I can assist you further. Regards, Dr. Sumanth Amperayani, Pediatrician, Pulmonology"
},
{
"id": 94860,
"tgt": "Pain in the stomach. History of abdominal tuberculosis. Lymph node present post treatment. Further treatment?",
"src": "Patient: Hello Sir, I am from New Delhi (India). My wife was treated with relapsed Abdominal Tuberculosis in 2011 and she completed her treatment (Lasted for one year) in the month of Apr-May 11....and post which she gave birth to my second child in the month of Feb 12. Now suddenly she is complaining some sort of strain in the stomach early in the morning hours hhowever during the day there is no such complaints. Once we finished the treatment for lymphnodes in stomach all the nodes were vanished except one, to which the doc said this could remain there and harmless since it did not increase. Could you please help me identifying the further possibilities of treatment? In case the desease reccure. My blessings to you for having such portal. Thanks alot sir. Best Regards Kapil Doctor: Hi, please get a check up with your treating doctor . the pain may be due to some other reason .A clinical check up and investigations may be needed to find out the cause and treatment to be taken accordingly . tuberculosis may relapse but it can only be diagnosed with appropriate tests and the treatment to be taken as per schedule . hope your wife is not having tuberculosis again and be well soon with treatment ."
},
{
"id": 170878,
"tgt": "What relieves the cough in a 4 year old?",
"src": "Patient: Hi, My 4 year kid(Weight 17.5 kg) is having cough for 25 days and we used Sporidex AF 375, twice a day for 8 days along with Relent(2.5) twice a day and Phenergan 2.5 once in night for 8 days. But still cough and flum persists. Now doctor prescribed Azithral 200 (2.5) once a day for 10 days along with Phenergan. Any suggestions please? I think he is having upper bhroncal respiratory .... etc .... Doctor: Brief answer :Pertusis. Detailed answer :Hi, welcome to HCM. Your child is probably suffering from pertusis. Pertusis usually takes time to relieve, usually it takes 2 weeks time. Azithral is correctly prescribed, continue it, i am sure that it will help. Take care."
},
{
"id": 52265,
"tgt": "What causes abdominal pain in a person with fatty liver while on Cipro and Flagyl?",
"src": "Patient: I m taking cipro and flagil for diverticulitis and it is causing liver pain. Should I stick it out for two more days? I ve been taking for eight days as of tonight. I have been diagnosed with fatty liver five years ago. I m 65. Haven t consumed alcohol for over a month. Doctor: Hello, The antibiotic is given for diverticulitis for one week. So as you have completed seven days of the antibiotic course, there is no need to take it anymore. If you have pain in liver area than check your sgpt and USG abdomen scan for further workup. For fatty liver, udiliv can be taken. Hope I have answered your query. Let me know if I can assist you further. Regards, Dr. Parth Goswami, General & Family Physician"
},
{
"id": 141961,
"tgt": "What causes irregular left-sided eye movement and teeth grinding at night?",
"src": "Patient: YES, MY CHILD FATHER IS HAVING SOME ISSUES WITH HIS LEFT EYE AND FACE NOT LOOKING RIGHT. HE HAS BEEN TO SEE PEOPLE AND HAVE BEEN TOLD HE HAS TMJ. SOME SAY HE MAY NEED TO SEE A DENTIST. I AM NOT SURE WHO HE NEEDS TO SEE AND HE IS UNCLEAR. BUT HIS LEFT EYE AND NERVE IN HIS EYES AND FACE DONT LOOK RIGHT TO ME. HE DOES GRIND HIS TEETH AT NIGHT WHEN HE IS SLEEP AND WE KNOW THIS MAY BE THE CAUSE. BUT HOW CAN WE BEEN SURE ITS NOT SOMETHING ELSE? Doctor: Hello!My name is Dr. Aida and I am glad to attend you on Healthcaremagic!His symptoms could be related to TMJ dysfunction (as he is grinding his teeth during the night) or a peripheral facial nerve palsy. Does he close his eyes completely during the night?Consulting with a dentist would help examine his TMJ. If this issues is clarified, consulting with a neurologist would help exclude other possible disorders. As he is grinding his teeth during the nights, I would also recommend checking his stool for parasite infection and performing some blood lab tests (complete blood count, thyroid hormone levels, iron levels). Hope you will find this information helpful!Best regards!"
},
{
"id": 122475,
"tgt": "Suggest treatment for swelling and soreness in knee",
"src": "Patient: Hi, I just got back to tennis after almost 10 yrs. (I am 63 yrs. old and active). My right knee has begun to feel tight behind the knee, my knee is a little swollen after a tennis lesson and there is soreness and just a little pain on the outer right side of my knee. A physical therapist told me I may have strained a labial ligament and perhaps have the start of arthritis or bursitis in the area. He suggested I rest the leg, ice it and take Advil. It does help a lot but then it comes back again now and then after a tennis clinic. Does this sound serious? I m afraid my days of getting back to tennis are numbered. Doctor: Hello, I agree with the doctor that the symptoms are related to tendinitis. I suggest using cold compresses in the area of the pain. I also suggest using Ibuprofen and rest. Hope I have answered your query. Let me know if I can assist you further. Regards, Dr. Dorina Gurabardhi, General & Family Physician"
},
{
"id": 50133,
"tgt": "Suffering from Hepatitis-B, done Nephoctromy. High SGPT, HVB and DNA positive. Taking treatment results negative. Present condition?",
"src": "Patient: HI RESPECTED DOCTOR! For your information i am hepatites B patient and at Feb 2011 my partial Nephoctromy done at right kidney and which was RCC, Two years back my SGPT was 318 and HVB DNA was High Positive and after treatment ( medicine isTelbovidine Group) Last one and half years my SGPT was around 40~50 and last one and half year to till now HVB DNA is Negative and in recent two test SGPT is 108 and 130. i will be grateful if you please advise what should i do now to control SGPT Level and what is my present condition. Doctor: when was your last HBV DNA done and was negative? have you stopped tenofovir or still contiuing it ?suddenly stopping tenofovir even after two years can sometimes increase viral activity and may result in increased SGPT levels.are you taking some other drugs which could have caused a high SGPT."
},
{
"id": 58478,
"tgt": "Liver function test done. Serum alkaline and serum total protein Values given. Is this normal and Treatment methodology?",
"src": "Patient: sir , my liver function test report is, serum bilirubin(total)0.98mg/dl, serum conjugated 0.17mg/dl, serum unconjugated0.81mg/dl, serum g.p.t.(a.l.t)46u/l, serum g.o.t(a.s.t)35U/l, serum alkaline phosphate(37deg.ctemp.)94u/l, serum total protein 7.8g/dl, serum albumin4.6g/dl, serum globulin3.2g/dl, albumin:glbulin ratio 1.4, sir my question is this report is normal or dangours for my health and what is the treatment soultion recovery from this type disease. my age is 31 , please give me the report as soon as in my mail Doctor: Hi. Your LFT report is absolutely normal and you need not worry regarding this. So stay cool and enjoy. Thank you."
},
{
"id": 5769,
"tgt": "Trying to conceive. Started irregular period. How to calculate ovulation date?",
"src": "Patient: Hi. I had menstrual (30 days) cycle every month regularily. After marriage my cycle has been started for 35 days. And I am trying to get pregnant. But every month dissappointing us even though we having sex on my ovulation dates. Now I gotten an another worry that I got periods two weeks early this month. What s happening to me. Now how I should calculate for my ovulate date for this month. My period date was oct 18,nov 22,dec 27,jan 2 and jan 25(two weeks early). (Note:I have been taking folic acid tablet since december,prescribed by Dr.mala ,vijaya hospital). Doctor: Stress could be one of the reasons for irregularity in periods,so one advice is to remain stress free. Secondly get your thyroid test done which might cause irregularity in periods. For knowing your ovulation day we now have kits available ,these ae LH kits (i-sure kits) with simple urine test using this kit you would know when you ovulate. Continue with your folic acid tablets and don't worry everything would be fine."
},
{
"id": 4201,
"tgt": "When should i have sex after taking an injection?",
"src": "Patient: i am 24yrs, last month my periods got delayed (70 days ), after 14 days of my period my follicule was 18 mm, doctor advised to take hcg 5000 iu, So after taking injection how long i have to wait for sex, and what is chance to get pregnancy, i waiting for last three years. Doctor: HAI WELCOME TO HCMYOU have to had sex 32-36hr after injection when the egg will rupture by this time the chances of pregnancy is more.consult an infertility specialist to solve your problem."
},
{
"id": 153255,
"tgt": "Could keeping on oxygen at time of worse health conditions in cancer cause death?",
"src": "Patient: 6 years ago my Mom died. She had lung cancer and was on oxygen at the time of death. She had a seizure or shock at the time of death. She sat up stiff as a board and shook. We put her back down and peace came over her face. I thought this episode was a result of lack of oxygen to the brain. anoxic shock or seizure? I have a sister 45 yr. old dyiing of colon cancer. she was just put on oxygen. Another sister said today we don't want the same thing to happen that happened to Mom. I said, whats that.? she said too much oxygen tio the brain and foaming at the mouth. My mother did not foam at the mouth.. and as I already stated...I rember the doctor saying lack of oxygen;.....we kept her oxygen going as a comfort measure andI I was under the impression there was no adverse affects of keeping oxygen on at the time of death. Please help....this is really bothering me as I was my Mom's health care proxy.....and feel responsible. Doctor: Hi,Thanks for writing in.It is possible that your mother had spread of lung cancer to the brain. There was seizure activity at the time of her death. Many times patients are not aware of the fact that cancer has spread to the brain. This causes electrical discharge in the brain and might lead to uncontrolled seizure activity. This is what happened to your mother and probably she had a severe seizure at the time of her death. It does not look like too much oxygen caused her death. The oxygen saturation does not go beyond a particular level and therefore it is less likely that given her oxygen caused her death.Your sister having colon cancer might have been put on oxygen because the saturation was falling down. This will not cause her death. Unfortunately at advanced disease there is a delicate balance in the functioning of the organs and this requires to be maintained. Please do not worry."
},
{
"id": 5890,
"tgt": "Trying to concieve. Having pink spotting, sore breasts, cramps. HPT positive. Doctor says negative. Suggestions?",
"src": "Patient: Hello, I m 23 years of age...me and my husband is trying to get pregnant so bad. I last time I started my period was August 18th, 2012 lasted for five days. Since then I still haven t had my period...It s now Nov 21, 2012. I ve spotted brownish discharge that appeared once which I thought I was expecting my period, this happened Oct 15th, 2012. I ve had all the symptoms of pregnancy which really I ain t sure anymore if I m pregnant or not...I did took a home pregnancy test Oct 30th, 2012 it came out positive, went to the health clinic there s came out Negative...They told me I had high blood pressure and am over weight....I am 5 7 weigh 241, I lost 10 pounds. This week my breast felt sore plus my nipples...yesterday I had cramps like I m expecting my period...But nothing...today I pee and when I wipe, I noticed a little pink spot on it and my panty had a white milky discharge...I am so confused, as much I am hoping to get pregnant, I ain t sure if I am or not because I get positive and negative test results....What does all this mean to you? Please help. Doctor: Hello, I appreciate your query. Based on your history, it appears that you have not had your period for the past three months, apart from the slight discharge once. Did you have regular monthly menstrual cycles earlier ? If that is so, then pregnancy is a very likely possibility. You have had equivocal Urine Pregnancy Tests so far, hence please do not perform any more. You should visit a gynaecologist at the earliest, for an examination and an ultrasound, which will confirm your pregnancy. It is possible that it is early pregnancy, or an ectopic pregnancy ( pregnancy outside the uterus ).. however nothing short of an ultrasound will tell you for sure. Also, you are definitely overweight, which also might be interfering with your menstrual patterns, in case you are not pregnant, you should definitely try to exercise, eat healthily and lose more weight. If pregnancy is not possible, please rule out other conditions like PCOD ( polycystic ovarian disease ), hypothyroidism etc, which cause such abnormalities of periods. Everything should be taken care of once you visit a gynaecologist for further treatment. Hope this helped you."
},
{
"id": 38458,
"tgt": "Suggest remedy for inflammation post sex",
"src": "Patient: doctor i am 46 year old. For the last two years, whenever i do sex, inflamation starts after 2 or 3 days and afterwards having unbearable itching. If i don t get any medicine than infected discharge travel to anus and it also gets unbearable itching. my doctor gave me AF-kit and inserting tablets. I get relief. but the problem is if again i do sex i get infected. why it is so. what is the permanent remedy please help I am suffering for last two years. It is not easy to eat af-kit after every sex. Doctor: HelloThank You for contacting HCM.First thing that i need to know is that do you have children? Was there any such symptom when you had sex for the first time? What i am observing from your description is that you might be having allergy from sperm(semen) of your partner. Although this is a rare disease but it can cause these symptoms. To prove this you need to visit allergy clinic where doctors can confirm by injecting your partner's semen intradermally. If their is a wheal formation then it will confirm this and further treatment for intravaginal desensitization can be done. If not then they will further investigate the case. Hope this answers your question. If you have additional questions or follow up questions then please do not hesitate in writing to us. Wishing you good health."
},
{
"id": 15558,
"tgt": "Red rash on the back of both of hands, lump in the back of neck, sore armpit. Is biopsy required ?",
"src": "Patient: Hi! I have a red rash on the back of both of my hands, seems minor at the moment but persists for about two weeks now; to compliment (though unsure if related), I have a lump in the back of my neck and a recently new lump-like thing in my right armpit. The one in my armpit is sore and the one on the back of my neck alternates, usually, it is fine unless provoked. Curious for some feedback, not sure if I should seek immediate medical aid, as my Family Doctor has ordered a biopsy on my neck bump/lump since it has persisted for, well, I ve lost count of time on it, but well over a few months. Thanks in advance for your time! Doctor: Hello,Thanks for the query.The red rash and lump in arm pit may not be related.The lumps can be a cyst or a bacterial furuncle.Treatment with antibiotics might be required.A photograph would have helped in giving a closer diagnosis and to decide whether biopsy needed or not.Please meet a dermatologist and discuss regarding this.Let me know if you have any other doubt.you can ask a direct question to me on this forum, following the below link.https://urldefense.com/v3/__http://www.healthcaremagic.com/doctors/dr-rahul-kumar/64818Wishing__;!!Mih3wA!SBzm6_kI6hCZ58EPH6N_05MFfiPbxWXT0a2TJCdFQObRWm5mV5ur7hUOMa8clQ$ you a good health.Thank you"
},
{
"id": 59576,
"tgt": "Metabolic panel blood test shows elevated SGPT, SGOT levels. Family history of diabetes. How to lower levels?",
"src": "Patient: I recently had a metabolic panel blood test done. My SGOT level was 81 and my SGPT level was 144. Of course these are definitely high and a cause for concern. I have never been diagnosed with hep but I am overweight and have a family history of diabetes (I have not been diagnosed). Are these levels extremely high and what can I do to lower them? Doctor: Hi, Thanks for writing in. High SGOT and SGPT levels associated with overweight is usually due to fatty liver. This can be diagnosed by a good ultrasound examination. You need to take care to bring these levels to normal at the earliest. You will need a integrated treatment comprising of exercise, liver medications and dietary modifications. Regards"
},
{
"id": 177023,
"tgt": "Suggest ways to feed newborn other than breast milk",
"src": "Patient: Hi..... I have 3 month old baby girl...... I don t have breast milk from the beginning due to some reason....... I can not give formula milk to baby...... So right now i m giving buffelo milk........ What else i should give her for better nourishment?????....... She has constipation problem may be due to milk but i cant give her breastmilk or formula....... How to overcome it?????......... Please help me Doctor: HIWell come to HCMAs long as the food is concern for baby so breast feeding is the best option and this need to be given only if you do not have enough lactation then you can try, Tab metoclopramide but better to discus this with your gynecologist, hope this information helps, have a nice day."
},
{
"id": 34103,
"tgt": "Can yeast infection medicine be continued after losing mucus plug?",
"src": "Patient: I am 36 weeks pregnant and just had my first physical exam at the doctors yesterday. I have had some spotting since the visit, which I understand is normal. I have been treating a yeast infection for the past 4 days with a 7 day treatment approved by the doctor but this morning it seems I have lost my moucus plug( blood clot/snot egg yolk looking). I am wondering if I can continue the rest of the yeast infection medicine to make sure it is completely cleared up? Doctor: Thanks for posting you query to health care magic.you need to take complete treatment if you have been diagnosed to yeast infection as incomplete treatment causes recurrence of infection . yeast is a fungus and require long treatment to cure so even if you relieved from your symptom take treatment completely to cure remaining subclinical infection .Hope you would be satisfied with my answer . Feel free to communicate if any query .regards,Dr.Manish PurohitInfectious disease specialist"
},
{
"id": 99706,
"tgt": "What causes itching under the skin on hands and feet?",
"src": "Patient: I m 54 year old female and have itching under the skin on hands and feet that is moving up my arms and legs. What could be causing this? I don t have a rash and sometimes it feels like my skin is going to split. All blood test have come back normal. Doctor: hi,see itching under hands or foot can be bcuz of dermatitis,fungal infection etc but a doctor need to see this by his naked eye rather than diagnosing only on the basis of history and description of the patient,so u go to dermatologist and take appropiate treatment.Dont worry u will be alright"
},
{
"id": 135101,
"tgt": "Suggest remedy for tingling sensation in neck",
"src": "Patient: could the tingling I have in back of neck (over many months) cause me to shun light, not want to watch tv or walking exercise ( all these things make my eyes tight and I just want to close them. I have had neurological tests etc and the only thing they have found is low cortisol. thank you Pamela Doctor: Hi Dear,Welcome to HCM.Understanding your concern. As per your query you have tingling sensation in neck . Well there can be many reasons for symptoms you mention in query like cervical spondylosis , radiculopathy or disc problem . Sometimes trapezius muscle also create such problems . I would suggest you to consult orthopedic surgeon for proper examination . Doctor may order CT scan ,MRI or physical examination . Doctor may prescribe nerve supplement along with anti inflammatory and recommend physical therapy and stretching exercises . You can also visit endocrinologist for cortisol problem as it indicates adrenal or pituitary gland problem .Hope your concern has been resolved.Get Well Soon.Best Wishes,Dr. Harry Maheshwari"
},
{
"id": 65289,
"tgt": "What does small lump in the middle of leg indicate?",
"src": "Patient: Hello, I have small lump in the middle of my leg between my kneecap and my shin right near the bone. It's the color of my skin, and is painless. It's only visable when I put pressure on that leg and it's soft (visable due to a slight shift to a whiter hue). When I push it in, nothing happens and there isn't any pain. It seems to completely disappear as soon as pressure is taken off the leg - when I sit down for instance. What the heck is it? Doctor: Hi ! Good evening. I am Dr Shareef answering your query. From your history, it seems that you have got a bursa (fluid filled cyst) over the area which might occur especially those who put chronic pressure on their knees . For a diagnosis, if I were your doctor, I would advise you to get a FNAC (fine needle aspiration cytology) to have a cytological diagnosis and further management depending on the report.I hope this information would help you in discussing with your family physician/treating doctor in further management of your problem. Please do not hesitate to ask in case of any further doubts.Thanks for choosing health care magic to clear doubts on your health problems. I wish you an early recovery. Dr Shareef."
},
{
"id": 39668,
"tgt": "Will nurokind Plus permanently cure Blepharospasm?",
"src": "Patient: Hi, I am suffering from Blepharospasm amd my eyes closes when in stress situation or when infornt of bright light, doctor prescribed me Nurokind Plus and Stematil tabalets, will it cure my condition gradually or will I have to keep taking these medicines for life long ? Thanks, Shyam S Doctor: Dear Friend.Welcome to HCM. I am Dr Anshul Varshney. I have read your query in detail. I understand your concern.No, Neurokind is a vitamin supplement. It is just going to give supportive help.It is required to see the cause of Blepharospasm.I would suggest you:1. To get Retinal Examination2. To get Fundus examination.In case you have any headache, Vomitings, fever associated with it.Please write to us in detail.This is my best advise for you based on available details. If you have any further query please ask me.Stay Healthy.Dr Anshul Varshney, MD"
},
{
"id": 116756,
"tgt": "What causes cold and hot sweats in the night?",
"src": "Patient: Hi, I'm a 21 year old female, active healthy lifestyle, medical history includes thallasemia, ostomalitis and recurrent patellar dislocations but none of these limit my daily activities. I had an appendectomy on Wednesday just gone and have been home since Thursday evening, the surgery went well and I am a lot more mobile these last two days with little pain in my abdomen other than when I cough and laugh. However, I've been very lethargic and having intense cold sweat at night to the point I have to change the blankets etc in the middle of the night, no fever as such but just feeling hot and cold and sweating. Does this mean anything I should worry about or just more bed rest? Doctor: Hi,Thanks for asking.Based on your query, my opinion is as follows.1. Not to worry. Possibly medication induced, as you are already on painkillers which also have anti-pyretic effect.2. Continue your active life.3. Take rest. Be mobile soon will affect the wound healing and may weaken the wound, resulting in hernia.Hope it helps.Any further queries, happy to help again."
},
{
"id": 96955,
"tgt": "Suggest remedy for eye and nose injury",
"src": "Patient: am 27 yrs old, 6ft and 88kg, was playing over the weekend and someone jammed their knee into my eye, got blurred vision and a nosebleed for about 1.5 minutes, all was fine after that, however for the past two days my spitum has mucus and dark blood in it, my doctor said i should not be worried that the blood will stop after sometime, however its now two days and everytime i blink or sneeze my, the back eye feels very uncomfortable as though it is congested and its twitching, plus when i blink i can feel as though am blinking through the nose, should i be worried, or will this go away. Doctor: U have sustained occulo nasal trauma. Based on your history injury looks like trivial to mild.All those blood tinged sputum n blurred vision are just due to congestion. Symptoms will be resolved soon.U can use otrivin or xylometazoline nasal decongestant drops.Lubricating eye drops."
},
{
"id": 124394,
"tgt": "Suggest possible treatment for fibromyalgia",
"src": "Patient: I am living in Bangalore temporarily and have Fibromyalgia. I have been reading that the underlying cause can be due to an under active thyroid. I was doing better but recently began to feel much worse. I don t know what to do or who to see here in the city. Doctor: Hello, I hail from Bangalore. Fibromyalgia what it is must be known to you. Having a good hands-on physical examination can help understand the extent of the fibromyalgia involved in the muscular system. Since you didn't mention anything about which are is more painful it is difficult to put words on how to resolve the issue Hope I have answered your query. Let me know if I can assist you further. Regards, Jay Indravadan Patel, Physical Therapist or Physiotherapist"
},
{
"id": 5589,
"tgt": "Diagnosed PCOS. Trying to conceive. Folicullar study done. Took HCG injection before IUI. Can conceive?",
"src": "Patient: Hi Doc,i am diagnosed with PCOS and i am trying to concieve,my 3rd day follicular study is ROF-6x6mms LOF-5x5mms ET-4mms,my doc prescribed me Ubiphene 100mg from day 3 to day 7 and from 5th day i have to take hMG injection and hCG injection for ovulation after that i will go for my 2nd IUI . Can i become pregnant this time? Doctor: Hello. Thanks for writing to us. There are follicles present in both of your ovaries which are likely to grow further. The ultrasound scan report is favorable. You can continue with the treatment as advised by your gynecologist for successful conception. I hope this information has been both informative and helpful for you. You can consult me again directly through my profile URL http://bit.ly/Dr-Rakhi-Tayal Regards, Dr. RakhiTayal drrakhitayal@gmail.com"
},
{
"id": 29532,
"tgt": "Suggest treatment for abdominal pain while on dengue treatment",
"src": "Patient: My relation 14 years3 days back got dengue fever. Now platelet count is coming down from 250000 to 92000, PCV is 38. today started complaining abdominal pain. vomited once. now has low grade fever. started Iv line.infusion rate is 25 drops/min. Urine out put is normal up to now. Please let me know what should they do? thank you. Doctor: HI, thanks for using healthcare magicThe fluids are very important in the treatment of dengue. He or she needs a lot of fluid.If the platelets drop lower, he or she may need an infusion of platelets to bring them back up.The doctors would also be monitoring for any signs/symptoms of bleeding and would be checking the blood pressure, pulse and platelet count at regular intervals.I hope this helps"
},
{
"id": 155343,
"tgt": "Can i masturbate during rehab after a Robotic Prostecotamy?",
"src": "Patient: Hello, I'm a 50 year old male. I underwent a Robotic Prostecotamy one month ago for Prostate Cancer. My Doctor has me taking Cialis at this time. I had a great sex life and hard erection before surgery, no problems at all. The cancer was detected at a very early stage. I'm wondering if masterbation will aid in my rehab at this time or is it detramental at this point after surgery?? Thank you for your time. Doctor: Thanks for your question on HCM. I can understand your situation and problem. Ideally intercourse and masturbation is not allowed atleast 1 month after prostatic surgery.Since you are having early cancer, I advice to avoid it for 2 months.The reason for this is to promot good healing in operated part.So better to avoid masturbation atleast 2 months post surgery."
},
{
"id": 43909,
"tgt": "Report shows sperm count as 102, motility during 1st hour rapid progressive at 10%. Symptoms of infertility?",
"src": "Patient: Hi , I am attaching my semen analysis report , please suggest -Do I have infertile semen ? . Regarding Thickness , Motility & presence of (PUS Cell + Germ Cell ) please suggest -is it normal . I am 40 yrs old . lately married & truing for baby . Please give some good suggestion. SEMEN ANALYSIS SAMPLE INFORMATION TIME OF COLLECTION 2.25 p.m. TIME OF ANALYSIS 2.35 p.m. ABSTINENCE (IN DAYS) 5 Days PHYSICAL EXAMINATION COLOUR - Off white VOLUME 5.0 ml VISCOSITY Thick REACTION Alkaline pH 7.2-8.0 0 LIQUIFICATION TIME 25 20-30 minutes MICROSCOPICAL EXAMINATION SPERM AGGLUTINATION 1 2 - 3 scale SPERM COUNT 102 millions/ml MOTILITY 1ST HOUR MOTILTIY RAPID PROGRESSIVE 10 % SLOW PROGRESSIVE ---15 % NON PROGRESSIVE-- 25 % IMMOTILE --50 % 3rd HOUR MOTILITY RAPID PROGRESSIVE--- 05 % SLOW PROGRESSIVE ----10 % NON PROGRESSIVE 25 % IMMOTILE---- 60 % SPERM MORPHOLOGY NORMAL-- 80 % ABNORMAL ---20 % OTHER CELLS (per 100 sperms) ROUND CELLS (pus cells + germ cells) 2 - 5 /hpf EPITHELIAL CELLS OCCASIONAL RBCs---- OCCASIONAL FRUCTOSE TEST--- Present With Regards Doctor: Hello, welcome to HCM, Your semen analysis report shows Normal physical examination. Sperm count is within normal limit. Motility of the sperm is lower than required ( normal is >40% as per WHO 2009). Morphology of the sperm is within normal limit ( normal is > 04% as per WHO 2009). Among the other cells, round cells are mildly increased ( normal is occasional as per WHO 2009). Presence of fructose is also a normal feature. So, overall, the semen analysis report shows normal report except the motility and presence of pus cells. Consult with your treating doctor."
},
{
"id": 78834,
"tgt": "Should i be worried about the severe pain in my chest after i cough?",
"src": "Patient: I get crippiling pain (an 8 or 9 on the pain scale) whenever I have a hard cough. The pain occurs between the collarbones and the first rib...right above my pectorals. The pain lingers a good 2-3 minutes after I cough for any reason. Is this something to be concerned about? Doctor: Thanks for your question on Health Care Magic. I can understand your situation and problem. Your pain is considered as severe and significant because it is associated with coughing and lasts for 2-3 minutes. So possibility of lung Infection (pneumonia) is more. Pneumonia is known to cause pleuritic pain. So better to get done chest x ray to rule out pneumonia. If pneumonia is there than you need antibiotic, antihistamine and anti inflammatory drugs. So better to rule out lung infection. Hope I have solved your query. I will be happy to help you further. Wish you good health. Thanks."
},
{
"id": 120533,
"tgt": "What causes joint pain below abdomen while having UTI?",
"src": "Patient: Hi , Im 22, I had a UTI one month back and some vaginal infection. I underwent the entire course of medication and since then im suffering from pain in the joints just below my abdomen. I still have 4-5 pus cells in my urine. I got a abdomen scan done that said normal. My doctor has said that Its not related to the infection and referred me to an ortho... Please tell me what is this ...as I have no idea how to deal with this.. I do not have calcium deficiency and I have a good diet and exercise in moderation. Doctor: Hello, I think your pain is not because of uti or any calcium deficiency. It may be because of any wrong exercise or may be because of cramp which may occur when you lift heavy objects. Did you have any record of doing such activity in past few days please let me know so I can help you. Hope I have answered your query. Let me know if I can assist you further. Take care Regards, Dr. Rohan Sham Deo"
},
{
"id": 132080,
"tgt": "Could impingment syndrome cause weakness in shoulders, back and neck?",
"src": "Patient: I was diagnosed with impingment syndrome in my shoulder, physical therapy was going well until one day even touching parts of my shoulder, upper back or neck area caused me to become lightheaded, sick to my stomach, weak and dizzy. I m 25, 6 foot 2 inches and about 200 lbs. Doctor: Hi you have impingement syndrome in your shoulder and one day you had light headedness ,sickness in stomach weak and dizzy. In my practice I would diagnose this is Vaso vagal Syndrome where pain sometimes causes these features . If problem comes again then i would recommend thorough medical check up."
},
{
"id": 52862,
"tgt": "What causes enlarged spleen?",
"src": "Patient: hi, my teenage brother has an enlarged spleen, he's had it for a long time now, the doctor keeps doing ultrasounds and has confirmed that is is getting bigger. however he can not figure out what is causing it. he has done some blood tests to rule out leukemia. the doctor now says that because he doesn't know what the cause is, he wants just keep waiting and watching out for symptoms, while being careful not to do anything that could make it burst. also i'm not sure if this could be related at all, but he also had his gallbladder removed due to gallstones, and it was because of his gallbladder problems that his enlarged spleen was discovered. Doctor: Hi and welcome to Healthcaremagic. Thank you for your query. I am Dr. Rommstein, I understand your concerns and I will try to help you as much as I can.Enlarged spleen is called splenomegaly and there are various causes possibly leading to this. In first case these are infections such as mononucleosis Parasitic infections, such as toxoplasmosis or Bacterial infections. It means that any infection in your body may lead to spleen enlargement. Second most common causes include autoimmune diseases such as hrombocytopenia. If this is ruled out by specific tests then some other causes may be considered and some are more or less serious such as Leukemia Lymphoma, or someInflammatory diseases such as sarcoidosis, lupus, and rheumatoid arthritis. In every case, infectious causes are treated with antibiotics and other causes may require surgical removal of spleen.I hope I have answered you query. If you have any further questions you can contact us in every time.Kindly regards. Wish you a good health."
},
{
"id": 25512,
"tgt": "How to treat high BP?",
"src": "Patient: Hi, i was diagnosed with High BP in 2006. 160/110; went on medication for 8 months. Then deliberately stopped taking drugs with the alterative of natural remedy and lifestyle changes. Just came back for general check up and my BP reads averagely 160/104. I have been without medication for more than 3 years. Except time to time casual headaches and palpitation (currently) , I have not been feeling that bad. How come, if my BP has been so high for so long? Doctor: Thanks for your question on Health Care Magic. I can I can understand your. In my opinion, you should take antihypertensive drug along with lifestyle changes. Your Intermittent symptoms of headache and palpitations are suggestive of high blood pressure. So you are having this high blood pressure since starting. No harm in taking antihypertensive drug regularly. Continue other natural remedy and lifestyle modifications along with antihypertensive drug to control your blood pressure. Don't worry, take your antihypertensive drug regularly and you will be alright. Hope I have solved your query. I will be happy to help you further. Wish you good health. Thanks."
},
{
"id": 14645,
"tgt": "What causes rash on ankle with pain ?",
"src": "Patient: My 3year old daughter has what looks like a mosquito bite, that has been scratched off on her right ankle and it hurts her to touch it. She also has redness on her left foot, from the bottom of the foot on her heel, and there are red streaks traveling from that to her ankle. She says her legs hurt. What could this be? Doctor: HIWell come to HCMThis is not the mosquito bite but this could be allergic reaction due to hypersensitivity, and this may come around without the medicines, local application of Diphenhydramine can be tried, no need to worry about this, hope this helps."
},
{
"id": 174349,
"tgt": "What causes positive in hair follicle test in a kid?",
"src": "Patient: my fifteen month old grandson came up positive with a hair folicle test, we were told coiuld only go back maybe 60 days. what kind of exposure would have to happen for this to happen. It was apparently high amount. parents claim they never smoked in same room as him. Doctor: Hi,Welcome to HCMIn hair follicle test,Cocaine, Marijuana, Opiates (Codeine, Morphine & 6-Monacteyl Morphine), Methamphetamine (Methamphetamine/Amphetamine & Ecstasy), and Phencyclidine (PCP). These five drug classes are mandated for testing by the Federal Government.Environmental contaminants are the big problem for hair testing. Microscopic amounts of smoke and powders cling to the hair shaft and wind up as a trace amount on a false-positive hair test. To make matters worse, contaminants cling to dark, coarse hair much more readily-making African-Americans, Latinos and Asians significantly more susceptible to a false-positive result. According to one report, people with dark hair are 10 to 50 times more likely to test positive for drug use from a hair-follicle drug test.So in my opinion, if your grandson is not exposed to any of the drugs, then it might be just a false positive report due to contamination.For further questions, do contact us."
},
{
"id": 68102,
"tgt": "What causes lump in back?",
"src": "Patient: I a m a 48 year old female, last week while friving h felt some discomfort on my back left side, the last couple of days my stomich on only my left has been numb and tingling on and off and today I could feel a lump below my left ribs on my backwere the discomfort is Doctor: Hi, dear. I have gone through your question. I can understand your concern. You may have some lipoma or neurofibroma or some other tumour. You should go for fine needle aspiration cytology to check the natura of tumour. Then you should go for surgery according to the diagnosis. Hope I have answered your question, if you have doubt then I will be happy to answer. Thanks for using health care magic. Wish you a very good health."
},
{
"id": 74146,
"tgt": "What are the symptoms of lung cancer?",
"src": "Patient: 3 weeks ago I had a large mole on my back removed, which was discovered to be a malignant melanoma. A few days later I had more of the area removed (Widened by 20mm) and a skin flap to cover the region.The area has healed well, however I am now beginning to experience a shortness of breath (Particularly when I first wake up, it is actually quite difficult to breathe, however improves throughout the day.)I have an appointment with an Oncologist in a few weeks to begin further testing, and was wondering whether this could be related and I should move the appointment forward? I understand breathing difficulties can be associated with the cancer spreading into the lungs, however I would have assumed this would occur in a later stage and I would be experiencing other symptoms?I do also have a sore throat and minor swelling bellow my jaw, so it's entirely possible I just have a \"cold\". I am a 27 year old male. Doctor: Thanks for your question on Healthcare Magic. I can understand your concern. By your history and description, possibility of post infectious bronchitis is more likely for your breathlessness. Bronchitis is very common after viral upper respiratory tract infection (URTI). So drink plenty of fluids orally and keep yourself hydrated. Do warm water gargles 5-6 times a day. You will mostly improve with all these. Since you are also having malignant melanoma, pulmonary involvement should be looked for. So if your breathlessness is not improving in 3-4 days, make an early appointment with your oncologist and get done CT thorax to see whether lungs are involved or not. Hope I have solved your query. I will be happy to help you further. Wish you good health. Thanks."
},
{
"id": 152786,
"tgt": "Can having suffered from cystic lymphangioma result in nerve pain?",
"src": "Patient: I was diagnosed with cystic lymphangioma at the age of 5, I am now 38. 3 years ago i weight 240 and now i weigh 150. I have been in the hospital for 2 years, a doctor removed a large tumor. Since then pain as radiated throughout my whole body. My tumor is growing back extremely fast and I have had radiation, and many different complication with this. They tell me the pain is from the nerve endings, they have me on 34 pills a day. What do you suppose i do? Doctor: Yes this is rapid loss of weight and the pain radiating all over the body is concern. The fact that they have removed your tumour and is growing back again means that the tumour is growing back again means it's malignant tumour with pontentual regrowth It's better to continue with treatment as directed by your doctor and you can seem a second opinion from another oncologist or pathologist."
},
{
"id": 177307,
"tgt": "What causes stomach pain in a 8 year old?",
"src": "Patient: My 8 yr old son keeps waking up at midnight crying and holding his stomach in pain in the upper left part of his stomach. He says it feels like hard poking and pushing. It gets better for a bit, he tries to go to sleep and wakes up a few times more until he can fall asleep for the rest of the night. Doctor: Hello. I just read through your question. The most common reason for the type of abdominal pain you describe in this age group is constipation. Even very mild constipation can cause left sided abdominal pain. I recommend making sure he goes top the bathroom before bedtime and has a bowel movement. If the pain persists, I recommend consulting with your doctor so other methods of softening the stool can be discussed."
},
{
"id": 98023,
"tgt": "Is it OK to give fish antibiotic form of tetracycline for tooth infection ?",
"src": "Patient: my sister has a tooth infection with swelling and fever .... i had the identical issue and was prescribed antibiotic so we know she has an infection and it needs to be treated but shes very poor and also has panic attacks w/ severe anxiety just in general and refuses to go to the doctor or dentist , she is healthy in all other aspects and does not have any allergies to anything including antibiotics of any kind .....would it hurt her to take the fish antibiotic form of tetracycline? and would it help her infection? i ve also researched that people take it 500mg 3 times daily and the fish brand is 250 mg is the dosage for people that i have correct? Doctor: 1. can you please let us know your country of origin because fish antibiotics are not approved by FDA in US for human use and giving them to your sister will be unlawful and moreover without any license. 2. as long as supply of human antibiotics is available, don't go for fish antibiotics. 3. direct evidence is lacking about its safety and suitability for human use, thus talk with your health care professional about the same. 4. most people because they don't have health insurance, or are unable to pay the expenses they just go to pet stores and buy antibiotics meant for fish, as your sister is very poor I think taking fish antibiotic for that matter will be good for her. 5. most importantly tetracycline for fish is used for EXTERNAL diseases while in humans it is used INTERNALLY."
},
{
"id": 31852,
"tgt": "Can chronic hepatitis B infection be transmitted via intercourse?",
"src": "Patient: I'm 27 years of age, 5'5\" Asian ,137lbs. I had chronic hep b.I have chronic hep b and had sexual intercourse with my wife. She had a test on hbsag and the result is negative, so i advised her to have vaccine 1st vaccine Dec.2010, 2nd vaccine Jan.2011 and her upcoming vaccination will be on june 2011. Can i infect my wife if we have sex between months of feb or march.???? Doctor: HiThank you for asking HCMI have gone through your query. There are chances to spread hepatitis infection to your wife through unprotected sex if you are positive. This can be avoided by taking full course Hepatitis B vaccine. Till she complete the vaccination better to have sex by protected way. That is by using condoms.Hope this may help you. Let me know if anything not clear.Thanks."
},
{
"id": 180861,
"tgt": "How can swelling and infection in the lower gum be treated?",
"src": "Patient: I have an infection under a lower tooth on my jaw. I'm taking metronidazole prescribed by my dentist, but there is an apparent increase in swelling under the tooth. Am I in danger of that spreading through my body and making a problem into a nightmare? Doctor: Hi..Thanks for the query.Your explanation is pointing towards tooth abscess formation that is due to deep infection in the tooth leading to pus formation below the root tip of the tooth..It can spread to the soft tissues of the face leading to Cellulitis and Space infection..So my suggestion is to consult a Dentist and get evaluated and a clinical examination should be done along with an IOPAR [x-ray] of the tooth to confirm the diagnosis..You can be advised a course of antibiotics and painkillers to reduce infection and along with it the tooth has to be treated..If tooth can be saved then root canal treatment can be done and if there is gross decay extraction is advisable..Hope this helps..Regards.."
},
{
"id": 162218,
"tgt": "Is corn starch a better remedy than petroleum jelly for treatment of diaper rash?",
"src": "Patient: Don t understand how covering diaper rash with petroleum jelly ( isn t petroleum fuel ) is doing any good , isn t diaper rash more like a boil or a scold and don t you want the area to dry and heal , also wouldn t using corn starch alone be better for the rash , it dries and soothes the pain so it can heal Doctor: Hi, Please do not use corn starch as sometimes it can irritate the area. The best will be to use a diaper cream which has got 15% Zinc oxide in it, please. Hope I have answered your query. Let me know if I can assist you further. Take care Regards, Dr Sumanth Amperayani, Pediatrician, Pulmonology"
},
{
"id": 25656,
"tgt": "What does sinus rhythm and left atrial abnormality in ECG report mean?",
"src": "Patient: hi..i had just an ecg recently and i git a remark of sinus rhythmn and left atrial abnormality..i am a reglar 10 km runner..and a long 21km run on week ends..does this mean any abnormalities of my heart..my RHR is 55BPM...pls any comments on my concern...thanks Doctor: Thanks for your question on Health Care Magic. I can understand your concern. Since you are running and doing regular exercise, no need to worry much for this ecg report. But better to rule any structural pathology of heart because ecg is stating left atrial abnormality. So get done 2d echo. 2d echo is sonography of heart. It will tell you more about heart chambers, pumping, valvular functions etc. It well also tell if any structural abnormality in left atria is present or not. So get done 2 d echo. Hope I have solved your query. I will be happy to help you further. Wish you good health. Thanks."
},
{
"id": 60951,
"tgt": "What do lumps on the lower ribcage indicate?",
"src": "Patient: I have lymphedema and do compression therapy twice a day.For the past 4 years I have noticed thick lumps along my lower ribs that started as just a nuisance. Spoke to a surgeon then and he said he would need to open me up all across my upper abdomen. The are now larger, painful and I have more areas below my ribs. These are painful, they hurt when I breathe, even a gentle touch. I cannot lay in bed on my side, so I have been sleeping in a recliner. I have a history of MRSA and VRSA and have had 12 spinal surgeries, al least 3 were found to have massive infections in my spine and surrounding area. My GP and my pain specialist just blow me off, thinking they are lymph nodes inflamed from my lymphema. No xrays or CT scans have been ordered. I do see my internal medicine Dr. on March 10th. I want to find what these are and what treatment needs to be done. I have a history of uterine cancer in 1985, and both parents died from colon cancer that had spread after several surgeries and chemo. Why do Drs. not listen? I have alot of pain from other issues as my spine and legs, but I think they just thing I'm complaining more....when I am serious and need help, I am age 56 female. Doctor: Hello,Considering the family history of cancer and your previous agony, I vote this condition not to be normal infection.In our clinic we recommend soft tissue ultrasound , CT scan and biopsy of one of the lesion to get precise diagnosis and management in accordance to that only.Hope I have answered your query. Let me know if I can assist you further.Regards,Dr. Bhagyesh V. Patel"
},
{
"id": 10232,
"tgt": "Suggest treatment for less hair growth in moustache region",
"src": "Patient: Hi, Good evening Doctor.One year back suddenly, hair growth stopped in my part(around 2 cms) of my left mustache. I consulted one of dermatologist and he suggested to apply Minoxidil in that area and i used it for two months and I didn't find any hair growth. Because of this I have to shave regularly. sometimes I am getting a feel like insect moving on the skin. Please give me the reason for this and how do I get hair growth in that area again. Please help me in this regardThanks in advance for your advice. Doctor: Hello and Welcome to \u2018Ask A Doctor\u2019 service. I have reviewed your query and here is my advice.In my opinion, you might be suffering from alopecia areata. Do continue minoxidil and it will take 6 months for treatment response. Be patient. You can also use topical corticosteroid cream under supervision of your dermatologist. Use corticosteroid cream only if the diagnosis of Alopecia areata is confirmed by a Dermatologist.Hope I have answered your query. Let me know if I can assist you further."
},
{
"id": 49807,
"tgt": "Recurring kidney stones. Have hypertension, diabetes and gout. PCNL and ESWL done. Suffering from severe pain",
"src": "Patient: I am very much worried about my mother with recurrence of her kidney stones . My mother is having Blood pressure and diabetis and 2 years back she got gout disease and immediately with a kidney stone and had operated with PCNL immediately after that. She is doing fine for last 1 and half year and now few days ago she got kidney stones in both kidneys which are of 6-8mm size.Because of heavy pain she got operated with ESWL in one kidney 15 days ago and preparing for another kidney ESWL operation in a month later. Doctor: hi, Any person who develops a renal stone has 50% risk of forming stone in the next 10yrs. Your mother is more prone for stone formation because of gout. In this condition she will have high blood level of uric acid which is excreted in the urine in high concentration. This leads to recurrent stone formation. If you have the stone with you, get it analysed for composition of stone. Few tips for preventing the stones are 1. Drink adequate water atleast 2-3ltr per day 2. low salt diet 3. She can take urinary alkalising agents like potassium citrate after consulting her doctor 4. Regular ultrasound of abdomen to detect stone formation at its earliest stage. 5. Regular followup with your urologist."
},
{
"id": 160670,
"tgt": "What causes bumps on the thighs of a child?",
"src": "Patient: Hi, my 9 year old daughter has big pimple like bumps on the inside of her thighs. They seem to come to a white head but she can t pop them. Two of them are getting red around them. We have been to her pediatrician and they keep passing it off as molluscum, but I am worried it is somethig different. Doctor: Hi, How long these bumps are there? Is it painful now? Small, round, whitish pimple which was there for few days is likely molluscum. Usually this will disappear its own by few months. If needed, you can ask a dermatologist for removal by chemical cauterization. Don't try popping up, as this will cause spreading and new lesions nearby. But if there is surrounding redness, there is chance of secondary infection. If no pain or swelling around, try applying antibiotic creams like mupirocin twice daily for 3-5 days. See you doctor for antibiotic treatment if not getting better by then. Hope I have answered your query. Let me know if I can assist you further. Take care Regards, Dr. Muhammed Aslam TK"
},
{
"id": 163215,
"tgt": "Could intake of excess fluids be related to hemochromatosis in a toddler?",
"src": "Patient: my 4 year old daughter has been drinking more fluids tha ususual and going to the toilet very frequent about every 15 min. i have had blood tests done and it stated that she should be further tested for haemochromatosis gene.what is this and are drinking fluids and going to the toilet symptoms of this? Doctor: Hello,Hemochromatosis is a genetic disorder in which excess iron deposited in the body in a various organ like liver, kidney, pancreas. It is not related to excessive fluid intake, though excessive fluid requirement may occur in its central nervous system involvement. It is incidentally detected in blood report where high hemoglobin with high iron level will be present.Hope I have answered your query. Let me know if I can assist you further.Regards,Dr. Sachin Kumar Agarwal"
},
{
"id": 56853,
"tgt": "What causes pain in gall bladder?",
"src": "Patient: I am a 73 year female. I have seen my doctor and am having an ultra sound done Wednesday. I have had a little gallbladder pain off and on for about a week. Not a bad pain, very light. I also have pressure in my lower right groin area. Not pain just pressure. Can my gallbladder cause this? Doctor: Hello,The pain in the right lower groin area is unlikely to be due to your gall baldder, I would be more concerned about looking for a hernia or urinary stone or ovarian diseaseAn ultrasound should help to determine the cause.Hope this helpsRegardsDr Samir Patil"
},
{
"id": 181446,
"tgt": "How is an open cavity covered when the crown is not replaced?",
"src": "Patient: Hi i had a crown that did not get replaced as wouldnt stay in. My dentist did not take out the root and just covered the hole with cement. this was weeks ago now and the socket has not healed. Instead there is something hard covering the hole at the top. What could this be and what will happen now Doctor: Hi..Welcome to HEALTHCARE MAGIC..I have gone through your query and can understand your concerns..As per your complain it seems that as the root is not removed then it can act as continued source of infection leading to unhealed socket..Another thing is that if the root is not removed and covered with cement then possibly post and core treatment has been done..I would suggest you to consult an Oral Physician and get evaluated and a thorough clinical evaluation and investigations like x ray can help in diagnosis and treatment can be done accordingly..You can be advised to get a complete removal of tooth done followed by placement of a bridge..Hope this information helps..Thanks and regards.Dr.Honey Nandwani Arora."
},
{
"id": 196630,
"tgt": "What causes itching and rashes on the penis after a unprotected sex?",
"src": "Patient: Dear doctor, I m happy to find this site. I don t really know what is wrong with me, my story goes thus. First, I am a Tboy. I will be 19 this year. I never have sex till last year August, a month before I became 18. It was unprotected, and that is the only sex I ve had. Fee days later, I started seeing rashes on my arms, I got confused, I washed the bed spread (which is the same we used while having the sex) it disappeared but I wasn t the same anymore. I felt itches on the tip of my penis so I took a natural medicine produced here in the country (Nigeria) I think it stopped. I thought that was the end, with time, I started having strange feelings on some one of my bone, joint or muscle once in a while. I don t know how to explain it, it wasn t painful neither was it making me tired but it was as if [it was somehow rusting ] I hope you understand. I feel sometimes even till today. A month or two ago, I started seeing rashes on one side of my groins only for me to to start discovering some grain-like solid spots or whatever on my penis, when I managed to force the pus out of one, it was white and solid. It seemed to me that somehow, the skin of my penis had been like that for some time before my first sex, but not this much now. It is so noticeable. Also, a few weeks or there about after the sex, I started seeing one or two red spots on my body, like about three to five on one lap not really swollen nor painful or itchy not only on the lap but, some,other parts of my body although rare. Currently, I m confused and getting afraid. Before my sex, I ve been diagnosed of staph I don t know whether this I contracted Syphilis, HPV, HSV or my greatest fear... You know. Thanks alot I hope You ll reply soon. Tboy Doctor: Hi,Fungal infection on penis is the most common cause of itching and Rash after having unprotected sex. If possible, please send us the photograph for property valuation as differential diagnosis also include human papillomavirus, Herpes simplex virus and recurrent folliculitis. Hope it helps. If you have any other question please do not hesitate to contact us.Regards,Dr. Atishay Bukharia"
},
{
"id": 96561,
"tgt": "What causes swelling and numbness in the knees after an accident?",
"src": "Patient: I was in an accident almost six weeks ago on a scooter where I hurt my knee there was a deep scab and bruising and it felt numb for 4 weeks. I had acupuncture on the knee which seemed to irritate it and now it\u2019s swollen up like water on the knee but it doesn\u2019t hurt unless i bump into something. The feeling has returned also. My question is does this swelling seem like a normal part of the healing process after it being numb. Doctor: Well come to HCMThank for asking to HCMI really appreciate your concern, I could say that the given condition could be due to accumulation of blood (Hematoma ), imaging study can confirm the condition if this is so the it may resolve on its own but care needs to be taken for infection, it is not the normal condition, but as long as the differential diagnosis is concern it may be something else than the blood collection, it may be abscess formation, it may be escaped synovial fluid, better to see the orthopedic surgeon hope this information helps"
},
{
"id": 126458,
"tgt": "What causes weakness in the legs when diagnosed with idiopathic myopathy?",
"src": "Patient: i have been diagnosed with idiopathic myopathy i have a history of lupus which is well controlled with palquenal . my symptoms include difficulty standing from a sitting position, inability to run or jump, difficulty climbing stairs, and occasional numbness ,tingling in feet, difficulty gettng up from a crouching position. i am awaiting further testing from a neuologist. my concern is my legs seem to be gettng weaker but i am very active walking and working out at gym 4 times a week. what can i expect as time progresses nd numbness in my feet and lightheadednessto jump or run, Doctor: Hi, Consult a neurologist and get evaluated. Hope I have answered your query. Let me know if I can assist you further. Regards, Dr. Shinas Hussain, General & Family Physician"
},
{
"id": 46844,
"tgt": "Recommend treatment for kidney failure with high sugar levels",
"src": "Patient: Sir, my father is 68 years old & have CKD, both the kidneys have failed ,have sugar level high & on dialysis. Last few days he become rest less, not even sleep for a moment, severe body pain,movement of limbs is stopped or even very pain full,his Hb at present is 5.6 & we have tranfuse at least 12 unit blood in last 3 months ,pl. advise what ido . Doctor: Hello and welcome to HCM.As an Urologist and transplant surgeon,i can understand your anxiety.Your father has end stage kidney failure,which can be treated with dialysis or a kidney transplant.Anemia is usual in CKD and low blood level, is treated with a packed cell transfusion,erythropoietin injection,and oral iron tablets.His Hb,creatinine,FBS,A1C,LFT,platelets,and Na,K,is done with 3 days gap.His Hb,sugar,creatinine and electrloytes must be under strict control.If you've any doubts,send all the reports to me,as a direct question.Dr.Matthew J. Mangat."
},
{
"id": 192282,
"tgt": "What causes pain in my testicle after hernia surgery?",
"src": "Patient: Sir, I had undergone a Hernia laproscopy surgery 3 weeks back. after that am feeling slight pain on my testicles. is it an after effect of that operation or do i need to consult a doctor? if so which specialist? Iam 41 yrs, 171cm ht, 65 kg wt. no pre, no suger. Doctor: Hi, Some times after hernia surgery there is testicular pain. It may persist for couple of months. If pain increases in intensity than you should consult a urologist. Hope I have answered your query. Let me know if I can assist you further. Take care Regards, Dr. Ashish Kumar Khandelwal"
},
{
"id": 154497,
"tgt": "What does pain & swelling in ankles in patients with multiple organ tumor suggest?",
"src": "Patient: My mother has one kidney after nephrectomy due to renal cell carcinoma. She is out of state on a trip and ct has revealed mutiple organs with tumors as the cancer has spread. She was released from hospital thursday and we have a flight for tomarrow to bring her home. She just called and said her ankles are swelling. What do i do? Can she fly? Emergency room? Is she in renal failure? She is weak and tired as cancer is taking over. I am a 5 hour plaine ride away, help Doctor: Thanks for your question on HCM. I can understand your situation and problem. In my opinion your mother should avoid flight journey. There are many causes for her ankle swelling and pain. But the common causes are1. Renal failure 2. Hypoproteinemia 3. Liver metastasis 4. Heart failure due to heart metastasis. So better to consult doctor and get done renal function test, liver function test, serum protein level, 2d echo to rule out above mentioned causes. Prognosis in her case is very poor as she is having metastasis in other organs. And ankle edema will further deteriorate her life expectancy. So better to consult doctor and first treat her."
},
{
"id": 161046,
"tgt": "What causes pus cells in urine test of a child?",
"src": "Patient: Hi! My 2year and 9 months old boy urine test resulted pus cells with range 8-11 and our doctor has suggested for urine culture and abdominal scanning. He mentioned that boys shouldn t have pus cells in urine. Is it a serious problem for the boys having pus cells in urine? Doctor: Hello, It could be a urinary tract infection. Go for a urine culture and start antibiotics according to the culture results. Nothing much to worry and most cases respond well to antibiotics. Hope I have answered your query. Let me know if I can assist you further. Take care Regards, Dr. Shinas Hussain, General & Family Physician"
},
{
"id": 46292,
"tgt": "Suggest treatment for pressure on the kidney and urinary incontinence while suffering from lung cancer",
"src": "Patient: Hi doctor, I would like to inquire about some discomforts. Starting from a month back, I can feel dull pain (like pressures) on kidney area (back waist and sometime sides waist area) after sleeping for around 5hrs onwards at night. However, no issue during Daytime or when I'm not lying down. I also feel that I have a little shorter tolerance for urine management (when i feel the urge to visit the loo, it's usually already at a quite urgent stage. This is not my usual self. Could this be any potential medical issue that needs attention or maybe just aching due to a less active lifestyle? Thank you for your attention.P.S: I've no prior kidney or bladder issue. Currently under medical treatment (Iressa) for lung cancer and was told that this drug doesn't cause any side effect on other organs. Doctor: hi, As per your description it seems like some bladder incontinence which is very mild. Actually at night time we don't void frequently so there is pressure build up in bladder after 5-6 hours and due to some incontinence, there is a urgency in urination that cause some discomfort in kidney area due to back pressure. there is nothing much to worry about. You just take Tab Solicept/ Soliten 10 mg at bed time daily for 15 days. You definitely feel some relief.thank you"
},
{
"id": 222994,
"tgt": "Can VBAC be tried prior to dermoid removal?",
"src": "Patient: I have a 1 cent. Dermoid on my right ovary found via ultrasound. I am 16 weeks pregant. I am really wanting to attempt a VBAC this time. I have only had 1 previous c-section. but my doc is talking about just scheduling a c-section to remove the dermoid at the same time. Could I still try for a VBAC and just remove the dermoid later after delivery?(if it remains small of course) Doctor: yes u can do it but the safer option will be to go for another caesarian section, removal of dermoid,and ligation at same time"
},
{
"id": 137016,
"tgt": "Suggest treatment for hip fracture in patient with blocked heart arteries",
"src": "Patient: Yes, thank you. My father had a stroke over 20 years ago. He was a miracle, they said as 2 arteries were completely blocked and only 3% of 1 artery open. They were unable to do any surgery on the damaged part of his heart. Well, he has lived way beyond we ever thought. Question: He fell this evening and broke his hip. It is not a clean break and demands surgery. However, the cardiologists at the hospital are too concerned due to his heart and are waiting for his personal cardiologist to evaluate him. What course would seem reasonable? Doctor: Hello,I can understand your situation.Since your father has a triple vessel block it would be a very high risk surgery for the hip fracture.A very high risk consent will be taken from you even if the cardiologist gives fitness for surgery.You have not mentioned the age of your father but i would assume it to be above 70,since he had an heart attack 20 yrs bck.Till the time the crdiologist gives fitness i would suggest traction with 3 kgs on the affected limb. Usage of pain killers to relieve tension.Aspirin or any other blood thinning drug should stopped five days prior to surgery.If surgery is not possible, bed rest with traction is advised."
},
{
"id": 126430,
"tgt": "What causes leg pain after hiking?",
"src": "Patient: I have had small lumps in my legs for a few years and was told they are benign and related to veins. They have a slight blue tint. About a month ago after a several hikes in the snow my leg started hurting and walking is difficult. A couple of the bumps are painful. Doctor: Hi, It\u2019s simple musculoskeletal pain and will settle by itself. As of now, take adequate bed rest and take analgesics like Acetaminophen or Tramadol for pain relief. If symptoms persist, better to consult an orthopedician and get evaluated. Hope I have answered your query. Let me know if I can assist you further. Regards, Dr. Shinas Hussain, General & Family Physician"
},
{
"id": 186003,
"tgt": "What could a greyish white film with red dot post a molar extraction be?",
"src": "Patient: Hello, I had a molar extraction in my upper right side on Tuesday of last week. On Friday, I went back because I had a dry socket and was inserted a gauze with anesthesia. Today I had to go back and get the gauze removed- the assistant scraped the top part of the extraction site to make it bleed again (it was the most painful experience), it bled, and was rinsed and cleaned out, inspected by the dentist, and I was sent home. Now, in the evening, I have a grayish, white-ish film in the extraction site and a dark red dot or hole right next to it. Is this normal? Please advise. Doctor: Thanks for using Health care magic.Read your query.It can be the healing site after applying medication to the socket.Take the antiobiotic as your dentist has prescribed for you.If You smoke avoid it as it will prevent healing.Don't carry on vigorous forceful gargling . Do warm saline gargling.If the pain is persistent or the swelling is increasing ,consult back to your dentist.Hope this was helpful.Thanks and warm regards."
},
{
"id": 223569,
"tgt": "Should I take emergency contraceptive again after taking it, having periods and unprotected sex?",
"src": "Patient: Hi am 23 years old,68kg height 5 4. I took emergency contraceptive pills 7th of march and got my periods which started on16th of march,I had unprotected sex on 28th of march,is it advisable to take emergency contraception again?is my cycle back to normal?what should i do? Doctor: Hallow Dear, Emergency or post-coital contraceptive pills give protection against the pregnancy if the pills are taken within 72 hours after the unprotected sex. These pills do not offer any protection against pregnancy from the unprotected intercourse performed after the consumption of the pills. Hence for such intercourse after the pills, the dose of the emergency contraceptive pills needs to be repeated within 72 hours of that sex. In fact, emergency contraceptive pills should be taken only in an emergency situation and its routine use as a regular contraceptive is not advised. If you are engaged in a regular sexual activity, I would advise you to adopt some regular routine safe and reliable contraceptive method like condom, oral birth control pills or condom. I hope this clarifies your doubts. Dr. Nishikant Shrotri"
},
{
"id": 142786,
"tgt": "How does it take to brain damage without right amount of oxygen supply?",
"src": "Patient: Hi. I hope you can me. I have a brother with lung problems. he is having problems menitally. The hospital has sent him home. They say he is not having trouble with his heart. He is on a breathing machine but menitally not doing well. How long without the right amount of oxygen will there be brain damage? Doctor: Hello.I am Dr Mittal.I have read your message.I need to know exactly what is the mental problem.To answer your question- it depends on the severity of lack of oxygen.After 1 minute, the brain cells start to die.After 3 minutes, the damage is usually serious.After 5 minutes, the damage is usually major.After 10 minutes, the damage is usually irreversible.After 15 minutes, recovery is usually not possible.Best of luck.Feel free to contact us for more information on the related issue.Dr Mittal"
},
{
"id": 77877,
"tgt": "What causes pain near my ribs?",
"src": "Patient: I have sharp pain on my right side near my rib. When my bladder fills up all the way my right side starts to hurt. When I go to the bathroom 1 or #2 the pain go down some, when I don t drink water it feels the best. do u know what might be the cause of this pain? Doctor: Hi. I can understand your concern. You could be having kidney stones. You will need a detailed evaluation like ultrasonography , consult a urologist for the same.Don't worry, you will be alright. Hope I have solved your query. Wish you good health. Thanks."
},
{
"id": 31761,
"tgt": "Suggest remedy for ringworm infection",
"src": "Patient: I used to be a competitive wrestler and have had Ringworm hundreds of times. I have not wrestled in years and just broke out in four spots. I'm getting married in one week and would like to kill this by then if possible. I just started to apply Lotrimin about an hour ago. Can I do anything else to quicken the process? Doctor: Hi..Welcome to HEALTHCARE MAGIC..I have gone through your query and can understand your concern..As per your complain Ringworm is a fungal infection of the skin and is treated with topical antifungal agents while if you want to get treated faster you can take oral antifungal medications along with topical treatment..You can continue applying Lotrimin [Clotrimazole] while you can also apply ketoconazole and miconazole containing topical medications..You can start taking oral antifungal medications that are effective in resolving infection faster like Griseofulvin, ketoconazole, fluconazole etc..Keep the infected area dry and clean and do not let moisture to accumulate to prevent spread of infection..Wear loose fitting clothes and preferably cotton fabrics..Avoid sharing clothes, towels etc with other individuals..Hope this information helps..Thanks and regards.Dr.Honey Nandwani Arora.."
},
{
"id": 130198,
"tgt": "What causes tightness in the knees post knee replacement procedure?",
"src": "Patient: I had total knee replacements, both knees, approximately two years ago with great success. Today, however, I am experiencing a tightness and some pain in my knee joints. Is it possible to experience gout in prosthetic joints, or what other ailment may be the culprit? Doctor: Hi,GOUT IS NOT CULPRIT BUT LACK OF PHYSIOTHERAPY IS.ALSO GET XRAY OF BOTH KNEES STANDING DONE IN AP AND SEND ME THE REPORT.IN ADDITION TO PAINKILLERS ADD amitriptyline tab.SO DO PHYSIOTHERAPY DAILY.THANKS."
},
{
"id": 57274,
"tgt": "Suggest treatment for abdominal pain and pus cells in liver",
"src": "Patient: MY BROTHER IN LAW IS IN GREAT PAIN AND UPON A READING OF THE SCAN REPORT, THE DOCTOR TREATING HIM HAS SAID THAT HE SUSPECTS THAT HIS LIVER MAY BE HAVING PUS CELLS. DESPITE THE MEDICINES PRESCRIBED, HE IS IN GREAT PAIN. WHAT IS THE BEST TREATMENT IN SUCH A CONDITION ? Doctor: Hi thanks for your query....first let me clarify to you that pus cells in liver suggest liver abcess predominantly of infectious origin....such an abcess is evaluated first with the help CT ABDOMEN AND ELISA IGM FOR ENTAMOEBA ANTIGEN.LATER ITS TREATED INTIALLY WITH TAB METRONIDAZOLE 750mgm po tid for 21 days.generally 70 to 90 %abcess resolve by this time...in case an abcess has not resolved with this therapy,then surgical drainage of the abcess is done under ultrasound guidance.please consult an expert hepatologist for further queries.thank you."
},
{
"id": 218492,
"tgt": "How can paternity of an unborn child be confirmed?",
"src": "Patient: Hello, please my friend av been sleeping with 2 guys, she use postinor2 after sex with first guy after sex and does not take postinor2 after sex with the other guy bcos he his her husband, wen she is pregnant ,is it the guy or the husband that impregnated her, Pls reply Doctor: Hello and Welcome to \u2018Ask A Doctor\u2019 service. I have reviewed your query and here is my advice.In first time she had used emergency contraception pill. So pregnancy from that guy seem less likely But to confirm paternity genetic testing should be done Take care Hope this will help you"
},
{
"id": 17794,
"tgt": "What is it called to have upside down heart?",
"src": "Patient: At the age of 52 after falling out at worked and being taken to the hospital I found I have an upside down heart. Half of the blood pumped to the heart drains back into the main artery before it is moved on. All these years I thought I was having mini-heart attacks. What is the name for this condition and why isn t it recognized for disability benefits. Doctor: Hello, After going through your medical details I understand your concern and I would like to tell you that you require a proper cardiac evaluation and a few medications to avoid regurgitation consequence. Kindly get an Echocardiography done and consult a Cardiologist personally for physical evaluation and treatment. Hope I have answered your query. Let me know if I can assist you further. Take care Regards, Dr Bhanu Partap, Cardiologist"
},
{
"id": 104523,
"tgt": "18 year old suffering from asthma and allergies. What would be triggering it?",
"src": "Patient: Hi there. I am an 18 year female who has asthma . The thing is, I can t point my finger on what causes my asthma attacks. I get them in weather changes, when I am being physically active, and sometimes just randomly in the middle of the night. I also have severe allergies. I don t smoke . But I was wondering what you think might be triggering them.. ( I get about 4 a week, and almost every morning around 2) Doctor: With my experience it may be food allergy either milk,chana,potato,which exagerates on changing season and temperature variations you can get blood serum specific antibodies test for these food items"
},
{
"id": 92625,
"tgt": "Stomach pain, shift to abdomen. Diagnosed heart burn, prescribed prilosec. Taken hydrocodone and naproxen to control pain",
"src": "Patient: For the past couple of months I have been experiencing pain in my stomach. This pain came about 4 times a week and it lasted approximately 2 minutes. It was excruciating, but it passed. When I spoke with my doctor she stated it was probably heartburn and recommended using prilosec. Ive tried that plus other over the counter meds. I do take alot of tums as well. Last Thursday I was experiencing that pain and then the pain went to my upper left side of my abdomen, where it has been constant sharp pain. Ive taken naproxen and some hydrocodone. It mainly takes the edge off. I do have an appt on wed as that was the soonest available. Is there anything else you could recommend for pain until my appt Doctor: youmust investigate it seems from your statement that your doctor said it is heartburn means that he didnot find any problem but still confirm from your doctor for gastritis duodenitis gall bladder liver or digestive enzyme problemif nothing you can consult allery specialist doctor to get blood serum tests for speciic antibodies for milk wheat potato and common food you takeas the food protein reaction with body protein have same type of problems after detection you ithdraw those for good results"
},
{
"id": 99754,
"tgt": "What causes productive cough with brown/green mucus?",
"src": "Patient: Hi, I have a really productive cough, its especially bad when i lay down, i sometimes sound like i'm drowning and choking, i constantly have to cough to clear my airways or i have a 'bubbling' sound when i breathe. The mucus is light brown/light green. Doctor: Hi, since how long u are having productive green or brown sptum,u have not mentioned which is important in every case.anyway it always suggests some infection in lungs like acute bronchitis or pneumonia or rarely COPD but u need to see pulmonary physician and get your sptum examined before u start taking antibiotics. Don't bother with full course of antibiotics u will be alright."
},
{
"id": 129858,
"tgt": "What causes painful head, neck and chest after an injury?",
"src": "Patient: i was snowboarding and hit my head , blacked out for a few seconds , had blurry vision for about and hour , but my head still hurts my neck, back and chest hurt and i feel sick . should i wait , and see if i feel better tommorrow before seein a doctor Doctor: Hi,Immediately visit emergency room.It shouldn't be ignored.There the doctor will do your phys examination and act accordingly.He may advise you CT scan or MRI scan depending upon your phr examination.Thanks."
},
{
"id": 107977,
"tgt": "Suggest treatment for severe back pain",
"src": "Patient: My husband is having severe back pain. He aeriated our lawn on Sunday and the pain started Monday. But he has had kidney stones previously so we are just trying to figure out if it might be that as it seems to be getting worse instead of better. What is better to take for back pain? Ibuprofen or Naproxin? Doctor: I would not recommend to take either of these with history of stones until or unless you can't move.There are better and harmless options available for managing pain and stones both. Safe, economic, no side effects alternatives in Ayurveda may prevent you from catching side effects of pain killers and dissolving stones and expeliing without surgery ( depends on size and location of stones). The worst may be no response of these herbal medicines.Gokshar guggul, Cystone/ Neeri tablets, Panchakol herbal powder ( five herbs group) decoction prepared at home and Rumalya and / or Mahanarayan oil have the qualities needed for your treatment . Little turmeric in warm milk or water is of added advantage. Strict medical supervision is not required for it. Only if you can find Ayurveda doctor near you, he can guide you in selection and procedure of medicines out of above or else. If you could know the details we can guide on net itself.You have always open option to go back to toxic drugs if you do not like naturality of Ayurveda.There's one more opportunity. On 21 June World Yoga Day will take place all over the globe. Get 35 minute package training free of cost. Follow it later. Be disease free.Hope it helps you to explore natural pain and stone management options."
},
{
"id": 73481,
"tgt": "Suggest treatment for MDR-TB",
"src": "Patient: hello doc, my son age 23. wt 49kg height 5'9 inch engineering stduent . He is suffering from mdr tb last four year,he was infected in mumbai during his education he was direct infection case of mdr tb. According to his sensitivity test he is sensitive to kanamycin , ethide .pas and cla rithromycin we are giving these medicine along with pza and ethambutal after six months kanamycin was stoped. But he is not cured still his culture is positive. Please doc. suggest how he will be negative which medicine should be added to his regiment and how long he should take the kanamycin injection. Doctor: Thanks for your question on Healthcare Magic.I can understand your concern.Persistent positive culture even after taking MDR (MultiDrug Resistance) TB treatment is suggestive of possible XDR (extremely drug resistance) TB or TDR (total drug resistance) TB.So better to get done sputum culture and sensitivity for XDR TB.He might need capriomycin injection, moxifloxacin, high dose INH, rifabutin, cycloserine, moxifloxacin etc.No need to give kanamycin injection now. First rule out XDR TB.Hope I have solved your query. I will be happy to help you further. Wishing good health to your son. Thanks."
},
{
"id": 195654,
"tgt": "Does masturbation daily cause any health issues?",
"src": "Patient: HAI, IAM 27 IAM A REGULAR MASTURBATER RIGHT FROM MY 18, BUT KNEW THAT THE NERVES ARE GETTING WEAKEN, SO RIGHT NOW IAM MASTURBATING AND IMMEDIATELY AFTER THAT TAKING METHYLCOBALAMIN TABLETS OF ABOUT 3000MCG TABLETS, AND GETTING RELIEF FROM THE PAIN I HAD WHEN I MASTURBATE.I WANT TO KNOW MASTURBATING IS WRONG DAILY. Doctor: Hello and Welcome to \u2018Ask A Doctor\u2019 service. I have reviewed your query and here is my advice. It\u2019s a myth that masturbation is harmful and weakens the body. Do not repent after masturbation. Take it easy. You may take vitamin E cap daily. Take Anxiolytics like Fluoxetine daily to reduce anxiety. Hope I have answered your query. Let me know if I can assist you further. Regards, Dr.Ilyas Patel MD"
},
{
"id": 54002,
"tgt": "Can actibile 500 treat enlarged liver?",
"src": "Patient: I am a hepatomegaly patient.Docter has given me actibile 500 twice a day before food for two months.Is it proper diagonosed for treatment?I am 50+,height 170cm., weight 65 k g. Suffered stomach pain particularly in empty stomach for last two months. Doctor: Hi welcome to health care magic... You are having hepatomegaly and stomach pain.. And given ursodeoxycholic acid... But I want to know your reports to give further comment.. This drug given in gall stone and fatty liver mainly... Your work up done with.... -USG-Liver enzymes estimation sgpt and SGOT -serum bilirubin-peripheral smear with retic count etc.. According to cause further treatment can be guided.... Take careAdvise : Consult physician for examination and further work up...."
},
{
"id": 146116,
"tgt": "Suggest treatment for tingling sensation in feet and legs",
"src": "Patient: Hi my husband 43yrs old diabetic has had high blood sugars this evening. Just a few minutes ago his left side went tingling including his tongue. He said this tongue has a metallic taste. I took his blood pressure and it was ok & he did take some baby aspirin. Blood sugars are coming down now and the tingling feeling is diminished to both feet and shin area. Doctor: Hello madam,I understand your concern about your husband.A diabetic patient can have tingling sensation in his body as a result of nerve damage due to high blood sugars,but the pattern of these sensations is usually chronic, symmetrical in both feet extending upto legs(sometimes upto thighs and trunk also) and both hands extending upto forearms(sometimes arms and face also).But this is usually symmetrical and chronic.In your husband's case it appears to be an sudden episodic, sharply demarcated to one side of the body including face and tongue(which is unusual).Being of sudden onset and restricted to one side of the body followed by recovery,these sensory symptoms mandates a neurological consultation to rule out a vascular event like a transient ischemic attack or even stroke(although,sharp demarcation of sensory abnormality including face and tongue is rare in stroke).Improved and now limited symmetrically to both feet and shin area,is something i mentioned above and well explainable with diabetes.I hope it will be of help."
},
{
"id": 177391,
"tgt": "What causes white bump on tip of tongue?",
"src": "Patient: My daughter was born with a small white bump in the tip of her tongue. I took her to the ENT doctor and they told me it's my go away by itself. Today I noticed it's growing a little be since last 2 months. I'm a little worry and want to know what could that be? Doctor: Thanks for your query, I have gone through your query.The white bump over the tip of the tongue can be a irritational fibroma or a lipoma secondary to trauma or injury to the tongue. Nothing to be panic, consult a oral physician and get it evaluated. If it is interfering with function then get it removed surgically.I hope my answer will help you, take care."
},
{
"id": 146769,
"tgt": "How to treat spinal stenosis?",
"src": "Patient: what is the best treatment for spinal stenoisis I am 5'9'' and67 years old. My weight is 167lbs. 15 years back I had two epidurel shot. Last July I had again one more shot. One month I went for a Aurvedic treatment. Is it good? or what I have to do next. Doctor: the treatment depends on your goal as a patient. the only cure for spinal stenosis is surgery but this is only to relieve the symptoms caused by compression. there are usually many other related issues that cause pain such as disc herniations and facet arthritis that will not be fixed by the surgery. complete pain is an unrealistic goal. you should make a goal such as \"to be able to walk a mile\" or \"to play golf\" as these are treatable goals for which medications can be tried. you can't get a back transplant so pain relief is unlikely. if the aurvedic treatment helped then continue doing it. same thing chiropractors and medical massageetc. the only absolute need for surgery is loss if bowel or bladder control"
},
{
"id": 133670,
"tgt": "Do water pills increases gout risk?",
"src": "Patient: I am a female age 63 , have a gout attack and unable to get to doctors for RX, what can I use to get me back on my feet, have been bedridden a few days, using walker now, but unable to get down stairs. Also isgatorade helpful for gout. I take water pills that increase gout risk? Doctor: hithank you for providing the brief history of you.A thorough musculoskeletal assessment is advised.As you have gout, it is due to increase in the uric acid levels in the blood stream and the pain is majorly in the foot.Undergoing physical therapy for the same will provide instant pain relief and get your functional ability to get active. Also, drinking more water and passing more urine should help further.In my clinical practice I see gout cases and majority of patients respond well to physical therapy.RegardsJay Indravadan Patel"
},
{
"id": 87771,
"tgt": "What causes sickness, abdominal pain, tiredness, hair fall and abnormal liver bloods?",
"src": "Patient: i have been having abnormal liver bloods since i have been pregnant.My little girl is now 4 months,with my first i had no problems.It started out with being sick ,abdominal pain,tireness.I was on steriods. My consultant doesn't know what i have. yesterday i had a biopsy my Alt was 1040 its been gradualy going up since i stopped the steriods my hair is falling as well.I'm clueless as to what i have and so is my doctor. Doctor: Hi.Thanks for your query and an elucidate history.I have read and understood your problem , which is related to the high liver function tests which started since second pregnancy. Pregnancy induces liver function derangement in many patients but get corrected automatically after delivery. I would suggest you to go for MRCP as there may be an underlying problem related to the liver, gall bladder , pancreas or the bile ducts. The treatment will be according to the report and will be specific. This may be a medical treatment or a surgical one if something like a choledochal Cyst. gall stones or such things are found on the investigations."
},
{
"id": 116970,
"tgt": "What does it mean by Hemoglobin-12.5, TLC-9,200 in a blood report?",
"src": "Patient: Hello Doctor I am suffering viral fever and i have all ready take a blood report here i am mention please tel what is exactly problem with me and what should be avoid Hemoglobin --- 12.5 TLC ----- 9,200 DLC NEUTROPHILS----70 LYMPHOCYTES---------26 MONOCYTES------------02 EOSINOPHILS-----------02 BASOPHILS------------00 E.S.R - ------------------28 PLATELET---------------1.52 L SERUM BILIRUBIN TOTAL -----1.0 SGPT (ALT)------------------------25 THIS IS MY BODY RESULT Doctor: Hi, dear. I have gone through your question. I can understand your concern. Your hemoglobin, total wbc count and differential count are almost normal. Your platelet count is at low normal range. Your esr is slightly high. You you may have mild infection. No need to big worry. Just take treatment according to your doctor's advice. You will be cured eithin 2-3 days.Hope I have answered your question, if you have doubt then I will be happy to answer. Thanks for using health care magic. Wish you a very good health."
},
{
"id": 58178,
"tgt": "Chronic pain under rib cage. Colonoscopy, endoscopy normal. Ultrasound shows fatty liver. Medication?",
"src": "Patient: Current issue is chronic pain in upper right GI area just under rib cage. Colonostomy normal, endoscopy normal except some diverticulitis, ultrasound of GI normal except fatty liver; gall bladder function tested at 92%. Doc says 92% is normal so gall bladder is fine. Online resources disagree that 92% is normal. Some say it should be less than 85% or it is considered hyperactive / hyperkinetic. Which is true? Also could the constant pain be coming from my liver? My PA says your liver has no nerves so it can t hurt. I am hurting and need help. What should I do? Doctor: Hi I read carefully all you concern and I can say that your pain is not from fatty liver.yes you may have a hiperkinetic gallblader and it is associated with abdominal pain. your chronic pain may be related to bowel (large colon )spasms,So you should eat bit and frequently, and please try to avoid stress. All the best DR.Klerida"
},
{
"id": 217773,
"tgt": "What causes chest and neck pain?",
"src": "Patient: Hi, I have a question, In July I had a bad feeling in the center of my chest, it was painful and felt like a bubble expanding and popping causing me to burp. Their was slight pain in my back around the same area and the chest pain, and sometimes i felt a sharp one secound pain on the left side of the neck. This lasted for about a week. Any Ideas of what it could be? Doctor: Such psins usually are a result of prolonged untreated acidity because of acid peptic disease leafing to oesophagus inflammation. This leads to intermittent swelling of lymph nodes in the left side of neck. To confirm you need an endoscopy to be done."
},
{
"id": 18130,
"tgt": "What causes elevated blood pressure levels?",
"src": "Patient: husband was in ER last night with BP 215/110. took dose of clonidine and it slowly came down so they sent him home. Was seen by a PA in small hospital. This evening took BP and it is 227/94 sitting and resting in chair. He doesn t want to go back to ER just to get sent home again. Takes clonidine 3xqd BP shoots up high then comes down after pill. Then shoots back up again before next dose. Doctor: Hello and Welcome to \u2018Ask A Doctor\u2019 service. I have reviewed your query and here is my advice. If the single molecule is not working in hypertension then second molecule can be added or changing the drug is the trend physician are doing. The best and effective combination are diuretic and betablocker, it gives excellent result and you can discuss with your physician for that. If patient does not have any symptoms then it is nothing to worry. Reading the blood pressure by non medico person with electronic machine may give much error. Blood pressure always needs to be read by doctor with authentic sphygmomanometer. Hope I have answered your query, let me know for further assistance."
},
{
"id": 103293,
"tgt": "79 years. Feeling of cold in head, cough, sneezing, fatigue, lungs filled with fluids. Prescribed nasal spray. Advice",
"src": "Patient: I am 79 years - in January this year spent five weeks in Samuir Island Thailand followed by five weeks in England due to feeling I was ill went to doctor there who prescribeed antibotics - since return have consulted doctor here who has prescribed nose spray as head feels I am most of the time full of cold - have harsh cough and sneezing often -feel hav e build up of fluid on lungs overnight and much fatique (usually very energetic person) Doctor: Hi, Thanks for using HCM.The symptoms of cold, cough and sneezing may be due to allergy, may be allergic rhinites. You can continue nasal spray with maoetasone furoate for its management with paracetamol as anti inflammatory and alaspan as anti histamine drug For feeling to have fluid in lungs get done one chest xray and echo to rule out any major problem. Or else the above drugs will help. Have adequate regular sleep, plenty of fluids, nutritious diet with required calorie and regular outdoor exercises for your wellbeing. Hope I answered your question. Consult your doctor for further management.Feel free to ask me if you have any further queries.Wish you good health. Take care.RegardsDr. Lohit K"
},
{
"id": 216707,
"tgt": "What causes intermittent pain between shoulder blades, back pain and stomach ache?",
"src": "Patient: I have been experiencing pain intermittently for a month or two... sometimes just once a week, sometimes more frequently. The pain usually begins in my upper back between my shoulder blades and increases over a quick period of time, ending in upper and lower back pain, more like a strong ache. And then also in my gut/stomach area below my ribs... all at once. I feel nauseaus. it usually occurs in the evenings and lasts into the night. At times it carries over into the next day. I am wondering if it is an ulcer or something more serious. It has increased this past week... Doctor: hi,thank you for providing the brief history of you.As you have pain in the upper back, shoulder blades and lower back you need a thorough clinical Musculoskeletal exmaination.As most aches are muscular in nature but post examination we can come to a conclusion if the pain is related to the pinched nerve.Also, such cases in my clinical practice responds well to physcial therapy.RegardsJay Indravadan Patel"
},
{
"id": 89186,
"tgt": "Why am I suffering from bloating, headaches, nausea and bad stomach cramps?",
"src": "Patient: I ve been having really bad cramps in my stomach the past week, with headache, nausea, I can t eat without getting sick to my stomach. I also am bloating and I recently lost 60 pounds in the past 2 months without trying. I didn t change my diet at all. Can you help me figure out what s wrong? Doctor: Hi.Loosing 60 pounds in the last 2 months is a very serious matter . IT is possible tht you are suffering from Cancer, tuberculosis or such a disease which needs urgent admission to a hospital, investigations of blood, urine stool, CT scan of abdomen, Enteroclysis for intestinal obstruction, Diagnostic Laparoscopy and SOS procedure. The treatment will depend upon the findings."
},
{
"id": 112463,
"tgt": "Pinch nerve, vomit, left side nagging pain. With MRI, esophagus cap did not open/close properly. Help?",
"src": "Patient: Hi, awhile back was in urgent care of a pinch nerve had an MRI done and showed that my esophagus cap did not open and close properly. I do vomit almost every other night there is no blood and I try to note what I'm eating that makes it happen and try to eat early.. I also been suffering left side nagging pain hard to pin point exactly where but feels constantly there. Tired of vomiting and this irritating pain. Help! Doctor: Hello. Thanks for writing to us. The pain in the left side could be related to repeated vomiting that you are having due to improper esophagus cap closure. You can consult a gastroenterologist for treatment.I hope this information has been both informative and helpful for you. Regards, Dr. Praveen Tayal ,drtayal72@gmail.com"
},
{
"id": 84282,
"tgt": "How to get rid of the itching caused by cortisone?",
"src": "Patient: I just got a shot for poison ivy, this happened last year as well. My poison ivy is gone and now i have this itching feeling all over my whole body, way worse then pioson ivy, i think im allergic to i think is cortisone, what can i do to get this out of my system and get past this? Doctor: HiCortisone is a steroid.The main role of steroid is reduce inflammation of any cause.Steroids reduce allergy due to all causes.Hence it is not possible to be allergic to steroids.Any preservative added to steroid may rarely cause allergy.Another brand of steroid can be used along with antihistamines.Hope I have answered your query. Let me know if I can assist you further. RegardsDr.Saranya Ramadoss, General and Family Physician"
},
{
"id": 137373,
"tgt": "What causes pain in ribs and on back side?",
"src": "Patient: I have pain in my ribs and back on my right side. I have had this for about a week. If I take pain relief it dull the pain for a short while . Then it starts again. My right side is very tender to the touch. And I can t lay on that side without it becoming very painful. Doctor: your symptoms suggest that you have soft tissue injury over right side of chest may be due to an unnoticed trauma. these injury takes at least 15-20 days to heal.Rest, icing, local application of non irritant analgesic gel and anti-inflammatory analgesics such as ibuprofen will decrease pain and swelling. deep breathing exercise will also be helpful. Hope this helps. If you have additional questions or follow up questions then please do not hesitate in writing to us. I will be happy to answer your questions. Wishing you good health."
},
{
"id": 68006,
"tgt": "What could the strange bump near the thigh be indicative of?",
"src": "Patient: On the inside of my thigh up near the groin theres this strange bump...it used to be painful now not so much...tonight some liquid came out...it was like a clear light yellowish color....I ve had these bumps before...but this is a bit larger than what I ve had before Doctor: Hi, dear. I have gone through your question. I can understand your concern. You may have some enlarged inguinal lymphnode due to infection or you may have tuberculosis with abscess formation or some other lesion. You should go for fine needle aspiration cytology or biopsy of that lump. Then you should take treatment accordingly. Hope I have answered your question, if you have doubt then I will be happy to answer. Thanks for using health care magic. Wish you a very good health."
},
{
"id": 123129,
"tgt": "What cause swollen and painful knee after exercise program?",
"src": "Patient: I am 37 and recently started an exercise program, (cycling jogging and plyometrics) I have been working out for about a month and am starting to get sharp pain behind my knees when i walk , also notice that they are sometimes swollen for an evening after i work out. what do i do? Doctor: Hi, In your case, there are few possible causes and if there was no injury then some other chronic disorder may be present. Without more information, it is not enough to determine the diagnosis. It could be a hematoma (bleeding in the tissues) or a swollen knee joint capsule. If it's in the back of the knee then it could be a Baker's cyst. Clinical examination (and perhaps fluid aspiration if there's any) would help to determine the diagnosis. Either an orthopedics specialist or your primary care physician can do that. Until then you can try to apply ice packs (not directly on the skin - wrap them with a towel) and perhaps use some painkillers starting with paracetamol (acetaminophen) and advancing to NSAID if paracetamol is not enough and there's no contraindication for NSAID. Hope I have answered your query. Let me know if I can assist you further. Regards, Dr. Ivan R. Rommstein, General Surgeon"
},
{
"id": 179583,
"tgt": "What causes sudden deafness and dumbness in a child?",
"src": "Patient: My daughter is deef and dump after unknown sickness doctors says she is fine some psychiatric problem is become just one before two months she was perfect and going school.her body become shaking for almost 30 days ,there was no fits diagnosed now she is ok but cant speak and listen,rarely she speak and hear. Doctor: HiThanks for writing to health care magic.Brain infection called as.meningitis or encephalitis can cause such effects.Get an MRI brain and EEGWishing your child good healthRegardsDr Arun"
},
{
"id": 109818,
"tgt": "Suggest remedy to relieve pain and tenderness in back",
"src": "Patient: dear doctor i am 24 year old boy with low back pain and tenderness in l5s1 area. mri shows mild posterior bulge at l5s1 level.this is effacing epidural fat and indenting ventral theca. i am on aceproxyvon and muscle relaxant. pain improved in 2 weeks . none in morning. after car ride comes back. sometimes diziness for second or so when i am walking. sometimes leg weakness for a second or so. i am on active rest.my neurologist says bulge minimal n cant be cause of pain since it has been there for last 3 years. this time pain came after heavy wt lifting. please advice. Doctor: Hi, I had gone through your question and understand your concerns. Back pain after weight lifting may have too reasons: 1. Soft tissue (muscular) damage2. Herniated disc diseaseIf dhe MRI that you report is done after weight lift and shows only bulging, then there is soft tissue damage, rest and pain medication should improve the pain, if MRI is done before the weight lift maybe that bulging was transformed to herniated disc disease and you need another lumbar MRI to confirm it. The treatment options depend on MRI findings. Hope this answers your question. If you have additional questions or follow up questions then please do not hesitate in writing to us. I will be happy to answer your questions."
},
{
"id": 116957,
"tgt": "What causes giant platelets with carcinoma?",
"src": "Patient: hi doc, I m a woman who was tested +ve br.carcinoma in 2009, I had 6 cycles of chemo. Now I am on harmonal therapy from last 2 yrs. from last 1yr n 8 months i was not having periods.. I had undergone D&C in March 2011 coz of endometrial thickness. Now again I am to be operated for removal of my uterus n ovaries coz of heavy bleeding. I have done pre surgery tests day before n the reports say m having GIANT PLATALETS. I m worried very much pl. help...... Age: 37Yrs wt: 62 kgs ht: 5\"4 Doctor: Hi, dear. I have gone through your question. I can understand your concern. You have giant platelets. Don't worry about that. Giant platelets may be seen in many normal individual. If your other parameters are normal then you can go for surgery. It has no significance with normal count. Hope I have answered your question, if you have doubt then I will be happy to answer. Thanks for using health care magic. Wish you a very good health."
},
{
"id": 218924,
"tgt": "Is Macgest-SR safe to be taken during pregnancy?",
"src": "Patient: My wife is pragnent of 8 month 10 days but my doctor prescribe macgest sr 200 till now. doctor prescribe this tablet from starting of pragnancy. this tablet is good for at this time or not. and how many time this tablet take in pragnany time. starting three months? starting six months? please tell me. Doctor: u can take dis medicine as soon as discount pregnancy is confirmed in normal pregnancy without treatment till 12 weeks.1 tablet twice daily...its safe in pregnancy..in case of infertility treated pregnancy u can start after ur doctor says ovulation has occurred."
},
{
"id": 199909,
"tgt": "Suggest remedy for low sperm count",
"src": "Patient: i check my sperm two to three times but no sperm found ...i have no any problem in childhood but when i masturbate i saw some thing like sperm before marriage.. i am married ( 1 year ho gaya hai).some specialist say to eat permen Tablet 3 months,... but i eat 1 month becoz of two expensive it is..in one report shows that dead cells are there... i though i have low sperm count productin. wht u have to suggest me... and my testicle size are small so is that reason of low or no sperm count..i eat Gonadil f tablet 2 months continuosly..evion also Doctor: HelloThanks for query.You seem to be confused about low sperm count (Oligospermia ) or No sperm (Azoospermia).Have you done your semen analysis and what has been reported .Absence of sperm indicates that there is no sperms are being produced by Testicles. whereas Oligospermia indicates the quantity of sperms (Sperm count)is less than normal range .Please repeat your semen analysis to confirm the same .If your testicles are small you need to consult qualified Urologist for clinical examination and hormonal assessment to rule out hypogonadism which may be the basic cause of absence of sperms in your semen .Dr.Patil.."
},
{
"id": 147066,
"tgt": "Could fever, nausea, mild headache, dizziness and red itchy bumps on hands be meningitis?",
"src": "Patient: I have fever, nausea, mild headache, dizziness and red itchy bumps on my hands. Is this a sign of meningitis? I went to a gp a week ago to check (did not have the rash at this point and symptoms were milder) and he said everything was OK (blood pressure, heart rate and no ear infection). Doctor: HIThank for asking to HCMI really appreciate your concern and looking to the history given here I would like to say that this could be viral infection and this would take some time to come around and during this period the condition can be managed with symptomatic medicines and the best option is Tab Acetaminophen and Tab Ibuprofen three times in day, have a nice day."
},
{
"id": 91478,
"tgt": "What treatment is useful for severe stomach ache and fever?",
"src": "Patient: I have woke up yesterday morning with extreme nausea, headache, severe chills. I haven t vomited or had diarrhea. But I had a 100 degree fever. Everytime I eat something today, my stomach hurts a bit. I have some Doxycycline left over from a malady few months ago. Will it kill a stomach bug? Doctor: it is infection of stomach best would be metronidazole course as it act fast on stomach infections for fever you can use mefanic acid tab sosadding antacid gel tds will relief your stomach from hurtuse plenty of fluids no oily greesy and fatty food for some days"
},
{
"id": 44441,
"tgt": "I was exposed to lot of heat and vibrations, will that cause reduce in sperm ?",
"src": "Patient: hey I m a marine engineer .as because of my job i was exposed to lot of heat and vibrations for long time. Is this will cause reduce in sperm count ? what test shall i do.plz help Doctor: Excessive heat near pubic area, vibration and tight underwear, using very hot water for bath can also result in low sperm count and motility. The test that analyzes semen is \"semen analysis.\""
},
{
"id": 104726,
"tgt": "Has severe allergy cold. Underwent surgery for adenoid and tonsils. Difficulty breathing. Why?",
"src": "Patient: HELLO SIR My son 12 years old is suffering from severe allergy cold for the past 4 years.last year he underwent a surgery for adenoid and tonsils.for the past 4 years he is under medication and takes levocetrizine tablet daily.but now he suffers from severe block in the nose and finds very difficult to breathe.now again he is suggested to go for a laser surgery in the nose for sinus and bent in the nose bone.we are helpless now.please suggest us the correct solution.please Doctor: Hello dear,Thanks for writing at HCMAs your son is only 12 year old.and last year he went for surgery and now in present if he has very much difficulty in breathing due to internal structure of nose the medication alone will not help.If their is a severe block then it could pressure on his other veins damaging it.So it is better to go to a specialize,who could diagnose the actual problem and suggest you betterHowever if your child goes after the doctor say for surgery then make sure it is done from specialize person in that field .But you dont need to worry, these surgeries do not have much complications and now days done successfully without any complication but a proper study of that individual case or condition is neededThanksDr M.IqbalPsychologist & Psychiatrist"
},
{
"id": 124995,
"tgt": "What other remedies apart from compression stockings are advisable for swelling in the legs?",
"src": "Patient: I have swelling (let leg = 17 , right leg = 16 ) I am 5 and weigh 195. I have tried to get 20/30 compression stockings on and it is impossible (83 yrs old). Is there any home treatment equipment that I could use that would pump and massage my legs to help this situation? Doctor: There are such lower limb air massage equipment available, which can be worn like a trouser, and when connected to the plug, with electric driven motor, the air flows and causes compression and a massage effect on the limbs, This is helpful to reduce swelling of the lower limbs caused by venous and lymphatic edema. Once the legs loose the swelling, you could wear the compression stockings."
},
{
"id": 49610,
"tgt": "USG results increased cortical echogenicity, proteins, granular casts, in urine analysis. Do I have kidney problem?",
"src": "Patient: hi.. my USG shows that i have increased cortical echogenicity . i have also had proteins and granular casts in my urine analysis. my cortico medullary differentiation is shown to be fair. i don have any other abnormality in my urine analysis except for some pus cells. i feel nauseous also.Please let me know if i have serious kidney problem.. Doctor: Hello user,First of all the presence of proteins, granular casts, in urine is NOT normal finding, unless it is correlates with the clinical findings.Your USG report also says some structural anomaly in your kidney.It would have been of more help if I had got your age and Diabetes or Hypertension history, and the blood Sodium Potassium and Creatinine values.Hope my advise helped you.RegardsDr Hari Om ChandrakarMBBS"
},
{
"id": 125459,
"tgt": "How to treat shin injury?",
"src": "Patient: i got hit directly in my shin by a team mates knee but it wasn t so serious. However the next day there was a bruise and when i rub the bruise there is a sharp stinging pain that moves down my leg to the lower part of my shin. what is this injury and stinging pain i have? Doctor: Hi, Pain, swelling, redness etc. are classic signs of inflammation. And trauma is a cause of inflammation. Unless it is significantly impairing function, it will go on its own. At best you need analgesics and if there is severe pain and limp, you may get an x-ray done. Hope I have answered your query. Let me know if I can assist you further. Regards, Dr. Syed Tehmid, General & Family Physician"
},
{
"id": 164702,
"tgt": "What causes bluish discoloration of face in a child on antibiotics?",
"src": "Patient: Just got back from the doctors and my 15 month old has strep. Just started her on the antibiotics and baby motrim. Her hands, arms and then face turned blue for a couple minutes then went away then came back slighty and went away. Is this chills from the fever or a reaction to the antibiotics? Doctor: HiHands , feet , ear , nose may turn blue due to cold temperatures, chills. If lips and youngsters are turning blue get her evaluated."
},
{
"id": 48879,
"tgt": "Could the masses found on the pancreas indicate a benign?",
"src": "Patient: Hi...my name is Gail...Just had a CT of the kidneys done because of blood in my urine...I have no pain but my Dr suspected kidney stones..they found a mass on my pancreas.....I scheduled an MRI for tomorrow morning...My daughter, a nurse, researched it and said that 75% of the masses found on the pancreas are benign......Any truth to that? Doctor: HIThank for asking to HCMI really appreciate your concern and I would like to say that most of the mass or growth found inside the human body are benign and very rarely these turns into malignancy so this is nothing to worry, hope this information helps you, take care and have a nice day."
},
{
"id": 16306,
"tgt": "Stinging pain in the groin, on prednison, had taken Kellogg shot for poison oak rash. Reason?",
"src": "Patient: I recieved a bad case of poison oak around my body, mostly in the groin area so i went to my doctor to get some help. they gave me a Kellog shot of 80mg and also put me on prednison. the rash and poison oak has gone away for the mostpart it looks like but i still get a very strong stinging pain in my groin that is preventing me from sleeping. Is this something i should be worried about or is it the result of my medications? Doctor: Hi welcome to H.C.M.Forum. the attending doctor would have asked you to come next day. as there is no complete remission you have to use antibiotic to prevent secondary infection, antiinflamatory analgesic for pain relief, anti histamine to reduce toxic effect.under the supervision of the doctor. thank you."
},
{
"id": 223452,
"tgt": "What is the effect if one takes pill after unprotected sex forget to take before sex ?",
"src": "Patient: i am on cerezette contraception and missed one pill yesterday and had unprotected sex on the same day where my boyfriend ejaculated inside me and my calnder tells me im fertile just now what are the chances of being pregnant? i took my pill after the unprotected sex Doctor: if u miss one pill..u have to take two pill within 24 hrs...i mean(d same day pill n next day pill)otherwise use other modes of contraception for next 7 days...pills provide 99 percent protection...if u didnt take 2 pills within 24 hrs ..wait for ur period...den do urine pregnancy test for confirmation"
},
{
"id": 41816,
"tgt": "Suggest treatment to attain pregnancy",
"src": "Patient: I have had two ectopic pregnancy in the past two years. My first ectopic pregnancy was located in my right tube, however the tube was salvaged. The second ectopic pregnancy was in the left tube and it was not salvaged. The doctor told me that my remaining tube is now partially blocked. He said I am a candidate for IVF. I have researched IVF and I have found that it is only about 25 percent affective which causes me to worry. I would like to know what to do next. Should I get a second opinion? If so where do I go and what do I do? I'm so confused and saddened that I might not be able to have children. Doctor: Hi welcome to healthcaremagic.I have gone through your question.There is no reason to worry at all. As in this era there are so many options available to conceive with this reports also.In ivf invitro fertilization you might get success in 1/2 cycle , it happen so many times if it is done in good hospital and in good set up.Search for best hospital in your town and go there for it.Hope i answered your question.Would be happy to help you further.Take care."
},
{
"id": 125497,
"tgt": "Suggest treatment for neck muscle spasm",
"src": "Patient: dear dr i have being diagoinised with cervical spine shows loss of normal lordosis due to spasm of neck muscles anddisc space reduced at level c5/c6 andlumbar spine also shows loss of normal lordosis due to spasm of back muscles and disc space reduced at level L5/S1 BODY OF L1 SHOW MINIMAL LOSS OF HEIGHT ANTERIORLY. NOW I AM GETTING CENTER AND LOWER PAIN AFTER GETTING WAKE UP IN MORNING PL SUGGEST THE MEDICINE AND EXCERCISE THANKS SURYA GARG. Doctor: Hello, As a first line management, you can take analgesics like Paracetamol or Aceclofenac for pain relief. Most probably it will be due to conditions like cervical spondylitis. If symptoms persist better to consult a neurologist and plan for an MRI scan. Hope I have answered your query. Let me know if I can assist you further. Take care Regards, Dr Shinas Hussain, General & Family Physician"
},
{
"id": 171396,
"tgt": "What is the treatment of cold in children ?",
"src": "Patient: My 7 year old daughter got a cold last week. Me and my son have the same cold but our has not progressed the way hers has. She has had a fever every morning with a headache, sore to move her eyes (mostly her left eye), and a dry cough. She started antibiotics 3 mornings ago but last night she started getting leg cramps. This morning her fever was up to 103 and her legs are very sore. She has a dry cough that hurts her throat and seems to have some laryngitis when she talks. I gave her motrin and her fever went down to 100.7. I have had her stretching her leg muscles to try to ease the pain from them cramping. I am concerned because her symptoms seem to be getting worse instead of progressively better Doctor: Hi...Thank you for consulting in Health Care magic.Fever of few days without any localizing signs could as well a viral illness. Usually rather than fever, what is more important is the activity of the child, in between 2 fever episodes on the same day. If the kid is active and playing around when there is no fever, it is probably viral illness and it doesn't require antibiotics at all. Once viral fever comes it will there for 4-7 days. So do not worry about duration if the kid is active.Cough and cold are viral 95% of the times in children. For cold you can use anti-allergics like cetirizine and for nose block, saline nasal decongestants will do. Paracetamol can be given in the dose of 15mg/kg/dose (max ceiling dose 500mg) every 4-6th hourly, that too only if fever is more than 100F. I suggest not using combination medicines for fever, especially with Paracetamol.For cold you can use Cetrizine at 0.25mg/kg/dose every 12 hourly for 3 days.For nasal block, plain saline nasal drops will do, every 4-6th hourly to relive nasal congestion.Hope my answer was helpful for you. I am happy to help any time. Further clarifications and consultations on Health care magic are welcome. If you do not have any clarifications, you can close the discussion and rate the answer. Wish your kid good health.Dr. Sumanth MBBS., DCH., DNB (Paed).,"
},
{
"id": 20696,
"tgt": "How long will it take strep to damage heart valves?",
"src": "Patient: my father has been going down hill health wise for the last 6 mths maybe even the last year . we took him to the hospital this last time and with in 24 hours they said he had a streap in his bloodstream and it has caused heart valve damage . what I want to know is this an infection that he had for a while or is this something that can happen in 24 hours . I just think it would take time for this to happen to the heart and being that he has be declining for a time now could he have before we took him to the hospital? Doctor: Hello Welcome to HCM. Blood is usually sterile I.e. devoid of any bacteria or viruses. Bacteria or viruses can appear in blood after a throat infection, urine infection or lung infection. Once the bacteria enter the blood they can infect other organs including the heart. The bacteria settle on the valves of the heart and cause infection and destruction of the valves. This happens over time and cannot happen in 24 hrs. But he must have had symptoms like fever, breathlessness, shivering etc. You have not mentioned any of the symptoms. here. The treatment of infective endocarditis is long term treatment with antibiotics and some residual damage of valve always remains. Wishing him good health Regards"
},
{
"id": 192474,
"tgt": "Suggest treatment to stop the addiction of masturbation",
"src": "Patient: Hi, may I answer your health i am 19 year old boy .i doing hand practice twice a week or some time thrice a day . i cant control it and it is an addiction to me and length of my penies is 5inch it is normal or small . how can i stop it .......queries right now ? Pl Doctor: Hello, Masturbation is considered normal, natural and healthy. Your habit of thrice a week masturbation is never harmful at all. So please do not worry. Therefore, I suggest consulting a psychologist for physical examination, diagnosis and treatment. Hope I have answered your question. Let me know if I can assist you further. Regards, Dr. Sameen Bin Naeem, General & Family Physician"
},
{
"id": 43713,
"tgt": "Difficulty in conceiving, advised fertyl. What are the advantages of this tablet?",
"src": "Patient: hello hi doctor,i want to ask that i married befor 3 years but still i m not getting pregnent becoz of irregular periods so my doctor advice to take fertil..so it is helpfull me or not or whaT IS THE ADVANTAGES OF FERTIL TABLET MY HUSBUND IS ALSO TAKING BECOZ HE ALSO GOT SOME PROBLE.....IN SPUM NOT MOVING SO PLZ ADVICE SUTIALBLE TREATMENT REGARDING FOR THIS THANK Doctor: Hi Mrs Bhakti naik, As you are correctly responding it is the correct time to be perticular about conceiving. as your doctor has correctly diagnised there is possibility of P.C.O.S. which causes irregular periods and infertility. But there are other causes as well, as insulin sensitivity which can be corrected by drugs controling diabetis. If present it should be corrected first to get successful correction of the P.C.O.S. Fertyl (clmid) is a good choice for correction of anovulation, besides it is also important to take good food rich in zink, iron, protiens, multivitamins, and minerals. Take more of green leafy vegetables, pulses, fruits, juices, milk, meat, chicken, and egg. Fnally mating recommendations have sex between 11th and 20th days. avoid having sex upto 10th day especially male partner. Male above pattern is preferable. Should not be washed immediately after the completion of the sex to be fertyle. wishing for a quick and fruitful result, and thanks for calling."
},
{
"id": 44801,
"tgt": "What medicine should I take to increase the ovulation ?",
"src": "Patient: I AM 33 YEARS OLD , WHEIGHT 52 KG , I HAVE LOW FERTILITY PROBLEM MY OVALUATION IS WEAK I GOT PREGNANT 2 TIMES BUT GOT MISCARRIAGE , PLEASE SUGGEST ME AMWAY MEDICINE TO INCREASE MY OVALUATION ( EGGS ), COMPARETIVELY MY EGGS ARE IS VERY LOW QUANTITY Doctor: hello, welcome to HCM i think you have problem called luteal phase deficiency. you may have some other problems like infection or genetic abnormalities of baby. so you have to consult a gynecologist for the cause. if you have problems of ovulation then drugs like clomiphen is availabe. so first find out cause.it may not be ovulation problem take care"
},
{
"id": 168400,
"tgt": "What causes fever and IBS in an infant?",
"src": "Patient: my daugther is 4 and ahalf months old and has been having a fever for 4 days of 38C and above and also she has a bowel movement very often about 10-12 dirty diapers a day with change in color and mucous . I m worried that she could have amoebiasis or something similar. is it possibel she has it Doctor: Hi,Welcome To Healthcare Magic.I can understand your worry regarding to your child\u2019s health.I have seen number of patients of this type. So, no need to worry for this and I would advise you to go for laboratory investigation of stool R & M. After getting the report, please share it on healthcare magic for proper diagnosis and treatment.You have to give some medicines till you will not go for laboratory investigations such as syrup Zifi (if no allergy), 50 mg, 2.5 ml, twice in a day, at the interval of 12 hours, for 5 days. Also add syrup Metrogyl (if no allergy), 2.5 ml, 3 times in a day, at the interval of 7-8 hours, for 5 days. Start giving maximum liquid to her by adding ORS tetra pack (2 packets daily). After completion of whole the treatment, if you will not feel better, then go for consultation of pediatrician.After doing all these things, I hope your child will get well sooner and please share your review about my answer on healthcare magic.Take Care,Thank You.From:- Dr. Nupur Shah"
},
{
"id": 190504,
"tgt": "Excessive brownish color saliva in mouth, bitter taste, unpleasant breath. Quit smoking. Reason?",
"src": "Patient: I am 50 years old and have been relatively healthy all my life, but over the past 18 months months since quitting smoking I wake up every morning with excessive saliva in my mouth (brownish color) and a very bitter taste that lasts all day. My wife tells me my breath is unpleasant even though I clean, floss, and gargle, several times a day. Could this be something serious ?. Doctor: Hi, I read your problem. See , cause of brownish saliva i mouth is presence of nicotine in mouth.Usually smokers may find that their saliva remains brown, even if if they quit smoking, as their lungs rid themselves of cancer causing nicotine. Firstly, you should consult a doctor to rule out potentially dangerous disease.And unpleasant breath is due to sulphur produced by anaerobic bacteria.So, also visit your dentist for oral prophylaxis. Use antibacterial mouthwash also. Have a nice time.. take care"
},
{
"id": 102794,
"tgt": "Took adderall, tongue feels swollen, swelling below adams apple, hurts. Is this an allergic reaction?",
"src": "Patient: Hi. Im in college and it s finals week. I just took adderal for the first time ever. I took it Tuesday dec. 10th at 10am and now, 1:53am on Wednesday the 11th my tongue feels swollen, theres swelling below my adams apple and when I swallow it kind of hurts.i still have my tonsils and about a year ago I had a really bad case of tonsillitis. Is this an allergic reaction? Or something else? Doctor: Hi and thank you so much for this query.I am so sorry to hear about these symptoms after taking adderall. These symptoms are very unlikely to be related to a side effect of this drug, This may very much represent acute tonsilitis or pharyngitis. I will suggest that you get this looked into and treated accordingly and don't just ascribe it to possible side effects.Please, i understand the reasons for taking adderall but it never makes it a god idea. That is a substance of abuse and doesn't settle good. it can make you prone to future substance abuse.II hope you find the above information helpful.Thank you so much for using our services and please feel free to request for clarifications or ask for more information if need be. I wish you the best of health.Dr. Ditah, MD."
},
{
"id": 7974,
"tgt": "What could be the reason to have swollen legs even though all reports are normal ?",
"src": "Patient: A few weeks ago I woke up with my left thigh swollen towards the inside of my leg. I didn t really see any pimples or infected hairs that might have caused the swelling and pain. It became so tender to the touch and I couldn t hardly walk. I went to the doctor and because I had a history of a blood clot, the doctor sent me to have an ultrasound on my leg. They didn t find any blood clots in my leg and they said everything looked fine to them. They did however give me a medication called Naproxen and it is 550 mg. They said it is supposed to help the swelling go down, I have been taking them and my leg is still swollen and I can physically see how one leg is bigger than the other. Just today the back side of my thigh just under my left butt cheek is swollen and extremely painful. I didn t injure my leg in any way for this to happen. I need a second opinion. Any ideas why my leg might be doing this? Doctor: Thanks for the query it is ver difficult for me to tell and come to a diagnosis without examining u. From what u r telling it looks like u r suffering from cellulitis. U will have to be pt on antibiotics if so. Please take a second opinion, it is an emergency if u r a diabetic as well Have a healthy living"
},
{
"id": 51949,
"tgt": "What is the pathophysiology of Chronic Renal failure?",
"src": "Patient: What is the pathophysiology of Chronic Renal failure. Is it atherosclerosis of renal artery leads to nephrosclerosis leads to hypercalcimea leads to supress haemopoesis jj Doctor: Hi Welcome to health care magic \u00a0\u00a0\u00a0\u00a0\u00a0The most common causes of Chronic Renal Failure are diabetes mellitus, hypertension, and glomerulonephritis. Together, these cause approximately 75% of all cases chronic renal failure. Certain parts of world have a high incidence of HIV nephropathy.Atherosclerosis is also one of cause \u2018Hope I have answered your query, I will be available to answer your follow up queries, \u201cWish you Good Health and speedy recovery\u201d Disclaimer"
},
{
"id": 74188,
"tgt": "What causes chest tightness and breathlessness after walking?",
"src": "Patient: Trouble catching my breath after stairs or walking 20-25ft. Stomach feels like I ve done a lot of sit ups and tightness is causing me to walk bent over. Longer I go or further I walk worse breathing and I ve got to stop and take a deep breath and start again. Doctor: Thanks for your question on Healthcare Magic. I can understand your concern. You are having breathlessness on exertion. This is commonly seen with heart and lung diseases. So better to consult doctor and get done ecg, 2d echo, stress test, chest x ray and PFT (Pulmonary Function Test). If all these are normal then no need to worry for lung and heart diseases. Don't worry, you will be alright, first diagnose and then start appropriate treatment.Hope I have solved your query. I will be happy to help you further. Wish you good health. Thanks."
},
{
"id": 130966,
"tgt": "What causes fluttering feel in hands?",
"src": "Patient: I have a strange fluttering that i can feel with my hand and see in the mirror about 2 inches below my belly button. It has been happening for weeks now but especially when my legs are drawn up. It is not painful at all just very annoying. I am def not pregnant and have one child who is 9. Should I be concerned or it his just air or a muscle jumping? Doctor: you should not be concerned unless you have any changes in period you can also check you magnesium and zink levels in blood Good Luck"
},
{
"id": 125386,
"tgt": "Suggest treatment for foot injury",
"src": "Patient: My daughter fell down some stairs and now complaining top of her foot between 2nd and 3rd toes hurts cannot not walk or put pressure on front part of foot. There is no swelling and she is able to wiggle all toes... Can her tendon be torn? She stated when she fell that the tops of her feet is what hit each step.. Doctor: Hello, It could be a minor fracture or contusion. As of now, you can use analgesics/anti-inflammatory combination like aceclofenac/serratiopeptidase for symptomatic relief. She can also apply ice packs for faster recovery. If symptoms persist better to consult an orthopaedician and get evaluated. Hope I have answered your query. Let me know if I can assist you further. Take care Regards, Dr Shinas Hussain, General & Family Physician"
},
{
"id": 201058,
"tgt": "What causes foreskin tightening?",
"src": "Patient: Hello, I m a 30 yr old healthy male, I always have a yearly check up with all blood work and test etc.(last dr. visit about 3 months ago) and within the last year or so I ve noticed a discomfort in terms of the tightness of my foreskin which becomes even more painful when erect. My foreskin used to be very elastic and almost loose a few years ago, but now it feels like a tourniquet even when flaccid. Any advice would be greatly appreciated. Doctor: Thanks for asking in healthcare magic forum.In short: Any infection of glans and prepuce can cause this.Explanation: As you told you were able to retract your foreskin previously but is difficult now. Any infection of glans and prepuce (balanoposthitis) can cause adherence of skin to glans and make it difficult to retract. So, you can apply oint clobetasol locally and try retracting it daily once for few days. If it subsides its ok otherwise consult your doctor for this or for surgery(curcumcision). It is better to consult your doctor before taking any medication or treatment as it is a presciption medication. Good luck"
},
{
"id": 116572,
"tgt": "What does the blood test result indicate regarding health conditions?",
"src": "Patient: I have just gotten back my blood work after applying for life insurance. I am a 56 year old woman 115lbs and 5 133 and have had no health issues. I am reasonable active - play golf - walk - bile et. My globulin level came back at 1.4 and I have an appointment with my doctor next week. What kinds of things will he be looking for? Doctor: without any symptoms if you have only a low globulin, and normal albumin I suggest you repeat the test. if it is still low you need to look for HIV which is known to cause low globulins. if negative, you may need a bone marrow evaluation, vaccination to prevent bacterial infections like pneumococcus and consultation with a haematologist ."
},
{
"id": 100908,
"tgt": "What is the cause of sudden sickness, sweating and dizziness?",
"src": "Patient: I think I am allergic to codeine ( double vision, excessive dizziness, hives, sick to my stomach which is in my records) I was prescribed Percocet yesterday for a procedure, 1 hour after taking it, I was seated on the exam table waiting for the dr. when all of a sudden I felt very sick, started sweating and dizzy then I passed out. (face first) When they woke me up I started vomiting. I spent the rest of the day in the er. My blood sugar was perfect, all tests came back healthy and fine. I was so amazingly tired for the rest of the day what are normal side effects and do you think I had an allergic reaction and should the dr. have given me something else knowing that I could not take codeine? Doctor: Hi, thanks for using healthcare magicThe hives that you experience when you use codeine are definitely signs of an allergic reaction.Dizziness,drowsiness , nausea and vomiting are potential side effects (not allergic reactions) of this class of drug, some persons may experience these side effects more than others.Percocet does not contain codeine and this may be the reason that the doctor gave you this medication despite your codeine allergy.I hope this helps"
},
{
"id": 62837,
"tgt": "Suggest treatment for purple lumps under the skin of inner thigh",
"src": "Patient: I have a question.. over the past year i've been getting these like dark purple lumps under my skin on my inner thigh near the groin and sometimes i get these big bumps under my skin near the groin area on the inner thigh that if i squeeze them then it pecome hard and like there is a point on top that looks like its going to pop but it doesnt.. and yesterday i noticed a purple bump ontop of my skin on the inner thigh and today it was open and like half a centimeter deep.. i didnt pop it and i dont know what they r or what to do.. do you have any suggestions? Doctor: hi.it is best if you consult with a doctor, preferably a general surgeon, for medical and physical examination. based from your description, it could be a cyst (a sebaceous or a keratinous type). these lesions can occur anywhere in the body and have the tendency to recur. the one near your groin area might be lymphadenopathies secondary to an infection or other pathology. management (medical and surgical) will be directed accordingly.hope this helps.good day!!~dr.kaye"
},
{
"id": 206448,
"tgt": "Suggest treatment for breathing problems and panic attack",
"src": "Patient: I had a panic attack last night, associated with my dx of PTSD, from military service. While trying to go back to sleep as I was drifting off I would wake up as I felt I had stopped breathing. Is that possible. Those sensations lasted for another 45 minutes. Doctor: DearWe understand your concernsI went through your details. I suggest you not to worry much. Panic attacks always happen due to underlying anxiety. Therefore you must address the underlying anxiety first. Please understand anxiety and related problems. Once you understand your anxiety, you will be able to keep yourself away from panic creating situations. Therapy always starts with avoiding such situations along with coping techniques when forced to be in such situations. Start slowly, progress gradually, attain neatly.You should know that these problems are not mental diseases, but are mental disorders. Many researches and researchers confirm that medicines alone cannot cure mental disorders. Life style changes, change in thinking pattern, relaxation etc are as essential as medicines. Psychotherapy can help you changing your lifestyle and thinking patterns. Yoga and meditation help you to streamline your metabolism and neurological balance. Please consult a psychologist for further information.Psychotherapy techniques should suit your requirement. If you require more of my help in this aspect, Please post a direct question to me in this URL. http://goo.gl/aYW2pR. Make sure that you include every minute details possible. I shall prescribe the needed psychotherapy techniques.Hope this answers your query. Available for further clarifications.Good luck."
},
{
"id": 198244,
"tgt": "Can a ooze seen before a vasectomy a cause of concern?",
"src": "Patient: My husband had a vesectomy last friday, and starting about 2 days ago the pain has gotten worse, and he has begun to ooze something from the stitches. We have tried calling the dr office numerous times, leaving messages, and have yet to be contacted. What can we do? Doctor: HelloThanks for query .Your husband has undergone Vasectomy few days back and now has oozing from the stitches .The oozing is strongly suggestive of wound infection .Take antibiotics like Augmentin along with anti inflammatory medicine like Diclofenac twice daily .Consult your operating surgeon for evaluation .He may need to open up one stitch to drain out the collected infected fluid .Do not worry it will heal up within a week.Dr.Patil."
},
{
"id": 223417,
"tgt": "What is the cause of bleeding?",
"src": "Patient: I got the depo shot recently for birth control but i've noticed i haven't stopped bleeding since its been effective. should i get the shot again and maybe my body will be use to it and i will stop bleeding. Or will i continue to bleed with my 2nd shot. Doctor: Hello,I have gone through your query and understood the concern. When you use hormone contraception, menstrual irregularities are often observed as the body adjusts itself to the new hormone milieu. This takes a couple of cycles. However, prolonged bleeding should be evaluated and managed properly. In the current scenario, you have to take the advice of your health care provider and plan further management. I do not suggest another dose. Instead, you may need to manage the bleeding with styptics and if necessary, hormone course. Hope this helps."
},
{
"id": 56396,
"tgt": "Can elevated liver enzymes cause boils?",
"src": "Patient: My son is 28 years old. He just told me that he went to a Dr. because he was getting sporatic boils that would not go away. The doctor did blood work and said that his liver enzymes are elevated and his cholestoral is also on the high side (225). He is not addressing the boil issue and wants him to have a liver ultrasound. Are the boils associated with the liver problems or something unrelated?? Can you suggest what he can possibly be dealing with. He has always had stomach issues since he was a young boy and he does drink socially, I think sometimes to excess (socially). Also he has not weight himself but feels he has been losing weight without trying. Doctor: HiThanks for choosing HCM for your query.I appreciate your concern for your son's illness.I have gone through the problems described in the question.I am of the opinion that your son's present problem is due to alcohol induced liver disease with a possibility of suspected fatty liver that is why the treating doctor has advised ultrasound to confirm the diagnosis and to know the extent of liver disease.High cholesterol and stomach related issues are due to alcohol consumption.Sporadic boils (pyodermas) are due to decreased immunity also caused by alcohol resulting in recurrent infections.The sporadic boils can be treated by appropriate course of antibiotics or surgical drainage. But the main problem of liver disease due to alcohol has to be defined and managed.He must stop taking alcohol immediately. The high cholesterol level should be decreased by using a safe medicine in presence of liver disease.I hope your doubts have been clarified.Kindly give your feed back after getting convinced.Regards"
},
{
"id": 157254,
"tgt": "Have dime-sized lump on lower neck for years. Can it turn into cancer?",
"src": "Patient: Hi I'm 20 male and I've always had a dime-sized pebble under my skin on the lower back of my neck. My friends tell me if it doesn't hurt or grow in size then I should not worry about it. But I don't want this to become cancer because so many people passaway by cancer. Please I need advise I don't want to have surgery where I'm gonna be \"Out\" like what if I don't wake up? It can be scary Doctor: HIThank for asking to HCMNothing to worry about this, this could be simple swelling happening because of collection of some fat, called \"Lipoma\" this does not harm, neither turn into the cancer, this will remain there, without causing any harm you can remove this for cosmetic purpose, have nice day."
},
{
"id": 25888,
"tgt": "Suggest treatment for steal syndrome",
"src": "Patient: I have steal syndrome but the symptoms have greatly improved. One doc says it s due to graft narrowing and wants to cut on my arm from the top of my arm down to my wrist to create a blood flow to my hand. I m affraid and the dialysis tech s haven t mentioned anything about the graft blood flow. Doctor: Hi there are many treatment available for steal syndrome.ex.ligation and banding.Tapered graft can be used.distal revascularization-interval ligation (DRIL) procedure also is in being used.distal radial artery ligation, (DRAL) provide sufficient blood flow to hand.proximal arteriovenous anastomosis (PAVA) and revision using distal inflow\u2019 (RUDI) are also in being increased use."
},
{
"id": 200241,
"tgt": "Suggest treatment for early ejaculation",
"src": "Patient: Hai sir, I m 28 years old. Last month I dated with my wife.. 40 sec we did the starting stage. finally I insert my penis into her after 2 minutes of inserting my sperm released. This was my problem. So kindly tell the some solution to improve my sperm release time.. thanks Doctor: HiThank you for asking HCM. I have gone through your query. You can practice squeeze and stop technique. You can wear a condom or condom with anesthetic which will decrease the excessive stimulation on penis and there by prevent premature ejaculation. You can also do thought changing when about to near ejaculation. Anxiety and stress if you have should be controlled as these can cause premature ejaculation also. Breathing exercises for stress relaxation will be helpful for this. If all these not helping then you can approach a psychologist or sex therapist for behavioral therapy. If that is also not helping then SSRI group drugs like dapoxetine will be helpful. Hope this may help you. Let me know if anything not clear. Thanks."
},
{
"id": 160202,
"tgt": "I would like to know what I can expect during chemo ?",
"src": "Patient: I have to start chemo soon and I was told there are some new meds out there to help with the nausea. I would like to know what they called. Also when having chemo does your hair always fall out or does it depend on the person. Basically I would like to know what I can expect during chemo. Doctor: The chemotherapy drugs used depends on the cause also, so the side effecta also wil vary. General hair loss and weightloss are the usual side effects along with nausea. What problem are you having? And what chemotherapy has been prescribed for you???"
},
{
"id": 213413,
"tgt": "Had mental trauma. Why is my mental ability suppressed? I have lost calmness, why?",
"src": "Patient: After mental trauma in 2005, my mental ability is supressed, I am not able to do proper verbal communication. I am not satisfied with the position/order of things on my table or say my meal plate and keep reorganizing things. I do not experience calmness during long period of time, mentally fatigued many times and also not sleep soundly. Is this a problem of psychiatry or neurology . What type of diagnosis and treatments available. Regards Doctor: Hello From the brief history and onset with some stressor it points to a psychiatric disorder and appers to be close to a disoder called OCD and it's treatable . You need to consult a psychiatrist for the same and you can be absolutely fine after sometime on treatment. Will need treatment for some duration to get releif."
},
{
"id": 153088,
"tgt": "Is swollen feet and hands while on medication for pancreatic cancer normal?",
"src": "Patient: My father has pancreas cancer( metastatic stage). He was taking all sorts of drugs starting from gemcitabine, gemzar, avastin and morphine for pain. He recently started a new treatment that combine 4 different drugs. He is feeling a lot better, he is not in pain anymore but his feet are swollen and now his hands are starting to do that. What does this mean? Should I be concerned ? Is this just an side effect or does it have a bigger meaning? And what can we do to stop them from swelling more? Doctor: Hello dear. Because your father is feeling better, it means chemo is working. The cause of swelling can be either venous thrombosis(clots) in lower limbs or it may be due to low proteins level in the body or may be due to heart related problem. Please discuss these with your doctor so that he can arrange test for your father and then further management. Thanks"
},
{
"id": 98068,
"tgt": "Suffering from anal fissure with burning sensation. Which treatment should I take?",
"src": "Patient: Sir, I am suffering anal fissre from 15 days... I had taken allopathy tablets and also homeopathy medicines.. but the disease is not cured. Go to stoolside after 15 minutes or half an hour very severe burning sensation and scabbies sensastion.. pl. medicines may be informed in allopathy or homeopathy and suggest to me. thanking you sir, prabhakar Doctor: 1. are you in practice of altered food habits,supression of natural urges,firstly make sure you are not constipated: take fiber rich diet and adequate amounts of fluid alongwith stool softeners and then in morning daily sitz bath (sitting in warm water) for 15-20 minutes. 2. pain is for the last 15 days, thus it can be the pain itself or due to non healing of the fissure. 3. because fissure is painful, digital examination is usually avoided, but to know the exact position it is recommended, so consult with a surgeon. 4. take leafy vegetables except kale (palak) 5. Application of 'matra basti' 'jatyadi tail' at frequent intervals before/after passing stools is helpful."
},
{
"id": 113306,
"tgt": "Lower back pain. Abnormal L4 and L5 discs. MRI shows dark spot on spine. What could this be?",
"src": "Patient: For the last several months, I have been having lower dack pain . I went to the doctor and he sceduled me to have an MRI . He told me my L4 and L5 disc were abnormal. He sent me to a neuro specialist, which when he read the MRI, He started I had a dark spot on my spine near the affected disc.. I now have to take a bone scan and I an extemely anxious about what this could be. What do you think? Doctor: Hi and thanks fort the question, the dark spot could indicate a variety of abnormalities. These could range from artifacts, things that do not really exist to tumours. Itis therefore interesting to do an ESR frist. A very high ESR or erythrocyte sedimentation rate could suggest further investigations for spinal TB. Then blood calcium levels, alkaline phosphatase. Elevated values for these enzymes indicate a bone tumour. An abdominal ultrasound and CT scan and a chest X ray could be important in ruling out any existent or spread tumour. I suggest a consultation with an internist first or a neurosurgeon for further investigations, Thanks and best regards, Luchuo, MD."
},
{
"id": 57596,
"tgt": "Can I take marijuana after stone removal from gall bladder?",
"src": "Patient: hey i just had a sugery to remove my galbladder after they found small stones in it. I was prescribed tylenol 3 to ease the pain but the pain is still a bit strong at the moment. My sugery was 10 hours ago. would taking marijana be more of a con than pro at this time to help ease the pain? Doctor: Hi and welcome to HCM. Thanks for the query.Marihuana is not justified as analgesia method so from out point you should not take it especially not so early after surgery. it may have very bad effect on your digestion and may interact with anasthetics that were given during surgery. So if you need more analgesia consult your doctors about it and take some of the regular painkillers WIsh you good health. Regards"
},
{
"id": 165936,
"tgt": "What causes vomiting in a child after feeding particular food?",
"src": "Patient: I have an 18 month old and in the past have fed her fish on 3 separate occasions and she got violently ill. Vomiting everywhere for a couple hours. I wasn t sure if it was the fish that did it till this 4th time I tried and the same thing occurred only this time it made her lifeless after. Scared me!! I will stop feeding it to her now that I know its the fish. What I m wondering is, does this sound like this is an allergy and if it happens again that she eats it by accident could her reaction be worse. Doctor: Hello. I just read through your question. This can most definitely be a good allergy to fish. I recommend discussing this with your pediatrician. Until then, avoid fish."
},
{
"id": 89050,
"tgt": "What could intermittent lower abdomen pain suggest?",
"src": "Patient: Hello, I ve been having a burning sharp pain in mid/lower abdomen on the right side for about 2 1/2 months. I have been through physical therapy the last 3 months for lower right back pain and muscle tightness. The abdomen pain doesn t seem to be related to food/ eating. It comes and go. I ve been noticing that at times it comes when I am in the car and reaching to the right to get something behind my seat. Other times, it just occurs for no particular reason. I ve had an abdomenal ultrasound which was normal. Please help!!! Doctor: hi welcome to hcm..It could be just a muscular pain if your ultrasound is normal..repeat your ultrasound abdomen after 2weeks to rule out any interval appendicitis..drugs like buscopan will be helpful..use an antibiotic course and antacids to treat underlying infection if any..consult your doctor for further queries.thankyou"
},
{
"id": 174148,
"tgt": "Does high fever and body aches occur due to menangitis?",
"src": "Patient: my daughter is 4 and never gets sick but she has has several fevers of 102 through out the night lasting upto 45 mins or more. sore head, neck and stomach accompanied by vomiting. what do i do and how can i help her? she is not walking around and achy all over. could it be menangitis? Doctor: HI your kid needs evaluation properly though she is having a viral pyrexial infection which often presents with the above complaints.Please show to a local child specialist who will examine further and gives you symptomatic treatment"
},
{
"id": 175175,
"tgt": "What causes green color bowel movement in an infant?",
"src": "Patient: Hello doctor, Past 1 month my baby(sixth month) motion is in green color, still it is going as it is. I need a solution for this. kindly inform me if there any solution for this. i am working women. i give baby to \"nan pro milk power\".waiting for your reply. Doctor: Greenish stools are normal in a child of this age. It is caused due to rapid transit of fecal matter through the large intestines where it does not get adequate time for the bile pigments to convert into the yellow pigment of stool. It is not a cause of concern and does not need to be treated.Breast milk is the best possible feed for a baby and there can be no alternatives for the same. It would be appreciable if you can express your breast milk and refrigerate it for later use when you are away to office. If that is not feasible, giving Nan Pro would be fine. From the 6th month, inclusion of semi solids into diet would be advisable."
},
{
"id": 60869,
"tgt": "What does a lump in the throat indicate?",
"src": "Patient: i was sick had jaw pain and swollen lymph nodes seems better now but now have lump in throat like something stuck, like acid reflux makes me feel like puking and do pretty much every day. Also belly gets sorta upset and bowels. ER says GERD but it just came out nowhere and wont go away. And no test where performed. Whats wrong with me? Hard to live Doctor: Hello,As per my surgical experience, the lump in the throat with swollen lymph nodes suggests an underlying bacterial infection of upper or lower respiratory tract.In our clinic, I recommend:- Gargles with salted lukewarm water- Hydration maintenance- Antibiotic, anti-inflammatory medicines- Avoidance of smoking if doingHope I have answered your query. Let me know if I can assist you further.Regards,Dr. Bhagyesh V. Patel"
},
{
"id": 213500,
"tgt": "Husband behaving irrationally, violent, aggressive, mood swings and depression. Can he be suffering from some form of bipolar? What can I do?",
"src": "Patient: Hi, i ve been married to my husband for about 10 years and always thought his behaviour was sometimes completeley irrational, he has been violent and aggressive, suffers from extreme mood swings and has periods of depression . I firmly believe that he may be suffering from a form of bipolar but when i try and approach him about his behaviour he gets angry and thinks that it is all in my head and there is nothing wrong with him. His mood swings are becoming more frequent and it is starting to affect our 8 year old daughter, he needs help but i can t get through to him as he thinks there is nothing wrong with him. What can i do? Doctor: Your situation is very difficult and I am deeply disturbed and sorry to hear about your suffering. It 's all the more painful because you are not the one with the problem, it's your husband. My first piece of advice to you is to protect yourself and your child from this man's outbursts. If you can, you should take your child and leave the house each time these outbursts occur. Notifying family members (and perhaps friends) of the situation will help to protect and support you to some extent. Your husband may indeed have bipolar disorder as you suspect, but more importantly he has a horrible lack of insight into his problem thereby making it impossible for him to understand his need for treatment. Because of this, his prognosis for improvement is very poor indeed. You cannot force him into treatment if he refuses it; however, a nicely written lawyer's letter on your behalf may help you. You may have to leave this man if he does not seek treatment for this intense problem. Best wishes and I hope things improve for you!"
},
{
"id": 178720,
"tgt": "What is the dent in the head of a child after an injury?",
"src": "Patient: Hi our 2 year old son bumped heads with his 4 year old sister a few months ago, we put ice on his bump & swelling went down, but has left a dent in his head. He s still the same, no changes in his normal activities as what we can see. We ve taken him to the doctors & doctor said its fine, but we re still worried. We haven t had a ct scan, should we? Could it be serious? Doctor: Hi...if he is his usual self and not vomiting and coherent and playing around, you need not worry. No need to do a CT scan.I will suggest you danger signs of head injury -1. Vomiting continuously2. Seizures3. Watery of bloody discharge from ears and nose4. Unconsciousness5. Altered sensorium or behavior. If these are not there, you need not worry.Regards - Dr. Sumanth"
},
{
"id": 55216,
"tgt": "What is the reason for high SGOT and SGPT?",
"src": "Patient: I had dengue 3 years back but 6 months later developed late night acidity problem, I checked it found moderate fat in liverRecently doctor started Viboliv to cure my liver but even after 2 months my SGOT and SGPT remains slightly high 43 and 56 respectivelyWhat is the reason and what are the risks Doctor: Hello! Thanks for putting your query in HCM. I am a Gastroenterologist. Fatty liver has many cause like Hepatitis B, C, Metabolic disorders, NAFLD, alcohol intake etc. So I will suggest you to visit a Hepatologist for evaluation who will advise tests and then manage accordingly. Meanwhile there are few medicines which I found very useful in my patients are ursodeoxycholic acid and vitamin E. Do regular exercise and reduce your weight if you are overweight, avoid alcohol if you drink.I hope I have answered your query and this will help you . Wish you a good health"
},
{
"id": 162610,
"tgt": "Is a dent on the forehead a matter of concern?",
"src": "Patient: Hi Ihave a 7 yearsold doughter .she hits hear forehead against wooden table in school 2 weeks ago.when i saw her in the afternoon there was nothing wrong with its head but extremely light bruise that was not clear without close attention.yesterday i suddenly noticed a dent which can be touched more than seen. Is it serious thanks for your attention Doctor: Hello and Welcome to \u2018Ask A Doctor\u2019 service. I have reviewed your query and here is my advice. First be reassured that your kid is normal. I will suggest you danger signs of head injury - 1. Vomiting continuously 2. Seizures 3. Watery of bloody discharge from ears and nose 4. Unconsciousness 5. Altered sensorium or behavior. 6. Excessive sleepiness If none of them are present, I don't think you should worry about his trivial head injury. Best regards - Dr Sumanth Amperayani"
},
{
"id": 200927,
"tgt": "How to treat swollen and hard testicles?",
"src": "Patient: good evening sirmy name is jagadish from indialast six year back I have occur hydrocele of my left testicle. A local doctor cleared the water with a needle. And second time gave injection after removal of water with needle. from that time my testicle has become hard and swollem. plz suggest me the solution Doctor: Hi, Needle aspiration of Hydrocoele is only going to get you temporary relief of swelling. I am sure collection has recurred, maybe with blood surrounding the testes.I suggest you have a scan to confirm that testes are normal and then go for a formal surgery like eversion which will give permanent relief."
},
{
"id": 170889,
"tgt": "What is the medication for cough?",
"src": "Patient: my son suffering cough problem from last 2.5yrs and now he is 5yr old. I visited doctors and as per prescibed medicine tritofen syrup 5ml twice a day and Telekast-5 tablet at night continued since last 6 month. Still my son getting cough after certain interval of 15days -20days. kindly suggest further treatment ? Doctor: Cough in small children, if happens again n again, indicates that they are prone to falling prey easily to allergens, changing atmosphere with seasons esp winter and rains and spring, infections etc.The reason is assessible commonly i.e. lack of resistance to such factors or weak immunity.Antibiotics, antitussives, bronchodilation in moder drug is a good measure in acute problems but not for recurring ones or for chronic cases.If I were your family doctor, I would have insisted you to resort to Ayurveda's time tested, herbal combo of four classic herbs viz Balchaturbhadra CHOORNA with Piper longum as central to all these having effect of detoxification, carminative, appetite enhancer, cough and fever controller, anti emetic for such cogh conditions and what not. It enhances immunity. Cyperus rotundus, Aconitum heterophyllum, Pistacia integerrima are other three herbs.It's child friendly from very small babies' to 12 yr old children according to directed doses.No side effects except overdose which may cause gastritis.Non toxic medicine and completely natural. Commonly available in India or globally can be ordered on net.For further query you may contact any Ayurveda doctor or contact me through direct question.Hope it helps."
},
{
"id": 102895,
"tgt": "Stomach cramps in lower abdomen since had lobsters. Is this an allergy?",
"src": "Patient: Hi, we had full lobsters for supper and about half hour later my 10 year old daughter is complaining about stomach cramps in her lower abdomen. These were pre-cooked frozen lobsers so we steamed them to warm them up - that's it. She had the stomach flu 2 weeks ago and said this pain is worse. It has been about an hour and it still hurts her. She isn't having any trouble breathing, nothing is swollen, no tingling around the mouth. I can't recall if she has had lobster before, I know she has had crab legs (2 years ago atleast, maybe 3) and she loves shrimp and mussels. Do you think this could be an allergy? Doctor: Hello I have gone through your query. As she previously suffered from stomach flu (gastroenteritis) this could be aggravation of the lobsters she had and I don't think this is anaphylactic reaction. If she was prescribed with antibiotics before for gastroenteritis use them. Hope this helped you. Thanks."
},
{
"id": 168014,
"tgt": "Suggest remedy for stomach ailments",
"src": "Patient: i have an 7 yr old boy who often gets complains of stomach pain. The last three days he says he feels puuky, he is not eating much, has a cough and is very quiet. He has been getting a low grade fever at night and then it is gone in the morning. He is just not himself, he has put himself to bed early every night and is very cuddly.???? kinda getting worried Doctor: Hi... I have just read through your question. Whatever you are describing is called Chronic Pain Abdomen in medical terms. The differential diagnosis of abdominal pain in children varies with age, gender, genetic predisposition, nutritional exposure and many environmental factors. The causes are many including - constipation / acid peptic disorders / inflammatory bowel disorders / irritable bowel syndrome / worm infestation etc.I suggest you see your pediatrician or a pediatric gastroenterologist for this.Hope my answer was helpful for you. I am happy to help any time. Further clarifications and consultations on Health care magic are welcome. If you do not have any clarifications, you can close the discussion and rate the answer. Wish your kid good health.Dr. Sumanth MBBS., DCH., DNB (Paed).,"
},
{
"id": 10705,
"tgt": "What are the natural remedies to stop hair fall?",
"src": "Patient: hi i have been having alot of hairfall since a month i am studying for my exams i have exams in 3 weeks.i know stress could make my hairfall more but can you tell me other remedies naturally that may reduce my hair fall? could a shampoo that doesn't suit you also increase hairfall Doctor: Hairfall has multifactorial causes like lack of proper nutrition, stress, hormonal imbalance etc..first of all take a proper till full of proteins and vitamins and minerals.Include eggs, chicken and pulses(if vegeterian) you may also try a good hairvitamin supplement available in the market...in girls iron deficiency is also a common cause of hair fall for which you can get your Hb checked...Avoid stress as much as possible...Include exercise in you routine...Massage your hair atleast twice a week as it would increase the circulation to your hair follicles..comb your hair frequently atleast 2-3 times a day...Shampoo in general has no effect on hairfall, use a mild shampoo with conditioning effect..."
},
{
"id": 91729,
"tgt": "What could be the reason for having a sharp pain in my abdomen?",
"src": "Patient: I felt a sharp pain in my abdomen, left side ,just below the ribs. I ve felt it for a couple of days.Tender to the touch now. Have had a bad sneeze tonight that really seemed to infuriate it. More tender to the touch now. Had back headache yesterday. Doctor: Hi.This is the site for the muscles to get attached to the ribs. Any strain can cause pain like you have , which can increase with sneezing and is tender to local pressure. This can occur in flu, which might have given the headache too ."
},
{
"id": 108468,
"tgt": "Suggest cure for back and rib pain",
"src": "Patient: Hello I am a 40 year old female I started off with back/rib pain almost a year ago. At that time the dr stated he thought I had pulled the cartilage away from my ribs and recommended physical therapy. I did go to therapy for months and the problem continued only to progress to low back pain and leg weakness ,heaviness, twitching, cold sensation in the legs and groin area. Ive had 2 mri done in which they stated there was nothing significant I had a small annual tear and cyst between vertebra but They don t feel that it is the cause my issues. I start my day off with very stiff legs within a half hour that s usually better. And I feel well for several hours then the back pain and leg pain ,weakness, and heaviness become horrible and by the end of the day im barely walking. Ive also noticed some balance issues. This issue is effecting my job and my ability to work. I need some advice of where to go from here Doctor: As per your explanation looks like you also have deficiency of calcium as well vitamin d I suggest to give your mri report as it is here too. So that I can guide well. But as per what you have explain I understand you have done physio therapy as well also have taken basic treatment. I would like to suggest that you also need to do spinal exercise properly which includes stretching, twisting, spinal extention, as well also strengthening. If you need it properly do talk to me on my mail nirajmodish@gmail.com and will try to mail you exercise detail. Apart from. This do take regular calcium supplement. Take care.."
},
{
"id": 73886,
"tgt": "What does this lung CT scan test result indicate?",
"src": "Patient: I had a critical scan w and w/o contrast..it showed a spot on lung. Radiologist said it looked like a swollen lymph node. Not normal but commonly seen. What does that mean? Dr said it had to do with my stomach and bowel carrying blood to my lungs? Or something like that can you explain please Doctor: Thanks for your question on Healthcare Magic.I can understand your concern.There are many lymphnodes in chest cavity.Lymphnodes carry lymph from organs. Lymph increases when there is infection in the body.So lymphnode enlargement is commonly seen in infection of draining organ.Stomach and intestine lymph drain in chest lymphnodes.So any infection of stomach or intestine will enlarge chest lymphnode which is seen on your CT.Hope I have solved your query. I will be happy to help you further. Wish you good health. Thanks."
},
{
"id": 166411,
"tgt": "What causes bumps all over the skin?",
"src": "Patient: my daughter (1 & 1/2) had started running a high fever at the beginning of the week, was cranky and has lost her appetite. And now she has these little red bumps all over her skin. I ve already taken her to the doctor this week and they couldnt tell me anything but now that the bumps are there i m not sure if its all connected or not and what I should do about it. Doctor: Hi...the possibilities could be - 1. Hand foot mouth disease2. Urticarial rashBut...Skin conditions are best diagnosed only after seeing directly. I suggest you to upload photographs of the same on this website, so that I can guide you scientifically.Hope my answer was helpful for you. I am happy to help any time. Further clarifications and consultations on Health care magic are welcome. If you do not have any clarifications, you can close the discussion and rate the answer. Wish your kid good health.Dr. Sumanth MBBS., DCH., DNB (Paed).,"
},
{
"id": 185930,
"tgt": "What could a hole in gum be?",
"src": "Patient: i have a hole in my gum, on the side behind my last right molar where my wisdom tooth supposed to come out which it havent done so. and there is another hole right besides my molar it been there for about a year now . I havent gone to the dentist because I dont have insurance but I am getting worry because it wont go away. Doctor: Thanks for posting your query to HCM.After going through your history, I would advise you to visit a dentist as in your case proper oral and radiological examination needs to be carried out.The hole which you are talking may be due to an impacted wisdom tooth or spread of infection from adjacent tooth.Without proper examination, no correct diagnosis can be made.Hope my answer will help you."
},
{
"id": 224055,
"tgt": "When should I change my contraceptive from evra to cerazette?",
"src": "Patient: Hi! I'm condisering changing contraceptives. At the moment I'm using Evra (depotplaster), but I'm changing to Cerazette. Can I change before day one of my menstruation? Or do you advise me to wait? Which complications can I get by changing? In advance, thank you! Doctor: Hello, and I would be happy to answer your question today.You certainly can switch from the contraceptive patch to a birth control pill without waiting for yourself to start to bleed. Just wait until your patch is due to be removed, and start the new pills the next day. The Cerazette will be efffective immediately as a contraceptive, so you do not need any backup method.However, you should be aware that Cerazette is a progesterone-only pill, which means that it will not give you regular menstrual cycles, and it is less effective than combination birth control pills or the patch. The patch is greater than 99 percent effective Cerazette is only about 92 percent effective, unless you are breastfeeding. So you will not get any complications from changing your birth control method, however, you should be aware of the decreased effectiveness of Cerazette compared to the patch you have used previously.I hope i was able to adequately answer your question today and that this information was helpful.Best wishes,Dr. Brown"
},
{
"id": 223810,
"tgt": "Does having Implanon cause changes in pregnancy test?",
"src": "Patient: I have been feeling like I am pregnant. I ve had the implanon in for 2.5 years and I feel movement in my stomach and I have gained at least 5lbs. I m shocked because I work out at least 3 times a week. My last menstrual cycle was the 24th of July. I know it may be soon to tell but I just have a strong feeling. I have done 4 pregnancy test and they all came out negative...is there a possibility that I may be pregnant?? Can the implanon give multiple false negative?? Doctor: Hello, and I hope I can help you today.If your pregnancy tests are negative, then you are not pregnant. The Implanon will not cause a false-positive pregnancy test. However, the hormone in the Implanon, called Progesterone, can cause many similar symptoms to pregnancy, because the same kind of hormone is produced in your body by pregnancy. So feeling bloated, having breast tenderness, spotting or irregular periods and weight gain can all just be side effects of the Implanon. Many women think they feel fetal movements, but most women don't feel fetal movement until they are at least four to five months pregnant, so you are likely mistaking the movements of your intestines for fetal movement.I suggest you discuss your situation with your healthcare provider, as you may prefer to try an alternate method of birth control which may not give you these symptoms. Your Implanon will need to be replaced at 3 years anyway, and other hormonal contraceptives that are not systemic, like the hormonal IUD, can give long-term pregnancy protection without as many systemic side effects.I hope I was able to adequately answer your question today and that this information was helpful.Best wishes,Dr. Brown"
},
{
"id": 26299,
"tgt": "What causes low BP and suggest treatment?",
"src": "Patient: I am a vegetarian (30 years), do not eat junk food (white flour, sugar, no preservatives, no soda, do not drink, smoke etc....) work out yoga, cardio, etc..... My blood pressure has always been low... around 82 over 60. I feel great but some say that is too low. Doctor: hello,I have gone through your query.Thanks for using HCM.Your BP of 82/60 is definitely low.even if you feel OK you must get your cardiogram and cardiac check up done.you must take plenty of liquids and extra salt in diet.But you should consult a physician for further work up.My best wishesDr.Rajesh Teli,MD"
},
{
"id": 61398,
"tgt": "Suggest treatment for a lump in the throat",
"src": "Patient: So Ive started working out w/ small group training @ my chiropractors office. Its been a hard startup for me and Im in my 4th month. About 1/2 way thru my hour workout I get a lump in my throat & feel my heartbeat in my throat as well. Last week it made me feel as if I was going to puke. Is this normal till I get better in shape? Doctor: Respected user , HiThanks for using Healthcaremagic.comI have evaluated your query thoroughly .* This seems in relation with upper respiratory infection may be likely tonsil or adenoid gland enlargement . It has no relation with exercise or training .Hope to clarify your query .Welcome for any questions .Thanks for using Healthcaremagic.comWishing fine recovery .Regards dear take care ."
},
{
"id": 223289,
"tgt": "Is it normal to bleed for two days after removal of copper IUD?",
"src": "Patient: hi my question is i whant to concive took off my iud 2weeks ago the first week i thought i had my peroid due to to days after takeing it off bleed like a my period usually dose but only last 2 days intead of the normall 3 days i had the copper iud is that normal Doctor: Hello,Thanks for sharing your health concern with us. In the current scenario, I think your bleeding is due perhaps to the instrumentation during the procedure of removal of the IUD. It can also be your natural period, if it is due. Do not worry if the bleeding is lesser than normal, worry if it is heavier. If the bleeding recurs, if you have pain, signs of infection etc., please inform your health care provider and get a proper evaluation. Please do not douche or use strong soap solutions to clean the genital region. Do not use tampons. Hope this helps."
},
{
"id": 212597,
"tgt": "Diagnosed with PTSD. Taking lexapro and diazepam. How can I stop medication?",
"src": "Patient: I have diagnosed PTSD, my meds were changed 2 weeks ago to Lexapro and diazapam when needed,I haven t been able to get back to the doc to check in as I know I was supposed too because of floods in my neck of the woods. Along with the med change I m being \u00ebncouraged to start seeing the psychologist again as I haven t been to see him for nearly 3 years. I didn t want to start taking meds again or seeing the psych doc, now the meds are making me feel all over the place. I feel that all the good work I did a few years back to get on top of the PTSD has been for nothing, I m back where I started from...is that normal with PTSD? Seems like one step forward and a whole heap back. Doctor: Hi there ~ I am sorry that you are feeling this way. I understand that you have been having problems with depression along with post traumatic stress and the floods in the neck of your woods are not helping any. I am sure you are having a tough time at this time of the year. I would highly suggest that you go back on the lexapro and the diazepam if you still have enough supply. I think that this is going to help you out a lot and make life easier for you as it had been for a long time. You should start feeling better instantaneously. I hope this helps and you start seeing a psychiatrist as soon as you can. Take care and have a lovely day !"
},
{
"id": 56773,
"tgt": "Can masturbation cause further liver problems?",
"src": "Patient: For one month i am facing liver problem. Cause my liver is enlarge. In my blood billirubin is also high. So i am very ill. But i am still masterbating daily. So i want to kno at this time is my masterbation effect on my health? And plz tell me what are the effects of this? Doctor: Hello,Welcome to Healthcare Magic.It is completely wrong that masturbation causes liver problems.There is no adverse effect of masturbation on health.Liver problem may be due to alcohol, drugs, injury or infections.In liver injury, billirubin becomes high.Please remove myths and take care.Have a healthy life ahead..."
},
{
"id": 76176,
"tgt": "Suggest treatment for nasal and chest congestion and wheezing",
"src": "Patient: I have nasal and chest congestion, wheezing, hoarse in the throat, stuffy ears and a headache. About four weeks ago had a bad sore throat and was treated with amproxcillan which cleared up the sore throat; then about Been taking mucenix DM and contact cold and flu for about 7 days now but it not treating/clearing up the condition only taking the edge off so I can function...Please advise. Doctor: Thanks for your question on Healthcare Magic. I can understand your concern. By your history and description, possibility of post infectious bronchitis is common in your wife's case. Bronchitis (inflammation of airways) is common after viral upper respiratory tract infection. She was having viral infection 4 weeks back and now her wheezing, chest congestion etc are more suggestive of bronchitis. So better to consult pulmonologist and get done 1. Clinical examination of respiratory system 2. PFT (pulmonary function test). PFT will not only diagnose bronchitis but it will also tell you about severity of the disease. And treatment is based on severity only. She will need inhaled bronchodilator (formoterol or salmeterol) and inhaled corticosteroids (ICS) (budesonide or fluticasone).Don't worry, she will be alright. Hope I have solved your query. I will be happy to help you further. Wishing good health to your wife. Thanks."
},
{
"id": 47451,
"tgt": "Would dialysis be suggested to improve kidney conditions ?",
"src": "Patient: My father had colon surgery 2 months ago and has been in the hospital 4 times with complications. After the incision ruptured and he became septic, he had emergency surgery and was given a colostomy until the colon is healed. He is now in the hospital because of a seizure which was caused my his electrolytes being out of balance. This is the 2nd time in a month he has been admitted due to a seizure, out of balance electrolytes and poor kidney function. The doctor said his kidneys are not working properly because the location of the colostomy is expelling the waste before the liquids are being absorbed in the body. It looks like they are going to start dialysis for now until his kidneys are working properly. All other test looks good and readings look good. Is this a normal practice? Many thanks! Doctor: Hello and welcome to HCM.As an Urologist, i've understood your anxiety and express my concern too.I'll need certain details to be able to give an expert opinion on dialysis.Seizures are commonly seen in people ,who have electrolyte imbalance, with low sodium levels. With kidney failure, seen by rising creatinine levels, septicemia can set in, and if rise of creatinine,urea,and potassium level takes place, or fluid retention is seen on ultrasound scan of abdomen,or chest X-ray/CT for chest fluid, a dialysis will be urgently required.Unless i know his urea,creatinine,sodium,potassium, and scan reports,it's difficult to answer your query. kindly mention,which colostomy is done.For my expert opinion, send reports, as a direct question to me.Dr.Matthew J. Mangat."
},
{
"id": 83754,
"tgt": "Could body pain and lower stomach pain be due to side effects of back pain medications?",
"src": "Patient: For the last two days my whole body has been achey. today three different times i have had shooting pains in my lower stomach that brings me to the ground. i ve been taking three different medications for my back pain. could this possibly be having a reaction to the medicines? Doctor: Hello,The pain into the stomach is related to possible gastritis or ulcers. Having used anti-inflammatory drugs such Advil or naproxen for back pain may have contributed to this stomach pain. I can advise you to stop anti-inflammatory drugs and start taking the antiacids such Omeprazole.Hope I have answered your question. Let me know if I can assist you further. Regards, Dr. Olgeta Xhufka, General & Family Physician"
},
{
"id": 107636,
"tgt": "What causes severe back pain?",
"src": "Patient: Hi i am 17 have a 2yr old dauaghter i didnt find out i was pregnant with her until i was 2 half months and i start birth control depo right before they released me in october of 09 then stoped in january of 11 and started the patch and then switch back to depo in april and was due for my second shot in june and missed it and had unprotectd sex i had some spotting and cramp then in july got my depo that same day and the lab lady droped my cup of urine so i took two pregnacy test came up positve and went back to the doctor and one negative urine and one negative blood test it is now its august and i still am spotting every time i go to bed brown or pink blood and cramps but no bleeding some times and constant peeing it feel like preasure on my bladder nasuea horribe back pain my lower are bloated in my lower adomen Doctor: For treatment of severe back pain you can take some pain killer like biozobid-plus or osteonac-th only when the pain is worse enough or disturbing your daily work along with some calcium supplements like shalcal-d or toscal-gem once daily. Take some multivitamins like metilda-af or neurobian-plus once daily. Do some exercises after an expert opinion of physiotherapist. I will also advice you to sleep on a plain surface."
},
{
"id": 163133,
"tgt": "What causes hives in children?",
"src": "Patient: Aloha! My daughter, who is 9, has had hives off and on since May. She had a blood test that said she was sensitive to dust. We are curious what your recommendations would be to figure out what is causing the hives. Is it possible it could be a growth spurt? Thank you! Doctor: Hello and Welcome to \u2018Ask A Doctor\u2019 service.I have reviewed your query and here is my advice.Please upload an image of the skin condition so that I can see and get back to you. This is not related to the growth spurt.I suggest that you use hydroxyzine. But this is a prescription drug, and you will require a doctor's prescription for it.Hope I have answered your query. Let me know if I can assist you further.Regards,Dr. Sumanth Amperayani"
},
{
"id": 187066,
"tgt": "What causes pain in the jaw, numbness in the left arms and mild pain in chest?",
"src": "Patient: Hi ive started with a pain in my jaw and my left arm went numb, i felt very light headed and really shacky and pins and needles in my fingers then went on to feel sick, my left arm has stayed numb but feel ok in myself now apart from a dull ache on my heart, what is it causing this? Doctor: Hi,Thanks for posting the query, This could be due to any lesion, impacted wisdom tooth.What are the other dental symptoms associated? is there any swelling?I would suggest you to get a checkup done once.At home take tab diclomol for pain relief.Hope you find this as helpful."
},
{
"id": 124226,
"tgt": "Are Allopurinol, Lansoprazole, Lisinopril, Naproxen, Simvastatin and Hydroxychloroquine advisable for rheumatoid arthritis and GOUT?",
"src": "Patient: Hi, I have rhumotoid arthritis and goit and I am taking 300mg allopurinol,30mg lansoprazrazole,5mg lisinopril,250mg naproxen,40mg simvastatin,200mg hydroxychloroquine and 500mg paracetamol.My hands,elbow joints,neck and shoulders are still very painful,am I taking too many tablets or are they the wrong combination.I am male and 69 ,thank you. Peter Doctor: Hello, To comment first on the history with respect to medication, it is totally a physician decision based on the symptoms provided at the time of writing the prescription. So he will be the right person to give justification on why so many medications were prescribed. Now coming to your pain part. The pain is due to inflammation of the joints. Which you might be aware of. RA is a moving pain and can lead to pain at small joints and big joints also at times. You have a take a bigger precaution and that is to keep yourself warm. Like staying in cold weather will aggravate the symptoms. So keeping yourself warm will be first line of action. Next will be since how long you been facing this pain and when was this RA diagnosed to you? this plays a key role in the overall guidance of the treatment plan. Regarding the medicine part please visit a rheumatologist as he will be the right person to take care of it. Now treatment part. First thing is to keep the muscle and joint mobility to the maximum. So a physiotherapist will play a key role here. They might perform some passive or active exercise based on a physical assessment. Also, physical therapy pain reduction modalities like Ultrasound therapy and TENS therapy will be of benefit to reduce the pain. Also, please understand that this is only symptomatic for the pain. But to improve the metabolism and keep the body warm internally is only in your hand. The more the active you are the more the better the outcome score. Usually, I see chronic RA cases in my clinical practice. They all respond well to exercise. Any kind of deformity present needs correction via physical therapy or with the help of splints. But most patients respond well to exercise in long term rehabilitation. Hope I have answered your query. Let me know if I can assist you further. Regards, Jay Indravadan Patel, Physical Therapist or Physiotherapist"
},
{
"id": 213416,
"tgt": "Feeling unhappy, anxious, failed exam, no energy to revise. May have depression",
"src": "Patient: I think I may have depression or some sort of anxiety problem. I m not sure if what I m feeling is severe enough to fit into either definition but I know that I ve been pretty unhappy for quite some time and I need to do something about it. I ve always been pretty unmotivated when it comes to studying, at the moment I m at uni and I have to retake an exam I failed simply because I didn t feel like I had the energy to revise. I ve always been a bit anxious over trivial things that others seem to have no problem with, like going to work and presenting something in front of the class. My anxieties really hold me back and I feel like I can t do anything worthwhile whilst I feel this nervous. Most days I just feel down and find it difficult to keep going. Every day I try to meet up with a friend and spend as much time as possible with them so that I can distract myself, if I m alone for too long I just feel down in no time. No one else really knows the extent of my problem. Recently I got very close to someone and started a new relationship only for it to fail within the first week, so that s definitely contributed, but I feel it goes way beyond that because I ve been feeling pretty down all year. I don t want to rely on drugs particularly to get me through this, but as a student I d find it difficult to pay for therapy . I m stuck, and could do with some advice! Doctor: Hi ...your problem appear to be of depression....need evaluation for it's cause...is it Secondary to personality disorder or any persistent stressor or endogenous..because management is different...as far as treatment is concerned medication are good and you should not worry about dependence....most of medication are not dependence producing and all medication if used as advices and regular follow up done , can be tapered and you can be off treatment after some time..you just need to tell your concerns to treating psychiatrist and follow their advice religiously . Take care"
},
{
"id": 102364,
"tgt": "How safe is using Zenhale inhaler for cough?",
"src": "Patient: Hi i was recently prescribed an inhaler by a doctor at my clinic for wheezing she heard when i coughed. I've had the cough for about 2 weeks now and I've only been using the inhaler for 2 days. But when I cough now I get this horrible taste in my mouth I can almost smell it. Is this bad or are there ways to prevent this? Maybe I'm using the inhaler wrong? The brand is Zenhale if that helps at all. Doctor: U should take the inhaler close to ur nostril then whenever u press the top of inhaler at the same time u should b inspired so tat u can b able to take the medicine inside to ur lungs use this inhaler properly."
},
{
"id": 59862,
"tgt": "Liver test shows high SGPT, overweight, asked to exercise. Family history of diabetes. Treatment?",
"src": "Patient: Hello sir.. I am XXXXXXXX. Today my blood test report showed SGPT 45 U/L and remaining liver profile is normal. My doctor advised me to exercise. My BMI is 27. My age is 26(M). My parents have diabetes (high). Please suggest me whether it is ok or should I take some serious measures about my liver condition? How to reduce SGPT?? Doctor: Hi, Welcome to Healthcaremagic. SGPT mentioned above is not too high to cause any serious problems.That could be due to previous fatty meal if you had. Since your BMI is more loosing weight , low fatty food and good exercise would suffice.There is no need to worry also you having family history of diabetes though a risk factor life style modification would help you much. regards,"
},
{
"id": 109202,
"tgt": "What is the remedy for back pain after delivery?",
"src": "Patient: Hmi, omay I answer your health queries right now ? Please type your query here...hello dr, good morning iam 26 year old i delvered boy baby 10 months back .i was quite ok for 8 months . but past 2 months i had severe back pain . it starts at 4am at morning and it will continue till i wake up. it will disappear when i starts my work. Iam a house wife. i feed my baby. i use external creame only.but the whole day iam very active. thank you Doctor: Hi,From history it seems that you might be having stiff back muscles.It happens after delivery due to lack of daily physical exercise and some calcium and vitamin deficiency.Go for back extension exercise daily 10 minutes in the morning and evening.Take plenty of milk, calcium and multivitamin supplements.If require take Iron capsules as well.Ok and take care."
},
{
"id": 44505,
"tgt": "On clomid for unexplained infertility, pain in ovary, feels bloated, ovulation test positive. Pregnant?",
"src": "Patient: Hello, I am on clomid for the first time. I ovulate on my own days 32-35 been ttc for 4yrs with no lucky so been prescribe clomid for unexplained infertility I started my first clomid pill on 5/05/12 day 5-9 so i finished my last pill on 09/05/12 been feeling pain on both sides of my ovaries for the past 3 days in a role and very bloated I have been doing ovulation test from 11/05/12 with afternoon urine got a strong positive and since then been getting positive test for the past four days and been baby dancing for all the days i got a positive could I be pregnant? or there is something wrong with me please HELP!! Doctor: Hello, As you have mentioned, you took medicines and are doing a ovulation test which you are getting positive. This just means that ovulation has occured and this is the most fertile period. This does not mean that you are pregnant. If you miss your period this time or around the time of your expected period, do a urine pregnancy test using early morning urine sample. good luck."
},
{
"id": 10252,
"tgt": "What is the cause for hair fall problem?",
"src": "Patient: HI,I m dealing lots of problem with my hair.they are sraight naturally and are beautiful.but since a past few months i m facing a lot of hairfull due to which my hair have become slime.they are less in volume now and have become a little bit rough frizzy and less.please suggest something. Doctor: Hello and Welcome to \u2018Ask A Doctor\u2019 service. I have reviewed your query and here is my advice.If you lose more than 100 hairs per day, then it is significant hair loss. If you lose less than 100 hairs per day, you do not need to bother. For significant hair loss, treat dandruff first by using ketoconazole shampoo daily. If you do not have dandruff, then you can use minoxidil solution.Hope I have answered your query. Let me know if I can assist you further."
},
{
"id": 159459,
"tgt": "Bleeding throat, spitting blood. Have smoking addiction. Do I have cancer?",
"src": "Patient: I smoke ,, and have been for several years now. And just now, I was smoking , my throat started to bleed and I was spitting up blood, I can feel it in my throat, like a sore. The bleeding only left a small amount of blood in my salivia, out of about the 10 times I spit, I saw blood in probably about 6 times out of 10. It has stopped but I am wondering if this could be a sign of throat cancer or something like that? Doctor: Dear Luke, there is a chance that this sign could mean something serious but dont be concerned untill you do further diagnostic test. It s important to know do you vomit blood, coughing up blood or you just noticed it in your saliva. The first one means that you have some serious gastritis,gastric ulcer or esophageal disease. Coughing up blood is also a serious symptom which could be sign of pneumonia, COPD or even cancer so this needs to be ruled out. Blood in the saliva could be seen due to some lesion (ulcer, tumor, gingivitis, teeth disorders) of mouth or pharyngeal mucosa. So depending on what I ve just said you should definitely do gastro, pulmo and ORL examination. Wish you good health, Dr. Ivan Romic (Rominho)"
},
{
"id": 13932,
"tgt": "What causes spreading symmetrical rashes on face and body?",
"src": "Patient: I have a rash that keeps spreading. have seen 4 doctors some say tinea others say allergic reaction One said possible scabies? .tried fungal creams and cortisone creams still no change but keeps spreading. my husband got a week later and now my daughter(not living at home) has it. rash appears symmetrical on face and body very faint and hard to see ,comes a in various shapes usually roundish with a very subtle outline and dy-rish inner and itchy ( the new ones itchy not the old) Doctor: Hello, Symmetrical rash is most probably allergic rash and uncommonly tinea or scabies. But it is very important to rule out scabies and tinea. Because the drugs given for allergic rash can worsen the skin if the diagnosis is wrong. So, I recommend you to consult your Dermatologist as soon as possible. Hope I have answered your query. Let me know if I can assist you further."
},
{
"id": 213628,
"tgt": "Which is the best medicine to cure depression and anxiety ?",
"src": "Patient: Im a 17 year old male that has been suffering form depression and anxiety for about two years now. I have been on numerous medications and they are not helping. Doctor: Hello Thanks for your query. I will try my best to provide you with best possible professional recommendations to your questions. Treatment may include addressing any medical conditions that cause or worsen depression. For example, an individual who is found to have low levels of thyroid hormone might receive thyroid hormone replacement with levothyroxine. Other components of treatment may be supportive therapy, such as changes in lifestyle and behavior, psychotherapy, complementary therapies, and may often include medication. Only drugs will not be helpful along with that family support, lifestyle and behavioral changes are also required. Yet again, I do hope that you have found something helpful and I will be glad to answer any further query. Regards"
},
{
"id": 10924,
"tgt": "Suggest hair fall remedy",
"src": "Patient: hello, im a female aged 19 and i weight 115 pounds. i have a healthy background, but i have for the past year been loosing a lot of hair. it has recently started thinning out significantly and im freaking out, and i have even noticed my eyelashes are falling out. ive always bruised easily and had thin nails. how to i get my hair to stop falling out?! Doctor: Hi,You seem to suffer from telogen effluvium most probably. Your hairs are thin you easily bruised and also your nails are thin. All these suggest that probably you may be having some disease like anaemia, thyroid dysfunction or some other internal disease. You kindly consult the dermatologist.I would suggest...- blood tests for anaemia,thyroid dysfunction etc..- mild steroid scalp lotion to apply at night on scalp- biotin containing capsules daily- Vitamin E cap 400 mg daily- anti dandruff shampoo if there is dandruff - otherwise use herbal shampoo - almond oil to apply on hairsHave patience for the good result.. I hope this might help you.Thanks..Dr. Ilyas Patel MD"
},
{
"id": 147729,
"tgt": "Protruding spine at the top of the back post fall, prominent on bending the neck, burning upper back, no improvement with chiropractic therapy, X-ray normal. MRI required?",
"src": "Patient: Hello, I had fallen 2 months ago and jammed my shoulder causing my back and neck to hurt real bad. I notice that the top of my spine is protruding at the top of my back right where my neck starts. If I bend my head back it really protrudes even more than normal. I still have burning across my upper back at times and that spot starts to feel really sore if I start doing things with my arms. I don t have headaches anymore but for some reason that protrusion wont go back to normal. I see a chiropractor but he wont try to adjust my neck cause he doesn t want to cause further damage. I did have a regular xray and it should no broken bones or fractures . Its just so weird to me. Should I get an MRI? Doctor: Hi, Welcome to Health care magic forum. As you describe, it appears to be the dislocation of the shoulder joint, or may be some other joint nearby. I advise you to consult an orthopedic surgeon for diagnosis and treatment. You may need to have an M.R.I. for diagnosis. If there is dislocation, you may need to have a reduction of the dislocation, and immobilization. I usually prescribe to my patient with such symptoms neurotropic injections for 10 consecutive days,and then periodically,calcium suppliments, and pain killers. Avoid taking potato, other tubers,tomato,egg, chicken,and and sea foods. Wishing for a quick and complete recovery. Thank you."
},
{
"id": 150396,
"tgt": "Slip disc surgery 3 months back, tension in left sciatica nerve, bending waist possible",
"src": "Patient: hi Dr. i am 23 yrs old and i had surgery of slipped disc 3 months ago .... but i can still feel a little bit tension in my left sciatica nerve ....i m just wondering why is that .... and can i start my usual routine(e.g. bend my waist to pick things up) or do i have to wait more Doctor: Hi, thanks for writing to HCM. Basically i dont understand the meaning of having tension in the sciatic nerve? as its not a symptom as such. Also you have not mentioned the type of surgery that you underwent. If microlumbar discectomy has been done for you, all sorts of activities can be done after 3 weeks of surgery. In case of open discectomy, physiotherapy is to be done for the back muscles initially and regular activities are allowed after 2 months. Hope this helps"
},
{
"id": 171840,
"tgt": "What causes increased bowel movements?",
"src": "Patient: My 7 year old son has had a marked increase in bowel movements. From one a day to 6 or 7. Started with a big stomach ache. Then he just seems to go every couple of hours. Changed from soft to hard. Greenish urine slightly cloudy. No fever. No vitamins. No food changes. Normal activity. Normal sleeping Doctor: I can relate with all the other symptoms as they are caused by a stomach virus. However, I am not sure you got the urine colour right, as there are no descriptions of a greenish urine in medical literature as being due to ANY cause. I think you should feed him normally, and take her to give him plenty of water with electrolytes to prevent dehydration. He does not need any antibiotic. Just wait it out for a few more days and he should be okay.Dr. Taher"
},
{
"id": 224396,
"tgt": "Is pregnancy possible despite taking Unwanted-72?",
"src": "Patient: Hello Doc I had sex with my boyfriend on 30th nov on my 1st day of period & 1st dec (this day after having sex period end). We didnt use condom. i took unwanted 72 on 2nd dec. but as per the symtomps mentioned i didnt bleed not even vomitted n all. Can i get pregnant???plz reply urgently Doctor: Hi,NO, There is no chance of pregnancy if you had sexual intercourse on 1st and 2nd day of your period since it was not your ovulation period. If 30th November is your 1st day of your period then 1st december was your 2nd day of your period right. Unwanted 72 can also delay your periods or can cause irregular periods. Side effects of pill vary from one person to other, it is not necessary to have side effects of pill."
},
{
"id": 58789,
"tgt": "CT scan shows fatty infiltration of liver, mild hepatomegaly, dinfiltration of liver. Meaning?",
"src": "Patient: My Ct scan showed that I have some component of fatty infiltration of the liver. Mild hepatomegaly. My liver measures 22 cm. No space-occupying mass in the liver or the spleen. No abnormality in the pancreatic bed. The gallbladder appears normal. Adrenal glands appear normal. There is no renal mass. No hydronephrosis of the kidneys. There is no aneurysm of the abdoninal aorta. No enlarged adenopathy. No free fluid within the abdomen. There is no indujration in the nesentery. Mild Hepatomegaly. Dinfiltration of the liver. Otherwise, negative comuterized tomography of the abdomen. I used to drink whiskey when I got upset or depressed about something, but quit just recently, and I don't want to drink any kind of alcohol ever again. What does this medical report that is written about my Ct scan mean? Also my blood work came back that I have good liver function, kidneys, and etc. Does this mean I have some kind of liver damage in areas of my liver? Since I don't drink anymore will my liver get better? What can I do or take to help my liver and entire body get back in shape and back to being healthier again? Thank-you for your medical advise, please write me back soon. YYYY@YYYY Doctor: Hi, many thanks for the query!Well, your report is not that bad to be worried so much, but the decision not to drink anymore, is perfectly correct & all that is needed.You should start some medicines for reducing fatty infiltration of your Liver with your doctor's opinion.Rest all findings in your report are normal, also its good that your Liver function tests are normal.Do regular exercise, also yoga & meditation if possible.Wish you a good health.Take care.Regards."
},
{
"id": 99326,
"tgt": "Can Zyrtec be given for sore throat caused by allergies?",
"src": "Patient: My 15 year old daughter has a sore throat that has come and gone for the past five days She also has mild seasonal allergies this time each year and takes zyrtec. She is a singer, too. How do I know if this is something needing a doctor exam and possibly a prescription, or if it will run its course. She does not seem very sick, but I hate to have it worsen. Doctor: Hi,Welcome to health care magic.No,zyrtec alone can't cure your daughter's sore throat.Sore throat or pharyngitis or layringitis, epuglotitis could be due to viral infection( EB virus or adenovirus) but some time it could also be due to bacterial or parasitic.You should take her for routine blood count with chest X-ray and sputum examination to diagnose the underlying infective cause.She might require antibiotics ( quinolones or macrolides) to prevent superinfection with antiinflammatory and other supportive treatment like bronchodilators.It would be advisable that you must consult ENT doctor or pulmonologist for further advice.Thanks"
},
{
"id": 41740,
"tgt": "How can I conceive if my fallopian tubes are blocked?",
"src": "Patient: I am 25 years old and i have been together with my spouse for two years now but legally married for a year. My HSG report says i have both tubes blocked at the fimbrial end and my husband has a very low sperm count. I have 2 failed ivf cycles. I really need your advice because i am desparately in need of a child. Thanks and waiting for your reply. Doctor: Hi welcome to healthcaremagic.I have gone through your question.As your both rubbed are blocked at fimbrial ends then i would advise to go for laparoscopy to recanalization of tubes.For your husband do color Doppler ultrasound to rule out varicocele.And ivf invitro fertilization is the best option for you. You can take a chance for next 2 cycles.Hope i answered your question.Would be happy to help you further.Take care."
},
{
"id": 54836,
"tgt": "Where can I get Sebivo for liver disease?",
"src": "Patient: Morning Sir, I am 46 years old, 5 feet 6'' and weight is 55 kgs. I am Suffering from Liver Case and under the advise of my Doctor. I am taking the medicine Sebivo 600mg but I want to have the knowledge about the price in India and want to know where i will get from Doctor: Hi thanks for asking question.You are taking telbivudine drug in your case.So you might have chronic active hepatitis for which this drug is given...For getting this drug get prescription from doctor for it.And in mega city like Mumbai or Delhi you can get it from medical store.Price of 15 tablet is around 2500 rs...You have not mentioned about your liver condition.Otherwise we can guide you further.Meanwhile as you have liver disease tae low fat diet....More fruits and green leafy vegetables...Take care.Dr.Parth"
},
{
"id": 120129,
"tgt": "Suggest remedy for pain in knee and grinding noise in lower back while walking",
"src": "Patient: I can hear a loud grinding noise in my lower back while walking. I feel like bones rubbing together. I also feel pain in my left knee. Age=29, wt=80, mother of two kids and younger being 6 mnths old. I was experiencing back pain since the delivery.Doctor says it is beginning of arthiritis and I am very depressed now.Can u please guide Doctor: Hello,There is nothing to be depressed about. Such symptoms can be due to dehydration of disc or bone weakness. These are reversible at your age. You should do following to have relief in this:- Take a mild analgesic like acetaminophen for relief.- Apply a local analgesic ointment and do warm water fomentation with water bag, at least 3 times a day.- Start doing some muscle strengthening exercises. Yoga will be very helpful for you.- Take calcium supplements. Hope I have answered your question. Let me know if I can assist you further. Regards, Dr. Mukesh Tiwari, Orthopedic Surgeon"
},
{
"id": 188181,
"tgt": "Non-stop secretion from nose and sneezing after the novocaine shot for tooth removal. Is this normal?",
"src": "Patient: Two days ago I had my top wisdom tooth removed. During the novocaine shot on my palate it felt as if the medicine went through my nose. It felt like cold water and wasn't hurtful just odd. The surgery went fine. As I was driving home I could not stop sneezing and had non stop clear snot coming from my nose. Until this day when I breathe in through my nose it burns/tingles and makes me sneeze. Sort of feels like its dry but I have plenty of clear snot still too. Was it the shot? The extraction? Will it go away? I can breathe out fine. Doctor: its normal if local anesthesia goes to throat while injecting into oral cavity since its drug foreign body human body rejects by coughing snezzing so that drug can come out"
},
{
"id": 90114,
"tgt": "What causes fever and stomach pain in a child?",
"src": "Patient: my son has temperature of 38.4 degrees,woke late this morning complaining of stomach pain and discovered he had a fever.I gave him homeopathic remedy belladonna and hour later gave him 7.5mls of calpol.How long should it take for his temp to come down?Thanks Doctor: HIThank for asking to HCMI really appreciate your concern for your son Acetaminophen is the right drug (calpol) for fever and other system of medicine wont work here and not advisable to treat simultaneously with both the system of medicine, underlying cause of fever need to be found out, usually antipyretic takes half an hour to bring down the temperature if given with right calculated dose, take care and have a nice day."
},
{
"id": 136052,
"tgt": "Suggest treatment for swollen and painful leg and ankle",
"src": "Patient: I am significantly overweight, but my ankles, feet and legs below the knee are all incredibly swollen. I can understand if this is weight related but i often see many very overweight people whose feet and ankles are not inflated at all. Mine are ridiculous. Doctor: HiWelcome to healthcaremagicI have gone through your query and understand your concern.You can see for different causes of leg swelling by kidney function test ( blood urea, serum creatinine), thyroid function test ( T3,T4 and TSH),liver function test and heart function test. You can discuss with your doctor about it. Hope your query get answered. If you have any clarification then don't hesitate to write to us. I will be happy to help you.Wishing you a good health.Take care."
},
{
"id": 92054,
"tgt": "Can lower abdominal pain, shakes and pain in the left leg be the result of anxiety attack and how to cure it?",
"src": "Patient: Hi I'm having really bad lower abdominal pain it is a very tight feeling. I also have the shakes and pain in my left leg. I have anxiety so not sure if this could be the result of an anxiety attack or what. I'm freaking myseld out and don't know what I can do. Doctor: Hi. It is difficult to explain the tight feeling in lower abdomen and left leg alone symptoms by just anxiety. Is there any problem in the back of thoracic or lumbar region? If yes, please go for a check up by a Doctor then freaking out and having a big problem later."
},
{
"id": 118371,
"tgt": "Will a tear in blood vessel heal on its own?",
"src": "Patient: I tore a blood vessel in of calf today that s what the doctor tells me it s my right calf on the inside of my calf hurts bad he said time will and it will heal on its on. I took a step and it felt like you hit me in the back of my calf YYYY@YYYY Doctor: Hi,from history it seems that you might be having hematoma on the parts due to injured blood vessels.Apply ice pack 2-3 times a day for few days.Ok and take care."
},
{
"id": 171179,
"tgt": "Suggest proper dosage for meftal P for fever",
"src": "Patient: Dear Dr. My 4 years(13 kg) daugher is suffering from fever for last 4 days. We are giving crocin (6 ml) and and if the temparature is more than 102 then we are giving meftal P (2.5ml). For her age and weight, how much ml of meftal P can given at time and at what frequecy ? Doctor: Hi, if fever is not relieved in half hour after giving crocin then meftal P 5 ml should be given. However, since your child has fever for 4 days, i suggest you to do some blood tests like CBC, Malarial antigen test, NS1 Antigen test. Treatment will depend on above reports. Regards - Dr Deepak Patel, MD Pediatrics"
},
{
"id": 120266,
"tgt": "Is a neck muscle pain worrisome?",
"src": "Patient: The back of my neck up into the base of my skull on the left side is killing me I have had a violent cough from a sinus infection and the pain started right after a cough should I worry it is more than a pulled muscle, when I took hot showerr an let the water hit the offending spot it felt much better for a short time Doctor: Hi, Neck muscle sprain is not a serious problem, although, if it is caused by some underlying disorder then it has to be taken seriously. It is better to get an MRI scan done if pain lasts longer. Applying dicofenac gel and giving hot fomentation will relieve muscle spasms. Thank you. Hope I have answered your question. Let me know if I can assist you further. Regards, Dr. Jaideep Gaver, Orthopedic Surgeon"
},
{
"id": 17274,
"tgt": "Is it normal to get thigh pain after heart catheterization?",
"src": "Patient: Hello, I am a 25 year old female recently diagnosed with SVT. I had an EP test done 6 days go with a heart catheter put in my right femoral artery. Like i stated this procedure was done 6 days ago and I am still having pain in my groin that even makes my thigh ache. Is this normal? Should I be concerned and call my Cardiologist? Please help. Doctor: Hello There After going through your medical query I understand your concern and I would like to tell you that pain after cardiac catheterisation can persist for a week to 10 days , you can use ice pack to lower the discomfort Hopefully this information will guide you properly.Kind Regards Dr Bhanu Partap"
},
{
"id": 80856,
"tgt": "Suggest treatment for cough with excess mucus and breathing difficulty",
"src": "Patient: Hi I m a 25 year old wife and a mother of 3 about a month ago I had a really bad sinus infection and ever sents then at night when I cough I m out of breath and have to sit up and cough it all out excess mucus comes out with black specks in it but not when I blow my nose help me please Doctor: U needs to be evaluated more..get ur chest xray, pulmonary function test & sos CT chest &consult any pulmonologist. As in some conditions of chest, sinusitis coexist. & also take regular steam inhalations for ur sinus secretions make thinned out. & if no chest conditions are found, sos sinus surgery can also be performed, if extensive sinuses involved & repeated sinusitis occuring."
},
{
"id": 1814,
"tgt": "Is conception a possibility given the large follicle?",
"src": "Patient: i took letroz 2.5 from 3 to 7 day of my period took hcg shot on 15th day my largest follicle was 19mm and thickness of endometrium was 9mm i m taking gestmate from 15th day i have pco taking melmet 500mg thrice i miscarried twice yesr back can i conceive Doctor: Hi, there is a fair chance of conceiving with a dominant follicle. So don't worry. After taking gestmate for 2 weeks, do a urine pregnancy test at home after that. It will tell you if you are pregnant or not. You can try like that for 3 to 6 months. Hope I have answered your question. If you have any questions I will be happy to help. Regards Dr khushboo"
},
{
"id": 213264,
"tgt": "Suffering from ADHD, low blood sugar. Lack of appetite. Swearing episodes, angry outbursts and self destructive behavior. What is the treatment?",
"src": "Patient: Hi, I am a mental health professional but even that doesn t help me. I have ADHD and don t want to be on medicine because it can affect appetite and I already struggle to eat enough (just due to not being hungry, having the bad habit of using work as an excuse, etc.). I know the low blood sugar greatly affects my mood, so stepping up my eating can help. But honestly, typical anger management strategies don t help because of the impulsivity. My main issue is that when I m stressed at work I can t stop the repeated swearing outbursts and self-destructive behavior that feeds itself. I need to know how to handle things better. Focusing isn t an issue anymore as I ve learned to adapt. It s the anger and frustration and feeling overwhelmed I don t know how to deal with. I ve read books, used my own knowledge, etc. I saw a counselor (can t afford to now) I liked but to no avail. They can only give you so many strategies, I know that, the rest is up to you. But I can t seem to control my brain. I know ADHD is a defect in dopamine D4. How do I overcome a brain problem and all the anger, etc., that goes with it? Doctor: Hi, You are a mental health professional, do not want counseling or medicine and want to improve. In this situation, first, you should be non judge-mental for your treatment. Because you can not improve without intervention for such problems as you are wishing here. In my view medicines will help you greatly with counseling. So, meet psychiatrist and continue following anger management techniques. If problem is prolonged or severe and leading to frequent interpersonal problems, then for assessment and treatment, you can visit specialist like psychiatrist or psychologist. I hope this information has been both informative and helpful for you. Wish you Good Health. Regards, Dr. Ashish Mittal www.99doctor.com"
},
{
"id": 211487,
"tgt": "Trying to understand MMPI-2 test result. Offsprings have ADHD. What does the result indicate?",
"src": "Patient: I recently took the MMPI-2 and I have my results. I scored high on 4, 7, 8, and 9. My therapist gave me the test but did not really explain the results because I was looking to make an appointment with a psychiatrist. But I am having trouble finding a one. So I looked up the test online and looked at what the numbers meant. If I understand the basic outline of each number then I feel like I am pretty much out of luck. The numbers are not related, the condition each correlates to is not a progressive stage of one condition. So how exactly do I go about fixing things, when nothing seems to make sense? Both of my children have ADHD. Now having experience with the symptoms, I am pretty sure I have it as well, and the therapist agrees. But the other issues make me feel like I am fighting a losing battle to be normal . Do you have any suggestions? And any suggestions on finding a psychiatrist? Doctor: Welcome to Health care magic and thank you for asking the question.First I would like to say that scores on the MMPI are indicative of personality traits and not disorder. Scoring high on certain scales indicates that you have increased traits corresponding to that scale but it does not mean that you have suffering from that disorder. To make the diagnosis you need to consult the psychiatrist. Now the scores: (As you have not mentioned the scale names, I am considering the scale number according to the 'Total Code by Welsh'.)Scale 4- The Psychopathic Deviate scale. - High scores indicate that you have absence of deep emotional response. Such individual have episodes of abnormal behaviour but generally remain normal and lead a normal life.Scale 7 - The Psychasthenia Scale - It indicates that you have phobias and compulsive behaviour. The degree can vary from not disabling to disabling. If you can do your routine work nothing to worry.Scale 8 - The Schizophrenia Scale- High scores do not diagnose schizophrenia so do not worry. Schizophrenia is a complex disorder and need detailed assessment before commenting on it. Anyways high scores only indicate that you may have unusual thoughts coming into your mind. Again I will say high score does not mean that you have schizophrenia so do not worry.Scale 9 - The Hypomania Scale- High scores indicate that you have overproductiviy of thoughts and actions.In my opinion if you are having trouble with some features of any of the scales you need counselling and behavior therapy to modify the troubling behavior. I hope you find this helpfulRegards."
},
{
"id": 208409,
"tgt": "Treatment for memory loss problem",
"src": "Patient: I am 26 years old and have memory loss I had no accident or anything that I can think off no one hit me in the head! I just forget everything!!! I forget apts where I put my keys or anything even a half hour before I set them down! And cant remember my past or present What is wrong wit my brain!!!!!! Doctor: hi dear,memory loss problem are of two types 1. pseudo dementia 2. Dementia at this age dementia to be there is difficult to have so it might be pseudo dementia you haveor it may be due to underlying any metabolic disturbance.if possible do MRI brain.is there any associated symptoms you have ?like depressed mood, sleep disturbance, loss of interest, appetite disturbance, irritability, any history of epilepsy, Nausea , vomiting etc..consult psychiatrist for detail history and mental status examination.Thank you"
},
{
"id": 198314,
"tgt": "How to induce penile growth?",
"src": "Patient: Hi my name is Matt, as a teenager I used to masterbate and grew a lot of pubic hair all over my body. I also lost penis and testicle size. I haven't master bated for at least 12 years and still no growth. Just wondering if there is some kind of drug or treatment to promote growthThank you Doctor: DearWe understand your concernsI went through your details. The hair that grew all over your body was not pubic hair. They were body hair because of your strong male hormone. There is nothing unhealthy about it. Masturbation is normal and cannot harm your health in any way if you were within moderation. Masturbation cannot cause reduction in penis and testicle size, for sure. Masturbation and penile growth is positively related. If you masturbate thrice a week, penis grows and become stronger. There are no medicines available for penis growth. Stop being obsessive about penis and its size. There is a danger of you being anxious.If you require more of my help in this aspect, please use this URL. http://goo.gl/aYW2pR. Make sure that you include every minute details possible. Hope this answers your query. Available for further clarifications.Good luck. Take care."
},
{
"id": 219786,
"tgt": "What is the best treatment to get pregnant?",
"src": "Patient: i started my periods on the 7/7/10 and finished on the 13/7/10 and i started ovulating on the 17/7/10 and i have been having intercourse with my partner as we are trying for a baby but on the 6th day of me ovulating i started bleeding is it normal for this to happen if i am trying for a baby please help Doctor: Midcyclic bleedig can occur . it is normal in some cases. do not worry about it will watch for the next a few months."
},
{
"id": 211367,
"tgt": "Are hallucinations normal in a person taking morphine and oxycodone for broken back and neck?",
"src": "Patient: My adult grandson fell and broke his back & neck a year ago & is wheelchair bound. He has taken oxycolone (sp.)& morphine untill he started having hallucinations & went to rehab for the last 2 months. He is still having these hallucinations. Will this drug cause these & how long will they last? Doctor: Hiyes these drugs may cause hallucinations usually visual but at times auditory ( through ears )Good that at rehab he is being taken care ofIt may take some time . Please do not remain upset as he is already under treatmentWish him early recovery Dr Lal Psychiatrist"
},
{
"id": 65471,
"tgt": "Suggest remedy for lumps in children",
"src": "Patient: My 2 year old nephew gets these large lumps in different spots of his body. They're mostly on his scalp, and they appear on his arms, legs and he recently got one on his feet. Do you know what this could be? I don't know if it matters, but he has the sickle cell anemia trait. Doctor: Hi, dearI have gone through your question. I can understand your concern. He may have multiple lipomatosis or neurofibromatosis. It has no relation with sickle cell trait. He should go for fine needle aspiration cytology or biopsy of one lump. Then he should take treatment accordingly. Hope I have answered your question, if you have doubt then I will be happy to answer. Thanks for using health care magic. Wish you a very good health."
},
{
"id": 79090,
"tgt": "What causes recurring episodes of intense pain in chest with dilated pupils?",
"src": "Patient: man in mid 30s, periodic feeling of intense sternal chest pain, like squeezing when this occurs his pupils dialate with each heartbeatbegan appx 2-3 months ago went to ER on one occasion but by the time he got there the episode was overhe related his symptoms but was given something for gastritis none of the symptoms were evident at time he was seenepisodes continue to occur intermittantly, seemingly randomly Doctor: Thanks for your question on Health Care Magic. I can understand your concern. We should first rule out valvular heart diseases like aortic stenosis and aortic regurgitation because both of them causes intermittent severe chest pain and dilatation of pupils with heart beat. So first get done ecg, 2d echo to rule out these valvular heart diseases. If any of these is present than he needs surgical correction of valvular defect. If these reports are normal than sometimes stress and anxiety can cause similar symptoms. So ask him to avoid stress and tension. Don't worry, he will be alright but first rule out valvular heart diseases. Hope I have solved your query. Wishing him good health. Thanks."
},
{
"id": 219963,
"tgt": "Suggest best treatment to get pregnant",
"src": "Patient: Hi doc ..i am 26 yrs ild n married since 5 months .. i was on tab cerazette for a month and had on and off bleeding .. stopped the tab and took a scan which showed a small intramural fibroid of 1*1 cm .. other wise normal scan thickness ovary everything fine ..nw i don get bak my periods since 2 n half months .. 6 of pregnancy test showed negative .. nw i really willing for a baby .. tel me wat should i do to get bak my periods and be a mother .. am mentally being upset about this ,.. kindly help me Doctor: Hello dear,I understand your concern.In my opinion your ultrasound report is normal but for small fibroid which also might not be the reason for your failure of pregnancy.Actually there is nothing to worry in your case as you are just married 6 months back and you can try for natural pregnancy for another 6 months.Because usually infertiliy treatment is started if pregnancy doesn't occur even after 1 year of unprotected intercourse.But meanwhile rule out the reasons for irregular periods like any thyroid abnormalities,overweight or obesity,anaemia,stress etc.Weight reduction in case of overweight or obesity helps in regularisation of periods.Avoid stress and anxiety as they cause hormonal imbalance and delay fertility further.If all the above causes are ruled out the delay can be attributed to stress induced hormonal imbalance.Adapt healthy lifestyle like exercise and healthy nutritional diet daily.I suggest you to consult gynaecologist regarding medicines for periods as the pregnancy test is negative.Regular intercourse increases the conception chances.Nothing to worry as such.Hope this helps.Best regards....."
},
{
"id": 175799,
"tgt": "What causes constipation in a 3 month old with white specks and mucous in bowels?",
"src": "Patient: My son is 3 months old & has had trouble having BMs the past couple days (excessive straining, screaming in pain, etc...) Today I had to give him a glycerine suppository to help him produce a BM. When he finally did go, it was grey with white specks and had a mucous-like consistency. I could see the remainder of the suppository in his diaper, but this has never happened before when I ve had to resort to using them. Should I be worried? Doctor: Hi...this type of behaviour is common for kids of this age. This cannot be dealt with as constipation. Please do not use suppositories at this tender age. It could be his natural cycle to pass in 3 days (motion). You need not worry regarding the consistency of post suppository motion.Regards - Dr. Sumanth"
},
{
"id": 96536,
"tgt": "How can immobility in the legs after an accident be treated?",
"src": "Patient: i am a 54 year old female in june i got in a crash with my car {yes seat belt was on] but because i am 5'2 i have to sit up close a lady ran a stop sigs i hit her in the drivers side my right knee smashed into the dash board, i hve been going to pt sense. now a few weeks go my upper thight i can't move my leges i went to ther er they took exrays nothing and my bp ws 179/119 they gave me a bottle o perc's i refuse to take ,my question is along with the kneee, my shoulder ,d my back xn something like theis happen pore that 7dr wnt to surgery on my knee Doctor: Hello and Welcome to \u2018Ask A Doctor\u2019 service.I have reviewed your query and here is my advice.It has been long since the accident happened so the present complaints may not be due to that unless it is being proved clinically. If these are the condition then the old history of injury is the cause of present symptoms. It has to be managed accordingly, for the better mobility surgery is the best option as long as the knee joint is concerned. I hope I have answered your query. Let me know if you have any further questions. Regards, Dr. Akhtarhusain"
},
{
"id": 65314,
"tgt": "Suggest treatment for lump in groin area",
"src": "Patient: hello, I went to see my doctor 2 days ago about a lump that I found in my groin area on the right hand side. He said it was a small hernia, and said that there is nothing to worry about, he has arranged for me to go to the Hospital for a scan, but I am worried if the hernia becomes strangulated! I am 18 years old, 5ft 7. I am also average weight, skinny, yet healthy, and never had anything like this happen to me before in the past! Doctor: It may be an inguinal hernia and it has a chance of strangulation. So, as and when you see is developing please try to push it back with manipulation by your hand. If it does not goes back, rush to a general surgeon before the content of the hernial sac may get compromise of the their blood supply and the thing may become a life threatening condition. I will also advice you to get it treated as soon as possible because, in non complicated conditions, if elective surgery is done, the results are very good."
},
{
"id": 151966,
"tgt": "Everything speeds up in my head extremely unpleasant, please give me suggestion on this",
"src": "Patient: Hi, i have a query on another users query titled \"Everything speeds up in my head Extremely Unpleasant\". I have the same problem. As that user has explained, when this happens actions and sounds including voices speed up to a very unpleasant point. It happens when i least expect it. I have not yet been to the doctors to find out wha it is, but i have talked to a neurologist about it and has told me he has never heard anuthing of the sort. If anyone has any information about this please tell me. Also I think it only affects males. My father and brother have it or have experienced it, yet my sister hasnt. Thank you. Doctor: Hello, welcome to Healthcaremagic. The issue you have presented is very unusal and requires thorough evaluation. You must visit a psychiatrist. He would be able to help you in a better way after complete examination. Wish you good health."
},
{
"id": 13046,
"tgt": "Suggest treatment for prolonged red bumps and rashes around my waist",
"src": "Patient: Hi, i have always had small red bumps and rashes around my waist line and on my upper legs, they itch like crazy and i cant seem to stop this. what can i do to prevent this and what is causing this? every Dr. i have asked has never been able to tell me how to treat this. Doctor: Hello, The symptoms seem to be related to an insect bite. I suggest using Loratadine 10 mg for the itchiness. I also suggest using Hydrocortisone cream for local application. Hope my answer was helpful. If you have further queries feel free to contact me again. Regards, Dr. Dorina Gurabardhi General & Family Physician"
},
{
"id": 7630,
"tgt": "Pimples, marks on face, taken treatment, no relief. What to do?",
"src": "Patient: Hello Sir, I have a query. I am 26, still getting pimples on my face and back regularly. I have taken treatment 2 times before and recently shown to other Dermatologist . He suggested me to take Doxylab tab, Beplex Forte tab and to apply Deriva MS Gel. Also I have marks on my face. Last month suddenly I got many pimples on my face. Please let me know, it this problem curable and how long it will take to get all marks cleared. Thanks, Neha Doctor: Dear Neha, 26 is the noveau 16! smile :) I note your age & your tratment history. However, it needs more htan this to formulate an effective acne treatment regime. This is usually tailored to suit individual needs..so please consult a dermatologist near your place or you may consult me privately if you so wish to through this portal. Firstly, one needs to evaluate your goal..if my understanding is correct, you desire clearance of both acne & the marks left behind. Second, you need to do a battery of tests assuming we will start you on systemic isotretinoin therapy as a 'definitve' treatment if one can call it this. Please check the details of the medication & the side-effects & precautions et al (as the blood tests required are tailored with these in mind) with your treating doctor. It may take anywhere from 3-6 months for a decent result & your treating doctor again will be best placed to discuss this with you. Cheers & all the best...am sure you will do well with this treatment. Regds. Dr Praveen Rodrigues MD Dermatologist, Cosmetologist & Venreologist, Bangalore, India"
},
{
"id": 132851,
"tgt": "What causes swelling in legs and wrists and sweating at night?",
"src": "Patient: wrist and knee and ankle hurt s swelled up leg and knee and wrist and leg warm to the touch and clamy to the touch and sweet s when sleeping at night like they just got out to the shower it is red at first then turn s darker she the swelling get s worse Doctor: Hello, I have studied your case with diligence.As per your symptoms there is possibility of pain due to bad posture as your whole left side is involved, analgesic and muscle relaxant will reduce pain.If not relieved then possibility of polyarthritis needs to be consideredRheumatoid arthritis may involve small joints and early morning stiffness in joints, do you have such complain?Ankylosing spondylitis typically involve spine and hips with other large joint also. There is reduced chest function in this disease. Stiffness in spine is increased do you have such stiffness in spine? When such patients come to our hospital I advise them blood test like RA factor, ACCP, ESR, complete blood count, HLA B27Hope this answers your query. If you have additional questions or follow up queries then please do not hesitate in writing to us. I will be happy to answer your queries.\u00a0Wishing you good health.Take care"
},
{
"id": 150092,
"tgt": "Histroy of tumor removal from pituitary gland, excessive brain fluid, csf leak. Have lumbar shunt. Reason for more brain fluid?",
"src": "Patient: Last year I had a transnasal endoscopic tumor removed from my pituitary gland. During surgery there was a problem, too much pressure in my head and brain fluid. I was sent home and rushed right back csf leak. I know have a lumbar shunt I'm my back. Can you tell my I have so much brain fluid. My Dr said \" I don't know\" Doctor: Post trans nasal surgery some patients can develop a CSF leak . If it is self limiting there is no surgery performed to close it.if it does not close then a sea lent material is used to close the area.The lumboperitoneal shut they have put is to reduce the pressure in the brain.kindly consult your neurosurgeon as why you developed increased pressure.you could also consult a neurologist at your place for an opinion as we would need to look at all your reports and labs. Kindly meet a endocrinologist as well as very often post pituitary surgery hormonal imbalances can occur and life long supplementation of the deficient hormone is required."
},
{
"id": 99912,
"tgt": "What do frequent bouts of sneezing with pink flesh inside the nose suggest?",
"src": "Patient: Hello, I am a 17 year old female. When i wake up in the morning i have this very itchy nose and throat and the urge to sneeze, sometimes resulting in tears. I sneeze uncontrollably throughout day and night - especially when I m indoors. I sneeze and must blow my nose right after, I have also noticed some pink flesh inside my nose that wasnt there before, could possibly be polyps but i am not sure. I havent been to see a doctor in over a year since i tend to miss my appointments Doctor: yes.it may be polyp.you are suffering from allergic rhinitis.consult your family doctor.he will perform blood test for esosinophilia,allergy tests.your doctor would start tablet MONTAIR L CPUT STEROID NASAL DROPS.feedback."
},
{
"id": 97395,
"tgt": "How to treat fissures?",
"src": "Patient: Hi Doctor,In 2004 i had constipation once and approached a gastro enterologist.He inserted his finger to examine and i remember stool comming out.He suggested fibrogel and usage of fibre rich food.Till 2007 July i didn't have any problem. That year i had bleeding at the time of defecation. I approached a gastro enterologist and he confirmed that i have fissure. He suggested to heal with medicine. If it repeats he suggested to have surgery. Again in April 2008 August i had problem.In all the above mentioned periods, the pain went off soon and i was normal.But in 2010 on New Year eve i had the problem again with bleeding. I still have the problem on and off. The doctor suggested surgery. I have only one fissure. He suggested to have surgery before the count increases.But i am not inclined to have surgery.Do you suggest Homeopathy or Sidha in this case. Doctor: Hi Welcome to HCMI have gone thru your query regarding your fissure problem.Constipation is a boon of faulty lifestyle and wrong dietary pattern. Causes like unbalanced food habits, irregular sleeping pattern, low physical exercise are few of them. In elderly population, about 70% people are affecting or complaining about constipation .Constipation is mother of all illnesses . Fissure, piles are part of it .If you are severely constipated, the hard stools and strain during bowel movements will cause further harm to your condition by tearing - You may even bleed as the tear gets rubbed by the hard stools. You may feel an urge to itch the affected area. first thing you should keep in mind ,Prevention is better than cure so try to eat a high-fiber & a low-carbohydrate diet . Eat food rich in natural fibres to avoid this condition. Whole grains, beans, green leafy vegetables, carrots, figs, raisins and dates, prunes, papaya etc are some of the best foods you can include in your daily diet. Fibre helps to increase volume and soften the stools, thus preventing constipation. Soft stools will put less pressure .- Avoid sour spicy foods , refined flours foods , as fried , fast foods cakes, pastries, white bread and biscuits. White rice and sugary food items tea ,coffee alcohol corn, tortilla chips, and peanuts should also be avoided as they can cause constipation . Milk and milk products can cause diarhhea in some people, can also aggravate the pain in the anal fissures.A gentle instillation of olive oil into the rectum is an effective home remedy to treat anal fissures. You need to use around 50 ml of olive oil at a time. Repeat it daily whenever required,help to soften the hard stools.This prevent aggravation injury and helps heal the fissure quickly.Use of fresh aloe vera jelly compress on the anus will provide comfort from the painful symptoms and help heal the injury fast .Keep the affected area clean and dry with soft plain tissue .Sitz bath in warm water with little olive oil or aoe juice , can give soothing effect.Take aloe vera juice 20 ml twice before mealsPsyllium husks can help to heal problems within the bowels by moving waste more easily through the intestinal tract. 1 teaspoon taken in warm water or juice everyday will greatly assist digestion and excretion .Turmeric a T spoon with a cup of hot milk at bed time ,garlic 3-4 coves in the morning with water , lemon juice twice with water, before meals are great antioxidants ,help in early recovery by killing becteria on mass basis Lot of water every 2 hours should be part of your daily regimen .Regular exercise /walk /yoga /pranayam with above regimen will detoxify your system strengthening your immune system and paving way for happy ,healthy ,disease free life ahead .This is a preventive as well as curative regimTake Homepathic Acid Nit / a Dose weakly/ 3 doses Hope this helps you to solve tyour query ,if any doubt Mail at drsuchda.gmail.com Dont hesitate to get back if have further query ."
},
{
"id": 50385,
"tgt": "Cramping while urinating. History of kidney stones. Will that repeat again?",
"src": "Patient: For the last week or so, i am having severe cramping pain when i pass urine . frequent need to urinate. My son, one year old now, was through C-section. i have a history of kidney stones . A urine test about a month back indicated blood in my urine, though the X-ray did not show any stones. What do you think is causing this pain? Need your help pls. Doctor: Hi, many thanks for the query!Pain can be due to several causes more commonly due to stones, infection of the urinary tract.You need to do- CBC, RFT, Urine (Culture & Sensitivity),Random blood sugar level, USG KUB.Start appropriate antibiotic as per report, take antispasmodics, diuretics with your Urologist's opinion.Drink plenty of water so that at least 2 litres of urine is voided in 24 hrs.If diabetic, blood sugar levels must be in good control. Wish you a good health.Take care.Regards."
},
{
"id": 7688,
"tgt": "Want a fresh and healthy skin. Have dark acne spots on face. Which soap should be used?",
"src": "Patient: Hi am TJ,have always wanted a freshh and healty skin,I have like a light chocolate skin.I have use some skin lightening skin before,such as plamer s skin lightening complexion tone soap,but I don t want to use any of these skin lightening soap again cos am scared its might damage my skin.so today I searched for a normal soap,that is gonna be cool for my skin,and the soap theo DETTOL SOAP.am not sure if this is the right soap for me,so that s why am asking.I do have some dark acne spots on my face.so what do u think is the best soap 4 me? Is it dettol? Doctor: Hi...dear TJ..., Thanks for choosing HCM.., Soap is not keeping skin fresh and Healthy.., But gives some support.., If U get good selection of soap.., Always look for..., 1) TFM value of soap... TFM means Total Fat Mean Value.., If it is high that soap treated as good soap.., Good soap have minimum 70% TFM value..., So every soap have it's own TFM value.., 2) If soap have low Ph ....treatyed as good soap.., 3) Good soap have loow surface tension and softness., it can give good foam even hard water....High PPF water.., 4) Non-smell, Clolourless, soft and frothy soaps are good ..., ok,..thanQ"
},
{
"id": 174158,
"tgt": "What to do if baby has fever and has temporary effect from medication?",
"src": "Patient: My boy baby 7 month old is suffering from Viral fever from last two days & the body temperature is like sometime it is 101 & sometime 102. We have already consult with the doctor he told us to give calpol and meftal-p syrup 05 ml thrice a day but the dose is effective till 5 or 6 hours then again the fever comes back Doctor: HI if your baby is active in between with out fever and tolerating feeds, playing well and passing urine normally, no need to worry and continue Calpol as prescribed as fever will take 3-4 days to settle down.If he had any above features, needs blood and urine tests and antimicrobials intake based on your doctors prescription"
},
{
"id": 224944,
"tgt": "Will taking plan b after 5 hours of sex prevent pregnancy?",
"src": "Patient: How effective is plan b after ovulation . Note , I didn't ejaculate inside my partner , to be cautious we took plan b 5 hours later . Her period is 4 days long and we had sex on the 3rd . She expects her period on the 8th. Should plan b prevent the chance of pregnancy . Doctor: Of course plan B helps in preventing pregnancy . Though she took plan B pills after her first act , she did not take it on the 3rd , however she would have done. But anyhow she was in the safe period. My advice is that that whenever there is an act of unprotected sex , please consult your doctor .There is plan B step 1 which is being followed these days.`"
},
{
"id": 175815,
"tgt": "What causes shortness of breath and nose bleed?",
"src": "Patient: My 17 month old son has a shortness of breath and had a nose bleed incident don t know if they are connected and he s been having diareah and the sipping cough as we ll. is this a sign of a serious matter just today he developed the shortness of breath and the bleeding Doctor: Hi...I feel that he is going through a viral illness. But fast breathing s not a good sign to be left alone. i suggest you consult your physician or pediatrician regarding this. Bleeding can occur even due to trivial nose picking or due to viral illness per se. But as 17 month old cannot pick the nose so effectively, I think it is related to viral illness and I suggest you see a physician or pediatrician.Regards - Dr. Sumanth"
},
{
"id": 59011,
"tgt": "Chronic fatty liver. Numbness and tingling in arm, pain in shoulder while sitting or lying down. Any connections?",
"src": "Patient: Hi, Dr. Narasi. Thank you for your time. I suffer from chronic, non-alcoholic liver disease (fatty liver). I have noticed increased numbness and tingling in my right arm, along with pain in my upper right shoulder. The discomfort is present when I am sitting, or lying down (especially lying on my back). Is this pain caused by my enlarged liver?Thank you again for your time.Tia Doctor: The tingling in the arm can be caused by compression of the spinal nerve roots in the cervical spine or in the brachial plexus. The shoulder pain may be related to the fatty liver and will settle down as the probem resolves."
},
{
"id": 168531,
"tgt": "Can I give Allerga syrup and Ronclav 457 dt to my 4.5 years old for fever?",
"src": "Patient: My child is 4 1/2. frequently (A gap of 1or 2or 3 months) he is down with fever that comes after 5 hours. All this happens after his nose is blocked or throat is not good. Doctor usually advise him Fenoride cyrup tab.Adicef dt and Paraken SOS. But this time he says it is due to some allergy and changed medicine are Allerga cyrup and Ronclav 457 dt but fever is coming. My doctor is out station what should i do now. Doctor: Hi...Thank you for consulting in Health Care magic.Fever of few days without any localizing signs could as well a viral illness. Usually rather than fever, what is more important is the activity of the child, in between 2 fever episodes on the same day.If the kid is active and playing around when there is no fever, it is probably viral illness and it doesn't require antibiotics at all. Once viral fever comes it will there for 4-7 days. So do not worry about duration if the kid is active.Paracetamol can be given in the dose of 15mg/kg/dose (maximum ceiling dose of 500mg) every 4-6th hourly that too only if fever is more than 100F. I suggest not using combination medicines for fever, especially with Paracetamol.No need to use antibiotics like Ronclav or Adicef for viral illness. You can continue with the other medicines.Hope my answer was helpful for you. I am happy to help any time. Further clarifications and consultations on Health care magic are welcome. If you do not have any clarifications, you can close the discussion and rate the answer. Wish your kid good health.Dr. Sumanth MBBS., DCH., DNB (Paed).,"
},
{
"id": 203841,
"tgt": "What could be a bit of skin connecting the foreskin to the head of penis?",
"src": "Patient: Hi, My query is that there is a bit of skin connecting the foreskin to the head of my penis but it's not a skin bridge because I know what they look like and this isn't that.Also I am 11 almost 12 and I'm scared of telling my parents although I was going to tell my dad now except he is asleep and now I don't know if il tell them. Doctor: Hi. It is good that you know the skin bridge. At your age the skin tag attached to the head of penis can be just incomplete retraction of foreskin, which gets alright with time. Clean with soap and water at bath and try to pull this softly. It may be separated. Do not pull hard or with force. If this does not help tell this to dad. All the parents do observe their child all along their childhood and they would have noticed if something was wrong in a normal circumstances. If you still have a doubt , better show this to Dad or a Doctor ."
},
{
"id": 51054,
"tgt": "Kidney disease in one, other has failed. Is there any treatment?",
"src": "Patient: hi my name is orlando,and i desperately needs a help for my sister.my sister had a kidney disease,one of her kidney already failed.almost 70percent of her kidney doest work and only 20percent is working. - her creatinine is 1.7 and potassium 3.0.i desperately need a help.is there any means to cure here disease,what food she needs to eat and what foods she need to avoid.thank u very much Doctor: Hi Welcome to HCM. CKD can be managed but not cured unfortunately. The essence of treatment are 1. Restricted fluid intake. 2. Low salt diet. 3. Moderate protein diet. 4. Supplementation of vitamins, iron and calcium. Please contact your nephrologist as he is the ideal person to guide you. best regards DR GS"
},
{
"id": 8795,
"tgt": "What treatment should I take for pimple scars ?",
"src": "Patient: hai doctor , am XXXX age 28 , holes on my face due to pimples but it makes me feel more can u suggest m e any cream or laser treatment pls Doctor: Hi...dear Thanks for choosing HCM.., Holes on face...called Postpitted acne.., It is Atrophic scar..., Good treatment is available ..., LASER is the best.., There have been a number of exciting advances in laser medicine..., which have resulted in vast improvements in our ability to treat acne scars...., 1) In the early days of laser medicine they used a Carbon Dioxide laser.., which destroys the top layers of the skin, reasonable risk of side-effects with CO2 laser,now it is giveup.., 2) NON-ABLATIVE LASERS.. which, unlike CO2, do not remove the top layers of the skin..., and are non-destructive. These lasers have less risk and..., are more effective at treating acne scarring than the ablatives...., My treatment of choice.., 3) PULSED DYE LASER... One of the benefits of the pulsed-dye laser is..., that it can take the redness out of the acne scars...., Since often acne scars are red, this can very quicklY., within a month, improve the appearance of acne scars tremendously.., 4) FRACTIONAL LASER...., have been reported to produce remodeling in acne scars without the downtime of ablative resurfacing. So follow up the case ...do accordingly.., Consult good dermatosurgeon...good luck"
},
{
"id": 76383,
"tgt": "What causes breathlessness while sleeping?",
"src": "Patient: hi i am a 27 years old , 165 cm height and 63 kg weight suffering from cold which i am getting rid of it but it is accompanied with breathlessness which is disturbing me a lot specially while sleeping and i can not breath properly resulting in wakin me up every 2 houres, sometimes it is ok but occasionally it flares up and irritates me so what type of medication do i have to go through ? thanks a lot Doctor: Hi thanks for contacting health care magic.You have cold induce bronchitis mostly...Get your respi examination by auscultation ...If rhonchi heard then bronchoconstriction present....For which bronchodilator given like theophylin....To relieve congestion antihistaminic needed.Steam inhalation helpful in relieving cold congestion....If inflammatory narrowing more then short course of steroid given...If thick yellow phlegm also there antibiotic needed.If no improvement chest x ray done to look for consolidation.Sputum cytological examination also helpful.Take care.With keeping this in mind consult pulmonologist.Dr.Parth"
},
{
"id": 78285,
"tgt": "What causes pin prick pain in middle of chest?",
"src": "Patient: Hi Sir, my age is 53 years old. I think i am suffering from acid reflux since 5 days as i have a slight pin prick-pain in the middle of my chest only when i contract and when i expand my chest . DOCTOR ADVISED TO TAKE VELOZ D capsules for 5 days. What could be the reason. Is it curable. what type of food to avoid. Doctor: Thanks for your question on Health Care Magic. I can understand your concern. By your history and description, possibility of GERD (gastroesophageal reflux disease) is more. Along with drugs, you need to follow these lifestyle modifications for better symptomatic relief. 1. Avoid stress and tension. 2. Avoid hot and spicy food. Avoid junk food. 3. Avoid large meals, instead take frequent small meals. 4. Go for walk after meals. Keep 2-3 pillows under head in the bed to prevent reflux. 5. Quit smoking and alcohol if you have these habits. 6. Loose weight if you are obese. Don't worry, you will be alright. Hope I have solved your query. I will be happy to help you further. Wish you good health. Thanks."
},
{
"id": 156696,
"tgt": "Is there a generic for evista?",
"src": "Patient: I m confused. Is there a generic for evista. I m taking evista now after being treated for breast cancer and would love to pay less for the drug. I am now paying $500 for 90 pills. I m seeing references to generic but not manufactured by Lily. Thank you, Gwen Doctor: Raloxifene is generic for evista.It is an oral selective estrogen receptor modulator (SERM) that has estrogenic actions on bone and anti-estrogenic actions on the uterus and breast. It is used in the prevention of osteoporosis in postmenopausal women.Raloxifene(evista) was as effective as tamoxifen in reducing the incidence of breast cancer in postmenopausal women at increased risk. A major adverse effect of tamoxifen is uterine cancer;Raloxifene caused fewer cases of uterine cancer. Tamoxifen increased the risk of cataracts, but raloxifene did not. Both groups had more blood clots in veins and the lungs, but that side effect was more common with tamoxifen than raloxifene. On September 14, 2007, the U.S. Food and Drug Administration announced approval of raloxifene for reducing the risk of invasive breast cancer in postmenopausal women with osteoporosis and in postmenopausal women at high risk for invasive breast cancer.RegardsDr De"
},
{
"id": 213849,
"tgt": "What does it mean when you have persistant thoughts, ideas that people, the system have ruined your life?",
"src": "Patient: what does it mean when you have persistant thoughts, ideas that people, the system have ruined your life? ruined my hopes, my dreams and oppurtunities on purpous....because this is what i think at times. when i think about how i was abused, traumatised, victimized, bullied growing up at the hands of others around me...then because of all this i got mental health problems.... then i went off the rails then and committed a crime got in with the wrong crowd...then the system locked me up, put me through the psychiatric system and the jail system..messing my life up , giving me unwanted labels a crim record i never wanted...which will in turn in the present effect me trying to achieve my goals aspirations...employment oppurtunities..emmigration oppurtunities ( because that is my number 1 ambition, to move away from the uk)~ and originally all this wasnt my fault, i was a product of my environment.. so can anyone understand that at the age of 30 now i feel people, society ,the system,or whatever has ruined my life on purpous? i have bpd to,i feel theyve brandished me now im ruined Doctor: Thanks for the query Looks like u are in lot of distress. Please seek continuous medical advice from one single psychiatrist and have regular follow ups Schizophrenia is a treatable disease, most of the exacerbation can be either prevented or controlled Have a healthy life"
},
{
"id": 57548,
"tgt": "Ultrasound shows enlarged liver. Taking prescribed medication and restricted diet",
"src": "Patient: My son aged 21. USG of Liver reported mildly enlarged in size with normal shape & regular outline.parenchymal echotexture & echogenicity are normal.No focal parenchymal lesson is seen.Intrahepatic biliary radicles are not dilated. Liver span is 150mm. Doctor advice medicine( Hepamerz 2 tab daily for 3 months & restricted diet ( less oil) Doctor: hi, Enlarged liver is a fairly common entity among obese children. As other things are normal, you nee not worry. However, it is wise to rule out common causes of liver diseases. please post the details of liver function tests, viral profile etc, if done so that we can better understand his problem. All the best."
},
{
"id": 186589,
"tgt": "Can teeth problem lead to headache?",
"src": "Patient: Hi My name is Kaelah. I never brushed my teeth that well when I was a child. Sad thing is that now I have 3 broke teeth, and the dentist told me that I had to surgery. When i went to the doctor they told me to see a dentist, and now i had formed headaches as well as pain and a lump on the lower part of my left jaw were 2 of the broken teeth are located. Not sure if the lump is from the teeth of me applying pressure to help with the pain. Doctor: Hi,Thanks for posting the query, Yes this could be related, sometimes the pain in the tooth can spread to the surrounding areas.I would suggest you to get a checkup done for the broken tooth take an OPG radiograph, and plan for the treatment.Take tab augmentin TDS and tab aceclofenac TDS for 5 days.At home take antiseptic mouthwash rinses.Maintain a good oral hygiene.Take complete sleef, avoid anxiety and stress.Take care!Dr Ammara."
},
{
"id": 61780,
"tgt": "Suggest treatment for lump in the throat near jugular vein",
"src": "Patient: Hello, I have been having a lump on the left side of my neck right by my jugular vein. It s movable, round and aches sometimes. It feels like the pain connects to the far back of my tongue and when i protude my tounge I can feel a pulling , uncomfortable type of pain that feels like its connected . The cyst has not gotten smaller or bigger is size. I have had about 3 tonsillitis infections in about 4 years. Should I be worried? thanks. BTW : I have been to 2 ent s and an otolaryngologist and nobody seems to be willing to do a biopsy or further testing. I can t feel at ease until I get CT scan. Doctor: Hello and welcome to healthcaremagic.I am Dr Asad sami.I have read your question and i can understand your concerns.I will be answering you.What is your age..??For how long the lump is there..??Is it hard,firm or soft..??Is it increasing or decreasing in size..??Does it pulsate..??Are you loosing weight..??Does it cause pain when you eat..??Does it move upward and downward while swallowing..??Any family history of submandibular gland tumor..??Any history bad odour from mouth(halitosis)..??If were treating you in my clinic I would have examined that lump to look for its consistency and its relation to structures such as muscles, mandible and submandibular gland.I would have examined your oral cavity to look for any stone formation in the tonsills(tonsillolith) as you mentioned that you have chronic tonsillitis.stone can be removed by forceps and for chronic tonsillitis causing problems,tonsillectomy is the treatment of choice.I would have palpated floor of your tongue to look for any stone formation in the duct of submandibular gland(warthins duct).this condition is known as sialolithiasis.In this condition the duct is closed by stone formation and thus the gland swells and lump is felt under the mandible.for its diagnosis sialogram is the investigation of choice.the stone can be removed surgically.I would have done fine needle aspiration cytology of the lump to look for the cause of swelling , whether it is benign or malignant,whether it is lymphadenopathy(enlarged lymphnodes caused by tuberculosis,lymphoma,leukemia or any head and neck infection.) or mass arising from muscle..I would have ruled out lipoma,sebaceous cyst.In fine needle aspiration cytology it will be evident whether it is benign or malignant condition arising from submandibular gland..other conditions to rule out are aneurysms which are abnormal dilatations of arteries.I will suggest you to meet an ottolaryngologist or a general surgeon for a second opinion.in the mean time remain calm.Thanks.Hope the answer is helpful.Feel free to ask further questions.Regards.Dr Asad sami"
},
{
"id": 139106,
"tgt": "Suggest treatment for swollen and painful ankle",
"src": "Patient: I have been noticing swelling in my ankle area that gets better when I elevate my feet. The swelling is soft and non pitting. Yesterday my left supercavular area shows swelling, also soft and non pitting. Reading some research it appears it may be due to Thoracic duct or abdomen (which is larger than usually, and I have gained 10 pounds without significantly changing my diet. I am concerned and wondering if I should go to the ER today or wait to see my doctor on monday? Your thoughts? Thank you! Doctor: HiWelcome to healthcaremagicI have gone through your query and understand your concern.Non pitting swelling(edema) and sudden increase in weight in likely to be due to thyroid disease ( hypothyroidism). You should get your thyroid function test ( T3, T4 and TSH) to confirm or exclude it. However you can get your heart function test to exclude heart disease also as your swelling decrease on elevating the limb. You can discuss with your doctor about it. Hope your query get answered. If you have any clarification then don't hesitate to write to us. I will be happy to help you.Wishing you a good health.Take care."
},
{
"id": 180872,
"tgt": "How can constant pain in the jaws and face be treated?",
"src": "Patient: My daughter has been suffering from constant jaw pain with intermittent, zinging pain up her face and down her neck since having her wisdom teeth removed 6 weeks ago. She has seen an oral surgeon, neurologist and GP. MRI and CT scan were negative. She has a history of lymphoma; treated with chemo and stem cell transplant. GP started her on low dose Amitryptiline about 1 week ago with minimal results. Do you have any further advice? Doctor: Hi..Thanks for the query..It looks like it can either be a nerve pain or due to jaw joint related problems..Similar pain symptoms can be seen in Trigeminal Neuralgia as Trigeminal Nerve is the nerve of face..It can be due to trauma, infection or inflammation of the nerve..If the pain started after wisdom tooth extraction then it can be most probably related to jaw joint related problems..Amitriptyline can help in a few days, however the dose can be gradually increased if needed..Also the regular painkillers like Ibuprofen, naproxen, Ketorolac etc can be advised..Hope this helps..Regards.."
},
{
"id": 193694,
"tgt": "Can paralysis below waist affect male fertility?",
"src": "Patient: Hey my good friend was in a terrible car accident a few years back and has just recently found the love of his life they want to have a baby but doesn't know if this is possible because he's paralyzed waist down is it possible for him to get her pregnant or not? Doctor: Hi, Yes, there are options. You can do a testicular biopsy for sperm retrieval. minor procedure for extracting healthy sperm and go for an IVF treatment. Hope I have answered your query. Let me know if I can assist you further. Take care Regards, Dr S.R.Raveendran, Sexologist"
},
{
"id": 52931,
"tgt": "What is the cause for black spots on my nephew's liver?",
"src": "Patient: Hi, my young nephew has had awful stomach pains, and the specialist has now found black spots on his liver and been referred for further tests. We are all very concerned. What could the symptoms suggest? (He works a great deal abroad!) A concerned Uncle! Doctor: Hello,In most cases, these spots are benign growths such as cysts or hemangiomas. Hemangioma is very small and this can\u2019t cause such symptoms unless larger than 10 cm. He should do MRI scan or liver biopsy to find what type of lesion it is. Less common this may be malignant issues such as hepatocellular carcinoma. His abdominal pain may not be related to liver spots at all. It may always be a sign of some other issues such as IBD or gastritis. If these are just simple liver cysts then I don\u2019t think this is causing his symptoms.Hope I have answered your query. Let me know if I can assist you further.Regards,Dr. Ivan R. Rommstein"
},
{
"id": 17994,
"tgt": "Can elevated BP along with shortness of breath be signs of a heart attack?",
"src": "Patient: My blood pressure was 169/122, on Saturday evening (it is Thursday, now). Ever since then, I have been short of breath and under my left pectoral has been sore, especially when I take a deep breath. Do I need to be looking at a possible heart attack? The ER dr did not seem concerned with any of that. They gave me steroids and my blood pressure dropped to 145/92. Doctor: Hello, Regarding your concern, I would explain that chest pain which is modulated from deep breathing is typical of musculo-skeletal pain, not related to the heart. For this reason, I recommend taking acetaminophen or ibuprofen for the pain. If this situation persists, I recommend going to the ER for a physical exam and some tests: - A resting ECG and cardiac enzymes - Complete blood count, PCR, ESR - A chest X-ray study. Hope I have answered your query. Let me know if I can assist you further."
},
{
"id": 64351,
"tgt": "What causes lump above knee?",
"src": "Patient: My daughter has a lump/knot above her right knee ( approx 2-3\" above) about the size of a nickel. It is not painful and it does not affect her movement in any way. We have just discovered it. There isn't any bruising in the area nor has she hit it. It seems to be just under her skin on the quadricep. I would hope this would be a cyst at worst, but unsure. Doctor: Hi,Good Morning,Thanks for your query to My-HCM Clinic.I studied your query in depth.I understood your concerns.My opinion -Your daughter has-?Lipoma./Lipoma arborescence of knee(synovial origin) / Sebaceous Cyst-(skin origin) /Pre-patellar bursitis of knee mostly.-Hope this would help you to recover from worry-some query.Wish you a Fast Recovey.Wellcome to HCM again."
},
{
"id": 38764,
"tgt": "How to prevent groin infection?",
"src": "Patient: I have been battling a groin infection for the past nine days. The culture just came back as proteus mirabilis and I have taken a prescription of Cephalexin which did not help much. I am currently on my fourth day of Clyndamycin. The abscess is still draining a very smelly fluid and it is going on the eighth day of drainage. Is this normal? Also how do you come in contact with this proteus mirabilis bacteria? Is there anything I can do to prevent this from happening again? Doctor: Hi, Welcome to HCM.Proteus mirabilis is one of the bacteria which is normally present in the intestine and is excreted in the stool. It commonly causes urinary tract infection in persons with bad personal hygiene. It can cause wound infections and rarely abscesses. In your case I would suggest you get the abscess drained regularly and it will take about 7-10 days to completely heal. I would also suggest you complete the full course of antibiotics for 7- 10 days.Maintenance of personal hygiene is a must to prevent recurrence.Thanks."
},
{
"id": 45945,
"tgt": "What causes high level of micro albumin in urine?",
"src": "Patient: Hi I am writing it for mother who is a renal donor for me and 2 yrs back i did my transplant.after donating she is also doing good but the problem is the sugar level is in higher side. FBS - 195 and PPBS - 212. Then I have tested the urine micro albumin which is 109 and the creatinine - 1.0 .So Wanted to ask what is consequences of high sugar and what signifies microalbumin is 109 .Is there any risk to her health and kidney and whether it is curable or not ? and what should be the medication or precaution should be taken? Doctor: Hello and Welcome to \u2018Ask A Doctor\u2019 service. I have reviewed your query and here is my advice. Consequences of high sugars are many and effects every part of our body from eyes to foot. but her sugars are not so high and can be easily brought down to below 140..by exercise and avoiding sugar in diet and diet control. if still not controlled she may have to take a small dose of antidiabetic medicines...like metformin. pls check her hba1c also urine for microalbumin is no longer used and is not important. if you are worried about protein in urine then get spot protein creatinine ratio in urine checked....it is unlikely to be high. by keeping her blood sugars under control her health and kidney will be absolutely okay. get her eyes checked for retinopathy every year. Hope I have answered your query. Let me know if I can assist you further."
},
{
"id": 220571,
"tgt": "Would there be a miscarriage if the baby is diagnosed with trisomy?",
"src": "Patient: I am 16 and half weeks pregnant And was told yesterday at the Dr that my baby might have Trisomy 18 but they dont know for sure yet. because i havent had a amnio.But the dr I seen was not my actual dr. It was at a different hospital . But they said we where gonna lose our baby just from looking at the ultrasound. But ever since I have been pregnant i have had brown vaginal discharge which my dr says is normal and fine. But ever since yesterday after i went to the dr I have had the same light brown discharge but its now a light pink with clumps or chunks in it. When they said i was gonna lose the baby they said they didnt know when or how.But yesterday our baby was moving around like crazy on the screen and the heart rate was 153 and its been around that since ive been pregnant. But I was just worried after having the light pink discharge with clump or chunks. what that might be???? I havent had any cramps or pains or anything tho. But yesterday when they did the ultrasound they did a vaginal ultrasound. And I didnt know if maybe the light pink chunks where maybe the lining of my uterus or maybe dead skin cells or something and when they did the vagnial ultrasound it maybe knocked some of the dead skin cells or lining lose or something. Any info for me???? Doctor: Hi there,,Please do not be so tensed, having been told that you are going to loose your baby must be a devastating experience. But nothing is confirmed till a definitive test such as an amniocentesis is done. A transvaginal ultrasound does not cause any damage to the lining of the uterus or to the pregnancy, do not be very concerned about it causing harm.If some defects were identified on the ultrasound we will need to rule if they are isolated or associated with a genetic cause such as a trisomy. The amniocentesis will take some fluid from around the baby and analyse it for the DNA.But if the baby does have a genetic defect it is also more likely to miscarry itself, which could be a cause of the constant mild spotting you have already had through this pregnancy.If the amniocentesis comes out with a normal finding and there are still defects observed on the ultrasound, you will need to get detailed scans at 20 weeks and a cardiac ECHO of the baby at 22-24 weeks to rule out other problems if any.Hope this helps.Regards."
},
{
"id": 199781,
"tgt": "What could cause pain in the bladder?",
"src": "Patient: Hi, I have been using contiflow for a while, lately my transplanted kidney Dr. switched me to cardular 4 mg pretending contiflow has the ability to damage the transplanted kidney though contiflow is much cheaper in price is that wright ???1 - I am having pain in bladder as well as in the balls esp. morning can u pls. advice for reason ??? Doctor: Hi, dearI have gone through your question. I can understand your concern.You may have benign prostatic hyperplasia. It is prone to infection. So genito urinary tract infection is common in this. It causes pain in bladder and testis. You should take a course of antibiotics like levofloxacin. Consult your doctor and take treatment accordingly. Drink plenty of water.Hope I have answered your question, if you have any doubts then contact me at bit.ly/Drsanghvihardik, I will be happy to answer you.Thanks for using health care magic.Wish you a very good health"
},
{
"id": 36174,
"tgt": "Suggest medication for e coli infection during pregnancy",
"src": "Patient: i am four month pregent.i am suffering from ecoli infection and CMV infection .wat shouldi do to keep my baby procted from its adverse effect ,i am taking argiprime oral solution .i am 28 year ,5 ft 5 inch,weight 76 kg and had a missed abortion in june 2012 and again conviced in oct 2012. Doctor: Hello,Thank you for contacting HCMI understand your concern, I am Dr Arun Tank, Infectious Disease Specialist answering your query.E coli can be treated and has no effect on the baby. But CMV can have birth defect in the baby.CMV can cause retinitis and ear defect in resulting child.I advice you to take the cephalexin 500 mg four times a day for four days for E. coli infection.For CMV you can take gancyclovir. But take all above medication under your doctors guidance. Some cases of CMV has serious defect in children I advice you should discuss this with your treating doctor. For UTI I advice you should take plenty of water as much as you can take. IT will flush out your residual bacteria.Please wash your hand frequently in day time as it can also transmits the bacteria to the urinary tract.Maintaining hygiene is also a important part as it can also treat you equally as other antibiotics do.You can ask for further queries here on bit.ly/DrArun.Thank you,Dr Arun TankInfectious Disease specialistWe wish you a best health at health care magic."
},
{
"id": 180837,
"tgt": "What causes inflammation in the gums after tooth extraction?",
"src": "Patient: last night I felt like I had fever in my gums the area where my tooth was extracted 2 years ago is still sensitive I took 2 generic Tylenol p.m. and it helped some I also went to the ER 3 nights ago they gave me some numbing mouthwash for my cankersores they said I might have some virus or vitamin defency what is the problem because I always have them Doctor: Hi..Thanks for the query..As per your complain you are suffering from RECURRENT APTHOUS ULCERS which is causing you the eruption of ulcers frequently in mouth..It can be caused due to a number of reasons like Stress, Nutritional Deficiency [Iron, Vitamin B12 and Folic acid] , hormonal changes, acid reflux, tongue trauma due to some sharp teeth or tongue biting habit, excessive intake of sour and citrus foods, uncontrolled diabetes and other immunocompromised conditions etc..You need to consult an Oral Physician or a general dentist in this regard to get evaluated and treated..For apthous ulcer you can do warm saline garglesGargles with 1:1 solution of benadryl and milk of magnesia can help..Apply numbing gel over the sore for relief from pain especially during eating..Take a multivitamin tablet for a period of 15 days..Avoid spicy, sour and acidic foods..In case if still the condition does not subside the physician can prescribe you to take a Steroid course as it the ultimate treatment in case if nothing else works..Hope this helps..Regards.."
},
{
"id": 92391,
"tgt": "Why do I feel tenderness under breasts with loss of appetite and stomach issues?",
"src": "Patient: Hi, I m 30 years old. I have just discovered that I am tender underneath both my left breast and in both armpits. It doesn t hurt to move but to touch. Feels like it s bruised. I have not had a trauma. Also no appetite and bad stomach. What could be the cause? Doctor: Hi...u may have gastritis... though i am not sure of the pain in armpits...try taking cap XXXX 20mg twice daily on empty stomach one hour before food for two weeks and then make it morning only for another two weeks...eat less of spicy fooda, tea n coffee...Dr. Ashish Verma"
},
{
"id": 188587,
"tgt": "Minor swelling on left side of palatoglossal arch, gets inflamed on eating cold things, no complete recovery. Reason?",
"src": "Patient: Sir, I am having minor swelling on the left side of palatoglossal arch. It is persisting for over three months. whenever i eat cold things like ice cream, or spicy food it gets more inflamed & start curing very slowly but never completely. I have gone for medical checkup but nothing worked. please help me out what can be the cause for such symptoms. Doctor: Hello,Welcome to healthcaremagic forum.Usually swelling on the palate occurs due to spread of infection from the tooth into periapical tissues.A thorough clinical as well as radiological evaluation of the affected area is necessary to rule out the cause of infection.Pus has to be drained out completely.If there is deep caries of associated tooth,root canal treatment has to be done.If there is periodontal infection,scaling ,root planning as well as curettage has to be done.I would suggest you to take ciprofloxacin antibiotic along with analgesics.Visit a dentist for further reference.I wish you speedy recovery.Take care."
},
{
"id": 16719,
"tgt": "What causes increase in heart rate after smoking?",
"src": "Patient: Hello. I am from Serbia, and i have one question.As far as i know, i don t have health problems, but, when i smoke a first cigarette in a day, my heart rate grows to 120-130, which worries me much. Can you tell me what happens there and is it risky for my health? Doctor: Hello There After going through your medical query I understand your concern and I would like to tell you that smoking increase oxygen demand in the body and to fulfill the demand you heart act more rapidly and result in increasing your blood pressure.Hopefully this information will guide you properly.Kind Regards Dr Bhanu Partap"
},
{
"id": 185265,
"tgt": "Suggest remedy for thick & whitish tongue with bad breath",
"src": "Patient: Hello Sir. I clean my tongue daily.. but then also My tongue has become thick , whitish layer is present on it + when i open my mouth bad breath or smell comes out.Apart from all these now a days i am not able to pronounce words likereached the ed in it ghosts when i say ghosts shhhh kind of sound comes out & many more words I'll be very thankful if u can help me with this Doctor: Hello,Possible reasons for a coating on the tongue include:-yeast infection, thrush-gastric upset-acid reflux-mouth with soft tissue infection-dental abscess draining from a tooth-periodontal disease-mouth breathing-dry mouth-other medications or health issuesBad breath can be associated with these conditions. An increase in the size of your tongue due to inflammation can affect your speech. A growth or tumor is possible. Other disease states can have side effects that could affect the function of your tongue. I am glad that you clean your tongue daily. Be sure to practice good daily dental care. Warm salt water rinses have a cleansing and soothing effect. Be sure to keep well hydrated, proper nutrition, and balanced vitamins. You describe a change in the size of your tongue which could indicate swelling. You should rule out an allergic reaction. Think of any changes you may have made such as using a new dental product or change in your diet. Eliminate any possibilities and evaluate the result.You can try some anti-inflammatory medications such as acetaminophen and advil. An antihistamine such as benedryl may help, but can make you tired. These may reduce your symptoms and offer some relief.There are oral rinses that may be helpful such as chlorohexidine gluconate which reduces bacteria in the mouth or betamethasone sodium phosphate which is a corticosteroid used to reduce mouth soreness.You should schedule a dental exam. A routine exam and cleaning will evaluate your dental health. Periodontal disease and dental infections must be ruled out. Your dentist will review your overall health. Antibiotics or precription medication may be needed. A diagnosis will be needed to decide the best treatment. You want to evaluate for a more serious condition and possible need for a physical or blood tests. I hope this information offers you some relief and thank you for your inquiry."
},
{
"id": 199356,
"tgt": "How to treat saw testicles?",
"src": "Patient: My son is 14 and has been complaining of saw testicles, mainly his left side which has recently brought him to tears. He says that the one that is saw has dropped and is a little red. I have not seen it myself but am concerned. Not sure if it is puberty or something a little more. I have given him some Panadol that helps only slightly. Just wondering if I could get a little bit light on what to do. Doctor: HIWell come to HCMI really appreciate your concern let me tell you that this must not be such condition that could need any medicine but this could be just worry of your son because appearance of testis in this age makes the boy more suspicious and conscious in my opinion better to get done the clinical examination to rule out actual disease if it is ever there, take care."
},
{
"id": 22489,
"tgt": "When is nuclear stress test done?",
"src": "Patient: I have had 2 EKGs in the past motnhs that were both normal. I drank some cold water fast recenty and had what my doctor said was an esphogeal spasm but he ordered a nuclear stress test as a precaution. I had an echo stree test 7 yrs ago (at 50) which was normal. I have no pain or symptons. Is this test really necessary? Doctor: Hi,If you are not an elderly, no chest pain on exertion, no family history, no smoking or hypertension or high cholesterol, then test is not necessary. If some of these risk factors are present, still it may not be needed, but you can have it to be on a safer side.Hope I have answered your query. Let me know if I can assist you further.Regards, Dr. Sagar Makode"
},
{
"id": 135831,
"tgt": "What causes severe headaches after jaw fracture?",
"src": "Patient: Hi, my name is Pamela. I have an old fractured jaw. I have severe headaches. When I open my mouth, the left side of my face, my jaw bone protrudes outwards a 1/2 inch from my ear. Recently, my jaw had shut with considerable force. Yesturday, when I was brushing my teeth, my jaw would not open. I went into the bathroom and leaned into the sink to empty my mouth of toothpaste and then I was able to open my mouth slowly. I m considerably worried now. My problem is much worse and so isn t the pain. I have prior medical problems such as: Systemic Lupus and Angina...etc. Due to these problems, I am no stranger to pain medication. What I find more disturbing is I can still feel the pain of my jaw even while being on Morphine and Percocet. I don t know who I should see. I m not sure what to do. Please, can you educate me on my problem and give me guidance to whom I should see? Thank you in advance, Pamela Doctor: Thanks for your query, I have gone through your query.The pain and difficulty in mouth opening or closing can be because of the problems related to temporomandibular joint. It can be a disc related problem or a malunion of previously fratured bone. Nothing to be panic, consult an oral physician or an oral maxillofacial surgeon and get it evaluated. You need to get radiographs like OPG and MRI done to confirm the diagnosis. Once the diagnosis is confirmed we can treat accordingly.I hope I have answered your query, take care."
},
{
"id": 156293,
"tgt": "Diagonised with Metastesis cancer through upper abdomen & fractured 11th left rib",
"src": "Patient: Hi, my wife is a nurse and she was at work. She bent down to get a food tray in a lower compartment and when she stood up she Herat a pop and pain in her back left side. She went to emergency and did an X-ray and found nothing. Four days later on a ct scan they found a retroperitoneal lymphadenopathy in the aortocaval region. 16days later they did another ct scan and found metastesis cancer through her upper abdomen and a fractured eleventh left rib. Was the rib the triggering mechanism? Doctor: Since CT scan is showing metastasis cancer in upper abdomen the site of primary cancer needs to be found out.I would recommend your wife does a whole body PET CT and a biopsy test along with immunohistochemistry test to find out the primary site of cancer.She is in stage 4 of cancer and now aim of treatment is palliation and improving quality of life.Regards Dr DE"
},
{
"id": 4650,
"tgt": "Is it difficult to conceive when having many follicles, polycystic ovaries? Miscarried first conceived through IUI",
"src": "Patient: I am 33 yrs old and I conceived in first IUI (last year) but it got miscarried and now again we gone for treatment last month treatment got skiped due to functional cyst in my ovary (size-2.2x.....) and I got periods now and on 14thday of my period i.e(19th Nov.-13 I had scan and doctor prescribed that there are many follicles and bil polycystic ovaries and told to visit tommorrow)Kindly reply for me whether any major problem is there if we are having PCOS and difficult for conceiving.What food & fruits I have to intake to increase follicle size Doctor: Hello and welcome to HCM,Polycystic ovarian syndrome (PCOS) is characterized by formation of multiple follicles in the ovaries.In a ovarian cycle multiple follicles develop in the ovary. One of these follicles matures to form dominant follicle which ruptures to release ovum into the fallopian tube.In PCOS, no single follicle matures into dominant follicle.Multiple follicles are formed in the ovaries and none of them releases ovum.Thus, there is difficulty in conceiving.This condition can be treated by drugs.Clomiphene citrate is a drug preparation which is commonly prescribed for this condition.Consult your gynecologist for further investigations and treatment.Thanks and take careDr Shailja P Wahal"
},
{
"id": 137999,
"tgt": "Suggest treatment for severe waist pain",
"src": "Patient: I am having on and off sharp pain (not lasting long) on right side waist area and haven t had a bm since Saturday and today is Wednesday. Started taking Miralax yesterday and had very small bm tonight but very oddly snapped...bm looks as if it squeezed pass something. Curious if the Miralax will get things going or should I go see a doctor? I m not on any meds and no major surgeries in past. Doctor: Hello, I have studied your case.Waist pain can be due to nerve compression. Usually nerve compression cant be seen on x ray you may need MRI for this.There may be age related spondylitis but if there is any nerve compression then that may need specific treatment.I will advise you to do MRI spineTill time, avoid lifting weights, Sit with support to back. You can consult physiotherapist for help.Physiotherapy like ultrasound and interferential therapy will give quick relief.Hope this answers your query. If you have additional questions or follow up queries then please do not hesitate in writing to us. I will be happy to answer your queries. Wishing you good health.Take care."
},
{
"id": 125498,
"tgt": "Any suggestion for knee stiffness and mouth bleed?",
"src": "Patient: my mouth bleeds from 2-3 months from one side . no teath is there and nothing visible just the same one side i have one knee stiff and cannot bear all weight on it . I am afraid of operation , if i delay operation for 1-2 years what bad effects are . My age is 52 years female Doctor: Hello, Knee pain and mouth bleed are totally unrelated. The knee issues might be due to arthritis. As a first line management, you can take analgesics like paracetamol or aceclofenac for pain relief. If symptoms persist better to consult an orthopaedician and get evaluated. Hope I have answered your query. Let me know if I can assist you further. Take care Regards, Dr Shinas Hussain, General & Family Physician"
},
{
"id": 211051,
"tgt": "What could cause hallucinations, forgetfulness and abnormal behaviour, taking host of medicines for various conditions?",
"src": "Patient: My dad is 57 he lost his brother a week ago, shortly after that we had to call an ambulance for him. It was like my dad lost his mind he lost all reality, he don't rember any thing that he done for 4 days. He was shooting people with his fingers, pulling on his clothing, sound sensitive, thought people was going to hurt him would jump when you touched him, praying, mixing past with now and refused to eat for 3 days. The hospital tolled us he had an allergic reaction to a steroid he was on and they released him 2 days later. He was good for a day but now keeps slipping back to being lost and asking many questions. He keeps thinking that we are all trying to make him crazy , he thinks that the kids are testing him but there just playing like little kids, he gets confused really easy and even gets mad for no reason. He's on over 12 different meds for back pain, diabeties, heart problems, and depression. This is not the first time its happened. Its happened once before and they gave us no diagnosis then either. He's had a cat scan and it showed nothing, his blood test only showed a low electrolites and they said it was from a sinous infection. Is it some thing we should be worried about or get a second oppionion on, are they missing something?. Doctor: HiThanks for using healthcare magicI think your father has acute transient psychotic episode for which he needs a antipsychotic drug rather than antidepressant. Low electrolyte could be due to antidepressant side effect. You should consult a psychiatrist for his proper evaluation and in case you need our help, you can connect us.RegardsDr. Abhishek Kapoor"
},
{
"id": 128408,
"tgt": "What causes knee pain along with numbness in the thigh?",
"src": "Patient: Hello, My name is Katarina I had a questions about a knee injury well I hurt my knee back in Dec of last year while trying to get in a high bed when I was pushing my self up in the bed guessing I came down on my knee the wrong way since den its been feeling like its been sliding out of place if I make a wrong movement but it will cause me to fall off my feet with bad swelling in the knee I recently just had the same problems but lot worst went to the ER and the MD explain to me it was a fracture ligament not really sure which one they didn t say but the pains come from the left side of my right knee...it hurts when I climb and know im feeling numbness in my thigh and pain in my ankle to my thigh ...could give me answers on what you think it coud be and if the numbness can be something major Doctor: Dear patient you seem to have torn cruciate ligaments most likely anterior cruciate ligament. Both anterior and posterior cruciate ligaments are present in the joint and helps in maintaining stability of the joint. If not of them is torn there will be instability. Diagnosis needs to be confirmed by mri of the involved knee. it is very specific and sensitive for diagnosis of soft tissue injuries. Please get it done from radiology centre nearby you. Meanwhile start use of Long knee extension brace while walking. You need to consult arthroscopy surgeon nearby your area with report."
},
{
"id": 19264,
"tgt": "What causes heart beat feeling with cardiac ablation?",
"src": "Patient: why can I hear my heart beat all night long and during the day if not busy. I had a cardiac ablation 3 months ago for pvc's and three weeks later i eas in bigeminy again. I am now taking rhyrhmol 225mg sr which has me in rhythm and my blood pressure usually runs on the low side. Doctor: welcome to hcm ..please note that bigeminy and pvc etc give rise to abnormal heart beats and forceful heart contractions ..these are felt by some personsif rhythm is in control the low bp is due to drug effect if you are troubled by low bp as you feel dizziness or lightheadedness as you stand from sitting position then you should consult your doctor he may adjust dose of drug"
},
{
"id": 169652,
"tgt": "What are the risk of swallowing hair conditioner?",
"src": "Patient: My 9 month old may have swallowed hair conditioner. She spit up a couple time before I found the small foil thing they put over the top of it. I don t know what it came from other than the little conditioners that come with store bought hair color or if it was accidentally dropped on the floor. What do I look for incase she did swallow anything? Doctor: hi, nothing to worry about it will go out from her potty area. its very little pungent in taste and is mostly harmless. so dont worry just give water and fluids to drink."
},
{
"id": 69128,
"tgt": "What is the lump on the thigh following an accident?",
"src": "Patient: I am a 23 year old male. Lately I've been getting really bad migraines, trouble sleeping, and dizziness. About 16 hours ago I realized I got this lump or bump on my inner upper thigh. I had an accident when I was 11 years old where I had to get surgery because a testicle popped inside me. There was a complications with the surgery I had which had an outcome of me overproducing testosterone. Since then my mother was informed that I would possibly have aggression problems, depression, and headaches. My question is, would this lump be related to my accident many years ago? Doctor: Hello!Thank you for the query.It is possible that this lump is related to the surgery you have had many years ago. It might be a femoral hernia. However it should be movable and get smaller when lying down.More possible is that this is something not related to the accident. It might be some benign skin lesion like lipoma. The best way to find out what are you dealing with is to have an ultrasound of this area.Hope this will help.Regards."
},
{
"id": 103588,
"tgt": "Back pain after eating fruit. Any ideas?",
"src": "Patient: I am 33 years old and ever since the age of ten I've had a strange issue that couldn't be diagnosed. Every time I drank apple juice over the years my back would hurt so badly afterward that I have been afraid to even attempt it over the last ten years or so. Other things started to have he same effect about ten years ago. I also cannot drink fruit punch or eat apples. This evening I cut up a watermelon for my kids and it looked so good that I decided to have some myself and about half an hour later, here I sit in bed with excruciating back pain that does not seem to involve my stomach. It wraps around my rib cage but is most painful in my mid to upper back. I would love to figure this out as I am actually afraid of what certain foods can do to me. Any ideas would be greatly appreciated! Thanks, Jennifer L. Smith Doctor: IT IS FOOD ALLERGY IF YOU FIND OUT CAUSE IT WILL BE GOOD AS ELIMINATION IS NECESSARY FOR TREATMENTMOST LIKELY MILK BEHAVES LIKE THIS MANY PERSONS ARE EFFECTED LIKE THIS WHEN IN INITIAL STAGES IT LOOKS THAT ONLY ONE YTHING REACTING AND AS TIME GOES IT SEEMS EVERYTHING IS REACTINGJUST REMOVE MILK AND DIARY FROM YOUR DIET COMPLETELY NOTHING WILL REACT AFTER 3 WKBUT IF NO RESULT GO FOR TESTS AND ELIMINATE FROM DIETS"
},
{
"id": 155048,
"tgt": "Can I get a cancer prescription online?",
"src": "Patient: i recently moved to a dual area and have no doctor. over the last few years i have(and still do) struggle with kidney problems as well as cervical cancer, I ve been on a transplant list for quite some time now, and I ve been needing to refill my roxicodone&other medications. is it possible to legally online get a prescription without having to travel far to a doctor so i can continue to walk and manage a somewhat normal lifestyle? Doctor: Hi, dearI have gone through your question. I can understand your concern. Your treating doctor's can give you prescription online. From health care magic we can not give you prescription. Legal issues is also a problem. Just ask your treating doctor to prescribe drugs. Hope I have answered your question, if you have doubt then I will be happy to answer. Thanks for using health care magic. Wish you a very good health."
},
{
"id": 214874,
"tgt": "Had stepped on rusted nail, punctured foot, cleaned with hydrogen peroxide, bandaged. Home remedy or visit doctor?",
"src": "Patient: hi I stepped on a rusty nail on saturday afternoon and I barely punctured my foot bottom. I immediately cleaned it with hyrogen peroxide and put a bandage on it, I am currently walking ok on it and I barely feels any pain. my question is do I need to visit a doctor anyway or is there any other home remedies I can use?thanks in advance Doctor: Hi and . It is recommended to have antitetanus shot after such injuries within 3 days.But you are probably vaccinated. Next time go to ER right after injury. Now, you can treat it at home. Keep the wound clean and if you notice swelling,redness or pusy discharge you should visit doctor because these are signs of inflammation. If there are no such signs it will heal in 5-7 days. Wish you good health."
},
{
"id": 55326,
"tgt": "What is the treatment for liver cirrhosis?",
"src": "Patient: Hi, My dad was diagnosed with liver sirrhosis and hospitalised when he had esophageal bleeding. Now he is in hospital, he is getting well but we are scared of how long can he live and what will be the cure for him. He never had alcohol in his life and even smoking he stopped when he was 30. Please respond to my query Doctor: I thanks for asking question.I appreciate your concern.First of all there is no permanent cure of liver cirrhosis except liver transplantation..But we can prevent its complication.Here are few suggestion for you.Aspirin, ibuprofen , some antibiotic can damage liver ,so taken only after consulting doctor.Take low salt diet which will help in reducing blood pressure and preventing ascites.If fluid accumulate in belly means ascitis , diuretic can be taken.To treat variceal bleeding endoscopy treatment or balloon temponade done.Beta blocker especially useful in preventing variceal bleeding.To prevent ascitis limit your protein in diet.Take enough rest and avoid stress.Regularly visit your doctor.If variceal bleed occur immediately refer to doctor.With these suggestion he will be in less trouble.I hope these will help you.Wish you good health.DR parth"
},
{
"id": 136193,
"tgt": "Does yoga help regain flexibility post a hip surgery?",
"src": "Patient: I recently had drilling surgery bilaterally on hips, knees, and ankles. I find that after a month from surgery, i am still in immense pain in my knees. Would yoga be a suitable therapy treatment in helping with regaining my range of motion and flexibility? Doctor: HelloYes you may get relief with yoga therapy.Avoid full flexion and rotationYou may have good relief with physiotherapy like IFT [Interferential therapy] and TENS[Transcutaneous electrostimulation] along with ultrasound.So if possible you can start that.You can wait for a month more and then you may consult your doctor.Hope this answers your query. If you have additional questions or follow up queries then please do not hesitate in writing to us. I will be happy to answer your queries. Wishing you good health.Take care."
},
{
"id": 213843,
"tgt": "Is Hydergine effective in treatment of Alzheimer's Disorder?",
"src": "Patient: Hello Doctor , My mom is suffering from Alzheimer's Disorder. She had been prescribed Hydergine drug by the doctor. However, the drug is no longer available in the market. Can you tell me whether Hydergine is effective in treatment of Alzheimer's Disorder. So I can request the manufacturer to provide me the same. Also for my knowledge please inform me what are the therapeutic indications in which you prescribe Hydergine Thank you Doctor: Hi,thanks for query.This is a vasodialator drugs means it improves blood flow the part.Usefullness is to be decided by the treating doctor.Please visit the following link for more information and other brands. wishing her good health. http://www.drugsupdate.com/brand/generic/Co-dergocrine%20mesylate/11525"
},
{
"id": 180646,
"tgt": "What causes skin peeling inside the mouth?",
"src": "Patient: When I use mouth wash at night, the following morning, I have what seems like shreds from the inside of my mouth that are on my tongue and between my cheek and gum. I can rub my finger along the inside of my mouth and wipe it out and it almost looks like string cheese. There is no pain, bleeding or anything, just what seems to be dead skin from the inside of my mouth. Doctor: Hello,String cheese like material that is present on the tongue and between the cheeks and the gums looks more like a yeast infection causing oral thrush. So my suggestion is to consult an oral physician and get evaluated. In case if it is sloughing of the mucosa. Then the use of mouthwash should be immediately stopped. If it is fungal infection then you shall be advised antifungal mouthwash like nystatin and clotrimazole lozenges. Oral antifungal medicines like fluconazole or Itraconazole can also be advised.Hope I have answered your query. Let me know if I can assist you further.Regards,Dr. Honey Arora"
},
{
"id": 186820,
"tgt": "Could the lump on the lower jaw bone with severe pain due to the fracture?",
"src": "Patient: i was hit in the jaw the other day. i was unable to open my mouth for several days and although it still hurts, the pain severity has lessened. however, there is still a large lump on what feels like the bottom jawbone. could it be fractured and should i get an xray? Doctor: Hello, Welcome Thanks for consulting HCM, I have gone through your query, as you have injury on lower jaw now you have lump on jaw bone dont worry yes it can be due to injury in periodontal ligament may be there is root fracture in tooth that cause Periapical pathology or hair line fracture of segment of mandibular bone . For this you should consult dentist and go for oral examination and Investigations done OPG XRay . If there is periapical pathology then go for Root canal treatment or Apicectomy and if there is fracture of bone then you should go for splinting .Hope this will help you"
},
{
"id": 101048,
"tgt": "Does wikoryl help to treat coldness and fever?",
"src": "Patient: Hi Sir, I am suffering from cold and fever and i think it happened due to inhalation of lot of dust in my factiry,i am planning to take wikoryl 500 mg with ampoxin 500.So is the dosage ok or i should make some changes in it?My age is 26 years with weight 75 kg and height 174 cm. Doctor: HI, thanks for using healthcare magicYou can use the wikoryl which is an anti histamine and would help to reduce the allergic reaction but the ampoxin is an antibiotic and would only be effective in a bacterial infection.Colds are usually viral not bacterial so the antibiotics would not be needed for the normal common cold.A decongestant would help to reduce any nasal congestion. This can be oral or topical drops for the nose. Should be available at your local pharmacy.I hope this helps"
},
{
"id": 52084,
"tgt": "Will Clarithromyc treat a UTI ?",
"src": "Patient: Will Clarithromyc 500 mg treat a UTI ? Doctor: no. you need to have urine culture sensitive antibiotics and also urologist to let you know of the right antibiotic"
},
{
"id": 46466,
"tgt": "Can accident be the reason for high protein level due to kidney damage?",
"src": "Patient: Hi, I had a kidney transplant four years ago and have been doing great until I found out yesterday that I now have a very high level of protein in my urine and must have a biopsy tomorrow at the hospital. I had a car accident where I was trapped in by the seatbelt hanging sideways on my transplant kidney side...this was about six to eight weeks ago...I also had a fall from a set of stair a few weeks before that where I landed on my transplant part of my belly, pretty hard. Can either of these types of occurrence cause kidney damage resulting in the high protein levels? Doctor: HelloThanks for query .You have undergone kidney transplant 4 years back and have been detected to have high level of Proteinuria which you are suspecting it to be due to damage to the kidney resulting from trauma that you have sustained twice during last 4 months at the site of surgery (Transplanted kidney).Please note that trauma to the kidney will be reflected by passing blood in urine immediately after trauma where as proteinuria is due pathology in filtering unit of kidney (Glomeruli) which can be detected only on microscopic examination which needs to be confirmed by doing Renal Biopsy .Dr.Patil."
},
{
"id": 225503,
"tgt": "Taken pill. Is it possible to get pregnant, having sex on 3rd day of periods?",
"src": "Patient: I had unprotected sex with my partner on 3rd day of my periods and had unwanted-72 within an hour. I continued mensurating evn after that. Nowvits been a month nd i havnt got periods yet. I m hvg a vomitng tendency. I m confirmed tht whether it is due to pregnency or because m scared. Are der chances of getng pregnant?? Doctor: Hello,Very frankly speaking,you can not be pregnant because 1st you had unprotected sex during your periods,which is the safest period and 2nd you took unwanted 72 within an hour.Relax as pregnancy can not take place.As far as delay in your periods is concerned,it can be possible because of various reasons and the most important is STRESS.take good sleep,drink plenty of water and eat healthy and wait for a week.If you don't get it,you can consult a doctor who can prescribe you progesterones,which can induce periods in you.Thanks"
},
{
"id": 30715,
"tgt": "Suggest treatment for tumors",
"src": "Patient: My husband has breast cancer that has gone into lymph nodes and bones in his back and pelvis. He had problems with his left shoulder which was the same side of his mastectomy. He had pet scans and the cancer dr didn't think his pain was cancer related. He was sent to an orthopedic dr who gave him a cortisone shot which didn't help. Now after doing another MRI they found that there are tumors. Wouldn't this have shown in the Pet scan?? Could the cortisone shots have made it worse?? Why wouldn't the cancer doctor have found this?? Doctor: Thanks for posting your query to HCM.there are so many factor responsible for relapse for breast cancer. some times small tumor cells can be removed during surgery and they may be responsible for recurrence .bone pain may be due to metastasis of tumor cells to that site . You need to consult oncologist to receive chemotherapy regulary .Steeroid are not associated with this spread but you should not take streoid to releive pain .take care ."
},
{
"id": 17264,
"tgt": "Is angiogram required to diagnose coronary artery disease?",
"src": "Patient: Sir: I am past 60 who is on atenlol,statin & anticoagulants as advised by a cardiologist. I do not have any symptoms. I am advised to have an angiogram at a convenient time. I donot want to do that . What if I continue with the medicines and frequent check up , investigations etc ? Doctor: Hi, Cardiac catheterization and coronary angiography (CAG) provides a detailed hemodynamic and anatomic assessment of the heart and coronary arteries. If you are apprehensive of conventional CAG, you can opt for Coronary CT angiography in place of invasive coronary catheterization. Hope I have answered your query. Let me know if I can assist you further. Regards, Dr. Tushar Kanti Biswas, Internal Medicine Specialist"
},
{
"id": 159948,
"tgt": "Can you suggest the best hospital to treat mouth cancer ?",
"src": "Patient: in search of a best hospital in india for mouth cancer treatment Hi, i want to know which is the best hospital in india for mouth cancer treatment? Doctor: thanks for choosing health care magic.See there are many institutions and even private doctors are doing very effective and good treatment of mouth cancer.so depending upon where you are based this can be suggested.Please contact and consult your primary physician who will be having this knowledge."
},
{
"id": 169035,
"tgt": "Suggest if taking 2 month old baby to campfire is safe",
"src": "Patient: Is it harmful to have a 2 month year old baby that has had no health problems at all around campfires? Me and my family are trying to go camping tomorrow and we just are trying to make sure. And if it is unhealthy for long periods at a time would a couple hours be okay for visiting? Doctor: How is the weather outside? do not expose to bad weather and ofcourse to camp fire smoke as bot are irritant to respiratory tract."
},
{
"id": 183867,
"tgt": "What causes gum disease?",
"src": "Patient: I m 16 and height 5.1ft and weight 50 medical history nothing diagnosed as yet. Hi my inquiry is I ve had 4 panic attacks this month where I ve been rushed into hospital and since this I ve have had cold sensations in my chest for over 4 weeks now since it happened and vomiting blood I ve seen my Gp but he seems to say I m healthy but I ve been recently very depressed. And have had problems with eating and have suffered with gum disease I ve had a blood test today to check for any allergies but I don t feel this has anything to do with my chest and feeling weak and dizziness all the time. Doctor: HIThank for asking to HCMI really appreciate your concern and looking to the history given here I could say that you have not mentioned the disease of the gum in specific what type of disease you have for the gum, any way in my advise better to have a fresh fruits and vegetables, if the condition persist then better to see the dentist, have a nice day."
},
{
"id": 222397,
"tgt": "Is it possible to get pregnant without penetrative sex?",
"src": "Patient: heelloo..yesterday me and my bf had a sex...but we both r with clothes on..he wear a night suit and i have wear a dress..he rub his penis arond virgina .... i dnt know that he get the sperm or not...bur my underwear get wet..so is it possible that i might get pregnant???? Doctor: Hi,If this sex play done during fertile period, there are all chances of having pregnancy as even few sperms entered into vagina might give rise pregnancy.Go for pregnancy test if you are late for 7-8 days.Ok and take care."
},
{
"id": 42897,
"tgt": "Suggest treatment for infertility",
"src": "Patient: Hi me and my husband have been trying to have a baby for last 6 months. We both are working at different place and not living together. I am sure atleast 3 times we were in contact during the week of my ovulation. Last month i had consulted my gynaecologist and did follicular study. On day 13th the size was 18 mm. She gave an injection and we kept contact for next 3 days. But That month also i got my periods. Can u guide me on next step. Is too early for me to go to doctor for check ups for me and my husband. Doctor: Dear member,Thanks for writing.The chance of conceiving in each cycle is only 15-20% in healthy couples.In your case you both are also staying away and meet less often.Be sides you are trying since only 6 months. There is a good possibility of you conceiving in few months as your ultrasound showed that you ovulated normally.you can use ovulation detection kits at home to time intercourse with ovulation.Please do basic blood investigation like hemoglobin, thyroid profile, sugar test.Husband semen analysis also to be done.If normal no need to worry. Please write back with report.ThanksDr bhagyashree"
},
{
"id": 211676,
"tgt": "Suffering from mental pressure. Solution?",
"src": "Patient: i have alway mental pressure when some boby rebuke me or say some bad word. But some time i have no reason and i have greatest mental pressure. with the cause of my mental pressure(tension) i never like people. i live alone and my age is 19 years. it is going on from 4 years. i have no friends. i nerver like my brothers my mother. i mean no one. In those 4 years i always watching a movie. please solve my problem. i will be very thankful to you. Doctor: I must be greatful to you for seeking help for yourself by asking a question. this is the beginning of your treatmentYou will clearly improve in \"inner happiness\", \"functioning\", \"academic performance\", \"excessively watching movie\" etc., if you are in regular touch with a psychotherapist / psychiatrist.we often such cases like you, yours is nothing uncommonYou may be benefited by medication, that will be decided after thorough assessment of your life from birth to till date, and of your mind, by your psychiatristI presume, you would worsen, if you do not seek treatmentKindly, do not treat this answer as a general nonspecific answer. I would be most happy if you give a serious thought to this discussionWish you a happy life"
},
{
"id": 83105,
"tgt": "Blood pressure high, lupus, sjogren's, shortness of breath",
"src": "Patient: I got my blood pressure checked at a fair at work today. My blood pressure was 115/84. My resting heart rate was 102. The RN there suggested I call my doc because of the resting heart rate being high. I am a 27 year old female with Lupus , Sjogrens. I ve been getting shortness of breath lately, out of breath walking to the parking lot right outside my office. Should I be worried? Doctor: You should definitely consult for symptoms of shortness of breath. It could be related to sjogrens."
},
{
"id": 59595,
"tgt": "Frequent nausea. Prescribed metaclopramide, no relief. Blood tests normal. Due to gall stones?",
"src": "Patient: I have been feeling nauseous on a pretty well daily basis for about three months. I m never actually sick however. I ve had blood tests - all OK. I ve had a gastroscopy which showed nothing but a small flat polyp which was removed at the same time. I m due a follow-up on this at an out-patients clinic later this month. I m also due an ultrasound scan as my GP thinks it might be gallstones, however I have no pain in my digestive system. I m prescribed Metaclopramide but this does not really have much impact. It s really getting me down now. Doctor: Hello, brucea1, Persistent nausea could be a result of inflammation of the stomach lining. You mention that the doctor found a small polyp and removed it. Usually, these are called Hyperplastic polyps or inflammatory polyps. In other words this happens from inflammation of the lining of the stomach and we call it \"Chronic Gastritis\". Metaclopramide or Reglan is a drug that improves contractions of the stomach and helps emptying of gastric contents into the intestine and it for control of nausea. It does have potential side effects such as shakiness of the hands, almost like Parkinson like symptoms in some, especially with higher doses. If you are not on any PPI like Omeprazole, Lansoprazole, Rabeprazole or Esomeprazole, these are available over the counter in drug stores. Try taking one of these once or twice a day about an hour to an hour and a half before meals. This should help. I am glad that you are scheduled for an ultrasound exam to rule out gallstones. I wish you well."
},
{
"id": 73446,
"tgt": "How can emphysema causing breathlessness be treated?",
"src": "Patient: I have emphysema. Nearly a week ago I recovered from nearly six weeks of bronchitis and sinusitis. I still become much more breathless than I did before this, with minimal exertion. A couple months ago I could walk a couple miles quickly. I still have problems with a couple blocks. What can I do? Doctor: Thanks for your question on Healthcare Magic.I can understand your concern.Emphysema is chronic progressive lung disease.It is further worsen by respiratory tract infection.So your current symptoms are due to worsening of emphysema.So you should take inhaled bronchodilators (formoterol or salmeterol) and inhaled corticosteroid (ICS) (budesonide or fluticasone) regularly. Get done PFT (Pulmonary Function Test) every six monthly to know the progress of disease.Hope I have solved your query. I will be happy to help you further. Wish you good health. Thanks."
},
{
"id": 94339,
"tgt": "Low grade fever, increased WBC count and bilirubin. Normal? Had appendectomy",
"src": "Patient: First week after appendectomy: I had headaches , nausea and constipation but was advised to stop the pain killers and symptoms stopped. End of second week: I had a burning sensation after eating followed by high fever 38C + which made me go to the hospital. They gave me medication to balance the gastric liquids - tests showed no signs of infection. On the third week after appendectomy, I have been having low grade fever (37,2 - 37,7) for the last 5 days. WBC went up to 10,000 in the first week but then reduced to 6,400. Tests and abdominal scan don t show that there are no signs of an infection. Billirubine total is a bit high. No tests for thyroid / hormones levels. Is it normal to have low grade fever? For how long after the operation? Please advise. Doctor: Hello, mardin78160, I am assuming that your surgery was done through laparoscopic approach and not open surgery. If , at the time of surgery you had a ruptured appendix with a little bit of infection of the covering (Peritonitis) , it is possible that you could still have a small pocket of hidden infection. Toatal bilirubin being a little high with the indirect fraction more than the direct fraction may indicate a little hemolysis or destruction of red blood cells which is not very serious and is temporary. I am sure that they did blood cultures also. You may be on some antibiotics. So long as they are keeping a close watch , I would not worry. Keep the hydration good. I wish you well."
},
{
"id": 8077,
"tgt": "Does tooth paste work good on pimple ?",
"src": "Patient: Hi, I m having pimple break out and everyone told me to use toothpaste on it. will it works? I have a important meeting in 3 days I need make my pimples disappear asap. im using brevoxyl on my pimples now. is auriderm xo vitamine k effective for pimple marks and dark scars? Doctor: hi rachel toothpaste has no role in acne,it will make it worse. brevoxyl is good one,u have to apply over pimple. its better if u mix CLINDAC -A GEL with brevoxyl. it will give quick rresult. vitamin k has minor role in scar. use face wash containing salicylic . drink more water."
},
{
"id": 154110,
"tgt": "What is the treatment for carcinoma tongue?",
"src": "Patient: Dear sir,I am K.Jeyakumar(46years,Male) of south India. I had wound and pain in my tongue left side for the past two months I have been diagnosed as a case of Carcinoma Tongue, through MRI diagnostic investigation.I need your valuable advice. MRI OF NECKTECNIQUE: Axial,sagittal, coronal SET1 and TSE T2 weighted sequences.OBSERVATION:The study shows presence of an irregular ulceration of left lateral border of posterior third and adfacent undersurface of tongue, with thickening and induration of tongue muscles and adjacent of of soft tissues. The lesion appears isointense on T1 and slightlyhyperintense on T2, IR weighted sequencesw. The lesion measure about 3.0X1.7X0.9cms in size.Base, right half of tongue, floor of mouth , oral, buccal, palatal,submandibular soft tissues and RMT show no involvement. No mandibular or maxillary alveolar bony erosions or marrow oedema are seen.Buccinator, ptergoid muscles, nasopharynx, oropharynx, hypopharynx, epiglottis, vallecula, palate, supra glottic, glottic, subglottic regions, trachea, pyriform sinuses, parotid, submandibular and thyroid glands appear normal.No evidence of cervical or supra clavidular lymphadenopathy is seen. Carotid, jugular vessels, sterno mastoid, other muscles, pre and para vertebral soft tissues appear normal.Cervical spondylotic changes are seen. No evildence of bony erosions are seen. Hyoid bone, other bones and cervical spine appear otherwise normal.IMPRESSION: MRI OF NECK SHOWS,SMALL IRREGULAR ULCERATIVE SOFT TISSUE LESION, INVOLVING LEEFT LATERAL ASPECT AND UNDER SURFACE OF TONGUE - SUGGESTIVE OF LIKELY MALIGNANT GROWTH OF TONGUE.NO EVIDENCE OF BASE, FLOOR OF MOUTH, ADJACENT SRTUCTURES INVOLVEMENT, BONY EROSIONS, CERVICAL OR SUPRA CLAVICULAR LYMPHADENOPATHY IS SEEN. Doctor: Hi,Thanks for writing in.The MRI scan says that there is a 3.0 x 1.7 x 0.9 cm mass in the posterior third of tongue on left side. This is a cancerous mass and causing firmness in the muscles attached to the lower portion of the tongue.In the above case, the tumor appears to be stage 3 cancer of tongue. If possible, wide excision surgery is to be done and this is to be followed by multiple cycles of chemotherapy and radiation therapy.As the response rate of tongue cancer is good, there are chances of cure and recovery after treatment. There are no lymphnode metastasis mentioned in your report and therefore it is still not spread. Please take complete treatment and closely follow up with your doctor. Please do not worry."
},
{
"id": 2635,
"tgt": "Is it safe to be pregnant after repeated miscarriages?",
"src": "Patient: Hello MadamMy Wife had 2 time misscarrage, once in august 2012, n second agust 2013, my doctor advise for tourch test n othe more test, all test result postive, n now my doctor gave me escosprin 75, n folic acid, told to keep contct it is safe again for being pregant as her doctor advice for it, Doctor: Hi,It is safe to get pregnant again after being treated for the torch infections. You should repeat the test and go for pregnancy if the test came back negative for recent infection.Hope I have answered your query. Let me know if I can assist you further. Regards,Dr. Salah Saad Shoman"
},
{
"id": 49301,
"tgt": "What could be the reason for the urine to smell along with stained discharge?",
"src": "Patient: I have noticed recently that when I urinate it smells funny Its never smelt when I ve gone toilet till recently, I have also sometimes had a reddy discharge when i pee but not very often not aure if it has anything to do with the smell i just want to know what does it mean ??? Doctor: Good Day and thank you for being with Healthcare Magic! You may have a urinary tract infection. The only way to be sure is to get your urine examined with a routine urinalysis. That way we will know if you have infection or if there are other sediments in your urine. It will also help to drink plenty of fluids around 2 liters of water a day would always do you good. I hope I have succeeded in providing the information you were looking for. Please feel free to write back to me for any further clarifications at: http://www.HealthcareMagic.com/doctors/dr-manuel-c-see-iv/66014 I would gladly help you. Best wishes.Regards, Manuel C. See IV, M.D. DPBU FPUA"
},
{
"id": 55459,
"tgt": "What causes hepatitis B?",
"src": "Patient: my wife came up and told me she was Hep. b positive and wanted me test, and she does not know how she got it.. I went to a doctors and got checked up and i came up negative in fact, i am immune can t get hep b ever. So my question is how can she get it if there is no foul play? to add we have not had sexual intercourse without condoms.... Doctor: HiSorry to hear your problem.your wife hep B positive and you are negative.more ever you are saying you are immune.its very unlikely to spread from your wife as you are immune.there are so many ways for spread.blood and blood product transfusion,contact of infected persons secreations into open cut or abraded skin / mucosa, sexual relation,from mother to baby & very rarely from sweat.so your wife might have got any of those above ways.better to keep low viral load to reduce her infectivity.evalute hep B viral serology to know acute or chronic and to know viral load.if high viral load better to take treatment with lamivudine / interferon.Thank youVasundhara"
},
{
"id": 138759,
"tgt": "What is the best medicine for knee pain and hip pain?",
"src": "Patient: I was taken off of Bonivia about 3 months ago by my doctor after being on it for 7 years. I started having hip pain and knee pain and the pain has gradually increased as time has gone on. I m an active 62 year old woman The pain wakes me up at night. is this normal when taken off this medicine. Doctor: HIWell come to HCMAppreciate this Bonivia used to treat the osteoporosis in menopause and not for pain.Underlying cause of knee pain is matter of concern.Pain is not due to the medicine taken off.Advise:Do not bend (Flexion) of knee joint beyond 90 degreeMRI could reveal something Avoid standing for long time Blood sugar need to be done to rule out diabetes Medication:Tab Diclofenac 50 mg once in dayTake care, have a nice day"
},
{
"id": 80552,
"tgt": "What is the remedy for the sore throat and lung infection with white spots near the tonsils?",
"src": "Patient: I have had sore throat since last Monday, I have trouble breathing since Tuesday morning. Saw doctor that evening and he said i have throat and lung infection and gave me antibiotics. Today is Sunday and I am still coughing, no more runny nose, at times I cough out pleghm with blood and I notice a tiny black dot on my throat and 2 white spots (look like ulcer) inner throat near tonsil. What could this mean? Doctor: Hello dear, thanks for your question on HCM. I can understand your situation and problem. You are having haemoptysis ( blood in sputum ). And it is seen in following conditions. 1. Tuberculosis2. Pneumonia3. Bronchitis4. Lung cancer etc.So better to get done1. Chest x ray. 2. PFT ( pulmonary function test ). 3. CT thorax if required. Chest x ray and CT thorax are needed to rule out tuberculosis, pneumonia and lung cancer. PFT is needed to rule out bronchitis. So better to first diagnose yourself and then start appropriate treatment."
},
{
"id": 40332,
"tgt": "What is the remedy for skin infection?",
"src": "Patient: Dear Doctor, Myself Pradipa Das from Bhubaneswar,Odisha.I have been suffering from skin infection in my left hand palm concavity,just like fungal infection.It aggravates after cleaning of clothes by detergents and use of hair oil.It produces dry scales and cracks even upto bleeding of skin.It does not show any irritation,no spreading,no itching ,but does not heal since three years.I have used so many antifungal creams,ointments,gels but of no use.Now I am continuing Xerina ointment and Blisderm ointment at night since one month.Only it is suppressive but no complete cure.If oneday I will not use,again the same dry,scally and crack scenario.For this I feel very uneasy and shy. Hence, humbly requested to suggest any ointment or cream from your expertisation for which I shall be grateful to you. With regards and thanks. Pradipa Kumar das Bhubaneswar odisha Doctor: Use clotrimazole cream mixed with triamcinolone ointment two times a day. use warm soaks to your hands and then after area is soft apply cream. good luck!"
},
{
"id": 187709,
"tgt": "What relieves the pain in the jaw due to which neck movement is painful?",
"src": "Patient: hi. i have pain in m left side of jaw by ear area. i have difficulty moving my neck to the left. . have not injured my jaw. i feel headachey on left side. painful to eat or open my mouth wide. it does sound crunch.y. no pain releif helps. this has now been for a few weeks. Doctor: Hello, Welcome Thanks for consulting HCM, You have mentioned that you have pain in your left side of jaw and area of ear and difficulty in opening of mouth , dont worry you have Temporomandibular disorder left side take this medication for 1week you will get relief, Tablet Flexon MR one tab in a day Ointment Diclofenac sodium Local application twice daily on left side of face where you feel pain Capsule vitamin B complex one capsule in a day Some points you should remember are take soft diet , and avoid excess mouth opening. If your problem continues then consult physician. Hope this will help you."
},
{
"id": 155388,
"tgt": "What are the survival rate of a patient suffering from multiple myloma?",
"src": "Patient: One of my relative has Multiple Myloma. His age is 72 years, 5 8 height and will be around 70 kg of weight. Along with many test, he had the bone marrow. In which it was detected that its around 88%. What should be further course of action and what are the chances of his survival? Doctor: Thanks for your question on HCM. Multiple myeloma is tumour affecting bone marrow. So it is rapidly spreading and very notorious in course.Usually it is responding to the chemotherapy. And pain in bones are treated by local radiotherapy. Surgery is not indicated as most of the time,at the time of detection Iit is already metmetastatize to distal organs. So chemotherapy is the corner stone of the treatment. And multiple myeloma has various types. And prognosis is based on types.Usual five year survival rate is 60-70%. And life expectancy is usually more than 4-5 years."
},
{
"id": 59473,
"tgt": "SGOT, SGPT out of range, weight increase, on Udiliv. Other tests normal. What to do ?",
"src": "Patient: HI DOCTOR FROM THREE YEARS MY SGOT AND SGPT ARE OUT OF RANGE BUT AFTER CONSULTING DOCTOR I GOT THEM REDUCED TO SGOT AROUND 60 AND SGPT AROUND 80 BUT RECENTLY MY WEIGHT WAS HIGH AND I TESTED IT CAME ALT 258 AND AST 114 AND MY WEIGHT IS AROUND 69 AND MY HEIGHT IS 5 FT 3 INCH. MY OTHER TESTS IN LFT AND LIPID PROFILE IS ABSOLUTELY NORMAL . I HAD BEEN TAKING UDILIV THAT HAS HELPED ME BUT I HAVE NOT CONTINUED IT.PLEASE ADVISE ME. FROM AAAAAA : KASHMIR Doctor: Hello, Alcohol consumption is a very important cause for high liver enzymes.\u00a0 There is no specific medicine to bring down your SGOT and SGPT levels. Although UDILIV is prescribed for this by many doctors,but it's role is doubted by many others. however there are ways to reduce your high liver enzymes.These are... 1... Stop alcohol totally,if you are taking it. 2.. Regular aerobic exercises for 40 minutes a day \u00a0\u00a0 3.., avoiding junk foods like maggie,noodles,pasta,pizza,white bread,burgers and colas. \u00a0\u00a0 4.. Consumption of green leafy vegetables and fresh fruits. \u00a0\u00a0 5.. Avoiding deep fried and fatty foods \u00a0\u00a0 6.. Take super B complex vitamin along with 1000 mg VIT C,once a day. \u00a0\u00a0 7.. Keep your weight in your required BMI range. \u00a0\u00a0 8... Avoid sweets and colas. Your SGOT and SGPT levels would \u00a0become normal in 3 to 6 months time. \u00a0 Thanks"
},
{
"id": 195181,
"tgt": "How can phimosis be treated?",
"src": "Patient: hi i am 46 , i have normal sex life no issues , i was dignosed with diabities 3 years back , initially all was normall suddenly i was not able to pull my forskin back not it has become very tight I am not able to have sex due to this m and if i try to pull back i get cracks in the skin and it very painful. Doctor: Hi, The only effective treatment option available for treatment of phimosis is surgical removal of foreskin. Hope I have answered your query. Let me know if I can assist you further."
},
{
"id": 100893,
"tgt": "Suggest treatment for lumps appeared due to allergic reaction to hair dye",
"src": "Patient: I have had an allergic reaction to hair dye. I have taken antihistamine, It started off with 2 lumps 1 either side of the back of my neck. This morning I woke up with quite a large lump behind my ear and some throughout the bottom of my scalp. I am worried as they wasn t there this morning. What should I do and what could this be? Is it serious? Doctor: Hi, thanks for using healthcare magicThe lymph nodes are present in different points in the body. They respond to infection and inflammation present with the vicinity of their location.There are lymph nodes in the neck, in the area of the chin.They would have responded to the allergic reaction on your scalp.You can consider using anti inflammatory pain killers such as ibuprofen, cataflam, aspirin or naproxen.In addition the use of anti histamines would also be very helpful. This would include meds such as benadyl, allegra, zyrtec, claritine, aerius.I hope this helps"
},
{
"id": 145468,
"tgt": "What causes dizziness along with heaviness in leg?",
"src": "Patient: My medical history over the past few months has been kind of rocky, but the main thing I am concerned about is a finding on an MRI done on my brain. The results say a pineal cyst measuring 4.4x10.5 mm was found with no other abnormalities. My appointment with the neurologist is not until the third. I believe it to be symptomatic. Can you share any knowledge with me about what symptoms these can cause and if it could relate to my dizziness and heavy feeling throughout my legs? Thanks. Doctor: Hi,Thanks for writing in.The MRI brain scan shows a pineal cyst of size 4.4 x 10.5 mm and is small to cause any symptoms. I assure you that a pineal cyst of that size is unlikely to be causing your symptoms. You can have a detailed discussion on that with your neurologist during appointment.In medical literature, there have been less than 50 patients with symptomatic pineal cysts till date. Most of these patients had history of headaches and symptoms concerning vision. Symptoms occur usually when the cyst causes obstruction to the flow of brain fluid causing hydrocephalus. Unless your MRI scan shows features of hydrocephalus due to CSF obstruction, it is difficult to say that your pineal cyst is symptomatic.Causes of your dizziness can include reduced blood supply to the brain, vertigo and anxiety. Please do not worry for the pineal cyst."
},
{
"id": 103450,
"tgt": "Red painful itchy rash on back, arms, legs, fungal rash, reduce by using anti fungal creams, anti inflammatory diet, receiving herbal teas, supplements to cleanse and clear system of poisons,",
"src": "Patient: mI have a red painful itchy rash. Sudden onset seven months ago on sun exposed areas of upper back, front of lower arms and both legs. I have been told by GP it was a fungal rash. I was treated with anti fungal creams for a month with no relief. My back has become more healed and few small left over rash spots are apparent, though seem to be healing . My arms have improved and currently have more of a slight spotty rash, healing and reappearing, as rash continues to develop on my legs. The closeness of the rash appears from a distance as a bad burn . It does not fester, weep or bleed. It is quite painful and itchy. I have been seeing a DOM, receiving herbal teas, and supplements to cleanse and clear my system of poisons. I am on an anti inflammatory diet. I have been a vegetarian , avoiding much processed foods, and have been for over 15 years. I have been using Mediterranean based diet for three years. I avoid perfumes, and processed soaps, do not use any pesticides, or cleansers, and have been low allergen diet due to problems with allergies all my life. I do not have sinus problems. In November I removed any vinegars from my diet. Doctor: hi, rashes on the sunexposed areas suggest polymorphic light eruptions or pellagra. usually fungal infection is seen in warm, moist ad covered areas. hence see dermatologist. take multivitamins also. all the best."
},
{
"id": 50042,
"tgt": "Have pain in kidney due to stone, have ureteral stent placed. Should I go to ER?",
"src": "Patient: Hello. About 3 weeks ago I had a ureteral stent place on my left side. I had a 7mm stone (they say they no longer see) and discovered I have a stricture. Im pending authorization for laser procedure for stricture. I am having horrific waves of pain jn my left kidney that is not relieved by pain medication. Should I go to the ER? Doctor: HiThanks for the queryYou did not mention the location of the stone.The cause of pain could be movement ydratedof the stone in the ureter, obstruction or infection.So you need to see the physician who can examine you and do further tests like an ultrasound and urine exam to determine the cause.Meanwhile keep yourself well hydrated.Hope this helpsGood luck."
},
{
"id": 6078,
"tgt": "Premenpausal, trying to concieve, on Clomid, having hot flashes, food cravings, bloating, constipation, low back pain, pregnancy tests negative. Could I still be pregnant?",
"src": "Patient: I m 45 years old, trying to conceive on Clomid . My periods are very irregular as I m premenopausal. I ovulated on 7/26/12 and had relations for 2 days following. A few days after I started feeling extreme exhaustion, had some hot flashes, and food cravings . Now I have abdominal bloating, some constipation and low back pain . I have done over 5 home pregnancy tests but all have been negative. I have not gone to my doctor because I was ridiculed when I mentioned desire for a baby at my age. My OBGYN was more open, put me and hubby on Clomid (he had low sperm count after vasectomy reversal and how his levels have more than doubled). Could I still be pregnant? How can I know? Doctor: Hello I think you are having perimenopausal symptoms Still you can get your beta HCG levels done in the blood Take care"
},
{
"id": 154659,
"tgt": "Does the beginning of lung parenchyma mean cancer?",
"src": "Patient: hi there, my mom was a cancer patient in the year 2004 and she had cervix cancer. Some thing was detected in her lungs and after doing the test i got this report in my inbox. IMPRESSION:CT guided biopsy of lung lesion : -Serial sections reveal benign lung parenchyma.There is no evidence of metastasis.Im really confused and not able to understand if its a cancer or no. Please help me with the same. regardswasim Doctor: Hi,Thanks for writing in.CT scan guided biopsy is a confirmatory investigation for any suspected cancer areas. In your mother the test has been done to confirm any abnormal lesion in lung.The report says that there is no cancer process in the cells studied under the microscope. These cells are harmless and benign.Presence of benign cells on biopsy is a good news and there is no need to get worried about the cancer returning. Please do not worry."
},
{
"id": 48930,
"tgt": "What causes back pain,low urine output and jaundice?",
"src": "Patient: my husband has had heart problems (arrythmia) for 10 years with medication- amioderone, beta- blockers etc. gets back pain periodically- being sick, no urgency to pass urine for past 2 days & appears jaundiced. Is this likely to be a kidney problem? Doctor: HI THANKS FOR POSTING YOUR QUERY.YOUR SYMPTOMS ARE MOST LIKELY SUGGESTIVE OF A STONE/CALCULUS.A STONE IN THE URINARY TRACT IS LIKELY TO PRESENT WITH LOW BACKACHE,DIFFICULTY IN PASSING URINE DUE TO OBSTRUCTION.THIS STATE CAN BE TERMED AS \"ACUTE URINARY RETENTION\"JAUNDICE CAN BE DUE TO COEXISTENT GALL STONES IN THE BILIARY TRACT CAUSED \"OBSTRUCTIVE JAUNDICE\".AN HIGH RESOLUTION CONTRAST ABDOMINAL COMPACT TOMOGRAGHY(CT) IS USEFUL TO EVALUATE BOTH THE DISORDERS SIMULTANEOUSLY.RENAL STONE SIZE CAN BE ASSESSED AND TREATMENT CAN BE PLANNED ACCORDINGLY.RENAL STONES CAN BE REMOVED BY PCNL,ESWL.ALSO HAVE REGULAR FOLLOW UP FOR YOUR HEART DISEASE.CONCLUSIVE OPINION:RENAL AND GALL STONES CAN BE THE CAUSE.CONSULT EXPERT UROLOGIST FOR FURTHER MANAGEMENT.THANK YOU."
},
{
"id": 170235,
"tgt": "Suggest remedy for persistent fever in an infant",
"src": "Patient: My son is 10 month old He has fever from last night We gave him pacimol till today evening After visiting our doc he advised to give him mental p if fever don t came down with pacimol So last night @10 pm we gave him meftal p Fever came down But againt at 3 am fever clocked to 103.5 Againt we gave him mapthol around 3.30 am But still the fever is same 103.3 @ 5.10 am Please advise Doctor: HiDont worry.its quite common problem.only drugs wont help to releive from persistant fever .just try to go for tepid sponging along with drugs.the drugs your taking is sufficient.follow for 3 days.if it not releiving consult paediatrician to evaluate.Thank uWith best regardsVasundhara"
},
{
"id": 220277,
"tgt": "What does a positive pregnancy test suggest?",
"src": "Patient: Dear Dr. Jain, I would like to know if i might be pregnant. I stopped using the depo provera in June 2009 and have nt received my normal periods only for 1 day in May 2010 i have seen some brown discharge on my panty and nothing afterwards. 2 weeks a go my stomach has grown and it looks like a pregnant stomach, i feel movement, bloated and sometimes pain in my lower abdominal. This past monday (28/06/2010) i have done a home pregnancy and the results was positive, could it be that i m pregnant? I don t have any symptoms of being pregnant exampl: morning sickness, sore breast, increase of appetite ect.) What could it be. Thank you so much for taking time reading my query. Regards Doctor: Hallow Dear,1. Your periods were very scanty, just few spots of brown blood.2. You have morning sickness.3. Your breasts ar sour.4. Above all your pregnancy test is positive.All these findings are supporting the diagnosis of pregnancy. Still if you are in doubt, you have two options for confirming the disgnosis:1. Beta hCG test which is very sensitive and specific, hence the chances of false positive results are almost nil.2. Ultrasonography will provide you direct visual evidence of pregnancy status. I hope this will resolve your dilemma.Dr. Nishikant"
},
{
"id": 11336,
"tgt": "Shall I take Biotin capsules for hair loss?",
"src": "Patient: Hi,I am Aditya 22 years old working professional and I am suffering from hair loss kindly suggest me some steps or any medicine to stop it and to regain the same. I have heard about biotin capsules if its good for hairs so please let me know in bangalore where its available. Doctor: Thank you for the question.Hair loss is associated to many factors .But hair loss due to heredity is usually seenKindly get yourself first checked for routine blood investigations such as hemoglobin etc. If all the tests are normal, then you could start applying Minoxidil solution 2% or 5% . this should be supplemented with multivitamins .Biotin is usually found in multivitamins. Regular exercise/yoga ,stree free life style and balanced diet will all be helpful in reducing hair loss. You will begin to notice hair growth after a period of 6-8 weeks after following all the steps which is advisedBest Regards,Dr Nithin"
},
{
"id": 222128,
"tgt": "Can you tell me why should I take Loprin?",
"src": "Patient: I was in 9month of pregnancy in 2009, I went for scan doctor diagnosed the baby is dead in the womb so they have induced pain and i delivered normally the baby. again in the month april 2010 I got concieved when I was in 8week complete absorption. Later in the month of september i had thyroid now i m taking tablet. Last month i went for checkup , doctor advised me to take Loprin 75 for two month and asked us not to plan for two months? Can you please tell me why I need to take loprin, why doctor advised us not to plan for pregnancy? Doctor: HelloReally very sorry about the loss of baby in 9th month and again abortion.Tab Loprin 75 is given in a patient who is having repeated pregnancy loss to take care of any disorder of blood which are responsible for pregnancy loss.follow your Dr's advice and trust me dear you will have successful pregnancy.All the best and don't lose hope.Dr.Mira Butani"
},
{
"id": 96619,
"tgt": "Can a hematoma on the forehead post an accident be treated?",
"src": "Patient: I fell in the subway and hit the train and received a very severe blow to my head and also a very deep cut with deep stitches that will dissolve and 11 topical stitches two days ago. CT was clear and I was released same day from ER. After two days the large hematoma on my forehead has turned into dark red covering my eyes and sides of nose. My question is about flying. I am on vacation in NYC and I am scheduled to fly home to TX in 4 days. I don t know if it would be a problem to fly with this blood pooling around my eyes and nose. And also concerned about how flying would affect the stitches. Please advise whether I should fly or not. Thank you. Doctor: Hi,Sorry to hear you had an accident. Thank God the accident never caused any brain damages as revealed by the CT. As for the hematoma, it will resorb progressively and may not pose any problem. Flying back home in four days will not as well pose any problem as people with more severe brain injuries can always be transported via flight when need be.All you need now is drugs to control your pain and rest well.Hope this will help. Let me know if I can assist you further.Wish you speedy recovery.Regards,Dr. Mbuomboh Veranso"
},
{
"id": 7028,
"tgt": "Is it safe to take letrozole while conceiving ?",
"src": "Patient: is letrozole 25 mg good for me .i want to get preg fast? 29 yrs,i have pcod ,irregular periods overweight taken three month course of dolute-L finsihed on march[jan-2011 to march2011] after that i have taken duphaston for 15 days. next month sysron-n for periods.all my tests were normal i.e insulin, but little hormon problem i.e fsh-4.56. LH-5.11 .now my doctor gave me letrozole2.5mg[intraget]tablet for five days from my day2 of period.i have this tablet. 1--Is this tablet safe for me because i have heard some side effect like birth defect ,triplet etc.? 2--now is my third day(9-8-2011) 3--plssssssssssss suggest is this a right way .any other suggestion Doctor: Hello Thanks for query Letrozole 2.5 mg per day for 5 days is the standard dose for the ovulation induction. No medicine is fre from side effects ,but the drug has been approved by the FDA & Govt.of India for use in India. We are using it our more than 200 patients without any significant side effect. Hope I have answered your question well to your satisfaction. Wish you speedy complete recovery at the earliest."
},
{
"id": 139329,
"tgt": "Suggest treatment for severe foot pain",
"src": "Patient: I have intermittent pain in the ball of my foot. The pain feels like hot needles piercing my foot. I am not overweight, don t wear high or tight shoes. This pain is intermittent, occurring for 5 seconds two or so times a week. On the pain scale it is a 10 (out of 10). Thanks Doctor: Hi, I value your concern regarding the symptoms. I have gone through your symptoms, and in my opinion this pain is not to be taken lightly, I would recommend to get yourself examined, get X ray and blood investigations done after consultation with your doctor and then take proper treatment, without examination of investigation we cannot advise you. Hope this answers your question. If you have additional questions or follow up questions then please do not hesitate in writing to us. I will be happy to answer your questions. Wishing you good health.Special note- Any medication prescribed needs to be taken after consultation with your personal doctor only."
},
{
"id": 188365,
"tgt": "White spot on the gums, a common mole sized. Smoking. Anything to worry?",
"src": "Patient: Hi, I have a white spot that will not go away on my bottom gum directly below my two front teeth.... it is the size of a small common mole...perfect in its circular pattern...it has been there for over one month....should I be concerned...and yes I am a light smoker.otherwise in good condition for my age....with above average oral hygeine.thank you! Doctor: Hello.Thanks for sharing your concern with us.I have gone through the description you given,it seems that the white spot is normal.It is not related with any pain or discomfort so I would advice you just check it regularly for increase in size or related symptom.And now Don't worry at all. Just be careful.Keep good oral hygiene and do routine dental check up.Take care.Regards."
},
{
"id": 187439,
"tgt": "What is the cure for peridontal disease?",
"src": "Patient: Hi...I have a dentist appointment in a week. It has been 6 months and they are insisting that besides a cleaning, I MUST have x-rays plus examination. It is very costly. I do have periodontal disease. My dentist seems to not want to clean if I do not do everything. What do you suggest, please? Doctor: Hello, Welcome Thanks for consulting HCM, I have gone through your query, you have mentioned that you have Periodontal problem , dont worry see Xray is neccesarry for treatment as doctor see bone loss and Periodontal ligaments and if any Periapical pathology so you you go to dentist and say your problem and you should go for Atleast Cleaning I.e Scaling and root planning Do warm saline gargle two - three times a day you Hope this will help you. Wish you good health"
},
{
"id": 8412,
"tgt": "Suggest a remedy for small white facial pimples",
"src": "Patient: I have small white pimples(not exactly pimples but i don't know what they are called) all over my face since last 5 years.I can't remove them manually but after deep pressing the beauty parlour people are able to remove one or two.Recently I visited a doctor in BTM and he suggested that I should undergo a treatment in which he will apply a layer all over my face for half an hour and then he will remove them.It will cost around 2000 bucks.He confirm that they will get removed but new one can come out.When I asked reason he said it depends on skin type.Please suggest me what i can do .Thanks in advance. Doctor: Hi,You seem to have condition called milia. There may be whitish,small,round elevated lesions. The lesions are cystic containig whitish material. The condion is benign and harmless except cosmetic concern.You cosult dermatologist for firm diagnosis.The milia may be treated in OPD by removing the whitish material puncturing the cysts manually. Or the lesions may be removed by electrocautery done under local anaesthesia.I hope you got my answer.Dr. Ilyas Patel MD"
},
{
"id": 111292,
"tgt": "What could back pain (on right side), dizziness and hiccups indicate?",
"src": "Patient: i have lower back pain just on my right side, it hurts to sit to stand to lay to bend down to bend to the side no matter what i do it hurts, the longer i sit the more it hurts, i also get light headed when i stand up its kinda strange but i have had to hic cups five times today and three times yesterday which is unusual Doctor: Hi,The fact that it is accompanied by hiccups could be a mere coincidence, or at times related. Hiccups generally arise from the irritation of the muscle that separates the thorax from the abdomen, called the diaphragm. A chest x ray and proper physical examination would be useful. Kind regards"
},
{
"id": 85202,
"tgt": "Will diflucan affect the quality of ova?",
"src": "Patient: I just took a single dose of diflucan, also had another dose last month that didn t clear up a yeast infection. Can this medication affect eggs. We are going to try and have a baby, and were going to start next month. Can this medication harm an egg? Doctor: Hello, Diflucan does not affect the quality of the ova. Nothing much to worry and your treatment is in the right track. Hope I have answered your query. Let me know if I can assist you further. Take care Regards, Dr Shinas Hussain, General & Family Physician"
},
{
"id": 219198,
"tgt": "Late periods, cramps, backaches and fatigue. Pregnant?",
"src": "Patient: I have a missed period, I am 4 days late. I have cramps, backaches, I'm tired more often, and nauseas, can I be pregnant? Last month I had my \"period on February 21 or was it implantation bleeding? it was light and a week before that I was bloated, nauseas, headaches,cramps and had tender breast. At the moment I don't feel myself and I think I may be pregnant. Please help Doctor: hello,Period date may alter up to 7 days and it is considered as normal thing. It occurs mostly due to hormonal imbalance & mental stress.Here, you need to undergo one blood test for beta-hcg & TVS scan to rule out conception at the earliest.Following test report, you can consult with your gynecologist for next line management. In exclusion of pregnancy, other relevant investigation may be required.All the best."
},
{
"id": 191472,
"tgt": "What causes elevated blood sugar levels post tooth extraction?",
"src": "Patient: Hello Dr. Rynne, I have Type II Diabetes and my PCP is with Kaiser Permanente who has a Diabetes class which has a diet that they want you to go by for your food consumption which is 1800 calories a day and 45 - 60 carbohydrates a meal. They also tell your that your dentist in your best friend. So after trying the diet and gaining 10 pounds which I quickly stopped and lost the 10 pounds plus 5 more. So after years of putting off going to the dentist I went back because of a toothache and was told I had advanced periodontitis, after two surgeries on my gums and two extractions my daily glucose reading have gone up. My fasting (a.m.) numbers were always high 120 to 150 and my p.m. numbers were always 105 to 120 now after improving my dental hygiene my numbers went up a.m. 145 to 170 and p.m. 130 to 150. What would cause my numbers to go up? I was and still am walking 2 miles every morning, working out on my Total Gym and stretching 6 days a week. Doctor: Thank you for your questionfirst of all I would request you not to worry for the rise in blood sugars, I agree that your blood sugar values have gone up after the dental procedures. the reason is, any kind of stress to your body will increase the sugar values, dental procedure is itself a stressor, the anxiety and pain involved in the procedure will cause a rise in blood sugars..these values will get back to normal once the infection and pain subsides. pls do write back to us, if u want any suggestions on ur medications"
},
{
"id": 156183,
"tgt": "Can a lump on the neck be a sign of leukemia?",
"src": "Patient: Hi,My son has a lump on the right side of his neck.My husband mother side there have been people with leukemia, his cousins son had it and died when he was 9 and now his aunty his mum sister... what do i do and what do i do i am so worried as my son is 4... There are no doctors open as it is 2:30 am and i am so scared..please help Doctor: Welcome to Healthcare Magic.. All lumps are not necessarily due to cancer.. However given the family history of multiple cancers it is better to consult your GP.. likely it is a benign enlargement of the nodes of the neck.. Just have a physical exam including ENT exam followed by FNAC or Biopsy if indicated.. Hope this helps answer your question"
},
{
"id": 138834,
"tgt": "What causes swelling of legs and arms with bruise?",
"src": "Patient: My boyfriend tripped and hit a edge of a wall, inner bruising on arm and leg, both swelling, Knee feels like it popping, constant burning sensation above inner calf muscle unable to straighten or bend knee. What should I do to help ease the pain? Does he need to go to the hospital? Doctor: Hi,Thanks for your query.From description it seems to be soft tissue injury . swelling might be due to haematoma formation(collection of blood).You need an x-ray to rule out any bony injury. I suggest you to consult an orthopaedician nearby for a detailed examination. You may get the X-ray of the area affected under his/her guidance. Beside icing and elevating it, you can take anti-inflammatory drug like motrin to reduce pain and inflammation.Hope this helps. Let me know if you have any more concern.Warm Regards."
},
{
"id": 101362,
"tgt": "Suggest treatment for breathing problem due to asthma",
"src": "Patient: SIR i am affect with asthama , i consult with my doctore, he told me 6 month take motack lc and inhalder.now 12 month compleled. now i suffering very breathing problem. now i am working out of my home city. now i take the same tablet and inhalder last 1 week . its any broblem will come Doctor: Hello.Thank you for asking at HCM.It is very important to understand asthma. Montack LC and inhalers control the symptoms but they do not \"cure\" asthma. Again, there are two types of inhalers - controller and rescue. I would highly recommend you to know the type of your inhaler. Controller inhalers are to be used daily on regular basis. While rescue inhalers are to be used when you have symptoms of breathlessness, wheezing despite using controller medicines. (Montack is also a controller medicine).SO the inhaler that you are using is not a rescue type, I would suggest you to purchase salbutamol inhaler which you should keep with you while away from your city. You can use it when you have symptoms.Regarding duration of asthma treatment, it is determined by activity of asthma. Allergy & asthma are dynamic, they tend to improve or worsen with time. So decision regarding decreasing your drugs and inhalers can only be taken by your physician after asking you about your symptoms. So please do not stop/decrease drugs by yourself.I would also suggest you regular breathing exercises and avoidance of smoke, dust & air pollution. This will help you in a long run.Hope this will be helpful to you.Wish you the best of the health.Regards."
},
{
"id": 212351,
"tgt": "Worsening anxiety and depression, feel cold and clammy followed by hot flushes, dizziness, jelly feeling in legs. More serious?",
"src": "Patient: I have recently been to the doctor, and diagnosed with severe depression and anxiety. This was around a month ago now. I have had up and down days, but more recently, as in the last couple of days, I have been getting worse, and feeling worse. Most of the time, I am cold and clammy. Then, I will get hot flushes and a dizzy, light-headed feeling, and like my legs are made of jelly. I'm also sweating a lot, and very fatigued; I can not concentrate on even the most basic tasks anymore.My question is this; is this just my depression and anxiety getting slightly worse, or should I go and see my doctor in case it is anything more serious? And if it is anything more serious, can you suggest what it may be? I really don't want to bother my doctor if I don't need to, as I do spend a lot of time in her office! Doctor: Hi there ~ You seem to have an exacerbation of your anxiety symptoms. However, there may be other reasons you are going through these symptoms. One of the other reasons that you might be having these symptoms is because you might be going through perimenopausal symptoms. You might want to increase the dose of your medication but make sure you are following the instructions of your doctor. Using yoga, pranayama and regular exercise may also be helpful to relieve your anxiety. Contacting your social services department may also be of help. I hope this helped with your question. Take care and have a lovely day !"
},
{
"id": 56659,
"tgt": "Suggest treatment for liver enlargement",
"src": "Patient: Hello Doc, my SGPT level is 95 and I have fatty liver.. the diagnosis my doctor had of my condition after I underwent an ultrasound and other laboratory tests, hence I am undergoing treatment for that now. My concern is about my family history which I tried to relate with my current medical condition. My father passed away 4 years ago due to pancreatic cancer, does my condition has something to do with it too? Doctor: Can't say confirmly since there r n no of causes of pancreatic cancer and liver too .mainstay for u is too avoid causes leading to fatty liver, and to continue treatment and follow up with 6 monthly usg"
},
{
"id": 164434,
"tgt": "Suggest treatment for vomiting in an infant",
"src": "Patient: Hi, I have a 5 month old son who has reflux and has always been a vomiter and currently receives 1mg/kg Omeprazole BD . Because of this we started solids at 4 months in the hope that the thicker substances would lessen the vomiting. However he is still vomiting excessively and I d like to know if there is anything else I can do to decrease his vomiting? Doctor: you are doing right,, giving thicker feeds will help in reducing vomiting and using Omeprazole is also help in reducing gastric acidity,, mostly your kid might be suffering from GERD ,gastro esophageal reflux disorder.. kindly get evaluate for GERD, do not make the baby lie down immediately after sleep and do not feed the baby 1hr before going to sleep, better try lansoprazole, which is more effective in GERD."
},
{
"id": 199633,
"tgt": "Would practicing masturbation be unhealthy?",
"src": "Patient: Dear Doctor, Hi, I am 27 Male. I have problem with masturbating. I am still a virgin. Never tried sex with any girl. But, I have started masturbating from my late 14 and now almost 13 years and I am not able to come out of it. When I am young around 14 I used to do 3 to 4 times a week. Now past 8 to 9 years I masturbate weekly once. Now, I am trying to control it because which I found myself it\u2019s not very healthy practice (By putting me in a lot of social activities, studies and sports, sometimes exercises). But, recently I found some kind of problem. When I am talk to my girlfriend or when I touch her some kind of liquid is coming out of my penis (this happens over the last 2months). Also, I have problem with control of my ejaculation (means when masturbate ejaculation happens out of control within 50secs). I think I lost control also. I feel it serious problem because of masturbation over the years. Due to this, I had lot problem over the time. Headache, back pain, losing memory and I used to get more frustrations and I play sport (Badminton) during this I observed. if I stress more I am getting temperature(fever) while playing and it stays for the whole day and after the proper rest it goes. This kind of problem I am facing recently over last 1 month. I never faced these kinds of problem before when I masturbate. Now I have lot confusion. Whether this masturbation will lead to Erctile dysfunction or any other problems I read lot of forums and blogs say masturbation is not a problem and I went through this website also http://yourbrainonporn.com/erectile-dysfunction-and-porn Now, I think I need help from doctor which helps me. I don\u2019t know whom to contact or which doctor I have to consult to solve my problem. Thank you. Doctor: HelloThanks for query.Sexual satisfaction is a normal desire of a man kind at the age of adolescence which in absence of female partner he gets by doing masturbation .However as a law of nature anything that is in excess leads to addiction and causes desensitization of penis resulting into problems like loss of libido ,poor erection etc.All other problems that you have are mind related and due to anxiety Following general measures will help you to boost up your confidence and improve your general health .1) Practice regular exercise for 45 minutes followed by meditation for 1/2 an hour in the morning.2) Take high protein diet rich in vegetables and fruits and Vitamin A,C,D,E.and Zinc3)Take anti oxidants like Almonds 5-6 everyday..4) Avoid alcohol and smoking..Consult qualified Psychiatrist for counselling and treatment .Dr.Patil."
},
{
"id": 135281,
"tgt": "Suggest treatment for open cut and bruise on ankle",
"src": "Patient: I closed a door on the back of my ankle and it cut it open but there is also a pretty big bruise above the cut, I cannot bear weight on it or walk on it, also can t move it downwards or push it against a hand. I want to know what this could be and if I should see a doctor. Doctor: Hi Dear,Welcome to HCM.Understanding your concern. As per your query you have open cut and bruise on ankle. Well there can be many reasons for symptoms you mention in query like hematoma , soft tissue muscle injury or bone injury. I would suggest you to clean wound with betadine , take proper rest , and consult orthopedic surgeon for proper examination . Doctor will examine carefully , order CT scan or x-ray to confirm the diagnosis . Doctor may stitch the wound after cleaning it with hydrogen peroxide and prescribe antibiotics along with anti inflammatory .Hope your concern has been resolved.Get Well Soon.Best Wishes,Dr. Harry Maheshwari"
},
{
"id": 180977,
"tgt": "What causes headache, fever and swollen cheek post a tooth injury?",
"src": "Patient: 2 weeks ago three of met top back teeth hurt. X-rays were clear but took 7 days of antibiotics. After exam by dentist my cheek was swelled for 4 days. This past wed. I had severe headache neck pain and fever. Now sore throat. I'm on my 4th day of this. What could be happening? Doctor: Hi..Thanks for the query..If the x ray of teeth was clear and you still have pain in upper teeth along with cheek swelling, headache, neck pain and fever then it can be probably related to infection in your sinuses or Sinusitis..So my suggestion to you is to consult an Emergency room and get evaluated.A PNS View x ray of sinuses can be done to confirm the diagnosis..In case of sinus infection you can be advised another course of antibiotics for atleast a week along with anti inflammatory painkillers like Advil, decongestant nasal sprays and steam inhalation..For now you can take Advil to reduce pain and swelling..Hope this helps..Regards."
},
{
"id": 11316,
"tgt": "What causes hair fall?",
"src": "Patient: i Have hair fall problem...hi i am siva from France.. i am 25 yrs old. for the last one year i have hair fall problem. i lost my hair and i look like little old.. i dont know why it happens.. i consult the doctor. he prescribe \u00e0 m\u00e9dical shampoo but it doesnt stop. can you give me some advice... Doctor: Hello,Thank you for posting on HCM.I appreciate your concern regarding hair fall. Hair fall is usually ascribed to multiple factors like diet and nutrition, hormones, stress, acute or chronic medical conditions,drugs,cosmetic products etc.From your description, it seems you might be having Telogen Effluvium, which is hairfall secondary to stress.If i were your dermatologist/trichologist, i would like to take through history including family history, history of recent major trauma/illness/stress/medications etc and recommend some basic investigations like CBC,blood sugar,Thyroid function test.Some other special investigations like trichogram and dermoscopy and further aid in diagnosis and prognosis.I would suggest you a course of oral tablets containing biotin and other essential vitamins and minerals for minimum 3 months. Also, would put you on solution containing 5%/10% minoxidil once a day and a hair serum containing peptides for hair growth at night.(Q sera etc). I would advise use of gentle shampoo and conditioner on regular basis and use of coconut oil twice a week.Also enquire from your dermatologist about upcoming treatment options like mesotherapy and platelet-rich plasma.Avoid combing in wet hair and let them dry by wrapping in towel. Avoid blow dryers and hair-color/dyes. Take plenty of fresh fruits and vegetables in your diet and try to de-stress your routine life.Hope your queries are resolved and wish you best of health.Thank youDr Hardik Pitroda"
},
{
"id": 21811,
"tgt": "What causes elevated BP and chest pain?",
"src": "Patient: My dr. is monitoring my bp for about a week, over the weekend it was 174/105, 185/109 and this am it was 137/92. I was hosp. overnight last week due to chest pain. All tests were negative. Is it normal to monitor bp this long? I am 49, 5'2\" and 220 lbs. Doctor: Hi,You need to take medication, as it is dangerous to leave such high blood pressure without treatment. For now don't worry , as the tests were normal, but talk to your doctor to prescribe a medication.Take careCome back if you have any further questions"
},
{
"id": 40827,
"tgt": "Suggest remedies to regularize menstrual cycles that are erratic despite treatment",
"src": "Patient: I've been going through a problem with my husband. I'm a 21 yo Female weighing 220lbs 5'6\" and my menstral cycle has been erratically irregular for almost 3years now.. since I've been with my husband. I've gone to get checked in May of 2016 but they couldn't find anything wrong. They said I was borderline diabetic borderline have fatty liver and not as consistent with POCS. Although they did give me pills to try and regulate my menstral they were called Progesterone but there was only a minor change and they wouldn't prescribe it to me anymore. My husband told me he will leave me because of this matter and I'm desperate to resolve this and get my menstral back on track. He says it's subconsciously decided that I don't want to have kids. That I purposefully made the choice and my body only followed what my mind thought. Please help me.. Doctor: Hi, I think you should go for evaluation first. Do a thyroid profile and prolactin levels. Sometimes these can also cause irregular periods. So, you may need some treatment for that. Also, if your husband is ready for a baby, medicines are available which can help you in getting pregnant. Talk to him regarding this. Hope I have answered your question. Regards Dr khushboo"
},
{
"id": 5541,
"tgt": "Had taken naturogest to conceive. Discontinued cause developed body pain, got periods. Neutralizing pill?",
"src": "Patient: i am 31 years old. i am plannig for pregnancy i used naturogest 300 mg for nearly 20 days as per my physician suggestion but did not get pragnancy. i am having severe body pains and i got period but its not normal is this due to use of naturogest..now i stoped using that pill and wanted to know whether i have to use any medecine to nutalize that naturagest pill effect . kindly help me out Doctor: Hello, Thanks for your query. Its progesterone side-effects, its very benign, just dont worry your periods will be alrite without any further medications. I hope this information has been both informative and helpful for you. In case of any doubt, I will be available for follow ups. Wish you good health. Regards, Dr Arif"
},
{
"id": 63633,
"tgt": "What does a formation on collarbone imply?",
"src": "Patient: I had an episode of rapid heartbeat two weeks ago, my heart dr. did several tests and sent me home with a beta blocker to take every day, and also i am wearing a heart monitor...since i have been wearing the heart monitor, (one week 1/2) i noticed a small area on my right collarbone that looked a little puffy so i felt it and it feels like a small marble moving around...my dr. looked at it and put me on antibiotics...i had a chest xray and chest c/t scan at the hospital march 2nd...my dr. was going to do an xray, but after i told him i had one march 2nd he said ok then he doesnt need to do another one...from what i am reading it sounds like lumps/marbles could be cancerous and i am extremely scared now...what do you think it most likely is? i was sick in feb. with congestion/cold symptoms??? Doctor: Hello,In my opinion, you have a collar bone boil with the infection for which your doctor started antibiotics. CT and x-ray chest bothered you much and you started worrying about the possibility of cancerous lumps in your collarbone area. By no means, this lump or collar formation is cancerous and is due to the infected boil on the collar bone area.So just forget it as antibiotics would take care of it. Cold and congestion are part of the seasonal illness and should not worry you much. So keep cool. Consult your emergency room, surgeon and physician, if need be, who would treat it accordingly. So don't build up wrong concepts and create more psychic complications in you which would increase risks and costs to you.Hope I have answered your query. Let me know if I can assist you further.Regards, Dr. Mukund Savaskar"
},
{
"id": 19404,
"tgt": "What does slow heart beat indicate?",
"src": "Patient: i got a slow beating heart.. 42-44 while resting.. 21 year old male.. 1,79 cm, 72 kg.. do sports but not extreme like running every morning.. should i be concerned? i got a very relaxed character though.. i hardly get excited or scared.. my ECG was fine.. nothing abnormal but i am just curious whether this is normal or not.. i just cant stop listening to my heart beats by the way.. Doctor: dear mam. I wld suggest u to do an echo and this s very unusual for any person. v need to look in to symptoms like dizziness and syncope. but meet a cardiologist. he wld examine and suggest u pacing. thank u"
},
{
"id": 24769,
"tgt": "Is it safe to have another surgery for stents within months?",
"src": "Patient: Twelve. Weeks ago I had a heart attack , I needed to have stent surgery and they implanted 2 stents, afterwards I still have angina not all the time but a couple of times a week, after going back to see the doctor who did the surgery he thinks it would be best to do 2 more stents in another artery which has 3 narrowings, this is planned for January how safe is it to do this so soon after the last op a? Doctor: Hello and thank you for using HCM.I carefully read your question and I understand your concern.I will try to explain you something and give you my opinion.You should know that if it is a narrowing in your coronary artery than the intervention shold be as soon at is it possible.If it os not an emergency, than you can take time to plan it.But you should not worry for the time you did the first intervention.This does not interfere with the next one.The firs time you had an infarction and the stenting was an urgent situation.The next procedure, is a planned one to reduse chest pains and prevent another heary attach from tje other blocked arteryes.As son as you do them the better is for you organism, so dont worry.Hope I was helpfull.Best regards,Dr.Ervina."
},
{
"id": 223957,
"tgt": "Can I take primolut-n and i pill for birth control?",
"src": "Patient: hi, my wedding is on 10th may and I m planning to take primolut-n till my first night and then I plan to take i-pill after the first night and discontinue primolut-n.My menstrual cycle is 28days so aaccordingly I m expecting my periods to start on 9th may. please advice whether there are any chances of pregnancy Doctor: HiDr. Purushottam welcomes you to HCM virtual clinic!Thanks for consulting at my virtual clinic. I have carefully gone through your query, and I think I have understood your concern. I will try to address your medical concerns and would suggest you the best of the available treatment options.1)First of all do not panic.2) If you are already on primolut N and expecting periods on 9th May, you are already in safe period, i. e.beyond day 20 of the cycle. And you need not take i pill after the first night.3) There are no chances of pregnancy at all even if you do not use i pill.Secondly, you will resume your periods within 3 to 10 days, after stopping Primolute N. I hope my answer helps you.Thanks.Wish you great health.Dr Purushottam S Neurgaonkar"
},
{
"id": 29622,
"tgt": "Suggest remedy for severe abdominal cramps with vomiting",
"src": "Patient: I threw up 8x today after eating pollo tropical at 12 midnight , But this happens so often I don't think its a stomach virus. My stomach feel like someone is clamping down on it with pliers or a wrench . I'm sweating ,scared to eat , and certainly afraid. I want to say after I eat , I immediately go to sleep but somehow I wake up starving like I didn't eat nothing late night . Can you please help me? Doctor: Hello,I really appreciate your concern, it is definitely a food allergy and you will have to give some trial like stop eating the food that causes the vomiting if you do not have the vomiting after this then it could be food allergy but is halfway done and you have to start eating that food again and if you have a symptoms of vomiting then you can be sure that it is the food allergy, and if this not the food allergy then it could be infection or hyperacidity, in short underlying cause of vomiting is matter of concern.Hope I have answered your query. Let me know if I can assist you further.Regards,Dr. Akhtarhusain"
},
{
"id": 3225,
"tgt": "Will she get pregnant from pre-ejaculate even after taking i pill?",
"src": "Patient: we had oral sex and my pre cum has touch on the tip of the vagina i i didnt ejaculate inside too and the pre cum too as i was using condom but during the oral sex i was rubbing with my penis the vagina and condom remove and my pre cum has touch on the vagina and after that i have given ipill in just a hour of it after having oral sex. is it possible if she gets pregnant. Her last period was on 21/5/15 and we did oral sex 6/6/15 Doctor: Hello dearI understand your concernIf precum goes close tot the vaginal intoitus then there may be chance of the pregnancy.But you have given I pill with in one hour of sex.It is highly effective in preventing the pregnancy.It has 95-97% success rate to prevent the pregnancy.So very less chance of the pregnancy (0-5%).But I pill cause change in the menstrual cycle as it cause hormonal imbalance.It may cause early period, delayed period by 1-2 week, excessive bleeding and intermenstural bleeding.If period will delay by the more than 2 week then go for urine pregnancy test and or blood HCG to confirm the pregnancy.If pregnancy will be there then still you can terminate the intrauterine pregnancy by the abortion pill under the supervision of the gynecologist.Meanwhile avoid stress, take healthy diet, drink plenty o water, maintain proper pelvic hygiene, and do regular exercise.Use condom in future to prevent unwanted pregnancy.Hope this may help youContact HCM for further health queryBest regardsDr. Sagar"
},
{
"id": 142764,
"tgt": "Is Valparin effective in controlling seizure in a child?",
"src": "Patient: sir, my daughter is having seizure for the 5th time. she is now 2yrs & 4 months. seizure occurs while sleeping. she vomits & passes motion before having seizures. all the 5 times she had in the same manner. her MRI & EEG were normal. after 1 month of last seizure again we conducted eeg. it was also normal. does she require valparin? doctors say it will subside after she is 5 years so don t need valparin. please advice. Doctor: HIWell come to HCMI really appreciate your concern, if this is the grandma seizure then patient requires the anti-convulsing drug or anti-epileptic drug that includes the sodium valproate, what ever the cause of the convulsion is there but to control the fit sodium valproate can be given, I would advise you to follow the advise given by doctor. hope this information helps."
},
{
"id": 24775,
"tgt": "What causes episodes of tachycardia in a 17 year old?",
"src": "Patient: my 17y/o son has brief episodes of tachycardia about once a week. these episodes last about 20 seconds and only cause palpitations, no other symptoms. these episodes never occur with exercise. He is a cross country runner and never has palpitations when he runs. He does not consume caffeine or other stimulants. He has never had dizziness, chest pain, SOB OR OTHER SYMPTOMS. His resting heart rate is around 50. RECENT NORMAL LABWORK INCLUDING THYROID, CBC, ELECTROLYTES AND EKG. ALSO, HE HAD NORMAL 24 HOUR HOLTER. SHOULD I BE CONCERNED? Doctor: Hello and thank you for using HCM.I carefully read your question and I understand your concern.I will try to explain you something and give you my opinion.You should not worry about.This thinks are quite normal in jung ages.Fast heart rate might be related to emotion, physical activity and different pathologys like anemia, hyperthyroidism , electrolyte imbalances , cardiovascular disease.As you explain, you already have done all the examinaltions and they result normal.This is a very goog think.So, this fast heart rate might be just sporadic, related to emotions.The only thing that I would suggest is a cardiac echo to complete all the examination, but in my opinion you should not be concern.We meat in our daily practice such complains in jung ages.Hope I was helpfull.Bestregards,Dr.Ervina."
},
{
"id": 191866,
"tgt": "Suggest remedy for urinary hesitancy and constipation in a diabetic",
"src": "Patient: hai sir,my mother 60 years was suffering with diabetic since 15 years giving insulin now a days when insulin was given she was unable to go to bathroom not urine,stool last month blood report was in normal fasting 80 whats going wrong with my mother ?doctor suggest me your answer sir Doctor: Thank-you user for your query.I understand your concern.as per your situation all I can suggest is that ask your mother to start having high fiber diet such as oats, cornflakes, salads, and along with it ask her to drink plenty of water, at least 3 litres a day. This will help in reducing the constipation and regularize the bowel habits. You may also give her dulcolax tablet at night. You may also get an whole abdomen ultrasound done too. But before starting this consult any gastro medicine doctor.He will help you much better. And yes its great that your mother is having controlled fasting sugar.god bless"
},
{
"id": 161532,
"tgt": "Suggest medication for flu",
"src": "Patient: i have a baby who was prem by 6 weeks her due date was 16th december all oue family which is me (mother) her farther and her two year old brother have flu like symptoms she is taking her feeds normal has normal nappies and a normal temp but is very unsettled would you advise me to call out a doctor or see how she goes on during the day? thankyou amy. Doctor: Hello, If she develops symptoms like fast breathing or respiratory distress or inability to feed or high-grade fever then approach her pediatrician, please. Till then she requires only monitoring of her symptoms. Hope I have answered your query. Let me know if I can assist you further. Take care Regards, Dr Sumanth Amperayani, Pediatrician, Pulmonology"
},
{
"id": 138420,
"tgt": "What causes swollen knee and ankle?",
"src": "Patient: very active 55 year old male (in excellent physical condition) who works out regularly with a trainer and has developed a very painful (inside of my right knee) and swollen knee. This occurred two weeks ago and has not subsided. Two days ago calf started to hurt and now right ankle has been swollen for two days. Strange part is no bruising (I typically bruise easily). Doctor: Sir you should check your CRP and ESR, also check Rheumatic Factor and anti-CCP, check levels of uric acid ! Joint effusion can be caused by a lot of rheumatic diseases but to put the diagnose we need to check your blood signs !"
},
{
"id": 157987,
"tgt": "Food not going down stomach, block. CT scan showed cancer, no chance of cure due to disbursed food pipe. What can be done?",
"src": "Patient: Sir, My brother is 62 years of age and suddendly on April 18, the food was not going down in stomach and it started blocked. After that no water, and no fluid was to enter into the food pipe and when the CT Scan was done, it was found that there the food pipe was blocked and there was cancer . After that 5 days RT were done on June 10 to June 15. After one month doctors advised to operate the food pipe. But when they took him for operation and started operated , after one hour the doctors told us that the operation can not be done because it is disbursed and started going upward. Now the patient is on death bed and doctors are doing nothing and nothing is going inside him. Yes they operated a tube on side of the stomach and only a small quantity of milk they had given to patient. As there any doctor can help us by giving some medicine. We are helpless before GOD. Can anybody help me without any payment or fee. Promila Kapoor Doctor: By your description, your brother is suffering from last stage of cancer of the food pipe. Technically whatever has been done so far seems to be scientifically correct. At this point, it is not a matter of money or fees, but there is not much that can be done for him except to provide him whatever comfort is medically possible with pain-killers and the like. Your brother presented at an advanced stage and that is the main reason for his present state. It is nobody's fault. If there is any other advice you need, then please let me know."
},
{
"id": 143887,
"tgt": "Could back pain,neck pain and numbness in back be due to strains or something else?",
"src": "Patient: I was in a car accident 6 months ago, the day of the accident I went to the ER and was given a CT scan. The doctor asked me if I had ever been diagnosed with degenerative disc disease? I said no, and she said nothing more about it. I was told I had Strain of the C- Spine and T- Spine. I followed up with a doctor who sent me to 8 sessions physical therapy. I felt a bit better but as soon as I returned to my regular life and activities, I was back in sever pain again. I have back pain and occasional neck pain. I have spasms and numbness in my back and sometimes my leg. Are these Strains??? It seems that there is something more going on...Please Help! Doctor: symptoms ur experiencing are due to muskuloskeletal injury....these will subside with time....continue physiotherapy and u can take analgesics SOSIf pain is severe and there is paresthesia over limbs or weakness...then u need MRI spine to rule out spinal injury.....regards"
},
{
"id": 134647,
"tgt": "What causes severe stiffness in right index finger?",
"src": "Patient: I am experiencing a very stiff right index finger middle knuckle. It feels like it gets stuck in either straight or bent mode. When I try to either straighten or to bend it feels like it gets stuck and the two parts of the knuckle are rubbing against each other. When it finally moves it is very uncomfortable. Doctor: Trigger finger. Tendon usually glides easily through the tissue that covers it (called a sheath) because of a lubricating membrane surrounding the joint called the synovium. Sometimes a tendon may become inflamed and swollen. When this happens, bending the finger can pull the inflamed tendon through a narrowed tendon sheath, making it snap or pop and it some feels like locking. This can be caused by a repeated movement or forceful use of finger. Rheumatoid arthritis, gout, and diabetes also can cause trigger finger. So can grasping something, such as a power tool, with a firm grip for a long time."
},
{
"id": 2715,
"tgt": "Can I get pregnant after removal of right fallopian tube?",
"src": "Patient: I am 31 years old, fell pregnant in September 2013 sadly it was an ectopic pregnancy and had to be terminated due to large fibroids. Due to the ectopic pregnancy I lost my right tube and my doc performed the myomectomy the same day. Please advise my chances of falling pregnant again as it was our first baby. Doctor: Hi,I guess your concern is about chances of getting pregnant with one fallopian tube and after myomectomy. Firstly let us discuss about the ectopic pregnancy, it is a type of pregnancy in which pregnancy occurs other than the womb, mostly in fallopian tubes. If it ruptures, it requires removal of that site of fallopian tube, and this same thing happened with you. But you have the one healthy fallopian tube so relax. You are having high chances of getting pregnant. You also come under the age group in which there is maximum chances of getting pregnant. So you have double chances of getting pregnant. Secondly I want to tell you that you have already undergone myomectomy for fibroid as told by you on same day. Usually pt having fibroid, facing difficulty in getting pregnant. So there is TRIPLE MORE CHANCES of getting pregnant because you already had myomectomy. You can follow my suggestions:1. RELAX, DON'T WORRY. there is triple chances of getting pregnant. just keep patience.2. If you are in hurry then revisit your treating doctor, he/ she can help you.i hope this information is useful, informative and helpful for you. I will pray to God that you will be getting pregnant as early as possible and will be blessed with \"TWINS\".Regards,Dr Sudha Rani Panagar"
},
{
"id": 112074,
"tgt": "What is the reason for upper back pain while standing up and breathing in ?",
"src": "Patient: Hello. I have been having back pain for about a day now located in my upper back. I can feel it when ever i stand up and breathe in. Someone told me it may be pneumonia because it has been very cold out were I live for the last few days, and I have been outside for quite sometime. I'm pretty sure that is all but if you need anymore information feel free to E-mail me via YYYY@YYYY Doctor: Hello, I have studied your case. Due to compression of this nerve root there is pain in your chest.I will advise you to do MRI thoracic spineFor these symptoms analgesic and neurotropic medication can be started.Till time, avoid lifting weights, Sit with support to back. You can consult physiotherapist for help.Physiotherapy like ultrasound and interferential therapy will give quick relief.I will advise to check your vit B12 and vit D3 level.Hope this answers your query. If you have additional questions or follow up queries then please do not hesitate in writing to us. I will be happy to answer your queries. Wishing you good health.Take care"
},
{
"id": 175592,
"tgt": "Suggest treatment for lung disease",
"src": "Patient: hi i work in home home health pediatric first time. 14 year old with lung disease. with nasal cannula, her O2 sats are 90-98 /1L. but im supposed to use bipap at night and she is often 87- 89%/5L for awhile then 90-96/3L in a few hours is something wrong with the bipap? Doctor: HelloFluctuation in saturation(SpO2) can occur while on bipap. This may be due to the transient displacement of saturation probe.You need not to raise the O2 if it is transient.But if saturation persistently remain below 90 look for nasal blockage or check the BIPAP. Usually there are some indicators which shows whether machine is working well or not.Regards"
},
{
"id": 212033,
"tgt": "Have tingling butterfly feeling, light in mind, chest and arms. What can it be?",
"src": "Patient: Hi, I have this feeling of being light in my mind, chest, and arms --like a butterfly feeling throughout my upper body--. It's been going on all day. I seem to be able to function normal, but this tingling-butterfly sensation is a very strange feeling. I'm wondering if I have anything to worry about, Can somebody please help me? Sincerely, Wendi Turner Doctor: DearWe understand your concernsAs you already are aware, modern medicine cannot define the problem you are narrating. But age old psychology can. We came across so many cases who has such fluttering or tingling butterfly feeling.This is due to some sort of nerve activity along your upper torso which cannot be seen always. Nothing to panic. Just a simple step will cure your problem. Take bath in warm water with mild soap twice daily and sit in front of god for 15 minutes and pray for others' health.Wish you speedy recovery"
},
{
"id": 15253,
"tgt": "Red rashes on legs, thighs, ankles, warm to touch, raised bumps. Due to drug usage?",
"src": "Patient: hi, i have a slightly red rash on my legs from my thighs to my ankles on the back and front of the legs. It is tender to touch and warm/hot feeling when you touch it. It doesn t itch at the moment though but is slightly bumpy/raised. Could this be because of smoking ice over the weekend? It happened last time I took drugs aswell so i thought there may have been a connection. Should I be worried about this? Doctor: Hi i think it is allergic vasclittis.it is caused by drugs.food allergry etc.it presents as bumps all over the lower legs which are hot to touch and non itchy.rule out other associated features like abdominal pain .vomiting .haematemesis to rule out any internal bleeding.it should be cured by vitamin c chewable tablet..a antibiotic twice daily and application of calamine lotion.use short course of steriods like wysolone 30 mg tapered to 10mg can cure it"
},
{
"id": 160428,
"tgt": "How can this abdominal cystic lesion finding be treated?",
"src": "Patient: my daughter is 7 years old , she is having abdomen pain.After done the U.S.G Abdomen Report says thats A 2.1* 1.4 cms in size well dfined anechoic cystic lesion is noted in the right mid abdomen region(iliac) may s/o ? mesenteric cyst and duplication cyst. rest excess of gas is seen in the smal bowel loops Doctor: Hi,The scan says there is cyst (fluid filled mass) in the abdomen, the origin of which is not clear. The size is somewhat significant, and if causing pain, this may need surgical or laproscopic removal. Kindly take him to a pediatric surgeon. You may be asked for a CT scanning to get a more clear picture.Take care. Hope I have answered your question. Let me know if I can assist you further. Regards, Dr. Muhammed Aslam T. K., Pediatrician"
},
{
"id": 141644,
"tgt": "Does a surgery for spinal stenosis cause difficulty in walking?",
"src": "Patient: I had surgery for spinal stenosis (cervical) in 2015. the reason numbness in hands and arms. After 6 wk. at home recovery went back to work on light duty. When walking I begin dragging my left leg. It was intermitting. The surgery was in Jan. In Nov. same year went hunting, Fell 16 and was knocked out. Don t remember why I fell but I had been in that stand hundreds of timeswith no problems. Had a concussion, broke shoulder, and 3 broke ribs. My suspition was my leg stiffened while decending. Time in hospital and since then haven t been able to walk without assistance and very slow. P.T. 3 times with little results. I know its a long story and I just hit on some of the highlights. My question is could the stenosis surgey be a cause of difficuty walkong? Doctor: Hello and Welcome to \u2018Ask A Doctor\u2019 service. I have reviewed your query and here is my advice. One of the complications that can occur in cervical stenosis surgery is cord contusion leading to spasticity of limbs and difficulty in walking. Also, there can be cervical instability leading to cord compression. You need to get a fresh MRI done and consult a spine surgeon. Hope I have answered your query. Let me know if I can assist you further."
},
{
"id": 221080,
"tgt": "What does the following follicular study indicate?",
"src": "Patient: Hi, This is Sasirekha. i am 27-year-old female, planning for kids. Dr. suggested me to have follicular study on day 11, 13, and 15. Report on day 11 as follows, Right follicle dominant measuring 5.0 mm Left follicle no dominant. E:E: 7.0 mm POD: No free fluid. my query is 5.0 mm dominant right follicle on day 11 is a problem or not. Does report indicate any problem, if so please explain. Thanking you. Doctor: HiDr. Purushottam welcomes you to HCM virtual clinic!Thanks for consulting at my virtual clinic. I have carefully gone through your case, and I think I have understood your concern. I will try to address your medical concerns and would suggest you the best of the available treatment options.1]As such comments can not be made from single day's report.2] Growth of follicles in any ovary should show that follicle grows 2 to 3 mm per day and around day 14 or 15 of the cycle i should rupture to release an egg. At the same time, E.T. _ endometrial thickness should go above 7.5 mm.3] This kind of report with little bit variation can be considered as normal.I hope my answer helps you.Thanks.Wish you great health.Dr Purushottam"
},
{
"id": 36375,
"tgt": "Suggest treatment for sebaceous cyst",
"src": "Patient: I have a sebaceous cyst on my back. It got infected and burst about 5 weeks ago. I drained much of it.Saw a dermatologist (while I was on the road for the whole summer) 2-3 weeks ago and he thought it looked like it was healing fine and it would be okay. Just contact him if it swelled or got infected again. Well, it started to get infected again (on my way home). A few days ago I had a first in that it developed some dark spot inside it near the surface that looked as best I could tell like dried blood.My dermatologist (now at home) left her practice without notice, couldn't get in to see another in 2 weeks, but it burst again. Looks like clear oozing has subsided, now just blood. Any recommendations? We leave for our family vacation in two days. Should I swim in the ocean or pool? Doctor: Hello.Until you be seen by another dermatologist, would recommend taking an antibiotic by mouth, for example Cloxacillin or Ciprofloxacin.You should avoid contact cyst with water.Please consult your doctor for a prescription.I wish you good health.(If the answer has helped you, please indicate this)"
},
{
"id": 183871,
"tgt": "What causes swelling in the jaw?",
"src": "Patient: Hi, my right side of my jaw feels swollen, also I can t close my mouth without one of my molars hitting my top molar, feels as if its out of place or has risen which may explain the swelling? Also the tooth in question hurts when pressure is applied. Any help would be greatly appreciated Doctor: Pain on right Side of your jaw is probably because of erupting third molar. An erupting third molar applies pressure to rest of the teeth causing tenderness. To confirm this diagnosis you need to get an IOPAR done or OPG."
},
{
"id": 24254,
"tgt": "What is the ideal pulse rate to assist in shrinkage of an enlarged heart?",
"src": "Patient: What is the ideal pulse/heartrate to assist in the possible shrinkage of an enlarged heart, please? I am 49 years old, under 9 stone, 5ft 6 and a half inches tall. I have had an overactive thryoid and partical thyroidectomy in 2006 and also anaemia. I had a chest infection with SOB in 2009 and a very bad chest infection this year May-June 2011. SOB worse, xray enlarged heart and possible dilated cardiomyopathy & left bundle block. Both ventricles affected and left atrial. No evident coronary disease. Drugs made symptoms worse. Doctor: Hello, The blood pressure in the ER and even now ( diastolic of 101 ) are in the higher range . The symptoms you describe headache and all other can be explained because of blood pressure being on the higher side . So a 24 hour ambulatory blood pressure monitoring ambulatory if the reading are high there ,starting ambulatory do anti hypertensive medication should be the way forward. Giving main med may be a temporary way of treating the symptoms particularly if the reason for the symptoms is higher blood pressure. As soon as the blood pressure comes under control the symptoms should vanish . Regards Dr. Priyank Mody"
},
{
"id": 59735,
"tgt": "Dizziness, pain under the rib cage radiating to back. Strongly feel due to gallbladder. Opinion?",
"src": "Patient: I think my gallbladder needs removed immediately, but can t get results at ER - they just send me home. What do you think? When I looked up my urine, blood results and the ultrasound results, it shows normal except: the gallbladder wall thickness is 3.5 mm (which they say in report is normal - but online, says that is high) pH 5.5 A/G Ratio - .8 (it says below low normal) MCH - 32.5 (it says above high normal) Neut% - 68 ALP - 104 MDX% - 9 I have had gallbladder attacks since 1994 - but never any gallstones. I have also been diagnosed with asthma , fibromyalgia and Crohns (though I am overweight ). I had stopped going to the emergency room because the answer is always the same - no gall stones, so go home, don t eat fatty foods. I went to the ER last night bc I have had constant pain and flare ups daily since Oct 25, and now the pain never goes away (under right rib cage, middle of rib cage, back under shoulder blade), chills, dizziness , nausea. My GI dr told me that I should ask for the test which shows how much bile is going in/out of the gallbladder, but of course, they did not - they did an ultrasound as usual - and surprise (not) - no gall stones, follow up with GI dr. - I feel that I am in need of immediate surgery. What do you think? Some things which would indicate possible infection or other disorders are on the high side of normal: This morning - once again - dizzy, pain in the same places, getting worse in middle abdomen Doctor: Hello! Thank you for the query. As surgery is a very serious procedure it needs some indications to have it performed. Your symptoms might be caused by gallbladder inflammation without gallstones (this is very rare) but it can also be caused by peptic ulcer, severe pancreas disease or even heart problems (including heart attack). That is why before any surgery you should have abdominal CT (to make sure the problem is not pancreas), liver tests (AST,ALT,GGTP,AP - if last two elevated ERCP should be performed), bilirubin level, amylase and lipase level, urine analysis. Heart ECG and heart enzymes should be performed as well (CK-MB, Troponine) to see if there is any ischaemia. As you do suffer from asthma and it very often is associated with gastric reflux disease you should also have upper GI performed to see if there is no peptic ulcers or duodenum swelling. Only after all this tests if no other reasons of your symptoms you may start thinking about surgery. Hope this will help. Regards."
},
{
"id": 218059,
"tgt": "What is the pain in my upper arm?",
"src": "Patient: i HAVE BEEN HAVING PAIN IN MY UPPER ARM WHEN I MOVE IT CERTAIN WAYS. IF I HOLD IT STRAIGHT OUT TO THE SIDE IT CONSISTANT. SOMETIMES IT CRACKS. I THOUGHT IT WAS FROM CARRYING A HEAVY BAG TO WORK DAILY. SOMETIMES IT HURTS TO SLEEP ON IT. CAN YOU GIVE ME SOME INFO ON WHAT I'M EXPERIENCING? Doctor: HelloThanks for the queryIf you have carried something heavy then it is most likely to be a cartilage tear. I advice you to give rest to the arm for 5 days, if the condition does not improve then you will have to get an MRI of the arm in order to evaluate for the painI hope I was of helpRegards"
},
{
"id": 101028,
"tgt": "Suggest remedy for allergic spots",
"src": "Patient: hi im 28 yrs old. im having allergies red/pinkish spots their starting to get plenty i don't know what food causes these allergies but eversince i was a kid i never got any allergies until now they started at the back of my legs and now it's in my stomach whom should i consult to? should i visit a dermatologist? Doctor: HIThank for asking to HCMI really appreciate your concern and looking to the history given here I could say that this is the allergic condition and in my opinion this need to be treated with the antihistamine tab Levocetrizine 10 mg three times in day and right now no need to see the dermatologist if the symptoms does not improved then you can see the skin specialist, have a nice day."
},
{
"id": 152258,
"tgt": "Pain and burning sensation in the left side of neck and feet with numbness",
"src": "Patient: hi, my mom who is 54yrs has this pain and burning sensation in the left side of her neck, down to her feet. The burning is inside the skin whiles there is numbness outside. The pain sometimes gets so severe she has to take strong pain killers. It actually started with some kind of stroke (at least that s what the doctors told us) which disabled her for a while. She now walks but not as perfect as before. Doctors here have tried so many medicines but we are yet to find one that can help. What do you think is wrong,Please help. Doctor: Hi dear, Any sort of burning sensation, numbness, pain without any injury or specific pathology be it after a stroke of central nervous system like the brain or due to reasons like peripheral nerve impingement (compression) shows an inability of the nervous system to transmitt proper response of sensations to the brain. Painkillers is not the absolut solution. As she had weakness causing disability which is now recovered it seems more or less a residual symptom of a stroke. If medications have not been effective, I would like you to try Sensory Re-education therapy done by a Physical Therapist involving different tecniques of electrotherapy and exercise therapy combined with oral nutritional supplementation for the nerves. Hoping it helpes. Dr.Shirish Sachdeva Orthopaedic & Neurological Physical Therapy & Rehabilitation Lifestyle Medicine Counsellor"
},
{
"id": 133338,
"tgt": "What causes stiffness and severe pain in the joints suddenly?",
"src": "Patient: I came to europe last month and all of a sudden i had stiffness and severe pain in neck, shoulder, both hand and finger joints, mild fever and flat reddish rashes in under arms and thighs. As per doctor advise, I took aspirine, caltrate vitamine D3 and voltarene L.P.75mg for few days. I had noticed blood in the stool and stopped the medicine. After 15 days the pain subsided in the neck and hands but slight pain persist in finger joints. kindly advise Doctor: hi,thank-you for providing the brief history of you.As you have pain in the joints a thorough clinical examination is advised.Also, such sudden pains can be probably due to either of the factors like - Rheumatoid arthritis or Viral arthritis.As both contains similar symptoms and needs a clinical examination and also the blood routine for RA and Viral arthritis.Based on the clinical findings and laboratory finding the diagnosis will be easy to make and drug therapy will be condition oriented.Also, physical therapy will help to reduce the pain and aches along with Strengthening of the muscles.In my clinical practice most cases with RA and Viral arthritis respond well to a combination of drug therapy and physical therapy.Regards Jay Indravadan Patel"
},
{
"id": 127890,
"tgt": "What causes muscle cramps in the legs?",
"src": "Patient: I am experiencing severe muscle cramping. It sometimes starts with the back of my arms and then moves to the entire arm. This last attack was in both arms lasting about 5 minutes. It sometimes happens starting in a hip moving yo my entire leg. It s been getting progressively worse and happening more often. Doctor: Hello,For cramps in muscle; electrolyte disturbance should be ruled out first. Check your serum calcium, sodium, potassium level.If electrolytes found alright, then neurological nerves and muscular examination can be done. If needed EMG and NCV testing can be done. Consult neurologist for examination if needed.Hope I have answered your query. Let me know if I can assist you further.Regards,Dr. Parth Goswami"
},
{
"id": 80096,
"tgt": "Suggest treatment for upper chest pain",
"src": "Patient: Hello , Im 30 year old african american woman, and have been having alot of pain and problems with the left side upper chest. I recently found outi have a bone spur in my left shoulder and the doctors though that was causing the pain. however the pain is not in my shoulder its like a burning sensation on the top of my chest . like the musceles are tight Doctor: Thanks for your question on Health Care Magic. I can understand your concern. Bone spur can cause muscle irritation and muscular pain. So better to avoid heavyweight lifting and strenuous exercise. Avoid movements causing pain. Avoid bad postures in sleep. Start painkiller and muscle relaxant drugs. Apply warm water pad on affected site. If you are not improving by all these then you need surgical removal of this bone spur. Hope I have solved your query. Wish you good health. Thanks."
},
{
"id": 160693,
"tgt": "Suggest remedy for fever and redness and swelling in scrotum",
"src": "Patient: I have a 3 1/2 yr old toddler who has a fever of 100.0 and his scrotum is red and a little swollen. He s not complaining of pain or itching and isn t having any difficulties urinating. He s eating and drinking fine. What do you think it is? Thank You, Mammie of Bub Doctor: Hi,Fever along with redness and swelling of scrotum can suggest a diagnosis of acute epididymo orchitis (infection of testes) or less commonly a scrotal abscess. As these conditions usually need antibiotic therapy, I will recommend you to consult your doctor. Meanwhile, you can paracetamol for fever.Hope I have answered your question. Let me know if I can assist you further. Regards, Dr. Muhammed Aslam T. K., Pediatrician"
},
{
"id": 210357,
"tgt": "Suggest remedy for confused mind, mood swings and irritable behaviour",
"src": "Patient: hi doctor, i am srividhyafrom 4 to 5 months i am not abel to take any correct decision.frequently my thoughts are changing and suddenly i get angry for silly reasons.i have a elder sister she was in us, and my younger sister also trying to go there from past 4yr i have no plannens to go. but suddenly i am getting jealousy on my sisters.i dont why.i am in relationship he is a gud guy but compare to my family status his family is poor and he spends lots of money for unnecessarily.i wont like that. At present i want to break my relationship and some time i dont want to break, i dont what exactly i want to do.please help me out Doctor: Hello, looks like depression is creeping up. it is better that you start some antidepressant like : escitalopram or sertraline in low dose ( 5 mg for escitalopram and 25mg of sertraline), and see how you feel. The effect usually comes in two to three weeks. You may need a higher dose once there is improvement to get 100% response. it is best to avoid making any major decision in lie ( fro relationship etc. ), till you become better as the illness may make you take wrong statements and wrong actions for which you may regret later. Hope the reply is usefulDr. Manisha GopalMD Psychiatry"
},
{
"id": 162220,
"tgt": "What should be the dosages of Tylenol and Motrin in a 2.5-year-old suffering from high fever?",
"src": "Patient: My daughter is 2.5 and been battling high fevers for going on the 5th day now. I\u2019ve been giving Tylenol round the clock and started alternating with Motrin yesterday. She is due in 1 hour for Motrin (she refused Tylenol imbetwen so I decided to skip it). She is back up to 103.8 now. Can I give her her next dose of Motrin or do I need to wait the hour?? Doctor: Hi, This looks like a viral illness. Paracetamol (Tylenol) can be given in the dose of 15mg/kg/dose (maximum ceiling dose of 500mg) every 4-6th hourly that too only if fever is more than 100F. Ibuprofen can be given at the dose of 10mg/kg/day every 8th hourly and you can alternate these two drugs at the specified intervals, please. Hope I have answered your query. Let me know if I can assist you further. Take care Regards, Dr Sumanth Amperayani, Pediatrician, Pulmonology"
},
{
"id": 210740,
"tgt": "Why do I feel body numbness? When will I become normal again?",
"src": "Patient: I am having a hard time writing and typing the past few days. I was experiencing full body numbness. today I feel better but my right arm and hand are slow responding. I went to the doctor and they concluded it is stress/anxiety related. How long before I feel completely in control again? Doctor: HiThanks for using healthcare magicI think, you have writer cramps syndrome. It is a psychological disorder due to excessive use of hand and due to underline anxiety. You can take low dose antidepressant and benzodiazepine. That would help to decrease anxiety. For these drugs, better to consult a psychiatrist for further management.Thanks"
},
{
"id": 191581,
"tgt": "How to treat high blood sugar levels naturally?",
"src": "Patient: Hello Doctor, I was diagnosed with Pre Diabetes. I try to watch what I eat and am delayed on ex resizing bc I fractured my foot last year. once Ash Wednsday I feel extremely tired, so many bathroom trips, irritable and brain is foggy . I had a veggie burger this morning and some chick peas but my stomach is so BLOATED I never had this. It looks like I am pregnant . Please help me with my high blood sugar so to heal quickly. I tried calling my doctor but the office is closed and I will remain in bed . Please help me . My email is XXXX Please leave a message if I do not pick up bc I may be in the bathroom for the thousandth time today . Thank you Doctor: check blood glucose levels fasting and post prandial ,hba1c.you need lifestyle modification & exercise daily along with a proper diet plan specific to u."
},
{
"id": 152279,
"tgt": "I fell swollen in the back of my head close to my neck. Is my dizziness caused from my head injury ?",
"src": "Patient: hi my name is maria and I fell and hit the back of my head close to my neck and its swollen on that area, for the past 2 days I have been getting dizzy spells , and nautious, is this serious where i would need to go to the emergency room or can i wait til tomorow to see my doctor ? what can I do ? is my dizziness caused from my head injury? Doctor: you have to meet the doctor if you experience severe headache and vomiting otherwise you can wait and take some pain killer which you are not allergic and take rest. dizziness may be caused by the disturbance of your balancing organ present in your ears which gets disturbed when there is fall."
},
{
"id": 85719,
"tgt": "Is tetralysal 300mg advisable for acne treatment?",
"src": "Patient: i having anche in my face.earlier i had lot so doctor suggest to take medice tetralysal 300mg for 3mths that time the anches gonne, this happen 4 yrs back.now again i am getting this pimples.so i though of taking this same medecine again.now i am in india. Doctor: Hello,Tetralysal may be taken for acne. On the other hand, it is best to talk to your doctor and have a through check before starting any medications, especially when it comes to taking an antibiotic (tetralysal) for 3 months.I hope this answers your query. I remain at your disposal in case further medical assistance is needed.Regards,Dr. Antoneta ZotajGeneral and Family Physician"
},
{
"id": 177882,
"tgt": "What treatment to be taken for the gastric problem for new born?",
"src": "Patient: My son was born 12 days ago and for the last 2 days we are having problems with gas pains. He is being fed Similac sensitive which is intended for gas but is still suffering. He will cry for several hours at a time turning red in the face and tensing up his abdominal muscles passing gas every so often. I have tried massaging is belly and doing the bicycle leg motion which seem to help the gas pass but do not relieve the pain. Both parents have no known allergic reactions. I am wondering if giving him simethicone gas relief drops like Pediacare or Little Remedies or a product like Tummy Calm or Colic Calm would be safe? If he is awake and moving around after feeding there does not seem to be a problem with the gas, it is only when he is in his sleeping hours and doesn t get enough movement in to help push everything in the right direction. He was born 3 days late with no complications and is healthy. Doctor: Thank you for the query.It seems to be lactose intolerance. kindly pay attention to the timing of pain with feeding.Mother should be fed wholesome diet without chilies.Simethicone is quiet safe for some times but not to be used routinely.You can try putting baby on belly while sleeping it will help get rid of gases.It is still advisable to check with a pediatrician."
},
{
"id": 54821,
"tgt": "Suggest remedy for liver problem",
"src": "Patient: there is some problem in my lever so i got my lever tested and i came to know that my sgot is 60.6 which is more then normal and now i am taking live 52 and all kind of precautions and i also want to tell u that i took some steroids for long time and i was feeling a heaviness and some burning sensation on the upper abdomen from very long period and i got digestion problem too, and i think i got toxic hepatitis, so please tell me the solution and which is the good medicine for liver and the doses if possible Doctor: Hi hanks for asking question.You have not mentioned reason for using steroid.It can affect liver if taking for long time.So try to avoid use of steroid unless it us indicated for some diseases.Usg done in your case with complete LFT profile...If fatty liver comes udiliv 300 mg tablet can taken...Avoid use of alcohol.Eat more fruits....Regular follow up done...Trans fat diet like non veg , butter , cheese taken very less....Take care...No need of using other drugs."
},
{
"id": 203790,
"tgt": "Why is the tingling pain sensation in the testicles having hurt while playing?",
"src": "Patient: My fianc\u00e9 is having a severe tingling pain sensation in his testicles.....he has already seen a doctor and is being sent to see a fertility specialist to examine his sperm count. He had one semen sample taken and it showed that his sperm count was 1 million vs a normal male sperm count of about 20 million. Now this I don't think has anything to do with his symptoms but what could be the cause of this tingling pain in his testicles. He has not hit them or damaged them while playing a sport for example. please help. Miss Marie Doctor: Hi ! Good morning. I am Dr Shareef answering your query.Most of the time, this happens due to unnoticed repeated trivial injury to the testes even during normal day to day activities if the testes are not supported by properly fitting undergarments. If I were his doctor, I would advise him for a well fitting testicular support, some anti inflammatory, and a broad spectrum antibiotic if need be after sending a urine sample for routine and culture sensitivity test to rule out a urinary tract infection which could give rise to similar problem. If there was no relief, I would advise him for a Doppler ultrasound of both testes to rule out a chronic torsion of the testis. Further management would depend on the investigation reports. The sperm count could get affected in case of untreated infection of the urogenital system and also a condition called varicocele. So, it has to be ruled out by a physical examination of your fiance by a doctor.I hope this information would help him in discussing with his family physician/treating doctor in further management of his problem. Please do not hesitate to ask in case of any further doubts.Thanks for choosing health care magic to clear doubts on your health problems. Wishing you an early recovery. Dr Shareef."
},
{
"id": 86918,
"tgt": "What causes stomach pain and headache?",
"src": "Patient: Hi, may I answer your health queries right now ? Please type your query here...I have been sick to my stomch everynight for the last 3 weeks and today for the first time Im sick in the morning. Also accomanied by a headache. There is no way I can be pregnant cause we use condoms and I have also had a pap smear and 2 years ago a colon oscopy and at the same time a endoscopy. All they found was that I may have IBS. What couls these symptoms possibaly be? Doctor: Hi.Thanks for your query.Read the history and understood your problem.You have already mentioned IBS , a known problem that exists in you. You have a classical symptoms of pain in the abdomen every night , again indicating more of IBS as timing can be a problem in IBS, although you had the recent attack in the morning. It is always better to get the investigations repeated again. This is to rule out any new problem.I would advise the following:Blood, urine and stool.Ct scan of the abdomenColonoscopy and Upper GI EndoscopyAll these investigations will either prove or rule out any problems. -If the investigations are normal, get all the dosages rescheduled or altered."
},
{
"id": 144882,
"tgt": "What could cause seizures after drinking, have history of epilepsy?",
"src": "Patient: ive had 3 seizures in the last month after having got a little drunk in japan before that i hadnt drank for 4 years but before that i had a severe drinking problem and epilepsy, i was hoping that my epilepsy had stopped but i know i shouldnt have drank and i have stopped and will not continue ever again, what advice can you offer Doctor: Hello,Feel sorry for your condition but seems you got the solution.Let me explain; it is well known that there exist a wide group of trigger factors for epilepsy to happen.Main subject here are metabolic disorders which could be caused from excess of drinking alcohol,also high temperature,physical and mental stress...etcIt seems in your case that drinking means seizures, so STOP DRINKING.BUT an appointment with your doctor is recommended to determine epilepsy type, etiology so as to derermine treatment.Wish this helped and feel free for any other information."
},
{
"id": 121916,
"tgt": "How to get rid of severe pain and numbness of leg ?",
"src": "Patient: A 29 year old male presents with extreme burning and cramping pain upon exertion in the lower limbs. The patient is a competitive marathon runnerand recently ran the New York marathon despite the pain. The patient reports that the pain coincides with the start of physical activity. When the pain was first noticed, it subsided almsot immediately after the completion of physical activity. Over time, the pain has become more acute annd now it takes some time after the cessation of activity for the pain to subside. The patient also reports that recently numbness and and extreme weakness of the calf will result after sustained physical activity. Doctor: Hello, His symptoms could be related to a pinched nerve. Another possible cause to consider would be a muscle disorder or chronic venous insufficiency in the lower limbs, or a connective tissue disorder. For this reason, I would recommend consulting with a neurologist for a physical exam and some tests: - nerve conduction studies - a Doppler ultrasound of the leg vessels - CPK plasma levels - complete blood count, PCR, ESR for inflammation - ASOT titer. You should discuss with your doctor on the above tests. Hope I have answered your query. Let me know if I can assist you further. Take care Regards, Dr Ilir Sharka, Cardiologist"
},
{
"id": 220762,
"tgt": "How to treat lateral ventricle of 10.2mm and the other as 12.6 indicate?",
"src": "Patient: My wife is 34 weeks pregnant. During an ultrasound today one latertal ventrical was 10.2mm the other was 12.6. Up to now all scans have been fine. We had an amino earlier in preganacy and that was ok. What could have caused this and what are the complications for the baby? Doctor: It indicates mild venticulomegaly. Go for regular scans if it doesn't increase in size then no need to worry but if it increases in size then consult your gynaecologist as well as paediatrician."
},
{
"id": 26117,
"tgt": "What causes faster heart beat and tight feeling in throat?",
"src": "Patient: Lately my heart has episodes of beating so strongly it feels like something pounding on my chest and my throat feels a little tight and breathing somewhat constricted. I have had this occasionally in the past but it happens several times a day now for the past three or four days. I have never had cardiac problems. Doctor: hello, I have gone through your query. Thanks for using HCM. I need to know following details before guiding you properly. 1.What is your age ? 2.Do you have high blood pressure,high cholesterol or Diabetes? 3.Do you have family history of heart disease? once i get this information i shall be able to guide you,Meanwhile get and ECG done to know about possibility of heart problem. My best wishes Dr.Rajesh Teli,MD."
},
{
"id": 49999,
"tgt": "Elevated creatinine level. Takes less protein, wine, OTC herbal medicines, no anti inflammatories. What to quit?",
"src": "Patient: My creatinine level was 1.03 on 5/7/13 and 1.27 on 8/8/13. Both abnormal and elevated. What does this mean. My doctor said to reduce protein intake by 20-30%, avoid alcohol and antiinflammatories and reduce OTC herbal meds. Protein is not much, alcohol is limited to wine and I have not taken motrin, etc in several weeks. However, I do take OTC herbal meds. So, which ones do I quit? Doctor: Hi, thanks for using healthcare magicIn terms of the over the counter herbal medications it would be difficult to say which to avoid. The exact ingredients of many herbal medications is not known.There is also no regulation that requires testing of ingredients or the effect of different body systems such as the kidney.It may be best to avoid them.In addition to the other precautions your doctor mentioned it would also be important to keep tight control on your blood pressure and blood sugar.I hope this helps"
},
{
"id": 180730,
"tgt": "What causes sharp pain in the jaw upon walking?",
"src": "Patient: Sounds strange, I know, but I have a sharp, stabbing pain in my upper left jaw when I walk. With each step down on the left foot, the pain shoots from my jaw to my lower eye area. It almost feels like a cracked tooth, but I have dentures, so I assume it's not that! Doctor: Hello,It looks like nerve-related pain as the pain seems to be occurring along the course of a nerve. If you are on dentures and do not have any tooth, then it is not related to the tooth, but still, you need to get an x-ray done to get checked that you have any remaining root pieces.Also, the sinuses need to be checked. In case if it is nerve-related pain, then you will be advised Neurotic painkillers like Gabapentin, Carbamazepine etcetera for relief.Hope I have answered your query. Let me know if I can assist you further.Regards,Dr. Honey Arora"
},
{
"id": 24837,
"tgt": "What causes high blood pressure?",
"src": "Patient: hi, i turned 40 in November. I have never had a problem with high blood pressure. The past week i have not felt well. I have a weird feeling in my head sometimes. Just a sense of overwhelming or pressure. my arms have been aching and hurting some; as well as my hims and legs at times. Diabetes and heart disease are in my family. I felt really bad one day and checked my sugar with my father s meter 2 hours aftet eating. My blood sugar reading was 140. But i have checked it before eating lunch the past days and it was 79 and 78. I have checked my bp at home also. It has been readings such as 134/99, 140/101, 123/90, 134/96..and elevated other times too. I ve been checking it several times the past few days. What should i think and do? I had already made an appointment with a new medical doctor for march 31 to get established. I called back but the receptionist said he had no openings before that date. And if i needed attention before to go to a local clinic. I am making it but i just don t feel well. Should i be more concerned or fine to keep waiting till my appointment? Doctor: Hello!I carefully passed through your question and would explain that your symptoms do not seem to be related to any cardiac disorders. The muscular pain could be related to a viral infection or electrolyte imbalance. This does not seem to be an emergent situation and you can wait until your doctor consultation. I would recommend performing some tests: - complete blood count- muscular enzymes- PCR, sedimentation test for inflammation- liver and kidney function tests- blood electrolytesRegarding your blood pressure values, I would recommend to closely monitor your blood pressure different times during the day ( in sitting position, after 10 minutes of total relax),write them down and show them to your new doctor. The values you refer are within the normal ranges, and some of them are borderline. This means that you do not need to start antihypertensive therapy. Some lifestyle modifications would be enough to help you maintain a normal blood pressure: - a healthy diet ( low salt and caffeine intake)- a lot of physical activity (aerobics, walking, jogging, recreational sports, etc.)- reduce your weight if you are overweight. Considering your blood sugar levels, they are within the normal ranges and do not indicate diabetes. So do not worry about them!Hope you will find this answer helpful!Kind regards!Dr. Iliri"
},
{
"id": 56152,
"tgt": "What causes stomach pain above navel with normal ultrasound result?",
"src": "Patient: I have stomach pain above the naval. Feels like I got punched in the stomach. Also pain for the last week s alternating right and left but lately on the right. Never had this before. Had an abdominal ultrasound aboutt 3 weeks Ago which was normal. We were looking for gallbladder problems. Doctor: Hi there,Thanks for posting in HCM.There are many causes for pain in the abdomen above the belly button. Some of the common causes are peptic ulcer, pancreatitis, gall bladder disease and liver disease. Ultrasound scan has ruled out gallbladder and liver disease. So now you to get tested for other causes. Consult a gastroenterologist for an upper GI endoscopy and other blood tests.I hope that answers your query.Regards."
},
{
"id": 20396,
"tgt": "Suggest treatment for uncontrolled hypertension",
"src": "Patient: Hi, may I answer your health queries right now ? Please type your query here... About a month ago they said i had a TIA. I have weakness in my right leg still but they can't control my blood pressure. i have tried 6 different medications to no avail and have almost eliminated my salt intake. Please help me. Doctor: Thanks for posting in HCM. I understand your concern. There are multiple causes of high blood pressure and they need to be investigated thoroughly before changing medications. I can help you understand the type of medications you are taking and how they are treating you. Mainly you will need to have about two medications for hypertension to be started now and then change it two weekly if it is uncontrolled.Till then you can start taking increased amount of green tea daily, avoid alcohol or smoking etc, reduce your stress levels and get your fresh investigations done especially RFTs and share with me so that i can help you and also state the type of medicines you have used so far.I hope you will understand the importance of medicines and investigations for me to decide for you. Get well soon and dont miss the medicines your doctor advised you until you are advised not to take."
},
{
"id": 83291,
"tgt": "Could vitamin B12 deficiency be caused by taking escitalopram?",
"src": "Patient: Can B12 deficiency caused by Escitalopram? I ve been taking 10 mg Escitalopram for 5 years and now I have a 107 B12 level. I have been eating red meet, eggs, spinach and lentils for years. Pernicious anaemia test was negative. So can it be the Escitalopram? Doctor: Hello, Your vitamin B12 level (107) is below normal is not due to Escitalopram. There is no proven such side effect of decrease vitamin B12 level. If you have symptoms of deficiency of vit-B12 like numbness, tingling sensation to hand or feet then you van take 5 injections on alternate day of Vitamin B-12. Work with your primary care doctor to achieve this. Take care. Hope I have answered your question. Let me know if I can assist you further. Regards, Dr. Pramod Kokare, General & Family Physician"
},
{
"id": 86750,
"tgt": "Suggest remedy for a persistent pain in the lower abdomen",
"src": "Patient: General Surgeon, I have been in pain for a great while . Lower Lt, Abdomen . Know I think I discharged OR have broken mesh , I added weights to my yoga work out Know My husband and son have G P S s me . I can t do anything till I get results CT / IV push {no results as yet. However I think the Dr. was on medication herself . In fact I asked her twice if she was alright. My PC-is on Vacation. Should I tell her about the Dr. and have a different test? or is that the test I would have been given by P C Regards M S. Doctor: Hi.Thanks for your query.It looks you had a Hernia prolene mesh repair and now you have the pain at the same area after Yoga. Yes, you have to get a proper clinical evaluation and Physical Examination done by a General Surgeon and get a final diagnosis. If there is appearance of an expansible lump which is reducible the diagnosis is obvious and you need to get this operated."
},
{
"id": 177211,
"tgt": "What causes head sweating and skin coolness?",
"src": "Patient: My son is almost 3 months old. When he fell asleep in my arms earlier I noticed his head was quite sweaty but his skin felt cool but then went away. Is that a cold sweat? Is that normal and what causes that? He does get sweaty sometimes but I never really noticed what I noticed today. Im really nervous. HELP PLEASE !! Doctor: Sweating is normal body's mechanism to regulate temperature. Infants head body surface area is also more. So excessive sweating is normal. Take care that you don't cover the body more.Keep the extremities covered to keep them warm. Regards"
},
{
"id": 112541,
"tgt": "Chronic back issues, have calf tear. On methadone and oxycodone. Done discectomy. Will I get better?",
"src": "Patient: my pain mng doc put me on methadone today and oxycodone for my chronic back probs had 2 surgerys 1 a discectomy siatic prob 2 same prob cage plates so much pain is the prob also have calf tear gastrominiscus/also planter fasitious with a groth in arch around akillies.surgery is to fallow foot calf soon my guestion is will I benefit from the methodone and oxycodone for the pain Doctor: Hello, Thanks for writing to usYou will benefit from the methodone and oxycodone for the pain as adviced by your doctor. Beside these you need good physiotherapy exercises to strengthen your back and calf and for pain control.I do hope that you have found something helpful and I will be glad to answer any further query."
},
{
"id": 75718,
"tgt": "What causes severe breathing trouble?",
"src": "Patient: I am a 19 year old female. I have had pneumonia 4 times now. This last time I tried going to the doctor for 3 days to get shots but on the 2nd day my breathing was so severely compromised I was admitted to the hospital. While there my potassium levels dropped, my white blood cell count was almost nothing, and my liver enzymes were elevated. The doctors tested me for cancers, such as leukemia and lymphoma, STD's including AIDS/HIV, and Hepatitis which all came back negative. I'm wondering if this can be normal or if I might have something to worry about? Doctor: Hi thanks for contacting HCM...You are having very young age ....You are suffering from repeated pneumonia Infection....So your immunity might low ...Also look for diabetis in which chances if infection high .If diabetis present drugs for that started.....This last time your infection might be severe and spread through blood into liver so liver enzymes were high ..Bacteremia sometime can lead bone marrow involvement , so wbc count was low ....At that time I.V antibiiotic infusion given according to blood culture report ....If breathing severely impaired at that time oxygen level should be checked in blood ...Take care .Advise : pulmonologist consultation for examination"
},
{
"id": 183143,
"tgt": "What causes swollen jaw and body aches while having toothache?",
"src": "Patient: My husband has a bad toothache and can't see the Dentist until they are open again on Monday. He has swelling in his jaw and is now experiencing all over body aches and possible fever. I'm worried there's a bad infection that has spread... will he be okay to wait until Monday or is this serious enough to take him to ER? Doctor: Hello,Thanks for consulting HCMRead your query as your husband had toothache dont worry this toothache can be due to carious tooth, decayed tooth , or periodontal problem for this I will suggest you to consult oral surgeon for its treatment inmeamtime do warm saline rinses 4 to 5 times and take medication like Diclomol SP or ketroldt for pain and bodyache .Hope this will help you."
},
{
"id": 186508,
"tgt": "What is the reason for tightening of gums near the molars?",
"src": "Patient: I recently had wisdom teeth surgery about a month ago, and I think everything is healed up now. but just recently, the left side of my mouth, near the molars the gum is feeling tight. explanation for this? and what i should do? im only 15 and have no history of cavities Doctor: thanks for your query, i have gone through your query, the tightening near the gums or at the site of the exatraction site could be the healing of the extraction socket with fibrosis because of the approximation of the buccal and lingual flaps and sutures. nothing to worry, do saline gargling and massaging over that area. i hope my answer will help you. take care."
},
{
"id": 145301,
"tgt": "Suggest treatment for headache and degenerative disc disease",
"src": "Patient: I have severe headaches but I only get the in 1 spot or a another at one time, like in the top right of my forhead piercing pain, then I will get them on the left back side of my head, my doctor says its due to the degenerate disc disease in my lower spine, but I don t feel like that is it, my legs and hips hurt alot most of the time,wrists and fingers, I have dry cracked feet like a diabetic only i m not diabetic or even pre diabetic, can all of this be related? short term memory loss like forgetting what I was doing while I was in the middle of it like completely forgot Doctor: Hello!I read your question and understand your concern.In my opinion the way pain on your back, problems with feet and headache may be related to a cervical disc problem. But trouble with memory is not caused by cervical disc. I would recommend a cervical spine CT scan and a head CT scan for the memory problems.A full blood work for inflammation would be helpful in rheumatic pain.Hope to have been helpful!Thank you for using HCM!Best wishes Dr. Abaz Quka"
},
{
"id": 194377,
"tgt": "What cases constant semen flow after having sexual intercourse?",
"src": "Patient: Hi, you know, i once had a sexual intercourse with this girl and for the first time in my life felt like to pee at intervals of about 2hrs then contacted a doctor who made numerous test and found all 10 negative. later, i started feeling this constant semen flow and pushing me to the toilet. May i know what to do about it? Thanks Doctor: Hi, If it's a constant semen flow, need to think of prostate infection, urethritis. it could also be simple urinary tract infection. Doing an ultrasound pelvis and urine culture sensitivity will be helpful. Hope I have answered your query. Let me know if I can assist you further. Regards, Dr. S.R.Raveendran, Sexologist"
},
{
"id": 125215,
"tgt": "Suggest medication for red swollen joints,red spots on lower limbs & amonia odour in urine",
"src": "Patient: I have painful red swollen joints, i have brusing, i have red spots on lower limbs, Skin is hot to touch, my urine smells like amonia this is on both sides difficultly walking. in my shoulders elbows wrist, ankles, knees and hips my ana was negative .. my primary dr gave me predinsone. my mom has genetic gout and has been on allopurino for 20 years. she gave me one of her tablets i was able to walk a little better. dr doing uric acid level after my attack got worse no results yet. Doctor: Hello, The red and swollen joints are suggestive of polyarthritis. Few blood tests are needed to find the cause. Till then mild pain killers can help symptomatically. Hope I have answered your query. Let me know if I can assist you further. Take care Regards, Dr Praveen Tayal, Orthopaedic Surgeon"
},
{
"id": 160826,
"tgt": "What causes stomach pain and blood in stool in a child?",
"src": "Patient: My daughter, age 6 has been complaining about her Tummy hurting. She has had loose stolls for a week or so and the past few days has had mucus and blood in her stools. She seems fine, she plays and eats and pees normally. What might be the problem? Doctor: Hello, Possibility of a gastroenteritis must be ruled out. Consult a paediatrician and get evaluated. A stool routine is required to make a diagnosis. Antibiotics like ciprofloxacin may be required empirically. Hope I have answered your question. Let me know if I can assist you further. Regards, Dr. Shinas Hussain, General & Family Physician"
},
{
"id": 70124,
"tgt": "What is the lump on my anal opening that usually bleeds?",
"src": "Patient: I have a quarter sized lump on my anal opening that bleeds fresh blood mostly all the time it is kind of tender and does not itch or anything like i have had it for a couple years too embarrassed to tell my doctor there are times i wake up with dried blood in my boxers quanity quite a bit for what this is what is it? Doctor: Hi ! Good evening. I am Dr Shareef answering your query.The lump on your anal opening could be a prolapsed hemorrhoid which would be bleeding on friction with clothes. The tenderness could be due to super added infection on it.When you visit your doctor, you should have complete faith in him and tell him all the details of your problem without any hesitation. This way you would be better examined by your doctor, and would derive maximum benefit from his services apart from getting yourself treated well. You have to get it examined by your doctor /surgeon for a definitive diagnosis and treatment.I hope this information would help you in discussing with your family physician/treating doctor in further management of your problem. Please do not hesitate to ask in case of any further doubts.Thanks for choosing health care magic to clear doubts on your health problems. Wishing you an early recovery. Dr Shareef."
},
{
"id": 182505,
"tgt": "How to treat a gum infection?",
"src": "Patient: HI doctorm i had RCT 9 yrs before. at first stage i was feeling free of any problem. but for the last 3 years i have been suffering from an infection inside the gum beside that teeth. Its swollen there. some times it get burst and white liquid comes out. how can i cure it? Doctor: immidiately go for a radiograph. probably u have developed one periapical lesion beneath that tooth. may be you have to go for periapical surgery. this is a common phenomena patients sometimes stop root canal treatment after 1st sitting. as after that they dont feel any pain. but this way u may loose the tooth also. catch hold of a dentist also"
},
{
"id": 140269,
"tgt": "What causes constant headaches,dizziness and tingling in hands?",
"src": "Patient: Okay so here are my symptoms, feel free to ask any questions... Constant headache (ranging from a 3-9) Periodic Dizziness and bad balance (usually accompanied with a bad headache) Random Neck Pain Laying on where my neck and head meet causing headache, nausea and dizziness Random shaking of my hand/forearm (lasting for 30seconds- to 30 minutes (on and off)) Random numbness and tingling of hands and feet Random nerve thing ( great description huh?) my hand will be poked/pinched/ect and my foot or part of my face will go numb/tingly, this will happen when my foot is poked/pinched/ect Random Memory loss (hard time remembering things) Random Temporary confusion Random Difficultly finding words Random Slurred speech I ve had the headaches and dizziness for almost a year and the other symptoms have made themselves know and have been getting worst for a couple of months. Also, symptoms will be very calm, less frequent and not as intense for a week or two and them get worst for a 2 weeks or so at a time. Doctor: Hello, I would explain that your symptoms could be related to a metabolic disorder or anxiety. But, before concluding that it is just anxiety, I would recommend consulting with your attending physician and performing some tests: - complete blood count for anemia- PCR, ESR for inflammation- a cervical spine X-ray study- blood electrolytes- thyroid hormone levels for possible thyroid gland dysfunction- vitamin D and vitamin B12 plasma levels for possible deficiency. Hope I have answered your query. Let me know if I can assist you further. Regards, Dr. Ilir Sharka, Cardiologist"
},
{
"id": 180034,
"tgt": "What causes fever, blisters on mouth and throat and swollen lips?",
"src": "Patient: 2 of my grandchildren are running low grade fever and have blisters in their mouth and throat with very swollen lips..it started with one child and the second one the next day.We are afraid it might be contagious as well. I Don t know if I have given you enough information. Also we know they did not eat anything unusual. Doctor: Thanks for expressing your concern on HealthCareMagic. There are a number of viral infections that give rise to fevers with rashes, although certain other infections also be causative. The nature of rash can distinguish between them. Based on the information that you have provided, it could be herpes or chicken pox but the information that you have provided is not enough. It would have been best if I could have a direct look but that is not possible. I suggest that you consult a doctor. The infection could be contagious and so I would advise isolation upto the point of time when you consult a doctor. He would also be able to tell you whether it is a simple case of vitamin B deficiency.I hope that answers your query. Feel free to revert back with any further queries if any."
},
{
"id": 74527,
"tgt": "What causes breathing trouble after taking bath at nights?",
"src": "Patient: when my husband takes a shower at night, he puts lotion on his face and body, but has a hard time breathing the rest of the evening. He says his skin feels like the hard foam you would put in a box. He doesn't have this sensation when he takes a shower in the morning. Wakes up all night not being able to breath. Doctor: Respected user , HiThanks for using Healthcaremagic.comI have evaluated your query thoroughly .* This is in relation with allergy or sensitivity to the lotion consumed after night bath .Hope this clears your query .Welcome for any further doubts .Regards ."
},
{
"id": 13521,
"tgt": "Cause for rash more frequent occurrence?",
"src": "Patient: YES, I hAVE HAD BEEN DEAling with a rash for a long time, I first thought that I had glad stuck in my hand , well now these crystals came out all over my arms when I put on Cortizine 10 cream, Then I had paint stuck on my hands and I used some mineral spirits to get it off and I now havean enevelope full of the crystals, WHat is it from? Doctor: Hi, It will be difficult for me to come to a conclusion without a photograph of your disease. If the rash is for a long time, chances of rash being an eczema are high. I request you not to apply any irritants or chemicals on your skin and do consult your Dermatologist for arriving at a final diagnosis and for initiation of the treatment. Hope I have answered your query. Let me know if I can assist you further. Take care Regards, Dr Siva Subramanian, Dermatologist"
},
{
"id": 222607,
"tgt": "How can the gender be found in a TIFFA scan?",
"src": "Patient: i am having 26 weeks 4 days pregnancy..my cervix length z 4 c.m..nd am having less RBC..nd i have done the Tiffa scan...how to find a baby boy r girl in this scan?nd my WBC,PLATELETS every thing z good.placenta z anterior upper nd mid segment.fetal weight 944+/-145gms.is it evrything z good? name: nikhshiptha.. Doctor: HiDr. Purushottam welcomes you to HCM virtual clinic!Thanks for consulting at my virtual clinic. I have carefully gone through your case, and I think I have understood your concern. I will try to address your medical concerns and would suggest you the best of the available treatment options.1] As per law of your country , ask the USG doctor if it is possible to know sex of the baby. In India, it is not permitted by law.2]TIFFA Scan is aimed at knowing or to rule out structural abnormalities in the baby.3] From your reports all seems to be well.4] Please regularly take IRON and CALCIUM supplements, protein powder, folic acid,B 12.5] have healthy diet, regular exercise, eat plenty of fruits, salads.ENJOY your pregnancy. May be blessed with bundle of JOY.I hope my answer helps you.Thanks."
},
{
"id": 21885,
"tgt": "What causes shortness of breath?",
"src": "Patient: Hi I beome short of breath only after I try to walk \"erect\" for more then a half block...if I stop for a short while I can then proceed for say another half block...I'm told it is either my heart or lungs...what's your diagnoses? Age 72, Height 4ft 11 3/4\" Caroline Doctor: it's difficult to say it's related to heart or lungs without any detailed evaluation.Get your 2d Echo done,Chest X-ray,ECG and stress echo.Then only you will come to know what it's related with."
},
{
"id": 95138,
"tgt": "What is the cause and cure of gripey pain in stomach ?",
"src": "Patient: Hello I have a gripey pain in my lower stomach right in the centre as low as poss before you reach the pelvic bone . Not sure if it s gynae or bowel related sort of feels like a mixture of the two. For the last couple of weeks me, my parnter and I have had mild runny tummys. The kind that dont really feel that bad and you just get on with things. I now have less of a runny tummy but this gripey feeling is different. When i go to the loo as a result of the feeling the pain gives me I dont have diahorrea but I do go. I also have been feeling hot and sweaty. I am never ill and this pain/symptoms are a new sensation for me so wanted to ask about possible causes and relief of symptoms please? thanks Doctor: hello chrissie.fitzerald, gripy pain in stomach-may be due to acid peptic disease,associated with amoebic colitis ( in this case pt. have motion soft,oily but not diahorrea) goes upto pelvis, advice- avoid- hurry,burry,curry,spicey,oily food ,alcohol,smoking, tea, coffee,- drink cold water & cold milk, tablet- pentaprazole 40 mg for 21 days on empty stomach once daily, tablet-tinidazole 500mg twice after meals for 5 days, tablet-drotavine twice daily after meal for 5 days, even pelvic disease tinidazole is use ful, you did not mention your age, hot &sweaty- may be due to spasm or in menopause( age after 45years), if associated with burning sensation in urine then may be due to urinary calculus, hope my advice is useful to you, thank you."
},
{
"id": 177693,
"tgt": "How to treat albumin traces in urine?",
"src": "Patient: Hey.. my daughter is 2 month old. 1 month earlier she was admitted into the hospital as she had blood and urine infection. She had albumin traces in the urine. After treatment everything was fine but now she passes urine only 3 - 4 times in a day. Again in urine test albumin traces are present. What does it mean? Is it related to kidneys? What should we do now? She cries a lot when passes the urine. Please suggest. Doctor: Hi...a 2 month old baby crying while passing urine is normal and it cannot be deemed as pain. I am telling this because the sensation of full bladder is perceived by adults as such and we urinate and relieve ourselves. But, babies of this age do not realize that and the only means of expressing that abnormal sensation which they are perceiving is by crying.2nd issue is that , urine containing traces of albumin. Please be rest assured. This is normal. It is related to kidneys only, but it is a normal finding in urinalysis.Hope my answer was helpful for you. I am happy to help any time. Further clarifications and consultations on Health care magic are welcome. If you do not have any clarifications, you can close the discussion and rate the answer. Wish your kid good health.Dr. Sumanth MBBS., DCH., DNB (Paed).,"
},
{
"id": 174075,
"tgt": "What causes decrease in HCG level during pregnancy?",
"src": "Patient: Last week I did ultrasound and blood test. There was no heart beat of the baby from ultrasound and my HCG level initially it was 35000 then after 2 days it dropped to 31000. Now I am 9 weeks, still today the ultrasound result is the same. My doctors told me to do D&C but I really stressed Doctor: HiFall in HCG level with no heart beat is cosistent with finding of non viable pregnancy. As the repeat USg shows the same result, D and C is the right treatment"
},
{
"id": 149037,
"tgt": "Suffered brainstem minor hematoma, MRI indicates white matter ischemic changes. Prescribed statins. Correct treatment? Stop further strokes?",
"src": "Patient: My mother suffered a brainstem minor haematoma in April this year. She has made quite good recovery. Her consultant has recently reviewed her MRI and indicated white matter ischaemic changes. He has prescribed statins. Firstly, is this a good first course of actions. Secondly, should we be worried about her cognition and risk of further strokes? I need to help my father and mother understand the issues of which they need to be aware and future management Doctor: Hi,Thank you for posting your query.Your mother has two types of problems in the brain- the first is a hematoma (bleeding), which is often due to high blood pressure. So, her BP needs to be kept under good control to prevent any recurrence of bleeding in the brain.Second problem is the ischemia- lack of blood flow to the brain. This is often due to high sugar, cholesterol, in addition to high BP. So, if these risk factors are present, then, they need to be controlled. Also, in addition to statins, an anti-platelet such as aspirin needs to be started to prevent further strokes and decline in cognition.I hope it helps.Please get back if you require any additional information.Best wishes,Dr Sudhir Kumar MD (Internal Medicine), DM (Neurology)Senior Consultant NeurologistApollo Hospitals, Hyderabad,My personal URL on this website: http://bit.ly/Dr-Sudhir-kumar My blog: http://bestneurodoctor.blogspot.com/"
},
{
"id": 145699,
"tgt": "What causes severe neck pain and pressure in head?",
"src": "Patient: I have had a nri 4yrs. ago and have a broad based c5-c6 osteophyte complex with out focal disc protrusion or central canal stenosis. My neck pain is worse and it hurts to even hold my head up at times. My head feels so heavy. it is very sore in the back of my neck and i can not sleep with a pillow. Could it have gotten worse maybe need another MRI? Doctor: Hi,The symptoms which are getting severe are indicative of progression of cervical spondylosis. You will need a repeat-MRI scan and have it compared with the previous scan to measure the canal diameter and look for disc protrusion or nerve compression. A nerve might have got pinched as it emerges out from the spine which is causing the severe pain. It needs to be looked at and basing on the severity of symptoms you may need to undergo surgery."
},
{
"id": 178079,
"tgt": "Is pain in left side with fever in a child serious?",
"src": "Patient: My 6 year old daughter woke up with left side pain. This afternoon she had a nap and woke up with a fever and pain on her left side and back and abdomen. Her mother has just taken her to the doc. I am worried and wondering how serious this could actually be. Doctor: Thanks for contacting HCMYou are concerned about your daughter who woke from sleep with a fever and left back and abdominal pain. This sounds like she has a urinary tract infection and she needs to be seen right away. I agree with her being taken immediately to her doctor. This condition is easily treated with antibiotics but may require hospitalization if serious.Hope I answered your medical question. Please contact us again with your medical questions and concerns"
},
{
"id": 190358,
"tgt": "Swollen gum on the front teeth, increasing in size. Have a set of braces put in. Cause?",
"src": "Patient: My gum is swollen in the front of my two top front teeth and is is also swollen behind. I have had it for 2 weeks and thought it should be gone by now. It started growing a little after i got my 4th set of braces and they put a double chain on my teeth. I don t know if this is from the braces or something but i really need to know. Doctor: HELLO AND WELCOME, THE braces you are wearing now ,is interfering with your oral hygiene maintenance. food debris and plaque has deposited in between your teeth resulting in inflammation of gums called gingivitis.there is pseudopocket formation along with pus accumulation. get ultrasonic scaling by your dentist.if necessary,root-planning and curettage has to be done. the enlarged gums can be reshaped by gingivoplasty and gingivectomy. use orthodontic toothbrush to clean your teeth. chlorhexidine mouthrinses can be used to prevent plaque accumulation. take tab.doxycycline-100 mg,once daily for 7 days to make your gums intact,firm and resilient again. regards DR.BINDIYA"
},
{
"id": 10231,
"tgt": "Suggest treatment for hair loss with baldness",
"src": "Patient: i lose my hair frome some place of my head . its looks round &round if i touch that place by my finger then many many hair r come out . i sufer by this problem frome 5 to 6 month. i m very upset by this illnes & i loos my confidance also so sir plz help me. Doctor: Hello and Welcome to \u2018Ask A Doctor\u2019 service. I have reviewed your query and here is my advice.In my opinion according to your complaints, you are suffering from Alopecia areata. You can apply Minoxidil solution. Along with Minoxidil you should use topical or oral corticosteroid according to the size of bald patches. Do consult your dermatologist to confirm the diagnosis and initiating therapy.Hope I have answered your query. Let me know if I can assist you further."
},
{
"id": 117032,
"tgt": "Suggest treatment for ITP",
"src": "Patient: hihis is srihari my daughter P sneha suffering with ITP from septemer'13 still now last one week back her platlight count is 17k. at the time of diagnosis they used IVIG & anty D after few day again platlights dorps down. now we are taking christen medical college Vellour. now given 3 months course of folic Acd & Dapson 50 mg per day. is this cornic or acut ITP. kindly giveme correct treatment for this mobil.0000 Doctor: Hi, dear. I have gone through your question. I can understand your concern. You have idiopathic thrombocytopenic purpura. It may be chronic if long time. In that there is immune mediated destruction of platelets take place. So you should take immunosuppressive drugs like steroids to control that. Consult physician and take treatment accordingly. Hope I have answered your question, if you have doubt then I will be happy to answer. Thanks for using health care magic. Wish you a very good health."
},
{
"id": 196885,
"tgt": "What causes semen leakage from penis?",
"src": "Patient: Hi, I'm 27 and facing a problem as I'm having semen leakage from penis a lot of the times in a day, specially if I got a little excited while talking to my girl then calmed down, could you please advice if I need to run any tests with my doctor or it is normal ? Doctor: HelloThanks for query ,To get emission while talking to female friend is normal at the age of adolescence and is not a leakage of semen but a excessive secretions of a mucus by mucus secreting glands located in Bulbar part of urethra.These glans get stimulated and secret more mucus by audio visual stimuli triggered even in presence of female partner .It is normal at the age of adolescence and does not require any treatment. It usually gets resolved spontaneously over a period of time and when one gets engaged in physical intimacy of a female partner .Dr.Patil."
},
{
"id": 82517,
"tgt": "What causes shortness of breath?",
"src": "Patient: I am 28 years old with a history of irregular smoking cigarettes, maybe 1 per day on and off with nothing regular, and a history of marijuana smoking, regular for 10 years but discontinued now. I have not had shortness of breath until recently. I kissed a girl who went to an urgent care clinic earlier that day complaining of shortness of breath. She was told it was asthma and it was not contagious. We kissed, and a couple of days later, I have developed shortness of breath. My doctor prescribed me a z-pack, an inhaler, and asked for a chest x-ray. The chest x-ray revealed what the radiologist called mild pulmonary hyperinflation otherwise negative chest x-ray When I look this up, it says COPD and emphysema, and my doctor referred me to a pulmonologist. I had a chest x-ray about a month and a half before this to check for TB for a health screening to enter China, which I will do next month. No TB and no comments about hyperinflation. My symptoms started after I kissed this girl. I m still experiencing shortness of breath or the feeling that I m not getting enough air in. Doctor: Thanks for your question on HCM. In my opinion you are having chronic bronchitis. It is caused by smoking and marijuana. And cause shortness of breath.Chest x ray can be normal in early stage.And kissing can not cause this. But by kissing viral infection can spread and this can exacerbate bronchitis.So better to consult pulmonologist and get done PFT (Pulmonary Function Test).PFT is must for diagnosis of chronic bronchitis. It will also give you idea about severity. And treatment is depending apon severity of the disease.So consult pulmonologist and get done PFT."
},
{
"id": 193198,
"tgt": "What causes abdominal cramps with stomach pain, odor and hematuria?",
"src": "Patient: My stepson is in the hospital now and has had abdominal cramps for the past 5 days they staff at the hospital do not know what is wrong with him. His symptoms added to the pain in his stomach are blood in his urine, adbnormalities in his liver function and I have noticed a funny smell on him. He is 16 and normally healthy. A few years back he had giardia with similar symptoms when he swam in an infected dam Doctor: Hi, The smell may be due to fungal infection. The cramps are caused by liver abnormalities, usually liver patients have bad odor during breathing. Have you completed your treatment? Does your step son have constipation and bad odor during breathing? I hope I have answered your query. If you have further doubts , I would be happy to help you. Happy day.Regards, Dr. S. R. Raveendran, Sexologist"
},
{
"id": 135142,
"tgt": "Could bump on leg after injury be a blood clot?",
"src": "Patient: Hi. I hit my leg pretty good in the front it was bruised and the bruising travel down my leg the bruising has gone but I still have a big hard pump on the front of my leg and hurts to put presser on it ,if I wear socks it put a indent could this be a blood clot Doctor: Yes, that could be an organised blood clot or fat growth. Get an Ultrasound to confirm. Apply ice to that region."
},
{
"id": 72165,
"tgt": "What is the cause of sharp chest pain?",
"src": "Patient: Hi, may I answer your health queries right now ? Please type your query here... Hi my mom says she has a real sharp pain right below her chest where her stomach and chest area meet in the middle by the rib cage...it causes her cold sweats and the pain goes to her back Doctor: Hello dearWarm welcome to Healthcaremagic.comI have evaluated your query regarding your mother in details .* This pain has different possible causes as - acute gastritis or duodenitis - ulcer stomach or duodenum - issues with pancreas - coronary insufficiency giving ischemia - others .* Needs physician evaluation and EKG .Wishing her fine recovery .Feel free to ask any further queries .Regards ."
},
{
"id": 28414,
"tgt": "What causes dizziness,pressure in chest and heart palpitations?",
"src": "Patient: This only happens on occasion, but is concerning. I feel dizzy and like I am off balance, when this occurs I think I get anxiety because then I feel short of breath and my chest feels a bit heavy. It doesn t last long but is scarey. I sometimes have heart palpitations, I am 51 and wondering if could be peri menopausal symptoms Doctor: Hi, Your symptoms may be explained as presyncope. I want to know about 1) Presence of diabetes, high blood pressure problem, cholesterol problem if any. 2) Current medication if any. 3) Habits like smoking, alcohol. 4) Any recent illness like loose motions, vomiting, excessive sweating, high grade fever. I will suggest you to undergo following test1) ECG electrocardiogram preferably during episode. 2) Holter's monitoring, electrophysiological study of heart. 3) 2DEcho (echocardiography) 4) Complete blood count, hemoglobin level. 5) Blood pressure check in supine position and In standing position to know postural hypotension. 6) If all these tests are normal then there may be need of MRI brain with angiography and carotid artery doppler. In your case there may be possibility ofCardiac arrhythmia (abnormal electrical activity of heart), postural hypotension, problems related to blood supply of brain. These symptoms cannot be explained as perimenopausal symptoms."
},
{
"id": 202464,
"tgt": "Is this true that hair loss treatment may lead to male infertility?",
"src": "Patient: Hi,Am 28 years old male. I am suffering severe hair loss for the past two years. My doctor asked me to use Morr-f solution. Thought I did not get hair growth, hair loss is under control somehow.Some of my friends say that hair loss treatment may lead to male infertility. Is this true? Doctor: Welcome to the forum.One of the 2 components is finasteride, which blocks the formation of active formation of male hormone.Its known side effects are decrease male sexual function and loss of desire.Male infertility is not described as a side effect, but in theory it can cause.Hope this helps,sincerely"
},
{
"id": 141891,
"tgt": "Does a swollen, stiff and painful neck require medical attention?",
"src": "Patient: Hi. I ve had a sore stiff neck for about a week. starting Thursday it started to get more painful. Today it is very painful at the base of my skull.. It feels swollen and tender and it hurts down my neck now too. Just recently it started to have a random nervy type of pain on one side. Wondering if I should have it looked at tomorrow or if it will probably go away. Doctor: Hello!My name is Dr. Aida and I am glad to attend you on Healthcaremagic! Your symptoms could be suggestive of a bulging intervertebral disc in the cervical spine. For this reason, I recommend consulting with a neurologist and performing a cervical spine CT scan or MRI. Inflammation tests (PCR, complete blood count, sedimentation rate are necessary too). You should discuss with your doctor on the above issues. Kind regards!"
},
{
"id": 177577,
"tgt": "What could lump behind right ear indicate?",
"src": "Patient: My 3 year old son today showered and I was dressing him and found a lump behind his right ear, in the past he would say he gets dizzy and legs would hurt. Then his pre-k teacher stated he sweats while he naps and I witnessed that one day. Could this be cancer Doctor: The growth is unlikely to be cancer and you need not jump to conclusions. To be able to comment on what exactly it is, it needs to be examined properly. Please visit a doctor and get your child evaluated. Sweating during naps may be normal if it is humid to some extent."
},
{
"id": 93394,
"tgt": "Tenderness in left pectoral muscle, pain on touching. Taking lercanidipine hydrochloride. History of high BP. Cure?",
"src": "Patient: I have tenderness on the left pectoral muscle. It starts right at the top of my left breast . In the same spot on my right side the pain is minimal barely noticeable but to my left its tender and painful to the touch. I ve been using Algesal Diethylamine Salicylate 10%w/w for the last couple of days to massage it. I am on Lercanidipine hydrochloride 20mg daily. Kindly advise please Doctor: Hello,You're complaining of left side chest pain and you're a known hypertensive on treatment.Any chest pain should not be taken lightly unless the cause is found.This becomes all the more important due to increase cardiac risk with hypertension.If it is a musculoskeletal pain than any movement and deep breathing will aggravate it.If the pain is more central and towards the left and radiating to the arms than most likely cardiac pain.Consult your doctor for more help."
},
{
"id": 126062,
"tgt": "Why do I feel pain and vibration in my elbow and wrist region?",
"src": "Patient: Hello, I would like to explain a problem with my wrist region as well and my elbow region. It seems that most of the times if lift something, such as weights, I sometimes get a buzzing/ vibrating feeling in my elbow or wrist region. It is somewhat painful. A similiar feeling is when you hit your funny bone. Doctor: Hi, Those tiny, uncontrollable spasms are called muscle fasciculations. It sounds scary, but in most cases, they\u2019re nothing to worry about. These spasms are often caused by an imbalance of electrolytes in your muscle fibers as they fatigue. So drink plenty of fluids and fruit juices. Spasms that last a long time or occur on a regular basis may be a sign of kidney or thyroid dysfunction, fibromyalgia, or other neuromuscular disorders. Please consult your physician he will examine and treat you accordingly. Hope I have answered your query. Let me know if I can assist you further. Regards, Dr. Penchila Prasad Kandikattu, Internal Medicine Specialist"
},
{
"id": 14086,
"tgt": "Can I take valtrex for shingles?",
"src": "Patient: I have a rash on my neck, itches very much went on sat. to a walk in clinic doctor said it looks like shingles. My question, when will it starts hurting. They put me on Valtrex. until Monday when I go to my doctor. Is that a good medicine to take for shingles if that s what I have. Doctor: Hello and Welcome to \u2018Ask A Doctor\u2019 service. I have reviewed your query and here is my advice.Valtrex is a good medication for Shingles. Do continue it as per your doctor's advice. Some patients may get pain after shingles which we call as Post herpetic neuralgia. If Valtrex therarpy is initiated quickly after the appearance of rash, the chances of occurrence of postherpetic neuralgia is less.Hope I have answered your query. Let me know if I can assist you further."
},
{
"id": 193194,
"tgt": "Suggest treatment for delayed ejaculation",
"src": "Patient: I m 26 weeks pregnant and my boyfriend hasn t been able to ejaculate while we have sex for a few weeks now. he doesn t any problems getting an erection but can t come. this has never happened before. is this to do with the pregancy, and will it go awy in time or is there a bigger problem? thanks Doctor: Hi, Take hormonal tests it can be due to thyroid, renal calculi and prostate inflammation or infection. Take tests for these mentioned hormones. Had any treatment? I Hope I have answered your query. If you have further doubts, I would be happy to help you. Happy day. Regards,Dr. S. R. Raveendran, Sexologist"
},
{
"id": 50819,
"tgt": "Swollen kidney with stones. Blood in urine. How serious is it?",
"src": "Patient: hi,,my name is mike and i have a kidney stones.my left kidney is swollen and as per my doctor says i have big stone in my left kidney and some in my right..he gave me medicines to intake,and i took the med. yesterday,when i pee in the morning i encounter some blood clots..is this a normal or there is something to with what i'm taking,,how dangerous is my situation right now....thank you for your advice.... Doctor: Hi, Welcome to the forum. I understand your condition. Multiple bilateral stones with swelling is not to be taken lightly. You need to undergo urine analysis, blood tests and if necessary CT scan or IVP. The treatment depends on the stone size, location and kidney function. You need to meet a urologist immediately. Best regards"
},
{
"id": 216737,
"tgt": "What causes pain in lower right back?",
"src": "Patient: I have a dull pain that has been in my lower right back that I can also feel from the front, when it is hurting. Sometimes the pain can also radiate upward and effect my right side. Not very painful, just nagging. Stretching can help, but it never really goes away. I m thinking sciatic pain but how can I be sure? Kelly Doctor: hi,thank-you for providing the brief history of you.A thorough clinical neuromuscular assessment is advised along with MRI of the lumbar spine.This pain symptoms mentioned by you may be dude to pinch nerve and so as the symptoms are arising may be due to this. After an thorough neuromuscular assessment taking an MRI will help understand the soft tissue status.Also, as you feel better with Stretching the pain can be due to Sacroiliac joint issues which post clinical assessment and MRI will be understood.I will recommend you to undergo physical therapy as it will help reduce pain and inflammation by Therapeutic ultrasound therapy and TENS therapy. Performing exercises at a later stage will help provide better muscular strength and joint stability.In my Clinical practice we recommend patients to physical therapy and they respond well with outstanding results.RegardsJay Indravadan Patel"
},
{
"id": 80330,
"tgt": "What causes a feel of liquid on the upper left chest muscle?",
"src": "Patient: I feel as though there is a liquid near the surface of my upper left chest muscle near the arm pit that burns when I lean foreword in a standing position. The burn is very intense like a white hot iron is slouching in that area. just started 4 hours ago and is increasing in area. feels liquid in nature, but i feel no bump and it may be inside rib at that location? do not feel it by putting finger pressure in general area. Doctor: Thanks for your question on HCM. I can understand your situation and problem. In my opinion you should first get done 1. Chest x ray 2. Ultrasound examination of local part. Lung infection or skin infection (left upper chest area) can cause symptoms of burning and fluid sensation. If both these are normal then no need to worry for major diseases. Also get done vitamin b 12 level, hemoglobin estimation as b 12 deficiency and anemia can give rise to similar sensation. If any of these are deficient then you need supplements. Don't worry, you will be alright."
},
{
"id": 130321,
"tgt": "What does dent on left arm with weakness indicate?",
"src": "Patient: Hi, my name is Candice. I have a dent/dimple on my left arm (outter side of my arm, where the muscle is) inside of the dent is a small round little knot/ball. doesn't really hurt to touch it just feels weird. this arm is a little weaker then the other but I am right handed. what is this.....should I be worryed Doctor: you should do a usg for that swelling....is that increasing? how many years u have it? is it moving with movements of arm...?"
},
{
"id": 153700,
"tgt": "How long survival is possible with gallbladder cancer?",
"src": "Patient: 80 year old female. Diagnosed with gallbladder cancer that has spread. She was treated with chemo from feb 2010 to feb 2011. It was discontinued because it was \"causing more harm to her than to the cancer.\" Her abdomen started to swell about a month ago. This week her feet and legs are huge. She seems to be getting senile very quickly this past 3 weeks. Her back hurts along with her 'tummy.' My question is this. When do I gather the family? Is there anyway for me to know how long it will be before the end? Doctor: Hi, dearI have gone through your question. I can understand your concern. Prognosis of gall bladder cancer depends on type and stage of cancer. General health factors and treatment also affect the prognosis. Of course chemotherapy hurts more then the disease at her age. So it is better not to take chemotherapy. Prognosis is poor and life expectancy is not good. Hope I have answered your question, if you have doubt then I will be happy to answer. Thanks for using health care magic. Wish you a very good health."
},
{
"id": 90113,
"tgt": "What is the growth found in appendix?",
"src": "Patient: my friend has a growth on her appendix identified by CT scan. if it burst it can be very bad, it might be maligent and needs to be removed. They might do path while she is under the knife that will determine how they remove it. what is this growth called Doctor: HIThank for asking to HCMI really appreciate your concern and looking to the history given here I could say that this might not be growth because this is very unlikely but this could be swollen appendix, get it out and send this for histo-pathological test, hope this information helps you, take care and have a nice day."
},
{
"id": 61772,
"tgt": "Suggest cure for small bumps on the thighs",
"src": "Patient: my 7 year old daughter has small bumps on the inside of her thighs, they started about 3 months ago, but just today I noticed some have spread and 2 are now bigger, lik a big pimple that needs to pop. The rest are small, and seem to have a clear liquid inside. What is this and should I take her to the doctor? Doctor: Hi.Thanks for your query.The inner aspect of the thigh can develop multiple sebaceous cysts like bumps due to friction with the clothes, allergy to clothes and so on. Since the condition started 3 months ago, you may please check which clothes are causing this sort of allergy and to avoid using these.Apply liquid povidone iodine locally to avoid secondary infection.Another reason can be scaboid form of dermatitis.Consult a Dermatologist so that a proper diagnosis and treatment can be undertaken for cure."
},
{
"id": 176140,
"tgt": "What causes 6 weeks baby to be fussy about having formula milk?",
"src": "Patient: i have a 6 week old daughter...no one in my house smokes. when we go to my in laws house (my mother in law smokes outside of the house only) my daughter gets fussy and doesnt like to drink her formula. She is normally a good baby and doenst fuss all that much and drinks her bottles like she should. everytime we go over there and she gets like that my mother in law insists i spoil her by holding her all day or that the formula is making her gassy or her tummy is bothering her. but when we are at home she is fine. my mother in law was watching her for a few hours the other day and when we went to pick the baby up she hadnt slept for more then 10 minutes and had only had about 5 oz of her formula. my husband and i are convinced it is the smoke that lingers on my mother in laws clothing and in the house that is affecting her and causing the fussiness...my mother in law thinks we are crazy is the lingered smoke be the cause of this? Doctor: I have carefully gone through the details. Although smoke could indeed be a cause of the fussiness of the child, it probably plays an independent role. It could be that formula milk itself is the cause of the fussiness. Let me emphasize on the fact that breast milk is the best possible feed for a baby and there are can be no alternatives for the same. Most formula milks are cow milk based and contain more proteins than the tender intestines can tolerate. Hence the formula milk itself could be the cause of the fussiness. I would strongly insist that you try to switch back to breast milk either in the form of breast feeding or as expressed breast milk. The production of breast milk is based on the child's demand and more the baby sucks on your breast, more would be the amount of breast milk produced."
},
{
"id": 77303,
"tgt": "Does upper chest pain after a fall need looking into?",
"src": "Patient: 68 year old female fell and took all in pact on left side of chest... Had pain when breathing and when not breathing... Felt very tired and slept that night and most of t he next day.. The next day no pain...went about doing things ...the next day had pain and after putting up Christmas tree my left upper chest hurt. Doctor: Hi thanks for asking question.Mostly by fall she had bruise or muscle tear that can lead to chest pain.Rest is done.Analgesic taken for pain like iibuprofen.Hot compress can be useful.If ribs are tender to touch then x ray done for rule out rib fracture.Mostly you will recover in few days.If still problem not clear then blunt trauma induced hemothorax has to be ruled out.X ray will useful for it.Take care...Dr.Parth Goswami"
},
{
"id": 90086,
"tgt": "What can be the reason for pain and a bulge in the lower abdomen?",
"src": "Patient: feels pain and a bulge on my left lower abdomen 5 months after hysterectomy especially with deep inspiration, coughing, laughing & sneezing. Pls help I am 50yrs, my height is 1.63m & my weight is 67kg. The indication 4 hysterectomy was grossly enlarged uterus due to fibroids. My name is Adaobi Doctor: Hi.Thanks for your query .The bulge looks to be a hernia, what we call as Incisional hernia. In females the hernia can occur just a place away from the incision- it appears to be . The best way is to get an examination by a Surgeon. Confirm the diagnosis and get operated to get a relief."
},
{
"id": 88079,
"tgt": "What is the treatment for lower abdominal pain and soreness in the skin?",
"src": "Patient: Hi I'm been getting stomach pains mostly in the lower part of my stomach. It's been happening for 7 years since I was 14. I get dizzy and week. My skin can be sore to touch, normally my legs. I feel sick but rarely am and I'm constantly tired.I've had 3 smears that have all came back borderline changes but keep getting told to go back in 6 months. From March 3rd Ive had my period every 2 weeks. I'm not pregnant, don't have a problem with my bowels (IBS), not anemic and don't have endimetriosis.My doctors just send me away saying 'See what happens next month' I'm scared they're missing something important but don't know what to ask them to test for. Can anyone please help? Doctor: Hi.Thanks for your query and an elucidate history.All the detailed history you have provided is suggestive that you may have a problem in the spinal cord. Sine you must been done with almost all tests of abdomen, I would suggest you the following. if not done yet ::Investigations:*Routine blood investigations like Complete blood picture, blood sugar, urea, creatinine, liver function tests, thyroid function tests.*Urine -routine and microscopy*Stool: routine, microscopy, occult blood, culture and sensitivity.*Ultrasonography of abdomen,*Colonoscopy All these investigations should be within normal limits for one to say you have IBS.In addition get an MRI done f the whole spine: This may show why you have sensitive skin along with pain and also have a second opinion of a Neurologist."
},
{
"id": 162356,
"tgt": "How can fever, headache and constant clearing of throat in a child be treated?",
"src": "Patient: 7 yr old has been blinking hard for days (started after given Ritilin, but discontinued), been clearing his throat, today had fever 102+ (gave childrens Tylenol) and complained of headache. After Tylenol fever seemed to break and sweated a lot but now temp seems to be coming back up. Doctor: Hi, Fever and headache are less likely to do anything with intake of Ritalin. In my opinion, it could be a viral fever. You have to keep giving Tylenol. If the fever does not subside, then go for blood investigations. Hope I have answered your query. Let me know if I can assist you further. Regards, Dr. Himani Saroha, Pediatrician"
},
{
"id": 203488,
"tgt": "How to treat leakage of stingy fluid from penis?",
"src": "Patient: I had oral from a girl three days later had the discharge and burning but three weeks later after a lot of anti biotics I only have a slight sting in the head and when that happens I squeeze a few times and a tiny amount of clear stringy fluid will appear Doctor: DearWe understand your concernsI went through your details. Isuggest you not to worry much. Your penis was burning after a mouth job due to the rashes you had during that. You cannot blame that incident alone, your hygiene is also questionable. Three days antibiotic treatment may not be enough.Take a five day course under a competent physician. Why do you want to squeeze your penis? The liquid escaping on squeezing could well be semen mixed with urine. Do not take self medication. Please consult a physician.Hope this answers your query. Available for further clarification.Good luck."
},
{
"id": 35586,
"tgt": "What causes painful baltod?",
"src": "Patient: I am frequently suffering from baltod since after marriage. Once I recovered from the first one the second one occur. It has been more than 12 Baltod within 3 month. Its really paininful. Although I am using Antibiotic and Pain Killing medicine advise by doctor. But my query why this is constantly happening. Doctor: HelloAs far as known to in this area \"BALTOD \" is known as frunculosis or ingrown hairs infection, causes may be :1 Diabetes ( not necessary in young persons ), in spite of this get in blood sugar examination for confirmation.2 Streptococcal infection i.e. resistant infection.3 Unhygienic care ,some times I have noted that due to improper skin care there develop infection the hairs root .So take bath with anti bacterial soap .Get in blood examination for culture and drug sensitivity test for proper antibiotics.Also get in stool examination as sometimes worm infection also produces recurrent baltod.So consult a dermatologist and get his opinion.Good luck."
},
{
"id": 222646,
"tgt": "What are the early signs and symptoms of pregnancy?",
"src": "Patient: Hi, I a 20 days late for my period. For the last 2 weeks I have had tender nipples, white spots on them, almost pimple like, constant nausea, I had small period like stomach cramps 2 weeks ago but no bleeding. I have had 3 negative HPT. I am not stressed about anything nor worrying about anything. Do you think I could be pregnant? I did not get any of these symptoms with my 1st child. Doctor: Hello dear,I understand your concern.In my opinion the symptoms said by you are similar to pregnancy symptoms but the negative test rules out pregnancy.The delay in period might be due to hormonal imbalance.If the test is negative the medicine for withdrawal bleeding can be taken after consulting doctor.Best regards..."
},
{
"id": 139864,
"tgt": "Can MRI find a healed herniated disc or slipped disc?",
"src": "Patient: Hello, I was in a wreck in September. I was rear ended at about 60 mph. I m still having lots of back pain, especially after therapy. My pain doctor wants me to get an MRI. My question is will an MRI still find a healed herniated disc or slipped disc? Doctor: Hello,A herniated or ruptured disc does not usually heal given the poor vascular supply to cartilage - this is similar to the menisci in knees that do not heal when they tear. So, yes the MRI would be able to still see it.Hope I have answered your query. Let me know if I can assist you further. Regards, Dr. Janne Nissinen, Neurologist"
},
{
"id": 74348,
"tgt": "Suggest treatment for chest tightness after inhaling toxic fumes",
"src": "Patient: I inhaled fumes from cutting steel with a torch that had fireproofing behind it.the fumes took my breath away and stopped me from breathing for a second.when I got home my nostrils were a little burnt feeling.Now my chest feel tight.What can I do to get this out of my system? Doctor: Thanks for your question on Healthcare Magic. I can understand your concern. Sudden inhalation of toxic fumes can cause bronchospasm. And thus spasm causes tightness in chest. Best treatment is inhalation of fresh air. You can also take antihistamine like levocetrizine or fexofenadine. In severe cases inhaled bronchodilators (formoterol or salmeterol) are also beneficial. Hope I have solved your query. I will be happy to help you further. Wish you good health. Thanks."
},
{
"id": 77113,
"tgt": "Suggest remedy for right sided chest pain upon doing any physical activity",
"src": "Patient: I have had shingles for about 5 days. Was doing very well with pain medication, sleeping mostly. A couple of days ago, felt I should stop being a slug. Just lifting jugs of milk and water out of refrigerator started pain in right chest and around side that has not stopped. Pain meds have been upped. Question: Do I go back to being a slug or just try to ignore pain? Doctor: Thanks for your question on Healthcare Magic. I can understand your concern. By your history and description, possibility of musculoskeletal pain is more. Your pain is associated with activities, so musculoskeletal pain is more likely. So no need to worry much. Do following things for better symptomatic relief. 1. Apply warm water pad on affected areas. 2. Avoid movements causing pain. 3. Avoid bad postures in sleep. 4. Avoid heavyweight lifting and strenuous exercise. 5. Continue painkiller. You will mostly improve in 1 week. If not improving even after 1 week then consult doctor told rule out internal cause for pain. Hope I have solved your query. I will be happy to help you further. Wish you good health. Thanks."
},
{
"id": 157283,
"tgt": "What is the possible life expectancy of a person with cancer spread throughout the body with seizures?",
"src": "Patient: My sister in law has been living with later stage Pancreatic Cancer for 2 years receiving Chemo. About 2 months ago the doctors informed her that the chemo was no longer retarding the growth of the cancer and she opted to stop chemo. She was told that the cancer had spread throughout her body. About 3 weeks ago hospice started coming to care for her. It is now in her brain. She had a seizure today and was taken to the hospital by ambulance. How long are we looking at. We live in Florida, they live in NJ and we would like to know approximately when to plan our trip to NJ. Thank you. Doctor: Hi...the life expectance at this stage will be maximum six to eight months...maybe even lesser...Dr. Ashish Verma"
},
{
"id": 139974,
"tgt": "What could cause brief tingling in head and limbs when running?",
"src": "Patient: my head, back of legs and feet get numb/tingly when i look sideways (left or right) while i run. the sensation lasts only between 3 to 5 seconds but it s been like that for mayber over 6 years. i don t feel faint, but i m curious as to what it could be. Doctor: Hello, Your symptoms could be related to physical or emotional stress. Anyway, I recommend performing a cervical spine X-ray study in order to exclude a possible bulging disc in the cervical vertebral column. Hope I have answered your query. Let me know if I can assist you further. Take care Regards, Dr Ilir Sharka, Cardiologist"
},
{
"id": 165371,
"tgt": "Suggest remedy for constant vomiting after food intake",
"src": "Patient: always vomiting...its not like a projectile vomit but hes always vomiting even like 2 hours after food or milk and i dont know why, im not giving him too much and hes not sick....its not hurting him because it does nt seem to phase him at all...ive changed his formula twice but its not only after milk...if i give him a yogurt an hour later he ll probably vomit it up...i jus dont understand Doctor: Vomiting in a baby may be due to infections, reflux disease, cow milk allergy, overfeeding etc. Some serious causes\u200b of vomiting are intestinal obstruction, meningitis, uremia etc. but such causes seems unlikely in your case.You ask your doctor to prescribe some antiemetic medications and if not relieved ask for investigations like USG Abdomen, CBC etc."
},
{
"id": 198395,
"tgt": "How to treat infection post circumcision?",
"src": "Patient: I got circumcision about a week n a half ago..I've been taking care of it properly but I think it might b infected..if so is there anything I can do to treat the infection..like anything at home so I won't have to take off work to go to the doctor..I'm 21 if that helps. Doctor: HelloThanks for query .You have undergone surgery of circumcision and worried about it getting infected .You need to take broad spectrum antibiotic like Augmentin and anti inflammatory drug like Diclofenac twice daily .along with topical antibiotic ointment like Neosporin twice daily.Ensure to wash your genitals with warm water and anti microbial solution Betadine twice daily.Ensure to avoid sexual encounters till it heals up completely.Dr.Patil."
},
{
"id": 150937,
"tgt": "Numbness in hand, 3 fingers, difficulty in holding or grasping things. History of stroke. Suggestions?",
"src": "Patient: Im a 46yr old male ,history of heart problem, on warfarin 9ml an also taking inhibace plus 5mg+12.5mg, metoprolol succinate 23.75mg, also having inr test my last test result was 2.8 but i have not had one for the past 3mths . my problem is that i have numbness in my left hand but only in the index ,fourth an little fingers, having also had a stroke about 8yrs ago,finding it hard to hold on to or grab hold of things properly Doctor: Hi, Thank you for posting your query. Isolated numbness of the hand fingers may be due to a peripheral nerve problem of the hand, usually caused by the mechanical compression of the hand nerve, such as while sleeping. As of now, it does not look like stroke. However, if the numbness spreads to involve forearm, arm and face, it could suggest a new stroke. Best wishes, Dr Sudhir Kumar MD DM (Neurology) Senior Consultant Neurologist"
},
{
"id": 28755,
"tgt": "Suggest treatment for chills, swallowing difficulty and low grade fever",
"src": "Patient: husand which we thought was having flu symptoms (chills/hot and cold), but is also having problems producing mucas to swallow, definitely dehydrated (giving electrolyte water as of yesterday). this is day 7 of his fatigue/dizziness when standing. Still having low grade fever/ chills with hot and cold, no appetitite, urination remains good, no diarrhea / no vomiting - just gagging. He s on just fluids and clear broth. He s taking Ibuprofin so I can t feel a high fever, just a low grade one which aspirin may be hiding. Very tired. Thought it may be norovirus symptoms. Doctor: Hello and Welcome to \u2018Ask A Doctor\u2019 service. I have reviewed your query and here is my advice. I understand your problem. As the symptoms you mentioned above, your patient may have tonsillitis or pharyngitis or viral fever. Check his throat with a torch or consult a ENT specialist. Do not take aspirin. Treatment advised is: 1) Drink plenty of fluids 2) Warm salt water gargle 4 times a day 3) Take Tab. Combiflamm twice daily and Tab. Azithromycin once a day for 3 days. Hope this will help you. Get well soon thank you"
},
{
"id": 14302,
"tgt": "How to cure medium size red itchy puss filled bumps on top of feet and legs that is worsening ?",
"src": "Patient: I ve got medium size red itchy puss filled bumps on top of my feet & all the way up my legs started yesterday & has gotten worse! I have took benadryl &applied hydrocortisone it seems to help a little but it s driving me crazy. only different thing I can think of is a new pair of jeans I got & maybe someone tried them on at store then put them back maybe caught something from that??? I did not wash them before wearing as they were new. PLEASE HELP!! Doctor: Hi since the rash seems to be pus filled, you will need an antibiotic course along with local treatment. Please consult your doctor for this--regards"
},
{
"id": 46906,
"tgt": "Is kidney transplant, effective for high creatinine levels?",
"src": "Patient: My sis. has been suffering from a kidney disease since past 4yeras. Her S. cretine level is increasing day by day.. her blood urea nitrogen Serum is 36 and creatinine serum is 4.3 on dated 26.02.11. is there any solution to control or cure it..? Is kidney transplant is the only solution? Doctor: Hello and welcome to HCM.As an Urologist and kidney transplant surgeon,i understand your anxiety.Your sister has end stage kidney failure and can be treated with 2 options :1. Dialysis or 2. Kidney transplant.Dialysis is mainly of two types : Hemodialysis or Peritoneal dialysis.A kidney transplant is the best option,with better quality of life,in long term.If she has no major medical risk factors,she should go for it immediately.If you've any doubts,send them to me,as a direct question.Dr.Matthew J. Mangat."
},
{
"id": 102064,
"tgt": "Suggest medication for sinusitis",
"src": "Patient: Hello, I suffer from chronic sinusitis and asthma. I am currently taking Clarinase, trying to clear up my sinus. It keeps on returning as my right paranasal sinus is very narrow. Is there anything I could do to prevent a really severe onset of the sinusitis? Thanks, Mel Doctor: in your case the sign and symptoms suggest that you are suffering from allergic sinusitis and medicines can control it not cure it in my advise if you get allergy tests to find out allergens causing this repeatedly and eliminate avoidable allergens and go for immunotherapy you will come out of this in my experience of treating cases like you"
},
{
"id": 164230,
"tgt": "Suggest treatment for cold and breathing difficulty in an infant",
"src": "Patient: Doctor.. My baby girl is 24 days old. She got cold and when she is breathing, she is making noise as though something is stuck in her throat.. what shoould I do? I live in a place where temperature is 30 degree centigrade. We switch ON AC during night times in our bedroom..is it because of AC that she got cold? I am very very worried..as she is too little to bear cold ...plz suggest as what I should do Doctor: Hi, this could be due to nasal blockage. This is very common in small babies. However, an examination by doctor should be done. Take care."
},
{
"id": 6409,
"tgt": "Which medicine should an obese with irregular periods and pcod take to get pregnant ?",
"src": "Patient: i have pcod . overweight irregular periods 9mmfollicle found doctor gave duphaston for 15 days but now 16days past i have no periods .tests were neg. is there any chance of preganancy still no periods .my question is how could i be preganant?. we inter course regularly .what are about my periods ?what procedure should i take to get preganant fast which medicine should i take. Doctor: welcome to nhealthcaremagic you should consult gynecologist and get all tests done minimum thing required is your patent tubes and ovulation and normal semen you should take regular exercise and healthy balanced diet and reduce your treatment should take regular treatment for pcos if needed laparoscopy should be done for infertility it is required to take regular treatment for both if husband also having problem"
},
{
"id": 759,
"tgt": "Will I get pregnant by having sex during ovulation?",
"src": "Patient: hi doc iam 27yr old,ihad duphation for 5days but i dint get my periods so v consulted doc she said hormone is low so she precribed me pivox xr tablets and too she prescribed me yasmin tablet for 11days andby the way i had my periods on 11/12011 and on fouthday of menses i had clomid tablets according to docs prescription and on the 14th day ihad a scanning she said iam ovaluted and we had love,doc my question is will i get pregnnant?as well as iam having duphaston on second stage for 10days?how to know tht iam pregnant?will it work wth this kinds of tablets?iam worried pls help me thnk u Doctor: Hi,I understand your concerns.Following is my reply:1)\u00a0\u00a0\u00a0\u00a0\u00a0Yes. There are chances of pregnancyYou can contact me anytime directly to ask question by pasting following link in your browser:http://bit.ly/askdrsoumya"
},
{
"id": 118104,
"tgt": "Suggest treatment for fluctuating platelets counts",
"src": "Patient: Hi, I am an ITP Patient. I am 27 years old. First diagnosed in 2002 when my platlets count was 40,000. That time Bone marrow test result was Pheripheral destruction. after this upto Jan, 2011 i was completely normal even i never felt that i have any problem. But my platlets count once again fall to 5,000 only when i shifted to a place very far from my home where the air and water(chlorinated water) was very poluted. my treatment was done with blood transfussion + Deltacortil tablets. Then my platlets count was normal for a few months but then fall down even taking the deltacortil tablets... then the doctor suggested to discontinue medicine. After this my platlets were falling down but when it reached to 30,000 it then start increasing. After remaining normal for 3 months now again falling... I want to ask.. 1. what treatment U suggest... 2. Changing living place is a factor or not... 3. Removing Spleen is a solution or not... Doctor: Thanks for contacting HCM. Dear patient there are many approaches to treatment and following treatments can be tried :-1 Steroids, like your doctor gave you delacortil.2. Human anti D immunoglobulin helps in some cases.3.Immunosuppresants like azathioprine or mycofenolate mofitil.4.intravenous immunoglobin (IvIg).5. Thrombopoitin recptor agonists like romiplostim and eltrombopag.6. H.pylori eradication in some cases has helped.7. Platelet transfusion should be used only in dire emergency as it has short lasting effect causes autoimmune destruction donor's pletelets.Splenectomy can help as cause long term remission in 60-65% case but there is a definite risk of bleeding during surgery.Changing place will hardly have any effect.regards"
},
{
"id": 20148,
"tgt": "How quickly will Ramipril bring down the blood pressure?",
"src": "Patient: Hello. Sorry bothering you, but I am concerned about the condition of my partner. He is having a constant headache the last 2 weeks. We went to doctor, who measured his blood pressure along with other checks. My friend's blood pressure stayed constantly around 165/115. He was sent to do Chest X-Ray, blood test, urine test, ECG. Unfortunately it will take 2-3 weeks to get the results. Meanwhile he was prescribed to take Ramipril once a day. There are 5 days passed, and his headache is not disappearing. We are going later to the doctor to check the blood pressure again. I am so concerned that I don't know how long it will take my friend to get rid of this terrible headache. Could you tell me how long it takes for Ramipril to affect the high blood pressure to lower? Thank you very much! Doctor: Hello, Thanks for posting your query on Healthcare Magic...* Brief Answer: Other causes of headache must be excluded, Ramipril is a long acting drug and it would take on or two weeks maximum to fully act.*Explanation: My opinion is that high blood pressure on many occasions is an indication for control .That happens on many levels. First, be sure that high blood pressure is not secondary to any other diseases as Cushing or renal failure.Then ,Diet with low salt and less fat content is crucial. Exercise and other healthy activities are also a mean to control blood pressure. If all that dose not help, medications are a must .That is usually decided by your doctor according to your condition and comorbidities .Some drugs that are perfect for patients are not necessary good for others.Regular blood pressure monitoring is as important as starting treatment so that we could assess efficacy of the medication we started.I suggest my patients the following : headache may be a symptom of hypertension but we should exclude other causes of headache such as teeth problems, eye, ear and neck stiffness.*Conclusion: I suggest you take simple analgesia such as paracetamol , acetaminophen . If you have any further questions I will be happy to help, If not, you can close the discussion and rate the answer. Wish your partner good health."
},
{
"id": 167875,
"tgt": "Is pinching sensation on left chest and high pulse in children a cuse for concern?",
"src": "Patient: My daughter is exerperiencing a pinching sensation on the left side of her chest just the the left of the breast bone with a high pulse following. It hurts when she takes deep breaths. I have experienced the same thing and several deep breaths make the pain go away, as if my lung is pinched in between my ribs. She has had it happen twice in one day, and she was not exerting herself during either instance. She is 7.5 yrs. Im wondering if I should be concerned with something more. Doctor: your are describing a child with costochodritis , which is the inflammation of the cartilage that holds the ribbon bones , it hurts when you push on it and some times with breathing , ibuprofen is helpful in this case .if your child senses palpitation or you feel her heart is racing then you should try to count the number of beats in one minute, and take her to her pediatrician to order some more tests"
},
{
"id": 173340,
"tgt": "What cause cause walking delay in a 1 years old child?",
"src": "Patient: Hi my baby is on eyear and 4 months. She is not walking yet, but she can walk on furniture or with support. All her developments were delay. She turned up by 6 months only. Her left foot is not in correct shape when she placing it at floor it not completely touching the floor . please suggest me what i can do for it Doctor: hi thank you for consulting me in health care magic...1.From what i have read here,i assume your baby is cruising but not walking independently...which is ok for her age and it is apt to wait till 18 months, and children usually should walk by 2 years and if not u need to get her development assessed with a developmental specialist then2.Regarding the next concern about delay in all the milestones,i need more details about her past (antenatal,birth and post natal history) to rule out any insults from history and also prematurity..so u send me the above details which will help me to guide u better3. last being the left foot ,its shape and position ...please be seen by a orthopedician for the same,because if it is fixed / positional talipus (club foot) or anything else it would need their interventionregards,dr Sowmya"
},
{
"id": 78659,
"tgt": "What could cause sharp fluttery feeling in lower rib cage?",
"src": "Patient: for the past two days i have had this fluttery feeling in my lower right rib cage, it feels like a baby moving around and every once in awhile it feels like a baby kicking it has gotten worse happening more ofter through the day, there is no pain just annoying and doesn't seem right. i have in the past had sharp pains in my chest a couple of weeks ago but have had none recently should i be concerned and make an appointment with my dr. i do not drink alot of caffeine, i do have a cup in the morning but i drink a lot of water and some juice though out the day i may have one to two soda's a week Doctor: Thanks for your question on Health Care Magic. I can understand your concern. In my opinion, you are mostly having stomach related symptoms. Stomach contractions can cause similar kind of flattery sensations. But better to rule out heart diseases like arrhythmia. So first get done ecg, 2d echo and Holter monitoring (24 hour continuous monitoring of ecg). If all these are normal than no need to worry. Your symptoms are mostly due to contractions of stomach. And to avoid these symptoms start proton pump inhibitors. Avoid hot and spicy food. Avoid large meals, instead take frequent small meals. Don't worry, you will be alright, but first rule out heart diseases. Hope I have solved your query. Wish you good health. Thanks."
},
{
"id": 123677,
"tgt": "What herbs to take for swelling in the body due to lymph nodes?",
"src": "Patient: i have swelling in most of my body due to lymph nodes. they are under my armpits, groaing area, my neck, behind my knees, middle of my arms, under my throat, behind my ears. and some many areas of my body, i could feel some of the skin have lumps too. what kind of herbs i can take. thank you. Doctor: Hi, Generalized lymphadenopathy is a very serious condition. No time should be wasted in treating with herbs. Go to a hospital and get attended by qualified doctors. Hope I have answered your query. Let me know if I can assist you further. Regards, Dr. Nirmal Chander Gupta, Orthopaedic Surgeon"
},
{
"id": 112258,
"tgt": "Have chronic lower back pain. Had spinal laminectomy. Took tylenol, aleve. Give me suggestion",
"src": "Patient: Hello Dr. I am a 67 year old woman who has chronic low back pain. I have had slipped discs in my forties and a spinal laminectomy in 2010. I have had physical therapy.injections of cortisteroids and only had temporary relief of pain which is in my left hip now. I have been doing Aqua exercises and more recently been going to a \"Bowen Technique\"therapist. I cannot stand or walk for any length of time without pain. I cannot tolerate strong pain pills and get some relief from Tylenol and Aleve. However I want to be pain free. Do I consult with an Orthopedist Dr. or another Neurologist.(my original neurologist has moved away) Thank you for your opinion. Doctor: Hello, Thanks for your query.As you are suffering from slipped disc and laminectomy already done previously, I suggest you to go for new MRI to see the nerve compression and plan of management acc. to that. Consult a spine surgeon for proper evaluation and management. I do hope that you have found something helpful and I will be glad to answer any further query.Take care"
},
{
"id": 182112,
"tgt": "Suggest treatment for cyst in food pipe",
"src": "Patient: helo sir my father is having sist in food pipe. he has gone through endoscopy where doctor suspects it as a esophagus cancer but we are not seeing any symptoms like problem in swallowing food, pains etc which are forcing us not to rely on the reports of the doctor. kindly guide us what to do? Doctor: HelloIf you are not sure of the doctors opinion, you could take a second opinion from other gastro enterologists for confirmation.If you are not having any symptoms How did you findcoutcabout this problem? Some form of cysts at first are very small which wouldn't cause any difficulties in the initial stages but could spread rapidly if not looked into.Thus if you are not sure about the doctors opinion kindly consult other specialist cause it needs to be checked by some clinical tests only,endoscopy being one.thank you."
},
{
"id": 134704,
"tgt": "How to cure numbness in fingers?",
"src": "Patient: hi,i have been waking up for the last 2 weeks with numb fingers,and only get the feeling back when i open and close them and hold them down,i use power tools in work,ive been talking 225mgeffexor for 1and a half years,and i have done a lot of srenght trainng over the last 10 years,could it be that im just lying on my hand when im asleep?thanks. Doctor: Hi Thanks for your query and Welcome to Healthcare Magic. I am Dr Akshay from Fortis Hospital, New Delhi.If it is a transient numbness which becomes better as you wake up, then it may not be a lot of worry especially if it is one side towards which you are sleeping .It can be due to a transient compression of the nerves, but if it is a persistent issue even while at work and is associated with Neck pain, or other symptoms like weakness, paresthesias ( Pin prick or needle prick sensations ) etc, then it requires evaluation in form of blood tests, clinical examination and maybe if needed a nerve conduction study .Do not hesitate to contact me if you need any further assistance/SOS You can also discuss your case and treatment plans with me in a greater detail in a private consultation.Thanks & RegardsDr Akshay Kumar Saxena"
},
{
"id": 222852,
"tgt": "What are the chances of recurrence of ectopic pregnancy?",
"src": "Patient: hi, i m 27 years female. i have no health problem till now. I had tubal pregnancy and laproscopic surgery was done 1 month back. what is the chance of getting second pregnancy to be tubal? Who is the better doctor to consult in CMC, vellore for the above mentioned problem. Doctor: Hi.I am unsure about which doctor you can consult in CMC now as things have changed a lot since I visited that beautiful institution. But there do exist chances of a tubal pregnancy to be brutally honest, which is why you need to report to your doctor for even the mildest form of lower abdominal pain with a history of a delayed period or unprotected sexual contact. Your doctor will also help you with a few modalities of treatment to help prevent that.Best wishes."
},
{
"id": 224157,
"tgt": "Can i use sprintec as emergency contraceptive?",
"src": "Patient: so, i had unprotected sex two days ago. i can't find a place that sells plan b over the counter. i didn't miss my birth control pill. i'm just really paranoid. i don't want or need a kid right now. i'm just wondering if i could use sprintec as emergency contraception? Doctor: Hallow Dear,Sprintec is a combination of female hormones norgestimate and ethinyl estradiol. It is oral contraceptive pill. Sprintec pack is of 28 pills. First 21 pills contain active pills; i.e. pills containg norgestimate and ethinyl estradiol while the last 7 tablets are non-hormonal reminder pills. They are provided so that women do not loose the habit of taking pills and continues to take the pills. Usually during these reminder pills, the menstruation starts. So complete these reminder pills and then go to next pack. Each pack should be started from day 1 of the pill. Every day one pill is to be taken, preferably at the same timing every day so that the period between 2 pills is maintained by 24 hours. Generally it is a practice to take the pills after dinner. If any pill is missed, it should be taken next morning, in addition to the These pills are regular birth control pills. However, if double dose is taken , it can give you protection from pregnancy. Ideally, Plan B pills like Postinor 2 are good emergency contraceptive pills. They should be taken within 72 hours of unprotected intercourse. After these pills, there may be some bleeding after few days and the menses may be delayed by few days to a week. It is due to the effect of the hormones. I hope this helps you. Dr. Nishikant Shrotri"
},
{
"id": 22857,
"tgt": "What are the symptoms of a heart attack?",
"src": "Patient: Did I have Heart attack? When I broke up with my girlfriend, I felt something is constricting my heart, felt a bit weak, a bit breathless. It lasted for about 5 mins and I suddenly felt Flushing (warm) sensation in my chest. It lasted for a couple of mins. Was it mini heart attack? I'm 26 years old. Doctor: Hi,This is Dr Sameer, cardiologist.Heart attack at your age is pretty rare but not impossible.To rule out any cardiac event, you need certain tests like ECG, 2D Echo, Cardiac Troponins. After doing these test consult your doctor.I'll also be there to help you.RegardsDr Sameer Maheshwari"
},
{
"id": 149617,
"tgt": "Pea size bump on lower spine, painful. Had back spasm, shot with relaxer. What could be this bump?",
"src": "Patient: I have a small pea size bump on my lower spine and it hurts. I had a bad back spasm for 6 weeks and the doc shot me with a relaxer and now the bump on the spine and I believe it is on the 3 vertabri where I have a bulging disc? What it it? I am a Veteran and going to the VA again to have an MRI. Hope they can take care of it? Doctor: Hi and welcome to HCMFrom description it seems you might be suffering from degenerative disc disease & it occurs due to repetitive overload or stress to the disc & it increases the risk of disc herniation .For this condition you have to do stretching & strengthening exercises in which you have to stretch tight muscle & strengthen weakened ones which will alleviate stress on facet joint and disc.I suggest you to get an MRI scan of the area done and consult a neurophysician for proper diagnosis and treatment. Hope this will helps you. Take care."
},
{
"id": 207987,
"tgt": "What causes stress and depression in a 27 year old?",
"src": "Patient: Hello,Iam 27 year old man iam still single i try to find my ideal wife to marry with but i cant succed.I has been more than a year that iam searching,but no satisfactory result.Iam very much under pressure of lust.I feel some kind of pain in my brain iam ofraid not to be in a big health problem.I cant do any thing i masturbate several times a week!!i dont know what to do!!wait for my ideal wife or..... Doctor: HiThis is reactive depression and based on social situations and social stress.Need to motivating yourself.Usually all has this kind of situations in their life but you have to face it and find the way.Need support from friends and family.Social support is also necessary.Some natural ways to treat anxiety and daily stress areexercise meditationdeep breathing and hyperventilationproper sleepstress free activitydo pleasurable activitymusic and hobbiesdiscuss with friendssee all thing with different anglepositive outlookconfidence all this will help you to counter daily stressMedicinesI have treat so many patient with mild dose of benzodiazapine and mild dose of SSRI,but before taking this medicines you have to consult psychiatrist and evaluate your selfGet well soonThank you"
},
{
"id": 9057,
"tgt": "What are the benefits of ozone hair treatment and how does it work ?",
"src": "Patient: what are the benefits of ozone hair treatment and how does it work? Doctor: Hello. Thanks for writing to us. Ozone hair treatment causes elimination of all the infections from the scalp- is a best disinfectant, improves blood circulation- helps in hair growth and enhances the texture of the hairs. It also stops the premature hair fall. It is a safe therapy with no side effects. I hope this information has been both informative and helpful for you. Regards, Dr. Rakhi Tayal drtayalrakhi@gmail.com"
},
{
"id": 47905,
"tgt": "Could passage of kidney stones damage/scar ureters?",
"src": "Patient: I am 48 years old and have had kidney stones since I was 19. I just recently had lithrotripsy to take care of a 14 mm stone. Sometimes I can feel them pass, sometimes I cannot. Does this mean my ureters are damaged or scarred? Also, how many CT scans are too many. I seem to average about 4 a year. Also, I have already had a parathyroidectomy and watch my diet. Any ideas? Doctor: Hello and welcome to HCM.As an Urologist, let me advise you, that as a stone former, you need CT- scan (plain), once a year. ESWL is ideal for kidney stones upto 2.5 cms size. Stones above 1 cm size or causing pain need treatment.The fragments do no harm to ureter. If stent is placed,(optional only),it can injure the ureter. Drink 12-15 glasses liquids daily.You should be more careful after para-thyroidectomy, as small stones need no treatment. You must have done blood calcium,phosphorus, and uric acid.You're welcome to send a copy of your reports, with last CT, here, in my name, for an expert opinion. Get well soon."
},
{
"id": 57654,
"tgt": "Suggest treatment for elevated liver enzymes",
"src": "Patient: I have FAP, recently participated in med study using erlotinib & sulindac. Liver enzymes riased and have not returned to normal. Prior to study - 21 AST & 26 ALT. On meds, up to 48 and 115. Now at 42 & 79. Currently taking omeprazole, lumiday (mood enhancer), zyflamend (herbal/spices) and turmeric. recent test also showed hpyoglycemia (64). normally in the 90 s. thoughts? Doctor: HIThank for asking to HCMThere is no particular treatment available which could normalize the liver enzymes but of course the underlying cause is matters if it has some underlying cause then this should be treated, the clinical manifestations are also important and have great concern than the test value, the test value may go on variation from time to time this could be normal sometime, if it is being done without any obvious reason, hope this information helps you take care."
},
{
"id": 57952,
"tgt": "What is the reason for bloating on right side with numbness after the gall bladder surgery?",
"src": "Patient: hi I am a 55 yr old female I weigh 165lbs.had gall bladder surgery in 2008 I had severe bloating on my right side feeling of numbness inside of stomach area severe elec.shocks,pns needles.numbness,in this area radates into my right side.Ifeel like nerves are jumping in area making my heart feel the same get shortness of breath.Icant touch the areas where my surgery was.Since then I cant lay down,feels like my circlelation gets cut off in my head.the right artery I my neck swells,and I feel the pins,needle numbness in my throat,mouth,causing me to feel like I cant swallow.I didn't have gall stones Some reason my gallbladder stopped working.What cause your gallbladder to just stop working?Im in terriable pain,not layed down since the surergy,PLEASE HELPTHE DRS.DONT KNOW WHY. THANK YOU WWW.WWWW.WW Doctor: Hi, Welcome to Health care magic forum. The stones in the gall bladder is always associated with the gastric hyper acidity, and cause the pain in the stomach and liver area. you got operated for the gall bladder, might not have treated completely for gastric pain. I advise you to consult a gastro enterologist for diagnosis and treatment. I usually prescribe to my patient with such symptoms omeprazole, domperidone, and antacid gel. Take more of green leafy vegetables, pulses, sprouts andProtein rich foods for recovery of the G.I.tract. Wishing for a quick and complete recovery, Thank you."
},
{
"id": 132328,
"tgt": "Does seizure disorder cause fibromyalgia?",
"src": "Patient: Yes please, I think I may have had a seizure yesterday, this was the second time on a month. I have been diagnosed with fibromyalgia and the last test were pointing towards lupus also. This is what happened; it felt like my wars were clogged and my vision kind of went black for a I think just a split second, then was blurry. I felt a vibration in my head and felt like I was in a fog. I was standing and felt weak but did not fall. I think it was just seconds but I m not really sure because it was so foggy in my head. I don t know how to explain it other than foggy! Doctor: HiI dont know what you subjectively describe fogginess and blackness to be a seizure, it has to be diagnosed by EEG and scans by a neurologist.Generally, convulsions, passing out, tremors are seizure symptoms.Acute Fibromyalgia symptoms may produce the symptoms not necessarily a seizure, which is a separate disease, its another matter that both disease process are there, but it needs confirmation.If seizure is diagnosed, there are different drugs needed to treat for long term, but why worry..it may not be...you may be requiring just Fibromyalgia treatment aloneBest wishes"
},
{
"id": 102235,
"tgt": "Is nabuliser the right treatment for asthma?",
"src": "Patient: HImy son is 8 yrs old and he is asthma patient. He is already undergoing the treatment of asthma with daily two puffs of Brocade and one puff of asthaline inhaler. Usually during nights he started coughing and this leads to vomit after that he feels fine and sleep. we had consult doctors for the same but as per them it\u2019s pretty normal and suggest to follow the treatment (inhaler). Now from last couple of weeks his situation is getting worst during night. He is not able to cough properly , its feels like something is blocked at his neck. We rushed him to nearest hospital where they provide him Nabuliser (Brocade and asthaline) and usually this help in this situation, but now we had noticed its not working as good as it was earlier in his case. One Doc also suggest to take : acebrophyllinesyrup, L-montus and omanacortil , 5 ml dose once a day... just wanted to ask, are we on right track of tratment? we had started these medicine from yesterday but the boy is still coughing although that neck choking feeling is not there.thanks Doctor: Hello dear,The symptoms as mentioned in your post can be attributed to acute exacerbation of asthma, secondary to a respiratory tract infection.Management involves:1. A course of antibiotics to eradicate the causative organism.2. Asthalin & Brocade inhaler- provide symptomatic relief by causing broncho-dilation (dilating the smaller airway passages, relieving the obstruction & increasing airflow to lungs)Can be used whenever he is having an acute attack.3. Montelukast preparations- used as a maintenance therapy to relieve symptoms of asthma.4. Antihistamine & anti tussive preparations for symptomatic relief from cough.5. Maintain adequate hydration & a healthy balance diet.6. Also make sure that he is well protected from cold, dust & other allergens.If symptoms still persist, kindly consult a Pediatrician/ Pulmonologist for proper clinical evaluation.Wishing him a good health.Take care."
},
{
"id": 115061,
"tgt": "What causes high levels of THC?",
"src": "Patient: can severe trauma alter blood thc levels i am male 80kg 19 years old had car accident ruptured spleen puntured lung multiple rib and spinal external process fractures recreational cannabis user but not prior to accident had thc level of 13 nanograms in blood 105 minutes after accident Doctor: Hi,Thanks for asking.Based on your query, my opinion is as follows.1. No trauma cannot increase THC levels.2. Recreational use of cannabis - can be positive upto 5-6 days.3. Long term use get stored in fat and be positive for upto 100 days. Unless you are looking for false positives, which could be due to medications given post accident or before accident, it will due to cannabis. Hope it helps.Any further queries, happy to help again."
},
{
"id": 76675,
"tgt": "Suggest treatment for shortness of breath",
"src": "Patient: Hello I'm 36 yr, female and I've been having shortness of breath for about two weeks i went to the ER they did EKG, Cat scan and Qvac test and x-ray and they only came up with I have asthma I never had any breathing problem before and the inhaler isn't helping I very worried and I do have High blood pressure and i'm over weight 5'8 and 300+ Doctor: Hi thanks for contacting HCM...Obesity can lead to apnea and dyspnea....So loose your weight by regular exercise...Low fat diet taken ....Second here you are mentioning you have diagnosed with asthma....For it's confirmation spirometry with pulmonary function test useful....If it is not controlled by inhaler during attack then nebulization might given with steroid for reducing inflammatory bronchi constriction...If asthma diagnosed and repeated attack then maintainance inhaler taken with formetrol and low dose steroid....One tsp garlic juice with honey good herbal remedy....Take care....Dr.Parth"
},
{
"id": 204371,
"tgt": "What causes excessive stress while at work?",
"src": "Patient: I have very long, stressful days at work as an RN discharge planner. I go none stop, at full throttle for 8 hours. I notice that on my first day off I am wiped out and my body feel depleted. Is this a type of effect from prolonged periods of adrenaline rush? Doctor: Hello and Welcome to \u2018Ask A Doctor\u2019 service. I have reviewed your query and here is my advice. Yes. Too much stress at work can make you tired. Whatever might be the amount of work, try to take a short break of 2-3mins at least every 20-25mins. You can just go for a short walk, have a glass of water and return back to work. This gives your body a chance to replenish energy stores required by the body. Feel free to ask further. Thank you. Regards, Dr Vikas Achampet MD Psychiatry"
},
{
"id": 27540,
"tgt": "How long will the graft lasts after cardiac surgery?",
"src": "Patient: Eight years ago,I had a procedure which caused a nick in my aorta. A week later, I had another cardiac cath procedure and a dissection occurred and I had bypass surgery. I am curious as to how long I can expect this graft to last. I have no known heart disease at this time. Doctor: Hi,In case you follow healthy lifestyle, take your medication regularly, there are good chances that grafts will last for years, there are cases, with normal functioning grafts for more than 20 years.Wishing you good healthIn case of further questions don't hesitate to askRegards,"
},
{
"id": 195076,
"tgt": "Is frequent watery fluid penis discharge a concern?",
"src": "Patient: Sir, When ever I am getting a little tempting by seeing any porn or a beautiful lady I was getting sperm released without any masturbating. Even some times during normal working time also I was getting sperm out. Is this any problem kindly please suggest me some medicines to cure this problem. Doctor: Hi, Usually, in orgasm, there's precum which will be released and it does not contain much sperm, it is the common phenomenon. Other possibilities like medications, such as antidepressants, mood stabilizers, and some hormone treatments may also cause semen leakage. Avoid watching porn, drink plenty of liquids, avoid alcohol consumption and smoking. If symptoms not improved please consult your physician, he will examine and treat you accordingly. Hope I have answered your query. Let me know if I can assist you further."
},
{
"id": 133739,
"tgt": "Does a toenail infection spread ?",
"src": "Patient: My 7 month old daughter has an infected toenail and the infection came out today. I am going to soak it 3 times a day and put antibiotic cream on it. At what point would I need to take her to the doctor. I freak out a lot bout ingrown toenails because there was a cop in my town who died from and infected toenail because the infection spread. What would be signs of it spreading. Doctor: Do not worry. Death is not common after toe nail. But still as your daughter is only 6 months old, i would recommend you to get show it to doctor as soon as possible. putting antibiotic solution with infection still inside will not going to work rather can create resistant among bacteria for drugs.Thanks"
},
{
"id": 223278,
"tgt": "Suggest remedy to avoid pregnancy",
"src": "Patient: i didn t use my nuvaring for the required dosage. I took it out, fridged it and 1 week later put it back in. 1 week after that I had unprotected sex. 2 days after that, took it out again for 1 week. im scared to death that I have gotten myself pregnant. what should I do?? Doctor: Hello,Thanks for sharing your health concern with us. In the current scenario, your cycle is no longer protected. I would ask you to go for a home pregnancy test a week after your expected period if you miss it. Since you have used the contraceptive irregularly, you can expect abnormal menstrual pattern. If you wish to have an earlier detection, please go for the estimation of the serum beta-hCG titres ten days after the intercourse since ovulation, if at all has occurred, cannot be predicted here. See a specialist for further help. Take care."
},
{
"id": 100320,
"tgt": "How to get rid of food allergy?",
"src": "Patient: hi, i am 49 years old, and i am suffering some food allergy past 2 to 3 years and i am not sure whether it is food or blood allergy, some red rashes, itching and red boils occurs on my body, skin, eyes and feel vomiting, acidic. Please tell me what would be the course, tenure and fees for this type of sickness and also what exactly this illness is? Doctor: HelloThank You for contacting HCM.Some people are allergic to particular food. So when they take such food then allergic reaction occurs causing rash, swelling, etc. sometimes it goes away with the passage of time and some times it persists for ever. In that case you simply need to avoid that particular food.i would suggest you to take montelukast one daily for one month. Research has shown that it decreases the allergic symptoms considerably.I would also suggest you to under go allergy testing at allergy clinic as you have chronic allergy problem. It will tell that you are allergic to what specific thing. The results will help an allergist to prescribe you immunotherapy to that specific allergen and it will improve the problem.Hope this answers your question. If you have additional questions or follow up questions then please do not hesitate in writing to us. Wishing you good health."
},
{
"id": 9921,
"tgt": "Suggest treatment for hair fall",
"src": "Patient: hello sir,my name is dileep. from karnataka.i am 21 year old.i hv seviour hair falling from wen i was 13 year old.i don't have much hair now.i consulted nearly 10 doctors.but its of no use.i request you please solve my problem,my hair falling is not heridity, i didnt use soap for my head.now hair is falling too much and luking bald.please help me Doctor: Hi Dear, Understanding your concern. As per your query you have symptoms of hair fall which could be due to fungal infection, stress, hormonal fluctuations, improper hygiene and due to nutritional deficiency. Need not to worry. I would suggest you to maintain proper hygiene of your scalp. Consult dermatologist for proper examination and go for required testing done as without complete observation it's not safe to take treatment. Doctor may also prescribe supplements containing biotin, Acetylcysteine, copper as active ingredients. You can use finasteride preparations as well. Hope your concern has been resolved.Best Wishes,Dr. Harry Maheshwari"
},
{
"id": 47306,
"tgt": "How can a hard and painless swelling on the testis be treated?",
"src": "Patient: I did my kidney transplant a month back. I hd leg edema and scrtal swelling from day one to two weeks post op day .In the last one week i have noticed a painless,non hot non tender sweelling of the rt testis. i have no problem in micturation .What will be the cause and how can i get treated?I am 50 yrs old, 1.67cm in hight. dibetic for the last 15 yrs on insulin,hypertensive for 4 years controlled after transplant. I am taking cyclosporin,cellcept,predinsolone Doctor: HelloThanks for query .swelling of the testicle that you have after renal transplant surgery is due to retrograde secondary infection of the Testis (Orchitis) due to catheterization,Get urine culture done to trace out causative organisms and take appropriate antibiotics as per culture report .You need to continue antibiotics and anti inflammatory medications like Serropeptidase twice daily for 2-3 weeks .Ensure to drink more water .This will get resolved soon .Dr.Patil."
},
{
"id": 57786,
"tgt": "Cause for itching, sweating, skin rubbing off inner thighs after taking treatment for gall bladder issues?",
"src": "Patient: I was in the hospital last week gallbladder issues. And I was itching all over my body. I had them to change the sheets...then my inner thighs started sweating and itching...it s been a week now and my thighs consistently itching, skin rubbing off and bumped up severely! Doctor: NO RELEVANT INFORMATION ON LIVER FUNCTION TESTS HAVE BEEN GIVEN BUT MUST HAVE BEEN DONE IN THE HOSPITAL YOU WERE ADBITTED. YOUE SYMPTOMS STRONGLY SUGGEST OBSTRUCTIVE JAUNDICE IN WHICH SERUM BILIRUBIN WILL BE RAISED AND CAUSE OF OBSTRUCTIVE JAUNDICE NEED TO BE FOUND."
},
{
"id": 45188,
"tgt": "Body CT scan during menses",
"src": "Patient: can I do whole body ct scan during menses Doctor: do u mean pelvic scan , if its not emergency u can do after menses ,other body parts can b get scan done even in menses."
},
{
"id": 219407,
"tgt": "Can thyroid levels during pregnancy affect the fetus?",
"src": "Patient: hello sir, im in15th week of pregnancy, just now we took test and came to know that my thyroid level is 7, my baby brain got affected already because of this? i started taking tablets from today on ly. pls give me a reply, that will help for me. im in so confusion. Doctor: Hallow Dear,Confusion also can happen due to low function of Thyroid. Thyroid deficiency definitely affects the development of the baby. Hypothyroidism during pregnancy is associated with adverse pregnancy outcome which includes:\u2022 Increased incidence of miscarriages, recurrent pregnancy loss, preeclampsia, anaemia, Diabetes, placental separation, heavy bleeding after delivery\u2022 Foetal growth restriction, foetal distress during labour, foetal death, preterm birth\u2022 Cognitive, neurological and developmental impairment in the baby.Guidelines recommend screening of all pregnant women at risk for hypothyroidism. Please get your complete Thyroid Function; i.e. T3, T4 and TSH estimated.The normal ranges for First trimester: 0.1 to 2.5 mIU/L; Second trimester 0.2 to 3 mIU/L; Third trimester: 0.3 to 3 mIU/LLevothyroxine is the drug of choice for treating hypothyroidism. Starting dose can be 25 mcg /day while for TSH level > 10 mIU/l it should be 50 mcg/day. Thyroxine should be taken early morning on empty stomach. L-Thyroxin ingestion and the ingestion of iron supplements, calcium supplements and soy-based food should be separated by at least 4 hours.TSH should be repeated after 4-6 weeks of starting the treatment to see the response. Target TSH levels should be kept below 2.5 in first trimester and below 3 during second and third trimesters. If target TSH level is not achieved, the dose should be increased by 25 mcg/day. Dose should be adjusted according to TSH levels. At any time TSH below 0.1 should be avoided by decreasing the thyroxine dose by 25 mcg from the current dose.After delivery, most patients need to decrease thyroxine dosage received during pregnancy. It is very important to continue monitoring thyroid function tests for at least 6 months after delivery.With this treatment of Thyroid you can prevent the further damage of the foetal brain. I hope this helps you. If you feel so, you may vote for 'Helpful' answer."
},
{
"id": 197360,
"tgt": "Are the semen analysis reports normal?",
"src": "Patient: We have been married for the past two years, and have no children as yet. My Husband has gone through a Semen Analysis the result are as under : Color - Greyish Viscosity - Gel-like PH - Alkaline Liquefaction time : 30 Min Sperm Count : 45 Milliom/ml Motility Total Motile : 50 % Actively Motile : 30 % Sluggishly Motile : 20 % Morphology Normal Spermatozoa : 50 % Abnormal Heads : 30 % Abnormal Middle Pieces & Tails : 20% Total Abnormality : 50%Cytology Pus Cells: 6-8 / HPF R.B.C : 1-2/HPF Please advice on the result...........is everything is okay. Regards Navika - YYYY@YYYY Doctor: Hi ....According to WHO guideline normally semen should have .....Motility 40% Active motile 32%Morphology ideally >30% , strictly >4%Count minimum 15 million per ml.So your semen report is absolutely normal....The only positive finding is presence of few pus cells...Clinical correlation necessary for that ...If needed antibiotic can be given to clear infection...You can able to reproduction according to report....So don't worry.Take care"
},
{
"id": 95782,
"tgt": "Stomach and abdominal Pains after eating steak",
"src": "Patient: I eat meat regularly, but rarely eat steak. The other day I had steak for dinner and later that night it felt like there was a knot in my stomach, then finally had a large bowel movement and felt like a million bucks. The next day I had the leftovers and later that night the cramping came back and has lasted 2 days so far. I keep feeling like I have to have another large bowel movement but am only able to get rid of a few pebble-sized feces. My wife who had the same dinner is fine. What can I do to alleviate this? Doctor: try digestive enzymes for digesting red meat. Your body might not process or digest rich meats well. I have the same problem and if I eat a real good lean steak I get bad gas and have to \"purge\". sometimes the same night"
},
{
"id": 85346,
"tgt": "How to relieve redness due to exposure after using tretinoin cream?",
"src": "Patient: To treat my acne, late last year I applied Tretinoin cream to my face. Soon after applying the cream I went outside and exposed my face to the sunlight with no protection. Its October 2012 now and my face is very red/sensitive/irritated. Could you guide me in the right direction to possibly fix this problem? Thank you. Doctor: Hello, This is seven years later. The problem isn't relevant anymore. Well, it is a sensitizer to sun so, you'll want to avoid using it and having sun exposure at the same time. Hope I have answered your query. Let me know if I can assist you further. Take care Regards, Dr Matt Wachsman, Addiction Medicine Specialist"
},
{
"id": 33154,
"tgt": "What causes tiredness and chills?",
"src": "Patient: Hi, I feel really tired and get chills. This happens every 2/3 days. I'm fine one minute and then then the next minute I'm absolutely shattered. I'm 43 and a female. Also I keep forgetting things and say the wrong things eg 'turn the volume down on the gas'! Recently had shingles in my head on the right side but the memory loss and tiredness/flu like symtoms were before I had shingles Doctor: HelloSince you are 43 years old and feels tiredness, chills , shattered , forgetfulness .All these symptoms are off and on , may be due to these reasons :1 Perimenopausal syndrome , just before onset of menopause the levels of estrogen and other hormones starts decreasing , so this may be the main reason. Diagnosis can be confirmed by blood examination for estrogen , androgen , progesterone . 2 Anxiety or tension ( stress ) is another most common reason of such symptoms .3 Thyroid hormone imbalance . Diagnosis can be confirmed by blood examination for T3, T4 , TSH . As I have noted due to imbalance many patients feel such symptoms .So deal as mentioned above , consult a primary care physician and get his opinion ."
},
{
"id": 75686,
"tgt": "What causes cough?",
"src": "Patient: my chest hurts in the middle and my lower back hurts when i breathe i have had a dry hacking cough for 3 weeks went to er and they gave me tussionex which didnt help i waited five days and went to my dr.he gave me allergy &antibiotic injections also clairnex and antbiotic pills called avelox.im still coughing and now vomiting w/diarreah what could this be.i am asthmatic and i have bronchitis i am 40 years old and i am overweight i am 5'3 and 290lbs Doctor: Thanks for your question on Healthcare Magic. I can understand your concern. Since you are asthmatic, your continuous coughing is mostly due to acute exacerbation of asthma. And best treatment for this is inhaled treatment and you are not taking it so you are not improving. So better to consult pulmonologist and get done 1. Clinical examination of respiratory system 2. PFT (pulmonary function test). PFT will not only diagnose bronchitis but it will also tell you about severity of the disease and treatment is based on severity only. You may need inhaled bronchodilator (formoterol or salmeterol) and inhaled corticosteroids (ICS) (budesonide or fluticasone). Don't worry, you will be alright with all these. Hope I have solved your query. I will be happy to help you further. Wish you good health. Thanks."
},
{
"id": 143467,
"tgt": "Proper medication for seizure?",
"src": "Patient: Hello Doc,I have a query regarding myoclonic epilepsy which was diagnosed to me at the age of 26. I am 31 now, weight 95Kg, height 183cms good built. When this was first reported, the doc advised Valparin Chrono 200mg (4 tab/day) for 3 yrs,EEG and MRI normal. I took it for 6 months and discontinued, then again after 1 yr of first occurrence I had the seizure again. This time I took it seriously and started valparin course again and continued for 3 yrs. After 3 yrs of regular medicine (Sep 2012), I had the EEG done and it came normal so doc advised me to discontinue the medicine. However the issue is the problem has resurfaced in Sep 2013. This time I am in Bangalore and here the docs advised me to take Levepsy 500 twice a day. I just had another seizure out of the blue in Dec 2013 again even after taking the medicine. Need a second opinion from you that if any other scan or something is required. This time after few days have gone by I still feel a soft pain towards the left side of my body from head to toe. May be the after effects of the seizure, I am not sure. Kidnly advise if I need to undergo any other tests or scan and if this is curable through proper medicine course? Thank you! Doctor: unfortunately this type of seizure is longlasting but fortunately its easy to control on valparin or levepsy.since you were control on valparin I recommend you to continue on it.levepsy is a very good drug for seizure control but valparin is better for this type.since you got multiple attacks after stopping the drug its better to stay on this medication for your life"
},
{
"id": 163386,
"tgt": "How can a red rash on the face be treated?",
"src": "Patient: Hi..My one month old baby started having these tiny red rashes filled with white pus all over her face since she was 2 weeks old..Apparently i learnt its called baby acne..It comes and dissapper but its getting worse now and has spread to other arears beneath her chin and around her cheeks and ears..They look soo tiny and very bsd..I dont use any cream on her face, i just wash and wipe with warm water almost a month now..Pls what moisturizer or cream is good or recommended to make these rashes go away for good..Thank you..xx A Worried mom Doctor: Hello,In my opinion, you should let them go from themselves. There is nothing to worry about, as it is common for babies.Washing with warm water helps. These baby acne spread in other areas due to warm temperature.Hope I have answered your query. Let me know if I can assist you further.Regards,Dr. Elona Dashi"
},
{
"id": 224408,
"tgt": "Does Fluoxetine affect Implanon implant?",
"src": "Patient: Hi, I've been using the Implanon Implant for 5 months and everything has been working fine. I got perscribed \"Fluoxetine\" and been taking it for the past 2 weeks. My period is a few days late, I maybe being paranoid but does Fluoxetine affect the Implanon Implant? Doctor: Hi dear and thanks for your query.There were no interactions found in our database between Implanon and Fluxetine.However, this does not necessarily mean no interactions exist.Hence a pregnancy test done to exclude pregnancy ,if you haven't had any other delay periods .All the best"
},
{
"id": 118558,
"tgt": "Low WBC count, headaches, joint pain, nausea. What could be wrong?",
"src": "Patient: Hello,A friend of mine is having difficulty getting diagnosed. She is currently in the hospital with symptoms of low white blood cell count, headaches, joint pain and nausea. She recently visited Mozambique and started to show symptoms about 3 days after returning. Do you have any possibilities of what could be wrong? Doctor: Hello and welcome to HCM,Low white blood cell counts may be seen in infections especially bacterial infections.A high load of infection consumes the white blood cells rapidly.Headache, joint pains, and nausea can be a manifestation of infection.You need to get investigations done to look for infections.A blood culture can be done to isolate the micro-organism causing the infection.Consult your treating doctor for prescription of further investigations.Thanks and take careDr Shailja P Wahal"
},
{
"id": 145435,
"tgt": "Suggest treatment for popping up and down of spine",
"src": "Patient: I ve been experiencing multiple popping up and down my spine with all activities. It s not especially painful, but happens all day long the past 2 months. I do have low back issues and had a cervical spine C4-5,C5-6 fusion 6 months ago. This new symptom is becoming a concern. What is going on with my spine? I m a active 55yr. Old female. Doctor: Hi,Thanks for writing in.Though neck surgeries work in most patient, a few of them might have post surgical reasons leading to a pop sound in the neck. This is especially more when doing activities that involve movements of the neck. It is likely a change that has developed during the healing process. There are many small ligaments and joints in the region of surgery and after surgery the area heals and might cause popping sounds. As pain is not there, it is not due to pinching of nerve roots or failure of surgery. Many people experience these popping sounds and with time it gets lesser. Please avoid doing heavy physical activities involving your spine. If at any time there is development of pain then please consult your doctor and you might require a MRI scan cervical spine after clinical examination. Please do not worry."
},
{
"id": 31799,
"tgt": "How to treat cough?",
"src": "Patient: I have developed a very violent cough which occurs several times a day and causes me to choke and vomit in order to clear the persistent irritation. No drinks, sweets or cough mixture can stop it. It only stops when I choke or vomit. I have experienced this several times this year and have been treated by antibiotics, but it keeps reoccurring. Doctor: HI, I understand your concern. repeated cough suggests possibility of- - Allergic origin- asthmatic bronchitis / toncilar involvement., - the infecting organism is resistant to the antibiotics given, - Diabetes with uncontrolled blood sugar To manage you need few investigations like CBC/ sputum culture & sensitivity/ X ray chest/ blood glucose ( fasting & post meal ).The treatment planned by a chest specialist,in light of these results would help you. A healthy life style with a balanced diet/ regular respiratory exercises & physical exercises/avoiding exposure to allergens is essential along with the treatment advised. Thanks."
},
{
"id": 67718,
"tgt": "What causes lump in the middle lower jaw?",
"src": "Patient: I have a lump in the middle lower jaw behind my teeth close to the chin. It is very tender to the touch. The dentist has given me Penicillin and it has helped a little but remains swollen and sore. It is hard to even eat because when I chew it irritates the bump. I am almost finished with the Penicillin and am scheduled to see the dentist Monday. The bump was larger but has shrunk. I have had a bump there for a long time but it has never been painful. Suggestions? Doctor: Hi,It seems that you might be having infection in the root of tooth giving this trouble.Continue with antibiotic.Take NSAID anti -inflammatory medicine to get recovery fast.Ok and take care."
},
{
"id": 164078,
"tgt": "Can my daughter travel as she has stomach upset and fever?",
"src": "Patient: My 6 year old had diarrheoa yesterday with some vomiting no fever, then fever started last night and she still has mild fever today. She s been in bed all day, sleeping on and off, taking some water and diarolyte. She feels uncomfortable when sitting up because her belly is still a bit sensitive. We re booked for a long haul flight home tonight and I m so worried! Should I cancel the flight or continue with plans and hope for the best? Doctor: hello. in best interest for your child she should not travel till the time she is better. for vomiting you can give her syp ondansetran. continue giving plenty fluids."
},
{
"id": 163363,
"tgt": "What is the treatment for bicuspid aortic valve in a child?",
"src": "Patient: my son is 20 mnths old. he was diagnosed with bicuspid aortic valve when he was born due to heart murmur2/6 sem. no diastolic murmur. we see the cardiologist 1/year. however he does not have the heart murmur anymore. we saw the cardiologist last week and he said that the left ventricle, tricuspid valve, and mitral valve are normal. the valves are fused togter since birth, and not all the fused valves cause problems. and based on his expericen he doesnt think he will face any problem before 40 s and might not even face anything at that time. when i asked him if the aortic root is dilated, he said that they are a little bit dilated comparing to someone who does not have BAV. and he said not to worry. his valve s are goog functioning valves, and they are at the good end of it. whats your openinion regarding my sons condition. is all BAV cases have some degree of aorta dilation, my son has no other medical condiotn, how fast the aorta become dilated. he has no stenosis or regneration in the valves. thank you, Doctor: Hello,If doctors are satisfied regarding aortic valve functioning, then you also no need to worry. Plan for the Echo, ask your children to avoid heavy games and exercises.Hope I have answered your query. Let me know if I can assist you further. Regards,Dr. Hina Javed"
},
{
"id": 102952,
"tgt": "Asthma attack, tight feeling in chest, feeling more tight after drinking soda",
"src": "Patient: Ok, so I have been drinking soda for the past 15 years and I have never had this feeling and 4 days ago I was with my friend and he started smoking weed I on the other hand did not partake on smoking according to my asthma and for the past 3 days I have been having Asthma attacks and today I felt fine and now that Ive had my first soda in 3 days I've been feeling like my chest is getting tight after I drink it why do I feel like its getting tighter after I drink Doctor: any thing can start reacting with body proteins at any age suddenlyit can be effect of smoking by your frien 3 days back or soda started to rectuse anti allergic tab montelucast bdshort course of steroids and bronchodilator sypconsult allergy specialist who can find thecause and youcan treat accordingly"
},
{
"id": 29596,
"tgt": "Could Hepatitis B relapse despite taking Interferon and Ribavirin?",
"src": "Patient: I had hep c and b I took the treatment interferon and ribovaron I was a success story back in 2001! Now I have cellulitis and found hep b active but not c, I have to c. Doctor next week! Could this be severe I was lead to believe I would not have future problems Doctor: Hi,Welcome to health care magic,I understand your anxiety regarding recurrence of Hepatitis B even after treatment. On an average, 90% cases recover completely, 5%-10% go for chronic carrier stage, of these chronic carriers few may progress as chronic aggressive hepatitis with repeated attacks of jaundice. I suggest you get in touch with your doctor and get investigations to rule out chronic stage of hepatitis B. You have to get tested for HBeAg, AntiHBsAg. PCR also be done to detect the exact viral load.Wishing you all the best. Thanks"
},
{
"id": 135879,
"tgt": "Suggest treatment for severe pelvic discomfort",
"src": "Patient: I have moderate to severe pelvic discomfort. I am seventy years old and in reasonably good health till the past few months. I did do some hiking in Arizona in March and had a fall although not a dramatic one. However....the discomfort bordering on real pain from time to time continues. Should I consider ultra sound or MRI procedures? I m thinking it is muscular......or could it be something more serious? Doctor: Your symptoms have appeared after a fall during hiking.I think you should ake it seriously & get i investigated.Get a ultra sound examination done. It may show a hematoma in the pelvis.MRI will be more useful.kindly inform me about the final diagnosis."
},
{
"id": 194297,
"tgt": "Do the masturbation reduces the sperm count?",
"src": "Patient: Hell doc.how r u.m just want to ask to u abt hand practise.how i rid from it.my learning power reduces,my knees become weak.i do not feel any power in my body...i do this 2 times a day for 4 years and my age is 23 and nw my sperms also reduces after this process.is this any problem wid me.do i need speasialist...and is there a problem married life u know childerns.pls reply me Doctor: Hi, Masturbation is normal, it\u2019s just another way of having sex without a partner. It\u2019s a good practice, helps to calm your mind and release happy and stress bursting enzymes. Having 2 to 3 times a week is not an issue, more than that is also not an issue, but needs adequate nutrition and exercise which will be difficult to fulfil. Ways to reduce Masturbation 1. Sex education 2. Physical exercises 3. Sleep adjustment 4. Quality of sleep 5. Healthy and balanced diet 6. Programming for leisure time 7. Development of spirituality 8. Taking a cold bath If you still find it difficult need to meet a psychiatrist in person. Hope I have answered your query. Let me know if I can assist you further. Regards, Dr. S.R.Raveendran, Sexologist"
},
{
"id": 7413,
"tgt": "Have acne problem. Not cured by clinda c gel and diprogenta cream. Safe remedy?",
"src": "Patient: i have never had any major pimple or acne problem till now. but some 2 months back i started getting very small rash like stuff on my cheeks. after some 3 weeks they kind of receeded. however again in a weeks time they resurfaced. this time some of them started converting into full blown pimples . i visited my dermatologist and he prescribed me clindac gel and dipgenta cream. i have been using it for the last one month and it hasnt given me any results yet. my small rash like things havent yet disappeared and few of them keep turning into pimples leaving behind dark scars..could you please help me with a fast cure and a safe remedy?? Doctor: Hello I would recommend that you stop diprogenta cream. That is giving rise to pimples. You can treat with pimples with benzoyl peroxide 2.5% gel once in the day and clindamycin gel once in the day. Also use a facewash regularly. Do not pick on the acne or it could leave dark marks or scars. Take care"
},
{
"id": 142972,
"tgt": "Suggest treatment sensory processing disorder in children",
"src": "Patient: My 6-year-old son is normally eager to please, outgoing, happy, very bright and something of a perfectionist - most of the time. Other times, he wanders around with a vague smile or spins around, often swinging things and running into people or hitting them with no apparent recollection afterward. When he s in this state, he has to be physically redirected because he seems oblivious to verbal commands. Is it possible he s having seizures? He s also been diagnosed with sensory processing disorder - mostly over-sensitivity to touch and auditory stimuli. The behavior I ve described above has been ascribed to the sensory issues, but while O.T. has helped with his other sensory symptoms, the spinning and zoning out persist unabated. Doctor: Hi, Welcome to HealthCareMagic.com I am Dr.J.Mariano Anto Bruno Mascarenhas. I have gone through your query with diligence and would like you to know that I am here to help you.This condition required detailed in person assessment using Clinical Evaluation Scales and this cannot be manged over internet. Please consult a Clinical Psychologist and Psychiatrist in your placeHope you found the answer helpful.If you need any clarification / have doubts / have additional questions / have follow up questions, then please do not hesitate in asking again. I will be happy to answer your questions. In the future, for continuity of care, I encourage you to contact me directly in HealthCareMagic at http://bit.ly/askdrbruno Best Wishes for Speedy Recovery Let me know if I can assist you further.Take care."
},
{
"id": 185389,
"tgt": "What causes the condition of enlarged tongue?",
"src": "Patient: Age: 19Height: 5ft7Weight: 10 stoneNo previous medical conditions worth mentioning.I have had an enlarged tongue for about a year now and I don't know why. It may have something to do with drug use but I can't find anything online that links the two. Doctor: Hello,Your medical and dental history would be helpful in determining if a drug has caused this condition. There are many reasons and it can signal a systemic change occuring to your body - hormonal, endocrine disorder, organ disfunction or damage, allergy, diet change, infection, tumor or cancer. I would suggest a blood test and physical evaluation as well as an updated dental exam. Clinical evaluation of the tongue would be helpful. Other conditions such as texture, color, coating on tongue may give some direction and help with a diagnosis. Trauma or habits can contribute. Does anything seem to affect the tongue? Any pain or sore spots? Thank you for your inquiry.I would be glad to expand my answer with additional details. I am glad to offer as much assistance as possible without seeing your condition."
},
{
"id": 14054,
"tgt": "Suggest remedy for a scar on the face",
"src": "Patient: hey sir, i have a marks which has been cut by falling from stairs when i was to small. now i am 18 years old & the marks also getting bigger on my face. everyone told me to get medicine for remove that marks which make me ugly then i tried so many medicine but it still same so i want to know from you sir is that possible to remove the marks? if it is possible then plz i beg you sir give me the idea for remove this marks. plz sir reply me as soon as possible i am waiting for your reply....hope you will give me good response for my question....plz sir Doctor: Hello and Welcome to \u2018Ask A Doctor\u2019 service. I have reviewed your query and here is my advice.For scars, you can use Vaniza gel twice daily on the scar. If you do not get improvement after 1 months, do consult your Dermatologist and discuss for cosmetic treatment options.Hope I have answered your query. Let me know if I can assist you further."
},
{
"id": 22011,
"tgt": "Is it okay to have pulse rate of 90-103?",
"src": "Patient: I am a 53 year old lady -recently had neutropenic sepsis. Since then my normally low resting pulse rate has increased to 90-103 beats per minute. Should I be concerned?-I have a low bp[normalfor me ] am slim and usually quite fit and very active. -thank s Doctor: it's pretty normal ma'am. Need not to think too much for this sometimes with exertion sepsis fever can cause high pulse rate"
},
{
"id": 225278,
"tgt": "White brownish discharge after taking after pill for unprotected sex. Could get pregnant ?",
"src": "Patient: Hi! My girlfriend ended her period november 30th and we had unprotected sex on december 9th. Late evening on december 11th, she noticed a brown/reddish discharge on her underwear. The next morning she took the morning after pill before the 72 hours. Later on that night, she had a white/brownish discharge that looked like moucus. Could she be pregnant? Doctor: Hi THERE THANKS FOR POSTING YOUR QUERY AT HCMSee its too early to predict about the pregnancy of the patient . That can only be done if has one missed periodRegarding the discharge it could be inter menstrual spotting which is known as mittelschmertz or it could be due to a sexually transmitted disease that the infection has occurred . I SUGGEST THAT YOU CONSULT A GYNOECOLOGIST Hope you are satisfied take care"
},
{
"id": 79680,
"tgt": "Are hives a normal symptom of bronchitis or pneumonia?",
"src": "Patient: Are hives a normal symptom of bronchitis or pneumonia? My 3 year old daughter has bronchitis in possible early stages of pneumonia and has broken out in hives over 4 times now. She's never had them before and got them before she ever took any medicine. Doctor: Hives is an allergic skin reaction causing localized redness, swelling, and itching.A wide variety of substances may cause hives in sensitive people, including foods, drugs, and insect bites or stings. Common culprits include:nuts, especially peanuts, walnuts, eggswheatmilkstrawberriesfood additives and preservativespenicillin or other antibioticstreatment of hives is with antigistamines and sometimes steroidsin severe pneumonia urticaria can happenthanks/ regards feel free to ask more questions may god bless your child wiith good health"
},
{
"id": 95897,
"tgt": "My Mother is having severe liver problem- please suggest treatment",
"src": "Patient: Hi My Mother is having severe liver problem. Water is to accumulate in stomache and hemoglobin percentage is dropped to 7 %. She has this problem since many years. Already consulted physian, he told liver was damaged and she need to be under medication for entire life. And should not eat any hard food. After taking medicine she used to have frequent urination and weakness feeling. After 3 months of medication hemoglobin percentage improved to 8%. Please suggest better treatment for my mother Doctor: you should consult some liver specialist,if possible he can treat with medicine & if liver is completely damaged irreversibly,she can undergo liver transplantation now a days at any good liver transplant center"
},
{
"id": 112741,
"tgt": "Hip pain, painful buttocks and legs, spasms, herniated discs. Have hypothyroidism, calcium and vitamin deficiency. What can I do?",
"src": "Patient: My hips hurt soo bad, buttocks, legs. Sometimes in my lower legs ankles/feet serious spasms my feet curl in. My hip pain seem to be setting off pain like a pin ball machine. What type of dr should I see. I have two herniated discs and other issues w/ back for years other. Had it under control. Until pain in my hips. I do have hypothyroidism (removed). I have vit D,B and calcium def. and taking supplements. Doctor: You should see the orthopedician and neurosurgeon. Surgical management may be needed to decompress. The radiating pain in the leg and buttocks is due to disc prolapse. Vitamin and calcium supplements are of least use in this condition."
},
{
"id": 203094,
"tgt": "Is it healthy to do masturbation?",
"src": "Patient: i am 20 years old i can do the handjob or not.after complete this i am feeling so light and i feel my energy is fully lost and my stamina level is dereased fully is it any solution. here after i do it or not and is it good for health or not. please reply me Doctor: HiDON'T worry you can do mastubation.its a normal and healthly excerise which every men do which will not lead to loose your stamina. Its totaly healthy.thank you"
},
{
"id": 165484,
"tgt": "Could drinking milk cause headaches?",
"src": "Patient: After drinking a bottle of warm milk, my daughter often says she has a headache and gets quite upset until i give her some medicine. I have asked her to point to where it hurts (she is 2 1/2) and she points to above both eyebrows and the centre of her forehead - is there any reason the milk could be giving her a headache? Doctor: Hello,Milk does not cause headache. Some children are allergic to cow's milk and in that case they have symptoms like itching, bloating, running nose, etc.But it does not appear to be a case here. Probably she doesn't like the taste of milk that's why she might be complaining of headache so as get rid of milk.You add some natural flavors to milk or make milkshakes, etc., so that she likes the taste of milk.Hope I have answered your query. Let me know if I can assist you further.Regards,Dr. Khan Shoeb Mohammad Sher Mohammad"
},
{
"id": 130152,
"tgt": "How to relieve muscle spasm near pelvic area?",
"src": "Patient: I'm 35 weeks and the pain on my left side is severe it's around my pelvic area. It feels like muscle spasm, stubbing pain.. it's hard for me to walk, sit down, specially to get up.. Cant even lift my legs up without feeling the stubbing sensation. i don't know what to do :( Doctor: Hi I am Dr Gopal Goel Orthopedic Surgeon. Pain during last weeks of pregnancy is generally due to stress on muscles and joints, which in your case might be affecting Sacro iliac joint on left side. I would not advise any medicine for this at present , but certain stretching exercises will help you , which will be demonstrated by any physiotherapist.Also you can apply hot packs at the site of maximum pain. I do hope that this line of therapy does benefit you."
},
{
"id": 184693,
"tgt": "Can taking metformin result in dry mouth?",
"src": "Patient: I take serveal meds and evert night I get a very dry mouth. Could one or more of these meds be the cause? I tak Dilgizdm 120 mg, Metformin 500, Lisinopril 20 g;Warfarin 2.5 mg six days a week and 5 mg once a day, Amoldipine 5 mg, Prevastatin Sodium 40 mg. Thank you Bob Doctor: Hi,Thanks for posting the query, Prolonged use of medications can cause the condition.Take lukewarm saline rinses at home.Drink plenty of water to keep yourself hydrated.Take care!"
},
{
"id": 153534,
"tgt": "Will a cancerous mole have spread in 7 months?",
"src": "Patient: my mom caught a mole today that she has had for seven months. it turns out it is cancerous and she is going to the doctor tomorrow to discuss surgery to see if the cancer has spread. if she caught the mole within 7 months, what are the chances she caught it in time? Doctor: Thanks for your question on Health Care Magic. I can understand your mother's situation and problem. Yes, a mole can develop into cancer in seven moths time. Skin cancers like malignant melanoma, squamous cell carcinoma etc can present as cancerous mole on skin. Usually prognosis is excellent if early detection and surgical removal of mole is done. So better to get done staging of this cancer. And if it is localised cancer than surgical removal with negative margins is curative. Seven months duration is sufficient for spread of tumour. If she undergone biopsy earlier than early detection was possible. So first get done staging of her cancer and than decide treatment strategy. Hope I have solved your query. Wishing good health to your mother. Thanks."
},
{
"id": 69162,
"tgt": "Why do I have lumps on my chest?",
"src": "Patient: I have 3 hard lumps on my chest - / to bottom near sternum and one just below breast area. I'm a 35 yr male2 are pea sized 1 about size of small finger nailThey don't hurt, and you can't notice them unless you touch. Also have one on my back - this is about the size of a marble Doctor: Hi.Thanks for your query.Most probably these lumps , which are painless and have been noticed almost simultaneously are lipomas.There is chance that these can be sebaceous cysts or other forms.The best way to get a diagnosis is to do get them checked by a Surgeon , who on clinical examination can tell. Another option is to do FNAC- fine needle aspiration cytology.IF you are worried get them all excised.You get 2 benefitsThe disease is out of body and you get a perfect diagnosis."
},
{
"id": 156378,
"tgt": "Can treated carcinoids cause severe back pain with swollen knee?",
"src": "Patient: Hello Dr. My Dad's age is 67, weight: 57 height: 160cm. He was diagnosed with Nureo Endocrine tumor in thrid part of duodenum. Carcinoids are spread to spinal bones. He had unbearable back pain so he was adviced to take up radiation therapy for 10 days by an Oncologist. After the radiation treatment he feels releived but still the pain is there. Radiation therapy was taken up two months back and he was on medication of Idrofos 50 mg tablets and a vitamin tablets (folic acid and livogen). Now his right thigh to knee get swelled and feels irritation. Please prescribe the possible treatments for my dad. Thanks Doctor: The swelling in your father's right thigh needs to be investigated. A clinical exam and a radiograph of thigh would help. If there is metastatic disease in thigh bone it may need radiation therapy as well.What treatment is being done for neuro-endocrine tumor. Was an Octreotide scan done. If Octreotide scan is positive then Sandostatin LAR is a treatment option. Otherwise palliative chemotherapy is a treatment option. I would also suggest that instead of ibandronate (Idrofos), Zoledronic acid should be given which is a newer generation bisphosphonate compared to ibandronate. I hope I have answered your query. You can discuss it with your treating oncologist. If you have any further questions I will be more than happy to answer."
},
{
"id": 35320,
"tgt": "Is it safe to put a drawing salve on cyst on tailbone for draining?",
"src": "Patient: If I have a boil or Pilonidal Cyst on my tailbone, upper cleft of my tailbone, which has popped some and is now leaking slowly. It still has a long way until it is emptied, my question is, is it still safe to put a drawing salve on the boil/cyst to try to get it to drain out better? Thanks. Doctor: Hello! My simple answer is 'drain it out...!'Hi! this is a very common surgical problem and the patients generally suffer for years but don't opt for surgery till it is very bad a disease! Unfortunately, surgery is the only cure for this and needs antibiotics also! What happens? a blind canal appears due to pressure, ischemia or some birth pits in the butt-crack; hairs/hair follicles get collected in the cavity and invites organism creating abscess, pus drainage, itching etc...Hope you got the answer and now understand why it is necessary to consult a surgeon!regards,"
},
{
"id": 98715,
"tgt": "Is the talc in Serc a corn-based component?",
"src": "Patient: I m sorry to bother you - this is not really a health question but I have not been able to get an answer from anyone. I want to know if the talc in the tablet Serc is corn-based - I have an allergy corn and I need the prescription for my vertigo. You are my last hope. Thank you. Doctor: Hello I have gone through your question and understood your concern.Serc contains 16 mg of betahistine dihydrochloride as the active ingredient:It also contains-colloidal anhydrous silica-microcrystalline cellulose-mannitol-citric acid monohydrate-talcSerc does not contain gluten, sucrose, lactose, tartrazine or any other azo dyes.The talc is not derived from corn.so,don't worry.Hope this information is helpful for you.Take care."
},
{
"id": 36693,
"tgt": "Experiencing numbness on the upper right thigh after a bee sting",
"src": "Patient: hi i have a question.i was stung by a hornet or bee back on my upper right thigh back in aug 2011 and when i stand for long periods or do alot of walking for some reason indoors only that area becomes numb and i have a tinkling heat like sensation why is that? Doctor: Thanks for your query at HCM,I think it has nothing to do with the past bee bite. I would like to know your weight? I advice that you avoid standing for long time take intermittent rest. Get your blood sugar test for fasting and post prandial and HbA1C. Reduce weight if obese. Follow up with investigations or visit a local doctor.Take care!"
},
{
"id": 3280,
"tgt": "Can having unprotected sex followed by intake of pill cause pregnancy?",
"src": "Patient: Hello I took the day afyer pill on the 22 the same day I has sexual intercourse and we didnt use protection be he didnt ejaculate in me later I started feeling nauseous and dizzy allso my breast hurt and had headaches and im bloated I took a pregnancy test and it came out negative on the 30th could I be pregnant oris this all because of the pill? Doctor: Hello dearI understand your concernYou have taken the emergency contraceptive pill on the same day of unprotected sexual intercourse.It is highly effective in preventing the pregnancy if taken with in 24 hours.It has 95-97% effective in preventing the pregnancy.Nausea, dizziness, breast tenderness, bloating are due to hormonal imbalance caused by the day after pill.Pill also cause delay in the period by the 8-12 days.Meanwhile avoid stress, take healthy diet, drink plenty of water, do regular exerciseTablet Pantoprazole and domperidone, tablet gas x to relieve nausea and bloating.Hope this may help youBest regardsDr. Sagar"
},
{
"id": 42857,
"tgt": "Does ovarian cyst cause complications in conceiving?",
"src": "Patient: Hi doctor, i got married 2 yrs back still have no child. I have choclate cyst on left ovary of size 23x18 mm, my weight is 58 and height is 156 cm. i am 26 yrs old. Is there any big complication. My husband also has low sperm count. Is this cyst size safe for pregnancy. Pl help me Doctor: HAI WELCOME TO HCM you need laparoscopy surgery to remove your choclate cyst. then only we can stimulate you for ovulation. low sperm count will need IVF to conceive. endometriosis removal followed by IVF will gelp you conceive soon."
},
{
"id": 214598,
"tgt": "Suggest natural remedy for treating cough",
"src": "Patient: MY baby is 3.5 years old and she has coughing fits since last 10 days. v tried all sorts of medicines- ayurvedic, aloepathy, homeopathy. but it keeps on increasing. Please suggest me some natural home made therapy. We no more want to give her aloepathy anti biotic high dose. Doctor: hai,I don't find any term like coughing fits in modern science.Normally only bacterial origin cough will need antibiotics. Cough can be assessed by its frequency/severity/timing/nature. Cough indicates any infection (viral/bacterial)any block in airway,Reflux diseases and systemic involvement.Cough without any other major symptoms can persist 2- 3weeks.so i suggest you take some cumin seed powder(1g) with honey thrice a day.if till worsening consult a pediatrician to evolve pathology under the problem.Thank youHope i answered your query.Wish you a good health for your baby."
},
{
"id": 75032,
"tgt": "Suggest treatment for chest infection and numbness in the hands",
"src": "Patient: This for my mother 44yrs past 20 days Is suffering from joint pain(Finger,ellow,ankle,spine,fever and lost of appetite) Doctor told to do RA Test,Xray,Anti CCP anbody test,CA-Protien but all come negative,her ESR is 29 Homoglobin 10.6 Doctor has give anti inflammation injection and pain killer medicine,now she is fine but she can\u2019t lift her both hands and hands become very weak,pls help us what can be caused for this,which doctor we have to show her,she is asthama patients. She had last month fever and chest infection.so RA doctor told it ts post viral artherty. Thanks Lakhi sharma Doctor: Respected Lakhi Sharma, hi I evalauted your query.Yes post-viral effects sometimes last longer in which tests turn out negative and patient suffers positively in agony.You should follow certain measures at home Dear to lessen it`s intensity and faster recovery :* Light exercises and Physiotherapy of all concerned joints at home* Leukwarm water fomentation for 10-15 min. 2-3 times a day.* Maintain her hydration, BP , Blood sugar levels.* Make her ambulatory with normal distance walking till there are no joint pains.* High protein & iron diet for Hb and general weakness also. Hope this helps you to good extent so far as present query is concerned. Thanks for using Health Care Magic and taking time reviewing my answer. Regards wishing your father fast speedy recovery from the same. Bye Dear Take care."
},
{
"id": 1373,
"tgt": "Suggest me treatment to get pregnant as chemical pregnancy didn t work?",
"src": "Patient: Hi Sir, My last month period was on 1/03/2014. On 30/03/2014, I got dark positive on first sample of the day, then it started fainting in next tests. Today morning, the test line is barely visible. Ultra sound scan shows no sign of pregnancy in uterus (inter or intra). I am not having any kind of bleeding or spotting but sometimes cramps in lower left abdomen. I had 1 chemical pregnancy which ruptured in jan 2014 and one abortion done through DNC due to infection in uterus on 14/05/2012. Now, I am planning for a baby. My beta hcg which is done yesterday depicts 5.11 mlU/ml I am worried about my health and baby. Can you please guide me further. Doctor: Hi , How are doing ? Your history suggests that you don't have problems in getting pregnant, but after you get pregnant the health of pregnancy is not good enough. Value of But don't lose hope . May I suggest the following investigations to see if any problems & get them corrected if presentHusband- Semen analysis at least 2- done 4 weeks apart preferably in an infertility center ( examination done by andrologist), Blood sugarsWife - Blood Sugars, Thyroid profile Ultrasound scans to evaluate Uterus , Ovaries & adnexa Confirm tubal patency- most important- Tube is the connection between uterus & ovaries( the bridge where sperm meets the egg & forms a baby) this is size of hair follicle & cannot be seen on scans. So check with HSG( Xray with dye), or SSG ( Scan with dye) or Laparoscopy aPTT, ACA ( IgG & IgM ) & LA - blood tests- these cause repeated abortionsAll these can be corrected if problemIf all this done & found normal , nothing can stop a pregnancy with God's willHope I have clarified your query, do write back if any queriesAll the bestDr.Balakrishnan"
},
{
"id": 113754,
"tgt": "Having throbbing pain in back. CT scan normal.Took anti inflammatory pills. Is it normal?",
"src": "Patient: Hi, I was treated for a severe kidney infection, and was given levaquin for 14 days. The pain in my back is still here- ot a throbbing pain but, a deep dull ache al ost like I am brused in oth sides and lower back. I have had a ct scan with contrast, all was normal I have no fever , just pain. I contacted my uro and was told to take ant inflammatory pills or 48 hours. Is this normal. When will this pain away? Doctor: Hello. Thanks for writing to us. After a severe kidney infection, mild remnant inflammation is likely which is causing the pain. It can also be due to a muscular spasm. Taking anti inflammatory medicines and hot fomentation in the area will be helpful. I hope this information has been both informative and helpful for you. Regards, Dr. Praveen Tayal drtayal72@gmail.com"
},
{
"id": 128254,
"tgt": "Suggest treatment for shoulder tendonitis with numbness in the fingers",
"src": "Patient: I have been diagnosed with tendonopathy of shoulder via MRI and impingement of same. I have have much pain in right scapula, shoulder to elbow, and all the way to front of chest. I am also experiencing numbness and tingling all the way down to my fingers. Doctor: dear sir/ madamif i were your treating doctor i would examine your shoulder and also your neck to find cause of this radicular pain,i would examine neurological status of your upper limb then decide on management.for tendinopathy i would recommend rotator cuff strenghtening exercises, analgesics, i would also add pregabalin for this radicular pain and again review after 3 weeks."
},
{
"id": 217166,
"tgt": "Suggest remedy for severe pain in ribs on left back side at bottom of shoulder",
"src": "Patient: I have severe pain in ribs on left back side at bottom of shouldered that is radiating down left arm and some pain in front of chest.. I saw my chiropractor yesterday who thought rub was outta place and proceeded with adjustment and manipulations to put back in place after which the pain is worse and almost unbearable. BP is fine...just hurts to breathe, move, or even lay down in bed. What else could be wrong? Doctor: As per your explaination I feel there is a problem at spinal level to and that is why it is radiating. How ever the conformation can be done by only MRI so you may go for it covering cervical thoracic spine or consult a orthopedic first and as per his suggestion go ahead with needed tests. Now as per report here the ortho doctor will suggest you either on surgical correction if there is major problem or conservative management with medication and physiotherapy. You can also use hot pack at home to have some more pain relief.One more thing here we need to see that if you have breathing difficulty with chest pain on walking few steps or going up hill then it can be related to heart. To rule out you may need to do electrocardiogram with tredmile test.I suggest to follow above steps and take the required treatment. And I am sure you will be fine.Take care."
},
{
"id": 223920,
"tgt": "What causes irregular vaginal bleeding after intake of tri sprintec?",
"src": "Patient: I have been on tri sprintec for about 6 months the now, I have started bleeding for the 3 rd time in 2 months. I have been taking amoxicillian for a sinus infection. And today I was put on sulfamethoxazole-trimethoprim and I began bleeding 6 hours after taking it. What could be the issue here? What can I do to fix it? Doctor: Hi Hope this message finds u in good health.I have gone through ur msg and understand your concern.it may be due to some sort of hormonal imbalances or variations in the body,though there may be other reasons as wellNothing to worry about, You should eventually get back to normal.Take multi vitamin and calcium supplements Do consult a gynecologist if symptoms worsenGet back to me for any FOLLOW UP QUERIES anytime.Regards,Dr Mahaveer Patil...(MBBS,MS,Mch)"
},
{
"id": 14252,
"tgt": "Could intermittent itchy skin rashes be connected to the lump in the lung?",
"src": "Patient: my wife has rashes sometimes on her skin that itchs terribly and then goes away and comes back again and also a lump in the lung which may or may not be connected. we traveled in China also to a spa for the last five years but this started 1and 1/2 years ago. What do you think? Doctor: symptoms points towards hives which require a full course of antihistaminics under a dermatologist prescription...also consult a physician for the lump most probably they are not connected"
},
{
"id": 205891,
"tgt": "What causes weight loss and nightmares and night sweats?",
"src": "Patient: dear doctor my son has just turned 4yrs old and for the past few months he has got very thin. His stools are always soft and sink to the bottom of the toilet, on occasion they are diahorea. He recently has complained of his bum being itchy and it is red. He is at nursery school and I know he is active and growing but has always had a good appetite and we would eat healthy home cooked meals, however the food in school is somewhat processed and I think at times he does not eat a lot of his dinner in school and this may be a contributing factor in the weight loss. Also about 2 weeks ago he seemed to be having bad nightmares, sweating/fever and his body was twitching a bit. He was like that for 3 nights running and then it stopped. he has night sweats regularly but this was more like a fever. I am concerned about his weight, and was wondering if you would have any advice, or would this give you an idication of any possible diagnosis? Doctor: DearWe understand your concernsI went through your details. I suggest you not to worry much. I hope you consulted a child specialist regarding the symptoms you mentioned here. I could see the problems are a bit more complicated. Realistically, these symptoms could more physical than psychological. Please consult a child specialist before approaching any psychologist or psychiatrist.Psychotherapy techniques should suit your requirement. If you require more of my help in this aspect, Please post a direct question to me in this URL. http://goo.gl/aYW2pR. Make sure that you include every minute details possible. I shall prescribe the needed psychotherapy techniques.Hope this answers your query. Available for further clarifications.Good luck."
},
{
"id": 150626,
"tgt": "MRI scan showing bulge resulting in effacement of the ventral thecal sac and mild central stenosis. Suffer from blurred vision. Are these related?",
"src": "Patient: I have recently been having a lot of problems with my left arm all the way down to my fingers. I have a plate in my neck covering c6-c7-T1 and the MRI stated I have a 3mm bulge resulting in effacement of the ventral thecal sac and mild central stenosis . Is this what is causing it? I am also haing issues very recently with blurred and double vision . could this be caused by this issue.. They also commented on the MRI that there was some increased signal intensity within the cervical cord in the region (C4-C5) is noted n the sagittal T2 weighted sequence Doctor: Hello, Thanks for the query, I understand your problem Your problem in the left arm are related to the MRI changes that you have described. Cervical spondylosis can cause these symptoms. Regarding blurring of vision, I do not thin that it is related to the MRI changes. Causes of blurred vision are many and it would require joint evaluation by neurologist and eye specialist so as to establish the cause and deal with it. Hope this helps Best wishes Dr Gopal K Dash MD, DM, Post-doctoral fellowship (Epilepsy) Consultant Neurologist and Epilepsy specialist Nrayana Hrudayalaya Hospital, Bangalore My Blog in the Web site: http://www.healthcaremagic.com/doctors/dr-gopal-krishna-dash/64344"
},
{
"id": 102707,
"tgt": "Allergic to dust and almost every fragrance. Why am I feeling cold in upper arms?",
"src": "Patient: Hi,I both upper arms feels cold usually during the sleep, I feel same thing during the day but it doesn't bother me much. I have an allergy to dust, palm, sour grape almost every fragrance, I used to think that is way my upper arm feels cold. Even though room temperature is about 75 F or above, still my upper arm feels same thing. What do you think.Thanks in advance! Doctor: Hello, I have studied your case.As per your symptoms neck pain can be due to diffuse disc bulge compressing your cervical spine.Another possibility of thoracic outlet obstruction leading to vascular compromise leading to cold feeling.Some medication like methylcobalamine with muscle relaxant and analgesic will reduce pain, you can take them consulting your treating doctor.MRI and colour Doppler will help in diagnosis.Also check your vit B12 in your blood.Hope this helps. Wish you speedy recovery. Take care"
},
{
"id": 134654,
"tgt": "Suggest remedy for pain and instability of knee",
"src": "Patient: I landed on my left leg (Straightened) while playing football approximately 3 weeks ago. The right side of my left knee hurts, along with the back and calf muscle. Knee is yet to regain full range of motion, it hurts when doing a quadriceps muscle stretch/tightening (right side hurts) no history of injuries (on this leg). When I straighten my leg out right side feels very tight and sore. I can walk with no limp now (crutches for two days, limped for a week after), I followed the RICE principal. My knee did not swell and I still feel slight instability. There was a click as injury happened (not a pop) which makes me think it could be meniscus or possible PCL. I really want to get back to football, what do you suggest. Doctor: Hi Thanks for your query and Welcome to Healthcare Magic. I am Dr Akshay from Fortis Hospital, New Delhi.Well really if 3 weeks are already over and you still have any instability in your knee with off and on pain, it will be wise to get MRI scan done of the affected knee simply because you want to get into active sports and i want to be sure that there is no major injury there.Till that time you are advised to continue with your physical therapy exercises so that there is no muscle atrophy/wasting or any other stiffness.Do not hesitate to contact me if you need any further assistance. You can also discuss your case and treatment plans with me in a greater detail in a private consultation.Thanks & RegardsDr Akshay Kumar Saxena"
},
{
"id": 126972,
"tgt": "What causes severe shoulder pain?",
"src": "Patient: I have tremendous shoulder pain that feels like a scalpel has cut me open from my armpit straight up and over (like a strap) and the pain is also acute in the bottom of the shoulder blade. It hurts for my arm to hang at all, it is especially prominent when I m walking or doing anything which makes the hang or bounce more pronounced. I can not sleep or put pressure on it. It feels like an open wound! (It is not a torn rotator cuff.) Doctor: Hi, The symptoms are more or less consistent with a tendon or ligament injury. As the first line of management, you can try analgesics like tramadol or diclofenac for pain relief. If the symptoms persist, you can consult an orthopaedician and get evaluated, you might require an MRI scan in sever case. Hope I have answered your query. Let me know if I can assist you further."
},
{
"id": 949,
"tgt": "How to confirm pregnancy after missing a period?",
"src": "Patient: i got a pregnancy test from the hospital in the negative control it had a line and in the positive control it had a tick yet i got told by a doctor tonight that i was pregnant i went and bought a pregnancy test and that had came up negative what should i do Doctor: Hi, I think you can do a repeat urine test after 24 hours or blood test for pregnancy bhcg. It will clear your doubt. Sometimes urine test can be false positive or negative."
},
{
"id": 157956,
"tgt": "Strep throat, back pain from thorax to thighs followed by sores on thigh, buttocks, painful and open, bloating, sore nipples. Cancer signs?",
"src": "Patient: Hello I am a 30 year old female, about 218lb (I lost 80 lbs about a year ago) just last week I had strep throat to the point that I could not work for 5 days. In the midst of my illness the main worry was my back pain , from the Thorasic down to behind my theighs, I took some Alives and Ibprofen/Keflex (for infection). My symptoms bettered,, but then I found bad bite like soars on outside of my left theigh, and on lineing of buttchek and rt theigh. Also a 3rd one in the inside of my left theigh. These are open (as a bad mosquito bite) and are a bit painful. I have been feeling a bit bloated (lft side of stomach pain) and sore nipples . I don t know what is going on and I am getting scared. Cancer runs thru my mind, but only because there is a history of it in my family. In 206 I was diagnosed with a Fatty Liver due to rapid weight loss . Please help me. ------Dee82 Doctor: Hi, the bad bite like soars on your thighs may be a infection of the skin or hair follicle. the presence of pain usually rules out the malignant causes unless there is evidence of infection. the presence of cancer your family needs genetic counselling and screening for the early diagnosis and treatment of cancer. if the lesions are irregular and bleeding much, you can consult the dermatologist to rule out common skin infections and need for the biopsy of the same."
},
{
"id": 113432,
"tgt": "Lower back spasms, stiffness, pain. What is going on?",
"src": "Patient: Years ago a dr. Said I have lower lumbar strain. I had and have spasms in my lower back, stiffness , dull pain and spasms in my lower to mid back. Also, often when I lay in bed in any direction my back just akes and parts of my lower back feel like it is literally ready to crumble.I have a pulling sensation and sometimes numbness in my lower back and butt when crossing my legs and laying down or bending. Also, a few years after my diagnosis, I started getting pain running down my butt threw my legs. Another said this is sciatic nerve problem. What do you think is going on with me and what is the prognosis for the future in your opinion? Also, do you have any suggestions for me? Doctor: Hello thank you for sharing your problem. I feel you are in pain due to your symptoms. I am not aware of your age and occupation and details of any blood tests or xray/mri scans. I feel I need more information on this regard to guide you further into a pain free life"
},
{
"id": 124133,
"tgt": "What causes tightness around the neck after a work out?",
"src": "Patient: I play tennis for exercise..after about 10 to 15 minutes of warmup hitting groundstrokes I have started having a tightness all around my neck and upper bronchial area, and breathing becomes difficult...particularly if strenuous effort is required to hit the ball. After 10 minutes or so I seem to work through the worst of it. I have had surgery twice for ruptured discs in my neck at age 39 and 40, my neck is always tight and I do exercises to loosen it up. I have a little trouble occasionally swallowing during meals, particularly grainy foods. I also had asthma from about 5 years old till I was 17 which may play into the breathing issues. I am now 62 years old. Your thoughts would be appreciated. Doctor: Hello, As you had issues with the neck and surgery was performed I think any strenuous shot played by you using the muscles of the neck and shoulder girdle gives a direct pull to the nerve root leading to tightness which is not a good thing to hear as it may go on to damage it more and more. You need to be careful with it. Coming to your breathing issue, as you are asthmatic there are possibilities of you to forget doing breathing exercise which is most important in asthmatic cases. As some time in their life asthma cases do develop asthmatic attack and that can be a painful situation. Also please have a self assessment or ask someone to notice you, most of the times your breathing pattern will be from mouth to mouth. inspiration also through mouth and expiration too. In this case, the ability of the alveolar tissue to function can be reduced and may lead to additional symptoms which are not expected by asthmatic patients. Tennis is an aerobic game. it demands lot of energy. Since you are asthmatic the overall functional ability of the lung to cope up with this becomes less. Its always good to undergo a good pulmonary rehabilitation so the overall functional ability of the lungs improve. Also, will recommend you yo buy a respirator which is a small machine and can be used by asthmatic patients to assess their FEV1 values from time to time to assess the ability of their lung tissue. Please take a piece of advice from the Surgeon who operated you for the neck disc, before indulging in the sports. As the pull of the shot is directly hitting the nerve root to get trouble and muscles are trying to get stiff to avoid any major injuries to occur. In my practice of 12 years, I have seen both cases with discectomy and asthma and have responded well to physical therapy and precautions on when necessary to be followed. Hope I have answered your query. Let me know if I can assist you further. Regards, Jay Indravadan Patel, Physical Therapist or Physiotherapist"
},
{
"id": 179988,
"tgt": "How to improve appetite in a child?",
"src": "Patient: My 11 months old daughter doesn t like to eat anything . We tried all varities of food. She likes to eat herself. We tried that as we ll.. But of no use... Please suggest how can we make her eat... We consulted a doctor ghe gave one medicine but of no use... Doctor: HelloThanks for writing in HCMI had gone through your query and understood your concerns.I suggest you train her to eat food by saying that if she eats that day properly you will give her reward & give her small gift .this is reinforcement technique.Start her on syp aptivate i spoon thrice dailyHope this guides you. If you have additional questions or follow up queries then please do not hesitate in writing to us. I will be happy to answer your queries. If you do not have any clarifications, you can please close the discussion. Thank you.Wishing you good health"
},
{
"id": 65115,
"tgt": "Suggest treatment for lump under my chin",
"src": "Patient: Hi! I am 43 female caucasian. Overweight. Nerve injury in foot. I have a LUMP under my chin that is swollen and painful. I have also been having much IMBALANCE since Dec. Last time at Dr. no ear infection. I had small injury outer ear that seemed to swell in my head. Jaw hurts a bit, throat mouth a tiny bit. The imbalance feels like I'm on board a boat. I have never tried this online. So glad you are here. Thank you. Doctor: Thanks for your question on HCM. I can understand your concern. By your history and description, in my opinion you are mostly having enlarged lymphnode. Since it is painful, possibility of Infection is more. So you need to rule out ear, nose and throat infection. Better to consult ENT doctor and get done clinical examination. You may need course of antibiotics. And if still swelling is not reducing then you need fine needle aspiration cytology for the diagnosis of cause. For imbalance, you might require MRI brain to rule out cerebellar pathology. But first get done clinical examination of CNS (central nervous system) for cerebellar signs. Hope I have solved your query. I will be happy to help you further. Wish you good health. Thanks."
},
{
"id": 171176,
"tgt": "Is Amikacin sulfate 125 mg safe to use for 10 month old baby?",
"src": "Patient: hello, need your help asap. my baby is 10months and for the 2nd time is diagnosed to have UTI..now after the urine was cultured, they said it has E. coli.. so they looked for a much stronger antibiotic for her... Amikacin sulfate -- 125 mg every 24hrs thru IV --- so my question is, is this safe for her? im so afraid of the side effects --- kidney damage/hearing loss....need your feedback asap... Doctor: Hi, amikacin is safe to use in 10 month baby if used in proper dosage. In some conditions like high blood urea the dose needs to be decreased. Amikacin is also given in newborns. I think that you need not worry about this. Regards - Dr Deepak Patel, MD Pediatrics"
},
{
"id": 172506,
"tgt": "What causes nose bleeding after a head injury?",
"src": "Patient: hi my little 2 year old hit his head when falling two days ago he then got a nose bleed. It stopped quickly the next day he had another nose bleed took him to the doctor who said it was because he had a cold, he now has had blood come out of his nose on and off today Doctor: Hello,If nose bleeding occurs after head injury which is reccurrent and if it is associated with headache or vomitting then admit your child . Keep under observation for 24 hours and do Brain imaging to rule out any skull injury.If its only nose bleeding without vomitting or headache then consult ENT surgeon to check for any nose injury.Take care."
},
{
"id": 38425,
"tgt": "What causes the presence of ring worm inside the knee since 2 months?",
"src": "Patient: I had ring worm on the inside of my knee for about 2 months. I was using tea tree oil as recommended by a friend, but I developed a rash where the tea tree oil would slide down my leg in addition to my ring worm. I then went into a walk in clinic and they prescribed: fougera clotrimazone and betmethasone dipropionate cream. I have used this for 9 days, twice daily. The second rash was completely healed by day 5, the ring worm I believe is healed as well. It has not been itchy, pussy, or flakey for weeks (even before using the prescription). The area that my ringworm was is just a slightly raised bump and there is a very small red dot. To the touch however, I can feel two hard bumps underneath the skin. The small red dot is on the side of one and the other is smaller and to the side. Is this still ringworm? I can take a picture if there is a way to upload it. Doctor: Hello,This is Dr. Klarida Papaqako answering your question.From your description looks like the treatment was the right one, and it healed the ring worm but not completely. In general, it needs 2 weeks or more to cure it with the creams that you have mentioned.If I was your caring doctor, I would recommend to use the cream until it cleared the skin bump/red area completely, which may take from 2 weeks to 6 weeks.Try to keep the area as dry as possible, because humidity makes the ring worm infection worse, or doesn't allow it to heal.Take care."
},
{
"id": 88502,
"tgt": "Suggest treatment for abdominal pain caused after eating a spicy curry",
"src": "Patient: Hi, yesterday I was experiencing really bad burning sensations in my throat and esophagus plus pains in my stomach, today it wasn't as bad but after eating spicy curry i started to get really bad pains in my stomach, the pain is also going through to my back. I was at a music festival from Friday through to Monday and it involved alot of drinking, I was experiencing acid reflux but continued to ignore it, on the Sunday I stupidly drank half a bottle of straight whiskey and I'm wondering if I've maybe caused myself to burn the lining of my stomach or given myself a stomach ulcer. I've been bringing up bile and foam with what looks like tiny specks of blood. I took an anti acid tablet a while ago, the pain is still there and I'm still bringing up foam but the pain is still there although it's not as bad as it was. Doctor: Hello....(1) you r suffering from G.E.R.D i.e GASTRO ESOPHAGEAL REFLUX DISEASE (2) kindly take this treatment for continously 3 weeks and dont miss any of the medicine - (a) Tablet RANITIDINE 2 times a day i.e one in morning n one tablet in evvening (b) Capsule OMEPRAZOLE once daily in morning before breakfast (c) Tablet DOMPERIDONE 10 mg 2 times a day (d) syrup SUCRALFATE 2 teaspoon full 4 times a day (e) avoid spicy food/alcohol....follow this advice and trust me,your problem will disappear like i have done some magic :-) ......regards...dr rahul"
},
{
"id": 109256,
"tgt": "Suggest treatment for back pain and symptoms of flu",
"src": "Patient: Woke up yesterday morning with a head cold, and minor symptoms of the flu. Progressed throughout the day, until my back starting hurting extremely bad. Im completely bed ridden and need to get back to work... I do see a chiropractor regularly, however my symptoms are much more severe than just back pain . Any advice would be most appreciated. Doctor: thank you for sharing your complain,symptoms you described suggests respiratory tract infection-flu like symptoms. take simple medication like paracetamol three times per day, cetirizine-10mg at evening. you can also take antibiotic like levofloxacin/amoxycillin to prevent secondary infection.this will clear all your symptoms. if back pain severe you can take ibuprofen.if symptoms appear severe consult your family doctor first.hope this answer will be helpful to you.for more queries plz don't hassitate to ask.get well soon dear friend"
},
{
"id": 213397,
"tgt": "Have visual disturbance and panic due to distress. Marijuana?",
"src": "Patient: Ocassionally after a long walk or high stress i will get visual disturbences and a disimbodied expericence which leads to panic which causes this felling to become stronger, it is almost as if i am passing out but for very breifly and once i sit down or take ideporofen it eventually goes away. I first had this feeling when i started smoking majauna but i have stopped ever since Doctor: hai ianmayberry , with your information , it appears you had bad trip with marijuana once . also u started experiencing anxious spells suggestive of panic attack at stressful situations. you may be having adjustment disorder/panic disorder. are u using any other substance like alcohol... don't worry it is a quite treatable condition. kindly meet your nearest psychiatrist. also practice relaxation techniques, time management, learn & acquire problem solving skills. I wish u better physical &mental health"
},
{
"id": 221981,
"tgt": "Will the morning after pill cause harm in pregnancy?",
"src": "Patient: hi i just want to ask i had sex with out protection next day i took the pill and that week was supposed to have period nothing came i took test and i,m pregnant is that problem for my baby that iwas on the pill please help me i,m worried that the is gona effact the baby healht Doctor: if you take my opinion then i would advice you not to get worried ,usually the pill dosen't affect your child health, so don't worry"
},
{
"id": 92871,
"tgt": "Have problem with stomach. Have diabetes, colitis and metoclopramide for which taking a number of medicines. What to do?",
"src": "Patient: i have diabetes and have 2 years with intestinal problems I tried a lot of medicines natural and metoclopramide and a lot of medicines for colitis but I m still having problem with my stomach they said I have hernia but I don't believe that the xrays only shows intestines are swollen all the time, I am little desperate I do know what else to do I do not eat meat or too mucho bread or bad stuff that is my worst frustration Doctor: Hi, it appears to be the hernia which is causing the problem. or there might be some allergy to some foods, like milk and milk products, gluten containing foods like wheat, and barley etc. I advise you to consult a gastroenterologist for diagnosis and treatment. You may need to have M.R.I. for confirmation of the hernia. Thank you."
},
{
"id": 162748,
"tgt": "How can a ureterocele in a child be treated?",
"src": "Patient: My daughter 5 years old found out deplux collective system with dilated upper pole moiety. Ultra sound says the dilated right distal ureter with ureterocoel likely at the ectopic site of insertion of upper pole moiety ureter. How to treat this issue and can it be treated without Surgery Doctor: Hello and Welcome to \u2018Ask A Doctor\u2019 service. I have reviewed your query and here is my advice. No is cannot be treated without surgery. I will advice to parents and child to go through genetic counselling also if planning for next child and also risk of recurrence in further generation. Hope I have answered your query. Let me know if I can assist you further."
},
{
"id": 189157,
"tgt": "Swelling in the upper lip and sore in the inside of the lips after a fall. Pus formation. What to do?",
"src": "Patient: I fell on a sidewalk three days ago and landed on my face. The swelling on my upper lip has gone down but there is a huge sore on my inside upper lip and it seems to be torn by the impact of my upper teeth during the fall. I have been washing it with salty water several times a day but it is still present and when I put a tissue on it and apply light pressure, a yellow \"pus-like\" smelly effusion comes out. Not much. But should I go to a dentist or doctor if if continues much beyong this weekend? One other thing, I am diabetic so the mouth canker-like sore is of especial concern to me. Doctor: Hi !Thank you for your query.Dear sir,You seem to have an infection that has been superadded over the wound you got by falling over the upper lip.Since you are a diabetic the tendency of getting infections is increased and there is delayed wound healing. You should check with an endocrinologist for this immediately so that he can monitor your sugar levels which is essential for effective recovery and plus start with an appropriate antibiotic coverage to heal the wound.If you have any further queries feel free to ask.Wish you good health.Take care.Dr. Shruti"
},
{
"id": 178612,
"tgt": "Suggest treatment for child suffering from cold and cough",
"src": "Patient: Hi, My Son is 3.4 years old. We shifted from Mumbai to Bangalore in Feb 2014. He was fine for 3 months but since June he has been having cough and cold every second week. This is after he started going to school. The doc prescribed Relent, Budecort 200 and duolin. Please suggest if you can help me with some suggestions. Doctor: Dear Sir/ Madam,Thank you for posting your query at healthcaremagic.com.Welcome to Bangalore!Usually the Bangalore weather is known for chronic respiratory allergies. The medicine prescribes seems to be okay.Along with these avoid grapes, guava, chocolates and aerated colour fruit juices. Keep the child away from fragrance and scented perfumes. Have a practice of giving him steam inhalation at night and give him Luke warm water to drink.Please revert back if you have any further question, I will happy to assist you.With best wishes,Dr. Vishwanath Patil"
},
{
"id": 150011,
"tgt": "Diagnosed with dementia, short term memory seems to be worsening. Suggestions?",
"src": "Patient: Yes, I am worried sick about my sister; her doctor said that she has dementia but her short term memory is getting worse all time; she is forgetting how to cook for one thing; she had company over the weekend; theyleft to run into town and she forgot that they would be back and went to bed. she rremembers things that happened years ago when she was a child. Her doctor hasn't perscribed any medicine for alhermiers and her husband is just satting there waiting. she is 83 years old what should we do? Doctor: Hello, Thanks for the query to H.C M. Forum. 83 years old, memory weak. Old age in itself is a disease . SENILE DEGENERATION is a type of disorder in which as age advance patient's memory diminishes. This is natural process. Bharamhi is an ayurvedic drug which increases the memory. I have tried over many patient. In my opinion there is no any need of tests . Hope I have answered your question. If further any question I will help. Get well soon. Dr. HET"
},
{
"id": 23907,
"tgt": "What causes pain in the left arm after an angioplasty?",
"src": "Patient: Dear Sir, my father had got angioplastry surgery in Last year. few months later he got pain on left arm and shoulder . I discussed the doctor who done the Surgery and he advised us to check ortho specialist. Now my father taking four medicine which was suggested by surgeon as below clopilet a 75,amifru 40 mg, revolol xl 12.5mg, rosufit 10mg. is there any cause for left arm pain? Doctor: Hi,Thanks for writing to Health Care Magic, I am Dr Asad Riaz, I have closely read your question and I understand your concerns, I will hereby guide you regarding your health related problem.The arm pain that you told about is unrelated to a procedure done through his radial artery as there is no nerve supply over arteries that could cause him to sense pain.If his pain increases with arm movements especially when raising his arm above his head, it may be due to adhesive capsulitis that is the inflammation of the linig of shoulder joint For that he may have phydiotherapy, use warm compresses, apply pain killer gels and take some over the counter pain killers like Tablet NaproxenI hope this answered your question, if you have more feel free to ask.RegardsDr.Asad Riaz.General and Family Physician.."
},
{
"id": 108123,
"tgt": "Suggest treatment for fever and pain in lower back",
"src": "Patient: My daughter is having lower back pain, fever, mild cold or sinus symptons. She thinks that she may be pregnant and is 8 days late. She has been off of the pill since June of 2010 because she and her husband are planning on starting a family. First, can she take Tylenol to help with fever. She plans on calling her doctor in the morning for an appt. unless she can't get the fever down tonight. Otherwise, she may go to the ER. What would you suggest? I am online checking for her because they are travelling home from out of town right now. Doctor: Hi,It seems that she might be having viral infection producing fever and lower back pain.As she is late of her period there is chance of having pregnancy.Go for urine test for pregnancy.During this time she can take Tylenol as needed.give her with full stomach.Ok and take care."
},
{
"id": 6381,
"tgt": "Can I ask a doctor to look into my reports as I was on medication for no periods till 33 and want to conceive ?",
"src": "Patient: note i m 33 years old and my period is not started till date - my ultrasound and hormone report as is: i m 33 years old married women not started my periond till date i gone thur with my 4D ultrasonnd and the report is: uterus is anteverted and normal in size. it measures 7.0x2.2 cms. the myometrium and endometrium reveal normal echogeneiy. no mass lesion is seen. bothe ovaries are normal in size and echotexture. right overy measures 2.2x1.3cm. lift ovary measures 2.1x1.3cm. cul de sac is unremarkable. 3D/4D scan was performed which reveal no significant abnormality. pls. advise as above report my both overy is ok can i will be able to convice? is overy play a very important role to concive? if overy is ok than where is the problem is ? pls advise asap my Hormone test report as: luteinizing hormone, serum - 15.13 Follicle stimulatingg hormone, serum - 10.19 Prolactin, serum 4.78 Free Testosterone 0.17 Insulin, serum 6.76 insulin, serum (post- prandial) - insulin - 30.13 Glucose fasting , plasma -90 flucose,pos-prandial, plasma - 95 Testosterone, total, serum - 41.24 Hemoglobin 13.0 Hematocrit -40.1 White Blood cell count -4.3 Differential count Band (Stab) cells - 00 Segmented neutrophils - 54 Eosinophils - 02 Lymphocytes - 36 Monocytes - 08 Basophils - 00 Differential count performed on : EDTA SMEAR Thyroid panel by chemiluminescence, serum T3 - 93.23 T4 - 9.70 TSH - 1.53 Comment : Note \\: the reference range for insulin given on the report is established for a fasting specimen . there is no clinically significant reference range established for a postprandial specimen or any specimen other than a fasting specimen. the in Range & out of range ncolumns are not appicable for this value of serum insulin. Pls note my gyaecologist give me Restrone (5mg) 2 tabs twice a day for 8 days and some vitimns also and she said that my period will start within a 8 days AND finally my periods is start NOW is question is as above mention reoprts my both ovearys are ok now my periods is also start Can i conceive now ? Doctor: Hello. Thanks for choosing HealthcareMagic forum. Now that your period has started, now you need to watch your period if it comes normally without any medicines also. If it starts next month normally then your chances of conceiving are pretty high and you can be hopeful of the good news. Dr. Rakhi Tayal drrakhitayal@gmail.com"
},
{
"id": 175501,
"tgt": "What causes itchy spots behind ear spreading to face?",
"src": "Patient: Hi, my son is having an out break of some sort with spots on his face. It started behind his ear and gradually spread to his face. I don t see him scratching or complaining however it doesn t look okay. His father has the same issue however he uses a shampoo called Nizoral which he says clears it up. I did my research but I m worried about using it on my son who is only 3 years old. Doctor: Hi... By what you quote it seems like eczema or skin allergy to me. The trick of the management is keeping the skin moist and never allowing it to be dry. For this oil massage and lotions like Oilatum will help. Regarding cure options - eczema is a sort of allergy...allergies can only be controlled and can never be cured. But usually kids grow out of it...that is as the age progresses.But , skin conditions are best diagnosed only after seeing directly. I suggest you to upload photographs of the same on this website, so that I can guide you scientifically.Hope my answer was helpful for you. I am happy to help any time. Further clarifications and consultations on Health care magic are welcome. If you do not have any clarifications, you can close the discussion and rate the answer. Wish your kid good health.Dr. Sumanth MBBS., DCH., DNB (Paed).,"
},
{
"id": 87832,
"tgt": "Suggest remedy for abdomen & back pain",
"src": "Patient: yes sir I do have abdominal problem longtime ago I do feel too much bleeching abdomen babling heavy sholder windy back pain feeling fatigue headach when I bend left side or twist ifeel pain under chest abdomen bloating I feel burning under the feet and hands I feel light headed Ifeel whole body burning sometime when I eat I feel food stack in my throat I feel pain on chest both side my arms dead I feel fever I did almost all checkup and many medication especialy anti acid pls advise wha to do Doctor: Hi ! Good evening. I am Dr Shareef answering your query.After going though your history, if I were your doctor, I would advise you for the following:1) Try to remove your anxiety on this issue or any other one.2) Elevate the head end of the bed with two blocks on each side, so that the regurgitation does not occur while lying down, and the contents of esophagus and stomach flow down with gravity.3) Some modification in your diet like more of roughage, enough liquids, avoidance of oily, fried, and fast food, and avoidance of alcoholic beverages and smoking if you do.4) Regular routine exercises like morning walk depending on your medical fitness.5) Prescription for a prokinetic agent and a proton pump inhibitor for symptomatic relief and to have a control on the acid production and facilitate movement of gastric contents downwards.6) Routine stool test for ova and cyst and treat them if positive.7) If the problem continues, then you might have to consult a gastro enterologist for a possible endoscopic evaluation of your intestines, and also an ultrasound to rule out any intra abdominal pathology.I hope this information would help you in discussing with your family physician/treating doctor in further management of your problem. Please do not hesitate to ask in case of any further doubts.Thanks for choosing health care magic to clear doubts on your health problems. I wish you an early recovery. Dr Shareef."
},
{
"id": 30962,
"tgt": "How to treat finger nail infection?",
"src": "Patient: I have in infection in my fingernail. I have cephalexin cap. 500 mg. doxycycline hyclate cap. 100 mg. and amoxicillin 250 mg. I have controlled high blood pressure and controlled diabetes. can I take any of these medicines for the infection in my finger? Doctor: Hi,All antibiotic medicines you mentioned are good medcines for soft tissue infection.Go for one course of any one antibiotic medicine for 5-7 days.Go for dressing of finger nail as well.Ok and take care."
},
{
"id": 184524,
"tgt": "Suggest treatment for yellow discoloration of teeth",
"src": "Patient: Hi Doctor, I have a yellow tooth from my childhood or from the time I got my 1st tooth. I consulted a doctor once and he mentioned it should be flurosis. Please suggest me with a treatment to get rid of the yellow color and would also like to know if there is a age constrain for this treatment. Doctor: Thanks for your query, I have gone through your query.The yellowish discoloration could be because of the fluorosis or secondary to deposits.Consult a oral physician and endodontist to rule out these two conditions.If it is fluorosis, then you can get the tooth treated with veneers or tooth color crown. If it is because of the deposits, the get the teeth cleaned once and maintain the oral hygiene after that.i hope my answer will help you, take care."
},
{
"id": 225552,
"tgt": "Taken nordete after 21 hours of unprotected sex. Any chance of pregnancy?",
"src": "Patient: hi. im from philippines.im 22 yrS old. my first day of menstruatioN was sept3 to 7,2013. i have a regular cycle.we have sex inteRcourSe last sept26. we do the withdrawal, even my bf do that i stil doubt coz im thinking that there miGht be a sPeRm that drop insidE my vagina beFore he release his penis. after 21hrS,i took NordEtTe.is there posSible that i will get pRegnant? im expecting my mens. next weEk firSt weEk of oct.pls help me thank u Doctor: Hi,Thanks for the query. The possibility of pregnancy is less in your case. As:- In case of regular 28 periods to 30 days cycles, usually ovulation won't occur on around 23rd day of cycle. -And you took the emergency pill. But emergency pill can alter the menstrual period depending on the period of menstrual cycle when you took the pill. So your periods may get delayed. If you do not get periods even one week after expected date of periods, go for pregnancy test once. Take care."
},
{
"id": 38841,
"tgt": "What causes fever even after taking antibiotic therapy?",
"src": "Patient: My son was diagnosed with right basal pneumonia 3 wks ago. He was given zithromax for 3 days. 4 days after the end of medication, he again had fever. After having blood test, it was found out that infection was high again. He was prescribed with augmention for 7 days. 5 days after the medication is done. He is again having low fever. Could it be that medication is not enough? Doctor: HiWelcome.I also think so.It must be either the medicine is inadequtely dosed or it may not be right antibiotic or there might be some othe focus of infection. Get a culture sensitivity of all the sites suspected to be infected, be done. Then treat accordingly. God bless. Have a nice day."
},
{
"id": 12659,
"tgt": "Scalp psoriasis",
"src": "Patient: i am having scalp psoriasis problem. anybody there online to chat about this Doctor: puva theraphy is good for scalp psorasis. also you need to apply scalp shampoo every alternate day to keep the scalp cleen"
},
{
"id": 91650,
"tgt": "Got stomach pain on my left abdominal side and pass smelly gas. Suggest medicine",
"src": "Patient: Hi I have had 3 bowel movements today. All have been fairly normal that is soft. The first one was right after breakfast. The second was during lunch. The last one was as soon as I got home at 1:00 pm and it was normal, but I have had stomach pains on my left abdominal side. I had some herbal tea. I seem to have pass gas and it smells very bad too. my pain on my admonen is soo bad that I have heatig pad on my stomach. Doctor: Hello!Thank you for the query.Your pain in that area can be caused by several issues including stomach issues (peptic ulcer, stomach inflammation), urinary tract stones/inflammation, bacterial infection of gastrintestinal tract or even bowels obstruction.Stomach issues usually give upper abdominal pain which is associated with foods intake. The pain is dull and does not go away with painkillers.Urinary tract issues give also frequent urinating, burning while urinating, lower abdominal pain, fever.Infection usually gives also loose stools, bad smelling stools, fever, nausea.Bowels obstruction (partial) can give cramps and bloating.I suggest you to consult your doctor. Abdominal ultrasound, blood work, urine analysis should be done at first. If nothing will be found, gastroscopy should be next.Hope this will help.Regards."
},
{
"id": 51570,
"tgt": "What is the generic name of the tablet leon 500mg ?",
"src": "Patient: can i know the generic name of tab leon 500mg? Doctor: Hi Welcome to HCM It is levofloxacin an antibiotic used in respiratory tract infection..also available in 250,750 mg. Hope I have answered your query.take care Thanks"
},
{
"id": 18719,
"tgt": "What causes high triglyceride levels with weakness?",
"src": "Patient: I am 37 years old. High Bp patient for the past two years.I take Nebicard 5mg,daxid 50 mg in morning and taking cresar 40 mg and somproz 40mg in the night. My triglycerides is 366 when glucose test done for sugar in appollo hospital. sugar range is before 110, after 210 mg.doctor give capsule maxepa for lowering tgl. after taking 1 month, i feel that tired and standing is difficult. doctor recommend lowtgl tablet instead of maxepa capsules. But after that all times hugry and no energy and strength totally. What is the reason? Doctor: Hello and Welcome to \u2018Ask A Doctor\u2019 service. I have reviewed your query and here is my advice. Muscle weakness due to medication for decreasing lipid is pretty common. I suggest you to hold the medicine for few days. Get a test called Serum CPK done. Decrease fat intake in diet and do some exercise. Thanks and take care."
},
{
"id": 119426,
"tgt": "Suggest remedies for weakness in legs after spinal cord surgery",
"src": "Patient: good doctor,it s been four month now i have spinal cord surgery in this prestige hospital,after the surgery it seems that my condition has improve after a month but now my leg or the way i work is becoming weak.please advice me. i m from nigeria.thanks Doctor: Hi,Spinal cord surgery can lead to delayed complications like you due to fibrous scar / thickened ligaments or tendons / slipped discs from other sites due to weakened spine. Any of these could have caused you the post operative problem. Physiotherapy would be ideal management in such cases, also, give results. In case of no relief, repeat surgery may be needed but such surgery has more risk of complications. Please consult your surgeon for personal advise. Take care. Hope I have answered your question. Let me know if I can assist you further. Regards, Dr. Vasudha Jayant Athavale, General & Family Physician"
},
{
"id": 213541,
"tgt": "Under lot of stress, dealing anxiety and depression. Tickling sensation in heart and stomach. Cause?",
"src": "Patient: I am under a lot of stress and have been having a lot of emotional issues lately. I ve been dealing with anxiety , stress and depression . The past 3 days I get these sensations in my heart and stomach. It feels like it tickles and it s uncomfortable. it s tingly and tickles. Do you know what is causing this? I feel like maybe there is a lot of stress and emotions that build up that may be causing this? Doctor: Welcome to healthcaremagic. Anxiety, stress and depression is very tricky type of illnes, it produces many unusual somatic feelings inside the body. Please a doctor / psychiatrist to know the actual problem and treatment. I am sure this helps you."
},
{
"id": 83786,
"tgt": "What is the suggested dosage of losartin?",
"src": "Patient: I am taking losartin but I am lightheaded and get dizzy for a spell when I lie down. I ve bee gettin occasional readings of 100s/70s. Can I cut the dose in half for a few days until I can call mt physician since it is a holiday weekend? What is the safest way to reduce the dose? Doctor: Hi, Based on the history you seem to have developed hypotension (low blood pressure) apparently due to a slight high dose of losartan. You can take half the prescribed dose at bed time to prevent postural hypotension till you consult your physician. Hope I have answered your query. Let me know if I can assist you further. Take care Regards, Dr. Mohammed Taher Ali"
},
{
"id": 23334,
"tgt": "Suggest treatment for headache and low blood pressure",
"src": "Patient: Hello Dr. Koregol I am a 46 year old female with chronic low blood pressure. I was in an accident several years ago which resulted in permanent nerve damaged (I have been diagnosed with RSD/CRPS) and was on so many meds it was driving me crazy. I don t take any medications for Bp or anything else currently. I have been having aches and pains, headaches lately and really just feeling run down so I decided to take some readings on my BP. It wasn t that low today 109/73, that is after coffee and food. Is everything ok as long as the bottom number is above 60? It has been as low as 52. I have low platelets (hangs out around 90-110k), and low temp (96.5-97) as well. I have learned that this is my normal. But I feel off lately. When should I be concerned? Thanks. T Doctor: Hi dear every body is different what is abnormal for one person is perfectly normal for othera number of systems work together in our body to maintain the blood pressure,as because of accident you nerve has been damaged therefore your body finds it difficult to maintain a normal blood pressure ,but our body has power of adaptation,slowluy and slowly your body has adapted to the changes caused by nerve damageas per your query for a normal person lower blood pressure should be above 60mm of hg but in your case if your body is not having any symptoms like dizziness ,feeling of impeding doom ,a blood pressure of around 50 is normalas i said earlier until you are symptomatic yo need not to worry,any symptom like palpitation ,gidiness ,feeling of passing out is to be taken seriously ,if you feel any problem it should be a matter of concern till the time you are asymptomatic dont worry"
},
{
"id": 195848,
"tgt": "How to treat hydrocele on the left side of the testicle?",
"src": "Patient: I have had a left sided hydrcele for about 9 months. Its about the size of a large lemon. The docs and consultants i have seen here in the UK have basically refused aspiration so i am going to do it myself. Could you tell me what i need and the safest way of doing it? Thanks Doctor: Hello, Best way to treat hydrocoele is surgery. In those who are at high risk of complications, needle aspiration done. Kindly follow with any query or consult with a surgeon.Hope I have answered your query. Let me know if I can assist you further.Regards, Dr. Shyam B. Kale"
},
{
"id": 187773,
"tgt": "Could an ingrown wisdom tooth be the cause for a painful lump under chin near jawline?",
"src": "Patient: Okay so my wisdom teeth are growing in and one of them hurts a lot. I have to get them removed but in the mean time I got this lump under my chin near my jaw line. It's almost directly under my hurting wisdom tooth I was wondering if that normal or something else is wrong. It's pretty painful Doctor: Hi,Thanks for asking the query, Yes this could be because of the impacted wisdom tooth.I would suggest you to consult to an Oral surgeon get a thorough checkup done take an OPG radiograph and plan for the removal of tooth.Take a course of antibiotics and analgesics.At home take lukewarm saline rinses and antiseptic mouthwash gargles.Take care!"
},
{
"id": 207629,
"tgt": "What causes distraction of thoughts while talking to people?",
"src": "Patient: hi , i am suffering from shizophernia and taking risdon 4 mg and pecitine 2 mg. but sometimes i have problems interacting with people and sleep problems. when i talk to people that things keeps on coming into my mind sometimes. also my family forces me to talk to people which i am uncomfortable with. i also feel financially insecure and i am in no position to work though i do housework but i feel my brother and his wife will throw me out of the house or will make such a situation that i myself leave the house . i am 44 years of age and not married, i live with my mother and doing spiritualism. can you suggest me some good medication so that i control my disease? Doctor: DearWe understand your concernsI went through your details. I suggest you not to worry much. Talking and interacting with people is treated as inherent therapy for schizophrenia affected people. Vocational therapy is also considered advantageous. Once you are sure that your schizophrenia is under control with these medicines, then why don;t you try for a job? Work with your psychiatrist for a better treatment / therapy plan.If you require more of my help in this aspect, Please post a direct question to me in this website. Make sure that you include every minute details possible. I shall prescribe the needed psychotherapy techniques.Hope this answers your query. Available for further clarifications.Good luck."
},
{
"id": 66932,
"tgt": "What does small, painful bump near buttocks mean?",
"src": "Patient: Hello. The other day, I noticed that it was very painful for me when I sit, lay on my back, bending over, or anything that involves using my butt muscles. I looked in the mirror, and noticed that I had a thumb sized bump near my crack. Its very painful, and I believe its a cyst. Can you help me? Doctor: yes, you are right!it is a sebaceous cyst that gets infected every now and then!do an FNAC test for confirmation and meet a surgeon if necessary!all the best!"
},
{
"id": 9582,
"tgt": "Extremely dry skin, hard looks hard with excessive dead skin. Solution to overcome this?",
"src": "Patient: HI, I have skin related problem. no soap suits on my hand . hand r 2 much dry and have 2 much dead skin. hands r 2 hard and yesterday i have cut my one finger through my own nail due to 2 much hardness and dead skin.. hands looks 2 much dry and sow 2 much deadness.. please give any solution how can i can i overcome frm that. Doctor: Hi The dry skin can be due to several conditions including hypothyrodism and liver disease there have somes tests like TFTs, LFTs and hepatobiliary system ultrasound. if they come normal then use moisturizer at day time and petroleum jelly at night time Thanks"
},
{
"id": 171823,
"tgt": "Should loose dark green stool in an infant be of concern?",
"src": "Patient: dark green poop my son is 19 months and for the last few days has been having loose dark green poops..so dark that in some spots it alomost looks black..it also has a real unusual strong smell to it.she also doesn t like me changing his diaper since it started and although it s loose stool he has a hard time pushing it out. he is a healthy todler the has a (for the most part) healthy appetite should I be concerned ? Doctor: Hi,Thanks and welcome to healthcare magic.Dark green stools are not normal.It suggests infection.It requires treatment .Better consult pediatrician .continue feeding as usual.Sporolac one sachet twice daily may help.Hope this reply is OK for you.Further queries invited.Dr.M.V.Subrahmanyam."
},
{
"id": 57484,
"tgt": "SGPT -98 U/L, SGOT-72 U/L, TOLAL BILIRUBIN IS .92 mg/dl, Diet and care?",
"src": "Patient: hello Dr., my father is suffering with some of liver problem test says that SGPT -98 U/L, SGOT-72 U/L, TOLAL BILIRUBIN IS .92 mg/dl please suggest what kind for meal should he have to talk & what kind of care to be taken. he is nonalcoholic person some times avail tobeco. ECG reports are normal. Doctor: say no to tobacco first.diet should be light.nonspicy,if nondiabetic then plenty of glucose in the form of juice."
},
{
"id": 6732,
"tgt": "Is it safe to get pregnant in early thirties ?",
"src": "Patient: Hi, I am 33 years old, I am married for 4 years now. My husband do not like to have kids as he feels they are just a headache and they are like stress busters. I am not able to convince him and most my friends have already have kids, when ever i meet them I feel insecure and I feel sad that I do not have any kid. My husband do not even touch me.. I feel that I have made a wrong choice of marrying him. Doc, please help me if i can get pregnant after 33 years..? Doctor: Hello. Thanks for choosing HealthcareMagic forum. Yes. You can get pregnant after 33 years of age until you have your normal periods the only difference is that it will be a high risk pregnancy and you will need regular medical care during the pregnancy.First you need to convince your husband. Dr. Rakhi Tayal"
},
{
"id": 112438,
"tgt": "Had pain in side lower down, lower back hurts, stomach pain. Suggest?",
"src": "Patient: I have had pain in my right side low down for months, its getting worse, its now starting to hurt on left side, my lower back hurts, and my stomach, the pain on my right side is making my right leg hurt, I am in constant pain, I have seen a consultant at hospital year ago he said it was my lower bowel, im in so much pain, fed up going doc, s I know my own body and think and I think it, s something elsealthough I dont know what please can you help Doctor: It may be wise to undergo tests.use hard bed,avoid lifting weights,hot fomentation of back.Take anti-inflammatory pain killers and take high fibre diet"
},
{
"id": 141255,
"tgt": "What to do for chronic backache transitioned into sciatic pain in the lower extremities along with body tremors?",
"src": "Patient: Hello. Without a full history, I understand it is impossible to respond with confident specificity; still, I have dealt with sporadic lower back pains for about 7 years, and around the first of last May, the pain returned and was crippling; the worst it had been, without a particular, immediately precipitating injury. After a week, the lower back pain transitioned into sciatica in my left leg, although it is faintly present too in my right thigh and right ankle. The pain is now worst on my left ankle, but present too in my left shin, my left index toe is numb most of the time, and sometimes feels as though jammed. I have the burning patch on my left thigh, and the hallmark symptom of sciatic too--but I am now dealing with nerve tingling and soreness in my left arm, at times right arm, and in both hands--which also slightly tremor, my leg too if crossed--but these tremors are not new to this recent onset. I feel tenderness and pain at times in my neck/shoulder, left side. My stomach is constantly spasming, its persistent contractions distinctly visible. I am dealing also at times with jerky movements of my legs and arms, and sometimes my head. I seem now to frequently urinate with great pressure, but I do not feel urgency to micturate beforehand, and my bowel movements are now often difficult. It very much seems to be a comprehensive and progressing thing, and now rapidly so, affecting certain peripheral and motor nerves primarily. Doctor: Hello and Welcome to \u2018Ask A Doctor\u2019 service. I have reviewed your query and here is my advice. You need to consult spine surgeon as early as possible and get MRI scan and X-ray of the back. If it shows any abnormalities compressing the nerve or spinal cord. you need to undergo surgery. Hope I have answered your query. Let me know if I can assist you further."
},
{
"id": 138840,
"tgt": "Suggest treatment for numbness and tingling on knees",
"src": "Patient: Lately I have been waking up with that sleeping feeling in both of my knees. They are numb and start tingling once I move them and stretch. I have no history of knee problems, and currently do not have any pain or swelling. This only happens when I am sleeping on my back, I have tried to sleep with my legs elevated to see if that helps, but is does not seem to make a difference. Is this something I should be concerned about? Or is this just a minor inconvenience I have to deal with? Doctor: Hi,Thanks for your query.The symptoms that you are having are likely to be due to a nerve root compression . You need to get an MRI scan of the area done and consult a neurophysician for proper diagnosis and treatment.Meanwhile you can follow these measures:- If the pain is severe, you need bed rest till the pain resolves. - Get some analgesics prescribed and apply analgesic spray or ointments.- While resting, keep a pillow under your knees if it doesn't bother you.- Avoid lifting heavy objects.- Back strengthening exercises and stretching exercises daily as advised by a Physiotherapist.Hope this helps. Let me know if you have any more concern.Warm Regards."
},
{
"id": 37080,
"tgt": "Are my uncle's lung infection and non functioning kidney interconnected?",
"src": "Patient: My Uncle is suffering from Multiple organ Malfunction. he was recently admitted to hospital and kept in ICU. The CT scan showed infection in his lungs due to some fluid and also his kidney is also not functioning properly. Is there any interconnection between the two. Doctor: Hello,I understand your concern.I am Dr. Arun Tank, infectious diseases specialist, answering your query.In my opinion patient should be investigated for systemic illness.There are some connective tissue disorder which can affect multiple organs like lung, kidney, brain etc. So if your father is investigated further, you may find something pertaining, and van be helpful in treatment.Please maintain optimum blood pressure it is surely can help you in maintaining the blood pressure.I will be happy to answer your further concern, you can ask me on bit.ly/DrArun. Thank you.Dr Arun TankInfectious diseases specialist."
},
{
"id": 120692,
"tgt": "Suggest remedy for cramps and claw foot",
"src": "Patient: I was first thought to suffer from epilepsy, then the diagnosis changed to conversion disorder. I am highly stressed and the slightest upset can cause me to shake. I suffer from chronic facial twitch/tic/spasm in my chin which has been every day for the last 5 months. The strain is felt all through my neck and head, and my scalp will often spasm as well. When a spot is pressed under my right arm, it causes my right arm to spasm uncontrollably for prolonged periods. My nervous system doesn t feel right, but my doctor says its all just to do with conversion disorder? My cramps are also getting worse, and I am getting claw foot.... Please help Doctor: Hello,Your symptoms seem to be related to muscle cramps. I suggest using a spasmolytic such as Baclofen three times a day. I also suggest using magnesium supplement for muscle relaxation. Warm shower can also be helpful.Hope I have answered your question. Let me know if I can assist you further. Regards, Dr. Dorina Gurabardhi, General & Family Physician"
},
{
"id": 34211,
"tgt": "Suggest treatment for stomach ache and fever",
"src": "Patient: My son is 6 he went swimming yesterday came home and went straight to sleep at 5pm. he woke up at 9 with a fever and stomach ache he now has a headache and has not been eating his stomach hurts him all over his temp has gotten up to 101.5. Do you think its a stomach flu or could it be something else. Doctor: HIThanks for posting your query. It seems like he has sustained infection of the stomach. Infection could be due to Virus or bacteria. Most of the viral infections are self limiting and will go off in 2-3 days . So just give me tylenol for 2 days and observe . If fever or Stomach pain persists there is a high possibility that he could be having Bacterial Infection of his gut. In that case you need to take him to a doctor who shall do certain blood tests and prescribe him the right antibiotic. Till then give him hot water to drink, less oily food and no non veg food . Hope this information was useful to you. Any clarifications feel free to ask."
},
{
"id": 2739,
"tgt": "What is reason for not being pregnant even after follicular study?",
"src": "Patient: hi doctor.. I have got married just 1 year.. before of my marriage my periods was regular..but after marriage it was irregular,, when I was undergone a ultrasonic scan ,it was known 2 us tat due 2 overweight..pco and irregular periods nu..after marriage my weight increased about 10 kgs... nw its reduced as 65kgs ..last month I have undergone follicular study and egg ruptured ..doctor advised 2 have interecourse and also v had..but this month I got my periods...but reason don t know???? why negative pregnancy test?? doctor advised my husband also 2 take semen test...his test of sperm count s 52 millions... n its normal,,but motility is low about 10 millions....so doctor gave him MAXOXA L powder and doxy 1oo mgs tab....for 2 weeks tab and 3 months powder..... I should want d reason why I didn t get conceive even after follicular study .. help me doctor waiting for ur reply ... Thank u Doctor: Hi, I think the motility of the sperms of your husband is a little low. You can go for IUI. that means the semen of your husband will be processed and it will have much better motility and then it will be instilled into your uterus on day of ovulation. This may work for you.Hope I have answered your query. Let me know if I can assist you further. Regards,Dr. Khushboo Priya"
},
{
"id": 7431,
"tgt": "Acne problems. Prescribed microdox lbx, moisturising lotion, isotroin. Safe to take?",
"src": "Patient: I am 21 years female suffering form acne problem. My doctor advised me to take microdox lbx for 30 days and with moisturizer lotion and a cream during night. After completion of this course for one month, he has asked me to continue with same treatment and along with Isotroin -20 for 30 days. Looking at the side effects of isotroin - 20 I am in state of confusion to take it this tablet or it. Please guide me as soon as possible, Since I have not started my course yet. Doctor: Hi, There is nothing wrong in the prescription suggested by your doctor. Isotretinoin is considered to be a wonder drug to treat acne. It has its side effects, however if doctor's advice is followed properly it is quite safe to use. The only side effect, you might notice is dryness of skin and lips and that can be prevented by keeping your skin well moisturised. Other so called serious side effects occur only when it is continued for a prolonged period without doctor's supervision. Your doctor must have evaluated the safety and efficacy of the drug in your case. So trust your doctor, start the course and remain in proper follow up for an acne free skin. Take care."
},
{
"id": 63523,
"tgt": "Suggest treatment for a lump appearing under the left clavicle",
"src": "Patient: Hi, may I answer your health queries right now ? Please type your query here...i am a 38 yr. Old female about 9 months ago I found a olive size lump under left clavicle its mobile its soft and painless I had a ct scan on my collar andmy lungs about 9 months ago lungs looked good (i am a smoker) and ct scan showed nothing on my collarbone what did dr. mean when he said nothing there when I feel it.by the way it hasnt changed in size. Also had a negativw mammogram that same day...thanks...ml Doctor: Hi,Dear I studied your query in all it details and I understood your concerns.Cause and Remedy - On whatever limited facts given from you about the lump under the clavicle, and with my experience My opinion is as follows-YOU seems to have underwent a breast check up with a negative mammogram.And your doctor seems to have opined on presences or absence of breast lump in axillary breast tissue.Hence To reduce your worry Please consult for opinion from ER Surgeon about the olive size lump under the left clavicle,who would confirm if its Sebaceous Cyst or any other lump like lipoma and thereafter treat accordingly for the cause of the lump.Plz hit thanks and write excellent Reviews if this would resolve your query.Plz don't worry and do Welcome for any further query in this regard to me.Have a Good Day.Dr.Savaskar M.N."
},
{
"id": 189833,
"tgt": "Tooth removed, sore throat and tonsil. On antibiotic, difficult to swallow, pain not subsiding. Is it normal?",
"src": "Patient: I had my right lower & upper wisdom tooth out at my local hospital Jan29th 2013, topday 22nd feb 2013 I still can not open my mouth more than say 6-7mm. My right side of my throat and tonsil are really swollen. I vistied my normal dentist Mon 18th feb 2013, who prescribed my anitbiotics but the lump under my jaw and the swelling of my throat/tonsils does not yet seem to be subsiding, is this normal, swallowing my own saliva is even painful. Doctor: thank U for the Query, pain, swelling and difficulty in opening ur mouth are common problems faced after wisdom teeth removal. As days pass by , its keeps healing accordingly and symptoms gradually subside. the duration is approx 1 month now from the time of ur teeth were extracted and looks like the infection still persists. Advise u to visit the specialist and get it checked at the earliest to avoid the problem from agravating. U will be probably need a change in medication if required and further assesment. regards, Dr. Manmeet"
},
{
"id": 216092,
"tgt": "What causes pain in upper abdomen?",
"src": "Patient: hi,2 years back i had gall stone surgery, now getting pain at right top of the abdomen below the ribs, i went for EUS test every thing is normal , mild echotexture formed at tail & bottom of the pancreases, and in ultra sound test, liver is in normal size ( 15.0 cm) with increase in echotexture.can you please explain is it life threaten, what care can i take in further.Please clarify Doctor: Hello and Welcome to \u2018Ask A Doctor\u2019 service. I have reviewed your query and here is my advice. Several features of pain can be helpful in clarifying what is going on and the next steps for diagnosis and treatment. Type of pain. Burning mostly means a nerve is having problems. This can be either a sick nerve or a nerve that has had an injury. Electrical, shooting would be others implying nerve source for pain. Location of pain. There is a lot to this. First, if something goes in the pathway of a nerve, then that is the nerve involved. Nerves are long and the pain is in a belt-like long pathway. Smaller areas are smaller nerves. This also tends to say what might be going on. Obviously if there was an injury to the area and then there was pain outward from it \"hit a nerve\" would be a possibility. Sick nerves are MULTIPLE areas and tends to be the TIPS of the nerve not the whole nerve. What hurts. Moving a particular way that triggers the pain tells what is hurting. Specifically it can distinguish outward structures like skin, muscles, tendons, joints from deep structures like organs and nerves. Can be irritation of the liver or pancreas--blood tests are quite good at indicating this. It rarely happens after gall bladder surgery. Hope I have answered your query. Let me know if I can assist you further."
},
{
"id": 89008,
"tgt": "Suggest treatment for mass of 10mm growing in the stomach",
"src": "Patient: Hi, My husband 65, was told that he has a mass of about 10 cm growing inside his stomach, next to the bladder, no obstructions yet. He is scheduled to have a biopsy tomorrow. Is that risky? Is it necessary? He also was told the there is possibility this mass might be a lymphoma. What to do? Doctor: Hi ! Good evening. I am Dr Shareef answering your query.Although it is a matter of concern, one has to wait till the histo-pathology diagnosis and then the appropriate treatment could be planned. There is no need to panick and the treatment of any such problem like a lymphoma has improved over the years with better prognosis then what it was a few years back. Whatever it comes to , one has to face the reality bravely specially you and other members of the family to strenthen your husbands morale in receiving the appropriate treatment.Please do not hesitate to ask in case of any further doubts.Thanks for choosing health care magic to clear doubts on your health problems. Wishing you all the luck. Dr Shareef."
},
{
"id": 119322,
"tgt": "Diagnosed with aplastic anemia, advised increase in platelet transfusions post rabbit serum-based treatment. Any other treatment options?",
"src": "Patient: Hi, My 65 year old brother was diagnosed with aplastic anemia in December 2011. Six weeks ago, he had a rabbit serum-based treatment. He has since been in the hospital with blood on his lungs; it is thought that his platelet transfusions should be increased to prevent the bleeding . In any event, his regular doctor told my sister-in-law yesterday that it would be a miracle if he survives. I realize that this is a very unusual disease with not much history to go on. However, aren t there other alternatives than just giving up? Thank you. Doctor: I'm very sorry to hear about your brother. Aplastic anemia is a very serious condition. The bone marrow shuts down and no longer makes any white blood cells (protective immune cells), red blood cells (vital oxygen carrying cells), or platelets (clotting cells). This can happen due to cancers, radiation exposure, certain drugs, and many other causes. Treatments vary and include steroids, injectable colony stimulating factors, transfusions (putting the cells in), and sometimes bone marrow transplantation. Transplantation is very hard to do because there must be an exact match (often a family member or child must donate, but even then a match is not guaranteed). However sometimes, depending on the actual reason for the anemia in the first place, the body starts making cells again on its own. Providing supportive care in the hospital is sometimes all we can do. I hope things get better for your brother and I will keep him and your family in my thoughts."
},
{
"id": 172014,
"tgt": "How to treat pneumonia in a 2 years 11 months old child?",
"src": "Patient: Hi,My daughter (2 yrs 11 months) was suffering from pneumonia, was hospitalized for 6 days and was discharged 2 weeks back.However she started getting mild fever (99-100) since yesterday and we gave her crocin syrup. Today we took her to our pediatric and the doctor said it does not seem to be some stomach infection and suggested 5ml Vizylac and 5 ml zifi 100 twice a day and to use crocin or meftal-p only as SOS.It is night now and she is running fever again (100.5). i have given her Crocin Syrup (5ML) but the fever is only down by 0.5 degrees in the last 60 minutes. Should i give her some more crocin? Doctor: Hi...Thank you for consulting in Health Care magic. I don't think this is related to her pneumonia 2 weeks back. Fever of few days without any localizing signs could as well a viral illness. Usually rather than fever, what is more important is the activity of the child, in between 2 fever episodes on the same day. If the kid is active and playing around when there is no fever, it is probably viral illness and it doesn't require antibiotics at all. Once viral fever comes it will there for 4-7 days. So do not worry about duration if the kid is active.Paracetamol can be given in the dose of 15mg/kg/dose (maximum ceiling dose of 500mg) every 4-6th hourly that too only if fever is more than 100F. I suggest not using combination medicines for fever, especially with Paracetamol.Meftal-P contains Mefenamic acid which is a NSAID. It carries the risk of gastritis and renal problems in the long run. Please do not use Meftal-P. Hope my answer was helpful for you. I am happy to help any time. Further clarifications and consultations on Health care magic are welcome. If you do not have any clarifications, you can close the discussion and rate the answer. Wish your kid good health.Dr. Sumanth MBBS., DCH., DNB (Paed).,"
},
{
"id": 20785,
"tgt": "Does electrocution affect blood pressure?",
"src": "Patient: i was electricuted at work.now im having blood pressure problem it is real high so i got on meds ive tried 6 differant kinds nothing seems to keep it stable it goes way down or way up my heart rate goes up when my blood pressure is low and down when its high is that normal can bieng elctricuted mess with your blood pressure long term Doctor: Hello!Welcome on HCM!I passed carefully through your concern and would like to explain that an incident electrocution doesn't seem to be related to persistent hypertension.Coming to the point that your high blood pressure values keep persisting, it is necessary to perform a comprehensive investigation on the possible predisposing factors (anemia, thyroid, renal, adrenal, metabolic disorders, etc.)A careful physical exam followed by several medical tests would definitely clarify this issue:- resting ECG,- complete blood count,- thyroid hormones level,- BUN & creatinine,- blood electrolytes level,- renine and aldosterone level,- urinary metanephrines,- fasting blood glucose, etc.After excluding secondary hypertension causes, a well tailored program for optimizing your daily life-style (lowering body weigh, a healthy diet, avoiding smoking contact, alcoholic drinks, triggering factors like caffeinated drinks, etc.) would be necessary in addition of an appropriate anti-hypertensive medication scheme.Hope to have been helpful to you!Kind regards,Dr. Iliri"
},
{
"id": 178451,
"tgt": "What to do if child is suffering from throat infection, vomiting?",
"src": "Patient: My son is 8+ years old , he s suffering from throat infection for the past 2 weeks and is unable to eat anything. As he eats he keeps vomiting. .. but he s not recovering I ve shown him to a doctor n he had advised him Pan 20 before food once a day, azibact tab once a day and Syrup Ascoril , 5 ml 3 times a day. Plz let me know whether any blood test is required to be carried out. .... I m very much worried as he s become weak. Doctor: Dear Sir/ Madam,Thank you for posting your query at healthcaremagic.comMake is his diet simple, liquid and semi solid. Nourish him with lots of fluids in form of soups, fresh fruit juices, porridge.Continue Pan 20 and azibact, and stop Syrup ascoril(as it caused vomiting in some children).If symtoms persist consult the pediatrician and get blood test done to rule underlying infection, if any!Hope you find my answer informative, Please revert back if need any further information.With best wishes,Dr. Vishwanath Patil"
},
{
"id": 140684,
"tgt": "What causes the sensation of blood?",
"src": "Patient: I have been smelling and tasting blood for the past three days, but my nose is not bleeding. I have also had a slight headache (I have a history of migraines) for the same amount of time. I have a neurological problem that is still being diagnosed, but with these symptoms, do I need to go in to see my doctor? Doctor: Hello, I have read your query and here is my advice. Since you are under diagnostic procedures for a neurological disorder and there are new symptoms, I think that you should see again your Doctor and report to him these new issues. So, I think that you should go to see him. Hope I have answered your query. Let me know if I can assist you further. Regards, Dr. Erion Spaho, Neurologist, Surgical"
},
{
"id": 56639,
"tgt": "What causes bleeding in the liver?",
"src": "Patient: What causes bleeding in the liver. My mother is currently in the ICU. She has been on comudin (sp?) since she was 28. The doctor I spoke with said the medicine does not usually present with bleeding in the liver. They are putting a fliter in her stomach in about 2 hours. Please help me understand! Doctor: HIWell come to HCMI really appreciate your concern for your mother this could be portal hypertension, there must be some mistake in understanding because it is not like that to have the filter in mouth, it need to be explained in little medical term so this could be easily to advise you in that way, hope this information helps, take care have a nice day."
},
{
"id": 154596,
"tgt": "What could cause a back pain after a breast cancer?",
"src": "Patient: had breast cancer left breast 2005 have constant pain in center of left shoulder blade area since 2010 so sore it hurt to sleep or even get a hug physical theraphy on back and neck for over a year brusitis on right hip tendonitis in right knee have had ct scan mri bone scan and still not showing anything for back pain no way to live limited to most of the things i loved to do seen every ologists still no long term resolution any ideas where i should go from here Doctor: I would advise you to do a pet scan.For pain you can take tablet morphine 10 mg thrice daily after food."
},
{
"id": 56160,
"tgt": "Can high estrogen level lead to gall bladder dysfunction?",
"src": "Patient: I was just told by my chiropractor that I have gall bladder issues. She found this through muscle testing. In the past few weeks I have had migraines, nausea, dry heaving. Those symptoms cleared when I stopped eating wheat and this week I have had diarrhea, buning stomach when I drink tea, and fatigue. Can all of this be related to gall bladder? Also she suggested that I might be over producing estrogen which could lead to gall bladder disfunction. Does this make sense? Doctor: Hi there,Thanks for posting in HCM.Dry heaving, migraines, diarrhea and burning symptoms are not due to gallbladder issues.But yes high estrogen levels can cause gallbladder dysfunction.I hope that answers your query.Regards."
},
{
"id": 11144,
"tgt": "Prescribed Minscalp, betadip lotion for hair loss but having dandruff. How new hair will grow?",
"src": "Patient: Am having hairloss and no growth to my new hair ..only existing hair is growing .i consulted doctor he gave me minscalp ,betadip lotion and onablete vitamin and hair stimulising tablet.Previously i dont have dandruff and now i have the same.Please help me and i need growth to my new hair. Doctor: Hello. Thank you for writing to usYou seem to have androgenetic alopecia or male pattern baldness, as can be judged from the medication that has been prescribed i.e topical minoxidil solution.Androgenetic alopecia is characterized by gradually progressive thinning and hair loss from the frontal scalp and vertex. It is also associated with receding anterior hair line.Topical minoxidil and oral finasteride are approved remedies for androgenetic alopecia.Goal of treatment of androgenetic alopecia is to arrest or slow down further progression.New hair regrowth may be seen if the problem is not a very long standing.Treatment is long term. Noticeable improvement is usually seen after 3-6 months of regular use.Response to treatment is variable and varies between individuals.Dryness/ dandruff to minoxidil is a known side effect and is due to crystallization of minoxidil on scalp and also because the solution is available in an alcohol base which can cause dryness ans scalp scaling.Therefore I suggest you to add an antidandruff ketoconazole based shampoo along with minoxidil to take care of dryness/ scalp scaling.If I was the treating doctor I would have also added oral finasteride in addition to topical minoxidil.Finasteride and minoxidil together produce better results as compared to minoxidil alone.Regards"
},
{
"id": 149087,
"tgt": "Sharp pain in the temple area after a concussion. Frequent headaches. Need immediate medical attention?",
"src": "Patient: I suffered a mild concussion in June 2011 and have recently been having severe sharp yet brief pain in my right temple. I had hit my head during a fall right on the bone above my right temple and had pressure all about my head. These headaches came and went for about 3 months and then nothing... now they are back. Should I see my doctor asap? Doctor: Hi,Thank you for posting your query.These symptoms are suggestive of mild head injury and the symptoms should subside in a few days.You can take ibuprofen or diclofenac tablets for pain relief.At present there is no need to worry. However, if you develop vomiting,or double vision, or fits, you should see a doctor and get a CT scan of the head done.I hope it helps.Please get back if you require any further information.Best wishes,Dr Sudhir Kumar MD (Internal Medicine), DM (Neurology)Senior Consultant NeurologistApollo Hospitals, Hyderabad,For DIRECT QUERY to me: http://bit.ly/Dr-Sudhir-kumar My blog: http://bestneurodoctor.blogspot.com/"
},
{
"id": 200468,
"tgt": "Suggest treatment for pain in scrotum",
"src": "Patient: Hi, I recently had a pain in my scrotum. Whenever i sit, my balls used to ache. When i visited a urologist, he said that i have Varicocele. Now i am taking medication for Varicocele. I normally sleep in Foetus position. Is it safe if i continue my sleep in Foetus position or as my one friend suggested sleep in Starfish position with a pillow placed under my back as a support to my balls. My another friend suggested to dip my balls in Cold Ice water. Is it safe to dip balls in ice cold water to improve the pain. Please suggest Doctor: Thanks for asking in healthcaremagic forum Please do not listen to your freinds. You may use scrotal support if its aching while you stand for long time/walk. You can sleep as you want. All the best."
},
{
"id": 46428,
"tgt": "What is polycystic kidney and its treatment?",
"src": "Patient: rmy age is 76 suffering from pollycystic kideny first time diaganoses inthe year2000 with 25% kidney left and serum cretenine 2.5 presently my serum cretinine is 7.1 haemoglobin7% blood urea is 185 i am being treated by one of the renound doctor dr nirbhay kumar at kanpur Doctor: Hello and welcome to HCM. As an Urologist,i can understand your anxiety.You're having APKD(adult polycystic kidney disease)with kidney failure.That's seen in your creatinine report, with anemia and previous report.You'll need to see your nephrologist,with the following tests:1. urine routine,culture and 24 hours urine protein.2. blood routine,uric acid,electrolytes,LFT,RBS,PSA and platelets.3. ultrasound scan-KUB,with residual urine.4. isotope renogram.According to reports,expert treatment can be advised.Dr.Matthew J. Mangat."
},
{
"id": 120425,
"tgt": "What causes the pain in the hip after a ankle surgery?",
"src": "Patient: Hello. I recently broke my right ankle, and had surgery this week. All is well with the ankle, but my left hip is so sore, and I have started to feel painful if fleeting pin prick sensations. I assume it s because I have been sitting with my right leg elevated unnaturally and overcompensating for my non-weight bearing right leg , but would be interested in comments. Is there anything I can do, heating pad? Ice? Thanks. Doctor: Hi, Take tablet gabapentin 400 mg and nortriptyline 10mg in night for your pick and prick sensation, it could be due to nerve compression of various causes, apply diclofenac gel, hot pack. Hope I have answered your question. Let me know if I can assist you further. Regards, Dr. Jaideep Gaver, Orthopedic Surgeon"
},
{
"id": 176767,
"tgt": "Suggest remedy for recurring fever",
"src": "Patient: Dear Doctor, my son is 14 months old, he is having recurring fever for past 6 days, his platelet is 446000, but hemoglobin count is 8. I am giving him distacolor syp, mepthal p, ibugesic, for fever. After medication fever is gone for 5/6 hrs, but again his fever rises to 102. What treatment do u suggest? his dengue and malaria results are negative. Doctor: Hi...Fever of few days without any localizing signs could as well a viral illness. Usually rather than fever, what is more important is the activity of the child, in between 2 fever episodes on the same day. If the kid is active and playing around when there is no fever, it is probably viral illness and it doesn't require antibiotics at all. Once viral fever comes it will there for 4-7 days. So do not worry about duration if the kid is active.Paracetamol can be given in the dose of 15mg/kg/dose (maximum ceiling dose of 500mg) every 4-6th hourly that too only if fever is more than 100F. I suggest not using combination medicines for fever, especially with Paracetamol.Meftal-P contains Mefenamic acid which is a NSAID. It carries the risk of gastritis and renal problems in the long run. Please do not use Meftal-P. I have publication against the use of Mefenamic acid in a renowned journal. I am attaching the link, please go through it.medind.nic.in/ibv/t10/i5/ibvt10i5p453.pdfRegards - Dr. Sumanth"
},
{
"id": 210139,
"tgt": "How to cut hair for someone with dementia?",
"src": "Patient: My mother is 90 years old and bedridden. Her hair is getting very long and I was wondering how to cut the back of her hair. The top and sides i have already done. But I would now like to trim the back of her hair without any discomfort to her. She has dementia and every little thing upsets her from her normal routine. Doctor: DearWe understand your concernsI went through your details. I suggest you not to worry much. Dementia is a broad category of brain diseases that includes any disease that causes loss of cognitive ability (the ability to think and reason clearly) that is severe enough to affect a person's daily functioning. It must also be a worsening of functioning compared to how the person was previously. The most common form of dementia is Alzheimer's disease. Other common forms of dementia include: Lewy body dementia, vascular dementia, frontotemporal dementia, progressive supranuclear palsy, corticobasal degeneration, normal pressure hydrocephalus and Creutzfeldt\u2013Jakob disease. But whatever may be the form of dementia, hair cut is not related to the form, it is related to the patient. You and your people know the patient better than we here. You must have tried all conventional methods. A visit to the hairdresser, or a hairdresser coming to the house may be a better alternative. Many people with dementia continue to enjoy having their hair cut and styled and this can continue to be a pleasurable experience. It is like feeding a 2 year old child. You do so many roles to make the child eat. Discover some role and act it out and do the haircut.Hope this answers your query. Available for further clarifications.Good luck."
},
{
"id": 87545,
"tgt": "What causes abdominal pain after TB treatment?",
"src": "Patient: sir iam troubled by abdominal discomfort last to year before these 2 years i got the treatment of tuberculosis for 9 month.i feel tention in my stomach .sometime ifeel fever after treat ment of t.b my weight was 63kg but now my weight is 55. ifeel discomfort arround nabhi area Doctor: Hi.Thanks for your query and an elucidate history. The causes of abdominal pain after TB treatment are:Intestinal obstruction or adhesions can cause this or the rare problem of blind loop syndrome.I would advise you the following:Enteroclysis is a barium study of the intestines.Consult a Surgical Gastroenterologist to read the report and see if there is an active tuberculosis or just a sequel or complication. And get treated accordingly. You may need an abdominal surgery."
},
{
"id": 38341,
"tgt": "Suggest treatment for swollen tonsil",
"src": "Patient: Hello. My right tonsil is swollen where stuff comes out of it, my throat is irritated, I have a knot of back of neck & just noticed I have cracked lower back molar tooth. I have jaw/ear/neck pain & have low grade fever & feel awful. I was given Cephalexin 500mg 3X day (didn t work) & just finished Amoxicillin 3X day. I m on Nasonex & Sudafed and can t clear sinuses or throat drainage. I ve been in the worst pain ever & feels like my tonsil, jaw, ear, head & shoulder just want to exploid. Is it possible to have a abcessed tooth AND abcessed tonsil at same time?? (Since age2, my ear drum burst & I grew up with ear infections & pharengitis & still have those problems today. I m age48. Any help would be so much appreciated. Thanks for all your help. =) Doctor: HIWell come to HCMThis is not the abscess of any kind and so taking antibiotic for this condition wont work here, in my opinion such case need to be treated with Tab Acetaminophen and Tab Ibuprofen 400 mg three times in day and Tab Loratidine with pseudoephedrine three times in day drink more water, have hot drink, no stress, every thing would be fine soon, take care."
},
{
"id": 54188,
"tgt": "What are the symptoms of liver disease?",
"src": "Patient: I'm a care giver for a mildly retarded man (due to meningitist at 3 now 53) who is an alcoholic. His recent liver test result is 173. His primary wants him seen for an evaluation but he's refusing. What signs should I be aware of to determine the severity of his condition and when I must intervene and demand he be seen somewhere. Doctor: HiThank you for contacting healthcare magic.He needs to be evaluated now because of high liver enzyme. Since you haven't mentioned which liver enzyme is raised, I can tell u symptoms in general like *vomiting, *jaundice, *itching alongwith clay colored stool & dark colored urine (in case of obstructive jaundice),*fever*painHope my answer helps you."
},
{
"id": 11865,
"tgt": "Skin darkening, pigmentation. Using melacare cream, stopped usage. Alternative fairness treatment?",
"src": "Patient: im XXXXXX im using melacare since 3 yrs. if i stop using it my skin tone is going dark before using i was fair but after using it i became more fair but if i stop im loosing my natural clr also im becoming so dark please do advise me any cream or help me out of this.i dont want addict to this cream... please help cant take this cream Doctor: dear homan1977 please stop using cream like this or with similar composition for such a long duration else you might suffer from a condition called topical steroid dependant facies continous use of topical steroid for long period will cause the skin to thin, with dryness, itching, pimples like lesions, excess hair growth , sensitivity to sun -- the list goes on we can help you get rid of the addicton but first please let me know - when was the last time you used this cream?? regards dr amit"
},
{
"id": 85708,
"tgt": "What causes tingly face after stopped celexa intake?",
"src": "Patient: I have been on Celexa for about 2 months. I do not like it as I can not sleep (without Klonopin) and I am weaning off of it gradually. I took 20 mg for the majority of the time. Then I cut down to 10 for a week, the 5 mg. for 3 days and then I quit... It has been 3 days.. I am feeling a very tingly face today. Is there anything I can do to ease the symptoms of withdrawal? Thank you! Doctor: Hello,Celexa is a long-acting drug and if you have been taken it for a couple of months, you will need to very gradually step down before you stop it completely.In your case, you have jumped from 20 mg a day to 5 mg a day and then stopped completely within 2 weeks. This is a very quick dose change and patients may experience withdrawal symptoms in these cases.I would advise you to go in 10 mg for at least 2 weeks, then 5 mg for another 2 weeks and then stop it.I hope this answers your query. I remain at your disposal in case further medical assistance is needed.Regards,Dr. Antoneta ZotajGeneral and Family Physician"
},
{
"id": 14261,
"tgt": "Can a body rash be due to warfarin?",
"src": "Patient: Yes, please. My husband has a rash all over his body. He s on a gluten free diet and the dr. does t think it s from anything he has been eating. He is on warfin, could this be a problem? I read that a rash could be from warfin. What does the rash from coumadin look like? I m interested in warfin and coumadin. Doctor: hi,thank you for putting up your query on HCM.mam, i would like to inform you :--warfarin rash mostly occurs in fat abundant areas like thighs, buttocks, abdomen etc after weeks to months of starting warfarin therapy.-- rash from warfarin looks like necrosis of skin (which starts with a purpule bruise like rash, which over a period of time turns to bluish black with a red rim).-- rash from warfarin is rare. still, consult a dermatologist nearby to be sure.-- as the doctor has said, he may have got urticarial rash which can easily be controlled by proper diet and a soothing lotion like calamine lotion.-- as of now, you may give him tab levocetrizine twice a day.do consult a dematologist for further treatment.wishing you good health.thank you."
},
{
"id": 168139,
"tgt": "Suggest treatment for cough and congestion",
"src": "Patient: my 7 month old daughter has a very horse cough and is very congestant it seems to hurt her everytime she coughs. She had a chest x-ray done and it came out o.k., but that was on 5-18-11. Could there be something wrong now, when had x-ray done she didin t seem to be having pain when coughing. Just worried @ it. Doctor: Hi...Thank you for consulting in Health Care magic.Cough and cold are viral 95% of the times in children. For cold you can use anti-allergics like cetirizine and for nose block, saline nasal decongestants will do. Paracetamol can be given in the dose of 15mg/kg/dose (max ceiling dose 500mg) every 4-6th hourly, that too only if fever is more than 100F. I suggest not using combination medicines for fever, especially with Paracetamol.For cold you can use Cetrizine at 0.25mg/kg/dose every 12 hourly for 3 days.For nasal block, plain saline nasal drops will do, every 4-6th hourly to relive nasal congestion.Hope my answer was helpful for you. I am happy to help any time. Further clarifications and consultations on Health care magic are welcome. If you do not have any clarifications, you can close the discussion and rate the answer. Wish your kid good health.Dr. Sumanth MBBS., DCH., DNB (Paed).,"
},
{
"id": 65660,
"tgt": "Suggest treatment for a painful lump in the back",
"src": "Patient: Hi I have a small lump almost like a little ball I can feel it when I stand it hurts if I press it its tender not sharp or shooting pains also it's been here since last summer it hasn't bothered me except once in a while if I lay a certain way or touch it what might this be? Doctor: Welcome to health care magic. 1.It looks like a ganglion arising from the joint space of your spinal region.2.In this case one need to examine to help you in better way after seein and palpating the area.3.These ganglion's arise from joint space, tendon and nerves - usually painless and does not need any intervention unless it is symptomatic. 4. Investigations recommended in this case is an ultrasound of the region - that will evaluate the nature of the lump, its content, source and extensions. 5.MRI will be definitive. You initially get examine first and follow the physician. Good luck.Hope i have answered your query,any thing to ask ? do not hesitate to ask.http://doctor.healthcaremagic.com/doctors/dr-ganesh/62888"
},
{
"id": 29075,
"tgt": "Is a doctor consultation advisable for sweating, dizziness and vomiting?",
"src": "Patient: my significant other who is 38 has had a bug for almost 3 days now that has progressively gotten worse....he has tried sweating it out and sleeping it off. tried to keep fluids in however yesterday puked more than taken in and got dizzy and light headed. his cough has gotten worse especially today the 3rd day. he has major body aches and pains. is freezing. now he said he doesnt remember feeling this bad since young kid. back then he ended up in hospital and had siezure. I am thinking itis the flu but starting to feel he maybe should go in. 3 other people live in home but none are really in much contact but me and im ok. please help Doctor: Hello,Based on your description, it is likely the flu. The common cold would not present with such severe symptoms.Though some persons with flu can manage at home, if it is severe, it is best to see a doctor for assessment.In his case, with worsening symptoms, this would be advisable. It is very likely that with appropriate care, he would be fine.Hope I have answered your query. Let me know if I can assist you further.Regards,Dr. Michelle Gibson James"
},
{
"id": 30046,
"tgt": "What causes a round large rash on the shoulder while on Duromine?",
"src": "Patient: I have been taking duromin for about 1 week. I have felt well and no side effects that I know off. I have found a round rash on the top of my shoulder quit large , I have scratched it and it has not spread. Ive never had a rash like this before , its as big as a 10c piece. Could this be a side effect from Duromin. Doctor: Hi,Yes this could be drug induced but it more seems to be ring worm infection (Fungal infection) and I would suggest to try Miconazole gel first, if this fails to responds then anti-histamine along with steroid can be tried orally and best drug of choice would be Tab Levocetirizine 10 mg three times in day and Tab Dexamethasone 10 mg three times in day for at least three days, if still the lesion is there then it needs to be clinically examined.Hope this information helps. Let me know if I can assist you further.Regards,Dr. Akhtarhusain Shaikh"
},
{
"id": 136659,
"tgt": "What causes severe shoulder pain after an electric shock?",
"src": "Patient: I had gotten shocked from the neutral of a 277 vac circuit at work about a month ago. The electricity entered my right fore finger and exited my left thumb. The burn on the finger is almost healed the thumb was more severe and is still a little raw. I was experiencing severe pain in my left shoulder and intermittently in my left elbow to shoulder. The shoulder had the most pain in something they called the arch of pain. I have virtually no pain lowering my arm but still when I lift it. I have had xrays and currently go to physical therapy twice a week. Now I am experiencing intermittent burning and or shooting pain from elbow to shoulder and also every now and then a pain in the lower back radiating across my but. Since this happened at work and workman s comp is covering the medical care, Is there something else I should ask them to check or is there something else I should do like pay for a second oppinion.? Doctor: Hello, I have studied your case. Shock can lead to sudden jerk to spine leading to disc bulge.Due to compression of nerve root there can be pain in shoulder and fingers.Also there can be large disc bulge which is compressing over spinal cord.I will advise you clinical examination and accordingly investigation like MRI cervical spine or shoulder will help.For these symptoms analgesic and neurotropic medication like pregabalin can be started consulting your doctor.Till time, avoid lifting weights. You can consult physiotherapist for help.Physiotherapy like ultrasound and interferential therapy will give you relief.I will advise to check your vit B12 and vit D3 level.Hope this answers your query. If you have additional questions or follow up queries then please do not hesitate in writing to us. I will be happy to answer your queries. Wishing you good health.Take care."
},
{
"id": 225080,
"tgt": "Mirena inserted, getting bloody discharge along with cramps after sex. No sign of any infection",
"src": "Patient: hi, Ive had my mirena since august of 2011 with no problems till the past 2 monthes ive had bloody discharge once or twice a week usually after sex. i went in to the doctor and was tested for chlymydia which was negative and no sign of any other infection. Also ive started to get bad cramps during and after sex sometimes sharp pains. Doctor: Hi. Mirena causes irregular bleeding and sometimes spasm also. But with treatment it tends to settle after some time. I will advise you to get one pap smear done for post contact bleeding, to be sure of everything. Consult your Dr. for treatment for pain and it will be settled after few cycles.Mirena is a continuous progesterone releasing device and will not do any harm as such."
},
{
"id": 176653,
"tgt": "What causes severe behavioral changes during headaches?",
"src": "Patient: The twins are 17 years old and have been in fairly good health. The pregnancy was great besides the period of extreme stress when my husband lost his job and I took on an additional job during months 3-6 of gestation. I also suffered from a severe episode of the flu at the time. One twin has been complaining about headaches in the right side of his head, mostly clustered around the frontal portion of his brain. Whenever he experiences these headaches I note that he becomes unglued and his personality changes. After the headaches subsides he enters a state of exhaustion and shows little or no affect for days. His friend has also commented that during school he will suddenly show outbursts of inappropriate emotions. When the class gets scared he never understands why. I noticed an issue because he was found in a broom closet because he was terrified of his locker. Hes been under a lot of stress recently because he was working on a science project where he had to breed mice, but his female mice were not becoming pregnant. Since Ive noticed something wrong with him I feel I want to make sure my other twin Joanie is okay as well. At the age of 17 Joanie has not yet begun to menstruate and does not have any secondary sex characteristics. I noted that Joanie has always been a very athletic child, had very little body fat and has always preferred more male activities. I would like to know a possible diagnosis for my son Richie and daughter Joanie and if it has anything to do with why his science project wont work. Doctor: Hi dear,This may either indicate conditions like vascular head ache or neurological causes like subtle seizure,behavioral disorders etc.Kindly see a pediatrician for evaluation. He can guide in further proceedings.Hope this answer will help.Thank you."
},
{
"id": 6810,
"tgt": "Is having a follicle on 12th day, a positive sign of conceiving ?",
"src": "Patient: i had pcod bt after treatment nw on 12th day i have follical of 20mm on right ovary and 9.8mm on left ovary.... is this positive sign of concieving Doctor: Hi welcome to HealthcareMagic The above measurements in the report suggest that, there is growth in the size of the ovarian follicle... This is a good sign,.. Only if the ovulation occurs and fertilization takes place, then we can confirm pregnancy... continue the given medication I hope i have answered your question Thank you..."
},
{
"id": 132233,
"tgt": "How to relax a painful and restless leg?",
"src": "Patient: I have pain in my legs at night, I have been told that I have IT BAND SYNDROME, but I have also been told I HAVE RESTLESS LEG ,once I sit down at nite the pain starts and I cant get comfortable to sleep. It anges from burning sensations to actual pain. pain meds such as ultram or muscle relaxers do not help. Doctor: Hi with your kind of symptoms , night leg pain with burning sensation i am inclined to think in terms of some pressure on the nerves in the back- . Now do you get relief after sleeping on your tummy? If that is so then it is probably due to narrowing in the spinal canal. You may have to undergo a MRI to confirm the diagnosis"
},
{
"id": 27170,
"tgt": "Suggest remedies for Achalasia cardia disease",
"src": "Patient: I have the Achalasia cardia disease. I already made the endoscope dilation in August, but last two weeks ended up like before doing the operation...not able to eat and drink sometimes even liquids... I am waiting to have another try...What do you suggest in this case and even on what to eat / drink? Doctor: achalasia cardia with dilation will relieve symptoms for some time, it is not the final answer. at times repeated dilation is required. so please consult the gastro surgeon for further plan of treatment for good result. till then you need to maintain your nutritional status with what you can take orally. frequent and small feeds may help."
},
{
"id": 67146,
"tgt": "How can growing leg lumps be treated?",
"src": "Patient: My 5yr old son has red bumps on his legs and the seem to be spreading now to his trunk area. The first ones to shoe seem to be growing in size. They are not itching. He does not have a fever or otherwise seem to be ill. He recently had his second dose of chickenpox vaccination (10 days ago) and he did have a reaction to that so he is current taking an antibiotic for that. Doctor: Hi.Thanks for your query and an elucidate history. The bumps can be due to reaction to chickenpox vaccination or it appears more to be due a condition called ''pyoderma'' which is common in this age group. Continue antibiotic treatment. Send pus or secretions form the wound for gram staining and culture and sensitivity. You may please apply liquid Povidone Iodine to the affected parts so that the skin is made sterile and the spread may be avoided. Give multivitamin syrup too.Consult a Dermatologist just to confirm it is pyoderma and to get a prescription for the medicines."
},
{
"id": 211097,
"tgt": "What could lack of concentration be along with messing up stuff in a 42 years old individual?",
"src": "Patient: I AM 42 , AND I M HAVING A VERY DIFFICULT TIME , KEEPING THINGS STRAIGHT.. ONE DAY I WAS DRIVING ON THE WRONG SIDE OF THE ROAD, FOR ABOUT A MIN, UNTIL I REALIZED THAT THE CAR THAT LOOKED STRANGE ON THE LEFT WAS WERE IT WAS SUPPOSED TO BE, IM TRANSPOSING EVERYTHING I WRITE, I CAN HARDLY TYPE CORRECTLY WITH OUT HAVING TO GO BACK AND FIX IT ALL , IM WRITING BACKWARDS, AND I JUST CANNOT CONCENTRATE ON WORK, IM MESSING STUFF UP , THAT TOO SIMPLE.. IM REALLY GETTING WORRIED.. Doctor: Hello,Thanks for your question. The information you have provided is unfortunately not sufficient enough to comment upon.Are you are also suffering from any medical disease specially, Hypertension, Diabetes, Obesity or has a previous history of any head trauma? Is there any significant family history of medical disease like Hypertension, Diabetes etc?On the basis of symptoms you have mentioned, you should consult ideally a neurologist or a physician to rule out any brain related problem which can cause these symptoms. Please feel free to discuss further if you have any other question.Kind regardsVikas"
},
{
"id": 84628,
"tgt": "Will keflex interfere with my birth control pills?",
"src": "Patient: I have been on the birth control pills for 3 straight years now and had no problems. I started taking the medicine keflex for a rash and im taking it morning and night. I had sex without a condom tonight. Should I get plan b tomorrow or should I be ok? Doctor: Hi,Yes, Keflex can reduce the contraceptive effects of birth control pills. Keflex is an antibiotic which can reduce the absorption of birth control pills from the intestine and hence can reduce their contraceptive actions thus you are probably having a risk of getting pregnant. If you do not wish to become pregnant then you need to take Plan-B within 72 hours following the unprotected sex.Hope I have answered your query. Let me know if I can assist you further. Regards, Dr. Mohammed Taher Ali, General & Family Physician"
},
{
"id": 97535,
"tgt": "Suggest alternative medication for pain in neck",
"src": "Patient: I am a software Engineer & facing Neck pain from last 9-10 months. I am also facing slight vertigo due to this. The Orthopedics have diagnosed this is due to Occupational-posture issue & issue is with muscles. Which product of Santulan Ayurved can cure this? How to apply that product -- may be oil, tablet or in any other form? Doctor: hai,As your problem came only because of your working nature.santulan auyrved didn't have any product for neck pain in the form of tablet and oil.General ayurvedic preparation- narayana tailam and pinda tailam fermentation in neck and surrounding area will give a relief. most of ayurvedic drug company will prepare above mentioned oil.whatever oil or oral medication will give you a temporary relief unless you change your mode of working pattern.don't keep your attention towards the system for a prolonged time. have neck movements like (forward bending,backward,neck rotation clock and anti clock wise)frequently every 20 to 30 minutes.fro every 40 to 60 minutes have a gentle walk which will relax your spinal bone and muscle.prolonged neck posture in a particular direction will lead to neck muscle pain.this type of exercise will keep you away from neck pain.Thank youHope I answered your query."
},
{
"id": 171298,
"tgt": "What causes knee pain in 4year old?",
"src": "Patient: I have a 3 almost 4 year old son who woke up yesterday morning crying and complaining of knee pain in his left lef. Since then he has been refusing to stand on it or walk. There us no obvisious swelling, deformity or brusing. He willmove around inand bed andor on thethe couch but.... besides that little bit of pressure will not do anythingu and keeps it bent up and wi unbend it with assistance and encouragement but then returns to hag itgain. He has an appointment today to have it looked at but was wondering about any other opinions . Thanks Doctor: Hi, This could probably be septic arthritis or muscular pain due to trauma. In my opinion, an ultrasound knee and aspiration of synovial fluid with x ray knee should be done to make a clear diagnosis. Treatment will depend on above reports, probably antibiotics, analgesic for 1 week. I hope this has helped you. Take care."
},
{
"id": 100484,
"tgt": "How to treat severe reaction of mosquito bites?",
"src": "Patient: I have quite severe reaction to mosquito bites. They itch initially then turn into two inch diameter hard swollen circles getting progressively itchy. They then get a head which looks like a chicken pox pustule. I have tried most mosquito sting relief creams lotions but do not get a lot of relief. They eventually go down after 4/5 days. Can you suggest a cream / lotion with a more instant relief. Thank you Doctor: HI, thanks for using healthcare magicThe response to insect stings can vary from one person to the next.You should try to reduce the bites. This would mean checking your environment for any containers that may be collecting water. Even small still water collections can act as breeding ground for mosquitoes.You should also apply a mosquito repellent and consider screens on the windows of your home.An oral anti histamine may work in addition to your topical creams to help reduce your response.Most of the topical agents are similar.I hope this helps"
},
{
"id": 124567,
"tgt": "What is the sudden bump on the achilles tendon?",
"src": "Patient: Hi! I have a hard painless bump at the base of my achilles. I m a regular runner logging over 30miles per week. The bump looks like a bony protrusion at the back of the heel. It doesn t hurt at all, but I m kinda worried it might be the beginning of something bad. Another thing, both my achilles tendons have lately felt somewhat sore and stiff. Been doing regular stretching routine for calf muscles and otherwise. So I am really bummed out about the sudden appearance of the bump. Doctor: Hello, Consult an orthopaedician and get evaluated. An MRI scan is required for further evaluation. Hope I have answered your query. Let me know if I can assist you further. Regards, Dr. Shinas Hussain, General & Family Physician"
},
{
"id": 172067,
"tgt": "Is it ok if the baby lies upside down for a long time ?",
"src": "Patient: my baby likes a lot to spend some time upside down. I mean When he is starting to foss. I lay him on his back and let his head hanging off the side of the bead. Is it ok? For how long? He is three months old and he likes it a lot.. Thank you very much Doctor: Did you mean upside down or prone? If he prefers to be prone, and you make sure that his nose isn't getting pinched by the bedsheets and pillows, then there is no problem ... except that you should be near him when he is lying on his stomach. There will be no issues with his digestion or anything else. Best wishes,Dr. Taher"
},
{
"id": 94546,
"tgt": "Diarrhea, frequent urination and discomfort in the lower abdomen. Appendicitis?",
"src": "Patient: Hello.. I have a warm sensation in the lower right side of my stomach , I ve been doing some reading and see that it could be appendicitis . The pain isn t extremely discomforting, it s a bit frustrating if anything because I m constantly shuffling around to attempt to make it feel better. I ve gone to the bathroom with diarrhea , thinking it was the food, but this pain just started happening after the food as well. I ve noticed I ve been urinating considerably more then usual as well. Hope to hear back soon, thanks! Doctor: While it can be Appendicitis from symptoms was there any fever? Any pain on right side back radiating to the right side of stomach?Has Urine test been done to rule out Urinary Infection or Stone on right side? If Urine is normal,CLinical Exam by Doctor and Ultrasound may pick up Appendicitis specially if the total count on blood test is very high Do get back if findings dont confirm the diagnosis Best wishes for early recovery"
},
{
"id": 170159,
"tgt": "Suggest treatment for loose motion",
"src": "Patient: my 2 yrs baby boy is suffering from loose motion 10-12 times since last 5 days. before 2 days his motion became reddish. he drink a red drink one time that is energile( mix fruity)...but still he is doing reddish loose motion 5 times a day...i am giving him entamizole syrup....should i worry abt the red color??? Doctor: how are the stools. if it is having mucus and red in color it may b dysentery. get stool routine done. if it shows blood mucus or plenty pus cells then antibiotic needs to be started"
},
{
"id": 63273,
"tgt": "Suggest treatment for a swollen lump on the armpit",
"src": "Patient: Yesterday morning I woke up with a very sore lump in my left armpit. There is a pea size or smaller lump under the skin that is sore and underneath a sort of swollen puffy sack. I started my period yesterday. I m very concerned. I am a healthy 24 year old. Doctor: hi.inflammatory changes or lymphadenopathies are common if you have a recent infection. if it is in your armpit tho, it could be hidradenitis suppurativa. this lesion usually comes from a folliculitis following plucking or shaving of your underarms. it is best if you consult with a physician and have a physical examination done. if drainage is needed, you'll be referred to a general surgeon for excision and drainage. antibiotic treatment will be prescribed accordingly.hope this helps.good day!!~dr.kaye"
},
{
"id": 11764,
"tgt": "Brown patches on nose, both sides of upper lip, spreading to forehead. Using carofit, sunban lotion. Suggestions?",
"src": "Patient: I am 50 yrs old lady.Last 3yrs some brown patches seen on my nose and on both sides of upperlip. Now its spreading to my forehead .I consulted a dematologist he did Microdermabrasion on my nose and upperlip. all came in vain.Now I am using carofit during night and a sunban lotion during night.Dr please advise me. I am so worried. Thank you Doctor: Beena, recurrence of spots after Micro dermabrasion indicates that the triggering cause is still there. Generally these spots are triggered by hormone estrogen, melanin or sun. Chloasma spots occer due to age. Sunscreen formulations may help. Hydroquinone topical ointment at night can be used over the spots. It decreases melanin production. This helps only if the pigmentation is superficial & not deep. Pigmentation may reccur if it is discontinued. Monobenzone, a derivative of hydroquinone should be avoided as its action is irregular & it may cause permanent blanching. Apply almond oil on the spots for 10 minutes before you wash your face."
},
{
"id": 206393,
"tgt": "What causes fits of rage?",
"src": "Patient: I have a question about my six year old daughter. Occasionaly, she will have fits of rage where she starts hitting, kicking, biting, scratching, and fighting in general. She won't make eye contact and does not seem to be mentally there. She will repeat herself over and over and will not change her answer despite what you ask her. She has done this since she was born in some form or fashion. It is not her personality to be this way, and she does not remember why she acted that way or why. She doesn't even remember what she does during what I have come to call \"episodes\". She normally gets a blank stare but completly goes into fits over nothing. The only thing I have really picked up, is that she will almost always have one of these \"episodes\" when she is tired. Any ideas on what could being going on to make her do this? Doctor: DearWe understand your concernsI went through your details. I suggest you not to worry much. As you say, such episodes could happen when they are tired and out of synchroniation. But in this case, this seems to be a bit more than normally expected. Attenton Deficit Disorders, Hyper activity, etc triggers irritation and anger and rage follows. In every possibility this child needs psychological evaluation with the help of psychometric tests. Please consult a psychologist. If you still need my help, please describe the whole problem in detail and post a direct question to me. I shall definitely help you with psychotherapy techniques to over come your problems.Hope this answers your query. Available for further clarifications.Good luck."
},
{
"id": 183232,
"tgt": "Suggest remedy for the fallen part tooth",
"src": "Patient: Hi, may I answer your health queries right now ? Please type your query here... PLEASE HELP! Part of my tooth fell out on the lower right side of my mouth months ago-no insurance or finances to pull the rest out. There is a piece that was left with a point that has been rubbing against the right side of my tongue and irritating my tongue-making it quite sore and causing some pain and bleeding. The right side of my throat and into the right side of my head aches so much at night I am finding it hard to sleep. I think I am infected and could die. I am alone much of the time so I am frightened. Also I had a cyst about the size of a nickle removed 12 years ago from the center of my chest. It was benign. Several years later it grew back and it is bigger now. Please tell me what I need to do about these problems and please suggest a doctor or a hospital in New Orleans where I can go to get some type of inexpensive help. Doctor: Hello, thank you for consulting with healthcaremagic. When a part of tooth is fallen the rest of its cusp becomes sharp, that is why it is causing soreness of tongue. You have to get it extracted, as it can cause problem in future, so better to visit a dentist and get it removed. Hope it will help you."
},
{
"id": 34282,
"tgt": "When should anti-rabies and anti-tetanus injection be taken after being bitten?",
"src": "Patient: Let's say the dog that bit me is positive with rabies. If I am not mistaken i need to get a shot immediately of anti rabies which is followed by 6 shots within 28 days. do i need to have anti tetanus?.. how many shots? is it okay to have shot the same time with anti rabies? thank you Doctor: Hello,Welcome to HCM,Rabies is a disease which is transmitted by dog bite which is 100% fatal but it is 100% preventable by proper and adequate treatment.If you are bitten by a suspected rabid dog, according to WHO categories it is categorized into Cat III, which requires following treatment1.Wash the wound thoroughly with soap and water.2.Active immunization with anti rabies vaccine on days 0,3,7,14 and 28.3.Passive immunization with rabies immunoglobulin around the wound.4. Inj TT, 0.5 ml, IM.Thank you."
},
{
"id": 106027,
"tgt": "I am suffering with allergy. any medicines for my problem",
"src": "Patient: Community Questions on I am getting cough due to dust elergy I am getting cough due to dust elergy Recently we have shifted house I am getting cough due to dust elergy little bit cold also there. Please advice medicine. I am getting tired also. evrything is over. i need to settle now Doctor: hi, welcome to hcm thanks for the query since you have dust allergy the best thing is to avoid the allergen i.e dust during the episodes of allergy you can take antihistaminics like levocetrizine,and some antinflammatory drugs which will improve your symptoms take care"
},
{
"id": 198377,
"tgt": "Should i be concerned about the scar tissue with puss/semen discharge after vasectomy?",
"src": "Patient: It s been 4 days after my vasectomy and I noticed in the shower that it was firm around the left set of stitches. Almost like scar tissue. I squeezed it like a pimple and puss or semen came out. Is it semen or puss? I ve been reading a bit on possibilities but still concerned. Should I be concerned? Doctor: HelloThanks for query .You have undergone vasectomy 4 days back and has noticed white discharge fro the wound .During vasectomy Vas difference (a tube) that carries sperms is ligated at both the ends it is unlikely to be semen ..It is mostly a puss .However Get it confirmed by microscopic examination and culture of a whitish material that you have noticed .This will help to allay your anxiety and help in taking a appropriate antibiotic to combat infection.Do not worry taking antibiotic and anti inflammatory drug will resolve the issue .Dr.Patil."
},
{
"id": 89682,
"tgt": "Why having abdominal pain after colon removal?",
"src": "Patient: I had a colectomy 11 months ago-- About 3-4 inches of the colon were removed to take out a flat sessile polyp. I have a four inch incision above my navel and a one inch incision on the right side of my abdomen. I am a weight trainer and returned to the gym about 10 weeks after the procedure. For the past three months I have either reoccurring or constant pain in several places in my right abdomen. The surgeon says no sign of hernia and offers no other possible cause. The only time it doesn t hurt is when I m working out . What could be the problem???? Doctor: Hi.Go for CT scan , Enteroclysis and endoscopies and if required for Diagnostic Laparoscopy or laparotomy. This will hep you to get a reason and a proper treatment. There may be associated IBS as you are not getting pain when you are working."
},
{
"id": 87561,
"tgt": "What causes intermittent pain in abdomen along the navel line?",
"src": "Patient: yes, I have been experiencing a sharp pain that ebbs and flows in the middleof my abdomen around the navel line. I am 62 and wonder why it comes and goes inn the same minute - not long duration - maybe 10 seconds, gone for 1minute and then repeats. been going on for the afternoon and evening. Doctor: Hi! Good afternoon. I am Dr Shareef answering your query.If I were your doctor, I would after a detailed clinical assessment of your vitals (pulse rate and blood pressure) and abdomen along with the cardio vascular system, might go for some basic tests like a CBC, a blood sugar, a serum amylase and lipase, an ultrasound of abdomen to know the status of organs like gall bladder and pancreas, a base line ECG for you, with X ray chest being an option if need be. Till then , I would prescribe you a proton pump inhibitor and an anti spasmodic drug for a symptomatic relief and would also advise you to keep a srobitrate handy on consultation by your clinician.I hope this information would help you in discussing with your family physician/treating doctor in further management of your problem. Please do not hesitate to ask in case of any further doubts.Thanks for choosing health care magic to clear doubts on your health problems. I wish you an early recovery. Dr Shareef."
},
{
"id": 125775,
"tgt": "What causes pain in the heels?",
"src": "Patient: I started to feel pain in my left heel earlier this afternoon. I little numbness up my leg. The pain has gotten worse. Haven t had time to rest it yet. Felt a slight pain begin in my right heel begin but has subsided since. I ran 4 miles yesterday with no problems. Any ideas as to what it could be? Doctor: Hello, It could be due to conditions like plantar fasciitis. As a first line management, you can take analgesics like Paracetamol or Aceclofenac for pain relief. If symptoms persist better to consult an orthopedician and get evaluated. In that case, steroid injection to foot might be required. Hope I have answered your query. Let me know if I can assist you further. Regards, Dr. Shinas Hussain, General & Family Physician"
},
{
"id": 173239,
"tgt": "Does light brown, mushy stool indicate lactose intolerance?",
"src": "Patient: My 13 month old has been having light brown, mushy stool for about 3 weeks now. We've been trying to get him to drink milk for the past month & a half & he's refused So he's been eating a lot of baby cereal with milk, whole milk yogurt, and cheese. Does his stool indicate he could be somewhat lactose I tolerant? Doctor: Hi...by what you quote I feel that it could be a cow's milk protein allergy.I went through the history points you have provided - I was expecting that there should have been a history of cow's milk and formula feed (this too contains cow's milk protein) given to the baby. I feel that your baby is having cow's milk protein allergy. Unusually babies grow out of this sort of allergy by 1 year of age. My suggestions for you - 1. Mother should go off cow's milk protein completely. This means that you should avoid consumption of anything and everything related to cow's milk - like - milk/ curds/ ghee/ butter milk/ chocolates/ biscuits/ ice creams etc. Even while buying commercial food products, you need to see the ingredients and if they contain milk - do not consume them.2. If this is followed scrupulously - the baby will stop having diarrhoea in another 3-4 days and then start gaining weight too.I request you to keep me posted about the recovery of the baby and follow of the case.Regards \u2013 Dr. Sumanth"
},
{
"id": 160688,
"tgt": "Suggest treatment for milk allergy in a child",
"src": "Patient: My old-year old boy had cows milk as a drink for the first time last weekend and several hours later developed a rash across his face, mainly on his nose and hairline and behind his ears. It didn t seem to bother him. The next day he had it on his body, front and back but not on his arms and legs. It looked a bit like a heat rash, was flat and pale. It all cleared up 24 hours after it appeared. Could this be an allergic reaction to milk? He has been having cheese, yogurts and cows milk on cereals since 6 months with no reaction. Doctor: Hi, If he was already using cow milk in weaning diet without any issues, the present rash is unlikely a reaction to milk. It might be an unrelated rash, like viral, which is unlikely allergic (as it is flat and non-itchy). Anyhow, since it has already disappeared and was not hurting him, we need not worry. We can evaluate further in case the rash returns or he becomes febrile. No need to withhold cow milk as of now.Hope I have answered your question. Let me know if I can assist you further. Regards, Dr. Muhammed Aslam T. K., Pediatrician"
},
{
"id": 143622,
"tgt": "Suggest treatment for pseudotumor cerebri",
"src": "Patient: My wife was diagnosed with pseudotumor cerebri by mayo clinic. She had been prescribed IV vials of Benadryl 50mg high potency to take im every six hours to help with the headaches and vision problems. They also had her take IV promethazine 25mg as needed. We are unable to return to mayo so my wife is unable to get updated scripts for this medicine that we could fill in Illinois. Is there anything you could suggest. We have everything but these two medicines. Doctor: Hi, Welcome to HealthCareMagic.com I am Dr.J.Mariano Anto Bruno Mascarenhas. I have gone through your query with diligence and would like you to know that I am here to help you.Pseudotumour Cerebri can be managed by1. Drugs like Diamox, Lasix2. Optic Nerve Fenestration3. Sub temporal decompression4. LP ShuntHope you found the answer helpful.If you need any clarification / have doubts / have additional questions / have follow up questions, then please do not hesitate in asking again. I will be happy to answer your questions. In the future, for continuity of care, I encourage you to contact me directly in HealthCareMagic at http://bit.ly/askdrbruno Best Wishes for Speedy Recovery Let me know if I can assist you further.Take care."
},
{
"id": 75648,
"tgt": "Suggest treatment for pain in lower rib radiating to chest and back",
"src": "Patient: Age: 28. Weight: 65. Height: 5 ft 8. Medical history: persistent catarh. I have a dull aching pain that seems like a muscle strain. It usually starts around lower right rib but spreads to the chest, stomach and back. Sometimes, I also get this funny burning sensation around my chest area. Lastly, I've been feeling dizzy lately. Doctor: Hi welcome to HCM....Here the area of pain you are locating is suggestive of gall stone or mild cholecystitis like condition ..For that investigate with USG ....USG will also rule out if gastritis or any other inflammatory condition there leading reffered chest burning ...Take omperazol capsule on empty stomach before meal for two week....Avoid heavy fatty meal ...Avoid excess spicy foods ...Depending on physical examination further investigation done and causative treatment done.If no any cause found pain can be from muscular strain as you are suspecting ...Take care ."
},
{
"id": 199534,
"tgt": "What could be the reason for pain during ejaculation in a 74 yr old male?",
"src": "Patient: I am a 74 year old healthy male and for the past several months I have pain DURING ejaculation. I would describe the pain as moderate. My urologist has had me on two antibiotics and the situation is no better. 14 days on Levofloxacin 500mg and now half way thru a 37 day series of Bactrim DS 800-160. Ideas? Doctor: HelloThanks for query.You have pain during ejaculation .This is mostly due to Chronic Prostatitis .Get your Semen examination and Semen culture done to find out the organisms causing this infection so that you can take appropriate antibiotic as per culture report to combat infection.Discuss with you Urologist about the other modality of treatment like daily prosatatic massage.This helps to drain out all the secretions from prostate and there by helps to control infection.Dr.Patil."
},
{
"id": 90021,
"tgt": "Why do I have severe pain in lower abdomen having undergone IUI?",
"src": "Patient: hi doctor, i had a positive pregnancy test with IUI done and then visited doc so she confirmed it as 6 1/2weeks and in scan they told single intrauterine gestation , and no evidence of gestational sac yet so repeat scan again after 2 weeks ............. then on the 7 weeks day i have very severe pain in my lower abdomen and consullted doc ... they scanned and confirmed it was ectopic pregancy in the left tube and got ruptured so they went for emergency abortion ............ now my doubt is what abt the one declared in 1st scan single intrauterine gestation ...... was i having twins ........ one went to uterus and was the other in the left tube ................ plz confirm and tell me whether i am still pregnant with a baby in my uterus or no ??? Thanx in Advance Doctor: welcome to Health care magic.1.Even the first scan didn't say that you have a line fetal pole or cardiac activity - as by 6 1/5 weeks one should see them. But not seen - it could be just a collection or blighten ovum. 2.And in procedures like IUI there are many chances of twin pregnancy also, you said you had underwent emergency abortion was done, no scan was done at that time. 3.If not done , to get your mind free, you can get a ultrasound scan get done which will gives you a peace of mind.4.Get an appointment and get the scan done.5.Hope it helps you.Anything to ask ? do not hesitate. Thank you."
},
{
"id": 32436,
"tgt": "How lymph node TB with liver infection be concurrently treated?",
"src": "Patient: hi my mother is diagnosed with lymph tb, she is started using atk4 medicines for one month, suddenly she got liver infection, her bilirubin and sgot and sgpt levels are too high, they stopped her medication except rmp, etm, and few antibiotics, now they started with inh and her sgot and sgpt levels are elevated within one week, now they stopped inh, how long this may continue , she is 65 years old Doctor: Hi, dearI have gone through your question. I can understand your concern.She has lymphnode tuberculosis. But INH is hepatotoxic drugs and she can not tolerate INH. INH causing liver damage in her. SO she should take 2nd line or 3rd line anti tuberculosis treatment. Consult your doctor and take treatment accordingly.Hope I have answered your question, if you have any doubts then contact me at bit.ly/Drsanghvihardik, I will be happy to answer you.Thanks for using health care magic.Wish you a very good health."
},
{
"id": 213870,
"tgt": "I am having dizziness, stomachache, headache. I was diagnosed with UTI",
"src": "Patient: Hello Dr. I am 33 yrs. old and female I have 1 baby, normal delivery. I was diagnosed with UTI, inflamed left kidney and both my kidneys have crystals...I felt dizziness , headache , sleepy, stomachache, feeling of hungry even i m full but not bloated , more thirsty, feeling of vomit. And two weeks ago i felt cramps im uterus. I weigh 48 kg. I feel not motivated and tired. I am living in hot region. What s happening to me? Doctor: HELLO, , SINCE YOU ARE FEELING A SERIES OF SYMPTOMS, THESE POINT TO SOME SORT OF INDISPOSITION. THIS MEANS YOUR BODY WILL OVERCOME IT ITSELF. U HAVENT MENTIONED ABOUT ANY FEVER, SO IF LETER ON U START WITH TEMPERATURE(MEANS OF SOME INFECTION), WITH ALL THESE SYMPTOMS, THEN U NEED TO UNDERGO SOME BLOOD TESTS ie CBC, URINE TESTS ETC. AND TAKE THOROUGH TREATMENT. THE CRAMP MIGHT BE DUE TO CRYSTALS ASLO. IN SUCH CASE YOU NEED VISIT THE DOC. I HOPE I HAVE BEEN ABLE TO ANSWER YOUR QUERY. WISH YOU GOOD HEALTH"
},
{
"id": 220133,
"tgt": "What are the symptoms of potential pregnancy?",
"src": "Patient: HiToday is the 8th day after taking 2 ipills together. I hav experienced spotting today only for few hour, the blood is abit darker than usual period. I have also being feelin dizzy past 2 days. Could you please tell me if i am pregnant or is this implantation bleeding or is the side effect of ipill. Please reply as early as you can Doctor: Hello dear,I understand your concern.In my opinion the spotting might be sideeffect of ipill.The ipill is 95% effective if taken within 72 hours of unprotected intercourse.If you had taken within stipulated time the chances of pregnancy are less.The ipill causes hormonal imbalance leading to irregular bleeding spotting and the period might come early or late.So the spotting might be mostly the ipill sideeffect.The implantation bleeding usually presents as spotting around the date of expected period.The first sign of pregnancy is missed period.And all the symptoms of pregnancy like nausea,vomitings,increased urination,sore breasts start 2 weeks after missed period.Dont worry.Avoid stress.Wait and check for the period.As you have taken 2 ipills there is chance of more hormonal imbalance as compared to single ipill.So there might be more chances for irregularity in periods.Overall the chances of pregnancy are rare.I suggest you to restrict ipill for real emergency situations and not to use frequently to avoid complications.Hope this helps.Best regards...Dr .Srilatha"
},
{
"id": 58782,
"tgt": "CT scan shows hepatic lesions typical of hepatic hemangiomata. Precautions?",
"src": "Patient: Hi,I am a 43yo male and have just had a ct scan in which it showsi have 3 lesions, 1 right heptaic solid hypoechoic lesion measuring 3.3mm x 2.5mm and 2 hyperechoic lesions right lobe measuring 1.0 x.8mm and 1.9 x 1.6mm hypoechoic lesion with peripheral achogenic rim.Post contrast scans were performed, durind the portal venous phase. All thrre lesions show a similar appearance being hypodense with lobulated peripheral infilling during the portal venous phase. All become isodense with liver on delayed phase scans. Appearance are typical of hepatic haemangiomata. I have now been advised to have another ct scan in a years time as well as a blood test in 6 weeks time. What are your thoughts on my precautions? My GP has advised me to stop drinking alcohol until my blood test is done to see if there are any improvements, I consume 8std drinks per week. Doctor: Hi,Thanks for writing in.Going by your report findings, lesions measuring 3.3 x 2.5 mm, 1.0 mm x 8 mm and 1.9 x 1.6 mm are very small. Also words hypoechoic and hyperechoic are terms used in ultrasound scan. Please re confirm size and words used in your report.Even if we take the findings of post contrast scan to hold good, you really need not worry about these lesions. Hemangiomata of size mentioned by you are always benign.A blood liver function test can be done and ultrasound scan can be done after 6 month intervals. If felt necessary (increase in size and number) then a CT scan can be done.Alcohol might be temporarily restricted to get proper blood test results. Please discuss you blood test results with your doctor.Hope this helps."
},
{
"id": 212327,
"tgt": "Suffer from seasonal depression, on zoloft. Need anti-depressant that works well with 12mg suboxone",
"src": "Patient: i am trying to find an anti-depressant that works well with 12mg suboxone..i am currently on 150mg zoloft, but the side effects are extreme tiredness..could sleep all day. i spoke with my primary dr.on thursday and he increased zoloft 50mg because i have been feeling extremely depressed. but i see on drug interaction page that there is a moderate interaction..i usually only suffer from seasonal depression...any suggestion would be greatly appreciated as the way i am feeling tired depressed and lack of energy will lead me back to opiate use as this was the reasons that i enjoyed them in the first place..thank you jessica Doctor: Hi, thank you for using health care magic. Zoloft is very good anti depressant drugs that has very less drug interaction with suboxone. Your extreme tiredness may be due to 1) Opiate withdrawal 2) as a symptom of depression 3) as a side effect of Zoloft 4) Vitamin B12 or Vitamin D Deficiency You should consult a psychiatrist for detailed evaluation and proper case management. the treatment depends on the cause of treatment. For seasonal affective disorder, many times tricyclic anti depressants like clomipramine are very good. I hope you will be fine soon. thanks Dr Nehal Shah M.D.PSychiatrist"
},
{
"id": 181710,
"tgt": "Is there a home remedy for teeth tingling?",
"src": "Patient: I have filling on my far back tooth, lower. I have tingling on the fill area like metal is touch aluminum or metal touch metal. Anyhow it fill like I have a batteries in my mouth. So uncomfortable, what so I do? I don t if this feeling is normal or is a bad thing. Doctor: Thanks for your query, I have gone through your query.The tingling sensation and the altered taste can be secondary to the presence of metal restoration. Nothing to be panic, consult a oral physician and get the teeth restored with composite.I hope my answer will help you, take care."
},
{
"id": 177444,
"tgt": "Suggest treatment for fever in a child",
"src": "Patient: Hi thank you. My 7 years old has had a low fever for 10 days..he turns pale some time during periods of time and get emotional from time to time saying he feels tired. But then gets back to normal behavior. But low fever of 99 to 99.6 stay the same through the day. He has low appetite but eats. He has developed a cough since yesterday. What could it be? Should I be concerned. It has had happened in the past. When I mentioned to his pediatrician my concern last year, he listened to his heart and said it was nothing. Should I persue and get a second opinion or is it normal for a child to get low fever for a while from time to time? Thank you. Doctor: It is quite normal for children to suffer from fever from time to time and most often it is due to a viral infection. Last year the doctor had probably found the chest (lungs) to be clear (not the heart). You can definitely go for a second opinion but all I would say that you need not be concerned. Just give paracetamol to bring down the temperature. You may give steam inhalation which would be helpful in diluting cough if any."
},
{
"id": 118187,
"tgt": "Should i be concerned about the blood clot in my arm after dialysis?",
"src": "Patient: Hi. I m Charles. I have a fistula in my righf arm and ive had it since 2009. Anyway I havent used it since January. But the other day it stopped working. Now its been clotted before but I had emergency surgery to open it back up but im on peritoneal dialysis now. Im just womdering if its safe for me to be walking around with a clot in my arm, or will I be ok,...? Doctor: HI Thank fro askin to HCM I really appreciate your concern and I think you must be talikng in the sense of shunt that must have been palced in your arm and which is being blocked, if this is correct then just do nothing because such shunt usually get blocked soon, it would be better to remove the shunt if possible even if you do not remove this then also it does not matter, take care and have nice day."
},
{
"id": 183720,
"tgt": "Should Meftal- forte tab be taken for toothache?",
"src": "Patient: Hello doctor. I have been havin a toothache since few days. I ws told few months back that the tooth needed a root canal. A temporary coverin ws done but within days it came off and exposed tooth since then. I ws advised meftal - forte tablets to take for pain by a chemist. Please advice ASAP if i should take to subside the pain for now. Within 2 or 3 days i would be visiting a good dentist. Thank you! Doctor: Hello,I am glad to assist you. The temporary filling placed would be expected to wear away if only placed to cover the tooth while expecting you to return for a root canal therapy. The filling needs to be removed to enter the center of the tooth and clean out the canals to complete the root canal. You will have a temporary filling until the canals are sealed and you can plan restorative treatment with your dentist. An antibiotic such as Amoxicillin may be necessary if pain and swelling is present. This will help control the spread of inflammation from the infected tooth. I usually recommend taking a combination of two Advil and two extra strength Tylenol to control a more severe pain until you are able to visit the dentist again.Thank you for your inquiry. I hope you will feel relief soon."
},
{
"id": 26705,
"tgt": "What causes difficulty in taking deep breaths and tingling and numbness in fingers?",
"src": "Patient: Hi, For the past few days now I have been finding it difficult to take deep breaths through my mouth, I also get tightness in my chest from time to time and also tingling and numbness in my fingers. I also have balancing issues from time to time also any feedback will be highly appreciated. Thanks. Doctor: Hello. Thank you for your question and welcome to HCM. I understand your concern. It is almost impossible for anyone, healthy or not, to constantly take deep breaths. It is one's organism that do not allow to do this, because of the organism balance disturbance that results from hyperventilation. Well, when this becomes worrisome to \"Why can I not take deep breaths\" an anxiety feeling is what comes after, and can also produce the tingling and numbness. My opinion is that you should try really hard not to give much attention to the deep breaths and/or try thinking about something else, when you capture yourself thinking about deep breaths. As for the loss of equilibrium, more information would be welcome, so I could precisely assess this symptom. Anyway, if it is coming after your briskly switch positions, such as from lying down to briskly stand up, the quick redistribuition of fluids in the organism, can cause dizziness and loss of equilibrium for few seconds - it is called orthostatic hypotension, not harmful, and is triggered by a sudden drop of blood pressure and attributed to the fluid restribuition explained above. It also can come from problems in your inner ear, a small organ responsible for our balance. If you have a history of life with ear infections or ear problems, then I recommens you to consult an ear-nose-throat (ENT) doctor for further evaluation and treatment. I hope I was helpful with my answer. Wish you a good health. Regards. Dr. Meriton"
},
{
"id": 122093,
"tgt": "What causes bruising in calf area?",
"src": "Patient: In the calf area. I have this bruise like feeling in the upper area on the tendon/ muscle. I can feel a slight bump (can t be seen). but the area near this bump has a bruised feeling when touched. I can feel it some (like a tightness if I stretch my calf). I am not an athlete or out of shape Doctor: Hello, The symptoms seem to be related to a muscle contusion. I suggest resting. Protect your injury from further damage by stopping unnecessary physical activity. Apply ice to your injury to reduce pain, swelling, and bleeding. Wrap the bruising with a bandage to provide additional support. Use anti-inflammatory medications such as Ibuprofen to relieve the pain. Hope I have answered your query. Let me know if I can assist you further. Take care Regards, Dr Dorina Gurabardhi, General & Family Physician"
},
{
"id": 77241,
"tgt": "Suggest treatment for embedded piece of nail under skin on chest",
"src": "Patient: I'm a woodworker. A few weeks ago I was cutting on a table saw and the blade hit a small nail and a small piece hit me in the chest. I didn't think it went in but it did. I can still feel it under the skin but it feels like it's gone deeper. The material was steel and it was clean and just out of the box (no rust). Will it dissolve or should I be concerned about something serious? Doctor: Thanks for your question on Healthcare Magic. I can understand your concern. No, steel will not dissolve. And yes if it remains inside the skin then it may create problem. This kind of matel particles causes local inflammation and granuloma formation. So if you kept it as it is then it will produce hard swelling consists of granuloma. And this can be dangerous. So better to first get done chest x ray to locate the particle. Once it is located then under local site incision, we need to remove it. Don't worry, once it will be removed, no future complications well occur. Hope I have solved your query. I will be happy to help you further. Wish you good health. Thanks."
},
{
"id": 149110,
"tgt": "Dark circle around outer edge of eyes, shaking hands, weakness in arms and legs, slow reaction time. MS?",
"src": "Patient: I have a dark circle around the outer edge of my eyes. The right one is worse than the left. I have had this going on since May, and it betters and worsens at times. It is hard for me to read, but it isn't blurry, it just strains my eyes. I have been to numerous doctors, none of which have any idea what this could be. The black circle is not completely solid, but close to it. I researched and found a disease called Retinitis Pigmentosa, which in turn led me to read about Multiple Scleroses. As I read about MS, I came to realize that I do have other symptoms of MS, such as shaking of my hands, weakness in arms and legs, slow reaction time. Do you have any suggestions as to what this might be? Could my age of 14 years old have anything to do with this? Doctor: Hello,Welcome to Healthcare Magic.Common causes of dark circles around eye are stress, overwork, vitamin deficiency, poor sleep and dehydration. It is very less likely due to MS in your case. MS diagnosis is not very simple to make. It will need examination from neurologist and imaging studies like MRI brain to confirm it. Your other symptoms (shaking of hands, weakness in arms and legs, slow reaction time) can also occur due to stress/anxiety and vitamins deficiency. So evaluation by general physician and psychiatrist will help you.Wish you good health and all the best.Regards,Dr Ashish Kumar Mittalwww.99doctor.com"
},
{
"id": 104678,
"tgt": "Allergies, skin itching, bleeding on scratching. Tried allopathy, homoeopathy. Permanent cure?",
"src": "Patient: Hello sir, since childhood at age 8 yrs suffering with skin disease, allergy ,there is itching in my body when i rub, the blood comes out become red & turn to black spot, even if ant or mosquito bit also become red, use many allopathy , hoemopathy but no use, pls advise me is there parment treatmnet or not, we are middle class cant spend more money. my parents want to do marriage, can i go forward, or stop any problem in married life. pls advise the parment treatment thanksing u Doctor: U need an allergy testing to find out the probable allergen. If u r found to be sensitive to many proteins/allergens, u may need a short course of systemic steroids and then antihistamines for prolonged period of time. If antihistamines found to ineffective, u may need to take a low dose steroid daily. But, u also need to rule out any infective causes like scabies, body lice etc"
},
{
"id": 156374,
"tgt": "Can spots on liver be symptom of liver cancer?",
"src": "Patient: My dad had colon cancer 23 yrs ago and had his colon removed and has been cancer free ever since. He went in to have his gallbladder removed and they found spots on his liver but all bloodwork was normal. They are doing a biopsy, but I m too anxious for the results. Are his odds higher for liver cancer? Doctor: Hi. The biopsy will clear the doubt. There are chances of having spread (metastasis) from his old treated cancer as well as metastasis fron some other cancer and liver cancer itself. If his blood tests of alpha fetoprotein (AFP) and CEA are normal, odds of liver cancer and metastasis from colon cancer are low. Biopsy is the recommended procedure to make things clear. Hope I have answered your query. If you have any further question, I will be happy to help."
},
{
"id": 77855,
"tgt": "What causes pain in chest with little discomfort in breathing?",
"src": "Patient: Hi Doc, Im having a sort of pain in middle chest. It is a i feel pain when i expand my chest or i certain positions. my age is 25. i feel a little(not much) discomfort in breathing as well. the pain is very sharp like sort of poking. before 2 years i had problem of acid reflux due to which my throat and chect started showing similar symptoms. Now can i assume this is also due to acid reflux? If yes what should i do to cure those burns on my food pipe? Doctor: Thanks for your question on Health Care Magic. I can understand your concern. Yes, possibility of GERD (gastroesophageal reflux disease) is more in your case. And it is definitely treatable. GERD is due laxity of gastroesophageal sphincter. Because of this the acid of the stomach tends to come up in the esophagus and cause the symptoms. So follow these lifestyle modifications for better symptomatic relief. 1. Avoid stress and tension. 2. Avoid hot and spicy food. Avoid junk food. Avoid large meals, instead take frequent small meals. 3. Take proton pump inhibitors and prokinetic drugs. 4. Quit smoking and alcohol if you have these habits. 5. Go for walking after meals. 6. Keep 2 - 3 pillows under head in the bed to prevent reflux. Don't worry, you will be alright with all these. Hope I have solved your query. I will be happy to help you further. Wish you good health. Thanks."
},
{
"id": 28381,
"tgt": "What causes pain and discomfort in chest when started roseday after angioplasty?",
"src": "Patient: hi, my mother has undergone Angioplasty in Month of August 2013 an doctor has prescribed her medication Roseday a week ago and since she has started this medication she is feeling chest pain and discomfort around the area. So what should be done in this regard??? Doctor: Hello! Thank you for asking on HCM! I understad your concern and I would like to assure you that Roseday therapy (that is lipid lowering treatment) may be associated with some side effects in certain patients, like muscle pain and break down. Some blood lab tests should be performed: AST, ALT, CPK, complete urine test, coupled with a medical ckeck up by the attending physician. If skeletal muscle involvement is confirmed, and altered blood tests are found, then Rosuvastatin should be stopped immediately.Hope to have been helpful to you. Greetings! Dr. Iliri"
},
{
"id": 150367,
"tgt": "Diagnosed with herniated disc and pinched nerve after an accident. feeling pressure in the lower spine. Advice?",
"src": "Patient: I was diagnoses with a hernatic disc. From car accident in 1997. 2002 I had a pinch nerve and inflamed disc ball on right side of back and leg where it went to the bottom of my right foot. Last year I was diagnoses with sciatic nerve on my left side of my back. I went to doctor they had did x-ray. Waiting for results because I go back Thursday. Now pain is worst. It's like on and off. But when I have a bowel movement I can feel pressure on lower spine and my testicles feel like its being smushed. Basically pressure. Is it possible that its the nerve in my back. Because when I do stretch I can feel irritation nerve in bottom of right foot. As if I stretch any further I can cause another pinch nerve Doctor: Hello Tony, Welcome at HCM. I have read complete details of your question. It looks from your details that you are having prolapse disc in your lower spine which is irritating your nerves supplying to leg. You should do following to have relief in this- -Confirm your diagnosis by having a MRI on prescription from a orthopedic doctor. -At initial stage with mild prolapse the treatment is complete bed rest followed by exercises under supervision of an expert- http://orthoperson.blogspot.in/ -If your pain is severe then it may need surgery to prevent death of nerve. -You may take a good analgesic like tramadol with paracetamol or pregabalin on prescription. -Avoid forward bending & lifting heavy weights. Hope this all will help you. With best wishes."
},
{
"id": 174987,
"tgt": "How to relieve stomach pain and possible constipation in 5 year old?",
"src": "Patient: my 5 year old son has had tummy pains that have been coming and going .. he feels like he has to poop but he sits there and nothing comes out i took him the hospital cause i thought he might have appendicitis but blood and ultrasound and xrays came back just fine so could he be constipated if so what can i do for him? Doctor: Hi. You can include lot of vegetables and fruits in his diet. Dates and prunes are especially good for constipation. He must take around 1 litre of water per day."
},
{
"id": 209032,
"tgt": "What causes agitated depression?",
"src": "Patient: Well i have been feeling very agitated, paranoid and depressed for 2 months plus i had insomnia. I recent found out that Bipolar Disorder runs in my family. I asked my old psychiatris and he doesn t know what i have but thinks i need antidepressants again (Prozac). Now sure if it mixed episodes of bipolar or something. He describes me as agitated depression. What do you think? Doctor: HiThanks for using healthcare magicIt is normally seen that patients with positive family history of bipolar disorder get either depression or bipolar disorder. If you do not have any manic or hypomanic episode in past, then current episode is of depression and you can start taking antidepressant like prozac. In your case, chances of antidepressant switch is there, but you can also take mood stabilizer as back up. In case, you need further help, you can ask.Thanks"
},
{
"id": 178192,
"tgt": "Suggest treatment for seizures in a child",
"src": "Patient: Yes my niece is 7months old, she's having seizures that have been treated, she has been to the emergency room a lot, she has an appointment with her neurologist tomorrow. However the seizures last for about 1min to 2min and after wards she cries for 30 min. my question is, is there pain that she's feeling, what can I do besides giving her meds to help ease her when she has an episode? is their anything that i should be doing to not cause them? Doctor: Hi, I had gone through your question and understand your concerns.During an episode of seizure , keep baby in lateral position to avoid risk of aspiration of secretions , do not give anything including medicines by mouth , usually seizures abort in 2-3 minutes ,in case of prolonged seizures you need to learn use of rectal or intranasal drugs or rush to emergency to get seizures aborted. Cause of excess cry is post seizure irritability due to effect on brain .Preventing seizures from occurring needs proper evaluation of cause and regular medicines to control seizure.Hope this answers your question. If you have additional questions then please do not hesitate in writing to us.Take care.\u2022 DISCLAIMER: - All the information provided here is for information purpose only , it is not a substitute for the advice of a physician after physical examination , it is not intended to replace your relationship with your doctor. This information in no way establishes a doctor-patient relationship. Before acting anything based on this information do consult your doctor. I recommend that online users seek the advice of a physician who can perform an in-person physical examination."
},
{
"id": 102294,
"tgt": "Will antihistamine medicine help cure dryness of mouth and swelling of throat?",
"src": "Patient: I think I have an amaranth allergy. i am finding it hard to breath, my throat feels swollen, i feel as though under my ribs are being squashed in, dry mouth. its not serious enough to go to A & E but I would like to know what to do until i can see my GP on Monday. Should I take ant histamine ? Doctor: Hello! Great question, with a \"mixed\" answer!Antihistamines are indeed quite useful in controlling the symptoms of an allergic reaction. Sadly to say, however, every antihistamine on the planet causes dry mouth (or makes it worse if you already have dry mouth).Overall, I would advise that the benefits of antihistamines in your situation outweighs the disadvantages. You can always use lozenges, drink plenty of water, or suck on ice chips to help with the dry mouth.You should not hesitate to go to the A & E if your symptoms should worsen. Too much mouth or throat swelling can cause serious breathing difficulties.For now, be sure to avoid amaranth exposure. Stay indoors if necessary. Again, drink plenty of water, and avoid strenuous activity until you are feeling better.I wish you the best, and a speedy recovery..."
},
{
"id": 200379,
"tgt": "What causes night fall thrice a week?",
"src": "Patient: Hi am 24 yrs male boy. i am suffering with night fall very frequent 3-4 times in week . and now my male organ have been very weak, small and its veins are also weak . medical says it does not harm physically , but i am felling very dull and my face is going to be scaring and not felling well . the main problem occuring during urination. after urine disposal still some drops of may be urine or what have self poured in pent after interval of every 10 minutes for 2 hours , specially for 2 days after night fall. i am in very much tension and kindly suggest me what should i do to stop night fall and this urine problem due to frequent night fall . now i am taking confido medicine , is it affective . kindly suggest how i can come out from these both problems . Doctor: Thanks for asking in healthcaremagic forum Masturbation and night fall are not diseases to treat. Its a normal phenomenon. Please visit a psychiatrist for counselling regarding this. All the best."
},
{
"id": 157101,
"tgt": "Unexplained weight loss, frequent bowel movements, pain in middle of abdomen. Do i have symptons of small intestine cancer?",
"src": "Patient: do I have symptons of small intestine cancer?-had capsule endoscopy yesterday-unexplained weight loss of 34 lbs in 16months-frequent bowel movements-especially after meals-pain in middle of abdomen-haven t had results of test yet-is small intestine cancer a possibility? Doctor: HiThanks for your query.Yes, it is a possibility. But there are other possibilities, like malabsorption syndrome or inflammatory bowel disease. The lab tests will help in formulating exact diagnosis and treatment plan. Hope this helps.Regards"
},
{
"id": 44064,
"tgt": "Spotting after taking Letrol. Ultrasound showed pregnancy negative. Am I pregnant?",
"src": "Patient: Hi,i had a 2 month miscarage October 2011,i had to make a D&C.April 2012 my period was 12 days late so i took a home pregnancy test & it came positive.Next 2 times i took more 2 HPT & it again come positoive.But May 2012 when i made a ultrasonograph ,doctros said i m not pregnant at all.After my 1st miscarage doctor prescribe me a medicine if i found the test positive,& its name is Gestrenol.So,when i found that i m not pregnant i was totally disappointed,i really want to have a baby.After that for last 4 month we have been trying but no result.Doctor also gave me a hormone tablet for pregnancy name Letrol.Though there is no progress.But last month 4 days before my period i had a very little bleeding just like spotting for 2 days.Then there is nothing happened like period.Am i pregnant ?If not what should i do now ?I really want to have a baby,please doctor help me.Thank u. Doctor: Go for all hormonal tests.Sometimes urine pregnancy tests may give us falls results.Do uterine sonography after 3 wks.If no pregnancy, surely Ayurvedic treatment will help you to fulfill your getting a own baby dream.All the best."
},
{
"id": 51578,
"tgt": "What does pus cells 60/HPF in urine indicate and would it affect his wife after sex ?",
"src": "Patient: hi , my sister 22y ,she was married last month , now she has burning sensation during elimination of urine ,his urine analysis report show aspect turbid , color amber yellow , Reaction acidic , (pr ,glu , keto , bili ) nil , pus cell 60/HPF , RBCs 3/HPF , epithelial cell ++ ,casts nil, crystals not seen , bacteria ++++ , Nitrit ++++ , what is the most probably diagnosis? Doctor: Welcome to Healthcare Magic She is having severe urinary tract infection. She needs to be on antibiotics treatment. Best would be to get a culture and sensitivity to find out the bacteria causing this and treat correctly. Let her drink 2-3 litres of water to reduce the symptoms. Drink Cranberry juice which will also give relief. Get her shown to her Urologist for the right treatment. Don't delay."
},
{
"id": 13352,
"tgt": "What is the redness and itching in my wrist?",
"src": "Patient: I woke up with an itching sensation coming from my wrist area. There were two small bumps that the redness seemed to be surrounding. At first I did not think much of it, but by now (night), The redness has reached all the way from my wrist to my elbow and is firming up. What is causing this? Doctor: Hi, It maybe insect bite dermatitis most probably. Consult the dermatologist for the perfect diagnosis and proper treatment. You may take antihistaminics like Cetirizine along with oral steroids in a tapering dose. If needed, antibiotic may be taken. Apply mild steroid with antibiotic cream. Hope I have answered your query. Let me know if I can assist you further. Regards, Dr. Ilyas Patel, Dermatologist"
},
{
"id": 200451,
"tgt": "How to treat redness around pee hole?",
"src": "Patient: Hi, for a few weeks now I have had redness on the top and around the peehole I have a small discomfort in my abdomen on the left side. My only question is what your thoughts are as the redness and the discomfort are not going away. I also have chunky sperm and have been dehydrated a lot lately. Doctor: Thanks for asking in healthcaremagic forum Redness may be due to dry and rough handling of genitals during masturbation/sex. If you are dehydrated have ORS(oral rehydration solution) and other fluids in plenty for relief. All the best."
},
{
"id": 107408,
"tgt": "What causes bed-wetting and left leg pain while having issues in L5-S1?",
"src": "Patient: Hi I have a back problem for the past 2years I m on a waiting list to be seen by a specialist I have been seen by one doctor and have had a few mri scans done they have said that I have a swonomma and l5 s1 problems too and my left leg is always in pain from my button down into my left foot and my leg is weak from the pain... but I have a new problem I wet the bed last night and don t know if I should go see my doctor Doctor: Please don't worry ..go and consult your doctor ..he will clarify all your queries better after seeing MRI and by clinical examination.. as it's scwanoma already you are suffering from sciatic pain bladder symptoms ..which are indications to immediate intervention.."
},
{
"id": 101585,
"tgt": "Does shortness of breath need medical attention?",
"src": "Patient: asthma question I hav most of the symptoms of asthma but have not taken any medication or been looked at for it I have shortness of breath and sometimes it feels like im holding my breath but I can't breath in or out at all but it doesn't happen while im asleep at least I don't think it does should I get it looked at or is it something I shouldn't worry about Doctor: Thanks for your query.Brief answer:You do not need to be worried much but you should not neglect the symptoms.Detailed answer:Asthma is variable disease and the patterns of symptoms often vary from person to person.Cough,wheeze or tightness of chest and breathlessness are the typical symptoms of asthma.Your pattern of breathlessness is suggestive of asthma but we can not confirm it without objective test(Spirometry).Again you have to identify whether any triggers(dust,cold,emotion etc) causebreathing difficulty or not.If found,please try to avoid trigger so far it is possible.So my suggestion is :please meet your local doctor to confirm the diagnosis that causes your breathing problem and get treated if required.Wish you good health."
},
{
"id": 143121,
"tgt": "What could cause dizziness,nausea and blurred vision?",
"src": "Patient: i have been experiencing dizziness, nausea and blurred vision for several years. It is worse around 3 am best in evening. Getting incapacitating. Had MRI to rule out MS. Lesions in brain but not spine. Neurologist thought it was normal 52 yr old brain. Saw an ENT, did all the tests. No answer. Quality of life terrible. Have tried every anti nausea med, Serc, .. I also have very labile BP. Diastolic from 40-110.Suggestions? Doctor: Hello!Thank you for asking on HCM!I carefully read your question and understand your concern. You should know that blood pressure fluctuation can cause these lesions in the brain. They are related to low blood flow to the brain and damage to the small vessels. Migraine can also cause similar troubles. For this reason, I would recommend consulting with a cardiologist to help you maintain stable blood pressure values. I would recommend taking flunarizine for your troubles. Sertraline can also help. It is also necessary taking aspirin 80-100mg daily to help prevent future strokes. Hope to have been helpful!Wishing all the best, Dr. Aida"
},
{
"id": 162241,
"tgt": "What causes baldness on the head of a 7 year old?",
"src": "Patient: My 7 year old Asperger son has a horse shoe uniformed bald shape on the top of his head. At first I figured he was pulling his hair to feel the tension to feed his sensory issues. Now I am wondering if that is not the issue due to the perfect uniform shape starting at his temples to just over half way on the top of his head again forming a perfect U shape. Any ideas? Doctor: Hi, I have gone through your question and I understand your concerns but please send a photo to be able to help you further. Hope I have answered your query. Let me know if I can assist you further. Regards, Dr. Salah Saad Shoman, Internal Medicine Specialist"
},
{
"id": 48492,
"tgt": "What is the treatment to remove kidney stones?",
"src": "Patient: Hello my brother has kidney stones and has to have them removed. However though he has been using crystal methamphetamines,and the Dr. Say s he can t preform surgery on my brother, because of his meth use. What can I do or how can we clean his system. Doctor: Helloif stone size is less then 10 mm it can be removed by medicines and if size is more then 10 mm kaser can be done"
},
{
"id": 23552,
"tgt": "What causes chest pains with pins and needles in left shoulder with shortness of breath?",
"src": "Patient: Hello. I am 25 years old. Very active person. I'm a runner. For the past week I've been having chest pains that have at times put me to the floor. I 'm having shortness of breathe as well pain and pins and needles in my left shoulder and fingers. My temperature was up this morning. I also cannot sleep as if I lay down, the pain gets worse and I can't move. The only comfort I get is when i sit up and lean forward a bit. There have been major heart troubles in my family. Should I just see a family doctor or should I go to the ER? Doctor: Mostly its pericarditis....inflammation in the wall of the heart...all you need is rest and any analgesic and and antipyretic...it will resolve spontaneously"
},
{
"id": 117541,
"tgt": "Suggest the cause for low hemoglobin in infants",
"src": "Patient: My 8 month old grandson has been admitted to the hospital. He's had a cold and fever for about 3 days. He's not eating or drinking well and has become somewhat dehydrated. They got some blood work back. His WBCs are elevated and his hemoglobin is low. What could cause the low hemoglobin? Doctor: Hi,Thanks for asking.Based on your clinical history and query, my opinion is as follows:1. Infants are very much prone to iron deficiency as breast milk is deficient in iron. 2. Along with it, malnutrition and diarrhea episodes can cause low iron and also Vitamins like B12 and folic acid3. Nutritional supplementation should be adequate to improve hemoglobin levels. WBC is elevated because of fever and will get corrected, when treated.Hope it helps.Any further queries, happy to help again."
},
{
"id": 179039,
"tgt": "Can Flutivate cream be applied on a baby?",
"src": "Patient: Hi Doctor, shall I use flutivate cream for my 1 year baby? She is having rashes in her legs and when we consulted the doctor he told that because of milk only this kind of allergy will come. And also he prescribed flutivate cream.. When I check the internet it shows some side effect and also mentioned not used for children (Fluticasone Propionate). So, I m in confusion. Please clear it. Doctor: Thanks for posting on HealthCareMagic. Flutivate contains a steroid called fluticasone which is intended to reduce an inflammatory allergic manifestation as exhibited by the rashes. Apply it as per your doctor's advice. It is hard to comment without looking at the rashes but might be needed. It can be used in children on doctor's advice and so you need not worry.Such allergies may occur not only in response to milk but also practically anything that we are exposed to, things like dust, pollens etc.If your doctor advises milk free diet, you can include formula like Zerolac. But after semisolids have been introduced in the diet, milk can be avoided anyway.Hope that helps. Feel free to revert back in case of further queries if any."
},
{
"id": 89351,
"tgt": "Suggest treatment for severe stomach aches",
"src": "Patient: My husband is having severe stomach pains. He says it feels like someone is reaching into his insides and twisting them. He doesn t have diarrhea or have to pass a stool. He did vomit a little after drinking some water. I ve asked him if it is in one area but he says it feels like all over his stomach. He asks me to rub his stomach, and when I do it feels better to apply a little pressure and in the middle of his stomach, not lower or upper but pretty directly in the middle. I ve given him a 600mg ibprophrine and 2 tablespoons of Kaopectate. Not sure if we should go to the ER. He says it feels better also when he is laying on his side. Please advise. Doctor: Hi.The causes of acute pain in abdomen in this can be pancreatitis, intestinal obstruction, intususception , Your husband needs to be taken to the ER for blood tests including that of amylase and lipase. Ultrasonography to see the pancreas and all . Needs X-ray standing abdomen for intestinal obstruction. Admission and further management."
},
{
"id": 161260,
"tgt": "Is fever, cough, blocked nose and diarrhoea symptoms of swine flu?",
"src": "Patient: Dear sir, my son 10 years old had fever past one week, he having running nose,some time blocked nose, and dry cough,and he feeling tried,and not taking normal food, yesterday i took him to decoter he examined,and said he had infection in his chest, and gave augmentin but he had now diarrhea, is it symptoms of swin flu. plesae i am waiting Doctor: Hi, Swine flu usually presents with severe fast breathing, cough, cold, respiratory distress and high grade fever spikes. Confirmatory test are by h1n1 PCR from nasopharyngeal swab. Even if it is swine flu, no need to worry. Treat with Oseltamivir for 5 days, it is a good drug. Hope I have answered your query. Let me know if I can assist you further. Regards, Dr. Prasanna Lakshmi, Pediatrician"
},
{
"id": 158320,
"tgt": "Have cancer of tyrhoyde, removed, throat always pain, red, problems eating. What could be wrong?",
"src": "Patient: I have cancer of tyrhoyde .the remuve ol my tyrhoyde ,...after that my thoat alwys haves pain,is too red, and have yellow thins in the back ish side and to dried and problems eating and my ears pain feels like coverup and my heart is too slow or too fast am too tired and angry.my tang is tingling some times.if they do test for thort throat is negative but I can see the yellow in the back.I have reflex too and am scared it is cancer Doctor: Hi, Thanks for using HCM.Your throat symptoms are could be due to throat infection or acid reflux.you can do salt warm water gargling 3 times a day.avoid taking heavy meals in night times.avoid immediately going to bed after taking food.consult a ENT specialist for complete examination of your throat.check your TSH level to know your thyroid level now.Hope I answered your question. Feel free to ask me if you have any further queries.Wish you good health. Take care.RegardsDr. Vidya"
},
{
"id": 93614,
"tgt": "Reoccurring abdominal pain and nausea. Low wbc, taking medication. Pregnant. Any ideas?",
"src": "Patient: Every so often, my girlfriend gets abdominal pain and nausea. It becomes quite severe and we end up having to go to the ER. When we do, they always just give her anti-nausea meds and pain killers take a urine sample, tell her that her white blood cell count is low, and then tell her she's fine and send her on her way. But it keeps coming back, generally several months apart, and now she's pregnant and it came back again last weekend. Any ideas or advice? Doctor: Hello, thanks for the query to H.C.M. Forum I would like to say one thing that she has Hyper acidity ( gastritis) , that's why there are pain and nausea. Anxiety, Fried food, Alcohol, Over feeding, Junk food, All these increases gastritis hence symptoms recur. Now she is pregnant so pregnancy also increases hyper acidity due to certain hormones , hence nausea, vomiting, pain in epigastrium region. Now only one option is there consult a gynaecologist and have a treatment for HYPER EMESIS GRAVIDARUM ( nausea and vomiting in pregnancy ). Lemon water luke warm sometimes gives relief in few patients. Good luck Dr. HET"
},
{
"id": 66077,
"tgt": "What is the lump which rises up into my chest?",
"src": "Patient: Hi, i'am a 36yr old female.my concern is when ever i bend over or lean forward i feel a lump rise up into my chest it gets stuck at top ov my ribcage.then it takes my breath away, i sit up straight then i see pop back down.what is it and should i be concerned.Thank you. Doctor: Hi.Thanks for your query.This is a bit atypical and rare presentation of any problem heard so far. Read you are female patient of 36 years - feel a lump rise up into the chest which gets stuck at the top of the rib-cage, on bending down and takes your breath away. It pops back when you sit straight. The possible cause is that this is a rare sort of a tumor which has gone through the rib-cage and hence can see it grow or come back to original position. This can also be a broken segment of the rib from the previous accident. This can be diagnosed well with :Clinical actual physical examination by a Surgeon as proper history and examination alone can help the best, to be supplemented by CT scan of the chest and other relevant investigations. If required a surgery may be needed."
},
{
"id": 5380,
"tgt": "Have PCOD. Trying to conceive. Suggestions",
"src": "Patient: Hi Doc,i have a major concern here....I am 26 and 2years back i got married i trying to concieve but in vain.2wice went to the doctors for check up and treatment but they say that is because of the PCOD i have and i need to loose weight but i have read that even women having PCOD can concieve so why not me..plzzz hel having pressure from inlaws..husband was suffering from epilepsy but with gods blessings its cured now he had an operation long back in 2007 now all well he is on medication doctors say that his semen count is low as he use to take high dosage now he takes 2500 mg per day..and i take dasmularishta 3 spoons with water after lunch and dinner and glycomet 1000.please please help Doctor: Hi,Thanks for the query.Don't worry, every problem will have a solution. But patience is required in infertility cases.As your husbands sperm count is low and you are having PCOD, both these conditions should be corrected first.Let your husband continue the medicines. And after one month you go for ovulation induction with your doctor's advice.Then plan unprotected intercourse around the time of ovulation that will increase the possibility of pregnancy.For more details you can ask me through: http://www.healthcaremagic.com/doctors/dr-sree-gouri-sr/63429Take care."
},
{
"id": 214830,
"tgt": "High BP. History of polio in left leg. Any natural or herbal remedy for BP regulation?",
"src": "Patient: I am 60 years old had polio in left leg so blood flow is not good of course and i have High blood presuer of course i do not like to take the meds / it causes low libdo , also i do not like the idea of taking eriction pills I would like to find a method / food / hurbs etc that would help better them the blood presurer meds Doctor: Hi,I agree that few medicines for high blood pressure do have effects on libido.Ask your doctor to go for medicine which has no effect on libido.Take low salt diet.Go for daily exercise like walking, swimming etc.Ok and take care."
},
{
"id": 144562,
"tgt": "What causes quivering chin?",
"src": "Patient: I wake up in the morning and when looking in the mirror, I see the lower left side of my chin slightly quivering. Possible causes suggested are stress, coffee intake, bells palsy, and others. I don t think any of these are possibilities. But I am puzzled. What do you think? Doctor: persistent quivering chin can be a variant of essential tremors, which is genetically influenced. If there is history of hand tremors/neck tremors/chin tremors in close blood relatives, then high possibility of essential tremor is there."
},
{
"id": 182041,
"tgt": "What causes tooth loss?",
"src": "Patient: I'm a well raised 44 year old female, who grew up with good dental care. In my adult life, I have my teeth cleaned every three months and practice good hygiene. In the last 5 years, I've been forced to replace 5 teeth with implants and another tooth has chipped. Why are my teeth failing? As a professional, this is crazy! Doctor: Thanks for your query, I have gone through your query.The loss of teeth can be because of aggressive periodontitis, even though there is no deposits. This is caused by aggregator bacter which results in severe bone loss. Nothing to be panic, consult a periodontist and get it treated with root planing and grafting with bone graft. You can save the remaining teeth with the above said treatment, but the extracted teeth can be restored with implants or a bridge.I hope my answer will help you, take care."
},
{
"id": 38071,
"tgt": "Suggest remedy for foul smelling wound on leg",
"src": "Patient: MY mother at age 70 has a skin wound on leg. It has been very slow to heal and has now developed a slight odor. She say a doctor just days ago and they told her to keep doing what she is doing. (changing dressing ets. What does the odor mean? Infection? Doctor: Hello, Thank you for your contact to health care magic. I understand your concern. If I am your doctor I suggest you bad odor in wound is suggestive of infected wound, which is also suspected with slow healing. I advice you to go with culture and sensitivity of the organism. Take treatment according to the sensitivity report and your infection will go in a week or two. Kindly check her blood sugar and if its high kindly maintain it to normal it will help you in healing fast.I will be happy to answer your further concernYou can contact me. Dr Arun Tank. Infectious disease specialist. Thank you."
},
{
"id": 4868,
"tgt": "Trying to conceive. Irregular period. Detected thyroid issues. On duphaston, fertility medication and thyroxine. What is going on?",
"src": "Patient: Hi.. I am 29 years 10 months old married from past nine months.. Had a history of irregular menstrual cycle since its start in teens.. Detected of thyroid in may which was not there earlier. Visited a senior gynaecologist who advised me to go on fertility drugs for conception and prescribed me with fertomid 100 mg from day 2 to day 6 and duphaston 10 mg from day 16 for 12 days along with thyroxine 25 mcg. I started the course in mid June and since then my periods have been exact 30 days.. I just finished my third month of the course and my period is delayed by seven days now.. It should have started on 13th September and today is 19th.. In the third month I took duphaston after breakfast than previous two times which I took after dinner.. I took a hpt on 15th which was negative and repeated it on 19th which showed a faint line in test window.. Have sore nipples for more than 20 days now.. And slight on and off cramps in mid stomach near naval also now feel some fluttering towards left side of abdomen .. Am I pregnant .. Is it possible to miss periods this long with fertomid if not pregnant.. Plz help Doctor: welcome to HCM!,there is chance of u getting pregnant.get urine pregnancy test done.there is already faint positive.get it confirmed after 2 days."
},
{
"id": 144198,
"tgt": "What causes numbness in left leg with sacralisation of l5 vertebra?",
"src": "Patient: hi.. I have numbness in left leg toe and numbness in left leg and left hand. I have MRI 1.transitonal vertebra with sacralisation of l5 vertabra. 2.modic type-II end plate changes l4-l5. 3.l4-l5 diffuse disc bulge indenting thecal sac. 4.changes of lumbar spondylosis. Doctor: Hi, I am Dr.Bruno. I have read your question and understand your concerns. Let me try to help you Question : What causes numbness in left leg with sacralisation of l5 vertebra?Answer : Two Possibilities 1. The Diffuse Disc Bulge in probably compressing the nerve root in that area and this causes numbess in Left Leg .2. You also have numbness in Left Hand. So This can be due to some generalized problem like Diabetic Neuropathy. Please get your sugar levels checked at the earliest Hope you found the answer helpful.If you need any clarification / have doubts / have additional questions / have follow up questions, then please do not hesitate in asking again. I will be happy to answer your questions.Let me know if I can assist you further.Take care."
},
{
"id": 31252,
"tgt": "How can chest infection with pneumonia causing fatigue be treated?",
"src": "Patient: i was diagnosed with chest infection/phenomia a week ago. I spent 3 days in hospital on an iv drip. Since i have been home i feel more tired and fatigued then what i did when i went into hospital, im only spending about 5 hrs out of bed a day. Simple things like having a shower are exhausting. I am always hot and sweaty but i have no temp. Doctor: hi thanks for your question... fatigue can occur after any disease in convalescent stage...just take proper diet and increase fluid intake... this will take time to be in your proper state of health...."
},
{
"id": 164577,
"tgt": "What causes loss of appetite in infants?",
"src": "Patient: My 4 month old baby use to eat 5 oz every 3-4 hours but the last week he will only eat 2-3 oz and he is really restless during the feed.He move his head away from the bottle and cries after 2 oz.I am worried about him losing weight.He is happy and smiles a lot but never seems hungry. Doctor: hi...this is totally normal with infants...usually with every Change in milestone their feeding habits changes ...unless there is any gross weight loss nothing is to be worried ...feed him on demand...this will change in few days and he will start taking feeds as usual"
},
{
"id": 101228,
"tgt": "Could blister on bottom lip be related to allergies?",
"src": "Patient: I have this cold feeling on my inner right knee. it has only been that way for a couple of days. I have been battling my allergy/sinuses and also have recently gotten a fever blister on my bottom lip... also allergy related I think. just wonder what's the sensation on my knee about? Doctor: HIThank for asking to HCMI really appreciate your concern and let me tell you that your condition of lip lesion may be due to the allergic condition and viral fever and this could come around as the time period over of the allergic condition and this is nothing to worry about this, take care and have a nice day."
},
{
"id": 208628,
"tgt": "What is the treatment for depression and stress?",
"src": "Patient: I need some help to get me through this rough time right now. I feel like I have tried so hard to be the man I thought that my girlfriend wanted me to be but my insecurity is getting the best of me. I want know what I need to do and understand. I don t have the money right now to go to a local psychiatrist. Doctor: Hi dear,for depression and stress related to girlfriend's insecurity then you should need counselling rather than medication if you feel very depressed and not able to cope up than also medication is needed.so first consult psychiatrist if you do not have money than go to government hospital where also psychiatrist available .discuss about your problem.Thank you"
},
{
"id": 29759,
"tgt": "What causes throat ulcers and swelling in lymph nodes after a bout of cold?",
"src": "Patient: I have a low grade fever, ulcers on back of tongue and really raised bumps and feel like they re down my throat. My doctor is treating with antibiotic. It s the tail end of a cough and cold. (3 weeks) he said if it is strep this antibiotic would take care of it... I have a little post nasal drip, enough to cause an annoying cough. Not congested. Other than upper respiratory, what could be cause of the ulcer feel down my throat and swollen glands. Btw I m 60. Doctor: It looks like an infection from what you've said. A course of antibiotics must take care of it. Since you are 60, we must be sure its not cancer. If you are a smoker and the raised bumps are painless please ask your doctor about this."
},
{
"id": 222976,
"tgt": "Is scan showing gestational sac of 16mm, no yolk sac, fetal pole and cardiac activity serious?",
"src": "Patient: Hi I m 8 weeks pregnant now as per my lmp I.e.,dec 19th 2010.but the today s scan says it is 5.1 weeks and there is only a gestational sac of 16 mm seen, no yolk sac,fetal pole and cardiac activity.the doctor says to repat scan after a week and in the meanwhile to get the bhcg done.I m very much worried as to hw to go about it and morever I had a misscarriage at my first pregnancy the same time last year.pls advice Doctor: Hello dear,I understand your concern.Iam very sorry for your condition.In my opinion the presence of empty gestational sac without yolk sac ,fetal pole and cardiac activity might need to be evaluated to rule out any blighted ovum.Usually fetal pole and yolk sac are visible in abdominal ultrasound by 7 weeks.Anyways repetition of ultrasound after a week and serum B Hcg will help in diagnosis.Take progesterone and folic acid supplements.Nothing to worry.Have hope and avoid stress.Best regards...Dr,srilatha"
},
{
"id": 32281,
"tgt": "What causes small red painless bumps on arms and legs?",
"src": "Patient: Hey I have small red painless bumps on my arms and legs. They aren t ichy or anything. I have had it for 6 years. They don t cause any problems but they seem to be slowly spreading in the course of the years. Any ideas on the cause or how I can stop it or remove it all together? I went to a dermatologist but she said it was a common skin problem caused due the parents. But I wasn t born with it. My parents and brother have it too but it isn t as noticeable as mine. Please help. Doctor: Hello. you seem to be having keratosis pilaris, also called 'chicken skin disease'. It's a very common, non-serious problem, and runs in families. The best way to get rid of them is exfoliating well with a store bought scrub, or even granules of sugar rubbed into the soap lather will do. It makes a visible difference and may even disappear entirely.If the exfoliation doesn't work, creams containing salicylic acid, alpha hydroxy acid or retinoin are advised.Hope this answers your question."
},
{
"id": 173440,
"tgt": "What type of food to give for the baby?",
"src": "Patient: hello sir, my child is 1yr4months old and weighs 12kg...he was given nan formula milk from birth.i worry wether any diet drink should be given as i wish to stop nan.he eats 2 meals at breakfast and lunch.but that to not nutritious as i guess.iam aneamic from birth and i dont want him to go through it...please help me Doctor: Hi,Thanks and welcome to healthcare magic.Milk formula can be reduced to 2-3 times daily.Give natural foods like rice with milk and sugar,vegetable soups,rice with dal or rice with curd.Gradually see that he eats the diet taken by the adults minus spices and chillies.You can try soft bread with milk,or milk with biscuits.Iron containing foods like leafy vegetables or iron supplementation prevents anemia.Hope this answer serves your purpose .Please feel free to ask further queries if any.Dr.M.V.Subrahmanyam."
},
{
"id": 54671,
"tgt": "Will Hepatitis B with liver swelling affect heart or other organs?",
"src": "Patient: Hello, my blood pressure is usually 170/110. It sometimes rises to up to 230/150. I am diabetic and taking insulin. Sometimes my blood sugar rises to 180 and up to 250. I have LDL 123 and HDL 40. The ratio for cholesterol was 5.44 on my last lab test. I had 265 for triglyceride. I also have Hepatitis B with some swelling and irritation in the liver with normal liver lab tests. Please let me know how dangerous this on my heart and other organs. Doctor: Thank you for posting your query. As per now, it will NOT affect adjacent organs but you should followup your HepB serology results with your treating physician. If liver enzymes are normal, and HepB viral load is low, then Nothing to worry.furthermore, strict glucose control especially with diet is advised to avoid complications of diabetes mellitus. Less Salt intake advised to counter the high BP. Low fat diet to counter the increase lipids. In case, you are fond of oily(fried) food, try to use OLIVE OIL BUT in less amount.remember, Prevention is better than Cure.if any further questions, feel free to ask.Health professionals aim to diagnose properly and manage patients according to their limited knowledge. Cure is blessed by the ONE who Created us, whose power and knowledge is unlimited .wish you good health.regards,Dr Tayyab Malik"
},
{
"id": 39944,
"tgt": "What is the treatment for scabies?",
"src": "Patient: the family and i have had scabies for 6 weeks now went to doctor got cream done twice washed all clothing and bedding but still have it and its getting worse my youngest aged 4 is the worst infected and wont stop scratching new lumps keep coming the itch is sending us crazy we need help what else can we do Doctor: Benzyl benzoate or asca biol emulsion is the drug of choice. take bath and in weight body apply the lotion. do not wash it for 1 2hours. wash the clothing under garment and bed sheet pillow cover dry it under sun. reaper it it for ten days"
},
{
"id": 209666,
"tgt": "Why do I hear loud music in my head?",
"src": "Patient: I have a question about my mental health.When I was little, I had voices, like people screaming, shouting, and raspling voices whispering at me. It doesn't happen during the day anymore. Just in dreams. But now I have music in my head. It plays loudly, and the music I have never heard before. It doesn't cause me problems, but I thought I should ask. Sometimes I can't hear myself thinking. Doctor: DearWe understand your concernsI went through your details. I suggest you not to worry much. A paracusia, or auditory hallucination, is a form of hallucination that involves perceiving sounds without auditory stimulus. There are three main categories into which the hearing of talking voices can often fall: a person hearing a voice speak one's thoughts, a person hearing one or more voices arguing, or a person hearing a voice narrating his/her own actions.Under musical ear syndrome people will hear music playing in their mind, usually songs they are familiar with. Reports have also mentioned that it is also possible to get musical hallucinations from listening to music for long periods of time.Treatment to this condition is largely based on normal hallucination and individual condition. Ignorance is the best treatment. Unless the hallucination does not produce any unacceptable result, learn to ignore them.If you require more of my help in this aspect, Please post a direct question to me in this website. Make sure that you include every minute details possible. I shall prescribe some psychotherapy techniques which should help you cure your condition further.Hope this answers your query. Available for further clarifications.Good luck."
},
{
"id": 9357,
"tgt": "Suggest treatment for dryness and redness on cheeks",
"src": "Patient: Hi, Recently, I ve had dry and red cheeks. It started when I went on vacation in relatively hot weather where I would be in the sun a lot. Every time I d use my regular cleanser my cheeks would sting and burn. Before that they d get dry no matter how many times I dranj water or how early I went to bed. It stopped after a while and I thought it d go away. However, recently I ve been outside daily under the sun for long periods of time and my cheeks are back to being dry, red, a little flaky, and stingy (when I put my cleanser on). I have a theory that I might have sensitive skin, but I m not totally sure. My skin was never like this before. It just recently started this past month. Do I have exema or some kind of infection? If so, what can I do to treat it or at least ease the pain? Please help! Sincerely, Rain Doctor: Hello,Thank you for posting on HCM.The red flaky skin over your face seems to be due to sunburns.Acute sun burns are sometimes frightening but nothing to worry as its pretty easy to manage.Generally, I advise short course of prednisolone along with local application of mometasone cream and soothing calamine lotion twice a day.Apply good moisturiser like Cetaphil everytime before going to bed.Antihistaminics and antioxidant tablets can additionally be added.Use high SPF broad spectrum sunscreen everytime before going out in sun.Cold sponging will definitely help.Hope this will take care of your issue.Wish you best of health.Thank youDr Hardik PitrodaM.D Dermatology"
},
{
"id": 116936,
"tgt": "What does WBC count of 14.3 in complete blood test suggest?",
"src": "Patient: WBC COUNT 14.3 H [3.7-11.0] RBC COUNT 4.82 [3.74-4.94] HEMATOCRIT 33.1 L [34.1-44.9] HEMOGLOBIN 9.5 L [11.2-15.7] MCV 68.7 L [79.4-95.3] MCH 19.7 L [25.6-32.2] MCHC 28.7 L [32.2-35.3] RDW-CV 16.9 H [11.6-14.4] RDW-SD 42.2 [36.4-46.3] PLATELET COUNT 393 [150-400] MPV 10.9 [9.4-12.3] AUTO NEUTROPHIL 54 [37-78] AUTO MONOCYTE 6 [4-13] AUTO LYMPH 38 [15-45] AUTO IMMATURE GRANS 0.3 [0.0-0.5] AUTO EOS 1 [0-7] AUTO BASOPHIL 0 [0-2] ABSOLUTE NEUTROPHIL 7.71 H [1.56-6.13] ABSOLUTE MONOCYTES 0.92 H [0.24-0.36] ABSOLUTE IMMATURE GRANS 0.05 H [0.00-0.03] ABSOLUTE EOSINOPHILS 0.19 [0.04-0.36] WBCs are always elevated, but usually only to 12. New onset anemia. Not sick that I know of just tired all the time and have pretty wicked heart palpitations, leg pain and cramps at rest, Doctor: Welcome to H.C.M.The WBC count in total CBC count suggest total leukocytes or white blood cells in blood.The WBC count in normal value range in your lab reports is 3.7-11.0 in Adult.High value of total WBC suggest infection. Your Total WBC count is 14.3 suggest that it is higher than normal value so follow your doctor to rule out infection.Thanx for query."
},
{
"id": 221388,
"tgt": "How can TSH levels be reduced in a pregnancy?",
"src": "Patient: Dear Dr, I am pregnant Report given in Scan gestational age is 6 weeks 6 days and Menstrual age is 8 weeks 1 day has on 10 feb 2011 I have tested thyroid on 9th Feb 2011 below are the test result Free T3 - 2.23 Free T4 - 0.39 TSH - 39.68 Please let me know what medican I need to take to reduce TSH level Doctor: Hi dear, I have gone through your question and understand your concerns.You have increased TSH levels which make you hypothyroid.You need to take thyronorm or Eltroxin to normalize the thyroid hormone levels and the dose should be adjusted according to the TSH levels.Hope you found the answer helpful.Wishing you good health.RegardsDr Deepti Verma"
},
{
"id": 163037,
"tgt": "Suggest remedy for cough",
"src": "Patient: hi, my 3yr old has been fighting a cough with quite alot of flem which after a while makes her sick. She has also just over the last couple of days developed a runny nose. Please is there anything i can do to ease all this as the cough especially is making her not well. Doctor: Hello and Welcome to \u2018Ask A Doctor\u2019 service. I have reviewed your query and here is my advice. Parent by your description it seems the child has developed respiratory infection. Please check with your pediatrician, as this may or may not create trouble. Better be supervised. Hope I have answered your query. Let me know if I can assist you further."
},
{
"id": 7898,
"tgt": "What medication should I take for acute acne problem ?",
"src": "Patient: m a female, 25 years old and still ave acute acne problem, esp on both the cheeks . the acne starts developing and becoming big just a week before my menses , it looks really ugly and has left a lot of scars too. please help. Doctor: Acne vulgaris is the teenage problems mostly between age of 18-25yrs Females. You must try for the Ayuvedic Drugs availble in the market rather than Chemical drugs. Some of the ayurvedic herbal drugs 1.Blood purifier (Syp or Tablet) Tab Radona, 2.Local iontment (Gel) Zap & Clear 3.Herbal Facepack Perolap Advice: Drink plenty of water and washthe face time to time Use herbal soaps and aware of commercial soap available in the market"
},
{
"id": 19328,
"tgt": "What causes headache, chest pain and breathlessness?",
"src": "Patient: my husband had a head ache, couldnt breath, chest pain and he said that it went silent and dark, he was in the hospital but they couldnt find anything wrong with him. he will have these spells where he just looks straight ahead and he said that his body starts to tingle all over, do you know what it could be Doctor: Hello!Welcome and thank you for asking on HCM!I passed carefully through your question and would explain that your husband symptoms could be related to TIA (transient ischemic attacks) or seizures. A cardiac arrhythmia can not be excluded either. Coming to this point, I would recommend performing further tests: - an ambulatory 24-48 hours ECG monitoring to investigate for cardiac arrhythmia - a brain MRI study- an EEG. You should discuss with his doctor on the above issues. Hope you will find this answer helpful!Kind regards, Dr. Iliri"
},
{
"id": 76252,
"tgt": "What causes chest pain while swallowing food?",
"src": "Patient: Hi, I'm 29 yrs old at 5'8 and 202 lbs. I'm pretty healthy with no medical problems. My allergie is latex. I'm having pain in my chest area when I swollow liquids or food. The pain is severe to the point I dont want to eat. This has been going on for two consecutive days so far. What could be wrong and is it necessary to see the doctor? Doctor: Thanks for your question on Healthcare Magic. I can understand your concern. You are having dysphagia (chest pain in swallowing). Common causes for dysphagia are GERD (gastroesophageal reflux disease), esophageal mass, esophagitis, achalasia cardia etc. So better to consult gastroenterologist and get done upper GI (gastrointestinal) scopy fir the diagnosis of your symptoms. Better to avoid hot and spicy food. Take pantoprazole tablet on empty stomach. Further treatment will be directed on the diagnosis. So consult gastroenterologist a. D first diagnose yourself and then start appropriate treatment. Hope I have solved your query. I will be happy to help you further. Wish you good health. Thanks."
},
{
"id": 209049,
"tgt": "What causes shaky hands with nervousness?",
"src": "Patient: I will think about something (ususally things in the future I have to do or second quessing the past) and feel nervous inside and them my left hand will shake. The nervousness inside will subside. I have this happen maybe every other day. What can be the problem? Doctor: Dear friend, it appears from your description of your problems that you are having anxiety related to your thoughts and excessive preoccupation over the same. I feel you are having a underlying anxiety disorder like generalised anxiety disorder etc. For which treatment is available both medicines and psychotherapy. You should consult a psychiatrist near by and discuss your problems."
},
{
"id": 67184,
"tgt": "What causes lumps in lower abdomen near pelvic region?",
"src": "Patient: Hi, I have a few small lumps in my lower left abdomen region near the pelvic area. It is also slightly painful to press the area, but no changes in any other functions. I did have my tubes tied a while ago, so I don t know if that would have to do with it, since I have only recently noticed this. Doctor: Hi! Good evening. I am Dr Shareef answering your query. In case you had a laparoscopic tubal ligation, and the lumps are in the area of the scars, it could be a hypertropic scar or keloid which could be related to the previous surgery. If not, then these could be completely different independent pathology like a benign lipoma. However, these could be diagnosed only by a physical examination , and so I would advise you to take an appointment with a general surgeon in your area for a proper diagnosis and further management. I hope this information would help you in discussing with your family physician/treating doctor in further management of your problem. Please do not hesitate to ask in case of any further doubts.Thanks for choosing health care magic to clear doubts on your health problems. I wish you an early recovery. Dr Shareef."
},
{
"id": 183395,
"tgt": "Suggest treatment for tooth pain and redness of gums",
"src": "Patient: My problem is I am terrified of the dentist and the last time I saw one was 10 years ago when I had some fillings put in, when she numbed me it numbed my whole face to which I couldn't even close my eye and that freaked me out also they wanted to scrape my gums and after hearing many horror stories I never went back well now I have been in pain for the past 2 weeks, my tooth came out upper right side and I am having pain in this location and also in the tooth next to it. when I looked at it I can see a hole in the tooth where the gum is, the area is red and I see white in the hole at one point the whole side of my face was hurting. Do I have to have this other tooth pulled? also i feel maybe a piece of tooth on the upper gum where my tooth fell out.I don't know what to do because I am scared to death. Doctor: Hi,Thanks for posting the query, First of all i would tell you dont get frighteneed it is not a major problem i would sugest you to get an x-ray done of the filled tooth to see if there is any formation of secondary caries beneath the restoration, see if there is any presence of high points if so get it trimmed, if there is presence of secondary caries get it removed and root canal treatment can be performed. For pain relief take tab brufen BD for 3-5 days, go for complete mouth scaling and polishing, at home take antiseptic mouthwash rinses.Hope you find this as helpful,Take care!"
},
{
"id": 11250,
"tgt": "When and how long should evion 400 be taken?",
"src": "Patient: hello sir,myself ml.a.of age 25yrs. As such no one suggested me prescribed me this capsule bt i have started taken it myself EVION400. and i take it at night tym.....before i go to bed.I just want to know is there any specific time to take this medicine??and for hw much days its intake should be continued??Thanks Doctor: thanks for asking,It's actually better to take it with food (right before you eat or shortly thereafter) because when you eat, your digestive system produces substances which help your body absorb fat from your meals. Since vitamin E is a fat-soluble vitamin, taking it with meals will increase the efficiency with which your body absorbs it. It is safe to take Evion 400 just after dinner.Vitamin E is a fat soluble vitamin and, as such, it's important not to ingest high levels of it on a regular basis.regards"
},
{
"id": 153636,
"tgt": "Is it safe to get a Cortisone shot during a chemotherapy?",
"src": "Patient: I have had one chemo treatment for colon cancer and then had to stop because I had to have surgery redone due to blockage.I then had infection.I will be resuming chemo in the next 2 weeks.Is it safe for me to get a cotisone shot in my elbow at this time? Doctor: Hi,Thanks for writing in.The cortisone shot to your elbow is local medicine. It will not affect your system. Having chemotherapy in two weeks is completely safe after cortisone injection.The cortisone is a steroid and gets deposited in the elbow. The steroid stops the nerves from carrying pain impulse and this will keep you comfortable. There is no risk form the shot and it will be safe for you to have chemotherapy a few weeks later. If you are given oral steroids then there is a need to discuss with your doctor. Please do not worry."
},
{
"id": 62314,
"tgt": "Suggest medication for painful lump on right side under the ribs",
"src": "Patient: I have a very hard lump on my right side just under my ribs and into the center of my abdomen about 2 inches. i also have just above the lump on my rib cage is an indention that i do not have on my left side. The lump is not painful unless you press very hard. but the area surrounding it is very painful. is this something i should be concerned about. I had it about 2 months ago and it last about 2 weeks then disappeared. It has been back now for about a week. Doctor: Hi,Dear thanks for the query to HCM virtual clinic.I studied your query in full details updated from you.I understood your health concerns.Based on your query data, In my opinion ,Dear You seems to suffer from-a hard painful lump under right ribs,which pains on pressure,with surrounding tissue-very painful.This indicates Spigelian Hernia-which is in the weakness in the Para-rectal fascia.The impacted / herniated pre-peritoneal fat, or omentum /mesetery is the cause of the severe pain around the lump.Remedy-Surgeons opinion/and USG of the part would fix the cause.Tab Voltaren with brufen to reduce the entrapped tissue.If not reduced manually under local anesthesia,Surgical planned repair is the only treatment for non-reducible Right Spigelian Hernia.Thus this lump should be concerned by you and need to take care as suggested urgently.Hope this reply would help you to evaluate your case and treat it with your doctors in time to come.Hope this would resolve your query and worry and Anxiety accompanied with it.Welcome for any further query in this regard to ME.I would love to help you out.Awaiting for any further query.Wish you fast recovery from this intriguing health problem.Have a Good Day.Dr.Savaskar M.N.M.S.Genl-CVTS -Senior Surgical Consultant"
},
{
"id": 59408,
"tgt": "Diagnosed with fatty liver, dyslipidimea, reduced weight. How much time will it take to be normal?",
"src": "Patient: sir i have been diagnosed with grade 2 fatty liver and dyslipidimea,iam 27 yrs of age,my lipids were also also raised,but now after taking medicines and strict diet they are under control in 1 and a half months of time and i have reduce 8 kgs..any guidelines for me??how much time it will take for a fatty liver to be normal Doctor: hello welcome to HCM from the details u have given , u were having a clinical condition called as NON-ALCOHOLIC FATTY LIVER . it is most commonly seen in obese persons.. fatty liver is asymptomatic and doesnt create any problem for you, instead it is a warning signal for u ..to take acre of ur body ok..now u already had realised and started doing work outs..which is a good thing..ok fatty liver gradually subsides , when you start shedding your weight and end up with mild grade only, some times , it may remain like that with out causing any symptoms. so dont worry much about fatty liver now,,first take care of your weight and check your cholesterol levels ..try to keep them low. hope this will help u ..bye"
},
{
"id": 151065,
"tgt": "MRI shows multiple cervical levels posterior disc bulge, protrusion causing thecal sac indentation with effacement of bilateral neural foramina. Suggestions?",
"src": "Patient: Dear sir My father suffering in last 5days because right side shoulder and R.H Arms and fourarms pain. Yesterday he gone to hospital and doctor suggested to take MRI scan . today we done scan and received report. But the report shows that the following problem: Disc dessication changes at multiple cervical levels posterior disc bulge/protrusion at C2-C3, C3-C4, C4-C5 and C6-C7 levels causing thecal sac identation with effacement of bulateral neural foramina at C3-C4 and C4-C5 levels. I request you to kindly tell me what is the real problem causes and solution. please guide me next procedure for treament to cure this problem Doctor: Dear Venkat, It sounds like your father is suffering from degenerative cervical disc disease. This basically means that because of wear and tear, the cervical discs at many levels in your father's spine are causing some compression on the nerves and spinal cord. I would advise you to consult a Neurosurgeon who will examine him and decide if a surgical option will be advisable keeping his age, general health, clinical examination findings and the MRI scan findings in mind. Hope this helps."
},
{
"id": 123137,
"tgt": "What could cause throbbing in thumb and numbness in finger next to it?",
"src": "Patient: Hi, I had right thumb fusion in 2008. Thumb was still aching so I went back to the doctors and just had the hardware removed and tenolysis done 3 weeks ago. The last couple of days, my thumb has been throbbing and the finger next to it has been going numb. It this normal. Doctor: Hello, It could be a neuropathic pain. As a first line management you can take drugs like gabapentin as it is quite effective for neuropathic pain.In severe cases you might require an MRI scan. Hope I have answered your query. Let me know if I can assist you further. Regards, Dr. Shinas Hussain, General & Family Physician"
},
{
"id": 12398,
"tgt": "Suggest medication to completely cure psoriasis",
"src": "Patient: I have psorirasis from one year. I have used so many medicines up to now. Homeopathy also i have implemented but invain. Please guide any suggested medicines for my psoriasis. I having high percentage on my scalp, on body 20% of it. Please guide me Doctor: Hi..As you said you suffer from psoriasis.Psoriasis is a chronic relapsing and autoimmune disease. You may be having scaly plaques on various parts of the body,scaly lesions on the scalp. Exact cause is not known. Genetic tendency, autoimmunitity,stress,dry wheather...etc may be responsible for precipitation and exacerbation of the disease.You consult dermatologist for firm diagnosis and treatment.There are different modalities for the control of psoriasis. Relapses may be prevented by regular and proper treatment. I usually recommend cap acitretin 10 mg twice a day along with methotraxate 15 mg weekly in three devided doses 12 hourly. If there is itching, you may take antihistaminics like levocetirizine. Treatment may be taken for long time till good response. Blood tests may be done to monitor the treatment. You may apply mild steroid cream, combined oint of steroid and salicylic acid or calcipotriol oint on the lesions. You may apply moisturiser to improve the skin texture. Avoid soap bath for few weeks.. You should avoid stress and worries as that might worsen psoriasis. You might get good result after few months regular treatment. Have patience and avoid tension.I hope you got my answer.Thanks.Dr. Ilyas Patel MD"
},
{
"id": 70883,
"tgt": "Can air conditioners cause chest discomfort?",
"src": "Patient: i have never been effected by old air conditioner that was previously installed in my house four years ago. I am not using any air conditioner since last four years. Yesterday i bought a second hand small air conditioning unit; i slept a night with this unit running and after that, i am feeling LITTLE CHEST HEAVINESS . Could this be cause of this second hand air conditioning unit or something else ? I dont have any other medical issue Doctor: Hello, Yes, this can lead to chest tightness. Old air conditioners are not hygienic. Air provided by them is not humidified. So inhalation of this air can lead to bronchitis and this in turn leads to chest tightness. So better to check filters and humidifiers of your second hand AC. This chest tightness usually recovers in 1-2 days. But if it is prolonged then consult pulmonologist. Hope I have answered your query. Let me know if I can assist you further."
},
{
"id": 157709,
"tgt": "Subserosal fibroid cyst on ovary. Bleeding, nausea. Reports show growth in cyst with less debris. Cyst removal? Cancer?",
"src": "Patient: Hello, Could you so kindly help me out? I have questions regarding a subserosal fibroid cyst on my ovary. I am post menopausal, have had a small amount of bleeding for two years and now have nausea daily. |The cyst was 7cm three months ago now it is 9cm. The report said there is less \"debris\" in the middle of the cyst now. What are the indicator/s for cyst removal? What does a cancerous cyst look like? Thank you. Doctor: HelloThanks for your query.A cancerous cyst generally has the following features -Rapid increase in size.Echogenities, calcifications and solid components inside the cystIrregular cyst wallPresence of ascitesPositive markers like CA 125Symptoms like weight loss, abdominal distensionFamily history of cancers.A cyst more than 5 cm, especially one that has increased in size over time, warrants removal, especially after menopause.All the best."
},
{
"id": 13499,
"tgt": "What causes rashes on the back and shoulders?",
"src": "Patient: I have a rash on my back, shoulders, underneath my breast, groin with a burning and soreness on my shoulders and bra area, with skin peeling all over my body. I take hydroxychloroquine 200 mg. Can you tell me what\u2019s going on. This is my second episode in 2 weeks. Doctor: Hi, The rash could most probably be a frictional dermatitis or a Tinea infection (fungal infection). As you are having sore, I request you to apply an antibiotic ointment and take some antihistamine tablet for a week. If you do not get any improvement in a week, I recommend you to consult your dermatologist for further management. Hope I have answered your query. Let me know if I can assist you further. Regards, Dr. Siva Subramanian, Dermatologist"
},
{
"id": 171854,
"tgt": "Suggest remedy for chronic cough due to accentuated bronchovascular markings",
"src": "Patient: my 2.5 yr old son is suffering from cough dry initially and now quite mucous for last one year. His latest x ray shows that he has accentuated bronchovascular markings. Nad he weighs only 13 kgs and ht is around 3 feets. He looks weak and chronic coughing . What should i do Doctor: HiCan you please elaborate the symptoms.Is the cough associated with cold or breathlessness? Is there any family history of similar complaints? Is there any exposure to allergens or pollen? The accentuated bronchovascular markings on the x ray chest can be because of repeated lower respiratory tract infections. You should visit a pediatrician for detailed evaluation as we need to rule out whether if it is cough variant asthma or if there is any evidence of lower respiratory tract infection like tuberculosis or any cardiac involvement."
},
{
"id": 183136,
"tgt": "What is the treatment for dental cavities?",
"src": "Patient: my daughter went to a dentist and have 4 cavities filled on Tuesday. She bit the inside of her cheek and lip several times and now it's very sore, white/yellow in color and one has cracked open, she can hardly open her mouth. Should I take her to the doctor? Doctor: Hello,Numb areas can include the cheek, lip and tongue with dental anesthsia. A risk can include biting these areas before the anesthetic wears off. This trauma will cause swelling. In general you don not have to worry. Initial placement of ice is helpful to limit swelling. Tylenol, at the appropriate dose, can reduce inflammation. Warm salt water rinses and benzocaine gels can help provide some relief. Help your daughter avoid the area and maintain a soft diet. Normal sensation should have returned to her mouth.Inform your dentist of this condition. Consider completing less dental treatment in a visit, so there is less numbness afterwards. A follow-up visit will be needed if condition seems to be getting worse rather than slowly improving. Time to heal will usually be all that is needed.Thank you for your inquiry. I hope that your daughter feels better soon."
},
{
"id": 170087,
"tgt": "Suggest treatment for mild rash and vomit in a 15 weeks old child",
"src": "Patient: my 20 week old son - corrected age is 15 weeks , born via emergency c sec i had major previa and haemmoraged and he was born not breathing , with fluid on his lungs nd was revived then ventilated and remained in scbu for 2 weeks - he has a virus 2 weeks after getting his first immunisations but i am concerned about his face being grey around his mouth eyes and nose, chest is clear he has a mild rash and is vomiting. i often thought the grey areas were wind but the eyes and nose shouldnt be so grey should they !? he sometimes wakes up gasping because he has missed a few breaths in his sleep ( apnea ? ) should i be concernedthat his oxygen levels are low? he was also born with blood level of 8.9 but turned out to not need a transfusion, his levels dropped to 8.6 at 4 weeks old but they ave not followed it up since. he does ave folic acid, sodium feredate andd dalivit daily so i presume these help maintain his levels. sorry if this is hit n miss info, so many points to make to try to get a more accurate diagnosis. eoh and he is still feeding and soiling nappies well sincere regards sincere regards Doctor: Hi, Welcome to hcm.I have read your question in detail and i know that you are very concerned about your child.Mild rash and vomiting is normal in a 15 week child, for this you have to do burping more. All babies vomit a little milk after feed , this is a normal thing.Greying of face around mouth, eyes and nose with few apnea episodes is a matter of concern. It could be due to congenital heart disease or hypoxia. I think an echocardiography should be done to rule out congenital heart disease. Also a complete bolld count should be done to know theexact hemoglobin levels. It will guide us that whether we should continue hematinics or not.I hope this will help you.Take care."
},
{
"id": 90232,
"tgt": "What causes the constant pain in my right lower abdomen?",
"src": "Patient: i have just turn 55 this pain started more than 18 months ago, constant pain in my right lower abdomen, blood test, CT scan of abdominal/pelvic, hysteroscopy and abdominal biosy, colorectal and the last is MRI where I drank liquid and the they gave IM die during the MRI session awaiting the result I am feed up. I can not afford to pay any money that is why i am trying to find out through the internet if my GP or the team have over look or missed any thing out and I live in England Doctor: Hi.Thanks for your query and faith in HCM.The commonest causes of pain in right lower abdomen in a female patient of 55 years are:AppendicitisEnteritis of a chronic nature like tuberculosis, Crohn'sColitis of chronic nature.swelling or a mass in the right side of the fallopian tubes and / or ovary.This will be helped by the investigation you have undergone and you will get a proper treatment."
},
{
"id": 33454,
"tgt": "Suggest remedy for red and inflamed sores on thigh",
"src": "Patient: I have a big spot on the inside of my thigh, it is very sore when i walk. Its red and looks inflamed. I've put some cream on to see if it helps, but its very sore when i walk and i have golf tournament tomorrow how will i stop it hurting for tomorrow. Thank you. Doctor: Hi,From history it seems that there might be having ingrown hair follicle infection producing induration and swelling.Go for one antibiotic and anti-inflammatory medicine course for 3-5 days.Do not try to pop up the spot as it might aggravate the infection.Ok and take care."
},
{
"id": 189681,
"tgt": "Pre removal of wisdom teeth caused major swelling of cheek. Should I go to a dentist?",
"src": "Patient: Hi Doctor, I went to the dentist to remove my 5th wisdom tooth (in the upper right jaw). The local anesthesia which was given pre-removal caused major swelling of my cheek. I was told by the dentist that the injection hit the blood vessel . Immediately after swelling I applied ice pack for about an hour with an interval of 1 min. I took antibiotics for 4-5 days and the swelling is reduced a bit but not completely. I have terrible pain when i touch my cheek on the injection spot. Its been 6 days now. Could you please tell me what needs to be done now? Right now I am in another city and could not go to the same dentist... Please advise what can be done. Thanks,, Doctor: Hi, Thanks for asking the query, The swelling which appeared is known as Hematoma , it is painless distended swelling . It occurs due to effusion of blood into the extravascular spaces during the injection of the local anesthetic solution . Immediate management of hematoma is application of external pressure over the affected area . It can be relieved by application of ice packs on the affected area. Take complete course of antibiotic and analgesic . Usually it takes 7-14 days to resolve . Hope this helps out . Regards..."
},
{
"id": 75349,
"tgt": "What causes unshapely chest?",
"src": "Patient: My chests are different in shape. What is this? I'm going for gym. I felt no problem other than shape of my chests. 17 years 5feet 7 inches 60kg at the age of 3 i got a heart surgery for hole in heart.but i had never felt difficulty of that till now. So please help me doctor. Doctor: Hello and thank you for asking.I have seen several patients with this condition as you are.If you do not have any complains for example difficulty in breathing cronic cough with sputum or not or any other concerns and you do normal daily activity i recommend you not to worry.Sometimes according to the type of heart surgery you had thoracic cage may not have the same form.But if this situation is new it is better to see the GP and to do a chest X Ray.Wish you good healthThank youDr.JolandaPulmonologist"
},
{
"id": 168673,
"tgt": "Is vomiting during medication normal?",
"src": "Patient: hi i have a 2 years old daugther who is been sick for 2 days by now I start giving her this medication (enterogermina) at the begining she was vomiting so that is why I gave her the medication and now she is not vomiting but is having diarrea Is that normal ? Doctor: HiNo, this isn't normal and she needs to see a doctor since she may have either a viral or a bacterial intestinal infection and she may need a course of antibiotics"
},
{
"id": 147349,
"tgt": "What causes tingling sensation and goosebumps on thigh?",
"src": "Patient: Hello. I have been having tingling and goosebumps in a strange pattern on the top of my left thigh. I have been having neck, upper back and shoulder pain as well and pain that radiates down my arm. Went to the Dr. and she said it couldn't be from my neck. Just wondering if it is something I should get checked. It started last Friday and has been more frequent happens sitting or standing. Doesn't hurt at all, feels like a chill. Thanks. Doctor: Hi,Thank you for posting your query.I have noted your symptoms. The description of your symptoms is suggestive of a condition called as meralgia paresthetica.In this condition, a skin nerve (lateral femoral cutaneous nerve of thigh) located in thigh gets pinched at the level of groin. This can result in pain, tingling and burning in the thigh region.You can get good relief from medications such as gabapentin or pregabalin capsules.I hope my answer helps. Please get back if you have any follow up queries or if you require any additional information.Wishing you good health,Dr Sudhir Kumar MD (Internal Medicine), DM (Neurology)Senior Consultant NeurologistApollo Hospitals, Hyderabad, IndiaClick on this link to ask me a DIRECT QUERY: http://bit.ly/Dr-Sudhir-kumarMy BLOG: http://bestneurodoctor.blogspot.in"
},
{
"id": 147782,
"tgt": "Is there any other methods to get cure from herniated disc and sciatica?",
"src": "Patient: Hi doctor, I'm suffering from herniated disc and sciatica, I've followed the treatment so it relieves me. but when I stop this treatment, the pain come back. so I'd like to ask if is there any solution without drugs, example walking or swimming ...I don't know.thanks. Doctor: Hi , As diagnosed with PIVD with radiculopathy it should be mananaged conservatively with regular physiotherapy (spinal extention exercises), avoiding sternous work/exercise bending forward , warm compression with vitamins , analgesic , muscle relaxant (if pain ) n PPI under guidance of orthopaedition . If symptoms aggravates, bladder bowel involve , neurological involvement ...... then may require decompression . So in my opinion be in regular follow up and treatment accordingly including physiotherapy .hope it ll help Thanks n regards"
},
{
"id": 219867,
"tgt": "What causes pink vaginal discharge during early pregnancy?",
"src": "Patient: Hi i had a positive hpt on 38 th day of my cycle ,i had visited my pcp for conformation .she said that i am 4 to 6 weeks then an today is 47 th day of my cycle and i found very light pink (creamish)dischare.i din't book appointment with gynic yet. Is there anything to be worried?i am not having any cramps and this is my first pregnancy Doctor: Hello, and I hope I can help you today.15 percent of women have some bleeding during the first trimester of pregnancy. Lots of new blood vessels are growing in your uterus and cervix, and as the placenta develops it too can bleed sometimes. As long as you don't have any pain along with the bleeding (occasional cramping is normal, but it should not be as bad as a period) there is really nothing to worry about.I hope I was able to answer your question today and that this information was reassuring.Best wishes,Dr. Brown"
},
{
"id": 155906,
"tgt": "Suggest treatment for Acute Myeliod Leukemia",
"src": "Patient: I have AML and the pulsing in my ears that used to go when I had a blood transfusion, doesnt seem to go any more after receiving blood. I used to only notice it at night, but now Im aware of it in the day as well. Any suggestions of what it can be and what can I do? thank you very much Doctor: well such ear symptoms are seen in lekemias with high wbc and this can be treated by diluting blood or with corticosteroids. all other measures are the same as for every type of leukemia so consult your hematologist about it.wish you good health. Regards"
},
{
"id": 207977,
"tgt": "Suggest remedy for mental health problem",
"src": "Patient: Hi, Two nights ago I had a panic attack. Probably the worst attack I have ever had. I ve had panic attacks in the past, but this was the first time where I felt a tingling sensation throughout my body and face. It felt the same as when your leg or arm falls asleep. Since that night, I have noticed a random burning rubber smell. Not all of the time, just once in a while. Should I be concerned or shrug it off? I am 23 years old, 6 2 , and 150lbs. Doctor: thanks for your query, In my opinion, if you had panic attacks in past and the last two episodes are also the same except that they were in the night and that these episodes were associated with tingling sensation and there were no other symptoms , then you need not to do anything special. wait for some time, practice slow, deep breathing, if it does not get better, consult a physician"
},
{
"id": 179594,
"tgt": "Suggest treatment for skin rash along with loose motion",
"src": "Patient: baby recently started scratching his lower back persistently and a rash appeared. He is also refusing meals and seems to be more tired as usual. He has no nausea or vomitting but appeared to have more frequent stools. He has not gained weight since 4 weeks Doctor: HiThanks for writing to health care magic.Diarrhea is usually viral and may last for 5 days.Give ors and zinc.Ensure minimum 6 times urine output in a day.Try avoiding dairy products Siloderm ointment for rash can be appliedWishing your child good healthRegardsDr Arun"
},
{
"id": 110411,
"tgt": "What is the treatment for avascular necrosis of hip?",
"src": "Patient: Hello, I am a 51 year old female that weighs 130 lbs and is 4'10\" I have been diagnoised with avascular necrosis of the hip.. I as wanting to know why my right leg is getting shorter than my left and can it be fixed.. have an appointment the 1st with ortho doctor.. was just curious Doctor: Hi,As you are diagnosed with avascular necrosis of hip.As ball of your hip joint collapsed leading to shortening of effected limb.You will probably require total hip replacement ,after which both of your limb should be equal.I think this will be of help to you."
},
{
"id": 89284,
"tgt": "Suggest remedy for vomiting sensation, abdominal pain",
"src": "Patient: Hi my name is luis i have recently started having stomach trouble i am always feeling very sick wanting to vomit and having abdominal pain i was wondering if you can help me find the problem my stomach is very sensitive with the foods i eat and the amount i consume as we ll Doctor: Hello,If you have constant stomach upset/ discomfort associated with vomitings or nausea, you have to get evaluated , pain in the stomach can be because of various reasons, ( Dependimg on the site of the pain- appendicitis , Gastritis , pancreatitis or kidney stones or anything non specific), please kindly consult your doctor and get an USG abdomen, UGI scopy and few blood tests done. Discomfort related to food could be sometimes a gastric trouble, Kindly consult and get evaluated. Hope I could help you.Thank you."
},
{
"id": 84429,
"tgt": "Can unwanted 72 increase HCG levels and how HPT positive?",
"src": "Patient: I have 7 months old daughter and missed period on 26th June. Then had unprotected sex and taken unwanted 72. Now I am vomiting with severe stomach pain and raised heart beat. I test positive for preganancy after a week. would it be possible that HCG has increased due to this pill as stated on test instructions? Doctor: Hello,It may be due to pregnancy. No any contraceptive gives 100 %protection. So for further assessment you may require beta hcg and obstetrics ultrasound after gynaecologist consultation. Further treatment mainly depends upon the underlying condition. Keep your self hydrated Hope I have answered your query...Let me know if I can assist you further.Regards,Dr Shyam kaleFamily and general physician"
},
{
"id": 151343,
"tgt": "Persistent speech problems, no fluency in reading in writing after brain stroke on left side, undergoing speech therapy. What else can be done?",
"src": "Patient: Dear Sir, 7 month before my husband had a brain stroke in left side. Now he can move his body parts easily and still he is in medical supervision. he still have the BP problem and it is now 100/80 and taking medicine that is Amtas 10, Qpill 10 and hytide 12.5. He also took strocit plus for 3 month according to the doctors advice. The problem still now is in speech and can not read and write fluntly though he is having speech therapy . can you advice me what should i do to improve his situation and how long it will take to became as previous. Doctor: Thank you for contacting Healthcare magic. Speech problems (aphasia) are common after stroke on the left side of brain (in right-handed people). This is because the speech areas (for both understanding and producing speech) are located on the left side of brain. Recovery is a gradual process and can take up to two years (though some people do not recover completely). The best treatment is speech therapy (which your husband is undergoing). There are no specific medications to improve sppech. However, in some research studies, piracetam has been found to be useful. You may use pitracetam (nootropil) 1200 mg twice daily for three months. Please consult your doctor regarding this. Best wishes, Dr Sudhir Kumar MD DM (Neurology) Senior Consultant Neurologist Apollo Health City, Hyderabad"
},
{
"id": 142661,
"tgt": "Could the disability in the left hand be due to the MS flare up?",
"src": "Patient: hes , hi i have MS and i have had 4 strokes ( TIA S ) in the past 4.5 yrs , well its been a month and i have no function of my left hand and arm is numb , had a MRI with and without contrast and showed no stroke HOWEVER hospital wanted to keep me over night for IV Steroids but could not because i care for my parents who are very ill . Could it possible that i have had a MS flare up ? i have had them before but not this bad where i have no use of my L hand ! Also i had a chest xray done today and i have had a R node on my lung ,and today its still there but now my L lung has a bigger node what could this be ? Doctor: Hello!Welcome on Healthcaremagic!I read carefully your question and would explain that it is necessary investigate for the nature of these lung nodes, which could just inflammatory lymph nodes or a tumor. An infection can not be excluded either. Biopsy may be needed if they persist. From the other hand, the fact that the doctors wanted to put you on IV steroids after the MRI results, indicates that your symptoms are related to a relapse of MS and IV steroids would help improve your situation. You should discuss with your doctor on the above issues. Kind regards, Dr. Aida"
},
{
"id": 140532,
"tgt": "Do i need a surgery for an injured coccyx bone?",
"src": "Patient: I have pain in my left upper leg and spinal, due to this i could not sit for log time then it is purning. I have taken MRI Spinal but no issues in Spinal but coccyx bone is injured. Doctor adviced me to go for surgery. Is it required ? because it is root cause for my backpain, foot and heal pain?. Doctor: Hello, Coccyx injury may cause back pain, which increases on sitting. After surgery that pain would go. However, foot and heel pain are not related to coccyx bone injury. You require further investigations such as nerve conduction studies and serum uric acid to determine the root cause of your problem. Hope I have answered your query. Let me know if I can assist you further. Take care Regards, Dr Sudhir Kumar, Neurologist"
},
{
"id": 202216,
"tgt": "What is the solution for chronic nightfall?",
"src": "Patient: i m suffering from chronic nightfall..in starting it was normal but know it coming three days in a week.... i dont do masturbation still it comes i am fed up stomach constipation stress all happening want to get permanent solution please suggest some remedies for thios Doctor: HelloThanks for your query,based on the facts that you have posted it appears that you have problem of night emission.Night emission is common at the age of adolescence and there is nothing abnormal.Following general measure will help you to boost up your confidence and getting good sleep and improve your health and there by reduce night emission.1) Practice regular exercise for 45 minutes followed by meditation for 1/2 an hour in the morning.2) Take high protein diet rich in vegetables and fruits and Vitamin A,C,D,E.and Zinc3)Take anti oxidants like Almonds 5-6 everyday..4) Avoid alcohol and smoking..There are no medication that can reduce the night emission.It normally gets cured itself without any treatment.Dr.Patil"
},
{
"id": 194662,
"tgt": "What causes bumps with puss on genital area?",
"src": "Patient: okay, i am helping out a guy friend. He is to scared to go to his parents about it.so he has little white bumps on this genital area and when he squeezed it some puss came out and a little blood followed. its pimple like but a little smaller. what could he have? Doctor: Hello, He may have an infected gland in the scrotal skin. He may need to wash it with water and soap and betadine. if it is persistent he may need oral antibiotics. Hope I have answered your query. Let me know if I can assist you further. Take care Regards, Dr Manuel C See IV, Urologist"
},
{
"id": 66180,
"tgt": "What is causing painful lump on my elbow ?",
"src": "Patient: I have a painful lump about and inch in size, in the inside crease of my left elbow, lower than where the nodes are. I have not hurt it.. I am right handed ...meaning i haven't pulled it .i have no money..is there a forum perhaps where someone else had this problem Doctor: Hi, thanks for writing to HCM.In fact, If I were your treating Doctor for this case of an elbow lump, I would come up with four possibilities, these include: 1.\u00a0\u00a0\u00a0\u00a0\u00a0lymph nodes due to some chronic disorder like some skin/joint infection or chronic throat / neck infection or even malignant diseases in elderly people!2.\u00a0\u00a0\u00a0\u00a0\u00a0The second possibility is of benign and simple conditions like lipoma or neurofibroma; there is nothing to worry about these!\u00a0\u00a0\u00a0\u00a0\u00a03.\u00a0\u00a0\u00a0\u00a0\u00a0The last possibility is of sebaceous cyst or some other cysts like ganglion or bursa! Nothing to worry about these also!4. parasitic cyst; must be treated...!I suggest you to go for a needle biopsy of the lump for confirmation and to relieve your concerns!Hope this answers your question. If you have additional questions or follow up questions then please do not hesitate in writing to us. I will be happy to answer your questions. Wishing you good health."
},
{
"id": 223320,
"tgt": "Is cycleonorm E and P after having unprotected sex safe medication to avoid pregnancy?",
"src": "Patient: Dear sir, i have done unsafe sex with my partner....what will i do ?? bt i dont know my sperm ejaculated in vagina so i have given cycleonorm E and P to my partner...she is taking first time this tablet....and her period has gone 30 December....so please give me advice i dont want pregnancy.... Doctor: Hello,Thanks for sharing your health concern with us. I have gone through your query and would like to offer some suggestions. Firstly, ovulation occurs two weeks prior to the onset of the next period and this is the fertile phase in the cycle when an unprotected intercourse can cause pregnancy. Around the menstrual period, it is safe to have intercourse without risk of pregnancy. The medication is not of much use as an emergency contraception. You have to use a proper pill or go for the insertion of an IUD as an emergency contraceptive. This should be done as soon as possible to prevent failure. Hope this helps."
},
{
"id": 19928,
"tgt": "Suggest treatment for chest pain",
"src": "Patient: hii have chest pain right in the middle of by breast that comes and goes, i have also had a cough for the last two weeks i'm bring up green flem. i must add that i quit smoking four days ago because im worried about what im feeling and what it could be.thanks in adavance Doctor: Hello, Thanks for posting your query on Healthcare Magic...* Brief Answer: chest infection can cause chest pain and tenderness, nothing serious.*Explanation: My opinion is that chest pain has quite a differential diagnosis. first of all we should exclude serious causes such as cardiac conditions. One of the most common causes of chest pain and tenderness is musculoskeletal pain accompanying chest infestion and chronic cough. as you are a chronic smoker that is a high probability.I suggest my patients the following : chest examination , chest x ray would be helpful , colorful sputum is an indicator of chest infection.The information provided by you is not sufficient to provide a good opinion. If someone comes to me with this condition I would ask them about other risk factors and comorbidities like diabetes , etc.*Conclusion: I suggest you check up with your general physician. If you have any further questions I will be happy to help, If not, you can close the discussion and rate the answer. Wish you good health."
},
{
"id": 39622,
"tgt": "Suggest treatment for jaundice",
"src": "Patient: hi i suffering from Jaundice from Nov 2011 still it is not recover I am on full diet like not having spici, oil, nonveg etc. Taking Tablet Liv52ds & Liopill My Burlibin is t 1.72 & d 1.32 I getting leg pain heavely Plz suggest me the soluation Doctor: Dear Friend.Welcome to HCM. I am Dr Anshul Varshney. I have read your query in detail. I understand your concern.We need to find out the cause of Jaundice.It might be Gilbert syndrome.But i can comment better if you can get me following investigations:1. Liver Function Test2. Viral Markers3. Ultrasound Abdomen.Review with me once you have the reports.This is my best advice for you based on available details. If you have any further query please ask me.Stay Healthy.Dr Anshul Varshney, MD"
},
{
"id": 31642,
"tgt": "Will pus cell infection in urine during pregnancy affect fetus?",
"src": "Patient: hi I am 6 weeks pregnant and my urine test shows that my pus cells are 180 - 200. my doctor put me on anti biotics while waiting for the urine culture rrsult. I am worried that this infection might have affected the fetus already. what are the possibilities? Doctor: HI, thanks for using healthcare magicIf you are not systemically unwell and the pus was discovered by routine examination, this is not likely.The baby would likely be ok. Continue the medication as prescribed and speak to your doctor if you have any complications of the infection or the medication.I hope this helps"
},
{
"id": 120651,
"tgt": "Suggest treatment for multiple sclerosis",
"src": "Patient: I have MS and U have had a weird fluttering feeling in my left side. Can t tell if it s in my rib cage or my lung. Doesn t hurt, I don t feel out of breath but it s annoying! Could it be an MS symptom, just saw neuro he said hard to tell ! However, it doesn t seem to be as bad as it was a week ago. Doctor: Hello,I read carefully your query and understand your concern. Your symptoms seem to be related to multiple sclerosis. The MS hug is a collection of\u00a0symptoms caused\u00a0by\u00a0spasms\u00a0in the intercostal\u00a0muscles. These\u00a0muscles\u00a0are located between your\u00a0ribs. They hold your\u00a0ribs\u00a0in place and help you move with flexibility and ease.So,I suggest using a muscle relaxant such as Baclofen for muscle relaxant. I also recommend using magnesium supplement. Hope my answer was helpful.If you have further queries feel free to contact me again.Kind regards! Dr.Dorina Gurabardhi General &Family Physician"
},
{
"id": 106641,
"tgt": "What causes abdominal pain along with lower backache?",
"src": "Patient: Seen in the ER for severe abdominal pain, bloating, fullness, extreme lower back pain, and gas/indigestion. Ruled out gastrointestinal but doctor stated I have inflammation in my cervix. No signs of any STDs. CT scan AP pelvis shows endometrial fluid, appears to be mild intrauterine fluid, cervix is prominent but not definitely changed. What could be the underlying cause? Doctor: Hello and Welcome to \u2018Ask A Doctor\u2019 service. I have reviewed your query and here is my advice. Can be due to disc related cause or IBS I hope this information has been helpful for you. Regards, Dr. Praveen Tayal"
},
{
"id": 74359,
"tgt": "What is the life expectancy for pulmonary fibrosis patient?",
"src": "Patient: My 90-year-old mother who has pulmonary fibrosis and has just survived a bout of double pneumonia is back in hospital with suspected but not diagnosed pneumonia. We children are in agreement that we should be focused on making her as comfortable as possible and her doc has boosted her oxygen to 100% and put her on morphine. Any way of estimating how long she might last in this situation? Doctor: Thanks for your question on Healthcare Magic. I can understand your concern. It is very difficult to predict exact life expectancy in IPF (Idiopathic Pulmonary Fibrosis). Since her age is 90 years and she needs 100% oxygen, I think end is near. In my opinion, her life expectancy is not more than 3-4 months. Hope I have solved your query. I will be happy to help you further. Wishing good health to your mother. Thanks."
},
{
"id": 201596,
"tgt": "Suggest treatment for painful and swollen scrotum",
"src": "Patient: Hi Doctor. Inside my penis I had some small growth on the skin that the doctor told me is called verucca tyle of whats. I was given a cream that has made my srotum swell and cracks due to the thickness. Am now in serious pain and am fearing an infection since am diabetic.Please advise on how to tavkle the swelling and the pain...tom Doctor: Thanks for contacting HCMI am sorry to hear that you are having swelling and pain of the scrotum. This sounds like you do have a bacterial infection and need additional treatment. I recommend that you seek medical attention from a urologist or your primary care giver. You are at a higher risk of infection due to you being a diabetic. Hope I answered your question. Please contact us again with your medical questions or concerns"
},
{
"id": 202627,
"tgt": "Redness on scrotum, smell, discharge, loss of appetite, sleeping a lot. Treatment?",
"src": "Patient: Hi, I ve just changed my little boys bottom and noticed the end of his penis , where his scrotum is, is very red, when I pulled it back it smelt and it looks like there s a bit of discharge , he s not been very well this week, behaving oddly, crying a lot, hardly eating, only drinking milk and sleeping lots, and now this. I m really quite concerned as he has come back from his dads today having still not eaten or drank much and still behaving weird. Usually he eats more at his dads! Something isn t right. Please help! Doctor: Hi,From history it seems that there might be having some bacterial infection due to poor local hygiene giving rise this problem.Due to this infection he is not feeling better, fatigue and loss of appetite.Clean the part with antiseptic lotion and apply antibiotic cream locally.Give him one course of antibiotic medicne for 3-5 days to cure infection.Ok and take care."
},
{
"id": 8663,
"tgt": "Will the use of anti-aging skin cream cause peeling and thinning of skin?",
"src": "Patient: Hi dr.I m 29 years old female with sensitive,mixed and hairy skin.I hve heard tretinoin creams are helpful in anti aging .I have recently purchased a tube retino-A WITH 0.025% tretinoin bt little afraid of using it as I hve read that it makes skin very dry often resulting in peeling and prolonged use can thin skin.please help can I use it with my night cream as an anti aging product. Doctor: Hi Piya, To prevent ageing multiple options are available like sunscreens, retinoids( retinol creams or tretinoin), vitamin c based creams, Glycolic acid creams, peptides, estrogen based creams, chemical peels, lasers, microdermabrasion, Botox and fillers. These can be used alone or in combination as per the skin's requirement. in your case, you may begin with these Use a good sunscreen daily Taking vitamin c and antioxidant supplements would be beneficial. Vitamin c helps in collagen synthesis and antioxidants prevents free radical damage Use a good moisturiser suitable to your skin type to keep your skin well hydrated. There is no harm in using tretinoin( vitamin A derivative). Tretinoin induces collagen production and helps in shedding of dead layers of skin thus helping in reducing fine lines. However it may irritate your skin. Use it in small quantities in night and do not rub it over face. Use of sunscreen in morning is mandatory with retinoids as it makes skin sensitive to sun. Initially in the beginning use it for two hours and as the skin sensitivity decreases you may apply it overnight. As you are still 29, you may also use a night cream with vitamin A,C,E to prevent ageing. The chances of irritation with such creams would be less as compared to tretinoin. Hope that answers your query. Take care."
},
{
"id": 128169,
"tgt": "What are the symptoms of leg syndrome ?",
"src": "Patient: My granddaughter is 2.5 years old. She is obese by the weight charts. Her legs are very heavy. This is where most of her wieght is. She is active and happy, but has trouble sleeping at night. Could this be an indication of swelling or water retension? Or some kind of a restless leg syndrome? Doctor: Hello, I have studied your case. Due to weight gain there is excess pressure on legs ,leading to swelling.there can be associated kidney function imbalance.You can take high protein, and vitamin diet,Avoid fatty, oily, junk food.Drink more water daily.Hope this answers your query. If you have additional questions or follow up queries then please do not hesitate in writing to us. I will be happy to answer your queries. Wishing you good health.Take care"
},
{
"id": 34708,
"tgt": "What could elevated WBC and neutrophils despite regular intake of antibiotics indicate?",
"src": "Patient: Hi I have cushing disease, lyme disease, fibromyalgia, occipital neuralgia and have been treated with antibiotics for over a year now and wbc and neutrophils are still elevated and don't understand why if being on antibiotics for over a year could this mean something Doctor: Cushing disease itself can cause elevated WBC, so this may not necessarily be related to an infection. Certainly a year of antibiotics should have taken care of most infections, depending on what antibiotic was used. Unusual infections or internal infections (eg, abscess in a muscle or organ) might cause a persistent elevated WBC, as might a chronic inflammatory condition. You would probably be having other symptoms, such as persistent fevers, pain, or weight loss, if that were the case. You don't say how high the WBC count is, so it is hard to assess its severity. Be sure the Cushing disease is well treated first. If you are still having elevated WBC, you might need further evaluation or need to see a hematologist to get a specific diagnosis. Hope this helps."
},
{
"id": 125711,
"tgt": "How can extreme pain below the ribcage after an injury be treated?",
"src": "Patient: I am a martial art player. I was hitted on left side of middle back just below the rib cage in fight about 1 year ago. From then i have an extreme pain just 2 month before hit in left back muscle and in spinal cord from 6 months i can t sit longer if there is no support to my back and when i immediately turn or kick then i feel a splash in my that point . I use many medicines but i can t get relief from pain. Plz i need help Doctor: Hello, It might be due to minor contusions after the injury. As of now, you can use analgesics/anti-inflammatory combination like Aceclofenac/Serratiopeptidase for symptomatic relief. If symptoms persist better to consult a physician and take a CT scan. Hope I have answered your query. Let me know if I can assist you further. Regards, Dr. Shinas Hussain, General & Family Physician"
},
{
"id": 196781,
"tgt": "What causes gelatin like lumps in the semen?",
"src": "Patient: my boyfriend's semen is normal looking but has chunky pieces in it jello-like in texture and they stay jello-like they dont melt or anything i know he used to do a lot of drugs before we started dating is it just side effects from smoking crack? (he's been clean for a year and half now) or is it infertility? does he have an STD? cause before we started dating i made him get checked out considering his past and he turned up clean on the tests. he says that his semen has had chunks in it for quite awhile but it kinda conserns me like i dont want to catch anything and i do want to have a child someday Doctor: Hi and welcome to Healthcaremagic. Thank you for your query. I am Dr. Rommstein, I understand your concerns and I will try to help you as much as I can.These symptoms and signs are suggestive of STD especially if there is associated pain, fever, urethral discharge and recent sexual intercourse. So this is the first thing to rule out by doing STD testing. Second, it may be sign of uroinfection o urinary stones and some other genital infection but this is less common. In every case, antibiotic therapy may be required so test should be done as soon as possible so that appropriate treatment can be started. Till that try to drink plenty of fluids, avoid sexual intercourse and rest.I hope I have answered you query. If you have any further questions you can contact us in every time.Kindly regards. Wish you a good health."
},
{
"id": 116771,
"tgt": "Why is RBC blood test ordered in case of abdominal pain?",
"src": "Patient: My daughter is in the hospital right now with abdominal pain. She does have anemia and her b12 levels are low. She had gastric bypass about 9 years ago and since has had a foot of her small intestin removed and hernia surgeries She complains of pain constantly and the doctors said that she does have scar tissue that is causing the pain. They ordered a RBC6 blood test what is this test? Doctor: Welcome to H.C.M.I am Dr Krishna Dubey.I think that they advised CBC.As you told your daughter is suffering of abdominal pain with history of gastric bypass and surgery of intestine, doctor is thinking about anemia and some infection so they advised this test.This test will help find out type of anemia like iron deficiency and infection by seeing total white blood cell count.Follow your doctor and hope she will be fine .Thanx for query."
},
{
"id": 23200,
"tgt": "What causes pressure on the chest and difficulty in breathing while using heart stents?",
"src": "Patient: I am am actove 74 yr. old female. About 6 wks. ago I received 2 heart stents, one 80% blocked LAD. I have not gotten 100%. I wore a 2 wk. heart monitor and my doctor didn't see anything to be concerned about. When I walk or do anything strenuous, I have a hard pressure feeling in my chest. I feel like I can hardly breathe. Is this normal after stents? Can stents be displaced? Doctor: Dear Mrs,Once stents are placed, they can't be displaced, and it is not normal to have such complaints after stent implantation. Maybe it is anxiety because you were worried too much for the procedure. I would advise you to see your doctor to be evaluated for your complaints. So they canfind out the cause.Take careCome back if you have any further questions"
},
{
"id": 212582,
"tgt": "Have head tightness, depression, nervousness and fatigue. Head scan fine. Recommendations?",
"src": "Patient: Hi,I am have severe head tightness from the center of the head to neck . Below are my symptoms: I cannot concentrate on the work, become slow. sometimes this also increases if my seating position is wrong I feel like depressed, nervesness, angry for small things, dizziness , less energy, less active, sometimes when have more workload not able to respond.. some times when i went walk in cold (4 degrees and less), I felt like fainting . I have taken head scan when I went to india, but nothing was detected. I used many medicens,i used pain killers, multi- vitamins but not changing the condition. Doctor: Hello, welcome to Healthcare Magic. From your history, many of the symptoms are suggestive of depression. In depression head scan usually come normal. Depression usually occur neuro transmitters imbalance in brain, primarily involving serotonin, due to that medical treatment is effective in treatment of depression. For that you can consult nearby psychiatrist. Dizziness in cold may be due to thyroid hormone problem, so also get your thyroid function test done. Thyroid problem also increases chances of developing depression. Wish you all the best. Regards, Dr Ashish Kumar Mittal www.99doctor.com http://bit.ly/Dr-Ashish-Mittal"
},
{
"id": 112796,
"tgt": "Tired, back pain and cannot sit or stand for long. Diagnosed with Sinus. Hair is falling. Treatment?",
"src": "Patient: I stay very tired, my back and neck hurt so bad I almost can't sit or stand for long periods..although I walk every day..and watch what I eat I still cont. To gain weight and stay dizzy..went to the e.r recently they sent me home with a severe sinus infection and told me my lymph nodes was swollen/enlarged..but I have felt like this for over a year now..my hair is dry and starting to just fall out and I am only 29 please help.. Doctor: Hi !Thank you for your query.Dear ,Looking into your problem i would say that you have provided very non specific history . Since you mention about no joint pains it is not arthralgia .considering your young age and history of hairfall along with sinus infections suggestive of weak immunity , iron deficiency anaemia and thyroid dysfunction needs needs to be evaluated first . You need to get a CBC , S. ferritin and Thyroid profile needs to be done. Third possibility will be to rule out any autoimmune disease, but first these two common causes needs to be evaluated.Hope you are satisfied with the answer.If you have any further queries feel free to ask.Wish you good health.Take care.Dr. Shruti"
},
{
"id": 29182,
"tgt": "Do body ache, cough and a skin rash indicate flu?",
"src": "Patient: My husband and I thought we had the standard flu....body aches, headache, coughing and a fever of 101.+ for three days...fever has subsided and now just dealing with a wracking cough but I broke out with a rash on my chest last night and still have it. Doesn\u2019t itch. He broke out with a rash on his back and it does itch...is that from the flu or are we dealing with something else..? Doctor: Hello -Influenza causes a cough, fever, and body aches, and is \"going around\" in the US right now, so there is a good chance that it is flu. But flu doesn't typically cause a rash. However a viral infection (which is what flu is) can cause a post-viral exanthem (rash). Still, I advise you both to go in to be checked as rashes can be from other causes (such as a strep throat) and other reasons as well."
},
{
"id": 118002,
"tgt": "What does these blood test indicate?",
"src": "Patient: I am 78 yrs was not feeling good past couple of days so doc said to run a few tests where in my haemoglobin was 9.2, and my bilirubin -0.5(direct serum), sgot -87.5 and sgot-141.2, and gamma gt(ggtp)serum(ifcc) was -221.5 also alkaline phosphate serum was 294.9 please advice Doctor: SGOT, SGPT, GGT & alkaline phosphate are the liver enzymes increase in any liver insult. your liver enzymes are slightly increase so most probably you are suffering from disease which affects liver. moreover your hb level is also low. for that you should search the cause for anemia. and take treatment accordingly. and your liver function test is abnormal, for that consult the M.D. Physician to deal with that."
},
{
"id": 58463,
"tgt": "LFT elevated, bilirubin normal, RBC low, cholesterol normal. Prescribed folic acid, B12 shots, recovered from IBS. Suggestion?",
"src": "Patient: I had routine blood work done the other day & my liver functions are elevated. Never have been before. ALT 202, AST 167. Bilirubin was normal. RBC was very slightly low. Everything else normal. Lipids were all better than ever and in optimal range. Cholesterol only 150. My PCP didn't seem real concerned but I am. He just put me on some folic acid, gave me a B12 shot & said we'd recheck it in a couple months. I recently have been having chronic bowel issues with severe diarrhea. Just had a colonoscopy that only showed some inflammation. GI doc says it's all just IBS but I'm not convinced especially now with the liver enzymes. What are your thoughts? Doctor: Hi! welcome to HCM! Irritable Bowel Syndrome is a Functional gastrointestinal disorder and has no known organic cause.In many cases, you can control irritable bowel syndrome by managing your diet, lifestyle and stress. Your ALT 202, AST 167 suggest that there is inflammation or injury of liver cells,you must consult a gastroenterologist to find out the cause.A simple ULTRASOSONOGRAM will clarify things along with few viral markers. hope this will help you! take care!"
},
{
"id": 83325,
"tgt": "Why am I having bad sweats even after coming Chantix and finishing antibiotics for side effects ?",
"src": "Patient: I am female, 50, quit smoking 22 days ago, with the help of chantix. 7 days ago my tongue swelled up , I had earache sore throat. Went to dr, he took me off chantix, gave me z pack, told me to take Zantac,and Zurtec am and Zantac and benadryl pm. Ok finished antibiotics and ears still heart break out in sweats for no reason day snd night, feel lightheaded sometimes. Just feel weird . The sweats are really bad, Doctor: HiAllergy to chantix should be ruled out.Steroids should be used to reduce inflammation.Your menstrual history has not been mentioned.Sweating and hot flashes can occur as perimenopausal or menopausal symptoms.Kindly check your thyroid levels.Blood sugar and blood pressure levels should also be checked.Hope I have answered your query. Let me know if I can assist you further. RegardsDr.Saranya Ramadoss, General and Family Physician"
},
{
"id": 21555,
"tgt": "What causes heavy chest and breathing difficulty?",
"src": "Patient: hi doctor, my wife is having dust allergy and being a teacher has to travel to remote village by bus, she is feeling heavy weight in chest and unable to breath properly, she coughs but sputum extract is not there, kindly suggest, this problem is there since last 1 year, she is 32 years now Doctor: Hi ThereAfter reading the details provided by you i understand your concern for your wife's health. As she is asthmatic and allergic to dust the most important precaution is to avoid dust and if her job is in village area then it become difficult to avoid dust. Discuss with her if possible change the job location or else she she need to wear dust proof mask all the time while out.Its also advisable that she should consult a pulmonologist for evaluation. I would like to put her on some MAST CELL STABILIZERS like MONTELUKAST for her dry cough. You have to consult a doctor personally for a written prescription.I wish her good health"
},
{
"id": 131956,
"tgt": "Experiening weakness,numbness & cold in the right arm,shoulder & right chest",
"src": "Patient: I m experiencing weakness, atrophy, numbness, coldness primary in my right arm, hand, shoulder, right chest ... progressively getting worse over the last several months ... the tendon on the inside of my right elbow is now experiencing some pain and tightness. Doctor: hi with your symptoms I am inclined to think in terms of neck pathology where pressure on one / two nerves may cause all these symptoms. My advise is Pain killers , Pregabalin, Muscle relaxant , physiotherapy and I would suggest an MRI neck."
},
{
"id": 26876,
"tgt": "What is the treatment for difficult breathing inspite of an history for sarcoidosis?",
"src": "Patient: I have been having difficulty with breathing. I have had a CT scan which shows light and dark areas in my lungs. The pulmonary specialist that I started seeing put me on 30mg Prednisone for 6 weeks and now 20mg. along with oxygen use at 2L. None of this therapy has really corrected my problem. In fact, now, I need to use the oxygen most of the time. I will be seeing my doctor this week for follow up. Do you have any suggestions what diseases/problems could be causing this. I have had history of sarcoidosis which affected my electrical system of my heart (now have pacemaker for it). Looking for alternate treatments, tests etc. to be done? Doctor: Hello!Thank you for asking on HCM!I read your question carefully and understand your concern. Your past medical history is really concerning, and your symptoms seem to be related to sarcoidosis. But I recommend performing some other tests like : pulmonary function tests, a blood gas analysis, a resting ECG, to examine better your cardiac and pulmonary function. An ambulatory 24 hour ECG monitoring may be needed if suspicions of arrhythmia are raised. Steroids can be a therapeutic test for sarcoidosis (which is a disorder which improves by steroids). But in your case, the dose of steroids may need to be raised, in order to have the wished effects. The cardiac complications, improve too, when sarcoidosis starts responding to the therapy. You should discuss with your attending physician on the above possibilities. Hope to have been helpful!Best wishes, Dr. Iliri"
},
{
"id": 141717,
"tgt": "Diagnosed with Arteriovenous malformation & a procedure with brain glue resulting in vomiting",
"src": "Patient: My brother-in-law has been in a coma since March 20, 2012. I'll give you the short list, diabetic, high blood, high cholesterol and overweight. He suffered a stroke will in ICU. He was later diagnosed with Arteriovenous malformation - cerebral and had the procedure with the \"brain glue\". Currently he's in assisted living, with a trach, and feeding tubes. Yesterday they informed us that he's been vomiting but there's no need for concern. Should we be concerned? The family knows someone that had a similar situation and was told he started vomiting and then a few days later, she lost him? Doctor: Vomiting can be a symptom of raised intracranial pressure, and so in someone who has had a stroke, we will worry about the possibility that this is a sign of increasing pressure in the brain.Often times we will order brain imaging just to make sure.However, if it is clearly related to another issue (feeding tube, slow stomach emptying, etc) then a scan may not be necessary."
},
{
"id": 121804,
"tgt": "What causes tightness in neck and shoulder and headache after walking?",
"src": "Patient: hello dr, i am 41ys old female, , untli i a was 30ys old my wt was 125lbs, but i now weight 144lbs, i did play lot of sports until i was 17yr old, ihave a 13 yr old daughter, anyways i am trying to loose wt, by walking , but when ever i stat to walk , even for 5 to 10 min my head hurts, and my neck and shoulder remains so tight all the time , if i massage it , will be good for a while and it will go back to being tight, i also had a pinched nerve 4years ago , so my left arm i am told not to life too much wt , and i have a cysict near c5 , which is form birth but not growing and is benign, pl help, thank u Doctor: Hello, The tightness in the muscles can be related to spondylotic changes in the spine. You need regular stretching exercises to maintain the tone of your muscles. Hope I have answered your query. Let me know if I can assist you further. Take care Regards, Dr Praveen Tayal, Orthopaedic Surgeon"
},
{
"id": 45825,
"tgt": "Can gout infection hamper kidney functioning?",
"src": "Patient: I am a 44-year-old kidney donor and am experiencing gout. Can the gout damage my remaining kidney? Are the drugs used for gout toxic to the kidneys? I am 6'2 and 215 pounds and a vegetarian. The pain is very disabling. I see a nephrologist on Friday, because my PCP cannot get the gout under control. I have taken two courses of steroids and the joint is still swollen and stiff. Doctor: Hello and Welcome to \u2018Ask A Doctor\u2019 service. I have reviewed your query and here is my advice. Most of the drugs used in the treatment of gout can harm your kidneys. Generally drugs including NSAIDS and Colchicine are used for the treatment of gout and both of them can cause renal damage on long-term use. You can start on newer drugs like Febuxostat which has shown lesser nephrotoxic effect and it is safe in kidney disease patients. You can consult your nephrologist and he will direct you accordingly. Hope I have answered your query. Let me know if I can assist you further. Regards,\u00a0\u00a0\u00a0\u00a0\u00a0 Dr. Shinas Hussain"
},
{
"id": 87971,
"tgt": "Can abdominal pain, with yellow stools be related during pregnancy?",
"src": "Patient: Hi, I am 36 weeks pregnant and have been having upper right hand side abdominal pain. I went to my OBGYN today and he thought he may be a muscle tear. I forgot to tell him that I've been having yellow stool for about a week. Could the two be related? Please advise. Thank you. Doctor: HI.The pain in the upper abdomen at 36 weeks pregnancy can be due to muscle sprain. Tear is not common at all unless there is severe sneeze or such activity. The tear would give you severe pain, hematoma and swelling. You may please get high resolution ultrasonography of the abdomen, and the abdominal wall to see what the problem is. Yellow stool is in fact normal and it is possible that you are taking less of fat so that the bile is not utilized to give your stool the yellow color. You can get the relevant blood, urine ans stool tests done. Take full bed rest and full diet."
},
{
"id": 78775,
"tgt": "Suggest treatment for severe shoulder pain during coughing",
"src": "Patient: I have this pain that manifests certain times of day in my left shoulder for 2 days. It is intensified when I cough, laugh, or cry. Sometimes the pain makes me cry. It seems more associated with my head movements than my arm. I can lift arm, no issues. When I ran last evening, it did hurt me at times like a dull ache and it is only on the left shoulder. I did some light arm exercise 2 days ago. Also have had this dry tickly cough with occasional phlegm in morning for 3-4 weeks. Is it just a muscle pulled or is it something with cough..bronchitis? Doctor: Hi. I understand your concern.Chest pain with cough commonly seen in bronchitis and lung infection. Get a chest x ray done, if its normal, no need to worry for lung infection. Possibility of bronchitis is more in your case. So better to consult pulmonologist and get done clinical examination of respiratory system and PFT (Pulmonary Function Test). PFT is needed for the diagnosis of bronchitis. It will also tell you about severity of the disease and treatment of bronchitis is based on severity only. You may need inhaled bronchodilators and inhaled corticosteroid (ICS). Hope your question is answered."
},
{
"id": 52041,
"tgt": "Can you have back pain when your hb is to low ?",
"src": "Patient: if my hb is low can it give me a backache my hb is around 11 i seem to be getting back pain all the time when i walk, so i cant walk far, but when my hb was 13 i could walk any were and far, i have kidney failure, and the doctors dont like it to go above 12, please help me, the doctors keep saying i need to see a back doctor, they wont listen, is every one difrent when it come to there hb Doctor: Hi Tracy, Kidney failure can give rise to hyperactivity of your parathormone which could then erode your bones and take away the calcium from it. You need to check your calcium and parathormone levels and then get treated for the same, if that's the case. Regards, Joey"
},
{
"id": 4680,
"tgt": "Delayed periods, having early pregnancy symptoms. HPT negative. Nursing baby. Chances of pregnancy?",
"src": "Patient: My period is 2 weeks late this month. Hpts have all been negative, but I've had excessive saliva, nausea/mild vomiting, menstrual like cramps, and fatigue. With my 3rd pregnancy, it wasn't detected on a hpt until my second missed period. I am nursing my 1 year old, but he is in the process of weaning, so it's not nearly as often. I have had a regular period since 8 months pp. Doctor: Hi, Because of breast feeding ( though in weaning phase) &the menses were regular, there is possibility of an anovulatery cycles/ late ovulation in between . Or a span of no menses in between. In such a case you won't have coseption. In case you had late ovulation you may have conceived at a later date & pregnancy test will turn +ve afterwords. Please repete blood HCG test after 15 days & consult a gynecologist for evaluation after detailed examination. In case there is no pregnancy ,you have to adopt a contraception method to avoid such tension in future. Thanks"
},
{
"id": 144568,
"tgt": "Suggest treatment for disc bulge at C3/4,C4/5 & C5/6 & mild spinal canal stenosis",
"src": "Patient: My MRI Report:1) Degenerative disc diseases at L5/S1 level.2) Central and both paracentral disc bulge at L5/S1 Level Causing thecal sac indentation, mild spinal cannal stenosis and bilateral neural foraminal narrowing.3) Screening of whole spine shows disc bulge a c3/4, c4/5 & c5/6 Levels causing thecal sac indentation & mild spinal canal stenosis. Doctor: Disc bulge causing thecal sac indentation and mild spinal canal stenosis, seems to be manageable by conservative management. Conservative treatment options include- Analgesics (NSAID), Physiothrapy, Gabapentin/Pregabalin.Surgical treatment is indicated, if:1. Muscle weakness or atrophy in supplied muscles.2. Neuropathic changes in EMG in supplied muscles by affected nerve roots.3. No symptomatic improvement on conservative treatment4. Associated Vertebral instability"
},
{
"id": 191858,
"tgt": "Suggest remedy for ear pain in a diabetic",
"src": "Patient: I ve had severe ear pain that started yesterday, today I can t seem to catch my breath and I am having pain under my right breast. Pain is worse if I move or try to get a deep breath. I have used my proair inhaler and some ear drops with no relief. Temp is 100.8under toung.. I have asthma and I m a brittle diabetic glucose is 212 .my question is what is wrong with me.? Doctor: HI, thanks for using healthcare magicEar pain can be due to an infection in the ear , this would cause a fever and can be associated with chest infections as well.Infection would cause the blood sugar to increase.Pain in the chest on movement and taking a deep breath may be due to muscle injury, pleuritis (inflammation of the lining around the lung), costochondritis.It would be best to see your doctor to be treated.I hope this helps"
},
{
"id": 42647,
"tgt": "Could the brown jelly indicate the embryos that were rejected during an IVF?",
"src": "Patient: Hi I had an IVF 10 days ago, with 3 embryos transferred. Since then I am taking progesterone (pills and gel). Yesterday i saw a 5mm brown jelly like substance, and since then nothing. Could this be the embryos (that when transferred are in a gel) that were rejected? Doctor: Hi,Thanks for writing to HCM .The chances of that brown jelly being the embryo is more since this is the usual time when embryo get rejected from the uterus.That you will be confirmed after 4 weeks when on transvaginal ultrasound number of fetus could be made out.One less common cause could be due to use of progesterone gel may lead to such discharge. Hope I have been helpful. RegardsDr.Deepika Patil"
},
{
"id": 169757,
"tgt": "what causes seizure in kids?",
"src": "Patient: I have a young six years old baby girl called (Noor) weights 21 kgs , normal delivery. she is active, smart & quite most of the time. She has no serious medical history of any sickness except a past complains of ear inflammations. - A sudden change happened to her started on 27-5-2011 at 3:00 AM while a sleep suddenly she had a seizure- semi-shocked (stretched , eyes wide open, whole body shaking and stiff, trouble in breathing , pale lips , redness in face) lasted for 5 MINUTES. - on 28-5-2011 at 7:00 AM she had another seizure which was the same except : * Breathing was normal * Eyes open but turning left and right with no focus and no initial response to any comands but eyes movments * Right side body shaking (leg and arm) * It lasted longer than 5 minutes ended with a headache complain then vomited. - Same day at 2:00 PM While she was resting, An arm raised up suddenly with hand fingers overlapped, head heading backward , eyes wide open lasted for 3 minutes. - On 26-6-2011 10:00 AM and this time she was awake having breakfast with her brother when she suddenly fell back and had another seizure that lasted for 3 minutes, I asked her if she remembered any thing or had any pain before the incident but she didnt recall any. - On 29-6-2011 10:00 AM Happened again during her sleep accompanied with body shake. a report of the EGG doctor attached to the email together with his report Please Help & inform : what is going on with my girl? Doctor: It is seizures mam..as there is no fever she is labelled as seizure disorder mam..anticonvulsants preferably sodium va probate to be started at 10mg per kg per day and ct brain and eeg to be taken to localise the cause sir..then only we can comment as she had got recurrent seizures."
},
{
"id": 98828,
"tgt": "What causes a sensitive throat with bubbly phlegm?",
"src": "Patient: I am 44 years old, and have a long history of asthma and chronic bronchitis. Lately I am developing a very sensitive throat and have tons of tiny bubbles when I cough up mucus. Lately hot flashes/head rushes are accompanying the cough. I used to smoke a variety of substances, but gave it up for the most part for dipping tobacco. I went through a horrible lung fungus but God healed me miraculously a few years ago. There are no signs of the fungus, but the bubbly phlegm seems to keep my throat coated and irritated/sensitive. I have lost all but 6 teeth due to severe periodontitis over the last 25 years. Guafenisin doesn t seem to loosen it, and dextromethorphan helps very little. I use albuterol, but it doesn t help the cough, just the asthma. Doctor: Thanks for your question on Health Care Magic. I can understand your concern. Cough indicates that your asthma is not under control.So better to consult pulmonologist and get done clinical examination of respiratory system and PFT (Pulmonary Function Test). PFT is needed for the diagnosis of bronchitis. It will also tell you about severity of the disease and treatment of bronchitis is based on severity only. You may need inhaled bronchodilators and inhaled corticosteroid (ICS). Don't worry, you will be alright. Hope I have solved your query. Wish you good health. Thanks."
},
{
"id": 33981,
"tgt": "What causes recurring fever?",
"src": "Patient: my mother has fever around 2 after all checks not able to diagnose the cause, what could be the reason, the fever is from last 7 days and ultrasound is also ok. Is it possible that cannot detect typhoid initially or what could be the problem, please guide Doctor: fever which is more than 3 weeks and after 1 week of hospitalisation and still without diagnosis is called PUO-PYREXIA OF UNKNOWN ORIGIN .I WILL ADVICE TO GO FOR BLOOD CULTURE,CBC,CXR,URINE R/E,CRP,RFT,LFT,ESR,ANA,Rheumatoid Factor,Mantoux,ECG.CHECK AGAIN WITH MD PHYSICIAN."
},
{
"id": 134645,
"tgt": "Can hifenac mr be taken for muscle pain?",
"src": "Patient: I fell some minor problem in my shoulder or neck muscle , i feel some clicking sound specially in the space of around my neck.yesterday i taken a tablet of hifenac mr and got relief in next day morning.is this medicine helpful for body muscles .can i take two tablets in a week for relief of my muscles .pls suggest Doctor: it may work to an extent. but getting sound in your neck and shoulder region can be due to muscle weakness. kindly visit a close by specialist to assess your muscles or can talk to me via this forum."
},
{
"id": 71597,
"tgt": "Suggest medication for respiratory tract infection",
"src": "Patient: I had a respiratory infection for a few weeks, went to the nurse practioner and she gave me amococillan, did not work but took for 10 days, then say my doctor and he said I had a bad sinus infection and put me on a different antibiotic took ot for 10 days felt better for 3-5 days and the runnny nose, watery eyes, cough, coughing so hard was losing my breathe, urinating in my pants and eventually throwing up. Whenever I went outside even just from the house to the car it fel like my lungs were closing. They put me on another anibiotic that kills the good and bad and gave me to inhalers.. Going outside still is a problem and my chest hurts all the time and I am completely exhausted. My body hurts Doctor: Hello,This is a case requiring extensive evaluation with basic lab tests, x-ray chest, and hospital treatment in form of intravenous medications. Kindly get admitted to your nearby hospital immediately.Hope I have answered your query. Let me know if I can assist you further.Regards,Dr. Bhagyesh V. Patel"
},
{
"id": 213448,
"tgt": "Difficulty in forgetting past, have low self confidence, get shortness of breath when emotional. Solution?",
"src": "Patient: mam i cannot able to forget my past negative things happen in my life...man made feelings so bad on myself...i want to forget that and also i used to think always wrong oly in myself eventhough others did wrong to me also very low self confidenet i want to die always thinking future oly iam always dreaming i should go to high but iam not starting to achieve that....one day i had breathing problems that day my relative one man did feel bad on me Doctor: Hi, Thanks for choosing HCM, you seem very depressed, everyone has a past...good or bad you need to go on with your life find out the purpose of your life its good you are trying to forget your past life sad incidents why do you think you are wrong all the time acc to me you are wrong only when you think low of yourself and lack self confidence what you are doing is good,dream big....be serious in fulfilling your wishes you said you are not starting to acheive your dreams...remember dear small steps make a big journey the problem behind you not starting to acheive is you have no clarity in what you want to do and how i will try to help you with it ... try these tips 1.put on paper what all are your dreams , 2.divide them as big ,average and small 3.divide them into other kind also , a.practically possible(reality) b.imaginations 4.take a big goal ,devise smaller steps to reach a goal and start working on it seriously 5.work about your goal daily ,however small thing you may do ,do it daily let me explain example:you want to cook for your self dream about what you want to cook,how tasty you want it to be now get your ingredients one by one veggies,onions ,oil,etc check for facilities you got like a stove or electric cooker plan and learn how to cook practice well cook it you can do it this is a small example i want to give,dont think otherwise because you have not mentioned your gender age etc i have comeout with this so leave about cooking try to understand what i want to say start now think positive you dont need any outsiders help to lead your life happily alwaya be confident you have a purpose of your life , go ahead coming to your physical health ,it is quite common to get ill,and thats not your mistake no one can command you or illtreat you just because you have a health problem if you still have doubts go get a complete health scan done,take medication for your health problems, be health conscious take good diet mental health is equally important dear........ stay cool,relax,feel comfortable to live your life king size involve yourself in various activities do sternous exersice,it throws out all the bad stuff from your mind as well as your body try meditation perrform yoga it brings you confidence never try to hurt or harm yourself!! your family loves you,they need you the most realize yourself,leave all imaginations behind ,come out into reality read some inspirational books,watch such movies dont waste time,you have a lot to acheive if your situation gets worse and seem like your depression is dominating you,go meet your doctor ,family doctor is sufficient to give you counselling but all this helps only when you seriously want to do something great in life and wiiling to be confident and happy in your life hope i answered your queries feel free to contact me for further queries thank you"
},
{
"id": 10720,
"tgt": "Suggest effectiveness of platelet rich plasma cell therapy for hair loss in males",
"src": "Patient: Hi doc. I m 23 year old male I m having hair loss problem from past 3-4 years but It became visible to others from last one year before that I had hair fall but not that acute. Doc prescribed me a androanagen tablet and alpecin liquid to apply on scalp I m not getting any result in visible nature yes but hair fall has dropped to considerable effect....doc. Asked me to perform prp cell therapy for better result...did it work..? Doctor: hi,thank you for putting up your query on HCM.as per you history, you are already on antiandrogen tablet and alpecin liquid for your hair loss.i would like to inform you that, results will be visible only after a period of 6 months. since your hair fall has dropped already, you may continue using the same.as for prp, it is a good option for immediate results. you may undergo prp for better results as the doctor has already informed you. it is a safe procedure. you need not to worry much about the procedure. your consulting dermatologist can guide you better, regarding the procedure.hope, it helps.thank you."
},
{
"id": 66786,
"tgt": "What could lumps behind knee with pain in thigh?",
"src": "Patient: i have seven hard lumps behind my knee on the right leg on the inside .i have pain in the calf knee and down the inside of the leg.along with pain in my thigh.i injured it while stepping down to walk.this happened 10 months ago .do you know what these lumps may be? Doctor: thanks for sharing your health concern with us!well, these are possibly neurofibromas or lipomas and these are all benign tumor-like conditions; not to worry much! never poke it as this may invite infection!FNAC test can confirm the diagnosis. all the best!"
},
{
"id": 225133,
"tgt": "Can oral contraceptives lead to scanty periods over a time period?",
"src": "Patient: I have been on oral contraceptives for about 12 years - 3 different brands, but similar types - I'm currently on Orsythia. I have missed my period for the 3rd month in a row, however it's gradually been getting lighter and lighter after switching to Orsythia. Do you think it's just keeping me from bleeding? I am very careful and never, ever miss a pill. Doctor: Hello ma'am and welcome.Oral contraceptive pills have a hormonal content and can easily influence the hormone levels and thus, the functions they perform.They can thus easily influence the menstrual cycle (frequency, flow, amoutn, etc.), cause breast tenderness, soreness, etc.So the answrer to your question would be yes, they can.Hope this helps.Best wishes."
},
{
"id": 32843,
"tgt": "Suggest appropriate dosage of antibiotic for a monkey bite",
"src": "Patient: Hello, I was bitten on 17th february by a monkey. (one of my hands finger). The doctor gave me the tetanus vaksin, only one dosage, and antibiotic, but I\u00b4ve read that maybe in some cases it \u00b4s necessary 3 dosages vaksin. At the moment I\u00b4m changing the bandage/curative in two days. I am in Indonesia, and in a week I Will return to my country Portugal. What do you think that I should do? Could you sent me an e-mail please? Doctor: HI, thanks for using healthcare magicMost persons would have received initial vaccination during childhood and is booster is given if needed eg with injury.It is likely that your doctor gave you a tetanus booster which is one vaccine.Vaccination against rabies should have also been considered.I hope this helps"
},
{
"id": 148173,
"tgt": "Alzheimer patient with enlarged liver started shaking violently with breathlessness. Was it panic attack?",
"src": "Patient: My father has Alzheimers, but he also has an enlarged liver, which is being investgated - slowly. This evening, he started shaking violently all over with breathlessness. I treated it like a panic attack, but I am worried it may be something else. Hope you can help. Doctor: Hi,Thank you for posting your query.Based on your description, your father most likely had a seizure.Seizures can occur in people with Alzheimer's disease. You should consult his neurologist and get him evaluated further. A CT scan of brain and EEG would be required.For control of seizures, he may be given levetiracetam tablets.I hope my answer helps. Please get back if you have any follow up queries or if you require any additional information.Wishing you good health,Dr Sudhir Kumar MD (Internal Medicine), DM (Neurology)Senior Consultant NeurologistApollo Hospitals, Hyderabad, IndiaClick on this link to ask me a DIRECT QUERY: http://bit.ly/Dr-Sudhir-kumarMy BLOG: http://bestneurodoctor.blogspot.in"
},
{
"id": 154216,
"tgt": "Can bladder cancer cause anus and testicle pain?",
"src": "Patient: My husband has a bladder cancer and is schedule for operation next 2 weeks. According to his ct scan the cancer has not spread to other parts of his body. My query is why is my hussband experiencing pain in his anus and testicles. Is that has something to do with his cancer. He is just taking strong pain killer. After the operation will the pain go away. His bladder will be remove . Thank you and God bless, Doctor: Hi,Thanks for writing in.The nerve supply to urinary bladder, rectum and testis are closely related. These nerves originate in the lumbar segments of spinal cord which is in the lower back. There must be a nerve getting stimulated or irritated due to the bladder cancer which is also supplying the anus and testis. This might be causing pain to be felt.Surgery for urinary bladder removal is a major surgery and recovery might take some time. However, the severe pain will improve but mild pain might persist for a few weeks till the surgical wound heals completely. In 3 months time after surgery the wounds should be healing well and his pain will likely improve. The testis pain should disappear by then. Please do not worry."
},
{
"id": 167865,
"tgt": "Suggest remedy for problem in bowels",
"src": "Patient: What could be the cause of a 7year old boy to have foul smelling stools? He was born fine. growth has always been below 5%. He suffers from severe constipation so takes miralx daily. aside from growth and asthma he apears normal and healthy. his apgar score was slightly low when he was born , he had rsv within the first month and followed it with pnemeonia, he also had several ear infections ending in surgery for ear tubes. Please help the doctors cant seem to figure out why?? important to note- he has had the salt/sweat test for cycstic fibrosis which came back neg. and growth hormone test also came back fine. with the exception of his last cbc which showed he was low in vitamen D nothing else has been found. More about the stools- they r dark brown in color w/an orange tint, they stick to bottom of stool and are difficult to get flushed also the closest related smell to his stools would be vomit. the smell is so strong it can be smelt in the rooms surrounding the restroom even and remains pressent for at least 30minutes following the stool. Doctor: the bad smell of the stool is because of the sever constipation , the stool which is stuck at the colon starts to be fermented by bacteria and cause this bad smell the regular use of laxatives and evacuation of the bowel can help with this problem a boy who has a growth below 5th centile and has several constipation should raise the suspicion of celiac disease I recommend doing total serum IGA level , anti tissue trasglutminase IGA level to exclude celiac disease I hope this helps"
},
{
"id": 69099,
"tgt": "What could be the small painful and itchy bump on knee?",
"src": "Patient: It started a a small bump on my knee that I thought was a bug bite. It was a little itchy and I felt like it needed to be popped, like it has pus, but nothing came out. Now the area surrounding the initial bump is red, swollen, hot and kind of hard (3 days later). Not sure what it is or what I should do, but it is painful and I m wondering if I should go to the doctor. Doctor: Hi! Good evening. I am Dr Shareef answering your query.Yes, I think you should see a doctor, as you have developed cellulitis (infection of skin and the subcutaneous tissues) due to fiddling with the initial bump. He after an initial clinical examination might advise you for a broad spectrum antibiotic with an anti inflammatory drug, or might go for an incision and drainage of an underlying abscess over there. The management would depend on the result of clinical assessment.I hope this information would help you in discussing with your family physician/treating doctor in further management of your problem. Please do not hesitate to ask in case of any further doubts.Thanks for choosing health care magic to clear doubts on your health problems. Wishing you an early recovery. Dr Shareef."
},
{
"id": 109597,
"tgt": "What causes sudden pain in the coccyx?",
"src": "Patient: I have had a sudden onset of pain in my coccyx. I haven t fallen to cause the pain. It hurts when I sit and stand and it almost feels like I need to move my bowels when I don t have to. I have had a herniated disc between L5 and S1 that was corrected with surgery 3 years ago. I m wondering what I can do to ease the pain and if this is something to see my doctor about. Doctor: Hi Welcome to healhcaremagic After going through your query I concluded that you are suffering from coccyxdynia. It's treatment is hot sits bath, avoid sittings on hard seat. Avoid long sitting. Analgesic such as ibuprofen can be taken for pain relief. vitamin B and C is useful. Vitamin D can be of help if it's level is low. You can discuss with your Doctor about it. Hope your query get answered. If you have any clarification then please don't hesitate to write to us. I will be happy to help you.Wishing you a good health. Take care."
},
{
"id": 26663,
"tgt": "What causes palpitations and increased heart rate?",
"src": "Patient: Hello, I am 31 year old female, 5'4\", and 154 lbs. My blood pressure is always normal and heart rate is normal when I do see my doctor. I have severe allergies and am on allergy shots every two weeks. The only regular meds I take are zyrtec and yaz. I work out two to four times a week. I usually just do cardio workouts. My heart rate usually stays within a good range and I recover well after my workout. This weekend, I did some traveling and found that walking up stairs (several flights), my heart was pounding so hard I felt my chest would explode and it was much harder for me to recover. I was breathing slow and deep, but do not feel that I was completely short of breath. I was told I had low iron in the past, but to just take over the counter iron supplement and eat more foods with iron. This horrible pounding and higher heart rate have me nervous. I also occationally feel like my heart his pounding harder (not faster), when sitting. Any suggestions on what is causing this pounding and increased heart rate when going up stairs? Doctor: Hello and welcome to HCM.I carefully read your question and I understand your concern.But dont worry.I'll try to explain you something and give you some advises.As you explain,my opinion as cardiologyst is that we are dealling whith a rhythm issue.You may feel increased heart rate and this pounding feeling for different reasons.It may be a simple tachycardia.It means that your heart beats over 100 for minute.Its normal range should be between 60-100 beat for minute. You may experience tachycardia for different reasons beginning from a simple emotional stres to different pathologys.You descript an iron deficiency long ago.Sometimes this iron deficiency may lead to anemia and this anemia can couse simple tachycardia.It is a compensatory mechanism of our body.You may experience palpitations becouse of extrasistoles.This are escapet beats originating from different parts of the heart.They may have a lot of couses.So,if I was your treating doctor I will recommend you to do some examinations like a simple blood analysis to look for possible anemia,your thyroid hormons to exclude a hyperthyroidism,electrocardiogram,echo cardiography and a holter monitoring to document your rhythm tendency.This for a simpme check up.At mean time avoid coffee and caffeine products becouse they rise your heart frequency.I hope I was helpful.Wish you good health."
},
{
"id": 102582,
"tgt": "Can bacterium taken for UTI cause red itch rashes all over the body?",
"src": "Patient: Hi, I have been on bactrim for a UTI for about a week and throughout the week when in the shower under hot water my legs and feet get extremely red and itchy.. Several hours ago I drank two beers (2 within about 4 hours). At first my chin, side of my face, neck and ear felt itchy and hot, I went to take a shower and noticed a small rash where I had itched it seemed, I got in the shower and kept the water luke warm then turned it to hot because I got cold my legs and feet got red and itchy as always, about 10 minutes after getting out of the shower the redness on my feet and legs turned into a rash and now the rash is everywhere and it's bump and itchy. My first thought is that it's an allergic reaction to bactrim (I took cephalexin before and reacted badly with it as well), what is your opinion? And should I not take Benadryl because I drank alcohol earlier? Doctor: HIThank for asking to HCMYes you are right the drug which you have taken is a sulfa group of drug and known for hypersensitivity reaction, you have to change the drug, for the rashes you can take the long acting second generation antihistamine drug.1) Livocetrizine 10 mg twice in day take care and have nice day."
},
{
"id": 89877,
"tgt": "Suggest treatment for abdominal pain",
"src": "Patient: Hi, I am aged 24 height 5' 10\", weight 75kg. I have been having this condition for around 3 years now. When i eat certain foods, I have abdominal pain, followed by burping, farting and subsequent stool which is a little loose but formed. I see that there is froth around the stool and there is a burning sensation in my rectal area. I have undergone a scope on my intestine which indicated small red irritation spots along my colon until my rectum. I was treated for gastric which i was finally cleared from. The types of food that immediately cause this are coffee wt milk and tea with milk. Tea without milk does not cause any issues for me. PLease advice. What are we looking at? Doctor: HI.The only treatment is such cases is NOT to take such foods and beverages which cause the trouble you explained. One can undergo a course of a steroid in reducing dosage along with probiotics and multi-vitamins of 3 weeks. Try these foods again. IF still a problem , then make it a life-time ban."
},
{
"id": 188211,
"tgt": "Have had sinus infection with numbness in upper lip, gums, nose and hands. Does it indicate infection?",
"src": "Patient: Hi there,I've had a sinus infection the last week, and I thought I am on the men's now.However, about an hour ago, I blew my nose, and now my nose, upper lip and teeth/gums have gone numb on the left hand side (which was/is the infected sinus). Very strange.It's numb like it would be had a dentist injected the gumsThanks for your advice Doctor: what is ur age....have u completed..antibiotic course?/it may indicate infection...is there swelling near lips/nose..have u history of exposure to extreme cold??u must b watchful...as infection around lips can spread to baseof skull...head causing serious sequele.../early symptom for cerebral insufficiency."
},
{
"id": 141865,
"tgt": "Suggest treatment for exploding head syndrome",
"src": "Patient: I am having brain explosions which is taking away a lot of my memory. I have high blood pressure, and when given a new sample BP med (since my old one was not generic), I tried it and it caused these explosions. Irbesartin was the culprit. Since them I have tried to use a natural BP med, but the explosions are still happening. These are mainly at night when going to sleep or waking up. Can you help end these explosions? Thanks!! Doctor: Hello!My name is Dr. Aida and I am glad to attend you on Healthcaremagic!Your symptoms could be related to ice peak headache. I recommend consulting with a neurologist for a careful neurological exam and some tests: - a brain MRI study- complete blood count, PCR, sedimentation rate for inflammation. I would also recommend wearing a blood pressure Holter monitoring in order to exclude possible high blood pressure during those episodes. Meanwhile, I would recommend taking naproxene before sleep. Hope you will find this answer helpful!I remain at your disposal for any further questions whenever you need!Kind regards!"
},
{
"id": 45843,
"tgt": "Suggest treatment for kidney problem in child",
"src": "Patient: Hello Doctors My baby as is 1.2 years, and who suffering from kidney problem because there one extra nali ... doctor suggested Anti-batik i.e. Amoxicilin once a day in one month and next month Cefelaxin once a day. both medicine run continuously please suggest me Thanks Doctor: Hello and Welcome to \u2018Ask A Doctor\u2019 service. I have reviewed your query and here is my advice. You can continue antibiotics. If symptoms persist, better to consult a nephrologist and get evaluated. Wishing you good health. Thanks."
},
{
"id": 167115,
"tgt": "What causes stomach pain and vomiting?",
"src": "Patient: my son is of 4 yrs,, all of a sudden yesterday he was souded uneasy,, in the evening he started telling of stomach pain,,he ommited all the food in the night,, never had food at night,, he says ommitting is comming in the morning,, he had only half glass of pediacure,, pls suggest Doctor: HiThe symptoms are most likely due to an episode of gastritis. A dose of antiemetic such as domperidone or ondansetrone along with antacids such as ranitidine can be of help. Visit a paediatrician for clinical evaluation if symptoms persists.Take care"
},
{
"id": 206686,
"tgt": "What causes difficulty in identifying objects?",
"src": "Patient: I make random relations with objects and other objects, (almost like synesthesia but not with numbers and colors) is there a name for that? As soon as I look at random things, like a dog I think of some weird thing that has no relation but makes sense to only me. I feel like I m crazy when I do it. Doctor: DearWe understand your concernsI went through your details. I suggest you not to worry much. In one sense everyone are crazy. Our brain learns and understands things relatively. That is the underlying factor of perception. Synesthesia, is not considered as a disease, but is a perception related disorder. In your case, your condition is also defined as a perception based, sensory organ based disorder, but not considered as a disease which requires treatment. Ignore these symptoms and concentrate on your career and education.If you require more of my help in this aspect, Please post a direct question to me in this URL. http://goo.gl/aYW2pR. Make sure that you include every minute details possible. I shall prescribe the needed psychotherapy techniques.Hope this answers your query. Available for further clarifications.Good luck."
},
{
"id": 136902,
"tgt": "What causes fingers tremors and muscle twitches in woman?",
"src": "Patient: Hi I am a 35 year woman whom is experiencing finger tremors and muscle twitches. Furthermore, I experience off and on night sweats and fatigue and constipation.The worst is the hand/finger tremors and the muscle twitches for I hope no one notices.I have been to a neurologist approx. 2 years ago because that was my first experience with the tremors.MRI brain was performed which was normal Any ideas what may be causing this because no one said what is causing these symptoms Doctor: Hello,I had gone through your question and understand your concerns.Looking at your problems I am thinking of following possibilities01) Over active thyroid( hyperthyroidism) : All your symptoms are consistent with this condition. A thyroid profile test will help in diagnosis. I02) Alcohol abuse/withdrawal: If you are alcoholic03) Liver failure: Liver diseases also causes tremors in hands, constipation etc. A battery of tests like blood investigations (CBC,ESR etc) Liver function tests, USG abdomen will help in diagnosis.A MRI of brain rules out any kind brain related problems. It is advisable to consult your doctor and get yourself examined for above conditions.Additionally, avoid mental stress if you have any. Include fruits,vegetables in your diet.Hope this answers your question.You can ask any followup question or any other queries. I'll be happy to help.Wish you good healthRegards,Dr.Prathap Kumar"
},
{
"id": 194438,
"tgt": "Suggest treatment for hydrocele",
"src": "Patient: Hello, I was diagnosed with a hydrocele some time ago, but it never really bothered me (or very infrequently) so i never got it fixed. The past couple of days I have felt pain similar to the feeling of the hydrocele, but it is now constant. I am going to get the hydrocele operated on, I was just wondering if this could have turned into something more serious? Doctor: Hi, A hydrocele is a sac filled with fluid that forms around a testicle. They generally don\u2019t have any threat to the testicles. To diagnose a hydrocele, the physical examination needs to be performed. If you have a hydrocele, your scrotum will be swollen, but most ofen it does not have any pain. It is not possible to feel your testicle through the fluid-filled sac. If it is tender, it needs to be treated by intake of analgesic medications. Most often, hydroceles typically go away within six months with conservative management by giving adequate scrotal support. Surgery is indicated if the swelling does not show signs of regression and would be causing more discomfort. Kindly consult Surgeon in this regard for clinical evaluation and appropriate management. Hope I have answered your query. Let me know if I can assist you further. Regards, Dr. Ivan R. Rommstein, General Surgeon"
},
{
"id": 19722,
"tgt": "Suggest treatment for abdominal pain and weakness while having vitamin-d deficiency/hypotension",
"src": "Patient: Hi my name is Katie. I am 22 years old. I have been in and out of doctors offices since my csection in Feb. of 2009. I had my gallbladder removed in June of 2009 due to gallstones. My symptoms didn't subside after that/ I still felt the same. Fatigue, Abdominal Pain, weakness etc. I found out that I had vitamin d deficiency (18L). And Hypotension. (Get out of breath, fainting etc)The dr told me to increase my salt intake and exercise more. I did, and it hasn't worked. During my pregnancy I developed ICP? (high liver enzymes)I had severe itching that didnt subside. Since I had no family history of it, I demanded a Ct scan of my abdomen to check everything visually. My results came back normal. As with all my bloodwork. I am at my wits end and don't know what to do. Do you have any suggestions? Doctor: Hello Katie!Welcome and thank you for asking on HCM!I passed carefully through your question and would recommend checking your adrenal gland function and also your thyroid gland function. Your symptoms could be caused by a hormonal disorder or an autoimmune disorder. For this reason, I would recommend performing some tests: - cortisol and aldosterone plasma levels for the adrenal gland function- ANA, ANCA to investigate for an autoimmune disorder- complete blood count, fibrinogen plasma levels and PCR for inflammation- thyroid hormone levels- a Head Up Tilt test for postural hypotension. A cardiac ultrasound is also necessary considering the shortness of breath when walking. In the meantime, I recommend continuing taking Vitamin D. Hope you will find this answer helpful!Kind regards, Dr. Iliri"
},
{
"id": 10858,
"tgt": "Suggest remedies for hair fall",
"src": "Patient: Hi, I had been experiencing hairfall since 5-6 years, so about a year back i went to a hair clinic. I was treated there for 5 months. Though the hairfall had somewhat stopped, hair growth is still questionable. During the treatment, i took the tab Follihair, applied Anastim solution oj the scalp and they also injected some medicine into my scalp. Again since last month my hairs have started falling at an accelerated rate. Please suggest me some tablets or medicines to apply on ghe scalp to stop hairfall and if possible revive hair growth. Doctor: hello, welcomei explain everything about hair loss in point wise manner1. 100-150 hair fall every day is very normal. So should be consider hair fall if number of hair more than this.2. Generalized hair fall in female is mostly due to stress, poor sleep, poor nutrition and excessive blood loss due to menstruation3. Stress alopecia is also common in which female keeps their hair very tight in certain manner which creates tension in root and cause hair fall.4. Straightening and other procedure also cause weakness of root5. Other condition like fever, ptyphoid, maleria, thyroid, etc also cause hair fall( less common)Treatment1. Have good sleep. food., green vegetables , fruits2. Take iron and multivitamines like follihair atleast 3 month.3. Check you blood hemoglobine if it is less iron and folic acid supplements should start.4. Check thyroid and diabetes to rule out other caus"
},
{
"id": 144806,
"tgt": "What are the chances of getting speach back after severe head injury?",
"src": "Patient: Hi sir, I am bharath from bangalore. My brother named Sunil has met with an accident last month June 5th. He has been admitted into SHALBY Hospital ahmedabad since then. His forehead got damaged internally, he went to serious state. Now his condition is stable. He is not able to speak still, and still it will take long time to recover the internal head injury. Sir I would like to ask few questions related to his situation. From Last two days only he has started eating food. Will he be able speak? Will he be able to recognise and back to his normal life? Doctor: Welcome to HealthcareMagic.com. Sorry to hear about your brother. Please know that it is really hard to tell, two different people can have the same injury and gain different functions at different rates. The best hope for him is for him to have the appropriate physical rehab and therapy for his condition. I have seen speech come back after intense speech therapy, also a rule of thumb is that the longer the days pass without regaining whatever function it is you inquire about, the less chance that this will get better. I hope and pray that he does well, but don't lose hope as his individual drive and vigor after the injury will go a long way into his healing as will all your support for him. I hope this helps and best of luck."
},
{
"id": 94183,
"tgt": "Have right abdominal cramps and weakness. Had found cyst in pancreas. Which specialist to see?",
"src": "Patient: September 2011 I had a fall at work. January 2012 8mm cyst on pancreas found incidentally after MRI January 2013 started to have discomfort on right side. Saw physician who told me it was related to fall 2 years earlier. Told me to see my nerosurgeon as back & stomac pain are related to back nerves. March 2013 Started having fairly severe right sided discomfort and crampy pain abdomen right and generalized weakness. Why was it ok not to biopsy just because ithadn t grown in 1 year? Which specialist do I see? Doctor: Hi A cyst can occur in pancreas due to trauma. No need to take biopsy for pancreatic cyst. If the size of the cyst is more than 6cms,then surgery has to be done. You should consult a Surgical Gastroenterologist. Hope surgery is not needed.But,please consult him. Wish you good health Regards"
},
{
"id": 102345,
"tgt": "Is bronchitis contagious?",
"src": "Patient: Hi, I am a 54 Year old woman . I used to suffer from asthma and bronchitis as a child. I have just found out I have bronchitis., which I haven't had for 20 years or more. I have been spending lots of time with my grand daughters and I'm worried that i might be contagious. What are your thoughts. Thank you Cathy Doctor: Hello, Thanks for your query.Yes. Most cases of acute bronchitis are caused by viral infections. Influenza viruses are a common cause, but many other viruses also have been identified in people with bronchitis. These viruses spread from person to person through direct and indirect contact.To reduce the risk of \"catching\" bronchitis:Avoid close contact with sick peopleWash your hands often or use hand sanitizers regularlyAvoid touching your eyes, nose and mouthGet an annual flu vaccinationI do hope that you have found something helpful and I will be glad to answer any further query.Take care"
},
{
"id": 79286,
"tgt": "Suggest treatment for bronchiectasis",
"src": "Patient: Hi My son heard a programme on radio 4 concerning a lady with CBID and feels I may have this too. I have this year been diagnosed with bronchiectasis, I have had psoriasis for the last few years, a persistent vitamin D deficiency, inspite of sunny holidays, and I find it very difficult to shake off colds which usually develop into laryngitis or/and chest infections. I have had ct scans of my chest and sinuses, the sinuses now being clear and no tumours in my lungs. Other than this I am very fit and healthy, my cholesterol is fine and blood pressure on the low side. I am 65 and slim. Should I have a test for immunoglobulin and is there any treatment should I have CBID? Many thanks. Sue Doctor: Thanks for your question on Health Care Magic. I can understand your concern. Possibility of immune deficiency is low in your case. Immune deficiency diseases present at early age. But better to get done IgE, IgM and IgG levels. If all are normal than no need to worry for immune deficiency. Bronchiectesis is chronic, structural lung disease. It needs life long treatment. It requires inhaled bronchodilators, inhaled corticosteroid (ICS), oral expectorant, mucolytic and antioxidants. Chest physiotherapy and deep breathing exercises are also important in bronchiectesis. Vaccination with pneumococcal and Influenza vaccines are also helpful to prevent recurrent respiratory tract infection. So consult pulmonologist and discuss all these. Hope I have solved your query. Wish you good health. Thanks."
},
{
"id": 55279,
"tgt": "Can Livogen z, Becosules and Vitamin D3 be used during pregnancy after liver transplantation?",
"src": "Patient: Namaste.my name is ERKA.i am from Mongolia.I have been donor of the liver for my mother. liver transplantation was in india 2012.04.26.Now iam 11 week pregnant.Can i use livogen-z, becosules and shalicall or vitamin d3 calci ? and avter 1year pregnant it is bad for my health and for my liver. Also iam becoming fat, what i can use for my liver ? please consult for me.sorry my english not very well.Thanks Doctor: Hello! Thanks for putting your query in HCM. I am a Gastroenterologist. Yes you can take tablet livogen, vitamin D3, becosules and shelcal without any harmI hope I have answered your query and this will help you . Wish you a good health"
},
{
"id": 160799,
"tgt": "What causes high fever, runny nose with weakness and tiredness?",
"src": "Patient: MY 3 YEAR OLD SON HAS A HIGH FEVER, COME ON ALL AT ONCE AROUNT 7 P.M. (CENTRAL TIME) FEVER WAS 102 DEGREES. GAVE 1ST DOSE OF TYLENOL AT 7:15 P.M. CHECKED TEMP AT 12:15 A.M. AND IT WAS 103.4 DEGREES. HAS A SLIGHTLY RUNNY NOSE NOTHING MAJOR THOUGH, NO COUGH, NO VOMITTING, NO RUNNY BOWEL MOVEMENTS. ACTS WEAK AND TIRED BUT IS STILL UP AND MOVING AROUND FINE. WHAT COULD THIS BE? AND SHOULD I BE CONCERNED? Doctor: Hi,From what you said, he is having a flu. Need not worry at this point of time. Mostly these are viral and fever will subside in 2-3 days. In such cases I advise my patients to give paracetamol every 6-8 hours for 3 days and saline nasal drops if there is congestion or block. Antihistamine like levocetirine will help if significant nasal discharge or dry coughHope I have answered your question. Let me know if I can assist you further. Regards, Dr. Muhammed Aslam TK, Pediatrician"
},
{
"id": 82914,
"tgt": "Can blotchy rashes be lupus?",
"src": "Patient: i i have osteoarthritis and fribromyalgia symtoms and constant inflamation in different parts of body, and a small blotchy rash rash that comes and goes can it be lupus had one blood test that came out positive but my dr does not beleive because i dont have a rash on face? Doctor: HiBlood test positive doesnot mean that you have Lupus. I assume that you are talking about ANA test. It may be positive in normal population also.You could have lupus even you donot have rash on face.What are the other clinical features? other tests?"
},
{
"id": 136152,
"tgt": "Suggest treatment for left lateral incisor pain",
"src": "Patient: I am having severe pain in my left lateral incisor. I went to the dentist and there is no apparent reason for the discomfort, no breaks or cracks, no cavities, no obvious swelling. The pain is very localized to the one tooth. What other possibilities could it be? Doctor: There may be a dentigerous cyst which is just a benign cyst. other causes maybe some nerve irritation for that you might need root canal treatment."
},
{
"id": 65723,
"tgt": "What could lump behind right ear indicate?",
"src": "Patient: I was diagnosed with follicular carcinoma about for years ago. I had my thyroid removed and have not had any problems. In the last couple of months, I noticed a lump behind my right ear. It s about the size of an eraser and tender to the touch. I have also been itching where my incision was. It almost looks like some sort of rash. Is it possible that they are related to my thyroid cancer? Doctor: Hi, dearI have gone through your question. I can understand your concern. You may have some enlarged lymphnode. It may be due to some reactive hyperplasia due to infection. Or you may have some soft tissue tumor or cyst. Chances of follicular carcinoma metastasis is veru rare. Follicular carcinoma rarely metastasize to lymphnode. Still you should go for biopsy. It will give you exact diagnosis. Then you should take treatment accordingly. Hope I have answered your question, if you have doubt then I will be happy to answer. Thanks for using health care magic. Wish you a very good health."
},
{
"id": 23195,
"tgt": "What could cause chest pain?",
"src": "Patient: Hi. I am 23 y/o male 6 1 about 250 pounds. Thursday evening i was awoke from sleep with a pain in my chest and an inability to breathe deeply without pain. The pain subsided after 10-15 minutes. Unfortunately, this morning around 1 a.m. it has return with a vengeance, along with a rapid heart rate and more pain with breathing deeply. I managed to calm myself down after some hours, although it felt like my heart was going to jump out of my chest, and was pumping way too hard, and i had heart palpitations that can be felt by placing a few fingers on my left breast. It did initially feel like i was having a heart attack (although i have never had one so i really wouldnt know what it would feel like). The pain is a sharp type of pain in the left side of my chest just to the right of my left breast, and can also be felt in the same area in my back. The pain happens at the bottom of my breath which makes it difficult to take deep breaths without pain. Although my heart rate has returned to normal, the pain is still there some 13 hours later. Some additional information: I sleep on a bad bed and tend to have back pain here and there. Sometimes i stretch my shoulders and chest out to crack my upper back until a pop can be heard in my chest which relieves stiffness. I feel as this can be somewhat related to the cause of my pain because i have been doing it the past few days as part of a normal routine. When the initial pain in my chest started, i thought that i may just need to crack my back in the same way i described to relieve the pain, but when doing so found that i could not and there was no pop in my chest. I have been researching the subject within the past 12 hours and come to find a condition known as Costochondritis. I read that the this condition affects the areas around the sternum, upper rib cage and cartilage between the two, and can be identified as localized pain the the left side of the chest that can cause difficulty breathing deeply. I read that it can be diagnosed simply by apply pressure to the area where it hurts, and if it does it is most likely Costochondritis and not another more serious condition. Well after hearing that i decided to give myself a little test, and applied pressure (not too hard, but not softly either) on my left breast with the tips of my fingers where i described the pain. I found that it was very sore and hurt very much to do that, and since i ve done it, the pain seems to have increased and now i can sometimes feel the pain at a stand still. It is not excruciating pain but significantly uncomfortable, especially when breathing deeply through the chest. I have since been focusing on breathing deeply through my belly, which doesn t hurt as much as if i were to breath through my chest, but the pain is still there. I do not feel as this is a condition with my heart, but more of a bone type of problem. With the information given what do recommend as a course of action and to provide relief to this area? Does it sound like Costochondritis to you? if so, how common is this condition and how long with the pain last before subsiding? As i said before the first onset lasted for about 15 minutes and went away but this time it has not subsided yet and many hours have past since. Going to the doctor is difficult right now as i do not have any money nor insurance which limits me to going only for a medical emergency. Thank you for your time. Doctor: any associated fever, cough or loss of weight or appetite.if none of these can treat like costochondritis. can take acetaminophen tablets and diclofenac gel for local application.pain can continue intermittently for 6-12 months"
},
{
"id": 16701,
"tgt": "Suggest treatment for ventricular septal defect",
"src": "Patient: My sister is 69 and the doctors just diagnosed her as having a hole in her heart after what they call a mini stroke and a blood clot in the neck. Why would the hole in the heart just now show up? What will they do to prevent another incident? What does it take to close the hole in the heart if that is what they decide to do? Doctor: Hello, I would like to tell you that structural heart disease generally needs surgical correction via open-heart surgery. Hope I have answered your query. Let me know if I can assist you further. Regards, Dr. Bhanu Partap, Cardiologist"
},
{
"id": 50167,
"tgt": "Frequent urination, blood in urine. Bruises on the penis. Due to kidney stone?",
"src": "Patient: My boyfriend has been having problems urinating on random occasions. He says that he pees blood and recently showed me the result. it looked like light pink wax has been dripped in the toilet. His genital area has minor bruises on the tip of his penis . Please explain to me what this is, I am very worried and I can not find anything on the net about this. is it kidney stone? Doctor: From what you have described he seems to be having viral infection of tip of penis.However it will be prudent to rule out a kidney/bladder disease .You may get urine test,blood tests,USG kidneys & bladder,STD test& consult a specialist in person"
},
{
"id": 80978,
"tgt": "What causes pain in chest while bending down or standing up?",
"src": "Patient: Hi, I have had chest pain in left hand side towards the top of my chest. It hurts especially if I bend down then stand back up, but also later in the day it hurts when I breathe in and out I would say its like a sharp ache. I m 41 years old , I don t smoke, reasonably fit. Does seem to improve when rested. Just concerned as I have been under some stress recently. Please can you give me some advice Doctor: Thanks for your question on HCM.I can understand your situation and problem.In my opinion we should first rule out cardiac cause for your left sided chest pain.So get done ECG first.If ECG is normal then no need to worry much for cardiac cause.Your pain is increasing with movements like bending and subside on rest, this favours muscular pain mostly.So try to follow these steps for better symptomatic relief.1. Avoid heavyweight lifting.2. Avoid strenuous exercise3. Avoid bad postures in sleep.4. Avoid movements causing pain.5. Start pain killer and muscle relaxant.6. Apply warm water pad on affected site.Don't worry,you will be alright."
},
{
"id": 7474,
"tgt": "Pimples, black spots on face. Prescribed clyndamycin and benzyl peroxide lotion. Can I use?",
"src": "Patient: Hi my skin is oily.. i used cinthol for many years.. But recently i changed to Dove. I get lot of pimples and black spots in my face..It happen last year also when i use one new cream at that time a dermatologist suggested clyndamycin and benzyl peroxide lotion and dermadew soap. my face become soft and clear.. shall i use the same lotion and soap now.. and am using Lactocalamine lotion now. Doctor: Hello, Thanks for writing your query to us.You can continue clindamycin, benzyl peroxide lotion & dermadew soap. Continue with lactocalamine lotion daily.Avoid touching your face frequently.Use daily suncreen.Have good night sleep.Avoid stress & exertion.Have healthy nutritious diet. Drink plenty of fluids.Avoid dehydration.Take care."
},
{
"id": 77940,
"tgt": "Suggest treatment for persistent cough with watery sputum",
"src": "Patient: Hi! Happy new year to all doctors.My problem is I was alright 15days before then I had fever for 1day, followed by cough and greenish expectoration.I had levofloxacine 500mg for 5days with Brozedex cough syrup.Sputum became watery and scanty and cough persisted without fever. I had cetrizine and syrup dextromethorphan & tripolidine. Cough with little watery sputum persisted.Then I had cefixime 200mg bid 5days.But symptoms not resolved.Please give me your expert advice. Thank you. Doctor: Hi,Dear,Thanks for your query to HCM.Dear I read your query and reviewed it with context to your query facts.I understood your health concerns and feel Concerned about them.Based on the facts of your query, you seem to suffer from-Chronic Cough? of 2 weeks period.Remedy-Reduce on Anti-tussives and anti-histaminics-Plenty of fluidsTab Zincovit x 10 days time.Blood tests for eosinophilia and Stool for Ova Cyst to rule out worm infestation.Antibiotics would not help if no relief after 5 days course.Shift to above regime.If no relief Review case with your doctor,Who would consult Specialist -Chest Physician-and get Sputum AFB done with IGRA blood test rule out any old TB infection in your case.Hope this would help you to resolve your health issues in the best way possible.Welcome for any further query in this regard. Good Day!!Dr.Savaskar M.N.Senior Surgical SpecialistM.S.Genl-CVTS"
},
{
"id": 202144,
"tgt": "Could clear discharge and burning sensation on tip of penis be due to recurring chlamydia or urithritis?",
"src": "Patient: Hi I tested positive for chlamydia 2 weeks ago. I was treated with doxycycline and ended treatment this Monday. The symptoms seem to disappear. I was retested on Tuesday with a result of negative. I am experiencing clear discharge again and have a burning sensation at the tip of my penis. It doesn t burn when I urinate though. I am awaiting results for bloodworm for other stds. Does it sound like chlamydia is back or could it be urithritis or something else? Doctor: HelloThanks for your query,based on the facts that you have posted it appears that you have been diagnosed to have Chlamydia infection and treated with Doxicycline for a week Now you have burning sensation and have clear discharge from penis .This is mostly due to inflammation of urethra (Uretritis) ..You need to continue antibiotics for couple of days .Dr.Patil."
},
{
"id": 20661,
"tgt": "What causes panic attacks?",
"src": "Patient: Hi my name is Nathalie and I live in Sweden. I'll have to excuse my bad english but I'll try. I have a problem and I hate it. It ruins my life. Every time I go to school, I get panic attacks so I must go home again. I dare not go to a doctor for real. what should I do.? Doctor: Hello Nathalie!Welcome on HCM!Your symptoms seem to be related to agoraphobia. I would recommend consulting with a psychotherapist, which is not a real doctor and can help you deal with this fear. If the problem persists, the next step would be consulting with a psychiatrist. Hope to have been helpful!Wishing good health, Dr. Iliri"
},
{
"id": 32464,
"tgt": "What causes cold,violent shivers,headaches,fever and nausea?",
"src": "Patient: Hi. My boyfriend has had the following symptoms maybe 3 or 4 times in the span of about a month or a little more. He gets cold, shivers violently, has a headache, a fever, and feels like throwing up. Sometimes he gets one or two of the symptoms, sometimes all, but it keeps coming back. What's wrong with him? He is 16, about 140 pounds, 5'8\". He's only had some minor sports injuries. Doctor: Hi, dearI have gone through your question. I can understand your concern. He has fever with chills and rigor, headache and nausea. It can be due to viral infection, malaria or urinary tract infection. He should go for complete blood count and urine routine and microscopic examination. It will give you exact diagnosis. Then he should take antibiotic treatment accordingly. Hope I have answered your question, if you have doubt then I will be happy to answer. Thanks for using health care magic. Wish you a very good health."
},
{
"id": 7312,
"tgt": "Does laser treatment affect conceiving ?",
"src": "Patient: Hi All! Kindly help me providing information : Is any side effect of Unwanted hair Removal of FACE by Laser Treatment. Can it create any problem If she is going to conceive a child. Please suggest better Treatment : LASER or anything else . Doctor: hello, it may cause itching, redness, and swelling ,hypo or hyper pigmentation around the treatment area.it can be reduced by treatment with an appropriate laser type .sometime allergy may occur.but it will not cause problem in conceiving the child. take care"
},
{
"id": 93765,
"tgt": "Intense pain in stomach. Diagnosed spasming and inflammation. Relief with serax. Safe?",
"src": "Patient: hi I have a problem with my stomach it was diagnosed after some test I was told it was spasming and inflamed and at times twisting causing intense pain when it happens, I take I or 2 imovane for sleep and have discovered serax helps when I get an episode of trouble,to take during the day I don,t know what it does but it sure helps does this sound ok to you people there....thanks Doctor: Hi welcome to Health care magic forum. Tnks for choosing H.C.M.Forum. You have Problem with stomach, Diagnised as spasming and inflammation and twisting. You are getting relief with serax. Whenever there is pain of stomach or intestines, there is severe contractions of the musclefibres which causes pain. some times due to skeletal muscle contractions there will be twitching and external pain. Any how you got relieved of the pain so don't worry. this can repeate at any time so it will be a guide for you for further treatment. Wishing for a quick and complete recovery. Best wishes."
},
{
"id": 29999,
"tgt": "Suggest treatment for UTI along with pain in the hands",
"src": "Patient: Problem just started last night when I realized I was having difficulty urinating. My hands started to ache. I took cranberry pills and UTI discomfort dissipated, until morning. UTI discomfort returned as did pain in the hands. Took Azo, and drank water, but discomfort has remained throughout the day. Doctor: Thanks for your query . First I advised you to send urine for culture and sensitivity examination to know the causative agent and effective antimicrobial agent .you should drink plenty of water , can take antispsmodic medication like drotavarine or dicyclomine for pain relief , alkalinizer syrup can be taken if there is burning during micturition .review me wih urine report then I wil suggest you further managment."
},
{
"id": 202731,
"tgt": "Rash in hand, blisters, rash on penis, itching, huge bump next to it",
"src": "Patient: I have been getting this rash on my hand in the form of 3-dot patterns and the dots tend to blister . Now I have it on my penis (3 dots on the tip, in a triangle shape) and it itches really badly. There is also a huge bump/sore next to the head and it s red/black. Could it be poison ivy? I think that s what is on my hand. Doctor: HIThank for asking to HCMI can understand your concern, this could be scabies at least cab be said from the history given, if this is being diagnosed as scabies then in my opinion this need to be treated with the following medicines,1) Gama benzene lotion for local application apply this for consecutive three days 2) Tab Levocetrizine 10 mg three times in day3) Tab Azithromycin 500mg once in day for three days hope this information helps you, have a nice day."
},
{
"id": 57184,
"tgt": "Suggest treatment for stone in the pancreas",
"src": "Patient: Hi,I am Nayan from Bangladesh. I have a stone in my pancreas head. so i need help to meet up a doctor in cmc hospital. The size of my stone is (1.5*1.6)cm. And if i suggested to surgery how much money needed for this purpose. so Please help me and oblige thereby.ThankfullyNayan Doctor: Hello Mr Nayan,I am so sorry to hear regarding your condition. Stones in the pancreas head are not very common, but do cause a lot of problems including pain, poor absorption and malnutrition and can also lead to diabetes due to pancreatic failure. There are two ways of dealing with the stone, based on how close it is to the pancreatic duct opening. If it is close to the opening, then it could potentially be treated with endoscopic technique (ERCP). If however the stone is big, then it will necessitate surgery to remove the stone. There are two CMC s in India. Vellore in the south and Ludhiana in the north (where Dayanand Medical College is also located). If needed I can put you in touch with the Gastro-enterologist in CMC, Ludhiana. Regarding the cost of the procedure, if it can be done endoscopically, then I would assume a total expenditure of about 35-50 thousand rupees (including the necessary scans). If Surgery is required, then the cost could be around 1.5-2 Lakhs. Hope this helps and please do not hesitate to contact me for any other details required."
},
{
"id": 9745,
"tgt": "Can alopecia be stress related?",
"src": "Patient: Hello, I am a 69 yr old female and I live in Quebec, Canada. In the last 6-8 weeks, I have noticed my usually thick and plentiful hair is thinning, especially in the front and top of my head. Also, in the last couple of weeks, I have felt a small dip/depression at the top of my skull at the spot where I have a patch of hair loss. One doctor calls it alapecia caused by stress but wasn't concerned about the depression in the skull. The Dermatologist didn't seem to have an answer for the hair loss but prescribed a cortisone type lotion for the chronic itching of my scalp, which seems to help. However, I think the depression in the skull is getting larger and I am wondering if you have any idea what is causing that. I have had three once-a-year infusions of Aclasta (the Canadian term for Reclast) for my advanced osteoporosis and am thinking that might be the culprit for the hair thinning and patches of hair loss. I've been dying my hair for a number of years with no apparent allergic reactions. My hairdresser has been using environmentally friendly (\"green\") products in the last few years, but I can't say if that's what causes the itching or the hair loss. Please let me know what your thoughts are on all of this. Thank you. Doctor: Hello, Perhaps you have androgenetic alopecia. I suggest you to use minoxidil 5% solution, twice daily. Hope I have answered your query. Let me know if I can assist you further. Take care Regards, Dr Kakkar S., Dermatologist"
},
{
"id": 154544,
"tgt": "How to treat stage 4 bladder cancer?",
"src": "Patient: my dad was diagnosed with unknown priamry cancer in feb 2010, we wnt to MD Anderson where he was told it was terminal and he could do chemo for pallitive reasons, fast forward to now.....he has been done with chemo for 3 months, he as in the lasr 3 wks really gone down hill. he is 69 btw. His left kidney is non working, right kid only 40%, they placed a nephosotomy tube to help drain, as he was not urinating x 3 days, edema build up, his pain is increasing, and they have found stage 4 bladder cancer. He came home tonight, walking with a cane, weak, I just wonder how long this can go on...its so sad to me that he is in so much pain. I know the prognosis isnt good Doctor: Hi, dearI have gone through your question. I can understand your concern.Treatment of stage 4 bladder cancer is chemotherapy and radiotherpy. Treatment is only palliative. It can not cure the disease. It just bhelp to increase some life span. Pain killer like morphine and other symptomatic treatment is needed. However despite of all treatment prognosis is not good and life expectancy is very low. I am very sorry to say but one should not expect much from this.Hope I have answered your question, if you have any doubts then contact me at bit.ly/Drsanghvihardik, I will be happy to answer you.Thanks for using health care magic.Wish you a very good health."
},
{
"id": 148171,
"tgt": "CTA of neck revealed a pharyngeal lesion. Is it a bad sign?",
"src": "Patient: I just had a CTA of my head and neck, with and without contrast. The vascular findings came back normal however; the CTA of the neck revealed a pharyngeal lesion. How likely is it that a CTA would reveal a soft tissue abnormality and is it a bad sign that it did? Doctor: Hi,Thank you for posting your query.CT angiogram is mainly done to look for a vascular lesion. So, the normal angiogram riles this condition, and that is a good news.Pharyngeal lesion seen or suspected on the CT angio needs to be further evaluated. This would require an evaluation by an Head and neck surgeon, and followed by MRI scan of the neck region.There is no need to worry about the pharyngeal lesion, as it can be treated well after proper diagnosis. I hope my answer helps. Please get back if you have any follow up queries or if you require any additional information.Wishing you good health,Dr Sudhir Kumar MD (Internal Medicine), DM (Neurology)Senior Consultant NeurologistApollo Hospitals, Hyderabad, IndiaClick on this link to ask me a DIRECT QUERY: http://bit.ly/Dr-Sudhir-kumarMy BLOG: http://bestneurodoctor.blogspot.in"
},
{
"id": 8066,
"tgt": "I am getting pimples and blackheads because of dust pollution, what can i take ?",
"src": "Patient: daer sir , i am 18 years old and i have problem regarding blackheads and pimples . whenever i ride my bike lot of dust and pollution cause my face to pop up a pimples and blackheads . i have used many blood purifires , lacto calamine cream , facewashes and ep-ac soap.it shows there result but after some time pimples and blackheads again starts popping up ? will u please recommend me some advice ? Doctor: Dear Sagar Wash your face regularly, if possible with soap Wearing a helmet is good, however the humidity inside the helmet might worsen your problem You can try a salicylic acid based face wash too try benzoyl peroxide on the pimples that appear twice daily All the best"
},
{
"id": 67715,
"tgt": "What causes lump on the ear with pink eye?",
"src": "Patient: Hi, I went to see the nurse practitioner yesterday who said I had pink eye. He gave me eye drops. This is day 2 no improvement. On the same side, I just found a lump near front of ear lob near jaw line. Could this all be related? Some type of infection? Doctor: Hi,From history it seems that you might be having conjunctivitis giving this problem.Due to infection there might be having pre-auricular lymph node enlargement.you might require one course of oral antibiotic medicine.Put antibiotic eye drops every 2-3 hourly.Ok and take care."
},
{
"id": 181515,
"tgt": "What causes red skin tag in the gum?",
"src": "Patient: so my tooth has been very loose for a while and I was eating and it came out and so it seemed normal but then I moved my tongue to were my tooth that fell out should have been and I felt this blob that tasted like blood so I went to a mirror and i have this weird flappy red thing and I'm not sure what to do Doctor: Hi Dear,Welcome to HCM.Understanding your concern. As per your query you have red skin tag in the gum which could be due to oral fibroma, tissue hyperplasia, inflamed mucosa fold Plica Fimbriata. It could be due to trauma, oral sex, infections and burning with hot foods or chemical substances. Need not to worry about it. Apply cold compresses on gum area. I would suggest you to apply lignocaine over it as it will relieve pain as it will act like numbing gel. You should take anti-inflammatory painkiller like Ibuprofen. Avoid hot, sharp food substances. do gargles with Listerine mouth wash. Visit oral pathologist once and get it examined if symptoms keeps on persisting.Hope your concern has been resolved.Get Well Soon.Best Wishes,Dr. Harry Maheshwari"
},
{
"id": 139152,
"tgt": "Suggest treatment for swollen and painful arms",
"src": "Patient: I came home from the hospital two days ago. Yesterday my left arm swelled up turned red and discolored all the way from my fingers to my shoulder. I had been in the hospital eight days to have fluid drawn off from around my heart. The Pick (Pic?) Line was in my left arm. I am three hours away from the heart hospital that I was in. My arm feels better when I keep it elevated in my recliner chair and I am keeping a heating pad on it. The normal flesh color is returning and some of the swelling is going down, but I m not sure what to do about it or what caused this problem. I have no fever or any other symptoms. Doctor: Hi these symptoms come due to the intravenous fluid extravasation, or due to infection at the site of putting canulla called as thrombophlebitis. in both situations symptoms get corrected if you keep your limb elevated constantly do fisting of fingers tightly and release, there is one ointment by name of SUMAG which can be locally applied over the limb it helps to reduce swelling. if your symptoms do not settle or you have fever, then your consultation with your doctor will be must"
},
{
"id": 15138,
"tgt": "Dark ring around mouth. used coco butter, Palmer ended as got rash and irritation. Treatment?",
"src": "Patient: I'm a pre-teen, I don't have any spots but what I do have is a dark ring around my mouth I'm a girl and it makes me look like I have a mustache. I've used coco butter and tried palmers and some other things but it just makes my mouth darker or I end up having an irritation or get a rash! I've had this for a while now PLEASE HELP! Doctor: usenon oily applicatins no toxic soap shampoo and creams non spicy foods non greesy non fatyy foodskeep skin dry and airyif you want more results withdraw milk and diary for few monthsapply clindamycin gl bdtretinoic acid or retinoic acid gel nightswitch off light after tretinoic acid gel as it reacts with lightdo long to get it clear"
},
{
"id": 204489,
"tgt": "What causes confusion and memory loss while suffering from depression?",
"src": "Patient: I am 47, healthy, some history of depression, ... last couple of weeks noticed significant confusion, memory issues, and general symptoms resembling dementia. Today, experienced moments of extreme confusion and some fear at grocery store, and daughter came to help. Other symptoms today: nausea, lethargy, confusion, hot flashes ( I assume, since I\u2019ve never experienced) Doctor: Hello and Welcome to \u2018Ask A Doctor\u2019 service. I have reviewed your query and here is my advice. The conclusion, nausea, lethargy etc are concerning for possible delirium. Delirium can occur due to several.underlying metabolic, electrolyte imbalances, infections, medications etc. A detailed evaluation is warranted and please be advised to follow up with emergency room. Hope I have answered your query. Let me know if I can assist you further."
},
{
"id": 32289,
"tgt": "Is anemia, related with itchy skin infections?",
"src": "Patient: I was told by my doctor earlier in the month that I was anemic and lately I've noticed that my skin had broken out into small rashes all over my body. They are dry, itchy patches of raised skin and do not seems to go away even when I moisturize. Could my skin problems be due to my anemia? Doctor: Hello, I understand your concerns. Anemia does not cause any itchy skin rash. A picture of your rash would help to narrow the possibilities. If it just feels like 'chicken skin' on your arms and legs! try exfoliating with a scrub. If it is very dry, keep moisturising with an emollient like liquid paraffin or coconut oil (which work wonders when moisturisers don't) It is important that you take iron supplements for your Anemia . Hope this helps you."
},
{
"id": 112781,
"tgt": "Have middle back pain, shoulder blades pain, headaches, had non functioning pituitary tumour. What is the cause?",
"src": "Patient: Hi, My name is Todd, I'm 50 years old, and have been experiencing chronic sharp pain in my middle back just below, and sometimes between my shoulder blades for 9 months. The pain went away for 6 of the 9 months, but returned about 2 months ago. Laying down helps; I wake with no pain, but by the middle/end of the day it becomes severe. I started having severe headaches 8 months ago and discovered that I had a non-functioning pituitary tumor which I had removed in November. The headaches stopped, but have recently returned and am wondering if they could be related to the back pain? I am in good shape, have no weight loss and appetite is normal, and am wondering what could be causing the back/head pain? Thanks so much! Todd Doctor: You should be meeting your doctor who removed pituitary tumor to evaluate if there is any residual growth which can be done on MRI of head..Headaches couldn't be related to your back problem."
},
{
"id": 37903,
"tgt": "Need treatment for lyme disease",
"src": "Patient: My question is regarding possible lyme disease or coinfections. I have been having horrible symptoms since january that seem to be attacking my nervous system and joints and muscles and tingling on my left side of face and leg balance issues and random anxiety attacks. Also had some high heart rate issues for about 3 weeks back when i first started feeling bad and then the other above symptoms all came afterwards. So would a western blot test or another late stage test need to be performed or do i need ot see a neurologist or infectious disease doctor. I havent felt 100% better in the last 5 months but better from when this all started but have no energy or as active as i once was. Thanks for your feedback Doctor: Hello,Welcome to HCM,As you were diagnosed to have lyme disease which is an infectious disease caused by Borrelia bacteria and it is a tick borne disease transmitted to humans by the bite of infected ticks.The symptoms are suggestive of the lyme disease.I would suggest oral doxycycline, if you have taken this and if there is resistant to this you can take either cefuroxime or azithromycin. These medications should be taken for atleast one to four weeks.The drugs used to treat this condition is not known to cause any cancerous conditionsThank you."
},
{
"id": 184567,
"tgt": "Suggest treatment for tooth pain with breaking off into bits",
"src": "Patient: good morning, I am 37 year old having problem in my teeth.20 years back I had pain in my teeth during my examination perriod . doctor suggested me for RCT and given NISE tablet. After taking tablet my pain get subside, and later on that start to break in particles without much pain. so it was ignored. but now only root is left without to much pain. should i go to doctor for complete removal of it. what will be expected cost. I do not take tobacco,smoking or beetle leaf Doctor: Hi Sir,When a patient with similar problem like yours visits my clinic, I will first check the integrity of the remaining root.I will examine clinically and take X-ray.If your remaining root is not mobile and atleast very little amount of tooth structure is remaining we can save the tooth by doing Root canal treatment, then place a screw like structure inside the root which will be projecting above the gums.Using tooth filling material we can artificially build tooth shape.Then place a strong cap over this. But if your root is in bad condition, remove it.It costs from Rs.200 to Rs.1000 depending on which tooth and the difficulty in extraction.Then replace the extracted area with artificial tooth soon to prevent the adjacent teeth from moving into the extraction space.Hope your query is answered.Please dont heditate to contact me for further doubts.Happy to answer you."
},
{
"id": 81919,
"tgt": "Suggest treatment for frequent episodes of chest pain",
"src": "Patient: Hi I have been having chest pains had heart checked out it s not that .its uncomfortable comes and goes throughout the day feel it now in bed.take deep breaths goes for abit. Under a lot of stress at the moment mum dad died I have had this before when upset. Could you please help thank you x Doctor: Thanks for your question on HCM. Since you have ruled out cardiac cause for your chest pain, I think it is anxiety related only.I believe anxiety is the cause because you clearly said that your pain worsens when you are at stress, after death of your mom and dad. This clearly indicates anxiety as the cause.So better to consult psychiatrist and get done counselling sessions. Try to identify the stressor in your life and cope up with it.Avoid tension. Be relax and calm. You may also need anxiolytic drugs too. Newer anxiolytic drugs are very safe, non habit forming and less sedative. So no harm in starting it, if necessary."
},
{
"id": 82734,
"tgt": "Can chlorine in stem cell product be used to treat lupus?",
"src": "Patient: I never use Chlorine. However, there is a stem cell product being advertised, which contains Chlorine. I assume the amount is minimal. However, I have fairly full-blown autoimmune dysfunction, including lupus, Hashimoto s thyroiditis, and about 15 others, along with about 30 other conditions. I am considering the product because it professes to heal so many issues. Any idea on whether or not I should use it? I am 67 and female. Thank you. Doctor: Good evening. I am not sure how much chlorine will help for curing your autoimmune conditions. These are all experimental things and how good they will prove it or not., it s doubtful. Please look into the side effects before going for it as the stem cell theraphy can be harmful too"
},
{
"id": 90635,
"tgt": "What causes stomach pain at night?",
"src": "Patient: Hello, i have had stomach pains for the last few weeks or so but they have been becoming less and less often. They occor mostly around 3 at night, waking me up. The pain is as though the upper area of my abdomen is being twisted. Ive been eating my normal diet, slowed down my exercise a small bit though. Doctor: Hello! Welcome to HCM.This is the typical history in a case of ?duodenal ulcer.This is due to excess secretion of acid producing an ulcer.This can be due to hot/spicy food or late night meals or excessive steess or alcohol/smoking..Avoid this.Endoscopy is advisable for confirmation of diagnosis.Consult your doctor for medicines like omeprazole, syp. sucralfate etc.Hope this information is useful.Wish you a good health.Thanks.Regards."
},
{
"id": 18356,
"tgt": "Is it safe to get pregnant while on stents due to heart weakness?",
"src": "Patient: I was 38 when I had my first heart attack. Within a month I had a massive 3rd heart attack. Which done about 45% damage to the heart muscle. I have also had 4 stents. I was married about a year ago and we would really like to have a baby together. My question is, is there a chance that the baby and I both could survive pregnancy with proper medical care. Doctor: Hello and Welcome to \u2018Ask A Doctor\u2019 service. I have reviewed your query and here is my advice. The pregnant woman has an increased risk of acute coronary syndrome because of hemodynamic, hemostatic, and hormonal changes. Atherosclerosis is one of the main causes of myocardial infarction in pregnancy. So better to plan before conceive, please consult your cardiologist he will examine through echo and other cardiac function tests. Hope I have answered your query. Let me know if I can assist you further. Regards, Dr. Penchila Prasad Kandikattu"
},
{
"id": 147543,
"tgt": "What is the pain on the right side of my head?",
"src": "Patient: Hi doctor, I'm feeling pain in the right side of my head. Sometimes it feels like just a heavy head but still at the right side. Pain in my right eye and sometimes ear. Feels more pain when bend down to pick something up and it ends in a matter of second. I don't know what to do and how to proceed. Dont have anyother problem. Is it something to do with my eyesight? I'll be grateful if you could suggest some solution to my problem! Thank you! Afsheen Doctor: HIThank for asking to HCMI appreciate your concern, looking to the history given here I could say that this could be frontal sinusitis, you have to drink plenty of water and have steam inhalation as many time as you can, if I would be your doctor then I would treat this with the following medicines,1) Tab Acetaminophen three times in day2) Tab Ibuprofen 400mg three times in day3) Tab Loratidine with pseudoephedrine three times nothing to worry about this, this would come around, hope this information helps you have a good day."
},
{
"id": 125616,
"tgt": "Is increasing swelling in the ankles overnight a sign of something dangerous?",
"src": "Patient: I noticed my ankle starting to swell yesterday but thought nothing of it cause I haven t had any injuries , but this morning I could barely put on my shoe , it s huge. It doesn t hurt or anything just really uncomfortable to walk on. What could cause my ankle to swell like this overnight and Could this be dangerous or life threatening? Doctor: Hi, You should investigate for sudden swelling over a foot. Hypoproteinemia, liver malfunction, cardiac problem, kidney disease etc can be the reason for that. For that, I suggest you investigate with SGPT, SGOT, EKG, urea, creatinine estimation, serum protein estimation for further work up. According to cause specific treatment given. You should consult a physician for examination and discuss all these. Hope I have answered your query. Let me know if I can assist you further. Take care Regards, Dr Parth Goswami, General & Family Physician"
},
{
"id": 71939,
"tgt": "Is drop in the body temperature with bronchitis normal condition?",
"src": "Patient: My son is 2. He has been diagnosed with asthma. I took him to the dr Friday and they said he has bronchitis and a touch of pnuemonia. His breathing seems worse today. He is on an antibiotic, prednison, and dy G. Also we are doing albuterol breathing treatments every 3 hours. His O2 is between 90 and 93 when he is awake, but when he sleeps it drops to around 88-82. Is this normal? Doctor: helloUsually O2 drops during the sleep cycle.But in his condition you should discuss with his treating doctor for this.RegardsDr.Jolanda"
},
{
"id": 162026,
"tgt": "Suggest medication for fever",
"src": "Patient: Hi, My 9 month old is running a fever, mainly at night of 101 to 102 for the past two weeks. We went to the Dr. last Monday and there is no sign of infection. She was very fussy and lethargic the following Friday, Saturday she was normal, Sunday she was lethargic and feverish again. Hardly no sleep last night but lots lots of crying no matter if we were holding her or not. What could cause this other than infection (ear, nose, throat is clear) and is there more we can do other than give fever reducer? Doctor: Hi, Fever for 2 weeks in a 9-month-old infant, is something that we need to worry. Since your child is lethargic, it is better to evaluate her. Simple Viral infection usually won't go for 2 weeks with lethargy. Rather than controlling fever with antipyretics, it is better to identify the cause and treat it, which will control the fever. For fever, you can give paracetamol or meftal. I would advise you to consult a paediatrician, for further evaluation and doses of medications. Hope I have answered your query. Let me know if I can assist you further. Take care Regards, Dr Nirubhan Bharathy, Pediatrician"
},
{
"id": 217085,
"tgt": "Suggest solution for persistent knee pain after knee replacement",
"src": "Patient: Have had both knees replaced, but 4 years later I still have pain in both knees. It is constant in lower middle back and radiates through the groin or bottom cheeks along thighs and down both legs. Doctor prescribed tramadol, morphine patches of 50 mgs, etc as a support. Should I settle for pain relief that is ineffective? Doctor: Thanks for keeping trust with us. ..as per your history under went knee replacement ....and using patches for analgesia. ..In my opinion physiotherapy is main thing u should should focus on it ..."
},
{
"id": 158367,
"tgt": "Chronic lymphoma, enlarged prostrate. Radiation therapy scheduled, cancer cells on forehead. Prognosis?",
"src": "Patient: my husband is 74 years old 3 years ago diaggnosed with chronic lymphoma today microglobulin 5.9 deos it need urgent treatments also sufferes from enraged prostate wears cateter scheduled for prostate surgery plus 2 months ago had squamous carcinoma removed from forhead cancours cells remained and is scheduled for radation on forehead which problem most urgent my mail is YYYY@YYYY Doctor: Hi there, Thanks for your query. If I understand correctly he has old diagnosis of lymphoma and the question is whether to go for prostate surgery first or radiation first for forehead skin cancer. In such situations, cancers treatment is a priority and should be done first. Prostate surgery can be considered later. Take care Regards"
},
{
"id": 214879,
"tgt": "Is cranberry juice and pomelos good for health?",
"src": "Patient: i heard about pomelos being good for high blood pressure , UTIs esp. in pregnant women. i also heard that Cranberry juice thins the blood...... this suggests to me that a life-long healthy diet will prevent health problems from starting. And drugs for each problem are derived from the foods / vitamins / minerals that the patient is actually avoiding. would you agree with my conclusion. I have no science or healthcare education or qualification. Doctor: Hello! Ofcourse Healthy and balanced diet is important for life. No doubt,it is better to prevent a disease rather than consuming energy and money on cure. Here are some facts you want to know... Pomelos It is citrus fruits containing abundant of Vitamin C that enhances your immunity, beside that it is rich in fibers, bioflavonoids (protects from cancers) and minerals (regulate blood pressure). Cranberry juice It contains substances that may affect your health, for example antioxidants help body to prevent cancer development. It also prevents bacterial growth in urinary tract and may benefit in UTIs. Beside all this excessive use may decay your teeth and oxalate in juice may lead to kidney stones. In short, proper use of healthy and balanced diet with regular exercise is best way to a happy life. Take Care!"
},
{
"id": 111657,
"tgt": "What do you suggest for the addiction to roxycontin for back pain?",
"src": "Patient: I think my daughter is addicted to roxycontin although she says that is the only thing that helps her pain,she says she has severe back pain from a wreck she had 4yrs ago she was left with some alterations in her moter. Skills,when she walks it looks as if one leg is weaker than the other Doctor: Hello, I have studied your case. Dependence on medication can be hazardous ,you need to contact your treating doctor and start alternative withdrawl medication.You can take neurotropics like methylcobalamine with pregabalin for pain.Hope this answers your query. If you have additional questions or follow up queries then please do not hesitate in writing to us. I will be happy to answer your queries. Wishing you good health.Take care"
},
{
"id": 189707,
"tgt": "Jaw feels swollen and hard to open mouth. Had tooth filled. Is this normal?",
"src": "Patient: Hello , well i ve just been to the dentist to get an old filling removed and cleaned and got a new one one. the problem is my jaw feels abit swollen and i find it hard to open my mouth now close to the side where my tooth was filled. it was hurting like this before i went to the dentist . is this normal? and what can i do to make the pain go? Doctor: Dear friend.Thanks for sharing your concern.This is not normal.I think the tooth which has been restored(filled) now after removing the old filling is already infected and also it has involved the pulpal tissue.( site where nerves are located.)Further the hardness that you are feeling in the jaw, now is the initiation of abscess around that tooth.First of all i would suggest you to revisit your dentist and get your tooth clinically evaluated .also if required get the filling removed.An x ray will be helpful along with clinical evaluation.After that suitable antibiotic and analgesic can be started.under its coverage RCT can be done and the tooth can be saved.hope it helps .thanks"
},
{
"id": 197390,
"tgt": "What causes pain in upper right chest and shoulder?",
"src": "Patient: sir, My name is shamsudheen, now I am living in Dubai, since today morning I have felt pain on upper right chest,shoulder and just back side of upper right chest. it is also feeling when i eating something, or coughing. still it is continuing ,when i will handup righthand and down hearing a small sound, my history.......... I am a diabetic, since 10years, did angioplasty 2 years ago with stenting 3. now physicaly ok, no left or midle chest pain. what I do sir, I think tomorrow morning will go to doctor, now time is midnight. hoping reply thank u sir , with lovingly shams Doctor: Hi Dear !! Thanks for your query to HCM .Read and reviewed your query and health concerns. IN the given situation, You seems to have suffered from-TOS(Thorasic Outlet Syndrome) with Neuromuscular pain in right upper chest and shoulder.Other causes like-Constochondritis right chest / GERD with coughing needs to be ruled out.CT chest and X-ray Cervical would fix the causes of right shoulder and upper right chest pains.Hope this would help you to plan further of this complex illness of yours.If need be, update any health issue 24 x 7 by a direct question to ME, at following HCM link-http://doctor.healthcaremagic.com/Funnel?page=askDoctorDirectly&docId=70229Dear, if satisfied,Don't forget to close this query with YOUR pleasing feedback comments to rate this reply and service, to boost the morale of incoming Emergency patients like YOU, at HCM services.If you want to update more details and ask more update queries ,You are most Welcome herewith !!Good Day!!Wishing Good Healthy Life in time to come!!Dr.Savaskar M.N.Senior Surgical SpecialistM.S.Genl-CVTS"
},
{
"id": 222461,
"tgt": "What causes increased white discharge while urinating or passing stools during pregnancy?",
"src": "Patient: Hello, I am almost four month pregnant with my first child and when i go to the toilet to ither urinate or have a poo i notice in the toilet a thickish white discharge which has only been happening the past week. I obviously have had discharge before but it seems to have increased when going to the toilet. What is it?? sould i be worried?? and can it harm the baby?? Doctor: Welcome to the healthcaremagic.com. I honour your consult here and being your expert healthcare provider I am assigned to help you further in this case. I have reviewed the details you have given.This is normal Physiology in pregnancy and nothing seems to be wrong. In pregnancy, it is normal to have discharge off and on. What happens is the mucous inside the vagina gets thicker a bit, and there are many more hormonal changes during pregnancy affecting the viscosity of the mucous secretions inside vagina. If there is no itching in or aound the vagina; no smelliy discharge; then it is ok. Otherwise it may have been a bacterial or fungal infection and needs definite treatment.If you think it is bothering you a lot, kingly discuss it with your OBGYN for evaluation in person and diagnodtic labs if needed. She will help you further in this regard.I am at your disposal for expert care. You may land again to healthcaremagic.com anytime you want. We are striving hard to make the forum better for you.Healthiest Regards!Dr. Sumaira Kousar"
},
{
"id": 189396,
"tgt": "Spot on tongue, not bleeding or painful, smooth, shiney",
"src": "Patient: I had a spot That looked like a bite on the top far right side of my tongue almost two weeks ago. The spot has spread along the whole right side where it looks as if a layer of the right side of my tongue has eroded away. It is not bloody or painful, but is smooth and in shiney in places. There is also a small area same condition on the front of tongue. Doctor: Hi, Thanks for asking the query, As per your clinical history i suppose the papillaes on your tongue are erroded and inflammed resulting in smooth and shiny tongue, this occurs in the condition known as geographic tongue, vitamin B12 defieciencies. Start taking multivitamin suplements, use chlorhexidine mouthwash gargles twice daily. Maintain a good oral hygiene. Take complete balanced diet. Avoid eating of hot and spicy foodstuffs. Hope this helps out. Regards."
},
{
"id": 210076,
"tgt": "What causes mental stress?",
"src": "Patient: when I was little a friend of my moms would try to beat confessions out of me, trying to get me to admit I did things that her own son was responsible for, she would hold me underwater in the bathtub till I nearly drowned and she would repeat this till I confessed. could this cause mental or physical damage that would affect you six or seven years later? Doctor: DearWe understand your concernsI went through your details. I suggest you not to worry much. You are right. Such traumatic experiences in childhood leads to anxiety episodes in future. In childhood human mind is under development. Childhood has been classified into several stages of development by several eminent psychologists. Certain predesigned developments are supposed to happen every stage. If a child is deprived of any such development, there is trauma later. I think you should be more clear here. Please post a direct question to me in this website. Make sure that you include every minute details possible. I want to know what are all things you are experiencing, There are so many possibilities. Be open. I feel there is something negative in future for you. Please do not ignore these symptoms. Then, after learning you, I shall prescribe some psychotherapy techniques which should help you cure your condition further.Hope this answers your query. Available for further clarifications.Good luck."
},
{
"id": 192558,
"tgt": "How to improve penile strength during intercourse?",
"src": "Patient: hi....after 2-yrs of our marriage we are planning for a baby, but we both are worry bcoz my husband is getting problem in pennis, he had not any pennis problem before 2-yrs. but now he does not get his pennis strong enough while intercourse......plz guide us wht to do..... Doctor: Hello,It may be due to performance anxiety. For further assessment you may require complete hemogram, lipid profile and electrocardiogram after physician consultation. Along with that Avoid stress and strenuous activity. Take balanced diet and proper rest. Avoid smoking and alcohol. Till then you can take tablet sildenafil one hour before sexual act.Hope I have answered your question. Let me know if I can assist you further. Regards, Dr. Shyam B. Kale, General & Family Physician"
},
{
"id": 29084,
"tgt": "What causes MRSA?",
"src": "Patient: What is MRSA??? I was given Mupirocin pre op fer my SCS procedure by my nero sergeon . But everything i read, it says not to get it in the nose area. I have the implant now. But, im having a horrible sinus infection thats is going around at the fitness center i work at. Is it ok to use this to help get over the infection, and will it help also prevent it?? Doctor: Hello, MRSA is a specific bacteria that have developed resistance towards a group of antibiotics, known as Methicillin-resistant-staphylococcus-aureus. It is very hard to deal with, and I would recommend you to go and visit infections or dermatologist. These cases mostly treated with vancomycin. Hope I have answered your query. Let me know if I can assist you further. Regards, Dr. Eni"
},
{
"id": 42322,
"tgt": "How can semen count causing infertility be treated?",
"src": "Patient: i am 29 yrs old,weight is 56, Height is 4\"10... medical history is slightly allergic to dust and cold(only seasonal) and my husband is 32 yrs, Height is 5\"7, weight 72, medical history is normal.. we had been married for 3 yrs.. we are not using any contraceptive measures..still we are failed to get progeny.. once, my husband's semen counts is tested... but it showed slightly count is low.. but not severe could be called as infertility.. please guide me , is any treatment is required Doctor: Hellothanks for using health care magicIt would have been easier to comment if you had stated the value of the sperm count. True, low sperm count causes infertility but it has to be really low.There are other possible causes of infertility such as less frequent intercourse, infections, hormonal imbalance etc.You should meet a fertility specialist or a gynecologist to do appropriate investigations and make a right diagnosis so that the right treatment should be given.Wishing you good healthDr Achuo"
},
{
"id": 224831,
"tgt": "Birth control pills, depo provera injection, missed periods for 9 months, ovulation?",
"src": "Patient: Hiya! I ve asked my doctor and hes useless so hoping you can help. Since I ve been fourteen I have been on some sort of birth control , either microgynon (30ED) contraceptive pill or depo provera injection . 1 1/2 years ago, I went on the injection and got 3 shots, and after this I went straight on the pill for 3 months again. Then 9 months ago I got another depo shot (I am 18 now), and I haven t had a period since I was on the pill, so no period in 9 months and depo ran out 6 months ago. I know it stays in your system for a few months afterwards but I have cleanse my body, drinking only water and eating healthily and still no sign of it coming. For the last 4 months I ve have swollen boobs, that are a little tender under the armpits however I know I am not pregnant because 1) I haven t had a period therefore i can t be ovulating 2) I have done several tests recently - all negative. I have regular unprotected sex with my boyfriend because I wish to have his baby as we are stable and I live with him and he wants his first child (he s 21). We have sex usually thursday - sunday up to 4/5 times a day and still nothing. I am slim - 5 foot 5 and 118 pounds, and I do not have much body fat which I heard is where it is stored. Today I was diagonosed with a kidney infection, which isn t bad but I have no idea why I haven t had a period? Is this normal for someone my size and after the detox? I had fake pg symptoms which you normally get after it leaves your body or as it is doing so. Today my doctor just said get on with it basically however he did give me utovlan 5mg three times a day to try and kick start my period, however I am worried because this is the same hormone as depo that it will cause more damage then good and I will have to wait another 6 months afterwards for things to go back to normal. I am TTC so can someone please help and say what I can do to help ovulation? I have started taking vitamins etc. sorry for the essay, also 2 months ago I had brown discharge for 2-3 days - could this have been my first post depo period? Doctor: Hello and welcome, It is a long and exhaustive history but it helps a lot. It seems that you have post pill amenorrhea. It is common and sometimes there is a delay for menstrual function to return after depo and pills. You had spotting so that may have been a sign of return to fertility. However not having periods does not mean not having ovulation. Periods occur 14 days after ovulation. So ovulation comes first and then periods. Many times women who are not using protection conceive in the first ovulation and so periods do not come as there is a pregnancy. So please get a pregnancy test done if you have not done it already. If it is negative then look for other causes of absent periods like thyroid dysfunction, abnormalities in prolactin hormone, ovarian cysts, premature ovarian failure and stress which all delay periods. See a gynecologist and get examined and evaluated. Treatment will depend upon the cause. Also you may need some ovulation inducing drugs to help in conception. Do not worry. All the causes can be treated. Hope this satisfies your query. Thanks for using HCM.\u00a0\u00a0\u00a0\u00a0\u00a0Feel free to ask any more questions that you may have. Dr Madhuri BagdeConsultant Obstetrician and Gynecologist"
},
{
"id": 59267,
"tgt": "High bilirubin levels. Liver ultrasound suggested. Worried about liver disease. Due to alcohol or high protein diet?",
"src": "Patient: Hello, I am a 25 year old female and just got blood test results back that my bilirubin level is at 3.4. The assistant who called wasn t very helpful beyond saying I am supposed to get an ultrasound of my liver. I am starting to freak out and get really worried I have a serious liver disease or something due to lack of information. I know speaking with my doctor is the best thing, but she is out for a week and I won t be able to. Do bilirubin levels ever randomly spike due to alcohol consumption or could it be due to a diet high in protein? Help! Doctor: Hi and welcome to HCM. It might spike, dut not such high. This is severe elevated bilirubin and it should be evaluated especailly in such young people. It can be caused by liver disease in most cases but also some blood disorders as well. He recommended ultrasound because without it we cant find out the degree of liver damage. After that you ll know is this just fatty liver or there are signs of fibrosis. ALso it can be caused by biliary duct obstruction and gallbladder issue. All in all you dont need to worry, this is something treatable and reversible. YOu cant have cirrhosis or malignancy. It can be alcoohlic damage,autoimmune or caused by drugs and dietary. Whatever it is it will be found and you ll know further tratment, WIsh you good health."
},
{
"id": 185081,
"tgt": "What causes redness in gum?",
"src": "Patient: i have hep c, about a year ago I noticed my gum turning red and a white film, my doctor told me I was alergic to whitening toothpaste which I had switched to, because of the hep c I have erthema nodosum and I am on minocycoline, my dentist said this med will help w/the gums, but its not, why Doctor: Thanks for your query. i have gone through your query. the redness in the gums could be because of gum infection secondary to deposits or it can be because of allergic stomatitis. consult a oral physician and rule out these two conditions. if it is because of gum infection then get the teeth cleaned. if it is allergic stomatitis change the tooth paste and apply topical steroids like triamcinolone acetonide(0.1%). i hope my answer will help you. take care."
},
{
"id": 173011,
"tgt": "Suggest treatment to cure vomiting",
"src": "Patient: My 8 month old has vomited twice this eve, after dinner & all his bottle. He was very snuggly & clingy & a bit floppy before bed. Should I worry? how do I tell if he s hungry or just upset in the night? And if I give him milk how can I help him not vomit again? Any advice appreciated. Doctor: HiWelcome to the HCMI understand your concerns but don't worry. Infants in this age group are prone to mild self limiting viral gastroenteritis episodes. This is due to their tendency to bite on everything in their reach.Usually these episodes do not last for more than a week. Only things that you need to take care of are:1. Proper hydration by regularly offering him feeds and oral fluids. Do not give excess sweetened juices as they may precipitate osmotic diarrhoea.2. Start a zinc supplement such as ZnD or Zinconia for next 2 weeks. This will help in improving his immunity to such infections and early recovery.3. Maintain a good hygiene in his surroundings to prevent such episodes.Hopefully this will help you to take necessary measures. I would be happy to help you for any further questions.Take care"
},
{
"id": 84416,
"tgt": "Is it safe to stop using Omnacortil abruptly?",
"src": "Patient: Dear Sir, My wife aged about 43 is taking Omnacortil 40 for the last one week as per doctor s advice for allergy cough. Cough has reduced I understand that it is steroid and it should not be stopped immediately and it should be tapered. Please advice Doctor: Hi,It is not safe to stop using omnacortil abruptly if it is used for a long period of time. Omnacortil is a steroid (prednisone) commonly prescribed to treat various inflammatory diseases such as bronchial asthma or allergic bronchitis. Sudden stoppage of omnacortil can produce 'acute adrenal crisis' or withdrawal symptoms such as severe fatigue, weakness, and joint pains. This happens if omnacortil is used for a long period of time. Since your wife has used omnacotril only for one week such withdrawal symptoms are unlikely to occur if omnacortil is stopped abruptly. So, it can be stopped as per her doctor's advice.Hope I have answered your question. Let me know if I can assist you further. Regards, Dr. Mohammed Taher Ali, General & Family Physician"
},
{
"id": 79755,
"tgt": "Suggest treatment for fever and allergic bronchitis",
"src": "Patient: Hi My dad is about 64 yrs and he has fever which ranges between 103 - 101 since last night. He also has allergic bronchitis. He is taking Sporidex 500, Dolo and deriphilin. Is this right medicine ? Secondly, he may have got the fever from my mom who had about 102 fever a few days ago. She took Novamox and Solo and she is now fine. Should his medicine be changed ? I feel Sporidex is the wrong medicine for his problem. Please advice. Thanks Arvind Doctor: hi , thanks for your questioni completely understand your problemactually in respiratory tract infection antibiotic is mandatory if sputum is purulent and choice of antibiotic depends. treatment starts with an empirical antibiotic after that if the patient is not responding we send a sputum pyogenic culture and senstivity according to which we can specifically know the pathogen and we can also know the antibiotic which will kill that organism causing infection . u can continue with the same antibiotic or else start on with a new second generation cefalosporin adter consulting a pulmonologist who can always guide you betteru can wait for a day or two to see the response. thanks/ regardsfeel free to ask more questionsmay god bless your family a good health"
},
{
"id": 52252,
"tgt": "Suggest cure for multiple ailments",
"src": "Patient: My mother is having health problems. Her dr is doing nothing. She spent 5 days in the hospital with no solutions. She passed out and when I took her to the hospital her BP was 80 over 40. She just had her gall bladder removed and lots of stones removed from the duct. She had a colonoscopy and everything was ok. She feels bloated. No appetite and when she does eat she get full fast. She has lost 16 pounds since the operation. When she has to have a bowel movement she says it feels like there is still more in there and she can t get it out. She said today she doesn t want to live like this. She has taken laxatives most of her life because of painful hemroides. Her heart rate is fast. Oh and she gets a fever. They don t last long but she shivers from them. In the hospital it was 102. They gave her Tylenol and it went away. Please help us figure this out. She is very weak also. Doctor: Hello, Your mother is having symptoms of Post cholecystectomy syndrome. She is having flatulence, bloating, dyspepsia, nausea, vomiting, indigestion, abdominal discomfort & fever. She can take antacids, PPI's & laxatives to get relieve. Let her take home made food cooked is scanty oil without spices. She should stay hydrated & cut on addictions(smoking, caffeine, alcohol) if any. She is suggested to get physically examined by her GP, get Blood (CBC, LFT) & Imaging tests (MRCP & ERCP) done to detect ductal dilatation, stricture, ampullary stenosis, retained stones or any other structural GI problems. Her treatment will depend upon the source of the pathology detected. Hope I have answered your query. Let me know if I can assist you further. Take care. Regards, Dr. Nupur K. General & Family Physician"
},
{
"id": 28133,
"tgt": "Is there any need to continue BP medicine with low blood pressure?",
"src": "Patient: my mom s blood pressure over the last hour has been very low, mostly the bottom number. the last reading was 145 over 64 but was as low as 51 on the bottom number. her question is should she take her blood pressure medicine as normal. She s kinda worried it may make it drop in the night too low. Doctor: Hi Welcome to Health care magic.I understand your query and concern.Yes she certainly needs to continue her blood pressure medications.Low number of the denominator need not be worry at the moment.All she need is good hydration with 3 litres of water .Reduction of daily salt intake to less than 6 gm/day will help.Monitor the blood pressure daily over 2 weeks to adjust adequate dosage.I advise you to have a baseline 2D echo and ECG on semi annual basis for better cardiac assessment.Consult an expert cardiologist for further help.Post your further queries if any.Thank you."
},
{
"id": 55974,
"tgt": "My Uncle is diagnosed with mild hepatomegaly with caudate lobe hypertrophy. Is there anything serious?",
"src": "Patient: My Uncle is diagnosed with mild hepatomegaly with caudate lobe hypertrophy. He is 45 year old and consumes alcohol on daily basis since last 12-13 years.According to the reports, his lever size in 170 mm. he has a swollen stomach. Is there anything serious.... any risk with his life? Doctor: Hepatomegaly with caudate lobe hypertrophy that may be because of excess alcohol consumption,and if continues to drink alcohol the liver may further compromise and land up to cirrhosis.And obviously there will be risk for his life once he is diagnosed as cirrhosis.You need to check for his liver function test ,prothrombin time with INR,and distension of abdomen is because of free fluid in abdomen (ascites),if so then he is already in decompensated stage.Get an gastroscopy to rule out esophageal varix.Hope this information help you thank you"
},
{
"id": 164342,
"tgt": "Can laxatives be given to a child for constipation?",
"src": "Patient: My 10 year old daugher is always constipated. When she does manage to open her bowels, usually every 4-5 days, the movements are very large and painful. She then complains of lover abdominal pain for several hours afterwards. I have tried to get her to take more fruit and plenty of fluids but this doesn t seem to make any difference. Is it safe to use a laxative/stool softener? Doctor: Hi, welcome to HCM. Can understand your concerns. Laxatives can given to a child only after examination by doctor. It should not be given without prescription. Take care."
},
{
"id": 96275,
"tgt": "grey stool in children, what can be the cause ?",
"src": "Patient: My daughter is 4 months old and recently passed a gray-stoned colored stool.what can be the cause ? Doctor: hi, if your child is on exclusive breast feeds, then u neednt worry. if it occurs more than once please take her to daughter & get her stools tested to look for some infection. otherwise make sure she feeds very very regularly."
},
{
"id": 198281,
"tgt": "Noticed a lump on the nipple with pus in it",
"src": "Patient: Hello, my boyfriend had a little lump under his nipple a few weeks ago. Last week, it grew even bigger to the point where it looked like he grew a left boob over night. He then compressed a hot towel on it to open the pores and puss started to come out and a lot of it. Now its back to just being a bump again but a hard bump. Doctor: Hi,From history it seems that he might be having ingrown hair follicle infection and now produced abscess.He might require one course of antibiotic medicine for 5 days to cure infection.Dress the wound with proper cleaning the pus.Apply antibiotic cream.Ok and take care."
},
{
"id": 32326,
"tgt": "Suggest treatment for swelling in cheek",
"src": "Patient: My father-in-law was here in america for 6 months and this is his 5th month. All of a sudden from yesterday his cheek was swollen and by reading the previous posts, i am sure it is abscess by listening all of his same symptoms like breaking teeth little by little from 6 months which he didnt told us till now. I do have enough amoxillin 500mg tabs with me. Can i continue them to him and wait till he goes back so that he can pull his teeth over there in india? Shall i add ibrufen to that too? how much mg of ibrufen should i give? Is it ok to travel in flight without pulling this teeth? Is it dangerous? It is too costly for me here which seems like i cant bear it as we had teeth treatments done for me and my husband also this month. His travel insurance people said after all deductables he give 80% of the bill which is again only till 100$. please help me with your valued suggestions... Thankyou MEDHELP!!!! my email is YYYY@YYYY Doctor: Hi,It seems that there might be having developing abscess at the root of broken tooth.You can give Amoxicillin 500 mg3 times a day for 5-7 days.This will clear infection but once pus is formed then it is to be drained.You can give Ibuprofen 400 mg or 600 mg twice a day as per severity of pain.Apply ice pack 2-3 times a day.Ok and take care."
},
{
"id": 200339,
"tgt": "What causes lack of erection except in the morning?",
"src": "Patient: Hi dr I m 34 married 4 yrs back. I dont get aroused my penis does not get eraction not once in a day except in the morning i.e. morning wood. Generally after 10 days I get the feel to have sex. Sex thoughts come into my mind but still my penis does not eract untill I start foreplay but as soon as my thoughts are diverted I lose eraction. I cant keep up my penis eract even for 1 min. I have to start sex right after I get eraction. I want to know whether I m suffering from ED or something else and what are the causes and cure for my problem. plz help me. Doctor: DearWe understand your concernsI went through your details. Please do understand that most of the erectile dysfunction problem are due to stress and anxiety and obsession. In your case too, I suspect the same because you do have erection in the morning. The details you gave in the query are not enough to diagnose your anxiety problem. You may consult a psychological counselor or sexologist who will advise you more on this aspect.If you require more of my help in this aspect, please use this URL. http://goo.gl/aYW2pR. Make sure that you include every minute details possible. Hope this answers your query. Available for further clarifications.Good luck."
},
{
"id": 3565,
"tgt": "Can pregnancy occur from unprotected sex?",
"src": "Patient: hi, i am 22 &; i havent gotten my period since november. I have irregular periods, i just started my birth control on may 13th. What are the chances of me getting pregnant this month cause i accidently had unprotected sex &; my doctor told me to use condoms the first month no matter what. Doctor: Hi,Welcome to Healthcare magic. I am Dr Ramadevi wani. I will be answering your concerns today. From your notes it appears that you started contraceptive pills without your periods. I hope that you have done pregnancy test and ruled out pregnancy before starting contraceptive pills.Contraceptive pills are effective in preventing pregnancy from day 8 of starting the pills when you start them at any time in a period cycle. So if you have had unprotected sex after a week of starting the pill, you are protected. The chances of pregnancy are almost nil.If you have had sex within a week of starting the pill, then there is a possibility of pregnancy. The chances of pregnancy are about 20% (one in five).I hope this is helpful."
},
{
"id": 114679,
"tgt": "What causes swelling and fluid discharge from skin when diagnosed with lymphoma?",
"src": "Patient: My husband has lymphoma and stage 3 kidney disease. His creatinine level has jumped from 2.21 to 2.97. His legs and hands are very swollen and he is secreting a yellow fluid through the skin on his legs mostly but also on his arms. How can I help him? Please advise me ASAP. I am so worried. Doctor: You need to take care of his kidney status. He might be requiring dialysis. There are various lymphoma with different types of treatment. So few drugs can still be given in low doses."
},
{
"id": 175571,
"tgt": "What does frequent urinating without pain in 7 month baby indicate?",
"src": "Patient: My 7 month old daughter urinating very frequently like every 15-20 minutes, In a day around 20-30 times. Night time while sleeping it is less may be 1-2 times only. most of the time, urinating little bit only not fully. i think there is no pain coz she wont cry during urinating. Doctor: please check her urine routine ,report if turns normal forget about it if any thing wrong then we will advise accordingly ok"
},
{
"id": 181988,
"tgt": "Suggest treatment for pleomorphic adenoma",
"src": "Patient: Hello Doctor,I am 34 years old and I am having a swelling at end of jaw and biopsy is done which says pleomorphic adenoma. I remember it was there atleast from 7-8 years when i was in 10th standard but the size was small and not noticeable. It start growing from 2 years when I had an road accident. It is my hubme request to kindly inform the possibility of medical treatment rather than surgery. Doctor: A growing pleomorphic adenoma sadly has to be operated upon for its removal completely . The sooner it is the better as a bigger size will require more surgical intervention. Take care ."
},
{
"id": 213762,
"tgt": "How can i improve mental strength ?",
"src": "Patient: i am having problem in quick dicision making in current situation, i give-up very easily in any situation and loose confidence , easly get dominated by second person, cant handle little complex situation, as a result i have poor perfomence in office, less importance in friends, Doctor: Dear Sachin,thanks for the query,Your problem is low self esteem.You need to trust your self first then only other will respect you.Take good sleep,avoid persons who always criticize you,take help of good friends who can help you at times.Read motivational books,do Yoga and meditation which will increase your confidence.Stop listing to people,only listen your inner voice.Try to avoid taking multiple issues at times.When you are thinking of giving up,just hold little longer things will start changing.All the best."
},
{
"id": 12113,
"tgt": "How can i get rid from dark patches on my nose ?",
"src": "Patient: I am Twenty three and i have dark patches on my nose around nose and on my cheeks .How can i get rid of them? Doctor: Hi! greetings from health care magic these dark patches are hyper pigmented patches, melasma. avoid sun heat. apply the Cassea sophera \u2013 Q (homeopathy) with ice ."
},
{
"id": 68478,
"tgt": "Suggest remedy for a hard, painless knot at the base of my thumb",
"src": "Patient: I have a very hard, painless knot on the base of my thumb where it meets my palm. It is about the size of a pea and it has gotten bigger since I first noticed it about 5 months ago. You can actually see it pushing out against the skin. Just wondering if it is anything to be concerned about. Doctor: Welcome to Health care magic.1.There is no remedy for this kind of lesions other than small incision and drainage the collection followed by a antibiotic treatment. If the lesion causing discomfort or pressure effect.2.The history suggest - its a ganglion cyst, pain less hard slow growing from the joint area.3.Other possible cause could be lipoma ( fat cells ) lesion.4.Its not a emergency or concerned issue - but if you have any discomfort see your doctor and get needful treatment done.5.MRI is good investigation for this - but depend if its easy to get it done in this small issue ( as it is helpful aim informing - joint tendon,nerve better assessed in MRI ) Good luck.Hope it helps you. Wish you a good health.Anything to ask ? do not hesitate. Thank you."
},
{
"id": 113194,
"tgt": "Constant back pain causing numbness in arms, leg pain. Back seemed to be curved at the top. Remedy?",
"src": "Patient: I have seen my doctor. I told her about my constant pain in my back. She checked my back and said it was curved at the top. I have pain that run s threw my arm s,numbing uncomfortable feeling,can t sleep when it happens. My right leg go s threw pain at the top. These thing s come and go. My Dr. gave me t3 s. They don t touch the all over body pain I feel all day,every day. I can t get out of bed. I will be in for an x-ray nexy month. Doctor: Hello. Thanks for writing to us. The symptoms that you are having are likely to be due to a nerve root compression or a pinched nerve at the level of cervical spine. An MRI scan of the area will help in the proper diagnosis and further definitive management. I hope this information has been both informative and helpful for you. You can consult me again directly through my profile URL http://bit.ly/Dr-Praveen-Tayal Regards, Dr. Praveen Tayal drtayal72@gmail.com"
},
{
"id": 99802,
"tgt": "How to get rid of itchy spots on legs?",
"src": "Patient: i have about 13 spots on my legs. they are big and have water coming out of them, they itch alot. i recently went to greece but the rash cam about a week later after i had slept in the same bed as my aunty who went to croatia and got a minor rash on her arms from there. my one is different as it will not go away and her one did. what can i do to make it go away? i did not get bitten in greece so it could not be that. Doctor: Hi, this kind of skin or may be surgical problem, doctor must have to examine with his naked eye on site.it may be due to allergy, insect bite,infection ..so many reasons either can be.so u first go to a dermatologist to see if there any skin problem or something else."
},
{
"id": 39784,
"tgt": "What does swollen lymph node with fever, chills and body ache indicate?",
"src": "Patient: I have had a swollen lymph node on my left temple for a few months. Within the past few days I have noticed that it continues to swell and get bigger. I am now experiencing headaches, being light headed, fever, chills, body aches, fatigue, and not feeling well after I eat. What is wrong with me? Doctor: Dear Friend.Welcome to HCM. I am Dr Anshul Varshney. I have read your query in detail. I understand your concern.Fever with Swollen lymph nodes can be a symptom of any of the following:1. Bacterial Infection2. Tuberculosis3. Lymphoma.I advise you to get following:1. Hemogram with ESR2. Xray Chest3. Blood picture4. FNAC of the lymh node.You should not wait, you should get investigated.This is my best advice for you based on available details. If you have any further query please ask me.Stay Healthy.Dr Anshul Varshney, MD"
},
{
"id": 131079,
"tgt": "Having noticed severe pain on left arm,elbow & shoulder",
"src": "Patient: Bicipital problem or shoulder problem? Last thursday went to gym did the 30 minute circuit, which exercises everything in 10 different machines. After doing the 3 leg machines which are the first 3 mschines, felt severe pain in right arm between above elbow and shoulder. Iced my arm all thursday night in bed. Friday when I went to work. Still had severe pain, hurt to even lift arm at any level. Saturday had mild-moderate pain in arm. Sunday and today barely any pain.. what would have caused this? Haven t been to gym since thursday. Should I have gotten it checked out? Who to see? What if I do nothing? Went to gym today, did biceps/triceps, no pain. Always wear a tennis elbow strap doing tricep exercises, otherwise pain in elbow joint. Currently no insurance. Doctor: Ok lets keep it simple and really specific for you.I dont like giving generic answers to my patients. so heres something to get your money's worth1) first are you relatively new to the 30min circuit or have you added seomthing in the regimen, if yes you have a DOMS ( delayed onset muscle soreness).Postexercise muscle soreness, also known as delayed-onset muscle soreness (DOMS), is defined as the sensation of discomfort or pain in the skeletal muscles following physical activity to which an individual is not accustomed to.The incidence of DOMS is difficult to calculate, because most people who experience it do not seek medical attention, instead accepting DOMS as a temporary discomfort. Every healthy adult most likely has developed DOMS on countless occasions, with the condition occurring regardless of the person's general fitness level. 2) then your next question would be, why does this happen?its caused by accumulation of waste by- products after muscle activity, these waste products can serve as a pain chemical.small tears in the muscle after an accustomed exercise is an additional factor3) doc what do i do?well DOMS just like in your case is a self limiting disease, meaning it wont get worse. it usually resolves in a week (max) so if the pain goes away on its own, best thing to do will be to stretch before you exercise, adjust the timer to 40mins initially, or switch to a more basic exercises for your arms first then switch gradually after a week to your preferred 30mins regimen. ( if you have a trainor now is the time to employ his/ her expertise)as of now no urgent need to see a doctor :) advanced merry christmas ( Dr. Charles, Orthopedic Surgeon)"
},
{
"id": 132171,
"tgt": "What remedies are suggested for swollen & bruised knee with difficulty in walking?",
"src": "Patient: I hit with a softball just below my knee 2 days ago and have iced it and taken ibuprofen throughout the day, my whole lower leg is swollen and firm, a large bruise is forming and sometimes it feels like someone is pinching me under my skin. It is painful to walk and bend my knee. Do I need to see a doc or should I wait and see how it progresses over the next few days? Doctor: Hi you have been hit by a softball below knee and are having large bruise and swelling.It is painful and walking and bending knee is difficult. To me it appears that you have probably torn some muscle like gastrocnemius and there is hematoma( blood clot fromation) with spread to Calf causing pain and disability. Take complete bed rest with legs elevated on two pillows , pain killers , creppe bandage for few days. If it does not subside then go for medical check up including Doppler and x rays."
},
{
"id": 92382,
"tgt": "Why there is pain in upper abdomen and sudden weight loss with all the blood tests normal?",
"src": "Patient: Hi, We are dealing with something of a medical mystery. The patient is 54 year old male. Has upper left abdominal pain for 4 months with 25 pound weight loss. Pain is chronic and localized. Unrelated to food intake or body position. No diarrhea, vomiting, nausea, reflux, or other GI changes. Negative Upper and Lower endoscopies, negative H pylori; negative EUS; negative MRI of pancreas. Negative Pill camera swallow. Normal bloodwork including liver and pancreatic enzymes with exception of borderline high sugar. Imaging all negative (CT, MRI, Ultrasound of celiac artery). Pain presented in days following a throwing injury that pulled oblique muscle on same side. Muscular pull healed within days. Negative Carnett's sign. MRI of Thoracic spine negative. Trial of Celebrex ineffective. Any guesses? Doctor: The pain can be due to fibrosis of the muscles after healing. On exploration they appear pale and hard to touch. Or can have a small hernia trapping the omental fat which is easily missed on USG and MRI unless specifically scanned for. The weight loss may not be explained by this. It can be due to high sugar, particularly the postprandial . OGTT (oral glucose tolerance test) and other investigations for blood sugar may help. May be the patient has a fear-factor or phobia causing this much weight loss."
},
{
"id": 54745,
"tgt": "What causes hepatitis-c and raised SGPT?",
"src": "Patient: HI Dr my father got hep c yr back then he took interferon he complete course for six months and when he test after treatment that was negative bt after four months he was not feeling well when he tested again and hep c is positive and his sgpt is 79 can u please tell me why it occur again?? and why sgpt is raised?? Doctor: Hi thanks for contacting healthcare magic.Noted he has chronic hepatitis c...In which chances of chronic hepatitis is very high as much as 80%....He might be in active stage so antiviral drug given....It suppress viral multiplication...But now you are hepatic c positive...So yes you are In chronic phase...Regular follow up done.Take more fruits .Use less oil while cooking for lunch and dinner....Protein diet taken in adequate amount...In morning take one tsp licorice with honey...Yoghurt , carrot , tomato good in your diet.Avoid alcohol...So your sgpt high because of liver affection as this enzyme present in liver hepatocytes.Take care."
},
{
"id": 70986,
"tgt": "Suggest treatment for constant post-nasal drip and cough",
"src": "Patient: I have a constant post nasal drip and a constant cough. I had allergy testing done 25 years ago and wasn't allergic to anything. Last year I had sinaplasty surgery and still have same problem. What type dr do I need to go see an allergy, ent or back to sinus dr. Doctor: Hello and Welcome to \u2018Ask A Doctor\u2019 service. I have reviewed your query and here is my advice. If you had a surgery & you have the same problems again means that your sinusitis did not respond to the surgery, else you could have got relieve. However, get re-examined & report your symptoms to the treating doctor, you need medications. You may have to get an nasal X-ray done to diagnose the problem. Chronic cough is an indicator of some developing lung dysfunction,bronchitis or COPD. Get sputum examination & other investigations done, get treated. Presently, take antihistamines & Cefuroxime & multivitamin supplements for relief. Hope I have answered your query. Let me know if I can assist you further."
},
{
"id": 143947,
"tgt": "Can I give tylenol to child suffering from concussion headache?",
"src": "Patient: I was research a prescription that a dr gave my 14 year old son for a cucussion headache I am am now concerned he should not be taking it after reading about online it is called Cambia and all the information I found says not for patients under 18 and it also states it s for migraines, I gave him a dose already and it didn t help him at all can I give him Tylenol Doctor: Hello,I can understand your concern. It is true that Cambia is given for migraine headaches, but do not worry about the one dose that you have given to your son as it is less likely to cause severe side effects with just that one dose.As your son is 14 years old, you can safely give him Acetaminophen 500 mg (Tylenol) or Ibuprofen 400 mg (Brufen) up to 3 times a day (8 hourly). This will give better relief in concussion related headache. If he does not feel any relief in pain in 3 days or if the pain worsens, you should consult a physician to examine the injured area.I hope this information helps. Thank you for choosing HealthcareMagic. I wish your son feels better soon.Best,Dr. Viraj Shah"
},
{
"id": 103649,
"tgt": "Swollen tongue with numbness at tip, blotchy inner lips. Do not have change in routine. Cause?",
"src": "Patient: My daughter is 23 and has been in Israel for 2months- she had an episode yesterday that caused her tongue to swell up and the insides of her lips to get blotchy. It was very scary and today she says the tip of her tongue is numb and the lips are still looking blotchy. She went to a doctor and he wrote a script for tests but they will run 1500 and the results will take 2 weeks. She is due home in 3 weeks. It sounds like allergy to me but she said she did not eat anything unusual. Any ideas? Doctor: Hello,By the very history it looks like angioneurotic edema which a form of allergic reaction.This type of allergic reaction is a cause for concern.Most of the times this type of reaction is related to food allergy.It is strongly advised to avoid any allergen which is suspected to cause the allergy.Due to edema of the tongue it may fall back and obstruct the airway.Keep her under constant observation.Sleep on her sides to avoid tongue falling back and obstructing the airway."
},
{
"id": 108974,
"tgt": "What causes severe back pain?",
"src": "Patient: hi i had unprotected sex on the 8th and 9th of december , i ov for 7 days i have felt funny ill for weeks!!! on the 28th i had bleed for day and half , one week before i should have !!! my back kills me!!! now i have clear watery discharge!! AM I PREG , i have taken 2 test a week ago neg Doctor: Dear patient Since you have got bleed and your pregnancy test is negative you are not pregnant. Having unprotected sex and that too ofter puts you at risk of sexually transmitted infection. Since you have got watery discharge and severe back pain that might be due to pelvic inflammatory disease , a condition due to spread of genital infection to pelvic organs. Please do not neglect and take urgent visit to gynecologist and get yourself examined. Never ever have unprotected sex bcoz it puts you at risk for STD And HIV. Start taking tab metronidazole thrice a day. Take tab ciprofloxacin 500mg twice a day both for 10 days. Take tab diclofenac plus thiocolchicoside combination twice a day for back pain relief. You will be relieved."
},
{
"id": 195178,
"tgt": "What causes warm sensation in the penis?",
"src": "Patient: My husband has a warm sensation when peeing and now says that his penis feels hot. Went to urgent care that said the sugar in his urine was high especially since he had eaten 4 hours earlier. Confident no STD, but not given any ideas. I know he is in significant discomfort. Doctor: Hi, Discomfort and warm sensation in the penis may suggestive of urinary tract infection and the possibility of mild ulcers in the tip of the penis. Since sugar in urine may indicate diabetes. kindly work it out for diabetes. in a diabetic patient, urinary tract infection and penile ulceration and pain are the commonest symptoms. Hope I have answered your query. Let me know if I can assist you further."
},
{
"id": 197005,
"tgt": "Suggest treatment to increase sperm count",
"src": "Patient: i am aravind i got married three years ago i participate sex with my wife every day but when ever i have sex the semen comes low i got tested semen analysis count was 2 millions only please help me that how can i increase my sperm count. i eat all but when ever i have sex with wife the semen comes at 1 hour Doctor: medications rarely increase the sperm count.. you should consider having an IVF cycle if you seek having childern.. also you should consider freezing a semen sample in case your analysis got worse"
},
{
"id": 184668,
"tgt": "What could nosebleed after undergoing root canal treatment indicate?",
"src": "Patient: i have never had a nosebleed before until today. I am 42 and had a root canal on the tooth next to my front tooth. I was told the root was calcified due to trauma to the tooth which happened in a car wreck 18 years ago. Do you think the nosebleed is related? Should call the endodontist? Doctor: Hi, Thank you for posting your query. I had gone through your problem. Nose bleeding is not seen after doing root canal therapy in relation to the tooth which is next to the front tooth. Because it is no way related to the maxillary sinus. So i would like to suggest you to consult a general surgeon who will give you better treatment for your problem. Hope my answer satisfy your query."
},
{
"id": 125524,
"tgt": "What causes stiff neck?",
"src": "Patient: Hello Doctor. I am a 42 years old male. Since some time, I am facing a peculiar problem. After some time of using my laptop I feel very stiff neck. Now it remains for so many hours and recently I feel that this stiffness is existing every time. Can you suggest the reason and what should I do to correct it pls? Doctor: Hello, As a first line management you can take analgesics like paracetamol or aceclofenac for pain relief. If symptoms persists you can consult an orthopaedician and plan for an MRI scan to rule out possibilities like cervical spondylitis. Hope I have answered your query. Let me know if I can assist you further. Regards, Dr. Shinas Hussain, General & Family Physician"
},
{
"id": 38193,
"tgt": "Is anti rabies vaccination needed after being bitten by a house dog?",
"src": "Patient: Hi, I have a 3 month old labrador pup who was vacinnated with anti rabies (first shot given on 15th May 2010 and second yesterday i.e. on 23rd June 2010). It playfully bit my thumb while playing with me and there was a incision and and some blood came out. I washed my wound under running water and also washed it with detergent soap and later with dettol. The bleeding stopped after some time. The pup remains within the house and has no exposure to other dogs. Do I need to take any antirabies vaccinne.Please advise Doctor: Thanks for contacting HCM with your medical question.Since this is your own animal and you do know for sure that it was vaccinated then the chance of rabies is very low if not non-existent. For a dog to contract rabies it must be out doors and then bite or eat an animal that has rabies. Since this is a pup I doubt it has been exposed to rabies. I would not be concerned about rabies and you will not need vaccination.Hope I answered your questions. Please contact us again with your health related questions"
},
{
"id": 2429,
"tgt": "Suggest precautions to be taken to conceive after fetal death due to increased uteroplacental and fetoplacental flow resistance",
"src": "Patient: hi dr. i am priyanka sharma from india..it has been since 6 months back i had my last pregnancy and now we are planning a baby.i am 25 years old and i had experienced 1 fetal death,due to mildly increased uteroplacental and fetoplacental flow resistances with mild fetal ascites.what should i do now.... Doctor: Before getting pregnant start taking folic acid.. And as soon as you get pregnant visit gynecologist.. Get usg done and when cardiac activity is present start taking ecosprin and progesterone."
},
{
"id": 7476,
"tgt": "Pimples and dark spots on face. Using exerge tablets, clear gel cream. Effective?",
"src": "Patient: gud mrng... my name is prathyusha, i am 20 years old. i have pimples and dark spots on my face. i am using exerge tablets and clear gel cream in the morning after breakfast and roxid tablets and tretin cream after dinner in the night. will it be helpful in getting rid of my acne? how long should i continue using these tablets and cream? Doctor: Hi, I think roxid is not the drug of choice go for Minoz OD once a day and i am sure in 2 months it would be great, the creams can be continued. take care"
},
{
"id": 103757,
"tgt": "Persisting skin rashes on the leg. Allergy?",
"src": "Patient: Hello Stephen, I have rush on my legs. I cannot identify the source. It is not a permanent rush. I have my own guessing about my allergic reaction . I do not want to influence any diagnostic. I do believe your question will lead to a reliable answer. My English is not very good, so please, accept my apologies for spelling. Best regards. Vasile Galan Doctor: CAN BE DUE TO SOAP OILS BODY APPLICATION MATERIAL HENNA DYE MEDICINES LIKE PCM BRUFEN CITAMINS MULTIVITAMINS DIET SUPPLIMENTS CALCLCIU, OT MULYIVITAMINSCAN BE FOOD LIKE PAPYA BANANA RICE EGG NUTSYOU CAN THINKAPLY CALAMINE LOCALLYNO OILY GREESY FAT FOODLOT OF WARM WATER"
},
{
"id": 218908,
"tgt": "Why is Cremaffin emulsion advisable during pregnancy?",
"src": "Patient: Hi, 6 to 7 days before delivery doctor has prescribed Cremaffin emulsion to initiate the delivery process. My Wife is not suffering from any complication related to during that period and her health is normal, so I advise her to NOT administered Cremaffin emulsion (She not administered single dose of it).1) As per data in https://glowpink.com/meds/cremaffin/ it leads to fetal injury and also mentioned on few website contradictory medicine in pregnancy. Please advise its safe or not? If the pregnant lady is normal (means she is not suffering from any constipation like issue) then why doctor prescribed it 2) As per doctor cremaffin used to initiate delivery process, then why it\u2019s not mentioned in USES of that drug or in indication? Doctor: in my opinion..I never prescribe cremaffin emulsion but yeah some old docs do describe castor oil or cremaffin...if u have nt taken yet..don't take it...ask ur doc to use alternative mode of induction lik cerviprim gel"
},
{
"id": 38101,
"tgt": "Will my UTI based on urine culture get cured on its own?",
"src": "Patient: I have had numerous UTI S and I requested a copy of my last lab report, 1) for vaginal culture, and 1) lfro urine culture, final report on the urine culture was Morganella MorganII, b colony count 30,000, but no antibiotic was prescribed. Can I get over this on my own? Doctor: Hello, Thnx to contact us. I understand your concern. If I am your treating doctor I advice you that urine culture with colony count of less than one lac is not UTI. It wont requires a treatment. Morganella morganii many a time isolate as a saprophytic organisms or Laboratory contaminant . So if the colony count is less, it is not UTI and treatment is not required. I will be happy to answer more of your concerns, kindly know me,Wish you a very good health at health care magic. Dr. Arun Tank. Infectious Disease."
},
{
"id": 25506,
"tgt": "How to reduce BP without medicines?",
"src": "Patient: Hi I'm 31 years of age and due to have surgery in 2 weeks time. My last blood pressure reading was 140/101 but prior to this it was averaging 121/76. I do suffer with anxiety and was wondering if you had any useful tips on how to bring my bp down without medication. Doctor: Hello 1) try maintaining blood pressure upper under 135 and lower under 85mm hg with lifestyle and medication as required 2) if overweight, losing weight should be a priority, try achieving bmi less than 25.3) 40-45 mins of aerobic exercise daily4) be regular with your life , eat and sleep at proper times5) in diet -low salt. Max 3 gms a day. - avoid preserved food as all preservatives have salt. - no pickles, papads, chips, farlam as all have hill salt and. - atleast 2 serving of fresh fruits in a day. - 1 serving 35-40 gms of nuts like walnuts, almond to be included - if you take fish 2 serving of fatty fish per will like salmon. -avoid red meat, egg yellow, fried and spicy oily food - may shift to better oils like olive in cooking 6) may consider supplements, like cod fish oil capsules, flax seeds, chia seeds, krill oil as they have good content of omega 3 fatty acid. We generally know everything, it's just following which is important. Lastly if there is any stress event going on in life, dealing with it in a mature way so that it doesn't affect your health is important. Regards Dr. Priyank Mody"
},
{
"id": 68818,
"tgt": "What causes knot between shoulder and neck in child?",
"src": "Patient: My 3yr old has complained about her shoulder and neck hurting when she woke up a few days ago. At first I thought she slept on it wrong and she wouldn't let me touch it. Last night after she fell asleep I felt a knot halfway between her shoulder and neck. Could this just be a pulled muscle and how could she do it in her sleep. Doctor: welcome to Health care magic.1.The location and consistency you have explained looks like brachial cleft cyst.2.Which is generally seen in that location and age.3.Lymph nodal mass in case of any infection / inflammation / systemic disease.4.However better to see your doctor and get an ultrasound scan done, which will help in better understanding.Hope it helps you. Wish you a good health.Anything to ask ? do not hesitate. Thank you."
},
{
"id": 132314,
"tgt": "What causes excruciating pain in the hip, groin spreading into the knee?",
"src": "Patient: Agonising pain in right hip. Pain in groin which goes down front of thigh and into knee. Very painful in bed. On Zapain tablets which do not help much. Been like this for a month. Dr. told me to rest and come back in a fortnight. Have disabled partner and live in a first floor flat. thank you JL Doctor: HiHope this msg finds you in good health.I have gone thru ur msg & understand ur concern. it may be due to disc prolapse causing radiating pain. get mri done. compression bandage will help Feel free to ask me a FOLLOW UP QUERY anytime.Take care.God bless."
},
{
"id": 9001,
"tgt": "How to remove dull complexion ?",
"src": "Patient: hi. i have a bigest problm. my complexion is very dull nd eyes around dark cyrcle plz help me . nd give me a some medicine or tablet or cream plz help me and i am very thank full to you Doctor: hi u have dull skin means u have sensitive skin. u need to take care of ur skin daily use good moisturizer twice a day use sunscreen twice a day avoid direct sun exposure drink more water u can apply NILTAN cream at night do exercise and eat healthy diet"
},
{
"id": 10129,
"tgt": "How should seborrheic dermatitis with hair loss be treated?",
"src": "Patient: Hi Doctor. Iam Afsal from kerala. Iam suffering from saboric dermatitis from last few years..I cant find any effective treatment for this disaese.it causing very high hair loss. Please suggest the effective medicine and food for this treatment Doctor: Hello and Welcome to \u2018Ask A Doctor\u2019 service. I have reviewed your query and here is my advice. I would recommend you to apply Ketoconazole shampoo twice weekly and apply lotion containing Beclomethasone and Clotrimazole on the affected areas twice daily. Also take capsule Itraconazole twice daily for 3 weeks. Hope I have answered your query. Let me know if I can assist you further."
},
{
"id": 221343,
"tgt": "Is delayed menstruation while on Petogen a sign of pregnancy?",
"src": "Patient: i was using pertogn since this year my retuning date was 8october,last month so i didnt retun after 2weeks i start feeling the is something in my stomach and is moving ,so i took 2 pregnecy test they come negetive and now movement a strong than before ,so i was asking that it is posoble that it is a pregnecy\uff1f Doctor: Hi, I understand your query & concern. Pitogen is an injectable progesterone.. it's a contraceptive which offers contraception for 3 months after injection. It has a known side effect of delayed/ no/ irregular bleeding. So delayed period while on Pitogen is not a sign of pregnancy. thanks."
},
{
"id": 43371,
"tgt": "How to know the problem for not having baby?",
"src": "Patient: hello docgood day i just want to ask if where i can consult the doctor related about my pregnancy we have a problem with my husband 3 years allready we been together until now we dont have a baby.,, i dot know if i have a problems or my husband what is the best way to know that if we have a problem or not?... best regards Mrs.Catherine Pajado Doctor: Hi and thanks for the query,The first thing to do is actually making a proper diagnosis of infertility. You have to be living with your husband for at least one year, having unprotected sex at least thrice to week. It s important.The cause of infertility could be from the female, male or at times unexplained. The first consultation usually should be with you and your husband for a initial battery of tests to be done. These usually range from screening of sexually transmitted infections, hormonal abnormalities, malformations, spermograms an d temperature monitoring in the female to ascertain effectiveness of ovulation.I strongly suggest you and your husband consult an infertility specialist/experienced gynecologist.Thanks and hope this helps,Bain LE, MD."
},
{
"id": 15310,
"tgt": "Fast spreading itchy reddish rashes on the arms, armpits and inner thighs. Will it become chronic? What type of treatment is needed?",
"src": "Patient: Hi I m a 28 year old female and int he last 2-3 days I ve grown a symmetrical fast spreading red itchy sometimes burning rash that is a little on my arms but mostly on my inner thighs, right behind my armpits (technically on my back), my lower back, and butt. The soonest doctor appointment is in 2 more days and I want any information I can get in the meantime about what this rash could be, if it is likely to be dangerous, if i can lessen it in any way, if it will spread even more, if it seems like it will be chronic/how long will this take to go away, and what type of treatment is involved? Doctor: Hello,Welcome to healthcare magic,From your symptoms and the distribution of the rash it is more likely to be a fungal infection (Tinea). You may apply an over the counter antifungal cream like clotrimazole 1% cream twice a day. It is unlikely to be anything dangerous or chronic but without an opportunity to closely examine the lesion, I would not be able to say for sure.Hope this helped,Take care"
},
{
"id": 216855,
"tgt": "What causes body ache and headache?",
"src": "Patient: My wife had a stomach ache and the followed by Body ache, Headache and fever. She is also iron deficient and don t like eating healthy food at all. Can you please suggest me some medicine ? Also, her period was due on 25th, but it s getting delayed and she is still waiting for the period to start. She had taken i-pill on 8th of this month. Doctor: Hi Dear,.Understanding your concern. As per your query your wife have symptoms of body ache and headache along with stomach ache as well which could be due to impaired immunity of your body and due to viral infection of body. Need not to worry. I would suggest you to boost your immunity by taking multivitamins and diet rich in vitamin A. You should take NSAID medication along with antiviral medications such as Acyclovir. You should take soft and bland food. You should take fresh juices in diet. Try to divide your meals in small fractions rather than having bulk at a time. Visit general physician once and get it examined. Go for blood tests as well. Start treatment after proper diagnosis. You should take plenty of fluids. Avoid taking any sharp and oily food substances.Hope your concern has been resolved.Get Well Soon.Best Wishes,Dr. Harry Maheshwari"
},
{
"id": 68808,
"tgt": "Suggest treatment for cyst in breast",
"src": "Patient: hi, i am 19 years old and have had a cyst in my breast for around 2 years, i had a biopsy a little over a year ago and results showed it was just a cyst. the cyst grows and gardens during periods so i have decided to have it removed. i forgot to ask my doctor who i was examined by at the hospital today if i would have it removed under local or general anesthetic, do you know? Or can i request general anesthetic to be used? thanks. Doctor: welcome to Health care magic.1.If it is proven cyst, and not causing any discomfort - you can leave it alone, may resolve by time.2.If as you said causing discomfort during periods - better get rid of it.3.It is generally done under ultrasound guided needle aspiration with no anaesthesia or local. General is not at all recommended. 4.Hope i have answered your questions.Anything to ask ? do not hesitate. Thank you."
},
{
"id": 99685,
"tgt": "How to treat pressure in head and loss of appetite?",
"src": "Patient: For the last couple days ive been really cold, body aches, headache, no appetite. Third day no body aches, don t feel cold, still no appetite and if I move my head to quickly I feel like my head is going to burst or I m going to pass out. I ve been taking allergy meds and naproxen. Doctor: HI, thanks for using healthcare magicBased on the symptoms described, it is possible that you have an acute viral infection such as the flu or cold.This can cause the symptoms that you are describing including the dizziness or abnormal sensation in your head.You can continue the allergy medication and pain medication. You should also rest and drink enough fluids.I hope this helps"
},
{
"id": 42042,
"tgt": "What problem does the semen analysis indicate?",
"src": "Patient: dear sir i have done semen analysis in 6 days of abstinent.motility report is as followes : 40% active motility , 35% sluggish motility ,25% non motility, morphology, normal forms-70% obnormal forms-30% pus cells 2-3 /hpf. What each indicates and what is the problem in my sperm Doctor: HelloAfter going through these reports I want to tell you that Motility should be 100 % while in your case it is 40 % ( active ) . Morphology is 70% , abnormal cells 30 % , for normal fertility It should be > 13 % of exhibiting normal morphology.Now read this carefully , as this may help you.Extensive studies suggests that normal FERTILITY is associated with sperm counts > 48 million/ ml with a motility of >63% ( in your case it is 40 % , much less )with 13% exhibiting normal morphology.But subfertility is associated with sperm counts As far as about conception you should consult an INFERTILITY specialist and get his opinion .Good luck."
},
{
"id": 96779,
"tgt": "Do I need TIG injection after anesthetic injection?",
"src": "Patient: Hi!! I m Delwar from Bangladesh. because decay a teeth,day before yesterday i have extracted the teeth by a Dentist(Assistant professor). he was push Anesthetic injection and after extract the teeth. my question is due to push Anesthetic injection, may i need to push any TIG injection for this? my dentist told me not require TIG. what is ur suggestion about TIG??? Doctor: Hi dear , I can completely understand your concern regarding your health issues .The need for active immunization (tetanus toxoid), with or without passive immunization (tetanus immune globulin [TIG]), depends on the condition of the wound and the patient\u2019simmunization history .In my opinion ,if you have 3 or more tetanus doses in past and wound is clean n minor ,as in your case it is .i don't think you need TT or TIG .please rate my answer if you found it helpful .regardsDr.harpreet Kaur"
},
{
"id": 108077,
"tgt": "Suggest remedy for stomach pains with severe back pains and nausea",
"src": "Patient: Hi well Im an 18 year old female n I lost my virginity October 2. An the very next day my period came on ten days earlier than normal n it only lasted four days then on November 1 it came on again for three days and me and my husband had sex on november 11 and 23. And then my period started again three days later and only lasted two and a half days. Throughtout October and November I've had stomach pains. Severe back pains. I've been sick. Throwing up. Nausea. Dizzy and confused. And all I have been doin is sleeping but yet I'm always so tired. I took some pregnancy test buy all of them said negative so what could it be??? Doctor: no need to worry if still pain take some pain killers..and there might be infection.it Can also cause lower abdominal pain.regarding multiple mensturation..it might be due to homonal. causen.just have rest and take medicines. if not then nneed to.go hospital to see doctors.will do some tests"
},
{
"id": 146700,
"tgt": "Does a pineal gland cyst cause superficial numbness and dizziness?",
"src": "Patient: Hello. I am 43 years old and 130 lbs, 5'7\". Two months ago I was diagnosed with 15.8 X 12 X 9.7 mm pineal gland cyst. I have constant low-grade headache, extreme eye sensitivity to light, constant dizziness, and mental fogginess. More irritating to me, however, is that I have superficial numbness in all four extremities, legs much worse than arms, a deep constant feeling that my legs are falling asleep, pain in legs, and a feeling of needing to move my legs constantly, resulting in twisting movements of ankles while awake. The numbness started in my left lower extremity and has progressively worsened and spread upward over the past two months. Additionally muscles feel sore and like they have been strained while at rest. Pain is unrelenting. Please help if you can. Thank-you. Doctor: Pineal gland cyst don't cause such symptoms. Probably you are having Restless leg syndrome. Try tab Pregabalin 75 mg at night and also do Serum ferritin and TIBC. Hope my answer will help you. Take care.dont forget to rate me."
},
{
"id": 73954,
"tgt": "What is the treatment for chest pain?",
"src": "Patient: Hi, may I answer your health queries right now ? Please type your query here...hi my husband is a dialysis patience and he has mild chest pain in the middl e of his chest.... what causes that and he did hes treatment today 4hrs and 15 mins of treatment Doctor: Thanks for your question on Healthcare Magic.I can understand your concern.Dialysis patients are always at risk of heart diseases.So we should first rule out heart diseases for his chest pain.So get done ecg and 2d echo.If both these these are normal then no need to worry for heart diseases.Sometimes gastritis can also cause similar kind of chest pain. So gives him combination of pantoprazole and levosulperide twice daily before food.Don't worry, he will be alright but first rule out heart diseases.Hope I have solved your query. I will be happy to help you further. Wishing good health to your husband. Thanks."
},
{
"id": 189076,
"tgt": "Gum swelling, painful lower molar. Neoclav taken, ospamox, suprazole, ibuprofen prescribed. Advise?",
"src": "Patient: Hi. Im Stephen from Kuwait. i just started taking Neoclav 625mg tablets about a 3rd time since yesterday to treat mg swelling gum and painful lower molar. Today i just came from the Ministry of Health, after the check up from the dentist. They prescribe me 3 different tablets one including Ospamox (Sandoz) 500mg. Suprazole 200mg and Iprofen 400mg. Ospamox was prescribe to me the same as Neoclav, 1tablet every 8hrs 5days duration and the other 2 medicines Suprazole and Iprofen 1 tablet 3xday for 5days. my question is do i have to continue taking Neoclav instead of Ospamox? Thank you Doctor: Hi,Thanks for asking the query,Tooth pain along with swelling indicates infection of tooth that had spread to the surrounding periapical tissues.I would like to tell you that antibiotics and analgesics will provide you temporary relief, i would suggest you to get an x-ray done and start with root canal treatment of the tooth.Unless the tooth is devitalised, disinfected and sealed to the apex you will face pain , and swelling.Maintain a good oral hygiene, use lukewarm saline and antiseptic mouthwash rinses.Hope this helps out.Regards.."
},
{
"id": 7697,
"tgt": "Recurring acne, taken isotane, azento, benzac ac 2.5%gel, clinmiskin, using cetaphil face wash",
"src": "Patient: I had bad acne 5 yrs back..I had a dose of isotane. I recovered and then had a mild acne breakout last yr..again low dose of isotane for 3 months. I was clear till february.I then threaded my face which caused me to break out along the sides of m,y face. I was prescribed azento for a month and application of benzac ac 2.5%gel and clinmiskin at night and morning. I wash my face with cetaphil . I am getting married in less than a month and I have 10 odd acnes on my face. It is scary .I am 25 yrs old Doctor: hello, thanks for using healthcare magic. i will be glad to be of assistance i can understand your concern. acne is seen in individuals from teenage years till the age of 25 years. so do not worry , i will recommend a good routine. hope you have not been picking up on any of the lesions as that will lead to deep scars and pigmentation. you need to follow a routine of cleansing, sunscreen , creams daily. cleanse your face with cetaphil cleansing lotion for acne prone skin thrice daily. sunscreen needs to be used daily , neutrogena would be a good choice a course of tab. doxycycline 100 mg once a day for a month will take care of the remaining acne. also taking saliclylic peels from your dermatologist once in 2 weeks will also help. in case you are short of time, you can use retinoic acid cream 0.025% and glycolic acid cream 6%on alternate nights on full face for 2 hours every night. and in the morning , application of benzac ac and clinmiskin should be continued. and using azelaic acid cream 10% for 2 hours every evening will also be beneficial. please let me know your queries. regards"
},
{
"id": 92237,
"tgt": "Ultrasound finding shows Hepatomegaly and acute appendicitis. Appendix removed but how to treat enlarged liver?",
"src": "Patient: My wife had a pain in right lower quadrant from past 10-15 dasy. Doc prescibed for 3 D ultrasound. Two findings came out one HEPATOMEGALY and ACUTE APPENDICITIS. Four days back appendix was removed. But her liver measures is 189mm which is worry for me. Kindly prescribe what to do. Doctor: Hello friend,Your query makes me feel that you would have removed your wife's liver also if it was possible just as you removed her appendix. Behind every symptom, whether it is pain or swelling, there is some language of nature that warns you that something is wrong with her. If a burglar's alarm is ringing, it means a thief has gone into your house and you have to find him out to rescue your valuable properties. Instead of finding out the thief, if you destroy the alarm because it is a nuisance for you, how can you qualify your action? Now you have removed the appendix because it was giving her pain. You are asking what to do next. What I mean is not the diagnosis of the disease but the diagnosis of the patient is more important. What is her genetics? What is her constitution? What is her life situations? What excited her to become sick 2 weeks back?As a Homoeopath, my approach in your wife's condition will be like this : I will take her case in detail. I will try to understand her genetic constitutional remedy and if taken by her, that will cure her. Once she is cured, her liver also will become less and less swollen and sick.You can get back to me through the direct question facility of Healthcare magic.With best wishes, Dr C. J. VargheseHomoeopath"
},
{
"id": 156097,
"tgt": "Suggest chances of survival with stomach cancer",
"src": "Patient: My Mum has been diagnosed with stomach cancer. She is 85 and it seems like it has not spread anywhere else. The doctors have told us that the treatment would be too harsh for her as she is quite frail. How long can one live with that kind of diagnoses. Thank you (Ireland) Doctor: Hi welcome to HCMI have gone through your query and found that your 85 years old mother is caught with stomach cancer . In a way ,doctors are right . Instead of giving her any harsh treatment ,give her some Alternative therapies . I would like to know since how long she is suffering ?What all the symptoms of her sufferings ? Is she active Any vomiting ? Colour ? Has no constipation ?other things to be mentioned ?Can be administered proper remedy after knowing the details .Meanwhile give her CHomeopathy is one them ,tried and tested , successfully ,in some cases G Meanwhile give her Carcinocin 200 weekly twice 1 dose . Then fortnightly thereafter . Keep vigil on her diet .Give her simple , nourishing diet containing fiber vitamins ,minerals antioxidant diet suitable to her age .You can check from treating doctor ,or a dietitian .Supplements which have no side effects can be administered to her which are rich in antioxidant properties .Lemon has strong anti-carcinogenic properties that are already been proven. Mixture, of extract of Lemon ,Ginger,Garlic , Vinegar of coconut water ,Bark of Moringa ( sahajana ) tree , Honey all in equal quantity in a dose of 15 ml , twice a day . with same quantity of water, before meals for 31 or 41 days as per requirement .a great antioxidant . Is good healer and also preventive . Take a 1/4 t spoon of turmeric powder twice a day , in a cup of hot milk after meals , Meals should be simple nourishing and easily digestible .Take care not to have constipation . Do pranayam - Deep breathing slowly , according to her age & capacity & Walk for resistance against diseases and to strengthen the vital part of the body . Many heart and cancer patients have been benefited from this .You Can also apply Treatment by magneto therapy , is very simple and effective treatment gives resistance against diseases ,is curative as well as preventive method of treatment ,having no side effects Regular fort nightly or monthly check up from treating doctor is highly appreciable ..Hope this gives reply to your query Take care .All the best & get well soonDon't hesitate to get back if have any further queryIf any doubt get back at drsuchda@gmail.com"
},
{
"id": 79374,
"tgt": "What causes weird bubbling in chest?",
"src": "Patient: Hello. So today I woke up and was perfectly fine and I have anxiety. I always feel sick and have a little bit of acid reflux. After I went garage saling I started feeling a weird bubbling in my chest. It got worse and worse and when I walk or use my muscles it gets worse Doctor: Thanks for your question on Health Care Magic. I can understand your situation and problem. By your history and description, possibility of gaseous sound due to GERD (gastroesophageal reflux disease) is more. Stress and anxiety are common cause for GERD. So avoid stress and tension, be relax and calm. Start proton pump inhibitors and prokinetic drugs. It will help in gaseous trouble. Avoid hot and spicy food. Avoid junk food. Avoid large meals, instead take frequent small meals. Avoid smoking and alcohol if you have these habits. Loose weight if you are obese. Don't worry, you will be alright. Hope I have solved your query. Wish you good health. Thanks."
},
{
"id": 197080,
"tgt": "Suggest treatment for psoriasis on penis",
"src": "Patient: My boyfriend has been applying anti-creams to his penis because he has psoriasis and recently I had a miscarrage, would the itch cream that he has been applying been the cause of what happened to me. Do the anti-cream cause serious and or even further damage. Doctor: HelloThanks for query .Miscarriage that you had can not be due to cream that your boyfriend is using fas a part of his treatment for Psoriasis .Miscarriage is the result of failure of products of conception to grow further and is mostly due to hormonal imbalance or deficiency of HCG ,You need to consult your Gynecologist immediately after you get pregnant next time for hormonal assessment and treatment to continue pregnancy .Dr.Patil."
},
{
"id": 6770,
"tgt": "Is it possible to get pregnant by rubbing the organs ?",
"src": "Patient: hello doctor We are newly married couple, me aged 23 n she 21, on aug2 she gave me job on my organ me, we dint have intercourse, due to this she had my fluid on her hand. then we stopped. she washed her hand with liquid soap handwash in bathroom.. nearly 45min after she washing her hands she rubbed her same hand on her organs as she was expecting her periods. on aug 5 she had her normal periods. now our concern is due to her rubbing her organs is there chance of she getting preganant ? actually we dont want to have children at this point of time now she had her normal periods, is there chance hat she gets preganant next month for same act ?? if yes then how to avoid it ? Doctor: no its not possible to get pregnannt just by rubbing the organs....penetrations is a must thing to become pregnant............"
},
{
"id": 29119,
"tgt": "What causes itchy red spots all over the legs along with earache and tonsillitis?",
"src": "Patient: Hi there Doc...I wonder if you can please assist me... I started getting ill last week first it looked like a cold or something in my eye then I started getting ear ace and now sitting with tonsillitis and I have been getting red dots on my legs and feet... can measles start on you legs and feet? I have spots all over my legs and feet very itchy or can it just be a rash? Doctor: Hello dear. Thank you for using HCM. I read your query and understand your concern. I looks you are suffering from a viral infection , Hand-foot-and-mouth disease a mild, contagious viral infection common in young children is characterized by sores in the mouth and a rash on the hands and feet. Hand-foot-and-mouth disease is most commonly caused by a coxsackievirus.There's no specific treatment for hand-foot-and-mouth disease. Frequent hand-washing and avoiding close contact with people who are infected with hand-foot-and-mouth disease may help reduce your risk of infection. Measles rush comes from head to feet and goes away from feet to head. so is not it. Best regards."
},
{
"id": 4851,
"tgt": "Had intercourse. Planning to have baby. When will she be ovulating?",
"src": "Patient: We are planning to have a baby now from the past 4 months. We have had intercourse on the right days, we both are healthy but still my wife could not concieve, she is 28 years old and myself is 35 years old now. Her periods are regular with a cycle of 28 days. but somtimes its like 2 days earlier or 3 days late. She had her period on the 18th of this month and will end on 22nd... please advice us what to be done & when are the perfect days for sex, so that she can get pregnant. Thank you very much Doctor. Doctor: Hi,Go for ovulation time testing for your wife.After ovulation or just ovulation took place, go for intercourse daily for 3-4 days.Do not allow her go to toilet for about one hour to prevent semen coming out.Ask her to touch her legs to her chest to prevent semen coming out and this position helps sperms migrating into her vagina and cervical canal and in the uterus.Try for few months.If there is no pregnancy then go for your wife for investigations for having any problem.at the same time go for your semen examination as well.Ok and take care."
},
{
"id": 190821,
"tgt": "Why does the puss pocket on the tooth treated with root canal hurt ?",
"src": "Patient: I had a root canal done on my front tooth about 5 years ago. It has an enamel cap on it. It has 3 puss pockets over it and hurts. What could be wrong with it? I am 73 years old, 6 3 tall. I am currently taking an antibiotic. Bill Doctor: Hi, It looks like there is some infection in the particular area. You have to consult your dentist to identify the reason of infection. Infection could be of the tooth or gums.Accordingly treatment can be obtained."
},
{
"id": 148095,
"tgt": "What causes spinning and shock waves in my brain?",
"src": "Patient: When I lay down I get really dizzy,it feel s like I am spinning..I have been getting shock waves in my brain,just the top though,They make me sit straight up.The spinning started out to be just every now and then and now it s every day..I have had a cat scan of my brain and ekg s and such and they say they see nothing.My ears are fine..It s really scary,I am on 320 mg of Diovan and 100 synthroid..I have PTSD,Anxiety,Major Depressive Disorder,COPD,Ulcerative Colitis and kidney problems,none of which I take medicine for..Any clue of what s going on here? I just turned 45 in case age is a factor..Thank s,Melissa. Doctor: Hello, Welcome Thanks for consulting HCM, I have gone through your query, you are feeling dizziness , brain shocks waves, dont take stress, first of al, stress creates many problems, Dizzeness, can be due stress related disorders, and can be due to variations in level of T3 T4 Tsh also You should consult physician for that. Hope this will help you."
},
{
"id": 12126,
"tgt": "What should i do for black tiny spots on cheek bones ?",
"src": "Patient: Doctor I have recently started seeing the black tiny spots on cheek bones. I never had any. Derm said its frakles and can go by radiofrequency. Is it true? How painful is the treatment? 1 She has prescribed with BAN A TAN creame to use at night 2 medicated sunscreen . I can t see much difference , what should I do to combat these,it really bothers me all the time. Please Help.I heard hydroquine is cancerous and not be used. I am highly allergic to dust. Doctor: hi u need to go for radiofrequency or co2 laser no other medicated cream is effective continue with sun screen avoid direct and indirect sun exposure it has property to reoccur its not much painful,if u feel more pain then topical anesthesia can be used."
},
{
"id": 80930,
"tgt": "What causes cough and difficulty in breathing?",
"src": "Patient: Hi. I have been sick for about a week, my sides hurt from coughing and I am having a hard time breahing, and when I breathe in it feels cold, I havent ate in four days and I have been couhging to the point of throwing up and I am weak and have been coughing so bad I feel like I am going to pass out. I have the shakes and I am freezing but my temperature is at 38.4Celcious. I have been to the emergency in my town and they sent me home and siad it was nothing, any suggestions? Doctor: Thanks for your question on HCM.I can understand your situation and problem.In my opinion you should immediately consult pulmonologist and get done1. Clinical examination of respiratory system.2. Chest x ray3. PFT (pulmonary function test).There are few possibilities in your case. Like1. Lower respiratory tract infection (LRTI) like pneumonia or tuberculosis.2. Bronchitis.So, chest x ray is needed to rule out lung infection.PFT is needed to rule out bronchitis.So better to first consult pulmonologist, diagnose yourself and then start appropriate treatment.Don't worry, you will be alright."
},
{
"id": 8370,
"tgt": "Can 50+ sun block cause premature wrinkling?",
"src": "Patient: HI! i have a question about a sun block. I've been using eucerin 50+ for about 3 years. almost every day when i go out. Now i am afraid ill damaged my skin and that sun block ingredients its to heavy for my facial skin and that this might cause premature aging. I've been using sun bloc because i want to be fully protected from the sun and its damaging effect. Can this 50+ sun block cause premature wrinkling? Doctor: Hello. Thanks for writing to us at healthcaremagicContrary to what you believe, a broad spectrum sunscreen is rather protective against sun induced damage (photoageing) and delays the onset of fine lines and wrinkles.Photoageing is characterized by gradually diminishing collagen and elastin from the dermis due to the effect of the UV A induced reactive oxygen species (ROS) which damage dermal collagen and elastin.UV A part of the electromagnetic spectrum is mainly responsible for photoageing.A broad spectrum sunscreen (spf 30 and more) filters both UV B as well as UV A part of the electromagnetic spectrum and is therefore protective. Therefore, you may continue with the sunscreen for its protective effects against photoageing.To be effective a sunscreen should be applied every 2-3 hours and in adequate amounts on the sun-exposed skin, specially when moving outdoors. A sunscreen should be applied 15-20 mins before stepping out in the sun to be fully effective.Regards"
},
{
"id": 49025,
"tgt": "Is taking BP medication with Kidney stone medication,safe?",
"src": "Patient: Hi, I am Aniruddha from Pune India, I am having high BP since many years untreated... Currently doctor suggested me Stamlo 2.5mg monring, Atorsave 20 mg lunch, and ecosprin night time. I am currently also taking medicines like meftal, K-Cit syp, and Alfoo for Kidney stone, Is it ok? Doctor: Hello!Yes, you can take anti-hypertensives with your stone medication.In my clinic, I do prescribe them together.Tkank you."
},
{
"id": 80688,
"tgt": "What causes hoarse cough with shortness of breath?",
"src": "Patient: I ve had a hoarse cough with shortness of breath for 8 months now. Lately my lungs have been sore too. I ve had xrays, ct scans, bloodwork, a pulmonary functions test and even a bronchoscopy. Nothing was found. During the bronchoscopy, he removed a bunch of excess phlegm and told me that I was inflamed, but he couldn t see why. Both the fungal and bacterial cultures also came back negative. From the pulmonologist I was sent to the allergist. There I had blood work and allergy testing. All came back fine. What could this be???It feels like my airways themselves are open, but I have something in the middle of my chest. I was thinking maybe a hernia, but would that cause a hoarse cough? Also, they put me on gerd medicine. No improvement. Doctor: Hello dear, thanks for your question on HCM. I can understand your situation and problem. In my opinion you are having uncontrolled GERD ( gastroesophageal reflux disease ) mostly. Since your extensive pulmonary work up is normal, no need to worry much for pulmonary causes. Uncontrolled acid reflux can cause chronic cough and breathlessness due to micro aspirations. And following lifestyle modifications along with drugs are needed for complete control of GERD. So avoid stress and anxiety. Avoid hot and spicy food. Avoid large meals, instead take frequent small meals. Go for walk after meals. Keep 2 - 3 pillows under head in bed to prevent reflux. Avoid alcohol and smoking, if you have these habits. Loose weight if you are obese. Don't worry, you will be alright."
},
{
"id": 103631,
"tgt": "Extremely itchy welts, stuffy nose, itchy throat, cough. On cytoxin treatments. Using protopin for itchy face. Help",
"src": "Patient: I am using protopin for skin itching on my face. I am currently getting monthly cytoxin treatments and my allergies have gone crazy. I now am getting welts that itch extremely bad and stuffy nose, sneezing, coughing, itchy throat and ears but the itching is the worst. What can I take or do for some relief, its driving me insane!!! Doctor: Hello,Thanks for posting your query,For immediate relief on severe allergic situations, you might require injectable forms of antihistamines ( a few couple administered) or steroids. When treating my patients, I usually administered them antihistaminic agents such as promethazine or give them injections of steroids such as dexamethasone or betamethasone. Most patients usually get relief after a few shots. I suggest you to the ER as soon as possible so a doctor can review you and provide you with appropriate medications. Best wishes"
},
{
"id": 118367,
"tgt": "hat can be done for RBC count of 7.3?",
"src": "Patient: my name is dorothy i am 47 years old .i have been told that i am severely anemic my red blood cell count is 7.3 i have recieved 2 units of whole blood.but my red blood cell count came up to a 7.3 the colonoscopy came back clear what else could be wromg and what tests should i get next? Doctor: hi, You need serum ferritin, TIBC and Tf saturation along with complete hemogram. there are many causes of anemia and hence i suggest you to consult a hematologist. Hope this helps."
},
{
"id": 33407,
"tgt": "What causes bumps on back,legs,neck,face & arms?",
"src": "Patient: My son has bumps on him there on his back, legs, neck, face and arms. One doctor said scabies and another said chicken pox he has bee around other kids and no one is getting them. He has scratched them tell they bleed what could they be if I can get an email address I can send a picture Doctor: Dear- you might have an allergy . Sometimes skin can be very sensitive and develops these lumps. Chicken pox is an acute viral condition that produces lumps but the will not last more than 10 days. I hope my advise will be helpfulDr.Sara"
},
{
"id": 207774,
"tgt": "What are the symptoms of oneirophrenia?",
"src": "Patient: Since I was 15, I have been in a dreamlike state, which I feared to be Depersonalisation disorder. I am 29 now and this feeling has not gone away, even for a moment, in the last 14 years. I have done more searching and found this thing called Oneirophrenia. I am aware that I have sugar problems and need to get it under control, but do not know exactly what to get tested for. Can Oneirophrenia last this long without any relief?Which tests would determine this diagnosis? Doctor: Oneirophrenia is a clinical diagnosis that your doctor makes after reviewing your history. It is a diagnosis that is now RARELY made.You should consult a psychiatrist who will evaluate you in detail. Dream-like state is a symptom of several conditions. Depending your complaints and history, the doctor may or may not order some tests.You say that you have sugar problem. If you are diabetic, then follow guidance of your doctor regarding medicines, diet and exercise.I hope this answers your question."
},
{
"id": 91747,
"tgt": "What should be done to treat a minor appendix symptom?",
"src": "Patient: yes, please, I have a very light kind of syntoms for appendix and I wonder if it can be helped with a treatment so I dnt have to go surgery. I feel like a very ight cramp in my right side lower abdominal area, does not stay, its like a nerve pinch butnot hurting nd I need some advise what to do, or what not to eat. Thank you Doctor: HI.You can take a full course of an antibiotic like cefixime or ofloxacin with metronidazole for at least 5 days. Take plenty or oral fluids, liquid home-made diet, take bed rest. Many of the times this will settle the attack. Get an ultrasonography done before starting antibiotics."
},
{
"id": 107071,
"tgt": "What does pain in the upper back on the right side indicate?",
"src": "Patient: I had a back injury last Thursday. My back is much better, but I have severe pain in my upper right quadrant. Especially when I lie Dow, and it is very tender to the touch , especially when lying down. I am on blood thinners for afibrillaton. This is the second be Ack injury I ve had within the span of one month Doctor: HiIt is possibly due to inflammation of muscles and soft tissues in injured area.Consult your heart doctor if you can take Motrin or Advil in short period course for 3 to 5 days.Do hot fomenting and apply anti inflammatory gels locally.take dolo neurobion also.Avoid bending,straining back or lifting any object.with adequate rest and pain meds it should subside"
},
{
"id": 94810,
"tgt": "Severe abdominal pain with diarrhea. Had appendicitis in the past. Any suggestions?",
"src": "Patient: I have had a bad stomach since few hours including bouts of diarrhea . However since two hours I get occasional bouts of severe pain on the upper abdomen/ lower heart region? I had got an emergency appendicitis surgery six months back, but this pain is different from what I was experiencing earlier during appendicitis? Should I be concerned? Doctor: Hi, Thanks for posting your query. You are most likely suffering from acute viral gastroenteritis. Presence of pain along with diarrhea indicates the diagnosis. You should consult with physician and should go for complete blood count, stool examination & ultrasound imaging. If you are suffering fromdehydration then you should get admission in hospital for 1-2 days. You should try to maintain your hydration by taking oral rehydration solution. You should also take antibiotics, anti-spasmodics and probiotics. You should also avoid milk and all milk relted products. Take care, Dr. Mayank Bhargava"
},
{
"id": 6714,
"tgt": "IS there any chance of me getting pregnant as the condom got stuck inside my vagina and later I took it off ?",
"src": "Patient: hi.am 23years old female. and my weight is 65kgs. i had sex with my partner last night.it was fifth day of my periods. he was wearing condom but at the end it stuck inside me and i immediately pulled it out. are there still any chances of pregnancy? Doctor: hi welcome to HCM read your prob, at 5thday of period there is less chance of getting pregnant, as ovulation starts around 8th day of period. n you did protected so there is no chance of getting pregnant. so do not worry. Dr aseeem aseemadhuri@gmail.com"
},
{
"id": 198408,
"tgt": "what causes brown color mixed with semen after ejaculation?",
"src": "Patient: Hi, I am a 59 year old male and when I ejaculated, there was a brownish color mixed in with the semen! I have never had this before, could you please tell me what would be causing this? About 6 weeks ago I was in spain and had what I thought might be a UTI bacause of burning when I urinated. After drinking 3 litres of water a day it went away. Could this be part of the cause and do I need to worry? Many thanks. Steve W. Doctor: HelloThanks for query .The brown coloured semen is mostly due to infection of either Seminal Vesicle or Prostate and needs to be investigated .Please consult qualified Urologist for clinical and digital rectal examination and get following basic tests done to confirm the diagnosis.1) Urine routine and culture.2) Ultrasound scanning of abdomen and pelvis3) Serum PSA.4) Semen examination and culture .In the mean while take antibiotics like Doxicycline and anti inflammatory drug like Diclofenaac twice daily .Further treatment will depend upon result of these tests and final diagnosis.Dr Patil.."
},
{
"id": 130336,
"tgt": "What causes hard pea sized bumps under each elbow incision?",
"src": "Patient: I had elbow arthroscopy about 6 weeeks ago and have very good mobility and litle or no pain of any kind. However, since resuming very strenous acitivy inthe past few weeks such as surfing and swimming, I have developed some hard pea sized bumps under each incision location. IS this OK? Or have I over done it and developed scar tissue? I am a 54 year old male in excellent health and athletic. Doctor: there is nothing to fear about , it is just fibrosed tissue on the site of incisions , nothing to be worried for it is a normal response of our body to many type of incisions.Be fearless it's everything just close to normal."
},
{
"id": 78391,
"tgt": "Will quantiferone test be effective for TB?",
"src": "Patient: i have mild leucopenia(3.6) ana absolute neutropenia( 1.79 )and i was contact with open TB case and i made tuberculin test and it was 7 mm ...is it false negative ...and is the quantiferone test for tb suitable for my case?? Also i have no drug history and no autoimmune disease and normal thyroid function Doctor: Hi. I can understand your concern. Sometimes viral infection and low immunity can cause leucopenia. Best way to improve immunity is by doing regular 30 mins exercise daily and having good nutritious food like green leafy vegetables, fruits, an egg every day and leading a stress free life. Tuberculin test and quantiferon are the same tests only thing is quantiferon is more expensive and does not give any added information apart from what tuberculin test gives. Don't worry, you will be alright. Hope I have solved your query. Wish you good health. Thanks."
},
{
"id": 11327,
"tgt": "What causes a spot of missing hair on scalp?",
"src": "Patient: My son has a small spot of missing hair on his scalp. It itches but nothing else seams to be of an issue. The spot has increased over the past 2 days. I thought it might be ring worm but its just a an area of missing hair no red spot no discoloration. What could this be? Doctor: HIWell come to HCMFungal infection is very likely and this can be best treated with \"selenium shampoo\" apply the shampoo on affected part leave it like that only for 30 minuets then wash out it, just do it once in day, new hair would be started growing, if this does not give any response then you need to see the Dermatologist, hope this information helps, take care."
},
{
"id": 1701,
"tgt": "Can i get pregnant while taking eltroxin for underactive thyroid?",
"src": "Patient: Good Day I'm 30 years old, just found out i have an underactive thyroid, and is using eltroxin for 2 weeks now. I want to get pregnant and underwent a laporoscopy to check if everything is still in good order. For how long do i have to use the eltroxin and while on this medication can i fall pregnant. Doctor: Hi, once you have normal thyroid profile, you can conceive. There is no problem in conceiving while taking eltroxin. The dose needs to be adjusted according to the value of TSH in pregnancy. Hope I have answered your question. Regards Dr khushboo"
},
{
"id": 176447,
"tgt": "What causes high body temprature in a child?",
"src": "Patient: my baby girl is 5. he had no health problems fro birth up wen she turned 3. She now has a problem of body temperature that normally reaches 40. This is even though there will be no other signs of illness. If she is given some paracetamol, her temperature immediately drops to normal or slightly below normal. She plays, eats normally even though the temperature is high Doctor: get her blood tests done to rule out any infections . get a cbp crp widal parasite v and f done and review wit a doctor"
},
{
"id": 8469,
"tgt": "Which to do first, the hormonal or laser treatment?",
"src": "Patient: hi Doctor, I m Keerthi (female 22 yrs, height:5.2 feet.weight:65). 2 yrs back i had abnormal periods i.e, once in 2 or 3 months. i got my harmonal tests done. testosterone was 86.10 NG/DL..got few tablets from my gynec. i have not checked it currently but my periods are regular as i reduced my weight. but my problem is i still have thick hairs on my chin. Thought of going for laser hair removal. I needed your advice whether i need to go for harmonal treatment first because i saw many posts of ppl on net who are done with hair removal but still hair is growing back.... please do suggest me.... Doctor: yes you are right, it is better you go for hormonal check up first to ensure that any hormonal balance is not there, then only laser hair reduction treatment will give you d desired results."
},
{
"id": 101615,
"tgt": "Suggest permanent solution for asthma",
"src": "Patient: i am a male 49 years old. In the year 1997, i shifted permanently from Ahmedabad to Bangalore. and Since then i have been suffering from severe asthma problem. i tried allopathy, homeopathy, siddha, but non gave me any relief. i am till suffering from the severity of asthma attacks. Nowadays my appetite is also gone down. no hunger at all and hence the gas problem. pls suggest me some permanent cure for this asthma problem Doctor: Hello dear,Asthma is caused due to broncho-constriction (obstruction of smaller airway passages) which is indicative of Hyper-responsiveness of air passages.Management consists of:1. Asthalin+ Seroflo inhaler- provide symptomatic relief by causing broncho-dilation (dilating the smaller airway passages, relieving the obstruction & increasing airflow to lungs)So, you can use it whenever you are having an acute attack.2. Montelukast preparations- used as a maintenance therapy to relieve symptoms of asthma. This will also give protection from recurrent allergy.3. Antihistamines like Cetrizine can also be used to provide symptomatic relief.4. Need for anti biotics can arise if there is a secondary infection. But this should be taken only under the guidance of your Physician.5. Steam inhalation & deep breathing exercises.6. Maintain adequate hydration & a healthy balance diet.7. Also protect yourself from exposure to cold, dust or other allergens.If symptoms still persist, kindly consult a Pulmonologist for complete clinical examination including Pulmonary Function Tests.Wish you a good health.Take care."
},
{
"id": 57072,
"tgt": "Can backpain be a result of sludge in gallbladder?",
"src": "Patient: Diagnosed sludge in gallbladder 5/52 ago got back pain RUQ pain steathorrhoea- pain wakes me up every morning around 3am pain is constant but gets very severe however my surgeon is not convinced a cholecystectomy would be the answer?? I am waiting a hida scan and a colonoscopy! I am very frustrated and convinced the gallbladder is causing all my symptoms I am normally fit and well! Since the start of my symptoms 5/12 ago I have felt awful and been admitted to hospital 4 times (sludge found on 3rd admission) with increased symptoms raised alt and crp of 187!! Why are they delaying surgery i am in agony!!!! Doctor: HIThanks for posting your query to Healthcaremagic . I personally feel that your pain is due to Gallbladder Sludge . Sludge increases the chances of stone formation and the stone and block the bile ducts and cause extreme pain ( Colic ) You said your AST is elevated . Cholecystitis can elevated AST but has any other blood tests done to look into other cause of elevated AST for example Hepatits A,B,C E , Dengue infection . It would be more helpful if you could upload all the tests done so that I can advise you better. But as of now even I feel a Cholecystectomy is the right option for you ."
},
{
"id": 453,
"tgt": "Is it possible to get pregnant through medicine without sex?",
"src": "Patient: hello doctor i had intercourse with my husband so many times but last was in month of DEC 2013, after that i got periods on 9th Jan 2014 doctor suggested me to take tables to continue tablets for regularity of periods now me taking thyroid medicine for 1 month , doctor says by the month of April i will be pregnant with out having any intercourse in between medicine time, and doctor giving me some pills to use for march month to get pregnant, is it worth full or its really work kindly suggest me Doctor: Hi, Thanks for the query.I understand your concern. * Normal ovulation &normal sperm are two basic necessities for conception. The sperms are deposited in female Vagina during her fertile days by- - Normal unprotected sex - intrauterine insemination(semen injected artificially in uterus. *the only possibility of conception without intercourse is test tube baby. * no tablets or medicines can impregnate without contact of ovum with sperms. Please take second opinion in case of doubt. Thanks"
},
{
"id": 88265,
"tgt": "What causes pain in upper abdomen?",
"src": "Patient: i recently suffered from a fever and loss about 3-4 kilos. I was taking amoxil and pain killers. Since the fever, I have felt stomach pain in my upper abdomen that i never felt before, the last three times i masterbated. I find that when I eat some food, the does pain eases abit. Any suggestions please? Doctor: Hi.Thanks for your query and an elucidate history of easing of pain on eating some food. The most probable diagnosis in your case looks to be Gastritis or gastric ulcer caused by the painkillers.I would suggest you the following:Zantac 12 hourly.Pepcid every 3 to 4 hours when on empty stomach. Add Domperidone or such motility regulators.Soft bland diet. No aerated colas / soda/ drinks.Early dinner. No late nights.Treatment for anxiety (as prescribed / needs a prescription by your Doctor)Lying in a reclining positionWalk around after dinner.Think of the factors which increased the present problem, GERD, anxiety and try to avoid. If no relief, get Upper GI endoscopy done and get prescription for added Medicines."
},
{
"id": 173774,
"tgt": "What causes a very bad smell during urination?",
"src": "Patient: babyMy son is 14 months old..lately his urine has smelled really bad to were i want to throw up ive been giving him lots of water so i dont understand whats going on. Also hes not circumcised n i went to pull his skin back he had a little red bump help me please Doctor: Hi...by what you quote I feel that it could be an urinary tract infection. But it is unlikely with out fever or vomiting. Moreover your son gas been circumscised. The other possibility includes - 1. Concentrated urine.2. Cystitis or inflammation of bladder.3. Inborn errors of metabolism.I suggest your pediatrician for this.Regards - Dr. Sumanth"
},
{
"id": 104796,
"tgt": "Having breathing difficulty and cough. Had fever. Is it serious?",
"src": "Patient: Hi. I have been feeling pretty unwell for about 2 weeks. For about 3 days I felt I had a very bad flu but after my fever has subsided and I m no longer very conjested. Now I have a really persistent cough , but this is weird too. I don t constantly cough, I am virtually fine for periods of time (3min-1hr) and then will cough really hard and not be able to breathe. I also have a bit of a burning itchy sensation when I inhale deeply, but not in my throat . I m a bit confused what this could be? Doctor: Hello. U were initially having upper respiratory tract infection with symptoms of throat congestion and flu. Now u probably developed lower respiratory tract infection and sinusitis with involvement of lung parenchyma. It probably is Pneumonia. U had to perform Chest Xray and labs such as Blood CP for elevated TLC count to check for infection in blood and Xray PNS to rule out Sinusitis. U should visit hospital and go through the foolowing tests and examination by your physician for appropriate antibiotics and anti histamines that can be levofoxacin and Loratidine and Pseudoephedrine in ur case and Paracetamol as antipyretic."
},
{
"id": 218282,
"tgt": "Is normal delivery possible when placenta is lower but away from the opening of the cervix at 35 weeks of pregnancy?",
"src": "Patient: Today I.e. 25 October my wife had an ultrasound in which details were aas follow :- 1:- placenta is lower but away from os. 2:- placenta maturity is 3 3:- placenta age is 35 weeks 4:- EDD is 29 November Edd in last 3 ultrasound was 10,11,12 November so when should I expect baby and it will be a normal delivery or c-section. Doctor: actual date of delivery is 2 weeks late or earlier than EDD ..now as for now placenta is away from os so there is no problem .i suggest repeat scan 3 weeks before EDD to see location of placenta then it will ne decided cs or normal vaginal delivery"
},
{
"id": 112284,
"tgt": "Lower back pain, swelling, purple bruising, tenderness. Is it something very serious?",
"src": "Patient: Hi, I fell down a flight of stairs in the outside stair well of a flat block 3-4 weeks ago. I landed on the top of my bum/lower back, at time there was a lot of swelling and after a few days a large purple bruise. Once the bruising faded I noticed a large oval shaped lump on my bum with yellow bruising around it, and since then it's never reduced. It's hard to touch and quite painful after sitting down for a long time. Any suggestions to what this could be and whether I should see a doctor? Doctor: Hi and thanks for the query,I do think you need to see the doctor. the color and the fact that it is still painful deserves that a collection of some pus inside be checked for. It is simple through a needle aspirations. In case of pus, a needle aspiration would be sufficient. If not, this should simply be the consequences of the trauma, with either mild bleeding into the tissue and swelling. In this case, you should be ok with time, But in case there is an infectious process going on inside, an incision could help. I suggest you see your doctor to be quite sure of the status. thanks and kind regards."
},
{
"id": 164582,
"tgt": "Suggest treatment for eye discharge in kids",
"src": "Patient: Hello. My four year old has a very small amount of something that resembles watery eye bugers coming from his left eye. He states that his eye hurts and it feels like there is something in it. Can I put a drop of Visine in it to get out whatever is in it? Doctor: Hi... by what you say I feel that your child might be having a viral conjunctivitis. I suggest you to clean his eyes frequently with clean room temperature water.But, eye conditions are best diagnosed only after seeing directly. I suggest you to upload photographs of the same on this website, so that I can guide you scientifically.Regards - Dr. Sumanth"
},
{
"id": 161244,
"tgt": "How can hyperactivity in a child be controlled?",
"src": "Patient: Dear Sir, I have a 5 years old kid ,he is very hyper in his activites but he is very intelligent.parents and teacher could not control him.when he alone in your home with your mother that s time he little quitebut what s the time any one entry in your house at the moment his activitiesis hyper. what s the procedure control him? Doctor: Hello, It needs a certain amount of sophistication with a pediatrician. this is a very common concern and therefore pretty much any pediatrician deals with it on a daily basis and a specialist generally is not needed. There are both drug and non-drug treatments. Hope I have answered your query. Let me know if I can assist you further. Take care Regards, Dr Matt Wachsman, Addiction Medicine Specialist"
},
{
"id": 128542,
"tgt": "What causes radiating pain under the arms despite a normal X-ray report?",
"src": "Patient: I ve had a pain against my body located few inches down from underarm which radiates towards back but X-rays only show very old broken bone and no new ones. Dr thought it might be muscle in Between ribs because few days before placed a box a stretched up high. It lessened after I stopped lifting anything but had to one day lift about a 6 pound bag. So it s sore now Doctor: It could be suprascapular nerve entrapment, you need to consult shoulder surgeon for further evaluation."
},
{
"id": 164499,
"tgt": "Can alcohol during pregnancy cause lower nasal bridge in kids?",
"src": "Patient: My daughter has a low nasal bridge. I drank heavily before I found out I was pregnant which was about 7 weeks and after that I had the occassional glass of red wine which my Dr said was fine. Now I am really worried because she has a low nasal bridge. She is very clever, alert and growing well but could I have done her damage? Doctor: taking alcohol during early pregnancy will cause birth defects and also fetal alcohol syndrome, but in your child,, it may not be due to your alcohol intake because flat nasal bridge is not a birth defects.. if flat nasal bridge along with mental retardation (down syndrome) is caused due to maternal alcohol during pregnancy.flat nasal bridge occurs in many healthy baby also.."
},
{
"id": 202149,
"tgt": "What should be done for bump on genital area after unprotected sex?",
"src": "Patient: Had unprotected oral sex about a week ago. Not a day or two later i got tingling on my penis head and slight increase in urinating. I just found a week later a bump on my genital area and more itching on my thighs around the genital area. Also now i have lower back pains. What should i do? Im highly worried. Doctor: Do you have burning while passing urine or fever. It seems you could have contracted some infection. Might not be anything serious. You should get a VDRL test done and see a Venerologist. Atleast take Tab Azithromycin 1 gm or Inj Ceftriaxone. Best would be to get checked by a Venerologist. Take caee and feel free to ask us again."
},
{
"id": 145535,
"tgt": "What causes tingling on face?",
"src": "Patient: I have slight tingling on my face. My forhead is feeling different I keep touching it. My eyes feel like they are part of it had a head injury about 6 days ago but no trouble It was light and I never lost consciousness or displayed signs of concussion. right by my left eybrow close to my temple, the doctor put dermabond. The head is close to healed so I don t htink it is connected Doctor: Hi,Thanks for writing in.The nerve supply to the face is through divisions of the same nerve. Therefore getting injury in one area can cause tingling in another part. That is why the injury near the temple can be cause for tingling in forehead.The nerves originate in the brain and divide in to 3 branches, they supply the skin over face and forehead region. An injury in one region will cause build up of chemical transmitters which carry information on pain and strange sensations like numbness and tingling. When the wound is healing and the nerves also heal then the tingling like sensations appear. Since the nerve branches come together in the brain therefore the sensations from one nerve branch might appear as coming from another branch."
},
{
"id": 148707,
"tgt": "Sore neck, dizziness, nausea, heaviness in eyes, fatigue, tingling face",
"src": "Patient: i have a very sore neck ,the pain is on the lower base of my skull . ive been to the neurologist he said that my spine sint oval shaped but heart shaped and it might be putting presure on the back of my spinal cord and i have some disc that are getting thinner. I get very dissy and get nausea, sometines my eyes get very heavy and i become disorientated, very fatigue , tingling in lower part of face, sensitive to smells and sound, very irritable , weird feeling in my head i think thats the dizzyness ,some short term memory loss Doctor: Hi there,Thanks for your query.Firstly, I agree with your neurologist's opinion. Spinal cord compression is causing most of your problems.Further, other problems like nausea, heaviness of eyes, fatigue, occasional disorientation, irritability, short-term memory loss and sensitivity to smells and sounds, may be due to either mental depression, Thyroid problem, allergy to certain smells or a type of epilepsy.Consult your doctor/psychiatrist and apprise him of my opinion. I am certain that he will agree with me, order required tests, and- based on the reports- will prescribe suitable/modified treatment as advised.In case of any further queries, revert back to me THROUGH DIRECT PREMIUM QUERY (Paid), addressed to me personally. You will receive- almost immediately- my response in great details.If you find my response helpful and informative, do not forget an \u201cexcellent\u201d (5-star rating) to my answer, to ENCOURAGE all doctors- engaged in social service- to render sound advice to the FREE queries. Fond regards and have a nice dayDr. Rakesh Karanwal"
},
{
"id": 24410,
"tgt": "What is the treatment for rheumatic heart disease?",
"src": "Patient: NAME ; LOKESHAGE ; 20I AM SUFFERING FROM RHEUMATIC HEART DISEASE I TAKING ALREADY TREATMENT FOR 21 DAYS PENCOM LA 12 PENCILLINMITRAL VALVE AND AORTIC VALVE GET DAMAGED NOW AORTIC VALVE IS NORMAL BUT MITRAL VALVE ; THICKENED LEAFLETS MILD SUBMITRAL FUSION,MVOA 2.0CENTIMETRE SQUARE CONCLUSION; RHEUMATIC HEART DISEASE ,MILD MITRAL STENOSIS WITH MILD MRFOR HOW MANY YEARS I HAVE TO TAKE THE TREATMENT Doctor: Hello...don't worry just need precaution and regular follow up... As there is not much obstruction or leak in your valve... So just need good follow up at least 6 monthly.... penicillin injection minimum needed to b take upto 40 yr ... Preferably life long .. Rest medication as per your valve status and symptoms..."
},
{
"id": 162695,
"tgt": "What causes stomach ache in a child?",
"src": "Patient: my child is complaining of a tummy ache. he just went poop, and it looked normal, not hard. he does not have a fever. any ideas of what i can look for? When i ask him to pinpoint where it hurts, he does focus around the tummy area, not the abdomen. Doctor: Hello and Welcome to \u2018Ask A Doctor\u2019 service. I have reviewed your query and here is my advice. There is no need to worry as this could only be due to the urge to defecate or accompanied gases. Hope I have answered your query. Let me know if I can assist you further."
},
{
"id": 189696,
"tgt": "Gums swollen, painful, red, mouth ulcer, getting wisdom tooth. Stolin astringent can be used?",
"src": "Patient: Hi my name is Ayesha uzma sultana I have sever mouth problem specially my gums it s hurting a lot dr I need your help a month before dr said me I have gingervits so he gave me tablets but unfortunate I don t continue that now my all gums r v swollen painfully red,I got mouth ulcer ,and am getting wisdom tooth am in to much pain can I use this stolin astringent for my gums it s too much painng thanx Doctor: HELLO AND WELCOME, Thanks for sharing your concern. your gums are affected with severe infection and ulcers. The wisdom teeth eruption has aggrevated pain. The gum infection can be due to- Poor oral hygiene maintenance. Nutritional deficiencies. SYstemic diseases. Faulty brushing methods. I would suggest you to get a thorough scaling done. Take nutritional rich diet. Maintain good oral hygiene. Rinse your mouth after every meals. Please do visit your dentist and get either flap overlying wisdom teeth removed or get it extracted if normal eruption is not possible. HOPE THIS HELPS"
},
{
"id": 72033,
"tgt": "Suggest treatment for severe chest pain",
"src": "Patient: Morning, I have a question regarding my heart. I think this may be either heart burn or anxiety but dont want to book an appointment at the doctors because I dont want to waste their time if it is only one of the above. Basically a few months ago, I had to run to catch my train home and I experienced chest pain, couldnt breath, had an aching cold pain in my bottom teeth, pains in my back etc so I called NHS direct when I got home as i was wheezing even after a 2 hour journey home and they sent me to the hospital to get checked over, I was in there for approximately 70 secs and the doctor sent me away and said im not having a heart attack. I booked an appointment the next day to see a doctor and he checked my chest and said I had a cold coming on and he could only put it down to a panic attack unless he made me go and do a run. This happens quite frequently when I have to sprint which I advised the doctor. Since this happened, I keep getting twinges in my heart, quick sharp pains, I googled it and found that it may be down to anxiety, but I get it when I walk round the house and often when i run up the stairs. Last night I went to the gym and im quite concerned now as I was fast jogging on the running machine and started to experience this pain in my heart after about 15 minutes, it was quite mild but I continued to run for an extra 5 minutes and the pain got quite painful but only happend when I breath in, it was as if my lungs were pushing against my heart when I inhaled. I obviosuly stopped running and when my heart rate got back to normal, the pain went away. Does this sound like heart burn, anxiety or acid reflux or should I go get checked out to be on the safe side? Please advise Regards Aimee Doctor: HelloAs you explain the history you should consult a cardiologist for further examination for the heart.Discuss with your doctor for this.RegardsDr.Jolanda"
},
{
"id": 86619,
"tgt": "Suggest treatment for umbilical hernia",
"src": "Patient: hello am a 57 year old man who has umbilical hernia done 14yrs ago immediately after the operation I notice that by abdomen was hard and sticking out like I was 9months pregnant. I went back to the surgeon and he said, there was nothing wrong. I went to a plastic surgeon and he said my abdominal muscles were stretched. but he told me a general surgeon might be able to help me I tried exercise running and got little results. I would like to get rid of this problem for good. Can you possibly help me with this. Doctor: Dear thanks for the question. Firstly I would like to inform you that you need to visit general surgeon as your treatment depends on physical examination of the abdomen. If I were your treating surgeon I would have gone for ultrasound scan of your abdomen after thorough examination. It could be recurrence of umbilical hernia or simply sagging related to obesity. Umbilical hernia would require surgery again and for obesity you have several options (liposuction, exercises, diet control).I hope I have clarified your queries. Do write me for follow up queries. Please rate my reply. I wish you good health. Thank you"
},
{
"id": 21477,
"tgt": "How to get cured completely from high blood pressure?",
"src": "Patient: I had quick heart beats about 130 and when I checked my blood pressure it was 160/110. After one week now it is 140/90 and normal heart beat but I am taking blood pressure pills 2.5 which my doctors says I have to continue using and consider myself a blood pressure patient. He also told me there is a great chance that I get cured if I lose my extra weight which is 130 Kg. At work I face lots of stresses and at home also. What are my chances of being completely cured from blood pressure? Doctor: Hi There I completely understand your concern for your health and I would like to tell you that STRESS and OVER WEIGHT are two important factors that causes high blood pressure. So if you want to get rid of high Blood pressure and medicines completely then you need to follow these things regularly for 3 months:1.Daily walks for a minimum of 45 mins for 5 days a week2. low dietary salt intake, avoid caffeine containing drinks, 3. stop smoking ( if you do )4. avoid junk food and include more vegetables and fruits in your daily diet. Follow this strictly for 3 months and you probably won't need any medicine.Wish you Good Health"
},
{
"id": 179676,
"tgt": "What causes swollen bump on right side of forehead after injury?",
"src": "Patient: Hi my son 8 has just walked into the side of a door and has a bump on th right of his forehead an inch above his right eye the bump had swollen up rapidly in seconds and quite large should we be worried and get it checked out as he has calmed down and seems ok apart from a little pain how should we treat it Doctor: Hi, thanks for asking. I can understand your concerns. IN my opinion there is nothing to worry at present. Just press some ice on the affected area. Give him some paracetamol for the pain. I hope I have helped youThank you"
},
{
"id": 184341,
"tgt": "What causes painful blisters in mouth after teeth removal?",
"src": "Patient: I had 3 teeth removed last monday and also 3 root canals done. When I left the dentist I had a big blister below my nose that formed within an hour after ariving home. Now its Friday and I have small blistors on the corner of right mouth. They are very small and painful. What caused the blisters??? Doctor: Thanks for your query, I have gone through your query.The blisters near the corner of the mouth could be because of the herpes virus infection. You might have got the infection while you were taking the treatment or it can be because of the stress. Nothing to worry its a self limiting condition but you can take topical antiviral ointments like acivir. you can also take topical anesthetic and analgesic like anabel gel. Apply 3-4times daily for 5 days.I hope my answer will help you, take care."
},
{
"id": 38431,
"tgt": "How to treat swollen glands and sore throat?",
"src": "Patient: Hi I had a sore throat with swollen glands for 4.5 weeks, my GP did a blood test to rule out glandular fever but bloods were satisfactory I m now experiencing small blister like lumps in the back of my throat. Could you give an indication of what this May be? Doctor: Hello, Thnx to contact us. If I am your treating doctor I would like to advice you that you should think of culture and sensitivity at blisters and lump you have. Once you have reports of sensitivity in your hand start antibiotic accordingly. You will get the results in a week. If you have anything else to ask please contact me. Thanx. Dr. Arun Tank; Infectious Disease Specialist"
},
{
"id": 49107,
"tgt": "What to do if the vein is tender, graft pumping well and no blood in coming in the syringe in a dialysis patient?",
"src": "Patient: hi,i am an dialysis patient.i have an av graft in my right upprr arm.went for dialysis,when needle was inserted.was in extreme pain,plus blood clot came out into syringe,nurse tried 4 times ,no blood was coming out,now vein is very tender,graft is still pumpimg well Doctor: Quite likely there is acute clot formation in your graft as its pulsatile and there is no blood flow. In a graft the likelyhood of improvement in that is less although its good to meet your vascular surgeon at the earliest."
},
{
"id": 164244,
"tgt": "What lead to sudden urinary incontinence and stomach ache?",
"src": "Patient: hi my daughter is 3 years old and has been potty trained for well over 6 months day and night with few to no accidents. For the last week she has lost all bladder control and is urinating more frequently, she is also complaining of tummy ache up to 3 times a day. She has been checked for a water infection and the results have come back negative. Doctor: You need to check also for constipation signs..Her psychological status can affect her and causes urinary incontinence.How long did she succeed to control her urinary bowel?Try to encourage her and restart the botty training bit by bit"
},
{
"id": 173267,
"tgt": "What causes watery pale stool in 12 months old?",
"src": "Patient: Hello doctor my baby is 12 months old.She is passing watery pale stool in 2 or 3 times a day.Before she was passing thick one and i never had face that problem as he is taking formula milk NAN.Now the stool is watery and i am upset.Please tell me what to do. Doctor: The concern in your child's case is understood. The watery diarrhoea diarrhoea is viral diarrhea usually. But as your child is formula fed, I would consider an infective cause.YOu would have to do a stool microscopy test.I would advise to stop bottle feed and give only breast milk always. YOu must give boiled water or ORS to prevent dehydration everytime she passes loose stools, in as much quantity as the loose stool lost. Based on this result we can start antibiotics or anti amebic drugs. Hope this answers your query and I will be glad to help further."
},
{
"id": 34146,
"tgt": "Experiencing trouble with my intestines",
"src": "Patient: There a puss filled lunchbox on the inside of my this I had a yeast infection on the inside of my upper third 2 month ago since then I've been having trouble with my intestines my dr treats me like a drug addict so he does not seem to care about my health trying to find new dr Doctor: Hello and thank you for your question.Not sure what you mean by a puss filled lunchbox. Please clarify that in another question. The best way to care for your intestines is to eat a healthy diet with lots of fresh fruits and vegetables and whole grains. Avoid alcohol, narcotics, tobacco, processed and fast food. Take a multivitamin daily and a probiotic. If you have serious bowel problems see a gastroenterologist.Regards"
},
{
"id": 8761,
"tgt": "Double process bleach, nausea, blisters on scalp. Remedy?",
"src": "Patient: I m a licensed hairdresser and yesterday I did a double process bleach on a girl who has had her hair done with hi lifts before. It turned out great..all during the process I kept asking her how she felt..if her head was irritated..nothing at all bothered her... today she emailed me and said she was nauseous and sick feeling last night and this morning has little blisters on her scalp!! Obviously had a reaction to the lightener this time but what I can do for her to calm her scalp down? Doctor: Hi...dear user..., Thanks for choosing HCM.., According to Ur medical history..., she had....IRRITANT DERMATITIS.., It is due to chemical irritation by using of cleansers.., So tell her immediately use the following treatment.., 1) Tab Cetrizine 10 mg daily night times for 7 days.., 2) Morning times ..use Betamethasone 1 mg tablets...,7 days 3) Tab Ofloxacin 200 mg daily 2 times for 3 days.., 4) Use topical on hair area,....Fusidic acid and Betamethasone gel.., on Scalp area after repeated head washes...by using.., Antibacterial and Antifungal shampoos... within day it will come to normal side..., thanQ"
},
{
"id": 189821,
"tgt": "Bleeding on roof of mouth, no soreness. Had implants before bleeding started. Cause?",
"src": "Patient: It seems as if the roof my my mouth is bleeding . I have top and bottom dentures and have had false teeth since I was 14. It only bleeds when i take my teeth out and suck and then the blood flows. I have identified it coming from the roof of my mouth. Nothing is sore right now but two years ago i had a fungus in my mouth. I had impants on the bottom just before all this began to happen Doctor: dear friend, thanx, for sharing your concern, bleeding from the roof of the mouth (palate) could be due to mechanical injury to the underlying tissues. your history shows that , you are a denture wearer since long time. so i am sure your denture are well fitting, to the tissues ? if not it requires to be examined . you have mentioned that it bleeds when you remove the dentures and try to suck . this could be due to the pressure developed on sucking on the weak or inflamed palatal tissues . for this i would suggest you to get clinically evaluated by dentist, also take an xray to see if there is some pathology underneath . have you checked for any sharp edges in your dentures? also some times ill fitting dentures lead to accumulation of saliva and fungal and bacterial growth these could be the pre disposing factors in your case . get your self clinically examined once . always store dentures in water when not in use . regular cleaning them is also mandatory. gum massage and warm saline rinse might help in your case. thanx"
},
{
"id": 38191,
"tgt": "What causes an ulcer with white head at bottom of the lip?",
"src": "Patient: I am an 18 year old female and i have a huge white ulcer on the inside of my bottom lip and it rubs against my teeth and is very painful. i have had it for 7 days and it seems to only be getting bigger. i have never had an ulcer. what can i do to make it go away? Doctor: Thanks for contacting HCM with you health concerns.From your description I would venture to guess that the ulceration you have is called an aphthous ulcer. These ulcers are possibly caused by a virus but we are not completely sure about that. There is little to help heal the ulcer other than time and they take a few days to a week to heal. To help reduce the pain I recommend for my patient a 1/2 and 1/2 mixture of Benadryl and Maalox. Swish this in you mouth and around the ulcer and then spit the mixture out. The Benadryl with help numb the ulcer and the Maalox will help protect the ulcer by coating it.Hope I answered your question. Please contact us again with your medical concerns"
},
{
"id": 108902,
"tgt": "Can fibroids in ovary lead to back pain?",
"src": "Patient: Hi, may I answer your health queries right now ? Dear doctor, I am 33 yrs old, married with one daughter age 9. In recent times I have developed severe backaches which continues for several days. I had a miscarriage recently, during my last scan my doctor informed me that my left ovary is slightly oversized compare to the right one. Can it be due to fibroid. I always hear woman saying if having terible backpain always, it might be because of fibroid, is that true? My daughter (9 yrs old) was born through vacuum, and my doctor recommended epidural during my delivery due to very long labor. Can you pls advise? Doctor: hi,welcome to HCM As I can understand from your history, your major complaints is of back pain.A slight oversized left ovary, as reported by doctor. A history of miscarriage.considering this information following conditions appears likelyovarian cyst, uterine fibroid, endometriosis.all these conditions can have presentation similar to yours.If a patient with complaints and history as yours comes to my clinic, I would advice an Transvaginal sonography to rule out uterine conditions. A consultation to gynaecologist will definitely help.welcome for further suggestions.Regards,Dr Pankaj Borade"
},
{
"id": 123831,
"tgt": "Suggest possible treatment for sprain in ankle & swelling like a bump",
"src": "Patient: I think I sprained my ankle three weeks ago. I got an x ray and came back fine. It is still swollen but doesn t really hurt? I have iced ot and heated it the swelling like a bump around bone I think? Don t know what to do. I can walk and run on it. But worried about why it is swollen still? Doctor: Hello, As there is no fracture this us a ligament sprain. I will advise you to use the ankle brace or crepe bandage so the swelling comes down and also it will provide stability to the joint while walking. Please allow 3 weeks of time and you should have no symptoms. Hope I have answered your query. Let me know if I can assist you further. Regards, Jay Indravadan Patel, Physical Therapist or Physiotherapist"
},
{
"id": 146437,
"tgt": "Can a bulging disc in L5-S1 area cause problem with bowel movements?",
"src": "Patient: Can a bulging disc in the L5 S1 area cause major pain, troubles with bowel movements numbness and tingly in my butt legs and feet? Also is this why I have allot of pressure and pain in my butt crack? I was also wondering if this can cause me to have issue with head aches? Doctor: Hi, I had gone through your question and understand your concerns. If the bulging is compressing your lumbar spine and nerves, then it may cause your symptoms, especially if you have a narrow lumbar canal. You need to consult a Neurologist and if there is spinal stenosis you need decompresive surgery as the condition may progress. Hope this answers your question. If you have additional questions or follow up questions then please do not hesitate in writing to us. I will be happy to answer your questions."
},
{
"id": 161340,
"tgt": "Suggest remedy for indigestion and loose stools in a child",
"src": "Patient: my son is 10 years old and is having loose motion maybe due to indigestion as he had fried food yesterday. He has passed watery stools about 4 times since early morning today. He also had a temperature of 101.3 underarm. I gave him some ginger tea. What else can I do? Please advice Doctor: Hello, If a similar case comes to my OPD, I would first check his weight because unless we know weight can't prescribe medicines for kids. Ors has to be given with mild hot water, Sporolac sachet in 5 ml water and zinc syrup 5 ml. if he is a healthy kid of normal weight and height then give him paracetamol 250 mg 2 times a day. If it doesn't reduce in a day kindly meet your doctor. Hope I have answered your query. Let me know if I can assist you further. Take care Regards, Dr Jilu Joseph, General & Family Physician"
},
{
"id": 209457,
"tgt": "Is diazepam recomanded anxiety and depression?",
"src": "Patient: Hello, I am a 50 year old woman and I need help, please! I had oral surgery on 4/10/14, was given versed, lots of swelling, everything seemed to go well, on hydrocodone, Advil. Got a bad head cold and took OTC cold meds, decongestants, antihistamine, nasal spray, Tylenol etc. A few days later I m crying all the time, I feel so bad, anxious, scared, don t want to be alone, don t like the dark. I saw my nurse practitioner and she gave me diazepam (10mg) to calm me down . Scared to take the full dose so I took 1/2 and that drugged me enough to calm down but didn t last. Next week I beg to see my Nurse again and she gives me zolpidem (5 mg) to sleep, and take diazepam as needed, and Zoloft. I took the Zoloft for 2 days(25mg) an I quit as I have never felt more anxious and nervous in my life, also took the diazepam(2.5mg) at a time but it made me feel worse. A couple days later I felt like I might be getting a handle on this thing. Had one day without any diazepam...but then I started to feel weepy again and more anxious. I need help! What happened to me? I have never had anxiety or depression. I am so tired of no answers. Please help. I m sorry I don t have the money can you help me anyway??? Doctor: Hello,Thanks for choosing health care magic for posting your query.I have gone through your question in detail and I can understand what you are going through.Yes diazepam is indicated for treatment of anxiety disorders. However along with it other drugs like escitalopram or paroxetine should also be used. Hope I am able to answer your concerns.If you have any further query, I would be glad to help you.In future if you wish to contact me directly, you can use the below mentioned link:bit.ly/dr-srikanth-reddy\u00a0\u00a0\u00a0\u00a0\u00a0\u00a0\u00a0\u00a0\u00a0\u00a0\u00a0\u00a0\u00a0\u00a0\u00a0\u00a0\u00a0\u00a0\u00a0\u00a0\u00a0\u00a0\u00a0\u00a0\u00a0\u00a0\u00a0\u00a0\u00a0\u00a0\u00a0\u00a0\u00a0\u00a0\u00a0\u00a0\u00a0\u00a0\u00a0\u00a0"
},
{
"id": 143901,
"tgt": "What causes nausea, headache and cold post consumption of Advil?",
"src": "Patient: Hello Doctors.. I have been having a problem with migraine but it occurs about 3 times a year only. But the other day, my migraine hits and I took advil, slept for 6 hours and the headache is gone. But today when I woke up around 11 am, I felt a throbbing pain on my right eyebrow near the nose-bridge and if I press it the pain is excruciating, When I stood up I felt nauseated and I wanted to vomit. I took another advil and went back to bed. I slept for 2 hours and the headache is less but still there. I wanted to take sinutab because I felt like I caught a cold as well as I m having a running nose yesterday. Can you give some advice? Thank you so much. Doctor: HelloThis sounds like a sinus infection and, in that case, it isn't advisable to take a decongestant like sinutab.You can try a Netipot or a saline nasal irrigation system but if this continues for more than three days you need to see your doctor for a course of antibiotics"
},
{
"id": 102414,
"tgt": "How to heal cold,sneezing,runny nose and lousiness with recent history of hospitalisation with flu/pneumonia/COPD?",
"src": "Patient: i was recently discharged from the hospital with flu/pneumonia/COPD. I have been home for about 1 wk. 2 days ago i acquired a cold which my physican put me back on antibiotics. Never have been able to produce any mucus even in the hospital. Have been using a nebulizer since discharged 4 times a day and started this really painful cough today. Still have sneezing,runny nose,and just feel lousy. What should I do at this point? I only received my antibiotic late yesterday even though I called my doctor early yesterday. Could I have the flu or pneumonia again or is this just a cold which is what my doctor suspected. never mind I'll just call the doctor. What a lousy thing to do to someone who is scared and really feeling BAD! Doctor: It may be simple viral with allergy. Take levocetrizine analgesic and vit C. Take temperature. Can take cough sedative and nasal drop. If symptom still persist then consult physician."
},
{
"id": 80926,
"tgt": "What causes vomiting, pain and pressure in chest and throat?",
"src": "Patient: Hi. I have had feelings of pain and pressure on my chest after getting sick and vomiting a lot about a week ago. I have a sore throbbing pain in my asopgagus and mild headache. I have been taking a lot of Zantac and it has not gotten any better. I have never had issues like this before except when I was pregnant. The pain is more pressure on my throat than burning. Could this be Be or Gerd? Will it heal? Doctor: Thanks for your question on HCM.I can understand your situation and problem.In my opinion it is mostly related to GERD (gastroesophageal reflux disease).But better to rule out cardiac cause first. So get done ECG. If ECG is normal then mostly it is due to GERD.It is because of laxity of gastroesophageal sphincter.Due to this, the acid of the stomach tends to come up in the esophagus and cause the symptoms like chest pressure, throat discomfort etc.So control of GERD is needed to relieve your symptoms.Try to follow these steps for better symptomatic relief.1. Avoid hot and spicy food.2. Avoid large meals, instead take frequent small meals.3. Avoid stress and tension.4. Start proton pump inhibitor.5. Go for walk after meals.6. Keep 2-3 pillows under head in bed to prevent reflux.7. Avoid smoking and alcohol if you have this habits.8. Loose some weight if you are obese."
},
{
"id": 51858,
"tgt": "Kidney problem with calcium and phosphorus in blood",
"src": "Patient: my father is suffering from Kidney damage.. he is having excess of Ca & Ph in blood .. can a dialysis be avoided using medication? He was having hemoglobin at 6 when admitted to hospital but after giving Espogen , Iron IVs, it has improved.. but the BUN and Calcium and Phosphorus is not being cleared... His legs are paining a lot and its difficult for him to walk.. Doctor: hello, welcome to HCM. You need to specifically tell the values of blood Ca and Phosphorus. If blood creatinine and BUN are high your father will definitely need dialysis. If the levels are marginally high they can be brought down to normal levels with help of medicines. But overall blood levels are the deciding factors. it would be advisable to undergo dialysis if the levels are high as they can be toxic for health if retained for long time in excess. Wish your father good health."
},
{
"id": 120192,
"tgt": "Can injury to neck and bulging disc after an accident affect energy level?",
"src": "Patient: Hello I was involved in a car accident in September of this year. Since then I've been sleeping a lot, and my energy i very low. I suffered injury to my neck; bulging in three discs, bulging in 3 discs on my lower back, tear of my shoulder, bone impingement (shoulder), problems with rotation cuff. Are these problems affecting my energy level? Doctor: Hello,Neck\u00a0stiffness and tenderness, and shoulder aches can cause tiredness and low energy levels. If you have pain, it should be treated with painkillers. I also suggest using a vitamin B supplement to boost the energy levels. I recommend to drink liquids and rest.Hope I have answered your question. Let me know if I can assist you further. Regards, Dr. Dorina Gurabardhi, General & Family Physician"
},
{
"id": 167483,
"tgt": "What causes fever in a child after being treated for lung infection?",
"src": "Patient: Hi there, my three year old had a lung infection and was treated with orilox for 7 days. He still has a fever and red bumps that comes and goes over his body. he s also a little light sensitive and complains of a headache. Can there still be an infection after antibiotics? Should i take him to the ER? Doctor: Hi... by what you say I feel that this could be a viral illness. Fever with Rash is usually a sign of viral illness. But the kid should be active in between two fever episodes and he should not be sick. If he is not well in between two fever episodes and is sick looking, I suggest you take him to the emergency room.Regards - Dr. Sumanth"
},
{
"id": 126234,
"tgt": "Suggest cause for recurring joint pain with difficulty in walking",
"src": "Patient: I have been suffering from severe joint pains now for a month and a half.it all seemed to start after a virus I had for weeks.No matter the medication I take it just masks the problem and it returns. In the morning when I get up I have problems getting out of bed and walking. Some times even the bottom of my feet ache.Can you give me any ideas what I may be dealing with? This is starting to affect my daily life big time! Doctor: Hello, Consult an orthopaedician and get evaluated. You can take analgesics like Ibuprofen or Diclofenac for pain relief. If symptoms persist you can consult an orthopaedician and get evaluated. Hope I have answered your query. Let me know if I can assist you further. Take care Regards, Dr. Shinas Hussain, General & Family Physician"
},
{
"id": 147591,
"tgt": "How to cure paralysis in a 75 year old female?",
"src": "Patient: my grandmother age 75 is suffering from paralysis and can just only talk and eat very little food. she is being given dexona injection and aptron. is it sufficient for the treatement or some other medications are required and how long this disease will take to get cured? Doctor: Hi,Thank you for posting your query.I have noted your grandmother's symptoms.We need more details regarding her illness to help you understand better regarding the treatment options.What was the cause of her paralysis? Was it brain stroke? If possible, please upload the scan reports.If the paralysis is due to brain stroke, then, complete cure may not be possible. However, with good physiotherapy and exercises, her strength can be partly restored.Please check about \"aptron\" tablets, as i am not sure about it. Dexona is a steroid used to reduce the brain swelling.I hope my answer helps. Please get back if you have any follow up queries or if you require any additional information.Wishing you good health,Dr Sudhir Kumar MD (Internal Medicine), DM (Neurology)Senior Consultant NeurologistApollo Hospitals, Hyderabad, IndiaClick on this link to ask me a DIRECT QUERY: http://bit.ly/Dr-Sudhir-kumarMy BLOG: http://bestneurodoctor.blogspot.in"
},
{
"id": 195213,
"tgt": "What causes discomfort in the groin area?",
"src": "Patient: My 17 yr old son told/showed me a recent balloon-feeling area near his groin. Im suspecting an inguinal hernia because he helped move a console tv{ when I wasnt home to stop him). didnt notice pain then but was too heavy to lift . Coughing alot due to asmatic brobchitis since last week is causing him to notice this area. Your thoughts ? Guess I wont find out now. No money. Widowed mother of 2 , working 7 days a week for last 15 months has no extra money, just trying not to worry myself silly till we see the doctor. Thought this was free. Sorry to bother anyone, Lisa Doctor: Hello and Welcome to \u2018Ask A Doctor\u2019 service. I have reviewed your query and here is my advice. Very difficult to comment without seeing pictures. If the swelling increases during cough it could be a hernia. Need a direct examination to confirm. Hope I have answered your query. Let me know if I can assist you further. Regards,\u00a0\u00a0\u00a0\u00a0\u00a0 Dr. B. Radhakrishnan"
},
{
"id": 59568,
"tgt": "Suffering from mild jaundice, severe stomach pain with vomiting. All reports normal. What to do ?",
"src": "Patient: I am suffering from mild jaundice since 6 months.Last year I had bilirubin level(conj.-0.6&unconj.-1.2).I visited a doctor then it was reduced to 1.4.He then stopped treatment & suggested to take rest.But this year one day I felt sever stomach pain with vomitting.Doctor told me to go for LFT.Again bilirubin level increased to 2.84(conj.-0.7&unconj.-2.14).After a treatment of one month it was reduced to 1.1(conj.-0.7&unconj.-0.4).Then she suggested folvite tablet as repeatation for next one month.After 3 months of this again I checked 3 weeks before and it was again 1.45(conj.-0.78&unconj.-0.67).My USG report of whole abdomen was in normal range. Stool test report was normal & hemoglobin report was also normal.Now my problem is I m feeling tired & drowsy,loss of apetite,energy,&stomach pain.So,what should I do?? Doctor: Hi Kunal, Welcome to healthcare Magic! You have mentioned the bilirubin levels only, rather you should have mentioned liver function test fully. I think you are having Gilbert syndrome and which is a very mild liver disease. I would like you to get fresh CBC, CRP, LFT, PT, serum Albumin levels and with reports put a specialist query to me or get reviewed by an expert local physician. You should take tab. Supradyn a day and tab. Ultracet for malaise as required basis till you get reports. Wish you a great health!"
},
{
"id": 151126,
"tgt": "Wide spinal canal, central disk extrusion causing impingement upon both L5 nerve root. Meaning ?",
"src": "Patient: I have multilevel degenerative lumbar spine disease. Spinal canal is wide at l5/s1 l4/5 there is a moderate size broad based central disk extrusion causing impingement upon both l5 nerve root more pronounced on the right just had this report back from mri what does this all mean and what can doctors do to hel is surgery required? Doctor: Hi, Thank you for posting your query. I have gone through your MRI lumbar spine report. The report shows compression of one of the lumbar nerves (L5), which travels from lower back to the foot. As a result, you may experience pain along the distribution. In most cases, surgery is not required. Treatment consists of medicines such as pregabalin to reduce pain, and physiotherapy. If medical measures fail, surgery is required in about 10% of cases. Best wishes, Dr Sudhir Kumar MD DM (Neurology)"
},
{
"id": 104175,
"tgt": "Suffering with allergic bronchitis, get pain in back, lower part of stomach. Could this be kidney problem?",
"src": "Patient: HI, I am suffering from alleric bronchitis . For this I use rota caps surflo inhaler(100 mg daily once). After using this I am getting pain in lower of the stomach front ,some time fornt and back side of stomach.is It related any kidny problem? if yes .how to check kindy problem? Pls help me And sime time I am getting light pain in left ear . Doctor: the inhalers and rotacaps cause acid reflux leading to symptomps with inhalers and rotacap it is required to drink lot of water take lot of warm watere add liquid antacisd and rhinitidine 150 mg empty stomach if it persists can go for ultrasound kub and other investigations of blood to rule out extra problems"
},
{
"id": 185466,
"tgt": "What causes boils in the gum?",
"src": "Patient: Yesterday a gum boil just popped up in my lower left molar area on the outside. The tooth isn't sore and there is no pain at all associated with it. The boil is still there today. Should I see a dentist right away or might this likely go away on it's own? Doctor: Hello, Read your query, as you have boils in the gum this boil can be due to infection in gums, inflamation in gums and periodontal ligaments , poor oral hygiene , trauma due to brushing, or irritants such as tobacco containing products. For this I will suggest you to on consult dentist and go for investigations IOPA x ray of that tooth , if there is Abscess formation Periodontal Abscess then you should go for Oral prophylaxis and curretage . In mean time do luke warm saline gargle two - three times a day, do chlorhexidine mouthwash twice daily, Brush your teeth twice daily. Consult dentist for oral examination and ots treatment. Regards, Dr. Priyanka tiwari"
},
{
"id": 43982,
"tgt": "Done semen analysis test. How to activate dead sperms?",
"src": "Patient: i got my semens anaylysis test report.... Total sperms count 53million 1% sperms actively motile 2%sperms sluggish motile 97%sperms non motile 20%sperms are abnormal RBCS....nill WBCs....15-20/HPF Epithellal.....nill 1 want to know how to active dead sperms and how much time it ll take....i totally vegeterian....never take any type of drugs...... Doctor: Hello, Welcome to HCM, I am Dr. Das, Firstly, Sperm count is within normal limit ( normal is > 20 millions /ml as per WHO 1999). Motility is very much low ( normal is > 50% as per WHO1999). Pus cells are very much increased. It indicates an ongoing lower urinary tract infection. For having back the normal condition, you have to take antibiotics. Consult with a infertility specialist/ general physician for prescribing ciprofloxacin group of drugs for minimum 7 days. After completing the course of antibiotic you have to repeat the test again. Regards."
},
{
"id": 183888,
"tgt": "Suggest remedy for recurring mouth ulcers",
"src": "Patient: hi doctor my frnd is 25 yrs old and he has mouth ulcer problem since childhood doc is sayng it started spreading in his body if he take givn medicine regulary till he bcome 30 years old then only he can survive..is it true.. just wanted to ask is there other solution on this or treatment i want to save him doctr said there are only 20 % chances to survive Doctor: Thanks for using Health Care Magic.Read your query.It is nice to have concern ,but it is not clear with this history to suggest and confirm any diagnosis of your friend .It can be some auto immune disease which can be the reason behind this.I would advice to follow the treatment correctly done by his doctor and maintain the health well.Have a nutritious diet which boost the immune system well.Use the antiseptic mouthwash or the wash prescribed by his doctor.Hope this was useful and he has a speedy recovery.Thanks and regards."
},
{
"id": 223607,
"tgt": "Can I use Tri-Sprintec as emergency contraception?",
"src": "Patient: Can I use Tri-Sprintec as emergency contraception? There s a list of birth control pills that can be used as such on Planned Parenthood s website, but Tri-Sprintec is not on there. However, a doctor on another site said it can be used and I would like a second opinion. Doctor: Hello, and I hope I can help you today.It is true that any brand of combined oral contraceptives can be used as the morning-after pill. They may list the most commonly prescribed pills on the Planned Parenthood website, however I will illustrate how you may use Tri-Sprintec correctly as a morning-after pill.The most important part of the regimen is making sure that the pills that you take total more than 100 \u00b5g of estrogen per dose. The dose is need to be taken twice, 12 hours apart to be effective. It is recommended that emergency contraception be taken within 72 hours of unprotected intercourse, however there may be some benefit for up to five days. So for example, with a pill with 20 \u00b5g of estrogen (like Aviane, for example) you need to take to five pills per dose, in two doses, 12 hours apart.Tri-Sprintec contains 35 \u00b5g of estrogen in the darkest colored pills the week before the placebo week. So if you take three pills from that section of the pack in two doses 12 hours apart (a total of six pills) you will be able to use Tri-Sprintec as an emergency contraceptive.As a warning, using estrogen containing products as emergency contraception usually causes severe nausea as a side effect. Try to eat right before taking your first dose, as you may not feel up to eating when the second dose is due.I hope that I was able to adequately answer your question today, and that this information was helpful.Best wishes,Dr. Brown"
},
{
"id": 19194,
"tgt": "What causes tingling sensation from head to toe with fast heart beat?",
"src": "Patient: i have had this weird hot warm tigling sensation from head to toe and i feel like if i pee it is really hot and my heart rate goes FAST went to urgent care once and said it might be the vaso vagel vein and i went to the er a few days ago and they did an EKG CAME BACK fine they gave me ativan but no help Doctor: Hello!Welcome and thank you for asking on HCM!I passed carefully through your question and would explain that your symptoms could be related to a metabolic disorders (low blood pressure, thyroid gland dysfunction, etc..). Coming to this point, I would recommend consulting with your GP for a careful physical exam and some tests: - a cardiac ultrasound- a chest X ray study- thyroid hormone levels- blood electrolytes- a Head Up Tilt test for orthostatic intolerance. Hope to have been helpful to you!Wishing all the best, Dr. Iliri"
},
{
"id": 179701,
"tgt": "What causes bleeding through the nose after a fall?",
"src": "Patient: my 12 year old son fell and hit back of head yesterday on wet concrete he ad a big nose bleed straight away which lasted about 20 min then stopped he had head ache but is fine now but had a small nose bleed this morning for only a couple of min but it was runny aswell Doctor: HiNasal bleeding is quite common in head injury children. In the absence of loss of consciousness there is no cause for concern.Local application of ice pack will take care of the issue.Take CareBest RegardsDr T Shobha Deepak"
},
{
"id": 75430,
"tgt": "What causes pain in my neck and lungs while breathing deeply?",
"src": "Patient: I have a lot of pain in my neck and lungs when I breathe in deeply. Another problem I have is I have been randomly choking, not on food... which is strange but I just choke... maybe on an air bubble? I also have endometriosis and had surgery about 7 months ago, now my pain is back and worse... I m on birth control but I am still lightly bleeding and having clots come out... WHATS WRONG WITH ME??? 19, female Doctor: For your breathing problems and choking you need to consult a pulmonologist who will examine you in detail and decide on the line of management. Regarding endometriosis, pl consult a gynaecologist."
},
{
"id": 144475,
"tgt": "What causes dizziness when walking, sitting, standing and lying down?",
"src": "Patient: Hi Dr, I have just taken up a new hobby, I am on the trampoline for 1-2hrs at a time and after my last session I have become dizzy at random points when walking around, standing, sitting and laying down! This has been going in for 2days now on and off. Have i coursed some damage or is it am inner ear issue? Never had anything like this before and never get motion sickness. Thanks for your help Doctor: Dear, Welcome to HCMWe understand your concernsI went through your details. Equilibrioception or sense of balance is one of the physiological senses. It helps prevent humans and animals from falling over when walking or standing still. Balance is the result of a number of body systems working together. The eyes (visual system), ears (vestibular system) and the body's sense of where it is in space (proprioception) ideally need to be intact. The vestibular system, the region of the inner ear where three semicircular canals converge, works with the visual system to keep objects in focus when the head is moving. This is called the vestibulo-ocular reflex. I don't think there is anything to worry. it is just your new hobby. The ups and downs in trampoline definitely affects your balance because it affects the vestibulo-ocular reflex. This habitual reflex stays there till it gets used to it. Could take a week or more of continuous habitual practice. Don't worry.If you require more of my help in this aspect, please use this URL. http://goo.gl/aYW2pR. Make sure that you include every minute detail possible. Hope this answers your query. Further clarifications are welcome.Good luck. Take care."
},
{
"id": 180643,
"tgt": "How can an elongated uvula be treated?",
"src": "Patient: Good evening I have have now had an elongated uvula for almost 3 weeks. 4 different Drs including ENT. First one said probably Gerd gave me Prilosec, 2nd one concurred have me lidocaine even tho I advised no real pain when swallowing. Went to third dr who adv to try antibiotics and prednisone, and saw post nasal drip. referred to ENT. Finally my Dr who advised to stop prednisone and antibiotics and said saw no infection. Now ENT said continue taking. uvula looks slightly elongated and he could remove part of it. I fail to believe this my only option w so many different answers Are there test I can run to see why this has happened and not gone down? Doctor: Hello,Elongated uvula can be due to a number of causes like infection, hereditary hyperplasia, inflammation etcetera. At times it does not do to any underlying issues. So my suggestion is to consult an ENT physician and get evaluated. If it is not infected or due to any pathology then it does not need to be surgically reshaped. Hope I have answered your query. Let me know if I can assist you further.Regards,Dr. Honey Arora"
},
{
"id": 108409,
"tgt": "What causes back surgery with hip fusion in a 45 year old?",
"src": "Patient: I m a 45 year old woman with prior back surgery hip fusion and Rods the surgery was and a half ago I am maybe 20 pounds overweight which I m working on I m already down 25 pounds I m 5 foot 3 non-smokerand I have not ate anything high in sodium North Salt,,/also I am on 15 milligrams a percocet 4 times a day no Tylenol thank you Doctor: If there is a degenerative arthritis for your hip as well problem with degenerative disc issue at your back then over the period of time the cartilage in joint will be damaged for hip joint. In this kind of cases patient needs to get replaced with joint or if the joint is degenerated a lot and not in position to get replace you need to undergo for joint fusion too. Both this option depends on your doctor how he is willing to operate as per patient condition.And he might have taken option to fuse the joint.Now as per your back is concern may be due to degenerative disc problem at your back at multiple level as well due to loss of lumbar curve and may be due to multiple disc prolapse your doctor may have taken dision to stabilise it with rode and screws.Now here since your explanation was not clear I have given the general idea on this. I am sure this will be useful for you. Take care."
},
{
"id": 101542,
"tgt": "Suggest medication for occular rosacea",
"src": "Patient: I was diagnosed with occular rosacea. After many treatments including steroid drops and allergy drops as well as artificial tears daily I have had no relief. I have also recently developed a red dry patch on top of my eyelid. I am unsure what to do next. I have seen dr 6 times in regards to this. Symptoms are worse than ever... Doctor: Rosacea is caused by inherent defects in the body's immune system and vasoregulatory processes. Treatment is directed toward symptomatic control and disease prevention rather than cure.When treating ocular rosacea, a stepwise approach can be undertaken, using first lid hygiene and artificial tears, followed by topical and oral anti-inflammatory medications, with late surgical intervention can be considered.Treatment of corneal perforationsCyanoacrylate tissue adhesiveLamellar keratoplastyPenetrating keratoplasty"
},
{
"id": 116316,
"tgt": "What causes low blood count?",
"src": "Patient: Hi I JUST RECEIVED THE RESULTS OF A COMPEHENSIVE BLOOD TEST AND MY DCTOR STATED MY BLOOD COUNT WAS LOW . I TAKE WARFARIN AND DIOXON FOR A-BIB ,HOWEVER I AM CONCERNED ABOUT THE LOW COUNT . DOCTOR WANTS ANOTHER BLOOD TEST IN ONE WEEK TO LOOK FOR ANY CHANGE. WHAT CAUSED THIS LOW COUNT . I NEVER HAD ANY PROBLEM PRIOR.THANK YOU Doctor: Hi,Thanks for asking.Based on your query, my opinion is as follows.1. Most common cause of low blood count is nutritional. Usually Vit B12 or folic acid deficiency.2. Full report necessary to give a complete opinion. One week is too short to notice any changes. At present, a reticulocyte count would be more accurate in evaluating bone marrow response.3. Continue therapy. Not to worry. If necessary bone marrow biopsy could be necessary at a later date for complete evaluation. At present nutritional improvement along with Vitamin therapy is essential.Hope it helps.Any further queries, happy to help again."
},
{
"id": 225672,
"tgt": "Have copper IUD. Have panic attacks, vertigo, ringing in ears, headaches, runny nose, heartburn. Reason?",
"src": "Patient: Hi there. I have had copper iud for 9 years altogether, last one has been in for 4. I have recently started to suffer with panic attacks out of the blue. I have had severe vertigo and a ringing in my ears for last 2 years. I am a very busy hairdresser and very sociable so as you can imagine it is hard work. I have been getting a lot of headaches and feel very low and 2 weeks before my period I get very tired- dizzy and very moody. Also seem to always have a runny nose and heartburn as well as constantly getting bv and other infections- nail infection- athletes foot and now experiency really itchy armpits- possibly yeast infection. I have been looking into copper toxicity and wandered if it sounds possible that this may be as a result of my coil? Doctor: Hello, It is very unlikely that your problems are because of your coil. Copper toxicity due to copper T IUCD has not been reported. The dose of copper in the coil is very small and it primarily is released into the uterus and acts locally to p roduce contraceptive effects. First, you need a thorough general checkup including an ENT check up. You also seem to be suffering from premenstrual syndrome which explains the problems you are facing before your periods. It is very important for you to have a check for Diabetes. Frequent infections may point out to undetected Diabetes. Fungal infections may be related to your occupation. Finally you may be stressed and you may have to consider to slow down a bit and find some relaxation techniques. Hope this helps."
},
{
"id": 178091,
"tgt": "Suggest medication for fever",
"src": "Patient: hello doctor my daughter is two years and nine months old. she is having fever occasionally for the past five days. my doctor advised mefanorm and clampkid antibiotic. when I give mefanorm five ml. her fever reduces. but after six hours she gets fever. so I give her mefanorm after seven hrs. even in the night time. this I contiued for four days. I had finished a bottle during this course. my doctor advised to monitor the temperature for two moredays. so that v can go for bloodtest. today I gave her around four o clock in the evening. now the time is 1.45a.m she is having 100.5.shall I give her mefanorm . Doctor: Thank you for the query.Kindly follow instructions of the doctor. Do not give mefanorm more than prescribed. At 100.5 you need not give any medicine. Fever is a form of body's defence mechanism too.At times a disease gets longer to heal.Kindly be patient and follow instructions. He may decide after results of investigations of may prescribe further investigations.This is a normal protocol."
},
{
"id": 140842,
"tgt": "What is the treatment for numbness in the hands and feet?",
"src": "Patient: I have numbness in my lips, tongue and slightly in my hands and feet. I have nausea and weakness. My doctor had me do a 3 hour glucose test and determined I am hypoglycemic. When I feel this way and check my blood sugar it is in a safe range. My problem is I have these symptoms from low 100 s at times, but very bad once in low 90 s and 80 s. It is not dropping below 70. Do I even need to worry about this and do I treat it the same as if it were lower? Doctor: Hi, If I were your physician I would do a good neurological examination of the face/neck/extremities to map out the exact areas of numbness and to see if there were any altered sensations. I would send you for a variety of labs having to do with thyroid, (FT4, TSH), Vitamin B12, methylmalonic acid, homocysteine, Folate, Vit. D, and not worry any further about blood sugars. Hope I have answered your query. Let me know if I can assist you further. Regards, Dr. Dariush Saghafi, Neurologist"
},
{
"id": 79109,
"tgt": "Cause for pain in the arm and chest?",
"src": "Patient: I had open heart surgery (orthoscopic) in April to remove a myxoma tumor from the left atrium. I had an appointment a week ago with my surgeon and he said everything looked good, but, for the last 3 days I've had periodic pain radiating down my left arm and very tight chest, pain with any deep breathing. I am over in Germany and wondering if I need to see a physician or if this is usual to the healing process.Thank you Doctor: Thanks for your question on Health Care Magic. I can understand your concern. In my opinion, you should definitely consult doctor and get done ecg and 2d echo to rule out heart damage. Your symptoms are commonly seen in heart damage and you are you just underwent heart surgery, so possibility of heart damage is more. So immediately consult physician and rule out this. If ecg and 2d echo are normal than no need to worry much. Normal healing process can also cause similar symptoms. So take painkiller and anti inflammatory drugs. But first rule out cardiac diseases.. Hope I have solved your query. Wish you good health. Thanks."
},
{
"id": 71671,
"tgt": "Why do fissures appear?",
"src": "Patient: last night i was in the hospital due to having chest pains and blacking out. they ran a few tests on me, and I got a call back this afternoon about a fissure appearing at the base of my heart. What do this mean exactly? they want me to come in next week so they can monitor me. Doctor: Hello dearWarm welcome to Healthcaremagic.comI have evaluated your query in details .* Usually this is normal physiological variant appearing at the broncho pulmonary segment in many cases .Hope this clears your doubt .Wishing you fine recovery .Welcome for any further assistance .Regards take care .Dr. Bhagyesh ( MS , FMAS - consultant surgeon )"
},
{
"id": 26871,
"tgt": "How serious are unfolding of aorta and mildly positive for myocardial ischaemia?",
"src": "Patient: i am 53 yrs old on health check up x-ray report indicates unfolding of aorta also treadmill test for 9 minutes shows mi;dly positive for myocardial ischaemia. how serious these findings are?what next investigations required? i am also taking treatment for gastriotis.pl advise Doctor: Hello, thanks for posting on HCM.Unfolding of aorta is xray finding which suggests that there might be aortic aneurysm. But nowadays with recent advances like 2D ECHO it can be easily picked up.Regarding positive treadmill test, it suggests there might be a chance of you having some blockage in artery(s) of your heart. Further workup will include a 2D ECHO study, if that also picks up some changes, your cardiologist might ask you to undergo an coronary angiogram which will visibly show any blockage if at all present and depending on the severity he might decide about doing angioplasty or not.I hope I answered your query well.Thank you"
},
{
"id": 132720,
"tgt": "What is the cause for leg strain?",
"src": "Patient: i am a 42 yrs old female that was diagnosed with polymalagia rheumatica ...i am taking 50 mg of prednisone daily what else can i do ... i also feel like the muscles in my legs strain today i pulled the leg muscle and it really hurts is this a symptom of the diseases ? YYYY@YYYY Doctor: Hi Hope this message finds you in good health.I have gone through your complaints and understand your concern.You seem to have strained your calf muscles which generally take around 3-4 weeks time to heel,if given adequate rest,The treatment you are taking is absolutely right,but i guess you are not taking sufficient rest.But still,it will eventually heal.Take analgesics,anti-inflammatory tablets,crepe bandage,rest.Nothing to worry about.I hope your question has been answered.If you have any follow-up queries,feel free to consult me anytime.Thanks,Take care,God bless."
},
{
"id": 120760,
"tgt": "What is the sensitive bump between the bunion bone and the heel?",
"src": "Patient: hello. while showering this A.M. I notices a dime sized soft lump or bump located on the inside of my right foot centered between the bunion bone and the heel, It is ever so slightly sensitive to the touch. You can press it in, not fluid-like, but soft. It almost looks like it could be a large bug bite. I have been wearing non supportive shoes lately. I am on my feet all day as a teacher. weigh 117 pds, 5 4 tall, 53 years old. I did walk a great deal this past weekend wearing good supportive sneakers. Do you have a clue? Doctor: Hello,The symptoms can be related to a ganglion cyst. Ganglion\u00a0cysts\u00a0are firm fluid-filled sacs that occur all over the body, but when they occur around the big toe joint they\u00a0can\u00a0be confused with a\u00a0bunion. I suggest to do an X-ray of the foot to exclude other possible causes. Ganglion cysts does not need treatment if they don't have any symptoms.Hope I have answered your question. Let me know if I can assist you further. Regards, Dr. Dorina Gurabardhi, General & Family Physician"
},
{
"id": 197972,
"tgt": "Can frequent masturbating causes any effect on ejaculation?",
"src": "Patient: Hi sir . i don't know how many times i did hand practice. now my penis at erection becomes like iron rod means too hard and also curved. does hand practice effects ejaculation time. i never fucked a girl i dont have idea about my time of releasing. i only did hand practice and my max time on it is 30 secs...plz tell my how can i increase my timings.. Doctor: Dear, We understand your concernsI went through your details. Frequent masturbation or masturbation habit do not have any adverse effect on your sexual health or ejaculation time or erection. Masturbation is normal and natural. Masturbation is never unhealthy. But you should make sure to reduce the frequency of masturbation to thrice or four times a week. Masturbation causes tiredness. The more tiredness, the more stressed and week you feel. These issues are temporary. If you take sufficient rest and fluid, the tiredness will go away.Penis too hard and curved is also normal and is good for perfect sexual intercourse. In view of all the above, you have nothing to worry about your sexual future. You will perform well. You may consult a psychologist or sexologist for counseling.If you require more of my help in this aspect, please use this URL. http://goo.gl/aYW2pR. Make sure that you include every minute detail possible. Hope this answers your query. Further clarifications are welcome.Good luck. Happy New Year. Take care."
},
{
"id": 3575,
"tgt": "How to get pregnant?",
"src": "Patient: About getting pregnantOk so my husband and i are trying to get pregnant we have been trying for 10 months and we still cant get pregnant. I have tryed the preseed lub hoping that it would help the situation which it didnt do anything ummm is there anything else i can do to try and get pregnant? Doctor: Hi,Welcome to Healthcare magic.I am Dr Ramadevi Wani. I will be answering your concerns.About 85% of couple conceive in the first year of marraige and 95% of couples conceive by the end of second year. So don't be stressed. Give time. You will get pregnant.It would have been nice if you had provided me some more information like your age, BMI, regularity of menstrual cycles, whether your hormone assay, semen analysis are normal. If it is bothering you first get baseline investigations like hormone assay, HSG, and semen analysis done. Based on the results, you can be treated.To increase your chances of conception maintain normal BMI, quit smoking ( if you're a smoker) and have intercourse at least three to four times in a week.Take a tablet of folic acid daily.If you have any further concerns do contact me through Healthcare Magic."
},
{
"id": 113147,
"tgt": "Swelling of stomach, hands, legs, feet. Stomachache, backpain. On antibiotics. Suggestions?",
"src": "Patient: Hello I am swelled up in my stomach like I am 9 months pregnant my legs feet and hands or swelled up to I am in a lot of pain in my stomach and lower back I constantly have to go to the restroom I have took Imodium ad but that don t help I went to the hospital and they said my small intestine was inflamed and gave me antibody but I in so much pain I feel like I am dying this is worst then labor I believe I am going to die plz help me I hurt so bad Doctor: Hi Such a pain is really worse and makes us feel very bad about it. For a immediately relief, I will suggest you to take Anti - Spasmodic medicines so that your pain in the abdomen gets relieved. I will suggest you to visit your local doctor or Homeopath to get immediate treatment. Further, I will suggest you to have a Complete Blood count (CBC), a Electrocardiogram (ECG) and a Ultrasound Sonography (USG) of your Abdomen and Pelvis. By these 3 investigations you will be able to know what is the underlying cause for the swelling of your feet, hands and hands with legs. Take Care With Warm Regards Dr. Apurva Tamhane"
},
{
"id": 90066,
"tgt": "Suggest medication for severe lower abdominal pain",
"src": "Patient: Moderate to severe lower left abdominal pain. Feels better temporarily when I have a bowel movement or urinate. Back ache improves then too. Wear a pessiray (sp?). Removed it this morning and some relief. Muscle from lower left back to front by pubic bone very tight causing hip pain. Not constipated but have constant bearing down feeling. Doctor: U might be having a kidney stone.or a urinary tract infection. Try having plenty of fluids. If possible get your urine tested for pus cells or crystals. If pain persist have cyclopam sos, urikind sos.It would Be better if u get a Ultrasound scan of your abdomen."
},
{
"id": 133437,
"tgt": "Suggest treatment for pain in bone",
"src": "Patient: I have really bad pain in a bone in my left hand....google search says it s the pisiform bone. Is there a way I can treat this myself? I don t have a doctor and it costs me a lot to go to one out of my insurance group. I m 70, don t have arthritis and haven t hit my hand to break anything. It s been on ongoing thing for several weeks now. Thank you. Doctor: Go for ice pack and have wrist mulligan mobilization and wrist strengthening excs. U can have checkup with physiotherapists in 1nce a week."
},
{
"id": 161018,
"tgt": "Suggest medication for loose motions in an breastfed infant",
"src": "Patient: Name: vrushali majage Date: 2010-29-10 My daughter who is 43.5months is having loose motions for last 2 days. She has been going for motions 6-7 times a day. Doctor has prescribed walamycin, vitazume drops, ( 3 times a day). She chuing her hand and cloths all time. She is completely on mother s feed. we complete 3 day course of this medicines. when we stop giving medicine , after 2 days again loose motion start. Best Regards, vrushali. Doctor: Hi, You mentioned 43.5 months - which is almost 4 years. It seems your kid is having viral diarrhea. Once it starts it will take 5-7 days to completely get better. Unless the kid's having low urine output or very dull or excessively sleepy or blood in motion or green bilious vomiting. You need not worry. There is no need to use antibiotics unless there is blood in the motion. Antibiotics might worsen if unnecessarily used causing antibiotic-associated diarrhoea. I suggest you use zinc supplements (Z&D drops 1ml once daily for 14 days) & ORS (Each small packet mixed in 200ml of potable water and keep giving sip by sip) as hydration is very important and crucial part of treatment. If there is vomiting you can use Syrup Ondansetron (as prescribed by your paediatrician). Regarding diet - You can use cerelac. Any flavour will do. Avoid fruit juices as they might aggravate diarrhea. You can give zinc supplements & ORS apart from normal vegetarian porridges & soups. Hope I have answered your query. Let me know if I can assist you further. Take care Regards, Dr. Sumanth Amperayani, Pediatrician, Pulmonology"
},
{
"id": 134273,
"tgt": "What causes severe bone pain and bone loss?",
"src": "Patient: I have had severe bone pain for 4-5 years, worsening as time passes. My Vitamin D level has been low at 15 but with supplement at 50 units once a week has recently come up to 64. I have had multiple blood tests that are normal. I have had a bone density test and been told that I have osteoporosis. My bones feel like they are breaking, especially my long bones. I have significant bone loss in my shoulder. I am very concerned, but my doctors just want to tell me that I have fibromyalgia. I do not believe that this is the correct diagnosis. What do you think? I am very concerned! Doctor: hi,thanks for the brief history of you. since you been diagnosed with osteoporosis and vitamin D deficiency let's focus on this rather going straight to fibromyalgia.let's assume that you have been into medication I am sure they are helping. now let's look the other way. due to the ageing of tissues the metabolism of the tissue functioning is reduced which has to be boosted , how? try doing regular exercises or under a guidance of a physical therapist. You should take more of the calcium through diet and vitamin D though sun. along with it, regular weight bearing exercises will help the calcium deposition consumed in the form of medicine and diet. also the vitamin D is getting absorb as you are taking sunbaths and also the metabolism will improve by regular exercises. talking about the pain and aches you feeling will also lower down as EXERCISES will lead to benefit in the form of improving metabolism and boost the immune system.talking about fibromyalgia is to be under stood in this way. as the patient tells the symptoms the doctor comes close to diagnosis.I am sure with regular exercises you will be better soon.with the grace of God I wish you a speedy recovery"
},
{
"id": 170248,
"tgt": "What causes congestion, cold and running nose in a 1 year old?",
"src": "Patient: My baby just turned one year old last week. Around then she started getting sick. She has a cough, a running nose (she woke up yesterday morning with her nose glued shut from snot). Today, when she breathes I can really hear her breathing from congestion. She s been really fussy and refusing food more than usual (we still get her to eat, she s just a bit fussy about it). Her grandma pointed out what looks like a rash on her forehead but it faded yesterday it seems like. Today especially she s been gassy and has had a slight fever off and on. On the 7th we introduced her to whole milk so maybe it could be an allergy? Doctor: HiWelcome to the HCMI understand your concerns but don't worry. Your baby is most likely suffering from viral infection which are self limiting and common in this age group. She needs symptomatic treatment. Give antihistamine such as Chlorpheniramine or fexofenadine for cough and congestion. Give acetaminophen for fever episodes if they occur. Also, start a zinc supplement such as Syp ZnD or Zinconia and probiotic such as enterogermina. Ensure regular feeds to avoid dehydration.Hopefully it will be of help.Take care"
},
{
"id": 207379,
"tgt": "What is the treatment for anxiety , dizziness and sweating?",
"src": "Patient: My doctor precribed me effexor for my anxiety and it was causing my heart to flutter and I was in the hospital for it. I stopped cold turkey taking it and now I am having constant dizziness feeling, sweating, and other things. Is this withdrawal and how long will it last? Doctor: DearWe understand your concernsI went through your details. I suggest you not to worry much. If the problem detected was anxiety and the diagnosis was correct, then the given medication should be suitable. You must also understand that Many researches and researchers confirm that medicines alone cannot cure mental disorders. Life style changes, change in thinking pattern, relaxation etc are as essential as medicines. Psychotherapy can help you changing your lifestyle and thinking patterns. Yoga and meditation help you to streamline your metabolism and neurological balance. Please consult a psychologist for further information.If you require more of my help in this aspect, Please post a direct question to me in this URL. http://goo.gl/aYW2pR. Make sure that you include every minute details possible. I shall prescribe the needed psychotherapy techniques.Hope this answers your query. Available for further clarifications.Good luck."
},
{
"id": 202005,
"tgt": "Suggest treatment after analyzing the semen analysis report",
"src": "Patient: sir, i'm male /age37,height 5'6'',wght 66kgs,in my semen analysis found sperm count 45mill/mil,morphology 70% normal,motility 25%& pus cell++,I have no any other disease . what type of treatment or medicine(alternative) should I take? thanks nasir,Bangladesh. Doctor: Hello Thanks for writing to usYour semen analysis report suggests infection with decreased sperm motility.Infection is indicated by the presence of pus cells in semen. Normally there shouldn't be any pus cells in semen.Infection may be due to prostatitis,UTI.You need antibiotics for infection.You need few more investigations like routine hemogram,RBS,Ultrasound of pelvis should be done.Trans rectal sonography of prostate can be done if needed.Proper antibiotics should be prescribed depending upon sensitivity report.You also have low sperm motility.Sperm motility should be at least 55 %.Fertility will definitely improve with control of infection.You should do exercise and take nutritious food.You should avoid tight underpants.Avoid cigarettes and alcohol if you take these things.Get well soon. Take Care Dr.Indu Bhushan"
},
{
"id": 115209,
"tgt": "What causes discoloration of fingers to blue?",
"src": "Patient: Our 15-yr-old granddaughter's fingers turn blue at different times for no apparent reason. No numbness, they just feel weak when it happens. It's not when she's cold. What would cause such a thing? (It's not her feet...just fingers and part of her hands)We have asked the pediatrician and other doctors...even her sports therapist (she's a cheer-leader). Thanks Doctor: Hi, dearI have gone through your question. I can understand your concern. She may have cyanosis. It may be due to malformations of some vessels or cardiac problems. You should consult physician for examination. If needed go for echo cardiogram. Then start treatment accordingly. Hope I have answered your question, if you have doubt then I will be happy to answer. Thanks for using health care magic. Wish you a very good health."
},
{
"id": 214423,
"tgt": "What is adiposis dolorosa aka dercum's disease?",
"src": "Patient: hi i'm looking for information on adiposis dolorosa aka dercum's disease. my mother has chronic pain for years and was finally diagnosed last year. but we haven't been able to get information on treatments, pills are NOT the answer. what can you suggest? Doctor: Thanks for contacting HCMI am sorry to hear about your mother and her recent diagnosis of adiposis dolorosa. This is a disease where fatty tumors or lipomas cause pain. There are little evidence for treatment of these painful lesions. Several trials on removal of painful lipomas, traditional anti-inflammatories and steroids. There has been some improvement by injecting the painful lipomas with lidocaine which may help from 10 hours to several months. I wish your mother luck in finding relief from her adiposis dolorosaHope this answers your question. Please contact us again if you have questions or need help."
},
{
"id": 73192,
"tgt": "What causes constant cough and abdominal pain?",
"src": "Patient: I have two unrelated problems: I have been coughing for over 2 years, its productive and the mucus is clear and sticky. It sounds bronchial however I am negative for asthma, have a clear chest xray and have many airborne allergies. i have tried ventalin,flovent,flonase to no improvement. My blood pressure is elevated from the constant coughing and I find it sometimes difficult to breathe or go long periods of time without coughing as I feel as if there is a constant pool of fluid in my throat. I also have frequent headaches that I believe to be related to this.And my second problem, unrelated is regarding a sharp pain i've recenntly developped in my left lower abdominal region. I'd guess around my ovaries or kidneys. It is only present during sexual intercourse or after achieving climaxes. It is pretty recent and has been since the start of my last menstrual cycle about three weeks ago. Doctor: Hello Thank you for trusting HCM Any cough more than 8 weeks called chronic cough May be your suffering from chronic allergic bronchitis :in this condition airwaves inflamed due to certain air born allergen produce cough. Other possible causes 1.post nasal drip2.GERD with laryngopharyngeal reflux 3 . cough variant of bronchial asthma4.drugs like ace inhibitor and etc. 5 . infections 6.occupational lung disease Treatment :avoid allergen, avoid cool drinks, tab. Moterleukast twice a day may help Post intercourse pain may be due to Pid or dryness Or simple muscle pain. Please consult your doctor if symptoms not improved."
},
{
"id": 168296,
"tgt": "Suggest treatment for inability to speak in child",
"src": "Patient: .hi doctor. my nefew is only 15 months and he has a delevery problem. actually his brain is have problem. we did c.t scan. and doctor said he can not speak all life.but on ther hand a militery doctor said he will speak after 2 years. we are very upset for him. plz tell us what we should do. Doctor: Hi....I suggest you do a hearing assessment for him and if the ears are normal then go ahead with speech therapy. Many times for speech impairment the most common reason will be hearing impairment.Regards - Dr. Sumanth"
},
{
"id": 149683,
"tgt": "Have brain lesions. Slurring words, memory loss, twitching hands, dizziness, headache, nausea, vomiting. Your opinion?",
"src": "Patient: Hello, Symptoms- slurring words, memory loss, twitching hands and legs, dizziness, headaches every day, migraines, sore neck, nausea, vomiting, feel cold water dripping on my arm and get burning sensations on my skin on my leg. I have 3 lesions on my brain. 6 years ago I had a stroke or virus in the lining of my brain was never given a definite answer. What is your opinion I am at the end of my rope. With never getting a answer for all my symptoms that have been getting worse and worse. I am a 37 year old female I have had left ventricle surgery and my gallbladder removed. I have had 2 unexplained viruses as a youth. I also have been diagnosed with Ellers Danlose syndrome. That is me in a nutshell. Thank you for any suggestions. Doctor: Hello,First of all I would advice that never feel you are at the end of rope. You have few diagnosed and a few undiagnosed problems. For your symptoms you may need a nerve conduction study to find out whether the abnormal sensations on your limbs are due to nerve involvement. Secondly the location of the lesions in your brain will also explain some of the symptoms. In certain situations it may be better to aim for the solution (treatment) rather then chasing the diagnosis. Neurologists can definitely prescribe you medications to relieve you of these symptoms.Good luck."
},
{
"id": 187063,
"tgt": "Why having numbness on face and tingling sensation in lip due to mouth sores?",
"src": "Patient: Hi I have had mouth problems for the last 9 weeks or so starting with a numbness on one side of my face,tingling sensation and burning of half of my top lip on the same side of the numbness, I later found mouth sores or ulcers and the roof / palet of my mouth felt spongy and unpleasant again on the one side I have had to take pain killers and mouth wash every few hours for the first 3 or 4 weeks.The pain has now gone and the numbness reduced but still present.My palet seems better and I am not in any pain.23 years ago I had a cancerous tumour ( a lymphoma) behind my left eye. This was succesfully treated with radio therapy but leaves me more anxious that this vould be cancer.I have seen the dentist had an ex ray and no problems with my dentistry.I have been to the hospital had blood test and biopsy of ulcer. Both showed no adnormalities.The ct scan showed something possible suggestion of synositus but this is on the opposite side of my face.I have just had an MRI scan and waiting for results.Could this be cancer and can it be treated.Do you have any thoughts or experience of anything else it could be Doctor: hello thanks for consulting at hcm..it could be due to anaemia which results in tingling and numbness,,kindly have a consultation with oral medicine/general physician..for check up and treatment..since dental and systemic prblms r not present..hope it helps.tc"
},
{
"id": 5953,
"tgt": "Had two miscarriages. Is it ok to take progesteron tablets while I've conceived ?",
"src": "Patient: Hi doctor, i have had miscarriage twice very early in my pregnancy 5/6 weeks.here in uk they dont do any test until it happens 3 times. but i want to avoid it next time .I might be pregnant now as we were trying to conceive this month and my period is due next week. After reading loads of blogs and forum , i heard may be it is caused by low progesteron. Do you think its ok to take progesteron tablet if i find out im pregnant without consulting doctor as doctor in this country doesnt do anything until it happens 3 times. If there is no side effect , can i not take them ? please advice . Kind Regards Doctor: Hi, Thanks for the query. Miscarriage in early weeks of pregnancy can happen due to many causes and progesterone deficiency is one of them. But there are many other causes like chromosomal problems, structural defects etc. So just by supplying progesterone we cannot prevent miscarriage due to other causes. But it will help in case of progesterone deficiency. So you can use them, prefer natural micronized progesterone as much as possible. Take care."
},
{
"id": 6859,
"tgt": "Why am I unable to conceive after an abortion ?",
"src": "Patient: hello doctor , i am 26, i aborted (40 days baby) my 1st child before 3 years.after that i am not getting preganent. last month i took clomefine tablet.i want have a baby please give me answer.is it possible for me to get preganent.if it forms.please answer. Doctor: Hi Kavitha, Welcome to HealthcareMagic Forum. Yes, you can get Pregnant, Kindly don't lose hope. You are not the only one to experience this. You will conceive if your Periods are regular which means you are ovulating. Have you gained weight recently? If yes, you may want to exercise regularly and go for regular brisk walks for 30 to 40 Minutes daily. Avoid Junk and High calorie foods. Eat healthy Nutritious balanced diet which should include fresh fruits and Green Leafy Vegetables. To increase the chances of conception, have alternate day Unprotected sex, starting from 10th to 20th day of your cycle. Take your Medications regularly and go for regular follow up. All the Very Best, Take Care."
},
{
"id": 121463,
"tgt": "Suggest treatment for hematoma in buttock and shortness of breath",
"src": "Patient: 11/3/11, fell down 1/2 flight of steps on right buttock. I was in shock and had pain up to my right chest area around ribs. Taken by ambulance to hospital. CT Scans/xrays showed to breaks or cracks. Developed grapefruit size hematoma in right buttock. Two months later still have shortness of breath and life-stopping pain in right chest. Doctor: Hello,A chest X-ray study is necessary, coupled with a thoracic spine X-ray study, in order to exclude a possible bulging disc in this level, caused by the trauma. You should discuss with your doctor on the above tests.Hope I have answered your query. Let me know if I can assist you further. Regards, Dr. Ilir Sharka, Cardiologist"
},
{
"id": 116408,
"tgt": "Can stent in the veins cause blood spitting or stopping tablets for heart bypass?",
"src": "Patient: my husband has had a by pass 10 years ago in 3 major veins he also had a stent put in one of his veins lately. HE ALSO HAS DIABETES.He stopped all his heart pills except those of diabetes.He is now spitting blood especially in the morning. Do you think that this is because of the stent? Thank you Doctor: Hi, dear. I have gone through your question. I can understand your concern. Spitting of blood may be due to many reasons. He may have cirrhosis of liver or some problems in lung. You should go for ultrasound abdomen and x ray chest. Search the cause and take treatment accordingly. Its not due to stent. Hope I have answered your question, if you have doubt then I will be happy to answer. Thanks for using health care magic. Wish you a very good health."
},
{
"id": 6593,
"tgt": "What are the chances to conceive after breaking of a condom while having sex and later taking birth control pills ?",
"src": "Patient: hi, i was just wondering what the chances were of becoming pregnate if a condom breaks and you take birthcontrol every night and have never missed a day?.... thanks:) Doctor: hello welcome to healthcare magic. if you have been taking birhtcontrol pills regularly without even missing a single dose, then the chances of you becoming pregnant are almost nill. which phase of your cycle are you in, as in what day of your cycle? a lot depends on that also"
},
{
"id": 161295,
"tgt": "What causes vomiting and drowsiness in a child after having programmable shunt?",
"src": "Patient: i have a 2n half yr daughter with hydrocephalus who got a programmable vp shunt at 5 months of age , past 4 mths she has had 3-4 days phases of vomitting and drowsiness , her ctscan n mri seem to be fine n the doctors hve tried first reducing n the increasing the pressure to 90 . need to know what could be troubling her. Doctor: Hello, Is vomiting early in the morning? If so it could indicate an increase in intracranial pressure. Please get a ct redone and check for the tube block. Hope I have answered your query. Let me know if I can assist you further. Take care Regards, Dr Prasanna Lakshmi, Pediatrician"
},
{
"id": 92923,
"tgt": "Steady pain at waistline, no digestive problems. Cause of pain?",
"src": "Patient: OK. I'm trying to find the possible source of pain at my waistline (in the front only) and directly above it - like the whole midriff area. It's been going on for a couple weeks now. It's doesn't move around and is always steadily there whether I eat or not. I don't usually have digestive problems, so it puzzles me. I don't seem to have any other symptoms. Doctor: need to investigate for inflamation of bladder spleen or other pelvic organscan confirm by getting ultrasound abdomen and know cause to treat it"
},
{
"id": 152441,
"tgt": "Is another urinalysis test required to confirm a cystocele and rectocele when diagnosed with squamous cell carcinoma of the esophagus?",
"src": "Patient: I recently had a urinalysis test and the results showed 7 HPF squamous Epithelial and few bacteria. I have a cystocele and rectocele and am waiting for surgery to correct. Also have had squamous cell carcinoma of esophagus (43 years ago) radiation only and Malt Lymphoma of stomach. Should I request another urinalysis test? Doctor: Hello, There is nothing much to worry about it. Increased epithelial cells may be due to a cystocele. You can go for a cystoscopy to get a better clinical picture. A urologist can help you further. Hope I have answered your query. Let me know if I can assist you further. Regards, Dr. Shinas Hussain, General & Family Physician"
},
{
"id": 9114,
"tgt": "How to get rid of chicken pox scar on face and what could be the cost?",
"src": "Patient: I am a 40 year old female living in Chennai. I had a severe attack of chicken pox 3 weeks back and have recovered now. I now have more than 50 dark spots on my face alone, and have about 200 dark spots all over my body. These dark spots are are not deep. As suggested by a doctor on this site I am using Kojivit cream and Sun Cross UVA cream for the past one week. I am indeed worried whether these dark sports would go off and wonder whether I should go in for any laser treatment for removing these spots on my face, at this stage. Kindly advise me on this, and if you would suggest laser treatment, please also suggest a good clinic providing this treatment in Chennai, how long will it take for a complete cure and the approximate cost for the same. Doctor: Hi!vidya v, Welcome to healthcareMagic forum, Since its only 3wks that you had chicken pox,you should let all the lesions completely heal .At this moment you should only use a sunscreen daily esp for face.You can continue kojivit if you want.Don't peel off the skin from the lesions ,let them heal on its own.As far as laser are concerned,you should wait for at least a month or two more.If you see any depressed lesions on face then you can go in for lasers.Till then you should take all precautions from direct sunlight.If you land up with depressed lesions on face then you can go for fractional laser either co2 or erbium laser.You might require 3-4 sessions of the same.Cost depends upon the number of lesions you have. take care, Dr.chawda"
},
{
"id": 42888,
"tgt": "Does the lower back ache after taking fertility pills?",
"src": "Patient: Hi.am 26 and hav be trying for 2yrs now to conceive but 2no avail..and after taking some fertility pills seems as if it comes and goes...but my problem now is hav seen my period full last month but since then am having serious lower back ach..will it be pregnancy.pls help Doctor: HAIWELCOME TO HCM,you need pelvic and abdominal scan if you urine pregnancy test is negative.low back ache produced by bone and ligament diseases.if you have persisting pain consult your gynaecologist."
},
{
"id": 114166,
"tgt": "Can back pain be a symptoms of horseshoe kidney ?",
"src": "Patient: I am 33 and just told I have a horseshoe kidney I went to doctor with severe back pain I am 33 and just told I have a horseshoe kidney I went to doctor with severe back pain I always have back pain but this was 10 times worse the acute care doc thought it might be a kidney stone even though CTscan didn t show anything she said blood showed up in my urine sample sent me home with pain pills and strainer and had me make appt with my regular doc the next day my reg doc said she thought it was more just back related as I am overweight and she said no blood really showed up in urine sample even though other doc said it did? She is the one that told me about my horseshoe kidney she ordered xrays of the back it showed calcification of the ligament and narrowing of the spinal canal then scheduled me for an MRI a couple weeks later and put me on light duty at work the MRI didn t show a whole lot and it came back as no narrowing of the spine?? Along with my back pain I sometimes get pain in my left leg and like a heaviness or numbness in it even though MRI didn t show any nerve pinching?? Back pain also seems to radiate into my sides and low abdomen this has been going on for years oh and left ankle always swells I am doing physical therapy for my back but so far its not helping I am starting to feel like I m crazy any suggestions on what s going on? Could the horseshoe kidney cause any of this? I also have PCOS(polycystic ovarian syndrome) any information would be appreciated! Doctor: Hi, Welcome to Healthcare Magic Forum. Horseshoe Kidney is a Congenital Urologic Condition which is the most common type of Kidney fusion anomaly and is usually found as a Incidental finding. Usually it is symptomless and in rare cases it causes abdominal pain and distension as a result of urinary obstruction, vomiting. It is a predisposing factor for formation of Kidney stones, which may be the cause of back pain in your case along with spinal muscle spasm, if at all stones are not present. You said you are overweight and pain radiates to your sides and legs, it may be related to Spinal Arthritis owing to your excess weight. Intravenous pyelography CT scanning is the best initial radiologic study to determine anatomy and relative renal function. You need a thorough work up and Physical evaluation by both Orthopedist and a Urologist. Good Luck and Take Care."
},
{
"id": 159553,
"tgt": "History of sunburns and acne. Does sebaceous hyperplasia lead to cancer? Treatment?",
"src": "Patient: Female. Age 52. Fair skin. History of sunburns, and acne . Family history of skin cancer (basal and mm that metastasized to brain) Went to dermatologist for moles on my face that are increasing in size and number. He said it would be expensive, cosmetic (not covered by ins)and would require stitches on each incision to close and each of the ten would be sent to pathology at a whopping $1000 additional cost. He said Scarring would be more unsightly than mole. I left dissatisfied. He never mentioned sebaceous hyperplasia . But now I think that is my condition. My questions? Does sebaceous hyperplasia lead to cancer? (squamous is what I read) Will moderate to deep peels help to keep the gland and cell growth at bay? What other treatments should I investigate? Laser, electro, needles? I would like a treatment (diagnosis) that my insurance would cover. Doctor: Hi, Welcome to HCM. Firstly, a sebaceous hyperplasia is increased number of cells that forms hair follicle glands. It does not lead to squamous cell carcinoma rather it leads to sebaceous gland carcinoma . Now, a Mole is proliferation of melanin containing cells which may lead to melanoma.It is a devastating disease. So, dont take it lightly, it does not matter what is the cost of surgery, whether there will be any scarring or not, whether it will be covered by insurance ...... Just GET RID of that. Thats all."
},
{
"id": 209418,
"tgt": "What treatment is suggested for Schzophrenia?",
"src": "Patient: y son, 42 yrs, has schzophrenia. He has been prescribed Mirtazapine 300mg, diazepem 5 mg, serequil XR and was on aAdvancer slow elease (I think ). as well his psychiatrist gives him scriptsss for valium which he takes from time to time. Drug induced pyschosis and he says he never gets a full nights sleep, just wants to zonk out for some hours. Then I hear him say he had morph tablet last week. A tablet apparently. Should I just contact his psychiatrist as I m not the patien and don t know how to help. Doctor: Hi,Thanks for writingYour son will benefit from a detailed session with his psychiatrist. The medicine regimen may be adjusted based on current symptoms, and he may also be helped with his habit of doing unprescribed drugs (morphine).Hope that helps,Dr A Rao"
},
{
"id": 186103,
"tgt": "Is bleeding in gum symptom of pyria?",
"src": "Patient: hi, my gums are very weak its bleeds and even slight touch to my gums its bleeding and they are separated from my teeth and due to blooding its giving bloody test and bad breath and again due to lose gums my teeths are paining a lot so is it symtom of pyria Doctor: Hi! Welcome to Healthcaremagic.I read your query. Gum bleeding is the first sign of gum disease. However, detachment of gums from teeth occur at later stages of disease which may involve heavy bleeding, bad breath,pain, etc.I suggest you visit a Periodontist and get your gums checked. You will be required to go multiple times. You will be given oral prophylaxis and curettage if required.Usually with proper cleaning, medicine, maintainence of hygiene and nutrition, the gums start to re-attach with in 2-3 months. Else further treatment like flap surgery may be required.Along with treatment, you need to maintain good hygiene. Brushing twice daily with soft tooth brush, mouthwash, warm saline rinses, gum massage, proper diet, multivitamins. You may be prescribed antibiotics and anti inflammatories.Take lots of fluid, fruits, vegetables like spinach, carrot, lemon, etc.Hope the answer helps you. Thank you."
},
{
"id": 6592,
"tgt": "Is there chance to conceive if touched a penis and touched the vagina ?",
"src": "Patient: what if you touched your Bf s penis and he had a pre-cum.. then afterwards you peed and touched your part down there to wash it.. could you get pregnant? Doctor: Hello. Thanks for choosing HealthcareMagic forum. You are not likely to get pregnant. You need not be so panicky.For pregnancy to happen your boy friend must penetrate inside you and has to leave the sperms directly inside the vagina and that too preferably deep inside you. Do not worry now. Dr. Rakhi Tayal drrakhitayal@gmail.com"
},
{
"id": 120205,
"tgt": "Suggest opinion about realignment surgery on finger",
"src": "Patient: Hello, i went to the Er in my area the other day and found out that i broke the very tip of my pinky finger right on the first joint. They told me that i may have to have surgery to realign the joint. How likely is it that i definitely will have to have surgery on my pinky. Doctor: Hello, It will be a minor surgery like K wire insertion, surgery will be a best option. Hope I have answered your query. Let me know if I can assist you further. Take care Regards, Dr. Shinas Hussain"
},
{
"id": 121713,
"tgt": "What could it be if having swelling in the outside of knee which grows when running, sitting?",
"src": "Patient: I have an odd swelling in the outside of my left knee. It tends to get larger when i run, but it does not really hurt. It can often swell just while sitting at my desk or at a restaurant for a couple of hours - when I get up it feels really tight tna there is a bit of soreness. When I reach down, I can feel a larger bump than usual. If I sit and cross my feet so my left and right feet cross under the chair the bump goes away almost completely. I have been told it could be an aggrivated IT band or hamstring but no one has been able to provide a firm diagnosis and hence no firm treatment except expensive Chiro that does not seem to be helping. Thanks, PDC Doctor: Hello,The swelling on the outside of the knee that grows on sitting can be due to an inflamed bursa. You need to get an MRI scan of the knee done for a proper diagnosis.Hope I have answered your query. Let me know if I can assist you further. Regards, Dr. Praveen Tayal, Orthopedic Surgeon"
},
{
"id": 97202,
"tgt": "What to do as the babies have swallowed 4 to 5 birth control pills?",
"src": "Patient: Hi , I am Javid Hussain, it is too panic situation for me that my two nephews babyboy 2 years and babygirl 2 years has swollowed 4 to 5 birth conrol pills each in one time( birth controle pills available in 30 tablets strip for one month) sir i a seeking an urjent answer to go through proper medical aid for children. Doctor: the birth control contains hormones,it is advisable to take them to the ER the emergency room and they will take care of it.I nshalah they will be fine."
},
{
"id": 132555,
"tgt": "What is the treatment for calcium buildup in shoulder?",
"src": "Patient: my friend has a calcium buildup in his shoulder. And he is hurting really bad. He has been to a doctor and they gave him a shot in the shoulder. It eased up, but now it has started back to hurting again. What can be done to stop the pain. And it is also swallon. Doctor: Hello. If your friend hasn't had an mri scan of the shoulder he needs one to check for tendonitis or a tear of a rotator cuff tendon and he may need arthroscopic surgery to repair the problem he's having."
},
{
"id": 10988,
"tgt": "Suggest medication for hair loss",
"src": "Patient: I have been losing hair on and off for 5 years. The condition was more severe 5 years ago where I lost about 60% of my hair. It did grow back back not as full and thick as before and ever since, I've been finding I lose twice as much hair than expected when I wash my hair. Prior to this - I would say about 6 years ago I lost my father to cancer. I wash my hair every other day. I am 40 years of age. Do you have any idea what my condition could be? Doctor: My DearWelcome to Healthcaremagic!I have noted your concerns and find that your history and details closely match that of Telogen Effluvium, a diffuse condition of scalp hair loss in which multiple factors might be playing an important part and where the symptoms may come and go just as in your case.The condition can follow stressful factors like it started with the tragic loss of your father.my patients recover from TE with following measures.1 Use of comb or brush frequently but without vigorous force being applied2 balanced diet rich in fresh fruits and vegetables3 Biotin tablets4 Stimulants for hair roots like Folica solution or cantharidine oil5 Treating the hairs gently and avoiding use of chemicals on the scalp6 Stress relief by meditation and engaging in hobbiesI am sure that with some of these measures suitable to your temperament, you will be benefited to a good measure.Best wishes!"
},
{
"id": 99970,
"tgt": "Suggest treatment for cough and vomiting due to allergy",
"src": "Patient: My husband is 58 years old. He just found out that he has allergies. He has been having this bad cough that brings water to his eyes. He coughs so bad that he vomits. He says he has this tickling in the back of his throat and that this is why he coughs like thatThe doctor keeps giving him cough syrup. He says that he doesn't know why my husband is coughing like that. Can you give me any idea of what it may be? Someone suggested that he go to a pulmonologist. Doctor: HelloYour husband is having very severe cough , so much so that even ,he vomits and also tears in the eyes. He is also complaining that there is tickling in the throat , he is right , actually this is also due to some sort of allergy .Such cough may be due to these possibilities , these include;1 asthmatic allergic bronchitis , due to some allergen there develop bronchoconstriction , this produces cough . Dust , mite , pollen , pests , smoking are few predisposing factors of such cough . When such patient visit I advise them to take montelukast+ bambuterol tablet according to severity of symptoms .Dextromethophan tablet chew will relieve ticking sensation in throat .2 Sometimes , such sudden provoked cough may be due to some inhaled particle in airways ( in lungs ) .3 Septic pharyngitis with allergy may be the next reason of such cough .So deal as mentioned above and take a prescription for above mentioned drug. However , it is very difficult to establish any diagnosis merely with these findings , so consult a physician and get his opinion."
},
{
"id": 161747,
"tgt": "What causes cold , cough and fever in a child?",
"src": "Patient: Hi my son (2years and 7 months old)is having cold and mild cough&fever (99).doctor told he is having mild vizzing.doctor listed some 5 medicines.crocin Ds for fever,cosome ,betnesol(tab),bricarex,bromolin DT(tab). i just started all the medicines,now i read some review about cosome and scared.which are the medicines i can follow?plz can u guide me Doctor: Hi Dear,Understanding your concern. As per your query your child have symptoms of cold, cough and fever which seems to be due to upper respiratory tract infection or poor immunity of body. Need not to worry. Avoid intake of cold carbonated beverages. Avoid intake of sharp spicy food. You should apply mist humidifiers in room and take throat lozenges. Visit ENT specialist once and get it examined and start treatment after proper prescription. You can go for blood and sputum tests as well. You should take proper course of antibiotics along with anti inflammatory medications. You should maintain oral hygiene. Do warm saline gargles. Hope your concern has been resolved.Best Wishes,Dr. Harry Maheshwari"
},
{
"id": 148280,
"tgt": "Blackouts after getting up from bed/couch followed by a fall and inability to remember whatever happened",
"src": "Patient: Sometimes after I get up from bed or couch , my vision goes black and i cant control my muscles and it feels like my body would shake :/ Most of the times I fall on the ground and I have hit my head many times because of these . I have had these many times for 2-3 years . Im 16 years old male . And i cant hear anything when i blackout and it feels like my mind is somewhere else and i get flashback memories when im having these seizures but afterwards i cant remember what i saw :/ i weight 65kg and i am 180cm tall Doctor: Dear User,Thanks for posting a query at healthcare magic.From your description it appears hat you are suffering from syncope.The other possibility is dissociative disorder and epilepsy.You need to contact a physician with full details to ascertain cause of your syncope. If you never injured yourself while falling and every episode occurred in presence of others and never in alone, Than you can meet a psychiatrist as soon as possible.Thanks"
},
{
"id": 166603,
"tgt": "What causes redness in eye area with itchy and blistered skin?",
"src": "Patient: My 8 year old son suffers from allergies. He gets severe reddness under both eyes and across the bridge of his nose. His skin looks blistery in the red areas. The skin is bright red like the color of a sun burn. He says in only itches now and then. The rest of his face in normal colored. Please advise. Doctor: Hi...Thank you for consulting in Health Care magic.By what you quote it should be an urticarial or a simple skin allergy. You can use Hydroxyzine at 1-2mg/kg/dose every 6th to 8th hourly for 7-10 days. Most important thing to be remembered is that it has a propensity to recur (called as second crop) within 10-14 days. If this happens, you can start using the same medicine but I suggest you get the kid evaluated with your paediatrician.Hope my answer was helpful for you. I am happy to help any time. Further clarifications and consultations on Health care magic are welcome. If you do not have any clarifications, you can close the discussion and rate the answer. Wish your kid good health.Dr. Sumanth MBBS., DCH., DNB (Paed).,"
},
{
"id": 215828,
"tgt": "What causes pain in lower right side of stomach and below the hip?",
"src": "Patient: i am having pain in my lower right side next to my stomach and right below my hip it started today about 7 hours ago and started on the right side of my stomach it has now become worse and has moved in the direction I just described dose this sound like the appendex to you Doctor: Hi, It can be an appendicitis. Consult a general surgeon immediately and get an ultrasound scan to rule out appendicitis. As of now, you can take analgesics like Acetaminophen for pain relief. Hope I have answered your query. Let me know if I can assist you further."
},
{
"id": 183118,
"tgt": "Suggest remedy for tooth abrasions with bleeding",
"src": "Patient: my bottom left k9 tooth was crooked and now seems to being pushed further into the bottom of my mouth, under my tongue its very painful and bleeds a lot, my wisdom teeth havn't broken skin but are starting to come in. what do i need to do about this tooth? Doctor: HelloPlease mention your age. Wisdom tooth can take some time for eruption.In some cases before complete eruption gums over the tooth becomrs infected and shows the symptoms like swelling bleeding and difficulty in mouth opening. this condition is called pericoronitis.I would suggest you to gargle with lukeworm saline water.If the problem persist then you should visit a dentist.they can take a xray to see whether the tooth is coming straight or not.RegardsDr shesh"
},
{
"id": 47776,
"tgt": "What causes enlarged kidney?",
"src": "Patient: Hello. Roughly 3 years ago I was in an accident and had a battery of tests done at the hospital. The only thing that was said is that on my x-ray it showed an enlarged kidney, but that was told to me by the doctor in passing as I was laying on the gerny in the hallway. Ive had blood work (yearly) and urine tests since then for physicals and nothing out of the ordinary has ever been mentioned. Im at a point in my life where Im fully enjoying it and want to get it checked out so I had an ultrasound earlier this year. Can there be anything that could come of this if it was years ago it was mentioned an nothing has shown up on recent tests. On the ultrasound (done in January 2014) nothing was shown either. And I specifically asked about it being enlarged. At times I get sore in my back side by kidneys and start to worry. Maybe just being nervous and I don t want anything to happen to me for a long long time. Doctor: Hi,Thanks for writing in.A normal kidney size measures 80 to 120 mm in length with the difference in length between two kidneys being not more than 20 mm. Therefore of your kidney is within this range then it is not a serious concern.There are patients in who one particular kidney might be larger than usual because of hypertrophy. This is only an increase in size and does not mean the kidney is under any type of stress or is diseased; this is an acceptable normal variant condition.If your blood tests for kidney function are normal and no significant abnormality is noted then your kidney is not a concern. Please do not worry."
},
{
"id": 117954,
"tgt": "Is there any relation between SGPT and SGOT with alcohol?",
"src": "Patient: I have had at least 180ml of alcohol every day for the past 20yrs .My sgot and sgpt levels are above 100. but physicaly I dont feel any health problems no BP. diabetic level is normal cholesterol is in 200.Am I suffering from any liver disorders right now?Dr should I do an immediate checkup? Doctor: HiYes, your liver enzymes are deranged possibly due to chronic alcoholism.You need to get a complete evaluation for alcoholic liver diseaseRegards"
},
{
"id": 138440,
"tgt": "Suggest treatment for arm pain due to rotator cuff tear",
"src": "Patient: i have this pain in left upper arm for a long time. MRI showed my rotor cup isn t the greatest but there is a small tare in tendon where the muscle is. The orthopedic suggest I get shotsbut how will that heal the tendon, I am right handed also and haven t injured my arm. Any suggestons? Doctor: HiWelcome to healthcaremagicI have gone through your query and understand your concern.You has small tear in tendon. It will heal with rest so it is advisable to do as less work with your shoulder as you can. Vitamin B and C help in healing. You can take analgesic such as ibuprofen for pain relief. You can discuss with your doctor about it. Hope your query get answered. If you have any clarification then don't hesitate to write to us. I will be happy to help you.Wishing you a good health.Take care."
},
{
"id": 176028,
"tgt": "Suggest remedy for high fever in child",
"src": "Patient: my 3 year old has had a hi fever, went up to 104.6, due to some type of virus as they told me in the hospital, now its day 5 and his right upper thigh to his but is covered in welts, almost looks like white areas surrounded in red, big spots, what can be the cause of that? now i dont have any money available on my credit cards, i just have some questions, can you please help me, or is their any sites i can get some answers without credit card information? Doctor: HelloMost likely it is a viral infection causing fever and rashes.Red big spots may be urticarial/allergic rashes too.Give him paracetamol syrup,also do tepid sponging to bring down the temperature.Consult a pediatrician for proper clinical evaluation or send some pics of the rashes so that a diagnosis could be made.Regards"
},
{
"id": 82819,
"tgt": "Suggest B complex without Cyanocobalanin B12",
"src": "Patient: I am wondering which is the best Vitamin B Complex that does not contain Cyanocobalanin B12, as I am learning contains Cyanide. I have Lupus and strive to be in optimal healh and am currently taking one with this added. I see i can take them separately but want all the B s working together for adequate balance. Does Dr. Weil advise a specific one? and is Cyanocobalanin harmful? I thought it was awesome finding one without fillers, corn or wheat, as I need to omit those as well, now to find out another scare with this synthetic B12. Thank you! Doctor: You can try becasule , it doesnt contain Vitamin B12 but I dont know if its available at your place.Dr. Shruti"
},
{
"id": 126009,
"tgt": "How can a bruised and stiff toe be treated?",
"src": "Patient: Hi, I stubbed my toe very bad on Tuesday night, and since then my toe has felt stiff it also got bruised all around the toe within the second day. Yesterday the bruising got darker but it does hurt less to walk on. Is there something I am able to do to help the bruising? Doctor: Hi, In your case, there is a possibility of a small fracture in your toe. I suggest you get an X-ray of the toe before embarking on any treatment. Hope I have answered your query. Let me know if I can assist you further. Take care Regards, Dr Gopal Goel, Orthopaedic Surgeon"
},
{
"id": 190944,
"tgt": "Does ecosprin have any effect on gums ?",
"src": "Patient: sir ,cn i know the effects of ecosprin on gums Doctor: Hi, Ecosprin or Aspirin is a blood thinner. It usually has no effect on gums as long as the gums are healthy. If the gums are injured or bleeding already due to gingivitis, Ecosprin may worsen the bleeding. So please get a complete oral examination done and contact a doctor immediately if you notice any bleeding in the mouth."
},
{
"id": 116811,
"tgt": "Suggest treatment for elevated lipid levels in blood",
"src": "Patient: MY SELF MAHENDER PAL, AGE ;29YRS ,SEX;MALE, RBS COUNT 5.44 ,MEAN CORPUSCULUR VOL. 81.2 , RED CELL DIST WIDTH (RDW) 15.8 ,CHOLESTEROL 212 , TRIGLYDERIDES 195 , CHOLESTROL LDL 126, VERY LOW DENSITY LIPOPROTEIN 39 PLEASE GUIDE PATHOLOGIST SAYS U HAVE HIGH LIPID, WHAT WILL I DO NOW GUIDE ME.... Doctor: Hi, dear. I have gone through your question. I can understand your concern. You have high LDL cholesterol and higb triglycerides levels. You should take low fat diet with high amount of polyunsaturated fatty acids. You can also take drugs like atorvastatin or pravastatin to control your cholesterol level.HoHowever it is prescription based medicine. So consult your local doctor and take treatment accordingly. Hope I have answered your question, if you have doubt then I will be happy to answer. Thanks for using health care magic. Wish you a very good health."
},
{
"id": 107938,
"tgt": "Suggest treatment for lower back pain",
"src": "Patient: I have pain in my lower back on both sides. I feel full all the time, it hurts when I breath deeply. I am having trouble with my bowl movements. Such as very little coming out & not as frequent as I used to. I am having trouble sleeping, ie getting into a comfortable position. It is not a constant pain. Last night was very bad for me after I ate a few handfuls of peanuts. Also, constantly flatulent... Doctor: your symptoms suggests that you might have severe paraspinal muscle spasm in the lower back.because of pain you could not generate enough abdominal pressure to pass stool .same things happen with deep breathing . eating peanuts may some times cause flatulence. if such patient come to my clinic i first start analgesic such as ibuprofen,muscle relaxants , stool softener (laxative) and hot fomentation of back.if symptoms do not subsides in 4-5 days then you should visit to an orthopaedician.Hope this answers your question. If you have additional questions or follow up questions then please do not hesitate in writing to us. I will be happy to answer your questions. Wishing you good health."
},
{
"id": 216729,
"tgt": "What causes severe pain in lower leg?",
"src": "Patient: About 5 years ago I had a drum full of steel crush my lower leg above my shin, It swelled up along with my foot and I was unable to walk for over 4 months, The doctors originally thought acute compartment syndrome but no compartments where the injury was originally. Time has passed and I get pains in the area A lot, especially if pressure is applied to the area, I have seen doctors who have taken X-Rays of the area and cant see anything. I had an MRI that didn\u2019t show much of anything either. The doctor wasn\u2019t sure what was causing the pain, saw a physical therapist and that didn\u2019t help either.. any idea on what the issue could be or what I could do? Doctor: hi,thank you for providing the brief history of you.As you had injury in the past and could not walk for about say 4 months, one needs to understand the biomechanics of the spine and pelvic. As you were not able to walk there must have excessive usage of the lumbar spine muscles which must have lead to degenerative changes due to propulsion of the body for Gait mechanism.Also, in my clinical practice we believe to find the root cause and attack over there to control the pain. Majority of patients with Past injuries will have spine issue and are treated accordingly.also the discomfort you are facing has to be differentiated from pain and ache. Pain is neuropathic and ache is muscular.If you mention it pain than it is neuropathic and needs proper examination and physical therapy. If it is ache then the treatment will be symptomatic and can help to a limited time.RegardsJay Indravadan Patel"
},
{
"id": 6909,
"tgt": "Is there a chance of getting pregnant if the condom is on ?",
"src": "Patient: hi , i want to know if you can get pregnant when you use a condom, the man did not ejaculat in myself . He pulld out and the condom came off he came at a later time no were by me lower have of my body? Doctor: Hello, Welcome to Health Care Magic, Failure rate of condom is upto 5% per hundred women using this method.Sometimes not placing the condom properly or if the tip gets torn during intercourse may lead to pregnancy."
},
{
"id": 160168,
"tgt": "What are the possible treatment for liver cancer with hepatitis b ?",
"src": "Patient: hellow doctor my brother suffering for liver cancer and he has hepatitis b and c also. the affected area is near about 9 cm. as per reports and doctor is saying it could be hcc. please tell me what possibilities are for treatment. height- 5.2 feet. weight- 60 kg. hbsag- positive, anti hbc igm- positive, afp- 91.76, portal hypertension Doctor: Hello \u00a0\u00a0\u00a0\u00a0\u00a0surgical intervention followed by chemotherapy is the right answer ,contact any good hospital& doctor & start treatment. \u2018Hope I have answered your query, I will be available to answer your follow up queries, \u201cWish you Good Health and trouble free speedy recovery\u201d"
},
{
"id": 97981,
"tgt": "What are the natural ways to get fairness ?",
"src": "Patient: black skin turn white skin ,so iam a girl in black colour how can i turn white colour please please i kindly request to give me tip ,my native place is tamilnadu. Doctor: Hallo, Your age is not mentioned. Skin color is due primarily to the presence of a pigment called Melanin, which is controlled by atleast six Genes. People with light complexioned skin mostly produce Pheomelanin, while those with dark colored skin mostly produce Eumelanin. Nature has selected for people with darker skin in trapical lattitude, especially in nonforested regions. Try this , but not sure to get white skin., Avoid hot sun light during noon hours. Apply sandalwood powder with rose water mix all over body. Take pure safforen with milk at night daily."
},
{
"id": 30736,
"tgt": "What causes sores on neck and white blisters in throat?",
"src": "Patient: Hi my name is matthew and I have a feeling i have hpv or gonorrhea but in late stage I have white ball bislter like in my throat and sores on my body neck and face and I'm all way s itchy and hard to swallow sometimes could it be either one and has gotten into my blood stream and is starting to cause me to have RRP ? And I have no energy and bones and muscle aches and feel dizzy and confused sometime s? Doctor: dear mathew,i advice you to get a complete blood count test,first of all.you might be suffering from low immunity due to itchy sores are formed all over your body.it might be an std but not gonorrhea.to rule out infection get a blood test done and then contact a md medicine doctor for further advice.hope that answers the question"
},
{
"id": 213543,
"tgt": "PCOS, obese, anxiety post episodes of body tremors, weakness. Advice?",
"src": "Patient: I keep having episodes where I feel rlly weak I shake and feel like I m going to faint. I m a 24yr old woman 15stonne I know I m over weight but the dr tells me its anxiety . I get anxious after these episodes happen but not before. I have been told I have poly cystic ovaries and I was wondering if it was related. I ve been like this now for 17months and its rlly beginning to take over my life. Please can somebody help me. Doctor: Hi and welcome to Healthcare Magic. Thanks for your question...Sudden bouts of anxiety which come in episodes is most likely to be panic attacks, and you may be suffering from a Panic Disorder. During these panic attacks one can feel shaky, dizzy, have a rapid heart beat, sweating, chocking, feel that there is something bad going to happen to them or even feel like they are going to die. Since these attacks typically come 'out-of-the-blue', you may not be able to predict an attack and hence may not feel anxious just prior to an attack.Usually, the period between these attacks is relatively symptom-free. However, sometimes, people can experience some amount of anxiety after or inbetween these attacks, which is predominantly due to a fear of getting another attack. This is called 'anticipatory anxiety'. This may be the reason why you experience anxiety after such attacks.Regarding your question about the relation to PCOS, more than half the women who have PCOS have obesity. Though there are some reports of anxiety disorders and depression being more common in PCOS, a definite causative link has not been established.All the best.- Dr. Jonas SundarakumarConsultant Psychiatrist"
},
{
"id": 14985,
"tgt": "How to cure the itchy rash on the outside of anus that bleed when wiped ?",
"src": "Patient: I have a rash on the outside of my anus. It's very itchy and now a lot of sores have formed along the outside. I haven't had sex in over a month. I'm a gay male. My lymphnode in my groin on the right side is inflamed. I used babywipes recently could've that have done it? I nomrally just use toilet paper. When i wiped recently it started to bleed a little from the outside. It's towards the top of my rectum. Doctor: NamastheWelcome to Healthcare-MagicGreetings of the dayIts likey that you have developed some sort of STD.Would suggest you to consult surgeon for further assesment.In the meantime hot seitz bath, a course of antibiotic like ciprofloxacin and analgesic like ibuprofen will help.In case you need any further assistance, will be glad to assist you.Wishing you happy and healthy lifeTake CareBest RegardsDr Deepak K KMBBS,MS,MCH"
},
{
"id": 11318,
"tgt": "What is the solution for hair loss problem?",
"src": "Patient: My hair has alot of new growth in it, but when I brush off my hair, I can see it shedding. It happens all day every day, but its like about 5 strands, sometimes more, sometimes less. I m scared that my hair might be falling out. (I relaxed it before, but I stopped in February.) Doctor: HIWell come to HCMIf you are male person then this could be Male- Patron-Baldness and to prevent this for some time Minoxidil 5% solution can be applied on scalp even female can be used this, but depends upon the age and underlying cause, (To have medical advise age and gender is very very important without these information nothing can be said for sure ) take care and have a nice day."
},
{
"id": 89559,
"tgt": "Does spicy food cause pain in navel area?",
"src": "Patient: Hi I had pain yesterday in my belly button area. I felt I wanted to fart but I couldn t. I felt like cuddling or laying down on my bed all warm and take some hot tea to feel better. I had eaten that day chips with limes and this not so spicy powder. Should I not eat spicy foods? Doctor: Do these symptoms come often then it may be acid peptic disorder. Get an ultra sound abdomen done to rule out any other pathology. Do take good care of your food habits like fiber full diet ;fruits ; lots of water.you should avoid tea;coffee and spicy food for time being. During acute phase of pain you can take antacids. If constipated take mild laxatives.If not improved see a doctor."
},
{
"id": 188701,
"tgt": "Are these black stains in the front tooth caused because of scaling and polishing ?",
"src": "Patient: Recently I did my dental check-up wherein the doctor did scaling and polishing. After two weeks I noticed some black stains in the front teethes? Could scaling and polishing cause this? If yes what is the remedy. Doctor: Hello and welcome to HCM, As scalig and polishing are the treatment to relieve the stain and tartar problem, so, these stains are not because of it, but may be a second appointment was't scheduled by any of you or you have not followed the post scaling instructions, like not having tea or smoke for some time,.. So check completely and visit the dentist. Thank you"
},
{
"id": 189777,
"tgt": "Decayed tooth removed. Pain in the gum. Can I take neuflo and naproxen together?",
"src": "Patient: hi, i have a sleepless night due to,had just remove my decay tooth at the dentist . but i notice that my another tooth is shaky but dentist did an xray the day before i pluck it out. Dentist mention it is fine. The gum surrounding the shakey tooth pain on and off. Only medication given was a paracatemol 500mg. Is it ok for me to consume neuflo/ lysozyme 90mg together with naproxen 220mg? please advice.Thank you darreanto,Singapore Doctor: HELLO, Thanks for writing in. AS youmantioned,you got your teeth extracted. Dentist prescribed you only analgesics because the extracted teeth was mobile. Please take warm saline gargle frequently to relieve pain. I would suggest you to visit the dentist soon since you have pain in your gums. Antibiotics can be administered as per dentist's prescription. Please do not take medicines without medical supervision. Also maintain good oral hygiene. Rinse your mouth after every meals to prevent accumulation of food debris on the extraction socket. ragards"
},
{
"id": 162745,
"tgt": "How can mucus in stool be treated while on Enfamil A+ Gentlease?",
"src": "Patient: Hello My baby is 3 months old. When we came home from hospital when she was a few days old her poop became hard. So I transitioned her to Enfamil A+ Gentlease from Enfamil A+. Then a couple weeks ago her poop becam explosive so I thought maybe she was ready to switch back to a regular formula. So I started the transition and her poop became mucus so I switched her back to Gentlease. It\u2019s still mucus and I called her doctor and the nurse said to alternate formula each feed. There is no change. She still has a lot of mucus in her poop. Please help Doctor: Hello and Welcome to \u2018Ask A Doctor\u2019 service. I have reviewed your query and here is my advice. Is she gaining weight? I would like to suggest you breastfeeding is only best food for babies till 6 months age. Hope I have answered your query. Let me know if I can assist you further."
},
{
"id": 156943,
"tgt": "Had whipple procedure for pancreatic cancer, still having diarrhea issues. How to reduce tumor markers?",
"src": "Patient: My husband had the Whipple procedure for pancreatic cancer and still has diarrhea issues 8 months later. He lost 60 lbs and is gaining it back slowly. On TPN food bags for the last 2 months and still on them and I would like to start using a juicer for him but don't know what fruits or veggies to use for this type of cancer. Presently his CA19-9 tumor markers are at 160 after surgery they were at 24. This concerns me. Any ideas to get those tumor markers down? He has already gone through the radiation and chemo treatments. Thank you. Dawn Skiba Doctor: Hi and thank you so much for this queryI am so sorry to hear about this persistent diarrhea and the rising tumour markers. The rising tumor markers mean that the tumor was not completely removed and nothing else can be done to address this at this time. It is rather unfortunate but a majority of persons with incompletely excised pancreatic cancer would most likely not be alive after 6months. There is nothing that can be done about this diarrhea as it is due to digestive secretions that flow through especially bile. About starting a juicer, talk to his treating doctor to give his accord.I hope this addresses your query fully. Thank you so much for patronizing our services and please do feel free to ask for follow up clarifications and information if need be. I wish you the best of health.Dr. Ditah, MD."
},
{
"id": 168398,
"tgt": "Suggest treatment for cough due to post nasal drip in children",
"src": "Patient: My baby is almost 6 weeks old, has caught a cold (or similar) from his 18 month old sister. Sister was given antibiotics (5 days ago) by GP since her cough was not improving after a few days and he believed she was getting bronchiolitis. Sister is now mostly recovering but baby is getting worse: - screamed for most of an hour trying to feed him 6 hours ago, refused approximately 40 of 100 ml bottle. - has been lethargic most of the day. - refused to shut his mouth to drink his next bottle ( approx 2 hours ago) and took only 30 ml of 100 ml bottle - normal is 130 - 150ml for this one. - has persistent cough (though no whoop thankfully). GP suggested chest clear, cough due to post nasal drip, though lethargy and refusing food are new symptoms. Doctor: HiThe symptoms are suggestive of respiratory tract infection rather than post nasal drip. I would recommend you to go for a chest X Ray and blood sepsis screen which includes CBC, CRP and blood culture. If the child is not feeding well and seems lethargic then hospitalization and management will be useful."
},
{
"id": 66491,
"tgt": "Does swollen lump on penis indicate sign of syphilis?",
"src": "Patient: I have a lump on my penis. It started suddenly and became very swollen within a day. The second day, the swelling had reduced a bit but my lymph glands in my groin/thigh area were tender. The third day, the tenderness had gone away, along with most of the swelling. Its been 6 days now. The bump is secreting puss. Not much. It seems to me that it is healing well. I would like to know if i should be worried that i have syphillus. Doctor: Hi, dearI have gone through your question. I can understand your concern. You may have syphilis or some other sexualy transmitted diseases. You need examination. You should consult your doctor and go for examination. Then take treatment accordingly. Hope I have answered your question, if you have doubt then I will be happy to answer. Thanks for using health care magic. Wish you a very good health."
},
{
"id": 44602,
"tgt": "What is the reason for failure in conception post regular intake of medicines for thyroid and premature ovulation ?",
"src": "Patient: hi, i am 28 yrs and trying to conceive since 2 years. i have premature ovulation problem. I am taking Ubiphene 50 (2 - 7 day of my period) and Ovacare forte and Obimet Sr also. i did thyroid tests and its normal tsh is 3.36. but Dr has prescribed Thyronorm 12.5 mg.. since 3 months am taking the tablets but still i dont know whats wrong. why am not conceiving? Pl help Doctor: Hi Jimmy, I hope that your Doctor has ruled out male infertility. Whileyou are taking these Medicines, you must be doing test for ovulation to see the effects of these Medicines on your Ovulation. There are certain ways to confirm ovulation like trans-vaginal Follicular study, Home Ovulation test, and hormonal levels in your blood, besides some simple method like BBT, Cervical Mucus test. If you are ovulating with these medicines, and still not able to become pregnant in 3-4 months time, then you should go for further tests for tubal patency test, uterine factors, immunological factors, and so on. For infertility, you must go step wise approach and proper follow up. You should be knowledgeable about the management you are going through, and talk to the Doctor frankly. One positive point, i wolud like to tell you is that , it is easy to treat infetility if it is due to ovulation dysfunction. best of luck. best of luck."
},
{
"id": 101763,
"tgt": "What causes body aches and red spots on forehead/chest/back/arms/neck?",
"src": "Patient: I am a 41 year old male. Symptoms started as body aches, flush feeling, just feeling like no energy. A day ago i noticed what looked like the start of a pimple on my forehead. Today several red spots on forehead, upper chest, some on back, crease of arms, neck area. They dont really itch to bad. I also feel constipated at times, some belly aches and small amount of diarreha. My wife is convinced its chicken pox. I also started an anxiety med two nights ago to help me sleep. Didn't notice rash till after i started the med but the other symptoms were present the day before i started med. Started med due to lack of sleep i was getting from wearing a cpap mask at night. Doctor: Hi, It will be better if you can upload the photos of the lesion. This will help in making diagnosis.if chicken pox you need to take acyclovir with other symptomatic treatment. Chicken pox presents similar to flu with vesicular rashes. It cannot be pimples as all the lesions cannot apprar in a day.Since you started anxiety medicines this can be due to its reaction also. Without examination and knowing about medicine we can't confirm the diagnosis. So better to upload a photo of the problem or else get examined by your doctor once.I hope this information helps you. Regards"
},
{
"id": 146052,
"tgt": "What does an MRI report of oval calcification indicate?",
"src": "Patient: HI dear doctor My MRI has shown an oval calcification between my cervical C5 and C6. I fell numb in my left hand and sometimes my right one. I have to go to toilet every one hour. I think it is because of that calcification. I do some static exercises as my physiotherapist has recommended but I am really worreid when I dont do them for more than 5 or 6 hours again I feel pain. I am 42 and I think I am too young to do surgery. what should I do? Doctor: hello,any chronic inflamation of spine heal into calcification, that could be evident in Xray or MRI investigations. Rheumatic conditions like Rheumatoid spondylitiis, ankylosing spondylitis, are the most common cause for such chronic inflamation of spine. There is no specific cause, as they are mostly due to an underlying auto immune mechanisms. I would advise a regular exercise and keep the stress levels low by daily yoga and healthy lifestyle."
},
{
"id": 181720,
"tgt": "Suggest treatment for painful white spots on palate and red spots on tongue",
"src": "Patient: I had a high risk before 3 years ago. And was tested -ve for Hiv1/o/2 abs qual after 15 months. and I am Negative . But here is something which worries me. 1. 2 pain white spots on roof of my mouth and a red tongue spots. Im married for 2 years. I got married after I got tested for HIV. So after 3 months from my marriage my wife got pregnant and we have healthy daughter have healthy daughter, and now my wife is pregnant. She need to be tested -ve on Hiv 1/2 ag ab plus O which was her screening at 5 months of pregnancy. I am really worried about her and my case. Should I go for a 4 gen or a NAAT test? And how much percentage will you say that I don t have HIV+? I sometime think that my body didn t produce antibodies so far. Please say your advices? Doctor: Thanks for your query, I have gone through your query.The white spots over the palate and the red spots on the tongue can be a fungal infection like candidiasis. Nothing to be panic, consult a oral physician and get it evaluated. Regarding HIV infection, consult a general physician and get it examined. You need to get investigations done to rule out HIV infection.I hope my answer will help you, take care."
},
{
"id": 113802,
"tgt": "Having lower back pain with tingling feeling in both legs. No relief with paracetamol",
"src": "Patient: HI, I have severe pain in my lower back on the left hand side above my bum, I bent down to pick something off the floor this morning and when I came back up right the pain started, I have a tingly feeling in both legs but my left leg really hurts when I walk on it. I have taken paracetomol for it but it s not done anything. Please help! Doctor: It seems you have developed as PIVD ie a slip disc in layman`s language and the nerves coming out from your spine are getting pressed causing tingling. Get a xray of dorso lumdar spine or a MRI. For immediate relief take some pain killer, muscle relaxant and local heat treatment and rest. Thereafter consult an orthopedic surgeon who will advise you accordingly."
},
{
"id": 210932,
"tgt": "How to treat feeling of depressed and low self esteem?",
"src": "Patient: Hi, I have problem with speaking, I cannot speak in front of peoples or in crowd many times. But I am happy (Confident from inside) am able to speak anything in front of anyone. But most of the time I feel ashame in front of peoples, in meetings and in group discussion. and Now i am 29 years old, this problem i have from starting of my life, I never mind it. thought it will go with confidence boost. but it never get sorted out. I am feeling depressed and low self esteem. can t speak in front of peoples. Please suggest some of medicine and cure of my speaking. Doctor: Hello,Thanks for choosing health care magic for posting your query.I have gone through your question in detail and I can understand what you are going through.This condition that you are having is called social phobia and it is a type of specific phobia. This requires medical as well as psychological help. In medical help, medicines such as paroxetine or venlafaxine or mirtazapine will be helpful. Among psychological treatments cognitive behavioural therapy and systematic desensitization would be helpful.Hope I am able to answer your concerns.If you have any further query, I would be glad to help you.In future if you wish to contact me directly, you can use the below mentioned link:bit.ly/dr-srikanth-reddy\u00a0\u00a0\u00a0\u00a0\u00a0\u00a0\u00a0\u00a0\u00a0\u00a0\u00a0\u00a0\u00a0\u00a0\u00a0\u00a0\u00a0\u00a0\u00a0\u00a0\u00a0\u00a0\u00a0\u00a0\u00a0\u00a0\u00a0\u00a0\u00a0\u00a0\u00a0\u00a0\u00a0\u00a0\u00a0\u00a0\u00a0\u00a0\u00a0\u00a0\u00a0\u00a0\u00a0\u00a0\u00a0\u00a0\u00a0\u00a0\u00a0\u00a0\u00a0\u00a0\u00a0\u00a0\u00a0\u00a0\u00a0\u00a0\u00a0\u00a0\u00a0\u00a0\u00a0\u00a0\u00a0\u00a0\u00a0\u00a0\u00a0\u00a0\u00a0\u00a0\u00a0\u00a0\u00a0\u00a0\u00a0\u00a0\u00a0\u00a0\u00a0\u00a0\u00a0\u00a0\u00a0\u00a0\u00a0\u00a0\u00a0\u00a0\u00a0\u00a0\u00a0\u00a0\u00a0\u00a0\u00a0\u00a0\u00a0\u00a0\u00a0\u00a0\u00a0\u00a0\u00a0\u00a0\u00a0\u00a0\u00a0\u00a0\u00a0\u00a0\u00a0\u00a0\u00a0\u00a0\u00a0\u00a0\u00a0\u00a0\u00a0\u00a0\u00a0\u00a0\u00a0\u00a0\u00a0\u00a0\u00a0\u00a0\u00a0\u00a0\u00a0\u00a0\u00a0\u00a0\u00a0\u00a0\u00a0\u00a0\u00a0\u00a0\u00a0\u00a0\u00a0\u00a0\u00a0\u00a0\u00a0\u00a0\u00a0\u00a0\u00a0\u00a0\u00a0\u00a0\u00a0\u00a0\u00a0\u00a0\u00a0\u00a0\u00a0\u00a0\u00a0\u00a0\u00a0\u00a0\u00a0\u00a0\u00a0\u00a0\u00a0\u00a0\u00a0\u00a0\u00a0\u00a0\u00a0\u00a0\u00a0\u00a0\u00a0\u00a0\u00a0\u00a0\u00a0\u00a0\u00a0\u00a0\u00a0\u00a0\u00a0\u00a0\u00a0\u00a0\u00a0\u00a0\u00a0\u00a0\u00a0\u00a0\u00a0\u00a0\u00a0\u00a0\u00a0\u00a0\u00a0\u00a0\u00a0\u00a0\u00a0\u00a0\u00a0\u00a0\u00a0\u00a0\u00a0\u00a0\u00a0\u00a0\u00a0\u00a0\u00a0\u00a0\u00a0\u00a0\u00a0\u00a0\u00a0\u00a0\u00a0\u00a0\u00a0\u00a0\u00a0\u00a0\u00a0\u00a0\u00a0\u00a0\u00a0\u00a0\u00a0\u00a0\u00a0\u00a0\u00a0\u00a0\u00a0\u00a0\u00a0\u00a0\u00a0\u00a0\u00a0\u00a0\u00a0\u00a0\u00a0\u00a0\u00a0\u00a0\u00a0\u00a0\u00a0\u00a0\u00a0\u00a0\u00a0\u00a0\u00a0\u00a0\u00a0\u00a0\u00a0\u00a0\u00a0\u00a0\u00a0\u00a0\u00a0\u00a0\u00a0\u00a0\u00a0\u00a0\u00a0\u00a0\u00a0\u00a0\u00a0\u00a0\u00a0\u00a0\u00a0\u00a0\u00a0\u00a0\u00a0\u00a0\u00a0\u00a0\u00a0\u00a0\u00a0\u00a0\u00a0\u00a0\u00a0\u00a0\u00a0\u00a0\u00a0\u00a0\u00a0\u00a0\u00a0\u00a0\u00a0\u00a0\u00a0\u00a0\u00a0\u00a0\u00a0\u00a0\u00a0\u00a0\u00a0\u00a0\u00a0\u00a0\u00a0"
},
{
"id": 205187,
"tgt": "What causes crawling sensation in the skin at night?",
"src": "Patient: Good evening, for the pass three or four days I ve been having the feeling of something crawling on me. I was awaken in the night a couple of time. It seem like something extremely small and light. Mostly on my left arm but on different part of my body like and face and leg, I m extremely healthy and don t have any health issues I know of. any suggestion? Doctor: please give details about any drug usage.cocaine can cause similar symptoms.Please specify if you have any sleep disorder,narcolepsy etc.also normal human beings can have hypnogogic and hypnopompic hallucinations.please consult a psychiatristalcohol withdrawal is to be taken into acount if you drinks ."
},
{
"id": 48126,
"tgt": "What could cause constipation, external hemorrhoids and pelvic cramps after passing kidney stones?",
"src": "Patient: I passed a kidney stone four days ago. I was very painful and I was put in the hospital but have since been released. Immediately after being discharged I was constipated for a day then began having serious lower pelvic cramps. After the first bowel movement which was tuff I now have the cramps and it s mostly blood. I do have some external hemorrhoids since the constipation episode yesterday. The blood is bright red. Is the cramping and blood from the hemorrhoids or is something else going on? Doctor: Bleeding is from hemorrhoids but cramps are because of passing stones.cramps occurred due to severe pain while passage of stone .Urinary system and bowl system are linked with each other that is the reason there are cramps and constipation due to pain."
},
{
"id": 166591,
"tgt": "How to identify if child has swollen vitamin B12 tablet additionally?",
"src": "Patient: i am not if my three year old swallowed vitimin b12 additionally she could taken green tea loss weght pills, this happened an hr agom my 6 years old states that she did not see her swallow anything, and she told me it took me almost not time to find her playing with these pills. I am not to concern abt the diet pills coz the HUGE, i am sure i would listen to her trying to cough, but still kids at this age can swallow even batteries without making a sound, should i take her to the ER? or what symptoms should I wait to see on her if she happen to swallow something? please help I am very nervous, although i cannot be sure she had swallowed something... Doctor: Dear parent, I understand your concerns but there is no need to worry as swallowing vitamin b12 tablets will not cause any complications ."
},
{
"id": 172438,
"tgt": "Should i be worried about low milk intake in my child?",
"src": "Patient: MY 5 month old son has a viral infection, he saw a dr on Sunday. He has a bad throat, cough and a cold. HIs temp has been fine. MY concern is his reduction in milk intake he use to have between 25 and 30 fluid oz now he is having14 but he still wants to eat his solid food. HE will not drink water either. I have been giving him calpol and ibrufen. He is happy in himself and is active. SHould I be worried? Doctor: Hi....no need to be worried. The most important thing you have quoted in your query is that he's happy by himself and also that he's taking his solid food. This is a sign of recovery from viral illness. I suggest you have a watch on his urine output, if he's voiding well every 6th hourly once, that is 4 times a day.....then he's well hydrated, you need not be worried about his fluid intake.Regards - Dr. Sumanth"
},
{
"id": 87075,
"tgt": "Suggest treatment for headache and lower abdominal pains",
"src": "Patient: Hi I keep having headaches just above my left eye,lower abdominal pains, feel nauseous ,weak at times, limbs feel numb and sharp pains in the left side in my chest and back . I had a missed abortion about six months ago at 10 weeks. used the ortho evra patch for 2 1/2 months , I stop cause of a rash I kept developing.the doctors cant seem to find anything. i did two home pregnancy tests and they are both negative . the doctor did tests of my sugar , liver, blood count and it was fine. They did an ecg and t was also fine Doctor: Hi.Thanks for your query.After reading the history you have provided, the occurrence of headache above the left eye may not be associated to other symptoms and can be due to Supra-orbital neuralgia.The lower abdominal pains, nausea, weakness, numb limbs and sharp pains in the left side of the chest with normal sugar, ECG liver and blood counts can be due to intestinal infection or IBS giving the symptoms of inappropriate absorption which lead to other symptoms of weakness and all other. Missed abortion might have instigated the changes and enhanced the problems. I would advise you the following :: Remaining blood tests for Typhoid, thyroid and relevant as per the clinical findings of a Doctor on examination. CT scan of the abdomen to rule out any changes. Colonoscopy and Gastroscopy.The Clinical examination and reports of the investigations will tell us the proper diagnosis and lead to the correct treatment . . ."
},
{
"id": 7361,
"tgt": "Is there any chance of pregnancy?",
"src": "Patient: I have taken postinor2 twice in a month and I had unprotected sex is there any chances of pregnancy Doctor: Hello Welcome to health care magic forum Two tablets of emergency contraceptive don`t protect you through out the month.It is an emergency contraceptive to be used with in 72 hours of unprotected sex & not vice versa . Wish you good health Disclaimer"
},
{
"id": 118258,
"tgt": "What could be wrong if having severe indigestion and low blood platelet count?",
"src": "Patient: Hi, I am having occasional bouts of severe indigestion (like right now) though I am generally very fit for a 56 year old male. My doctor is puzzled that I have quite a low blood platelet count and she has run the blood tests for three concurrent months... Any ideas what may be going in would be helpful, thanks Doctor: Hi, Persistently low platelet count could be ITP (Idiopathic Thrombocytopenic Purpura). check if other blood counts like WBC and RBC are also low or lower side of the normal. You may need further investigations like bone marrow examination and auto immune panel. You can post the details of tests done for complete understanding of your problem and possible diagnosis.Hope this helped."
},
{
"id": 93057,
"tgt": "Had severe stomach pains. MRI,X-ray, medicines tried. Still have pain. On aspirin. Suggestions?",
"src": "Patient: I have had tremindous stomach pains. I have had MRI, X-Rays, tried new meds and I still suffer from the same pain. To me it feels like it is getting worse but my dr. keeps saying nothing seroious. I went to an abdominal dr. also and he said just to keep on taking the asprin I was taking to help the pain. What else Can I do? Doctor: Hi,Thanks for writing in.I suppose you have stomach pains for a while now and started taking aspirin for it with some ?new drugs. MRI and X rays have been done for abdomen or any other part is not clear from your query.With this information, I would like you to stop taking aspirin for stomach ache. You may take proton pump inhibitors (pantoprazole, omeprazole ) and some anti spasmodic medication. Aspirin may worsen your pains by causing gastritis and that might exactly be happening with you.As investigations, I suggest you to have complete blood tests and ultrasound scan abdomen. A gastroduodenoscopy (oral endoscopy) might be done if necessary.Hope this helps."
},
{
"id": 190292,
"tgt": "Soreness around the back of gums, swollen cheek, gap between molar and guns, jaw feels stiff on chewing. Cause?",
"src": "Patient: Hi im 19 next month and i havent had my wisdom teeth removed yet and id just like to ask you why my mouth is sore around the back of my gums on the left hand side only I ve noticed that my left cheek is swollen and my gums too and just behind my last molar my gum is really tender and there s a little gap between my molar and my gum and it s really sore and food and decay get caught in there after each meal I eat and my jaw feels sort of stiff as well and hurts when I chew......what could be the cause of this? Could it be that I need my wisdom teeth extracted? Please help I don t know what to do and I d be very greatful if you could give me an answer to my question and help me out Doctor: hello and welcome to HCM forum, i would be glad to answer your query, according to the dental history provided by you i would like you to know that the gap between your tooth and the gums is due to inflammation of the gum tissue that surrounds the tooth, this is either a case of localised periodontitis, where a pocket is formed where all the food gets impacted causing pain and swelling. Also, as you mentioned that your wisdom teeth have not yet erupted, i would like to inform you that wisdom teeth may or may not erupt, wisdom teeth are not always present. I advise you to follow a few instructions in order to relief pain and swelling; 1. kindly see a dentist, 2. full mouth scaling is required, 3. warm saline rinses(3-4 times/day), 4. use a betadine mouthwash once/day. i hope i answered your query, i wish you good health, take care."
},
{
"id": 47108,
"tgt": "What treatment options are available for IGA nephropathy?",
"src": "Patient: Hi, I was diagnosed with IGA nephropathy at 10 years old, I am now 25 years old and still see blood in the urine each time I urinate and is easily fatigued. Whenever i have a blood test my kidney function is always normal yet I dont feel normal! What treatment options are available? Doctor: Hi thanks for asking question in HCM.Allow me to know you the fact that there is no single curative treatment available for it.Treatment will definately delay the renal complication and need for transplantaion and dialysis will be delayed.ACE inhibitors are the best drug use for secondary hypertension.Meat and dairy product has to be taken minimum.TAKE LESS GLUTEN IN DIET.Take low protein diet.The ultimate effective treatment for end stage disease is renal transplantation.In your case regular monitoring has to be done. Enalapril like drug will be helpful to you.I hope my suggestion will help you."
},
{
"id": 1074,
"tgt": "Will Ovamit tablets help in conceiving?",
"src": "Patient: hello doctor, i have been taken last month ova mit tablets ,and i got my period at dis month at right date i m quite happy for dis ,i m married girl since last 2 years ,we r tring for baby as well .i really want baby now,wil ova mit tablets help me for getting pregnency? Doctor: Hi, I think ovamit tablets will help you in conceiving. You can track your follicles growth by repeated ultrasound after taking the tablets. When follicles reach a size more than 17 to 18 mm, take injection for rupturing the follicles. Be in contact with your husband for 2 to 3 days after injection. Take progesterone for next 2 weeks. Do a urine pregnancy test at home after that. You can try like that for 3 to 6 months. Hope I have answered your question. Regards Dr khushboo"
},
{
"id": 2278,
"tgt": "Will it be difficult to get pregnant after a prolong use of loette tablets?",
"src": "Patient: i have taken loette tablets for 2 yrs and frm last 1 yr i have stopped, now willing to get concieved i have undergone Hsg test as well, now i have got a ist period after my HSG, i am now in confused status, i am scared whether i will get concieved or not Doctor: Hi there Please do not panic. If you have regular cycles, your fallopian tubes are patent and partner's semen reports are normal then you need not worry about the pregnancy. Low dose contraceptive pills do not have any negative impact on getting pregnant. Please start on Folic acid, Vitamin B 12 and Omega 3 supplements at least 3 months before you plan to get pregnant. As you might be aware that in a woman with regular cycles day 10 to 20 of the cycle is the most fertile period of cycle. If you happen to have unprotected sex act in this period, then You Stand chance to get pregnant. I hope this answer helps you. Thanks. Dr. Purushottam Neurgaonkar."
},
{
"id": 49545,
"tgt": "On antibiotics for kidney infection and virus, constantly sick. Is there a permanent cure ?",
"src": "Patient: I've been to the hospital as I was very ill throwing up every half hour for a full day, I got diagnosed with a virus and a kidney infection at the same time. It's now the fourth day, my virus has seemed to of gone, im just on antibiotics for my kidney infection, however every time I try to eat it feels like it's sitting in my throat, like I need to be sick constantly, and I haven't pooped since a week ago, I've tried to eat quite a bit but I don't need to poop at all? What's going on? Doctor: Hi and thank you so much for this query.I am so sorry to hear about what you have been experiencing. Having been unable to go to the toilet is probably caused by the very little amount of food that you eat. we need enough residues after digestion in order to be able to fill the rectum and excite the defecation reflex. As you get better with treatment, for sure you would start pooping good.Try to eat with much water and help relief yourself of the impression of food sticking in your throat. this will help improve your food intake.I hope you find this helpful and it addresses your query fully. Thank you so much for using our services and feel free to ask for more information and clarifications if need be. I wish you the best of health.Dr. Ditah, MD"
},
{
"id": 211950,
"tgt": "Taking Vibryd for panic disorder.Having panic attacks again. Suggestions?",
"src": "Patient: I've been taking Viibryd for almost a month now for panic disorder. I've worked my way to 40 mg on the sample pack. I felt things were looking up on the 20 mg and then I started my second week of 40 mg and I started having panic attacks again starting yesterday with today's being the worst. Any suggestions? I just am tired of feeling good for 2 weeks and then bad for a few days. Doctor: Hi, it appears that you are happy with the 20 mg of the drug. and 40 mg became over dose. I suppose if you use 20 mg two times will be better, if not comfertable with it you can go for 20mg once daily. try to pracitce yoga, meditation, reading books of your interest, and, hearing music. Thank you."
},
{
"id": 14815,
"tgt": "What may be the cause of itchy rashes on thighs?",
"src": "Patient: I have a rash on my inner thigh and on my arm pits and the inside of my arms and around my waist and cress of my leg for 2 weeks now I cant sleep it is so itchy my doctor said it will go away but it did not I have no bed bugs and I stopped using lotions and deodorant I want to take my skin off....... Doctor: Hello and welcome to healthcaremagicI would keep a possibility of fungal infection/tinea cruris. This is because closely opposing body folds e.g finger webs, thighs and groin, axilla, submammary folds etc can trap sweat, heat and moisture, therefore making the area favorable for fungal proliferation.If I was the treating doctor I would have asked you to use a topical antifungal like clotrimazole/sertaconazole/luliconazole cream, twice daily, regularly for 4 weeks.Since you are already using an Oral antifungal e.g oral terbinafine once daily. I would suggest you to continue with it for 4-6 weeks for a more complete cureAn OTC antihistamine e.g cetrizine 10mg once daily would help you in symptomatic itch relief.Keeping the folds dry by using an antifungal dusting powder would help in preventing the recurrence.regards"
},
{
"id": 178449,
"tgt": "What causes fever and cough in a child?",
"src": "Patient: Blood report of my 2 year old shows normal results except sgot of 47, lymphocytes 52 and rbc count 5.31. His chest xray also shows prominent bilateral para hilar broncho vascular markings . Hes had a fever rangung from 100-101 since 6 days, cold, cough and no appetite. Should i be worried? Doctor: Hi...this suggests that this is a viral illness...Cough and cold are viral 95% of the times in children. Fever of few days without any localizing signs could as well a viral illness. Usually rather than fever, what is more important is the activity of the child, in between 2 fever episodes on the same day. If the kid is active and playing around when there is no fever, it is probably viral illness and it doesn't require antibiotics at all. Once viral fever comes it will there for 4-7 days. So do not worry about duration if the kid is active.Regards - Dr. Sumanth"
},
{
"id": 23886,
"tgt": "How serious is Mild global hypokinesis with borderline left artrial enlargement?",
"src": "Patient: How serious is Mild global hypokinesis with borderline left artrial enlargement? I have been diagnosed with PVC's. They occur sporadically. I had an EKG and Echocardiagram and wore a holter for 24 hrs. I am active 40 year old male in good physical condition. I am 5'8\" 198 lbs. Doctor: Mild global hypokinesis refers to your overall function of the heart (left ventricle). A \"good\" heart pumps at least 52% of the blood that it receives. The mild category is used for a range of 41-51%. The most worrisome etiology is ischemia or otherwise blockages in your heart arteries. PVC's - which are premature ventricular complexes , if too excessive can cause depressed heart functions, but yours are only sporadic and not a concern. Borderline left atrial enlargement is typically not a concern, at times it can be associated with abnormal rhythms from top chamber of the heart but your holter monitor did not reveal this."
},
{
"id": 135546,
"tgt": "What makes my knee cap sting after popping?",
"src": "Patient: Hi my 13 year old son was at his dads house all week till about 10:15 tonight apparently he dislocated his right knee cap yesterday at about 7:05pm while helping to move a picnic table and he poped it back in place and his dad said nothing was wrong with his knee and made him walk on it till he got to my house at about 10:15pm today and now he says it stings and he can move his leg but he can t move his leg all the ways you usually can without it stinging really bad what do we do? Doctor: Hello,I have studied his case and i think that he must have injured his ligament which is useful for knee patella stability. It can be confirmed with the help of x rays and Mri. If there is ligament damage thence will need plaster for three to four week. if there is severe damage then he might need surgery. If you want then you can share his MRI details with me. I hope this answer will be useful for you. Let me know if there is any other followup questions.thanks"
},
{
"id": 116991,
"tgt": "What causes low wbc?",
"src": "Patient: What could cause a constant issue with a constant low wbc, low iron (which I have to get iron infusions every one and a half years),low lymphocytes, sores on back of head, severe muscle/joint pain mostly in morning, muscle spasms in hands and feet, memory fog to not remembering simple things. I have been to so many doctors and they have done many tests, but they are still baffled and still testing. Any suggestions? Doctor: Hi,Thanks for asking.Based on your query, my opinion is as follows.1. Possible syndrome or genetic cause.2. Test details necessary for full opinion. You are at present having pancytopenia, due to marrow depression.3. Bone marrow biopsy is necessary to evaluate the cause. Feltys syndrome is one of the common causes of neutropenia with rheumatoid arthirits manifestations. More details necessary.Hope it helps.Any further queries, happy to help again."
},
{
"id": 136901,
"tgt": "Suggest treatment options for tibial fracture",
"src": "Patient: Got a tibia plateau compressed fracture non surgical. I m PWB and wearing a brace. I have 110 degrees knee bend and can get full flexation with a little discomfort. I bent my knee quickly by accident and now have quite a bit of pain and throbing. What can I do to help it? My accident was 6wks ago. Doctor: Post fracture setting and fixing is done best by an Orthopaedic surgeon.Pain and throbbing in knee is medicinally controlled by an Ayurvedic Medicine Abha guggul and Laxadi Guggul with warm water. It's commonly available in India. Added advantage by turmeric powder drink 1/2 tsp to a glass of warm milk will make you comfortable more rapidly. No side effects, easy, economic.Hope it helps you."
},
{
"id": 218337,
"tgt": "Should I be worried about pregnancy while on birth control pills?",
"src": "Patient: I am 20 years old and I have been taking birth control pills constantly for about a year. I never miss a dose and always take my medication on time. The medication has never affected my period and I get a heavy 5 day period every month. About 23 days ago is the last time that I had sex. I was supposed to start my period this past sunday. However I have only been having light brown spotting since Tuesday. I am concerned but I never miss my birth control pills. Should I be worried about pregnancy? Doctor: Hello, There is nothing to worry about if you are taking oral contraceptives regularly as advised by your doctor, at same time of a day without a break then the pills would protect you from getting pregnant. The scanty bleeding can be a side effect of pill (which can cause irregular periods). Please continue pills as advised it would resume normal bleeding soon. If It does not then an consult your gynecologist for physical examination and some investigations to exclude other causes of irregular periods. In case you had missed pills in between and you have scanty bleeding with positive home pregnancy test then you need to terminate the pregnancy. Oral contraceptive pills are harmful and can cause congenital abnormality. Hope I have answered your query. Let me know if I can assist you further."
},
{
"id": 2860,
"tgt": "Can I get pregnant with unprotected sex?",
"src": "Patient: Hi I had unprotected sex with my husband on Saturday he went in me , since then I have been discharging clear / sticky stuff it doesn t stink or smell at all But it does scare me it s too early to take a pregnancy test it only been 4 days ago, my last period was the beginnings of April and it s irregular so I don t know when it s supposed to come or when I m ovulating like I said I had unprotected sex on Saturday May 10th can I be pregnant? ? Doctor: Hi,I understand your concern. Your last period was on beginning of April. And you had sex on may 10th. I think it is safe period of cycle so chance of pregnancy is less. I would suggest to go for blood HCG test 10 days after sex to confirm pregnancy id period will not come. Blood HCG test is more accurate compare to urine pregnancy test as you have irregular period. Clear discharge could be a cervical mucus, cervix irritation.If it will continue then consult gynecologist. Avoid stress, take healthy diet, drink plenty of water, wash vagina with warm water 2 to 3 times per day and wear pure cotton loose inner.Hope this may help you. Contact further if follow up needed.Best regards,Dr. Sagar"
},
{
"id": 21900,
"tgt": "Suggest treatment for high blood pressure",
"src": "Patient: I am a 37 year old female and starting a year ago, have developed high blood pressure. The highest numbers have been 156/95 and the numbers vary but are almost always high. All of the tests I had done so far have been normal (blood tests, holter moniter, echocardiogram, 24-hour urine test) I don't know what else to do and I am quite worried about this ... I am not on any medication anymore. The doctor took me off of my birth control pill and anti-depressant in hopes of finding the cause. Do you have any ideas?? Doctor: Hi Ma'amYes definately your bp is high and it need to adresed soon. As all of your basic test are normal I want to know two things first, if you have any family history of High BP and second your dietary salt intake and smoking habit? If every thing is alright then we need to start with LIFE STLYE MODIFICATION TRAIL first which includes Brisk morning walks daily 45 mins a day 5 days a week, low dietary salt intake, avoidance of excessive coffee intake, High fiber diet and fruits and preferably YOGA/Meditation if you can. Start and follow this regime strictly for 3 months and keep a daily record of your BP at same time. Hopefully it will lower down your BP. And if it doesnt then we need to start with antihypertensive medicines.Good Luck"
},
{
"id": 39333,
"tgt": "Are there any dietary restrictions after dog bite?",
"src": "Patient: Gud aft,doctor.....my father was bitten by a dog today morning.he has taken tetanus nd rabies vaccine today.should he change his diet????? Any dietary restrictions???it was a smal scratch mark.he has no pain .nor bleeding.injury site was left lateral side of leg Doctor: Dear Friend.Welcome to HCM. I am Dr Anshul Varshney. I understand your concern.There are no dietary restrictions after a dog bite.i would advise you to take the Rabies vaccination - full course.Also, get a tetanus vaccination.If you develop local swelling, see a doctor for antibiotics that might be required for cellulitis.This is my personal opinion for you based on available details. If you have any further query please ask me.Stay HealthyDr Anshul Varshney, MD"
},
{
"id": 207227,
"tgt": "Suggest treatment for anxiety",
"src": "Patient: I have been feeling tense lately and keep worrying about negative things and how can I stop worrying and this has lead me to anxiety problems. I have seen a doctor, done and ECG and a chest x-ray the results are normal but I still keep worrying about my health and sleep has also been affected and have been feeling down. I am a 18 year old male Doctor: DearWe understand your concernsI went through your details. I suggest you not to worry much. Worrying about health is usually called as health anxiety or hypochondriasis. You are just 18. You have to do a lot to advance in your life. Education and career are important. Concentrate on them. Exercise regularly, keep yourself fit, eat balanced diet and always be happy. If you do all these, at your age, illness should not come that easily. Also you may need psychological counseling.If you require more of my help in this aspect, Please post a direct question to me in this URL. http://goo.gl/aYW2pR. Make sure that you include every minute details possible. I shall prescribe the needed psychotherapy techniques.Hope this answers your query. Available for further clarifications.Good luck."
},
{
"id": 164315,
"tgt": "What causes persistent fever in infant inspite of Amoxicillin?",
"src": "Patient: my 19month old grandson has fever @ 42.4 for more than 2 days and Advil and Tylenol are not working much... we gave him luke warm bath and place towells over his head but that doesnt seem to help much...After 2hrs his fever is back to 42.4 C is back with... He is taking Amoxicillin as his pediatrician prescribed more than 2 days ago.Shall we take him to Sick Kids Emergency ??? Doctor: Hi...Thank you for consulting in Health Care magic.By what your quote I feel that your grandchild is having a viral illness. I will explain you how a viral illness behaves so that you can be more confident.Fever of few days without any localizing signs could as well a viral illness. Usually rather than fever, what is more important is the activity of the child, in between 2 fever episodes on the same day. If the kid is active and playing around when there is no fever, it is probably viral illness and it doesn't require antibiotics at all. Once viral fever comes it will there for 4-7 days. So do not worry about duration if the kid is active.Regards - Dr. Sumanth"
},
{
"id": 100230,
"tgt": "How safe is taking Xyloflo-Nasal drop and Clavam 625 for severe cold?",
"src": "Patient: this question is for my wife. doctor says due to heavy cold, her left ear has got some problem. doctor prescribed following medicine which i could not underdstand.1.Xyloflo-Nosal drop, 2. Ambronac, 3. Clavam 625 and 4. mucodyne. after 7 days pure tone audiometry test they have recommended.is any thing serious? Doctor: This audiometry test to diagnose pathology for middle or internal ear pathology.If there is no improvement after taking medicines then might your doctor suggest to do this test. So you sholud go for this testGet well soon..."
},
{
"id": 184335,
"tgt": "Suggest treatment for swollen lump above tooth",
"src": "Patient: i had a white swollen lump above my tooth that was filled with liquid ithink. i get these every once in a while usually above the same tooth. it errupted now and is all deflated. these past few days that same tooth has been feeling wiggly as well, like its getting loose Doctor: HiThanks for your query with HCM,It would have better if you had mentioned your age since you say it is erupting and whether the tooth is having any decay? Coming to lump it could be either periapical or periodontal abscessI would suggest you to visit your dentist for the best treatment don't delay if you want to save the toooth though it depends on the condition.Hope my answer would be helpful."
},
{
"id": 164372,
"tgt": "What causes fever and cough in a child?",
"src": "Patient: we went to the doctor on thursday with my seven year old. she had low grade fevers and a cough and was complaining her chest hurt and her right side. Dr. said ears fine, sinuses fine, lungs fine, and sent us on our way. today 3 days into this her cough is worse and still running low grade fevers what do you think? Doctor: Hi, welcome to HCM. Can understand your concerns. Fever and cough can occur in upper respiratory tract infection and viral flu infection. You need to get the child examined by a doctor. Take care."
},
{
"id": 184262,
"tgt": "What causes small red dots with white centers scattered across roof of mouth?",
"src": "Patient: i have small red dots with white centers scattered across the roof of my mouth. They are really sore. This doesn't feel like the strep I've had before and my lymph nodes aren't swollen. The dots aren't swollen or pussy or in my tonsils, just the hard roof of my mouth. I had a cold for about a week and a half but my sore throat is better now. I used zinc and cough drops constantly over the course of the week - could these little sores be from irritation of the zinc and cough drops? I had an STD test for a Visa and I have no STDs. I am very healthy and don't smoke or drink. Doctor: Thanks for using Health Care magic.Read your query.Roof of the mouth or the palate can become sore or it can develop a burning sensation due to may causes which can include drinking or eating hot foods excessively,spicy foods,canker sores ,throat infections or any tooth problems.* Since you have a history of cold and cough ,this could have triggered the formation of this lesion.I would advice you to get your throat infection checked with the ENT surgeon and have it treated .Avoid too much of over the counter medicines.Avoid spicy and hot foods .Avoid irritating the lesions by using toothpicks or by hard brushing.Apply a dentogel /mucopain gel on the lesion two or three times a day,Take nutritious food and lead a stress free life.Hope this was helpful.Thanks and regards."
},
{
"id": 63516,
"tgt": "What is the lump in the back of my throat with fever and weakness?",
"src": "Patient: I have had a sore throat (only on my right side) for about a week, and a lump behind my right ear. I started feeling better until I woke up this morning and now I have some kind of lump in the back of my throat (right side) I have been running a fever, and feeling really weak and achy. Doctor: Hi,Dear,These lumps need to be reviewed and consulted by your ER Ent-Surgeon.The lump in the throat seems to be Acute Tonsillitis.Lump behind right ear may be related with it and could be mastoiditis with abscess.So consultation with ER Ent-Surgeon would fix solution to your intriguing lump problem at 2 sites.This would resolve your query.Write excellent review if you feel that this reply has helped you a lot.Welcome for any further query in this regard to HCM and to ME.Have a good Day.Dr.Savaskar M.N."
},
{
"id": 154158,
"tgt": "What is the treatment for pancreatic cancer?",
"src": "Patient: my friend of 40 years has been diagnosed with pancreatic cancer which has spread to her liver and lynth nodes. she has had a ct scan and has been told nothing can be done although they are going to give her chemo through a stint, she was only diagnosed 4 weeks ago she is now yellow, Doctor: Hi, dearI have gone through your question. I can understand your concern.Treatment of pancreatic cancer depends on type and stage of cancer. In early stage surgery is possible. In late stage treatment of choice is chemotherapy. Despite of treatment prognosis remains poor. Life expectancy is not good. Consult your doctor and take treatment accordingly. Hope I have answered your question, if you have doubt then I will be happy to answer. Thanks for using health care magic. Wish you a very good health."
},
{
"id": 111964,
"tgt": "Lower back pain and vaginal numbness with antibiotics not working?",
"src": "Patient: hi i ave lower back pain like a dull ache an also vagina numbness an discomfort, i am on the pill an in relationship. i am 6 days back into my new packet of pill an have seen a doctor an was precribed antiboicts but dont seem to be working am on my last day of a weeks percription plz help Doctor: Hi there. You are probably suffering from Pelvic inflammatory disease. This sort of pain can be referred to the back commonly. If your antibiotic is not working then you need to get a vaginal swab culture and sensitivity test done along with a urine culture and sensitivity test. Start antibiotics according to the culture report. For back pain take an analgesic and muscle relaxant for the time being. I hope my advice has helped. Good luck"
},
{
"id": 209070,
"tgt": "Suggest treatment for depression",
"src": "Patient: my age is 29 years,5'3.5'i m feeling very lonely and sometimes all r looking fake....my parents finding a groom for me but i m feeling very irritating and sometimes feeling i want to go somewhere far from my near and dear.........sometimes i m crying.........after my sister's merriage i m feeling more depressing d;nt know why Doctor: Hi,I read your query and can understand the distress that you seem to be having due to your symptoms. From what you have mentioned, it seems that you are suffering from a depressive episode which is further worsened by the fact that you are feeling lonely after your sister's wedding.I would suggest that sou seek a psychiatric consultation and start treatment. Anti-depressants like escitalopram n would be effective in your condition. In addition, you should also start exercising daily. You can also start writing diary daily. These methods will help you feel relaxed.Hope this information was helpful. Wishing you speedy recovery."
},
{
"id": 13372,
"tgt": "Suggest treatment for red rashes on joints and armpit",
"src": "Patient: Hai docter. i am shaji from Harippad Kerala Alappuzha( dist).Sir last five month s i am using cosmelite cream so i have a better result in my face &skin .But i stopped the cream last two weeks then i have growth red rashs in my joints & armpit. How to recover from side effects of Doctor: Hello, Perhaps you have a fungal infection. I suggest you use an otc topical antifungal e.g. Clotrimazole cream. Hope I have answered your query. Let me know if I can assist you further. Regards, Dr. Kakkar S., Dermatologist"
},
{
"id": 191275,
"tgt": "Suggest economically reasonable ways of controlling diabetes",
"src": "Patient: I am a 65 year old type 1 diabetic. My sugars are hard to control, and my doctor suggested a meter that monitors sugars continuously. I checked the meter she recommended, but there were monthly fees and automatic updates with it. Seemed expensive. Any alternatives? Doctor: Welcome to HealthcareMagicplease see that there are many glucometers available which can be bought and you need only needles to prick to take sample .but you have to monitor your sugar level and need insulin regularly your sugar must be within normal range if you have any further query then you can come back to HealthcareMagic i will definitely help you and guide you appropriately .With regards dr varinder joshi."
},
{
"id": 172246,
"tgt": "What causes pain during urination in child?",
"src": "Patient: my 2 1/2 year old is complaing about his pee pee hurting ...his little balls are red and really hard and when i touch them he crys it hurts boo boo he says ......the doctor not in today and waiting on a nurse to call me but want to take him to er....what could this be im very concered Doctor: Hi,Thanks and welcome to healthcare magic.You history suggests that he may be suffering from orchitis.He needs antibiotic and pain killer like ibuprofen .Better take him to ER immediately .Hope this answer serves your purpose .Please feel free to ask further queries if any.Dr.M.V.Subrahmanyam."
},
{
"id": 124022,
"tgt": "Should i continue the medication for body cramps even after symptoms disappeared?",
"src": "Patient: I am a type 2 sugar patient. One year back i used to have body cramps especially in shoulders and back. Doctor advised me to take one capsule of Mega neuron OD Plus daily. I am taking the same now close to 15 months. Wants to know how long I need to continue the capsule. I feel quite okay now. Doctor: Hello, As you been taking the medicine for so long, I hope you may have not had any symptoms of cramps for now. Before stopping the medicine you need to consult the diabetologist who can do a physical examination and blood, urine routine then might decide the future of the medication. Also, a team of diabetologist will include a physiotherapist who can help you improve the metabolism and also control the blood sugar levels with exercise. Taking too much medication isn't as good as per science. We need to take control of our diabetes by - diet and exercise too. Hope I have answered your query. Let me know if I can assist you further. Regards, Jay Indravadan Patel, Physical Therapist or Physiotherapist"
},
{
"id": 84186,
"tgt": "What are side effects of tablets for delaying periods?",
"src": "Patient: hi my doctor gave me some tables to delay my periods as i was on holiday and it said i should cum on 3 days after stop takin them which was was sunday and i havent but todaay i been sleep feeling sick bad stomach cramps and sweeling stomach and heart burn Doctor: HiYou can wait for 7 to 10 days after stopping the drug, for your periods to resume.Abdominal cramps can occur with the drug and will settle in a couple of days.Hope I have answered your query. Let me know if I can assist you further. RegardsDr.Saranya Ramadoss, General and Family Physician"
},
{
"id": 203640,
"tgt": "How can i get rid of side effects of masturbation and gain weight?",
"src": "Patient: sir i am 19 and due to over and excessive masterbation lost my weight and face smartness......i was 1 of d most powerfull and smart boy of my school but i find myself now as a thin and weak boy. plz can tell how to get over it and regain my weight. i feel shy telling my parents my problem so plz help me out. Doctor: Hello Masturbation is a normal method of sexual self gratification . There is nothing like over or excessive masturbation. You have done what most other individuals do in the world. I suppose you have gained height so looking to be thin . Please don't have guilt feelings because you have done nothing wrong. Please take good and balanced diet and play a outdoor game.DR SAATIISH JHUNTRRAA"
},
{
"id": 48971,
"tgt": "Could nephrolithiasis be the reason for discomfort in the leg and heaviness on testes?",
"src": "Patient: is there any way i can get away with discomfort caused by varicocele, my case is different, its on the right side. i suffer discomfort in the leg and i feel heaviness on my testes. and my kidney ultrasound impression is nephrolithisis on the right, is this a complication or not? Doctor: Hello! Welcome to HCM.Varicocele and kidney stones are two different things.Discomfort in legs & heaviness in testis can be due to varicocele.In my clinic, I advise operative treatment for symptomatic varicocele.For kidney stones, I advise renal function tests, urine routine, X-ray KUB, ultrasound abdo-pelvis to know size & location of stones & hydroureter/hydronephrosis if any.. these are back pressure changes in ureters & kidney pelvis causing their dilatation due to distal obstruction to urine outflow by stone.Treatment is done according to the diagnosis.Hope this helps.Wish you a good health.Thanks.Regards."
},
{
"id": 200765,
"tgt": "What causes drop in blood count after prostate removal surgery?",
"src": "Patient: My husband s blood count is 24 after 4 days for surgery for removal of prostate. He wants to go home but dr wants to keep him. Is there anything that can be done to raise the level. And if he was given transfusion today would he have to stay in the hospital? Doctor: Thanks for asking in healthcaremagic forumIn short: Your report is incompleteExplanation: Blood count cannot be just 24, please send me complete report of your husband for proper suggestion. Stay in hospital depends upon the condition of the patient and that can only be done after physical examination. So, consult your doctor and trust him."
},
{
"id": 74273,
"tgt": "What causes shaky after taking advair for bad cough?",
"src": "Patient: I have had a bad cough for quite a while and the doctor gave me Advair because he thought my lungs were probably inflamed from all the coughing. I thought he told me to use it twice a day but now I am feeling shaky. Should I only use it once a day. I do not have asthma or copd Doctor: Advair is a combination of Fluticasone, a corticosteroid, is the anti-inflammatory component of the combination, and salmeterol, a long acting beta-adrenoceptor agonist, treats constriction of the airways. Together, they help prevent symptoms of coughing, wheezing and shortness of breath. It may sometimes cause shakiness but other Asthma medications are causing it more frequently. I would suggest consulting with your physician before changing the dose, it might make more sense choosing a different drug altogether."
},
{
"id": 59236,
"tgt": "Diagnosed fatty liver in pancreas. Can any online doctor help me to read the results?",
"src": "Patient: Hi,I am male/37 years/diagonised fatty liver(164 mm)& fatty infiltration in pancreas,normal total bilurubin,but slightly elevated sgpt,56 u/l,high uric acid ,9.2mg/dl,low platelet count,0000,lipid profile quite normal,slight central obesity,fasting sugar 113 mg/dl.Feeling worse physically.what should I do? PLEASE SUGGEST THE MANAGMENT Doctor: Hi and welcome to HCM. You should only change your lifestyle and dietary habbits. Fatty liver is common condition. It can be found in healthy people, but usually is resulting from elevated blood lipids, overweight and elevated blood sugar. Do exercise, avoid alcohol,smoking, fats,fast food and carbonated drinks. Eat more fruit and vegetables, boiled meat and follow some of hepatoprotective diets. Findings usually improves on such measures. If not, then some medicines can be presribed, but I suggest to go step by step. This is nothing serious and in the beginning it is completely reversible. Wish you good health."
},
{
"id": 184434,
"tgt": "Does pain in wisdom tooth require its extraction?",
"src": "Patient: I have been advised to get my wisdom teeth removed since the age 25 .I am now almost 50 and am having slight pain in the lower left wisdom tooth amd also the upper left .I bite my inner cheek often if I dont eat with care. Do I have to extract my W teeth ? I am 5 feet 3 inches 145 pounds and I take Levothyroxine 88 mcg /day.I also took vit D prescription medication for three months and now I have a good vit D result. Doctor: Thanks for your query, I have gone through your query.The cheek bite could be because of abnormal position of the wisdom tooth in the arch. The wisdom tooth can be removed if it is troubling, since it is a non functional tooth.Consult a oral maxillofacial surgeon and get the tooth removed under local anesthesia. You can take a course of antibiotics like penicillin 500mg tid for three days(if you are not allergic)I hope my answer will help you, take care."
},
{
"id": 61650,
"tgt": "What causes lumps on knee incision area?",
"src": "Patient: I had ACl surgery about 5 yrs ago my knee has never felt right since. however lately I have been getting a lot of pain below and to the inside of the knee near the insision. Today I noticed near the surgical insision there has have formed a large hard lump,and the pain has subsided. What could be the cause of this lump forming? Doctor: Surgical intervention for joint after some years can cause ofosteo- arthosis of the affected which in turn may initially cause lot of pain limiting the movements with joint fluid acculumating. This causes sac Bursa around the knee to swell once the joint space exceeds its capacity and presnts as a Swelling like a cricket ball . Please get it confirmed and addressed by orthopaedic"
},
{
"id": 193915,
"tgt": "Does bigger chest size than stomach for an overweight person suggest gynaecomastia?",
"src": "Patient: hi i am 25,and little overweight 71kg but my chest is big then my stomach,am i suffering from gynecomastia.my height is 5.6.i do not suffer from any other diease.i was underweigh till 11 but after leaving hostel in 5th std my eating completely changed and became an overweigh and since my stomach appears small then chest.i also don't exercise regularly. Doctor: Hello, The chest muscles of an overweight person tend to sag a little and that is never gynecomastia. Continuous exercise and chest muscle tightening exercises are the best. Hope I have answered your query. Let me know if I can assist you further. Take care Regards, Dr K. V. Anand, Psychologist"
},
{
"id": 205489,
"tgt": "Suggest treatment for disorientation caused by drug abuse",
"src": "Patient: A few years ago I had a phase where I experimented with a lot of drugs. Although I wasn t an addict I certainly abused them. My ability to engage in conversation and my once big imagination and creative streak is now more or less non existent. I believe this is a result of a large ketamine and LSD intake and it may have damaged my brain. I stopped taking drugs 2 years ago in hope that it would improve but it hasn t and I m really worried because it has effected the way I act, my confidence and how I feel about the future. I slur my sentences a lot even when they are simple, my memory has collapsed and I find it hard to immediately process anything anymore Doctor: it may have some side effects but they can be at low dose also.i generally add on some other drug instead of increasing dose.u consult your psychiatrist."
},
{
"id": 54718,
"tgt": "What causes fatty liver?",
"src": "Patient: I had an ultrasound today and it states I have a \"fatty\" liver. What does that mean exactly, and my gallbladder is 3mm in thickness with sludge visualised. here is a copy of the report... REASON FOR EXAM: Right upper quadrant pain. Nausea. History of diabetes. The gallbladder wall measures 3 mm in thickness. Slight sludge is noted. No definite gallstones. Liver shows increase echo pattern. No liver mass. Portal vein shows antegrade flow. Common hepatic duct measures 1.7 mm. Common bile duct measures 7.1 mm. No intrahepatic biliary duct dilatation. Visualization of the common bile duct is suboptimal on this study. Pancreas is obscured by bowel. No free fluid. IMPRESSION: Gallbladder shows a thickened wall and this large. No gallstones. Correlate with the clinical findings and if needed with hepatobiliary scan. Normal caliber common hepatic duct and intrahepatic ducts. Bile duct is slightly dilated measuring 7.1 mm. Because of the poor visualization, the size of the common bile duct could be over estimated because of technical factors. Echo pattern of the liver is increased. Consider fatty and or fibrous infiltration. Doctor: Hi thanks for contacting HCM.Noted you have fatty liver disease.Gall bladder thickness increased might be by chronic inflammation...Fatty liver exactly means steatosis.In which fat accumulate in hepatocytes...There are few causes for it like high cholesterol, diabetic , drugs , alcohol etc.You need to change life style for its treatment.Take less fried food.Use less oil in food ...Use only good oils like sunflower or canola oil.Regular exercise.Avoid alcohol....Take udiliv 150 mg tablet daily...Regular follow up ...Usually with above measure it will resolve in few monthsDr.Parth"
},
{
"id": 95989,
"tgt": "I am suffering from sharp chest pain. I am 17 yrs old girl. Should I be worried ?",
"src": "Patient: I have sharp pains in my chest and sometimes in my stomach rib area. and im a 17 year old girl. should I be worried ? Doctor: Hi, Pain may be due to chosto condritis but you go for ECG to rule out coronary cause. if it is normal then consult your orthopedic surgeon."
},
{
"id": 100145,
"tgt": "Why do I keep coughing after I have quit smoking?",
"src": "Patient: I am 39 years and quit smoking 7 years ago. I never coughed, coughed anything up, etc after quitting. I have always had chronic sinus issues and been on lots of antibiotics and take mucus relief fairly often. I get shortness of breath out of nowhere. I have inhalers, and albuterol for nebulizer, prednisone, etc to help, but it takes a while to get back to breathing normal. I excercise regularly and occasionally get bad headaches from shortness of breath. Been to doctors and they just tell me my lungs are clear and give me an antibiotic and/or an inhaler. Doctor: The inner lining of our sinuses as well as the respiratory tract has lining of small projections called cilia which beat in a definite direction and help in bringing out the dirt and mucus. This is called muco ciliary clearance. But the smoking has damaged the ciliated epithelium and hence this clearance is disturbed. Hence you need to cough out the thick mucus instead of it automatically coming out even after you have quitted. This is also responsible for the sinusitis and the infective agents do not get cleared out from the sinuses. Since the lungs have also been damaged to some degree while you were smoking, you sometimes have respiratory distress or shortness of breath. It will take significant time to recover and will recover only to a near normal stage. You will have to be patient till then. Steam inhalation on regular basis may help."
},
{
"id": 122998,
"tgt": "How to treat dark brown rash that turns red is also itchy on hands?",
"src": "Patient: My 32 year old daughter developed a flat dark brown rash that turns dark red and is not itchy. It is on both sides of her hands and up one thumb. . Has been there for 2 weeks and is non changed . She works in Many Daycares daily , could this be a virus ? Doctor: Hi, It could be a contact dermatitis that occurs due to the contact with several materials at work or at home, materials that we make contact in our everyday life. She should meet an allergist or dermatologist and do a Patch test. Hope I have answered your query. Let me know if I can assist you further. Regards, Dr. Gjustina Loloci, Allergist and Immunologist"
},
{
"id": 111587,
"tgt": "What is the remedy for the pain in neck, shoulders and back?",
"src": "Patient: Hi sir, gm, myself Amith from bangalore. I was suffering from neck,shoulder and backpain in the right side 2 years, which I consulted and got cured. Now after 2 years the same pain has started again, in past Doctor has told if the pain repeats then I should undergo MRI scan, so I just wanted to know the cost of MRI scan for the back portion. Thanks in advance Doctor: Hello and welcome to health care magic. Your doctor is suspecting cervical spondylosis and he want to rule out any Vertebral disc prolapse through an MRI. The cost of MRI spine various from different hospitals depending upon the machine they use. Bigger hospitals use high resolution machines with higher kilo Tesla magnets and the cost will be higher, but we will be able to see the minute details. Typically the cost varies from Rs 5000 - 10000. Hope this discussion will be helpful to you. Thank you."
},
{
"id": 28008,
"tgt": "Suggest treatment for rapid heart rate",
"src": "Patient: Hi there! So as of recently, maybe in the past two weeks or so, every time my heart beats faster than it normals does, it has this weird feeling to it and if it's really quiet you can hear it squeaking. For example, if I stayed in bed all day and get up to go downstairs to the kitchen, when I get downstairs I hear the squeaking with every heart beat. I'm turning 17 in a week so I assumed it wasn't any serious heart condition but it wouldn't hurt to be 210% sure. Doctor: Dear Patient, Thank you very much for your question. If your heart rate is 100 or above beats per minute, it is called as tachycardia or your heart is beating faster than normal. There are many causes for tachycardia. 1. Anxiety and severe stress.2. Hyperthyroidism.3. Anemia.4. Damage to heart tissue during heart disease.5. Smoking and Alcohol intoxication.6. Fever etc. Also, there are many types of tachycardia,too. Atrial Fibrillation, Atrial flutter, Supraventricular tachycardia (SVT) are some of them. In order to diagnose these conditions and differentiate them taking an ECG test is mandatory. Therefore, in your case, I would recommend you to consult your family doctor and take an ECG. Also, I would recommend you to do a thyroid test because many young individuals those who have atrial fibrillation have hyperthyroidism. Do not worry. Therefore,see a doctor soon and check your thyroid and do an ECG. If your conditions are related to anxiety or severe stress, your doctor will direct you to a Psychiatrist. Vagal maneuvers can slow down your heart rate. Putting an ice pack on face , coughing are examples for Vagal maneuvers which can slow down your heart rate during palpitations. Anyway, I again do not recommend you to rely on those temporary solutions. Seeing a physician and observing his or recommendations is the best solution. I hope that I answered your question.Good Luck!Dr. Dinesh Nuwan Weerasinghe. MD"
},
{
"id": 217908,
"tgt": "What cause anus hurts when sneezing?",
"src": "Patient: Hello, for a few days my wife has been complaining that when she coughs or sneezes her anus hurts, she also states that her anus is inflamed, it hurts when she sits on a wooden chair, there is no blood in her stool or on toilet paper when she wipes, we have tried preperation H and that has not done anything Doctor: Hai, pain while cough and sneezing, is due to increase in intra abdominal pressure. She could have a possible fissure or any other radiating pain.u can consult a general surgeonundergo pr examination, or proctoscopy"
},
{
"id": 207780,
"tgt": "Suggest treatment for lack of organizational skills",
"src": "Patient: i am a 26 year old female i weigh 128 pounds, and i have 3 kids. i am a stay at home mom, and i can never get motivated. i try to clean my house and never seem to get finished because i always switch rooms. i cant seem to get organized, and it is very cluttered in my home!!! i forget so many things!!! i seem to zone out all the time. i am on cymbalta but i am not depressed. what is wrong with me. Doctor: DearWe understand your concernsI went through your desciption. I suggest you not to worry too much. First of all you must understand that you should not diagnose yourself for any mental health issus. Secondly your symptoms do correspond to Obsessive Compulsive Disorder, you need to get an expert diagnosis. Psychologists use psychometric tests to assess mental conditions. Please consult a psychologist and he shall help you diagnose.If you need more of my help in this regard, please post a direct query on this portal. I am happy to help you.Hope this answers your query. Available for further clarifications.Good luck."
},
{
"id": 163101,
"tgt": "Suggest treatment for fever and skin rash in a child",
"src": "Patient: My 8 year old has been running a fever for almost 48 hours now. Her temp is now at 103 degrees, and she has developed a rash on her chest. She also has the symptoms of a cold. She is drinking fluids and has cold washclothes on her head and legs. What should I do? The fever does not seem to be breaking. Doctor: Hello,Sometimes just drinking fluids, cold baths, cold washcloths help but are not enough so you should use paracetamol or ibuprofen syrup for the temperature. About the rash, in the chest, I think it comes from the temperature. If your child has more than 3 days with high temperature you should consult to your doctor.Hope I have answered your query. Let me know if I can assist you further.Regards, \u00a0\u00a0\u00a0\u00a0\u00a0Dr. Elona Dashi"
},
{
"id": 126730,
"tgt": "How can cold feet due to neuropathy be treated?",
"src": "Patient: my husband is a Va Vet and we have gone to the podiatrist in utah and very unhappy. my husbands feet get ice cold like no blood flow, other days they are warm they say he has neuropathy but pretty much nothing to do for it there are days the skin feels like itis being peeled of burning and can t walk some days. we need help from someone. Doctor: Hi, The cold feet due to neuropathy can improve with methylcobalamin supplements and by treating the cause of neuropathy. Hope I have answered your query. Let me know if I can assist you further. Take care Regards, Dr Praveen Tayal, Orthopaedic Surgeon"
},
{
"id": 190925,
"tgt": "What treatment is suitable for tooth decay and gum abscess ?",
"src": "Patient: please help me , since from my front middle upper teeth from root some white pulse continously coming since i shown to doctor he insert calciaum hydraoxide but still problem remain same. since this theeth was broken at child hood when i was 8yrs now i m 26yrs old.since doctor told me to remove this teeth but i dont want to remove . please guide me what to do. Doctor: Hi, since there is a history of trauma to the tooth, there is formation of periapical abscess. That is why pus is oozing out from the root apex. The pulp tissue inside the tooth and root have necrosed and since become infected resulting in pus formation. The tooth needs root canal treatment. The infection can be treated with this treatment and the broken part can be restored by the help of a ceramic crown after the treatment. Please consult a good dentist. All the best!"
},
{
"id": 199167,
"tgt": "What care and precautions to be taken post circumcision?",
"src": "Patient: Hi doc..I m a 20 year old male and I ve been circumcised this morning. I just wanna ask is it safe to sleep on your stomach or should I just sleep on my back?.. After reading and understanding the procedure to do after circumcision, is it okay to treat and clean your penis at home or should I just go everytime for check-ups at my nearest clinic or doctor? Doctor: DearWe understand your concernsI went through your details. Circumcision is a minor surgery. Still you need to adhere to the hygienic healthy procedure associated after surgery. For atleast aweek you should take precaution so that there should not be any infection. For cleaning and dressing, please approach a nursing home near you or go the same hospital.If you still need my assistance in this regard, please use this link. http://goo.gl/aYW2pR. Please remember to describe the whole problem with full detail.Hope this answers your query. Available for further clarifications.Good luck."
},
{
"id": 181371,
"tgt": "Is wisdom tooth extraction advisable while on Fosamax?",
"src": "Patient: I took Fosamax from about 1995 thru 2003. Because of acid reflux problems my dr. switched me to Evista. Now I need to have a wisdom tooth extracted and am worried about jaw problems because of having taken the Fosamax. The area is not painful. The tooth has just broken. Am wondering whether to go ahead with the extraction. Doctor: Hi..Can understand your concern..As per your complain Fosamax contains Alendronic acid and is a bisphosphonate and is seen to cause osteonecrosis if dental extractions are carried out when it is being taken..On the other hand Evista contains Raloxifine and is Selective estrogen receptor modulator and it also alters the bone loss but is not seen to cause osteonecrosis, therefore you need not to worry about getting extraction done as you have already been off Fosamax for more than ten years.But for being on a safer side consult an Oral Surgeon and get evaluated and a thorough clinical examination and evaluation for bone density has to be done for diagnosis and treatment can be done accordingly..Hope this information helps..Thanks and regards..Dr.Honey Nandwani Arora."
},
{
"id": 53003,
"tgt": "Suggest treatment for fatigue,diarrhea and nausea after surgery",
"src": "Patient: Hi, I had a whipple procedure done in 2009 at age 56(benign mass on the head of the pancreas) had my gallbladder and part of my stomach removed. nearly 2 years down the line and trying kreon and nexiam, i have feeling weak,tired, increased pulse rate, weight loss, constant diarea,nausia and labour type cramps. I burbe alot and recently had another scan done. was told that there was no recurrance other than hernia and polyps on my colon. I am due to have a colonos--that word shoild i be worried Doctor: Hi, I had gone through your question and understand your concerns.Whipp\u010de is extensive and complex surgery with clear negative efect on digestion so certain degree of abdominal discomfort is expected. it looks to me like malnutrition syndrome which has all these symptoms and this occurs in about 30 percent of patients who undergo whipples procedure. usually this is transient, and rarely persistent or lifelong. However, tumor recurence should be ruled out first. Some individuals may benefit from diet modification, such as a reduced fat diet, following cholecystectomy since it may be more difficult for digestion of fatty foods. Typically, the patient is recommended dietary restriction table with fatty foods, enzyme preparations, antispasmodics and Kreon pills.ALso, octreotide may be beneficial but is pretty expensive medication.Hope this answers your question. If you have additional questions or follow up questions then please do not hesitate in writing to us. I will be happy to answer your questions. Wishing you good health."
},
{
"id": 115695,
"tgt": "What is the treatment for Anti-nuclear antibody?",
"src": "Patient: my mummy is having her Antibodies antinuclear test is positive so doctor asked her to take other tests and her result is SGTT 29 RAF is negative anti CPP is 3.7 CRP negative and she is having swollen hands and legs with joint pain. Can u please guide us what the problem is. Doctor: Hello,A positive anti-nuclear antigen (ANA), and a history of swollen hands, swollen legs, and joint pains suggest an autoimmune disorder. There are a number of autoimmune disorders which show positive ANA. The closest differential of above-mentioned symptoms is rheumatoid arthritis. Rheumatoid arthritis is characterized by joint pains especially the joints of hands. Rheumatoid factor (RF) is usually positive in rheumatoid arthritis however, it can be negative in a large number of cases. I suggest you consult your rheumatologist for further workup.Hope I have answered your query. Let me know if I can assist you further. Regards, Dr. Shailja Puri"
},
{
"id": 60378,
"tgt": "Can any online doctor tell me about my blood report ?",
"src": "Patient: hello doctor ..im 28 yrs age frm nepal..my current report is Hbeab+ve, Hbeag-ve, Total brilium=0.5mg/dl, c brilium=0.10mg/dl, ALT=19.0 iu/ml, AST=23.5iu/ml, Total protin=7.3g/dl, Albumin=4.8g/dl...and HBV DNA Viral copies/ml plasma=750...so doctor i want to know my stage of disease and condition of liver ....should i start medicitions..?..if i start how long i ll take an what is it cost per month..? Doctor: u r well enough,but if u want more about ur liver profile u can go for liver MRI .after all u can have some ayurvedic treatment. arogyavardhini vati-2tab tid with warm water yakrit plihari lauh - 2tab tds with warm water kumaryasav - 15 ml +15ml water bd after meal"
},
{
"id": 10972,
"tgt": "Suggest remedy for hair loss",
"src": "Patient: Hi, may I answer your health queries right now ? Please type your query here...Currently I am 21.5 years old Hi I suffered from Typhoid in July 2010.Hair fall started around September mid. In early days it was heavy hair loss. The bedsheet and pillow used to have a lot of hair strands.I used a homeopathic medicine and got my head shaved in Feb 2011. After that there was no further hair loss. But now again since 1 month I am observing hair loss. Around 10-15 strands per day. Density of my hair has also reduce.Kindly guide me. Doctor: Hi,Welcome to HCM..You seem to have telogen effluvium. You had enteric fever in past. Due to that you had severe hair fall. But, then you were alright. Now you got hair loss again. The reason may be different. It may be tension,vitamin deficiencies,anaemia,thyroid dysfunction or some other. You consult dermatologist to have perfect diagnosis. After thorough examination and investigations,some conclusion may be there.I would like to suggest as below..- biotin 10 mg tab daily- vitamin E 400 mg daily- if needed..vitamin b complex and iron preparation - if some cause is found..treatment may be taken- mild steroid lotion to apply on scalp at night- herbal shampoo and oil may be usedHave patience for the good result. Take long time treatment.I hope this would help you.Thanks.Dr. Ilyas Patel MD"
},
{
"id": 79356,
"tgt": "What treatment is suggested for cough?",
"src": "Patient: i have noticed that i developed a cough, when i cut out splenda the cought stopped and stayed gone for a while,i did not stop drinking diet soda but now the cough is back. Again i stopped the artificial sweetner, is this just another side effect of splenda/artificial sweetners? Doctor: Thanks for your question on Health Care Magic. I can understand your concern. No, cough is not side effect of artificial sweetener. It is mostly due to respiratory tract infection. Most common is viral upper respiratory tract infection. So better to avoid oily and spicy food. Avoid junk food. Drink plenty of fluids orally and keep yourself hydrated. Start antihistamines and anti inflammatory drugs. Do warm water gargles 5-6 times a day. You will mostly improve in 1 week. If not improving in 1 week than get done chest x ray to rule out lower respiratory tract infection (LRTI). Hope I have solved your query. I will be happy to help you further. Wish you good health. Thanks."
},
{
"id": 96572,
"tgt": "What do headache, a numb mouth, sweating and an inability to stand indicate?",
"src": "Patient: I have had to go to the emergency room 3 times in the last month. The problem starts out the night before where I sweeting badly by the next morning my mouth becomes numb and my tongue I have a sinus headache I have a problem breathing I become really weak and lose my ability to stand. The next thing I know I having convulsion. My feet and my anal area start having a burning pain and my hands feel as though their turning numb. After awhile at the hospital everything starts to return to some what normal. All the tests they do at that time come back normal. What do you think is happening to me? Doctor: HIWell come to HCMI really appreciate your concern, the only matter of concern is convulsion if this is the convulsion then better to get done the CT of brain and EEG just to rule out the late onset of epileptic seizure if it is not detected anything in tests then it can be hysteria, it could be due to general weakness, but it does not seems to be anything serious, in differential diagnosis, anxiety and depression is likely, you have not given any basic information like age and gender else something better could have been thought hope this information helps."
},
{
"id": 31801,
"tgt": "Can dengue re-occur after treating it?",
"src": "Patient: I had Dengue about 5 months back. During the period I had severe myalgia. Now I played cricket after about 5 months and I had similar kind of pain in the muscles which has lasted for about 7 days now. The pain reminded me of Dengue muscle pain. Does it happen like that Doctor: Hello and welcome to HCMI have read and understood your queryI hope this will help you.Dengue fever is caused by DENGUE VIRUS transmitted through bite of mosquito AEDES AEGYPTI.July to Oct are the months in which the disease has peak incidence.. The virus has got 3 subtypes.. Once affected by a typical subtype of the virus causes life long immunity in our bodies..but infection with a second strain causes most severe form of the disease called dengue hemorrhagic fever..as you previously had the disease you will be knowing that disease accompanies high grade fever..At the moment from the history provided by you I don't think that's the case.I think you should consult your local doctor.. As any febrile illness can cause muscular pains especially viral illnesses..I don't think that you are having a second attack of dengue fever..I would advise you to have a temperature record of yourself and mean while consult your GP get a full blood count investigation advised and proceed accordingly..Mean while tab paracetamol 1 tab 3 times day will be helpful for you.I hope I answered your queryYou can contact me any time for more questions.ThanksDr faeza"
},
{
"id": 165390,
"tgt": "What causes hematuria?",
"src": "Patient: Hi, My 5 year old daughter has blood in her urine. This has been happening throughout the day. I have been to hospital with her and have provided the doctor with a urine sample which will be tested and back with my local GP on Wednesday. I am just beside myself with worry. The GP said in most cases it would be a urine infection. The waiting is distressing. She is very pale and off her food and also has tonsilitus. Please can you offer any advice on what this may be or what to ask the gp to further investigate? Best regards, Julie. Doctor: Episode of hematuria, which usually starts within a day or two of upper respiratory tract infection like tonsillitis may be seen in disease called IgA nephropathy.Though other more common cause of hematuria (blood in urine) are urinary tract infections, kidney stones, bleeding disorders etc.Ask your doctor for investigations like urine examination, USG abdomen, KFT, CRAP, ESR etc.to confirm diagnosis."
},
{
"id": 45926,
"tgt": "Is postponing kidney stone removal surgery safe?",
"src": "Patient: my college-aged daughter has a 5 mm kidney stone in her right kidney. They have worked her in to have it surgically removed (basket procedure) next week, and will have a stent for a week afterwards.She has a required field trip over that weekend and she doesn t want to have to endure that while having a stent (she had a stent for a month last semester and was miserable the entire time). She had a kidney infection the last week of January,the ultrasound they did then found this stone. She has had one short bout of pain/nausea since then (2-3 hours). She wants to wait until after the field trip to have the surgery. The next time available would be the week of March 21.Would it be dangerous to wait? Doctor: Hello and Welcome to \u2018Ask A Doctor\u2019 service. I have reviewed your query and here is my advice. Yes it would be safe to wait until after the fieldtrip. Stent sometimes causes irritation in the bladder and urination. Hope I have answered your query. Let me know if I can assist you further."
},
{
"id": 114949,
"tgt": "Suggest treatment for myelofibrosis",
"src": "Patient: My dad has myelofibrosis. His platelet count is ridiculously low. His spleen is enlarged and he s been complaining his left side below his ribs are hurting. I think its his liver. His doctors are doing great but his pain keeps getting worse. He was diagnosed last April. Do you think with these signs he s getting close to the end? I dont want him to go but dont want him suffering either. Any info will help my family. Thank you Doctor: Hi, dearI have gone through your question. I can understand your concern. Myelofibrosis is one type of bone marrow cancer. In that blood producing cells are replaced by fibrosis. Treatment options are stem cell transplantation. Other options are splenectomy, androgen and alkylating agents. Despite of treatment prognosis remains poor. Life expectancy is not good. Consult your doctor and take treatment accordingly. Hope I have answered your question, if you have doubt then I will be happy to answer. Thanks for using health care magic. Wish you a very good health."
},
{
"id": 5048,
"tgt": "Trying to conceive. Had miscarriage and abortion. Will it affect pregnancy? When does one ovulate?",
"src": "Patient: hi i am 29 a mother to a 4 year old and me and my partner are trying for another baby, my partner is a HGV driver which means he is sat down all day then when he gets home he is atired from work so will just lie down on the sofa until bedtime i have heard that being sat down for long periods reduces your sperm count is this tru? Also i have been reading through loads of info from the internet but still cannot figure out when i will be actually ovulating each month is it 2 weeks prior my period or 2 weeks after and if it is after do i count 2 weeks from the day my period starts or the day my period ends. We have fallen pregnant twice before whilst i was on the pill the first time round unfortunately we lost it and the second time around we were not financially ready my partner had just lost his job so we decided to have an abortion but now im worried that this may have effected my body someway as we have been trying for 4 months now - well 4 months since i stopped taking the pill and still nothing i thought we were quite fertile considering we fell pregnant twice before whilst on the pill but that was 2 + years ago and my partner was not a HGV driver then so r all these causing barriers in my falling pregnant? Please help dont know if i can face another month of disappointment :( thank you, Jemma. Doctor: hello,thank you for using healthcare magic,if you have consulted gynecologist in past for miscarriage and abortion and it was done properly then it does not reduces your fertility.. perhaps you might take little long to concieve because of difficulty in implantation..O.C.pills does not reduces fertility neither sitting for long time reduces sperm count..after leaving o.c.pills it take around month to obtain your complete fertility.. if your cycle is of 28 to 30 days then you will ovulate on 13th to 15th day.. usually on 14th day...you can confirm by it looking your vaginal secretion it will become thick, take your temprature on morning before getting out of bed.. start taking it from 12th day.. if ovulation occurs your temp will increase by 0.5 to 1 celsius..12 th to 18th day of your cycle considering first day of your period as a first day of your cycle will be fertile.. during this period have intercourse 3 to 4 times or more if possible...keep trying for at least a year then too you are not able to concieve then you need to worry..hope this helps you,Regards,Dr.Nehal.."
},
{
"id": 15242,
"tgt": "Allergy on right hand, rash with white dots, itching. Medication ?",
"src": "Patient: sir i have an elergie on my right hand . i think its from sun because in winters it will be normal skin but when summers comes . my right hand is like some rash with white dots ... its itch to much and it is from last 4 to 6 years .. i consult many doctors and homopethic also .... but its nt good .. and sir i am using ( flutibact skin ointment cream ) its not to much good but it can stop itch.... what i do now ??? plezz sir reply me as soon as possible .... Doctor: Hi, it appears to be the seasonal allergy allergy to the foods fruits, cotton dress or flowers of the summer season. When it has already there you can consult a dermatologist fordiagnosis and treatment. To prevent it try to trace out the things allergic to you by avoiding the things you usually use in summer. And avoid them for ever. Thank you."
},
{
"id": 87934,
"tgt": "Suggest cure for severe pain in the belly button & nausea",
"src": "Patient: My 14 year old son has had deep pain behind his belly button and intermittent waves of nausea (about a 6 on a 1-10 scale) constantly for a week to the point that he can't run track.X-ray was done yesterday (1/14/11)and shows \"nothing\".Doc says to go to ER if he gets a fever or begins vomiting.Sounds like possible appendicitis according to the symptoms I've read online but hate watching him feel bad and have no course of treatment. Can it be diagnosed (or ruled out) in any other way rather than waiting for it to get worse? It hasn't worsened or improved in a week's time. Doctor: Hi,From history it seems that he might be having acute gastro-intestinal infection giving this problem.In my opinion he requires one course of Ofloxacilin, tinidazole combination medicine for 3 days.Give antispasmodic medicine like Meftal spas or Cyclopam for pain.Give him light diet.Ok and take care."
},
{
"id": 144586,
"tgt": "What could be the cause of sudden behavioral changes?",
"src": "Patient: My mum had a stoke about two years ago but got better there was some issues with her left hand but everything else was good this week her hands turned blue the next day she was talking and saying things that did not make any sense and now I have been told she has a shadow on the right side of the brain and her oxygen is low. She also got violent today in the hospital. Visitor told me she was talking about her childhood today Doctor: If stroke occur in specifically frontal lobe, there can be immediate behavioral changes. Also similar symptoms can be due to involvement of speech area (broca's & wernicke's area of brain).Few patients also get secondary depression after stroke.I advice you to consult the neurologist."
},
{
"id": 167669,
"tgt": "What causes numbness in the right hand?",
"src": "Patient: Hi, my almost six year old daughter has said 2x today that her right hand felt numb and prickley. Once in the morning and once in the evening. She has done nothing out of the ordinary today or yesterday. She is otherwise healthy. Is this a sign of a serious medical problem and can a six year old get MS? Doctor: DearWelcome to HCMWe understand your concernsI went through your details. Numbness and prickly feeling of right hand should be investigated. But nothing to worry about. Check the child's writing position and hand position while sleeping. these are tow basic causes for numbness.If you require more of my help in this aspect, please use this URL. http://goo.gl/aYW2pR. Make sure that you include every minute details possible. Hope this answers your query. Available for further clarifications.Good luck."
},
{
"id": 158213,
"tgt": "Lump above jawline near parotid gland. Was treated for triple negative BC stage 1 grade 3. Anything to worry?",
"src": "Patient: I am 32 years old and was treated for triple negative BC last year, stage 1 grade 3 (invasive ductal carcinoma). Just complete 6 chemotherapy with lumpectomy and 33 rounds of radiotherapy January this year. I felt a 2cm lump above my jawline near the parotid gland on my right face yesterday, not sure was it there before. (BC on left)My appointment with my oncologist for check up is next month. Should i be worry?Thank you in advance. Doctor: Hello.For your background that relates, if you have lump above of jawline near the parotid gland is necessary to rule out a recurrence of tumor.Por please go to your oncologist as soon as possible for examination.I wish you good health."
},
{
"id": 100072,
"tgt": "Does Cortisone cause severe hiccups?",
"src": "Patient: Does Cortisone cause severe hicups, I was given a shot yesterday around 2. I went to bed and had hicups throughout the night and I havent had hicups in a long time. I woke this morning and had them again. Currently I am having very strong hicups. Ive had them total of 5 times today. What should I do Doctor: HIWell come to HCMIrritation of diaphragm never is likely and this could be due to hyperacidity and some time be may nonspecific, and condition can be treated accordingly most of the time this gives good response to proton pump inhibitor, if this fails to response then this need to be clinically examined, have a nice day."
},
{
"id": 134901,
"tgt": "What caused the Heterotopic Ossification in my hip?",
"src": "Patient: I WAS IN A CAR ACCIDENT AND BROKE MY PELVIS. I HAD SURGERY THE NEXT DAY. AFTER THE SURGERY THE DOCTOR SAID THAT HE DIDN T GET ALL THE BONE FRAGMENTS AND I WOULD HAVE ANOTHER SURGERY THE NEXT DAY. AFTER MY X-RAY HE SAID HE GOT ENOUGH OF THE BONE AND I DIDN T NEED THE SURGERY. A COUPLE MONTHS LATER MY X-RAYS SHOWED HETEROTOPIC OSSIFICATION IN MY HIP. COULD THE BONE HE LEFT IN ME CAUSE THIS TO HAPPEN Doctor: Hetero topic ossification is known to happen in hip joint and etiology is multi factorial. It would not be possible to identify single factor causing it."
},
{
"id": 173051,
"tgt": "What does blood test results of 22 month old mean?",
"src": "Patient: My 22 mos old son's Hemoglobin A1c came back as 6.0 high % 4.8 - 5.6 01under that info it stateincreased risk for diabetes: 5.7 - 6.4Diabetes: >6.4Glycemic control for adults with diabetes: What does this mean? Does he have childhood diabetes? He also has balanitis which made think he does. Doctor: I wonder if these test was done as directed by a doctor ? Well the high HbA1C of this level is suggestive of prediabetes. But as your child is 22 months old, more tests have to be done to make sure if he is a case of Type 1 Diabetic which also raises doubts of any other family member is diabetic, like father or grand father, The balanitis also arises doubts of sugar in urine. It is inappropriate to make any authoritative comment with just one report as why the report was taken itself is information needed to assess the condition. I would do a FBS, 1/2 hourly Glucose tolerance test after a glucose feed and evaluate the whole family history. Please reassess with your doctor who ordered the test and please com back to me Regards."
},
{
"id": 140385,
"tgt": "What could cause dizziness?",
"src": "Patient: For the past few months, for seconds at a time, I feel very faint - then it goes away. At first it happened 2 or 3 times a month, once or twice each day. It usually occurs when I m sitting or standing still, but today, for the first time, it kept happening while doing pilates exercises at home, and I had to stop. I do have SVT, for which I do not take any medication. I started taking Livalo a few months ago, but stopped after feeling muscle aches and stomach discomfort. However, my internist felt I should try it 3 x a week with COQ10 because my cholesterol rose from 190 to 304. The last few years have been very stressful for both my husband and me in terms of health issues and family - I m wondering if it could be due to anxiety, but how do you tell? Doctor: Hi, With your medical condition and worries happening at once it is sometimes difficult to tease things out, however, the right steps to always follow in such a case involve making sure to the extent possible that nothing organic is really going on. Therefore, I believe you should do what you can to control the SVT's. Perhaps, this medication is causing intolerable side effects so you need to have a discussion with your doctor to see if another suitable drug can be chosen. Often times that solves a lot of problems. Second, I would check your orthostatic blood pressures (your doctor knows about this) and make sure any other medications you're taking are not potentially lowering blood pressure, you need to be well hydrated at all times, and not becoming hypoglycemic (i.e. skipping meals for weight loss purposes, etc.). Beyond that, if there are still problems your doctor may consider an ultrasound of the carotid arteries and do other blood work to guarantee there are no blockages in the arteries going to the brain and that you don't have an anemia that should be treated (especially since these feelings have now occurred during exertional activities). Hope I have answered your query. Let me know if I can assist you further. Regards, Dr. Dariush Saghafi, Neurologist"
},
{
"id": 21849,
"tgt": "What is the treatment for heart palpitations?",
"src": "Patient: Strange issue -- I keep skipping very random heart beats, followed by no chest pain but constantly, daily, I seem to have sudden hard 'whacking' hard heart beats in my chest seemingly randomly. Is this an issue I should see a doctor for, even though my mother complains of having the same thing most of her life, however, not as frequent as myself? Doctor: Hi,Welcome, this is Dr Sameer, cardiologist.Skipped heart beat could be in form of premature ventricular contraction (PVC), which are premature contraction from a normal part of heart or could be more serious arrhythmia in form of Atrial Fibrillation.For the diagnosis, you need to do a ECG at the time of this skipped heart beat. If the duration of these skipped beats are short enough to reach a nearby hospital, we advice our patients for a 24 hr ECG monitoring called Holter Monitoring.The treatment will be as per the report of above tests. So get these tests done soon.ThanksTake care"
},
{
"id": 92480,
"tgt": "What is wrong if I have abdominal pain when eating or drinking?",
"src": "Patient: Hi Doctor Samuel I had pain under my left rib cage on Monday lunchtime. Went to hospital and had blood, urine and ECG checked and was clear. Doctor then gave me laxative for constipation and after 2 dosage I had severe runs for a few hours. Runs had stopped but now when I drink or eat it hurts and feels like the food is stuck inside mid abdomen. Also when I walk my left side hurts. Please let me know what s wrong. Jan Doctor: HIThank for asking to HCMIt is nothing to worry at all, hyper peristalsis wave causing such symptoms, while we eat a kind of wave appears in GI system that helps the food to propagate ahead when this wave becomes more active then it gives such kind of pain as you are feeling, you have to eat slow, eat little less in quantity, the purgative or laxative will aggravate the condition, :Dicyclomine\" is drug of choice and this slow the peristalsis wave no need to worry even if you do nothing then condition goes away it self have nice day."
},
{
"id": 107227,
"tgt": "Is Nucynta advisable over Fentanyl patches for severe back pain?",
"src": "Patient: HI I AM TAKING NUCYNTA 100mg ER FOR BACK PAIN. I have also found it to be far more effective than Fentenyal 12mcg patches and as a bonus it is far more effective than Fentynal patches for my peripheral neuropathy. Without these painkillers I would be in constant pain t. I was very careful not to raise my Fentynal dosage over the course of using them for the last 4 years, enduring some occasional periods of pain. My question: is Nucynta more or less effective for long term pain reduction than Fentynal? Thank you. Doctor: Nucynta is the first and only FDA-approved long-acting opioid designed to control both pain and neuropathy. Nucynta is also used in patients for whom alternative treatment options (e.g., fentanyl) are ineffective, not tolerated, or would be otherwise inadequate to provide sufficient management of pain."
},
{
"id": 191993,
"tgt": "Suggest remedy for anxiety when having diabetes",
"src": "Patient: My husband, age 57 , has been recently diagnosed with Type 2 diabetes. He is working with an internal medicine doctor to slowly regulate his blood sugars. Since being on Glypizide/Metformim 5/500, his blood sugars are coming down nicely and he is not having to use humolog. However, he is having severe mood swings with anger outbursts which are not characteristic of him.Should this moodiness be treated? He has a huge amount of anxiety about this diagnosis and he is struggling to understand how he became diabetic since he has always been thin his entire life.Any suggestions? Doctor: Hi, welcome to our site. I am Dr. Kalpesh Kavar, MD.Read your query. That is a very significant question and i appreciate your problem. I will try my best to answer your query. Many a times when a person is diagnosed for any chronic long term illness, there is psychological stress associated with it, which need to be cleared by proper counselling and patient education. The same thing happens with your husband as he fails to understand why he is suffering from such disease. I would like to assure your husband as Diabetes is not a rare disease and almost majority of patients live with diabetes like totally normal life by controlling blood sugar levels with medications and few lifestyle changes. As far as his anxiety and mood swings are concern, it needs psychological support and needs to be counselled properly, can be cured without any medications. I hope this helps you. I have given you the answer to the maximum considering the information provided.Please do understand that some details could be extracted from a detailed history and examination.Please feel free to ask another query. I would be glad to help you. Looking forward to your return query with the details asked so that I can help you further. (If the answer has helped you, please indicate this)Take Care. DR. KALPESH S KAVAR, MDCONSULTANT DIABETOLOGIST"
},
{
"id": 173630,
"tgt": "Suggest treatment for urinary incontinence in a 5 year old girl",
"src": "Patient: My daughter who is 5 years old goes to pee every 5 mints at regular intervals during the day. Her water intake is also not so much that she has to pee every 5 mints. She has been this since last few days. I don t know what to do except take her for routine urine examination. Doctor: Hi,Thanks and welcome to healthcare magic.Increased frequency of urine suggests urinary tract infection .It may be some local infection around urethra.Local speculum examination to detect any local condition .urine microscopic examination and culture and sensitivity ..Better consult pediatrician for prompt diagnosis and treatment.Hope this answer serves your purpose .Please feel free to ask further queer if any.Dr.M.V.Subrahmanyam."
},
{
"id": 107985,
"tgt": "Suggest treatment for persistent back pain",
"src": "Patient: I have always had some difficulty falling to sleep. I just stay awake because I just cannot get tiny irritations not to be noticed. It is just something that has always bothered me. I have lived with it. Now the problem. I am an identical 79 year old male twin. My twin has had severe back ache for many years - and difficulty sleeping. I know he has this sleep challenge. It dawned on me that his pain might be eased if he could fall asleep easier - bringing my wonderment that if I can identify a solution to my situation I could get him to try to bring him comfort. He has had back operations etc over a 20 year period. Thanks. Larry Esswein Doctor: Now , he is suffering back during sleeping position. its a muscular pain.but if standing pain is occure so you do MRI LUMBR SPINE ."
},
{
"id": 80082,
"tgt": "What are the symptoms of bronchitis?",
"src": "Patient: i am nidhin vn from india.In my childhood days i suffered with asthma and after age 12 it was not there.Now i shifted to city and last 4 years i am at hyderabad.I am facing problem with continuous cold.Once it come,the running nose,Throught pain,suffocation it is continuing for 2..3 weeks.And after reading some google pages i fearing that it is bronchitis.Please suggest me a solution Doctor: Thanks for your question on Health Care Magic. I can understand your concern. By your history and description, possibility of allergic bronchitis is high in your case. So better to consult pulmonologist and get done 1. Clinical examination of respiratory system 2. PFT (Pulmonary Function Test). PFT is needed for the diagnosis of bronchitis. It will also tell you about severity of the disease and treatment is based on severity only. You may need inhaled bronchodilators and inhaled corticosteroid (ICS). Anti allergic and antihistamine drugs are also useful in this disease. Since all these drugs are prescribed medicines, you need doctor 's prescription for them. So consult pulmonologist and discuss all these. First diagnose yourself and then start appropriate treatment. Hope I have solved your query. Wish you good health. Thanks."
},
{
"id": 148790,
"tgt": "Experiencing constant headaches, lightheadedness, faintiness, tremors, vibration in face, change in blood pressure, heart rate. Indications?",
"src": "Patient: My teenage daughter, 13, is experiencing constant headaches that come in the morning and last all day. Some days they change in frequency of pain and area of the head. This began in early October '13 and is still persistant as of Nov. '13. As of a couple of weeks ago, she began feeling light headed at various times of the day. This has also been constant but changes in intensity (comes and goes). As of a week ago, the light headedness escalated to fainting. Also, having her left arm tremor (shake) at time of fainting and a \"vibration\" sensation in her face occur at the same time. These symptoms have remained constant for one and a half months. Also, her blood pressure has been recorded high on some days and other days low or normal (141/85 being highest and 98/57 being lowest). Also, flucuations in heart rate at times (120 bpm highest). Doctor: Hello,Thanks for the query to H.C.M. Forum.I would come up with these possibilities for headache and fluctuating blood pressure, these includes.1 Congenital heart disease , diagnosis can be confirmed by E K G and ECHO.2 Hormonal imbalance develops anxiety and tension in teenagers .Diagnosis can be confirmed by blood examination for oestrogen, androgen, FSH/LH ratio.3 As you mentioned that she develop tremors in left hand with lightheadedness and intense headache so may be due to brain lesion.Diagnosis can be confirmed by MRI of brain for for tremors.In my opinion first of all consult a physician and get his opinion.Good luck.Dr. HET"
},
{
"id": 46180,
"tgt": "Suggest treatment for renal failure",
"src": "Patient: My mother suffer from Chronic renal failure. her age is 72. Hemo Dialysis in weakly thrice. She has taken amlodipien 5mg b.d., Flavedon MR o.d., Livogen b.d., cap.Rocatrol 0.25mg o.d., calcium 500mg, b.d., Tab, sorbitrate 5mg, ing. Eithropoietin 2000i.u weakly twice,These are all useful doses. Now she has swelling the both leg, Doctor prescribed Moxclav 625mg and Acetaminophen 625mg both b.d. dose, To day taken the uric acid test . the present of uric acid is 5.2. Doctor told to me if the uric acid present is more than 7 , you may get tab. Febuget 40mg. may i give the drug Febuget 40mg to my mother? Doctor: No we don't generally treat uric acid level if the patient doesn't have any symptoms, below level of 8...in your mother's case she is CKD stage 5 on maintainance hemodialysis patients and pedal swelling most likely because of fluid overload, not because of increased uric acid... So restrict her fluid intake 1-1.2 litre daily and at this point of time febuget is not required.."
},
{
"id": 84869,
"tgt": "Can xanax cause dizzy?",
"src": "Patient: Hullo my name is anthony and start first pill of cipralex ( escitalopram )10 mg just 10 hours before and i still feel bad symtoms like affraid sensation and vomit .i ask my doctor he said to me to take 0.25 xanax .and actually i feel little bit dizzy and i wanna ask you if it is recomended please to take the xanax to stop these bad sensation Doctor: Hi, Yes, for sure. Xanax can make one sleepy or dizzy and can slow down the intellect, thinking and motor skills.Hope I have answered your query. Let me know if I can assist you further. Regards, Dr. Ajeet Singh, General & Family Physician"
},
{
"id": 121852,
"tgt": "What to do if having excruciating pain while sitting, sleeping after hurting tailbone?",
"src": "Patient: I fell off a skateboard today and landed on my tailbone. It doesn t hurt too much to stand or even walk but after sitting for a while I get pretty bad pain even more so after lying down which is just excruciating. Also I have had an insane amount of gas. Is this usual? Should I see a Dr? If so When is the ideal time frame? Doctor: Hello,Your symptoms seem to be related to the injury. I would recommend to do an X-ray of the tailbone to exclude the possibility of a fracture. Meanwhile,I suggest using anti inflammatory medications such as Acetaminophen to relieve the pain.Hope I have answered your query. Let me know if I can assist you further.Regards,Dr. Dorina GurabardhiGeneral & Family Physician"
},
{
"id": 165175,
"tgt": "Suggest treatment for constipation in a child",
"src": "Patient: hello! dr my baby boy is 4 month old , he is on exclusive breastfeedig, he used to pass 4 stools in a days from bith uptill now, but from last 2 weeks he was passing 2 stools on every 3rd day and now he has not passed any stool from last 48 hours , what i should do now , plz reply, thanks alot Doctor: For breastfed babies frequency of normal bowel movements varies significantly. Some pass 4 to 5 times per day others 3 to 4 times per week. If your baby is comfortable, abdomen is soft, feeding normally, no vomitings etc. then these findings are more important than mere frequency of bowel movements. Constipation is passing hard and dry stools causing discomfort in defecation.Constipation in breastfed babies might be due to dehydration or less milk supply. Increase frequency of feeding. Continue exclusive breastfeeding. If no relief consult your doctor and ask for guidance on transition to weaning, laxatives, suppositories etc."
},
{
"id": 176867,
"tgt": "What causes loose motion in kid?",
"src": "Patient: My baby has lose motion from 5 days.he is 1 year 2 month.not recover even medicen.what I give him in food.he is not eating well.thats why he become seck to much . cough and other probles are also.recently I change country.before this he was ok.plz help Doctor: Hi Drear Welcome to the HCM,loose motions and cold cough can occur in any child frequently in such a tender age.Take care of the hydration and adequate oral rehydration solution to given @ 150 ml of ORS per loose motions.As soon as the infection is expelled out of the stomach baby will be alright.cold and cough can be taken care by giving the babay some antihistaminic and decongestant drug.Hope the query is answered.Thanks"
},
{
"id": 12405,
"tgt": "Suggest treatment or cure for psoriasis in legs",
"src": "Patient: Doctor good day\" I am 48 yrs male staying out of india in Dubai United Arab Emirates since 22 years. My problem is i am having Psoraisis in my legs (foot) and sometimes it comes and goes. tried many treatment. recently i went to bombay and took ayurvedic treatment which was partly beneficial. But doctor i believe i need to take a trip to meet you in person and show the condition whereby you can recommend the treatment. I have been told that there is no permanent cure. but there should be some way out. please help me. Ajay Vyas.... Doctor: Hi, Ajay...As you said..you have relapses of psoriasis on lower extremities..You have tried many treatment,without good response..right? You should be aware of the pathology of the disease.Psoriasis is a chronic relapsing and autoimmune disease. You may be having scaly plaques on various parts of the body , even on the scalp. There may be dryness of skin,which might cause itching.Exact cause is not known. Genetic tendency,autoimmunitity,stress, dry wheather...etc may be responsible for precipitation and exacerbation of the disease.I advise you to consult dermatologist personally for firm diagnosis and perfect treatment. Avoid anxiety and tension related with your disease.I usually recommend cap acitretin 10 mg twice a day along with methotraxate 15 mg weekly in three devided doses 12 hourly. This may be taken for long time till good response. Blood tests may be done to monitor the treatment. Apply mild steroid cream or calcipotriol oint on the lesions. If you have scalp psoriasis,tar shampoo may be done. You may apply moisturiser to improve the skin texture. Avoid soap bath for few weeks.Avoid stress and worries as that might worsen psoriasis. Have pateince for recovery. You take treatment under observation of dermatologist.I hope you got my answer.Thanks.Dr. Ilyas Patel MD"
},
{
"id": 185564,
"tgt": "What causes itchy gums with sore throat?",
"src": "Patient: I have recently been unwell with a common Cold.. Symptoms such as runny nose sore throte a cough aches and pains with a headache... I have also noticed that I have got itchy gums... I have been taken cold a flu tablets along with paracetamol.. I brush my teeth twice a day and i use corsodol mouth wash.... But my gums are still itching... Please help! Doctor: Hello, Read your query, understand your concern this itchy gums can be due to viral stomatitis or gingival and periodontal ligament inflamation, for this you can do is do luke warm saline gargle two - three times a day. You can apply ointment Gum paint like stolin or Acivir , and take vitamin B and C supplements by consulting with your local doctor . Consult dentist for visual examination , scaling and root planning for healthy gums .Hope it helps you. If you have further query I will be happy to help. Regards , Dr. Priyanka tiwari"
},
{
"id": 96084,
"tgt": "I had yellow diarrhea , What should I do ?",
"src": "Patient: 2 days ago i pooped out a clear jelly. then about 20 min later i had diarrhea which is a yellow color with very painful gas. What do i do? Doctor: The ENT has done a colonoscopy and some lab tests (blood only) and says I am \"perfectly healthy\" I continue--about once or twice in every 2 weeks to have explosive, unxontrolled btight yellow diarrhea--the Ent says this is \"IBS\"---- Bull-----. I am not having any pain, and I had an ultrasound done of my gall bladder which was normal. BTW, this has been going on for over 4 years now. I have no constipation and am up to taking 4-5 immodium at a time so that I can at least go out to the grocery store once a week. But I have no abdoninal pain, ever. I am fed up with the doctors here who don't seem to care or know anything. What should I do next? majic"
},
{
"id": 158999,
"tgt": "Thyroids removed due to cancer. Taking levothyroxine. Feeling tired, pain in the temple and lower back. Why?",
"src": "Patient: Hi, I have a number of health questions, if I may. First, I was diagnosed with thyroid cancer about 10 yrs ago and had surgery to have both thyroids removed. Have been taking Levothyroxine daily since. But I havent had my blood levels checked lately because of no insurance, and have been feeling really sluggish and tired for past few months. Am still taking medicine (levothyroxine). Second, I have piercing, sharp stabbing pains in my left temple area and on top and behind my left ear off and on for past several years. Pain only lasts a few seconds and comes and goes. Dont have migranes or any other unusual headaches besides normal headaches from time to time. Lastly, I have been having pain in my lower back on right side kidney area. I notice it when I take a deep breath.Thank you Doctor: Thank you for your query. There are some possibilities of your problem. Long standing tiredness after thyroidectomy may be due to hypothyroidism. The levothyroxine dose you are taking probably is not sufficient. Simply t4 & tsh test will guide you about the dose. Next the pain you feeling at various part of the body particularly the low back pain should be concerned. It may be simple mechanical pain. But the disease recurrence & bony metastasis should also considered. Nuclear scanning of the whole body have to be done. Please contact with your concerned doctor. Take care."
},
{
"id": 203188,
"tgt": "How to treat pressure between scrotum and rectum,rectal pain and penile discomfort post sex?",
"src": "Patient: pressure between scrotum and rectum,seems to be worse after strenuos work or long periods of standing.sometimes almost feels like hind end is falling out.it seems to come and go,when it is present it almost feels i have discomfort in penis especially after sex.itchy almost hurtful feelings.sometimes this also comes with rectal painns or discomforts.might last couple days then go away the return in a few weeks sometimes months. Doctor: Hello!Thank you for the query.Your symptoms are very characteristic for chronic prostate inflammation. This issue is sometimes hard to diagnose with any test. The treatment is long antibiotics therapy.I suggest you to start with urine analysis. There might be some signs of infection. Its better to have urine analysis after prostate massage. Please try to massage the area between the rectum and scrotum and right after it have urine analysis.You should also consult urologist.Hope this will help.Regards."
},
{
"id": 147285,
"tgt": "How to cure slipped disc?",
"src": "Patient: Hi doc, im a 40 yrs female having a slip disc problem and on simple medication. Doctors have adviced me bed rest for few days. Im very confused and want to know what actually slip disc is? is it cured properly? can i live a normal life doing all activities like driving,climbing stairs at home and all normal household affairs? Please help me because im getting very depressed. Thankyou. Doctor: The vertebrae in the spine are separated by a spongy tissue which is the disc. If the disc ruptures, the tissue in it can slide out of its normal place. This narrows the space between the vertebrae and may cause pressure on the nerves in the spine. This is called a slipped (\"herniated\") disc. This usually heals up on its own without any specific treatment except control of the pain. Some people may need physical therapy or even surgery to fix it. Once it has healed, you can live a normal life and do all your activities as before. I personally had a very bad slipped disc when I was your age. I took some medication for the pain and did not have surgery. The disc healed and I have been able to do all the activities that you mention since then. So there is certainly much hope that this will happen with you too, and no need to get depressed.Hope I have answered your query. If you have any further questions I will be happy to help."
},
{
"id": 169871,
"tgt": "Suggest treatment for a vaginal skin tag in a child",
"src": "Patient: my daughter is 21 months old. No problems throughout pregnancy, and born term, vaginally. She was born with a small skin tag slightly inside the vagina. Recently i had noticed it becoming larger in size. It doesnt seem to bother her a whole lot. I had asked her doctor about it a few months ago and he said he doesnt think i have anything to worry about. Now to me it seems like her vaginal hole is slightly bigger than it should be and im not real sure if its from the skin tag growing and stretching. Right now we currently dont have health insurance until the beginning of july and i dont have a whole lot of money to take her back to the doctor. Doctor: Hi.... by what you say I feel that your kid might be having a condition called vaginal synechiae. This is a very very common condition and it can be most of the times relieved by using Premarin cream as external application on the skin tag. But it has to be done under a specific technique and it needs to be taught by a pediatrician or physician. If the Premarin cream is not working then only we need to release the synechiae surgically. This is also is a day care procedure and is not major procedure. So you need not worry about it. I suggest you consult your pediatrician regarding this with the tip I have given.Regards - Dr. Sumanth"
},
{
"id": 62749,
"tgt": "Suggest treatment for a itchy lump on the armpit",
"src": "Patient: I have removed a deer tick from under my arm and it has now developed a knot around where the tick was attached and itches terribly. Do I need to seek medical help for this. I have pulled off several ticks in the past with no side effects whatsoever. Thanks so much for your advice. Doctor: Hi,It seems that there might be having allergic local reaction due to tick bite.Take Benadryl or cetrizine medicine for 2-3 days.Apply triple action cream locally.Avoid perspiration.Ok and take care."
},
{
"id": 113898,
"tgt": "How do I check if my diagnosis for Annular Tear L5-S1 with fragmented narrowing is correct or not ?",
"src": "Patient: I have been diagnosed as having an Annular Tear L5-S1 with varying degrees of fragmented narrowing. Mild to moderate degenerative disk disease. I was deny a position because of medical restrictions. I was told that I could not lift more than 25lbs, and could not work in bent over positions. During routine daily life I perform these activities with no significant problems, only a little stiffness went bent over, but it subsides when I stand upright. This does not occur on a daily basis. I am wondering if the diagnosis was inaccurate, or could the restrcitions be too extreme? Doctor: welcum to healthcaremagic better u go n consult any spine centre for second opinion. and if u want to reduce ur restrictions thn start physiotherapy in any spine care centre. tk cr"
},
{
"id": 150048,
"tgt": "Lump in back of head, increasing in size during stress. Cure for symptoms?",
"src": "Patient: Hi I have a lump in the back of my head behind my right ear for 10 months. Sometimes I feel aching pain in it but it doesn't hurt. Its the size of a quarter. There is no abnormal shape or no discoloration. Every time I stress it get as big as a quarter when im not stressing it gets as small as a nickel. I dont have any signs or symptom of infection. What could this be. Doctor: Hi, you have a lump on the head behind the right ear. since 10 months. It appears to be the dermoid cyst, or lipoma, it wont do any harm, if it is growing very big you can consult a surgeon for removal. If you want to get it removed now it self you can get it removed with out any risk. Thank you."
},
{
"id": 95443,
"tgt": "Bright red blood with bowel movements.I have intermittent constipation and diarrhea",
"src": "Patient: I have intermittently had blood showing on toilet paper and droplets, some size of dimes, bright red blood when I have a bowel movement. This has been ongoing for about a year, in recent 2-3 months I started having upper abdominal pain and feeling of lump, but cannot feel it by physically feeling the area. Now I am having at times, severe pain over my bladder area or to the side of it and my abdomen is very bloated almost everyday. I am not obese and eat a pretty healthy diet. I have noticed the blood more when I eat red meat as opposed to fish or chicken. I eat a good amt of steamed vegetables and brown or wild rice. I'm wondering what these symptoms may be, especially because the blood is bright red. I also have intermittent constipation and then I'll have diarrhea. I do have an extreme amt of stress. Please advise... Doctor: You should visit a doctor, it seems to be quite a problem"
},
{
"id": 190433,
"tgt": "Frequent canker sores, stopped orange juice, still have sores, bleeding gums, terrible sore throat that pains on swallowing. Help?",
"src": "Patient: Hi, I have been getting canker sores quite often but i use to be told its from orange juice and too much acid well now i dont drink much and i havent been i have two canker sores and feels like one on my tongue but think it might just be a swollen taste but also have a terrible sore throat not so much hurt kinda tickles unless i swallow my spit and only hurts on one side i have had my tonsils and adnoids out currently takin amoxicilllan and garling salt water as much as possible also just brushed my teeth and my top upper gums were sore and started bleeding . PLEASE HELP!! Hilary Doctor: hello and welcome, the problem is due to aphthous ulcer.this commonly occurs due to vitamin deficiency. please do visit your dentist and take treatment according to that.please do not take antibiotics without medical supervision. take tab.flanzen-100mg,thrice daily for 3 days and tab.wysalone-5mg twice daily for 3 days.these tablets after consulting with your dentist. also get your teeth professionaly cleaned as your gums are bleeding due to gingivitis. check also whether any of your impacted wisdom tooth is inflammed ie,pericoronitis.if present,get operculectomy done or extraction.pericoronitis can cause sore throat.continue gargling salt water.maintain good oral hygiene. take care."
},
{
"id": 212721,
"tgt": "Miscarriage, still birth, extreme fatigue, history of depression and hypertension, advised thyroid tests. Is that right?",
"src": "Patient: I m 35. 2 mc and 1 full term stillbirth Nov 2010. I m rh neg. was given 12 shots of rogram after c section. History of depression , htn (on meds since daughters birth). Extreme fatigue . CBC are off. I m now being checked for thyroid issues. Do I need to see a specialist that studies blood? Something is wrong and I need answers. Thank you. Doctor: Hello....... Thanks for your query. As you mentioned many of symptoms of hypothroidism like extreme fatigue, generalised weakness can mimic depression. Also sometimes throid disorders can have onset after a still birth. Hence, i would suggest to go ahead with thyroid function tests and consult an endocrinologist. You would also require an evaluation for depression by a psychiatrist. Hope you find the suggestions useful. Regard DR Sundar Psychiatrist"
},
{
"id": 40507,
"tgt": "How can infertility be treated?",
"src": "Patient: Hello my name is Paris I am 26 years old, my husband and I have been trying to get pregnant for over a year now, I myself honestly believe I have had two miscarriage s with in the year of trying. After experiencing what felt to be the first miscarriage, I made an appoint to be seen at my local gynecologist practice in my home town. The doc only did a basic check up which only consisted of five minutes of her checking my vital signs and breast. The doc really didn t want to hear or see what I need to tell or show her of what I believe to be a misfortune event. I was also scheduled for blood work along with other things that was listed for checking into during that appointment, with out preforming the complete check up and no explanation as to why; she calmed it be unnecessary and rushed me off. Because of what I had in-contoured during that visit, I felt overwhelmed at the fact that I was concerned about my health as well as well being but felt as thou it didn t matter, I did not return to be seen the second time that it happen three months after to the first time. What should be our next steps? or our best option for trying naturally or even alternative s Doctor: Hello and Welcome to \u2018Ask A Doctor\u2019 service. I have reviewed your query and here is my advice. I am sorry that you felt unattended and ignored but basic checkup is the first line management, without which we cannot progress anywhere. Miscarriage is an event which can happen cause of numerous causes and in certain case is unavoidable in some countries it is not advisable to take any severe measures until it is a case of recurrent pregnancy loss (more than 2 abortions). I know it is an unfortunate and traumatic experience and you had to suffer it twice. Get screened for APLA ( antibodies form which cause abortion). Undertake basic USG take progesterone support in next pregnancy. You and your partner can take up karyotype study to rule out genetic problems, if you had taken up karyotype study of previous abortus it will definitely be helpful. Hope I have answered your query. Let me know if I can assist you further."
},
{
"id": 78686,
"tgt": "Suggest remedy for persistent coughing and sticky phlegm",
"src": "Patient: For the past 2 weeks every day i snort & cough up bloody smelly sticky phlegm. I do cough up alot of clear phlegm in the morning - non smoker. Also recovered from chest infection about 1 month ago - finished antobics. No fever no pain, once i get it out i feel fine. Doctor: Thanks for your question on Health Care Magic. I can understand your concern. Since you had chest infection in the recent past, possibility of post infectious bronchitis is more in your case. Your symptoms like constant cough and phlegm since 2 weeks are commonly seen in bronchitis. So better to consult pulmonologist and get done clinical examination of respiratory system, chest x ray and PFT (Pulmonary Function Test). Chest x ray is needed to rule out recurrence of lung Infection. PFT is must for the diagnosis of bronchitis. It will also tell you about severity of the disease and treatment of bronchitis is based on severity only. You may need antibiotics, antihistamines, anti inflammatory, inhaled bronchodilators and inhaled corticosteroid. Don't worry, you will be alright. Hope I have solved your query. Wish you good health. Thanks."
},
{
"id": 185018,
"tgt": "What causes thick gum like secretion on gums,lips and tongue?",
"src": "Patient: I have a thick gum like secretion on my gums,lips and tongue and I am constantly trying to spit it out and it is invisible, you can't see it but you can feel it with your tongue, my lips and tongue are inflamed but gargling wit salt water helps with that. What is this stuff? Doctor: Hello!Thank you for posting here.This can be thick ropy saliva you can feel with your tongue.Keep hydrated.Take 3 liters of water every day.You did not mention if you suffer any other health problem.Chemotherapy,radiation,Diabetes, allergies and sinus problems also cause this.If this is just because of dehydration, this will become normal once you start having ample water.Do not smoke or drink.Maintain a healthy life style.Hope this helps."
},
{
"id": 164245,
"tgt": "Is Moxclav advisable for cough while on Calpol for cold and congestion?",
"src": "Patient: Hello doctor ,My Daughter is 2 months old , she had cold , stuffy nose and cough , with saline nasal drops and calpol the stuffy nose and fever got cured . But she is coughing too much , doctor prescribed maxclav 228.5 can I give to my daughter , she weigh 4 kg Doctor: Hi, welcome to HCM. Yes, moxclav Syrup is often prescribed for upper respiratory tract infection but it should be given on prescription of doctor only after examination."
},
{
"id": 180626,
"tgt": "How can trismus be treated?",
"src": "Patient: Unable to open my mouth due to eating Gukta which I have left 5 years ago now I am dental problems want to go for door canaling but it not possible unless my mouth opens had S.M.Fibro for one month twice daily but it is still not opening request remides Doctor: Hello and Welcome to \u2018Ask A Doctor\u2019 service.I have reviewed your query and here is my advice.As per your query, the symptoms of reduced mouth opening due to Gutka chewing can be due to most common reason that is Oral Submucous Fibrosis in which there is formation of thick fibrous bands below the mucosa of cheeks and lips leading to stiffening and difficulty in mouth opening. It is a precancerous condition [although not cancer] and should be taken care of.It is commonly seen in individuals with tobacco and areca nut chewing habit. You need to consult an Oral Physician or a Dentist and get evaluated and he can advise him to apply a steroid ointment in combination with hyluronidase containing ointment to break fibres. SM Fibro should be continued.Intra lesional injections of steroid and hyluronidase can be advised to be injected after every 3 to 4 days. Physiotherapy like blowing cheeks, trying to open the mouth under pressure etcetera can be demonstrated and advised to be done daily. Complete cessation of habit of tobacco if you have this habit.Hope I have answered your query. Let me know if I can assist you further.Regards,Dr. Honey Arora"
},
{
"id": 106863,
"tgt": "What does lower backache along with itching on the right thigh indicate?",
"src": "Patient: I had pain in my lower back, right side. It was accompanied by sensitivity to touch , along with some itchiness in my right thigh. Now, the pain and sensitivity are gone but a rash has developed. I thought the \u201crash\u201d may have been a result of constant rubbing and scratching, but I\u2019m not sure. The rash does not cover my thigh; it\u2019s like 3 separate rashes. All rashes are in fatty parts of my thigh. They hurt as a fresh bruise would and are not overly itchy. I\u2019m taking ibuprofen if necessary and have put both anti itch cream and antibacterial cream on them. It\u2019s only been about 24 hours since the rash developed. Doctor: Hello,The symptoms that you have described points to disc prolapse and in my opinion an investigative modality like an MRI should be of help. Kindly consult a doctor for the same.Hope I have answered your query. Let me know if I can assist you further.Regards, Dr. Fahim Sheik"
},
{
"id": 31035,
"tgt": "How can yeast infection be treated?",
"src": "Patient: Hi, I have a mild yeast infection and I crushed a garlic clove and inserted it in my vagina and at first it burned but now its ok and Im on my period will I be ok or should I go and soak so the garlic in come out because i read online that it will cause me to have sores Doctor: yeast infections are very much curable . most vaginal yeast infections can be treated with an over-the-counter vaginal cream or suppository. Vaginal creams are typically placed inside the vagina with an applicator to kill off the yeast that causes infections. these antifungal creams usually come with an applicator . common examples include Mycelex(clotrimazole) , Femstat( butoconazole) , monistat (mucanozole nitrate). in a dose of three to five days. Some of the same medications (such as clotrimazole and miconazole) are manufactured in vaginal tablets or suppositories to treat yeast infections. You insert these into the vagina and let them dissolveavoid using any fragrant body washes or any harsh soaps and intercourse during treatment course .Be sure you actually do have a yeast infection before you start treatment."
},
{
"id": 213388,
"tgt": "Occasional swelling of the right side of the body. Is it a symptoms of any mental health case?",
"src": "Patient: I m a 30 yr old female. Occasionally my legs, feet , hands, and sometimes my face will swell for no apparent reason. It s typically limited to the right side of my body. After I begin to swell I start saying and doing things that make no sense and serve no purpose. I never have any recollection of these events, but as my husband begins to reiterate them to me I can recall small bits (almost like viewing photographs of myself). With each episode it gets more extreme. Last time, my husband came into our hotel room and found me standing in the window, arms outstretched, telling him that Trent Reznor(from the band Nine Inch Nails ) had just been in the window talking to me. I realize that this mainly sounds like a mental health case, but I assure you that its something else wrong with me. I m not sure if my liver could be to blame, but for some reason I feel that thats the base of the problems. I m a mother of 3 and I m beginning to fear the next time this happens. Im scared Doctor: Hai From the information given i could not conclude anything but i suspect some mental health problem.problem with liver will not cause such problems.further information about whether others also can notice your body swelling,your marital relationship,any stress in the recent past are also needed. Better to consult a psychiatrist as early as possible. Bye"
},
{
"id": 36707,
"tgt": "Is it needed to apply some medicine on the infected part of the injury?",
"src": "Patient: Hi Dr Prasad,My son had an ankle injury and was advised a plaster . We were also advised to avoid the plaster from getting wet or if he must take a bath, he could use a plastic cover over the plaster. We used a plastic cover over the plaster, but unfortunately the water went inside. Within 3 days my son started complaining of itching iin the feet. I insisted that the plaster be cut and to my horror, found that his foot was completely soddened . The doctor then advised us to leave it open. It has dried up, but i still need to know, if he needs to apply some medicine on the infected part or should he just let it be. Doctor: Hi,Clean the affected area with antiseptic lotion and apply antibiotic cream.Keep local part open and airy.First time clean the part by the doctor so proper cleaning of the part is done.If your doctor feels proper, he will go for one course of antibiotic medicine course.Ok and take care."
},
{
"id": 190610,
"tgt": "Dry patch on the inner part of lip, feeling of roughness on the site on running the tongue, apparent on eating, no discoloration or pain. Treatment required?",
"src": "Patient: Hi, I m a 33 year old healthy woman. I have a strange dryish feeling patch on my upper inner lip. The two sides of my upper lip don t feel symmetrical - when I run my tongue over the left side (where the dry patch is), it feels almost a little rough, like very fine-grade sandpaper. the right side feels normal and slick. Also, when I purse my lips , the right side still feels smooth and plump on the inside, but the left side, where the patch is, creases into sharp wrinkles . I notice it especially when I m eating - especially drier foods like cereals - which feel like they catch there. And the lip sometimes feels like it catches on my teeth on that side if they re dry. I first notices this about 3 weeks ago. I can t see any discoloration, and it s totally painless, just odd. Should I see a dentist? Or a dermatologist? Thanks! Doctor: hi as per the information provided by you i would recommend you to see a dentist to rule out any dental structure(natural or prosthetic) responsible for this patch. sometimes chronic local irritation from sharp edge tooth or dental prostheses lead to such patch appearance.am sure that a dental doctor can fully take care of this particular problem. take care."
},
{
"id": 213763,
"tgt": "How can I come out of depression and regain my self confidence ?",
"src": "Patient: I have been diagnosed with depresion and anxiaty 4 yrs ago. But with all the difrent pills I have tried nothing has helped. I can be happy one min. and very sad the next I am up and down like this all the time. I have had cbt witch didn t help. I went in to the priory witch helped for a bit but I came out morbadly obeast which has made me feel even worse. I am always looking at others and wanting there lives. I despise my self so much. I don t want to go out any were but when I do I get so anxtuse I have to leave. I also have spending splurges wich i can t realy afford. My doc says I may be bipoler my partner says I am not, I am so confused. Please could you give me any idea on what to do. I would be so greatful, i really am at a loss with my self. I m scared no one will answere this aspeople will just think I am being stupid. Doctor: hello welcome to health care magic its better u consult some psychiatrist u need to have some medications required for depresssion and u r not suffering from depression its better u contact some psychiatrist"
},
{
"id": 175309,
"tgt": "How to treat vaginal rash?",
"src": "Patient: My daughter has a rash looking area on her vagina that she use to say burn.I thought it was a rash due to her wiling herself so hard, I asked her if anyone touched her inappropriately and she would always say no, what are like signs or images of a abused child Doctor: Thanks you for follow up . There lots of causes of itching in vagina, more often enterobiosis,it isisorms, then different infection, as fungal-candidiasis, bacterial-streptococcus,esherichia Coli. For deep examination you should go to gynecologist , she can do bacteriological analysis of secret from vagina, visual examination and she can tell she was abused or not, she will check her hymen , see hematomas. You can try to speak with child not directly -ask,\" Which person did you not like?\" Consult at psychologist alsoBest regardsDr.Svetlana"
},
{
"id": 44167,
"tgt": "Trying to conceive, unable to. Asked to take semen analysis test, HSG, blood test. Suggestions?",
"src": "Patient: Hi Sir I am 30 year old & my husband is 36 we have one daughter 5 years old and we are now planning for another child. First child was spontaneously conceive. Now we tried for two - three months but there is no result. Presently we are taking treatment from gynecologist and she suggested HSG Test , Semen analysis & some Blood test . Is it necessary to do the test or we tried by natural method for another months ? Doctor: Hi, Welcome to HCM, i am Dr. Das It is confirmed that you both are not primarily infertile as you have a child. Have you tried for child by doing regular intercourse for one year? If not, then wait. First go for natural method by doing intercourse during 10-18 days in a menstrual cycle counting the day of first bleeding as day one. If the natural method fails then there may be secondary infertility which needs a thorough work up. So, it is better to wait for some month and keep trying."
},
{
"id": 193949,
"tgt": "What should I do to remove large nipple?",
"src": "Patient: hello doctor my name is uday. from the age of 14 my chest nipples are growing larger shape at present my age is 20 i am very uncomfortable with this recently i went to the doctor and he said that it requires surgery and he also said that scars will appear at my chest portion after the operation i am afraid of that one.so, i am requesting to please send me the suggestion to remove my larger nipples.my weight is 65kgs.my height is 5.5ft Doctor: Hello, There is no need to worry and no need for any surgery if you exercise daily and concentrate on your chest exercise. Hope I have answered your query. Let me know if I can assist you further. Regards, Dr. K. V. Anand, Psychologist"
},
{
"id": 220708,
"tgt": "How to check pregnancy at home?",
"src": "Patient: HI, I missed my period, i took apregnancy test and it was clearly psitive within 2 minutes (slightly light pink) then i wnt to doc. her test was negative .again next morning i testet it turned positive after two days test turned again negative and it was firs mornig sample.Am i pregnant????? Doctor: Haallow Dear,Your two positive tests are more supporting the possibility of pregnancy. The two negative tests may be false negative. These tests if performed in the first week of missed period may give false negative results. Moreover, since you are performing the test at home on the overnight first morning urine sample, the hormones (hCG) in this urine are concentrated and hence the chances of more reliable results. When you go to the doctor, the urine you collect is a random sample which may have dilute hormones in the early stage of pregnancy. However, your second test with the doctor was on morning first urine sample. To get rid of the dilemma, I would suggest you to get Beta hCG test done which is very specific and highly sensitive. The results of this test are quite conclusive even few days before you miss your period. I hope this guides you to the better method. Dr. Nishikant Shrotri"
},
{
"id": 199009,
"tgt": "Suggest treatment for a small itchy bump on the penis",
"src": "Patient: Ive having a constant battle with jock itch. I work outdoors so i sweat hevily throughout the day. Yesturday while showering i noticed a small itchy bump in the shaft of my penis. No pain or leaking any fluids and not pus filled. I do have masquito bites on my arms from work and the bump looks and feels like a bite also. Doctor: Hello dear,Considering the case history that you have recurrent infection with jock itch, the lesion as mentioned in your post can be attributed to contact dermatitis or any fungal infection.Application of ointment like Lobate GM (pharmacologically Clobetasol + Miconazole + Neomycin) will reduce the symptoms.1. Clobetasol being a steroid will reduce the inflammatory process2. Miconazole being an anti-fungal agent will take care of any fungus causing the symptoms3. Neomycin being an anti-bacterial agent will prevent secondary bacterial infections.In addition, keep the area dry & avoid too much exposure to water or sweat formation.Wishing you a Good Health.Take care."
},
{
"id": 119993,
"tgt": "Suggest treatment for subchondral sclerotic focus",
"src": "Patient: Sir,I Dr. Parmar had segmental resection of pelvic colon for cancer in 2003 followed by chemotherapy then had right hepatectomy for solitary secondary in2006. now having backache for one month. On MRI small subchondral sclerotic focus in D-12 seen. I am fearing it may be secondary in bone. What investigation & treatment do you advise at this time? Doctor: Hello, It is unlikely a bone secondary. You can opt got PET-CT to look for any recurrence. Hope I have answered your query. Let me know if I can assist you further. Take care Regards, Dr. Shinas Hussain"
},
{
"id": 145276,
"tgt": "Suggest treatment for pain on right side of back by scapula and spine",
"src": "Patient: I am 40y male with hypertension and I have had back pain for the past week. Last week I was throwing a football and over extended my throw. It caused some brief pain in my right back by the scapula and spine. Later that night I was sitting on the couch leaning forward and started to feel an intense sharp pain in my right chest. I sat up straight and it went away. It did happen a second time that night from leaning forward. The next couple of days I felt like I had a knot in my back and some tightness. Eventually this led to me waking up with the right side of my back swollen and having random shooting pains in my back and chest. It got to the point where I was in so much discomfort that I went to the ER. The hospital did verify that it is not a cardiac issue. I am out of the hospital now and was told to take ibuprofen and rest up. Which is what I am doing. I am just concerned about my symptoms. I generally feel very run down with feelings of having a hot face, a little diarrhea (maybe from the pain meds at ER?) and elevated blood pressure. Is it normal for back pain to cause symptoms like this? Doctor: Hi there, it sounds like the doctors at the hospital thought this might be costochondritis or perhaps muscle strain/ spasm given the recommendation for ibuprofen and rest. If no CT scan or MRI was done of your Thoracic spine, that would be the next step if your complaints of shooting pain don't get better (especially if they wrap around your chest to the front) to make sure there is not a bulged disc there that is causing your symptoms. Back pain specifically is not known to cause elevated blood pressure, but pain certainly can do this."
},
{
"id": 25488,
"tgt": "What medication is suggested for elevated BP levels?",
"src": "Patient: I am a white female age 56 and recently my blood pressure has been consistently high. Last week, at my doctor's it was 140/100 and it has gone up since. In the past few days it has been 158/110, 188/120, 165/114, 150/113/ 179/113 (10 minutes ago as I am sitting at the computer doing nothing strenous). Should I be concerned? My father had many heart attacks, the earliest one at 35 years of age. My mother had several TIAs before having a more serious stroke. Help? Doctor: Hello, I saw your question .Most of the values you mention come in the hypertensive range.Considering repeated persistent high values of blood pressure and your strong family anti hypertensive medication is certainly indicated.Heart ailments usually occurs due to variety of risk factors which include obesity diabetes high cholesterol levels, kidney disease and genetics. Therefore at age screening is necessary for all other factors.The anti hypertensive medication is determined by all such factors .The usual choice of an anti hypertensive in absence of all other factors would be a class of drugs known as angiotensin receptor blockers or diuretic. Hope you start it soon"
},
{
"id": 206130,
"tgt": "What causes Alzheimer s with depression and psychosis?",
"src": "Patient: I am a psychiatric/mental health ARNP consulting in LTC. 76 yowf with hx of alzheimers with depression and psychosis, copd, and pain. On Tramadol 25 mg BID; the notes available to me are from Feb 2014 forward. She has been on Tramadol since Feb; however, not certain site of pain or length of use prior to hospitalization. Hospitalized in Feb for exacerbation of COPD with pneumonia and has been on Prednisone since, has been titrated down to 5 mg for past 2 months. D/C from hospital to AFL with new behaviors of increased confusion, verbally agitated, wandering in and out of residents rooms, resistive to redirection. She had weight loss and problems sleeping and I started her on Remeron 15 mg HS and increased Seroquel. PCP added Cymbalta 20 mg in April. Once I saw that I d/c Remeron worried about serotonin symdrome. I am being told her delusions are causing her emotional pain with resulting behaviors more problematic. I will check with pulmonologist to see if Prednisone is going to be an ongoing med for COPD. MY QUESTION: drug/drug interaction with Tramadol: is REMERON safer than CYMBALTA; does one have less potential drug/drug interaction than the other when used in combination with Tramadol? Doctor: Hello thanks for asking from HCMI have read the query and I will try to solve it. She has history of Alzheimer's, Depression, Psychosis, COPD and is on Tramadol 50 mg per day, and was given Mirtazepine, Qutiapine for sleep. Now coming to question-- Mirtazepine and Duloxetine both increases serotonin and can interact with Tremadol but as per my opinion mirtazepine should be more safe and should have less interactions with Tremadol.- Both can drugs can be used under controlled condition for depression with tremadol still risk will be there.Thanks, hope this helps you"
},
{
"id": 121532,
"tgt": "What medication is suggested for Livedo reticularis on the right thigh?",
"src": "Patient: My daughter age 24 ht. 5 5 wt. 165 has been diagnosed with livedo reticularis on the outside of her right thigh. The marks seem to be moving upward. She has a desk job and has gained approx. 25 lbs. over the last year. She keeps a heater at her desk on that side. Any nutritional cures? The doctor said it is caused by being cold. Doctor: Hello, It is kind of vascular dilation and will settle by itself. Generally no treatment is required.If you are very much concerned, newer techniques like LASER may be attempted. Hope I have answered your query. Let me know if I can assist you further. Take care Regards, Dr Shinas Hussain, General & Family Physician"
},
{
"id": 72204,
"tgt": "Is asthma a side effect of lobectomy surgery?",
"src": "Patient: My 20 year old son had a partial right lobectomy for a mio fibro blastic tumor.[non cancerous] he has developed asthma due to a reaction to his surgery. have you come across this before? one of his consultants put him on prednisolone and inhalers. two weeks later he went back to consultant and blew out half a litre more into the machine. hes hopeful this will go, but im not so sure. he is a non smoker. Doctor: Thanks for your question on Healthcare Magic.I can understand your concern. In my opinion, he is mostly having bronchitis. Bronchitis is inflammation of airways. It is common after thoracic surgeries especially lung surgeries. It is because of handling of lungs with instruments during surgery. It is usually short lived and recovered with steroid and inhaled bronchodilators (formoterol or salmeterol).So continue with prednisone and inhalers. He will mostly improve with all these in 1-2 months.Hope I have solved your query. I will be happy to help you further. Wishing good health to your son. Thanks."
},
{
"id": 91463,
"tgt": "How to stop stomach pain and feeling of throwing up?",
"src": "Patient: Hello,my stomach has been hurting for a really long time. But it s unusual because I feel like I m going to throw up because of every move I make. And when I stop moving around,the pain stops. What do I do? & I can t really take any medicines because I m allergic to penicillin. Doctor: Hi,I would suggest that you undergo endoscopy to rule out any hiatus hernia or esophagitis.Once diagnosed specific treatment can be commenced.Regards"
},
{
"id": 50430,
"tgt": "Stones in kidney. Had a stent put. Why is there swelling and infection?",
"src": "Patient: I have been in the hospital 3x because I have a lot of stones on my right and I have bad infections. I just had a stint put in because of I have another stone on my right. He did not break them up. Does this mean I will still get them back? They told me my kindey has been sowllen so much and infection I could lose my kiney. What else can I do? Doctor: Hi, many thanks for the query!Stone in the ureter causes obstruction to urine outflow, so urine accumulates in kidneys causing so called swelling of kidneys i.e. Hydronephrosis, also, proximal to obstruction- Hydroureter.If this obstructed urine gets infected, it causes inflammation in kidney & ureter, also, renal parenchyma is thinned out due to prolonged obstruction causing decrease in renal function.Hence, a stent (DJ) is placed starting from renal pelvis through ureter upto bladder, so that infected urine is drained, obstruction is bypassed,inflammation is reduced.After say 15 days, (duration depends on extent of inflammation), of DJ stenting, stones can be removed.To know more about renal function, you need to do-DTPA renal scan renogram.First you must do some basic investigations like-CBC, RFT, Urine (R/M), USG KUB.Take antispasmodics, antibiotics, diuretics with your Urologist's opinion.Drink plenty of water so that at least 2 litres of urine is voided in 24 hrs.If Sr. Uric acid is raised, start uric acid synthesis inhibitors like allopurinol, urine alkalizing agents with your Urologist's opinion.Wish you a good health.Take care.Regards."
},
{
"id": 134028,
"tgt": "Does swollen & tender arm/elbow suggest fracture?",
"src": "Patient: I fell on my arm a couple hours ago right below my elbow. It is swollen, tender and tight, but I am mobile. I can move my elbow very little pain, move my fingers no pain just really tight on my forearm, but when I flex my wrist up or any pressure is applied it bring excruciating pain. Could it be a fracture or just sprained? Doctor: Hi Thanks for your query and Welcome to Healthcare Magic. I am Dr Akshay from Fortis Hospital, New Delhi.Since you have suggested an injury due to a fall, i will suggest X Ray of the affected region to rule out a bony injury because the management may be entirely different in case of a bony or a soft tissue injury.Do not hesitate to contact me if you need any further assistance. You can also discuss your case and treatment plans with me in a greater detail in a private consultation.Thanks & RegardsDr Akshay Kumar SaxenaConsultant Orthopaedics Fortis Hospital, New Delhi"
},
{
"id": 98811,
"tgt": "What causes chest/back pain, hoarseness and sore throat?",
"src": "Patient: Hi. I get pains in my chest and upper back occasionally. About 18 months ago they got so bad that I was sent for an x-ray. It came back clear. They flared up again recently, and I had another taken. Again clear. The doctor said it must be costochondritis. I also suffer with pretty regular hoarseness and sore throats, plus what's been diagnosed as allergy-induced asthma (without inhalers I have random coughing fits). About a month ago the doctor gave me some antibiotics for a sinus/tonsil complaint. I have no cough. This week I again began to wake with chest pains, which became back pains later in the day. When I expectorated this morning I had mostly white sputum, but then two were a dirty green. More worryingly, one had a pea-sized lump of dark red blood in it. Almost purple. Should I go back to the doctor? Could the previous x-rays have been wrong? Or might it be a problem with my aesophagus?I am 37 and otherwise pretty fit, cycling for an hour a day and playing sport on the weekend. I don't smoke now, but did for about 15 years. Doctor: Hi. I can understand your concern. Chronic cough is commonly seen in bronchitis and lung infection. Since your chest x ray is normal, no need to worry for lung infection. Possibility of bronchitis is more in your case. So better to consult pulmonologist and get done clinical examination of respiratory system and PFT (Pulmonary Function Test). PFT is needed for the diagnosis of bronchitis. It will also tell you about severity of the disease and treatment of bronchitis is based on severity only. You may need inhaled bronchodilators and inhaled corticosteroid (ICS)Don't worry, you will be alright. Hope I have solved your query. Wish you good health. Thanks."
},
{
"id": 34930,
"tgt": "Suggest remedy for soreness in toe",
"src": "Patient: I've had an ingrown toe nail on my rt big toe for over a month now. I went to the e.r. 10/31/11, to seek help. Before removing the nail completely, the doctor did 3 attempts of anesthesia. There were complications with the anesthesia. They finally removed the nail and applied ointment (Triple-antibiotic cream). Since the procedure, my toe has gotten worse and so has the pain. I apply the cream twice a day. There is fluid flushing out of everywhere and where the anesthesia was injected, looks like 2nd degree burns. It looks like it's spreading throughout the entire toe and foot. I am also soaking it in epsom salt and warm water. How ever, this still doesn't make it any better. What else should I do? Please help!! Doctor: thank you for posting your query in healthcaremagic.com....well probably you had paronychia and secondary infection following which the nail had to be removed.....but the infection persisted in spite of antibiotics and there had been further abscess formation.....i would suggest you to visit a GENERAL SURGEON as soon as possible for inspection of the area and also for further management...an examination is needed before giving you further advice.... hope this was helpful....regards"
},
{
"id": 119900,
"tgt": "What causes pain in the site of bone grafting?",
"src": "Patient: I fractured my foot and ankle slamming directly into a piece of wood in September of 2010. The fifth metatarsal fractured at the distal head and was quite significant. I was treated with a boot, and told I did not need surgery. The metarsal was slow to heal but the ankle healed and the other fractures. Last x-ray in November and December showed a healed fracture. I began having pain and swelling in April and May of 2011. X-rays revealed a bone mass in the fracture site which had characteristics of a tumor. Emergent surgery was performed, a bone graft completed (taken from heel), and biopsy and cultures were clean. I am 13 weeks status-post surgery, and recently began to have significant pain in the graft site. Is this normal? If the site is a non-union, what is the next step? Doctor: Hello, Bone has no nerve supply,its is the covering of the bone( perosteum,) that has sensory nerve supply. So after bone grafting when overlying breached periosteum develops newer nerve endings which are highly sensitive as they are un myelinated and are responsible for pain. Hope I have answered your query. Let me know if I can assist you further. Take care Regards, Dr. Rohan Shanker Tiwari"
},
{
"id": 90164,
"tgt": "What causes pain in the stomach?",
"src": "Patient: Male, I have pain in the central and upper stomach, just below my sternum. Feel very sick, but not been sick yet, No diarrohea. Feels like a punch in stomach. Have sharp pain at left side about two inches from naval, occasionally right. Also, have un explained bruises on legs, arms, stomach and back. Also sudden onset of severe right hip pain spreading down the thigh. Symptoms came on two days ago and not able to sleep. Doctor: With the history of bruises and pain in abdomen I would suggest you :::First of all to go for the coagulation Profile of blood. loke Bleeding timeClotting timeProthrombine timePlatelet count. Complete blood picture.This will explain both . Consult the Physician ASAP. This may be a serious problem. The similar rash in the upper abdominal musculature can give rise to pain. Apso watch for blood in urine, stool and mouth."
},
{
"id": 71963,
"tgt": "suggest remedy for pain due to inflammation on phrenic nerve during pregnancy",
"src": "Patient: I m pregnant and have a pain in the upper left part of chest, the doctor say that it s possible I have the infiammation of the phrenic nerve may be the result of the position of the baby. I take some pills tachipirina but no relief at all. What else shall I do? No positions at bed is good. Thanks for answering. (from italy) Doctor: thank you for writing me kindly go for chest xray PA with masking cage of abdomen *(to prevent baby from radiation) and you will get your answer"
},
{
"id": 72457,
"tgt": "Suggest medication for cough and nausea",
"src": "Patient: My son developed a chesty cough and last night he threw up while i was feeding him chicken noodle soup. That was the only time he threw up. He also has a runny nose, but no fever. Could he have a stomach virus or just a bad cough that upset his stomach to puke? Doctor: Thanks for your question on Healthcare Magic.I can understand your concern. No, nausea is not due to stomach viral infection. Bad cold due to viral upper respiratory tract infection (URTI) is the cause for his cough and nausea.So asking him to follow these steps for better symptomatic relief. 1. Avoid oily and spicy food. Avoid junk food. 2. Drink plenty of fluids orally and keep yourself hydrated.3. Take levocetrizine and paracetamol combination twice daily.4. Do warm water gargles and steam inhalation 4-5 times a day.Don't worry, he will be alright with all these in 1 week.Hope I have solved your query. I will be happy to help you further. Wishing good health to your son. Thanks."
},
{
"id": 175115,
"tgt": "What is the treatment for runny nose?",
"src": "Patient: my 5 year old girl has recurrent runny nose ,sore eyes but has now developed very black rings under her eyes and her facial color is very pale she is saying under her arms are very sore and she had a raised iga she was told she is allergic to egg whites and other things 2 months ago she appears to be disimproving generally Doctor: Hi...sometimes allergies can take even months together to get better. It doesn't depend only on the medications alone. The most important thing is to take off the triggers. Even if your kid is using medicines regularly, if she is constantly exposed to the allergens which are causing this, then the improvement may not be satisfactory. The allergen can be as simple as a common house dust mite. So I suggest you concentrate on the removal of triggers from her vicinity.Regards - Dr. Sumanth"
},
{
"id": 196166,
"tgt": "What causes clear liquid discharge through the penis when erected?",
"src": "Patient: hi , when i get slightly excited , or when am hard ..something colorless comes out of my penis , i saw in one of your answers that its called pre cum and natural body fluid .Can that make a women pregnant ? even it did not ejaculate.Am 21 years old , will it stop one day ? Doctor: precum is usually watery.that do not contain adequate sperm to fertilise women.forceful ejaculation will provide adequate sperms to make women conceive."
},
{
"id": 77841,
"tgt": "Can shortness of breath due to aortic stenosis and stiff lungs hurting lungs?",
"src": "Patient: I have always been offended by the smell of onions or garlic cooking but now it seems to make my lungs burn and gives me a headache. I suffer from shortness of breath due to aortic stenosis and stiff lungs. Do you think these odors are actually hurting my lungs? Doctor: If you are allergic to these odours then it will result in shortness of breath. burning of your airways is more attributed to irritant substances. Either one will result in symptoms which are detrimental to the airways and the lungs.."
},
{
"id": 223052,
"tgt": "Is excessive amniotic fluid harmful to the baby?",
"src": "Patient: Hello doctor....am 7 months pregnant, in scanning they had told me that i have more aminoitic fluid (AFI19), is it harmful for my baby.my docotor has done the diabetic test its normal and even baby s growth also is very good . i just wanted to know is it harmful for the baby Doctor: amniotic fluid 5 to 25 is normal ,so you need not worry,but yes excess amniotic fluid suggest congenital abnormalities"
},
{
"id": 139457,
"tgt": "What causes light headedness and shaking?",
"src": "Patient: Hi, My name is Jose and I had a question. For some reason, for the past few months there are times when I get light headed and start shaking a little. I m not sure if its because i m hungry or if there is something else that is wrong. What do you think it could possibly be ? Doctor: Hello and welcome to \" Ask a Doctor\" service.I have read your query and here is my advice.If you feel hungry, you should measure your blood sugar levels.Discuss with your Doctor for these issues.Hope this helps.Feel free to ask if you have further questions."
},
{
"id": 150702,
"tgt": "MRI shows degenerative disc narrowing, presence of prevertebral osteophytes, diffuse disc bulging, neural foraminal stenosis. Indications?",
"src": "Patient: Dear Doctor Can you help me understand the severity of my cervical mri. Am I looking at surgery. nerve pain bi lateral through shoulder, arms and into middle and ring fingers. back and neck pain . the pins and needs into my hands is so bad that I have to sleep sitting with an ice pack. My MRI results are as follows: straighting of the acervical curvature. degenerative disc narrowing is prsent including c-5 c-6 and c6- c7. Small prevertebral osteophytes are present . c3 -c4 mild diffuse disc bulging uncinate process spurring. moderate right neural foraminal stenosis . c4-c5 mild diffuse disc bulging mild facet degeneration and uncinate process spurring mild righ neural foraminal stenosis c5-c6 mild diffuse disc bulging larger in the left paracertral region. mild central canal stenosis with mild moderate spinal cord deformity , greater on left. Uncinate process spurring. Marked left neural foraminal stenosis. c6-c7 mild diffuse disc bulging. mild spinal cord deformity but relative patencyor central canal. this may be associated with hypertrophy or early ossification of posterior longitudinal ligament. Thank you Doctor: Hi, Thank you for posting your query. Your symptoms are suggestive of cervical radiculopathy, where pain, tingling or numbness may occur in the arm upto fingers, due to compression of the nerves in the neck region. MRI findings suggest mild compression of the nerves. In the initial phase, medical treatment along with physiotherapy would be sufficient. Medications helpful in this condition are pregabalin or gabapentin. I would advise you to start these. If your condition does not get better, you may see a neurosurgeon for a possibility of surgery. Best wishes, Dr Sudhir Kumar MD (Internal Medicine), DM (Neurology) Senior Consultant Neurologist Apollo Hospitals, Hyderabad, My personal URL on this website: http://bit.ly/Dr-Sudhir-kumar My email: drsudhirkumar@yahoo.com"
},
{
"id": 85359,
"tgt": "Does folitrax have any side effects?",
"src": "Patient: My Mother recently is diagnosed with Rheumatoid Arthritis 6 Months back. Doctor has prescribed 1)Folitrax 15mg once a week 2)folic Acid 5mg once a day 3)Sazo 500 twice daily is this treatment good? shud we continue these medicines?as The Doc says she has to continue forever. folitrax(Methotrexate),Sazo 500 taken for long term will not have side effects? Should she go for Homoepathy/Ayurveda? should she use these drugs only when there is a flare or should she continue this forever? Doctor: Hello, Indeed, her current treatment is a bit hard for her, but that's the perfect treatment for her rheumatoid arthritis. Usually, when used in long term we advise to run liver/kidney and other blood tests to evaluate if their function has altered, in order to change dosage or therapy of her actual treatment. She can go for homeopathy, there is no problem. Unless, she might feel something unusual or if improved, should discuss with her doctor to stop her current treatment. All the best."
},
{
"id": 59290,
"tgt": "Heavy drinker having a lesion on the liver. Severe abdominal pain, diarrhea. Any suggestions?",
"src": "Patient: My husband is a heavy drinker and they just discovered a lesion on his liver and he has an apointment in a couple weeks with the gastro dr . But he is having another problem we thought he had hemorrhoids but its been a yr and its only getting worse he has had diahrea for a yr also the hospital checked him for chrons disease and diverticulities and all tests came back normal i just cant figure out what the problem is he is in constant pain and hes has lower abdominal pain ive been searching online and cant figure it out please help im despreate to help him Doctor: Hi, thanks for writing in HCM. The investigation of choice for your husband would be a double contrast Ct of the abdomen. This will help in picking up the exact lesion. In addition he needs to undergo a detailed evaluation of the liver function as well as pancreatic function. Pancreatic insufficiency can lead to loose motions. Regards"
},
{
"id": 219395,
"tgt": "What are the symptoms of potential pregnancy?",
"src": "Patient: Hi, I m 21 years old I m often have unportected sex so I m always stressing if I m pregnant. I ve maybe puke a total of 10 times my entire life so I don t believe I would have morning sickness to soon. I m having short periods up to 3 to 4 days, there is heavy bleeding in my bowels, bad cramps, lack of appietite, seem to be tired alot, moody, I do have alot of gas and I ve notice constant jerking in my stomach and I m always having indejestion I ve also been faint my blood pressure seems to drop alot. I do have alot of stress going on so it s hard to tell if all this is just stress or something bigger. When I was 16 a doctor told my I have hpv. I ve taken pregnancy tests but they either come out negative or unreadable. Have any suggestions? Doctor: i need to know if your periods are regular and normal each month. Do the pregnancy test only if you are late for your period.HPV can be diagnosed by getting pap smear test done now so you will know the current status."
},
{
"id": 143344,
"tgt": "Suggest treatment for hyperlordosis and hyperkyphosis",
"src": "Patient: Hello I pulled my lower back out and now I have hyper lordosis and hyper kyphosis which is straining my thoracic muscle which are pulling muscles in my head, in which its causing my c1 to become misaligned and cause tight pinching headaches in my temples. I ve went to the chiropractor but nothing really has helped. I work as an exterminator in which I have to sit and drive from place to place, and my back starts to hurt after a few minutes. I also have to bend over often which has put futher strain on my back. I m in alot of pain. Advil hasn t helped any. What should I do? Doctor: According to ayurveda, we can take it under vatrakat disease. So you should do basti treatment for 30 days it will really help you."
},
{
"id": 67850,
"tgt": "Could the removal of bump below the collar bone harmful?",
"src": "Patient: Hi my daunghter has a tuberculous lymphadenitis said by his doctor and was treated 9 mos.of antibiotics but it doesn t work! It became a little bigger now. The size about half of an egg below collarbone ..is there any option how can i know the real results or can you advise me to have a city scan? is it not dangerous to remove the said bump ?thanks Doctor: Hi! Good evening. I am Dr Shareef answering your query.If I were your doctor, after the apparent failure of the antibiotic for 09 months and increase in size of the lump, I would advise you for a FNAC (fine needle aspiration cytology) of the lump. If this is inconclusive, I would redo the FNAC along with a CT scan of your neck, chest and abdomen. OVer and above the findings of a detailed physical examination would also help in co relating with the repots of investigations. Further management would depend on all of the above.I hope this information would help you in discussing with your family physician/treating doctor in further management of your problem. Please do not hesitate to ask in case of any further doubts.Thanks for choosing health care magic to clear doubts on your health problems. I wish you an early recovery. Dr Shareef."
},
{
"id": 19619,
"tgt": "Suggest treatment for irregular heart beat and breathlessness",
"src": "Patient: from since a young age i always had irregular heart beats. eg i was playing football when all of a sudden my heart started beating alot faster than usual, with chest pains on the right and short of breath. i ocassional suffer these attacks even when i sit down. now i regularly get irregular beats, where my chest hurts and at times short of breath from lying down... Doctor: Hi thereAfter going through your query I understand your concern.I would like to tell you, palpitations along with shortness of breath is more likely to happen due to cardiac Arrythmias and it requires further evaluation to avoid fetal cardiac events. Kindly get your ECG, treadmill test or stress echocardiography done and see a Cardiologist with reports for further management.Hope to have been helpful.Kind Regards Dr Navneet Bansal"
},
{
"id": 103275,
"tgt": "Having seasonal allergy symptoms, tried Fexofenadine, antihistamines, no relief. Solution ?",
"src": "Patient: I am having seasonal allergy symptoms such as sneezing, stuffy nose, watery eyes, sore throat, etc. I tried an allergy medicine with fexofenadine hci tablet USP, 180mg/ antihistasmine , (generic form of Allegra), and got no relief. It was a 24 hour tablet. Can I take anything else, such as benedryl, for my symptoms? Doctor: Hello dear,The symptoms as mentioned in your post suggest that you might be having Allergic rhinitis.Management includes:1. Antihistamine preparations like Allegra can be taken for symptomatic relief.2. Montelukast, which is used as a maintenance therapy to relieve symptoms of seasonal allergies & asthma.3. Maintain adequate hydration & take a healthy balance diet.4. Also make sure that you are well protected from cold, dust & other allergens.Wishing you a good health.Thanks & take care."
},
{
"id": 141236,
"tgt": "What causes vomiting post a concussion despite a normal CT scan report?",
"src": "Patient: My daughter has a concussion from a car accident last week. She threw up all weekend and on Monday they did a CT scan. It didn t show a bleed but she is in college and has slept most of the week - and still feels like she cannot answer questions or read or focus. How long should she rest or sleep before she will be ok to return to her heavy Ph.D. schedule? Doctor: Hello and Welcome to \u2018Ask A Doctor\u2019 service. I have reviewed your query and here is my advice. She may be suffering from post concussion syndrome. Post-concussion syndrome is a complex disorder in which various symptoms, such as headaches and dizziness that last for weeks and sometimes months after the injury that caused the concussion. Most common symptoms, such as headache, dizziness and sleep problems, which are similar to those often experienced by people diagnosed with depression, anxiety or post-traumatic stress disorder. There is no specific treatment for post-concussion syndrome. Your doctor will treat the individual symptoms you're experiencing. The types of symptoms and their frequency are different for everyone. Please consult your neurophysician, he will examine and treat you accordingly. Take care."
},
{
"id": 144776,
"tgt": "Suggest treatment for numbness and weakness in the arms",
"src": "Patient: I was injured at work. I was grabbed by a intoxipated patient that was in a chair. She grabbed my left arm and pulled me down and grabbed me by my neck. I immediately had pain and tingling down my left arm. Now I have numbness/tingling and weakness in both arms/hands. My MRI C-5C-6 right paracentral disc extrusion is impinging the spinal cord and narrowing the AP diameter of the canal to 4 mm. No cord signal abnormality. C-6 C-7 spondylosis with central canal narrowing to 6 mm with ventral cord impingement. Moderate bilateral neural foraminal stenosis. I have done PT and injections. Now they are talking surgery???? Doctor: Hi there, thanks for your question at HCM.Surgery is definitely the last step in treatment ladder of Spine related pain.Since you have already tried pt , injections and medications.And I presume they have not worked. Surgery is also indicated on new onset weakness. On the other hand if u have significant improvement in pain and weakness, may be you can wait. Hope this helps.All the best. Regards. Dr.SBK"
},
{
"id": 91943,
"tgt": "What causes sharp pain in mid and right section of abdomen?",
"src": "Patient: I have had gerd like symptoms for about two years lately the sharp pain in the mid and right section of abdomen is so painful it's left me unable to move, I have changes all kinds if dieting habits but pain is getting more intense after every meal no matter what it is I have pain that's for a few hours Doctor: HI. A simple test of Upper GI Endoscopy and Ultrasonography would diagnose the case. As the pain is getting more intense with every meal and lasts for about few hours - suggests me about probable diagnosis of an ulcer in the duodenum or stomach, can also be die to an obstruction somewhere in the intestinal tract."
},
{
"id": 197663,
"tgt": "What causes rash on penis while on Champix?",
"src": "Patient: Hi. My fiance has like a rash on his penis. On the tip is like a raw red spot and just below the head is all read and puffy. He said it itches and burns. We are both on our 5th day of champix. I also have about the same on my viganal area but not raw its tender, burning ect. Its like a yeast infection but im not on antibiotics at the moment I finished them the beginning of the week and mine only started 2 days ago and my fiance only last night and we havnt had intercourse. Could it be a side effect from the champix? We really worried. Thank you. Doctor: Hi. Welcome.It is not possible to comment on skin lesion without examination, so kindly see local doctor or upload the pictures in here to help you with specific advice.This provisional advice provided by me is based entirely on the input provided to me. I would suggest examination by the doctor.I hope this suggestion might be of some help to you. Please accept my answer in case you do not have further queries.Take care."
},
{
"id": 19956,
"tgt": "What could pressure on diaphragm suggest?",
"src": "Patient: I have undergone bypass surgery eight months ago.now at present i am feeling pain in the upper portiom of diaphragm in the left side of the chest and breathing trouble too.i am feeling a pressure in that area always.what is the remedy and what should i do now. Doctor: Hello!Thank you for asking on HCM!Regarding your question, I would explain that it is important performing some tests to investigate for the possible causes underlying your complaints: -a chest X ray study-a resting ECG and a cardiac ultrasound-cardiac enzymes-complete blood count, PCR, ESR for inflammation. A cardiac stress echo would help investigate for coronary artery disease. A fibrogastroscopy may be needed to investigate for possible gastritis or gastro-esophageal reflux, especially if your symptoms are relived by PPI (omeprazole, esomeprazole, etc.). Hope you will find this answer helpful!Wishing all the best, Dr. Iliri"
},
{
"id": 87017,
"tgt": "What causes severe abdominal pain near cervix?",
"src": "Patient: I am a 24 year old female, I am 5'7 and 183 pounds. I have severe lower abdomen pain, feels very near cervix. It hurts to sit down, stand straight, and when I do sit to go to the bathroom, it feels like everything is going to come out. The upper stomach pains have now started again, and if what happened before happens again, I will vomit and then not be able to move at all. When I went to the ER in August all they did was check my urine to see if I was pregnant or had a bladder infection. The only medical problems I have is a reoccurring case of BV, and possible ulcers, but have not had a test done because my health plan does not cover it. The pain is in increasingly getting worse, when this happened in August I was writhing in pain for three hours. Doctor: Hi,From history it seems that you might be having severe infection in your lower genital tract giving this problem.As you got problem of having Bacterial vaginosis in the past so it seems that this problem re-occurred.There is another possibility we should keep in mind about having prolapse of urinary bladder called cestocele or lax pelvic floor muscles.Consult Gynaec and get examined.Ok and take care."
},
{
"id": 98059,
"tgt": "Is there an alternative medication for weight gain, candida, hypothyroidism, anemia and PCOS?",
"src": "Patient: LOOKING FOR ALTERNATIVE OR WHOLESTIC THERAPY IN ARKANSAS OR OKLAHOMA WOULD LIKE MORE INFO BEFORE PROCEEDING MY THOUGHT IS CANIDIA, WEIGHT GAIN , GASTRIC BYPASS THEREFORE POOR ABSORBTION OF RX, IRRITABLE, HYPOTHYROID , PCOS, MEMEORY PROBLEMS, SEIZURES, ANEMIA, TOXINS, FOOT FUNGUS, SCALP SORES EXTREME FATIGUE, I JUST AM TIRED TRYING AND TRYING TO FIND ANSWERS YET GETTING WORSE. SEEMS TO ME THAT THERE WAS A NATURAL DR THAT PRACTICED IN NORTH AR YOU COULD WRITE HIM AND HE WOULD SEND YOU AN ANSWER BY MAIL. ANY SUGGESTIONS Doctor: Hi, TIGHTWADHAG Two things come to the discussion: 1.\u00a0\u00a0\u00a0\u00a0\u00a0Low digestion due to gastric bypass, Candida-yeast overgrowth, low pancreatic function. It can lead to many nutritional deficiencies and internal toxicity because the foods are fermented 2.\u00a0\u00a0\u00a0\u00a0\u00a0Hormonal changes such as hypothyroid, high insulin and insulin resistance To normalize conditions like those that you have we recommend our patients \u2022\u00a0\u00a0\u00a0\u00a0\u00a0Healing Anti Candida and alkaline diet \u2022\u00a0\u00a0\u00a0\u00a0\u00a0Whole body cleansing with colon hydrotherapy and restoration of the friendly intestinal flora \u2022\u00a0\u00a0\u00a0\u00a0\u00a0Nutritional supplementation with cellular magnesium-potassium, iron, vitamins, probiotics, and amino acids \u2022\u00a0\u00a0\u00a0\u00a0\u00a0Herbal Therapy \u2022\u00a0\u00a0\u00a0\u00a0\u00a0Acupuncture \u2022\u00a0\u00a0\u00a0\u00a0\u00a0Drinking Karlovy Vary healing mineral water \u2022\u00a0\u00a0\u00a0\u00a0\u00a0Custom medical hypnosis CD for self-hypnosis at home, etc To my experience, without participation in your healing and changes of your life-style, it is very difficult to help you. Do not give up knock all doors. Some useful info you can find in my EBook: Healthy Pancreas, Healthy You. Why pancreas? Pancreas is a core of digestion. Your pancreas is weak. Good Luck in your road to proper health."
},
{
"id": 162859,
"tgt": "What causes cracks around the sphincter?",
"src": "Patient: my 13 yr. old son has cracks around his sphinkster . he said he has had them for years . I just found out about this. he was using a lot of wet wipes so I asked him if every thing was alright. he said my ex wife has known this for years. im concerned . he has cronic bloating and diaria . my ex recently had him checked for food allergies. the result was 1 for wheat 2 for soy 1 for dairy. very low , but for 2 months now I have got him foods without these things. he has told me he feels 50 to 60% better. I am seeking info on and diseases that have this MO. THX! Doctor: Hello and Welcome to \u2018Ask A Doctor\u2019 service. I have reviewed your query and here is my advice. Your child has developed cracks around anus due to chronic diarrhea. As you have put child on dietary restrictions, there will be control of loose motions and cracks also. At present, ask your child to do gentle wiping of area around anus after every stool. In addition, you may apply any steroid ointment as Beclomethasone for healing. Hope I have answered your query. Let me know if I can help you further."
},
{
"id": 32554,
"tgt": "What causes stomach ache, fever and vomiting in a child?",
"src": "Patient: hi my 11 year old son had come 2 weeks back from school complaining of headache and weakness then in the evening he vomitted.he complained of stomach ache .He had cough and we told him to take rest after giving digene and crocin.By Sunday he had fever around 99-101 fever.So we took him to our doctor who gave him crocin and he was off school for 3 days.Again after a week on monday he vommitted in school and i rushed to get him back.he vomitted around 7 times.no loose motions.he was given ondem inj and put on light food diet.he was fine.today(friday) again he is complaining of stomach ache and he vomitted once.wwhat could be the reason. Doctor: Thanks to HCM I can understand your son concern headache ,recurrent vominting and stomach pain fever in children generally have infection here I would like to suggest just undergo once fever profile like CBP. ESR, WIDAL and MALARIA test along with chest x ray you can give the paracetamol,along with antiemetics like vomikind , antibiotic like cefixime for 3 days try avoid oily food ,sweets in diet let him drink plenty of water don't worry in children it is commonest problem once with report consult your family doctor or HCM OK ."
},
{
"id": 2071,
"tgt": "Suggest when to meet obstetrician after two miscarriages and now not able to conceive",
"src": "Patient: I have been trying to conceive for 12 months. I am 30 years old and have no children yet. I have had 2 miscarriages. The first was in August 2010 at 7 weeks and the second was March 2011 at 5 weeks. I have read that you should see an OB if you haven't gotten pregnant within 12 months. When should I see an OB, since I did get pregnant twice in that time? Doctor: Hi I think you should meet a obstetrician now because you had 2 miscarriages. You should get yourself evaluated to find cause of abortions so that some medicines can be started to prevent happening that again. do a thyroid profile. Hope I have answered your question."
},
{
"id": 150173,
"tgt": "Severe pain in the forehead, nausea, vomiting, dizziness. What could this be ?",
"src": "Patient: From yesterday, my wife has severe pain in the forehead and has been feeling nausea. She has vomitted a few times (3-4 times) over night and now she is also complaining of dizziness. She says she gets a little bit of relief from the pain after she vomits. Her BP is usually between 110/70 to 120/80, though we haven't checked it in the recent past. Dear doctor, can you please suggest what could be the cause. Thanks Doctor: Hi userSevere headache with nausea vomiting, and dizziness are really red flag signs of some brain disease. I do not say all persons having these symptoms suffering from brain disease but those who diagnosed brain diseases most of them having these symptomsI will advise you consult doctor immediately because in few brain disease require early treatment"
},
{
"id": 121218,
"tgt": "What causes pelvis pain with bladder problem?",
"src": "Patient: I am having pain the lower pelvis which has increased over the past few days, and really hurts wthen I got to the restroom. Also I have been having this light-headed spells and I am always tired. I also have noticed over the past few weeks I have had a lot more bladder control issues. I am not sure what to do, just I just wait it our get to a doctors? Doctor: Hello,I read carefully your query and understand your concern. Your symptoms seem to be related to a a bladder infection. I suggest to do a urinalysis and a complete blood count further evaluation. Meanwhile,I suggest using anti inflammatory medications such as Acetaminophen to relieve the symptoms. I also recommend to use cranberry juice for antibacterial properties. Hope my answer was helpful.If you have further queries feel free to contact me again.Kind regards! Dr.Dorina Gurabardhi General &Family Physician"
},
{
"id": 147118,
"tgt": "What causes feeling like having brain shocks?",
"src": "Patient: ok it seems when i get mad my body tempature drops dramaticly why does it do that Also sometimes its like im gettin like brain shocks when something is loud or i move my head i have ct scans and xrays all come back clear but nobody can answer these question Doctor: I don't get any sense of a true neurological problem with the symptoms that you are relaying. I can tell you that the temperature control in your body would not be something that could drop that quickly that you'd be able to tell in an instant that it were changing so that is a perception...it's not really happening within the brain or body. As far as the symptom of anger having something intimate to do with the onset of the other symptoms you mention tells me that again, what you are saying is not of a neurological origin but rather psychological which is causing you to perceive and feel certain things but the brain itself is wholly intact. This is supported by the scans and other studies you have said coming back as negative. Good luck to you."
},
{
"id": 91613,
"tgt": "Are antibiotics recommended to treat pus cells shown on urinalysis causing abdominal pain?",
"src": "Patient: hello doc.. my query is i already take antibiotic last 4 days ago name is monurol,, antibacterial but now i get urinalysis my pus cell increased from 0-2 to 0-3,, 4 days later,, is antibiotic not effective? or ill ask another antibiotic,, i still have abdominal pain ,, Doctor: i think the medicine is not working due to resistance of bacteria to anntibiotic these are the chancesi suggest you to go in for urine culture which can tell you the exact antibiotic working on the bacteria causing the urinary infectionyou can have lot of fluids and liquid antiseptic syp which excretes through urine till you get results of culture"
},
{
"id": 90360,
"tgt": "What causes severe pain in the right quadrant of the abdomen?",
"src": "Patient: I am hurting in the top of my abdomen and there are small sharp pains in the right quadrant of my abdomen and some on the left. I am burping a lot currently diagnosed with gerd taking 40 mg of protonix a day. My stomach is bloated also. I have gallstones which had not been causing me any issues but the Dr said I need to get it taken out soon. I wasn t in pain before and its not unbearable but is very uncomfym should I call a Dr to make sure I m not having a gallbladder attack? Doctor: Hi.Thanks for your query.Thanks for a good history and understanding you have. The pain in abdomen looks to be due to gall stones , unless proved otherwise. The Surgery for removal of the gall bladder may relive of many problems including bloating, pain and may be burping too.Better to call a Doctor and make sure why the symptoms- plan for a surgery too."
},
{
"id": 140553,
"tgt": "What causes bruising and pain in the lower spine after a fall?",
"src": "Patient: I fell down bottom 2 stairs,lurched forward and then back. I fell on my buttock and the bottom of my spine. This was 2 days ago. I have a nice bruise but no pain when I feel arround the spine but going up staisrs and moving certain ways causes sharp pain. Is this just bruisinfg which will sort itself out I am 72! Doctor: Hello, Well, there is really no way to predict without an x-ray what the status of your skeleton is since it sounds as if you took a pretty could clobbering falling to your buttock. You likely have a condition known as COCCIDYNIA (extremely painful bruising of the COCCYX or tailbone). This is usually not treatable, even if fractured because it is such a small and inaccessible bone. Whether or not you may have suffered a bone chip or compression fracture of one of the lumbar vertebrae can only be known if you get an X-ray. You would be making a smart move to call your doctor in the morning to get an order for some testing and maybe even a good physical exam as a follow up to what happened. Hope I have answered your query. Let me know if I can assist you further. Take care Regards, Dr Dariush Saghafi, Neurologist"
},
{
"id": 29733,
"tgt": "What causes burning sensation and redness in scrotum after intercourse?",
"src": "Patient: 8 days ago, I had protected sex and days later have a slight burning sensation and redness on my sac below my penis. This has never happened before. I have been living in a very humid climate the last two weeks....Cartagena, Colombia. Is this skin irritation a threat to future women I have sex with? Doctor: Hi, I have gone through your medical history carefully and understood your concerns. Your actual problems of:- burning sensation- redness in the sac below your penisstarted 8 days after protected sex while living in a very humid climate the last 2 weeks are indicating a local irritation/infection of your genitalia. A photo of the affected area would be of great help. If no painful urination or other urinary problems, I advise to :- keep your genitalia well clean (wash with warm water and soap)- apply clotrimazole cream locallyIf no improvement, then, dermatologist to consult as you might need prescription medicines. Wish fast recovery."
},
{
"id": 107572,
"tgt": "What causes back and rib pain, nose and perineum bleeding, gums swelling?",
"src": "Patient: I have been reading about leukemia and I think this may all be psychological but i have have pretty bad lower back pain, right lower rib pain, headaches for months, my nose bleeds only slightly, but often,, my gums are swollen, and a little blood comes from the skin in between my v@g!n@ and my rectum. Also, I am very tired almost all the time.. and recently i have been waking up very sweaty.. what do you think could be wrong? Doctor: Hello, I have studied your case with diligence.Due to malignancy there can be osteoporosis leading to back pain.There can be degenerative wear of bones, muscles, ligament and loss of elasticity of muscles, due to deficiency of calcium, vitamin D, VIT B12 and carnitine.so supplements of these nutrients can relieve pain.In post cancer osteoporosis is very common osteoporosis leads diffuse pain in back.Check your bone strength with DEXA SCAN and you can start medication according to level of osteoporosis,You need to start supplement of calcium with vitamin D, vit B12, and if osteoporosis is severe then bisphosphonates can be started.I will advise you to do regular physiotherapy and exercises as my patients find much relief by these therapies.You need to take diet rich in calcium and vitamin D.Hope this answers your query. If you have additional questions or follow up queries then please do not hesitate in writing to us. I will be happy to answer your queries. Wishing you good health.Take care."
},
{
"id": 83467,
"tgt": "Can Adderall cause severe chapping with swelling in tongue and mouth?",
"src": "Patient: I m prescribed to Adderall XR 60mg a day, however, due to this shortage, I ve began taking 2 (20mg) standard adderall. Now I cannot control my lip movements, and other people are noticing. My lips are severely chapped, and everything inside my mouth (tongue, mouth, cheeks) are swollen. I imagine this has something to do with my change in adderall. It s extremely painful and embarrassing. Doctor: Hi, People with ADHD fidget and this includes mouth and tongue movements. You'd have to wonder if there were another neurological cause such as Tourettes, basal ganglion problems, use of anti-psychotic meds long term, or other causes. Hope I have answered your question. Let me know if I can assist you further. Regards, Dr. Matt Wachsman, Addiction Medicine Specialist"
},
{
"id": 8648,
"tgt": "Can kojivit and propionate be used to remove black spots from face?",
"src": "Patient: dear sir ... am sandhiya 26years ... recently (last 2months) am having some block spots in my face specially below and around to my mouth. naturally am little dark in color.now am using propynate nf cream for block spots. and now am going to use kojivit it s given by dermotologist only... is it posiible to go ? in that particular medicine. am waiting for your reply.... am really feel bad about my face..... Doctor: Hello Sandhiya Welcome to healthcare Magic Propynate nf is a steroid and i would not advice you to continue that for your pigmentation around mouth. You need to rule out some conditions like Vitamin B12 deficiency, Hypothyroidism, Addison's disease and Seborrhoeic melanosis in your case. Your dermatologist will be able to help you out in this regard. Kojivit is a safe cream which you can try for your pigmentation. It will improve but usually won't go completely. Have a balanced diet. Eat plenty of citrus fruits (Vitamin C), Carrots and other colored fruits(Vitamin A). Meat, poultry and fish is rich in Vitamin B12. Avoid excessive sunlight exposure and wear a sunscreen daily. Hope this helped Take care"
},
{
"id": 214234,
"tgt": "Suggest home remedies for loosening skin around sphincter",
"src": "Patient: Hi Dr. I am a 38 year old mother of 2 who has suffered from hemmorhoids during my pregnancies and since. When I am not suffering, I have loose skin around my sphincter. The skin does engorge during a flare up. Is there something I can do at home to remove that loose skin without surgical intervention?? Doctor: Hi, I understand your concern. If there is no involvement of spincter & just skin loosening please don't worry. You can try to tighten it by- - avoiding constipation/stress to anus ( fibers in diet/ plenty of water) - alternate hot & cold rinses/ -sitting in hot/cold water tub.schitz bath. -Perineal exercise- alternate closure & loosening of anus( to be done after evacuation of bowels only) Thanks."
},
{
"id": 210834,
"tgt": "Could Anxit be taken after having food and can two tablets be taken in a day?",
"src": "Patient: I'm Ayan Dolai . I'm taking Anxit 0.25mg 1 tablet/day .But it's not works. If I take 2 tablet in a day what it will be after take some food ?? If I take 2 tablet after food what will be that time ? It works or not after taking food ?? please give me the answer right now . Doctor: Hello,Thank you for your query.Anxit contains alprazolam which is a benzodiazepineused to treat anxiety disorders and certain other condition.This medicine should be taken strictly as per your doctors guidelines as it has its own indications and dose varies accordingly and higher doses may harm or may lead to addiction.If your doctor told you to take 2 tablets then you can take this with or without food.If you getting stomach upset then take with food no problem.Ones again do not increase or reduce or stop the dose without consulting your doctor.Hope this helps.Take care.Thank you."
},
{
"id": 19962,
"tgt": "What causes shooting pain from left breast side to back during pregnancy?",
"src": "Patient: hi, i am 28 weeks pregnant and have had this pain that shoots from the top of my breast on the left side through to my back and i have to breath slowly for about 10minutes and then it goes it has happened before maybe twice, am i having heart spasame? what is it also my doctor wants to test me for systolic murmur il have that at the end of the month, other than that i have had no problems so far alh. Doctor: Thanks for your question on Healthcare Magic. I can understand your concern. Possibility of musculoskeletal pain is more likely. But better to get done ecg and 2d echo to rule out heart diseases. If these tests are normal then no need to worry for heart diseases. Avoid movements causing pain. Avoid heavyweight lifting and strenuous exercise. Avoid sudden jerky movements. Apply warm water pad on affected areas. Wear rib belt to prevent mobilization of ribs. Don't worry, you will be alright with all these.. Hope I have solved your query. I will be happy to help you further. Wish you good health. Thanks."
},
{
"id": 21670,
"tgt": "Suggest treatments for Rheumatic heart disease",
"src": "Patient: Hi, My age is 26 and weight is 54. I am having rheumatic heart disease. My question is that should i go for valva balloon or should i go for mitral valva. Another question if i am getting preganant now than would my baby be safe also my delivery OR If i am doing operation above any of this in August 2011 can i concive baby after the operation immediately after 2/3 months. Pls waiting for ur precious answer. Doctor: Hello Ma'amI have read the details provided by you and I understand your concern for starting your family. Making a right treatment decision for Rheumatic heart disease patient is a multidisciplinary approach taking into consideration History, physical examination and some tests like ECHOCARDIOGRAPHY. After echo we can be sure how much severe is the disease and what treatment option will suit the best.so I would like to advise you to consult a cardiologist personally, get an echocardiography done and then the right plan will be decided. After treatment a three to six months wait before conceiving is usually advised. wish you all the best"
},
{
"id": 110938,
"tgt": "Suggest treatment for swelling pain in lower back",
"src": "Patient: Hi, I am a mother of 3 children. Since giving birth to my youngest child, I will have swelling pain in my lower back near my buttock almost everytime after my period has ended for 2-3 days then it will disappear until after my next period. What is exactly causing this pain? Doctor: Hello, I have studied your case.Gynaecological reasons may also lead to back pain but first you need to correct structural problems if they are present.I will advise ultrasonography pelvis.Many patients have back pain during menstruation due to weakness and pain. You may consult physiotherapist for further guidance. He may start TENS, or ultrasound which is helpful in your case.Hope this answers your query. If you have additional questions or follow up queries then please do not hesitate in writing to us. I will be happy to answer your queries. If you find this answer helpful do not hesitate to rate this answer at end of discussion.Take care."
},
{
"id": 34517,
"tgt": "Suggest treatment for swelling and pain after dog bite",
"src": "Patient: my husband 240 lbs 6'2' 64 years old was trying to break up a fight between our two dogs and was accidently bitten on the hand. The wounds were puncture wounds and bled slightly. He wash the wounds and put bactine on the wounds. This was a week ago, the wounds have healed and look okay however his hand is swollen. We are now treating with heat and cold in half hour intervals Is this correct. I am having him call the doctor in the AM but was just wondering if we should do something immediately? He is a VET and on vet disability so our ER options are limited. Doctor: Thanks for posting you query to health care magic.first I would like to suggest you weather dog were vaccinated or not if not thhope you are satisfied with my answer . feel free to communicate if any query .en first step is to take rabies vacination .If your dogs were vaccinated then no need for it.now coming to current problem as dog bite causes penetration in crude form so damage of tissue and blood vessels laceration takes place which require long time to heal. but no need to worry as you should take precaution to avoid secondary bacterial infection by maintaing proper cleaning and dressing of wound with antibacterial ointment .For pain take :Tablet of Diclophenac potessium 50mg with Serratiopeptidase 5 mg two times a day .it will help in releiving pain and decresing swelling .Tablet Pentoprazole 40 mg once in morning to prevent drug induced gastritis .dont do hot and cold compression instead do either hot or cold compression. .there are increased chances of infection so take any antibiotic orally for 5 days like Amoxycillne 500mg 3times a day .regards,Dr.Manish PurohitInfectious disease specialist"
},
{
"id": 162941,
"tgt": "Suggest treatment for fatigue and neck pain in a child",
"src": "Patient: my 12 yr old daughter Alexis has had severe back pain, leg pain, neck pain, fatigue, shortness of breath and generally just feeling unwell for 2 months. All blood tests came back normal except for 2 Lyme titers which came back positive. However the dr feels this is a false positive and wants to run another test in 3 months. He doesn t want to start her on antibiotics. Doctor: Hello and Welcome to \u2018Ask A Doctor\u2019 service. I have reviewed your query and here is my advice. There are so many complaints that Alexis's illness is complex medical condition. I am assuming that musculoskeletal, cardiac, lung, endocrine, and anemia have been ruled out. I would like to know whether she has had a tick bite. Does she worry a lot? Has she missed any time from school in the past few months? I wish to give you information about a condition known as Somatic Symptom Disorder(SSD). It causes one or more bodily symptoms, including pain, nausea, dizziness, fainting, and abdominal pain. The symptoms may or may not have a physical cause. For the patient, they cause excessive levels of distress. The distress from pain or other problems are real, whether or not a physical explanation can be found. The distress from symptoms significantly affects daily functioning. Doctors need to perform many tests to rule out other possible causes before making the diagnosis of SSD. Some doctors feel that while the tests are being obtained the child ought to be evaluated simultaneously for SSD. In SSD the brain-body message system is not functioning properly: you may have such a strong emotion, e.g. anger. that your brain is overwhelmed and it sends signals so that you experience the emotion in your body as abdominal pain rather than as a feeling in your mind. As time goes by and if there is no diagnosis, you might consider asking if a developmental and behavioral pediatrician would be a good choice for children with complicated medical problems. I hope I have been able to give you some additional thoughts to help your daughter.Please return if we can assist you further. Arnold Zedd, MD, FAAP"
},
{
"id": 70659,
"tgt": "How can haziness over the bronchioles area be cured?",
"src": "Patient: I have been experiencing problem in breathing since a few days. Whenever I inhale, I feel pain in my mid-rib section in both front and back. It feels as if it is getting constricted or something. Due to this, I cannot breathe to my full capacity, and it gets even worse when I run, or sneeze, or any activity that involves heavy breathing. I had a previous case in which I experienced the same type of pain in my lungs, some 2-3 years back, due to accumulation of particles from mosquito coils and cigarette smoke (I don t smoke). The X-ray report showed some haziness over the bronchioles area, which the doctor describes as clogging due to which I am unable to breathe properly, This has resulted in dizziness on heavy workouts, and I also feel tired all the time. Can you help me out? Doctor: Hello, As per my clinical experience, the issue with haziness over the bronchioles and symptoms suggesting dizziness on heavy work out indicate compromised airway path and enough oxygenation is not taking place at the alveolar space level. The precise diagnosis with pulmonary function test is mandatory for the root cause, evaluation of elastic lung recoil, details of all capacity and lung reserves upon which the definite management should be built up. It is best recommended to get a consulting with an expert pulmonologist at earliest. Till that time, deep breathing exercises help greatly to relieve the issue partially. Hope I have answered your query. Let me know if I can assist you further. Regards, Dr. Bhagyesh V. Patel, General Surgeon"
},
{
"id": 147421,
"tgt": "Could it be a pinched nerve in the neck if having numb fingers and tingling in tongue?",
"src": "Patient: I have been painting my cottage and now when I sleep at night I am frequently awoken with numb fingers. My middle fingers aches so much I have to get out of bed and move around until the pain goes away. Today my tongue started to get tingly. could it be a pinched nerve in my neck as friends have suggested? Doctor: Hi,Thank you for posting your query.I have noted your symptoms. The most likely cause of your symptoms could be carpal tunnel syndrome (CTS), where the median nerve gets pinched at the wrist level.Pinching of nerve in the neck region would result in pain in the neck, and radiating pain in arm/fingers, etc. Tingling of the tongue can not be explained by either of them. Please consult a neurologist for evaluation.I hope my reply has helped you.I would be pleased to answer, if you have any follow up queries or if you require any further information.\u00a0\u00a0\u00a0\u00a0\u00a0Best wishes,Dr Sudhir Kumar MD (Internal Medicine), DM (Neurology)Senior Consultant NeurologistApollo Hospitals, Hyderabad,For DIRECT QUERY to me: http://bit.ly/Dr-Sudhir-kumar My blog: http://bestneurodoctor.blogspot.com/"
},
{
"id": 144688,
"tgt": "Suggest treatment for foggy brain with slow down in thinking process",
"src": "Patient: I have problems in the mornings with my brain being in a fog. I have often taken Sudafed which has helped to clear it up and sometimes Excedrin, but I try to stay away from it unless I have a bad headache. I do have some problem with hayfever and the Sudafed clears up my congestion. I also take an antihistamine for hayfever. My question is: Is there anything else I could be doing about the foggy brain, because he slows down my thinking process? Doctor: Hello!Welcome and thank you for asking on HCM!I understand your concern and would explain that your symptoms may be related to the raised sinus pressure from the nasal obstruction. The fact that your headache is relieved by Exedrin and sudafed is another argument in favor of this cause. From the other hand, antihistamines can cause these symptoms (slow thinking and sleepiness) as a common adverse effect. Do you have these symptoms when you don't use antihistamines? What type of antihistamines are you taking? I recommend you to consult with your allergologist and try to find another therapy for hay fever (immunotherapy or a non sedating histamine).Caffeine (in the coffee) could also be helpful if you tolerate it well. Hope to have been helpful!Best wishes!Dr. Aida"
},
{
"id": 102504,
"tgt": "Can maxiflo inhaler be used by child who is less than 5years for asthma?",
"src": "Patient: Hello Doctor, My Son is 4 years old and has Asthma. A Doctor suggested to use Maxiflo 125 Inhaler for 60 days. However, on CIPLA s website it is stated that Maxiflo 125 is not advisable for children under 5 years of age. Request you to let us know on how to proceed now. Doctor: Hello,Welcome to HCM,Maxiflo or other inhaler is not safe to continue for long time in children.Asthma is curable with homeopathic treatment. He is only 4 year old then don't make him habituated with allopathic medication when you have a better option in terms of homeopathy.Homeopathic treatment is based on total understanding of patient on physical and mental level. And that's why detail history of patient is needed for proper homeopathic treatment. Your problem can be cured permanently with homeopathy.So consult to good classical homeopath nearby you or contact us.Have a better health."
},
{
"id": 21631,
"tgt": "Suggest proper diet for controlling blood pressure",
"src": "Patient: Hi, My mom is 53yrs old, 5 ft tall and weighs 51 kgs. She s been taking medication for high blood pressure for past3-4 yrs. Her blood pressure is under control but recently her blood test has shown higher creatinine level (around 2.3) and higher urea level (100). Her potassium level was also high but is now under control. Sodium and phosphorus levels are normal Please suggest what diet she should follow to control these levels. Thanks Doctor: for diet your mom has to restrict her salt and have to cut down on fats n can add physical activity of one hour in her routine , but one concern is which medicine is she taking for hypertension? increase in urea n creatinine level can because either medicine she is taking or kidneys are involved now"
},
{
"id": 9958,
"tgt": "Should i continue using mintop 2% solution for hair fall?",
"src": "Patient: Hi, I m 17 years old and i m suffering from massive hairfall and male pattern baldness. For the past one year i lost a lot of hair. I had excellent hair before. I had been to a dermatologist, he presrcibed me mintop 2% solution, vb7 forte tablets and to take arythromycin 500mg one tablet a day for 3 days. I ve been using mintop and vb7 for 10 days now but there is no effect. Should i continue using it? Doctor: Hello, I have gone through your query and you need to continue these medicines for a long time. Hair fall will take time to settle and 10 days is very less time to see any results. You need to continue these medicines for at least 4-5 months to see significant results. Hope I have answered your query. Let me know if I can assist you further. Take care Regards, Dr Asmeet Kaur Sawhney, Dermatologist"
},
{
"id": 173812,
"tgt": "Does overdose of calpol and nerofen cause vomiting?",
"src": "Patient: My child 14 month old child spiked a temp of 39.1 and i gave him 10ml calpol and 5 mls of nerofen childrens liiquid.... He vomited both straight away so i repeated the doses and he kept it down.... Did i overdose him?My email YYYY@YYYY Thank u Doctor: It is very irritative for stomach and can cause overdose. When baby has very high fever you better use Adol-suppository-Acetaminophen-125mg. Give tepid sponging as well or usual bath. Kind regards"
},
{
"id": 48465,
"tgt": "What is the remedy for kidney stones?",
"src": "Patient: Was diagnosed with kidney stones by ER and 3 cat scans. Very painful lower abdomen and lower back. Also urge to have bowel movement and urinate is constant. Still after 2 weeks pain continues. Just had x ray and was told I had phleboliths. Is this something that is a sign of something worse my father just passed from stomach and colon cancer. Also the pain is extreme and unbearable. Is that normal what need to be done to correct this and will it go away on it s own Doctor: Hi,Thanks for writing in.Kidney stones can be confused with phleboliths on X rays. Phleboliths are benign findings and requires no treatment. This is confirmed by CT scans and does not have anything to do with bowel and urination. It is not something you should worry about. They are are literally vein calcium depositions, and represent calcification within venous structures. They are particularly common in the pelvis though can be seen anywhere in the body. This is a very common condition in the elderly when almost half of normal people have it. Since CT scans have been done and no serious conditions like tumors have been found hence there is no need to worry."
},
{
"id": 153883,
"tgt": "Is it necessary to remove testicles if suffering from testicular cancer?",
"src": "Patient: why haven't the doctors removed my partners testicle if they suspect testicular cancer? He has been receiving radiation therapy but not responding and now has to have chemo, but testicle still hasn't been removed. I was under the impression that this was the first thing to be done. Doctor: Hi, dearI have gone through your question. I can understand your concern.Your partner has testicular cancer. Treatment depends on stage and type of cancer and spread. In many type of cancer first radio or chemotherapy is given and then surgery is done. So it is common practice. No need to worry. Give me report of his tumour so that I can help you further to choose treatment options.Hope I have answered your question, if you have any doubts then contact me at bit.ly/Drsanghvihardik, I will be happy to answer you.Thanks for using health care magic.Wish you a very good health."
},
{
"id": 2067,
"tgt": "Should i undergo IUI?",
"src": "Patient: hi doctor i am NASREEN BEGUM height 5.5 weight 80kg age 30yrs married life 4yrs today i visit Dr. Anu clinic first time an they advise me IUI therapy,my problem is hypothyroid and mild pcod shall i go for IUI therapy till now i did not concive i used manymedicine an gone for laproscopy also nothing mention its normal ovaries normal utres normal endometrium normal plz waiting for ur responce Doctor: Hi I think you can go for IUI. it is a good option for you. It has chances of pregnancy around 10 to 15 percent. So you can try it for 3 to 6 months. Try to lose some weight. It will help you a lot. If it doesn't work then you can think about undergoing IVF. Hope I have answered your question."
},
{
"id": 23568,
"tgt": "What causes chest pain spreading to the left arm and leg?",
"src": "Patient: I woke up from a sound sleep with pain in the chest that went into my left arm and then my left leg and arm went numb to the point I had trouble moving. The next morning I was VERY tired and sore. My father has already had several heart attacks and we had genetic heart disease in our family. I have gone to the hospital several time and was told it was Angina. Nothing to worry about. My concern is this time it seemed a little different. Is this also normal for Angina? or could it have been a mild heart attack? Doctor: it was angina and nothing to worry aboutthis line seriously raises concern about the consulting dr you are undera dull aching pain usually left side with radiation to left arm is a classical presentation of patient with angina.angina happens when blood supply to your heart is less then what it needs to work properlyso an angina is basically a warning sign that your heart cannot recieve the amount of blood which it needs to work properlywithout losing any timego fore.c.ga 2 d echo cardiographyand tread mill testall these test will be helpful in ruling out if it is angina or any other pain mimicking angina (as radiation to leg is very unlikely)thank you"
},
{
"id": 28081,
"tgt": "Experiencing sharp burning pain in the side & front of the stomach area",
"src": "Patient: i have diabetes, i have extrem high blood pressure 4 meds for it, i have sharp buring pain in my side or in front of the stomach area. It is a sudden buring pain with out warining. It last seconds but the pain sends me to my knees or makes me stand up and become verbal because of the degree of discomfort. On a scale of 10 twenty mins. ago I had an 7 or 8 buring sensation andserious pain this has been going on now for several months. Last year had a xray of my left side and stomach nothing was found but the sudden pain and buring still continues.. Doctor: Hi. It sounds like gastritis. Drugs like pantoprazole will help. However in view of diabetes even cardiac or heart problems can mimic this condition. If it recurs plz do an ECG. Take omeprazole and observe for 12 hrs. Take care."
},
{
"id": 184802,
"tgt": "Why does food get stuck in between the gums of my teeth?",
"src": "Patient: Hi. I have a tooth that does not fit in my mouth, so it s on the side of another tooth, on top, believe it s a molar. Iv had it for about 8 years but in the past year latley iv been getting food stuck between it in my gums that becomes extreamly painful. Iv also been feeling tired, sluggish, dizzy, lightheaded, just simply out of it. I was wondering what this could be? It s running my everyday life Doctor: Thanks for your query, i have gone through your query. The food lodgement is because of the malaligned teeth. consult a oral physician and get your teeth cleaned and after that maintain oral hygiene. You can take an opinion from the orthodontist to correct the malaligned teeth with braces. you can use mouth washes and you should rinse your mouth soon after eating anything. i hope my answer will help you, take care."
},
{
"id": 122074,
"tgt": "Suggest treatment for painful bump on knee",
"src": "Patient: i was workiing on my truck last week and i knealt down on the concrete. when i got up i heard /felt a loud crack and since then i have a bump on the inside of my knee and its extremely painful...when i bend it ,it hurts real bad when my leg is straight iit doesnt hurt hardly at all.... Doctor: Hello, Your symptoms seem to be related to bursitis. Bursitis is inflammation of the bursa. A bursa is a small sac of fluid that cushions a joint and helps it move easily. Bursitis usually gets better if you avoid the activity that caused it. Meanwhile, I suggest using anti-inflammatory medications such as Ibuprofen to relieve the pain. I also suggest using cold compresses for local applications. Hope I have answered your query. Let me know if I can assist you further. Take care Regards, Dr Dorina Gurabardhi, General & Family Physician"
},
{
"id": 5322,
"tgt": "Have irregular periods. Had history of pcod and taking myp ionisitol and gestofit. Want to get pregnant. What to do?",
"src": "Patient: hello sir i am 28 year old and planning to conceive in coming cycles.i had a history of pcod and taking tab.myo ionisitol .iam having irregular periods also.i took t gestofit sr 300 for inducing menstruation (5 days od) i didn't get my periods 9 after taking the tab.did i wait or take the tab again.please give me an appropriate answer sir. thank you. Doctor: Hello,Thanks for your query. Gestofit is micronised progesterone, ideally you should get periods by 2 to 7 days after the last dose. I strongly suggest you to get a urine pregnancy test. I hope this information has been both informative and helpful for you. In case of any doubt, I will be available for follow ups.Wish you good health.Regards, Dr ArifYou can consult me again directly through my profile http://www.healthcaremagic.com/doctors/dr-arif-n-khan/65133"
},
{
"id": 129446,
"tgt": "What could cause recurred severe pain in knee with history of knee fracture and on brace and ibuprofen?",
"src": "Patient: bWell, the first time i was trying to run and do exercise movements and all of a sudden...i heard a noise in my right knee area. It hurt a lot and was shooting down my leg and on my side of my top right foot which still hurts a lot. I had gone to the doctor...she sent me for xrays....and about 6 weeks later did an mri. They said nothing except they found a fracture on the knee and did nothing more about it. About a year later...i fell...landed on my knee...it was a different kind of pain....kinda made me sick feeling...couldnt feel anything and for a second or two, i felt like i couldnt move it. Doctor took xrays...she said nothing but a lot of mass. A week later, i fell again and in the same knee. This time doctor thought i was faking it and did nothing...no meds, brace...nothing. A few days later i couldnt stand the pain anymore, so i went into the ER. She took an xray...same old fracture showed up, she put on a brace and is having me take ibprophen and using an icepack and have it higher then my heart. When they put on the brace...it pushed my knee cap back into place. Very painful...will this go away?? What is the problem?? Cartlidge?, tendenitious??, bursitus?? Its numb under my knee cap along with a bump and a bump on each side of the knee. Help me doc...what do you think??? Doctor: Hello,Thank you for your question.From the information that you give I think that you are developing what we call post traumatic arthritis. You say that in X-ray and MRI it was a fracture on the knee that may have been tibial plato or femoral condyle or knee cap. The fracture is healed but since it was an intraarticular (inside the joint) fracture, it heals with some step off and may cause later arthritis.Of course I would like to see the images of the MRI to confirm this diagnosis.If this is the case I would recommend- Loose weight- Physical therapy , range of motion , quadriceps muscle strengthening- Glucosamine and condroitine treatment from mouth- Consider intra-articular injection with Acid Hyaluronic combined with cortisone injection.- cold therapy ( ice packs) against the swelling- anti-inflammatory drugs according to level of pain and knee swellingI hope I helped you"
},
{
"id": 80212,
"tgt": "Suggest remedy for persistent cough with little help from Asotolon and Brozedex",
"src": "Patient: Hullo Rynne, I greet you from Uganda. I have this cough that has persisted for like 3 or more months. I have checked and medicated my self, but it does not seem to go. right now am taking allergy tabs (Asotolon) and an expectorant syrup (Bro Zedex). It probably is working. Would advise some that i may not be doing. Doctor: Thanks for your question on Health Care Magic. I can understand your concern. In my opinion, we should first find out the cause for your cough and then start appropriate treatment. You are having chronic cough (cough for more than 2weeks). Following are the possible causes for this. 1. Tuberculosis 2. Chronic pneumonia 3. Bronchitis 4. Bronchiectesis. So better to consult pulmonologist and get done 1. Clinical examination of respiratory system 2. Chest x ray 3. PFT (Pulmonary Function Test). Chest x ray is needed to rule out infection like tuberculosis and pneumonia. PFT is needed to rule out bronchitis. So first diagnose yourself and then start appropriate treatment. Hope I have solved your query. I will be happy to help you further. Wish you good health. Thanks."
},
{
"id": 154174,
"tgt": "Can chemotherapy cause death?",
"src": "Patient: yes hello my mum has just been told to stop her chemo treatment she is a breast cancer patient was having echemo every 3 weeks has had 4 sessions and the last lot of chemo caused her to faint and angles are swollen and slightly red has a pimply rash on her chin ... my question is can chemo therapy be harmful and can it cause daeth Doctor: Hi, dearI have gone through your question. I can understand your concern. Chemotherapy has many side effects. It worsen thr the quality of life. Tolerance of chemotherapy is different in different person. In some person it becomes fatal. It is better to live with cancer then takin chemotherapy is true for person who can not tolerate that. So doctor advice to stop chemotherapy. Hope I have answered your question, if you have doubt then I will be happy to answer. Thanks for using health care magic. Wish you a very good health."
},
{
"id": 39170,
"tgt": "Suggest remedy for purple wound from insect bite",
"src": "Patient: I was stung or bitten by something on my stomach....very strong sting at first, then maybe again...then I didn't feel and pain after a little while...when I looked at it later, it was bright red and swelling and itching...now, 3 days later, it has turned purple...what should I do? Doctor: HI, thanks for using healthcare magicThe purple discolouration is due to the bruising as a result of the bite sting. It will resolve in a few days .It does not signify a problems.If it is painful then the use of an anti inflammatory pain killers would be helpful for a few days. ( Eg- ibuprofen, cataflam, aspirin, naproxen, celebrex)I hope this helps"
},
{
"id": 114027,
"tgt": "I have sores on penis. I use to masturbate twice-thrice a day",
"src": "Patient: Hi, I have red sores on my penis head . they are small, around 1-2 milimeter in diameter. They are coming and going for last 5 years. I also have lower back pain for last 5 years. I always have a pain in my penile shaft. when I ejaculate/urninate I feel a burning in the shaft. Please help me. I have donated blood . And in final screening i was cleared off of any VD, malaria etc. In KUBE test ( ultrasound ), I was diagnosed with a Cystisis like symptoms. Thickening of UT walls. I never have sex out of wedlock, but use to masturbate twice-thrice a day. Doctor: Welcome to Healthcare Magic Recurrent lesions on penis could be due to allergy to certain drug or can be due to STD like Syphilis among causes of such a clinical picture. Are you on any medication. Masturbation may not cause such a condition unless penis head is dry and repeated stimulation can cause erosions and redness. If it is such a case you can use lubricant like KY Jelly while masturbating. For cystitis you will need to drink lots of water to prevent symptoms like pain, burning sensation on passing urine. Diagnosis of cystitis can be associated with certain infections or conditions which can be found out by biopsy of urinary bladder by cystoscope which may help more in treatment. You can consult your Urologist these options."
},
{
"id": 126767,
"tgt": "What causes pain in the lower leg?",
"src": "Patient: I have had pain in my right leg. Usually it is in the back of my thigh and wraps around my knee down to my ankle. Popping noise when straightening my leg. I did fall over a year ago but the pain has gotten more severe of the last few months. Really hurts to straighten my leg. I do have loer back paid also. Doctor: Hi, Your symptoms are suggestive of radicular pain in lower limb as result of disc prolapse in lower back. Radicular pain occurs due to nerve root compression in lower lumbar spine and pain is like radiating from knee to ankle and foot as you described. Diagnosis can be confirmed by MRI of the lumbo sacral spine. MRI is very sensitive and specific for diagnosis of spine pathology. Meanwhile start tablet Pregalin one at bedtime with tablet Ultracet twice a day for pain relief. Visit to spine surgeon for detailed clinical examination is recommended. Hope I have answered your query. Let me know if I can assist you further. Regards, Dr. Jayesh Vaza, Orthopaedic Surgeon"
},
{
"id": 19963,
"tgt": "What can cause palpitations?",
"src": "Patient: How fast can heart disease develop? I had a stress test and ultra sound about 5 years ago and was told my heart was very healthy at that time. I had an oblation about 11 years ago, and just recently I've had a lot of heart palpations and fluttering heart again, along with a feeling of butterflies in my stomach and some heart burn. I've been suffering from dizziness and headaches and fatigue for the last year and that was diagnosed as a type of migraine. Should I be concerned about heart problems? I'm a 46 year old female, the only heart disease in my family was my grandmother. I'm not overweight and typically have low blood pressure 90/56, with a pulse of 90-110. Doctor: Thanks for your question on Healthcare Magic. I can understand your concern. Women are at increased risk of heart diseases during and after menopause. Since you are approaching menopause, you are at risk for heart diseases. There is no definitive period for development of heart disease. So at your age, it is always recommended too rule out heart diseases first for palpitations. So get done ecg, 2d echo and Holter monitoring (24 hours continuous recording of Ecg). If all these are normal then no need to worry for heart diseases. Hormonal imbalance during menopause can also cause palpitations. So consult gynecologist for this. Hope I have solved your query. I will be happy to help you further. Wish you good health. Thanks."
},
{
"id": 92139,
"tgt": "How to heal a prolonged pain in the lower stomach?",
"src": "Patient: I have pain in my right groin for 15 yrs getting worse have pain in lower stomach area is really sore when I DO THINGS LIKE CUTTING GRASS OR LIFTING. dR HTHINKING MAYBRE HIP BUT i'M NOT SO SURE WITH THIS PAIN IN LOWER STOMAH AREA WHEN i DO SNALL THINGS CUT GRASS, BEND OVER. tHANKS nOREEN Doctor: HI. I hope by this time your Doctors have seen you for hernia, which commonly causes this problem , but missed many times so easily. Is there a small bulge in groin area if you strain or cough ?"
},
{
"id": 43445,
"tgt": "Trying to conceive, had unprotected sex. Prescribed duphaston. Correct medication to conceive?",
"src": "Patient: hi doc i had my lmp on 11 march and had unprotected sex on 17 march.....since last three months i was on ginnette 35 and now my doc told me to stop taking it and start with dupharton 10 mg from 15 day of lmp......i want to conceive should i take dupharton as i had unsafe sex and want to conceive a healthy baby................plz help Doctor: Hi,Thanks for your query. I read your query and I understand your concerns. Following is my reply:1) The medicine your doctor has mentioned is a progesterone.2) This will help to carry the pregnancy if conception has occurred.3) Continue medicine without worryI hope I answered your query. I will be available for any follow up query you have.Regards,Dr. Mahesh KoregolIVF & Infertility Specialist"
},
{
"id": 126899,
"tgt": "What could cause acute pain in the elbow and numbness in the arm?",
"src": "Patient: My girlfriend and I recently traveled to Colorado from Texas. We have been here about a week, and she has began to experience an acute pain which started in her elbow. The pain has lasted about six hours now, and has progressed from something she described as shin splint like pain in the left elbow to numbness and tingling down the entire left arm. The pain is not increased with palpation. She takes oral contraceptives, and Synthroid. She has hypothyroidism, and NKDA. Any idea what this could be? Doctor: Hi, The pain you described can be radicular pain in entire arm due to nerve irritation in the cervical spine. The cause may be long distance travelling with sitting in an odd position. Ask her to take a rest. Avoid lifting weights with the left upper limb. Start tab Pregalin one at bedtime with tab Ultracet twice a day for pain relief. If not relieved in 5 days, please consult an orthopedic surgeon nearby your area. Hope I have answered your query. Let me know if I can assist you further."
},
{
"id": 99826,
"tgt": "Suggest therapy for allergies",
"src": "Patient: I am a 23 year old female, mother of a 2 yr old son. I have severe allergies that are moderate all year except August-October. Had allergies all my life, am allergic to dust, mold, pollen, dander, penicillen. Been on allergy shots when I was 13-16 and all kinds of medicines such as Zyrtec, Benedaral, Allegra, Claratin, over the counter medicines like Chlorotabs. Nothing worked. The only thing that does help is the Methylprednisolone pack. This past entire month I have had a completly stuffy/drippy nose, sneezing at least 80 times a day with fits of 10-15 each time causing me to get dizzy and out of breath, headaches, itchy legs. I have a very powerful air purifier with a hepa filter and some kind of zapper in it, I also run a humidifer on 24/7. I try to keep my house as clean as possible, I have a latex bed and pillow so dust mites is not an issue. I cannot get relief inside or outside. I go through a box of tissues every 2-3 days. My eyes are burning and itchy as is the inside of my face. Some days its so bad I can't even open my eyes. The more I move around the more I sneeze. The stuff coming out of my nose is clear. Vicks vapor rub does not help, no nasal decongestants help. Hot showers do not help. What else can I try for relief, this is killing me! Is there anything similar to the Methylprednisolone packs that are safe to take continously? Doctor: Hi, it seems that u are self medicating since long time, if u are allergic to so so substances and u are taking predinisolone, wrll ,steroids are strong anti inflammatory drugs and will defiantly gonna help but the question is that how long u should dependt on.u consult an ENT specialist for mainly sinusitis or any other nose problem. Here u take steam inhalation as much as it 100% safe and will help to relieve from congestion if any."
},
{
"id": 155233,
"tgt": "What is prognosis for oesophagus cancer?",
"src": "Patient: my uncle has been diagnosed with oesophegus cancer just this morning. there was alump on his neck which we got checked out and they said somethng about cancerious cells. he does not feel any discomfort when eating and no vimitting nothing! he feels normal so i mguess it is inthe initial stages. he is about 55 years of age and no health problem. quite fit in fact! i wanted to know about the survial rate and everything in general! =( Doctor: Thanks for your question on HCM.Treatment, prognosis and life expectancy of any cancer is depend on its staging. And this goes true with esophageal malignancy.So to determine staging in his case following investigation s are needed.1. CT scan of neck to see for local extent of disease.2. PET scan to see for distal spread of disease.If distal spread is present then it is known as stage 4 cancer and it carries poor prognosis. Life expectancy is less than 1 year. Only palliative chemotherapy and radiotherapy is given.In the absence of distal spread surgical removal of esophagus with clear margin is the treatment of choice. Post operative chemotherapy and radiotherapy is given to prevent local as well as distal spread. In these cases life expectancy is less than 5 years.Overall esophageal malignancy is not having good prognosis because of poor feeding issues after surgery."
},
{
"id": 55151,
"tgt": "What causes elevated LFTs?",
"src": "Patient: Taking 10/500 Lortab for 11 years for chronic pain. Last two LFTs showed elevated, the last one two days ago being very high (although I don't know the numbers). Sonogram of my liver to be done in two days.upper right abdominal pain, but has been there for over three years. Could my elevated LFTs indicate cancer? Doctor: hi.noted history of elevated LFTs. some possibilities are of infective and obstructive (maybe secondary to a gallstone or a tumor lesion such as malignancy) causes. it is best if you consult with a doctor, preferably a gastroenterologist or a general surgeon, for physical examination and clinical evaluation. diagnostics (such as ultrasound, ct-scan, bilirubins, blood count, etc.) and management (medical and/or surgical if indications are found) will be directed accordingly.hope this helps.good day!!~dr.kaye"
},
{
"id": 19156,
"tgt": "Suggest treatment for high BP",
"src": "Patient: Dear doctor,After my return from tour my bp was 200/120.I am having hipertension for which i am taking for 15 years attenol 100mg plusenalparil5mg twice a day plus simstat 10 for cholestrol. Doctor ,now, prescribed amlong 10 plus vertin 8. My age is 63 years.After 4 days of this medicene along with my usual tablets my bp is 150/90. Please offer your opinio.n/ advice Doctor: siras per ur BP u require 2 or 3 drugs so what ur taking is essential.with atenolol 100 mg u may experience weakness and dizziness if u don't have these then u can continue atenolol same dose.if u r a smoker then stop it.if u drink too much coffee then decrease it.if take too much alcohol decrease it to 2 drinks per day."
},
{
"id": 93931,
"tgt": "Hard, painful, big formation near the naval. Hernia?",
"src": "Patient: I believe I have a hernia in my stomach. A month or so ago I laid on my stomach (at night ) and felt a formation by my naval. Its hard and the pain demanded I turn over. Well the holidays came up so I delayed going to the doctor. Then in Feb. I had to have openheart surgery, a triple bypass. That prevented me from laying on my stomach so the mass was forgotten until last night. I turned over and there it was, bigger and harder than before. Could it be a hernia? Doctor: Hello, Thanks for posting your query, From your description, if the bulge is retractable or reducible then the chances of it being an umbilical hernia is high. As you described that it usually occurs when you lay on your stomach, meaning that your abdominal are highly tensed/contracted and if there is bridge on your abdominal wall, that will force intestines to bulge through the orifice hence you feeling it on your navel. It will be best if you could pay a visit to a doctor who can take a closer look at it to determine exactly what it is. Other possibilities are intraabdominal benign tumors. Abdominal scan could help determine the nature of the bulge. Hope this helps and wish you the best"
},
{
"id": 219844,
"tgt": "Suggest treatment for vaginal bleeding during pregnancy",
"src": "Patient: hi doctor,i would like to know if someone is 18weeks pregnant and was getting light blood stains and out of sudden started bleeding when went to toilet. doctor has examined her and advised to have proper bed rest. he said the cord or something has started leaking or something. cud u please tell if the baby is going to be fine. doctor also said mentioned the baby is lying quite low.thanks Doctor: Hallow Dear,At any stage of pregnancy, any amount of bleeding should be taken seriously. Chord never leaks. Please undergo ultrasonography. It will visually locate the site from where you are bleeding. If it is from behind the placenta, you have to be on the bed. Scan also will inform whether the foetal heart activity is present or not. Also it will inform about the status of the cervix (mouth of the uterus). If it is short and internal mouth is opening, you may require reinforcement stitch to the cervix after the bleeding stops completely. If the baby has not been affected, I would advise you to take full bed rest and start Progesterone preparation with Obstetrician's advice. So please report to the Obstetrician ASAP. I hope this helps you.Dr. Nishikant Shrotri"
},
{
"id": 201556,
"tgt": "Is the sperm contagious outside human body?",
"src": "Patient: Respected Doctor, First of all THANK YOU for giving Sex Education. I am a Male guy,Age: 24.I have a 3 Questions,PLEASE HELP ! I musterbate a lot but i am feard bcause the sperms/liquid is touching/mixing with my undergarments,cloths,bedsheets,or other cloths.& sometimes I keep my Underwear as it is in bucket of water in bathroom & unknowingly it mixes with my Sister/Mother/Father s clothes for sure. I do not wash my penis after masturbation. Question1: will my sister becomes pregnant?also her stomach is growing/enlarging,why? Question2: bcause I donot WASH my PENIS,& it touches my underwear whole night,will it cause HIV or any other INFECTION? Question3: will there be any problem of HIV/AIDS/INFECTION to my parents? I hope You will do favor for me.PLEASE. Yours Respectfully. Doctor: Hi Thank you for asking HCM. I have gone through your query. You don't need to worry as sperms won't be viable outside body. But it is not good to mix your undergarments with others because it can spread any infections. Even fungal infections can spread like that. Wash your undergarments separately. Hope this may help you. Let me know if you have any further queries."
},
{
"id": 184652,
"tgt": "What causes swelling in cheek?",
"src": "Patient: I have a very swollen cheek along the jaw. I visited the dentist on Tuesday morning and was told I have an abscessed tooth where the infection has gone into the cheek. I have been on the prescribed medication since Tuesday lunchtime and today the swelling is far more prominent and I am having difficulty opening my mouth. Doctor: Hello,Your infection is not under control and symptoms are increasing. Call your dentist to report that your condition has not improved and is worse. Your dentist will need to increase the dosage or change your antibiotic. Make sure you are following directions on taking all medication. I usually recommend Amoxiciilin 500 mg or 875 mg depending on how often you take the antibiotic. Your dentist will recommend the next step for treatment of the abscess. You may need other oral conditions evaluated as well.Take anti-inflammatory medications in addition to the antibiotic. Tylenol and Motrin are effective choices. The choice of the antihistamine Benadryl also helps control swelling and control symptoms. Keep well hydrated. Do not stress your jaw. Avoid irritants. Continue daily dental care and rinsing.Thank you for your inquiry. I hope you will find relief with my suggestions and do not delay in contacting your dentist with an active infection present. I am glad to assist further if you have additional questions."
},
{
"id": 190199,
"tgt": "Tingling and swollen lump in the mouth. What is causing it?",
"src": "Patient: I was eating all of a sudden felt my inside of mouth against my lower teeth . Feels like when you bite inside mouth and it gets warm and swollen and tingling but no sores nothing inside or out but it feels like when you can feel swollen gland and there is bump but just under lip inside face. I have been sick for week not sure if related Doctor: It looks like that your lower tooth is infected. It can be because of decay. As you do not have any hot and cold sensation, it means the tooth has become non vital and pus formation is there. The pea size lump is the abscess which is formed on the tooth. For this you have to visit dentist, who will do clinical examination and take x-ray and find out the extent of decay. This tooth will require Root Canal Treatment. The abscess can only be drained out by Root Canal Treatment."
},
{
"id": 108837,
"tgt": "What causes pain and numbness in lower back?",
"src": "Patient: first i have to thanks to you doctor.since last 3-4 weeks i have pain my lower back.actuaiy that is num feelings.when i carry some hevy things that increece.when i have hot water bath that make less.when i want to pass urin feel more but afterget less.is it kiddney problem or rhumetic problem?becouse 1 year ago i had rhumetic atheriteas.please answerser me. Doctor: Hi,The symptoms you have explained aren't from the kidney problems, though they may be from the rheumatoid arthritis of the spine. But it rarely involves the lumbar spine which is the lower back region. I would suggest that you discuss it with your rheumatologist and get a MRI scan of lumbar spine done to confirm its cause. It can also be due to a pinched nerve due to a protruded disc."
},
{
"id": 176163,
"tgt": "What causes abdominal pain, bloating and foul smelling gas to a 6 year old?",
"src": "Patient: My six year old daughter has been complaining of mild abdominal pain for the past five days, yesterday her stomach was very bloated after lunch and she had a lot of foul smelling gas the rest of the day. This morning she had pale putty colored stool. She also says she s tired a lot. Doctor: Hi...these are symptoms are viral hepatitis. I suggest you consultant your physician or pediatrician and get her evaluated for acute viral hepatitis. If not it could as be a viral illness or an evildoing enteric fever too.Regards - Dr. Sumanth"
},
{
"id": 18365,
"tgt": "Suggest alternative medication for high blood pressure with aneurysm",
"src": "Patient: my mother is 76 yrs old and is currently on blood pressure medicines. she had a stint put in her aortic vein several yrs ago and since then the dr. s have found another aneurism. her primary care physician added a new bp med last week which so far has not helped to lower her blood pressure. what should I do? Doctor: Hello and Welcome to \u2018Ask A Doctor\u2019 service. I have reviewed your query and here is my advice. If blood pressure is not decreased then it may be controlled by alternate medicine otherwise it may cause dissection of aorta. In aneurysmal patients may be used beta blockers or nitroprusside to control blood pressure. Please consult your physician he will examine and treat you with alternate medicine. Hope I have answered your query. Let me know if I can assist you further. Regards, Dr. Penchila Prasad Kandikattu"
},
{
"id": 120789,
"tgt": "What causes severe pain in the armpit of a 2 year old?",
"src": "Patient: My two year old daughter fell and later last night she was complaining her arm hurt so we took her to the er they said possibly nursemaids elbow and she may have fixed it on the way there. So most of today she has been fine at bath time she started sayin her arm hurt and now she has pin pointed tgat her armpit hurts but the xray came back fine how do I help her and what could it be she is honestly in pain Doctor: Hello,I read carefully your query and understand your concern. The symptoms of your child seem to be related to the injury. I suggest giving her Tylenol for the pain.I also suggest using cold compresses at the place that he feels painful.I also recommend resting and avoiding the activities that can trigger the symptoms. Hope my answer was helpful.If you have further queries feel free to contact me again.Kind regards! Dr.Dorina Gurabardhi General &Family Physician"
},
{
"id": 195334,
"tgt": "How often does prostate cancer re-occur after a prostatectomy?",
"src": "Patient: I had prostate cancer and a prostatectomy in January 2014. It was contained in the prostate and my surgeon said all was clean. My last three annual psa test have been .03, .05, and.09. With the .09, the Dr says the cancer has returned. I'm not sure I understand how it is possible.also, I was told a little radiation would be the treatment once my PSAT reached .2. I will be 63 in May. Is this normal? How often does this type of recurrence happen?Thanks. Doctor: Hello and Welcome to \u2018Ask A Doctor\u2019 service. I have reviewed your query and here is my advice. Recurrence may happen after prostatectomy in case of prostate cancer. Serial PSA assessment is required if we see a rising trend in PSA level, we have to suspect a recurrence. In your case there is rising trend and we have to intervene. Get an MRI scan done as soon as possible. Consult your urologist and he will direct you accordingly. Wishing you good health. Thanks."
},
{
"id": 151853,
"tgt": "Do I need to take more medication after my course is completed to treat tuberculoma ?",
"src": "Patient: my tuberculoma was cured and my doctor said it was very less and i dont need to have any more my medicines whan the total course was completedbut sometimes still my left side of the brain pains plz help Doctor: Tuberculoma of brain is usually resolve with 18 months of anti tubercular chemotherapy. So don't worry for that.your pain may be due to other causes........."
},
{
"id": 54352,
"tgt": "Suggest treatment for SGPT level 43",
"src": "Patient: hello sir i was patient of HCV after treatment of six month it clear and after other six month i test again Thanks God it clear again i thank to allah but now i did my LFT test and in report my SGPT is 43. sir plz tell me it is normal or not if it is not normal then what i do Doctor: Hi thanks for contacting HCM...Noted you had infection with hepatitis c...In such case chance of chronic infection is high as compared to other hepatitis....So it is good that you have cleared this virus from your body ....By which method you have confirmed ???For HCV detection ELISA or equivalent PCR methods has to be used....So confirm your negative report by this method....Normal sgpt is up to 42 to 45 ....Here no need for worry....Just confirm HCV negativity by methods I have mentioned....Take care.Dr.Parth"
},
{
"id": 187143,
"tgt": "Can i take medicine for swelling on cheek?",
"src": "Patient: Hi, I can't get into my dentist for a few days. I was prescribed Nitrofurantoin for a urinary track infection. Can I use that for now. My face has a golf ball coming out of my cheek! I have heard that a dentist is going to tell me to take anitbiotics before they can do anything anyway. This way I can at least take the swelling and infection down. I am in SO MUCH PAIN. Doctor: Hi,Thanks for posting the query, This can be due to allergic reaction, i would suggest you to stop the medication consult to the nearby Dentist and get the checkup done.At home take lukewarm saline and antiseptic mouthwash rinses.For pain relief take tab brufen BD for 5 days.Hope you find this as helpful,Regards..."
},
{
"id": 64443,
"tgt": "Could the bump on lower lip be a cold sore?",
"src": "Patient: Bump on lip I had a bump on my lower lip like a cold sore about three weeks ago and now for the past week I have had a bump on my top lip. It looks as though its going away but is dark like it will leave a dark mark. Iam 36yrs old, Male, I smoke ciggarretes daily and I drink beer twice a week. I also drink coffee every morning and I have a dog. Doctor: Hi Dear,Welcome to HCM.Understanding your concern.Thanks for your query. The probable reason for bump on your lip is is canker sore or cold sore . There can be many factors responsible for canker sore like stress (very common) ,irritation of mucosa due to food ,fruits and drinks, vitamin deficiency and underlying diseases that weaken immune system.They usually go away by their own without any treatment in week or two or by avoiding trigger factor .I would suggest you to apply some antimicrobial ointment ,start multivitamin supplement containing vitamin B12 and folic acid , quit smoking and drinking at least for some time and apply gel containing benzocaine for pain .If condition persist after 2 weeks then consult dentist for proper examination and to rule out causes like HIV , herpes or oral cancer .Hope your concern has been resolved.Get Well Soon.Best Wishes,Dr. Harry Maheshwari"
},
{
"id": 191042,
"tgt": "Is ponstan 500mg advised in toothache ?",
"src": "Patient: I have been suffering with toothache and was told by the pharmacist to take ponstan 500mg...im 120kg and have taken 10 so far today and nothing!!! Help!!! Doctor: Hello and Welcome to \u2018Ask A Doctor\u2019 service. I have reviewed your query and here is my advice. Only pontan will not help in toothache. It needs to be covered with antibioticis + mouth wash for local effect. So visit a dentist for evaluation. Hope I have answered your question. Let me know if I can assist you further.Wish you a good health!Regards,Dr. M. Imran Javid"
},
{
"id": 401,
"tgt": "Can pregnancy happen after having non penetrative sex?",
"src": "Patient: I'm 17 and about 5'3 and about 120 pounds and healthy. I was making out with my boyfriend on top of him ad we were both Fully clothed. He pushed me off and said he \"came\" he said it was in his boxers and a tiny bit on his jeans. I wiped my pants to see if it got on me and I didn't seem to feel anything but I'm not sure. I went to the bathroom and without thinking wiped myself with the same hand. I'm deathly afraid I could get pregnant. I never had sex before and I plan on not till I'm married. I just want to know if I could or not. Thanks. Doctor: Yes pregnancy can occur without penetration however it is very unlikely especially in the case you describe."
},
{
"id": 75447,
"tgt": "What causes chest pain in 8 year kid?",
"src": "Patient: Yes my 8 year daughter was complaining of her chest hurting today so went to er they did chest x-ray and than ekg and found an abnormal reading.. They said it was the two top parts of the heart wiring were misfiring, what the hell is that.... They said its normal in kids and unless she is dizzy not to worry.... Doctor: Hello dear, Thank you for your question. I don't have sufficient information on the ECG report to diagnose your daughter. It is common for children to have strong heartbeat (palpitations) or in the case of your daughter \"irregular\" rrythm (benign arrythmias). The majority of these symptoms tend to fade as the child grows and the conduction system of the heart matures. As long as you consult periodically with a specialist there is nothing to worry about.I hope this helped.Best wishes,Dr. Alma"
},
{
"id": 107575,
"tgt": "Suggest treatment for severe back pain and bladder infection",
"src": "Patient: Went in to urgent care yesterday with severe back pain...bladder infection, sent me for ct scan found kidney and gallstones, very anemic and iron deficient (26) Also found spot on my lung that they want to watch for three months and are sending me for colonoscopy I am a 52 year old female What do you think? Doctor: Hi there.These problems need to be treated by a multi-disciplinary team like Cardiothoracic surgeon and Oncologist for the spot in the lung, Internist for the Anaemia, Hepatobiliary surgeon for the gallstones and Urologist for the Kidney stones for a proper approach to the best management.Drink 2 litres of water daily. Eat plenty of fresh fruits and vegetables. Take iron supplements. Usually Anaemia and spot in the lung can be suspicious for cancer and needs to be ruled out by your Oncologist."
},
{
"id": 167949,
"tgt": "Suggest treatment for developmental delays in a child",
"src": "Patient: I am a peds occupational therapist. I see a 4 year old boy who has some developmental delays. He is also seen for speech therapy. He has a very soft voice and doesn t seem to have much respiratory reserve to speak or sing more than a phrase or two. He has very slow processing time for following directions and seems to have some motor planning problems. He is a possible candidate for an autism diagnosis. My concern is that in addition to the soft voice he has a sporadic gasp-like breathing pattern - takes in a deep breath, holds it, then exhales. I can t pinpoint any specific trigger in his environment - anxiety, excitement, etc. Sometimes he repeats it several times. While he does this he seems to disconnect from whatever else he is doing. Others involved with him think it is a self-stimming behavior. I am concerned he has respiratory issues of some sort, or could it be related to seizure activity or something else? Doctor: neither a respiratory nor a seizure activity is likely, could you provide more information about his perinatal and natal history, and whether a brain MRI was done or not"
},
{
"id": 181952,
"tgt": "Suggest treatment for severe pain after root canal treatment and crown placement",
"src": "Patient: I had a root canal on a back tooth, number 25 I believe, the dentist did not fill the four root canal holes and he put a tempory crown on......it has been one year. I have had crazy pain.....how long can you wait before having the holes filled. It was a tempory cement used and a plastic tempory bridge on all three back teeth. Doctor: Thanks for your query, I have gone through your query.The pain after the root canal treatment can be because of the missed canal, or incompletely filled root canals. Nothing to be panic, consult a endodontist and get it retreated. You might have to get the bridge removed and get the tooth retreated. Mean while you can take a course of antibiotics like amoxicillin and metronidazole(if you are not allergic).I hope my answer will help you, take care."
},
{
"id": 58160,
"tgt": "Had gall bladder surgery. Pain in stomach just below ribs and on the back right side. Comments?",
"src": "Patient: Hi. I had gallbladder surgery two and a half weeks ago and am still having pain at the same levels as pre-op - in my stomach area just below ribs and often on the back right side as well. I ve been careful of what I ve eaten and haven t eaten much at one time. Sometimes the pain occurs perhaps a half hour or so after eating and sometimes it seems unrelated. I m supposed to fly to Sarasota from MN Wed. to see my mother and am getting a bit nervous. Comments? Doctor: Hi and welcome to HCM,thank you for your query.It can be postcholecystectomy syndrome which is common complication of gallbladder surgery. but also there could be stone in choledochus duct or pancreatic duct. Or your preop. pain was initially caused by something else,pancreas or stomach issue. However, you should follow hepatic diet and drink plenty of fluids till you find the exact diagnosis. Also,take some antispasmic medicines.if that helps,it may be stone in bile ducts.Wish you good health. Regards."
},
{
"id": 73007,
"tgt": "What causes dizziness and chest discomfort?",
"src": "Patient: I am 48 years old man my echocardiography report is normal but ECGsuggets blockage in walve as per doctors . I feel dizzyness & uneasy in chest afters every meals. my uncle , father & grandfathers died of heart attack I have sugar ,uric acid , cholestorol above the border level Doctor: Thanks for your question on Healthcare Magic.I can understand your concern. You are at high risk of heart disease because you are having diabetes, dyslipidemia and strong family history of heart diseases.So we should definitely rule out heart diseases for your symptoms.Ecg can be normal in some patients. So normal ecg is not always guarantee normal heart. So better to consult cardiologist and get done stress test and angiography to rule out coronary artery disease (CAD).You should strictly control your diabetes and cholesterol. Hope I have solved your query. I will be happy to help you further. Wish you good health. Thanks."
},
{
"id": 173896,
"tgt": "What is normal heart rate for children?",
"src": "Patient: Hi. My daughter is a over 20 months old and tonight I happened to notice that when I put my hand on her chest, I could feel her heart beating. It felt like it was beating hard and fast. This is the first time I've noticed this. Is this normal for a baby her age? Doctor: Hi, I can understand your concern. The normal heart rate for a baby of around 20 months will be between 90-110 beats per minute. There might be variations in rate with crying, playing, sleeping, post meals, fever, dehydration etc. Chest wall of baby is comparatively thin, it is normal to feel heart beat with palm even in adults. So don't worry about it. You can monitor heart rate, activity of baby, watch for fever and any other symptoms. If any then consult your Pediatrician. Hope this helps you, if so do vote."
},
{
"id": 162506,
"tgt": "Is it advisable to give Graniforce to our 13 months old child weighing 11 kg for persistent vomiting?",
"src": "Patient: My child is suffering from direhha since long...it got repeated actually within 10 days...main problem is wid vomiting mwdicine...we havevtried ondem,domstal,.both the medicine he again vomits...then we tried emefilm sachet..which doctor told to take if not getting control by previous medicine.Now changed the medicine and doctor advice to give graniforce.so please advise me his age is 13 months...and weight 11kg Doctor: Hello, Graniforce of Emefilm are all same family drugs. If it is not getting better with Emefilm (Ondansetron) I suggest intramuscular or intravenous injection of Ondansetron rather than changing to another oral medication. Hope I have answered your query. Let me know if I can assist you further."
},
{
"id": 31618,
"tgt": "Suggest remedy for sores in mouth when diagnosed with herpes infection",
"src": "Patient: I have just been diagnosed with HSV-1 but it was a painful genital breakout. First one. My wife has also been diagnosed with HSV-1 but its oral. She sometimes gets cold sores on her mouth. Is it possible for me to give her another infection of HSV-1 in her genitals if I m shedding in my genitals? thank you S Doctor: Hi..Welcome to HEALTHCARE MAGIC..I have gone through your query and can understand your concerns.As per your complain infection in genitals seems to be due to HSV 2 and in case if you have a breakout in your genitals I would suggest you to avoid intimate contact with you wife as you can very much transfer infection to her and she is susceptible to develop infection when she is already infected with oral herpes..I would also suggest you to consult an Infectious Medicine Specialist and get evaluated and he can do viral culture as well as blood culture to rule out the exact level of infection..He can start a course of oral and topical anti viral medications for you as well as your wife and can give oral doses of Acyclovir and topical application of antiviral ointments like Acyclovir, Valacyclovir etc for reducing viral count..Anti viral medication will reduce the frequency of recurrent infection as well as reduce the duration of prevent infection..Avoid sexual intimacy to prevent spread and severity of infection..Keep your personal use things like towels glasses etc separate..Always use washed and cleaned undergarments, towels, clothes etc..Hope this information helps..Thanks and regards.Dr.Honey Nandwani Arora."
},
{
"id": 95084,
"tgt": "Appendix operation. How soon can the patient travel?",
"src": "Patient: Hello DR My friend has just contacted me from Tunisia and his wife (my friend) has just been taken and kept in hospital and is due for an appendix operation at 0900 Tunisia time. They have a little child out there which as you can imagine is not easy. Could you tell me if she is able to travel home after the operation please or how many days she will have to wait? they are due to fly back on Sunday Doctor: Hi Welcome to HCM Appendicectomy is small procedure requiring not more than 3 day stay in hospital. Laparoscopic removal when done, it will have incision of about 1 cm. If non absorb-able sutures used it will be removed after week. You friend can travel after 2 to 3 days after surgery, if surgery is not associated with complication. Hope i have cleared your query. take care."
},
{
"id": 109172,
"tgt": "Suggest treatment for severe low back pain",
"src": "Patient: I have severe low back pain. My PcP has tested me for various diseases, all negative. But he diagnosed me with fybromyalgia....I cannot hold my stomach in and move/excercise without causing severe pain from a hemmoroids. Could this hemorrhoid be the reason for my low back/hip pain ? Samanda YYYY@YYYY Doctor: HelloWelcome hereI dont think that hemorrhoids can cause low back pain.Low back pain is usually due to disc prolapse. An MRI scan of lower back will help to find out the cause.Fibromyalgia can cause pain at multiple body sites. I would advice you to visit a physiotherapist for exercises to help you in back pain.Do hot compresses regularly. Keep your posture correct.Take pain kilker like tylenol when needed.Hope this helps.Thanks"
},
{
"id": 2959,
"tgt": "Is there any chance of getting pregnant after an unprotected sex?",
"src": "Patient: me and my gf were doing orgasm. my underwear and boxer was wet with sperm.. then i rubbed it on her vagina..she was also wet and undressed.. i touched my boxer and with that litle bit wet finger... i fingered her..so is there any chance of pregnency??? if so then before how many hour she can consume i pill Doctor: Hello and welcome to \u2018Ask A Doctor\u2019 service. I have reviewed your query and here is my advice. From your history it is clearly evident that you had just body play and had no insertional peno-vaginal sex. In non penetrative sex, there is no possibility of pregnancy. The semen has to be deposited in the vagina for conception. I-pill like post coital emergency contraceptive pills do give protection against pregnancy if taken within 72 hours of the unprotected sex. Any intercourse after the consumption of the pill is not protected.Hope I have answered your query. Let me know if I can assist you further.Regards, Dr. Nishikant Shrotri"
},
{
"id": 111549,
"tgt": "Is it muscular pain if back and chest hurts after falling on back?",
"src": "Patient: 45 yo male, 5'10\",170lbs., and healthy. Fell on my back 2 weeks ago, at which time the left lateral side of my chest hurt, midway up rib cage. Two days later I felt more pain in my back than my chest. Now both chest and back are mildly uncomfortable, with mild dull pain that feels like it's located deep, and becomes painful with exertion. It has not been too painful to prevent me from traveling or shoveling the driveway. Does this sound muscular, with rest being all that's needed? Or should I get checked out? Doctor: Hello,I had gone through the case and found that it might be muscular or ligament tear or hair line fracture of rib.So as for precaution go for X-ray of back and ribs.If report comes normal then take mild painkiller and local applicant of muscle relaxant gel and physiotherapy massage if pain not subside by itself.Hope my answer will be effective for you.Thanks"
},
{
"id": 56068,
"tgt": "Is taking prophylactic antibiotic cefepime and udiliv safe for congenital choledochal cyst?",
"src": "Patient: I have got 4mnths old baby girl who had congenital choledochal cyst for which she has been operated at the age of 34days following which she had 2 episodes of cholangitis. her doctoe suggested prophylactic antibiotic cefepime and Udiliv for a long time. her LFT is deranged and her liver is enlarged and firm. Could you please tell me how long we should give udiliv and antibiotic and wheather it is correct treatment or not. Thank you Dr.payel mukherjee Doctor: generally choledochal cysts treated by surgery implies cure of the same. when bile duct is connected to the bowel by surgery and it is draining well, there is no need for udiliv or prophylactic antibiotic. however when there is narrowing at the connected site, there can be fever and poor drainage which necessitates antibiotic and udiliv. I would advice you to be on regular follow up with your treating doctor to know when to stop these medications."
},
{
"id": 84533,
"tgt": "What are the side effects of taking norflox tz ?",
"src": "Patient: Hi, I just took a norflox tz tablet as I was having stomach ache, and I have severe itching over my penis. I think I have finally figured out the causal relationship between the two. Is there anything I can do right now to prevent the further stages of Fixed drug eruption etc. Doctor: HiKindly avoid further intake of the drug. Antihistamines and steroids can be taken in case of severe itching and eruption after physician approval.Inform your physician at every visit regarding your drug allergy history inorder to avoid its prescription.Hope that was helpful.Let me know if i can assist you further.RegardsDr.Saranya RamadossGeneral and Family Health Physician"
},
{
"id": 182659,
"tgt": "Suggest remedy for inflammation in the gums",
"src": "Patient: my right lower wisdom tooth has come out recently and a pocket has formed near that. The Tooth has fully erupted and a huge pocket has formed. When I brush, some blood used to come initially. Now a days I see pus and blood coming out from that pocket. Some pain and inflmation remains all the time. I have tried Betadine gargle for some time and also tried some antibiotics (amoxicillin - 5 day couse). Then the inflamation subsided but once i stop using those, the gum started flaring up again with the pus and blood. I have consulted a dentist and he suggested me to go for dental extraction. The teeth is healthy and fine. So I do not want to go for an extraction. So can you please suggest me any other alternative for this problem. I am 26/Male. No dental extractions done till now and no chronic disease history. Doctor: the dentist suggested you the perfect advice. you can always go for temporary treatments like betadine or hexidine mouth wash, worm saline water mouth rinses 8-10 times a day. regular visit to a dentist for cleaning of that pocket.but better once in a life time go for extraction of that wisdom tooth"
},
{
"id": 137802,
"tgt": "What causes bulging disc?",
"src": "Patient: I was diagnosed with a bulging disk at the L-3, L-4. I had pain in my back and pain that went down my right leg on the inside part of the leg NOT the back of the leg down to the knee. No pain below the knee. I am much better. Did my bulging disk receed back into place? Doctor: Hi I did review your concern. Herniating or bulging disk is a disease of unknown cause that causes the disk pulp to force through surrounding tissue and press on nerve roots as they come out of the spinal cord.It causes the severe pain like you are having and depends on which nerve roots are being irritated.It precipitates with increase pressure on vertebrae and with rest and relief of pressure, the disk pulp moves about and stops irritating causing relief of symptoms.It does not mean you cant have it again, once it happens, you are predisposed to it and it can happen again.I hope this helpsLet me know if you have any more questions or concerns.Wish you all the best."
},
{
"id": 184909,
"tgt": "Suggest remedy for yellow hew on tongue",
"src": "Patient: Hi doctor(s), i've been diagnosed thru surgery having Chronic Hepatitis Auto Immune Disease. I've been taking prednisone for about 8 years now learning how much to take accordingly from my doctor and has been under control for these years.... What I have now for about a week is a yellow hew on my tongue. I had this about 4 weeks ago and went away.. I check my eyes on a daily basis with them being normal. I live in a very hot with high humidity climate during the summer months, I was wondering if this could be from the heat or are the chances of my disease not under control any more... I personally feel fine etc. What do you think ? Doctor: Hello,I have read your inquiry. I recommend you first brush the tongue and make sure this is not a coating. Rinse with warm salt water. Second, I recommend due to the climate you are living in, that you make sure you are very well hydrated. A dry mouth is very stressful on all oral tissues. You are well balanced with vitamins and no deficiencies? Consider updating your blood test results if you have not had lab work done in a while. Autoimmune changes to your blood and muscle tissue are a concern. Is this change unique to your tongue?Thank you for contacting Heallthcare Magic. I am glad to answer additional questions if you provide a more detailed description of your tongue condition. I recommend following up with your regular physician as well as visiting your dentist for a routine exam and cleaning to observe the tongue."
},
{
"id": 53801,
"tgt": "Suggest treatment for hepatitis B and its effects on future",
"src": "Patient: Dear Sir, Iam a patient of hepatitis B---Positive, 25 years of age,my viral load is only 655.4 Copies/ml. but HBe Ag- Negative, and my chronic infection is also non-reactive. my doctor advised me to take a every check up of 6 months once. Now my queries is how to clear these infection, any specific doctors in chennai. i need to solve this 100 %. Can i able to marry. whether it will affect my future wife. when it will solve , Doctor: hi.thanks for posting query at HCM.you may marry but be sure that your spouse has been immunized against HBV before having sexual relationship.HBV can not be treated completely but complications and transmission can be prevented. Liver enzymes and viral DNA should be examined regularly. secondly, avoid sexual relationship if HBsAg and/or HBeAg is high in your blood and/or high viral DNA copies. hope to answer your concernregards Dr Tayyab Malik"
},
{
"id": 130311,
"tgt": "Why does my left side ache?",
"src": "Patient: In may i fell backwards from a tube door down to the floor on my left side. Since then I have had shoulder and upper back pain n my left side. Been taking NSAIDs and general analgesics, but even slight exertions like typing, washing dishings, or even gripping something from left wrist makes it ache. what is the problem ? is there a permemnant damage ==> please address. Doctor: Hello Thanks for consulting Healthcaremagic. I have read your query and understand your concern . As far as left side bodyache is concerned for this i will suggest you to consult Physician for examination and if needed go for investigation like radiograph . Inmeantime you can do hot formentation and you can apply ointment containing diclofenac gel localy on effected areas . Hope this will help you ."
},
{
"id": 8837,
"tgt": "How can I get rid of blemishes on my face ?",
"src": "Patient: i am suffering from blimshes on my face .would u suggest me something to get rid of them.i am 30 yr old getting newly married.i got this blimshes problem in the age of 27 after a few acne on my face then i got allopathy treatment now i suffered from blimshes Doctor: if u r not gatting responded with allopathic medicines,u can withdraw nad start natutal therapy.like allovera, etc.for more detail u can contact at a_k227@yahoo.com"
},
{
"id": 184195,
"tgt": "What causes needle like sting inside front tooth?",
"src": "Patient: I was woken up by an needle like sting inside my left front tooth. It is verry irritating and I have trouble going back to sleep. I tried brushing them, I used mouth rinse and flossed to see if there was maybe just something in there, but it wont go away. What can I do? Doctor: Thanks for using Health Care magic.Read your query.The symptoms that you have mentioned suggests decay in the tooth and the shooting pain is caused by this.(pulpitis)I would advice you to take ibuprofen (if you are not allergic to any medicine) for instant relief of the pain.Please visit your local dentist and have a radiograph done to evaluate the tooth.Get the tooth restored .Hope this was useful.Thanks and regards."
},
{
"id": 217420,
"tgt": "How can pain be managed throughout stomach and chest?",
"src": "Patient: Hi I have been having chest pain for over five years now. I have had many ekg's, had 2 stress tests. I'm 38 years old. I have no gallbladder. I also get severe abdominal pain with this and it radiates into my upper back on left. I have been to the drs so many times and seems like no one can figure it out. Doctor: dear friend,you can take rabeprazole/pantoprazole with simple pain killer like paracetamol or combination with ibuprofen for temporary relief.you are facing this problem from more than 5 years so i suggest you for yoga, exercise-physiotherapy and homeopathic line of treatment. if you are interested in homeopathy it will be much helpful to you.so plz consult a good homeopath nearer to you or in your city.hope this answer will be helpful to you.for more queries plz don't hassitate to ask.get well soon"
},
{
"id": 215281,
"tgt": "What causes pain in right ribs and high blood count?",
"src": "Patient: i had doctors say my gallbladder needed to be taking out over 5 days in the hospital they changed their mind was on antibotics got better for a short while went back to doctor had left arm muscle pain in my lower right rib cage around to my back white count was elevated to 12 my right arm feels very strained hurt in my armpit it almost feels like infection moves around up my back an into my neck I have high blood count at least once a month have had a back fusion my right arm seems to be getting weaker run low grade have been hospitialized 3 times in 6 months very concerned plz try an help Doctor: Hi, Cannot say in your particular case, and if concerned that would be a reason to be examined and treated. Reasons to increase concern is that high blood count is an indication of overwhelming and/or spreading infection. It is doubtful that it can spread directly to gall bladder, arm, back and ribs without a lot of obvious problems BUT, infection going out from the gall bladder to the surroundings could irritate the diaphram and this is often felt widely. There would also often be increased pain with deep cough. It's serious. Hope I have answered your question. Let me know if I can assist you further. Regards, Dr. Matt Wachsman, Addiction Medicine Specialist"
},
{
"id": 56477,
"tgt": "How to treat calcified granuloma in right lobe of liver?",
"src": "Patient: I got a sonography done on 2nd june, 2014 which showed calcified granuloma in right lobe of liver, but rest everything showed normal. Do I have to worry as my urologist stated that I am absolutely fine and normal. Just wanted to ask whether I have to worry because of calcified granuloma in right lobe of liver. Thanks in advance. Doctor: Hello You are rightly advised by your doctor.You are perfectly fine.Calcified granuloma in liver are healed stage of infection and calcification indicate that there is no activity in the lesion.Calcified granuloma in liver may be due to healed abscess etc.Infection naturally heals by calcification and fibrosis.Mostly it doesn't require any treatment.Get well soon.Take careDr.Indu Bhushan"
},
{
"id": 189226,
"tgt": "Had root canal done. Removed tooth after severe pain. Noticed slight pain. How long will the pain exist?",
"src": "Patient: hi,i ahve done root canal for my left upper firsat molar 2 mtnhs agao as i was having sever pain, and the doctor said it was infected.then he fixed cap for the same. but the pain didnt subsidzed. i was on contrionus pain killers.then he removed teh cap and he called a endodontist. he also did soemthign on my tooth. but yet pain was not going. one day it will not be tehre next day it will cumat last teh doctor decided w ell remove the tooth. and he extracted the same.1 week wound pain was there and after that it was better. i was not eatign that side.but these days i do slight sensitivity and pain over there.i feel pain over the next tooth also. but whne took xray he said ther eis no infection in any other tooth.after extraction how many days the pain will be there. Doctor: Hello there , Thanks for writing your query, Extraction of tooth often ends with complications like pain and swelling post extraction . usually it takes 7 to 10 days for the extraction wound to heal and close completely provided if there is no infection . After extraction antibiotic and analgesic coverage is must to prevent infection of the extraction site and to promote healing. i would suggest you to start with a antibiotic and analgesic course consulting your dentist , which helps in subsiding the swelling and relives post extraction pain. follow all the post extraction instructions and rinse the mouth with lukewarm saline or antiseptic mouth rinse. i hope this helps , take care."
},
{
"id": 144779,
"tgt": "What causes headaches with blurred vision and weight loss?",
"src": "Patient: Hi, I have severe headaches but I do have epilepsy. I have blurred vision and feeling hot and tired all the time. I am 44 and even though my hormone level was only 4 my G.P. hasn t ruled out that I could be peri menopausal. recently I have lost weight for no apparent reason, and even though I do have hypothyroidism too, do you think with my symptoms that I should ask for another blood test? My last test was normal after a dose change. Doctor: Hi there.This could be related to hot flashes during peri-menopausal period or also Migraine headaches. Avoid sunlight exposure, pollution, loud noise and stress. Loss of weight can be due to HYPERthyroidism and needs to be tested as this can also cause Headaches.Take pain killers and apply hot bag to the painful region. Eat healthy and keep fit."
},
{
"id": 97923,
"tgt": "Suffering from on and off cough. Took prednisolone and mucosolvan. Cough started again. Any alternate medicine?",
"src": "Patient: I m suffering from cough that is off and on . I was prescribe factive prednisolone and mucosolvan I just finished my medication last April 18 2012 but then my cough started again last April 27 and till now I m not taking any medicine because I m already scared for any side effect of taking too many medicine please advice Doctor: 1. Cough was dry or productive by which we can get into the pathology of either upper or lower Respiratory tract infection. 2. It could be occur due to post nasal drip (sinusitis/rhinitis), gastro esophageal reflux disease (GERD), or Bronchial Asthma, or side effect of blood pressure medication? 3. history of night sweats,weight loss,fatigue, chest pain on breathing, abnormal breath sounds,or of any addiction? 4. was chest x-ray done by the attending physician? 5. Seek Chest Specialist professional help who will have an insight into your problem."
},
{
"id": 163079,
"tgt": "What causes vomiting sensation and stomach pain in children?",
"src": "Patient: Hi, may I answer your health queries right now ? Please type your query here...my son is 22 months past few weeks he is suffering from vomitting and specially whenver he eats non-veg at lunch he is uncomfortable at nights he feels vomiting sensation and stomach pain . Doctor: Hello and Welcome to \u2018Ask A Doctor\u2019 service.I have reviewed your query and here is my advice.Causes of vomiting sensation and stomach pain in children are stomach infection, gastritis, outside feeding. You can give syrup ondansetron, syrup cefixime as per his weight. Maintain proper hygiene. Give clean water and food. Kindly consult with a pediatrician.Hope I have answered your query. Let me know if I can assist you further.Regards, Dr. Shyam B. Kale"
},
{
"id": 220520,
"tgt": "What is the best treatment to get pregnant?",
"src": "Patient: We got a baby after 3 abortions of 2 weeks each. Unfortunately that baby had pulmonory venous stenosis. He was operated on at Narayana Hrudayalaya Bangalore successfully. Unfortunately we lost him in the very next month. He was 10 months old. Now my question: we are having a big question mark in front of us on the status of our next pregnancy. The very though of that problem which our baby is pulling us back from next pregnancy. We are longing for a baby. Please advice us. What should be done? What are the precuations to be taken? What tests to be done? Doctor: Hello,I have gone through your query and very sorry for what had happened. In the current scenario, you need to have proper prenatal counselling before attempting another pregnancy. You should see a specialist in high risk pregnancy to help you. If you happen to stay in Bengaluru, you can approach Dr. Kamini Rao to help you. Several factors like consanguinity, blood group incompatibility, genetic abnormalities, infections, heredity etc., should be evaluated. Proper preparation before attempting another pregnancy gives better results. Hope you find this information useful. Take care."
},
{
"id": 64132,
"tgt": "What is black, pea sized bulge on tongue which burst causing bleeding?",
"src": "Patient: Hi! My mom is 53 years old and a few days ago she was eating dinner and suddenly had a prickling feeling on her tongue. When we checked she had a bulge that was black in colour and about the size of a pea. After 15 minutes or so it had tripled in size and then it burst causing a lot of bleeding. Could it be a burst blood vessel? Doctor: Hi,Dear,Dont get hopeless and desperate.Thanks for the query to HCM.I went through your query in-depth and Understood your health concerns.In my opinion the cause-- of your mothers bleed from black pea sized bulge is as follows-a-Sudden noticing of the tumour mostly -leads me to label the bleeding cause -is because of the--BITE-TRAUMA -which lead to bite HEMATOMA -of black colour.-As I am short of more clinical data,I would alert YOU for the CAUTION-of such a complaint-as I would advise Surgeons opinion-on toungue who would fix the diagnosis by clinical and FNAC Biopsy for any-Suspicious growth or the un-noticed-TOUNGUE Ulcer, which would cause such a event of bleeding at Dinner time as your mother had.-It could be a benign tumour-AV malformation HAMARTOMA-Hemangioma unnoticed and now found after bleed.Advisea-Consult ER-Surgeon who would according to the causal factor diagnosed after proper investigations as suggested above.Hope this would help you to work a plan with your doctor.Wishing you early and fast recovery.Wellcome to my HCM Clinic for further queries in this matter.Wishing you Good Health ASAP.Have a Good Day .Regards,Dr.Savaskar ,M.S.Genl-Thorasic CVTSSuper specialist in Asthma/Cancer and non-curable diseases."
},
{
"id": 172697,
"tgt": "What is the cause for seizure with swollen lips?",
"src": "Patient: my nephew is in ghana , he was having a siezure with swollon lips and was very white, he was shaken on controlably had no temp, so what was going on with him he is 6yrs old he has no history of siezures i wonder what will triger that episode pls get back to me am very worried about the kid. thanks lily Doctor: HiWelcome to the HCMI completely understand your concerns. It seems that your nephew had first episode of generalised seizure. It's very important to go for electroencephalogram (EEG) to check the reason for seizures. Also, a family history of seizures, certain drugs and electrolyte imbalances, perinatal risk factors can predispose a child to seizure disorder or epilepsy.I would recommend you to take him to a paediatric neurologist for complete physical examination and lab evaluation. He may even start certain anti epileptic drugs to prevent any further such episodes.Hopefully this will help you. I would be glad to help you with any future health related question.Take care"
},
{
"id": 8086,
"tgt": "Dry, rough, scaly and flaky skin under armpit and other parts",
"src": "Patient: dry, rough, scaly and flaky skin under armpit and other parts of the body hi i got this dry, rough, scaly and flaky skin under my armpit and other parts of the body, i ve been suffering for this for quite a long time already like more or less 5 years. i have already consulted a doctor but they cannot give accurate answers, they re just giving some oitment and etc. this rashes thing in my bosy sometimes are reddish specially when i sweat. please if anyone here can give me some answers i ll be happy to read it. i am 26 years old, male. Doctor: Hi!, Welcome to HealthcareMagic forum, It could either be psoriasis or a fungal infection or haieli haieli disease .You need to consult a good dermatologist for proper diagnosis.Kindly also specify about any symptoms associated with it...and exact sites involved. take care, Dr.chawda"
},
{
"id": 2957,
"tgt": "Will I be able to conceive after vaginal infection?",
"src": "Patient: hEllo doctor, i m tryin to concieve for 1 year 6 months. after one year of tryin, checked with gneacologist and found a polyp. then done polyp removal 5 months before.. now i got vaginal infection(yeast infection) and took fas3kit on 23rd day.. wil it affect in process of conceiving? Doctor: Hello and welcome to \u2018Ask A Doctor\u2019 service. I have reviewed your query and here is my advice. Yes, yeast or any other infection in the vagina does affect the fertility by creating a hostile environment for the sperms. However, once the infection is treated successfully, the environment regains its normalcy and thus fertility can be re-achieved. For yeast infection, please take some anti-fungal vaginal or systemic treatment. Along with it, use Lactobacilli vaginal ovules for at least 15 days and Lactobacilli perineal wash too. This infection can spread sexually also. Hence, irrespective of the symptoms to your husband, both of you should take the simultaneous treatment. I would advise both of you to follow the undermentioned tips for successful anti-fungal treatment: 1. Till both of you are declared completely cured, observe complete sexual abstinence. 2. Shave the private parts clean. 3. Clip the nails short - flush to the skin.4. Boil your underwear daily. Use panty liners. With the successful treatment for the fungal infection you can definitely conceive provided:1. You are forming the eggs regularly: you may ensure this by ultrasonography ovulation monitoring. 2. Your tubes are patent and there is no pathology in the uterine cavity : Hysterosalpingography, Laparoscopy and Hysteroscopy can provide the information about these factors. 3. Husband has sufficient healthy sperms: Semen examination will give the information. Please report to your gynaecologist for these investigations, treatment and further advice. Hope I have answered your query. Let me know if I can assist you further.Regards,Dr. Nishikant Shrotri"
},
{
"id": 212638,
"tgt": "Depression, weight loss, have knee replaced. On metformin and lesinopril. Suggest?",
"src": "Patient: My husband suffers from depression . He was injured at work and we had to deal with Worker s Comp. for 22 months. He had to have total knee replacement and the surgeon has told him he won t be able to return to work. He has applied for dissability but because he s only 47, he was turned down. We have a lawyer and feel he s doing a good job. My husband feels like he should be taking care of his family and of course, staying inside all day certainly doesn t help. Our doctor has tried him on Celexa (29 mg) and Effexor (150 mg). He had 1 refill left on the Effexor so he started taking it again. With all the medicines out there, we don t know which might be best for him. He has an appointment next week with anothe doctor to get a second opinion. He has been on Metformin and Lesinopril but has stopped those since his weight loss from close to 270 down to 217. He s such a wonderful person and am not giving up on getting him help. He just gets very frustrated trying different meds. Any advice you can give will be greatly appreciated! Thanks in advance. I m sorry I can t pay this since I am the only one working. But thank you anyway. Doctor: Hi there ~ I understand that not being able to work can be handicapping in a lot of ways for a lot of people. In your husband's case, I am sorry that his disability petition was turned down. If effexor had helped him before, it would be prudent to go back on that medication until he sees a doctor. However please watch for any side effects, most commonly gastrointestinal (nausea and vomiting) and increase in blood pressure. Celexa is also an effective adjuvant medication for depression. I am glad that he is working on making his day better. I hope he is eating at regular times and sleeping well, not too much not too little. Activity makes depression better, so the more he does with his hands the better he would be with his depression. I would not bother too much about the weight loss. Please wait for the doctor's appointment to see what the doctor says. I hope this helps. Take care and have a nice day !"
},
{
"id": 75291,
"tgt": "What causes deformity of ribs ?",
"src": "Patient: hi-one of my ribs (the top part on my left side) sticks out loads and looks twisted/deformed. My back is perfectly straight and my ribs on the right are completely normal. I'm not sure if it's the top part of the left side sticking out or that the bottom part has sunken in. Help!!! Doctor: HelloThank you for asking in HCM I have a question for you. Does this deformation of the ribs hurt to you? Have you had an accident or a trauma? If no of these factors you should do a chest X Ray to see better the ribs in both sides of the thoracic cage.Thank you Dr Jolanda Pulmonologist"
},
{
"id": 217544,
"tgt": "What is the cause for severe pain in the left temple?",
"src": "Patient: I have a severe pain in my left temple. It feels like a headache, but it's the worst I've ever felt. It's almost completely isolated to my left temple. I start to sweat when it hurts the worst. It's been happening for about 3 days now. What could this be? Doctor: HiI understand your concern.You need to investigate yourself for differential diagnosis.If it is associated with blurring of vision/difficulty in walking/giddiness/nausea and vomiting etc then go for investigation.1st thing is require is fundas examination.It will shows any papillodeama that suggest raise in interracial tension.MRI brain will helpful to rule out any pressure producing lesion or space occupying lesions.Analgesic like paracetamol help to reduce pain.If all report come normal then we have to think about migraine associated with tension type headache.But go for reports 1st then consult neurologist if require.Still have query then feel free to ask.Thank you.get well soon."
},
{
"id": 13902,
"tgt": "Suggest medication for skin rash in the armpit area",
"src": "Patient: Hello, I m a healthy 36 year old female with no known allergies or asthma. I m med free. However, twice within this month I ve broken out into a rash from under my left armpit up my chest that stops at my neckline. It runs around my shoulder blades. I ve noticed my left armpit had a purple like bruise each time. The rash is red, moderately itchy and a little swollen. It lasts for two days. I haven t introduced a new product into my diet or on my skin... Doctor: HiThe rash could be a contact dermatitis or erythrasma. It should be properly examined by a Dermatologist to come to a proper diagnosis. So, I would recommend you to visit your Dermatologist.Hope I have answered your query. Let me know if I can assist you further."
},
{
"id": 88714,
"tgt": "Could the pain on right of the abdomen be due to H pylori?",
"src": "Patient: I have H Pylori and am currently on a special diet to get rid of it, which I am nearly finished with. The main symptom for me is abdominal pain especially on the right side at night. I think the diet is helping, but I am still in pain at night, though not as bad. Could the pain be caused by something else? Doctor: Hi.The pain in the right side of the abdomen at night with H pylori can be due to a Stomach Ulcer. But there are other differential diagnosis such as gall bladder stone, Cholecystitis, liver inflammation, pancreatic problem, right kidney infection and stone and so on. A detailed history, physical examination by a General Surgeon, ultrasonography and relevant blood and urine tests as per the suspicion, upper GI Endoscopy can help to get a proper diagnosis and treatment."
},
{
"id": 25704,
"tgt": "Is it serious if having chest tightness, sudden chills, pain in left shoulder, heart racing?",
"src": "Patient: I feel a quick tightening in the middle of my chest and now I got a \"chilly\" feeling through my body, My chest stings, it does not \"hurt\" just stings, my left shoulder aches. I feel cold from head to toe. Now my heart beats quicker for like 2-3 beats and returns to normal. I have a history of anxiety, depression, I am type 2 diabetes. My total cholesterol is 102. I am 5 ft 6 inches tall 324 pounds, and I smoke approx 8-10 cigarettes a day. I am on Metformin, gemfibrizol, and rantadine, along with a birth control pill because I have disfunctional uterine bleeding. Should I visit the er? Doctor: Thanks for your question on Health Care Magic. I can understand your concern. Yes, you should definitely visit emergency room and rule out heart diseases. You have following risk factors for heart diseases. 1. Smoking 2. Diabetes 3. Obesity 4. Birth control pills. And your symptoms are more in favour of heart diseases like hypertension, arrhythmia, coronary artery disease. So immediately consult emergency department and get done blood pressure monitoring, ecg, 2d echo and coronary angiography if required. Strict control of diabetes and weight reduction are needed. And quit smoking as soon as possible. Don't wait at home, rule out heart diseases because without treatment these can be life threatening. Hope I have solved your query. I will be happy to help you further. Wish you good health. Thanks."
},
{
"id": 46228,
"tgt": "Suggest remedy for kidney stones",
"src": "Patient: hi..i am rinki and my age is 21 years old ..i have stones in my both kidneys..around 3 months before , i have taken 1 vovern injection in right buttock.its 3 moths spend bt still its paining alot and whenever i walk run its pain like i can never bear it out..plzz give ur suggestion.. Doctor: Hi and welcome to HCM. Thank you for your queryMost small stones in patients can be treated with observation and acetaminophen or other painkillers. More serious cases with intractable pain may require drainage with a stent or percutaneous nephrostomy. In cases of emergency, treatment of nephrolithiasis involves management of renal (ureteral) colic, including surgical interventions where indicated, and medical therapy for stone disease. In emergency settings where concern exists about possible renal failure, the focus of treatment should be on correcting dehydration, treating urinary infections, preventing scarring and identifying patients with a solitary functional kidney.Kindly regards. Wish you a good health."
},
{
"id": 16921,
"tgt": "Can one with reversible ischemia and CAD lead a normal life?",
"src": "Patient: I am suffering from CAD(TVD) and undergone ptca+stent to RCA EF is 45%. Jut now I have done TMT test and is shown mildly positive for reversible ischemia. Can I live a long life as I have not any problem or chest pain etc. I am doing my work easily and having sound sleep. Thanks , Tandon Doctor: Hello There After going through your medical query I understand your concern and I would like to tell you that positive TMT test means you might require to undergo Angiography again, you can live a normal life and need to consistent with Medication and regular follow ups.Hopefully this information will guide you properly.Kind Regards Dr Bhanu Partap"
},
{
"id": 186080,
"tgt": "What is the white spot on my son's gum?",
"src": "Patient: My 7 month old son has had a fever of 102.7 for about 3 days and it's finally going back to normal, hea been sneezing and has a runny nose. Today I noticed a white spot on his gums and when I touched it to see if it was a tooth it moved. It was attached to the gums. What is that? Doctor: Thanks for posting your query to HCM.The teeth is erupting out from the gums which is normal during the eruption procedure. So, you need not worry about it.Also, fever and all do occur during eruption.Hope my answer will help you.RegardsDr. Himesh"
},
{
"id": 150716,
"tgt": "Signs of confusion and dementia after radiotherapy for acromegly. What's the mortality rate ?",
"src": "Patient: I have a sister who had acromegly, which was undiagnosed for many years. She eventually had an operation to remove the tumour. And, had radiotherapy as all the tumour had not been removed. Over the last couple of years she has slowed down a lot and, has shown signs of confusion and what seems to be dementia . She is on lots of medication. Do we know the prognosis and mortality rate for my sister. She is 68 now and, it s about 20 years ago since acromegly was discovered. Concerned sister AAAA Doctor: Hi, Thank you for posting your query. The prognosis of a benign tumor of pituitary is quite good. So, if you have gone through the biopsy report of the tumor, and it is benign, then the chances of long term survival is quite good. There could be mild decline in her memory and cognitive functions due to her tumor and radiotherapy. However, we should investigate whether she has any other reasons for the same. This would include checking for vitamin B12 deficiency, thyroid dysfunction, etc. Proper treatment of her dementia would help her recover to an extent. Best wishes, Dr Sudhir Kumar MD DM (Neurology) Apollo Hospitals, Hyderabad"
},
{
"id": 86441,
"tgt": "Suggest treatment for terrible cramping in stomach and back",
"src": "Patient: In the last month I have been constantly nauseous. I have to make myself eat. I have lost 25 lbs. It all started one day when I was walking around having a good time at a casino (no alcohol). I had a terrible cramping feeling on my right side under my rib that went all the way through my back. I broke out into a sweat. This lasted about 30 minutes. I was weak and just felt like laying down. I had eggs and sausage and pancakes for breakfast that morning. I went to the restroom, and urinated an odd orangish color. I let it go until about a week later I noticed that i was passing undigested food through my bowels and having nausea. I felt terrible like I was going to die. I then went to the E.R. 3 times in 4 days. No answers. Now my pain is pretty much constant under the right rib. If I take a pain killer like hydrocodone it actually makes the area cramp more. So.....at one of my E.R. visits they checked my urine, and it showed high bilirubin level. A blood test did not show high bilirubin. I have belching (not normal for me), diarrhea, tongue coating is usually white or yellow, passing undigested foods, especially items like beans, yellow stool color, nausea, and bloating. I also have tenderness to the touch under the right rib area. Tests I have had.....CT no dye, upper abdominal ultrasound, Hida Scan with CCK, blood tests, urine tests. The only thing that ever showed anything it shouldn t was the urine have bilirubin. The rest of the tests are showing normal. The gallbladder hida scan showed ejection fraction of 59%. How am I having all the symptoms of gallbladder problems, and the tests aren t showing it. What does this sound like to you? Please give me a suggestion as to what else it may be, or other tests you feel would be needed to find the problem. Doctor: Hi.Thanks for your query. Read and understood your history and query. You have nausea for 1 month, a long time to keep you from a proper diagnosis; lost weight, have cramping in the right side under the rib, that is referred to the back, sweating, orange urine in the beginning. This followed after 1 week with undigested food in the stools and nausea, 3 ER visits, Hydrocodone increases pain, Urine shows raised bilirubin, coated tongue, all other tests are normal,.All the history together shows that there may be a problem of hepatitis or cholangitis or Sphincter of Oddi dysfunction. Sometimes you need to undergo repeat tests as sometimes the tests may not show the problem. I would advise the following:Get the full liver profile including GGT, this may give a clue. Therapeutic trial with an antibiotic and metronidazole may help to get well. Avoid food rich in fat and oil. Get an opinion of hepatobiliary Specialist."
},
{
"id": 218809,
"tgt": "Are delayed periods a sign of pregnancy?",
"src": "Patient: My boyfriend ejaculated on his stomach while I was lying down next to him with my skirt down with no underwear on....I was lying on my side so my legs were closed. After he came he did not touch me at all and we got up and he went to clean himself up. Last to last month my periods started on 21st then last month they started on 15th or 16th and I thought it was maybe because I had taken 3-4 different medicine at that time (for my migraine). Usually my periods start 3-4 days early but now it's 14th and my periods have not come. Could I be pregnant? I'mreally worried. Doctor: no chances of conceiving are negligible as intercourse did not happen. periods may shift sometimes due to hormonal changes. keep a check fr another week and then consult a doctor"
},
{
"id": 183215,
"tgt": "What causes swelling in the tongue?",
"src": "Patient: Hello, my daughter, who is otherwise healthy, has developed, in the past two to three days, unexplained swelling of her tongue. The only medications she takes are multivitamins and Lo-estrin birth control pills. She is under the impression the swelling may be due to a hormone imbalance.. What do you think? Doctor: Thanks for your query, i have gone through your query.The cause for the unexplained enlargement of the tongue could be allergic origin. Allergic reaction secondary to any drugs or food items might have resulted in enlargement of the tongue. Consult a oral physician and get it evaluated. Meanwhile, you can take anti allergic medications like cetrizine morning and night for one day. I hope my answer will help you, take care."
},
{
"id": 185456,
"tgt": "What causes sore inside the mouth with white patches on tongue?",
"src": "Patient: I have had a sensitive mouth on and off for about 8 wks. I have a sore on the inside of my right cheek (most likely a canker sore. I chew on that side and so salty and spicy food is irritating. The side and underside of my tongue has had some white patches and at the moment the (R) topside of my tongue feels like I ate something that was too hot (burned). Any ideas what I am dealing with. Doctor: Hello, thank you for consulting with healthcaremagic. The type of symptoms you are mentioning that you have sore cheek with white patches on lower side of tongue, it looks that you might be having an autoimmune disease known as lichen planus.Better that once you should visit an oral medicine specialist and get the necessary investigations done.Hope it will help you."
},
{
"id": 16586,
"tgt": "What causes burning pain in left upper chest and shoulder?",
"src": "Patient: I have a burning pain in my left upper chest (upper left of sternum) and shoulder area. Sometimes t radiates to my left back and arm. I also have Shortness of breath. EKG and Stress test came back fine. Blood work shows ok. At times it feels like fire is running up the sides of my neck in my veins also. Any suggestions. Doctor: Hello, I would explain that it is important to perform comprehensive differential diagnosis between different causes that may mimic this clinical situation: - gastro-esophageal reflux - gallbladder stones - pancreatitis - chronic degeneration of the cervical spine (a bulging disc in this level). For this reason, I would recommend performing some tests: - a fibrogastroscopy - an abdominal ultrasound - a cervical spine X-ray study - liver function tests - amylase and lipase plasma levels. You should discuss with your doctor on the above tests. Hope I have answered your query. Let me know if I can assist you further. Take care Regards, Dr Ilir Sharka, Cardiologist"
},
{
"id": 63708,
"tgt": "What causes thigh and foot lumps with itching?",
"src": "Patient: My son had itchy welts on one of his outer thighs and they went away. He also got them on the other thigh on a different day. He also said he had a wierd sensation on both of his hands. He then said that he had a lump on the bottom of his foot that he could not walk on it but that went away too. Could you explain any of this? Doctor: Hi,Dear,Thanks for the query to HCM. I studied your problem in depth and I understood your concerns.Causes-In my opinion your lump onthe foot /hands / thigh are due to Acute on Chronic Idiopathic Urticaria, and needs ER Dermatologist consultation and treatment.Many cause for CIU needs to be ruled out and treated accordingly.Your son has very sensitive and labile immune system currently due to many factors and causes inside the body ,which needs to be studied and treated by ER Dermatologist accordingly.So you need to take treatment under cover of a ER Dermatologits and don't self medicate,as your case has become complicated.So act fast and get ER Dermatologits consultation and intervention URGTLy.If you dont want to remove as you are afraid of its pain, then Show to ER Dermatologits if it grows and worries more.So don't build up wrong concepts and create more psychic complications in you which would increase risks and costs to you, but just ask a query to HCM and be comfortable to resolve your health issues.Welcome for any more query in this regard to HCM.Write good resume and Click thanks if you feel satisfied with my advise.Have a Good Day.Dr.Savaskar M.N."
},
{
"id": 14174,
"tgt": "What causes little bumps on shaft, inner thigh and elbows?",
"src": "Patient: rash which started at base of crotch as a couple bumps and is now spread in that area, on shaft as well (just red, puffy and pinkish), and a bit on my inner thigh (where it is red and pinkish more than anything with no dots or anything and now looks like it is starting on the inside fold of my elbow but as little bumps. Doctor: Hi.As per your case history you are having fungal infection called as tinea corporis.My treatment advice is \u2013 1. Maintain good hygiene and bath twice daily.2. Apply an antifungal cream like clotrimazole cream twice daily on it.3. Take an antihistamine like levocetirizine for 7-10days .4. Other treatment options are oral fluconazole, itraconazole and terbinafine given only after consulting a dermatologist.Thanks.Dr.Harshit Bhachech.MBBS, DDVL."
},
{
"id": 146872,
"tgt": "Suggest treatment for scoliosis",
"src": "Patient: I am 14 years old and I have scoliosis. I was braced a little over a year ago. It has ruined my life. I dont hang out with my friends outside of school anymore. I cry alot. if i am not wearing my brace i am really happy. but the second my mom or dad asks me to put it on, I get really mad and I go to my room and cry. My doctor says that I have until about January until im out of the brace. but no one knows for sure. I have been suicidal a couple times. Never tried but I have thought about it a few times. Help? Doctor: Hi, welcome to our site. I am Dr Saumya Mittal, MD.Read your query. That is a very significant question and i appreciate your problem. I will try my best to answer your queryYou really need to be patient son. You see this condition that you have developed is not a very comfortable disease. I have seen enough number of patients with scoliosis to understand your discomfort.But you really need to be very brave in your treatment. Courage is the essence of treatment. Do not loose hope. I am sure you have already searched the net about the possibilities, and I am also sure you have understood that the main solution is surgery if everything else fails. And I am sure you know that is not a good option.Frankly, I think you are ashamed of your brace, dont be. Ok if its there. Flaunt it. Ya I know it doesnt look good. But if your friends are exactly that- friends, they are going to be very supportive in your condition. You need not hide from them. You have to accept the condition yourself first buddy. Thats absolutely essential. I hope your parents know about the suicidal thoughts. Cause they are the best people who can help you short of a psychiatrist prescribing medicines.I hope this helps you. Inform the reports mentioned above/if any other so I can be of help further. I have given you the answer to the maximum considering the information provided. The results of the tests could further enhance my answer to you.Please do understand that some details could be extracted from a detailed history and examination.Please feel free to ask another query. I would be glad to help you. Looking forward to your return query with the details asked so that I can help you further. (If the answer has helped you, please indicate this)Best of luck.Dr Mittal.MBBS, MD (Internal Medicine), CC (Diabetes Mellitus), DNB (Neurology)Consultant Physician and DiabetologistJS HospitalEx Apollo Hospital, DelhiEx Kailash Hospital, Noida"
},
{
"id": 77205,
"tgt": "Suggest treatment for hard breaths & chest pain",
"src": "Patient: My daughter is 12 and said she is having hard time taking deep breaths and exhailing, said pain started at her left chest then moved to behind her left shoulder blade, now thats where it stays, I have seen online lots of diagnosis of pleursey or phenomia (spl?). Doctor: Thanks for your question on Healthcare Magic. I can understand your concern. Chest pain with breathing difficulty are commonly seen with pleurisy (inflammation of pleura). Common causes for pleurisy are lung infection, bronchitis etc. So better to consult pulmonologist for her and get done 1. Clinical examination of respiratory system 2. Chest x ray 3. PFT (pulmonary function test). Chest x ray is needed to rule out lung infection. PFT is needed for the diagnosis of bronchitis. She may need antibiotic, inhaled bronchodilator and inhaled corticosteroids (ICS). Don't worry, with proper treatment, she will be alright. First diagnose her and then start appropriate treatment. Hope I have solved your query. I will be happy to help you further. Wishing good health to your daughter. Thanks."
},
{
"id": 88656,
"tgt": "What causes severe abdominal and pelvic pain?",
"src": "Patient: I am having severe abdominal pain, flank pain, pelvic pain, nausea, vomiting, diarrhea, constipation, lower back pain, pain in middle of the back, pain in upper-center area under the breastbone, pain when urinating, feeling of needing to urinate frequently, feeling of full bladder, intense pain in pelvic and bladder area, left side lower back left side. Medical history is surgeries: Hysterectomy, Appendectomy, Tonsillectomy, Gallbladder removal. Do you have any idea what this could be? The Dr. s where I live do not seem to care to figure this out. Doctor: Hi ! Good morning. I am Dr Shareef answering your query.From your history and symptoms, it seems that you might be having a pelvic infection (PID) associated with a urinary tract infection (UTI). If I were your doctor, I would advise you for a urine test (routine, microscopic, and culture/sensitivity test), a complete blood count (CBC), a blood sugar test, gram-staining and culture sensitivity of vaginal discharge if any and a review ultrasound of abdomen to rule out any other intra-abdominal pathology. Till the reports are ready, I would advise you for a braod spectrum antibiotic along with a probiotic, a proton pump inhibitor, and an anti inflammatory drug.I hope this information would help you in discussing with your family physician/treating doctor in further management of your problem. Please do not hesitate to ask in case of any further doubts.Thanks for choosing health care magic to clear doubts on your health problems. I wish you an early recovery. Dr Shareef."
},
{
"id": 152895,
"tgt": "Is radiation necessary with thyroglobulin at 13?",
"src": "Patient: I had a total thyroidectomy 12 years ago due to thyroid cancer. Three months ago I stopped taking my Synthoid for a full body scan. The scan came back clear but my thyroglobulin was at 13. I had an MRI done of my neck area the radiologist said one of my lymph nodes was right at the limit of size. My doctor ordered a biopcy but I have not done that yet. He is also saying he thinks I need another radio active iodine treatment, but that bit carries risk of causing other cancers. I have heard of using ultra sound for diagnosis. Is this something that would work in my case or do I need to have the biopsy done? But the bigger question is do I need another radiation treatment with a 13 level thyroglobulin????? Thank you for your time. Lance Doctor: Thank you for posting your query. Thyroid cancer may relapse manifested with high Thyroglobulin level and appearance of lymph node.Lymph node needs pathological testing (biopsy) to confirm whether there is any recurrence.Radio Iodine Ablation is standard treatment of relapse.But, Thyroglobulin level >10ng/ml is also an indication of another course of Radio iodine ablation therapy."
},
{
"id": 29263,
"tgt": "What causes itchy rashes on the body along with diarrhea, cold sweats and nausea?",
"src": "Patient: My son is 16 yrs old. This morning he woke up with a itchy rash all over his body.Now he is complaining that he has diarrhea, cold sweats, and nauseated.Took him to the doctor this morning, but they didnt know what was wrong.They gave him a Z pack steroids meds to take.Any suggestions? Doctor: HI, thanks for using healthcare magicThe most likely cause is a viral illness. Many viral infections can be associated with a skin rash, called viral exanthem which normally resolves with or soon after the other viral symptoms.Viral infections can cause diarrhea, nausea and cold sweats. In some persons, antibiotics such as the z pak may worsen diarrhea and nausea though this may not be the case for himI hope this helps"
},
{
"id": 152262,
"tgt": "Is there any medication for brain and heart control during longer sex ?",
"src": "Patient: Dear Doctor Sahib, Please suggest best tablets available at Lahore_Pakistan which full control my brain , heart and give me powerfull erection pleasure during sex. Waiting your reply. Sheikh Imran. E-mail:kaleemexchange@hotmail.com I answer your health queries right now ? Please type your query here... Doctor: You only has the power,no tablet can give you anything,so do not wonder here and there.Control yourself ,develop self confidence,try yoga and medication which will increase your brain power and gives you mental stability,eat good healthy food,drink plenty of liquids .this are the only tablets you need.If you really feel low,get your self examined by your doctor and take medicines accordingly.do not worry."
},
{
"id": 194002,
"tgt": "Is a surgery required for hydrocele?",
"src": "Patient: hi sir,my name is ram iam suffering with hydrocele since 10 years with out pain.in between i meat doctors twice they checked and said no need operation just use they prefered medicine. but present one week on words iam geeting pain.what can i do? if iam going to operation. after operation days need sexually rest. please give me suggetation. Doctor: Hello, As per your history, it may be due to hydrocoele. For further assessment you may require ultrasound Scrotum after consultation After confirmation you may require surgical intervention after surgeon consultation. Till then you can take tablet acetaminophen for pain. Use scrotum support. Hope I have answered your query. Let me know if I can assist you further. Regards, Dr. Shyam B. Kale, General & Family Physician"
},
{
"id": 121983,
"tgt": "What causes stabbing pain at the back of the leg above the knee that lasts for a few seconds?",
"src": "Patient: Im getting shocking pain in the back of my leg just above my knee... it doesn t last long, maybe a 5-10 seconds. It just happening less than a week ago. Kind of feels like an electrical shock. Its It s weird, never felt anything like this ever.Whats wrong with me and is this a common problem? Doctor: Hello,Thanks for the query.It can be an arthritis or ligament injury.As a first line management you can take analgesics like paracetamol or aceclofenac for pain relief. if symptoms persist better to consult an orthopaedic and plan for an MRI scan.Wishing all the best.Thanks"
},
{
"id": 18975,
"tgt": "Is Aspirin advisable for elevation in the BP level despite taking Losartan?",
"src": "Patient: Hello, I am a 60 year old female who started taking 25 mg of Losartan daily about 4 months ago. my blood pressure has dropped some since I started taking it but is sill not in the normal range. Was 170 s over 90 s when I started taking it and is not normally 140 s-150 s over 70 s. I spent the last 24 hours with my daughter who is going through some traumatic circumstances and returned home this evening to find my blood pressure is measuring around 165-175 for the top number and 99 -102 for the bottom number. I am trying to relax and my blood pressure has gone down a bit since I got home (167-98) but I m wondering if I should go to the ER or if I could just take an aspirin to help for the moment? I actually have a physical scheduled for tomorrow afternoon so that will be handy but would like to know:1) exactly what my blood pressure needs to be where it is bad enough to go to the hospital (obviously not where I would like to spend the evening) 2) Is were something I can do right now, such as take a low dose aspirin to help reduce my blood pressure without having to go the hospital? Thanks for your help! Marie Doctor: Hello,Your blood pressure is a little high despite the use of losartan. I can understand that you have had a stressed day and that may contribute to higher blood pressure. If you don't have any other disease (renal disease), you can take another Losartan. If you were my patient, I would give you hydrochlorothiazide starting with early morning. It takes a few weeks to start acting. Meanwhile, you take daily or twice a day Losartan. It can low your blood pressure. Aspirin does not play a role in decreasing blood pressure. It only might increase it a little.Hope I have answered your query. Let me know if I can assist you further.Regards, Dr. Anila Skenderi"
},
{
"id": 68014,
"tgt": "What causes lump near the groin area?",
"src": "Patient: Hello, I was wanting to know if this lump next to my hip bone but not to close to my groin but close to my stomach is a swollen lymph node. I m sexually active but have only been with one partner. He gets swollen lymph nodes occasionally and I was wondering if that is normal. Doctor: Hi, dear. I have gone through your question. I can understand your concern. You have swollen lymphnode. It may be due to reactive hyperplasia most likely. You may have tuberculosis or lymphoma but Its less likely. You should take course of antibiotics. If lump doesn't respond to treatment then biopsy diagnosis is advised . consult your doctor and take treatment accordingly. Hope I have answered your question, if you have doubt then I will be happy to answer. Thanks for using health care magic. Wish you a very good health."
},
{
"id": 144386,
"tgt": "Suggest treatment for a hard lump on the base of skull",
"src": "Patient: I have a hard knot at the base of my skull. It s hard and won t move but it s cause headaches and light sensitivity, also the headaches start right off the bump it feels. What could that be? I m not sure if they are related but I ve also had nose bleeds since this. Doctor: Thank you for your query.The base of your skull houses some important structures namely the occipital lobe ( responsible for vision), brain stem ( responsible for regulating important body functions like breathing,blood,heart rate and also houses several workhouses of the cranial nerves including the occulomotor which supplies the muscles of the eye) and blood vessels , contained in a narrow space, that supply the posterior part of your brain.Any space occupying lesion (SOL), a general term used to describe 'knots', 'bumps', 'tumours', in this area needs to be examined promptly especially as any pressure to adjacent structures , mentioned above , can cause compromise of these functions and Increased intracranial pressure.Nose Bleeds may occur to help relieve these symptoms . All your symptoms fit in this scenario. The SOL needs to be checked out right away by CT or an MRI of the brain and surrounding structures. Based on the size, type and other characteristics, a treatment plan will be made and initiated. This may include surgery to relieve pressure symptoms and effects.I recommend a prompt visit to your doctor to get it checked out.in the meantime, worsening ofy our symptoms, occurrence of new ones like, tremors, breathing difficulty, severe headaches, vomiting warrants a trip to the ER right away.hope this helps and you get better soon.wish you good health and feel free to get back to us if you need any clarification"
},
{
"id": 120925,
"tgt": "Suggest remedy for pain in right thigh above the knee?",
"src": "Patient: Hello doctor, I m 25 years lady, i got pain in my thigh( above the right knee ) when it got pressed. Moreover it is difficult for me to lay on the ground joining my two legs. I don t know what the real matter is ? It happened when i was 16,but i didn t take it seriously. But now when i concern with my nearest doctor, he said it is the matter of some injury happened inside the nerve of my thigh. So, what should i do? What type of report will be effective for you to give feedback.. Will the MRI report be effective ??? Doctor: Hello,Looking at your details it looks to me either an adductor muscle contracture or as suggested old nerve injury. In this case MRI spine and affected limb will be adequate to rule out the problem. Hope I have answered your question. Let me know if I can assist you further. Regards, Dr. Mukesh Tiwari, Orthopaedic Surgeon"
},
{
"id": 37103,
"tgt": "Suggest treatment for pus filled bump on back of head",
"src": "Patient: My son has a spot on the back of his head at first I thought he may have got a scratch cause there was a pussie scab. But everyday another pussie scab forms over it. When we pull it off it doesn t bleed but now it s raised and a round red spot I think clear pus is on it and is getting bigger Doctor: Hi,It seems that he might be having some bacterial scalp infection.Give him one course of antibiotic medicine for 3-5 days.Wash his head with branded hair wash.Apply antibiotic cream on affected part.Ok and take care."
},
{
"id": 1341,
"tgt": "Why the pregnancy test is showing different results?",
"src": "Patient: HELLO DR. my wife had missed her periods, her periods were due on 9th dec. we have used paganews yesterday evening but it had shown negative result, again i hve heard from somewhere that it should be used in the morning, so i have used it today morning and the results were positive, so we have planned to visit a doctor nearby, today evening the doctor has done the same test again and it had came negative this time. what to do..? totally confused..? please help. Doctor: Hi ,How are you doing ?You have to confirm the pregnancy , urine test is not confirmatory. it can vary with time of the day, with the quality of the kit.Best way to confirm is Serum Beta HCG . It can diagnose the smallest of pregnancy & anywhere in the body.Urine test is positive at Beta HCG at 25iu/ml, even lower levels are detected by the serum beta HCG levelScan detects pregnancy only at 1500iu/mlHope I have clarified your query, do write back if any more queriesAll the best Dr.Balakrishnan"
},
{
"id": 143429,
"tgt": "What is the remedy for the seizures and depression with mood swings?",
"src": "Patient: Hi, my son is experiencing the seizures for the last 8 months and have 4 episodes for 30-40 seconds. It started in Aug 13. He is 16 years old and have 45kg weight. The doctor started the medicine KEPPRA 500mg twice a day and the increased it to 750gms twice a day after the second attack. After the 3rd attack in Dec his medicine potency was increased to 1000mg twice a day. Now 5 days back he has again experienced the attack and the Dr has advised to add another medIcine Oxetol 300MG twice a Day along with KEPPRA1000mg twice a day. But he says that this medicine will be replaced slowly with Oxetol. His Pet Neuro is normal and EEG & MRI are also normal. How to deal with this problem please as he is facing sometime depression and his behaviour also swings and all of a sudden he gets angry ? Doctor: Hello!Welcome on HCM!I carefully read your question and would explain that the changes in his mood can be related to Keppra adverse effects. From the other hand Oxetol can improve these mood changes. So, I would recommend follow your doctor advise to gradually switch from Keppra to Oxetol. Regarding his normal EEG and MRI, I would recommend performing a 24 hours EEG (prolonged EEG) in order to be able to register any possible epileptic activity. Sleep deprivation can help in this regard. You should discuss with his doctor on the above issues. Kind regards, Dr. Aida"
},
{
"id": 28459,
"tgt": "Suggest treatment for heart murmur and chronic hip pain",
"src": "Patient: I have seen an orthopaedic surgeon who has advised me to have a total hip replacement. Several years ago the garbage can fell on me knocking me down, and screws were inserted in the femur. It has been getting increasingly difficult to walk without pain, and I now use two crutches. I take no medicines, am 87 and have no health issues. The surgeon said that the position of the screws would mean they would soon penetrate the pelvis, making it impossible for me to walk at all and to be in constant severe pain. Today I went to my G.P. to be cleared for the surgery. She heard a heart murmur, and has suggested they might want to do an Echogram. Haven t heard back from the surgeon yet, but could this prevent me from having the surgery? The thought of pain progressing and being unable to walk has me very concerned. Doctor: Hi welcome to HCM.I understand your query and concern.Heart murmur needs quick evaluation for any organic heart disease.THR carries increased risk in the form of cardiac embolism.This has to be prevented by the following set of investigative work up.I advise you to have a baseline 2 dimensional echocardiography,ECG and lipid profile to assess the basic cardiac reserve of your heart.Restrict the intake of salt to less than 6g/day.Regular physical exercise in the form of brisk walk for 20 min a day for 5 days a week is pretty useful.Drugs like antihypertensives and antiarrhythmics will help.Reduce the intake of fatty and fried food.One pomegranate a day will help to keep your heart at good pace without clot formation. Review the orthopedic work up with regular x ray imaging of the screws and the plates.Consult a Cardiologist for further expert management.Post your further queries if any.Thank you."
},
{
"id": 85978,
"tgt": "What medical condition causes a stabbing pain in the stomach and abdominal bloating?",
"src": "Patient: I have severe stabbing pain in my stomach area every 20 minutes. it lasts only about 15 seconds. A few days ago when it started my stomach was bloated and very hard. Not it is only a little bloated but the stabbing pains are still coming. Any thoughts? Doctor: Hi, The recurrent stabbing pain in the abdomen that last for 15 seconds and comes every 20 minutes indicates that this is a spasmodic pain. The associated bloating indicates a possibility of sub-acute intestinal obstruction. I would advise you the following in such a situation: Rush to ER or nearby hospital for urgent clinical evaluation and investigation to know the cause as intestinal obstruction needs urgent care. X-ray of the abdomen in standing position can help to get an exact diagnosis aided by an Ultrasound and CT scan of the abdomen. If there is an intestinal obstruction, you need to get admitted and may need a surgery for cure. Hope I have answered your query. Let me know if I can assist you further. Take care Regards, Dr T Chandrakant, General Surgeon"
},
{
"id": 206073,
"tgt": "What causes fear and nausea after getting up from sleep?",
"src": "Patient: when i wake up in the morning, i can never remember dreams ive had, but they appear in my head as flashbacks, and they scare me, i freak out, cant remember anyone, hide myself in the bathroom.. there is nausea, and it scares me.. the longest lasting attack is RIGHT NOW and its been almost an HOUR.what is WRONG WITH ME Doctor: Hello,If there is no other symptom apart from bad dreams we usually find associated anxiety in that case during daytime as well. You havent mentioned anything apart from dreams in the query. Dreams are part of normal sleep but if they are constantly troublesome then it is found that patient has anxiety disorder as well.In such cases anti anxiety drugs are main treatment modality.if there is no associated anxiety at all then you can get an EEG and sleep study. You may also try relaxation exercise and sedative in low dose for some days. These approaches with individual or combinations usually treat the problem successfully.Nausea is because of the terror which you feel due to dream and subside on its own once the problwm is solved.Good luck.Dr. M GopalMD Neuropsychiatry"
},
{
"id": 54777,
"tgt": "Suggest remedy to cure hep b",
"src": "Patient: my mother age 65 year is suffering from Hep B from last one month. she has piles now due to piles and constipation there is lost of blood. kindly suggest what we can do for her to recover from this problem . Please suggest me medicine and diet for her. Doctor: Hi thanks for contacting health care magicShe has Hepatitis b since one month.For that bed rest done ..Take lots of fluid in firm if mainly fruit juices ex...papaiya or grapes juice ..Avoid physical exertion and alcohol.Fried food taken very less.One tsp licorice with honey can be taken....Udiliv tablet can be given....For piles take stool softener.Avoid straining at stool.Anal warm water sitz bath can be taken...Avoid excess spicy food.Local betardine ointment can be applied...Hope your concern solved...Follow up is necessary esp.viral narker for HBSag after six month....Dr.Parth Goswami"
},
{
"id": 7372,
"tgt": "Is the follicular cyst in the left ovary related to PCOD ?",
"src": "Patient: I Have irregular periods from 2 years. I am 29 yrs old. My Gynic advised to take TVS and the report says that my utarus and Right ovary is normal.But a follicular cyst in Left Ovary measuring 30X26MM and left ovary measures 38X31MM. No free fluid in POD. Is it related to PCOD. Doctor: Hello Welcome to health care magic forum ultrasound report is normal & there is nothing to suggest of PCOD.Single small ovarian cyst disappears with in a month or so.your Wish you good health Disclaimer"
},
{
"id": 16544,
"tgt": "How can excessive phlegm and chronic cough after having meals be managed in a heart patient?",
"src": "Patient: My husband gets full of phlegm after almostevery heavy meals and violently coughs, gags, and spits up mucous within 10 minutes of eating . This happens almost after every meal and not just with one type of food. With all types of foods. He\u2019s been placed on antibiotics nasal sprays etc but nothing helps. This has been going on for several years. He does have cardiovascular disease. Has had a bypass and is overweight by 30 pounds or so Doctor: Hi, Your husband should go and get an upper GIT endoscopy to show us if there is possible inflammation in his stomach. Hope I have answered your query. Let me know if I can assist you further. Take care. Regards, Dr. Salah Saad Shoman, Internal Medicine Specialist"
},
{
"id": 212887,
"tgt": "26yr old, having difficulty in sleeping, wakes up with a loud shout. Prescribed medicines. Little improvement. Suggestions?",
"src": "Patient: Dear and honorable online Doctors, I am writing this from Afghanistan, Kabul. My 26 year old son, for the last one year is suffering from restlessness, difficulty with sleeping, low mood, fear etc. He awake from sleep with a laud shout, even this happens to him during the day. Some time when he is busy with some work or computer all of a sudden, without any prior signs he shouts, and wants to run away ( Is it panic attack??). One year before we consulted a doctor in Delhi he prescribed: Symbal 20 mg Frisium 10mg, twice Clobazam , once at night Olanex, twice, 2.5 mg in the morning and 5mg at night Nitrosun, one tablet day and half at night Liberum, 5mg day and 5 mg at night He was better under medicine, however still was experiencing the attack. I was not comfortable with so many medicines. November this year again went to Delhi and consulted another doctor. After polysomnography test he prescribed him Depran 10 one tab. at night and Zapiz, 0.25 mg whenever he is in bad mood ( the later does not help him much). When he takes the medicine at night, he is feeling okay till next day evening, and then his mood is decorates and experience the same attacks. However, still he has difficulty with sleeping. I would appreciate if you honorable online doctors can advise him. Can he continue Nitrosun, which is a sleep medicine whit his medicine? Worth mentioning that I trust fully the doctor that we consulate, but he is far away from us ( in a second country) that difficult to reach him frequently. Appreciate your advice Best regards Gh.M Salem Email: YYYY@YYYY Doctor: Hello. Thanks for consulting healthcare magic. Salemsahab, from the history you have narated, your son appears to have some kind of seizure disorder. That can also be confirmed by getting and EEG done. The drugs mentioned in your first prescription are right, but i should say that they are too many. He probably doesn't need these many drugs. If he has some abnormal EEG findings, starting him on a single anti-epileptic drug such as carbamazepine or leveteracetam will surely help him. if there is good control of his epileptic attacks, his mood symptoms and sleep should improve automatically. I understand that the distance is the problem. hope you get a good psychiatrist nearby who can consult you frequently. Thanks."
},
{
"id": 88389,
"tgt": "What causes leison on pancreas and abdominal pain?",
"src": "Patient: Should I be worried about a lesion on my pancreas. I was very sick vomiting and really bad abdominal pain and went to the e.emergency room. A CT was performed and the DR there didn t note anything. How ever the official radiologists report came back and he noted pancreas lesion YYYY@YYYY Doctor: Hi.Thanks for your query. Since you are now diagnosed as a case of pancreatic lesion, you need to go further to get a proper diagnosis to decide the actual diagnosis and plan a treatment under a care of a Onco-Surgeon or a Gastroenterologist Surgeon. I would suggest you the following ::-Blood tests routine and particularly amylase, lipase, liver function tests, kidney function tests, -Urine and stool tests- occult blood.MRCP by MRI for further evaluation- FNACERCPThe proper diagnosis will decide whether this is a Cancer of a pancreas or a benign lesion and you may have to go for surgery."
},
{
"id": 117078,
"tgt": "What causes calcification in my lymph nodes?",
"src": "Patient: I went to get a breast exam and the doctor stated I had calcification in my lymph nodes? He had asked me if I d been out of the country or any tattoos. I said NO. He stated he thought I would need to go to MY OBGYN and get add l tests done. Blood work etc. They my OBGYN called and said it wasn t cancer but I should get a tuberculosis TEST to rule that out. I can t imagine I have TB. that seems so strange to me. Doctor: Hi,Thanks for asking.Based on your query, my opinion is as follows.1. Lymph nodes are enlarged due to numerous conditions. Commonly Tuberculosis, and cancer in elderly.2. Get an FNAC done. It will confirm it. Mantoux test is also helpful.3. Not to worry. You will be completely cured.Hope it helps.Any further queries, happy to help again."
},
{
"id": 195493,
"tgt": "Can I take Nacetyl cysteine to decrease the time of semen liquefaction?",
"src": "Patient: hey... my liquefaction time of my spemen analysis is 30 min (borderline), is it normal ? can I take like Nacetyl cysteine to decrese time of liquefaction, or what can I do ? is N acetyl cycsteine taken without Dr, Prescription is good ? or may alterate my sperm Thanks a lot Doctor: Hello and Welcome to \u2018Ask A Doctor\u2019 service. I have reviewed your query and here is my advice. The normal semen liquefaction time ranges from 20-30 min and anything above 60 min is dangerous and make the process of fertilization difficult, in your case i do not think that you should be worried about it. The main two reasons for increased liquefaction time is semen infection or dehydration. I would like you to share the complete details of your semen analysis so that i may rule out the infection. If there is no infection the the cause of increased liquefaction time would be dehydration and i would recommend you to increase your hydration status. Regarding n-acetylcystiene, studies has shown that it does improves the health of sperms but it has no documented effect on sperm liquefaction time and i would recommend you not to use it without doctors prescription. Hope I have answered your query. Let me know if I can assist you further."
},
{
"id": 133772,
"tgt": "What causes hip pain and difficulty in walking?",
"src": "Patient: Just looking up hip pain (feels bruised but isn t) and trouble walking sometimes. Short rest (sitting) makes it possible to continue. 58, male Oh forget it. Of course you ll tell me to see my doctor. I was looking online to get available info first... Doctor: hi,thank-you for providing the brief history of you.A thorough neuromuscular and musculoskeletal assessment is advised.As the pain is in the hip area, it can be related to either of the pathologies. Either a hip joint pathology or a lumbar spine pathology.Based on the thorough Clinical neuromuscular assessment and musculoskeletal assessment the MRI may be advised.Usually there is difference between the ache and the pain. Ache is muscular and pain is neuropathic. Now you should be able to differentiate both individually and based on that the treatment is planned.Following which with simple physical therapy exercises you should regain the strength in muscle. Also, your age is 58, so after a thorough assessment the treatment planning will be performed.RegardsJay Indravadan Patel"
},
{
"id": 73763,
"tgt": "Suggest treatment for cough",
"src": "Patient: i have been coughing for the last three weeks. I was examined and had malaria and was treated with Coatem anti malria tablets, antibiotics and a cough surup called Bro -Zedex. I am still coughing and i cough like someone who has some dirst in the chest. Kindly advice.i do hope to be adviced. Doctor: Thanks for your question on Healthcare Magic.I can understand your concern.You are having chronic cough (cough for more than 2 weeks).So we should definitely rule out lung infection and bronchitis in your case.So get done chest x ray and PFT (Pulmonary Function Test).You may need inhaled bronchodilators (formoterol or salmeterol) and inhaled corticosteroid (ICS) (budesonide or fluticasone).Don't worry, you will be alright. First diagnose yourself and then start appropriate treatment.Hope I have solved your query. I will be happy to help you further. Wish you good health. Thanks."
},
{
"id": 182226,
"tgt": "Suggest remedy for shift in right jaw after getting punched in face",
"src": "Patient: I got punched in the face about 5 days ago. It was an uppercut to the jaw on the left side. It wasnt until yesterday that i noticed any clicking but i felt no pain. The only time that i would feel it is when i would move my jaw side to side. I just woke up now and it hurts a bit but nothing absolutely unbearable. When i open my jaw wide i can feel a slight shift on my right side. It also hurts now to open wide. Doctor: Thanks for your query, I have gone through your query.The pain and deviation of the jaw, limited mouth opening can be because of two reasons.The first could be because of the disc displacement in the jaw joint secondary to the punch on the face. The other possible cause can be a fracture in the mandible. So consult a oral physician and get it evaluated. You need to take a radiograph like OPG to confirm the diagnosis. Later it can be treated accordingly. Till then, do not open your jaw too wide, take soft diet.I hope my answer will help you, take care."
},
{
"id": 167184,
"tgt": "What causes bad smells from child s body and how to retify?",
"src": "Patient: i have a 3yr old child.i notice that there are times his sweat smells like fresh squid.its funny but its true,i took him a bath thrice a day still he smells bad.though this isnt always but im so curious why sometimes it happened.i wanna know the causes and also the remedy for this..pls help..tnx Doctor: abnormal smell of body may b due to certail metabolic disorders with causes accumulation of certain substances in the body. you should consult a pediatrician for this smell."
},
{
"id": 100642,
"tgt": "Could diarrhea and cramps be due to allergy towards milk products?",
"src": "Patient: My grandson ate some sugar free pudding yesterday and within hours was cramping and suffering from diaharia. Could he be allergic to milk products? He couldn't take regular milk as a baby, but soy. My grandson is 2 yrs old and weigh 27 lbs, otherwise he is healthy. Thanks Doctor: Hi, thanks for using healthcare magicThe reaction can be due to : (1) gastroenteritis - infection induced after using the pudding. This would be more likely if other persons who also used it, were sick as well(2)intolerance or an allergic reaction to the foodIf he uses other milk products with no problem then it may be some other aspect of the pudding that caused the reaction.If no other persons who used the same pudding are unwell then it is unlikely infection and you would need to consider the other option.It is possible to assess for allergies by testing at your doctor's office.I hope this helps"
},
{
"id": 128144,
"tgt": "How to prevent traumatic back and neck injury from returning ?",
"src": "Patient: I am terrified. Don t know why. Maybe it s because I sufffered a traumatic back and neck injury and am in pain from that? I know there is pain but I am afraid it will be horrific. I am an old broad for this surgery--lavh Hello? Grief. Interesting name. Doctor: Hello, I have studied your case.Traumatic spine can lead to disc bulge.I will advise MRI spine to see for any nerve compression leading to back pain.Medication like methylcobalamine with muscle relaxant and analgesic will reduce pain; you can take them consulting your treating doctor.You may consult physiotherapist for further guidance. He may start TENS, or ultrasound which is helpful in your case.I will advise to check your vit B12 and vit D3 level. As MRI shows disc compressing on nerve root then surgical decompression is permanent solution.Hope this answers your query. If you have additional questions or follow up queries then please do not hesitate in writing to us. I will be happy to answer your queries. Take care."
},
{
"id": 205263,
"tgt": "What causes bad temper while having depression?",
"src": "Patient: Dr.my elder brother (age 33) is in depression near about ten year or more he do not visit doctor and take medicines. He is always in high temper and do not go any where from home. We are nothing know to what to do please suggest us to do something for him. Doctor: Hello,Depression becomes sometimes agitated so that can be one reason your brother gets angry. Or your brother has bipolar disorder where depression and agitation come and goes. So I suggest you take your brother to a psychiatrist and start him on antidepressant or mood stabilizer after confirming diagnosis. If he refuses to take medicines, better get him admitted for few days till he recovers and start taking tablets on his own.Hope I have answered your query. Let me know if I can assist you further.Regards,Dr. Prashant Gohil"
},
{
"id": 20345,
"tgt": "What causes pressure in chest with numbness on the left side?",
"src": "Patient: Q : Alright im not sure if I had a heart attack tonight or what, but im really worried. I was walking to a bus stop with a few of my friends at about 7:45pm-it was cold out and the ground had about 6 inches of snow. It must have been 36-40 degrees(not sure, thats a really vague guess...all I know is it was COLD). We decided it would be fun to try and run through a meadow to the highway, well we all began running for about 35 seconds. When we stopped I had a really hard time catching my breath-I couldnt make full breaths and my heart was pumping very fast and very hard. I was in a store buying something (right after meadow-store is next to meadow) and I still didnt have my breath back, for some reason putting pressure on my heart helped somewhat. I was at the checkout counter and suddenly my left arm and the left side of my face got really numb, I got really dizzy and light-headed and my breathing became even harder. I walked to a bench as fast as I could and lied down for a few minutes, I felt okay when I tried to stand back up, and I can breath again-but not big breaths. Whenever I take a big breath my chest stings and I have to cough. My chest is still really sore! Im 17 years old, 5 11 , and about 189 pounds. Im a little overweight but its not bad. I have been going to the gym regularly for a month and a half now and doing rigorous cardio exercises. I have a hard time believing it s just because im out of shape... ...im really worried. ...Did I have a heart attack? Does this need a doctors attention(immediatly?) Sorry for the long post! thanks Doctor: The symptoms that you describe have multiple possible causes. Being 17 doesn't mean that you are bulletproof. Yes. Go see a doctor immediately."
},
{
"id": 200437,
"tgt": "Is it healthy to masturbate twice a week?",
"src": "Patient: I am a 75 years old men in general heathy good condition but unable to have sex mith my wife becausse she sick l have a problem maintaning an erection but if I play with my pennis I masturbate and enjoy very much .My quetion is it\\s healthy to do it twice a week thank you very much Doctor: Hi, dearI have gone through your question. I can understand your concern.Yes it is healthy to masturbate twice a week. There is no harm in masturbation. It is normal phenomenon. Don't worry about that. Just be relaxed and enjoy your life.You are healthy and no treatment is required for that.Hope I have answered your question, if you have any doubts then contact me at bit.ly/Drsanghvihardik, I will be happy to answer you.Thanks for using health care magic.Wish you a very good health."
},
{
"id": 194538,
"tgt": "What causes the shaft to sting when touched?",
"src": "Patient: I masturbated with Cetaphil, it entered my urethra and it's a little more difficult and painful to urinate. My shaft stings to the touch when I press where the head base is. It throbs and stings every now again and when I urinate, and now I can't stay errect. I hadn't had sex 6 months prior to this incident, can I rule out an STI? Doctor: Hello, You can have acute urinary tract infection or STI. For further work up I suggest you investigate with urine culture report. According to the causative organism and susceptible antibiotic, medication can be prescribed. Hope I have answered your query. Let me know if I can assist you further. Take care Regards, Dr Parth Goswami, General & Family Physician"
},
{
"id": 119472,
"tgt": "Suggest treatment for bent back bone with difficulty to walk",
"src": "Patient: What is an actual bent back bone because i have been to the doctors an got told i had a bent back bone an since then just got on with my life but recently its starting to hurt me, when i walk short distances should i go to the doctors an let them have a look? Doctor: Hi, By saying bent back doctors means that a person is having scoliosis. Scoliosis is of three types and its progression depends on the type as well as on initial degree of the condition. Treatment of scoliosis is conservative with braces in initial stages and operative in later stages when scoliosis starts causing additional symptoms. You need to visit an spine surgeon to evaluate you clinically and to guide you regarding further management. Take care. Hope I have answered your question. Let me know if I can assist you further. Regards, Dr. Rohan Shanker Tiwari, Orthopedic Surgeon"
},
{
"id": 4409,
"tgt": "After taking first course of trigestrel, when should I have sex to conceive?",
"src": "Patient: Hi , I am a lady 31 years old ,I have one baby 11 years old ,I last have my period 8 months ago ,before I already have a period disorder ,now I am taking my first course of trigestrel so that I ca concieve ,now I am bleeding ,which days can I have sex so that I can get pregnant , yours Cihihijy Doctor: Hi,In a normal cycle of 28 days, the ovulation days are usually 12-16 day of the cycle. but now that you are taking trigestral, which is a contraceptive pill, which has been given to you for regularising your period. You cannot concieve while on it. Complete the course, and after you have regular periods, you can try on your fertile days/ovulation days."
},
{
"id": 94595,
"tgt": "Taken Refocox for pleural effusion. Now abdominal pain. Taken DOTS treatment",
"src": "Patient: Hello Sir, I am Manoj, I had treated for flurel effusion for 6 month, Refocox-H. Then had some abdo. pain and i contact to doctor he tells me that my intensites gets sensitive, he treated me but my prb is not get resolved. Doctor refered me to the tubroclossis expert doctor and he tolds me to make montox test and it gets possitive and He gave me the treatment of DOTS for 6 months. but my abdo. prb is still nor resolved. Please Helps me out sir.. Doctor: Hi, Welcome to HCM, After going through your history it is clear that you have Pleural Effusion that is a kind of fluid in pleural cavity around lungs,you have not mentioned which side is involved.But why this has not resolved in 6 months? This most commonly is due to irregular treatment as stopping drugs in between makes treatment difficult. Now you are taking DOT therapy please continue it regularly and if fluid is more in cavity it can be aspirated and send for AFB,and R&M study. Your abdominal problem may be related to TB itself or drug induced kindly check your liver functions as some time Antitubercular drugs may cause hepatitis. If liver function are OK, taking drug on empty stomach and symptomatic drugs prescribed by your chest physician will help to resolve or at least reduce them. Take care Good Luck"
},
{
"id": 119350,
"tgt": "Anaemic patient, hemoglobin blood test, level 4.2%, on hematologic tablets, new level 9.2%",
"src": "Patient: Respected doctor, my wife is a anemic patient ,we had a hemoglobin blood test before one & a half month before and the percentage of hemoglobin came 4.2%. we consult a doctor and he advice some hematologic tablets . my wife eaten every day the tablet and healthy food & fruits also. recently we again done a hemoglobin blood test and the % came 9.2% is it possible ? Please let me know. Thanks XXXXXXX Doctor: Hi Vinay, Taking hematological drugs for at least 3 complete months is known to raise the Hb levels. As evident by reports Hb levels have increased ,means your wife has completed the course. Let her continue for 1 more turn of medication and have Hb levels between 12 to 14%. Let her eat iron rich food like red meat, egg yolk, green leafy vegetables,dry fruits, jaggery and pea nut etc. Best regards, Dr. Neelam."
},
{
"id": 120863,
"tgt": "Suggest remedy for pain in arm upto the elbow",
"src": "Patient: i knocked my wrist very hard against a door knob about 6 weeks ago. i went to the E.R and to an ortho, both tech who did my x-ray said it looks to be fractured, however both Dr reviewing the x-rays say they see no fracture to the scaphoid or anywhere else. now, today (6 weeks after the injusry) i am having a hard time picking things up, putting dishes away, opening doors, etc with out pain. Originally i felt some tenderness in the snuff box but that has since subsided. However the pain when i move my hand a certain way, can be felt up to my elbow. is this just a bruised bone or something more? i have an MRI scheduled, but dont want to spend the money if i dont have too... help Doctor: Hello,Your symptoms seem to be related to a pinched nerve in the shoulder due to the injury. I suggest using anti inflammatory medications such as Acetaminophen to relieve the pain. I also suggest to avoid the movement of the arm to prevent further damage.Hope I have answered your question. Let me know if I can assist you further. Regards, Dr. Dorina Gurabardhi, General & Family Physician"
},
{
"id": 87955,
"tgt": "Suggest remedy for abdominal pain, nausea, gastritis, inflammation and weight loss",
"src": "Patient: Hello, i am a 56 year old woman with digestive issues. I ve had C-D twice and in April 2013 I had left-sided ischemic colitis. I ve bowel problems for many years, mainly loose bowels, dark stools, mucos. Since last april i ve had more bad days then good; abdominal pain, right and left side and across the middle, nausea, gastritis, inflamation, weight loss apprx 30 lbs. I ve had colonoscopies (inflamation of the large intestine), endoscopy (acute gastritious and stomach inflamtion), capsule endoscopy (normal small intestine) Doctor: Hi.Thanks for your query and an elucidate history..Since you are already well investigated, the treatment has to be well planned. This is because you have a complicated combination of colitis and gastritis.It is good that the small intestine is normal. I would suggest you the following and I hope you are already taking some treatment under the care of the Gastroenterologist. Soft bland diet to be equally distributed in 4 parts and spaced nicely to cover at 5 to 6 hourly duration.Ranitidine /PPI Antacid gel at 3 hourly interval.For colitis:The 5 day course of an antibiotic and metronidazole, probiotic. Avoid all the foods and beverages which you must have noticed by this time that increase your problem. Review endoscopies after one and half months. And to reschedule treatment as per the new findings."
},
{
"id": 36219,
"tgt": "Suggest treatment for dog bite",
"src": "Patient: I was just bit in the right calf by a dog. It broke the skin in a couple of areas, is bruising and the area has a burning pain sensation. I called the police and made a report; I also identified the two offending dogs. Their rabies vaccinations are not up to date. Would you suggest I go to the ED? Doctor: Hi,You should go to ER and get examined.As you are having skin breaching in multiple area with no proper information of vaccination of dogs, you require to go for,1, tetanus toxoid shot,2, ARV shots of five.3, cleaning and dressing of the lesions.Ok and take care."
},
{
"id": 78375,
"tgt": "Suggest treatment for cough in a multiple myeloma patient",
"src": "Patient: Hi, My 77-year-old mother was diagnosed a few years ago with smoldering multiple myeloma. She sees her hem/onc twice a year. She has started coughing within the last week. It may be an allergy or cold but, should she let her doctor know? Thank you, Lori Doctor: Thanks for your question on Health Care Magic. I can understand your concern. Yes, she should definitely tell about cough with her oncologist. Respiratory tract infection is very common in cancer survivor patients. So cough in such patients should always be evaluated first for lung infection like pneumonia. So better to get done chest x ray. If chest x ray is normal then no need to worry for lung infection. Possibility of cold is more. So tell your mother to drink plenty of fluids orally. Warm water gargles 5-6 times a day is useful. Don't worry, she will be alright, but better to get done chest x ray. Hope I have solved your query. Wishing good health to your mother. Thanks."
},
{
"id": 207914,
"tgt": "Suggest treatment for stress and depression",
"src": "Patient: i feel sick tired, i had a boyfriend, when dont call me i start crying, at that time i feel like to kill myself, i cry very badly, i even dont know whether i have migraine or something, please help me, why i am so concerned about his phone call, is there a major problem? Doctor: HiThanks for using healthcare magicI think, you are getting obsess with your boy friend. That could lead to such kind of symptoms. In that case, you need help of a psychologist. Psychologist would help you to teach how to come out of such kind of symptoms. In case, you need further help, you can ask.Thanks"
},
{
"id": 202893,
"tgt": "Surgery of testicular torsion done, testicle removed, silicon prosthesis inserted, pain and burning penis. What to do?",
"src": "Patient: i have a surgery of testicular torsion on 1 st feb.2012. right testicle is removed and a silicon prosthesis is inserted there. now, after ejaculation i am feeling a mild pain and burning in penis. although urine c/s report is normal with no bacteria. am i need any other tests? what should i do? give me some precosions after the surgery. am a ejaculate now? Doctor: HelloThanks for your query,based on the facts that you have posted it appears that you have pain in pain and burning in penis after ejaculation.You have undergone surgery of Orchidectomy for torsion of testis and a silicon prosthesis has been implanted in your scrotal sac.This is in no way related to your pain in penis.During erection and ejaculation blood flow to the penis is increased to 20 times more than normal blood flow resulting in to congestion of entire penis and urethra The pain that you have is due to congestion of the penis and there is nothing to be worried about .Drink more water so that this congestion will subside with passing urine after ejaculation.Dr.Patil."
},
{
"id": 210721,
"tgt": "Experiencing extreme rage and homicidal tendencies when not taking champix tablets",
"src": "Patient: Hi, I was taking Champix and when I ran out of tablets, experienced extreme rage and homicidal tendencies for 2 days. I had to be locked up in a room by my family. (Zero history of any mental illness, depression etc). So when I reported this to my doctor, all she offered me was some different tablets. Should she not have at least reported this??? Doctor: HIthanks for using healthcare magicIt happens with few patients due to sudden stoppage of champix. The suspected symptoms include hostility, agitation, depressed mood, suicidal thoughts and attempted suicide. There is no proof that these symptoms are any more common in people taking varenicline compared with the general population, but still there are few chances of getting such symptoms with it.Thanks"
},
{
"id": 210897,
"tgt": "How do I confirm if I have OCD?",
"src": "Patient: Do I have OCD. I took a test online and it said that it is likely. In 2011-2012 I use to continuously flick the lights on and off until I hit a certain beat. Idk what it was but the clicking had to be right. If I messed up, I would re-start. When I have something in my hand, I have to press my hands on the object until it feels right. Sorry if right isn't very clear but I don't know how to describe it. I use to have a large house and it would terrify me if I heard a noise. We had three floors and I was in the third floor with my older sister. I never told anyone. \ufeffMy parents always admire more for keeping everything clean. I was always awarded for this. I hate germs and I have to clean the bathroom before I take a shower,. Please help me.\ufeff\ufeff Doctor: Hello,Thanks for choosing health care magic for posting your query.I have gone through your question in detail and I can understand what you are going through.From what you have stated it appears that you could be having an OCD. OCD is a very common illness and effects many. It can be mild and some times unrecognizable and sometimes it can be severe. You certainly need to be treated for OCD. Although there are plenty of options available like clonil, fluoxetine, escitalopram, fluvoxamine, sertraline etc my best option for my patients is fluoxetine in higher dosages. Also there is an option of cognitive behavioural therapy. This therapy is taken by either a psychologist or a psychiatrist in which the therapist identifies impaired cognitions (Thoughts) and try to correct them. They are 15-20 min weekly session and there should be around 8-10 sessions to help you with your illness.Hope I am able to answer your concerns.If you have any further query, I would be glad to help you.In future if you wish to contact me directly, you can use the below mentioned link:bit.ly/dr-srikanth-reddy\u00a0\u00a0\u00a0\u00a0\u00a0\u00a0\u00a0\u00a0\u00a0\u00a0\u00a0\u00a0\u00a0\u00a0\u00a0\u00a0\u00a0\u00a0\u00a0\u00a0\u00a0\u00a0\u00a0\u00a0\u00a0\u00a0\u00a0\u00a0\u00a0\u00a0\u00a0\u00a0\u00a0\u00a0\u00a0\u00a0\u00a0\u00a0\u00a0\u00a0Wish you good health,Kind regardsDr. Srikanth Reddy M.D"
},
{
"id": 157874,
"tgt": "Cancer, radiotherapy done, blood flowing from passage of latrine, APC done, prescribed Gynaec CVP. Side effects?",
"src": "Patient: I am 78 years old and was suffering from cancer 10 years back. I got radiotherapy done and recently there was blood flowing from the passage of latrine. On the advice of a doctor, I got APC done. Even after a week of APC there has been bleeding. Thus the doctor has suggested 1 tab of Gynaec CVP after every meal. Could you please suggest what could be the side effects of taking this tablet? Doctor: Hello, Thanks for the query to H.C.M. Forum. 10 years back you were a patient of cancer ,but which organ? You took treatment for cancer as radiotherapy but 10 year back . Now there is bleeding per rectum . APC done ( argon plasma coagulation) , no response in bleeding . Now put on Gynae C V P capsule ( tab) one /day. As far as known to me this is menadione and uses in prophypalxis & treatment of bleeding due to clotting factor . If you have bleeding due to abnormality in clotting factor than you will get relief . As far as concerned about side effect it causes ( help ) in clotting, if any person who have high cholesterol , this is not good . So be aware about that . Good luck. Dr. HET"
},
{
"id": 191040,
"tgt": "Is prolonged use of Sensodent K toothpaste is dangerous ?",
"src": "Patient: Is prolonged use of Sensodent K toothpaste is dangerous ? Doctor: Hi, This is a medicated tooth paste which is for sensitivity relief. This tooth paste should be used for 3-4 weeks untill sensitivity is relieved and then you can revert back to the normal toothpaste."
},
{
"id": 9796,
"tgt": "Suggest remedy for hair loss",
"src": "Patient: HI, This Prakash I am having baldness problem since long back arround 7 years I got treatment from cutis clinic in bangalore in 2007 and medicines \" hair 4 u, sebowash, finabold, xtraglow\", i used these for arround 6 minths and it worked then i again visited the cutis clinic then the doctor gave me some different medicine and lotion which didnt worked so i use the privious one, few months later the improvement level was declined and now in two years i i came to the earlier position that means the same baldness as i had earlier, please advise Doctor: Hi Dear,Understanding your concern. As per your query you have alopecia problem. Well the symptoms you mention in query can occur due to many reasons like genetic factor, fungal infection, covering head with cap and hormonal changes. Nutritional deficiencies and stress are additional factor. Need not to worry. I would suggest you to maintain complete hygiene and avoid excessive use of chemical rich shampoo's. You should consult dermatologist for proper examination and go for skin patch test , blood test and physical examination . Doctor may prescribe you drug like finpecia at least for 3-4 months or anti fungal treatment along with minoxidil spray. Take biotin capsules on daily basis.Hope your concern has been resolved.Best Wishes,Dr. Harry Maheshwari"
},
{
"id": 101622,
"tgt": "Is it safe to take nebuliser 2 hours after using rescue inhaler for asthma?",
"src": "Patient: ht 5' 0\" wt 219 lbs past hx of asthma and diabetes. Current heart rate 116 bpm, shortness of breath. Took rescue inhaler 2 hours ago is it safe to take my nebulizer now? Fearful heartrate will get faster. been fighting an upper repiratory bug for a week and really struggling with the asthma Doctor: Hi, thanks for using healthcare magicThe increase in heart rate can be alarming to some persons but it is not dangerous. The heart can tolerate it. The risk of heart disease at these points is far less than the risk of complications due to asthma.In a hospital setting persons may require nebulisations every 20 t0 30 minutes or be on continuous nebulisations. It has been 2 hours since your inhalor, you can use the nebulisation or if you are in distress, go to a doctor as soon as possible.I hope this helps"
},
{
"id": 30067,
"tgt": "Suggest medication for cold and nasal congestion during pregnancy",
"src": "Patient: Hello.I am 5.5 months pregnant and I was wondering what medications can I take for a common cold? My symptoms are severe congestion, runny nose and a slight sore throat.I currently have Dayquil/Nyquil regular strength cold and flu and Alka Seltzer Severe cold and cough night time medicine.Thanks for your help. Doctor: Hi,Sorry to hear you are suffering from a cold. Common cold is mostly caused by respiratory viruses and may need only symptomatic treatment. Dayquil/Nyquil are drugs that contain:- a pain reliever and fever reducer (Acetaminophen)- cough suppressant (Dextromethorphan)- decongestant (Pseudoephedrine)- Anti-histamin (doxylamine)The above drugs are well indicated in your condition and well tolerated in pregnancy. I will suggest you continue with the drugs and try the following home measures:- Drinking plenty of fluids- Rest well- Adjusting your room's temperature and humidity- Saltwater gargle- Using saline nasal drops- Avoid environmental irritants like dust, cold and smoke. Hope this will help. Let me know if I can assist you further.Wish you good health!Regards, Dr. Mbuomboh Veranso"
},
{
"id": 74680,
"tgt": "Suggest medication for upper respiratory tract infection",
"src": "Patient: I have been advised to take tablet augmentin duo 1gm twice a day by one doctor but my gastroentologist says I should take only levodey 500 twice a day instead for my upper respiratory tract infection. I have MVP and am taking digoxin tab also. pl advise. Doctor: Hello and Welcome to \u2018Ask A Doctor\u2019 service.I have reviewed your query and here is my advice.For mild to moderate upper respiratory infection one antibiotic is sufficient to control the bacteria, any one molecule you can take both are good.The dose of Augmentin duo 1 gm once a day only and not twice a day that should be kept in mind.Hope I have answered your query. Let me know if I can assist you further.Regards,Dr. Bhagyesh V. Patel"
},
{
"id": 71600,
"tgt": "Suggest remedy for chest pain radiating to shoulder and jaw",
"src": "Patient: Chest pains that burn and goes to back of shoulder, sometimes to the left jaw, sometimes to the left elbow with finger tingles. Lasts for sometimes close to an hour. Before the symptoms were this bad had nuclear stress test (normal), echo ( EF 65% and dystolic LV dysfunction)calcium score(zero) holter monitor(inappropriate sinys tach) 24 hr. bp (normal). Ecg shows sinus rythym with poor inital anterior forces, nonspecific st-t wave changes... doctors says, heart is fine chest pain is anxiety but ativan does not take away symptoms. Please help... tired of ignoring worsening chest pains because people think I'm crazy... 36 yr old female. Doctor: Hello,\u00a0It is more of\u00a0stress-induced manifestations of underlying\u00a0gastritis\u00a0and hormone imbalance. Precise medications with a psychiatrist, meditations, relaxation\u00a0techniques, deep breathing exercises, yoga are the turning point in\u00a0your case.\u00a0Hope I have answered your query. Let me know if I can assist you further.Regards,\u00a0Dr. Bhagyesh V. Patel"
},
{
"id": 12310,
"tgt": "Suggest treatment for Psoriasis",
"src": "Patient: Hi, I have had a severe case of psoriasis (covering my entire body - apart from my face) for the last 3 months. I would like to try the puva light but my derm keeps saying that we can start that type of treatment as my skin is so red??? I would've thought that the light would have improved it signigicantly??? Thanks :)) Doctor: Hello. Thank you for writing to us at healthcaremagicSince you seem to have a generalized involvement with psoriasis; this type of involvement is also known as erythrodermic psoriasis Or unstable psoriasis.This type of psoriatic involvement is associated with thermoregulatory as well as metabolic complications as a result of the lack of normal temperature regulatory functions of the skin as well as loss of proteins, and fluids from the skin.In general unstable forms of psoriasis like erythrodermic, pustular psoriasis is managed conservatively with topical emollients and Oral / Systemic Immunosuppressives like Methotrexate, Cyclosporin etc.Before starting on PUVA, patients skin is tested for the tolerable effective dose of PUVA, known as MED (minimal erythema dose). The starting dose of PUVA is then calculated as 75% of MED. However, if all of the skin is already red, there cannot be an accurate determination of MED. Therefore, if inadvertant higher doses of PUVA are given in erythrodermic psoriasis without calculating MED, PUVA can further irritate the skin and therefore is not the first choice amongst dermatologist in unstable psoriasis.Nevertheless, PUVA can be given in low doses along with other systemic treatment like methotrexate, cyclosporin etc even in erythrodermic psoriasis.Regards"
},
{
"id": 186502,
"tgt": "What causes bleeding in gums after brushing?",
"src": "Patient: When brushing my 2 1/2 year old sons teeth his front left and the tooth to the left of it bleed around and between them. Its been going on for about a week now and Im wondering if its something I should be concerned about and why it JUST started happening? Doctor: hello thanks for consulting at hcm..it could just be becz of plaque,,,hence proper brushing with good massage with the pulpof ur fingers,,should help,,if it continues professional cleaning of teeth with a pedodontis..hope it helps,,tc"
},
{
"id": 68814,
"tgt": "What causes painless small lump of the head?",
"src": "Patient: I'm an 18 year old female and have had a raised almost mole like tiny lump on the right side of my scalp since i was like 6, it doesnt puss, hurt to touch or brush my hair, it hasnt grown or changed throughout the years but i just wanted to make sure it isnt cancerous or anything. I have a zinc deficiancy but no other serious medical problems. Any suggestions? should i get it removed? Doctor: welcome to Health care magic.1.This type of the lumps are of bony origin, they are or benign origin.2.You don't have to worry about that those.3.The zinc deficiency is not a major issue, by having healthy died will help to over come. 4.You can not do anything it this stage, only thing you can do is confirmation by getting a CT scan.5.It is difficult to get rid of it, unless it is causing cosmetic causes.Hope it helps you. Wish you a good health.Anything to ask ? do not hesitate. Thank you."
},
{
"id": 209034,
"tgt": "Suggest remedy for panic attack, depression and confused mind",
"src": "Patient: Hello,I had a panic attack about three weeks ago stemmed by the idea of mind control and now, I have been unable to work or go to school. I am concerned about my mental health because I keep questioning reality, everything, whether I am here, what is the meaning of life, whether we are in something like the matrix? Part of me realizes this is illogical, the other part is very much afraid it might be true. I have depression and am wondering if this is a symptom of this? Will it pass? Doctor: HiThanks for using healthcare magicIn depression, sometime, patients get idea of persecution and it is part of it. In your case, idea of control or avoid belief is part of depression. In that case, you need a antidepressant with low dose antipsychotic like aripiprazole. It would help to control such kind of thoughts. Better to consult a psychiatrist for treatment. In case, you need further help, you can ask.Thanks"
},
{
"id": 217624,
"tgt": "Can cervical spine surgery cause radiating pain in neck to head?",
"src": "Patient: Nine years ago I had cervical spine surgery with instrumentation. The last three weeks I am experiencing low-grade pain in my neck--radiating up the back of my head; and, I have noticed odd tastes (metalic) and smells (burned grain like) Could all this be connected? If so, what should I do? Can't afford fees. Doctor: Ok.. there's a nerve from the top of the neck that runs up the back of the head (C-1 nerve). Problems in it cause pain in that location, often triggered by neck/head movement.Taste/smells are from the totally opposite side of the head (nose and the first cranial nerve that runs under the brain near the nose). A stretch to connect it to neck problems.X rays of the neck to establish the surgery didn't fall apart then physical therapy and/or traction and/or injections, pain manangement would be standard treatments.Nasal inflammation/infection/sinus blockage can cause changes in smell and taste, I cannot say in your particular case without an exam."
},
{
"id": 180428,
"tgt": "What causes dark spots on the tongue?",
"src": "Patient: I have two faded dark spots on right side tongue an left side, I\u2019ve seen oral surgeon who says he isn\u2019t worried because it\u2019s not raised or hard an for me to keep eye on more than a couple dentist called them Amalgam tongue from all my silver fillings as well as over production of melanin, shld I be more concerned I\u2019am arching them constantly. Doctor: Hello and Welcome to \u2018Ask A Doctor\u2019 service. I have reviewed your query and here is my advice. Dark spots on tongue that are not raised can be Amalgam tattoo if you have amalgam fillings while it can also be black hairy tongue, infection, hyperpigmentation etc..So first of all you should start cleaning your tongue with a tongue scraper.Also drink plenty of water.Eat hard foods like carrots, apples etc.Maintain a good oral hygiene.To be confirmed that it is nothing serious you should once get it checked by an Oral Surgeon.You can also send me pictured of the tongue spots so that I can advise you better..Hope I have answered your query. Let me know if I can assist you further. Regards, Dr. Honey Arora"
},
{
"id": 25915,
"tgt": "How to relieve heart murmuring?",
"src": "Patient: welcome Dr. Prabhakar appearing online for consultations. I am feeling some heart murmering for the last two days as if the blood being pumped by the heart is passing with some force sometimes. Kindly suggest the cure. I am about 84 years old but quite active in my life............Syed. Doctor: Hi, here history of murmur is of 2 days only.Is there any previous cardiac disease you have that lead to narrowing of valve of heart.narrowing of valve(stenosis) could be due to thematic fever, infective endocarditis,lupus etc. Rheumatic fever associated with involuntary movement,skin patches,arthritis etc.CRP will be helpful.FOr lupus ANA level done.Treatment is according to cause.Regular treatment for hypertension has to be taken if present.If valve stenosis severe then its repair or replacement done.If heart failure present (edema over body),take diuretic."
},
{
"id": 29320,
"tgt": "How can an E-Coli infection be treated?",
"src": "Patient: wow and thankyou to you mds I have to believe that care. ive had 3 surgerys on my left tibia in less than a year, osteomyelitis-work in lab for Kaiser in sd. my surgeon s helmers has cut the hell out of my tibia trying to get rid of the (ecoli) infection! no one has answers and cant predict my outcome,anyinput ?I would be greatful Doctor: Hello,my name is dr,Dorina Gurabardhi.I understand your concern.Some advices that I can give to you are:-Drink Clear Liquids. Drink plenty of clear liquids, including purified water, clear soups and broths, gelatin, and juices. Avoid apple and pear juices, caffeine, and alcohol.-Get Bed Rest. Minimize all activities and stay in bed as much as possible. Getting as much sleep and rest as possible is a key ingredient in recovering from any infection.-Add Foods Gradually. When you are ready to eat solid foods, stick to low-fiber foods at first. Try soda crackers, toast, eggs or rice.-Avoid fatty foods, high-fiber foods or highly seasoned foods can make symptoms worse.Hope my answer was helpful,if you have any other question I will be glad to answer,thanks!"
},
{
"id": 87283,
"tgt": "Is Mukosta safe to be taken for abdominal pain ?",
"src": "Patient: hello, may i ask is mukosta safe to be taken by my 23 year -old son because this is the medicine prescribed by his doctor for he suffers abdominal pain. Although its bearable but sometimes it alarms me for he takes time in eating. according to him there's pain sometimes it connects to his head.he has not undergo endoscopy yet .but the doctor gave him this medicine .is this okay? Doctor: is the drug spelling is correct and pls give details of pain history and relation with pain abdomen with food and episodes of pain"
},
{
"id": 163981,
"tgt": "How can constipation with pain and unusual stools be treated?",
"src": "Patient: My 3 yr old son is constantly constipated and for 3 days he has been having very minor stools come out in his underwear which are a tree green colour. When he tries to pass more he just cries in pain. I have tried parachoc before with his constipation which has worked but it says not to use it too often. Previously he didn t have the green stools. Any known cause? Doctor: Hi...what you are quoting is exactly called encopresis. This occurs when the kid has hard stools, almost similar to constipation. But what happens here is the liquid stools will seep out on the sides of the hard stools and leak out. But, the main treatment of this is relieving of constipation.Suggestions:1. Natural methods are the best to relieve constipation.2. Constipation is a risk factor for UTI3. Maximum milk consumption per day should not exceed 300-400ml4. Minimum 3-4 cups of fruits and vegetables to be consumed per day5. Toilet training - that is - sitting in Indian type of lavatory daily at the same time will help a lot.Hope my answer was helpful for you. I am happy to help any time. Further clarifications and consultations on Health care magic are welcome. If you do not have any clarifications, you can close the discussion and rate the answer. Wish your kid good health.Regards - Dr. Sumanth MBBS., DCH., DNB (Paed).,"
},
{
"id": 207394,
"tgt": "Suggest treatment for aggressive behaviour and suicidal thoughts",
"src": "Patient: hello. I am looking for a psychologist for my mother. Shes very stressed out bcoz of my family. Sometimes shes so angry she just goes out of the house and we dont even know where she goes or what will happen to her. She even gets bad head ache and sometimes bofy ache. She even says that she doesnt want to live anymore. Pls help me. I am very worried for. Doctor: She is having psychotic symptoms with depression and suicidal ideation. Can be treated by some drugs like olanzapine..sodium valproate and escitalopram.kindly visit a local psychiatrist for above drugs as these drugs are available on prescription basis only. She will be fine soon. So dont worry. Hope my suggestions helpful and thankful to you. Take care."
},
{
"id": 171207,
"tgt": "Need treatment for hip pain and blue skin with heart murmur",
"src": "Patient: I have a 4 yr old grandaughter - she has a heart murmur but , with regular checkups, has no problems with this other than tiredness. She has started to develop pain above the waist and blue mottly skin while having a shower - this is not a hot shower, just luke warm and takes a while to disappear after. We have changed products with no avail. She does get quite mottled skin at other times, but not necessarily associated with hot or cold temperatures. Any ideas? Doctor: Hi, Since your grand daughter has heart murmer and an episode of bluishness of skin, there is a possibility of congenital heart disease. In my opinion, an urgent ECG, with echocardiography should be done with x ray chest to rule out heart disease. I advice you to do the above investigations and report back. I hope this has helped you. Take care. Regards Dr Deepak Patel, MD Pediatrics"
},
{
"id": 130741,
"tgt": "Suggest treatment for talonavicular luxation",
"src": "Patient: Hi there, i am a brazilian medical doctor and suffer from cerebral palsy.While recovering from a fibular reconstruction at the contra lateral foot(the normal one), i had a lateral talo navicular luxation(the talus in the medial position compared to the mid foot.MRI:no neurovascular demage,neither condral injury.Only some calcaneus,navicular areas of bone edema at paretic foot(.Because i am not recovered from the first surgery, i have to wait some months to go to that surgery.Considering that lateral luxation is very difficult to achieve a non open reduction,probably i ll have to submit myself an artrodesis. I don\u00b4t know with it will be a triple one,or a talonavicular,maybe associated with subtalar. I have no signal of osteoarthits.i would like to know how much time i ll need weight bering and the final prognosis(moviments that will not be able to do) . I don,t practice sports ,i just need to return to work with no or mild pain.Iam a psychiatrist.Because of the first surgery i couldn\u00b4t discuss that with my ankle specialist.Sincerely yours,leonardo Doctor: Hello and welcome to \u2018Ask A Doctor\u2019 service. I have reviewed your query and here is my advice. Based on your questions it seems that you have decided not to seek any medical help till your fibular reconstruction has improved. If you do seek help right away it might be possible to reduce the Talonavicular joint with pinning and since your non weight bearing currently it won't affect your lifestyle much anyways. If you choose to wait and get TN reduced later there is a chance you might ulcerate due to intrinsic pressure on soft tissue that is being created by navicular, not only you are increasing your chance to get a fusion which might over several years cause arthritis in other joints. Locking TN joint right away will reduce motion in subtalar joint significantly. Regarding if you choose to be under TN fusion I would not recommend getting STJ fusion with it because it will limit your motion significantly. In my opinion you should under Talonavicular reduction with temporary fixation right away before you end up compromising your soft tissue.Hope I have answered your query. Let me know if I can assist you further.Regards, Dr. Samuel Parmar"
},
{
"id": 183682,
"tgt": "Suggest treatment for bad breath while having white patches along gum lines",
"src": "Patient: I have small white patches all along my gum lines and my gums bleed if I just push on them, and I am experiencing bad breath. I floss and brush and use mouthwash...this has been going on for about a year and is progressivly getting worse. I am a single mother of four and cannot afford to go to the dentist. Is there something I can do over the counter? Thank you!! Doctor: Thanks for your query, I have gone through your query.The white patch could be because of lichen planus, candidiasis, or leukoplakia. Consult a oral physician and get it evaluated. The possible cause of halitosis or bad breath could be because of the deposits over the teeth causing gum infection. Or it can be because of the pus discharge secondary to gum or tooth infection or any respiratory tract infection.Consult a oral physician and get yourself examined to rule out these conditions.If it is gum infection get the teeth cleaned once and maintain oral hygiene after that. You can use mouth wash.if it is tooth infection get the tooth restored.If it is respiratory tract infection consult a pulmonologost.I hope my answer will help you, take care."
},
{
"id": 127392,
"tgt": "Does peripheral neuropathy cause tingling sensation in the leg?",
"src": "Patient: I can t get to the doctor s office right now but I m feeling weird. I have had bad acid reflux before and am going through a bout now. But I m also having wierd electric like pain/sensations in my leg. Could it be diabetic nerve pain. I don t have diabetes that I know of my my dad developed in late in life. I m 58, 6.4, 195 pounds and very active. Doctor: Hello and Welcome to \u2018Ask A Doctor\u2019 service. I have reviewed your query and here is my advice. As per your history, it may be due to peripheral neuropathy. You can take tab. Pregabalin after consultation. Kindly do MRI back, EMG NCV test after consultation. Continue physiotherapy as advised. for acid reflux you can take tab. Pantoprazole after consultation. Take care. Hope I have answered your query. Let me know if I can assist you further. Regards, Shyam B. Kale"
},
{
"id": 72036,
"tgt": "What causes lung pain, sinus infection with phlegm and mucus in diarrhea?",
"src": "Patient: I quit smoking April 19,2011. I was getting bad lung pain on & off until about 2 weeks ago. i now have a dark yellowish phlem in the morning and sinus infection. Sometimes my right lung will make a very low click when i take a deep breath. The past few days * on & off i will get mucus in diarrhea. I am lactose intollerant and when i eat some greasy food my nose gets itchy ( guessing iam allergic) i have been under a lot of stress ( loosing my dad suddenly arpil 16 then my dog april 22nd... any idea if the mucus in diarrhea is from my body cleaning itself from quitting smoking... sinus infection... stress or ???? Doctor: Hello dearWarm welcome to Healthcaremagic.comI have evaluated your query in details .* Respiratory infection will give you lung pain , sinus infection , phlegm .* Mucus in stool is in relation with intestine infection or other inflammation issue .Wishing you fine recovery .Feel free to ask any further queries .Regards ."
},
{
"id": 120013,
"tgt": "Suggest treatment for knee pain due to gap in knee",
"src": "Patient: i am 48 years of age (female), i have pain in my knee, i visited so many doctors,most of them suggested surgery because they found big gap in my knee just in one right leg,but all them says these is not right age , we will do surgery after 60 years ,because knee surgery you can do only one time,so please guide me, thanks Doctor: Hi, The doctor would need to see your X-ray to guide you properly. Take physiotherapy, use a knee cap, give hot fomentation, take over the counter painkillers. Hope I have answered your question. Let me know if I can assist you further. Regards, Dr. Jaideep Gaver, Orthopedic Surgeon"
},
{
"id": 159884,
"tgt": "Am I suffering from kidney cancer ?",
"src": "Patient: Hi, I ve had about 3 lumps behind my left ear - ranging in size from about 1mmX1mm to about 1cmX2cm - and my doctor keeps brushing them off. I have had them probably at least a 2 years now (yes, I told him this) but he insists it s, just the shape of my head . With my random bouts of dizziness and the fact they weren t there before, I doubt this. Please help! I m only 17 but I m very worried that it could be kidney cancer as I have a lump in my kidney as well (I know this because I had an ultrasound ), the hospital said they didn t know why I had blood in my urine, and I know lymph node cancer can stem from kidney cancer!!! :S Doctor: Hello There.. Welcome to healthcare magic forum. It looks like you have a lymph node enlargement behind your ears... You need to get a detailed investigation including the removal of the nodes and pathophsiological examination of those nodes..This will help you to detect whether it is related any cancer or infection or its just physiological enlargement of the nodes.. So consult a doctor and discuss with him about your case.. Hope this helps.. Take care.."
},
{
"id": 55706,
"tgt": "Can grade 3 fatty liver be curable by healthy diet?",
"src": "Patient: Dear Doctor, My recent ultrasound result reports I have Grade III fatty liver, My doctor said its a critical stage of the disease and suggested me a strict diet plan with Oats, Boiled Vegs& fruits, Green leafs.And regular exercise. However i am very much upset about the situation,,,, Is it possible to reverse my (Grade-3) fatty liver to lower Grades or a just normal fatty liver If i strictly follow this. Please guide me. Doctor: HI i had gone through your question.I understand your concerns. My opinion is that you should not much worried about this as the fatty liver is a reversible condition. This can be achieved by simple lifestyle modifications for weight loss. in your case it takes pretty much long time to solve your problem. you have to stick to your goal very seriously with out giving it up.I would advise you to follow these instructions given below. Try losing 1 or 2 pounds per week not more than that, as it would lead to health problems. Go for cardiovascular exercises like brisk walking, running, swimming for 45min a day, 6 times a week. Strength training 4times a week.Avoid alcohol , quit from smoking.Avoid fatty foods, junk foods, deep fried foods, simple sugars, pizzas, chocolates, ice creams, soft drinks. Have a balanced and healthy diet that should include complex carbohydrates like brown rice, oats, brown bread. High protein foods like egg white, pulses, beans, sprouts, soya beans, milk, cottage cheese. Low fat foods. Use olive oil. Have 4 to 5 servings of green leafy vegetables along with all mixed vegetables. 5 to 6 servings of all fruits except banana, mango, chickoo.Drinky 3 litres of water per day.If you have diabetes keep your blood sugars under control.Get regular check ups for cholesterol and triglycerides from your doctor. Hope i have answered your question, if you have further questions i will be happy to help."
},
{
"id": 99319,
"tgt": "Does clop-gm and dk gel help to reduce red spots on underarms/thighs/buttocks?",
"src": "Patient: I had few red spot allergy of plants on my underarms,thigh & my buttock so I took treatment for little while and then I discontinued , but now those red spots had turn into brown/ blk spots n few of them are raised. My doctor prescribed me CLOP-GM 15g cream & DK gel.is it correct ? Since my Dr expired few days back I do not have dermatologist to go to. So please suggest a good dermatologist too.thank you for you help! Doctor: Yes the drugs prescribed by your doctor are correct as far as red spots are considered. Along with clop-gm and DK gel you have to take cap doxycycline 100 mg twice daily after meals with tab avil thrice daily or tab levocet twice daily after meals with tab rantac or tab famotid twice daily empty stomach."
},
{
"id": 210583,
"tgt": "What is the remedy for panic and anxiety attacks other than taking Xanax?",
"src": "Patient: hi i just moved and i have been taking xanax for 7 or 8 years now. for panic and anxiety disorder. i was able to overcome agaorophobia. but i still have the attacks. i need a local doctor but they all worry about prescribing it and i understand why. however, i need to continue my regimen Doctor: Hi,Considering that you have panic disorder with agarophobia, it would be better that you should be treated with some medication that will take care of the disorder per say like selective serotonin reuptake inhibitors, rather than only taking xanax. Xanax will help reduce your anxiety but not take care of the disorder as such. i would recommend you to meet a local psychiatrist immediately.Best wishes."
},
{
"id": 160252,
"tgt": "I noticed a lump in my neck, how to get rid of it ?",
"src": "Patient: Hi My name is Melanie, Im a 20 year old female. and have lump on the left side of my neck rite next to my collar bone . I noticed it about 3 months ago, it is hard immovable and feels like there are other small round lumps around it. Sometimes it feels larger and I can find it as soon as I touch my neck, other times I have to dig around to feel it or for someone else to. Had blood work about a month ago, came back fine showing no sign of infection however there was blood that came back in my urine test . I also have had a couple swollen lymph nodes in my neck and behind my ear. I have an ultra sound booked 2 weeks from now that I have been waiting for over a month now, however im terrified if its cancer im waiting too long. scared, and desperate for help or advice. I should also mention I weigh around 115 lbs, my diet is impeccable, I hardly drink and do not smoke. Melanie. Doctor: Hi Melanie, You seem to have a good number of enlarged nodes. Though not always malignant, one should always get a biopsy, an x-ray of the chest and a scan done even if your blood tests are normal. There are a lot of infectious and other causes for enlarged nodes so I would not worry too much."
},
{
"id": 193123,
"tgt": "Suggest remedy to overcome addiction to masturbation",
"src": "Patient: Sir, I am 15. Sir I have a problem . In my family, friend circle and school I\u2019m very sincere and talented boy.My nature is very calm and polite. I\u2019m scholarship holder of NTSE, MTSE and silver medalist of Dr. Homi Bhabha Balvaidnyanik Competition(toughest exams of India) but there is one thing that no one can expect from me. I have masturbation addiction. Last year I did it just for fun but now it has become a habit.Sir I really wanna get rid of it but can't, please suggest something as early as possible. Thanking in anticipation Doctor: Hello,Very nice adolescent you are and very well talented young man you are. You are doing well. Very good. Go ahead and improve your talent and succeed in your career. do not worry about masturbation. Masturbation s normal, natural and safe. Make sure that you do not masturbate more than 3 times a week.Hope I have answered your question. Let me know if I can assist you further. Regards, Dr. K. V. Anand, Psychologist"
},
{
"id": 221247,
"tgt": "What causes abdominal cramps during pregnancy?",
"src": "Patient: i am 39 weeks and 2 days pregnant many times i get menstrual cramps and suffer from lower back pain so is it that i am in labour and if it is so then y i am not getting it properly and is it that i need to have artifical pain or what other things should be done in order to induce labour pain Doctor: Hi there,The last few weeks of pregnancy are the most uncomfortable and also filled with anxiety and stress. And as the baby grows larger compared to the womb you can feel tightening and pain which can be mistaken for labour pains.You need not worry, as some women may get into labour by 42 weeks, that is 2 week after their due date and this is absolutely normal. You are still at 39 weeks and should not be worrying.A true labour pain will be increasing in intensity and length of time and will be from loin to groin and is usually accompanied by bleeding or breaking of waters.Do not worry, if you dont get labour pains by 42 weeks then induction of labour will have to be considered.Hope this helps.Regards."
},
{
"id": 19949,
"tgt": "Suggest treatment for high blood pressure",
"src": "Patient: Hi, may I answer your health queries right now ? Please type your query here...I have been on 12.5mg 1x daily of hydrochlorithiazide for about 1 1/2 yrs. for moderate high blood pressure 145/90. At first my blood pressure improved to 120/80 or lower. About 4 mos ago at my yrly checkup my blood pressure was 130/85. I thought that was high but my Dr. did not say anything and did not adjust my medicine. Recently I am experiencing headaches, extreme fatigue and some intermmittent bouts of dizziness. These were the same sypmtpoms I had pre diagnosis and tx. I am now wondering if my medicine should be increased to at least 25 mg daily or higher or another medicine. I also sometimes can feel my heart beat or pulse in my ears and neck when lying down to sleep or even when I awake in the morning. Doctor: Hi. Sounds like you may need to adjust your BP medications. First thing I would do is check your BP at home with a machine. If you don't have one I suggest getting one. Omron is a good brand. Best to check first thing in the AM when you are relaxed or when you feel dizzy or fatigued. Several options. One is diet. Less salt and processed foods. Try organic foods if possible. Moderate exercise may also be beneficial.If this doesn't help, then the options are to increase HCT to 25 mg daily or a combination pill such as HCT/ACE INHIBITOR. There are quite a few of these meds and they are usually the first choice for high BP as per JNC (Joint National Commission) 8 guidelines. Hope this helps."
},
{
"id": 166375,
"tgt": "Suggest treatment for cerebral palsy",
"src": "Patient: my little child aditya, when he born he was not crying and his devlopement is not reguler his mind cell damage and he became a c p child at this time at the age of 3 year he cant sit,cant seeing anything,cant hearing and cand make his nack and back ballance. then what can i do for him? Doctor: Hello....Thank you for consultation.I knowyour concern . c p child is unfortunately having brain cells damage in certain parts of brain usually at birth time,leading to nerves damage,causing delayed milestone , weakness of muscles of body ,cannot maintain balance, so cannot walk & holds things properly.most important is to start physio-therapy to prevent muscle contraction-rigidity,best time is early age as muscles are still growing, so can easily be adjusted but need to see physio DR without waste of time,do not wast money on many neurology Drs as they cannot reverse this natural damage to brain cells.,but physio-therapy is very fruitful as it can make +ve change in her life ,reverse damagebut needs regular sessions ,if child able to eat by own hands means, she/ he is not 100% dependent on others,properdiet milk/protiens rich ,home exercises -massage of limb hand/ feet can be done at home ,also speech therapy is also advised ,Although first 1-2 years are harder time for you as it needs time/regularity,but it will give you good result in long run ,you can make you child not to be 100 % dependable on others , as Cp left like this-meaning,ignored,are depedent on people for rest/whole life .Hope u got the answer , thank URegardsDr M Ibrar khan"
},
{
"id": 1672,
"tgt": "Can high body temperature cause inability to get pregnant?",
"src": "Patient: hi, my age is 28. i am married and 3 yrs completed and i dont have baby. i am having thyroid. i am having lots of tablet to get pregrant. i am having high body temperature and i can feel the heat on my body. can high body temperature be one of the reason for not getting pregrant inspite of having pregrancy tablet. Doctor: Hi, high body temperature alone can't be a reason for not getting pregnant. So, you can go ahead with your treatment of infertility. As you have taken many medicines, so now you should go for IUI. Talk to your doctor regarding this. Hope I have answered your question. Regards Dr khushboo"
},
{
"id": 42002,
"tgt": "How can PCOS with lack of mature follicles be treated?",
"src": "Patient: Hi, may I answer your health queries right now ? Please type your query here... hello... i m 24 year old woman,i have PCOS.i have been trying to concieve from past 4 months.my doc. has advised me to take siphene 50 mg in feb,i went for follicle monitoring but no mature follicle was seen.Now for the next month she adviced me to take siphene 100 mg but again everything is in vein.I m little disappointed and frustated with all this.now what next can be done to get pregnant?i have already gone for FSH,LH,Prolactine,E2 tests. Doctor: Hi, Welcome to HealthcareMagic. I suggest you to discuss with your doctor regarding using FSH injection to induce ovulation. This will increase chances of ovulation. With that you need to get follicular study. Also go for IUI in same cycle if ovulation is there. It is intrauterine ingestion of sperms . Hope I have answered your query. RegardsDr.Deepika Patil"
},
{
"id": 147621,
"tgt": "Experiencing numbness of arms and hands",
"src": "Patient: Hi. I am a 41 year old female in pretty good health. I am finding that more and more my arms and hands are becoming numb. Just last night I had a fitful night of sleep as I kept waking up from my arms going completely numb due to my sleeping position. Is there a way to remedy this? Should I be worried? Doctor: Hi,Thank you for posting your query.The numbness of arms denotes a dysfunction of the nerves in the arms.We need to exclude a diagnosis of carpal tunnel syndrome, where the median nerve gets compressed at the level of wrist. The diagnosis can be confirmed by doing nerve conduction studies.There are good medicines to treat this condition, and hence, there is no need to worry.I hope my reply has helped you.I would be pleased to answer, if you have any follow up queries or if you require any further information.\u00a0\u00a0\u00a0\u00a0\u00a0Best wishes,Dr Sudhir Kumar MD (Internal Medicine), DM (Neurology)Senior Consultant NeurologistApollo Hospitals, Hyderabad,For DIRECT QUERY to me: http://bit.ly/Dr-Sudhir-kumar My blog: http://bestneurodoctor.blogspot.com/"
},
{
"id": 64539,
"tgt": "Suggest remedy for lump on the foot",
"src": "Patient: my foot got stood on playing soccer. i took my boot off and a vein had burst. it swelled up like a golf ball. it has since gone down and the blood has spread across my foot ( normal bruising) but there is still a lump where the vein popped. do i need this looking at or will it self heal Doctor: Hi,Dear,Good Morning.Thanks for the query to my HCM Clinic.I studied your query indepth.I am concerned about the worries you have.In my opinion,you have-Resolved-Traumatic Foot-Hematoma-with subcuticular petechial heamorrhages. .I would advise you to consult a ER Surgeon-who would treat as follows-Tbr-NSAIDs,Comprssion Bangage, Rest and Elevation of Foot at Night.This would resolve in 1 wks time.If not-X-ray Foot, Clotting profile,CBC, with Surgeons Review would fix your problem.This is the remedy for your lump.Hope with this you would be relieved of your worrysome query.Wellcome again to HCM.Have a good Day."
},
{
"id": 10789,
"tgt": "Suggest remedy for hair loss",
"src": "Patient: hello! this is sharmin. i've been suffering from severe hair loss since the last 2 years... and i am a patient of hypothyroidism. i am extremely tensed about my hair loss... its getting thinner day by day... what can i do to regrow hair? and to stop hair fall? and what vitamins do i need to take to regrow hair? Doctor: Hello and welcome to HCM,You seem to be suffering from excessive hair fall commonly seen with hypothyroidism. Though your levels maybe normal , the overall hair growth and rate of growth may be affected more in your case,even on treatment.I hope your thyroid levels are in check. In your case, the hair fall ,maybe prolonged due to other factors as well, such as poor nutrition, stress, lack of rest, low iron and hemoglobin levels, hormonal changes etc. Dandruff,Chemical treatments,dyes on hair can also aggravate hair fall.For your hair to grow well, it needs sufficient nutrition.Fever, typhoid, recent illness, water change, work stress, pollution etc is a factor.Low hemoglobin/iron levels need to be checked as well.These causes need to be considered and treated as they affect hair growth.You can consult a dermatologist regarding any tests that may be required to diagnose the causes. They can assess your hair in person and advise accordingly. They may advise hemoglobin, iron, hormone levels if needed.For now you may start a Recute solution or Tress gro serum to scalp at night , along with hair supplements daily like Keraglo or Biotee capsule. This needs to be continued at least for 2-3 months to see results. Use mild shampoo and conditioner like Anaphase or Renocia for hair growthDo start with the treatment as advised and see a doctor. And remember to check your thyroid levels regularly as advised and follow treatment.Due to the thyroid condition, hair growth will be a little slower than normal.If your hair is very thin and scanty on the scalp, and medical methods do not work well for you,after a few months of observation, certain new surgical procedures can be considered such as PRP, as will be advised by your consulting doctor after observation, if they feel that it will be a good option for you. This is a later option.Also get a good nutritious diet, avoid stress,pollution, get good rest.Hope this guides you."
},
{
"id": 154194,
"tgt": "What is primary site of metastasis for stage IV squamous cell carcinoma?",
"src": "Patient: Hi, may I answer your health queries right now ? Please type your query here... Hello, my brother has stage iv squamous cell carcinoma, primary is unknown. He is having surgery on his neck to remove the tumor and send to pathology for further testing and I guess to find the primary site of his metastasis. What should I expect? I have some medical knowledge. Thank you Doctor: hi ...it can be an unknown primary with secondaries in neck. you should ideally go for a PET scan to locate the site of primary which is the best investigation. however if you cant spend that much,you should visit an ENT surgeon to get a panendoscopy (esophagoscopy,nasal endoscopy, DL scopy,bronchoscopy ) done to assess all the primary sites of cancer.you will also have to undergo blind biopsies from sites like nasopharynx and tonsillectomy which are the sites for hidden carcinoma.squamous cell carcinoma mean it will be mostly from head and neck region especially nasopharynx or oral cavity or upper GI tract.if you get evaluated in a manner the way I told you will definitely get the primary site.all the best."
},
{
"id": 58266,
"tgt": "Taken antibiotics for tonsillitis. Lab work shows elevated liver enzymes. Is it mono?",
"src": "Patient: HI Im 23 years old, a year ago I was living in south america and got a bad case of what i thought was tonsillitis. After a two week period of antibiotics was able to come back to the states and saw my dr who after doing some lab work diagnose me with Mono. Recently my lab work showed very elevated liver enzymes so i had to see a specialist who found salmonella in my system. Could the original diagnose of mono had been incorrect and all along i had salmonella? Doctor: Hi and welcome to HCM,thank you for your query.Yes, it is possible but mono is more likely isnce there was throat infection asscoiated with elevated lft. Salmonela rarely cause lft elevation unless there was blood poisoning. So I suggest to do blood tests to verify is it mono. This is routine test today. After you establish right diagnosis,the treatment can be started.Wish you good health. Regards."
},
{
"id": 192732,
"tgt": "What causes pain in testicles?",
"src": "Patient: A couple of days ago I randomly experienced a very random sharp pain in my testicles for about 5-10 seconds, none since. But after, my testicles have felt kind of strange, and feels weird when I walk. They don't really \"Hurt\" but don't feel normal, and one... Doctor: Hello,There are number of possibilities regarding your symptoms like infection, inguinal hernia, Stones and twisted testis as well. I suggest you to visit your doctor to rule out all these possibilities and treatment if needed.Hope I have answered your query. Let me know if I can assist you further. Regards, Dr. Sameen Bin Naeem, General & Family Physician"
},
{
"id": 66167,
"tgt": "What does a swollen spot behind the ear indicate?",
"src": "Patient: I have a swollen spot about an inch behind my ear and at the base of my hairline. It is very painful and causing a headache & neck ache. It is not red and does not look anything like a pimple or boil. I have been quite healthy up until this showed up. Doctor: Hi! just today I came across such a patient of course who was aged and presented with a swollen painful nodule behind his ear for 3/4 days!it is quite a common clinical presentation in day-to-day practice!In such cases I just do a careful and gentle physical examination regarding size, consistency, mobility, feel and tenderness of it; then I do a needle biopsy that generally reveal:1. a lymph node with no significant abnormality2. a small infected inclusion cyst3. an inflammatory pseudotumor4. metastatic carcinomaTherefore you need to see your doctor today for confirmation and to relieve your tension.regards,"
},
{
"id": 50408,
"tgt": "Had 2 kidney transplants, suffering from side effects. Want stem cell done. Suggestions?",
"src": "Patient: hi my name is sunil deut i had first kidney transplant 1993 living related donar my brother last kidney 14 year fail kidney 2008 second transplant desese donar 2110 donig ok but so many side effect like diabetic artritis blood pressure verry high heart pelpetation not well a toll just surviving i am just 41 year old reason for fail kidney iga nep plus reflux pl doctor help me i wanna have stemcell done ------ YYYY@YYYY --i am in australia Doctor: Hi Mr Sunil, I can understand your problems and frustrations. Till date, stem cell therapy has not been approved for the treatment of kidney failure. it is under trial. As you mentioned, your kidney is functioning well. In that case you donot need stem cell therapy for kidney. Take care."
},
{
"id": 178315,
"tgt": "Suggest treatment for diarrhea",
"src": "Patient: my daughter has been feeling very tired, achy, and has had diarrhea every time she goes to the bathroom for about 3 weeks. She can function, and goes to work. but as the day progresses she is very lathargic. sleeps more than normal. not much a fever. Doctor: You have not mentioned the age of the child which is very important. The case would different in each of the situation if it is a newborn, 6 month age or 6 years of age. If a diarrhoea continues for 3 weeks it may mean some malabsorption. The role of diet and diet change comes into play here. Diarrhoea can happen when a semi solid food or some different food is introduced as in case of a 6 month old child undergoing weaning. Lethargy is not a good sign. It might either indicate that the child is having dehydration and requires to be managed with ORS. A blood investigation and the investigation of the stool is also required."
},
{
"id": 110630,
"tgt": "Suggest treatment for back pain",
"src": "Patient: i went crowd surfing last night and i was droped about 6 or 7 feet about the ground and landed on my back with my legs flying over my head. I hit so hard it knocked me out of breathe and made me throw up. Now today my back hurts so bad..everytime i move i have sharp pains going up my back and at certain points when im walking i just want to collapse. Should i go see a doctor? Doctor: Hello, I have studied your case with diligence.As per your symptom and history there is possibility of acute muscle spasm in lower back due to injuryFor these symptoms muscle relaxant with analgesic and neurotropic medication can be started consulting your treating doctor.Till time take rest and continue analgesic available over the counter take rest, and hot fomentation. Physiotherapy like ultrasound and interferential therapy will give quick relief.You may need to do MRI spine if symptoms persist.I will advise to check your vit B12 and vit D3 level.Hope this answers your query. If you have additional questions or follow up queries then please do not hesitate in writing to us. I will be happy to answer your queries. Wishing you good health.Take care."
},
{
"id": 122069,
"tgt": "What causes pain and stiffness below the knees?",
"src": "Patient: Dear sir I am having pain and stiffness ibelow my right knee. Dr prescribed Renerve inj and sammy 400 tablets. please let me know what for these medicines are prescribed and any possibility of increaseng BP? Because after using these medicines my BP increased and is remaining in the same state. Pl advise Doctor: Hello, Renerve Plus is a multi-vitamin/multi-mineral oral supplement used for nerve damage. Based on evidence studies, this medication does not cause high blood pressure. Hope I have answered your query. Let me know if I can assist you further. Take care Regards, Dr Dorina Gurabardhi, General & Family Physician"
},
{
"id": 142919,
"tgt": "Is it safe to undergo a conventional laminectomy if fever is there?",
"src": "Patient: Hello doctor,I will be having a conventional laminectomy operation for lumbar spinal stenosis in three days, but I have had a mild temperature of 99.2 for several days. Is it safe for me to go ahead with it? I'm 70 years old and have had the preoperative required tests. My email is: YYYY@YYYY Doctor: Hello and thanks for using HCM.I have read your question and understand your concerns.The answer to your question is no, you should postpone the operation until you are fever free, since your condition is not an emergency and anesthetic drugs may increase further the fever.Tests about inflammation levels ( CPR, ESR, WBC ) should be carried out until they are normal.Hope you found the answer helpful.Take care."
},
{
"id": 83146,
"tgt": "Should i discontinue Cefalexin after experiencing side effects like ulcers and lethargy?",
"src": "Patient: Hi, I get tonsillitis alot and am allergic to penacillian also struggle with other antibiotics as they cause vomiting and diarrhoea. My GP has prescribed Cefalexin but I have a really sore mouth (ulcers,lumps) am very tired and have a couple other side effects should I stop taking it? Doctor: Hello, You should consult your doctor to switch it to some other medication, meanwhile keep yourself hydrated with adequate water intake.Take care. Hope I have answered your question. Let me know if I can assist you further. Regards, Dr. Wajahat Baig, Internal Medicine Specialist"
},
{
"id": 76628,
"tgt": "Is it to be concerned about the chest pain and numbness in hand after workout?",
"src": "Patient: After a long work out doing back flips and heavy activity while doing this i had a couple energy drinks i came home and took a very hot bath and a muscle relaxer and tried to go to sleep about 10 minutes later i started to get an ache in my chest and a tingling numbing feeling in my left hand should i be worried? Doctor: Thanks for your question on Healthcare Magic. I can understand your concern. Possibility of musculoskeletal pain is more but better to rule out heart diseases first because left chest paid and left hand discomfort are commonly seen with heart diseases. So get done ecg and 2d echo. If both these are normal then no need to worry for heart diseases. Musculoskeletal pain due to long work out is more likely in your case. So follow these steps for better symptomatic relief. 1. Avoid heavyweight lifting and strenuous exercise. 2. Avoid movements causing pain. 3. Avoid bad postures in sleep. 4. Take painkiller like ibuprofen. 5. Apply warm water pad on affected areas. Don't worry, you will be alright with all these. Hope I have solved your query. I will be happy to help you further. Wish you good health. Thanks."
},
{
"id": 86093,
"tgt": "Does abdominal pain, bloating accompanied with rectal bleeding is signs of hemorrhoid?",
"src": "Patient: Hello. I'm a 29 year old female and I've been experiencing abdominal pain, fullness, and bloating, accompanied with rectal bleeding (passing clots, which nought blood to turn the water completely red). I'm also experiencing dizziness, headaches, and fatigue. Could this all just be from a hemorrhoid? Doctor: Hello and Welcome to \u2018Ask A Doctor\u2019 service. I have reviewed your query and here is my advice. * The above mentioned symptoms can be from various reasons as - Inflammatory bowel disease as Crohn`s or others - Hemorrhoid with anemia - Systemic issue of coagulopathy or others * I recommend to undertake a primary evaluation with your consultant surgeon for proctoscopy examination and manage accordingly. Hope I have answered your query. Let me know if I can assist you further."
},
{
"id": 84903,
"tgt": "Can morphine cause solid poops?",
"src": "Patient: Hey. I was in a car accident about 6 days ago. This morning there was blood in my stool. They gave me morphine and I did poop very solidly for about two days and my anus did hurt on the last day. I also normaly suffer from hemrhoids. Is this normal or should I get it looked at. Doctor: Hi.Yes, morphine is known to cause constipation as its one of the common side effects. I advise you take plenty of oral fluids and consume more fruits or vegetables. An osmotic or emollient laxative which soften the poop is also recommended.Hope I have answered your query. Let me know if I can assist you further. Regards, Dr. Mohammed Taher Ali, General & Family Physician"
},
{
"id": 126778,
"tgt": "How can tricompartmental degenerative joint disease be treated?",
"src": "Patient: Tricompartmental degenerative change most severe involving the medial compartment with chronic maceration and tearing of the medial meniscus, full-thickness articular cartilage loss, and marrow edema. Mild to moderate joint effusion what happens next Doctor: Hi, As the first line of management, you can take analgesics like Tramadol for pain relief and exercises for symptomatic management. If symptoms persist, you can consult an orthopaedician and plan for joint replacement. Hope I have answered your query. Let me know if I can assist you further. Regards, Dr Shinas Hussain, General & Family Physician"
},
{
"id": 24432,
"tgt": "What are the symptoms for portal hypertension?",
"src": "Patient: my son is 9. he has portal vein hypertension blocking outside and inside the liver. he has an enlarged spleen and platelets in the 60s. due to the severe nature of the blockage he is not a candidate for TIPS or splenatic shutning. We are currently undergoing esphogial banding every 4-6 months for the vericies. wondering your opinions on a partial speen embolization??? and what does the next few years look like for us??? Doctor: Thanks for your question on Healthcare Magic. I can understand your concern. Yes, you can definitely take chance by partial spleen embolization. You can actually go for therapeutic splenectomy. Also enroll in liver transplant waiting list because he is ideal candidate for liver transplant. Future complications are likely in his case. So you can expect 1. Hemetemesis 2. Intestinal bleed 3. Lower gastrointestinal bleed (hemorrhoids) 4. Hypoproteinemia and infection 5. Ascites 6. Hepatic encephalopathy 7. Hepatorenal syndrome, Hepatopulmonary syndrome etc. So consult gastroenterologist and plan for liver transplant as early as possible. Hope I have solved your query. I will be happy to help you further. Wishing good health to your son. Thanks."
},
{
"id": 112598,
"tgt": "Lower back pain, unable to lie down. How can I help myself?",
"src": "Patient: I was on my feet for three days. And now I have lower back pain where I can't stand, sit, or lay down with out it hurting. I was laying down and turn wrong and I had a sharp pain up my whole back. I couldn't move for a little bit. Now I can't really do anything. What could have caused this, and how I can I fix this pain. Doctor: Could be some acute sprain or even a disc problem of spine. Get evaluation by orthopedic specialist who on physical tests can determine the origin of pain and whether radiological tests are required for diagnosis..In mean time take combination of methacarbamol 750 mg and ibuprofen 400 mg thrice a day after meals,, apply hot packs, take firm mattress to lie down and maintain the most pain free posture on lying down in any position."
},
{
"id": 33761,
"tgt": "What are effects of getting a dog bite treated after a week?",
"src": "Patient: hello doctor, last week a dog bite me in hand, that's my friend dog, but that time i dont injured that means there is no blood bleading in that area, and am washed my hand with sope, but am afried that incident so am take treatment after 8 days. if any problem affected for me Doctor: Hello dear user!Welcome and thank you for asking us on HCM!Is your friend's dog vaccinated?If yes it doesn't make any problem in general. Anyway the best thing that should be done in these cases is the vaccine against the rabies and tetanus. If you have cleaned your hand properly and the wound was't deep the risk is very very small. I wish you a good health!Best regards!DR ERIOL."
},
{
"id": 66591,
"tgt": "Suggest remedy for lumps in rib cage",
"src": "Patient: My fiance has a hard lump under lweft rib cage. He has been feeling nauseus and gets sharps pain and sometimes feels out of breath. Also heartburn what.could this be or could we try he has no insurance? He is feeling bettee today but the area is sore and im worried. Doctor: HiWelcome to hcmHard lump under ribcage mostly is some bony growth. But soreness is not seen in common bony lumps unless some infection is there. You should see a surgeon for confirmation of diagnosis. Chest x ray should be taken and one needle biopsy is required. Soft lump with soreness may be tuberculosis of ribcage. Regards."
},
{
"id": 103867,
"tgt": "Having asthma, depression. Taking fostair, duloxetine, Zomorph, rivaroxaban. Treated with omeprazole. What are the risks and interactions between all these drugs?",
"src": "Patient: I am taking fostair for asthma, duloxetine long term for depression, Zomorph 60mg daily recently reduced from 80mg daily, with naproxen recently added in to get me off opiates, I have been on opiate drugs long term also for hip and back trouble and have just had a hip replaced, I am also taking rivaroxaban. In the past I have been endoscoped and gastric lesions were found and treated with omeprazole. It is documented in my gp's notes that I have abnormally low blood pressure and that I buise easily. I am bruising easily again and was treated last week for a suspected blood clot, but I am worried about a) interactions between all these drugs I am taking and b) the risk of a bleed somewhere and c) an asthma attack. Can you advise me what are the risks? Generally I am feeling just unwell weak and a bit wobbly. The hip replacement was done on 3rd April this year and we think the other one will have to be replaced soon, i am female aged 46 Doctor: Hello, Thanks for the query to H.C.M.Forum.Here I want to tell you that ZOMORHP tablets causes respiratory depression and in asthma there is respiratory depression.Fostair's ingredients are beclo+ antihistamine and when this drug is to be taken with antidepressant (duloxetine) than there are chances of cardiac arrhythmias ,so be aware about that.You are also using anticoagulants along omeprazoel.Omeprazole enhances the effects of rivaroxaban because it( omeprazole reduces excretion of rivaroxaban, so keep this thing in mind. 2.Risk of bleeding is there because of interaction of these drugs.3.An asthmatic attack's chances are more due to drugs interaction.So please take drugs in such a way that at least 3-4 hours gape remains in between them.Good luck , hope you will recover from all diseases.Dr. HET"
},
{
"id": 118402,
"tgt": "Can lower platelets count cause low energy levels?",
"src": "Patient: Hi I am a 65 year old male . My platelets are consistently 15 points lower than normal. My doc says so long as they are at a consistent level It is ok. I am consistently tired and have low energy levels. Is this ok? If the platelets were higher would I be less tired. I am fairly active? Doctor: Low platelets generally are not the direct cause of fatigue. It is usually only when low platelets lead to low red blood cell count that platelets play a role in fatigue. Make sure your other blood counts are within normal limits & follow up with your doctor regarding the severity of platelet counts."
},
{
"id": 116562,
"tgt": "What causes low WBC count and fatigue?",
"src": "Patient: My 75 year old brother-in-law has a low white blood count, chief complaint is fatigue. Otherwise, feels fine. Originally his family doctor thought the low WBC was due to his medicine for history of seizures and changed meds. Hematologist did a bone marrow biopsy on Friday to rule out aplastic anemia or leukemia and he will get the results on Wednesday. What else could cause his symptoms? He has had heart bypass surgery and non-cancer colon surgery about 8-10 years ago. THANKS! Doctor: Hi,Thanks for asking.Based on your query, my opinion is as follows.1. Only WBC count increase does not indicate aplastic anemia. 2. All the cell counts should be reduced for a possible diagnosis of aplastic anemia.3. Bone marrow biopsy could show occupying lesions. However, fibrosis or other causes require evaluation.Hope it helps.Any further queries, happy to help again"
},
{
"id": 52567,
"tgt": "Can Amlycure DS be taken along with other jaundice medications?",
"src": "Patient: HI i am suffering from jaundice. The billirubin is 5 and i am being treated from last 2 months there is 20% recovery. I want want to know that along with the allopathic medicine can i start taking amlycure ds for recovery. I was a heavy drinker and my age is 30. Doctor: Hello and Welcome to \u2018Ask A Doctor\u2019 service. I have reviewed your query and here is my advice. You can take Amlycure D.S along with your routine jaundice drugs. They do not interact with each other or affect the efficacy one another. You have to stop alcohol consumption at the earliest to prevent further liver damage. Hope I have answered your query. Let me know if I can assist you further."
},
{
"id": 176748,
"tgt": "Suggest treatment for mild fever and vomiting",
"src": "Patient: My 5 year old son has mild fever for the past 3 days and is continously vomiting also - almost twice a day for the last three days. His esr is 36 and crp is 24.Neutrophils is 84 , lymphocytes is 12 and SGOT/AST is 84 IU/L. Rest of the tests are okay.We are worried , kindly suggest. Doctor: Hi, the report suggests, raised ESR and CRP which points towards infection. Neutrophilia suggest acute infection( possibly bacterial). AST is deranged which suggest an acute infection causing derangement of liver function. The child might be suffering from Enteric fever(Typhoid) or Acute viral hepatitis. Get a serum Widal done(although not very sensitive in first week) and viral markers like Anti-HAV, HBsAG, Anti-HCV,Anti-HEV. Consult a physician as the child might require antibiotic course in form of third generation cephalosporins."
},
{
"id": 127775,
"tgt": "What causes lower leg pain and numbness in the foot?",
"src": "Patient: My husband has left lower leg pain in the front of calf by ankle,no swelling . Foot is numb some of the time . On warfarin 4 mg for AF due to complications of sepsis/influenza B was on respirator 16 days , 7 months ago Pain started a few hours ago . Has been back to working 1 month and is on feet after a 6 month recovery . He is not sure if he banged it . no pain in back of knee . no other symtoms Doctor: Hello,Does his profession require long-standing?Is he a smoker?. If he requires long standings in his profession, this should be reduced. He should quit smoking if he is a Smoker. And also get this investigation done and review with a general surgeon. Investigation: Arterial and venous doppler both lower limbs.Hope I have answered your query. Let me know if I can assist you further. Regards, Dr. Mohammed Abdullah"
},
{
"id": 150648,
"tgt": "MRI shows disc bulge causing indentation of thecal sac, narrowing of neural foramen. Solution?",
"src": "Patient: Hi Doctor.. Good day!!! I happened to get my MRI done due a neck pain and stiffness . Impressions are as follows: a. Minimal posterocentral and right paracentral disc bulge at C5-6 level causing subtle anterior indentation of thecal sac , mild narrowing of right neural foramen. No significant nerve root compression. b. Mild posterocentral and right paracentral disc bulge at C5-6 level causing subtle anterior indentation of thecal sac, mild narrowing of left neural foramen causing mild indentation of left exiting nerve root Please advice whether this can be cured ? what are the do s and don ts.. Can i swim ? and ride bike ? Appreciate your quick response.... Thank you.... Doctor: H, Thank you for posting your query. I have gone through your detailed MRI reports of the cervical spine. It is reassuring to note that there are no serious issues. Findings suggest mild disc bulges at multiple levels, leading to compression of nerves in the neck region, which travel to shoulder and arms. You should do regular neck exercises. Avooid carrying too much weight on shoulders or over the head. Swimming is fine. Bike riding for shorter distances and without too much jerking is fine. Please get back if you require any additional information. Best wishes, Dr Sudhir Kumar MD (Internal Medicine), DM (Neurology) Senior Consultant Neurologist Apollo Hospitals, Hyderabad, My personal URL on this website: http://bit.ly/Dr-Sudhir-kumar My email: drsudhirkumar@yahoo.com"
},
{
"id": 150954,
"tgt": "Sudden fall from bed causing a red scratch on head. Scheduled for radiation post recent diagnosis of benign brain tumor. Fall due to tumor?",
"src": "Patient: Last night my husband, 75, fell out of bed for the first time since a child. He just rolled out. No dreams involved, he said. Just found himself on the floor. He made no sound, just rolled off. He has a very slight red scratch on the right side of his head but no pain. He was recently diagnosed with a benign brain tumor and Monday, Jan. 28, he will begin prep for radiation . Could the tumor cause this falling out of bed? Doctor: Hi, Thank you for posting your query. Most likely, your husband had a seizure (fits) during the night and fell off the bed. Since, he has no memory of falling on the ground, it was most likley a geenralized seizure (with loss of consciousness). This seizure can be due to the benign brain tumor. He ideally needs treatment with anti-epileptic medications. Please discuss this with his doctor. Best wishes, Dr Sudhir Kumar MD DM (Neurology) Senior Consultant Neurologist"
},
{
"id": 688,
"tgt": "How can PCOS with excessive bleeding be treated for a successful pregnancy?",
"src": "Patient: hai madammm myself divya last 6 month i have excessive bleeding and i consult a doctor , as per advice i done usg pelvis its diagnosed as pcod for both ovaries now i have excessive bleeding but no clots but periods are regular am on metformin tablet also last 2 months planning 4 a baby but till now am not pregnant shall u give one suggestion? Doctor: Hi, I think you should take some medicines like clomiphene for the growth of your follicles and track your follicles growth by repeated ultrasound and when your follicles is more than 17 to 18 mm, take injection for rupturing the follicles. Be in contact with your husband every 2 to 3 days after your periods stop. Take progesterone for next 2 weeks. Do a urine pregnancy test at home after that. In pcod, the problem is with ovulation. So, you need medicines for ovulation."
},
{
"id": 179056,
"tgt": "Can combiflam syrup given to a toddler for recurring fever and runny nose?",
"src": "Patient: Hi, My baby is below 2year and got abrupt fever (more than 101 degree) since last 4 days and in last days she got nose running badly. I ad given Maftol P syrup 2.5ml (when fever comes) since last 4 days. But se is not showing any sign of improvement. Please suggest what to do? Should i need her to give combiflam syrup? Doctor: Thanks for posting on HealthCareMagic. I understand the cause of your concern. Most of fevers in children of such age are caused by certain viruses. Viral infections are generally characterised by high rise of temperature as in your daughter's case. Generally these also have an allergic component as exhibited by the running nose. Such fevers last for 5-10 days before they resolve on their own. Antibiotics are ineffective in such cases. All you have to do is to keep the temperature in control. This can be done either by cold sponging or by use of medicines. Meftal P contains paracetamol. Combiflam additionally contains ibuprofen and can reduce temperature more effectively and can be given. Give some antiallergic like levocetirizine if not given already. Cold sponging or bathing the child are effective methods and should be resorted to.Hope that helps. Feel free to revert back in case of further queries if any."
},
{
"id": 51661,
"tgt": "What is the need of giving insulin before a kidney surgery and is it required after surgery also ?",
"src": "Patient: Todays my father operation, he has stone in kidney , surgery which to be done is Percutaneous Nephrolithotomy so i wann know is there any kind of risk or danger in this sugery or its common and how much time it will take and one other thing is that he has sugar since 4 or five years and he has never taken insulin but yesterday when sugar report came sugar was 156 so why doctors are giving insulin before surgery when i talked to them they said it is to be given whether sugar is at normal and after surgery it will be continue for 3 days...so i wann know that insulin can be dangerous or it will den become habbit to give insulin regularly after surgery.....reply soon Doctor: the recurency of ranal calculi after operation is 99%.if u hv operation done ,than please u start the homeopathy medicine which will stop the recurrence of the stone formation.ur father should take at least 10 lit liquid / day.i m suggesting medicne please u start . thuja 1m (2 dose) take in night calcarea renalis 30 4 drops tds fragarai 30 4 drops tds hydrangea-Q 20 drops in a half cup water BD"
},
{
"id": 35004,
"tgt": "Can decreased platelets be associated with low grade fever?",
"src": "Patient: My husband has been on Gemzar for pancreatic cancer. He has not had it for about two weeks due to his low platelets, 72,000. He now has a low grade fever. His WBC and lymphocytes are fine. Can the decreased platelets be associated with a low grade fever? Doctor: Hello dear,Thank you for your contact to health care magic.I read and understand your concern. I am Dr Arun Tank answering your concern.No, low platelets has nothing to do with the low grade fever.There are many reason for low grade fever. But looking into your case I think the reason for low grade fever is the cancer cachexia.Neoplastic syndrome which is seen along with the fever can cause such a low grade fever along with other symptoms.So you can take the tablet paracetamol under your doctors guidance whenever you are having fever. It will drop down your fever immediately.You can later show this to your doctor and diagnosed yourself.Cancer and its therapy can cause immunosuppression so maintenance of good hygiene can put you away form the other known and unknown infections.I will be happy to answer your further concern on bit.ly/DrArun.Thank you,Dr Arun TankInfectious diseases specialist,HCM."
},
{
"id": 29016,
"tgt": "How can tonsillitis be treated?",
"src": "Patient: About 2 weeks ago I was told by my doctor that I had tonsillitis and prescribed me medication. After about i week I noticed no change so I was put on a higher prescription. I am now complete that prescription and that pain has gone away but my tonsils are still swollen and still have the white spots on it, should I book an appointment with my doctor? Or wait it out? Doctor: Hello,Persistent swelling and white spots are suggestive of persistent infection. You will need another antibiotic that is chosen according to culture and sensitivity results. Please be advised to follow up with your primary provider.Hope I have answered your query. Let me know if I can assist you further.Regards, Dr. Lekshmi Rita Venugopal"
},
{
"id": 160169,
"tgt": "Does cancer responds to mantoux skin test ?",
"src": "Patient: does cancer responds to mantoux skin test Doctor: hi mantoux is done to identify tuberculosis infection in body and skin it is bacterial infection cancer does not respond to mantoux test it has no relation to it ya,it is possible to have negative mantoux test due to low immunity"
},
{
"id": 192362,
"tgt": "What causes infection in scrotum?",
"src": "Patient: hiya been to my doctors said have infection in my ball sack, hurts like mad put me on flueloxacillin tablets, asked me stay of work wants see me 6 days time see what needs doing sighn me off work hurts but was to scared see doctor past few weeks carried on working lots lifting etc thxs tezz Doctor: Hello, You have to consult your physician to check whether infection in scrotum ( orchitis, epididimitis or funiculitis) is subsided or not. If no improvement heay change the the antibiotics and treat you accordingly. Hope I have answered your query. Let me know if I can assist you further. Take care Regards, Dr. Penchila Prasad Kandikattu"
},
{
"id": 67396,
"tgt": "Suggest remedy for a lump in the vasectomy surgery site?",
"src": "Patient: I had a bi-lateral vasectomy on the 26th of last month and I developed a lump around the incision site. I was told by my doctor that it was a hematoma and he gave me antibiotics in case there was also an infection. As of today the inflammation has lessened but the hard lump around the incision site is still present? One, is this typical and two, should I be applying ice or heat to the lump in order to help it heal faster? Any suggestion would be greatly appreciated. Doctor: Hi,Having some hematoma on the site is common and nothing to worry.As you have completed antibiotic ,there is no risk of having infection.you can apply ice pack for few days.Ok and take care."
},
{
"id": 22245,
"tgt": "What causes faster heart beat?",
"src": "Patient: hi last night when i went to bed I was almost asleep and my heart quit beating for a second.. I have heart problems and this happens sometimes..could you tell me why this happens?? I am 60 yrs old 5 ft 4 in. and 120 lbs. I have heart prob, and am a kidney transplant patient.. Doctor: DEAR USERTHANKS FOR CONSULTING AT HCMI UNDERSTAND YOUR CONCERN.. INTERMITTENT MISSED BEAT LIKE SENSATION CAN OCCUR EITHER IN VENTRICULAR ECTOPICS OR INTERMITTENT ATRIAL FIBRILLATIONI WOULD ALSO LIKE TO KNOW AS TO WHAT IS THE EXACT HEART DISEASE YOUR SUFFERING FROM AND THE MEDICINES YOUR ON FOR YOUR KIDNEY TRANSPLANTAS CERTAIN MEDICINES CAN ALSO PRECIPITATE THIS SYMPTOMSI WOULD ALSO ADVISE YOU TO CONSULT A CARDIOLOGIST AND GET A 24HR ECG MONITORING DONE..HOPE I HAVE ANSWERED YOUR QUESTION.. YOU CAN MESSAGE ME FOR ANY FURTHER CONCERNS"
},
{
"id": 211113,
"tgt": "Will loosing blood in stool cause stress and sleeplessness ?",
"src": "Patient: hello i occasionally have blood in my stool and am having severe trouble sleeping, only being able to when my body literally runs out and cant go any longer with out. i can end up staying awake for a couple of days at a time and when i do finally sleep it is usually only for a few hours..... i am under a lot of stress in not being able to find a job perhaps this is related. could the mystery blood be a culprit of exhaustion, if so are there any non pharmaceutical remedies for sleep loss... thank you Doctor: hithanks for using healthcare magicBlood loss could lead to anemia which can cause easy exhaustion. For this u should get urself properly evaluated to find the cause and get the treatment accordingly. Consult a gastroentrologist for further help. For sleeplessness, if u dont want to take sleeping pills, there is some relaxation exercise like JPMR that would help u to get sleep and keep u calm even under this stressful period.Thanks"
},
{
"id": 8375,
"tgt": "How can dark underarms be treated?",
"src": "Patient: hello, my name is liza and i have dark underarms, is not genetic cause my mom doesnt have it darker, i shave before and now i m waxing but its the same i thing they are getting darker than before, can i do a chemical peeling for my underarms? or what are the others options that i have.. thank you so mucho in advance Doctor: Hello Liza,Skin peel is a good option for Underarms pigmentation, for quick results.you may require 5-6 sessions.You can also use Retinoin based creams. Kojic acid, Hydroquinone also help in decreasing the pigmentation.Keep it covered. Avoid sleeveless dress in daytime."
},
{
"id": 130782,
"tgt": "What causes pain from thighs to calf?",
"src": "Patient: I get excruciating pain emaniting from the top of my thigh going right down to the calf of my right foot.This happens particularly after walking a while or simply standing.sometimes it goes away after a brisk walk.i get numbness in my toes while sleeping at night & have continuous pain in my right foot while asleep.i have a fused L5 n a slight problem in L3/L4 Doctor: Hello and Welcome to \u2018Ask A Doctor\u2019 service. I have reviewed your query and here is my advice. Radiating pain in leg may be attributed to sciatica where nerves are impinged in between their route. Try having some physiotherapy done and exercises do help. Don't do extreme range of movements at knee or hip. And use lumber corset while travelling. It will surely help in the condition.Hope I have answered your query. Let me know if I can assist you further.Regards,Dr. Harsh Swarup"
},
{
"id": 190144,
"tgt": "Aching after getting tooth extracted. infection?",
"src": "Patient: i had a tooth extraction 3 days ago one of my back molars,my jaw is still aching i had tooth pulled as the dentist said there was a likelihood that there was an abscess under the tooth, the socket looks white , am doing the salt water rinces but the aching is not going a way and can still be felt after taking tramadol or panandol ,shouls i be worried ,perhaps an infection? Doctor: Hello there...The first three days of pain is absolutely normal ...you need not worry about it.... You need to have a check on the duration and intensity of pain...is it the same since the day of removal or has it come down? increase in the pain intensity is factor to worry..Pain for more than 4 days with increased intensity could be a dry socket. Infection would set in only after 1 week's time....if the sterilization procedures were well maintained at your clinic infection wouldnt set in so fast...the abscess your doctor was talking to you would get cured by the antibiotics and atmospheric oxygen...Consult your dental surgeon in case of pain even after 4th day...."
},
{
"id": 198239,
"tgt": "Noticed a lump on top of the ball sack of the teticles",
"src": "Patient: Hi I found a lump/bump on towards the top part of my nut sack. It is visible sometimes and it doesn t hurt to touch. It is not attached to my testicle. It kinda is on its own and is only visibile when my nuts sag. Not sure if this is relevant but I currently have mono. No pain really just occasional discomfort for a couple seconds. Doctor: HelloThanks for query .A painless lump on the scrotal sac could be mostly a Sebaceous Cyst .The scrotal skin is rich in sebaceous glands and hence prone to get more sebaceous cyst due to accumulation of sebum beneath the skin. Normally they fade away without treatment however they need to be treated if increase in size or get infected. Take broad spectrum antibiotic like Augmentin along with anti inflammatory drug like Diclofenac twice daily.If does not subside within a week you will need to consult qualified General Surgeon for clinical examination to confirm the diagnosis and further treatment. If needed he may advise you to get it excised in Toto (Completely along with the sac) .Dr.Patil."
},
{
"id": 122277,
"tgt": "How safe is using catheter for hip replacement surgery?",
"src": "Patient: Yes, I am scheduled to have a hip replacement surgery on 4/9/12, and i am very anxious. I am dredding the surgery, and one reason is the use of a catheter. I have never had one in the past and do not want one now. I am afraid that this may be a reason that I will walk out of the hospital and not have the surgery. Doctor: Hello, The catheter is a simple and safe procedure and nothing to worry about it. As you have scheduled for surgery, it will be done after anesthesia only so that you won\u2019t feel any pain also. Hope I have answered your query. Let me know if I can assist you further. Take care Regards, Dr Shinas Hussain, General & Family Physician"
},
{
"id": 142320,
"tgt": "Suggest treatment for memory loss and lump on the head",
"src": "Patient: I have developed two indentations and a lump on the top of my head. The left one is getting longer - started off at approximately 1 in long and is now 2 and a half and is starting to go past my hairline. I don t experience headaches but have periodically experienced memory loss. I have had a ct scan which showed nothing and have recently had an mri scan which has showed nothing except a small cist in the sinuses. should I be worried. I have seen a neurologist, a neurosurgeon and a plastic surgeon. I am 53 could it be something to do with hormones and/or diet. I have also experienced considerable weight gain in the past 15 months which is when these indentations started Doctor: Hello!Welcome on Healthcaremagic!Your symptoms could be related to vitamin D deficiency. The indentations could be related to a lack of calcium. For this reason, I would recommend consulting with your doctor and checking your vitamin D levels. It would be advisable to check also your thyroid hormone levels, to exclude possible thyroid gland dysfunction. A follow up with your neurologist every 6 months, by performing cognitive tests is necessary to investigate for any possible dementia. Hope you will find this answer helpful!Best wishes, Dr. Aida"
},
{
"id": 144449,
"tgt": "What causes headaches with dent on the skull?",
"src": "Patient: hi I have a dent at the front of my skull I have had an xray on my skull, the radiologist said to me that I would need an MRI scan and commented on the depth and width of the dent. I called the doctor and she had on my xray nothing further required. although I am very happy nothing is technically wrong I have a gut feeling all is not right I have had headaches for 6 days straight, most are dull aches but there has been 2 which has been pretty intense do you have any recommendations? Doctor: Hello. Thanks for asking from HCM.I can understand your concern. The dent over skull can be due to: Old trauma : Bone moulding which occurred in childhood to give shape to skull: Age related degeneration like sclerosis and lysis. In your case, this may be the reason: Another cause can be bony lesion. It has been already ruled out by your doctor. But still you have doubt, you can go for MRI brain to relieve your tension.Regarding your headache, it is most probably \"tension headache\" as it is dull aching, occurring for few days. It can be confirmed by evaluating your symptoms like headache holocranial, causing band like sensation around forehead, increases in bright light/stress. To relieve it, you can follow few tips: Avoid stress both mental/physical: sleep 8 hours at least daily: Avoid bright light: while working on laptop/computer, take break after very hour: Head and neck massage: Do some fun activities: You can take painkiller like Diclofenac sodium after proper prescriptionStill no relief, you can consult doctor. Hope it will help you. Take care. Thanks."
},
{
"id": 147071,
"tgt": "What is a white spot on the left side of my brain?",
"src": "Patient: Hello, I am a 44 year old male who, until relatively recently, has been very fit all of his life, playing various sports, particularly football until only a couple of years ago. My problem started at the beginning of 2009 when suddenly I lost all vision in my right eye. There was no pain and no warning signs this was going to happen as I had no previous health problems. This lasted for about 3 mins until the vision returned as quickly as it went.I had a number of similar episodes over the next few months including what I can only describe as a curtain dropping down in my eye (always the right eye). I was examined by various specialists and was diagnosed with Amaurosis Fugax and placed on medication (aspirin and ezetimibe). Afterwards I felt that my physical strength had diminished a little but still felt relatively fit.I was free of any further symptoms for 3 years until a similar episode of loss of vision in September 2012 and after further examinations my medication was changed ( chlopidigrel and rusovostatin). I felt that I was getting tired very easily and was finding it hard to concentrate at times. Again I was symptom free for about a year until I suffered a further loss of sight in my right eye (October 2013). Not only did I lose my sight, I also lost coordination in my legs, I had 'tingling' in my right fingers and leg. I was admitted to hospital where I was told I was having a stroke. At this point I felt completely drained of all energy, I was sleeping most of the day, I had no strength in my arms or legs and was extremely 'moody'. Over the next few weeks I underwent further examinations including a transoesophogal echocardiogram and it was discovered I had a Patent Foramen Ovale (PFO) which it was explained is a 'hole in the heart' and was probably where small blood clots are forming. I underwent an MRI scan recently and it was found I had a 'white spot' on the left side of my brain which was explained May be a restriction in the blood flow. I have been referred to a cardiologist and a neurologist for further consultation but I have to wait a significant time before my appointments. I am extremely worried as I have never had anything like this before, I am not allowed to work (I am a police officer of 23 years with minimal sickness) and do not know what is likely to happen.Any advice would be welcome.Kind regards,Eamonn Doctor: Hai it is nice that your narration of symptoms is so clear.Initially your symptoms were due to amaurosis fugax ie a trasient loss of vision.you were advised aspirin, clopidogrel to prevent clotting of blood by their antiplatelet action and also rosuvastatin for cholesterol lowering effect.your symptoms of defective right eye vision weakness and paresthesias in right upper and lower limb weaness suggests a lesion in the left brain.the whitedot in mri may be due to infarct of the brain,or a granuloma or a tumor.dont worry about that white lesion.just follow doctors advise.please do regular physiotherapy and use medication regularly.avoid smoking alcohol and fatty diet.dont worry about patent foramen ovale.please consult cardiologist and he may advise devise closure procedure if required."
},
{
"id": 140841,
"tgt": "Suggest treatment for functioning of the motor nerve",
"src": "Patient: Iam a paralysed person,leftside taking homeopathic medicine I am not getting proper movement & strenght in y hand.I do all my ork,Office house work,marketing etc. How motor nerve will start fuctioning so that I get myself cured.Workship,Work Exercise ig going on from last 3 years Doctor: Hi, If you've been treating yourself to this point using homoeopathic means and things are not working very well then, I would recommend you see an allopathic doctor for a workup and suggestions to help you regain function and strength. Physical and Occupational Therapies would be highly recommended. Unfortunately, no medication exists (homoeopathic or otherwise that is known to \"kick start\" motor nerves into working again that have been damaged from a spinal cord injury). Hope I have answered your query. Let me know if I can assist you further. Regards, Dr. Dariush Saghafi, Neurologist"
},
{
"id": 124367,
"tgt": "Does surgery on shoulder and lower back cause fatigue?",
"src": "Patient: I was in an ATV accident about 3.5 weeks ago, I had surgery on my elbow, road rash ony shoulder and lower back, a hematoma on my left buttock and a dislocated clavicle, is it normal for me to be fatigued most of the time, I m experiencing several periods during the day when all I can do is lay in bed because I am so tired. Doctor: Hi, In my opinion, it is not normal to be fatigued most of the time following this kind of injury. I suggest you join an aggressive rehabilitation programme to take care of your fatigue. Hope I have answered your query. Let me know if I can assist you further. Take care Regards, Dr Gopal Goel, Orthopaedic Surgeon"
},
{
"id": 124866,
"tgt": "What can cause pain and stiffness behind the knees and thighs?",
"src": "Patient: I have pain and stiffness in the rear of my knees and thighs. I want to an Orthopedist and he gave me a cortizone shot. The pain and stiffness extends from my buttox to the rear of my knees in both legs. It often becomes difficult to stand from a sitting position especially over an extended period of time sitting. The Orthopedist seems to think that this is stemming from my back but I don't have back pain Doctor: Hello, As a first line management you can take analgesics like paracetamol or aceclofenac for pain relief. If symptoms persist, it is better to consult a physician and get evaluated. Hope I have answered your query. Let me know if I can assist you further. Regards, Dr. Shinas Hussain, General & Family Physician"
},
{
"id": 77490,
"tgt": "What causes tightness in chest?",
"src": "Patient: tightness in chest I'm having this tightness after just getting out of the shower. I was testing on my right arm and when I breath it was a mild tightness. I sat up and itbseems to have gone away.when something like this happens I think mild heart attack or stroke. other then that what could it be.nite I ate a salad from McDonald's and four wings from KFC. 2 or 3 hours prior. Doctor: Hi thanks for asking question.From eating outside food you might have got viral infection and it can lead to headache and muscular pain Like constitutional symptoms.your chest tightness might be because of that.Simple analgesic can be taken for it.Hot compress also helpful.If congestive symptoms like blocked nose present steam inhalation and antihistaminic useful.Second it could be because of heavy weight lifting or improper bed posture.If still your chest tightness increasing then chest x ray and electrocardiography might be done.Wish you good health"
},
{
"id": 185626,
"tgt": "What causes wet feeling on the left side of mouth?",
"src": "Patient: Feeling wetness, like water sprinkles on the left side of my mouth. It is on the side of my lips on the outside not in. Its been going on for a couple of days & when I go to feel the place it is dry. It feels like water sprinkles hitting the area. It does it both inside and outside of my house. Doctor: Hi,Thanks for posting the query, For when did you get such feeling? can you elaborate the symptoms? Is there any other symptoms like tingling, loss of sensation? is there any carius tooth in the region?Take care!"
},
{
"id": 65120,
"tgt": "Suggest treatment for a lump outside vagina",
"src": "Patient: I am 18 years old and i think i have a sabatious cyst. i have had a cyst in my finger before and a lymphoma on my back also. I have a a under the skin ball on the outside of my vagina. Can I go to the doctor without a parent?? I do not really want anyone knowing because it is embarrissing. Doctor: Hi, dearIhave gone through your question. I can understand your concern. You may have sebaceous cyst or other benign cyst or soft tissue tumor. You have history of lymphoma so it ia also a possibility. You should go for gynecological examination. If needed go for biopsy of that lump. Then you should take treatment according to diagnosis. Hope I have answered your question, if you have doubt then I will be happy to answer. Thanks for using health care magic. Wish you a very good health."
},
{
"id": 204922,
"tgt": "What causes anxiety attacks while sleeping?",
"src": "Patient: It s been 3 weeks...I sleep for approx 40 mins then wake up gasping for air and having an anxiety attack. I HAVE TO get up and walk around. Once I calm down I fall back asleep...for about 40 min...then it all happens again and again all night. I m sleep deprived and exhausted. I have a CPAP. Doesn t help. I ve tried melatonin...doesn t help. Doctor: in my opinion it looks like panic attacks check your thyroid functionsyou would need anti depressants and small dose benzodiazepines"
},
{
"id": 199149,
"tgt": "What to do for semen leakage and nightfall?",
"src": "Patient: Hello Sir, I have semen leakage daily and moreover I have also the problem of nightfall. Even in day time also, my semen get leaking everytime. I am 21 years old male but don't even have moustache and beard. My body is still underdeveloped and I can not concentrate on my studies. Please help me Doctor: HelloNocturnal ejaculation in a common condition and it may be due to over excited state,inadequate emptying etc.You should avoid provocative literature and videos.You should try to be calm and try to engage yourself in your job.Spontaneous leakage may also be due to straining etc.It is also important to evaluate for constipation.You should drink plenty of water and take lot of fruits and vegetables in diet.It is not a serious finding and he will be fine soon.If findings persists the you may need proper evaluation.Get well soon.Take CareDr.Indu Bhushan"
},
{
"id": 86832,
"tgt": "What causes lower abdominal pain and chronic fatigue with nausea?",
"src": "Patient: we were desperately trying to find out the cause of these symptoms, if you could help us please! Symptoms are lower abdominal pain, achy pain, number 6 on pain chart, starts lower in stomach, some sparatic sharper pain here and there, constant headaches all day everyday, nausea, bloating/fullness, difficulty eating due to upset stomach but no weight loss, dizziness/faint, white outs and hot flashes, always not feeling well, always tired and run down, severe pain somtime so bad that uncapable of moving or standing more so falling over because of the pain. Please help us! Doctor: Hi.Thanks for your query.You have symptoms of lower abdominal pain, sparatic pain here and there, headache -nausea, bloating, difficulty in eating - but no weight loss - dizziness, white-outs - hot flashes -sometimes not possible to stand or move. . Wants to know the reasons. Your mentioning of the bowel movements, your age and sex (gender) would have helped me much. With the complex history you have mentioned the possible causes are:- Subacute intestinal obstruction.- General disease like Typhoid causing all these symptoms.I would advise you the following to get a proper diagnosis:Blood: CBC, Widal, sugar, liver and kidney functions.Urine and stool.X-ray of the whole abdomen in a standing position.Ultrasonography and CT scan of the abdomen to get a more clear picture.- Clinical evaluation and physical examination by a General Surgeon may help a lot. Depending upon the investigations the treatment would be planned to get a permanent relief."
},
{
"id": 181221,
"tgt": "What causes severe tooth sensitivity and ear pain after dental filling?",
"src": "Patient: I had a filling one week ago. I started having pain in that tooth before the feeling came back. I contacted my dentist and was seen. He then performed an x ray, it revealed nothing. He prescribed Norco 5mg. It helped for a very short time. Now I am experiencing severe tooth sensitivity as well as severe jaw and ear pain. Doctor: Hi,As per your complain slight sensitivity of tooth after a dental filling is not abnormal finding and can resolve in 2 to 3 days. But in your case it seems that it is not just sensitivity but considerable pain that is also radiating to Ear and jaw and it is a clear I education of nerve exposure that needs Root Canal. Treatment of tooth and not just filling will work. At times in acute condition the x ray is not diagnostic and clinical symptoms has to be taken into consideration for diagnosis.If I would be your treating dentist I would first of all remove the filling under local anesthesia and start Root Canal treatment to reduce pain and discomfort. It will be followed by a course of antibiotics and painkillers. After completion of root canal treatment, tooth will be capped to provide strength to it. So better consult an Endodontist and get evaluated so that he can treat you accordingly.Hope this information helps. Let me know if I can assist you further.Thanks and regards,Dr. Honey Nandwani Arora"
},
{
"id": 82818,
"tgt": "How to treat SLE?",
"src": "Patient: Hello Doctor, i m 11 yrars old gir.i have sle problem since 6 months ,my height is 4'11\" and current weight is 48kgs.i am using preduisulin 20mg in alternate day.But my anti ds dna is still positive.so plz what is the best solution 4 me plz give ur advice...thank Doctor: Dear Madam,Your treatment is right , anti ds DNA signifies the activity of lupus and it will take time to settle down.Dr. Shruti"
},
{
"id": 25281,
"tgt": "How can PoTS be treated?",
"src": "Patient: Hello, I have been diagnosed with POTS a couple months ago and and now on midodrine Topril rubinal and Cymbalta. Two days ago and then again tonight my left arm has turned really red and blotchy and it is really hot. It does not itch and isn t painful, just looks bad and is really hot (like sun burned skin). Just wondering what this is and if I should e concerned Doctor: HIWell come to HCMI really appreciate your concern,what the symptoms being given here it could be infection and it need to be ruled out till the condition can be manage with Tab Diclofenac 50 mg once in day, it is least concern withe POTS there could be other conditions like arthritis condition muscular infection (Cellulitis) hope this information helps."
},
{
"id": 15569,
"tgt": "Severe rash on face, painful on nose, prescribed Metrogyl for Roseacea, getting worse. Solution ?",
"src": "Patient: I have a severe rash on my face. I cannot wear my glasses its VERY painful especially on my nose. My checks are raw also. I have been using metrogel prescribed for roseacea but it is making it worse. I have triple antibiotic applied now which seems to reduce the pain. but I cannot use my cpap which means very little sleep and my glasses make it much worse. It is now starting to peal which makes it itchy. I am miserable. everything that touches my face burns. it hurts to lay down. my neck aches. and my checks feel swollen and are very red. Doctor: Hello Welcome! I would be pleased to answer your question . Persistent photosensitivity or seborrheic dermatitis are other causes of facial redness. It might be worthwhile trying 1% cortizone cream mixed with 1% clotrimazole cream on the red areas - twice a day for 14 days - to see if this helps. If it does then seborrheic dermatitis is the most likely diagnosis. If it does not then and given the fact that you have no history of reacting badly to the sun it might just be that you have sensitive skin and this is what is causing the redness. You might also need a course of an oral antibiotic which will need to take by prescription.It will be worthwhile seeing a dermatologist about this. Please feel free to ask if you have any queries.. Lastly it will also be worthwhile getting your blood ANA levels done to rule out Lupus. This is not likely form your clinical picture but must be ruled out."
},
{
"id": 140447,
"tgt": "Suggest treatment for a sore lump on the coccyx",
"src": "Patient: I mountain bike daily and have for years. Two weeks ago I noticed a hard lump on my coccyx. It does not move. It is almost like it is part of the bone. It is very sore when sitting, lying, and to the touch. It does not seem to be clearing up on it s own. Does anyone have any ideas on what should be done here? Doctor: Hello, A local skin or underskin infection is possible. Consult with a General Surgeon for your problems. Hope I have answered your query. Let me know if I can assist you further. Regards, Dr. Erion Spaho, Neurologist, Surgical"
},
{
"id": 121606,
"tgt": "Suggest treatment for bunion and arthritis",
"src": "Patient: Hi, I am 32 years old and my GP said that I suffer for bunion and arthritis when I came to her with my painfull big toe. As a treatment she said it is only painkillers. The pain depend on the time of the day, and the shoe I wear, but from day to day it is more painfull. I can not do things with my foot like before. Should i take any other treatment ? Does the copper rings or magnetic help for this kind of pain.? Doctor: Hello, Copper rings or magnets will not help. Consult a rheumatologist and get evaluated. In severe cases you can opt for steroid injection also. Hope I have answered your query. Let me know if I can assist you further. Take care Regards, Dr Shinas Hussain, General & Family Physician"
},
{
"id": 174472,
"tgt": "What causes high fever in children?",
"src": "Patient: Early this evening my 3 yr old daughter spiked a high fever 102.0. I am assuming at this point it is the flu since 2 of my other children are recovering from it now. However, after a dose of tylonel and brief nap she was up and back to normal. I frequently checked her temperature after she woke and it very quickly dropped to 95.8 and has been between there and 96.8 since. Is this something I should be concerned or is this a way for her body to protect itself also? Doctor: HI fever with flu is common as it is a viral infection .Symptomatic treatment like giving fluids, paracetamol when having fever will help.No need to worry now"
},
{
"id": 221009,
"tgt": "What causes brown vaginal spotting during 4 weeks of pregnancy?",
"src": "Patient: Hi. I m about 4 weeks pregnant and I began spotting this morning a dark brown with occasional gummy/stretchy tissue of the same color. I have not experienced cramping. I have had two previous miscarriages this past year that both were accompanied by strong cramping and heavy, syrupy bleeding. I m wondering if it is possible that I could still be pregnant even with the above symptoms? Doctor: Hallow Dear, I am indeed sorry to note that you are having brown bleeding in the early pregnancy. Usually brown bleeding, which is old bleeding, denotes the risk to the baby. Most of the time, such bleeding starts after the baby has become non-viable (missed abortion). Ultrasonography will give you information about the status and integrity of the pregnancy. If the pregnancy is no more viable, you will have to undergo the termination of pregnancy. Ignoring may lead to clotting disorders (DIC) or infection.Please report to your Obstetrician about this brown bleeding. I hope this gives you the required message. Dr. Nishikant Shrotri"
},
{
"id": 145035,
"tgt": "Are Selzic 150 and Levomac 250 mg right medicines for epilepsy?",
"src": "Patient: Hello sir, good morning, My son master Arpit age 8 yrs he is suffering from epilepsy last two years, and we have give to him velperin 200mg approx 1 year and now we have given to hime salzic 150, and livomax 250mg, but result is nil, Sir we are very upset regarding this problem, because he is not fine and after one week he is suffer this problem. So i requested to you please give your suggestion and help me. sir we are realy very upset... Doctor: Hello thanks for asking from HCM for your health related queriesI can understand your concern. Your son of 8 years is having epilepsy disorder and he was taking Valprin 200 but seizures persisted. If we consider his current medicines then Selzic or Oxcarbazine is an anti epileptic drug and Livomax 250 is an antibiotic drug. If even on this medicine his seizures persist then in my opinion there is need to add some adjuvant drug for his seizures. Medicines like Levatiracetam can be added to his prescription as add on drug. Combination treatment helps to provide effective control to resistant seizures. Consult his doctor for proper re-evaluation and prescription of the drugs.Thanks, Hope this helps you. Take care"
},
{
"id": 206810,
"tgt": "What causes lack of motivation and energy?",
"src": "Patient: I have no motivation theses days, im always tiried and just out of it. I am in need of a job but I can not get motivated at all, I just can t seem to muster up any motivation to actuly go out to job interviews or anything like that. I m starting to think there is something wrong with me, because I used to have a load of energy and motivation and it s just not there anymore, do you think I could have some kind of problem? I feel I need help. Doctor: DearWe understand your concernsI went through your details. I suggest you not to worry much. Must understand that motivation comes from need. If you are hungry, you are motivated to eat. Eating is a behavior which is directed towards quenching your hunger. Only goal directed behavior can satisfy goal. Therefore, it is very clear that you do not have any goal in your life, hence there is no motivation. You should know yourself and your abilities to find a goal in your life. Once you set up a goal, stick to it and your behavior will change. You can consult a psychologist for further advise. Don't worry, because worry is not at all a solution to any problem. If you require more of my help in this aspect, Please post a direct question to me in this URL. http://goo.gl/aYW2pR. Make sure that you include every minute details possible. I shall prescribe the needed psychotherapy techniques.Hope this answers your query. Available for further clarifications.Good luck."
},
{
"id": 136745,
"tgt": "What causes recurring on and off pains in arms and lower legs?",
"src": "Patient: Im having pain in my bones. All the time but feels worse when im working or lifting anything. Especially in my arms and the lower halves of my legs. It s an achey pain. I would rate it as a 2 on a scale of 1 to 3. Its a recurring pain that comes and goes. Ive been dealing with it for a few years now. Ive been use using deep heat but it doesnt help as much as it used to. A few years a go I was told that I am deficient in vitamin D. Doctor: Hi,Thanks for your query.The symptoms that you are having are likely to be related to benign causes like electrolyte imbalance, iron,vitamin D and B 12 deficiency, etc. Most of the times these symptoms can be taken care of by taking electrolyte rich fluids, vit D and Methylcobalamin and iron supplements.I do hope that you have found something helpful and I will be glad to answer any further query.Take care"
},
{
"id": 40807,
"tgt": "What causes difficulty in conceiving 7 years after first conception?",
"src": "Patient: Hi! me and my husband are having a problem to have a baby for almost 2 years.We tried so hard a lot to get pregnant.I know its not my problem because i have one 7 years old daughter from my ex.Do you think there is something problem w/ my husband? How will we know it and what is the better we need to do to get pregnant? Is there any other way or need to take some medicines to be pregnant? Thanks Doctor: Hi,Welcome to HealthcareMagic .I suggest you to get your husband semen analysis. If any abnormalities is there then treatment can be initiated accordingly. Also there are chances of females getting fertility problems after normal pregnancy before. So I would suggest you to get your follicular study to know egg maturation and growth is normal and also timing . If any problem is noticed treatment will be given accordingly.Also if your periods are not regular then get thyroid profile, FSh and Lh , prolactin levels done.Hope I have been helpful .RegardsDr.Deepika Patil"
},
{
"id": 59195,
"tgt": "Pain in the back radiating to sternum. Gall bladder removed in the past. Could it be due to carbonated drinks?",
"src": "Patient: I have been experiencing pain that seems to radiate from my center back to the front in the sternum area. This has been off and on for about a year. I had my gallbladder removed 2 months ago and my liver has been checked also. I was told that the surgeon looks around to make sure nothing isn't going on out of the ordinary. I stop having this pain about 2 weeks after surgery. I recently started drinking carbonated drinks again and was told by my friend who is a nurse that the drinks that I've been drinking probably caused this problem. So I'm sticking to drinking water only. Doctor: Hi It looks like Gastritis. Please avoid carbonated drinks. Please take Rabeprazole with Itopride tablets. Avoid smoking,alchohol,spicy foods,caffeine products if you are taking them. It is not related to Gall bladder surgery. If the symptoms still persist,please get Upper GI endoscopy and ultrasound abdomen. Wish you good health Regards"
},
{
"id": 178932,
"tgt": "What causes diarrhea in a child?",
"src": "Patient: My 3 month old son has had awful diahrea for the past 5 days. It s very stringy and mucusy with white chunks which I m assuming is I digested milk. He is bottle fed. The poop has no odor and is leaving a very bad diaper rash. He has had no fever. What could it be? What can I try? Doctor: Thanks for posting on HealthCareMagic. For infants, formula feeds are not easy to digest and often semi digested portions appear in the stool. Semi digested milk can also give rise to substances that may cause loose stools to be passed. However, rotavirus is a very common cause of diarrhoea in infants and probably that is the cause of diarrhoea in your child. Unless associated with bloody stools, even without treatment of the cause, such diarrhoeas generally resolve spontaneously within 5-7 days so antibiotics are not required. All you need to do is to replenish the salt or water loss by the use of ORS. Had he been exclusively breast fed, I would have insisted you to continue breast feeding however.Hope that helps. Feel free to revert back in case of further queries if any."
},
{
"id": 113946,
"tgt": "Is Deanxit the right medication for backpain in spine nerve ?",
"src": "Patient: my husband had a backpain in spine nerve, his taking deanxit prescreption by his doctor , my question is it is the right medication. thank you & God bless! Doctor: welcome to healthcaremagic please consult with orthopedic surgeon and get a check up and investigations as advised the medicine your husband is taking may be due to some other condition not related to back pain"
},
{
"id": 79158,
"tgt": "Suggest treatment for cough and blood in sputum",
"src": "Patient: I am having severe Cough and at times sputum gets blood as well, i suspected some throat infection initially but when i had my CBP and Chest X ray, reports showed my Haemoglobin as 12.6 ( male ) and in Chest X-Ray it mentioned Prominent Bronchovascular Marking two lobes, please suggest ? Doctor: Thanks for your question on Health Care Magic. I can understand your concern. Possibility of bronchitis is more in your case. Bronchitis is inflammation of airways. And it can cause coughing, hemoptysis (blood in sputum) and prominent bronchovascular markings on chest x ray. If you are smoker than you should quit smoking as soon as possible. You need inhaled bronchodilators and inhaled corticosteroid (ICS). Antihistamine and anti inflammatory drugs are also important in bronchitis. So consult pulmonologist and discuss all these. Hope I have solved your query. I will be happy to help you further. Wish you good health. Thanks."
},
{
"id": 61457,
"tgt": "What causes a lump on the thigh?",
"src": "Patient: I have a had a lump on my upper right thigh below my buttocks. It is about the size of a egg and it usually isn't painful unless I have been sitting for awhile. It has been there for about 20 months or longer and it has gotten back her home n the last few months. I am about 95-100 pounds and am 41 years old. I'm hypertensive. Also have osteoarthritis in pretty much all of my joints. I feel sill making a appointment to the dr over this but it is becoming bothersome. Doctor: Respected User, hi I evaluated your query thoroughly.*according to the narration it`s lipoma (Accumulation of fat)of the thigh.*Should consult a Surgeon or else can consult with photograph of the same for perfect management aspects in details. Hope this clears your query. Thanks for using Health Care Magic & keeping trust in our medical services. Wishing you fast and speedy recovery for the same. Regards. Bye dear take care."
},
{
"id": 51992,
"tgt": "What step can i take in high albumin in urine ?",
"src": "Patient: my albumin level in urine is reported as 495 mg. where as the blood sugar level is 108 ( fasting ) may I have your advice for the next step I should be taking Doctor: Dear thanks for the query.Normally kidneys secrete albumin in very small quantity or nothing at all.When your kidney cannot retain protein you should look for the damage to kidney.Please visit a physician or nephrologist and get proper investigation and treatment accordingly.You made ultrasound and other kidney function tests.bye and take care."
},
{
"id": 87091,
"tgt": "What causes swelling in abdomen after CT scan?",
"src": "Patient: Since having a ctscan of my lower abdomen and pelvic region I have been experiencing a swollen/distended abdomen. I am very small and have never had a tummy and my stomach feels as if all the organs are swollen. Thank you for answering my question. Doctor: iodinated contrast may cause swelling in Lower abdomen. if you are suffering form any renal dysfunction or contrast allerfy"
},
{
"id": 178152,
"tgt": "How to treat Vitiligo white spots in a child?",
"src": "Patient: Hi.. my daughter has just shown signs of Vitiligo... white spots on back and pinkish on and around eye. Consulted a dermatologist and he said its early or pre vitiligo. we wish to know whether we opt for Allopathic or Ayurvedic or Homeopathic treatment. Choice of treatment is highly confusing for us. Want to treat her asap before it spreads and ensure mininal side effects. Please advice on method of treatment. Doctor: there no term like pre vitiligo please do not hurry to treat this conditon apply liquidparrafin one hour before bath expose the child to early morning sunlight ,avoid talcum powder .of cource the best treatment is allopthy"
},
{
"id": 116218,
"tgt": "What causes low platelets counts after delivery?",
"src": "Patient: What can cause pitika? My platelets were low (80,000) 5 months ago when I delivered my baby. Is this a sign that they could still be low, or is it OK to get this once in a while? I understand what it is but what can cause it other than low blood platelets? Doctor: Welcome to H.C.M. I am Dr Krishna Dubey.My pleasure to help you. Pitika are eruptions on the face of adolescents . In pitika, normal excretory function of skin is disturbed and waste material stagnated inside producing symptoms like itching, pustules and nodules formation.It is not due to Decrease platelet counts.Earlier your platelet counts was low so i suggest you to repeat platelet counts to rule out thrombocytopenia( Decrease platelet counts).The exact cause of pitika is not known, but hormones called androgens can play a role. Androgens increase in both boys and girls during puberty. Androgens make the skin's oil glands get larger and make more sebum.This blockage of sebum results in pitika.Thanx for query."
},
{
"id": 968,
"tgt": "Will i be able to get pregnant after suffering from chlamydia infection?",
"src": "Patient: Hi, may I answer your i had chlamydia, about 2 years ago, i went and got tested at the end of april and i got treated for it, now 2 years on, iv settle down with my partner, we are going to start trying for a baby soon, i am just worried if il be able to conceive, because of the STI i had, will i be able to get pregnant, as i didnt have the STI long, i caught it and had it treated it quite fast, i would say i had the infection for abut 2 months. so do you think i still have a chance at having a family . thank you health querie Doctor: Hi, I think you can get pregnant. Try naturally for 6 months. Be in contact with your husband every 2 to 3 days after your periods stop. It will increase chances of your pregnancy. If it doesn't work, then you can go for evaluation. Hope I have answered your question. Regards Dr khushboo"
},
{
"id": 189095,
"tgt": "Had 2 root canal done on molar. Left side of face is swollen and hard. No pain. Reason?",
"src": "Patient: Hi there :), i recently had 2 root canal treatments on a molar in the lower left side of my jaw. Before the 1st treatment, i had no pain in the tooth, but after the procedure my mouth felt like it was on fire. I returned to my dentist on friday, who carried out a 2nd root canal. It is now sunday, and following the 2nd treatment the left side of my face has swollen ans become hard to touch. There is no pain in the tooth, but i am a bit worried that this swelling has appeared almost 48 hours after the treatment. Any advice would be greatley appreciated. Thanks in advance :). Doctor: Hello and welcome, The pain after root canal treatment may be due to- Infected pulp remaining within the canals, Overpenetration of files through the apex. Ledge formation. I would advice you to visit a dentist and get a thorough clinical as well as x-ray examination done. Take complete course of antibiotics-analgesics . Maintain oral hygiene well. Apply cold packs to subside the swelling. Hope this helps."
},
{
"id": 57802,
"tgt": "Is it normal to have vomiting that smells like feces after having gallbladder removed?",
"src": "Patient: my mother is experiencing sever stomach pain. in the upper part of her stomach. she has been in and out of the hospital with this for a year, which she is back in again now. they have taken out her gall bladder, done a colonoscopy, today the did the scope, said everything looked okay, they took biopsy which wont be back for couple weeks. with this attack she was found on the floor she couldnt move, she had been vomiting which resembled feces and smelled like feces. is there anything i can mention to her doctors for them to check. the drs did say it wasnt crhohns or celiac Doctor: Hi, Welcome to Health care magic forum. It appears that your mother has a fistula, between the stomach, and some part of the colon, due to the intestinal obstruction, hence the food is coming back into the stomach. You should inform your doctor, regarding this, so that they may ask you to have a CT scan, for diagnosis. Wishing for a quick and complete recovery. Thank you."
},
{
"id": 219026,
"tgt": "Suggest treatment for difficulty in conceiving despite being sexually active",
"src": "Patient: I have a question I been trying to get pregnant for about 2 months and I have unprotected sex with my boyfriend every day and I even have sex on my ovulation days and still not pregnant. But all I can do is keep on trying. But I have noticed my period it looks weird like November 20 of 2016 I started my period it and the color was red like bright but it didn t stay red for long it started looking brown and then pinkish and the Same thing happened in December 19 of 2016 I took a pregnancy test and it was negative but I still feel like I am pregnant so I am just going to go get a blood pregnancy test because it s more stronger than a home pregnancy test. And I do also have unprotected sex during my period and before my period comes on and after my period comes off what do you think i should do. Doctor: First of all I want to say that if you are not getting pregnant after two months of unprotected inercourse it does not mean u can not get pregnant.. It's okay. But u must get investigated.. That s blood test, usg and husband semen analysis"
},
{
"id": 146531,
"tgt": "Can I stop leviplil after 10 years of injury?",
"src": "Patient: Hello doctor. My son is taking eptoin 100 mg twice daily for last ten years. He lost consciousness and fell down twice ten years back. His EEG report said non specific changes. But after that everything was normal. Now he has some gum problems and his doctor says to withdraw eptoin and introduce leviplil 500 mg twice daily. My two brothers were patients of epilepsy. My question is , is it necessary for him to continue such medicine? Doctor: As there is gum hypertrophy Upto in should be withheld. As patient is seizure free for past 10 years we can attempt withdrawn of drugs. There will be chances of recurrence say around 10 to 15 %Hope my advice will help you. Take care. Don't forget to rate me."
},
{
"id": 109340,
"tgt": "What causes pain in lower back?",
"src": "Patient: I have pain in my lover back, i did my PSA test and is elevated 8.21, no problems with urination at all same thing with ejaculatio is normal except I have much less sperm. My doctor examine my prostate, limf, abdomen and everything sims ok. I still fill ticling sansation in my blader but urine test was ok. also.All this heppened during the flu I had and comen symptoms off flu were gone but my ticling sensation and back pain still presuite. what is wrong? Thank you. Jake Doctor: Complete bed rest for a certain period of time is the initial mode of treatment in low back pain. Pain can be controlled by medication, exercise, weight reduction, lifestyle modification etc. take paracetamol 1 gm twice or thrice daily as required initially for some days. you can do an MRI of lumbo sacral spine to clearly diagnose spine conditions."
},
{
"id": 84624,
"tgt": "Can pregnant women take udiliv 150mg tablets?",
"src": "Patient: iam 26 weeks pregnancy women,last 2 weeks iam having itching problem in my body, last day i met Doctor.she gave me some blood test . i got that result sgpt count (45)its high in my blood. Doctor told me to take the UDILIV 150mg tabs. its good to take,i dont know please inform me. Doctor: Hi, Based on the given history you seem to be suffering from cholestasis of pregnancy, a liver condition causing intense itching, but without a rash. Udiliv (ursodeoxycholic acid) is commonly prescribed to treat itching associated with this disorder. It does not cause any harmful effects during pregnancy hence can be taken safely as prescribed by your doctor.Hope I have answered your query. Let me know if I can assist you further. Regards, Dr. Mohammed Taher Ali, General & Family Physician"
},
{
"id": 157276,
"tgt": "Is there any solution for not being able to operate pancreatic cancer due to internal bleeding?",
"src": "Patient: hi my brother was just diagnosed with pancreatic cancer and he has had 3 radio and yesterday he could not breath because his hemoglobin is very low and now his in the hospital and they notice that his bleeding internally and his tumor is the same size 4 cm and he can not be operated on because the pancreas in attched to his aorta Doctor: Hello,Thanks for choosing health care magic for posting your query.I have gone through your question in detail and I can understand what you are going through.Yes, a malignancy where in the tumor is attached to aorta is not an easy procedure and has high chance of mortality. this is the reason it is not being operated. If your doctor feels that there is no chance of survival as such, then as a palliative procedure he can be operated with very high risk consent. Hope I am able to answer your concerns.If you have any further query, I would be glad to help you.In future if you wish to contact me directly, or wish to know further about this problem, you can use the below mentioned link:bit.ly/dr-srikanth-reddyWish you good health,Kind regardsDr. Srikanth Reddy M.D."
},
{
"id": 11693,
"tgt": "Black spots on back, itching. History of high BP, Bipolar, Insomnia, Anxiety, High cholesterol, Panic. Advice/",
"src": "Patient: For about two years noe I have had what looked like black burn marks on my back only. At first I thought nothing of it since most of my life. Id have s back itch have one of my Kidd scratch it and it would stop instantaneously. But for about two years or better I have numerous black marks above my bra line. Right now one has turned into a sore with scabs around it. They now go as high as my hairpins and always itch like crazy. I am on several meds, but this concerns me like mad. I take; Subutex Clonazapam Lyrics Atenolol Potassium Elavil and Simvistatin. I am a 47yr old Black woman diagnosed with chronic pain: high blood pressure::bipolar: insomnia: anxiety; high cholesterol: and panic attacks. I know this is a lot but can you offer some advice as to why my back is covered with what looks like burn marks? I've consulted four me's to no avail. Doctor: Hi, You are a 47 year old black woman, Had burn like black marks on the back with itching. Also have pain, blood pressure, bipolar , insomnia, anxiety, high cholesterol, and panic attacks. I to my patients with such symptoms prescribe anti fungal drug, grisofulvin, BID, anti oxidant tablet daily one. and local application of combination of anti funga, antibiotic and steroid. So i advise you to consult a dermatologist for treatment. Thank you."
},
{
"id": 128982,
"tgt": "What is the indication of chills and joint pain in 93 year old?",
"src": "Patient: My 93 year old mother has been suffering from chills in the evening (no fever). Temp in the house is at 70. She complains of fatigue. Last blood work showed Creatinine level at 0.90, low EGFR Non Afr American, low WBC and low Neutrophils, Absolute. Was seen by her primary beginning of the week and he did not seem to be concerned. Of course, at the time, I was not aware of the additional symptoms of chills in the evening and joint pain. Would like to go back to the primary with some idea of what could be going on and what else I should be asking my mother. Doctor: Hello,Thank you for suing healthcare magic.I read your question and understood your concern.It looks like viral infection so you can not do any thing just Ibuprofen and drink plenty of water. If there are complications such as pneumonia then need antibiotic treatmentDr. Selmani"
},
{
"id": 185699,
"tgt": "What causes soreness in the jaw joint after having upper denture?",
"src": "Patient: I got a new upper denture when I try to chew my jaw in the joint gets really sore also my cheek bones my lower natural teeth get really sore on the outside edge and the ridge of my upper jaw (where the teeth were) the lower teeth feel like they are biting on a flat surface Doctor: Thanks for your query, i have gone through your query. the pain or soreness in the joint could be because of the new dentures. it can be because of sudden change from edentulousness to dentures, or any high points or fault in the denture. the soreness over the ridge could be because of the sharp edges of the denture impinging on the ridge. consult a prosthodontist and get yourself examined and correct the problems. i hope my answer will help you. take care."
},
{
"id": 66866,
"tgt": "What does a lump under skin on back indicate?",
"src": "Patient: I have a movable lump under my skin. I noticed after i hurt my lower back. I didn t touch it for days and then it seemed fixed. A simple rub and it felt like it detached from something. It doesnt hurt and my backbis feelin good. It might be in the thoracolumbar fascia area. Should i be worried? Doctor: not to worry much as this is benign condition like sebaceous cyst or a neurofibroma.if really worries you can go for an FNAC test for confirmationall the best!"
},
{
"id": 25026,
"tgt": "What relieves the pain in the chest having the stents inserted?",
"src": "Patient: I had an exercise stress test the other week due to repeated chest pains, Had MI 18 months ago with 3 stents fitted since the exercise test I have felt much worse. My Cardiologist has booked me in for another angiogram in a couple of weeks but i feel much wors. what should I do. Doctor: Hello and thank you for using HCM.I carefully read your question and I understand your concern. I will try to explain and give you my opinion. As you mention, you are a person diagnosed with ischemic heart disease becouse you previously had an infarction and you had a percutaneous intervention whith stents.When a person whith this diagnosis complains for heart pain than we should exclude the possibility of active ischemic heart disease, meaning that might still be a problem whith coronary arteries, new ones or those treated before. In these cases we recommend a strees test.If your doctor has programed you for another angiography this means that he might have seen something in your stress test that rises his doubt for what I mention before. But, if your chest pains are more intense now in their severity or duration this means that the angiography should be prompt. So, my opinion is that you should reffer this symptoms to your doctor, becouse this changes in the intensity of pains are reasonable to accelerate the angiography. During this time don't make big efforts, take your medicament regulary and just meet your doctor.Hope I was helpfull . Wish you good health. Best regards, Dr.Ervina"
},
{
"id": 35433,
"tgt": "How to get rid of yellow pus formation in chin?",
"src": "Patient: A CHILD OF 4 YRS GOT HURT AT CHIN BY HITTING IT AT TABLE. tHE BLOOD STOPPED BY COMPRESSING AND THEN A BAND AID APPLID. NOW AFTER 24 HRS THE BAND AID CHANGED AND FOUND PUS(YELLOW) DISCHARGE. CLEANED WITH ALCOHOL FREE SWAB AND PUT A GAUGE DRESSING. ADVISE FOR FUTHER ACTION FOR THE PUS FORMATION Doctor: Hi,It seems that there is infection in the wound producing pus.Clean the wound with antiseptic lotion and dress it with antibiotic cream.Give him a course of antibiotic syrup for 3-5 days after consulting your doctor.Ok and take care."
},
{
"id": 111517,
"tgt": "Severe back pain, throwing up and stomach pain after eating food. Should I visit ER?",
"src": "Patient: hi I am a 45yr old female I have been hurting on my lower right side that goes to my back very uncomfortable cant sleep but few hours. throwing up for a week, waking up in morning soaked. some foods I get a terrible stomach ache after I eat. Should I go to ER? Doctor: Hello,I had gone through the case and found that it might be due to any gastric infection or renal complaint or appendicitis.So immediate go for ultrasound of whole abdomen , urine test and blood test.Meanwhile take antispasmodic pain killer with anti emetic medicine. You need to take it serious.Hope my answer will be effective for you.Thanks"
},
{
"id": 142084,
"tgt": "What could cause pain and pressure in skull, dizziness, red eye with low grade fever?",
"src": "Patient: i have been having pain and pressure on the rt side of my skull, it has been accopanied by a red eye, and also slight dizziness, espcially when i raise my head if it is down like reading , when i do, it feels like my head wants to go backwards, i have had low grade fever associated with sewats, please help me what to do this is been about 3 weeks already Doctor: Hello!Welcome on Healthcaremagic!Your symptoms could be related to a sinus infection. Other inflammatory disorder (temporal arteritis, cavernous sinus inflammation) can not be excluded either. Do you have nasal congestion?For this reason, I recommend consulting with your GP and performing a sinus X ray study and some blood lab tests (complete blood count, PCR, sedimentation rate). In case of a sinus infection, a sinus discharge bacterial culture would help find the proper treatment based on the type of infectious agent. If a sinus X ray study is excluded, I recommend performing a brain MRI or CT scan to exclude other brain lesions. Hope you will find this answer helpful!Kind regards, Dr. Aida"
},
{
"id": 75881,
"tgt": "Suggest treatment for a severe cough",
"src": "Patient: chest x-ray report- bilateral congested hila shadow with perihilar accentuation, lung markings are shown in both mid and lower zones, costopherenic angles are clear, cardiac size is normal and no active parenchymal lesion could be shown. my wife is a sugar patient and is suffering from serious cough for last 2 months and as per Dr. advise taking antibiotic but with no significiant result. what to do? can antibiotic cure Doctor: Thanks for your question on Healthcare Magic. I can understand your concern. I have gone through the x ray report you have mentioned. This x ray report is suggestive of bronchitis. And bronchitis can cause cough which needs inhaled bronchodilator and inhaled corticosteroids. No need to give antibiotics further as it can cause side effects. Better to consult pulmonologist and get done clinical examination of respiratory system and PFT (pulmonary function test). PFT will not only diagnose bronchitis but it will also tell you about severity of the disease. And treatment of bronchitis is based on severity only. She will improve with inhaled bronchodilator (formoterol or salmeterol) and inhaled corticosteroids (ICS)(budesonide or fluticasone). Strict control of diabetes is also needed. Don't worry, you will be alright with all these. Hope I have solved your query. I will be happy to help you further. Wishing good health to your wife. Thanks."
},
{
"id": 153856,
"tgt": "What does this biopsy test report of small growth in lips indicate?",
"src": "Patient: I am 48 years without any complications. Recently I had a small growth in the lips which was bulging. As per medical advise it was excised and sent for lab test. Lab report says DYSPLAXIA. My doctor has again sent smear for lab advised to contact oncologist.Can u tell me whether it iscancer Doctor: HiThanks for your query.I appreciate your concern for the biopsy report.Dyplasia is a precancerous condition in which cells which are very similar to cancer cells grow locally in an organ without the ability to invade into tissue or metastasize (spread to areas distant from where they started). Dysplasia is the earliest localized form of pre-cancerous lesion which can be cured by surgical excision.This biopsy report needs to be correlated with your clinical examination by an oncologist.Hope this answers your query.Take care."
},
{
"id": 171456,
"tgt": "What does low growth hormone level indicate in blood test report?",
"src": "Patient: Sir, My sister s son is diagnosed with Growth Harmons problem as his height growth is lower than normal. He is around 12 years of age. Test result shows following; Growth Hormone 0.045 ng/ml 0.10-8.80 INTERPRETATION:Low levels which are not able to be stimulated to a level achieved by normal subjects occur in pituitary dwarfism in children.High or normal levels cannot be suppressed are found in gigantism and acromegaly .lGF-1 is a better screening test for acromegally and gigantism. REFERENCE: Simard MF.Neurosurg Clin N Am 2003;14(1):41-54 Please advise what should we do. Thanks Ram Doctor: hello ram,sorry to hear about your sis son problem.growth harmones level alone will not help in diagnosis.he has to undergone further evaluation to rule out other endocrine problems.short stature can be due to other causes too.i would suggest you to consult an endocrinologist.he will help with further management.have a nice day"
},
{
"id": 93643,
"tgt": "Lower left abdominal pain, more pain when bladder fills, sore lower back. Any ideas?",
"src": "Patient: Hi I would like some info regarding lower left abdomnal pain, which I have since last July, have had a gunny doc check my ovaries etc said all clear. Have bloods and swabs done and all clear. Am waiting for bowel clinic to see what they say. Also notice if bladder fills up the lower left pain gets worse or more uncomfortable, have myself worried about what could be wrong. Sometimes I also feel my lower back sore. Doctor: Hi welcome to Health care magic forum. Thanks for calling H.C.M.Forum. You have got lower left abdominal pain since 1 year, it gets worst when bladder is full. Some times you get lower back pain. It appears to be the urinary tract infection, or small stones in the left ureter or a big stone in the bladder. I advise you to consult an urologist for diagnosis and treatment. You may need to have M.R.I. besides other routine tests for confirmation. Wishing for a quick and complete recovery. Best regards."
},
{
"id": 163672,
"tgt": "What is the brownish thing coming out of my 2 years old s penis?",
"src": "Patient: My 2 year old son had a stomach flue, diarrhea, vomiting, not eating well or drinking, luckily he\u2019s coming back and things are getting better. The only thing is worrying me is that I noticed some brownish dry cinnamon like stuff in he\u2019s dipper, it seems like its coming out of he\u2019s penis and not wet just little bit of pee and the brownish thing stock to he\u2019s dipper. Can you tell what it is? Doctor: dear parent please find a pediatrician to take your child to. he needs to be examined and investigate dplease don't delay."
},
{
"id": 193564,
"tgt": "Is there a treatment to increase the sperm count?",
"src": "Patient: This is mohan kumar recently i am tested sprem count. Result is Nill , is it possible nill and how increase sprem count. Previous i am using pestrozon harmon medicines. is any effect this sprem plz give me answer sir, i can't understand, i am purely vegetarian and i am not drinking and smoking. give me sugession. my mail id is YYYY@YYYY sir i want wait for reply.thankumohan Doctor: Hi, There are few reason for your issue \u2022\u00a0\u00a0\u00a0\u00a0\u00a0Varicocele - swelling of veins in testicle / scrotum \u2022\u00a0\u00a0\u00a0\u00a0\u00a0Infection - Sexually Transmitted Diseases or Non Sexually Transmitted Diseases causing epididymitis, orchitis \u2022\u00a0\u00a0\u00a0\u00a0\u00a0Anti-sperm antibodies \u2022\u00a0\u00a0\u00a0\u00a0\u00a0Hormone imbalances \u2022\u00a0\u00a0\u00a0\u00a0\u00a0Alcohol & Smoking There is a procedure called sperm biopsy - you can check that out. Hope I have answered your query. Let me know if I can assist you further. Regards, Dr. S. R. Raveendran, Sexologist"
},
{
"id": 186388,
"tgt": "Suggest treatment for perioral dermatitis in corners of mouth",
"src": "Patient: Hello, I've had perioral dermatitis in both corners of my mouth for about 5 years now. Everytime I go to the doctor I get hydrocortsine (1%) and it never goes. What do I do? It makes me feel really self-conscious and unattractive and I just want it gone. Doctor: thanks for posting your query in HCM..perioral dermatitis is named to skin disease that manifest as multiple papules pustules or vesicles found surrounding perioral skin and nasolabial folds..it can be caused by many reasons likeallergy to certain medicationautoimmune diseasenutritional deficiencyhabits like lip sucking or lip chewing or licking etc..as u told that u have near corner of mouth it can also be due to dribbling of saliva at corner of mouth followed by infection most commonly by fungi candida...so i will suggest you to use anti fungal mouth ointment on corner of mouth and antiseptic mouthwash..also take multivitamin supplements..hydrocortisone use is limited to condition that are either allergic or autoimmune in nature with precaution to stop the drug gradually after improvement..hope my advice will help you positively..thanksDr. Priyanka"
},
{
"id": 58117,
"tgt": "High SGPT levels, non-alcoholic. Prescribed ursodiol. Chances of Hepatitis B?",
"src": "Patient: hi doctor, my hubby is non alcoholic and he was found to have a high level of SGPT around 235, then he was put on tab ursodiol 300mg bd, he took the same dose for three mnths thn he changed the dose to 150 mg bd since last mnth, i just want to know cah high SGPT cause hepatitis b positive condition ? and what all are the risk conditions of this high enzym level ? Doctor: Dear Mam.There are many causes of raised SGPT, and yes Hepatitis B is one of them.Your doctor must have tested him for Hepatitis. In case not, I advise you for the following tests :1. Liver function test2. HbsAg , Anti- HCV antibody.3. Usg Abdomen.No point in treating raised SGPT levels with Urosidiol without knowing the cause for its rise.Stay Healthy"
},
{
"id": 80075,
"tgt": "What do wet and choking feeling in lungs suggest?",
"src": "Patient: Hello I don t know if this sounds normal or not. my lungs feel wet! you know how when you go swimming and you choke on water. well that is how I feel. We went swimming a week ago but the feeling keeps progressively getting worse. no fever at all. very small couch every once in a while. should I got to the doctor or ride it out? Doctor: Hello,Kindly visit a physician for a chest Xray and clinical examination.If there is any infection that you may have got during swimming, it can be treated with antibiotics and cough expectorants.Regards, Dr.Riyanka"
},
{
"id": 101999,
"tgt": "What causes cold, sneezing, coughing up yellow phlegm and nose bleedings?",
"src": "Patient: I have either had a cold or bad allergies for a week - including low energy, sneezing, coughing, coughing up yellow phlem. I thought I was getting better today until I had a coughing attack so bad that my nose started bleeding. (Which has never happened before, and I have a history of sinusitis) Are nose bleeds normal when coughing? Doctor: Hi, If severe cold or due to more pressure on nasal mucosa due to cough or due to manual trauma, nasal bleeding can happen. If severe or repeated bleeding get examined by your doctor. Hope this answered your query.Regards"
},
{
"id": 206366,
"tgt": "How to taper of Diva OD and Qutipin?",
"src": "Patient: Hi , i am taking medicine DiVA OD-1000 m.g. and Qutipin-100 m.g. since july2013 so please help me for this regard how can reduce or left this medicine , i was some bad history with college time during april 2013 i was make misbehaviour with a student girl , i was touch her breast ,,,might be i was not slppeping well because my phd. supervisor presurrised me for papers/journals Doctor: Hello You are on Sodium Valproate 1000 mg and Qutiapine 100 mg since July 2013. You misbehaved with a girl prior to that and extreme pressure of papers caused disease to get precipitated. As you are on these medications and want to stop the medicine. I would advise you to consult your psychiatrist regarding tapering of drugs as he has seen your clinical case from beginning. The medicines are gradually tapered over 6-8 weeks. Usually the dose is decreased gradually, about 25% every 2 weeks and then stopped completely at about 8 weeks. In this way the withdrawal doesnt occur and individual can be weaned off completely.Thanks, hope this helps you"
},
{
"id": 48276,
"tgt": "Suggest treatment for heavy breathing with kidney stone blockage",
"src": "Patient: My Kidney function level is at appox. 55 percent due to kidney stone blockage some years ago. At time i experience some heavy breathing, only for a short period after exertion such as gardening or minor lifting. Does this seem normal with low kidney functioning? As i mentioned it passes quickly. Doctor: Hi,Thanks for writing in.Heavy breathing might be a symptoms of your general health and not specific to your kidney. It is appropriate if you can get examined by a general physician and get routine blood tests done. Anemia due to low hempoglobin counts s the commonest cause for sudden tiredness after short period of mild exertion. If you are past your middle age then getting your heat checked is also important. Abnormal thyroid hormones also cause suddenheavy breathing after exertion. The fact that it passes quickly indicates that the condition should you have any is in its mildest form and treatable with medication. Please do not worry."
},
{
"id": 136911,
"tgt": "How to cure tingling sensation on both hands,neck and shoulder?",
"src": "Patient: I am 46 yrs male having tingling sensation on both hands and pain in neck shoulder and both arms,I have done MRI of servical spine as per orthopedician advice.The report is as below 1)Multiple disc protrusion at c4-c5& c5-c6 level & compression of adjacent subarachnoid space 2)post traumatic partial vertebral compression of c5&c6 3)Cervical partial block vertebrae of c3-c4 with vestigial disc in between. 4)F/O Cervical spondylosis 5) Mild atlanto-axial dislocation. Kinly advice me for further medical treatment and precautions if any. Doctor: Hello, I have studied your case. Due to compression of nerve root there can be tingling numbness in your arm and pain.For these symptoms analgesic and neurotropic medication like pregabalin and methylcobalamine can be started.Continue your Physiotherapy like ultrasound and interferential therapy will give quick relief.I will advise to check your vit B12 and vit D3 level.Your Spondylitis will get aggravated due excessive travelling so better to avoid such travelling. If nerve compression becomes symptomatically severe then surgical decompression will give permanent relief.You can attach your MRI report or film so that I can help you better.Hope this answers your query. If you have additional questions or follow up queries then please do not hesitate in writing to us. I will be happy to answer your queries. Wishing you good health.Take care."
},
{
"id": 96647,
"tgt": "Suggest remedy for pus discharge from wound scab",
"src": "Patient: Hi Doctor. I have an abrasion from a car accident 2 weeks ago that looks like a gash. However it is not like a deep cut. I think an object must have hit it hard but did not penetrate too deep as I was wearing jeans. It's about 3cm long and 1/4cm wide. It seemed to be healing and there is even a thick scab on the surface. But I noticed a little pus coming out from the sides over the past 2 days. I am using Bernadine to clean and I leave it open without covering. Please advise if i have any worries. I will try to send a picture. Doctor: Hello and welcome...Don't worry buddy its OK...the pus are the debris and it should be out from there so there is rapid healing...When the pus accumulates then healing process halts...And its good to wash with antiseptic solution...Hope you'd understand...See you back in good health next time...bye and take care ."
},
{
"id": 206726,
"tgt": "Suggest treatment for eating disorder, hallucinations and suicidal tendency",
"src": "Patient: I have an eating disorder and I have been restricting a lot, I started cutting again, and I m feeling a little suicidal. I endured a sexual experience that I didn t want to be a part of and ever since then I have been experiencing visual hallucinations again. I really don t know what to do. Do I need to go back to the hospital? Doctor: DearWe understand your concernsI went through your details. I suggest you not to worry much. If you are correct about the given symptoms, you need to consult a psychiatrist. Eating disorder, Bulimia, is also a bit depressive and therefore suicidal thoughts are part of it. Hallucinations can be part of suicidal urge which may be linked to schizophrenia. Timely diagnosis and start of treatment can give you good result. Please do not hesitate to visit a psychiatrist.If you require more of my help in this aspect, Please post a direct question to me in this URL. http://goo.gl/aYW2pR. Make sure that you include every minute details possible. I shall prescribe the needed psychotherapy techniques.Hope this answers your query. Available for further clarifications.Good luck."
},
{
"id": 75861,
"tgt": "What causes pain in left side of chest, shoulder blades, neck and jaw?",
"src": "Patient: hi there I have a pain in my chest - left side, constant for 2 yrs now, constant painful sharp pains in shoulder blades. Last Thurs my neck and jaw ached too - took ibufuren, eased it. Worse on mornings or its just when i notice it more, also just as i wake strong pulsing lower back sometimes in head too. Doctor: Hi thanks for contacting HCM...This left sided chest pain also reffered to shoulder blades , so cardiac causes has to be ruled out ....Investigate with. ....1.Blood pressure measurement 2.ECHO 3.Tread mill test 4.Angiography (only if needed )If no cardiac causes present than thus could be from musculoskeletal pain which needs proper sleep posture and analgesic tablet ....Consult pulmonologist with keeping my answer in mind ....Take care ....."
},
{
"id": 66821,
"tgt": "What does a small lump on inner thigh indicate?",
"src": "Patient: Recently I have noticed a small lump on my inner thigh. It is sensitive to touch and slightly painful.I am quite concerned on what it is.I have been taking weight training classes and I m wondering if this may have caused the lump of form. I have no idea what it could possibly be and I am wondering what I should do next. Doctor: Hi, dearI have gone through your question. I can understand your concern. You may have some lymphnode enlargement or some soft tissue tumor. Next to do is fnac. You should go for fine needle aspiration cytology of that lump. It will give you exact diagnosis about lump. Then you should take treatment accordingly. Hope I have answered your question, if you have doubt then I will be happy to answer. Thanks for using health care magic. Wish you a very good health."
},
{
"id": 74463,
"tgt": "How can coughing, breathlessness, swollen feet and vomiting be treated?",
"src": "Patient: been coughing for 3-4 days. have shortness of breath, swollen feet, when lie down breathing becomes tougher, heavy snoring, wake up in the middle of the night to cough and catch breath, on 4th day vomited, if breathe out the lungs sound like a heavy smoker of many years.. Doctor: Thanks for your question on Healthcare Magic. I can understand your concern. By your history and description, we should definitely rule out heart (congestive cardiac failure) and lung (acute bronchitis) related diseases. So better to consult doctor and get done ecg, 2d echo, chest x ray and PFT (Pulmonary Function Test). Ecg and 2d echo are needed for heart failure. Chest x ray and PFT are needed for bronchitis. You may need antibiotic, diuretics, bronchodilators and other supportive drugs on the basis of diagnosis. Don't worry, you will be alright with appropriate treatment. Hope I have solved your query. I will be happy to help you further. Wish you good health. Thanks."
},
{
"id": 31769,
"tgt": "What causes swelling and bruise in the dog bitten place?",
"src": "Patient: I got bitten by one of my dogs when I broke up a fight between three of them. All of the dogs are current on vaccinations. The bite is a puncture wound in the web of the palm of my hand between the thumb & index finger. I know it must have punctured the muscle because it is oozing slowly (blood & clear fluid). This happened about 3 hours ago. The area is bruised and swollen. I have flushed it with cold water and put ice on it, & I took Ibuprofen. No stitches are needed, but I am wondering what else I can/should do for it to ensure that it does not get infected. Doctor: Welcome at HCM I have gone through your query and being your physician i completely understand your health concerns.For how long u are suffering from it? Any other associated problem? like diabetes? Are u running fever? Any other lymph node swelling in your body? You should really visit ER so that your wound can be examined by a doctor.Thorough cleaning of the affected area is must. and try to meet your doctor as soon as possible Meanwhile stay calm and use acetaminophen to relieve pain Get well soon Hope your query is adequately addressed if you still have any feel free to ask RegardsDr Saad Sultan"
},
{
"id": 140299,
"tgt": "What is the rate of survival in elderly patients with carcenoid tumors in brain with seizures",
"src": "Patient: my mom has carcenoid tumors metastasized to brain and ectopic cushing syndromes. she is 77 and has lost 15 kg. last 10 days she has been having seizures continuously. both generalized and focal. she has been on glucose drip and midazolam injections since past 10 days. no other medication or artificial respiration is being given any more. how long she will survive Doctor: Hello, In your mother's case, there are several factors that indicate a reserved prognosis. These factors are metastases, seizures, and the age. So, I think that even with medical support the prognosis is very reserved. Hope I have answered your query. Let me know if I can assist you further. Take care Regards, Dr Erion Spaho, Neurologist, Surgical"
},
{
"id": 55954,
"tgt": "What causes an increase in the sgpt count?",
"src": "Patient: sir, my age is 19 yrs, ihave no any problem of pain and anything, But in a regular test igot my sgpt count is 214 and sgot count is 121 . After 20 days elopethic medicines the results were down by 128 and 52 respectively. i am still in elopethic treatment . sir tell me upto how much time i have to take medicines and even after complete course is that possible that these counts not increase in future, or other wise i should choose any ayurvedic treatment . i am absolutely non-alcohalic. please advice me. thanks, Doctor: HelloIncreased SGPT and SGOT may indicate liver injury.It may be due to many reasons like hepatitis,alcohol intake,altered lipid profile,medicines,auto immune causes etc.You may need few more investigations like routine hemogram,random blood sugar,viral markers,ultrasound of abdomen.I suggest tablet ursodeoxycholic acid 300 mg twice daily for three months to my patients.It helps in regeneration of liver cells.It is good that you are non alcoholic and values are decreasing after treatment.You may need to take medicines for three month.It depend upon evaluation.You should avoid saturated fat.Get well soon.Take CareDr.Indu Bhushan"
},
{
"id": 78963,
"tgt": "What causes pain in breast bone and under my rib cage?",
"src": "Patient: i have had sharp pains through my breast bone. hard to breathe, pain just under my rib cage right where my stomach starts. doctor told me it sounds like heart burn but i have never felt heart burn like this. i am a 24 year old female. i have been having this issue for almost a year now Doctor: In the case you need to atleast undergo a chest x ray to rule out any underlying lung pathologies. Also do a breast examination to look for any lump."
},
{
"id": 165615,
"tgt": "What is the treatment for dizziness in a child?",
"src": "Patient: My five year old just got real dizzy and fell, I am not sure why. Could it be blood sugar? She had pancake for breakfast with syrup, a root beer at lunch and then a cookie. This has never happened. She had not had anything to drink for the day other than that root beer. Please advise! Doctor: Hi.... I understand your concern. By what you say I feel that this could be a transient ischemic attack or a type of seizure called Complex partial seizure or a rhythm disturbance in the heart. I suggest you consult your pediatrician and get an ECG, ECG and echo cardiography. We can proceed further based on reports.Regards - Dr. Sumanth"
},
{
"id": 82891,
"tgt": "What is the itchy and painful rashes on my buttock with lupus?",
"src": "Patient: I have lupus and am currently in a flare (red cheeks, etc.). I fell 3 weeks ago tomorrow, so I know I've compromised my immune system with that as well. I've a quarter-sized area on my right buttock near the base of the spine. It's itchy, painful and uncomfortable. Could this be shingles? I've never had it, although I've run the gamut of rashes with the lupus. This is new. I see my rheumatologist tomorrow...is this something he can help with, or should I call a dermatologist? I was treated months ago with Neurontin for facial/cranial nerve pain...same side. Doctor: Dear Madam,This can be involvement of dermatome and possibly right in saying shingles even.Dr. Shruti"
},
{
"id": 11543,
"tgt": "Suggest medication for skin burn due to reaction to dettol",
"src": "Patient: i got a chemical burn from using dettol antiseptic lotion on an acne mark..what should i do to treat the burn? my email id is YYYY@YYYY . i have an interview to attend on monday.i need to look good.i m really shattered now. please suggest something that i recover faster Doctor: Hello,Thank you for posting on HCM.It seems you have got contact dermatitis due to Dettol.Nothing to worry about as it will be fine soon.Avoid use of any facewash/soap and other cosmetics on face for few days.Use a good moisturiser like cetaphil DAM thrice a day all over face.Apply a mild steroid cream like desonide gently over affected areas twice a day for few days.Use high SPF sunscreen before going out in sun.Hope this will take care of your issue.Wish you best of health.Thank youDr Hardik Pitroda"
},
{
"id": 91775,
"tgt": "What causes pain in ribs and tenderness in stomach?",
"src": "Patient: Hi every now and then I m getting pains in the top part of both rib more to the side area some times the right side sometimes the left also wen I sit down my stomach feel really tender and gasy feel like it s guna cramp in areas can you help?i got a bad back but don t think that s anything to do with it Doctor: HI.As already explained , please go for an MRI of the thoracic spine., as this is easily missed by many clinicians. If it is normal , we are happy to rule out the spinal pathology .Also for for x-ray of the chest and other relevant investigation as advised by your Doctor. Let us know the results and reports for us to help you more. Tell us if you have another symptoms"
},
{
"id": 77029,
"tgt": "Suggest treatment for emphysema",
"src": "Patient: I have severe emphysema and heart desease ,which i have lots of medications for iwasnot suitable for cbg as too much lung damage ,so triple bypass out of the question. My chest has been really bad for a bout 3-4 wks, i am unable to breath without rattling /wheezing and can't even shake-up a pillow, without taking inhalers inbetween. my doctor put me on clarythromycin 500mg, he rang next day to see how i was and it had eased a lot. However by that afternoon it crept back and has been as bad since. my doctor thought it could possibly be pneumonia, when he rang, but asi say it had eased ..He said i had to see him early next week. He also put me on prednisolone ,which i often have, but he said keeping me on 6daily for a good while and eventually wean them down do you think it is pneumonia. i am coughing up lots of clear phlegm too.. Doctor: Hi thanks for contacting HCM...Noted you have emphysema and cardiac history...Here you could have exacerbation of emphysema....That can lead cough ...If pneumonia suspected at that time then chest x ray should had done to look for consolidation....Now you are given antibiotic and recovering.so no worry...As you have emphysema regular follow up needed.Avoid smoking....Keep minimal exposure to pollution....Ultimate treatment is lung transplantation.Take care...."
},
{
"id": 13880,
"tgt": "What is the treatment for rashes after knee surgery?",
"src": "Patient: I had knee surgery on Tuesday, November 20th. Five days after the surgery, I developed a rash above the area where the ace bandage was placed. I went back to my doctor and they gave me a strong steroid cream. Rash seems to be getting worse. Any suggestions??? Doctor: Hi, In my opinion, it could be a contact dermatitis. But it would be difficult for me to confirm the diagnosis without examining the rash. So, I recommend you to visit your Dermatologist to confirm the diagnosis and to initiate the treatment. Hope I have answered your query. Let me know if I can assist you further."
},
{
"id": 9161,
"tgt": "Suggest treatment for dry and scaly bumps all over the body",
"src": "Patient: i have red itchy bumps appearing randomly all over my body, they started between my fingers 4 weeks ago and are now showing in armpits wrists inner thigh under scrotem tops of arms, the itching is very intense and now my girlfriend has the same conditions. The itching comes and goes but when its there it is almost unbearable and i will scratch till it bleeds. I thought it was scabies but cant see any sign of burrows, have taken antihistimine which hasn't really helped did use hydrocortisone cream on my fingers which cleared the rash but has now turned them dry and scaley Doctor: Hi.As per your case history of dry skin.My treatment advice is \u2013 1. Don\u2019t scratch the lesion as it may worsen the condition.2. Apply an emollient cram like enmoist twice daily on affected skin.3. Take a folic acid supplement for 3 months and levocetirizine for itching.4. Other treatment options are topical steroid, oral steroids taken only after consulting a dermatologist.Thanks.Dr.Harshit Bhachech.MBBS, DDVL."
},
{
"id": 22545,
"tgt": "What causes abnormal blood pressure readings?",
"src": "Patient: my blood pressure is 135 over 107 and it was 131 over 99 two days ago.. I am concerened with the blood pressure readings. I am 5'11.. I weigh @ 205lbs. The only other health issues that I have is acid reflux and I have had 2 issues with kidney stones over the last 3 years. Doctor: Hi,Please let me know if you are obese, have any risk factors like diabetes, or if you are smoking. You have isolated diastolic hypertension, which needs to be treated. Best treatment for BP is lifestyle modification like avoid fatty, oily and high calorie diet. Have low salt diet and monitor blood pressure regularly thrice a day for one week then once or twice a week.Regular exercises like brisk walking, jogging according your capacity at 30 min a day and 5 days a week. Have lots of green leafy vegetables, fruits, fish once or twice a week, avoid meat. Avoid smoking and alcohol if any. There shouldn't abdominal fat deposition or obesity.Hope I have answered your query. Let me know if I can assist you further.Regards, Dr. Sagar Makode"
},
{
"id": 68665,
"tgt": "Suggest remedy for spongy lump on elbow",
"src": "Patient: I just discovered a soft spongy lump on my elbow - It does not hurt, can be moved around, and was not caused by any injury that I know of. It feels strange and is about the size of a quarter. Should I be concerned - will it grow in size or become painful? It is actually on the bone of the elbow itself. Doctor: welcome to Health care magic.1.The most possible cause can be lipoma - lump of fat content ( mobile, painless, no history of trauma)2.Othere possible causes are - ganglions (from joint capsule,tendon), Vascular malformations.3.An ultrasound lesion will help to find out the nature of the lesion and its content and sours and extensions. 4.Take a general appointment with your GP, after examination request for ultrasound lesion.5.You have to be concern as some thing growing on your body, we have to find out and treat the cause.Hope it helps you. Wish you a good health.Anything to ask ? do not hesitate. Thank you."
},
{
"id": 22994,
"tgt": "What causes sinking feeling in the heart?",
"src": "Patient: Since morning after getting up from the bed and till now I am having like sinking heart problem and seems like I am very tired and sometimes I have to take a deep breath....I took my breakfast, did shopping and made dinner and took dinner but the feeling is same through out the day.......what is that and how could I be normal? Please help...I am 45 and have two childrens 10 and 11. Doctor: hiit can be hypotension or because of brahdycardiaget your blood pressure and serum electrolyte measured some times loss of electrolyte can result in symptoms you are havinga 2d echo should be done to rule out the cardiac functionshowever intiallly get your blood pressure and blood sugar checked up ,both hypotension and hypoglycemia can cause feeling of sinking"
},
{
"id": 95270,
"tgt": "Why am I unable to pass motion with floated belly ?",
"src": "Patient: Hello, Motion is not going properly - once in 3 days and also irregular timings. I feel my belly always look like floating. Doctor: hi,thanks for query.There are many reasons for this constipation,first of all look for food habits,avoid maize products,have more of leafy vegetables and drink plenty of fluids.Do regular exercise,this helps in most of the cases.You can discuss with your doctor and take laxatives for time being till this above things helps. wishing you good health."
},
{
"id": 122341,
"tgt": "What is the treatment suggested for pleural effusion thickening?",
"src": "Patient: hi doc.. i just had my thoracentesis twice last week and they drained almost 1.2 L of fluid.. but on my recent follow to my doctor today and had my xray there s an impression of haziness on the lower base of my right lung with light solous and also pleural effusion thickening.. what would be the mode of treatment to that? It it possible that it will be resolve by medication? pls help.. thanks! Doctor: Hello, A CT scan is required to get a better clinical impression. If asymptomatic no treatment is required. Consult a pulmonologist and he will direct you accordingly. Hope I have answered your query. Let me know if I can assist you further. Take care Regards, Dr Shinas Hussain, General & Family Physician"
},
{
"id": 107647,
"tgt": "What does Diffuse disc bulge and mild kyphosis of cervical spine suggest?",
"src": "Patient: L5-s1 desiccated disk with posterial Annular tear and mild Protrusion causing Thecal Identation. C4 to C6 Desiccated diffuse disk bulge with mild Kyphosis of Cervical spine. What is this mean Help pls Recently I have been diagnosed for my back problem, i have done two MRI for my problem, First report saying that Acentric High Signal at L5-S1 with Margin Ingeriorly on T2 May likely Represent Type 1 Annular Tear, Secondly, Circumeerniation disk L4-L5 with Mild lock of Posterial Concativity identifing the Anterior Thecal SAC with Relative Ingerior Neural Forminal Compromis. Annular bulge at L5-s1 interverbiral disk with the prosterial central disk protrusion causing mild Anterial Thecal SAC compression, Traversinty Nerve Indentification with relative Neural Foraminal Comprimise. Second MRI says, L5-s1 desiccated disk with posterial Annular tear and mild Protrusion causing Thecal Identation. T4 to C6 Desiccated diffuse disk bulge with mild Kyphosis of Cervical spine. What is this mean Help pls Limited CT Sections show no evidence of Spndylolysis or Listhesis. What is my condition... I want to be pregnant soon, can i go for pregnancy or not? What care should i take for my health. My Man is healthy he has no problems, Does my kids get this problems. Please help me.Also i am unnable to feel better in my intercourse, i cant move my posture, infact he is not putting his weight on me at that time, he is using his knees and shoulders to support himself at that time. How can i get my problem solved. Please please help me doctors. Doctor: U need some rest. Avoid any intercourse for few weeks. U got posterior annular ligament tear. It might progress to diffuse disc bulge. Need not worry. It 'll not be transferred to ur kids. U need absolute life style modification for 4-6 weeks. Do some physio. Lie on flat couch with soft pillows underneath ur thighs n knees. U 'll be better soon as there s mild disc bluge. For cervical problem, avoid pillows avoid lifting heavy objects"
},
{
"id": 215766,
"tgt": "What causes pain and insensitivity in finger?",
"src": "Patient: I have had PHN for some time and have now developed insensivtiity in little finger of my left hand.My GP has examimed and said that I am insensitiv e with no suggestion as to what might done about it. The seneitivity is now effecting the tip of the next finger. Doctor: Hi, Consult a neurologist and go for nerve conduction study as I can be due to neuropathic causes like nerve impingement and require early intervention. Hope I have answered your query. Let me know if I can assist you further."
},
{
"id": 18841,
"tgt": "Could an enlarged heart be related to large red blood cells?",
"src": "Patient: i had an enlarged heart and a hole which was corrected via open heart surgery as a child. i have been told that i have had large red blood cells for a number of years. i would like to know if there could be a connection as the doctor has tested for a number or things to find the reason for the large cells and has found no answer. he did not look into my history Doctor: Hello,I don't think that your heart problem has to do with enlarged red blood cells. If you are also anemic this is called megaloblastic anemia and may have to do with your diet, your gastrointestinal tract or something else. The hematologist can see you for this problem. You may need vitamin B12 injections. Hope I have answered your query. Let me know if I can assist you further.Regards,Dr. Anila Skenderi"
},
{
"id": 197750,
"tgt": "Will masturbation leads to shortness of breath?",
"src": "Patient: I exercise everyday (morning and evening) morning running and evening fitness and i musterbate daily (sometimes twice times a daily)...From the past one month, i having shortness of breathe (not continuosly)..its just when i stop musterbating for a while, it won't have shortness of breathe...so, is it because i musterbate a lot and causes shrotness of breathe? Doctor: Dear, We understand your concernsI went through your details. Though masturbation cannot directly be reason for shortness of breath, it can affect your stamina and energy and therefore, indirectly it can affect your general breathing pattern. Please reduce the masturbation schedule to thrice a week and consult a physician. If you require more of my help in this aspect, please use this URL. http://goo.gl/aYW2pR. Make sure that you include every minute detail possible. Hope this answers your query. Further clarifications are welcome.Good luck. Take care."
},
{
"id": 144935,
"tgt": "Is intercourse okay when with lower back slipped disc and herniated disc in neck?",
"src": "Patient: Is it ok to have sex, whilst I have a slipped disc lower back and a herniated disc in the neck. Left leg is affected by sciatica, and left leg swollen & sore, as is left arm. Help please, sex is s good thing for me and I m hoping a recommended activity. Thank you for your time, Lisa. Doctor: Hi,Thanks for writing in to us.It is important to know how bad is your slip disc and the pain you are having. Slip disc is the displacement of cushion like discs between the bones in lower back. These cause pressure over nerves and there is pain and muscle spasm.Sexual intercourse is a fundamental human activity and also causes people to feel better. You might participate in sex that is slow and gentle and place a pillow under the lower back to support it. Please have a communication with your partner and have sex in a position that is comfortable to you. Sex is definitely recommended as a slow exercise. Please do not worry."
},
{
"id": 92189,
"tgt": "What could be the reason for having abdominal and back pain?",
"src": "Patient: hi am29 years old,every night after dinner i have stomach pains,after dinner i wait 3omins to drink a glass of water,after drinking the water i start to have abdominal pain and back pain also,i have gastric problem,but am really confused now i dnt know wats inside is it an ulcer or cancer am worried Doctor: If you have not done an USG, then do it first, after that proceed to an upper GI endoscopy if the results of USG inconclusive.By this time take tablet XXXX once daily in empty stomach in the morning."
},
{
"id": 213681,
"tgt": "What can I do to avoid the feeling of being a hypothyroid when I am not ?",
"src": "Patient: I have done two rounds of blood test , I haven t yet gotten the results from the second ones, the first one I had a TSH done and i was a 3.26 which is still normal but I am having ALL the symptoms of the hypo thyroid. what is there to do so I can stop feeling like I am? Doctor: hi welcome to healthcaremagic forum your tsh is with in normal limits so don't worry you don't have hypothyroidism.take good food ,do some exercise and meditation you will be allright"
},
{
"id": 173601,
"tgt": "Could fever be due to sprained ankle?",
"src": "Patient: My 6 year old daughter sprained her ankle last monday. we took her to a urgent care center, they did xray and said it was just sprained. she is still limping around and it is still a little swollen and bruised. yesterday she started with a fever. is this due to the sprain? Doctor: hello,It could be. The spraines ankle could lead to fever if it was severe enough as to destroy the structures incolved. Especially if you didn't take care of it after it happened. For example, after the injury of joints the best thing to do is to insert the joint in cold water and then to rap it up with onion and salt in order to suck and remove the fluids causing edema of the joint. If this is not done the fluids coming out because of the injury will be swolled and hurting. The ankle neess to rest as well but you say your daughter continues to move around thereafter. Therefore, after aome days in this conditions with fluid in the joint space the inflamation could occur leading to fever. However this is little probable if the skin is intact (not perdorated or torn out) as microbes dont have an entry point. My asvice: try to treat the ankle with grilled onion mixed with salt and rapping it with a piece of clothing or bandages and give paracetamol to lower the temperature. If the situation does not improve then you have to see a doctor.Best!"
},
{
"id": 214678,
"tgt": "Suggest home remedies for abortion",
"src": "Patient: i m doctor 37 year old and 8 weak pregnant i want to abort it because i m working in busy hospital.with pregnancy very difficult to work.i have 3 children last one 3 year old.please solve my problem i used cytotec 4 times but no benifit plz snd me home remidies to abort it soon.thanks Doctor: **1. since you yourself are a doctor, thus you are very well familiar that Medical termination of pregnancy laws remains the same even for ayurveda. Under the act, abortion can be provided only by a registered medical practitioner who has the necessary qualification, training and experience in performing MTP and only at a place which has the facilities, meeting standards specified in the Rules and Regulations of the MTP Act. ii. Moreover abortion is associated with serious complications, a few even life threatening ones. Therefore I do recommend all my patients to follow up with the treating doctor during and after the procedure. iii. Without an opportunity to physically examine the patient and monitor the process of termination, it is very unsafe and unethical on my part to suggest you the measures. So please visit your healthcare provider for safe termination."
},
{
"id": 69096,
"tgt": "What is the treatment for lump on left side of scrotum ?",
"src": "Patient: I ve had a hard like lump or bump on the left side of my scrotum it got to a point were it was really hurting. A day ago it popped I believe and white smelly pus began coming out, now I have a whole that is leaking pus and when I whipe the puss off the between my left leg and scrotum it becomes irritated which makes it difficult to walk. Idk what to do?! Should I go see a doctor? Thanks. Doctor: Hi! Good morning. I am Dr shareef answering your query.Yes! I think you should go to a doctor/general surgeon, as it seem to be an infected sebaceous cyst from your history. The treatment would be an excision biopsy under antibiotic cover. It is not a big issue. So do not worry, and go to your doctor to get it treated.I hope this information would help you in discussing with your family physician/treating doctor in further management of your problem. Please do not hesitate to ask in case of any further doubts.Thanks for choosing health care magic to clear doubts on your health problems. Wishing you an early recovery. Dr Shareef."
},
{
"id": 153314,
"tgt": "How to treat microvascular ischemic disease?",
"src": "Patient: I am a 60 year old male, 5'9\" tall, 155 lbs. Only significant history is a diagnosis of prostate cancer, believed contained to the prostate at this time. Seeing a doctor for ringing in the left ear, recently had a MRI to check for tumors/irregularities of the ear canal. The MRI report reads, in part,\"Axial T2 and flair sequences demonstrate small punctuate T2 and flair hyperintense lesions within the deep white matter bilaterally. This likely represents microvascular ischemic disease. Everything else in the report appears normal, I think. They did administer contrast and identified no abnormal areas of enhancement. The reading doctor's impressions are: 1. Small punctuate T2 and flair hyperintense lesions demonstrated within the deep white matter bilaterally. These likely represent small foci of microvascular ischemic disease. No confluent signal change suggestive of mass lesions or infarct is identified. 2. No abnormal areas of enhancement are seen.No mass lesions are identified.3. Grossly normal internal auditory canals. No mass lesion is seen. No abnormal fluid collections are present. I realize this is probably good news as far as the ear problem, but I'm a bit concerned with the microvascular ischemic disease part. Your opinion will be appreciated. Doctor: Hi,Thanks for writing in.Microvascular ischemic disease is a common condition found after 60 years of age. It happens due to the mildly decreased blood flow to areas of the brain. This is not a serious condition and more like an aging change and seen early in people who are having diabetes, high cholesterol and hypertension.Observing a healthy lifestyle and eating a nutritious diet helps prevent progress of this condition. Please take vitamin B12 supplements and this is a neuroprotective medicine. It will help in growth of damages nerves. Please do not worry."
},
{
"id": 7605,
"tgt": "How can I get rid of the acne scars? Have pimples on face and scalp",
"src": "Patient: i have a pimple prone skin... they only appear few times in many months.. but when they do....they leave marks..and they start 2-5 at a time.... what causes them to appear? why all of a sudden and not all along.. what can i do to prevent scars? pimples on my forehead and chin usally dont leave scars only ones onmy cheeks do. and I consulted a doctor once.. he said i dont have acne vulgaris ... i dont have pimples on my back but usually on face and sometimes on my scalp... Please help. Regards Doctor: Please mention your age and sex. your pimples can be easily treated by a dermatologist. Hormonal variations and stress are the main reasons. Do not pick your pimples which can cause scarring. once the pimples are treated u can go for acne scar treatment."
},
{
"id": 57035,
"tgt": "How to lower SGPT level?",
"src": "Patient: urgent please. i want to lower the SGPT level.i did blood tests for an embassy because i want to travel for work, the maximum level of SGPT that the embassy approves is 65. at first i had my SGPT level 71, i dieted so to lower my SGPT for a short time, the SGPT level went up to 79. i dieted even more for a longer time, the level got up even more to 82 . i need to lower my SGPT level to a maximum of 65 to get my visa accepted in the embassy, is there any medications that can work on my liver enzyme SGPT to a maximum of 65?? please urgent Doctor: hi, It is important to understand that elevated liver enzymes is a consequence of some underlying level liver disease like fatty liver or alcoholic liver disease.liver enzymes may drop down with some drugs like Ursodeoxycholic acid. you can try that drug and it is safe. Wait and see the response . All the best"
},
{
"id": 185607,
"tgt": "What causes dry mouth?",
"src": "Patient: My wife as dry mouth and really bad. She is at a point where they are thinking of removing all her teeth.She also as red finger tips and palms and for no reason when she wears socks her legs break into a rash.For the dry mouth she as tried everything sold in the drug store. Doctor: Hello DearThanks for your query, I will be most happy if I could help you out. First of all be relaxed and make your wife relax and reassure her. I have thoroughly gone through your query and found that you have not mentioned few things, so it is difficult to reach on any diagnosis on the basis of these two signs(dry mouth+skin rashes).If someone with same symptoms arrives in my clinic, I must ask few questions- Since how long time the problem is occuring??- Are you having any problem like dryness, redness or burning sensation in eyes??- Do you also feel pain in joints ??Now if we combine all these signs and symptoms i.e. dry mouth+skin rash+dry eyes+joint pain, it may take us to a condition known as Sjogren's Syndrome, which is an autoimmune disease and only symptomatic treatment can be provided.The problem may be due to any other reason also. So here our first attempt is to reach on a correct diagnosis which is very necessary to plan and initiate the treatment.So I advice you to must consult an oral medicine specialist as soon as possible so that the exact diagnosis can be made through thorough clinical examination and investigations.Hope this will help youIf there is any more query or question regarding this in your mind, you are welcome to ask me any time.Take CareRegards"
},
{
"id": 168114,
"tgt": "What causes prolonged low fever?",
"src": "Patient: Hi Doc, My son is 20 Months old, running temperatur 99 degrees since about 20 days. He has no other issue... is playful eating well, and chearful... but carring temperature and it sometimes it crosses 99 upto 99.05..my doc got him tested for typhoid and infection and say he had some infection and treated him with peracetamol and Ofloxocin for 10 days. but he still show temperature. Doctor: Hi...fever should be treated only if it is more than 100F. That too if the kid is otherwise well then you need not worry. this is not fever.Paracetamol can be given in the dose of 15mg/kg/dose (maximum ceiling dose of 500mg) every 4-6th hourly that too only if fever is more than 100F. I suggest not using combination medicines for fever, especially with Paracetamol.Regards - Dr. Sumanth"
},
{
"id": 79800,
"tgt": "What could painful ribs with pulsating sensation and nausea indicate?",
"src": "Patient: im getting very worried about a very strong pulse and kind of dull ache just below my rib cage in the middle where they separate it also makes me nauseous at times, this has been going on for around 2 years now getting increasingly worse and I ve had a sore head for a few days now Doctor: thanks or asking your queryI completely understand your problemthe most common cause of pain over that reason in young age group is peptic ulcer disease and much more commoner in smokers and people who take alcohol . if u r a smoker u should first quit smoking and drinking alcohol ,this will help u a lot with your pain .as for medications u can take a proton pump inhibitor like pantoprazole by asking your physician. if u r more worried about heart issues u can get an ecg done and consult a general physician after which u will feel relieved.thanks/regardsfeel free to ask more questionsmay god bless u a healthy life"
},
{
"id": 5427,
"tgt": "Had D & C. On puregon injection, clomid. Time to mature follicle?",
"src": "Patient: Hi, today is day 9 of my cycle.I had 5 injections of 150 iu puregon, and 5 days 150m clomid this cycle alternatively (tomorrow is the last clomid day). I did Scanning today (D9). but my biggest follicle was 8*8 others were 7... . my dr gave me 3 more injections for day 10, 11 and 12, and another scanning for day 13... do u think is there any hope to get mature follicle? How long I have time to grow the follicle? can they grow till day 13 or 14 also? and also the endometrial thickness was only 4! I had D&C on 1 march... Doctor: Hello, Thanks for posting your query on health care magic. Follicles grow at a rate of 2mm/day. 8mm on day 9 is a good size. There are chances that follicles grow at-least to 18-20mm. Ideal size of dominant follicle at the time of rupture is 18-22mm. Endometrial thickness of 4mm is not ideal for conception. Anyways, nothing is impossible. Wish you good luck. regards, Dr Nilofer"
},
{
"id": 117901,
"tgt": "What does MCV level of 111 suggest?",
"src": "Patient: Hi, I had a recent pre-op blood test and they were concerned my MCV level was 111. I have always been a drinker and currently consume, around 60 units a week, although sometimes it is zero or sometimes a little higher. I am thinking of seeing GP to discuss, should I ? Doctor: Hi, mcv is suggestive of rbc size.lts the mean value for rbc size. And its quite high in your case.so you have macrocytosis. Its quite common in alcoholics. Macrocytosis mostlydue to Vit B12 deficiency. so you need to check your vit B12 level and ps examination. if it is suggestive of b12 deficiency then take treatment accordingly."
},
{
"id": 12611,
"tgt": "Good doctor for psoriasis in Hyderabad",
"src": "Patient: iam affcted by psoriasis on past 3 years. i feel so depressed. pls suggest a good skin doctor in hyderabad.. pls help me... send the details to talathkausar91@gmail.com Doctor: HELLO WELCOME TO HCM... YOU HAVE PSORIASIS SINCE LAST 3YRS. WE HOMOEOPATH HAVE GOOD MEDICINE WITHOUT ANY SIDE EFFECT . FOR THAT WE REQUIRED DETAIL HISTORY FOR SELECTION OF MEDICINE . SO IF YOU WANT ONLINE HOMOEOPATHY FROM ME THAN MAIL ME drvipul2001@yahoo.com . MEET AGAIN FOR BETTER HEALTH"
},
{
"id": 150425,
"tgt": "Pars fracture and disc desiccation diagnosed. Using brace for back pain. Will a bone stimulator help?",
"src": "Patient: My 15 yr old son has been diagnosed by an orthopaedic dr (actually 2 of them) with a pars fracture (bilateral) and disc desiccation of L4-L5. He shut down for football season and rested in a brace for 6 weeks in October, then began PT/core strengthening for 6-8 more weeks. Back pain was better and he eased into wrestling season and did well for about a month, when the back pain came back. He has had xrays, 3 mris and most recently a CT scan. He shut down again at the beginning of baseball season and has been resting for 6 weeks. Still has back pain. Plan is currently to rest until June 1st and then try PT, swimming, core strengthening again and try to go back to football in August. I'm wondering if it is more likely that the pars fracture or the disc degeneration is causing the pain? What treatment do you suggest? Do you think he will have to have surgery of some sort to return to competitive sports? Sports are his world!! Would a bone stimulator help? Does he need a bone scan to determine if the fracture is hot or cold? Please help! :( Doctor: Hi, Thank you for posting your query. It is unfortunate that his pain has not completely settled even after about six months of rest and physiotherapy. Pars fracture pain is unlikely to persist for this long. Also, L4-5 disc degeneration pain should get better with physiotherapy. So, I would suggest further evaluation with bone scan and may be a local epidural injection for pain relief. Please get back if you require any additional information. Best wishes, Dr Sudhir Kumar MD (Internal Medicine), DM (Neurology) Senior Consultant Neurologist Apollo Hospitals, Hyderabad, My personal URL on this website: http://bit.ly/Dr-Sudhir-kumar My email: drsudhirkumar@yahoo.com"
},
{
"id": 143495,
"tgt": "What causes chronic pain in neck and back after cervical and lumbar fusions?",
"src": "Patient: I have chronic back and neck pain. I have had 2 cervical fusions and I lumbar fusion, but I m still in pain. I have numbing pain in my left leg from my hip to my foot. Can t stand having anything touching my foot. I also get what I call electric pain in my back when I go from a sitting position to standing. I can t walk or stand for very long. Sometimes it feel like my spine can t hold me up. I also get pain around my should blade. Doctor: hi Good morningthe implants in the spine make you stiff that causes pain. you need to get in touch with a Physical therapist who can help you maintain flexibility and get Functional"
},
{
"id": 189136,
"tgt": "Sensitive teeth, throbbing pain. Due to reduced enamel?",
"src": "Patient: I've just started getting sensitive teeth (front top & bottom), assuming it was related to cavities I made a dentist appointment, however he said I've been looking after my teeth and they were all ok. He said the ache would be caused by reduced enamel & to use sensodyne, however this has made it worse and when I go for a run the throbbing pain after is awful..... What is causing this? Doctor: Hello and welcome,The sensitivity may be due to -bone lossperiodontal pocket,gingival recession and regressive changes like cervical abrasion and erosion.I would advice you to brush your teeth using soft bristled toothbrush in back-forth and up-down motion.Avoid vigorous brushing methods.Get your teeth cleaned.Bone loss has to be corrected by bone grafting methods.Exposed dentin as well as root portion of tooth has to be corrected using tooth-coloured filling materials.Hope this helps."
},
{
"id": 172133,
"tgt": "Suggest remedy for frequent cough and fever",
"src": "Patient: my kid is know 3 years 3 months but every frequent she suffering with cough and not able to breath properly , dr said she at initial stage of asthama, I am very much worried? And one more since she is 3 years old but 10 to 15 times she is effected with fever? Doctor: since you said that your child is suffering from initial stage of asthma i am expecting that your kid do not have any night exacerbations of asthma . And since this is initial stage you can go for the routine short acting beta agonist for the treatment. Go retrospectively and watch out for any any allergens or any irritants causing breathlessness to child and try to avoid it. I saw quite a few cases of paediatric asthma cases who got re-leaved as the age progresses . but the recurrent episodes of fever must be investigated for any underlying causes . go for a routine CBC, ESR, chest X ray PA view can you tell me if any abnormal sounds from the chest or any other symptoms like cold, fatigue or any weight loss or reduced appetite"
},
{
"id": 88828,
"tgt": "What causes abdominal pain with swelling at base of ribs?",
"src": "Patient: Hi, may I answer your health queries right now ? Please type your query here...my 13 year old daughter has had some right quad abdominal pain for a couple of weeks that is has progressively gotten worse...especially after running in track or playing a basketball game. I also feel some swelling at the base of her ribs...possibly liver...spleen...? Any suggestions? Doctor: Hi.The commonest reason for pain on the right side after running or playing in a child of 13 are :AppendicitisCholecystitis due to stones in the gall bladderLiver infection like hepatitis or hepatic abscess.I would advise you the following : Blood- complete blood picture, kidney functions tests, blood sugar fasting and post-lunch, Urine- routine,microscopy, culture and sensitivity ( before start of antibiotic)Ultrasonography of abdomen.Once a proper diagnosis is made go for wither a medical or surgical treatment as is advised ."
},
{
"id": 34890,
"tgt": "How to treat staph infection?",
"src": "Patient: for 10 yrs now i have been looking for final solution to this illness but to no avail, does staph has cure? please doctor in the house i want to no. i started having this symptoms before i knew a woman. i have taken many drugs which are sensitive to the staph still the signs are still there some of the symptoms are : worm movement all over the body, pulse on the joystick with white pimple like liquid, weaks the joystick, tiredness .etc can any one who has suffered from this and cured it give me solution, cos am dying i silent for the bacteria i do not know how it enter my body. i thought is spiritual problem but i went to lab and i they say is staph culture: heavy growth of staph areaus... i have taking all kind of anti botics and herbs nothing works what can i do next. i tested hiv it was nagative. advise pls i will appriciate and thank your specially for your genuine solution. i suppose to get married this yr but the fear of staph and i do not want my partner to get it, the worst is that the lab report say it has affected my sperm motility. why me why staph!!!!!! help me pls and pls Doctor: Hello dear,Thank you for your contact to health care magic.I read and understand your concern. I am Dr Arun Tank answering your concern.Such severe infection can only occur if your immunity has weakened. I advice you should think of testing for your immunity.As you have tested yourself for staphylococcus you should also test its sensitivity for routine antibiotics. Please also test for MRSA testing.Taking the drugs according to report will cure your infections and you are free from the infection later on. If its MRSA than further treatment with mupirocin is necessary. This will clear up the staphylococcus from your body.Please maontian good living habit which includes daily bath two times a day. Good diet which includes fruits, green leafy vegetables, high protein diet.You can surely clear this bug from your semen if you take this much care.I will be happy to answer your further concern on bit.ly/DrArun.Thank you,Dr Arun TankInfectious diseases specialist,HCM"
},
{
"id": 84799,
"tgt": "Can intake of tranexamic acid and mefenamic cause clotting in next cycle?",
"src": "Patient: Hi. I am Aastha. I would like to know if taking Tranexamic acid + Mefenamic acid to stop excessive bleeding during periods causes excessive clotting in the next cycle? I took this medication last month and this month I am getting heavy clotting; like nothing I ve seen before! Doctor: Hello, If tranexamic acid was prescribed by your doctor, I think you should consult with him/her again about stopping taking this medication this time. To me, you should be using only mefenamic acid this time. If you will notice excessive bleeding, then, take tranexamic acid. Hope I have answered your query. Let me know if I can assist you further. Take care Regards, Dr Albana Sejdini, General & Family Physician"
},
{
"id": 59497,
"tgt": "Chances of damage to the liver on taking benedryl for a long term?",
"src": "Patient: 6 months ago i completed hep c treatment i am also alcolic only 2 weeks sober when I had a liver biopsy done four years ago I was between stages 2 and 3 of my liver I consume alot of benedrye over several years I have used it place of alcohol may times what if any damage could the benedryl do to my liver im talking for periods of six month intervils around 100 a week Doctor: Hello, Hepatitis C and alcohol are the two main causes for liver cirrhosis. Benadryl,in large doses,cause liver damage and since you already are a patient of cirrhosis,you should not consume either alcohol or Benadryl.Better be away from these,otherwise it may progress towards end stage liver disease very fast. Best of luck Thanks"
},
{
"id": 61554,
"tgt": "How can a sore lump on the tibia be treated?",
"src": "Patient: Hi, Over 3 weeks ago I was playing ice hockey and took a shot off of my shin pads, which left a lump on my tibia, the bruising has gone but i still have a lump (half the size of golf ball) that is sore to the touch. What, if anything, should I do about it? I have exercised on it since so it is not broken. Doctor: Dear user,When you get a hard enough blow, it causes blood to collect under the skin if there is no wound Hematoma, as we call it. That usually takes 4-6 weeks to go away, even after the redness is gone. it will usually change its colour from red to purplish to green and yellow before it completely fades away. If this continues to decrease in size, you don't really need to do anything. Just take pain killers and cold compressors as needed. If it becomes redder, or the swelling becomes more painful or increases in size, there are chances it has got infection. In that case you'll have to go your doctor to get it checked. In rare cases it needs operation to clean. There are very minimal chances that these kind of lumps persist after a few weeks, if it does persist though, you might need an x-ray to rule out small fracture, or any other kind of skin lump which you only happened to notice after the injury.Do write back if you need more help.Thanks"
},
{
"id": 168614,
"tgt": "Suggest treatment for lethargy and painful cervical lymph nodes in a child",
"src": "Patient: HI, my 3 year old son has been ill for 9 months, it started with a virus as such. He suffers with occasional night sweats and a fever every 21-28 days. He has a poor appetite although we do make him eat, he drinks plenty of fluids. He is lethargic and tired and is unbearably tired by friday. He looks washed out and tired. The last letter from our consultant stated He has persistent 1.5cm mobile shotty cervical lymph nodes and inguinal lymph nodes present on the right side of his nec, but with no obvious hepatosplenomegaly or masses. His abdo is soft and non-distended and no abnormalities of gait or limp are present. His bloods & tests include a -ve bone scan, normal AP xray of pelvis and chest, normal abdo ultrasound. His FBC shows mild microcytic anaemia, his HB 11.9, MCV 78, Ferritin 13, with slight lymphcytosis up to 5.2 x10*9/L, but a normal film. -ve serology for toxoplasma coeliac and ANA, normal u&e, LFTS, CK and thyroid and glucose Our consultant is saying its is post viral fatigue, but after 9 months ? could this really be the answer. Our 3 year old is now hoarse and complaining of pain where the glands are swollen. Can you offer any further advice ? many thanks in advance Doctor: Hi,From history it seems that he might be having possibility of Primary complex producing long lasting fever, generalized lymphadenitis, weakness and fatigue.He might require, Montoux test,Complete blood checking with ESR.After investigations one can have some clue about his chronic fever.Consult pediatrician and get examined.Ok and take care."
},
{
"id": 5866,
"tgt": "Trying to conceive. Back pains, carvings, sore breast, diarrhea. Pregnancy test negative. Hgc levels not high?",
"src": "Patient: Well we have been trying to conceive and have been baby dancing quite frequently throughout November. I have woke up with a to what I believe an allergic reaction on my breast nod left arm , serious lower back pains off and on, cravings, hungry spells constantly, creamy Milly discharge, headaches, sore breasts and diarrhea . However a negative on the cheapie pg test. Could my Hgc levels not be high enough? Doctor: Hi, Thanks for your query You have not mentioned your last menstrual period , so it is difficult to interprate. Actually in very early pregnancy ,pregnancy test may come negative depending on the sensitivity of your card test. You should recheck after one week if you dont get your periods. The symptoms are nonspecific and does not helps in diagnosing pregnancy. Best of luck and take care. Hope i answered your query.."
},
{
"id": 155273,
"tgt": "How to treat stage 4 ovarian cancer?",
"src": "Patient: I have been diagnosed with stage 4 ovarian cancer in Feb this year. I have had 6 cycles of chemotherapy and was due to have a debulk operation.....I was told there was no measurable cancer on my scan and they have decided not to do the op yet. I am worried that with no op the cancer will come back quicker....does this happen without the debulk op? Doctor: Thanks for your question on HCM.Many ovarian cancers are chemosensitive and respond great with chemotherapy.So your cancer is reducing with chemotherapy.So better to continue with it.Stage 4 ovarian cancer needs debulking surgery only when tumour is not regressing with chemotherapy.So no need for debulking surgery in your case."
},
{
"id": 36638,
"tgt": "What causes Sleeplessness, weak sensation and less working efficiency?",
"src": "Patient: sir, i had continued fever for last 20-22 days. Now fever is not there but Antigen 'H' continues in the range from 1-160 in WIDAL test. Symptoms developed post fever : Sleeplessness , weak sensation , working efficiency slow, uneasiness in understanding/grasping. Kindly suggest. Doctor: Thanks for your Query at HCM!You have not mentioned about your treatment?If you have completed the course of antibiotics and fever has subsided I see there is no need to be concerned. \"H\" titre usually persist for 6-8 weeks before they starts to decline. You must eat healthy diet, take some mineral and vitamin supplements.I see no serious problem with you unless you have some serious abdominal complaints.Happy to take more queries!Taker care!"
},
{
"id": 211572,
"tgt": "Using lexamil escitalopram as antidepressant medication for post partum depression. Sufficient?",
"src": "Patient: Hi doctor I am using lexamil escitalopram 10mg tabs daily, as an antidepressant medication, after I have been diagnosed with post partum depression, and is also undergoing therapy. Do u think that this is good enough? The other problem also though is that I have gained over 20kg while on this medication, and is a bit concerned about my weight. Do you have any advise for me on how to loose weight; while still on these antidepressants? Doctor: Hello,Thanks for choosing health care magic for posting your query.I have gone through your question in detail and I can understand what you are going through. Escitalopram is a good drug and is mostly free of adverse effects. Weight gain is a problem with some. The best way to take care of this problem is aerobic exercises with food restriction. If problem persists then other option is to start on glycomet.Hope I am able to answer your concerns.If you have any further query, I would be glad to help you.In future if you wish to contact me directly, you can use the below mentioned link:bit.ly/dr-srikanth-reddy"
},
{
"id": 8683,
"tgt": "Rhinoplasty done, disappointed as nose seems to be tilted at the right side after surgery. Could it be swelling?",
"src": "Patient: i had a rhinoplasty 6 days ago.my cast was removed on 4 day. i asked the dr. to pinch my nose at the bottom. i was happy with the result but i am very disappointed my nose tilted at the right side after surgery. i am worried is it just swelling or it is crooked.. it give unusal shape to my nose everyone cn easily notice that.. Doctor: hello after rhinoplasty the swelling remains for around 2-3 weeks but it should not give deviated or tilted appearance to the nose. for now just wait for the swelling to get reduced then only you will be able to know the difference. if needed revision rhinoplasty can be done latter on to correct any deformity regards"
},
{
"id": 86350,
"tgt": "What causes pain in lower abdomen after taking HMG massone injection?",
"src": "Patient: hy dr plz tel me about my health my age is20 yrs and i was married in9august2010 i took medicine for concieving a baby after2months of my marriage first 2month dr gave me medicine but i was not pregnant then she gave me inj hmg massone this time again i was not pregnat in jan 2011 my husband went abroad and after that medicine i hve pain in lower abdomen i m so confuse why there is no pregnancy and pain if there are side effect of medicine and now what i have to do Doctor: I think you are misguided about your complaint. there is no need of treatment for pregnency in just 2 months of mairraige. Also you say that your husband went abroad and you take treatment for pregnancy. Why? Primary thing is to live both of you together for atleast 1 year. Your problem will solve without any treatment. If not than go to doctor. Please stop all the medicine you are taking. Hormonal inj can give rise to paing in pelvic region."
},
{
"id": 138262,
"tgt": "What causes a sudden loss of strength in the neck and head?",
"src": "Patient: I have a 20 month old son.... he has an issue. not sure what it could possibly be. He will suddenly lose the strength in his neck and his head will tilt to one side. once that happens he will become very lethargic and then he is unable to walk because he has no balance. then.... 30-40 mins after this incident starts, he will vomit. I have taken him to the drs. several times. tests tests tests... no results :( any ideas?? Doctor: Hello,Welcome to the magical world of health care, I went through your query, and these are symptoms of problem with his brain, and the tests will only be significant when taken during the test. see a good neurologist.I hope my advice would have been useful, in decision making regarding your treatment, still if you have any clarifications or doubts feel free to contact back.I hope a 5 star rating if you feel guided in your treatment,Thanks"
},
{
"id": 16187,
"tgt": "Using betnovate cream on face, skin getting itchy and red, small zits on forehead, cheeks, chin. What to so ?",
"src": "Patient: Hi, i have been using betnovate cream on my face for years now. I wash my face before going to sleep every night and apply it on my face. I tried to stop using it but can t cos my skin gets very itchy and red and small zits start appearing on forehead , cheeks and chin if i do not apply it on my face for a day even. I have a combination skin and i started using betnovate cream without any consultation with a doctor. I am 24 years old. Now I have stopped its use from 3 weeks. My skin was itching the first week and red and small zits started appearing and now it has left dark spots. I am on homeopathic treatment my doctor told me to apply cold milk for itchy skin and it worked but what about the dark spots. What should i do? please suggest Doctor: Hi Betnovate is a strong steroid - long term use has several hazards, including skin atrophy. Your skin may end up worse in the end! You are wise enough to have stopped now. Self medication is fraught with dangers! It is wise to see a skin specialist - generally, they may give antihistamine, moisturising lotions, and if there be a need for steroid, they will advice the least harmful one like simple hydrocortisone. I am not competent to comment on other systems of medicine! Good luck"
},
{
"id": 50239,
"tgt": "Pressure in bladder, frequent urination. Diagnosed as cysts in kidney. Treatment?",
"src": "Patient: the nurse gave me the results of my ultrasound and told me that I have simple cysts on my kidneys and that in six months I should consider having another ultrasound. My question is is that the reason I have such pressure in my bladder and such frequent unrination and what can be done? Also, shouldn't the dr. be calling me with these results and informing me what steps are next to be taken? Doctor: good day!simple renal cysts are benign condition and rarely cause any signs and symptoms. the nurse is right in saying that we usually recommend biannual ultrasound to monitor these cyst for growth rate and changes in appearance. I would suggest doing a urinalysis to check for infection which is the most common cause of burning sensation and urinary frequency. the renal cysts are not related to your urinary complaints. I hope I have succeeded in providing the information you were looking for. Please feel free to write back to me for any further clarifications at: http://www.HealthcareMagic.com/doctors/dr-manuel-c-see-iv/66014 I would gladly help you. Best wishes."
},
{
"id": 53953,
"tgt": "What treatment is suggested for Carcinoid syndrome?",
"src": "Patient: This patient who is diagnosed with Carcinoid Syndrome and was operated for same. His Octreoscan was also positive i suppose. Now since his symptoms have recurred search on the primary tumor has to be done by repeat Octreo scan or CT scan with special look at the Pancreas and Retroperitoneum. Doctor: Hi and welcome to Healthcaremagic. Thank you for your query. I am Dr. Rommstein, I understand your concerns and I will try to help you as much as I can.It is treated by removing promary tumor so you should do scan to verify exact locus,then treatment can be planned.I hope I have answered you query. If you have any further questions you can contact us in every time.Kindly regards. Wish you a good health.DR. Ivan Rommstein"
},
{
"id": 49328,
"tgt": "Pain in kidneys after having stone in each kidney but was told that they are not obstructing. So, could the pain be from a pulled muscle?",
"src": "Patient: Why does ER doctor tell me that I have a kidney stone in each kidney - one is 9mm and the other 6.5 and said I shouldn t be in pain from them as they are in the kidney not obstructing. They said I pulled a muscle. I want them out and I see my urologist tomorrow. Will they take them out if they are located in the kidney. Doctor: Hi, Welcome to Health care magic forum. As your doctor describes, you have big stones in the kidney,and not obstructing. So they are pain less. At this position, doctors may not prefer to touch them. I advise you take plenty of fluids to have plenty of urine. It may make them desolve, and may bring in the way of the flow of the urine. Then it may cause pain, but make a way for treatment. I advise you to consult a urologist for diagnosis and treatment. Drinking water is continued, and some drugs to desolve the stones may be prescribed. Some doctors may prefer to keep a stent, to make a way for passing the stone, some may prefer to crush the stone from out side with out an opening in the skin. The powder will come out with urine. Take more of green leafy vegetables, pulses, sprouts, and protein rich foods to have resistance during the procedure. Wishing for a quick and complete recovery. Thank you."
},
{
"id": 174568,
"tgt": "What is the treatment for swelling in ear?",
"src": "Patient: Hello Doctor, My 26 days old baby has a swelling in left ear (part connecting ear and the face). When consulted with the local child specialist doctor nearby suggested to give 1 drop of Otogesic ear drop. When consulted with the baby s term doctor (from the hospital where we usually go), said that its not needed. Will there be any side effect in giving Otogesic ear drop to the baby? Please advice. Thanks, Madhanagopal Doctor: Hi,Thank you for asking question on health care magic.Otogesic drops do not do any harm if instilled in the ear.You may stop it if not indicatedHope this answer will serve your purposePlease feel free to ask any more queries if requiredTake careDr.M.V.Subrahmanyam MD;DCHAssociate professor of pediatrics"
},
{
"id": 199146,
"tgt": "What does the large hard lump in groin area with pus and blood indicate?",
"src": "Patient: There is a large hard lump on my groin area- in the groove where my inner thigh meets pubic hair. It has a spot like lump on top- which I can squeeze and blood mixed with watery puss comes out. It is getting bigger- not visibly but I can feel it with my hands. What the hell is it? Doctor: HelloI appreciate your concern It could be a infected boil or a sign of STI securely transmitted infectionIf you can upload a photograph and provide me more information like relevant medical history and since how long it has been there, I will definitely help you in best possible way.I see patients with similar complaints in my clinic and I would advise them for a course of antibiotics under supervision with cleaning and dressing with antiseptic solutionIf it doesn't heal in a course of a week I would have advised for baseline blood and urine investigation to establish an underlying cause.Maintain good hygienekeep it clean an apply antiseptic ointmentThanks for your questionBest wishes"
},
{
"id": 164633,
"tgt": "What causes platelet count of 1,000,000/ccm in infant?",
"src": "Patient: Hi I have a 6 month old daughter. She went for a routine check up yesterday and they took blood. They called me back today and told me to bring her back in to take more blood because she had a platelet count of 1,000,000/ccm. What can be the cause of this? Should I be worried? Doctor: Hello, no need to worry,it is absolutely normal if the baby is taking feed properly, no evidence of bleeding from any site ."
},
{
"id": 225994,
"tgt": "Irregular periods after stopping Microgynon. Light bleeding, excessive discharge. Normal?",
"src": "Patient: I came off the contraceptive pill (Microgynon 30) last May and my periods have been irregular since. Before I was on the pill my periods were always 4 weeks apart like clockwork. Since coming off the pill my periods have been 8-10 weeks late each time and my last period finished on the 26th March. A week later I had light bleeding for 2 days, a lot of discharge and then last Thursday (18th April) my period has started. It is very light and not like my other periods. Is this normal and what could be causing this? Doctor: Hi dear user, thanks for your query.Usually, ovulation and menstrual cycles returns to normal in 3 months after stopping oral contraceptives.As per your history, you are having light bleeding, lot of discharge and irregular period. In my opinion, first of all, you should rule out pregnancy for which you can use home pregnancy test or serum beta HCG levels( better option). Hope this helps."
},
{
"id": 52357,
"tgt": "Suggest treatment for haemochromatosis",
"src": "Patient: My doctor wants me to go on a statin for high cholesterol - LDL 182; HDL 68; Triglycerides 90. I have hemochromatosis and am concerned about live damage from the statin. I have managed my hemochromatosis well for the past 5 years. Are my numbers too high to bring my cholesterol down naturally or are my risks for stroke/heart attack too high that I need a statin? Doctor: Hello, How old are you? What is your gender? Cholesterol levels are different by age, weight and gender. Healthy levels of cholesterol don't vary much for typical adults. Variation of recommendation levels tends to change due to other health condition and consideration. For LDL cholesterol levels should be less than 100 mg/dL, levels of 100 to 129 mg/dL is acceptable for people with no health issues but maybe more concern for those with heart disease or heart disease risk factors. Levels of 130 to 159 mg/dL is borderline high and 160 to 189 mg/dL is high. A reading of 190 mg/dL or higher is considered very high. Generally, the earlier an adult starts living a healthful lifestyle, the better for their cholesterol levels. A sudden change in lifestyle will help eventually, but the older a person is, the less impact they will see in cholesterol levels. High cholesterol at any age puts a person at risk for heart disease, heart attack and strokes. These risk only increase over time, especially for adults who are not taking action to reduce their cholesterol buildup. Your cholesterol levels high so it is an indication to give you medicine. If you try to change your lifestyle and the cholesterol levels not improve you should use medicine. Occasionally, statin use could cause an increase in the level of enzymes that signal liver inflammation. If the increase is only mild, you can continue to take the drug. Rarely, if the increase is severe, you may need to try a different statin. You should follow up with your home doctor for liver enzyme tests. Hope I have answered your query. Let me know if I can assist you further. Regards, Dr. Heang Chan Raksmey, General & Family Physician"
},
{
"id": 136517,
"tgt": "Suggest treatment for pain and soreness in heel",
"src": "Patient: I jumped into shallow water to save my nephew from drowning and I landed flat foot. And my heel has been in thriving pain I can t walk hardly and when I try to put my shoe on it feels like a knife going in my heel please help also the pain is in my right foot Doctor: Hello, I have studied your case.X ray foot will help in diagnosis.There is possibility of injury to calcaneal bone due to fall from height.Shoe/footwear should be with soft sole. You can use silicon pad insoles.You should not walk bare foot at home also.Avoid exposure of cold to foot.Start plantar exercises consulting physiotherapist also Physiotherapy like TENS and ultrasound will help. Take calcium supplement with vit D.You can use supportive splint till further investigation.Hope this answers your query. If you have additional questions or follow up queries then please do not hesitate in writing to us. I will be happy to answer your queries. Wishing you good health.Take care."
},
{
"id": 215789,
"tgt": "What causes shooting pain in fingers?",
"src": "Patient: I have been having painful shooting pains down my left middle finger and it is now happening on my right middle finger also. I had an EMG last week and was told I have several abnormalities. I have been referred to The Utah University in Salt Lake City. I have done some research of my symptoms and ALS seems like a possibly. I would love your feedback. Thanks, Cheryl I just turned 70. Doctor: Hi, ALS is very unlikely. The symptoms are more consistent with a neuropathic pain. You can consult a neurologist and get evaluated. Hope I have answered your query. Let me know if I can assist you further."
},
{
"id": 28664,
"tgt": "How can hepatitis C be treated?",
"src": "Patient: My husband has Hepatitis C stage 4 did not clear medicine trials. He is experiencing symptoms that are disorientation that comes on all of a sudden where he can t talk and can t function somewhat clammy feels nauseous for a period of about a minute then he gets back to himself but is wiped out afterwards. usually only happens in the morning/afternoons never in the evening curious to know if this is possible due to his liver disease with his hepatitis c got a CAT scan came back fine and blood work in the normal ranges. Going to the neurologist tomorrow but I don t have alot of faith in a diagnoses because he only gets these symptoms once a day to once every other day. Any suggestions what to possible look for? Doctor: Hello and Welcome to \u2018Ask A Doctor\u2019 service. I have reviewed your query and here is my advice. Since he is having neurological symptoms, he has got very advanced stage of the disease with central nervous system involvement. Complete cure is not possible and only supportive management is possible at this stage. The best treatment option is liver transplantation and you can discuss this with your doctor.Thanks"
},
{
"id": 143493,
"tgt": "What causes tingling of lips and eyes and disorientation?",
"src": "Patient: Yes, my daughter 29 yr/old has been sick head/chest cold for a week. went to campus doc. symptoms light headed out of it . as asked to breath hard her lips and eye balls started to tingle, heavy breathing disorientation. no psneumonia (camp doc). still feels disoriented can t think with tingly lips and eyes? help Doctor: I do not know whether she is really disoriented i e cut of from surrounding time ,space and person.Otherwise it may be simply case of over breathing causing transient alkalosis thereby reduced ionic calcium which may cause tingling,numbness and light headed sensation."
},
{
"id": 72505,
"tgt": "Suggest treatments for brown phlegm and pain in chest",
"src": "Patient: my wife went to her doctor 5 days ago bout symptoms of brown flem and pain in her chest, the doctor told her that she had fluid in her lungs and a cracked rib the doctor did not order her a chest xray or send her to the hospital she gave my wife perscriptions for prednizone 20 mg 3 times a day, avelox 400 mg 1 a day for 7 days , omeprazole 20 mg 1 2times a day, clonazepam 2mg 1 2times a day PRN, and flovent 110 mcg 2 times a day. She has been taking these religeously and still doesnt feel better she acts like shes stoned and cannot stand at all. I dont know what to do Doctor: Thanks for your question on Healthcare Magic.I can understand your concern. Brown phlegm with chest pain is more suggestive of lung infection. So better to consult pulmonologist and get done chest x ray. If chest x ray is showing lung infection (pneumonia) then she should be admitted and start Intravenous fluid and antibiotics.So don't wait at home. Consult pulmonologist and discuss all these. Hope I have solved your query. I will be happy to help you further. Wishing good health to your wife. Thanks."
},
{
"id": 33326,
"tgt": "What causes itchy back with bumps on back and top of arms?",
"src": "Patient: During the evening, mainly, my back starts to itch terribly. What causes this? I am now sitting at my desk and or an unknown reason, which is strange, my back is now itching. It is because I drink mountain dew sodas? I am a 3rd year doctoral student who works full-time, and take care of two sisters who are ill. I thought it was because of the stress I m dealing with; but need to know how to resolve the itching. Also, I have bumps on my back and top of arm. I m a 59 year old African American female. Doctor: Hi, Thanks for your question. After going through your question there may be following possibilities for your symptoms of itching and bumps over back - 1) Allergic reaction to some food stuff, environment or to insect bite. As your symptoms start abruptly this is most likely cause. 2) Fungal infection which is rare possibility as your symptoms started abruptly. 3) Scabies most rare cause. In this case some family members may have similar complaints. Try some antihistaminics like cetrizine or hydroxyzine. Apply lactocalamine lotion. Watch for symptoms like breathing difficulties, swollen lips. If there is no relief or started appearing new symptoms better to consult dermatologist. Hope this helps you, if so do vote."
},
{
"id": 116326,
"tgt": "What causes low red blood cell count in blood test?",
"src": "Patient: My boyfriend had a routine physical. His WBC is 3.54 Red blood low (slightly- cant remember) and platelets 77k. He has been running and exercising reguraly. His hip started hurting recently and they found a small extra beat in his heart. He has been trying to loose weight recently- almost no red meat or fruits. Very low Calories. We are so scared beacuse his dad suffers from b12 Deficiency and his grandfather died of blood cancer. We are very scared. Doctor: Hello and welcome to HCM,A low red blood cell count can lead to anemia.Anemia leads to tiredness, fatigue, weakness and cardiovascular disorders like missed beats, palpitations etc.I suggest you to get peripheral blood film examination to know the type of anemia and thus its treatment.Thanks and take care Dr Shailja P Wahal"
},
{
"id": 221835,
"tgt": "How long should I wait post immunization to get pregnant?",
"src": "Patient: hi. im going to kenya next month and have to have immunisations including yelow fever, hep a and b, malaria tablets and was wondering if there is a certain amount of time we will have to wait after having these injections before i get pregnant as i don t want to increase risks to baby Doctor: Hello,After such immunization schedules, it is better to wait for three months before trying to conceive. Usually, a period of two weeks to one month is suggested after live vaccine injections. Since these are only prophylactic procedures, there is no specific restriction, but, as a safety precaution, you can use temporary contraception for 6-8 weeks at least. Please use prenatal vitamins right from now on. You can also see a specialist once you settle down for further plans. When immunization is advised for Rubella in TORCH infections, pregnancy is strictly avoided for 3 months after vaccination. Hope this helps."
},
{
"id": 33287,
"tgt": "What should be done for wound due to puppy's scratch?",
"src": "Patient: Hi, my wife just was scratched by a puppy's nail(we just took it 3 days), very few amount of blood. She already flashed the wound by water and cleaned by betadine. What should we do now? Should we get the rabies injection now or only observe out puppy in the next 10 days without doing others? Doctor: Hello,Rabies virus spreads through the salivary secretions of the biting animals. The chances of transmission are very, very rare through scratch wounds. However, since there were some blood drops came out following the bite. The wound might have come in contact with the puppy's mucus, then she should definitely be vaccinated against rabies especially if the puppy's vaccination status is not known.Since this disease is 100% fatal, it is best to give the anti-rabies vaccine on 0,3,7,14 and 28 days. If the puppy is under close observation by a specialist until ten days after the scratch and remains healthy, then you can discontinue the vaccination. Besides she must also receive injection tetanus toxoid.Hope I have answered your query. Let me know if I can assist you further.Regards,Dr. Mohammed Taher Ali"
},
{
"id": 129753,
"tgt": "What treatment is suggested for bruise on the elbow?",
"src": "Patient: Last night I fell off of a swivel chair and received some nasty bruises on my elbow, hip and the pad of my right foot. In just about five hours, the bruises have gone from red, to purple to greenish and are currently yellow brown. Should I be concerned about this? Doctor: Hello,It is probably just regular bruising, however, if you are having severe pain, extreme swelling or trouble walking or using your arm then you should be checked by a doctor to rule out fracture.Regards"
},
{
"id": 88953,
"tgt": "What causes abdominal pain after foreplay?",
"src": "Patient: my age 18height 5feet 6 inchweight 70 kilosi had very heavy foreplay with my boyfriendwith our under garments onafter that i am having lower abdominal pain for abt 30 hrsmy period is due on 14th may.is there any risk.why i am getting these pains.pls hlp me? Doctor: Hi welcome to hcmYour pain is related to exertion during foreplay.you need not worry.take antispasmodics like tab.mafenamic acid 10mg ,tab.and drink plenty of water to releive congestion during sex.maintain genital hygeine.take tab.norflox 400mg if you have symptoms like burning micturition,urgency,increased frequency of urine.consult your gynecologist for curther queries.thankyou"
},
{
"id": 157806,
"tgt": "Painful hands, back pain, swollen parotid glands, prostatitis. Taken bactrim and tamsulosin. Done US. Fear lymphoma?",
"src": "Patient: I have painful hands and back pain and was told that my parated salivary glands are swollen. I take Bactrim for prostatitis along with Benacar, HCT , trilipex and tamsulosin. Could my symptoms be side effects? I have been referred to an ENT for ultrasound of my salivary glands and will be screened for auto immune factors and lymphoma. Doctor: Hi,Thanks for consulting HealthcareMagic!There is chance for these being side effects of medicines. However, physical examination is mandatory to assess this. At this time, I believe that it is prudent to discuss review about all your medications with your treating doctor who had analyzed your problem better. Anyhow, there are various other reasons as well which cause similar symptomss as yours.Let me know if you have have any clarifications. If you like my answers and wish me to answer in future, bring it to my attention: Dr Vasanth. Url link is as follows:http://doctor.healthcaremagic.com/doctors/dr-vasanth/66057Wish you good health!"
},
{
"id": 31062,
"tgt": "Can strep infection cause elevated troponin level?",
"src": "Patient: i had a mi 3 yrs ago .STEMI, arrested ,cardioverted and 2 Cypher Sents.. got strep last week and was admitted to hospital for 3 days. 1st troponin was \"normal\" 2nd was 0.10, 3rd 0.11 4th 0.09. NO other cardiac symptoms. Could the slightly elevated troponin be due to strep infection and/or the existing damage from the prior MI? I am Male 67 with controlled hypertension go to gym 5 days a week for the past 3 yrs. Doctor: Hi & Welcome to HCM.Streptococcal infection is unlikely to cause elevated Troponin levels.A troponin test measures the levels Troponin T or troponin I proteins in the blood. These proteins are elevated when the heart muscle has been damaged, such as occurs with a heart attack.Most patients who have had a heart attack have elevated Troponin levels within 6 hours and which may remain high for 1 to 2 weeks after a heart attack.Since you do not have symptoms suggestive of heart problem and the BP also seems to be controlled the elevated levels of Troponin could be due prolonged exercise during gym. However, it is better to get an ECG done to be on safer side to rule out the ischemic heart disease.Hope this answers your query."
},
{
"id": 293,
"tgt": "Is Progyluton safe to be taken for conception?",
"src": "Patient: Hi I have been trying to conceive for yrs,recently following a gyno infertility doctor,She has been monitoring my eggs until they reached around 17.8mm big and prescribed me prygnyl injection and to start progyluton tablet. I am worried about the progyluton medication is it safe that am trying to conceive? Doctor: Hello and Welcome to \u2018Ask A Doctor\u2019 service.I have reviewed your query and here is my advice.Yes, it is safe. Progyluton (estradiol valerate) is used for preparing endometrium for implantation. You can also do TSH test, prolactin test, and husband semen analysis.Hope I have answered your query. Let me know if I can assist you further.Regards,Dr. Sheetal Agarwal"
},
{
"id": 116360,
"tgt": "Is there any danger to have ESR count as 80?",
"src": "Patient: hello, i am 52 yrs female, having done a recent blood test i came to know that my ESR count is 80, it is always on a bit higher side like above 50 for me, i have also been diagnosed with gallstones recently, is there any relationship between the two? Doctor: Hi,Thanks for asking.Based on your query, my opinion is as follows.1. ESR of 80 indicates severe inflammation.2. Correlation with C-reactive proteins level is necessary. If CRP is also elevated, then possible inflammation anywhere in the body needs to be evaluated.3. Gall stones can induce mild inflammation. Associated with anemia and your previous history of a high normal - I suppose gallstones could be a cause. However, get a CRP done to rule out high inflammation.4. If ESR crosses 100, autoimmune disorders, multiple myeloma or chronic infections like tuberculosis requires evaluation.Hope it helps.Any further queries, happy to help again."
},
{
"id": 167847,
"tgt": "What causes gastric problems in an infant?",
"src": "Patient: whethere by eating nachani it provokes gases trouble in infants-my child is 7months oldgood in health,his height is 69 cms and his weight is-8.780kgs- According to my paeditrition the health of my son is excellent but he is troubled by gases trouble which disturbs his sleep everyday---please advise Doctor: if its just gas, change his feeding pattern. give small healthy feeds multiple times a day. prefer liquids more. can also use antacids for few days"
},
{
"id": 90933,
"tgt": "What causes strong pulsation in my abdomen?",
"src": "Patient: I'm a 30 yr old woman, I do not have a history of medical problems, the last few weeks when I lay on my stomach I have been feeling a strong pulsation, much stronger than in my chest. When I lay on my back I feel an area with a very strong pulse on my abdomen, near my umbilicus. I know the major artery, the aorta runs in the abdomen... So is a strong pulse a concern. Or just normal aortic flow? Doctor: Hi.Thanks for your query and an elucidate history.Very good question...If you are feeling this for the first time ,, the surgical problems concerned with this feeling like a cyst or a mass lying on the aorta have to ruled out by an urgent ultrasonography and color doppler examination ,; the the findings are positive confirm this by a contrast enhanced CT scan of the abdomen ans of necessary of the chest too... If the investigations and clinical examination by your Doctor are normal , enjoy life- it is possible that you have lost a lots of weight and lost abdominal fat... ( this is on a lighter vain)"
},
{
"id": 4125,
"tgt": "When can I conceive after having medication for PCOS?",
"src": "Patient: Hi am 24 years old. I have been diagnosed with PCOS last month. All my hormonal level is normal. My doctor have prescribed me MET DM-500 for 30 days twice daily and Uterone 100 on my 18th day to 27th day. Can I know within how many months i will get conceived. Actually, am very depressed of it since we are trying for baby for more than an year and I had this irregular periods only for last 3 months. My husband is also found with pus cells infection as ++. Later on treatment it came to + and the number of sperm cells is 28 million after the treatment. Please give me your available suggestion. Thanks a lot in advance. Doctor: Hi,I read your query and I understand your concerns.Following is my reply:1) Your husband's semen count is fine.2) Please continue the medicines an keep getting regular follicular growth scans done.Let me know if you have anymore questions.Regards,Dr. Mahesh Koregol"
},
{
"id": 13221,
"tgt": "Suggest treatment for rashes on foot",
"src": "Patient: My wife has had a rash looking thing affecting her left foot for about 7-8 years. It does not respond to any medicines that have been perscribed to clear it up. She has had acute intermittant porphyria and wonders if this could be part of the problem. Doctor: Hi Dear,Understanding your concern. As per your query you have symptoms of rashes on foot which is very common due to skin infection, sun poisoning and as dirt and moisture get trapped.Need not to worry. I would suggest you to keep this area clean and dry. You should apply warm compresses with soaked towel and apply vitamin C serum at night. If condition doesn't get better then consult dermatologist for proper examination. Doctor will examine physically along with blood test and sample for lab test. Doctor may prescribe immunosuppressants and may prescribe benzoyl peroxide ointment. Hope your concern has been resolved.Best Wishes,Dr. Harry Maheshwari"
},
{
"id": 55717,
"tgt": "What is the treatment for a fatty liver?",
"src": "Patient: hello doc,my age is 33 years and having bmi around 30,i m diagnosed with fatty liver also cholestrol boaderline high level,i take alcohol occasionaly,for last one month i have started taking live 52 tab,please tell me will this be helpful in fatty liver n how long i can take this? thank you Doctor: Your obese and alcoholic with fatty liver. Taking liv52 is herbal medicine for improving liver functions and is supportive. Unless you improve your lifestyle by doing exercise to reduce weight and cholesterol and stop alcohol your liver condition will not improve. Liv52 and lipid lowering drugs are just supportive and usually given for 1-2 months as per patient response to drugs. Hope I have answered your question. If you have any further questions I will be happy to help."
},
{
"id": 37595,
"tgt": "Can shingles cause any side effects to the surgery?",
"src": "Patient: I have surgery scheduled in 2 weeks I was diagnosed with shingles last Saturday morning after noticing the area puffy,swollen and very tender.I immediately started taking the prescription right away. If I do not have too much discomfort can the surgery go as planned? Doctor: Thanks for contacting HCMI am sorry to her that you have shingles. What complicates the matter is that you are scheduled to have surgery in 2 weeks. My recommendations is that you contact your surgeon and inform them of the shingles. More than likely they will postpone the surgery until after you have recovered from shingles. Hope this answers your question. Please contact us again with your health care questions and concerns"
},
{
"id": 186443,
"tgt": "Cause for pain in the gums and headache?",
"src": "Patient: Hi, my husband had 8 teeth extracted a few years ago, the top and bottom back teeth. He recently has been getting severe pain in the gums where the teeth were along with headaches. This pain comes on quickly and not consistently, and lasts for a while. Is there anything we can do? Doctor: HiThanks for writing in.Pain could be due to traumatic multiple extractions.Just continue with course of analgesic & antibiotic.Do warm saline rinses 4-5 times a day.Consult dentist if problem persists.RegardsDr. Neha Sumra"
},
{
"id": 184515,
"tgt": "Is swollen gums with yellow stuff and hole in gums of a child concerning?",
"src": "Patient: My daughter has her 6 year molars coming in on the top, her bottom gums are swollen on one side it hurts the most when I took a look today she had yellow stuff on the gums I took a q tip and rubbed it off it was hard now she has a hole in her gums. She I take her to be seen? Doctor: Hello and Welcome to \u2018Ask A Doctor\u2019 service. I have reviewed your query and here is my advice. I read your query. As your daughter had swollen gum with yellow discharge from gum this can be due to periodontal problem. May be there is partially erupted tooth, and there is inflammation in gums or due to carious tooth present. Don't worry but you should take her to dentist and go for oral examination and investigations. IOPA x-ray of the region of swollen gum if there is carious tooth or any gingival problem then go for its treatment.In the meantime, advise her to do warm saline gargle 2 to 3 times a day, proper brushing twice daily, regular use of mouthwash like kidodent once daily. Hope it will help you. If your have futher query feel free to ask.Wishing your daughter good health.Regards, Dr. Priyanka Tiwari"
},
{
"id": 79431,
"tgt": "What are the symptoms of asthma?",
"src": "Patient: I have been dealing with a nice little interplay between pneumonia and asthma for several months now. That finally seems to be getting sorted, but over the last few days I ve developed what I believe is diaphragm pain. The pain is constant, and also sore to palpate. Do I need to do anything about this, or will it resolve with time? Doctor: Thanks for your question on Health Care Magic. I can understand your situation and problem. In my opinion, you should consult pulmonologist and get done clinical examination of respiratory system, chest x ray and PFT (Pulmonary Function Test). As pain can be due to either pneumonia or worsening of asthma. So chest x ray is needed to rule out lung infection (pneumonia). PFT is needed for the diagnosis of worsening of asthma. You may need antibiotics, inhaled bronchodilators and inhaled corticosteroid. So better to first diagnose yourself and then start appropriate treatment. Hope I have solved your query. Wish you good health. Thanks."
},
{
"id": 15747,
"tgt": "Raised red spots that go white after itching, started on legs, spread to stomach, arms and face. Medication ?",
"src": "Patient: I have raised red spots that go white when I itch them, the are extremely itchy and the larger ones can be felts under my clothes! They started with a few on my legs an now seem to be spreading every day to my stomach, arms an face:( I do work with horseses but have not been by one for 2 days now but they did first appear in the night after I finished work Doctor: HELLOThanks for visiting healthcaremagicREGARDING your problem of itchy red spots turns white may be due occupational hazards.NEXT it may be purpuric rashes which should be evaluated by doing Lab tests."
},
{
"id": 54213,
"tgt": "What causes unconjugated bilirubin reading to be very high?",
"src": "Patient: Hi, my unconjugated bilirubin reading is abnormally high, while the conjugated reading is normal. Besides, all other liver function indicators are normal. I had my gall bladder removed several years ago too. What do think is responsible for this unusual readin. Olayinka Doctor: Thank you for posting query.noted that you have abnormally high uncojugated bilirubin.increased bilirubin is termed as Jaundice. Bilirubin is conjugated in the liver.Bilirubin is derived from the breakdown of heme. Hemoglobin is a complex protein found in Red blood cell that plays important role in transport of oxygen/CO2 from lung to tissues and vice versa.Increased breakdown of hemoglobin will lead to increased heme production and ultimately increased bilirubin. These abnormally high bilirubin can NOT be conjugated by liver and circulate in blood.therefore, high uncojugated bilirubin indicates \"Hemolytic Anemia\" (abnormal (increased)breakdown of red blood cell.)You need to do following test:Complete Blood cell count (CBC)Peripheral Blood SmearLab results maybe posted here or visit your treating physician / hematologist/ hepatologist.my name \"Dr Tayyab Malik\"you may find me in gastroenterology panel@HCM.you may also put a \"direct question\" to me via this portal or by contacting HCM admin.any further questions are welcomed.wish you good and sound health.regards,Dr Tayyab Malik"
},
{
"id": 193750,
"tgt": "What causes slight breast development in men?",
"src": "Patient: dear sir,height 194 cmweight 83 kgslight breast developementenough beard growth ,pubic hair ,sparse chest hairtesticle size max of 9 ml i m worrying of having klinefelter syndrome in my teenage time i couldint mingle with peers still i hav that problem...please tell me how can i get treatment for this in chennai or delhi Doctor: Hello. I have reviewed your query and here is my advice. You have not indicated your age. Breast development should be treated with chest muscle physical exercise for 6 months. If no improvement, then we can think of checking your hormones.Hope I have answered your query. You can contact me for treatment options. Let me know if I can assist you further. Regards, Dr. K. V. Anand"
},
{
"id": 49257,
"tgt": "How to diminish the size of 7 mm kidney stone so as to facilitate its throwing out process?",
"src": "Patient: I have a 7mm kidney stone. Last time i was at the hospital it was in my ureter. It feels like it may have moved to my bladder. Is there anything i can do to make it smaller for when it comes out. I m afraid of it becoming stuck or tearing once it comes out if that sensitive place. Doctor: HelloThanks for your query,based on the facts that you have posted it appears that you have 7mm stone detected in your kidney and which has moved down in to bladder.There is no medicine that can reduce the size of the stone.The stone of a size 7 mm will pass out through urethra freely with slight discomfort when it comes out.Please ensure to drink more water so that it will pass out with forceful urine flow. Dr.Patil."
},
{
"id": 127673,
"tgt": "How are the symptoms of post-concussion syndrome treated?",
"src": "Patient: I had a bad fall on the ice back in late December. I bruised my tailbone and had a concussion and bruised my knee. Everything slowly healed but I m still having a vertigo type dizziness at night when I turn over in bed and upon rising in the morning. I checked with my doctor in early March and she said it was okay because of how hard I hit the back of my head on my concrete porch. But yesterday, I started having like flashes of lightening in my peripheral vision in my left eye. Any ideas what s going on? What s my next step? Doctor: Hello and welcome to \u2018Ask A Doctor\u2019 service. I have reviewed your query and here is my advice. Go for CT Scan head. I hope this information has been helpful for you. Hope I have answered your query. Let me know if I can assist you further."
},
{
"id": 182721,
"tgt": "What causes nausea and vomiting after tooth extraction?",
"src": "Patient: my friend has her tooth extracted 4 days ago, shes been nausious and vomiting on off the last 3 days, today she has been vomiting for 2 hours on and off, she described it as light green stomach acide, dark brown liquid, and bright red liquid (maybe blood) Doctor: Thanks for your query, I have gone through your query.The nausea and vomiting can be secondary to the drugs prescribed after extraction particularly antibiotics and analgesics. These drugs causes severe gastritis and cause vomiting and nausea. Nothing to be panic, If i am your treating doctor I would have suggested you take antacid like ranitidine 150mg BID before food and aniemetics like domperidone BID. If the vomitind does not stop or if it is more than 4-5 times a day then immediately consult a general physician. Because you will get dehydrated and feel weak so consult immediately. Consume lot of liquid, drink tender coconut.I hope my answer will help you, take care."
},
{
"id": 117750,
"tgt": "Should I be worried for being anemic?",
"src": "Patient: My husband is 34 years old and 6'4 tall and weighs 220 pds. He is very strong man, works out regularly and owns his own contruction business. He went in two months ago for sinus issues and the Dr. wanted him to come back for blood work because he is at age where it shoudl be done. He went in and his dr told him he is anemic with a hemoglobin level of 13.6 and hemocrit at 40.2%. Should we be concerned? Doctor: No. You dont worry about that. HemoglobinLevel 13.6is not consider as low.it is almost normal and no need of concern."
},
{
"id": 175372,
"tgt": "Suggest treatment for cough , cold and fever in a child",
"src": "Patient: Hi, My 3.5 years old son is suffering with High Fever, cold and cough and has 101.6 temperature. We are giving him calpol every 4 hours and Fenpaed (Ibuprofen) whenever it touches 102. I also have also given him Azibact-100 in afternoon 12Pm. Can you please let us know if we are doing the right think and can i give one more dosage of Azibact at night after dinner. Regards, Hemant Doctor: you should give antibiotic 10mg for kg of child. you wait 2-3 days then you will receive effects. You also give tulsi juice, sualin 1 tab 3 times, rince gurgle with chlorhexidine or soda and salt"
},
{
"id": 196752,
"tgt": "Suggest treatment for erectile dysfunction",
"src": "Patient: I'm 26 yrs old male ,6ft n 82 kg,non smoker occasional drinker being in a relationship for more than 6 yrs our love life was really good. I love her a lot,bt for the past 1 month my penis cannot hold it's erection I'm really troubled because of this and Iam making her life also miserable what should I do ? Doctor: Dear user,Thank you for writing to HCM.I understand your concern about inability to hold erection hence causing trouble in sexual satisfaction.Let me know whether you achieve and maintain a erection in sleep, in the early morning or during masturbation then there is no need to be worried about.It is likely due to performance anxiety. But If you cannot, then consult a urologist or psychiatrist to rule out other causes of erectile problem for effective treatment.Wish you good health."
},
{
"id": 26318,
"tgt": "Is there any relation between blood pressure and feeling run down?",
"src": "Patient: good morning... my blood pressure is around 180/115 at the moment quite regularly.. but i have started feeling really run down and achy. are the two related? have recently moved back to the UK and have registered with the doctors surgery, but I can't get an appointment for another 6 days. Doctor: hello,I have gone through your query.Thanks for using HCM.Your BP of 180/115 is really very high.Your symptoms may be because of this.Irrespective of symptoms you must consult your doctor at earliest to start treatment.In your case Ramipril 5 mg along with Chlorthalidine 6.25 mg may be started .kindly consult physician at earliest for quick control of BP and complete evaluation.My best wishesDr.Rajesh Teli,MD."
},
{
"id": 8389,
"tgt": "Suggest treatment for recurring pimples",
"src": "Patient: I have a pimple which is almost ripen and filled with pus on my back just above the buttocks..is there any ointment that can heal it faster. this now fifth time i get this in different parts of my body....fed up of eating antibiotics..is there any solution doctor? Doctor: Hi,As you said you suffer from pimples, also called acne vulgaris. There may be different types of lesions in this disease like white comedones,black comedones, inflammatory lesions,cystic and conglobate acne..etc. The lesions may be precipitated or excerbated by stress,anxiety,sleeplessness,drugs like steroids, hormonal imbalance,dandruff,oily food..etcI generally treat my patients explaining about long term treatment and to avoid anxiety related with treatment. Antibiotics like azithromycine or minocycline in proper dose may be taken. Isotretinoin in low dose 5 mg may improve the lesions. Clindamycine with benzoyl peroxide 2.5 % may be applied on the lesions at night. Antiseptic facewash may be used to cleanse the face.Patience is necessary to get relief. You consult dermatologist for firm diagnosis and treatment.Thanks.Dr. Ilyas Patel MD"
},
{
"id": 193598,
"tgt": "Suggest treatment for inhibited ejaculation",
"src": "Patient: sir, i ve tried masturbating since my maturity but never ejaculated once in my life and even not during sexual intercourse...... i m worried about the insensitivity or numbness of my penile glans ..... i think it is congenital. Is this caused by anejaculation or neurological problems or anything else.... i want to experience orgasm atleast once in my life.... give some solutions Doctor: Hello, Please tell us your age. There are many medical condition with which you experience lack of ejaculation. Talk to your urologist regarding this. Hope I have answered your query. Let me know if I can assist you further. Take care Regards, Dr K. V. Anand, Psychologist"
},
{
"id": 214536,
"tgt": "What is the home remedy for faileriasis?",
"src": "Patient: I am 50 yers old and i am suffering from faileriasis3 days before i had fever.my right leg is swollen a littele more from the day of fever.can u give me the solution ,is there any herbal medicine,what about using ginger.tulsi,onion,lemon leaves.can i eat chiken?just for 1 day. Doctor: hai,there is no home remedy to treat filariasis.you can take such home remedy as you mentioned in your quote will help in fever. but there is no study evidence for filariasis infection. taking chicken is not advisable when you experiencing fever.thank youhope i answered your query"
},
{
"id": 138246,
"tgt": "What causes electric shock like pain in inner hip?",
"src": "Patient: Recently I ve had lower back pain and am now, on occasion, suffering from simultaneous electric shock like pain in both my inner hip/groin along with sharp electric shock like pain in the back of my foot. The pain only lasts a few seconds but is quite unbearable. Any ideas? Doctor: Here you have told about symptoms like back pain and shock like pain or sensatiin in inner hip as well the back of the foot. This kind of pain is usually related to back pain(spinal issue related with disc and nerve) only and rarely it may also be related to vitamin B deficiency along with less water intake.I suggest to visit a orthopedic and get your self examine properly, if needed go for x ray and then do take blood report to know about vitamin B level in body. As per the report and evaluation doctor will suggest for the precautionary treatment with medication. I suggest also visit a physio for postural correction, advice on back and leg strengthening and stretching exercises. And also do follow it.You can also use a hot bag at your back for some relief in pain.As per your symptoms what you have given nothing looks major so I have given you simple guideline and I am sure you will feel better once you start taking steps on it. Take care."
},
{
"id": 221767,
"tgt": "What causes red spotting during 8th week of pregnancy?",
"src": "Patient: hello i am about 8 weeks pregnant and for the last 2 days i been seeing brown stuff when i wipe after the bathroom and today i have two period like cramps i dont have tender breast or really any symptoms at that i call the doctor but they seemed like they didnt care i am getting worried it may be a miscarriage Doctor: Hi,In the current scenario, I would ask you to choose a specialist who is trained in high risk pregnancies or who can manage such cases. If your attending doctor shows no concern toward your problem, please take a second opinion. The viability of the pregnancy should be established through a trans-vaginal sonogram. If there is any hemorrhage seen, the need to initiate hormone supplements must be thought of. Absolute bed rest, avoidance of all strenuous activity including sexual intercourse are mandatory. If needed, serial titres of serum beta-hCG may be taken since good progress of the pregnancy is seen when there are doubling titres every 2-3 days. Hope this helps."
},
{
"id": 202023,
"tgt": "Suggest treatment for phimosis",
"src": "Patient: My boyfriend is uncircumcised and when we have sexual intercourse his skin is pulled back and he says that it is very painful... Is there some type of cream that he could use to help? We have tried A&D and Vaseline but it is still now working. Sometimes his skin is inflamed and he has stretch marks on his penis.. What can we do?? Please help... Doctor: HelloThanks for your query,based on the facts that you have posted it appears that your boyfriend gets severe pain while getting erection and tries to retract his foreskin.This is due to what is called as Phimosis.The only best treatment option is to get circumcised .This is a simple surgery done under local anaesthesia as a out patient procedure and offers permanent cure and helps to have enjoyable sex for rest of the life.Dr.Patil."
},
{
"id": 196007,
"tgt": "What causes a lump in the chest?",
"src": "Patient: I'm a 25 yr old male, 5'5'', 150 lbs and have a small lump above my nipple on my right breast. It is about pea sized. It doesn't hurt at all and there is no rash or irritation around it. I would still like to know what it is though and if it should be be checked out. Should this be a concern? Doctor: Hi,thank you for querry.As you mentioned you said you have a lump in chest which doest hurt you.the most chances its a lipoma,but clinical examination is important to rule out lump becuase lump site,size,shape,consistancy,reducibility,compressibility and fix to underlying structure or not is important.lipoma usually not painfull,rubbery in consistancy freely mobile not connected to undelying tissues.no need to worry about it,if size increasesd pain or it mlbecome tenae or tender than consult your General Surgeon.hope the answer will help you in making decsion."
},
{
"id": 95590,
"tgt": "I have a stomach ache towards the middle",
"src": "Patient: hi. I have like a stomach ache. But its not normal its like an aching/stining in my middle stomach. I have had this before and it is usually when i eat lots of junk foor. I ussually throw up. I was wondering if there is anything i can do for it Doctor: Hello, These all symptoms due to acidity. You should take antacid, & omiprazole tablets on empty stomach. Avoid spicy & oily food. Take dinner up to 8 p.m.&do walking for at least 1 km after dinner. I hope this answer will be helpful to you."
},
{
"id": 16618,
"tgt": "Suggest treatment for rheumatic fever and asthma",
"src": "Patient: Dear Doctor, One friend of mine (aged 25) has a problem with positive Rheumatoid factor and has sometimes faced heart fever in the past. At the moment the fever is in control. But her doctor has cautioned that she might have a possibility of heart attack after the age of 30. She goes on for a regular health and blood check-up every year and is under several restrictions in terms of food habits strictly to keep the fever in control. Also she has a problem of asthma. Is there any permanent cure to her problems please? Thanks. Doctor: Hello, I would like to tell you that ablation in PVC is not generally recommended but you can consult your cardiologist and ask to start some antiarrhythmic drugs. Hope I have answered your query. Let me know if I can assist you further. Regards, Dr. Bhanu Partap, Cardiologist"
},
{
"id": 165789,
"tgt": "Suggest diet to avoid constipation",
"src": "Patient: i hav a 11 yrs old daughter who got disabled at the time of birth due to cord rounded around her neck. her survival was not complete and now she is a special child. she is completely bed ridden and dependent on liquid or mashed diet. she is bed ridden can not hold her neck and also feels fits. she passes stool after 6-7 days. she is on medicines like revotril drops, tab topamax.kindly suggest balanced diet for her particularly to keep her fresh and avoid constipation Doctor: Hello dear!I can understand your concern as i am a mother too. It feels heartbroken to see your child in misery.I would advise you to make a diet chart based on her caloric requirement according to her current weight and considering the ideal weight at this age. Suppose at this age,she should have a weightof atleast 35 kg,which i am sureshe wontbe.so you should start with ATLEAST 60% of the required calories of this weight that is 1800 calories (60% would be around 1000 calories)i am making a sample of diet chart which you can give to her and gradually increasing calories.starting from 8 am in the morning give her whole egg in the form of a puree made with poring egg in about 5 ounces of boiled milk and gradually stirring with a whisk.it will be like a custard.add in 2 teaspoons of sugar and a drop of vanilla essence for flavorit will provide around 220 calories.at 10 am you can give chicken stock made with about a 100 gm of chicken in water with selected spices like black pepper or paprika and salt,whichever she likes.it will provide around 150 caloriessave the meat of chicken and blend it with a cup of wheat cereal with a little stock if remaining)it will provide around 250 calories.At 12 noon you can give about a cup of rice pudding made with 4-5 ounces of boiled rice and 8-10 ounces of milk with added sugar if she likes and a cardamom to enhance the flavor.It will provide around 200-250 calories.at 2 pm you can give her mashed veggies like a mashed potato medium sized..you can also grill with butter and sprinkle some salt and then mesh it with chicken stock or milk or anything you like.It will provide around 150 calories and with added butter and stock around 250-300 calories.At 4 pm repeat the wheat and chicken stock (wheat cereal/porridge is high in fiber to avoid constipation).At 6 pm give her banana shake..2 bananas in 5-6 ounces of milk, it will provide around 200 calories.At 8 pm give her lentil soup (about a 100 grams) with selected spices. it will provide around 250 calories in almost a cup of 5-6 ounces.At 10 pm repeat rice puddingAt 12 midnight give her warm milk about 5-6 ounces with a tablespoon of olive oil. It will enhance the calories and relieve constipation.Hope it will help.You can always make variations in the diet with the choice of ingredients.Wishing your child good health :-)"
},
{
"id": 53806,
"tgt": "What could hepatomegaly with high bilirubin levels indicate?",
"src": "Patient: In my sonography i have been mentioned to have mild hepatomegaly with fatty infiltration of liver and in my liver function tests most of parameters are ok with bilirubin as 0.2....i want to know is is it a serious problem...i have a regular problem of bloated stomach...please help???? Doctor: Hello and thanks you for your query. I am Dr. Rommstein and I will try to help you as much as I can with my answer.This is not problem but you should change your dietary and lifestyle habits for some time since obesity and inappropriate diet are main cause of fatty liver. You should avoid alcohol and carbonated drinks,high fats sugar and liver toxic medications. You should eat more green and boiled food. All so,you can take Sylimarin to protect your liver in future. I hope I have answered you query. If you have any further questions you can contact us.Kindly regards. Wish you a good health."
},
{
"id": 39121,
"tgt": "Is bruising a side effect of ringworm?",
"src": "Patient: My doctor diagnosed me with severe ringworm on Tuesday (I am covered from head to toe, and it happened within 2 days-I noticed a few spots on Sunday and thought it was mosquito bites, then woke up monday with triple the amount, and the spots kept showing up during the day). Anyway, she prescribed me a cream that I used on Tuesday, two times- however it caused swelling and pain in my hands so I discontinued use. The doctor prescribed me something called grisofulvin? but I can't pick it up until today at 3 because the pharmacy didn't have it. When I woke up this morning I noticed that most of the ring worm was gone, but I have a LOT of bruises. Is bruising associated with ringworm? ( I did itch the bumps a lot, but only itched over my clothes). I used cortisone cream, but that's it. Doctor: I am not sure that whether your doctors has diagnosed the lesion properly as ringworm does not have such acute course it takes few days to week to appear the lesions of the ring worm. They itch so badly that you have to take anihisaminic to be relived off itching. even if you apply the local cream it will take few days to weeks to clear the lesions of the same. and most importantly they never bruise & go away wihout leaving any stress. So my frank opinion is that get it diagnose correctly befor having any medications as well as applying steroids ( cortison which may change the morphology of lesion & make it difficult to diagnose ). From the symptoms narreted by you lesions most likly point towards patechial haemorrages but as I can not make visual impression exact diagnosis regarding the same can not be given. So my advice is that you must consult a well experienced dermatologist for the problem."
},
{
"id": 109114,
"tgt": "Suggest treatment for back pain at insertion site",
"src": "Patient: 4yr old had LP yo rule out meningitis...negative LP but having back pain at insertion site next day...does well after some ibuprofen bu alot pain when 6 hrs are up...no signs infection at insertion site..no fever appetite good fluid intake good. YYYY@YYYY Doctor: Hi,It is good for negative LP for meningitis.After this procedure pain at the site for few days is common and nothing to worry.Give ibuprofen as needed for 2-3 days.Ok and take care."
},
{
"id": 81087,
"tgt": "Suggest remedy for spasms in chest followed by warm blood rush",
"src": "Patient: Hi. Since last week I ve been feeling spasms in my chest followed by a warm rush of blood sensation. I actually went to emergency because of this and was admitted to hospital for a few days with tachycardia. They reduced my heart rate with beta blockers but never addressed theses spasm sensations which are more disturbing to me than the fast pulse. Doctor: Thanks for your question on HCM.I can understand your situation and problem.Betablockers are known to cause bronchospasm.Beta receptors in bronchi are needed for bronchodilation, Especially beta 2 receptor.So whenever nonselective beta blockers are given, they will block the action of beta 2 mediated bronchodilation and cause bronchospasm.So better to start cardio selective betablocker (beta 1 selective). It Is having only beta 1 action and no action on beta 2 receptor in bronchi, So no bronchospasm. Beta 1 receptors are present in heart so beta 1 antagonist ( cardio selective beta blocker) will have effect only on heart.So consult your cardiologist and change to cardio selective beta blocker."
},
{
"id": 135904,
"tgt": "Are stints the reason for losing balance and strength in legs?",
"src": "Patient: My 87 year old Mother had stints put in both of her legs a few years ago to improve circulation. Now she s having problems with very little strength in her legs, feeling fatigued, and losing her balance. Her doctor believes Lipitor was the problem and took her off of it. Now he s prescribed steroids and anti-biotics for her to take, but she s getting worse. Is it possible that the stints may be part of the problem? Doctor: She needs assessment of the blood supply of both the legs.stents can get blocked over the years.A doppler exam for the lower leg arteries can give good information. A detailed neurological assessment will also be required."
},
{
"id": 20715,
"tgt": "What causes chest pain and artery blockages?",
"src": "Patient: my niece was having chest pains & had by-pass surgery. The Dr. thought she had 3 cholesterol blocked arteries, with surgery-the Dr. said that she had 3 split arteries & blood had dried and formed blockages. What could have caused this problem? She hasn't had any previoous problems. Doctor: Hello welcome to HCM. Atherosclerotic heart disease is a multifactorial disease affected by genetic and environmental factors. The risk factors for atherosclerotic heart disease are strong family history, obesity, smoking, male sex, blood ptessure, diabetes, age, improper diet and many others. A combination of these factors lead to deposition of cholesterol in the walls of the heart vessels causing them to narrow. Narrowing of the artery decreases blood supply to the heart and hence the patient gets chest pain. Narrowing of the arteries critically also leads to stagnation of blood cells called platelets causing clot formation which you are referring here to dried up blood. This can lead to a heart attack. A bypass surgery uses alternative grafts to bypass the blocks so that the blood of supply to the heart is restored. I hope I have answered your query satisfactorily.Wishing you good health regards"
},
{
"id": 87356,
"tgt": "What causes persistent abdominal pain after unprotected intercourse?",
"src": "Patient: hello,just had unprotective sex on saturday and my abdominal hurts..feels like a constant pain, the pain will go away for a period after taking ibprofien but comes back shortly... i dont ever have unprotective sex so i'm really worried tht i may have something now...i just got all my cultures back from doc last week and blood work ,everything fine so i hope i'm alright now... i do want to stress the fact tht i havent had unprotective sex in years and tht i always use condoms with my partner of 13 yrs Doctor: Hi.Thanks for your query. It is wonderful that you never had unprotected sex for 13 years. The possible causes of pain after unprotected sex can be ... PID- Pelvic Inflammatory Disease.Salpingitis.Ovarian problems. I would advise you the following :Ultrasonography and per-vaginal examination by a Gynecologist will help in the diagnosis and treatment .It is possibly best that both you and your partner should get treated."
},
{
"id": 191646,
"tgt": "Is Gabapentin advisable for pain related to peripheral neuropathy?",
"src": "Patient: I have some pain associated with peripheral neuropathy related to diabetes myelitis. The doctor has prescribed gabapentin. The paperwork that came with it causes me some concern as to whether or not this is the correct medicine for me. WHat do you think Doctor: HelloI have gone through your question and understoond your concern.You should know that Diabetic neuropathy has no known cure like all the other complications of Diabetes mellitus. Treatment for diabetic neuropathy focuses on relieving pain and slowing the progression of the disease.It is very important to keep blood sugar within a target range.This can help delay the progression of diabetic neuropathy and may even improve some of the symptoms you already have.Gabapenrin is a anti -seisure medication and it is used to treat nerve pain.Based on current practice guidelines,the first-line treatment of diabetic peripheral neuropathic pain includes tricyclic antidepressants (amitriptyline, nortriptyline ). If these agents are contraindicated, newer anticonvulsants ( gabapentin, pregabalin) should be considered.In my opinion,your doctor has carefully considered your functional status and potential adverse effects of gabapentin when he chose the treatment for your painful diabetic peripheral neuropathy.So,My answer is:Yes,Gabapentin is advisable for pain related to peripheral neuropathy.It is a \"good\" treatement and you have no reason to be concerned.Stay in contact with your treating doctor for every problem you may have during the therapy.Hope this is helpful.If you have other question,feel free and ask.Best regards."
},
{
"id": 153097,
"tgt": "What is the treatment for tumor growing between bile ducts?",
"src": "Patient: My father who just turned 90 this month was recently diagnosed with a tumor that is growing between the two bile ducts that lead into the liver. This was detected thru bloodwork and in the past two weeks he has had two seperate scopes, one of which involved the placing of a temporary stint into one of the ducts. The doctor who performed this discovered during the scope that the tumor was actually larger and was indeed blocking the other duct too. He did not have the dual stint that he needed so he just treated both blockages with temporary stints. We returned with my father yesterday to this same doctor who is a general surgeon. He advised to have the permanant dual stint placed as soon as possible but also added that there are risks involved. Namely not being able to place the left side stint due to the difficulty with manuvering to that side. My question in all of this is: should my father have this situation treated by this general surgeon (who is almost 70 years old) or should he be treated by a gastro specialist???? Doctor: Greetings. I appreciate your concern for your father. Your father is suffering from the cancer of the bile ducts which is an aggresive cancer. As he is very old, i think biliary stenting should be done to relieve the jaundice followed by some form of oral chemotherapy if he able to tolerate that.However if he is in a good health and the cancer is localised then upfront surgery is the option. Stenting will be done by gastroenterologist and surgery by a surgeon. Hope you now understand well and can chose depending upon the reports and health status of your father, whether to go for surgery or stenting first."
},
{
"id": 206047,
"tgt": "Suggest treatment for fatigue and depression",
"src": "Patient: I had viral encephalitis when I was age 12. I had multiply seizures and was induced into a coma to stop them. After that I had to relearn how to walk and talk and have little memory of events prior to infection. Some things such as photos, and conversation will bring back flashes of memories but still to this day at age 29 my memory is in and out about events that happen even now. Throughout my life I have had issues with catching everything around me, colds, strep throat, sinus infections, you name it I seem to catch it, although since the encephalitis I have only had 2 seizures. About 8 months ago I contracted a severe case of EBV (Epstein barr virus ) I am still have problems with extreme fatigue and hearing and vision problems. I begin to wonder if it will ever end of if this is now my life? The mood swings, depression, sleeping, spots, being off balance, is this just all becoming a part of my life now? It has come to a point where I do not work anymore and doctors say hmmmmm...and your ok it will pass, and eat better foods, exercise....I am wondering what I should do? If there are many of us experiencing this why is there no research group to help? Are these symptoms from the EBV or from the viral encephalitis? Doctor: Hello,Thanks for choosing health care magic for posting your query.I have gone through your question in detail and I can understand what you are going through.Fatigue could be a symptom of depression.The first line is the use of escitalopram 10 mg per day for depression.Hope I am able to answer your concerns.If you have any further query, I would be glad to help you.In future if you wish to contact me directly, you can use the below mentioned link:bit.ly/dr-srikanth-reddy\u00a0\u00a0\u00a0\u00a0\u00a0\u00a0\u00a0\u00a0\u00a0\u00a0\u00a0\u00a0\u00a0\u00a0\u00a0\u00a0\u00a0\u00a0\u00a0\u00a0\u00a0\u00a0\u00a0\u00a0\u00a0\u00a0\u00a0\u00a0\u00a0\u00a0\u00a0\u00a0\u00a0\u00a0\u00a0\u00a0\u00a0\u00a0\u00a0\u00a0"
},
{
"id": 203500,
"tgt": "How to cure sore penis skin, peels off, itches?",
"src": "Patient: hi iam haveing trouble with the foreskin on my penis with irration and being sore and some of the skin peels off then heals and it itchs some times some times it gets irratated after sex even when lubracated is there some kind of treatment i can use Doctor: Hi,From history it seems that you might be having fungal infection or collection of smegma under foreskin giving rise this problem.Keep local part clean by averting foreskin while taking bath.Keep local hygiene proper.Apply anti-fungal cream locally.Ok and take care."
},
{
"id": 12587,
"tgt": "What is the best way to cure psoriasis ?",
"src": "Patient: what is the best way to cure psoriasis? is mx3 capsule can help? im 5 ( height ), 63 kilos( weight )..thank you doc Doctor: Hi..dear angel.., Thanks for choosing HCM., MX3 CAPSULES.....these are natural pure XANTHONE.., It is powerful ANTIOXIDANT.., No doubt..good antioxidant..., And proved to work in. 1) ENHANCES SEXUAL ENERGY and athletic or sports related activities. 2) LOWER BAD CHOLESTEROL and enhances the production of good cholesterol. 3) PREVENTION OF CONGESTIVE HEART DISEASES cause by dietary deficiencies, genetic disorder, liver or kidney problem. 4) Recommended for patients undergoing CHEMOTHERAPHY (CANCER) because of its ability to detoxify the body of the toxic and ill-effects of the medication. 5) Prevention and TREATMENT OF OVERACTIVE THYROID..., But not proved in Psoriatic treatment, but little role in Psoriasis also.., But use as adjuvant ....,not a main drug...?, There is no cure ......only control the PSORIASIS..., Bcoz it is Autoimmune..and inflammatory disorder with..., genetic predisposition....ok.., Good treatment is there.., 1) Betamethsone and Salicyic acid 6% for application...night times.., 2) Morning TACROLIMUS...gel will give good remission..., 3) Tab..Methotrexate pack...Mext 7.5...Wallace company..., good result..., 4) PUVA...Tar terapy also give good result...ok..., Consult good dermatologis and take opinion...good luck..,"
},
{
"id": 176471,
"tgt": "What causes green coloured stool in a child?",
"src": "Patient: Hi My child since he was 3weeks old as per Dr. Advice I started Farex 1 alongwith breast milk as my nipples were sore and feeding was less. His motion has decreased from three,4 to once a day or alternate days.now he is a mnth old n poops lemon green motion and turns light red while passing motion. What may be the cause? Farex? Or is it normal? Doctor: HI...this is completely normal. This colored stools is normal. Unless the kid's having low urine output or very dull or excessively sleepy or blood in motion or green bilious vomiting...you need not worry. Regards - Dr. Sumanth"
},
{
"id": 56526,
"tgt": "Suggest treatment for bladder infection",
"src": "Patient: hi had a bladder infection went to the er and was having light to heavy bleeding through the vaginal but took a pregnancy test through blood work it was negative , was prescribed cipro but im still bleeding light to moderate and its been 7 days , what can my next step be am I goin through the changes at 37 years old already Doctor: Hi, dearI have gone through your question. I can understand your concern.You may have some gynecological cause like fibroid uterus or dysfunctional uterine bleeding. You can take tranexemic acid to control bleeding. Bladder infection can not causes such heavy bleeding. Consult your gynecologist and take treatment accordingly.Hope I have answered your question, if you have any doubts then contact me at bit.ly/Drsanghvihardik, I will be happy to answer you.Thanks for using health care magic.Wish you a very good health."
},
{
"id": 200097,
"tgt": "How to control masturbating addiction?",
"src": "Patient: Hii..doctor I m 19 years old. I dont able to sleep without masturbating. .I had this habit from 3 yrs early. I enjoy masturbating. ..and feel excited by releasing sperm. .too quickly. .I use to masturbate 3 times a day..not even a single day I live without masturbating. .plz tell me if it will affect my future sexual life... Doctor: Hello,,Based on the facts that you have posted it appears that You have been indulged in masturbation since many years and fear yourself to get into problem of infertility in future.First of all there is myth in a mind of common man that excessive masturbation is the reason of all sexual problems.But I would like to state that it has been discussed in scientific forums all over the world and proved scientifically that masturbation does not have any negative effect on any organ or system in the body. So do not worry at all.However it is advisable to get addicted to it. Thank you"
},
{
"id": 128533,
"tgt": "Is rheumatoid arthritis the right diagnosis for stiffness in the legs?",
"src": "Patient: Hi, I have had stiffness in my legs when I wake up in the morning, when I sit for a period of time. Bending down to tie my shoelaces is becoming difficult They did an examination and some blood test, said I have RA.. But I am not totally convinced, because some of my other blood tests are normal. Any suggestions Doctor: EARLY MORNING STIFFNESS IN JOINTS IS A FEATURE OF RHEUMATOID ARTHRITIS.HOWEVER YOU HAVE PROBLEM IN BENDING FORWARD SUGGESTIVE YOU MIGHT HAVE DISEASE IN LOWER SPINE, WHY NOT CONSIDER ANKYLOSING SPONDYLITIS.IT WOULD HAVE BEEN GREAT HELP IF YOU COULD TELL ABOUT RESULTS OF YOUR TEST, HOW MANY JOINTS AFFECTED AND OTHER SUSTEMIC ILLNESS.IN ABSENCE OF CLINICAL,IMAGING AND BLOOD INVESTIGATIONS PLEASE GO WITH YOUR DOCTOROR VISIT A RHEUMATOLOGIST FOR FURTHER MANAGEMENT AND NEEDFUL."
},
{
"id": 184700,
"tgt": "Suggest remedy for swelling of gums & jaw",
"src": "Patient: I am a 67 years young female. Have hypertension which I am taking meds for but otherwise I am in good health. Just had my remaining bottom teeth (10)pulled July 7th. All went fine but my gums on the right side of my face right about the jawline is sore and seems like its swollen. It's sore to the touch. Will see my dentist again on July 18th but how can I get the swelling down until then. Thanks Doctor: Thanks for your query, I have gone through your query. The swelling over the gum could be because of the infected gums or any root pieces left in that region or osteomyelitis. consult a oral physician and get a radiograph done and rule out any infection arising from any left out root pieces or bone infection. if there are root pieces get it removed, take a course of antibiotics like penicillin and metronidazole combination. do saline gargling. i hope my answer will help you, take care."
},
{
"id": 142300,
"tgt": "What causes brain lesions while experiencing lack of balance?",
"src": "Patient: I have secoundary progressive ns around 10 years or so/ I find lately my balance is getting worse and my memory is possible getting a lot worse.l saw my neuro said it was caused by deep lesion in my brain and normal.i would like to know what causes these lessions Doctor: Hello!Welcome on Healthcaremagic!You should know that your symptoms are quite normal at this stage of the disease. They are caused by chronic progressive damage to the brain. The main cause of the disease is very complex and it is not just one cause. It is related to chronic inflammation and an autoimmune response of your body, triggered by a possible infection, leading to progressive demyelination and cell loss in the brain. In the advanced stages there is more prominent brain cell loss. That is why your situation is worsening in a progressive way. Anyway, the drugs are not very effective at this stage. A lot of physiotherapy and a balanced diet are the main treatment to reduce the progression speed of the disease. Hope you will find this answer helpful!Best wishes, Dr. Aida"
},
{
"id": 93333,
"tgt": "Taking medicines for tachycardia, high blood pressure. Having chronic abdominal pain, back pain. On tylenol, no relief. Advice?",
"src": "Patient: I have been.doctoring and doctoring and cant get any answers. I a am on two different medications for tachycardia and high blood pressure and I have chronic abdominal and left side mid back pain . I have had chronic uti s but my last urine analysis have come nack normal. I have abnormal bruising but my blood work comes back ok...inn paper they say I look ok besides the heart rate and blood pressure and the fact im in constant pain that ia no longer relieved by Tylenol or Motrin. I am only 29 years old and a mother of two and just stumped on what to do anymore. on what to do anymore Doctor: Hello,You're hypertensive and on medication for it and tachycardia.You have chronic abdominal pain associated with backache.This is most likely of Acid peptic disease.Wherein the backache is most probably a referred pain from gastritis.Consult a Gastroenterologist."
},
{
"id": 14846,
"tgt": "What is the treatment for bumps and rash surrounding the mouth?",
"src": "Patient: quit smoking weed and cigarettes 1 week ago. 2-3 days ago a very itchy red small red bumps and rash showed up around the side of my mouth. I shaved one day and it spread to my neck. Was a daily pot smoker and had maybe 5 or so cigarettes per day. Is this due to withdrawal or is this just some kind of allergic reaction? Doctor: Hi,Thanks for posting the query, this is some kind of allergic reaction dont worry, but get a checkup done by a skin specialist.Take care!"
},
{
"id": 164415,
"tgt": "What causes slimy and liquid mucous in nose?",
"src": "Patient: is disudrin still has side effects? i am hesitant to use it though it was prescribed by my baby s pedia. now i only use muconase spray for her clogged nose. but im a bit worried cause her mucus gets slimmy and yellowish after 5 days of using muconase. please help me what s the best medicine to give.thanks! my baby is 1year old now, weighs 8.4kg and diagnosed with pneumonia last march 20,2011 with Gods help, she recuperated after 7 days of taking chloramphenicol Doctor: Hello and welcome to healthcare magicSlimmy and mucoid secretion in nose can be caused by common viral cold.This disease is self limiting and no treatment is required.You can do some help to make your baby comfortable.Keep her nose and air passage clear by using nasal bulb sucker.Application of Saline nasal drops 2 hourly can make the secertion thinner and easier to remove.Moreover give your baby more liquid diet to keep him hydratedThank you for using healthcare magic.Good luck"
},
{
"id": 153795,
"tgt": "Can mild dysplasia increase the chances of anal cancer?",
"src": "Patient: I'm female,41 years old. I have high risk HPV with mild dysplasia & I'm having follow pap smears to avoid cervical cancer. But I know that hpv can aslo cause anal cancer & we don't have anal pap smears in my country. Am I at risk of getting anal cancer? How can I prevent it? (I've been married & faithfull for 17 years & we never have anal sex & I don't smoke) Doctor: Hi,Thanks for writing in.You do not come under high risk group for HPV associated anal cancer. However you are doing the right action by getting regularly checked up for possible early cervical cancer. Pap smear done regularly is beneficial as you are having high risk HPV with mild dysplasia.Regarding the anal cancer, it is only a probability and this pattern is seen in those women who have had cervical cancer. Since cervical cancer features have not yet manifested in you therefore you might relax about anal cancer occurring due to human papilloma virus infection. Please do not worry."
},
{
"id": 65708,
"tgt": "What does a purplish lump in the vulva indicate?",
"src": "Patient: Hi, I have what appears to be a cyst on the top right hand side of my vulva. There had been some slight dark/ purple discolouration present before the lump, which developed over night. When squeezed, the lump produces a orange discharge that does not smell. What is this and how can I treat it? Doctor: Hi, dearI have gone through your question. I can understand your concern.You may have some benign cyst. Many type of benign cysts are common in this area. You should go for examination. Treatment of choice is surgical excision. Consult your doctor and take treatment accordingly.Hope I have answered your question, if you have any doubts then contact me at bit.ly/Drsanghvihardik, I will be happy to answer you.Thanks for using health care magic.Wish you a very good health."
},
{
"id": 190374,
"tgt": "Dark red bumps between molars on gums, bleeding on brush. What should be done?",
"src": "Patient: I have a very dark red bump between my 1st and 2nd molar on my gums. It doesn t go between my teeth , just in front. I had it checked out a month or so ago and my dentist said he can t find anything in the x-ray (taken 3 different times) so it s just something in my gums, nothing to do with a root or anything. Blood often comes out when I brush. Can you help me? Doctor: Hi, dark red bump between 1st & 2nd molar which bleeds after brushing is more likely a Granuloma. As dentist couldn't find any pathology in x-ray which is taken three times then involvement of periodontium is not the issue. Yes, Granuloma has to be treated by excision & that excisional biopsy should be sent for examination. Unless it excised, avoid forceful brushing as granuloma tend to bleed even on slight touch. Severity of bleed depends on the size & condition of granuloma. Also get your teeth cleaned by dentist (scaling) to avoid secondary infections. Take care."
},
{
"id": 153533,
"tgt": "How to treat stage 4 ovarian cancer?",
"src": "Patient: My mother has been recently diagnosed with stage 4 ovarian cancer following the debulking surgery. her CA 125 is 600. the surgeon performed the surgery to take out as much as possible tissue involved including both ovaries with mass,omentum but could not take the uterus out as it was adhered to the bladder. she is 54 years old and all her symptoms started very recently. it's very devastating to me and my family and we are in a big shock as we never expected this to happen to her. we dont know what to do next but we can go to any extent to cure her cancer. please help ! Doctor: Thanks for your question on Health Care Magic. I can understand your mother's situation and problem. Usually after debulking surgery, post operative chemotherapy and radiotherapy are advised. During surgery, possibility of local spillage of tumour cells is high. This is due to handling of tumour mass. So to prevent local site growth radiotherapy is given. And to prevent distal hematogenous spread, chemotherapy is given. Besides these, nutritional support and pain management are also important. By all these, we can reduce her sufferings and improve quality of remaining life. Psychiatrist counselling regarding end of life issues is also important. So these are treatment options in her case. Hope I have solved your query. Wishing good health to your mother. Thanks."
},
{
"id": 141019,
"tgt": "What causes eye floaters while having myopia?",
"src": "Patient: second time this issue has occurred in past few months - sudden light flashes - irregular pattern but constant in both eyes - so i m guessing neruro - just checked bp - too high - took meds just now - 158/93 then a few min late 163/95 - i m normally under lots of stress and normally take my bp right before bedtime but decided to take now due this issue - also am high myop and other issues (eye had blood vessel break) and md/opt did thorough check - pressures were elevated but no glaucoma actually just now noticed the light flashes have disappeared - lasted about 15-20 minutes the light flashes interfere with vision to the point that i have loss of vision in some areas - all ok at this moment. any particular concerns here - how urgent should i consider this episode? Doctor: Hi, If the ophthalmologist (not optometrist) has thoroughly examined you with a slit lamp, perimetry testing with a Humphrey or Goldmann machine, etc. and not found any ocular problem involving the retina, or other ocular structures then, another possible entity to consider in your case with what you present would be an acephalgic migraine headaches. In other words, migraine headaches without the headache part but with the visual aura which is the streaks of light you talk about that only last a brief time and go away. I would recommend seeing a neurologist and even a headache specialist for consideration of this diagnosis. Hope I have answered your query. Let me know if I can assist you further. Regards, Dr. Dariush Saghafi, Neurologist"
},
{
"id": 65526,
"tgt": "Suggest medication for breast lump",
"src": "Patient: hi im 27, and i have 3 big lumps in my left breast and 1 in my right, sometimes they are painful,. its already two weeks now that im feeling pain in my left part of my breast, armpit and back, lastyear i had a chek up and my doc told me that its benign Doctor: HelloBased on your query my opinion is1. Breast lumps need to be evaluated to rule out malignancy.2. There are many modalities for detection mammogram/ FNAC/ biopsy to find the exact nature of lump.3. You can get a FNAC done, if benign it can be excised.4. There are no medicines to reduce the size of the lump.Hope this helps"
},
{
"id": 193704,
"tgt": "What causes a ring of discoloration on the shaft of penis?",
"src": "Patient: Hi,I'm a 15 year old male so lately I'm really worrying about my penis, mostly because there is a ring of discoloring going around the top of the shaft, and also there is numbness on the skin down the front of the shaft why is this????? I exccesivley masturbate, and I also have a very poor hygiene history I am ashamed to say:( Doctor: Hello, No need to worry from now on you can take care of genital hygiene. Your issue - it can be candida infection or infected smegma. Daily washing with saline water can help you. If you can share a picture for better understanding - i can help you better. Hope I have answered your query. Let me know if I can assist you further. Take care Regards, Dr S.R.Raveendran, Sexologist"
},
{
"id": 7833,
"tgt": "Can I use Deriva MS gel for acne and acne scars ?",
"src": "Patient: I ve acne & acne scars about 3 months old. I m 22 , female, and getting married after 2 months & so want to have a clean clear face . Saw a dermatologist , he recommended Differin & Suprim & Reoccutane (Antibiotic + Tabs), but not much useful. My friend recommended me Deriva ms gel. What should I do? Doctor: Hi!, Welcome to HealthcareMagic forum, Treatment of acne depends upon the severity of acne.What you have received is good.In fact roaccutane which you have mentioned as an antibiotic is actually not an antibiotic but a retinoid.I hope your doctor has told you all the side effects of this medicine since you are getting married in 2 months time.Roaccutane is a wonderful medicine for acne ,it would give good result in one months time,but it has certain side effects like ,patient should not get pregnant during the course of medicine and at-least 3 months after stopping this medicine.It has teratogenic effect on the baby.Other precautions are that one should not donate blood during the course ,should not wear contact lenses,should have a normal liver function test & blood counts before start of the medicine.And if you get headache after this medicine then you should stop the medicine.I hope this was told to you before start of the medicine.In your case pregnancy is the most important precaution . If you can take all the above precaution then roaccutane would be best to treat your acne ,provided your hormone profile is normal.Once started this medicine needs to be given for at-least 6 months depending upon your body weight. Deriva ms is just a gel and alone it might help you a little bit.Patient with hormone im-balance with associated acne would need hormonal medicine to treat their acne. So,kindly discuss all this with your dermatologist and then proceed for treatment. regards, Dr.chawda"
},
{
"id": 123857,
"tgt": "Is a raised spot with burning sensation on the knee a concern?",
"src": "Patient: I have an issue with my right knee I have a slight raised place on the lower outside part of the knew and have had for about 2 weeks it doesn t hurt down deep like a joint or tendon but rather the skin hurts to the touch or when i have pants on I certainly have never had anything like it before Any Thoughts?? Doctor: Hello, As I understand with the query is all about the burning sensation. Now, this is not related to the knee. As you never mentioned about the injury so I suspect that you haven't had any. This burning sensation is related to the nerve root. due to weakness in the spinal region, the nerve root gets impingement. As the nerve has the function of carrying the sensory and motor, any impingement of the nerve will cause the dysfunction in the normal functions of the nerve. I think if you do core strengthening exercises then your symptoms will subside Hope I have answered your query. Let me know if I can assist you further. Regards, Jay Indravadan Patel, Physical Therapist or Physiotherapist"
},
{
"id": 23194,
"tgt": "What causes severe chest pain and constipation?",
"src": "Patient: my chest hurst and i cant go poop, i had this problem for 2 days now and i never experience anything like this, the pain on my chest feels like if something is swallen inside or like my food doesnt go down at all and it stays in the middle of my stomach Doctor: Hiconstipation can cause all the symptoms that are experienced by you at present , you have to take some of the laxatives ,laxatives will loosen your stools , once the stools are passed the heaviness which your are feeling will go away !The heaviness of the chest which you are feeling is not related to your cardiac system ,rather its because if constioation and production of gases inside.once this is relieved your symptoms will go away!"
},
{
"id": 63760,
"tgt": "What causes lump on forearm?",
"src": "Patient: Hi there,I have a larger lump on my forearm - just noticed it - feels like I hit my arm really hard, which I didnt, and it shows no bruising - but it is an area of what feels like swollen tissue. It is like a golfball is sticking out of my arm, not sore to touch and slightly pink in colour, minor itchiness? It is not a solid mass either. Doctor: Hi, dearI have gone through your question. I can understand your concern. You may have some ganglion cyst or other benign cyst like sebaceous cyst. You should go for examination. If needed go for fine needle aspiration cytology or biopsy of that lump. It will give you exact diagnosis. Then you should take treatment accordingly.surgical excision is the treatment of choice. Hope I have answered your question, if you have doubt then I will be happy to answer. Thanks for using health care magic. Wish you a very good health."
},
{
"id": 220639,
"tgt": "Will excessive alcohol affect the pregnancy and baby?",
"src": "Patient: hi Doc, I took 2 tablets orally and 2 on my vagina.. i did it twice but nothing happens.. I m 11 weeks pregnant and I already drink a lot of beer when I though I am not pregnant.. I m thinking that my baby will no longer be normal at all.. please advise. Doctor: i have read and understood your queryI hope this will be helpfulIst of all stop drinking at once.Drinking alchol in ist trimester does effect baby very negativelythough you tried to abort but still you could not,the best you must do now is to get an anamoly ultrasound scan to confirm the health of baby.if it is okay,continue with your pregnancy and take care of yourself.If it is not,discuss it with your ob/gynae or get back to us with detailsI will be very glad to know if you find it usefulThankyouRegards Dr Faeza Sherdil"
},
{
"id": 153004,
"tgt": "Suggest treatment for pulmonary arterial hypertension and adenocarcinoma",
"src": "Patient: My mother has been diaignosed with pulmonary arterial hypertension, resp. failure and now with adenocarcinoma. She has has accumulation of fluid around the heart and lungs and I believe that is caused by the cancer which was found in tissue samples from the pericardium. We have been given the option of hospice or comfort measures in the hospital. I think there may be something more for someone to do rather than just giving up on her. Is there a way to help her illnesses? Doctor: Hello dear. I appreciate your concern for your mother. From your details it appears as she is suffering from advanced stage of cancer along with other co morbid diseases. So primary aim will be to stabilise her chest and cardiac condition, If she improves then only definite treatment for cancer can be done as it will be extremely difficult for her to tolerate the treatment otherwise. Hope that makes it clear. Thanks"
},
{
"id": 176161,
"tgt": "What symptoms should i need to observe if my child swallows some anti flatulent oil substance?",
"src": "Patient: Hello., my 8 months old baby boy has accidentally swallowed an Aceite de Manzanilla for like 4 hours now., it is an anti-flatulent oil substance like that has mineral and manzanilla oil., its color is green.. my baby coughed immediately after it happened but has stopped eventually. I gave my son lots of liquids like water, juice and my milk..30 minutes after the incident, he vomited and there were small green oil like substance on it., He is not showing any symptoms as of now and he is sleeping peacefully my i am really worried that something might happen to him cause of that and i don t know im really panicung.. please help me and please advise me what possible things should i be cautious with or what symptoms should i need to notice,.. please please. Thanks you. Doctor: HI...usually any oily substance which is swallowed has to be noted whether it is volatile or not. As you say it is anti flatulent so it is definitely not volatile. So do not panic. But as it not a consumable substance beyond a certain dose, I suggest you take him to the physician or nearest emergency room.Regards - Dr. Sumanth"
},
{
"id": 165762,
"tgt": "Suggest treatment for small bumps on the hands of a child",
"src": "Patient: hi my 6 year old has small hard bumps only on his right hand just above is thumb and towards his index finger and on the back of his hand, they don t itch. He has had them for about a week, we were on vacation in Mexico, he said he got stung by a bee while swimming in the pool, they don t appear to be fadng Doctor: Hi...I feel by what you quote he should be having a - Hand Foot Mouth disease. This is one viral illness among all other exanthemas which can cause fever followed by rash over palms and soles. It is a self-limiting disorder and itching can be really worrisome. I suggest you use any over the counter antihistamine if you have one with you now. You can use Hydroxyzine at 1-2mg/kg/dose (Maximum 10mg) every 6th to 8th hourly for 7 days.This can even cause some peeling of skin in the next 4-6 weeks and do not worry about it.Regards - Dr. Sumanth"
},
{
"id": 192647,
"tgt": "Is bent penis and foreskin covering only half of penis serious?",
"src": "Patient: I am 20yrs male. My penis's glans is bent downwards and my glans are covered only half part with the foreskin. I mean half glans cn b seen. I jst wnt 2 ask lL there b any problem with my penis in my latr sexual lif? Howevr tl nw I dnt nt xperienc ney prblm. Pls hlp . Abt my penis's glans is bent slight downwards Doctor: Hello,Not serious but it may hamper your sexual life. For further assessment you may require urosurgeon consultation. You may require surgical intervention after surgeon consultation. Avoid forceful retraction of foreskin. Maintain proper hygiene Hope I have answered your query...Let me know if I can assist you further.Regards,Dr Shyam kaleFamily and general physician"
},
{
"id": 25795,
"tgt": "How to control BP and give suggestions to get proper sleep when alone?",
"src": "Patient: Hai, i am 29 year old from india, having my BP at 140 and my LDL is 120. my height is 182cm and weight is 95kg. how can i control my BP? please advise. i am not getting sleep when i stay alone. once i got sleep ,i have good sleep but only getting sleep is quite difficult for that too while staying alone.please advise for this also Doctor: hello,I have gone through your query.Thanks for using HCM.BP of 140 systolic is marginally high.LDL of 120 too is midly elevated.your prob;rm id related to stress of lonliness and anxiety.I suggest you to perform regularly yoga and meditation.Beside this you must walk daily for 30 minutes.Only as last resort you may take anxiolytic tab.Alprazolam 0.25 mg at bed time.my best wishesDr.Rajesh Teli,MD."
},
{
"id": 204617,
"tgt": "How can anxiety be treated while suffering from Alzheimer s disease?",
"src": "Patient: My mother in law has been diagnosed with Alzheimer. She has always had a fractious personality but lately she has been angrier and more irrational than usual. I she needs an anxiety or depression medicine but she is diabetic, has high blood pressure and is on a lot of medications. Suggestions please? Doctor: Hello,Alzheimer's can cause all the symptoms told by you. She needs to take medication for her Alzheimer's to prevent progression of her illness. Consult your neuro psychiatrist for the same.Hope I have answered your query. Let me know if I can assist you further.Regards,Dr. Rohit Kothari"
},
{
"id": 167285,
"tgt": "What causes stomach pain in child?",
"src": "Patient: My sister has a baby girl who has just turned 1 and a daughter who is almost 10. About 2 1/2 weeks ago, the 10 yr. old has been complaining with a stomach ache almost every night at bedtime. She has been to 3 different doctors and they found nothing wrong with her. She says that nothing is bothering her at school and that she is not jealous of her baby sister. She lashes out at her mother to do something, because she is the grownup and she is supposed to fix it. I asked her last night how her stomach felt and she said it was tingling. She calmed down after my husband and I visited her home, but before we went in I could hear her screaming at her mother. They don t want to take her to a phychologist if they can help it. Do you have any suggestions? Doctor: this seems to b a case of separation anxiety. elder sister is seems to b jeleous to new member in family. following steps may b taken1 reassure her that everythig is fine. she has its own place in family.2 give some resposibilty about younger sister to her.3 medication should be given only if behavioural therapy fails."
},
{
"id": 13908,
"tgt": "What causes itchy rashes on arms and legs with throat itching?",
"src": "Patient: I have an intensely itchy rash on my arms and legs and under my breasts and on my upper to md back-it ha raisd bumps and heat in some areas-there hav been no changes in my diet (no new foods or drinks) and no environmental changes, nor hav I been exposed to common poison ivy, sumac, oaks, etc. what cold this be-its making me craz and I am tearing my skin off scratching-which of course brings no relief-ad now my throat is starting to hurt/itch like its on my insides as well. please advise as to what this could be Doctor: Hi, Your symptoms seem to be related to an allergic reaction. I suggest you to use antihistamines. I also suggest you to use Calmine lotion for local relief at the rash area. Hope I have answered your query. Let me know if I can assist you further."
},
{
"id": 202991,
"tgt": "What causes red/oozing in testicles and sore on legs of handicapped?",
"src": "Patient: Severe handicapped son, 19, still in pull ups and here recently it has been happening a lot and doesn't seem to go away totally...This happened last summer but I thought it had something to do with the pull ups but it went away til about 1 month ago.. His testicles are super red and oozing, hes even sore in the creases of his leg. It gets better and then comes back again, bad.. What is going on? Doctor: Hi,It seems that there might be having some bacterial infection due to constant friction due to pull ups and more perspiration.go for one antibiotic medicine course for 3-5 days.Apply antibiotic cream locally.Keep local part clean,dry and airy.Ok and take care."
},
{
"id": 2104,
"tgt": "What is the treatment for having a baby if the couple have thyroid and sluggish semen problem?",
"src": "Patient: Hello Dr. mai 32 years ka hu, aur meri shadi ko 3 years ho gaye hai, lekin ek bhi bacche nahi hai. maine aur meri wife dono ne test karvaye hai, wife ko thiroid ploblem thi aur mera semen sluggish tha but after treatment now we are fine, lekin meri wife abhi bhi conceive nahi kar paa rahi hai. Can you help me? Doctor: Hi., Thanks for asking.I understand your concern. If both of you have thyroid problem.. the first thing to do is consult Endocrinologist.Get investigated about thuroif hormone &go for specific treatment advised. As you recovet.. you would get your sexual functon normal. St.illget both of you checked by a stetility specialist.& take whatever. Treatment advised. Yoy would be benefited this way. Thanks."
},
{
"id": 194005,
"tgt": "What causes white discharge from penis?",
"src": "Patient: Hello sir, my self Jitu, I am 26 year old and I am male.7 year ago itching occurred near my groin and latter resulted black spot. I have the problem of white discharge from my penis in night. So please tell me what could be problem and how it will cure. Doctor: Hello, As per your history, it may be due to sexually transmitted infection. For further assessment you may require complete hemogram, urine analysis and relevant blood tests after physician consultation. You may require broad spectrum antibiotics after consultation as per report. Along with that avoid unprotected sex. Maintain proper hygiene. Hope I have answered your query. Let me know if I can assist you further. Regards, Dr. Shyam B. Kale, General & Family Physician"
},
{
"id": 106542,
"tgt": "Is it normal to experience severe lower backache after a colonoscopy?",
"src": "Patient: I had a colonoscopy yesterday afternoon and I\u2019m experiencing severe lower back ache (similar to Menstrual period back ache but more severe). This has kept me awake most of the night. I had a polyps removed and I\u2019m female, weigh 11 lbs 9 ounces , 5 feet 7 inches tall and 72 years of age. Is this normal and can I self medicate with pain killers? Please can you help? Mrs Heather M Arrowsmith Doctor: Hello and Welcome to \u2018Ask A Doctor\u2019 service. I have reviewed your query and here is my advice. In my opinion this back pain is unrelated to piles and may be due to inflammation in pelvis or spinal canal stenosis. I would suggest pain killers, muscle relaxants and MRI of lower spine to assess the exact cause. The Final treatment would be depending on exact cause. Hope I have answered your query. Let me know if I can assist you further."
},
{
"id": 59074,
"tgt": "Echographie report shows liver increases echo structure diffuse hyper echoic homogeneous and regular contours. Meaning?",
"src": "Patient: hello.this is my echographie report.kindly suggest whet can i do?liver increases echostructure diffuse hyperechoic homogeneous and regular contours.permeable portal vein of normal caliber and flow hepatopetalgallbladder, spleen, head and body of the pancreas of normal appearancetail of the pancreas obscured by deigestifs gas (flatulence)kidney size, well differentiated and cavities pyelocalicielles fines.ils measure rd 119 * 46mm, rg respectively .............. 118 * 52 mm.absence of ascites.absence of deep adenomegaly.finding ..........echographic appearance can do evoke a hepatic steatosis probable.a compare to biology.flatulence + + (or other functional bowel disorder.)Nouveau !Conne Doctor: Hello.Diagnose is hepatic steatosis.Is a pathology related to overweight/obesity, fat food or frequent alcohol intake.It is not an a important pathology but you must control your weight andfat and alcohol consumption.Your doctor must control your pathology periodically requestingabdomen ultrasound and hepatics enzymes-GOT and GPT- levels in blood.The other abdominal organs are normal.I wish you good health.(In case the answer would have been useful please indicate this)"
},
{
"id": 68471,
"tgt": "Is it ok to exercise with lump in knee?",
"src": "Patient: I have a bump on my knee but it does not hurt at all. It is not hard and seems to be fluid filled. It is the size of a baseball. I am a 16 year old girl and I am very active but this past week I have been taking it easy. Could that have anything to do with it? It doesn't bother me at all, so can I still exercise? Doctor: Hi. I suggest before you start exercising get the lump evaluated by your Doctor. It could be a cyst or Synovial effusion which is a fluid collection inside the knee joint. It is definitely safer to be sure of what it is before proceeding with activity. Do let us know if you need advice even after getting it checked up. Take care. Dr Rishi, New Delhi, India."
},
{
"id": 117971,
"tgt": "What to do if am hypoglycemic and blood is thin and bright red in colour?",
"src": "Patient: i am not sure what to do or if i should be worriedI am hypoglcemic and I am having symptums thay my sugars are droping and I checked them and they were at 125 which is good but I noticed my blood was really thin and bright red which os not normal Doctor: Hi, first of all 125 sugar at any time is not hypoglycemic. Stilll if you have symptoms like faintness, temporary darkness, weaknes etc then check for sugar. In urgent need you can take sugar or some sweetor dextrose oral to releive your symptoms. the consult doctor. And you cant say definitely your blood is thin or not. In case of anemia medical person can say grosssly byseeing that but not definitely. So just go andcheck your hemoglobin level andmake sure.and colour of blood is red only forget about that"
},
{
"id": 12573,
"tgt": "Suffering from Psoriasis, itching. Taking oncotrex, folic acid. Treatment?",
"src": "Patient: Dear Sir, I am XXXXX. I am suffering from Psoriasis in fingers, I dont have any problems other than rarely itching and it looks bad. I am right now treated by Dr. XXXXX. His prescription is oncotrex tablet weekly once on wednesday and daily folic acid and levocitrizine. For topical application blizderm. As i presently work in XXXX, i am not able to visit my doctor regularly, i am visiting once in 2-3 months. last time i have visited on 29/5/2012. Kindly help me to come out of this psoriatic condition. My mail id YYYY@YYYY . Doctor: use white petrolium jelly regularly with urea and salicylic acid creams. avoid excess use of soaps."
},
{
"id": 82260,
"tgt": "What does constant pain in the center of chest mean?",
"src": "Patient: I have Constant pain in center chest, has been for several months. Feels like behind the breastbone. Usually better in the morning, gets worse throughout the day. A strange sensation like rubbing near the top of a full deep breath, and also pain is worse with deep breath. Pressing hard on the area / rib cage can temporarily help to relieve the pain sometimes, or sometimes make it worse. If in the car and drive over bumps, can feel like a snagging feeling. Doctor: Thanks for your question on HCM.In my opinion you are having GERD (Gastro Esophageal Reflux Disease). But better we should rule out cardiac and pulmonary causes. So get done ECG and chest x ray, PFT (Pulmonary Function Test) to rule out these.If all these are negative than it must be GERD.It is due to laxity og gastroesophageal sphincter. Due to this acid of the stomach tend to come up in the esophagus and cause symptoms. So tty to follow these for better control of GERD.1. Avoid hot and spicy food.2. Avoid stress and anxiety. 3. Avoid large meals, instead take frequent small meals.4. Take proton pump inhibitors. 5. Go for walk after meals.6. Keep 2-3 pillows under head in bed.7. Quit any habit if you have like smoking and drinking. 8. Loose weight if you are obese."
},
{
"id": 132221,
"tgt": "What causes tumors all over the body?",
"src": "Patient: My nephew has tumors all over his body. They are raised and is different sizes. My grandmother, sister and another child in the family has it. I am trying to find out the name of it, to help him get treatment. They disfigure his face and body. I found something by the name van Recklenhousers syndrome, (The Elephant Mans disease. Doctor: There are very few conditions that cause tumors all over the body like \"Von Recklinhausen's syndrome\" that is congenital neurofibromatosis. It is a developmental disorder of neurological tissue ( Tissue that forms nerves)that overgrows to cause a tumor . Since the nerves are all over the body , It can occur all over the body, but generally does not grow in the internal organs."
},
{
"id": 142607,
"tgt": "What is the treatment for cerebral cyst?",
"src": "Patient: good morning. my names james i have been diagnosed with a 53mm cerebral cyst wich they think is arachnoid 2 yrs ago. i didnt do any follow up when i found out because i was in perfect health besides the fall that required the ct scan . about 6 months ago i had a massive headache wich i couldnt move talk or do anything becuase the pain would increase. i vomited and passed out for 30 hours. then about 2 months ago i randomly collapsed and again vomited and passed out. so i went to the doctors after that an got a mri. they confirmed my cyst grew to 7cm and they also found a venous angioma and hemosiderin from early cavernoma. i just want to know how worried i should be and if i should tell my family. also a little medical backround about family history- 2 of my sisters had surgery to remove cancer cells in there ovaries, my farther and my brother are epileptic my brother was for 10 yrs and stopped taking meds and has been perfect since(10+yrs). can someone give me and idear of what i could be dealing with or some ideas of things i should do. Doctor: Hi, Welcome to HealthCareMagic.com I am Dr.J.Mariano Anto Bruno Mascarenhas. I have gone through your query with diligence and would like you to know that I am here to help you.You need to consult your neurosurgeon at the earliestPlease do not delay Hope you found the answer helpful.If you need any clarification / have doubts / have additional questions / have follow up questions, then please do not hesitate in asking again. I will be happy to answer your questions. In the future, for continuity of care, I encourage you to contact me directly in HealthCareMagic at http://bit.ly/askdrbruno Best Wishes for Speedy Recovery Let me know if I can assist you further.Take care."
},
{
"id": 207948,
"tgt": "Can i take an extra dose of adderall for a panic attack?",
"src": "Patient: I have suffered from severe agoraphobia since I was 17. I am adhd and take adderall. I have PTSD which caused the agoraphobia to go out of remission and return , now I take 8 mg klonopin, 60 mg adderall, 600 mg seroquel. I am physically extremely healthy, my blood pressure is low, it s now 100/80. I was having severe panic to the point of things looking unreal, I was terrified to get off my bed, my brain was going a mile a minute. Out of desperation I took an extra 20 mg adderall, it worked so well I forgot my 4 mg nightly klonopin, and only needed half my dose of seroquel, I tried it the next day with same results, I m afraid to tell my psychiastrists, what do you think I should do Doctor: Hello and thanks for writing in.Taking an extra dose of Adderall is definitely not advisable when you have panic attacks. Adderall is a stimulant medication and can worsen anxiety symptoms.Moreover, taking a double dose of Adderrall can lead to toxicity and result in harmful or dangerous consequences. So, I would advise you to refrain from taking any extra medication on your own. If your panic attacks are still persisting, then I suggest that you discuss with your psychiatrist about further treatment options.Regards,Dr. Jonas Sundarakumar MRCPsych.(U.K.)"
},
{
"id": 87032,
"tgt": "Can upper abdominal pain, be related to groin strain/pull?",
"src": "Patient: Hi, I have been having uper thigh/lowser abdomen pain for about 24 hours now. It is getting slightly worse over time. I really would like to know what this aching in this area could be. I have read about it and it looks like it could be a possible groin strain. can u please help me? Doctor: HelloLower abdominal pain may be due to many reasons like pelvic pathology, genitourinary pathology,colitis,musculoskeletal reasons like sprain etc.You need clinical correlation and investigations like routine hemogram,random blood sugar,liver function test,renal function test,urine RE/ME and ultrasound of abdomen.Ultrasound is important in your case.Proper treatment depend upon findings.Get well soon.Take CareDr.Indu Bhushan"
},
{
"id": 86522,
"tgt": "Suggest treatment for a large painful lump in the abdomen and dyspepsia",
"src": "Patient: Hi - I have found a pea sized hard lump in my upper left abdomen. I have been to see my doctor who has arranged for an ultra-sound scan next week. When I went to see my Doctor last week, I was also suffering from stomach discomfort, bloating and feeling full very quickly during eating. He has prescribed medication for dyspepsia and the bloatedness has mostly gone away. However, having researched on the internet, I m worried my symptoms might be more sinister as I have had a significant weight loss without dieting. I examined my stomach yesterday and found some more lumps on both the left and right hand side of my abdomen so now i m really worried. Doctor: Hi.The commonest lumps in the abdomen which you, the patient can feel and if multiple are lipoma unless proved otherwise. our stomach discomfort, dyspepsia, bloating and feeling full quickly during eating is getting settled with the medicines which your Doctor has prescribed you. Yes, unexplained weight loss is a sinister thing unless proved otherwise:The causes related to weight loss can be:- Cancer of the abdominal contents or somewhere else- Concealed infection like liver abscess- Chronic infection like Tuberculosis.A detailed clinical history and investigations as per the physical findings alone can give a proper diagnosis ."
},
{
"id": 188598,
"tgt": "Front 2 teeth s protruding, having problem in smiling and talking. Will braces be helpful? Treatment period?",
"src": "Patient: Hi,I'll be 23 in a couple of months. My front two teeth are protruding. Though it is not visible when I close my mouth I am having a problem in smiling or talking freely. Will braces be helpful? What will be duration and cost of the entire treatment? Also I dont want anybody to notice that I am wearing braces. Please help. Doctor: Hello,Welcome to healthcaremagic forum.From the details mentioned,its clear that your upper anteriors are protruded resulting in proclination of teeth as well as incomplete lip seal.Orthodontic appliances may help you out in such situation.Depending on the severity of malalignment,treatment duration can be planned.If you are willing to get invisible braces,which are very costly,you can ask regading this with your orthodontist.Hope this helps.Take care."
},
{
"id": 108515,
"tgt": "What is the treatment for spinal stenosis and cystic armpits?",
"src": "Patient: I have some spinal stenosis and a curvature of the cervical spine This has caused me some pain in my shoulder , so getting physical therapy. About 3 days ago, I have gotten a cyst under my armpit . it almost feels like an ingrown hair. I also noticed before I started therapy , that sometimes my deodorant wasn t working on that side along with some itching. I am 78 and otherwise in good health. Doctor: Cyst and cervical spine stenosis has no connection.for cervical spine spine stenosis take tab ultracet 1/2 tab tds and physical therapy.gor cyst take opinion from a surgeon"
},
{
"id": 103447,
"tgt": "Dried scabs on head, wound, no bleeding, allergy, fever",
"src": "Patient: Discovered two dried scabs on my 4 year old daughters head today. We gave her a bath and removed the scabs which left two small open wounds which are not bleeding. She is always scratching her head but it has never resulted in bleeding. She currently has a lot of allergies . No fever . Should we take her to the doctor or keep an eye on it? How do you recommend we treat this? Thanks Doctor: hi, she has fungal infection of the scalp, with secondary infection. see the dermatalogist for local ointment, shampoo, and antibiotics. all the best."
},
{
"id": 109761,
"tgt": "Suggest medication for severe back pain",
"src": "Patient: I went to er yesterday for ongoing back pain and the doctor would not prescribe any medicine. he just told me to follow up with the workers comp clinic next week. i have no medicine and my back is killing me. what should i do?. should i go back to er because the clinic is closed? Doctor: Hi, thank you for posting.I have gone through your query and I understand your concerns.I think that your symptoms might be caused by spinal arthritis.To relieve pain you should use Acetominophen 500mg.You need a spinal X-ray, a fibrinogen test and urine test to confirm the diagnosis.The results of the above tests will determine the definitive treatment.Ask me for anything unclear.All the best.Dr. Behar."
},
{
"id": 21626,
"tgt": "What causes faster heart beat and blocked throat?",
"src": "Patient: Hi,Nowadays, my heartbeat seems like pounding hard in a sec follow by hardly breath and I feel that my heart skip one beat in that one sec. I also feel like my throat being block. This happened for the last 2 years but now it become for frequent like every hour. For your info, I'm 18 years old. Doctor: Get your ECG,Echo done.And if required your doc may advise for.holter monitoring which will monitor your heart rate for 24 hrs.Based on its result a person xan interpret what is the cause of your situation either cardiac or non cardiac."
},
{
"id": 211409,
"tgt": "What are the benefits and side effects caused by psychotropic medication?",
"src": "Patient: what is a heat related side effect to psychotropic medication? why is it important for a patient to report to their psychiatrist if they have been taking a medication previously. what psychotropic medication drugs cause metabolic syndrome. what are some side effects caused by psychotropic meidcations what are some benefits of psychotropic medication Doctor: Hi what is heat related side effectsAs I have understood the question all psychotropic drugs effect heat desire and performance .There is long list of these drugs some effectsc more some less"
},
{
"id": 106336,
"tgt": "What is the treatment for critical stage in tuberculosis and pneumonia ?",
"src": "Patient: My 67 year old mother is critical due to Tuberclosis and pneumonia .Her right lung is almost destroyed due to TB..This is a 3rd attack of the disease in last 14yrs. She is having difficulty breathing without oxygen and her oxygen levels are pretty low without support..they dropped to 50% at one time when she was off it. pls tell me if she is a candidate of lung transplant and what are her chances. urgently help..i am told lung transplant isnt an option for her . Doctor: Hi. Welcome To Healthcaremagic Lung transplantation is indicated for patients with irreversible, progressively disabling, end-stage pulmonary disease (ESPD). The typical lung transplant candidate has a life expectancy of There are certains indications for lung transplanation. Infection in lung is not an indication. Her age profile >65, chronic TB makes her not suitable candidate for lung transplantation. Regards, Dr. Jagdish"
},
{
"id": 68797,
"tgt": "Suggest treatment for lump on elbow",
"src": "Patient: I have a very small, nonpainful, slightly moveable lump on the bottom of my elbow, on the olecranon, I think. It's just odd, and, as with anything involving a lump of sorts, it makes me slightly worried. When I straighten my arm, you can see it a bit more than when I flex. But you can feel it more when I flew. Doctor: Hi ! Good afternoon. I am Dr Shareef answering your query.From your history, it seems that you have got an olecranon bursitis, a benign condition which occurs due to repeated friction on the olecranon bursa normally placed in the elbow just above the olecranon bone. If I were your doctor, I would advise you to avoid pressure on the elbow repeatedly to avoid accumulation of more fluid in the normally present olecranon bursa. In case of need, some patients might have to change their profession if need be to achieve this. Apart from this, I might advise you for a RA factor test and a serum uric acid test to rule out Rheumatoid Arthritis and high uric acid levels in the blood. On the management part, I would treat any abnormality detected on the blood tests, and would give an anti inflammatory and an antibiotic drug if need be to control the inflammation and infection if present. In case of no relief, I would suggest you to visit a general surgeon/orthopaedic surgeon in your area, who after a physical examination, might go for an FNAC, or an excisional biopsy both for diagnostic and therapeutic purposes.I hope this information would help you in discussing with your family physician/treating doctor in further management of your problem. Please do not hesitate to ask in case of any further doubts.Thanks for choosing health care magic to clear doubts on your health problems. Wishing you an early recovery. Dr Shareef."
},
{
"id": 167073,
"tgt": "What is the remedy for thrush?",
"src": "Patient: My daughter has thrush (age 4) she has been suffering with this on and off for over 3 months and had a water infection about a month ago, she is now showing signs of another water infection (frequent toilet with just a dribble and stinging sensation) the GP says this is just thrush and we should just continue to use the cream prescribed 6 weeks ago that we have been using on and off since prescribed, he did not test her urine so how does he know she does not have another urine infection? Doctor: Hello,As per your complaint thrush is a fungal infection, and it can reoccur if the previous infection has not yet subsided, and also if the child is being given antibiotics for some infection. Also, frequent urination and dribbling urine and stinging can be due to urine infection, and you should get a urine culture done for it.I would suggest you first consult a pediatric dentist, and get evaluated and he can do microscopy of the scraping from the tongue done to confirm that it is thrush. In case if it is thrush, then you have to continue applying antifungal mouth paint like Candid mouth paint and along with it gives plenty of probiotics like Yogurt and garlic. Feed with plenty of fluids.Hope I have answered your query. Let me know if I can assist you further.Regards,Dr. Honey Arora"
},
{
"id": 194560,
"tgt": "What causes urine leakage and pain in anus,testicles and groin?",
"src": "Patient: Hi,I'm 16. For the previous month or so, I've been experiencing mild urine leakage after urination and occasional pains in my anus and scrotum (i.e. once a week). Over the past three days, I've been experiencing a mild ache in my groin/abomen/testicles. The ache is most prominant on the left side, although it is felt on both sides. I've since examined my testicles and nothing seems out of the ordinary. Furthermore, my semen is getting more thick and my orgasams are feeling more intense; this has been a development only over the previous few days, too. Do these symptoms pertain to anything in particular? I'm not experiencing any major irritation, but I would like peace of mind. Thanks. Doctor: Hi, The given symptoms do not significantly particular to any major problem. Still do talk to your doctor in person. Hope I have answered your query. Let me know if I can assist you further. Regards, Dr. K. V. Anand, Psychologist"
},
{
"id": 199702,
"tgt": "What causes painful testicles and semen in urine?",
"src": "Patient: Hey there I am 27 years old Initially I felt pain in my left testicle and afterwards I felt pain in both testes and today I have seen semen in my urine with pain while urinating and pain in testes after urinating and nausea feeling with weakness. Please help me as I am very much worried about this Doctor: HelloLet me explain your first complain i,e painful testis and painful urination.Painful testis and painful urination may be due to many causes like infection,hydrocele,urinary tract infection etc.You need proper clinical evaluation.Ultrasound of scrotum and routine investigations should be done like routine hemogram,urine RE/ME,random blood sugar.You may need to take antibiotics.Semen in urine may be due to reasons like over excited state,straining etc.It is important to evaluate for constipation etc.Mild leakage is normally found during defecation.Get well soon.Take CareDr.Indu Bhushan"
},
{
"id": 165273,
"tgt": "What causes vomiting after ingestion?",
"src": "Patient: hello i have a 2yrs old son and today everytime he ate he would throw it up within 10min of eating and the he goes back to his normal self running laughing and playing now when i put him to bed he is burning up but when i use a rectal temp he is only at 99.5 Doctor: Probably your child is suffering from gastreosophial reflux, gastritis, food allergy etc.Give domperidone, ranitidine and ORS solution and he will improve within 2-3 days.However if your child develops fever, stiff neck, headache, burning micturition, abdominal distension orther warning signs, then you will have to consult doctor.For further queries regarding this disease or its medications you may take my appointment at Healthcare Magic.Dr. Khan Shoeb"
},
{
"id": 60586,
"tgt": "Gamma gt level in liver is 26. What it means ?",
"src": "Patient: Hi, my gamma gt level in liver is 26. what it means? Doctor: that means the level is with in normal limits"
},
{
"id": 76277,
"tgt": "Suggest treatment for persistent cold",
"src": "Patient: Hi, I m reema from Australia (age 23) I have cold 365 days n it becomes worse during winters. in summers it will be normal but i sniff when there is cold breeze. Its not exactly cold but i sniff alot when the weather changes. I m not sure about the reason behind this. i have taken multi vitamin tablets but its of no use. can u please suggest me some solution for this problem? Doctor: Hi I can understand your concern...You are having a seasonal allergy to cold...If needed to search for cause of allergen allergic test done....Allegra like antiinflammatory drug helpful for allergic infection..If having blocked nose fluticasone or azelestine nasal spray can be used for shirt time....In cold season you can use room humidifier .Hot water shower taken.Steam inhalation also useful for relieving congestion....If dyspnea occurs then spirometry done to rule out asthma ....Drink more water....Take care.You can consult pulmonologist for it."
},
{
"id": 98618,
"tgt": "What causes nasal congestion, sinus pain and dizziness?",
"src": "Patient: I had a really bad allergy episode with a lot of congestion and sinus pain. I lost hearing and have felt a little off and had some dizziness. When I am around people talking it sounds really loud and almost hurts. Like I am very swollen. I have been on Prednisone and antibiotic, but neither helped much. It has been 4 weeks. Doctor: Either infection or allergy is responsible for your medical condition. What antibiotic did you take ? It is always better to do a culture and sensitivity before starting antibiotic. Was that done ? I would suggest you find out which microbe is responsible for take the right antibiotic which will deal with your infection."
},
{
"id": 86120,
"tgt": "What causes abdominal pain on the left side?",
"src": "Patient: Hello. I have had terrible stomach pains for months on the left side of my stomach. The only thing that I can eat is an English muffin and eggs. I have lost a lot of weight. When I eat most other foods I get so sick, that I can not really fully function. Oh yes, I can sometimes eat a banana. This may sound like a very minor problem, but let me assure you it is not. I have been to 2 hospitals and my regular doctor and have gotten no response to the problem or medications. Marlo YYYY@YYYY Doctor: Hello and Welcome to \u2018Ask A Doctor\u2019 service. I have reviewed your query and here is my advice. With the history your have provided about left sided abdominal pains with inability to eat anything other than English Muffin, eggs and bananas the probable causes can be one or more of the following:Intolerance to other foods thus causing severe pain and obvious weight loss. IBS that is irritable bowel syndrome. Gastritis. Since you have already visited two hospitals I would advise you the following in such a situation: Consult a Gastroenterologist for clinical evaluation and investigations. Upper GI Endoscopy, Colonoscopy, CT scan of abdomen. Tests of stool for 3 days. Tests of blood and urine. Any other tests as would be advised. All these will lead to a proper diagnosis. If the intolerance for food is present then avoiding these is the best way and try for newer foods so that you can have proper foods and nutrition. Take multivitamins and mineral supplements. Hope I have answered your query. Let me know if I can assist you further."
},
{
"id": 83102,
"tgt": "Sle patient, had done angeoplasty. Prescribed clopivas75mg, cilacar10mg, telma40mg. Trace of protein in urine. Any side effects?",
"src": "Patient: i am an sle patient and my angeoplasty has been done 3 years back now my doctor has prescribed clopivas75 mg,cilacar10mg ,telma40mg azoran 75,atorlip F 10/160mg my urine protine is ++ can i take these medicines does it have any hardcore side effects Doctor: Nw for the time being ,you have to take these medicines still u get stabilised.after some time you can slowly drop one by one considering your body condition or switching yourself into some safe and side effect free system such as Naturopathy and Yoga before the manifestation of side effects of the drugs which u have to consume nw. wishing u a speedy recovery !!"
},
{
"id": 4754,
"tgt": "Unprotected sex, took i-pill, bleeding in intervals, abdomen pain. Pregnant?",
"src": "Patient: Hiii,I made love with my fiance on 6th oct early at around 0100hrs early morning of 7th and as it was the unprotected sex I took I pill at around 1100hrs in d night of 7th.I had my last period on 2nd Oct which got over on 5th oct.now from 13th Oct afternoon iam bleeding in intervals and there is minor pain in abdomen is it a symptom of pregnancy or it is normal???? Please advise... Doctor: HelloThanks for your query.This is not at all a symptom of pregnancy.It is normal side effect of taking the i pill.In response to the hormones contained, most women have some kind of withdrawal bleeding and pain.also, the i pill would delay or prepone your normal periods by a week or more.take care."
},
{
"id": 30425,
"tgt": "What causes nausea,joint pains,hand numbness and low grade fever?",
"src": "Patient: help to see if I have lyme disease I think I have been mis diagnosed with CFS I am a 21 year old female 5-3 140lbs in good health other than for 7 years I have been really tired and have have joint pain hands stay numb almost constant nausa and vomit 3-4 times a week for no reason. Cant get enough sleep stay tired all the time. Run a low grade feaver alot. Head aches , Can not remember a tick bite but about 7 years ago when all this started I moved to a new house and spent alot of time in the woods. I have been to count less docs and have all brushed it off have had liver test galbladder test any sugestions just in my research and the time line that all this started it seems to be more likley than anything Doctor: As per your symptoms, your condition seems to be ranging in between chronic infectious and chronic rehmatological disease, you need proper investigation to be diagnosed specifically. it might be two condition running simultaneously, what you do is to do all basic labs, and disease specific test, which can only be recomended by rehmatologist. thank you"
},
{
"id": 37678,
"tgt": "What causes afternoon fever after appendix removal in aged woman?",
"src": "Patient: 84 year old woman had appendix removed 5 wks. ago. She has low grade fever each afternoon, into evening of 99.4-100.7. Has been to ER and dr. several times and all blood work is normal now. She was on Amox. for 5 days 3 weeks ago and the fever seemed to be more controlled. She takes a Motrin or two each day if temp. goes above 100 and it brings her temp. down to about 98.8. Ruled out absess with CT scans. Any suggestions? Doctor: fever lasting for 5 weeks could be tuberculosis. I'm assuming she initially presented with abdominal pain and appendicitis was diagnosed. I would ask if she has any bowel symptoms or abdominal pain since the fever began. colonoscopy would then be ideal."
},
{
"id": 93000,
"tgt": "How can I get rid of bad breath ?",
"src": "Patient: almost everything i eat causes halitosis,however proteins,fats,heavy starches and sugars,fruit,make it worse. i have also noticed that yeasty foods,probiotics and vegetables like cabbages,spinach and even broccoli make the bad breath worse too.what is my problem Doctor: Hi,Thanks for asking the query,Bad breath usually occurs due to poor oral hygiene, plague and calculus deposition, tongue deposists, GIT disturbances, sinusitis, pharyngitis.I would suggest you to go for complete mouth scaling and polishing in every six months.Take antiseptic mouthwash rinses twice daily.Develop a habit of brushing teeth twice daily.Use interdental cleaners, dental floss.Gargle after every meal.Avoid eating of raw ginger, garglic.Hope this helps out,Regards.."
},
{
"id": 64642,
"tgt": "Suggest remedy for lumps in eye",
"src": "Patient: I'm 26 years old, 5'11\", 200 lbs.I have had no medical history. I recently fell and hit my eyebrow on the wooden floor. I don't believe that I fractured any bones and have made sure that I still have eye sight and the ability to move my eye around. However, I have a huge soft lump about the size of a golfball above my eye where I got hit. There is also some numbness around the area of the bump. My questions is, do I need have the lump drained by a doctor or will the swelling go down on its own.If i don't need to see a doctor,is there something I can do to speed up the healing process? Thanks. Doctor: Hi,From history it seems that there might be having hematoma formation following hit on the part.Nothing to worry.Apply ice pack 2-3 times a day for few days.Gradually blood will be absorbed and it will be alright.Ok and take care."
},
{
"id": 129590,
"tgt": "What could be the reason for having pain below my last rib?",
"src": "Patient: I am having a pain about 3 inches below my last rib. and 2 inches from my navel. The pain is bothersome but I know it is there. I knelt 3 days ago and it was obvious. The pain isn t piercing. It is very suttle almost like a heart beat. This has been so for about 3 months. But I am feeling it more now Doctor: dear patient. Here are some possibilities for your pain:1)Liver disease: An enlarged liver may be pressing on the diaphragm. Get an LFT done along with a CBC and a physical by your GP2)Pleurisy: pleural fluid collection may lead to a a heaviness and it can be diagnosed by a simple xray. The causes of pleural effusion are a plenty and will have to be narrowed down by your GP3)Bowel abnormalities: Gas or fecal impaction may result in this sort of a presentation especially if you feel particularly bloated are are having trouble passing stool.a simple xray should rule it out.Other possible but rare causes can beappendicitispancratitisgallstoneskidney stonesThe pain in all these sharp and stabbing"
},
{
"id": 68552,
"tgt": "What causes a lump on lower back?",
"src": "Patient: Hi, I have a lump on my lower back. I thought it was a pimple so I tried to pop it but it does not pop, just really hurts, especially when my book bag is on it. I believe I have experienced a bump like this before on my back. Would you happen to know or have an idea to what it might be. Doctor: Hi,It would be difficult to make a diagnosis without examination. However, this may be a sebaceous cyst or possibly a lipoma which is a fatty lump. If your book bag has be pressing on it, it may have caused some inflammation. I suggest you see your doctor for examination and further advice. Regards,Dr K A Pottinger"
},
{
"id": 49634,
"tgt": "Cyst in pancreas, calculas in kidney, abdominal pain. Surgery required. Weight to be reduced. Survived cancer. Any connections?",
"src": "Patient: I just found out I have a 5cm cyst on my pancreas, I have an appt at Cl clinic on the 29th for it. I also a 1mm calculas lodged in the meat of my right kidney. One of the 2 is giving me a dull achy ab pain, like a mild hot poker. Surgery would be the only way to remove it with part of my kidney. Did one cause the other or visa versa. I had really high tryglyricerides in the past couple years but got it down. I am 5-3 162lbs, weight in my middle. I know I need to loose it. We are rolling on the pancreas, I had cancer in my past. Cant worry till you know a verdict. Is there a connection between all of this? Doctor: Hi, you have described that you have a stone in the right kidney, and it can be desolved by aurvedic drugs, like neeri, and can have lithotrypsy to crush the stone. the renal caliculus will be in the circulating system in the calisys, or pelvis. If in the meat it may be tuberculous, or some chronic infection. So it may need to be removed and biopsy done. The spleen is on the left side of abdomen, the cyst could be secondry to the malignancy. Other things may not have relation ship with the present things. Thank you."
},
{
"id": 37207,
"tgt": "Is tiny wart on my finger symptoms of HPV?",
"src": "Patient: Hi, I have a really tiny wart on my pointer finger. I m not sure if it s even a wart, it s that small. Smaller than a pencil eraser. I performed oral sex on my boyfriend last night, without thinking about this tiny thing on my finger. Should we be concerned about HPV or anything? I am 26 years old and have had all of the HPV vaccines as a teenager. Doctor: Hello ,I understand your concern. I am Dr. Arun Tank, infectious disease specialist, answering your concern.It is unlikely to have you HPV on your finger because you have taken all the vaccines of HPV.Glad to answer your further concern. We wish you a best health at healthcare magic. Thank you,Dr. Arun TankIf you are satisfied with my answer rate me as five star and close the answer"
},
{
"id": 204482,
"tgt": "What are the signs of schizophrenia in a teenager?",
"src": "Patient: Good evening, perhaps I feel a little lost. Our son is 16, typical teenager but gets so moody. He doesn t talk to us and often acts out, yells, screams go away when we try to help him. We are a normal relaxed fsmily, he is our only child. He has a loving home. He experienced a loss of a very dear friend, they were friends from their baby age, due to cancer, it was hard on him. He doesn t do well at School much, gives up easily. We are thinking about a psychologist or a psychiatrist visit. Why? My husband s brother is a schizophrenic, so we are worried. Is it something hereditary? Am I being totally irrational? What are the signs of schizophrenia youth? Thank you for your time!!! Andrea Doctor: Hello and Welcome to \u2018Ask A Doctor\u2019 service. I have reviewed your query and here is my advice. Well due to the lose of his friend the boy can change his behavioral in different way. But I think that your boy need professional help by a psychiatrist to help pass this period of time. And yes schizophrenia is hereditary illness but I will suggest to ask for help and let them make the diagnosis. No you are not being irrational you are a worried mother and in this case your son needs help and of course your support. Hope the best for you. Hope I have answered your query. Let me know if I can assist you further."
},
{
"id": 13637,
"tgt": "What causes spreading rash on inner elbow, chest and thighs?",
"src": "Patient: I am a 19 year old female. I've had a rash on my inner elbow for about 3 years now and it won't go away. It not raised and it doesn't itch. Its a little red, very noticeable. It almost looks like dry skin. It has recently spread to my inner thighs, all over my stomach and chest. Im starting to get worried Doctor: Hi, The rash could be an erythrasma or a pityriasisis versicolor. The former condition is due to a bacteria and the latter is due to fungus. So, I request you to consult your Dermatologist for confirming the diagnosis and for initiating the proper treatment. Hope I have answered your query. Let me know if I can assist you further."
},
{
"id": 172590,
"tgt": "How to get rid of my daughter's loose motion?",
"src": "Patient: hello sir/mamdam, my one yr. & three months old daughter is suffering from loose motions from almost 4 days , even she is not taking proper milk or solid food .On the begning her stool was very smelly & watery but now its not very smelly ...pls help me out from this. Regrds Doctor: Hi...Thank you for consulting in Health Care magic.It seems your kid is having viral diarrhoea. Once it starts it will take 5-7 days to completely get better. Unless the kid's having low urine output or very dull or excessively sleepy or blood in motion or green bilious vomiting...you need not worry. There is no need to use antibiotics unless there is blood in the motion. Antibiotics might worsen if unnecessarily used causing antibiotic associated diarrhoea.I suggest you use zinc supplements (Z&D drops 1ml once daily for 14 days) & ORS (Each small packet mixed in 200ml of potable water and keep giving sip by sip) as hydration is very important and crucial part of treatment. If there is vomiting you can use Syrup Ondansetron (as prescribed by your paediatrician).Regarding diet - You can use cerelac...any flavour will do. Avoid fruit juices as they might aggravate diarrhea. You can give zinc supplements & ORS apart from normal vegetarian porridges & soups.Hope my answer was helpful for you. I am happy to help any time. Further clarifications and consultations on Health care magic are welcome. If you do not have any clarifications, you can close the discussion and rate the answer. Wish your kid good health.Dr. Sumanth MBBS., DCH., DNB (Paed).,"
},
{
"id": 77267,
"tgt": "What causes tingling waves in chest?",
"src": "Patient: Hello, I am a 44yo male. I am on Indopril 4 for hypertention. Lately when lying down and feeling relaxed, I get the feeling of a tingling heavy wave sensation across my chest area (lasting 2-5 seconds), when I move or take a breath the sensation subsides. I am a non smoker. Any ideas?. Thank you. Doctor: Thanks for your question on Healthcare Magic. I can understand your concern. In my opinion, we should rule out heart related diseases in your case. There are two risk factors in your case that predispose heart diseases. 1. Hypertension 2. Age of 44 years. This age is consider as perimenopausal age. During this age due to hormonal changes (low estrogen), women are vulnerable two heart diseases. So better to first get done ecg and 2d echo. If both of them are normal then no need to worry for heart diseases. Don't worry, you will be alright. Hope I have solved your query. I will be happy to help you further. Wish you good health. Thanks."
},
{
"id": 173268,
"tgt": "Suggest treatment for diarrhea and cough in a child",
"src": "Patient: My 2.5 months child is suffering from diarrea, gastro with little cough. His stool reports shows it contains mucus, blood & bacteria. Doctor prescribed Zifi oral drops, Colimex drops & flatuna oral drops. Could you please guide me are these prescriptions can cure my child.? or any special treatment needed.? Please advise. I am very much upset about my child s health. Doctor: Thank you for contacting me. Infective diarrhoea in 2 month old is in most cases due to faulty feeding especially if on bottle feed . I would stop bottle feed and ask feed only breast milk. Is there fever ? YOu have not told if there is incessant vomiting also.In case there is bacterial dysentery the antibiotic is necessary and as stool culture is not always practical, we an empirical antibiotic. The colimex is for colic and flatulent for gaseous disturbance. This treatment should help and you watch for response. THe little dough you said can be due to a viral infection or allergic cough . YOu have not mentioned if there is fever. If not infective children will just cough if possetting ( regurgitation of milk which is common due to lack of strength in theier Gastro espahgic sphincter ( muscle that at junction of food pipe and stomach ) irritates them. I would try giving anti cold drops and see.YOu can review and let me know.Take care regards."
},
{
"id": 6687,
"tgt": "What are my chances to conceive if I had unprotected sex ?",
"src": "Patient: Hello, Please help me Have been taking Tetralysal 300 for a long time. At the same time i take the pill and one year ago i put the implant also. As my period started not to be regular i started taking the pill again to help it. When i have sexual relationship with my ex-boyfriend i did us condom. but with my boyfriend know we decided we wouldn t us it at i was taking the pill and had the implant. But today i went to my GP because of something else and he told me there was a risk of me getting pregnant . How big is this risk ? What should i do ? Doctor: Hi Welcome to HealthcareMagic If you take a contraceptive pill, it may not work properly while you are taking Tetralysal, especially if you have sickness or diarrhoea for any reason. An additional \u201cbarrier\u201d method of contraception (e.g a condom) should be used to make sure you don\u2019t get pregnant. If there are any doubts get an urine pregnancy test or preferably ultrasound done to rule out possibility of pregnancy. Take care."
},
{
"id": 161201,
"tgt": "Is it safe to keep the new born with pets and high risk of asthma?",
"src": "Patient: hi im due to have my baby in march but my doctor say my baby has a high risk of having cronic asthma just like I. However my baby father has 3 dogs and a cat the pets are not kept very clean neither is their house but he wants my baby to visit and spend nights over there is this idea safe Doctor: Hi, No, it is definitely unwise to keep pets at home when there is a strong family history of asthma, as child is predisposed to it. Keeping pets at home causes hyper reactive airway disease and it can cause skin allergies also. Sorry, you cannot have pets. Hope I have answered your query. Let me know if I can assist you further. Regards, Dr. Prasanna Lakshmi, Pediatrician"
},
{
"id": 138127,
"tgt": "What causes soreness and pain in the lower left rib?",
"src": "Patient: I am a weightlifter and I recently did vertical leg presses. After which, I noticed a little soreness and pain in my lower left rib cage. I went to an urgent care and they diagnosed it as costochondritis. The doctor said it was an inflammed cartilage due to the pressure of the weights (my legs/knees were pressed against my rib cage).. I was wondering if that s true, or if it could be a problem with my spleen or maybe a hernia. Doctor: Hello,Thank you for contacting us at \"Ask a Doctor\". I will try to answer your questions the best of my ability.I have reviewed your questions, and it is certainly possible that you have a so-called costochondritis. The other possibility would be a simple intercostal muscle strain in the same region. If you not have any swelling present then this is very unlikely to be something like a hernia. Also, spleen problems do not present in this way, and so this would also be very unlikely.I think at this time it will be sensible to avoid any lifting. You can certainly take over-the-counter remedies such as ibuprofen or Tylenol if it is particularly painful. A local application of ice in the form of a bag of crushed ice applied for about 10 min. 4 or 5 times a day, would certainly help. I would continue with this approach for the next 5-7 days. If after this time, your symptoms are worse or you have developed any new symptoms I would certainly consult with your primary care physician. At this time I do not see the need for further tests or x-rays, et cetera. I feel confident that this condition will settle in due course.I hope this information is helpful. Please not hesitate to contact me again if you have any further questions.Best wishes,Adrian Rawlinson, M.D."
},
{
"id": 10836,
"tgt": "Suggest remedy for hair loss",
"src": "Patient: Hi I am suffering from hair loss for 2 to 3 years and have frontal hair loss. I have been suggested with VB7 hair tablets, Hair for u 5% lotion for scalp and triclenz shampoo. Please advice me if I can use them for now or is there any better lotion than Hair 4 u,,, I have severe hair loss... Doctor: Hi..Hair 4 u lotion contains minoxidil 5%. Keep applying it 1 ml twice a day. It will really stop your hair loss and help you gain newer and thicker hairs. VB7 hair tablets will act as supplements. You can take them for about 2 to 3 months and then stop. Triclenz shampoo can be continued twice a week. This is a very good treatment if you follow it sincerely. I hope this helps. Take care."
},
{
"id": 151392,
"tgt": "Neurocysticercosis, EPTOIN 25 MG, effect on pregnancy",
"src": "Patient: HI, MY SISTER HAS Neurocysticercosys AND SHE IS TAKING EPTOIN 25 MG FOR THE LAST 3 YEARS.BUT DOCTOR ADVISE HER TO TAKE THE MEDICINE FOR THE NEXT 3 YEARS FROM LAST ATTACK.NOW ITS ALMOST 1 AND HALF YEAR IS COMPLETED FROM LAST ATTACK(TOTAL 3YEARS+2.6 YRS +5.6 YRS).BUT PROBLEM IS SHE GOT MARRIED IN MARCH 2012 AND SHE IS FEARING OF MISCARRIAGE IF SHE WILL GET PREGNANT .WHATS UR SUGGESTION. Doctor: Hello, I suspect you meant to say your sister is on 250mg of Eptoin ? instead of 25mg as mentioned above. (as 25mg is too low a dose and unlikely to provide any meaningful protection against seizures) The risk of seizures is highest in those with multiple lesions in the brain especially those with surrounding inflammation.Even in people with calcified inactive lesions longterm antiepileptic medications is necessary if the patient has recurrent seizures. In some it is possible to taper and stop medications after to 6 to 12 months of radiographic resolution of active infection. There should be no problem in continuing the antiepileptic medication when your sister gets pregnant.If she is apprehensive she can be switched to newer drugs like Levetiracetam which should provide the same quality of protection.She could also discuss with her Neurologist regarding a trial stopping of medication. Hope this helps!"
},
{
"id": 145017,
"tgt": "Does peripheral vertigo get cured over time by itself?",
"src": "Patient: I have had Vertigo for about 3 weeks, it has improved greatly. I went to ER had a kat scan and doctor said it is periferal vertigo. WILL IT JUST GO AWAY? Within 10 I went from normal to very dizzy, headache, neckache. Will wearing my night guard help with symptoms? Doctor: Hello. I have been through your question and understand your concern.Fortunately peripheral vertigo is a benign condition and very common. It is self limiting and tends to resolve in a few weeks time. It is very important that CT scan is normal. Still, I would recommend Betaserc 24 mg twice daily for a couple of weeks.Hope this helps. Please feel free using MHC again"
},
{
"id": 36665,
"tgt": "Suggest treatment for jaundice",
"src": "Patient: I am diagnosed with jaundice by my doctor. It all started with high fever (104\u00b0F) then doctor called for first blood test and urine test which turned out to be normal and after medication the fever was gone , meanwhile I had breathing difficulty which was treated (a chest x-ray was done , all normal) , then after 2 days a blood test was done again and I had billirubin (5.47) . I was told I had jaundice . medications are given and meanwhile an ultrasound was done which says : Mildly diffusely hyperechoic liver echotexture with diffusely thickened , oedematous walled of gall bladder and peri-portal cuffing suggestive of hepatitis . after 2 days i have pain in right part of abdomen . I wanted to know my doctor is giving me 2 liver tonics (Livfit & valueLiv) and 2 capsules (Essentiate-L & Pantocar-D) ,are the medications correct ? Because he is not an gastroentrologist . And how much days would take for my recovery? What food I should avoid or take ? And cautions etc ? And what does pain signify? Please help Ps : i used to have white coloured facies but now it is perfect and naturally coloured and the urine is reddish brown in colour. I have occasionally occasional coughing also. Doctor: HIWelcome to H C M You are suffering from Acute Viral Hepatitis.It is self limiting disease.No treatment is required-Take hygienic dietTake plenty of liquid Eg.fresh fruit juices,vegetable juices,milk,curd,soup.No alcohol & no soda.Get blood test for L F T (Liver Function Test) every 7 to 10 days .It takes 2to 3 weeks to recoverIncreased hunger indicate improvement.Your doctor would vaccinate against Hepatitis A & Hepatitis B.Take care"
},
{
"id": 102379,
"tgt": "Suggest treatment for breathing difficulty",
"src": "Patient: Hi Sir, Good day :)! I am a 23yr female with 5.2 height & 59 kgs weight. I have brething problem for pst 3 months whichis new to me. i had been sufferin from sinusitis more than 5yrs. And so am in homeopathic treatment coz am having little stomach ulcer too. my doctor after checking with ecg,hb and some tests said that am physically fit & my breathin prob s due to Mental stress & depression. Thats true coz i myself know am very depressed. Anyway am not able to recover now so fastly Dono what to do instantly to reduce my breathin problem. Yesterday it was severe & i had to take a vein injection too. Am practising Yoga to overcome. Can you please suggest me somethin on this & s t good taking a vein injctn. It aches like something. Kindly help me. Doctor: Hello, Thanks for your query.If you have asthma or allergies, eliminate household allergy triggers like dust mites and mold.Don't smoke, and keep away from secondhand smoke. Don't allow smoking in your home.If you have asthma, learn ways to manage it.When traveling by airplane, get up and walk around once in every few hours to avoid forming blood clots in your legs. Clots can break off and lodge in your lungs. If traveling by car, stop and walk around regularly.Lose weight. You are more likely to feel winded if you are overweight. You are also at greater risk for heart disease and heart attack.I do hope that you have found something helpful and I will be glad to answer any further query.Take care"
},
{
"id": 65009,
"tgt": "What treatment should I do for varied sized lumps under skin?",
"src": "Patient: Question is re: 5+ varied size lumps under the skin on the left calf muscle. Most on inner calf. Some low level periodic pain and some shooting pain. New within the last month or 6 weeks. Found by massage therapist today. She recommended MRI and physician contact asap. What to look for re cause, diagnosis and treatment. Background: Left ankle weakness from many youthful sports injuries. I am on my feet for many hours of the day. 60 year old female, 5 2 , 138 lbs. so about 20 pounds overweight. Generally good health. History of auto accident 2.5 years ago with shirred 5-6-7 and neck fracture. Massage is what alleviates any pain. Doing well in that regard. History of eye, colon and skin cancer in family. Only basal cell for me, many years ago. D Doctor: Hi,Dear,Thanks for the query to the HCM forum.I reviewed your query in depth.I can appreciate your deep concern and worry due to the family History you have.Though I dont have our gestational history-and no of children you have...Your atheletic history and strong physic at 60 yrs of age- with the standing history for many hours Strogly leads to the -non-cancerous-origin of these subcutaneous multiple lumps-as they are of painful nature -and the pain releives with the massage.In my opinion- you should not worry but it definitely needs care -by a surgeon near by.Though your leg photograph-are not there with me, In my opinion -these lumps in left calf-are the varicositie-developed over last 6 wks during which the pain has attracted your attention to these lumps-though they might be there for quiet a long time-as they were painless.Plz upload the photograph urgtly to my mail inbox.I await your photograph.I hope I clarified your doubts and hope you are satisfied with the answer.Thnks .Wellcome again if any query is there."
},
{
"id": 58649,
"tgt": "Increase in weight, flabby stomach. Used duromine. Is it liver disorder?",
"src": "Patient: Hello doctor how are you I have a question after coming back from a holiday I weighted myself and saw an increase in 5 kg in one week I managed to lose 1 kg using duromine I used to take duromine last year and I had one week supply left over so I used that I have also noticed my waist increase I never had an increase waist even when I was fat I had a flat stomach now it s all flabby. Also in past month my salt intake was high as well as I really like salty things is it just water retention or could there be something els I have stopped eating nearly nd still waist has not dropped do u think I would have some other sort of disease such as liver Doctor: Welcome to HealthCare Magic. Hope you are doing good. We often gain weight rapidly with out knowing it the cause could be eating habbits , limited physical activity and much more. Duromine tablets are not effective if you dont have control over your diet and do not physically active ( I mean enough exercise to burn fats). Advance re-search also shows that one of the cause of weight gain is not proper functioning of liver. But in your case i think don't blame the liver. Let Me give you simple plan: If you burn 500 more calories than you eat every day for a week, you should lose about 1-2 pounds.If you want to lose weight faster, you'll need to eat less and exercise more.For instance, if you take in 1,050 to 1,200 calories a day, and exercise for one hour per day, you could lose 3-5 pounds in the first week, or more. Limiting salt and starches may also mean losing more weight at first -- but that's mostly fluids, not fat.When you reduce sodium and cut starches, you reduce fluids and fluid retention, which can result in up to 5 pounds of fluid loss when you get started. you did not mentioned your body weight and height with that could give you better idea that how much should you loose. I hope have answered your query. feel free to ask if you have any query. wish you good health. Regards"
},
{
"id": 179487,
"tgt": "Suggest treatment for abdominal distension after having food",
"src": "Patient: my child 4 3/4 years old, child having abdominal distension after eating meals, HB is 10.3, Ferritin clia - 21.2, USG done mild hepatosplenomegaly with few enlarged mesentric nodes, dr s suggest C pink syp, Acilac 1/2 bd, review after 15 days for repeat HB WHAT I DO Doctor: Hi dear welcome to the HCM,Investigate him for Vitamin D deficiency also.Milder hepatospeenomegally with enlarged lymphnodes suggest some milder chronic infection in the body. kochs infection should also be ruled out.Hope the query is answered.thanks`."
},
{
"id": 60899,
"tgt": "What do painful lumps on the fingers while on Plaquenil indicate?",
"src": "Patient: I have been on Plaquenil for several years and I am getting many nodules on my fingers that are increasingly growing in size and causing pain and discomfort. My Dr. says it's unusual since I'm not on Methotrexate. When I was on Remicade they went away but I got a serious infection on Remicade and ended up with sepsis.Do you have any suggestions? Doctor: Hello,Painful lumps on the fingers indicate possible alterations at microcellular level with fibrous tissue changes to certain areas of the body. A better insight can be possible with a detailed history, physical examination and certain basic lab tests.Hope I have answered your query. Let me know if I can assist you further.Regards,Dr. Bhagyesh V. Patel"
},
{
"id": 141261,
"tgt": "What causes involuntary body movements while on Zyprexa?",
"src": "Patient: person who dx as schizophrenia psychologist give him/her with Po Zyprexa 5mg daily and IVega I m once per month the A/R is moving lips like shivering also involuntary shaking hand like Parkinson decease how to reduce those symptoms? please be advice thank you Doctor: Hello and Welcome to \u2018Ask A Doctor\u2019 service. I have reviewed your query and here is my advice. Zyprexa use may cause extrapyramidal adverse effects such involuntary movements. These adverse effects are dose dependent, so, if it possible to lower the dose, these adverse effects may settle down. Discuss with the Psychiatrist about these issues. Hope I have answered your query. Let me know if I can assist you further."
},
{
"id": 182510,
"tgt": "Is gum swelling, infection, redness and spots due to the temporary filling?",
"src": "Patient: My son, received a temporary tooth filling several months ago and now the gum around the said tooth is very swollen and has doubled in size within 24 hrs. It looks infected, with redness and white spots. Is the swollen related to the filling? He has doctors appointment and dentist appointment booked. Doctor: Thank you for chossing Healthcare Magic . I think your son has infection and is very good that you have an appointment with the dentist . this is because the tooth is not treated at the right moment . 1 endodontic treatment ,2 a treatment of infection ,3 and receipt of an antibiotic dose are the procedures that will be in your son's tooth . Thanks !"
},
{
"id": 104987,
"tgt": "Prescribed Seroflo and Levosiz for cough, cold and breathing problems. Do I have asthma or allergy?",
"src": "Patient: hi i am 25 years old.i had cough and cold last week and some breathing problem after 2 days. in the past i had sneezing problems when exposed to dust.doctor said i had asthma .she has prescribed seroflo 250 2 puffs 2 times a day and levosiz m tablet once a day.my absolute eosinophil count was 1.27*10^9/lt. She said i am suffering from severe allergy.i am confused it is asthma or just allergy? i have been feeling better after treatment for 5 days. Doctor: Hello, Greetings from healthcaremagic.com I am Dr Sourav Chakraborty and i will be helping you with your query Asthma is a disease where there is hyper reactivity of your respiratory tract to particular pathogen which could be an allergen , foreign body and infection. It may trigger asthmatic attack by narrowing your airways and you may feel breathless/ breathing problem The medications your doctor has provided for you , one of them is primarily is to relieve you from recurrent allergic attacks and the inhaler is to improve air entry through your hyper responsive air way so that you don't get any more breathing trouble. Follow your doctor's advice meticulously. Hope i have been able to help you. Take Care."
},
{
"id": 119573,
"tgt": "What is the treatment for swollen and painful knee?",
"src": "Patient: I am 46 years of age, 5 ft 3 in tall and am in good overall health. Approximately two years ago, I was at the beach coming out of my room, located on the 3rd or 4th floor, I was stung by something and immediately turned around and went back to the room. The sting or bite was painful, swelling up to my knee. I was bit or stung on my left foot leaving a three-prong bite or sting mark on my foot. I immediately put benedryl creme on my foot up to my knee. Some of the swelling went down after an hour or so. But it was still painful and swelled. I was ok for the rest of the trip Doctor: Hi, The effect of sting or bite remaining for 2-3 years is not possible, these are usually antecedating event which leads to unmasking of underlying pathology cause for knee swelling and pain may be infective or inflammatory in origin. You need to get a complete blood count along with arthritic profile, and if required after the results of blood tests knee aspiration and testing to be done furthermore to make a diagnosis. Take care. Hope I have answered your question. Let me know if I can assist you further. Regards, Dr. Rohan Shanker Tiwari, Orthopedic Surgeon"
},
{
"id": 52355,
"tgt": "What causes an elevation in the ammonia level while having acute pancreatitis?",
"src": "Patient: My son is currently in the hospital with acute pancreatitis - very sick but being treated appriopiately. I am concerned because Ammonia level is elevated at 71 - he rec d Lactulose ( 1 - dose so far ) . He is heavily sedated but seems oriented. I am worried that this could get worse in setting of pancreatitis . Please advise. Doctor: Hello, Sorry to hear about the condition of your son, increase in the level of ammonia could be suggestive of patient landing into mods multiple organ failure as a result of pancreatitis. Treatment is ok. Wait and watch is only we can do. Hope I have answered your query. Let me know if I can assist you further. Regards, Dr. Ramesh Kumar, Gastroenterologist"
},
{
"id": 136889,
"tgt": "Suggest remedy for swelling in knee joint",
"src": "Patient: Hi I am suffering from RA for last seven yrs.I was treated with folitrax inj once a week and medrol tab which controlled my disease to a great extent.My knee joint has again swollen and it has become immobile. folitrax inj. does not suit me now as it causes reststlessness and discomfort.Pls.advise me for my knee joint Doctor: Hello,I have studied your case and I want to tell you that rheumatoid arthritis is a chronic disease and it takes time for complete healing. You need to take DMARD drugs for long time. If it is not responding to medical treatment then it can be treated with surgery.I hope this answer will be useful for you. Let me know if there is any other followup questions.thanks"
},
{
"id": 137886,
"tgt": "Do tender points on thighs which feel like bruises indicate rhabdomyolysis?",
"src": "Patient: Hello. I have been prescribed to 10 mg of adderall twice a day for about a year and a half now, about 6 months after beginning the adderall, I began experiencing muscle tightness all over but mostly in between my shoulder blades. I have tender points in my thighs, and buttock area, and calves. I never thought until now after some research on side effects that my pain could be coming from the adderall. During my research that is when I learned about Rhabdomyolysis.. however I always have take my medication as prescribed. Recently I have felt as if my medication is not working for me anymore. My body feels as if I have the flu but I am not ill. The tender points feel like big bruises but are not visible to the skin. I have been to physical therapy after clear xrays from a local Orthopedic, that suggested poor posture.. this is not the case for me. My pain is to severe. I do no know where else to turn because I do not feel comfortable seeking help from any more doctors after several attempts, only leaving me with the blank stare and being treated as if I m only trying to score narcotics. My question is; 1. Are my symptoms something someone would experience with Rhabdomyolysis? 2. What steps should I take if so? Doctor: Hi i did review your concern. Adderall on long term use can cause rhabdomyolysis. However we need to do some tests that indicates presence of myoglobin in blood and urine. You should consult a physician who can get this test done and also examine your kidneys if they are under any damage. meanwhile i must advice you to take adequate water for proper urination to remove if there is excess of myoglobin body.i hope this helps. good luck."
},
{
"id": 224814,
"tgt": "Is there any chances of pregnant if i miss my periods this month?",
"src": "Patient: I dry humped my bf on 9th and even though nothing happend his pre cum might have come in contact with my underwear but as a precaution I took an i-pill within 24 hrs. my periods are delayed. is there any chance I might be pregnant if i miss my periods this month? Doctor: Hi. you took i- pill well in time and should be fully protected. there is hardly any chance of pregnancy. However, if you have delayed periods then check for pregnancy test once. if negative take medicine for withdrawl. Thanks."
},
{
"id": 222249,
"tgt": "What are the chances of getting pregnant after using ipills?",
"src": "Patient: I missed the 7th (friday mornings) pill in my packet (microlite) and only realised this and took it when i was taking my 8th day pill (Saturday morning). I had sex that saturday morning before i took the two pills. Whats the chances of becomming pregnant? am I still protected?? Doctor: Hello dear,I understand your concern.In my opinion even after missing a single pill the protection from hormonal pills is doubtful for that cycle.When you miss the pill you have to take two pills the very next day or as soon as you remember. So I suggest you to follow back up method with condom for this entire cycle.We cannot predict the chances of pregnancy exactly but the risk cannot be ruled out.So wait and check for the period.If the period is delayed by more than a week then a urine pregnancy test need to be done and contact your doctor.Best regards..."
},
{
"id": 225843,
"tgt": "Planning for taking birth controls for 3 months. Later will this affect pregnancy?",
"src": "Patient: Hi, i m in age 36 and never get pills. Me and my husband having plan to have kid but decided to get only three months pills dian 35 and then stop and go for pregency. My question is, is that affect my pregency? Is that make my pregency by delay or any other problems? Is there any changes in my hormones by getting 3 months pills? What s the fastet way to get pregnant after stop taking pills? Thanks, Sue Doctor: Hi Sue,Thank you for posting your question here I will try to answer it to the best of my abilities.Many women take contraceptive pills for months, even years before they decide they want to get pregnant and from what I have seen in my career they usually always manage to get pregnant unless is their some other problem with them.The best way to get pregnant is by having intercourse on the days when you are ovulating, so when you see your doctor for the prescription for contraceptives, ask him how to calculate the days when you are ovulating. In case he doesnt know how to calculate them, refer to me and I will let you know.I hope this answered your question."
},
{
"id": 178013,
"tgt": "Can Levolin be given for fever to a child?",
"src": "Patient: My daughter is 2 1/2 yrs old. We moved from US to India couple of weeks earlier. She w as 12 kgs when we moved here. She has cold, running nose and stuffy nose during sleep, she also has fever for the past 4 days. The temp goes till 104. Currently taking p 250, cetzin, levolin and an antibiotic. Has fever even now Doctor: Hi....Fever of few days without any localizing signs could as well a viral illness. Usually rather than fever, what is more important is the activity of the child, in between 2 fever episodes on the same day. If the kid is active and playing around when there is no fever, it is probably viral illness and it doesn't require antibiotics at all. Once viral fever comes it will there for 4-7 days. So do not worry about duration if the kid is active.I seriously doubt if antibiotic is really necessary. Without wheezing, no need for Levolin too. You can continue the rest of the medications.Regards - Dr. Sumanth"
},
{
"id": 31124,
"tgt": "How to treat a red, swollen and painful big toe?",
"src": "Patient: I went the the ER w/ redness, swelling, and pain in my big toe. they cut me open, drained some fluid, and gave me an anti-biotic. This was almost a month ago, and fluid is still draning, it's still red and swollen and it hurts. Does it normally take this long for a bacterial infecftion to clear up? Doctor: HI, thanks for using healthcare magicNo, it does not normally take this length of time.It would be best to see your doctor or re visit the ER for assessment.You would likely need another course of antibiotics and it may need repeat incision.An inflammatory pain killers can be used to help deal with any discomfort eg cataflam, naproxen, ibuprofen.I hope this helps"
},
{
"id": 221403,
"tgt": "Suggest ways to avoid pregnancy after intercourse",
"src": "Patient: I am on birth control and i take it everyday and never miss a pill! I always take them around 3 Pm but sometimes i will forget and take them at 4- or something 5 Pm. I had unprotected sex with my boyfriend and some of the sperm got in me and im kinda of freaking out. We always use condoms but this time we didnt. Doctor: HiDr. Purushottam welcomes you to HCM virtual clinic!Thanks for consulting at my virtual clinic. I have carefully gone through your case, and I think I have understood your concern. I will try to address your medical concerns and would suggest you the best of the available treatment options.Please do not worry.As you have mentioned that you take birth control pills very regularly, you will not get pregnant.As such it is ok to have 1 hour time lag in daily timings, but never ever forget to take the pills.Secondly it is a very good thing on your part that you are using condom,use of condom will also help to prevent STDs.I hope my answer helps you.Thanks.Wish you great health."
},
{
"id": 100939,
"tgt": "Experiencing cold ,itchiness of throat & burning of nose",
"src": "Patient: im 15yrs old and i think I'm starting to catch a cold but right now the back of my throat itches and it some what burning and my nose is stuffed and it feels like i have mucus on the back of my throat but nothing is coming up/out and my throat is sore, what can i use to make it go away? would regular tea help? Doctor: HI THANKS FOR POSTING YOUR QUERY ON HEALTHCARE MAGIC.THESE SYMPTOMS OF YOURS POINT TO \"POSTERIOR PHARYNGITIS\",TREATMENT INCLUDES TAB SINAREST WITH SALINE MOUTH GARGLE.ANTIBIOTICS CAN HASTEN RECOVERY.CONSULT YOUR DOCTOR FOR FURTHER QUERIES.THANK YOU,TAKE CARE"
},
{
"id": 191006,
"tgt": "I have a problem of bad breathe",
"src": "Patient: I am aged 24 and l am female. Have a problem of bad breathe. Was diagnised ulcers when l was nine years old. Have been to a throat specialist. Was given a two week course tablets for ulcers to no avail have gone for sinuses x-ray. Have just graduated from university and had a hell of a time in school please help me. Doctor: Hi, First of all it should be understood that malodor is caused by many reasons like decayed teeth, poor oral hygiene, gastric problems, liver disorder, diabetis, smoking, alcohol consumption, tobacco chewing. Gastric ulcer should be treated by a gastro-enterologist. You should consult a dentist to check for any problem in your oral cavity."
},
{
"id": 48427,
"tgt": "Should i take ultrasound to check kidney stones?",
"src": "Patient: I have been diagnosed with kidney stones.when I had pain once in my abdomen . Stones are Less than 5mm in size, 2 in both kidneys. I am having noculi syrup and no pain since past 15 days. Should I get ultra sound done now, to check status of stones ?? Doctor: Hello and welcome to HCM, Ultrasound of the pelvic region helps to determine presence or absence of kidney stones. Only a minority of kidney stones are radio-opaque on X-ray thus ultrasound will pick up all other stones. In case stones are picked up on the ultrasound, appropriate management can be done- conservative or surgical. Thanks and take care Dr Shailja P Wahal"
},
{
"id": 157072,
"tgt": "Is there any treatment to cure disability caused due to radiation?",
"src": "Patient: My grandson who lives with me and has had years of health issues starting with cancer when he was 3 and the radiation has caused many back issues. He has had 43 surgeries and been put on pain medicine many times. He is on disability and has low self worth. His mother isn t part of his life and lives out of town. He has abused the pain medicine which causes lots of problems He has been to concelors but I think he may need other help. He really tells lies to the point we question everything he say. What can I do to help him Doctor: Sorry to hear about your grandson.I think he developed dependency on the pain medication.You may consult psychologist regarding the issue.He/she will guide your grandson.pain medication also have to be changed. Cosultation with palliative and pair care specialist will help you."
},
{
"id": 167003,
"tgt": "What causes fever, gums bleeding, cold with lack of appetite?",
"src": "Patient: Hi, may I answer your health queries right now ? Please type your query here... My 5 year old daughter had crowns put on all of her back molars 6 days ago. She would not eat anything but yogurt or completely soft foods for 3 days. She has finally started eating but limited. She has been moody and seems to have a cold. She has had a mild fever off and on but tonight when we were brushing her top teeth in back started to bleed. Doctor: Hi..Welcome to HEALTHCARE MAGIC..I have gone through your query and can understand your concerns..As per your complain bleeding from the gums in the area where there is placement of crown can be due to infection in the gum in that area that can be due to infection caused due to lack of proper cleaning..Pain can lead to reluctance in eating too..For this consult a Paediatric dentist and get her evaluated and also get an x ray of the tooth done to check for infection..In case if there is gum infection then scaling will help..For fever and cold that can be due to viral infection you can give her Acetaminophen but f the fever continue to recur consult a Paediatrician and get a blood test done..Hope this information helps..Thanks and regards.Dr.Honey Nandwani Arora.."
},
{
"id": 167434,
"tgt": "What could cause passing of green stools ?",
"src": "Patient: Hi my baby is 6 weeks old and bottle fed, he as just started passing dark green stools with a bit of yellow he is crying more then usual and seems to have a bit of a cold sweat I was told that this maybe normal as its a sign of colic but im worried I am going to book him in to the doctors but was just wondering if you could help and advise me. Hes weight is 12lb 13oz Doctor: Hi...Thank you for consulting in Health Care magic.This is a normal finding at this age.Unless the kid's having low urine output or very dull or excessively sleepy or blood in motion or green bilious vomiting...you need not worry. Regards - Dr. Sumanth"
},
{
"id": 1093,
"tgt": "Is there chance of conceiving after opening the blocked tube by surgery?",
"src": "Patient: doctor, my name is RAHEL GIRMA i > am from ETHIOPIA and i am 32 years i have one of my > tube already removed the right tube is there but it is > blocked.A year ago i did surgery to open the blocked tube > but i couldnot get a baby so now i found your e-mail from > the web. and i want you tell me about THE PRICE and > how many percent that i can be sucssesful hope you will give > me all detail and also if you have agent here in ETHIOPIA > just give me your advise thanks. > Doctor: See i have no agents in Ethiopia .But A piece of advice. You can for procedure like ICSI GIFT Or IVF"
},
{
"id": 165108,
"tgt": "Suggest remedies for a concussiona and headache in a child",
"src": "Patient: My son has a concussion- a really bad headache for 5 days.A friend said he could take 2 5mg. oxycontin for the pain. I gave him that and it didn t help.What will and is the oxy bad for him? He doesn t take anything usually. I thought the oxy would take the pain and would sleep well.But that has not helped. Thankyou Doctor: what is the age of the child??? and what are other symptoms associated with headache?? Is it associated with nausea and vomiting?? Is there any family history?? Morning or evening symptoms?"
},
{
"id": 15051,
"tgt": "Recurring rashes on cheeks and neck. Allergic to wheat in skin products. What could be causing this?",
"src": "Patient: I have a recurring rash that I get on my cheeks and neck only, I thought it was due to wheat in skin products but seems to be occurring very regularly and I am very careful not to use any products that contain wheat on my face. It starts out by feeling itchy and sort of prickles and I get really irritated, then I get red blotches then they turn into tiny little pimplesAny ideas????? Doctor: THESE ARE ALLERGIC ALLERGY CAN BE FROM FOODS SOAPS OINTMENTS CREAMS OILS COOKING OILS AND OTHER BODY APPLICATION MATERUIAL IF YOU ARE ALLERGIC TO WHEAT NEED TO STOP COMPLETELY EVEN EATINGI SUGGEST TO GET ALLERGY SPECIALIST DOCTOR TO FIND THE CAUSE BY GOING THROUGH ALLERGY TESTS AND SPECIFICALLY LEAVE THAT FOT EFFECT"
},
{
"id": 211334,
"tgt": "Does having weird type of sensation in head, jaws, teeth, face are signs of sinus or anxiety?",
"src": "Patient: Hi, I am having this off balance feeling or wiered type of sensation in my head and my jaws , face. There is sometimes pressure on the top of the head and its sometimes accompanied by nasuea . i get wiered sesnation in the teeth or mouth which i cannot define but its a pressure sort of thing. my ear also get pressure often and speically in the evening. i also take praxatine hydrocholride due to my anxiety and IBS I had an MRI of brain and a CT of the brain which came clear , i also had a CT of sinus in which maxillary , ethemoid & frontier sinus were inflammed , with mucusol thinkening and deviated sputum on the right. My question is can this off balance thing is becuase of my sinus or its anxiety. Doctor: HIThank for asking to HCMThis of course because of the sinus but anxiety aggravate it , this will be treated easily with the long acting antihistamine and analgesic, along with this try steam inhalation frequently, have nice day."
},
{
"id": 110070,
"tgt": "Is upper back pain and left hand numbness normal?",
"src": "Patient: Hi. My brother has been experiencing pain in his upper back in the neck area and also numbness in his left hand from past few days. His age is 34 and is quite normal in health otherwise. His Blood pressure also seems to be normal. Is this symptom of something or taking pain killer is okay? Doctor: Hi,Welcome to healthcare magic. After going through your query I think your brother is suffering from cervical spine pain with radiation to left upper limbIt may be simple muscular pain.Treatment is Rest in position of comfort, NUROKIND GOLD ONCE DAILY and analgesics (Diclofenac three times a day after meals is effective). Sometimes strong analgesic(such as ultracet three times a day after meals) is required. OMEPRAZOLE 20 MG before meals prevent acidity caused by analgesics .Second possibility may be disc prolapse for which MRI cervical spine is needed to be done.I think your query answered.Welcome to any follow up query."
},
{
"id": 49278,
"tgt": "What could be the reason for small drop of dark urine? Has been diagnosed with slow kidney",
"src": "Patient: My 87 year old mother is seeing a small drop of dark urine when she goes to the bathroom. No pain in kidneys or bladder, no fever and she has been diagnosed with one slow kidney. She otherwise is in fairly good health for her age. Wondering what the small drop of dark urine might be? Doctor: hi pt is 87 yr old,small drop of dark urine, means nothing to worry , firstly do the test like - lft, kft,lipid profile and routine check-up .then consult with best physician."
},
{
"id": 19257,
"tgt": "How is uncoated aspirin better than coated aspirin to a heart patient?",
"src": "Patient: My neurologist told me to take 1/2 of a 325mg UNCOATED aspirin every day since I had a Stroke 12 years ago. the reason for the uncoated vs coated aspirin is that there have been studies that show uncoated aspirin protects against stroke 40% better than coated aspirin. I can t seem to find anyone that still makes the uncoated aspirin. Do you have any ideas where I can still get it? Any suggestions what I should do if it is unavailable? Thank you for your help email: YYYY@YYYY Doctor: Hello, You can find uncoated aspirin in generic formulation, in generic stores. Reason for coating is it prevents gastritis and gastric bleeding. But majority neurologist, do not stress upon the preference regarding this. I don't think there is any significant difference between the two. You should check the authenticity and reliability of those studies. All the best."
},
{
"id": 92363,
"tgt": "What does the lump in abdomen, near the spleen, causing severe pain be significant of?",
"src": "Patient: Hi, I am a student nurse and today during an abdominal assessment we found a squishy lump in the left upper quad. near the spleen. 2 of my instructors felt it and didn't know what it was. It doesn't hurt until you mess with it and then it is slightly achy but then goes away. What do you think it might be? Doctor: Dear Madam ,It can be enlarged spleen or the splenic cyst . USG needs to be done for evaluation of cause .Dr. Shruti"
},
{
"id": 12278,
"tgt": "Is medical marijuana recommended for psoriasis treatment?",
"src": "Patient: Hi, i have Psoriasis and it started small, but is spreading fast. i haven't tried anything for it yet. But it is always irritating my skin, and i think i might be catching arthritis too. Everyone i have talked to who has it said there doctors recommend Medical Marijuana? should i go for that too? Doctor: Hi, welcome to HCM,Psoriasis after long time affects joint and known as psoriatic arthritis.There is nothing like medical marijuana which is prescribed for psoriasis.I usually prescribe Tab methotrexate in weekly dose.super potent steroid mixed in salicylic acid and urea for local application. Emollient for whole body application.All these need medical supervision and titration of dose.Hope this is helpful.Thanks."
},
{
"id": 66854,
"tgt": "What do lumps on hairline indicate?",
"src": "Patient: My daughter is 17 years old and has lumps on the lower part of her head, at hairline, they do not hurt her and they have been there for appx 2 weeks. I took her to a derm. and he said that they are lymph nodes and that if they do not go away in 2-3 weeks see a general surgeon?? I made an appt with a family Dr. for a 2nd opinion, but was curios what kind of info I can find on my own while we wait until next week to see new Dr. Doctor: not to worry much as this is benign condition like sebaceous cyst / dermoid cyst or as you mentioned, lymph nodes!if really worries you can go for an FNAC test for confirmation and detection of the exact pathology there!all the best!"
},
{
"id": 181867,
"tgt": "Suggest treatment for tooth ache",
"src": "Patient: I feel like I have a tooth ache that's been aching moderately for about 3 weeks. It goes from my left top molar up to my eye. I also just noticed that when I was speaking on the phone, the other person's voice hurt my ear. I made him call me on the landline instead of the cell phone thinking it was the phone. Maybe it was voice. But, it still hurts. I'm going to the dentist later to check out the tooth. My question is.....is it an earache (which I haven't had since I was 7) or a toothache, or both? Thanks. Doctor: Hello,Thanks for consulting HCMRead your query as you have pain in ear this can be due to toothace due to wisdom tooth, your tooth might be carious ,or due to periodontal Infection . I will suggest you to consult dentist for oral examination and if needed go for investigation to confirm the diagnosis . Inmeantime do warm saline rinses 2 -3 times a day and you can take medicine like ibugesic or diclomol by consulting with your local doctor .Hope this will help you."
},
{
"id": 106552,
"tgt": "When should the ER be visited for lower backache accompanied by diarrhea?",
"src": "Patient: Hi, I have been having pain in my lower left side that started in my back and kinda expanded over my hip bone towards my groin area. That pain has stayed and I have also started having some pretty severe nausea and diarrhea. I had an ultra sound and x ray done last week but do not have the results yet. The pain, diarrhea and nausea have been getting worse. When should I go to the emergency room. Doctor: Hello and Welcome to 'Ask A Doctor' service. I have reviewed your query and here is my advice. Considering the lower back pain that radiates to the groin area, the fact that you have symptoms of nausea, vomiting and diarrhea, in my opinion this might be a bacterial gastroenteritis. Gastroenteritis is usually accompanied by these symptoms, but if the symptoms continue for more than 4 days and you haven't taken probiotics and oral rehydration solutions you should go to the ER for IV therapy and further treatment. You should also consider starting antibiotic treatment (Ciprofloxacin) for some days. You can use Buscopan for abdominal pain and Ranitidine. Hope I have answered your query. Let me know if I can assist you further."
},
{
"id": 66164,
"tgt": "What is the purple/reddish lump on the mid back of baby?",
"src": "Patient: My 4 month old baby has a purple/reddish lump on her mid back that seems to be getting larger. Her doctor said it could be a platelet issue, I can't remember the term she used, and that we would watch it closely. Do you know what the doctor is talking about and should I get another opinion?? Doctor: Hi, thanks for writing to HCM and sharing your baby's health concerns with us!Well, If I were your treating Doctor for this case of the purple/reddish lump on the mid back of baby, I would think of few possibilities :1. a purpuric spot/hematoma/ecchimosis due to low platelet coulnt or ITP/Thrombocytopenia2. a congenital mole or nevus3. an abscess/boil/infected cyst etcI think your clinician is thinking the same way like me...Hope this answers your question. If you have additional questions or follow up questions then please do not hesitate in writing to us. I will be happy to answer your questions. Wishing your baby good health."
},
{
"id": 201543,
"tgt": "Is bleeding 6 days after frenuloplasty procedure normal?",
"src": "Patient: Hi I had a frenuloplasty 6 days ago and I hadn t noticed any bleeding since the second day. I ve been bathing every pulling the foreskin back to clean the area and today I noticed a lot of blood compared to previously their are also white patches along the stitches line which I m assuming is normal. Was just wondering if I should still be bleeding after 6 days or not?? Doctor: Hi,Frenuloplasty is the release of a fold of skin on the underside of the penis. It is done to decrease pain during sexual intercourse. A dressing is not necessary, but it is normal for the wound to bleed or ooze slightly for a few days. You can have a shower after 24 hours and usually a dressing in not indicated until your inner clothes are getting soiled.Usually it stops bleeding 2 - 3 days after frenuloplasty as mentioned by you. The bleeding on day 6 can be falling away of the scab. This is a single event and one must not worry. If it bleeds continuously then it can become a concern and you must talk to your surgeon."
},
{
"id": 82791,
"tgt": "Can I have a second baby after being diagnosed with Lupus?",
"src": "Patient: I was told I was a carrier of Lups . I have a 2 year old daughter she is completly heathy and had a very normal pregnancy .. Does that mean ive had this in my body since i was born ? only reason i found out is becuase I contracted EBV ........ Wanting a 2nd baby what does this mean for me and getting pregnant again Doctor: HIThank for asking to HCMI really appreciate your concern and let me tell you that as long as the lupus and the conception are concerns these are two different phenomenon and they may not be related with each other, and this is just possible that you might be having effect of lupus at the time of your last conception, hope this information helps you, have a nice day."
},
{
"id": 107513,
"tgt": "Suggest treatment for severe lower back pain and pelvic pain",
"src": "Patient: Sever pain in lower back and plvic area it has me bawled over and comes and goes very sharp and dull feeling and i am not sexuly active at all it hurts to pas gas and bowles and to urinate it rus to my left shoulder as well as tighting cramps on voth sides of my ribs Doctor: u must tell ur age weight proper hisotry . any fever. burning micturition duration of symptoms.....take tab voltral 50mg twice daily after meals"
},
{
"id": 116834,
"tgt": "Why my SGPT level is 117?",
"src": "Patient: I have 5 months old baby and I am breastfeeding her. I went through annual checkup and my SGPT(117) and SGOT(59) levels are high and also my Lipid profile test results are high(Cholesterol\u00a0\u00a0\u00a0\u00a0\u00a0263, Triglyceride\u00a0\u00a0\u00a0\u00a0\u00a0201,HDL\u00a0\u00a0\u00a0\u00a0\u00a059, and LDL 164. Is there any specific reason for my SGPT high levels? Doctor: Hi, dear. I have gone through your question. I can understand your concern. SGPT is a liver enzyme. Its level increase in liver damage. Alcohol or drugs or fiet may caused liver damage in your body. Or you may have some infection in liver. You should go for complete liver function test. Then you should take treatment accordingly. Hope I have answered your question, if you have doubt then I will be happy to answer. Thanks for using health care magic. Wish you a very good health."
},
{
"id": 20580,
"tgt": "What does calcific fibrofatty plaque in artery as shown on tests suggest?",
"src": "Patient: hii, i m jagannath kumbhar, retired teacher, age- 63, i have done full body checkup, CTscan, angiography, cartoid doppler test.. doctor says that there is calcific fibrofatty plaque in artery's in left main coronary artery 50% plaque, in right coronary artery 90% plaque, Dr. suggested angioplasty.. bt i don't want to do that.. is there anyother solution? i m healthy person having weight 69 kg, height 174 cm. always having normal BP. I do farming in my town.i feel little chest pain when i do heavy work, but i never had deep chest pain.. or any major heart related problm. so i m confused . will u plze help me.. Thank you. Doctor: Hi,WelcomeThis is Dr Sameer.CT coronary angio is a pretty good test & has high positive predictive value for underlying blockage in arteries of heart. As your CT Angio is showing significant blockage & that too in Left main artery (which is the main artery to supply blood to heart) you should not take it lightly & should definitely go for a Angiography as soon as possible. And Blockage in left main artery is a very serious thing.Anyone can have blockage in the arteries of heart whether it be a farmer or a sedentary person.Angiography is a simple test to conform the findings of CT angio & their is absolutely nothing to be feared of. It is just a test which can be done through arm also. Further treatment will depend on the findings of Angiography.So don't worry & go for angio. Their is no alternative to this. Its your health & you should not take it lightly.Thanks"
},
{
"id": 36678,
"tgt": "Possibility of having epidermoid cysts",
"src": "Patient: I think I have several epidermoid cysts...quite small on my trapezoids and shoulders. I have had an infected one on my inner thigh....I thought my dermatologist called it a keratin cyst, however. I am 51, female, in terrific physical shape, and am medium complected. In my mid thirties, I have had some in the genital area that finally pop out on their own... Doctor: Hello,I understand your concern.I am Dr. Arun Tank, infectious diseases specialist, answering your query.Epidermoid cyst appears on the germline fusion at the time of birth.As the site you have mentioned is not the germ line fusion.So it is not epidermoid cyst it is likely to be other cyst. As doctor described you as keratin cyst. Treatment of the keratin cyst is the surgical excision.If it is infected, we should first treat it with antibiotics there after treatment should be started.I suggest cefixime tablet three tines a day under your doctors guidance. Local cleanliness and dressing is very important, this will clear the infection early.I will be happy to answer your further concern, you can ask me on bit.ly/DrArun. Thank you.Dr Arun TankInfectious diseases specialist."
},
{
"id": 122407,
"tgt": "What does a knot in shoulder and arm pit indicate?",
"src": "Patient: Hello, Last week I slept on a futon at a friend s house. The next day I discovered a knot in my left shoulder. The knot is about the size of a golf ball and inhibited me from full range motion in my shoulder. I noticed my shoulder seemed pushed further forward than usual. During stretching I noticed the knot was not only below my clavicle as well as I had another knot under my arm pit. Since stretching I m experiencing pulling and spasms. Doctor: Hello, The symptoms seem to be related to a pulled muscle. I suggest using muscle relaxant such as Baclofen. I also suggest using Voltaren gel for local application. I recommend gentle stretching. Hope I have answered your query. Let me know if I can assist you further. Regards, Dr.Dorina Gurabardhi, General &Family Physician"
},
{
"id": 224644,
"tgt": "Lower abdominal pain, headache and vomiting after taking emergency contraceptive pill. Are these side effects? Taking dispirin for headache",
"src": "Patient: Hi, I am 27 years old. I have taken one I Pill , 5 hours after having sex. Now after 2 days, I am feeling headache , lower abdominal pain and vomiting . Headache is increasing continuously. Please suggest if this is normal after taking I Pill or I need to do something. I have taken dispirin tablet for headache (one in night and one in morning), but its not helping. Doctor: Ye these are side effects. do not worry . just wait for a few hours and it will resolve. and try not to use emergency pills frequently."
},
{
"id": 78748,
"tgt": "What causes breathing difficulty with tightening feeling in chest?",
"src": "Patient: I went to the ER today cause I could not breath, 2 weeks ago went to urgent care for the same thing. They said it was acute bronchitis. Went to normal doc a week later, he said it might be asthma. But I am not wheezing or coughing, chest will tighten out of the blue and I won t be able to breath enough to hold a simple conversation or walk very far. Then I start fading out like I m going to faint and start going numb. That could be anxiety though. Prednisone helped. The ER said x rays were fine, O2 levels are fine, cardiogram was fine and put me on an inhaler and steroids, again. Any ideas? Vocal Chord Issues? Could it be occupational, adult onset asthma? I ve smoked for 6 or seven years, I m 23 and male. Doctor: Thanks for your question on Health Care Magic. I can understand your concern. Chest pain with breathlessness are commonly seen in bronchitis and lung infection. Since your chest x ray is normal, no need to worry for lung infection. Possibility of bronchitis is more in your case. So better to consult pulmonologist and get done clinical examination of respiratory system and PFT (Pulmonary Function Test). PFT is needed for the diagnosis of bronchitis. It will also tell you about severity of the disease and treatment of bronchitis is based on severity only. You may need inhaled bronchodilators and inhaled corticosteroid (ICS). Don't worry, you will be alright. Hope I have solved your query. Wish you good health. Thanks."
},
{
"id": 82646,
"tgt": "Is a thick tar like discharge in menstruation normal here?",
"src": "Patient: I have been being treated for possible lupus and have been on Imuran and Mobic for the last three months. I just saw my Rheumatologist last week and the nurse called yesterday to tell me I was anemic and to stop the Imuran for 4 weeks and come back in for labs. Now I have been on my period for 3 days and it is not it s usual color or consistency. It is dark brown (almost black) and pretty thick (almost like tar). Is this something I should be concerned about or is this normal in my situation. I am 32 years old and I have never had a period like this. I weigh 108lbs and I m 5 4 tall. Doctor: It seems to be normal but still if it doesnt revert back to normal that means you will need to seek a gynecologist appointment for the same ."
},
{
"id": 225678,
"tgt": "Spotting after stopping Femiplan. Had unprotected sex. Pregnancy possible?",
"src": "Patient: I have been on femiplan pills for the last two months now. I had a misunderstanding with my husband then i discontinued taking them for four days and then we reconciled. After i stopped, i started spotting bur after we reconciled, i started taking a new pack and used a condom that day. My husband works far from me then he come back after twelve days then we made love without a condom. Can i get pregnant? Doctor: Hi Dear,the spotting which you had after stopping the pill was the withdrawl bleedeing of the pill.Also, as you have used the pills for 2 months, the suppressive effect of pill on ovaries will be there. So, stopping a pill for few days and than starting a new pack with unprotected intercorse on Day 12 ideally should not result in pregnancy.Anyway, if you miss your period, get a pregnancy test done immediately"
},
{
"id": 25328,
"tgt": "Suggest treatment for premature venticular contractions",
"src": "Patient: I have Premature Venticular Contractions (skipped heart beats)? Why is it that on some days I am ok then others, maybe for a week or so they are real bad.I am 63 a type two diabetic. I was diagnosed with PVC's about two years ago. I take Bystolic 5mg for them but it does not seem to be helping very much. They seem to be more at night. Doctor: Hello and thank you for using HCM.I carefully read your question and I understand your concern.You shoud not worry I'll try to explain you something and give you my opinion. Premature heart contaction are a rhythm heart issue. They are electrical impulses that generates in some part of the heart different from normal sinus rhythm.You feel this like skipping beats, palpitations and anxiety. There are different causes of this rhythm issue like simple stress, coffee and alcohol consumption. Pathologys like anamia, hyperthyroidism or ischemic heart disease might be other causes. As you are a diabetic person it is very important to exclude ischemic heart disease. So, if I was your treating doctor I will recommend some examination like an electrocardiogram, a cardiac echo to evaluate heart function and if it is possible a stress test to exclude ischemic heart disease. If it is posible after checking your frequencyes we can rise the dosage of your medicament up to 10 mg a day.This can better control the premature heart beats.So I recomend you to do all above and discuss them with your doctor.Hope I was helpful.Wish you good health. Best regards."
},
{
"id": 182604,
"tgt": "What causes uneven jaw line?",
"src": "Patient: I have an uneven jowl line one side of my face right at my jaw to my chin and underneath a little bit around the neck is bigger(noticebaly) than the other side. People often ask if i have an abssess tooth because it kind of resembles that but i have had this since about the age 13 and it wont go away the more weight i gain the more noticable it gets. When i was about 13/14 yrs old i had lipo on it to try to get it away because its no mass or tumor. Do you have any ideas what could be causing this? it is making me very uncomfortable with my self image:( Doctor: Thanks for your query, I have gone through your query.The uneven jaw line can be because of any defect in the jaw bones like condylar hyperplasia or hypoplasia or hemimandibular hypertrophy or atrophy. So consult a oral physician and get yourself examined to rule out the above said causes. You may have to get some radiographic investigations done like OPG, PA view and TMJ views. Once we diagnose it later we can manage accordingly with surgery.I hope my answer will help you, take care."
},
{
"id": 117016,
"tgt": "What is the treatment for essential thrombocythemia?",
"src": "Patient: I have been diagnosed with Essential Thrombocythemia and am taking Hydra to control the platelets. haven t had a period in nearly 10 years due to the medication. These hot flashes started the week I began the medication and are still going strong. Can I expect this forever? Doctor: Hi, dear. I have gone through your question. I can understand your concern. You have essential thrombocythemia. Treatment is chemotherapy. You can take hydroxyurea like drugs. Permanent treatment is bone marrow transplantation. Your side effects are very common with any chemotherapy. And you have to tolerate the side effects. Hope I have answered your question, if you have doubt then I will be happy to answer. Thanks for using health care magic. Wish you a very good health."
},
{
"id": 78128,
"tgt": "Suggest treatment for painful red lump around solar plexus",
"src": "Patient: Hi, I have a painful red lump that is around my solar plexus, it has been there for 2 days now, it seems to be decreasing in size, however is still quite painful whenever any pressure is put on it, whether it be through touching it or lying on my stomach, Should I be worried? Doctor: Hi. I can understand your concern. If the lump if red and painful it could be abscess or severe infection. You will require a detailed examination for the same. As you may require drainage of the same. Consult a surgeon for the same.Don't worry, you will be alright. Hope I have solved your query. Wish you good health. Thanks."
},
{
"id": 5900,
"tgt": "Had 3 miscarriages. Trying to concieve. Tracking ovulation period, having healthy sex life, but no results. Suggestions?",
"src": "Patient: hello doctor. im 27 yrs old,married for 5 yrs now.i had 3 miscarriges in last 2 yrs.1 1)feb 2011 at 7th week of pregnancy,no hearbeat was detected 2)nov 2011 at 17th week of pregnancy, due to premature rapture of membrane 3)may 2012 at 7th week of pregnancy due to chromosomal anomaly. after 3rd miscarraige i n my husband has done all d tests.all test came out normal including blood karyotype . but my ANA TEST IS POSITIVE, so my doctor suggested heparin shots as soon as i conceive. at present im taking folic acid , baby aspirin and duphaston tab 1nce a day from 20th day of cycle till 28th day. i always had 30 day regular cycle. i am trying to conceive for last 3 mths with no success. with all my last pregnancies i could conceive at first mth of trying. now my cycle has become irregular by 32-34 days.is it becoz of medication im taking? do i need to see fertility specialist? im tracking my ovulation and having healthy sex life..still last 3 mths no success..kindly advice Doctor: Hello. Thanks for writing to us. The medicines you are taking are not likely to cause the irregularity in the cycles that you are having. It is just a chance that you have not conceived in last three months on trying. You need to try again for at least three months more before consulting an infertility specialist. I hope this information has been both informative and helpful for you. Regards, Dr. Rakhi Tayal drrakhitayal@gmail.com"
},
{
"id": 218147,
"tgt": "Is it safe to take Ampiclox and Flagyl during 13 weeks of pregnancy?",
"src": "Patient: Hi Doc, Please, this is my first pregnancy.I am in my 13th weeks and I am having a brownish thick discharge, boil and Itch on my vj area.The boil is on the Vulva area and has ruptured.I went to see the doctor and was given Ampiclox and Flagyl.I hope this is okay for me and the drug interactions too is what I am bothered about. Thank you. Doctor: Hello, Ampiclox (ampicillin) is safe to be taken during pregnancy while the data about metronidazole are a bit contradictory on causing birth defects. You can take ampicillin and possibly metronidazole since you are at the second trimester or can switch to suppositories. Hope I have answered your query. Let me know if I can assist you further. Take care Regards, Dr Olgeta Xhufka, General & Family Physician"
},
{
"id": 123727,
"tgt": "What causes tight feeling in left calf muscle?",
"src": "Patient: my left calf muscle has been very tight for about 4 weeks now, i cant walk properly as it feels like the tendons r being squeezed, two times it has felt like something has exploded and the pain becomes million times worse and then i cant walk properly. it aches constinly and starting to get me down. please help Doctor: Hello, As the history directs towards the nerve entrapment in its course. Do core stability exercise, spinal muscle strengthening exercises, hip and lower limbs strengthening exercises as a whole. This should get some good results. Use hot water fermentation for the calf muscle to reduce the vasoconstriction and keep the leg elevated over the pillow. Hope I have answered your query. Let me know if I can assist you further. Take care Regards, Jay Indravadan Patel, Physical Therapist or Physiotherapist"
},
{
"id": 180194,
"tgt": "What causes spreading rashes?",
"src": "Patient: My daughter is 4 years old she started out with rash on her legs then it went to her arms then to her belly now it s on her groin area and now tonight it s on her privates. It all started out as tiny raised bumps on her her no color to them then she would wake up the next morning with scratch marks all over her cause of the intense itching. I had taken her to the doctor they said it could be poison ivy or something so they gave me benidril and cortisone cream; been using that for about a week and it still the same. She has had the rash since about the 1st of the month. What could it be? Also noticed she is bruising easier than normal. Doctor: Hi...by what you quote it seems to be an allergy of the skin commonly called urticaria. Sometimes it can as long as 14 days to settle and sometimes within this time period it can completely subside and come again as a second crop. Benadryl and cortisone cream are good for this condition. If it is persisting even after 2 week or she is developing blood in urine or motion and intense pain abdomen - seek immediate medical attention. Otherwise you can wait.Hope my answer was helpful for you. I am happy to help any time. Further clarifications and consultations on Health care magic are welcome. If you do not have any clarifications, you can close the discussion and rate the answer. Wish your kid good health.Dr. Sumanth MBBS., DCH., DNB (Paed).,"
},
{
"id": 202725,
"tgt": "Suggest acupressure techniques for hydrocele treatment",
"src": "Patient: hello sir I am 18 yr old male..and I am suffering from mild hydrocele from past a year..it all happened after I got a knock to my testicles during sme wrk..And it was diagonised that my right testicle has mild hydrocele..while my left testicle is normal..plz suggest me some accupressure techniques Doctor: Hydrocele can be repaired surgically or drained. If drained, it is done with an accupuncture needle and a scarring substance can be injected as well to prevent recurrence. Please rate 5 Stars! I strive to provide you the best answer to your questions."
},
{
"id": 165477,
"tgt": "Suggest dosage of Colineaid for flatulence in a child",
"src": "Patient: HI Doctors, My daughter is of age 1 and 25 days, she has a problem of gass, we give her colineaid for instant releaf, my questions are 1) how may time colineaid can be given ? 2) does it have any sideeffects? 3) Is it alright to give colineaid when ever she has gass problems ? Please help me out with my questions. many thanks in advance. Doctor: Hello,Colicaid drops contain digestive enzymes. Digestive enzymes are prescribed for long duration only in case of certainly diseases like pancreatitis.You may give your baby Colicaid drops for few days for symptomatic relief of flatulence. However long- term use of these drops are not advisable. If baby is on formula feed you may try changing to some easily digestible formula feed.Hope I have answered your query. Let me know if I can assist you further.Regards,Dr. Khan Shoeb Mohammad Sher Mohammad"
},
{
"id": 224824,
"tgt": "Mirena coil fitted, low pelvic pain, free fluid in uterus, taking antibiotics, pain while sneezing and popping",
"src": "Patient: i had a mirena coil fitted 2 wks ago was hospitalised with low pelvic pain , scan showed coil was fitted low and had free fluid in uterus , was on iv antibiotics overnight then sent home. ive since been in pain still but have sneezed and had a popping sensation and pain increases, popping has happened twice this week, pain is quite bad now but have no blood loss Doctor: the symptoms that you describe are not usually associated with the mirena IUD. occasionally an infection might occur at the time of insertion, and it sounds like you were treated for that possibility when you had the antibiotics. it is possible that you are still in pain due to infection if the infection didn't respond to the antibiotics. that could've happened if you had an unusual or resistant organism causing the infection. I would imagine that they did cultures, and if this was the case then they should find it in the results and then thy would know which antibiotics to use. we now know that infections can be treated even without needing to remove the IUD. if they can't identify another organism, and the problem still doesn't improve, you will need to discuss with you doctor if the IUD should be removed anyway, or maybe they can identify another cause of the pain that is not associated with the IUD. I hope this helps, please let me know if you have any more questions,"
},
{
"id": 43166,
"tgt": "Have PCOS, irregular period. Trying to conceive. Advised Transvaginal ultrasound. Pregnancy test negative. Possible reason?",
"src": "Patient: I HAVE BEEN TRYING TO GET PREGNANT FOR 5 YEARS TO NO AVAIL SO AS MY GP WANTED TO REFER ME TO FERTILY CLINIC SHE REQUESTED FOR TRANSVAGINAL ULTRASOUND BUT DURING THE SCAN i WAS TOLD BY THE SONOGRAPHER THAT I COULD BE PREGNANT AND THAT I SHOULD DO A TEST (APPARENTLY HE SAW SOMETHING AS PREGANCY)HOWEVER AFTER DOING 2 TESTS ,IT ALL CAME BACK NEGATIVE .I AM NOW CONFUSED ,IF IT IS NOT PREGNANCY WHAT COULD IT BE ?(I HAVE GOT PCOS AND DO NOT REGULARLY SEE MY PERIOD SO I CANT RELY TO THAT ).ANNABEL,THANKS Doctor: hello Annabels! Your question has answers in it. If you need a baby, treat PCOS.Lifestyle modifications will be a good start and are considered first-line treatment for women with PCOS. Such changes include the followingDiet(A diet patterned after the type 2 diabetes diet has been recommended for PCOS patients) increase fiber; decrease refined carbohydrates, trans fats, and saturated fats; and increase omega-3 and omega-9 fatty acids.Exercise regularly and minimum 45 minutes a day.Take vitamin D as 75 5 cases of PCOS resulting infertility are corrected once low Vitamin D levels are corrected.Weight loss is the most important thing you need.Use following medications Oral contraceptive agents (eg, ethinyl estradiol, medroxyprogesterone)Antiandrogens (eg, spironolactone, leuprolide, finasteride)Hypoglycemic agents (eg, metformin, insulin)Selective estrogen receptor modulators (eg, clomiphene citrate)Topical hair-removal agents (eg, eflornithine)Topical acne agents (eg, benzoyl peroxide, tretinoin topical cream (0.02\u20130.1%)/gel (0.01\u20130.1%)/solution (0.05%), adapalene topical cream (0.1%)/gel (0.1%, 0.3%)/solution (0.1%), erythromycin topical 2%, clindamycin topical 1%, sodium sulfacetamide topical 10%)See a fertility consultant you will need assistance in fertility. Try some clomiphene citrate and follow calendar methods and utilize active days of cycles for conception.If my tips didnt work after you followed them to every single bit of them, then come and grab me from my collar. Cheers & regardsDr S Khan"
},
{
"id": 60398,
"tgt": "I have been having pain under my right rib cage, I find it difficult to take deep breaths. I had nausea and felt dizzy",
"src": "Patient: Hi I ve been having pain under my right rib cage that either starts late at night and comes on quickly or i ll wake from my sleep in agony. If I nap during the day I never wake up in pain. The pain is like i m being stabbed but also stitch like. When i m in pain I find it difficult to take deep breaths, i m quite gassy and feel constipated although i m not. The last time I had it (which has been a lot recently) I had nausea and felt dizzy. Hot baths give me some relief and it feels like I need to put pressure on the area although this hurts. Stretching also gives relief for about a second. I usually end up pacing as I can t sit still. The pain usually lasts about 5 hours and then it will quite quickly subside enough so that I can go to sleep. When I wake up again the pain is gone. During the days I feel mildly sore in the area but will never have the stabbing pain. Also, the pain will either start with back ache or will start under my rib cage and then move to my back. Sometimes my whole back hurts, sometimes it s my upper back, sometimes my lower back. It s hard to describe the pain when i m not feeling it. i went to my doctors who said that he didn t know what it was but that it could be plurisy. I don t think it s plurisy. He gave me some co-codemol but they along with paracetemol and neurofen don t work. What do you think this could be? gallbladder , IBS, muscle spasm?? Is there any kind of relief that might work that could help indicate what this pain is? Thank You!! Doctor: sounds like gallstone attacks ive been diagnosed with them and your symptoms are exactly like mine"
},
{
"id": 2769,
"tgt": "How can successful conception be achieved?",
"src": "Patient: Hello, I am trying to conceive but not able to get pregnant. I have history of MTP before 4 years but afterwards I had not tried. Now I want to concieve and m trying since 3 times but not successful. Last attempt was USG guided ovulation but then also not successful. Please le me know the treatment. My gynec has suggested me tablet ovaflo or tab. susten Doctor: Hi, First of all you go for some basic tests like thyroid profile and Prolactin, and a semen analysis of your partner. You can take tablets for growth of your eggs and then an injection for rupture when egg size is more than 17 mm. You should be in contact with your husband for 2 to 3 days after injection. Ovaflo is for increasing the number of eggs. So if you have less no of eggs, you can take it. Susten is a progesterone which you can take after the rupture injection for 2 weeks.Hope I have answered your query. Let me know if I can assist you further. Regards,Dr. Khushboo Priya"
},
{
"id": 125716,
"tgt": "What causes muscle cramps in the lower legs and pain in the temples?",
"src": "Patient: i tossed and turned all nite bad cramps and muscle cramps in my lower legs i have a back issue i got up and took a gabapentin took a couple sips of water then i had to throw up got up this am and went to get my coffee and my hand shook when i tried to drink it spilt all over me balance was off and slight dull ache in my temples Doctor: Hello, Most probably it will be due to vascular causes. Consult a general surgeon and plan for a vascular Doppler. As a first line management, you can take analgesics like Paracetamol or Aceclofenac for pain relief. Hope I have answered your query. Let me know if I can assist you further. Take care Regards, Dr Shinas Hussain, General & Family Physician"
},
{
"id": 88666,
"tgt": "What causes lower left abdominal pain?",
"src": "Patient: Lower left abdominal pain at about mid line of pelvic bone. Pain is limited to a small concentrated area. Pain is a 2 out of 10 but only get pain when palpated aggressively. Doctor thinks it is a muscle strain, possibly of psoas from golfing. Began to notice the day after playing golf. History of diverticulitis and the pain seems similar to a diverticulitis flare up. Resection of splenic flexure. No fever, no constipation, appetite normal. Feel fine. Question: can muscle strain mimic diverticulitis pain? Doctor: HI.Muscle strain can not mimic diverticulitis pain. First of all remember the flare up pain of diverticulitis is very strong along with associated symptoms of fever and others. Your pain is limited to a smaller area only 2/10 on a scale and pains when palpated aggressively. This type of pain can be due to a muscle pull as you are thinking. It may aggravate due to movements of the abdominal musclesApply rubrafacient cream and take painkiller like Ibuprofen after food, it may go off.Psoas muscle spasm can not cause such pains as it is a deep seated muscle at the back portion of the abdomen can it increases by movement of the thigh. Muscle strain can not mimic diverticulitis pain."
},
{
"id": 140444,
"tgt": "What causes a numb patch on the shin after a fall?",
"src": "Patient: I fell on my shin a couple weeks ago and a little patch beside the cut is completely numb and when I touch it it feels like I m touching someone else. I put peroxide on it a few times. I picked the scab and it bled for a while but than I noticed it was like a hole and it s still there a few days later Doctor: Hello, Since you have numbness a small nerve may have been damaged. I don't think that there is something you can do for it, it should improve by itself with time. Hope I have answered your query. Let me know if I can assist you further. Regards, Dr. Erion Spaho, Neurologist, Surgical"
},
{
"id": 36402,
"tgt": "How long does it take to cure bacterial infection?",
"src": "Patient: HelloI have had what I thought was cystitis initially about four weeks ago and I bought some over the counter powder satchets and they seemed to relieve the symptoms for a short while and then the heavy dull feeling in my abdomen returned plus the feeling that I needed to pass water. I took another course of the cystitis satchets and again it seemed to relieve the feeling for a while but on and off I have had these feelings of heaviness and wanting to urinate. Last week the symptoms got worse and I went to the doctors. They took a urine sample and gave me some antibiotics and a few days later I was still in pain so I returned to the doctors. The urine test revealed that I did have a bacterial infection and the doctor said that the antibiotics prescribed were not particularly good for the sort of bacteria I had and he prescribed nitrofurantoin 50 mg tablets, 4 to be taken per day. This was last Thursday evening. I have been taking the tablets and over the weekend I did feel some improvement. My tummy has been really really swollen and I have been wairing pajamas all weekend. Now back at work in my largest size trousers I am feeling uncomfortable again and my tummy is really swollen. I have a couple more days tablets left - should I be concerned or expect not to be fully recovered by now please?I am a 53 year old woman and would really appreciate your advice. Doctor: Hello.Althought you're suffering adverse effects (bloated stomach ...) it is important to take all the tablets prescribed by the doctor;if treatment is stopped, the infection may return.When you finish taking the antibiotic, adverse effects will decrease.I wish you good health.(If the answer has helped you, please indicate this)"
},
{
"id": 180980,
"tgt": "Suggest an alternative to Oxycontin for right-sided palate pain while EBV and B-Cell disorder",
"src": "Patient: I have EBV& B-CELL LYMPHOPROLIFERATIVE DISORDER on the right palate in my mouth that is very painful. I have stage 4 Lymphoma and am undergoing tests before starting the chemo. The biopsy reveals significant numbers of EBV& cells which have a similar distribution as the PAX5 cells. Of course there is more- but although I have OxyContin 5mg to take 1 to 2 every 4 to 6 hours as needed- it s not really helping. Is there anything else that I can do for pain while I wait for the chemo to start? Ny gums are swollen and there is necrotic tissue and it seems to be worse every day! What about an antiviral medication? Doctor: Hello,I hope you are feeling better now.You may ask your doctor regarding tapentadol PR for replacement of Oxycontin.Since you are already on high dose medication , I would suggest you to do saline rinses.Also apply a local anesthetic gel over the ulcers so that you get some immediate temporary relief.Maintain the oral hygiene. Drink more water. Vitamin B complex and Vit C would help in healing as well.Hope you get well real soon."
},
{
"id": 142339,
"tgt": "How long should I continue torleva after having mild brain stroke?",
"src": "Patient: Good evening Sir. My father (59) was mild brain stroke 2 month before. Now he is recovering after treatment without operation only by medicine. doctor advice him to continue Tab Torleva 500. May I know how long continue ? he is doses is OD. Is this right ? Doctor: Hi, Welcome to HealthCareMagic.com I am Dr.J.Mariano Anto Bruno Mascarenhas. I have gone through your query with diligence and would like you to know that I am here to help you.Torleva or Leviteracetam has to be continued as long as the EEG is abnormal Please repeat the EEG every six months When it is normal, you can taper the dose and gradually reduce it and stop it Hope you found the answer helpful.If you need any clarification / have doubts / have additional questions / have follow up questions, then please do not hesitate in asking again. I will be happy to answer your questions.In the future, for continuity of care, I encourage you to contact me directly in HealthCareMagic at http://bit.ly/askdrbruno Best Wishes for Speedy Recovery Let me know if I can assist you further.Take care."
},
{
"id": 135786,
"tgt": "What causes pain and swelling in left wrist?",
"src": "Patient: I was up most all night with pain in left wrist and swelling. I took 2 gen. Advil and put a carp. Tun thingee on the wrist...the pain was really bad. I laid on belly with wrist under pillow. Swelling went down a bit by time to get up. I broke arm when 5 and in early 30s the bridge that went over ligaments??? Was broken and never repaired. Sound like arthritis and should I use cold or hot. Thanks for whatever you say. Doctor: HiWelcome to healthcaremagicI have gone through your query and understand your concern. There is possibility of synovitis or arthritis. It can be distinguished by clinical examination and radiological examination. Treatment of both the condition is similar that is rest and analgesic . You can discuss with your doctor about it. Hope your query get answered. If you have any clarification then don't hesitate to write to us. I will be happy to help you.Wishing you a good health.Take care."
},
{
"id": 89269,
"tgt": "What causes when pushed where the ovary is, it hurts below breasts?",
"src": "Patient: Hello night before last my throat felt like it had a lump stuck in it when I woke the next morning I had a mild tummy ache as the morning went on it turned into severe cramps from my lower abdomen to under my breasts it hurt all day and worse everytime I ate or drank anything went to bed last night wiped out and woke today feeling better but sore all over like if I push gently on my lower side like where an ovary might be it hurts in my upper abdomen just below my breasts Got any ideas??? Doctor: Hello,Pain in the stomach associated with food or liquid intake associated with some lump like sensation in the throat could be ( either GERD, Esophagitis, Gastritis due to some bacterial infection) or any other local causes, please kindly consult your doctor , get examined, get an USG abdomen or UPPER GI endoscopy, so the cause can be evaluated and hence treatment planned. Hope I could help you.Thank you."
},
{
"id": 194042,
"tgt": "Can I mastubate after a month od laproscpoic pyleoplasty?",
"src": "Patient: Hi I am a 25 year old Male. My question is related to urology .I had gone through a laproscpoic pyleoplasty about one month back due to PUJ obstruction in my right kidney , now I am taking rest . two weeks after the surgury I started travelling and my body was shaking due to the vibration while travelling. Will it lead to any complications.? I have a stent inside. I am not getting any pain as of now. I can't see any blood stains in my urine as well. and also Can I masterbate now.? any problems.? Doctor: Hello, I don't see any problems for masturbating or ejaculating or any sexual activity 2 weeks after laparoscopic pyeloplasty. I'm sure you will do fine. just no exercise or strenuous activities for at least a month after. Hope I have answered your query. Let me know if I can assist you further. Take care Regards, Dr Manuel C See IV, Urologist"
},
{
"id": 85854,
"tgt": "Does intake of mobizox tablet cause pain on the right side of kidney?",
"src": "Patient: Hello Dr , i am suffering from lower backpain L4 and L5 compression towards right hip. Now started mobizox tablets with pantacid. After taking this medicne i am getting pain on the rightside of my kidney .Shall i continue the medine or not.please advice me. Doctor: Hello, Mobizox is a pain reliever and muscle relaxant. It does not have any known side effect on the kidney. I suggest checking the kidney function with examinations such as Azotemia level and Creatinine level. Hope I have answered your query. Let me know if I can assist you further. Regards, Dr. Dorina Gurabardhi, General & Family Physician"
},
{
"id": 155185,
"tgt": "Is there any chance to reappear the removed fibroid tumor?",
"src": "Patient: Some 10 to 15 years ago I had a fibroid tumor removed, a very large one. At this point in time I have finished going through menopause at 52, common for my mother's family, but I have note that now and again, I have been having pains in the exact same point. I'll experience the same pain and then won't make itself known for months or years. I've read that they can come back, should I be worried and see a doctor? Doctor: HelloPain may be due to some other reasons.Once fibroids are removed they do not grow back usually.Besides clinical evaluation,you also need ultrasound of pelvis.Pain may be due to other causes like pelvic adnexal pathology,genitourinary causes etc.Proper treatment depend upon findings.You should consult your doctor.Get well soon.Take CareDr.Indu Bhushan"
},
{
"id": 62485,
"tgt": "Suggest treatment for lump like sensation in throat",
"src": "Patient: Hi i have had a lump sensation in my throat for about 3 months now i wake thinking iv e swollowed odd objects it feels like a ball between my neck and chest i am a smoker and drink coffee alot and it seems to worsen after coffee. I ve also had a really bad pain in the right side of my stomach above my hip (like my ovaries) and lots of clear metalic smelling discharge i don t know if its all llinked but if someone can please help. Jess Doctor: Based on the facts of your query,you seem to suffer from Globus Pharyngeus with GERD.Causes of the GERD-with Acid Reflux needs to be investigated by Physician or Ent specialist,who would help you out.Suggested Treatment-Second Opinion from Ent Surgeon and scritpt from your GP doctor.Restrict Coffee and smoking for 2-3 weeks till you get relief.Bland Diet/Avoid street/ fried foodInvestigate the cause of the GERD causing Acid Reflux and Take its treatment accordingly.ProtonPump Inhibitors with Domperidone x for 2-3 weeks would help you.Antispasmodic to control stomach right side pain.Antibiotics with Metrogyl to control the acid reflux caused by intestinal infection.Invest-USG study with /Stool / with Blood eosinophilia would support the treatment.Your Ent surgeon would deal with needful treatments of other causes of the GERD with Globus Pharyngeus.I will not tax you with medical names.Hope this would resolve your worry.Welcome for any further query in this regardWill appreciate writing your feedback review comments,to help the needy patients like you at HCM.Good Day!! Dr.Savaskar,Senior Surgical SpecialistM.S.Genl-CVTS"
},
{
"id": 149310,
"tgt": "X-rays showed mild subchondral sclerosis. Having problem in walking. Treatment?",
"src": "Patient: Yes, last January my x-rays showed I have mild subchondral sclerosis in my left hip...my abilities for walking, up and down steps, standing as well as intimate relations has deteriorated....I am looking for a doctor in the Holland MI/Zeeland MI area for further consultation regarding hip replacement...this is a condition that runs in my family... Doctor: Hi,Thank you for posting your query.For more clarification in diagnosis, you may get MRI of the hip done. Then, you should consult an Orthopedic surgeon for opinion. Total hip replacement is a safe and effective treatment for your problem.Best wishes,Dr Sudhir Kumar MD DM"
},
{
"id": 123305,
"tgt": "Cause for the toe to itchy along with pain?",
"src": "Patient: hello? i cant pay for this but i dont know what to do my little pinky toe started to itch three days ago i itched it and now it has a red spot and a line connected going up my right foot...it hurts so bad i cant walk on it so good and i cant move it either... Doctor: Hello, The pain and itching in the foot can be due to a fungal infection in the area. This needs proper oral antibiotics to take care of the problem. Do consult your physician for proper prescriptions. Hope I have answered your query. Let me know if I can assist you further. Regards, Dr. Praveen Tayal, Orthopaedic Surgeon"
},
{
"id": 54802,
"tgt": "Suggest treatment for fatty liver",
"src": "Patient: yes i have been on Suboxone for a year I was on pain meds for 10 years or more i have a hard time breathing but i have had this problem before starting suboxone i remember a exam one time was told i had a fatty liver what do i ask for to get liver checked out my stomach stays swollen to i have no patience lots of anxiety starting to worry Doctor: Hi thanks for contacting health care magic.Noted you are taking buprenorphin drug.This drug used for opioid addiction patient or for chronic pain.You might have drug induce liver affection.The dyspnea problem might be from excess stress lead to panic attack and so dyspnea...Try to avoid stress in routine activity.Daily exercise is effective antidote ...Yoga and meditation helpful...Avoid excess fatty diet.Alcohol avoided.If need for smooth bowel movement fiber supplement needed.Kiwi , cooked oatmeal, avocados good food ...If still digestive problem remain USG or endoscopy needed.Take care..Dr.Parth."
},
{
"id": 32453,
"tgt": "Suggest better treatment for jaundice",
"src": "Patient: my father,s age of 74 years.he is suffering from jaundice since 7 wks.symptom-yellowish eyes & body,loss of appetite,gas formation & eructation,occasinally nausea & vomiting & weakness.Now serum bilirubin level is 34.7,conjugated-31.sgot-132,sgpt-68,alkaling phosphate-206,usg abdomen-mild hepatitis & no obstruction.endoscopy-mild gastral antritisMRCP-normal study,total protein-6.5,serum albumin-3.4,prothrombin time-13.9,HEPATITIS-A,B,C,E--NEGATIVE,ANA-22.25,AMA-NEGATIVE.What is the probable diagnosis?& what should i do? Doctor: Hello,Welcome to health care magic,Considering your billirubin value (especially high conjugated billirubin) you are suffering from hepatocelluar jaundice which needs to be treated.You will require antibiotic to prevent super infection with symptomatic treatment like Udilive 300 mg for liver enzyme supplements and some dietary restrictions. For diet,you should follow these guidelines,Avoid fatty fried and junk foodHave lots of fresh fruits, fruit juices and vegetables.You should take glucose water 3 to 4 times a day.Avoid oily,fatty or even high protein intake.You should have whole grain food items chapati and Daliya or Khichadi.Do you feel weakness?or How's your appetite?You should check your blood counts and liver function tests almost daily in initial phases.Consult your doctor or family physician for complete treatment.Regards,"
},
{
"id": 166156,
"tgt": "Suggest exact schedule for BCG",
"src": "Patient: good morning my baby was born on 08.03.2011 and the village nurse says that she will give BCG on 06.04.2011. So my question is what is the exact schedule for BCG and by giving BCG on 06.04.2011 we are in right time? baby-vaccination-schedule-india . Doctor: Hi, welcome to HCM. BCG vaccination is given at the time of birth as per vaccination schedule in India. Take care."
},
{
"id": 40298,
"tgt": "What is permanent cure for yeast infection which appears before periods?",
"src": "Patient: I have had chronic yeast infections for 9 years now. I have done everything to a gluten free diet, all kinds of medicines, you name it I have done it. I get them now before my period and it s very aggravating. I am so tired of just getting treated for my problem and no one is getting to the bottom of what is actually causing me to get this problem. Any suggestions would greatly be appreciated. Doctor: Hello,Welcome to HCM,Your condition should be evaluated completely to know what is the exact cause for your symptoms. You may need lab investigations like KOH mount to confirm the fungal infection. The fungus are normal comensals of the genitals which is kept under control by the helpful bacterias, whenever there is imbalance in the environment you may develop symptoms of fungal infection.For your condition I would suggest you to follow some remedies like1.Maintain proper hygiene2. Topical antifungal creams3.Oral antifungal like Tab Fluconazole once in a week for 3-4 months.Thank you."
},
{
"id": 207130,
"tgt": "Why do I get sudden attacks of blurred vision and sweating?",
"src": "Patient: For a while now I have been experiencing strange symptoms in certain situations. If i am injured (no matter how minor), if I see someone else injured (no matter how minor) or if I visit someone in hospital after they have had surgery I go into these phases where I begin to lose vision, it becomes narrow and blurry and then goes white, i then get extremely hot and sweaty and my face goes grey. In most instances my vision goes completely white and I am blinded and I usually lose any strength in my legs and fall over. After a short while of sitting down and putting water on myself it tends to pass. Is this some kind of anxiety attack or is it something more? Doctor: It is most probably an anxiety attack, something like phobia. But you must get your blood pressure and blood sugar checked immediately."
},
{
"id": 112006,
"tgt": "Why has the back pain not subsided after having a fusion with discectomy to compress an exiting nerve?",
"src": "Patient: had fifth back surgery last week. Last dec had fusion at L4L5 with discectomy due to compression on exiting nerve. my pain continued as it did prior to surgery. there was no change in pain even though dr had found a piece of disc 5 centimeter in length by 1 mm to 5cm in width on nerves. it was recently found that there was still a far lateral flattening of the L4 nerve, partial disc and scar tissue-it was removed. questions:wouldn t the fusion surgery have dealt with this issue and is it normal to have severe femoral learned pattern and other parts of leg hurt? burn, constant twitching-same as last years surgery. Is this the nerve repairing or most likely its been compressed too long to come back? l4 I had a fusion last year at L4L5, with continued pain that had not changed with this surgery it was then discovered that the L4 nerve was flattened (even though they found that it was already compressed with a far lateral herniation at the fusion surgery) evidently, it was flattened even more lateral as well, my question now is I am having a lot of femoral nerve symptoms, could this be just nerve rejuvenating (this did this after fusion as well), a lot of pain and constant twitching and burning along all L4 learned pathway as well as other parts of the leg, is this normal? and after a fusion why would there be a compressed nerve when this surgery was to correct this? Doctor: HiI think if the full discectomy was done which used to press the nerve, then after decompression your pain should have disappear gradually. But as yet you have pain you should try with short course of steroid along with methylcobalamine and pregabalin tablets for temporary solution and should get a fresh mri lumbar spine with whole spine screeing done...then consult good spine surgeon.Get well soon.Thank you."
},
{
"id": 223646,
"tgt": "Is pregnancy possible despite intake of contraceptive pills?",
"src": "Patient: Hi . I am Maushumi from India . I had sex on 15th of jan with my bf and took emergency contraceptive within 6 hrs. after two days I had sex again. I was drunk hence don t remember weather he has ejaculated inside of me or not . this time i didn t took any pill. however yesterday I had an urine test with my pregnancy test kit and the result was negative. still I am confused. Doctor: Hello dearI understand your concernThere is chance of the pregnancy as you had not use pill after second intercourse.Pregnancy test will give positive result 8-10 days after the missed period.Emergency contraceptive pill also cause delayed in the period y 10-12 days.Now if your period will delay by more than 12 days then go or urine pregnancy tests and or blood HCG to confirm the pregnancy.If you will be pregnant then still you can terminate the intrauterine pregnancy by ABORTION pill under the advise of the gynecologist.Avoid stress, take healthy diet, drink plenty of water and do regular stretching exercise.Hope this may help youContact HCM for further health queryBest regardsDr. Sagar"
},
{
"id": 12193,
"tgt": "Suffering from scleroderma for the past 12 yrs and black spots appearing on head, ears, chin",
"src": "Patient: hello Dr iam 23yrs scalaroderma suffer from 12 yrs black spotss appering on my head ears chin nosehow it hi doctor ......iam scalaroderma patient &23yrs girl &unmarried..... i have black spots in my head & face ....please help me ? prescribe some lotin, cream...? Doctor: hi they may be black head. apply adaferin gel at night on black spot dont prick or remove it use good cleanser avoid oily and spicy food"
},
{
"id": 115304,
"tgt": "What causes blood in saliva and after coughing?",
"src": "Patient: WHEN I WAKE UP IN THE MORNING I SPIT AND THERE WAS SOME FRESH BOOD WITH SLIVA AND WHEN I TRIED TO TAKE SOME WATER IN THE MOTH AND THAN SPIT IT IT CAME BACK OUT PICK AND THAN I TRIED TO COUGHT AND TRIED TO GET WITH IN THE THE THROUGHT THAN A SERIOUS OF FRESH BLOOD START COMING OUT. Doctor: Hi, dearI have gone through your question. I can understand your concern. You may have either haemoptysis or haematemesis. You need x ray chest, ultrasound abdomen and endoscopy. It will give you idea regarding cause. Then you should take treatment accordingly. Hope I have answered your question, if you have doubt then I will be happy to answer. Thanks for using health care magic. Wish you a very good health."
},
{
"id": 127858,
"tgt": "What causes left-sided shoulder pain and tightness in the arm and chest?",
"src": "Patient: Sudden pain in left shoulder, under shoulder blade. Took Tramadol, and rubbed topical pain relief. Pain gradually moved from back to chest mid-breast bone. Difficulty taking deep breath without heaviness in chest. Took 2 puffs albuterol, drank 16oz of water. Pain slowly moving to top left arm. Now tightness in arm and chest. All this began around 5:30 pm today. Pain and pressure went from severe to mild by 9:15 pm EST. Should I go to urgent care or emergency room tonight or check with Dr tomorrow? Doctor: first You do investigations like ECG x ray c spine.it will helpful for diagnosis until I will suggest you some Ayurvedic medicinetab yogaraj gugglu 2 bdtab sarpgandha vati 1bt"
},
{
"id": 167535,
"tgt": "What causes lower heart beat?",
"src": "Patient: My daughter s heart rate after childbirth was 32 and the doctors did not know why. She underwent a lot of tests and they told her she had a health heart and that this would go away by itself. It has been two weeks and it is only 40 and she sometimes feels light headed. Have you ever heard of such a case? Doctor: Hi.... I see that your daughter might be having congenital heart block. If the mother is having SLE, then this becomes all the most important. I think that your daughter should be worked up for conduction defect in the heart like heart block.Regards - Dr. Sumanth"
},
{
"id": 167713,
"tgt": "What could vomiting and red spots in mouth indicate?",
"src": "Patient: My 5 yr old daughter has red flat spots around her mouth. They are pin point in size. She threw up this morning out of the blue. She has no fever and has been vaccinated but she hasn't gotten her 5 yr shots (turned 5 in March 2011). Should I be worried? Doctor: Hello,I can understand your concern. The red flat spots around mouth indicates presence of allergic reaction or viral infection. As she is vomiting too, it seems more like to be a viral infection. I would advise you to visit a pediatrician for the physical examination and investigations regarding viral infection. Confirmed diagnosis and treatment can be based on the test reports. I hope this information helps you. Thank you for choosing HealthcareMagic. I wish your daughter feels better soon.Best,Dr. Viraj Shah"
},
{
"id": 50907,
"tgt": "Cured of diberticulitus. What can be done to combat this from coming back ?",
"src": "Patient: My doctors say I am cured of my illness . I had diberticulitus that poisonedmy blood and caused liver absesses that had to be drained 3times. It caused kidney problems also. My recent test show that everything is normal. Includeing my liver. I have been through a lot and am concerned that it will come back again. I was in hospital for a month and then home nurse for a month. What can be done to combat this from coming back. THANK YOU........AAAA Doctor: Hello and welcome to HCM You seem to have suffered from acute diverticulitis with infection that had gone to liver and formed and abscess. Recurrence of diverticular disease is common and can be best avoided by diet alterations. Patients should eat a fiber-enriched diet that includes 30 g of fiber each day. Fiber softens the stool and helps prevent constipation. It also can help decrease pressure in the colon and help prevent flare-ups of diverticulitis. High-fiber foods include: Beans and legumes,Bran, whole wheat bread and whole grain cereals such as oatmeal Brown and wild rice,Fruits such as apples, bananas and pears Vegetables such as broccoli, carrots, corn and squash. Drink at least 8-10 cups of fluid daily. Fluid will help soften your stool. Exercise also promotes bowel movement and helps prevent constipation. The incidence of complicated diverticular disease appears to be increased in people who smoke. Therefore avoid smoking. For long-term medical management of uncomplicated diverticular disease, rifaximin (a poorly absorbed broad-spectrum antibiotic), is associated with 30% less frequent recurrent symptoms from uncomplicated diverticular disease. Furthermore, the use of probiotics has been shown to decrease the incidence of recurrent attacks. Hope i have answered your queries. Wish you good health."
},
{
"id": 178035,
"tgt": "Suggest treatment for bone fracture in baby",
"src": "Patient: My 1 year old son just started walking not so long ago and yesterday he fell on his left arm and I took him to emergency room for x rays and it showed a fracture between the elbow and wrist but the hospital does not have a dr who works on patients that young so Im going to a childrens Khospital to have it fixed,Can you help me to understand what will be done to repair the fracture? Thank You. Danielle Doctor: Hi....if the fracture is not too much displaced they will put mild traction and put a plaster cast and advise a sling to be in place for 6 weeks.If the fracture us displaced too much, he might require an open reduction and wiring to keep the fractured segment in alignment.Regards - Dr. Sumanth"
},
{
"id": 6507,
"tgt": "When will ovulation start and when is the right time to have intercourse to conceive ?",
"src": "Patient: Hi i am 35yrs will be 36yrs on oct 22nd and my period is a normal cd26- cd27 but i was told by my doctor that i have pcos in my right ovary and my left ovary is fine. i do have a 12 yrs old son had an unwanted pregnancy and now i want to have another child. my period started july 27th 2011 and i started taking clomid cd 3-7 i waited for my period to start on the 22-23 august 2011 and it never came until august 27 2011 which took it into 31days being late. i will like to know if i did in fact ovulated which caused the change in my cycle and also i am now on clomid cd 1-5 which i took on sat 8-27-11 i will like to know when should i expect to ovulate and start having interourse? thank you Doctor: Welcome to HealthcareMagic forum normaly after the last dose of clomid over, one will ovulate within 5th to 9th days. it means if you took last dose on suppose 3rd dec. then you will ovulate between 8th to 12th dec. just have regular intercourse within the fertile period, or say atleast alternate day. if your menses is late it has nothing to do with ovulation under clomid. it has to do only if one is not on clomid. so be relax and try. I hope I answered your query. Wish you good health."
},
{
"id": 110797,
"tgt": "How to treat severe back pain?",
"src": "Patient: helloI'm 28yo male, 5 '9 \"tall, and 200 lb.I was leaning forward today (my back was about 60 degrees) and i was holding a not so heavy iron clothes, and suddenly I heard a quite pop and felt pain in my lower back (a little lower than the height of my bellybutton).I'm able to walk, sit down and stand up, but even that my posture looks right, I feel that I'm leaning forward a small bit, and I'd feel pain if i tried to stand fully right. and I do feel pain when leaning forward without holding onto something.I also feel pain when I try to take wide steps. I can only take small steps.I never had back injury .. I had usual back pain due to bad mattress but nothing major.I won't be able to visit doctor till tomorrow so i want to know if this is serious, and how to deal with it until then.thank you. Doctor: Hello, Thanks for your query.From description , it seems you might developed muscular or ligamentous strain over back .If pain is severe ,I would suggest getting this evaluated by an orthopedician for an accurate diagnosis and appropriate management. You may get the MRI of the area affected under his/her guidance.Meanwhile following measures will help with the pain\u2022 Lie on a hard bed.\u2022 Anti -inflammatory drugs like Tablet Motrin 1 tablet as and when required \u2022 Avoid forward bending.\u2022 Avoid strenuous activity & lifting heavy weights.\u2022 Apply diclofenac gel on the affected area. Warm compresses will also help.I do hope that you have found something helpful and I will be glad to answer any further query.Take care"
},
{
"id": 187172,
"tgt": "What treatment to take for tooth infection?",
"src": "Patient: Hello I've been on atiboitics for about a month now for a sever tooth infection the infection won't go way and pain meds aren't helping the pain at all nor is Ora gel working any more either could it be something more the just a simple tooth infection Doctor: Hello, Welcome Thanks for consulting HCM, I have gone through your query, as you have toothache , see pain in tooth can be due to if your tooth is carious or there is any periodontal problem that is problem in your gums . For this medication doesnt helps in giving you complete releif , go for treatment restoration or RCT ,scaling and root planning . Do warm saline gargle two - three times a day. Hope this will help you. Wishing you good health."
},
{
"id": 197690,
"tgt": "What are causes & treatment for pain in male genitals?",
"src": "Patient: My husband (28) is feeling a needle pricking kind of sensation in his a**hole from last 2 weeks? He tried keeping it clean and dry , applying talc powders but it doesn't seem to get better. Can you tell me what this is and how can it be treated orally ? Doctor: HelloI appreciate your concern for your husband.Needle pricking kind of sensation in anus might be caused be fissure in Ano, hemorrhoids or infected anal gland and perianal abscess.In my opinion he should apply anaesthetic ointment like lignocaine for symptomatic relief.Ask him to avoid constipation by taking laxative or stool softeners like liquid paraffin.Ask him to avoid fatty fried and junk food, advise him to take lots of fresh fruits, vegetables and dietary fibers in his diet.No smoking and alcohol for time being until it heals completely, if he is into it than.Sitz bath or hot water bath will definitely help him.Hope this answers your question, please do let me know if you have any more queries, I will happily answer you.Best wishes"
},
{
"id": 70133,
"tgt": "What is the lump behind my testies?",
"src": "Patient: hi there i shower every day or so n the other day i notice that i had a lump behind my ballsack and before the line starts for my bum, when i check it, it was quite small but over the last day or so it has incresed in size, its not a soft lump, and i know that its not my testicle Doctor: Hi ! Good evening. I am Dr Shareef answering your query.Most likely you have got a sebaceous cyst on the skin between your scrotum and starting of your bum. This is a benign condition, but might increase in size, and could give rise to abscess due to infection. It is better to get rid of it before it complicates. If I were your doctor, I would refer you to a general surgeon who would examine it physically and decide on further management of the condition.I hope this information would help you in discussing with your family physician/treating doctor in further management of your problem. Please do not hesitate to ask in case of any further doubts.Thanks for choosing health care magic to clear doubts on your health problems. Wishing you an early recovery. Dr Shareef."
},
{
"id": 154309,
"tgt": "How serious is Chondrosarcoma (Grade I) of femur bone?",
"src": "Patient: hi im 20 yrs old 5'7 i45-150 pounds i jus recently had surgery on my right femur bone they found a tumor so they removed it so 3 weeks past nd i went to mount sanai yesterday and they found the name for it its called low grade level 1 chondrosarcoma is this good Doctor: Thanks for your question on HCM. I can understand your situation and problem. Chondrosarcoma is bone tumour of younger population. If detected in early stage, survival is good. Early detection and surgical removal is the treatment of choice. Grade indicates severity. Grade 1 indicates low grade malignancy. So it has less chances of distal metastasis and local recurrence. So you had surgery and having grade 1 Chondrosarcoma,both of these favour good prognosis. So your Chondrosarcoma is having good life expectancy and five year survival rate."
},
{
"id": 179609,
"tgt": "How safe is taking Zifi,Ambrodil-lx and Merizyme drops for cough?",
"src": "Patient: Hi My son is 4 months old and weighs 7.27 kgs. He is having cough since two days. Today afternoon he vomited cough with milk(he is only on breast milk). He passed green stool once. I showed him to a doc. He has prescribed the following medicines Zifi50 - 3.5 ml twice a day Ambrodil-lx - 3.5 ml thrice a day Merizyme drops - 10 drops thrice a day Enterogermina twice a day I gave him the prescribed medicines and immediately within a minute he fell forcefully asleep. I am worried about the folkowing Are the medicines and doses prescribed correct? If not will it harm my baby? Is taking all these medicines together ok? Should there be a gap between 2 medicines? Plz reply asap. I m really worried Thank you Doctor: Hi Medicines prescribed are correct , there is no sedatives in these medicines, they will not harm in prescribed doses , you can give medicines in gap of 5 minutes . More important for you you is to see is baby active , sleeps well, accepting feed , no high fever, no large watery loose stools, passing urine every 4 hourly , there is no rapid breathing / chest wall retractions . thanks"
},
{
"id": 59252,
"tgt": "Not able to sleep on the back after gall bladder surgery. Ultrasound shows fatty liver. Bubbling sound from the ribs. Advice?",
"src": "Patient: well I am 44 yrs , 5'3 in height an about 87 kilo of weight, I had my gallstone removed in the year 2010, ever ssince the surgery I am feeling very sick especially at night i cant lie flat and doctors here have done ultra sound and they keep telling me that I have a fatty liver but more then if thats the sace why do I keep hearing bubbling noise in my right side of my ribs and I have to keep on changing my positions while sleeping I feel the pain getting less when I am sitting up, pls advise me Doctor: Hello, karmasonam69, You had gallbladder surgery three years ago and probably this was done through laparoscopic route and not open surgery. If the surgery was difficult due to location of the gallbladder or too much inflammation around , it is possible to develop some adhesions or scar tissue around that area. There is a part of the colon called the hepatic flexure that is close to the liver. If there is some kinking , gas can get trapped in that area and give you the bubbles. When you change your position the gas moves and you may get relief. Unless the problem is major you really don't need another surgery. Try to take simethacone containing pills that may help to break up the gas bubbles. Make sure you drink enough water also. You might consider getting an x-ray of your colon (Barium Enema) to see the shape of the colon. I wish you well."
},
{
"id": 182386,
"tgt": "What is causing a lump near the extracted tooth?",
"src": "Patient: hi there i had a tooth extracted just over 3 years ago now and have been back to the dentist many many times since then due to a big lump above extraction site plus keep getting abscesses , have burning and sting to right side of tongue, he always says it is caused by smoking or come back in 3 months but no other treatment now i have a new dentist and i demanded a x-ray, she states that i have a infection in the bone and that only surgery will solve it now, would this be what is causing the lump? how much damage will have been done as it has gone on for so long what will surgery do? Doctor: Hard lump below tooth extraction site can be there because of some tooth part still left in tooth extraction socket or can be a bony prominence present. It can also be there because of some cyst or some other pathology. Without clinical examination, it is difficult to give definite diagnosis. You can visit your oral surgeon for evaluation and treatment. If hard lump is because of part of tooth still left in tooth extraction socket, then it can be pulled out. Part of tooth still left in extraction socket can cause infection and pain later on. If it is a bony prominence present, then it will regress on its own or surgical intervention will be required for it. For more info on complications after tooth extraction,"
},
{
"id": 102869,
"tgt": "White substance in throat, have had tonsils removed. Used lemon water and allergy sprays. Why do I have flu?",
"src": "Patient: I have this disgusting white stuff in the back of my throat. The only way I can get rid of it is to gargle with warm lemon water. It is so sticky-it hangs in my throat. It won t go up and it won t go down--it just hangs there. How can I get rid of it. The lemon water lets me spit it up in the sink, but that is the only way and then it is back in an hour or so. I have had this for almost a year. I have tried allergy sprays and decongestants. I have had my tonsils out-recently-nothing is helping. They put my on clindamycin-150 mg--I can never blow anything out of my nose. I don t know what to do. I have never had allergies before--I had this problem after having the flu last December. Doctor: you may have allergies or silent chronic allergic sinusitis and tonsilectomy o not helpneed to investigate for sinusitis and allergies after consulting ent and allergy doctor and it can be treated accordinglyi have treated cases like this"
},
{
"id": 72026,
"tgt": "How to cure panic attacks,shortness of breath and difficulty in swallowing?",
"src": "Patient: hello,i recently have been having problems with panic attacks, including shortness of breath and problems swallowing.Tonight I had a chinese take away, which seems irrelevant, but ever since I have had problems. I had to drive home from where I had the food, and on the drive home I had to stop half way as the right hand side of my body felt numb. I am now home after my brother drove me back, and the right hand side of my body still feels the same.In my head I have a tired right eye, my ear feels blocked and my right arm/cheek and glands feel dull.I am freaking out and dont know whether to go to the hospital or not. Doctor: Hello According to the history you provide i think that you should go to consult your doctor.Regards Dr.Jolanda"
},
{
"id": 83220,
"tgt": "Can zestril cause any side effects for long period?",
"src": "Patient: i m 47/m. having hypertention since 3 years. my doc. advized me loprin 75mg and zestril 10 mg. i m regularly taking the medicins from last 1 and 1/2 years. my b.p. is normal now, but when try to stop the dose the b.p. becomes high. can i continue the medicins? what is risk for contnuing medicins? and what r the side effects for these medicins. thanks. tahir mahmood. Doctor: Hi, I feel that since after taking the medicine - Zestril your blood pressure is controlled, so you must continue taking the medicine. It is very important to keep blood pressure under control. Zestril relaxes the blood vessels in your body and reduces the stress on your heart. So it is a highly useful medicine. As far as side effects are concerned, commonly reported side effects are headache, dizziness, abdominal discomfort, or cough. I think you should continue taking the medicine and along with that keep a watch for side effects- like swelling of feet or hands, urine output, cough, tiredness. If any of these occur, you can consult the doctor immediately . also regularly monitor your kidney function tests- including sodium-potassium. Take care. Hope I have answered your question. Let me know if I can assist you further. Regards, Dr. Milinda Gupta, Internal Medicine Specialist"
},
{
"id": 154376,
"tgt": "What causes a swollen abdomen?",
"src": "Patient: I am having my abdomen getting bigger all the time, I have a history of stomach cancer with it being removed. I have not gained any weight as of recent. I gained wt after my car accident from 130# to 212#, I currently weigh 212#, but abdomen seeming to get larger with no wt. gain. Periods gone since stomach cancer Doctor: Hi, dearI have gone through your question. I can understand your concern. You may have ascites or other cause for stomach enlargement. You should go for ultrasound abdomen. It will give you exact diagnosis. Then you should take treatment accordingly. Hope I have answered your question, if you have doubt then I will be happy to answer. Thanks for using health care magic. Wish you a very good health."
},
{
"id": 198456,
"tgt": "Why is my scrotum growing more than its required size?",
"src": "Patient: hellow I have a query .............from last few months I have observed that one of my sex organs, ball has been growing up and have an odd shape. It is like ball is looking compare to bigger than the penis...........it is any symptom of bad diseases. Doctor: Hi,From history it seems that you might be having either,Hydrocele or,Indirect inguinal hernia.Consult surgeon and get clinically examined.On examination it will give exact nature of your problem.Ok and take care."
},
{
"id": 167915,
"tgt": "Suggest treatment for swollen lymph nodes with cold",
"src": "Patient: My son, who is almost 2, has swollen gums and I believe he also has swollen Lypmh nodes under his jaw. He has been complaining of pain in his mouth and has been fussy. He has been cutting a lot of teeth recently, which I think may be the problem, but whenever I brush his teeth recently he cries in pain and the gums bleed. I have taken his temperature and so far he has no fever. But, the swollen Lymph Nodes do concern me, and I wonder if that is usual for children cutting teeth, or if I should be more concerned. He also had a cold and a sinus infection and has just recently Since it is the weekend I can t take him to his doctor, so I was wondering what may be causing this and rather I should be more concerned or if it can wait until the weekend is over. Doctor: Hello,I can understand your concern. The gums might seem swollen as the primary teeth might be erupting into the oral cavity around this age. At this time, the gums overlying the erupting teeth become sore and they cause pain which makes the child fussy and irritable along with his denial to eat anything. However, the swollen lymph nodes indicates presence of infection which might be in the gums as an alternative to the diagnosis of the erupting teeth. The gums in rare cases swell and get infected and bleed a lot in small children. It may require treatment with professional cleaning/scaling and a course of antibiotics.I would recommend you to apply orajel over the gums for easy eruption of the teeth. In addition, consult a dentist or pedodontist after the weekend is over for clinical examination of the oral cavity and ruling out presence of infection. I hope this information helps you. Thank you for choosing HealthcareMagic. I wish your son recovers soon.Best,Dr. Viraj Shah"
},
{
"id": 75593,
"tgt": "How to cure typhoid fever?",
"src": "Patient: My mother is suffering from typhoid. She i 65 years old . Initially she had high fever for 15 days now, it is between 99.2 F for the last 5 days. How many more days it will take to have normal body temperature? She had paraxin and monocef injections for 7 days. Doctor: Hello and Welcome to \u2018Ask A Doctor\u2019 service.I have reviewed your query and here is my advice. Your mother is having typhoid infection. She was given an injection for 7 days. Now complete the course of prescribed oral antibiotic for at least 7 days.Advise her to take rest. Consume fruit juice more. Protein diet like egg albumin, boiled fish can be given. Small semi-solid meal taken at frequent interval. Avoid excess spicy and heavy meal. Fluid intake should be more.After symptoms improve, stool culture is done. If bacilli are still there then repeat the course of antibiotic given.If no improvement then by culture resistance bacilli should be searched. If resistant bacilli are there then ciprofloxacin like antibiotic is needed.Take care. Hope I have answered your query. Let me know if I can assist you further.Regards,Dr. Parth Goswami"
},
{
"id": 100445,
"tgt": "Suggest treatment for allergy",
"src": "Patient: Hi Doctor. I am 36 years old and I suffer from allergies. The winds were very high about a two weeks ago and set my allergy symptoms into overdrive. I have been taking my allergy medications, nasal rinse and nasal spray. I have had a sore throat for a couple of weeks and have pain eating. I noticed yesterday that I have enlarged taste buds also. My throat is definitely red, but I do not have any yellow or white spots on my tongue. I have also been terribly tired for the past couple of weeks. What do you think this could be? Doctor: HelloThank You for contacting HCM.This could be due to Sore Throat. As you have chronic allergy problem so i would suggest you following things:> Take hot water in a cup, add 1 table spoon salt in it. Perform gargles (mouth wash) with it three times a day for at least 3 days. It will give soothing effect.> Take boiling water in a pot, put your head over the steam and cover whole with a sheet. Take deep breaths in it.(Avoid getting closer else it might cause burn). Do this twice a day.> Take montelukast one daily for one month. Research has shown that it decreases the allergic symptoms considerably.I would also suggest you to under go allergy testing at allergy clinic as you have chronic allergy problem. It will tell that you are allergic to what specific thing. The results will help an allergist to prescribe you immunotherapy to that specific allergen and it will improve the problem.AVOID:> Dust & Pollen> Chilled water> Drinks> Spicy foodHope this answers your question. If you have additional questions or follow up questions then please do not hesitate in writing to us. Wishing you good health."
},
{
"id": 145864,
"tgt": "What does the MRI of spine indicate?",
"src": "Patient: Had MRI which showed bulging disc at c3-c4, c-4-c5 and c5-c6.C4-C5 and C5-C6 showed mild bilateral facet arthropathy and ligamentum flavum thickening.C3-C4,C4-C5 and C5-C6 all show mild bilateral neural foraminal narrowing.Pain in right side of neck, shoulder, upper back and upper arm.Pain in collarbone area and deltoid when putting pressure on arm or sqeezing and picking object up.Pain in arm when extended straight out to the side. Doctor: I read your question and I understand your concern.That MRI shows degenerative changes of the spine with herniated discs at several levels which narrow the foramina, the passage for the nerve roots, causing compression of these nerve roots and accounting for your symptoms.Treatment consists in pain killers and physical therapy. If those don't work local injections with steroids and anesthetics are the next step. If still ineffective, surgery could be considered. For the last two alternatives though, since multiple levels are affected, it should be determined first which level is to be addressed meaning which is the most affected and causing the symptoms, if not clear by history, exam and MRI, nerve conduction studies might help.I hope to have been of help."
},
{
"id": 47837,
"tgt": "What could cause severe pain in left kidney while taking deep breathe, yawning and burping?",
"src": "Patient: Good speed!! I feel pain in my left kidney everytime i take deep breath,even yawning burping and sneezing everytime i do this i feel soo much pain, when i walk for like 10 minutes i feel pain..feel this pain a year ago but it didnt bother me this much, but now for 3 weeks i am feeling this..hope to hear from u soon..tahnks, jean Doctor: Hello Jean and welcome to HCM.As an Urologist,let me advise you, that with each deep breath, the kidney moves downwards, and upwards as you breathe out.If the kidney is affected by any inflammation or infection or swelling, the pain is felt and becomes more severe, if not treated.You must find out the cause for the kidney pain, by doing certain tests, like urine routine and culture, blood urea and creatinine, and an ultrasound scan of kidney. Get an Urologist to examine you.You may send the reports here, for an expert opinion,in my name. Dr.Matthew Mangat."
},
{
"id": 86371,
"tgt": "What causes abdominal pain with heart burn and black stool?",
"src": "Patient: I am 20 years old , I have abdominal pain for 2 years by time it's getting worse , I went to adoctor once and she said it's inflammation and gavebme medications but it didn't work and the pain getting worse , i have heart burn and some times black stool , also some times I have difficulties in breathing (it's not that bad) , I thought it's peptic ulcer but the pain isn't in that area , please tell me what's wrong. . I can't pay I have no money pleasr help me Doctor: Dr. Hanif warmly Welcomes you!Thanks for consulting at my virtual clinic. I have carefully worked through your case, and I can realize your health worries. Being your physician, I assure you not to worry as I will take care all of your medical concerns. It is definitely a gastroduodenal Ulcer and the black stools, heart burn, pains (no matter in which area of abdomen it is) all suggest that you need an immediate gastroscopy. Black stools are due to bleeding ulcer and you feel short of breath due to low hemoglobin.I preferably suggest you to consult a Gastroentrologist right away. He will go for a gastroscopy after careful physical examination and assessment. Also ask him to go for a Complete blood count to see your hemoglobin levels.The treatment will be according to the diagnosis. But for now, start with Omeprazole before breakfast, until you get an appointment for consultation.Hope I answered all of your health concerns . If you have further questions in future, feel free to contact me anytime. In the start of your question write my name, and I will get back to you in no time. If satisfied, please click you found it helpful.Have a wonderful time ahead. Best Regards!DR. MUHAMMAD M. HANIFUSA"
},
{
"id": 20844,
"tgt": "Can history of atrial fibrillation and strokes lead to cramps in legs and slurred speech?",
"src": "Patient: 60yo male history of atrial fib with 2 strokes from them does not take his medications..now sleeping more than half the day..falling down at times no slurring of words but seems to be getting much weaker has severe cramping in legs and chest at times...is this from the atrial fib and what can i do for him ..he was in hospital last year for 1 week to get his blood thinned out took the meds for 1 month then quit hasnt been back to dr since Doctor: Hello Thanks for posting at HCM. Atrial fibrillation means irregular contractions of the heart. Due to irregular contractions of the heart, clots tend to form in the heart which may later go towards the brain and other organs leading to stroke. Hence such patients are kept on blood thinners which prevent the clots from forming. These clots when they migrate to the brain can cause falls. Also since the heart rate is irregular, the heart may not receive blood supply for short intervals which can also lead to falls. As you said he has stopped hid blood thinner medications, this may be due to formation of clots. Cramping in the legs is less likely due to clots or blood thinners, but some heart patients are on diuretic medications like lasix which can cause electrolyte imbalance and result in cramps. Kidney problems can also lead to electrolyte imbalance and cramps.Calcium deficiency, Vit D deficiency are also important causes of cramps. Please do an ECG, serum electrolyte levels, serum calcium, Vit D ad 2 D Echo. Brain imaging (MRI + MRA) and carotid (neck artery) doppler should also be done to rule out narrowing of the vessels of the neck and the head which may cause falls.Wishing you good healthRegards"
},
{
"id": 97021,
"tgt": "What causes a fluid discharge from the hurt hand?",
"src": "Patient: I am 79 years old and had a bad fall while pushing a shopping cart down a ramp. My right elblow took a big hit and is now leaking a clear fluid,sometimes yellowish. I have been applying neosporin but the leak continues. What should I do?My height is 5ft11\",weight 190lbs and have type 2 diabetes. Doctor: Considering the symptoms narrated by you & other co-morbid conditions such as high age & type 2 diabetes it is advisable not to neglect the wound. As exact opinion regarding the wound care can be given only after the visual impression of wound, it is advisable to have a physicain visit who will promt you regarding the further managment whether you need oral antibiotics, drainage, or just local antibiotics will sufficient enough. The clear & yellowish discharge (which might be pus) coming out of the wound on the elbow may be indicative of infection within for which you might need to have oral antibiotics. Further more as there is no any information about your dibetic control its better not to take the chances and have consultaion which will clarify picturte. Also you may need to have tetanus toxoid shot if not taken in previous couple of years."
},
{
"id": 68958,
"tgt": "Suggest treatment for painful lump on back?",
"src": "Patient: My son has a nickle size knot near his hip flexor you have to push deep to feel it. He can't sleep at night because it hurts so bad. He also plays ball and can't run for the pain no matter what he does he can't get relief.Do you have any idea what this could be and what he might do to help it. Doctor: Hi. Thanks for your query.This can e lump originating from the muscle only as it is so deep.I would advise you to go for ultrasonography guided FNAC .This can given an idea what it is . As this is symptomatic , Surgical removal is the best answer.This does not look to be infective in origin ."
},
{
"id": 199330,
"tgt": "what could be the painless lump on the penis?",
"src": "Patient: I m 50+ and noticed, when I went to the toilet first thing in the morning, a bruise about 1.25cm in diameter over the back of the shaft of my penis, around mid-point, maybe slightly towards the base. No discomfort to start with. At night, I noticed a small paler area, may be slightly reddish, in the middle of the bruise. On feeling it, it appeared to be a tiny lump, about 2-3mm in size, completely within the very tin skin of the shaft - I can actually pick it up and roll it between my fingers. Since its appearance, it has got a very slight burning sensation. I have only one partner and have no urinary or other symptoms. Normal erection and no pain during sexual intercourse. I would be grateful for your opinion as to what might it be. Doctor: HelloThanks for query.Based on what you have described I would state that you have either staphylococcal infection of a hair follicle (Boil) or infected sebaceous cyst over shaft of the penis,You need to take broad spectrum antibiotic like Augmentin along with anti inflammatory drug like Diclofenac twice dailyfor a week .If does not subside with the treatment consult qualified General Surgeon for evaluation and further treatment.It may need to get drained or excised in Toto.Dr.Patil."
},
{
"id": 23018,
"tgt": "Suggest treatment for PVC with acid reflux",
"src": "Patient: After a stress test showed nothing significant--what is there to help with PVC s. Have had them off on on for many years but the last few days they have been awful and really interfering with normal activities. I do have a mild prolapse mitral valve and am bothered quite a bit with acid reflux lately (Endoscopy yesterday showed nothing significant) Although I m in my mid 60 s I m not taking any medications other than Prilosec & maalox at times. 5 6 129lbs (low blood sugar) IBS otherwise nothing serious soooo why do I feel so miserable at times---Can acid reflux aggravate irregular PVC s? Doctor: if PVC are troubling are troubling then you can ask your doctor to start you on beta blockers that will suppress PVC. otherwise not a matter of concern"
},
{
"id": 40323,
"tgt": "What is the remedy for ear infection with low grade fever?",
"src": "Patient: I'm a college student and usually when I'm sick and need to see a doctor go home I go to my family physician where my parents live. And this fall I had blood drawn for SED, thyroid, cholesterol, and the nurses usually do a good job. A good job as in the only thing I see after I pull off the band-aid is the little red dot of where they stuck in the needle. And I have smaller veins so the doctor's office I usually go to has the nurses already aware to always use a smaller needle when they go to draw my blood. Plus, they always give my arm time before injecting a needle to allow a proper vein show up. They do this by slapping my arm, using the elastic band, or whatever. \u00a0\u00a0\u00a0\u00a0\u00a0Yet come this past week since i'm a college student I live on campus, and I got sick with an ear infection slash small stomach virus. And I could not go home to my normal doctor due to having to many things due this past weekend, so I decided to use the clinic on campus that is for students. And they end up having trouble diagnosing me because I had a low grade fever, yet I had drainage in my left ear. So they decided to do blood count test or sed rate. \u00a0\u00a0\u00a0\u00a0\u00a0Like I mentioned I never have problems after getting blood work done. But this time being nervous in a different doctor's office then I\u2019m used to I notice the nurse was not taking her time at all making sure I had a proper vein. Yes she did use the elastic bandage, and I told her I had smaller veins. Yet she did not really check as much as most nurses I have had in past tend to do. And when she went to inject the needle I noticed that she did not inject it in the normal area under my elbow that I usually get blood drawn from and she did to the side of it. I was slightly excited at the time, because I never had my blood drawn that fast. Yet it left me with a feeling of dread. \u00a0\u00a0\u00a0\u00a0\u00a0And that dread was right, after hours of having the cotton swab and Band-Aid on the injection sight, I lifted it up to see redness and two bruises on and around the sight. They were different in size, one about the size quarter and one about the size of a half dollar. And I showed one of my best friends, but she paid no mind to it thought it was normal. But over the weekend I developed another bruised around a nickel size near the other two bruises. Plus, I notice pain and painful lump had developed (probably a hematoma) between the size of a quarter to a half dollar's circumference 1 \u00bd inch to 2 inches away from the injection site. And now the redness has gone away but the bruises are still there, and so is the lump that is making it painful to bend my left arm. So what could have happened to cause this? And what can I do to make it less painful and to heal? I saw posts about taking vitamin c and applying compresses to it. But seriously, I never had this happen before and usually the only thing after blood work is the little red dot from injection nothing else. Doctor: Hi,Welcome to HCM.I understand your concern regarding redness and swelling in the area where the needle was pierced. Such swelling usually occurs due to thrombophlebitis due to bleeding and inflammation of the vein wall. It is one of the common complications of blood sampling. I suggest you do some hot fomentation on the area for the swelling. You can take over the painkillers for the pain. Bruises will resolve within 7-10 days.Thanks."
},
{
"id": 107499,
"tgt": "Is Cortisone injection the right medication for severe upper back pain?",
"src": "Patient: I woke up Saturday morning with severe upper back pain,mostly radiating on the right down my arm. Pain was excruciating. I am 57 female. I. Was taken to hospital Sunday and gave me IV drip with pain meds and Valium to fill at pharmacy. Yesterday I received a cortisone injection by a orthopedic dr who put me on tormadol. Hospital says use cold compresses not hot to relief of pain. Any suggestions please.it all came from no where. No warning signs at all. YYYY@YYYY . Doctor: Dr. Hanif warmly welcomes you with thanks for a consult! I have carefully worked through your case, and can well realize your health worries. Being your physician, I want to assure, I will take care all of your medical concerns according to the latest medical guidelines.It is a muscular spasm and is due to the excessive stretching of the muscles and you will get better in a few more days once you get off of this spasm.Oral medications you are on are all fine and no extra medicines needed at this moment.I suggest you to apply Voltaren gel on the back and massage it gently. Physical therapy is also very effective for this pain. Your primary care physician will help you refer you for the same.Take a rest and keep a balanced healthy diet. Avoid weight lifting and excessive work or standing. Keep an appropriate posture when you sit in a chair. Keep the upper body straight up and your legs and upper body should be at ninety degree angle to the thighs when sitting in a chair. Hope I answered all of your questions. Please click on 'I Find This Answer Helpful' and don't forget to add your precious positive feedback to help me able to assist you better in future too.For any disease you have, a timely follow-up regarding the treatment efficacy and to see if there is any need to change the treatment options is very necessary. So keep a follow up to see how you doing. Remember to write down my name in the start of your question next time, & I will be with you right away. I am available 24/7 for providing you expert medical opinion on any health issue. Have a blessed time ahead.Regards!DR. MUHAMMAD HANIFUSAFOR DIRECT QUESTIONS: http://doctor.healthcaremagic.com/Funnel?page=askDoctorDirectly&docId=70199"
},
{
"id": 68690,
"tgt": "What causes a lump on the right side of the groin?",
"src": "Patient: Hi, I am female and I recently noticed a small lump in my right groin area. In the past couple of weeks I have noticed that it has become noticably enlarged on occasion (and visible from the surface) before reducing to its original size. It is pain less but I am not sure what it is and whether it needs to be checked. Doctor: welcome to Health care magic.1.The most commonly seen in this area are lymph nodal mass - they are enlarged due to presence of underlying infections and inflammations.anatomically it skin2.The second chance could be vascular dilatation ( in competent venous channel)3.The possible chances are hernia ( in terms of your explanation - no pain reduces on the size) little higher to the groin area.4. When you have the lumps, get an ultrasound lumps done - helps in identification of nature of lump, its origin and extensions.Hope it helps you. Wish you a good health.Anything to ask ? do not hesitate. Thank you."
},
{
"id": 191804,
"tgt": "What causes burning sensation and itchiness in the foot while on Insulin?",
"src": "Patient: my wife is diabetic, she also have ashma. prior to her knee replacement she suffer wound ulcer in her leg and took several months to heal before the surgery was performed. everything is okay now and her blood sugar is under control except for some occasional spike though she is taking insulin twice daily. The issue now is having burning foot pain, itchy and swelling find it hard to walk with out the aid of rollator. questions. how she can improve her blood circulation, prevent poor blood circulation, pain and neuropathy.another than these, her knee replacement is a success. she is 78 year old now, is her age a factor. Doctor: Hello, In a person with long standing diabetes, peripheral neuropathy does set in, particularly if glycemic control is poor. But even with good control also neuropathy does develop giving rise to burning pain in feet, tingling and even numbness. Itchy feeling is due to improper blood circulation and affection of venous return. Strict blood glucose control is a necessity. Drugs like Methylcobalamine and pragabalin can help in controlling pain in the feet. Strict deitary control, some amount of regular exercise along with glycemic control will help. Age is definitely a factor. Fasting blood glucose should be closer to 100 mg, PP 160 to 180 mg & HbA1c%<7. Thanks"
},
{
"id": 157817,
"tgt": "Have lesions in liver, possible kidney stones. Done CT scan. Is it cancer?",
"src": "Patient: i just learned I hv 6 lesions on lower rt lobe of my Liver. I hv been w/American Airlines for 21 yrs,and they cancelled my insurance. I do not qualify for any programs,as they say I make to much money. (Us flight Attendents) just rake it in I TELL you. ;) ANYWAY, the things my Family & Friends hv sugested or came up with, oh God bless them all! Today someone said Milk Thistle. Which I know nothing about. What is Your take on that? Coarse i still need test done and find out if cancer. But noone will see me when they find out no insurance & no pay check since Sept 2012. Im scared for my mortality. Im 46 f trim n fit 3 children & 11 th Grandchild on the way. Yes, I started young! ;) but this Nana rocks n can still do anything they can or want n keep up too. ( on good days) any adviceat all? I dont know where to goor who to talk to at this point. All I have and keep hearing is \" Sorry,I cant help You\" then who can? I actually found out May 15 via CT Scan,as the ER thought I had Kidney stones, no stones,BUT. Doctor: HIThank for asking to HCMJust take it easy, no need to worry and do not think about the big disease, if you do not have any sign and symptoms regarding your kidney and liver then why to worry, kidney stones are easy to treat nowadays, here you have not gave your complete medical history, for proper advise we need complete medical history, please be in touch with HCM in future with detail health profile have nice day."
},
{
"id": 61369,
"tgt": "What causes a painful lump on the vulva?",
"src": "Patient: Question, only just the other day when I was using the restroom and i was cleaning up, i noticed a sharp pain. Upon further investigation I discovered a very firm bump at the back of my vulva that was very tender to the touch (irritated from presumably wiping after the toilet). The placement is to the very far back left side, close to my anus, on the inside of the lip. It is very tender and I m worried it is being aggravated when I use the rest room. Additionally my body has experienced sever pain near my lower abdominal, night sweats, warm skin to the touch (not quite a fever) headaches and chills. When I investigated with a mirror the bump looks like it s a white ish color but there is only one, and it is the size of a pea. Havent had sex in over 3 months, this only started 48 hours ago. I m an avid gym goer and periodically have to run erronds after working out in my gym clothes. Have to wait until morning to call for an appointment but any ideas what it could be? Doctor: Respected user , HiThanks for using Healthcaremagic.comI have evaluated your query thoroughly .* This seems most likely bartholin cyst , vulva abscess or perineal abscess * Require sound clinical examination to confirm the diagnosis and further line of management .Hope this clears your query .Welcome for further assistance .Regards ."
},
{
"id": 197814,
"tgt": "What does black patching of skin on penis indicate?",
"src": "Patient: Hello. I haven t noticed this before, but there are these black patching of skin between the glans and foreskin of the penis; in that empty, circular area. I have recently been able to peel the patching off, but I am a bit concerned. Is this normal? Doctor: HelloThis might not be normal, please uplaod a photograph and I would like further details likeany history of unprotected sex in recent past ?is there any discharge coming out of this?your age and relevant medical history like diabetes or urogenital disease?meanwhile maintain good hygiene, wash it with antiseptic soap or solution and keep it clean and moist.you may require a course of antifungal or antibacterial under supervision.please write back to me with these details so thta I will be able to tell you more accuratelybest wishes"
},
{
"id": 66902,
"tgt": "Suggest treatment for lump on wrist of a child",
"src": "Patient: i have a 23 month old daughter that has a large lump on her right wrist im assuming its a cyst. I wasn t alarmed at first because it was small but ive noticed it has gotten larger. My daughter can determine a bo bo and she acts as if it doesn t bother her. Im making her a appointment but should I be worried and could this be cancerous? Doctor: not to worry much as it is likely a ganglion cyst or lipoma and easy to treatit cannot be cancerous! FNAC test can confirm it.all the best!"
},
{
"id": 19389,
"tgt": "What is meant by bilateral grade 2 varicoceles ?",
"src": "Patient: i was undertaken a doppler test to check whether i have vericocoeles. after the test the impression was color doppler study is suggestive of bilateral grade 2 varicocoeles.. what does it mean what is its remedy as i have no children after two years of married life? Doctor: welcome to hcm ..brother the varicoceles are usuall cause of infertility as it decrease sperm quality ..please go for surgery ..bilateral vaeicoceles means veins are abnormally dilated both sides .."
},
{
"id": 43650,
"tgt": "Sperm analysis test done. Prescribed Oligocare, Maxzoa L. Will it help in increasing sperm count, motility?",
"src": "Patient: I have gone through sperm analysis test 2 months ago, and result was, sperm count- 5 million /cumm, motile sperms - occassionally present. Based on the result, doctor prescribed me 3 months course of oligocare and Maxzoa L. Please let me know will these medicines help me to increase my sperm count and motility? Thanks Rajesh, Doctor: Hello, welcome to HCM, I am Dr. Das Look, your semen analysis report indicates that sperm count is very low ( normal is >15 millions /ml as per WHO 2010). Sperm motility is also very poor. So, you are suffering from oligoasthenospermia. Oligocare and Maxzoa is really very helpful. So, just continue the medication. Regards."
},
{
"id": 59611,
"tgt": "Have high SGOT and SGPT level. Reason?",
"src": "Patient: Sir, I am male 44. My Glucose fasting is 115 mg/dl. SGOT (AST) is 91 U/l, SGPT ( ALT ) is 160 U/l. Are my SGOT & SGPT at genger level? Should I immediately start medication? What are the reasons and adverse effects of increased SGOT/SGPT. Can these be controlled by controlling the diet? Should I get an lever ultrasound? Am I be heading towards jaundice? Kindly advice other preventive measures to control them. Doctor: Hello, scsaini5294, Fasting blood sugar of 115 mg/dl is mildly elevated blood sugar that might indicate early diabetes. You should have HbA1C tested and also a 2-hr post meal blood sugar test to confirm the diagnosis. Your enzyme levels are elevated. If you are above your average weight for your height , this could all be Fatty liver related. After you get a Hepatitis screening tests for Type-B & Type-C to rule these out , then you can get a sonogram of the liver to see if you have fatty liver and it's grading. If you also have elevated lipid levels, you need to control that by dietary means and may be medications. You should be on a diabetic diet , cutting back on concentrated sugars. I also recommend that you start taking Vit-E 800 i.u. daily, along with Vit-C 1000 mg. daily, and also a Super B-Complex Vit. tab daily. Fatty liver is reversible if taken care of before cirrhosis sets in. I wish you good luck and good health."
},
{
"id": 47223,
"tgt": "Is transplantation advisable for nephropathy?",
"src": "Patient: my brother has been diagnosed with IgA nephropathy and he is 19 years old, his BP is 140/80 and is currently prescribed by Amlogard 5mg, arkamin and omnacortil 40mg with fish oil. Is transplantation advisable for him as his creatinine is 5.9 and blood urea is 182. Doctor: HelloThanks for query .Your younger brother who is just 19 years old has been diagnosed to have IgA Nephropathy with his serum creatinine being 5.9 IgA Nephropathy is a progressive disease that progresses very fast and 10 years survival rate is reported to be less than 60 % .And more over results of the medical treatment are not encouraging .In view of his age being just 19 years I would suggest you to get Renal Transplant done . Dr.Patil."
},
{
"id": 180927,
"tgt": "What causes unusual feeling and discomfort after tooth extractions?",
"src": "Patient: I had about 20 extractions and dentures put in about 6 months ago and never really got comfortable with them and the adhesive. Could the fact that I was grinding and fidgeting with them made it possible for the inner skin inside to peel off inside of my body as I have had feelings of string or rubber wrapping around my head and body sometimes it feels like my ear is actually pinned backwards but when I look I m the mirror it is fine. I went to the doctor and he said it was neurological so I went to the emergency room and get MRI and am awaitiny appointment with him. Have you ever heard of anything like this? Doctor: Hi..Thanks for the query..Well, if you have a habit of grinding teeth then the denture will unfortunately not set well causing difficulty in adaptation of the dentures..It can continuously dislodge he denture and making it difficult for you to wear them and use efficiently..However the denture will not cause any peeling of skin inside body or the other symptoms of string wrapped around the head..Yes, you are advised correctly that the symptoms can be neurological and even the grinding of teeth can be related to it..So my suggestion is to get your MRI evaluated..Also for teeth grinding habit you can do meditation and reduce stress..Also consult a Prosthodontist for manipulation of your denture..Hope this helps..Regards."
},
{
"id": 72319,
"tgt": "What causes bleeding while you cough?",
"src": "Patient: HelloMy girlfriend had cancer earlier, the doctors say thats its gone now, but she coughs up blood from time to time, but everyday. to stop the pain she smokes a cigarette each time at evenings. is that okey for her or could she get worse? Doctor: Thanks for your question on Healthcare Magic.I can understand your concern. Your girlfriend is having hemoptysis (blood in sputum).And it is not good sign. Hemoptysis is commonly seen with bronchitis, lung infection (pneumonia) and lung cancer.Since she is cancer survivor, we should definitely rule out lung cancer in her case. So better to consult pulmonologist and get done chest x ray and CT thorax to rule out lung cancer. Hope I have solved your query. I will be happy to help you further. Wish you good health. Thanks."
},
{
"id": 215671,
"tgt": "What causes muscle soreness after prolonged standing or sitting?",
"src": "Patient: Hi, I have tough physical job that I have been doing for over 20years, I often wake up in the morning with sore muscle in the morning. However I am now finding that if I sit down for 20 minutes that sometimes they are already stiff and sore. What can I do to help with this? Doctor: Hello, It is just a muscle cramp arising after prolonged standing. As a first line management, you can take analgesics like Paracetamol or Aceclofenac for pain relief. Nothing much to worry as the symptoms will settle in a couple of days. Hope I have answered your query. Let me know if I can assist you further. Take care Regards, Dr Shinas Hussain, General & Family Physician"
},
{
"id": 145364,
"tgt": "How long does brain take to heal after drug de-addiction?",
"src": "Patient: thank you my son will be 22 on wed and he s recovering from drug abuse or the past 7 years sadly and i want to know can his brain heal totally? He is under a dr care and is on subtitex and will ween soon. Will this be long enough to stop his craving and realign his brain. I read males brine continue to grow and heal and regenerate unto 30. Is this a true fact. Also does weed damage their brain as well and does it cause anxiety as well as bad memory Doctor: Hello. I have been through your question and understand your concern.Weed has several side effects like memory loss and anxiety but there is no report on brain damage. Craving instead is difficult but not impossible to recover from.You should consider your son as abusive to drugs who is not really sick. Brain has several mechanisms to recover from certain attitudes like addiction to drugs, but in this case there is no physucal dependency, just psycological.So you should also consider psycological approach which has the best results.Hope this helps.Please feel free using MHC again"
},
{
"id": 159068,
"tgt": "Have breast cancer. CT Scan shows hepatic cyst. Recommendations?",
"src": "Patient: hello, my aunt was diagnosed of breast CA stage 3c and undergo a series of chemo , we are now doing her work up laboratories..my concern was the result of her abdominal CT- Scan. the impression was 1. Hepatic cyst 2. parenchymal calcification 3. atrophic uterus with heterogenous enhancement and calcification 4. vacuum disk phenomenon Doctor: Hi, You should have mentioned the exact management protocol that has been selected for her. Whether surgery and radiotherapy has been completed or not. Anyway , hepatic cyst is not due to malignant deposit, this is most probably a benign condition. Uterus pathology is not related to breast cancer at all. Consult your gynaecologist."
},
{
"id": 55699,
"tgt": "Can using andro gel cause liver cirrhosis?",
"src": "Patient: Ok here is my question I currently have cirrhosis of the liver and for a good part of my life have been on andro gel I have looked into andro gels website now my cirrhosis is at stage 4 level b is it possible for the gel to be the cause of my cirrhosis bear in mind I am 33 and drink once in a blue moon Doctor: Hi, dearI have gone through your question. I can understand your concern.Androgel generally doesn't cause liver cirrhosis. Your cirrhosis may be due to some other reason. You should go for ultrasound abdomen and liver biopsy if needed. Consult your doctor and plan accordingly.Hope I have answered your question, if you have any doubts then contact me at bit.ly/Drsanghvihardik, I will be happy to answer you.Thanks for using health care magic.Wish you a very good health."
},
{
"id": 50465,
"tgt": "Bleeding after intercourse. Ultrasound showing a calculus in kidney, cortical cysts in ovaries. Treatment?",
"src": "Patient: i got married on 1st feb. ,during first month of my marriage i didnt had any problem during sex, but frm second month i see little blood during our intercourse. after examination the doc said thr is a small mole/poly or anything.so the doc advised tat it should be romed immediately.I havent understood the reason for this and also in ultra sound report its seen tat a calculus is seen in mird 3rd of right kidney measuring 3.3 mm in size, thr are features of endocervictis, small cortical cyst is seen in both the ovaries and minimal collection is seen in POD? what should i do now, what are the remedies? will i have any problem in conceiving a child???pls help?? Doctor: Hi,Thank you for your query.You have not mentioned the location of the mole/polyp. If it is on the cervix then it should be removed. Endocervicitis can be controlled by appropriate medications.The kidney problems should not be a cause for worry if the kidney functions are within normal limits. The doctor shall be seeing to this aspect.Cortical cyst in ovaries and minimal collection of fluid in the POD are normal. These should not cause any problem for you to conceive. However you need to be under constant medical supervision for your renal problems.I hope I have answered your query to your satisfaction.Wishing you all the best.Regards,Rajiv K Khandelwalhttp://goo.gl/SuCjl"
},
{
"id": 179731,
"tgt": "Suggest cause & complications of spinal fluids in infants",
"src": "Patient: Hi my name is Jennifer... Our family is going thru a rough time right now my nephew was involved in a car accident , was hit my a drunk driver he ran a red light but anyways now my nephew doesn t feel his hand and now was transferred to a childrends hospital now there thinking it might be a leak in his spinal between his neck and shoulders , my question is what happens when there is a LEAK IN THE SPINAL FLUIDS?? what are the damages and how can this effect him???? now his a baby he just turn 1yrs Doctor: HIWell come to HCMLeak in the spinal cord and the history of accident means this could be fracture in spinal column in my opinion the whole spinal column need to be scanned and the best choice would be MRI study, hope this information helps."
},
{
"id": 220007,
"tgt": "What are the early signs and symptoms of pregnancy?",
"src": "Patient: Hello Doctor,I got my period delayed by 3 days. From last few days, I am getting temperature high in a certain period of the day and after sometime it recedes. I feel heavy inside, even getting headache. Unable to sit in a certain position,Food craving is there. So am I pregnant? Doctor: you should visit your gynecologist and get blood pregnancy test done. that will give you a definite diagnosis of pregnacy. you should be examined what is causing the fever."
},
{
"id": 99841,
"tgt": "What are the symptoms of heat allergy?",
"src": "Patient: When I am outside, or inside exercising, I get really hot and my knees turn a red blotchy color. It feels fairly warm to the touch. It stays on my knees for about 30 minutes to an hour. This only occurs when my body temperature rises. Does this mean I am allergic to heat? Doctor: Hello,Thank you for asking at HCM.I went through your history and would like to make suggestions for you as follows:1. It is not exactly \"allergy\" to heat. I would think of a condition known as \"cholinergic urticaria\".2. It is a simple benign condition when increase in body temperature, either due to exercise, heat exposure, fever, etc causes red/pink itchy lesions over skin (any part of body), which usually subside by themselves.3. To control itching, I usually suggest my such patients use of antihistamines such as levocetirizine or cetirizine. 4. Please do not scratch the itchy areas as it would aggravate itching.Hope above suggestions will be helpful to you.Should you have any further query, please feel free to ask at HCM.Wish you the best of the health ahead.Thank you & Regards."
},
{
"id": 216578,
"tgt": "Suggest treatment for sever pain in thigh",
"src": "Patient: I was woken up in the middle of the night with a severely painful thigh which is causing me difficulty in walking and moving generally. As it is Sunday it would be difficult to obtain a Doctor s appointment. Not sure what to do. My name is Sandy Bell. Please reply to my partner s email address below. Doctor: Thank you very much for contacting healthcare magic. You have problem of severe thigh pain that may be because of diabetic neuropathy. Check your blood sugar level.Any injury to thigh may cause thigh pain.Take one x ray of thigh .Any bony lesion may cause severe pain at thigh.X ray give exact idea. Anemia may cause sever pain. Check your blood hemoglobin level also.Consent a physician for further diagnosis. Proper physical examination and history give exact idea. I hope my guidance is helpful to you. .Take care.Thank you very much."
},
{
"id": 179838,
"tgt": "Is swelling on head after an injury a cause for concern?",
"src": "Patient: Hello, my 13 month old grand child fell from the couch and hit her head on the wooden coffee table, she has a red mark about an inch on the side of he head, there is a little swelling but not much, she is acting fine, not crying or fussy, she doesn t seem drowsey, and no vomiting at all. should we still be concerned about taking her in to see a doctor, and also when can she go down for a nap Doctor: If there was no unconsciousness and no vomiting, after injury, no need to worry....yet you should consult your doctor once as soon as possible .."
},
{
"id": 167325,
"tgt": "Does wearing diaper have any effect on baby penis growth?",
"src": "Patient: does wearing diaper have any effect on baby penis growth my baby is 9 months old he wears diaper for whole day and at night for 5 to 6 hours i keep him without diaper when he is sleeping in the morning i pack him again does it have any effect on baby penis growth i m really conserned and would be very thankful to you for answering me Doctor: No it does not effect the growth of penis. however the diaper should be of proper size so that your baby feels it comfortable. The only thing you have to worry about is rashes. The baby might get rashes and seconday infection due to cobstant moistness and the ammonia present in babies urine. There has not been any proper mass research on this issue and in my career i havent seen any thing like that or geard about that. As far as theory is concern you dont have to worry about it. Just enjoy the childhood and motherhood."
},
{
"id": 74971,
"tgt": "What causes ESR fluctuation after treated TB?",
"src": "Patient: hi i am vani rao 26 year old lady, got married on two year ago, but last year on sep 2010 i got TB, and from sep 2010 i have been AKT-4 as per doctor prescription, at apollo new delhi, now i got my blood test done yesterday 21 june 2011, but ESR is now increasing 23 mm/hr (0-20normal range), what would be cause, still i am taking medicine. AKT-4 Doctor: HelloThanks for posting here. Tuberculosis usually causes an increase in ESR, so i suppose you had high ESR when you were diagnosed. Noe since you are on AKT 4 for more than 6 months, your ESR is coming down. an ESR of 23 is considered as normal. Please do not worry about that value. A value above 50 would be concerning. More important thing here is since you have completed 6 months on AKT-4, we have to evaluate whether you are cured from TB. I would recommend a sputum gene Xpert test, chest x ray and AFB smear. wishing you good health. regards."
},
{
"id": 89146,
"tgt": "Suggest treatment for sharp abdominal and groin pain",
"src": "Patient: I m a Female, 25yrs old. About a week ago, I had to hold urine for about 4hours during hospital rounds..The urgency went away then returned suddenly, I had an accident trying to find a bathroom.During voiding, I felt a sharp supra-pubic pain. Since then, every time my bladder is fulling to full, I feel sharp pain in lower abdomen and groin area. It has gradually worsened over the past week. Some times I don t feel the urge to void, I feel the pain first before the urge. When I have voided the pain reduces, but there s persistent dull pain in the lower abdomen that lingers for a few hours. Feels like bloating. I do not have increased frequency or urgency, my urine is normal. During voiding, some small amounts of retained urine comes out in a palsating manner after the main stream is over, about 1-2times. when I strain to push the rest out I feel severe sharp pain in my groin. Could it be a UTI? I have no chronic medical conditions. Doctor: Hello,The symptoms are probably related to the over stretching of the bladder walls and minor trauma to the urethra by your trying to hold the urine and then the sudden let go. It would disappear after a few days once your system has repaired itself.You can also get a urine examination done if you are worried. It would show if you have a urinary infection.Wish you good health.Dr. Noble Zachariah"
},
{
"id": 63241,
"tgt": "What causes a lump in the groin?",
"src": "Patient: i have been to 7 doctors i have had a ct scan and x rays i have a 1.0 cm lump in the right groin the ulta sound states papable abnormality and a small benin apperaing lymph node they sent me to a sport medcine doctor and he doesnt know what it is and said to get an mri and go to a otheropedic oncology dotor i called them and they cannot help without a diag , this injury was notice after a bike injury . its painfull in some ways i think its and infection or a blood colt a hernia was ruled out but everyone has a different answer i just want the pain to stop and the lump to go away , and dont know what doctor can help the primary hasnt the gyn hasnt and the sport doctor hasnt , what can i do Doctor: Hi, dearI have gone through your question. I can understand your concern. You may have some enlarged inguinal lymphnode. It can be due to reactive hyperplasia as a result of infection or lymphoma. You should go for fine needle aspiration cytology or biopsy of that lump. It will give you exact diagnosis. Then you should take treatment accordingly. Hope I have answered your question, if you have doubt then I will be happy to answer. Thanks for using health care magic. Wish you a very good health."
},
{
"id": 36111,
"tgt": "What causes pain and clear discharge in belly button?",
"src": "Patient: My belly button has been hurting & when I went to unbotton my pants due to pain, it was bloody & then oozing clear. Is it an umbilical hernia? Or just infection? I also currently have a bladder infection which I get all the time due to institial cystitis? Doctor: Thanks for asking in Healthcaremagic ForumIn Short: Umbilical granuloma/oomphalitis can cause thisExplanation: Bloody discharge from umbilicus points towards infection or infected granuloma. Sometimes persistent urachus(persistent connection between bladder and umbilicus in foetal period) can cause frequent discharge from umbilicus. So, visit a surgeon to get yourself checked for this. Hope you have contacted doctor for your cystitis to get treated. Good luck."
},
{
"id": 219237,
"tgt": "Can Susten, Ecosprin and Lonopin be taken together?",
"src": "Patient: Hi, my wife had 2 miscarriages in 2yrs now she Is pregnant again our doc has already given Her four doses of PTL before conceiving n now she has prescribed susten 200mg vignally, duphsotan both thrice a day n ecosprin tab with lonopin 300mg 0.4cc injection n three doses of h -cort injection will all this have any side effect on my wife or baby plz suggest Doctor: HI.No no, it will not. These medications will help to sustain the pregnancy and support it. These are all medications that are mandatory right now in helping to avoid/prevent a miscarriage.Please do not even think twice before taking any of the above mentioned medications, as the consequences of not taking these medications can cause real problems.Best wishes."
},
{
"id": 189084,
"tgt": "Front teeth big and projecting. Advised not to remove healthy teeth to fix clip. Suggest",
"src": "Patient: My name is tejaswi.my age is 23. My front teeth are big and projecting slightly.I consulted a doctor who advised that as there is no gap between the teeth two teeth from the lower jaw and two teeth from upper jaw will have to be removed to fix the clip.our family doctor advised that as the nerves of the teeth are interconnected with various organs like eye,brain,ear etc.., it is not advisable to remove the healthy teeth.It may carry over to digestive and other problems later.Please suggest me. Doctor: Dear friend.Thanks for sharing your concern.First of all i would like to explain you that the nerves of the teeth are not interconnected to any other organ like eye brain etc..extraction would not affect these organs for sure.Every small part in our body has its own muscle and nerve supply,which does specific function related to that organ.Now coming to your dentist advise,i think there is some misunderstanding regarding teeth extraction.He or she would have said that, you require four teeth extraction as there is no space in the jaw to move the proclined teeth.Secondly remember it is premolars(back teeth)which is usually extracted never the front teeth.Here the best suggestion that i can give you is please revisit our dentist and understand complete treatment procedure before taking any decision in haste.Extraction is required,for how many teeth, can be decided only upon taking an x ray called cephalogram,And IOPA x rays.Hope i have answered your query.for further query please let me know.Thanks.Take care."
},
{
"id": 159686,
"tgt": "Excessive flatulence, weight gain, low platelets and WBC after rectal cancer. Have scar and fatty tissue. What diet is advisable?",
"src": "Patient: After rectal cancer,good blood test and CEA,just a little low platelets and WBC . Oncologist not worried.He told me to lose weight . I have some scar and fatty liver .But stressed and moving in a new place,I had gained...Everything makes me pass gas. so very often ..there is a pressure on the anus,very umpleasant..when passing//No blood in the stool.O.k. diet and exercise,diet with what?Soup?No fruits?What fruits are O.K?A hernia developed and pressure from it..I took Metamucil more rarely because I enjoyed whole wheat bread and lots of gas and gained weight.Cottage cheese low fat and soup o.K.?K. Doctor: Dear K, I am so glad to hear that your rectal cancer is in remission. Low platelet count could be the result of some form of chemotherapy you might have received after surgery. I am sure your oncologist will be monitoring the blood tests. As for excessive gas, there are several reasons. Some people ingest a lot of air like during smoking or during chewing a lot of gum.Intestinal gas is also produced from firmentation of certain foods. If you are lactose intolerant (Milk,Dairy) you can try switching to lactose free milk or lactaid added milk. Eating too many fruits can also cause excessive gas. There is a product called \"Beano\" which does not require prescription. You can try taking that once a day.Essentially cutting back on fatty foods and have more protein rich foods and also limiting sweets combined with moderate excercise would help you lose weight. Good health,"
},
{
"id": 216950,
"tgt": "What is the treatment for pain in the thigh?",
"src": "Patient: When getting up on my legs, the pain down the back of my right legs is so bad it causing me not to be able to put pressure on my foot to walk. its like a pain from my thigh down my leg. pain is in the back of my knee. also when standing it feels like muscle tearing in my thigh on the left leg. Doctor: Hi there, thanks for your question at HCM .Pain radiating along your back of thigh and legs, suggests pinching of a nerve in your spine. Since its present mainly on movements and you are painfree at rest it suggests a dynamic stenosis. If its affecting your daily routine, I suggest you to get an X Ray if your spine at first. Most vlikly your pain wil respond well to pain killer medicines. physiotherapy protocol including core stabilisation and muscle strengthening. If you still persists to have pain 6 weeks into therapy , it would be prudent to do further tests whcih will include an MRI. Hope this helps.All the best. Regards.Dr.SBK"
},
{
"id": 111150,
"tgt": "Suggest treatment for lower back pain",
"src": "Patient: I have been experiencing a lot of low back pain... for starters if I stand up wrong or sit too long and go to stand my low low back, almost butt area, will lock up on the left side and it causes me to fal. I am 25 and I have beared 3 children. I am not overweight. Second about 5 hours into sleep every night I will wake up due to what feels like tight compression around my spine in my middle back, causing it to be very hard to breathe or even move. I can void sometimes but it doesn't help the pain, after about an hour or so of being up and moving the pain will subside a little. Do you know anything that can cause these symptoms? My x ray and mri of lower lumbar showed normal but obviously its not. Doctor: Hi,At this age there might not be having any problem with spines.But due to 3 pregnancies there might be some with curvature of spine giving rise this problem.There might be having some effect on lumbar lordosis giving this problem.Do physical exercise like back extension exercise.There might be having some calcium and vitamin A and D vitamins due to 3 deliveries.Take calcium, vitamin A and D supplements.Avoid segentary life.Ok and take care."
},
{
"id": 106523,
"tgt": "How can chronic lumbar back pain be treated?",
"src": "Patient: Sir I have a long history of chronic lumbar back pain, D.D.D., Stenosis this morning on my lower left side just above caudal equi area i am having horrific pain I take morphine sulf. 200 mg 2x daily and are having no effect I am 2 hours any from my primary and have tried heating pad ice nothing is helping???? Doctor: Hi, Since your pain is not relieved by strong analgesics possible cause may be 1. mechanical backache and 2. instability. Detailed investigation in form of 1. X-ray of lumbo sacral spine anteroposterior and lateral views and 2. MRI of the lumbo sacral spine with screening of whole spine needs to be done. Please get it done from radiology center nearby you. Meanwhile, start tab. Ultracet. Visit to orthopedic surgeon for detailed clinical Examination is recommended. Hope I have answered your query. Let me know if I can assist you further."
},
{
"id": 175000,
"tgt": "Could foul smelling stools with undigested food due to malabsorption in a 17 month old?",
"src": "Patient: My 17 month old son had GERD and several food intolerances that we are exploring with the help of a naturopath (I m in Canada, so we ve been on a waiting list to see a pediatrician) - we believe Dairy is a major factor, his naturopath also suggested we pull all refined sugar out of his diet and only sweeten his food with actual fruit; additionally we see ongoing stomach pain (twisting torso, screaming in short bursts, still cannot lay flat on his back, is only calmed in a warm bath or when I rub his tummy) on a regular basis. While the one test we ve done thus far doesn t indicate the following foods, we suspect oats, eggs, nuts are also still contributors to his ongoing discomfort. Joey is not gaining in weight or length significantly - certainly not since I weaned him at 13 months old. He still looks small in the same pajamas he wore at 8 months old, and he still weighs only around 20lbs. I finally got my husband to look at his poop last night and he s extremely stressed as Joey (my son) poops 4-5 times per day some days, the stools are always very foul smelling, they always contain these funny white granules and large amounts of undigested food. Is it possible we re not looking in the right direction? Our physicians seem to follow our lead in exploring what s bothering my son, so no testing has been done on him to my ongoing dismay. Is it possible there s some sort of malabsorption issue and not all these allergies/intolerances? Doctor: Thank you glfor following up, your baby has malabsorbtion,caused by dysbiosis of intestine. It is really pity that you don't check your baby . I suggest you should give next baby's analysis:- stool culture for flora and sensitivity to a tibiotics- stool analysis-stool for ova and cyst-blood analysis- urine analysis I would suggest to give your baby next treatment:1. For prophylaxis or for treatment-albendazol(zentel)10 ml once 3days in bed time2.Probiotic: Econorm(saccharomyces)1 sashe 2 times 3days3.Probiotics: Enterogermina 1 sashe 2times 10 days,then bifilac(darolac)1 capsule 1 time during 1 month4.Enzyme syrup during 1 month for good abbsorbtion of food .5.If your child were not allergic to vitamins i would prescribe him zinkovit 5 ml 1 time 1 month I will be in touch to follow his progress. Your sincerely, Dr.Svetlana Shrivastva"
},
{
"id": 211799,
"tgt": "Mentally weak since birth, violent action, has severe headache. On frisium, glycocate, olimelt. Sufficient?",
"src": "Patient: Sir my brother is mentally weak by birth and few days before he experience a mental problem in which he acts very violently at that time he is admitted in meerut hospital \"MIMHANS\" under the observation of an specialist neurologistand now he is discharged from the hospital but he is not well at present;the time he is given a medicine (Medicine name -Zostum,Encorate, Olimelt,Lonazep, Glycolate,Protonil & Frisium) he sleep and after few hours when he is consious he is wipping , suffering from headache . plz help me Doctor: hi, i would like to know the dosages of these meds, also whether he has seizure disorder. many of the intellectually deficient patients do suffer from behavioral disorders, for which medications are necessary to prevent harm to self or other.now adjusting these meds is a difficult process for the patient as well as the family, as these indivisuals find it difficult to verbalize their symtoms. in such case it is best to admit and adjust the dosages under psychiatric care.hope this helps."
},
{
"id": 16783,
"tgt": "Is hypothyroidism expected if having high triglycerides and cholesterol levels?",
"src": "Patient: dear doc I had high tryglycerides 340 and high bp 110 /190 and totalcholesterol 260 I was blood tested and I am under medication stil I have the heaviness in chest with a light headache its 8 days now iam on medicine.Coul I have hypothyroidism also? Doctor: Hello There After going through your medical query I understand your concern and I would like to tell you that high cholesterol levels depends on your lifestyle and genetics , but high cholesterol level doesn\u2019t mean hypothyroidism should be there. You can get you TSH levels done.For chest heaviness it is recommended for you to get an ecg and stress echo done and consult a physician with reports for further management.Hopefully this information will guide you properly.Kind Regards Dr Bhanu Partap"
},
{
"id": 122158,
"tgt": "What causes a wet water like sensation on the left of torso?",
"src": "Patient: I am a 44 yr old male and for the past two days I have experienced a rather odd occurance. I have been experiencing a wet water like censation where my left leg attaches to my torso. It last for just a second or two. Not painful but odd non the less. It feels almost like I have spilled something Doctor: Hello, Your symptoms could be related to a pinched nerve. For this reason, I recommend performing nerve conduction studies in order to investigate for this possible cause. Hope I have answered your query. Let me know if I can assist you further. Regards, Dr. Ilir Sharka, Cardiologist"
},
{
"id": 93383,
"tgt": "Abdominal pain, continuous bleeding, prescribed Esylate. Gestational sac without fetal node. Suggestion?",
"src": "Patient: Good evening Dr. Gestational sac without fetal node found in the right overy region. Patient is suffering from mild severe pain in left abdomenal region and bleeding is continous. Consulting doctor sugested to observe 10 more days during which patient will take E-SYLATE-500 ALONG WITH Ferrous Ascorbate & Folic Acid Tablets. What is your kind suggestion ? Doctor: Hi ! Esylate tablets are given to you to control you bleeding. Empty gestational sac may mean a miscarriage, but not always so. Your ultrasound might have been done too early to note any fetal part. Your doctor might have advised you to come after ten days, when she can examine you again, and may go for a repeat ultrasound. If a repeat ultrasound also shows an empty gestational sac, then one can conclude a miscarriage, along with a declining serum HCG level which might have done at the first instance.Wishing you an early recovery. Take care."
},
{
"id": 187265,
"tgt": "What are the side effects of getting teeth scaled?",
"src": "Patient: I am scared. The dentist said to have my teeth cleaned and insisted they be scaled. I have never had this done before. I hear that scaling hurts a lot. I read that two studies had shown that scaling was of no use, was intended only to fatten the dentist's pocketbook. The dentist's assistant called to say she would have to scale my teeth because just cleaning them would be too dangerous. So I said to cancel the appointment. What should I do? Doctor: Hello, Thanks for your query.No, none of these procedures cause any discomfort. If you feel any sensitivity or unpleasant sensation on your teeth, inform the dentist.Bleeding may occur during scaling if the gums are already inflamed. However, it is transient and stops soon after the procedure. You may not even know that your gums are bleeding. There are no side effects associated with scaling and polishing. In fact, it should be made a practice to do it once in six months. You will feel the difference soon after the completion of the procedure.I do hope that you have found something helpful and I will be glad to answer any further query.Take care"
},
{
"id": 108459,
"tgt": "What causes back pain, tiredness and hand discomfort?",
"src": "Patient: Hello Doctor. Am 24 yrs old. Got married and having a kid 2 yrs old. Mine ws a caesarean. My blood group is 'B Negative' and my haemoglobin count ws '7' whn i checked before 1 yr. Now on days am feeling very tired when i wash clothes or clean the house. when ever i wash clothes am getting back pain for almost 2-3 days. my height is 159 cms ans weight is 65 kgs. Am unable to do any kind of work giving pressure to hands continuously for 5 mins. Can u please suggest me what 've to do ? Doctor: your hemoglobin level is very low and that can cause all type of troubles that you are facing. its a must to get your HB level checked and take appropriate treatment. Also knowing the cause of low Hb is a must. moreover your symptoms will be cured once your Hb is improved."
},
{
"id": 119175,
"tgt": "Low platelet count. Readings show 299. Work out thrice a week. Have further testing done?",
"src": "Patient: I had a low platelet count 78, had mild headache for three days felt weakness for a few days. The following day of the blood sample I reapeted the test in an other lab. They took two different samples one was the regular sample red cap vial and a second was taken on a blue cap vial. The lab technician response was that some people that usually get low platelet results; sometimes have some type of reaction to the substance on the regular vial. She mentioned that she will double check with an extra blood sample taken with blue cap vial that has another substances. She uses a multipler factor to correlate results, readings were higher 299. I am concern with this because this is the second time it happend to me, last year I went through a similar situation. However I do not remember having low platelet in yearly teste in previous yearly examination. I am 62 years jog 3 miles 2 to 3 times a week, go the the gym the same number of days during the week an play tennis on the weekend. Have any idea what could be happening to me, should I have further testing done? I will like to have an Hematolist opinion if possible. Doctor: Hello, Welcome to HCM, I am Dr. Das Look, when a patient has this type of problems, we advise the patent to go for direct smear examination. that means, a slide should be prepared by pricking the finger directly. This slide should be examined by the pathologist for platelet count. This is the most reliable test that must be done in a patient with low platelet count. So, consult with your treating doctor about it. Regards"
},
{
"id": 88564,
"tgt": "Suggest treatment for persistent cough and abdominal pain",
"src": "Patient: greetings i started coughing about 2 weeks ago the coughs were very strong and firm but they were tolerateablethe night before yesterday i experienced very firm abdominal pain which was not going so we called the family doctor and he diagnosed me with apendix and i went to the hospital to run a test for it and it turned out i didnt have it however the morning after that i woke up seeing that my lips and some areas around my genitals were swollen with hives i asked the family doctor again and he thinked it was for the coterimoxasol i was taking (due to the coughing i took coterimoxasol and metronidazol ) and so he cut out and prescribed sefixim instead and prescribed injections of sefalexin and betametazon so i took those injections they made the condition a little less tender but the hives expanded vastly and now i have contipation can i know diagnosis please? millions of thanks! oh and by the way he said to me that i stop taking metronidazol too Doctor: Your cough was probably caused by a viral infection which is quite common. But it is probably unrelated to the hives or acute urticaria that your doctor has treated you for. He has additionally prescribed antibiotics to be on the safer side and has tried to cover both for the cough and the hives. Please stop metronidazole and the other antibiotics after completing the five day course. The constipation could have been caused due to the normal intestinal bacterial flora being disturbed. Taking plenty of curd or yogurt should help."
},
{
"id": 108990,
"tgt": "Suggest treatment for lower back and hip pain",
"src": "Patient: Hi there .. I recently lifted something heavy and my lower back and hips are in sever pain ... I have bursitis so that probably is the major problem and also a sciatic nerve problem but as well as my stomach and right side are sore especially after I eat ... will this go away when my back pain gets better or is there more damage I should get checked out Doctor: Hi,Your stomach and back pain are not related. Do not lift weights or bend forward it worsens the back pain. Hot water fomentation for back. Also start core muscle strengthening. For stomach pain visit a doctor."
},
{
"id": 137711,
"tgt": "Suggest treatment for swollen and painful toes after injury",
"src": "Patient: Hi there, I was playing baseball a week ago and did something to my foot where if I put pressure on my toes it s a sharp pain. I left it and just thought it would heal now a week later and getting worse with swelling. It s on the top of both my feet but one worse then the other. Top middle on big toe side Doctor: Dear patient You have got trauma to your both feet and since pain is not relieved even after one week possibility of fracture cannot be ruled out. I would advise Xray of both feet anteroposterior and oblique to rule out bony abnormalities. Visit radiology center nearby you and get it done. Meanwhile start tab diclofenac plus serratiopeptodase combination twice a day for pain relief. You need to consult expert orthopaedic surgeon with Xray report. Avoid playing till the time. All the best."
},
{
"id": 95009,
"tgt": "Child having abdominal pain. USG abdomen test done. Took tetadox 100. Suggest further chek-up and which disease can it be?",
"src": "Patient: my 3.6 year child was complaint abdominal pain then I consult to his Dr. he advised USG abdominal and Findings are slo mild thickening of terminal ileum and caecum with small mesenteric lymphadenopathy and minimal interbowelloops ascites , likely to be infective etiology. I went for follow up and Dr prescribed medicine ,Tab Tetadox 100 for 10 days and Syp metrogyl 2.5 mg thrice for 7 days and also advise for test CBC-ESR,Tb Quantiferon gold, Montonk test, S.WIDAL,Urine and chest X-ray. kindly suggest further chek-up and which disease can be possible. Doctor: further check up will be to rule out tuberculosis, a CT scan of abdomen will be beneficial."
},
{
"id": 209925,
"tgt": "Is Pexep Cr25 is better than Prothiaden 75 for depression?",
"src": "Patient: I was taking Prothiaden 75mg, one tablet every night. Now Doctor has changed and giving Pexep Cr25 every morning after break fast. I have depression since 9 years and on medication. Is Pexep Cr 25 is better than Prothiaden 75? Has any side effect? Should I take life long? Doctor: HIThanks for healthcare magicProthiaden is a tricyclic antidepressant and pexep contains paroxetine, which is a SSRIs. Proxetine is better than prothiaden in efficacy and in tolerance. It has lesser side effects as compared to prothiaden. It has few side effects like gastritis, nausea, sexual dysfunction, but these are very minor side effects. According to your duration of illness, may be you have to take it for life long.Thanks"
},
{
"id": 8037,
"tgt": "Is it OK to get a cheese smell from the pimples ?",
"src": "Patient: i have white pimples on my chin...when ever i rub them, i get an aroma of cheese, is this serious? Doctor: hi it is not serious ,its solid oil which look and smell like cheese, u need to take medicine for that. wash ur face with cleanser contain salicylic acid drink more water avoid sun exposure dont pick or popped it better to consult dermatologist"
},
{
"id": 76034,
"tgt": "Is throbbing pain while breathing normal due to inflamed chest?",
"src": "Patient: hi, a few hours ago i went to the A&E for chest pains, had an x-ray and an ecg with normal results..i was diagnosed with having an inflamed chest wall and given ketoprofen slow release capsules and paracetamol. I took 100mg of the ketoprofen and 1500mg of paracetamol and i am still experiencing pains, in fact it seems the pain has worsened. When i take a deep breath there will be a sharp throbbing pain. Is this normal? Doctor: Hi thanks for contacting HCM..You are having chest wall inflammation...So pain is present ....For pain you had take paracetamol and for inflammation relief ketoprofen...Here continue antiinflammatory drug like ketoprofen for relief from inflammation for five days . ..Avoid strenous work for few days....Hot pad application can be useful ...Avoid bad sleep posture ...Avoid movement causing pain...Take rest for few days ...Take care ..."
},
{
"id": 214306,
"tgt": "Suggest remedy for boils under the armpit",
"src": "Patient: i am a mother of a two month old baby, lactating, and am suffering from boils in the area under the right arm pit next to the the right breast for the past three days. there is a cluster of boils whish is extremely painful, the same looks like caused by either the bra strap or red ant bites. Can u pls suggest any home remedy since the boils looks to be spreading.This is the first time have boils for reasons still unknown...pls help... Doctor: HelloAs you mentioned that these boils are in the arm pit and for 3 days with pain .These type of boils ( frunculosis ) are usually due to bacterial infection ( usually streptococcal or staphylococcus are responsible ) .When such patient visit with this short duration of symptoms I advise them to take LEVOFLOXACIN 500 + AZITHROMYCIN500 ) in combination pack , one tablet after meal / day for 5-10 days .Also take pantaprazole-d with this tablet to avoid nausea and vomiting.Also take aceclofenac 200 mg c r twice in day for 5 days .Usually cold ice packing provide relief but not a cure . So for this you have to take tablets .Hope this will help you.Rest is also important .But if this lymphadenitis ( boils ) don't respond to this treatment after 5 days then consult a surgeon and get his opinion and all tests.Good luck."
},
{
"id": 147500,
"tgt": "What causes white spot on the lower right cerebral part of the brain?",
"src": "Patient: my son is in the hospital for a torn hamstring. he can only move his toes. the doctor did an mri with contrast today. they found one white spot on the lower right cerebral part of the brain. spinal tap is scheduled tomorow . hes twenty one very active. doctor is looking into ms or lymes. Doctor: Hi,Thank you for posting your query.I have noted your son's symptoms as well as MRI brain findings.The most likely causes for a white spot (hyperintense lesion) in the brain in a 21-year old are demyelination (such as multiple sclerosis) or infection. The confirmation of diagnosis can be done by lumbar puncture (CSF analysis) and MRI of spine.There is no need to worry, as there are good treatments available for both the conditions.I hope my reply has helped you.I would be pleased to answer, if you have any follow up queries or if you require any further information.\u00a0\u00a0\u00a0\u00a0\u00a0Best wishes,Dr Sudhir Kumar MD (Internal Medicine), DM (Neurology)Senior Consultant NeurologistApollo Hospitals, Hyderabad,For DIRECT QUERY to me: http://bit.ly/Dr-Sudhir-kumar My blog: http://bestneurodoctor.blogspot.com/"
},
{
"id": 151400,
"tgt": "Severe vomiting. Brain CT scan done. Taking treatment. What precautions should be taken?",
"src": "Patient: a 26 year old female c/o severe vomiting since 03 days.CT brain shows-two calcified granulomas in right cerebellum with no perifocal oedema. ECG- ST depression in III,avf .Tretment continuing Inj.Decadron,Inj.Ranitidine,Inj.Ondansteron,Valparin-200 tds,stemetil 01 tab tds,Metolar25MG bd,Monotrate 20mg bd.Further advise and management please. Doctor: DEAR FRIEND, CALCIFIED GRANULOMAS ARE VERY LESS LIKELY A CAUSE OF VOMITING IN THE ABSENCE OF HEADACHE AND PERILESIONAL EDEMA ON CT, KINDLY GET UR SELF REASSED WITH ADIFFERENT DOCTOR REGARDS Dr. Vineet Saggar (MCh) Neuro Surgeon / Spinal Surgeon"
},
{
"id": 8905,
"tgt": "Which is a good but economical hospital in Mumbai to do a nose job ?",
"src": "Patient: hlo i wanted to know a good doctor not extremely expensive but a good doctor and a good hospital for doing nose job in mumbai, i am 17 around 5 3 and weigt 53 Doctor: Dear Anamika, Basically the treatment would depend upon the requirement, like enhancement of nasal bridge or lifting of the nasal tip may be done both surgically and nonsurgically whereas reduction in the size of the nose is only possible surgically. Nose job means?.. are you looking for surgical correction of nose or non surgical correction ? Kindly specify your concern.. Thanks & Regards."
},
{
"id": 211414,
"tgt": "Why does one experience intense stress, tingling sensation, excessive anger and inability to focus due to no specific reason?",
"src": "Patient: Whenever I get really angry and streesed my head starts to tingle and my hands feel wet when their actually dry after I feel numb all over and nothing bothers me Also for some reason lately I can't seem to focus on things like if someone's talking to me I can hear them but I don't understand them as if someone pointed out that a truck was red and I still didn't know what color the truck was as if I never heard anything to begin with it sounds funny but it happens more times than it shouldIs this normal? Doctor: this is sympathomimetic action,that is reaction to increase in adrenalin levels in blood.which perist for short period.u need to cool down,do deep breathing..and relax."
},
{
"id": 183591,
"tgt": "Suggest treatment for lesion in molars",
"src": "Patient: hi i have a large well defined , oval, expansile , unilocular,lytic lession at juntion of body and rambus of mandible on right side associated with impacted third molar within the lesion , the lesion is extending upto the root of 2nd molar . there is acrtical break posteriorly. the lesion measures 2.5 x 1.3 cm cross section and in 2.6 cm height. the inferior mandibular nerve is exposed in the region of the lytic lesion Doctor: Thanks for your query, I have gone through your query.The unilocular radioluceny asssociated with an impacted tooth could be because of the dentigerous cyst or mural ameloblastoma. There might be a involvement of the inferior alveolar nerve canal with the nerve by the lesion.The lesion can be treated surgically by doing enucleation with removal of the wisdom tooth under local anesthesia or general anesthesia.I hope my answer will help you, take care."
},
{
"id": 160383,
"tgt": "What causes severe blocked nose and little foamy saliva from mouth?",
"src": "Patient: My Four year old son had cold on monday. When he was sleeping facing his face up, His nose was heavily blocked and very little foamy salvia came out his mouth. Is it cause of concern? Doctor: Hi,Does he have fever? Was the frothing associated with any jerking or stiffening of body? If not, we need not worry. Nasal stuffiness and block suggests rhinitis as a part of flu, common cold. A blocked nose can interfere with normal frequent swallowing of saliva to clear mouth, resulting in drooling. Just apply some nasal decongestants. I usually gives oxymetazoline nasal drops (2 drops on both sides, 3 times a day) for 5 days.Take care. Hope I have answered your question. Let me know if I can assist you further. Regards, Dr. Muhammed Aslam T. K., Pediatrician"
},
{
"id": 219214,
"tgt": "Is to normal to have no contractions even after the plug is passed?",
"src": "Patient: this is my first pregnancy, i went to the doctor last week on friday and was 3cm dilated. the plug passed saturday around 0215, i have had no contractions; is it normal to not feel any pain and when is it a have to go to the hospital time- i don t want to go there and spend 20 hours doing nothing and not being able to eat when i could of spent my time at home resting and eating. i am a bit worried -then again im a worry rat- cuz my baby (he) isnt moving that much, he moves at least once every half hour to an hour compared to before when he wouldn t move for about nine hours in a 24 hour period Doctor: Hello, Thanks for the query. Its not uncommon to find cervix dilated even up to 5 or 6 cm without pain although its not the norm. If your pregnancy has remained uncomplicated throughout and if you can reach the hospital within no time if your water breaks or pain starts then you can take that chance of staying at home for sometime. But its risky when your cervix dilates without pain and your labour progresses very fast and baby is not monitored. So I suggest you to carefully watch for pains or contractions and and get admitted for labour at least when your cervix is 5 to 6 cm dilated.2. Your baby movements are generally restricted during labour and late stages of pregnancy. Your obgyn will assess baby status by taking its heart beat graph( cardiotocograph) which will tell you how baby is doing. Hope this helps.Have a safe delivery. Good luck."
},
{
"id": 85640,
"tgt": "What is the use of oligocare and fertyl M?",
"src": "Patient: i want to know the use of oligocare and fertyl M as i am prescribed by two urologist, one prescribed oligocare and another Dr. Prescribed Fertyl M along oligocare for the Low sperm count (40 million/ml, low motility (60%), low rapid progressive (10%)) Thanks for your suggestion. Doctor: Hello,The medicines you have mentioned are prescribed mainly for patients with low sperm count who are trying to conceive and are unable to because of low sperm count.I would suggest you to follow your doctors advice on intake of the medicines as per the prescription for better results. As these medicines are prescribed after careful evaluation of the patient. If you have doubts then you can have a conversation with your doctor on your next appointment.Hope this helps. Feel free to contact for any further queries in the future."
},
{
"id": 123946,
"tgt": "What is the effect of a swollen ankle on plantar fasciitis?",
"src": "Patient: I was diagnosed with plantar fasciitis and just recently fell from a few steps while going down. My right ankle is swollen and it s been 2 weeks now. I noticed that my 2nd toe is really painful. Could have I aggravated my plantar fasciitis due to my swollen ankle? Doctor: Hello, As you been diagnosed with plantar fasciitis and a fall which may have twisted the ankle which led to swelling in the ankle and pain in the 2nd toe, it is a possibility that there will be ligament sprain which has led to swelling. But it's always on a safer side that x-ray is advised as sometimes it may be hairline fracture and the location need to be traced with the help of the x-ray. How are the movements of plantar flexion and dorsiflexion of the ankle? Are they also painful? Please do check the inversion and eversion too. For now, first, stabilize the foot in the crepe bandage or ankle brace. This will help to provide stability of the joint. You can also do hot water fermentation and elevation of the foot on the pillow above the level of the heart while supine lying. For the 2nd toe using quick icing will help reduce pain. In case the pain and swelling don't come down in 4-5 days of time than please opt for an x-ray on a safer side. Hope I have answered your query. Let me know if I can assist you further. Take care Regards, Jay Indravadan Patel, Physical Therapist or Physiotherapist"
},
{
"id": 140470,
"tgt": "What causes swelling and redness in feet and tingling in shoulder?",
"src": "Patient: I have had swollen feet the past few years usually after flying. it has become worse and there is redness. I have also been experiencing vertigo, I cannot close my eyes without holding on to something. I feel tingling from my left shoulder down to my elbow about 50 percent of the time. I regret trying meth because I ve been on it almost daily for 14 months. started snorting now smoking. Are these symptoms related. Male, Cauc, 44yrs 601, 200lbs. Doctor: Hello, Your use of methamphetamine is a risk factor in the development of your symptoms of dizziness/vertigo. The swelling in the feet may be due to what we call dependent edema and can be resolved if you are on a plane by taking walks up and down the cabin aisle every 1-2 hrs. The tingling sensation may be due to an entrapped nerve in your shoulder or neck that can be worked on by physical therapy and possibly released. I would recommend you consult your physician or an addiction specialist on the best path to take to be able and suspend this activity on a permanent basis. I formerly did head up an addiction center and so I know how difficult \"kicking the habit\" can be. But if you don't, the long term consequence of meth are unfortunately not good at all. Hope I have answered your query. Let me know if I can assist you further. Take care Regards, Dr Dariush Saghafi, Neurologist"
},
{
"id": 84632,
"tgt": "What are the side effects of steroids?",
"src": "Patient: Dear sir ,I am 22 years old male height-5 6 , 55 kg from punjab (india) i have planned to take deca durabolin steroid plzz suggest me some important things which should i knw about this injection What amount of dose is sufficient for my body and what are its effects ? Doctor: Hi, Side effects of steroids include severe acne, hair loss, heart disease, such as heart attack and stroke, altered mood, irritability, increased aggression, depression or suicidal tendencies, blurred vision, early cataracts or glaucoma, papery thin skin with easy bruising, insomnia, decreased immunity, secondary infections, raised blood pressure, fluid retention, weight gain and increased growth of body hair.Hope I have answered your query. Let me know if I can assist you further. Regards, Dr. Ajeet Singh, General & Family Physician"
},
{
"id": 10436,
"tgt": "Suggest remedy for hair fall & drandruff",
"src": "Patient: Hello, I am 25 years old and weight about 45kgs. I am having lots of hairfall and little dandruff for whihc I am using Tri-a-top as prescribed by a doctor for dandruff. I have scanty hair on both right and left side of the head. Please could you suggest me a good dematologist or trichologist in mumbai. I have scanty hair on both right and left side of the head. Doctor: Hello and Welcome to \u2018Ask A Doctor\u2019 service. I have reviewed your query and here is my advice. As per your case history of hair fall and dandruff, my treatment advice is - 1. Use a good herbal hair oil and shampoo for routine use. 2. Apply an antifungal lotion like Clotrimazole lotion twice daily on affected part. 3. Take an iron supplement and vitamin B12 supplement. 4. Other treatment options are oral Fluconazole and Terbinafine taken only after consulting a dermatologist. Hope I have answered your query. Let me know if I can assist you further."
},
{
"id": 220373,
"tgt": "Does transvaginal ultrasound kill fetus?",
"src": "Patient: i am trying to get pragnant . on day 16 of my cycle i went to my doc and had transvaginal ultrasound and the next day started bleeding and have cramps and pain like period also bled for 3 days. could it be i was in process to get pragnant and transvaginal ultrasound killed the process? Doctor: Hifeel sorry for your condition.transvaginal sonography as such doesnt kill the process. why you went for transvaginal sonography at day 16. are u going for follicuar study.if yes is that mentioned follicle ruptured at or before scans.if follicle ruptured and if chance of pregnancy expected, this type is bleeding is unlikely.it might be an incidental occurence. excessive manipulation during examination is slightly associated with abortions. dont have all such fears as it is usually done by trained personals.thank youregardsvasundhara"
},
{
"id": 31695,
"tgt": "What causes my daughter's redness of lips after a bout of fever?",
"src": "Patient: I just noticed today after school my daughters lips are a bright shade of red...almost like she has lipstick on. Over Christmas she ran a high fever for 4 days, had white patches in her throat and when the fever broke, had a bumpy red rash on her chest and then it all just went away about 3 days after Christmas. What could this be and do I need to be worried? Thanks Doctor: Hi Dear, Welcome to HCM. Understanding your concern. As per your query your daughter have symptoms of redness of lips after a bout of fever which seems to be due to viral or bacterial infection. I would suggest you to visit general physician once and get it examined. You should get blood tests done and start treatment after proper diagnosis. Do tepid sponging as well to lower down fever. Apply petroleum jelly on lips. Take diet rich in fresh juices and fruits. Drink plenty of water and fluids. You should take anti-allergic medication along with proper course of antibiotic along with Ibuprofen. Do tepid sponging as well to lower down fever. Take multivitamins to booster immunity of body. Hope your concern has been resolved. Get Well Soon. Best Wishes, Dr. Harry Maheshwari Hope your concern has been resolved. Get Well Soon. Best Wishes, Dr. Harry Maheshwari"
},
{
"id": 155892,
"tgt": "What causes coughing up blood clots?",
"src": "Patient: I fell down a flight of stairs a week ago. Now I am having facial pain and coughing up some blood clots. I had a cat scan and it showed my left cheek bone and around my left eye fractured. My nose and around my sinuses and teeth hurt. What is the answer? Doctor: Hello, Thank you for your contact to health care magic. I understand your concern. If I am your doctor I suggest you that as a result of injury to cheek bone it causes bleeding inside the sinus cavity. The clotted blood on the sinus cavity are gradually coming out now. This is the reason of clotted blood coming out of the cough. I will be happy to answer your further concernYou can contact me. Dr Arun Tank. Infectious disease specialist. Thank you."
},
{
"id": 16018,
"tgt": "Severe rash all along the abdomen, redness, no itching. Tried cortizone, no relief. Cure?",
"src": "Patient: I have a very bad rash along the area where I had an operation 40 yrs ago. It is very deep and aboujt 10 inches wide all along the bottom of my stomach . I have tried cortizone 10 and it helps a little but not enoujgh. I am 87 yrs old and it it rally bothering e. it doesn t itch . just very red. can youj tell me what to use? Doctor: Hi, Welcome to HCM. Rashes at a particular anatomical site can be due to excessive sweating, some allergy, non-infectious seborrheic dermatitis or contact dermatitis, etc. You should apply 1% hydrocortizone cream for a week or 10 days or 1% clotrimazole cream if you suspect it to be fungal infection, anti-histamines like cetirizine and medicated lotions and moisturisers can be helpful. Non-infectious rash generally go away by themselves in 10 days. Take care of your hygiene, wear cotton clothes, avoid eating oily or spicy food. Keep yourself well hydrated. If rash still continues, see a dermatologist, get clinically examined, take care."
},
{
"id": 145729,
"tgt": "Does a person feel pain when dying with catastrophic cva?",
"src": "Patient: My mom died of a catastrophic cva. It happened on Friday am. Was unconscious. Was intubated. Removed after a few hours per her wishes in proxy. Remained unconscious but breathing on her own until Monday am when she died. Dr said she didn t feel anything but her legs were moving had slight gag reflex when extubated. Was she in any pain & why did she not fie immediately but died 3 days later? Could she have felt any pain during this? Doctor: Hi,Thanks for writing in to us.Sorry to hear about the loss of your mother. The brain is an important organ and a stroke can cause deficient blood supply or leakage of blood in to the brain as the case might be. The end result is decreased functioning of brain and this continues till the brain cannot sustain itself. Coming to pain being felt, the brain by itself cannot feel pain but if there was some leakage of blood in to the brain coverings then irritation of nerve endings might happen. This does not happen in every patient and when it happens, there is restlessness in the patient. If your mom did not show any signs of restlessness then there was no pain that she possibly had. The brain was supporting itself and in 3 days there was deterioration of the other organ functions leading to her demise. May her soul rest in peace."
},
{
"id": 219739,
"tgt": "Suggest medicine to terminate pregnancy",
"src": "Patient: HI Doctor, Last month i got my periods on Jan 9th but i have skipped my periods for Feb month. I performed pergnancy test at home and the result showed positive. But we do not have plans for a baby at this point in time. I was suggested with Regestrone tablet for 2 times for 5 days. Is this safe or is there any other oral medicines that you can suggest. Doctor: Hello dear,I understand your concern.In my opinion the rogesterone is prescribed for the onset of periods when the periods are delayed due to some other cause other than pregnancy.So it is not the right choice.Usually the termination of pregnancy is done by medical and surgical procedures.Medical termination of pregnancy is possible only upto 7 weeks of pregnancy.Medical termination of pregnancy is done by combination of mefipristone and misoprostol.I suggest you to consult doctor and take the medicines for termination.Kindly never try the medicines on your own as they are associated with complications like heavy bleeding,incomplete abortion etc.But as the pregnancy is there rogesterone doesn't work.Kindly consult a gynaecologist.Hope this helps.Best regards...."
},
{
"id": 223584,
"tgt": "Can morning after pill avoid pregnancy after unprotected sex?",
"src": "Patient: hi im super worried because i realized I m in my most fertile period of my cycle and i had sex, but only precum was involved. should i take the morning after pill or how high are my chances of getting pregnant, cuz i hear the morning after pill is bad for your body Doctor: Hi,You did unprotected sex during fertile phase and it has high chance to become pregnant. Better to take morning after pill to prevent unwanted pregnancy.In some case it cause hormonal imbalance and side effects are tolerable. You can take it with in 72 hour of sex. But it has high effectiveness if taken as early as possible after sex.Avoid stress, take healthy diet, drink plenty of water and do regular exercise. Use condom in future to prevent unwanted pregnancy. Hope this may help you. Contact further if follow up needed.Best regards,Dr. Sagar"
},
{
"id": 77263,
"tgt": "What causes dry cough?",
"src": "Patient: I have dry cough and it feels like a tingle or a tickle at the top of my sternum that causes me to cough but nothing is there. It would usually have gone away by now but its been a few weeks now. Im not sure it it wad from working and cutting blackberry bushes and inhaling the pollen or what. Doctor: Thanks for your question on Healthcare Magic. I can understand your concern. By your history and description, possibility of irritant cough is more. And it can be due to all the things you have mentioned like cutting and working blackberry bushes, inhaling pollens etc. These things are highly irritant in nature. When they inhaled, they cause inflammation in the trachea (center of chest-sternal area). And this gives you tingle or tickel sensations at the top of sternum. So better to avoid such things or wear face mask while doing those activities. At present take combination of antihistamine and anti inflammatory drugs. Do warm water gargles 5-6 times a. Drink plenty of fluids orally and keep your self hydrated. Don't worry, you will be alright. Hope I have solved your query. I will be happy to help you further. Wish you good health. Thanks."
},
{
"id": 182132,
"tgt": "Suggest treatment for severe toothache",
"src": "Patient: HI, I have a severe toothache (upper back jaw)and I did see a dentist yesterday and they said all my teeth were fine.I even did an xray but everything seems normal. I suspected for sinusitis. I did have a sore throat flu and fever couple of weeks ago which doctor prescribed an antibiotic for it. Then my fever and flu gone. But I noticed my nose not fully clear always feels congested but not runny. Few days ago I started having this very painful toothache which I can't sleep at night. It came and gone once in a while . I took a painkiller to ease the pain(paracetamol 500mg).I went to see a doctor yesterday afternoon and she gave me loratidine,pesudoeprine,piritone and flomist spray.But last night I still have a very painful toothache,and this morning again after taking all the medication the doctor prescribed 2 times (last night and this morning).I will go to another clinic for second opinion after this.For information I did feel a ittle bit pain when pressing the front face beside the nose(test for maxillary sinusitis).It is really painful although it come and gone once in a while.Any advice or recommendation? I was thinking of taking antibiotic for maxillary sinusitis for a week or so and monitor before meeting ent specialist.What do you think? Please help me doctor.Thank you.Abdul Rahman Halim. Doctor: Hello,Your tooth discomfort may very well be related to your sinusitis. This is not uncommon. Please be careful before introducing more antibiotics. You may need a change in the antibiotic or a stronger dose, if what you have taken has not been fully effective. Please consult your doctor before taking any antibiotics. Definitely take medication to reduce discomfort due to congestion. This will help determine if you have an issue with the tooth or it is merely a reaction to the inflammation caused by the congestion. Inflammation and the accompanying discomfort needs time to heal. Keep this in mind and try and reduce your discomfort while giving more time to heal. My suggestions are to use a toothpaste for sensitive teeth, fluoride rinses, maintain good day dental care, follow a bland diet with no acidic and spicy foods. Your mouth may be acidic and this can cause a sensitive reaction at the gum line. This can be severe if you have root exposure or recession. You may also be breathing through your mouth with the congestion. This creates a harsh environment and dries out your tissue. Your plaque is much harder to remove. Please make sure you are well hydrated. Use of antibiotics can also cause a reaction such as an oral yeast infection. Taking acidophilus may prevent this. Other possible dental conditions may relate to grinding your teeth or a possible tooth with a fracture. You may continue to monitor the area and take an additional x-ray if your condition continues to rule out an abscess or other conditions.Thank you for your inquiry. I hope you find my suggestion is usable and that you feel better soon."
},
{
"id": 144292,
"tgt": "What causes dizziness and blurred vision?",
"src": "Patient: I collapsed twice and was misdiagnosed with vertigo. A week later, after an MRI , I was told I had a cerebellum stroke and a stenosis/occlusion of the left vertebral. I am female age 53. I went back to teaching after 7 weeks off and I am so exhausted and weak, dizzy and blurred vision at times. Is this okay? I was on medical leave and now I do not have sick days left and have to work. I do not have money for this answer. Sorry Doctor: HiI am Dr Mittal.I have read your query.I think I can help you.No need to worry about the financial part of the query.I will try my best to answer your query.A stroke takes time.The symptoms persist for a long time.And sometimes, they do seem to come on and off.I will suggest that you continue to take the antiplatelet and statin medicines that have been prescribed to you.Make sure you do not stop the medicine- that point is essential.You should have been taught exercises for balance in physiotherapy while you were in the hospital and recovering from the stroke.Make sure you continue these exercises as far as possible.I have tried to make it as simple as possible.Please feel free to ask a further clarification.Please click on HELPFUL.Best of luckDr Mittal"
},
{
"id": 94591,
"tgt": "Large distended stomach with pain in upper left quadrant and lower abdomen. What should I do ?",
"src": "Patient: I have a very large distended stomach with pain n upper left quadrant n lower abdominal pain . I hat a CT scan out of town said my uterus has falling out of place layoffs on my small bowel n the fluid in my abdomen where the pain is in the upper left quadrant where my colon descends seems to be kinked I was also told my cervix was already down on my vagina. I cannot use the bathroom only having 3 bowel movements in almost a month which were so painful I broke out n a sweat I ve taken stool softeners fleets n several other things just causes mote pain bloating n cramps I take take gas pills n no relief either and do not pass a lot of gas or feel the need to. I can t eat much very small amours n mostly liquid or soft foods. I often lose control of my bladder n sometimes feel I need to go so badly bit just trickles out or none at all. I was told I had a hiatal hernia a few weeks ago and last night I got the hiccups so very bad n couldn t stop the pain increased my husband said o kinda slumped over on the couch n was unresponsive for about 30 to 60 seconds. Do I need to go yo the emergency room or can I wait to schedule an appointment and with which h Dr do o need t Doctor: Dear madam, you need to go to ER if you feel constant pain, fever, bretahless, vomit blood or feel fatigue but your symtpoms definitely means something and detailed work up should be done. You obviously have some kind of intestinal passage disorder caused by either mechanical or non mechanical causes. YOUR GYNAECOLOGICAL PATHOLOGY U VE DESCRIBED COULD effect your intestinal motility and if there is a certain cause then pills wont be helpful. The best way to rule this out is to the colonoscopy,barium swallow or diagnostic laparoscopy and everything shoud be clear. Wish you good health, Dr. Ivan Romich"
},
{
"id": 35875,
"tgt": "What causes episodes of campylobacter, diarrhea and sickness?",
"src": "Patient: Hi there, I have had two episodes of campylorbacter in the last two years and numerous small episodes of diahrroea and sickness. Someone has mentioned to me that this may have been agrovated by taking omeprazole which i have been taking for a few years now for acid reflux. Is this right??? Doctor: hello there,The acid in your stomach acts as a barrier which prevents bacteria from entering your GI tract. Since omeprazole is a strong acid reducing agent we prescribe it only for small durations. If you are having it over extended periods then the stomach does not produce adequate acid that can neutralise bacteria. Hence you have a higher predisposition for infections. Also campylobacter is seen more in individuals having meat that may not be sterile. So be careful what you eat .Hope this helps,Dr Arun A"
},
{
"id": 95929,
"tgt": "I am pregnant and suffering from appendix pain",
"src": "Patient: I am 18 years old and had surgery a month ago now i am pregnant and haing pains.. can i loe my baby?? hi, i had my appendix removed about 6 weeks ago... and i just found out im a month pregnant!! i am having some pain in my stomach by one of the places they went in to get the appendix. Is this normal? Doctor: Hi Welcome to HealthcareMagic. This could be a pain due to surgery .Do you have any bleeding per vagina ? Do you have any pus from the surgical area ? Please mention these details as a comment here. There could be generalized pain in pregnancy.I advise you to visit your Gynecologist and get an Ultrasound done to ensure safety. Take care."
},
{
"id": 81688,
"tgt": "What causes chest pain?",
"src": "Patient: Why does it feel like I m being poked in my chest with needles? I ve been having chest pain for a while and I ve been to the doctor and they said it s nothing but it doesn t go away unless I take some ibuprofen or tylenol or something. I m 13 and I m really worried. What could be wrong? Doctor: Thanks for your question on HCM. In my opinion you should not worry much about this. As it seems muscular pain mostly. And you are improved with pain killers. So try to follow these steps for better symptomatic relief. 1. Avoid heavy weight lifting and strenuous exercise. 2. Avoid bad postures in sleep. 3. Take good painkiller and muscle relaxant. 4. Apply warm water pad on affected site. Another possibility is vitamin b 12 deficiency. So get done vitamin b 12 level and if deficient than start replacement."
},
{
"id": 172193,
"tgt": "What could cause body rash in a 6 years old child?",
"src": "Patient: My six year old daughter started of with a red rash all over her body. At first i thought it was german measels, there was no noticable fever. A few days later her hands and feet started peeling very badly. The rash is almost gone but the skin peeling is very bad. What could this be? Antoinette Doctor: Hi...I feel by what you quote he should be having a - Hand Foot Mouth disease. This is one viral illness among all other exanthemas which can cause fever followed by rash over palms and soles. It is a self-limiting disorder and itching can be really worrisome.I suggest you use any over the counter antihistamine if you have one with you now. You can use Hydroxyzine at 1-2mg/kg/dose (Maximum 10mg) every 6th to 8th hourly for 7 days. This can even cause some peeling of skin in the next while its recovering and do not worry about it.There's one another possibility with rash and skin peeling if the kid had high grade fever of more than 5 days duration and the kid has been sick....its Kawasaki Disease. If this is so he will require certain investigations and an ECHO of the heart. I suggest you consult your pediatrician for this.Regards - Dr. Sumanth"
},
{
"id": 131530,
"tgt": "Could erosion in bone connecting neck and shoulders have led to the intense pain?",
"src": "Patient: My mother (aged 64) has pain in her neck. Consulted a doctor and he said that there is an erosion in the bone connecting the neck and the shoulders and has prescribed some medicinesCan you please let me know what care should be taken now in terms of work, food , exercises so that there is no further damage.What would be the consequences of this situationPlease help Doctor: HiAs a precaution I may advise wearing of cervical collar for support and isometric cervical exercises under s physical therapist.arm could be supported in a sling.Get x-rays of neck and shoulder.For pain Ace-proxyvon 1 tab BD for 5 days.Consult orthopaedic surgeon"
},
{
"id": 170329,
"tgt": "What causes red itchy rash on the legs?",
"src": "Patient: My 4 year old daughter has bites that we believe to be mosquito bites on her legs. (about 4 bites) She has been complaining of itching & we have been applying hydrocortisone cream a few times per day. I noticed tonight that two of the bites (& one in particular) have a large (2-3 inch) reddened area around them & one has a defined white border around the red. Does this seem like a reason for concern? Doctor: Hi,It seems that there might be having allergic reaction to some insect bite producing itchy rash and bumps.Give her Benadryl or Cetrizine syrup for 2-3 days.Ok and take care."
},
{
"id": 116566,
"tgt": "Suggest treatment for fever and elevated SGPT/SGOT levels in blood",
"src": "Patient: Hi, My Father is suffering from Fever since last 10-15 days. 5 days ago, the fever increased and blood report showed increased SGPT and SGOT. The chest X-ray and sonography were normal. The doctors started giving anti-biaotic injections thrice a day and a dolo 650 tablet if fever above 100F . But the fever is coming back and is upto 101F once after 6- hours. Please suggest further investigations . Doctor: please do tests for leptospirosis and tuberculosis. In leptospirosis( spread by rat s urine), the liver enzymes are elevated but bilirubin does not rise till much later. TB can also cause raised liver enzymes due to granulomatous hepatitis. there are other sinister causes lymphoma which can be diagnosed by a bone marrow biopsy if the above 2 diagnosis are ruled out."
},
{
"id": 43095,
"tgt": "Is prolonged bleeding during follicular study harmful?",
"src": "Patient: hi, I m 28 year old.. I got married and we are try for baby.. As concult with doctor and done the scan which are all necessary. IN result i got multiple follicular in left overy. hence they prescrib the krimson 35 for birth control and to regulate the period. This month my period is start on 18th and 19th i have taken LH test which is normal so they ask me to take ccq tablet. Today i am in 7 days even though i am getting slight period. Is that any issu Doctor: Dear madam,I dont think it is much significant still if it continues seek your gynecologist advise.Dr. Shruti"
},
{
"id": 143518,
"tgt": "What causes emotional problems after recovering from coma?",
"src": "Patient: My friend woke up from a medically induced coma (was in it for 8 days) yesterday morning. She was confused and distressed tried to pull out the tubes and wires before her mum calmed her down and explained what happened. She doesn t stay awake for very long and each time wakes up confused and upset like she doesn t remember what was told to her the time before. Is this normal at this stage? She also hasn t spoken yet but tried to say something to her mum this afternoon but we couldn t make it out was too low and mumbled. Is this normal also? and will her voice eventually get stronger and become more than a mumble and be understandable? Also the fact she sleeps more than she is awake, is that normal too? Doctor: Hi,Medically induced coma is reversible. the symptoms you talk about are generally present in all patients admitted to and intensive care. they are temporary and will be reversed eventually. try to calm her with gentle music or talk to her about some happy moments also if allowed try to take her outdoor on a wheelchair, sunlight fresh air will make her feel beter and oriented.thank you"
},
{
"id": 7652,
"tgt": "Acne scars, acne prone skin, whiteheads on nose & chin. Any product to control scarring & acne?",
"src": "Patient: I have some acne scars on my skin . I have sensitive skin which is prone to mild acne . The skin type is combination and i have a few whiteheads on my nose and chin and it is try near the cheeks . Can you suggest some product to remove scarring and control acne .I have heard about glycolic acid products for the same. Can you suggest such a product easily available in India..? Thanks Doctor: Hi, what type of scar do you have?superficial or deep?If superficial you can undergo Microdermabrasion once in 3weeks 10-12 sittings.For whiteheads you can use saslicDS facewash twice daily.Glycolic peels do good for Acne and not acnescars."
},
{
"id": 225936,
"tgt": "Have mirena fitted. Have bleeding and cramping now. Had been taking northisterone. What is the remedy?",
"src": "Patient: Hi I have a mirena fitted 9 months ago, for constant heavy periods. It hadn't made any difference. I had a previous one for 6 years which worked like magic and I was period free for most of those 6 years. Now I have been prescribed northisterone which I have been taking for two months, with a break in between as I am waiting for an ultrasound to check if the mirena is ok, as its not helped as before. The problem us, I've started cramping and bleeding while taking northisterone! Will it stop? I also have transexamic acid to take, will this help stop the bleeding. I don't want the ultrasound cancelled because of the bleeding and I'm sick of this bleeding more than not. Can you advise please? Mandy Doctor: Thanks for using HCMYes tranexamic acid do help to reduce bleeding. Continue to take those medication as your doctors prescription.Though you are bleeding ultrasound can be done, not a problem.As you had responded well in previous mirena, this time also we expect a good response. Better to rule out other endometrial problems. Consult your gynaecologist for further evaluation.Have a good healthRegardsDr. Vidya"
},
{
"id": 125879,
"tgt": "Does ribs pain post an injury need medical attention?",
"src": "Patient: I fell on concrete 12 days ago. No head trauma,hit r side of body on sidewalk. R knee abrasions large hematoma,bruising inside r wrist,ect. Took care of myself with help @ home. Today I m having pain under ribs,not fix,lower. Called PMD,was told she couldn t see me for 10 days. I m only concerned for any internal injury. I m also a hematology patient,low white count,M spike,been followed for 6 years. No do of cancer. Should I insist on seeing a dr tomorrow? Doctor: Hi, Cannot say in your particular case, but generally, the M spike is a long term condition. It could become a cancer that generally goes into the bones, weakens them, increasing risk of fracture with pain. Most rib fractures do not need much treatment if they are not cancer. Hope I have answered your query. Let me know if I can assist you further. Regards, Dr. Matt Wachsman, Addiction Medicine Specialist"
},
{
"id": 81299,
"tgt": "What causes severe pain on the chest and shoulder?",
"src": "Patient: Hi, I am a 26 year old male. For the past 2 years I have suffered from terrible chest pain and left shoulder blade pain. It is not always a sharp pain but is more of a discomfort. I get the feelings of palpitations and also get sore jaw sometimes and sore pains in my left arm. I have been for numerous check ups and tests but everything is coming back normal. My doctor keeps telling me it anxiety which I have been on medication for, for quite some time now. I have also spent a lot of money on therapy. Nothing seems to be getting better. I don t feel stressed at all and I know there is something out of place going on in my body. I have just recently developed a couch which is now making me worry about my lungs( I have had hayfever though so could be why). I feel my symptoms get worse after physical exercise or a hard days work. Could it be that I need to see a chiropractor? Any help would be great? I m worrying myself sick about it. Thanks Doctor: Thanks for your question on HCM.I can understand your situation and problem.In my opinion you should consult pulmonologist and get done1. Chest x ray2. CT thorax if needed3. PFT ( pulmonary function test).Since your cardiac work up is normal , we need to rule out pulmonary causes now. You are also developing cough, this also favours possibility of lung related causes.Chronic infections can cause chest pain and pleurisy (inflammation of pleura). Pleurisy can cause shoulder pain. So chest x ray is needed to rule out infection.Chronic bronchitis can also cause similar symptoms. So get done PFT to rule out this. If all are normal, than mostly you are having anxiety related symptoms only.So consult pulmonologist and rule out pulmonary causes for your symptoms."
},
{
"id": 195698,
"tgt": "What causes cuts with severe pain on the head of the penis?",
"src": "Patient: I am having few cuts on pennis and very immense pain in the top of the pennis.I am 45 yrs old .Pl suggest some medincines Age:- 45 yrs sex:-Male I noticed after having sex with my wife, have multiple cuts on the top of the pennis when I try to push the foreskin back I feel pain Doctor: Hello and Welcome to \u2018Ask A Doctor\u2019 service. I have reviewed your query and here is my advice. It may be Candidal balanoposthitis. Apply antifungal cream like Sertaconazole with antibiotic cream. Take Ketoconazole tablets for few weeks. Keep away from sexual intercourse. Hope I have answered your query. Let me know if I can assist you further. Regards, Dr. Ilyas Patel"
},
{
"id": 125269,
"tgt": "What causes body pain even after vit B12 deficiency treatment?",
"src": "Patient: body ache..left arm to leg tingling..discovered that i had low b12 tht was 145 and d3 was 6..took injections n now am 450 b12 and d3 is 33,again the same pain started...cud it be the deficiency or fibrocities ?i also had spondolysis of scapula nerve n got treated by accupressure Doctor: Hi, In your situation you need to supplement more of vitamin B12 with injection. If pain persists then consultation by orthopedic surgeon may be required. Hope I have answered your query. Let me know if I can assist you further. Regards, Dr. Gopal Goel, Orthopaedic Surgeon"
},
{
"id": 90284,
"tgt": "What causes severe abdomen pain with sweating and chills?",
"src": "Patient: 22 yrs old male 5'3 130 Ib this morning I woke up fine three hours later I get a really painful stomachach with sweat and chills. I could not poop, but tried really hard after half an hour, diahrea came out. Still having the painful stomachache, I go to the bathroom again,but nothing, except for som kind of red liquid. I was constipated since yesterday. Is there any serious problem I'm having? Doctor: Hi.Thanks for your query.There looks to be a serious problem as per your history.The sudden onset of pain in abdomen- sweat and chills- unable to poop in the beginning , followed by diarrhea- pain continues and now red liwuid...This is very classical finding of a disease called ''intussusception''.In this condition the part of the intestine enters into the intestine to give a tube-in-tube appearance. You need to rush to ER.Do not take anything orally.You have to go for :X-ray of the abdomen in the standing position and-Barium enema- as for diagnosis and many times it is the treatment on its own. The pressure of the barium can resolve the intussusception.Once the problem is sorted out temporarily, you have to undergo definite diagnosis by way of CT scan , colonoscopy and other investigation and the treatment then is well planned to remove the cause that caused this. At the same time this helps to rule out any other pathology."
},
{
"id": 209063,
"tgt": "What is the treatment for panic attack and phobia?",
"src": "Patient: age 35 /height 5.10/ wt 64 kgs iam facing problem with fobia of alone drive bike car travel and any time i carry small water bottle with me but i never drink it if i dont get bottle sudden i became panic when i find it i get calm how to improve confidence and how to come out from fobia etc Doctor: DearWe understand your concernsI went through your details. I suggest you not to worry much. The symptoms you are explaining are not of phobia but are of anxiety disorder. You are feared or worrying always about your health and something might happen to you. You are finding alternative obsession in keeping a bottle of water with you. You are believing that water can save you for the time being. No problem. There are lot people around you like this. The obsession could be something else. That is all. Consult a psychologist and undergo counseling sessions. You shall be alright.If you require more of my help in this aspect, Please post a direct question to me in this website. Make sure that you include every minute details possible. I shall prescribe some psychotherapy techniques which should help you cure your condition further.Hope this answers your query. Available for further clarifications.Good luck."
},
{
"id": 162707,
"tgt": "Can bronchitis and asthma in the parents cause the same in the child?",
"src": "Patient: Hi Doctor my son is a year old he has been coughing a lot recently, took him to our family Dr and he gave him betalin, I just wanted to know whether I should continue with the medication. I am asthmatic, husband has a history of bronchitis and our elders daughter was diagnosed with asthma and lactose intolerant when she was 2 years old. Could my son have inherited our illness too?? Doctor: Hello and Welcome to \u2018Ask A Doctor\u2019 service. I have reviewed your query and here is my advice. Bronchodilator is given as a reliever medicine in asthma but to make a diagnosis you should get him checked whenever he has symptoms and then continue as reliever medicine only. further follow up will be required if symptoms persist. inhalers and nebulizer can also be used but for that initial diagnosis should be made. Hope I have answered your query. Let me know if I can assist you further."
},
{
"id": 111045,
"tgt": "Suggest remedy for back pain in patients with kidney stone",
"src": "Patient: I have a history of kidney stones and I had lithotripsy for an 11mm stone on my right side ureter a few weeks ago. I had a stent in since August 3rd. After the surgery I removed the stent after a three day wait. I have had pain in my lower back and kidneys everyday since. It get worse when I go to bed. I have been waking up in the middle of the night with this pain. I do have additional stones that are still in the kidneys. I have not passed any blood, so I don't believe that I have an obstruction. After I am up for awhile the pain subsides. If I walk around in the morning the pain is also present. If I sit down for a bit the pain goes away. HELP. Doctor: HIThank for asking to HCMI really appreciate your concern looking to the given history I could say that this is a renal colic and in my opinion this could be best treated symptomatically as long as the pain is concern and the best medicines for this is Tab Dicyclomine three times in day and Diclofenac 50 mg once in day take care and have a nice day."
},
{
"id": 171832,
"tgt": "Suggest treatment for penis injury in a child",
"src": "Patient: my 5yr old nephew was playing with his cousin who is mentally challenged. He bit him on his penis but he was wearing his pyjamas . there is redness with pain . should i worry , should i put him on antibiotics ,if yes can you please prescribe the antibiotics Doctor: Hi,Thanks and welcome to healthcare magic.If the injury is not severe and bleeding you need not worry.usually it heals of its own.If it is infected the he needs an antibiotic like erythromycin .Hope this answer is OK for you .Further queries invited .Dr.M.V.Subrahmanyam."
},
{
"id": 203745,
"tgt": "Can bumps and pimples on penis and groin area be symptoms of HSV 2?",
"src": "Patient: I am a 27 yr old male. After recent unprotected sex with my current girlfriend I noticed some pimple like bumps on the side of my penis... It started as two one on the mid right side and one on the mid left side.... The recent bout of sex was vigorous and slightly rough so I figured it was due to that and chalked it up as ingrown hairs or just zits... The bumps formed white heads and all but popped on their own. they appeared 1-2 days after this sexual instance. 2 more days went by and as I went to urinate I noticed the same bumps some with white heads, some without spread all around my groin area... Not in clusters but individual bumps scattered midway up my penis shaft to the base, above my penis in my pubic hair as high as my waistband of my boxers, on the upper inner part of both thighs almost in the crease of my groin. I am concerned it might be hsv 2. The bumps will pop and become tender afterwards but not any excruciating pain. They pop with clear pus normally no blood, and they hurt when popped to the point that I don t want to squeeze them all the way to fully drain them. My girlfriend said she got tested 3 months ago and was completely clean. she also is very trustworthy( I know many may doubt this but trust em she is) and she hasn t been with anyone else since being checked out. Is it possible for me to have an std from someone else long before that lied dormant until it was triggered by our recent spat of rough sex? I haven t slept with anyone else win over a year. Doctor: Hi, .HSV 2 is usually appeared in groups and confined to localized area and not spread over widely. They usually develop into grouped ulcers which remain for a week or so. It will not contain pus, only clear fluid. So you present lesions are unlikely to be due HSV 2 as they extended up to your pubic area and contains pus.Possibilities of hair folliculitis can be thought off. Oral antibiotics for a week or so along with some topical creams contains either antibiotic or antiseptic. Use of antiseptic soaps also would help. You consult your physician, confirm the diagnosis, rule out STDs with investigation and plan for the management. Get well soon. Dr S.Murugan"
},
{
"id": 151952,
"tgt": "Is posaconazole is available in india ?",
"src": "Patient: is posaconazole is available in india ? if yes please provide contact details Hi, One of my friend is having mucormycosis problem and I would like to know if posaconazole is available in india . So could you please provide contact details if possible. ? Please mail details on tillu.yogesh@gmail.com Doctor: No, posaconazole is not available in India. You can contact the company in USA asking for a shipment."
},
{
"id": 92359,
"tgt": "What could the reason for pain in right side of abdomen for a person having fatty liver?",
"src": "Patient: I am 43 years old female. I have pain on right side of abdomen for almost one month.(Appendix already removed 10 yrs back).USG 2 mts back showed fatty liver.Cholesterol ,triglycerides normal .Apetite & motions normal.What can be the cause of pain ? would probiotics help? Doctor: Hi!I read carefully your question!I think that fatty liver is not a reason to have pain.You should go to do :- Abdominal ultrasound- Hemogram- AST,ALT,Bilirubinemia (for liver function)- Lipazemia,Amilazemia ( for pancreas function)After them you should consultation with gastroenterologist.Wishing you all best.Dr Ilirian"
},
{
"id": 144637,
"tgt": "Are memory loss, hallucination and depression due to dementia?",
"src": "Patient: my mother has dementia, my dad has it too, but his stages were kind of a normal pace, but my mom is huliicinating, and her memory is lost , every 10 minutes or less she forgets what i told her and always gets hysterical, depressed , crying at night some nights worse than others. what stage is this shes unbelievable but is this normal dementia Doctor: there is no term called \"normal dementia\". There are many types of dementia. Behavioral abnormalities can happen in various kind of dementias. In few it can come in early stages like - frontotempral dementia and dementia with lewy body disease. and in few types it come in later stages of disease like in alzheimers disease. So its not possible to tell the stage of dementia based on hallucinations."
},
{
"id": 163566,
"tgt": "What causes fever, loss of appetite, fatigue, cough and cold in child?",
"src": "Patient: My daughter is aged 3 and been under the weather now for nearly two weeks it started off as flue type symptoms fever off food very tired. This lasted a week just over now has turned into a cold and cough she seems ok ish in the day it gets worse at night and she is up coughing all the time she is still off food and doesnt seem herself! Do you no what it could be? Doctor: Hello,All these symptoms are because of the flu. It may likely be viral fever or may be a bacterial infection.As these symptoms have lasted for 2 weeks, I would suggest that you get a chest x-ray, sputum examination and complete hemogram after pediatric consultation.For that, you can give syrup Ibugesic Plus 1 teaspoon, syrup Amox Clav 1 teaspoon and syrup Ambroxol D after pediatric consultation. Along with that you can give nebulization twice a day.Hope I have answered your query. Let me know if I can assist you further.Regards,Dr. Shyam B. Kale"
},
{
"id": 88291,
"tgt": "Suggest remedy for abdominal pains",
"src": "Patient: hi am 30yrs old and for the last month ive been having bad abomimal pains from the bottom of my breast to belly botton the doctors checked me for gallstones and liver and also pancres test came back normal no one knows whats wrong doctors are guessing Doctor: Welcome to health care magic. 1.The history and symptoms suggest possible cause in this case could be features of gastritis.2.In this case you need to maintain your diet in frequent intervals of time - could be helpful. 3.Other possibilities include - enteritis / colitis / colonic diverticulitis. In this case some dietary changes like avoiding spicy / junk / fat containing food and take more of salads / fruits / fibrous diet for better bowel moment. 4.For gastritis - you need endoscopy to see the stomach walls as diagnostic value.5.For diverticulitis or colitis you may need cross sectional studies like CT - to evaluate the cause.Good luck.Hope i have answered your query,any thing to ask ? do not hesitate to ask.http://doctor.healthcaremagic.com/doctors/dr-ganesh/62888"
},
{
"id": 129211,
"tgt": "What causes pain in legs, knees, inner thigh and buttocks?",
"src": "Patient: My legs and knees and inner thigh and even buttocks aching for two days and nights. I am very tired and a bit emotional. Been trying spiralina drink once a day, and swalk swam 4 days ago. Eases off with Panadol but comes back in a couple of hours. Feeling unwell, lethargic and had 7.1 ogtt reading. What should I do now about the heavy legs and aches. Thanks Doctor: well.... thank you for the information u have provided to us... please mention your age. weight and any co morbids... once get your fasting blood sugar and post prandial blood sugar heamoglobin and thyroid profile get done thanq"
},
{
"id": 111724,
"tgt": "What is the cause of cramping and lower back pain?",
"src": "Patient: i am 21 years old i took one depo shot nov 18th i never went back for my next shot me and my bf have unprotected sex and lastnight i had some brown discharge and then light pink spotting and this morning the brown is back is this implantation bleeding or part of coming off the depo i also get light headed and ive had cramping and some lower back pain as well Doctor: Hi and thanks for the query,The back might really not be related to the depo. Stopping the deopo means you could get back your menses. Screen for pregnancy and this could as well be related to menses coming back or pre menstrual pains. Kind regards"
},
{
"id": 117003,
"tgt": "Suggest treatment for very low HB with osteoarthritis",
"src": "Patient: I am 63 years old, 5'4 & 160. I have osteoarthritis and HTN. My family dr. told me to take 2000 mg of D a day/ then after taking my blood again tells me I need to go to a hematologist--RBC's 3.4Why did he tell me to take D (for 2 months) what does this have to do with Vitamin B-12 or Iron???--He did not tell me to increase these vitamins??? Doctor: Hi,Thanks for asking.Based on your query, my opinion is as follows.1. Increased osteoporosis and bone fracture risk is present. Vit D helps in improving calcium and hence bone strengthening.2. Improvement in hemoglobin can be through intake of Vit B12, folic acid and iron supplements.3. For osteoarthritis, NSAIDS painkillers, glucoran and chondroitin sulphate tablets are useful. Along with physiotherapy and resistance exercises, you can use medication alsoHope it helps.Any further queries, happy to help again."
},
{
"id": 213877,
"tgt": "Usages of tolaz and zepny-M tablet",
"src": "Patient: my sister with mental disorder aged 25 has been prescribed tolaz and zepny-M tablets by her doctor.what can be the name of the disorder she's suffering from? Doctor: Hello kim; welcome to HealthcareMagic Tolaz is Olanzapine is drug used to treat Schizophrenia in acute phase and for maintainence and the related psychosis.As for Zepny-M even it may be related drug but the exact information is not available as the exact content is not known. Thanks"
},
{
"id": 224050,
"tgt": "Suggest treatment for constant abdominal pain after IUD removal",
"src": "Patient: I had MirenaIUD removed due to constant pain,had laparoscopy that was normal and still had pain. I ended up having robotic hysterectomy 4 weeks ago. I have had severe pain in my right abdomen and groin also severe low back pain since. Ob doc has referred me to ortho. He prescribed a muscle relaxer. Also referred to pain clinic I see them next week. I never had problems before. Do you think it s related to recent surgery? Doctor: Thank you for the question.It sounds to me like there was some component of pain present before the surgery. It also sounds like there must have been pain present even after the IUD removal given that you went ahead with surgery. Sometimes the evaluation of pelvic pain is in an effort to answer the question \"is it GYN related pain or not\". Given that your uterus is gone and you still have the pain suggests that it is related to something else - perhaps the pelvic bones or muscles. Evaluation in a pain clinic is a good idea. Physical Therapy might also be able to help.I hope that this helps.Dr. Raichle"
},
{
"id": 208587,
"tgt": "Suggest management of a mentally retarded person with diabetes",
"src": "Patient: my brother is slightly mentally retarded, with a stubborn nature . He wont obey anything and torture others if gets violent. he is having hormonal imbalance, 40% retardation, always want to eat food, he is a having type 2 diabetes , he is obese. What can be done with him? Doctor: Hi dear,For mental retardation patient difficult to accept changes in environment.For behavior disturbance in mentally retarded patient there are many medication but choosing depend on personal profile and behavior problem.so consult psychiatrist for detail history and treatment.for diabetes you should consult physician.medication to control behavior are Mood stabilizer like carbamazepine, oxcarbazepine, sodium valproate, also antipsychotic like olanzepin, risperidone, haloperidol etc., also behavior therapy works best with medication.Thank you"
},
{
"id": 52625,
"tgt": "How long will it take to get infected with HPV?",
"src": "Patient: Hi, may I answer your health queries right now ? Please type your query here...I wanted to know how long can it take to get hpv. If I have been with the same partner for 5 years and one other before that could it take five years to get worse or show up Doctor: Hello and Welcome to \u2018Ask A Doctor\u2019 service.I have reviewed your query and here is my advice.Usually, if you infected with HPV, then the incubation period can be up to a few days to one year. But after 5 years usually, symptoms don't appear.If infected with HPV, then wart can appear over genital. If you have any clinical complaint, then kindly mention it for giving more comments.Hope I have answered your query. Let me know if I can assist you further.Regards,Dr. Parth Goswami"
},
{
"id": 219604,
"tgt": "What causes vaginal discharge and swollen ankles during pregnancy?",
"src": "Patient: I am 33 weeks pregnant and 34 weeks pregnant on Thursday. I have a mucousy discharge, swelling on my ankles, very bad back pain, cramps, and my abdomen seems soar. Also, I have pressure in my crotch. This has been going on for about a month now. I told my doctor about this but recieve no answers. What is happening? Is this normal? Doctor: Thank you for your query. The last trimester can be difficult. Your body is changing to accommodate your growing baby. The pelvic & abdominal muscles stretch, hips widen,adding more strain to your back. This puts pressure on your crotch and a constant need to be near the restroom just in case you need to urinate. Pillows are a blessing in disguise. Use them behind your back while you sit. Between your thighs and tummy when you sleep on your side to relieve pull on your back. The growing baby and uterus put pressure on the blood vessels that drain venous blood from your legs. This causes stasis and hence swelling in your ankles. Keep your feet propped up at hip level when you sit. Facial swelling, eye puffiness, persistent headaches, blurring vision, severe itching is not normal and need to be reported to your doctor immediately Occasional cramping and tightening of uterus are practice contractions called Braxton-Hicks. These may cause some discomfort. Remember that any tightening/ pain at regular intervals lasting more than an hour, a gush of water/blood could mean you are in Labour. See your doctor immediately. Your baby should stay in for at least another 3-4weeks, ideally 6weeks. Some mucus discharge is normal. A thick blob of mucus, however, with or without blood could also signal onset of Labour. Too much white discharge could indicate fungal infection (possible in pregnancy) which needs to be treated. Go for regular checkup. Monitor your baby's movements (2-5 an hour except when the little one is sleeping). Stay well rested, nourished & hydrated. Your symptoms are normally expected in pregnancy (although it varies from person to person). Just monitor for the warning signs. Here's wishing you and your baby a safe, happy and healthy delivery. Please get back to us with further queries."
},
{
"id": 203916,
"tgt": "Should I be concerned about blood on foreskin after having sexual intercourse?",
"src": "Patient: I am a 50 year old male married 32 years. For some time I've been noticing blood on my foreskin after having sexual intercourse. At times when my wife cleanses her vagina, blood stain is present in semen. At first I thought it was her having some vaginal irritation, but then I discover the blood actually comes out from around my foreskin in very tiny amounts. I've never had circumcision done and the flesh of my foreskin is quite substantial. Is this something I should be concern with? Thank You. Doctor: usually after 50 years risk of cancers and prostate enlargement are common. U need to be examined by the surgeon for any ulcers on ur fore skin to rule out skin cancer . And u should undergo U S G abdomen and K U B region to rule out prostate enlargment"
},
{
"id": 96976,
"tgt": "Is rabies injection needed after prophylaxix on monkey attack on a lady?",
"src": "Patient: my brother's wife was suddenly jumped over by a monkey yesterday, she had a scartch on lower part of leg and echymosis but no cut, by claw. She took a bath with dettol soap after that , had tetanus injection. what else should she do? Is rabies post exposure prophylaxix required? Doctor: Yes rabies vaccination is advised. Monkeys are one of the common cause of rabies in rural india. Kindly consult a nearby physician for the vaccine. Anti rabies antibodies might not be necessary but as early as possible get her vaccinated as rabies is a horrible disease with nearly 100% fatality. Why take the risk."
},
{
"id": 107073,
"tgt": "What does sharp pain in the lower back indicate?",
"src": "Patient: I have a very sharp pain in my lower back on the left hand side of my spine. It feels like a nerve but I m not sure because it doesn t go down my leg at all. It s painful to get up from a chair or bed. Ice seems to help little but of course I can t keep ice on it all the time. It is better the more active I am, which is also fine, but I also can t be active constantly....I have a really difficult time sleeping. I had a knee replacement on my right knee two years ago. Could that be attributing to it? Doctor: HiKnee replacement has nothing to do...sudden pain on movement means some pars articular small joint implication which can be assessed by an MRI. Utasof now,you may take pain meds like tramadol or Advil 2 to 3 times after meals,apply muscle relaxants locally till you seek consultation with spine doctor,if medicines do not alleviate your symptoms"
},
{
"id": 76455,
"tgt": "Suggest remedy for chest discomfort with gas issues",
"src": "Patient: Hi , myself suffering from discomfort of Chest and formation of gas. These symptoms are found since 1st July. Particularly I have felt some breathing problem during my staying at AC, or before sleeping. Plz advice me accordingly.Thanks & Regards Ramkrishna Parui Doctor: Thanks for your question on Healthcare Magic. I can understand your concern. In my opinion, we should rule out GERD (gastroesophageal reflux disease) and asthma in your case. Both, GERD and asthma commonly present together. Your chest pain and gaseous feeling are due to GERD. And your breathing problem in AC and at night are mostly due to asthma. So better to consult pulmonologist and get done clinical examination of respiratory system and PFT (pulmonary function test). PFT will not only diagnose asthma but it will also tell you about severity of the disease and treatment is based on severity only. You may need inhaled bronchodilator (formoterol or salmeterol) and inhaled corticosteroids (ICS) (budesonide or fluticasone). For GERD symptoms, take antacid like pantoprazole on empty stomach. Avoid stress and tension, be relax and calm. Avoid hot and spicy food. Don't worry, you will be alright with all these. First diagnose yourself and then start appropriate treatment. Hope I have solved your query. I will be happy to help you further. Wish you good health. Thanks."
},
{
"id": 190027,
"tgt": "Had dental abscess. Had an extraction. Having foul discharge from nose and mouth. Prescribed ibuprofen. Suggest?",
"src": "Patient: Hi, I had a dental abscess , from an upper right tooth , and I had an extraction today. I am having a large (I think) amount of foul, purulent discharge draining from my nose and mouth today. I gather that this is normal, that the abscess found a way to drain. What I am wondering though, is this - should I not be on antibiotics for the infection, as well? I am running on-and-off low-grade temps. They prescribed ibuprofen and acetaminophen for the pain, which I have been alternating every three hours as they told me, as well as soft foods, no sucking or spitting, etc. Any help at all is appreciated. Doctor: Hello and welcome to HCM forum, According to the dental history provided by you i would like to inform you that when a tooth is associated with an abscess, antibiotics have to be prescribed in order to treat infection. It is pus which is discharging and is responsible for foul odour , once the infection resolves, pus will disappear . Fever is also one of the symptoms associated with infection, currently your medication does not include antibiotics, therefore kindly see your dentist. Perform warm saline rinses(3-4times/day), it will help relief symptoms. use an antibacterial mouth rinse(twice/day). i hope i answered your query, i wish you good health."
},
{
"id": 114829,
"tgt": "How should hypoglycemia with blurred vision, muscle twitching, dysgeusia be treated?",
"src": "Patient: I was diagnosed with Hypoglycemia last year, which explained my blurred/tunnel vision when I stand or sit up, but I still have muscle twitches, weird tastes and smells is my mouth and nose, and I often feel detached or out of my body, I also have crazy thoughts. Doctor: My advice is to also get the following tests done if they have not been done before. The tests are serum C- peptide, Serum insulin , Insulin/ C peptide ratio and an abdominal CT scan. It is important to rule out the possibility of an insulinoma which is an insulin secreting tumor which causes hypoglycemia. Please note that insulinoma is a rare tumor . However it is important to rule out this type of tumor in patients with hypoglycemia.Regarding the fatigued feeling & other symptoms of hypoglycemia you may try to eat frequent small meals with complex carbohydrates such as oat meal ,ragi etc which sustain blood sugar levels for a longer period of time. Best regards"
},
{
"id": 99019,
"tgt": "How to treat asthma?",
"src": "Patient: I had surgery about 8 or9 yrs ago for a hiedal hernia in my stomach. My esphagus was damaged and for the last three weeks I ve had a dry spot in my throat. I caugh and I won t to throw up but nothing comes up. For awhile I felt like I had a ball in the middle of my throat it longer feels like I have the ball. The caugh is still their but irritating. I have asthma and when I needed it irritates and makes it worse. Since my surgery my voice changes sometimes I loose my voice. Doctor: it might be gastro oesophegeal reflux disease. due to weakness of gastro esophegeal spincture reflux of acid from stomuch will induce choking sensation in chest and throat. it will also provocate asthma.Avoid lying down position for at least 90 minutes after meal. omeprezole or pentaprezole or rabeprezole for five weeks will help. local anasthetic plus antacid preparation for four time a day will also help."
},
{
"id": 23707,
"tgt": "What causes spasm and pain in left side of chest?",
"src": "Patient: I am a 30 year old female I have been having some discomfort on the left side of my chest I did see a cardiologist and had a ekg & stress test done he thought the discomfort had nothing to do with my heart but it will not go away I also fill a fullness in that area that seems to move on its on it also seems as if I'm having spasms I did have two c-sections one in 06 & another in 2010 please help what could this be it is not painful just bothersome what should I do Doctor: Hi,The pain may be caused either by gastrointestinal or spine problems, or it may be just a muscle pain .I would advise you to take Ibuprofen 400 mg three tomes a day after the meal along with Pantoprazole 20 mg, two times a day 30 minutes before the meal for 7-10 days. If it doesn't help, then you should see your doctor to undergo some examinations to find out the cause.Take careCome back if you have any further questions"
},
{
"id": 107606,
"tgt": "What causes nausea , backache and cramping pain?",
"src": "Patient: Hello, for the past 2 weeks, I have been experiencing nausea, lower backache and cramping. My period started a day late, but it was only 2 days (rare) and yesterday I was potting a pikish color. Today my stomach has been feeling extremely tight and bloated and it s quite uncomfortable. I already have 2 small children, so I ve been pregnant before but I was just wondering what could be going on? I am also still urinating every hour and I have acheiness in my lower left abdomen. Please help Doctor: Dear- thanks for using our service, I read your question with detail and will go over your specific concerns.First of all, your symptoms could be secondary to an urinary tract infection due to nausea,back pain and cramping. You have been with bloating too, which can be part of the Uti. It is advisable to get a sample of the urine for culture and sensitivity and start antibiotic treatment if the urine shows RBC and WBC. I advise you to drink at least 16 oz of water and cranberry Juice.I hope that my advise has been helpfulDr.JC"
},
{
"id": 152266,
"tgt": "Suffering from paranoid schizophrenia. how to cure it ?",
"src": "Patient: sir my sister is suffering from paranoid schizophrenia .how to cure her .her age is 17.she is not sleeping not having bath and not listening to our words what we r saying. she is just lying in the bed and talking to her self Doctor: sir is this brain diseases r mental r psycho behaviourised r constant thinking .can medication leads them to normal life.she should take medication through out life. how to bring back them to normal life."
},
{
"id": 123755,
"tgt": "Is sore elbow normal if got hit on elbow ?",
"src": "Patient: About 2 weeks ago I fell on my elbow rollerblading. There was a large gash and swelling. The swelling finally went down but now the elbow is still sore when I touch it and it looks like the elbow is sticking out more - is this normal - YYYY@YYYY - Chris Doctor: Hi, No, it is not normal. I suggest you get an X-ray of the elbow and see an orthopedic surgeon for the needful else you can miss some injury which can trouble you later on. Hope I have answered your query. Let me know if I can assist you further. Regards, Dr. Gopal Goel, Orthopaedic Surgeon"
},
{
"id": 44215,
"tgt": "Done seminogram. Is my report okay? Any deficiency?",
"src": "Patient: Hi. I am Vivek from Delhi. I had my seminogram test today. Below is my report. Please let me now if the reports are OK or do I need some consultancy for any deficiency ? Below is the report : Time of Specimen 3:45 Time of examination 4:15 Duration of Abstinence 4 days ( 3-7 ) Liquefaction 30 min ( 30.00 - 60.00 ) Volume 3.00mL ( 2.00 - 5.00 ) Appearance Viscid Opaque Colour Whitish Viscosity Thick pH 7.5 ( 7.2 - 7.8 ) Microscopic Examination Total Sperm Concentration 52.00 million/mL ( 20.00 - 150.00 ) Percantage Motility 60.00% ( 50.00 -90.00 ) Grade A 20.00 % Fast progressive Grade B 25.00 % Slow Progressive Grade C 15% Local Motile Grade D 40% Immotile Agglutination Negative Negative Longevity 2 hours Pus Cells 2-3/hpf Nil Red Blood Cells Nil Nil Epithelial Cells Nil Nil Morphology Normal Morphology 45% ( 30.00 - 90.00 ) Abnormal Morphology 55.00% (a) Head defects 25.00% (b) Neck & midpiece 20.00% (c) Tail defects 10.00% Chemical Examination Semen Fructose , Qualitative Positive Others Nil Doctor: HI THANKS FOR CHOOSING HEALTH CARE MAGIC, your report is normal, all the values are in normal limits so far i dont think there are any deficiencies here i give you few tips to maintain such healthy levels: take good and balanced diet avoid fast foods avoid fatty and spicy foods reduce intake of processed and canned foods reduce foods containing preservatives decrease the usage of electonic gadgets like cell phones etc do not place laptop on your lap for long hours you may take multivitamin capsules for vitamin supplementation anti oxidants rich foods or supplements helps in reducing abnormal forms consult your doctor for using them hope i answered your query feel free to contact me for further queries thank you"
},
{
"id": 179047,
"tgt": "What could white chalky stain discharge in a 4 yr old boy indicate?",
"src": "Patient: Yes, I just discovered a white chalky stain in my 4 yr old sons underwear. He bathes daily, I have taught him to wash his genitals and I supervise to be sure he washes everything . He is gentle but through. He has not complained of any pain or anything, no signs of redness or swelling etc. I have noticed since being potty trained that he does not seem to urinate very often for as much liqued as he drinks and he does drink alot of calcium enriched orange juice. Any ideas? Doctor: Chalky discharge in underwear is calcium cystals and nothing else. if the child is not having any problem then there is not much to worry. try to estimate 24 hour urine output and get a Urine calcium creatinine ratio done. If they are normal you have to get concerned. Tell him to drink plenty of water all day long."
},
{
"id": 185650,
"tgt": "What causes small bump with white center on the roof of the mouth?",
"src": "Patient: I ate breakfast this morning and my tounge brushed the roof of my mouth and I noticed a hard bump. When I got a closer look at the bump I was small but it had what looked like a white center. I kept messing with the bump with my tounge until some jelly like materials came out . What is this? Doctor: Helloi have gone through your query. a bump on the palate (roof of mouth) can be due to various reasons:Irritation due to foodinflammation of incisive papillassmoker's palate (nicotine stomatitis)Torus palatinus or ExostosisMucous CystsCanker Soresplz mention your age.do you have any habit of smoking?irritation can occur due to burning after drinking or eating something that was too hot or by eating very spicy food. trauma to the tissue can cause micocele hormonal changes, poor diet and stress can also cause cancer sores.if i were ur treating doctor i wud suggest you to watch the bump for 3-4 days to see any change.quit any bad oral habitavoid taking too spicy and hot foodrinse your mouth with a antiseptic mouthwashif the condition persist you sud visit a dentist to check for the other possiblitiesHope this ans helpsRegardsDr. Shesh"
},
{
"id": 7344,
"tgt": "I have PCOS problem, can any online doctor tell me about my pelvic scan report ?",
"src": "Patient: Hello Doctor I am 27 year old and have been trying to concieve from past one year. 6 months ago I came to know I have bilateral polycystic ovaries .I took ginette 35 as per my doctor s advice for 3 months to regulate my period. After that my period occured delayed by 10 days. Is it normal or do I need to take those tablets for some more time? My pelvic scan report says: 1. uterus is normal in size. 2. Endometrium appears normal. 3.both ovaries appear polycystic. 4.no evidence of free fluid seen in the pelvis 5.no evidence of adnexal mass is seen Please let me know can I become pregnant in future?? Doctor: hi welcome to healthcaremagic yeah u can definately concieve..continue taking those medications..follow your doc advice..over a period of time u will have normal periods"
},
{
"id": 111737,
"tgt": "What is the cure for lower back pain without any side effects?",
"src": "Patient: my wife is suffering with her lower back when she has stood doing her jobs around the house she has had lower back trouble in the past but has been ok for some time now is there a recommmended pain killer that has no side effects she is 72 and is not on any hp/t tablets Doctor: Hello, Thanks for your query.The following measures will help with the pain\u2022 Lie on a hard bed.\u2022 Anti -inflammatory drugs like Tablet Motrin 1 tablet as and when required ( do not repeat a dose before 6 hours) with food will help relieve the pain. \u2022 Avoid forward bending.\u2022 Correct your posture while sitting\u2022 Avoid prolonged sitting \u2022 Avoid strenuous activity & lifting heavy weights.\u2022 Apply diclofenac gel on the affected area. Warm compresses will also helpI do hope that you have found something helpful and I will be glad to answer any further query.Take care"
},
{
"id": 180615,
"tgt": "What does swelling on the gums after tooth extraction indicate?",
"src": "Patient: I had a tooth extracted 10 days ago and I went to the dentist 3 days ago and he said I was healing really well. Today I am quite sore and I just loooked and it looks like the side of my gum is splitting and a little swollen, Is this normal? I understand a split on the top is normal since that\u2019s where the gum is growing over the bone graft, but on the side seems a little strange to me. Doctor: Hello and Welcome to \u2018Ask A Doctor\u2019 service.I have reviewed your query and here is my advice.As it is already 10 days so the gum must have undergone initial healing by now. So by now, the soreness and inflammation should have subsided. However, at times there can be injury to the healing area with any food that is hard and can cause inflammation. Food lodgement in that area can cause slight infection. So my suggestion is to do warm saline gargles and antiseptic mouthwash gargles. Do not let food particles accumulate in that area. Take anti-inflammatory painkillers like Advil.If it does not improve in 2 to 3 days then get it checked by your Dentist. Hope I have answered your query. Let me know if I can assist you further.Regards,Dr. Honey Arora"
},
{
"id": 28626,
"tgt": "Are Diflucan, Lamisil and Fluconazole effective in the treatment of fungal infections?",
"src": "Patient: Is the Diflucan and Lamesil and fluconozole pill what is used to treat nail fungi and does it clear up most nailbed infections? My chemo doctor stopped prescribing one of the pills for me after my last chemo and until then it had been helping clear up my nails. Now he passed me off to a pediatrist that told me once you get fungi of the nails then you always have it - (not true?) and he wouldn t prescribe me a pill either. So I been using topical and it doesn t work. I took a urine infection test at home and my leukasites were positive for infection. Not even the dermatologist at Stanford would give me a pill. The pill cleared up my nail infections until the doctor quit prescribing it and I think a low grade nailbed infection can give me cancer eventually? The website said I get a free question then after writing it and submitting there were several options to pay. If it is free like it said, the website, then you can send me an answer via my email please. YYYY@YYYY Doctor: Hello and Welcome to \u2018Ask A Doctor\u2019 service. I have reviewed your query and here is my advice. You are right nail infection caused by fungus can be cleared most of the times, and the mentioned drugs are effective in clearing it, and mostly needed to be taken orally however prolonged course is needed like it may be needed for up to 2 months to clear the infection. Hope I have answered your query. Let me know if I can assist you further. Regards, Dr. Mahboob Ur Rahman"
},
{
"id": 17477,
"tgt": "What can cause CAVSD condition for a 6 month old?",
"src": "Patient: MHV (mitral valve) is in my son, he got diagnosed with CAVSD, on anticoagulant every day and has had 3 major surgeries at 5-6 months of age. Now he is 16 months old, doing beautifully, developing nicely, his doses of anticoagulant is settled and he is tested in every 4 wks for INR, ECHO and ECG. He has 2 paravalvular leaks, both minimal that cause no problems. He was born at 42 wks, 3840g, 51cm, natural birth with no complications. All previous scans show no abnormality in his heart. I took no drugs, had no viral or any other infection at gestational 4-6 wks when the heart forming, noone in our families have any heart deficiency. I didn t work near any radiation and noone could tell me how come I have a healthy looking son with such chronic illness without any syndrome, any other health problems. Do you know how could this happen? Doctor: Hi, See there are genetic, environmental factors and sporadic factors or at best the congenital heart disease may have an ill-defined etiology. Once corrected early, if the other organ development is as par with a normal child, he should do well. Hope I have answered your query. Let me know if I can assist you further. Regards, Dr. Swarup Pal, Cardiac Surgeon"
},
{
"id": 95026,
"tgt": "Constant stomach pain. More after eating and passing stool. Endoscopy result normal. What is causing pain?",
"src": "Patient: Hi, My wife has stomach pain in 5-cm above the navel button and 5-6 cm to the left side from the navel button since january 28. I feel constant pain in that area 24 x 7, but after eating something it gives more pain, and after passing stool it pain more, it pains even if I drink water. I have lost 8 Kg since January 28, 2012. I had Endoscopy last week but it was clear, no H pylori Infection, no Ulcer , they took Biopsy sample, i am awaiting forthe results to come.... Can Anyone of you please tell me whats wrong with my stomach Doctor: Hi girish, Thanks for writing in. Although your symptoms are classic of peptic ulcer disease, the fact that the endoscopy was normal , points against it. I would still recommend a trial of oral proton pump inhibitors like omeprazole for 2 weeks and see whether it helps. Please talk to your doctor about this. also, I would like to rule out tuberculosis in your case as well. I would suggest that a MRI of the small and large intestine be done so that we can rule this out. Hope this helps Regards"
},
{
"id": 51336,
"tgt": "Ultrasound shows increase in size of kidneys. Have UTI. Reason for increase in size?",
"src": "Patient: Hi Doc, My dad was diagnosed with UTI and physician asked him to go for ultrasound . His ultrasound results shows that his Right kidney is 91*35mms and Left kidney is 97*40mms. The Report says there is an increase in size of kidneys. Hence the physician asked my dad to consult a Nephrologist . Could you please let me know what could be the cause for this increase in size? and is this increase is normal with age? Doctor: Hi, Thanks for query, You have not mention about corticomedullary difference ?is this maintained. Due to kidney diseases the kidney shows enlargement in size but cortex & medula both enlarged or only single one is an important aspect. A general rule is that small kidneys depict chronic renal disease and enlarged or at least normal sized kidneys depict acute and therefore potentially reversible disease. A further distinguishing feature is parenchymal echogenicity. Diseased kidneys in general show increased echogenicity, however in the early stages echogenicity may still be normal. Increasing echogenicity is directly correlated to histopathological abnormality. acute obstructive disease ,acute pyelonephritis or renal failure may be the possibility. The S.creatinine level is a better tool to measure kidney function. Thanks."
},
{
"id": 75275,
"tgt": "What causes yellowish brown mucus during cough with blocked nose?",
"src": "Patient: Hello, I am 17 years old, female and a smoker. I don't usually suffer with coughing, but recently i have developed a very chesty cough along with a blocked nose. I am coughing up dark yellow + brown mucus. I have had this cough for around 4 weeks now, and it seems to be getting worse. what could it be? Doctor: Hello and thank you for asking in HCMYou explain that you have cough with yellow phlegm since 4 weeks.and given the fact that you are a smoker it would be better to do some blood tests to rule out any infection of the lungs,and a Spirometry .Hole blood countCRPALT,ASTChest X RayX Ray of the sinusesIf these tests are positive you should have a medication antibiotics mucolitis some vitamins and simptomatics and you should be better.I strongly recommend you to stop smoking.Thank youDr.JolandaPulmonologist"
},
{
"id": 89387,
"tgt": "What causes intermittent burning like abdominal pain radiating to thighs?",
"src": "Patient: I have been experiencing intermittent right sided abdo pain for a few wks with no urinary symptoms or change in bowel habits/vomiting/nausea/wt loss etc I am 54 years old and have a Mirena coil which was changed 18 mths ago. A recent pelvic and vaginal scan in Aug 2011 for similair symptons was normal as were my bloods and CA125. The pain feels like an interntal burning sensation and can radiate down into my right thigh and sometimes my back. Any suggestions? Doctor: Hi. The burning character of the pain and the history of radiation to the thighs suggest me to think of Neuralgic pain originating either from the Retroperitoneal area or from the spine. I would advise you an MRI of the spine and the posterior abdominal wall to see for Destructed vetebrae due to cancer or tuberculosis, Compression on the spinal cord or affection of the ilio-psoas muscles . Also go for regular stool, urine and blood tests including quantiferon gold test for TB and so on. The treatment will obviously depend upon the diagnosis one gets"
},
{
"id": 108826,
"tgt": "What is the treatment for severe back pain?",
"src": "Patient: Hi, may I answer your health queries right now ? Please type your query here...hi i have a back side pain on the right side think its the kidney area,,,comes and goes for a long time now..did ultrasound that was normal and urine analysis was normal...what investigations should i do or what may be causing the pain? Doctor: HIWell come to HCMI really appreciate your concern, this pain could be due to muscular spasm and may be due to poor posture, and walking manner try to maintain good posture and for the pain this can be best managed with \"Tab Diclofenac 50 mg once in day\" try to away from stress, take care."
},
{
"id": 49316,
"tgt": "What is the cause for frequent stabbing pain in the groin area with a history of uric acid kidney stones?",
"src": "Patient: HelloToday, out of the blue, I was sitting at my desk and go a very sharp stabbing pain, near my left testicle.It is not in the testicle, actually up in my groin. It felt liked I was being poked with a needle. I got up and rubbed the area and walked around . . . and the pain went away. SInce then I have gotten the sharp pain about every couple of hours. It comes with no warning. The pain only last about 5 - 10 seconds, but hurts tremendously. It seems that moving helps get rid of the pain, but don't know for sure. The pain makes me have to move and walk.I just saw a dentist yesterday because of a tooth ache. He said I need a root canal and prescribe amoxicillin for the infection. Could this be related to the new stabbing groin pains ?Also, I have had uric acid kidney stones. Doctor: Hi. From your history this looks to be a stone impacted or trying to pass through the ureter on the same side of the pain. No relation to amoxicillin. Get the urine test and sonography done, Visit a Doctor for clinical examination as to see there is no hernia or so."
},
{
"id": 208024,
"tgt": "Suggest remedy for mental health problem",
"src": "Patient: i feel like my brain is telling me to walk to one side im feeling dizzy at times . I saw a MD and now she is sending me for a MRI. 10 yrs ago i had a menningoma removed and had scan every year with no regrowth. To ruled this out, the doctor is sending me for a MRI. YYYY@YYYY Doctor: HiiYes it is necessary to look for any secondary or regrowth.MRI will best suggest it.To take any further step MRI is essential.So go for it and do according to its report.Revert back with report or consult your doctor for further assistance.Thank you."
},
{
"id": 118237,
"tgt": "What is the cause for the drop in blood pressure?",
"src": "Patient: 37 y.o. male 5 7 180lb Very fit...work out 6 days a week (running,biking weightlifting) Started juicing(smoothie),second day. Experiencing a drop in blood pressure-105/60/60 Usually it s 120/70/72 What is the problem? P.S.For juicing used broccoli,banana,apple,carrots,avocado,kiwi,blackberry Doctor: hi, Though it may not be related, vegetable juices may reduce the blood pressure by regularizing your metabolism, reducing blood sodium levels and increasing potassium levels, which is healthy. if it is not causing any symptoms, you need not worry. if there are symptoms like fatigue or giddiness, you need to be liberal in increasing salt intake.all the best."
},
{
"id": 222407,
"tgt": "Is loss of mucus plug during pregnancy with bowels a normal occurrence?",
"src": "Patient: i am 36+3 days pregnant with my 3rd child, this pregnancy is completely different to my others. i am having bowel movements every morning, which is ab normal for me, i am feeling very tired and nauseous, i lost my mucus plug a week ago today, and im feeling an awful lot of pressure between my legs is this all normal? Doctor: Hi dear, I have gone through your question and understand your concerns.Loss of mucus plug and pressure between the legs in the lower abdomen is suggestive of the starting of labour.I will suggest you to consult your doctor as soon as possible for proper examination and treatment accordingly.Hope you found the answer helpful.Wishing you good health.Dr Deepti Verma"
},
{
"id": 75961,
"tgt": "Why am i suggested for apicolordotic view?",
"src": "Patient: I had my xray on my chest and the diagnosis is I have an ill-defined densities in the right apex and reticular infiltrates are seen in the lung bases. The impression is I have a bibasal pneumonitis, can you please explain to me what is this and why they suggest for me to have a apicolordotic view? Doctor: Thanks for your question on Healthcare Magic. I can understand your concern. Normal chest x ray is taken in posteroanterior view (PA view). In this view, lung apex (upper part) is not clearly visualized because of overlapping ribs, clavicle and lung shadows. So any opacity or lesion in apex is not properly visualized in normal PA view. So exact opinion about the etiology is not possible. To overcome this issue apicolordotic view is needed. In this, chest x ray is taken in such a way that apex of th lung is clearly visualized without hindrance of ribsribs and clavicle shadow. Since you are having right apex ill defined densities, apicolordotic view is demanded by radiologist for proper reporting and diagnosis. You are also having bilateral lower zone infiltrates, so possibility of bilateral lower zone pneumonia (lung infection) is more. Get done apicolordotic view for diagnosis of right apex ill defined densities. Hope I have solved your query. I will be happy to help you further. Wish you good health. Thanks."
},
{
"id": 104836,
"tgt": "Suffering from amoebaiasis. Using ciprofloxacin, metronidazole. Alternative medication?",
"src": "Patient: I am suffering from amoebaiasis for last four days . I am 36 yrs male ; body wt 70 kgs . Iam using tab Ciprofloxacin 500 mg only . I have strong allergy on Tinidazole / Metronidazole . what should be my alternative drugs for cure. Please suggest me. Doctor: Hi bandow, Welcome to healthcare Magic! How were you able to know that it is Amebioasis. If proven by stool examination then you must get a proper treatment under the supervision of a qualified physician. There are many drugs apart from tinidazole/metronidazole which are effective against amebiasis like Quiniodochlor, Diloxanide Furoate etc. to name a few. The dose and regimen your clinician would know. I do not agree with your choice of taking ciprofloxacin for amebiasis, you are harming yourself by self medication. I hope the advise would be informative and useful for you. Take Care!"
},
{
"id": 133792,
"tgt": "What causes intermittent pain and large mass between the ribs?",
"src": "Patient: I have been having intermittent sharp stabbing pain under left ribs and just noticed left rib protrudes quite a bit more than right ribs. Also, have noticed, what I believe is a hernia/large lump/mass between my ribs, upper abdomen. What could this be Doctor: hi,thank you for providing the brief history of you.a thorough neuromuscular assessment and respiratory assessment is advised.As the chances of hernia in that area of upper left quadrant is less you need to get a hands-on assessment post which a diagnostic ultrasound scanning will help further to assess if any thing in the abdominal region is there will be detected.I advice you to get the abdominal scanning done and see what the other inputs can be providedRegardsJay Indravadan Patel"
},
{
"id": 114718,
"tgt": "Suggest remedy for elevated WBC in blood",
"src": "Patient: Husband had an endoscopy during the procedure found white spots. Took a biopsy and thought it was fungus. Found out today not the case. White blood count low told might be an infection. Not for sure needs another scope in a few months. Taking meds for acid reflux. What could it be? Doctor: Hello thank You for contacting HCMWell first of all I must say there's too little details provided to work on however let me try__ 1. your husband's having acid reflux and on meds - since when? 2. Did he undergo endoscopy for that or there was some other problem(what was the problem that he's facing lately)?? 3. Is he having any other problems lately (fever, flu like symptoms, loose motion, pain abdomen etc. )?? 4. Apart form the meds for acid reflux, is he also taking any other medications for any other problems(Diabetes, Hypertension, Asthma etc.)?? 5. You've mentioned White blood count low - its better if you kindly attach the report of white blood count and also the defferential count along with next time it would be better 6. Does he smoke??Either way do check in with your gastroenterologist and also get back to me with some more details next time for actually telling what it could bep.s Havn't you got the report of the Endoscopy?? if yes then please attach along with in your follow-upHope to see you soon, thanks a lot for posting your query in HCM"
},
{
"id": 159983,
"tgt": "lung cancer survivor status",
"src": "Patient: I am diagnosed of lung cancer 7 years ago. Now, with God's grace I'm feeling all right. am i cured already ? Doctor: Hello, welcome to HCM. Good to know that you are feeling healthy. But the real condition of cancer can be ascertained only from your present and previous work up reports. So your doctor would be in a better situation to inform you about that. Wish you good health."
},
{
"id": 146890,
"tgt": "Suggest treatment for vertigo",
"src": "Patient: I am suffering from vertigo problem due to the early evidence of irritation in right vestibular end organ, which got detected after ENG test. My life has become hell as I have been suffering from vertigo for last 10 months and visited several Doctors but no soultion found till date. Please suggest medicine or remedies Doctor: HIWell come to HCMI can understand your worry but this is nothing to worry because the symptom of vertigo could be well controlled with Tab Cinnarizine 25 mg three times in day some time steroid can be tried nut for that you need to discuss this with your ENT surgeon because this needs some test result and clinical evidence, hope this helps."
},
{
"id": 69909,
"tgt": "What does small pea sized bump inside bellybutton indicate?",
"src": "Patient: I randomly noticed a small pea sized bump inside my belly button. It is blue in color almost like a bruise and is hard. It does t hurt unless I touch it but even when I do it isn't horribly painful. What could it be? I ruled out a hernia, but that still be it? Doctor: Hi. This is not hernia, otherwise it could have caused you tremendous pain. This is called an umbilical granuloma. Needs to be excise by a Surgeon if you want a permanent cure."
},
{
"id": 72151,
"tgt": "How can severe chesty cough while having pulmonary hypertension be treated?",
"src": "Patient: My mother is in the final stages of pulmonary hypertension a complication of her CHD. She was discharged from the hospital a week ago into hospice care and told the family to expect her to live no more than a couple of weeks. She has rallied somewhat. She is responsive and her breathing has eased. However, this mucus producing cough, a symptom of the hypertension I am assuming as she has widespread edema, is taking its toll on her. Would it be appropriate to give her an OTC cough suppressant or ask the doctor for something a little stronger? Doctor: HelloI do not think that they might help as much in such condition.However you can try them no worriesRegardsDr.Jolanda"
},
{
"id": 143956,
"tgt": "Does abnormality during pregnancy cause multiple sclerosis?",
"src": "Patient: Suppose someone argued that the reason twins often share multiple sclerosis is not because it\u2019s hereditary, but because of some abnormality in the mother\u2019s pregnancy such as maternal exposure to a toxin or virus. What argument could you give against this hypothesis? Doctor: I don't think so. MS is acquired primary demylination of central nervous system. Environmental factors do play role in its caution eq it's more common in western countries with low sunlight than India. Some other unknown factors including viruses also play role."
},
{
"id": 1859,
"tgt": "Are there pregnancy chances with non penetrative sex?",
"src": "Patient: doc please help me ! me and my boyfriend were kissing and we both had our pants off, he ejaculated about 10 minuites before we started kissing. we didnt have sex, were both virgins, but im afraid some of his cum got on the outside of my vagina. is there a possibility i could be pregnant ? Doctor: Hi, if anything has touched your vagina, there may be chance of pregnancy. So better take some medicines to prevent pregnancy."
},
{
"id": 106432,
"tgt": "What condition requires cardiopulmonary resuscitation ?",
"src": "Patient: A. stroke west nile virus cardiac arrest asthma Doctor: Cardiac arrest requires immediate CPR if necessary ACLS or ALS(Advanced Life Support). However in case of Bronchial Asthma if a patients develop Respiratory fatigue and his Oxygen concentration in blood starts falling put him on a ventilator, but wean him off as early as possible OR if he develops Cyanosis CPR + ALS. There are other protocols for all the above disease but you stick to cardiac arrest and asthma. Don't forget Cardiac Asthma, severe cases may requires ACLS. So it is the indication and symtomology of the patient that gives rise to CPR, ACLS etc. no hard and fast rule."
},
{
"id": 4225,
"tgt": "Suggest precaustions before conceiving",
"src": "Patient: Hi, I am 31 yrs old, IT professional, Had miscarriage 6 months back. Should I take any test before I get conceive the next time. Could you please let me know if there are any precautions that I need to take. I do not want to be in such a position again. Please advice Doctor: HiThank you for asking HCMIt is better to have a physical examination including pelvic examination by a gynecologist.If necessary an ultrasound examination also should be done to find the status of uterus.If your uterus is clean you can concieve.It is also advised to start folic acid tablet 3 months before concieving.It is also good to check thyroid function and hemoglobin level.Hope this may help you.Let me know if you have any further query."
},
{
"id": 102164,
"tgt": "Is taking Prednisone essential for the periodic attacks of asthma and Sinusitis?",
"src": "Patient: ok, I have asthma and sinustitus, I went to the doctor because I was not feeling very well and my specialist gave me 7 tablets of prednisone 50 mg for 7 days , I have three more doses to go. I am getting better slowly. I was wondering why I need to take this drug, periodically when I get sick like this. Thank you. Doctor: Hello,Welcome to HCM,As you are a known case of asthma for which your doctor has prescribed prednisolone 50 mg for 7 days, you are also known to have infection of sinus.Your doctor has prescribed the oral steroids for your symptoms of asthma, because it will reduce the edema caused by the allergens and it helps to dilate the bronchioles which helps to relieve the symptoms of bronchospasam. As your doctor has prescribed these medicine you have to take it for 7 days and you cannot discontinue abruptly.For sinus infection you may require the course of antibiotics, so i would suggest you to takeOral Antibiotics like Tab Bactoclav, 625 mg, twice daily for 5 days.Oral antihistamines like Levocast for 5 days.Thank you."
},
{
"id": 91527,
"tgt": "What is the treatment for stomach ache?",
"src": "Patient: I'm having a severe stomachached I feel a hard ball by my right abdominal toward my lower both back I have been feeling this for almost 6 months now I had a urine blood pelvic ultrasound and right stomach ultrasound all came out negative and I got monthly period but my symptoms is feels like I'm pregnant but my stomach hurts all the time the only thing it makes me feel better if Igot to sauna what could this be? Doctor: HI. This can be a form of a entero-colitis that is inflammation of small and / or large bowel... Get a proper diagnosis by barium studies / Enteroclysis. Many times the colonoscopy can help a lot. I hope you do not have anxiety symptoms. These too can give symptoms of like having pregnancy ."
},
{
"id": 220268,
"tgt": "Is regular anesthesia safe during pregnancy?",
"src": "Patient: Hello, I am 5 mos pregnant and I am due to have a wisdom tooth pulled due tp the migraine headaches its been causing me.... I am not getting put to sleep the dentist is just giving me regular anesthesia I want to know will the anesthesia harm the baby??? Doctor: Hallow Dear,Any dental issue should not be ignored during pregnancy, particularly infections to avoid further complications. However you seem to have impacted wisdom tooth which needs extraction. Most of the time, this procedure needs to be performed under general anaesthesia. Usually, dental exteactions are considered to be comparatively safer during mid trimester; i.e. between 14 weeks to 28 weeks of pregnancy as during this period, the pregnancy is more stable and the uterus is less irritable. Now you are in the 5th month of pregnancy; i.e. in the second trimester. So if the wisdom tooth needs extraction, this is the most suitable time for its extraction. I hope this helps you to take decision.Dr. Nishikant"
},
{
"id": 214351,
"tgt": "Suggest home remedies to reduce abdominal fat",
"src": "Patient: hi i am unmarried girl and have thyroid problem ,which is under controll as i take 50 mg thyronorm tablet every morning and i also have polycystic overies ..my problem is i have a big belly which looks as though im carrying..im upset as im not able to get a flat tummy .i have tried yoga ,excercies but did not help....can anyone suggest what can be done to get rid of belly fat..if possible any effective home remedy. Doctor: Reduction in Abdominal fat with Ayurvedic medicines or home remedies is a myth, if you are looking for a shortcut. The half answer is in the question itself .PCOD and Thyroid imbalance, both are metabolic disorders, both related to Fat metabolism and weight.This problem has to be treated with Invasive Ayurvedic therapies known as Panchakarma and a long time use of Ayurvedic medicines which can go up to 3 yrs. As you are you are unmarried, take this serious now itself because it may also more problems in future during pregnancy or postpregnancy"
},
{
"id": 9624,
"tgt": "What is a suitable treatment for dry skin ?",
"src": "Patient: dry skin Doctor: Kindly complete ur query dry skin needs daily application of moisturiser after bath n b4 sleep n drink lots of water. Thnx"
},
{
"id": 213194,
"tgt": "Panic attacks, stress, talking fast, slurred speech. Causes and treatment?",
"src": "Patient: i was feeling fine the other day i have panic attacks ive been overwhelmed with to much stress i was talking fast and slured my word alittle for about 2 or 3 min then i had someone on my mind i knew who i was talking to but i said the other persons name and then i really panicke what can i do to stop these attacks i worry for no reason Doctor: start taking paroxetine 25 mg in night. do deep btreathing relaxation exercices.. do regular aerobic exrcise and morning walk it will help you.."
},
{
"id": 113603,
"tgt": "Severe upper back pains, chest problem after delivery. Related to delivery?",
"src": "Patient: Good day to all of you here. I would like to ask about my problems. i felt upper back pains and a chest problem 8 months after my delivery. Is there a connection with my delivery? It s now 11 months after giving birth and the pain is worsen. It is right to go for an obgyne for a check up? or to another specialist? This pain really affect my work.... Please help me. Doctor: Hello renee, Welcome to the forum. You have a very common problem that is faced by many women post- partum. Do not worry. It is nice to know that you have already recovered and doing fine. I think it is time for you do start your regular exercises under an expert. You can even perform yoga under the supervision of an expert adviser. Take calcium and mineral tablets and food rich in vitamins anti-oxidants, fiber and low in lipids and carbohydrates. Look to your posture, never lean forward while working or standing. Take care of yourself. A hormone called relaxin secreted ( in excess ) during child birth may be a cause of your backache. As your hormone balance gets back to normal you will get fine. Yes, you need to see your gynecologist and get clinically examined and have all investigations and radiology done to rule out any causative factor which may be responsible for the pain. You need routine blood tests to rule out generalized weakness. Have agood day, take care."
},
{
"id": 138061,
"tgt": "Can Onglyza intake cause muscle spams?",
"src": "Patient: I was wondering if nighttime muscle spasms in the leg can be caused by Onglyza. I have started taking onglyza for a few months now. Granted, it has been quite hot and I gave not been drinking much water. But still, this is the first time I gave ever experienced them.Have, not gave My email is YYYY@YYYY Doctor: Onglyza, an antidiabetic medication might cause swelling in swelling in your hands, ankles and feet, but that is an form of allergic reaction to this drug and if that happens call your doctor right away. As per your concern the nighttime muscle spasms are not a side efffect of onglyza.thank you."
},
{
"id": 209189,
"tgt": "How to overcome the withdrawal symptoms of alcohol?",
"src": "Patient: I am a recovering alcoholic and was prescribed Campral after being clean for 60 days. It made my craving unbearable and to be honest made me walk around like I was highly intoxicated my eyes could not focus I was dehydrated nauseous. My eyes were tired but my body felt awake. I am trying to find how common this is? Doctor: HiThanks for using healthcare magicCampral contains acamprosate and it is a anti-craving agent. It has few side effects like drowsiness, anxiety symptoms, nausea etc. These symptoms would stay for few days and decrease with time. In case, it stays, then you can talk to your psychiatrist to shift on another anti-craving agent like baclofen. If you need further help, you can ask.Thanks"
},
{
"id": 165121,
"tgt": "What causes child to have high fever at night?",
"src": "Patient: Hi, may I answer your health queries right now ? Please type your hello my son is 3.5 years old he has high temperature for the past week...and we have been giving him medicine...when he gets up he is ok during the day but at night he consistently gets high temperature...his ears release dirt and his eyes now...what is wrong with him Doctor: Dear parent,you have not provided details of other symptoms, medication and also how much is fever at night? Are you giving OTC meds for fever?One week is enough time to wait on your own. I will suggest to see the nearest pediatrician or a doctor. Because such a fever can be due to many things, depending on your location. He may need a few investigations too, to come to a diagnosis."
},
{
"id": 28180,
"tgt": "Can I take more than two rhythm drugs per day for A-fib?",
"src": "Patient: I am being treated for atrial fib. I have had it most of my life. My wife has Alzheimer's and I am under a lot of stress during the day. When I lay down at night my atrial fib kicks in. It has happened three nights in a row. I take two pills in the morning one for rate and one for rhythm. I have been taking two pills for rhythm.Questiion: Can I take more that two of the rhythm pills during a 24 hour period. Doctor: Dear Sir,It depends what medication you take and what doses, also depends on your heart rate during rest and when you have complains.I would advise you to see your doctor, or provide us with mentioned information, so we can help youWishing you good healthRegards,"
},
{
"id": 221233,
"tgt": "What causes watery nipple discharges in a pregnancy?",
"src": "Patient: Hi, I am 27 years of age and currently pregnant with my first child. I am now 5 and a half months preggies. Today, for the first time I noticed a watery discharge from my nipples and would like to know whether this is normal for someone in my stage of pregnancy. Pls help Doctor: HiDr. Purushottam welcomes you to HCM virtual clinic!Thanks for consulting at my virtual clinic. I have carefully gone through your case, and I think I have understood your concern. I will try to address your medical concerns and would suggest you the best of the available treatment options.Please do not panic.As a part of preparation for breastfeeding the baby, breast glands start developing and start becoming active and in that process the secretions are formed.Please do not worry. this is absolutely normal.Please wipe out with moist, warm cloth at the time of bath, if any dried secretions are there.Also use some coconut oil or moisturizing cream around nipple and circular dark skin around to prevent cracks.I hope my answer helps you.Thanks.Wish you great health.With regardsDr Purushottam"
},
{
"id": 178001,
"tgt": "What does it mean for MRI of an infant to show faint hyperintense areas noted in bilateral periventricular regions?",
"src": "Patient: My son is 1 yr 8 months old...he is not walking on his own...so dr suggested MRI n MRI report shows .ventricular system appears mildly prominent,normal in shape position and signal morphology,cisternal and sulcal spaces are mildly prominent,small faint hyperintense areas noted in bilateral periventricular frontal and Pareto occipital regions.rests things are normal....pls tell me wat does it means Doctor: Hi...though I do not have the images of the MRI before me right now, by what you quote, I feel it should be - Periventricular Leukomalacia - a condition which occurs in prematurely or underweight born babies. Their mentation and IQ will be perfect, but they will have problems with their legs and walking. Usually there wont be any problem with the hands. This condition is called - Spastic Diplegia and is amenable for physiotherapy. Please do not worry as the kid will get better with physiotherapy.Regards - Dr. Sumanth"
},
{
"id": 216909,
"tgt": "How to get relieved from pain due to costochondritis?",
"src": "Patient: i was diagnosed with costochondritis in the ER pn 2/7/14 - I have been told it is a extreme case. I am currently being treated with injections three total at a pain management office. I cant take nsaids. I am still having constant pain. very painful to the touch. Please advise what can be done. I need pain relief. Doctor: HiYou can go for pain relieving gel application locally like diclofenac.Local neurolytic blocks will be of good help in pain relief for longer duration.please consult a pain specialist for the same.All the best.Take care"
},
{
"id": 101647,
"tgt": "Is softin 10 mg good for allergy in genitals?",
"src": "Patient: i am 28 years old, with 5.3 inches height and 56 kg weight. I have been suffering with this allergy in my private area since 2 weeks due to the discharge of the white liquid. My doctor has prescribed me Softin 10 mg once a day and diflucan 150 mg once a week. I am curious to know about the medicines and what are they used for?? please can u provide me details on these 2 medicines related to my problem. Doctor: HI, thanks for using healthcare magicDiflucan is an antifungal medication used for fungal infections. Softin is an antihistamine used for allergies.White vaginal discharge is often related to vaginal yeast infections. If the diflucan is not working, your doctor may want to consider a vaginal swab to determine which medications would best treat the infection.I hope this helps"
},
{
"id": 179293,
"tgt": "What causes soreness around the clitoris?",
"src": "Patient: Hi, my daughter is 5 years old and she keeps complaining about being sore around her clitoris. There is white stuff there and I clean this off and she is better then. I clean her every day but this keeps appearing. Also, I may be showing my naivety here, but she doesn t appear to have any lips over her clitoris. Is this normal at this age? Doctor: Hi...it is common for this age for clitoris to be appearing as out of the inner lips. The white substance might be slough which forms daily in certain kids. It can also be a fungal infection if its irritating her. I suggest you to use a over the counter antifungal cream and if it doesn't settle in a week, consult your pediatrician.Regards - Dr. Sumanth"
},
{
"id": 125219,
"tgt": "How can broken tibia and femur from an accident be treated?",
"src": "Patient: Dear Sir, I met a road accident by bike at 13-Dec-2011. Then my Pattella(knee cap broken into 3 pcs), tibia and Femure has broken. after that illizarove treatment done & bones are healed femure n tibia but broken pattella has left without any surgery. As a result broken knee cap(broken into 3 pcs) big piece in position as per my dr. but other 2 pcs are nearby n saperated from main pattella. I m able to straightly raise my leg and walk but problem is that, i can not bend my leg. So it s bending up to 10-15 degree. While walking i m putting my injured leg(right) in straight position because if it is little bit bend position then i feel strengthless. However, i want to bend my leg. Is it possible or not? If possible then what to do? Pls advice. My x-ray photos are attached herewith. Best Regards Sharif Doctor: Hello, Consult an orthopaedician and get evaluated. Surgical fixation will be the best option. Hope I have answered your query. Let me know if I can assist you further. Regards, Dr. Shinas Hussain, General & Family Physician"
},
{
"id": 13414,
"tgt": "Suggest medication for an infection on the face which causes itching",
"src": "Patient: halo why my face alays etching? who should i do si that it remove o nit come again. pls help me.. what product i use so that it list .and how to remove burnd scars in face. or pores. and dimples pls i need your help pls.. take care doc...i wait your reply. Doctor: Hello, As a first line management you can apply topical steroid antibiotic combination like Betamethasone- Neosporin combination for symptomatic relief. If symptoms persist better to consult a dermatologist and get evaluated. Hope I have answered your query. Let me know if I can assist you further. Regards, Dr. Shinas Hussain, General & Family Physician"
},
{
"id": 120432,
"tgt": "What causes pain in thumb joints?",
"src": "Patient: Fingers getting the appearance of arthritis, thumb joints at wrist area always have pain. Over the past year at the area of thumb joints they became red/purple like a bruise approx. half dollar in size, stay rhat way approx. 1=2 weeks and are very tender. This has happened about 3 times over past year. RA lab test comes back neg. Doctor: Hi, You do not mention your age and to know the exact diagnosis we need to have your blood reports like anti CCP, CRP, ESR ,CBC, vitamin D. it could be osteoarthritis for that we need to have your X-ray. take over the counter painkillers, apply diclofenac gel, hot fomentation. Hope I have answered your question. Let me know if I can assist you further. Regards, Dr. Jaideep Gaver, Orthopedic Surgeon"
},
{
"id": 200505,
"tgt": "Suggest exercises for longer periods of sexual act",
"src": "Patient: how to increase the length and breadth of my penis by means of exercise doc i want to increase the size of my penis by some natural methods. i mean without using medicines or pumps or weight hanging. is there any natural way i mean jast by physical exercises sothst it could be long , strong and thick? and also that i fall too quickly. is there any ways to make my sex period long , full of energy and with full of romance? plz help me doc. Doctor: Thanks for asking in healthcaremagic forum It is not possible to increase penile length and girth by medication. For long performance you can try kegel exercise. You can find literature about kegel exercise on net. All the best."
},
{
"id": 195923,
"tgt": "How to treat leaking penis and erection problems?",
"src": "Patient: I have been to the doctors and to the hospital and had the camera up my penus and they said there was slight scaring on the tube i leek when i come iito contact with my girlfriend i have tried viagara and other pills but cannot maintain a hard been to see a sex theropyst to to no joy can you help Doctor: hellothank you for trusting HCMdear If the cause of the leakage is physical, such as poor muscle relaxation or over-masturbation, then the general treatment might be to boost the parasympathetic nervous system as relates to erections, so that the penis gets enough androgen hormones, neurochemicals and oxygen for consistent function. Hormone replacements as well as arterial stimulation drugs could be a solution.sometimes surgical procedure useful even though it is complicated. please consult your doctor and get it done cavernosonography to confirm venous leak. please consult your doctor he will examine and treat you accordingly. I think I answered to your question if you have more questions feel free to ask.take care"
},
{
"id": 123152,
"tgt": "What causes pain in knees?",
"src": "Patient: I had a knee replacement 5 years ago. Within the last 2 months, I have started having intermittent pain and swelling in the joint. Sometimes it feels like something is catching in the left lateral aspect of the knee. I can press on the spot, when it is flared up and cause moderate pain. Any quesses? Doctor: Hello, As post knee replacement this pain is common to arise due to change in the normal biomechanics of the joint and the gait cycle. This is all due to the weakness of the knee Muscles post knee replacement. I will advise doing exercises like - static hamstring, static Quadriceps, ankle toe movements, straight leg raise, etc to improve the muscle strength and correction of the biomechanics of the knee joint. The gait also needs to be assessed and the muscles which are found weak needs correction. You should be fine with conservative management. Hope I have answered your query. Let me know if I can assist you further. Regards, Jay Indravadan Patel, Physical Therapist or Physiotherapist"
},
{
"id": 88851,
"tgt": "Suggest treatment for abdominal pain",
"src": "Patient: I had Fallopian tube removed (right side) with ectopic pregnancy a year ago, I have been having abdominal pain sometimes and feeling very tense right the scar, suddenly now I am having abdominal pain as same as the ectopic last year the night before my surgery? What shoul I do now ? Thanks Jackie Doctor: Hi.Such a severe pain after tubal excision, after a gap of a year is suggestive that either you are pregnant and have another tubal pregnancy or there is an intestinal obstruction.I would advise to get an X-ray in the standing position of the whole abdomen to find out the cause. You may also go for tests of blood, urine, stool.; ultrasonography and serum electrolytes .IF something of surgical nature is found you may need to undergo another surgery to get the problem solved for ever."
},
{
"id": 203448,
"tgt": "Why am I unable to think after masturbation?",
"src": "Patient: sir i am 22 years old and i have been masturbating for frequently for some days and after some my brain felt something like dizziness and now i cant think properly and my brain doesnt perform like before and there is leakage of semen .when i consulted the doctor he says that it is natural and you need to relax but my head doesnt work means i cant think especially my right and left nerve of the brain close to the ear distracts me while i think. i feel those nerves.so there is remedies for these problems Doctor: Hi and . I think u are getting too conscious about masturbation, so u are disturbed. Masturbation is no where related to your brain performance and thinking process. Masturbation is a very common phenomenon among adults. I can suggest u to come out of thst habit gradually. First thing dont keep things that provoke u for masturbation like any porn material, Develop some new hobby and dont be alone for long time. Go out and involve yourself with other family members, go on outing with friends, Play some sports, keep yourself busy. Do remember masturbation is not be ashamed of or its not a crime. Neglect these thoughts that it is affecting your brain. Regards"
},
{
"id": 225900,
"tgt": "Painful intercourse, pain in the lower abdomen. Should I take out the paraguard?",
"src": "Patient: I've had paraguard for 3 years now and intercourse has always been painful since and I keep getting pain in lower abdomen. Why is this? Should I get it took out? I don't have medical insurance to get checked out.. But if this is a sign of something bad I will definitely go to the hospital to have it taken out. Please let me know what you think I should do. Thanks. Doctor: Hi,Thank you for posting your question here, I will try to answer it to the best of my abilities.Pain during sex is one of the more common side effects oh having a paraguard IUD placed, if the pain is too much I would recommend you get it removed, after all there are other ways to avoid getting pregnant, which dont hurt.I hope this answered your question."
},
{
"id": 27744,
"tgt": "What could cause pain in chest?",
"src": "Patient: HI I am getting chest pain right now. I been sleeping on my side for hte past month but for osme odd reason yesterday my shoulder started to hurt. It became more sore yesterday after I smoked (hooka tobacco) I am not a smoker and barely smoke. I woke up this morning with unbearable chest pain. I have pain that has radiated from my should to my mid arm(left side only) breathing hurts and I feel a pull to my heart.laying down is even more difficult and causes pain in my chest. Doctor: The rest pain you described is more likely because of a muscle pull due some sleeping position, cardiac origin pain would not be related to position you described with a Characteristic pressure like sensation which would increase on exertion, associated with some breathlessness. Avoid smoking if you are used too as it will increase muscle soreness and some local warm compress and sos a analgesic for 2 days should help"
},
{
"id": 200830,
"tgt": "What causes tender black colored mole on testes?",
"src": "Patient: my husband has a black mole looking object on one of his testes that he says he is aware of because it is tender and sometimes bleeds. I think he is trying to make it bleed by opening it up thinking it was an ingrown hair...which I found out later after questioning him about small amounts of blood in shower etc. I am making him an appointment with an urologists tomorrow since I just found out about this tonight. He has NEVER had a physical and is in good health at age 58. should I be worried that this is cancer?? Doctor: Thanks for asking in healthcaremagic forumIn short: Visit a doctor Explanation: Black tender mole like swelling can be a sebaceous cyst infected or a mole as you think. So, it is difficult to differentiate without seeing. So, visit your doctor without hesitation for relief."
},
{
"id": 4847,
"tgt": "Had unprotected sex followed by intake of first inactive pill for period week instead of last active pill of the month. Chances of pregnancy?",
"src": "Patient: I had unprotected sex with my boyfriend on Saturday night. He ejaculated inside me like he always does. Yesterday(Sunday) was my last active pill for the month. Instead of taking that I accidentally took the first inactive pill for my period week. I did not realize this for a full 24 hours when I went to take my inactive pill today so i took the The last active one today instead. I'm really scared that I could of gotten pregnant from the semen that could of still been inside me. Please help. Doctor: Hi, There is no need to worry as you have passed the projected ovulatory phase of your cycle; moreover, you have consumed the pill at once and this will compensate for the mistake. You can safely wait for your next cycle and only if you miss it, go for a pregnancy test with a fresh early morning sample of urine 3-7 days later. Pregnancy can occur only if you happen to ovulate at the time of the unprotected intercourse which in your case is remotely possible. Hope you find this information helpful. Take care."
},
{
"id": 88099,
"tgt": "What causes extreme nausea and stomach pain?",
"src": "Patient: My 17 year old daughter has been suffering with recurring symptoms for almost 2 years now. At first the \"episodes\" were occurring about every 4 to 6 weeks. Now the \"episodes\" are coming weekly and last 2 to 3 days. Here are her symptoms--extreme nausea and stomach pains, extreme fatigue, body aches and sometimes headaches. Normally she does not have vomiting. When the symptoms hit, she cannot function. She cannot sleep at night and she stays in bed all day. She is missing 2-3 days a week from school almost weekly. Between \"episodes\" she is fine. We have had an endoscopy of her stomach and ultra sound of her gall bladder---all was fine. She has been tested for metal allergies (she had spinal fusion surgery at age 12 and has metal rods and screws in her back)--negative. He CBC came back with her C reactive proteins high--over 12. She has also been diagnosed with Polycystic Ovarian Syndrome. We have seen 5 different doctors! What could this be. Doctor: HiThanks for your query for your daughter who is 17 years old. I have read the history and understood the problems. Consultations with 5 Doctors and some investigations, PCOS, severe pain in abdomen making her miss school for 3 days every time ,The symptoms being : extreme nausea and stomach pains, extreme fatigue, body aches and sometimes headaches.. My probable diagnosis leans towards a Psychological factor, which can be confirmed by 2 things 10 all relevant investigations being normal and 2) Evaluation by a Psychologist.Investigations:*Routine blood investigations like Complete blood picture, blood sugar, urea, creatinine, liver function tests, thyroid function tests.*Urine -routine and microscopy*Stool: routine, microscopy, occult blood, culture and sensitivity.*Ultrasonography of abdomen,*Colonoscopy All these investigations should be within normal limits for one to say you have IBS.A trial by a Psychologist / Psychiatrist may be of help and the response with his medicines can be confirmatory."
},
{
"id": 151069,
"tgt": "Have epilepsy. On Levroxa, Valparin Chrono, Lobazalm. Can I reduce the dosage? Is it curable?",
"src": "Patient: Hi, I am having epilepsy since last 2 years (I get fits once every month for 10 seconds), am under following medication: 1) Full dose of Levroxa 500 (6 per day) 2) Full dose of Valparin Chrono 500 (4 per day) 3) Lobazalm (1 in the night) Due to these medications, my frequency of fits have reduced but am getting affected by severe side effects such as memory loss , weight gain, hair loss, rashes on body, frequent mouth ulcers , frequent mood swings, pessimism, getting irritated/angry/depressed on small/minute things, etc. Because of this, I cannot concentrate on my work and cannot be 100% productive at my workplace. I have started getting the feeling that I am not able to live/enjoy my life fully that I used to earlier. Please answer following questions that I have on this: 1) Kindly advice on how I can deal with this? 2) Is there a way that I can reduce the dosage by changing my daily routine? 3) Is Epilepsy 100% curable? 4) Do I need to be under medication for entire lifetime? If not, how long should I be under medication? Doctor: Hi sbeetle ! These medicines give you these side effect including constipation and oral ulcer with sucidal thoughts. 1) ask doctor for break , drug change, or dose change. 3). Epilepsy is not a disease, so no cure it is just over activity of brain. 4). You need medicine just to make your brain calm. But in long term drug side effect is a problem. Yes! you have to take it on and off for long time. And keep searching net for new treatment. Take care. Bye."
},
{
"id": 8819,
"tgt": "Are there any side effects of using a coffee mask for hair growth ?",
"src": "Patient: what s the dangerous if we use coffee to do mask ? female 23 years old....i use these mask(coffee mask) and i want to know if there is any dangerous or any side effect??as i heard that it help in hair growth after i use it .is it true?? and its really serious problem.....please i really need help... Doctor: hi welcome to health care magic, Coffee has no role in hair growth rather it may cause hair damage and discoloration. Better you to go to a dermatologist. thanks."
},
{
"id": 96386,
"tgt": "Heavy bleeding still Abdominal pain",
"src": "Patient: I m an 18 y/o F. Had my period Feburary 9th - 16th (heavy as usual) started bleeding again on the 20th and still bleeding. Heavy bleeding still Abdominal pain . Not sexually active. Bloated. Having a OBGYN appt monday.. wondering when and if to go to er. Doctor: oh my god,, you could have lost lot of blood by now. i suggest you to see an ER and they would call up your obgyn who can help them what to give to stop the bleeding."
},
{
"id": 63409,
"tgt": "What is the cause of painful lumps in the armpit that are filled with pus ?",
"src": "Patient: Hello! I am 23, 5 1 of height and 65 kls. I just want to ask about lumps in the armpit. Everytime I get tired, I usually get lumps on my right armpit (dominant hand). It appears boil and pus came out on it. What is this? It s painful whenever I have this and it affects my ADL. :( What meds or vitamins should I take? Doctor: Hello I have gone through your question and appreciate Your concern. From your history it seems that u r suffering from multiple boils. U should keep ur armpit clean.Shaving regularly of armpits.Fruits and vegetables and vitamin c To improve your immunity. Thanks You can write me back for any query."
},
{
"id": 76984,
"tgt": "What causes chest congestion in infants?",
"src": "Patient: Our 5 month old grandson has a chest congestion, sometimes with a slight weezing, for nearly 2 weeks now. He has no fever, does not look sick in anyway, eats and drinks well and plays and smiles as usual. I try to run the humidifier for him as I know it's not easy for babies to cough it all up. What should we do? Doctor: Hi thanks for contacting HCM....Noted your child having congestive symptoms and slight wheezing....Here there could be chance of bronchiolitis by viral respiratory infection....So your child must be examined by pediatrician for respiratory auscultation and respiratory rate....Symptomatic management done.Your doc might prescribed decongestent to your child if needed.Saline nasal drop may be prescribed....Take care....Your child will recover soon."
},
{
"id": 84141,
"tgt": "What causes dizziness after taking Dubagest?",
"src": "Patient: i have undergone laproscopic ovarian puncture last august.and doctor prescribe dubagest to be started on the 15th day from the1st of the periods..i started to have from 13th of this month but now i have dizziness some times for 2 days..is it normal?waiting for ur advise..... Doctor: Hello, It is unlikely related to dubagest. Possible causes like hypotension must be ruled out. Consult a physician and get evaluated. Make sure that your blood pressure is well within the normal range. Hope I have answered your query. Let me know if I can assist you further. Take care Regards, Dr. Shinas Hussain"
},
{
"id": 6264,
"tgt": "Trying to conceive, spotting, had unprotected sex, swollen breast, heartburn, upset stomach, pregnancy test negative. Pregnant?",
"src": "Patient: yes i touched my cervix today and saw very light pink blood i am trying to get pregnant i ovulate 9,10,11 of may i had unprotected sex 9, 12,and 13 of may. my breast are very swollen i have been sleeping earlier that usual and some time i have heartburn and an upset stomach could i be preganant? according to my calender i should start my period on 5/25/12 i am every 28 days. i took 3 pregnancy test one today and 2 3 days ago and all 3 are negative. + Doctor: Hello, I would be happy to help you today. If we assume that ovulation occurred on about cycle day #14, AND that you concieved, then a pregnancy test should be positive at this time. Given that the tests are negative, it sounds like you are NOT pregnant. It is likely that your symptoms are related to your cycle. It can be confusing when you are \"late\" and with negative tests, but here is what I would recommend: 1. Repeat the test in one week, regardless of your cycle 2. If you don't get your period in one week, it is likely because you had delayed ovulation or did not actually ovulate. In this case, you might need medication to start a period, such as progesterone. 3. If things remain confusing, your doctor could check a blood pregnancy test and/or a progesterone level to see if that clarifies what is going on. I hope this helps. Good luck!"
},
{
"id": 14435,
"tgt": "Are red horseshoe shaped rash on arms and diarrhea related?",
"src": "Patient: I have just noticed a red horseshoe shaped rash on the insides of both my arms, which became more pronounced after having a bath. They are itchy, but not crusty or oozing any fluid. They seem to appear worse when I am warm and almost disappear when I am cold. I have also had diarrhoea for the last 2 days don't know if it is connected? Doctor: HIWell come to HCMI really appreciate your concern, this could be hypersensitivity and this could be due to some allergens some time this may be associated with abdominal manifestation, diarrhea and nausea, condition is more or less is self limiting, wait for couple of days to come around take care for hydration, if this does not come then you can try antihistamine \"Tab Levocetirizine 10 mg three times in day, for diarrhea just do nothing, hope this information helps, take care."
},
{
"id": 69923,
"tgt": "Suggest treatment for lump in groin area",
"src": "Patient: hello i am female and 20 years old and found a lump in my groin (the very top of my right leg in the crease) about a year ago and its stayed the same since its about the size of a pee and feels a bit rubbery it can be seen and looks like a white lump under the skin when touched it moves about a bit but feels as if it is attached to something and is quite hard and i was wondering what it is and if you could help me find out thank you :) Doctor: Hi and welcome to HCM. Thanks for the query.It can present seve ral conditions. first it can be swollen lymph node whcih can be normaly found or can be sign of uroinfection or any other infection. also it can be subcutaneous benign lesions such as lipoma,fibroma or atheroma. in every case it shouldnt concern you if there is no pain, inflammation or rapid growing. it can be surgically removed if there is cosmetic concern. Wish you good health. Regards"
},
{
"id": 197705,
"tgt": "Could masturbation cause problem in married life?",
"src": "Patient: hi iam nick i use masturbation when i was 15 stil now iam useing it now iam 27 so can i get married n no problem for my married life and i wana know that my naves are visible near nies so it come becoz of masturbation can u give me detail about it n how to increase the sprum counts Doctor: Hello and .As an Urologist, i can understand your anxiety. Masturbation has no ill effects, on marital sexual health.It has no effect on sperm parameters either.First test your semen and send reports to know if it's normal.Kindly clarify about, terms naves and nies ?You may send your doubts to me, as a direct question.Dr.Matthew J. Mangat."
},
{
"id": 91669,
"tgt": "What is the cause for lower abdominal pain?",
"src": "Patient: i have alot of pain in my lower abdomen. i am post menapause, but started bleeding. the dr tried to biopsy my cervix but was not able to get to it, pap came back good. also i feel like i have to urinate all the time. i also get dizzy when i urinate, i'm scared. Doctor: Hi.All the symptoms make me think of a few things you should take decision about faster.You need a CT scan of the pelvis and abdomen to rule out malignancy/ UTI or any other problem which may be diagnosed and t get the treatment according to the diagnosis made.Dizziness can be due to weakness. also this happens as a disorder called syncope, please discuss this with your Doctor."
},
{
"id": 24052,
"tgt": "What causes heart to skip a beat that leads to dizziness in teens?",
"src": "Patient: I'm seventeen, and sometimes it feels like my heart seems to skip a beat or I can just feel it pushing really hard. I take Pristiq and am tilt positive (my heart rate changes severly when I change positions.) When my heart does this I get disoriented and dizzy and I sometimes feel like I can't breathe. It happens rarely to occasionally. Doctor: dear , you should make an Echocardiogram to check your Aorta and your heart functions ( systolic and diastolic ) , otherwise many causes can do it like :1- Anemia , can cause that escaped beat ( extrasystole ) .2- Anorexia and lack of sleep .3- psychic , especially if you are coming to an unusual event ."
},
{
"id": 115285,
"tgt": "Is it normal to have panic disorder and depression in patients with polycythaemia?",
"src": "Patient: My 21 year old daughter just had a physical and her blood test indicate she has abnormally high red blood count and at risk of polycythemia. She has frequent nosebleeds, her menstrual cycle is regular but only bleeds for a day. She complains of itching around the face and neck. SHe has been diagnosed with panic disorder and depression and has been taking anti-depressants and sleeping pills for the last 3 years. Is this common among young women with polycythemia or is it unrelated? Some of her symptoms like dizziness and difficulty breathing overlap with symptoms of panic disorder. Have we been giving her the wrong medication for a completely different disorder? Doctor: Hi,Sorry to hear about your daughter. Although some herbal drugs, androgen derivatives,etc can cause polycythemia, she has to be systematically investigated for the cause of polycythemia by a hematologist. You could also go through her past blood reports to ascertain which started first- polycythemia or panic disorder. Is she doing any drugs as that could lead to both."
},
{
"id": 91907,
"tgt": "What could be the reason for having lower abdominal pain?",
"src": "Patient: I have had pain in my lower abdomen/stomach for a little over a week now. It feels like a strained muscle in the area and hurts when I move around and particularly at night. It's been hurting several times through the night and I have to get up and use the restroom, thinking that is what is causing it. Doctor: Hello!Thank you for the query.The most common reason of lower abdominal pain is urinary tract infection. You may have also frequent urinating, burning while urinating, blood in the urine, fever with this condition.Lower abdominal pain should be also checked with inguinal hernia, adductor muscles inflammation, appendicitis (lower right abdominal pain), diverticulitis (lower left abdominal pain), ovaries inflammation, ovaries cyst, pregnancy.Please consult your doctor and start with urine analysis, abdominal ultrasound and blood work.Hope this will help.Regards."
},
{
"id": 211020,
"tgt": "Could Valtrex be used as an alternative to acyclovir?",
"src": "Patient: I have facial herpes. I have been under extreme stress for several months and have continual out breaks so my doctor put me on acyclovir 400 mg three times daily. I think this medication is causing depression. So I was looking at Valtrex as an alternative. Doctor: Hello,Thanks for choosing health care magic for posting your query.I have gone through your question in detail and I can understand what you are going through.Both acyclovir and valtrex are more or less the same and both these medications can cause depression as an adverse effect. The fact that you have already stated that you have a lot of stress states a possibility that your current depression could be due to the stress independently rather than the medication adverse drug reaction. You may take escitalopram or fluoxetine along with the medication for herpes. The stress needs to be managed properly. That will require cognitive behavioural therapy. This therapy is taken by either a psychologist or a psychiatrist in which the therapist identifies impaired cognitions (Thoughts) and try to correct them. They are 15-20 min weekly session and there should be around 8-10 sessions to help you with your illness.Hope I am able to answer your concerns.If you have any further query, I would be glad to help you.In future if you wish to contact me directly, you can use the below mentioned link:bit.ly/dr-srikanth-reddy"
},
{
"id": 34801,
"tgt": "Suggest treatment for mold disease",
"src": "Patient: Our home in Hudson Florida was tested for mold and my husband & I have been having a lot of health problems since this home was tested positive. We want to be tested to see if our health problems are related to the mold.Can you recommend where to have this testing done. Doctor: Hello dear,Thank you for your contact to health care magic.I read and understand your concern. I am Dr Arun Tank answering your concern.You should go for testing of blood. This can be tested in any of the advanced laboratories available near to your house.There are two tests that can differentiate the condition. One is blood eosinophilia and other is the fungal hypha antigen testing present in the blood.If this two tests are detected in blood and in the respiratory system. Than you are infected with the mold.Mold mostly come into system by the inhalation of the fungal hyphae.Once inhaled it causes allergic reaction in the body. This is responsible for the reaction in the body.To keep the antigen away from the body is most important thing. This is the final cure for the disease.If you are severely affectedbthan you can take the fluconazole tablet and prednisolone low dose tablet under your doctors guidance. Engineering and construction can remove the mold from the house so if you can do that, all your infection will remain away from you for life time.I will be happy to answer your further concern on bit.ly/DrArun.Thank you,Dr Arun TankInfectious diseases specialist,HCM."
},
{
"id": 190789,
"tgt": "Should I get root canal treatment re-done?",
"src": "Patient: Hi, I had Root Canal Treatment done a year ago. Did not have the last step done of capping the tooth , due to which the temporary filling from my tooth has come out. And now anything which i eat gets inside the cavity of the RCT tooth.. Do I have to get an RCT done again, or just the capping would suffice? Please help Thanks in advance Doctor: Hello Arzoo, Well it depends on how many days previously your filling has come out.. If its recent then may be you can go for a new filling and a Cap over that tooth.. If there hasnt been any discomfort on the tooth, a Xray can be taken and it can be decided if any infection is there or not. If not just go for a permanent filling and definitely a cap over the tooth. Thanks and regards Dr. Pankaj Malhotra"
},
{
"id": 122695,
"tgt": "Suggest treatment for soft tissue swelling in hip due to injury",
"src": "Patient: I fell skiing 2 weeks ago, injured my right hip. I had a good size black bruise which is going away, but i have a large bump of soft tissue that is still present. I cannot sleep on my right side because of the pain. Short distance walks are OK. Long distant as in a mile or more seem to irritate it. Doctor: Hello, The symptoms are consistent with a hematoma.It is nothing but a collection of blood underneath the skin.Generally it will settle by itself. In your case it is still there even after two weeks.Better to consult an orthopedic and plan for an ultrasound scan. If the size is large it has to be aspirated. Hope I have answered your query. Let me know if I can assist you further. Regards, Dr. Shinas Hussain, General & Family Physician"
},
{
"id": 100749,
"tgt": "Suggest treatment for asthma",
"src": "Patient: i were using erocort inhaler since last 3 year almost every night .I am little worried about it & recently i quit to use it. The whole day passes without any problem but in morning i feel watering from my nose & biggest throat problem, asthma symptoms. now i take advice form doctor he changed my inhaler now i m using budamate 200 as well as salbair I. Any other alternative which cure and treat my asthma . Few have suggested that i should quit my all inhaler .Please suggest if there are any side effects of using inhaler for long term . Doctor: Hello dear,Asthma is caused due to broncho-constriction (obstruction of smaller airway passages) which is indicative of Hyper-responsiveness of air passages.Budamate and Salbair inhaler- provide symptomatic relief by causing broncho-dilation (dilating the smaller airway passages)Being administered through inhalational route, they have minimal side effects and can be used in the long term.Additional management consists of:1. Montelukast preparations- used as a maintenance therapy to relieve symptoms of asthma.2. Antihistamines like Cetrizine can also be used to provide symptomatic relief.3. Need for anti biotics can arise if there is a secondary infection.4. Also make sure that you are well protected from exposure to cold, dust or other allergens.As the condition is related to hyper-sensitivity of your immune response, it can only be managed symptomatically.Complete cure will need complete shut down of your immune mechanism, which will make you prone for severe life-threatening secondary infections.So, kindly continue the medications as prescribed by your treating Doctor.Wishing you a good health.Take care."
},
{
"id": 213811,
"tgt": "Why do i feel better when it rains ?",
"src": "Patient: On an average sunny day i am tired, uninterested, calm, depressed. I don't care to do much more than the average and i am all around just living the day out. On a gray rainy day i have endless energy. I feel more happy than anything. I can concentrate and i get more accomplished. Everything seems so much more alive when it rains. when it's gray the world has more color. i normally have trouble waking up in the morning, i don't drink coffee. If it is raining that morning, i jump up as though i had been up for hours. Im excited to see rainy days coming. When everyone wants to be in i love to be out....why do i feel this way. Why can't i function like this on a regular day... what's wrong with me Doctor: More attachment towards the color gray signifies a form of depression. This may or may not be true in this case, but then, how good or bad the person's quality of life during sunny days needs to be assessed. You may require a consultation with psychiatrist to assess your positive and negative symptoms that you experience on normal days to arrive at a diagnosis. Perhaps you are completely normal, or may be suffering from a mild depression that wanes off when its rainy and pleasant.."
},
{
"id": 108612,
"tgt": "What is the treatment for chronic lower back pain?",
"src": "Patient: HI.. 5 days ago i while i was using the restroom.. i pushed my watse out with too much force and now i have had a chronic pain in my lower left abs, and slightly in my lower backside.. ive researched possible causes and the only things that i feel relate to me are a Herniated Disk, or Possibly a kidney infection, although the pain is mild and chronic, not sharp or severe. I also thought my appendix could perhaps be the cause? can you help me? Doctor: HIWell come to HCMExerting force at the time of defecation may cause some kind of pain and this may any where from lower abdomen to anal region some time this may be mild to moderate depends upon the force applied, do not apply such force to avoid pain, else this is nothing to worry, and this is not related with the conditions that is being discuss here, take care."
},
{
"id": 145646,
"tgt": "What causes pain and muscle spasms in lower back and abdomen?",
"src": "Patient: I was just wondering about having some pain and spasms in my lower back and abdomen. I had a full hysterectomy about 7 years ago, only thing left is 1 damaged ovary, and I started having some painful muscle spasms in my lower back and abdomen. Tonight I notice that I am bleeding and in pain. Should I go the dr. or may this pass? Doctor: Hello!I understand your concern.Your situation is serious. The pain you have shows that something is happening inside your abdomen. It is normal for this kind of pain to irradiate on your back and cause muscles spasms. The bleeding is very concerning. It may be related to the rudimental ovary or the vessels cut off during intervention. But it may also be related to other organs in your pelvis like rectum or urinary bladder.I recommend you see a doctor as soon as possible and have an abdominal echography.Hope to have been helpful!Thank you for using HCM.Dr. Abaz"
},
{
"id": 11713,
"tgt": "Pigmentation problem with skin on both legs, dry and itchy. Any solution ?",
"src": "Patient: I have problem with my skin on both legs. On rt leg, there is a distinct round shape, centrally pigmented (same as my skin color) and outer circle has less pigmented than my skin color and its bit shiny but not itchy. Initially, about a 6 months back, it was round, very dry, very itchy and when I scratch it used to become white in color and the skin in the area would fall as flakes. I started using steroids but I still have this problem. On the other leg, it was just the same but there were two circles. Now the two circles have joined and became like an elongated shape. It has similar characteristics: shiny, centrally pigmented (same as my skin color) and the outer regions have lighter skin color. I feel it is spreading outwards. Please advise. i appreciate any response. Thank you. Doctor: Hello,Welcome to health care magic.From your description, the possibilities of your skin problem include psoriasis and dry discoid eczema. You need to consult a dermatologist to confirm the same. A skin biopsy may also be required.Both the conditions benefit fom a topical steroid for a few days and also moisturisers. Both have remissions and relapses. However, psoriasis can become more widespread.Hope this helped.Take care."
},
{
"id": 10051,
"tgt": "Suggest side effects of kera xl for hair loss",
"src": "Patient: Hi sir, I m kayal. I m facing hair loss problem past 6 months. I was consult with doctor and he suggested kera xl hair growth serum(night).my question is any side effects for using this product. How many months I use this product. And another one question is morning I use any Hair oil? I using tresseme keratin smooth and shin shampoo and same conditioner is this good? Doctor: Hello, Kera XL serum is safe and doesn't have side effects. It can be used long term. Instead of hair oil I suggest you to use lacsoft lotion. Tresseme shampoo and condition is quite ok. Hope I have answered your query. Let me know if I can assist you further. Regards, Dr. Kakkar S., Dermatologist"
},
{
"id": 226283,
"tgt": "Discomfort with paraguard IUD. How long after switching to pills should I wait to discontinue them?",
"src": "Patient: So I have a question regarding removing my Iud , so I have the 10 paraguard Iud and it causeing me discomfort I ve had it in for about 4 month and I m considering switching both control pills but my boyfriend and I have talked about wanting a baby . So I wanted to know how long after switching to the pills should I wait to discontinue them so I cab get pregnant , because I know the Iud thins the uterus lining if I get on the pills will it still continue to thin the lineing ,I should also no that I miscarried in June of last year when I was three months the baby measured up to 5 weeks six days , I will be turning 18 in 4 months Doctor: Hello Thanks for your query. Normally return to fertility after stopping pill use occurs within 1 - 2 months, however there is a condition called post pill amenorrhoea, where you might not get regular periods for upto 6 months. There is no way to predict this. Pills would not thin the lining of the uterus once you stop using them. take care."
},
{
"id": 16552,
"tgt": "What can cause weight gain, fatigue and dry hair while suffering from hypertension?",
"src": "Patient: My blood pressure is 157/87. I have never had any issues with my blood pressure. my left side of my face feels as if my blood is continually rushing. I work out 4 times a week @ 1 hour. I am starting to gain weight, my hair is drying out and I am so very exhausted Doctor: Hi, You should check your thyroid function, any malfunction in your thyroid gland can cause these kind of symptoms. Hope I have answered your query. Let me know if I can assist you further. Take care. Regards, Dr. Salah Saad Shoman, Internal Medicine Specialist"
},
{
"id": 149427,
"tgt": "Leg pain, unable to balance. Prescribed Tribion, Resonix20, Clonatrill, no relief. How to cure?",
"src": "Patient: dear sir,i have a problem left leg,sometimes my leg pain ,sometime my left leg doesnot balance ,i droped into the yeard i going bangladeshi doctor chember he has 13 years experience of physical medicine & other i said doctor what is my diseases?doctor said my diseaes name CMT. i can not understand what is this,he check me & give me medicine Tablet TRIBION ,Tab- resonix20 & Tablet-Clonatrill0.5mg i eat tablet daily but my left leg not balance yet ,doctor said my diseases solve 60% plz suggest me how can solve my problem Doctor: HiThank you for your question to HCM.Leg pain and weakness can stem from pressure or intrinsic pathology of the nerve. To find out the cause and location of your problem I would recommend getting Electromyography and Nerve Conduction testing (EMG/NCV) of left leg. For my patients I would also get MRI of the lumbar spine because lumbar disc herniation is a common pathology that causes leg pain and weakness. Rarely thoracic disc herniations (midback) can lead to such symptoms and MRI of the thoracic spine would be the next step if no correlating picture is found in lumbar spine.Hope this is helpful. I will be happy to answer any questions you may have."
},
{
"id": 123032,
"tgt": "What causes lower back pain?",
"src": "Patient: Hi Doctor My wife is suffering from a low back pain since last year. All of the tests suggested by the doctors are normal. Even then also the pain is there. Exact position of the pain is lower back left side. Please let me know in case any other details required Thanks Mani Goel Doctor: Hello, This could be related to muscle weakness which is leading postural changes. Due to postural changes, there will be joint compression leading to pain. Undergoing physiotherapy sessions will be helpful to reduce the pain. Exercises like - core stability, hip muscle strengthening exercises and lower limb strengthening exercises will help to regain muscle strength and stability of the spine. Hope I have answered your query. Let me know if I can assist you further. Regards, Jay Indravadan Patel, Physical Therapist or Physiotherapist"
},
{
"id": 66133,
"tgt": "What causes lump on knee?",
"src": "Patient: I have these pink bumps along my left knee that have suddenly appeared a month or so ago. I thought that it may be simple bug bites at first, but they got bigger, and they each have a hole in them with this white, crusty-like substance in it. I'd say they are about the size of an average lady bug. Doctor: Hello there... According to your problem it looks like due to some infection...and now they have hole with white crusty substance...means it has been burstsed open and all the infectious material has came out...so it will heal soon.. just keep the part clean and dry... u can use betadine powder to make it dry... if u r feeling pain and redness.. u can use tablet brufen 400mg maximum upto three times a day...Thanks for asking....Get well soon..."
},
{
"id": 200725,
"tgt": "What does this sperm analysis indicate?",
"src": "Patient: hello i have gone through sperm analysis month ago and result was- sperm count-12million/ml, motility:- grade a(progressive)-70%, grade b(non linear)-10%, grade c(non progressive)-10%,grade d(progressive)-10%, morpholagy:- normal-60%, abnormal-40%, epithelial cells-1-2/hpf, pus cell-3-5/hpf. impression-oligozoospermia.Based on test result my doctor prescribed me T.Alene& T.Uniwell,maxoza-L SACHETS for three monthS..DO MY PUS CELLS ARE NORMAL OR ANY THING TO FEAR PLZ ANSWER ME? Doctor: Hello Your report suggest persistent decreased sperm count.Normally sperm count should be atleast 20 millions/ml.You may also need culture and sensitivity of semen due to presence of few pus cells.It is important to exclude infection.Sperm motility and morphology of sperm are normal.Motility should be at least 55 % and normal morphology sperms should be at least 16 %.You need proper clinical examination and further investigations.Besides clinical examination you need routine hemogram,hormone assay and colour doppler of scrotum.It is important to rule out any abnormality.You have been prescribed right medicines.You should take it continuously.You should take nutritious food and do exercise.If low sperm count persists then you may need assisted fertilization techniques like IUI.Get well soon. Take Care Dr.Indu Bhushan"
},
{
"id": 142389,
"tgt": "What causes abdominal soreness while doing therapy exercises?",
"src": "Patient: Hi, I had back surgery in Dec. and had vertebra and disc replaced and 2 rods put in. Surgery was done through the back and not the abdomen but my abdomen muscles hurt . I do therapy exercises and I am a active 70 year old. Surgeon just gave okay to bend and twist 3 weeks ago. Thanks Mike Doctor: Hello Mike!Welcome on Healthcaremagic!The abdominal pain could be related to the lack of physical activity. For this reason, I would recommend trying to perform regular physical activity to help reinforce your back and abdominal muscles as recommended. Your situation will improve spontaneously. You should just try to be more patient. Hope you will find this answer helpful!Kind regards, Dr. Aida"
},
{
"id": 185784,
"tgt": "Suggest treatment for two small bumps appear inside my mouth",
"src": "Patient: I just had two small bumps appear on the inside of my mouth. One inside my top lip, and on on the lower lip, closer to my cheek. Also my mouth felt a little bit numb.This occurred immediately after I had lunch, which contained fish soup. Does this sound like an allergy? Doctor: thanks for your query, i have gone through your query. the bumps inside the mouth could be secondary to burn caused by the soup or allergy to the contents of the soup or it can be mucocele or herpes virus infection. consult your oral physician to rule out these conditions. you can observe for two-three days and see. if it breaks and forms ulcer then you can use topical anesthetics and analgesics that will clear the issue. if it doesnt disappear then consult your oral physician for further."
},
{
"id": 37745,
"tgt": "Can I take non vegetarian food and alcohol after being bitten by a street dog?",
"src": "Patient: Hi Sir, My self, Thirupathi Reddy, 3 days I was bitten by a Street dog, that dog was looking very danger it s just looking like some infected or mad dog. Immediately I went to a doctor and taking the vaccines, the doctor said nothing to worry and nothing to avoid eatables or drinks. But my friends and relations are saying to avoid non-veg and alcohol, I am really confused and worrying about this, please confirm can I eat non veg and take alcohol, please help me in this regard, thank you sir Doctor: Hello,Welcome to HCM,Rabies is 100% fatal but it is 100% preventable by proper and adequate treatment. As you were bit by a dog for which you have taken care of your wound with thorough wound wash and antirabies vaccine of three doses on days 0,3 and 7.Adequate and complete treatment is of at most importance in any dog bite. As such there is no diet restrictions and you can have whatever you are having regularly.If you are a alcoholic and smoker you need to restrict that as it hampers the immune system to produce adequate and protective rabies virus neutralizing antibody titres.Thank you."
},
{
"id": 109312,
"tgt": "Is complete cure possible for lower back pain caused by gap between discs?",
"src": "Patient: hi, i am chaitra from mysore ,i am 30 years old having lower back pain,from september last week i am suffering i shown to all doctors in mysore but of no use.i dont want to go for surgery.they told in betwen last two discs no gap so its paining.now i am going to show to an ayurvedic doctor,they have suggested to go first for MRI scan then we will c.please suggest me whether i am able to come out from this problem or not Doctor: U have to take proper bed rest and back suport and then U can go for the physiotherapy treatment... so consult physiotherapist and follow that..."
},
{
"id": 40301,
"tgt": "How long will it take to heal staph infection drained in lower leg while being on oral antibiotics?",
"src": "Patient: Hi Dr. Grief, My daughter has a staph infection that was drained on her lower leg. It has been 2 full days, and there is still redness and a large wound. How long should we expect the symptoms to persist? She is on an oral antibiotic. Thank you, Christy YYYY@YYYY Doctor: Hello,Welcome to HCM,Once any wound is drained it indicates there may be a cellulitis of the wound. Before draining any wound I think your daughter was given antibiotics to make less infective and need to continue broad spectrum antibiotics like combination of amoxicillin and clavulanic acid for atleast 7 days.As your doctor as drained the infection, it will take minimum of 7 days to clear off the infection.She need to take good nutritious food to boost her immunity and fasten her recovery.Thank you."
},
{
"id": 77161,
"tgt": "What causes sharp pains in lung while on pneumonia medication?",
"src": "Patient: Hi I have pneumonia and have been on a z pack for 4 days now. I also have been taking mucinex and recently was prescribed cough medicine with codine. I am having EXTREME sharp pains in my infected lung. I seem to only get them at night or when I'm about to go to bed. They aren't only when I breath but are constant. Doctor: Hi dear I can understand your concern...Noted you have pneumonia....In which lung paranchyma inflamned so pain ...But a pain is constant and somewhat severe here mild pleural inflammation may be there and ya pleura very sensitive....So continue with antibiotic...and after resolution of consolidation pain will subside with resolving pleural inflammation.Analgesic can be taken if pain troublesome If any unusual symptom contact doc immediately.....Hope my suggestion is helpful.Dr.Parth"
},
{
"id": 217917,
"tgt": "Suggest treatment for pain in neck and shoulder",
"src": "Patient: i have much pain in my neck, also in my shoulder. there is a spot on my neck vertebrae that i can push on that seems to move and \"pop\". I'm wondering if this could be a ganglion cyst. I do have these on my wrist and notice a similarity in the movement Doctor: It cud be really a simple ganglion or alipoma or sum benign cystic growth compressing on ur pain nerve endings, not to worry, pls meet a general surgeon."
},
{
"id": 171280,
"tgt": "What causes sore throat and lightheadedness after tonsils removal?",
"src": "Patient: Hi, my son is 9 years old & he had his tonsils/adnoids removed on January 12/12. Now 3 weeks later he started complaining of a sore throat, lightheaded & neckpain on the left side. His temperature is around 97.5. Is it common to have these symptoms 3 weeks after? I Would really appreciate an answer. Thank you. Doctor: Hi, after tonsillectomy there are some minor complications which may persist upto 3 weeks and then they gradually disappears. This sore throat, light headedness and neck pain are minor complications of surgery and it will disappear with in 1 week. I hope this has helped you. Wishing your child good health. Take care."
},
{
"id": 213278,
"tgt": "Having excessive sweating after exercise. Why even after hours it wont stop? Why abdominal pain?",
"src": "Patient: I am having excessive sweating hours after my exercise routine and am also very fatigued. I took a nap which I rarely do because of my history of Depression and it flaring up when my sleep is disturbed. I have been on Cymbalta 60mg for a month and had some side effects but not either of these. I have no fever but a few abdominal pains, probably gas. It could just be a bad afternoon of tiredness but I can t explain why I am sweating so much in a cool room. Doctor: Hi, Excessive sweating, fatigue and sleep disturbance can occur due to anxiety and depression. Same symptoms can occur in the presence of thyroid hormone imbalance, so it is better to get thyroid function test done. For anxiety and depression cymbalta dose can be increased by your physician. I hope this information has been both informative and helpful for you. Wish you Good Health. Regards, Dr. Ashish Mittal www.99doctor.com"
},
{
"id": 14098,
"tgt": "Suggest medication for hives on skin",
"src": "Patient: iv been suffering from hives over 2 yrs. they say its because I have hoshimoto disease. but I believe theres something triggering them late at night when my body temp rises they seem to onset no food allerigies I had a biopsy and was told it was chronic urticara im miserable nothing seems to help them Doctor: Hello and Welcome to \u2018Ask A Doctor\u2019 service. I have reviewed your query and here is my advice.Chronic urticaria can be treated with long term antihistamines such as Levocetrizine and Fexofenadine. In case of lack of response to antihistamines, you can use cyclosporine, autologous serum injection etc.But, do take care of underlying thyroid disease as thyroid disease is one of the causes of chronic urticaria. If hives occur even after normalizing thyroid function, you can go for drugs such as cyclosporine.Hope I have answered your query. Let me know if I can assist you further."
},
{
"id": 163843,
"tgt": "Suggest treatment for constipation and anal pain in a child",
"src": "Patient: mY FIVE YEAR OLD GRANDSON COMPLAINS ALOT OF HIS BUTT HURTING...HE HAS TO TAKE IRON AND HE TAKES MIRALAX TO PREVENT CONSTIPATION, HE SAYS IT BURTS AND LIKES TO KEEP A BABY WIPE FOLDED UP IN BETWEEN HIS CHEEKS BECAUSE HE SAYS THE COOL FEELS GOOD TO HIS ANAL AREA Doctor: dear userabout your question i would like to tell you that constipation is common problem and at the same time the treatment is simple but always be losen....constipation mainly refer to bad food style that may contain starch or rice or tea drinking and less vegetables and less fruits....this can make severe constipation for the child ...there may be no pathogenic disease causing that but unhealthy food enough to be the cause...iron will not give improvement it some tumes make the problem worse so you can change his food style to food containing high fibers as fruits and vegetables with less rice less besquit less tea...you will find him after short time having normal defecation and the constipation ended gradually and anal pain will relieve as the constipation is the main cause of this severe pain....so treatment is simple but alot of people think that drugs will be good...healthy food is very effective and he will be fine.....i hope that helps you"
},
{
"id": 38886,
"tgt": "What is the treatment for influenza?",
"src": "Patient: I am getting over a bad case of flu. Have bad cough still have a feeling of pop rocks in throat after a prolonged coughing episode. Will this reolve on own or should I go and see Dr. Again. I was not on any rx for this illness. Do I need an antibiotic? Doctor: HelloWelcome to Healthcare MagicAs you are having cough and other constitutional symptoms which are in favor of flu like symptoms. Most of the flu are self limiting snd it will resolve by itself.If the symptoms are not improving I would suggest you to follow1. Drink hot water2. Salt water gargling3. NSAIDs like Ibuprofen4. Oral antibiotics like Tab Bactoclav.5. oral antihistamine. Thank you."
},
{
"id": 15808,
"tgt": "Swollen, sore joints. Red spots like bug bite on legs turning into bruised lumps after taking Cipro for stomach infection. Itchy new rashes on legs and arms. Help",
"src": "Patient: I was given Cipro xl 1000mg on April 3 for a possible stomach infection. I had severe stomach cramps and loose stools for 2 weeks prior. After taking one pill, legs started to beak out with spots like a bug bite. Within 8 hours the spots swelled and got big and red like a bee sting, and hurt the same. My legs also swelled. I went to emergency and was told to stop taking the cipro. By April 8, the joints in booth legs and ankles were swelling. I went to a local Dr. and he described Prednisone 5mg--2 tablets 3 times day for 3 day, then 2 tablets twice a day for 3 days, then 1 twice a day for 8 days. Today my joints are still sore and believe I have tendeitis in my right ankle and the spots on my legs are hard lumps turning into bruised areas. I am developing a new rash, on legs and arms which is very itchy Doctor: hi good day to you i think you may have drug reactionas you are suffering from pain and if u taking pain killer and steroid medicine better you dont take it for long timefirst you have to consult orthopedic surgenfollow his advise and though you dont feel good you have to take homoeopathic medicine from experienced homoeopathic physician"
},
{
"id": 165848,
"tgt": "Suggest treatment for ear discomfort in a child",
"src": "Patient: I took my 8 month old to the rodeo 2 days ago and there was a music performance with very loud noise. He was exposed for about 15 to 20 mins and has been crying since then and scratching his ears. Is there anyway I can help him? I will take him to the Doctor tomorrow but wondering what I can do in the mean time Thank you. Doctor: hello..i suggest to take him to a doctor to know the exact pathology and then start the treatment accordingly.."
},
{
"id": 197709,
"tgt": "What is the treatment for painful internal hemorrhoid?",
"src": "Patient: I feel Like I have an internal hemorrhoid and my scrotum has changed from a firm type of skin to a more stretchy sagging scrotum, and when I get a boner Its slightly painful and uncomfortable. My dick doesn t get as hard and when I pee my flow isn t as fast and it splits into two flows. This isn t normal and don t say it is and BS me with oh your scrotum sags when it gets warmer and shrinks when it gets colder because I know it normaly does this, but My balls are extremely saggy and balls don t sit still like usual. if I stretch this skin it stays warped WTF my balls will reach my knees soon HELP Doctor: HelloI share your concernSplitting of flow of urine might be related to prostate enlargement or prostate disease.Internal hemorrhoids can be managed conservatively by avoiding constipation and surgery (hemorrhoidectomy) if need be.Scrotal scratchiness and sagging might be related to fungal infection, eczematous lesion or inflammation of testis.Looking at your description I would advise you to consult a general surgeon for per rectal examination for internal hemorrhoids as well as prostate examination, ultrasound or colour Doppler of the testis.meanwhile maintain good hygiene, have lots of dietary fiber, fresh fruits and vegetables.Hope this answers your question, please do let me know if you have any more queries, I will happily answer you.Best wishes"
},
{
"id": 27114,
"tgt": "What does these chest x-ray after an injury indicate?",
"src": "Patient: Bilateral apical pleural thickening seen.This is what comes in my xray report . 2 week ago I got hit by rickshaw right side of my chest. I hav started facing breathing problem. Can't take complete breathe feel pain right side of my chest. How to cure this problem.? How long this Will take.? Can I continue my gym activity? Doctor: bilateral apical thickening in xray is not to be worried. before 2 weeks injury can lead to some lung contusion or collection in lungs, you need to get xray chest done again and if required go for CT scan of chest. if all normal, then do deep breathing exercise, spirometry and you breathing will improve and pain will subside gradually."
},
{
"id": 188568,
"tgt": "Had my wisdom tooth extraction. Swelling stopped. Stitches dissolved. Feel a bone protruding. Bit inflamed. Normal?",
"src": "Patient: It's a little over 6 days after my wisdom tooth extraction ( I had all 4 taken out). Pain pretty much stopped the second day ( didn't even need the painkillers doctor prescribed). The swelling stopped a couple days after that. Perceivably, things are going smoothly. This afternoon, however, I noticed that my stitches dissolved ( like my surgeon said they would) except on the bottom left side of my jaw I noticed something that, to me, seemed a bit peculiar. In that area, the site seems to be slightly open with, what looks like, bone slightly protruding from it. I still have no pain/swelling, and the area still looks the slightest bit inflamed. Is this normal? Is it infection or the beginnings of dry socket? Any help would be appreciated. Thanks. Doctor: Hello,Welcome to healthcaremagic forum.From the details produced,its clear that socket has not healed completely.To prevent occurrence of dry socket,take complete course of augmentin along with analgesics.Swish with lukewarm saline frequently.Irrigate the socket with betadine ,apply eugenol as well as lignocaine solution within the socket.Avoid spicy foods,smoking.Take adequate rest and stressful work has to be limited.If symptoms persists,please do visit a dentist.Hope this helps."
},
{
"id": 132135,
"tgt": "What is the indentation in the lower calf?",
"src": "Patient: I have been having back pain. Had a fusion in 2011. going to physical therapy because I keep injuring my lower back. Have had constant numbness and sensations throughout left leg. Noticed tonight I had like an upside down V indentation in lower left calf. What could this be? Doctor: Hi you had fusion in 2011 in your back ,have constant numbness and now have an inverted V indentation in your calf.This kind of indentation is generally seen if there is a spasm in the calf muscle , may be due to pain .Once you have relief from pain , this could well go away.v"
},
{
"id": 218021,
"tgt": "What is the sharp pain below my ribcage?",
"src": "Patient: Hi Im 44 years of age and in the last few days Ive noticed a sharp pain directly below my ribcage on the front left side. I had an ultrasound done on my gallbladder and pancreas and the doctors found a polyp on my gallbladder and a lesion in my pancreas. However after this I was sent to get a ct scan in which i was injected with a dye and nothing could be seen at all - the doctors said there must have been a glitch in the ultrasound - is this possible. Im worried about my pancreas as I m getting pain and wonder if I should go back for another ultrasound. Doctor: Hi, Welcome to Health care magic forum. As you describe, there is nothing wrong in the pancreas, as the pain is mainly on the right side. Your pain appears to be due to the peptic ulcer,due to irritant foods, anxiety or H.pylori infection. I advise you to consult a gastroenterologist for diagnosis and treatment.You may need to have gastroscopy,for diagnosis. Avoid taking spices, junk foods, and oily foods. Wishing for a quick and complete recovery. Thank you."
},
{
"id": 59598,
"tgt": "Diagnosed with hepatitis C. Altered stool consistency, fatigue and acne. What should be the treatment?",
"src": "Patient: Hi, I was recently diagnosed iwth Hepatitis C . I strongly believe I got it this past summer the month of August 2012. I was checked last year oct. 2011 and wasnt detected. But now I have it and I have symptoms of clay like stool, darkening of the liver , fatigue , tiredness, acne , loss of apetite. Is it possible to have symptoms approach this fast? If so, what can I do to change this reaction within my body? Doctor: Hello, amanda, Hepatitis- C can be contracted by a number of means. This occurs by coming in contact with contaminated blood. Drug abuse , shared needles or other paraphernalia even many years ago can be the source. Blood transfusion or blood products prior to 1992 (We did not have blood tests for Hep -C then). Other things that are risk factors include Tattoos,body piercing,sharing tooth brushes, razors etc. Accidental needle prick and exposure to contaminated blood (Lab technecians etc,surgeons ) I am giving you all this background to let you know , you can be completely well till the virus starts multiplying in the liver cells and causes damage. Now that you have been confirmed with blood tests, you need to get HCV -RNA viral count and the Geno-Type of the virus because the response to treatment or cure depends on this. Good news is ,there are very effective treatments using a combination of drugs. You can look for a clinic or a doctor's office that specializes in treating this. It takes patience, dedication and determination on your part to get cured and this can be done. You may have to put up with side effects from the drugs, again most people can tolerate well.Usually the treatment lasts for 24-48 weeks depending on the response of the virus count at 4-8 weeks. Please see the doctor and have all the proper tests done and start the treatment as soon as possible. I wish you well."
},
{
"id": 156699,
"tgt": "What does redness in the area arond my incision indicate after surgeries for papillary throid cancer?",
"src": "Patient: I recently underwent two surgeries to have my thyroid removed due to papillary thyroid cancer. I am two weeks post second surgery and noticed today that the area around and below my incision is red (like a sunburn). I didn't know if this might suggest an infection at the incision or a reaction to hormone meds. Doctor: Hi and welcome to HCM. Thanks for the query.In most cases this is wound infection considering what you ve described but if there is no fever or pus discharge at wound then you dont have to be worried. you should just see your doctor who can decide are antibitoics neceesary ot not. Wish you good health. Regards"
},
{
"id": 186886,
"tgt": "What causes swelling in mouth and jaws?",
"src": "Patient: Well it happened not to long ago. All day the back of my gum on the right side of my mouth has been swollen and for weeks my jaw on that side has been in so much pain. Well 15 minutes ago, I pressed against the gum and puss started shooting out from my gum. It's feeling better now. Was it an infection? Doctor: Hello, Welcome Thanks for consulting HCM As I have gone through your query, as you have pus discharge from gums dont worry it can be due to periodontal problem . There is formation of periodontal Abscess . Do warm saline gargle two - three times a day. Consult dentist and go for oral prophylaxis .Hope this will help you."
},
{
"id": 43702,
"tgt": "Semen analysis showed oligoathenozoospermia, bilateral hydrocele. Treatment?",
"src": "Patient: Hi doctor .. i m safy from egypt . married one year ago but no luck of getting pregnant .. i m 29 years . i went for consultation and my doctor told me every thing is normal and she asked to see me at time of ovulation . and every thing was just good .. my husband was asked for semen analysis and the results he got was : count : concentration 8.18 million .. grade a ( rapid progression ) : is 12.5% .... morphology : 24% so the diagnosis was ; oligoathenozoospermia .. by us: no varicocele but with mild bilateral hydrocele i m very worry please answer me what is the most suitable way to treat such cases .. i m dying to have a baby of my own .. i wish you can help me sir plz Doctor: Hello. Thanks for the query, Oligospermia refers to semen with a low concentration of sperm which is a common finding in male infertility. Asthenozoospermia refers to reduced sperm motility. Hydrocele needs operative corrections. Clomiphene citrate in oligospermia. Combination of tamoxifen plus testosterone. IUI-intrauterine inseminations can be performed. In more severe cases IVF, or IVF - ICSI is done and is often the best option. Hope this helps Good luck Dr Nilofer"
},
{
"id": 218582,
"tgt": "Is pregnancy possible despite taking an emergency contraceptive pill after unprotected intercourse?",
"src": "Patient: I had unprotected sex with my b.f on 4th of this month i took the pill within 3 hours but there is still no sign of withdrawal bleeding, my period date is 17 of the month, i'm concerned as there is no withdrawal bleeding happen, it's been more than 10 days, is it there any chances of getting pregnant? Doctor: Hello and Welcome to \u2018Ask A Doctor\u2019 service. I have reviewed your query and here is my advice. As you have taken emergency contraceptive pill within 3 hours of unprotected intercourse the possibility of pregnancy is less. Usually after taking the emergency contraceptive pill withdrawal bleeding can be seen in around 7 to 10 days. But the onset of withdrawal bleeding after taking emergency contraceptive pill may depend on which stage of the menstrual cycle you have taken the tablet. As you have taken the tablet in the second half of the cycle sometimes onset of the withdrawal bleeding may be delayed. So, better to wait for 3 to 4 days. Even after that if you do not get withdrawal bleeding better to go for urine pregnancy test once. Hope I have answered your query. Let me know if I can assist you further."
},
{
"id": 213746,
"tgt": "How can post operative effects be treated ?",
"src": "Patient: I am 57, male, 6 1 , 210 lbs, diabetic 2. I had a foot infection leading to the amputation of toes and debridement of my left foot. Prior to a below the knee amputation, I had a full leg by-pass which was successful, and did not need the second amputation, which was BRILLIANT :). However, during the bypass procedure, I woke up and, although I have no memory of this mid-operation awakening, I was subsequently visited by my anaesthetist explaining to me what had happened and asking me if I was experiencing any problems. At the time, I was so uncomfortable from the procedure, I felt nothing unusual. I have now been home for almost a month and have an almost overwhelming and unpleasant smell of antiseptic in my nose 24/7, and I am inclined to weep embarrassingly at any movie on TV. On the one hand this is funny and weird, on the other it is embarrassing and unpleasant. Can you offer any advice? Doctor: Hi,thanks for query.What you are having has got no medical base.This is not possible.This can happen due to some disliking about certain things.Try to forget the incident and start thinking some thing different.Most of the leg surgeries are done in spinal anaesthesia and only sedation is given to make you feel comfortable. wishing you good health."
},
{
"id": 164518,
"tgt": "Suggest the best treatment for vitiligo in 8 years old",
"src": "Patient: Dear sir, Im writing this mail for my brothers daughter..she is 8 years old We have spotted some white marks on her neck.. when we consult with our family doctor.. we came to know that shes suffering from vitilgo\u2026 It was really a shocking news to the whole family as we have seen some people in our village with large white marks all over their body\u2026. I We would like to know some things from u .. can u please reply to the following doubts 1, what are the main causes of virtilgo 2, what to do to prevent these marks spreading 3,what is the best available treatment 4, can u name a specialty clinic or hospital in india for treatment. Doctor: vitiligo can occur due to 1. family history2.auto immune disorders3. thyroid disease4. diabetes5. Addison disease6. pernicious anaemia7.hypo parathyroid hormone treatment is.1, new lesions: topical steroids and calcineurin inhibitors2, old lesions: topical puva solutions3, severe lesions: topical steroids+puva solutions+ oral steroids,"
},
{
"id": 144280,
"tgt": "What causes loss of balance and frequent falls?",
"src": "Patient: I am a 60 year old female who has been having trouble feeling unbalanced. This has resulted in me having a concussion. On the most recent fall I broke two ribs and a third one has a hairline fracture. I can tell when these episodes are coming on so I have been able to hide it. I don t want to hide this I want to know why this is happening. Doctor: hi,as you mentioned you are 60 and having frequent falls due to balance issue. Then consulting a neurologist will be help, also he may ask for an MRI of the brain, as there needs to be checked out something if going pathologically dysfunctional.As most patients when fall into ageing there will be cerebellum dysfunction. As the demand of the body is high in maintaining the coordination and balance. also , with the help of the Neurological assessment it will come close to the actual cause of the balance and coordination problems.Once you are done with an MRI and Neurological assessment, you may be onto some medicine and also on a physical therapist.the physical therapist will teach how to learn the balancing activity and also to avoid falls.Also with age there is mass reduction in the strength of the Musculoskeletal system which needs an attention as falls might be dangerous at this age due to degenerative changes in the bone.Fracture are not good at this age , as the pain bearing capacity gets reduced and also the recovery time is delayed.by doing physical therapy the brain again learns the balance and coordination functions which allows to reduce the number of falls and also improve the overall health at this age.with the grace of God I wish you a speedy recoveryregardsJay Indravadan Patel"
},
{
"id": 76515,
"tgt": "Suggest treatment for bronchitis",
"src": "Patient: A friend of mine has a 3 year old son that was running a fever of 105 and vomiting yesterday. She took him to the ER and they sent him home with a diagnosis of Bronchitis. His current temp is 92 with cold sweats. He will not eat or drink and he can't stay awake. What could be causing his temp. to be so low? Doctor: Hi thanks for contacting HCM.Noted he had runny nose and fever with diagnosis of bronchitis...Here treatment is done symptomatically.His temperature is slowing down so he is in recovering phase...So no need for worry...As he has frequent sweating mostly it is viral infection ...For vomiting ondensetrone containing syrup can be given...Apply cold sponging whenever fever...Paracetamol can given for high fever.....If congestive symptoms more then levocetrizine might needed to relieve congestion and inflammation...If still condition increasing then respiratory examination done by physician for further work up..Hope your concern solved....Take care...Dr.Parth Goswami"
},
{
"id": 154630,
"tgt": "Can a growing cyst be possibly stem of a malignant cancer?",
"src": "Patient: If a person has a cyst that grows from marble size to baseball size within a few weeks and a co-worker lances it, is there a possibility that if diagnosed later it is a stem of a malignat cancer, could that cancer have caused an infection within the body that would shut down all organ systems? Doctor: Hi, dearI have gone through your question. I can understand your concern. Growing cyst can be benign or it can be malignant. You should go for fine needle aspiration cytology or biopsy of that cyst. It will give you exact diagnosis whether it is cancer or not. Then you should take treatment accordingly. Malignancy decrease the immunity and it causes infection. Hope I have answered your question, if you have doubt then I will be happy to answer. Thanks for using health care magic. Wish you a very good health."
},
{
"id": 130231,
"tgt": "My daughter has been referred to a rheumatologist. What do i need to ask?",
"src": "Patient: My daughter (19) has been referred o a rheumatologist for the 2nd time n 2 years. High Sed rate, inflammation, and WBC. The fist visit, in 2012, showed a few slightly elevated results in CBC, but was not an issue at the time. What do I do as a mother, parent, to know what I do from here? What do I need to ask? Doctor: You have not cleared the reason why is she refferred to a rheumatologist?Any joint pain,rash on face,loss of hair ?apart from ESR,CBC what other blood tests are you asked to do?Any family history of autoimmune disease?inflammation and High ESR and changes in cbc are seen when you are ill due to some bacterial,viral infection and fever and in many other diseases.i would suggest you to meet a rheumatologist if you are adviced to because from the history you have given,can not come to any conclusion but its good to visit a rheumatologist with all previous history of illness.Its easy to make a diagnosis when as much information is given to the doctor."
},
{
"id": 66147,
"tgt": "Suggest medication for a hard red bump on the arm",
"src": "Patient: i just shot meth into my are in the open crease of the are where you bend your arm. well it didnt hurt during the hit but totally afterwards it started burning both up and down my arm. it even hurts when touched or when i bend the arm. still hurts 10 minutes later. Another ?. my other arm has a hard red bump on it. i cut it open myself and squeezed crap out of it, but its still hard, should i cut deeper cuz maybe i just didnt get all the crap out? Doctor: Hello and welcome to HCM,It is not advisable to cut open the bump on the arm yourself.Cutting the bump yourself can lead to infection and injury to soft tissues.Thus, consult a primary healthcare provider for clinical assessment.Proper dressing of the wound is required followed by application of antibiotic ointment.Thanks and take careDr Shailja Puri"
},
{
"id": 124927,
"tgt": "What could cause arm bruising?",
"src": "Patient: Quick background. Diagnosed with Hodgkin\u2019s Lymphoma on January 13th, started Chemo on January 27th, finished Chemo on June 29th. PET-scan on August 22th was all clear & blood work on the same day looked good according to my Ecologist. My question / concern. 3 days ago my right bicep started to ache\u2026that night, about 2 inches up my arm from my bicep it felt like I was punched. Now I have a huge bruise on my arm and I do not recall getting injured on that arm at all. What can this be? Doctor: Hi, It should be investigated. It could be a reactivation of the original disease or hopefully an infective process setting in. Hope I have answered your query. Let me know if I can assist you further. Regards, Dr. Nirmal Chander Gupta, Orthopaedic Surgeon"
},
{
"id": 193126,
"tgt": "How can pain due to groin hernia be treated?",
"src": "Patient: I had a groin hernia 5 years ago and then 6 months later they had to take out the mesh because it was not in right to repair my hernia. Now I believe it is coming back I have pain in that area plus pain around toward the baack area where leg joins Doctor: Hi, Anti-inflammatory drugs can be given at initial stage, in case of obstructive hernia, surgery can be done.Hope I have answered your question. Let me know if I can assist you further. Regards, Dr. S. R. Raveendran, Sexologist"
},
{
"id": 168308,
"tgt": "Can Ofloxacin suspensions be given safely to a child for loose motions?",
"src": "Patient: my daughter is 3 1/2 years old. suppose i take her anywhere she will have stomach infections due to change of food and have vomits, loosemotions, fever. Doctor detects stomach infection and gives ofloxacin suspensions regularly whenever she has so. how far is it safe to my daughter Doctor: Hi, ofloxacin is given in stomach infection. But it is an antibiotic and the medicine is given according to weight. Moreover, as child grows infective organism change so the antibiotic is also changed. I advice you to not to give ofloxacin by yourself. Take care."
},
{
"id": 63498,
"tgt": "What causes lumps on the scrotum?",
"src": "Patient: Hello Sir, I have one lumps or may be boil on skin of scrotum. for last 4 year i have and there size is some thing like half of vegetable pee. in this there is no pain. just like a piece of hard dead muscles. There is no irritation also. Initially when it grow that time i pressed it and i find no pain and in that i see some black colour thing come out. after that it increase and now it is fixed. But my worries is nearby the lump i find 2 new very small lumps also. same as first one. Please tell me what is this and why it happen to me. and how to cure it. Doctor: Hi, dearI have gone through your question. I can understand your concern. You may have sebaceous cyst. Multiple sebaceous cysts are common on scrotal skin. You should go for examination and if needed go for biopsy. Surgical excision is the treatment of choice. Consult your doctor and take treatment accordingly. Hope I have answered your question, if you have doubt then I will be happy to answer. Thanks for using health care magic. Wish you a very good health."
},
{
"id": 68483,
"tgt": "What does a swollen gland in neck indicate?",
"src": "Patient: I have a swollen gland in my neck.Have had it for about 3 months. I went to the dentist to rule out my teeth and had a routine checkup and cleaning. the next day woke up with ringing in my right ear,which I've had for a week now.I have an appt. with an ent doctor on oct 1st,but wonder what it is and if I doing something before then. Doctor: HiYou should be investigated. You should get FNAC ( Fine Needle aspiration Cytology ) of the swelling and confirm the diagnosis. Once the disease is confirmed, treatment will be given accordingly. Nothing to worry. Wish you good health. Regards"
},
{
"id": 168201,
"tgt": "Suggest treatment for restlessness and crying in a child having reflux problem",
"src": "Patient: my daughter now 10 mths diagnosed with reflux at 2 weeks old on .05 liquid zantac once daily. has always been uhappy, cries always, always had a cold, attacks her ears, shoves fists in mouth extremely hard to put and stay asleep...pulls legs up while rocking her off upright, leans forward while sleeping. as i have slowly introduced foods the texture is more chunkier and she is becoming so bad.... all symtoms are tripled and she is waking up all night instead of just stiring and farting badly at night. HELP im going insane my patience is now on overload!! Doctor: calm first.. feeding a child in the age group of 9 - 18 months ia an art. children start developing independency and they hate being fed. give them food in a bowl of their favourite colour and choice. put her food in it. let her spill and eat on her own. if you are using a feeding bottle kindly throw it away . that may be reason of frequent resp infections. apply coconut oil all over body. give a nice body massage with it and make her sleep. make her food energy dense with oil and ghee"
},
{
"id": 97348,
"tgt": "Suggest an alternative remedy for mylofibrosis apart from Hydroxyurea",
"src": "Patient: Good morning Dr I am Prakash Rana ,38 years old, from Nepal,now I am suffering from mylofibrosis and taking allopathic drug Hydroxyurea since three months from Medanta Hospital India,Now I want to know curative or better treatment about this Disease. Doctor: **1.Since you are already a diagnosed case and under medical supervision thus they must have excluded any secondary cause of the disease, and provided with necessary treatment and management, and because treatment is symptomatic, as such following things must be kept in mind:. Allogenic Stem Cell Transplantation is the definitive curative management. If you are asymptomatic, observation is best. If uric acid and hemolysis is there then a hyperuricemic drug and folic acid is essential. JAK inhibitors [ruxolitinib] have shown promising results in treatment of myelofibrosisPS. Management of individual symptoms in myelofibrosis is very difficult and there is no evidence of benefit for any of the conventional treatments for the same.2. So you need Immuno-modulation [Rasayan] Chikitsa to help strengthen your Immune system"
},
{
"id": 157713,
"tgt": "Lump in breast,enlarged lymph node in neck, tightness in the chest, pain when raising hand. Cancer?",
"src": "Patient: Hi there,Im a 21 year old male and I have a few symptoms that I can't get out of my headI have a small lump in my left breast and have had it for about 3 years. It hasn't gotten any bigger and I've had an ultrasound done on it that came back showing it to be non-cancerous. I also have an enlarged lymph node in my neck/throat just below my jaw line, only on the right side that sometimes swells up and sometimes is normal, but I've had this too for about 3 years.In about the past year I've developed a pain in my right shoulder, 99% of the things that I do in my everyday life don't effect it at all. But I can feel like it's slightly swollen/enlarged compared to my left shoulder. When I raise my right arm above my head like I'm pointing to the sky and bring it back down slowly and to the side my shoulder cracks, every single time that I do that motion.Recently, in about the past month I've started to get a tightness in my chest, almost exclusively associated to the days after I consume alcohol but not only on those days. I'm constantly thinking about these symptoms and second-guessing the ultrasound results. Cancer is in the forefront of my mind whenever I start thinking about all these symptoms and my chest tends to feel like it's tightening when I think about them as well. Doctor: HIThank for asking to HCMI really advise you to control your thoughts it may be dangerous later on, it is kind of psychological condition developing, and that need to be stop as early as possible and you can only do it, it is not at all the thing as you are thinking just relax and forget every thing about this swelling it will go away without medicine do not worry eve if it takes little more time for this, have nice day."
},
{
"id": 155607,
"tgt": "What could be the WBC level indicate in blood test when having pancreatic cancer?",
"src": "Patient: My husband has unrespectable pancreatic cancer...his wbc is 4.4, his lymphocytes are 0.4, his granulocytes% is 86.0, his RBC is4.04, his hgb is 11.0, hct is 35.3 and mchc is 31.2....I m very concerned...his nurse practitioner did not explain these to me...any help will be appreciated...my emails is angelicgaston @yahoo.com...thanks Doctor: Thanks for your question on HCM. I want to ask you few questions first.1. Does your husband on chemotherapy? 2. Does your husband have fever?His total WBC count is on slightly lower side. And this is seen when patient is taking chemotherapy. As chemotherapy drugs are immunosuppressive in nature, low WBC count can be seen. His neutrophils are very high, so of he is having fever than chances of bacterial infection are high.Otherwise report is good."
},
{
"id": 135053,
"tgt": "Is exercising advisable after suffering a contusion in the hip and sternum?",
"src": "Patient: I am 62 yrs old. I jumped off a 30ft cliff into the ocean in Hawaii. The dr I saw said I had. Contusion on my left hip and also sternum. I am wondering if I shudder be doing exercises for the sternum while it s healing. I am taking ibprophren 3 times a day and muscle relaxer at night only. The trauma happened a week ago today. Going home tonight to Denver on an airplane Doctor: hello....contusions take time to heal...if you continue to exercise ,healing time will prolong..i would suggest you to have some rest for few days and once you get relief then only start exercises..thanks"
},
{
"id": 14272,
"tgt": "What causes rash on chest?",
"src": "Patient: I have a rash on my chest that is right under and in between my boobs that has constantly gotten worse since i have had my daughter about 14 months ago. I have tried puting triple antibiotic cream on it and also suffered through alcohol and hexiderm. Can you please let me know what this is? Doctor: Hi.As per your case history you are suffering from tinea corporis (fungal infection)My treatment advice is -1. Maintain good hygiene, bath twice daily and wear loose cotton clothes.2. Don't use triple combination creams as it will only give temporary relief and skin rash will relapse and worsen on stopping it.3. Use an anti-fungal cream like clotrimazole cream twice daily and dust a clotrimazole powder. Anti-histamine medicine like levocetrizine can be taken for itching.4. Other treatment option includes oral fluconazole, itaconazole or terbinafine.Thanks.Dr.Harshit Bhachech.MBBS,DDVL"
},
{
"id": 70027,
"tgt": "Will ice and naproxen help for lump on knee?",
"src": "Patient: I was playing volleyball and I dove for a ball. I must have hit below my knee somehow. Now there is a lump below my knee about the size of my knee. It is starting to bruise. I was told to take naproxen and put ice on it. Should I expect this to go down? Doctor: Hi,Thanks for writing to HCM.That swelling can be a hematoma. Continue the same treatment you are already taking. It should go in a few weeks time.If pain increses or it becomes warm or red then show to a physician as it signifies infection in there.RegardsDr. Ashish Verma"
},
{
"id": 197540,
"tgt": "How safe is taking Nebido for low testosterone ?",
"src": "Patient: Hi I've been on Nebido injections for the past 6 months as I was put on them by my consultant as he said I'd never gone through puberty properly because of low testosterone. So basically my question is how long will all the changes take? Its just I've met somebody recently but I've always been very reluctant to get into a relationship as I wouldn't be able to have sex/get an errection. So I was wondering how long does it take till I can start getting erections and engaging in intercourse Doctor: Higreetings. I understand your concern. Since your consultant told you that with out testosterone injection you will not reach puberty meaning, either your testis is not producing enough testosterone or there is inadequate stimulus from your pituitary gland to the testis.Now I have no idea of your age , present hormonal status and your body stature.(I mean your hair pattern and size of the Penis etc).IF TESTOSTERONE IS NOT PRODUCED IN YOUR BODY YOU WILL NEED THIS FOR A LONG TIME TILL YOU HAVE TO BE ACTIVE IN SEX.It depends up on the dose and it is calculated as per your hormonal levels. It is safe and you will be able to have normal sex.So don't worry about it and to know about your present stage discuss with your doctor as I have no access to examine you.hope my answer helps you. Regards"
},
{
"id": 74925,
"tgt": "What causes a sensation of movement under the ribcage?",
"src": "Patient: Everyday I feel this movement under my left Ribcage..It does'nt hurt so I have'nt felt the need to go and see a doctor..But it is just getting on my nerves..When i touch it I can feel it like as a baby is kicking but i know that is not it..Can you please tell me what it could be..Thanks Doctor: Hey there, according to me there is nothing to worry for your complains,Its just some kind of anxiety like features."
},
{
"id": 91998,
"tgt": "Suffering from abdominal pain post total hysterectomy",
"src": "Patient: I have a sharp pain when i get up bend down,cough, and when i am pulling in my muscles there is a sore spot right in the centre below the belly button above the public bone. I am female.Have had in1985 a total hysterectomy to remove very large fibroid so was cut to that area to get the lump out. when i pull in the muscles eveything is hard as if there is another mass or scar tissue maybe. Had a tummytuck soon after so there is also a scar right round the bottom stomach perfectly healed on the outside. At the moment the pain has got worse daily very much like an infecton pain all round the inside of the abdomen but to touch the spot above pubic is very sore Doctor: Hi, I highly suggest to get checked by your physician and to run further tests for further evaluation: - abdominal ultrasound- urine analysis- vaginal swab testing- examination of stool for ova/parasitesI advise to: - avoid exercise that involve abdominal muscle- avoid sex- drink plenty of water- take acetaminophen or mephenamic acid for the pain- apply warm compresses to the sore areaHope it was of help!Dr.Albana"
},
{
"id": 35212,
"tgt": "Suggest remedy for persistent cough",
"src": "Patient: I have a cough lasting over 3 weeks now. I went through a course of zithromax 250mg, 1 tab a day for 5 days which did not help, then the doctor prescribed zyrtec - d for allergies, that has also not helped. My cough is productiive (with white phlegm), with a lot of crackling sound in my chest, however I don't have a fever or throat pain, please let me know - Ravi Doctor: Thanks for posting your query to hcm.as you are having couhh for 3 week first I would suggest you some investigation.1.gram stain and zn stain microscopy of sputum.2. Culture and sensitivity of sputum. It will help in detection of cause.however you can take syrup containing ambroxole, terbutalin and guaniphesin 2 tea spoonful 3 times a day , tablet cetrizine 10mg with monteleucast5mg once at night and pentoprazole 40 mg once in morning.it will give you symptomatic releied.further treatment will be based on your investigation report because you have already taken antibiotic ..feel free to ask if any query.review me after report comw.regards,dr manish purohit"
},
{
"id": 153796,
"tgt": "Suggest methods to prevent cancer",
"src": "Patient: I'm female,41 years old. I have HPV Type 16 with mild dysplasia in cervix. i'm having follow ups pap smears to prevent cervical cancer. But I know also that HPV can cause anal & oral cancers & there are no follow up tests for these in my country. I've been with y husband for 17 years & faithfull & we don't have anal or oral sex? Is there anything else I can do to prevent these cancers? How big is the possibility to get them because of HPV 16? Doctor: Hi,Thanks for writing in.HPV infection is common in women. It might happen even in those with a single partner though multiple partners increase the chances of infection. Prevention of cancer is best by getting screening tests done and recognizing symptoms.If you have HPV 16 infection then getting annual pap smear is recommended and I hope you are doing it regularly especially because of mild dysplasia in cervix. The possibility is small and not something that should be disturbing your piece of mind. Oral, vaginal, vulval and anal cancers are seen apart from cancer cervix which is the commonest condition among cancers present in women with HPV infection. Please take note of any oral or anal symptoms like non healing ulcers, wounds in oral cavity and skin tags in the anal region. Please do not worry."
},
{
"id": 143885,
"tgt": "Suggest remedy for spine pains",
"src": "Patient: Doc, I am having back spine pains. D pains is like a lightning, its preads to my shoulders. I have never injured my back or involve in an accident. I suspect that the pain was caused by a bullet proof. The pain gets more deep when it is cold. Can you help? Doctor: symptoms described by u are suggestive of cervical spondylosis with radical are pain.Try tab Etoshine MR and tab pregaline.Get MRI cervical spine done if symptoms persists.Cervical collar may help u. Regards"
},
{
"id": 196450,
"tgt": "What causes loss of sexual drive?",
"src": "Patient: I am currently losing my sexual drive. I usually cannot go more than two rounds these days. I have stopped drinking and regularly exercise. What is the cause? before, I could go 3 to four rounds, but now I just cant go beyond two. plus it take a long time say 20 mins before I can get an erection for the second time. Kindly advise Doctor: Hi welcome to the health care magic In your case you can do following work up.... -Blood sugar (for rule out diabetes) -Blood pressure measurement (for rule out hypertension) -Serum cholesterol and triglycerides (for rule out hyperlipidemia) -BMI for obesity -Drug history -alcohol, smoking -stress, depression -prostate problem -Serum testosterone estimation Hence rule out above cause and accordingly treatment given It's not how many times sex done that matter but to get climax and satisfaction that is more important If erectile dysfunction more then your physician after rule out above cause can prescribe you sildenafil like drug Hope your concern solved"
},
{
"id": 21940,
"tgt": "What causes pounding in chest?",
"src": "Patient: I am 43 years old I have had pounding in my chest for several days and now today I am very tired and sick to my stomach I have hbp ans high cholesterol and have been taking meds for years this is really starting to worry me not sure what I should do. Doctor: hello, This pounding sensation is most probably related to gastritis and reflux disease. However, get one ecg done during this pounding sensation, to rule out heart problem. Do you upper abdominal pain, nausea, bloating, burping, increase in pain on food, sour water feeling in throat or chest burning, if yes it further supports the diagnosis. You should avoid fatty, oily and high calorie diet. Have some walk after having food instead of taking rest. Have multiple small meals instead of heavy meals. Have regular sleep habits and avoid stress. Lots of green leafy vegetables, fruits. Avoid smoking and alcohol if any. You can get prescribed tab Pan DSR 40 mg beforebreakfast once a day for 2 weeks. For your other problems, you should have a healthy lifestyle like avoiding fatty, oily and high calorie diet. Have low salt diet and monitor blood pressure regularly thrice a day for one week then once or twice a week. If bp is persistently more than 140/90 mmhg, thenmedicines needs to be hiked up for it. Regular exercises like brisk walking, jogging according your capacity atleast 30 min a day and 5 days a week. Lots of green leafy vegetables, fruits, fish once or twice a week, avoid meat. Avoid smoking and alcohol if any. There shouldn't abdominal fat deposition or obesity. Get your lipid profile and sugars tested once.. For cholesterol, you should be started on medicine like rosuvastatin 10 mg bedtime, you can get it prescribed from local doctor."
},
{
"id": 222354,
"tgt": "What is required diet for a pregnant lady with Urticaria and Angioderma?",
"src": "Patient: Hi Doc I just suffered a bout of Urticaria and Angioderma for the first time.I am 6-7 weeks pregnant my rashes are gone since 2 days and today is the 3rd day. WHat all foods can I eat I am still on antihistamine tabs. I had brinjal, strawberry milk, pickle, & 6-7 cashews which were main causes of the outbreak says my Doctor. Please tell me what foods can I start taking. Can I eat Upma? Lentils in little quantity? Can i start eating tomato and Potato. I am fed up of having curds n rice all these days and have lost 2 and half kilos in 4 days. Please help URGENT!!!!!!!! Doctor: dearfood for urticaria patient is,avoid lime / lemon, tomatoes, orange, curd,pickle,sauces, chutneys, any tinned food, vegetable with seeds,avoid any perfume with strong odour, extreme change in climate.if you have cough or cold take care of thatavoid stress.dr. Mira Butani"
},
{
"id": 186572,
"tgt": "Suggest treatment for a bump on the lower front tooth",
"src": "Patient: Hi I have a small hard red bump on my lower front tooth and it hurts. I've had it for a while and it's bigger now. My teeth hurt and it hurts to touch my chin. Can u please help. I'm worried it's a tumor or something. I'm only a teenager. Please helpThank youAshley Doctor: thanks for your query, i have gone through your query, the red bump could be an infection from the tooth resulting in periapical abscess or granuloma or cyst formation. consult a oral physician and take radiographs like IOPAR and rule out the above said pathologies. if it is a tooth infection you can go for RCT Or if it is gum infection get your teeth cleaned. if it is a cyst, the cyst has to be enucleated. i hope my answer will help you. take care."
},
{
"id": 205829,
"tgt": "Suggest treatment for symptoms of ADHD and stress",
"src": "Patient: Hello, I have a 23 year old daughter that exhibits ADHD symptoms. When away from us i.e. university etc, she has abuse marijuana , alcohol , constantly lies to us and needs money. She is very intelligent and yes people go to her for help and yet she can't seem to score well all around. when immense pressure where she had to pass three remedial courses for a carribean medical school (after being home for the summer) she managed a 89 %, a 92% and on the third course repition only a 69%. she has issues with aleopicia and is very attractive and will be friends with guys who pay attention to her! Because she has long hair the alexia is well hidden. It has increased with the stress of medical school and i have asked her many times if she wants to leave the program. she refuses. i m lost and afraid , live in Toronto and want to try and help her . please help Doctor: DearWe understand your concernsI went through your details. If your daughter refuses to discontinue her course, please do not compel her. She is doing what her intellect says. Let her be happy and do it. You can ask her to consult a psychologist who can counsel her and teach her to cop with stressful situation and time management. In fact she should learn it from her experience. Prolonged stress can affect her health adversely. She shows ADHD symptoms may not mean she has ADHD. Please consult a psychologist.Psychotherapy techniques should suit your requirement. If you require more of my help in this aspect, please use this URL. http://goo.gl/aYW2pR. Make sure that you include every minute details possible. Hope this answers your query. Available for further clarifications.Good luck."
},
{
"id": 223484,
"tgt": "Should I get Plan B even after being on Yaz and condom breaking during sex ?",
"src": "Patient: So my boyfriend and I had sex last night and I have been on the pill (yaz) for 7 months and I had sex on my period the 4 day and I usually start a new packet on Sunday.. So we had sex and the condom broke and he came in me. I am nervous I could get pregnant because during the birth control I recently been good with taking it on time but missed one pill but took right away when remember it and been fine since and got my period on time . Should I go get the plan B? Or wait it out? I also have poliastic ovarian syndrom. Please help me Doctor: Hello,I have gone through your query and understood the concern. Since you are almost always regular with your oral contraceptive pills, you need not worry about possibility of conception, as there is none. Moreover, you had the intercourse soon after the onset of the menstrual period. There is no need for an emergency pill. Do not stress, go about with your regular contraception and wait for your next period. Taking the emergency pill is going to mess up your hormone balance and this is not at all advisable considering the fact that you have PCOD as well. Hope this helps."
},
{
"id": 92616,
"tgt": "Niggling pain in abdomen. Suggest a solution",
"src": "Patient: I have had a niggling pain in my abdomen for 5 days. It started as a very mild annoyance, it wasn't unbearable, and it was located on my right side, mostly towards upper right abdomen. It has progressively worsened, and today it feels as though my entire right abdomen is sore. It hurts slightly to press upon it. There have been times when I have felt the pain radiate through my groin also. I would not describe the pain as severe, more like a 5 on a scale of 1 to 10, but it is annoying. I cannot lie on my right side, in fact it is most easier for me to lay on my left side with my knees drawn up. This is unlike any pain I've ever felt, and it does not feel as though it is ovary pain. The pain is constant, dull and gnawing. Doctor: Hello,Thanks for the consultation.After going through your symptoms,The pain in this part of tummy usually happens because of Stones of the Gallbladder, kidney problems and Gut Problem such as appendicitis.Because of unavailability of clinical findings of a physcian and complete history, I want you to undergo few blood test Such as complete blood count,Liver Function tests and Urine examination.I would also advise you to undergo Ultrasound Abdomen and pelvis.Meanwhile you can start pain killer which suits you as i dnt know the allergy status.Please do all these test and can revert back to me .Hope You get well soon."
},
{
"id": 113504,
"tgt": "Pain in upper back due to swelling and bump. Taken painkillers",
"src": "Patient: Hi i have extream pain in my upper but crack swelling on the left side only and a medium size bump in the center it is painful all the time just noticed the pain 2 days ago. I have been taking painkillers for my back before this happened and when i take them the pain of this bump does not go away. I havent slept in 2 days not for more then an hour Doctor: Hello! Diagnosis: Fibromyalgia with Lump under screen Treatment: Get radiograph of spine followed by MRI of spine for swelling with Hb%, TLC,DLC, CRP, ESR done to monitor the course. Take antiallergic, antiinflammatory & sedative supportive till the reports are available when definitive treatment is advisable. Wishing you good health Dr Vijaykumar Sadulwar, M.S.ORTHOPAEDICS & FELLOW IN SPINE SURGERY"
},
{
"id": 141002,
"tgt": "What causes dizziness and whooshing sensations in head?",
"src": "Patient: Yesterday I was sitting drinking a cup of tea quite relaxed after nice picnic out with the kids when I got this sudden dizzy feeling or woosh through my head. I carried on as normal but anxiety took over during the night & I feel every feeling in my head now wondering if it s related / going to happen again. I have felt it once or twice before, oddly enough when I ve been in bed & about to fall asleep. I understand the anxiety side last night worsened my symptoms & can usually talk myself out of worrying but I m still concerned this morning about that wooshing sensation. Doctor: Hi, I don't think yours is a serious condition, in my opinion, benign positional paroxysmal vertigo is a condition to consider. It is a condition that may be triggered by certain sudden movements of the head and neck. So, try to avoid sudden movements of the head. If your symptoms worsen then consider consulting with an ENT Doctor. Hope I have answered your query. Let me know if I can assist you further. Regards, Dr. Erion Spaho, Neurologist, Surgical"
},
{
"id": 115764,
"tgt": "What causes lower level of hemoglobin?",
"src": "Patient: my father having problem of HB. B'for 15 days it was 5.00 and than we have supply VITCOFOL injection 5 times. But no such progress HB remain same. After that we have supply 3 bottles of blood and HB was 6.1. What may be reason..? what will be your advise..? Doctor: Hello,I understand your concern.I am Dr. Arun Tank, infectious diseases specialist, answering your query.There are many reason for such a high degree anemia.There is need for detail investigation to diagnose the exact cause. I advice you should attend nearby doctor to diagnose exactly.Though you have taken the blood transfusions, such a low HB requires some time to come to normal level.So from now your can start the injectable iron therapy. Continue it until your HB reaches a over 10 gm there after oral iron therapy till HB reaches to normal level is suffice.I will be happy to answer your further concern, you can ask me on bit.ly/DrArun. Thank you.Dr Arun TankInfectious diseases specialist."
},
{
"id": 163759,
"tgt": "What causes stomach ache, sickness with vomiting and cyanosis?",
"src": "Patient: my daughter had tummy ache all day on tuesday and started being sick at 10 pm at night, she was still being sick the next day even just drinking water, took her to the doctors and he just said it was a bug, she had some dry toast on thurs morn, and brought that straight back up, today she has eaten some dry toast and kept it down, she keeps saying she fells unwell, but have noticed that her lips are blue - is she ok??? Doctor: Hi... lips becoming blue is a very very critical symptom of illness. I suggest you to take her to the nearest Emergency Room immediately.Regards - Dr. Sumanth"
},
{
"id": 19166,
"tgt": "What is the treatment for low Blood Pressure?",
"src": "Patient: My blood pressure has been low ever since i'v had my first child. I had a c- secion with both my children and have not felt the best ever since. I'm tired all the time ( was diagnosed with sleep apnea after having my children) I have been having breathing difficulties on and off feeling like someone has their hands around my throat only two episodes of this within the last year and with this comes headaches, lighheaded, dizzy, can't consentrate and it only seems to be when i have this titening in my throat . Can all these symptoms be a cause from low blood pressure which 2 days ago had done and it was 113/54 Doctor: not because of low BP but due to sleep apnea u can get these symptoms ur BP recording is 113 so this will not cause ur symptomsif u r overweight or obese then weight reduction will clear ur sleep apnea syndrome and u will have good sleep and later on ur symptoms will disappear."
},
{
"id": 68301,
"tgt": "Suggest treatment for pea sized lump in the armpit",
"src": "Patient: I have two tiny pea sized lumps in my left armpit, and one on the right side of my adams apple. And oh, they're painless. I'm female. 17. What can it be? I've had the neck lump since a few years now, but, the neck ones formed a few months ago. One after another. Doctor: Welcome to health care magic. 1.The history suggest the lumps in the armpits are lymph nodal enlargement due to the glandular enlargement.2.The glandular / neck lumps which you say few months ago need to be evaluated.3.Get an ultrasound examination - that will evaluate the nature of the lumps, its source and extensions.4.An FNAC to confirm the nature of the cells involving in the process. I think you need to treat the neck lumps first the axillary / armpit lumps goes on its own.5.Suggest to get an appointment and get investigated and treated accordingly. Good luck.Hope i have answered your query.Any thing to ask do not hesitatehttp://doctor.healthcaremagic.com/doctors/dr-ganesh/62888 Thank you."
},
{
"id": 166515,
"tgt": "What causes body and head pain while having high fever in a child?",
"src": "Patient: hI THERE. IT IS 4 AM AND MY CHILD IS WITH HIGH FEVER AND CRYING. AND COMPLAINING OF PAIN ALL OVER THE HEAD AND IN THE BODY. ESPECIALLY INSIDE HIS ANKLES ON BOTH LEGS. dO YOU KNOW WHAT THIS COULD BE? IS IT WORTH THE RISK OF GOING OUT IN COLD AND AT NIGHT, TO TRY TO FIND A 24 HOUR PHARMACY THAT WHERE WE COULD GET SOME KIND OF PAIN RELEIVER.WHAT KIND OF PAIN RELEVER IS GOOD FOR A 4 YEARS OLD WHO WEIGHS 43.6 POUNDS? Doctor: Dear parent, I understand your concerns. to relieve fever and body aches you should give alternating acetaminophen and ibuprofen syrup every 4 hours . this means you give acetaminophen at one time then after 4 hours you give ibuprofen syrup and so on. the dose for your child will be 5 ml or one teaspoon. I hope I answered your question"
},
{
"id": 124907,
"tgt": "How to treat bruise across upper back & width of the shoulders?",
"src": "Patient: I have recently discovered that I have bruising across my upper back,across the width of my shoulders I have had no accidents, there is a niggling pain/aching,picking feeling where the bruising is it looks like big finger print markings should I go see my G.P.? From left to right it looks like nail marks,then bruises, a lot of redness and is like an s shape, It has been there now for some weeks they don t disappear, I would send you a picture to give you a better look at it but that option is not here please advise if you can. Thanks Gloria Doctor: Hi, I appreciate your concern. The picture you described gives an idea that you might be having some allergic phenomena or fibrofascitis. You are advised to consult a dermatologist as well as an orthopedic specialist. Accordingly, it will be investigated and advised. Hope I have answered your query. Let me know if I can assist you further. Regards, Dr. DEORISHI TRIPATHI, Orthopaedic Surgeon"
},
{
"id": 35502,
"tgt": "Can itchy blisters be microbial bacterial infection?",
"src": "Patient: I cleaned a cespool dirty Ice chest, 3yrs ago, and began to have black splinter looking things under my fingernails, It started to seperate my fingernail, little bubble like blisters began to show up and ich, whet to a doctor and they gave me terbinifine Hydrocloride, My hands got worse, an ameba looking circles began to move and spread on my hands. Went to another doctor, he gave me Ketocanonzole, My Hands are getting worse, Busting open cracks,Red and Pink, Fat pockets, and iching and spreading fast, I also have a fish tank and dogs. Im scared because they keep saying Fungal infection, but I fear It may be Microbial Bacteria Infection. And I don't want to loose my hands. What do you think? Doctor: Hi XXXX Thanks for your query at HCM! I am Dr. Sheetal Verma an Infectious Disease Specialist answering to your query! I went through your query and understand your concern. Itchy lesions are mostly due to fungal infection. Take scrapping test for fungal microscopy and culture. also bacterial culture and sensitivity to see secondary infection. Consult infectious disease specialist or dermatologist. Hope I have answered your question. If you would like some more information I will be happy to provide. It would be kind if you hit a thanks if found helpful. You can also write a review for me. Wish you good health Take care! Wish you best of Health. Dr. Sheetal Verma Infectious Disease Specialist"
},
{
"id": 89385,
"tgt": "What is the treatment for anal fissure and rectal pain?",
"src": "Patient: I have extremely and severe rectal pain due to a fissure. I also have abdominal pain on my right side that is sharp; seems like when my rectum hurts is when i ll feel a sharp pain in my side/same time. i have had several uti s this past year, and it seems i may have a bladder that has dropped which may be putting pressure on my rectum, will this cause the severe pain on my left side too? not so sure who to go to see; as i am scheduled to see a urologists for the dropped bladder; then a colon/rectal surgeon for the fissure. i don t know what to do. Doctor: The pain in anal region is due to fissure in ano. And the pain on right side can be due to colitis - which may be the original reason for anal fissure.Recurrent UTI s should be well investigated by ultrasonography and IVP that is intravenous pyelography. The treatment of the anal fissure is as follow: High fiber diet.Avoid all the foods and beverages and stress-like conditions which gives constipation. Laxative for softening the stool- never allow the dose too much to cause liquid stools.Sitz bath- very important- just before passing a stool and after passing a stool.Clean the area very clear of the fecal soiling that remains after cleaning; use cotton soaked in the same water to clean.You can apply liquid povidone iodine locally. Apply nothing else.5 day course of an antibiotic on prescription of a local Doctor.Add probiotics and multivitamins.Avoid stress and mental tensions at any cost; this is a very important factor.Continue this for 3 weeks and most probably you may not need surgery.If the results are positive you have to continue doing all this and take proper care.This may help to relieve your UTI to some extent"
},
{
"id": 155963,
"tgt": "Are my chances very high of getting some form of cancer?",
"src": "Patient: Hello.....Women on both of my parents' sides have had breast cancer for years, and on my Dad's side, cancer runs very high....my Dad just passed away from brain cancer, and his two sisters have had breast cancer, only 1 of them died from it. My mom gets benign breast cysts all the time. Also, my maternal grandmother has just been getting treatment for uterine cancer. Are my chances very high of getting some form of cancer? Should I get a DNA test done? Doctor: Hi Welcome to HCMI have gone through you query and found your concern about chances of getting some form of cancer? Dear, It is not mandatory that if some one has family history of cancer ought to get infected . Chances can be there . You have to have stronger immun system to be free of the disease.You have to be aware of the fact that everyone has cancer cells in their body, so why does one person never get diagnosed with cancer and another person is diagnosed with cancer? Because ---Nasty microbes and parasites get inside of the organs and make their homes there . The cells in the organ become weak due to the activity 0f microbes and weaken the immune system & cause growth of cancer cells indicriminately ,The cancer will not come back as long as the patient watches their diet and strengthen their immune system , so that it can fight out the microbes and parasites causing cancer or other diseases . If you think you may be at risk for cancer, you should discuss this concern with your doctor. You may want to ask about reducing your risk and about a schedule for checkups.Meanwhile , for your imun system I would like to suggest you to Adopt regularity in your life style . --Simple, healthy nutritious food including more of fibers , protiens , vitamins & minerals than starches like rice , potatoes . Also avoid tea ,coffee ,alcohol fried and fast foods PROPER regular EXERCISE , WALK,YOGA, pranayam-deep breathing proper rest HELP YOU TO STRENGTHEN YOUR IMMUNE SYSTEM and help you lead disease free life .Turmeric powder a Tspoon with a cup of hot milk before bed time help many types of bacteria under control .A Tspoon of fenugreek seed powder with water in the morning to control sugar level .Antioxidants like raw garlic ,ginger , onion in every meal .Use of lemon in water befor meals and butter milk in lunch help the body keep going Mixture of extract of Lemon ,Garlic, Ginger , Vinegar of Coconut water Bark of Moringa tree & Homey , Take 25-35 ml for 21 days to 41 days ,with equal amount of water for 3-4 days and pure mixture for rest of the days .PROPER regular EXERCISE , WALK,YOGA, pranayam-deep breathing proper rest HELP YOU TO STRENGTHEN YOUR IMMUNE SYSTEM and help you lead disease free life .All the above is full of antioxidendants having no side effects giving you immunity to live life without such disease and can go with any therapy . Condition is to be regular with this regimem .You can advise this regimen also to your mom , grand mom or any body who is undergoing treatment to save them from reurrance of the disease by STRENGTHENING THEIR IMMUNE SYSTEM .Hope this helps solve your query . If have any doubt pls mail at drsuchda@gmail.comTake care .All the best Don't hesitate to get back if have any further query ."
},
{
"id": 99060,
"tgt": "What causes coughing and chest tightness after drinking alcohol?",
"src": "Patient: I have a tight chest and sometimes cough after a night of moderate to heavy drinking. This only started a month ago or so. I am a 23 year old male, 5' 11'', 165 lbs, athletic trainer. What is going on with my body? Am I allergic to Alcohol or is it something else? Doctor: Coughing and chest tightness both could be an indication of upcoming cardiac disease too. It should be excluded. If it's not, thank God and try to treat other local reasons that could be chest congestion due to environment, infection, cold etc. Doesn't seem allergy to alcohol.Exclusion of any serious disease by clinical examination would be advisable.Hope it helps you."
},
{
"id": 137483,
"tgt": "Suggest remedy for swollen arms in chidren",
"src": "Patient: My 11 years old got a vicine yesterday. And right now He ia in pain. He says his arm hurts 6 in the scale of 1 to 10. I gave him.Tylenol and rubed his arm with mentol. Still in pain? Should I take him to.the emergency room,? Wharmt should I do? Why is He in pain. his arm looks fine, no swallowing. Doctor: Do iceing 10 to 15 minutes 4 time a day and ask you kid to move his limb actively Dont keep arm as it is move and do some sort of active exercise or at least play activity This will relieve spasm as well as pain ."
},
{
"id": 213145,
"tgt": "Tremors in hands. Difficulty in picking up things. Betacap advised by psychiatrist. Is it due to any mental problem or Parkinson's disease?",
"src": "Patient: Hi doctor, i feel some times tremors in my hands (not regularly my hand are shaken..some times i feel very normal)..i.e while signing or filling bank cheque, while handling dinner plates in any family function, while taking cup of tea or coffee and most important is sometimes i am unable to pick up any food item from plate when some body is near or when i feel nervous..to all the symptoms i have reported to psychiatrist in Vimhans institute in New Delhi..he advise me to take betacap-20 mg (Beta propranolol ) after morning meals but while taking from last 20 days i am still not feeling any improvement (sometimes shaking proceed as usual) whether it is Parkinson or any kind of depression or still should i increase dose of betacap 20 mg and make it to 40 mg..please advise what to do...Satya Doctor: hi..intention tremor may also be of neurological origin..so please consult a neurologist..propranolol is a good choice for tremor of any origin, but there are certain problems of increasing dose in case of hypotension or asthma..so before increment, please consult your psychiatrist..Parkinsonism usually presents with rest tremor, rigidity, slowness..so these are in need of assessment..have a good health"
},
{
"id": 78582,
"tgt": "Suggest cough syrup for tuberculosis patient",
"src": "Patient: Hi, my mom is tuberculosis patient, she is in cat II treatment for the second time, treament failed in the first time and nw she again taking cat II medicines. Can u pl suggest some cough syrup for her, since she is suffering the whole night without sleep. We tried many syrups but of no use. She is 57yrs old. Doctor: Thanks for your question on Health Care Magic. I can understand your mother's situation and problem. In my opinion, we should first rule out MDR (multi drug resistant) tuberculosis in her case. She has already taken treatment for two times. Now she is having tuberculosis third time and even after starting treatment she is not improving and having continuous coughing. All these favour possibility of MDR tb more. So better to first get done sputum AFB (acid fast bacilli-bacteria causing tuberculosis) culture and sensitivity report and rule out MDR tuberculosis. At present give her centrally acting (acting on brain cough centers) cough suppressant like codeine, levocloperastine, Benz pearls etc. All these are prescribed medicines, so you need doctor 's prescription for them. So consult your doctor and discuss all these. Hope I have solved your query. I will be happy to help you further. Wishing good health to your mother. Thanks."
},
{
"id": 78142,
"tgt": "Suggest treatment for exercise induced asthma with sharp pain in armpit?",
"src": "Patient: Hi- question regarding 23 year old 6 5 male with prior history of childhood exercise induced asthma (much resolved in late-teens according to pulmonologist). Recently having on/off symptoms of sharp pain under armpit area, down side and occasionally upper abdomen area. Does not take any meds. Doctor: Hi. I can understand your concern. Possibility that it is asthma again is there. Will also need a detailed examination to look for any local cause.So better to consult pulmonologist and get done clinical examination of respiratory system and PFT (Pulmonary Function Test). PFT is needed for the diagnosis of asthma. It will also tell you about severity of the disease and treatment is based on severity only. You may need inhaled bronchodilators and inhaled corticosteroid (ICS)Don't worry, you will be alright. Hope I have solved your query. Wish you good health. Thanks."
},
{
"id": 226183,
"tgt": "Delayed periods. Had taken emergency contraceptive pill after intercourse. Chances of pregnancy?",
"src": "Patient: Hi mam,I had sex with my bf in 3rd week after my periods. Though he was sure that cum didnt go inside the vagina.still i took unwanted 72. Just within half an hour. Next day i had anal with him and he cum in it. But surely it didnt went inside vagina. Its my period date and still not down. Is there anything to worry. What should i do?Warm Regards Doctor: Hi, thanks for asking in Healthcare Magic. Abstinence or withdrawal pull out is associated with a highest failure rate. Although unwanted 72 contains Levonorgestrel, a progesterone which prevents conception and can cause withdrawal bleeding, it will not protect you from the subsequent exposure you had. Since the second exposure was too close to your periods date, and happened after you took the pill, unwanted 72 may not have been effective. The withdrawal bleeding should have come within 5 to 7 days. Since the pill contains high level of hormone it may have disrupted the menstrual cycle and may have caused the delay. But still in your case, the chances of pill failure is more and so you have to rule out pregnancy through urine and blood tests. If your pregnancy tests are negative, then visit your treating Dr and follow a regular method of contraception suggested by your Dr and avoid using ECP frequently as it may completely disrupt your menstrual cycle. I hope this is helpful to you."
},
{
"id": 155362,
"tgt": "Would it be safe to take calmly for brain cancer?",
"src": "Patient: My boyfriend has just found out he has brain cancer and he is taking it calmly, he said that he is okay with it because everything dies sometime. He is 17 and he said that he is going to be 18 when he dies. Im scared that im going to lose him to the cancer and i would like to know how can i support him through this? Doctor: The best advice for you right now would be to stay calm as well and try not to talk about it all the time except when he wants to talk about it. Also, be with him as much as possible unconditionally, and do things that he likes about you. Go to his treatment sessions if he is okay with it. In short, just be yourself and be with him."
},
{
"id": 145917,
"tgt": "Suggest treatment for memory loss, disorientation and seizures",
"src": "Patient: My uncle is suffering from memory loss, disorientation and seizures. Currently he is in ICU and unresponsive for more than a week. He was confused and behaving strangely and completely dependent to do anything before admitted. Everything happened so suddenly. Doctors are doing their best. but he is inactive and on anti seizure drugs. Doctors are saying that there is no hope for recovery. Looking at the symptoms it seems like he has some type of Encephalopathy. Is it curable? what shall we do next as doctors are saying what is happening with him is mystery and couldn t find the root cause of all these problems. Doctor: HIWell come to HCMIf your uncle is in comatose condition so this need to be seen clinically, all the test need to be red carefully for diagnosis and prognosis, if this is viral encephalopathy then prognosis would be poor, still vital organs are intact then chances of recovery is good, patient's age is matter here, (You have not mentioned the age ). hope this information helps, take care."
},
{
"id": 146310,
"tgt": "Suggest treatment for slipped disc",
"src": "Patient: Hi,I have slipped disc problem. I read a lot that I cannot sit too much but thst is hard as I have to work every day.GP refered to xray but this did not show anything. I went to private MRI scan while i was on holiday since pain increased they suggested 6mm hernia L4-L5.Any suggestion will be highly appreciated. Doctor: Hi, I had gone through your question and understand your concerns. Herniated disc could be treated conservatively or surgically. Conservative treatment consists on short term rest ( in severe pain periods), NSAID drugs, warm procedures to the lower back, physical therapy ( when pain is mild or no pain at all), swimming etc. Some Doctors try affected nerve root infiltration with cortisonics and Lidocaine that may result in temporal relief. If the above steps fail to improve your condition and don't ease pain, then there is need for surgery. Hope this answers your question. If you have additional questions or follow up questions then please do not hesitate in writing"
},
{
"id": 82785,
"tgt": "Suggest remedy for lupus & antiphospholipid syndrome",
"src": "Patient: I am 61, female, weigh 56kg, height 5ft 3, I have Lupus and Antiphospholipid syndrome with small vessel ischaemia and arrhythmia, at last hosp appt bp was 159/91 and was told it needed monitoring by GP which I am waiting for, took my bp today on a machine and it was 135/69, why is the systolic now ok but diastolic much lower? Doctor: Hi ! thanks for the Query on HMC. No need of worry... You are in monitoring by GP. If any abnormality will occurs your GP will Guide you. For one thing, blood pressure fluctuates over the 24-hour cycle, depending on a person\u2019s activities . Also, some people have what\u2019s known as white-coat hypertension \u2014 their blood pressure spikes upward in a physician\u2019s office or other health care setting. Others have \u201cmasked hypertension\u201d \u2014 normal in a health care setting but high at home. Home monitoring can also provide a better sense of how well a medication or other blood pressure\u2013lowering strategy is working."
},
{
"id": 165567,
"tgt": "Is it safe to use an Epliator for removing underarm hair?",
"src": "Patient: My 10 year old daughter has terrible body odour (confined to underarm) although she washes daily and applys deodorant. I understand that it s probably rampant hormones as she s been like this now since she was 8. The question I have is... is it safe for her to use an epliator for the underarm hair? She is embarrassed about the smell and the hair and gets upset easily about it all! Please help. Doctor: hi,epilator is safe as far as it is used for single person use. epilator catch the hair and remove from root unlike razor.only drawback is that it produce pain while removing the hair"
},
{
"id": 98029,
"tgt": "Excessive hair loss, dandruff present. Problem reoccurs on stopping mintop. Homeopathy treatment will be helpful?",
"src": "Patient: I am a male,age 24. I am suffering from hair loss since 6years. In that period of time when i was moving my fingers in my hair dandruff was there and around 1000 s of hairs were falled 1day when i was doing coconut oil massage and than after hair loss started, each and every day 100 or more falling of hair get started , I have started mintop 10% and 5% treatment for around 1 and a 1/2 year and a medicine such as follihair and finax 1mg ,regrowth of hair of 10% was there but after stoppage of use the hair loss started again, still i have dandruff and using danket shampoo but still there is dandruff and hair loss is in routine (I am at 6th stage of hairloss as doctor has told me,my upper portion has some slite hair but my backmiddle and front hair is being falled) ,please suggest me about using the anti dandruff shampoo and a good medicine.I am now thinking of a treatment of homeopathy using SBL s arnica shampoo and medicine R89 drops will it be helpfull for my hair regrowth?? please give your reply as soon possible Doctor: Dear patient, Don't get panic!First the basic cause should be rulled out.The hair loss is relateed many other things.According to ayrveda Simhanad guggule 3tabs thrice a day with dadimadya/sukumara/amlakyadi ghritam is very benificial in such conditions.Avoid fast food,hypertension,lazziness,excessive drinking water etc.This will solve all your present hair problems for ever.Thanks"
},
{
"id": 37174,
"tgt": "How to avoid the staph infection occurring in future?",
"src": "Patient: Hi . I had an MSSA Staph infection about a year ago that was treated at the hospital and then at home with IV antibiotics. I was wondering if in the future I would be more suseptabile to having that happen again in the future than a person that never had that Doctor: Hello,I understand your concern.I am Dr. Arun Tank, infectious diseases specialist, answering your query.In my opinion you should complete the treatment.Having MSSA won't make anyone susceptible to reinfection if you have completed the treatment.I advice you should carry out culture and sensitivity again after the complete treatment if you found Staphylococcus aureus again than for the eradication purpose you again have to take the complete treatment.I will be happy to answer your further concern, you can ask me on bit.ly/DrArun. Thank you.Dr Arun TankInfectious diseases specialist."
},
{
"id": 136795,
"tgt": "Suggest treatment for generalised tickling sensation",
"src": "Patient: Hi, I suffer from a generalised tickling sensation. It feels like a skin crawling sensation. This has been going on since january 2014. It happens intermittently. Maybe every 5 weeks and lasting for a few days at a time. I have had an MRI and all clear. Neurological exam checked out fine. All bloods also clear. It is very discomforting and wondering have you and ideas?ThanksKaren Doctor: Hi welcome to HCm have gone thru your query regarding generalised tickling sensation . I can understand your concern . This is a condition not rare among the masses . Every 3rd person suffers from such sensation , one time or the other due to many causes . It can be tackled if the cause is removed .Dear ,Your MRI clear, Neurological exam checked out fine. All bloods also clear so lets check out of any other cause of concern Like any other health problems , this can be outcome of numerous disorders- physical or psychological , side effect of drug abuse , diabetic neuropathy , diseases of the spinal cord and peripheral nerves , menopause, allergies , skin cancer and extreme alcohol withdrawal with delirium tremens . As the sweat evaporates, it removes the protective oils on the skin can be cause of concern . A close study to find the cause and removal of it and remedies as under will help remove your agony .Eat healthy food which contains all the essential nutrients needed by the body. Especially the food that you eat should be packed with vitamins and minerals. Vitamin B12 is present in meat and other non vegetarian food, while other B vitamins are available in fruits and vegetables. Antioxidants like Aloe Vera juice ,Termeric powder a spoon with milk , Lemon juice all help eliminate toxins , srengthening your immune system and finally providing resistance from diseases .Keep well hyderated .Body massage helps in improving circulation. It also stimulates the nerves and relieves the abnormal feeling. Avoid alcohol and cigarette smoking , stress ,worry , anger , sleeplessnessKeep your skin clean by washing with hot or cold water whichever suits , with antibacterial /neem soap and apply neem oil . Take a cup of warm water ,mix 5-6 drops of neem oil and apply on your body as lotion for better result Living with the sensation of crawling skin can be extremely uncomfortable Therefore,if at all , this condition persist , consider medical attention / an expert dermatologist opinion is a must to rule out any obvious medical reason. in order to avoid further discomfort.Hope this helps solves your query .Take care , All the best .Don't hesitate to get back if have any further query"
},
{
"id": 154366,
"tgt": "How to treat enlarged prostate grade 2?",
"src": "Patient: Hi, may I answer your health queries right now ? Please type your query here... Age64 weight62 height5'4\" history- HB-11.1,Blood urea-35.12 ,Syrum-1.11,blood urea nitrogen-16.4 ,SyrumK-4.1,SyrumCa-1.9. The patient is catheiterised and prostate enlarged grade2. Requested for kind advice Doctor: Hi, dearI have gone through your question. I can understand your concern.You have benign prostatic hyperplasia grade 2. Alpha blocker like prazocin and finasteride are useful in this condition. It is prescription based drugs so consult your doctor and take treatment accordingly. Your other reports are normal so no need to worry about that. You should also go serum PSA level and ultrasound abdomen regularly.Hope I have answered your question, if you have any doubts then contact me at bit.ly/Drsanghvihardik, I will be happy to answer you.Thanks for using health care magic.Wish you a very good health."
},
{
"id": 17631,
"tgt": "Suggest treatment for a hole in the heart",
"src": "Patient: well I had a heart poblem when I was a child. Doctor said I had a hole in my heart, a lot of people do but mine was in a positon that couldn t be left there so I needed an operation. I think he told me it was in the left side bottom of my heart and I m researching about it now what it was called cause I forgot, I remember something marmalade or something similar. ring any bells pls cause I need to know the name :) tks Doctor: Hello, Consult a cardiac surgeon and get evaluated. Most probably you have a condition called VSD (ventricular septal defect). Generally, it will settle and no surgical treatment is required. Hope I have answered your query. Let me know if I can assist you further. Regards, Dr. Shinas Hussain, General & Family Physician"
},
{
"id": 160832,
"tgt": "Suggest treatment for loose stools and anal rash in a child",
"src": "Patient: I have a 3 year old boy , im really concerned at the minute as i took my son to school and got pulled aside not the first time either , I take him to my doctors as the school and my self are very worried about the loose stool i explained to the doctor who sent his stool away to test for diary intellorant ,which it come back fine . Ive a big concern as it has an awful smell to it as well and im going though so many items of clothing and cant potty train him as if he past wind his stool will come out , it has also made his whole bum sore , so sore you cant wipe it , its hard to get him in a bath ,its horrible seeing him in so much pain when i wipe it he is crying and screaming uncontrolable . I ve used bum cream but it goes then come back 3 days down the line Doctor: Hi,It will be very useful for me if you give few more details- 1) How long this problem is there?2) Does the stool contain blood / mucus / lot of air3) Approximately how many time a day he passes stools4) Whether he passes stools after each feedsIf its recent onset, of a few days only, this may be just a food poisoning which can be treated with antibiotics If it is there for a long time, we have to think other conditions like intolerance to particular food item, malabsorption conditions, chronic worm infestation etc. If I were your doctor (depending on duration), I would have given a short course of antibiotics, zinc supplementation and a probiotic to see the response, and plan further workup accordingly. Meanwhile, to control the rash there- keep the area as dry as possible, apple an emolient (like some diaper creams you used) 3-4 times a days for 7 days.Hope I have answered your question. Let me know if I can assist you further. Regards, Dr. Muhammed Aslam T. K, Pediatrician"
},
{
"id": 153040,
"tgt": "Suggest treatment for stage 4 stomach cancer",
"src": "Patient: my daughter is in stage 4 cancer, stomach, her urine is very dark and she was only going every 48 hrs. now the past 3 day she has been going every day very little and their are little tissues in it. she is eating no food only intake is water, maybe 8 to 16 oz a day Doctor: hello dear. I have read your case. It seems she is very sick and her nutrition is poor. She requires aggressive nutrition support may be in the form of injectable nutritional supplements. Once her condition is better then only chemotherapy can be planned. Meanwhile she will also require pain killers. So keep in touch with your doctor. Take careREGARDS"
},
{
"id": 129478,
"tgt": "What causes pain and swelling in the knee long after the surgery?",
"src": "Patient: Hi I have had knee surgery before . I first had my knee scoped then my doctor did a partial and my knee still hurt so he did a total replacement. It s been around eight or so.years ago now my knee hurts againg it looks like it swells and hurts to walk on it. Does that mean another surgery? Doctor: First you should do blood count for infection of the knee and X-ray of the knee to see the position of the prosthesis if it is worn out or migrated.8 years is not many years for a knee prosthesis but depending on the blood results and X-ray it may mean another surgery"
},
{
"id": 196886,
"tgt": "Suggest medication for erectile dysfunction and to improve sperm motility",
"src": "Patient: Iam a 37 year old male planning to get married. I feel i have a low lipido and erectile strength is moderate. I hit gym every day and workout for an hour, both weigth training and cardio for 5 days a week, I have no diabetes or BP. Can you please advice some general medicines that can improve lipido and erectile strength and also sperm motility? Doctor: helloThanks for query .Based on the facts that you have posted you seem to have facing problem of Erectile Dysfunction .In absence of any major health issue the cause of Ed at young age is mostly Psychogenic in nature and is due to what is called as performance anxiety .Following measure will help you to boost up your confidence and getting good erection and delay ejaculation. 1) Practice regular exercise for 45 minutes followed by meditation for 1/2 an hour in the morning. 2) Take high protein diet rich in vegetables and fruits and Vitamin A,C,D,E.and Zinc 3)Take anti oxidants like Almonds 5-6 everyday4) Avoid alcohol and smoking..Dr.Patil."
},
{
"id": 158531,
"tgt": "History of B-cell lymphoma. Taken radiation therapy and chemotherapy. Have leg pain, difficulty walking straight",
"src": "Patient: Sir/Madam, I am a 24 years old guy. I was diagnosed to be suffering from Mediastinal mass large B-Cell lymphoma in October 2011. I underwent 12+2 cycles of chemotherapy till 16th march 2012 and later on 20 episodes of radiation therapy. During chemotherapy,I was given heavy dose of steroid(75 mg of prednisolone per day) for almost 5&1/2 months. After 6 months of chemotherapy,I started to have some pain in my both legs which was occasional at the beginning and became persistent later on. From february 2013 i started to have difficulty in movements and could not walk straight and properly.Now it is diagnosed to be AVASCULAR NECROSIS, which is most probable to be 3rd and 4th stgae .Can you please tell me whether it can be treated through ayurvedic medicines completely. If yes,how much time can it take and how much can it cost me. Doctor: Hi, Sorry to hear about your condition. Avascular necrosis can be due to chemotherapy, excessive steroids use etc. Most common site is hip joint. Anyway, conservative management has little role . Many a time that can be benefited by surgery like total hip replacement, metal on metal resurfacing. Consult your doctors. Role of ayurvedic medicines is not known to me."
},
{
"id": 36810,
"tgt": "Suggest cure for a boil in the anal area",
"src": "Patient: Sir I travelled for 48 hours...been sitting. Because of sitting alot i had to force in the washroom to pass stools. I normally never have to force. I exercise regularly. I am completely fit. Never had constipation. When i finally arrived back home from that tiring long journey . The next day i felt a small bump like lump or a boil on the right side of my anal ring right outside anal opening. I checked in mirror. It had skin color. No pain at all. After 4 days i went to doctor. He gave me antibiotics i took for 2 weeks but still the boil is there and still no pain at all just an uncomfortable feeling of its presence.its smaller then a pea. Kindly guide me what to do about it. I am tavelling soon again in 4 days. But this time i changed the airline. Only 22 hours travel. Atleast.shorter then 48 hours. I am using an antibiotic cream on it. And i keep is clean. Doctor: Hello According to your history of travel ,this bump around the anal ring is hemorrhoid ,not a boil ( because of undue force for stool ).Since you develop this ,so next DON'T STRAIN at all ,usually straining ( force ) aggravate the condition.For this the best way is take some phenophtaleine tablet for constipation during journey.For this take some anovate type cream ( or nefidipine +local anesthetic cream ).I don't advise any antibiotics to my patient . In my opinion consult a physician and get a conservative treatment for hemorrhoids ( 2nd opinion ).Good luck."
},
{
"id": 166267,
"tgt": "What causes frequent urination in kid?",
"src": "Patient: Today, my daughter s teacher told my husband and i that she realize that my daughter has been passing water frequently at school. I have not realize it at home, but however, i remember she going to the bathroom frequently when we were at church often. My daughter is six years old. Doctor: hi, the most common cause of frequent urination in a 6 year old child is urinary tract infection. Other cause could be Diabetes mellitus. you should do urine routine microscopy test and urine culture test and blood sugar levels. Based on reports, treatment for urinary tract infection or diabetes needs to be started. Review with reports. Take care."
},
{
"id": 8578,
"tgt": "Any suggestions for laser/ chemical peel that can rapidly remove post acne dark spots?",
"src": "Patient: Hi! I have post acne dark spots.. and to treat it I have gotten a v beam laser.. mild peel.. and most recently (tuesday) a Matrix RF laser.. and nothing has really gotten rid of these spots.. I need something that will remove them quickly because i am leaving to EU and want my skin to be nice before I go.. Any recommendations for a laser or chemical peel i can do that will really reduce these spots?? Thank You! Doctor: Hello,I have gone through your query and understand your concern.I understood you are affected by dark spots due to acne.But it is difficult to expect the cure with in a few days.As you had laser treatment recently, Please wait up to 10 days after laser therapy.It may take up to 10 days to heel and your skin may look better than before.It work by initiating body's natural healing process sweeps away older, damaged tissue and rebuilds it with fresh, new collagen and elastin- the crucial building blocks of normal-looking skin.You need further seven to 10 sittings for complete effectiveness of laser treatment in acne.Thus please continue the treatment and wait for effectiveness.Hope this helps.Please write back for further queries.Wishing you good health."
},
{
"id": 213925,
"tgt": "I suffer from depression, please explain what is going on ?",
"src": "Patient: I am 17 years old I highly believe I suffer from depression. I am constantly sad and at times I can't even go out in public because I begin to cry and cannot stop it. Before my monthly cycle I begin to get excessively sad, I have nightmares and wake up very afraid. Yesterday and today I have had moments where I feel really sad and begin to cry but I am over 10 days away from my monthly cycle so i dont even know if that is exactly the cost. I have been this way for probaly 2-3 years now and I am tried of feeling so unhappy. Can someone please explain what is going on. Doctor: Hi Kelly. I can fully understand the way u feel before your monthly cycle because i feel exactly the same way. its terrible. i cry for no reason at all, am jittery, edgy, sad all the time. and this happens to me as much as 7 day before my periods. but as soon as my periods satrt i start to feel better. sometimes i take alprazolam when i cannot bear it. i have spoken to my mother and sister about it and they also have somewhat similar symptoms but much much milder than mine. All that i have been able to find out about it till now is that it is premenstrual syndrome (PMS) and it is common, but nobody seems to have a cure."
},
{
"id": 2591,
"tgt": "Could anal sex cause pregnancy?",
"src": "Patient: what is the probability of getting a girl pregnant if you position your penis between her buttocks so that the penis is heading towards the anal area but doesnt penetrate it and you just thrust up and down (girl has a large ass) and then pull out before ejaculating Doctor: hello...thanks for trusting the healthcare magic doctors for ur health related queries.I guess ur concern is about...could anal sex causes pregnancy.my straight forward answer is...NO, IT NEVER CAUSES A PREGNANCY, IT'S IMPOSSIBLE.let us discuss about factors required for getting pregnancy. for getting pregnancy, penis should be inside the vagina n ejaculation should occurs around the ovulation( during which follicles rupture n ovum comes out from the ovary). so sperm can fertilize the ovum n pregnancy will happen.but in anal sex nothing is happens as explained above. so there no chance of getting pregnant in case of anal sex.with above discussion u try out my suggestions-1. u r still worried then ask ur girl to do urinary pregnancy test to rule out the pregnancy.2. try to do sex with protection, so u can not in dellelima of pregnancy.I hope the above information is useful, informative and helpful for uregards- Dr sudha rani panagar"
},
{
"id": 86472,
"tgt": "Suggest treatment for severe abdominal infection",
"src": "Patient: Yes, a family relative has a prostate removal 10 days ago. Then yesterday the hospital found a nick in his intestines, and now he has major infection in his abdominal area. They had to do major surgery again. How long will it take him to recover? Thanks!! Doctor: nick in intestine means there is peritonitis due to irritation of infective materiel leaks from intestine. a exploratory laprotomy is needed to correct that. recovery depends on that during surgery what is condition of intestine. if nicked part is healthy they can repair it but if it is not healthy than they need to take it out to abdominal wall and wait till i become healthy. after that need to do 2nd surgery to repair and replace that part"
},
{
"id": 53512,
"tgt": "What causes higher SGPT levels?",
"src": "Patient: Dear Dr. I am 42 years old and my sgpt is 46 which remarked as high value (by Lab). Also I have a gallstone with no attacks signatures.my HIV & HBS results are negative. pls. let me know the meaning of sgpt high values remarks in my blodd biochemistry results. Doctor: Hi and welcome to Healthcaremagic. Thank you for your query. I am Dr. Rommstein, I understand your concerns and I will try to help you as much as I can.Liver enzymes are reasonably sensitive indicators of liver damage or injury from different types of diseases or conditions, and collectively they are termed liver tests or liver blood tests. So the next step is to look for certain liver damage and most common casue is fatty liver disease. It should be confirmed by ultrasound. Other causes are viral hepatitis,cirrhosis, medications, alcohol intake or autoimmune diseases. To verify exact cause, US, bilirubin levels and tumor markers should be additionaly done Then appropriate treatment can be started.I hope I have answered you query. If you have any further questions you can contact us in every time.Kindly regards. Wish you a good health."
},
{
"id": 52155,
"tgt": "What causes raised levels of infection in mu liver?",
"src": "Patient: Ithink I may have crohns disease,Ihaven t been diagnosed as yet buy blood and fauces test came back saying I did then the next day my dr recieved aletter from pathology saying disregard those results as they were re tested and nothing showedup but I stil have raised elevations of infection showing up in my liver.I have all the symptoms of chrohns. I am becoming metally drained as I feel so sick and my dr can t seem to get any answers or diagnosis Can you please help me. Fiona Monti Doctor: Hello, How do you know that your suffering from crhons disease? Is there any loose stools? Is there any weight loss? Is there any blood or mucous in stools? Is there any hemorrhoids or sinuses around the anus? It is to be diagnosed by colonoscopy and ultrasound abdomen. So please consult with your Gastroenterologist he will examine and treat you accordingly. Hope I have answered your query. Let me know if I can assist you further. Take care Regards, Dr. Penchila Prasad Kandikattu"
},
{
"id": 74650,
"tgt": "Suggest remedy for constant voilent cough, choking spasms and laryngospasms",
"src": "Patient: hi i am a 59 yr old female that has been dealing with violent coughing and choking spasms and also laryngospasms for over 5 years. i have been to ent specialists, pulmonologist, neurologist, general practitioners...taken dozens of tests and medications...ended up at stanford university ent department a couple of years ago without enough relief. am currently taking lyrica which seems to be about the only thing that helps...i HATE the side effects but it has mostly stopped the laryngospasms (which were terrifying) and has helped the frequency, duration, and intensity of the choking episodes by about 50%. but the other 50% still impacts my quality of life significantly...involuntary vomiting, stress incontinence, and the forceful expulsion of saliva, mucus, food and drink are major deterrents to interacting with people...both publicly and privately. so my question is... where do i go now, who do i see? PLEASE HELP! Doctor: Hello,Sorry to hear about your issue. I thoroughly have gone through your query. Now there is not any institute in the world for you to go for further filing. Another case and make the load of files heavier in your desk!Only thing that can help you:\u2022\u00a0\u00a0\u00a0\u00a0\u00a0Highly dedicated meditations \u2022\u00a0\u00a0\u00a0\u00a0\u00a0Deep breathing exercises \u2022\u00a0\u00a0\u00a0\u00a0\u00a0Concentration exercises \u2022\u00a0\u00a0\u00a0\u00a0\u00a0YogaOnly naturopath is left for you can try it, I am sure if you will do it honestly, sincerely, faithfully it will give you tremendous results than any other institute in the world and you would like to remember me in your life.Hope I have answered your query. Let me know if I can assist you further.Regards,Dr. Bhagyesh V. Patel"
},
{
"id": 38367,
"tgt": "What causes chest conjustion and cough?",
"src": "Patient: I have been feeling tired and headache for about 4 days. I starting to feel congested, when I take a deep breath, I start coughing. The last two nights I have woke up with upper back pain ans feeling breathless...I am 61 yrs old healthy...I take thyroid replacement meds but that is all Doctor: Hi, thanks for using healthcare magicIt is possible that you may have an upper respiratory tract viral infection (common cold or flu). This would present fever, congestion, fatigue, headache.Treatment would consist of : rest, fluids, decongestants and cough medication.Decongestants can be oral or topically applied to the nose (eg otrivine or normal saline). They are available over the counter and can be found combined with cough medication.If the cough becomes wet then you may need a mucolytic (medication to break up mucus in the chest). This is also over the counter.I hope this helps"
},
{
"id": 49111,
"tgt": "Is it good to see a renal dietitian for stage3 CKD?",
"src": "Patient: Hello Dr. I Have seen a diabetes nutritionist, although I don t have diabetes. It was helpful to a degree. I now have phosphorous issues, in my stage 3 CKD and am wanting to see a renal dietician. Do you think this is a good idea? Thank you for helping me, in advance, Sincerely, Cherril Doctor: Even being a CKD-3 also poses some restrictions on day to day life styles, like protein restriction, salt restriction, water in take. hence it is good to consult your nutritionist along with your treating nephrologist."
},
{
"id": 114258,
"tgt": "I am getting low back pain frequently",
"src": "Patient: Hi i am getting low back pain pain frequently Doctor: Most of them are due to poor muscular tone. Aerobic and stretching exercises along with heat therapy is of help to many people."
},
{
"id": 139138,
"tgt": "Suggest treatment for severe knee pain",
"src": "Patient: Sudden Sharp pain in right knee right side of knee . No injury no fall,? Just started as a instant sharp burning pain inside the knee from front right side of knee to the back of right side of knee . To painful to walk lifting up n down my knee. And even just resting the pain is realy bad 9/10 whats wrong with me Doctor: Hi,Thanks for your query.your pain is due to Arthritis. When bone is not able to take weight due to arthritis than it has to be shared by muscles. Also some time it can be due to nerve compression. You can judge your self. If your pain increases on walking and relieved with rest, it is suggestive of nerve related pain.Arthritis pain is present even when you are taking rest.I do hope that you have found something helpful and I will be glad to answer any further query.Take care"
},
{
"id": 6611,
"tgt": "Will it be possible to get pregnant after laparoscopy surgery ?",
"src": "Patient: Hi doctor , I am 29 year old woman..married from last five year. Have no children.. My periods are not heavy...only just for a one day \u2026.my weight is 45kg & my height is 5\u201d 4\u2026 i consulted doc n did my sonography but all report s is clear now doctor suggested me for laparoscopy surgery . \u2026.. after getting laparoscopy surgery can I get pregnant ... Doctor: Hi.. Laproscopy is done to know wat problem you are having and depending on that the treatment will start, laproscopy is not a treatment its done in ur case as investigation. All the best."
},
{
"id": 2303,
"tgt": "What could be the reason for not concieving and not having periods even after stopping YAZ birth control pills?",
"src": "Patient: My medical insurance ran out about 2 months ago and that is when I stopped the generic YAZ birth control pill. I am almost 24 yrs old & I have been on different types of bc pills for 10 years and this is the 1st time I have not been on it. Now, I have not had a period for this entire time and am concerned. Took 2 at home preg test and both negative. Also, I have dropped about 20 lbs over this time too (was 145 now 125) and have been having very severe anxiety/depression... Over the past 10 yrs I have also experimented with all types of drug use. Is my body changing for good?? Doctor: Hallow Dear,Pregnancy test on urine is reliable after about a week of missed period; earlier these tests can report false negative results. However, since you have repeated the test twice, it is likely that later test was done after about a week of missed period. If not you may repeat the test on overnight early morning first urine sample after 4 more days. If this test also reports negative, you are not pregnant. There are multiple causes apart from pregnancy for missing a period. However, from your history, it appears that you are under tremendous depression and anxiety. Your feeling for permanent change in the body also seems to be out of anxious feeling. This is one of the very important and common causes of missing a period; and then irregular menses subsequently. Please report to some counsellor/psychiatrist with your anxiety/depression issue for effective management. Hormonal imbalance also can cause irregularity in the menses. Please report to your Gynaecologist for thorough investigations and management.Meanwhile you may take some progestational preparation like Deviry for 5 days and then hope for the withdrawal bleeding with 4-7 days after that, if you are not pregnant. This medicine should be taken with a Gynaecologist's advice only. I hope this helps you. Dr. Nishikant Shrotri"
},
{
"id": 145347,
"tgt": "What causes persistent tightness in the neck and upper back ?",
"src": "Patient: Since 2011 I have been having c-spine issues. In 2011 I had eight injections in my c-spine and a full round of physical therapy. A couple of months ago I completed another round of injections and was to start physical therapy. During my PT evaluation, I was told c-2 and c-3 were shifted greatly to the right. The therapist was able to move them back into place but they immediately shifted back to the right. At that time I was unable to turn or tilt my head to the right. Two weeks later I went to PT and was told c-2and c-3 we re not shifted as bad and I was able to turn and tilt to the right. I have had continuous muscle tightness in my neck and upper back to below my scapula. Radial and ulnar nerve pain, weakness and tingling down both arms. Stabbing pain between my shoulder blades. Pain in both shoulders. The injections helped a lot of the nerve issues but have had no effect on the muscle issues. For the last several months I and others have noticed my head shaking. As of three to four weeks ago, I am now having upper body shaking both visibly and inside and my head is shaking worse. These are getting worse and worse where today I was up long enough to take a shower then was shaking so bad I had to lay down for 45 min to settle down. And to top it off, I can t eat and am almost always nauseated. Are these new symptoms all related to the c spine? Getting a little scared.. Doctor: Hello! I understand your concern! In my opinion the troubles you have may be related the a cervical spine problem. The pain, body shaking, tinglings on hands are all concerning symptoms of a serious cervical problem causing also irratation of the nerves. I would recommend you a cervical MRI to examine the vertebras and the spine for probable damage. The size of the spinal canal as it may be narrowed. If serious problems are encountered a surgical treatment may be needed . In the meantime I would recommend you a cervical collar. It will be very helpful to your situation. Hope to have been helpful!Thank you for using HCM!Best Wishes Dr. Abaz Quka"
},
{
"id": 13465,
"tgt": "What causes itchy rashes on the stomach after drinking apple juice?",
"src": "Patient: Hi Doc.Whenever I have apple juice I seem to get an itchy rash on my tummy and around my back it\u2019s not really red there is just 2 red lines that when I stretch the skin out they go away. I\u2019m not allergic to apples but I think I may guy be allergic to something in the apple juice like mabey the things they might add like reconstituted apple juice or artificial colourings. I just want to know what I have and if it just an allergy Doctor: Hi, You are most probably suffering from hives (urticaria). You can confirm the diagnosis by avoiding the apple juice and observing the absence of new skin lesions. The rash is managed by Calamine lotion application and taking antihistamine tablet for a week. If you have any other problems, do consult your dermatologist for further management. Hope I have answered your query. Let me know if I can assist you further. Take care Regards, Dr Siva Subramanian, Dermatologist"
},
{
"id": 117545,
"tgt": "What does esr level of 34 mean?",
"src": "Patient: hi Dr,my mother was complaing about neck pain and shoulder pain. she saw the doctor yesterday and the doctor advised her to get ESR test, uric acid test and Artharitis test.uric acid and artharits tests are within the normal range but her ESR is 34, can you please suggest Doctor: Hello, Thnx to contact us. If I am your treating doctor I advice you that your mother is free from arthritis and gout and had high ESR. ESR is non specific marker of the inflammation. So it raises in a variety of conditions. Which type of inflammation you have is we have to diagnose. I also want to know what age you have, your sex and further examination to establish the diagnosis ? I can only say that ESR is raised here from the history that you have mentioned. If you have anything else to ask, kindly contact me Dr. Arun Tank."
},
{
"id": 170069,
"tgt": "Can Nestrumgel be used with breast milk?",
"src": "Patient: Hi Doctor, I need to know about nestrumgel for mixing with baby formula and or breast milk - how long can the mixture be kept before mixing it into the milk? Can it be used with with breast milk? Please I need an answer a.s.a.p.Regards Dalena Mocke (Grandmother of a 6 week old baby) Doctor: Hi,Welcome to HCM. A 6 week baby should be given exclusive breast milk and nothing else should be given. Nestrumgel should not be mixed in breast milk for feeding. I hope this will help you. Take care."
},
{
"id": 46265,
"tgt": "What causes pain and tenderness in kidney area?",
"src": "Patient: Hello. Back in November, I had 2 kidney stones that were passing at the same time. They were found to be 1 mm and 3mm back when I was in the ER. They did a CT scan and found them. Since then, I have still been having some discomfort in my left side, and some tenderness around my left kidney. Sometimes I feel a sharp pain or like a pulling in my life side, not near my ovaries, but further up and toward my back. It is worse when I sit down. I am also feeling a bit light headed and nauseous. Could this be more kidney stones? I am not feeling intense pain, it's more of a throbbing or feels at times like a muscle when I put my weight on my left side. I can't lay on my left side without it being uncomfortable. I have a doctor's appointment on the 13th for this, but I don't know if this is related to the kidney stones, or something else. What are your thoughts? Doctor: Hi and welcome to HCM.As an Urologist, i can understand your anxiety.1mm and 3mm stones are insignificant and would've been flushed out by now.A proper CT scan would've detected any other abnormality in the abdomen.So most likely,you've some muscular sprain,which increases with strain.Anyway do an ultrasound scan-KUB before you see the doctor.Drink about twelve glasses fluids daily,as stones can recur.Dr.Matthew J. Mangat."
},
{
"id": 142246,
"tgt": "What causes tingling sensation around the calf area after experiencing vertigo?",
"src": "Patient: I had severe vertigo and had to be helped to walk, I vomited once later in the evening. I felt discombobulated the next day and the next morning I had dry heaves and felt an intense tingling sensation starting from mid-calf moving slowly up my body, it felt hot/cold and then once it hit my midsection, my entire body was clammy. Doctor: Hello!Welcome on Healthcaremagic!Your symptoms could be related to an inner ear disorder or a viral infection (considering the tingling in your body). For this reason, I would recommend consulting with your GP for a careful physical check up and some tests: - complete blood count- blood electrolytes- thyroid hormone levels- PCR, sedimentation rate. Consulting with an ENT specialist and performing labyrinthine test would help exclude an inner ear disorder. Hope you will find this answer helpful!Best wishes, Dr. Aida"
},
{
"id": 33556,
"tgt": "What causes abscesses in body?",
"src": "Patient: I have an 18 month old daughter who has had 6 abscesses in the past 9 months They have progressively been getting larger she has one now that is being treated but the last one was the size of a golfball and put her in the hospital on IVs and required surgery are their any conditions you can think of that may be causing this. Doctor: Hi i did review your concern.recurrent abscesses can be due to immunodeficiency due to malnutrition or primary immunodeficiency. If she is well fed I would recommend you to get her tested for primary immundodeficiency disorder especially chronic granulomatous disease where there is a enzyme defect in neutrophils and leads to formation of recurrent abscess.I hope this helps.wish you all the best.Thank you for using healthcaremagic."
},
{
"id": 163940,
"tgt": "What causes fever and runny nose?",
"src": "Patient: My three year old has a slight fever, runny nose, and watery eyes (they are crusty after he sleeps). this just started two days ago. Is this just a cold or infection starting. He has had a flu shot. Doctor: this is probably due to allergy. please give him an antihistamine as citrizine once daily before going to bed"
},
{
"id": 147673,
"tgt": "Is there any treatment for fibromuscular dysplasia of internal carotid artery?",
"src": "Patient: I was diagnosed with fibromuscular dysplasia of the right internal carotid artery at the base of my skull. I was told 17 years ago the technology did not exist to do a bypass on the artery. Is the technology there now and do I need to have it rechecked? I have been on warfarin ever since to keep the blood as thin as possible. Doctor: Hi,Thank you for posting your query.I have noted your symptoms and treatment received so far.I think you are on correct treatment and warfarin should be continued for long to prevent any brain stroke.You could see your neurologist for review. I do not think that a bypass surgery is needed, as you are doing so well on medical treatment.I hope my answer helps. Please get back if you have any follow up queries or if you require any additional information.Wishing you good health,Dr Sudhir Kumar MD (Internal Medicine), DM (Neurology)Senior Consultant NeurologistApollo Hospitals, Hyderabad, IndiaClick on this link to ask me a DIRECT QUERY: http://bit.ly/Dr-Sudhir-kumarMy BLOG: http://bestneurodoctor.blogspot.in"
},
{
"id": 52846,
"tgt": "Suggest remedy for low blood pressure and gall stones",
"src": "Patient: my wife just got a blood pressure reading of 107/51. she is 19 weeks pregnant, and has gall stones. she was supposed to have her gall bladder removed, but one month prior to the surgery we found out she was pregnant and had to postpone. she is in a lot of pain from it regularly. should i be concerned over the low bp? Doctor: Hi the low BP does not matter. For the gall stone pain ask her Gynaecologist if she can take Uro deoxy choleic acid, which I think she can if she has crossed 3 months of pregnancy--regards"
},
{
"id": 131943,
"tgt": "What causes splinter-like feelings on upper leg?",
"src": "Patient: My younger son has multiple splinter - like feelings on his upper leg, he originally thought he had glass in his leg but we couldn t find any splinters or glass shards, and he wasn t near anything glass as he had just got out of the shower. What should we do? Doctor: hi Your son has abnormal sensation of multiple splinters in upper thigh . In my opinion this kind of feelings come when there is pressure on the nerves originating is the spinal cord. I would recommend Pregabalin , pain killers, and suggest S Vit B12 and MRI Spine( Lower Spine)."
},
{
"id": 151576,
"tgt": "Facial paralysis, eye not closing completely, bells palsy. Taking ibuprofen, gupisone. Correct medication?",
"src": "Patient: hello doctor 2 days before my right side of face is not working and my right side eye is not closing full. yesterday i went to doctor.the doctor said it is bells palsy .and he have given me ibuprofen 400mg and gupisone 20 mg.he told me to use this tablet for 1 weak.i want to know that the doctor have given me right tablets and i want to know that how time it will take to become ok. Doctor: Hi, Thanks for writing in. Even though the treatment you're receiving is right, the dose is not ok, Usually, Bell's palsy is treated with steroids at a dose of 1 mg/kg for 6 days. In addition, you would need acyclovir or valacyclovir. The recovery time is variable and varies from person to person. Please see another ENT surgeon. Regards"
},
{
"id": 138633,
"tgt": "Suggest treatment for lower back fracture",
"src": "Patient: I was in a bad Dirt Bike Accident at a desert race saturday landed from about 15 ft square on my bum...thought I broke my lower back but now seem to be walking ok but havent pooped (sorry) since And have eaten a ton my insides feel very tender and kidneys are killing me. is that bad? I refused ambulance care btw...just tried to walk it off Doctor: Dear patient Fall from height puts you at risk of vertebral compression fracture and this needs to be investigated. Compression fracture may also lead to nerve compression and both lower limb weakness. I would advise Xray of dorsolumbar spine anteroposterior and lateral views to rule out bony abnormalities. MRI of dorsolumbar spine is also required. MRI gives detail of vertebra, nerves and even soft tissues. Visit radiology center nearby you and get it done. Meanwhile take strict bed rest and avoid stress to your spine. You need to consult expert spine surgeon with report. All the best."
},
{
"id": 196565,
"tgt": "Is it not advisable to shave in 13 years of age?",
"src": "Patient: hello doc, my name is james when I was 12 13 i was in cadet camp. My sgt gave me orders to shave when I only had peach fuzz. 6 months down the road I started growing a full beard my doc told me not to shave because it would mess up my pituitary gland and would stop growing. is this true? Doctor: Hi and welcome to Healthcaremagic. Thank you for your query. No, this is absolutely not true and shaving has no anything to do with hormones, pituiary glands or growing. So you can shave as much as you want if you have this need. You wil grow as long as your puberty last.I hope I have answered you query. .Kindly regards. Wish you a good health."
},
{
"id": 85961,
"tgt": "What does pain in the lower abdomen associated with aches in the testicles and lower back indicate?",
"src": "Patient: Over the last few days I ve noticed a dull ache in my lower right abdominal area associated aches in my testicles and lower back. I am also constipated. Yesterday, and strangest of all, I began noticing an unusual odor emanating from my groin area- I ve never experienced this. I m 63 years old, and still have my appendix. I ve had some recurring issues that were diagnosed and treated as prostatitis. I had a colonoscopy about 6-months ago, and a rectal ultrasound about a year ago- both were normal. Doctor: Hello and Welcome to \u2018Ask A Doctor\u2019 service.I have reviewed your query and here is my advice.As per your description, that could be appendicitis for which get evaluated first.Examination for guarding rigidity and possibility of a hernia should check if swelling in the groin occurs during coughing.You should go for ultrasound abdomen, and Diverticuli should also keep in mind.Hope I have answered your query. Let me know if I can assist you further.Regards,Dr. Bhadresh Lakhani"
},
{
"id": 64325,
"tgt": "Is cyst common problem for athletes?",
"src": "Patient: Hello. My son has a small cyst that he was treated for with cipro. It was opened then closed up. He is a college baseball player and the weird thing is, two other baseball players on his team, got them at the same time. One team mate had surgery. This past week, the team physician said my son needs to have his surgically opened again and cleaned out, then packed. My question is, What is the recoup period and is this a common problem amongst athletes? Is it coinsidental that other team mates got the samething, same time? Doctor: Welcome to healthcare magicI am Dr Fahim and I will help you with your problemI have gone through your question. You have not mentioned the location of the cyst. In athletes, joint bursa is common condition because of the increase wear and tear of joints. The treatment of which is surgical excision, if it is large enough and bothering the patient. You need to tell us, the location of the cyst, when and why he was operated. What problem is it causing. Only then we will be able to help you in a useful way.I hope my answer will help you. Do rate it, if you like it.Regards"
},
{
"id": 102381,
"tgt": "How to treat dust and smoke allergy mediated dark skin patches?",
"src": "Patient: HELLO DOCTOR . i HAD MET AN ENT DOCTOR. mY WIFE HAD SERERVE COLD AND NOSE LEAKING LOT. SHE WAS ADVICE TO TAKE RUPANEX -M . THE DOCTOR SAID SHE ALERGIC OF DUST, SMOKE, WHICH HAS CREATED PROBLEM. AFTER TAKING 40 TABLAT OF RUPENXE -M .THE DOCTOR STOP THE SAME AND SAID IT IS NOW OK. BUT TWO DAYS BACK IT STARTED AGAIN . HER FACE CHANGED WITH DARK PATCHES .NOW SHE STARTED TO TAKE THE RUPANEX -M .NOW AGAIN SHE IS OK BUT THE SKING COLOR HAS NOT CHANGED .pLEASE ADVICE. Doctor: Hi Rupanex m contains rupatidine n montelucast which are anti histaminic n mast cell stabalizer respective and was prescribed to you for alergic rhinitis . But it wad not indicated for you to take it for 40 Days .Hence ur ENT surgeon advised you to stop n have it vn symptomatic again. Again developed allergic rhinitis with skin rash so take same medication for 5 days and if symptom persists or aggravetes then visit versatility clinic n get reviewed . Prevent condition to which allergic to use mask while going in dusty env or applyn simple hanky can work, avoid food allergic , clean house n bed if possible with vacume cleaner n mop etcHope u stay in good health. Thanks n regards"
},
{
"id": 20964,
"tgt": "Is blood pressure of 144/101 dangerous to health?",
"src": "Patient: I've had high blood pressure issues with weight gain. I've come down in weight from 247 to 234 in 6-8 wks @ 6'1\" and my pressure is currently 144 over 101 from a home pressure gauge which also has come down. w/ diet and walking. Is this dangerously high at this point? Doctor: Hi,Welcome.Bp of >140/90 is considered abnormal & should be treated. High bp can cause serious life threatening side effects. So you should consult your doctor.Thanks"
},
{
"id": 149254,
"tgt": "Vivid hallucination, fear, anxiety. Possiblity of injuring amygdala in car accident?",
"src": "Patient: I have a few questions concerning the amygdala. 1) If I was in a car accident, is it posssible to injure your amygdala? 2) If my amygdala was slightly injured in an accident, would it be possible to have unconcious vivid hallucinations in which fear, anxiety, control, emotional trauma and loss of love were present. 3) Can an injury to the amygdala cause a person to become unconcious for a small period of time or go into a coma. 4) Can an amygdala be repaired? What are the steps involved? 5) Can a severe loss of a loved one cause a person to injure the amygdala? Thanks, Juan Doctor: Hello,welcome to Healthcare Magic.Amygdala can be damaged in car accident and do not get repaired as it contains neurons. Rather than worrying about amygdala, it is better to get your MRI brain done, which will show current situation of amygdala.From the nature of your query there is chances of having anxiety disorder in your case, which should be assessed and treated.Wish you good health and all the best.Regards,Dr Ashish Kumar Mittalwww.99doctor.com"
},
{
"id": 169698,
"tgt": "Suggest cause and remedy for itching all over the body",
"src": "Patient: i have a nephew of 2 years when he was 5 or 8 months old he started itching his whole body.his parents went to all the pediatric allurgist in middle east they say that he has enlarge liver and becouse of that his enzym system isnt working properly.Still they dont tell us the name of the disease till now we dont know what has happend to him please please please tell me what to do.he even dont have any food allurgy.i will really appriciate each and every reply thanks Additional Details we have obviously use body washes medicated soaps lotions medicines still no use we have change alot of pediatric allurgist Doctor: Hi Dear,Welcome to HCM.Understanding your concern. As per your query your nephew have symptoms of itching all over the body which is probably due to lowering down your immunity and making you prone for infection. It could be due to allergic reactions and contact dermatitis. Need not to worry. I would suggest you to apply wet cold sheets on your body to lower down body temperature. and itchiness. You should avoid touching or pricking rashes of body. Take hot milk rich in turmeric as it will boost immunity. Take combination of antiviral drug along with anti-inflammatory drug. Your symptoms will resolve in 4-5 days. If symptoms keeps on persisting visit ENT specialist/ dermatologist once and get some routine blood investigation and skin patch test done. Avoid hot, spicy and sharp food substances. Take diet rich in multivitamins and vitamin C. Hope your concern has been resolved.Get Well Soon.Best Wishes,Dr. Harry Maheshwari"
},
{
"id": 47766,
"tgt": "Does excessive smoking of marijuana lead to kidney stone formation?",
"src": "Patient: Been smoking marijuana for past few months. Recently developed urine problems and turns out it was small kidney stones. Medication to melt them away is working. Question is can excessive marijuana smoking lead to these stone formation or play a part? Thanks Doctor: Hi,Thanks for writing in.Marijuana directly does not lead to formation of kidney stones. However an addiction to marijuana might cause you to reduce drinking water and this might predispose to a condition leading to formation of kidney stones. What happens is marijuana is a recreation drug and the person gets busy and will not find time to drink adequate amounts of water and juices.Therefore indirectly smoking marijuana might drift your mind away from normal pattern of eating and drinking water and this leads to build up of kidney stones. Please maintain a normal daily water capacity of 2 liters for good health. If you have kidney stones then please drink adequate water and also visit washrooms regularly. Please do not worry."
},
{
"id": 150544,
"tgt": "Discomfort breathing, knot in spine at lung level, numbness in limbs, painful. Can you guide me?",
"src": "Patient: Several months ago I noticed it was uncomfortable for me to breathe and noticed a knot on my spine at lung level. I started losing feeling in my arms and hands and feeling tingly, the pain was awful. At the same time, I notice if I was laying flat and tried to sit up I had this horrendous pain in my neck and felt paralyzed. I was also limited in my range of motion in my neck; turning to the right, holding phone on right side, all caused significant pain and difficulty to return to normal position. Over the months I have felt this knot move up my spine, all the same symptioms, now it feels as though it is behine my shoulder blade; same painful sypmtoms, neck, arms, hands. I have no answers. Can you guide me? Doctor: hi thanks for Writing to HCM, your problem is surely very painful and bothering you a lot but you didn't mentioned any medical help/ treatment taken for it till now. The knot (or lump) is a major concern here. It has to be well diagnosed first before starting any kind of treatment. A X-ray spine cervico-thoracic showing that knot (both AP & LAT) view is required, and some blood tests- CBC, BLOOD SUGAR. If anything comes out in these tests then treatment has to be done accordingly. We definitely need to know the origin of that knot. Or if all the tests are normal and it is a muscular trigger point then physiotherapy can help you. hope this helps you wish you good health"
},
{
"id": 52074,
"tgt": "Bleeding during motion",
"src": "Patient: i had blood two times during motion... what is the problem? Doctor: Hello.. Welcome to HealthcareMagic.. go to ur nearby family doc for examination because it may be piles or fistula problem or if it comes in motion then it be any internal bleeding but without examination no doc tell u what is the exact problem... but one thing as u know there is some problem so to ur nearby family physician as soon as possible unless the problem may increase and tough to handle... Thanks.. Dr.Bhagwan dass"
},
{
"id": 202824,
"tgt": "Penis slighlty inclined towards left side, erectile dysfunction, underweight. Treatment?",
"src": "Patient: hi doc i am suffering from this problem for quite along time i hoped that with passage of time it will automatically resolve but unfortunately not yet happened.... my problem is that i had a bad habit of masturbation when i was 16, with passage of time that fascination came to end from last two or more years i have stopped doing that but i realized that despite of leaving that habit i have sperm injecting problem certain amount of sperm automatically discharges when i am asleep and some drops also come after i piss...... kindly tell me the solution of this problem i am 26 now and have weight only 51kg i am going to get married but due to this situation i am confused what to do moreover due to masturbation my pinis is slighlty inclined towards left side.... Doctor: HelloThanks for your query,based on the facts that you have posted it appears that you had been indulged in masturbation since many and now facing problems related to sexual activities.First of all there is myth in a mind of common man that excessive masturbation is the reason of all sexual problems But I would like to state that it has been discussed in scientific forums all over the world and proved scientifically that masturbation does not have any negative effect on any organ or system in the body.All the problems that you are facing now are mind related and due to anxiety.Following measure will help you to boost up your confidence and getting good erection.and help to improve your general health.1) Practice regular exercise for 45 minutes followed by meditation for 1/2 an hour in the morning.2) Take high protein diet rich in vegetables and fruits and Vitamin A,C,D,E.and Zinc3)Take anti oxidants like Almonds 5-6 everyday..4) Avoid alcohol and smoking..The problem of night emission is common at the age of adolescence and nothing to be worried about Dr.Patil."
},
{
"id": 66094,
"tgt": "Should i be concerned about the lump on my breastplate?",
"src": "Patient: I am a male age 55 and recently lost 40 pounds. I've noticed a lump on my brestplate the size of a nickle, just below the V of my colar bone. Was this there before and I never noticed it because of the extra weight or is it something I should be concerned with? Doctor: Hi, dearI have gone through your question. I can understand your concern. You may have soft tissue tumor or benign cyst or tumour related to breast. You should go for fine needle aspiration cytology or biopsy of that lump. It will give you exact diagnosis. Then you should take treatment accordingly. Hope I have answered your question, if you have doubt then I will be happy to answer. Thanks for using health care magic. Wish you a very good health."
},
{
"id": 16215,
"tgt": "Itching below breast, back and inner thigh, have red bumps, applied rubbing alcohol",
"src": "Patient: some parts of my body itch like underneath my breast , parts of my back and inner thigh. I scratch and then I start to see some little red bumps start to show it almost looks like when you scratch after ants bite. The itchness happens at intervals not all day long. I noticed that I scratch mostly at night and I have been putting rubbing alcohol after I scratch. Can you please help! Doctor: Hi I am Dr Manoj.... The kind of problem you are getting may be just a simple urticaria or a fungal infection which is very common in the areas u have mentioned....don't use alcohol....you can use any anti itch lotion during day hours and antifungal cream daily at night...try to keep these areas as dry as possible as friction and rubbing may aggravate the lesions.."
},
{
"id": 3606,
"tgt": "Could no bleeding after spotting during periods indicate pregnancy?",
"src": "Patient: Hi I have irregular cycles 21-25 days . I have been ttc for 6 months. I think I ovulated on CD12 (had follicle tracking) So my period is due today or tomorrow. Really feels like it is due , have cramps and pressure in my abdomen. No bleeding yet but after a bm this morning did have a creamy pink discharge . My question is really how long after spotting / CM like that would should bleeding start? Also if there is no further bleeding, could I be pregnant even though I feel like I'm having the ususal pre-menstrual cramps? Doctor: Hello dear,I understand your concern.In my opinion the period might start spontaneously or within 1-2 days after spotting but it might be variable.As the ovulation had occurred on 12 th day the period might come on 26th day in case if there is no pregnancy.The presence or absence of pregnancy cannot be definitely predicted just basing on symptoms like cramps,spotting etc.I suggest you to do a urine pregnancy test a week after missed period for confirmation.Usually a urine pregnancy test done after a week of missed period gives accurate results.So relax and avoid stress.Nothing to worry.Hope this helps.Best regards.....Dr.Srilatha"
},
{
"id": 20211,
"tgt": "Can heart spasms be treated as disability for social security?",
"src": "Patient: I have coronary spasms on the bridge of my heart which occurs at anytimme day or night and at rest. I take cardizem 240 1 daily, imdur 30 mg twice daily, plavix 75mg for blood thinner,nitro pills and the nitro spray along with diovan for hbp. I had a heart cath last yr and my cardiologist saw my heart go into spasms and flushed it with nitro. I am in and out of the hospital cause of the spasms which the call variant angina. Could this be a disablilty for social security? Doctor: The disability question can only be answered by your treating doctor and the social security administration. Be sure that you have discussed additional sources of discomfort that may be confusing. It is more important to get control of the problem and be sure that you are treating the right problem. I have encountered people with variant angina who also had abnormalities of the esophagus making the control of symptoms very challenging."
},
{
"id": 61966,
"tgt": "What causes a painless lump on the upper back of spine?",
"src": "Patient: bump on my upper back right side of spine, about 2x2 cm, no pain, stiff growth doesnt move, when squeezed by my wife, this almost dry white pus came out very slowly, like in a white head, wife had to sqeeze and pinch very hard, but still no pain even though i have very sensitive skin and low pain tolarence. what could this be Doctor: Hi,Dear,Welcome with your query to HCM.Studied your query in full depth of its details.Reviewed it in context of your health concerns.Based On the facts, You mostly seem to suffer from-Sebaceous Cysts on upper back of spine,which was popped by you.Dry white push is actually the cheesy material which comes out of the Sebaceous Cysts.I would suggest Excision of this Sebaceous Cyst by a Surgeon.Hope that ,This reply would help you to plan further treatment soon with your treating doctors.Best of Luck and early recovery.Welcome any further query in this regard,which would be replied in next session.Good Day!!Dr.Savaskar M.N.Senior Surgical Specialist"
},
{
"id": 154214,
"tgt": "Is mobizox effective for back pain with bellini duct carcinoma?",
"src": "Patient: My father is taking morphine for pain. He has been diagnosed with bellini duct carcinoma. He just had chemo and the pain now does nto subside even with 30 mg morphine dosage 3 times a day. Our doctor suggest mobizox because he is getting pain twinges in lower back when he moves. Can anyone corroborate this? Doctor: Hi,Thanks for writing in.Bellini duct carcinoma is a cancer involving the collecting duct in kidney. He is getting sufficient dose of morphine but sometimes patients develop what is called tolerance and then the pain relief does not happen with the usual dose of morphine.Mobizoz contains a combination of pain relievers and muscle relaxants for relief from boy ache after chemotherapy. It contains chlorzoxazone 500 mg, diclofenac sodium 50 mg, paracetamol 500 mg which are helpful to reduce pain. Please give him this medicine after food to avoid gastritis. Pain and twitching can be due to the chemotherapy and usually improves over 3 to 4 weeks. Please give pain medicines only when necessary. Please do not worry."
},
{
"id": 37346,
"tgt": "What is the treatment for cough and cold?",
"src": "Patient: Hi. Im 17 years old and female, i smoke regulary. I caught a cold a while back now and was coughing up mucus also. After the cold eventually dissapeard the mucus did not. Every morning i cough up mucus when i have the urge to cough and this has being occuring for about a month and half, also when im sat down and i stand up and walk around i feel dizzy and feeling like im going to faint also get tight chests sometimes. Looking forward to hear back from you, thankyou. Doctor: HelloYou are having cough with mucus ( phlegm or expectoration ) for 1 &1/2 months . Feeling faintness when stand up , dizziness and tight chest .All these symptoms may be due to these reasons , these include:1 Allergic bronchitis , as you are smokers and this causes bronchoconstriction and this produces cough and added infection produces mucus . So due to smoking or any other allergen you developed allergic bronchitis .2 Bronchiectasis , may be the 2nd reason of such symptoms . If the amount of mucus produced is more than 10 ml it may be due to bronchiectasis .3 Tuberculosis , diagnosis can be confirmed by X ray chest and sputum for a f b and blood for CBC & ESR .As you mentioned that you are also having dizziness and faintness , so these symptoms may be due to anemia , so get in hemoglobin examination .Since your are having these symptoms for long time , this is the best way to consult chest physician and get his opinion.Hope this will help you."
},
{
"id": 212621,
"tgt": "Feeling sleepy when reading, difficulty remembering. Need help",
"src": "Patient: Good day, am so glad to be here, av got a problem with my system in the sense that whenever am reading or having serious and important studies i begin to feel very sleepy and that usually cause a lot because i end up not also remembering all what av been thought. Please i really need your help on this, thank you so much. Doctor: Hi there ~ You feel sleepy when you start reading because it is something that you are not interested in. To get your interest levels up you might need to start doing other things to help with concentration. Yoga, pranayama, dhyana, and regular physical activity are known to boost concentration. I would also recommend that you get evaluated by a psychiatrist to treat your condition if it needs medication. I hope this helps. Take care and have a nice day !"
},
{
"id": 89655,
"tgt": "Can stomach pain lead to pain while urinating?",
"src": "Patient: The other day I experienced severe lower abdominal pain after peeing and passing a BM it started hurting immediately and the pain grew more and more severe as the hours went on there was a constant pain in my stomach that felt sharp and lasted as long as an aching pain while there were sharp shooting pains to my anus and vagina .... It was crippling and I even fainted once .this has happened before and I didn't get my period for 3to4 months after that . And sometimes it hurts when I pee on a normal basis . What could this be. Doctor: hithanks for askingi would like to ask about any contraceptive pills usage and sex activity your age........the history looks like a urinary tract infection.........but some relevant investigation needs to be done like urine RE,ultrasound abdomen and pelvis,full blood count.....meanwhile you can take painkillers and consult your doctor"
},
{
"id": 11716,
"tgt": "Dark pigmentation on skin, apota on forehead. Any ideas?",
"src": "Patient: I have dark pigment areas on my cheeks and side of face and bridge of nose and I have been to severl derms and no results.. some hydroquionone treatments and it made it inflamed and worse..I now have some light apota on my forehead... I am told I do not have vitiglio... I don't think you can help... frustrated.. I sit home in the dark avoiding the sun.. I have given up... everyone sells me a product a script or clueless advice I have to beg for someone to care enough to actually help me. After my insurance payment or out of pocket I walk away the same way I came in. My forehead itches and the left side over my brow is thickened. One doctor asked me what is going on in that area actually he is the head of dermatology in Houston... I have no faith in the profession..why should someone care at the end of the day it is my problem. Doctor: Hi,Thanks for writing to us.The site of your dark pigmented areas suggest that it is probably due to melasma.To treat the sameUse a broad spectrum sunscreen with SPF 50Take oral vitamin c supplements 500mg twice in a dayYou need to apply pigmentation reducing creams to reduce the dark patches. Hydroquinone might have inflammed the area due to excess application of the same. Reintroduce hydroquinone 2 percent cream over your marks in night. If you do not want to apply the same, you may use Glycolic acid/ Kojic acid/ Azelaic acid based creams. Also few sessions of chemical peels( Glycolic acid/ trichloroacetic acid) should help.As far as light spots over forehead are concerned, it is difficult to comment on the same without having a look at those lesions. If possible send us an image of concerned area, so that proper advise can be given.RegardsDr kalpana"
},
{
"id": 147552,
"tgt": "What do the findings in MRI scan of spine indicate?",
"src": "Patient: I have just had results of an MRI scan of my spine. I have had difficulty walking and pyramidal signs.the scans shows a significant osteophytic disc bar at level C5/6 with additional cord signal change.and the cord looks inflamed. what does all this mean Doctor: Hi,Thank you for posting your query.I have noted your symptoms and the findings of the MRI spine.As per the report, there seems to be pressure on the spinal cord due to disc prolapse in the neck region.Please discuss the report with the neurosurgeon.You may require surgery to relieve the compression from the spinal cord (decompression surgery).You should also use a soft cervical collar.I hope my reply has helped you.I would be pleased to answer, if you have any follow up queries or if you require any further information.\u00a0\u00a0\u00a0\u00a0\u00a0Best wishes,Dr Sudhir Kumar MD (Internal Medicine), DM (Neurology)Senior Consultant NeurologistApollo Hospitals, Hyderabad,For DIRECT QUERY to me: http://bit.ly/Dr-Sudhir-kumar My blog: http://bestneurodoctor.blogspot.com/"
},
{
"id": 217174,
"tgt": "Suggest remedy for stomach pain",
"src": "Patient: hello doctor... my girl is going through stomach pain specially before she is taking a dump in the morning... after taking the dump her pain is ceased.. right now she is going through her menstrual periods... just worried doctor.. kindly advise on this...Naveen Doctor: Stomach pain while period is common. Do not worry but still you can give her muscle relaxant while having her periods every month to reduce the pain. And after her period also if she has regular stomach pain then I suggest to visit a good doctor physician or gynec. And do ultrasound of abdomen once to know more what's going wrong as well to start treatment as per the problem..take care."
},
{
"id": 8809,
"tgt": "Can I do waxing of my face skin instead of electrolysis ?",
"src": "Patient: Iam teen 19 year old going to college, i ahve lot of hair on my face. I was belaching all theses days. Iam still not happy, but still my face looks sick due to this . Can i do waxing of my face to get rid of these hairs. I read about electrolysis,i feel it is very costly and i cannot afford this. So can i go for waxing instead ? Doctor: Hi!Feel nice to answer your question. Yes repeated waxing may not be good for your face.Its painful & not a permanent solution. But you can have a try for laser treatment.Laser hair removal is a better option than electrolysis. Few sittings of Laser treatment can give you permanent solution & also is not a very costly procedure. Thank U."
},
{
"id": 225201,
"tgt": "Which is the best and safest method of birth control?",
"src": "Patient: good morning Doctor. doctor me and my wife staying in saudi arabia. am working here and i have one kid. now his age is around 2 years.upto now my wife using capsule for birthcontrol . now i plan to go vacation for 10 days to kerala. doctor my planning is for 3 years we dont need another baby. now am afraid to take too much capsules. doctor can u suggest some other methids. i check in internet i found some methods, copeer t , sponge and rings etc... but my wife check with her friend regarding the copper t, they told its very painful and to much irtations. please give us good advice. Doctor: Hello!Thanks for your query.There are so many safe methods but hormonal Intra Uterine Device IUD (mirena ) are 99.9 % safe and have so many advantages: - They can be used for 5 years , - reduce periods cramps - make period lighter,- the ability to became pregnant returns quicklyThe have only one disadvantages large initial cost.Depo-provera is another method very safety (99,7%) and very convenient, because it is cheaper (than Hormonal IUD) and one injection prevents pregnancy for 11-13 weeks.She can use and copper IUD because is safe around(97%) and is very effective for at least 10 years and the best advantage is cheap.Thanks for your queryDr. Vjollce"
},
{
"id": 165554,
"tgt": "What is the treatment for Epstein-Barr and Cytomegalo Virus (CMV) Simultaneously?",
"src": "Patient: I am a nurse case manager, following a child with newly dx d CMV and reactive lymphadenitis. Also with longterm elevated leads which have not dropped down adequately despite multiple short runs q2mo of chelation. Are these dxs the reasons why his body is holding on to the lead? Doctor: Hello and Welcome to \u2018Ask A Doctor\u2019 service.I have reviewed your query and here is my advice.Oral Valganciclovir for CMV infection. It looks like patient may have a coexisting chronic lead poisoning blood levels more because of slow redistribution of lead from bone to soft tissues, which may elevate blood lead concentrations for months to years. If blood levels are more than 70 ug/dL, it\u2019s an acute emergency.If blood levels are more than 45 ug/dL to 69 ug/dL take only Calcium Disodium Edetate, if rebound blood levels are more than 45 ug/dL you can repeat after 5 days. If levels are between 25 and 44, chelation therapy is not required but have to be monitoredI have answered your query. Let me know if I can assist you further.Regards,Dr. Shivtej N"
},
{
"id": 123111,
"tgt": "What causes sharp pain on lower left side of shoulder during deep breath?",
"src": "Patient: i have a Sharp pain on the lower left side of my right shoulder blade when i breathe deep or move or sit certain ways and even hurts a little bit when i walk..its been like this since sunday night and is getting worse but i have not had a cold or been sick either, but feel like im short of breathe. Doctor: Hello, The sharp pin on taking a deep breath can be due to an injury to the intercostal muscles or an infection like pleuritis. Proper rest and anti-inflammatory pain killers can help. Hope I have answered your query. Let me know if I can assist you further. Regards, Dr. Praveen Tayal, Orthopaedic Surgeon"
},
{
"id": 186071,
"tgt": "What is the treatment for the inflammation of wisdom teeth?",
"src": "Patient: hello, I visited my dentist today, my wisdom teeth are coming up and my gums are inflamed, my dentist prescribed me augmentine 500/125mg and ponstan, do you think its ok to take 1 of each 3 times a day? I have taken 2 ponstan and would be afraid if taking augmentine would be too much? thank you Doctor: I have gone through your query.Augmentin consists of amoxicillin 500mg and 125mg clavulonic acid. This is a beta lactam antibiotic given to eliminate the infection in pericornitis.Usually augmentin is administered every 12 hours.In cases when the infection is severe , it can be administered every 8 hours. Consult your physician if there are signs of diarrhoea.Ponstan contains mefenamic acid which is a non-steroidal anti inflammatorydrug.Ponstan 250mg can be safely taken t.i.dhope this helps."
},
{
"id": 174643,
"tgt": "Is it safe to give Ambrodil for an infant for cough and cold?",
"src": "Patient: Hello Doctor, My child is 2 months 22 days old. His weight is 5.8 Kgs. He is suffering from Cough and cold . His Cold is quite a lot and we are giving him Nasoclear drops. As for his cough, we have been prescribed Atrax and Ambrodil S. Is it fine for giving Ambrodil S(2ml). Or is there any home remedy we can add-on. Doctor: Hi...there are no cough and cold medications which have been licensed to be used in this age group. I suggest you use only nasoclear saline nasal drops. Sometimes cough and cold medications make kids too drowsy and hinder feeding and another issue is that they might cause increased heart rate.Regards - Dr. Sumanth"
},
{
"id": 136723,
"tgt": "Suggest treatment for severe foot pain",
"src": "Patient: Having severe pain in right foot. Was fine this morning, came to work, sit at desk job. Got up about 3 hours ago and the right side of my right foot feels like it needs to crack if that makes any sense. Limping when walking as very difficult to bear weight. Almost feels like a Charlie-horse Doctor: Hi,Thanks for your query.Pain in arches can be due to flat fleet or muscle imbalances,or excessive fatigue of feet muscles and ligaments on prolonged pressure of walking,standing on hard surfaces without adequate shock absorbing cushions in insole or on faulty foot wear, high heels etc.Treatment is contrast bath (alternate dipping in hot and cold water for 10 minutes,massage,elevation of feet on sitting, use of ankle support in day time, use of silicone in-sole in footwear,Take anti-infllammatory medicines and acetaminophen .Take short breaks from walking and standing.Do toe and ankle exercisesI do hope that you have found something helpful and I will be glad to answer any further query.Take care"
},
{
"id": 46027,
"tgt": "Suggest treatment for shrinkage of kidney on an ultrasound",
"src": "Patient: My father is 72 years old and suffering from hypothyroid and high BP. Recenlty, his bp increased abnormaly and admmited to admitted to hospital for bp. The doctor has advised whole abdomen sonography. The sonogrophy reportreveals that right kidney is normal but left kidney is reduced in size & shows ill defined outlines with hyperechoiccortex & diffuse cortical thinning. It measures about 79x34x31 mm and shows loss of corti comedullary diffe-rentiation type II. The renal parenchymal echogenicity is equal to that of the central echo complex -Grade III.Please suggest the treatment.Mithilesh sinhaJabalpur Doctor: mr sinha ..your father has renal parenchymal disease ..or chronic kidney disease ..it may be the result of long standing high bp ..what are reports of blood urea serum creatinine s uric acid ...the treatment is directed at underlying blood pressure or diabetes or associated edema of patient ..please see a medical specialist at Jabalpur...he will guide you after taking all things in consideration"
},
{
"id": 210863,
"tgt": "What is the remedy for depression?",
"src": "Patient: hello doctor i masturbate continously 4 years now my age is 17 i do it from 13 now i feel more depressed losing self confidence poor memory and many more please give me some advice i feel inferior to speek people and i did not able to see others people eye Doctor: Hello dear,First of all, remove all myths & misconceptions from your mind about masturbation. In fact, Masturbation is completely normal & doesn't have any bad effect to the body and mind. Since you are feeling that your masturbation is interfering with your normal functioning, you can try out the following measures:1. Improve your social life by spending more time with friends and relatives.2. Keep your mind active and busy at all times either through curricular or extra-curricular activities.3.Regular physical exercises, yoga and meditation. This helps in improving concentration & builds up the confidence level. So, now stop thinking and feeling guilty about this all the time.Wishing a Good Health.Take care."
},
{
"id": 219221,
"tgt": "Is heart transplant fatal during pregnancy?",
"src": "Patient: hi my sister in law is 14 and 4 months pregnant. she has a hole in the heart and is currently on a waiting list for a heart transplant. she has a pacemaker. i need to know if she will carry ok or if it could be fatal. she has had 4 heart attacks in the past 12 months. Doctor: Hello, and I hope I can help you today.If your sister-in-law truly is on a heart transplant list, it means her heart function is very compromised. Pregnancy increases strain on a women's heart, and pregnancy also makes blood clot more easily, which can put women at risk for stroke if they have abnormal heart motion. The fact that she needs a pacemaker is another sign that her heart cannot beat properly on it's own.Therefore, I hope your sister-in-law is getting care at a major medical center and can consult with a high-risk obstetrician, because pregnancy may be an extremely risky state for her. There is no danger from maternal heart disease to the baby, but the mother may have an increased risk of stroke, heart attack and even death if the pregnancy continues.So without complete review of all you sister-in-law's medical records, I cannot exactly determine what her percent risks are, but all women with severe cardiac disease while pregnant have an increased risk of death.I sincerely hope her doctors will determine the best course of action for her health, which may be to not continue the pregnancy if that is an option.I hope I was able to help answer your question and sincerely wish your sister in law the best of luck for her health and her pregnancy.Sincerely,Dr. Brown"
},
{
"id": 92259,
"tgt": "What are the reasons for having brown discharge, abdominal pain and feeling of lump on right side of lower abdomen?",
"src": "Patient: Hi I have a brown discharge and horribly bad pain in my abdomin it is painful in abdomin when I pee and I feel like I have a lump on my right side of my lower abdomin when the pain happens I touch where the pain is and it is like I cant feel my hand touching the spot on my abdomin Doctor: Hi, The pain/lump in right side of lower abdomen suggest problem with the organs lying there.Viz.- - An appedicitis, Right sided tubo ovarian mass,Right sided ectopic pregnancy,Urinary bladder infection, pelvic inflammation. To differentiate these,a range of investigations suggested by a gynecologist after physical check up & treatment accordingly. please do not massage/foment the part before you see doctor. A painkiller can be taken till then,after food. Thanks."
},
{
"id": 135533,
"tgt": "Suggest treatment for swollen and painful hand",
"src": "Patient: My right hand became swollen overnight 3 months ago .anti inflammatory pills di nothing. Steroid pills reduced the swelling but returned before finished. Cortizone shot I n carpal tunnel, Cortizone shot in wrist. 6sessions of hand therapy. Still have a lot of pain . Still swollen. M.R.I. No results yet. Do you have any idea.no one else does. I have an appointment with rheumatologist tomorrow. YYYY@YYYY Doctor: Hi,Thanks for your query.According to your description , pain and swelling over your hand seems to be inflammatory in nature. It could be an acute attack of gout.I advice you to give you rest to the part affected, take anti-inflammatory drug like motrin 1tablet with food as and when required (upto 4 tablets daily) to reduce pain and inflammation and consult your doctor for thorough examination and rule out any infective cause.I do hope that you have found something helpful and I will be glad to answer any further query.Take care"
},
{
"id": 7409,
"tgt": "Taking tetralysal for acne. Reddish painful pimples. Can I increase the dosage?",
"src": "Patient: Hi, I ve been taking Tetralysal for a bit over four weeks now for acne . I had moderate acne before, but it was worse than I ve ever had. Now it has gotten completely worse, and I have broken out everywhere on my face. Also it has gotten extremely red and it s painful. I don t want to give up on the antibiotic , as it s my only hope, but I am really upset that this has happened. I am thinking of upping my dose to twice daily, because I know some people do that. I really need this skin problem to go away. Also I wash my face twice daily and put cream that the derm prescribed as well. My skin just feels really irritated and like it s getting worse daily. Any advice? Doctor: hi..Thanks for your query Acne can present in several forms. This includes whiteheads, blackheads, and red, inflammed patches of skin (such as cysts).In the course of evolution, you may have any or all of these.Sometimes ,over a period of time, even if you are under a specific for of therapy,one agent might not be enough to handle it completely.This is based on the various internal hormonal stimuli as well, which can change several times in. Do not use heavy/thick creams ..Use only gel based moisturizers/ sunscreen for regular use.U can use a cleanser containing Salicylic acid/ Glycolic acid on a regular basis. You can start taking Salicylic acid peels from a qualified dermatologist to achieve a better control of your acne.These are safe and latest methods whereby you can achieve a better control, but not be perturbed by the side -effects of long term antibiotic therapies. You should change the dodage only after F/U with your doctor.Also use the creams initially on alternate days , as you tolerate it ,you can ,make it daily.Since acne creams make the skin dry , they feel irritating to some on starting the application. Also acne is a very recurrent condition,as you see...It is essential to maintain yourself on a control by doing frequent followups with your doctor and changing meds as and when required.U definitely do not need to stop anything now, however please visit the doctor early so that there are no delays in the onset of more efficacious therapies. Hope this helps.Take Care!"
},
{
"id": 38537,
"tgt": "What is the treatment for cat bite?",
"src": "Patient: Hi, I am female, from the Philippines. Today I got bitten around the palm area by our pet cat. We were trying to treat her wounds when she attacked me. The bite punctured like a needle. It s a tiny bite, but it was bleeding. I pressed down firmly on the bitten area using my fingers, and I washed it with soap and water. My plan is to wait and observe the cat for 10-14 days for signs of rabies infection. Is it the right thing to do? Thank You. Doctor: No, you will need to get shots to prevent and treat infections. And also local dressings. Rabies prophylaxis will be a must irrespective of bite when bleeding is visible, but may vary as per local country guideline."
},
{
"id": 160728,
"tgt": "What causes infant to always lean towards the right side?",
"src": "Patient: Hi there, my 11 week old baby keeps going to the right side all the time for exsample when i put him in his baby bouncer his head drops to the right side he wont sit up in there straight also when he has a poo this is always on the right side off his nappy his never in the middle is this normal? Thanks Doctor: Hello, It is quite normal and does not indicates any disease process. If the growth and development is normal, nothing much to worry. Hope I have answered your query. Let me know if I can assist you further. Take care Regards, Dr. Shinas Hussain"
},
{
"id": 83856,
"tgt": "Is there any side effects for morr f solution and curlzvit tablets?",
"src": "Patient: hi i m 25 years old male and i m having problem with my hair as my hair suffer a substantial loss in the frontal and anterior over the last year when i consulted with a doctor she prescribe me with morr F solution and curlzvit tablet plz let me know the effectiveness of these medicine and their side effects. thanks Doctor: Hello, It may used for hairloss. It will work by increasing the blood supply to the scalp or affected area. Possible side effects are: 1.Dermatitis. 2.Excessive hair growth on face. 3.Headache. 4.Itching. Curlzvit is an multivitamin tablet may be useful to avoid the hair fall. So I think I answered your questions still have queries please feel free to ask. Hope I have answered your query. Let me know if I can assist you further. Take care Regards, Dr. Penchila Prasad Kandikattu"
},
{
"id": 152134,
"tgt": "I had an MRI of the orbitals done. how it affect white matter of the brain etc",
"src": "Patient: I am 17 girl living in NYC. I had an MRI of the orbitals done a few days ago because i was having eye pains. The doctor called and said the test came back clear, but he wanted to talk to my mom. I m afraid he is going to tell my parents I drink alcohol. I am in no way an alcoholic or anything, but I do sometimes go to parties, and drink to the point of getting drunk. I have researched that this sort of drinking and the effects it has on the brain shows up on MRI scans (how it affect white matter of the brain etc.). Will the doctor tell my parents or does doctor patient confidentiality rule this out ? Please help! Doctor: hi read your prob, as you r only 17 so your doc want to discuss every thing about your mom. dont be afraid, tell everything true to your mom, she can understand you more then others . this is nothing is nothing but phase of life , today you thing as you r daughter, leter you think different when become mom. so do not be worry take your mom to doc . Dr aseem 9982583020 aseemadhuri@gmail.com"
},
{
"id": 189204,
"tgt": "Got wisdom teeth removed. Have numbness in chin and lip. Taking nervijen-p tablet. Sufficient or any treatment?",
"src": "Patient: I got my lower left wisdom tooth removed last friday and since then my left chin and lip has been numb and I am worried now.. I went back to my dentist he says it is normal as the wisdom tooth was on the nerve.. He suggested me to take these tablets - NERVIJEN - P - which I am taking currently - is this sufficient ? or should I do something else to get out of this? Doctor: hello,thank you for your question.yes whatever your dentist said is right only.it may be a reversible nerve injury which was caused while extracting the tooth and nerve gets time to repair so you have to wait for some time and continue to take vit-b12 tablet which helps in nerve regeneration.wish you goob health ahed,,,"
},
{
"id": 217959,
"tgt": "How to treat pain in an old person arising out of recovering herpes?",
"src": "Patient: My mom got diagonised with Herpies a month back. Now it is subdised almost 75% however, she is under severe pain and not able to tolerate the pain.....what should i do.....she is 70 years old...and pain killers are not much helpful.....thanks in advance for your advice Doctor: Hello, Welcome Thanks for consulting HCM, I have gone through your query, as you have herpes infection dont worry as you have said it is painful , yes it is painful, for this you should consult Physician and go for Antiviral drug course . Drink plenty of water with liquid supplement Electrol . Take one tablet multivitamin once daily with antiviral drug . Hope this will help you."
},
{
"id": 64787,
"tgt": "What is the movable lump in my jawline?",
"src": "Patient: I have a lump under my jawline, about the size of a golf ball, but it occasionally shrinks in size. It doesn't hurt, and shifts slightly when I touch it. I went to the doctor, who told me that because it's \"mobile\" it's probably a blocked gland, but definitely not cancer. He also recommended an MRI, which I got. Upon examining the films, he now wants me to get bloodwork to rule out lymphoma, which he says is rare. Still, I'm very nervous about it all. Doctor: HI,Dear, Thanks for the query from you for your painless lump under the jaw line with slight mobility ,which occassionally shrinks and is non-cancerous on MRI study.1-I feel concerned that you are serious to know what is this jawline lump?,2-In my opinion this non-tender lump underneath the jaw line appears to be lymph node mass or a sebaceous cyst ,as it shrinks sometimes.2-a-If you have history of associated malaise or low grade fever in eveining ,then this would confirm for the TB lymphadenitis.2-b-One has to rule out the lymphoma-NHL and or Infectious mono-lymphadenitis.3-I would advise FNAC to fix the reason ans hence would refer the case to ER Surgeon.and would advise CBC,ESR,and chest -Xray would fix the cause of the slightly mobile. 4-Other possibilty would be adenoma of the sub-mandibular gland- as the tumour is mobile and not fixed,which rules out the cancerous nature of this jawline lump.5-Hope this would solve your worry of the mobile lump underneath the jawline.6-Wellcome to HCM for any more query."
},
{
"id": 69594,
"tgt": "What could cause red swollen lumps in the tonsils?",
"src": "Patient: i have a almost 4 yr old daughter, she is complaining that her throat hurts... i have checked her throat for redness and swollen tonsils... her throat appears slightly red, but her tonsils seem normal.... today while checking her throat i noticed a lump in the opening of her throat, behind her tounge.. i am wondering if this is normal or should i have her doctor check her throat out Doctor: Hi.Thanks for your query and elucidate history.This is not normal at all to have a lump behind the tongue.You have to get to see the child by an ENT surgeon, Has to undergo ultrasonography of the neck first to see whether the thyroid gland is in lower neck in its normal place or as at a place you described -- this is called lingual thyroid and is good that you noticed this at such an early stage. MRI of the whole area of neck and throat may be needed to confirm the diagnosis."
},
{
"id": 43341,
"tgt": "Unexplained infertility problem, 4 failed IUI cycles, poor egg quality. Suggestions ?",
"src": "Patient: I am facing unexplained infertility problem since 6yrs .In recent times I undergone IUI for 4 cycles but all were unsuccessful. I don't have any problems except hypothyroid which is in control. Dr is saying that I am having poor egg quality. I am of age 33 and weight 60. My husband is very healthy with no bad habits.Please suggest me doctor? Doctor: Hi, poor egg quality is one of the causes, how about the semen quality, whether it is tested or not. Besides you also must be an elegible person, means haemoglobin, anxiety status, etc. I advise you to take more of green leafy vegetables, pulses, sprouts, and protein rich foods, besides other medication. Thank you."
},
{
"id": 223296,
"tgt": "What does irregular bleeding and smell while on birth control indicate?",
"src": "Patient: For almost a month now I've been having on and off bleeding I believe it is from my birth control depo. But I've been having a smell that smells really bad almost like something dead but its coming from the discharge and blood. Could you tell me what's wrong Doctor: hello user.birth control pill has to be taken regularly and on time.Smell can be due to infection.you can have a local check up from your gynecologist.and take antibiotics accordingly.thanks."
},
{
"id": 190270,
"tgt": "Pieces of skin hanging, whiteness on the bottom post wisdom tooth extraction. Remedy?",
"src": "Patient: I got my wisdom teeth out Monday (3 of them) and on the bottom right it s just very messy looking. Some pieces of skin are just hanging there and it just looks weird. Some of the skin is white. I m more concerned about the skin that is hanging. It kind of looks like its been all cut up (which obviously it has) but I figured it should all be stitched. Doctor: Hello, Thanks for posting your query. How much time it took to get it removed and does dentist need to cut the bone along with it.?? As such after extraction there are nil chances of flattening of face because that area is covered by the coronoid process of mandible and lot of muscle and fat tissues. Only chance it if he has to remove the chunk of bone along with the tooth. You can go for extraction of other side as every case is different. explain your dentist about your previous experience. he will take care of you. Regards."
},
{
"id": 122747,
"tgt": "What causes pain in collarbone?",
"src": "Patient: What does pain at my collarbone mean? It s very tender to the touch - almost like a bruise. It hurts more when my arm is raised above my head and I lean my head back. There is also tingling ( in my right hand. I am 45 year old female in excellent health. Thanks. Doctor: Hi, This mostly is a spasm in your sternocleidomastoid muscle or your scalene muscle, sometimes called thoracic outlet syndrome, try doing stretching for your anterior chest wall muscles, and stretch your scm and scalene by doing same side rotation plus opposite side, side bending as if you are looking up for 30 seconds and repeat 5 times...combine this with breathing exercise. Hope I have answered your query. Let me know if I can assist you further. Regards, Ayman Darrag, Physical Therapist or Physiotherapist"
},
{
"id": 69408,
"tgt": "What causes pain in upper breast near armpit area?",
"src": "Patient: Hi, I have pain in my upper right side of my breast but its near the lower armpit area. When I touch it it hurts, its tender and I think it may be causing pain in my shoulder as well. Also it becomes painfull without any pressure applied(like a sharp pain). I can fell a large like lump. I have had issues with lymph node swelling in the past (dr thinks its lupus) but this feel differnt. I am nervous as my mother got cancer when she was young, Im 25. Any thoughts? Doctor: Hi.Thanks for your query and an elucidate history.First of all remember a simple dictum. Cancers are always painless to start with. I can understand your feeling as your mother had cancer. In your case this looks to be fibroadenosis only. This is a benign disease and never known to get transformed into cancers. To prove Let you undergo: MammographyUltrasonographyGuided Fine needle aspiration cytology.Of course a clinical examination by a Surgeon .All this will lead to a very proper diagnosis and give guideline for treatment."
},
{
"id": 116686,
"tgt": "How long will ventilator use be required while administering white blood platelets?",
"src": "Patient: my daughter is given white blood platelets but she is having breathing broblemso she is kept on ventilatormeans oxigen how long she will have to beath like that and her weakness is very much in blood transfusion. mydaughter's age is 33height5feet 2 inches and having thoriroid and was ill due to Dengue fever. Doctor: Welcome to H.C.M.I am Dr Krishna Dubey.In dengue fever ,patient recovery depends on his/ her body immune mechanism to fight with virus. Ventilator is for support to maintain oxygen saturation.when she itself start maintaing her oxygen saturation then your doctor will remove ventilator.Removal of ventilator completely depend on patient status, not on time period.When her platelets count itself start increasing at least in 2-3 consecutive repeat of platelets count test then it is very good news for you.Hope for her well being.All the very best.Thanx for query."
},
{
"id": 175466,
"tgt": "What is the treatment for cough and cold?",
"src": "Patient: Hi , my daughter is 20 months old and is having cough and cold from last one month. Presently doctor has told that she has mild asthmatic bronchitis and has told to give her telecast 4mg. How serious is this? Will she be fine and healthy once she recovers? Doctor: Hi DearWelcome to the HCM.Not to be panicked, asthmatic bronchitis is inflammation of the lung. quite a common problem among young ones. To be tackled skilfully with antihistaminic decongestant and bronchodilator drugs. Antibiotics if really required.Nebulization helps a lot in reliving the symptoms.Montileukast to be given far along time to suppress the allergic eliment in the body"
},
{
"id": 148263,
"tgt": "Why is the arachnoid cyst growing again after spinal chord surgery?",
"src": "Patient: Hello Doctor, I had undergone spinal cord surgery due to arachnoid cyst (4CM) on last sep 2012. Recently, I went for MRI check-up after surgery review. Now, the cyst(1CM) is growing again. Can you please clarify the below queries? 1. whether CSF fluid is not drained properly from arachnoid layer during the surgery? 2. Why it is regrowing again after surgery? 3. Whether I need to undergo another surgery? Thanks in Advance Regards, Doctor: Hello and welcome to HCM,Arachnoid cyst arises from the arachnoid layer of the meninges.It does not occur due to improper drainage of CSF.So, re-appearance of arachnoid cyst does not signify any alteration in CSF drainage.Arachnoid cyst is mom-neoplastic growth and there is every possibility of re=appearance and regrowth.Repeat surgery will be required when the lesion becomes symptomatic.This can occur when the arachnoid cyst becomes large enough to compromise CSF production or drainage, any pressure symptom, etc.You need to stay in touch with your neurologist/ neurosurgeon.Thanks and take careDr Shailja P Wahal"
},
{
"id": 173732,
"tgt": "What causes black spots on the testicles of a child?",
"src": "Patient: Hello. My 9 year old just told me he has black spots on his testicles. I had a look and the seem like splinters. There is a dark spot pushing out of the surface and I can see it's prolongment under the skin, 2 mm about. He had his testicle lowered when a baby and his foreskin cut to detract foreskin. He was also born with 3 kidneys. One removed as it was not working (8 months old). He is lively at times, frequently tired and seems to crave for foods containing vit b and iron. I will have a complete checkup done, but what are the black spots? I am terrified it cld be something serious. Thank you. Doctor: Hi,Thank you for asking question on health care magic.Usually the srotal skin is pigmented and normally looks black.Dark spots and tiny projections ate common.Nothing to worry.Hope this answer will serve your purposePlease feel free to ask any more queries if requiredTake careDr.M.V.Subrahmanyam MD;DCHAssociate professor of pediatrics"
},
{
"id": 50091,
"tgt": "Urinating in bed while sleeping. Started after removing kidney in surgery. Have mild dementia. Advice?",
"src": "Patient: My husband had a kidney removed last month due to cancer. Since the surgery, he has been peeing while sleeping without waking (soaking through 2 adult diapers, sometimes 2 or 3 times a night). He does fine during the day, but every night since surgery, he pees in his sleep without waking. He is 78 yrs. old and has mild dementia, but did not have this problem before the surgery. Hope you can help with an answer. He is on two new meds since surgery but the side effects do not mention urinary problems.Thank you,S. McAdams Doctor: Is be on any sedatives by any chance ?Deep sleep due to these drugs might be causing it.Does he have increased frequency of urine or burning during day time suggesting urinary infection.Whar are the renal functions now?Also take care that he doesn't drink excess water or liquids during evening and night time."
},
{
"id": 116200,
"tgt": "Could elevated liver enzymes and platelets(120)cause itchiness?",
"src": "Patient: I'm itching like something serious....both liver enzymes high and platlets 120 now what and could this be causing my itching? I am a 29yr old female with no hx of lots of over counter drug use and not more than average on drinking (in my opinion) and very very very tired. Doctor: Hello and welcome to HCM,Elevated levels of bile salts can cause itching all over the body, however, elevated levels of liver enzymes are not likely to cause itching.Elevated number of platelets can also cause itching however, the platelet count is not very large so as to cause itching.You need to consult your doctor for clinical assessment to find the cause of elevated liver enzymes and cause of itching.Thanks and take careDr Shailja P Wahal"
},
{
"id": 144850,
"tgt": "What causes dizziness and lightheadedness?",
"src": "Patient: At age of 12, I have been electrocuted while replacing a lamp for 5-10 mins. My family asked the neighbours about what should do about this matter and they said I will be ok through time. 2 years have passed and I was fine. But one day while sleeping I felt there is electricity moving in my body rapidly and I woke up in hospital after 15 mins. By that day, I experienced dizziness, light-headedness everyday for 3 years and I went to many doctors during the 3 years and made many tests but I was ok and they told me dizziness will go away throw time. However, until now I experiencing dizziness and light-headedness. Doctor: Thank you for asking Healthcare majic. My name is Dr Ehsan Ullah & I have gone through your query.As your age is quite young,there is nothing to much worry about if you didnt experienced skin burn due to electric shock and if there was no loss of consciousness along with vertigo and vomitting at time of incident.Get visit to medical consultant and do an ECG and complete blood and urine count.If these reports are normal then this might be due to psychological effects for which seeking advice from psychiatrists will be good option..Till then take good diet and drink plenty of fluds to be energetic. Hope this may help you. Let me know if anything not clear. Thanks."
},
{
"id": 80053,
"tgt": "After having a stent on the left wall experiencing pressure in lungs,heart & stomach",
"src": "Patient: had stent put in the left wall. was good for 3 months, was doing some moving , caught the flu, now , stomach feels full of gas pressing on lungs and heart, dr increased water pill and this made it worse, this morning i had plain yogurt , some pressure went away, i know meds cause these symptoms, try to burp , can t. I m wearing the zoll life vest but it don t go off ,so i assume the heart is beating right, maybe i just have to rest and exercise slowly. Doctor: Thanks for your question on Health Care Magic. I can understand your concern. By your history and description, possibility of GERD (gastroesophageal reflux disease) is more. It is due to laxity of gastroesophageal sphincter. Because of this the acid of the stomach tends to come up in the esophagus and cause burning chest pain, fullness in stomach and burping. So better to avoid stress and tension. Avoid hot and spicy food. Avoid large meals, instead take frequent small meals. Avoid stress and tension. You may need proton pump inhibitors and prokinetic drugs. Since all these are prescribed medicines, you need to consult your doctor. So consult your doctor and discuss all these. Hope I have solved your query. Wish you good health. Thanks."
},
{
"id": 21973,
"tgt": "How to increase ejection fraction of heart?",
"src": "Patient: i am 51 years old working women and had a heart attack 7 yrs ago and having 2 stents in my heart. After that with lifestyle changes and medication I was living a good lije but from past sometime i am feeling fatigue and recently having bad cold an during the chest x-ray done observed some Lung infection and heart enlargement and after that gone through a test where my heart pumping came to 26%. I am adviced for some rest and medication. Will my heart pumping will increase and i will be able to live normal avtive life. My weight is 83 Kgs and my working timings are from morning 8 to 6 p.m. advice. Regards, madhu Doctor: I would like to know what was your heart function in your previous report.....heart function never becomes normal so you have to adjust your lifestyle as well as decrease your work stress"
},
{
"id": 10193,
"tgt": "Suggest remedy for hair damage",
"src": "Patient: Hi doctor ths is abrar for india?i had an hair problem at my back side of head?ths problem happen at the age of 18?when was goin to haircutter for cutting my hairs?but i advice him to do the hairthinker?hairthinker is also a type of hair cutting but the differance is that it remove the curlyhairs?After 1mount i again went for haircutting?he told me that ur back said of hairs r falling and damgaed?and when i see that part their was two smalleholes?so after that i went for herbal treatment?its going day my day bad?DOCTOR PLEZ PLEZ PLEZ HEPL?MY FURTRE IS Going Bad? Doctor: Hello and Welcome to \u2018Ask A Doctor\u2019 service. I have reviewed your query and here is my advice.I feel that you are having alopecia areata. There are various causes for alopecia. Treatment options differ depending on the cause. Please do visit your dermatologist and get his advise. In the meanwhile you could take biotin supplements and can apply minoxidil solution on areas of hair loss.Hope I have answered your query. Let me know if I can assist you further."
},
{
"id": 93963,
"tgt": "Have pain between hip and belly button, nausea. Have pain due to ovarian cysts also. Appendicitis burst?",
"src": "Patient: What does it feel like when or after your your appendicitis bursts?I am having sharp pain in my right lower side in between my hip and bellybutton. My bellybutton never hurt but I am feeling nauseous. I get ovarian sists and might be mistaking this for a sist, but the pain is different. I dont believe I have a fever but i get lightheaded like my face is getting real hot when i move to fast, only for a few seconds. I have been to the bathroom. It doesnt hurt while I am going but hurts after. Once it bursts will I feel relief or will the pain become worse? I do not no if you can get use to the pain if you get sists and are use to that pain, but I dont want to mess with this if I need to go see a doctor. Doctor: Hi, it seems that you already have ovarian cyst right side. You have already experienced pain due to that. Now you are having a different pain which is located between the hip bone and belly button, nausea , pain after defecation and fever like hotness of face. It is typical location of appendicular pain. Possibility of appendicitis is more in favor. your expression of pain and other symptoms does not favor perforation. Get USG abdomen to rule out appendicitis & stone in the KUB area. The differential diagnosis for your complaints is ureteric colic either due to stone got blocked or recent passage of stone. Consult appropriate dr. as per the result of USG. and get treatment accordingly Best wishes"
},
{
"id": 152202,
"tgt": "Is my mother suffering from schizophrenia ?",
"src": "Patient: hi.. my mother is suffering from neurology problem from past 10 yrs..// she is getting angry so soon,, she is speaking alone ,scream alone... she is behaving like anything...//plz can u suggest me what kind of treatment is usefull Doctor: may be she is suffering from schizopherenia,consult with a psychitrist."
},
{
"id": 15297,
"tgt": "Itchy, red rash around eyes. Taking loratadine. Waiting for blood results. Suggestions?",
"src": "Patient: In July this this year I came up in a itchy red rash aroude my eyes. On my eye lids and the eye sockets and above my lips. I saw my G.P who prescribed Daktacort .This helped reduce the itchiness and the redness ,but does not last. I was advised to take an anti hystimine when I saw my GP last week and he queried if I had become sensitised to my makeup. I am taking Loratadine 10mg daily which has helped the symptoms ,but now the redness and itchiness is returning under my eye and a small red itcy patch at the side of my face .I am not using amy make up on my face to see if this makes a difference.I am 51yrs of age fit and healthy. weight 7.5stones and never ill. I do not have any health problems. My G.P has taken bloods to check for anaemia and I am awaiting the results. I have never has skin problems or allergies before. I do not know what else I can do except for ? try an antibiotic to see if this makes a difference. Doctor: you are allergic allergy can develop at any time with any thing at any age even a food component at once start reacting to body proteinsallergens can be oils shampoo soaps creams ointments body application materials medicines like pcm ibuprufen aspirin disprin calcium multivitamins and diet supplimentsfoods pollensget consultation with allergy specialist who can get you diagnosed by getting allergy tests and find the cause and treat according to result"
},
{
"id": 71837,
"tgt": "Sharp pain in arm while bending neck with sore chest and numbness in right hand",
"src": "Patient: I have a sharp pain that shoots down to the. Middle of my right arm when I bend my neck back. My arm hurts certain ways I move it I have a sore mild feeling in my upper right chest area and for the last to days my thumb and pointer finger. On my right hand has been num. Doctor: Hello As you explain the history it seems like a neurologic problem.Discuss with your doctor. Regards Dr.Jolanda"
},
{
"id": 10779,
"tgt": "Can Mononucleosis cause Hair loss?",
"src": "Patient: My 18 yr old daughter is having hairloss. Lots of hair in the shower and on bathroom floor after she brushes. She had a suspected case of mononucleosis in May. It was confirmed however it was some type if virus which presented with very high fever for 7 days, vomiting, weight loss due to not eating and severe fatigue. Could the hairloss be related to the virus from 2 months ago? Doctor: Hello. Yes, the hairloss is not caused by the mononucleosis itself, but by the prolonged and sharp febrile process your daughter went through. 3 weeks-3 months after the febrile process there can be diffuse hairloss. In a few months the hair will reappear."
},
{
"id": 51593,
"tgt": "Can I have renal transplant when I am on medication for HCV infection ?",
"src": "Patient: i am a diabitic type two patient. I am in dyalisys patient from one year and planned to transplant bo the kidneys. at the time of bood testing i found hcv rna positive GNA TYPE 1, even thouhgh i had under gone syngle use dialiazer through the period of dialisys. I had bood trnsfusion once during the month of march 2009. I under took treatment for hcvrna . I took PEgasysy interferon 180mcg treatment . After 8 injections my level of hcv rna become negative. i took further two inejction already. Now gastronics dr. says , i have to take further 12 injections to completely eradicate the hcv rna virus frommy blood . But the nephrologysts is assuring us to go head for kidney transplantation. HOW TO GO HEAD SIR. I AM AGED ABOUR 46 YEARS. kindly suggest the case and oblige thanking you shrinivas Doctor: Hi Shrenivas > i am a diabitic type two patient. I am in dyalisys patient from one year and planned to transplant bo the kidneys. at the time of bood testing i found hcv rna positive ----------------------------------- Dialysis has become a predominant mode of hcv transmission in world inspite of best dialysis practices. -------------------------------------------------- > GNA TYPE 1: This is difficult to treat genotype and is associated with poorer outcome. -------------------------------------------------- >I under took treatment for hcvrna . I took PEgasysy interferon 180mcg treatment . After 8 injections my level of hcv rna become negative. i took further two inejction already. Now gastronics dr. says , i have to take further 12 injections to completely eradicate the hcv rna virus frommy blood. Atleast 48 weeks of treatment is recommended for genotype 1. Shorter duration is associated with lower response rate and higher relapse (recurrence). ------------------------------------------ > But the nephrologysts is assuring us to go head for kidney transplantation. If you are maintaining a good health on dialysis, completion of 48 weeks of treatment will be advisable. But if you are rapidly deteriorating, and not able to tolerate dialysis, your doctor may suggest early kidney transplant with following caveats \u00a0\u00a0\u00a0\u00a0\u00a01. HCV may recur \u00a0\u00a0\u00a0\u00a0\u00a02. It is not possible to treat HCV posttransplant \u00a0\u00a0\u00a0\u00a0\u00a03. Your liver may suffer rapid damage in posttransplant period. \u00a0\u00a0\u00a0\u00a0\u00a04. You may develop HCV related problems like cryoglobulinemia Talk to your nephrologist DR VAIBHAV BANAIT, MD, DNB, DM Consultant gastroenterologist"
},
{
"id": 88496,
"tgt": "What is the treatment for left abdominal pain?",
"src": "Patient: good morning sir , i am shariq .... sir 6 month before i did check up and doctor said i have a stone in my kidny....and they operated it by injection .... but now again im feeling pain on my left side of stomach.. sir i dont want to go for surgery....plz plz give me some suggestion...... Doctor: Hi.Thanks for your query. History of a stone in the kidney and the procedure performed understood.First of all you should get a confirmation of the diagnosis, get the exact size of the stone to decided which route will be OK for its removal if present.How big is it?Since when it has got impacted ? Also you have to undergo the tests for :BloodUrine - routine and microscopy, culture and sensitivity. UltrasonographyC T Scan and intravenous Pyelography .The reports will decide whether you need a Surgery or other methods of its removal."
},
{
"id": 58710,
"tgt": "Pain in stomach from mid abdomen to back, worsened while lying down. CT scan showed fatty liver. Concerning?",
"src": "Patient: I have gone to my general practitioner for the last year with unidentified stomach pain. It is pain to the touch. I have had an abdominal CT and a fatty liver has been identified. Now my pain extends from my mid abdomen and to my back and my back pain is more extensive when I lie down to sleep at night. this has been the case for the last 6 months. I'm heading to a GI Specialist. Should this be a concern? Doctor: Welcome to HCM.The back pain would be due to musculoskeletal causes though spinal nerve root compression would need to be ruled out.A fatty liver would cause a moderate ache over the right subcostal area and hypochondrium.The management for the liver problem will include life style changes ,like a low fat diet with plenty of fruits and vegetables.Regular exercise like a daily 45 minute brisk walk would help.The back pain needs to be investigated and treated,use of NSAIDs like ibubrufen will help till yo see your doctor."
},
{
"id": 127393,
"tgt": "What causes sharp pain at the base of the thumbs?",
"src": "Patient: I\u2019ve been experiencing sharp stabbing pain in the base of my thumbs for the last 3-4 weeks. My thumbs both seem extremely weak, esp when gripping or picking something up. Over the last few days I\u2019ve moticed aching from my elbows down to my wrists as well. Doctor: Hello and Welcome to \u2018Ask A Doctor\u2019 service. I have reviewed your query and here is my advice.The pain in thumbs accompained by weakness might be related to the thumb joint or to the tendons and ligaments surrounding the joint problems.The thumb joint is commonly overused and this often leads to arthritis of the joint, or an inflammation of the tendons around the joint, that causes pain. The pain might also be related to carpal tunnel syndrome, that is a very common cause of sharp stabbing pain in the base of the thumbs , weakness and numbness, and occurs when your median nerve gets compressed at the wrist .In my opinion you should consult a doctor for a physical examination, to put the right diagnosis.Meanwhile you can use antinflammatory drug for pain and inflammation relief, Naproxen 500 mg daily.Hope I have answered your query. Let me know if I can assist you further."
},
{
"id": 212736,
"tgt": "Skin colored bumps on fingers, redden, liquid filled. Is it Herpes? Is it connected to OCD?",
"src": "Patient: I have small 1mm max skin coloured bumps on my fingers, sometimes in clusters not often near the nail. I mention this because my fear is that this could be herpes whitlow and I bite my nails often and the skin around them as a compulsion to reduce stress for my OCD . The bumps do not pop easily and when they do the skin sort of deflates and goes red. The liquid is clear like water. When popped they disappear after a couple of days and stop irritating within a few mins. I never thought you could get herpes on the hand or I would have fought harder not to bite my hands! I do not care about my health but rather the effect I could have on my wife or more specifically and children we decide to have as I know herpes in any form can kill babies! And that you can pass the virus on even when not having an outbreak! Not so bad if it s oral or genital but you use hands for everything! Can I hold hands? Again, engage in foreplay? Hold children? Suck food off my fingers? This is crazy? Why didn t I know about this and why is herpes called an STD when you can get it on your hand from a coldsore and give it to yourself through masterbation! I m really stressed and upset. I am also due to start training to be a mental health nurse soon. If i have herpes whitlow can I still be a nurse? Or will I infect others? This is crazy I can t belive you can get something this contagious on your hands! Doctor: Hello and welcome to Healthcare Magic. Thanks for your query. There are two major varieties of herpes - genital and non-genital, and both are contagious. It is possible to get herpes in the hands (herpetic whitlow). However, it is also possible that the lesions in your hands around the nail could be from the reccurent irritation caused due to excessive biting of your nails and the surrounding areas. Since, these you seem to be extremely worried whether these lesions could be due to herpes, I would suggest that the best way forward would be to get yourself tested for the same. So, please consult a dermatologist for a proper physical examination and further investigations. Wish you all the best. Regards, Dr. Jonas Sundarakumar Consultant Psychiatrist"
},
{
"id": 92827,
"tgt": "Hurt badly in lower abdomen, hurts to walk, stand and sit, feels dizzy. Reason ?",
"src": "Patient: Yes I was intimate earlier with my fiance and suddenly it felt like he hit something in me and it hurt sooo bad in my lower abdomen. It hurts to walk, stand straight, and even sit. I am pretty much in the fetal position. Its making me feel dizzy because it hurts so bad. It really hurts when I cough, laugh, or try to go to the bathroom. Doctor: Hi,From history it seems that due to hard hit there might be having some internal organ injury giving rise to this problem.Consult your gynaec and get examined.If you do not get appointment then go to ER.Ultra sound will help for having conclusion.Ok and take care."
},
{
"id": 129913,
"tgt": "What causes painless pitting edema of both legs?",
"src": "Patient: hi.. I am a girl at the age of 24...I am having painless pitting edema of both legs since the age of 12 years of mine ...all investigations done were normal except lymphangiography was incomplete..4 months back done microfilaria test (microscopic ) which was negative..initially at the start swelling was only on left foot den involves left leg and then right also ..but when I took DEC 300 MG and ALBENDAZOLE 800 OD for 21 days ,right from the first dose swelling decreased,,then after the course I discontinued it..should I go for antigen test for filarial to conform it..i want my both legs look normal at any cost ..I badly need it..please help Doctor: yes u should go for antigen. sensitivity test . . and microfilm ria test will be postie only if active and live . . but negative if not alive ut their might be antigenic reaction of filariasis inside to cause ur symptoms. . ."
},
{
"id": 154186,
"tgt": "Is it possible to get anal cancer due to frequent sex?",
"src": "Patient: My girlfriend and I have anal sex on a regular basis. We are both still very young at 15 years old. My concern is for her well being. I want to be sure I wont hurt her in some way. We are both 100% clean as we have never had any other partners. But can she get anal cancer? or some other disease or infection? I really enjoy doing it but I need to be sure that i'm not hurting her somehow. Thankyou Doctor: Hi, dearI have gone through your question. I can understand your concern. No risk of anal cancer due to anal sex. But you should take precautions. Because chance of infection is high in anal sex then the vaginal sex. Because muciss is tougher and chances of injuries and infection are high. Otherwise it is safe. No chance of cancer. Hope I have answered your question, if you have doubt then I will be happy to answer. Thanks for using health care magic. Wish you a very good health."
},
{
"id": 17874,
"tgt": "Can high blood pressure cause balancing problems?",
"src": "Patient: Hi, may I answer your health queries right now ? Please type your query here...i have been on altace and hydroclorizade for years recently a failure to take the meds which sometimes were needed and sometimes not resulted in a rise of blood pressure from the 130/90 to 184/115 which was felt as highblood pressure new meds were given after a hospital visit and a visit to a regular doctor the balance problem which is not visible to others after trying to stand still in the same spot for a few minutes has not has not gone away and the blood presuure towards the later part of the day has not been lowered below 150 or 160 /95 yet it is at this time that the balance issue seems to be felt and can high blood pressure cause a minor balance problem while trying to stand in one place a mri and a cat scan were done without anything showing will the problem likly go away with additional meds every 12 hours Doctor: Hi, If you are having symptoms of high blood pressure & taking medications, you are recommended to have a thorough cardiac checkup (Lipid function tests, ECG, Echocardiography, CBC, MRI, CT). High blood pressure does not cause co-ordination problems or headaches, these are side effects of some medications taken for hypertension. Co-ordination problems, loss of balance, sudden dizziness may lead to syncope or stroke. Get physically examined, evaluated & treated without much delay. Balancing problems occur in hypertensive crises (180/120), if you have it, you need medical attention. Get connected to your treating physician. Hope I have answered your query. Let me know if I can assist you further. Regards, Dr. Nupur K, General & Family Physician"
},
{
"id": 205420,
"tgt": "Suggest treatment for kleptomania",
"src": "Patient: I have a 20 year old daughter who has often, throughout her life been a habitual liar and petty thief from friends and family. She graduated from school, started a career and stole from her employer. She was terminated (obviously). Shows no signs of remorse and does not willingly help out around the house or keep up with her responsibilities. She has been seeing a psychologist. She has also been stealing from us. Ideas? Doctor: Hi dear,kleptomania is an impulse control disorder which is treated with medications and cognitive behavior therapy both.so please consult a psychiatrist along with psychologist."
},
{
"id": 63389,
"tgt": "What causes lump on tongue with indentation?",
"src": "Patient: I have a lump on the side of my tongue with indentation all around it and two sores on the back top part - the dentist shone the light in my mouth that s supposed to detect abnormal cell growth and it didn t sho anything -- are those lights effective or should I be worried? Doctor: Hi Dear, Indentations sometimes occur along the lateral borders (the sides), as the result of compression of the tongue against the adjacent teeth . When tongue becomes larger than normal, it will press onto your teeth. This tends to cause indentations. There can be many reasons for indentations like Swollen tongue , which could be due to allergies, some disease and conditions such as amyloidosis, hypothyroidism, tongue cancer, canker sores, scarlet fever and angioedema . If light test doesn't show anything then there is nothing to panic . The lumps you mention in query are probably due to canker sore due to frictional trauma . These usually go away in few days without any treatment . Do betadine rinses , speak less so that your tongue doesn't get hurt and do hydrogen peroxide rinses mix with equal amount of water twice a day . If condition persist then consult oral pathologist for proper evaluation . Hope your concern has been resolved.Dr. Harry Maheshwari"
},
{
"id": 180316,
"tgt": "Does a fall for a baby need looking into?",
"src": "Patient: My 3 week old son rolled off my chest whilst i was sitting on my bed. He fell right down the gap between the bed and bedside table.i picked him up and he cried for a couple of minutes but settled quickly. There are no visible signs of injury and his pupils seem to be reacting to light. He wanted to be fed almost immediately after and has passed wind and im almost certain a bowel motion since this happened (about 50 minutes.ago) Doctor: hi I think he is fine, but be careful in future as some injuries can be fatal to kids of this age. kindly don't sleep while feeding and don't sleep while your baby is on you as you might be tired and don't know when you may roll on to him.hope this is helpful to you, rate it if you like it. take care"
},
{
"id": 112695,
"tgt": "Back fused, swelling in hand, painful, high BP. On blood thinners, taking benadryl. What is wrong?",
"src": "Patient: Backing up 2 weeks I had my back fused. That went well but in the last 3 days I have developed swelling in my left hand pointer finger along with pain. The pain is localized in the palm when I force my hand open as far as I can I get red splotches in the palm that are painful if you push on them. The palms of both hands itch constantly . I am on blood thinners and high blood pressure meds along with pain and cholesterol meds. I have been taking benadryl for 2 days. Yesterday the same symptoms were on the bottom of my feet but are no longer having issues with them . Any thoughts would be great. Doctor: Hi. many thanks for the query!This is the picture of palmar space infection spreading from index finger infection that may be due to some minor trauma or it can be an inflammatory pathology.Get physically checked up by your doctor.If Abscess needs to be drained.Take antibiotics, anti-inflammatory drugs, anti-histaminic drugs as per your doctor's opinion.Wish you a good health.Take care.Regards."
},
{
"id": 68215,
"tgt": "What causes formation of lump on my inner upper thigh overnight?",
"src": "Patient: I had a 12 cm lump show up on my inner upper thigh almost overnight and have no idea what it may be.I know my immune system has been very much activated by all the alternative things I have been doing for my advanced prostate cancer and was thinking possibly dead cells and poisens looking for a another way to exit my body.Is this an absurd thought? Doctor: Hello!Thank you for the query.Your though is not quite correct. Cancer cells do not get from the body in such way. As the lump has appeared quite quickly some abscess or hematoma are most probable reason. In both cases there should be pain and swelling around this lump. The best way to find out what you are dealing with is an ultrasound. Abscess and hematoma should be incised and drained.Hope this will help.Regards."
},
{
"id": 67066,
"tgt": "What causes swollen,itching lump on belly button?",
"src": "Patient: I just found a swollen lump on my belly button. Im not sure how long it has been there, I noticed it now because it is itching really bad. Looking online it seems like it could be a harmless umbilical hernia, so I was going to wait untill next week to go to doctor. I am wondering if thats the only possibility you could think of or is there a chance that its something else and I need to go immediately? Doctor: HiWelcome to hcmYou have not mentioned the exact location. If it is arround the umbilicus then it can be hernia but less likely to cause itching. Hernia will disappear on laying down and becomes prominant on caughing. You should show it to your surgeon once ffor confirmation. If it is hernia it needs surgery. Skin infection can cause itching.Regards."
},
{
"id": 208685,
"tgt": "How to get rid of mood swings?",
"src": "Patient: I am suffering from severe mood swings. Sometimes I get really depressed and cry but also lash out and get really angry. I also get really hyper and I feel like I m going to explode, I used to really enjoy being like that but now I know I ll plummet down again. I regularly go out drink far too much and act very silly, especially sexually. I have always been like this but I m 21 now and it effects my daily routine. I don t know what s wrong with me but it drives me mad and it s affecting my relationships. Please help!! Doctor: DearWe understand your concernsI went through your details. I suggest you not to worry much. The symptoms you are mentioning here neither warrant any depression related disorders nor any bi-polar disorders. You could just be a victim of drinking and over negative thinking. You are just 21. you have enough other activities to do in the field of education and career. You cannot overlook your career/education/life and indulge in negative activities and thinking. You should consult a psychologist for expert opinion.If you require more of my help in this aspect, Please post a direct question to me in this website. Make sure that you include every minute details possible. I shall prescribe the needed psychotherapy techniques which should help you cure your condition further.Hope this answers your query. Available for further clarifications.Good luck."
},
{
"id": 108791,
"tgt": "Suggest treatment for lower back pain",
"src": "Patient: Ive had some variable lower back pain going on for about 3 weeks. Ive alturnated between tens unit, epsome salt/peppermit oil baths, massage, ice packs, stretching excercises. With a little relief. I finally resorted to 37.5mgs of ultram. However i can feel golf ball sized lumps on either side of my spine, they are squishy and move around a bit when rubbed. They feel good when i rub them and even better when i push on them hard and stretch. Should i be conserned about these lumps? Any advise on how to get rid of the pain permanently? Doctor: Hello, I have studied your case. Do not worry about lump in paraspinal spine muscle.Due to compression of this nerve root there can be pain in your back.I will advise you to do MRI spineFor these symptoms analgesic and neurotropic medication can be started.Till time, avoid lifting weights, Sit with support to back. You can consult physiotherapist for help.Physiotherapy like ultrasound and interferential therapy will give quick relief.I will advise to check your vit B12 and vit D3 level.Hope this answers your query. If you have additional questions or follow up queries then please do not hesitate in writing to us. I will be happy to answer your queries. Wishing you good health.Take care"
},
{
"id": 168800,
"tgt": "What causes simultaneous arms shaking and twitching?",
"src": "Patient: my 5 year old arms shake or twitch or bounce both at same time she is awake but does not seem to realise she is doing it she did this last month but stopped after a couple of hours but started doing it this before school and still doing it walking to school Doctor: hi, Welcome to this forum.Can understand your concerns.Involuntary movement of upper and lower Limbs could be due to calcium deficiency, seizure, tremor due to multi vitamin deficiency.you should do calcium levels in child, complete blood count,l electroencephalogram should be done if tremors persists. next time, this kind of involuntary movements occur then you should take the child to ER because it is very necessary to get the child examined during period of involuntary movements.Review with above reports.I hope this will help you.Take care.Regards, Dr Deepak Patel"
},
{
"id": 199245,
"tgt": "What causes swelling on frenulum?",
"src": "Patient: Hello, i have swollen frenulum even after one year prior to circumcision. I have no pain, and the scar is thick. Everything has healed. I also had lengthening of frenulum and its been swollen ever since after 2 months. It hasn't reduced, maybe its the scar wound preventing it? Doctor: HelloI share your concern, I had gone through your question diligently and I will try to help you out in best possible waySuch a swelling on frenulum can be caused by infection , Sexually transmitted infection and can be allergic as wellIf you can upload me a photograph and answer my these questions I will be able to help you betterYour age and relevant medical history if any like diabetes or liver disease?Have you ever had similar problem in pastany recent exposure to unprotected sex?since I can't physically examine which gives more clue to diagnose you more accurately So,In my opinion you should see your doctor for physical check up and investigation to rule out infective conditionMeanwhile you can apply antibacterial or antiseptic ointment like mupirocinMaintain good hygieneyou can take Ibuprofen for pain.Hope this answers your question, Please feel free to ask for more clarification, I will be happy to help youBest wishesThanks and regards"
},
{
"id": 42933,
"tgt": "Can bilateral tubular blockage that leads to infertility be removed without using surgery?",
"src": "Patient: Dear Sir,I am suffering from bilateral tubular blockage leads to infertility.I want to know whether same can be removed without surgery if so how long it will take to cure .pls reply soon.It is been 3 years after my marraige.I have a keen desire to get prgnant.Please help me... Doctor: hai,the cause and site of tubal block is the deciding thing.surgery is the final option to clear the tubal block.if it is not cleared with surgery go for IVF.CONSULT AN INFERTILITY specialist."
},
{
"id": 125924,
"tgt": "What causes swelling and soreness in the groin area?",
"src": "Patient: i have on the inter right leg groin area more to the buttock tho swelling and sore to touch it seems to be attached to the bone below the buttocks/upper leg bone. what could this be i have had it come and go over the last 6 months and lasting 3 or so weeks. Doctor: Hello, It could be due to musculoskeletal causes. As a first line management, analgesics like Aceclofenac or Tramadol can be tried. If symptoms persist better to consult an orthopedician and plan for an MRI scan. Hope I have answered your query. Let me know if I can assist you further. Regards, Dr. Shinas Hussain, General & Family Physician"
},
{
"id": 25447,
"tgt": "What medication is suggestsed for tightness in chest and left shoulder pain?",
"src": "Patient: i Have tightness in my chest. My baby is 5 months old and worried iv pulled a muscle. It hurts around my left shoulder pain and sometimes when I'm sitting alone there is no pain but when I'm active n up and about its quite tight around my chest area. I wondered if it was from bathingg, lifting, pushing pram etc? Doctor: hiThanks for writing to HCM.Paracetamol is a safe analgesic. Local application of Voveran Emul gel or Felbinac also might help.Dr. Pinak"
},
{
"id": 208925,
"tgt": "Suggest remedy for loss of concentration after masturbation",
"src": "Patient: hi.... i am a man,182 cm,82 kg, and i am 23 years old, i did masturbation two times a day after i did it i felt trouble i can not focus my mind on anything ,is that true or it's just imagination and if true what i need to return to my usual state again .... thanks Doctor: HiThanks for using healthcare magicAfter masturbation, getting tiredness is normal. It is due to loss of energy from the body. It happens with every one after doing masturbation or sex. In that case, you can take 5 or 10 minutes power nap and that would help you to come out of that tiredness. In case, you need further knowledge about that you can consult me.Thanks"
},
{
"id": 17890,
"tgt": "Suggest treatment for hypertension, borderline diabetes and chest pressure",
"src": "Patient: Hello, I was told I have intra-atrial conduction delay with moderate high-lateral re polarization disturbance, consider ischemia or LV overload. Slight right-precordial repolarization disturbance, consider feminine pattern. This was on the ekg report. P/PR: 138/166 QRS: 92ms QT/QTc: 370/465 ms P/QRS/T Axis: 82/34/86 deg Heart Rate: 95 BPM I had an allergic reaction to the flu shot I was given. I believe it s from the preservative. I m not allergic to eggs. usually get rapid heart rate, high blood pressure, and chest pressure sensation. I took two 25mg Benadryl and was put on a prednisone pack. I have an appointment to see a cardiologist in two weeks. The triage doctor said I need to be careful of taking certain medications. My father passed away from a massive heart attack at 65 years old, he was also diabetic and had high blood pressure that started when he was in his early 30 s, and my mother had a 90 percent blockage about 5 years ago, she has high blood pressure, and she s borderline diabetic. So I see a cardiologist once a year because of my family s history. I have high blood pressure when I was 30 years old (I was 115 lbs so they checked my kidneys and they were normal). I was diagnosed with a slight heart murmur that is not always present, was heard when I was a teenager once, when I was pregnant, and a couple of times after I had my daughter. I m a borderline diabetic, A1C 5.7, and my cholesterol is normal. Sandra Doctor: Hi, For blood pressure you can take tablet Metoprolol after consultation with cardiologist. Follow diabetic diet after dietitian consultation. For chest pain kindly do angiography after consultation. Hope I have answered your query. Let me know if I can assist you further. Regards, Dr. Shyam B. Kale, General & Family Physician"
},
{
"id": 153450,
"tgt": "Can I get colonoscopy done without drugs?",
"src": "Patient: hi: I need to have a colonoscopy to evaluate probable colon cancer but I m unable to find anyone to do an unsedated exam (allergic to the drugs)......had a virtual but need a regular colonoscopy (polyps,masses etc). most of the world does these without drugs; any suggestions ? Doctor: Hi,Thanks for writing in.Please discuss with your doctor for alternative pain relief solutions and it might be possible to get through the colonoscopy without the requirement of allergy causing medicines. It is possible that you might be able to tolerate certain medicines and a test dose might confirm this.Discuss the options available and take test dose of local anesthetic and then know from your doctor if you are able to tolerate those medicines and get colonoscopy done. Please take note of the medicines which you are allergic to."
},
{
"id": 174438,
"tgt": "Suggest treatment for persistent wheezing",
"src": "Patient: My son is 8 yrs old and he is having wheezing for past 3 yrs.For past 3 yrs his wheezing has increased only. When ever he gets cold ,he gets cough and starts wheezing . It worsens in winter. Please suggest some medications and exercises for wheezing so that he is not attacked by wheezing when he gets cold. Doctor: Hi,Thank you for asking question on health care magic.Cold acts as a trigger for wheezing.MOnteleucast a leutriene antagonist daily at bed time prevents attacks.Ebastine an antiallergic medication reduces nasal allergy and prevents attacks.Beta agonist like salbutamol inhalation will abort an attack.Steroids reduce inflammation and controls severe asthma.Hope this answer will serve your purposePlease feel free to ask any more queries if requiredTake careDr.M.V.Subrahmanyam MD;DCHAssociate professor of pediatrics"
},
{
"id": 37307,
"tgt": "Suggest remedy for itching ringworm infection",
"src": "Patient: I had what was told to be ringworm on my arm/ side of elbow...which has spread to my back, Both sides of my thigh and a patch on my back leg and on my neck..it inches so bad that I scratch and it spreads and never goes away... What can I use and What should I do Doctor: Hello,Welcome to HCM,The ring worm is caused by a fungus. They thrive on moist surfaces due to sweat formation, they infect that area and the infection spreads if not treated. Wearing wet clothes and not wiping that region completely dry creates a favorable condition for the fungus to multiply. Ringworm infection needs to be treated with antifungals.You can apply clotrimazole cream or terbinafine cream. Either of these has to be applied twice a day. They have to be used for two to four weeks depending upon the area infected.I would advise you not to use clotrimazole combination with steroid cream since it basically masks the infection. You should use a plain clotrimazole cream.If the infection is not controlled by application of cream for a month then oral antifungal might be needed.Thank you."
},
{
"id": 218655,
"tgt": "Are Drotaverine and Progesterone safe during pregnancy?",
"src": "Patient: Hello Sir I am a Veterinary Surgeon. My wife is in one month pregnancy and doc. has recommended her Tab-Drotaverine 80mg BD along with Tab-Micronised progesteron 200mg BD and few supportive drugs for morning sickness, owing to complaint of mild abdominal pain. Is that ok Sir? Doctor: Hello,Yes, the trust's okay drotaverine reduces pain, which his wife's having and progesterone supports pregnancy, so it will help the pregnancy to continue. So nothing to worry.Hope I have answered your query. Let me know if I can assist you further.Regards,Dr. Mandavi Rai"
},
{
"id": 62729,
"tgt": "What is the treatment for a lump in the forehead?",
"src": "Patient: Doc, my daughter fell off from BBQ in the park, stand on the grill but wasnt stable so she shoot down to the ground and bump her head; bruises in her forehead and lump; i have put ice after 5 mins of the accident. should I worry? How to get rid of her bump on her head? Thanks. Doctor: hi.did she have any loss of consciousness, changes in mentation and vomiting when she hit her head? if there's none, observation may be done. if there is, a cranial ct-scan might be of help to further evaluate her condition. the swelling/inflammation is normal and will eventually subside after 2-3days time.hope this helps.good day!!~dr.kaye"
},
{
"id": 77383,
"tgt": "What causes pressure in chest, constant burping and difficulty in breathing?",
"src": "Patient: i have developed a condition that seems similar to gerd. i have so much pressure in my chest that i cannot breathe sometimes. ialo have to burp constantly .i have been to the hospital er several times thinking i was having a heart attack .i dont have insurance and am very low on money what can i do? Doctor: Hi thanks for asking question.Here you seems to have Acid reflux disease if associated abdominal pain also present.omeprazole taken for it.At night put your head above pillow as head rise position.avoid smoking .Second if you are doing heavy exercise or weight lifting then also such symptoms can occur.If you are taking excess stress then anxiety induce chest tightness can occur.so try to avoid stress.You can do chest x ray for rule out respiratory causes.Avoid smoke.Garlic and ginger good for respi tract.I hope my suggestion will help you"
},
{
"id": 15265,
"tgt": "Rash on face spread to whole body. Taken Piriton, antibiotics. No relief. What is this rash?",
"src": "Patient: My daughter is 9 yrs old, she as cum out in rash it started on face bt as now covered her whole body, taken her to one doctor he said he doesn t no wot it may be and gave me some pirtion, then it spreaded more so went and seen another doctor who said it could be either viral or bacterial infection , so gave me some antibiotics . The rash/spots on face as faded bt have gone onto other side of face. Tonight she is being sick. Do u know wot this could be?? Thanks a worried mother. Doctor: Hi rash on face followed by rash all over thre body without any fever or fever after 5days indicates that is not viral exanthem or dengue fever.fever might be in this case is due to secondary infection.it should be treated with atarax 25 mg at night.antibiotic ceftriaxone twice daily and application of calamine lotion can cure it.if no response start oral steriods"
},
{
"id": 105604,
"tgt": "Asthma, green phlegm with red spots. Had taken antibiotics, steroids for chest infection. What should be next step?",
"src": "Patient: I have had an infection in my chest for around last Monday, i went to the doctors on Tuesday, saying i an Chest Infetion/ Viral Infection, they put me on Antibiotics and Steriods. On Wednesday, the pain started at the side of my chest, leading up towards the top. I went to my out of hours GP on Friday, she said i should go to A+E. For an chest x-ray . I had blood tests and around 7/8 hours of tests. And they still weren t certain on what it was. Or if they are going to keep me in. I left anyway an hour later with a final decision that it might be Pleurisy , i couldnt take a deep breath in probably during the x-ray, so they gave me more Antibiotics and Steriods, until today. The pain still continues around the back of my rib cage, i haven t been sleeping well. Im still very tired/weak. I also have asthma , but normally a fit 19 year old, female. Also i am still coughing up green/yellow plenum, which this morning i noticed red spots? What do think i should do, just leave it and see how it goes? Or do i go back up the doctors? Help please. Doctor: Hi, Thanks for writing in. You seem to be suffering from pneumonia. This will usually cause a chest pain (which increases on breathing in), difficulty in breathing, fever and cough. An empiric course of antibiotics would be definitely the way out. You would require antibiotics like amoxicillin-clavulanate and levofloxacin. A total course of 7-10 days would be required. Please see your physician. This should not be ignored. Hope this helps"
},
{
"id": 194153,
"tgt": "Will frequent masturbation affect sexual life?",
"src": "Patient: hey doc i want to ask that i am masturbating since i am a child now i am 24year..i am doing this daily if i mean daily means daily so im really concern about that regular masturbation may efeect my future sex life or having problem in kids plzzz answer me....waiting for your reply Doctor: Hello, Masturbation is not bad but regular masturbation needs more energy and nutrition. You need a constant supply to prevent tiredness. For reducing masturbation you can join an athletic sport or yoga. If the problem persists need a psychiatrist review for medication to control masturbation. Hope I have answered your query. Let me know if I can assist you further. Take care Regards, Dr S.R.Raveendran, Sexologist"
},
{
"id": 193888,
"tgt": "Can maca root balance hormone levels?",
"src": "Patient: Can maca root give a male a bigger butt? I'm 5'8, 140 lbs, 25 y/o. I'm also a ts who is on hormones? I heard maca can balance your hormone levels, but will that affect me badly (more testosterone and less curves)? Should I just stick to my regular hormone regimen and workout routine? Doctor: Hello, I would not recommend using the mentioned herbal product without doctor advise to increase your libido. Instead, you can take testosterone supplement if your testosterone level low. Try to maintain your weight within the normal range. Kindly let me know your serum testosterone level and USG pelvis report to guide you further. Hope I have answered your query. Let me know if I can assist you further. Take care Regards, Dr Parth Goswami, General & Family Physician"
},
{
"id": 223067,
"tgt": "How to get pregnant?",
"src": "Patient: pawan 30 nidhi 29 we do not have any child since last 5 years of our merriage.we do not have any problem .all the reports are normal.our sexual life is healthy.we have consulted many doctors all give us treatment.but there is no any profit. please tell us what we can do. Doctor: Hi,If you have been for evaluation for infertility and all reports have come back normal. And, if both you and your wife do not have any medical or surgical illness and have no problems with intercourse, it makes this a diagnosis of unexplained infertility.Unexplained Infertility of 5 years is best helped with IVF. And since both of you are in 30 years, the results of IVF can be promising.I would advise to go directly for IVF, without wasting much time and money on other treatments, if you both have no objections to IVF.Hope this helps .Best regards."
},
{
"id": 20174,
"tgt": "What causes heaviness in chest after a short walk post angioplasty?",
"src": "Patient: I am 6 month status post angioplasty I had LCX at present I am taking delisprin 150, pasugrel 10 mg& Rosovas 20 mg. I am vegetarian and non alcholic i am feeling heavyness in chest after a short walk but at times I can do medium load physical work but no such symptoms.Can u suggest me the cause of such symptoms at times.? Doctor: Welcome and thank you for asking on HCM!I have gone through your question. As you are having chest pain onexertion post angioplasty an angiogram (repeat ) is recommended.Hope to have been helpful!Wishing good health, Dr D Sunil Reddy"
},
{
"id": 225204,
"tgt": "On B-Yaz and looking into taking Hydroxycut. Will it have any negative affect on the birth control ?",
"src": "Patient: Hi I am currently taking B-Yaz and was looking into taking Hydroxycut but I wanted to make sure that it wasn t going to negatively affect my birthcontrol . I know when I took the Complete Nutrition CTS 360 for a few days it made me spot really bad so I figured I should probably not take it. Do these supplements affect my BC? Doctor: Thanks for query. as you mentioned you previously had a bad experience with such products it better yo avoid.it is difficult to be certain about the effects of hydroxycut as it is not a licensed pharmaceutical.so information about its interaction are limited.coming to weight loss it is always healthy to cut down weight by negative energy balance i,e burning more calories than you eat .hope you got the answer.regards"
},
{
"id": 80984,
"tgt": "What causes difficulty breathing?",
"src": "Patient: It feels like I have an air pocket in my lung on the right side of my chest. When I swallow air to try and release it I feel it there but it will not release. This has been happening for a few years now and it seems to be getting worse. What could this be? Doctor: Thanks for your question on HCM.I can understand your situation and problem.In my opinion you may have bulla or blebs in lung.They are air filled structures in lung parenchyma.So you should consult pulmonologist and get done1. CT thorax2. PFT (pulmonary function test)CT thorax will be needed to know whether bulla or bleb present or not.PFT will be needed to rule out bronchitis present or not due to bulla and blebs.So consult pulmonologist and discuss all these.If both are normal then no need to worry much. You may have anxiety only. So avoid stress and tension. Be relax and calm."
},
{
"id": 82026,
"tgt": "What causes chest quaking?",
"src": "Patient: I have chest quaking and then a cough, it comes and goes, last year it lasted a month and it has started again, when I am not coughing it seems like my heart Is beating a little hard, no discomfort or pain. I have felt it for a number of years but the last two years it seems to have been worse Doctor: Hi, thank you for asking Healthcare magic.With this presentation. I think it could probably be due to allergy especially asthma. You need to be assessed clinically by a doctor and some tests like a chest X-ray, spirometry, a complete blood count and IgE level may be needed.I suggest you see you see a general practitioner for orientation or a specialist on chest medicine for proper assessment.I hope that helpsI wish you well"
},
{
"id": 182650,
"tgt": "How to remove black dots on teeth permanently?",
"src": "Patient: hi Dr Justin Glaister , i would like to ask u , i have a problem i have smaaall black dots on my teeth and i removed them for a couple of times before but they keee on comming back how can i remove them once and for all are they a kind of bacteria ? or maybe a deficiency in something and thats y they come bk thankxx... Doctor: Thanks for your query, I have gone through your query.The stains can be because of the habits like coffee, tea, or smoking. The other possible cause of black stains can be because of the chromogenic bacteria or it can be secondary to the use of mouth wash also.Consult a oral physician and get it evaluated. If it is because of the coffee or tea or smoking, get the teeth cleaned and avoid the habits and rinse your mouth soon after eating anything. If it is chromogenic bacterial infection you need to consult a oral physician and take a course of antibiotics and get scaling done.I hope my answer will help you, take care."
},
{
"id": 27282,
"tgt": "What causes heartburn with excess sweating?",
"src": "Patient: I have had heartburn since 2011 it is not heartburn because it burns every single day of the week even.when I don t eat anything I can.just walk or bend over and get bad burning in my chest plus the chest pain and it apears that underbmy left arm pit is a burninh and it feels as if blood.vessels are swollen to.touch I can feel.them moving when I put my hand there and.rub it up and down also when I touch it my chest starts to hurt very badly I have been.sweating none.stop all day and feel very hot when I have the air on and tired.. I have no clue what this could be Doctor: Hello. Thank you for asking on HCM. I read your query and understand your concern.Now, the description of the heartburn you provided, is with highest probability from the stomach, as the origin. This symptom is called pyrosis, and is produced when the highly acid content of the stomach is returned to oesophagus or, sometimes, throat and mouth. This can happen from various reasons, mostly from a disbalance between acid-producing and acid-inhibiting factors in the stomach and from the lower oesophageal sphincter insufficiency. Without digging further more into theoretical details, I would suggest you to refrain from certain food and beverages that favour acid production and/or inhibit acid-inhibiting factors and/or lower the pressure of the lower oesophageal sphincter:- alcohol and tobacco- chocolate- tea, coffee (xanthyne derivatives)- spicy and grilled food, and picklesI would also recommend you to start a course with a proton pump inhibitor (PPI): omeprazole 20 mg x 2 tb/d for the first two weeks, then 20 mg x 1 tb/d for four other weeks, and accompanied with domperidone x 3 tb/d. Also, you should not lie down, but sit up or stand, for at least 90 minutes after you have had your last meal. If the symptom persists after all these measures are undertaken, I would recommend a upper gastro-intestinal (GI) endoscopy, for further investigation.I hope I was helpful with my answer. Best regards."
},
{
"id": 153852,
"tgt": "Suggest treatment for stomach cancer",
"src": "Patient: Hello Doctor, I nee need to ask abt my uncle. By reports we come to know that my uncle got stomach cancer and cancer cells are sped to liver.Now doctors removed the stomach of my uncle and they told us that they gonna cut the infected parts of the liver. but i need to know after doing this is my uncle will out of the cancer and can he lead the normal life...? Doctor: Hi, dearI have gone through your question. I can understand your concern. Your uncle has stomach cancer. If cancer cells dpreaf to the liver then it becomes stage 4 cancer. It is difficult to treat. He should take chemotherapy like oxyplatin amd other drugs. He should regularly check CEA level to check cancer cells. Consult your doctor and take treatment accordingly. Hope I have answered your question, if you have doubt then I will be happy to answer. Thanks for using health care magic. Wish you a very good health."
},
{
"id": 11619,
"tgt": "How to treat darn patches on the face when the rest of the body is normal?",
"src": "Patient: Sir, i am 47 years old working lady. I am having dark patches on face. I am wheatish but my face is very darker than whole body. It looks very shabby. I have taken treatment of five to six Dermitoligsts but no use. Kindly help me in this matter.Mrs. Patil YYYY@YYYY Doctor: Hello,Welcome to healthcare magic.The dark patches on your face could be melasma, lichen plan us pigmentosus or Reihl' s melanosis.Since you have met 6 dermatologists, I presume an appropriate diagnosis has been made as each condition has a different prognosis.However, some common steps you can follow are - 1. Always use a broad spectrum sunscreen with SPF 30 or above every 3 hours.2. Cover your face with a scarf in addition to using a sunscreen.3. Eat plenty of colored fruits and vegetables.4. Take oral antioxidants.5. You could try lightening agents containing kojic acid, liquorice, arbutin and niacinamide at bedtime.6. You could try chemical peels like glycolic peels and retinol peels.Hope these help you.Take care."
},
{
"id": 197447,
"tgt": "How to get rid of masturbating habit?",
"src": "Patient: i am 21 year old.i started masturbating at the age of 16 year.i did it twice or thrice in a week since then.now i want to stop doing it anymore but the thing is that now i m worried that is masturbating leads to impotency.my ejaculation reduces to 1-2 min and even not sure about the semen count..plz help as i m young enough to recover if i have any problem Doctor: Hi welcome to health care magic....I can understand your concern...It is good now you are masturbating only 2 to 3 times per week which is normal...If frequency more than that then there is worry...Still to avoid overmasturbation watch less porn if habit....Yoga , meditation and regular exercise done.Healthy and balanced diet taken.Fruits more....Avoid bad habit like smoking and alcohol.For your sperm count you can investigate with semen analysis ...Collect semen in clean container by masturbation...Take care.Hope your concern solved."
},
{
"id": 185681,
"tgt": "Suggest treatment for a painless flat pale spot on the gum",
"src": "Patient: I have a painless flat pale spot on my bottom gum, below one of my canines. It looks a bit like a round blister but is slightly irregular at the bottom of it. It doesn't appear to have any fluid in it. It feels only slightly irritated if I touch it with my fingertip. Doctor: Hello,Mouth ulcers are fairly common. Although uncomfortable, they will heal in a few days. Treatment is symptomatic only. Salt water rinsing, gentle care to avoid area, check for vitamin deficiencies and avoid spicy foods. There are some liquid or gel medications that can offer a proteciive coating with an analgesic to offer relief. Stress or trauma can cause these to occur and can reoccur. Commn in the mouth: apthous ulcer.If the area is not resolving and symptoms increase, your concern could include an infected tooth with a fistula. This means the abscess is draining at the spot mentioned. I advise you to have a dental exam for a correct clinical diagnosis. There are multiple medical conditions that can manifest as mouth sores. The range can be from a simple allergic reaction to a cancer."
},
{
"id": 166899,
"tgt": "What causes the feeling of wetness after peeing?",
"src": "Patient: i have a 6 year old boy that uses the bathroom and when he s done he feels like he peed his pants, but nothing is wet. i have noticed a couple of days ago that he has a black spot with a hole around his ring. he hasen t complained or noticed it, but i seen it when i was drying him off after his bath. could the black thing be a blackhead and whats causing him to feel like hes wet? Doctor: Hello and Welcome to \u2018Ask A Doctor\u2019 service. I have reviewed your query and here is my advice. I have gone through your question. Do not worry. This can be black head or pigmented naevi. I would recommend you to visit a pediatrician for complete clinical evaluation and definitive treatment.Hope I have answered your query. Let me know if I can assist you further.Take care.Regards,Dr. Archana Verma"
},
{
"id": 37302,
"tgt": "What to do for kidney infection and cystitis?",
"src": "Patient: hi, iv been suffering with kidney infections and cystitis on and off for the past 2years iv seen plenty of doctors and had scans and what not done, iv had to try cut alot of alcohol and sugars out of my diet, i take D-Mannose tablets to try prevent it they work but if i forget them even once i get the stinging sensation i woke up early this morning with a random stinging sensation i took my tablets and had bread soda with water and milk to try get rid off it. its very uncomfortable and making me feel nausues the only thing that eases it a bit is sitting on the toilet Doctor: Hello,Welcome to HCM,You are suffering from what we call recurrent uti. Since it is recurring frequently you need some investigations to find the cause. You need urine culture, vaginal swab culture, ultrasound of abdomen. Check your blood sugar. You may need an x-ray call voiding cystourethrogram to look for reflux of urine into the kidneys. If everything is normal you may be a person more susceptible for infection because of your genetic make-up.I would like to advise you the following:1. Recheck you infection parameters like Total white blood cell count.You can also get your Urine routine examination done (culture may be done, but it may be negative). A course of antibiotics may be repeated.2. Get your Random Blood sugar levels done.3. If not relieved, you can get a USG of kidney/ureter/bladder done, since stones in the urinary tract can lead to persistant UTI and flank pain. 4. Drink plenty of water/fluid (>2/day)5. Avoid using unclean washroom and maintenance of personal hygiene is important.Please consult your physician for further clinical check up and investigations.Thank you."
},
{
"id": 56925,
"tgt": "What could the blood test report indicate?",
"src": "Patient: I had my gallbladder removed on Jan 25 2012. After being released I had blood work done. I received a call on Feb 15, I was told my alt was at 84 and I need to see the doctor. My appointment is on monday but I am very worried. Should I be? I don't do drugs, or drink a lot, no hep. Or jaundice. Doctor: Hi, dear. I have gone through your question. I can understand your concern. Your alt level is slightly high. There are many causes for that. You may have some mild liver insult. Its not a cause of big worry. You just consult your doctor and go for complete liver function test. Then if needed take treatment accordingly. Just be relaxed. Its not serious. Hope I have answered your question, if you have doubt then I will be happy to answer. Thanks for using health care magic. Wish you a very good health."
},
{
"id": 85814,
"tgt": "Is it safe to take Guafinex for more than a week while suffering from COPD and allergies?",
"src": "Patient: I have COPD and have allergies. I have been taking Guafinex (600 MG Guafinacin) every 12 hrs for 7 days. I am also a liver transplant patient (16 yrs. out). Is it safe to continue for a few more days until allegires subside completely? I am drinking plenty of water. Doctor: Hi, It's a really safe medicine that is in most over the counter medicines. Lots of people take it for months. I hope this answers your query. I remain at your disposal in case further medical assistance is needed. Regards, Dr. Matt Wachsman, Addiction Medicine Specialist"
},
{
"id": 25725,
"tgt": "What causes palpitations?",
"src": "Patient: I was sitting here watching tv and i I was sitting here watching tv and i felt my heart beating irregular i felt my pulse and it was as if my heart would beat twice then pause then twice and pause. It has since then returned to normal I just recently saw my doctor for palpatations that have been lasting for weeks the attempted to. Doctor: Hello!Welcome and thank you for asking on HCM!I understand your concern and would explain that your symptoms could be related to cardiac arrhythmia. Anxiety could also mimic this clinical situation. I recommend consulting with your doctor for a careful physical examination and some tests: - a resting ECG and a cardiac ultrasound to examine your heart structure and function- an ambulatory 24-48 hours ECG monitoring to examine your heart rhythm trends during the day- lab blood tests (complete blood count, thyroid hormone levels, blood electrolytes). I recommend discussing with your doctor on the above issues. Hope to have been helpful!Best wishes, Dr. Iliri"
},
{
"id": 191496,
"tgt": "What causes low blood sugar level and dizziness while on Metformin and Glipizide?",
"src": "Patient: I am 5 9 300 pounds, diabetic taking Metformin, Glipizide, fluoxetine and Lisinopril. My a1C was at 5.8 on my visit last week. I am also taking a Fat burner pill from wakaya, Turmeric and Ginger... The past couple of weeks my blood sugar has been dropping in the mornings after I take my pills. I get light headed, dizzy, anxious and sweaty. I recently repurchased a glucometer and the past three episodes my blood sugar has been at around 57 when I have these symptoms. But it was at 120-140 when I woke up. I am wondering if I should stop taking the glipizide. Doctor: Hi,I understand your concern regarding your low sugar levels.Let me tell you that if your a1C was 5.8,then there was no way that you continue taking Glipizide.You are at a very good a1C level and you just have to maintain it so Metformin will be sufficient. All the symptoms that you were experiencing were of hypoglycemia.Please stop Glipizide ASAP.Hope my answer helps.Please feel free to ask more questions.Thanks,Dr.Prabhas Verma"
},
{
"id": 46660,
"tgt": "What causes pain and discomfort in and around kidney area?",
"src": "Patient: hello 3 days ago i had discomfort on the lower left side where the kidney is (i thought i was maybe passing a stone, my brother had this not too long ago). it got better after 12 hours now its comming back daily same place with occational pain in the left groin and shooting towards the right kidney maybe once or twice a day briefly. Also my hip on the left side feels numb on the surface from the kidney area to the full left buttock. the pain can go away for a few hours then come back id say the maximum level of pain is a 7 from 1 to 10. i smoke and am overweight by 15 kilos all the fat seems to be in my spare tire if know what i mean. i am usually in good health. Doctor: Hello and welcome to HCM. As an Urologist,i can understand your concern.The kidneys aren't located in the groin area,but in the loin region.!!To know the cause for your pain,you'll need to do a CT-KUB scan.That will clear your doubt about stone and if your pain is due to it.Also,it may give some idea about your hip pain.According to the report,an expert opinion can be given,about treatment.You may take an anti-spasmodics for pain relief,if the pain is severe.If you've any doubts,send all the reports,to me as a direct question.Dr.Matthew J. Mangat."
},
{
"id": 9665,
"tgt": "Swollen ankles with red shiny skin on calves",
"src": "Patient: I am 34 and I have been suffering with swollen legs and ankles , to touch the skin feels like i am being stabbed with glass the skin isnt broken and its not a dvt as i have had tests for that i have also had xrays and there is no breaks, they are really itchy and very sore Doctor: hi, there are two chances for this 1-RHEUMATOID ARTHRITIS 2-GOUTY ARTHRITIS Any way consult one rheumatologist or ortho doctor and do the best thing early thanking you christyjoseph1@gmail.com"
},
{
"id": 91555,
"tgt": "How to treat abdominal pain due to enlarged lymph nodes?",
"src": "Patient: my sone has left sided abd pain..lt pelvic and rt pelvic pain in the groin area.. can only stand for a short time and sitting impossible for the pain..ct showed inlarged lymph nodes..slight elevation in wt count. he has been 10 weeks and unable to get any relief other than rest and pain meds.. he is 32 6ft 3..approx 220 lb Doctor: Hi. If there is no relief , it is mandatory to get the biopsy of the node done ASAP to get a correct diagnosis . it may be a serious problem.As a rule - anything not getting OK within 1 to 3 weeks maximum should be removed and tested. Noe-a-days we can do this by laparoscopy too , one day care surgery..."
},
{
"id": 126763,
"tgt": "What can cause severe pain in the shoulder and stiffness in the arms?",
"src": "Patient: Husband has had 3incidents of severe pain in right shoulder. Down his arm and left hand also stiff. He has gone to clinic been given steroid and pain meds. He also has again and signs of neuropathy in feet. Has been told shoulder pain could be rotator cuff or punched nerve. Possible? What could this be? Doctor: Hi, Acute pain in shoulder without trauma can be due to calcific tendinitis of rotator cuff tendons. If there is trauma pain can be due to cuff tear. Diagnosis needs to be confirmed by clinical examination. 1. X-ray of involved shoulder anteroposterior and axillary views. 2. MRI of the shoulder. X-ray helps in identifying calcific tendinitis. MRI is sensitive and specific for diagnosis of soft tissue pathology. Please get it done from radiology center nearby you. Treatment depends on cause. For pain relief you can take tablet Ultracet twice a day. Visit to orthopedic surgeon for detailed clinical Examination is recommended. Hope I have answered your query. Let me know if I can assist you further. Regards, Dr. Jayesh Vaza, Orthopaedic Surgeon"
},
{
"id": 167562,
"tgt": "Can Ucerax syrup be used in 10 month old child with eczema?",
"src": "Patient: my son is 10months old now, he been suffering from excema since he was only less than a month old... he have very dry skin also he scratches himself really bad, today we just got ucerax syrup(Hydroxyzine Hydrochloridel) by GP prescribed,just wondering if its safe to use on a 10 months old baby, bcs on the leaflet actually says not suitable for children under 12 months...thanks Doctor: Hi....better to avoid a drug whose safety has not been established in this particular age group. It might cause excessive sleepiness. The trick of the management is keeping the skin moist and never allowing it to be dry. For this oil massage and lotions like Oilatum will help. Regarding cure options - eczema is a sort of allergy...allergies can only be controlled and can never be cured. But usually kids grow out of it...that is as the age progresses.Hope my answer was helpful for you. I am happy to help any time. Further clarifications and consultations on Health care magic are welcome. If you do not have any clarifications, you can close the discussion and rate the answer. Wish your kid good health.Dr. Sumanth MBBS., DCH., DNB (Paed).,"
},
{
"id": 207092,
"tgt": "Could the bad and persistent thoughts towards a person be due to OCD?",
"src": "Patient: hi my name is emily and i want to know if i have ocd i keep having this bad thoughts and i keep having to go back to make sure a loved one is ok also i have to make sure i did something right or said something right and if i touch something i have to make sure i touched it a even number of times and if i dont im scared someone is going to get hurt do i have ocd? Doctor: Hello Emily,You are right in diagnosing your problem. The problem is of OCD. For the treatment of this, you have to start with drugs like sertraline. As the severity of symptoms will decrease, u can take sessions of behaviour therapy from your nearest psychotherapist.Thanks."
},
{
"id": 33012,
"tgt": "Could yellow skin be related to viral infection?",
"src": "Patient: Hi, may I answer your health queries right now ? Please type your query here...I had a blood test to determine yellowing of skin, everything came back normal. Awhile back another doctor said I had a viral infection this was 3 months ago. Could this all be related? I am a 35 yr. male 5ft 11dont have never had medical conditions. Doctor: Hello,Yellow skin or jaundice is caused by liver problems. Make sure that your doctor checked your liver enzymes and did specific tests for hepatitis B and C. It depends upon what type of viral infection you had as to whether or not that is the cause of your yellow skin.If your doctor can not find the cause of yellowing of skin then I suggest that you see a liver specialist.Regards"
},
{
"id": 101238,
"tgt": "Suggest treatment for consistent dry heaving cough",
"src": "Patient: I have a consistent dry, sometimes heaving, cough. This has been going on for a year or more. I am diabetic and have high blood pressure, high cholesterol, asthma, and gastroparesis. My med list is overwhelming...up to 12 tablets a day plus insulin. Could the losartan I take for pressure be the cause of my crazy cough spasms? Am I having an allergic reaction to it or the combination with my other meds. 65 units of Novalog insulin before am/pm meals 10mg Zetia daily 3.125mg Carvedilol twice daily 5mg (2) Glyburide twice daily 20mg Omeprazole daily 25mg Hydrochlorothiazide daily 5mg Metoclopramide 4 times daily (be efore meals and at bedtime) 20 mg Pravastatin daily 50mg Losartan daily 160/4.5 Symbiotic 2 puffs twice daily 90mcg Abuterol 2 puffs four times daily 59mcg Fluticasone Propionate 2 sprays each nostril daily 2500mcg Methycobalamin (B12) daily 1000 IU Vitamin D daily 500mg Inositol daily 3000mcg Biotiln daily 50mg Tramadol as needed for pain every 8-12 hours Doctor: Welcome. Ace inhibitors r known to cause dry cough. losartan is an arb receptor blocker but it too can cause dry cough less so than ace is. u need to do 2 things 1st) ask ur doc to change ur bp med to observe any improvement, perinodopril is a good option. 2ndly) go to a pulmonologist n get ur pulmonary function tests, cxr pa view done to rule out any co morbidity n do proper referal. thanx hope that helps."
},
{
"id": 131549,
"tgt": "Suggest solution for muscle optimalization?",
"src": "Patient: i am 62 yrs old, Indonesian man, ideal body weight, hypertension (mild ; 150/90 mmHg), routin aerobic (cycling) and lawn tennis, ECG normally, MRI ; a little hypoxy at the left brain. But I have hyperaggregated blood, is it caused my brain hypoxya ? My chief complaints is suddenly, my right lower arm is weakness. During hospital care is getting better, but not yet optimal condition. Would you like to give me other solution for muscle optimalization ? physiotherapy and clopidrogel and irbesartan 150mg are continuosly treament. thanks. doc... Doctor: HiI may suggest addition of ecospirin 150 mg too.There should be periodical check of platelets count,CBC,PT..if you are on blood thinners.Avoid smoking.Your doctor may do Color Doppler to forearm for vascular study, clot,thrombus..A physician must explore causes for hyperconjugation factors of blood, possibly may suggest some plasma infusions periodically if some deficiency is detected in blood..you may consult an clinical hematologist also for thorough evaluation as you stand risk for clot, emboli in vital organs,brain, lungs, limbs..."
},
{
"id": 81264,
"tgt": "Suggest treatment for chest pain during night",
"src": "Patient: I have acid reflux disease and am on meds for it. I have been getting a pain in my chest at night mostly where it feels like someone is trying to push their fist through my chest. This just started in the last couple of weeks. I try to relieve the pain by taking gaviscon and a few tums. Is there something else I can try? Doctor: Thanks for your question on HCM.In my opinion we should first rule out cardiac cause for your chest discomfort.Because night time chest problems are commonly seen heart disease.So get done ECG to rule out cardiac cause.If ECG is normal than no need to worry much for cardiac cause.You are mostly having chest discomfort due to GERD (gastroesophageal reflux disease).For better control of GERD,along with drugs, you should follow below mentioned lifestyle modifications.1. Avoid hot and spicy food.2. Avoid stress and anxiety.3. Avoid large meals, instead take frequent small meals.4. Go for walk after meals.5. Keep 2-3 pillows under head in bed to prevent reflux.6. Loose weight if you are obese.7. Avoid smoking and alcohol if you have these habits."
},
{
"id": 138197,
"tgt": "Suggest treatment for severe jaw pain",
"src": "Patient: I have been having jaw pain on the left side near my ear. It does not have the popping and clicking associated with TMJ although the pain in that general area. It interferes with the opening of my mouth. At times I cannot open my mouth wide enough to put even a small spoon into my mouth. Other times, I get stabbing pains when chewing. Other times I can open my mouth for regular talking. Sometimes I get a stabbing pain in the tmj area for no known reason such as .... not chewing, not talking, not opening mouth for eating or drinking etc. Can you lead me to some reason this may be happening? Doctor: Dear patient You are having symptoms of left tm joint arthritis. This needs to be investigated with mri of left tm joint to confirm diagnosis. Visit radiology center nearby you and get it done. Meanwhile avoid solid foods and put yourself on liquid diet semisolid diet. You can take tab diclofenac 50mg twice a day for pain relief. You need to consult expert ENT specialist with mri report. All the best."
},
{
"id": 52254,
"tgt": "What causes pain from breastbone to right side of the lung area post gallbladder removal?",
"src": "Patient: i have been having sharp pain , then mostly dull aching pain, from breastbone to right side inside right lung area, very tender, coupled with orange stools, i have had orange stools for two and a half to three months i had my gallbladder removed, it almost feels like a gallbladder attack, some days even water can trigger these pains Doctor: Hello, Abdominal pain after gall bladder removal suggestive of postcholecystectomy syndrome. For that Colestipol like a tablet can be prescribed. But you need to investigate with USG abdomen scan and MRCP like investigation according to need to rule out any stone in biliary tract. Hope I have answered your query. Let me know if I can assist you further. Take care Regards, Dr Parth Goswami, General & Family Physician"
},
{
"id": 219451,
"tgt": "Is Cefolac and Gestakind good for cough and cold during pregnancy?",
"src": "Patient: i have 14 weeks pregnancy, age 28 yrs old, during last one week i caught wih cough and cold and sometimes feeling pain in my lower abdomen. Today during my routine check up with my doctor, I asked about cough and cold and about pain. My doctor prescribed to take Cefolac - AZ and Gestakind- SR. Is the medicine prescribed by my doctor is ok ? Please let me know. Doctor: Hi there,,Any infection in preganncy needs to be treated promptly a lot of women do not take medication thinking it may adversely affect the baby, but it is very important to get the infection treated.The cefolac is a antibiotic useful in respiratory tract infection and is safe in pregnancy. Moreover the first 12 weeks is a very crucial time and after this period it is generally safe to take medications.Do complete the course of this antibiotic.The gestakind is a medication which is helful in cough and also relaxes the womb so do use it.Hope this helps.Regards."
},
{
"id": 224994,
"tgt": "What could missed periods, fatigue, dizziness with HPT negative after family planning operation indicate?",
"src": "Patient: Hi Doctor I had family plainng operation last 18month back now I miss period from 9days and iam felling tried and dizziness n I had blood test also that iam not pregnant and no thyroid everything is fine .what it s mean I only miss period in pregnancy I have three kid last one is 18 months and iam feeling symptom like pregnancy why tis happying? Doctor: hellothanks for your questionif you had family planning operation than you dont worry about pregnancymissed periods fatigue and dizziness was due to another reason also and if you have missed periods than it will be because of some hormonal changesthanks"
},
{
"id": 28505,
"tgt": "Does ureaplasma urealyticum causes infertility?",
"src": "Patient: Dear Sir/Madame, I'm writing to you because I was diagnosed with ureaplasma urealyticum abroad (routine STD testing) - no symptoms. I was prescribed seemingly strong antibiotics (looking at what other diseases it is used to treat), doxycyclin. I have learned that in some countries this is not considered an STD since it's presenton 70% of people. I also found a study online that advises against treating ureaplasma without any symptoms (because of the development of bacteria resistance), but also found articles that says that it's a very dangerous infection which can lead to infertility or joint problems. So I'm properly confused as what to do about it. Should I take the antibiotics? What about my partner? I don't want to develop antibiotics resistance either, or well I don't want the bacterias in me to do that at least. Can you please share your thoughts about this with me? Thank you, S Doctor: Hello, Ureaplasma urealyticum is a common bacteria that lives in the vagina. Usually, it does not cause any problems, and while it has been suspected to cause many different kinds of infection, it is hard to say whether this bacteria was responsible, or if it was due to another bacteria. While it is possible that it could cause an infection that would lead to infertility or many other problems, if you are not having any symptoms, such as vaginal discharge, fever, or pain, then it is not causing any damage, and there is no reason you need to treat it. So I would recommend not taking the antibiotics unless you have any symptoms that suggest an infection. Hope I have answered your query. Let me know if I can assist you further. Take care Regards, Dr Aaron Branch, General & Family Physician"
},
{
"id": 36787,
"tgt": "When will throat pain, tonsils and glands swelling subside?",
"src": "Patient: I have mono but im not tired at all. My throat hurts really bad and my tonsils and glands are very swollen. I also have mucus or snot in the back of my nose that i cant blow out. Im taking predisone and ibu and sudafed. How long should these symptoms last? Doctor: Hello and thanks for your question. Unfortunately mononucleosis symptoms can last for months. There is no real treatment for mono so the best you can do is treat the symptoms. Taking an anti-inflammatory like ibuprofen can help with the sore throat and glands. Drinking lots of water will help thin secretions. Using nasal saline in your nose may help with congestion and mucus. It is good that you are not tired. If symptoms worsen or if you get high fevers see doctor immediately. Good healthy to you and I hope you are feeling better soon."
},
{
"id": 202008,
"tgt": "Suggest medication for swollen clit",
"src": "Patient: Hi I started taking microgynon 7 days ago. And 3 days ago have rather rough sex. I took the morning after pill the next day is condom broke. And by the after noon my clit started to feel uncomftable Slight swelling. No abnormal discharge. Is this just from the sex being to rough Doctor: HITank for asking to HCMI really appreciate your concern and looking to the history given I could say that the swelling of your genital organ may be due to the fungal infection and in my opinion this need to be treated with Miconazole cream, and this would come around, take care and have a nice day."
},
{
"id": 214190,
"tgt": "What causes recurring tightness in throat with frequent burps?",
"src": "Patient: sir i was suffering from : burning sensation in stomach and near throat and i feel like something blocking in my throat with heavy burps for 3 months now i dont have burning sensation but sall bubbles getting formed in my mouth which becomes big and gets n=blast and after sometime my throat gets blocked and i get relief if burf only . i have drego d , etirox tablets Doctor: u have to improve ur food habits do1. take 2/ 3 glass of warm water in the morning2. take more water in day3. take meals at fixed hrs4. take small amount of food at a time, take more frequent meals - may be five times a day.5. no spicy/ fried/ fast/ junk/processed food.6. no milk for few days, can take curd, no uncooked salad7. avoid constipation8. use nibu pani (lemon water) 2/ 3 times a daydo for 7 days u will feel better"
},
{
"id": 140552,
"tgt": "Could my paralysis be due to gluten poisoning or allergies?",
"src": "Patient: I have MS. When I eat wheat products my symptoms are exacerbated. The numbness in my feet spreads. Could my paralysis be due to gluten poisoning/ allergies? Any recommendations onhoq to best detox my body and how long it will take to get it totally out of my system. Doctor: Hello, Gluten sensitivity may cause poor absorption of nutrients like vitamin b12 and iron. Lead to peripheral neuropathy. Also, check for other possibilities like diabetes or autoimmune disease etc. Please check your blood sugar and hemoglobin levels. Serum b 12 and folic acid may also useful. Take vit D and folic acid with b12 supplement. If symptoms not improved please consult your doctor he will examine and treat you accordingly. Hope I have answered your query. Let me know if I can assist you further. Take care Regards, Dr Penchila Prasad Kandikattu , Internal Medicine Specialist"
},
{
"id": 114170,
"tgt": "I have acidity,gaseous and back pain, please suggest treatment",
"src": "Patient: i have acidity ,gaseous problem and severe back pain with it. Doctor: Yes, there are many problems due to gastric. The simple thing which will help you is to avoid foods that you find it hard to digest. And next is the use of raw onion in your diet. It shall work wonders. Do give a try."
},
{
"id": 163794,
"tgt": "What causes rash and swelling on legs of an infant?",
"src": "Patient: my son is 19months old and for the past few days he developed a rash and his left leg become a bit swolen and he found it diffcult to walk. We took him to the doctors and they performed a blood test and the result showed that his esr levels was a bit high. what could this mean? Doctor: Hi... by what you say this could be an allergy or angioedema. in both these conditions ESR elevated.You need not worry about the raised ESR if the kid days otherwise active and playful and feeding well.Regards - Dr. Sumanth Amperayani"
},
{
"id": 158286,
"tgt": "Diagnosed with stage 2 endometrial cancer, has a blood clot, about to start chemo. Is clot a side effect ?",
"src": "Patient: Hi my sister was diagnosed with stage two endometrial cancer. Had a full hysterectomy and unfortunately they found cancer cells on her ovary from the pathology report. She is now in her final cycle of chemo and will start radiarTigon in a few weeks. Her team of drs told her this is for prevention purposes. Last week she found out she has a blood clot and is being treated for it. She is really down and her emotional state is fragile. Can you confirm that a blood clot is in fact a side effect. Every time she gets a new side effect she thinks the cancer is spread thanks for your input. Doctor: Hi,Thanks for writing in.Your sister has had a total hysterectomy with removal of ovaries. On pathological analysis there are cancer cells found in the ovary specimen. She is on her last cycle of chemotherapy and due for radiotherapy.Form what you have written, I understand there is a clot in the leg deep viens. This condition is known as deep vein thrombosis and confirmed by venous Doppler for lower limbs. This requires slow treatment with anticoagulants which are blood thinning agents.I reassure you that it is a side effect of cancer treatment and not a sign of cancer spread.God Bless your sister for a quick and complete recovery.Hope this helps"
},
{
"id": 98711,
"tgt": "What causes itchiness of the skin, without rashes, all over the body?",
"src": "Patient: I have itchy skin that I believe could be some allergic reactions but no red marks or rash until I scratch I have used various types of lotions and creams and they work for a period of about 2-3 hrs there are times I feel like I am being bitten by something and there is nothing on my skin no after effects this is my entire body not just a specific area Doctor: it can be allergy due to any thing season clothes use fexofenadine 120 MGM once a day and atarax 10 mg at night for few days may be 10 days along with local application of histocalamine"
},
{
"id": 189189,
"tgt": "Red flat lesion on left of tongue since recovering from pneumonia. Cause?",
"src": "Patient: Good Morning! I am a 40 year old mother of three children. Over the past year I contracted pneumonia and noticed that ever since I have recovered I have developed a red flat lesion on the left side my tongue . The lesion tends to correspond to how much exercise I m doing (believe it or not). However when I take at least 5 days off from exercise the lesion begins to grow. Please advise on what s going on? Is this related to the health of my lungs? I look forward to your response. Doctor: Hello there , Thanks for writing your query, Pneumonia is inflammation of lungs and it can be bacterial or viral . Tongue lesions can be a secondary oral signs of pneumonia.other reasons include stress ulcers which are long standing and painful, apthous ulcers, traumatic ulcers especially seen on sides on tongue. Red flat lesion on tongue in the elderly group people usually associated with smoking can be Erythroplakia. such lesions often show malignancy. do no panic but consult your doctor as soon as possible as such lesions should not be ignored. mean time maintain good oral hygiene.rinse your mouth with lukewarm salt water, apply topical numbing agents over the lesion i hope this helps , take care."
},
{
"id": 25827,
"tgt": "What causes pounding heart beats?",
"src": "Patient: My hearts pounds really hard against my chet to where at times i cant breathe. Also sometimes it pounds so hard that you can see it against my chest and sometimes makes me move side to side. Please respond fast i feel this is a problem that needs to be adressed. You can email me at YYYY@YYYY Doctor: Thanks for your question on Health Care Magic. I can understand your concern. Pounding of heart is known as palpitations. And most common cause for palpitations is stress and anxiety. But better to first rule out heart diseases (arrhythmia). So consult cardiologist and get done blood pressure monitoring, ecg, 2d echo and Holter monitoring (24 hour continuous monitoring of ecg). If all these are normal then no need to worry for heart diseases. If you are having stressful life then avoid stress and tension, be relax and calm. Consult psychiatrist and get done counselling sessions. Try to identify stressor in your life and start working on its solution. You may need anxiolytic drugs too. Don't worry, you will be alright. Hope I have solved your query. I will be happy to help you further. Wish you good health. Thanks."
},
{
"id": 96789,
"tgt": "What should be done if vesicular fluid flows into eyes of a doc?",
"src": "Patient: I am an ob/gyn in private practice. Today, while examining a lesion known to be herpes, some of the vesicular fluid flew into my face and at least one of my eyes. I washed out my eyes and removed my contacts, and took 1000mg valtrex. Do I have to do anything else to prevent infection? Doctor: Hi. no,there is no need for anything else ,there is no need for preventional antiherpetic drugs but in next 1-2 weeks watch for signs of herpes infection and start medication if it occurs. Regards"
},
{
"id": 219912,
"tgt": "What causes brown discharge during periods?",
"src": "Patient: To whom it may concern,I take birth control regularly and normally get my period on the last Monday of each month, almost every month. I haven't missed a pill and I take it at the exact same time each morning. The day before I was supposed to get my period (Sunday) I went snowboarding for the first time, which involves a lot of exercise. I didn't get my period on Monday and was freaking out, scared of a pregnancy. I didn't get my period until Tuesday and there was only a little red blood at first. Now my tampons have been with brown blood instead of red. If I don't have a tampon in I do not leak on a pad. Am I pregnant or what would cause this? Doctor: it could indicate just a very light period. but at the same time do pregnancy test to exclude the pregnancy."
},
{
"id": 177833,
"tgt": "What causes recurrence of typhoid in kids?",
"src": "Patient: My son is 3yrs 7 months old ,17 kg weight.He is having typhoid o as 1/80 in widal test and was detected e.coli(urine infection).He was given antibiotics(cefixime for typhoid,oflaxin for urine infection).He became ok.After 20 days again he is having fever and same report of widal is there.His L.F.T test report and tc/dc is ok.Doctor is again giving him antibiotics of typhoid.pls help as why relapse of typhoid is there. Doctor: Hi Dear welcome to the HCM,Typhoid disease is a water borne disease. usually spreads through the contaminated eatables/water.Only widal test is not the conclusive test for the diagnosis.Repeated detailed work up should help in the exact diagnosis like CBC, ESR, TYPHI DOT TEST, MALARIA ANTIGIEN,URINE ROUTINE EXAMINATION. CHEST X RAY. BLOOD CULTURE etc.This case may be also partially treated case or some other concurrent infection may also be there.hope the query is answered.thanks."
},
{
"id": 84726,
"tgt": "What could be the side effect of glucoredforte?",
"src": "Patient: Sir I am diabetic patient since 2006 (now 45 yrs) and last one year i take the glucoredforte tablet-01 no morning and evening . Request clarify whether take this tablet daily any side effect with this or required change the tablet and my sugar level is now normal Doctor: Hi,Glucored Forte is a combination of Glibenclamide and Metformin commonly prescribed in the treatment of type 2 diabetes mellitus. It improves the blood glucose levels in adults when taken along with proper diet and regular exercise. Generally it is well tolerated by most of our patients however it may cause low blood sugar level (can be adjusted by the dose), abdominal cramps, diarrhea, deficiency of vitamin B12 and a serious condition called lactic acidosis.Hope I have answered your query. Let me know if I can assist you further. Regards, Dr. Mohammed Taher Ali, General & Family Physician"
},
{
"id": 77858,
"tgt": "What could cause sharp chest pain?",
"src": "Patient: Why do I keep having sharp chest pains? I went to the doctor and had an EKG and chest x-ray, both were normal. But I got no answer to why my chest hurts. He just said anxiety. But I keep having sharp chest pains. Anywhere from left to center to right sides. When this happens I get very anxious and worried and my heart races. Sometimes the pain will also be dull like in the muscle or spasms. But this sharp chest pain scares me. Please help. Doctor: Thanks for your question on Health Care Magic. I can understand your concern. Sharp pain in chest is mostly due to heart diseases. But since you are having normal ECG, possibility of heart diseases is less. But normal ecg can show abnormality when heart is any stress. And hence get done 2d echo and stress test. If these are also normal, no need to worry for heart diseases. Stress and anxiety can cause similar kind of sharp chest pain. So better to consult psychiatrist and get done counselling sessions. Try to identify stressor in your life and start working on its solution. You may need anxiolytic drugs too. Don't worry, you will be alright. Avoid stress and tension, be relax and calm. Hope I have solved your query. I will be happy to help you further. Wish you good health. Thanks."
},
{
"id": 155844,
"tgt": "What are the symptoms of cancer?",
"src": "Patient: My best friend went to an oncologist today, with extreme white blood cell count at 21. Her symptoms are sore tounge, pain in left side of neck, fatigue, pain in lower left side of the body which gall bladder has been ruled out. The Vitiam B-12 shots are not work for her anymore. Could this be a sign of cancer? Doctor: Hi and welcome to HCmyes but this is not the first thing to think of. this is more likely to be some systemic infection and this is the first thing to evaluate. blood disorders can furthetly be evaluated by hematologist but as I said you should be optimistic and do compelte work up.,, Wish you good health. Regards"
},
{
"id": 74169,
"tgt": "What causes chest pain while on Metprolol and Succinate?",
"src": "Patient: nes my husband is passing small kidney stones everyday sometimes at a time, he is on Metoprolol 25 mg succinate, linsinopril 20mg, has some pains in his chest just like soreness all over can you give some advice are you really serious I have to pay for an answer Doctor: Hello! Can you be more elaborate about his complaints. Chest pain in his case can be due to something very common like GERD (Reflux disease) or it can indicate something to do with his heart or lungs. Since when is he having the pain ? Where exactly in the chest does he have the pain ? is it continuous ? is it mild or severe enough to disturb sleep ? Any particular thing that you have noted to aggravate or relieve his pain ? Any other chest complaints ? Does he smoke ? Looking forward for an elaborate history. Since you mention soreness all over , it seems more likely to be reflux disease in which case he will need medications to control that and needs to avoid spice n caffeine in food, head end elevation during sleep Do consult a physician in person too."
},
{
"id": 47628,
"tgt": "How to treat Renal haemorrhage when not responding to Amoxicillin and Macrodantin?",
"src": "Patient: I am a 66 year old female born with one kidney but I ve never had any problems with it prior to August 17th, 2013. I had open heart surgery scheduled and had to start taking 3 medications including Coumadin. I began to have a lot of blood in my urine and I developed a UTI. I was given some strong antibiotics but had renal hemorhage 2 days later which required a four day hospitalization. The surgery was delayed a month for my kidney to heal. On September 17th 2013 I had successful open heart surgery (valve repaired/valve replaced/epicore maze) and I have been ok until about 2 months ago when I started having UTIs again. Tests have shown leukocites esterase, nitrates, protein and blood. My urine is brown right now and burns. They are treating me with amoxicillan and then macrodantin but I have not responded to either of them. Due to the hemorhage after the stronger meds were prescribed, they feel those two are the strongest things I can take but it is not working. Do you have any ideas? Doctor: Hello and welcome to HCM.As an Urologist, i must advise you, that before starting treatment for UTI in future, always get an urine culture done. It's more sensitive than nitrates in urine. The other reason being,it tells us,which is the best antibiotic to use.Don't worry about being born with one kidney, as it's seen in many people.Your doctor may change the antibiotic, to a cephalosporin soon. In active bleeding, your aspirin/ Clopidrgel/ Coumadin, would've been stopped by your cardiologist. Your prothrombin time,INR,and platelets must be checked periodically. If you need an expert opinion,you may contact me directly.Dr.Matthew J. Mangat."
},
{
"id": 226383,
"tgt": "Taken Postinor2 and other pill after sex. Does taking medication late affect its effectiveness?",
"src": "Patient: I took Postinor2 less than 24 hours after sex and the other pill 13 hours after that. I know I need to eat it in 12 hours time but something happened and I took it one hour late. Does that affects the effectiveness of Postinor? Do I need to get another set of pills and take two right away to reduce the risk of getting pregnant? Doctor: . Hi thanks for your question. You have to take postinor 2. Postinor 2 can help to prevent you from getting pregnant, after unprotected sex, The first tablet has to be taken within 72 hours (3 days) of unprotected sex. The tablets are more effective if they are taken as soon as possible after unprotected sex. You need to take 2nd tablet 12 hours (but no more than 16hours) after first. So relax you have taken the medicine well within the prescribed limit. Hope I answered your question."
},
{
"id": 126297,
"tgt": "How can pain in the right hip post injury be treated?",
"src": "Patient: Hi I missed a step and slipped down four stairs hitting bottom. Now my right side hip hurts as if someone took a bat to it. I can t lift my leg past my ankle height or turn my leg a certain way anymore. It hurts especially when I lay down and try to wake up. Like something s broken or out of place. Doctor: hisince you fell down to a bottom and got hurted it might be a hip fracture. if there is any reduction of movements it's better to take x-ray of both the hips and consult a orthopedician once. just apply the ice packs over the hip area and under rest would be better.thank you."
},
{
"id": 10497,
"tgt": "Why have I started to lose hair with the texture growing rougher?",
"src": "Patient: Hi, I am deepika, I have started losing hair like anything & even the texture has become rough. i noticed the change when I relocated to new apartment. Underground water doesn't suits. As soon as I moved for a fortnight to cosins place, I regained the texture to a certain extent. Now situation is again the same. What to do? Doctor: Hello and Welcome to \u2018Ask A Doctor\u2019 service. I have reviewed your query and here is my advice. As per your case history you are having telogen effluvium. My treatment advice is - 1. Use a good herbal hair oil and shampoo for routine use. 2. Take good nutritious diet full of green leafy vegetables and milk. 3. Take an iron supplement and vitamin B12 supplement once daily for 3 months. If problem persists then consult a dermatologist. Hope I have answered your query. Let me know if I can assist you further."
},
{
"id": 146496,
"tgt": "Suggest treatment for leg weakness with limping in leg",
"src": "Patient: I am a 61 year old male I believe to be in excellent health. The only surgeries I have had is shoulder surgery. For the past several months I ve noticed a gradual limp with my left leg. It s fine in the morning. As I walk on it, it starts to become weak feeling. Left leg feels heavy, almost weighted down. When I lay on my back and raise my left leg straight out, it is weaker than my right leg. When walking rapidly, left leg seems to almost lose control and wants to turn outward. I have had minor chronic lower pain for many years, no surgeries so far (too many auto crashes). Rarely I have minor left knee and hip pain. I attribute this to regular age related pains. When standing to put on pants I can t raise my left leg as high as my right leg without minor discomfort. Help Doctor: Hi, I had gone through your question and understand your concerns. I think from your description of your problem, the probable diagnosis is lumbar herniated disc disease. I suggest you to consult a Neurologist and to discus the possibility of imaging study of your lumbar column (MR scan). You need plain radiographs of lumbar column as we'll, since there been trauma, to evaluate any bone damage. Th treatment depends on imaging studies results. Hope this answers your question. If you have additional questions or follow up questions then please do not hesitate in writing to us. I will be happy to answer your questions."
},
{
"id": 224554,
"tgt": "How can I postpone my period with yaz?",
"src": "Patient: I want to postpone my period, which is regular (28-29days), I also use Yaz for about 10 months. I don't want to stop using Yaz, so I wanted to know can I use Primolut Nor with Yaz. If answer is \"no\", how can I postpone my period with Yaz, if it's that possible at all? Doctor: Hello!Thanks for your query.Yes you can use YAZ to postpone your periods .Just skipping the placebo pills and start the new pack .All the best"
},
{
"id": 219224,
"tgt": "What was the small thing in gestational sac?",
"src": "Patient: WELL I HAD A PROBLEM I DIDN T KNOW I WAS PREGNANT APPARENTLY I THINK I M STILL PREGNANT BUT MY HCG LEVEL IS LOW NOW SEEING THAT I DIDN T KNOW I THOUGHT I HAD MY PERIOD I GOT LOTS OF CRAMPS THEN SOMETHING LIKE A BIG CLOTH PASSED AN I VE BEEN BLEEDING HEAVY AT FIRST THEN IT WENT LIGHT BUT TOOK A TEST AND IT SAYS I M PREGNANT WENT TO GET AN U/S THEY SAW SOMETHING SMALL IN A GESTATIONAL SAC BUT COULD NOT FULLY RECONISE IT SO I HAVE TO GO BACK CAN U TELL ME WHAT HAPPENED THER Doctor: Hello.It seems you have had an incomplete spontaneous abortion. What you did see pass out 'as a cloth' might well be partly the products of conception and partly blood clots. The unrecognisable mass they saw in the ultrasound might be the leftover products of conception. It needs to be evacuated completely as it might leads to avoidable complications later on, hence I believe you have been called back for the same.Hope this helps.Regards,Dr Geeta"
},
{
"id": 147283,
"tgt": "What can i do for a nerve pain?",
"src": "Patient: i got blood drawn three days ago and the nurse hit a nerve. it felt like i was being electrocuted. i thought the pain would go away but it hasn't. i cant lift anything with my right arm and i still feel like i am being electrocuted throughout my whole arm. what can i do Doctor: HIThank for asking to HCMI really appreciate your concern looking to the history given here I could say that this might not be a never damage but because of the very sharp needle edge this could cause trauma to the soft tissue and trauma to tender tissue induce the pain as you had, but this is nothing to worry, take care and have a nice day."
},
{
"id": 104339,
"tgt": "Body pain, salty taste in mouth. Taking amoxicillin for cold and cough, nasal spray, no relief. Cure for symptoms?",
"src": "Patient: I have recently been on a short course of amoxicillin 500mg 3 x day for 5 days for a very bad cough , cold, infection. The course has stopped but I am left still feeling full of aches and pains but the worse thing is that I cannot taste anything other than a salty taste in my mouth and salty mucus . I went to my doctors last week who prescribed some nasal spray for allergies!? Its not working anyhow, whats wrong with me please? Doctor: ALLERGIES DONT RESPOND TO ANTIBIOTICS YOU NEED ANTIALLERGICS tAKE METROGYL 20 MG BD FOR 7 DAYS TAKE MONTAIR FX BD SYP VENTROLIN 1/2 TSF BD SYP TOSSEX 1/2 TSF NIGHT TAKE ANTIALLERGICS FOR 3 WK METROGYL FOR 7 DAYS ADD ANTACID SYP FOR TDS TO END TASTE YOU DESCRIBED"
},
{
"id": 54006,
"tgt": "Suggest treatment for enlargement in liver",
"src": "Patient: My son who is 13 years old has been diagnosed with an enlarged liver. The hepatitis tests have come back negative and the one mono test is negative. We are waiting on one more Mono test to come back. What else would cause a viral infection that enlarges the liver? Doctor: Hi welcome to health care magic... The enlarged liver can be from viral hepatitis infection as well as from fatty liver or from hemolytic anemia like condition... For viral hepatitis work up done with... -sgpt and sgot estimation-hbsag detection -anti hcv detection-serum bilirubinFor hemolytic anemia work up done with... -Peripheral smear examination -retic count -LDH estimation If needed hb electrophoresis done to rule out hemoglobinopathy if present... Like thalasemia also can lead this... According to cause further treatment can be guided.... Take careAdvise : physician consultation for examination..."
},
{
"id": 211441,
"tgt": "Are Kallman's syndrome and Borderline Personality Disorder interrelated?",
"src": "Patient: Hi , My 30 yr old daughter has Kallman's Syndrome and has never really looked her age or acted her age , she is showing signs of depression. Is there a relationship between Kallmans and Borderline Personality Disorder? She also doesn't think she has a problem , its everyone else ......this includes jobs, friends etc. Doctor: Welcome on HM These two conditions are not interrelated clinically ,not knowing that she had any problem is because of both the conditions .In my hospital i will send her for through clinical psychiatric and grnetic evaluation before giving further suggestionThanks for seeking opinion HMDr Lal Psychiatrist"
},
{
"id": 1342,
"tgt": "What could be the reason for not getting pregnant?",
"src": "Patient: My wife had misscarriages twice & under gone DNC . Doctor advised us restrict intercourse for six months & we followed as Doctor said. After six months we started intercourse & already 1 year have been elapsed but still my wife didnot become pregnant . What shall we do to get pregnant ? plz advise me. Doctor: Hi , How are you doing ?Let me give a positive note to begin with. If she has got pregnant once , no matter what the result was, she has very good chance of getting pregnant.Kindly see if the following checklist of investigations is doneHusband- Semen analysis at least 2- done 4 weeks apart preferably in an infertility center ( examination done by andrologist), Blood sugarsWife - Blood Sugars, Thyroid profile Ultrasound scans to evaluate Uterus , Ovaries & adnexa Confirm tubal patency- most important- Tube is the connection between uterus & ovaries( the bridge where sperm meets the egg & forms a baby) this is size of hair follicle & cannot be seen on scans. So check with HSG( Xray with dye), or SSG ( Scan with dye) or LaparoscopyIf all this done & found normal , nothing can stop a pregnancy with God's willHope I have cleared your query, do write back if any more quriesAll the bestDr.Balakrishnan"
},
{
"id": 12882,
"tgt": "What causes purple and yellow rashes on my legs?",
"src": "Patient: Hi,i am a 20 year old female i am 5'2 127.lbs i have ashma and i have had bone corrective surgerys.i also have nerofibromatosis sorry dont kno if i spelled it righ. lol anyways lately i have been noticing in the mornings wen getting ready for my day that i have these purplish/yellow bruises all over both my legs.they start in my hip/thigh area and end just bellow my knees like almost my calf area. and i have done nothing to cause them. can you help me Doctor: Hi, I would recommend you to go for certain lab tests to find out its cause. I would recommend you to go for complete blood count and coagulation profile. Hope I have answered your query. Let me know if I can assist you further. Regards, Dr. Asmeet Kaur Sawhney, Dermatologist"
},
{
"id": 151846,
"tgt": "What does brain herniation mean and can it be detected through MRI ?",
"src": "Patient: A year later i still have... -Tingling in face , tongue,hands. Severe pains in my neck,head which are mostly rite on top or at the base of my skull.Feeling off balance,feeling of burning sensation on tongue and in head, neck pain , lower back pain ,facial pain,vision disturbances.Im in pain all the time, not just when I stand up I dont think but it does feel like pulling sensation on my head. have seen 3 nuerologist a infecious disease, cancer dr,autoimmune dr and stayed in the hospital last year test done were mri / mra of brain ,neck,lower spine,spinal tap,Ct of brain and mri 3 days after the spinal tap. Bloodwork showed b12 deficiency and vit d ( LAST YEAR THEY ALSO FOUND ABNORMAL ANTIBODIES IN MY BLOOD ), im miserable and have a 3 month old.I am only 21 years old.I have no energy and always feel ill.I had a compound fracture above my elbow ten years ago which I don t think has anything to do with it, but it was a major surgery. I have not had any test besides a CTscan of brain and a brain MRA ( not MRI )for about 6 months because drs. dismissed me and said it was stress or migraines. Is there a chance of brain herniation or anything life threatening after almost 10 months with nothing found on tests or the spinal tap? Could this be a CSF leak...or MS? Chiari? would that not show on those mris?What else could it be? Doctor: Hi,Magan, Thanks for query, As all reports are normal you do not worry, It seems that you are having anxiety neurosis. There is no question of having brain hernia. Forget all and develop self confidence. Take B1,B6,B12,medicines. If anxiety persists consult psychiatrist. ok and bye."
},
{
"id": 36938,
"tgt": "Suggest treatment for recurrent mild fever, cold and fatigue",
"src": "Patient: I've been running a fever for a year when it started I'd get sick flu like for a few days then it go away for a week or so then come back I've been to the dr had ain't meds they can't find anything worng with me and it's just getting worse I have a Good day ever now and then but I stay sick it never gets bad fever never gets over 99.5 I stay cold and some days I don't have the engery to get out of bed it fill just like the flu like I comming down with something I've lost 40lb look like a dead man walking I don't eat plz if u can help plz Doctor: HelloYou are having for ONE year and off & on , gotten worse , weak , unable to walk , feeling cold .You didn't mention about other tests and symptoms , as these things are important for diagnosis . Such type of long standing fever with such type of symptoms may be due to these possibilities , these include.1 Tuberculosis , diagnosis can be confirmed by tuberculin test , blood for CBC , ES R and sputum examination for AFB ( if present).2 Typhoid , for confirmation get in Widal and serology test.3 Drug induced fever , if you are taking antibiotics for prolong time.However , there are so many other possibilities of such fever , as HIV , anemia , urinary tract infection .In my opinion consult a physician and get his opinion ( 2nd one ).Jaundice is another such common cause of fever and weakness.But physical examination , clinical history is also important."
},
{
"id": 66880,
"tgt": "What causes small pea sized lump in thigh?",
"src": "Patient: I have a small pea-sized ( or small ) lump, located inside my left thigh, and under the skin. I squeezed it a bit, to find out that it s hard. I can t move it around, so it appears in a completely knew spot, but discovering it, it urks me, to know what the heck it is, and if it s dangerous or not. Doctor: not to worry much as this is benign condition like sebaceous cyst; but could be a lymph node and in that case you have to know the cause of it!if really worries you can go for an FNAC test for confirmationall the best!"
},
{
"id": 9193,
"tgt": "How to treat dry facial skin?",
"src": "Patient: skin,im 26 and married from chennai....i have normal skin as my dermatologist said......for the past two months i have noticed dry skin or tightness around the mouth and chin area and in someother body parts as well....when i checked with the doctor she said it is caused my sun senstivity....and the other parts are okay now excepting my face...also i sweat if i use anything on my face....and if i use any rich body butters on my body i sweat it when there is no current...my doc has suggested nmfe lotion for my face but m a person who cannot stick to one fragrance so i alternate it with my olay moisturiser....m also obsessed with stocking up moisturisers in the quest of finding the right one for me.....also i would like to know when is the right age to use anti ageing and night creams.whether they should be oil based or water based? please help! Doctor: Hi Dear,Welcome to HCM.Understanding your concern. As per your query you have symptoms of dry facial skin which is due to abnormal skin reaction to sunlight specially to ultraviolet radiation and it is due to hormonal changes sometimes. Need not to worry about it. I would suggest you to apply cold compresses to affected area and consult dermatologist for proper examination. Doctor may recommend phototherapy to desensitize the skin, may prescribe drugs like azathioprine to suppress the immune system. Drink plenty of fluids and take diet rich in multivitamins. For now avoid sun exposure , use PABA-free sunscreens that protect against UVA with sun protection factor of 30 and do not use any other skin product.Hope your concern has been resolved.Get Well Soon.Best Wishes,Dr. Harry Maheshwari"
},
{
"id": 140571,
"tgt": "What causes lower back pain radiating to left leg?",
"src": "Patient: I am a software proffessional. I am suffering from Back pain from last 1.5 years. It started suddenly. One day I got a sudden pain in my stomach and back at a time. That day onwards I am getting lower back pain. If I bend for a short time also I am getting pain. This pain spreads to the left leg if I keep on doing the work. I can not lift weights also. Sometimes If I sneeze or cough also I am getting back pain. Some time I get middle back pain also. Please let me how you diagonize this and what you suggest? which doctor do I consult and can I get rid of this permanantly? Doctor: Hello, Back pain radiating in a limb suggests compression of one of the nerve roots exiting the spinal column. The most common cause for the compression is disc herniation, displacement of one of the cartilage discs located between the vertebrae. Chronic changes to the vertebrae with formation of osteophytes, bony spurs, compression the nerve root are another cause. The best means to diagnose it is a neurological physical exam and a lumbar MRI, I would start with a neurological consult. Most cases improve gradually, but it may be a recurring issue, as our columns continue to age with the passing of the years often it is difficult to get rid of it permanently. Physical therapy is important to prevent recurrences. Hope I have answered your query. Let me know if I can assist you further. Take care Regards, Dr Olsi Taka, Neurologist"
},
{
"id": 109584,
"tgt": "What causes pain in lower back with bloating and loss of appetite?",
"src": "Patient: I've been having sever pain in my back stomach side and neck I'm soo bloated and stomach feels very heavy I've been getting pains in my vagina aswel tingles down my face and head also I'm so worried its cancer or something I've lost my appitite aswel and breathing has gotten lighter Doctor: Hello. Thanks for writing to us. The bloating and loss of appetite are not related to the back pain that you have. Gas and indigestion can cause these symptoms.I hope this information has been both informative and helpful for you. Regards, Dr. Praveen Tayal . For future query, you can directly approach me through my profile URL http://bit.ly/Dr-Praveen-Tayal drtayal72@gmail.com"
},
{
"id": 88202,
"tgt": "What causes pain in upper abdomen radiating to mid-back?",
"src": "Patient: Hi,Today was the third time that I have experienced a pain in upper abdomen, (below rib cage around diaphragm area), that seems to radiate to my mid-back. I get really nauseous and have an almost \"outer body experience\" when it happens, and I feel like I'm going to throw up, but I never do. I feel clammy and weak, and then it goes away after about 20 minutes or so. Should I go to see a doctor or just wait to see if it happens again? What are the different things it could be?Thank you. Doctor: Hi.Thanks for your query and an elucidate history.You have a classical history and you should not wait, You should see a General Surgeon preferably with Gastroenterology specialty to get a proper investigations and diagnosis.My probable diagnosis would be as follows as per the history you have provided:-Gastric Ulcer-Pancreatitis-Volvulus of the stomach-Rarely some problem of the bile ducts, aorta ans so on.I would advise you the following in such a case:X-ray of the abdomen in standing and lying down position.UltrasonographyUpper GI EndoscopyCT scan of the abdomen to confirm the findings. The treatment will depend upon the diagnosis on the basis of clinical history, physical examination, and investigations."
},
{
"id": 120316,
"tgt": "What causes pain in shoulder blades shifting to the left arm?",
"src": "Patient: I have had a pain in my back left side near shoulder blades. Thought I slept wrong because pain was in my neck on left side also.. Now after about 1 1/2 weeks pain has moved to muscles in my left arm and at times arm goes numb down to fingers.. Feel ok in chest area but concerned. Doctor: Hello,I read carefully your query and understand your concern. The symptoms seem to be related to a pinched nerve in the stomach. I suggest using anti inflammatory medications such as Acetaminophen to relieve the pain. I also suggest to maintain the shoulder immobilized. If the symptoms continue, you should consult your neurologist. Hope my answer was helpful.If you have further queries feel free to contact me again.Kind regards! Dr.Dorina Gurabardhi General &Family Physician"
},
{
"id": 195341,
"tgt": "What causes lumps on the foreskin of the penis?",
"src": "Patient: Hi, im 19 on the ridge of my penis forskin ( right upnder the head) i have 3 skin ( a bit transparant ) flat slightly raised but barely bumps or patches im not sure what to call them I recently noticed them acouple weeks ago and have been researching what they could be but havent found anything alike they were ive only had oral sex ( blow job) and ive had unprotected sex once from what ive read it might be masterbation rash or something I cant really tell, they are only really noticeable when falcid when erect you wwouldnt be able to notice them Doctor: Hello and Welcome to \u2018Ask A Doctor\u2019 service. I have reviewed your query and here is my advice. As per your description it seems like something called genital wart, nothing much to worry as it is benign (non cancerous) and can be easily cured. You can get it removed either by cautery or laser. Consult a dermatologist and he will explain various treatment options. Always stick on to safe sexual practice. Wishing you good health."
},
{
"id": 24253,
"tgt": "What is the benefit of 2d echo test?",
"src": "Patient: my doctor has told me to go for 2d echo test and PFT, i can hear my own heartbeat, i have a bloating abdomen, fatigue. i am 34 years female. 5 feet 2' is my height, with no family history of heart problems. please suggest me that how is this test beneficial for me?thanks.... Doctor: Hello , 2D echocardiography is a sonography of your heart . It sees the anatomy of the heart and you van see the heart beating in real time . So one can get information anatomy bout any structural abnormality, the pumping of the heart , valves and there function .The great arteries and veins near the heart . Basically it helps us identify the structural defect of the heart which may be the cause of your symptoms. Regards"
},
{
"id": 789,
"tgt": "Can PUJ obstruction cause difficulty in conception?",
"src": "Patient: hi i am 25 years of age weight around 79 kg i had puj obstruction which was diagnosed 8 years before and i went to a proper treatment of open surgery with stent.i am now married and want to have baby does puj will lead to any abnormality in getting pregnant or no pregnancy at all. Doctor: Hi,I understand your concerns.Following is my reply:1)\u00a0\u00a0\u00a0\u00a0\u00a0PUJ obstruction is unlikely to cause difficulty in conceptionYou can contact me anytime directly to ask question by pasting following link in your browser:XXXX"
},
{
"id": 14903,
"tgt": "A cut in right pinky knuckle that is spreading like a rash",
"src": "Patient: I've gotten a cut in my right pinky knuckle and I'm not sure how I got it. Now it is a day later and it has a rash and is spreading down my pinky and across my hand. Do you know what it is or what is causing it? The spot where the rash/cut is, is very dry and cracking and painful to touch. Doctor: Hi ,\u00a0Thanks for the query.\u00a0I completely understand your concern and I hope I am able to help you for the same.\u00a0Looks like you have developed\u00a0an infection at the site of previous trauma.\u00a0We can try conservative treatment first but if the condition continues to worsen , then you might need to consult a physician.\u00a0Avoid touching it too much.\u00a0Avoid using any kind of cold dreams or oils, as it might increase the lesion\u00a0.You can start using any topical antibiotic/antiseptic\u00a0cream(like mupirocin cream) over the lesion for 2-3 days\u00a0.Take some pain killers along for relieving the associated pain and discomfort.\u00a0Keep\u00a0hand elevated esp at night in order to prevent any swelling.\u00a0Oral antibiotics are required if the cellulitis increases, which a dermatologist / physician can examine and suggest\u00a0.Take care,\u00a0Wishing you speedy recovery,\u00a0Dr Geetika Paul\u00a0MD ( Dermatology)"
},
{
"id": 175473,
"tgt": "Need to worry about premature child s intestine?",
"src": "Patient: Our Extreemly Premature son (born 17 weeks early) had bowel reconstructive surgery and as a result his appendix and Ileocecal valve were removed leaving him with 50 CM of Small Intestine. All Large Intestine still intact. I keep reading how some have problems and others not. His dr s do not seem concerned because he has good intestinal length. Should I be worried? Doctor: Hi.. it will depend on how much he will be cope up with as he starts growing up. But as of now, if he is gaining weight well and not having vomiting or diarrhea, you need not worry. Future prognosis will based on his growth and development.regards - Dr. Sumanth"
},
{
"id": 91873,
"tgt": "Experiencing stomach pain and dizziness",
"src": "Patient: Hi there im 23 yrs old and it s been a week iv had really bad stomach pains like period pain plus I feel dizzy. I haven t been throwing up or have toilet issues. I had my period last week so im not due for it but this pain In my stomach is very unusual iv never had this problem I can t sleep properly too. Doctor: Hi.You say the pain is like period pains and you had period last week.No vomiting and toilet issues.IT is possible that a clot or so is retained in the uterus causing you this type of pain. An ultrasonography and a Gyne opinion should be sought immediately."
},
{
"id": 170128,
"tgt": "What causes red dot above lips?",
"src": "Patient: My 3 yr old son has a small red flat dot above his lip. It has no drainage or swelling, and has remained the same size (like a pinprick). It doesn t go away. I have tried OTC cortisone and neosporin. If I put vasoline, it looks better but doesn t completely go away. It is a perfect circle and flat. He recently went through a round of antibiotics for an infection and the dot remained. It doesn t hurt him but my pedi and I don t know what it is. He has no significant medical history. I am looking for a pedi-dermatologist in my area. My pedi doesn t believe it s anything to be concerned about. My son does have sensitive skin and a hx of eczema - last bout was 2 yrs ago. He tends to wipe his and rub his mouth a lot. Doctor: Hi...Thank you for consulting in Health Care magic. Skin conditions are best diagnosed only after seeing directly. I suggest you to upload photographs of the same on this website, so that I can guide you scientifically. Hope my answer was helpful for you. I am happy to help any time. Further clarifications and consultations on Health care magic are welcome. If you do not have any clarifications, you can close the discussion and rate the answer. Wish your kid good health.Dr. Sumanth MBBS., DCH., DNB (Paed).,"
},
{
"id": 177156,
"tgt": "Suggest treatment for cold sweat in a baby",
"src": "Patient: Hello, My name is tomi. I have a 1year old daughter who has been in a cold sweat for a couple days. Her appetite is iffy. she feels cold to the touch. please help me it scars me so much! my monitor has been broken so there s no way to take her temp! please help. Doctor: cold sweats in a child can be seen during fever defervescence or if the child has been overclothed. If the child has been clothed properly, child might be having fever. If the child has any other symptom like refusal to feed, lethargy or vomiting or rashes, he might require antibiotic treatment and sepsis screening."
},
{
"id": 124460,
"tgt": "Suggest treatment for swelling in feet in a paraplegic patient",
"src": "Patient: My father has been a paraplegic since the early 80 s. His feet always swell during the summer but it is worse this summer. They get very swollen and look almost purple. I know that there are many causes (diet, his condition, etc) but what can he do to keep the swelling down to a min? Would compression socks help? Doctor: Hello, First of all sorry to hear about your father's medical condition. Let's get to point. As he is paraplegic and there is a presence of swelling in his feet I would definitely appreciate your idea of using the stocking. Stocking provides great help to avoid the swelling. It should be used on a regular basis. Now to stretch a little more, as your father will be ageing by now, his cardiac condition might have become weak and this swelling is a cause of loss of venous return, which is common in most paraplegic cases. I will advise to look for a cardiac physician and also a nephrologist to rule out some basic parameters. Also coming to the therapy part, doing exercises in the mat, like rolling in bed should help the body move and avoid allowing it to become weak. Next is to focus on the upper limbs on both sides as it is important to have a proper blood circulation to happen. for the lower limbs, one needs to be assisted by passive Range of motion exercises which will help him about a lot of swelling. Also, the upper limb strengthening exercises will make his heart stronger and improve the venous return. Hope I have answered your query. Let me know if I can assist you further. Take care Regards, Jay Indravadan Patel, Physical Therapist or Physiotherapist"
},
{
"id": 196384,
"tgt": "What causes swelling on one side of penis?",
"src": "Patient: My girlfriend and I had a lot of intimate time yesterday, she used her hand on me twice (once in the shower, once out, both times very firm and rough and also dry) and once through protected sex today I have swelling on one side of my penis. No pain, no trouble going to the bath room. What is this? And is there anything I need to do to prevent it, or help it go away? Doctor: HiGREETINGS I can understand your situation. Is there any bleeding ? It is very difficult to comment without seeing pictures. I suggest you to consult your doctor. Hope you are convinced. Regards"
},
{
"id": 156497,
"tgt": "What does CAT scan showing enlarged lymph node in back mean?",
"src": "Patient: My husband was recently diagnoised with Testicular Cancer of the rt testi and had it removed. The CAT scan however shows that he has an enlarged lymph node in his back she said 45 cm and that the liver has some spots that were inconclusive. What does this mean about his stage and how does the liver come into play here? Doctor: You have not mentioned the histopathological subtype of the testicular tumor. Most common are the germ cell tumors with two subtypes seminomatous and non-seminomatous germ cell tumors.Apart from lymph nodes in back (retroperitoneum), serum tumor marker levels ( including serum alpha-fetoprotein, serum beta-HCG and serum LDH ) have to be taken into account for treatment decision and response monitoring.If only lymph nodes are there, it is stage 2 and if liver involvement is also there it is stage 3. Good thing is that Germ cell testicular tumors have such good prognosis (outcome) that there is NO stage 4. Please discuss it with your oncologist.If you can provide the above mentioned information, would be more than happy to comment on treatment options as well."
},
{
"id": 202392,
"tgt": "Found problems in erection",
"src": "Patient: hi i am vicky my age is 29 yrs weight 79 kg my lipids are cholestrol 265 tg 233 ldl 160 hdl 47 and my bp is always 100/140 but i have no headache i found problem in erection also i have very poor erection kindly suggest the right way for erection and bp problem Doctor: HIThank for asking to HCMI really appreciate your concern, looking to the history given here I could say that the erection actually stimulated through the cortex area in the brain and this depends upon the certain stimuli like visual, touch, thinking, that is directly related with the sex, and this stimuli greatly affected by the anxiety and depression which may cause the erection problem and this is not the disease, that can be treated, take care and have a nice day."
},
{
"id": 160654,
"tgt": "Is it normal for a child to have diarrhea and vomiting while suffering from fever?",
"src": "Patient: My son is 4 and woke up this morning with diarrhea. He didn t have much of an appetite and was sleepy and felt slightly warm and then this afternoon he began vomiting as well. His temperature has been around 104 and only gets down to 102.5 or so with the Ibprophin. How do I make sure he isn t getting dehydrated? When is his fever too high ? He s tired but still verbal and somewhat alert when awake. He has been sipping water as well as pedialite. He has ate a few crackers. He is currently sleeping, has been recently medicated (Ibprophin), and is running a fever of 102.5 (taken under his tongue). When do I seek medical attention? Doctor: Hi,If he is active in between fever and taking feeds, you need not worry now. Just give ibuprofen every 8 hours and give frequent small sips of ORS / pedialyte to avoid dehydration. Also give a soft diet in frequent but small amount. If he not excessively thirsty, passing urine as usual and eyes are not sunken- there is no dehydration. Do tepid sponging if temperature doesn't come below 100 after 1 hour of ibuprofenIf fever is not coming down after 48 hours, he becomes lethargic and not taking orally, or signs of dehydration persists despite giving fluids- you need to take him to doctor, as this may need antibiotics & fluid therapy.Hope I have answered your question. Let me know if I can assist you further. Regards, Dr. Muhammed Aslam T. K., Pediatrician"
},
{
"id": 148861,
"tgt": "Internal fever with stomach pain. Partial paralysis in mouth. What do you suggest?",
"src": "Patient: for 15 DAYS my mother feels internal fever, i mean the kind of fever she do feels but the thermometer does not show any temperature increase. It weakens her body and she cannot walk herself without others support. Along with these she suffers from stomach pain,Unable to eat food due bitter taste, gone partial paralysis in mouth, right leg is getting weak, Reports shows: Mri of brain shows normal, bilurubbin=2.1, Urine test reports: Ecoli=0000,pus cells:8-10,RBC:2-3. what is the name of this fever and what are the causes? Doctor: HelloThanks for the queryThere is nothing called as internal fever, most probably either the temperature is not being recorded properly or the thermometer is spoilt.It looks like your mother has multiple medical problems which needs to be taken care of in a multi speciality hospital.Here are my suggestions to you1. Approach a hospital with gastro enterology and neurology as a speciality2. If MRI is normal then the next step would be to get a nerve conduction test to check for weakeness3. Urine report is indicative of an urinary tract infection which could be causing the stomach pain and vomiting, please send a urine culture and take a physician opinion to start antibioitcsI hope I was of helpRegards"
},
{
"id": 39381,
"tgt": "Is viral fever fatal enough to cause death?",
"src": "Patient: Hai Doctor, We are in a village in tamilnadu my friends wife age35 was diagonised viral fever and was hospitalised,after few days was taken to a specialitily hospital and treated for the sama mean will she had her periods after beging in hospital for 5 days she did not recover and passed away will viralfeaver be that fatal? Doctor: Thanks for contacting HCM. Usually viral fevers are self limiting but in patients with low immunity they can be life threatening. Also some viral fevers are also very severe type where death do occurs as in dengue haemorrhagic fever and ebola virus infections. What you described about your friend's wife appears to be a case of Dengue haemorrhagic fever where blood clotting is impaired due fall in blood platelet levels and patient starts to bleed spontaneously from different sites and unfortunately in your friend's wife's case she had periods at that time and she probably died of excess bleeding. Feeling sorry for your friend and his poor wife. Regards"
},
{
"id": 149798,
"tgt": "Vagal nerve damage, constant pain, receives blocks every week. Any way it can be repaired ?",
"src": "Patient: After a hysterectomy several years ago, a friend now has vagal nerve damage, resulting in numerous health issues. First there was gastorporesis and she underwent the insertion of the gastric stimulator. For the constant pain, she receives blocks every 4-6 weeks, but with the last 2, has aspirated, so now is on oxygen . Within the past year, she developed type 2 diabetes after steroid treatment resulted in weight gain. Central line for the numerous infections. Obviously, her life is now quite frustrating and very depressing for her. Is there any way the vagal nerve can be repaired? Thank you, A Friend Doctor: Hi, there is no relation between the hysterectomy, and vagal nerve damage. the nerve damage could be due to the hiatus hernia, or some sort of pressure on the vagal nerve. You may need to have vagatomy, for relief. Diabetis must be strictly under controle for any process to be done . Thank you."
},
{
"id": 156611,
"tgt": "What causes lump on armpit, heavy breast with family history of ovarian cancer?",
"src": "Patient: Im 55 yr old female I have a lump in my left armpit ...not painful and my left breast feels heavy and tight near my armpit. I have not had a period for 7 mths. My mom died 2 yrs ago ovarian cancer. Grandma,Father and 3 uncles all had cancer. Should I be concerned? Doctor: Yes this lump and heaviness in breast needs medical attention.You need to have clinical breast examination, a mammogram and a fine needle aspiration for cytological examination of the lump.Please see your doctor at the earliest."
},
{
"id": 49309,
"tgt": "What does ultrasound post left nephrectomy indicate if showed mild compensatory hypertrophy of right kidney, mild prominence of right pelvicalyceal system?",
"src": "Patient: Hi female with one kidney (left was removed before 6 yrs) i made US : post left nephrectomy status .. Mild compensatory hypertrophy of right kidney .. Mild prominence of right pelvicalyceal system.. However normal ureteric jet noted from right vesico ureteric junction.. Significant post void residual urine .. Correlate clinically Doctor: HI. The kidney condition shown on a scan is perfectly normal finding on nephrectomy on one side. What bothers me is post-void residue in the bladder which is significant. Your Doctor who operated you will be better to be contacted for this."
},
{
"id": 156441,
"tgt": "How to treat the pain while passing urine for a person having sqamous carcinoma?",
"src": "Patient: i am a 55 years old female with symptoms of pain in lower abdomen,loss of appetite and freqent urine pass and post menopausal white discharge for last five months.Recent biopsy report indicates endocervical growth POORLY DIFFERENTIATED sqamous carcinoma.DIAGOSED for ||B stage cancer.first cycle of chemotherapy givenon 19 and 20th december 2013.Frequency of urine pass is reduced but pain still persists.there is no change in abdomen pain and white discharge.kindly guide me further in the matter.Regards Doctor: Treatment of choice for stage II B cervical cancer is concurrent chemo-radiation therapy with external beam radiation therapy followed by brachytherapy boost and not chemotherapy alone.The pain and discharge will settle with treatment as the tumor regresses in size. Meanwhile I suggest that you take analgesics for pain relief.Please discuss it with your treating oncologist."
},
{
"id": 7330,
"tgt": "I am taking fertyl 50 and having PCO will I be able to conceive ?",
"src": "Patient: i had unprotected sex for 1 year now ive had PCO now its treated...can i still get pregnant?? am i infertile?? i ve been taking restor f for months now and fertyl 50 for 4 days....we really want to have a child...thanks for any answer.... Doctor: hello, since you are already treated for pcod you can conceive now. continue the the drugs given by your doctor. try to maintain sexual relatioship during your fertile period.it will maximise your chances of pregnancy. take care"
},
{
"id": 213991,
"tgt": "I hear imaginary voices and can see some shadows that doesn't exist. Am I suffering from any mental disorder ?",
"src": "Patient: since i was small maybe like seven i had been hearing voices calling my name i dont think they are in my head because they sound so real. but then as i got older it gets more worst becouse know i start to hear other stuff besides my name like its trying to get my attention by making weird noises or whistling and know i think i m seeing shadows fallowing me everywere but when i m not alone i dont hear nor see anything like that. i sometimes think that i m going crazy. i once told my mom about this and she just said that maybe i was just imagining stuff. am i actually going crazy? Doctor: There is nothing called \"crazy\". To you the experiences are real. That is all that matters. 60% of people hear voices or see things that others can not experience atleast once in their life. This is a proven fact. The only difference is that your experiences are persisting for long time and are troubling you. The best thing to do is to get professional help. It could be that the most simple thing could be the answer to your problem, like talking about stress that you are experiencing at home or at school or with friends, or stopping the occasional pot or marijuana that you may be doing or whatever. I am just guessing all the simple things that could help you with these things apart from the notion that you are going crazy. Do consult a psychiatrist. There are lots of reversible causes that may cause experiences like these. And we can help in all of them. Half the battle is already won that you are seeking help for the experiences. Goof Luck."
},
{
"id": 113001,
"tgt": "Have lump in back along with pain. Should it be taken serious?",
"src": "Patient: Hi! I'm a healthy female 15 year old and i was wondering if it's normal for me to be experiencing extreme pain in my lower back? Based on my observations, there is a small lump about 3\" in diameter on the area where i feel extreme shooting pains. Could it simply be because of carrying heavy nap sacks to school or the long hours of sitting at a ninety degree position? If so, could this lead to anything dangerous? Oh and i have no history of being involved in any dangerous incident regarding my back. I haven't been.Since i haven't consulted a doctor on this case, my decision to push through with a medical check up will depend on your advice. So i look forward to your reply and i thank you so much for your time! Doctor: Hello. Thanks for writing to us. The pain in your back seems to be a muscular pain and is likely to be due to a postural defect and carrying heavy loads on your back. A direct examination and investigations like MRI scan will help in detecting the cause of this pain. I hope this information has been both informative and helpful for you. Regards, Dr. Praveen Tayal drtayal72@gmail.com"
},
{
"id": 120339,
"tgt": "What causes arm bruising?",
"src": "Patient: I have a bruise on my arm that has popped up a few times. It is in the same spot and has the same shape to it every time. I know I didn t hit my arm on anything or pinch it or anything either. The bruise doesn t hurt and if it is the same as the other times and will have it for about a month which is much longer than normal bruises that I know how I got. What could be the reason for this?? Doctor: Hello,Bruises without any cause can be related to bleeding disorders. I suggest to do a complete blood count and platelets level for further evaluation.Hope I have answered your question. Let me know if I can assist you further. Regards, Dr. Dorina Gurabardhi, General & Family Physician"
},
{
"id": 195003,
"tgt": "What causes testicular pain while treating diabetes, CVD and Alzheimer s disease?",
"src": "Patient: My father, age 80, has type 2 diabetes (treated with Novolog and Lantus), plus cardio vascular disease (Lipitor and he had open heart surgery app. 14 years ago), neuropathy in legs (Neurontin), Alzheimer s (Aracept), and dizzy spells, has just come to live with us. He takes a very long time to urinate (10 to 15 minutes), and now says he has worsening pain in his right testicle, which is spreading up into his back. Doctor: Hello, He is a duabetic so there's is a possibility of neuropathic pain that radiation to back from testicles. Other possibilities like infections like orchitis or epididimitis due to mums or other bacterial Infections in diabetic. It is a immunosupressed condition. Torsion or hydrocele or hernia may present like this. Until examination is done at the local site it is difficult to say what it is. Please consult your physician/surgeon he will examine the area and do ultrasound scrotum if any queries then treate you accordingly. Hope I have answered your query. Let me know if I can assist you further. Take care Regards, Dr. Penchila Prasad Kandikattu , Internal Medicine Specialist"
},
{
"id": 195492,
"tgt": "Suggest remedies for cysts in the groin",
"src": "Patient: iam 35yrs 5'10\" 205lbs no health problems but i have what i think is cyst of the scrotum i had these for about 17ys and is starting to efect me with my sex life with my wife. what kind of dr do i need to go see to have them looked at and removed ? They do hurt at times when they swell and the white fluid comes out. Doctor: Hello and Welcome to \u2018Ask A Doctor\u2019 service. I have reviewed your query and here is my advice. I would like to have a look at them before giving you any advise if the cysts are on the skin of the scrotum then it might be sebaceous cysts which is fairly harmless condition and you can get it removed by general surgeon as they may get infected. If the cyst is felt inside the scrotum above or around the testes then it might be spermatocele which is also known as epidydimal cyst which is the swelling of testicular tube,it is also a harmless condition and nothing needs to be done except pain control if required. Hope I have answered your query. Let me know if I can assist you further."
},
{
"id": 57527,
"tgt": "What is the whitish area on my right lobe after an ultrasound of liver?",
"src": "Patient: I had a ultrasound done of my liver, found a whitish area the size of 5.4 cm on my right lobe, I clearly saw it with my own eyes, now I had a MRI done and doctors say there's nothing on my liver when you can clearly see it on an ultrasound. I'm 21 weeks pregnant. What can it be or why can't they see anything on the MRI? Doctor: Hi, thank you for posting.You don't have to worry...Ultrasound is a widespread method all over the world. It is cheap, fast and harmless method. But it is not an exact method. Ultrasound images sometimes are fallacy, like in your case. An ultrasound technician with work experience can easily find mistakes.MRI is a sophisticated and exact method. If MRI is negative, there is no reason to worry about.Regards.Dr. Behar."
},
{
"id": 187633,
"tgt": "Is the lump on inner cheek after having tooth extracted dangerous?",
"src": "Patient: Hello , I have my tooth extracted #3 out on Tuesday January 4 and there's a lump in the side of my mouth near my cheek were the tooth was taken out of . I'm not sure of that make any sense . But it's hard like a bone . It doesn't hurt or bother me but it just weird knowing that it's there . Should I check with the dentist or will it heal soon ? Please help . Thank you ! Doctor: Hello, Welcome Thanks for consulting HCM, I have gone through your query, you have undergone extraction of tooth , now you are feel like hard swelling , dont worry it can be Bony Spicules also , for that you should visit your dentist and Investigations (IOPA) done , if it is Bony Spicule go for ALVEOLOPLSTY done , Hope this will help, Wish you good health."
},
{
"id": 152361,
"tgt": "Recent advances in the management of G.B.SYNDROME",
"src": "Patient: RESPECTED DR.Kindly let us know the recent advances in the management of G.B.SYNDROME. Doctor: No cure yet. Scientists are concentrating on finding new treatments and refining existing ones. Scientists are also looking at the workings of the immune system to find which cells are responsible for beginning and carrying out the attack on the nervous system. The fact that so many cases of Guillain-Barr\u00e9 begin after a viral or bacterial infection suggests that certain characteristics of some viruses and bacteria may activate the immune system inappropriately. Investigators are searching for those characteristics. Neurological scientists, immunologists, virologists, and pharmacologists are all working collaboratively to learn how to prevent this disorder and to make better therapies available when it strikes."
},
{
"id": 119314,
"tgt": "Blood test showed low B12 levels, diagnosed as prenicius anemia. What treatment should be taken?",
"src": "Patient: a blood test back in February indicated that my daughter s B12 was on the very low end of the spectrum, yet the dr. said that was not a problem. Another dr. just told us recently she had prenicius anemia yet does not treat that diagnosis. Should prenicius anemia go untreated and what happens then. What specialist should I be looking for to treat this? Doctor: Hi, Thanks for query. As your daughter has very low B12,it requires treatment with injection B12 and she will be alright. If pernicious anemia untreated she will feel extreme weakness,dizzines,loss of appetite,cramps,muscle weakness. You may consult Hematologist or Physician. Ok and bye."
},
{
"id": 70756,
"tgt": "Are heterogeneous opacities of the right mid-lung and left lung base a reason to worry?",
"src": "Patient: Hello, I was being checked over because I am going to have surgery soon a T10 PSO C3I had a chest X-Ray that stated Findings: Heterogeneous opacity in the right midlung and left lung bas, Mild elevation of the left hemidiaphragm. Postsurgical changes of the lower cervical spine. The heart is within normal limits. Impression: Heterogeneous opacities of the right midlung and left lung base. Whether these are acute or chronic is unknown. These appear to be new compared to images of a scoliosis series however the lungs are not well seen on the prior series. Direct comparison with any outside studies is recommended. Should I be worried? Doctor: Hello, * There can be various issues regarding this as chronic or acute inflammatory reason or systemic issue from the malignancy or other disorder. * Details of symptoms with CT plates are mandatory here to guide further. Hope I have answered your query. Let me know if I can assist you further. Take care Regards, Dr Bhagyesh V. Patel, General Surgeon"
},
{
"id": 191463,
"tgt": "Can Insulin shots be safely avoided for a few days on doctor's advise?",
"src": "Patient: my father inlaw who is 74 was sent home and told not to take his insulin on Tues , until he consults his doctor who then said he was to only take 10 units a day ( was 56 and 58 units) his levels are at 17.1 now yesterday was 14.9 at 7 pm , day before 15 at 7 pm his doctors apt isn't till tues afternoon should we be concerned , he just got out of hospital ( kidney were 1000) and his heart is only pumping 25 % ( past couple years Doctor: Hello and thank you for asking!I can understand your concern regarding to your father's health.The dose of insulin should be increased because the blood glucose levels you reported are too high.You should report the type of insulin he is taking actually and the number of injections per day, so i can help you to increase the doses depending on glucose level.You should not wait too much as this high levels may be problematic for his situation.Will be a pleasure to help you further.Best wishes,Dr.Mirjeta"
},
{
"id": 97977,
"tgt": "Getting treated with chemotherapy for breast cancer. Is this the proper treatment? Is radiotherapy useful?",
"src": "Patient: hello iam kapilachander, my mother affected by breast cancer we have treated with 2 rounds of Chemotherapy . in first round has 6 cycles and then after 6 months we gone for check up the doctors report say that CA cells spreads to lungs as well as bone marrow in rib cage. in this 2013 January we treat my mother with 2nd round of chemotherapy with 3 cycles as January.febuary and march. now my question is this treatment is enough or further we have continue with chemotherapy. or else i may go to Ayurveda treatment. 1. if iam move to Ayurveda treatment is there is possible of any side effect to my mother? 2. else iam may continue both the chemotheraphy as well as ayurveda treatment is this possible to my mother since my mothers age is 60 years old? plese give me a solution to me... if there is some other treatment like radiotherapy is good for my mother to cure the Cancer plz give me some good solution because iam in total confusion..... Doctor: 1. Cancer has spread to lungs and rib cage, Metastasis is there, which itself means grave condition, thus prognosis is not good. 2. This is stage IV of CA and whether TNM Classification was done? 3. were Tumor Markers test carried out, and whether they find out that CA is hormone positive or negative? 4. any family history?, her lifestyle, any addiciton, use of OC pills, Weight, diet rich in fat? 5. at this age and stage apart from medicine she needs emotional and group support 6. Ayurveda medicines can be given along with Chemo/Radio-Therapy which helps in slowing the progress of the Tumor., you can contact with an Ayurveda Physician there for the same."
},
{
"id": 74170,
"tgt": "Suggest treatment for cold and chest congestion",
"src": "Patient: My wife is 76, generally in good health, began taking 25 mg metoprolol er succinate last Friday. she has had a cold with congestion. She just woke up and said she felt uncomfortable breathing but said she is breathing ok. her blood pressure is 128/80, pulse is 59. should we be concerned? Doctor: Hello! looks like your wife has developed bronchospasm (constriction of her airways) due to metoprolol . Also her heart rate is 59 (bit below normal). Hence kindly review with your physician regarding the reason for starting on metoprolol (a beta blocker)and discuss regarding feasibility of shifting to some other safer medication .For symptomatically benefit for her breathing difficulty she might bronchodilator medications (drugs that dilate the airways n make it easier to breathe) for a short term ."
},
{
"id": 41024,
"tgt": "Suggest remedy for difficulty in conceiving and painful sex",
"src": "Patient: i am 29 years old, i have a 12 year old son. i have been trying to conceive for eight years and i have been unsucessful. my husband has no medical issues. i went to speak with a doctor and the only thing they told me was i have a backwards uterus. i have pain when i have sex and i still have not conceived. what should i do? Doctor: HelloBackward uterus(Retroverted uterus) is a normal variant of uterus position.It is most likely not related to infertility.Infertility may be due to some other reasons.It need proper evaluation.Your partner also need proper evaluation.Semen analysis should be done initially.Besides routine investigations you may need hormone profile assay,thyroid profile,follicular study etc.Further investigation depend upon findings.Pain during sex may be due to many causes like dry vagina,problem in cervix etc.This need proper local clinical examination.You may need few medicines after evaluation.You should consult renowned infertility center.Get well soon.Take careDr.Indu Bhushan"
},
{
"id": 133832,
"tgt": "Can i take crushed lortab to relieve pain in hip bone?",
"src": "Patient: I have stage IV lung cancer which has traveled to my hip bone and is giving me pain. I have been prescribed Lortab 7.5/32 to relieve the pain. I take it as seldom as possible, but if I wait too long I have bad pain for up to an hour after I take it before I get relief. If I crush the pill and dissolve it in water will it take effect sooner? What happened to the Free? Your ad is deceitful. I will be reporting you to the dept of consumer affairs. Doctor: hi,thank-you for providing the brief history of you.A thorough neuromuscular assessment, Musculoskeletal assessment and respiratory assessment is advised.Based on you query about hip pain it will be a muscular pain and shall be treated by a physical therapist. It is non invasive therapy. With Therapeutic ultrasound therapy and TENS therapy your pain will come into control.Also, the physical therapist will be interested to help you learn breathing exercises as you have a lung cancer. With the help of the breathing exercises, the lung capacity improves, the muscles of respiration also Improve.As per the clinical experience, patient suffering from lung cancer need a combination of medication and physical therapy. As breathing exercises has its role to control the symptoms of the lung cancer and sometimes make and ease of breathing in normal conditions.You can discuss the same with your oncologist.RegardsJay Indravadan Patel"
},
{
"id": 137038,
"tgt": "Can lasix be taken to reduce severe swelling of legs and ankles?",
"src": "Patient: my legs and ankles are severlly swollen I took 80 mg of lasix this afternoon and a pil that is supposed to boost the lasix and Ihave not seena difference in them it i really painful to walk would a trip to the er do any good or is there something else I can do Doctor: Hi there.Lasix is a dangerous medicine and should not be taken without Doctor's prescription. Lasix is a water drug and helps drain body fluid through the urine. I advise you to keep both your feet elevated over a couple of pillows or a chair. Flex your feet and ankles up and down for about 10 times a day. This will help drain the body fluid back to the heart. You may be suffering from deep vein thrombosis or varicose veins. You must see a Surgeon for the same. Avoid squeezing the calf. Keep well hydrated. Wishing you a speedy recovery."
},
{
"id": 196140,
"tgt": "What are the side effects of masturbating?",
"src": "Patient: Hi, I'm 58 years old and still can't grow a full beard and I have no hair on my legs or chest. My older brother could grow a beard in high school. Was using asthma medicen when I was a pre-teen, or masterbating since I was able to get an erection cause this? Doctor: hii.masturbation is not related to beard growth.biotin,a vitamin helps in beard growth.take multivitamin tablets daily in the night.take nutritious diet.applying eucalyptus oil on beard area may help you in beard growth.thank you."
},
{
"id": 153762,
"tgt": "Suggest treatment for elevated BP after pituitary tumor removal?",
"src": "Patient: Thanks,i had a pitutury tumor removed 6weeks ago and i am on medication of hydrocortisone,i have sometimes high blood presure which i never had before ,i donot know how to lowe don my blood presure and i get scared,i had my lad blood work everything was ok but my creatinine is 101 and tour prefernce rate is 40-95 and my Gfr is very low and is 49,please let me know what to do Doctor: Thanks for your question on Health Care Magic. I can understand your concern. Since your creatinine is high and GFR is low, possibility of hypertension due to renal damage is high. Altered creatinine and GFR suggest renal (kidney) damage. And kidney is having important role in regulation of blood pressure. So kidney damage can cause high blood pressure. So better to consult kidney doctor and start appropriate treatment to reduce creatinine level. Once this is normalise, your blood pressure will also come to normal. Hope I have solved your query. Wish you good health. Thanks."
},
{
"id": 80134,
"tgt": "How to cure shortness of breath due to pneumonia?",
"src": "Patient: I was in the hospital with pneumonia for five days at the end of March. After my discharge, I was told it would take a few weeks for me to feel completely better. I am still experiencing a lot of shortness of breath and have had a CT Scan, heart cath and everything comes back fine. My pulmonary doctor says it s because I need to lose weight. I won t argue that fact, but I have been the same weight for at least six years and never had this problem. I have RA and rheumatologist has suggested that it could be related to that, but my pulmonary doctor says, no . I had a PFT and was told that on a scale of 10, my lungs are an 8 , That doesn t make it any easier to breathe. Doctor: Thanks for your question on Health Care Magic. I can understand your concern. In my opinion, you are mostly having post infectious bronchitis. Pneumonia causes inflammatory changes in lungs. Due to these changes, lung function is hampered. Inflammation causes edema and structural changes in lung. This is the main reason for post infectious bronchitis. So better to start inhaled bronchodilators and inhaled corticosteroid (ICS). Since all these drugs are prescribed medicines, you need doctor 's prescription for them. Don't worry, you will be alright. Hope I have solved your query. Wish you good health. Thanks."
},
{
"id": 138421,
"tgt": "Suggest a remedy for the severe ankle sprain",
"src": "Patient: I rolled my ankle five days ago, resulting in a trip to the ER. After X-rays revealed no obvious fractures, I was diagnosed with a severe ankle sprain. I wreturnedd in a splint, prescribed crutches, and encouraged to make a follow-up appointment with an orthopedic. There was no initial bruising, but 24 hours later the bruising around the ankle was significant. After 48 hrs I noticed bruising between my second and third toes. At the visit with the orthopedic, I was placed in a cast. Today I noticed the bruises on the toes had almost faded entirely. But now, I see that they have returned and appear as dark or darker than before. Is it normal for bruises to get lighter and darker like this? Doctor: You should be very careful if you have diabetes ! Try to check if your toes are warm or cold, do you still have pulsations and can you feel any sensations with them-cold and hot, press gently with needle! If they are cold and you do not have sensations you should see a doctor immediately!"
},
{
"id": 84796,
"tgt": "Why do I have extreme groin & medial thigh pain two weeks after a gluteal IM injection of progesterone?",
"src": "Patient: Why do I have extreme groin & medial thigh pain two weeks after a gluteal IM injection of progesterone? Did my husband give the shot in the wrong place? It feels like the entire groin & inner thigh are tight & painful to touch making walking or sitting very painful. Doctor: Hi I cannot say in your particular case, but generally, pain, swelling especially if there is warmth and redness or fever can be an internal infection or internal bruise or internal collection of infection all of which are quite serious problems. Hope I have answered your question. Let me know if I can assist you further. Regards, Dr. Matt Wachsman, Addiction Medicine Specialist"
},
{
"id": 6101,
"tgt": "Off depo, had unprotected sex, tender swollen breasts, mood swings, light spotting, lard abdomen, negative pregnancy tests. What does this indicate?",
"src": "Patient: Hi, I m really confused, I came off the depo around 2 months ago and had unprotected sex with my partner frequently for the first month. Now I have extremely tender swollen breasts, mood swings and about 3 weeks ago I experienced light spotting. My lower abdomen also feels quite hard. However I ve taken two pregnancy tests , one around 3 weeks ago and another a few days ago and both came out negative! Could I still be pregnant? Doctor: Hello Welcome to HCM forum Depo inj can cause delayed periods and irregular periods. Breast tenderness can be due to depo shot To be extra careful you should repeat pregnancy test after a week and if possible get an ultrasound done. Best wishes"
},
{
"id": 190664,
"tgt": "Loss of taste post taking a tablet that stuck on the tongue for seconds, in a state of cotton mouth. How can I get rid of this?",
"src": "Patient: I was on prednisone for 9 days. I have been off for 3 days now. On the 3rd day of treatment, a pill stayed on my tongue for several seconds before I swolled it. Since then, I have had no taste sensation. I am in a constant state of cotton mouth now. It has affected my tongue and the roof of my mouth. Will this go away? Is there anything I can do? Doctor: hello, the lesion on your tongue and palate are chemical burn/irritation caused by tablet coating.. these lesions take time in resolving especially when they occur on tongue.. but they will resolve on its without any medical help.. as the new epithelium/covering on tongue is formed.. if possible avoid spicy foods and have good balanced diet.. take antioxidents- like lycopene and vit-b , vit-c tablets as they help in re-epithelization of mucosa.. 10-15days can be taken for resolution of problem .. if they persist for more then that consult oral medicine and diagnosis specialist.. they will help you out.. use antiseptic mouthwashes to prevent infection on denuded surface.. maintain proper oral hygiene.. take care.."
},
{
"id": 178472,
"tgt": "What causes continuous vomiting and diarrhea in a child?",
"src": "Patient: I have a 9 year old daughter that has had continuous vomiting and diarrhea for four days, she has no fever and blood pressure has been fine. She vomited sunday and Monday and was fine until just now. She had blood drawn at the drs today but then came home and is vomiting again... Doctor: Probably your child has gastroenteritis. It causes diarrhoea and vomiting that can continue for 5-7 days. Give her Ondem MD (ondansetron mouth dissolving tablet) half an hour before food so that she does not vomit it out. Give ORS so that she does not suffer from dehyration due to loss of salt and water from body. The blood counts will indicate if there are signs of infection. If you post the reports, I will try to help you further."
},
{
"id": 162011,
"tgt": "Suggest medication for treating severe cough and chest congestion in a 6-month-old",
"src": "Patient: Hi my son was born on 9th Jan 2018 and is a 6 and a half months old. He has been suffering from Cough and Chest congestion for about 3 weeks now. Finally after trying a few milder medicines, finally the doctor has recommended treating him with Nebulizer for 3 days (twice a day, with Saline water) and Moxclav BD 228.5mg Antibiotic. He seems to be slowly recovering but the congestion and cough still seems to be bothering him a lot. Kindly suggest what can be done from here on. He has completed 4 rounds(2 days of Nebulization as well as Moxclav BD) in addition to the cough syrup which is Kufril LS and Levocet. Doctor: Hi, If there is no fever then nothing to worry much. Just ensure temperature monitoring through. Continue the same medication & nebulization for upto 5 days. Ensure complete cover of body including head & ears. Give luke warm soups and even ORS. This will give soothing effect on throat thus decrease the cough bouts. Dr. Ajay Gupta, General & Family Physician"
},
{
"id": 63224,
"tgt": "Suggest treatment for lump in neck with abnormal PAP",
"src": "Patient: Hi, I had a blood test done about 8 months ago and everything was normal. I had another blood test done because of an abnormal pap & it said my absolute eosinophils were high (1100) It says normal range is 5 - 500. I got a biopsy for the abnormal pap today. I have also been a little worried about a little lump in my neck, left side, moves around easily but in the same area. Would like to get your thoughts on these issues, Thanks. Doctor: hi.it could be a separate entity. tho there's still a possibility for it to be related. have your neck lump checked by a doctor. a physical examination would be of help. it could be a lymphadenopathy/lymphadenitis which could be secondary to an ongoing infection. or it could be lesion with an etiology other than infection. further diagnostic examination (such as neck ultrasound or ct-scan) and management will be directed accordingly.hope this helps.good day!!~dr.kaye"
},
{
"id": 140145,
"tgt": "What could cause drowsiness?",
"src": "Patient: i am 30 years old height 1.55 weight 70 kgs i dose off for a few minutes excessively during daytime. it doesnt matter where i am in church, sitting talking to people. this has been happening since i was a child and now its becoming embarassing. i cant visit anymore Doctor: Hi, You have features of excessive daytime sleepiness. Most likely it is a part of sleep disorder. The common causes are narcolepsy and obstructive sleep apnea. A sleep study would confirm the diagnosis. Hope I have answered your query. Let me know if I can assist you further. Take care. Regards, Dr. Sudhir Kumar, Neurologist"
},
{
"id": 145289,
"tgt": "How to reduce really bad sciatica that starts in sacra area spreads to left hip with pain in knee ?",
"src": "Patient: I have really bad sciatica starts in my sacra area and into my left hip and terrible pain into the knee and down into my ankle bone of that same side. sitting is the worst standing okay and lying down flat what exercises should i be doing to elevate the pain want to get it right as travelling on long flight in a couple of weeks time Doctor: Hello ! I read your question and understand your concern. I would first recommend a lumbar CT scan to examine better the disks in this region. If I were your doctor I would recommend to hold a lumbar brace, have a lot of physiotherapy, take Acetaminophen and Ibuprofen three times daily for 10 days. I would advice you to avoid standing in sitting position for long periods , as it can aggravate the problem.If the pain is disabling most of the time you may need to consult a surgeon.Hope to have been helpful. Best wishes Dr. Abaz Quka"
},
{
"id": 148176,
"tgt": "Suggest remedy for on and off blank stare, sudden check out of mind and loss of memory with intake of Topamax and Dilantin",
"src": "Patient: new to seizures. had first one at 41 and still trying to figure out what kind I am having. I have a blank stare and kind of check out for a few minutes and then for about 10-20 minutes later I come back to I don t know. MRI shows brain is good so far. Currently on Topamax and Dilantin. Losing memory . I can follow directions during a :checkout: time but am not self so I am told. but have no recollection of it. HELP PLEASE :( Doctor: Hi,Welcome to Health care magic forum. It appears that you are getting seizures due to the hypertension, and there also may be atherosclerosis. It should be confirmed that the hypertension is always under controle, and never touching the higher reading. I advise you to consult a neurologist for diagnosis and treatment. You may need to have a C.T.scan,and E.E.G. for diagnosis. Avoid taking the drugs, having seizures as a side effect like antihistamines, fluroquinolone antibiotics etc. Wishing for a quick and complete recovery. thank you."
},
{
"id": 196362,
"tgt": "What is the cause and treatment for pain in testicles?",
"src": "Patient: i am 27 years old bachelor. i never had any sexual intercourse till now.i masturbate frequently and i had hurted my epididymis an year ago. it started paining with infection and i have been taking treatment since then. the Surgeon had adviced me nothing was wrong and everything was fine. recently the doctor told me that no more treatment is needed and i do not have to visit him again. he prescribed me to take cataflam tablet whenever i fee pain. the pain is coming on the right side of testicles and the doctor told me one vein is still need to be recovered. having said that,he mentioned that he doesn't know what to do with that and i do not have to visit him again. also he does not advice any operation as everything is fine. i took a second opinion from another doctor and he did some ultra scanning 6 months ago and all the results were normal.I am worried now as my doctor said as he do not know how to treat this pain. please help me with your suggession as how should i proceed? the pain is not stopping at all and making me stressed despite it is not swelling very high.many thanks.. Doctor: Hi and welcome to Healthcaremagic. Thank you for your query. I am Dr. Rommstein, I understand your concerns and I will try to help you as much as I can.All these symptoms are suggestive of epididimitis which is inflammation of part of testicle and this not uncommon in males especially in young age. This is usually treated with 10 days of antibiotics, usually ciprofloxacin and there are no permanent consequances. MAsturbation and intercourse should be avoided till pain persist, if possible since I may prolong inflammation course. Some more serious conditions such as tumors or torsion are rare but in doubtful cases you should do Doppler ultrasound or scintigraphy. In this particular case, I don\u2019t think this is required.I hope I have answered you query. If you have any further questions you can contact us in every time.Kindly regards. Wish you a good health."
},
{
"id": 210224,
"tgt": "Suggest treatment for suicidal thoughts",
"src": "Patient: Hi, I tried killing myself on thursday night but unfortnatlly i was found by police and it didnt work....my stomach is churning all the time and my heads dreaming up weird things....is there anyway i can make these stop? or i feel like im going to try killing myself again just to stop feeling like this. thank you Doctor: Hello and welcome to Healthcare Magic. Thanks for your query.I understand that you are going through a very difficult time. Suicidal thoughts can be due to intolerable stress, depression or other psychological problems. However, you needs a detailed evaluation to arrive at a definitive diagnosis and initiate appropriate treatment. Since you appear to be quite distressed by these thoughts, I would advise you to seek help urgently. Please share your feeling with someone close and I would also advise you to consult a psychiatrist for a detailed evaluation and further treatment. Effective treatment is available in the form of medication or counselling.Wish you all the best.Regards,Dr. Jonas SundarakumarConsultant Psychiatrist"
},
{
"id": 63427,
"tgt": "What is the painful lump on chest side?",
"src": "Patient: I have a long lump on the right side of my chest for a week. It's like a straight line over the rib cab. It's about 6 inches long, moves with the skin, it is not red, but painful in a couple spots. If I stretch my arm up, I can actually see a lump in the skim. What could it be, and should I go to a doctor. Doctor: Hi,Dear,thanks for the query to Health Care Magic.Dear, your question has multiple facets which would be replied to satisfy your query.After meticulously studying your facts and your concerns following possibilities need to be considered in coming to its diagnosis -Shingles/herpes zoster for a weeks time and in process of development -as its 6 inch long along rib cage and is painful at couple of spots.Its very difficult to think of the probabilities as the facts are very scanty and hence I would advise you to consult a ER Surgeon and would work out the clinical check up and would fix its diagnosis and its rightful treatmentI would advise to consult ER surgeon who is the right specialist who would work up and would plan treatment for you.Don't worry and go ahead as suggested to resolve the lump issue of yours Hope this would give you confidence to plan treatment with your doctor on your next visit ,which you should plan definitely.Feel free to ask any further query in this regard to HCM and to ME.Write excellent review if you feel satisfied with this reply,which would help my rating for the benefit of the other patients.Wishing early and fast recovery for your lump.Dr.Savaskar M.N.Senior Surgical ConsultantM.S.Genl-CVTS."
},
{
"id": 176376,
"tgt": "Suggest remedy for sprained ankle",
"src": "Patient: Yes, my daughter sprained her ankle Friday night;she went to the ER and was told that it was sprung and that a blood vessel. The ER wrapped it, but she has been in such much pain with sever burning sensation and also she has a blister on the same foot; the foot is cold to the touch. She went back to the ER again last night and on her way back again to night. What can we do for her? Doctor: Hi...the treatment for sprained ankle is complete rest and a POP (Plaster of paris) cast or a crepe bandage keeping the ankle in the correct position of comfort. When the bandage or POP is too tight then the symptoms of blistering and cold limbs occur. I suggest you get back to an orthopedic surgeon and get the bandage loosened appropriately.Regards - DR. Sumanth"
},
{
"id": 189031,
"tgt": "Woke up with a sore jaw, teeth. Now having a drippy, runny nose with blood. Problem?",
"src": "Patient: I woke up yesterday with the right side of my teeth and jaw sore, thinking I had clenched my teeth while I slept. Waking up this morning it was just as sore maybe worse and I started having a drippy, runny nose with blood. Because of the clear liquid and blood it was sort of watered down. This started at 9am and it is now almost 8pm and it has settled down a lot but is still happening. I sat in steam for 30 minutes, which helped the blood decrease. I also took a hay fever pill. What's going on? Help please. Doctor: Hi,Thanks for asking the query,Pain in the tooth can be due to bacterial infection, sore jaw can be due to the radiating pain. I would suggest you to visit the Dentist and get the checkup done take an x-ray of the tooth, such type of infected tooth is to be treated with root canal treatment to save it permanently.Take a complete course of antibiotics and analgesics.Take antiseptic mouth wash gargles.Maintain a good oral hygiene, drink plenty of water and keep yourself hydrated.Take care!"
},
{
"id": 179183,
"tgt": "What does bumps on armpits and behind knees in a baby indicate?",
"src": "Patient: My 8mo old has several flesh colored bumps in his armpits, elbow creases, and behind his knees. I noticed a few in his armpit 4 days ago and they seem to be spreading quickly. Some are turning red and look like they re scabbing over. What could it be? Doctor: Hi...by what you quote it seems like the kid has contacted a hand foot mouth disease. There may be some ulcers and similar lesions in the mouth too. Kindly check and if so it is almost confirmed. Usually it is a self limiting disease and trivial viral infection. Do not apply anything on the skin as it can spread the lesions. The best way is to give some over the counter antihistamines before consulting your docotr and in 4-6 weeks you should be expecting a skin peeling too and do not think that it is another unrelated skin disorder. Regards - Dr. Sumanth"
},
{
"id": 52290,
"tgt": "What causes persistent nausea while having elevation in the liver enzymes?",
"src": "Patient: I have had nausea everyday for months. It just started out of nowhere. Everyday it is mild to moderate nausea and it can come and go or last all day. Other days I will have severe nausea lasting hours to days. I have had bloodwork that show elevated liver enzymes. I have been to a gastroenterologist who found nothing. Do you have any suggestions what to test For? I just want my life back. Doctor: Hello, The elevated liver enzymes values are significant only if elevation is two-fold that is greater than 90. Or else it's not significant. If your liver enzymes values elevated two folds than ultrasonography of abdomen should be done for further work up. If no liver pathology present than endoscopy can be done for further work up. Hope I have answered your query. Let me know if I can assist you further. Regards, Dr. Parth Goswami, General & Family Physician"
},
{
"id": 196882,
"tgt": "Suggest treatment for the round open sore on the penis",
"src": "Patient: I have a round open sore on my penis shaft about the size of a pencil top eraser. It is painful when bumped. It also scabs and then when the scab comes off it looks sort of whitish. The sore is slightly raised around the edges but otherwise basically flat.I visited my doctor and he tested for a host of std's. All results were negative. He is suggesting that I may need to see a specialist. Meanwhile, I have been applying a triple antiobiotic ointment to encourage healing, but this does not seem to be helping any. The sore has existed for about a week now with no real healing.I will go see a specialist as suggested, but just to put my mind at ease, is there any idea what this may be? Doctor: HelloThanks for query .You have ulcer over shaft of your penis since one week .You have not mentioned about sexual history in your query .whether it occurred after unprotected sex ?You need to consult sexual disease specialist and get swab test and culture of a scrapping from the base of the ulcer to establish diagnosis .You need to take broad spectrum antibiotic like Cefotaxime along with anti inflammatory drug like Diclofenac twice daily along with topical antibiotic ointment like Neosporin twice daily.Ensure to wash your genitals with warm water twice daily.Ensure to avoid sexual encounters till it heals up completely.Dr.Patil."
},
{
"id": 44870,
"tgt": "What is the normal sperm count and how to increase it ?",
"src": "Patient: What is the normal values of sperm count?How can improve sperm count? Doctor: hi welcome to healthcaremagic forum normal spermcount vary from 30-60 million per ml. it may be vary from person to person you need to eat healthy and do regular exercise there are medicine but must be prescribe by doctor so better to consult doctor i hope i have answered your question"
},
{
"id": 215282,
"tgt": "Suggest treatment for clenching pain in cheekbone",
"src": "Patient: I m finding some information that pertains; however, here goes. I m having clenching pain in left cheekbone that radiates to my left eye, along with sight disturbance. This concerns me because I had an acoustic neuroma removed from my right side (2.54 cm, retrosigmoid approach) in 2007 and had eye twitching related to it that I originally put down to stress. My guess is that I just need to go back to my neurosurgeon but have High Deductible plan and hate to go it not necessary. Doctor: Hi, On the one hand a new symptom that goes in the pattern of a definite nerve problem is going to be 1) real and 2) needing attention. On the other hand, it is going to be addressable. Worst case is that nothing is broke and the TRIGEMINAL NERVE IS IRRITATED. Nerve dampers like any anti-epileptic work really well on this. Hope I have answered your question. Let me know if I can assist you further. Regards, Dr. Matt Wachsman, Addiction Medicine Specialist"
},
{
"id": 139564,
"tgt": "What causes tingling and fogging feeling in brain in mornings?",
"src": "Patient: Hello, I m 26 I consider myself super healthy eat friuts and vegtables daily I exercise a good amount. I recently stopped smoking marajuana for a month and a half after I ve been smoking for 5 years. Here s my question, my brain especially in the morning feels like its tingling and foggy. Is my brain healing from all the weed? or should I be worried? I would really like not having to go to the Dr if I absoutly don t have to. . Doctor: Hello,Some symptoms of withdrawal after quitting marijuana are acceptable. I think you shouldn't worry about. You should see your Doctor if there is a worsening of the symptoms.Hope I have answered your question. Let me know if I can assist you further. Regards, Dr. Erion Spaho, Neurologist, Surgical"
},
{
"id": 5107,
"tgt": "In my HSG test the left fallipan tube is not visible. Blood report is normal, no thyroid. Worried about injecting sperm. Suggestions",
"src": "Patient: hello dr yesterday i taken HSG test in this test my right fallopian tube appeared to be normal bt my left fallpian tube is not visualised. after i show my report to my gynecologist said next time we have take another test that test was my husband sparm collect and inject with me that system wast not satisfied with me ,my last 3 months i had take the pregnancy treatment my gynecologist every month give ova shield taplet only without any scan.my blood report every thing is normal i have no thyroid problem also,i m worry about injection sparm not naturally and this system mostly become operation also i dont want what can i do next pls clear my doubt,my hsg report not satistied what can i do which one is correct pls tell me docter Doctor: Hi thanks for your question.Your one tube on HSG is blocked, so chance of pregnancy are reduced by 50%.since you have not conceived by natural method of injection sperms into uterus. You have to go for artificial method of injecting sperms into uterus, the method is called as IUI.It too has only 50% chances of success as you have only one tube patent, and chances are that sperms will travel through that tube to fertilize the egg. If 3 cycle of IUI fails then you have to go for IVF,that is in vitro fertilization.Hope this answers your question."
},
{
"id": 132564,
"tgt": "Suggest treatment for pain and swelling in ankle",
"src": "Patient: I have pain in my left ankle it hurts the most when I walk. I notice swelling around the outer bone after I have been on my feet awhile. I do not recall a injury to it. I am on my feet a lot but my right foot is fine. It has been bothering me for about a month now. I can bend it in all directions but it hurts when I step forward on it. I can t put all my weight on it either, What do you think? Doctor: Hi Hope this message finds you in good health.I have gone through your complaints and understand your concern.U seem to have a soft tissue swelling or a ligament inflammation that might be causing the pain.it generally occurs due to a simple sprain on the joint.U should get an Xray done to find out the exact cause.an mri should provide more information regarding the soft tissues. another possibilty is of a raised uric acid level that might cause swelling. it can be diagnosed by a blood test done. a callosity in that region might also cause pain.Hot fomentation,analgesics,anti-inflammatory drugs should help. A shot of local steroid injection should be useful.Nothing to worry about.\u00a0\u00a0\u00a0\u00a0\u00a0I hope your question has been answered.If you have any follow-up queries,feel free to consult me anytime.Thanks,Take care,God bless."
},
{
"id": 175635,
"tgt": "What causes blood in bowel movement?",
"src": "Patient: My 5yr old daughter just had a bowel movement and asked for help wiping. When I wiped her, the toilet paper had light pink blood on it. Her bum looks a little irritated, and she said it hurt a little when she went potty. What could be the cause of this and should I take her to the ER for this? Doctor: Hi,From history it seems that there might be having some soreness on the part due to having some local skin infection giving this problem.Due to constipation and giving more pressure to pas stool might hurt this region.Having worm infestation might produce irritation or itching might caused this lesion.Apply antibiotic cream locally.Avoid constipation.Give plenty of water.Ok and take care"
},
{
"id": 208652,
"tgt": "Suggest a remedy for a phobia of needles",
"src": "Patient: My son has developed an extreme phobia of needles over the past 2 years. He is overdue for his second Hep shot, and now needs to have a blood test done. He said the pain is not what bothers him, it's just the idea of having something in his body. He starts to hyperventilate at the sight of the needle, and can become irrational and violent if pushed. He is 14 years old. Would an oral sedative be enough to calm him for the procedure? Doctor: Hi,Your son seems to be suffering from specific phobia of needles/syringes. It is a condition which is amenable to treatment. You can consult a psychiatrist for the same. Medicines like escitalopram would be helpful. Alternatively he can be treated with behavior therapy sessions which will gradually help reduce his fear.For the purpose of vaccination, he can be given oral benodiazepine agent like clonazepam or lorazepam prior to the procedure which will help reduce his anxiety and go through it.Hope that this information was helpful. Best wishes."
},
{
"id": 4914,
"tgt": "Delayed periods, pregnancy symptoms with light brown spotting. On birth control. Possible pregnancy?",
"src": "Patient: Hi, so im about six days late now and feeling really off. I have some pregnancy symptoms such as slight nausea, headaches, boobs are tender, feeling bloated, some cramping and moodiness (i have randomly cried about 3 times this week for no big reason). I have never been late before, if anything Im usually early. I am on birth control but we don't use condoms and I know its supposed to be 99% but I've heard of others getting pregnant while on birth control.. also today i started getting light brown spotting but only when i go to the bathroom.Just wondering with all this information if there's a chance I could be pregnant? Doctor: Thanks for your question. It is important to realize that if you are sexually active, there is always the chance of pregnancy, even if you are using birth control. Oral contraceptives are, in fact, 99% effective. They are so effective that if 100 women used a birth control pill as directed, only one of them would become pregnant in any given year. However, that is still 1 percent of women pregnant using a birth control pill as directed.Birth control pill doses have gotten much lighter over the past twenty years, so formation of ovarian cysts is still possible. I did a study on this several years ago, showing that women may still form ovarian cysts on lighter dose birth control pills, although they usually do not ovulate. That is the mechanism for prevention of pregnancy.Ovarian cysts can make hormones which may produce symptoms in women that mimic pregnancy. A pregnancy test for you is in order and your gynecologist may also recommend an ultrasound of the pelvis to detect any ovarian cyst formation. Hormone production from the ovary may also produce the light spotting you are experiencing. Remember, however, that any amount of bleeding on a birth control pill may signify a menstrual period. Menstrual periods on birth control pills are drug induced and not really \"normal\". I hope this answers your questions. I am available for further consultation if needed."
},
{
"id": 206519,
"tgt": "How to reduce stress and hypertension?",
"src": "Patient: Hi, I would as Cardiologist. I am 44 years old, weight 106 kg, and my father died last year on heart operation, due to angina pectoris (heart veins blockage). I start getting hypertension from 3 years ago (then was overweight 130kg), then get medicine attacand 4 mg. From then, my BP was under control (about 135 /85). Five months ago, I start getting problems with sudden dizziness and BP increase, and my medicine is changed to Concor 5mg. I am busy on my work, it was happening again during the stress and long meetings my condition went wrong: dizziness, weakness and slight body shaking, pale face, mouth get dry, heart start running, complete loss of concentration, then need to leave, have a walk, take some food, and condition improves. Due to this, my concor daily dosage is reduced 3 months ago, on concor cor 2.5mg (BP under-pressure was suspected). I am having normal life, walk in evening near the beach about 3km, no problems, but it is happening during the work after stress, after tough and exhausting discussions, in physical peace /sitting, I start feel week, dizzy, my heart start running, jumping, body weakness, almost shaking, dry mouth, complete loss of concentration, generally very unpleasant feel. When come home, and measure my BP it is about 135 /85. Sometimes, when I expect stress day on work, I am taking 1.5mg Bromazepan, rarely 3mg Bromazepan. Maybe I should try to fully quit Bromazepan. I am wondering also is any difference between Concor and Concor COR? Doctor: DearWe understand your concernsI went through your details. I suggest you not to worry much. Blood pressure and stress or anxiety are always related. Sometimes measuring BP may become obsessive for an anxiety patient. Whenever the measurement is taken, BP shows higher readings, otherwise normal. Physicians know this and read the measurements accordingly and in that case, you need to understand your obsession and stop worrying.Many researches and researchers confirm that medicines alone cannot cure mental disorders. Life style changes, change in thinking pattern, relaxation etc are as essential as medicines. Psychotherapy can help you changing your lifestyle and thinking patterns. Yoga and meditation help you to streamline your metabolism and neurological balance. Please consult a psychologist for further information.Psychotherapy techniques should suit your requirement. If you require more of my help in this aspect, Please post a direct question to me in this URL. http://goo.gl/aYW2pR. Make sure that you include every minute details possible. I shall prescribe the needed psychotherapy techniques.Hope this answers your query. Available for further clarifications.Good luck."
},
{
"id": 88814,
"tgt": "Suggest remedy for pain lower abdomen",
"src": "Patient: my mother has a pain in lower abdomin and her age is 46 and doctor advise to do scan and after scan the report shows that 1) urinary bladder: normally distended 2) uterus: it is bulky and measure 9.9*5.0cms size. anterior wall of uterus shows evidence of a hypoechoic mass of 11mm size. endometrial is uniformaly thickened(thickness is 14mm)cervix shows evidence of 2-3 nabothian cysts largest measure 7 mm size largest follicle in right ovary is 19mm.left ovary do not show any mature follicle doctor are saying for the operation. is there any thing to worry or either continue with the doctor advise. plz reply me. wat is actuall condition and operation is necessery with this report. Doctor: Hi ! Good evening. I am Dr Shareef answering your query.Even though it needs a clinical assessment along with the investigation reports to opine on such a situation, from the history, it seems that she has got a fibroid,and and also endometrial thickening. In such case, I would advise you to follow your doctor's advice for a surgery. There is nothing serious to worry about, but the specimen should be sent for a histo-pathology report. Further management if any would depend on this report.I hope this information would help you in discussing with your family physician/treating doctor in further management of your problem. Please do not hesitate to ask in case of any further doubts.Thanks for choosing health care magic to clear doubts on your health problems. I wish you an early recovery. Dr Shareef."
},
{
"id": 174599,
"tgt": "How to treat typhoid in a 8 years old child?",
"src": "Patient: Son, 8 years, 30kg, diagnosed with typhoid. He had vaccination 6 months ago. Before diagnosis complained of terrible pain in chest (more to left side). Had explosive diarrhoea but did not have high fever. What could the pain in his chest be and what should be done about it? Doctor: Hi Dear Welcome to the HCM,Antibiotic according to the sensitivity report of blood culture .with adequate doses and duration will eradicate the infection.symptoms like fever,loose motions,pain may be treated accordingly by your pedeatrician.Hope the query is answered.Thanks"
},
{
"id": 199216,
"tgt": "Any suggestion to get sexual interest?",
"src": "Patient: i am 40yrs old and blessed with 3 children. my husband is 50 yrs old.he is not having interest in sexual life,but i am much interested. he is not giving importance to my feeling and often avoiding me. i am much worried.sir please sir help me what shall i do to get sexual interest to my husband Doctor: Hi,Decreasing sex libido is common at this age due to low testosterone level.,Remember that sex is an art and not merely mechanical action.So it require to prepare him for sex by,1, exciting sex talks, verbal, fantasy.2, have kissing, pushing,3, wear sex exciting clothes or your husband likes clothes more exciting,4, create quite and exciting atmosphere.5, see sex videos and porn sites.6, go for changing sex positions,This will give him sexual excitement and you will enjoy sex.Ok and take care."
},
{
"id": 207384,
"tgt": "Suggest remedy for short term memory loss",
"src": "Patient: Hi, I ve been having this problem for the past 5+ years. I ll have these random spurts of (what I call) amnesia. I ll be doing something or talking to someone and my mind goes completely blank. My body will still go through the motions and continue what I was doing, but my mind just stops. I won t recognize anyone around me. Or remember who they are. I can remember who I am, how old I am. It doesn t happen often, but it freaks me out when it does. Can you help me determine what it is I have? Doctor: DearWe understand your concernsI went through your details. I suggest you not to worry much. The description you gave is not enough for a proper diagnosis. But such things do happen with everyone. Cannot be termed amnesia without proper evaluation. Busy people do have momentary \"shut down\" of the system. You must understand, human brain is a power full computer and it do need to have some breaks. When information overload happen, it sulks. That is the way to think about your episodes. Please consult a neurologist if need be. If you require more of my help in this aspect, Please post a direct question to me in this URL. http://goo.gl/aYW2pR. Make sure that you include every minute details possible. I shall prescribe the needed psychotherapy techniques.Hope this answers your query. Available for further clarifications.Good luck."
},
{
"id": 129108,
"tgt": "What causes numbness in the left arm and nose with puffiness in face?",
"src": "Patient: I have a friend that ate far too many desserts over thanksgiving and this morning he has numbness in his left arm, numbness on his nose and his eyes and face seem to be puffy. He has had this problem before after eating too much candy. Do you have any idea what's wrong? Doctor: Hello,Thank you for using Healthcaremagic.I read your question and understood your concern.I think your friend may be allergic from a food , maybe candy ingredients , so he needs to see a allergic disease doctorDr. Selmani"
},
{
"id": 12335,
"tgt": "Suggest treatment for psoriasis on head",
"src": "Patient: My Husbad has Psoriasis on his head and it is like flakes of dandruff on his scalp. He does head bath daily. His face, neck and hands have become very dark due to falling of dandruff but his body is white and this problem he has almost from 3 years. Doctor: Hello. Thanks for writing to us at healthcaremagicScalp psoriasis commonly presents as scaly itchy patches. It is frequently confused with dandruff. Normal antidandruff shampoos are usually not effective in psoriasis.Treatment of scalp psoriasis is based on coal tar and salicylic acid based shampoos. Coal tar is antiinflammatory and inhibits epidermal hyperproliferation that accompanies psoriasis. Salicylic acid is keratolytic and helps removes scaling.Since psoriasis is a steroid responsive condition therefore topical steroid lotions are usually effective and first choice therapy in scalp psoriasis. They can be used alone or in combination with salicylic acid.I would suggest you to visit a dermatologist for a confirmatory diagnosis and appropriate prescription.Regards"
},
{
"id": 74088,
"tgt": "Suggest cause for mild chest pain",
"src": "Patient: Hi, I have a very tiny chest pain in the center maybe slightly right side of my chest that I only feel when I move a certain way. I can take very deep breaths with absolutely no problems at all so it's kind of weird. I have a tiny bit of shortness of breath when I exert myself but that's it. My room mate and I are both getting over viral sickness. I am a smoker of cigarettes and marijuana for a long time however have quite cigarettes about 2 weeks about and quit the pot about a week ago. I have no cough but notice a tiny bit of wheezing but not all the time. Pain started about a week ago. Doctor: HelloAs a smoker better to consult your doctor for a Spirometry test for examining your lungsRegardsDr.Jolanda"
},
{
"id": 192881,
"tgt": "What causes smelly thick puss and blood from my right nipple?",
"src": "Patient: Hi, may I answer your health queries right now ? Please type your query here...im a male and today my right nipple felt really tender,when i sqeazed it lots of smelly thick puss and blood came out and nipple is still tender and dischrgeing small amounts some time later Doctor: HI, I can understand your concern for your symptoms, It can be due to bacterial infection and hormonal imbalance. If it looks severe, you must have to take biopsy. I Hope I have answered your query. If you have further doubts , I would be happy to help you. Happy day"
},
{
"id": 97308,
"tgt": "What is the cause of swelling on the top of my head after a car accident?",
"src": "Patient: Good afternoon. Since my car accident a year ago, I still have swelling on top of my head, like soft spots, which are tender and a headache normally follows. I have been treated for this past year for a concussion by a neurologist. Today she said to take meds for swelling and thats it. I am worried. Doctor: Hi thanks for using HCM.Swelling after a injury could be encapsulated one with remained fluid with in swelling, presence of pus needs to be ruled out. Which needs surgical treatment.Not to worry first cause needs to be evaluated and treatment accordingly.Regards"
},
{
"id": 14766,
"tgt": "Suggest treatment for a body rash",
"src": "Patient: I was working outside and have what looks to be poison Ivy now I have blisters on my tongue should I worry? I've had the body rash for about 5 days now? just spent the day traveling and noticed the blister have been treating with calamine lotion and it helps Doctor: Hello,Thanks for writing to us.As per your case history,you are probably having allergic rash.It will disappear on its own,but will cause itching and discomfort.Use cold water spongings for soothing effect.Use cotton clothes.Avoid soap as far as possible.Continue with calamine.Antihistaminic like allegra 180 mg will help in reducing itching and redness.Take this for 4 to 5 days.Hope this will help you.Take care."
},
{
"id": 77524,
"tgt": "Suggest treatment for chest pain",
"src": "Patient: hi im having chest pain it started 4 days ago with a little sharp pain on the right side and traveled to to the middle and got bigger. now i cant get up without pain in my chest. it feels like weight being put on my chest. when i breathe deep it hurts and when i poke it, it also hurts... i am a young male Doctor: Thanks for your question on Health Care Magic. I can understand your concern.Possibility of musculoskeletal pain is more but better to rule out pleuritic pain first. You are having chest pain associated with breathing. This is characteristic feature of pleuritic pain. Most common cause for this is lung infection. So first get done chest x ray. If chest x ray is normal then no need to worry for pleurisy. Musculoskeletal pain is more likely. So avoid heavyweight lifting and strenuous exercise. Avoid bad postures in sleep. Avoid movements causing pain. Apply warm water pad on affected areas. Take painkiller and muscle relaxant drugs. Don't worry, you will be alright. Hope I have solved your query. I will be happy to help you further. Wish you good health. Thanks."
},
{
"id": 117644,
"tgt": "How to increase hemoglobin level?",
"src": "Patient: Hi sirGood morning My daughter is one and half years old. Her weight is 9.5kg. Last I consult wit her doctor , last one week she s suffering from running nose and severe cough. So doctor ask to test her blood. In that he find that her hemoglobin is very less she s having only 7.8. She s very lazy to have food. Still she have only breast milk no other milk. Sir please advice me. How can I increase her hemoglobin. Is ter any problem wit her health. Waiting for ur fast replyThank you Veena Doctor: continuing the exclusive breast milk till the age of 1.5 years is not advisable. exclusive breast milk is advisable only upto 6 month of age. after that gradually you have to start with the food supplements. low hb is because of this. You should start with the food supplements gradually and than you have to stop the breast feeding to her. this will improve your way. thnx"
},
{
"id": 36555,
"tgt": "What causes fatigue and low grade fever along with body aches?",
"src": "Patient: I have been feeling tired about 2 yrs now. I m 56 & for about 6 weeks now I have been running a low grade temp, my body aches, head hurts & I can t hardly do anything without being so tired that it wipes me out for the rest of the day. I m to my witts end & don t know what to do. The Dr. gave me Celexa, but that doesn t help. Doctor: Thanks for your query at HCM!According to me you are possibly suffering from tuberculosis.You need to confirm it by culture & sensitivity of sputum and X-ray. You might need contrast enhanced CT to establish the diagnosis. IF positive you will require treatment with ATT for 6 months. You will recover from illness.If it is negative for TB you will need investigations to rule out malignancy, auto-immune disease, etc.Visit a physician for examination & investigations.Take care!"
},
{
"id": 78050,
"tgt": "Noticed pain on left side of rib cage & tightness in the esophagus",
"src": "Patient: I have been experiencing an intermittent pain under my left side rib cage for several years. When it happens it seems to last for a few weeks then goes away for a while. I have been bothered by this bout so much that I feel like I can't catch my breath (not because my lungs aren't clear but because it feels like my lung just can't expand completely). I had blood tests yesterday and ruled out pancreatitis. Still waiting for results on the bloodwork for ulcer. I have had an ultrasound during one other bout and they couldn't see anything. I get heartburn occassionally but not regularly. Today, I not only feel the pain under my rib cage but a \"tightness\" in my esophagus and still feel like I can't catch my breath. I am scheduled to see a gastroenterologist but I can't get in for over a month. I am very frustrated because it seems like the symptoms I have don't fall into any obvious diagnosis. Do you think seeing a gastroenterologist is the correct next step? There is no food that I've noticed to aggrivate it. In fact the only thing I notice that aggrivates it is abdominal exercises (like sit ups). Doctor: Thanks for your question on Health Care Magic. I can understand your concern. In my opinion you are having GERD (gastroesophageal reflux disease) and bronchitis. GERD is due to laxity of gastroesophageal sphincter. Because of this the acid of the stomach tends to come up in the esophagus and cause symptoms of Central chest pain, tightness, pressure etc. Reflux causes micro aspiration in the lungs. And this causes bronchitis. Bronchitis manifest as breathing difficulty. So control of GERD is must. And hence gastroenterologist consultation is needed. You should avoid hot and spicy food, stress and tension, junk food. Avoid large meals, instead take frequent small meals. Take proton pump inhibitors and prokinetic drugs. Don't worry, you will be alright. So consult pulmonologist and discuss all these. Hope I have solved your query. I will be happy to help you further. Please hit thanks."
},
{
"id": 204756,
"tgt": "Can Trazadone be taken while on Ambien?",
"src": "Patient: I take paxel for depression,clonozapan for anxiety.recently I replaced ambian for insomnia with trazadone ,which is an antidepressant. I was feeling very chatty and unfocused.Should I be taking two antidepressants? My doctor said yes,the trazadone is for the insomnia and it would be better to stop the ambian.there is definitely conflict with combination of the trazadone and the panel.What do you recommends? Doctor: you can stop ambien if you get enough sleep with trazodone. Trazodone can be continued for its antidepressant effects."
},
{
"id": 124377,
"tgt": "What causes numbness on upper back?",
"src": "Patient: I have a approx. 2 inch by 2 inch area on my upper back, next to my spine on the left side, that has been numb for the last 4 months. I m just wondering if it s something I should be concerned about. There is a mole in that area, but the numb area is bigger than just the mole. Doctor: Hello, The numbness in the area is due to a nerve related problem. It can be due to a mild nerve root compression. In case the numbness is increasing then do consult a neurologist for proper treatment. Hope I have answered your query. Let me know if I can assist you further. Regards, Dr. Praveen Tayal, Orthopaedic Surgeon"
},
{
"id": 34102,
"tgt": "What causes fever along with sore throat,body aches and fatigue?",
"src": "Patient: Good Day, I have been sick for close to a week with what has been flu-like symptoms. I m quite healthy and figured Id power through with fluids, rest, fever reducers, Etc. I have suddenly gone from a 100+ fever to a subnormal 96.5-97.7 and the symptoms (aches,chills,sore throat that feels like Ive gargled broken glass,fatigue, no appetite) have all doubled in intensity since the temperature drop in my body. The hospitals call lines are useless because they cant give medical advice on the phone and I live so far from a hospital that an ER visit for something that will clear up by tomorrow is unfeasible. Could someone possibly give me some advice on if I should pack up and go to the ER? Thank you for your time. EAK Doctor: Thanks for posting you query to health care magic.you are suffering from upper respiratory tract infection. Usually these kind of infections are caused by viruses and sometimes by bacteria.you dont need to worry do following thing:1.take good nutrious dite 2.warm saline gurgle three times a day to relieve soreness in throat.3. take tablet of Cetrizine 10 mg with Monteleucast 10 mg once at night for relieving allerglic reaction.4.tablet paracetamol in adult dose two times a day for body ache.contact immediately to health care settings if complaint aggravated .medicine should be taken under supervision of physician.review me after 5 days of treatment .Hope you would be satisfied with my answer . Feel free to communicate if any query .regards,Dr.Manish PurohitInfectious disease specialist"
},
{
"id": 8267,
"tgt": "Suggest effective fairness cream for dark skin tone",
"src": "Patient: Hello sir i using sungrace sunscreen of glanderma spf40+ but my face is still dark and i also using melaglow cream in night but my face colour is dark please suggest me which 1cream 2 face wash 3 sunscreen is better work on my face to glow and which cream is use to glow my neck and hand skin also Doctor: Hi Dear,Welcome to HCM.Understanding your concern. As per your query you have dark skin tone. Well i would suggest you to wash your face with cold water twice a days with facewash containing benzoyl peroxide , then apply cream containing kojic acid which is natural ingredient , very safe and effective . Also exfoliate your fac twice a week to reove the dead skin and oper the pores of the skin . Use broad spectrum sunscreen with SPF 30 + while in exposure to sun , eat healthy diet including fresh fruits and vegetables and drink plenty of water . If condition doesn't get improved then consult dermatologist for proper examination . Doctor may prescribe retinoids and hydroquinone or may recommend laser therapy or chemical peeling . Hope your concern has been resolved.Get Well Soon.Best Wishes,Dr. Harry Maheshwari"
},
{
"id": 45315,
"tgt": "How long after giving up smoking does fertility come back ?",
"src": "Patient: how long after giving up smoking does fertility come back? I've only been smoking for a year. Just stopped smoking yesterday. So far so good, but my husband and I are trying to have a baby, so just wondering how long it takes for the reversible effects of smoking on fertility to go away. I know some are irreversible, but what about the ones that aren't? Doctor: Hi Amy, Welcome to HealthcareMagic Forum. The return of fertility will vary from person to person.You should have no nicotine at all in your system during pregnancy, since nicotine can constricts your blood vessels including the ones to the placenta and the baby.Smoking and secondhand smoke can increase the risk for miscarriage, birth defects, and other complications of pregnancy. You need to be free from smoking for atleast a month before trying to conceive. Take care"
},
{
"id": 211962,
"tgt": "Sudden silence, stumbling for words, confusion, blank mind. Have taken many medicines. No relief. MRI, xray normal. Suggestions?",
"src": "Patient: happy gud noon doctor.....i am Shivani..age 24...i have some mental problem...now i am consulting government mental hospital doctors..i m a middle class ..single..Now i am taking Fluoxetine Capsules. i itself avoided sleeping tab . i will sleep normally but not a gud sleep.i searched in net this tab would be taken in condition as major depression my problem's are 1. most of the time's in silent.2. cant able to speak normally. (i know 1ly tamil lagu but it itself i have more difficult to put words in correct situations). blank mind. while speaking in between times will be silent in myself and thinking what i am saying or telling to opposite person. i itself ask them ' what i am speaking or telling now\"..? 3. unknowingly sudden tear's came.4. Unknown Fear5. chest pain6. Headachemy family friend said please go to consult Dr. rangarajan Psychiatrist in chennai. i dnt have a money to spent. i dnt want to take medicine's too. i took more medicine's with in 3years..they took mri eeg ecg xray everything will be normal.. now i m doing yoga too... but they itself told please go to doctor.above conditions are there in 2010 onwards... i went psychiatrist and neurologist Dr. Sekar in chennai he 1ly said took mri (headache) den he gave me more medicines that time i dnt now to ask for what i will take this and that?...he just told me ''u must take these medicines otherwise u will be more trouble in future(operation)\".s my headache will be gone...other neurologist government doctor said 'u no need to take this medicines at this age' each day 10 to 15 tabs...i spent more money to him also...why they are like this????????? am i a rat???? or what???? now i have 1ly little problem (rarely silent in myself, block in my chest, unbelievable dreams (so that in my wokeup time i will be more tired)...i am confused a lot....Please give me a correct solution for my problem.....!!!!(without medicine)i think can i do my yoga practices it itself OK or not? Doctor: Hi,Thanks for using healthcare magic.I have gone through your available history.You are having major depressive disorder. Physical causes like thyroid dysfunction, vitamin B 12 deficiency or anemia should also be rule out.You may get benefit from medications as well as as counselling.Fluoxetine is very good, potent antidepressant drug. Usual dose of it is 20-80 mg per day. If taken under proper supervision, it should not harm you in future or you may not get addicted to it also.You should also continue to do yoga, stress management and relaxation therapy like breathing exercise.I hope It will help you.Thanks."
},
{
"id": 139855,
"tgt": "Suggest medication for treating dementia",
"src": "Patient: SIR MY MOTHER IS 80 YEARS OLD AND SUFFERING FROM DEMENTIA SINCE 15 MONTHS BUT RIGHT NOW SHE IS NOT COOPERATING US AND CONTINUOUSLY SPECKING AND DO NOT TAKING PROPER SLEEP THOUGH HER MEDICINES LIKE ADMANTA,STROCITPLUS,AND FOR BLOOD PRESSURE NUDON 5 IS CONTINUE NOW WHAT OTHER MEDICINES SHOULD GIVE Doctor: Hello,It is necessary starting anti-psychotic therapy. In this regard, quetiapine or risperidone would be possible treatment options, which would help improve her behaviour and also her sleep.Hope I have answered your query. Let me know if I can assist you further. Regards, Dr. Ilir Sharka, Cardiologist"
},
{
"id": 102454,
"tgt": "Is asthavent, mucospect and linctospect suitable for a 3 years old child with asthma and bronchitis?",
"src": "Patient: hi my daughter is 3 yrs old having asthma and bronchitis and she currently taking chronic medication foxair and singular sprinkler. For the past five night she is coughing throughout the night and the pharmacist gave me Asthavent, mucospect and linctospect can i give them to her? Doctor: Hello,As your daughter is having asthma and bronchitis for which your doctor as prescribed the drug to control the symptoms as cough as increased in past five days.You can give these medicine given by your pharmacist which helps to relieve the bronco constriction, take out the mucus from the lung and reduces the cough.Inform your doctor regarding the medicines which you have given to your daughter.Thank You."
},
{
"id": 125628,
"tgt": "Should a doctor be consulted for swelling in the ankle post injury?",
"src": "Patient: A had an incident take place on Thursday ! I hurt my ankle pretty bad , I even heard a popping noise as my ankle was being injured. It s been a few days and my swelling as not gone down . I have a swollen ankle and was wondering if I should see a doctor Doctor: Hello, I just have read your problem. As you said you heard a popping sound when you get injured. There may be possibilities of tendon tear or muscle rupture or even fracture or a mild fracture in your ankle. As swelling is still persisting. See if your joint is movable?Are you able to walk properly?Do you feel intense pain while walking and you feel to discontinue to walk?Does pain settles when you take some rest?I would suggest you to see a good ortho in your area. You need to get your X-Ray done, then only the clear picture will comeI hope this helps. Wish you a speedy recoveryThanks"
},
{
"id": 112071,
"tgt": "What can bring relief to soreness and pain in back and spine?",
"src": "Patient: hi I am just recovering from the flu and I had a very sore back just at the bottom of the spine area. now the flu and back pain has gone but I have very sensitive skin around the lower back upper buttocks and central stomach area. would you have any ideas what this is? no rash or other symptoms. Doctor: Hello. Thanks for writing to us. The soreness and pain in the lower spine can best be relieved by an improvised posture, regular physiotherapy and back strengthening exercises, aerobics and swimming in addition to the medicines. I hope this information has been both informative and helpful for you. Regards, Dr. Praveen Tayal ,drtayal72@gmail.com"
},
{
"id": 49852,
"tgt": "Unwell, thirsty, frequent urination. Blood count, sugar level, urine sample, normal. O rh d positive. Suggestion?",
"src": "Patient: Hi I have been feeling very unwell lately tired all the time just want to sleep.. very sore back feels like my kidneys.. bad heaches only in the evenings thou.. and thirsty all time I need to urinate every 20-30 mins... ive had a full blood count done sugar levels and urine sample they have all came back normal wat is wrong with me? Also I am o rh d postive with anti cw antibodies and a cold antibody don't no wat that is but I have to carry a card incase I need a transfuaion in an emergency thank u Doctor: Hi, If your blood count, sugar levels and urine sample were resulted normal, then, diabetes insipidus is suspected as you are having extreme thirst and frequent urination. I would suggest to consult an endocrinologist to perform some tests to diagnose diabetes insipidus and take proper treatment. Hope it helped!"
},
{
"id": 179606,
"tgt": "How are adenoid treated in children?",
"src": "Patient: My daughter is 4 years old suffering from adenoid as she is breathing from mouth while sleeping we went for xray and found adenoid was developed, whats your suggestion since few doctors advising surgery and other are advice wait...whats your suggestions. since she is very young is there any complications in future if we go for surgery... Doctor: HiThanks for writing to health care magic.Try Nasal steroid sprays like mometasone dailySometimes adenoids shrink with nasal spraysRestrict sweets, chocolatesWishing your child good healthRegardsDr Arun"
},
{
"id": 205888,
"tgt": "Is asperger, related with difficulty in social interaction ,sensitive ears and shaky hands?",
"src": "Patient: Hi, I have several symptoms about a family member from that started from birth that I would like to know the diagnostics for. Her ears have has always been extra sensitive to load noises but is more tolerated now. Around age 1 she used to bite her fist when she got excited. We got her to stop that but she started to shake her hands widely instead of biting fist and at her now age of 9 she still shakes her hands widely when she is excited. She was not very sociable until she started school. She is mentally a normal child. Whatever she has is very mild and getting better with age. Do you think her symptoms are related to Asperger? Doctor: DearWe understand your concernsI went through your details. I suggest you not to worry much. Asperger syndrome (AS) is an autism spectrum disorder (ASD) that is characterized by significant difficulties in social interaction and nonverbal communication, alongside restricted and repetitive patterns of behavior and interests. The social interaction should come naturally after some time. Given situation is not that bad. Shaking of hands, if it is really a problem, should be analyzed. For that consulting a neurologist should help you.I can help you in this aspect. Psychotherapy techniques should suit your requirement. If you require more of my help in this aspect, Please post a direct question to me in this URL. http://goo.gl/aYW2pR. Make sure that you include every minute details possible. I shall prescribe the needed psychotherapy techniques.Hope this answers your query. Available for further clarifications.Good luck."
},
{
"id": 104868,
"tgt": "Suffering with asthma, have cough during winter, dust allergy in summer. Effective remedy?",
"src": "Patient: Hello doctor, My father is suffering from asthama for about 10 years. My father s current age is 55. My late grand father also suffered from asthama. In cold season asthama is trigger at night and father can t walk long distance and trouble in steps up stair. he always suffer from cough in cold season and in summer season he has allergy from dust, smoke . Please give your suggestion and suggest some medicine, and hospital name for asthama treatment in India. Doctor: Hi, Your father seems to be suffering from bronchial asthma. You need to get him evaluated by a Pulmonologist/ Chest Physician. Investigations that may be required are chest xray, serum IgE levels and pulmonary function test in consultation with the Pulmonologist. Depending on the evaluation results he may be prescribed certain inhalers and oral medications along with a nasal spray. Also he is a candidate for yearly flu vaccination and 5 yearly pneumococcal vaccination. Depending on your location in India, you may find a pulmonologist. A list is availbale on this website: link: http://www.healthcaremagic.com/doctors/Pulmonologists-in-India/L18860S37 You may search as per your location. Regards Dr. Gyanshankar Mishra MBBS MD DNB Consultant Pulmonologist"
},
{
"id": 106909,
"tgt": "What causes sharp pain in the back upon bending?",
"src": "Patient: last night while lying in bed I rolled over and there was a sharp pain in my back that went from my bottom rib next to my spine and out towards my left side. it hurt so bad it took my breath away. but breathing doesn't hurt it. neither does coughing. it feels like a bruised or cracked rib. but there was no trauma and it only hurts when I lean forward to pick something up with my left hand or when lying down and try to roll over. Doctor: Welcome to HealthcareMagic i think that you are confusing symptoms as the rib fracture occurs with trauma or violent cough and cough will cause pain .you are having problem in spine so go for xray lumbar spine ap and lateral view ...MRI lumbar spine will be a better option than xray .Get these done as soon as possible .till the investigation you may take analgesic as tab aceclofenic plus chlorzoxone two times .Avoid excessive bending of spine and lie on hard bed .If you have any further query then you can come back to HealthcareMagic i will definitely help you and guide you appropriately With regards Dr Varinder Joshi"
},
{
"id": 112092,
"tgt": "Is it normal to have constant back pain even after removing cyst and left fallopian tube?",
"src": "Patient: Hi,I had laproscopic surgery to remove left fallopian tube and two cyst right and tube. I still have lower back pain it has been two months since surgery. I thought having the cysts out will give me relieve still have sharp and dull pain in my lower back and hip. Is this normal? Will I get better soon can't stand too long or sit or laydown for a long time?I thought it was because I was lithotomy positon for long time during surgery? Doctor: HelloThanks for writing to us with your health concern.Yes, prolonged lithotomy posiiton can cause back pain later.However, laparoscopic surgery has lesser incidence of post operative back pain, because spinal anesthesia is not given for the same.Please consult an Orthopedic surgeon.You might be taught physiotherapy and postural stretching exercises to alleviate back pain.ALso, mild painkillers might be temporarily taken.Imaging should be done to rule out any actual lesions - prolapsed disc , disc swelling etc.Get your calcium and Vitamin D levels checked - deficiency can cause back pain.Do not delay investigation and treatment for long,Take care."
},
{
"id": 127710,
"tgt": "How often should Ibuprofen be taken for left-sided shoulder pain?",
"src": "Patient: My left shoulder/neck area has been hurting for about 3 weeks. It feels like I ve slept on it wrong. Some of the pain feels like a tight muscle (the neck area) and some is lower in the shoulder, beneath and at the bottom of the shoulder blade. Taking ibuprofen lessens the pain, but I don t like to take it often. Neither heat nor ice seems to affect the pain. Doctor: Hello,You have got muscle spasm in your neck and only ibuprofen will not help. Muscle relaxants like thiocolchicoside are also needed with NSAIDS. Ibuprofen can be taken thrice a day. Avoid jerky movements of the neck. Use soft pillow of low height just to support the head. Local application of diclofenac gel is also needed. If not relieved within a week visit to an orthopedic specialist is needed.Hope I have answered your query. Let me know if I can assist you further.Regards,Dr. Jayesh Vaza"
},
{
"id": 163867,
"tgt": "Suggest remedy to stop yellow colored motion in infant",
"src": "Patient: Hi, may I answer your health queries right now ? Please type your query here... mr doctor i want to know about my 3 month baby motion discharge yello colour last 12days i have been given medicine from children specialist but there is not stoping motion pls sugest me thanks Doctor: Yellow color stools is a normal finding in children. There is no need to give any medicine for this."
},
{
"id": 201961,
"tgt": "How to treat premature ejaculation?",
"src": "Patient: have a problem of quick release of sperm at the time of sex with my wife. I am 25 years old. Height 5 10 and weight 65. I am very frustrated right now for this problem every time. At the time of sex when she touches my penis or sucks my penis, at that moment automatically my sperm is releasing. So please help me. I can t do sex also with her for this problem. Before starting sex only my sperm is releasing. I have acidity problem and asthma also. now I need a very fast recovery from this problem to do a happy sex with her next time. Doctor: Premature ejaculation causes emotional and relationship distress. There are several ways to treat Premature ejaculation. But in your case i would suggest you to take Dapoxetine 30mg tablets daily at 7 pm for 10 days as a course, later if necessary you can take them 2 hrs before intercourse. Along with the tablet try some local analgesic creams like PRILOX. You have to apply it to your Penis 10 mins before Intercourse and wash it off after 5 minutes. Otherwise it can numb your partners vaginal mucosa. If this is not working Take 50mg sildenafil tablets along with dapoxetine.There are several other techniques like Stop and squeez, but they take long time to show benefits"
},
{
"id": 80798,
"tgt": "Is the mild pain in chest normal?",
"src": "Patient: I was diagnosed with a pulmonary embolism in October. I am on warfarin and as of November 10th my clots are gone. It is believed to be a result of smoking and birth control. Here I am 3 months later and I still have pain in my chest. The initial doctor told me I had an infarction and the second er doctor told me I did not have an infarction but he did see some thickening of the lung tissue in my left lung where the large clot was. Is the mild pain in chest normal? Doctor: Thanks for your question on HCM. I can understand your situation and problem. Mild chest pain in pulmonary embolism patient is mostly due to infarction. Since you have rule out infarction, it is mostly due to local thickening of pleura at the area of embolism and infarction. So no need to worry much for this mild chest pain as you have ruled out Infarction. But if pain worsens then get done repeat CT pulmonary angiography to rule out relapse of embolism and Infarction. At present, no need to worry."
},
{
"id": 213577,
"tgt": "Not interested in study, tensed and confused",
"src": "Patient: sir,i am an guy of 25 age.i am trying to complete BE in haliyal college from last 6 year and still unable to complete.now now i have lost interest in studying and every thing. i am very very tensed and confused about my life.our family is middle class and dont no what to do now and where to go.pls doctor atleast you show me way Doctor: Hi Raghavendra, Welcome to Healthcare Magic and thanks for your question... I'm sorry to hear that you have been having a difficult time, esp. with regards to completing your studies. Firstly, the very fact that you have realized that you are having difficulties coping up and have seeked help means that you have taken the first step in solving your problems. I understand that it can be quite dejecting to keep failing repeatedly in something and sometimes this can give an impression that all is lost and that there is no hope. And that feeling can often sap our confidence and leave us with a negative view about anything and everything. It's like seeing the future wearing dark glasses... everything will appear only dark! What I usually advise my patients is to look only at the 'here and now' and not keep brooding about the past or using it as a yardstick to predict the future. So, firstly get yourself together and say: Okay, this is the situation I am in... it may be a difficult situation... but what I'm going to do is look forward and find ways to solve this... rather than saying: I have failed so many times before, so I don't think I can succeed. This attitude will give you enough confidence to face any problems that may come your way and if your confidence is high, then your performance is also going to be better. Also, try to avoid any stressors which may be preoccupying you and hindering your ability to concentrate. It is also important to talk to someone when you're feeling down and low - it may be a relative or close friend or if necessary, a counsellor or a psychiatrist. I'm sure you'll do well. All the best... - Dr. Jonas Sundarakumar Consultant Psychiatrist"
},
{
"id": 40918,
"tgt": "Suggest remedy to get pregnant as i have PCOS",
"src": "Patient: I have been married in the year 2013, and for last one year trying to get pregnant. My age is now 35 and I have PCOS, for which I suffered from headache, nausea, bulging of stomach, pressure in lower abdomen and acute pain in lower abdomen during periods. Please help. Doctor: Hi, Thanks for posting in HCM. I understand your concern. Polycystic Ovarian Disease (PCOD) is a treatable disorder caused due to alteration in hormones leading to irregular menstruation and untimely ovulation, resulting in infertility. You need to be treated for getting your cycles regular by use of medications like birth control pills. You may also need supplementation of ovulation inducing drugs like metformin and clomifine along with. Along with it, you need to reduce weight by means of intake low calorie foods and moderate exercise. Kindly get a regular follow-up done and follow advise accordingly. Have patience as it may take some time to improve the chances of conceiving.Hope the information provided would be helpful. All the best."
},
{
"id": 15874,
"tgt": "Rash on skin. Used betnovate. No pigmentation, white patches. This permanent?",
"src": "Patient: Hi I have been using Betnovate 1/5 cream on my hands and arms for approximately 4 years to treat an eczema/ psoriasis type rash. I have noticed resently that the pigmentation has been stripped in those spots and I now have white patches. I have stopped using the cream and the rash seems to have improved but not disappeared. Will the pigment regenerate or is the damage permanent? I am a 35 year old female in good health. Thankyou for your assistance. Doctor: Hi thanks for writing here.Betnovate is a steroidal preparation which is used for treating eczemas,it is a potent steroid and has to be applied under the observation of a dermatologist. common side effects are skin atrophy,secondary bacterial infections like bacterial or fungal,loss pigment leading to white patches,steroid induced acne etc.Based on your history i can assume that you have got loss of pigment,stop applying cream now apply placentrex gel two to three times over affected part,which will help faster repigmentation.TAke care."
},
{
"id": 114956,
"tgt": "Suggest treatment for lymphadema other than wearing high compression stockings",
"src": "Patient: I wear high compression stockings 24/7 for lymphadema since 2000. Is there any new treatments out there that I could try that are not painful when the stockings gather or they stockings slide down or gather at certain places. It is very painful since I cannot keep these stockings from gathering in certain places. I have tried everythine to keep the stockings from doing this and it is very painful and I get minor infections from this. Please help.sincerely, Rosemary Hull. calicoarrts@ yahoo .com Doctor: Hi rosemary, i can understand your concern and pain. I would advice you to keep your legs raised and keep doing foot exercise to aide in your venous return, a pillow or two below your leg area of bed before you sleep is a good idea. also stockings do help especially when you are standingapart from this there are surgical manuovers that many surgeons perform to aid in proper drainage or remove excessive skin and tissue and fascia.you need to consult a surgeon for that especially one who has done lot of surgeries for filariasis.i hope this helpsdo let me know if you have any more questions for me."
},
{
"id": 6565,
"tgt": "Does thick white vaginal discharge indicate that I am nearing ovulation date ?",
"src": "Patient: i got my periods on july 24th 2011, today is 30th july,mine is a 28 dayz cycle but now doctor says make it 30days.....now i am getting thick white discharge from morning little frequent. am i nearing ovulation?? Doctor: Hi,Shehnaz, Thanks for query, Ovulation time is not possible as it is too early. You might have some infection in genital tract. Go for vaginal swab examination and after report go fo treatment accordingly. Take plenty of water. Ok and bye."
},
{
"id": 126393,
"tgt": "Suggest treatment for fluid retention in the feet",
"src": "Patient: My brother-in-law is extrememly overweight, with very very swollen feet that appear shiny. He also has neuropathy and sometimes cannot even walk. He seems very depressed about his weight. He does have some heart problem but has not seen a cardioligst. The dr. wants an mri with a camera and he did not fit in the closed one. Because of his feeling very badly and icy roads he missed the open mri. This is thru VA. Dr. will not see him until he has the MRI. My sister wants to know if she should take him to the ER because of the shinyness on the feet which is a very new symptom. thank you. They do not have a computer. Doctor: Hi, Swelling could be of many reasons like hypothyroidism, kidney disorder, cardiac disorder. So you should not ignore this recent development and should consult the doctor urgently. I would like to advise you that there is one medicine available in market which increases your BMR and helps you loosing weight. many of my patients are benefited from that. Hope I have answered your query. Let me know if I can assist you further. Regards, Dr. Anuj Gupta, Spine Surgeon"
},
{
"id": 224608,
"tgt": "On ortho tri-cyclen lo. Missed pill and delayed periods. When to start the next cycle?",
"src": "Patient: I ve been on birth control ( ortho tri-cyclen lo) for 5 years. I m 22. I did not take pills the past month (did not have the pills available until two days into week 2 of active pills) because the doctor told me not to with the delay and was instructed to start the Sunday after my period. I should have started my period this week (it s now Friday) but it has not happened. Can I still start the new cycle of pills on Sunday to re-regulate things? Doctor: HelloThanks for writing to us with your health concern.How many pills have you missed exactly ?It is not clear from your question.If you have missed more than 3 pills, then do not start a new pack.Wait for your period, use back up protection, and start a fresh pack from day 2 of your periods.Take care."
},
{
"id": 42637,
"tgt": "Any treatment after failed consecutive IUI to conceive ?",
"src": "Patient: hi i am 30 years old and trying for a baby since 1 year.My and my husbands all reports are normal . including sperm test and my fallopin tubes. i tried for 2 iui for 2 concecutive months. but both failed. i dont understand what should i do further and what is hte reason for not conceiving. i was suggested to use aloes compund table by my ayurvedic doctor. should i go ahead with that . please sugeest. should i go for any other treatment. someone suggested to get ur uterus clean. should i try that ? Doctor: Dear member,Thanks for writing.I can understand your anxiety.The chance if pregnancy in normal husbandand wife us only 15% per cycle.So in a treatment cycle like IUI the chance of success is only 15% You have to peruse for atleast 5-6 cycles to say failure of IUI.Please send scanned details of reports and semen analysis. I may be able to suggest some changes.If treatment has been following in right manner then I would strongly recommend not to change even though there is failure.Please keep faith. You will conceive by 5-6 cycles as I have noticed 70- 80% success is seen by the end of 5-6 cycles in my clinical practice.Please write or call back to DR BHAGYASHREE I can surely help.ThanksDr Bhagyashree"
},
{
"id": 36755,
"tgt": "Suggest treatment for water infection",
"src": "Patient: Hi there I had a water infection a couple if weeks ago and gave me some tablets for it then went bk to doctors and it was still not fully gone so they gave me another set of tablets and now they are saying my wee is over the boarder line what does this mean? Anything to worry about. Doctor: Hello,I understand your concern.I am Dr. Arun Tank, infectious diseases specialist, answering your query.In my opinion you should do stool culture and sensitivity.As you had undergone treatment trial for two times, but haven't received any improvement. So there is possibility of resistant organisms growing in the intestine.Once you do culture and sensitivity testing, report will be available and we can take the drug accordingly. This will give you a relief very instantly.I will be happy to answer your further concern, you can ask me on bit.ly/DrArun. Thank you.Dr Arun TankInfectious diseases specialist."
},
{
"id": 112104,
"tgt": "Should i meet doctor after having severe back pain?",
"src": "Patient: I was sitting on recliner last night and I started getting severe pain in my lower back. It's hard to sit walk and even coughing the pain is severe. Today it's worse and I can't even walk, sit or stand without the severe pain.Should I be going to the ER? Doctor: Hi thereBack pain of such severity warrants immediate medical attention.Firstly get an Xray of the affected part.Get expert opinion from an orthopedic surgeon after xrayFor the time being , complete bed rest alon with a pain killer and muscle relaxant like Flexispaz and tramodol should suffice.You can also apply hot water bottle along with a local gel like Oxalgin nanogel massage.Good luck"
},
{
"id": 27198,
"tgt": "What is the cause of high blood pressure?",
"src": "Patient: This am my bld preasure was 149 73 66 Some time I awake with high bld preasure other time 129 55 72,,, I change to normal & high & lows that scare me I had a 98.45. 72 awhile back why should there be such radical changes? I'm 87 yrs old exercise walking a dog ..sometimes walk at a good clip & slow down wt 125 5'2\" Diabetic controled 5.7 tp 6.4 AC1 What is normal? My cardiologist thinks it's OK BUT THE READINGS SCARE ME Doctor: If you have long standing diabetic even if well controlled and also age related , what happens is your nerves get weak over time what we call autonomic dysfunction, where there would be fluctuations in the BP. Till you don't have symptoms don't worry about the fluctuations. However I would personally prefer your BP on the higher range like between 135-150 and lower 80-90, secondly don't suddenly change position and things should be all fine. Regards Dr Priyank Mody, Cardiologist"
},
{
"id": 34230,
"tgt": "What are the symptoms of mrsa?",
"src": "Patient: I am a 25 y/o F w/ PMH of strep in 6/04 and bronchitis in 93. On Tuesday, I was contact w/ a patient with MRSA, and then on Thurdsay (2 days later) I ended with fever, chills, swollen lymph nodes, and white bumps on my tonsils. The doctor said I had strep throat, but i was now wondering if an MRSA cause white bumps on the back of the throat. Doctor: Thanks for posting you query to health care magic.MRSA infection can cause symptom alike your symptom but you need to first investigated to confirm diagnosis of MRSA and then take treatment for it as Streptococcal infection can produce same illness.you should be investigated for :1.Throat swab for culture and sensitivity2.Blood culture and sensitivity.these two test will help you in find out MRSA and effective antibiotic to treat it .review me after investigation report then i will suggest you complete treatment .Hope you would be satisfied with my answer . Feel free to communicate if any query .regards,Dr.Manish PurohitInfectious disease specialist"
},
{
"id": 162886,
"tgt": "What causes vomiting and breathing issues?",
"src": "Patient: hello, my friends son has been throwing up for 3 days and has been having breathing issues.. he was taken to the local ER this morning and then admitted to the hospital for more tests.. do you have any suggestions as to what might be going on with him? Doctor: Hello and Welcome to \u2018Ask A Doctor\u2019 service. I have reviewed your query and here is my advice. He might be having pneumonia. For this he will need Chest X ray and some blood test anf doctors will give antibiotics. You didn't mention the age of the child. If I know age then can narrow down the diagnosis. Hope I have answered your query. Let me know if I can assist you further."
},
{
"id": 47819,
"tgt": "How to increase appetite after kidney transplantation?",
"src": "Patient: my father is 8o year old and he is kidney transplant and he also blood pressure and diabetics he takes insuline ,my question is last few day he takes food very little quantity and he feels very weakness , what can i do , my father came in normal routine. i am appreciated if u tell me solution thanks Doctor: Hi,I understand your concern.I would appreciate if you could mention the details regarding drug treatment,test reports.blood sugar levels.As loss of appetite with generalised weakness in a kidney transplant patient may be a presenting symptom ranging from a simple upset stomach to uncontrolled blood sugar levels,serious infection etc"
},
{
"id": 58534,
"tgt": "Health check up showed elevated SGPT and SGOT levels. Had wisdom teeth taken out and was on medication",
"src": "Patient: Hi, My name is Ravinder. My company has doen a health checkup wherein the SGPT level is 191 and SGOT level is 149, Bilirubin is 0.6. Last year my SGPT level was 144 and SGOT was 97. I have taken out my wisdom tooth 15 days back and was on medication for 10 days. Kindly suggest what preventive measures should i take and how to go further. Doctor: Hi, it appears that you had medicines after removal of the wisdom tooth are hepato toxic, the other reason may be you might be having hepatitis B, so i advise you to consult a gastro enterologist for diagnosis and treatment. Thank you."
},
{
"id": 166385,
"tgt": "What causes purple mottle on the body?",
"src": "Patient: My four year old daughter has intermittent purple mottling of her skin over most of her body. she also has blue veins that are very noticable on her temple and along her jawline. She had a rigourous course of antibiotics at 5 weeks old for viral meningitis that I was told would be hard on her veins. Doctor: this is normal in this age no need to worry give her more fluid /water intake should be adequate as not drinking sufficient water can cause this"
},
{
"id": 194003,
"tgt": "What is the seed like thing on the shaft of the penis?",
"src": "Patient: Hi. I have a thing on the shaft of my penis. It looks a bit like a sesame seed and it's been there forever, it looks like its starting to come out but I dont want to touch it. Theres 3 of them I think, all in the same spot. The top of it is like a brownish color and inside it looks almost white. I'm not sure what to do Doctor: Hello, As per your history, it may be due to smegma. As it's asymptomatic so no need of any active management. In case of bleeding you may require laser surgical treatment after consultation. Maintain proper hygiene. Avoid harsh cosmetic products. Use plain water for washing. Hope I have answered your query. Let me know if I can assist you further. Regards, Dr. Shyam B. Kale, General & Family Physician"
},
{
"id": 103571,
"tgt": "Child with persistent swelling on chin with redness followed by swelling on forehead, under the eyes with tiredness. No improvement with Zyrtec. What is the cause?",
"src": "Patient: my 6 year old woke 2 days ago with a red forehead and chin . by that night it looked slightly swollen so we took him to the doctors. he said it looked like a reaction to something and gave him claratyne to take. today the redness is worse and swelling is too. his forehead nose and under eyes swollen. back to the doctors we went and he told us to take Zyrtec. He has had two doses of that and tonight still looks like the swelling is getting worse? Is this just a reaction to something or is it something else. He is showing no other symptoms but being a little tired Doctor: it is acute sinusitis which is allergicactually this is coomon to develop in children any timeit is mostly milk induced as the child ned milk for 2 years only breast and after that no milkas we add milk and diary in diet these causes allergy and vaious diseases as they are animal proteins incompetible with hbuman proteinsfor time add metronidazole 200 mg bd for 5 days as it clear sinuses quicklyadd antiallergic fexofenadine 30 mg bdsyp cpm 1 tsf nightapply local antibiotic mixed with antiallergic ointment in affected partsadd liquid antacidmefanic acid sos for pan or feverwithdraw milk and diary from diet for next some years"
},
{
"id": 200920,
"tgt": "What do red dots under penis foreskin indicate?",
"src": "Patient: The area underneath my foreskin has red dots. The red dot area once pulled back just looks like inflamed tiny blood vessels with no bumps? This has happened twice after oral sex. But there is also a possibility it may have appeared after not properly cleaning up after masturbation? Could it be a yeast infection or just inflammation/friction from the oral sex? Doctor: Thanks for asking in healthcaremagic forumIn short: If its Fordyce spots(white spots) then need not worryExplanation: Fordyce spots are visible sebaceous glands which are harmless. As you thought allergy to semen/not cleaning up of smegma can cause allergy and red spots like this. For time being you can clean your glans and wait for 2-3 days. If it does not subside then visit a doctor."
},
{
"id": 20729,
"tgt": "How to confirm artery blockage before surgery?",
"src": "Patient: Hi, may I answer your health queries right now ? Please type your query here...my brother is advised to do the bypass surgery by next month. He has a 90% block in his artery.Another block was there in a major vessel . But it was cleared by an angeoplasty. My question is that before doing the surgery will the DR do any test to cnfirm that whether the blocks are still there or not.So many people are praying for him.So we hope that God might have done the mirracle.Except angeogram any other test is there to cnfirm the block. Doctor: hello Thanks for posting at HCM According to your description, he has critical blockages in the heart of which one has been stented earlier. I suppose more than one artery is involved (or the left main artery is involved) that's why your doctor has asked to go for a bypass. There are other non invasive methods like CT angiography to detect blockages in the heart. But of all that an angiography is the most important and gives the maximum information. No doctor will operate on a patient without doing an angiography at first. Further, a bypass surgery also has good results these days. It will take about 7- 10 days stay in the hospital and after that a month for recovery at home. But the long term effects of bypass are very good and he may not require another procedure for next 10-15 years. He has got significant blockages in the heart. So please don't go searching after other options and go ahead with the bypass surgery. Thanks for posting here.Regards"
},
{
"id": 191449,
"tgt": "Is an internal doctor advisable for checking A1C level for diabetes?",
"src": "Patient: I am 68 years old and need a primary care doctor. Is an internal dr a good choice ? I m needing to check my AC1 for diabetes...also thyroid....haven t had a doctor in MANY years. Had a lumpectomy Nov 2013 w radiation and have been taking estrogen blocker. All is well with oncologist. Now have lab work twice year, surgeon once a year. Just know I have a lot to have checked and finding the right primary care is hard for me. Doctor: Hi, Thanks for your question.Hba1c can be advised by your internal doctor or family physician if Diabetologist not available.Its is advisable to get checked once 3 months and modify the dose of anti diabetic medicines accordingly."
},
{
"id": 144191,
"tgt": "Is shoulder pain and breathing difficulty normal with spinal tumor?",
"src": "Patient: I have a spinal tumor on my L5-- now I have horrible pain under my shoulder blade making it hard to breath. I have 2 mris next week but until then I want to know what this could be.. I haven t done anything strenuous what so ever so I am completely confused.L la Doctor: Hi, I am Dr.Bruno. I have read your question and understand your concerns. Let me try to help you Question : Is shoulder pain and breathing difficulty normal with spinal tumor?Answer : No. L5 Spinal Tumour Will NOT cause Shoulder Pain and Breathing DifficultyThe Shoulder Pain and Breathing Difficulty are due to some other problem, most likely a problem in Cervical SpineI would suggest you to consult a Neurosurgeon at the earliest Hope you found the answer helpful.If you need any clarification / have doubts / have additional questions / have follow up questions, then please do not hesitate in asking again. I will be happy to answer your questions.Let me know if I can assist you further.Take care."
},
{
"id": 177339,
"tgt": "What causes red patches on torso?",
"src": "Patient: My 11 month old son had some red blotchiness on his torso and back of his neck yesterday that seems to have spread to his arms and legs now and up his face. What could be causing this? He doesn t have a fever and it doesn t seem to be itchy. How long do I wait to see if it just goes away? Doctor: Hello. I just read through your question. This is either a harmless viral rash, or a reaction to something. Following it's course over the next 2-3 days is sufficient. If it continues to spread, you should consult your doctor. If not, you can leave it be. In either case, it is not serious."
},
{
"id": 8670,
"tgt": "Most efficient and permanent way to remove body hair at home?",
"src": "Patient: I am a 54 year old male. I have always been very shy especially about my body hair and I want to know what is currently the best/most efficient and permanent way to remove body hair, preferably at home? I know that Professor Angela Christiano of Columbia University in New York was working on a hairless gene , and was in the process of developing a topical solution to stop hair growth, but there hasn t been any information/update that I can find. Doctor: hello.i am sorry, but i haven't heard of any topical solution to permanently stop hair growth, from recent literature.LP Nd.YAG or Diode LASERs would help you, though you would require multiple long sessions."
},
{
"id": 21633,
"tgt": "How to treat elevated heart beat?",
"src": "Patient: My friend: Male, 44 yrs old, 6 2 , 185 lbs. normal BP & temp. smoker: pack.5/day. (I think he has rapid heartbeat but I m not in medical field.) Has rales and rhonchi. Problem: Has what appears to be pitting edema? in one mid-calf. Sudden onset 3 weeks ago as he was walking 5 miles and hadn t walked that much in a while. Had cramping at that time. Worse when he points his toes. Gets occassional cramping still. Swelling in ankle/calf/maybe upper foot. Swelling hasn t resided and I can see a small red spot with inconsistent edges that is warm but not hot to the touch. When he wears a sock the swelling is reduced within the sock area so then the calf hangs over where sock was. Has what I would call a cankle . He also has had for a long time small varicose veins, all little ones all over and around both ankles. Some blue but all small and skin a bit red poched on both ankles/lower calves. How does he get rid of swelling and is the red spot an infection? Doctor: Hi ThereAfter reading the details provided by you, i understand your concern and i want to tell you that the condition of your friend doesn't seems to look good. The symptoms you are telling goes in favor of PAD (peripheral artery disease) he probably have developed Deep Vein Thrombosis (DVT) as he had a history of small varicose veins and now he is having CLAUDICATIONS while walking.Now i would like to advice to consult personally a vascular surgeon as well as a cardiologist for BILATERAL LOWER LIMB DOPPLER and ECHOCARDIOGRAPHY, as smoking increases the risk of MYOCARDIAL INFARCTION also. Most Importantly he needs to STOP SMOKING IMMEDIATELY. He needs medical treatment to get rid of the swelling. No its not an infection.I wish him good luck"
},
{
"id": 162345,
"tgt": "How can high fever and cold in a child despite taking Crocin and Azivent be treated?",
"src": "Patient: Hi, my 4yr old daughter , who weights 11.3kg has high fever with cold from past 2days. She had a fever session 10-12days back which went down with paracetamol and cefixime. Today i took her to paediatrician he suggested crocin 240 , wikoril and azithromicin(azivent) . I gave her azivent also crocin with 4hrs interval. But fever is not going down it went to 104 then still at 101.3. I called dr he suggested meftal p. As i just gave crocin can i give meftal without 4hrs gap?. And suggest me good treatment of her fever please. Thanks Shabana Doctor: hi! your history demands few more questions.is the fever is high grade , having any pattern like at night,is this having rigor n chills??kindly answer these all so I could suggest you for sometime else to take."
},
{
"id": 81973,
"tgt": "What causes buildup of fluid in pleural cavity?",
"src": "Patient: My dad is on hemodialysis. Recently he had a thoracentesis and the doctor extracted more then 2 liters of clear yellow luquid from his right pleural cavity. He took a chest Xray today and it shows the fluid is back and worse. The lab tests on the liquid drawn previously were negative. What is causing the buildup of liquid in his pleural cavity? Doctor: Thanks for your question on HCM. Fluid in pleural cavity is known as pleural effusion. And broadly speaking there are two types. 1. Transudative.2. Exudative. Transudative effusion is due to causes other than pleural disease. It is seen in cardiac, liver and kidney diseases. It is basically due to fluid overload and low protien. And both of these are seen in kidney patients. Hemodialysis causes protien loss and this is the cause for Transudative effusion in your father's case. So better to get done protien level and start protien supplements."
},
{
"id": 138402,
"tgt": "What causes joint pains in the fingers with headaches and nausea?",
"src": "Patient: Hi, these past few days, I ve been experiencing joint pains on the fingers of my left hand, followed shortly by pulsating headache, usually at the back of my head. This is accompanied with mild nausea and sensitivity to light. The throbbing headache would last for about a day or two. Can you please tell me what s wrong? Thanks in advance. Doctor: Welcome to healthcare magic!Do you have difficulties when moving your head forward (trying to touch your chestbone witn chin) ? You have to see an infectionist because this photosensitivity accompanied by nausea and headache can be signs of serious disease known as meningitis! Be very careful and see a doctor because some additional tests will show for sure ( liquor fluid testing with lumbar punction) !"
},
{
"id": 10197,
"tgt": "Why am I tired and have breathlessness with severe hair fall?",
"src": "Patient: ive been reallly tired lately, and i can barely breathe, i have been this way for about a month. i dont know if it has anything to do with this but my hair feels thinner and whenever i take a shower more hair comes out when im washing my hair. Im really weak too. Doctor: Hello and Welcome to \u2018Ask A Doctor\u2019 service. I have reviewed your query and here is my advice. Fatigue, breathlessness and hair loss are signs of anemia (low hemoglobin levels). It is better you opt for a complete blood work up to identify the problem. Hair fall can be secondary to any illness/stress in the body. The underlying problem has to be corrected to reduce the hair loss. Hope I have answered your query. Let me know if I can assist you further."
},
{
"id": 28123,
"tgt": "What does QRSD acronym stand for in a cardiogram result?",
"src": "Patient: what dies QRSD acronym stand for in a kardiogram result? I have a score of 130 I am 73 years old. had no medical history of heart condition. My Pr is 135 QT 424 QTc 470. The P is 1 the QRS is -31 and the T is 132. I am going for a Nuclear Stress Test. I walk evey day for 45 minutes, not out of breath. Otherwise I m feeling fine. Doctor: Hi Welcome to HCM.I understand your query and concern.Yes definitely your symptoms are suggestive of AF due to undue stress and anxiety. .Increase in heart rate is known to alter the blood supply to the brain leading to impaired function in a transient way.Few drugs like beta blockers are certainly known to reduce the heart rate and accelerate the dizzy spells.I advise you to have a complete 24 hour holter ECG which can accurately determine the cardiac conduction abnormalities.Monitor your blood pressure and heart rate.Drugs like Isoproterenol will help to increase the heart rate.Drugs like amiodarone can also help to minimise the rhythm irregularities.A 2 Dimensional echocardiography will help to assess the functional reserve of the heart ruling out organic heart lesions.Electrophysiological study and Radiofrequency ablation will also help to eliminate abnormal foci of rhythm.Consult your cardiologist for expert management.Post your further queries if any,Thank you."
},
{
"id": 205383,
"tgt": "What causes irregular heart beats while having anxiety?",
"src": "Patient: I am a 51 year old female. History, have a heart murmur, Microvalve regurgitation, irregular heart beats, history of anxiety and depression. Currently take atenonol (Heart palpitations), Premarin, Xanax (Anxiety), Vytorin (not sure why), Naphazadone (depression) and Clonazepan (Anxiety). I have had my tonsils and adenoids removed, two hernia surgeries, a total abdomen history, broken wrist and those are my surgeries. The last two months, I have been experiencing irregular heart beats, like my heart is out of rhythym and won t beat right. All my joints are hurting, it s heart to get out of bed, my knees are hard to bend, my back is aching and I m stumbling around and dropping things, plus I m saying the wrong thing, like instead of a red sock, I ll say sock red. I m not sure what is going on. My family doctor just retired, I need to go to the doctor, what type of doctor would you recommend, and also do you have any ideas on what I could be experiencing. I appreciate any advise. Thank you! Doctor: Hi,I have gone through the question and seems you are suffering from multiple ailments. You may require a thorough physical and systemic examination which is performed best by a good physician. I would suggest you may please consult with a doctor having postgraduate degree in internal medicine.Hope I have answered your query. Let me know if I can assist you further. Regards, Dr. Chandra Shekhar Gupta"
},
{
"id": 169174,
"tgt": "Suggest treatment for a lump on the face",
"src": "Patient: my baby of 7 months has a raised bump on face its red but yet skin coloured no white head or anything like that i treid many time to squeeze soflty as i thought it was a pimple but nothing its been there for 4 months now and shes getting another on her eye lid. Doctor: HiI understand your concerns but don't worry. The most likely reason for such lesions is molluscum infection. I would recommend you to visit a dermatologist for proper clinical evaluation. If present, it's usually harmless and self resolving.Take care"
},
{
"id": 153158,
"tgt": "Is there a complete cure for multiple myeloma?",
"src": "Patient: My mother (60 yrs old) is suffering from multiple myeloma since last one year. When she was diagnosed she had high M about 5.53 gm/dl but with no other symtom like bone lesion or bence jones protein etc. Her treatment started with Leningio, for 4 cycles, which very well controlled the diseases. However, Myeloma protein recurred within a month, time during which she was on thalidomide. She was started with valcade, after two dose only M protein went down to Zero. Only two cycles of valcade were given as she complained of severe neuropathy in hands and legs. After that, as M was under control, she underwent stem cell transplant. She still has neuropathy in hands and legs after almost three months of peripheral blood stem cell transplant. Otherwise she is fine. But since past week she has started complaining of poor eye sight. She has blurred vision and can not see distant objects very clearly. I am worried and would like to know why is it so? Will as this disease progress my mother will have other problems also like kidney failure, inability to walk or loss of senses because of severe neuropathy etc. Apparently this disease looks worst with no sign of relief even after having transplant etc. Will there be any disease free survival period. Please guide. Doctor: Good morning. With the current knowledge and advancement in medical science, there is still no cure for multiple myeloma. Only the survival can be prolonged. Transplant only provides a prolong disease free survival but the disease again recurs back. There are still newer drugs available with little side effects and your mother can be prescribed those drugs. More over a repeat transplant can be done once her condition improves. So you need not to worry. With the current available treatment options patient of myeloma can live upto 5- 7 years.regards"
},
{
"id": 121024,
"tgt": "How to treat joint pain?",
"src": "Patient: hi. doctor im 43 yrs weight 115kg this increased due to the medicine im taking r cinex 600, combutol 1000 and l cin 500. i want to know the joint pain which is very severe will go after the course. if taken overdose of any of these medicine what will be the side effect. thankyou. Doctor: Hello,It looks that you are taking anti tubercular treatment with Rifampicin, ethmbutol and isoniazid / pyrazinamide. The joint pain as a side effect is associated with rifampicin and isoniazid. The best thing is that you should not worry too much about this because this effect is temporary and will go away after taking complete course and stopping drugs. At present your preference should be to complete your anti tubercular therapy. I can understand your pain. You should keep your morale high. In case of severe pain you may have a mild analgesic like paracetamol for relief.Hope I have answered your question. Let me know if I can assist you further. Regards, Dr. Mukesh Tiwari, Orthopedic Surgeon"
},
{
"id": 58592,
"tgt": "Detected HbsAg quali reactive. Had no hep B, all other reports negative. Advise ?",
"src": "Patient: Hello, last 2009 im applying for a position in a government but then it came up that my hbsag quali is reactive. Im not contented with the result i got my second opinion to a higher institution all my labs are negavite hbsag quali and quanti, anti hbc,anti hbs, hbeag, sgot, igm and igg are all negative.. The doctor explain everything, i dont have a hep b. last 2012 i had my qualitative test and it showed non reactive.. Im just bother with my self and my career because im applying to work abroad.. Please give me advised.. Ty Doctor: Hi! Welcome to HCM! HBsAg positivity means recent active Hepatitis B infection . It is tested as a routine in many govt set ups by a simple screening test.If the screening test is reactive then a confirmatory test is mandatory because there are false positive results and if the confirmatory test is non reactive and a quantitative test is also negative that means person is free from disease as in your case. So I think it was just a false positive result which gave you worry!"
},
{
"id": 106489,
"tgt": "What should I do for pain in the lower back and abdomen after lifting a heavy object?",
"src": "Patient: I was lifting a very heavy object and felt a loud pop in my back which caused me to see stars, feel like I was going to through up and pass out. Made it to a chair and sat for a while until pain eased some. Was able to get up and walk but with pain in my lower back. Now my abdomen feels sore and abnormal. What do you think has happened and what should I do? Doctor: Hello, The pain in the lower back you have is likely to be a muscle spasm. You need to have rest and do a hot compress. Local analgesic gel and oral muscle relaxants will also help. Hope I have answered your query. Let me know if I can assist you further. Regards, Dr. Praveen Tayal, Orthopaedic Surgeon"
},
{
"id": 220319,
"tgt": "What are the symptoms of potential pregnancy?",
"src": "Patient: My girlfriend has been on her period as scheduled but she is concerned that she may be pregnant because there has not been any lining in her period. She has been on birth control but has been taking it later than she normally does do to college. We had four play a few weeks before her period but I did not ejaculate. Is this normal or could she be pregnant? Doctor: Chances of her being pregnant are very remote as she was using contraception and you did not ejaculate also.even then if she is doubtful get her pregnancy test done and confirm or rule out the same"
},
{
"id": 173847,
"tgt": "Is nan-2 be given in place of lactogen to a 6 months old?",
"src": "Patient: my baby is 6 months old.i need to know if i can switch over to nan-2 from lactogen-1? baby has normal stools from his early months as at least 1 time a day but now it is like 2-3 times a day.last stool mostly comes watery.should i introduce water to him now? he has started with banana as his 1st solid food. Doctor: yes after 6 month shift from stage 1 to stage 2 is advised. give cererals , banana. one by one. u can give water but no give to much."
},
{
"id": 101032,
"tgt": "Could the welts on the body be due to allergy to bug bites?",
"src": "Patient: My son has been getting welts on his body (reminds me of mosquito bites) usually waking up with them. Appears to be some type of bug bites. I know it is not bed bugs, have dealt with that with that before. Have done the deep cleaning of room, washing everything. Could this be an allergy to something? Doctor: Hello.Thank you for asking at HCM.I went through your son's history.Yes, they can be due to bug/mite bites (there are several types of bugs/microorganisms, some of them can not be visible with naked eye).If he does not get welts daily, I would also think of food allergy. For that, I would suggest you to observe his evening meal and the days he gets symptoms. This will help you to identify a correlation between food and his symptoms.Were I treating him, I would prescribe him levocetirizine/cetirizine/hydroxyzine for 5-7 days and to wear loose, preferably cotton-like clothes (to avoid sweating). I would also suggest him to apply lotion like calamine on his skin.Hope above measures will be helpful to him.Should you have any further query, please feel free to ask at HCM.Wish him the best of the health.Thank you & Regards."
},
{
"id": 212234,
"tgt": "Have Serotonin Syndrome twice, Major Depressive Disorder, chronic migraines. Taking Celexa. Cure?",
"src": "Patient: I have had Serotonin Syndrome twice as I have Major Depressive Disorder , Recurrent, Severe and Treatment Resistent and also Chronic Migraines, plus many other musco-skeletal issues in my neck and advanced stage TMG. I could keep listing diagnises, but wont. I have had an intractable migraine for 4 weeks, so this week my Neuro has put me on 2 doses of 300mg of Neurontin a day. Currently I am taking 20 mg of Celexa , just went off of 5 mg of Abilify and 60 mg of Straterra as I need a break from the side effects. I couldn t get a definitive answer at the pharmacy. Doctor: Hi there, thanks for asking. Many medications should be used cautiously with Celexa. Combination of many medications with celexa can cause serotonin syndrome. Many medications which abort migrain should not be taken with celexa for avoiding the serotonin syndrome. Serotonin noradrenergic reuptake inhibitors also may do this (one of your medications belongs to this class). You can ask your doctor about this issue more. Best wishes"
},
{
"id": 154089,
"tgt": "Is esophageal cancer best treated with Taxotere and Cisplatin?",
"src": "Patient: My husband has esophageal cancer and it has spread to his liver and lungs. They want to use taxatere, xoladan and cistplatin. Is this the best treatment. He has already had radiation and 5fu, carboplastin, and xoladen, and eperubicin. that treatment is not working any longer Doctor: Hi, dearI have gone through your question. I can understand your concern. He has esophageal cancer. His cancer is spread to the lung and liver. Sp it is considered as stage 4 cancer. Surgery is not possible at this stage. So only treatment options left are chemotherapy and radiotherapy. He has taken radioe and chemotherapy. That regimen is not working so other regimen cisplatin can be used. However despite of all treatment prognosis remains poor. Life expectancy is not good. Consult your doctor and plan accordingly. Hope I have answered your question, if you have doubt then I will be happy to answer. Thanks for using health care magic. Wish you a very good health."
},
{
"id": 64081,
"tgt": "What is the treatment for a pea-sized bump on the head?",
"src": "Patient: I have a pea-sized bump on my head. It is hard, but moves with my skin. It hurts to put pressure on it, but not more than if I was putting pressure anywhere else on my skull. It has been there for a couple of years, and I think it may have grown slightly in the last few months. A few areas around it hurt a little bit more than they should, almost as if it has tendrils that hurt whenever the bump hurts too! Doctor: Hi,Dear,Thanks for the query to My HCM Clinic.I studied your query in details and understood your health concerns.-Treatment and Cause of your lump in upper neck-a-Mostly it is-Dermoid ? or it could be-Burnt out Chronic boil with reactivated infection due to excessive handling.b-Show it to Er Surgeon-, who would treat it as follows-i-would study the Xray Skull for any intra-skull connectivity as in Dermoids.ii-Would Excise with Biopsy-of the sore lump , which would fix its cause.This advise would help you to plan treatment with your doctor.Hope this resolves your query.Wellcome for further query in this regard. Have a Good Day...!!With Regards"
},
{
"id": 129719,
"tgt": "Neck pain and small painful lump above collar bone",
"src": "Patient: Neck pain and small painful lump above collar bone. Is this just swollen glands? I have neck pain and noticed a small painful lump just above collar bone area of neck..pain is running up from lump. I thought it was maybe a neck strain but also feel very tired. When I press on the lump it hurts and I start getting mucus build up in my throat. What does this mean? Doctor: it could be a sequestration dermoid cyst or a plunging ranula. The history and location you have provided is too non specific for me to come to a diagnosis but I would advise you to go to a doctor for a full examination of the swelling and make sure that he/she rules out metastatic lymph node although that is highly unlikely in your case.I'm sorry I could not be more of a help."
},
{
"id": 112399,
"tgt": "Severe pain in lower back, legs after taking cortisone injection for lower vertebrae problem. Suggestions?",
"src": "Patient: I recented received cortizone shots into two lower back vertibre to help with the shooting pain in my hips and legs. The GP who suggested this procedure promised that the worst that could happen is that the cortizone injections would not take and I would not be any worse off. Wrong! I now suffer from quite severe pain in my lower back andfrom time to time have shooting pains into both legs. I now have resorted to using a cane. Would do I do now? Doctor: Hello,Thank you for the query. I have gone through your query and understood your concerns. I suggest you to have epidural analgesia instead of cortisone to get relief. But as you have recently took the shot you can have spinal stimulation therapy which is showing good results now a days. Thank you."
},
{
"id": 29791,
"tgt": "What causes recurring painful blisters on the soles?",
"src": "Patient: Yes, I have these red pus bumbs located on the bottom and sides of my feet. They clear up and then a month or so later they return. Sometimes it hurts me to walk. I have no idea what this is or what s causing it. Can you give me some suggestions? Doctor: Hi and welcome to Healthcaremagic. I understand your concerns.It may be impetigo pyoderma which is a dermatologic disease caused by bacterial skin infections You need to see dermatologist and do further tests to see if this is your case. Also, skin biopsy should be taken since tese changes are unusual and should be evaluated.I hope I have answered your query.Kindly regards. Wish you a good health."
},
{
"id": 81390,
"tgt": "What causes severe chest pain in sleep?",
"src": "Patient: hey i am a female, age 22 and every morning when i wake up or during my sleep i have a severe chest pain right in the center of my breast.i don t smoke nor drink nor lift heavy weights. currently am on birth control for like almost a year now and 2 months ago is when this chest pain started. my question is what may be the cause of my severe chest pain during the nights and when i m awake? Doctor: Thanks for your question on HCM.There are few possibilities in your case.1. GERD ( Gastroesophageal Reflux Disease)2. Stress and anxiety related ( mood disorder)Birth control pills can cause GERD and mood swings (anxiety). And you are taking it since 1 year. So chances of side effects Are high. And GERD and anxiety are very well known side effects of birth control pills.So better to consult gynecologist and discuss about these side effects.At present take good proton pump inhibitor and avoid stress and anxiety.You are too young for cardiac disease. So possibility of cardiac cause is very less.Side effects of birth control pills appears more in your case."
},
{
"id": 33741,
"tgt": "Suggest remedy for a cyst on the shoulder",
"src": "Patient: l have a hard knot on my shoulder it has been there for 2 years now i started messing with it today an clear stuff an blood came out but i dont know what it is why is it hard knot an why is clear stuff coming out an what is it???? im 17 years old i weigh 160 pounds an ive had lost of kidney problems with dehydration an i do have a cyst on my ovaries could this be a cyst of some kind? Doctor: Hi i did review your concern. A cyst on shoulder is generally due to some infection or some bursa inflammation. i would recommend you to get it evaluated from a surgeon and probably get it removed. You can get a diagnosis before removal by a needle aspiration cytology which is a day office procedure which includes needle puncture and aspiration of fluid and analysis of cells present inside it by a cyopathologist. Eventually you might anyways would want to remove it if it is causing you any kind of discomfort.I hope this helps Wish you a very healthy life ahead."
},
{
"id": 172751,
"tgt": "How to treat constipation in a 3 months old child?",
"src": "Patient: hello,wanted to ask that my baby is 3 months old n he has not passed stool from since last 3 days ,today hepassed little bit but with lots of force n blood also came old ,still some stool left,m very worried ca u pls suggest me that what can i do/??/ Doctor: HiWelcome to the HCMI understand your concerns but don't worry. Infants can have wide variation in poops in terms of frequency, consistency and colour. It's not recommended to treat an infant not passing stools up to a week unless and until he has any other associated symptoms such as refusal to feeds, vomiting, abdominal distension etc.But since your baby has hard stools and has marked difficulty in passing stools along with blood stains, I would recommend you to use per rectal glycerin suppository to help it. Glycerin works by osmotic effects. Do not use it more than once a day. Also, try to establish exclusive breastfeeding as it has less chances of constipation in baby. If on formula feeds, mix it in a proper proportion. Give regular feeds to avoid dehydration.Hopefully this will help you. I would be happy to help you out in any future health related questions.Take care"
},
{
"id": 170254,
"tgt": "Is frequent, watery stools in an infant concerning?",
"src": "Patient: my daughter is 3.5 months old. normally she passes 3-4 stools a day. from last week every alternate day she is passing 5-6 stools a day.sometimes she passes stool while drinking my milk. sometimes the stool is totally watery. she is oly on mothers feed. her weight is 5.2kg.iss it a cause of concern? Doctor: HiWelcome to the HCMI understand your concerns but don't worry. Infants are prone to wide variation in frequency, consistency and colour of poops. I appreciate that you have kept your baby on exclusive breastfeeding. So the chances of infective diarrhea is not likely.So, continue with regular feeds and no other treatment is required. Take care"
},
{
"id": 187084,
"tgt": "Will antibiotics will really cure tooth infection?",
"src": "Patient: I have got a recent new bridge on my upper 6 front teeth because my lateral incisors are missing, and my central incisors have gone under RCT, and one of them developed an infection at the apex of the root but the rest of the bridge are fine, my dentist prescribed Ornidazole and told me to take it for 3-4 days and we'll check about the improvement after a week, but if it doesnt get better after 2 months then I might need apectomy.I just want to know if the antibiotics will really cure the infection? Is it a serious problem? because I dont want to get an apectomy operation. Doctor: Hello, Welcome Thanks for consulting HCM, I have gone through your query, yes sometime infection is healed by taking antibiotics . You dont worry take medication for few days it it doeant heal then go for apicetomy it is simple procedure not to worry about it much.Hope this will help you."
},
{
"id": 153830,
"tgt": "Is it safe to have chemotherapy for a old aged woman?",
"src": "Patient: age77 years H:5ft,4in W:100lbs I am a prostate cancer patient TURP & Orchidectomy operations have been done at KOlkata in Jannuary -2011Palliative radiations have been given in the pelvic region & prostate gland areas in June'11Doctor has given Bis phosphonate injectionto relieve back pain Now is suggesting to take Chemotherapy injectionI am in dillema whether at this age & health condition I shall be able to withsta nd chemoBlood PSAvalue is >100& Hb%is 8.5 Need your valued advice Doctor: Hi, dearI have gone through your question. I can understand your concern.You have prostate cancer. You have gone through surgery and radiation. Now you need chemotherapy. It depends on general condition of patient that he can tolerate chemotherapy or not. If you are otherwise ok then you should take chemotherapy. If you can not tolerate that then it can be stopped. Consult your doctor and take treatment accordingly.Hope I have answered your question, if you have any doubts then contact me at bit.ly/Drsanghvihardik, I will be happy to answer you.Thanks for using health care magic.Wish you a very good health."
},
{
"id": 28895,
"tgt": "How can bronchitis and breathing difficulty be treated?",
"src": "Patient: a few months ago, I came down with my almost yearly dose of bronchitis and walking pneumonia, had breathing treatments and a z-pak but my symptoms have yet to go away. I have a hacking cough that makes me urinate at times when really bad and I have trouble getting air in or out correctly. I can be doing absolutely nothing and have trouble breathing, sometimes waking me up at night. Help? Doctor: Hello,Airway closure and inflammation are the most common reasons for a persistent cough with infection.You may need a short course of oral steroids to help reduce the inflammation in the chest. Continued inflammation is often the most common reason for the persistent coughing.You may also need to use a medicine to open the airways, either an oral medicine or the temporary use of an inhaler such as ventolin.Hope I have answered your query. Let me know if I can assist you further.Regards,Dr. Michelle Gibson James"
},
{
"id": 51540,
"tgt": "What precautions and diet should on take before and after dialysis ?",
"src": "Patient: my dad has creatine level 8.5 & dr has told to start dialysis from tmr. i want to know how mant times dialysis needs to be done in week and will frequency reduce from next week?. what diet and precautions to be taken after dialysis Doctor: hello their please go for serum potassium investigation which is more important then creatine level, then let me know regards"
},
{
"id": 220178,
"tgt": "What causes swollen vaginal lips during pregnancy?",
"src": "Patient: I am 5 months pregnant and about a month ago I started feeling really sore around my vaginal lips. No burning or pain when I pee, just aching on my lips (especially towards the front of my body) and on my buttox right around my vaginal opening. I also started experiencing lower abdominal cramps. All of these symptoms just keep progressing, until last night I tried massaging myself and my vaginal lips swelled up afterwards to the point where I couldn t close my legs. Please tell me if this is normal or if I need toget into the doctor now. Doctor: hello5 months of pregnancy with swollen vulva can be a feature of hypertension in pregnancy.u need to go to a doctor immedietely and get urself locally examined, also measuring blood pressure and other tests need to be done."
},
{
"id": 147039,
"tgt": "How to treat fits in a child?",
"src": "Patient: hi doctor. my daughter is a CP child of age five n a half years old her weight is 10.8kg she has a fitz problem since 3.5years . we hav consultd with many doctrs .no one gv us satisfactory answer. last year one neurologst says aftr seeing her MRI tht her outer surface of brain is plain. incmplete formation ,in EEG report he says its not gud, aftr tht v medicatd her with SEIZONIL syrup (tegratol) .nowadays i m gvng her 9ml thrice a day but we cnt overcome her fitz. cn u gv us any advice for her betterment?????????? Doctor: HIThank for asking to HCMI really appreciate your concern and looking to the history given here I could say that if the child is having cerebral palsy then convulsion may be the symptoms of that and the given medicine is alright but only matter is dose the dose of this need to be adjusted properly for getting good result, take care and have a nice day."
},
{
"id": 88648,
"tgt": "Suggest treatment for severe abdominal pains after marriage",
"src": "Patient: hi, doctor, im 21 yr old girl, just recently got married, 8yrs before i went under ASD DEVICE CLOSURE, (HOLE IN THE HEART). i never get my menstruation regular. just twenty days i have got married, now im having pains in ma lower abdomen, as if im getting my menses, but im not getting it. the pain so severe and unbearable doctor. please be kind enough to advice me in this painful situation! Doctor: Hi ! Good afternoon. I am Dr Shareef answering your query.If I were your doctor, I would go for a complete blood count, an HbsAg and an HIV test as a routine, a serum HCG test and an ultrasound of the abdomen to rule out any pelvic pathology and an unsuspected pregnancy as well. Till then I would prescribe you with a mild anti spasmodic drug along with a proton pump inhibitor for a symptomatic relief. Further management would depend on the reports of investigations.I hope this information would help you in discussing with your family physician/treating doctor in further management of your problem. Please do not hesitate to ask in case of any further doubts.Thanks for choosing health care magic to clear doubts on your health problems. I wish you an early recovery. Dr Shareef."
},
{
"id": 158840,
"tgt": "Having loose motions. Had bladder removed. Undergone chemo and radio therapy. Had colonoscopy and endoscopy. Taking diabex and Glayde. Suggest",
"src": "Patient: I am 78 years old,taking diabex500xr now 1 tablet 3 times a day also 2 Glayde tablets in the evening.Sometimes in the morning and during the night I feel not very well also after 1st motion I very often have a very loose motion 2-3 more (Ihave my blooder removed due to cancer and had chemo and radio therapy in 2010, also have a bag) . I also have problems in controling my motion when I have to go.Can you help.I had a colonoscopy and endoscopy a couple month ago and both are negative results, OK.Thank YouMark Doctor: Hello; Welocme to healthcaremagic. Good to know you do not have cancer now because your test results done recently are normal. Regarding problem with motion ,it is probably due to radiotherapy in pelvis.The problem is more common in older age group where sphincter that control motion is already weak. Although the problem is bothersome it is usually self limiting. Taking lots of fibres and greens which are easy to digest can help. Regards"
},
{
"id": 114055,
"tgt": "Lower back ache it hurts to even walk during the pregnancy period of 23 weeks",
"src": "Patient: Hi I am 23 weeks pregnant with severe lower back ache that It hurts to even walk, other than lying on my left side or useing a heating pad is there anything I can do... Plz help im in alot of pain. Doctor: Hi,Luv, Thanks for query, At this time lower back pain is common but intensity is not much. so consult orthopedic to rule out any orthopedic cause. Ok and bye."
},
{
"id": 10766,
"tgt": "Would Mintop and Novophane capsules have any side effects on a pregnancy?",
"src": "Patient: i m having hair fall. 2 years before i used mintop 10% solution , Novophane and finax tablet . within 3 months i got better result. after that i used only mintop 10% solution for past 1 years but all my hairs starts falling and now my doctor advice to use morr12.5% solution along with novophane plus tablet . whether this combination works ? then i want to know wheather is there any side effects because i m looking for second baby. Doctor: Hi,First of all I would say that Finax tablets should not be handled by a woman who is pregnant or who may become pregnant. If you accidentally come into contact with this medication or a broken or crushed tablet, please wash the area right away. Also Finax tablets (Finasteride) are not indicated for use in women. You can use Novophane shampoo, and mintop 12,5%. For the mintop solution I would advice to use it only if the situation is severe. You will notice the during pregnancy your hair will be fuller, because you will loose it more slowly as you normally do, because of the high levels of estrogen.Hope I have answered your query. Please feel free to contact me if anything is unclear.Wish you good health! Regards,Dr. Migena Gega"
},
{
"id": 220499,
"tgt": "Why Decdan injections are used in pregnancy?",
"src": "Patient: Good Evening Ma'am. . Ma'am I wanted to know why Decdan Injections are used in pregnancy? My wife is pregnant and its her 9th month.. 6th June is her due delivery date. Yesterday our concerned Dr told her to take decdan injection (2 doses, one after another in12 hrs).. Its her 1st pregnancy so I am a little tensed .. Please throw some light on this.. Thank you Doctor: Hello,Decdan contains dexamethasone as ingredient. It is given after 32 weeks of gestational age for better fetal lung maturity. It reduces respiratory distress problem during delivery or after that. It is quite safe.You have nothing to worry. Proceed follow up check up as per schedule after consultation with obstetrician.All the best."
},
{
"id": 122989,
"tgt": "Suggest remedy for swelling and numbness in legs",
"src": "Patient: well it started 4 years ago i broke my right femur had to have a tittium rod from hip to knee now they say i have a bugline disk pinched nearve to my legg well just did my 3 rd injection in lower back now it worse and my right leg is sweeling up feels like pin and needles and goes numb both legs when i walk Doctor: Hi, As post the implant there will be change in the biomechanics of the gait which is normally considered as walking. Due to which there will be abnormal and unplanned stress & strain on the spinal muscles leading to changes in the normal anatomical position of the intervertebral disc. Due to which there is numbness, tingling , pain etc. The swelling will be due to lack of venous return as the muscles of the injured limb would have undergone weakness. I will advice to start some simple low intensity exercises like - breathing exercises, core stability exercise, spinal muscle strengthening exercises, hip muscle strengthening exercises, static Quadriceps, static hamstring, straight leg raise, ankle toe movements. This will help stabilize the spine and reduce the symptoms along with swelling. If required you can take guidance from the physiotherapist as well. Hope I have answered your query. Let me know if I can assist you further. Regards, Jay Indravadan Patel, Physical Therapist or Physiotherapist"
},
{
"id": 8390,
"tgt": "What is the remedy for Moles in face?",
"src": "Patient: hi.. my name is ansar im 50yr old.. i had moles on my face which usualy developd after menupause.. nw 4 weeks ago i had Laser treatment for the moles ..its been 4 weeks since treatment and my skin is not coming bak to normal as there are brown coloured spots left on the treated areas... now what shud i do? is there any medicine or cream for this? Doctor: Hello. Thanks for your query.You seem to have developed Post-inflammatory Hyperpiegmentation , which is common after a Laser procedure, specially if sun protection was not strictly adhered to.If I was the treating doctor I would suggest that you use a broad spectrum sunscreen, throughout the day. Your sunscreen should cover both UV A as well as UV B part of the spectrum. You may find a sunscreen OTC from a drug store or you may ask for a prescription for it from your dermatologist.For the spots, I would have normally recommended you a skin lightening cream containing one or more of the following ingredients e.g Kojic acid, Glycolic acid, Arbutin, Hydroquinone etc. These ingredients are available in various combination in a cream formulation. These creams are prescription products and you may visit your dermatologist for the same.Regards"
},
{
"id": 95248,
"tgt": "What treatment should I take to get relief from abdominal pain which is more while walking and standing ?",
"src": "Patient: I am a 33 yr old female .I have lower right sider abdominal pain which goes toward my inner thie since last 4-5 month also same sided back pain .month ago my laproscopicappendectomy is done but the pain haven t stopped infact increased(my abd ct andsonography and bld test are normal) pl. advize..it is kin of very much of tension coming on my nerves also my stomch feel very heavy and give me pain more when i m standing and walking although it is continuous even after taking pain killer .some relief i there only when i lie down. also i have motions 3-4 time a day i m really worried and feel like misguided .(what is this exactly ibs, or nerve or anything else)I would like to add some more points which according to me must be considered as I don\u2019t want to keep a single point to be neglected, yes this is my life and I have to be really concerned about it at least now looking into the past history. I had a small accident before 2 yrs back where I was collapsed from nearly 2ft high platform unknowingly ,that time I had ligament rupture in my right foot and ankle after 2 month of rest and physiotherapy I started walking gradually. I am really worried and want to seriously look into not only for present issue but for the future. Doctor: Hello Snehal, . Do you have Nausea or vomiting? It need not necessarily be IBS, but get yourself examined by a Gastroenterologist to confirm it. Get a repeat Ultrasound scan and a CT scan done after consulting your Doctor. If everything comes to be normal, Kindly get yourself examined by a Urologist so that he can examine you and suggest Urine routine or Blood Work up to rule out Urinary Tract Infection or Kidney stones, since your history suggests dragging type of pain to your Thigh. Get a Ultrasound scan concentrating more on your Kidneys to rule out Kidney stones. Once confirmed the treatment has to be started. Drink as much as 2 to 3 liters of water everyday. The treatment depends upon the size of the stone. No treatment is required if the stone is less than 4mm. Eat less salt and less protein diet. Eat Fresh fruits regularly. Wish you Good Health, Take Care."
},
{
"id": 144414,
"tgt": "What causes soft spot on the head along with headaches?",
"src": "Patient: I noticed a soft spot about a month ago,but didnt think much of it. I just recently noticed it was extreamly bigger than it was. Ive had bad sinus trouble for about a year or longer with painlful headaches that sometimes feels better when i squeeze my head. Could tge soft spof be realted? Doctor: Hi Dear,Welcome to HCM.Understanding your concern. As per your query you have soft spot on the head along with headaches. Well there can be many reasons for symptoms you mention in query like migraine , follicullitis , fractured bone or hematoma . I would suggest you to consult dermatologist for proper examination. Doctor may order CT scan , ultrasound and physical examination to check tenderness and consistency . Doctor may drain the hematoma , prescribe antibiotics along with anti inflammatory or may prescribe vasograine along with diuretics and betablockers. For now apply warm compresses and take ibuprofen or acetaminophen for pain .Hope your concern has been resolved.Get Well Soon.Best Wishes,Dr. Harry Maheshwari"
},
{
"id": 4626,
"tgt": "How long does it to get pregnant after implanon removal followed by periods?",
"src": "Patient: hello i am 21 years old I just got my implanon removed on Nov. 22nd and me and my husband are trying to get pregnant.We do have unprotected sex and recently did Nov. 28th. I am wondering how long is it going to take for me to get pregnant? Now I was on my period a week before I got it removed then came on again for two days earlier this week after it had been removed so I came on twice this month. My husband is a diabetic and due to financial situation I had to get the implant removed after it expired which was 9months after it expired. Which leads to my second question does that slim our chances to get pregnant? Any answers will help thank you. Doctor: Hello,It differs from person to person.It may take from few weeks to few months to get pregnant after removal of implanon.Removal of implant improves the chances of pregnancy.It has been seen that implanon keeps working for around a year more after the expiry date.Thanks"
},
{
"id": 41106,
"tgt": "What does elevated LH and estrogen levels with normal ultrasound indicate?",
"src": "Patient: i am trying to get pregnant and did iui w/ clomid this month. i got my period on friday when i was suppose to go in for a pregnancy test. the flow was normal although it only lasted 3 days and i had no cramping which is unusual for me. i went in for baseline bloodwork today and they said my lh and estrogen levels were high although my ultrasound looked normal. any explanation? Doctor: Hello, these are all because of clomid intake in a patient with PCODIn case you have any questions in future you can contact me directly on http://bit.ly/drmanishajain"
},
{
"id": 70921,
"tgt": "What causes shortness of breath and fatigue?",
"src": "Patient: i have shortness of breathe, fatigue sensation of something in my throat, arm and hands tingle and prickling feeling acid reflus slight burning feeling at top of stomach, had ekg good xray clear,pft for copd clear bloodwork for copd crest clear 100% oxygen level, anybody with a clue so desperate.. Doctor: Hello and Welcome to \u2018Ask A Doctor\u2019 service. I have reviewed your query and here is my advice. By your history and description, possibility of uncontrolled GERD (gastroesophageal reflux disease) is more likely. GERD is due to laxity of gastroesophageal sphincter. Because of this, the acid of the stomach tends to come up in the esophagus and cause chest pain, burning epigastric pain, nausea, breathlessness, throat irritation, etc. So follow these steps for better symptomatic relief in GERD. 1. Take pantoprazole and levosulpiride combination on empty stomach twice daily. 2. Avoid heavy meals, instead take frequent small meals. Avoid oily and spicy food. Avoid junk food. 3. Keep 2-3 pillows under head in bed to prevent reflux. Go for walking after meals. 4. Quit smoking and alcohol if you have these habits. Don't worry, you will be alright with all these. Hope I have answered your query. Let me know if I can assist you further. Regards, Dr. Kaushal Bhavsar"
},
{
"id": 58197,
"tgt": "End-stage liver failure. Nausea, lack of appetite, wasting. Precautions?",
"src": "Patient: My 55 yo son has end-stage liver failure . He recently completed the medication regimen to eradicate his Hep-C, and last couple of lab results show no discernible trace of Hep C . He has an appointment at UAB on Tuesday for his three-day workup prior to being placed on the liver transplant list. He has continued to decline, suffers almost daily with nausea , lack of appetite , wasting. Today his thigh-high edematous legs began to weep. I have cleaned them well and wrapped them with absorbant gauze , and elevated them above the level of his heart. I will try in the morning to put on compression stockings after - hopefully - some of the swelling decreases. If I am unable to do so, should I wrap them with ACE bandages, and if so, what pattern, fig 8 or circle up? The trip to UAB is about 4 hrs. What precautions should I take when driving him there? Thanks so much - this is the hardest thing I ve had to do in my 76 years! Doctor: If compression bandage is not possiable then you may use ACE bandage by lightely wrapping it to cover maximum area in circle. while travelling antiemitic may be given in advance. Liver transplant waiting some times very long and patients some times cannot wait so long. Mesenchymal Stem cell therapy may be used as a tool to pass time till liver transplant as this delays the progression of complete liver faliure and improves quality of life."
},
{
"id": 117362,
"tgt": "Suggest treatment for anemia",
"src": "Patient: I had my gallbladder out a few months ago, due to biliary dyskinesia and I recently started having itching everywhere with breathing difficulties and headaches. I think it s anemia, but I m not sure of the cause. Would an iron supplement help with the anemia? Doctor: HIThanks for posting your query to Healthcaremagic . First of all it's very unlikely for anemia to cause Itching though it can cause Headaches and Breathing difficulty . We need to find out the cause of your problems and then comes the treatment . I advise you to get CBC , Chest X ray , test done , Get your BP checked and eyes tested . Revert back to me . Any clarifications feel free to ask ."
},
{
"id": 63469,
"tgt": "Suggest remedy for lumps on the inner arm",
"src": "Patient: I was lying down typing on the computer when all of a sudden my inner arm on my wrist where the vein is started itching. I looked at it and saw that it was all red and broken out with bumps and they were swelling. I immediately jumped up and felt dizzy so I asked my brother to rush me to the ER. Then I changed my mind because I assumed anxiety was taking over taking a sit for WHILE the bumped started gong down and the dizziness left. there are still lumps/bumps there but they are small. Doctor: Hi,The lump near your inner arm appears to be insect bite/bed bug bite?.I would advise following treatment under cover of your PCP doctor-Soda-bicarb application locally at the bite site to neutralise the insect-toxin.Anti-inflammatory drugs x for 5 days time.Cold compresses x 2-3 days time.Hope this would resolve your query.Welcome for any further query in this regard to HCM.Write good review and hit thanks if this reply is helpful to you.Have a Good Day.Dr.Savaskar M.N.Senior Surgical Consultant.M.S.Genl-CVTS"
},
{
"id": 122279,
"tgt": "Is there any non surgical remedies for bow leg?",
"src": "Patient: hello im 44 year old having bow leg..Is there any non surgical remedies for that Doctor: Hello, At this age, corrective surgery is the only possible option. Consult an orthopedic and plan for surgery. Hope I have answered your query. Let me know if I can assist you further. Take care Regards, Dr Shinas Hussain, General & Family Physician"
},
{
"id": 3982,
"tgt": "As per Follicular study report when should pregnancy be attempted?",
"src": "Patient: Hi, Iam 28 yers old ,My weight is 56 kgs .My doctor suggested Follicular study , Yesterday was 11 th day and the 11th day report -right ovary -nil,left ovary-DF:13mm endometrium -8mm POD-free. Can you let me know when i should have intercourse to get conceive . Generally how will the normal report look like Doctor: Hi, hanks for writing..I understand your concern.. Repeat follicular study on alternate day till the follicle size reaches 20 mm. It is time you should have intercourse. Repeat scan need to be done to see for ovulation status after 2 days. Post ovulatory period take progesterone for luteal phase support. Hope I have answered your query. good day."
},
{
"id": 88917,
"tgt": "Suggest treatment for upper abdominal pain",
"src": "Patient: I've been having pain in the right upper quadrant, it's a stabbing pain that goes into my pain, up to the right shoulder & at times takes my breath away. I've been having this constant pain/nausea for 10 weeks now and all tests are saying negative. I did have the HIDA scan done but for whatever reason, it did NOT include the percentage of function of the gallbladder. I've had GB attacks before and it feels like the same exact pain, however, no one seems to listen to me. I just recently had an endoscopy also that does not show any ulcers or obstructions. I am still saying it's the GB, what do you think???? Doctor: Hello ,Welcome to HCM.In my opinion, it may not be gall bladder,because if your HIDA is normal, you should not be in doubts about its result. After going through your question, Gastroeosophageal reflux disorder seems very strong possibility.My advice to my patients would have been to take pantoprazole and domperidon combination for two weeks without interruption.Bland , and full of fiber diet, with plenty of oral fluids.If constipated take mild laxative.If you are already on some medication for some other ailment, make sure they are not contributing to your problem.Please do inform us on out come of your case.good luck."
},
{
"id": 9347,
"tgt": "Why does one have an itchy flaky skin in the scalp?",
"src": "Patient: On the back of my scalp I have itchy flaky skin. It spread to the back of my neck about 3/4 of an inch down and maybe 1 and 1/2 inches across. This spot on my neck is itchy and flaky, as well as red around the edges. I'm using head and shoulders shampoo and it has improved some but it's not clearing up well. Do you think this is dandruff, psoriasis, ringworm or something else. Doctor: HIWell come to HCMLooking to the history given here this could be seborrheic dermatitis, most of the time skin condition of the scalp being treated with combination of three agents, like steroid, antibiotic, antifungal, and the best option would be combination of \" Neomycin, Beclomethasone, Minconazole, try this combination, hope this information helps, take care and have a nice day."
},
{
"id": 205708,
"tgt": "Suggest treatment for stress and depression",
"src": "Patient: My son who is gay, had been in a relationship with a person for16 yrs. over this time , this person has been found to be a liar , cheater, has lost jobs, ran of the family with his horrible and unacceptable behavior. My son who is very bright, mannerly, with a ton of friends, sees some of these behaviors, but find excuses to excuse them all. He can see shortcomings in others, but not his sig other. The problem is : this guy is a cop who has just been charged with child pornography! He tells my son he's being framed! My son believes him! Doctor: DearWe understand your concernsI went through your details. Relationship between gays are almost equivalent to straight people. The relationship basics are love and affection. In your sons case, there is a blind love and affection and you will be able to see some possessiveness. Possessiveness can be attributed to the gay relationship too. You cannot force your son to distrust his gay friend. Circumstances will do that for you. Wait and be patient. You will get opportunities, where you will be able to expose. But, beware, as your son is a gay, he will find another partner very soon. Then you should not worry.If you require more of my help in this aspect, please use this URL. http://goo.gl/aYW2pR. Make sure that you include every minute details possible. Hope this answers your query. Available for further clarifications.Good luck."
},
{
"id": 104010,
"tgt": "Shaky hands, palpitations, pain in the stomach muscles. History of asthma. Opinion?",
"src": "Patient: hi from a very young age i have always randomly got bad shaking hands when it got really bad or i would feel really stressed my heart would start beating really fast it would hurt the muscles in my stomach and i would get really weak and find it hard to talk. i dont smoke drink rarely or at all. i have mild asthma and i go for walks every second day so im not unfit or unhealthy i never get take outs either. Doctor: Hi Thanks for writing to Health Care Magic The symptoms ur reporting could possibly be due to anxiety. When we get stressed out and anxious our body shakes, we become breathless, our heart beats rapidly, there will be sweating, dryness of mouth, stomach discomfort, feeling of extreame weakness and lack of energy, confusion. This may even cause difficulty in speaking. Try relaxation and breathing excercises. Take proper rest. If the problem is still persisting consult a psychiatrist for advise. Hope this helps u Thanks and regards"
},
{
"id": 149691,
"tgt": "Done cervical laminectomy for spinal stenosis. Neck pain, light headedness. Numbness in hand, neck tremors",
"src": "Patient: I had cervical laminectomy surgery for spinal stenosis in Aug 2010. Since then I have a real tight neck that wants to keep dropping , neck pain , light headedness, back pain, occasional loss of balance and trouble walking and climbing steps. I recently lost my job because of these issues. My Neurologist thinks it is all coming from the neck tightness. My Neuro Surgeon disagrees and sent me to Physical Therapy and a Pain Management Doctor. They both agree with my Neurologist. The Pain Management Specialist told me that there is nothing that can be done. He told me 1 out of 10 or 1 out of every 12 with this surgery end up like me. In the past 3 months I started experiencing tremors in my right hand, right arm, shoulder, neck & head. Aggressive at times. I have tremors pretty much daily now. My Neurologist retired but wanted me to get a second opinion because I am progressing. I have not yet had a second opinion away from the same Medical Practice. I was about to when my Neuro Surgeon called and wanted me to get a Neck MRI. She referred me to another Neurosurgeon she works with to have more surgery. I saw the new Neurosurgeon. He explained he wants to take the hardware from my previous surgery out of the back of my neck and put it in the front to hold my head up. I was told it would not relieve the pain or any other symptoms I am having. I was also sent to another Neurologist in the same practice. She has no idea what is going on. Parkisons was ruled out. Both Neurosurgeons think there is something else going on and it s not coming from my neck. I can feel everything coming from my neck. I have had since May 2 neck MRI s & 4 Botox Injections, in the last 30 days an EEG, Brain MRI & CT Scan. All are normal except CT scan. It was to check healing for removing metal in my neck. I have an appointment to talk about that in a few weeks. Do you think there could actually be 2 different problems? My neck is really tight and like I said before I feel everything but my hand and arm tremors coming from my neck. I am really confused. Doctor: Thanks for writing to us.You have complex story with multiple issues and not pointing to a particular one problem. Tremors can happen in severe cervical spine stenosis due to weakness in extremities. Do you have significant weakness in hands? neck drooping is unlikely from c-spine problem. Difficult to tell you the exact answer however if neurologist ruled out other brain related disorder then you may consider doing anterior cervical surgery. Thanks."
},
{
"id": 173515,
"tgt": "Suggest possible treatment for stomach pain & passing of yellow urine",
"src": "Patient: Hi, my 3 year old son has been complaining about a stomach pains for one day then the following day he's been passing a yellow urine (its been 2 days now) and I give him enough water during the day.He has the flu now and I'm giving him his flu medication Doctor: HiThanks and welcome to healthcare magic.Stomach pain specially in right hypochondrium followed by yellow urine suggests hepatitis.Get the liver function tests done to rule out liver disease.If liver disease is confirmed then consult pediatrician for proper treatment.Hope this reply is OK for you.Please feel free to ask further queries if any.Dr.M.V.Subrahmanyam."
},
{
"id": 103678,
"tgt": "Diagnosed with sjorgrens, itchy hives on upper body. Medications?",
"src": "Patient: I have recently been diagnosed with sjorgrens. I have been exeriencing itchy hives on my shoulders, back (torso region), neck and arms. The hives will come and go and show up in other areas. I have been taking allergy tablets to help relieve the itching. Is hives a symptom of sjorgrens? If so....is their any relief for itching and to make the hives go away? It is very frustrating! Thank you! Doctor: this disease causes dryness of every part so it is essential to make skin moist rather than antiallergicsyou should use moisturiser like petroleum jelly to keepskin moist and ad it becomes moist the problem decreases autmaticallyanti allergic can extra benefit for itching but not treatment of it"
},
{
"id": 158861,
"tgt": "Have anemia, bradycardia. History of carcinoid in stomach. How to reduce tumor growth?",
"src": "Patient: I am 40 Yrs old. Diagnosed @ 30 Yrs. for Aneamic, done blood transmission, diagnosed Bradycardia. Implanted Dual Chamber Permanent Pacemaker. In 2011, diagnosed with Carcinoid in Stomach. No Symptoms except weight loss of about 7 kgs in 8 months. Underwent partial Gastrectomy in June 2011. Again it has recurred now in March 2013. My present Sr. Gastrin level is 2445 & Sr. Chromogranin-A is 645. No metastasis as per Dotonac PETCT. IHC (Ki-67) is less than 2%. Is is a cancerous tumor. What is the medication for reducing the growth of tumor / Sr. Gastrin & Chromogranin-A levels and also what is the medication for controlling / curing the growth of the tumer. Also suggest the next course of treatment Doctor: Hello; Gastric carcinoid is low grade tumor meaning it is slow growing tumor compared to typical 'carcinoma.' They have long course and are less responsive to traditional anti-cancer therapy like radiotherapy and chemotherapy. Somatostatin analogues like octreotide LAR have been utilized to reduce the gastrin/hromogranin levels and have been shown to reduce recurrences, yet its use is not routine. You can discuss with your oncologist regarding this therapy.Another option is wait and watch with periodic DOTA-scan. Wish you good health."
},
{
"id": 188296,
"tgt": "Large, hard, tender knot in the area of deep filling on molar. Have temporary crown. Concerning?",
"src": "Patient: I had a deep filling on a molar performed on Monday and tonight I felt the area on the cheek side of my gum to see if there was any swelling and there is a large, hard, tender knot there. Is that a problem? I have a temporary crown on and am not supposed to go back to the doctor for three weeks. Could this just be from all the shots he had to use to numb me. It was very difficult to get numb as I can't use the anesthetic with epi. He had to apply lots of pressure to get the area prepped and then said he took several precautions to try and preserve the pulp. Should I be concerned? Doctor: thank you for posting your query.Looks like you had a very deep decay with respect to the tooth that was restored. During the procedure the pulp has been exposed and hence the lump. Get back to the doctorand get an xray done. This tooth will require root canal treatment.take care."
},
{
"id": 218808,
"tgt": "Can manual stimulation of a woman's vagina after ejaculation result in pregnancy?",
"src": "Patient: I had oral sex with my girlfriend and ejaculated on her chest and rubbed it , we were wet as were under the Shower.i fingered her using the same hand . She washed it with soap water immediately after that .she had her periods 6 days ago and she has a normal cycle .is there a chance she can get pregnant ? Please help Doctor: no. chances are negligible as procedure for intercourse did not happen. moreover she was in the safe time period as her periods ended 6days back"
},
{
"id": 175997,
"tgt": "What is the remedy for ADHD when on medication?",
"src": "Patient: I have ADHD and I dont feel like my medcine is helping verymuch. I am still very loud and distracted sometimes. I m already on the highest dosage of my medicine and take more at unch. How can I calm down at school so I can be less loud and more focused? Doctor: DearWelcome to HCMWe understand your concernsI went through your details. You must understand that medicines alone cannot cure mental disorders. ADHD is a mental disorder. To cure ADHD, you need to change your life style, attitude, thinking pattern etc. Cognitive behavioral therapy (CBT) is very much effective on ADHD. Rational emotive therapy too is useful. Meditation, mindfulness, exercise etc should also help you. Identifying and deep understanding ADHD is the first step of cure. Please consult a psychologist who is willing to treat in this manner.If you require more of my help in this aspect, please use this URL. http://goo.gl/aYW2pR. Make sure that you include every minute details possible. Hope this answers your query. Available for further clarifications.Good luck."
},
{
"id": 107804,
"tgt": "Suggest treatment for chronic lower back pain",
"src": "Patient: Hi , I am 66 years old . Two years ago I had a fall and fractured my lumbar bone . I had a vitobroplasty surgery done. After that , I had been having chronic lower back pain . The pain comes on when I stand or sit too long . It goes away when I rest .Is it ok for me to take Neurobion ? Doctor: Hi.after knowing your complains I feel this is a residual pain which is seen after most of the vertebroplasty. The metal implants most of the time benign a foreign body in your body cause various reactions which may ultimately cause low back pain. In my clinic with such scenario I advice some life style modifications like to avoid unnecessary n extra ambulation whenever possible, not lifting have objects n taking rest whenever possible in between the walks. Some painkillers like ibuprofen and some muscle relaxant like thiocolchicoside for a few days added with local painkiller gel application to avoid systemic side effects of above painkillers. Even after this precautions and medications pain persists, I would have advised the metal implant removal surgery n then an MRI to know the exact pathology inside. U can discuss with your orthopaedic surgeon the above options n get a proper management under his guidance. U should continue neurobion to prevent any further damage to your nerves. I wish you a painless back n healthy life."
},
{
"id": 186098,
"tgt": "How long after filling does tooth ache stop?",
"src": "Patient: I have toothache after a filling and she said the hole was quite deep and if it continues to hurt I may need root canal I'm scared but am I being inpatient? will the pain go away? it's only been two hours since the filling and the numbness has only just worn off Doctor: HiWith respect to your query, it look like you have irreversible pulpitis reasons for your pain might be due to inadequate removal of decay which is deep.if pain persists definitely you must undergo for root canal with cap which is the ultimate treatmet. Hope I answered your question"
},
{
"id": 107139,
"tgt": "What causes pain in the neck and lower back?",
"src": "Patient: I m trying to determine the diagnosis, causes and best treatments for my pain. My neck hurts,also upper middle and lower back, my hips hurt. When I lie on my back I get intense burning in my right thigh. I have atrial fibrillation on a continuous basis and have medium sleep apnea and use Cpap. I have nodules under the skin along my spine, thighs and arms which are painful to touch like bad bruises, I am white female 67 years and weigh 260 pounds. I eat very little but gain weight. In the past 3 years my mobility continues to deteriorate and I have poor balance and major difficulty going down stairs. Doctor: Hi, I understand your question and I will help youWith such cases in my clinic I would like to exclude serious conditions as Rheumatoid arthritis by doing certain laboratory investigation (Rheumatoid factor). Rheumatic nodules of (Rheumatic fever should be excluded also). Also I advise you to receive NSAID and muscle relaxant. I suggest doing X Ray for your lumbosacral spine and your hips."
},
{
"id": 19125,
"tgt": "Is Metoprolol an effective alternative to Atenolol?",
"src": "Patient: My Doctors nurse called me a couple of days ago to say Atenolol has been taken off the market. I have been on it for several years for my blood pressure. She said my Doctor has recommended that I now get Metoprolol. My Atenolol dosage was 50mg once a day. The substitute is 100 mg per day. I am 75 years old and have been on high blood pressure meds for more years than I can remember. In addition to the Atenolol I take Lisinopril 40mg/day, Hydrochlorothiazide 25 mg/per day, Amlodipine Bestylate 5mg/day, Atorvastatin 40mg/day and once a week Alendronate70mg once a week. I take a Multi Vitamin as well as other supplements daily. Please advise me as to the change in this drug to another and the dosage. The price of drugs is a major concern for me as my Medicare has not increased in the last 3 years. I appreciate your expertise and help in this matter. Thank you, Pat Malcheski mrs Doctor: Hello Yes both metoprolol and atenolol are equally effective as they both belong to Beta blocker group of medicine.Don't worry you can safely go with metoprolol.Thanks"
},
{
"id": 195518,
"tgt": "What does this semen analysis report indicate?",
"src": "Patient: hi i got a semen analysis done the result is volume 2ml, color greyish white, viscosity viscous, Ph 8.0, sperm count 140 million, pus cells 5-6/hlf, epithelical 2-3/hpf, actively motile 50%, sluggishly 10%, dead sperms 40%, morphology normal 80%, abnormal head-05%, neck-10%, tail-05% pls advice on the report Doctor: Hello and Welcome to \u2018Ask A Doctor\u2019 service. I have reviewed your query and here is my advice. Your semen analysis report shows good total sperm count with 50% motile sperms which is also quite good number but still there are 40% dead sperms which might be due to mild pyospermia (pus cells in semen). Pyospermia should be treated if it is severe which means more than 10-15 cells / HPF. Usually a course of antibiotics gives excellent results. I would like to know more about your history any symptoms plus the need for this semen analysis. Hope I have answered your query. Let me know if I can assist you further."
},
{
"id": 195753,
"tgt": "Does sulfa/trimo treat epididymitis?",
"src": "Patient: HI, i am a male 35, 160 lbs and healthy.I have been diagnosed with epididymitisdue to a bacteria infection. Taking sulfa/trimoantibiotics for 2 weeks had to go off cipro because of bad side effects. Should the sulfa/trimo antibiotics cure this? If not what is the next step?thanks. Brandon Doctor: Hello and Welcome to \u2018Ask A Doctor\u2019 service. I have reviewed your query and here is my advice. It depends up on the organism.In some patients who have been treated with other antibiotics sulfa might work.Again it should be taken till your infection subsides. So consult an urologist and take treatment according to his guidance and supervision. Hope I have answered your query. Let me know if I can assist you further."
},
{
"id": 129461,
"tgt": "What causes sudden hot sensation and tingling sensation in the arms and legs?",
"src": "Patient: so i havent been getting enough rest.i was driving home from work when all of a sudden i had to pull over do to feeling being constricted and hot and my arms and legs were going tingly and numb. and my hands started jerking and stiffing up and i couldnt relax it. Doctor: It sounds like you are suffering from stiff person syndrome more common in females could be caused by several causestriggered by severe emotional stress vitamin B12 deficiencies autoimmune where anti bodies against nerve protein Amphiphysin formed or neural chemical Glutamic acid decarboxylase deficiencyit could be feature of lung or breast cancersmuscular tetanyor epilepsy or focal seizures it is good to see your doctor to get investigated muscle relaxants like baclofenvitamin B12 and Bcomplexcalcium and Vitamin Danti epileptic medicines gabapentineor IV immunoglobulins or immune modulating drugs all are helpful and should be prescribed by your doctor based on the cause of your symptoms.psychosocial support and counseling would be helpful wish you good luck."
},
{
"id": 107138,
"tgt": "What causes worsening burning pain after an MRI scan long after a back surgery?",
"src": "Patient: dr wagner is treating me! he did surgery on my back 2014 but I was in recently to see him ! he did an mri ! my symptoms have become worse and some other unusual things are occurring ! I am concerned because I don t see him til march 30th !~ pain and burning have increased and spread! problem because I am taking ibiprophen but found out that it may not be good for me taking that either ! any advice for me til I see him again! very worried! another doctor had to do other surgery to remove csyt too at same time and now I had a recent occurrence and Ive already been through menopause as far as I know! Doctor: Hi., I understand you question and I will help yoy If I were your treating doctor I will ask for new MRI as It will demonstrate the cause of recurrence of pain (failed back surgery or not) I advise you to receive muscle relaxant , Indeomethacin and to wear a lumbosacral belt"
},
{
"id": 54028,
"tgt": "Is an IVC filter necessary for a paraganglioma removal?",
"src": "Patient: When is an IVC filter necessary for surgery? My daughter is having surgery to remove a paraganglomia near her liver. My daughter has a blood clotting discorder of factor 5 and low protein s. She has not had any clots nor is she on any blood thinning medicine. Her father has a blood clotting disorder and is on coumidin to regulate it. The surgeon is suggesting heprin after surgery and then lovenox for 21 or 28 days. Is this appropriate? Doctor: Hi and welcome to Healthcaremagic. Thank you for your query. I am Dr. Rommstein and I will try to help you as much as I can. I suggest it too since surgery is in abdomen and prolonged resting may be required. She has postive family histroy of clot disorders and this is not worth risking. I hope I have answered you query. If you have any further questions you can contact us. Kindly regards. Wish you a good health."
},
{
"id": 100323,
"tgt": "What is the remedy for mosquito bite allergy?",
"src": "Patient: sir, my age is 23 my height 167cm..from childhood onwards i have mosquito bite allergy. many creams i tried but no use. at present i am using fusiderm-b cream by doctors advice. is this cream will help me to get rid off my mosquito bite scars?..or there is any other good remedy to fade off my scars...i am from kanyakumari, tamilnadu..waiting for ur reply please help me . Doctor: HelloThank You for contacting HCM.Yes it will improve the it. Meanwhile for recurrent allergy i would suggest you to take take montelukast one daily for one month. Research has shown that it decreases the allergic symptoms considerably.Hope this answers your question. If you have additional questions or follow up questions then please do not hesitate in writing to us. Wishing you good health"
},
{
"id": 33477,
"tgt": "Can recent blood works show past Chlamydial infection?",
"src": "Patient: Three years ago, before i met my husband, I was diagnosed with Chlamydia. I have been treated and it is gone. I am now pregnant, had my first OB appt. they did the DNA probe for Chlamydia and Gonorrhea, and a lot of blood work. Will these tests show I have had it in the past, even if I don't have it anymore? NO ONE knows I have had it once. Not my OB, not my husband. Doctor: HI, thanks for using healthcare magicThere are tests that can detect present infection or past infection. It is likely that your doctor is looking for present not past infection.It is not likely that the test would detect the previous infection.I hope this helps"
},
{
"id": 71229,
"tgt": "Does pleurisy cause pressure in chest?",
"src": "Patient: I have a pain in the middle of my chest plate area, and in the past, I ve been diagnosed with pleurisy. I m not sure if that s what causing that, but yesterday, it was fine. I m not sure if sleeping in the wrong way may have caused that, but it s been bothering me when I lift up my arms, or when I breathe a bit deep. It s like there s a pressure in the middle of the chest plate area when I breathe deeply. I m 20 years old, and 200 lbs. I m 5 7. Doctor: Hello and Welcome to \u2018Ask A Doctor\u2019 service.I have reviewed your query and here is my advice.No, I don't think your current symptom is due to pleurisy in the past. The possibility of musculoskeletal pain is more likely. Incorrect sleeping posture can cause a similar kind of pain. Your pain is associated with movements and specific positions. So follow these steps for better symptomatic relief in musculoskeletal pain. 1. Avoid incorrect sleeping posture. 2. Avoid heavyweight lifting and strenuous exercise.3. Apply warm water pad on affected areas of the chest.4. Sit with proper back support.5. Take simple painkiller like paracetamol or ibuprofen.Don't worry, you will be alright with all these. Hope I have answered your query. Let me know if I can assist you further.Regards, \u00a0\u00a0\u00a0\u00a0\u00a0Dr. Kaushal Bhavsar"
},
{
"id": 53270,
"tgt": "Can cholecystectomy be performed safely after suffering from cardiac arrest?",
"src": "Patient: my mum has a difibulator and gallstones.She was admitted last month with gallstone pancritus and had six cardiac arrests as they were pushing iv fluids into her.She is now home and following a no fat diet but is worried she needs to have the gallbladder removed and has been refered to another hospital as the surgeon at her local hospital wont now do the surgery.Will she be able to get the gallbladdr removed Doctor: Hi and welcome to Healthcaremagic. Thank you for your query. I am Dr. Rommstein, I understand your concerns and I will try to help you as much as I can.Yes, cholecystectomy can be performed if there is no current heart disorder such as arrihmias or low ejection fraction. So you can have surgery but you should see cardiologist preoperatively and anesthsiologist will judge if there is danger from general anesthesia and procedure.I hope I have answered you query. If you have any further questions you can contact us in every time.Kindly regards. Wish you a good health."
},
{
"id": 198773,
"tgt": "Suggest treatment for blood in semen",
"src": "Patient: Hi, I m a 22 yr old male. I have had blood and blood chunks in my semen for as long as I can remember (about 8 years). I have never had pain. It is normal colored for only about a week at a time and this happens about once every 4 months. I don t really want to go in and get checked right away because I m not in any pain or discomfort. Since this is an abnormality that has always been present, I was hoping you could recommend any medication for me to start?? Thanks Doctor: DearWe understand your concernsI went through your details. Seeing blood in the semen can make a man anxious. Fortunately, it doesn't always signal a major medical problem. For men younger than 40 with no related symptoms and no risk factors for underlying medical conditions, blood in semen usually disappears on its own.In your case, you are 22 and you are frequently witnessing the problem. I suggest you not to oversee this and consult an urologist for further investigation and treatment.If you require more of my help in this aspect, please use this URL. http://goo.gl/aYW2pR. Make sure that you include every minute details possible. Hope this answers your query. Available for further clarifications.Good luck. Take care."
},
{
"id": 41264,
"tgt": "What causes infertility after miscarriage?",
"src": "Patient: I am Anita Jha \"32years old . I had a baby girl of 8 years old . Twice I had a miss carriage But since 3 years I am planning , but I am unable to consume now . I had my Follicular Test since from four month every thing is normal Kindly advice me reason for not consuming . My weight is 70 kg , height is 5.3 . Doctor: Hello,Welcome to Health Care Magic.Thanks for writing.I am Dr.Nitasha Khan. I have read you question completely, i understand your concern and will try to help you in best way possible.infertility is very common after miscarriage and can take a while to cncieve but nthing to worry about .....you wil soon m sure just do your cmplt work up like ultrasound etc and also do your husbands work up as wel its v imp even if you have children before .....do al the investigations m sure you wil be fine and wil cncieve soon mean while live with your husband have intercourse on ovulating days you wil be fine I hope this answered your question.If you have more queries I am happy to answer.Otherwise rate and close the discussionRegards.Dr.Nitasha Khan"
},
{
"id": 86103,
"tgt": "Can Dicyclomine be taken for abdominal pain while suffering from diverticulitis?",
"src": "Patient: Hi dr. I suffer from diverticulitis, I was in the hospital for 3 days March 13 . I was put on. Gluten free diet by my Dietition and I'm slowly introducing some fiber into my body. However, I saw my GI dr. Last week, I was feeling good until I started taking Metamucil almost a week ago which he told me to do. I have some Dicyclomine hci 10mg here, which my other GI dr had prescribed when I was seeing her. Do u think the Dicyclomine would hurt me if I took one for this belly ache, it's not severe pain but I know it's there. Can u pls give me your thoughts. Thanks in advance. Doctor: Hello and Welcome to \u2018Ask A Doctor\u2019 service. I have reviewed your query and here is my advice. There is no evidence that dietary modifications are helpful in the management of diverticulitis. If you have simple diverticulitis (no abscesses or perforations) or fever, I suggests that you stay on semi-solid foods (soup, yogurt, etc.) for a week or so, and take over-the-counter pain medication, preferably aspirin. To prevent recurrent diverticulitis, I advise a high-fiber diet, and abstinence from smoking and NSAID use. Anecdotally, Rifaximin, which is a locally acting antibiotic that is poorly absorbed from the gut, may provide symptom relief. Hope I have answered your query. Let me know if I can assist you further."
},
{
"id": 197007,
"tgt": "Suggest treatment for azoospremia",
"src": "Patient: Hi, I m 37 yrs old (183cms/75kgs), got married 9 yrs ago, no kids, tested and found azoospermia(less semen counts), underwent varicocele surgey 18 months back, no improvement, wife(31 yrs old/58 kgs) is alright. What is the alternate solution to give birth to a child? Doctor: you should go for a testicular biopsy.. maybe the biopsy will reveal sperms in the testicular tissue that could be used for an IVF cycle.. IVF might be the only possible way for you to have children..goodluck"
},
{
"id": 81673,
"tgt": "What causes rib cage pain with difficulty in breathing?",
"src": "Patient: pain in right side upper quadrant, just @ rib cage,feels like there is pressure there, onset pain started last week, on a 1-10 scale pain @ 3/4 just bad enough to take motrin several times a day, may have been onset from sneezing attack, bothers me more when sitting than standing , i do have dr appt tomorrow. breathing is somewhat difficult on r/s, no other issues, no pain anywhere else, all other body functions ok, please advise, i know you can not positvely diagnois over internet but opinion would be great Doctor: hellowelcome to hcmthere can be following possible reasons for your pain on right side of chest(1) PLEURISY i.e inflammation of the outer covering of lungs(2) PNEUMONITIS i.e infection of the lung tissue(3) COSTOCHONDRITIS i.e inflammation of cartilages of rib cage(4) MUSCULOSKELETAL due to strain/sprain of the muscles of the rib cagehowever precise diagnosis can be made by physicaql examination and from getting a XRAY OF CHEST which im sure your treating physician will do before prescribing you the appropriate treatmentregardsdr rahul rathee"
},
{
"id": 167166,
"tgt": "What causes vomiting and high temperature in child?",
"src": "Patient: hello i have a 17 month old baby girl she has been ill the last 5 days with been sick vomiting not keeping any thing down had high temperatures she will only eat and drink when she wants to and when she does it dont stay down for long she has bright green mucus coming out her nose one min she is ill the next she is her normal self we have had her to hospital and the gp surgery and i just feel were not getting any were and i am starting to panic about my little one this is not like her. Doctor: Hi...it might be a prodrome or starting stage of viral diarrhea.As of now for symptomatic treatment you can give Syrup Ondansetron.She might develop diarrhea and if that occurs, keep giving her ORS and zinc supplements might help.Regards - Dr. Sumanth"
},
{
"id": 169672,
"tgt": "Does Alpen need to be given once more?",
"src": "Patient: My daughter had 2,3 half cm worms coming out of her anus.she was given 10ml single dose of albendazole ip of 400mg under brand name alpen.Next morning in her stool she had live worms.Is it necessary to give her another dose to relieve her of all the worms.also is the worms related to her eating ready to eat noodles often.Thanking you Doctor: Hi,Albendazole is very effective against all types of worms.It will take some time to eradicate all worms from the system so you can wait for some time to get all worms eradicated from GI tract.One course is enough but you can repeat the course after 1-2 weeks if required.Keep her nail well cut and maintain good nails and hand hygiene.Ok and take care."
},
{
"id": 38574,
"tgt": "Can rabies be transferred through blood?",
"src": "Patient: Can rabies be transferred through blood? I may have ran over something and when I came inside after cleaning it I forgot to wash my hands and began playing a video game then I realized I needed to wash my hands so I did then I came back and forgot to clean my mouse and I have a bad habit of biting my nails. So if the animal did have rabies could I have rabies now? Doctor: Hello thanx to contact healthcare magic. If I am your suggestion giving Doctor I suggest you that you should not be worry regarding rabies. Rabies only spreads through damaged nerves, which happens in the case of biting by dogs. But it wont spread by biting of nails. But if suppose when you biting the nail you got injured the fingerbed you might be infected with rabies. But I think you have not injured your finger during biting the nail so do not worry regarding that. If you have anything else to ask kindly contact me Dr Arun Tank, Infectious Disease specialist."
},
{
"id": 18190,
"tgt": "Does strenuous exercise cause high troponin levels?",
"src": "Patient: I ran a twelve mile run a couple of days ago. 5 hours later I started to feel funny. Felt a little unstable standing, started seeing spots, felt nauseous, started shaking. Went to Er. Said I was dehydrated. My sodium was a tad low and I have had hyponatremia earlier this year. I had two electrocardiograms that were perfect. No chest, shoulder, arm, jaw or neck pain. They did a test and I had a very high troponin level. Two cardiologist at other larger hospitals would not take me. One even questioned the ER doctor on why he did the test. He said it wasn t a heart attack. I have read that strenuous exercise can cause a high troponin reading. Is this true? I have been scared to death. Doctor: Hello and Welcome to \u2018Ask A Doctor\u2019 service. I have reviewed your query and here is my advice. Troponin is an enzyme produced during muscle damage as in case of heart attack. It is also produced during strenuous physical activities as in your case. prolonged running might caused some trivial damage to your calf muscles and that might have caused high troponin levels. Nothing to worry and no treatment is required. Drink plenty of water and take adequate bed rest. Hope I have answered your query. Let me know if I can assist you further."
},
{
"id": 196972,
"tgt": "Does masturbation affect the medical checking examination?",
"src": "Patient: Hi, 'm 21 years old and height 180cm weight 62. I masturbate daily and some time it leads to 3-4 times a day. I feel very bad about it. And 'm very much passionate towards armed force i want to join indian Army. But i do masturbate daily, and if i get select will it effect in my medical checking exam?? Doctor: masturbation in an excessive manner makes you weak as it drains the calcuim and protein content of your body.. protein is the building block of ur body.. so masturbation makes you lean and affect metabolism of your muscles.."
},
{
"id": 23773,
"tgt": "What causes giddiness with high blood pressure?",
"src": "Patient: dear sir, I have giddiness past three years, every day after getting up from bed and after having shrot nap i feel slight gidiness, i had very bad B.P, and i cheak my B.p. every month and i taking aten25 tab, now i have gidinness past 2 weeks now my doctor priscribe colpid, stuguron Doctor: giddiness with high BP cause are 1) uncontrol BP 2) side affects of drugs like aten/beta blockers induce bradycardia etc 3) autonomic dysfunction,check sugars,BP postural hypertension 4)cerebellar signs, cervical spondolysis 5) heart disease check ecg/echo first of all kindly control your BP strictly"
},
{
"id": 144701,
"tgt": "What causes fizzing noise in the back of head?",
"src": "Patient: Hi I have a fizzing noise in the back of my head right where the neck it is connected I ve had this for on and off possibly a year so I can t really remember exactly what happened but I was wondering if it is because of my acid reflux and air just trying to make its way out? Doctor: You could be having ear problems or possibility of Migraine. Consult an ENT Doctor and a Neurologist. Avoid smoking and drinking alcohol. Avoid junk food and colas."
},
{
"id": 113848,
"tgt": "What is the cause of pain in the lower back and leg ?",
"src": "Patient: My son is complaining about lower back ache and and left leg. Three doctors has already told him that he has a Herniated disc. He went to a Chiropractor today and he saids that he does not have a herniated discs and that his leg hurts and has little strengh because a nerve is pinched. Based on this mRI reading attached please give me your opinion Doctor: Hi...your mri file i cannot access since its not there...but what looks like is your son is suffering from sciatica...get him neurobion(vit B) injections 5 in total every alternate day...and sometimes without any surgical intervention the problem shall come down in intensity...but for long term treatment surgery might be needed depending on the location of nerve entrapment."
},
{
"id": 110312,
"tgt": "What causes discomfort/tightness in lower back",
"src": "Patient: I have discomfort in left side lower back which tightens and makes my legs jerk. I have constant leg pain in different locations\" knee, foot, ankle. Does this sound like a spine? age 62 height 5'4\" weight 190 I take mirapex for restless let symdrom Doctor: Hello, I have studied your case. Due to compression of this nerve root in spine there is pain in your thigh and leg.I will advise you to do MRI spineFor these symptoms analgesic and neurotropic[pregabalin] medication can be started.Till time, avoid lifting weights, Sit with support to back. You can consult physiotherapist for help.Physiotherapy like ultrasound and interferential therapy will give quick relief.I will advise to check your vit B12 and vit D3 level.Till time you can continue your medication.Hope this answers your query. If you have additional questions or follow up queries then please do not hesitate in writing to us. I will be happy to answer your queries. Wishing you good health.Take care."
},
{
"id": 98860,
"tgt": "What causes hives breakout along with pain in wrists and forearms?",
"src": "Patient: i have been haveing what the drs and i thought was allergic reaction too somethin unknown since my daughter was 6wks old 5 yrs ago now when this happens i break out in hives swell sometimes and now i am suffering with painfull wrists forarm shoulders and sometime my knees Doctor: You will need a detailed evaluation as the cause for your allergic reaction needs to be identified. As sometimes allergic reaction can be life threatening called as anaphylaxis. So kindly consult a allergist and it is advisable for you to undergo skin prick test to identify the allergen."
},
{
"id": 29172,
"tgt": "What causes pain in the scalp, legs and back?",
"src": "Patient: I WOKE UP THIS MORNING AND THE TOP OF MY HEAD WAS SORE TO THE TOUCH, NOT PAINFUL UNLESS i TOUCH IT....THE ENTIRE SCALP. IN JUST A FEW HOURS I BEGAN TO HAVE ACHING PAIN IN MY LEGS, ARMS, BACK, ALL OF MY MUSCLES NOW ACHE. i AM TIRED AND HAVE CHILLS HOWEVER, i DO NOT HAVE A FEVER? Doctor: HI, thanks for using healthcare magicThe chills indicate that there is a change in body temperature even if you are not detecting a fever.Based on the symptoms mentioned, it is possible that you have a viral infection. Most viral infections can cause these symptoms but you there are no symptoms mentioned that point to a particular virus.The most common viral infections are the cold/flu and gastrointestinal (causing vomiting and diarrhea)Since the symptoms are so severe, it may be best to be assessed.I hope this helps"
},
{
"id": 15603,
"tgt": "Have itchy scalp, neck, arm, upper back. Have rashes, bumps. Is is burning also. Had used iodine supplement. Treatment?",
"src": "Patient: Hi, this morning, I started itching my scalp and then my neck, I then started itching my upper arm, my upper back, I have rashes and a lots of bumps and ridge looking bumps that just popped up about an hour ago, the itching hasn't stopped but getting more and now burning, I haven't changed soap or lotion or used new medication, I only started using iodine supplement 3 days ago, can you please help Doctor: Hello,Taking Iodine can worse rash caused by Dermatitis Herpetiformis.Other than that Iodine is safe and doesn't cause any problem.You may take antihistamines to control the itching and rash.Consult a Dermatologist for further management."
},
{
"id": 134649,
"tgt": "What causes persistent pain in left arm?",
"src": "Patient: Hello, I am a female, age 53. I am feeling pain in my left arm and same side in the back ( whole back up to half)! The pain is more severe in the night and some time during the day! It comes for few days and goes away. This time it is continues from last week! I am under lots of stress. Is it spasm? Doctor: Slight tearing or spasm of connective tissue around upper back region can cause pain in upper back pain and may radiates to surrounding areas typically as a result of bending, lifting, twisting or shoulders forwards activities or trivial movements, such as picking up a pencil or sneezing. May cause upper back pain with or without symptoms radiating along shoulder, arm, forearm, hand or fingers. Pain may increase on firmly touching the affected level of the spine and spinal movement is often stiff or restricted. Pins and needles or numbness are occasionally present in affected arm or hand. Symptoms may increase with repetitive or prolonged sitting (especially slouched sitting or during computer use), bending, lifting, shoulders forwards activities, coughing or sneezing and are often worse first thing in the morning."
},
{
"id": 13164,
"tgt": "What are causes & remedy for spreading rashes on face with fever?",
"src": "Patient: My frnd has been taking trika for more thn 1 yr. He has been taking it like 10 to 12 pills a day n sometimes he also takes alchol along wid trika... He has memory prob difficulty in spking n sleep prob... Is he at any kind of risk n wht is d solution out for this prob Doctor: Hi,Alprazolam is a sedative antidepressant. It should be taken as 0.5 mg tablet at night. Your friend is taking excessive dose... And also with alcohol. Tell him to consult psychiatrist and get the proper dose. High doses might harm. ...okHope I have answered your query. Let me know if I can assist you further. Dr.Ilyas Patel,Dermatologist"
},
{
"id": 213269,
"tgt": "22 year old girl, shy, anxious behaviour, frequent urination. Prescribed risdone, tranxene. Safe to take?",
"src": "Patient: hi, my girl 22 years, is anxious, shy and goes frequently to toilet. she is in holiday in another country and the local doctor prescribe risdone 1 mg, for 7 days, he prescribe too Tranxene 5 mg for 7 days and Lonoxyl drops....are these medicines can be taken for this kind of problems??? can she take these medicines and stop after 7 days??? thking you in advance! Doctor: do not take risperidone. other medicine take as before. and take fluvoxin cr 50 mg"
},
{
"id": 160953,
"tgt": "Suggest treatment for chest pain in kid",
"src": "Patient: My 10 year old son started last night complaining of chest pains and feeling really dizzy. He was in tears. He went to bed and was fine this morning. The school called and said he is complaining of the same thing. Should I be worried? Do I need to take him to the Dr? Doctor: Hi, At this age chest pain will not be serious don't worry, but I would like him to be evaluated first for mitral valve prolapse. If it is still persisting then costochondritis has to be considered.Hope I have answered your query. Let me know if I can assist you further. Regards, Dr. Sumanth Amperayani, Pediatrician, Pulmonology"
},
{
"id": 96336,
"tgt": "Is there any cure for Medical Renal Disease?",
"src": "Patient: Is there any cure for S.creatinine showing 5.2% and sonography indicating Medical Renal Disease ? Doctor: Hi!!! Welcome healthcaremagic community forum. High serum creatinine indicates renal failure. Details of symptoms, past medical history and clinical examination findings are essential to determine if it\u00e2\u0080\u0099s an acute or a chronic renal failure. Acute renal failure can be reversible. Adequate hydration, blood pressure corrections and treating the cause will normalize the creatinine values. On the contrary, chronic renal failure is a persisting condition which can be irreversible. Discuss with your doctors for a specific treatment plan."
},
{
"id": 190505,
"tgt": "Crown placement in the mouth. On amoxicillin. Developed numbness, tingling, cramping and vertigo. Swollen glands. Due to infection?",
"src": "Patient: I had a crown placed about 1 month ago and have not felt well since; I was placed on Amoxicillin 850 mg 2x/day. I developed numbness, tingling , and cramping in the extremities and then developed a vertigo . I constantly have an electric type feeling along and underneath my tongue and lower jaw. I also have swollen glands. My dentist says its nothing and I have to get used to the crown. I think it is an infection? Thoughts? Doctor: hello and welcome to HCM forum , i am glad you wrote in, according to the dental history provided by you it seems that the tooth still has infection due to which you have been in pain, i assume there is no swelling around the tooth, but if there is then kindly revisit your dentist and get the crown removed. i am not sure about the numbness and tingling in the extremities,but its not due to the tooth for sure. for this you will have to consult a neurologist. swollen glands is something which needs to be checked thoroughly, i would have appreciated if you would have attached an x-ray of your RCT treated tooth. on the whole, crown is nothing to do with pain , it is the infection which has not resolved yet, therefore kindly visit a specialist(endodontist) soon. i wish you good health, take care."
},
{
"id": 148863,
"tgt": "Why do I get a warm fuzzy feeling when it rains ?",
"src": "Patient: Why do I get a warm fuzzy feeling when it rains ? When it s precipitating and gloomy outside, I feel warm and energetic inside but any other sunny day I feel depressed and bored. Is this common? Doctor: Welcome to HCM!I have read your query carefully and I can say from my clinical experince that you have mood problems associated to seasonal changes. In usual seasonal disorder, one has gloomy feeling and lethargy during winters and cloudy weather. In your case, you feel better the other way round, which is quite possible, it can vary with different people.I think you should meet a psychiatrist as it is well treatable.bupropion 150-450 mg is usually seen to have good response in this illness.Hope the reply is helpfulFeel free to ask more questionsDr. Manisha Gopal"
},
{
"id": 87531,
"tgt": "What causes pain in the abdomen?",
"src": "Patient: I am having a sharp pain on right side of abdominal and pressure in the bottom stomache area with bad back pain lower and now vomitting i have gone to doctor and they say nothing is wrong now my chest is hurting I am not sure what to do I am in pain and the doctors say nothing is wrong. Doctor: Hi,Looking to the symptoms possibility of having acute appendicitis is there.You might require ultra sound of abdomen to rule out appendicitis or other lesion in abdomen.Clinical examination while having pain will give some clue about your problem.Meanwhile take diclophenec, paracetamol combination medicine as needed.Ok and take care."
},
{
"id": 62667,
"tgt": "Suggest treatment for a lump on the nipple",
"src": "Patient: I am 18 years old and my problem is that my left breast nipple has become lumpy and sore. When I squeezed it a whitish fluid came out and a little trace of blood. I had faced a similar thing 2 months ago when I was having a missed period because of stress.I've had normal periods after that.Is the nipple discharge because of some infection? Please help Doctor: hi.it is best if you consult with a doctor, preferably a general surgeon, for medical and physical examination. based from your description, it could be a cyst, a benign breast lesion. it could also be a fibroma or fibrocystic lesion. these lesions are best evaluated clinically. further diagnostic examinations, such as breast ultrasound, will be requested as needed. regular breast examination and follow-up with your doctor is highly recommended in these cases. management (medical and/or surgical) will be directed accordingly.hope this helps.\u00a0\u00a0\u00a0\u00a0\u00a0good day!!~dr.kaye"
},
{
"id": 72310,
"tgt": "Suggest treatment for tuberculosis",
"src": "Patient: dr i am 30 yr old, when i went to my dr he ask me to go for all test, i did all test are normal but IGM test was positive, then my dr ask me to go for mox sken test for TB and that is positive . he had given me AKT 4 ( rifampicine, isoniazide, pyrazinamde and ethambutol ) in due corse i got my utp positive as on today the growth of baby is arround 8 to 9 week , my worry is is my baby will be good and healty will there be any problems Doctor: Thanks for your question on Healthcare Magic.I can understand your concern. Yes, your baby will be healthy and no need to worry much. AKT 4 is having rifampicin, pyrazinamide, isoniazid and ethambutol. All these anti tubercular drugs are safe in pregnancy. These drugs don't cause any side effect to developing baby or mother. So nothing to worry about these drugs. Just finish the total course of 6-8 months and you will be cured.Maintain good nutrition by taking protein powder and multivitamin tablets. Drink adequate water. Hope I have solved your query. I will be happy to help you further. Wish you good health. Thanks."
},
{
"id": 116052,
"tgt": "Is nausea and lightheadedness related to high blood pressure?",
"src": "Patient: I'm a 16 year old girl, over the last few days I've been very tired and week. I've also experienced constant nausea and light-headedness. It worsens when loud noises are made. I went to the obgyn for an annual visit yesterday, she said my blood pressure was 90/50. I was wondering if this could have anything to do with low/high blood pressure? Should I seek medical attention? Doctor: Hi,Thanks for asking.Based on your query, my opinion is as follows.1. At present, you seem to have low blood pressure.2. The blood pressure, at present is very low which could cause light headedness. This could be secondary to dehydration and constant nausea could be secondary to this.3. Also get a complete hemogram done to evaluate for anemia. More fluids and salt intake necessary to improve blood pressure. concentrate on nutrition.Hope it helps.Any further queries, happy to help again."
},
{
"id": 91752,
"tgt": "Should I be alarmed by severe abdominal pain after taking laxative when having two hernias?",
"src": "Patient: I took a laxative about 4 hours ago. Now I feek like I may vomit. I have abdominal pain on my left hand side of my stomach. It is not severe pain, but uncomfortable enough for me to notice it. I am afraid because I do have two hernias in my abdominal area. There is one above the navel and one below my navel. Should I be alarmed? Doctor: Hi.Laxatives itself can cause such problem.The para-umbilical hernias which you have mentioned may not cause this problem. Do not think about them as a cause unless there is non-reducible swelling with local pain at the hernial site.Visit ER if the problem increase or the loose motions continue / vomiting / fever / loose motions."
},
{
"id": 45544,
"tgt": "Does a ureteral stone get cured on its own?",
"src": "Patient: hello. yesterday i had a sonography. result hydronephrosis and hydroureter. the Doctor who done my sonography he said 3.5 mm stone in my ureter, will it pass by own.. currently i am feling no problem in passing of urine and drinking loads of water.. Doctor: Hello, Stones less than 4mm usually pass by itself. As of now drink plenty of water as it will help to flush the stone away. If symptoms persist or in case of obstruction, surgical removal of the stone may be needed. Hope I have answered your question. Let me know if I can assist you further. Regards, Dr. Shinas Hussain, General & Family Physician"
},
{
"id": 192721,
"tgt": "What does recurring tears and dry skin on foreskin indicate?",
"src": "Patient: hello. the foreskin of my penis gets dry and there are some tears there that seem difficult to heal, i have tried not to \"annoy\" the area with sex for 2-3 days and seemed to heal (just the dry and some red remained) but the tears came back with just a very little \"action\". what should i do and what can i apply on the area in order to moisten it but not harm it? (i heard something about clean aloe cream). Thank you!! Doctor: Hello,It may be due to dry skin. For that you can apply hydrocortisone ointment locally. In case of itching take tablet loratidine. Avoid sweating. Avoid hot and humid environment. Maintain proper hygiene.Hope I have answered your query. Let me know if I can assist you further. Regards, Dr. Shyam B. Kale, General & Family Physician"
},
{
"id": 173105,
"tgt": "Do sacs in diaper indicate worms?",
"src": "Patient: I just changed my 10.5 month olds diaper and found something that looked like the sacs of juice in an orange. He hasn t eaten an orange in weeks. He is currently on antibiotics for bronchitis. Should I be worried about these sacs? Could they be worms? Doctor: Hi, I had gone through your question and understand your concerns. You should be worried but not too much. You may easily exclude it by giving stool analysis for ova and cysts.If test were positive i would prescribe Albendazole syrup-Zibend 5 ml once time. The other possibility is undigested food due antibiotic therapy. In this case you can use probiotics- babylac for 1 month. Hope this answers your question. If you have additional questions or follow up questions then please do not hesitate in writing to us. I will be happy to answer your questions. Wishing you and baby good health."
},
{
"id": 33312,
"tgt": "Suggest treatment for right lymph node TB",
"src": "Patient: my wife is suffering from right lymph node TB after found +ve in FNAC report of Right Lymph node. she was on dots treatment for one month.but she had severe side effects like severe vomitting, joint pains, severe fever of 102-105(F).aong the combination pyrizidine was most severe on her. Once after taking it she developed uneasyness, fever of 106(F), joint pains, severe headache. she has now stopped taking DOTS treatment. she is now taking BICOX-E medicine which does not contain Pyrizidine but it contains the rest of the combination as in DOTS treatment and has also a different strength. My question to u is whether this drug is sufficient enogh to cure my wife's right lymph node gland TB. Please answer SIR, waiting eagerly for ur reply. Doctor: HiThanks for your query at HCM.I understand your concern and situation.The tubercular drugs must not be stopped at any cost. Your doctor has done the correct thing removing pyrazinamide. She must complete 12-15 months of treatment. If even 2 drugs are effective the cure is achieved nothing to worry. In my opinion one fluoroquinolone may be added in the regime and would be quite effective. Eat protein rich healthy and balanced diet and drink plenty of water. Do take multivitamins and calcium supplement regularly.Hope I answered your query.Best of healthDr Sheetal Verma"
},
{
"id": 26307,
"tgt": "Do i need a special dentist if i am on heart medicine?",
"src": "Patient: Lost a tooth/cap and will have to see a dentist. Do I see a special dentist, because 10 years ago I had an emergency AAA surgery and pressure of some sort cracked rear teeth during the long operation. I haven't been to the dentist and am heart medicine. Doctor: No you don't need a special dentist for the same, however a skillful and senior dentist should be preferred. Also the dentist should be well versed with precautions to be taken in a heart patient and knowledge about infective endocarditis prophylaxis. Regards Dr Priyank Mody"
},
{
"id": 105917,
"tgt": "What are the after effects of using an inhaler post the asthma attack ?",
"src": "Patient: after a severe asthma attack over the weekend where i had to use my inhaler just about every hour, i have experienced some voice loss and pain in my throat that can be described as feeling like someone is cutting my throat with a sharp knive. i find it difficult to swallow and can only eat really small portions at a time. are these the after effects of the asthma or the inhaler? Doctor: Hi, Thanks for query, The problem you had might be due to inhaler as frequent use of inhaler there is dryness of mouth and local effect of inhaler content. Use inhaler when needed the most. Take more water,and along with food have sufficient water. During attach take liquid diet. ok and bye."
},
{
"id": 24151,
"tgt": "How to treat tiredness and sleepiness post heart by-pass surgery?",
"src": "Patient: My husband is heart patient went twice heart by-pass surgery My husband is a heart patient & twice operated by-pass surgery & in oct got 2 stents in main art.now a days he is feeling sleepy all the time & lost interest in all the activities . What to do? Doctor: Hi,He needs some examinations to find out the cause of his tiredness to receive correct and effective treatment. I would advise you to monitor his blood pressure, heart rate and glucose for couple of days. If the data are normal, then he will need to have complete blood count and serum electrolytes to find out the cause. The further strategy will depend on results.Take care"
},
{
"id": 79269,
"tgt": "What is the treatment for recurrent URTI?",
"src": "Patient: Recurrent URTII have a recurrent URTI presenting with chesty cough, with whitish sputum and nasal discharge on and off for 2/52 duration. Consult a doctor and was given antibiotic (Amoxacillin) and Diphenhydramine Hydrochloride.But the cough doesn't improve at all. What should I do? Can i go to the hospital and ask for SAFB? Doctor: Thanks for your question on Health Care Magic. I can understand your concern. Recurrent cough should be ruled out for tuberculosis. So better to consult pulmonologist and get done clinical examination of respiratory system, chest x ray and sputum examination for AFB (acid fast bacilli-bacteria causing tuberculosis). Chest x ray is needed to rule out pneumonia. Sputum AFB will diagnose tuberculosis if present. If both these are normal than no need to worry much for tuberculosis or pneumonia. You should take Influenza vaccine annually. It will prevent recurrent upper respiratory tract viral infection. Also take high protein diet, multivitamin and antioxidants tablet to improve immunity. Hope I have solved your query. Wish you good health. Thanks."
},
{
"id": 9812,
"tgt": "Suggest remedy for sinus issues with hair fall and dandruff",
"src": "Patient: am 38 years old i had sinus problem,so i cant drink liquid item,it mean drinking only boiled water.so i cant eat butter milk,fruits,wn i ate buttermilk or fruits immidiately headche and cough coming.i had so much dandruff my head all hair falling,so wt can i do.my son have esnophylia problem.he have cough.cold Doctor: Hi Dear,Understanding your concern. As per your query you have symptoms of hair fall and dandruff which is mainly due to fungal infection, improper cleanliness and stress leading to telogen effluvium. Sinus issues could be due to upper respiratory tract infection.Need not to worry. I would suggest you to consult dermatologist for proper examination. Doctor may order skin patch test, blood test and physical examination and may prescribe finpecia at least for 3-4 months along with antifungal medications. Take Indian gooseberry powder (half spoon) with water once a day and use almond oil for hair massage. Avoid use of hard cosmetic products, gels, sprays and shampoo's. Do nasal saline rinses for sinus issues and avoid intake of outside food. Hope your concern has been resolved.Best Wishes,Dr. Harry Maheshwari"
},
{
"id": 84142,
"tgt": "Can avil tablet use during pregnancy?",
"src": "Patient: pregnancy test positive after FET but am allergic to inj gestone . taking Hald injection, another brand of progesterone but am still getting thev allergic rashes. i have been advised to take tab avil , (chlorpheniramine ). is it safe in early pregnancy/ Doctor: Hi,Oral antihistamines, like cetirizine (Zyrtec), chlorpheniramine (Chlor-Trimeton), diphenhydramine (Benadryl), fexofenadine (Allegra), and loratadine (Claritin) seem to be safe. Avil is an antihistamine commonly used for the treatment of symptoms associated with allergies and cold. It is considered safe to take during pregnancy, but because it can cause drowsiness, so I would suggest less sedative loratadine or cetirizine for the treatment of allergic rashes.Hope I have answered your question. Let me know if I can assist you further. Regards, Dr. Mohammed Taher Ali, General & Family Physician"
},
{
"id": 8208,
"tgt": "White pimples on my nipples",
"src": "Patient: hi im 22 i have white pimples on both my nipples my nipples are very sore they become erect when i touch them what are they? Doctor: Oh it could be due to blockage in ducts I guess. Once that is cleared the swelling just disappears."
},
{
"id": 57316,
"tgt": "Increasing swelling in the legs causing a feeling of skin tightness, spreading to abdomen, arms and face, excessive fatigue. EKG, ER CAT scan normal, low liver levels. What should be done?",
"src": "Patient: female, 44, non drinker, smoker (1/2 pack a day) 5 nights ago had a little swelling in the legs/ feet , woke up the next morning gained about 12 lbs and swelling increased by 2-3 times skin tight in the legs felt like going to explode went to ER cat scan and ekg there said all was normal but liver levels a bit low. sense then has moved to the abdomen arms and face, increased fatigue now feels like water around the lungs. Doctor: This generalised swelling all over the body is usually due to low protein levels.As initially it was in lower limbs, it may be kidney related.Get Renal function tests & Serum protein levels done.Consult ur dr with reports."
},
{
"id": 108304,
"tgt": "What causes severe back pain and headache?",
"src": "Patient: I don t know how I can explain but I will try to. My nose is burning and ive been having chills and headaches and my back also hurts. I almost feel as if I put bengay all over my body. It is a very weird feeling and when I close my eyes to sleep I cant really sleep because I been having this feeling and im a little scared. My chest starts to hurt as well. I don t know what I should do. I tried taking Tylenol cold and sinus and that hasn t helped. Doctor: Hi,From history it seems that you might be having viral infection producing generalized body-ache, fatigue, headache, chills etc.This will remain for 2-3 days.Take plenty of water.Take Tylenol or ibuprofen and some decongestant medicine to get symptomatic relief.Take complete rest.Ok and take care."
},
{
"id": 12930,
"tgt": "What are the causes & complications of pityriasis rosae on genitals?",
"src": "Patient: Hi, I was just diagnosed with pityriasis rosae by my Dr. I have a herald patch on my back and it seems like every day I notice about 10-20 more red dots all over the place from my back to chest, to arms, stomach, and now I see them on my genitals. Can you tell me anything more about pityriasis rosae on genitals? Is this normal for it to spread there? Doctor: Hi, Pityriasis rosea is a self limiting viral infection of the skin which takes 6-8 weeks to resolve. It is normal to have genital lesions in pityriasis rosea. Hope I have answered your query. Let me know if I can assist you further. Regards, Dr. Asmeet Kaur Sawhney, Dermatologist"
},
{
"id": 45172,
"tgt": "19 year old girl got lutofolone for oligomenorrhea . But there is no bleeding",
"src": "Patient: 19 years old female with oligomenorrhea . Last time she got her period was 6 months ago. She is overweight (95kg) with excessive hair in her face and pubic area. She got one shot of lutofolone and no bleeding occurs after one week. What should she do? Doctor: Hi Ahmad, Welcome to HCM. Most likely she is having polycystic ovaries. If she is married, pregnancy test has to be done to exclude pregnancy."
},
{
"id": 169050,
"tgt": "What causes stomach pain?",
"src": "Patient: my 7 yr old daughter has had a stomach ache for 4 days. Thurs night she couldn't sleep at all, and last night was also a night of no sleep. Now she says it feels like she has a heart beat in her stomach, she i take her to the Dr. Toady i did give her a laxitive thinking she was constipated. DeAnna Doctor: was she constipated first of all before you administer laxative to her? stomach ache is a common symptoms for kids and does she eat? does she vomit? does she poop? how severe is the pain? if you say yes to all, then do not wait and visit the doctor."
},
{
"id": 67989,
"tgt": "What causes lump toward the back of neck?",
"src": "Patient: I have a lump on the right side of my neck, towards the back. I ve had it for several years, but recently its gotten a little bigger, about 1inch, and is now visible above the skin. Its painless, doesn t move and is hard. I m not sick right now and I feel fine. Should I get his checked out by a doctor? Doctor: Hi,As you have had it for several years then it is likely to be a sebaceous cyst. However, to be certain, I suggest you need to have your doctor examine it to confirm the diagnosis. Regards,Dr K A Pottinger"
},
{
"id": 9169,
"tgt": "What is the treatment for dry skin patches?",
"src": "Patient: I was experiencing numbness in my lips then a day later what seemed to be a cold sore appeared but, it is more just a small patch of dry, dark skin in the corner of my mouth. I just changed my facial wash so I thought I was allergic. I haven't used the wash in days but the dry patches are increasing and the numbness seems to be spreading to my nose and chin. What should I do? Doctor: Hi,It may be angular cheilitis. Riboflavin deficiency may be the cause. Kindly consult the dermatologist for the perfect diagnosis and proper treatment.I would suggest...- riboflavin containing tablet like FDson plus thrice a day-vitamin E & aloevera containing cream to apply on the lesionsI hope this help you.Thanks.Dr.Ilyas Patel MD"
},
{
"id": 87541,
"tgt": "What causes lower right abdominal pain?",
"src": "Patient: My lower right abdomen is in slight pain and i felt like a pop an er doctor pressed on it and it hurt a lil and he said it wasnt my appendix, but i still have pain there. Another issue i have had it was like my breathing went shallow i felt like i was going to pass out. Then my heart would race the right side of my face felt like a tingling sensation, my hands felt clamy and i started to slightly shake, then i felt exhausted. I tasted like i had metal in my mouth before i felt like i was going to pass out. Doctor: Hi! Good evening. I am Dr Shareef answering your query. If I were your doctor, I would advise you for a stool test for ova and cyst, and a urine routine, microscopic (and a culture sensitivity test if need be) test to rule out a UTI and treat it if positive. In case of a female patient, I would inquire about history of any missed period, and go for an ultra-sound to rule out an ectopic pregnancy (specially in those who are sexually active) which could give rise to your other symptoms along with the pain on the right lower abdomen. If all these are negative, I would give a serious thinking to the appendix as the culprit, and re assess you clinically followed by a diagnostic laparoscopy if thought necessary. Till then I would prescribe you with an anti spasmodic and a proton pump inhibitor drug for a symptomatic relief.I hope this information would help you in discussing with your family physician/treating doctor in further management of your problem. Please do not hesitate to ask in case of any further doubts.Thanks for choosing health care magic to clear doubts on your health problems. I wish you an early recovery. Dr Shareef."
},
{
"id": 213713,
"tgt": "How can i get a relief from my marital life stress ?",
"src": "Patient: My wife has told me that she has some funny relation with his jija before our marr. on my forcement like non-veg sms,non-veg talk etc. but on my more forcement then tell once that she also has some physical relation with him.but after that time ,she refuses for physica relation.Do you think it is possible that some one has only non-veg talk relation no physical relation. I am very tensed Doctor: your wife could have had premarital relationship at any extent.But if you extend you understandings with love to her she will turn back to you.Its better to take her for a councelling."
},
{
"id": 99369,
"tgt": "Suggest treatment for asthma",
"src": "Patient: I have been taking Seritide Acuhaler and Flixonase for over 10 years for what I am told is Asthma. I am choked with catarrh all the time and these inhalers don t seem to help at all. Am I better taking a daily antihistamine , as the congestion is affecting my sleep in a big way ? Doctor: HI, thanks for using healthcare magicThe congestion may be related to allergic sinusitis or allergic rhinitis which may be associated with asthma.The use of topical steroid nasal sprays along with your other medications would help to reduce your symptoms.Eg of sprays are flonase, rhinocort, nasonex, nasocortIf you can identify the source of the reaction and reduce your exposure then it may help as well.I hope this helps"
},
{
"id": 126680,
"tgt": "Is surgery recommended for this MRI test result causing severe neck and body pain?",
"src": "Patient: I deal with chronic neck and body pain. Due to car accident it is effecting my ability to use my right hand It hurts to even send an email. The MRI shows the detrition of C-4-C7 I have seen a PCP but should I follow up with a Neurosurgeon or Orthopedic Surgeon to discuss surgery? Doctor: Hello, As the pain is severe, it's better to consult them immediately before it worsens. Hope I have answered your query. Let me know if I can assist you further. Regards, Dr. Phanindra Dulipala, Diabetologist"
},
{
"id": 191839,
"tgt": "Suggest treatment for vaginal itching in a diabetic",
"src": "Patient: Hi I am 47 years old and was recently diagnosis with type 2 diabetes and recently I noticed I have bad vaginal itching only in the evening. I went to my doctor several times and they said it might be the soaps but I have used the same soaps before it. Can you please tell me what can cause vaginal itching? Doctor: Hello and welcome to HCMI have read your queryI hope this will helpDiabetic patients have decreased immunity to counter infections..Often in female diabetic patients vaginal itch secondary to infection is the only presenting symptom at time of diagnosis... You might be having VAGINAL CANDIDIASIS which is very common in female diabetic patients...I will advise you to visit a gynaecologist and have your vaginal swabs taken for examination.. And receive medications according to the results.. Meanwhile good diabetic control and local hygiene are necessary.. The response to treatment is excellent most of the times..ThanksDr faeza"
},
{
"id": 220100,
"tgt": "What are the chances of miscarriage having the fetus heartbeat normal?",
"src": "Patient: hello, I think I am 7w1day pregnant but just had a scan which gave a CRL measurement of 1.4mm and a yolk sac measurement of 3.7mm (mean). there was a heartbeat which was described as 'nice and fast'. I had a miscarriage last October and it looks like it is happening again... Doctor: Hallow Dear,The reports of the scan at this stage do not indicate any risk or impending danger to your baby or pregnancy. The growth of the baby and the heart sounds are well within normal limits. Had you mentioned the events of your previous miscarriage, I would have been better informed about the probable cause of miscarriage. You can continue this discussion further by asking me a Direct question. I do not know why you feel that miscarriage is happening again. Are you having any other symptoms? Please get back to me with more detailed information so that I can guide you further. Till that I hope this information is sufficient to encourage you. Dr. Nishikant Shrotri"
},
{
"id": 93790,
"tgt": "Frequent, severe stomach pain. Diagnosed high AST. Reasons?",
"src": "Patient: I was in the ER the other night with severe stomach pain. After doing a blood test, they discovered that I have a high AST (90) and a normal ALT. I'm 30 years old and exercise regularly (3-4 times a week) and drink 1-2 times a month. I don't have any other significant health history other than frequent stomachaches. Can you tell me any possible reasons for this? Doctor: Hi, thanks for using healthcare magic AST is normally used to assess any structural or functional changes to the liver though it can be found in other parts of the body. An increase in AST may occur in as a result of : (1) infection to the liver - the most common infection is hepatitis but this would normally be associated with other symptoms such as fatigue, fever, headache, loss of appetite (2) trauma (3) alcoholic liver disease (4) non alcoholic fatty liver- this is increasing in prevalence. It is the build up of fat in the liver as a result of diet, it results is inflammation of the liver (5) gallstones (6)presence of any mass in the liver The increase is AST may be related to one of these conditions. Your doctor may suggest repeating the value in a few weeks to see if it changes. An ultrasound or some other form of imaging may also be necessary. I hope this helps"
},
{
"id": 85662,
"tgt": "Can lightheadedness and breathing difficulty be the side effects of Prilosec?",
"src": "Patient: Hello, I have been on prescription Prilosec for a few days to combat reflux. Today I noticed some light headedness and difficulty breathing (getting a deep breath). Could these be side effects of the drug or what s happening with my stomach acid? Thank you. Doctor: Hello, Even without lung problems, GERD (reflux) can cause shortness of breath and difficulty breathing. Treatment can be a double-edged sword. GERD medcines like PPIs can actually increase pneumonia risk, according to several studies and very rarely light-headedness. You should take an appointment with your doctor to evaluate your health condition. Hope I have answered your query. Let me know if I can assist you further. Regards, Dr. Ajeet Singh, General & Family Physician"
},
{
"id": 50301,
"tgt": "Have back pain when dehydrated. Had Urinary Tract Infection spread to kidneys. Related?",
"src": "Patient: I am having pain in my back on my right hand side. A few weeks ago I had very painful gas like cramps , mostly on the right hand side but throughout the front. About 1.5 years ago I had a urinary tract infection that was thought to have spread to my kidneys because of the fever and pain that I experienced. I have had one urinary tract infection since then and every now and then I feel this kind of pain when I am dehydrated. Could this be a problem with my kidneys? Doctor: Tanks for using HCM!You have to do some investigations to come to some conclusion.You haven't mentioned your age, also, whether your UTI in past was diagnosed, with which investigations.Now, you should do Urine culture & sensitivity, Renal Function Tests,Sr. Uric Acid, CBC, Ultrasound KUB & X-ray KUB.Further management depends on these investigations,Consult your Urologist with the reports.Till then you may take, with your doctor's opinion, antispasmodics, analgesics, tamsulosin & diuretics (if required), antibiotics (as per reports).Drink plenty of water throughout day, approx. 2-3 litres/day.Hope this explanation is satisfactory for you.Wish you a healthy life ahead.Regards."
},
{
"id": 123576,
"tgt": "What causes sharp pain behind right knee?",
"src": "Patient: i keep having a sharp pain behind the right knee...feels like a vein pulling, very wierd feeling. Then I stood up and walked to my kitchen and had absolutley no sense of balance. I was dizzy and falling to the right without control. What could this be? Doctor: Hello, Sharp shooting pain can be due to ligament instability. Take hot water fermentation and do knee exercises. You should be fine. In case the pain is not coming down for a week or so then an MRI is advised to rule out the possible soft tissue injuries. Feeling dizzy and having no control could be due to low blood sugar and has nothing to do with the knee injury. Hope I have answered your query. Let me know if I can assist you further. Take care Regards, Jay Indravadan Patel, Physical Therapist or Physiotherapist"
},
{
"id": 47873,
"tgt": "Are fatigue and bone pains due to high creatinine and borderline calcium?",
"src": "Patient: I just received a phone call from my drs nurse saying my labs taken yesterday show that my creatatine (sp) is high and borderline calcium. She scheduled me to see him at his first available appointment, which is next Tuesday. So, I m left wondering if this could cause my extreme fatigue, bone pain, problems mixing up my nouns, increased forgetfulness, and heat intolerance? Doctor: Hello, Welcome to Health Care Magic. I read carefully your query and I understand your concern.I see your description; you say that you have high creatinine blood test and a borderline level of blood calcium. The creatinine high level shows us that you have problem with your kidney function and this is related with the borderline blood calcium and your symptoms of fatigue and bone pains.You should go to nephrologist to determine your stage of your failure kidney function or your kidney disease that cause these problems and to follow up with medications and other blood tests.Hope this answers your question. If you have additional questions or follow up questions then please do not hesitate in writing to us. I will be happy to answer your questions. Kind regard,Dr .Marsida JANKO"
},
{
"id": 203350,
"tgt": "Will hemeroids have an affect on ejaculation?",
"src": "Patient: Can hemeroids have an affect on ejaculation in men? \"OR\" can it be that i am 55 years old and semen is becoming thicker because of age? (First pulse of ejaculation is fine but after that, semen is hard to PUSH out of penis. Semen seems thick and looks more like mucus rather than whitish color semen use to be) Doctor: Hello,Thanks for posting on HCM,Hemorrhoids will not influence ejaculation but a urogenital condition can influence semens. A urogenital condition such as a prostate diseases can influence semens. Normally your seems quality should decrease with age given that your prostate becomes less productive. If you seem to have developed a thickened mucus-like discharge after sex, then it will be advisable to get that checked for an STD. I would not want us to jump into conclusion without running a semens analysis. Ones we are sure there is no infection, we might start looking at other possible causes of difficult discharge and thickened semens.I suggest you see a urologist who can further assist you.Hope this helps"
},
{
"id": 214806,
"tgt": "Could you suggest home remedies for the Phimosis and a cut on the fore skin?",
"src": "Patient: I am 52yrs,. what will be ayurvedic or home remedies for phimosis. at present I am using Alovera leaf gel resulting swelling got down. but i am not able to masterbate and feel pain and I have also seen a very very minutely cut on foreskin of penis. please suggest. I am not able to pay anything. Doctor: Hi,If phimosis is not since beginning and appeared recently then this might be due to collection of smegma and strictly adhere to glans called Belano-prostitis, can be corrected.Clean the smegma by gentle cleaning while taking bath.Apply some moisturizing lotion or antibiotic cream locally.Ok and take care."
},
{
"id": 45555,
"tgt": "Is hydronephrosis in the ultrasound of the kidneys a cause for concern?",
"src": "Patient: Usg of kidneys Rt. 8.5/4 and lt 8.7/4.4 Report states Bilateral grade one cortical echogenicity increased with altered cm differentiation s/0 bilateral grade one medical renal disease No e/o calculas or hydronephrosis .No e/0 focal mass What does all these mean Doctor: Hello, Mild hydronephrosis does not cause any problems and only close monitoring is required. You can go for a renal function test(RFT) to check the functional status of kidneys. Hope I have answered your query. Let me know if I can assist you further. Take care Regards, Dr. Shinas Hussain, General & Family physician"
},
{
"id": 51544,
"tgt": "Do dimensions play an important role in kidney scarring ?",
"src": "Patient: i had urine problem..i went for ultrasound 1 year ago and report came as ,left kidney-is normal in size 10.1cm and outline.cortical thickness and echogenicity are normal.cmd is maintained.mild pelvicalyceal system prominence is seen..no calculus is seen..same for right kidney only size differs i.e is 10.5cm. now i had ultrasound yesterday and it says everything is normal execpt,,there is mild prominence in renal pelvis..ap dimensions of renal pelvis is 25mm.i am worried about the dimensions of renal pelvis.what should i do.i had this problem from 1 year can it cause scarring in kidneys.i had no protein or wbc in urine.my doctor did t say anything.plzz reply i m very much worried Doctor: Hi Nawab, Your kidneys seem to be normal even though there is mild prominence in renal pelvis. The AP dimensions are also normal. You should stop worrying about this. Please drink plenty of water. Wish you good health. Regards,"
},
{
"id": 17955,
"tgt": "Suggest remedy for weakness, chills and racing heart?",
"src": "Patient: Im in bed now body feels weak chills a lil no fever light headed and bad headaches grouchy want to sleep life away been like this over a few years no good diagnosis yet I have small children I WANT TO HAVE MY LIFE BACK TO MUCH COMODTION MY HEART RACES i sit still my body feels like im spinning sometimes? Doctor: Hello, After going through your medical details I understand your concern and I would like to tell you that racing heart rate and weakness can be because of fever. Kindly consult a physician personally for physical and diagnostic evaluation of fever and further treatment. Hope I have answered your query. Let me know if I can assist you further. Regards, Dr. Bhanu Partap, Cardiologist"
},
{
"id": 128734,
"tgt": "What does this spinal MRI test result indicate?",
"src": "Patient: My MRI Conclusion: 1. Small (approximately 1.5cm) are of subcortical gliosis involving the left superior parietal lobule most likely from remote infarction. 2. Several scattered punctate foci of increased T2 and FLAIR weighted signal typical of early chronic small vessel ischemic disease. What does this mean? Thank you in advance for your help. Doctor: Dear patient your mri is of Brain region and not spine. human brain is divided into four lobes and parietal lobe is one of them. your report suggests old infarct in parietal area and subsequent fibrosis in same area. there are also multiple small infarct of recent origin which might be because of pathology in small brain vessels. what is your age? always mention age and complaints. I would advise visit to neurologist for further management. thanks."
},
{
"id": 124680,
"tgt": "What is the cause for extreme pain in the hip?",
"src": "Patient: hello my grandpa has extreme pain in his hip,he used to be very active, never babied himself or anything,he never let pain get him down. now he can hardly walk, he has totally quit working. And he is in non-stop pain. Please help,we are desperate...and his Dr. just laughs and says hes watching it...what is the matter? Doctor: Hello, It could be muscular pain or conditions like sciatica. As a first line management you can take analgesics like paracetamol or aceclofenac for pain relief. If symptoms persist better to consult an orthopedician and plan for an MRI scan. Hope I have answered your query. Let me know if I can assist you further. Regards, Dr. Shinas Hussain, General & Family Physician"
},
{
"id": 101730,
"tgt": "What causes yellow sputum in cough?",
"src": "Patient: Dear Sir,I am 58 year old male and having asthma for the last thirty years. Initially I used to have Asthma wheezing problems till the time I was on oral medication. But things are quite comfortable for the last 24 years or so when I started using inhalers. Now I am having 3 puffs each of Foracort 400 morning and evening and also 2 Puffs of Asthalin morning and evening. Breathing and life is absolutely normal,but one problem I am facing is that I get yellow sputum which keeps coming out when I cough just little. Kindly advice some remedy and oblige.G.S.Sethi YYYY@YYYY Doctor: Hi, Yellow sputum may be due to thickened secretion or bacterial colonies. For thickened secretion avoiding antihistamine drugs and daily breathing exercises with steam inhalation will help. If bacterial colonies you can take a course of antibiotics like azithromycin. This will help.Daily breathing exercise like pranayama will give more benefits.Hope this helps you. Regards"
},
{
"id": 152011,
"tgt": "What does it mean when you get a bad headache with racing heart?",
"src": "Patient: whats it mean when you get a bad head ache when your heart races whats it mean when you get a bd head ache when your heart races Doctor: Hi Fred,thanks for query.Please visit your doctor and get your blood pressure and physical check up done.There are chances of rise in blood pressure and the problem could be because of this.Please take rest,avoid any form of physical strain and see your doctor. wishing you good health."
},
{
"id": 21097,
"tgt": "Could the unstable angina be treated?",
"src": "Patient: I have recently been discharged from hospital with probable crescendo angina due to some discomfort I experienced last week. All my tests in hospital were normal. I had a stent op 18 months ago and have been fine since then. I am awaiting further appointment for angiogram. If diagnosed with unstable angina is it a lifetime diagnosis or can it become stable again? Doctor: Dear, since you have a crescendo angina that is unstable angina, you have to have a coronary angiogram as early as possible. Generally a person with unstable angina is not discharged without an angiogram. But since you have been discharged and now you are waiting for the angiogram, I would strongly advice you against any activity that would need you to exert. And also take your medicines very religiously.Most of the times unstable angina would not become stable on its own but definitely it can be reduced by curtailing the level of activity and by taking some particular medicines.All the best."
},
{
"id": 63675,
"tgt": "Suggest remedy for severe pain due to lump",
"src": "Patient: I ve had severe left side pain about a year. I have gallstones and nausea but pain is on left side. There is a small marble sized lump on left and pain is in that area radiating to left back. Lump is between ribs. I went to er today had ct scan they found nothing. Pain is becoming unbearable Doctor: Hi,Dear,Thanks for the query to HCM. I studied your problem in depth and I understood your concerns. Treatment for severe painful lump on left side-Old lump on left side could be of -Cold abscess with TB Spine.MRI done would fix the cause of this painful lump.Your case needs ER Ortho-Surgeon's consultation , who would treat it depending on the cause of it.Second opinion from ER Surgeon and from ER Chest Surgeon would fix the cause of this 1 yr lump and pain radiating to left back.This advise is based on the facts from the history you give and needs further clinical check.So don't build up wrong concepts and create more psychic complications in you which would increase risks and costs to you.Hope this would relieve your problem.Welcome for any more query in this regard to HCM.Write good resume and Click thanks if you feel satisfied with my advise.Have a Good Day.Dr.Savaskar M.N."
},
{
"id": 89391,
"tgt": "What is the cause of stomach cramps after having intecourse?",
"src": "Patient: hey my girlfriend and i had protected sex wednesday or thursday last week 7/13-7/14 and she said she feels stomach cramps in the middle of her stomach and they started monday 7/18 and she still has then now 7/22. I didn't come at all we just stopped. I was wondering if it was the kind of position i did that caused her pain in her stomach, i was wondering what you guys think i should do? Doctor: Hi.This is called dyspareunia, painful sex and has many causes. A particular position can be really causing this, should be avoided altogether. If there is pain in all positions, your partner needs investigations :Per vaginal examination by the Gynecologist to ascertain the cause and assisted by ultrasonography and / or CT scan. It may show the following.Salpingitis, Ovarian problem. PID- Pelvis inflammatory disease. Many times it can be due to unwillingness of your partner, or fear of somthing."
},
{
"id": 87936,
"tgt": "Suggest treatment for severe upper abdominal pain",
"src": "Patient: I had an episode of severe upper abdominal pain. Ultrasound revealed dilated bile duct, mildly enlarged liver, no pancreatic lesions. Liver enzymes, amylase, cbc, normal. My cnp wants to consult with a surgeon. Is this appropriate or are there other non-invasive tests which might aid in the diagnosis? Doctor: In regard to this present status, the best and most dependable Non-invasive test would be a MRCP scan."
},
{
"id": 191469,
"tgt": "What is the prognosis for a woman with type 2 diabetes undergoing menopause?",
"src": "Patient: I am 52 years of age and live in Alaska. I had blood work done, but not sure if I type 2 Diabetes is my problem, because of the results I received? I am full menopausal and trying figure out what I really need? I just found out my doctor is no longer in practice and trying to find a new one. Doctor: Hi and welcome to HCM! You have not mentioned what the problem is with your blood report. In general, postmenopausal woman are prone for osteoporosis and heart disease. Uncontrolled Type 2 diabetes in the long run can lead to complications like weak bones, ischemic heart disease and others. You do not have to worry as postmenopausal might increase the risk. Keeping optimal blood glucose under control, vitamin B12 and vitamin D supplements, taking medicatons regularly along with lifestyle modification will have good prognosis."
},
{
"id": 100829,
"tgt": "What could cause sneezing,eye itching,nasal discharge?",
"src": "Patient: I am having continuous sneezing problem for last two days, other problems are eye itching, watery nasal discharge. I have taken Avildom but no joy. Pl suggests me the remedy. I am 32 years old lady ans stay in delhi. My emailid YYYY@YYYY Doctor: HI, thanks for using healthcare magicSneezing, itching of the eyes, watery discharge are all in keeping with an allergic reaction.Oral anti histamines would help to relieve your symptoms.These are available over the counter.In addition the use of topical steroid nasal sprays or topical allergy drops would help. You may need a prescription for these.I hope this helps"
},
{
"id": 200360,
"tgt": "What treatment to be taken for sperm count improvement?",
"src": "Patient: Hi Doctor, My husband has a very low sperm count . it is 1% and motile sperm imotile sperm 99%. we both are 33 years old. can my husband have a normal count suitable for pregnancy by any kind of medication . If so How long does it take for him to recover. is there any possibility of me... Doctor: hello,,there is no such treatment to increase the number of motile sperm.but you can get pregnant by artificial methods like IVF by your husbands sperm.i shall advise you to consult an infertility specialist for that.hope this is helpful.regards."
},
{
"id": 178885,
"tgt": "Is squishy bump on head with indentation in the bones of skull around it serious?",
"src": "Patient: My 5 year old son collided with his sister(heads) full speed over a week ago and knocked him back on the cement and he blacked out for a few seconds. He recovered well but now has a half baseball sized squishy bump on his head near where he hit the cement. He has not complained and it is not terribly sensitive to the touch but I am concerned because I feel an indentation in the bones of his skull around the squishiness. He has acted completely normal and energetic. Doctor: Thank you for posting your question.The bump on the head that you feel is most likely due to local swelling following the hit. The indentation of bones however is not likely as the bones of a five year old are not normally supposed to be soft enough to give way to local swelling.Also, if your child is playful, eating well and not complaining of pain, serious consequences can be ruled out to a great extent.Having said that, if you still feel that his skull is showing some bony depression, I would suggest a medical consultation and if required atleast an X-ray skull may be done to rule out skull fracture.If you have any more doubts , do write back."
},
{
"id": 183785,
"tgt": "Will restoring the vertical dimension of the teeth cause pain?",
"src": "Patient: Hello. The last time that I was at the dentist office, a dentist polished down the enamel in the center of my premolar tooth a bit too low. I feel regret now because I asked her to do it. It was fine when I left the office. Now, a month has passed and I m having trouble speaking properly and smiling around that tooth. It feels too low in the center and I feel stress and pressure when the teeth come together while saying words and smiling. Some words, I can not say properly. It is not comfortable. Sometimes, a very small twinge occurs when the teeth come together. I was reading online and noticed something about not having enough vertical dimension, and it affecting phonetics. This is what I think is going on here. Is there any way to restore the vertical dimension in the center of the tooth where she removed the enamel and make it feel natural, so that I feel more comfortable smiling and speaking again? The location is on a biting surface of a premolar. There is no cavity where she removed the enamel and oddly, as of now, I do not have trouble eating. Will restoring the vertical dimension cause pain here? How can this be done that will not cause new pain or problems eating? or should I leave the tooth alone so that matters don t become worse? I am very sad because I just want my normal functions back (smiling and speaking properly) and am afraid to file anything else down. Hopefully, I won\u2019t have to choose between living with this forever or ending up in pain. Unfortunately, my dentist is booked for a month, so I have to wait before seeing her. I pray that you and she will be able to help me. ): Age:29 Gender: Female No medical problems (don\u2019t smoke or drink) Only had fillings done in my teeth Doctor: Hi,Thanks for posting the query, I would suggest you to take an X-ray of the tooth that was trimmed and see if the pulp horn is close to the occlussion, as you cannot see the Dentist right now take tab aceclofenac BD to relief pain.....Hope you find this as helpful,Take care!"
},
{
"id": 189990,
"tgt": "Had molar extracted, did root canal, loss of sleep, on nurofen. What can help pain?",
"src": "Patient: hi i have a second molar extracted yesterday,the dentist try to save the tooth doing a root canal treatment but the tooth still heart after so i decide to extracted but last night i could not sleep i take nurofen 400 mg three times a day and is very painful and my check is swallow ,what can I do or what tablet to take,wait your answer thank you Doctor: Some amount of pain and discomfort is expected for few days after tooth extraction. Amount of pain which occurs after tooth extraction is dependent on procedure which is used for the extraction of the tooth and the degree of trauma to the adjacent tissues.Pain can be managed after tooth extraction withover the counter pain medicaments.Analgesics are given after the tooth extraction to relieve the pain.During the first 24 hours after tooth extraction, you should take soft and liquid diet and should avoid taking food from the side where tooth was extracted. It is advised to do warm saline rinses 24 hours after the tooth extraction to reduce the pain and swelling. Take rest.You should avoid spitting, sucking, drinking anything with straw for minimum of 24 hours after tooth extraction and should not smoke for minimum of 2 days after tooth extraction.Smoking can create negative pressure and can cause dislodgement of clot and lead to dry socket. It is also common to have pain in the jaw after the extraction as the mouth is held open for long time during the tooth extraction. Massaging the muscles gently, hot compresses application and over the counter analgesics can reduce the soreness of the jaw. Allow relaxation of the muscles and the jaw joint and don\u2019t take hard crusted foods."
},
{
"id": 52015,
"tgt": "What does this result of mine say Urea=33 mg, BUN=15.41mg,S. Creatine=1.0mg ?",
"src": "Patient: I am 26yrs Male. My B.Urea=33 mg, BUN=15.41mg,S. Creatine=1.0mg, Uric Acid=6.5mg. Kindly comment on these results. Doctor: Hi! Welcome to Healthcaremagic.com.Except Blood urea all other values are well within normal range.slight elevation of blood urea may occur in dehydration .Please repeat after adequate hydration.Meanwhile what are your complaints? Oliguria? etc,please let us know."
},
{
"id": 58347,
"tgt": "Have echogenic liver, gallbladder removed, abdominal pain,evidence of benign hemangiomas, BMI 31. Any treatment?",
"src": "Patient: I was told from a CT and ultrasound that I have a \"slightly echogenic liver\". And there is also evidence of \"most likely benign hemangiomas\". I have had my gallbladder removed due to gallstones and continue to have upper abdominal pain. My BMI is 31, I am not diabettic and do not drink. Should i be concerned? Do I need further treatments or test? Doctor: Thanks for query on HCM.Benign hemangiomas does not require any active treatment at present.Keep regular check up for liver.Upper abdominal pain is most like due to indigestion,it usually occurs after gallbladder removal.Your bmi 31 indicates obesity grade 1.You have to take care for your general health.Take simple,high fiber diet,drink more liquids like buttermilk,fruit juices,vegetable soups.Avoid oily,spicy,high calorie diet.Try to reduce your weight.Go for exercise,yoga,swimming,cycling.Keep in touch with your doctor."
},
{
"id": 211060,
"tgt": "Have weird behavior because of autism",
"src": "Patient: I have a great nephew that was diagnosed with Autism about 6 1/2 yrs ago. For the last 18 months he has been having episodes where he tries to hurt himself by banging his head on a hard service, biting himself, scratching, pinching himself, as well as anyone trying to stop him. Are the episodes typical with children with Autism? He is strong as an ox during an episode. It takes 3-4 adults to hold him down until he is calm again, to keep him from hurting himself. Doctor: HiThanks for using healthcare magicYes, all these self mutilating behaviors are part of autism. Autistic children sometime are impulsive and due to impulsivity, they do all such behavior. In case of your nephew, you should consult a psychiatrist. He needs low dose of antipsychotic that would help in controlling his aggression. In case you need our help, we are happy to help you.Thanks"
},
{
"id": 181317,
"tgt": "Is numbness in mouth and tongue after a root canal normal?",
"src": "Patient: I had a filling re-filled in my lower right molar yesterday morning and am still experiencing mouth and tongue numbness on that side of my mouth, 30+ hours later. The dentist told me I would experience prolonged numbness, but this seems extreme. I've put a call into the office, but haven't heard back after 3 hours. Is this normal, or could there be a possibility there is some nerve damage? Doctor: Hi..Welcome to HEALTHCARE MAGIC.I have gone through your query and can understand your concerns..As per your complain as you have numbness of mouth and tongue even after 30+. Hours it is definitely abnormal as the numbness due to dental anaesthesia generally resolves in about 2 to 3 hours at a max..It can be due to needle prick injury to the nerve that is leading to prolonged numbness..Many times mild injuries tend to resolve soon and the numbness may stat subsiding in next 2. To 3 days..But in case if it still does not improve consult a Neurophysician and get evaluated..As of now take Vitamin B complex supplements and do alternate warm and cool compresses..Hope this information helps..Thanks and regards.Dr.Honey Nandwani Arora.."
},
{
"id": 21799,
"tgt": "What causes numbness in left hand,feet, odd heart beats at night?",
"src": "Patient: Hi, I am 28 years lady. Height 5 ft 2 inch. Weight 60 kgs. Have 2.5 yrs old son. I face numbness in left hand and feet. The heart beats I feel are very odd. I mean I feel as if it has stopped beating. I feel like givin pressure on hands and heart in such situation. It always happens mid night. Please suggest Doctor: Hi,It can be cause from stress or overtiredness, I would advise you to take valeriane for coupe of days before going to bed.Take careCome back if you have any further questions"
},
{
"id": 91945,
"tgt": "What causes to pass ginger color stools with abdominal pain?",
"src": "Patient: My little boy is 5 years old. He sometimes gets abdominal pain, esp before needing a poo. He has recently (tonight) been passing wind almost every 5 minutes, but he is also passing what looks like little white bubbles too. His stools recently have been a ginger colour. What do you think it could be? Many thanks, Ruth Doctor: Hi Ruth. This is obviously suffering from an intestinal infection. What he needs is a course of an antibiotic and supportive medicines. As these are prescription medicines , I would advise to see a Pediatrician for further management."
},
{
"id": 225667,
"tgt": "On and off spotting followed by heavy period with large clots, lightheadedness when getting up. On mirena IUD. Anemia?",
"src": "Patient: I have had the mirena iud for 3 months with approx 6 days of no bleeding here and there. Mostly it's been a tolerable spotting, for the past week I have been on a heavy period which is becoming lighter. Although I've had clots throughout the periods today I have seen a somewhat large size the length of a small tampon. Should I be worried? I have also been becoming lightheaded when I get up from laying down or sitting down in my own mind I think this is maybe anaemia? Doctor: Hello,Welcome to Healthcaremagic.Yes you may have anemia.But this needs either iron tablets or injection depends on your hemoglobin level in blood.Regarding your spotting and heavier bleeding, It is very common with Mirena IUD for initial 3 to 6 months.Thus your body needs adjustment for the Mirena IUD.After these 3 to 6 months, your body may adjust and your periods may resume to normal.So you do not need to worry too much about this. If you are my patient, I will prescribe iron tablets for 2 months after checking the hemoglobin concentration in blood. I ask you wait for 3 more months and review.Hope this helps.If you have additional questions or follow up queries then please do not hesitate in writing to us. I will be happy to answer your queries. Wishing you good health.Regards,Dr.Manjeth"
},
{
"id": 3986,
"tgt": "Are there pregnancy chances with precum coming near vagina?",
"src": "Patient: Was with my boyfriend last night and he started humping bt didnt enter the vagina and there was no ejaculation... while doing this he was wearing his underwear... could i be pregnant may be because of pre ejaculation fluid?? should i take an ecp??? Im expecting my period a week later.... Doctor: dear madam since the intercoure not happened ther might not be any chance of pregnancy.no need to take any pills i suggest you to relax and enjoy life regards"
},
{
"id": 210723,
"tgt": "What combination of the below drugs can cause cognitive impairment?",
"src": "Patient: I take 10 mg of lexapro, 5 mg of ambien, 150mg of seroquel and 2 mg of klonopin: I am repeating myself a lot what combination of these drugs could cause cognitive impairment. I am going on 73 years of age. I also take zofran for nausea from General anxiety disorder; I also have depression but I am not b-polar Doctor: HIThanks for using healthcare magicContinuous use of ambien and klonopin could lead to cognitive impairment. Better to use them on sos basis when you feel any anxiety or sleep disorder. Lexapro and serquel you can use. It would control underline depressive and anxiety symptoms. Seroquel also helps to get sleep at night, so you can easily avoid ambien.Thanks"
},
{
"id": 24267,
"tgt": "What is the purpose of nurokind gold 1cup injection?",
"src": "Patient: hello doctor, my mother who is 50 yrs old, was sleeping in chair and felt stiffness in had for more than 2 hr. when we checked with doctor we came to know she got high bp.. she gave nurokind gold 1cup injection. what is the purpose of that injection? Doctor: Hello Neuro kind gold is a multi vitamin injection particularly given for Vitamin B12 , which in turn may help strengthing of nerves . It's generally not given for higher blood pressure and your doctor must have given it for other symptoms he might have thought were because of vitamin deficiency. Regards Dr Priyank Mody"
},
{
"id": 190598,
"tgt": "Had orthognathic surgery. Still feel that I do not have enough space for tongue. Is it due to surgery?",
"src": "Patient: i have a question. i am an adult. i have had orthognathic surgery ..and then a revision of it along with total joint replacement of both tmjs this past july. i am still in braces. when i was a child the orthodontist extracted 4 bicuspids. the orthodontist i have been seeing prior to the most recent surgery..has helped me tremendously by reopening the spaces on the maxilla . i will hav implants when stable. the problem is..that i still feel that i do not have enough space for my tongue . i was okay prior to the 2nd surgery bc the orthodontist pushed the teeth way out..to where they should have been all along. i was told i never needed the extractions..just the surgery. now that the occlusion is back to the proper place..i now feel very crammed with the tongue..especially on the lower palate . my dr is trying to help me keep lips together..et..and tongue to roof of mouth..i am doing this the best i am able..but at times..the tongue literally seems too long. it is uncomfortable..and i am concerned that when the braces come off..i will have problems. i had a long face syndrome..from allergies i guess as child. i have kept in shape..however..my neck posture ..and the entire posture all around is off...and always will be. i dont know if this could be making the tongue posture forward. i just am to the point i am about wanting to have a tongue reduction. is it possible that my palate, upper and lower..are so underdeveloped that the tongue is too large..despite the sugeries. i was concerned of this prior to the 2nd surg..because this was a major problem after the 1st surg...and there was relapse. my oral surgeon is excellent..but he and the orthodontist are so focused on what they specialize in..im not sure they understand..there may indeed be a problem. the only other thing..is that maybe it is swelling...even so...it still extends past the lower arch..when at rest.. they do not want me tongue thrusting..et..or mouth breathing ..so if i keep lips together..et..tongue is crammed. i have had so much surgery..and other things..i just dont know if something rt the surgery is causing this.. i am concerned if i dont do something..i will continue to have problems...my palate is very narrow..despite that they split it..on the last surgery.. Doctor: Hello, Firstly, quite a strong person you are. You have had so much in past & still you are standing strong. You might be suffering from macroglossia or pseudomacroglossia due to narrow palate. please provide some radiographic records so that we can understand the situation in a more better way. Consult some other maxillofacial surgeon to have a second opinion. You might need a tongue reduction surgery. I hope this might help you. Please do follow up with the proceedings of your case. Get well soon. With Regards, Dr. Himanshu Govil Gurgaon"
},
{
"id": 52871,
"tgt": "What causes fatty liver grade 2?",
"src": "Patient: I saw a doctor this morning for severe pain in my lower abdomen. Doctor prescribed Rabeprazole, domperidon, sparfloxacin and tiemonium methylesulfate for 7 to 10 days for my ache. With medication I am feeling better now. He also advised me to do tests: ultra-sonogram of whole abdomen, blood CP and urine RE. Report says that my lever has got enlargement (13.89 cm) and I have fatty liver grade 2. My urine RE report is ok. However, blood CP says WBC total count is little high (11.6). I am worried about the liver enlargement. Please advice me what should I do now? Doctor: Hi and welcome to Healthcaremagic. Thank you for your query. I am Dr. Rommstein, I understand your concerns and I will try to help you as much as I can.There are some medications which may cause fatty liver and domperiodn is one of it, however,in most cases this is caused by diet and alcohol.There are two main types of fatty liver disease so treatment depends on this: Alcoholic liver disease and Nonalcoholic fatty liver disease. You can get alcoholic liver disease from drinking lots of alcohol. It can even show up after a short period of heavy drinking, other causes are obesity and hepatitis B and C.If you have alcoholic liver disease and you are a heavy drinker, quitting is the most important thing you can do. Talk to your doctor about how to get help. If you don't stop you could get complications like alcoholic hepatitis or cirrhosis. Even if you have nonalcoholic fatty liver disease, it can help to avoid drinking. If you are overweight or obese, do what you can to gradually lose weight -- no more than 1 or 2 pounds a week.Eat a balanced and healthy diet and get regular exercise. Limit high-carb foods such as bread, grits, rice, potatoes, and corn. And cut down on drinks with lots of sugar like sports drinks and juice. This should lead to improvement. You should repeat your liver markers in 3-4 weeks.I hope I have answered you query. If you have any further questions you can contact us in every time.Kindly regards. Wish you a good health."
},
{
"id": 90370,
"tgt": "Why am I feeling nauseated and stomach pains?",
"src": "Patient: Hi, I have been experiencing dizzy spells lately and have never before. I sometimes feel nauseated and have been experiencing stomach pains. My stomach pains hurt more when I am walking. The pain feels like there is something pushing down on my stomach. Doctor: Hi, thank you for posting.I have gone through your query and I understand your concerns.According to your symptoms I can say that you suffer from gastritis. Gastritis is the inflammation of the mucous of stomach.To determine the causes you need:1. Fibrogastroendoscopy.2. Liver function test.3. Abdominal ultrasound.4. Complete blood count5. Urine test.To improve your health condition you should follow these guidelines:1. Eliminate spicy and acidic foods from your diet.2. Do not drink alcoholic drinks, carbonated drinks, coffee and milk.3. Quit smoking(if you do smoke).4. Take proton pump inhibitor medicines such as Omeprazole for 6-8 weeks.Contact your gastroenterologist.Take Care.Dr. Behar."
},
{
"id": 69745,
"tgt": "How to remove sebaceous cyst without surgery?",
"src": "Patient: hi, I have a sebaceous cyst on my forehead, and it doesn't pain. But I still want to get it removed. so could you please guide me if it can be done without surgery? I don't want any mark on my face, so worried to go for a surgery. Is there any other method of removing it? Doctor: Hi.Is not this you are asking too much???Dear friend - how is it possible?Do the Doctors be directed like this where you are at the problem end.Well there is no way to remove this without surgery.Yes, you can be helped_ the best surgeon can give you the best scar, may not be visible later on. If the swelling is looking better on the face than the mark- do not get operated."
},
{
"id": 81157,
"tgt": "What causes breathlessness with increased heart rates?",
"src": "Patient: I recently turned 20 and just last month I started to have trouble breathing along with my heart beating really fast I feel.as if its popping out of my chest! It does this several times a day. I can t breathe deeply anymore unless I concentrate. What do I do,?? Doctor: Thanks for your question on HCM.I can understand your problem and situation.In my opinion you are having anxiety mostly. But better to rule out cardiac and pulmonary cause first. So get done ECG to rule out cardiac cause and chest x ray and PFT (pulmonary function test) to rule out pulmonary cause (asthma). If all are normal than no need to worry much.Anemia and thyroid problems can also cause similar symptoms. So get done CBC (complete blood count) and thyroid function test (TFT) to rule out anemia and thyroid problems respectively.If all of the above are normal than you are having anxiety only.So consult psychiatrist and get done counseling sessions. Try to find out stressor in your life and start working on its solution.Avoid stress and tension. Be relax and calm. Don't worry much, you will be alright."
},
{
"id": 82269,
"tgt": "Is coughing up blood with phelgm and clots after intubation concerning?",
"src": "Patient: My husband was intubated for 2 1/2 days while sedated after a seizure waiting for MRI. This was last Thursday. He coughed up phlegm and blood for several hours. Then it was just phlegm. It dissipated over the weekend. When he came home today, he started to cough really hard and coughed up what looks like a 1 square of flesh. Then a few hours later, he coughed up a larger piece. Is this something we should worry about or take him to the emergency room? Doctor: Thanks for your question on HCM.In my opinion you should get done atleast chest x ray to rule out other causes of haemoptysis (coughing out blood). If chest x ray is normal than no need to worry. The blood might be due to friction of endotracheal tube during the time of hospital stay.And the description you are giving favours old clotted blood, rather than fresh blood. As fresh bleeding appears bright red in color.So get done chest x ray first and if this is normal than no need to worry much."
},
{
"id": 144400,
"tgt": "What causes tingling and numbness in limbs and face?",
"src": "Patient: I ve recently learned that I have scoliosis and have had a nettle-like burning/itching and sometimes numbness in my limbs, face and chest. I don t work out much, but if I do anything that gets my heart rate up it seems that I get brief electric shock pulses going through those same areas. I ve also had problems with migraines and severe nausea over the last 2 years. My neurologist says it could be an early symptom of diabetes and I am quite overweight (6 2 , 280 lbs). I m just not sure I agree. I ve had an MRI and an EEG. MRI was completely fine. EEG he first said was just slightly abnormal, then he said it was normal. And I did break my tailbone a few years back. I m wondering if this could all be tied to the scoliosis in my upper thoracic vertebrae or if it could be MS or something else. Doctor: DearWelcome to HCM.Understanding your concern. As per your query you have tingling and numbness in limbs and face because of nerve impingement , pressure on facial nerve which can lead to facial paralysis . Need not to worry about it. Visit neurologist once and get it examined to rule out nerve compression , neuralgia or other conditions like neuropathy. Soemtimes the symptoms also occur due to deficiency of vitamins and mineral like vitamin B 12 . Go for physical therapy of facial muscles. Also do facial muscle exercises . If it is not due to nerve and due to dental infection then consult dentist once and get it examined. Doctor may prescribe OPG of .It could be related to stroke. Neurotropic medication such as nerve supplement , methylcobalamin is effective to improve your symptoms. In case of sinusitis consult ENT specialist and take steam several times a day . Eating healthy will also be very helpful .Hope your concern has been resolved.Get Well Soon.Best Wishes,Dr. Harry Maheshwari"
},
{
"id": 15997,
"tgt": "Swollen reddish fingers with itching on the knuckles. Severe pain in the arms with tingling. Need help",
"src": "Patient: Hi. My fingers are red and swollen but my knuckles are white and they itch like crazy. I have for the past 5 months been waking up in the middle of the night with sever pain in my arms from the shoulders to my fingertips. I am exhausted, my coordination is off. My arms and hands are tingly constantly. When the pain wakes me up the only way to make it go away is to walk around for an hour or so. If I am awake when it starts, I hold my arms out but bent down at the elbows the pain and numbness subsides. HELP Doctor: Hi welcome to Health care magic forum. What is missing is your age, It appears it is neuropathy secondary to hypertension, or diabetes, or old age. I advise you to consult a physician for diagnosis and treatment. You may need to have M.R.I.scaning besides other routine tests for diagnosis of any cerebral ischemia. As anaemia also play a role in this, i advise you to follow some diet recommendations as take more of green leafy vegetables, pulses, sprouts, milk, egg, meat, and sea foods to provide body with iron, proteins, vitamins and antioxidants to cure it by natural way. Avoid taking, potato, other tubers, tomato, cauli flower, and coconut, till you get complete relief of the symptoms. Wishing for a quick recovery and thanks for calling."
},
{
"id": 128439,
"tgt": "Can an ankle fracture be treated when the X-ray report is normal?",
"src": "Patient: WHILE STEPPING DOWN FROM STEPS, SLIPPED AND MY RIGHT FOOT TWISTED . AFTER THAT MY ANKLE BOUND BY DOCTOR. BUT IT WAS NOT PROPERLY BOUND AND NOT HEALED PROPERLY. BECAUSE OF NERVE TWIST FULL BODY OF NERVES ARE EFFECTED INCLUDING LEFT LEG AND RIGHT SIDE OF SPINAL CORD. AS PER X- RAY, BONE CONDITION OF ANKLE IS GOOD. NOW TWO MONTHS OVER, NOT HEALED PROPERLY. WHETHER INFECTION OF NERVES MAY EFFECT THE BONES? PLEASE GUIDE. Doctor: Dear patient If Xray is normal means there is no fracture. On the one hand you are saying fracture is healed and then you say xray report is normal. This is not possible. If your xray is normal you may have sprained your left ankle with ligament injuries. For which you are given slab for month. Sprain should be relieved by now and u should be painfree. If there is constant pain there should be other possibilities than sprain. I would like to advise mri of the left ankle to rule out other pathology. Please get it done from radiology centre nearby you. Meanwhile start tab diclofenac sodium 50 mg twice a day for pain relief. Please consult orthopaedic surgeon nearby your area with report."
},
{
"id": 131616,
"tgt": "What causes pain in the thighs, hand and finger joints?",
"src": "Patient: I had typhoid & paratyphoid in last but last week. I completed entire antibiotic course given by my MD physicia. I was absolutely right till yesterday. But since last evening, my thighies are considerably paining & left hand finger joints are paining Doctor: Hello and welcome to Healthcamagic. These symptoms indicate rheumatoid arthritis and you should see immunologist and do findings to rule out most common types of arthritis.I hope I have answered your query. I wish you a nice day."
},
{
"id": 156288,
"tgt": "What is the side effect on skin post chemo therapy?",
"src": "Patient: I am an esthetician and have a potential client who is a breast cancer survivor. She is currently taking a medication similar to tamoxifen and has little blister looking pustuales on her face. I told her I would be more comfortable with a dermatology visit before I facial her to come up with a skincare plan. Is that a common side effect for skin post chemo? Doctor: Side effects on skin post chemotherapy can be redness, rashes, itching, peeling, dryness, or acne. Nails may become darkened, yellow, brittle, or cracked, and may also develop vertical lines or ridges.Sometimes, chemotherapy causes the skin along the vein to darken, especially in people who have very dark skin. After treatment ends, the darkened areas often fade in a few months.RegardsDR De"
},
{
"id": 212632,
"tgt": "Child stubborn with certain things. Does this denote OCD?",
"src": "Patient: My daughter will be 4 in june. On sunday she started with a few odd habbits. Not sure what they mean or if there is a problem with OCD or something. First she is keeping her hands in the fists position, she wont touch certin things, is asking for blankets, bedding, etc to be washed even if they were just washed. When I question her she doesn t have an answer for me, which I expect because of only be 4. There is no consistant pattern to any of it. Doctor: Hi There, Thanks for asking. Childhood onset OCD usually begins around the age 10 to 12 or sometimes in the years after. For many of them a family stress exists which contributes to the maintenance of symptoms. On the other hand, many children have some behaviors to show their autonomy (sometime by such behaviors you described). These behaviors manifest around age 4 and then gradually subside. However, it could be a sign that the child has a transient anxiety about a family or social stressor. I wish you a good health."
},
{
"id": 112355,
"tgt": "Back pain, spasms, abdominal pain, bloated, spine pain. On tramadol. Is there a kidney stone?",
"src": "Patient: hi I have had a bad back for a few months and 6 days ago I layed on the sofa shuffling down and I couldnt move from the pain since this I have been in a lot of pain my tummy and sides are blooted I have been to the doctors I am on diaxepam codeine and today they switched me to tramadol, I have had no xray just felt it and told me my back was in spasm as u could see the muscle on the lect side of my spine, do u think this could be something more than a back spasm as it seems to me odd It has. ot felt right for a few months now something has gone to bring this ofull pain and now the swelling I looked on here any it told me 92% chance its kidney stone? what do u think? Doctor: Hello.Symptoms of renal colic by kidney stone include pain in the lower back spine radiating to the abdomen and genitals and problems in urination (increased frequency, difficulty starting urination ....).To rule out kidney stone is needed, plus a abdomen radiography, an urine analysis .Go to your doctor ,as soon as possible,for urine analysis and confirm or exclude the diagnosis.I wish you good health. (If the answer has helped you, please indicate this)"
},
{
"id": 77805,
"tgt": "Suggest medication for severe cough and phlegm",
"src": "Patient: i have a ton of phlegm in my chest and lost my voice. just a nasty cold. i am also trying to get pregnant-any suggestions of things i can take so i can get my voice back to normal? i feel fine, and when i cough, i am coughing up some nasty green stuff Doctor: Thanks for your question on Health Care Magic. I can understand your concern. Green expectoration is suggestive of lung infection and bronchitis. So better to treat these things first before pregnancy. It is advisable to be normal when you are conceiving. So first consult pulmonologist and get done clinical examination of respiratory system, chest x ray and PFT (Pulmonary Function Test). Chest x ray is needed to rule lung infection. PFT is must for the diagnosis of bronchitis. You may need Inhaled bronchodilators, inhaled corticosteroid (ICS) and antibiotic drugs. Don't worry, you will be alright. Hope I have solved your query. I will be happy to help you further. Wish you good health. Thanks."
},
{
"id": 126212,
"tgt": "What causes body cramps and walking difficulty?",
"src": "Patient: hi I am a 49 year old male don t smoke or do drugs I work it started about 3 years ago body cramps its just getting worse I cant walk legs will cramp up if I use my arms they will start cramp up and now I think my back muscles are trying to cramp up and when they do its very painful Doctor: Hello, The possibilities are neuropathic pain and peripheral vascular disease. Consult a physician and get evaluated. You have to get a vascular Doppler to look for vascular causes. Hope I have answered your query. Let me know if I can assist you further. Regards, Dr. Shinas Hussain, General & Family Physician"
},
{
"id": 67241,
"tgt": "Suggest treatment for a lump on the buttock",
"src": "Patient: Hi, I have a lump right underneath the surface of my butt cheek and I was wondering what it is. I haven t fallen or done anything to cause one to be there, it just sort of appeared there and the spot where it is has slowly gotten more tender and painful to sit on. I can feel the lump when I touch it, but you can t tell there s anything there from looking at it. It is pretty small, maybe a bit wider than my index finger Doctor: Hi ! Good evening. I am Dr Shareef answering your query.Although it requires a clinical examination to opine on the lump, from the history it seems to be an infected lump, ranging from a simple abscess to a pilonidal abscess in the area. I would advise you to get it assessed by a general surgeon in your area for an appropriate management. Till then, you could go for an anti inflammatory drug along with a proton pump inhibitor for a symptomatic relief.I hope this information would help you in discussing with your family physician/treating doctor in further management of your problem. Please do not hesitate to ask in case of any further doubts.Thanks for choosing health care magic to clear doubts on your health problems. I wish you an early recovery. Dr Shareef.I hope this information would help you in discussing with your family physician/treating doctor in further management of your problem. Please do not hesitate to ask in case of any further doubts.Thanks for choosing health care magic to clear doubts on your health problems. I wish you an early recovery. Dr Shareef."
},
{
"id": 2801,
"tgt": "What are the chances of getting pregnant from pre-cum?",
"src": "Patient: This may seem pretty silly, well it is very silly but I m a bit paranoid.. I fingered my girlfriend today, but then I had to adjust myself, I didn t touch my penis, I adjusted it over my pants, I was wearing two layers (briefs and shorts) I didn t feel any precum on my pants but in the moment I may not have, and then used my fingers to well go at it again... I doubt there was any precum on my fingers as I don t think it could get through two layers of cloth but in case it did what are the chances of her getting pregnant..? I don t generally have a lot of precum, the usual drop or two Please help, I m just a little freaked out Doctor: Hi,There is no chance of pregnancy in your case as you described your act. You was on both brief and pant. You did fingered your girlfriend without touching to penis. If you have touched penis then also extremely less sperm can attache to finger. By fingering, sperm can not enter in to vagina. Sperm require seminal fluid as a liquid medium for movement inside uterus. To become pregnant, penetrative sexual intercourse with or without ejaculation during fertile period of cycle is required. Use condom in future to prevent unwanted pregnancy. Avoid stress, take healthy diet, drink plenty of water and do regular exercise.Hope this may help you. Contact further if follow up needed.Best regards,Dr. Sagar"
},
{
"id": 135470,
"tgt": "Suggest proaction to prevent bunion on left foot",
"src": "Patient: Hi, my 14 yr old daughter dances ballet, jazz, modern etc 6 days a week and she has bunions that are beginning to bother her, especially her left foot. I want to know what we can be doing proactively (proper walking shoes, wrapping her feet for dance...anything that may help alleviate her discomfort. Also, I ve considered taking her to a sports podiatrist for a consultation and I am looking for a reputable one in our area. I appreciate any insight you can lend. Tracy Doctor: She is at risk for bunion if she wears shoes with narrow toe box. Apply ice, use protection bunion pads and conformable shoes not tight."
},
{
"id": 55531,
"tgt": "Could shaking, weakness and difficulty in focusing be the symptoms of gall stones?",
"src": "Patient: Hi I m Summer I m 26. I found out over a week ago I have gallstones so they are going to take it out on thursday(Thank God I am so over this pain) but yesterday i started getting shaky and felling very weak today I m having trouble focusing my eyes with out getting a head ache but if I dont try to focus the everything looks burry. Are these possible side effects Doctor: HelloNo,these are not the usual symptoms related to gall bladder calculus.Your symptoms may be due to anxiety and nervousness.You may need anti-anxiety medicines like alprazolam,clonazepam, etc.You should try to be calm and take rest.You are rightly advised for cholecystectomy(surgical removal of gall bladder).Get well soon.Take CareDr.Indu Bhushan"
},
{
"id": 15666,
"tgt": "On keflex after surgery. Had drug rah. Dry skin with itching. What can be done?",
"src": "Patient: I recently had surgery and was placed on a week of keflex, then doxycycline and clindamycin - while on these i got a drug rash and stopped taking them after 6 days then i was placed on zyvox for a week. Im finally off all these meds for about 1.5 weeks now and itching with extremely dry skin which gets quite red when i scratch. I am using bath oils and lotions but can not seem to get relief. Doctor: Hi, thanks for your query. For dry itchy skin you may do as follows:-use a oat meal based cleanser for bathing.-bath with luke warm water instead of hot water.-apply a good emollient cream containing shea butter or olive oil after bath and repeat it twice or thrice as required.-avoid scratching as it initiates scratch-itch cycle which increases the intensity of itching. Take oral anti hisatmine like fexofenadine or levocetrizine by prescription.-Drink about 2-3 litres of water per day. -You may use a calamine lotion or over the more itchy areas as it gives a soothing effect. Hope this helps you."
},
{
"id": 106590,
"tgt": "How can backache be treated?",
"src": "Patient: I pulled my back out this morning. I made it home barely and now in bed I can\u2019t move without agony. I also have a cough from being sick last week and it\u2019s making my back pain horrendous. I don\u2019t have anyone at home until 5pm. Struggling to get out of bed to use the bathroom. I have had 2 nurofen plus but not helping at all. Doctor: Hello and Welcome to \u2018Ask A Doctor\u2019 service. I have reviewed your query and here is my advice. In my opinion your pain in the back is due to a condition called lumbo sacral strain or grade 1 slipped disc. You should take pain killers and muscle relaxants . Local heat application would also be helpful. If you do not feel better in few days then I wold recommend MRI examination of lower back. Hope I have answered your query. Let me know if I can assist you further."
},
{
"id": 103575,
"tgt": "Stung by yellow jacket. Area washed and ice applied. Started vomiting and stomach pain. Is this an allergic reaction?",
"src": "Patient: My 5 year old son was stung by yellow jacket last night 4-5 times (one looked more like a small irritated pore as opposed to a sting but he claims it was a sting). Stingers were scraped out, area washed, ice applied. No severe swelling . Today he began vomiting. Severe vomiting with stomach pain. No fever , just very tired and and can not hold even water in his stomach. Is this an allergic reaction and should he be seen by a doctor? If so, how soon? Doctor: it can be allergic to sting bite or vasomotor reaction to biteit can take some timedrink lot of water warmtake antiemetic for vomitindadd liquid antacid 3 times a dayanti allergic can add ebstine 10 mg bdsyp cpm itsf nightapply local antibiotic and antiallergic combination ointment in affected partfor 3 wk to heal completelytake mefanic acid sos for fever"
},
{
"id": 157411,
"tgt": "Has paranoid psychosis. Fear of cancer. Why is he uneasy? What can be done?",
"src": "Patient: my brother has paranoi sychosis, had to get him admitted to carseview dundee nearly 3 wks ago as the same day we got a call to say my older brother had died due to cancer after bein diagnosed in late april, i myself have advanced breast cancer which is in my liver now, my brother keeps thinkin things are on the net about him and people speakin about him etc, he now think cos my brother died of cancer and iv had it 5 yrs he will get it to, how do i help him over this, some days hes taking a step forward then the next hes awa back, do you get leaflets he can read about paranoi to help get these thought out his head, sometimes wen he fones and says that was all in my head was it, we say yeh keep goin forward, then he will step back and say if you somethin about me would you tell me, i told him wen you get they thoughts in your head just keep sayin out loud its only rubbish its not true, he lives with my mum whos 78 so its a lots for all of us too cope with, he had a day at home then the next day it was over nite but he could not handle it, foned me 7.30 in mornin and i told him fone the hospital , tell them wats wrong with you, my mum had to get in the middle of the nite with him til he settled, does paranoi eventually leave you Doctor: I think it's purely an psychiatric problem and I would strongly recommend to contact a psychiatrist immediately for appropriate therapy ASAP . I hope this clarifies your queries and would be glad to answer more if any."
},
{
"id": 219374,
"tgt": "Suggest medications for terminating pregnancy",
"src": "Patient: I got my last period which was on 21st January 2014, I missed my period this time... I checked with the home pregnancy test on 32nd day...and the result was negative but then again on 26th february 2014, I checked it... its showing positive... which means I m in my 5th week of pregnancy; but the issue is we both husband and wife dont want to keep this baby, as we have a 2 yr old boy and we are not at all prepared for another baby at this moment... please advice me the best way of abortion, Is there is any side effect of M T Pill or what else can be done without surgery and without side effects... If we plan a second baby after 6 months is it possible without any severe problem.... Seek your Advice ... Doctor: Hello dear,I understand your concern.In my opinion the termination of pregnancy can be done by both medical and surgical methods.The medical termination of pregnancy is possible up to 7 weeks.And anything after 7 weeks surgical procedure like D&C is tried.But as you are 5 weeks pregnant the medical method can be tried.It might not cause any problem if taken properly under doctor supervision.So relax and consult your doctor.After the termination you can choose any contraception method to delay the pregnancy.Hope this helps.Best regards..."
},
{
"id": 213313,
"tgt": "Panic attack after tapering the dosage for IBS, taking Escitalopram, Clonazepam, want to discontinue them. Guidance?",
"src": "Patient: I had sufuring form IBS from 1990 to 1995. The dr. give me Fluoxetine 20 mg + amitriptilyne 12.5 mg daily for five years. Then I was OK . After Six months by taoering the dose I discontinue these madicines.But I had panic attack.So my psychiartic give me Escitalopram 20mg a day + clonazepam 0.5 mg 3 time a day ( I am also suffuring from verious type of phobias ). From 2001 to till date I have takeing this madication. Now I want to discontinu this medicine ASAP. Kindly give me guidence. ( If i mis one time dose of clonazepam ihave panic) Doctor: Hi, Panic, phobia and IBS are common in anxiety disorders. Anxiety disorders are chronic illness and patient may need treatment for years, even for lifelong in some cases. In your case clonazepam doses can be tapered gradually. so ask for that from your psychiatrist. As you are still having some symptoms then it is not advisable to stop treatment in your case. I hope this information has been both informative and helpful for you. Wish you Good Health. Regards, Dr. Ashish Mittal www.99doctor.com"
},
{
"id": 221600,
"tgt": "What are the chances of getting pregnant without penetration?",
"src": "Patient: Okay so my bf and I had sex for the first time today, with no condom we were caught up in the moment. If he pre-came in his pants and then we had sex and he pulled out could I be pregnant? Also my mom doesn t know so I can t get birth control or the morning after pill or anything. Doctor: Hello dear,I understand your concern.In my opinion there might be rare chances of pregnancy in your case.There is scope for pregnancy when the semen is ejaculated inside or around the vagina during the fertile period.As nothing such happened you need not worry.The missed period is the first suspicion of pregnancy.If at all the period is delayed do a urine pregnancy test after a week of missed period.Avoid stress regarding pregnancy as stress might also delay period due to hormonal imbalance.So wait and check for the period.Best regards..."
},
{
"id": 93270,
"tgt": "Skin irritation and vomiting. Prescribed oflomac. Why does diarrhea also persist?",
"src": "Patient: my girl child aged 41/2 years is suffering from loose motion, skin irritation and vomiting for the last 2 days and the doctor prescribled oflomac forte, mephtal forte, and hicet and domstal. She is already taking telecast tablet. Now the vomiting is under control. But the skin irritation and diarrhea persits. Do I need further consultation Doctor: Hello,Your daughter is suffering from Acute gastroenteritis.The toxins produced by the bacteria causes itching.Loose stool and itching will subside gradually with the antibiotics.If the frequency of the stool is reduced than the previous day it shows signs of improvement."
},
{
"id": 115241,
"tgt": "Does low hemoglobin cause dizziness?",
"src": "Patient: am a girl 19 years old having some health disturbances cuz of my busy life complicatted with studying .. a lot of stress is likely the baddest issue of mine ... i have sex using condom and before 2 months and delayed period until now i closed 2 moths without it.. but within the first month i had only little droplets of blood thinking it was pregnancy .. i measured my hemoglobin only and its 8mg .. is that thing indicating pregnancy or does the all these dizzness cuz of the low hemoglobin ? Doctor: Hi, dearI have gone through your question. I can understand your concern. First of all your hemoglobin is low and it causes dizziness. You should go for investigation like complete blood count and peripheral smear examination. Then take treatment according to type of anemia. You should go for urine pregnancy test. It is easy and you can perform at home. Go for it. Then consult your doctor accordingly. Hope I have answered your question, if you have doubt then I will be happy to answer. Thanks for using health care magic. Wish you a very good health."
},
{
"id": 17799,
"tgt": "Suggest treatment for takayasu disease with severe pain in the neck",
"src": "Patient: Hi doctor, i have diagonsed with Takayasu Disease 6 months ago.. I have severe pain running down from my neck to my left arm... I have difficulties raising my arm up for 1 miniute... The pain is unbearable.. I cant carry or shampooing my hair, small things are difficult to do... I am very devastrated as i am only 29 and not married.. My future is a question mark in my life.. I was given presodine every 6 hours for two days when i was hospitalised.. But once im discharged i stopped my steriods of 20mg as the side affects are scarry.. Im already suffering, and i dont wish to suffer with other diseases cause by this drug.. Please advise me on how to be cured without steriods.. and what wat happens if it not treated or got worse. Thank you Selvi Doctor: Hi, Takayasu\u2019s arteritis is progressive, and there is no definitive therapy. Glucocorticoids and immunosuppressive agents are effective in some patients during the acute phase but have the risk of side effects as you have experienced with prednisolone... Surgical bypass or endovascular intervention of a critically stenotic artery may be necessary. What you are having is some large artery occlusion in the upper limb, causing ischemic/claudication pain on active movement of the limb. Angiography is needed to delineate focal or segmental changes/occlusion in large arteries in the upper extremities. Synthetic or autogenous bypass grafts need to be placed onto disease-free segments of vessels. For focal lesions, there have been reports of success with angioplasty and may provide relief. Hope I have answered your query. Let me know if I can assist you further. Regards, Dr. Tushar Kanti Biswas, Internal Medicine Specialist"
},
{
"id": 59606,
"tgt": "High BP, groin pain, fatty liver, has skin pain. Could this be due to infection?",
"src": "Patient: My husband has been feeling ill for almost 1 1/2 onths now. He lays block for a livng and is not a small guy. He weighs 130 and is 6 1 and has high blood pressure (recently started meds). He is a light smoker, maybe 2 or 3 a day, and he does not drink. He has recently had an internal ultrasound complaining of groin pain. They have found that he has borderline excessive fat around his liver . Now he is complaining of his skin hurting, clothes are uncomfortable. He has a ct scan scheduled for the 27th. Could he have an infection? What should I look for? What could be causing this? Thanks for any assistance. Doctor: Hello, hollicks5, Your husband is under weight for his height, considerably so. I don't know if this weight loss happened during the past month and a half. You state that he is a light smoker ,half a pack a day, but if he has been a smoker for a very long time , that is of concern. I am glad that he does not drink and we can eliminate that as a possible cause of fatty liver. Poor nutrition can cause fatty changes in the liver. Diabetes , obesity, other metabolic disorders can do this. Certain medications like NSAIDs, Ibuprofen, Naproxen,Sulfa drugs, Tetracycline family of drugs can cause fatty liver. In women, during pregnacy they can develop fatty liver. If your husband does not have fever,chills, nausea, vomiting, diarrhea, it is probably not an infection. He seems to have developed an increased sensitivity of the skin You mention that he was recently started on medication for blood pressure. Look at all the possible side effects listed . I am glad he is scheduled for a CT-Scan. I hope they will do a chest CT along with an abdominal CT. to rule out anything serious. I am sure that he had a complete blood work done by the doctor. Find out about his liver tests, Protein levels, and his blood count to see if he is anemic. My personal feeling is, some of these tests could be abnormal. He should be taking a good Super B-Complex vitamin along with Vit-E 800 i.u. daily. He should be on a high protein diet ,may be with some supplements like Ensure or Ensure Plus or similar drinks 2-3 times a day. I wish him well."
},
{
"id": 95707,
"tgt": "I have been smoking marijuana for 4 years so this is weird. Felt like I was going to faint and abdominal was hurting really bad",
"src": "Patient: Yesterday i was smoking marijuana , ive been smoking for 4 years so this is weird . Im not sure if that had anything to do with it , . . and all of a sudden my head got hot , I went to the bathroom and knelt as if i had to throw up i felt like i was going to faint . My body shook for a couple seconds , my vision was grey ish and i had extreme ringing in my ears . nausea stopped and i got really sweaty i went to throw water on my face and my face and lips were really pail after my abdomnail was hurting realy bad i felt gassy . It went away but thats the 3rd time thats happend its been awhile though . Whats wrong with me ? Doctor: hello welcome to health care magic all these are side effects of this drug ....quit smoking marijuana .have many hazardeous effects and you will face many problems in future...drink 12 glasses of water every day for 25 days to wash out toxins,,visit gud physician near you for routine check up. take care payal mail me at payal rawat16@yahoo.com"
},
{
"id": 175899,
"tgt": "Suggest treatment for imbalance issues in children",
"src": "Patient: My son is diagnosed with Cerebral plasy by birth, he is 03 yr now, his body parts looks very normal but he does not achieved body balance to sit, stand on his own. he speaks few words and understands others speaking better. Please suggest me any treatment/therapies to make him achieve body balance. Doctor: HI...The best way to treat and expect some improvement in this issue would be Physiotherapy. If the kid is having seizures, they have to be controlled effectively. Occupational therapy helps these kids in getting accustomed or to be trained to their own activities on their own.Regards - Dr. Sumanth"
},
{
"id": 93091,
"tgt": "Severe pain in right side of stomach area, low WBC count. Reasons ?",
"src": "Patient: my child is complaining with severe pain in her right side and stomach area .It hurts when she walks as well as trys to eat.We took her to er last nite they said she had mono as well as a swollen spleen there were no test done except for a blood test they said her white count was at 0000 i dont believe what she was diagonosis with there wasnt any antibotics given xrays or mri plz help Doctor: Hi,There is usually only one cause of low WBC count. That is typhoid fever. Also, since you have history of stomach pain as well, this seems to fit the diagnosis pretty well. What you need to do is to meet a MD Internal MEdicine who will diagnose the patient and then start the appropriate treatment. I hope this helps. Good luck.Dr Vineet"
},
{
"id": 111640,
"tgt": "Could back, neck and pelvic pain be attributed to spondilitis?",
"src": "Patient: hi can you plz help me ? l have very bad lower back pain but only when l am standing! it gets worse if I stand longer than 10 min. it gets so bad that I can barely walk!! it is like my back , hips &pelvic area freeze ! it makes it hard to even sit back down !! I have lumps on both sides of my spine . it is ruining my quality of life!!!!!! could this be a episacral lipoma?? however l have had a m.r.I done...and have spondylolthesis!!! I don t think that is what is causing my pain.??!!! I can touch my toes &bend backwards without any pain. no leg or foot pain. no trouble using the bathroom either. I think it s a back mouse! lol plz help thanks, tammey Doctor: Hello, Thanks for your query.These cold be related to degenerative disc and spondolytis.I will advise you to do to do x ray spine and MRI spine.You may need muscle relaxant with analgesic,and neurotropic medicine.lumbar corset and hot fomentation will give faster relief.Physiotherapy like USG and SWD will give quick relief.Spine exercises done regularly will prevent reccurance of such episodes.I do hope that you have found something helpful and I will be glad to answer any further query.Take care"
},
{
"id": 149299,
"tgt": "Seizures at interval of 2,3 months. Taking medication. How can this be completely controlled?",
"src": "Patient: My father is 80 years old and gets seizures almost at intervals of two to three months . He is taking 1000 mg leviplil morning and 2000 mg Levipil in the evening after dinner . He has also been prescribed 5 mg Frisium which he DOES NOT take regularly as he feels lot of dizziness He takes Frisium only twice in a week. Please advise how his seizures can be completely controlled . Doctor: HelloWelcome to HCMYour father is 80 years old and he is having seizures episodes at intervals of 2-3 months and he is currently taking 3000 mg Levipil and 5 mg Clobazam daily. Even on the medicines the seizures are not under proper control. He is not taking Frisium or clobazam regularly. First of all Levetiracetam and Clobazam both are newer drugs and both are adjunctive drugs for epilepsy especially for generalized seizures. They are more useful in partial seizure. Although you have not mentioned the details of his symptoms still I assume that he is having generalized seizures because seizure in general terms denotes generalized seizures. Consult his doctor for management as many broad spectrum drugs like Valproate, Carbamazepine etc are very useful in seizures.Get his MRI done to rule out any organic cause of seizures.Thanks"
},
{
"id": 95283,
"tgt": "Is it good to remove gall bladder, will it affect in any other way ?",
"src": "Patient: hello, I suffer from intermittent stomach pain and the diagnosis revealed gall stones of 2 mm in size in a sludge in the gall bladder . Doctor suggests gall bladder removal for the same. Is it good to get the gall bladder removed? will it affect me in any other ways. I am just 27 years old. Doctor: Hi, Intermittent pain can be because of gallstones, however if the stone is solitary and a very small size less than 5 mm it is better to have yourself a repeat scan after few days of normal diet and the scan to be done after overnight fasting. Small solitary stones can be treated by medicines. Removal of gallbladder does not alter the health and digestive system in anyway. wishing you a good health."
},
{
"id": 171226,
"tgt": "What causes the condition to constantly swallow and expel tiny burps?",
"src": "Patient: I believe that my daughter is suffering from anorexia. She has developed this condition where she constantly swallows and expels tiny burps. Is this part of anorexia or could it be something else. Also, she was born with Corrective Transposition with Pulmonary atresia Doctor: Hi...it would have been better if you had mentioned the age of your kid. By what you quote I do not feel that it is anorexia. I feel that she is doing it by habit and it is habit disorder. The best way to manage this is by tender loving care and get her out of the habit. If not it could also be due to attention seeking behaviour in which case it is better to ignore it. If nothing is working I suggest you get in touch with a clinical psychologist.Regards - Dr. Sumanth"
},
{
"id": 207389,
"tgt": "What causes hallucinations during pregnancy?",
"src": "Patient: im 27 weeks pregnant, 2 years ago i lost my first child to cot death, during this pregnancy ive started to get to a point in my mind where i think im paranoid i think people want me to have this baby for them, i think people dont want me to have the baby they want the baby, like people are watching me everyhing i do trying to point out any reasons they can to make me feel like a useless mother, im scared that people are trying to take away my baby. whats wrong with me? Doctor: HIwell come to HCMI really appreciate your concern, this is not the hallucination but this is phobia, distrust, false accusation, delusion, and these all happened due to anxiety and depression, I would advise you to get way from all these may not be good for baby, hope this information helps."
},
{
"id": 165212,
"tgt": "What causes UTI with soreness in stomach and urine?",
"src": "Patient: My daughter aged 6 years 112cm and 20kg had double STING procedure to correct her defected urethra tubes. She suffered constant UTI s. She still complains of sore tummy and her urine is still very cloudy but no foul smell. Her urine samples have come back negative.Her last ultrasound showed a slight swelling in her left urethra. Is there anything more that can be done or needs done? Doctor: he must done MRI to know the type of swelling an must take agood amount of water and fluid and eliminate the salt from diet"
},
{
"id": 145881,
"tgt": "Are dizziness and tingling signs of a seizure being blocked?",
"src": "Patient: I take Keppra for Epilepsy twice a day. I am also taking Tylenol for arthritis recently. I have had some dizzy spells with tingling in my arms. I have also had some wine consumption. are the symptoms due to the combination of the three? Is the dizziness and tingling signs of a seizure being blocked? Doctor: As per your narration it is not clear about the type of seizure. Localized symptoms to one side is usually associated with some lesion in brain. Sometimes such patients who initially were having partial seizure with secondary generalisation or generalised tonic clonic seizure may have only some minor sensory phenomenon to the corresponding side due to drug control. Rarely some patient have lesion at sensory cortex which may present with tingling in one arm or leg or half body. Alcohol usually don't present this way but it is advisable to stop it's intake as it lowers the threshold of seizure and affects drug level in blood leading to increased risk of seizure. Hope you understand and comply. Wishing for your good health."
},
{
"id": 83517,
"tgt": "Why duphaston is banned in USA?",
"src": "Patient: Why duphaston is banned in USA? I am from Poland and I am 12 weeks pregnant. I got 3x1 tab perscription from my doctor. I am worried as duphaston is banned in USA, UK. In Poland they still use this. I ve got haematoma 21x12mm under the placenta. This is my second pregnency. First one was 100% delivered on time with no issues. Doctor: Hi, Based on the evidence duphaston is banned in UK as well as in US for the commercial reasons. If you are residing in US then ask your doctor for an alternate effective drug to support and maintain the pregnancy as well as to prevent miscarriage and preterm delivery for which Duphaston has been widely used. Hope I have answered your query. Let me know if I can assist you further. Take care Regards, Dr. Mohammed Taher Ali"
},
{
"id": 91158,
"tgt": "Suggest treatment for abdomen pain?",
"src": "Patient: ENGIOPLASTY (CAD) 2009 taken ECOSPRIN 75 OD DEPLETT 75 OD FIBATOR 5MG CARDIC 1.5 MG OD BUT SUFFER STOMATCH PAIN NOW I AM TAKEN PANKEROFLAT OD AND GANATON 50 OD BUT STOMATCH NOT FIT ALSO SUFFERING PAIN IN LOWER ABDOMAN TAKEN ULTRASON IT IS OK ALL PROSTATE TEST EVERY THING OK NOW WHAT I CAN DO Doctor: You are having gastritis. I would advise you to take Tab Pan 40 twice daily empty stomach and syrup Gelusil 1 tsf three times a day. Drink plenty of water, avoid spicy food, add lot of dietary fiber in your diet."
},
{
"id": 144187,
"tgt": "What causes back pain in a person with disc herniations?",
"src": "Patient: Hi, I am experiencing lower back pain, right above my buttocks on both sides, left is worse. Blood work just came back normal kidney function etc. I do have cervical, thoracic and lumbmar disc herniations but this has been killing me for the past few days. Doctor: Hi, I am Dr.Bruno. I have read your question and understand your concerns. Let me try to help you Question : What causes back pain in a person with disc herniations?Answer : The Herniated discs impinge on the nerve roots and this causes pain Hope you found the answer helpful.If you need any clarification / have doubts / have additional questions / have follow up questions, then please do not hesitate in asking again. I will be happy to answer your questions.Let me know if I can assist you further.Take care."
},
{
"id": 149850,
"tgt": "Chronic back pain due to fusion from L3 to S1 and have a spinal cord stimulator. IS there a possibility of loose wire from the stimulator?",
"src": "Patient: Hello, I have been on a lot of meds for yrs with no changes. I have Chronic Back Pain . I have a fusion from L3 Tito S1 and c(5 to C6. I also have a Spinal Cord Stimulator . Over the last few weeks I have been found during the night at 5am, 7am, and last night I got up myself at 4.30am. There is a possibility that there may be a loose wire f From my Stimulator. Some nights I get up from bed because of pain, thus the reason my husband does not miss me from bed. I am really worried at this stage but am trying not to add more anxiety to my family. Have you any idea what it could be. Thank You. Doctor: Hello, Fusion both L3- S1, C5-C6. Spinal cord stimulator???What for???Meet your neurosurgeon to checkout things, if not satisfied, find a good neurosurgeon in your locality.hope you got the answer.Regards."
},
{
"id": 212192,
"tgt": "Difficulty in managing anger. What can be done?",
"src": "Patient: my uncle had a pleasant personality, he was calm headed but since few years he looses temper very quickly secondly he becomes hyper and starts using languages which is inappropriate and in a nut shell he gets out of control He lost his job last year since then he has been trying but as yet there has been no luck, even places who have given him positive feedback after interview take a hell lot of time to reply as to whether they have accepted him or not we all are very worried at this behavior of his Doctor: Dear, I understand your concerns. It is not clear from your description whether he has anger from long time or not. If it has started just some days back, it might be due to anxiety about his self worth Losing jobs and inability to find a new job undermines self worth and self confidence, which in turn creates stress and anxiety. Take him to a mental health counselor. Instead of controling anger, he should learn to adjust to the anger creating situation. Then anger is just an emotion. Let him know his life and he will become alright. Wish him speedy recovery"
},
{
"id": 76936,
"tgt": "What causes dull pain in chest?",
"src": "Patient: Hi, I am a 34 year old woman with unexplained chest pain. I have had several tests including a chest x-ray, EKG, stress test, echo, and ct of the chest. All test were normal. However, I am still having pain in my chest on the left side. I do have a Lap Band and experience heart burn at night. I am on meds for the acid reflux. What could be causing the dull pain in my chest? Doctor: Hi thanks for contacting HCM...Noted your all test comes normal...So here two causes seems according to your history.....-reffered pain from acid reflux or-musculoskeletal pain....Continue treatment for reflux with drug.Head elevated while sleeping...Avoud excess spicy and fatty food...Chocolates, coffee avoided in excess.Avoid heavy meal at night.Second pain could be musculoskeletal.Resolve by its own within few days.Rest needed.Hot pads can be applied....Avoid heavy weight lift ...and strenous exercise ....Take care.Dr.Parth"
},
{
"id": 186670,
"tgt": "What causes pain in the ear?",
"src": "Patient: The wisdom tooth on my left side has broken off I guess in half you could say and has been causing me a lot of pain. But now I'm also noticing pain in my ear going into my neck, sort of like an ear infection. Could the wisdom tooth be connected to this problem? Doctor: thanks for your query, the pain could be radiating pain from the wisdom tooth..consult a oral physician and take a radiograph and get the tooth removed..you can take a course of antibiotics and analgesics..do saline gargling...i hope my answer will help you, take care.."
},
{
"id": 220146,
"tgt": "Could smoking during pregnancy effect the fetus?",
"src": "Patient: Hi my name is nia and i m 10 weeks pregnant i went to my obgyn and they did an ultrasound and they said that the baby had a good heartbeat and at my next doctor visit they will hear the heart beat well the problem is that since i smokes ciggarettes could anything go wrong ? Doctor: Hallow Dear,Smoking - whether active or passive - is definitely harmful to the mother as well as baby in utero. Nicotine in tobacco causes detrimental effects on the blood vessels. Hence rise in mother's blood pressure and placental starving due to constriction of the blood vessels are common effects of tobacco. This placental effect leads to intrauterine growth retardation of the baby. In addition to high blood pressure, there are many effects on the mother's health like hyperacidity, confusion, sleeplessness, risk of cancer, etc. Tobacco in any form is harmful - smoking, chewing, snuffing or applying to gums and teeth (mishri). So avoid tobacco use completely, and also avoid passive smoking also totally. I hope this gives you message. Dr. Nishikant Shrotri"
},
{
"id": 202400,
"tgt": "Lump under foreskin, bleeding, pain post ejaculation, red ring around penis head. Treatment?",
"src": "Patient: For a couple of months now i have had a small lump under the foreskin . It doesnt sting in the shower but when i am having sex with a condom on it, when i release it stings just where the lump is as the condom is holding everything in around it. I never passed too much heed of it. Recently my partner and i have been able to have sex without a condom and it improved a bit until the last time and after we had sex i seen it was bleeding , we realised i must have been bleeding during the sex too. This lump is so small sometimes you would wonder is it just a vein just slightly raised? I dont know how it was bleeding or if i got a small cut. There is a red ring around the bottom head of the penis , it would be lower down than the lump. you can only see the lump or red ring when erect or foreskin pulled back. Should i be worried. will this heal and go away itself or do i go to a doctor? Doctor: Welcome to the forum.The lump may be an insignificant lesion, but the red ring needs examination, as it can be bacterial or fungal infection.It is better to consult a urologist or a general surgeon.Sincerely,"
},
{
"id": 65601,
"tgt": "What causes a hard lump on the arm?",
"src": "Patient: I am a 32year old female, I weigh 120lbs. I was seen 4yrs ago because of a discoloration(appeared to be broken blood vessels) they did a biopsy and found it to be nothing. The broken blood vessels went away and I had no problems until a few weeks ago my left arm went numb and when I rubbed it I noticed a hard lump the size of a gumball. It does not really bother me, I gave it a few weeks to see if it would go away...still no change Doctor: Hi! thanks for your health query on HCM!Well, you have a significant medical history behind the hard lump on the lump. You had broken vessels that healed itself; you have numb left arm; and the lump appeared after you rubbed the numb area; it does not seem to reduce...I must consider following possibilities in this case:1. post traumatic neuroma-like benign nerve tumor2. inflammatory stromal nodule3. neurofibroma or schwannoma4. parasitic cyst5. aneurysm or vascular malformationYou need a doppler/ultrasound study for confirmation, in addition to a clinical examination; therefore please see your doctor but nothing to worry really!regards,"
},
{
"id": 60295,
"tgt": "Is Portal Hepatopathy curable ?",
"src": "Patient: hello sir, my father Dr. PRAVINCHANDRA RAO is suffeing from PORTAL HEPATOPATHY.ON 9 TH FEB HE HAD BLOOD OMITTING 1ST TIME. THEN HE HAD BLOOD OMITTING ON 20TH APRIL AND 6TH JUNE. HE HAVE UNDERGONE 3 ENDOSCOPY S AND DOCTORS HAVE FITTED IN ALL NOW 7 CILESTIC BANDS ON HIS VEINS NEAR LIVER . THEY SAY THAT VEINS HAVE SWELLING.CAN YOU PLEASE SUGGEST ME SOME TREATMENT . PLS SIR REPLY FAST WE ALL ARE LIVING IN PANIC. IS PORTAL HEPATOPATHY CURABLE???? Doctor: Hi, Welcome to HealthcareMagic Forum. Was he a Alcoholic? Your Dad's History suggests, he has Portal Hypertension which is an increase in the blood pressure within a system of veins called the portal venous system. What other symptoms does he have? The treatment for this focuses on preventing or managing the complications, especially the bleeding from the varices. Diet, medications, endoscopic therapy, surgery, and radiology procedures all have a role in treating or preventing the complications. He should stop drinking alcohol completely, He will probably be required to consume no more than 2 grams of sodium per day and reduce his Protein intake as well. If endoscopic therapy, drug therapy, and or dietary changes do not successfully control his variceal bleeding, he may require Transjugular intrahepatic portosystemic shunt (TIPS), This procedure involves placing a stent (a tubular device) in the middle of the liver. The stent connects the hepatic vein with the portal vein which reduces the Portal vein Blood Pressure. Kindly don't panic, speak to his Doctors and discuss these aspects with them I Wish him Good Health, Take Care."
},
{
"id": 155095,
"tgt": "Is chemotherapy advisable for a third brain tumor?",
"src": "Patient: I have recently been told that chemotherapy to try and reduce my third brain tumour has stopped working. I am 50 and I'm weak. Im trying to decide whether to try aain or to give the treatment up and live a better life, how long do you think i have left? Doctor: Hi, dearI have gone through your question. I can understand your concern. Treatment depends on type of your tumour. BBecause chemotherapy is useful in certain tumours. And prognosis and life expectancy depends on type and stage of your brain tumour. Send me your all reports so that I can help you further. Hope I have answered your question, if you have doubt then I will be happy to answer. Thanks for using health care magic. Wish you a very good health."
},
{
"id": 2167,
"tgt": "Can you still get pregnant using a condom?",
"src": "Patient: Me n my frnd had protected sex with condom on, now she has missed periods for 3 months continuos. She fears she would be pregnent ,but we are sure that she is not what could be reasons she is missing her periods and what are the chances if getting pregnent, she also did UPT , it was negative please suggest Doctor: Hi, Thanks for asking. I understand your query. Sex with condom can cause pregnancy .. when the condom tears in vagina.-ve pregmancy test excludes pregnancy. Though it needs to be repeated to confirm. Your friend should get investigated for finding the cause of stopping the periods for 3 months. (It could be severe anemia/ chronic or debilitating illness/ hormonal imbalance(thyroid,ovarian cause or pcod). Lrt her consult a gynecologist for finding the cause followed by specific treatment. Thanks."
},
{
"id": 210420,
"tgt": "Does taking Cymbalta,Welbutrin and Xanax together cause mood swings?",
"src": "Patient: PLEASE YES ..my daughter was given cymbalta -welbutrin-xanax and adderall all at once, by a pa.we have noticed wild mood swings , loss of motivation and a great deal of emotional ups and downs.is it possible this is a bad drug interaction and is it common practice to perscribe these together? She had orginally mild depression following a breakup with her boyfriend . Do you as a MD feel its ok to use all thse drugs together.... whats more she is drinking a lot almost every night. Doctor: Hello and welcome to Healthcare Magic. Thanks for your query.From the information that you have provided, your daughter had been siffering from a mild depression following a personal stressor. In my opinion, I find it a bit unusual that she has been prescribef a combination of 4 mefication, including 2 antidepressants, for a mild depression.I'm also not sure why she was prescribed Adderral which is a stimulant medication usef in the treatment of ADHD.According to most national guidelines for treatment of depression, ther person has to be startef on one antidepressant and has to be observed for at least 4-6 weeks on an adequate dose, before switching to or adding on another antidepressant.It is possible that the current combination of medicstion could be precipitating her mood swings. Excessive alcohol intake could also be another reason for this. I would suggest that you take her to a good psychiatrist for a detailed evaluation and further treatment.Best wishes."
},
{
"id": 106977,
"tgt": "How can backache on the right side be treated?",
"src": "Patient: I have had pain in my middle right side back area for 10 days. Sleeping is difficult and painful. Can only sleep on my back, yet uncomfortable. Bending, stumping difficult and painful. Getting to sitting position and rising up painful and difficult. Advil doesn t help. Doctor: hi sir/madam,Thanks for your question on Healthcare Magic.Let me advice you for your problem.Low back ache may be a pre-monitory (prodromal) symptom in fistula in ano, sciatica, gouty arthritis, hernia, emaciation disorder etc. Pain is confined to lower part of the spine (back bone) especially lumbar region or lumbo-sacral area (rarely sacro-iliac region also). If it is secondary, earlier history of fall or injury may be associated. Rarely, radiating pain may be complained by the patient towards lower limbs. But it is quite common in low back aches if the defect is in the discs between vertebrae (back bones). Often the movements of lumbo-sacral region like flexion and rotation are hampered either partially or completely.Low back pain may be found in mild form in case of anemia, sciatica, rheumatoid arthritis, hemorrhoids, urinary calculi, uterine disorders etc also.Line of treatment as per Ayurveda:-The general principles of treatment of vata dosha are adopted in case of katishoola (low back pain). It includes various measures to suit its varied clinical entities, stages and associated complaints.1. Snehana (oleation) \u2013 by sneha dhara (pouring oil), abhyanga (oil massage), avagaha (tub bath with oil or oleaginous medicaments), kati basti (retaining medicaments on the back) etc.2. Swedana (sudation) \u2013avagaha sweda (sweating treatment with tub bath), pizichil (kayaseka), nadi sweda (sudation through a tubular device \u2013 local sudation), panda sweda (sudation through medicated paste or powder) etc.3. Mridu Samshodhana -mild purgation.4. Basti (medicated enema) like eranda basti, vaitarana basti, pippalyadianuvasana basti etc.Formulations indicated in Low back ache as per Ayurveda:-1. Dashamoola kwatha.2.Maharasnadi kashaya.3. Rasnaerandadi kashaya.4. Sahacharadi kashaya.5. Gandharvahastadi kashaya.6. Trayodashanga guggulu.And for local application these oils will help:-1. Ksheerabala tail.2. Mahanarayana tail.3. Dhanwantaram tail.4. Maha narayan tail.Avoid these for better results:-1.Bitter, astringent and pungent food2.Cold water3.Fear4.Exhaustion5.Standing6.Driving7.Cold food and beverages etc.Yoga is also very helpful in treating your back ache.Now for your backache do these yoga aasana:-1. Ardha Matsyendrasana(Sitting Spinal Twist).2. Dhanurasana (Bow Pose).3. Marjariasana (Cat Stretch).4. Balasana(Child Pose).5. Halasana(Plow Pose).6. SUPTA MATSYENDRASANA(THE TWO KNEE SPINAL TWIST POSE).7.Adhomukha Shwanasana (Downward Facing Dog).8. Urdhva Mukha Svanasana(Upward Facing Dog Pose).9. Paschimottanasana(Seated Forward Fold).Follow these yogas you will get best results for your problems.Hope i was helpful.Have a healthy day."
},
{
"id": 197891,
"tgt": "Suggest treatment for burning sensation inside foreskin of penis",
"src": "Patient: Hello doctor, Am 30 years old. I am having a problem with my penis. I am getting burning sensation inside my foreskin and I could find some whitish dead tissues/cells when I pull down the foreskin. It stopped many times when I apply some ointments. But not getting rid off it permanently. Please help me. Doctor: Hi,It seems that you might be having chronic prosthtitis.Common cause is non cleaning smegma collected on glans.Clean smegma daily by averting foreskin while taking shower.Apply triple action cream for few days.Ok and take care."
},
{
"id": 22879,
"tgt": "How long it will take to reduce the high BP?",
"src": "Patient: This is Darryl, I am 26 years old, native and a resident of India. I am a regular alcohol(whiskey) consumer and frankly immoderate in my quantities. Last noon I had an episode of high blood pressure that reached 230/140, ever since then it has reduced quite a lot, I stopped alcohol consumption immediately too.. Now 30 hours since the episode my B.P. still exists at around 150/110.. How long would this take to completely subside. I do have plans of stopping alcohol from many days, but since yesterday s episode, perhaps the time has come... Kindly guide me..... Doctor: hi, you must having underlying blood pressure problem , over and above you had alcohol binge leading to excessive rise in blood pressure. In normal person blood pressure will not rise this much. So blood pressure may not reach normal in your case. You need to monitor your blood pressure and Star on blood pressure medicine.Also consume low salt diet, regular exercises."
},
{
"id": 8354,
"tgt": "How to treat dark patches and tan lines?",
"src": "Patient: Hi when I was younger I live in a country which was hot 24/7 and I didn t leave until around 5 years later, but ever since then and now I feel that my skin colour become darker because I see tan lines and patches around my body and my face, I currently live in the Uk, I was wondering is there any way I can remove the dark tan I ve got an get my natural skin colour back please? Doctor: HIWell come to HCMBecause of the hot climate and sun your skin color must have been change, although this would come around on its own and may take some time, still condition can be treated with, \"Hydroquinone cream\" drink more water, have fresh fruits, be patience every thing would be fine soon, take care and have a nice day."
},
{
"id": 125656,
"tgt": "Extremely painful fissure that doesn t go away with creams",
"src": "Patient: i had fissure for 4 months now and is extremley painful.ive been given cream pain.some days standing is unbearable.i also take pain killers to and always make sure stools are soft by taking laxatives but still very painful for hours after.is there anything else i can try?sometimes i take iomium just so i get some relief and have to go somewhere.please help.im 42 cant work properly and like my life back.ive had mastectomy october and know my body trying to repair more than one thing.thankyou. Doctor: Hi, Rectal fissure is a difficult condition to treat. If you getting relief with the current treatment please continue till you get complete relief. Mastectomy has no direct relation except constipation (due to bed rest and change of diet) which you will have to take care. Hope I have answered your query. Let me know if I can assist you further. Regards, Dr. Gopal Goel, Orthopedic Surgeon"
},
{
"id": 22986,
"tgt": "How to treat tachycardia and increased heart rate due to vasoconstruction?",
"src": "Patient: Hi, I'm a student at a Belgian university and I kind of ran out of time to finish learning my exam. So someone here in the studenthome gave me a mix of herbal extracts an caffe\u00efne for the night. I think this was not the case, I've had tachycardia + increased heartrate all night long, also my limbs are a bit cold and numb, due to vasoconstriction (also blue knees and weird skin colour) My roommate told me I was probably given mephedrone! and actually I think he's right, it matches all the symptoms. My exam is tomorrow morning, I recieved the \"booster\" yesterday around 9pm. Can't fully concentrate and also I'm quite \"amped up\" if that's the correct word. I know I've been stupid but is there anything I can do about it, I've tried eating chocolate for the vasoconstriction, but eating is hard. I'm 21 years old, weigh 85kg and 175cm. I'm actually a dentistry student, and I know it's not lifethreatening at the moment, but I just want to be able to focus asap. Thanks in advance! Doctor: you can tab nifedipine 60 mg extended release preparation for vasoconstriction. if you are having too much tachycardia or palpitations you can have tab ciplar LA 40 mg, it will resolve palpitations and anxiety."
},
{
"id": 72609,
"tgt": "Is Amoxicillin ideal in case of cough and sore throat?",
"src": "Patient: I was a soprano. 3 weeks ago, I came down with a cold, starting with s sore throat. Did not miss a day of work, since I am a teacher. Did not lose my voice neither. Got a prescription of Amoxicillin, 500 mg., and took it for a week and started to cough. Ever since then, I can't even hitthe lowest note. I can talk but can't sing at all. Anything I should do to gain back my singing voice? Doctor: Hello dear , hiWelcome to Healthcaremagic.comI have evaluated your query thoroughly .* Amoxicillin does work better for upper and lower respiratory tract conditions but as you have already consumed it for a week and current symptoms intensity demands change of the antibiotic .* Additional guidelines - Gargles with salted lukewarm water added peppermint oil 3 times a day- Drink plenty of liquids .- Balanced nutritious diet with avoidance of oily , sugar , spicy .Hope this may help you .Wishing you fine recovery .Keep constructing the future of the world by teaching millions .Regards ."
},
{
"id": 105622,
"tgt": "Ear and throat hurts while swallowing, used allergy medicine. Should take nasal allergy spray?",
"src": "Patient: Hi my problem is my left ear and left side of the throat hurts when I swallow ....Im not sure if its allergies or a cold.....I use allergy medicine and have bad allergies but i havent taken my medicine lately......and allergy season is super high right now, I m really hot and my forehead is like hot and stuff and its been warm all week except for this morning, so should I just take my nasal alergy spray or what Doctor: take antiallergic medicine tab fexofenadine 120 mgm daly for 5 days application of neosorin h ointment inside nose if not controlled you can take flomist nasal spray after 5 days"
},
{
"id": 222023,
"tgt": "Suggest treatment for vaginal bleeding during pregnancy",
"src": "Patient: 5 weeks pregnant, last sunday was bleeding black/dark red blood about 2 pads full - only had bleeding for 1 day still feel sick.. done a 2 test last monday one said negative one positive, ive had a miscarriage before the blood was bright red and there was loads of it.. not sure whats going on, i rang my doctor he said he didnt want to see me he said id had a miscarriage..can you advise? Doctor: I think you had miscarriage and I suggest you go for usg for confirmation. Because upt test remains positive at least for 1 week after miscarriage so usg is best for confirmation."
},
{
"id": 13947,
"tgt": "Suggest treatment for itchy rash on various parts of the body",
"src": "Patient: Hi, I have a rash in various parts of my body, it has what appears to be small whiteheads, they are dry. It seemed to have started on my pelvis with a red area the size of a lid of a pen that was itchy and had a scaley look, a second one the same size appeared on my lower stomach area with same look. The scaley look has now vanished and it s simply a red mark that s itchy, it began around 2-3 weeks ago. I m 36, over the last 2 months I have been training hard every day, can t point to any change in products or food. Thanks Doctor: Hello,There are many probable diagnoses for your complaints. It could be a folliculitis or Pityriasis lichenoides chronica. Treatment options differ for these conditions. So, I would recommend you to kindly consult your Dermatologist for confirming diagnosis and initiating treatment.Hope I have answered your query. Let me know if I can assist you further."
},
{
"id": 171040,
"tgt": "Does epileptic discharge lead to high grade fever?",
"src": "Patient: my son is 5 year old, his wt is 12.5 kg & ht.is 3ft sir his problem isthat when he is 1y old he got suffered from high grade fever its 105 &he had seizers and this month again same situation had happened this time doctor suggest us to EEG in EEG we found Epelepsic discharge from left side .sir, i want to know that fever is related to that problem only Doctor: Hi...by what you quote i feel that your kid is having simple febrile seizures. It is common for some kids to have this during very high temperature. 4% of normal kids in general population can have this type of seizures form 6 months to 6 years of life.For just one episode there is no need to do an EEG or any other brain scan. The EEG will obviously be abnormal if performed within one week of any seizure. This doesn't mean that the kid will require anti epileptic drugs.All the treatment that is required is using of Clobazam for the first 2 days of any fever. This is a prescription drug and you need to consult your pediatrician for this. I am telling to use Clobazam only for the first 2 days because the probability of the kid getting seizures is most in the first 48 hours of fever.Coming to your query, the seizures are due to high fever, but the fever is not due to seizures.Regards - Dr. Sumanth"
},
{
"id": 13911,
"tgt": "How to treat itchy rashes all over body after delivery?",
"src": "Patient: I had a baby 7 weeks ago and have had an extremely itchy rash all over my body since week 3. I'm told it's caused by the hormone prolactin. Any diet changes recommended or how can it be treated? I have been using an anti-itch cream to aide w/ the itching, and I've taken Zyrtec for the last week, but it isn't helping. Suggestions? Doctor: Hi, Your symptoms seem to be related to prolactin hormone. I suggest you to take antihistamines daily to relieve the itching. You can also use Calmine lotion for local relief. Hope I have answered your query. Let me know if I can assist you further."
},
{
"id": 33734,
"tgt": "Suggest treatment for painful urination along with whitish vaginal discharge while suffering from chronic E.coli infection",
"src": "Patient: Hello Doc, My wife Age 29 Yrs, Ht - 5'3\", Wt - 51Kgs. Suffering from Chronic E-Coli infection since more than one year. She was medicated with various antibiotic courses , includes Amycycin, Gentmycin, Fluconazole, Nitrofurtion ,pencillin and even 3rd Generation antibiotc was also administered but no relief at all. kindly help how to get rid of E-coli, she is complaining of severe burning sensation during ,after urination, stomach pain, white dischagre from vagina. she feed up and looking suicide as her last solution, kindly help us for correct treatment in Hyderabad , India Doctor: HiThank you for asking HCM.I have gone through your query. E coli is a bacteria found in stools. Her urinary tract should be examined by an urologist to rule out any fistula. Cleaning of anal side should be done in one direction and don't have contact with that hand on urethra and vagina before washing. Using intimate washes will be helpful in maintaining normal vaginal PH and flora. Suicide is not a solution for anything. I am sure that an experienced urologist can solve your problem.Hope this may help you. Let me know if anything not clear.Thanks."
},
{
"id": 97875,
"tgt": "Diagnosed with fibroid inside uterus. Started homeopathy & herbal. Now in ultrasound uterus seems smaller. Suggestions?",
"src": "Patient: HELLO DOCTOR. I HAVE BEEN DIAGNOSED WITH A LARGE FIBROID INSIDE MY UTERUS. I HAVE STARTED HOMEOPATHY & HERBAL MEDICINE & JUST REC'D MY ULTRASOUND RESULTS AFTER 1 MONTH OF THERAPY. MY UTERUS HAS GOTTEN SMALLER BUT FIBROID DIMENSIONS GOT BIGGER.WHAT DOES THIS MEAN??? IS THE UTERSU SHRINKING AND CAUSING THE FIBROID TO DISTORT IN SHAPE Doctor: Hello, Thanks for the query to H.C.M. Forum.The size of fibroid has increased that is the cause of concern. In my opinion you should get in an operation (surgery) of fibroid. This surgery is the only one option. Good luck. Dr. HET"
},
{
"id": 207679,
"tgt": "Suggest treatment for personality disorder with depression",
"src": "Patient: i think i have a personality disorder. im constantly sad, depressed, i HATE myself, i cant get along with anyone, i cant be around to many people at one time or i freak, i cry for random reasons, and sometimes i cant have my back to people because they can read my mind..... and thats only a few things i feel. help? Doctor: DearWe understand your concernsI went through your details. I suggest you not to worry much. You may not be having any personality disorder or depression aas you suggest. This feelings and worthlessness could just be due to some set back or negative experiences happened in your life. You need to understand disappointment and clinical depression. Experience makes a man perfect. Small and big negative life experiences creates a perfect and wise man. Understand why you are like this. Talk to your parents, friends. If need be consult a psychological counselor.If you need more of my services, please post a direct question to me in this website. I am happy to serve your purpose.Hope this answers your query. Available for further clarifications.Good luck."
},
{
"id": 130031,
"tgt": "Suggest treatment for pain in neck and shoulder",
"src": "Patient: I have SHARP STABBING PAIN IN LOWER SPINE LIKE THE TIP OF A PAIRING KNIFE AND HANGS AROUND IN PERIODIC INCREMENTS. ALSO HAS A ELECTRICAL SHOCK THAT RUNS FROM NECK DOWN BOTH ARM WHEN RAISE ARMS AND LOWERRAISE HEELS AND RETUN TO FLOOR WHEN I REACH OVER HEAD AND LOWER ARMS PAIN IN NECK CONSTANTLYRIGHT AT THE SHOULDER AREA THEN AFTER THAT IT GRADUALLY WORKS ITS WAY DOWN TO LOWER BACK. Doctor: Hi,It may be due to cervical spondylosis.So get xray of cervical spine done and send me the report.Till then have tab.aceclofenac100mg+thiocolchicoside4mg twice a day by prescription of family doctor.Thanks,Dr.CHANDER MOHAN SINGH."
},
{
"id": 76107,
"tgt": "Can acid reflux induce chest pain?",
"src": "Patient: I have pain in my chest, under my left breast, happens quite often, recently had an ekg, doctor said everything fine, could be just acid reflux; I have been under a lot of stress in the past year or so taking care of my mom; my primary doctor says life is tough, life goes on and to just deal with the stress Doctor: Hi thanks for contacting HCM...Yes acid reflux and gastritis can lead reffred chest pain....Avoid taking stress that can aggravate your condition...For stress you can try yoga and exercise daily...Psychiatric counselling session will also be helpful....If needed psychiatrist may prescribed you anxiolytic drugs.. .You can take omeprazole acidity ...Avoid excess spicy foods...Avoid smoking, alcohol , meat like irritants...Excess tea and coffee avoided...Drink lots of water.....Don't worry .Try to be tension free....If chest pain increasing consult physician for examination..Take care"
},
{
"id": 12989,
"tgt": "What causes bumps on calves and arm?",
"src": "Patient: My 6 yr old has small bumps on her calves that are itchy. They are not in any particular shape,however almost on same portion of calf and inner calf, on both legs. They are raised,hard and swollen. A few,on her thigh are red w swollen rings around them. A few more are now on upper arms;much more tinier, Doctor: Hi,It may be lichen urticatus. Consult the dermatologist for the perfect diagnosis and proper treatment. I would recommend to apply calamine lotion and to give antihistaminics like Hydroxyzine HCL syrup to reduce itching. Protect him from insects.Hope this helps.Dr. Ilyas Patel,Dermatologist"
},
{
"id": 86394,
"tgt": "Suggest treatment for severe upper abdominal pain",
"src": "Patient: Hello Doctor, Good Morning. I have occasional pain in the right side upper abdomen. I get the ultrasound test and then found that there is partial PUJ obstruction in the right kidney. I also get the DTPA check up for this and found that kidney is working fine. The ultra sound report says that there is swelling in the right kidney and PUJ obstruction. Please suggest , what further treatment I should go for get rid of this problem. Thanks, Gautam Doctor: Thanks for consulting. I have carefully worked through your case, and I can realize your health worries. Being your physician, I assure you not to worry as I will take care all of your medical concerns.for the pain you can take analgesic like tablet tramadol twice a day and anti spasmodic like tablet buscopan twice a day.since the obstruction is due to a mass. further evaluation has to be done about the swelling. consult a nephrologist they may advice you regarding he treatment based on the scan and biopsy report if we can cure by medicine or chemotherapy or may need surgery to remove.Hope it helps. If you need further, detailed and quick assistance related to any health issues in future, feel free to 'ask me a question' directly from my profile.Have a wonderful time ahead. Best Regards!Dr. Arunmozhi varman"
},
{
"id": 27861,
"tgt": "What causes severe chest pain and abnormal blood pressure?",
"src": "Patient: I have serve pain in the left part of my chest, ringing of my heart in my ears and bloodpresure of 164 over 45. I don t know if I should go to E.R. in our local hospital or wait and see my family doctor tomorrow. Would appreciate and suggestions. Marilyn A Doctor: hello Marilyn, I have gone through your question.I would like to know from you whether you got chest pain for first time? or you had similar pain in past and it was evaluated?your blood pressure 0f 164/45 is abnormal especially difference between upper and lower pressure is significant and it may point towards leaking aortic valve.However final opinion can be given only after your clinical examination and investigations like ECG and echo.so better you go to ER and get yourself evaluated early,My best wishesDr.Teli,MD"
},
{
"id": 65013,
"tgt": "Suggest remedy for lump on the back with puss",
"src": "Patient: i had a small lump on my back/shoulder that i assumed was a kind of spot, but which after a few months i squeezed and thick white 'paste like' stuff came out, lots of it. but the thing is, this stuff smells awful! and every now and then it gets a bit more inside that i have to squeeze out. what is this? it doesnt hurt, ever. any ideas? Doctor: Hi,Dear thanks for your query.this is a chronic folliculitis on the back with chronic -recurrent absess,due to improper drainage.In my opinion you need proper drainage and scraping of the abscess wall, for it to heal permanently.This is the remedy for this chronic follicular abscess on the back of shoulder.Thnks again.Hope you got the answer.Wellcome again."
},
{
"id": 32157,
"tgt": "What causes a swollen open wound after bitten by a insect?",
"src": "Patient: hi i got a bite the other day when i was at a beach. i was sitting in grass but i was on a towel. at first it looked like a mosquito bite but it didn't itch. then each day it got bigger and bigger and looked like an open wound. it looks like a scab in the middle and my dad looked and thought it could be a tick bite but didn't see anything in there. Doctor: Hi..Welcome to HEALTHCARE MAGIC..I have gone through your query and can understand your concerns..As per your complain it seems that you have got infection at the site of insect or bug bite leading to inflammation and abscess formation..You need not to worry and clean the wound with antiseptic solution and apply a triple antibiotic over it and take a short course of antibiotics like Ciprofloxacin oral..You can also take diclofenac or tylenol for pain relief..If there is itching present you can take anti-allergic medication like Fexofenadine [allegra] or benadryl..Hope this information helps..Thanks and regards.Dr. Honey Nandwani Arora."
},
{
"id": 105365,
"tgt": "Developing flu leading to tonsillitis after smelling perfume, dust. How can it be cured?",
"src": "Patient: everytime i wear a perfume or pass someone wearing a perfume i develop flu that leads to tonsilitis always. even if i just start sneezing it ends up in tonsilitis, what is wrong with me? if i sense/ smell dust i catch a flu. two days ago i started sneezing, my nostrils are blocked and i am having a slight pain in my throat when swallowing . please help me. Doctor: THIS IS CASE OF ALLERGIC SINUSITIS WITH POST NASAL DRIP WHICH CAUSES TONSIL PROBLEM REQUIRED XRAY PNS TRATMENT TAB METROGYL 200 MGM TWICE A DAY TAB MONTAIOR FX TWICE A DAY SUP PIRITON 1/2 TSF WITH WARM WATER AT NIGHT TAB ZINETAC 150 MGM EMPTY STMACH FOR FEVER TAKE VOVERAN D TWICE A DAY OR SOS TREAT FOR 7 DAYS GET XRAY POST AGAIN AND DISCUSS"
},
{
"id": 216337,
"tgt": "Suggest cause for persistent fever with metallic taste and muscle pain",
"src": "Patient: Good day team. I appreciate to b part of this health forum, my complaint goes thus; since october 2013, I have been having almost a permanent fever feeling with symptoms listed below: head discomfort far from headache,altered taste-metallic taste, sour taste, very dry tase at different intervals, very heavy weakness and feeling very light especially on the legs, internal hotness, joints pain, muscle pain, bitter mouth, slight dizziness, lower back pain, shoulder pain on the right arm, and lots more whch can not b described. I ve been living with all od this as part of my evryday life buh very disturbing as I spend most of mydays at home. If I must say It is becoming a life threating experience as I have.not been.able to get a proper diagnosis to acertain what could be the cause of permanent general fever in my life. I wiill appreciate if I caan get favourable feedbacks, suggestions to help me deal with it. I have bn to very many physicians and proffesional and all of them seem clueless. I rilly nid help. Doctor: perment fever is not symptom of any disease..if u do all investigation n its normal..plz don't worry..how is ur body temp??..plz check it with thermometer.."
},
{
"id": 161578,
"tgt": "What causes cohesive, sticky and smelly stool in a child?",
"src": "Patient: hi my baby is 1.5 months. gas forms while passing stool. stool is too smelly and very cohesive, sticky & very smooth. is it normal??? does stool depends on baby s body tempreature? normally how much temperature is necessary to be maintained for baby s body?? does this temperature has any effect on stool?? Doctor: Hello, Possibly he is having a food intolerance possible milk intolerance. The body temperature in a child is normal below 37\u00b0C.He can avoid eating too many fruits and vegetables, brown bread and fiber-rich foods. Hope I have answered your query. Let me know if I can assist you further. Regards, Dr. Olgeta Xhufka, General & Family Physician"
},
{
"id": 148921,
"tgt": "Have MS, epilepsy and frequent vomiting. Feeling dizzy and indigestion. Suggest the cure",
"src": "Patient: I am a 53 year old female with MS and epilepsy. For the last 2 years, I have been having episodes of frequent vomiting. These have been averaging about one to two or more times per month. Sometime I am laid out for a day up to four and have no appetite. I have indigestion, my skin with the vomiting turns bright red and I am dizzy and sometimes confused. I experience weight loss and have had several blood tests and they all come back negative. I'm getting extremely frustrated that no one can figure this out. Doctor: HIThank for asking to HCMJust do not worry every thing will be okay ! this is not the indigestion but could be acidic problem and will come around soon with the Omeprazole 40 mg and the domperidom start this tab, just one tab at evening time at least for six week, one more thing which is very important and that is to keep your anxiety level very low no stress at all, try to sleep well eat more alkali food no smoking to alcohol no coffee before bed time, you will definitely going to have a good result have nice day."
},
{
"id": 124735,
"tgt": "Is morphea with atrophy related to muscle cramps?",
"src": "Patient: I have had severe muscle cramps in my left neck, shoulder and shoulder blade since 8/4/12. Nothing relieves the pain. I also have morphea with atrophy on the same side both upper and lower. Is this just related to that. I have an appt with a rheumatologist , but not til 10/4. Earliest available. Doctor: Hello, Consult a neurologist and get evaluated. We have to rule out possible causes like neuromuscular causes. Hope I have answered your query. Let me know if I can assist you further. Regards, Dr. Shinas Hussain, General & Family Physician"
},
{
"id": 123107,
"tgt": "What causes a red spot in bruise after spraining a wrist?",
"src": "Patient: Hi, so I got a bruise on my wist from hitting something a few days ago. Yesterday I spraigned the same wrist. I have been wearing a brace and I just took it off and on the bruise there s now a bright red spot in the middle. Could this be a blood clot? Doctor: Hi, The answer is yes, in all probabilities. I suggest orthopedic consultation with X-rays ASAP. Hope I have answered your query. Let me know if I can assist you further. Regards, Dr. Gopal Goel, Orthopaedic Surgeon"
},
{
"id": 91508,
"tgt": "Why am I experiencing constant stomach pain for a long period?",
"src": "Patient: greetings! I have had stomach pain since about 11.00am today, felt my tummy wasnt that good after eating a delicous asian. dinner last night. I have had this feeling in the last six months or so, I cant even drink without feeling soon after tummy pain. I had a dried date a few hours ago and sure enough not long after, tummy pain, drinking and then tummy pain. Have decided not to have dinner tonight cause my tummy isn t that good. If theses symptoms are like my last, this will now go on for a few days. Mmmm...thought I may have been over it, or maybe it is last nights meal, but I don t think so. Regards Vicki Doctor: Hello dear,The symptoms as mentioned in your post can be attributed to Acid Peptic disease.Sensation of pain and discomfort after eating food is characteristic.An Ultrasound scan of Abdomen & upper GI endoscopy will be helpful in clinching the diagnosis.Symptomatic relief can be obtained with intake of Pantoprazole preparation (to be taken twice a day before meals) & antacid gel (to be taken after meals).They prevent formation of excess acid in the stomach & provide symptomatic relief.And it is also needed that you take certain precautions regarding your diet & lifestyle, such as:1. Take soft, plain, non-spicy foods.2. Drink lots of water to maintain adequate hydration.3. Avoid fast foods & gas producing food products like cabbage, etc.4. Avoid smoking & alcohol.5. Keep away stress, think positive.If symptoms still persist, kindly consult a Gastroenterologist for proper clinical examination.There is no need to worry, you will be fine.Wishing a good health.Take care."
},
{
"id": 131089,
"tgt": "Any suggestion for feeling shoulder separation?",
"src": "Patient: I am a nurse. Twelve weeks ago, I was moving a 485 pound woman and after the episode I felt like my shoulder had separated. I had to strap the arm to my body with a belt because it felt so unstable. By the second day I could not lift it. Any subtle movement, I would feel it. MRI showed fluid in the capsule, no tear. MRI arthrogram didn t show a slap tear, although every time my ortho guy does the tests in his office, they come up positive. This is workers comp case, so they can t do surgery unless definitive objective evidence. Physical therapy has made things worse. I have not gotten any better. Today I am back to strapping my arm down because I am sick of the feeling of looseness . I m seeing the top shoulder guy in the area. Got any ideas? Is it still a slap tear just not showing up? Doctor: if it is painless then you should look for lumbs on your shoulder blade , if it is extremely painfull ,then MRI can possibly not show it try putting your arm in abduction for 24 hours , if its nothing problem should go away if you keep your arm rested in abduction Good Luck"
},
{
"id": 220849,
"tgt": "What are the symptoms of potential pregnancy?",
"src": "Patient: Hi I have this problem, in January I had sex with my ex boyfriend and ever since then I havent had my period.. i have a feeling that I'm pregnant but I dont want to be and I dont want to tell my family because I already sent them through this problem..Im 18 and I can barely take care of myself...my stomach hasnt grown I havent had any symptoms but I thought maybe I had a miscarriage but I didnt have any heavy bleeding please help me Doctor: Hello and I hope I can help you today. I know it is difficult, but you need to have a pregnancy test to find out for sure if you are pregnant or not. The stress of worrying about pregnancy and other hormonal abnormalities can cause you to miss your period and pregnancy symptoms are unreliable. You can buy a urine pregnancy test over the counter and follow the directions if it has been at least three weeks from your unprotected encounter.The earlier you find out if you're pregnant the more options you have in terms of what to do next. So I encourage you to just do a pregnancy test as long as your period is late, and if it is negative, it can also ease your piece of mind about the other issues in your life. I hope I was able to adequately answer your question today and that my advice was helpful. Best wishes, Dr. Brown"
},
{
"id": 204444,
"tgt": "What causes panic attacks in the morning?",
"src": "Patient: Twice now and five months apart, I experienced these symptoms: awakened in early morning with a rush of euphoria lasting about ten seconds, immediately followed by about ten seconds of panic like I am going to die. Then, I experienced the same thing about three more times throughout the day (each time). There are no physical symptoms. I take no medication and have never used recreational drugs and I don\u2019t drink alcohol. What do you think would cause this. The euphoric feeling is great but the panic makes me afraid to think I could have another episode. Doctor: Hello and Welcome to \u2018Ask A Doctor\u2019 service. I have reviewed your query and here is my advice. Please explain your panic symptoms because you say that there are no physical symptoms. Panic attacks are always due to underlying anxiety problems or stress response. I suggest you to provide all the details so as I can help you. Hope I have answered your query. Let me know if I can assist you further. Regards, Dr. K. V. Anand"
},
{
"id": 138861,
"tgt": "What causes weakness and twitching in arms and legs?",
"src": "Patient: Hi I have been having the feeling of weakness in my left hand. I also been experiencing fasciculation in my arms, legs, hands, feet, eye lids, and lips. My joints crack a lot and I am nineteen. I have a history of anxiety so I am fearing that I have ALS. Is there anyway you could put my mind at ease? -Mack Doctor: Hi. The symptoms you are having could stem from anxiety also.But before that we should try vit supplements and see if that works.Get bit B12 level done.along with it get your serum calcium and vit D 3 levels.Hope that helps"
},
{
"id": 84575,
"tgt": "What is the purpose of gestofit tablet?",
"src": "Patient: I m 31 year old married for last 3 yrs,me and my husband trying for a baby from last one and half months, i m suffering from pcod.except this my and my husbands all reports are normal.doctor prescribed me gestofit 200 mg.what purpose/and what is the use of gestofit tablet.pl.tell me pl pl pl Doctor: Hi, It is used to support a healthy pregnancy. Gestofit tablet contains natural micronised progesterone and is commonly prescribed to support and maintain healthy pregnancy and prevent possible miscarriage in early weeks. Continue taking it as prescribed by your doctor.Hope I have answered your query. Let me know if I can assist you further. Regards, Dr. Mohammed Taher Ali, General & Family Physician"
},
{
"id": 26364,
"tgt": "What causes rapid heart beats and dizziness?",
"src": "Patient: My 14 year old son has been having a rapid heart beat off and onFor the last couple days--he often gets dizzy when standingup..he seems concerned and I can only go by what he says andHe knows his body..should I be concerned and take him to the hospital? Doctor: Hi,Sometimes there are blood pressure fluctuations in his age. They can cause dizziness while standing. They are not dangerous and usually disappear with age.But if he has palpitations, I would advise you to check his pulse, if it is over 100 bpm, or not regular, I would advise you to see your doctor and have an ECG performed.Wishing a good health to your sonIn case of further questions don't hesitate to askRegards,"
},
{
"id": 182275,
"tgt": "Suggest treatment for lumps behind the molars",
"src": "Patient: When i was younger i had a sort of bump behind my back molars that i could pull around with my tongue, it sat on the gum and was like a hinged little bit of flesh that fitted into a recess in the gum. it used to heal over then reappear , then one day just never back. what exactly is this? Doctor: Thanks for your query, I have gone through your query.The lumps behind the molars can be a inflamed pericoronal flap over a partially erupted tooth. If the tooth is covered only with the soft tissue lump then you can get aperculectomy done (surgical removal of the flap). If the tooth is covered with bone and soft tissue flap, then the tooth has to be removed.I hope my answer will help you, take care."
},
{
"id": 180534,
"tgt": "Suggest treatment for a sore under the tongue",
"src": "Patient: recently my brother has sore type under his tongue after preliminary investigation doctor's conclusion was that is due to bacterial infection and he started antibiotics for weeks it totally disappeared .After fortnight it again appeared what might be the reason and what is the treatment to be taken . Doctor: Hello and Welcome to 'Ask A Doctor' service. I have reviewed your query and here is my advice. A sore under tongue can be a canker sore or yes, it can be bacterial infection as it went away completely after treatment with antibiotics..There is a possibility that the infection was not completely resolved and it is the cause of the recurrence.So it is better to take another course of antibiotics.Along with it do warm saline gargles, antiseptic mouthwash gargles and take Vitamin B complex for a week.Hope I have answered your query. Let me know if I can assist you further."
},
{
"id": 196285,
"tgt": "Suggest treatment for premature ejaculation and itching of penis",
"src": "Patient: I am suffering of ABNORMAL DISCHARGE in the course of my sleep accompanied by itching inside of my pines. I have done a number of tests including urination and sexual special visits but the resulf was still negative. Mainwhile, secretion can be seen straightaway of urination or togather at the first some times. So, the consequencies of the aformentioned matters are weakness in the sexual organs, back chronic pines, and over all abnormal situations. So, I much appreciate your vaulable advices in getting off of this situation. thanks a wrold. Doctor: Hi and welcome to Healthcaremagic. Thank you for your query. I am Dr. Rommstein, I understand your concerns and I will try to help you as much as I can.Premature ejaculation is uncontrolled ejaculation either before or shortly after sexual penetration. It happens with minimal sexual stimulation and before the person wishes. Most cases of premature ejaculation do not have a clear cause. With sexual experience and age, men often learn to delay orgasm. Premature ejaculation may occur with a new partner. It may happen only in certain sexual situations or if it has been a long time since the last ejaculation. Psychological factors such as anxiety, guilt, or depression can also cause it. In some cases, it may be related to a medical cause such as hormonal problems, injury, or a side effect of certain medicines. In many cases premature ejaculation gets better on its own over time. Treatment may not be needed. Practicing relaxation techniques or using distraction methods may help you delay ejaculation. For some men, stopping or cutting down on the use of alcohol, tobacco, or illegal drugs may improve how well they can control ejaculation.I hope I have answered you query. If you have any further questions you can contact us in every time."
},
{
"id": 130127,
"tgt": "Reason for swelling in the feet?",
"src": "Patient: My cousin, who is 37 years old, had a baby by C-section on 4/23/14. She had some high blood pressure 2 days after delivery, and was put on BP medicine for 2 weeks. Now she is at home and says both her feet are really swollen. They were slightly swollen when she left the hospital, but she says they are much worse today. She said she feels fine, has a good appetite, and no shortness of breath. She is breastfeeding. Do you have any idea why her feet are so swollen? Doctor: Hi i am Dr Ahmed Aly thanks for using HealthcareMagic site ,I had gone through your question and understand your concerns .. In my opinion i think she may have a kind of vascular insufficiency due to low venous return . this could be due to standing for a long time ,sitting , bending knees , bad sleeping postures . She may use ant inflammatory tabs , elevate her feet above a pillow while relaxation , wear compression stockings , hot massages with topical gels , 1 daily aspirin pill after breakfast will be helpful for most of cases . If swelling increases or persists i suggest Doppler U/S to rule any vascular injury and visit her physician for proper evaluation and management . Please click and consider a 5 star rating with some positive feedback if the information was helpful. Wish you good health,Any further clarifications feel free to ask."
},
{
"id": 133731,
"tgt": "Suggest remedy for pain and swelling in the knee",
"src": "Patient: hello I m concern about my knee two days ago my family and took a drive and on the way I started peeling off the scabb I have a bad habbit taking off the scabbs well the next morning woke up and had pain on my knee later on the day it got swollen and hurting more I put ice packs on it and today its the secobd day and its still the same its hot and swollen can you tell me whats going on and if I need antibiotics Doctor: hi,thank-you for providing the brief history of you.As you had a scab and you tried peeling it off which was not the need, as it is the normal healing process and it heals on its own and falls away.Since, now the symptoms are hot skin and swelling, continue performing icing and do gentle massage near the area to improve the healing process. Also, kindly avoid any peeling of the scabs in future to avoid infection due to nail injury.RegardsJay Indravadan Patel"
},
{
"id": 123839,
"tgt": "How is facet joint disease cured?",
"src": "Patient: Hi ...i have facet joint desease but over last few monts ive been gettin a numbness in both legs on and off over last few months...they feel numb at the top of my legs but are so painfull feels like my bones have been crushedthe lower half...do u think i should be concerned or is the part of my problems with my facet joint problem Doctor: Hello, As I see the above-mentioned history I feel it is related to lumbar spine intervertebral disc injury and so does the symptoms of numbness, tingling and radiating pain has come. I will advise for an MRI scan of the lumbar spine so we can understand the soft tissue pathology better. With core stability, spinal muscle strengthening exercises and lower limb strengthening exercises you pain should come down. Use hot water fermentation which will help reduce the spasm of the lumbar spine muscles. Hope I have answered your query. Let me know if I can assist you further. Take care Regards, Jay Indravadan Patel, Physical Therapist or Physiotherapist"
},
{
"id": 136187,
"tgt": "What causes bruise and numbness on knee?",
"src": "Patient: I had a minor motorcycle wreck and the weight of the bike landed on my knee. It s bruised and sometimes feels crunchy when I move it- as though a tendon is shifting. Also it almost feels numb on the surface. Symptoms have persisted for five days after the wreck without getting much better. Any thoughts? The trauma caused no twisting/straining- only impact. Doctor: Hello, I have studied your case.As per your age and history there is possibility of soft tissue injury in knee.Most commonly meniscus injury occurs. As per your symptoms you need to do physiotherapy and knee exercises to improve range of motion .If pain persist then you may need to do MRI again to see for any functional defect.Small meniscus tear can be managed conservatively with physiotherapy with USG and TENS.Meniscus injury can be managed by arthroscopic repair,and same day you can go home.Hope this answers your query. If you have additional questions or follow up queries then please do not hesitate in writing to us. I will be happy to answer your queries. Wishing you good health.Take care"
},
{
"id": 54201,
"tgt": "What does fatty liver and scattered diverticulosis mean?",
"src": "Patient: What is fatty liver? What does scattered small pelvic lymph nodes adjecent to the obturator regions, iliac vessels, inguinal regions etc. What is calculi or hydreonephrosis. What is scattered diverticulosis? And bladder wall thickening? Doctor: HiThank you for contacting healthcare magic.1) Fatty liver is fatty deposition inside the liver. It is caused by obesity. No management needed except lifestyle modifications like brisk walk daily for 30mins for 5 days a week, avoid excess sugar & oily foods, reduce weight, quit alcohol.2)Calculi & hydronephrosis - calculi are stones. Sometimes stones in the ureters (tubes that connect kidney with the urinary bladder) can block the urine flow from the kidneys leading to its accumulation and a swollen area of fluid around the kidney (hydronephrosis). Once stone is removed, obstruction will be gone.3)Diverticulum is a pouch that comes through the intestinal wall. It can have infection & present a with abdominal pain.4)Lymph nodes are small collections of immune cells that fight against infections. Once infected or diseased, they become enlarged. Treatment includes treating the underlying infection or cause.I have tried to explain as simple as I could.I hope I was able to make you understand.Take care"
},
{
"id": 136515,
"tgt": "Suggest treatment for low grade fever and severe joint pain",
"src": "Patient: About 6 months ago I had an increase in joint pain primarily in my knees, beyond any pain that I had before, I became lethargic and could barely move at times. I have improved some and now my 26 year old son that lives with me is having some of the same symptoms. Could this be a joint virus of some sort? I have had a low grade fever on an off over the last month. Doctor: HelloI have read your query. There is possibility of polyarthritis leading to pain. I will advise you clinical examination and further investigation like CBC and Esr and Crp and RA test. Another possibility of viral infection leading to fever and joint pain. I hope I have answered your questions. If you have further questions please feel free to contact us. I will be happy to answer. Take care."
},
{
"id": 197824,
"tgt": "Can masturbation cause insomnia, anxiety, muscle soreness an palpitation?",
"src": "Patient: I have been masturbating since about the age of 14, I am now 24. For the last couple of years, I have felt various negative symptoms but hadn't started to relate them to masturbation until this past year or so. These include but aren't necessarily limited to: constant fatigue, insomnia, hair loss, muscle soreness, ringing in the ears, lightheadedness upon standing, stress, pretty severe anxiety, frequent panic disorder / attacks, heart palpitations, irritability, etc. I used to masturbate daily and did so for years, it seems. And, even though it was just a handful of rare times, I remember doing it two times per day on a couple occasions (less than 10-15, I'm sure). The past couple of months I have been trying to limit the frequency of this. I have done it 3-4 times per week. I even went about a month without ejaculating at one point recently but did still somehow always manage to stimulate myself so I didn't feel all that better. I have ejaculated about two times so far this new year and am now on day 4 of no masturbation. I'd like to know if you believe my symptoms are due to this and what I can do to really start on the road to recovery. In February, I went to a local emergency department for heart palpitations. I had tests done: EKG, blood, urine, x-ray, etc. All came back normal gratefully.Thank you so much in advance for reading my inquiry. I look forward to your knowledgable response very soon.Bradley Doctor: Hi,Thanks for writing in.1. Masturbation is a safe sexual behavior when done in limit. The average man masturbates 2 to 3 times in a week. It should not be done more than once daily.2. Like any sexual activity some people might get addicted to masturbation. This leads to added stress and anxiety causing palpitation. Forceful masturbation can cause muscle soreness.3. It is good if you realize that come of the symptoms like anxiety, fatigue and insomnia can happen from too much masturbation and you are taking measures to control it.4. Please continue for a week or longer without masturbation. If you feel uncomfortable then you can masturbate once a week to release stress.5. When you achieve control over your masturbation behavior then you can do it 2 to 3 times in a week.6. Please meet your friends, do study and keep your mind in work to avoid sexual thoughts.7. Try and avoid pornography and when you get sexual thoughts then read a book and listen to music or go for walks. Do not touch your penis except during urination. Please do not worry."
},
{
"id": 115132,
"tgt": "How will the large clot in the leg break down?",
"src": "Patient: After a vaginal hysterectomy the patient developed a severe blood clot in the leg from ankle to hip and clots in the lungs. She was then given Coumadin and Fragmin. Before release from the hospital her lungs were clear but the large clot remained. After 2 weeks on Fragmin the Coumadin Clinic advised that the INR was within the safe range the and Fragmin could be discontinued. How will the large clot in the leg break down and be released from the body and is the patient still in danger? She is 41 and has asthsma, but no other recognized defects. Doctor: Hello and welcome to HCM,The drugs prescribed- coumadin and fragmin act by dissolving the clots in body.These drugs are monitored by INR.Thus, these drugs would have acted by dissolving the blood clots.The effectiveness of these drugs is monitored by INR within range.There is nothing to worry.The clots formed as complication of surgery however, the clots will not form once the patient becomes ambulatory or mobile.Thanks and take careDR Shailja Puri"
},
{
"id": 152836,
"tgt": "What causes rib soreness with rectal bleeding and mucus?",
"src": "Patient: hi i have just had a basal cell aminoma removed but that is the least of my probs just thought that ia would mention it I have a sore right bottom rib and below it. I have had blood in my stools and also mucus it seems to happen when i get stressed I am booked in for a colonoscopy just wondering if it is likely to be cancer i am only thirty two Doctor: HelloOf course, colon cancer has to be considered here, but, considering your age, hemorrhoids, polyps, or inflammatory bowel disease are much more likely diagnoses"
},
{
"id": 42844,
"tgt": "Is there any chance of pregnancy after taking infertility medication?",
"src": "Patient: Good afternoon doctor. we are trying for a baby since 8 months. i have mild pcos and my husband less count. i.e. 43millions . doctor suggested him to take fertisure and me glycomet, cabergoline. again we consulted one more doctor who just gave medications for me . i.e folinine and berocin . do we have chances this time? is my husband can improve his count? since its affecting our phy relation? thanks in advance Doctor: Dear member,Thanks for writing to healthcare magic.I am Dr Bhagyashree. Consultant obgyn specialist.I can understand your anxiety as infertility treatment requires lots of patience.If pics is mild it can be treated with weight loss and tab ginette 35.The glycomet has to be taken regularly and a diet has to be followed according to BMI.The husbands count of 43 millions is ok if motility is good.The treatment for you will be ovulation induction. In my clinical practice I have noticed that mild pcos can be treated easily if you have patience and show discipline in diet.Please call back with reports to Dr BhagyashreeThanks Dr Bhagyashree"
},
{
"id": 73666,
"tgt": "What could sharp stabbing lung and chest pain suggest?",
"src": "Patient: HI, the right side of my chest has been hurting all day. Really sharp stabbing pains near my heart and lung. Followed by troubled breathing. I have asthma, and have been using my inhaler all day. The pain still persist. Not sure if I pulled a muscle or something minor. Just concerned. Any advice? Doctor: Thanks for your question on Healthcare Magic.I can understand your concern.Sharp stabbing chest pain is mostly seen with musculoskeletal cause like pulled muscle.So avoid movements causing pain. Apply warm water pad on affected part of chest.Take painkiller and muscle relaxant drugs.You will mostly improve with all these in 2-3 days.Of chest pain still persist then consult doctor and get chest x ray and ecg to rule out lung and heart diseases.Hope I have solved your query. I will be happy to help you further. Wish you good health. Thanks."
},
{
"id": 219298,
"tgt": "What is the best treatment to get pregnant?",
"src": "Patient: Hi, I am 31 year male married 8 months ago and have sex with partner 2 days in a week on an average and never used condoms but still wife not conceived yet? Whenever i have sex the sperm coming out of the penis lies outside the vagina it is not going inside(98% of the times) and have4.5 inches of penis when it is erected and 1.5 inches when it is in normal. Doctor: Hello,I have gone through your query and understood the concern. There are several causes for failure of conception. For the male part, the sperm count, progressive motility and liquefaction are important. For the female part, regular ovulation, proper tubal and uterine function, absence of anti-sperm antibodies are important. Intercourse should be done at least on alternate days around ovulation, i.e., two weeks prior to the onset of the next menstrual cycle. If your wife has a retroverted uterus, lying on the stomach for sometime after the intercourse will help to retain the sperm longer. Alternatively, placing a pillow beneath the hip region may also be tried if there is a spillage of semen. You can get further help from a specialist. Take care."
},
{
"id": 13925,
"tgt": "Suggest remedy for itchy red burning rashes",
"src": "Patient: Hi Dr. Andrew. My name is Matthew and I am suffering from a skin condition sporadically for 2 weeks now. It s an outbreak of resurging itchy bumps all over the body that started with red burning rashes on the underarms and upper chest/neck area after I used a new deodorant. Now it s very itchy and pops up anywhere. I tried rubbing ice and even applied boric powder mixed with a mild talc over the infected regions. The itch would persist but the bumps would go away overnight only to pop up again next morning on newer areas. Will a topical cream do the trick? Doctor: Hello, I feel that you are suffering from contact dermatitis. In that case, you should find the probable cause for the allergy and avoid that. But, according to me, you must visit your Dermatologist to confirm the diagnosis and to initiate proper treatment. Hope I have answered your query. Let me know if I can assist you further."
},
{
"id": 5860,
"tgt": "Trying to concieve. All tests normal. Ultrasound shows bulky uterus. Chances of pregnancy?",
"src": "Patient: hi doctor i hv married for 2yrs and i am nt able to concive. and i hv seen d gyn she told me t o do my test i hv done all d test all my blood reports and thyroid test r normal. i hv done d TVS which says dat my utrus size is normal butm my left oviery is slightly bulky. . . is dat d reason i am nt able to consive and i am wory and tensed. wat to plz help me. wil i never bcm mother ?plz help Doctor: Hello I appreciate your query I hope you are aware of the fertile period ( peri ovulatory period ) during which intercourse leads to maximum chances of conceiving. This is generally Day 14 if you have a 28 day cycle, so day 10 - Day 17 are your fertile days. Apart from that, you need to get your husband's semen analysis done. You should have your routine blood tests, thyroid profile, blood sugar levels, hormonal tests, serum prolactin etc., and an internal check up . TVS is also important, however your report seems to be normal Slightly bulky left ovary does not signify anything, unless your report is suggestive of polycystic ovarian disease, which interferes with ovulation. Inspite of all normal reports, if you fail to conceive, then you should go for some form of tubal testing, either a hysterosalpingogram or a diagnostic hysteroscopy and laparoscopy. If tubal block is present, you need ART ( assisted reproductive technologies ) to help you. Please mention your age. Do not lose hope, as today there are various modalities of treatment to help you further. Please consult an infertility specialist. All the best."
},
{
"id": 173815,
"tgt": "Pancreatitis in child can be cured?",
"src": "Patient: Dr. My Child is 4 years old and she is diagnosed pancreatitis on 18th may 2012. after 2 months she is diagnosed swealling in pancreas again . He r treatment is on from PGI. Please let me know whether this disease is curable ? And if curable please let us know the best possible treatment. Doctor: Yes, it is curable, don't worry ,she will get better. In most cases, pancreatitis gets better on its own. I strongly recommend putting your child on a low-fat diet with plenty of fluids. This is less likely to irritate the pancreas and helps it to heal faster.Kids with a more severe case of pancreatitis who become dehydrated may need a short stay in the hospital. Treatment involves resting the pancreas by not eating or drinking, and receiving intravenous (IV) fluids and pain medication. Other medications may help to reduce the amount of acid in the body. For some kids, this is enough and they can go home after a few days of treatment Your kid should stay on a low-fat diet to help heal the pancreas, which she should follow until their enzyme levels are back to normal. A follow-up blood test will probably be done to confirm normal enzyme levels. For improving digestion you can use papaya, Earth apple, artichoke, Enzyme's syrup,probiotics-Darolac. Kind regards Dr.Svetlana Shrivastva"
},
{
"id": 31767,
"tgt": "Can one contract TB through sexual intercourse?",
"src": "Patient: Hi i am 31 yr old male , i was diagnosed with Miliary TB some time back and i am under medication. my last X ray did not show signs of the disease, but wanted to ask if still TB can spread if i have sexual intercourse with my spouse as i plan to start a family. Doctor: Hi & Welcome.I can understand your concern regarding the mode of transmission of TB infection.Tuberculosis DOES NOT spread through sexual intercourse.Tuberculosis is a disease caused by bacteria that spread through the air droplets from someone with the untreated, active form of disease coughs, speaks, sneezes, spits, laughs or sings. Although tuberculosis is contagious,it generally takes a few weeks of treatment with TB medications before you're not contagious anymore.Following preventive measures are recommended to prevent the spread of infection to house hold contacts (family members):1. Ventilate the room. Tuberculosis germs spread more easily in small closed spaces where air doesn't move. If it's not too cold outdoors, open the windows and use a fan to blow indoor air outside.2. Cover your mouth. Use a tissue to cover your mouth anytime you laugh, sneeze or cough. Put the dirty tissue in a bag, seal it and throw it away.3. Wear a mask. Wearing a surgical mask when you're around other people during the first 3 weeks of treatment may help lessen the risk of transmission.4. And all the family members should be given the medication Isoniazide (INH) daily for 3 months if the Mantoux test shows POSITIVE reaction.Hope the suggestions given above could be useful."
},
{
"id": 188011,
"tgt": "Can severe pain in the tooth on which root canal treatment was done be due to gaseous necrosis?",
"src": "Patient: RCT was done on my left 2nd molar and 3 roots were treated about 4-5 weeks back. Since 2-3 weeks I am getting severe pain in same tooth which is relieved by cold water. I have been advise ipobrufen 600mg TDS with amoxicillin and metronidazole which I took for 6 days without any improvement. I suspect the detist failed to see the possible 4th root of the tooth which is now giving problem. I am suffering from gaseous necrosis in the possible 4th root or what? If the dentist is unable to see the 4th root with naked eye, do dentist use magnifying instrument to see minute roots? A followup xray and OPG was unremarkable. Pls advise what should be done. The pain is unbearble even with ipobrufen. Thank for sending reply to YYYY@YYYY Naqeeb Khalid Doctor: Hello, thank you for consulting with healthcaremagic. If there is pain present in the same tooth after the root canal treatment also, it means that either the RCT is incomplete or it has not been done properly. I think you should visit another dentist to get a complete examination of the tooth done with x - ray, as without x - ray it can not be diagnosed properly. And if needed get it retreated. Hope it will help you."
},
{
"id": 118664,
"tgt": "High WBC, other values normal. Get stressed all time, have tooth problem. Related ?",
"src": "Patient: Hi, I went to the doctor for a complete physical and everything came back great except my white blood cells were high, 13,000. My thyroid, platelets, red blood cells and everything else were completely fine. I never get sick but I do have bad teeth and have had a few root canals and procedures done. I am also extremely emotionally stressed all the time. Could my teeth and stress be causing my elevated WBC count? Doctor: hello increase in wbc count indicates - any kind of infection in body- any reaction to a drug-bone marrow disease-any immune system disorder there are certain diseases in which wbc count gets increased... yes if you are undergoing some kind of infections in your teeth the count can riser but also please consult a general practitioner... hope it helps"
},
{
"id": 105193,
"tgt": "Sinus, chronic cough with mucus, wheezing, no fever. History of dust allergy. Treatment options?",
"src": "Patient: Hi, I ve sinus . I ve been coughing since a month or so. Last year around Jan/Feb 2011, i had the same problem i.e. cough with mucus , wheezing , but no fever, I had X ray and blood test done but the doctor said its dust allergy. This time also I m facing the same problem, cough, mucus, wheeze but no fever. What should i do? Doctor: allergy can be tested by allergy tests you get your allergy tests and you wll find to which item you are allergic to and then you can get permanent cure by immunotherapy you can start anti allergics tab singular 10 mgm bd tab zinetac 150 mgm od syp tossex 1/2 tsf night syp ventrolin 1/2 tsf bd you can take 3 week and you ca take sos till your test esults"
},
{
"id": 91074,
"tgt": "What causes abdominal cramps after taking unwanted 72?",
"src": "Patient: My Wife has taken unwanted 72 on the second day of her period when we had sex. Now its 12 days but she is having regular stomach pain and sometime in her breast and in leg most of the times. Her stomach has swelled and her left side of vegina has also swelled (red). Touching the stomach causes the pain and even at the time of urinating its gives burning feeling. Some time few drops of blood comes out. It seams like bacterial and fungal infection. Should we take any medicine is it ok? Doctor: Hi, thanks for using HCM.Your wife's symptoms are suggestive of urinary tract infection or genital infection. Nor related to unwanted 72 pill intake. Complete history and examination required to confirm condition. Antibiitic treatment required. Good hygienic measures required. Treatment of partner is also essential.Intercourse on 2nd day of cycle is safe, unwanted 72 pill was not required.Regards"
},
{
"id": 161460,
"tgt": "What is the meaning of protein allergy in children?",
"src": "Patient: Hello, I have a newborn 3 weeks old. 6 lbs 4 oz. He keeps spitting up and choking to the point he can t breath within an hour or two of eating. The doctors say he has a protein allergy but I don t think so. We have already had to call an ambulance once before because he wasn t breathing. What do you think is wrong? Doctor: Hi, He could be having GERD or gastroesophageal reflux disorder. While lying flat some milk may be returning back from the stomach to the feeding tube and into the throat. A few drops of this milk can enter the breathing tube and cause choking. The solution is to keep him in an upright position for as long as possible after feeding, and when you keep him down, maintain an angle of 45 degrees, using pillows. Antiemetic medication like Domperidone would also help. Hope I have answered your query. Let me know if I can assist you further. Regards, Dr. Cajetan, Pediatrician"
},
{
"id": 170816,
"tgt": "What causes dry skin and patches of blotches on legs of a 3 month baby?",
"src": "Patient: My grandaughter has dey skin and patches of blotches of whitish discoloration especially on her legs and patches of hair and her scalp is dry. We especially started noticing this after taking her to the beach. She is only 3 months old but it is becoming more noticeable. Both parents have a history of eczema. Not sure what to do about it??? Doctor: Hi... I sympathize with the kid and understand your concern. By what you quote I feel that it could be eczema. The trick of the management is keeping the skin moist and never allowing it to be dry. For this oil massage and lotions like Oilatum will help. Regarding cure options - eczema is a sort of allergy...allergies can only be controlled and can never be cured. But usually kids grow out of it...that is as the age progresses, they become better starting from 6 months.Hope my answer was helpful for you. I am happy to help any time. Further clarifications and consultations on Health care magic are welcome. If you do not have any clarifications, you can close the discussion and rate the answer. Wish your kid good health.Dr. Sumanth MBBS., DCH., DNB (Paed).,"
},
{
"id": 225901,
"tgt": "Brownish discharge after using expired implanon on arm. Reason?",
"src": "Patient: hiya .. i have an impanon in my arm which expired on the 12th of feb and now i am experiencing brown blood/discharge which has lasted for 6 days now .. it only leaves a bit in my pants but is mainly when i wipe. i have been having unprotected sex with my boyfriend but when i did a test 2 days ago it came back negative .. is this just a symptom of my implanon? Doctor: Hi,Thank you for posting your question here, I will try to answer it to the best of my abilities.Brown discharge on its own is nothing to worry about, it is a common thing which happens to a lot of women after intercourse. Another common reason for brownish discharge is that your endometrial lining didnt clear fully in your last period thats why the left over lining from your last period, is flowing out now. This reason is only valid if you are having your period right now.Anyway as far as I can tell there is nothing to worry about, unless you are feeling any other symptoms.I hope this answered your question."
},
{
"id": 41320,
"tgt": "How is conception possible with PCOS and irregular periods?",
"src": "Patient: Hello Dr.Jain, How are you,i would wish to know the solution for the problems which am undergoing that its going to be 2 years since have got married am planning for a family but am unable to conceive because of irregular periods and polycyctic ovaries due to which follicles are not growing.i tried Homeopathy and other english medicines to make the follicles grow but in vain.And the doctor where am consulting says that i need to do laproscopy but am really scared of that.More over am working and 26 years old i feel not to undergo such treatment as am very scared for which it causes too much of pain.So i would like to kn by any chance will i get conceived soon without such treatments. Please advise.That will be a great help. Doctor: Hi, I have gone through your question and understand your concerns. Polycystic ovaries and irregular periods is suggestive of the fact that you are having polycystic ovarian disease. Conception is difficult in PCOS as there is irregular or no ovulation, hence no conception. The line of management in PCOS is stepwise, the next step to be done when the previous step fails:1. Ovulation induction with drugs2. Intrauterine insemination3. In vitro fertilizationThere is no role of laparoscopy in the treatment if PCOS is confirmed and bilateral fallopian tubes are patent.I would suggest you and your partner to consult an infertility specialist and get properly examined, investigated and treated accordingly.Hope you found the answer helpful.Wishing you good health."
},
{
"id": 196054,
"tgt": "Suggest diet for penis health",
"src": "Patient: dear sir i am 21 year old my height is 5.9 feet my weight is 82 kg , my penis size is 5.9 inch and it is not straight it is downward and it is skin is also not good if you see you will think it has extra meat, and it is not strong. what should i do i not rich so if you tell me natural things to eat , let me tell you that i eat onion too much. i am waiting for your response Doctor: hi dear.welcome to healthcare magic.i have analysed your query completely.i understand your problem.my opinion is that,1.avoid stress.2.avoid cigarette smoking and alcohol intake.3.take nutritious foods and avoid junk foods.4.you should sleep for 8 hours in a day.5.do yoga and exercise.6.avoid masturbation more than twice in a week.these steps will help you to make your penis healthy.thank you.hope i have answered your question."
},
{
"id": 83459,
"tgt": "Does corticosteroid injections causes headache?",
"src": "Patient: Yesterday morning I had a corticosteroid injection in my right knee for a patellar tendinitis. No reactions yesterday except a slight headache. Today however, I ve been experiencing palpitations and slight headache. I took my blood pressure and it was completely normal at 120/68 and my pulse was 68. The palps began after exercising for 1/2 hour on my treadmill this afternoon at 3pm and haven t completely subsided yet, it s now 9:30. Will these symptoms disappear eventually? Doctor: Hello, Yes corticosteroids injection causes headache. side effects of steroids very much depend on the dose and how long they are taken. If your dose is low, your risk of serious side effect is quite small. Hope I have answered your query. Let me know if I can assist you further. Take care Regards, Dr. Arup Majumdar"
},
{
"id": 167858,
"tgt": "What causes cough in a child after tonsillectomy?",
"src": "Patient: My three year old son had a tonsillectomy last wednesday and he has had a fever of 101 to 102 since and he has had to go to the hospital for fluids twice now and still is not drinking very much and today has a cough should I call the doctor again today? Doctor: your child is unable to drink and dehydrated because of pain after the tonsillectomy, which is quite common .but fever indicated an infection , a possible complication of any surgery or a concomitant Upper respiratory tract infection , which is usually vial and this explains the cough , anyway , your child must be carefully examined by his doctor to exclude any complications of the surgery .I hope this helps with the answer"
},
{
"id": 99608,
"tgt": "Can Zyrtec be taken for allergies despite other medication?",
"src": "Patient: My mom is having a very hard time with allergy but is afraid to take anything because she takes so many other medicines. She takes Metoprol Tartrate (50 mg.) Hydroxyurea (500 mg.) Folic Acid (1 mg.) and an aspirin. Can she take a Zertec or Zertec D? My email is YYYY@YYYY Doctor: Hello,Thank you for asking at HCM.I went through your mother's history and would like to make suggestions for her as follows:1. There are so many types of allergies so I would like to know more about your mother's symptoms as well as details of the other medications as well as medical disorders. This will help me to make more specific suggestions.2. As you have asked about Zyrtec D (use for allergic rhinitis - nose allergies), I assume your mother has nose allergies - nose obstruction, running nose, sneezing etc. 3. Zyrtec (Cetirizine) usually does not interact with the mentioned medications. However, Zyrtec D contains a decongestant medicine which may not be desirable for heart patients. As your mother taking Metoprolol and aspirin, I assume she might be having some heart-related problem.So, Zyrtec would be safer for her rather than Zyrtec D.4. If your mother has kidney or liver problem, then Zyrtec can cause more side-effects. So in that case, necessity as well as dose of Zyrtec should be discussed with your doctor before taking it.5. Another option is to use local nose sprays. Intranasal antihistamine (Azelastine) as well as intranasal corticosteroid (Fluticasone propionate or mometasone) are available in market. They act more locally, on nose membrane. As very small portion of these spray enter blood, they usually do not interact with other medications. They have less generalized side effects also.6. If your mother has severe nose congestion, please get her nose examined by an ENT surgeon as long term aspirin use can cause nose polyps. Also, metoprolol can cause nose congestion in some patients. Please discuss this possibilities with your mother's physician who will review her history and complaints in detail and then decide accordingly.7. Please do not reduce/stop metoprolol and aspirin by yourself as it could be dangerous.8. I would also suggest your mother allergy testing for common air-borne allergens such as house dust mites, pollens, molds, insect proteins, animal dander, etc. This will help her to identify the substances causing allergies to her as well as to know the measures to avoid them.9. In general, I would suggest her to avoid exposure to dusts, smokes and air pollution as much as possible.10. Regular breathing exercises and steam inhalation would also help her in a long run for nose symptoms.Hope above suggestions will be helpful to your mother.Should you have any further query, please feel free to ask at HCM.Wish your mother the best of the health ahead.Thank you & Regards."
},
{
"id": 222494,
"tgt": "What causes brown coloured discharge during pregnancy?",
"src": "Patient: Hello, I m 8 weeks pregnant and just went to the restroom to urinate and saw that I had a brown color after I wiped. I had my first doctor appointment last week and saw the heart beating in the sono. He said everything looked good. What could this mean? Doctor: HI, I understand your concern. Brown colored discharge in 8 weeks pregnancy can be due to various causes like- fall/ trauma/exertion /infection . BUt most common & bother sum cause is deficiency of .. Hormone progesteron which is essential for maintenance of pregnancy Hormone replacement in time ,recovers the condition & pregnancy can be saved..Please don't worry. You should contact your gynecologist immediately, get examined & start with treatment soon. Till then take rest/ avoid physical or psychological stress.IN case you can not see youe doctor immediately, you can add Tablet Duphaston 10 mg 12 hourly.. & seek doctor's advise as early as possible. Thanks."
},
{
"id": 131143,
"tgt": "Is it suggestible to use blood thinners for reattached left hand?",
"src": "Patient: Hi Doctor, my husband was 51 years old. In 2007 he had an unfortunate accident where his left hand was cut off. The doctor was able to reattach it but he could no longer use it. He developed high blood pressure. When he and I got together in 2010 his blood pressure was ok but he was still on medication for it. He had about 11 surgeries on that hand. He had a muscle taken from his right thigh and put on his left hand. My question is should he had been on blood thinners? Doctor: breiflyNo i see no reason to use blood thinners with amputation , reconstruction surgerys and high blood pressure"
},
{
"id": 103832,
"tgt": "Baby having loose motions. Prescribed Z&D suspension and enterogermina. Showed pus in report. Refuses feed. Medicines?",
"src": "Patient: My 8 month old baby is passing loos motions (green color with mucus) since last 5 days. On very next day I have visited doc who suggested Z&D suspension (.05 ml once a day), entrrogermina ( 5 ml once a day). But on 3rd day, my baby had fever so that doc gave oflox 50 syrup (4 ml twice a day) and suggested stool routine test. yesterday the report showed pus cell 10-20 and doctor suggested the same treatment should be continued. Now, she is passing yellowish green 2-3 times a day but there is some reaction on her whole body skin. Its like some allergy on whole body skin. She is refusing feeds other than my breast feed and also not taking water. Is it a side effect of medicines? Doctor: Welcome to HCM! What\u2019s your baby's weight? Is he/she on breast feed alone or you have already WEANED the baby at 6 months age i.e. started solid simple food like easy-to-digest preparations of cereals, pulses and vegetables? Please stop all tinned dietary supplements (at times babies are allergic to the preservatives used in them). It\u2019s okay if the baby has been given Z&D or zinc sulphate suspension (aid in healing damaged intestinal villi), Enterogermina (contains Bacillus Clausii, a new class of probiotic to help stimulate the immune system to protect the Gastro-Intestinal tract against infection). However Oflox syrup or Ofloxacin is contraindicated in children due to risk of permanent injury to the musculo-skeletal system. If there are pus cells in stools; a stool culture may be done for bacteriologial detection and appropriate anti-biotic may then be given. In most cases its Rota viral diarrhoea and only the baby's hydration is to be maintained with ORS (Oral Rehydration Solution) and breast feed. Since there is no fever the possibility of a bacterial infection is low; As a foul smelling stool with blood and mucous suggests amoebiasis which is not so in your baby's case. As your baby is 8 months old the possibility of MEASLES can't be ruled out (since vaccination against measles is given at 9 months of age and maternal antibodies protect against measles till 6 months of age); refusing feeds could be because of this viral exanthema (rashes) and treatment has to be supportive by continuing BREAST FEED alone."
},
{
"id": 19650,
"tgt": "What is the treatment for high blood pressure?",
"src": "Patient: good morning iam a woman 52 years of age i have a bit of high blood pressure and i noticed when i started to take a medicine called Disothiazide i started to notice my blood sugar began to go up a bit and in the morning fasting sometimes ll2 sometimes ll4 and today l22 i stopped it a week ago and i started my diet. it goes down after a meal and it will be normal after 4 hours so please doctor tell me what it is iam worried thankyou and god bless you... Doctor: madam ...your sugar was rising as a side effect of thalidone drug or drug you were taking for high BP ...but your BP if it is too high or more than 140/90 it should also be brought down .please visit ur local doctor he may prescribe another medicine for high BP which does not raised sugar .."
},
{
"id": 131750,
"tgt": "How to treat the swelling and bruise on the upper thigh?",
"src": "Patient: I bumped my upper thigh on a desk corner a little over a month ago. I hit it so hard I nearly dropped to the floor but I was able to get up and walk around normally several moments later so I continued my day as usual with very little pain. The next morning I noticed a small bruise and a large area of swelling within my upper thigh. There is what looks like an indented horizontal line at the distal border and swelling above almost to my hip. I was also limping AND had severe pinching sensations. An x-ray was ordered five days post injury and was negative. An MRI was ordered two weeks post injury and was also negative. From week two to three the pinching was replaced with a more tolerable aching and my gait was almost normal but the swelling remained. About three days ago (a month post injury) I started to experience the pinching sensation and I ve been limping again. I should also mention I ve been on my feet quite a lot the entire time (except on weekends) as my job requires me to be. The swelling has not noticeably receded since the day it appeared. What should I do at this point? Doctor: Hi, welcome to HCM, since x-ray and MRI were normal, what you need is local treatment. I would suggest the following. Apply ice fomentation for 10 minutes followed by heat 10 minutes followed by an anti inflammatory containing pain balm (ibuprofen/Diclofenac) Repeat twice a day and give the thigh as much rest as possible. You can strap it if it comforts you. Regards"
},
{
"id": 77084,
"tgt": "Why there is an air bubble in the centre of my chest?",
"src": "Patient: Hi, I have what feels like either a gas bubble or air bubble in the dead center of my chest. I am a 34 yo female, no major health issues myself. However, father passed at 48 of heart attack which was determined to be heart disease. Just recently suffered a miscarriage. Thoughts? Doctor: Thanks for your question on Healthcare Magic. I can I can understand your. Better to first rule out heart diseases for your symptoms. So get done ecg and 2d echo. If both these are normal then no need to worry for heart diseases. Sometimes, stress and anxiety can also cause similar symptoms. You might be having stressful life due to father's death and miscarriage. So better to consult psychiatrist and get done counselling sessions. Try to identify stressor in your life and start working on it's solution. You may need anxiolytic drugs too. Don't worry, you will be alright. Avoid stress and tension, be relax and calm. Hope I have solved your query. I will be happy to help you further. Wish you good health. Thanks."
},
{
"id": 176925,
"tgt": "Suggest treatment for swollen lymph node in jaw area",
"src": "Patient: My 2 year old daughter recently had an I&D done on an abscessed lymph node on her jaw. It came up for no apprent reason. They are doing a biopsy on the contents. She has a follow up appt next week but I don t want to wait to ask a question. She has a bruise come up on her ulna side of her and it doubled in size in less than 24 hours and has pea size knot in it that can be move around. What could cause that? I m worried that she may have lymphoma because her lymph node doubled in less than 24 hours and had to be drained within a week. And no antibiotics helped at all. Doctor: dear user, thanks for your query.form you history its suggest that your child has lymaphadenopathy in neck region. it may be due to bacterial, viral or mycobactirial infection or may be due to leukemia or lymphoma.i suggest you to wait for biopsy report, because only by this report proper diagnosis can be made.regards- dr deepak"
},
{
"id": 168825,
"tgt": "What causes repeated bilateral inguinal hernia?",
"src": "Patient: hello , my daughter is 21months old and she had a bilateral inguinal hernia repaired when she was 11months.hernia come back both side after the repair then she had second time operation doctors found pea sized lump on her both side and they did a biopsy and also they checked her viginal lenth which is normal . Now i wonder my daughter is born with CAIS ,can somebody clarify for me please Doctor: hi, Welcome to this forum.Can understand your concerns.Repeated inguinal hernia can occur in a childin cases of postoperative complications, concomitant diseases like to Tuberculosis, premature birth. sinc, your child has already been operated earlier, we have to keep the possibility of postoperative complications as the first possibility.There are some diseases like tuberculosis,which is present can result in delayed healing of the hernia. This can also lead to repeated inguinal hernia.It is also seen that repeated in one hernia is more common in premature babies.I hope this will help you.Take care.Regards Dr Deepak Patel"
},
{
"id": 62785,
"tgt": "How should a forehead cyst with scarring, irritation and bleeding be treated?",
"src": "Patient: I have what i think is a cyst on my forehead. I got into a fight and was hit in the head and since that happened there has been a bump of my head. Doctors have told me its a cyst but i have also heard its built up scar tissue... The bump goes from being soft back to being hard im guessing when its irraitated. Do you think this is a cyst? Ive tried poking it with a probe but all that comes out is blood. Doctor: Provisional Impression-Right Leg Cramps /Under InvestigationHi,Welcome with your query to HCM virtual Clinic.I reviewed all the details of your query,in context of the of the forehead lump and its progress.Based on the facts of your query,You seem to have resolved hematoma with fibroma at the forehead hit.If it was a cyst as told by your doctors,cystic fluid should have come after your poking,but as you confirm in you query that blood came out after poking with a probe.This reduces the possibility of this forehead lump being a Dermoid or Inclusion Cyst /or Sebaceous Cyst of the forehead.To resolve this ambiguity in its causes-USG of the lump and FNAC Biopsy with HPR report,would fix the nature of this forehead lump.CT study to find out any intra-cranical connection with any other lump or cyst there and Second Opinion from a Surgeon and if need be from your NeuroSurgeon would resolve the diagnostic worries for you.Treatment- Would depend on the cause confirmation by a team work of your doctors as explained above.Your Surgeon would be the right doctor to work out the cause as told above and would plan a best possible treatment accordingly,after confirmation of the forehead lump.Excision Biopsy of the Lump with HPR study would resolve all the issues related with this forehead lump.Hope this reply would help you to plan treatment with your doctors and resolve your worries.Will appreciate writing excellent review comments to help the needy patient visitors like you at HCM.Welcome any further query and information in this regard.Good Day!!Dr.Savaskar M.N.M.S.Genl-CVTS"
},
{
"id": 888,
"tgt": "Can pregnancy happen when precum in exposed to vagina?",
"src": "Patient: I'm a 16 year old girl who forgot to take her birth control two days in a row then had sex without a condom. He didn't ejaculate inside me but we are worried about precum and the chances of me being pregnant. I'm probably about a week away from my menstrual cycle. I believe I'm 5'2\" and weigh 130. I'm completely healthy. I've had mono, strep twice each and bronchitis once, but that was last year. I'm healthy now. Doctor: Hi, I think there is a chance of pregnancy. If your periods get delayed, do a urine pregnancy test as soon as possible. If positive consult a doctor if you don't want pregnancy. If negative wait for your periods. Hope I have answered your question. Regards Dr khushboo"
},
{
"id": 117984,
"tgt": "Suggest treatment for low hemoglobin and ferritin level",
"src": "Patient: I am a female of age 48. I am suffering from low Hb level i.e. 3.3. Ferritin level is 1.4 whose range is 10-120.I have pain in my teeth and mouth. my hands and feet swell by even walking a few steps. I have become quite weak. eyesight is also affected.How can I bring my Hb and ferritin level to normal? Doctor: Hi,After looking at your lab reports, I am wondering why you are not admitted to hospital..Severe anemia has led to cardiac failure leading to limb edema.You need a red cell packed cell transfusion to improve your hemoglobin at the earliest.Later on injectable iron can be used and iron tablets can be taken at a later stage. Other vitamins also needs to be added to improve hemoglobin levels. Any further queries, happy to help again."
},
{
"id": 177633,
"tgt": "How to treat a horizontal cut on eyebrow of an infant?",
"src": "Patient: Hi, my 15 month old son fell and cut horizontally through his eyebrow 2 days ago, it was glued back together and 2 steristrips were placed over the wound. They are understandably beginning to peel off as they're stuck to his eyebrow and he rubs his face when he's tired. If they fall off should i be concerned/take him back or do nothing? Doctor: The doctor had done the right thing in fixing it that way. Generally steristrips, dermafix strips or dermabond glue is used to appose the skin properly. The healing occurs spontanously without leaving much scar if the apposition is proper. If the steristrips get peeled off, you would have to assess, how much healing has already occurred. If the wound has already closed reasonably and there seems less chances of gaping, you need not worry and can leave it like that. But if you feel that there are chances of separation, you can take the child back to the doctor who can put steristrips back again. The maximum difference that can be there is in the amount of scarring and nothing else."
},
{
"id": 172347,
"tgt": "What is the treatment of puffy eyelid in children ?",
"src": "Patient: my 5 years old lower eyelid is slightly puffy. He said it was itching only after I mentioned it and when I gave him cold compression to put on it he said no longer itchy and he seems fine not complaining any more. I am not sure what it might be and what to do to it. can you advice? Doctor: Thanks for asking I gone through your question and understand your concern most common cause of itching and swelling in eyelid is some insect Bite .as it is no more Itching .only swelling there now. in next 24 to 48 hours swelling should also disappear.so there is no need to worry Hope i answer your question. Wishes you healthy family"
},
{
"id": 67616,
"tgt": "What causes a lump under the ear?",
"src": "Patient: I got my ears gauged yesterday and today later in the evening a lump under my ear on the left side of my neck appeared out of nowhere. Its sore and I can cover it with the tip of my index finger. The ear lobe isnt infected. its not swollen or super sore so is it just my lymphnode gland reacting to this injury ? Doctor: Hi,It seems that there might be having enlarged post auricular lymph node.Due to ear piercing there might be having some infection.Clean prick site and apply antibiotic cream.Ok and take care."
},
{
"id": 138021,
"tgt": "Suggest effective treatment for swollen and painful coccyx bone",
"src": "Patient: I have had spinal surgery almost 2 years ago. Upper spine. c5,c6, Infused with metal. Last October had another car accident. In pain ever since. Was always in pain but now worst. Slipped on stairs 1 week ago. Now coccys area is swollen and hurts. How long does this injury lasts? Doctor: Hi,Thanks for your query.You are suffering from coccydynia- inflammation of the bony area (tailbone or coccyx)Cure for your coccyx pain-1.Use a well-padded seat when sitting and avoid long periods of sitting when possible.2.Do sitz bath- take warm water in a tub and sit in this.It will relax the muscles around the coccyx.3.Do exercises that will strengthen your tailbone's supporting muscles and stretch ligaments near the pain. Consult a physiotherapist for this. Therapists may perform a combination of:Levator ani massage and Levator ani stretching.4.Correct your posture. Poor posture may be contributing to your tailbone pain. Try to sit upright. If you get a sharp pain when getting up from a sitting position, lean forward and arch your back before rising.5.You can take anti -inflammatory drugs like Tablet Motrin 1 tablet as and when required ( do not repeat a dose before 6 hours) with food will help relieve the pain.6.Apply diclofenac gel on the affected area.I do hope that you have found something helpful and I will be glad to answer any further query.Take care"
},
{
"id": 208176,
"tgt": "What is the treatment for overactive imagination?",
"src": "Patient: Hi Doctor, My mother lives with my 2 brothers, one is married and has a daughter, the other is studying. She has this problem of over imagination. she thinks the way my elder brother behaves with her is because of his wife, she talks wrong about her.....etc..... She always feels insecure. This is been going on since 4 years, now she doesnt trust my brother as well, she feels that \"he is a wife pupet, he does what she says.,...& he doesnt listen to her at all...\" This has strained the relationship with my sister-in-law too. Please help. Doctor: DearWe understand your concernsI went through your details. I suggest you not to worry much. The given description is not enough to get into a proper diagnosis. Though I assume that your mother is actually troubled with insecurity. This happens with every mother of male childs. When children marry, mother feels insecurity and start behaving to ensure that son is taking care of her. One sort of attention seeking disorder of old age. This couples with unsettling the new daughter in law. There is nothing to worry as such phenomena is normal. Your mother should feel secure and you should make her feel secure and that is childrens duty. If you require more of my services in this regard, please post a direct query and I shall be happy to provide you psychotherapy tips.Hope this answers your query. Available for further clarifications.Good luck."
},
{
"id": 16172,
"tgt": "Bright red itchy skin in groin, tried various creams, no help. Had angioplasts inserted through the groin. Related ?",
"src": "Patient: The skin in my groin is very bright red and itches very much. Further under the groin the skin looks like a bruise . I have tried creams to try to deal with the complaint including Lamisil, Mycil Cream, Corstisone cream and finally an iodine spray, but nothing seems to work. I have had angioplasts inserted through the groin about six weeks ago and I was concerned that this may be related to the situation. Is this something or nothing to be concerned about please? Doctor: Hello ic.brooks, Thanks for posting your query. I think your problem is not the infection but the intertrigo, it is a condition of moist raw itchy skin which becomes reddish and develops in areas that are in contact all the time, like in the armpits, the outer side of labia/scrotum and inner side of the thighs. Wearing synthetic under garments hampers the circulation of air and retains moist making the problem worse. You should avoid tight pants and synthetic under garments and keep the region dry most of the times. Immediately wipe and dry yourself with thick cotton towels after you take bath. You should use a medicated dusting powder containing Clotrimazole/miconazole/salicylic acid, thrice a day. The cannulation for angioplasty through local skin does not seem a probable cause. I hope this answers your query. In case you have any follow up queries or doubts, you can forward them for me, and I shall be glad to help you. Please accept my answer in case you do not have further queries. Wishing you good health"
},
{
"id": 129582,
"tgt": "Is severe muscle pain due to the prolonged use of air conditioners?",
"src": "Patient: Hello Doctor, After 3 years of working in open space office under air conditioner, which blew air into my leg, I have terrible pain in the muscles and tendons. I was trying every possible over the counter drug and cream. Nothing helped. Naproxen help a little. I do not know what to do next. I would appreciate your help. Thank you. Nadia Doctor: Dear patient thanks for the question.it may have something to do with body hydration and humidity which is relative to ambient temperature. Try hydrating with isotonically balanced fluids around a pH of 2.0 (acid -Citrus juice 2 x Lemons, 2 x Limes + 2 x Oranges with equal parts chlorine free water with the addition of magnesium salts (Magnesium carbonate, MgCO3 and or Magnesium sulphate MgSO4) all day and half the night. If this does not help please go to your doctor and discuss the possibility of fibromyalgia"
},
{
"id": 83048,
"tgt": "HCT test result high, CRP, BUn high. Can high HCT give a false ESR ?",
"src": "Patient: Hello.....I just received some test results back today. The HCT test is HIGH 45.2 - HGB is 15.1. I also have a CRP at 1.4 HIGH and BUN at 18.... Creatinine 0.9. The ESR is 8. I have read where a HIGH HCT can give a false ESR Rate. I have a Postive ANA 1:320. Back in 1996 I was diagnosed with Lupus....then he said Undifferenciated Connective Tissue Disease and Systemic Fibromyalgia. I am in horrible pain and in a flare. I just broke out with sores in my nose again which my doctor never sees. I went to my Dermatologist on Tuesday. He thinks I have Impetigo . I also have Granuloma Annule. Something is going on and they can t figure it out. Could I have Lupus? I have a lot of the same Symptoms. But, my doctor says he has never seen anyone have Lupus without a High SED RATE. I have read a High HCT and interfere with the SED Rate. Thank You For Your Assistance! AAAAA Doctor: Hi It could definitely be lupus. The esr may be false, that is true, and many times it does not correlate with the disease activity of lupus."
},
{
"id": 157689,
"tgt": "Abdominal and testicular pain, hot flashes, high calcium level. Done biopsies, CT scan and ultrasound, negative for cancer. Suggest",
"src": "Patient: I am a 42 year old male with abdominal (lower right) and testicular (right) pains with Hot flashes in both legs. It started in May 2013. Every time I'd put something in my mouth, I'd get nauseated. Then I started having stomach pains. I had an endoscopy and everything was normal. While they were in there, 3 biopsies were taken, all negative for cancer and H. palori (probably spelled wrong). Then CT scan and ultra sound on abdomen, gallbladder and appendix was normal but they did find 2 \"spots\" on my liver (another CT and ultrasound scheduled for December 2013). Colonoscopy performed and only found internal hemorrhoids. Now the pain is in my lower right abdomen and right testicle. I saw a Urologist this past Tuesday and they did an ultrasound on it and said it appeared normal. Had blood tests done, LDL slightly elevated, bilirubin was a little off, no diabetes, lead levels normal, calcium elevated, B12 normal, but they said something else was also off (can't remember the name). Because of that they want to check my thyroids. Would or could my thyroids be causing the pain. Sometimes I feel good and others times it feels like I was kicked in the abdomen by a horse and sometimes it feels like someone is squeezing my testicle. Any info would be gratefully appreciated. Thanks. Doctor: confirm what the spots in liver are but getting the scheduled tests done. your symptoms are not related to this but since the imaging of rest of the body is normal, there is nothing serious about it."
},
{
"id": 96823,
"tgt": "How to treat head pain and fever caused by trauma?",
"src": "Patient: On Friday my step son and husband were playing being at the wrong place wrong time, my step son fell back and hit his head on my tooth really hard and I got an immediate throbbing pain in my head. Lasted a couple mins and went away. Saturday I woke up with a mild pain on my far right side toward my back. Sunday I woke up with intense pain on that same side and ran a fever and a throbbing headache. I took some pain and fever otc meds. Monday and Tuesday I had the same symptoms. Now it s Wednesday and I still have a fever and chills and the throbbing headache or migraine or whatever and the pain is still on the right side but moved toward the front. I went to the er last night to get some help but the wait was about 8 hours so I left. What is wrong with me?? Help! Doctor: U need to get a jaw x ray done as u could have a tooth fracture that is now infected. Also antibiotics and painkillers might help. Try using hot compresses in area of back pain."
},
{
"id": 13989,
"tgt": "Suggest treatment for rash caused by sweating",
"src": "Patient: I have had a rash for about 6 weeks now it started by my riding my bike and sweating a lot..i used powders for a while and it got a lot better..but it just wouldn t quit clear up..so went to my dr and he gave me this (clotrimazole and betamethasone dipropionate ...it is helping I been using it about 12 days but it just seem to me its being stubborn...should I be using something else or is there something I can do different... Doctor: Hello and Welcome to \u2018Ask A Doctor\u2019 service. I have reviewed your query and here is my advice. I would recommend you to apply topical steroid like Clobetasol Propionate cream on the affected areas and take anti histaminic like tablet Cetrizine depending on the intensity of itching. Regards."
},
{
"id": 156770,
"tgt": "What are the chances of cancer if having cyst on ovary and having surgery to remove ovaries?",
"src": "Patient: Am 53 years old. Ultrasound shows 5 mn complex cyst. Catscan shows prominet ovary. Had hystomtory 25 years ago. Ca125 test came back good. Had severe endometriosis wheb younger. Surgery schedule next week to remove ovaries. What is chance of cancer Doctor: complex cyst will increase the chance of cancer. removing ovary will only give exact answer, whether it is cancerous or not. If u have completed your family, then this is the best option that is to remove ovary. In early ovarian ca, you may not get ca 125 levels elevated."
},
{
"id": 186291,
"tgt": "What causes difficulty in closing lips over teeth after removal of braces?",
"src": "Patient: My upper lip has been, for two weeks now, requiring a thoughtful effort to close it over my teeth. It is strange to describe, but I wore full-metal braces as an adult, and I had to \"work\" to close my lips over the braces. Now, my husband describes the lip thing as a similar motion to the era of the braces. This feeling is only on the right side. It is an unusual, but concerning thing. I am a teacher, so I speak before students constantly. I find I need to be more deliberate in my pronunciation. Thank you. Doctor: thanks for your query, i have gone through your query. the habit of closing your lips or path of closure of lips will change as you get used to the normal way of closure. keep practicing daily 2 times sitting in front of the mirror. nothing to worry it will get solved by its own. i hope my answer will help you, take care."
},
{
"id": 95494,
"tgt": "Is there any side effect oflooz syrup ?",
"src": "Patient: I have been taking looz syrup 90ml per day at night is it have any side effects i had been using it since 3 years i am not gaining any weight is it due to looz,i have intestine abstuction so i am taking looz otherwise i get stomach pain please guide me. Doctor: hi satya, welcome to hcm thanks for the query looz contains lactulose. It is a stool softener, and it reduces the pain which you get while defecating it will not cause weight loss.it may cause gas or flatulence,abdominal cramp,sometimes diarrhea and electrolyte imbalance. but is over all safe to use take care"
},
{
"id": 161305,
"tgt": "Does severe epilepsy with seizures, tremors and vocal noises need ER admission?",
"src": "Patient: My son has severe epilepsy with many different types of seizures. Today he stared to have tremors, they are involutary and he shakes a bit and some vocal noices as well. This keeps happening like evry 30sec. He is on many meds but we just recently stared Banzel. Im wondering if i should take him to the ER they have no Ped.Neuros at this ER . Doctor: Hi, If your child is having seizure every 30 seconds, then he must be taken to emergency at the earliest, without delay. Hope I have answered your query. Let me know if I can assist you further. Regards, Dr. Varinder Joshi, General & Family Physician"
},
{
"id": 56173,
"tgt": "Is yellow eyes and dark urine a symptom of jaundice?",
"src": "Patient: Hi. My name is Alana an my husband has been have a few symptom s and I am highly concerned.. The symptom s are: yellow eyes dark urine pain in lower back on right side (@ times) chronic fatigue if u could plz get back with me asap it would be highly appreciated Doctor: Hello, I can appreciate your concern for your husband and i will try my best to resolve your query. I would like to tell you that yellow sclera and dark yellow coloured urine mostly indicative of jaundice. So i would suggest your husband to check the bilirubin level as well as SGPT level as soon as possible to confirm the diagnosis. In most of the cases of jaundice, no treatment is required. Take plenty of glucose, have a healthy diet, take sufficient rest and avoid unnecessary medications. Check your SGPT level to check the improvement of condition. Usually it will resolve completely in 10 to 14 days. So don't get much bothered, get blood examine done and follow the guidelines, your husband will be fine very soon. Hope this answers your question and let me know if you have any further relevant queries, i will be happy to guide you."
},
{
"id": 121338,
"tgt": "What is the cause and treatment for severe pain in ribs after an injury?",
"src": "Patient: i am 350 pounds but very strong and can easily touch my toes and lift my own wieght, i fell flat on my back 2 weeks ago, it felt like i broke my ribs but exray said no broken bones, but in the morning i feel not to bad just a bit of pulling on my left side ribs, but by night time im almost in tears, i ve been off work for 2 weeks and dont know what to say anymore when asked when i can return to work ...i lift on 50 pounds hoses and drive tank trucks pls help, how long should this pain stick around Doctor: Hello,It may be due to rib fracture. For further assessment you may require chest x ray, pulmonary function test after chest physician consultation. For pain you can take tablet acetaminophen. Further treatment mainly depends upon the underlying condition. Continue breathing exercises. Hope I have answered your query...Let me know if I can assist you further.Regards,Dr Shyam kaleFamily and general physician"
},
{
"id": 199206,
"tgt": "What is the solution for a bump on the side of my shaft?",
"src": "Patient: I've had a bump on the side of my shaft for over a year now. Last month I had a blood and pee std test and everything came back negative, though I do not believe those tests include hpv. It hasn't spread, but is on a cycle of growing (and then shrinking), and I pop it and a little pus comes out which shrinks it down but even after popping the bump doesn't feel empty. What could this be? Thank you in advance for your help. Doctor: HIWell come to HCMI really appreciate your concern, if the pus is coming out of this lesion then this could be infection some time this may need surgical intervention for to drain the pus and antibiotic may be necessary, else this is nothing to worry, hope this information helps, take care and have a nice day."
},
{
"id": 64218,
"tgt": "Suggest remedy for tender hard lump under armpit",
"src": "Patient: Hi my boyfriend has a tender red hard lump under his armpit, which appears to have pus. It has appeared quite suddenly over a couple of days and he believes he should try open it up to get the pus out.....what is your advise? As well as the (roughly 4cm diameter) he also has smaller pimple looking spots forming a rash around the area. He is 33 years old, 187cm tall, 95kg in weight and with no major medical history, the most notable item being indigestion - which he has suffered for years. Doctor: Hi dear your boy friend is having a - acute follicullitis of the armpit hair, which is about to open out.- pointing head and red rash around the point indicates acute supputative process inside the lump.Treatment advised by me under care of Er Surgeon is as follows-a - tb- oflox - Oz x 5 days, b- tb- declofenacx 5 days.c-Surgical proper drainage with dressing for 5-7 days timed- care of any associated diabetes if any to be planned by FBS and PPS testing.Hope this helps to resolve your boyfriends armpit painful lump.Wishing fast recovery. Wellcome toHCM for any help you need."
},
{
"id": 74424,
"tgt": "What could intermittent chest pain suggest?",
"src": "Patient: i keep getting horrible uncomfortable chest pains that come and go, but when they do theyre unbearable, sometimes hard to locate, lastnight it seemed to be within my neck to, is this something to worry about? please help me, im hardly getting any sleep due to the amount of worrying i do. Doctor: Thanks for your question on Healthcare Magic. I can understand your concern. In my opinion, we should first rule out heart related diseases for your chest pain. So get done ecg, 2d echo and stress test. If all these are normal then no need to worry for heart diseases. Sometimes, stress and anxiety can cause similar symptoms. So consult psychiatrist and get done counselling sessions. Try to identify stressor in your life and start working on its solution. You may need anxiolytic drugs too. Don't worry, you will be alright with all these but first rule out heart diseases. Hope I have solved your query. I will be happy to help you further. Wish you good health. Thanks."
},
{
"id": 158615,
"tgt": "Ultrasound shows triangular solid nodule in breast. Due for biopsy. Chances of cancer?",
"src": "Patient: I had a breast ultrasound done and results says I have a triangular shaped solid nodule 8mm in size. During ultrasound the techs were talking about a blood vessel running to it. I have a biopsy Tuesday, I would like to know what are the odds that this is breast cancer. My maternal Aint died in 2009 at the age of 48. I just want to prepare myself, can you give any insight? Doctor: Hi, Simply based on this description nothing can be commented. There are other causes of solid swelling in the breast also. Do not worry at all. This is a small size lump. If anything serious can be found on biopsy then also total cure is possible. Nothing serious will happen to you. Do not get nervous. There is very good treatment option for the disease whatever it may be. Consult your surgeon."
},
{
"id": 35654,
"tgt": "Suggest treatment for sepsis",
"src": "Patient: I was recently diagnosed with sepsis, asked to return to hospital where I stayed for 3 days as was treated with I don t know what. I was released with an antibiotic Ceftlin 500 mg q 12 hrs. Apparently the sepsis was brought on by urinary tract or kidney infection, not sure. I m a diabetic (2) and I just want to know if Ceftin is a good antibiotic for this illness.. and how long will it take before I should feel better? Doctor: Sepsis is a bacterial infection in the blood. It often starts in the urinary tract. It can be caused by many different types of bacteria. The correct antibiotic to cure it depends on the type of bacteria you have. Since you were in the hospital, they did tests to diagnose the type of bacteria and to see what antibiotics it is susceptible to. In that case, you would initially be treated with IV antibiotics, and when the infection was under control, you would be switched to oral antibiotics. Typically, the oral antibiotics would be the same or similar to the IV ones were, so the Ceftin should be a good antibiotic in your case. You can have residual effects after such a serious illness for some days or weeks afterward. You should also be keeping a close eye on your diabetes control as sepsis could upset it and poorly controlled diabetes also reduces your immunity. Hope this helps."
},
{
"id": 114642,
"tgt": "Suggest homeopathic medicine to reduce creatine levels in blood?",
"src": "Patient: my father aged 80 years is having kidney problem. one of the kidney has reduced n size and is not functioning. second one is having slight infection. Dr started dalysis as the patient is having creatine level of 5.5-6.2 varying. but when he was in the hospital the creatine level has come down to 3.2 from 7. now alternate days he is undergoing dialysis treatment and the doctors are telling the creatine level is varying in between 5.5 to 6.2. my father used to take only homeo medicine since his childhood. now I want to know is there any medicine in homeopathy for reducing the creatine level? Doctor: I am sorry to hear about your father. It looks like he has chronic kidney disease (CKD) from your description. It is a slowly progressive disease due to multifactorial causes such as hypertension,diabetes, recurrent kidney infections,chronic use of painkillers etc. The condition itself is not reversible.The best option for him would be to continue with dialysis till his metabolic parameters stabilise. The problem is not only the creatinine but the glomerular filtration rate (GFR) which is bound to be low in his case considering his age. I would suggest the you continue with supportive treatment as advised by your nephrologist and avoid any other medication"
},
{
"id": 130628,
"tgt": "What causes shoulder blade pain and stiffness?",
"src": "Patient: Hey i am 33 year old female for about 6 months i have had right shoulder blade pain stiffness, and over the last month it has become worse when moving the arm or holding it straight out the pain has since moved down as well to elbow making it hard to open and close my fingers Doctor: Hi,A lot of causes for shoulder stiffness and pain like the bones, ligaments and tendons that make up your shoulder joint are encased in a capsule of connective tissue. Pain attacks or stiffness occurs when this capsule thickens and tightens around the shoulder joint, restricting its movement.Doctors aren't sure why this happens to some people, although it's more likely to occur in people who have diabetes or those who recently had to immobilize their shoulder for a long period, such as after surgery or an arm fracture. In my opinion physiotherapy, yoga, hot fommentations with topical gels is helpful, advil, acetaminophen, muscle relaxants, rest is recommended it may take sometime. In some cases if persists you may ask your physician for imaging and physical examination for the best evaluation for proper x-rays, MRI, other investigations like RBS, urine analysis, SUA, RH factor, and ASOT titre. I think this is only muscle sprain and will go away soon.Hope I have answered your query. Let me know if I can assist you further.Regards, Dr. Ahmed Aly Hassan"
},
{
"id": 109860,
"tgt": "What causes back pain and pain during urination?",
"src": "Patient: hi im a 37 year old woman i have got constant back pain on the left hand side and in my stomach area an i was sick after my meal last night. when i need to pass urine the pain is like a dull ache and it fells like ive got to go straight away or im in more pain. i suffer from pcos Doctor: Dear mam ,u r sufferening with urinary tract infection.go for simple complete urine examiation depending upon the infection u have to use antibiotics."
},
{
"id": 191169,
"tgt": "I have a dark brown bump on my tongue, What could it be?",
"src": "Patient: Dark brown/black spot/bump on my tongue, What could it be? I have a tiny dark brown/black spot or bump on my tongue, what could it be? I m female. Doctor: which part of the tongue, how longit is present"
},
{
"id": 158989,
"tgt": "Terrible pain in neck and back, causing nauseas and lump behind ear. History of cancer x2. Suggestion?",
"src": "Patient: Hi, I m a 33 year old female with a history of cancer x2. For the last 4 months I ve not been feeling well. Tired, run down, exhausted, etc. I should mention I have quite a bit of stress and have attributed most of this to stress, although my gut tells me something may be wrong physically. Most recently I ve had terrible neck and upper back pain (but seems to stem from the neck pain ) that gets very severe to the point of nausea and severe headaches. A few days ago I noticed a hard, immobile, lump (I m assuming a lymph node), behind my left ear to the right of my jaw bone. It is tender upon touch. I ve been trying to call my primary to schedule an appointment, but the offices are closed today. Thank you Doctor: Hi, You have not mentioned what cancers you suffered from , not also mentioned the treatment you received. Back pain may be due to spinal injury, muscle spasm, disease progressing to invade the neural structures. Clinical examination is mandatory to have an idea for identifying the cause. Tender lymph node generally is infective, proper antibiotic course is necessary. You should consult a physician."
},
{
"id": 119448,
"tgt": "Suggest treatment for leg pain and swelling due to arthritis",
"src": "Patient: my husband 30 years old, height 5 8 & weight 68 kg. he suffering arthritis last 4 years. total heep replased. last 2 years have no pain but now he suffering pain his legs. legs are swelling and have a pain. so please suggest me best doctor in cmc vellore for treatment this problem Doctor: Hello, Thirty year age is very less for arthritis. If there is pain at knee then this does not looks to be due to arthritis. There may be weak muscles or some cartilage issues. You may ask him to do some exercises which helps in strengthening his thigh muscles. There are also some medicines like glucosamine, chondroitin and collagen preparations which can be helpful for him. Take care. Hope I have answered your question. Let me know if I can assist you further. Regards, Dr. Mukesh Tiwari, Orthopedic Surgeon"
},
{
"id": 80624,
"tgt": "Can tea served with honey be of help to a toddler suffering from bad cough?",
"src": "Patient: My son is gna be 2 years in the middle of april, and he has a really nasty cough. Other than this harsh cough hes okay. hes playing fine, eating fine, and being himself. he isnt sleeping well because of this cough though. and i wanted to know if it would be alright to give him a little bit of tea with honey or if there is anything else i can do to help him other than just medicine. Please give me your best advice. Thank You. Doctor: Hello dear, thanks for your question on HCM. I can understand your situation and problem. Yes, you can definitely give tea and honey to your grandson. Honey is good for soothing action in throat. So this will reduce irritation in throat. I advice you to give him honey in green tea. As green tea contains antioxidants and they are good in protection from infection by increasing his immunity. You should also give him steam inhalation 5 - 6 times a day. This will also reduce coughing. Ask him to drink fluids more.If he is not improving with all these in 5-6 days then don't hesitate to consult pediatrician. As he may need medicines for cough."
},
{
"id": 205625,
"tgt": "Suggest cure for depression and attention deficit",
"src": "Patient: Hello. All my life I have never been able to concentrate and my attention is always diverted. I am always depressed but keep it hidden among others.. I want to seek medical attention but I don t think my insurance covers it. I think an antidepressant would help me. I don t really know what to do. Doctor: Hello Welcome to health care magic I can understand your situation. I appreciate your effort to seek online support. As per your details, you are possibly having underlying anxiety/depression. For this consult a psychiatrist for detailed evaluation. Following treatment strategies would be helpful- 1) Antidepressants/Antianxiety agents like SSRIs (escitalopram, sertarline, paroxetine etc)2) Antianxiety agents like clonazepam, lorazepam, alprazolam etc3) Psychotherapy/ Counselling sessions4) Relaxation techniques like yoga, meditation, deep breathing etc Hope it helps Good luck Regards Dr. Gourav Monga Consultant Psychiatrist"
},
{
"id": 123134,
"tgt": "What causes pain inside leg and hand?",
"src": "Patient: hello doctor, since last two years, i get pain inside of my leg and hand with heat. but i dont feel any heat nor any fever outside. i cant walk when i get this pain. can u suggest me what problem is that doctor. I am from Bhutan. sorry for my bad english Doctor: Hello, As a first line management you can take analgesics like paracetamol or aceclofenac for pain relief. If symptoms persist, it is better to consult a physician and get evaluated. Hope I have answered your query. Let me know if I can assist you further. Regards, Dr. Shinas Hussain, General & Family Physician"
},
{
"id": 30469,
"tgt": "Suggest treatment for low immunity",
"src": "Patient: i have been diagnosed with H.S. and it cause me to have a staff infection about a year ago my doctors in dayton told me that my immune system has been weakend and my nerver get back to what it used to be. since then i have been getting sick all the time even now i have a cold, however this cold feels more like a flu how do i treat a cold with a weak immune system? Doctor: hi sir, welcome to HCM, i understand ur problem, tobacco and alcohol also reduce immunity, u have to stop that if u have that habit, take protein powder with milk twice a day, get enough sleep and manage stress, eat plenty of fruits, green leafy vegs, nuts, seeds etc, take cyperheptidine syrup thrice a day after food 10ml, walk everyday one hour, hope u satisfy with my answer, thank u"
},
{
"id": 95354,
"tgt": "Is typhoid a common cause for left-sided abdominal pain ?",
"src": "Patient: I have pain in my left side and back since yesterday. I have been on medication for typhoid upto last week - 31st july. I am a diabetic and am on medication for the same,as also for hypertension . Doctor: Hi,Vinny, Thanks for query, If temperature is alright then nothing to worry, Pain may be due to as in typhoid infection and soreness in the intestine causing pain in abdomen. Due to constipation which is common in typhoid may also cause pain. Take liquid diet and plenty of water. Ok and bye."
},
{
"id": 172685,
"tgt": "How to improve the appetite of a 15 month old?",
"src": "Patient: hi, my name is Meraj my son is 15 months old but still he is not eating and he is very week even he not stand for long tile his weight is decreasing day by day i have consult maby doctors but there is no possintive response from them please suggest i am very worried. SHAIKH MERAJ Doctor: Hi.. I know how difficult to eat child of this age. U have improvise some of following things1.give frequent small diets and small diet should be with extra calories(add sugar or ghee or oil). 2.never force the child for feeding. If he or she is hungry he will come ask or food definitely. Till 1yr parents are responsible for child feeding after that child can walk some can come and ask for it. 3. Always take breakfast, lunch and dinner in groups so child can enjoy feeding. 4.give minimal amount of milk5.give syp vitcofol and syp zincovit bot are tonic which may improve diet. Hope u r getting ur ans"
},
{
"id": 72079,
"tgt": "What causes excessive cough in the night despite taking medication?",
"src": "Patient: I have been to the Dr for severe night cough...The medication she prescribed has been taken, However the cough is still there. I have never smoked, I work out daily, however cannot sleep.At night due to coughing. Please advise some relief...thank you Doctor: Thanks for your question on Healthcare Magic.I can understand your concern. Most common cause for night time coughing is Asthma. So better to consult pulmonologist and get done clinical examination of respiratory system and PFT (Pulmonary Function Test).PFT will not only diagnose asthma but it will also tell you about severity of the disease and treatment is based on severity only. You may need inhaled bronchodilators (formoterol or salmeterol) and inhaled corticosteroid (ICS) (budesonide or fluticasone).Oral combination of antihistamine (levocetrizine or fexofenadine) and anti allergic (montelukast) once a day at night is also beneficial.Don't worry, you will be alright with all these. Hope I have solved your query. I will be happy to help you further. Wish you good health. Thanks."
},
{
"id": 91427,
"tgt": "What is the cause of stomach pain?",
"src": "Patient: I have has a cold for the past 3 days, the more those symptoms subside the more pain in my stomach I have. It has been a dull achy pain constantly. But when I inhale deeply it is a sharp intense pain from the center of my ribs to my belly button. Should I go to the doctor or will this go away? Doctor: Hi. Sure , you have to go to a Doctor fro clinical examination and investigations like X-ray Chest/ spine and ultrasonography to decide the cause and get a proper treatment thereupon.This can be due to inflammation of the pleura, or some problem with liver or so in the abdomen giving you a typical pain."
},
{
"id": 197297,
"tgt": "What causes testicle pain post surgery for varicose veins in leg?",
"src": "Patient: Hello, I have been operated for vericose veins in my left leg in March this year. About two months ago, I began feeling a pain in right testicle. It seems the testicld list its axis. Colour Doppler scan was done of the scrotum and vericeole detected in right testicle. An ongoing medical treatment and have been advised scrotum support. I am wearing sports underwear even at night since two weeks now. While the pain seems to be s little less, I feel restricted in any sort of bending or streatching movement. Raiding the right knee to a right angle while standing causes discomfort and pain the right testicle region. Will this pain go away with time ? Are there any excercisrs that would help ? Don t wish to take pain killers. Is a surgery recommended ? Doctor: HelloThanks for query .Pain in testicle after surgery for Varicose veins is mostly due to secondary Varicocele that develops due to Varicosity(Dilatation) of testicular veins that drain in long sephanus vein which is ligated during surgery of varicose veins of leg ..This usually gets resolved over a period of few months after collateral circulation gets established .Till then use scrotal support and pain kllers as and when necessary .Dr.patil."
},
{
"id": 31955,
"tgt": "Suggest remedy for chest pains and headaches due to dry hacking cough",
"src": "Patient: I have had a dry hacking cough for over a week now. Its giving me chest pains during the night when I lay down and headaches during the day. Have been taking day nurse during day and night nurse at night. Am never normally ill so this is getting me down. What can I do? Would be grateful for any advice please. Thank-you Doctor: Hi Dear,Welcome to HCM.Understanding your concern. As per your query you have chest pains and headaches due to dry hacking cough which is due to upper respiratory tract infection. It could be due to impaired immunity of your body. I would suggest you take nasal decongestant sprays, Antihistamines and antibiotic combination as infection is present. Co amoxiclav is best treatment option. You should take steam inhalation. Visit ENT specialist once and get it examined. You should get chest X-ray and pulmonary function tests done to check extent of infection and start treatment accordingly.You should take Take Diphenhydramine and Dextromethorphan for 4-5 days. Maintain proper hygiene and avoid cold liquids and spicy food. Do warm saline gargles. Hope your concern has been resolved.Get Well Soon.Best Wishes,Dr. Harry Maheshwari"
},
{
"id": 53025,
"tgt": "What is the recovery time for stent in gall bladder?",
"src": "Patient: my 79 yrs old aunty has had a stent in her gall bladder but recent problems ,with growth not being operable,has been moved from hospital to a nursing and put on morphine syringe.could you kindly guess how long approximately we have to see her again?we live 300miles away in cornwall ,she aunty ann is in chichester.THANKYOU. Doctor: Hello, I hope you are well, it is important to know what was the cause for the placement of the stent since they are usually placed before benign processes such as lithiasis in the bile ducts or as a form of palliation in malignant obstructive processes. Bile stents should be monitored against the risk that they may clog, migrate or even cause infections such as cholangitis, so many cases and depending on the type of stent they should be replaced every 3 months."
},
{
"id": 143146,
"tgt": "What is the cause of pain in arm and tingling in hand?",
"src": "Patient: Hi!Yes - if you could answer this question. My name is Fran and I walk often. Last April I was taking a walk on a dirt path in Yarmouth, MA. I tripped on a tree root and took a hard fall. I put my arms out to break the fall. I thought I had broken both upper arms it hurt so bad. I could move my arms and saw no sign of fracture. The pain eased up but continued to hurt for about a month. My left arm is fine but my right arm continues to hurt at night. It's an aching throbbing feeling. I end up taking some Ibuprofen or Aleve. I tend to sleep on my right side so I think it may be the weight of my body. I, also, sometimes get a little tingling in my right hand. I still work out 3 times a week and lift 10 lb. weight and do pushups. My arm doesn't bother me during the day - just at night. Do you think this is muscle, tendon, or nerve pain and do you think it will eventually go away?. Should I stop working out with my arms for a while and, if so, how long? Thank you for your help. Doctor: Hi, Welcome to HealthCareMagic.com I am Dr.J.Mariano Anto Bruno Mascarenhas. I have gone through your query with diligence and would like you to know that I am here to help you.Background1. History of Fall Complaints1. Arm Pain2. Continuous Pain - aching throbbing feeling - Right Arm 3. Tingling ImpressionThis is suggestive of Cervical Radiculopathy How to Proceed Needs Clinical Examination and Nerve Conduction Studies, XRay Neck and Other Radiological Evaluation What to do Please consult a neurosurgeon in your placeHope you found the answer helpful.If you need any clarification / have doubts / have additional questions / have follow up questions, then please do not hesitate in asking again. I will be happy to answer your questions. In the future, for continuity of care, I encourage you to contact me directly in HealthCareMagic at http://bit.ly/askdrbruno Best Wishes for Speedy Recovery Let me know if I can assist you further.Take care."
},
{
"id": 221258,
"tgt": "What causes back pain, leg numbness, sickness, stomach pain during a pregnancy?",
"src": "Patient: hello my name is Samantha i am 36 weeks pregnant i am back pain that comes and goes about every 15 to 20 min and i am have numbness in my right leg that dose not go away i have also gotten sick 4 times today in the last 2 to 3 hours my stomach is very hard and hurts Doctor: Dear Samantha,Low back pain at this stage is highly suggestive of opening of the cervix or merging of the cervix into the uterus (effacement). In both these processes, the cervix (mouth of the uterus) gets stretched which causes pain in the cervix. Cervical pain is felt at the low back area as referred pain. If this is your first pregnancy, cervical effacement starts sometime after 36 weeks of pregnancy. Usually the pains start anytime after a week or so. So you have to be prepared for your delivery. You need not be concerned about delivery after 8-10 days as the baby is sufficiently matured after it attains 37 weeks of gestation. Wish you safe happy delivery with a bonny baby. Please breastfeed your child exclusively for 6 months, starting it within half an hour after delivery. Dr. Nishikant Shrotri"
},
{
"id": 93923,
"tgt": "Abdominal pain, numbness in legs, knees.What could this be?",
"src": "Patient: I'am a 23 year old female. I have very bad pain in my left lower abdominal sharp cramping pain for about a couple of weeks now. Now its getting so bad that I also have numbness in both of my legs down to my knees. Have a doctor appointment next week with my doctor. earliest I could get in. the pain is getting almost to much to bare. Doctor: Hi and welcome to HCM. This sounds like typical lumbosichialgia with nerve compression. You should do lumbal xrays or MRI to see is there disc bulge or any other spine disorder. ALso some neurologic disease may be considered but I am sure this is benign and you ll have to rest and take painkilers or corticosteroids till symtpoms improves. Sometimes is surgery neccessary too. WIsh you good health."
},
{
"id": 176926,
"tgt": "Suggest remedy for pain in head",
"src": "Patient: My niece who is 7 has been complaining a lot that her head hurts, she shows that it hurts right in front. Some days it hurts more and some days not so much, sometimes she closes her ears saying that it is too much noise and her head is pounding from it. Her doctor says that it is really nothing serious. Can you please tell me what it can be and what can be done to prevent those headaches? She is overall a healthy 7 year old girl. She does love salty foods, can that contribute to the headaches? Can she be dehydrated? Would appreciate your answer. Thank you! Galina Kolodyi ( YYYY@YYYY ) Doctor: Hi If the pain is so severe she needs complete work up including checking blood pressure,eye examination and CT head .If everything I us ruled out it may be migraine.Pl take her to a pediatrician or pediatric neurologist. God blessDr uma"
},
{
"id": 94157,
"tgt": "Severe pain n left leg, lot of discomfort at bottom of stomach, with discharge. Solution?",
"src": "Patient: I have severe leg pains especially in my left leg and both thighs I have also a lot of discomfort at the bottom of my stomach and some discharge sometimes it is yellow and sometimes a greeny colour I have not been sexually active in over 10 years I am very tired and don t have great circulation what would you advise for me Thanks. Doctor: Hi welcome to Health care magic forum. Thanks for calling H.C.M.F. It appears that you have got a pelvic inflammatory disease, all these symptoms are secondary to the infection. I advise you to consult a gynacologist for diagnosis and treatment. You may need to have C.T.scaning besides other routine tests for confermation . Simple urinary tract infection may also cause these symptoms . Wishing for a quick and complete recovery . Best regards."
},
{
"id": 5319,
"tgt": "Tired, lower back pain. On medication for pregnancy. Pregnant?",
"src": "Patient: Hi,My wife is under medication for pregnancy. After 3 months of medicines for controlling her irregular periods, it got regularized to 32 days cycle. Now is the in the first month of trying. Since 14th day after timing (Mar 9th) she is taking strone 200 mg twice a day orally.We mate as prescribed (14th - 20th day). Since taking strone she was feeling sleepy in 2 - 3 hrs after taking the medicine. Now since March 31st, she feels more tired, lower back pain and very mild abdominal abnormalities. Is she pregnant? Is this too early to expect anything?Kindly advice. Doctor: Hello,Thanks for your query. Strone is PROGESTERONE, what your wife is feeling is side-effects of progesterone. If she misses her periods get a urine pregnancy test done. Side -effeccts of progesterone:Stop using this medication and call your doctor at once if you have any of these serious side effects:sudden numbness or weakness, especially on one side of the body;sudden headache, confusion, pain behind the eyes, problems with vision, speech, or balance;fast or pounding heartbeats;chest pain or heavy feeling, pain spreading to the arm or shoulder, nausea, sweating, general ill feeling;unusual or unexpected vaginal bleeding;migraine headache;nausea, stomach pain, low fever, loss of appetite, dark urine, clay-colored stools, jaundice (yellowing of the skin or eyes);swelling in your hands, ankles, or feet;fever, chills, body aches, flu symptoms;a breast lump; orsymptoms of depression (sleep problems, weakness, mood changes).Less serious side effects of progesterone may include:mild nausea, diarrhea, bloating, stomach cramps;dizziness, spinning sensation;hot flashes;mild headache;joint pain;breast pain or tenderness;cough;acne or increased hair growth;changes in weight; orvaginal itching, dryness, or discharge.Less serious side effects of progesterone may include:mild nausea, diarrhea, bloating, stomach cramps;dizziness, spinning sensation;hot flashes;mild headache;joint pain;breast pain or tenderness;cough;acne or increased hair growth;changes in weight; orvaginal itching, dryness, or discharge.I hope this information has been both informative and helpful for you. In case of any doubt, I will be available for follow ups.Wish you good health.Regards, Dr ArifYou can consult me again directly through my profile http://www.healthcaremagic.com/doctors/dr-arif-n-khan/65133"
},
{
"id": 146727,
"tgt": "Suggest treatment for blood clots in brain",
"src": "Patient: Sir,My father has blood clot in brain he is about 61 years old . he is suffering with paining with hand and leg and when he is speaking some what different than before. please suggest us , i cant spend more money, Please give your suggestion to survive him.Even though last doctor can cure with medicine. How much correct i dont know.Iam very serious particular about that Doctor: Hi,Thank you for posting your query.Based on description, it seems to be a brain stroke. We need to continue physiotherapy to improve his chances of recovery.Medications are needed to prevent stroke, such as aspirin and statins.For pain relief, we may use pregabalin capsules.Best wishes,Dr Sudhir Kumar MD DM (Neurology)"
},
{
"id": 89520,
"tgt": "What does on and off sharp pain in lower left side mean?",
"src": "Patient: For about 9 months now I have had pain in my lower left side that comes and goes. I experience some sort of pain at least a couple of times a day. Most of the time it is a dull pain. Also, I have extreme discomfort if I try and reach for something. Also, for the last couple of weeks I have started noticing that I have a dry tongue. Well, today for the first time it has been a wave of sharper, shooting pains moving closer to the back, but still will pain in the side. Ever since I have noticed the pain it has always been tender to touch the area. any thoughts on to what is going on? Doctor: Hi Thanks for asking HCM. There are two main possibilities 1.Chronic constipation 2. Urinary system involvement like chronic infection or a small stone.If you are a female ther may be possibility of any gynecological problem. Get your ultra sound abdomen done things will become clear.Mean while drink lots of oral fluids,as your dry tounge also indicative of lack of hydration, take fibrous diet, you can even take mild laxatives if constipated.One more possibility of abdominal wall old small injury which can be differentiated by your doctor only.Do visit your doctor for complete and final diagnosis and treatment."
},
{
"id": 65617,
"tgt": "What is the treatment for lump on left rib cage ?",
"src": "Patient: Hi, I have a slightly enlarged spleen so I m told from my gp, I can see a lump on the side of my left ribcage and I have pain there, feels like its pushing on my ribs or something like that. every time i breath it hurts..... I m 6ft 5 , 23 , m , Caucasian I need help :( Doctor: Hi, dearI have gone through your question. I can understand your concern. You may have enlarged spleen. There are many causes for spleenomegaly. Malaria, congestion, leukemia, lymphoma all can lead to enlarged spleen. You should go for ultrasound abdomen and routine blood examination. It will give you exact diagnosis. Then you should take treatment accordingly. Hope I have answered your question, if you have doubt then I will be happy to answer. Thanks for using health care magic. Wish you a very good health."
},
{
"id": 35364,
"tgt": "Does positive TB IgA, IgG and IgM indicate tuberculosis?",
"src": "Patient: Hellow Docter,i am 38 years old.my serology report is as follow TB IgA-409(positive),TB IgG 229(positive),TB IgM .60(negative),montoux text 20 mm (positive)and ESR 28 mm 1st hour .Mostly my daigation disturb,weakness,body pain ,back pain,headuch.Does this mean i have TB. Doctor: Hello ,I understand your concern. I am Dr. Arun Tank, infectious disease specialist, answering your concern.TB antibody test is not the reliable test to diagnose the TB.This test is banned by the World Health Organisation.Even your test is stating that you do not have active TB infection. As we are living in the country where TB is very common so you may be exposed to the TB but are not having active TB infection.Even your ESR is not much high so this not makes you a TB patient.If you are suspecting of TB than you should test for sputum for AFB.I advice you to start doing daily exercise. This will clear all your weakness body pain indigestion and other things. Initially you wont feel comfortable with the exercise but gradually you like doing exercise.I will be happy to answer your further concern you can contact me here or you can contact me on bit.ly/DrArunWe wish you a best health at healthcare magic. You can hit thank you button if you are helped.Thank you,Dr. Arun Tank"
},
{
"id": 167411,
"tgt": "What causes ridges and soreness of upper palate?",
"src": "Patient: Good Morning. My little who is almost 4 is complaining of having a sore mouth, when he's pointing to it, it's the roof of his mouth that he's point too. When I look inside his mouth, the roof of it is rather lined, with ridges on on the gums. Unsure whether this is normally how it is supposed to be, not having paid much attention to it before. Wondered if this would be an infection? Thank you. Doctor: Hi..Welcome to HCM..As per your complain it seems that your son has an inflammation in his mouth leading to Stomatitis and it can be most probably due to viral infection leading to Herpetic gingivostomatitis ..The pain and inflammation can be followed by eruption of sores or bumps..Other common cause can be Apthous stomatitis that can progress into eruption of canker sores..As far as ridges over the palate is concerned they can be Palatine rugue that is a normal anatomic structure that are folds of mucosa..I would however suggest you to consult a Pedodontist and get him evaluated for the cause of the symptoms and after a proper evaluation only treatment can be given..As of now to relieve the symptoms you can make him gargle with numbing mouthwash containing Lignocaine..You can also make him suck ice popsicles and give him soft and bland diet..Feed him with plenty of fluids and avoid spicy and hot foods..Hope this helps..Regards,"
},
{
"id": 168757,
"tgt": "Why my 4 years old is vomiting in the morning?",
"src": "Patient: My 4yr old has been throwing up only in the morning for the past 3 mornings. Thurs was 5am, Fri was 4am, and this morning was 2am. He seems fine all day long, eats and plays. He had runny poop today but other than that, there are no other symptoms. Doctor: Hi...Thank you for consulting in Health Care magic.It seems your kid is having viral diarrhoea. Once it starts it will take 5-7 days to completely get better. Unless the kid's having low urine output or very dull or excessively sleepy or blood in motion or green bilious vomiting...you need not worry. There is no need to use antibiotics unless there is blood in the motion. Antibiotics might worsen if unnecessarily used causing antibiotic associated diarrhoea.I suggest you use zinc supplements (Z&D drops 1ml once daily for 14 days) & ORS (Each small packet mixed in 200ml of potable water and keep giving sip by sip) as hydration is very important and crucial part of treatment. If there is vomiting you can use Syrup Ondansetron (as prescribed by your paediatrician).Regarding diet - Avoid fruit juices as they might aggravate diarrhea. You can give zinc supplements & ORS apart from normal vegetarian porridges & soups.Hope my answer was helpful for you. I am happy to help any time. Further clarifications and consultations on Health care magic are welcome. If you do not have any clarifications, you can close the discussion and rate the answer. Wish your kid good health.Dr. Sumanth MBBS., DCH., DNB (Paed).,"
},
{
"id": 102594,
"tgt": "What could cause a serious pain in the neck for an asthma patient?",
"src": "Patient: I have been bothered with a serious pain in my neck(right side)along with a pain in my right arm that feels like something is pulling the muscle in my arm(the pain is on inside of arm,above the elbow). I haven't been to a doctor for anything but the acute asthma that I've had a hard time regulating but I will have insurance beginning February. This has really been bothering me and I would like an answer please. Doctor: hi, pain in hand and asthma are unrelated. you may be having associated disc disease in your cervical spine that is causing impingement of nerves, causing pain in your hand. next time you visit your doc, ask for a neck x ray. An important suggestion- donot use pain killers with out consultation as they exacerbate your asthma. hope this helped."
},
{
"id": 87228,
"tgt": "Suggest medication for stomach pain when suffering from TB",
"src": "Patient: Hi. My dad had stomach pain Doctor prescribed taking AKT-4 for 3 months and after he was told to take R-cinex. He tool R-cinex for 2 months. Now wihtout informing any body he has stopped taking the tablets since month and stomach pain is back. What is cause, Now if he continues take the tablets will TB be healed or is it bad? Doctor: Hi! Good evening. I am Dr Shareef answering your query. If the diagnosis of tubercular abdomen was confirmed, then he should have completed the course of anti tubercular drugs as advised by her doctor. Extra pulmonary tuberculosis is notorious in taking much longer duration to heal and so ATT is prescribed for at least 9 months. Stoppage of medicines, that is leaving the treatment in half could make his disease relapse and also cause more damage to the intestines with adhesion formation for which he might be getting these pains. Therefore, I would advise you to consult your doctor again for getting your Dad properly investigated and treated.I hope this information would help you in discussing with your family physician/treating doctor in further management of your problem. Please do not hesitate to ask in case of any further doubts.Thanks for choosing health care magic to clear doubts on your health problems. I wish you an early recovery. Dr Shareef.I hope this information would help you in discussing with your family physician/treating doctor in further management of your problem. Please do not hesitate to ask in case of any further doubts.Thanks for choosing health care magic to clear doubts on your health problems. I wish you an early recovery. Dr Shareef."
},
{
"id": 136922,
"tgt": "Suggest treatment for severe calf pain",
"src": "Patient: What are symptoms of plaque build up in leg arteries.. My husband has pain in his calf muscle and it came on suddenly. He has issues with his feet and his gait has changed because of bunions. This is a long time situation. The Dr. Says the pain is caused by the plaque buildup in an artery. Is this possible? Doctor: Hello, I have studied your case.Yes There is possibility of atherosclerotic changes in lower limb blood vessels due to chronic disease.Your symptoms suggest possibility of Deep vein thrombosis also.Ultrasonography/colour Doppler leg will help to rule pathology leading to shrinking of muscle and swelling like deep vein thrombosis. If required you may need to do blood investigation for prognosis and recovery.I will advise to check your vit B12 and vit D3 level.Hope this answers your query. If you have additional questions or follow up queries then please do not hesitate in writing to us. I will be happy to answer your queries. Wishing you good health.Take care."
},
{
"id": 77594,
"tgt": "What causes severe cough and fever at night?",
"src": "Patient: My brothers wife age is 29 suffering from severe cough, fever at night and suspected for pneumonia and TB but x Ray did'nt show the result again we went to our family doctor and according to his suggestion we scan and he is saying that it is primary complex, Is there any chance of primary complex attacking her? her heamoglobin level is 10 Doctor: Hithere is a wide range of differential diagnosis for fever with cough. Your Sister in law is 29. any contact with TB patient ? Has she lost weight ?What is differential counts of CBC. ESR LDH beta microglobin .Liver function. CT chest perhaps will help. how long had she had this problem? She might need a bronchoscopy to wash her lungs for TB culture. Has she ever had raw milk ? this can give Brucela.Lymphoma is uncommon but can happen and give symptoms like you have said.There is no such thing as primary complex as diagnosis. it is a radiology and pathology term. Whether she has got TB or something else you to see a chest Physician. hope this helps."
},
{
"id": 158797,
"tgt": "Red lump on stomach, pain in testicles. Can it be cancer?",
"src": "Patient: Hello my name is frank I'm 23 years old. I have a red/purple lump on my stomach. Sticks out almost as big as a marble. It's like discolored with red and purple. Doesn't hurt at all. I can't really think of any side effects what's so ever. Maybe that when I sit down and drive for too long I have pain in my testicles but I'm have no idea if its due to the lump. I've had this lump for so long I'm just scared to see a doctor I'm afraid of what they might say. Like cancer or anything serious. Like I said its red and purple about the size of a marble. Any IDE what this could be? Doesn't hurt at all if I touch it or push it. It's been about 3 year I think I've had it. Thank you very much! Doctor: Hi and welcome to HCM. If this is there for 3 years thenit wont be malignant. It sounds to mi like haemangioma or any other subcutaneous lesion such as lipoma,fibroma or cyst.It can be surgically removed. Pain in testicles shouldnt be related to it.It can result from hernia or infection including STD. You can do ultrasounds of both to evaluate the exact diagnosis. WIsh you good health."
},
{
"id": 103352,
"tgt": "Suffering with sinus, feel jumpy and light headed with mucinex. Advised mucinex or robitussin. Suggestion?",
"src": "Patient: Just came from walk in where I was told I had a bad sinus infection . Will be taking an antibiotic but my question is when I take Mucinex it makes me very jumpy and light headed. I thought the dr said it was from the guaifenesin and to take mucinex or robitussin exportorant. It looks as though all the mucinex products contain guaifenesin. Did I misunderstand? Doctor: you understand goodbut sinus infections are all allergies and after neglecting milld allergies off and on you get infected sinussand you need to take antiallergic instead of bronchodilatorsif infected can take doxycycline 100 mg bd for 7 days off and on as hey are fast clearing of infectionstake anti allergic bd doseadd antacid and syp cpm and codeine combination one tsf nighttake flunarazine 5 mg night continue 3 wkbut get xray pns waers view and take ent doctorconsultation to cure it completely"
},
{
"id": 15850,
"tgt": "Skin rashes, persistent cough. Diagnosed as viral infection. Chest x ray suggests repeat procedure. Suggestions?",
"src": "Patient: Hi im having a skin rashes for 3 days and a persistent cough for two weeks in the absence of fever, i seek consulation yesterday and diagnostic exams was carried out, it appears that i have a viral infection but what really worries me, my chest xray impression suggest that i should repeat procedure but this time in apicolordotic view.. Is my rashes related to this? Doctor: Hi welcome to hcm.Thanks for writing here. Skin rashes appearing after any viral infections are known as post viral exanthem. There are number of viral infections where rashes appear .Most of them disappear within 1 to 2 weeks. You can apply calamine lotion over rash for soothing effects.Drink plenty of water ,Take bed rest. Second view of your X ray may be related to rule out any lung infections and start your drug treatment accordingly."
},
{
"id": 169693,
"tgt": "Suggest remedy for fever, abdominal pain and diarrhoea in a child",
"src": "Patient: Hi,sir/mam, My daughter is 4yrs old from Tuesday night she is suffering from fever we used Calpol 250mg from Wednesday she is suffering from stomach pain & motions we treated her with Doraloc powder she didn t got relief so on Friday we saw a doctor who prescribed Fevril-DS suspension & Nizonide for three days,now she is k with stomach pain but still getting fever when enquired he told us that it s a viral fever without doing blood test and asked us to continue both the medicines for next 4 days Is it a viral infection fever or shall we go for a blood test Doctor: ya,your doctor advised you properly,so don't get panicked when he has advised it is better to stick to him.ok"
},
{
"id": 175808,
"tgt": "Suggest treatment for staph infection on head and redness on face and head",
"src": "Patient: Hi, my baby has staph infection on his head and it would weep heavily. He was prescribed mupirocin and it worked after 3 days but the infection came back with a vengeance. He now has redness all over his head and face. He was switched to cephalexin and weve been on it for 5 days with very little progress. We were told to use petroleum jelly on his head for moisture but i feel like thats helping the infection more that get rid of it. Are we on the right track? Doctor: YOUR BABY IS ALLERGIC TO OIL APPLICATION(ALL) TO APPLY ON HIS HEAD/HAIR/BODY.THIS WILL CAUSE HIM ITCHING,REDNESS,BACTERIAL INFECTION ETC,FIRSTLY YOU WILL TO STOP USE OF OIL,AFTER START USING BETNOVATE-N OINT THREE TIMES A DAY,ALONG WITH CEPHELEXIN FOR 5 DAYS THEN SEE THE RESULTS OK"
},
{
"id": 40408,
"tgt": "Treatment for glandular fever, overweight, tiredness?",
"src": "Patient: I have two straines of glandular fever, over weight and get tired easily. Found out I have a high level of allergy to certain grasses. Also found out the cancer runs in the family. My mum mum died with pancrease cancer, my dad died of aggressive cancer which started as a turmour in the bial duct, an uncle with pancrease cancer and an uncle with prostate cancer. Unable to loose weight, I eat right and exercise and hour per day and my house has 14 steps. Doctor: Hi,Welcome to HCM.I understand your concern regarding losing weight and staying healthy. Glandular fever is caused by a virus which usually is self limiting but this virus has the tendency to stay dormant in the body and can get reactivated when the persons immunity is low. You should target at keeping stress levels to minimum and take healthy diet to maintain a strong immune system. Exercise definitely will help. You have to calculate the number of calories you can eat per day and start a regular exercise which can be done everyday. Please do not expect to lose weight suddenly, slow weight loss is easier to maintain later. Weight loss of half a kg per week is ideal. Predisposition to cancer in the family may increase the risk of cancer for you. Routine health check ups will help you to deal with that. Allergy triggers have to be identified and avoided.Thanks."
},
{
"id": 182159,
"tgt": "Suggest treatment for itchy palate",
"src": "Patient: For the last two days the inside of my mouth has been itching. It started on upper roof of mouth near the hard palate I have not eating anything hot or spicy or any citric fruits. Now I'm having a issue with my tongue irritating and my upper lips is very chapped it looks like a rash but not signs of sores. What could it be? Doctor: Hello,You may have an oral fungal infection. Try some acidophilus and continue to avoid acidic and spicy foods. Rinse your mouth with warm salt water to soothe and cleanse. If you do not get relief or feel your condition is worse, see your dentist for an evaluation. A prescription anti fungal medication, Nystatin, may be required to treat a yeast infection. Other possibilities include viral infections will take time to heal. You should continue to avoid all irritants including smoking and alcohol.Consider your sleeping habits and congestion causing you to breath through your mouth. Any sinus concerns? This can dry out you palate and cause dry lips. Keep well hydrated.Any recent food changes? An allergic response could be responsible for this condition. Antihistamines can offer relief. Itching can be treated with Benadryl. I hope that my suggestions are helpful. Thank you for your inquiry."
},
{
"id": 25660,
"tgt": "How to treat chest tightness?",
"src": "Patient: Hi there, recently I have felt tight in my chest (a bit short of breath) and my eyes are a bit sore. I occasionally have a headache and sometimes my joints are a bit sore as well.I'm not in a lit of discomfort, and don't have a fever - but am a bit concerned as my chest has now felt like this for a couple of weeks.Do the symptoms sound familiar to you? Doctor: Hello!Welcome and thank you for asking on HCM!Regarding your concern, I would explain that it is important perform a comprehensive differential diagnosis of different causes that may lead to this clinical scenario: - A viral infection, could cause breathing problems and sore eyes. - An inflammatory disorder could also lead to these symptoms. I recommend consulting with your GP for a careful physical exam and some tests: - a resting ECG- a chest X ray study- some blood lab tests (complete blood count, PCR, sedimentation rate, kidney and liver function tests, fasting glucose, blood electrolytes including calcium plasma levels). You should discuss with your doctor on the above issues. Hope you will find this answer helpful!Greetings!Dr. Iliri"
},
{
"id": 202490,
"tgt": "Is there a cure for foreskin attached to penis head?",
"src": "Patient: Hi, This is abhi 23 years old and i am working as a software engineer in a private company in bangalore. I have a problem with my pennies, as it gets exited the pennies head is not coming out the fore skin is this a problem whether this will affect while having sex with my girl friend please help me out. n we i reverse my foreskin then i can see a small joint which is attached to pennies head. can it be removed without surgery??? Doctor: THE CONDITION YOU ARE DESCRIBING IS CALLED PHYMOSIS MEANING INABILITY TO RETRACT THE FORESKIN. THIS NOT ONLY WILL LEAD TO PAINFUL ERRECTION AND PAINFUF INTERCOURES BUT WHAT IS MORE IMPORTANT IS THAT SINCE YOU CANNOT RETRACT IT YOU WILL NOT BE ABLE TO MAINTAIN GOOD PERSONEL HYGIENE. YOU THEN WILL BE MORE PRONE TO RECURREND INFECTION OF GLANS PENIS CALLED BALANITIS. THE PERMANENT CURE AT YOUR AGE IS SURGERY- CIRCUMSICION. WISIT A SURGEON FOR THIS."
},
{
"id": 158953,
"tgt": "Have lower abdominal pain, period like cramps, vaginal pain. Ovarian cancer in family history. Help?",
"src": "Patient: My lower abdominal on my left side hurts , but im realizing the pain is starting to spread. I also realize that when i push down on my stomach it hurts and somtimes i feel a sharp pain in my chest. Its so weird because when i get my period my cramps are horrible and my vagina gets this stabing pain. (im a virgin so i know its no std ) please help. ovarian cancer runs in my family so im pretty scared. Doctor: Hello! Thank you for the query. Lower left abdominal pain can be caused by diverticulitis, urinary tract infection, ovarian cyst. Ovarian cancer can also give such symptom as well as ovarian cyst. Stomach pain seems to be caused by stomach disease like peptic ulcers or acid reflux disease. Sharp pain in the chest can appear due to rib nerves irritation. But most likely your stomach pain and sharp chest pain is caused by stress (as you have admitted to be scared about cancer, this can trigger such symptoms). If an ovarian cancer runs in your family (and maybe breast cancer?) you should consult oncologist as some of such hereditary cancers are caused by genes mutations. You should have checked BRCA1/BRCA2 mutations. Ca-125 level and transvaginal ultrasound should be also performed. Lower abdominal pain should be consulted with gastroenterologist and some blood tests and urine tests should be performed. Hope this will help. Regards."
},
{
"id": 87709,
"tgt": "What causes abdomen pain, bloating and shoulder pain?",
"src": "Patient: Pain in lower left side of abdomen. Also in upper left abdomen radiating to side and back. Bloating/pain/diarrhea after eating. Recently have had several bouts of left shoulder pain which radiates along what feels like a bone that goes from shoulder over to chest just underneath neck. Doctor: Hi.Thanks for your query..Your history of radiation of the pain to the should and the neck has first to be checked by an urgent ECG and cardiac work-up. The history of pain in the left side of the abdomen with loose motions, bloating and pain radiating to the sides and back is suggestive of severe colitis of the left colon. I would advise you the following:Take a course of an antibiotic, metronidazole and probiotic with activated charcoal and symptomatic treatment. If required undergo the test of blood, stool and urine , a colonoscopy and biopsy and CT scan of the abdomen. and get the further treatment as p[er the reports."
},
{
"id": 215466,
"tgt": "Why is Oxycodone prescribed in place of Oxycontin while suffering from fibromyalgia?",
"src": "Patient: I have MS, fibromyalgia and degenerative disc disease. my doctor had me on OxyContin 60 mg a day, twice a day, morning and night. It worked wonders. he also put me on, 120mg Cymbalta which helped with the neurological pain and 15mg Dexedrine ER a day for energy due to a genetic disorder called Beta thalassemia minor, which makes my hemoglobin run from 8 to 10 on any given day. I actually felt normal for in my life. As child I was diagnosed with low so I had to get painful iron weekly. I was terribly sick until I was finally given the right diagnosis. I could keep up with the best of them. (Well, the middle of them) Recently, my doctor has taken off OxyContin and put me on 40mg a day of oxycodone. He no longer gives me Dexedrine so my energy is down again. I have asked for blood to help, and smaller dosage of Dexedrine and he says he won\u2019t prescribe that anymore. I asked him why he cut the Dexedrine and lowered me to oxycodone and his answer was vague and a little strange. I have been seeing him for several years and I have always passed any drug given. Please help me understand what is going on in the medical field or advice my to seek another doctor thank you for help. Sincerely, Edyee Doctor: Hello, Both belong to the same family of opioid drugs. You can safely use both the drugs interchangeably. Hope I have answered your query. Let me know if I can assist you further. Take care Regards, Dr Shinas Hussain, General & Family Physician"
},
{
"id": 128685,
"tgt": "How to treat the hard lump caused by a fall?",
"src": "Patient: hi, I fell down the stairs 2 weeks ago and hurt my leg, the way I fell I landed on my shin, there was a bruise but now that its gone there's a small hard lump under the skin about the size of a pea. its sore to touch, and I still feel pain while I'm walking. do you know what this could be ? Doctor: hi I understand your situationyou got hard painful lump after traumaDiagnosis:its most probably a sub-periosteal hematoma formation due to direct trauma to your shin then gradually became harden due to fibroses.it will be painful due to movement and touchingTreatment:you can let it and apply gentle pressure over it with taking topical or oral painkillersif no benefit you can excised by surgery by specialist."
},
{
"id": 105076,
"tgt": "Which supplement can be given to asthma patients to help gain weight?",
"src": "Patient: hi i Harsh i just want to know that which suppliment we can give to asthma patient for weight gaining actually my mother is having asthma but now a days she is having Aleovera juice and she better felling from last three months but her weight is getting reduce so plz tell me any diet for food suppliment for her Thanks Doctor: To gain weight she has to take good nourishing diet especially rich in proteins. If she is a non vegeterian she can take eggs, chicken and fish regularly. If vegeterian she may take milk and milk products like cheese, curd, paneer etc. You may give her protein drinks which are available in the market. As such there are no drugs prescribed for gaining weight. She can take a vitamin supplement that may improve her appetite."
},
{
"id": 149920,
"tgt": "Suffer from chronic pain syndrome due to neck injury. d\\Developed neuralgia and trigeminal neuralgia, microvascular decompression. Any help?",
"src": "Patient: hi, I am a 69 y.o. woman suffering from chronic pain syndrome ,d/t aneck injury. I later developed occipital fioroset bid and hydrocortisoneand levothyroxine ,zoloft 50 ml neuralgia and trigeminal neuralgia . Had a microvascular decompression of my trigem. nerve . The surgeon also cut my ociptal nerve whaich has put me in chronic 8/9 pain. I am also in adrenal failure have thoracic outlet syndrome and a rt. knee torn meniscus from falls. nothing is more frustrating than my faltering speech. I am taking dilaudid p.o. and by pain pump,ativan1mg b.i.d. Doctor: Hi,Thank you for posting your query.It is unfortunate that you are suffering from chronic neuropathic pain due to multiple problems.Medications, which can help in relieving this pain, would include pregabalin, gabapentin or duloxetine.You can get them prescribed by a local doctor.Best wishes,Dr Sudhir Kumar MD DM (Neurology)Senior Consultant Neurologist"
},
{
"id": 205010,
"tgt": "What causes ADHD?",
"src": "Patient: My son was diagnosed with ADHD 3 years ago and during our reevaluation this summer the neuro recommended an EEG to rule out any seizures. He had a 45 minute one in the doctors office and all came back fine and then had a 48 hour one and results came back with some abnormality during sleep. He is having an MRI to check for nocturnal seizures and to check structure to see what may cause the abnormality. What are examples of structure problems if he had any Doctor: HaiAny Psychiatric diagnosis is made after medical or neurological causes are ruled out.If seizure or other organic cause is found ,treatment of that is the primary importanceADHD =cause : multifactorial -- Genetic,antenatal /birth/post natal insult .... thank you"
},
{
"id": 115987,
"tgt": "What does CBC smear showing leukocytosis & neutrophilia suggest?",
"src": "Patient: I have had an elevated wbc count 11-15 for at least 5 years. My platelets typically run 350-415. My doctor recently ordered a CBC smear and the results said leukocytosis and neutrophilia. There were no immature cells noted. She doesn t feel like it is a problem, but is sending me on to see a Hematologist. Should I be more concerned? Doctor: Hello and welcome to HCM,An elevated white blood cell counts occurs in case of infection.And amongst white blood cells, the neutrophils are increased.Thus, high neutrophils suggests bacterial infection.You need to consult your primary healthcare provider for clinical assessment and treatment of bacterial infection.The focus of bacterial infection has also to be found out.Thanks and take careDr Shailja P Wahal"
},
{
"id": 78812,
"tgt": "What causes consolidation in my right lung?",
"src": "Patient: I began to feel unwell in May, my doctor missed pneumonia and I was only diagnosed when I went to hospital in June, I now have consolidation in my right lung and it is not clearing up, is this a sign of lung cancer? I am a 34 year old normally healthy female. Doctor: Thanks for your question on Health Care Magic. I can understand your situation and problem. Possibility of non resolving pneumonia is more in your case. At your age lung cancer is very rare. But in NY opinion, you should get done bronchoscopic evaluation with BAL (bronchoalveolar lavage) analysis and biopsy. These will diagnose cancer if present. It will also isolate the causative organism for pneumonia. And so we can decide optimal, appropriate antibiotic treatment. Also rule out tuberculosis and fungal infection in your case. So consult pulmonologist and discuss all these. Hope I have solved your query.Wish you good health. Thanks."
},
{
"id": 100994,
"tgt": "What could cause pain below the right breast & that goes to the back?",
"src": "Patient: Hi, my problem is, I have pain below my right breast that goes to my back, I had bronchitis last winter with 2 anti-biotic prescriptions. The pain is still there. When I sneeze I have to hold my side because the pain is bad. I had a scan test done and today my doctor gave me an appointment to see a specialist and needs to have another test done, they will insert a tube that will go down to my lungs with a light and camera. Doctor, what is wrong with me ? Doctor: hi thanks for your query.the doctors want to know if the bronchitis infection of yours has gone lower into the lungs forming an abcess.this condition can cause painful cough.this investigation is called as bronchoscopy.it can reliably diagnose chest and lung infections.the pain isnt related to your breast.consult your doctor in case of further queries.thank you"
},
{
"id": 59310,
"tgt": "Diagnosed chronic infection of Hep-B with low viral load, suspicious coarse in USG. On Tenvir daily. Does coarse mean cirrhosis ?",
"src": "Patient: My name is AAAAA , residing in Mumbai.In the year 2010, it was diagnosed chronic infection of Hep-B with low viral load=457IU/ML and in USG , it was found suspicious coarse also.so on that facts my Dr.started treatment and prescribed antiviral called 1 dose of TENVIR-300 daily basis. After 2 consecutive years my all reports covering VL, ALT ,AST,cretenine,alpha feto etc..normal but suspect of coarse is still been identified.I am in yearly basis routine test and current reports are normal. She is recommending tenvir-300 -antiviral continue only on the suspect of coarse. Is coarse mean cirrhosis? is cirrhosis reversible or treatable? Regards AAAAA 0000 Doctor: Hi and welcome to HCM. It can mean liver damage at any stage. Cirrhoiss is one of the worst consequence of hepatitis but it goues through phases and develops during many years. First stage is fatty live then fibrosis and then cirrhosis. If your liver enzymes are fine then this probably isnt cirrhosis,you just have minor liver damage which is inevitable in hepatiits. Enzymes would be more than 10x elevated in cirrhosis and you would have some serious symptoms. Fatty liver and fibrosis are almost completely reversible, cirrhossis is ireversible,but even small healthy part of liver can compensate its function so there is a long way from hepatitis to failure. YOu need to do frequent enzymes check ups and ultrasound to follow all these and everything will be fine. WIsh you good health"
},
{
"id": 80907,
"tgt": "Does Foracort inhaler taken daily as a treatment for breathing problem cause bitterness ?",
"src": "Patient: Hi, My daughter , 9 years old, getting treatment for breathing problems. she is taking foracort 100 inhaler daily. she is telling that the new foracort 100 which i bought after the present inhaler is over is more bitter than the old one. shall i discontinue it and buy a new one Problem with medicine or in her health conditions. pl advise Doctor: No usually it doesnt causes bitterness. If possible take this inhaler with spacer device as it deliver desired amount of drug and dont forget to rinse mouth after taking inhaler as some of drug might get deposited in throat and after long term us ecan cause problem."
},
{
"id": 125949,
"tgt": "What prognosis is recommended for walking disability caused by nerve damage in the feet?",
"src": "Patient: We are looking for a dr. In the Mpls. Minnesota area or in Burnsville Mn. That would specialize in illness due to mold exposure. My son has been working in a toxic invironment and has developed many of the symptoms of mold exposure and can no longer work for this company. A claim has been filed with OSHA because he has been feeling so ill. Doctor: Hi, It takes a very long time for complete recovery. Better to consult a physiatrist (a doctor specialized in pain and rehab) for further assessment. Hope I have answered your query. Let me know if I can assist you further. Regards, Dr. Shinas Hussain, General & Family Physician"
},
{
"id": 118539,
"tgt": "Low platelet count after fever. Urinalysis shows high RBC in urine. Is it related to fever?",
"src": "Patient: hi, my mother have fever last week for 3 days as, i check her cbc and platelet after the fever it shows that her platelet it is is slowly going lower and lower... since my mother don't want to be admitted to any hospital, and she dont have any problems after the fever, all i do is watch over constantly checking her platelet to make sure it don't go dangerously lower lever... i advise her to drink plenty of water and eat a lot and after 3 more days her platelet have gone up but her urinalysis shows that she have rbc 20-22 in her urine... is it related to her fever? and is there something i could do? Doctor: Hello dearThanks for your query at HCM.Low platelet counts and high rbc count in urine are sequelae of the infection in the body. You need to immediately consult doctor if platelet counts are dropping.Repeat urine analysis after 1 week of subsiding infection even then it shows rbc in urine further investigation might be necesssary.regards."
},
{
"id": 60046,
"tgt": "Had colic jaundice and infection. Stones in bile duct. ERCP planned. Is it safe to wait till then?",
"src": "Patient: i have just had bilary colic jaundice and infection had ercp but woke half way through procedure and could not cope very sore lack of appertite pain between shoulders known stones in bile duct ercp re planned under general but cancelled due to other emergencies at hospital still in pain what should i do mcp planned two weeks time dont know if i can survived until then Doctor: Hello. Thanks for writing to us. An acute pain due to impacted calculus in CBD is often unbearable. It usually needs an indoor treatment with fluids and intravenous antibiotics. You need to consult your physician for proper treatment. I hope this information has been both informative and helpful for you. Regards, Dr. Rakhi Tayal drrakhitayal@gmail.com"
},
{
"id": 145219,
"tgt": "Suggest treatment for myasthenia gravies",
"src": "Patient: My father age is 67 years old. Last month , he had dedeceted myasthenia gravies . Dr started the medicine myestin 60 , one tab three times a day. My father has 10% relif. Now dr suggest to start a new medicine azoran 50 g. But this have some side effects . Blood test report is _ ACHR ANTIBODIES , ACETYL CHOLINE RECEPTOR ANTIBODIES _ 20.1. Nmol/l please suggest me.what can I do??? Doctor: Hello ! Than you for the question on HCM!Regarding your concern I would say that his antibodies are really high . The small effect of myestin, is very indicative for a possible progression of the disease. This disease can lead to respiratory insufficiency, causing high morbidity and mortality. So I agree with your doctor to have a more aggressive treatment. We should put into balance the side effects of this drug and the benefits, taking into consideration the severity or this disease. Another option would be steroids. But they are contraindicated in hypertension, diabetes or gastritis and ulcers . I don't know if your father has any contraindications to oral steroids. I would also recommend a thoracic CT scan for thymoma, or thymic hyperplasia. This high level of antibody may be related with this disorder. Surgery would be very helpful in that case. Hope to have been helpful!Best wishes, Dr. Abaz Quka"
},
{
"id": 153478,
"tgt": "What are the signs and symptoms of colon cancer?",
"src": "Patient: I have a friend who is a 40 year old male who was diagnosed last year with Stage 3 Colon Cancer. He had surgery to remove the cancerous tumor and is currently undergoing chemotherapy. When his doctor removed the tumor he found 3 out of 16 lymph nodes testing positive. For over three years he complained to his doctor about symptoms and wanted further testing done but his doctor kept telling him it was not necessary and it was due to having a fatty liver and being over weight. He went with a different doctor last year and the doctor had a CT scan done which detected the cancer. We just want some insight and to know if his cancer was detected three years ago would it have been at an earlier stage and not effecting his lymph nodes. Doctor: Hi,Thanks for writing in.Colon cancer even after treatment with surgery and chemotherapy, might spread to liver in half of the patients. This is confirmed by many studies and therefore a close follow up is suggested in every patient with colon cancer even on treatment.Please get regular ultrasound scan abdomen done and CT scan abdomen might also be done less frequently. It is possible that the cancer grew over tha last 6 months before detection. To know the appearance of the cancer, it is important to know the CT scan report. Though it is less likely to be detectable on CT san done 3 years back. If the cancer involves a large segment with obvious wall occlusion then a CT scan done earlier might have given a clue. Please do not worry."
},
{
"id": 93433,
"tgt": "Have severe abdomen pain and don't seem to orgasm during masturbation. What to do?",
"src": "Patient: I am a woman in her late 20's experiencing two problems: 1.) I experience severe lower abdomen pain when I'm aroused and do not get relieved, I double up and cannot get comfortable and often feel what is a 10 on the pain chart: what could be causing that? 2.) Masturbation to attempt to relieve this doesn't go anywhere, I cannot seem to orgasm during masturbation at all, so I just grew a distaste for it in the end and am still stuck in pain. I have no idea what to do. Doctor: I DONT KNOW YOUR HISTORY ACCORDING TO WRITTEN SYMPTOMPS THINK OF ANY MEDICINE TOOK FOR ANY OTHER DISEASEPCM BRUFEN DIETRY SUPPLIMENTS VITAMINS OR MULTIVITAMINSANY CHANGE OF COOKING OILTHERE CAN BE FOOD ITEM CAUSING THISGET BLOOD SERUM TEST FOR FOODS IF CAUSING AS AT ANY AGE AT AY TIME ANY COMMON FOOD LIKE ILK WHEAT OR OTHER START CAUSING SYMPTOMPS GET DIAGNOSED AND TREAT ACCORDING TO DIAGNOSISIN MEAN TIME TAKE LOT OF WARM WTER NON OI;Y NON FAT NON SPICY FOODTAKE GELUSIL 2 TSF TDSMETROGYL 200 MG FOR 7-10 DAYS FOR INFECTION IF ANY BUT MUST TAKEMEFTAL SOS FOR PAIN"
},
{
"id": 207179,
"tgt": "Suggest treatment for depression and to increase concentration",
"src": "Patient: Dear Sir, i'm feeling tired all the time, i'm feeling depressed, i can't concentrate i got married two and a half years ago. after marriage i gained 6 kg and when i got pregnant i gained 24 kg. my girl now is one year old. my Weight movement: i lost 16 kg using protein diet in 10 weeks - i lost 4 kg by a nutritionist diet and having sports 3 times aweek - i still have 4 kg but i can't control my food anymore, i don't have energy to do sports. my life: -i was working and my income was more than great but my parents needed most of my salary, after getting married i stopped working full time job i'm working partial time job and i get payed once a year and give all the sum to my parents since my husband isn't rich but have his medium society and has a gooood income - i'm not happy in marriage because i feel that i need to beg love from him and remind him that i exist. he works from 8 till 8 either when he comes back he sleeps or go to play card with his friends, all we talk about is asking me to prepare him sometimes dinner, he is kind but we rarely talk - there is 12 years between me and my husband i'm 26 he is 38 when i knew him he was alot younger in shape and spirit, now i'm feeling the difference and he is feeling that he is getting close to 40 without saying it i know what he is thinking about, we go out once a month with his friends (i'm not enjoying being with his friends anymore they are all arround his age, and make me feel old though they are very kind and when we go we go for dinning and dancing but i need this crazy feeling of being young) - sex is the worst thing in my life, as i mentioned i feel i'm begging for some tender kiss or hug, most of time he is tired or he feels that he ate too much, when i get sad he calls me for sleeping next to him and hugging me, when he is at home he sleeps in front of TV, and when i ask him to sleep in bed next to me most of the time he shouts, and still i insist to call him, because i need this hug before sleeping. - when having sex i get satisfied but like a robot i have to receive and can't give, i other words i'm not allowed to touch him or suck his penny, or kiss any part oh his body unless a frensh kiss and touch his \"boobs\" to help him getting his back... - he doesn't care if i was waring sexy or not he finds me beautifull when in a party every body complemets me, and when i pass infront of someone he can't takeof his eyes on me, it starts to give me a good feeling when a whole table looks at me when i pass (i know i'm beatifull and he married me because of being it + later on he found that i'm clever and well educated and i have the character of the girl he wishes to live with) - even when i prepare food, many ask me the recipe then he tells me that my food is good, if we were alone either he says it is regular or he says it is simple to cook and what i did is almost nothing. and since i'm always on diet i can't try what i do! - to forget and not over-think i thought to continue studying, when i achieve this year, next year i'll start preparing for the PHd, (in computer engineering) i will be havin in 10 days to deliver a 30 pages report + some tasks in a common project + 2 exams, there is still alot to do in the report, i'm having some problems with the team in the project tasks, i can't study for exams unless at night when my daughter sleeps because she's full of energy. - i don't sleep well, i send my girl to my mother in low the morning either to study or to go to my work, and i get her around 14:00 to go to my mother i have my lunch at my mother's place she is so demanding, i hate going to her because she makes me feel tentionned, since if anyone arround the world do anything with their husbands she wants me to apply it with my husband. and usually i don't accept any interference in my personal life so we end by a small fight and i'm always wrong, and if i don't go to her she gets sad and i don't like to make her sad. (knowing that i always had problems with her but it ends up with satisfiying her) and i can't even tell her that i'm having an unhappy life because they were against my marriage because my husband and i, we are from two different religions. besides she feels lower than my husband, and she thinks that he is arrogant, when the problem is that he has communication problem, and he is not that much richer than she is but se under estimates her sitution. a new complaint to make pressure on me - i'm neglecting my house, i can't kleen it the way i want - i forgot how the shopping in life is - i feel that i'm not giving enough care to my girl - i can't concentrate or study anymore - i feel exhausted - i need help pleaaaaaaaaaaaase help Doctor: Hi, thanks for the query;you indeed have a long list of problems & seems like not only you but also your husband is also suffering from depression or some form of stress related psychiatric disorder. First & foremost you 2 need to get together & consult your GP/Psychiatrist about these problems so that he/she can refer you to a Psychologist/counselor for systematic discussion & solution finding for these problems. This type of counselling is popularly known as family therapy or marital therapy. In addition you two also need medications to help improve your mood, your tiredness, sleeping difficulties & to boost your sexual life. Many antidepressants like bupropion, agomelatine, SSRI drugs (depending on your clinical profile) can help you with these. Hence Consult your Psychiatrist ASAP or get back to me for further assistance Hope this helps; good luck"
},
{
"id": 14745,
"tgt": "How can rashes and bug bites on skin be treated?",
"src": "Patient: hi yes my daughter has raised blotches on her skin some look like a rash and some look like a bug bite. she has not had any abnormal food that would maybe cause this and she did play ourside yesterday but not really in a high wooded area or by the water Doctor: Hello,Thank you for posting on HCM.The condition you have described is called Papular urticara, which is an allergic manifestation to insect bites.Apply a cream containing hydrocortisone and fusidic acid twice a day.Soothing calamine lotion will also help additionally.Take antihistaminics like Benadryl as required.Hope this will take care of your issue.Wish you best of health.Thank youDr Hardik PitrodaM.D Dermatology"
},
{
"id": 128794,
"tgt": "What causes recurrent scabs on the lumbar area and right-sided leg and knee pain?",
"src": "Patient: I have a small linear rash. Much like a scab at the base of my spine. It is about an inch long and 1/16 of an inch wide. It is reoccurring. When it occurs I have pain in my leg, right side just below the knee area. The pain is similar to a muscle cramp, but there is no cramping. It hurts even to touch it. Doctor: Hello,It sounds like you may be having shingles. If so there are medicines that may help with the pain. See your doctor when you have the rash for an opinion.Regards"
},
{
"id": 146432,
"tgt": "What causes dizziness on standing up?",
"src": "Patient: Practically every time I stand up from either semi laying down, laying down or sometimes from even just sitting... I completely black out, close to passing out, I have to completely stop and wait until it passes as I cannot see a thing and my legs get weak.. Some of the time for about 10 seconds as it s easing and stopping I have troubles remembering where I am or what I was doing ... I do also have troubles breathing sometimes that I have been advised is due to anxiety... Which causes me to feel as though I have a tight chest So I am constantly trying to take deep heavy breaths..which also may affect the dizziness, because of too much oxygen going to my head? A lot of Mosquitos live around my house and obviously they all find their way into my house, do any of these symptoms indicate the Ross River Virus?? Help! Doctor: Hello dear,First of all, please do not get concerned with Ross River Virus disease, it presents with a whole lot of different symptomatology.The symptoms as mentioned in your post can be attributed to pathology in the Vestibular Apparatus in inner ear (it is associated with maintenance of body posture & balance) most probably Labyrinthitis or Benign Paroxysmal Positional Vertigo.Symptomatic relief can be obtained with intake of Vestibular sedatives like Betahistine or Cinnarizine preparations (to be taken only under the guidance of a Physician).So, kindly consult your Physician/ ENT Specialist & get a complete clinical examination done.Investigations like complete blood count, estimation of serum electrolytes & vestibular function tests will be required to rule out any pathological cause for the symptoms.There is no need to worry, you will be fine.Till then, maintain adequate hydration & proper nutrition status and avoid stress.And also take precautions like getting up from bed slowly & avoid sudden head movements.Wishing you a Good Health.Take care."
},
{
"id": 189196,
"tgt": "Pain in teeth, lower lips is three times size. got cavities filled. Taking medication. Anything to worry?",
"src": "Patient: Hello I recently went to the dentist to get a few cavities filled. After the visit i noticed that the pain in my one tooth was increasing. Yesterday I phoned my dentist at his residence and he prescribed me metronidazole 3x a day. I also have been taking Tylenol 3 and oragel ..when I woke this Morning my lower lip is three times its size. So my question is do i have to be concerned or should i continue with the medication the doctor prescribed? Doctor: hi there thanks for asking your query pain and swelling after filling of a tooth means the cavity was deep reaching pulp,so that tooth needs a root canal treatment deep cavities should not be filled as the lesion progresses deep in canals so it needs to be expirated cleaned and then filled with inert material i will suggest you to consult a dentist and get an xray done of the tooth to see extend of lesion mean while take antibiotic augmentin 625mg three times/day along with metronidazole 400mg two times/day for a week"
},
{
"id": 83016,
"tgt": "Have symptoms similar to SLE, ANA weakly positive. Symptoms persist, with new ones",
"src": "Patient: I have been having many symptoms that are similar, if not dead on to those of SLE. I had a positive ANA but when they did more blood work they said it was a weak postive and that was it. No more tests nothing, but still the same symptoms persist a year and half later. They were better for about a 5 month period but now starting again with new ones on top of the old symptoms. Doctor: you probably need a repeat ANA by immunofluorescence . that will give us a definite answer about your SLE. ANA weakly positive means that it has been done by elisa which is not the recommended method to do the test. what other symptoms do you have. do you have photosensitivity,skin rash, oral ulcers, ever had pleural effusion, CNS or renal symptoms-these are the symptoms of SLE."
},
{
"id": 183551,
"tgt": "Suggest remedy for pain caused after a molar extraction",
"src": "Patient: I had my right, second molar from the back extracted 3 days ago. I went back yesterday & they flushed it with disinfecting mouth rinse, and packed it with something that taste like cloves. Can I use Peroxyl mouth rinse or.doni need to wait until it heals? It has been hurting this morning & staggering 800mg ibuprofen and Tylenol isn't helping Doctor: Hello,You are describing a delayed healing from the extraction. You may be familar with the term dry socket. I recommend that you take a prescription antibiotic. Call your dentist to inquire if you are not taking an antibiotic at this time. You should contact your dentist and report your symptoms. Try to take two Ibuprofen and two Tylenol at the same time. Keep taking at regular intervals to control pain. You may need to return to your dentist for packing additional medication in the socket. Follow all instructions provided by your dentist closely. You should not rinse after packing has been placed in the socket until directed by your dentist. Usually this will be the next day. Rinsing with warm salt water or peroxyl will assist healing. Keep the ara clean and do not let any debris get caught in the socket.Thank you for your inquiry. I hope you feel better and have improvement with the healing soon."
},
{
"id": 23362,
"tgt": "Suggest exercises for heart ailments",
"src": "Patient: I am 57 year man. 3 months back two stents installed because of double vessel heart desease. I have my BP in range of 140-90. I am on prasugrel 10, ecosporin150, avastro, losartan 50, betaloc25. I don't feel much problem except some pain in left hand. Not daily. I walk for 45-50 minutes and feel much better. What type of more excersise/yogasan and breathing excercise can i do? Doctor: HI 3 months after stenting after a myocardial infarction and 1 month after a planned stenting you are free to do any exercise you want you can live a perfectly normal life ,you are free to do every exercise apart from lifting weights more then 15 kg,infact a strenous exercise like brisk walking at a rate not less then 5km/hr a day for atleast 6 days a week should be done and is recommended ,you walk daily but this walk will be effective only when it is brisk in nature ,practically speaking you should be sweating after 45 minutesbreathing exercises like Anulom vilom and bhastika should be done daily for 15 minutesone thing to remember is that do not over do any exercise ...start slowly and increase gradually thanks"
},
{
"id": 195018,
"tgt": "What do warts on the penis indicate?",
"src": "Patient: I have noticed small wart like lumps on my pubic lower shaft of penis region they dont seem inflamed i hit 1 with a razor and it bleed like crazy its how i 1st noticed they were there a couple have cleared but more have arrived only noticed in last few weeks help thanks Doctor: Hi, It\u2019s a genital wart. You can consult a dermatologist and get it removed either by cauterisation cryosurgery. Nothing much to worry as it is a common and harmless condition that can easily be cured. Hope I have answered your query. Let me know if I can assist you further. Regards, Dr Shinas Hussain, General & Family Physician"
},
{
"id": 3244,
"tgt": "Am i pregnant if pregnancy test is faintly positive?",
"src": "Patient: Hi, 2 days ago I did a home pregnancy test as I was having symptoms. It came up with a faint but clear positive. I took another this morning to be sure and again it came up a faint positive but took a bit longer for results to be clear. Am I pregnant or could the test be wrong. Doctor: Hello dearI understand your concernFaint positive pregnancy could be due to1. Done very early in the pregnancy2. Done with diluted urine and Failure to follow instructions properly while doing test3. Chemical pregnancy/miscarriage4. Positive pregnancy test as some time pregnancy test will give false negative testPregnancy test will give positive result 8-10 days after the missed period.I would suggest to go for blood HCG test after consulting the gynecologist for final confirmation of the pregnancy.Blood HCG is more accurate test and give positive result only 10 days after the unprotected sex.Meanwhile avoid stress, take healthy diet, drink plenty of waterHope this may help youBest regardsDr. Sagar"
},
{
"id": 72333,
"tgt": "What causes shortness of breath followed by headaches?",
"src": "Patient: i am currently experiencing breathing issues. the shortness of breath gets better and worse as i swivel my back and keep my hips in place. it also gets bad when i lay on my LEFT side. It feels as though the air in my lungs is escaping. i also have a tight pain in my chest and have been burping non stop. what could cause this? i also wake up with strong headaches which have been getting worse. should i go into the ER? Doctor: Thanks for your question on Healthcare Magic.I can understand your concern. Yes, you should definitely go to Emergency room and rule out hypertension and other heart diseases.You should get done blood pressure monitoring, ecg, 2d echo and stress test (trade mill test).If all these are normal then no need to worry for heart diseases. Sometimes stress and anxiety can also cause similar symptoms. So avoid stress and tension, be relax and calm. Consult psychiatrist and get done counselling sessions. Try to identify stressor in your life and start working on its solution. You may need anxiolytic drugs (propranolol and flunarizine combination) too.Don't worry, you will be alright with all these. Hope I have solved your query. I will be happy to help you further. Wish you good health. Thanks."
},
{
"id": 16964,
"tgt": "What does my EKG report indicate?",
"src": "Patient: Hi! im46 yrs old and i am latina. My concern is that I took an EKG and my results where that. I have sinus rhythm (slow) left axis deviation consistent with LAFB RBBB QRS= 145 ms RSR in V1 S 30 ms in 1v5 v6 Borderline ECG should i be concern? I weight 135 pounds Please let me know concern. Thank You! Doctor: Hello, What is the level of your blood pressure? It should be under control as high blood pressure is one of the causes of left axis deviation. Hope I have answered your query. Let me know if I can assist you further. Take care Regards, Dr Varinder Joshi , General & Family Physician"
},
{
"id": 88201,
"tgt": "What causes left side abdominal pain after childbirth?",
"src": "Patient: My wife gets these bad abdominal pains on her left side and in her back ever since child birth. They don't happen all the time, but whe it does she's bent over in pain. She's gone to the er a couple of times but all tests come back inconclusive. So now she's afraid to go to the doctor. Please Help!!! Doctor: Hi.Thanks for your query. The problem of pain in abdomen after a childbirth can be a serious problem.You have not mentioned whether this was a normal delivery or by a Caeserian Section (operation).If going to ER is not helping Her, I would suggest you the following:Consult a Gynecologist or a General Surgeon for an internal / per-vaginal examination. Clinical examination is very important as it gives the information that no tests can give. Go for the investigations as suggested , which may be :Ultrasonography of the abdomenColonoscopyCT scan of the abdomen if there are positive findings on USG or need further evaluation.Psychological evaluation is there is a chance of post-puerperal psychosis.The treatment will be according to the investigations reports and clinical judgment of the Surgeon."
},
{
"id": 222400,
"tgt": "What causes blood clots during menstruation?",
"src": "Patient: Hi, may I answer your health queIries right now ? Please typei your query here...i have any hcg of 8 my lmp was nov 21. Nothing showed on ultrasound. Er doctor said I wasn't pregnant. I am bleeding and passing clots but it's not a flow. Any thoughts? Doctor: Hi dear, I have gone through your question and understand your concerns.HCG levels of 8 is considered to be non pregnant status.Passage of clots can be due to excessive flow in periods.Hope you found the answer helpful.Wishing you good health.Dr Deepti Verma"
},
{
"id": 176853,
"tgt": "Could cough, chest spasms, wheezing and allergic colds be due to asthma?",
"src": "Patient: Hi Doctor, My Son is 4 Years 4 months old. At age 4 years 1 month one day he suddenly got fever, severe cough, spams in chest when his chest was going inside and wheezing. he was diagnosed with LRTI and was treated in hospital for 5 days with nebulizers and antibiotics.HIs IGE serum and count was higher than normal. After 5days doctor discharged him from hospital and prescribed to give him budecort 100 2X2 puffs and motair lc kid 5 ml everyday for 3 months. He was very fine for 1 month and we stopped medications as we were unsure about steroids. again after 2 months he had severe dry cough and spasms.no fever now. We took him to hospital emergency where doctor nebulized him once and told he can be treated at home with levolin puffs every 6 hours for 4 days, budecort and montair to be continued for another 3 months... We wnated to know if this is asthma and he said he can t say now. This are allergic colds and he shuld be on montair lc and budecort and this may go away after child is 5 years. We are not sure if these steroids are safe....and if we can give him this for 3 months...is this asthma or can this be a different underlying disease? are the medicines safe and ok ? should i reduce his milk or egg intake? Doctor: Thanks for your query on HCMFrom history it seems your child has \"multi trigger wheeze\". it may be asthma but asthma usually diagnosed if this episodic wheeze persist even after six year of age.There is more chances of persistent of this wheeze even after 6 year if child has food allergy, allergic rhinitis, atopic dermatitis, or parents or other siblings have asthma.Treatment of multi trigger wheeze is similar to asthma and include salbutamole/levosalbutamole to control symptoms and inhaled inhaled steroid to prevent episodes.Inhaled steroid is safe and usually have only local side effect like oral thrush which can be prevented if child do gargle after each puff. these medicine is necessary other wise child may have recurrent episode Your doctor is treating in right direction, so i suggest you to follow his all direction and dont leave medicine without his advice. regular followup with doctor is necessary because this condition require modification of dose according to response.For more queries you may contact me at this site.Regards-Dr Deepak Vaishnav"
},
{
"id": 159276,
"tgt": "Lump on right side of chin, blood discharge on popping. Injected it with cortisone. Cancer?",
"src": "Patient: I have a small (little larger then a pea) lump on the right side of my chin . i went to a dermatologist and he opened it expecting to find material inside. there was just blood . He injected it with cortizone and it didi get alittle smaller. 3 week later i went back and he injected it again. It has gotten even small but is still there. I go back in two weeks and if it is not significantly gone he wants to biopsy it. Does he think this is cancer? Doctor: Hi, biopsy does not always indicate a cancer. without proper history regarding nature of swelling and associated features it is difficult to comment on diagnosis. anyway as he has ordered biopsy you should go for it . exact tissue diagnosis then can be easily made and treatment can be done. cosult dermatologist."
},
{
"id": 184804,
"tgt": "How to treat gingivitis around the teeth?",
"src": "Patient: I have many nodules, two of these have been biopsied with negative results ... (one - 2 yrs ago) New symptoms have surfaced now although maybe it is something else ... what? ...... gingivitis around several teeth, mouth sores, sore throat and glands. Also I believe one nodule must be enlarging... What are the throat and mouth sores all about?? Doctor: Thanks for your query, i have gone through your query. The gingivitis can be treated with scaling or oral prophylaxis. after scaling maintain the oral hygiene by brushing your teeth twice daily, and you can rinse your mouth with mouth washes. the soreness in the mouth and throat could be because of the aphthous or herpetic ulcers. consult a oral physician to rule out above said causes and treat accordingly. The nodules can be rebiopsied and get it examined by a good oral pathologist for a proper diagnosis and management. i hope my answer will help you, take care,.."
},
{
"id": 107740,
"tgt": "What could burning ribs with back pain suggest?",
"src": "Patient: Hi there I keep feeling funny, I keep getting.Burning/warm feeling under bottom left ribs moving down to my waist around the left side of my belly button and left hip.My head feels like its heavy and someone is pushing it around (sometimes this gos).I feeling of dread/panic. Like a cold feeling in my chest.Throat closes up when trying to eat sometimes, as if someone had their hands round my neck.Back pain, neck pain.Blurry eyes.Balance unsteady sometimes.Brain fog.The tips of my fingers on my left hand are all sore and have a rash or something on them.Feel presure or a kind of tightning of my muscles above my belly button up towards my chests.Digestive problems.Hair on my face (im female)Hair thinning on my head.Ive had blood tests and my doc is saying they cant find anything, but why do I feel like this. Doctor: Simply get yourself moulded to take antacid herbals for the time being. A kind of oesoophagitis, acidity ( in Ayurveda Pitt humor vitiation) takes place. It could be result of mild hiatal herniation symptoms too.Avipattikar chooran ( a herbal powders combo) is the best bet. It's bitter a bit but effective. No side effect, safe, no counteraction with other medicines.Second required thing is Haridra Khand - a classical Ayurveda medicine having main heb turmeric.These medicines are available all over Indian subcontinent. I m sure you won't need so many procedures and tests again. Once you start these, other things will follow to correct. Pl try to find these. If HCM allows we may try to send you.Next step related to hairs etc will be advised after completing this phase.Hope it helps you."
},
{
"id": 149542,
"tgt": "Have spinal compression and nerve compression. Prescribed Rylica, Oxetol and Sibilium. Which one to use?",
"src": "Patient: i got Posterions spinal compression in cervical C6-C7, C-4-C5 and exit nerve compression in C5-C6 due to osteophytes. I am presicribed Rylica 75 by one doctor and Oxetol 15m mg 1 tablet and Sibilium ( 5 Mg) by another doctor. Which on is to use?. I have pain in arms and numbness on hands along with shooting pain on Head. Also neck pain and shoulder pain occure Doctor: HiThank you for your question.In this scenario of nerve compression with pain and numbness, Lyrica is a better option to offer pain relief."
},
{
"id": 194064,
"tgt": "Suggest remedy to control libido when stressed?",
"src": "Patient: my brother cannot control himself and will have oral sex with anonymous men.he says he does it when he is stressed. he is married and has a daughter. he is 38.He is 5 feet 8 inches.he has consulted many doctors. They gave him advices but of no use .Know he is under the treatment of a pychatrist in Hyderabad He gives him flexartin 60,an antiaddiction medicine and an injection to reduce his libido Doctor: Hi, I can understand your concern for your symptoms, 1.\u00a0\u00a0\u00a0\u00a0\u00a0Infections 2.\u00a0\u00a0\u00a0\u00a0\u00a0Diabetes , hypertension. 3.\u00a0\u00a0\u00a0\u00a0\u00a0Hormonal disturbances- Low testosterone, low levels of thyroid, high levels of prolactin. 4.\u00a0\u00a0\u00a0\u00a0\u00a0Obesity 5.\u00a0\u00a0\u00a0\u00a0\u00a0High cholesterol 6.\u00a0\u00a0\u00a0\u00a0\u00a0 Chronic disease 7.\u00a0\u00a0\u00a0\u00a0\u00a0Drugs 8.\u00a0\u00a0\u00a0\u00a0\u00a0Anxiety Find out the cause & treatment will be according to it. Hope I have answered your query. Let me know if I can assist you further. Take care Regards, Dr S.R.Raveendran, Sexologist"
},
{
"id": 156975,
"tgt": "What causes bloating in upper abdomen after had colonoscopy?",
"src": "Patient: Hello - I had a colonoscopy in late January of this year, and my upper abdomen has been bloated ever since. Could this be related to the colonoscopy? Or perhaps the Prepopik preparation I drank the day prior? I am a 52 year old female, and this is my first colonoscopy. Doctor: Thank you for asking! Colonoscopy has its effects one amongst which is gut motility compromise for some time. It takes a few weeks for the gut peristalsis to get back to normal. It is normal and self resolves.Keep the diet soft and bowel friendly.Meanwhile some pro-kinetic would help the bloating like Metoclopromide. Stay in touch with your gastroenterologist in case the problem persists.Take care"
},
{
"id": 92617,
"tgt": "Abdominal pain, prescribed silbostin and BP tablet. Had pain before and diagnosed fatty lever stage-I and cured",
"src": "Patient: Sir, i feel my upper aqbdoman pain and go to cunsult with doctor doctor advise me to full abdoman ultrasound and fount fatty lever stage -I and after 6 month medicin i got releaf. and now after feeling realif 4 months feeling pain. again i cunsult with doctor he is giving me silbostin tablet. and my BP is also 96 134 he also giving me BP medicin. kindly guide me. Doctor: it look to me food proteins reacting with body proteins sometime body protein start reacting with food proteins suddenly to cause these and they can effect other organ alsoget blood serum tests for specific antibodies for milk wheat and other major food you take daily after consultation with allergy doctorwithdraw those proteins you will be benefitted"
},
{
"id": 21449,
"tgt": "What causes electrical shock on left arm when approaching target heart rate?",
"src": "Patient: i've recently started using the treadmill at the gym, and i haven't done anything for years. when i approach my target heart rate (123 and up) i get a feeling of small electrical shocks on the outside of my left arm. i do not feel any other symptoms. when i maintain below 120 its fine. Doctor: Sir i think these should is are muscle twitching or fasciculation . You need to confirm it then you please get your proper treadmill test in cardiac lab. Some routine blood investigations to see electrolytes and ECG 12 lead. i am sure if you continue your work out it will gradually went off.Best regards"
},
{
"id": 163633,
"tgt": "What causes fever and cough in a child?",
"src": "Patient: My son (age 5) was diagnosed with pink eye on Sunday; he took Tobrex, which has cleared his eyes. Yesterday he got small hives on his face, arms and legs... his ped believes it was an allergic reaction. We gave him Benadryl and the rash is gone. However, he has continued to have congestion, low grade fevers (in the evenings), little appetite, and is pretty lethargic (sometimes bursts of energy, followed by long naps... he usually does not nap). Today, my younger son (age 3) was sent home from school. He is also VERY lethargic. He as slept most of the afternoon (usually does not take naps). He, too, seems to have a very low grade fever (in the 99 range) and has no appetite. He also has a mild cough and congestion which just started. Any ideas? Could this be mono? Doctor: Hello,With two family members down with similar symptoms I feel that it could be a viral illness. But at the same time whether it is mono or not could be diagnosed exactly after doing a blood test only.Mono is also a viral illness and exactly behaves the same way.Hope I have answered your query. Let me know if I can assist you further.Regards,Dr. Sumanth Amperayani"
},
{
"id": 144466,
"tgt": "What causes restless leg syndrome?",
"src": "Patient: Hi My 83 year old father, who is diabetic, recently had a short session of involuntary leg, arm, shoulders jerking and spasms. He does have restless leg syndrome but this was considerably more exaggerated. Any thoughts as to what this may be or what caused it? Doctor: Hi Dear,Welcome to HCM.Understanding your concern. As per your query you have restless leg syndrome. Well there can be many reasons for symptoms you mention in query like chronic condition, such as diabetes, kidney disease , peripheral neuropathy , pregnancy , sleep deprivation, use of alcohol or caffeine, certain medications, including lithium and neuroleptics or iron deficiency . I would suggest you to do mind relaxing exercises like meditation and yoga , take healthy diet like fresh fruits and vegetables ( spinach ) , drink plenty of water and avoid caffeinated product . If condition doesn't get well then consult neurologist for proper examination . Doctor may order certain test like blood test , check your symptoms and may refer you to nephrologist for kidney problem . Doctor may prescribe vitamin supplement , ask you to cahnge your life style or prescribe sedatives for few days . Hope your concern has been resolved.Get Well Soon.Best Wishes,Dr. Harry Maheshwari"
},
{
"id": 201333,
"tgt": "What causes red bumps in the genital area?",
"src": "Patient: I have like ten little red bump starting at my waste line down to my pubic hair, but still like an inch from my penis. They have been there for like a month and can t figure out what going on. I also ich a lot and never really see anything biting me. I thought it was bed bugs but I have moved since and it has not stoped. Doctor: Hi,It seems that you might be having fungal infection or irritation due to shaving giving this problem.Apply anti-fungal cream locally.Keep local hygiene clean.Keep local part well shaved.Ok and take care."
},
{
"id": 45151,
"tgt": "How asthenospermia can be treated ?",
"src": "Patient: what is the latest drug approved to treat asthenospermia Doctor: There is no allopathic medicine to treat ashthenospermia successfully."
},
{
"id": 21103,
"tgt": "What causes rapid heart rate and body tremors?",
"src": "Patient: i am 32years male, 65kg, 182cms tall. Recently a few times i felt my heart beat was rapid and my body was shaking when i climbed the stairs and when i was walking carrrying a 5kg bag .. It was for a couple min and wud ease down once i rest. There was no pain nor sweating. Will it be a case of heart disease?. I dnt smoke nor drink but eat a lot of fried foods and my family has got a history of heart disease. What simple tests could help me confirm that i dnt have aheart disease. Help Doctor: Hello! Do not worry, because what you refer does not look a serious problem. It can be a form of normal tachycardia. However, I would recommend you to do a cardiac evaluation; electrocardiogram (ECG), cardiac ultrasond and monitoring of a 24-hour rhythm (Holter rhythm 24 hours) and an evaluation of thyroid hormones.I hope to have helped you with my answer. You are welcome to write to us again.I wish you good healthDr. P. Cerri"
},
{
"id": 209314,
"tgt": "Does stress cause dizziness and chest pain?",
"src": "Patient: Hello, I'm 30 yr male, Ht 5'6'', wt 55Kg I'm feeling bit dizzyness at the chest and the pain would be for say 10 - 15 mins and comes back after a while....I never had any other health issues...but off late feeling bit pain in the chest and also I'm mentally stressed due to my personal problems...Is this anything to do with the pain ?? Doctor: Dizziness could be because of dehydration if you live in a sunny place , unhealthy eating habits or lack of sleep. You should consult a general physician if your chest pain is too severe. Stress directly effects our heart , which in turn makes our chest pain. Don't ignore this problem and kindly don't keep on deviating the problem by taking pills for it. Consult a doctor as soon as possible."
},
{
"id": 151669,
"tgt": "Sciatica from one year, compression in vertebrae, Priformis syndrome",
"src": "Patient: hy sir i m suffering with sciatica from 1 year i m 32 yrs old male married 1 year ago doctor check my x-ray n says there is an compression in vertibrae but sir can u plz explain me that i cant feel 1% of backpain only pain and inflammation in hips and under knee if a compression in vertibrea then thank god i dont have pain in vertibrae i think there is a PRIFORMIS SYNDEROM whts ur great opinion sir itsjust my thinking and if i m suffering in PRIFORMIS SYNDEROM then wht can i do and wich test shows exactly PRIFORMIS SYNDEROM causes i m very thankfull 2 u sir Doctor: Hello. Thanks for writing to us. Your symptoms are suggestive of a nerve root compression at the spinal cord level. For proper diagnosis, an MRI of the lumbosacral spine is helpful. The diagnosis of Pyriformis syndrome can only be done after a physical examination. I hope this information has been both informative and helpful for you. Regards, Dr. Praveen Tayal drtayal72@gmail.com"
},
{
"id": 197807,
"tgt": "Suggest treatment for sore foreskin",
"src": "Patient: foreskin problemthe foreskin has become sore, small sores and it is almost like it is too tight to be pulled back. I have recently ended a course of augumentin for tonsilitis and got thrush during this time. I was given canesten which cleared the thrush but this problem is still there Doctor: Hi,It seems that due to sticking of smegma on glans cause prosthatitis producing unable to pulled back foreskin.Clean smegma by averting foreskin and apply antibiotic cream locally.Afterwards make a habit of cleaning smegma while taking shower.Ok and take care."
},
{
"id": 157698,
"tgt": "Have a rare form of lung cancer. Taking chemo. Done bone scan. Have swelled thighs",
"src": "Patient: I have a rare form of lung cancer, and I am going thru chemo right 6 sessions so far. I just had a bone scan doneand it came up that both my knees, left thigh and left hip have degenerative changes. My oncologist today said hemay want to biopsy my thigh. It does not hurt me, but it is swelled and feels hard. I guess I'm just looking for eithera second opinion Doctor: HIThank for asking to HCMI really understand your concern in fact for second opinion I will be in need of complete medical history along with all the reports without these it is little difficult still from the given history I could say that, if wee want to rule out the possibility of metastasis then the biopsy is necessary you better follow the advise of your physician have great day."
},
{
"id": 120931,
"tgt": "What could cause foot seizure?",
"src": "Patient: I have just had a very day of I think Foot seizure, I one about 3 years ago. the pain was unbearable wnet to doctors and am going for xray and ultra sound today. Will these show up a seizure? I can walk today with a limp and only 50% of pain of yesterday. Many thanks Doctor: Hello,Any type of seizure can not be detected by Ultrasound or X-ray. Looking at your details it looks to me a muscle cramp at your foot. Some time due to severe contraction of calf muscle or muscles at foot there can be a painful cramp. As your record is showing that you are having more then 50% percent relief. In my opinion you should do rest and apply a ace bandage. This will help you in giving relief. You may have a mild analgesic like aceclofenc 100mg on your doctor advice. Hope I have answered your question. Let me know if I can assist you further. Regards, Dr. Mukesh Tiwari, Orthopaedic Surgeon"
},
{
"id": 168885,
"tgt": "What causes bumps on armpits of a child?",
"src": "Patient: My 7 year old son has these 3 bumps on each of his armpits. They look like bites.. has been treated for scribes yet they don t go away. They basically look like regular bug bites. He even has two of them on his ball sac. He has had them for about a month...they won t go away. What can they be? Doctor: Hi...Thank you for consulting in Health Care magic. Skin conditions are best diagnosed only after seeing directly. I suggest you to upload photographs of the same on this website, so that I can guide you scientifically. Hope my answer was helpful for you. I am happy to help any time. Further clarifications and consultations on Health care magic are welcome. If you do not have any clarifications, you can close the discussion and rate the answer. Wish your kid good health.Dr. Sumanth MBBS., DCH., DNB (Paed).,"
},
{
"id": 38781,
"tgt": "Suggest remedy for thrush",
"src": "Patient: hello, i found out i had really bad thrush on tuesday and they gave me some stuff, but the itchy is still there and its really getting to me now. i took the pessary but read not to take it on my period but the man new i was and i dont thing it has worked, what should i do? Doctor: HI, thanks for using healthcare magicVaginal yeast infections can be itching and they are associated with a white- yellow discharge.There are oral tablets that can be used to treat this infection that would not depend on whether you were having your period or not.The main one is fluconazole 150mg. Only one dose of this medication is taken.This is the best option since you are currently menstruating.It should be available at your local pharmacy though you may need a prescription.I hope this helps"
},
{
"id": 50485,
"tgt": "Diagnosed with severe fatty liver. Kidney stones, abdominal pain, nausea, indigestion. Liver function test normal. Symptoms from liver or kidney stones?",
"src": "Patient: I have been diagnosed with severe fatty liver by ultrasound. Liver function tests are all normal I also have kidney stones in left kidney high up that require lithotripsy. Iam experiencing right sided abd pain under the ribsthat comes and goes , nausea, and indigestion can these sx be coming fro the liver or the kidney stones Doctor: Hi!. Thanks for your query. I think your pain is due to Kidney stones and you better follow your doctors advice for lithotripsy. Take care."
},
{
"id": 111722,
"tgt": "Could pain in left side of back near kidneys be due to a pulled muscle?",
"src": "Patient: I have pain in my left side on my back near the kidneys. It is extreme pain when I sneeze, so bad that I almost fall on the floor if I am standing up and moderate to severe pain when I cough or turn in bed, but it does not hurt when I press on it. There is no fever, no high blood perssure. Could it be a pulled muscle or should I go to a clinic to see a doctor? Doctor: HiThanks for writing to us.keeping in mind your complain may i ask do you have nausea and vomiting along with pain radiating from back to front ? If yes ,then it's better to get ultra sound scan of the abdomen done.Mean while you can have drugs in combination of muscle relaxant and pain killer.I shall be glad to answer if you have further query.Take care,Dear."
},
{
"id": 166622,
"tgt": "Suggest treatment for cortical dysplasia in a child",
"src": "Patient: Hi, My Child has a cortical dysplasia near the language area. he is 5.5 years old. he had 5-8 seizures a day. The Doctors get to conclusion that he need a surgery cause the all medicines he take do nothing. We live in Israel. We look for the best surgone in the world for that. Do you know someone? Doctor: HelloI have gone through your question and understand your concern. It must be difficult as a parent to see a child suffer, surgey will help and will take care of seizures. For your information, many of the world\u2019s top pediatric hospitals are located in the United States. According to various survey's carried out in the public interest, Dr.Benjamin S. Carson, M.D., one of the world's foremost pediatric neurosurgeons, is professor and chief of pediatric neurosurgery at Johns Hopkins University Medical School is known to be the best. Ben Carson revolutionized his field in several areas, including hemispherectomies (removal of half of the brain to prevent untreatable severe seizures, such as those caused by Rasmussen's encephalitis). He dramatically increased the safety of the procedure by developing several major surgical innovations, which include better ways of controlling bleeding and infection, as well as an innovative system of incrementally removing specific brain parts as units rather than in whole sections.Does not matter which part of world the surgeon is from, he will put all his effort in the best interest of patient. At the end of the day all the treating doctor's have same goal, to treat the patient with care."
},
{
"id": 92462,
"tgt": "Stomach pain aggrevated at night, vomiting, flu like symptoms. On magnesium citrate for cleansing. Advice",
"src": "Patient: 52 year male, 250lbs, I go to bed and awake with a stomach ache, which progresses into the night... I can t go to the bathroom, number 2, I feel blocked, finally I vomit, retch, and feel only a lil better. Fall asleep at 6am and sleep all day with flu like symptoms, feel cold, headache coming and going, sleep needed. I cleanse myself with magnesium citrate, which starts the healing process. This has happened 5 weeks ago, 3 weeks ago, and then this last Friday Doctor: you might be suffering from abdominal organ disease like gastritis duodenitis gall bladder liver or other organ get it investigated by consulting gastroenterologist if nothing found go to allergy specialist to find if you are allergic to any major food proteins these can be reasons for your problem and getting investgation for diagnose and treatment according to cause can help you in my opinion"
},
{
"id": 186489,
"tgt": "Reason for severe tooth ache?",
"src": "Patient: Hi,about a month ago i had a really bad toothache and had to go to the dentist. He removed my wisdom tooth as it was growing sideways in the cheek and caused a really bad infection. the last few days my cheek is hurting and it feels exactly the same as when i had that infection. It also looks similar. It is really tender and it hurts when i chew. It looks like it has a pussy centre.I'm worried cause this is the second time I've had it in a month. Is this something serious? Doctor: Thanks for posting your query to HCM.It shouldn't be serious. You should visit a dentist and get proper oral and radiological examination done of the concerned area.Till then you can do Betadine rinses 3-4 times a day.Hope my answer will help you."
},
{
"id": 13830,
"tgt": "What are the treatments for recurring itchy, red rash on wrists?",
"src": "Patient: I have a reoccurring rash on the insides of my wrists. It's read and itchy. I used to wear jewelry but stopped. I do use essential oils and thought this might be the reason. But I am not applying to the effected area. I'm not so concerned a bout what it is (but am curious) but how to stop it. Doctor: Hi, It could most probably due to jewelry, the condition is known as contact dermatitis. Does it resolves after stopped wearing the jewelry. In that case, the diagnosis is most probably contact dermatitis. Do visit your Dermatologist to confirm the diagnosis and to initiate the treatment. Hope I have answered your query. Let me know if I can assist you further."
},
{
"id": 150382,
"tgt": "Have indent in the thecal sac, hypertrophic facet sac. Is that the reason for pain in the lumbar area?",
"src": "Patient: hi there if l have Indent Thecal Sac L2/3 4/5 5/si should thi s cause much pain...l also have Hypertrophic Facet Sac throughout ....Modic type 2 endplate at 2/5 L2/3 dehydrated and reduced.....l dont underatand my MRI could you please expalin .....there are no signs of Spinal Canal Stenosis or Discal Neural Compression....Thank youI have sever pain in the lower lumbar area and l am on Butrans 20mg and Zapain 4000mg per day Doctor: Hi, Your MRI spine describes the various features of degeneration of spine and vertebral body. Thecal sac is the space where roots from spinal cord comes out which is mildly indented.Hypertrophy facet is thickening and enlargement of joints between veretebra called facets.Modic type2 endplated indicated degeneration of spine. Your back pain is mainly due to facet hypertrophy and ,thecal sac indentastion.You need to use lumbosacral belt. Consult neurologist or a neurosurgeon for neuralgic drugs.If syptoms still persisting then go for thecal steroid injection"
},
{
"id": 58924,
"tgt": "Slightly elevated bilirubin levels but within normal limits. On medication for gout. Suggestions?",
"src": "Patient: I went to the doctor today for a routine check up. He always goes over the blood tests with me but this time what I saw was that the bilirubin was slightly elevated but consistently increasing for over, more than five years but still is in the normal range. I am seventy years of age, excellent health, except for gout, I take allopurinol 300 mg/day for the last 10 years and aspirin. Doctor: Hello and welcome to HCM.Thank you for your query.I am very proud of your lifestlye and dedication to maintain good health. Gout is a common condition that takes place due to high serum uric acid levels. I would first like to inform you that serum uric acid levels can be controlled to quite an extent through diet. Why don't you consult a nutritionist for the same. If you succeed, then we can wean you off the medication.Allopurinol has shown to cause a rise in bilirubin levels, which is why you notice only the rise of bilirubin levels. As long as other parameters to measure your liver function are within normal limits, there should be absolutely no reason to be worried. Try controlling your gout through your diet, which will help your doctor in reducing the dose and gradually weaning you off the allopurinol. Wishing you loads of luck.I hope I have succeeded in providing the information you were looking for. Please feel free to write back to me for any further clarifications at: http://doctor.healthcaremagic.com/doctors/dr-shoaib-khan/64581 . I would gladly help you.Best wishes."
},
{
"id": 92550,
"tgt": "Mouth full of saliva when having severe abdominal pain. Possible reason?",
"src": "Patient: Hi , I woke up with this extreme pain in my stomach it lasts for about 5 seconds but during the pain mouth fill with saliva ! It's happenedvon and off for the last hour . It happened before and i put it down to something I ate ! However, I havent eaten in about 9 hours ( sleeping) and havw drank plenty of water ..I also dont smoke ir drink. What could it be ? Thank you. Doctor: Dear patient...You must be having a stomach upset.The abdominal cramps is causing the increase salivation.I suggest u see a doctor.Mostly the abdominal pain will be either accompanied by vomitting or loose motions or both.You have to start on ors.....antibiotics and probioticsDrink plenty of fluids like coconut water..lime juice ......rice water...it will prevent you into going into dehydration.Please consult a doctor if it worsens ..u may require iv antibiotics.Hope i could help you out."
},
{
"id": 151895,
"tgt": "Can't blood clots be dissolved completely ?",
"src": "Patient: HI, My mother is suffering from weakness in right leg, hand and arm since 6 days, also only 20% movement was possible then. First 48 hours we could not find any root cause of this sudden illness. However, on third day after MRI we came to know that there are blood clots in the arteries leading to brain . (no clots in brain). I am not able to understand exact reason for this clots. Doctor is giving 2-3 injections everyday since then so as to liquefy the thickness in blood. Now there is 50-60% improvement in physical capabilities on my mother only in right side of body, face is alright. But doctor said, that he cannot dissolve the clots completely. And only 70% recovery from paralyzed leg and hand is possible, even after physiotherapy . So is it real, that blood clots cannot be dissolved?, Also, can my mother recover 100% from this ? and can u please help me interpret meaning of 70% recovery on right hand side?..left side is alright. Doctor: Hi, Through arteries blood clot might have traveled in the brain causing this problem. Now as treatment for clot and making blood thin are going on so do not worry. But as doctor says 100%improvement is not possible it means due to clot some part of the brain area is damage due to cut of blood supply during initial period before treatment is started. you continue the treatment and physiotherapy more improvement may be there. ok and bye."
},
{
"id": 51842,
"tgt": "Can i get a kidney damage after few hard punches ?",
"src": "Patient: I received several hard punches on my kidney are by my friend. Can my kidneys get damaged? Doctor: Thanks for the kidney Kidney is a retro peritoneal organ so chances are that ur kidneys might not be damaged. I still suggest a usg abdomen and pelvis to rule out blunt injuries Have a healthy living"
},
{
"id": 53469,
"tgt": "What is the treatment for liver hemangioma?",
"src": "Patient: I have the exact same symptoms - I have had many tests (including CT, MRI for Abdomen & Back, and Xrays) which showed Liver FNH and Liver Hemangioma which would not cause pain supposedly, as well as 2 Herniated Discs in Back and Costochondritis. Still however the doctor wants to do one more test now, just to get a better idea because of the fact that my pain is so severe and I am not responding to Physical Therapy...Does this sound familiar to Anybody?? Doctor: Hello and Welcome to \u2018Ask A Doctor\u2019 service. I understand your concerns and I will try to help you as much as I can. FNH and liver hemangiomas are benign tumors and these should not be surgically treated unless there are symptoms such as pain, bleeding or jaundice or these exceeds 10cm in diameter. So surgery is not an option. Your spine problems may be treated with medicines, physical therapy and surgery in some advanced cases.Hope I have answered you query. If you have any further questions you can contact us anytime.Wish you good health.Kind regards,Dr. Ivan R. Rommstein"
},
{
"id": 22630,
"tgt": "Is a biopsy necessary after finding a spot on aortic window?",
"src": "Patient: my 20-year-old son has a spot on his aortic window, which was also a shadow in a 2004 but no one saw it then. So he has to have a biopsy; the spot is between his heart and his left lung. What are the odds of him bleeding fatally? If it s cancer, will it be terminal? Doctor: Hello and welcome to \u2018Ask A Doctor\u2019 service. I have reviewed your query and here is my advice. The chances of it being malignant are less than it being there for long time. If it's malignant then I would have progressed a lot till time. Also risk of bleeding depends upon what sits there. If there is possibility of hemangioma then risk is very high and biopsy is not recommended and if there is possibility of other tumor, then risk may not be that high. Please let me know the detailed report if you want further clarification. Also MRI can be considered if details are not provided by ct.Hope I have answered your query. Let me know if I can assist you further.Regards, Dr. Sagar Makode"
},
{
"id": 23805,
"tgt": "What causes shortness of breath?",
"src": "Patient: Good day, I 'm 27 years old and i've been having a shortness of breath, specially when i'm outside, i feel like i'm gonna faint,i drink plenty of water all day long and rtoday i'm feeling so tired when i slept ok last night.and i haven't done anything. Doctor: Hey,Breathlessness on exertion at this age can have a lot of causes1) Low Haemoglobin2) Cardiac Disease3) Respiratory DiseaseSo first of all check your Hb than get a xray and echo done to know the cause of breathlessness and easy tiredness. Wishing you good healthDr Sameer Maheshwari"
},
{
"id": 129642,
"tgt": "What could cause spreading, itchy knuckles?",
"src": "Patient: have these Itch s on my knuckles (really itchy) and it has been there for bout a week I have now noticed it has gone to my other knuckle of my Outher hand. It feels like a nettel sting but isn t a nettle sting. I can take a picture of them so u can see but it is getting me worried as it cud be contagious, seeming how I have the same on my outher hand Doctor: Dear patient this is a problem for a dermatologist and without a picture it would be quite impossible to point out what the condition is and what could have caused it. All I can say is try a tablet of Levocetrizine 10mg and see if it helps the itching.I strongly urge you to show it to a dermatologist"
},
{
"id": 209239,
"tgt": "How to treat disturbed mind?",
"src": "Patient: my wife is very much suspecious about every thing including his parents and also me. she complaint about somebody charging electricity at night. her age is 39 yrs. she has 2 daughter. some time she behave like erratic women who has no one in this world. one time she also left home without informing anybody and return next day. she always try to clean the room by sweeping. Doctor: DearWe understand your concernsI went through your details. I suggest you not to worry much. I am afraid to suggest here that your wife could be suffering from Paranoid personality disorder (PPD). It is a mental disorder characterized by paranoia and a pervasive, long-standing suspiciousness and generalized mistrust of others. Individuals with this personality disorder may be hypersensitive, easily feel slighted, and habitually relate to the world by vigilant scanning of the environment for clues or suggestions that may validate their fears or biases. Paranoid individuals are eager observers. They think they are in danger and look for signs and threats of that danger, potentially not appreciating other evidence.Best method of treatment is to consult a psychiatrist as soon as possible and start prescription medicines and then combine it with psychotherapy. If treated earlier, success rate is multifold. If you require more of my help in this aspect, Please post a direct question to me in this website. Make sure that you include every minute details possible. I shall prescribe some psychotherapy techniques which should help you cure your condition further.Hope this answers your query. Available for further clarifications.Good luck."
},
{
"id": 154286,
"tgt": "What is the life expectancy for metastatic bladder cancer?",
"src": "Patient: Hi, may I answer your health queries right now ? Please type your query here...hi my mum has just been diagnosed with cancer that started in her bladder and she had surgry 6 weeks ago and tumor was to big to remove so they bypassed it to bag outside her body.She had a scan on wednesday and tolled her she has cancer in her whole body.they did not give her an idea of about how long she has left to live and the pain is starting to get worse in her bones Doctor: HiThanks for posting.I understand your concern.I am sorry to know about your mom. This is an incurable condition but with chemo the average survival time is 1 year. it can be much more or even less depending on the response and tolerance. But still we can palliate her symptoms and give her some quality life.Hope this helps.Please get back if you need any more clarificationsRegards"
},
{
"id": 68921,
"tgt": "Is surgery required for removing hard movable bump on right side?",
"src": "Patient: My right knee has a hard movable bump on the right side. I skateboard a lot and for sure it's from falling on it multiple times. I've had it for a long time and it doesn't hurt. Also it sometimes swells up and ice it and wear my knee brace. What my question is I guess, will the bump eventually disapate without surgery? Doctor: Hello and welcome to HCM,The hard movable lump on the knee has to be investigated to know its site of origin.Repeated trauma can be the cause of the lump formation.The lump could be inflammatory due to repeated trauma to bone and soft tissue or could be neoplastic arising from soft tissue or bony tissue.I suggest you to consult an orthopedician for CT scan or MRI of the joint.Imaging will determine the site of origin extent of the lesion.Further aspiration cytology or biopsy may be required for exact characterization of the lesion.Management will be planned after the result of investigations is obtained.Thanks and take careDr Shailja P Wahal"
},
{
"id": 123583,
"tgt": "What does a hard swollen flat mass below the tail bone indicate?",
"src": "Patient: red, hard swollen flat mass below the tail bone with one big red blotch in the middle. i get a rash there often in summer because of wearing wet bathing suits but this time, it s not that. it appearred yesterday and is bigger and more awkward to sit down comfortably Doctor: Hello, As I find the history to be directing towards important thing is you have a gluteus muscle weakness due to which there is excessive pressure over the sacrum region. When the pressure is applied again and again there will be a beginning in the physiology of the skin tissue and the changes in the form of swelling, bruising, wound etc. Doing exercises for the gluteus muscle strengthening should be good and also using piles pillow to avoid excessive weight transferring over the sacrum. The icing is also a good thing to allow healing. Hope I have answered your query. Let me know if I can assist you further. Regards, Jay Indravadan Patel, Physical Therapist or Physiotherapist"
},
{
"id": 81172,
"tgt": "Suggest treatment for chest infection and metallic taste",
"src": "Patient: I have had a chest infection which has left me with metallic tasting cattarh and mucous that goes down my nose to throat. Is this normal. I gave been given steroids, then antibiotics, then naval spray, then inhaler with tube. Should it take 8 weeks to go away? Doctor: Thanks for the query. The condition you mentioned is known as post nasal drip and is a common cause of chronic cough. Treatment consist of antibiotics to cure infection and steroid nasal spray if you have nasal allergy. Any way nasal spray has to be continued for 3 months. Best wishes"
},
{
"id": 113476,
"tgt": "Lower backache after exercising. Some relief after applying ointment. Solution?",
"src": "Patient: hi , i have a lower back ache problem. i doesn t happen regular but whenever i do some exercises or jogging it happens. today i had been for some exercise & my back started paining. plzz suggest what to do i am using correct body postures during my exercise . my height is 5 8 & weight is 74 kg i am 30 yrs old. i applied solans balm for some relief. Doctor: Hi, Thank you for posting your query. The commonest cause of back pain is mechanical, due to prolonged sitting required for jobs, computer use, studies, driving, etc. One needs to strengthen the back muscles to prevent pain in the region. These exercises are referred to as back extension exercises. You can learn them under the guidance of a physiotherapist. For pain relief, you can use ketorolac or diclofenac gels over the painful area of the back. Wishing you good health, Dr Sudhir Kumar MD DM (Neurology) Senior Consultant Neurologist drsudhirkumar@yahoo.com"
},
{
"id": 17779,
"tgt": "What causes high BP?",
"src": "Patient: My systolic jumped very high for about two weeks. My diastolic is always normal 65 - 73 but my pressure jumped to 170 to 192. My doctor told me to double up on my Ramipril 2.5. But I feel like I have the flu or is this just anxiety. I ve had eye headaches for two weeks so I thought it was sinus or flu. I feel very anxious. I m taking 1/4 mg. of xanax to calm me down. I have never taken flu shot Doctor: Hello, A sinus infection can raise blood pressure values. For this reason, I recommend performing a sinus X-ray study and complete blood count for infection. In the meantime, I agree with your doctor on the recommendation to raise the dose of Ramipril as long as your blood pressure values continue to be high. Hope I have answered your query. Let me know if I can assist you further. Take care Regards, Dr Ilir Sharka, Cardiologist"
},
{
"id": 204272,
"tgt": "How can sadistic personality disorder be treated?",
"src": "Patient: My parents have said many times that i m cold blooded because i don t feel sad or pity when people die in terrorist attack like they do , i really don t understand why i should, it s not me. I tried to drown my sister out of jealousy but i played the victim so I didn t get punished. I like to break friendships up and make my friends hate each other. I like to wreck other people s groups. I like to twist people s words to make them feel bad and apologize/ comfort me. I only want to become a surgeon when i graduate because i want to operate on people not because i want to help them and my mother always say that of me, but even though that is not my goal, they are still helped in the end, so what s the problem? I once ran away from home for a while so that my parents can get worried and be punished. I pretend to be the innocent and lovely person around other people . I once accidentally poisoned a cat but i dont feel regret. I was happy when my baby cousin had a hole in his heart because he was born near my birthday and he took the spotlight away from me. I was mad my granddad died a week before my birthday so my family wont celebrate my birthday cause they are greiving. I fantasize about murdering or torturing people that has wronged me , but i wont do it cuz i will get a criminal record . I wasnt brought up in a abusive family and i have never had a traumatised childhood. I dont know what s wrong with me. Doctor: Hello and Welcome to \u2018Ask A Doctor\u2019 service. I have reviewed your query and here is my advice. This occur generally in Bipolar personality disorder. It is treated by Oxcarbamazepine, Aripiprazole and Clonazepam. Hope I have answered your query. Let me know if I can assist you further."
},
{
"id": 44712,
"tgt": "Which antibiotic should be taken for presence of pus cells on the semen ?",
"src": "Patient: we had an intercourse in my 5 days of my menstruation.can we have infection?i want to know my ovulation day usually im 26 days/28 days.. give me some advice i would like to become pregnant ..f im not fertile we dont need anymore to intercourse bcoz maybe the sperm will die suppose to be we will conceive?is it true?mu husband had a semenalysis.written there is infection bcoz the pus cells is high.. what he will do/take antibiotic?for how long?pls reply..thanks! Doctor: Hi Welcome to health care magic Intercourse during menstruation generally increases chances of infection.chances of infection is to the both partners.ovulation is in the mid cycle i.e.14 days before next cycle.infection in the semen can be treated with a course of broad spectrum antibiotics. \u2018Hope I have answered your query, I will be available to answer your follow up queries, \u201cWish you Good Health and speedy recovery\u201d Disclaimer"
},
{
"id": 45932,
"tgt": "Suggest treatment for kidney stone",
"src": "Patient: Dear sir, I m suffering from a 9.6mm kidney stone in my left proximal ureter causing mild hydroureteronephrosis. My doctor told me I have to go for stint operation to remove it. So how can I get rid of it? Do I actually need to go for operation? Or is there any medications available to melt the stone? Plz give a valuable suggestion ASAP. Thanks, Hari. Doctor: Hello and welcome to \u2018Ask A Doctor\u2019 service. I have reviewed your query and here is my advice. treatment of renal stone depends upon the size and site of stone, in your case the stone is in proximal ureter so you have 2 main option available for the removal1- start tablet alfuzosin 10 mg once a day daily, this drug dilates the ureter which helps the stones to pass via urethra2- get endoscopy removal of stone via urotrescopy where a camera tube is inserted via your penis and it goes up into bladder and then into ureter where stone is visualize and it is manual broken and retrieve piece by piece. Hope I have answered your query. Let me know if I can assist you further."
},
{
"id": 119357,
"tgt": "What does low WBC count in an obese, diabetic with high BP having arthritis and gout indicate ?",
"src": "Patient: my husband as been havinf a low white blood cell count for 2 months now hte last test showed lower results of 2.3 his hemeglobin is 11.8 AND PLATELets of 127 Iron Saturation of 12 an ferritin serum of 19 we are very scared and concerned cause you always think the worst and the hemotologist appointment is not the close My husband is a type 1 diabetic has high blood pressure, over weight , arthritis and gout can you give us any ideas of what could be wrong Doctor: Go with your family physician advice"
},
{
"id": 60972,
"tgt": "How can a lump on the breast be treated?",
"src": "Patient: Hi, I have a lump on my left breast near the border which is moving under the skin when I pressed it to the side. It does not give any pain. I am male, married and 48 years now. It was seen from last five years and size was remain same. I consulted it with the local doctor, they said it is the accumulation of fats under the skin. Need not to worry. Another point when I pressed my nipples it discharges liquids from both breast. Discharging liquid continued till I was 18 but there was no pain. Is it bad for me? Doctor: Hello,The soft lump indicates benign nature, but the long-standing liquid\u00a0the discharge does not sound normal. I strongly recommend\u00a0MRI\u00a0evaluation with further management to cure\u00a0the condition.Hope I have answered your query. Let me know if I can assist you further.Regards,\u00a0Dr. Bhagyesh V. Patel"
},
{
"id": 103089,
"tgt": "Have constant dry cough. Feel throat is irritating. Allergy specialist gave rescue inhalers. No improvement. Treatment?",
"src": "Patient: I have had a constant dry cough for over 2 months. It feels like something is irritating my throat because the cough sounds like it is coming from my throat and not my chest. I saw Asthma/Allergy specialist and he prescribed a daily and rescue inhalers. My cough hasn't gone away. It's becoming annoying and irritating. What could it be? Doctor: hello dear,thanks for your query at health care magici understand your concern regarding coughcough can be due to irritation caused by allergens on pharynx.1 drink lot of water that will soothen your symptoms2 try to avoid allergens , pets , dust3 try avoid exposing to pollution4 take tab montac lc once daily5 continue medication that was prescribed get well soon regards"
},
{
"id": 159031,
"tgt": "Huge acidity problem, taking omoprazoll 40 mg, burping. Is it the symptoms of cancer? Can NEXPRO help me?",
"src": "Patient: hiiiiiiiiiiiii..doctor.. i have face huge acidity problem for last 2months .i take omoprazoll 40 mg ..but nothing is improved..i feel a uneasyness near my throat ..like something is there..nd burping too much..i am very nervous..is it the symptoms of cancer?? plzz help me..what do i do??tab NEXPRO CAN HELP ME?? PLZZ SUGGEST.. Doctor: Hello Subhra ,your symptoms remind me of Gastro-esophageal Reflux disorder(GERD),a condition where the lower esophageal sphincter is incompetant leading to symptoms of acidity like heart-burn and chest discofort. You should take twice a day dosage of any PPI along with domperidone (eg. Rablet-D),which has a prokinetic effect ,for atleast 6 wks under guidance of a doctor. If symptoms persist ,an upper G.I. endoscopy to confirm diagnosis of any complication of GERD may be done to alleviate your fear and provide a possible surgical treatment. Please do keep me informed of your treatment proceedings,tk care."
},
{
"id": 189403,
"tgt": "Itchy bumps on soft palate. Taking antiallergic medicine. What could it be?",
"src": "Patient: So every now and then my soft palate gets extremely itchy, and I have no clue why. I have been taking antiallergic but now thay have bacame ineffective, my hard palate appears white in color and soft palate reddish with transparent highly itchy bumps. It seems like to scratch my palate with my nails.My teeth and gums are healthy.I am allergic to smoke, less sleep and but they aren't related to the itchyness of my palate, because it'll happen randomly. It'll pop up in the middle of the night or just randomly in the day for a few hours n its been now for months. please need your advice.Does anyone have any clue what it could be?? Doctor: Hello there , Thanks for writing your query, According to your symptoms i suspect your palatal mucosa of hard and soft palate are inflammed , may be due to allergy to certain food stuffs , physical or chemical trauma , allergy to smoke etc. It may be caused due to several reasons like deficiency or certain vitamins especially vitamin B12 and vitamin C. Maintain good oral hygiene by practising regular mouth rinsing with lukewarm saline water or antiseptic mouthwash like betadine or chlorhexidine. start with multivitamin supplements for 7 to 10 days and vitamin C lozenges will help in resolving the problem. avoid alcohol , carbonated drinks , tobacco chewing and smoking as these may worsen the area affected. Apply oral antiseptic ,analgesic ointments like dentogel or mucogel topically over the affected area i hope this helps , take care."
},
{
"id": 154122,
"tgt": "How long will the stitches of ovarian cancer take to heal?",
"src": "Patient: Hi, may I answer your health queries right now ? Please type your query here...I have 4th stage ovarian cancer. Had the debulking 6 weeks ago. My husband is packing me do to stitches coming apart 1 inch open. inside Dr. said there in a hole possible size of lemon, my husband packs me twice daily...do you have an idea how long this will take to heal from inside out? Doctor: Hi,Thanks for writing in.It is important to know if there is any infection in the area of stitches. If there is an infection then it must be treated first and then the wound healing will progress. By 6 weeks the stitches should have started healing. The time for complete healing of surgical wound is approximately 12 weeks. However since you have an open wound therefore first this wound must be cleaned and resutured. This will permit healing of the skin layers by first intention and progress to normal healing over the next few weeks.Since there is a lemon sized hole in your abdomen therefore it must be repaired by putting additional sutures in the area. Please discuss about the open wound in detail with your doctor and when he is likely to close it. Healing is not possible with a persisting open wound. Please do not worry."
},
{
"id": 140471,
"tgt": "Can a person have both multiple sclerosis and lupus?",
"src": "Patient: My 25 year old daughter has been diagnosed with MS. She had 2 MRIs and spinal plus loss of use on the right side of her body. Lupus keeps being mentioned because her SED rate is high. We see a PA tomorrow for results of some blood work the neurologist ordered. It is being sent to her GP but we are seeing the PA. My question is can you have both MS and Lupus? She has no fever, no rash, no joint pain, but two separate blood test say SED rate very high. They want to treat her with copaxen. When or how do we get a final diagnosis? Doctor: Hello, It is rare for the same person to have both MS and lupus. The symptoms can be different, however, there are strict diagnostic criteria for both the conditions. So, your doctor would be able to correctly make a diagnosis after reviewing the reports. Hope I have answered your query. Let me know if I can assist you further. Take care Regards, Dr Sudhir Kumar, Neurologist"
},
{
"id": 155889,
"tgt": "Can chemotherapy reduce the size of a ganglion near the carotid?",
"src": "Patient: My husband was treated last year for CA base tongue stage 4, his tongue is clear but he has now had 2 of 3 sessions of Al Saraff chemotherapy, the 3rd has been postponed as he needs to build himself up. The specialist advises there is no tumour in the lymph gland but he has a ganglion which currently makes surgery high risk due to its proximity to the carotid.He says this needs reduced before surgery is an option. Can chemoptherapy reduce this sufficiently?If it is not an option what would a treatment plan be?Also he has mucositis which impacts on his eating, how long does it last?MALE 67 years 8ST Doctor: 3 cycles of chemotherapy needs to be given followed by pet scan.mucositis lasts for 1 week.RegardsDr DeI"
},
{
"id": 199610,
"tgt": "Any medication to stop sperm drop and will it affect ability to have child?",
"src": "Patient: Hi, I am 21 years old. I am not sure If what I have is normal or not. Every time I speak to my girlfriend even on the phone, little drops of sperm come out. Should I get some medicine for it? Would this effect my ability to have kids.Thanks for your help Doctor: HelloThanks for query.You are getting discharge of clear fluid from urethra whenever you are talking to your girl friend on pone ..This is a excessive mucus secreted by mucus glands located in urethra which get stimulated even with sexual thoughts or when one is engaged in talking to female partner.This is normal at the age of adolescence and nothing to be worried about it .You do not need any treatment .This problem will get resolved over a period of time and once you get involved in sexual relationship with your girlfriend.This will not affect your sexual potency nor affect your ability to have a child.Dr.Patil."
},
{
"id": 211840,
"tgt": "Panic attack after eating food, unable to swallow, throat dry, appetite loss. Due to anxiety?",
"src": "Patient: So it started when I had a panick attack from eating a cooking and couldn't swallow after my throat became really dry. Then I drank it with water and my swallowing mechanisms were fine. I could swallow food but sometimes I would be afraid and be aware of my reflexes and started panicking. This panick cause me loss of appetite.I went to my doctor and did blood test and told me I was fine. My mom who is also a doctor told me its an anxeity. Her friend who is also a doctor told me Im just being paranoid. I could swallow my saliva fine but when I feel or have an urge to swallow without food saliva or water I couldn't and I need to push my saliva so I can swallow. So is this psycholigical? Doctor: Hello, Thanks for the query to H.C.M. Forum. In my opinion this is case of folds ( fold ) in esophagus . When there , fold in esophagus patient fears to eat. When eat something solid , it feels pain and very difficult to swallow without the help of water. In my opinion please consult your mother and insist for examination of esophagus for any folds. Good luck. Dr. HET"
},
{
"id": 218120,
"tgt": "Are Prednisone and HCQS pill advisable during pregnancy?",
"src": "Patient: Hi Doctor, I am XXXX. I had two miscarriage ealier and my doctor prescribed me to take HCQS 200 mg for sixmonths and Ecospirin 75mg. After six months we planned for child and my result is positive. and now my doctor wants me to take prednisone 5 mg and HCQS 200 mg throughout my pregnancy. Is using these tablets safe during pregnancy? Doctor: Hi,Both Prednisone and HCQS are grouped under pregnancy \"D\" category which means studies in pregnancy women have demonstrated a very low risk to the fetus. However, the benefits of therapy in a life threatening situation or a serious disease like lupus, may outweigh the potential risk. In view of the above, Prednisone (5 mg) and HCQS (200 mg) are generally considered to be safe throughout pregnancy so continue taking the treatment as prescribed by your doctor.Hope I have answered your query. Let me know if I can assist you further. Regards, Dr. Mohammed Taher Ali, General & Family Physician"
},
{
"id": 73606,
"tgt": "Suggest treatment for pain and discomfort in chest",
"src": "Patient: HI i took mushrooms about three weeks ago and i also smoked marijuana when i did it. about one hour into the trip i had horrible chest pain and heard my heart beating extremly fast..i was scared but then it went away..After that ive smoke weed 4 times but all four times were painful and days after my chest was hurting but not too bad..i also smoke cigarettes and today i feel when i breath in very hard that theres something in my chest. maybe like air..i dont know what to do i havent smoked weed or done anything other than cigarettes in about 2 weeks now so that the pain goes away... Doctor: Thanks for your question on Healthcare Magic.I can understand your concern.You are chronic smoker. So you are at risk of heart disease.So get done ecg, 2d echo and stress test (trade mill test) to rule out heart diseases for your chest discomfort.If these tests are normal then no need to worry for heart diseases.Better to quit smoking as soon as possible as this is not good habit.Hope I have solved your query. I will be happy to help you further. Wish you good health. Thanks."
},
{
"id": 1097,
"tgt": "Will i get pregnant even after taking contraceptive pills?",
"src": "Patient: hi have two question. i had my period on the 27th of jan, i had it for 5 days which is normal but flow on the last two days was a bits smaller than usual. i took a test today and it came back negative. is it right.today i had sex with my boyfriend we used condoms but the broke a some point i had also 2 tabs of today contraceptive. he didnit ejculate in me. i was scared so went to the pharmacy and the pharmacist gave me primolut N as an EC - would i get pregnant and is primolut N a god EC Doctor: the pregnancy test is right, since you got your period then you are not pregnantif you had sex just after your period ended then you won't get pregnant as you are not ovulating at this timealso taking the after pill decreases the chances of getting pregnant by 95 %"
},
{
"id": 10402,
"tgt": "Suggest remedy for alopecia universalis",
"src": "Patient: hi im 19 and im 5'4 and 8st and i am 3 months pregnant with my 2nd child and i have alopecia universalis, i have been completly bald for over 1yr now, and i used to have bald patches (alopecia areata) i did suffer with bald patches when i was a child but i recovered now i have lost all my hair everywhere. i just need some professional advice on the matter.. is there anyway i can pass it to my baby?? where i am pregnant and in this state. plus is there any hope for me in the future with regrowing my hair loss as i dont think ill accept the way i have become.. Doctor: Hello and Welcome to \u2018Ask A Doctor\u2019 service. I have reviewed your query and here is my advice. For alopecia universalis I would suggest pulse therapy with methyl prednisolone. Other remedies includes oral cyclosporine and more recently Tofacitnib. Hope I have answered your query. Let me know if I can assist you further."
},
{
"id": 145803,
"tgt": "Suggest treatment for back pain due to mass on spine",
"src": "Patient: my 66 yrs old husband has recently been diag with osteomyelitis he has been on dialysis for polysystic kidney for 16 years he has a stint in the heart has had 1 heart attack he has two stints in the leg also has had shingles a mild case on the right upper chest under the arm pit area and a sm amount on the shoulder area no open or blester anymore but still has pain on occaision he has a mass on his spine recently bio y=this it came back no cancer but he cant lay down in bed without having sever pain lasting fo 20 min to an hour what can I do to help with the pain im giving rocephan iv at home after his dialysis treatments Doctor: Hi,Thanks for writing in.Your husband has a mass on the spine and this is to be known in details regarding the exact location, size and pressure on surrounding areas. The spine is a delicate part of the body and any mass over there can cause pressure on the structures within the spinal cord or adjacent to it. There are many nerves in the area and pinching of nerves can cause significant pain in particular positions. Since it is not a cancer, still attempts are to be made towards decompression of the mass at its location. Till such time that surgical treatment is decided upon, he can be given prescription strong pain killers in the form of skin patches to reduce the severe pain when it happens."
},
{
"id": 99930,
"tgt": "Suggest treatment for sniffling",
"src": "Patient: My husband has constant sniffling. He says it feels like his nose is dripping but when he blows it nothing comes out. He has had sinus surgery from constant head aches which helped but continued the sniffling. What can he do to get rid of it.....driving us all crazy Doctor: Hi thanks for using HCM...Continuous sneezing can be due to1.Allergy2.post-nasal drip from sinusmild steroid nasal sprays are available which can reduce sneezinganti-allergic tablets can be used for post nasal drip..you can also use mucolytes and steam inhalation .By clearing the sinuses post nasal drip can be reduced..we can try these options thank you..."
},
{
"id": 58983,
"tgt": "Had vertical sleeve weight loss surgery, hysterectomy. CT scan shows cyst on liver. Feeling weak, dizzy. Advice?",
"src": "Patient: i had verticle sleave WL surgery then month later hysterectomy kept ovaries thrn obgyn seen unbilical hernia I am in pain there going to have it fixed I am extremely tired and wake up weak and a little dizzy I stay in bedmost of the day had ct scan mri they didn't show hernia only cycst on liver going back to doc tomorrow extreme ftique robbing my life surgery 3/21/. 4/26/2013 Doctor: Hi and welcome to HCM, thank you for your query.I dont think this cyst is associated with your symptoms. You should check your estrogen and thyroid hormonal levels. There are some surgical complications, but it woldnt cause such difficulties. So just do exstensive blood tests. Also it can be psychosomatic and changing life and dietary habits would be of benefit.Wish you good health. If you have any question I ll be glad to help."
},
{
"id": 144998,
"tgt": "Suggest treatment for polyneuropathy with diabetes",
"src": "Patient: Hi Dr Samuel, My Mother is suffering with Polyneuropathy from 2012 March we got to know about it in 2013 September. She is on bed from 2012 March. She can move her upper body but not the lower bod, she has the movements in the lower body too however can not lift the body from the lower portion. She is also diabetic. Doctors said that she will be living for next 6 or 12 months. I am really worried. If you could please guide me how i can proceed. Please let me know, Hoping to hear back from you soon. :) Doctor: Dear Patient, your mother's inability to move the legs may or may not be due to diabetic neuropathy. She should also be checked for spinal stenosis in the lower back and primary muscle disease and maybe thoracic spinal cord problems. Thank you."
},
{
"id": 100384,
"tgt": "Suggest therapy for dust allergy",
"src": "Patient: Hi Doctor, I am writing this for behalf my brother. He is suffering from the dust allergy since last 3 months, we have taken him to a allergy doctor in our city, and he has prescribed him medicines for 3 months. But after the course, again it is started. So can you please tell me the remedy for his dust allergy and can you please suggest a good dust allergy doctor in Bangalore ? Doctor: Hello.Thank you for asking at HCM.I went through your history and would like to make following suggestions for your brother:1. I would suggest your brother allergy testing which will help him identify the substances causing symptoms to him and also to know the measures to avoid them.Based on the allergy testing, an Allergist-Immunologist may prescribe him allergen specific immunotherapy which gradually improve allergy symptoms by working on immune system.2. At present, I would suggest him regularly montelukast and levocetirizine on daily basis.3. In general, I would suggest him to avoid exposure to dusts, smokes and air pollution.Allergies is a big problem in Bangalore because of air pollution.There are many Allergist-Immunologist in Bangalore. You may consult any of them who is easy to approach with convenient location. There are some institutes like KIMS who have many Allergists-Immunologists with a department of Allergy-Immunology.Hope above suggestions will be helpful to your brother.Should you have any further query, please feel free to ask at HCM.Wish him the best of the health.Thank you & Regards."
},
{
"id": 208021,
"tgt": "What can cause loss of sleep?",
"src": "Patient: Hi, i totally confused. i dont really know what is wrong or right. i mean i did not sleep wel because i just cant stop thinking. i went to the gym 4/5 times in week. i dont have any appetite whatsoever. i hardly eat. i know i suppose to eat more than 5 times because of my work out routine. but i just cant eat. maybe once or twice of amount of food. i m continuously feel depression. because i moved a new place and sure i have homesickness. and to be honest sometimes i do believe that being on this earth was not my choice and i came this world against my will. so i dont give a honeywaffle if commit suicide or not. then i think about my dad (the best dad ever) and my mom. i dont know what i do. any good advice would be truely appreciated. thanks.... yours lonelyboy Doctor: HiiI admire you for positive outlook.Daily routine and stress cause interfere in sleep.certain routines and lifestyle modification will help you likeFix sleep schedule and stick on itNo day time sleepExercise that makes you fatigueStress free activities and meditationsAvoid alcohol and smokingAvoid heavy meal in dinnerAvoid tea coffee soft drink at nightUse bed when you feel sleepyread up to when you will sleepyNo talking or use mobile in bedCalm and cool bed roomall of this will help you to restore good sleepBesides this there are some medicines like benzodiazapine and zolpidame which are useful in treat sleep problemsbut you do not require just follow what i have saidGet well soonThank you"
},
{
"id": 127179,
"tgt": "What does a painful blister with a red spot on the heel indicate?",
"src": "Patient: I have developed a blister with very little fluid and a red circular dot in the middle of my left heel...I stepped on a rock through my house shoes about 2 months ago and not long after this the red dot appeared and now a blister has developed. What can this be. It is painful Doctor: Hello and Welcome to \u2018Ask A Doctor\u2019 service. I have reviewed your query and here is my advice. It could be due excessive friction while stepping on stone.If it is very painful interfering with walking then you need to remove it by piercing and taking off fluid. It need to be covered with cotton dressing. So you need medical attention. Hope I have answered your query. Let me know if I can assist you further."
},
{
"id": 193187,
"tgt": "How to increase sperm count?",
"src": "Patient: doctor, i'm 30 years old men now. recently i have done a semen analysis, the result stated that sperm rapid progression 0%, slow progressive 33%, non-progressive 14% and immotile 53%. The doctor told me that i only have <20% can fertilise the egg. is that true? can you suggest me how to improve the progressive of sperm? i'm not a smoker and seldom take alkohol. Doctor: Hi, Take antioxidant drugs, fresh fruits, vegetables and nuts for better results. Do yoga and exercise. I Hope I have answered your query. If you have further doubts, I would be happy to help you. Happy day. Regards,Dr. S. R. Raveendran, Sexologist"
},
{
"id": 189398,
"tgt": "Pulled out a loose tooth, gums seem hanging. Roots still inside?",
"src": "Patient: My daughter (age 6) pulled a very loose top front tooth out this evening. The tooth came out easily, but there is a dangling piece of her gums left hanging in its place. The tooth itself is hollow, it looks as if the hanging bit of gums came from inside the tooth. Will the hanging gums go away or is there a chance the roots of the tooth are still inside? Do I need to take her to the dentist? Doctor: Hi, Thanks for aksing the query, There is nothing serious about this, there is exfoliation of primary tooth and erruption of permanent tooth due to which there can be a piece of loose gum hanging. As the permanent tooth is not fully errupted, it will disappear when the tooth will fully errupt. The age of your child is six, so there you can see the errrupting permanent tooth from inside which is not the roots of primary tooth. The roots of primary tooth undergoes physiological resorption during its exfoliation. Do not worry, maintain a good oral hygiene. Hope this helps out. Regards."
},
{
"id": 13331,
"tgt": "Suggest cure for severe prickly itching on the arms",
"src": "Patient: I have had severe prickly itching of my upper arms for several years. I have seen 2 different dermatologists and been treated with steroid creams and OTC creams, and UV light therapy. I am still experiencing the pain and itching and have considered acupuncture. I have rheumatoid arthritis, and am on methotrexate and enbril but have stopped all meds for the past 8 weeks because I cannot stand the itching anymore. I thought perhaps it was an allergy. I still have extreme pruritis of my upper arms. I am an RN. I need some help Doctor: Hi,The lesions on upper arms may be keratosis pilaris. The skin on upper arms may be xerotic so,you get severe itching. Apply moisturiser cream containing liquid paraffin frequently.Avoid soap bath. Take bath in olive oil containing water to keep the skin moist. Take hydroxizine HCL 25 mg tablet daily at night and desloratidine 5 mg tablet in the morning. Do not stop treatment of rheumatoid arthritis. Hope I have answered your query. Let me know if I can assist you further. Take care Regards, Dr Ilyas Patel MDDermatologist"
},
{
"id": 120906,
"tgt": "What causes numbness in fingers?",
"src": "Patient: This morning I had numbness and tingling in the fingers on my left hand, it lasted for about 4 hours. I am now experience the same sensation in my lower lip on the left side. I am 29, do not smoke or use drugs, occasional glass of wine. I recently had my bloodwork done and it all came back within normal range. My mother has MS, not sure if this could be related in any way. Doctor: Hello,The feeling of numbness that you are having can be related to electrolyte disturbances or decreased fluid intake. You need to get your electrolyte levels and serum Vitamin B 12 levels checked and start with proper supplements accordingly.Hope I have answered your question. Let me know if I can assist you further. Regards, Dr. Praveen Tayal, Orthopedic Surgeon"
},
{
"id": 179767,
"tgt": "Is it safe to give Duolin to months old baby?",
"src": "Patient: Hello. My son is 7 months old, having very bad coughing n cold frm last 3 weeks. Doctor initially adviced me to mix half respule of duolin 2.5 mg and half respule of budecort 0.5 mg. he didn t recover so then adviced to give half duolin of 2.5 mg first. After 10min budecort 0.5 mg full. It s week nw but nt recover. Why it s not affecting and what should I do now ? I m giving 2.5 mg duolin to my 23 months old doughter for cold and cough. Is it safe. Thank you. Alifiya Doctor: Thanks for asking your question at HealthCareMagic. Your son is probably suffering from a condition called bronchiolitis which is caused by a virus and affects children between 6 months to 3 years of age. In most cases it is a self limiting disease but often requires nebulisation. If he is wheezing, duolin needs to be given. But if he is not, rather give him nebulisation with 3% NaCl solution (hypertonic saline) and it should help. Do not give budecort more than twice a day. I am unable to listen to the chest of the child and needed to evaluate whether there is likelihood of superadded bacterial infection which needs to be treated with antibiotics. Some antiallergic like levocetirizine also might be required. He may also need to be assessed for chronic diseases like tuberculosis. Get in touch with your doctor who would be able to tell you more.The commonest side effect of duolin is increased heart rate. About the side-effects, they are there but you need not worry. You need to get the disease treated first.Hope that helps. Feel free to revert back with further queries if any."
},
{
"id": 215303,
"tgt": "What causes tingling feeling and sharp pain in my feet?",
"src": "Patient: I have recently moved from Minnesota to Arizona. I have a history of ankle swelling, along with tingling and sharp shooting pains in my feet. My physician in MN diagnosed this as peripheral neuropathy. I am not diabetic. I am 28 years sober from ETOH, marijuana. For several yrs I have been prescribed antidepressants, most recently Effexor XR and Bupropion. Because I knew I would not immediately have insurance, I weaned myself from 225 mg of Venlaxafine to 75 mg daily. This was done over a 2 month period. I have also run out of the Bupropion, as well as the HCTZ & Metropolol, Pravastatin and folic acid I was taking. The main problem I have is the extreme swelling of my calves, ankles and feet. I stay off my feet as much as possible and elevate them above my heart, but that only helps for the time I m doing it. I have no insurance, my bp is in the neighborhood of 123/78. I m 5 7 , 190#. I hope you can offer me some help. Thank you. Joy D. Webb Doctor: Hello!Welcome to Ask a Doctor service!I understand your concern and would explain that your symptoms are suggestive of increased fluid retention. For this reason, it is important performing some tests (a cardiac ultrasound, blood electrolytes, kidney and liver function tests). Starting a diuretic like furosemide may help improve your situation. Hope to have been helpful!Wishing all the best, Dr. Ilir Sharka, Cardiologist"
},
{
"id": 104070,
"tgt": "Having allergic reaction after certain food. Applied moisturizer. What can i apply ??",
"src": "Patient: Hi I have sensitive skin and over the last few years notice reactions with redness and burning after certain foods/drink but then recently, I had a baby and now I have dry flaky skin on my forehead and cheeks- nothing clears it Ive tried barrier cream top of line moisturisers for sensitive skin and just plain moisturiser.. now my face is burning to touch do i have a condition or is this hormonal and what can i do to make it better? Doctor: this can be after effects of pain killers like paracetamol brufen calcium bcomplex dietry suppliments and multivitamins you think about any medicoine taken you can take anti aleric cleritin 10 mg night for 10 days can apply moisteriser and mix with clindamycin gel becase if you apply moisteriser alone it can form pimples on face use non toxic soaps oils cream and shampoo no oils and spicy food"
},
{
"id": 63910,
"tgt": "What is the treatment for a lump on the shoulder?",
"src": "Patient: I have a lump, its more of my shoulder than my neck. But its not in the back area of my shoulder. If u were to run if finger down from my ear the Kim as just as my neck turns into the top of my shoulder, almost closer to 2 inches down from where my back hairline stops. I popped up around 4am while I was working. Doctor: Hi,Dear,Good Evening and Thanks for the query to HCM.I studied your query in details and understood your concerns.@My impression of your lump on the Shoulder-is as follows--Could be-Sebaceous Cyst?Epidermoid Tumour?Lipoma on the nape of neck and shoulder.Treatment Suggested-Surgical Excision Biopsy - is the better surgery.at the hands of best Surgeons.FNAC biopsy from Surgeon would fix diagnosis -from the cancerous and would treat accordinglyThis would help you to plan treatment with your doctor.@Hope this would resolve your worrisome query.Welcome for any further query in this regard,and would love to help you to resolve this problem till it is solved to your satisfaction.@ One Small Request to you to Write strong recommendations promoting my services for the benefit of my new and old patients at HCM clinic.Wishing you fast recovery..!!Have a Good Day.With Regards ,Dr.SAVASKAR M.N.Super-specialist in NCCD-Non-Curable Chronic Disorders and Rejuvenation therapies in tissue and organ failures."
},
{
"id": 19140,
"tgt": "What causes sudden nasal bleeding and drooling while having hypertension?",
"src": "Patient: my husband drove a big truck. He was found unresponsive with a nose bleed and drooling still in his truck and he died.. I was told my EMS that he was gone by the time they got there. They resusitated CPR, shocks, fluids, but they couldn t get his heart started. He was 59. What could have caused the bleeding from the nose and drooling? He was obese and was hypertensive, but it was controlled well. Didn t have usual heart symptoms. I just wish I had answers to what happened to him. Doctor: Hello!Welcome and thank you for asking on HCM!I am sorry about your husband. He seems to have suffered a sudden heart attack. The nose bleeding could be indicative of uncontrolled high blood pressure, probably related to the chest pain. Drooling is a normal reaction to the pain and does not indicate any specific medical disorder. As your husband was obese and hypertensive, which are known cardio-vascular risk factors, a heart attack seems to be the most probable cause related to this clinical scenario. Hope to have clarified some of your uncertainties!Wishing all the best, Dr. Iliri"
},
{
"id": 97400,
"tgt": "Can alternative accupressure therapy be given along with homeopathy for cancer?",
"src": "Patient: my father age 65,having tongue & throat cancer for last 3 months.he has been taking homeopathy medicine for last three months.he cant eat or chew any solid food,every thing in liquid form,i want to give him the alternative acupressure therapy,so please show me the path. Doctor: Hi Welcome to HCMIhave gone thru your query regarding your father's sufferings due to cancer of throat & tongue. It is a matter of great concern .Yes you can apply Accupressure Pls consult a specialist .In my opinion , you should take guidance from an oncologist for actual treatment as the condition of the patient has gone beyond the hands of Alternative therapies .Alternative therapies can help you in the begining of the problem & help you strengthening your immune system by following regular life style - proper food habits - intake of fiber , fresh fruit , green leafy veges, vitamins ,minerals all antioxidents as garlic , ginger , onion , in raw form , lot of water Turmeric powder with a cup of hot milk in b/f and at bed time twice and lemon juice with water before meals are anti biotic ,kills bacteria in mass bases and gives resistance to our body from diseases . Aloe vera juice freash 20- 25 ml twice and coconut water also can stimulate the immune system to produce cancer killing chemicals. >Regular exercise ,yoga pranayam -deep breathing andpropr, rest ,meditation ect; and avoidance of fried, fast foods , te coffee , alcohol smoking , mental worry anxiety .and constipation , all strengthen our immune system ,helps to lead healthy happy life .ask him to do gargles with salty luke warm water twice daily to detoxify the system I suggest ,You can try Magnato therapy treatment is good potant tool of treatment without any side effect goes with any therapy and kills becteria on mass bases If you have any therapist around ,or if you can arranage a set of 2 magnets of SUM 500 power ,inform me. I can give you info about system of using and how it works , though it is simple to use but dos and donts are there to be followed . Give him Homeopathic Carcinocin 200 fortnightly 3 doses .If already giving pls ignoreHope this helps solve your query ,Take care .All the best Wish him fast recovery Don't hesitate to get back if have any doubt at drsuchda@gmail.com"
},
{
"id": 16255,
"tgt": "Rash like areas around the sides of nose and upper lip that itch and flake. Need medication ?",
"src": "Patient: Hey. Im a 28 year old male and my condition is on the face. It started about 4 years ago. What happens is that i will get these rash like areas around the sides of my nose and upper lip that will itch and flake once i wash the areas. It will go away for a bit but it comes right back in the exact same areas and looks virtually identical as the previous affected areas befor. Doctor: hello I would be pleased to answer your question . Seborrheic Dematitis: These grains sound like small crusts and your condition sounds like seborrheic dermatitis. This is a condition where the body loses tolerance for a yeast called Malassezia (which is present normally on all our skins) and starts attacking it leading to symptoms. This condition responds fastest to a steroid and antiyeast lotion applied on the area twice a day. This requires a prescription and your doctor can write one for you. If you cannot see your doctor for any reason then here is what I would recommend trying: Nizoral- cream: to apply alternate day (e.g Mon-Wed-Fri). Use regularly for 4-6 weeks then start decreasing the frequency of application gradually till you reach once a week and maintain. Also use over the counter cortaid cream ( steroid creams) Avoid picking the area . Please feel free to ask if you have any queries.Thanks and Take care"
},
{
"id": 166792,
"tgt": "Suggest medication for redness around the base of penis",
"src": "Patient: My 6 month old baby boy has had redness around the base of his penis for about a week. it seems to have gotten worse and a little more red towards the tip and he is very restless and getting upset... I put A&D ointment on it earlier but seems to have given him no relief. Doctor: Hi,Skin conditions are best diagnosed only after seeing directly. I suggest you to upload photographs of the same on this website, so that I can guide you scientifically. Hope my answer was helpful for you. I am happy to help any time. If you do not have any clarifications, you can close the discussion and rate the answer. Wish your kid good health.Regards,Dr. Sumanth MBBS., DCH., DNB (Paed)."
},
{
"id": 151957,
"tgt": "Birth asphyxia symptoms",
"src": "Patient: i need a doctor regarding case of birth Asphyxia Doctor: a girl is born on 27 jan in birth Asphyxia, she cyired after 30 days, unable to suck milk yet"
},
{
"id": 52536,
"tgt": "How further to treat liver infection with hyperthyroidism?",
"src": "Patient: Hi All, My bother(age-27) is having liver infection, hyper thyroditis and kidney stone. he has been suggested the following medicines: PAN D(mfg:ALKM), Neo-Mercazele 10(mfg:PIRML) and Zanocin OD(mfg:RNBXY). Could you please advise me in this regard. Kindly let me know if you need additonal informations Doctor: Hello and Welcome to \u2018Ask A Doctor\u2019 service. I have reviewed your query and here is my advice. You can continue all the medicines prescribed by your physician. Most of the cases improves even without treatment. Do an LFT(liver function test) to assess the progress. As of now drink plenty of water and take adequate bed rest. If in case of any symptoms like abdominal pain or fever do an ultrasound abdomen also. Hope I have answered your query. Let me know if I can assist you further."
},
{
"id": 100704,
"tgt": "What causes swollen tongue and itchy lumps on the feet?",
"src": "Patient: My tongue swells I get large patches of hives and itchy lumps in my feet. My allergist put me on cetirizine and my symptoms have lessened but not completely. I haven t had to use my epi pen. What could be causing this? I had read something about an autoimmune disorder but I forget what it was. Doctor: Hello,Thank you for asking at HCM.I went through your history and would like to make suggestions for you as follows:1. From your symptoms, I would think of urticaria with angioedema.2. Common causes are allergies, infections, drugs (mainly analgesics & antibiotics)sometimes autoimmune and sometimes cause may not be found. Also some physical factors like exercise, sunlight exposure, cold/heat exposure, water exposure, pressure etc can also trigger these symptoms.3. If the trigger is known, avoidance of the trigger is the best option.4. For symptom relief, an antihistamine like cetirizine or hydroxyzine can be used. It may take 3-7 days for complete resolution of symptoms.5. The autoimmune disorder you read may be \"autoimmune urticaria\".Hope above suggestions and information will be helpful to you.Should you have any further query, please feel free to ask at HCM.Wish you the best of the health.Thank you & Regards."
},
{
"id": 84657,
"tgt": "What are the side effects of metamucil?",
"src": "Patient: Hello, I m 29 years old and I was taking Metamucil once twice a day. Then a month later I m having trouble pooping and my urine flow has slow down. Sometimes I can urinate with medium stream and sometimes a light to dribble stream. At first I was urinating every 10 to 20 mins. But through the week I m not having to go for hours at a time. I have no blood in my urine and I have no pain at all. I will have a tense in the abdominal sometimes but not all the time. Doctor: Hello,Your symptoms seem to be related to metamucil use. It is a bulk-forming fiber laxative and can cause changes in bowel habits, flatulence, gas. So, I suggest to stop the treatment and use simethicone medication such sa Espumisan to relieve the bloating. I also recommend to eat a health diet and drink liquids. This medication does not influence in the urinary tract,so you should do a urinalysis for further evaluation.Hope I have answered your query. Let me know if I can assist you further. Regards, Dr. Dorina Gurabardhi, General & Family Physician"
},
{
"id": 28472,
"tgt": "Suggest treatment for stroke and high blood pressure",
"src": "Patient: Hi my mum had a stroke a bit over a year ago. all has been fine except for now her blood pressure is high and the Karvea 300mg tablets do not seem to be working anymore her GP doesn t seem to want to put her on anything else so we suggested could she swap to 150mg of Karvea twice a day to see if it worked in keeping her blood pressure down during the evening because that is when it rises to around 190/100. He has agreed to split the dose from 300mg once a day to 150mg twice a day but I am now wondering if we did the right thing or will that raise her blood pressure even more . Do the 300mg tablet and the 150mg tablet release the same amount throughout the day Please help can you give me your advice on this as I need a second opinion Doctor: Hi welcome to HCM.I understand your query and concern.300mg and 150 mg release different amount of drug everyday.Hence it has to taken as 300 mg of Karvea to have adequate good control to prevent future stroke.I advise you to have a baseline 2 dimensional echocardiography,ECG and lipid profile to assess the basic cardiac reserve of your heart.Restrict the intake of salt to less than 6g/day.Regular physical exercise in the form of brisk walk for 20 min a day for 5 days a week is pretty useful.Drugs like antihypertensives and antiarrhythmics will help.Reduce the intake of fatty and fried food.One pomegranate a day will help to keep your heart at good pace without clot formation. Consult a Cardiologist for further expert management.Post your further queries if any.Thank you."
},
{
"id": 88550,
"tgt": "What causes lower abdominal pain after IUI?",
"src": "Patient: LMP - 6/feb/2014IUI done on 24/Feb/2014.Severe pain in lower abdomen for the next 2 days. day by day it decreased. Doctor gave us Miprogen 400(Vagina) to use daily. 4Mar/2014 - discreet pain in lower abdomen again. Is it pain of pregnancy? Nipples are paining for the past 4 days.When can i check for pregnancy? Doctor: For IUI or intrauterine insemination, after speculum examination, a probe needs to be inserted through the cervical canal into the uterus and the processed seminal fluid is injected. During the speculum examination, it is natural for the muscles of the area to get strained and hence localised muscle spasm is a possible cause of the pain that you are talking about.Another probability is that the ovarian hyperstimulation that is required for such a procedure might be a possible cause of such a pain. It might also be related to the nipple soreness.In either case, the pain should gradually subside.It is recommended to wait at least 14 days before taking a pregnancy test after IUI."
},
{
"id": 43073,
"tgt": "Should i go for infertility treatments like IUI or IVF?",
"src": "Patient: I have been trying to concieve past 8 months , have been taking clomid for ovulation , i have had prolactin level 57 when i started treatment , now my prolactin is 34 , kindly let me know if i will be able to concieve, should i go for any other method like iuf or ivf for concieving , Doctor: Hi,Welcome to healthcare magic.I read your query and understand your concerns.Following is my reply :1) Prolactin is still high please take treatment from an endocrinologist for the same.2) Once prolactin levels are low, you can go for IUI depending upon your age or IVF. 3) Please provide more details which will help me to answer you betterLet me know if you have any more queries. I will be happy to answer.Regards,Dr. Mahesh Koregol"
},
{
"id": 153903,
"tgt": "How can appetite be improved?",
"src": "Patient: Hi my Dad is 87 with MDS - rbc transfusions only, every 2 weeks, Prostate Cancer (inoperable), hormone therapy at present, had radiotherapy, previous non-hodgkins lymphoma 20 yrs ago - small bowel, operated & chemo, large harmless cyst on kidney, varicose veins, heart failure & ab fib. Has loss of appetite, sometimes metalic taste in mouth, reflux and very sore gums, which are pale in colour ie not red/blistered. Often has white foam/froth in his mouth. Any suggestsions to improve appetite & why is gums are so sore would be appreciated. He does not want to go on those food supplement drinks :-) Doctor: Hi,Thanks for writing in. Please see that your father gets 1800 kcal food energy daily and most of this might be given as fluids for better acceptance and energy utilization. Sore gums and generalized weakness might be due to malnutrition and combined vitamin and mineral deficiency.His general condition requires to be improved and this can be done by providing him minimum daily nutritional and fluid requirements. You can include protein and vitamin supplements to give him extra energy. It is possible that he is getting emotionally weak due to illness. Please encourage him to eat and drink fluids regularly. Please do not worry."
},
{
"id": 188316,
"tgt": "Cuts on corner of mouth, irritation below lower lip. Taken Desonide for eyelid dryness",
"src": "Patient: i believe i have angular Cheilitis. I have had cuts on the corner of my mouth and irritation below my lower lip for over 3 weeks now. Difficult for them to heal because i open my mouth a lot to eat. what can i do? i was given desonide for 0.05 for eyelid dryness about 8 months ago. I still have some, should I apply some of that? Doctor: Hi, No, I would not recommend use desonide. Instead, do mouth swab culture testing for candida infection, a complete blood profile for anemia. Until the test results are in, can use a thin layer of butter or olive oil and honey. Drink plenty of water and take good hygiene of your mouth.Hope it helped!Dr.Bardha"
},
{
"id": 102028,
"tgt": "Could the outbreak of caner sores be due to an allergy to Pistachios?",
"src": "Patient: I get canker sores a lot and I've been trying to figure out why. I love Pistachios and noticed that the last couple of times I've eaten them, I end up with severe canker sores. I eat other nuts and have never had a reaction. Could I be allergic to pistachios and not to other nuts, or could it just be a coincidence that I broke out after eating pistachios? Doctor: Hi, it shows that you are allergic to pitachios , but are you sure that you are not allergic to other tree nuts like almond or cashew, peanut and ground nut are not tree nuts. even this is possible as some people are allergic to some specific nuts protein and least to others. but it can be diagnosed by patch test and it can confirm where you are allergic to other or not also, so go for this and till then completely avoid its consumption. for the treatment of your canker you can take local glycerine borax, B complex forte tab, maintain good oral hygeine by aqueous chlorhexidine 0.2% mouthwash , can use triamcinolone local paste , for pain you can take lignocaine mouth gel or paracetamol tab 6 hourly. if your patch test comes negative for pistachios then you should meet an ENT specialist to rule out other causes and treat them accordingly. thanks and take care."
},
{
"id": 87271,
"tgt": "What causes sharp pains in my lower abdomen?",
"src": "Patient: HI Dr. Jain.I have sharp pains by my lower right abdomen which I assume is my right fallopian tube. I am 21 years old and I am curious why I have these sharp pains that come and go. What can I do to prevent this and how can I treat it. I have been told many times its because my feet are always cold but I expect a more logical explanation. Thank you for your help.Best, Marina K Doctor: Hi,From history it seems that you might be having chronic appendicitis giving this problem often.Consult your doctor when pain is present and get clinical examination done.Appendicitis can be well judged when pain is there.After examination plan of treatment can be decided.Ok and take care."
},
{
"id": 88483,
"tgt": "Suggest treatment for abdominal pain",
"src": "Patient: 2 days ago I had a couple drinks. Yesterday I had a bad migraine with vomiting. The migraine went away but I still had nausea then last night I started having diarrhea. The nausea and diarrhea have not left and in the middle of the night I started having g bad abdominal pain i my higher abdomen that hurts when touched or any pressure is put on it. It hurts on both sides but the right side hurts worse. Doctor: HelloAll these symptoms started after alcohol . Usually alcohol causes hyper acidity and this produces vomiting , nausea , abdominal discomfort , headache ( of course some times if alcohol is more may develop diarrhea).If there is severe vomiting , diarrhea and abdominal discomfort then consult a primary care physician and get his opinion.Also take some antacid and tablet pantaprazole-d according to necessity .Usually diarrhea is self limiting in such cases but if severe take anti diarrheal tablets.So deal as mentioned above."
},
{
"id": 147894,
"tgt": "Meaning of chronic microvascular ischemic change in MRI of brain?",
"src": "Patient: I am a 65 year-old white female who recently had an MRI of the brain. Symptoms include numbness and tingling on the right side of my face, my right hand lower part of the right arm, 2 1/2. weeks in duration so far. The MRI showed chronic microvascular ischemic change. What does that mean? Doctor: HelloThanks for writing to HCMMicrovascular ischemic changes(decreased blood supply) in brain typically involves small blood vessels.You need monitoring of conditions that lead to ischemic changes like hypertension,altered lipid profile,diabetes mellitus.You need investigations like routine hemogram,RBS,LFT,RFT,Lipid profile,ultrasound of abdomen.If there is any more physical problems then it should be treated to prevent further progression.Get well soon.Hope i have answered your query.Take CareDr.Indu Bhushan"
},
{
"id": 136654,
"tgt": "What causes tight feeling around total knee replacement?",
"src": "Patient: I had a total knee replacement 9 weeks ago. This was my second replacement and it took forever (4 months) to get off the pain meds with the first knee. I was hoping for a better result this time. I do, however, have a different problem with this procedure. All around the incision, I feel tight, very tight. The skin surrounding the skin is very tender. There does not appear to be any infection, just warmness from time to time. I am wondering if this is a scar tissue problem and what can be done about it? I finished therapy a month ago. Doctor: Hello, I have studied your case. As pre your history suture site infection is likely diagnosisDO NOT IGNORE INFECTION IN POST TKR .You may need to investigate like CRP ,CBC,ESRPCR culture is very useful to find culprit organism.If infection is confirmed then debriedment is required .Persistent infection revision operation is required.Hope this answers your query. If you have additional questions or follow up queries then please do not hesitate in writing to us. I will be happy to answer your queries. Wishing you good health.Take care"
},
{
"id": 142110,
"tgt": "How to treat partial loss of the vision caused due to PML?",
"src": "Patient: My son has had MS for twenty years now. He ws treated with Tysabri and contracted PML He is still alive and trying his best to keep strong. His Doctor has not given him any medication since h left the hospital Is there a treatment for him? The PML caused a stroke and took most of his eyesight. He is also dependent on his wife and health care person to get around in a wheelchair. I feel there has tobe something that can helphim improve. Can you give him some direction as what to do. PLEASE Doctor: Hello!Welcome on Healthcaremagic!I am really sorry for your son's medical condition, but you should know that there is not too much to be done in his clinical situation. I would recommend taking a vitamin supplement and continue performing physiotherapy. Unfortunately there is no cure to his damaged nervous system. Physiotherapy and a balanced diet, can help improve his situation. Hope you will find this answer helpful!Best wishes, Dr. Aida"
},
{
"id": 188631,
"tgt": "Why am I having pus in the teeth ?",
"src": "Patient: sir in my teeth pus is coming sinse 3 month Doctor: Hello Pus indicates infection, you problem needs thorough investigation and proper management, as pus from teeth is indicative of many diseases, like Periodontitis, Periodontal abscess, Endo-Perio or Perio-Endo Lesions, Anachorectic infections, Alveolar bone degenatation and many more. THough you probably just have Periodontitis, a propar diagnosis and course of management are required. Antibiotics alone WILL NOT HELP. Please consult a good Dentist."
},
{
"id": 210426,
"tgt": "Is cleaning everything in room that was touched after masturbating an abnormal thing?",
"src": "Patient: when after i masturbate in the room, i wipe sperms with a old cloth.. but problem is not that..... after that i want to clean everything i touched (Like key board, mouse, computer table, door handles, chairs and everything i touched) in my room with Dettol... is it a sick? i spend more than 3 or 4 hours to do this... if i didn't do this i cant live in the room... it's unpleasant.... please help me... Doctor: Hi friend , After reading your complaints the symptoms suggestive of OCD (Obsessive Compulsive Disorder) . After masturbation a thought of unclean touched objects comes repeatedly in mind & you cant resist or stop this unpleasant thought - the symptom is called Obsession. Due to obsession you develop anxiety and to relieve anxiety you clean those touched objects - the symptom is called Compulsion . For firm diagnosis we need detailed history , Mental status examination & investigations. Generally we give SSRI group of tablets and Behavioral therapy to treat OCD . You should consult Psychiatrist soon for firm diagnosis & treatment. Take Care.Regards"
},
{
"id": 217945,
"tgt": "Having noticed pain on the right hand after having a fall",
"src": "Patient: I was playing soccer about 3 weeks ago and fell akwardly onto my right hand. It hurt but not too much so I ignored the pain and kept playing. I thought it wasn't a big deal and didn't go see my doctor. But today while I was doing a squats with the bar in front for weight room the pain got so bad I couldn't finish. It was painful around the outside, where the joint bump is. Doctor: this type of falls during playing usually leads to hair line fractures. which might need atleast an XRay( skiagram) to get diagnosed. you better visit your doctor get a prescription for the Xray and seek help accordingly."
},
{
"id": 222107,
"tgt": "How long does it take for one to conceive?",
"src": "Patient: ease type your query here...hi there iam a 31 woman ive been recieving the depo injection for 2 and a half year ive just came off it as my partner would like to have a baby ive started bleeding and its fresh blood iam highly fertile as ive got other kids how long do you think it will take me to concieve . Doctor: Hello,Once you get off the depo injection, ovulation can return after 3 weeks at the earliest. Thus, it is possible to conceive in the first cycle itself. In the majority, however, this is uncommon and it takes 3-6 months to achieve regular fertility status. Some may take even longer, as long as 2 years to return to normal hormone balance. Please maintain a healthy weight, regular exercise and a healthy diet and thus, you can expect to return to fertility sooner. Hope this helps."
},
{
"id": 164194,
"tgt": "What causes red rashes under eyes of a 14 month old?",
"src": "Patient: my 14 months old daughter has a dark red rash under both of her eyes and on her eye lids. I cannot think of anything new she possibly ate, so I am not sure that this would be a reaction to food. She did have red bumps around her mouth 2 days ago (this lasted about 4 days). I think this was a reaction to pineapple... She also had this rash around her eyes before about twice and it lasted a couple of days ... Doctor: Hi...Thank you for consulting in Health Care magic. Skin and eye conditions are best diagnosed only after seeing directly. I suggest you to upload photographs of the same on this website, so that I can guide you scientifically.Regards - Dr. Sumanth"
},
{
"id": 160535,
"tgt": "What causes severe itching in leg after taking Calpol?",
"src": "Patient: My son is 5 years nd 10 months old. He has got fever on last wednesday and after that he has been sent for diagnosis for dengue. The result was negative. However on sunday he as got fever again in night 101 deg F & as instructed my doctor we had given him calpol, After that on monday he was normal an after half day (after noon) he starts itching his leg and now at night the itching is so severe that we are sitting there and itching his leg, basically near toe,bottom most part. Please advise. Doctor: Hi dear,If you had given him paracetamol in the past without a rash or itching, this is unlikely related to paracetamol. It is probably a part of fever itself.If he do not have any other symptoms, fever and itching indicates a viral illness. He may also develop a rash in coming days. If he is active and playful, we need not worry. Just continue paracetamol 3 times a day for 3 days. Also give antihistamine to relieve itching. Syrup hydroxizine(10mg/5ml) given 4ml 3 times a day will be sufficient for him.If he is not getting better by 2-3 days, kindly take him back to his doctor.Hope I have answered your queryLet me know if I can assist you furtherDr Muhammed Aslam, Paediatrician"
},
{
"id": 212274,
"tgt": "Have fidgeting, distraction, sleeplessness. No medication till now. What to do?",
"src": "Patient: hi I am a 14 year old female and I am worried I might have adhd. I always seem to be figeting, my hands are always moving when everyone else is still (like in photos). I alwqys struggle to do my prep on time because I get distracted by other things or people easily, I often forget what my homework was so I don't do it. I cant sleep at night but always wake up early. I seem to always be talking when im around people buyt not when im alone and I don't like silence! please tell me if I should see someone about this or if its just hormones or whatever!!! Doctor: Hi there ~ Thank you for your question and I am glad that you ask this on a public forum. It may be that you actually do have ADHD, however it needs to be corroborated with a history from your parents and you need to take yourself along with them to a psychiatrist who can help diagnose you and if needed treat you for the condition. I hope you do this as soon as you can. I hope this helps. Take care and have a lovely day!"
},
{
"id": 65733,
"tgt": "Suggest treatment for hemorrhoids",
"src": "Patient: I am 44 yr old female 5'3\" 110 lbs with a history of hemorrhoids.They usually go away by increasing fiber. This time they are getting worse. the pain is unbearable at times.This has lasted for 6 days. There is also a significant amt. of bright red blood during b.m.s which has happened in the past. What would be the best method of treatment? I'm going to see my PCP today. Doctor: Hi,It seems that there might be having infected hemorrhoids giving this problem.It might require surgery to get complete cure.Meanwhile go for one antibiotic medicine course for 5 days.Avoid constipation.Avoid fried, chillies and junk food.Take plenty of water.Ok and take care."
},
{
"id": 51307,
"tgt": "Have kidney stones. Can I gain weight? Why am I not interested in sex?",
"src": "Patient: hi i m 27 years old,weight 58kg ,height 5.9 ... i wanna gain weight.. well i have kidney stones from a long time does it effect me in gaining weight? and recently my haemoglobin also reduced but i m taking iron content food and have almost gained normal haemoglobin..one more query is that i had a real good sex drive until the age of 20 n then it reduced by time and now i m not so interested in sex i can ve sex but i dont enjoy it.. i live in a hill station ooty(tn) and my body contains a lot of heat.. well i want to gain weight trough endura mass should i?? or what is the best way to solve all these problems??? i m still single n marriage is on the way in about 6 months or so..within that time period i want to gain atleast 10kg,n wanna feel more horny like before .. please tell me what should i do?? Doctor: Any high protein diet with higher amount of calcium Or animal protein like meat or egg is likely to increase ur chance of developing renal stones. Anyway sex before marriage is more harmful- try avoiding that if u wanna stay healthy. Once Ur settled in life Ur sexual life will be like before."
},
{
"id": 106091,
"tgt": "Medications in Asthma, should I go for steroids ?",
"src": "Patient: I on deriphylline tablets for asthama, but when it worsens then only i will use asthalin pumps, now a days neither the tablets nor the pumps help me. i am tired most of the times, should i go for steroids ? Doctor: is better to go with steroids and inhalers under strict supervision of chest physician or physician."
},
{
"id": 98351,
"tgt": "How can hay fever be treated?",
"src": "Patient: Hi I have hay fever with all the symptom and the line across my nose. I have post nasal drip and I deal with it through my nose. I\u2019ve taken antihistamine for this but fine very little lasting relief. Now, due to the drip I have the fluid in my chest. I\u2019m coughing a lot and now appear to have a temperature. I\u2019m affected by dust/dander and plants. I currently have all of these in my life due to job and neighbors tree. It\u2019s an overload to my system and I can\u2019t cope. I need help as I\u2019ve had this for some time and my doctor seems to have no idea how to effectively deal with this Thank you Sharon Doctor: Hello and Welcome to \u2018Ask A Doctor\u2019 service. I have reviewed your query and here is my advice. You have severe allergic rhinitis and allergic alveolitis. For this you have to go for skin allergy test with 140 allergens and it will tell you why actually you have problems. Other things total serum IgE and IgE specific for aspergillus. Treatment options are long term steroids Montelukast and Levocetirizine , Omalizumab and immunotherapy treatment Hope I have answered your query. Let me know if I can assist you further."
},
{
"id": 123923,
"tgt": "What causes shoulder pain and headaches after giving birth?",
"src": "Patient: Hi, I have a pain in my left shoulder/ arm and left side of my chest the last week or so. I also have sharp shooting headaches for about a week also. I had a baby ten weeks ago if that makes any difference? When I was in labour I had a severe pain in the top of my spine which also hurts every ow and again. Doctor: Hello, As I see that most people usually don't complain of the upper limb pain post delivery, but very few complains have been received. This might be due to degenerative changes in the intervertebral disc. Taking an MRI of the cervical spine along with lumbar spine after a gyncec certification would help understand the pathologies better. Using a soft cervical collar will help stabilize the cervical spine and reduce the cervicogenic headache and upper limb pain. In my clinical practice of over 12 years, most cases post delivery undergo Physiotherapy for better health and muscle strength. Thereby reducing the symptoms and helping them for ease, will recommend you the same. Hope I have answered your query. Let me know if I can assist you further. Take care Regards, Jay Indravadan Patel, Physical Therapist or Physiotherapist"
},
{
"id": 184281,
"tgt": "Can tooth decay cause heart problems?",
"src": "Patient: 23yr old son has a heart murmer, tooth decay. He passed out at work and they done ekg. doctor told him the right side of his heart wasnt working. he has to go to a cardiologist. They asked if he done drugs. He did for about 2yrs, pain pills. my question is wont bad teeth cause heart problems? what does this mean for someone with a murmer? Doctor: Valvular heart disease can be accentuated with dental bacteria . Literature has ample proofs. That is why prophylactic amoxycyllin is given to cardiac patients before dental treatment."
},
{
"id": 196800,
"tgt": "What causes lower groin pain and testicle pain which worsens with erection?",
"src": "Patient: i am 36 for the last three years i have had a dull aching pain in the right lower groin and occasionally testicles. it worsens with sitting and erection. I have seen four different drs. including a surgeon and a urologist. it seems to be more constant now. What is wrong with me Doctor: Hi and welcome to Healthcaremagic. Thank you for your query. I am Dr. Rommstein, I understand your concerns and I will try to help you as much as I can.All these symptoms are suggestive of epididimitis which is inflammation of part of testicle and this not uncommon in males especially in young age. This is usually treated with 10 days of antibiotics, usually ciprofloxacin and there are no permanent consequances. MAsturbation and intercourse should be avoided till pain persist, if possible since I may prolong inflammation course. Some more serious conditions such as tumors or torsion are rare but in doubtful cases you should do Doppler ultrasound or scintigraphy. In this particular case, I don\u2019t think this is required.I hope I have answered you query. If you have any further questions you can contact us in every time.Kindly regards. Wish you a good health."
},
{
"id": 203995,
"tgt": "Not getting a proper orgasm. Do I have ejaculatory anhedonia? Had vasectomy in the past",
"src": "Patient: The last three times I ve had sex with my wife I had no feeling of orgasm at ejaculation . I m doing nothing different, I ve never had a problem before, I m 55 years old, in great shape. I have read about ejaculatory anhedonia and it describes exactly what I have experienced. What can I do to get things back to normal? I also had a vasectomy about 15 to 16 years ago. Any suggestions? Doctor: Hello,Thanks for choosing health care magic for posting your query.I have gone through your question in detail and I can understand what you are going through.Ejaculatory anhedonia could be due to psychological causes. You need to be evaluated for depression. If there is depression, there could be ejaculatory anhedonia. Hope I am able to answer your concerns.If you have any further query, I would be glad to help you.In future if you wish to contact me directly, you can use the below mentioned link:bit.ly/dr-srikanth-reddy"
},
{
"id": 40478,
"tgt": "What causes vaginal spotting after stopping Duphaston?",
"src": "Patient: hello , i went through iui on 4rth april and was taking Duphaston 10mg twice a day of 15 days ... i continued taking dupaston and did my first pregancy test which was negative ... than i took beta HCG test which was also negative. then i called my doctor and she told me to stop duphaston and wait for 3 days . she told me that that either i ll have period or a positive result. today i again take home pregancy test it is negative and light bleeding .. as i have intercourse yesterday .. so the bleeding started by that .. Doctor: Hello and Welcome to \u2018Ask A Doctor\u2019 service. I have reviewed your query and here is my advice. You have been getting negative pregnancy test results and started spotting, it is likely to be premenstrual spotting. you are likely to get your period soon. If not, repeat the pregnancy test. Hope I have answered your query. Let me know if I can assist you further."
},
{
"id": 155657,
"tgt": "Does small dark spot on testicle means cancer?",
"src": "Patient: My dad (59) yrs old did a sonogram on his testicle and was found 3 small dark spot. Could it be cancer? There are the sonogram finding: There are 3 irregularly bordered small hypoechoic defects present in the left testicle. The largest is 0.3 cm, towards the mid- lower pole. The other two are upper pole. On image submitted there was no Doppler suggestion of vascularity within these lesions, on repeat real-time reviews, there appears to be internal vascular to the lesions. There is a small anechoic defect within the midsole of he left testicle which real time color dropper indicated was a vessel. Doctor: Thanks for your question on HCM. In my opinion no need to worry much. As these spots indicate vessels only. So get done repeat ultrasound after 3 months. If spots are similar than no need to worry. They are just vascular lesion only.But if they are enlarging than biopsy is needed.If you really want to rule out malignancy at present than get done CEA , AFP and HCG level. If all these are normal than no need to worry about malignancy. Get done repeat ultrasound after 3 months."
},
{
"id": 97466,
"tgt": "Does acupuncture have any side effects?",
"src": "Patient: I've been getting acupuncture for abdominal pain. The acupuncturist will put a needle on the inside of my ankle. It almost shocks me when he puts it in and it makes my whole leg jerk. This is supposedly for my rectum, as I have inflammation in my rectum and colon. I have been for 4 or 5 treatments. Just today I've noticed when I get out of my car seat or move in a particular way that I feel this tingling weird sensation. It has me very concerned. Any thoughts? Thank you. Doctor: Hi Welcome to HCMI went through your query and found you are going through a painful therapy . I really feel concerned .You asked ' if it has side effects ' No , no side effects .as far as I know . But only problem is pinch of needles you have to tolerate . I have worked with Magnets & magnetised water and acupressure very effectively . At least they are painless therapies giving 90% positive result . Rest depends on receptivity of the subject .Hope it helps you in your query All the best take careDon't hesitate to come back with next query in any ."
},
{
"id": 3226,
"tgt": "Are there pregnancy chances after having sex while on inactive pill?",
"src": "Patient: Hi. I m on ginette for 2 months,I just started my 3rd pack, and awaiting my period.. I had unprotected sex,whiles on my inactive pill,I have also taken pain killers,and I read that certain tablets mess with your contraceptive.... Could I get pregnant? Do I need to take a morning after pill? Please help. Thank you Doctor: Hello dearI understand your concernThere is no chance of the pregnancy.You are on ginette since last two months.It is highly effective in preventing the pregnancy.It has just 0.5-1% failure rate.Pregnancy will not occur as you did sex while you was on inactive pill.Again it is very safe period of the cycle.Pain killer does not affect the efficiency of the ginette.Some antibiotic like Rifampicine, Amoxicillin, ampicillin, griseofulvin, metronidazole, tetracycline, sulfamethoxazole/trimethoprim. tetracycline, minocycline, metronidazole, and nitrofurantoin.So there is no chance of pregnancy.Avoid stress, take healthy diet, drink plenty of water and do regular exercise.Hope this may help youContact HCM for further health queryBest regardsDr. Sagar"
},
{
"id": 223012,
"tgt": "Why is one unable to conceive?",
"src": "Patient: Hi doctor my name is Sarah. I am sending this message from bangalore. (south india) I got married at 2005. But still I don't have children. I have visit one of famous hospital here in bangalore. As the doctor report I have thyronorm problem in overy. I have all doctors report details. So can I hear help from your side? Doctor: I think thyronorm is a tablet which you must be taking for thyroid problem. There is no such disease of ovary. There are various causes of infertility and both male and female investigation is required."
},
{
"id": 39757,
"tgt": "Suggest medications for puss filled painfull bumps on thighs",
"src": "Patient: For the past few years I have been getting these painful bumps on my inner thighs. They range from dime size to quarter size. It comes to a point when these bumps or blister pop open and produce blood and puss? What are these and what can I do to stop them? Doctor: These appear to be folliculitis from the history provided. Do you sweat a lot? The best way to avoid them is to maintain proper hygiene. Take care"
},
{
"id": 94695,
"tgt": "Only on soft foods, soup after getting partial colon removed. Still having pain. Suggestions?",
"src": "Patient: I had 18 inches of my colon removed in August then two weeks later developed abcessess and was hospitalized , they placed a JP drain and I went home on I ve abx. And a PICC line. I am still having a lot of pain and nausea have been hospitalized with this again recently. I am finding that I can now only eat pudding, soup very soft foods, or my pain inreases in my stomach greatly. What could be the problem? My surgeons say there is nothing more they can find. I still feel like there is something wrong. Doctor: Hello! Thank you for the query. First of all it would be good to know the reason of the abscess creation. Usually there is no reason for abscess to create during the regular colon removal. Such abscess presence usually means that there is a leakage in the intestinal anastomosis (surgical connection between two parts of intestine previously detached). Such leakage results with stool leakage to the abdominal cavity. Such situation causes inflammatory process inside the abdominal cavity causing adhesions creation and anastomosis narrowing. Both adhesions and narrow anastomosis impedes stool passage and results with symptoms like yours. That is why you should have abdominal CT with oral contrast performed (so called enteroclysis) to see if are there any obstacles. Full blood work with CRP (inflammation marker) should be performed to see if there is no other abscess. If no findings in CT, you may have performed laparoscopy as very often adhesions can not be see in the CT. Hope this will help. Regards."
},
{
"id": 101703,
"tgt": "What causes sneezing with watery eyes?",
"src": "Patient: hello doctor am 55 yr old female weighing 90 kg, suffering from continuous sneezing accompanied with watery eyes and slimy, oil like discharge from nose throughout. am suffering this kind of probelm for more than 15 years. i tried various tablets, anti allergy tablets but of no use. i need some relief so that i can do my day-to-day work without running nose and sneezing. plz help doctor. mallika n Doctor: hello,thanks for your query, continous sneezes followed by running watery secretion could be due to allergic rhinitis. as you are having since 15yrs, u need to evaluate for allergic polyp and chronic sinusitis by ct-pns. as such steroidal nasal sprays are best medicine for allergic rhinitis , we dont know ur diabetes status.kindly visit an ent surgeon evaluate by doing ct pns, grbs.all the best. take care."
},
{
"id": 24659,
"tgt": "What could cause breathlessness?",
"src": "Patient: Hi, my Grnadmother is 84 yrs old and has been having kidney haem dialysis for 8 weeks. She was okay for a few weeks but has lately been very breathless while receiving it and has been unable to receive the full amounts due to this. Could this be due to an underlying heart problem? She also smokes so it may be a sy ptom of emphysaemia? I also noticed in a few articles about this type of dialysis that eating bananas is to be avoided and she should be on a low potassium diet, however the hospital have advised her to eat at least one banana a day because she is low in potassium? I dont understand this, please help Doctor: Thanks for your question on Healthcare Magic. I can understand your concern. Breathlessness in patients with dialysis and active smoking can be due to fluid overload, pulmonary edema and COPD (chronic obstructive pulmonary disease). So better to get done chest x ray and PFT (pulmonary function test). If chest x ray is suggestive of pulmonary edema then she will need diuretics. If PFT is showing obstructive Airway defect then she will need inhaled bronchodilator and inhaled corticosteroids (ICS). In any case smoking cessation should be done first. About potassium level, if she is having low potassium level then banana should be given. Actually kidney patients are having high potassium level in the blood due to decreased urine output. But due to dialysis, some patients develop low potassium level. So in such patients, you should give banana to supplement potassium.Hope I have solved your query. I will be happy to help you further. Wishing good health to your grand mother. Thanks."
},
{
"id": 197771,
"tgt": "Suggest treatment for brown semen",
"src": "Patient: Presently I have brown semen. A little history and maybe you can help me. 12/23/11 I had sex and in the condom was bright red throughout, scared the shit otta me and my girlfriend. I went to the emergency room the next morning and they ran blood and urine tests and found nothing unusual. Just minor amounts of blood in the urine. I did make an appointment with an urologist and seen him a week and a half later. But prior to seeing the Dr. I started to have quite severe pains in my lower left back and nausea, and then the pain moved to the front lower abdomen. The Dr. preformed a prostate exam and urine test and found no blood in the urine. But I still had brown semen. He prescribed Flomax and a pain killer and set up a CT scan. That was yesterday. I don't get the results for another 4 days and still my semen is brown and watery. I have no pain when i urinate or ejaculate. My pain in my stomach is dull but still I have nausea. Is there anything or test I can do? I'm drinking lots of water and urinating frequently. I have had a kidney stone problem 7 years ago, and this almost feels like that, but really no blood in the urine pretty much just in my semen. Please let me know if I can do anything else for the next 4 days while I'm awaiting the test results. Thanks. Doctor: Hi,Dear,Thanks for your query to HCM.Dear I read your query and reviewed it with context to your query facts.I understood your health concerns and feel Concerned about them.Based on the facts of your query, you seem to suffer from-Chronic Bacterial Prostatitis(CBP) with urethritis mostly with ?Orchitis- from Incomplete or unnoticed- STD recurring infections lately Or could be accompanied UTI with Old Kidney stone with STD / CBP urethritis , as you have first attack of severe lower left back pains with nausea, which moved from lower left back to front lower abdomen, indicating Cysto-urethritis and ? Orchitis.Presence of no pains in urethra while ejaculation indicates chronic low lying Sub Acute CBP-infection from the bygone STD in the past, which has resurged lately.Pain in stomach and nausea points to -pyelonephritis / and could also be from Epididymo-orchitis.Presency of Kidney stone with Pyelo-nephritis with cysto-prostatis, could mimick this and needs to be ruled out and treated.Persistent brown coloured semen indicates definitive Prostatitis with all this complex story and points to the possibility of Sub acute long Prostatitis with Epididymo-orchitis lately.CT is awaited.Remedy Suggested -Continue plenty of fluids Good Sex hygine and avoid illicit sex contacts to avoid risk of recurrent exposure .Good Diet with full rest 8 hrs a day with Supplementary Vitamins / antibiotics after sending samples to Bacterial Culture.In this above discussed complex scenario- I would suggest LCr test with Semen from prostatic massage which could rule out Gonorrhea and STD from Chlamydia infections, which were incompletely treated in the past.Urine Culture / or prostatic massage semen fluid Culture would give definitive lead of proposed antibiotic usage, as your case seems to hibernating without much of clinical complaints from you.CT Prostate and Testes and cord with Cysto-urethrography would heilp in proper assessment of -Causative factors.This would help in targeting the treatment accordingly.USG Abdomen in initial stages and LCr test of Urine and semen and prostatic massage by this time would save dealy in initiating right antibiotic attack- either by oral / or by Intratprostatic Injections / with hyaluronic acid infusions to increase antibioitic reach in chronic fibrotic prostatic tissues, which is giving you tough times now.Addition of Tamsulosin/ Flomax with Pyridium would reduce your discomforts along with Chymoral forte /and Diclofencac Sodium for 2-6 weeks period, which would assure you better outcome.Hope the above discussion and points in it, would help you to plan further treatment from your Urologist at the expeditious speed Hope this would help you to treat your health issues in the best way possible. Welcome for any further query in this regard.Good Day!! Dr.Savaskar M.N.Senior Surgical SpecialistM.S.Genl-CVTS"
},
{
"id": 7869,
"tgt": "What are the causes of adult acne and how can acne scars be treated ?",
"src": "Patient: hello, what is the most common acne treatment. i have pimples that come and go, but i have some scars that are permenant. so what can i use, that has absolutley no side effects?? Doctor: ACNE OR PIMPLES ARISE IN YOUNG ADULT DUE TO INFLAMMATION OF OIL SECRETING GLANDS BY INFECTION OF AN-AEROBIC BACTERIA. TREATMENT AIMS AT: 1)GOOD SKIN HYGINE, MAINTAINENCE:LIKE FREQUENT WASHING. 2)USE OF ANTIBIOTIC LIKE OXY-TETRACYCLINE FOR FEW DAYS. 3)TOPICAL RETINOIDS TO THICKEN YOUR SKIN EPIDERMIS LIKE RETENOID -A 0.25%W/W 3)AVOIDING FAT LOADED FOOD ITEMS,AND GOOD AMOUNT OF WATER CONSUMPTION. \"HAVE A HAPPY ADOLESCENCE. \""
},
{
"id": 117674,
"tgt": "Can hematoma during pregnancy causes anemia?",
"src": "Patient: I had a hemotoma after or during the delivery of my child 28 yrs again.(while inthe labor room duaghter head came out sooner .than expected ..- no pushing lol)..everytime I take a blood test they say that I am anemic ...is this the reason for the abnormalicy or what else can be done..no other problems....no high blood pressure no other heath issues....how else do I explain this or is this common with a hemotoma Doctor: Hi,Thanks for asking.Based on your clinical history and query, my opinion is as follows:1. Hematoma, if large, can cause anemia at that time. Not any time after that.2. Females commonly have anemia, due to decreased nutrition and monthly periods loss.3. Improve your nutrition and take suppplements of Vit B12, folic acid and iron, your hemoglobin level will improve.Hope it helps.Any further queries, happy to help again."
},
{
"id": 121667,
"tgt": "What causes yellow bruise on the arm with pain?",
"src": "Patient: Hi, A Few days ago I found my upper left arm very painful, very randomly without injuring myself, or even banging my arm. The next day I discovered yellow bruising which has slowly turned into a rather large red/purple mark which looks like blood surfacing to the skin. This has now spread therefore is now a little bigger. This worries me as it has come out of nowhere Doctor: Hello, Your symptoms are suggestive of a hematoma, which could be related to a ruptured artery or local inflammation. These are the normal stages after the hematoma and I don't think that you should worry about it. If the problem repeats, I recommend consulting with your attending physician and performing coagulation tests. Hope I have answered your query. Let me know if I can assist you further. Take care Regards, Dr Ilir Sharka, Cardiologist"
},
{
"id": 54826,
"tgt": "How to cure 8.8x2.5x7.4cm hematoma in gallbladder?",
"src": "Patient: I'm a 25 year old female and just had my gallbladder removed 2 weeks ago, since the surgery I have been in constant pain and discomfort. 2 CAT scans have revealed a (roughly) 8.8x2.5x7.4 cm hematoma in the gallbladder fossa. How common is this and how does the body re absorb something like this? What if my body does not reabsorb it? I'm afraid that I will feel like this forever. I can't stand or walk and the surgeon doesn't want to do anything about it Doctor: Hi thanks for asking question.Noted you have removed your gall bladder.After surgery you have hematoma in gall bladder fossa from bleeding .Hematoma is collection of blood ...It is not tumour .so no need for extra worry.Hematoma gradually will be resolved...Close follow up needed.Avoid excess work load and straining.If it not resolved then drainage can done.But just now close follow up need.Take care."
},
{
"id": 226378,
"tgt": "Had unprotected intercourse, on contraceptives. Having cramps but not got periods. Worried ?",
"src": "Patient: Hi I had intercourse with my boyfriend on Saturday and he cum inside me, saturday is the same day i was meant to get my periods. I have been on the contraceptive pill for a hile now. I took the morning after pill on Monday and stopped taking the contraceptive pill the same day (Monday). I began to have cramps on Tuesday (thinking my period was coming) but than they stopped and it s Wednesday today and I still haven t gotten my periods ! Is it possible to fall pregnant when my periods haven t arrived after taking the morning after pill ? And stopped taking the contraceptive pill? (normally after 2-3 days of not talking the contraceptive pill I have a withdrawal period) I haven t even had that yet. I m starting to worry.. Doctor: Hello there.If you has taken your oral contraceptive pill regularly on fixed time then pregnancy is unlikely. Also you had intercourse in the safe period that is after finishing your pack.Slight delay in periods is common after emergency pills.But if you do not get periods after a week of stopping the pills then you need to take a pregnancy test.If negative, then it could be just aside effect of long term oral contraceptive pill. These cause thinning of the lining of uterus. It is this lining which is shed in menses. SO when thinned out it can cause light periods or no periods at all.Take care."
},
{
"id": 38407,
"tgt": "Suggest treatment for low grade fever",
"src": "Patient: I have low grade fever from last two weeks. My body temperature remain normal when I wake up after sleep but increase gradually up to 99.6 - 99.8 F. If I do work I get 100 F maximum. I am using ciprofloxacine from last week as I got widal test positive. but there is no improvement. Doctor: Hello, Thank you for your contact to health care magic. If I am your doctor I advice yw to do blood cultures. If you have widal positive salmonella will be isolated and once you have sensitivity pattern in hand you will take antibiotics accordingly. I also advice you ESR suspecting the tuberculosis. If you have to ask anything kindly contact me. Dr Arun Tank. Infectious disease specialist."
},
{
"id": 206370,
"tgt": "Suggest treatment for nervousness",
"src": "Patient: Hi, may I answer your health queries right now ? Please type your query here... I have the problem of nervousness when a little trouble occur in my body i will take as serious matter for that the problem is increase very much which affect my body and mind also. i cant ignore such situation. i want to escape this situation but i cant do so. Kindly give the suggestion for tackle the situation. Doctor: Hello, Welcome to Health Care MagicYou have complaints of nervousness even on day to day routine problems. You can't ignore the problems and this results in increased anxiety. Most probably these are symptoms of anxiety especially Generalised anxiety disorder. As it is causing significant impairment to your routine there is need of treatment. Meficines like SSRIs as fluoxetine, Sertraline, Escitalopram etc are approved for generalised anxiety. Apart from medicines try relaxation exercises, breathing exercises, yoga to keep yourself relaxed. You can consult a psychologist for Behavior therapy for anxiety.Thanks, Hope this helps you"
},
{
"id": 173920,
"tgt": "Why is my 20 month olds urine is attracted by ants?",
"src": "Patient: Himy son is 20months old and his urine is being attracted by ants.he is reluctant to accept any food.he dont drink water as well through out the day.his age infants were able to speak, but i dont see any progress on this.he is becoming lean also.what would be the problem ?Thanks,Rajesh Doctor: Hello and Welcome to \u2018Ask A Doctor\u2019 service.I have reviewed your query and here is my advice.If your baby is still breastfeeding or on milk feeds mostly, sometimes ants are attracted to the baby\u2019s urine because of the presence of lactose sugar in the urine.Some babies learn speech very late.If the baby is not speaking after two and a half years, then you may consult a speech therapist.Not accepting feeds is a common problem during the second year.It needs patience to feed the baby.Hope I have answered your query. Let me know if I can assist you further.Regards,Dr. Venkata Subrahmanaym Mukkavilli"
},
{
"id": 46522,
"tgt": "Which activities can be performed after perinephric haematoma and renal injury?",
"src": "Patient: im a 23 year old male, i have aleft perinephric haematoma, 1cm grade II renal cortical injury, undisplaced, Spinous process fracture L1 and L3, it was treated on the 8th of may. i was just wondering, what i can and can't do in terms of eating, drinking, and exercise. can i consume alcohol. can i ride a bike( not at the same time). as i was told to rest by the hospital, but 'rest' is a vague term i wondered if you could suggest some more clear guidelines. im normally a very active person so not knowing what i can do is driving me a little bit mad Doctor: Hello and welcome to HCM. As an Urologist,i can understand your anxiety.You've had a major injury to kidney, with fractures of the spine tip.In your case,rest would mean,avoiding lifting weights and straining.After 2-3 months of the injury,a CT or MRI scan,should've been repeated.This would show,if the healing is progressing normally and if rest is needed.You can take normal diet and certainly drink wine or social alcohol intake.If you've any other doubts,send all reports to me,as a direct question.Dr.Matthew J. Mangat."
},
{
"id": 8178,
"tgt": "Diagnosed with UTI and bacterial vaginosis. I have been experiencing itchy sensation and seems to be dry",
"src": "Patient: Hi I m 19h and been diagnosed wit a uti and bacterial vaginosis about two years ago now I Been experiencing a itchy sensation and my labia seems to be dry and inside are some tiny white dots that look like pimples that are under the skin of my labia it doesn t burn when I pee and there isn t any unusual discharge what can this be? Doctor: Hi Welcome to Healthcare Magic Forum Eruptions on the labia can be due to an infection (like herpes, lichen etc ) or an allergy , contact dermatitis , or a result of itching & scratching. You may get yourself examined by a dermatologist. Takecare."
},
{
"id": 201309,
"tgt": "What causes open sores under the head of the uncircumcised penis?",
"src": "Patient: Hi, recently I have formed three small open sores underneath the head of my uncircumcised penis. I have been applying Neosporin and they do not appear to be getting worse. I am sexually active and do use protection. I do rarely have unprotected penetration to tease before sex. What could this be? Doctor: Hi,It seems that there might behaving some soreness and some minor injury on frenulum due to sex giving this problem.Continue applying antibiotic cream.If require go for one antibiotic medicine course.Keep local hygiene proper.Ok and take care."
},
{
"id": 165198,
"tgt": "Suggest remedy for runny nose, dry cough and green stools in infant",
"src": "Patient: My five month old is congested with a runny nose and dry cough. She doesn t have a fever and output is normal but has had 2 green poops. Should I take her to a doctor? Also, she has a red birthmark at the base of her neck which is the same place mine is but it has dry skin that keeps peeling. I ve been putting olive oil on it but it seems to bother her, do you know what this could be? Doctor: Dear Parent,Cough and cold in an infant is always better evaluated by a pediatrician who can examine your baby to rule out lung infection and give medications for the relief of symptoms. Red birthmark at the base of neck can have various diagnoses. I would suggest that you see your pediatrician who can examine the birthmark and give you the diagnosis and more information regarding it."
},
{
"id": 153035,
"tgt": "How long do effects of chemotherapy last in body?",
"src": "Patient: I am 68 years young, 5 3 after Whipple surgery on Pancrease I lost 26 pounds (which I don t mind) After surgery one year ago in May I was 108 pounds....now after 6 months of chemo I m up to 112-114 which is fine. But I have IBS and continue to have sleep issues; and am having problems remember things. pancratic whipple surgery to remove cancer from opening of pancreas. Taking enzymes and photonics for the IBS. Having difficulty sleeping. Melitonan not working now. Sometimes Ambian takes quite a while before I get to sleep. How long will the chemo I was given for 6 months stay in my body? I m working with a trainer for Pilates but still feel tired and sleepless. Any recommendations? Doctor: Hello dear. I have gone through your case. The symptoms of IBS you are suffering from are due to pancreatic deficiency. You should take pancreatic enzymes replacement therapy after consulting your doctor. The effect of chemotherapy vanishes quickly after 3 weeks of last chemo, so no need to worry. For sleep problem, there are many better drugs and therapies available now days. You can meet a sleep physician who will help you in this matter. Hope i have made you understand. Thanksregards"
},
{
"id": 98514,
"tgt": "Are Loratadine and Phenylephrine the right medication for allergy?",
"src": "Patient: My allergies have become worse this year. I have had wheezing and productive cough with clear or light green mucous, and intermittent nasal congestion most at night and early morning for atleast the past 6 weeks. I believe I may have bronchitis. It is tolerable, however I m concerned because my symptoms have not gone away or decreased on there own. I take allergy medicine, Loratadine and nasal decongestant phenylephrine to help cope with the symptoms. Can bronchitis lead to infection? Do I need to take something else to get better? Doctor: Hi,Allergic bronchitis can sometimes be aggravated by a bacterial/viral infection.For the viral infection the treatment is just supportive care (good hydration and rest).If you think its bacterial infection (change in sputum color , clinically in bad shape) then I would advise you a course of antibiotics.As for the medicine for allergy, the ones you are taking are fine. Give it 2 to 3 weeks if it does not help then you can change it to other medications.Wishing you good health.Regards."
},
{
"id": 202872,
"tgt": "Bums on penis, scaring. Is it due to masterbaiting or an std",
"src": "Patient: Well I have notice 2lil bums on my penis behind the head and something I think is scaring on my penis. Idk if it came from sex masterbaiting or an std . Any help?? Can send pic if needed Doctor: hifrom your description of your symptoms you require to do a urine test to rule out an infection.you could be having a fungal infection it itchiness is present.i would suggest get reviewed by a doctor to examine you and get a urine test done as well.you also have to have a good personal hygiene to avoid reinfections.hope to have answered your querythanking you."
},
{
"id": 191565,
"tgt": "What causes dizziness in a diabetic?",
"src": "Patient: I am a 61 year old woman with type 2 diabetes. My bottom lip, the tip of my tongue and fingers on both hands are are numb. I am also very dizzy whenever I stand up or bend over. It started yesterday after working in the garden for several hours. I did drink at least 80 oz of water during the time. Doctor: Hi,Thank you for asking your question on HealthcareMagic.I have gone through your query. Your history suggests that you are a long-term diabetic. Diabetes over a period of time affects different body organs and may show symptoms due to same. The dizziness can be due to low blood pressure but needs to be evaluated to confirm.Are your sugar and HbA1C levels within normal limits? I would suggest you get the random blood sugar and HbA1C levels done and consult your doctor. Also, consult a cardiologist who would like to do an Echocardiography to assess your heart function. I hope I have answered your query. Let me know in case of any further questions.BestDr Vishesh"
},
{
"id": 185750,
"tgt": "Suggest remedy for intense tooth pain",
"src": "Patient: I had a tooth surgically removed 5 days ago, now the toot beside it hurts alot!! when i tap on it the pain is so intense i think i may pass out. I never had a problem with this tooth and had exrays recently and my dentist never mentioned anything about that tooth, is it just sensitive from having the one beside it cut out? Doctor: Hello:)Welcome to HCM.You mentioned you had a surgical extraction done.There are chances the next tooth can be moved in the process of removing the extracted tooth.This will settle down in a week. Take the antibiotics and analgesics prescribed by your dentist.Do not chew on this side for a few days.Gargle saline water twice daily.See your dentist to grind it out of occlusion.Consult your dentist if it persists more than a week.Also make sure you do not have any hidden caries in the tooth.Regards."
},
{
"id": 158920,
"tgt": "Diagnosed with stage 3 lymphoma. On chemotherapy. Twitching in the eye. Cause?",
"src": "Patient: Hi,i m a 45 year old female. i have alot of health issues that didn t seem to occur until after I d been diagnosed with stage 3 lymphoma & had to take 8 intense chemo treatments, that lasted for 6/7 hours each. Lately, i ve had this twicting around my entire right eye. it s not constant, but when it happens, i can actually feel it jumping & it s very scary. what could it be? Doctor: The twitching in your eye may or may not be related to the lymphoma. It is more likely that there is a disturbance in the nerve which supplies the eye. The reason for this needs to be investigated. This may be the cause you are having twiching in your eye. You should consult with your doctor who can examine you and treat accordingly."
},
{
"id": 103493,
"tgt": "Hit on butt. What to do about the sore?",
"src": "Patient: I fell during a strength training program - we were doing wall chairs and I didn t see it but there was water on the floor needless to say my feet went right out from under me, my butt is sore, but the the pain that has me scarey is that is right down the center of my chest. Can take deep breaths , can cough, and sneeze it just moving etc. Any suggestions. Doctor: Hello,Fall on the back can cause some real damage to the muscle, bone and nerve.The pain may be due to musculoskeletal cause.That is you it is aggravated during movement.Rest and analgesics may help."
},
{
"id": 75414,
"tgt": "What causes cough after chewing sugarless chewing gum?",
"src": "Patient: Age: 30 Sex: female Height: 5 1 Weight: 108 Question: When I chew gum, sugar-free OR bubblegum/non-sugar-free, I generally start coughing from the initial juicy sugar. After the intensity of the sweetness or the amount of saliva decreases, the coughing/choking stops. Additionally, just a few minutes ago, I had a much more severe coughing attack from chewing a cherry candy-cane, which has real sugar and corn syrup in it (artificial colors and flavors as well). This time, I actually was wheezing and struggling to breath. Not coughing. I believe it was an asthma attack. Asthma does run in my family. It just never occurred to me that I could have it; my potential symptoms never were as obvious as other family members. Could sugars and sweeteners be asthma triggers? Doctor: You seem to be having allergies. The allergen is more likely to be the artificial colours and flavours in the chewing gums and candies rather than the sugar itself though high concentration of sugar can also cause cough, wheezing and other asthma like symptoms. You need to consult your PCP who may test you for allergies and may start anti allergic treatment like montair lc and inhalers for wheezing."
},
{
"id": 32762,
"tgt": "Suggest treatment for infection that may have caused due to child abuse",
"src": "Patient: hi one of my friend was abused when she is 7 yrs old know she got married she is afarid to go to gynacalogist because if something they come to know about her then marriage may break out ..will she be having any cancer like colonal ..or will she having any kind of infections she wants to go for ultrasound wht can be found in ultrasound Doctor: Welcome to Healthcaremagic.com. I am sorry to hear about your friends issue. Please let her know that it is highly unlikely that she has an infection now when she is married due to abuse she had when she was 7 years old, however, if she contracted a STD during that time then it is possible that she has long term effects, what comes to mins is untreated chlamydial and gonorrheal infection. In any event, she needs to take the opportunity and get help because if she needs treatment then the infection will not go away on its own. Please let her know that she will also need to talk to a professional that she can confide in and help her to heal from the physical and emotional wounds that she has carried all this time, I truly sympathize and hope that she can get some resolution to this matter sooner than later. We are here to talk and chat with her anonymously if she needs that. I hope this helps, good luck and all the best. Yours Doctors at Healthcaremagic.com."
},
{
"id": 71863,
"tgt": "What causes chest pain having mitral and pulmonary regurgitation?",
"src": "Patient: Hi, My chest seems to hurt a little in the top centre. I have lupus, and I did an ultra sound on my heart, and the results said.: Grade 1 systolic function, physiological mitral and pulmonary regurgitation, enlarged eustachian valve in right atrium. I am on plaquenil. What might be causing my chest pain? Doctor: HelloRegarding probably the fluids in your lungs due to heart problems might be the causeRegardsDr.Jolanda"
},
{
"id": 79065,
"tgt": "Suggest treatment for pain and tightness in the chest",
"src": "Patient: Hi Ive had problems with chest for 2 years, feeling of something stuck in my throat, coughing phlem. Treated for reflux but nothing. Then develop chest infections and cannot breathe, require steroids, antibiotics, inhalers which dont seem to work. Then improve but still get tightness feeling in chest, voice comes and goes, pains in front and rear of back near lungs Doctor: Thanks for your question on Health Care Magic. I can understand your concern. By your history and description, possibility of post infectious bronchitis is more in your case. You were having chest infections. Some infection may cause prolonged inflammatory reaction. So bronchitis is common after lung infection. It can cause chest pain, tightness, heaviness. So better to consult pulmonologist and get done clinical examination of respiratory system and PFT (Pulmonary Function Test). PFT is needed for the diagnosis of bronchitis. It will also tell you about severity of the disease and treatment of bronchitis is based on severity only. You may need inhaled bronchodilators and inhaled corticosteroid (ICS). Hope I have solved your query. I will be happy to help you further. Wish you good health. Thanks."
},
{
"id": 11686,
"tgt": "Dark red patches on hands, under armpits, thighs. Due to less water consumption?",
"src": "Patient: hi my skin have developed dark black-red patches only on my hands. mainly underarms and very little on theighs. its since almost 3 years. but now its visibility is increased. when i visited doc he said, : my consumption of water is very less so its happened and suggested me to apply moment oinment. i hardly go in sun can you suggest me about this ? Doctor: Hi,Thanks for writing to us.I can think of two probabilities in your caseAcanthosis nigricans where skin becomes thick velvety and pigmented.Erythrasma an infection where the area becomes red or brown in color.I would advise that you meet a qualified dermatologist so that proper diagnosis based treatment can be offered. A steroid cream like momate would not solve your problem.Take care"
},
{
"id": 187291,
"tgt": "Is an exposed bone caused by a mouth ulcer,a matter of concern?",
"src": "Patient: I had a mouth ulcer for a couple of weeks and after healing I have bone exposed now where the ulcer once was. It is located in my lower jaw on the inside of my mouth. The pain is just a dull ache. I expect it will heal. Should I be worried or go see a dentist? Doctor: Hello, Welcome Thanks for consulting HCM, I have gone through your query, as you have ulcer on lower jaw dont worry , you do is avoid spicy food first. Take ointment Mucopain or Pansoral apply on ulcer three times a day. You can take Capsule Zevit one capsule twice daily . Do warm saline gargle two - three times a day Consult dentist and go for oral prophylaxis . Hope this will help you. Wishing you good health."
},
{
"id": 163440,
"tgt": "What causes drop in sugar level, nose bleed and distended abdomen?",
"src": "Patient: my daughter is 4.10 years old.looks like 3.5 and weight is 15kh. she has distended abdomen.doctors believe she has metabolic lever disorder.sugar level drops if she does not eat any thing for 7-8 hrs. She had nose bleed. but now her blood does not stop so easily. platelets count ok.hb=10 sgpt=54 Doctor: Hello,Yes, it could be metabolic liver disease. A distended abdomen may be fluid in abdomen, seen in liver disease.Bleeding may occur due to deranged PT/INR, a test and function of liver, which may lead to bleeding.I suggest you a battery of test as ultrasound abdomen, complete Liver function test, alpha feto protein, and repeated blood sugar. Further tests are based on reports of these tests.Hope I have answered your query. Let me know if I can assist you further.Regards,Dr. Sachin Kumar Agarwal"
},
{
"id": 175689,
"tgt": "How long it takes for the bilirubin values to come normal?",
"src": "Patient: sir. 5 days back my wife gave birth to male baby, at that time his wieght is 2.8 kg. after two days he got symptoms of hypoglymcemia as mother didnt get enough milk to feed him. very next minut he got admitted to NICU. when doctor checked him and did bilirubin check , jaundice is confirmed as his bilirubin lele is 17.6. since two days he has kept for light therapy. hpw long he should be in NICU and when his bilirubin ccomes down to normal level? plz help me sir Doctor: good question,normally because of immaturity of liver enzymes,hyperbilirubinemia is commonly by 3-4th day usually subsides on its own by 7-8 day of birth.2.8kg baby having bilirubin 17.6mg is almost normal ,only it is dangerous if it is increasing more thna 5mgs /day,then only phototherapy is advised,otherwise now a days we pediatrician have habit and also parents feel safe to baby under that blue light.please donot give phototherapy unneccesary"
},
{
"id": 165588,
"tgt": "Which baby food is advisable to 1 year 2 months old baby?",
"src": "Patient: My 1 year, 2 months baby is on Isomil 3 and the doctor has advised that I need to change her to Pelagon ? She had reflux (she has since bben operated and seems fine although she doesn t eat as normal kids do) . I ve started her on parmalat cow s milk for infants (1-3 yrs), however I only give this to in the morning as well as during bedtime. Whicj is the best formulae as I ve never tried Lactogen ? I ve tried Pelagon and Nan and she refuses to drink it Doctor: Dear parent,Your baby is now one year and 2 months old. you can give her normal boiled milk(packet or local).There is no need for formula milk. If you are not convenient with local milk ,formulae i would recommend is lactogen stage 3 /similar stage 3/enfamil stage 3. Give her milk three times a day around 500 to 650ml per day. Other 3 feeds should be solids like rice,dal,vegetables,fruits.As u told ur baby had reflux problem,reflux would decrease spontaneously in infants after starting solids.For proper growth of the baby ,she should be provided with balanced diet not only milk."
},
{
"id": 93066,
"tgt": "History of small bowel obstruction. Have CHF. Caused by soluble or insoluble fiber?",
"src": "Patient: I have had numerous episodes of small bowel obstruction which the doctors say is probably caused by ruptured appendix surgery when I was 3. I am now in a 70's with CHF and would like to know what type of food I should be eating. I have been told by every physician I have consulted that it does not matter what I eat but I am sure it is related to the amount of fiber I consume. Is soluble or insoluble fiber causing this ? It seems to be related to fiber and I have tried to reduce my intake but it is difficult since I am then prone to constipation and hemorrhoids. Any advice would be appreciated. Thanks. Doctor: Hello,If it is a post surgical obstruction then it may due to adhesions in the lumen of the intestines.Fiber diet is very important in your age because it prevents constipation, hemorrhoids, hyperlipidemias, heart blocks and even colonic cancer.The only thing is that the diet should be easily digestible.So cook the fiber well(Vegetables, greens).Preferable you may take soluble fibers."
},
{
"id": 168449,
"tgt": "Suggest remedy for pain in groin area",
"src": "Patient: my 11 year old daughter has pain in her groin area on one side. She is very athletic. She says that it feels as if it needs to pop. IShe says it has done this before, but when she pops it, it feels better. t bothers her to walk. She says it s like a bone is in the way... she says it won t pop. Doctor: Hello. It sounds to be like your daughter's groin pain is from being very active with athletics and the fact that her body is growing and changing quickly. I would make sure there are no other symptoms like really bad hip pain, fever, abdominal pain or anything else unusual. If it's just the groin then I would make sure she is warming up with some walking or light jogging before she does sports and she stretches afterwards. If the problem continues or gets worse I would consider having her see her doctor for an examination and maybe some x-rays. Hope she feels better!"
},
{
"id": 217408,
"tgt": "What causes tendon pain in my left arm and shoulder?",
"src": "Patient: I had a ganglian cyst removal some years ago just above the left wrist on the back of my hand.I am having problems with tendons etc and have muscle/tendon pain in my left arm and shoulder.Could this be due to nerve/damage from the surgery? I understand this can sometimes be an unfortunate side effect? Doctor: No, the surgery on the wrist will not give rise to pain up in the arm or shoulder. You might have injured a muscle during excercise or sport. Yoy have not mentioned your age. If you are above 40 years, then you should also evaluate yourself from your cardiac point of view. Show yourself to an orthopaedic surgeon and also a physician if muscle trauma is ruled out."
},
{
"id": 66757,
"tgt": "What causes lump/bump on the upper arm and on top of shoulder?",
"src": "Patient: 28-year old man: a lump/bump on the upper arm and on top of the shoulder appeared suddenly one after another within 2 days. One of them (on the sholder) is almost round and the other one (on the upper arm) is ellipsoid-like and is about 1.5 long and about 2/3 wide. What could that be? Doctor: Hi, dearI have gone through your question. I can understand your concern. Multiple lump can be lipoma or neurofibroma. You should go for biopsy of one lump and ho for histopathological examination. Excision is rhe treatment of choice for both. Consult your doctor and take treatment accordingly. Hope I have answered your question, if you have doubt then I will be happy to answer. Thanks for using health care magic. Wish you a very good health."
},
{
"id": 86151,
"tgt": "Suggest cause for lower abdomen and back pain with loose stools",
"src": "Patient: For the past few days I have had sharp pains in my lower absomen and lower back pain. Today I had loose bowel movements. I also have had gas on and off when the pain starts. I don't know if it was something I ate. I exercise daily and am in good health with no medications. Doctor: Respected user , HiWelcome to Healthcaremagic.comI have evaluated your query thoroughly .* This seems in relation with intestine infection via some sort of bacteria or virus which may be consumed from some sort of food or liquids .Hope this clears your query .Welcome for any further doubts .Regard dear take care ."
},
{
"id": 198903,
"tgt": "Suggest treatment for small painful bumps on the penis",
"src": "Patient: Sir Good Morning, I am 42 year old man last 15 days i fill aching inside my panis after 3 or 4 days there was boil there. after that there is some other boil and leakage like water. I apply some talk on the spot its ok. after one weak i have sex with my wife we fill aching there and same repeat above mentioned. please do advice for the same. a.k.verma Doctor: Hello Mr AK VermaI had gone through your question very diligently and I can understand your concern Bump followed by aching inside the penis might be related to Inflammation inside the penis.Here I would like further information for better assessment of the conditionSince how long you have this problemany history of unprotected intercourse? If I were your treating doctor I would have advised you for battery of test like blood for hemoglobin, CBC, Liver and kidney function test, blood sugar, urine testing for microscopy as well as culture.Meanwhile you should take plenty of fluids 3 to 4 liters a day.You can take Ibuprofen for pain inside the penis.Avoid sex or use condoms until you recover fullyBest wishes"
},
{
"id": 209844,
"tgt": "Why the behaviour of my son is unusual?",
"src": "Patient: sir my son is 18yrs old nd he talks vch has no sense his eys r gone blk.he has started hating me ,his frnds,his teachers nd few others.his behaviour is also unusual.he is lacking in his studies.vat evr v say he does d opposite.he keep distance frm others.vat should v do now Doctor: Welcome to healthcare magic,It is quite usual at this age that children behave like this. He has partially entered adulthood. We know this, but not your son. Blame it on the hormones. He is confused. All he need is parental support and love. Your must not force your love to him. To love and support means to be there for him when he wants. To give what he wants. Nothing more nothing less. Even though sometimes it might seem that by doing so you are going away from your basic principles of life, still then its fine if it gives him happiness. Make sure that you are providing him the right family atmosphere at home. Try to be friendly by just listening to him. Never advice if he seems irritated. Children hate advises. Watch him and join him if he is fine with it. There will be a reason for his behavior or simply it could be hormonal which needs no treatment but love and support. If you still find no clue, better get him a friendly consultation from an expert.Thankyou"
},
{
"id": 30290,
"tgt": "How to treat chronic infection on nose that peels and dry skin?",
"src": "Patient: My daughter has a rare metabolic syndrome in which she can not absorb animal proteins, and fats. She is also very sensitive to phyto estrogens. She takes Creon 5 (pancreatic enzyme). Her skin is dry and has tiny bumps. She also has a chronic infection type rash on her nose that constantly peels, heals and reacts. She is afraid of using cortizones, and your average otc creams etc are not sufficient; any suggestions? We are from northern NJ NY metro area. Doctor: The condition sounds like seborrheic dermatitis, which is treated by proper skin hygiene (washing face twice daily) and with shampoo/soap containing one or more of these ingredients: Salicylic acid, Selenium sulfide, Coal tar, Ketoconazole, and Zinc pyrithione. Other treatments include: Antifungal products, Corticosteroid lotions, Prescription-strength medicated shampoos, and Sulfur products. Make sure your daughter is not allergic to any of these ingredients by testing a small patch of skin on her arms (not face). She should be evaluated by a dermatologist, who can prescribe a stronger cream with an antibiotic to treat in case of infection and for proper diagnosis. In the meantime, there are also natural essential oils and emollients you can use like Neem oil, which is naturally anti-microbial and soothing."
},
{
"id": 74776,
"tgt": "What causes sudden tightening of the chest?",
"src": "Patient: Recently while driving through the mountains in Pennsylvania I felt a strong tightness in my chest. I was on HRT in the fall, but was taken off because of loss of eye sight in one eye...also had this tightening in my chest. Since then I have been fine...up until now. What do you think? Doctor: Respected user, hi I evaluated your query thoroughly.* Relates to decreased lung oxygenation at heights in mountains causing sudden tightening of the chest , a normal physiological phenomenon observed in majority of people without any underlying pathology.Hope this helps you.Welcome for any further assistance.Thanks for using Healthcaremagic.com & giving me an opportunity to assist you.Wishing you a very happy healthy life ahead.Regards dear take care."
},
{
"id": 208467,
"tgt": "Suggest remedy for mental health problem",
"src": "Patient: hi,dont even know if it is a problem.i just cant get over being sad.i get these bouts of crying and dont know what to do??i have ateenage daughter n a 8 yr old son.i have been having regular disagreements with my husband.he seems to think i dont care about him at all.and to think i didnt make my career and have been looking fter them for 15 years....!!!! Doctor: Hi,I can understand what you must be going through. Having trouble in relationship with the person you care about a lot really makes us feel bad. Your symptoms of being sad and having crying episodes suggest that you are developing depression as a result of your marital conflicts.I would suggest that you consult a psychiatrist. Ant-depressants will help in overcoming your sadness and other symptoms. In addition, you and your husband must consider marital counseling as an option. It will help out ease the tension between you two and help you both feel better.I do hope that I was able to answer your query. Best wishes."
},
{
"id": 136241,
"tgt": "What is the swelling and numbness at the site of injury?",
"src": "Patient: I have a bruise on my shin from a collision in a soccer match. It is just below my knee, but not on it. It is VERY swollen and an area about 2cm by 4cm feels similar to when your foot has gone numb. I don t feel pain unless I exercise or apply a lot of pressure. When I apply pressure I don t feel my fingers on the surface, but I feel the pressure deeper down. Should I be concerned? And is there anything I can do to make the nasty contusion go away? Doctor: Hi Thanks for your query and welcome to healthcaremagic. I have gone through your query and understand your concern. I think you are having contusion. Treatment of this is icing, elevation and analgesic such as ibuprofen for pain relief. There is not much to worry. It will take its own course to resolve. You can discuss with your doctor about it. Hope your query get answered. If you have any other don't hesitate to write to us. Wishing you a good health.Take care."
},
{
"id": 192283,
"tgt": "Suggest treatment for pain and burning sensation in penis",
"src": "Patient: hi...........i have been enjoying my sexual life since 4 years. recently 3 months ago some small pocks like pimples with unbarrable burrning were arised. i went for a doctor and had taken treatment. now it's fine .but pennis became too sensitive. internally some pain and burning is there.could you suggest me the solution? Doctor: Hi, From your complaints it seems to be a case of UTI. Any h/o diabetes mellitus ? Any difficulty in passing urine apart from burning ? Any perineal heaviness or pain ? Go for Urine R/E & C/Sreview with investigation. Hope I have answered your query. Let me know if I can assist you further. Take care Regards, Dr. Ashish Kumar Khandelwal"
},
{
"id": 170897,
"tgt": "What causes chronic diarrhea and intestinal cramps in a 9 month old baby?",
"src": "Patient: This is a senario; a 9 month old girl was admitted to hospital with chronic diarrhoea, intestinal cramps and flatulence. her weight gain had been slightly below normal for the first 6 months of life, but had become considerably below normal in the past 3 months. diarrhoea first became obvious at about 6 months. microscopic analysis of the faeces did not reveal any abnormalities. a sample of urine is available for analysis. What could her possible diagnosis be and why did the diarrhoea start at 6 months ? ( I assume it is because she was weaned off breast milk and it has something to do with amylase enzyme) Doctor: Hi...Thank you for consulting in Health Care magic. I understand your concern. I have a few questions and will suggest few possibilities.Questions:1. What was the birth weight and what is the current weight?2. How is the attaining of developmental milestones?3. Is the stool oily and floats in the pan?4. Is there any colour change in the hair or skin?5. Is there any history of recurrent serious infections?Possibilities:1. Chronic diarrhoea - ? Fat malabsorption2. Micro Nutrient deficiency3. Immunodeficiency either primary or acquired.Hope my answer was helpful for you. I am happy to help any time. Further clarifications and consultations on Health care magic are welcome. If you do not have any clarifications, you can close the discussion and rate the answer. Wish your kid good health.Dr. Sumanth MBBS., DCH., DNB (Paed).,"
},
{
"id": 196687,
"tgt": "Can excess masturbating cause phlegm in throat and depression?",
"src": "Patient: sex maleAge 21ht 5'11''wt 55kgi do excess of masturbate.twice daily.my throat is very sensitive.it gets tounciled very frequently.for the last 2 years phlegm is forming which i used to expell from mouth as it doesnt comes fron nose itself.6 months earlier i suffered from typhoid without fever.and even now i get jerks in stomach and stomach pain sometimes.i am also suffering from depression for 8 years.is there any link of excess masturbation and any of my symptoms that i told?is there link between any of above mentioned problems?do advice me Doctor: Hi,I can understand your concern. Masturbation does not cause excessive mucous discharge (phlegm), tonsil infection or depression. Masturbation is a normal activity for sexual satisfaction. It does not harm you. Kindly consult an ENT specialist for recurrent discharge and phlegm.Hope I have answered your query. Let me know if I can assist you further.Regards, Dr. Chintan Raval"
},
{
"id": 220450,
"tgt": "What are the chances of me getting pregnant?",
"src": "Patient: My husband suffers suffers from asthenospermia.His sperm motility was just 10% but count is okay.since last 20 days he is taking medicine called oligocare.Just want to know that how fair is the chance of conceiving and what all measures should we take.Does this treatment requires longer time? Doctor: HiDr. Purushottam welcomes you to HCM virtual clinic!Thanks for consulting at my virtual clinic. I have carefully gone through your case, and I think I have understood your concern. I will try to address your medical concerns and would suggest you the best of the available treatment options.Male subfertility treatment takes time. With treatment, once the count of motile sperms increases, you can opt for IUI- intrauterine insemination. Or even IVF ICSI- that is one type of test tube baby technique wherein the sperm is directly injected in the egg.Please do not worry. With the help of expert infertility doctor, you have fair chances of getting pregnancy.WISH you be blessed with a bundle of Joy.I hope my answer helps you.Thanks.Wish you great health.Dr Purushottam"
},
{
"id": 53431,
"tgt": "Can a low functioning gallbladder cause shortness of breath?",
"src": "Patient: I was recently diagnosed with a very low functioning gallbladder. I have not had any recent attacks, therefore I haven't taken any steps toward surgery. I have recently started having some shortness of breath upon exertion. (do not smoke and blood pressure is good, overall good shape) I was wondering if there could be any connection with the shortness of breath and my low functioning gallbladder? Doctor: Hi,To think that such breathing difficulties can be caused by gallbladder so it is more likely there is some lung or heart diseases and so this should be ruled out first. Gallbladder stones will cause more likely sharp pain right upper part of abdominal cavity. Wish you good health.Hope I have answered your query. Let me know if I can assist you further.Regards, Ivan R. Rommstein"
},
{
"id": 162480,
"tgt": "How can cough, fever and chills in a child be treated?",
"src": "Patient: A 10 year old boy is send home with a diagnosis of viral bronchitis. Symptomatic treatment is prescribed. The mother calls and states that her son is now 48 hours late having purulent productive cough, temp of 103 and chill and not eating. What plan is most appropriate? Doctor: Hi, A respiratory infection such as viral bronchitis can develop into pneumonia. The symptoms that you describe are compatible with pneumonia. I believe this child needs to be taken to either his primary care physician or the nearest emergency room as soon as possible. Hope I have answered your query. Let me know if I can assist you further."
},
{
"id": 81755,
"tgt": "What causes chest pain while breathing and lying down?",
"src": "Patient: I ve been suffering from chest pain for nearly six month now it get worth when I breath , lying down or sneeze I know I have 3mm granulomas within the left lower zone The pulmory vasculature is not congested Pain is getting worth I m not doing anything to activated even I stop going to the gym I don t know but I think I have Costochondritis Doctor: You need to get a CT chest done so that we can know exactly what is the problem. A 3 mm granuloma will never cause such pain as you describe and a costochondritis is a clinical diagnosis. See a good pulmonologist and he should be able to answer your diubts"
},
{
"id": 66956,
"tgt": "Suggest treatment for small lump on lower back",
"src": "Patient: im 37 yr 165 lbs male i have a small lump on my lower back on the left hip side. It is not paining much, but once in a while I feel small pinch. This small lump was there for almost 3 years. I remember I was playing Tennis in hot sun in Arizona one day and was dehydrated and got this lump the next day and it stayed there. Was this lump due to the muscle spasm or anything else is I am worried of. Please let me know. Thank you very much. Doctor: Hello and welcome to HCM,A lump on lower back on left hip region is most probably due to soft tissue swelling.Muscle spasm is unlikely to cause a lump or swelling.Imaging of the lower back will help to determine the relationship of the swelling with the bony structure.Imaging can be followed by aspiration cytology of the swelling.In aspiration cells are aspirated, spread on a slide, stained with appropriate stains and studied microscopically.This investigation will determine the origin and nature of the swelling.Management of the lesion can be done after this investigation.Consult your primary healthcare provider for clinical assessment, investigations and thus management.Thanks and take careDr Shailja Puri"
},
{
"id": 163396,
"tgt": "Does Neopeptine syrup help treat constipation in children?",
"src": "Patient: Hi, my daughter is exactly twenty six month old. Fur the past seven months she is having difficulty in passing stool. She gets highly constipated. She s not a great eater! My question is will neopeptine syrup help passing stool? Will it solve the constipation issue? If yes, what ll be the dose? Please suggest. Doctor: Hello,At such a young age, anything to help pass motion is not advisable. No, Neopeptine doesn't help passing motion. If she is a poor eater, then you have to figure out ways to create her interest and see to it that she gets enough fiber and fluid. Give her a glass of warm water in the morning and make her run around so as to get the defecation reflex. Take advice or short-term medicine if she has hard stools and try to regularize her habits. It will be helpful in the long term. Hope I have answered your query. Let me know if I can assist you further.Regards, Dr. Ratna Mulay"
},
{
"id": 47745,
"tgt": "Could pain in back below shoulder blade while turning, coughing and burping be kidney related?",
"src": "Patient: I have right back pain below my shoulder blade when I turn, cough burp any movement really. I didn t think my kidneys were that high and I have not done anything to strain the area. I have no signs of bladder infection meaning burning when urination. could it be kidney or liver related Doctor: when you turn, cough and burp if you have pain then it could be due to a muscle involvement or a pleural problem initially. Hence check in those lines. if nothing abnormal on that side then proceed with the evaluation for other associated causes."
},
{
"id": 23618,
"tgt": "Suggest treatment for palpitations, chest tightness and heart pounding in chest/throat",
"src": "Patient: i am 36 yrs old and have been dealing with heart palpations, my chest is tight, get dizzy often, and now when i lay on my side (either) i can feel my heart pounding in my chest and throat which makes me feel as if i have something stuck in my throat. I am at my wits end with this and dr dont seem to know just yet Doctor: chest heaviness and choking sensation can be a feature of angina ...though 37 year is early age for angina but it can present at any ageplease mention your smoking habits and sexan ECG should be done for palpitations to rule out supraventricular (sinus tachycardia) or any other arrythmiai would suggest you to go for a test called holter which records every beat for 24hoursa tread mill test and 2d echo cardiography is to be done to find any ischemia in heart walls"
},
{
"id": 50589,
"tgt": "Had severe abdominal pain and constipation. Found left kidney is hydronephrotic. Can see blood spots in stools. What does the finding mean?",
"src": "Patient: Dear Doctor , Last May 2012 end I had severe abdominal pain starting from middle of stomach to till lower part on left side.I couldn't able to sit or stand at that time. Then I went to Gp's clinic near by.He send me to Ultrasound scanning.The radiologist found my left kidney is hydronephrotic (due to stone in my uretor).After that doctor gave me Tablets , and late after 2 weeks again I undergone another scan.This time hydronephrotis gone.They discovered 10mm renal calculiin left kidney.From that time to till3 months before I am continuing the medication.Cyctone is the tablet given for kidney stone.While taking it ,I have contant constipation...some times laxatives are needed.I stopped the medicine and again went to GP's ,this time he change medicine,that also causing constipation.After that last week i Have severe pain in my stomach lower abdomen part and lower joint.Stomach bloated and inside some itching feeling .constipated for almost 2-3 days .then doctor gave medicine.Even though constipation is ok I have severe discomfort in stomach and sometimes stool is so hard to pass.some times ok.When it is hard some times I can see blood in stools.I did Ultrasound scan of Abdomen and pelvis.After that I went to Urologist .He send me to CT KUB .Kidney stone size reduced to 3mm .My urologist is telling Stomach pain is not due to Kidney stone have to do ectoscopy. .Gp is telling it is due to kidney stone.Last october I did Ultrasound scan for utreus also.Now I am so stressed.Once I know ,nothing wrong in my stomach only I feel ok.I have a small kid .I am praying to God nothing bad to hear.Symptomsin and off stomach inflammation after passing stool and urinenow after taking medicine constipation ok,now it is normalbut I can see blood spots in stools.almost 2-3 months I have in and off constipation,last week only it got worsed.Have kidney stone in left kidney with 3mmplease suggest what I have to do, colonoscopy,ectoscopy or CT abdomen Doctor: take plenty of fluids,get checked with surgeon,to rule out piles ,fistula.get protoscopy first.get blood test -Hb%.take high fiber content food,laxative if very hard stools,"
},
{
"id": 77025,
"tgt": "Does durasalyn liquid help with the cough?",
"src": "Patient: My kid who is 3 yrs old has been suffering from severe cough for the last 15 days. Doctors have prescribed many drugs but nothing worked fine and finally he is on medication with 'Durasalyn liquid' syrup. Could you please guide us if this syrup is effective for cough. Doctor: Hi thanks for contacting HCM...Noted your child having cough since 15 days.Here you have prescribed theophylin syrup by your doctor...If bronchoconstriction is present like that in asthma or bronchitis then it is useful..Your doc might have examined him so giving this syrup...If cough is dry then dextromethorphan like cough suppressant syrup also useful.Your child must be physically examined with auscultation for searching cause.Respiratory rate also counted....You can discuss this with your doctor and syrup can be taken...Take care.Dr.parth"
},
{
"id": 157719,
"tgt": "Cough, taking Sutent for kidney cancer, kidney removed, spread to both lungs. Cause of this cancer?",
"src": "Patient: I am taking Sutent for kidney cancer I will have a CT scan soon to see if it has been effective. I also had a kidney removed and know that it ha spread to both lungs. I can't seem to ask questions of my doctor in person, just to real, and reading is eaiser. How did I get this cancer, Inever had any sympotms sxcept for a cough which lead to this discovery. Doctor: Hello!Thank you for the query.It is hard to tell where does the cancer come from. Its a good question. Some kidney cancer are genetic (in few percent). Some are related with smoking, obesity, estrogens and some chemical agents. But so many people smoke and do not have kidney cancer.Generally cancer is caused by single cell mutation which is missed by immune system. Such mutated cell can not be controlled anymore and its only aim is to grow and spread. This process can not be stopped by human body and thats the main problem with cancer.The other thing is that many cancers does not give any symptoms at all.So no one really knows where does the cancer comes from.Hope this will help.Regards."
},
{
"id": 161830,
"tgt": "What causes swelling in throat?",
"src": "Patient: my daughter is 11 years old she has a small swelling in throat we met doctors and take all test for tyroid the result shows nothing . but now she become little fatgue, less nail ,sometimes joint pain we feel fear about tyroid Doctor: Hi, By what you say I feel that she might be having an autoimmune disease - keeping the description/age and gender in mind.I suggest you discuss this with your pediatrician and get her appropriately evaluated and managed. Hope I have answered your query. Let me know if I can assist you further. Regards, Dr. Sumanth Amperayani, Pediatrician, Pulmonology"
},
{
"id": 153524,
"tgt": "What causes leg cramps after treated cancer?",
"src": "Patient: Hi, may I answer your health queries right now ? Please type your query here... my husband was diagnosed with non hodgen lymp and under went chemo n radiation treatments. He is cancer free, however, he suffers from servere leg cramps. What can he do? Doctor: Hi, dearI have gone through your question. I can understand your concern. His leg cramps may be due to low calcium, dehydration or electrolytes imbalances. He should investigate for that and take treatment accordingly. Hope I have answered your question, if you have doubt then I will be happy to answer. Thanks for using health care magic. Wish you a very good health."
},
{
"id": 42950,
"tgt": "What should I do to increase my chances of conceiving?",
"src": "Patient: Hi Doctor, I would like to know my options towards conceiving. I do not have regular mensies and my last pap smear which was about 2 years ago came out ok. I had a miscarriage about 3 years ago. What should I do to increase my chances if conceiving and if any, what lifestyle changes do need to put in effect? Doctor: Hi,I read your query and I understand your concerns.Following is my reply:1) You have to loose weight and attain healthy weight.2) Have balanced diet and exercise regularlyLet me know if you have anymore questions.Regards,Dr. Mahesh Koregol"
},
{
"id": 44062,
"tgt": "Trying for a baby. Normal reports. What is unexpected fertility?",
"src": "Patient: hi,i am 33 yrs old ,i am trying for a baby last 9 month,married for 7 yrs.all of my reports is normal said my gynecologist .now she give me letrol-2.5,2 tabs for 5 days,i also in take metformin .folison.letrol taken for 2 month,today is the 4th day in first cycle as she prescribed.what is the meaning of unexpected infertility ? Doctor: Unexpected infertility means all required reports of male & female are normal & after trying medicines not be pregnant.Can try Ayurvedic medicines for that.Wish you all the best."
},
{
"id": 186495,
"tgt": "Suggest medications for blisters in gum",
"src": "Patient: I had lower back tooth pulled today 2nd fromBack. Had two temporary crowns put in front and behind extraction site along with stitches and foam gel inserted in socket. Then temporary bridge was attached over site. I have what looks like a huge blood blister on side of my gum I guess due to trauma. It was a hard tooth to get out. I am feeling weak dizzy and nauseated.. Is this normal? I have ate some soup and taking ibuprofen. Still bleeding after 12 hours Doctor: hello thanks for consulting at hcm..it could be due to immediate placement and the socket has to heal,,u r feeling dizzy becz of blood loss, hydrate urself well, kindly get back for a check up for continued bleeding in the extraction site,,so consult with ur oral surgeon/dentist.for pain relief and stop the bleeding.hope it helps,,tc"
},
{
"id": 97635,
"tgt": "Suggest treatment for Hydrocephalus in ayurveda or homeopathy",
"src": "Patient: Is there any kind of Treatment (in Holopathy, Ayurveda, Homeopathy) for Hydrocephalus (a baby born with almost 80% of her brain filled with water) & also suggest whether there is any kind of assurance (that baby is completely healthy and completely out of this Hydrocephalus) that I get about baby health after the treatment, in the long run? If not tell us what kind of problems that may arise in the baby after the treatment in the long run? Doctor: the child needs a combination of homeopathy and physical therapy for a long term solution.there will be improvement in vocabulary with speech therapy,also the physical therapy or rehabilitation therapy will make your child sit ,walk and help himself as much as possible,and definitely much much better then if not treated.reduce the heat of the body is one of the alternative therapy but that is only at birth.since the child is older he can take one spoon of cumin ,black pepper,sonf and amla ,Chrystal sugar,ginger . Mix and take one tea spoon in the morning with a glass of water.or honey with ginger and place a drop of this in his belly button over night."
},
{
"id": 27976,
"tgt": "What causes faster heart beat?",
"src": "Patient: Sometimes I can feel my heart beat really hard once or twice. It doesn t hurt or give me any side effects after that. It just beats hard sometimes. And it used to happen only once or twice a month, or even less, but lately its been everyday two or three times a day. What s going on? Doctor: Hi, i did review your concern.Palpitation as experienced by you can be due to multiple reasons. most common are coffee intake increase, thyroid illness, anxiety or panic attacks and hypoglycemia.rrarely it could herald some underlying heart condition for which we need to do investigations and rule it out as early as possible. I would advice you to consult a doctor and get yourself evaluated.basic test would include thyroid function test, ECG for heart and a blood report.I hope this helpsWish you a healthy life ahead."
},
{
"id": 125735,
"tgt": "What causes sudden numbness in the upper body and tingling sensation in the arms?",
"src": "Patient: this morning, while sitting on the toilet, my upper body went numb and I had a sensation of burning and tingling in both my arms and abdomen. the sensation was extremely scary and I felt like I was going to pass out. after about 5 minutes of sitting there I was able to get up. Doctor: Hello, The symptoms are consistent with neuropathic causes like nerve impingement. Consult a neurologist and get evaluated. A detailed evaluation including MRI and nerve conduction study is required to make a diagnosis. Hope I have answered your query. Let me know if I can assist you further. Regards, Dr. Shinas Hussain, General & Family Physician"
},
{
"id": 127638,
"tgt": "What to do if having pain in wrist, arm, shoulder and legs after being diagnosed with Scheuermann's disease?",
"src": "Patient: A doctor said I had scheuermann's diesase because I did an X-ray because I was having really bad back pain well I tried physical threapy and lost some weight and tons of pain pills...nothing helped. Now I'm getting wrist,arm,shoulder,leg,knee,ankle pains plus swollen ankles...what should I do Doctor: Hello and Welcome to \u2018Ask A Doctor\u2019 service. I have reviewed your query and here is my advice. Scheuermann's disease is mainly seen in young adults. I understand that you have a progressive back deformity. You should take pain killers and some of which lead to fatigue. However, I also consider to rule out spondylitis which also leads to kyphosis of thoracic spine along with other features. Consult your treating orthopedics surgeon for guidance and further treatment Hope I have answered your query. Let me know if I can assist you further."
},
{
"id": 69783,
"tgt": "Is there a minimally invasive way of removing hemangioma?",
"src": "Patient: Hello Doctor,My husband has been diagnosed with hemangioma in vastus medalis. Symptoms: Pain in thigh. Pain increases in the evening esp if he has a had an active day.The MRI report is as follows.\u201cSmall Patchy irregular hyperintense area in lower third of right thigh medially involving the vastus medialis muscle with minimal extension into adjacent intramuscular planes and extending close to the lower third of femur with no periosteal reaction.Features are s/o soft tissue (muscular) hemangioma in the lower third of right thigh involving the vastus medialis muscle.\u201dDoctors have suggested a surgical excision of the hemangioma along with the some part of the surrounding muscle.My question: Is there a minimally invasive way of removing/treating it?Regards,Worried wife Doctor: Hi.Sure there is.With the advent of the new techniques it is possible to embolize the feeding vessel/s and this should be sufficient to cure the problem.but all this will depend upon the angiography reports.Look for interventional Radiologist or Cardiothoracic surgeon.This is but possible, go ahead for this therapy ."
},
{
"id": 153181,
"tgt": "Is it normal for albumen levels to drop while suffering from gullet cancer?",
"src": "Patient: My mother has advanced gullet cancer and has lost her appetite completly. She is refusing all foods, only living on iced water. Her albumen levels are very low, this surely isn't a good sign. How long can she survive without eating or is this just the natural way the body copes with terminal cancer? Doctor: Hello! I have read your concerns and feel sorry for your mother. Gullet/esophageal cancer's primary problem will be nutrition. A patient with advanced cancer of this type usually has poor nutrition, since he/she is unable to tolerate food intake. A low albumin count only confirms that she is poorly nourished, with this status, most patients succumb to infection due to very low immune system. No patient can survive without eating, the time frame varies on how her body will cope with the stress. If I was the treating physician, I will offer options for alternative route for feeding (tube feeding, intravenous/parenteral feeding, placement of stent), to boost her immune system through nutrition."
},
{
"id": 187543,
"tgt": "Which medication to be taken for terrible tooth ache?",
"src": "Patient: i have terrible tooth ache and the end of a virus which is now attacking my sinuses. I was taking mucolit (finished last dose this morning.) I have taken ibuprofen but it had no effect on the tooth ache. can i take paracetamol with codeine (combined)? Doctor: HelloWelcome to HCMFirst of all you should know the reason of tooth ache so get your tooth x ray done. IOPA or OPG xray done which will reveal the extent of infection in tooth or surrounding structures. Meanwhile you can taken flexon A for your pain.Visit your dentist and according to xray report, treatment will be decided. Thank you"
},
{
"id": 96401,
"tgt": "After i had my last kid it has been hard for me to have a bowel movement",
"src": "Patient: after i had my last kid it has been hard for me to have a bowel movement is it normal also sometimes i bleed when i have one is it possible that something is wrong? Doctor: Ofcourse something is wrong.,after all you are bleeding!"
},
{
"id": 122965,
"tgt": "Suggest treatment for sore and swollen leg",
"src": "Patient: my leg is sore and swollen. it began a week ago when I bent down and stood up. My knee swelled and then the bottom half of my leg. It s stiff and painful. I notice more swelling when I m on my feet but it s bruising now too. Very painful and when I press on around my shin it leaves an indentation. Doctor: Hello, It could be sprain only. As of now, you can use analgesics/anti-inflammatory combination like aceclofenac/serratiopeptidase for symptomatic relief. You can apply icepacks for symptomatic relief. Hope I have answered your query. Let me know if I can assist you further. Regards, Dr. Shinas Hussain, General & Family Physician"
},
{
"id": 119187,
"tgt": "Diagnosed with low hemoglobin and iron level. Chances of getting lymphoma?",
"src": "Patient: my sister very recently has been investigated because of low hemoglobin and iron. they discovered some sort of genetic issue that may be in our family (alpha ?) can that lead to lymphoma? I recently received tx for lymphoma and am quite concerned for my kids and grandkids, especially since my son in taking Remicade q 6 weeks, which makes him more at risk for it too. All these together make me quite anxious for their increased risk. do you have any information or links i can further research? Doctor: Hello, Welcome to HCM, I am Dr. Das Look, your sister is suffering from alpha thalassemia most possibly. It is a defect in hemoglobin synthesis but is minor in clinical course than beta thalassemia. So, do not worry at all. It is not a lymphoma. But her spouse should be tested by HPLC study to rule out any chance of homozygousity or dual heterozygosity in their children. Hope this will become helpful. Regards."
},
{
"id": 177125,
"tgt": "What causes diarrhoea and rashes on chest in infant?",
"src": "Patient: My daughter is 10months old, she has had diarrhea for 15 days. About 4-8 times a day. We have not changed her diet and she drinks soy. The diarrhea has subsided in the past 2 days but she now has a small rash on her chest and sround her armpits. Her mood has perked up but for 3 days she would just lay on ke or the floor. We have been to the doctor 3 times and have no anwers. I am worried there is more to it then just a stomach bug Doctor: Hi...a 10 month old kid with prolonged diarrhea and rash on her chest and sround her armpits....I would like to consider the possibility of zinc deficiency (especially with a background of diarrhea). Other possibility is incidental eczema (skin allergy).Regards - Dr. Sumanth"
},
{
"id": 23402,
"tgt": "Suggest treatment for ischemic heart, chest pain and breathing problems",
"src": "Patient: patient is suffering from angiodysplesia(obscure G.I.bleeding) for 8yrs. she has low haemoglobin level at around 7.5-8%. has to be transfused blood every 4-5 mths. she has been diagnosed with ischemic heart...chest pain, breathlessness in performing daily chores and especially climbing bed. pls advice Doctor: Thank you and pleased to answer you.The term intestinal angiodysplasia has been defined as a superficial acquired single or multiple acquired vascular lesion developed in the mucosa and / or submucosa of the gut wall without being associated with a cutaneous or visceral angiomatous lesion. The etiology and pathophysiological mechanisms leading to the formation of angiodysplasia are not yet fully understood. A hypothesis developed by Boley is the appearance with age of a chronic partial obstruction of the submucosal veins in connection with an increase in contractions of the muscular layer of the colon wall. Investigations leading to the diagnosis of angiodysplasias are closely related to the clinical presentation and severity of digestive bleeding. The development of endoscopy techniques and the quality of video endoscopes used, particularly in terms of image resolution, make endoscopy the first pathway for the diagnosis of angiodysplasias. Treatment is indicated when there is external bleeding or when the bleeding is occult, after eliminating any other cause of bleeding. Asymptomatic I mean non-haemorrhagic, angiodysplasias of incidental discovery should not be treated. The principle of argon plasma coagulation treatment is based on the simultaneous release of an electric current and a gas, argon, which in its ionized form will destroy, in a targeted way and without contact (3 to 5 mm Of distance), the superficial mucosa (0.5 to 3 mm deep), destruction done with an electric arc. It is the reference technique for the destruction of angiodysplasias, whatever the location. Electrocoagulation treatment, or coagulation of angiodysplasias with hot forceps, was performed in the 1980s but is no longer recommended because of the high risk of perforation.The patient also has anemia following recurrent bleeding, and therefore what explains the symptoms. However, treatment is not only based on transfusions, but also on argon plasma coagulation to stop hemorrhage. A recent retrospective study showed a significant reduction in transfusion requirements in all patients treated with argon plasma coagulation. However, hemorrhagic recurrence occurred in 42% of patients after an average follow-up of 4.5 years.In this patient, the anti-aggregation treatment for ischemic heart disease should not be interrupted because of the risk of harming her life expectancy.The heart procedure indications which formally require further anticoagulant treatment may lead discussions with the patient, the surgeon and the cardiologist about the indication of a segmental intestinal surgery.May this informations help youBest wishes"
},
{
"id": 104431,
"tgt": "Chronic sinus, headaches, tingling in ears and temples, tightness in chest. Been taking liquid chlorophyll. What now?",
"src": "Patient: I have had chronic sinus for years that has never been treated properly or gone away. I have been taking liquid chlorophyll in water the past week to help cleanse my system and feel like I have water in my ears, severe headaches with tingling in my ears/temple region. My throat is very hoarse, sneezing always and tighness in the chest when I breath. Im also getting nauseous. Can you tell me if this is the sinuses trying to clear and what I can do about it. Im also taking vitamin supplements and vitamin B liquid. I am sick of having chronic sinus and no solution. My head, neck and shoulders ache constantly. Any advice? Doctor: Your chronic sinuses symptomps are converted to acute sinusitis it got infected Newer studies show that synthetic vitamins increases sickness Stop all vitamins and dietry suppliments Take cefixime 200 mg bd tab Montair fx(combination of montelucast and fexofenadine )twice a day put see water drops in nose 2 drops tds and diclofenic sos for pain add antacid syp do for 10 days get your xry pns waters view consult ent specialist and treat your sinuses properly as they are outcome of allergies"
},
{
"id": 130922,
"tgt": "Why am i having fluttering sensation behind knee?",
"src": "Patient: HiI have a fluttering sensation behind my left to knee to the left side - it's intermittent (mostly at night in bed) and feels like a strong pulse. I am a 60 year old Caucasian male in otherwise good health - only had it for a few weeks and it's not every night) any ideas? Doctor: Hi,Get USG of knee joint done and send me the report.Avoid squatting and weight lifting.If pain occurs then take tab. aceclofenac+thiocolchicoside three times a day.Thanks,Dr. CHANDER MOHAN SINGH."
},
{
"id": 121426,
"tgt": "Suggest treatment for purple discoloration in finger due to an injury",
"src": "Patient: My husband smashed his pinky finger today at work about 14 hours ago. He just told me what happened. about 30 mins or so after smashing his finger he got a pain in the back of his neck. he stated that he does not have anpain in the back of his neck now, but about 1/2 of his finger is purple. He has a blood clot in his leg and is on coumadin. he has had the blood clot for about 7 years now. his blood pressure is 127/76 pulse is 95. Doctor: Hi, Purple discoloration after the increase simply means there is blood deposit in your finger. Do you have any X-rays of it? As it might be a fracture or a soft tissue injury which should be confirmed. If not then please get the X-ray done, apply ice pack and this discoloration may takes 15 days to go off. Hope I have answered your query. Let me know if I can assist you further. Regards, Dr. Jaideep Gaver, Orthopedic Surgeon"
},
{
"id": 141066,
"tgt": "What causes excessive anger and aggressiveness while on Effexor in a bipolar patient?",
"src": "Patient: My daughter in law is bi polar. She is on lithium among other meds. Three and a half months ago she was placed on Effexor. As of right now my son is at home with her watching her burn thngs, break thngs. She had guns, but he managed to get those away from her. As far as we can tell she is angry at the world. Could this be from the Effexor??? Doctor: Hi, t her symptoms could be related to Effexor adverse effects or acute psychosis. Coming to this point, antipsychotic drugs may be needed. For this reason, I recommend consulting with a psychiatrist as soon as possible to help prevent her from doing harm to herself and the others. Hope I have answered your query. Let me know if I can assist you further. Regards, Dr. Ilir Sharka, Cardiologist"
},
{
"id": 46408,
"tgt": "What is the treatment for a kidney infection?",
"src": "Patient: I found out about a week ago that I have a kidney infection and was put on antibotics. Since then I have had sex twice and both times have bleed during sex. I had an ablation about a year ago and dont have periods anymore so I know its not my period. Doctor: Hello and welcome to HCM. As an Urologist,i can understand your anxiety.To confirm kidney infection,after examination by Urologist,do these tests :1. urine routine and culture.2. blood routine,RBS, creatinine,LFT,uric acid and platelets.3. Ultrasound -KUB or CT-IVU.According to reports,treatment is advised.Drink about 2 liters fluids daily.If you've any doubts,send the reports to me,as a direct question.Dr.Matthew J. Mangat."
},
{
"id": 21952,
"tgt": "Suggest treatment for sinus tachycardia",
"src": "Patient: Previously I was suffering from Junctional Tachycardia, I was admitted in B M Birla at Calcuta, and Angio and RF ablation has been done by the Dr. Dipankar Mukherjee and Dr. Azizul Hazue . No block found in the Angio and RF Ablation has been done successfully. Now my pulse beat are going up . Sometimes it goes up 120 bpm. My doctor diagnosed it as Sinus Tachycardia and told me it will not be fatal. Kindly help me by advising by your valued opinion. how can I get rid of it ? Doctor: Hi ThereIts a pleasure answering your query, as you have alredy gone through angiography and RFA for your juctional tachycadia focus has already been ablated, you need not worry much.Now sinus tachycardia is a condition which we all have when we exert or do more than usual physical activity. But if its troubling you to much then you can start a low dose beta blocker after consulting your treating doctor. And yes it is not fatal and even anxiety can cause tachycardia so relax yourself.Good Luck"
},
{
"id": 159041,
"tgt": "Cancer. Used steroid. Diagnosed with avascular necrosis. Had hip replacement surgery. Suggest?",
"src": "Patient: My father, age 78, has been diagnosed with avascular necrosis due to steroid treatment of cancer . He also has a broken hip. His team of doctors cannot find any way to help him since he is not strong enough for hip replacement surgery. He has had a couple of heart attacks and quadruple bypass about 14 years ago. He also has an abdominal aortic stent . As you could imagine, he is in alot of pain and looking for relief. He is planning on trying medical marijuana to alleviate his pain and in hopes of increasing circulation to the femoral tip. Is this a safe for him to try? He is very determined to try it immediately. Please help me to know how to help him. Doctor: Hi, Sorry to hear about your father's condition. chronic use of steroids can lead to problems like avascular necrosis of femoral head, in this case surgery can only relieve the pain. You should better consult all the specialists like cardioanaesthesiologist, orthopaeditian, cardiac surgeon for discussion regarding possibilities of operation, otherwise you may also consult pain specialist for chronic pain management."
},
{
"id": 145044,
"tgt": "How to treat numbness in fingers after taking trigger point injections for neck pain?",
"src": "Patient: I Have numbing in my finger tips after getting trigger point injections to help with nerve pain n pain in neck, arm n hands, in the morning usually... Now I have constant numbing n tingling in my fingertips.I m supposed to go for an epidural in 2 weeks. ....But can not deal with this, what could this because of and what can I do to help it in the meantime. ? Doctor: Hello dear, Your narration seems to be related to cervical spondylosis with radiculopathy. Treatment depends upon the extent of involvement. Physiotherapy, neck collar may help in many patients. Medical treatment with pregabalin, gabapentin or amitryptiline reduces perception of pain and methylcobalamin may be useful for nerve nutrition. Anesthetic agent in subdural space improve pain complains. At last surgery is required. You need to consult a neurologist or neurosurgeon for proper examination and evaluation. Hope you found the answer helpful. Wishing you good health Dr Neeraj Kumar Neurologist"
},
{
"id": 35321,
"tgt": "Suggest treatment for bone infection due to osteo mellitus",
"src": "Patient: Hi i would liketo ask you a question about osteo mellitus. My son is on antibiotics for atotal of five weks. he had a sever foot infectioon. the infection is to the bone. iIs it anyway possible that he could avoid amputation. The doctor says there is no way medically the antibiotic wouldclear theinfection up. He said every bone inis foot wasinfected. So the doctor said when he takes him off in 6 more days he will spike a fever and his foot will blow back up. Just curious aboutyour answer Doctor: Hi! thanks for sharing your son's long-standing and severe health problem with us!In such cases of chronic non-healing or non-responding osteomyelitis, especially affecting the lower portions of legs, when a child visits me, I usually consider following things:1. any injury to skin or loss of skin/soft tissue is dangerous!2. microbiological culture/sensitivity report is essential to kill the offending organisms3. chronic infection of bone leads to bone death which is irreversible4. in long-standing cases tuberculosis, fungus and other unusual infection also take place5. left untreated, the disease is self progressive that means it spreads quickly to adjacent areas/bones6. months of antibiotic often fails to heal it and surgery is then must to prevent further lossTherefore, in my opinion, in your son's case, at least partial surgical debridement would be helpful if not an amputation!Hope got the answer; anyways, Also don't forget to write to us in case any more queries.Wishing your son good health!"
},
{
"id": 2804,
"tgt": "Are tamoxifen and ova care effective for pregnancy?",
"src": "Patient: I'm trying to get pregnant and my Gynecologist prescribed me tamoxifen-10mg 3 times a day for5 daysovacare forte 2 tmes a day for 15 daysARG-9 twice a day for 7 daysbarkeit 3 times aday for 7 days (this tab for me and my husband)i wanted to know how these medicines help Doctor: Hi, Tamoxifen is for increasing growth of your eggs. Ova care forte also helps in building up your pregnancy chances as it contains some vitamins. Arginine is for making your uterus receptive for conceiving. Barkeit helps in thinning the secretions of cervix and helps in increasing motility of sperms. Thus these medicines will help you in conceiving.Hope I have answered your query. Let me know if I can assist you further. Regards,Dr. Khushboo Priya"
},
{
"id": 31614,
"tgt": "Suggest the vaccine schedule for hepatitis c positive",
"src": "Patient: dear doctor, my name is nafees. my elder brother's wife is about 45. she has been diagnosed hepatitis c positive. sir, it is mentioned ANTI HCV AB. \"POSITIVE\". should she be given vaccination? what is the schedule of it. how long will it continue. please tell me . God Bless u! thanks. nafees. Doctor: Hi, THanks for posting in HCM. I understand your concern for your brother's wife. The goal of HCV treatment is to cure the virus, which can be done with a combination of drugs. The specific medications to be used and the duration of treatment depend on a number of factors. There are a number of approved therapies to treat HCV, which include: 1. Harvoni - sofosbuvir/ledipasvir, 2. Daklinza - daclatasvir, 3. Olysio - simeprevir, 4. Sovaldi - sofosbuvir, 5. Technivie and Viekira Pak. Kindly consult Physician for proper evaluation and appropriate treatment upon assessment. Hope the information provided would be helpful. All the best."
},
{
"id": 106915,
"tgt": "What causes severe lower back pain radiating towards the legs?",
"src": "Patient: Suddenly this morning I got really excruciating lower right back pain. I was standing in the kitchen and it came on suddenly. It s the lower right side. It does not go down in the my legs it s just isolated to my back. I have no nausea or no blood in my urine. Any idea on what it could be? Doctor: Hello! Welcome to HealthcareMagic! Lower back pain may cause by right lumber disc prolapse which compresses the nerve. but exact other causes are diagnosed by physical examination or by MRI. Common causes of lower backache and leg pain is a lumbar ruptured disc or herniated disc. Symptoms of a herniated disc may include dull or sharp pain, muscle spasm or cramping, sciatica, and leg weakness or loss of leg function. Sneezing, coughing, or bending usually intensify the pain.Rarely bowel or bladder control is lost, and if this occurs, seek medical attention at once.Sciatica is a symptom frequently associated with a lumbar herniated disc. Pressure on one or several nerves that contribute to the sciatic nerve can cause pain, burning, tingling, and numbness that extends from the buttock into the leg and sometimes into the foot. Usually one side (left or right) is affected. Take care Regards, Dr. Akhilesh Kumar, Physical Therapist or Physiotherapist"
},
{
"id": 182942,
"tgt": "Suggest remedy for ulcers inside mouth",
"src": "Patient: Doctor I am having ulcers in my mouth from past few days due to which I am not able to eat anything...any time I eat sumthing homemade my mouth use to burn like hell..hav got vizylac capsules from the chemist...so is it ok for me to hav it...twice a day as the chemist prescribed....thank you Doctor: Hello, Thanks for consulting HCMRead your query ,as you have ulcer in mouth dont worry ulcer can be due to vitamin B deficiency , drug induced , Nutritional deficiency , hormonal influcences. For ulcers you can apply ointment Mucopain or Mugel for topical application two - three times a day, you can take drug like vitamin B complex..Hope this will help you. Wishing you good health."
},
{
"id": 163774,
"tgt": "What causes pus filled bumps on the forehead of a child?",
"src": "Patient: Hi, I have a question and I m hoping you could answer it.. My 11 month old daugther developed a zit on her forehead this past week.. It looks like it is filled with puss.. I just noticed another small one on the other side of her forehead. What could be causing it? Doctor: Hi... by what you say I feel that your daughter may be having pyoderma of this call which is bacterial infections of the skin and subcutaneous tissues.Skin problems are best diagnosed and treated on seeing them directly. I suggest that you upload an image of the same on this website and get back to me. if it is pyoderma your daughter might require an antibiotic therapy which is a prescription drug.Please revert back to me with images so that I can guide you better.You can approach me at the following link.Once the page opens there will be an option below my image as \u2013 ASK ME A QUESTION \u2013 click on it.Please find the link below -www.healthcaremagic.com/doctors/dr-sumanth-amperayani/67696Regards - Dr. Sumanth"
},
{
"id": 171836,
"tgt": "What is causing the phlegm to stuck in his throat?",
"src": "Patient: My 8 year old son has been constantly clearing his throat for some time, I took him to the pediatrician last month, she precscribed a nasal mist for him, but it was not covered,(he doesnt want something up his nose either). Now he is complaining and crying because he says there is phlegm he cannot get out. It seems to be stuck in or around his throat. He is making barking noises and really trying to get it out. I dont know what to do for him at this point, any suggestions on what could be happening or causing this? Doctor: Thanks for asking Healthcaremagic. Phlegm is formed as a result of bacterial infection. I would suggest to give him a syrup which contains terbutaline and Guaifenesin which help phlegm to loosen and come out. Give him 5ml three times a day. Steam inhalation is also helpful. Tell him to have plenty of fluids and donot forcefully try to get phlegm out as it can cause further damage. He can be started on appropriate antibiotics if still the problem persists. Hope it helps. Thanks and Regards."
},
{
"id": 126256,
"tgt": "What causes right-sided pain in the trapezius muscle and chest?",
"src": "Patient: I am having a stabbing pain in my right trapezius muscle through the lateral aspect of my right shoulder blade by my rotator cuff. It is more of an awareness of discomfort that turns into a stabbing or shooting pain when I take deep breaths and my lungs are close to full expansion. On full breath expansion I am also having right sided chest discomfort about mid chest area, below the breast, that started laterally, early this morning, but is more medial now. I am not sure if the pain is radiating from the chest to my shoulder or vice versa. I am wondering if it is a pinched nerve, but don t want to ignore something much more serious. Symptoms started about 4am? I am an ICU nurse and move patients (including my shift last night) as well as recently restarted back at the gym. I lifted upper body weights last on Friday December 29th. Doctor: Hi, It may be due to musculoskeletal causes. As the first line of management, you can take analgesics like Ibuprofen or Diclofenac for pain relief. If symptoms persist you can consult an orthopedician and get evaluated. Hope I have answered your query. Let me know if I can assist you further. Regards, Dr. Shinas Hussain, General & Family Physician"
},
{
"id": 182598,
"tgt": "What is the use of silver tooth in a kid?",
"src": "Patient: okay...My kid has 2 silver tooths at his side they re both connected he s 12 and one of them is falling out...Is silver tooths just to prevent a disease even if you have baby teeth or is it protecting from your adult teeth? Doctor: Hello, Thank you for consulting with HCM.Actually these silver tooth which you are mentioning are artificial crowns on the tooth, these artificial crowns are placed to maintain gap between teeth for permanent teeth, as now permanent teeth are erupted you can get them replaced.Hope it will help you."
},
{
"id": 176584,
"tgt": "What causes recurring cold and cough in an infant?",
"src": "Patient: Hi, my on is 19 month old and weight 10 kgs. He gets frequent cough and cold. Right now he is suffering from blocked nose, fever and bad cough. Giving him sinarest af and cherricof for cough. Plz suggest if relent or ambrodil s is better? Can I give them with zyrtec drops instead of sinarest? Doctor: Hello Welcome to Health care magic Children are more vulnerable to catch cold and suffer from coryza. if not treated well the condition can convert to Whooping cough which is much more irritating and little difficult to cure.Homeopathy has good response over such case and provide complete cure. you can visit to any homeopathic physician for treatment.Meanwhile i am writing some medicine you can start as prescribed Agrahic Nutens 30 6 pills twice in day Antim tart 30 6 pills twice in day Dulcamara 30 6 pills twice in day Ars Iod 3x 4 pills twice in day Take the medicine for 15 days Hope this is clear to you in case of any query feel free to ask Have a good health Thanks Dr Ratnesh"
},
{
"id": 222721,
"tgt": "Suggest treatment for pancreatitis during pregnancy",
"src": "Patient: Does having pancreatitis cause fertility or pregnancy problems? I am 31, 145 pounds and 5 5 tall. I have had two live births and three miscarriages. My second baby was born 7 weeks early. There was no cause listed as to why they couldn t stop my labor. He is 3 now and doing well. But we were thinking of trying again but I started having symptoms of pancreatitis while pregnant with him I had my gallbladder removed and this past 6 months my sypmtoms have returned and are worse than before. My insurance won t pay to have the tests done to verify that it is pancreatitis. Doctor: Hello, and I hope I can help you today. Pancreatits can be a life-threatening condition of it is not diagnosed and treated properly. Even if you had your gallbladder out, thickening of your bile can still cause blockages and pancreatitis. Because pregnancy makes bile thicker, I really advise not trying to get pregnant until you have a diagnosis. If you have symptoms of upper abdominal or back pain, and nausea, you really should go to an emergency room to be evaluated... I find it difficult to believe your insurance doesn't cover even a regular visit to a medical doctor to evaluate your symptoms. But you certainly shouldn't get pregnant until you have a complete evaluation because pregnancy can make pancreatitis worse and harder to treat. I hope I was able to answer your question today and that my advice was helpful. Best wishes, Dr. Brown"
},
{
"id": 53996,
"tgt": "Suggest treatment for gall bladder cyst",
"src": "Patient: Hi Doc, I'm been diagnosed with Gall Bladder Cyst its about 0.8 cm in size. But I been getting pain on the lower right adomen. Its pain comes and goes away between 2 to 4 times daily. I check few tests and I was told I should get the cyst removed by surgery. Do you think its necessary? Because I don't mind going to sugery as long as the Gall bladder is not removed, only the cyst. Doctor: Hi and welcome to Healthcaremagic. Thank you for your query. I am Dr. Rommstein, I understand your concerns and I will try to help you as much as I can.If there are symptoms then surgery is recommended which is simple and routine procedure today. This cyst may only grow in future and cause problems so it is better to do permanent solution by removal of whoe gallbladder.I hope I have answered you query. If you have any further questions you can contact us in every time.Kindly regards. Wish you a good health.DR. Ivan Rommstein"
},
{
"id": 112367,
"tgt": "Lower back pain, no relief with physical therapy, yoga, cortisone injections. MRI done. Possibility of Multiple Sclerosis. Suggestion?",
"src": "Patient: Hi, My 25 yr old daughter has been on bed rest for lower back pain since February and has tried all kinds of things medical and naturopathic. It was first thought to be sciatica. It has gotten progressively worse even with all the physical therapy, cortisone injections, accupuncture, yoga, etc. She just had an MRI done with a note of Multiple foci of subcortical and periventricular white matter and have told her that MS is a possibility. She isn't having any balance issues or numbness and tingling. Can you think of anything else we could look in to? She is being scheduled for a lumbar punch and a contrast MRI. We are at a loss as are several orthopedic specialists. Doctor: Hello, I have studied your case.Another possibility of multiple myeloma.You may need to do serum electrophoresis and urinary ben jones proteins.If both test comes normal you can do lumbar puncture.Hope this helps. Wish her a speedy recovery. Take care"
},
{
"id": 55086,
"tgt": "Is gall bladder functioning at 25% a cause for concern?",
"src": "Patient: I had a hida scan and my results was my gallbladder is functioning at 25%. I declined to go to a surgeon because I was informed a normal gallbladder functions at 35% or more. My thought was my ejection fraction was a high abnormal so I shouldn t immediately assume my gallbladder needs to be removed. I can go days/weeks without any attacks but when I have one they are severe. What is the pros/cons in waiting until the attacks are more frequent? Doctor: hi.noted history of gallbladder pathology. if it is secondary to gallstones and your symptoms are occurring frequently and increasing in severity, it is best if you consult with a doctor, preferably a general surgeon, for physical examination and clinical evaluation. what you have is chronic cholecystitis, probably secondary to cholelithiasis (gallstones). the most important thing to take note here is that an elective or scheduled cholecystectomy has better post-operative outcomes and less risk of developing complications as compared to the one done emergently. diagnostics (such as ultrasound, liver function tests, blood count, etc.) and management (medical and surgical) will be directed accordingly. definitive treatment for gallstones is cholecystectomy or surgical removal of the gallbladder along with the stones. it can be done laparoscopically (goldstandard) or thru open technique. low fat diet is also recommended.hope this helps.good day!!~dr.kaye"
},
{
"id": 108917,
"tgt": "What causes lower/upper back pain?",
"src": "Patient: my 14 year old son has been complaining for over a year about back pain. it comes and goes, but mostly there's always some pain. Earlier it was his lower back, now it's his upper back. besides medication, what can I do? what should I attempt to rule out? doctors just say he's strained it, but it has been going on for too long now. Doctor: Hi,At this age there can not be any thing wrong with his back.It seems that there might be having lack of physical exercise, remain sitting in front of computer for long time.Ask him to go for regular exercises like running, swimming, walking.Pass some time in sports.Give him high nourishing protein food.Ok and take care."
},
{
"id": 222597,
"tgt": "What causes white yellowish discharge during pregnancy?",
"src": "Patient: hello im wondering if you can help me, im 34 weeks pregnant with my third child and for the last week or so ive had a really heavy thick discharge, its kind of a white yellowy colour with a bit of brown, i know this mite disgust you but im a smoker and im really worried there is something wrong the baby doesnt move much but hasnt rite through my pregnancy can you tell me if this discharge is normal. Doctor: Hello dear,I understand your concern.In my opinion thick yellow vaginal discharge needs examination to rule out any infection.The optimum fetal movements should be atleast 10 in a span of 12 hours.I suggest you to consult doctor and get vaginal examination done to check for any infection.In case of any infection antibiotics might be needed for it as these infections might predispose to preterm pains.Smoking during pregnancy causes growth retardation of fetus and low IQ in baby.So stopping smoking during pregnancy is essential.Best regards..."
},
{
"id": 8642,
"tgt": "Doctor suggested not to bleach face. Applied oxy bleach resulting dark patches. Fucibet cream or calamine lotion will work?",
"src": "Patient: Hi Doctor,I am taking Kaya's treatment for almost 1 & half year for acne. and it really worked for me. Doctor suggested not to bleach. But i am getting married in two days and hence my beautician used Oxy bleach over my face. After bleach i have got dark patches on my cheeks and forehead. i am really scared now, how to get rid of it into 2 days. i cant even go out as i within 2 days its my wedding. Fucibet cream will work for me ? or a calamine lotion? please suggest. Doctor: Hi, Welcome to HCM. you should have followed your doctor's advise. One should not use facial bleach while taking peels for acne. Now to treat the dark patches, apply mometasone cream twice in a day and keep the area moisturised by using cetaphil moisturising lotion many times in a day. Hope that helps. Take care"
},
{
"id": 177329,
"tgt": "Suggest treatment for cough in 3 month old baby",
"src": "Patient: My small baby is 3 Months old.He had a Lymphatic Node swelling in left Axilla.Doctor removed that by surgery before 3 days ago.Now he is in hospital but having more cough,due to which he face too much pain at operative area & just crying every time.Which cough expectorant I should give him to relief very fast.Plz help me . Doctor: cause of the cough is more important which u failed to elaborate in the historyanyways try giving coscopin syrup 2 ml thrice daily it's a suppressant"
},
{
"id": 92699,
"tgt": "Pains on upper left side below the middle of rib cage. Specific causes ?",
"src": "Patient: well i have had pains on my upper left side below the middle of my rib cage it has been going on for a while now and sometimes it will move from there to my stomach . i have lost my apatite for things but when i get it back i eat like i haven t eaten in days. I am a skinny guy 22 years old 125lb last time i went to the doc was for this about a week ago and now have an appointment with a stomach DR but is there anything i can do to ease the pain for the 4 weeks that it will be b4 i see him Doctor: Hello, Thanks for the query to H.C.M. Forum. If I were your treating for this case of pain in the left rib cage , I would come up with these possibilities , these includes.1 The first possibility is of peptic ulcer disease or Hiatus hernia. Diagnosis can be confirmed by upper G I endoscopy . Try raising the head of your bed 4 inches above with blocks.It might also help to avoid drinking or eating 2 hours before you lie down. To help control the stomach acid , one should not drink alcohol or drinks with caffeine in them or eat chocolates or spicy food or greasy good. Also take some antacid but if, symptoms are severe you may need drugs like proton pump inhibitors.2 As you mentioned that you lost appetite so this may be due to some disease in liver , diagnosis can be confirmed by ultrasound of liver . Also get in blood for liver function test as jaundice is the main cause of fatigue and loss of appetite. 3 tension is the one cause , which may be responsible your problem , so if you think so , avoid the same. In my opinion antacid with oxetacaine in them will help you till your doctor's consultation. Good luck, Dr. HET"
},
{
"id": 219389,
"tgt": "Suggest remedy for dysuria during 5th month of pregnancy",
"src": "Patient: Hi, my wife had 5th month pregnant and suffered from pain while passing urine.. gyanec presicribed for urine analysis and urine culture determination. after 48hrs eptilial cells observed 10-12 on reports. my wife little bit tensed. may I know the treatment?? how to reduce these cells throughout pregnancy?? eptilial cells causes any damage to pregnancy?? Doctor: Hello,Thanks for writing to us.Hello,Thanks for writing to us. Followings are my comments:According to your information, it is suggestive of Urinary Tract Infection. 10-12 epithelial cells are detected on culture test. They usually don't effect in pregnancy but needs to be treated.TREATMENT: I suggest to take one course of ANTIBIOTIC (like Cephalexin or Nitrofurantoin) & URINE ALKALISER (to be taken with water). It will help to control urine infection. Kindly, procure complete prescription from your doctor.PREVENTION: You must drink plenty of water (2.5 to 3 lit/ day) to flush out the body toxin, take cranberry juice (one glass) regularly, maintain genital hygiene properly, empty bladder in time and wear clean-cotton underwear only.Hope, it helps for you. Wish you good health and take care."
},
{
"id": 163545,
"tgt": "What does this thyroid test result for developmental delay in a child indicate?",
"src": "Patient: Hi,7 month Baby Development delayed...done THYROID test and result show up as below...is it normal ? TOTAL TRIIODOTHYRONINE (T3) C.L.I.A 189 ng/dl 60 - 200. TOTAL THYROXINE (T4) C.L.I.A 9.3 \u03bcg/dl 4.5 - 12.0,THYROID STIMULATING HORMONE (TSH) C.L.I.A 1.62 \u03bcIU/ml 0.30 - 5.5,FREE TRIIODOTHYRONINE (FT3) C.L.I.A 4.15 pg/ml 1.7 - 4.2,FREE THYROXINE (FT4) C.L.I.A 0.97 ng/dl 0.70 - 1.80 Doctor: Hello,This report is suggestive of normal thyroid function.Hypothyroidism is one cause of developmental delay. Your child needs to be investigated for other cause of developmental delay.Hope I have answered your query. Let me know if I can assist you further.Regards,Dr. Priyank Ghanchi"
},
{
"id": 33887,
"tgt": "How to prevent getting infected with Gram positive Cocci?",
"src": "Patient: i work in the health care field, and a client of mine, that I work directly with everyday, was diagnosed with Gram positive Cocci, along with several other bacteria realted issues. I have a 2 year old at home,and I'm worried of the fact that I could become infected and bring it home with me? Help me, PLEASE!!!! Doctor: hi..thanks for your queryAs your paramedical staff, you need to be careful while handling patients and body fluids. One simple and best measure to avoid infection is hand washing with antiseptics. If you follow this you will not get any infections from patients. Moreover, person with good immunity will not easily get infection. I suggest you wash your hands before going home.Thank you"
},
{
"id": 20657,
"tgt": "What is mortality rate after getting stent inserted?",
"src": "Patient: My mother had a cypher stent placed in her artery. She did not have a heart attack but her artery had collapsed and they caught it before anything more serious happened. She is very concerned that her life is shortened and is worrying that her life will be drastically decreased as multiple people she has known died within seven years of a stent procedure. She is getting very stressed about this because she cannot get an answer about her mortality rate, I want her to stop stressing as obviously its not good for her, so please advise if her mortality rate is effected in anyway due to the circumstnce. Doctor: Hello Welcome to HCM Angioplasty or stent implantation is a very reliable treatment for heart blockages. Now a days with the drug eluting stent the chances of stent narrowing again or not being accepted by the body has decreased significantly. The cypher stent is a drug eluting stent. Drug eluting stents usually remain patent for 10-12 years. With regular check ups with ECG, 2 d echo and stress test any possible complications can be diagnosed and avoided. Also a lot depends on the pumping function of the heart. If the person has sustained a heart attack earlier and the heart pumping function (LVEF) had decreased, then the life expectancy goes downm But this is due to the damage caused by the attack and not because of the stent placement. People with decreased pumping function and with stents also do well if proper precautions are taken and medications are taken regularly. In my practice I have seen people with stents doing well even after 40 years requiring no additional procedures except routine tests and check angiographies every 10 years. So please stop worrying about the life expectancy. It is quite good. One of the indications of putting in a stent is to improve the quality of life and expectancy. She should be compliant on her medications and regular brisk walking or cycling with diet restrictions.A health check up should be done every one year with ECG, 2d echo and a stress test.Wishing her good healthRegards."
},
{
"id": 113266,
"tgt": "Severe pain in back, which reaches to leg. MRI shows degenerative diffuse disc, low signal intensity oval / curvilinear lesion. Suggestion?",
"src": "Patient: Doctor i have severe pain in my back and consulted to doctor who further advised to go for the MRI test which is expalains . 1.excruciating pain on my back and the pain reaches to the legs ass well . reports says 1.L4 -L5 degenerative diffuse disc bulge compressing thecal sac with focal right paracentral disc protrusion with mild upwards migration, compression right L5 nerve and narrowing right neural foramen. 2. There is well defined low signal intensity oval / curvilinear lesion T1 and T2 in right aspecct of the spinal canal , epidural to L4-L5 disc , it may represent extruded disc material which also contribute to thecal sak compression on right asspect. Doctor: Hello. Thanks for writing to us. The findings of your MRI are suggestive of a degenerative changes in the spine with disc prolapse which is causing a nerve root compression. Most of the times, the symptoms can be taken care of with regular exercise and medicines. I hope this information has been both informative and helpful for you. Regards, Dr. Praveen Tayal drtayal72@gmail.com"
},
{
"id": 50785,
"tgt": "Small left kidney, no problem in left side body part. Recommendations?",
"src": "Patient: HI Im Roberto Dela Pena of Philippines,My problem is i have a small left kidney,it was found during my first medical test which KUB is mandatory,the doctor found out that my left kidney is small it was 2007 ago,can you explain me about this situation because everytime when i conduct a medical exam for work abroad they always found thesame result small left kidney,but i did not experience any trouble in my left side body part,can you explain me about this one,here is my e-mail add: robertodela YYYY@YYYY , Thank very much,more power to all of you.. Doctor: Hi Thanks for your query. Uni-lateral small kidney is usually acquired after a disease affecting it. In your case, you may correlate with your past medical history. In such a case the other kidney enlarges to compensate for the function. Thus the person has no problems from it. Sometimes it is present since birth. Then it is called as congenital hypoplasia. The pelvocalyceal system is small and usually opacified in intravenous urography. Again compensatory hypertrophy of the contralateral kidney happens. Associated anomalies of kidney are present. You may watch for symptoms of urinary tract infection and other kidney disorders as it may affect the function of the other kidney. Take care. Dr Vaishalee"
},
{
"id": 155218,
"tgt": "Is painful intercourse,back pain and pain in vaginal area, symptoms of bartholin gland cancer?",
"src": "Patient: I have been to my gynelogist several times this year with different issues. Painful intercourse, leg and lower back pain. I was prescribed a hormone replacment therapy that did not work. I went back to him a week ago with severe pain in the vaginal area and buttocks. I was diagnosed with a barthilon abcess and was prescribed an antibiotic therapy. Didn't work. Went back yesterday and now he is telling me that I need surgery - possiblity of cancer(!) Needless to say I am in shock today. The symptoms I have today were present when he first saw me only now that have excellated to unbearable. Where can I search for answers to this issue? I have to go to a surgeon next week and I want to be fully armed with my questions so that I will receive the right treatment. What are the symptoms of barthilon gland cancer? Doctor: Thanks for your question on HCM.I can understand your situation and problem.Bartholin gland cancer can present as bartholin cyst or abscess initially.And if not relieved by routine treatment like antibiotics than possibility of cancer should be raised.Symptoms are vaginal discharge, lower abdominal pain,pain during sexual act, low grade fever, weakness and weight loss. Sometimes lump can be felt too.But you need biopsy and histopathological examination first to confirm cancer.So consult surgeon and ask him about biopsy first."
},
{
"id": 140513,
"tgt": "What causes sudden fainting with loud sound coming from the throat?",
"src": "Patient: hallo, my father aged 61 yrs old was unconscious on 31st jan. & 4th feb for 40-50 seconds and a loud noise was coming from his throat. while consult with M.D.(Med), he told that it seems to be a neurological problem so we consult with neurosurgeon at mumbai. the neurosurgeon asked us to perform few tests like MRI OF BRAIN, Angio , 2D Echo, Lipid profile & EEG of Brain., but every report concludes Normal. we cannot diagnose the exact reason for that 2 incidents why it was happen i.e. Unconscious incident. plz suggest any treatment or solve this mistry. thank you. Doctor: Hello, Since the neurological problems were cleared out, in my opinion, cardiac conditions (arrhythmias for example) should be considered. In addition to the performed tests, I think is necessary a Holter monitoring for 24-48 hours. Discuss with a Cardiologist for these issues. Hope I have answered your query. Let me know if I can assist you further. Regards, Dr. Erion Spaho, Neurologist, Surgical"
},
{
"id": 198091,
"tgt": "What is the suggested treatment for inner anal scar tissue?",
"src": "Patient: Hello,I am a gay man and my boyfriend has inner anal scar tissue from a prior bad sexual relationship. It has become increasingly difficult for us to be intimate because of this. Is there any way that this can be rectified? Or at least take the pain down? Doctor: Use local anesthetic cream to reduce pain. If still persists, you may need to consult doctor to get a check up as sometimes anal fissure may cause such pain. Try using lubricants during sex."
},
{
"id": 77606,
"tgt": "Suggest treatment for sharp chest pain while breathing",
"src": "Patient: I have asthma and I have a cold at the moment, for some reason, when I bend over and take a breath, I occasionally have a sharp pain in the left side of my chest. I am not sure if this is because my bra is too tight, or if this is something that needs to be taken care of by a doctor or through the use of medicine. Doctor: Thanks for your question on Health Care Magic. I can understand your concern. Possibility of muscular pain due to tight bra is more. But since you are asthmatic, we should rule out lung infection and worsening of asthma because both these can cause similar symptoms. So consult doctor and get done clinical examination of respiratory system, chest x ray (to rule out lung infection) and PFT (Pulmonary Function Test) (to rule out worsening of asthma). If all these are normal then no need to worry much. Wear appropriate size of bra. Apply warm water pad on affected areas. Take painkiller. Don't worry, you will be alright. Hope I have solved your query. I will be happy to help you further. Wish you good health. Thanks."
},
{
"id": 203100,
"tgt": "How to treat itchy brownish/reddish bumps on penis and scrotum?",
"src": "Patient: Hi, ive got brownish/reddish bumbs, they are all together its not like i got one in my penis and another one in my sack its like a spot with bumbs, on my ball sack and it itches when i touch it im a virgin so i dont think its herpes. ps, ive got it since i was born. Doctor: HelloThanks for your query,based on the facts that you have posted it appears that you have Sebaceous Cysts over scrotum and shaft of the penis.The scrotal skin is rich in sebaceous glands and hence prone to get more sebaceous cyst due to accumulation of sebum beneath the skin.Normally they fade away without treatment however they need to be treated if increase in size or get infected.Please consult qualified General Surgeon for clinical evaluation and further treatment.If needed some of them which are bigger may need to get excised in Toto (Completely along with the sac)Dr.Patil."
},
{
"id": 173772,
"tgt": "What are the cluster of little red bumps on my toddler's body?",
"src": "Patient: my toddler has developed a rash with little red bumps on clusters on her chest, thighs, nevk, legs and arms. the bumps mainly appear in the evening and fade in the morning. nothing has changed in her diet or enviroment . doc said flea bites but I don't think they would just appear in the evening. Doctor: HelloIt appears to be urticarial (allergic) rashes.The reason for coming up in the evening and night time can be due to hypersensitivity to insect bite or to any particular chemical to which he get exposed in the evening.Use of antiallergic like cetrizine helps in relieving the acute symptoms.Adding montilukast helps in controlling the repeated attack.So my suggestion is to give montilukast cetrizine combination and try to find out the causative agent so that it can be better avoided.Regards"
},
{
"id": 218945,
"tgt": "When can test for pregnancy after unprotected sex?",
"src": "Patient: Hello i am a 21 year old female and am scared. I had unprotected intervoyrse 5 days after my period ended and im stressing over the fact that i thought it was one of my safe days but it was day 10 of my ovartion cycle . Should i take a pregnancy test and if so how soon can i take the one at home. Doctor: please don't get scary about pregnancy . otherwise you will develop anxiety , will cause further harm to your body and pregnancy too. safe period vary from person to person as your cycles may not be regular or on same date . if you have over due date you were expecting and it's few day over your last menstrual period then you must get UPT, urinary pregnancy test done at home . That is single step method & quite efficient ."
},
{
"id": 81837,
"tgt": "Suggest remedies for discomfort in stomach and tightness in chest",
"src": "Patient: Hi, I am having a really hard time to burp since yesterday and it seems like all the food that I ate was just stock inside my stomach. I m feeling a bit tight on my chest right now and there s this little feeling that I want to vomit. Can you give me a home remedy? I ve tried to drink sodas to burp but was unhelpful. Thank you. Doctor: Thanks for your question on HCM. In my opinion you are having GERD ( Gastro Esophageal Reflux Disease ). It is due to laxity of gastroesophageal sphincter. Because of this the acid of the stomach tends to come up in the esophagus and cause the symptoms. Try to follow these steps for better symptomatic relief. 1. Avoid hot and spicy food. 2. Avoid large meals, instead take frequent small meals. 3. Avoid stress and anxiety. 4. Start proton pump inhibitor and anti emetic combination. 5. Go for walk after meals. 6. Keep 2 - 3 pillows under head in the bed. 7. Avoid smoking and alcohol if you have habit. 8. Loose weight if you are obese."
},
{
"id": 132052,
"tgt": "Could pain in left shoulder after bending be sprain?",
"src": "Patient: my grandson was falling out of his chair. I reached out and grabbed him, and felt bad pain in my left shoulder. now, if I try to lift even a small object by lifting it laterally, I can only lift it to a horizontal position. I can lift it straight out I front of me all the way up, but not to the side. while it does cause discomfort, that is not what prevents me fromlicting it, ijust cant do it. any thoughts Doctor: dear user, do not worry about shoulder pain for your grandson.at home just ask him to apply any pain reliving ointment or diclofenac gel for 15 - 20 mins. must cover the area applied by gel. after 20 mins gently massage it with warm water which relives pain.then some shoulder excercise like, lifting hand full up and down,lifting hand straight over the head from side direction.standing near the wall and make hand lift up gently with the support of fingers.all these excercises can be done 10 times twice a day for a week.if you think still no improvement then take hiim to physiotherapist for ultrasonic therapy."
},
{
"id": 190662,
"tgt": "What could be the small white bump on gums? Possible reason for recurring lesion on cheeks?",
"src": "Patient: i had leukoplakia , which is more or less cured now, it took nearly 9 mths to heal. but i have a very small whitish bump on my gum just behind the lower left molar. my lesions were on both my inner cheeks. i used to drink occassionally, smoke and chew tobacco heavily. i have left them all completely for nearly 9 mths, though i drink whiskey/vodka/beer very rarely,say once in a month. what could be the possibilities of my lesions to be recurrent? why am i having that bumb? Doctor: hello, the most common causes of recurrence of leukoplakia is repeated trauma in the region, either by tobacco,brushes or from sharp edges of tooth/traumatic occlusion.. its good your lesion is curing now.. since 9 months have been passed and some white lesion still exists. i will advise you to consult an oral medicine specialist and get your occlusion checked and remove if any irregularties are noted there.. even if it doesnt regress then you can also go for excisional biopsy of the lesion which will remove the lesion completely.. you should stop all adverse habits.. antioxidents will help you in treating the lesion.. get nutrional and healthy diet full of raw fruits and veggies rich in antioxidents.. take care.."
},
{
"id": 111985,
"tgt": "What does congenital spinal cavalry stenosis due to short pericles, disc bulges and spinal canal stenosis mean?",
"src": "Patient: hi there is a suggestion of congenital spinal cavalry stenosis due to short Pericles L1-L2 mild disc bulge spinal canal and neural foramen are patent L2-L3 there is a disc bulge there is a right paracenral annular tear there is a mild facet hypertrophy there is spinal canal stenosis there is bilateral neural foraminal stenosis L3-L4. mild disc bulge mild facet hypertrophy spinal canal is mildly narrowed mild narrowing of the neural foramen L4-L5 there is a disc bulge with a small superimposed central disc extrusion there is facet hypertophy small right sinovial cyst extends into the right paraspinal soft tissue there is a spinal canal stenosis with effacement the lateral recesses and impingement of the traversing L5 nerve roots there is a bilateral neural foraminal stenosis Doctor: These findings are suggestive of disc prolapse with very little to no availability of any space for nerves. So chances of getting nerve compression are high. Patient with these finding will complain pain in both legs and some times in thigh which may or may not get relieved with rest. Smoking can also increase pain. Patient may need disc surgery if pain persist after medication ."
},
{
"id": 180832,
"tgt": "How can pain in the gums after tooth extraction be treated?",
"src": "Patient: had wisdom teeth pulled 3 weeks ago. had dry socket in both lower holes. still having alot of pain in lower left and yellow pus coming out. been \"sick\" with cold/flu like symptoms for 2 days also. is there something that I can do at home, or should I go back to oral surgeon? Doctor: Hi Dear,As per your query pain in the gums after tooth extraction seems to be due to presence of remaining infection and infected tooth sockets. I would suggest you to consult dentist once and get thorough examination done and do warm saline gargles. Apply numbing gel over the sore area for relief from pain. Avoid eating from that side. Avoid spicy, sour and acidic foods. Take proper course of antibiotics along with anti inflammatory medications. Go for quadrant cleaning.Hope your concern has been resolved.Get Well Soon.Best Wishes,Harry Maheshwari"
},
{
"id": 30268,
"tgt": "What is the treatment for epstein barr virus and depression?",
"src": "Patient: A 65 y.o. friend of the family was diagnosed with Epstein Barr about 5 years ago. She has chronic fatigue and severe depression in association (although I am not 100% convinced she is not predisposed to depression anyway). She takes 150 mg of Cymbalta, which has allowed her to resume some of her normal activity; although, her Lupus can also leave her immobilized some days. I believe that the depression inhibits her from living a more active life more than the pain and exhaustion from the EB or Lupus (she is also getting tested for Lymes Disease). What kind of doctor should she see to address the depression? I have already had the discussion with her that she will still have \"bad days\" but it would be nice if in the morning she wasn't at such an emotional deficit that her energy is sapped no matter how good she feels physically. Doctor: As your question suggest, You must take her to psychiatrist to diagnose and address the problem of depression. You can also do family counselling to elate her mood like take her for some pleasorous activity."
},
{
"id": 206861,
"tgt": "Suggest treatment for stress and depression",
"src": "Patient: Hello Dr. Sudhir Hebbar, I am Shabna Rashid. I would like to take your advise on my Niece Health Problem. Her Name is Nubla and she is 14 Yrs Old. She doesn't behave normally that is her mind is not stable. She does things without her knowledge. She is getting treated with one of the Psychiatrist and Neurologist in Cochin. We understood from them that she has got depression and she is taking medicines also for that. But in the last few days her activities are increased. Like she doesnt sleep in the night and jus keep on Dancing, She forget to chew the food before swollowing. She doesnt pass the urine completely and so on. We had heard about Nimhans Hospital in Bangalore. We would like to take her there and consult with a specialist. Can you please guide me on this and to Which Doctor I should consult her so that i can take prior appointment with that doctor. Please Advise, Regards, shabna Doctor: Hello,I think your niece has switched to manic phase from depressive phase due to use of antidepressants.You can consult Dr. Shobha Srinath in nimhans. She is child and adolescent psychiatrist and a wonderful clinician.Thanks."
},
{
"id": 90366,
"tgt": "What causes lower back pain, abdominal pain and leg pain?",
"src": "Patient: I have extreme low back pain that never stops along with pain in my abdomen & back of my legs. I have just been diagnosed with a 1.7 by 1.5 cm posterior fundal intramural fibroid. Could this be causing my symptoms? I am 43,130lbs,5'6\". They say this pain is Fibromyalgia but I don't really have any of the other sx's related to that dx. I also have several Hemangiomas, one desribed as being 2.4 cm in size & \"almost entirely replacing the vertabral body\". There is also mention of some mild Arthropathy, minimal buldging & mild stenosis? Doctor: Hi.Thanks for your query and coherent history.The uterine fibroid can not cause such symptoms as you have. It is small enough.The pain in abdomen, back and abdomen looks to be due to combination of all the problems you have noted. There are hemangiomas, arthropathy, bulgind of the disc together with the canal stenosis can cause all these symptoms. Every problem contributes in a smaller way to have this collective effect."
},
{
"id": 43878,
"tgt": "Taking Reccogon and Menocap during follicular study. What to do?",
"src": "Patient: I am from India, taking infertility treatment, two iui s failed. now going for third one. today is 6th day. taking reccogon injection on 3rd, 5th and 7th day. later on 9th day would take menocap inj. from 11th day onwards folicular study, if need ed injections there on till iui procedure. very much sad and stressed. dont know to be happy. what to do Doctor: Hi, Thanks for your query. I have read your query & I understand your concerns. These injections help in follicular growth. If you are getting follicular growth and ovulation, no need to worry. If you dont conceive by IUI in another 2 cycles of IUI please visit an IVF centre for proper evaluation and next step of treatment. I hope I answered your query. I will be available for any followup queries you have. Regards, Dr.Mahesh Koregol IVF & Infertility Specialist."
},
{
"id": 162008,
"tgt": "Suggest cure for a large bruise on the stomach",
"src": "Patient: my 7 yr old daughter has lots of large bruise type marks to her stomach sides and back, she has had them for 3 months with new ones developing each day. GP ran basic blood tests about 6 weeks ago which were nad. the marks do not look anything like a truama type bruise. Can anyone help// Doctor: Hi, I am sorry to say that skin conditions are best diagnosed and treated only after seeing them directly. May I suggest you to upload some images here so that we can have a look at the skin bruising and then guide you better. Hope I have answered your query. Let me know if I can assist you further. Regards, Dr. Sumanth Amperayani, Pediatrician, Pulmonology"
},
{
"id": 137566,
"tgt": "Will I be able to stretch and flex my hand after knuckle bruising?",
"src": "Patient: I badly bruised my knuckles over 2 weeks ago and there are no broken bones shown on the xray. The discoloration haa faded and most pain is gone, but the area is still sensitive. Will I be able to stretch and flex my hand fully as it could before the injury? Doctor: Hello,I have studied your case. This type of bruising takes some time in healing so yes you will be able to stretched it but it can take one to two months. I would suggest you to do physiotherapy and hot water fomentation. This will help you to increase recovery speed. I hope this answer will be helpful for you. Let me know if there is any other followup questions.thanks"
},
{
"id": 5992,
"tgt": "Absence of periods, negative pregnancy test. Taking fertyle and oligocare. What are the chances of preganncy?",
"src": "Patient: hi, my LMP was3-Mar-12, since then i have not had my periods. i have tested twice for pregnancy but the result was negative. Since I wanted to conceived i was stated on fertyle tablets and my husband was started on oligocare. kindly advise if there are chances of me being pregnant since the test done twice has come negative Doctor: Hello. Thanks for writing to us. Since you are overdue for your periods and your pregnancy test is negative, the delay in the periods is likely to be due to a hormonal disturbance. You need progesterone withdrawal therapy to resume your periods. I hope this information has been both informative and helpful for you. Regards, Dr. Rakhi Tayal drrakhitayal@gmail.com"
},
{
"id": 151087,
"tgt": "Tingling, numbness in right fingers. MRI scan shows shadows on the spine, transverse myolitis. Treatment?",
"src": "Patient: My daughter has had tingling and numbness in her right fingers for a couple of months Just had the results of an MRI scan which showed shadows on the spine and doctor thinks she has transverse myolitis. Are my symptoms likely to get any worse. I have read up a bit on the TM and she does not have any problems with her lower limbs or problems with bladder or bowel control Doctor: Hi ,welcome to HCM. Transverse myelitis does not cause tingling and numbness in fingers only. It will be more severe like loss of bladder bowel control, lower limb weakness. If clinically it looks like transverse myelitis then must be treated with steroids. If you want details discussion please upload MRI images of spine in our experts opinion forum. Then we can be able to come on conclusion. Regards."
},
{
"id": 215235,
"tgt": "What cause stomach cramping and pain with vomiting?",
"src": "Patient: Yes I have been in pain for the past three days my first day I had sever pain in my stomach cramping to the point I became nausea and I through up I haven t been able to really eat I through up a pinkish red and that day I had not have anything of color Doctor: Hello,It could be a gastritis secondary to severe acid reflux. As a first line management antacids like omeprazole or pantoprazole can be taken for symptomatic relief. If symptoms persist better to consult a gastroenterologist and get evaluated. An endoscopy may be required for further assessment.Hope I have answered your query. Let me know if I can assist you further. Regards, Dr. Shinas Hussain, General & Family Physician"
},
{
"id": 45898,
"tgt": "What are the eligibility and recovery period of a kidney donor?",
"src": "Patient: Sir'I'm a 58 yrs. old man having 5'4\" and 63kgs. I would like to donate my kidney and liver. So far, by HIS grace, I'm healthy. No sugar, no pressure. Only complaint is that I'm a chronic allergic for cold. I'll take inhaler for Asthma maximum once in a month. Am i eligible for that. If so how long to recover to lead a normal life? Is both can be done at a stretch? Is my age is a hindrance for the receiver?Kindly answer me.Thank You Doctor: Hello and Welcome to \u2018Ask A Doctor\u2019 service. I have reviewed your query and here is my advice. Your age is not suitable for a donor as the age advances the chances for rejection will be more and whole procedure will be spoiled. The criteria may vary from country to country and you can enquire about the rules in your country. Hope I have answered your query. Let me know if I can assist you further."
},
{
"id": 159281,
"tgt": "Diagnosed with endometriosis. Have simple simple hyperplasia with glandular crowding. Meaning?",
"src": "Patient: I was diagnosed with endometriosis when I was 27 and am now 44. I had an endometrial biopsy done in 2010 because my lining was 12.6mm. It came back normal. In 2011 I had an ablation done, and the pathology report states that I have simple hyperplasia with glandular crowding with an increased gland to stroma ratio, but no diffuse hyperplasia or cytologic atypia was identified. I had two fibroids then, the largest measuring 44mm x 40mm x 31 mm. This fibroid is now showing that it is calcified based on a recent ultrasound. I was told the other fibroid is changing, and that I have a new one on outside of my uterus. I am having bleeding issues and pain in my lower back and buttocks. My doctor was going to perform a hysterectomy and decided not to without an explanation. I am seeing a new doctor next week. Should I be concerned about cancer? Doctor: Hi, simple hyperplasia not associated with cancer. most important report is your specimen did not show any atypia that means you should not be worry about cancer. management of fibroid solely depends on your gynaecologist.as they are quite large in size better to go for hysterectomy.consult gynaecologist."
},
{
"id": 117127,
"tgt": "What causes the increase of WBC count?",
"src": "Patient: Hi I am 34 years old and my wbc count increases suddenly over night with fatigue, headache, nausea and fever. The wbc count shoots as high as 29K and it comes back to normal the next day without the help of any medicine. This has happened alm,ost thrice in 3 month period. Doctors tested for all kinds of infection and have not been able to find any source of infection. Why is this happening to me. My heart beat also increases to 120 + during these times Doctor: Hi, Welcome to Health Care Magic. I read your history and your are worried about episodic rise in WBC count. I must tell you the causes which are responsible for increase in WBC are following-infection... can be viral, bacterial or fungal-inflammation anywhere in the body due to foreign insult-allergies of any type-leukemia-other bonemarrow disordersI would suggest you get the following tests done -Complete blood count with differential count-Liver and renal function test-blood & urine culture and sensitvity-chest xray consult your local physician to formulate future plan based on these tests. If everything comes normal you can go for bone marrow biopsy too.Hope I have answered your query. If you do not have any clarifications, you can close the discussion and rate the answer. Wish you good health."
},
{
"id": 113402,
"tgt": "22 year old with back pain. On ibuprofen. Do I have pulmonary embolism?",
"src": "Patient: Hi, I m a 22 year old female. I m having some back pain that started a little over a week ago. At first I thought it was a knot in my muscle, then it didn t go away. I ve been taking 4 ibeproferin every six hours and still the pain wont go away. It was really bad when I woke up this morning and could barely get up. I woke up several times coughing and that caused and even more sharp pain. I looked up on Web MD and put my symptoms in and it pointed most likely to pulmonary embolism . I m scared. Doctor: Hello, in this question you have not mentioned which area of your back is hurting, is it upper or lower back. PE can mimic back ache (in the upper part of spine), but the pain is more on the front than on the back, there is cough with sputum and often blood can also accompany. The embolus usually arises from the lower limb vessels and after a period of immobilization or major surgery. also your age dosent fit into the risk factors for PE. But other than that there are several other causes of back ache with cough. you should consult a doctor and get your tests done. It can be disc disease or acute spasm also. do not worry. take care."
},
{
"id": 120588,
"tgt": "Suggest remedy for throbbing and itching in left palm",
"src": "Patient: I have a wierd feeling in my left palm. Close to the area between the thumb and pointer finger (sorry, I m forgetting all the correct terms) It s almost throbing at times, and then I almost get some sort of itch and not only do I just scratch it, I end up biting and pulling with my teeth to get rid of the feeling. I think it happened before, and got better later. Never lasted more than a day. But this is probably the third time it happened, so I m wondering if there s a bigger issue? Doctor: Hello,I read carefully your query and understand your concern. The symptoms can be related to many medical conditions.One of the common causes is dermatitis of the palm.I suggest using an antihistamines such as Cetirizine 10 mg daily.I also suggest using a steroid cream for local application. If the symptoms continue, see your physician and get evaluated. Hope my answer was helpful.If you have further queries feel free to contact me again.Kind regards! Dr.Dorina Gurabardhi General &Family Physician"
},
{
"id": 92371,
"tgt": "What could be the reason for abdomen pain on left side?",
"src": "Patient: I'm having pain in my abdomen on the left side. I have seen a doctor & it is not a hernia. I felt a sharp pain at work when I was doing some heavy lifting 2 weeks ago but the pain is at times unbearable. I'm not sure what to do at this point. The doctor provided me with some pain medication but it isn't really working. Doctor: Hi...it could be a ureteric calculi or a ligament injury following your weight lifting... get a usg abdomen done to rule out calculi...Dr. Ashish Verma"
},
{
"id": 54442,
"tgt": "What are the side effects peg placed in the small intestine?",
"src": "Patient: My daughter just had a j peg placed in her small intestine. She has been throwing up green bile every day since the surgery. She has no gall bladder. They stopped the tube feeding but she still throws up over 2 liters of at one time. This has been going on for over a week. Doctor: Brief Answer:She needs to undergo further examinations .Detailed Answer:Hello,Thank you for using HCM.Dear , I understand your concern , and i will try to clarify what's happening. PEG tube is not placed in her small intestine, it is passed into the stomach through the abdominal wall, to provide a means of feeding when oral intake is not adequate. The fact that from the PEG tube comes out bile, means that there is something which does not allow it to go in the natural route, from the bile duct (which comes from liver and pancreas) to duodenum (which comes directly after the stomach). So, due to any obstacle ,mechanical one, or due to the lack of adequate bowel movements, the bile turns back to the stomach and goes out from the PEG tube. I would suggest you to see a gastroenterologist to advice you further examinations, such as abdominal ecography and fibrogastroscopy . A series of blood exams are also needed , such as bilirubin levels in blood and inflammation indeces.Hope this is helpful.Let me know if you need more clarifications. Otherwise please close and rate the answer.Kind Regards, Dr. Juarda"
},
{
"id": 164095,
"tgt": "What causes sudden fever and skin infections?",
"src": "Patient: My son is sick in bed with a fever. Began during the night, went to dr this morning. Checked for flu -negative. His skin issues are in other locations but I don t know where, but he told me about his hands. He was in Japan for a month for work. He has been home for 4 days. They have a daughter who is 21months old. Just thinking Doctor: Hi...I feel by what you quote he should be having a - Hand Foot Mouth disease. This is one viral illness among all other exanthemas which can cause fever followed by rash over palms and soles. It is a self-limiting disorder and itching can be really worrisome. I suggest you use any over the counter antihistamine if you have one with you now. You can use Hydroxyzine at 1-2mg/kg/dose (Maximum 10mg) every 6th to 8th hourly for 7 days.This can even cause some peeling of skin in the next 4-6 weeks and do not worry about it.BUT PLEASE BEAR IN MIND THAT SKIN CONDITIONS ARE ALWAYS BEST DIAGNOSED AND TREATED AFTER SEEING THEM DIRECTLY. SO I SUGGEST YOU TO UPLOAD SOME IMAGES ON THIS WEBSITE AND GET BACK TO US SO THAT WE CAN HELP YOU SCIENTIFICALLY.Regards - Dr. Sumanth"
},
{
"id": 88030,
"tgt": "What causes abdominal pains and nausea?",
"src": "Patient: I am wondering what could be wrong with my wife. She is having upper abdominal pain, in the center, and its fluctuating. One minute its ok, and the next its not. She is also experiancing nausea and its a lot worse when the pain comes. She is also having upper back pain. A few hours ago we did some heavy lifting of some furniture and she said her back hurt a little bit then, but thats normal for her. She then had some chips and salsa, which I shared with her, and then she had a turkey and mayo sandwich. 20 min later all of this started. She has taken pepto, Levsin, Zofran, and nothing is helping. Any ideas? BP and Pulse are normal and lung sounds are all clear. Doctor: Hi.Thanks for your query and an elucidate history. I have read your wife's history and understood Her problem.The causes of the pain in upper abdomen in a female patient. Nausea is common with any sort of problem of the abdomen is just an indication that something is not alright in the abdomen.The character of the pain which you explained is called spasmodic pains. This occurs in case of intestinal obstruction. or some problem in the initial (proximal) part of small intestine. She also have a back-pain , you say,.I would advise in such a patient the following:Standing X-ray of the abdomen.EnteroclysisUltrasonography of the whole abdomen to start with and confirmatory CT scan of the abdomen if something is positive. Relevant blood, urine and stool tests after a clinical examination The treatment will depend upon the final diagnosis."
},
{
"id": 100459,
"tgt": "How can pet allergy with sneezing be alternatively treated?",
"src": "Patient: I'm going to a friend's house tonight and spending the night. I love going there because she's one of my best friends, and we always have a great time, but her house triggers my allergies even when I've taken my medicine. She has 2 cats, 2 dogs, and a guinea pig in her house. I thought taking my medicine (Allegra of some sort) an hour or so before I go would make it fine, but it's always really bad; I'm sneezing like crazy and my nose is runny, and I cough a lot. I'm not allergic to cats or dogs or guinea pigs per se, but they do bother me a little but not as much as they do at her house. I'm only supposed to take it once every 12 hours, but my mom is a nurse and she said it wouldn't kill me to take it again when I was at this girl's house before. So I took it a few hours after I had the first time, and my allergies were fine after that, but I was SOOO sleepy, I was the only one to sleep that night because a few other friends were there too. I want to stay up and have fun, but I don't want to be miserable. What should I do about it? Doctor: Hello.Thank you for asking at HCM.I went through your history and would like to make following suggestions for you:1. Yes, antihistamines (commonly used for allergies) can cause sedation. Were I treating you, I would suggest you to take combination of fexofenadine and montelukast which will be more effective than fexofenadine (Allegra) alone. You can take a single dose approx 1-2 hours before going to your friend's house.2. If this does not help you, I would suggest you to keep a intranasal spray like azelastine with you, you could use it when you develop symptoms. As it is a local spray, it causes least sedation.Hope above suggestions will be helpful to you.Should you have any further query, please feel free to ask at HCM.Wish you the best of the health ahead.Thank you & Regards."
},
{
"id": 181164,
"tgt": "What causes excessive saliva production in the mouth?",
"src": "Patient: Hi there I've been having excess saliva production from a monthIt's been coming and going Actually , from the internet , I checked that it. Can be due to bad oral hygiene I had pretty bad oral hygiene I had gone 2-3 weeks without brushing and never brushed before sleepingAfter the things started, I've been taking extreme good care from 2 days now and brushing every 2 hoursI feel like it's reduced but it's not yet completely gone How much time will it take to go as I have exams coming up and it'll be a distractions to my concentration Doctor: Hi Thank you for your query on Health Care MagicI understood your concern. I appreciate that you have already noticed that poor oral hygiene caused problems like hypersalivation.There are other causes for hypersalivation that includes injury or inflammation of salivary glands,hot or spicy food diet, rabies, pellagra, side effects of drugs etc.I also appreciate that now you follow brushing regularly. But brushing in an improper method can harm your teeth and gums if done every 2 hours. I suggest you:1.To brush twice daily(morning and night)2.Avoid sticky foods3.Brush in a proper brushing technique 4.Visit a dental clinic and know about the brushing technique for you.5.Use Dental floss and mouthwash6.Poor oral hygiene also causes gum disease. So you need to get the professional scaling and treatment.7.Consult a Dentist and undergo an oral check up and get the right diagnosis and treatment.Dont worry , Get treatment. Avoid distractions, get refreshed and do exams well. Hope this information helpThank you"
},
{
"id": 145945,
"tgt": "Suggest treatment for spinal stenosis",
"src": "Patient: I have L4-5 posterior disc bulge with increasing central spinal stenosis and the AP canal diameter is about 5 mm. I see a neurosurgeon soon. Is this 5mm diameter not good? I also have severe facet arthropathy at this level and a thick walled synovial cyst. I have bulging discs and moderate facet arthropathy at 4 more levels. I am most concerned about the stenosis. What can I expect the surgeon to do? Doctor: Hi,Thanks for writing in.You seem to be having a significant spinal canal stenosis. There is a passage in the spine running the neck to the lower back through which the spinal cord passes. In the lower back the spinal cord but the nerves continue to exit through small foramina on both sides of the spine. Disc bulges cause narrowing of the canal and pinching of the nerves leading to severe lower back pain. A spinal canal dimension of 5 mm in AP direction is small and can be the cause for significant pain and discomfort. A visit to the neurosurgeon will help in knowing the amount of discomfort you are in and if surgical correction of the bulging disc at L 4 - L 5 level will improve your pain."
},
{
"id": 86373,
"tgt": "Suggest treatment for severe lower abdominal pain",
"src": "Patient: Hello i have a uncomfortable pain not severe in my lower right abdomen i have been two the hospital the past two days first for acid reflux and then urine infection and am worried about this, i keep thinking appendicitis but have been telt its severe pain Doctor: For severe lower abdominal pain you can take some antibiotic preferably norflox or metrogyl after an expert opinion along with cyclopam or meftal-spaz for severe pain and cramps in abdomen. You can take omez-d or rekool-d empty stomach at-least one hour before meal if acid reflux is present."
},
{
"id": 186369,
"tgt": "Suggest remedy for dental problem",
"src": "Patient: my question is about dental work ihave a problem to the point i will not go any where i have not smiled in a very long time i am disabled and still waiting on a settlement i do not have dental insurance i know this not good for my heath my 2 front teeth are almost gone i am only 54 i need help my name is judie wright tell what i should do Doctor: Hello, Thank you for consulting with HCM.It looks that your teeth are severely damaged due to caries and infection.In This condition you have to visit a dentist, as the problem can not be solved on your own or home remedies.Better visit a good dentist and get the treatment done.Hope it will help you."
},
{
"id": 63937,
"tgt": "What causes red lump on arm?",
"src": "Patient: Hello, my girlfriend is 31 years old, 5 feet 6 inches, and 125 lbs. She discovered a lump on her arm two days ago. The lump is underneath her skin, toward the inner bicep and just above her elbow. At first it was painful when pressed and has become more tender since. Over the past two days it seems to have grown and is now about 1/3 the size of a golf ball. It is red and does not move. Any idea what this might be? Thank you. Doctor: Hi,Dear,Good Evening from INDIA.Thanks for the query to HCM.I went through all the details of your query. on the details given by your query,In my opinion you suffer mostly from-though the information given in your query is less-on the facts there in-mostly you suffer from-lump with a bug bite mostly,as its red and non-movable.Treatment suggested would be-a-Tab NSAIDs x 5 days b-Cold compressesc-Tab Antihistaminic drugd-Antibiotics if need beconsult ER doctor for the script of drugs.Hope this would resolve your query.Welcome for any more query to HCM and ME,in this regard.Wishing you fast recovery.Write review ASAP with strong recommendations for HCM and for ME,for the benefit of other new and old patients.Have A Good Day.With Regards,Dr.Savaskar M.N."
},
{
"id": 94701,
"tgt": "Taking medication regularly for PCOS. Missed taking a pill. Had unprotected intercourse. Missed periods. Am I pregnant?",
"src": "Patient: hi, i m a PCOS patient. i have been taking my pills correctly for a span of 5 months. but in the 5th month i missed 1 pill and took other pills correctly. after having my 21st pill i had unprotected sex with my bf on that night only, i.e. 5th nov. and took ipill on 7th nov evening. i haven t got my periods yet. i m worried. m i pregnant? and my pcos has been cured or not? plz help. thanks in advance Doctor: Hello, I would be happy to help you with your question. Pregnancy is very unlikely. Just the one missed pill does not increase the chance of pregnancy that much. In addition, the use of emergency contraception was appropriate. It is not uncommon for emergency contraception to delay your period - this is the most likely explanation. Just to be safe, take a pregnancy test now and repeat in 2 weeks. I hope that this helps. Please contact me through the PREMIUM SERVICE if you would like more detail or a more timely response - Dr. Timothy Raichle"
},
{
"id": 182039,
"tgt": "What causes red swollen roof of mouth and difficulty eating?",
"src": "Patient: I have what looks like, by sight, an extremely red and swollen roof of my mouth, including the gums and around the teeth. I can barely contend with the discomfort or can I eat well either. It's come to my not eating anything except drinking, and soft foods. I have been swishing with either warm salt water or a product on the market called Colgate Peroxyl Mouth Sore Rinse. Awhile back I tried Hydrogen Peroxide with an equal amount of water and rinsed with that and I did notice my mouth became worse, so I'm possibly allergic to Hydrogen Peroxide. I do have a partial which is very hard to wear, although I can't hardly not wear it because it's in the front on top. The tooth next to the partial on the left side does bother me periodically - it's totally chipped off in the back of it. I just can't believe all of this and then starting yesterday I have a tooth on the bottom, last one on the left, starting to hurt for the first time. I take Ibuproferin, Aspirin, Tylenol, whatever, for the 2 teeth that are bothersome. I'd like to have teeth implants and I've seen the commercial for \"Clear Choice\" but haven't contacted them yet. I am going to the dentist on Tuesday, 11-18-2014. Right now I'm very concerned over the inside of my top mouth and gums - it looks very bad and of course, hurts very bad. Thank you. Wanda Sandlin Doctor: Dear Wanda The oral mucosa (the soft tissues inside your mouth ) are a very flagile and undergon rapid changes over years . if you are still continuing to wear your old partials (since they are your front teeth and you have no choice as of now ),i strongly recommend to discontinue the usage of the same as it might be a misfit for you now and the swelling and pain is most probably dus to the undue pressure the denture has exerted on your delicate mucosa . i also advice you to undergo deep cleaning procedures for the health of your gums .peroxide is a mild irritant for a certain ratio of population so i suggest you to discontinue the usage . please stick to warm water salines /betadiene gargles . good luck with your appointment ,hope i have provided you with some points to ponder upon .take care ."
},
{
"id": 68920,
"tgt": "Does cyst causes lump ?",
"src": "Patient: I have a sebaceous cyst on the back of my head. Around the same time I noticed a small knot like lump on the side of my neck about 2-3 inches under my left ear. It is painful. Do you think that the lump is from the cyst? Or does it need to be checked for cancer? Doctor: Thank you for query.The lump behind the ear may be due to lymphnode enlargement due to infection in the scalp or ears.It may be any infection.So you may need oral antibiotics and examination for looking the site of infection so please consult doctor."
},
{
"id": 75028,
"tgt": "Is it safe to take cifloc 500 along with prednisone and spiractin?",
"src": "Patient: I am a emphysema patient for quite a long time now and use a variety of medication . Once a year I end up in hospital with an excerbation. Then I m given aminophyllin and sollycortef (I probably misspelt these) on a drip and I always respond quite good. I use symbicord 320, theophyllin 300 twice daily, onbrez((started about 10 days ago with these) before that I was on Forvent , I also use an Venteze inhaler, and take 10mg of prednisone in the morning. 10 mg Monte-air , 25mg Spiractin, 2.5mg of bilk or, h5mg of Zytomil, As I have glaucoma I also have Alphagan as well as Ocuprost eye drops which tend to lessen the work of the lung medications, is that correct. After going for a flu shot, I have this clear sputum/mucus in my airways which makes it difficult to breath, I don t cough, just force it out and spit it out every now and then. I have a watery, runny nose, and SAfrica does not have Vibrocil Nasal available until the end of June and if this does not stop I know from experience that this will go down to my lungs and than I ll get this thick, colored mucus. Lastly, I have cancer in remission after a mastectomy in March 2010 and am on Tamoplex 30mg, as Kessar and Neophedan are not available in South Africa at the moment. My QUESTION IS: when taking all these medications, is it safe to take Cifloc 500 twice daily? It seems asof the American FDA is not happy with these pills for the side-effects. Further I firstly have to know much I have to pay for you reply, please. Thank you. Mona J. v. Rensburg Doctor: Hi and welcome in HCMI am sorry for your situationYes you can take safely ciflox with steroids and spractin,no problem,do not worryDr.Jolanda"
},
{
"id": 140321,
"tgt": "What causes a twinge in the leg while lying down?",
"src": "Patient: I am 56 woman and have been working out and doing yoga for about a year and have loset 70 lbs. 2 nights ago I started to get this twinge in the right side in one spot. It only happens when I lye down. I have sciatica but I didnt think that this could be the problem. Whatever it is is seems to be near the surface muscle. Doctor: Hello, Your symptoms are suggestive of benign fasciculations (BFS). There is no need to worry, as it is not a serious condition. It would stop on its own. It commonly occurs due to muscle exertion. Hope I have answered your query. Let me know if I can assist you further. Take care Regards, Dr Sudhir Kumar, Neurologist"
},
{
"id": 213508,
"tgt": "Severe mood swings, anxiety, anger and insomnia. Had postpartum depression. How can I cure this naturally?",
"src": "Patient: Hi, I m 33 yrs old female and I m experiencing severe mood swings, anxiety , anger and insomnia . 18 mons ago I gave birth to my second child and experienced postpartum depression , which lasted 7-8 months after the birth. Although I do not think I have the postpartum depression any longer, I do not feel like myself anymore. I don t think my hormones ever fully went back to normal. I would like to know what your professional opinion is regarding this matter and what I can do naturally to heal myself. I do not believe in pharmaceutical drugs if not completely necessary. Thank you for your response Doctor: You may be experiencing the mania of an undiagnosed bipolar disorder. If your mood is more irritable and/or elevated, rather than depressed, this could well be the case. Bipolar disorder can manifest first as a major depressive disorder and the ,ania can appear later in time. Do consult a mental health professional about it if you have concerns. I hope you beginning to improve!"
},
{
"id": 57938,
"tgt": "Why is the pain under the right rib cage having a history of liver infection and syphilis?",
"src": "Patient: Good day sir.I'm a 22years old I had acne since I was 16, I had a liver infection last year... I think that's caused because of alcohol and syphilis, I survived all of it but now I have a pain just under the right rib, sometimes in my back and sometimes a bit far from the rib, same side just down.. So I don't know if it's my liver again or its my stomach. Pls tell me what is should do about it. I spoke to my doctor about he gave me pain killers for the pain and the doxitab for my acne. The acne dosnt Completely go away I just getting better but as soon as I stop it Comes back Doctor: Hi and welcome to HCM.It can be both liver stomach but also gallbladder stones so these are first things to check. you should do liver ultrasound and check gallbaldder stones and liver enzymes. it will reveal is there some serious patology which I doubt. Till you do this you should change dietary habits and avoid alcohol completely.Thanks for the query. Wish you good health. Regards"
},
{
"id": 161136,
"tgt": "What causes dark green and frothy stools in kid?",
"src": "Patient: my 2yr old daughter has had bad bowels last 3 days, yesterday was dark green and frothy and today was dark burgundy red in colour and loose, quite worried, otherwise she is well though apart from being overly fussy with eating last 2 days? Pls can you advise what this might be? Doctor: HiI 've gone through your query. I can understand your anxiety. Frothy Green stools often results from 1. Green leafy vegetables 2.iron supplements 3. food coloring 4.Breast feeding (sensitivity to some of the mother's diet e.g : cow's milk) 5. Bacterial or parasitic Infections. 6. Medications that turns stool green Based on the dehydration status, start your baby on * Electral powder or sugar 8tsp in water * Do not stop breastfeeding * Give rice cunjee, dal water * Probiotics* if not settling, go for antibiotics for 5-7days * If the episodes are increasing , consult your child specialist Hope I have answered your query. Let me know if I can assist you further. Hope for speedy recovery of your baby. Regards, Dr. Yogapriya V, General and family physician."
},
{
"id": 6600,
"tgt": "Why I dint get periods from past 3 months ?",
"src": "Patient: hi i m 32yrs female married, diagnoised with PCO, doctor treated me with Kremson for 6 months and then moved to evecare and sutane for a month.After completing month course i never got my periods nor i conceived, its almost 3 months. Pl suggest Doctor: hi there, you need to get once again checked by a gynecologist who after proper investigation will suggest you the remedy right now your question is incomplete in its investigation parts like,ultra sound,blood profile, thanks"
},
{
"id": 40435,
"tgt": "Can I take anti malaria tablets while trying to conceive?",
"src": "Patient: We are trying for a baby but will be going to bali visiting some of the countries that may be at risk of malaria. Are there any anti malaria tablets I can take during this time, as I know some aren't suitable if trying for a baby. Some forums state that you shouldn't try whilst taking the tablets. Please can you advise. Doctor: Hello,Welcome to HCM,Avoid travelling to areas of the world where there is a risk of malaria.Pregnant women have an increased risk of developing severe malaria. There is also a greater risk that both the mother and their baby will experience complications if they get malaria.If you are unable to postpone or cancel your trip to an area where there is a malaria risk,Mefloquine is the drug of choice during pregnancy.Thank you."
},
{
"id": 17539,
"tgt": "What causes fast heart rate during aerobic exercise while having COPD?",
"src": "Patient: Have COPD, 40 yr old female 5 2 exsmoker 20+ years 20 min aerobic exercise and heart rate climbs to 230-215 for about 10 mins after exercise and is very slow to decline, shaky and feel as though I could pass out with weird burping sensations too, overheated not any sweat to speak of ever either. Can weight lift but when cardio is involved I get the above result and feel as though I will die. Doctor: Hello, It can happen as a side effect of bronchodilators. However, it is better to get an ECG to rule out possible causes like cardiac arrhythmia. Consult a cardiologist and he will direct you accordingly. Hope I have answered your query. Let me know if I can assist you further. Regards, Dr. Shinas Hussain, General & Family Physician"
},
{
"id": 6580,
"tgt": "Is there a possibility to conceive because of this watery sperm ?",
"src": "Patient: hello doc, I tried to have unprotected sex with my girlfriend and i feel a little spam(kamarash) like water in top of my penis. then i tried to push it but we abort sex(unprotected sex). i just push it about not more than 5 cm and get it out. but did not come the final spam. is there any possibility to pregnancy with that water like spam? normally i fell it when i was very excited and must before having sex? my final Question is Can my girl friend get pregnant ? Doctor: hi, welcome to healthcare magic that water like semen also contains sperms,if your girl friend is in her ovulatory dates then there is a chance for pregnancy"
},
{
"id": 110626,
"tgt": "How to cure pain in the tailbone region after an injury?",
"src": "Patient: Hello, My wife fell down the stairs and after slowly got up acusing a lot of pain in the tailbone she became pale, felt nausea and she almost fainted. After washing her face with cold water she managed to get in bed and she only feels the taibone pain now. Should we get medical attantion or it is ok to wait till tomorrow to see how she feels in the morning? Doctor: Give her a soft cushion underneath the buttock during sitting posture and also give paracetamol tablet.In the morning she may sit in a tumbler of warm water forhot fomentation for 20 mins See a doctor if it doesn't helpf"
},
{
"id": 77046,
"tgt": "What causes pain in chest and right arm after taking HCG drops?",
"src": "Patient: Hi, I was just wondering if you've ever heard of anyone getting a slight pain on the right side of their chest and right arm after taking HCG drops? I feel the pain when I breathe in and move my arm. I'm 30, female, good health, a little overweight. Doctor: Thanks for your question on Healthcare Magic. I can understand your concern. No I have not heard about this correlation. No need to worry for this kind of chest and arm pain. This is not associated with HCG drops you have taken. Your this pain is mostly musculoskeletal. And it is just co incidence that this pain is felt after HCG drops. There is no scientific connection between these two. Hope I have solved your query. I will be happy to help you further. Wish you good health. Thanks."
},
{
"id": 85963,
"tgt": "What does pain in the lower abdomen in a diverticulitis patient indicate?",
"src": "Patient: I am having pain in my lower abdomen. I believe it could be from an internal hemorroid but I am not sure. The pain is right in the front. I am a 71year old woman. It also have diverticulosis but I don\u2019t think it is diverticulitis. I have had that before. I know the bladder,lower colon and uterus are all kind of right there so I\u2019m not sure. Doctor: **Thanks for contacting with your health concern1. As there was a history of diverticulosis and now having pain abdomen as the symptom, thus following should be your approach:i. At present, it might be a 'gastrointestinal' symptoms [diverticulitis, celiac disease thus you should contact either a general surgeon, gastroenterologist or Gynecologist and if not sure consult your GP who after reviewing your case history and physical examination will refer you to specialist concerned.PS. The most common diagnoses of lower abdominal pain relate to uterine, gonadal, renal, and bladder complications, [less common causes like primary dermatologic and musculoskeletal problems as well as referred pain and neuropathies related to underlying vertebral and spinal cord complications can also occur], so examination is mandatory for further workup and management.2. since the pain is unexplainable thus it is wise to check with your doctor since persistent or chronic pain should be examined to rule out any serious underlying causes.PS. A doctor can diagnose the cause of the pain and develop a treatment plan that will address the underlying cause of lower abdominal pain"
},
{
"id": 93360,
"tgt": "Underwent Tubectomy operation. Have stomach pain. Suggestions?",
"src": "Patient: i underwent tubectomy operation on 1st may 13 and gettng severe stomach pain starting some times whole (starting from left back then thighs, leg, hand and whole left partfrom left and sometimes right (same as mentioned to right side also mainly at belly). This pain will be continous till 2 to 3 days and automatically off for one day. pl. suggest. this pain started nearly 3 yrs back. but now after tubectomy its severe and continous. Doctor: Hi, you underwent tubectomy operation, on 1st may, and getting severe stomach pain, 3 years back you got this pain and again getting now. Your description suggests that there is urinary tract infection 3 years back, now it has recurred again. Consult an urologist for treatment, Thank you."
},
{
"id": 47009,
"tgt": "Recommend the best solution for kidney problems in elderly",
"src": "Patient: i have my father here in kuwait, and he is suffering for kidney problem for how many years and today we got the updated result that 20 percent of his both kidney are only functioning. what can u suggest the best solution? do we require a kidney transplant or another alternative? Please all in family now is in shock and chaos.Please advice Doctor: hithanks for posting in HCMI have understood your concernyour father is in the situation of need for renal replacement therapy1.hemodialysis - done in the hospital2.peritoneal dialysis-can be done at home3. renal transplantif his age is less than 65yrs and cardiac condition is good he can go for transplant whether cadaveric or live .if he is not for for transplant, be needs life long dialysis weekly twice or thrice depending on his symptoms.hope this helps youany further questions please let me knowthanks"
},
{
"id": 100487,
"tgt": "Is prednisone safe for allergy?",
"src": "Patient: I was given prednisone for my allergies - 40 mg per day for five days. When I have used prednisone in the past, I usually was given lower doses and had to taper off. I am a bit concerned because of the high dose level and sudden stop at the end of five days. Thanks you... Doctor: Hi. Oral prednisone is very useful in allergies as it ceases the inflammatory response to the allergen almost instantaneously.It is necessary to taper steroids only if they have been taken for more than 14 days since that is the duration required by the body to start getting dependent on an external source of steroid (Hypothalamic pituitary axis). A 5 day course of steroid can be stopped abruptly without any foreseeable complications. Do keep in mind that high doses of prednisone may affect your vision so do see your physician who started you on these if you develop blurring of vision or changes in colour. High steroid doses may also make you vulnerable to infections so try to avoid anything that exposes you to a source of infection.A 5 day course of steroids is usually nothing to be concerned about. Follow up with your physician for the remaining management of your allergies.Do let me know if you have any other questions."
},
{
"id": 217006,
"tgt": "Suggest treatment for hip and arm pain after an injury",
"src": "Patient: I fell off the steps of my camper, twisted around, my right arm fell and scraped on the corner of the picnic table, then my hip hit the corer of the picnic table bench and I fell onto the ground. My arm was scraped and brused. My hip had about a half inch hole with the indention of the corer of the picnic table on the skin and also scraped and brused. I have a hard lump about 3 long where the hole is healing with a scab but it hurts when I breath and I have a lot of trouble sleeping because of pain. It has been about 2 weeks since I fell and I am 65 years old. It also hurts a lot when I cough or breathe in deeply. The pain is pretty much all of the time and sometimes a lot worse. The pain goes from under my bra to just below my waist on the right side. I thought it would go away by now. Is this something I should be concerned about to go to the doctor maybe for x rays or something? I am afraid maybe I hurt something inside. Doctor: thank you for opinion .from your history it looks like you might have chest bone injury as well.please check it with an X-ray. for the rib fracture.treatment is liberal pain killer.light breathing exercise .check with thoracic surgeon if no improvement."
},
{
"id": 106252,
"tgt": "What should i do for asthma and wheezing ?",
"src": "Patient: my son has asma and hes weezing a lot and when i went to give him hes therapy the tube of the therapy machine broke im concern on what should i do? Doctor: Hi, welcome to health care magic forum, Homeopathy is the best way to get rid of asthma problem, without any side effects as inhalers will contain steroids and have numerous side effects on human body. well here are few tips for you. Know what triggers your Asthma attacks. Don't smoke. Asthma or no Asthma. Declare war against dust mites & cockroaches. Avoid smoke, especially cigarette smoke, vapours and chemical fumes. Vacuum and dust your home regularly. Stay indoors during the humid season or the change of seasons especially when the pollen count is high. Do not use fans in closed and dusty places. Cover mattresses and pillows with plastic covers. Wash the bedding in hot water every week. Do not use a vaporizer or humidifier unless it is thoroughly cleaned. Do not have close contact with pets. Try to prevent yourself from dust while driving, You should always cover your mouth, and nose if possible, with a scarf. Take Homeopathic treatment for complete cure of Asthma & stay healthy. For More Information Mail Us At :- info@drkackerhomeopathy.com or visit us at www.drkackerhomeopathy.com"
},
{
"id": 155484,
"tgt": "Is prostate cancer, related with enlarged prostate and calcifications?",
"src": "Patient: Im 62 years of a age, and on a routine lab exam PSA level was 6.5, Digital rectal exam was normal, echography shows enlarged prostate with calcifications and all the rest was normal.. FREE PSA/TOTAL PSA was about 22.5 and according to the lab this result is normal, urine analysis shows RBC levels of 20 - 22 and WBC 2, can we know from these results if it s a gland hypertrophy or it s prostate cancer? thank you Doctor: Thanks for your question on HCM. In my opinion you should not worry much about cancer. As calcification is 100% benign. It is not at all seen in malignancy. So you should not worry much about cancer. Your other lab reports are also normal for cancer. You are having BPH (benign prostatic enlargement) which is causing red blood cells in urine.So better to consult urosurgeon and get done removal of prostate. This will cure rbc in urine. And no need to worry about cancer."
},
{
"id": 195855,
"tgt": "Suggest remedy for pimples in genitals",
"src": "Patient: hi Dr I have some issues with my male organ, which i noticed long time ago. initial i had a like a pimple on the side of my male organ and it become a wound. after few months it healed itself but i had a healed wound mark on the area. I thought its was just pimple but i am not sure whether it is something serous or not. Right now after two year i feel that i need to get a second opinion because it seem that the size of the mark some what increasing. I did not have sex for two and half years and i got that issue after three months of my last sex. i am attaching a photo of current look of my male part for your consultation Doctor: Hello,For pimples on penis, you can take tablet Amox Clav with tablet Dolo. Apply Neosporin H powder, Lactocalmine lotion. Keep it clean and wear loose dress.The causes are improper hygiene, STDs (Sexually Transmitted Diseases), contact dermatitis, allergic reaction, bacterial infections. Treatment depends upon underlying conditions.I will suggest you to follow up with detailed history or consult with urologist for detailed examination.Hope I have answered your query. Let me know if I can assist you further."
},
{
"id": 64759,
"tgt": "Suggest remedy for lumps on cervix",
"src": "Patient: I am 17 and my mom recently died from oral cancer caused by Human papillomavirus also know as HPV. I recently found a lump in side my vagina on what i BELIEVE is my cervix. Its between the size of a nickel and a dime in diameter and about half a cm in height, I also have pain in my ovaries. like im on my period even when im not. My periods have lately became longer and more painful and i have them ever 21 days. For the past 3 months ive been very lethargic, I always want to sleep and have no energy and i feel sick all the time even when im not. Im scared to death it is cancer can anyone shed some light on what it could be ? Doctor: Hi,DEar,thanks for your query to HCM.I studied your query in depth.1-In My opinion the lump on your cervix,is mostly due to-Cervical Cyst-with polymenorrhagia.2-I would advise gynaec consultation and FNAC / or excision biopsy of the cervical lump-3-Treatmen is by -Surgical Exicion and biopsy to rule out cancer-as you have fear of death , as your mom died of oral cancer.3-Hope this would solve your query.4-Wellcome for any more questions on new or this subject also...Have A Good Day..!!"
},
{
"id": 184100,
"tgt": "What causes swelling in between gums after root canal?",
"src": "Patient: i had a tooth that had a root canal done and then the next day i had to get it pulled, i have a huge swelling between my gum and check, it started at the injection site and has expanded the Dr put me on steriods, it didnt get rid of it, the lump is bigger and it is hurting again, the dentist said he thought it was fluid build up Doctor: Thanks for your query, I have gone through your query.The swellingin between gums after root canal treatment could be because of the trauma cause to the gums while instrumentation or it can be because of the gum infection. Consult a oral physician and get it ruled out. Mean while you can start with a course of antibiotics like amoxicillin 500mg and metronidazole 400mg tid for 5 days (if you are not allergic). Do saline gargling. i hope my answer will help you, take care."
},
{
"id": 18308,
"tgt": "What causes recurrent burning sensation and tightness in the legs?",
"src": "Patient: Hi! About one week and a half ago, I had my vascular surgeon do an exploratory on my deep veins to see if I had any blockage. I have had swelling feet and ankles for years. But, after he did the procedure (first in the groin area then back of the knees, he found no blockage. Unfortunately, I have been experiencing awful leg symptoms, such as a burning feeling, tightness, throbbing, etc. in my calf down to my ankles, now feet. I went back to him and they checked for blood clots - none, and I don t seem to have an infection. I took today some ibuprofen to see if it may help for inflammation and tonight the symptoms returned and I think I am getting the symptoms in the other leg as well. I cannot sleep - there is no position that does not disturb my legs. Please see if you can help me as I do not want to go to the emergency room if I can help it. Thank you, Lynne Albrecht Doctor: Hello and Welcome to \u2018Ask A Doctor\u2019 service. I have reviewed your query and here is my advice. After going through your medical details I understand your concern and I would like to tell you that Hopefully this information will answer your query query. Kind Regards Dr Bhanu Partap"
},
{
"id": 30096,
"tgt": "Suggest remedy for a cyst",
"src": "Patient: I have an intermittent smell of smoke...when no smoke is around. I do have a draining cyst next to my adeonid remnants (am 48 yo F). Not sure if it is a thornwaldt cyst Could the phantom smell be coming from that? Also just put on Fe for low level, had to decrease levothyroxine from 175mcg to 150mcg.. and just stopped 2mos of coumadin for a basilic thrombus. Hx of allergiesl rhinitis and sinus surgery x 2 for sinusitis Doctor: Good afternoon, I am sorry you are experiencing these symptoms. To discover the treatment for the cyst, I highly suggest you first discover the makeup and what is in the cyst as to treat with the correct medication, dose and or intervention. A cyst is defined as a abnormal membranous sac or cavity containing fluid. Often times it contains infection, and the treatment is merely drainage and antibiotics. Other times however, the contents may be more serious or even benign. There could be simple fluid, drainage being the treatment as well, or a neoplasm, cancerous or otherwise, in which treatment is more complicated and very dependent on many factors. I highly suggest you obtain a formal workup from a physician in which you can obtain a biopsy, study of the histology of and or workup discovering exactly what is in that cyst. Afterwards, you and your physician can determine a proper course of treatment. I hope this discussion helps you towards a proper course of action and you find it helpful :)"
},
{
"id": 101276,
"tgt": "Suggest medication for nasal allergy leading to running nose",
"src": "Patient: I have a problem of ENT problem of allergy, im badly effected, nose allergy is like flu, nose is raining, on regular basis, i have taken, Baydal, Rigix, and also Fexet 120 still no relief, kindly suggest proper medicine...thanksM.Ali Karachi Pakistan Doctor: Hello M. Ali,Thank you asking at HCM.I went through your history. From your brief history, I would first think of allergic rhinitis.Were I treating you, I would suggest you to take montelukast and levocetirizine daily. However, if this does not control your symptoms adequately, intranasal corticosteroids would be recommended.Along with above, I would suggest you regular nasal cavity irrigation with saline nasal spray 3-4 times a day. Please avoid exposure to smoke, dust and air pollution as much as possible.Personally, I would also suggest you to consult an Allergist-Immunologist who will perform allergy testing and suggest you avoidance measures and allergen immunotherapy, if appropriate.Hope this will be helpful to you.Wish you the best of the health.Regards."
},
{
"id": 1162,
"tgt": "Is there a way to get pregnant after a recent depo shot?",
"src": "Patient: Hello, my husband and I just decided we wanted a baby and we don't want to wait. The only problem is I just got my Depo at the end of February and I have been on it for quite some time. Is ther any way to get pregnant on it or an tips to help increase the chances? I heard if you take another birth control they cancel each other out, is this true as well? thank you. Doctor: Hi there, I have understood your concern. I will suggest you the best possible treatment options. As you might be aware that whenever depo injection is given en it works for the duration for which iy is supposed to work for. Usually, depo I jectoon of medroxy progesterone hormone 150 mg is taken intramuscular, then it works and remains I. The body for 3 months. As the medicine can not be taken out of body, it's contraceptive effect will last for 3 months from the date of injection. So I will suggest you to wait for 3 months from your last injection for your fertility to return. Till then please start on Folic acid, Vitamin B 12 and Omega 3 supplements . This helps to prevent many problems during pregnancy and delivery. .I hope this answer helps you .Thanks. Dr. Purushottam Neurgaonkar"
},
{
"id": 192295,
"tgt": "Can unprotected sex cause pus like penile discharge and itching?",
"src": "Patient: Hi, my penis has been leaking a white pus like substance and I get this itchhing sensation where it leaks out at. I am 19 years old and I had unprotected sex about 5 weeks ago. Its very little pus but it still worries me. I found out about 2 days ago. I'm not on any medication. Can someone help? Doctor: Hi, From your complaints it seems to be a case of UTI. Any h/o burning urination ? Any h/o perineal heaviness ? Any h/o suprapubic pain ? Go for urine r/e & urine c/s so that i can suggest you better. Hope I have answered your query. Let me know if I can assist you further. Take care Regards, Dr. Ashish Kumar Khandelwal"
},
{
"id": 12506,
"tgt": "Diagnosed with psoriasis. Flared up after using medication. Causing stiffness in both knees. Psoriasis arthritis?",
"src": "Patient: hi. i have recently been diagnosed with psoriasis . This was done with a visit to my local GP a couple of weeks ago. At the time i had a couple of small areas on my right hand and wrist which were causing irritation. I also had a couple of areas on my left forearm which were causing me concern. Two of my fingers on my right hand were also affected. Unbeknown to me, my knees also had psoriasis. Since using the medication , it seems that this has only encouraged the psoriasis to flare up even more in the areas i have mentioned (apart from the knees). I now have two large red areas on my right hand and wrist, although it does not look as raw as it did previously. Although it still looks as though it is spreading slowly, the skin is peeling away around the edge of the affected areas. On my left forearm, the areas look as though they are slowly scabbing over. The two fingers on my right hand are also red in colour, but peeling at the edge. As this has only developed in the last month or so, does this sound like the psoriasis is clearing? Also, i do suffer from quite a bit of stiffness in both of my knees. I wasn t aware that you could suffer from psoriasis arthritis! Doctor: YES IT IS COMMON WITH PSORIASIS TO INVOLVE BONES LEADING TO PSORIATIC ARTHRITISPSORIASIS IS NOW CATAGORISED AS CHRONIC INFLAMATION WITH CERTAIN DAILY FOODS GET BLOOD SERUM TESTS FOR SPECIFIC ANTIBODIES FOR MILK WHEAT POTATO CHANA NUTS EGG RICE AND COMMON FOODS ELIMINATE FOR CURETILL THEN TAKE TAB METHOTREXTATE 2.5 MG TWICE A WEEK ANTI ALERGIC TAB ONCE A DAYAPPLY SALYSILIC ACID OINTMENT ON AREA OF SKIN INVOLVEDNON TOXIC SOAPS SHAMPOO AND OILSAN TAKE TAB ALLOPURINOL BD FOR 5 DAYS OFF AND ON FOR PAIN IN JOINTS AND BONES"
},
{
"id": 40980,
"tgt": "Suggest treatment for infertility problems",
"src": "Patient: Hello, I am 27 and i am 230lbs. I am 5 4 . My boyfriend and I are trying to get pregnant. We made sure to have sex 1 and 2 days before suspected ovulation (which was supposed to be this past monday). I noticed that wednesday and thursday my urine was bright yellow and also a bit cloudy. Today it was still bright yellow and it was clear (not cloudy). Normally, it is clear color or pale yellow, no cloudyness. I am not having unsual urine frequency, or pain or any discomfort (it feels completly normal, but is just different color). I drink iced coffee and I drink water, I havent had any soda or other juice beverages. I do not take multivitamins, I havent changed my diet (egg whites, sausage patty and american cheese on a wrap for breakfast). small soup (either chicken noodle or beef noodle) for lunch with a side of veggies, and usually a meat, veggie and starch for dinner. Also, please note that I have PCOS but do not have diabeties. Could the urine change be pregnancy, or perhaps something else? Doctor: Hallow Dear,From your history, I have not understood whether you have missed your period or not. Of course, though the first cardinal symptom of pregnancy is missing the period, since you are a case of PCOS, your menses could be irregular. Hence missed period cannot be relied upon in you . However, the urinary changes that you have explained are not the signs of pregnancy. Still if you have got doubt, please get your urine tested for pregnancy. Cloudy appearance of your urine could be due to crystals in the urine. Please get your urine examined to find the cause. If you are keen on becoming pregnant, you have to get your PCOS treated. In this condition, the eggs get stuck up in the ovaries. The doctors will give you medicines to release eggs; however you also have to cooperate in the management of PCOS by bringing down your weight. I hope this helps you. Dr. Nishikant Shrotri"
},
{
"id": 8698,
"tgt": "Open pores and acne scars due to waxing facial hair. Effective and low cost treatment?",
"src": "Patient: hello sir, I am trapti. I am very fair in color and getting married in november. i am having facial hair and do wax on that..due to this i am having open pores and acne scars ..One of my beautician has advise me to take treatments and given home care , but i am worried it would work or not and its too costly..she has given me products and treatment of cheryls s Doctor: Hello. Waxing facial hair cannot give rise to acne scars or open pores. Acne scars occur following acne. No Cheryl products can treat acne scars. Scars have to be treated with a procedure like lasers or microneedling. for open pores application of retin A cream applied once at night will be helpful. Application of the cream on affected areas only at night should be done. Please let me know if you have any queries."
},
{
"id": 178837,
"tgt": "What causes intermittent fever in a child?",
"src": "Patient: My granddaughter is 15 months old and has intermittent temps ranging from. 101-103 sometimes as high as 104. My daughter generally ends up in the ER with no answers. I m wondering if she needs a CBC blood test to check for some kind of lymphoma or other blood disorderes? This has been going on for. Couple months now? Any thoughts? Doctor: I think she is suffering from recurrent influenza infections which are common in this age group and causes repeated episodes of fever. To rule out other possibilities get a blood test done for CBC, CRP Chest Xray and urine routine examination. If all the reports comes out normal get a INfluenza vaccine yearly which will help the child."
},
{
"id": 205312,
"tgt": "What causes severe nightmare?",
"src": "Patient: I just had the worst nightmare of my life. I have been taking clindamycin HCl 300mg tid and norco 10mg as neede f For an access I had drained. I also quit smoking cigarettes 2 months ago. Is any of this the reason for the bad dreams? I ve had them 3 nights running and been on the clindamycin since Tuesday. Doctor: hi, no any of this causes nightmare. it might be due to quiting smoking, so either u take tab.clonotril(0.5) once a day in night for sleep or use relaxation technique for getting a better sleep."
},
{
"id": 128956,
"tgt": "How can a sprained wrist be treated?",
"src": "Patient: Hello, so i sprained my wrist about 5 weeks ago. I ve been taking it easy on it since then. I still have slight pain when I put stress on it (i.e. push up). But its not unbearable pain. At this phase in my healing process will it reduce pain and speed up healing if I were to stretch it or do light exercises that incorporate it? Thanks Doctor: Hello,Thank you for using Healthcaremagic.I read your question and understood your concern.I think light exercises will speed up the process of healing so you should do it.Dr. Selmani"
},
{
"id": 117052,
"tgt": "What could cause high cholesterol, low BP, pain in legs and eyeballs with shivers and headache?",
"src": "Patient: Pain in legs, headache, pain upperside of eyeballs, shivers, temperature up the first day, still pain in both legs now and headache and pain in eyeballs but no sign of cold or nose or throat problems. I m 54 years old, high cholesterol (7.4) but low blood pressure. All other bloodtest were ok few weeks ago ( except cholesterol) Doctor: Hi,Thanks for asking.Based on your query, my opinion is as follows.1. High cholesterol is due to altered fat metabolism due to obesity, low activity and high fat intake. 2. Low blood pressure is possibly due to chronic dehydration and also possible postural hypotension.3. Variations can occur within a few weeks. Get yourself on low fat diet, along with anti-lipid medications. Also moderate exercise with weight reduction will be very helpful.Hope it helps.Any further queries, happy to help again."
},
{
"id": 119159,
"tgt": "Severely anemic, feeling weak, racing heart, cough and burning sensation. Reasons ?",
"src": "Patient: hi I am severely anemic my last level was told to be 12 and they are in process of setting up an infusment for me for this, but today I feel as if something as change i feel reall out of it, sick and weak, if I exert any energy I feel like im gonna pass out my heart start racing and i have to sit down, now im also exprencing a cough that feels like its only coming from the right side which has a burning sensation. I need to also to let you know that I had a PE a year ago and was being treated but recently my hematologist took me off bc of the low iron Doctor: Dear Ericar4988, Thanks for your questions. From your statement, I can gather that your heart may have started experiencing more strain from the severity of the anemia, than it can handle...it may be failing. So, you need to urgently check yourself into a competent hospital (with specialist Hematologist or Cardiologist service) for immediate treatment (which would most likely include blood transfusions). Additionally, the cough may even be a sign of a repeat PE, which can co-exist with heart failure. Whichever it is, you need to be in hospital. Best wishes."
},
{
"id": 167475,
"tgt": "What causes high carnosine and anserine levels?",
"src": "Patient: high carnosine, and anserine levels? My son, 3 yrs. He is devlopmentally delayed and Autism has been cansidered. He had genetic testing and a full blood work up. the results were no chromosome problems. glad for that. However his. anserine was 377 and carnosine at 93. these were said to be high. A blood plasma test has been ran for inborn metabolism disorders. The nurse made me feel as if all this is diet related but I really don t feel we eat tones of meat. Are their any disorders that match those two levels being high? and are they treatable? if it is diet related? shat can I do as a mother to ensure he gets more of the other food groups? A side note. Years ago I was told I could not donate plasma because of a protien being missing. they thought it a mistake and told me to come back in a year. I never did. Could it have been right and related to my sons problem with speech delays. I too was delayed as a child for unknown reasons. sorry for the long question but I m worried. thanks for advice. Doctor: High Carnosine and anserine levels in blood have not been co-related with any definite health disorder, as yet. There is no scientific proof as yet to make these levels point to a diagnosis. Diet does not have much to do with it.It is claimed by proponents of alternative medicine that Carnosine and anserine is helpful in treating autism, though there is no such proven clinical evidence and we do not recommend its use. Have you, by any chance, been giving your child any such supplements containing Carnosine and anserine? This may have led to an increased level.Regarding diet, a child with autism can be given any diet, which contains adequate amounts of all nutrients, just like any other child.The finding of a 'protein missing in your blood' should be looked into, but it most likely has nothin to do with Carnosine and anserine levels, which are not part of the proteins that are looked for when testing a blood sample! So dont worry."
},
{
"id": 24932,
"tgt": "What are the symptoms of orthostatic hypotension?",
"src": "Patient: hello i am 17 years old, male, about 140 pounds, skinny, and have this weird thing going on, ok so whenever i stand up my heart beats hard and at a low rate but not life threatening rate maybe like to 55 or 60 then it speeds up really quickly, when im having this thing, i feel kind of out of breath but i can breath fine but it feels like i have to breath a little harder, and rarely it happens whenever im sitting down. also when im taking a hot shower my heart races. i tryed to look this up all i found simular to this was orthostatic hypotension, because it feels like im going to black out, but it onlys happens for about 5 seconds then my heart races goes to normal after that please can anyone tell me what do i have, also i have not fainted or blacked out before, my family and i have gone through 10 tests on my heart and went to a cardiologist to take a cardiogram, to take pictures of my heart, and they said i was fine, but dont know what going on right now, so if anyone can help me with this or if i can get treatment for this? Doctor: Hello!Thank you for asking on HCM!I carefully passed through your question and would explain that your symptoms could be related to orthostatic intolerance. In orthostatic hypotension there is a decrease in your blood pressure during standing up. I would recommend measuring your blood pressure in sitting and then in standing up position. A decrease more than 20mm Hg when standing up from sitting position could indicate possible orthostatic intolerance. Another disorder, related to orthstatic intolerance is Postural tachycardia. In such case heart rate increase more than 30 bpm while standing up from sitting position. A head Up Tilt test is necessary to rule in/out these possible disorder. Hope you will find this answer helpful!Kind regards, Dr. Iliri"
},
{
"id": 128222,
"tgt": "What causes muscle pain and stiffness in the body?",
"src": "Patient: I have muscle pain and stiffness throughout my entire body. Especially my legs and hands. The stiffness and pain increases at night and gets better when I get up and move. When I kneel down it is virtually impossible to get up. Sometimes it even seems the leg muscles are not adhering to the brain signals telling it to get up. Doctor: Dear patient you may be suffering from early arthritis like rheumatoid arthritis. Morning stiffness in joints and muscle pain are the symptoms. Diagnosis can be confirmed with serum level of RA factor. Please get it done from pathology centre nearby you . Start cap indomethacin 25 mg thrice a day with tab rantac 150.mg twice a day before meals. If report is abnormal please consult rheumatologist."
},
{
"id": 169703,
"tgt": "What causes fever to shoot up every few hours?",
"src": "Patient: I have a 1yr old...he has been running temperature for the past 2 days...its not high fever though 100 degree F. I am giving him crocin syrup (5ml) that is equivalent to tylenol. The fever comes down after the medicine is given, but shoots up again after 3-4 hrs. Please let me know how many days I can continue this? Doctor: Hi Dear,Welcome to HCM.Understanding your concern. As per your query you child have symptoms of fever to shoot up every few hours which is due to impaired and reduced immunity of child. Need not to worry about it. Visit pediatrician to find out the cause of fever after thorough investigation. If required request your pediatrician for any lab tests which may be required and start treatment as per that. You can give Crocin as well every 4-6 hrly. Give him paracetamol as prescribed by doctor and along with that do tepid sponging in between depending on intensity of fever. Hope your concern has been resolved.Get Well Soon.Best wishes,Dr. Harry Maheshwari"
},
{
"id": 147139,
"tgt": "Is it safe to work with L4/L5 slip?",
"src": "Patient: Hello, I had a spinal fusion in May 2008. (L4/L5) Six months later during physical therapy something happened during a stretch. I have been in pain since. I pain has been increasing. I had an MRI in July that shows an L4/L5 slip. I have not worked since 2007. I was offered a job as a special education teacher. Is it safe to accept this position? I do not want to end up in a wheelchair. Doctor: Hi, As you had lumbar lecithesis at L4-5 level post trauma during physiotherapy so if the injury is stable which can be decided upon MRI Finding & clinical examination it can be managed conservatively by1) Avoiding bending forward 2) regular physiotherapy 3) warm compression4) Vitamins (methylcobalamine, folic acid etc )5) analgesic , proton pump inhibitor , muscle relaxant as and when required 6) LS Corset belt if required Otherwise if unstable then operative fixation may require . So in my opinion Visit an orthopaedic surgeon preferably spine centre for further management and opinion ( If you have stable injury and can perform the job if spinal mobility can be imobilised during working hours ).Hope this will help. For any queries feel free to ask THANKS N REGARDS"
},
{
"id": 79258,
"tgt": "How to cure breathing issues during weather changes after jaundice and chickengunea?",
"src": "Patient: I'd jaundice and chickengunea two years back. I have been having breathing issues since then. They come up and aggravate in weather change or a week before my periods. It makes it really difficult for me to breathe or perform any activity. It's accompanied by a mild yet persistent headache. Doctor: thanks for your questionin my opinion possible causes can be-bronchial asthma- bronchitisyou need to consult a Pulmonologist who can examine you and investigate regarding the respiratory issue in my patients I usually conduct a spirometry and a peak expiratory flow rate which can lead to a diagnosis and if it's an obstructive cause some inhaled drugs may be started as per your pulmonologists opinionthanksfeel free to ask more questions"
},
{
"id": 63264,
"tgt": "How can chest and back lumps be treated at reasonable cost?",
"src": "Patient: Dr i am male 29 years old. i have bumps like pimples around the chest,my back and now its spreading in my face also. I consulted a Dr and he said its fungal infection. and he wrote creams, tablets which was quiet costly. is that require such costly medicines? Doctor: Hi, dearI have gone through your question. I can understand your concern. You have fungal infection. You should take antifungal antibiotics. You can take oral tablets also. If you feel that it is costly then you can change the company and take generic drugs. Consult your doctor and take treatment accordingly. Hope I have answered your question, if you have doubt then I will be happy to answer. Thanks for using health care magic. Wish you a very good health."
},
{
"id": 60919,
"tgt": "What does tinnitus along with a lump behind the right ear indicate?",
"src": "Patient: Few day ago, there was ringing in my right ear and a tiny painless hard lump behind my right ear . My doctor suggested me it is all because of throat infection and mucus. They prescribed me nazel sprey nose drop steam and acetcysteine ambroxol HCL tablet. Now my throats feeling better. Ringing in ear has gone away. But I am worried after little lump. It is still here in same position size moreover today I notice a stiff and tender pain in my right neck and shoulders . I am worried Doctor: Hello and Welcome to \u2018Ask A Doctor\u2019 service.I have reviewed your query and here is my advice.The hard lump behind ear is reactionary lymph node enlargement, which gets slow resolution once the acute phase of infection is well managed.Hope I have answered your query. Let me know if I can assist you further.Regards,Dr. Bhagyesh V. Patel"
},
{
"id": 77997,
"tgt": "What is the meaning of homogeneous opacity seen in right hemithorax?",
"src": "Patient: What is the meaning of Homogeneous opacity seen in righthemithorax of RT costophonic angle and obscuration of RT hemidiaphragmn .Doctor says this means Lungs are filled with water (Approx 1.5 liter)Kindly let me know is this curable with medicine?Is Operation required. Is this causes serious problem now or later on. Doctor: HelloYour findings may suggest huge amount of fluid in right pleural cavity.This condition is called as gross right pleural effusion.You may need aspiration of fluid immediately. Inter coastal drainage may be placed in right side of thorax.You may need proper clinical evaluation and investigations.Investigations include routine hemogram,random blood sugar,pleural fluid (routine and biochemical analysis).Culture and sensitivity of pleural fluid should also be done.This is generally a manageable condition and it can be managed by medicines.You may need proper evaluation for tuberculosis etc.Get well soon.Take CareDr.Indu Bhushan"
},
{
"id": 57703,
"tgt": "Suggest the diet for increased AST and ALT levels",
"src": "Patient: Hi Team, I have check my LFT , in that my AST My lipid profile tryglecrides raised to 300 so it raised , right now i have caught and cold , caught in the troat past 6 mnth my throat always dry water feel , even my 2 ear always water inside fell while movement , I dont have thyroid both hypo,hyper. I have vitamin d deficiency right nw came to 13 before i have taken 8 week course during that time it was 50 . after completed 1 month i have checked it was 23 thn i have taken support 2000k per day but its reduce to 13 now . i have check with endocrine he told my complaint is not related to endocrine , check with gastro so i have checked with them they have checked my blood test both virus hb hbc etc its all negative . so they ask to take endoscopi to check is there any water retension is happening because i feel my whole body flabby ness as well water sence . i my head , ear, adn throat and body and stomach are getting soft thinner . I went to ayurvedic doc he told that my body bad colostrol not there so body getting softer and flabby and thinner . so i think he is telling the write thing , because i have checked uro , neuro told that sensory neuropathy but i dont have sugar ...my EMG normal ....so please tell to reduce AST my previous opening doc told its may be prosta condition caused by Enzyme protease deficiency. The water retention, decreased strengths and pain in the muscles without any protein loss is a way of presentation in Protease deficiency. 1.So if deficiency presist what to do , and how to over come it , my total body feel like water packet when i touch my muscle so soft like water packet . suggest for LFT issue as well thiss issue . Doctor: unsatturated oils like palm,sunflower oilsis the gold standard.donot consume groundnut,sweets,& nonveg diet. alcohol , smoking banned.start rosuvastatin+finofibrate daily till next estimation of your lipid profiles."
},
{
"id": 107248,
"tgt": "Can eating orange cause lower back pain and fatigue?",
"src": "Patient: My 41 yr old husband is feeling lower back pain on one side and he s very fatigued, and says he doesnt feel well. I dont think its an emergency; but he has had painful kidney stones before-where his back suddenly hurt so bad- he collapsed in the mall and was ambulenced to hosp. It s 7 p.m. and he s already asleep. Hes been feeling this lower back pain for a few days; but he only mentions it about once /day. It could be kidney stones agin. ugh. He has been eating an orange everyday for last several months. And I just read that could cause them again. hmmm.. Doctor: Answer: Hello,Thank you for using Healthcaremagic.I read your question and understood your concern.I think you may have a recurrent kidney stone so get ultrasound examination or ct scan to confirm it. oranges are citrus fruits and prevent stone formation so you may continue eating them.Dr. Sandeep patil"
},
{
"id": 45758,
"tgt": "What can cause hydronephrosis in kidney?",
"src": "Patient: sir recenly my kidny stone operation is done of ledt kidny for the 14mm stone ,it is removed and one tube is inserted in between kidny to pu junction......... after that a 6 mm stone is present in lower side of the kidny doctor tell this stone remove by tablets in few days ......then after 20 days doctor check the stone by using sonography....it is remove but the report shows the mild hydronephrosis in left kidny.....please tell me causes of that and tretment ...........iam confused it generate because of pipe.....but in this i have small pain if larg physical activity please help me ............ Doctor: Hi, Hydronephrosis is nothing but fluid collection in kidneys. It is quite common after surgical procedures and in your case, it is mild and will settle in a couple of weeks. No treatment is required. Wishing you good health. Hope I have answered your query. Let me know if I can assist you further."
},
{
"id": 120521,
"tgt": "What causes shooting pain in feet after ligament tear treatment?",
"src": "Patient: I am a female 41 years old. My weight is 79 kgs. About 15 months back I had a ligament tear on my right foot. Had the treatment with an ortho for 2 months. after about 3-4 months, i had a pain shooting from the right foot till the buttoks. this happens mostly when I sit on the floor. close to six months i am working out in a gym (1 hour a day, 5 days a week.) i do not have problems while working out. But this shooting pain has increased recently and i suffer almost daily. i am unable to sleep during night. request your suggestion on this. Thanks Doctor: Hello,It does not looks that your present problem is due to old ligament tear.Looking at your details it looks that you may have compression of nerves at your back, this may be related to your exercises at gym.At present you should following to have relief:- Avoid weight lifting or heavy exercises- Avoid forward bending at your back- Take a good analgesic with muscle relaxant activity- Apply warm fomentation over back and leg.Hope I have answered your question. Let me know if I can assist you further. Regards, Dr. Mukesh Tiwari, Orthopedic Surgeon"
},
{
"id": 30236,
"tgt": "How to cure swelling and redness on ankle due to welts?",
"src": "Patient: I have been getting terrible reactions to bug bites, outside. Never had any issues, started this year. We have mosquitos, wasp, blackflies, ants and horse flies here. Not sure exactly is biting me. I get a normal looking bug welt, then couple hours later it turns red, hives up, swells and hurts. I got a bite above my ankle two days ago and my ankle is still red, swollen and sore. Swelling is from top of foot up past my ankle. Got one on my calf and it hurts so bad if you press on it, feels like your getting pricked, now turning red and aching. I do get dizzy and have been taking benedryl when it gets bad. Is there a reason all of a sudden I am having such bad reactions? Doctor: hello,This is kind of how the immune system reacts in some people. The first few times you get an exposure the reaction is small then in subsequent exposures there is a much larger reaction. The things you can do in this situation are to avoid the bites or see an allergist for getting testing and treatment.Regards"
},
{
"id": 176490,
"tgt": "Suggest treatment for blocked nose in infant",
"src": "Patient: Hi Doctor, My 9 months old infant baby girl has cold and nose blockage. She is breathing by nose and is facing difficulty while feeding & sleeping. We visited 2 different doctors from different hospitals, they checked and mentioned that chest is clear and prescribed Fenistil Drops & Ocean Spray. One of them also prescribed Sinecod (Butamirate Citrate) while other advised to stop it. its since 2 days but the baby is still uncomfortable, could you please let us know if it will take more time to cure and suggest any other practice which we can follow at home to ease her discomfort. Doctor: i suggest otrivin pediatric nasal drops 1 drop in each nostril 4 times a days along with oral drops atarax 1 ml twice a day . both for 3 days . stop other medications"
},
{
"id": 215434,
"tgt": "Suggest remedy for precordial catch syndrome",
"src": "Patient: Im 19 and have a history of precordial catch syndrome but it usually lasts only minutes and it comes whenever i last a few days without excercising i run 2 to 3 times a week i rested 4 days due to leg injury and it suddenly came and now i ve had it for 2 hours and it just hurts when i exhale what should i do? its never lasted this long Doctor: Hello, Cannot say in your particular case, but generally, Aspirin-like drugs that work on inflammation would be the first thing to try. This includes naproxen, maybe ibuprofen at high doses, and several others. Hope I have answered your query. Let me know if I can assist you further. Take care Regards, Dr Matt Wachsman, Addiction Medicine Specialist"
},
{
"id": 96943,
"tgt": "Is the swelling in the shin and ankle related to back injury?",
"src": "Patient: I injured my lower back about 12 years ago. I have a herinated disk, over the years I have noticed other things going on. I have over active bladder and now recently I have swollen in my shins and ankle when I push on it it leaves a indent, Are all these related from my back injury? Doctor: What we call it as pitting edema.Nothing but fluid accumulation.It is unrelated to back injury.Causes of it include-Renal failureCongestive cardiac failureSevere anemia and hypoprotenemiaPlease rule out these causes"
},
{
"id": 3222,
"tgt": "How to confirm pregnancy after missing a period?",
"src": "Patient: My last period was June 15 and I have not gotten yet, I was wondering if I am pregnant and make a dr. apt. or if I should wait to see if I will get it . Is it too early to take an ultra sound? For a week I feel pressure in my lower stomach even if no one is touching it Doctor: Hello dearI understand your concernYou will go for home pregnancy test when your period will delay by the 8-10 days.Blood HCG test is more accurate test and give positive result even 10 day after the sexual intercourse.USG will detect the pregnancy at 5-6 week of pregnancy.Meanwhile avoid stress, take healthy diet, drink plenty of water and maintain proper pelvic hygiene.Hope this may help youBest regardsDr. Sagar"
},
{
"id": 44166,
"tgt": "Semen Analysis shows Volume 2 Non Motile 70%. After taking Paternia Volume 3.5 Non Motile 50 %. How long to take ?",
"src": "Patient: hi My First Semen Analysis Report shows Volume 2 Non Motile 70% Pus Cell 2-3 after that doctor adviced me Paternia whcih i took for 2 months Reult was Volume 3.5 Non Motile 50 % Pus Cell 1-2 BUt then Doctor Shifted to Some other Hospital so i didn t took his advice i tried another doctor who adviced me to semento Capsules then which i took 1 moths reports shows Volume 3 Non Motile 60 % Pus Cell 1-2 Please advce what to do should i carry on with Paternia Doctor: hello thank you for writing to HCM The tablets suggested by both docotrs are good and help many men to improve the sperm parameters. But if the counts show higher number of non motile sperms after 3 months of medication you need further investigations with a urologist to rule out any other problems Dr Nandita Thakkar"
},
{
"id": 210793,
"tgt": "What is the remedy for low back pain with anxiety and dizziness?",
"src": "Patient: low back pain , anxiety , dizziness , very hard to concentrate tired all of the time , very bad tingling in both feet , burning dry eyes need drop in early morning ,prescribed by doctor i000mg each day, diagnosed by a doctor with naturopaty ect_ Doctor: Hello,Thanks for choosing health care magic for posting your query.I have gone through your question in detail and I can understand what you are going through.Neuropathy can present in this way with burning sensation over the limbs. You have not quoted the name of the medicine that you doctor has prescribed but one of the common causes of neuropathy is B12 deficinecy. It needs to be replenished with alternate days IM injections. Also get a thyroid levels tested as hypothyroidism can also lead to this kind of symptoms. You can get a TSH level done. Further if nothing comes from the investigations then depression can also lead to this kind of symptoms. But all said the b12 treatment and thyroid investigations should be done before.Hope I am able to answer your concerns.If you have any further query, I would be glad to help you.In future if you wish to contact me directly, you can use the below mentioned link:bit.ly/dr-srikanth-reddy\u00a0\u00a0\u00a0\u00a0\u00a0\u00a0\u00a0\u00a0\u00a0\u00a0\u00a0\u00a0\u00a0\u00a0\u00a0\u00a0\u00a0\u00a0\u00a0\u00a0\u00a0\u00a0\u00a0\u00a0\u00a0\u00a0\u00a0\u00a0\u00a0\u00a0\u00a0\u00a0\u00a0\u00a0\u00a0\u00a0\u00a0\u00a0\u00a0\u00a0"
},
{
"id": 213338,
"tgt": "Sudden angry, impatient, high heart rate. Calms down easily. Why is it happening?",
"src": "Patient: Hello. I was speaking to someone on the phone earlier (part of my job) and they were not nice and I noticed myself becoming impatient and angry. I tried to remain calm, but I started to get this weird feeling in my head like it was going to explode and my heart started to race. When I ended the call and calmed down, those feelings went away and my heart rate was back to normal. Can you please explain possibly why I had been feeling that way? Thanks! Doctor: Hi, Low frustration tolerance and anxiety may be the cause of your symptoms. You need evaluation for the similar problem in other life circumstances. Other reason may be disliking for the nature of job. You may seek psychiatrist help if symptoms are severe and affecting your job/social life. Apart from that proper rest/sleep, stoppage of substance use (if any), aerobic exercise, healthy diet and supportive counseling will help you. I hope this information has been both informative and helpful for you. Wish you Good Health. Regards, Dr. Ashish Mittal www.99doctor.com"
},
{
"id": 197702,
"tgt": "What causes red and rough skin on the scrotum?",
"src": "Patient: hello.. im an indian boy... and my scrotum is having some peeling skin type things on it.. and when i touch it, it really pains.. it has also turned red and rough... when i try to peel of the skin, it burns and it is very painful... what should i do, please help me... Doctor: HiWelcome.I have gone through your query.An examination is required to exactly say what it is. Do not peel the skin, it increases the chance of secondary infection. It might be due to friction. Avoid wearing tight under garment and use loose cotton underwear. It is better to get yourself examined by doctor to know what it is.Hope this helps.Take care"
},
{
"id": 29856,
"tgt": "What is the cure for a constantly running nose?",
"src": "Patient: My husband is 83 and his nose runs constantly .... ven dripping before before he can retrieve his handkerchief. What can he do to help prevent this embarrassing condition? Doctor: Hello,Thanks for using healthcaremagic.I have gone through your query and I can understand your concerns.Running nose or rhinorrhea can be due to many causes,most often it is allergic in origin.It can be due to viral infection also.Some other rare conditions like CSF rhinorrhea have to be ruled out before commencing the treatment.As a first line therapy you can try the following:1.Vapor rubs and nasal decongestants.2.Antihistamines like levocetrezine.3.Steam inhalation.4.Nasal sprays(olapatidine containing sprays are highly effective)5.Avoid dust and cold.If symptoms persist despite all these consult an ENT specialist and get a detailed evaluation done.Hope this information helps.Regards.Dr.Shinas Hussain"
},
{
"id": 99443,
"tgt": "What causes itchy bumps on hands/legs and swollen lymph nodes in groin?",
"src": "Patient: I am allergic to something/s I consistantly break out into these tiny itchy bumps on my hands as well as my legs often. My lymphnodes in groin are swollen and I also have a spot near the vulva that I get tinty bumps/blisters its not too itchy ... Are there different allergies or diseases that cause these symptoms other than an std Doctor: Helloenlarged inguinal lymph node along with bumps on clitoris could be related to STI or infection of genitourinary tractin my opinion you should consult a gynecologist for examination and a battery of investigation like blood for hemoglobin CBC VDRL test urine test for culture and sensitivity and a swab test if discharge is there you should be worried but not too much you will be alright after a course of antibiotics which your doctor knows have plenty of fluids and maintain good hygienei hope you will find my response helpful and informative please don't hesitate to ask for further clarification if need beThanks for using our serviceswish you good health"
},
{
"id": 132579,
"tgt": "Reason for severe pain in the legs?",
"src": "Patient: I have very severe quadricep pain in both of my legs. I had a mild cramp wednesday night, then Thursday when I woke up both of my quads were in severe pain. It s very painful to sit down, then stand back up. Friday when I woke up the pain was worse then today it was even worse when I woke up. It hurts to walk, going up and down stairs and moving after sitting even for short periods. There was no direct incident that caused this, no lifting, no moving, no fall. The only thing I can compare it to is when I used to lift heavy weights, it feels like the next day recovery but so much more painful. Doctor: Hello--if this is muscle pain and cramping then it isn't a disc problem in the low backIt could be a low potassium, high calcium, low magnesium or low vitamin D blood level and you need to see a doctor for an exam and blood work"
},
{
"id": 146843,
"tgt": "Does sciatic nerve pain cause fluttering in foot?",
"src": "Patient: I have been diagnosed with L4/L5 prolapsed disc, and have associated sciatic nerve pain, but recently (6 weeks from onset) have been having fluttering in my foot (lasts a few seconds, mostly when standing). Is this something to mention to my doctor, or just another symptom? Doctor: Hi, welcome to our site. I am Dr Saumya Mittal, MD.Read your query. That is a very significant question and i appreciate your problem. I will try my best to answer your queryThere is a strong possibility that the condition may be due to the nerve compression. The nerve damage may cause the symptoms, especially when the muscles are getting affected secondary to the nerve damage.Fluttering may be due to a number of reasons, the most common being fatigue. The fatigue may occur due to any cause. The muscles are basically stressed out. And so when they have to keep contracting and acting, there may be a feeling of flutter.Fluid and electrolyte balance is crucial to smooth acting of the muscles.As depicted, there are too many causes of flutter. So I would suggest that you may mention it to your neurologist the next time around when you meet him/herMeanwhile, test a FBS/TSHI hope this helps you. Inform the reports mentioned above/if any other so I can be of help further. I have given you the answer to the maximum considering the information provided. The results of the tests could further enhance my answer to you.Please do understand that some details could be extracted from a detailed history and examination.Please feel free to ask another query. I would be glad to help you. Looking forward to your return query with the details asked so that I can help you further. (If the answer has helped you, please indicate this)Best of luck.Dr Mittal.MBBS, MD (Internal Medicine), CC (Diabetes Mellitus), DNB (Neurology)Consultant Physician and DiabetologistJS HospitalEx Apollo Hospital, DelhiEx Kailash Hospital, Noida"
},
{
"id": 163495,
"tgt": "How can bed-wetting be treated in a child?",
"src": "Patient: My son (9) has had trouble with nighttime dryness. We have tried to limit liquids, waking him up in the middle of the night to void, and now have an alarm he wears but nothing works. I am getting worried that he will not outgrow this issue as his pediatrician says. I am wondering if I should seek a UA to see if there is an infection and blood work to see if there is an underlying issue. Thanks Doctor: Hello,Besetting is usually resolved by the age of nine years. In some cases, it remains present until twelve years of age. First, we have to look for any illness if present. I advise for ultrasound KUB, renal functions, urine routine, culture, calcium, blood sugar level. If all is normal, then try to manage conservatively.Hope I have answered your query. Let me know if I can assist you further.Regards, Dr. Sachin Kumar Agarwal"
},
{
"id": 60340,
"tgt": "What is the possible treatment for a cortical prominence measuring 3 cm in the mid pole of the left kidney normal variant vs mass lesions ?",
"src": "Patient: got abdominal ultrasound with mild enlargement of the liver wihtout evidence of focal hepatic lesions. There is a cortical prominence measuring 3 cm in the mid pole of the left kidney normal variant vs mass lesions. pls. interpret causes and possible treatment Doctor: welcome to healthcaremagic consult your doctor he will be best to guide you you may need further investigations like m r i and the nature of the ultrasonogram finding is to be found for diagnisis and treatment"
},
{
"id": 146794,
"tgt": "What causes dent in the lower spine?",
"src": "Patient: Hi yesterday I noticed a golf ball size dent in the lower spine of my son I am on holiday will contact my gp on Monday still have a week before we go back home I am very scared what should I do is it serious he is not in pain but does say his legs hurt when he stands for long should I cut my holiday short and get back to uk Doctor: Hello, Thank you for trusting HCM.I dont think you need to cut your holiday, what you need to do is to watch out for him not fall or injure himself until he gets his physical examination by his doctor. Probably its not something that is progressive in nature in a week period, contact your GP on Monday for further discussion and reassurance.hope that helped.Dr. Nazzal"
},
{
"id": 48601,
"tgt": "Suggest treatment for burning after urination",
"src": "Patient: sir , i am 35 years old and i have been diagnosed with kidney stone of 3.4mm in january 2014, and i have tested fot UTI but it was negative, i have burning sensation after urinating and its like again i feel urinating but it continues, it sometimes severe or mild, i took urilizer sometimes but whether there is some permanent cure? Doctor: Hi,Thanks for writing in.A stone measuring 3.4 mm is too small and if found in the kidney, it is usually non obstructive. This will not cause any urinary obstruction in most patients. If this stone is on its way out then it can cause symptoms like burning sensation after urination and feeling of urge. This can also happen if there are any other small stones in the urinary tract which can be held up any where along the lower urinary tract. I suggest you drink lots of water for some days and also continue taking urilizer which will prevent acidic urine formation. Please pass urine frequently.This can be done for about 15 days and if there is persistent burning and pain then please report to the urologist. You might require a CT scan and cystoscopy to evaluate other causes of burning urination and urge."
},
{
"id": 14270,
"tgt": "What to do for the rash on the arm?",
"src": "Patient: I have a rash or brusing on my arm between the wrist and elbow. It occurs most often when my arms are covered with long sleeves, like a jacket or sweater, If I wear a watch or any metal, it will also happen if I carry any item such as a bag containing any item over 8 oz Doctor: Hello and Welcome to \u2018Ask A Doctor\u2019 service.I have reviewed your query and here is my advice.From your description of your rash it could be one of these possibilities:Allergy or eczema, which usually appears in such areas, especially, if you have a prior history of any allergies. It could be aggravated by contact with the synthetic/woolen fabric or any metallic object, most commonly containing nickel such as watches and jewelry. This is allergic contact dermatitis. Without treatment it may persist and aggravate each time you come in contact with certain items.Sweat/certain cosmetics/strong soaps can also make it worse. The strap of the bag may be causing a reaction rather than the weight. The bag rubbing on the rash may also aggravate it.Also, rule out similar rashes elsewhere on the body and in family members to check for fungal infections, etc.First step is to note down exactly all the causes that give you a rash and avoid them. Once you are allergic to something, repeated exposure will cause repeated and worse rashes.Check with a dermatologist to confirm and certain blood and skin tests may be done to check allergy levels and to find out allergens as well such as patch testing.Avoid direct sun/dust exposure on the area. Wear loose cotton clothing. Use a mild nonfragranced soap and always moisturize with a hypoallergenic product such as Cetaphil.For the rash apply Calamine lotion 2 to 3 times a day when itchy. During a recurrence of new rashes or severe rash apply a steroid-based cream like Mometasone for 5 to 7 days till it heals. Do not use for a prolonged period, more than 2 weeks.Antihistamines like Levocet or Benadryl can be taken till itching subsides.Keep in mind that it is recurrent if exposed to allergic items and will take a while to heal. Don\u2019t scratch the area. Visit a dermatologist for follow-up or if it persists.I have answered your query. Let me know if I can assist you further.Regards,Dr. Dinah Levillard"
},
{
"id": 47989,
"tgt": "What causes pain during movements when suffering from cyst in kidney?",
"src": "Patient: Diagnosed with a right kidney upper pole 20 mm simple cyst. Am experiencing some pain in normal movements like putting on socks on my right side. Pain is sometimes mild and sometimes strong. I'm 57 and in general good health with slightly elevated blood pressure (taking metaprolol er succinate 25 mg daily. Doctor: HelloSimple renal cysts are generally asymptomatic in nature and usually it only require follow up ultrasound.You may need clinical evaluation and latest ultrasound of abdomen.Renal cyst may cause pain if size increases or there is infection in the cyst.It is important to correlate from earlier findings.It is also important to evaluate musculoskeletal causes of pain.Treatment depend upon findings.Get well soon.Take CareDr.Indu Bhushan"
},
{
"id": 48006,
"tgt": "Suggest medication for treating multi septated cyst in kidney",
"src": "Patient: Dear Dr.I am 38 years female and diagnosed with multi septated cyst noted in the interpolar cortex of the right kidney. it measures 5.6*5.1*3.4cms (52.2cc), no evidence of solid component. is it dangerous? what will be the medication?Kindly suggest.ThanksRekha Doctor: Hello Rekha and welcome to HCM.You don't need to worry about a simple cyst in the kidney.Unless, it causes complications like block to the kidney drainage or pain, you don't need to worry at all. Cysts will not clear with any medicines.Get an annual Ultrasound scan of the kidney to know if there's any change in size or content. If you're satisfied with the answer, hit the thanks button.Dr.Matthew Mangat."
},
{
"id": 142349,
"tgt": "Suggest treatment for blackouts and violent tremors",
"src": "Patient: When i stand up I have been having increasingly violent blackouts. Sometimes its just loss of vision and balance, but sometimes its violent shaking that often knocks my off my feet. The problem seems to get worse when i drink more water. I am in my 20s, and female. Doctor: Hello and thanks for using HCM.I have read your question and understand your concerns.Postural ( orthostatic ) hypotension or lowering of your blood pressure should be considered in your case.It is necessary to have a continuous study of your blood pressure and heart rhythm with a Holter 24 monitoring device in order to get a correct diagnosis.Discuss with your Doctor about this.Hope you found the answer helpful.Greetings."
},
{
"id": 208422,
"tgt": "Suggest remedy to overcome, lack of confidence and concentration and negative thinking",
"src": "Patient: i am presently unable to remail cool, to concentrate effectively as lots of fear inside, low self esteem and negative thinking, somw time feel restlessness.... i am 34 years old, height 5 feet 9 inch and medical history of healthy individual but always feel nervousness. Doctor: Hi,From what you have mentioned, you seem to be manifesting symptoms suggestive of a depressive episode. It is resulting in inability to relax, low self esteem, negative thinking. You should consult a psychiatrist for initiating treatment. Anti-depressant medications like escitalopram would be helpful. In addition, you can also consider treatment with psychotherapy in form of cognitive behavior therapy which will help you learn to relax and will help in changing the negative thought pattern you have.I do hope that I was able to answer your query. Best wishes."
},
{
"id": 133737,
"tgt": "How to treat the pain in the knees?",
"src": "Patient: My right leg aches when falling asleep and often wakes me up at night. It seems to start in my knee and radiate in each direction. I ve never had knee problems. I am female, 56 years old, 5 3, and weigh 150. Do you have any thoughts on how to relieve this pain so that I sleep better. I do sleep with a pillow between my knees and have done so for many, many years after suffering from back problems. I would think that the pillow would provide relief, so I m totally at a loss. I sleep 95% of the time on my left side, but it sure would be nice to change position every once in awhile. Thank you Doctor: hi,thank you for providing the brief history of you.A thorough neuromuscular assessment is advised along with MRI of lumbar spine.As the pain starts in midnight there are two possibilities here which I can understand. first - due to the back injury in the past now there's will help a pinched nerve , so is the symptoms and second - restless leg syndrome, a possibility of irritation to the sensory part of the Nerve.I will advice you to undergo physical therapy like - TENS therapy to reduce the pain and exercises on a later stages to strengthen the muscles of core, spine, pelvic floor and lower limbs.In my clinical practice, majority of patients respond well to physical therapy.RegardsJay Indravadan Patel"
},
{
"id": 191793,
"tgt": "Does unintentional weight loss indicate diabetes?",
"src": "Patient: My 21 year old daughter is concerned because she's losing weight without a any change in diet or exercise. She's gone from 117 to 113 which because she's 5'2\" is significant for her. That's the chief complaint. I told her it may be diabetes and if it continues she should get checked out. Doctor: HelloI have gone through your question and understood your concern.Glucose levels in our body are controlled by a hormone called insulin, which is made in the pancreas. Type 1 diabetes usually appears during childhood or adolescence.So if your doughter has diabetes it is more likely that it is type 1.In type 1 diabetes, the pancreas does not make enough insulin and this condition produce evident symptoms.Type 1 diabetes signs and symptoms can come on quickly and may include:-Increased thirst-Frequent urination-Extreme hunger-Unintended weight loss-Irritability and other mood changes-Fatigue and weaknessSo my answeris is Undiagnosed or untreated type 1 diabetes can cause weight loss but if your daughter has not other symptoms of the list,maybe the diabetes is not the culpit of her weight loss.Anyway, my advice is to check a fasting and random blood glocose or a Glycated hemoglobin (A1C) test.For a personalized comprehensive evaluation, treatment recommendations, and individual therapy,you can ask me at HealthCareMagic at my private link: bit.do/hcm-Dr-Xhardo.Hope i have answered to your question.If you have other doubts,feel free and ask.I will be happy to help.Take care."
},
{
"id": 186945,
"tgt": "What causes soft gums with swelling on face?",
"src": "Patient: hello Dr, The entire left side of my face has been swollen for nearly a week, my gums in that area is bubbling up, soft but scary feeling. This is very painful, I thought it would go down after a few days. But days pass and no kind of improvement has occured. Should I go to a doctor at a emergency room or go to a dentist? Doctor: Hi, in my opinion u should see a dentist at the earliest and not ignore it, as per ur symptoms it seems to be an infection which needs to be investigated for the reason for infection and treating it with antibiotics and analgesics and treating the reason as well, if u want me to help me more u can upload a few pics of ur swelling and intra oral pics and if u hv any latest xrays.wish u early recovery n all the bestDr Adarsh Desai"
},
{
"id": 167355,
"tgt": "What does extended gum/skin growth between two teeth in a child indicate?",
"src": "Patient: my six year old daughter has what appears to be an extended gum or skin growth between her two front teeth on the bottom. It seems to have grown. The dentis didn't observe but I found it. She sent us to an oral surgeon for follow up but doesn't think it is anything serious. Doctor: Hi.Welcome to HEALTHCARE MAGIC..I have gone through your query and can understand your concern..As per your complain extended gum growth in between the two front bottom teeth can be due to gum inflammation causing swollen gum while it can also be due to spacing between the teeth causing gum to overgrow in that area..It can also be a high frenulum attachment..It is true that is not appearing to be anything abnormal and you need not to worry about it..You can consult a Pedodontist and get evaluated and the cause can be clinically evaluated and then only treatment can be advised..In case of gum infection scaling and antibiotics will help..If it is high frenulum attachment then surgical relieving of frenulum has to be done..Hope this information helps..Thanks and regards.Dr.Honey Nandwani Arora."
},
{
"id": 200905,
"tgt": "What causes itching in groin?",
"src": "Patient: I am male, 25yrs. Doctor I am facing consistent itching in the groin. I consulted a doctor some 6 months back he prescribed Onabet B cream. After applying the cream itching get reduced. Now even though I am applying the cream the itching continues. There is no red dots or rashes as in Fungal infection. Groin appears to be normal.Please guide me the medicine. Thanks in advance. Doctor: Thanks for asking in healthcaremagic forumIn short: Keep area clean and dry(you may trim/shave)Explanation: Moisture can cause fungal infection frequently over there. So keep that area clean and dry to prevent such infection and itch. Visit a doctor if it doesnot subside."
},
{
"id": 191791,
"tgt": "Is blurred vision with hoarse voice in a diabetic patient cause for concern?",
"src": "Patient: A diabetic. Age 50 male. All of a sudden for last 3 days, unable to read anything close, especially small letters. For examples, cannot read the notifications on the cell phone. Especially on the left eye feels like there is something bluring the vision. Was told the feeling is like if the pupils are dialated.For the past 3 days, he is also having flu like symptom with flem and voice change.Should he be worried? Doctor: HelloI have gone through your question and understood your concern.Chronically high blood sugar from diabetes is associated with damage to the tiny blood vessels in the retina, leading to a chronic complication of diabetes called diabetic eye disease or diabetic retinopathy.The risk of diabetic eye disease increases the longer a person has diabetes.The disease often progresses unnoticed until it affects vision.Rapid tightening of blood glucose control leads to transient deterioration of diabetic retinopath.In my opinion this can be a potential cause of his sudden visual problems.Anyway i suggest to consult an ophtalmologist for a comprehensive dilated eye exam for the right diagnose and treatement.Hope this information is helpful.If you have other questions ,feel free to ask.I will be happy to help.(you can ask me at HealthCareMagic at my private link: bit.do/hcm-Dr-Xhardo)Take care"
},
{
"id": 2208,
"tgt": "Will i face any problem during pregnancy?",
"src": "Patient: I anitha, I have pcod problem I done laprascope and j concive, after 13 weeks baby is miscarrage reason, no heart beat then we done dnc and I try iui 4 times failure after 4 years I concive in normal cycle with help of medicine then I affected by diabeties but it normal value after 25 weeks the severe oilgomo some problems ,doctor said no water in your stomach so the baby plus is 47 no chance to protect baby , so they devier normal , now we done blood test is all normal, now I am taking ovanac tablet, this problem continue next baby please advicememm Doctor: Hi.Speak to your doctor to help regularise your menstrual cycle, and then initiate the same medication you used to conceive the first time. Yours is a complicated case, and based on the information at hand, I cannot guide you well enough.Best wishes."
},
{
"id": 156059,
"tgt": "Is diarrhea with bleeding symptoms of cancer?",
"src": "Patient: I have had diarrea for 5wks. It then improved but began to bleed.My doctor has taken blood and there are no nasty bugs and I am waiting to see if there are cancer markers.There is cancer in all of my mothers sided of the family.If the diarrea has eased could it still be cancer? Doctor: Severe inflamation in intestines can also cause bleeding while passing stools.Any bleeding while passing stools shoul be evaluated as it is a sign of cancer.Since you have a family history i strongly suggest you toGet a colonoscopy to ruleout cancer. Meet a gastroenterologist."
},
{
"id": 64319,
"tgt": "What is the treatment for a pea sized bump in the lower leg?",
"src": "Patient: Hi I am a 52 year old woman who is moderately active. I walk 22-24 miles a week at a quick pace and sometimes bike 26 miles also. I have noticed small pea size bumps in my lower legs which are tender to the touch. What could they be and what should I do for this. I take 1200 mg of calcium daily. Thank you. Doctor: Hi! Good evening. I am Dr Shareef answering your query. Eventhough it needs a complete physical examination to opine on these bumps, if I were your doctor, after a clinical assessment I might advise you with a dopler ultrasound of both the legs to rule out chances of any varicocities in the legs. If excluded, I would advise you for a cytological dignosis of these by a FNAC (fine needle aspiration cytology) of the lesion. Further management would depend on the reports of these investigations.I hope this information would help you in discussing with your family physician/treating doctor in further management of your problem. Please do not hesitate to ask in case of any further doubts.Thanks for choosing health care magic to clear doubts on your health problems. I wish you an early recovery. Dr Shareef"
},
{
"id": 139759,
"tgt": "Can the botox treatment for dystonia lower the WBC count?",
"src": "Patient: I have dystonia in my right arm/hand and also my neck. I have been receiving Botox injections for 4 years. I see a neurologist at a movement disorder clinic for the dystonia. I take gabapentin, clonazepam, and baclofen to also help with the dystonia symptoms. I recently had some routine lab work done at my PCP s office and my WBC count was low at 3.2, I am 47 years old. Can the Botox treatments and low WBC count be associated? Doctor: Hello, Botox treatment does not cause low WBC count. You can repeat the test after two weeks. If the white blood cells are persistently on the lower side, you can consult a physician and get evaluated to rule out hematological causes. Hope I have answered your query. Let me know if I can assist you further. Take care Regards, Dr Shinas Hussain, General & Family Physician"
},
{
"id": 167497,
"tgt": "Can Petechiae like bruising, fatigue and a rash all be related?",
"src": "Patient: Hello my son is 5 and has had some concerning symptoms over time. He has been bruising very easily in a petichiae like way he also has had increased fatigue and decreased appetite( still eating adeqetley). Just recently he has developed a painful white streak on the inner left cheek. I was wondering if these symptoms could all be related? He has no significant medical history other than pneumonia as a baby,few indoor allergens such as dust, dogs and cats. And he has currently been battling molluskum and eczema on the back of his leg. Doctor: petechiae may be due to platelet malfunction{giant platelets,platelets non adhesiveness}many causes is there for platelet malfunctionbernard soulier syndromeglanzmann thromboastheniaautoimmune disorders like ITPbone marrow supressionif u r baby is having repeated infection,eczema,petechiae it fits in WISKOTT-ALDRICH SYNDROMEIT IS X LINKED DISORDER WORK UP HIS PLATELET COUNT, IMMUNOGLOBULINS{IG A IG E MORE}THEN CONSULT PAEDIATRICIAN TAKE APPROPRIATE ACTION"
},
{
"id": 200808,
"tgt": "What causes a tiny white bump on the testicle?",
"src": "Patient: Hi I have a tiny white bump on my testicles right before the shaft of my penis. It has been there for a long time and if I try to pop it, it won t. But the other ones will. I am 19. And have had unprotected sex. I took a yerin test recently. The bump looks like a white ish cloudy, and a blends in with my testicles a little. It doesn t hurt and you can feel it. Hoping you can help Doctor: Thanks for asking in healthcaremagic forumIn short: Those are fordyce spots and are normal.Explanation: Fordyce spots are the visible white sebaceous glands over there.They are harmless and please do not meddle with it, it may get infected if you do. If it is causing you cosmetic problem then visit a dermatologist for its removal. Hope this helps you, please let me know."
},
{
"id": 98178,
"tgt": "Is 500 mg Krill oil safe for my 2 year old ?",
"src": "Patient: Will 500 mg Krill oil hurt my 2yr old? Doctor: hi; why you want to give this Krill oil or any oil for as your baby is 2 yrs old. The basic thing is oil directly is difficult to swallow ;there can be chance of aspiration pneumonia so better don't experiment. thanks"
},
{
"id": 37403,
"tgt": "Is Tylenol safe for fever?",
"src": "Patient: My son is 12 and work up with a 102.9 fever and I have been giving him Tylenol. And with the Tylenol it seems to be going down. Now it's up to 103.4 and I just gave him more tylenol. He is not complaining about anything should I wait and see what happens of call the dr. Doctor: Thanks for contacting HCM. It is safe to give your child Tylenol for his fever. I recommend in my practice that you can give Tylenol every 4 hours for fever. You can also use a tepid bath. Place him in bath water that is about 99 degrees and place him in it. That will slowly lower his temperature. Do not use a cold bath as it will make him shiver and actually increase his core body temperature. Overall temperature is not a bad thing. We only reduce a temperature if the child has a history of febrile seizures. Remember fever has been around of thousands of years as a defense against infection. The fever reduces the ability of the infection to grow and it also brings more white blood cells into the body to help fight that infection. So fever is not a bad thing.You can wait and see how he behaviors over the next day or so. Hope I answered you question. Please contact us again with your medical questions and concerns."
},
{
"id": 167594,
"tgt": "Suggest alternative medicine for Impetigo in a child",
"src": "Patient: My son aged 4 (normal hieght and weight, no medical contitions) is diagnosed with impetigo. and the doctor prescribed oral antibiotics. I don t really like to give antibiotics unless it is an emergency. Clould you please help me with some alternate options. thanks Doctor: hi madam , it's important to treat impetigo because it can turn into a serious cellulitis, the earlier treatment the better , and both oral and local antibiotics are used , I recommend a 1st generation Cephalon poring like cephalexin and local fuscidinic acid .I hope this helps"
},
{
"id": 126323,
"tgt": "Suggest treatment for pain in the ball of the feet",
"src": "Patient: Thank you. I felt pain in the balls of both feet a month ago. I was moving into a new apartment at the time and just figured I had over done it going up and down stairs, jumping off the rental truck. Days later I woke up with pain in all 10 toes. It feels like there is no circulation. No swelling. Now it s traveling up to my knees, top of my feet, and ankles. I can barley walk without wobbling. Any suggestions of what it may be? My insurance doesn t start until March first. I d appreciate any information. Thank You. Doctor: Hi, You can start ultrasonic therapy for this and there is no relief after 2 weeks then you can get local infiltration done from your orthopedic surgeon. Hope I have answered your query. Let me know if I can assist you further. Regards, Dr. Anuj Gupta, Spine Surgeon"
},
{
"id": 139209,
"tgt": "What causes swelling in feet and ankles and joint, cervical and lumbar pain?",
"src": "Patient: I have been hoarse for 3 months. The ENT diagnosed it as thrush from Advair and Gerd. I am now in medication for the Gerd, but I have come off of Advair. I have also been diagnosed with Aortec Stinosis and now my feet and ankles are swelling every day and very tight feeling in my lower legs. I also have extreme joint pain in cervical and lumbar. On medication for pain. Have had many procedures to reduce pain. Getting very concerned. Thank you for your opinion. Doctor: Hello, I have studied your case.Most probable reason for your symptoms could be postural problem as you may be sitting for long time.I will advise you to do MRI spine to see for any disc bulge or spinal canal stenosis.And also check your vit B12 and vit D3 level.Medication like methylcobalamine and analgesic will reduce pain.Some exercises which can be done include- Spine extension exercises, Lying on your stomach flat lift leg 6 inches from ground, do it for other leg.Now lift both hand and leg simultaneously, 6 inch off the ground and stayPosition for around 10 breathes. Core stabilizing spine exercises will help.You may consult physiotherapist for guidance. He may start TENS, or ultrasound which is helpful in your case.If not relieved you can take epidural injection.Hope this helps. Wish you a speedy recovery. If you find this answer helpful do not hesitate to rate this answer at end of discussion.Take care."
},
{
"id": 97963,
"tgt": "Is there any treatment in Ayurveda for curing hypersensitivity pneumonitis ?",
"src": "Patient: Is there any treatment in Ayurveda on Interstitial Lung disease in the form of Hypersensitivity pneumonitis?My father is complaining of dry cough from January to May every year for 3 consecutive years and pulmonologist has diagnosed him to have Hypersensitivity pneumonitis after the HR Ct scan and PFT reports.He gives history of smoking,dust,exposure to fungus or black moulds as it has started growing on our house walls for last 3 years in rainy season which we clean after rains go away.He is on cortico steroids and anti allergic medications.Can you suggest any ayurvedic treatment for the same? Doctor: Hi, Most cases of Hypersensitivity Pneumonitis develop only after many years of inhaling allergens, which include microbe, animal or plant proteins and certain chemicals that form haptens. Hypersensitivity pneumonitis is charecterized by diffuse inflammation of lung paranchyma and airways in previously sentisitized patients. Based on the length and intensity of exposure and subsequent duration of illness. Clinical presentations of hypersensitivity pneumonitis are categorized as acute , subacute and chronic progressive. The best way to treat HP is to avoid the antigen that caused it Your petbird, moldy carpet, or hot tub is the source of the antigen you can remove it from your home. Properly dry and store farm products to decrease the risk of developing hypersensitivity pneumonitis. Indivituals who may be exposed to allergens should work in ventilated areas and use respiratory protection [ masks ] YOGA, Limited human study exists for yoga in the treatment for lung diseases , such as bronchitis or airway obstruction. VASA, Adathoda vasika and Ocimum santum an indian herb will control allergens. ."
},
{
"id": 167859,
"tgt": "What causes leg pain in a toddler?",
"src": "Patient: respected sir, I have a baby boy who is 2 years and 8 months old.He tells of pain in his both legs since past 1 month. Ours is a small house on 3rd floor and my baby is in front of my eyes 24 hours , I know there is no overactivity that my child does; instead I can say he cant get enough place to play. Then why he complains so.Pain is in daytime as well as in the evening. Doctor please help me . I d be grateful to you. Doctor: first let's try to locate the pain is it in his knees ? lower legs ? calf muscles ? or is every where ? does this pain prevent him from playing ? does it awake him in the night ? if you press against his legs , does it hearts ? OK, if this pain is discrete and can't be located , and doesn't prevent him from activities, and no tenderness is felt when pressing on legs , and mostly pain is afternoon or the night , then there is nothing to worry about , it's called the non specific lower limb pain of childhood , or some like to call it growing pains , anything else besides what I mentioned here is worrisome and needs to be investigated by his pediatrician I hope this helps"
},
{
"id": 100227,
"tgt": "What causes coldness, hot flashes and gagging?",
"src": "Patient: I believe My friend is currently going through heat exhaustion. She is shaking, said she feels cold / hot internally, but she doesn t feel hot to the touch, gagging but no vomit. Put her in a cold tub and gave her plenty of water / Gatorade. It s been about an hour of pain for here, now and she is asleep. Should I still be worried? Doctor: HIWell come to HCMSuch symptoms most of the time happens due to functional condition and may not be having any pathology behind this, better to keep her under the observation for fever and other sign of infections if ever have, otherwise this would come around on its own, ask her to have some rest, better to keep the stress level low, and no need to worry about this, have a nice day."
},
{
"id": 139585,
"tgt": "Can I smoke marijuana with nerve pain and spinal surgery?",
"src": "Patient: I am 60 years old ,5 ft 3 in, 186 lbs , Had spinal surgery and was prescribed to take 200mg of gagapentin every 8 hours for nerve pain Had 6nerves repaired from spinal stenois,bone graph for fusion to the last 2 disc taken from the left pelvic bone. Can I still smoke marijuana with this medication. Doctor: Hello, Marijuana and gabapentin, both may cause excessive sedation, so, it's advisable for avoid simultaneous use. You can smoke marijuana ( small amounts ) before sleep. Hope I have answered your query. Let me know if I can assist you further. Take care Regards, Dr. Erion Spaho"
},
{
"id": 143793,
"tgt": "Suggest treatment for loss of memory and less opening of eye",
"src": "Patient: IN 2012 i had an incident extreme nausea and sweating- i thought my blood sugar had fallen-I went to the bathroom because I THOUGHT I was going to throw up and have diareah at the same time--- so as I sat on the toilet -- i began to shake I woke up my husband to get me something seet to eat-- he did and I took a bite of cake and then handed it back to him and told him that I couldn t eat any more.. he left to bring the cake to the kitchen-- when he came back I had fainted - he called my daughter to come down stairs - she said that my arms were rigid at my side and jaws were clinched.. she called 911 and the ambulance took me to the hospital... the ER doctors said that the EKG was eratic...and said that I probably had a heart attack... I had complained that my back was hurting--- however it was because of the rough ride in the ambulance-- I was extremely thirsty.. but no one in the ErR listened to me-- my husband had to sneak water to me... any way I was transferred to another hospital and there they continued tests for a possible heart attack-I was tested alterriorgram I think that is how you spell and there were no blockages.. the cardiologist said that my kidneys were very dry and I was put on IV fluids.... before the incident I was taking HTCZ blood pressure meds...the Dr s diagnosis was synchopy which I know as fainting.. but I have lost memory some has come back- my right eye does not open as wide as it used to- I know that the day it happened I did not drink much water Doctor: Hi ,you probably had syncope due to dehydration and hypotension which has caused hypoxic injury to brain leading to memory loss and third nerve dysfunction.This cannot be reversed totally and will gradually improve with time. Thanks"
},
{
"id": 39281,
"tgt": "Do I need to take vaccination if the dog dies after scratching me?",
"src": "Patient: i was bitten by a dog around a year back. i took all the 5 doses of verorab. 20 days back a pet dog non vaccinated scratched me on my hand & blood came out. i again took2 doses of rabipur (0 & 3). but after 15 days the dog died. idont know the reasons of its death. Do i need to take more doses of the vaccination. Doctor: Hello,Welcome to HCM,As you was bit by a fully vaccinated dog one year back for which you have taken all the shots of antirabies vaccine and once again you were scratched by a stray dog on your hand which led to bleeding from the site.According to WHO you were categorized into Re-Exposures and you should receive only two doses of vaccine on days 0 and 3. Even it is a Cat III bite there is no need of passive immunization.You will be having protective antibody titres and the vaccine on day 0 and 3 will boost up the immune system by anamnestic response and protects you from getting rabies even if the dog dies of rabies.Thank you."
},
{
"id": 37699,
"tgt": "What is the treatment for bladder infection?",
"src": "Patient: My mother is 54 years old and seems to be in good health passing all her test they have given her. but her bladder seems to be swollen. they thought it was just a bladder infection. and she seemed to get better. she is getting a vag ultra sound soon. wonder what it is. Doctor: Hello,Welcome to HCM.I appreciate your concern for your mother's health.If her bladder is distended, it means that either there is some lesion interfering with the free flow of urine from her urinary bladder or there can be some problem with the contraction of the bladder wall muscles. This can occur with a urethritis if she had a recent infection or with strictures from old infections.There can be some thickening of the urinary bladder wall from a urinary infection or it can be due to other lesions, benign or otherwise.A transvaginal ultrasound examination with post micturation volume will help to come to a proper diagnosis.Wish her a quick recovery."
},
{
"id": 201993,
"tgt": "Does masturbation affects the knee?",
"src": "Patient: does masturabtion affects the knee ?? and how can i fill my knee with sperms im 14 years old sometime after masturbation my knee hurts me and is masturbation healthy ? what kind of food i need to eat to fill my knee with sperms or wich medecin Doctor: HIWell come to HCM You are you too young for this act and in my opinion better stop this or if you are still doing this then stop thinking weird the knee pain cold be due to the strength that comes while masturbating and this is nothing to take with the semen, hope this helps."
},
{
"id": 161829,
"tgt": "Suggest treatment for child to improve growth and muscle tone",
"src": "Patient: Hello, my daughter 17 months is suffering from global delay in development is not reaching her growth milestones and are providing occupational therapy. we have done all the tests like MRI, BERA, VEER etc,, and all reports shows no signs of abnormality and has very low muscle tone. please advice Doctor: Hi, You can do physiotherapy and at the same time use L Carnitine and UBQ suspension for muscle strength. Hope I have answered your query. Let me know if I can assist you further. Thank you. Regards, Dr. Sumanth Amperayani Pediatrician, Pulmonology"
},
{
"id": 118122,
"tgt": "What is the remedy for high blood pressure when on clonidine?",
"src": "Patient: I have high blood pressure I am on medicine but it still stays high they put me on .1 milligram clonidine last night to take when I think I need to, my arm also hurts from elbow up and also my neck and head, could this be caused from blood pressure, they also said I had arithritis in my shoulder Doctor: Thanks for contacting HCM. Dear patient you have not mentioned why you are on clonidine, as clonidine is not used for normal high blood pressure cases. Usually when kidneys are affected then clonidine is needed and that too on a regular basis not when you need to. You have not mentioned which of your arm hurts as pain in left shoulder and arm can be due to cardiac problem like Attack or angina. Please get your blood urea and creatinine done to rule out renal damage. If there is renal damage you can take clonidine 3 times a day and if necessary add prazocine 5mg at bed time. please also get an E.C.G. done to rule out cardiac cause of shoulder pain. regards"
},
{
"id": 217902,
"tgt": "What causes constant pain in one of the knee?",
"src": "Patient: I am having constant pain in one of my knee. It happened after i heard a crakling sound in my knee bones while i was trying to stretch my feet. for the past 4 days i am unable to walk properly and am in constant pain. Also i did not take any medications yet. what do i do? and whats wrong with my knees? Doctor: Hai u might have an acute knee injury either meniscal or small ligament injury or could be a sprain, u can go for pain killers, avoid vigorous upstairs and down stairs as the injury may aggravate. Pls go for mri scan of the knee, and an orthopaedic opinion"
},
{
"id": 224188,
"tgt": "Had painful mirena insertion. Having contraction like pain. Possibility of pregnancy?",
"src": "Patient: Hello, I went to have a merina coil fitted this morning. It was a failure the pain was excrutiating like labour and the GP said my cervix was rock hard and contracting against the procedure. She spent 20 minutes trying to fit it. I am still having contractions (4 hours later) and have vomited once. The GP asked if I had had sex after my last period and I had not. Though i did have unprotected sex on day 9 of my period. My periods are 9 days. My period is due in one week, could this reaction be a result of a possible pregnancy? Doctor: Hi,Pregnancy makes the cervix softer rather than harder. And It would have been easier to insert, though it would have caused bleeding as well. Some Mirena insertions are painful and result in crampy pain. The maneuver of insertion involves holding the cervix, which could cause nausea and uterine cramps.I would advise you to go for the mirena insertion on the 8-9 day of your next period. This will not only ensure you are not pregnant, but also that the cervix is slightly open at the end of the period, which could make the insertion easy.Hope this helps.Regards."
},
{
"id": 133240,
"tgt": "What can cause coldness in hands?",
"src": "Patient: sir first of all i want to tell u that i am not good in english so pls ignore ny faults. sir...sometimes i feel my left hand cool, it happened in summer season. earlier...about a month ago i felt same thing. a doctor advised me to take NEUROKIND GOLD injections. that time everything was okay. but again i feel same thing. please advice me some thing. another thing is that...when i feel cold in my left hand..then i dont feel any weakness. Doctor: Welcome at HCM I have gone through your query and being your physician I completely understand your health concerns. Based upon description it is difficult to comment. kindly meet your attending physician for proepr assessmment. Meanwhile stay calm and use acetaminophen to relieve pain Get well soon Hope your query is adequately addressed if you still have any feel free to ask RegardsDr Saad Sultan"
},
{
"id": 205149,
"tgt": "What causes anxiety and depression?",
"src": "Patient: Hi Thank you I have a 55 year old boyfriend and we have been dating for about 10 months now , he recently cheated he has anxiety and depression issues, is this normal behavior he said he was drinking and things just happened he is not being properly treated for his issues I am extreamley up set what can I do thank you Doctor: Hi and thanks for questions.anxiety and depression mainly occur due to decreased level of serotonin and norepinephrin in brain. it will occur due to major life event like breakup, divorce,someone close died, business loss etc. it will treated with medicine.thanks"
},
{
"id": 37187,
"tgt": "Is there any treatment other than silver nitrate for the abscess on the area of Hysterectomy?",
"src": "Patient: Hello, I had a hysterectomy in Sept 2013, shortly after the surgery an abcess developed above the stitching which had to be drained. A catherer was inserted for drainage. After it was removed I started to slightly bleed and had a discharge. It has been treated with silver nitrate every month since the surgery, I also applied Primerin creme, nothing s helping the situation (bleeding stopped but not the discharge). I am desparate for ending this problem. Would there be any other treatment I could have beside the silver nitrate? Thank you. Doctor: Hello ,I understand your concern. I am Dr. Arun Tank, infectious disease specialist, answering your concern.In my opinion you should do culture and sensitivity from the discharge.Silver nitrate act as a good antiseptic agent against variety of organisms. But in your case some resistant organisms has grown in the wound.With Grams stain, culture and sensitivity we get the idea about organisms and so we can treat accordingly.Continue your primerin cream it helps in healing. Please maintain the local hygiene and cleanliness. It will help you in healing.Glad to answer your further concern contact me on bit.ly/DrArunWe wish you a best health at healthcare magic. Thank you,Dr. Arun TankIf you are satisfied with my answer rate me as five star and close the answer"
},
{
"id": 194615,
"tgt": "Suggest treatment for having semen during urination",
"src": "Patient: Sir, i am 18 year old boy i stared masterabulating when i was 13, and i still continue it i can't stop practising it if i stop it i get head ache or i am not able to concentrate on my studies i am also getting grey hair due to this bad habit if i stop masterbulating for a week then sperm comes out when i urinate or when i am sleeping. Please give me a good solution to over come these problems and stop getting grey hair. Doctor: Hello, Excess masturbation can cause psychological and stress issues like you have mentioned here. I suggest sex therapy to remove the habit of masturbation. Hope I have answered your query. Let me know if I can assist you further. Regards, Dr. K. V. Anand, Psychologist"
},
{
"id": 58887,
"tgt": "Liver problem. Upset stomach, acidity. Took vitamins. Advise?",
"src": "Patient: hi, I am suffering from liver problem for a long time , like i cant eat much at a time which i used to take 1year back also, stomach upset, acidity is a daily problem... the most important is that my doctor gave me some medicines (vitamins) but they are not work, to make me hungry and grew my body weight... I am 21 at my Weight is . 54kgs. Doctor: Hello,If you have acidity problem it may be because of more liver acid and bile acid release in your stomach. You should eat something to take up all the acid that secreted for digesting if there is no enough food then you will get acidity. Try eating little food many times rather than bulk food at a time. Use some antacids if you have more acidity problem but limit it.Have a Good Health."
},
{
"id": 216331,
"tgt": "Suggest treatment for tightness and pain in left breast area",
"src": "Patient: Hey, My name is Tayyab. I am male 22. I have been experiencing pain in my left breast area. Sometimes its like a sharp tightness and squeezing feeling. Its been going on for three weeks now. I am under a bit of stress these days. Should I be worried about it? Doctor: Hi how are you?I have read your concern. Stress is among the major factors responsible for majority of health hazards these days. Anxiety and depression usually result when one takes undue stress in their busy lives. I suggest you to get relax as soon as possible to combat these symptoms. More than 50% of pain and discomfort goes away when we do relaxation exercises. You are young and exercise can make you stronger and calmer reducing stress. Join gym and increase work out slowly. Furthermore, an evaluation of chest pain by your medical specialist is essential in case your heart is undergoing any changes due to stressful life episodes. I will not recommend any medication at this stage. Just change your lifestyle, eat healthy diet and make exercise your routine."
},
{
"id": 144941,
"tgt": "What is the best medicine for parkinson s disease?",
"src": "Patient: my father is being treated for parkinsons and has been prescribed CoQ forte and other medicines - Galamer OD 8 mg (1), Pramipex 1 mg (1x3), Rasalect 0.5 mg (1), Qutan 100 mg (1) and Trazonil 0.25 mg. he is now having vision problems, confusion, hallucinations, mental/mood changes. is the medication right? he has once been prescribed wrong medicines - syndopa plus (1x3) and pacitene (1/2) for two months and reached a critical state and nearly collapsed before being examined by a neurosurgeon and prescribed the above medicines. Doctor: Hello dear, There are many drug treatment available for parkinsons. The best effective is levodopa. Dopamine agonist like pramipexole, ropinirole can also be tried. other drugs like rasagiline, amantadine are also used in patients. The choice and dose of drugs depends upon stage of disease and tolerance of drugs. Postural hypotension is common side effect in early stage of dopamine prescription. you need to consult a neurologist for proper evaluation and treatment. Hope you found the answer helpful. Regards Dr Neeraj Kumar Neurologist"
},
{
"id": 103244,
"tgt": "Celiac disease, have dryness in mouth, small cuts around cheeks and roof of mouth, itchiness on gums, ears after drinking soup. Reason?",
"src": "Patient: My mouth has been feeling dry though I m not thirsty and have small cuts around my cheeks by my mouth, the roof of my mouth and feel itchy on my gums, in my ears and in my mouth. I feel like I ve had a similar reaction before when I ate at a mongolian restaurant several years ago. I ve been eating a lot of soup since Saturday night and feel like I may be having a reaction to that maybe - I ve recently been diagnosed with celiac disease and have also taken dairy out of my diet . Could it be too much salt? Would benadryl help? Doctor: celiac disease can cause this you need to stop whole wheat instead of gluten as there are many proteins in wheat and any protein can reactyou can use antiallergic like ebastine 10 mg odand apply localapplication on areas of cut in mouth and lips triamsolone mouth paint and use liquid antacid tds"
},
{
"id": 138050,
"tgt": "Suggest treatment for neck fracture",
"src": "Patient: A man i know was ejected 50 ft from his car. The doctor said that he needs nck surgery because there is some broken bones in his neck. Today the doctor said that there are unforeseen complications. Do you know what the complications maybe? They said if he dont get the surgery he maybe wheel chair bound Doctor: Yes the neck surgery is indeed dangerous. The vital structures and the spinal cord all pass through neck. There are chances of them being hurt although the chances are minimal but still they are there. Moreover the cord symptoms may not fully recover. But usually not going for surgery has more complications than going for."
},
{
"id": 156011,
"tgt": "Does a female urethra have a bulb like thing at the end?",
"src": "Patient: I am a 70 year old female, had bladder cancer which is now cured. I don t know what an uretha looks like in a female but noticed that the end of the uretha tube in me was surrounded by what looks like a bulb the size of a large marble. It wraps itself around the uretha. Does a female uretha have a bulb at the end? Doctor: Hi and welcome ot HCM. this is normal and last part of urethra is surrrounded with such bulbs which may increase in size during aging. WIsh you good health. Regards"
},
{
"id": 46534,
"tgt": "Can neck pain and respiratory problem be caused by dialysis?",
"src": "Patient: Hi zdoctor, my dad is going through dialysis weekly twice. and he went for an angioplasty 2 years back. He is having diabetes also, ex smoker. now some times he is getting pain in hand neck and respiretory problems at night specially after food. i want to avoide dialysis, is tghere any suggestions or can i get few tips Doctor: Hello and welcome to HCM. As an Urologist and kidney transplant surgeon,i understand your anxiety.Your dad has renal failure,diabetes,CAD,and H.T.(hypertension).CAD is coronary artery disease,for which he's already had an angioplasty.Dialysis is done for people with end stage kidney disease ( ESRD ).The only two options available are: 1.dialysis or 2.kidney transplant.You've not written your dad's age and medication.He's has a high risk for a kidney transplant surgery.So,no option to dialysis.If you've any other doubt,send all the reports to me,as a direct question.Dr.Matthew J. Mangat."
},
{
"id": 105472,
"tgt": "Suffering with essential tremors, aggravates with sulbutamol intake, normal thyroid tests. What treatment is advisable?",
"src": "Patient: Dear Sir, I am 26y M. I just have Checked my Thyroid blood test coz I am suffering from Essential Tremor(Shaking hands) Since last 8 yrs. I also have allergy with salbutamol group of medicince. when i take my tremor increase dramatically. My Test results as below from thyrocare test lab: TOTAL TRIIODOTHYRONINE (T3) C.L.I.A 125 ng/dl TOTAL THYROXINE (T4) C.L.I.A 8.8 \u03bcg/dl THYROID STIMULATING HORMONE (TSH) C.L.I.A 0.89 \u03bcIU/ml Comments: SUGGESTING THYRONORMALCY kindly advice me the right treatmentorany other meditation. I feel very stressed due to my tremor. My life is like unuse to stay live. thanks in advance for you early replies.. Doctor: Dear friend, welcome to HCM. what are you taking salbutamol for? how long? what dose is it must? have bronchospastic disorder? because salbutamol can usually cause tremors. TFTs are normal. pl. revert with deatils . take care and pl. keep me posted on the progress."
},
{
"id": 160869,
"tgt": "What causes fever,vomiting and leg pain?",
"src": "Patient: My son is 4 yrs old and has had a feaver for 3 days now with vomiting @ night He also complains that his legs hurts all the time I have him a dr apt today I just want to make sure they check him for all possible threats of luekimia cause he seems to have alot of the symptoms what should I do Doctor: Hi, These symptoms are typical for any viral fever usually fever disappear and pain decreases after 5 to 7 days. I do not know why you are thinking of leukemia? Yes, prolonged fever and bony pain is a sign for leukemia, if you want to exclude that screen it with few investigations.Hope I have answered your question. Let me know if I can assist you further. Regards, Dr. Deepak, Pediatrician"
},
{
"id": 132297,
"tgt": "Suggest treatment for chest pain after gardening",
"src": "Patient: I have been gardening and while pulling some stubborn weeds I could hear a dull crack in my chest, just above my stomach. I stood up and taking a deep breath, it was very painful. Now I can not bend down to pick up anything of the floor. Have I fractured my sternum? I am 77 years old and have osteoporosis. Erica Lind Doctor: You are having osteoporosis and age 77 yrs.While pulling heavily you might have fractured one of your ribs.Though sternum might not have fractured but there are chances of rib fracture.You can get an Xray done to confirm it.Minor crack will get healed in 4 to 5 weeks.But you need to take rest and avoid heavy work till pain reduces."
},
{
"id": 168763,
"tgt": "Suggest remedy for navel pain in a child",
"src": "Patient: My 8 year old son had a pain above his belly button this evening - got him a hot water bottle. just been in to check, he is fast asleep but felt cold to touch on forehead...temperature of 35.3. he did respond when I tried to wake him to check he was ok but now asleep again. should I be concerned? Doctor: Hi... by what you quote I feel that your kid might be having a worm infestation or it could be a mesenteric adenitis which means but he might be having some small lymph node enlargement in the tummy. This could be the reason for pain. I suggest you not to do anything as of now. But if he develops green colour vomiting or blood in motion or continuous Diarrhea and decreased urine output or rash and joint pain then approach the nearest Emergency Room or your child's pediatrician.Regards - Dr. Sumanth"
},
{
"id": 226083,
"tgt": "Had unprotected sex. Should I take pill to avoid pregnancy?",
"src": "Patient: genital-genital rubbing took place last night with my bf but there was no ejaculatory fluid or penetration that occurred. I was on my prime day of ovulation according to my cycle and I read debates on whether pre-ejaculatory fluid contains motile sperm- however he says this did not happen but it says sometimes guys don t notice...I am pretty paranoid and am debating getting a morning after pill/wondering if it is necessary. Doctor: Hi,Welcome to HCM,Thanks for posting query to this forum,It is unlikely to have pregnancy without penetrative sex but theoretically possible.According to you it occurred near ovulation period hence it is better to be safe than sorry so the morning after pill may be taken if it is with in 48 to 72 hours to avoid any chance of pregnancy.Remember it can cause withdrawal bleeding or irregular cycle you can also consult a Gynecologist for her expert advice.Remember to practice safe sex as sexually transmitted diseases and unwanted pregnancies are prevented.Hope this helps.Take care.Good Luck.Dr.Akhilesh Dubey M.D."
},
{
"id": 177161,
"tgt": "Is enlarged lymph node in front of right shoulder in an infant serious?",
"src": "Patient: Hello. Thank you. My 12 month old baby girl has had a 1 centimeter palpable lump in front of her right shoulder, arm pit area. We first noticed it around month 8. Dr. Suggested we keep an eye on it for changes. No change in size and consistent blue/green hue on skin where lump is. Two weeks ago had ultrasound and received note from dr. today that radiologist states it is an enlarged lymph node, and referral will be made to children s hospital for pediatric surgeon consult for baby. Plan on seeing surgeon as soon as possible but in meantime if you are able to provide any additional info on possible cause. The lump does not appear to be painful and baby is thriving with no fever, etc... Doctor: Hi...I understand your concern. I feel this is best diagnosed only after seeing directly. I suggest you to upload photographs of the same on this website, so that I can guide you scientifically. Hope my answer was helpful for you. I am happy to help any time. Further clarifications and consultations on Health care magic are welcome. If you do not have any clarifications, you can close the discussion and rate the answer. Wish your kid good health.Dr. Sumanth MBBS., DCH., DNB (Paed).,"
},
{
"id": 21918,
"tgt": "Can insect bite and wine intake cause atrial fibrillation?",
"src": "Patient: I have AF which has been treated successfully for 18months, over the last few weeks it has returned, many nights after dinner and a glass of wine. Tonight I skipped the wine but still went into AF. Could it be a reaction to some insect bites that I have been getting? Doctor: Hi thereThe AF which returns after getting successfully treated once is know as PAROXYSMAL AF. AF is commonly assosiated with advanced age, hypertension, valvular heart disease, CAD, CCF but it has no proven or documented relation with alcohol drinking. So it cant be the reason in your case.But what you need to do now is to visit your cardiologist, get an ecg done to confirm, and you need to be started on ANTICOAGULANTS as AF is a risk factor for clot formation in heart chambers which can later lead to Stroke or other devastating complications.Good Luck"
},
{
"id": 92851,
"tgt": "Lower abdominal pain, soreness, cough and sneezing extremely painful. What does this show?",
"src": "Patient: Hi doc, I have a terrible pain in my lower abdominal muscles. I hurt them as I continuously strained them before two months by playing. I also reckon not warming up properly. The pain is not normal soreness after a hard workout as it is ok when i cough or sneeze but when I shoot a football or try to change direction quickly from stationary position, it pains a lot. Doctor: Hi! Due to irregular exercise by an unprepared or non warmed up body, possibly there is a repeated tears of muscle fibers with accumulation of small amount of ooze at places of unsuspected trauma. You have to give sufficient rest for these to heal and have to have some anti inflammatory drug for some time so that the inflammation is gone and you are relieved. If this does not solve your problem, then you may have to go for an ultrasound of abdomen to rule out any other organic problem. Thanks and good luck."
},
{
"id": 216292,
"tgt": "How to reduce post operative pain?",
"src": "Patient: I had revision lumbar surgery L2 thru S1 on January 7th , 2014. The nerve pain has been resolved,however, I am experiencing severe post operative pain at S1 where the surgeon anchored screws into my pelvis. The surgeon wants me to stop using Hydromorphone, which I understand . I am not dependent on the medication, but it does resolve the pain. Therefore, what can I Take to help with the pain? What can I do physically to reduce pain other than physical therapy? What is the prognosis and success rate of this surgery? What, if any, are physical limitations ? Thank you. Doctor: Hi, I am a practicing anaesthesiologist and pain consultant.I have gone through your question. Its been 3 years now, after your surgery. If you were taking hydromorphone for such long duration it is better to stop it now, though you are not dependant. It is better to go for other safe pain killers like tramadol and paracetamol. I would like to suggest, you should go for repeat MRI/CT scan of spine to rule out any other possibility like nerve pain, loosening of the screw etcSurgical results of these surgeries are excellent if done by expert hands.Daily walking, good protein diet,wrm fomentation will also help a lot."
},
{
"id": 80003,
"tgt": "What causes shortness of breath after undergoing lung surgery?",
"src": "Patient: Hello, my husband just had surgery 7 weeks ago he had a paralyzed right diaphragm the dr. shored up the diaphragm and peeled the right lung. today he has more swelling than ever a little short of breath and still cannot do anything hard to lift even a gallon of juice his dr. keeps saying this is normal what do you think? his right lung is still folded over and has not inflated and when he breaths he can fill the diaphragm rubbing up and down. he also has something going on when he breaths he has some kind of sigh or something which he never had before. Doctor: HIWell come to HCMI really appreciate your concern, if this is the post surgical complication so this could be normal one if this is the likely to happens in such surgery and easily be managed but if this is not so then it may not be normal, as matter of fact such query needs complicate clinical examination and case study then and then it can be said anything perfect, better to have second opinion, hope this information helps, take care."
},
{
"id": 87674,
"tgt": "What is the treatment for severe abdominal pain and nausea?",
"src": "Patient: I had a D & C wednesday and went home started feeling better thursday. By Friday I was having severe abdominal pain and was very nauseated I went to the Emergency Room Twice. The first time they gave me iv fluids and morphine then sent me home. The second time the same but they did blood work and pushed on my stomach and a ct scan. I was told the blood work and ct scan came back fine but to follow up with my doctor on monday. Now once again I am in such pain i can hardly stand it and was running a temp of 102.3 but came down to 99.1 with Ibuprofen and it now burns when I urinate. There has to be an answer! Please help! Doctor: Hi,From history it seems that you might be having infection in your genital tract or urinary tract following D & C procedure giving fever and pain.You might require one course of antibiotic and anti-inflammatory medicine course for 5 days.Consult your Gynaec and get examined.Take plenty of water and liquids.Ok and take care."
},
{
"id": 193113,
"tgt": "What causes deposition of white substance between foreskin and glans?",
"src": "Patient: there is white deposition between foreskin n glans penis....its bit sticky.....white deposition is there when i skip bath for even one day otherwise its only sticky....it smells when i skip bath....i m taking alcohol regularly for 2years n i m 30yrs old....i took 4 fluconozol tablets n even applied hcl cream but its has only reduced the problem but did not altogether finished it.....i have no sexual contact for last 13 months.....n i got this problem 4 months back....is this any thing to do with HIV.... Doctor: Hi, The white normal deposition is smegma which is normal, if it is excessively deposited, it can be due to bacterial or fungal infection. Hope I have answered your question. Let me know if I can assist you further. Regards, Dr. S. R. Raveendran, Sexologist"
},
{
"id": 25243,
"tgt": "What causes multiple muscles spasms on the left side of chest?",
"src": "Patient: Hello am 23 years female , I have had multiple muscles spasms, on the left side of my chest and sometimes near the heart area for the last couple of days. The last a few seconds and the happen a couple of times a day. Is it something serious, what should I do????? Doctor: Hello and thank you for using HCM.I carefully read your question and I understand your concern.You shoud not worry about this . I will try to explain you something and give you my opinion. Typical heart chest pain have some characteristics. They usually are retro-sternal pains like squizzing , tightness or heaviness. They may irradiate to both arms, neck and jaw. They last from 5 to 20 minutes, might be associated with nausea, vomiting, tachycardia.In my opinion this isolated pains to left side near heart lasting for seconds are not related to heart problems. They might be muscular, sceletic or neurotic pains.The fact that you previously had muscular spasms fortifies my opinion that this might be a muscular problem.So, you should not worry about. This is nothing serious. Only if this symtoms persist and begin to bother you I would recomend some examination like an electrocardiogram, a cardiac echo and a strees test. This examinations can reassure you that its nothing serious about the heart.Hope I was helpful.Wish you good health. Best regards."
},
{
"id": 39848,
"tgt": "Suggest treatment for typhoid fever",
"src": "Patient: Good day,sirMy wife is suffering with typhoid fever at 99-99.5 degree for the last 1 month.he is taking medicines, but no recovery. temprature does not go to 98.4 from 99-99.5. doctor has not given any tablet for temprature like crocin.he said fever is due to infection.pl advice. Doctor: Hello,Welcome o HCM,As your wife is having fever for last one month it may be a fever of unknown origin (FUO) which should be thoroughly evaluated to find out the causes for the fever. As she is having for last one month all the relevant investigations should be done to rule out the cause of fever.As your doctor has prescribed paracetamol to reduce the fever, this drug will act for 6 hours and it will control the fever for this duration but it cannot cure the condition. The source of infection should be found out by the investigations.To confirm it has typhoid after having fever for last one month your wife should undergo stool and urine culture to confirm it. There are many conditions which can lead to fever of more than one month which should be evaluated thoroughly.Thank you."
},
{
"id": 93440,
"tgt": "Sever abdominal pain. Have Hepatitis C. On dilaudid. Is it a side effect?",
"src": "Patient: Ever since I was put on treatment for Hepatitis C I have had severe abdominal pain.. no pain before the treatment. had to go off the treatment as it caused severe side effects. that was app. 4 years ago and I still have the pain but doctors can't seem to explain why .. have been on Dilaudid for several years but do not want to continue taking any pain killers as I don;t like the side effects . I want to get off the pain killers and find a way of dealing with the pain or stopping it.. any solutions Doctor: you can go for cause finding instead of being in for pain etting fine by analgesicultrasoung abdomen to rule out gall bladder inflamayion and stonelier function yests to find out an gamagespecific anibiotics for common foods in blood to fing out an food causing this problem like milk wheat etcafter fing out caue and teat permanently instad of worrying for side effects of nalges"
},
{
"id": 131054,
"tgt": "What is the treatment for pain in bones and muscles?",
"src": "Patient: HELLO,AND THANK YOU.I HAVE HAD NECK PAIN AND SEVERE BONE AND MUSCLE PAIN FOR THE PAST 2 WEEKS NOW,I AM A CHRONIC PAIN PATIENT AND ON PAIN MEDS. ALSO.SHOULD I SEE MY G.P. FOR THIS ORMY PAIN MNGMT.DOC'S,WHICH ARE TOPS IN THE PHILA. AREA?THANK YOU BERNIE BRUCKS SPINAL STEONCIC PAT.FEEL FREE TO CALL ME ANYTIME.215-633-8231 Doctor: Hi,Get xray of cervical spine in AP and Lateral view done and send me the report.Till then have tab. aceclofenac+ thiocolchicoside twice a day, tab. pregabalin+ methylcobalamine at night, antacid in empty stomach.Apply hot packs.Definitely pain will go.If pain still persists after 7 days you can consult your doctorThanks.Dr CHANDER MOHAN SINGH."
},
{
"id": 78776,
"tgt": "Suggest treatment for cough and chest congestion",
"src": "Patient: I have had a cough for almost two weeks now. I noticed the chest congestion goes away after lying down and when I wake up it s gone until I have been up for 30 minutes. The congestion starts to come back and about mid day I have been getting sinus pressure and headaches. I have started to notice a tightness in my right jaw as well. Could this be a serious concern? Should I see a dr or is it just bronchitis that will go away soon? I have noticed a slight improvement in the chest congestion but the jaw pain has me worried Doctor: Hi. I understand your concerns.Cough of 2 weeks is commonly seen in bronchitis and lung infection. Get a chest x ray done, a normal rules out lung infection. Possibility of bronchitis is more in your case. So better to consult pulmonologist and get done clinical examination of respiratory system and PFT (Pulmonary Function Test). PFT is needed for the diagnosis of bronchitis. It will also tell you about severity of the disease and treatment of bronchitis is based on severity only. You may need inhaled bronchodilators and inhaled corticosteroid (ICS). Treating sinusitis is also very important as that may be also the cause for cough. You will need a detailed evaluation for the same."
},
{
"id": 103692,
"tgt": "Nose bleeds. On allergy medicine. Why does it happen in kids?",
"src": "Patient: My 4 year old daughter has nose bleeds all the time. It s so bad she will soak a small towel. They usually lasts for about 20 min. They can start at any time. She will just be sitting and playing and it will start. She has woken up to a bloody face because she had a nose bleed while she slept. It usually starts when she gets upset. It doesn t have to be a full blown tantrum either. I don t know what to do. Her dr just says kids have nose bleeds. She gave her some allergy medicine but it didn t work. What do I do/ Doctor: it s sinus or adenoid bled which is normal in many childrenget xray pns and waters viewxray stn for adenoidsaccordingly we can treatt present give antiallergic fexofenadineapply neomycin ointmt in nosebdsea water 2 drops at night each noseshewll feel good till tou get diadnosis"
},
{
"id": 18438,
"tgt": "What causes dizziness despite taking Amlodipine Besylate for high BP?",
"src": "Patient: I have been taking amlodipine besylate 2.5 grams for a year for high blood pressure.I am a healthy weight women with no other problems.In June of this year I had an epoxied of feeling faint and unsteady on my feet.It lasted for 4 days.I thought I was dehydrated.Now I am in Arizona for the winter and had this same thing happen 2 more times.Could my dosage be wrong or the blood pressure? I went to the doctor and have no infections so I am wondering if this could be the problem.I Have had problems with being dehydrated in the past so I drink tons of water.Being in Arizona I sweat more so that doesn t help my dehydrating problem.I am confused and my husband thinks is my thyroid.I had blood test before leaving home and all was well. Doctor: Hello and Welcome to \u2018Ask A Doctor\u2019 service. I have reviewed your query and here is my advice. Some time high dose of Amlodipin cause dizziness n also swelling in leg High dose of Amlodipin create problem in kidney. Due to high dose oxygen level goes down towards brain that's why you feel dizziness. Hope I have answered your query. Let me know if I can assist you further."
},
{
"id": 61567,
"tgt": "What causes a painless lump on the buttock?",
"src": "Patient: I have a lump in the middle of my left butt muscle. The limp is hard but surrounded by softer area that has clear fluid that comes out when squeezed. Alot of it. No pain. There is no discoloration of the skin and you can t see where it is without applying pressure to feel it. What could this be? Doctor: Hello and welcome to HCM.This may be benign subcutaneous lesions such as fibroma,ateroma or lipoma and you should see a surgeon who can then perform removal under local anesthesia and it can be sent to patohistologic analysis.Wish you good health. Regards"
},
{
"id": 211999,
"tgt": "Have problem with memory, low vitamin and celiac. No medication till now. What to do for this?",
"src": "Patient: female aged 60 years is a celiac and low in vitiam d not exactly memory loss but seems to have. problem with time keeping and always running late for everything in recent times. and seems to not have any moviation for anything from getting up in the mornings to getting prepared for leaving to attend things ie church, work ect Doctor: DearWe understand your concernsYou are not the only person who complains about memory and time management. We all are like that. Hard pressed for time and therefore hurry and struggle and therefore forgets so many things and complains that we have some memory problem.We forget so many things when we are busy. This happens even in childhood and adolescent and adulthood. Just try to remember. We didnot worry then. Why worry now? Because we are afraid? Natural. Just learn relaxation and time management technique from a psychologist. I can help you if you give all the detaills through \"Ask a specialist\" link of this website.Wish you speedy recovery. God bless you."
},
{
"id": 208855,
"tgt": "What can be done if my child is not talking to any one?",
"src": "Patient: grief of my child on issue of name change due to teasing of college friends and lecturers & we tried to change the name through court formalities but we came to know that as per Govt G.O name change will be done only alternation , they should not change the full name my daughter from two months she is not talking to any one neither family members nor out side please suggest me Jayasree Doctor: DearWe understand your concerns.I went through your details. We all know \"what is there in name?\". Such thought itself is childish. Also your child is being childish now. That is normal. Instead of pacifying her and convince her, you did wrong by trying to change name. No problem. Taking her to a psychological counselor shall surely help. Only that shall help, because you already tried whatever you can. Now an approach psychologically can only change her. Please do not ignore.Hope this answers your query. Available for further clarifications. Good luck."
},
{
"id": 97489,
"tgt": "Suggest natural remedy for sound sleep",
"src": "Patient: I AM 31 YEARS OLD WEIGHT 49 KG AND HEIGHT 5 FT...OFFLATE I HAVE PROBLEM HAVING SOUND SLEEP...I GET UP REALLY EAR;Y DREAMING ABOUT VARIOUS THINGS AND THEN DONT FALL ASLEEP AGAIN...HAVE STARTED FEELING TIRED VERY SOON AND IRRITATED TOO.ANY HOME REMEDY FOR SOUND SLEEP Doctor: Hi Welcome to HCMThis is Dr Suchda Naturopath - Homeo path - MagnetoTherapist Taking care ofyour query.Not having sound sleep is very common problem and it is very natural feeling tired , listless & irritated .and there are various reasons to lead to this situation I would like to know about your over all health ? If you take nourishing diet & take lot of waterDo sufficient exercise Have no constipation .No HBPiF YES You need not worry about any thing 1.Take 1 cup of hot milk with 1/2 spoon of turmeric in it.1/2 hour before bedtime.2.Take Nux Vomica 30 1 dose at bedtime for 7 days3.Do Bhramari - a pranayam postureDon't take unnecessary tentions .All this will help you to get sound sleep Don't hesitate to comeback if have any further queryAll the best"
},
{
"id": 67995,
"tgt": "What causes painful and swelling lump under armpit?",
"src": "Patient: i am 37 years old, 5 3 and weigh approx. 150lbs. I have a lump in my armpit. I have had it for a very very very long time. It usually is very small and doesnt hurt, about the worst usually is it kinda gets in the way of shaving. lol. But twice now (starting two days ago being the 2nd time) it suddenly swells very large and hurts pretty bad plus it slowly leaks this fluid that is kinda like snot. Im not sure if it that stuff that smells bad, or the fact that its hard to actually clean the area during this and i just have bad b.o. i m afraid its something bad....what should i do? or what the hell is it?? Doctor: Hi, dear. I have gone through your question. I can understand your concern. You may have enlarged axillary lymphnode due to tuberculosis or rractive enlarged or malignancy. You should go for fine needle aspiration cytolog for exact diagnosis. Then take treatment accordingly. Hope I have answered your question, if you have doubt then I will be happy to answer. Thanks for using health care magic. Wish you a very good health."
},
{
"id": 103154,
"tgt": "Pain while swallowing, looked like strep, prescribed Penicillin. Palms red and covered with small bumps. Allergy ?",
"src": "Patient: I saw a doctor two days ago due to painful swallowing. I had no other ailments, no congestion, runny nose, or sore throat, simply painful swallowing. The doctor checked my throat and said it looked like strep. I was prescribed penicillin, which i have been taking twice daily since. This morning, I noticed the palms of my hands were very red and covered in small bumps. What could this be? I have had strep before and was prescribed penicillin every time to treat it. Could this be a newly developed allergy, or a rash due to the strep throat? Thank you for your time. Doctor: Hi,Thank you for asking question on health care magicrash is a part of strep. sore throatnoseparate treatment except treatment for strep. infectionHope this answer will serve your purposeTake careDr.M.V.Subrahmanyam MD;DCHAssociate professor of pediatrics"
},
{
"id": 39786,
"tgt": "What precautions need to be followed after dog bite?",
"src": "Patient: hello doctor, i am hiren from gujarat, india... before 10 days a street dog bites me. i have taken rabipur vaccine within an hour of that incident. then after 3 days and 7 days i have taken another two rabipur injections. still two injections are remaining. what types of precautions i should follow. please tell me. Doctor: Dear Friend.Welcome to HCM. I am Dr Anshul Varshney. I have read your query in detail. I understand your concern.Since it has been 10 days and you are on Vaccination schedule, no special precaution you need to take now.In general, if a dog bites following things need to be done:1. To clean the area with lot of water and soap.2. To get wound dressing done3. To take Rabies vaccine4. If high risk bite- Immunoglobulins should be takenCurrently, i would advise you just to complete the vaccination course.This is my best advice for you based on available details. If you have any further query please ask me.Stay Healthy.Dr Anshul Varshney, MD"
},
{
"id": 172881,
"tgt": "Need treatment for red raised rashes on both inner thighs",
"src": "Patient: My 3 yr old son has a red blotchy raised rash on both inner thighs after getting out of the pool today. He had a swimmer and a bathing suit on as well as a life jacket for toddlers that snaps between the legs. What can I use to put on this rash to treat it? Doctor: Hi,I understand your concern ,it seems allergic reaction to chlorine water. I suggest Cetirizine syrup,betamethasone ointment for 3 days.Next time apply oil to protect skin and hair from chlorine water, take shower before and after pool. After pool apply Jonson's cream. Hope this answers your question. If you have additional questions or follow up questions then please do not hesitate in writing to us. I will be happy to answer your questions. Wishing you and son good health."
},
{
"id": 95992,
"tgt": "i am suffering from severe pain and nausea during periods",
"src": "Patient: HI, I AM A 28 YO FEMALE WITH FIBRO AND RA, STARTED WITH PERIODS OF SEVERE NAUSEA ALTERNATING WITH PERIODS OF EXTREME HUNGER FOR 4 DAYS, UPPER ABDOMINAL PAIN STARTED 2 DAYS AGO, VERY BLOATED AND BURPY, AND CONSTIPATED, VOMITTED WHITE FOAMY SUBSTANCE ONCE, TAKING PEPTO AND PEPCID OTC NOT WORKING...ALSO TAKE ASPIRIN AND TYLENOL FOR ARTHRITS...NO INSURANCE WHAT CAN I DO AT HOME? TO SEE A DR I HAVE TO GO TO ER... Doctor: do you mean fibromyalgia?with RA?,then 2nd one is more ecnomic burden for any1 as it runes long 1c comes,if you have fibromyalgia along with RA, at least you should keep strict suprivison on you diet by ensuring it contains rt amount of protein along with fibre indiet. as far as drugs, u take cap omez-d20 along with syp gelucilmps 2to3tsp on empty stomach,followed by cap TRAMAZAC50 1 OR 2(MAX) AT NIGHT&TAB BURLGAN IN DAY TIME to relieve pain,for more,consult physican nearby.worry never solve problem,rather it make you more painful to worry abt pain itself.godbless you,best of luck."
},
{
"id": 27897,
"tgt": "Should angioplasty medication be continued due to uneasiness and sleeplessness?",
"src": "Patient: I had angioplasty in September 2007. I used to take Aspirin and other drugs continuously. Since I feel better and normal. I stopped taking tablets.However I feel little uneasy and loss sleep. Doctor could you please advice whether I should continue the medicine? Doctor: It's not a good idea to discontinue such medication without discussing with your doctor first. You don't say what else you are receiving besides the aspirin, but I suspect they are protective drugs for your heart. Aspirin acts as a blood thinner and a small daily dose is strongly recommended in people who have had stents or heart attacks. You do have underlying heart disease which can continue to progress, even without symptoms for a long time. I recommend you talk to your cardiologist and get back on your medicines unless s/he tells you something different, Hope this helps."
},
{
"id": 92795,
"tgt": "Had lathroscopic hysterectomy. Total hysterectomy with rectocele correction near rectum. Soreness in abdomen. On Advil. Suggestions?",
"src": "Patient: Just home from robotic lathroscopic hysterectomy. Total hysterectomy with rectocele correction and removal of skin tag near rectum. Experiencing a lot of soreness yet in the abdomen, Very sore and burning rectal area. Trying to keep on top of this with Advil (2 every four to six hours) /been sleeping in recliner. Hurts to sit on hard surface.How long does it take for the abdominal/bloat/gas to dissapate? With the rectal area stitches, I feel a constant burning in the vaginal/rectal area. Is the rectocele repair part of this discomfort. Felt pretty good in the hospital with the IV pain drugs and then had trouble with hydrocodone and constipation. Switched to Advil only.'Help??? Doctor: Hi, you had robotic laproscopic total hysterectomy. There will be so many complicated stitches and contractions, you may have all these for nearly 10 days, if it exceeds you consult your surgeon for follow up. It may also be due to anxiety and tension. i to my patient with such symptoms as usually happen to occur prescribe alprazolam .5 mg Hs for few days. And continue the antibiotic for few more days. Thank you."
},
{
"id": 156893,
"tgt": "Does stem cell treatment cure oral cancer?",
"src": "Patient: Hi Doctor, My brother in law have the oral cancer past few years, we done operate two times, now we found the cancer will form again, on his throat. There is any possible to complete recovery for him, more over we need to more about the stem cell treatment will help this kind of cancer treatment Doctor: HIThank for asking to HCMI really appreciate your concern for your brother, early detection of malignancy gives excellent result if your brother's disease detected early, then no need to worry about this, proper treatment will give good results, of course this could take some time, hope this information helps you have good day."
},
{
"id": 55440,
"tgt": "What causes fatty liver disease symptoms for non-alcoholic ?",
"src": "Patient: hii doctor,my husband is suffering from fever ,loose stool and continuous vomiting from wednesday onwrds..his sgot and sgpt,bilirubin levels are still normal..he went through clinical examinatin yesterday in a local clinic in our city itself,as per clinical symptoms doctor diaganozied as fatty liver disease..my husband is not a chronic alchaholic person,but he regularly uses beer..still ne is sick..i am actually confused about his symptoms...but he couldnt eat anything and the vomitus is yellowish clear fluid..can u give me a proper answer pls? Doctor: Hi,Thanks for writing in.There are many causes of liver showing fatty changes (i) Increased intake of fatty foods and alcohol(ii) Over weight and obese patients.(iii) Those who have fatty liver running in family members.(iv) Any liver condition that is changing appearance of liver.(v) Mild fatty liver can also be there in many normal peopleYou can get his liver function test and lipid profile done to know if the liver enzymes are raised and cholesterol numbers are increased. Please discuss results with your doctor and take required treatment if your tests results are higher than normal.Total calories in food should be 2200 kcal and low fat food is suggested with decrease in amount of beer. Please do not worry."
},
{
"id": 157883,
"tgt": "After MRI for suspected cancer spread jaws were locked and foaming at mouth. Died after lapse of heartbeat",
"src": "Patient: A friend of mine went for an MRI to determine if cancer had spread to the brain. After he received the injection the medical team did not realize his jaws were locked and he was foaming at the mouth. He was at an imaging center & had to call for ambulance which for some reason took approx 15 min. They didn't even begin CPR until the ambulance arrived. He had no heartbeat or pulse but when they got him to hospital they were able to get his heart started again but he never regained consciousness and died 8 hours later. What could have gone wrong? Doctor: Hi, you have not mentioned about the type of cancer your friend had? because the spread of cancer to brain depends on the site and type of cancer. the injection I.e the contrast agent used for MRI scanning is unlikely(but very rare) to cause the symptom which appears to me with the above description as a convulsion. the convulsions may also be caused by a spread of cancer to brain. the only caution should have been to start the patient on anticonvulsants for the patients who are at risk for the same. The convulsion may also lead to death. Hope i have answered your question."
},
{
"id": 156423,
"tgt": "Will eating right help in chemotherapy?",
"src": "Patient: My friend has cancer and she is not eating right. She has had radiation and now she is getting ready to do chemo. Can eating right help? She is going to need plenty of energy. I ll just ask my Dr. For free thanks but no thanks. It s a price for everything even a question about someone s life. Sorry didn t mean to inquire. Doctor: Hi. Eating a healthy diet definitely help patients. it keeps body energetic and boost immunity which is otherwise compromised due to effects of chemotherapy. Patients most of the time do not feel like eating or have nausea or vomiting. Patient should be encouraged to take small but frequent meals containing nutritious foods.Hope I have answered your query. If you have any further questions I will be happy to help."
},
{
"id": 53298,
"tgt": "Suggest medication for raised SGPT levels",
"src": "Patient: i am 44year old and my weight is 82kg i was affected by heptits c in 2007 and awent through intefetron on alternate day along with robrivion.the result cameas not detct of virus but at end of thropy my SGPT was 78.again in 2010 virus became positve and in quantitative PCR the number of virus was 598452 and SGPT was 128 the physician conceltated for pege interfron (begasus) nowat the copmliation of course in quantitve PCR the number of virus is less than 30.i am worried about my SGPT which is 80.i am non alcholic ,i am daibaticwhat should do about raised SGPT?kindely suggest any medicine because i belong to rural and backward area of the state. Doctor: Hi and welcome to Healthcaremagic. Thank you for your query. I am Dr. Rommstein, I understand your concerns and I will try to help you as much as I can.ALT (SGPT) may indicate viral hepatitis or other types of liver damage. So the next step is to look for certain liver damage and most common casue is fatty liver disease. It should be confirmed by ultrasound. Other causes are viral hepatitis,cirrhosis, medications, alcohol intake or autoimmune diseases. To verify exact cause, US, bilirubin levels and tumor markers should be additionaly done Then appropriate treatment can be started.I hope I have answered you query. If you have any further questions you can contact us in every time.Kindly regards. Wish you a good health."
},
{
"id": 32830,
"tgt": "Suggest medication for fever",
"src": "Patient: Hi. I'm having a fever which has lasted for 3 days as of now, it doesn't seems to subside with Paracetamol, temperature is about 37.5 degree Celsius. Besides, I feel very tired and sleepy even though I have just woke up from a nap. The doctor has took some blood for Full Blood Count test, but the result is not out yet. Is it necessarily to go to the hospital, or what should I do? Please help. Doctor: Hello, I really appreciate your concern. From your description I think you are suffering from viral fever. First of all, 37.5 degree Celsius temperature is within normal range for an adult. So there is nothing to worry about it. Your temperature is in control. Tiredness and sleepiness is because of viral fever itself. You should drink plenty of water. Fresh fruits and juice are advisable. Have adequate rest and diet. There may be some feeling of anorexia. Try to divide your meals in small fractions to have enough nutrition. Further management if any needed, will be initiated by your physician after results of blood reports. Hope my answer has solved your query. Take Care.Thank you."
},
{
"id": 12567,
"tgt": "Psoriasis, broken skin under feet, pain. Taking tricort-40 injections. Cure for psoriasis?",
"src": "Patient: my son aged 12 years suffering from psoriasis for more then 3 years, skin under feet is becomes very fragile and broken paining heavily. we were consulted one homeopathic doctor but he suggested tricort-40 injection every month for next 6 months. its been more then 3 months we had given. but when I search on internet for tricot injection, I see there are lot of side effects. I just pledge you to suggest best process or why to cure his psoriasis. Doctor: Hi, There are very good drugs other than tricort for psoriasis. Steroids are not the treatment for psoriasis.On stoppping tricort psoriasis may increase suddenly. Please consult a good dermatologist for treatment of psoriasis. Discuss the option of Methotrexate and NBUVB phototherapy for psoriasis. Cyclosporine is also a very good option. Hope this helps. I will be available for follow up. Regards. DrSudarshan"
},
{
"id": 196330,
"tgt": "What causes oozing of ejaculate with urine during chicken pox bout?",
"src": "Patient: hello sir...i am 23year male..before one week i suffered from chicken pox.During that night time,i found sperm coming along with urine..this is not my usual case of sperm coming with urine.i want your answer sir.but still after 1week its continuing.i need your help..advance thanks sir.. Doctor: Dear userWe understand your concernsI went through your details. Please do not panic. This is quite natural because the oozing sperm was already inside you. The stored sperm is always a waste product for your body. So the body is removing the waste product. Please do not worry about it.If you require more of my help in this aspect, please use this URL. http://goo.gl/aYW2pR. Make sure that you include every minute details possible. Hope this answers your query. Available for further clarifications.Good luck."
},
{
"id": 13260,
"tgt": "Suggest treatment for itchy rash on face & neck",
"src": "Patient: Hi i have had a itchy rash that started on my face and neck.Now it seems to be all over and it itches more in the evening.Also have little bit of hair falling out everyday.Have headaches at times and almost like the flu.Little bit of sore throat and feels like mucus draining in my throat.Ears also have water in them constantly. Doctor: Hi Dear,Understanding your concern. As per your query you have symptoms of itchy rash on face and neck which can occur due to our environment factors, bodies response to the trigger factor and due to certain systemic causes.Need not to worry. I would suggest you give cold compresses to rash several times a day to prevent from swelling and itching and take over the counter antihistamine drug like Benadryl. If symptoms still persist the consulting dermatologist for proper examination and treatment. The doctor may perform skin test and try to find out the trigger factor and prescribe treatment accordingly.Hope your concern has been resolved.Best Wishes,Dr. Harry Maheshwari"
},
{
"id": 87872,
"tgt": "What is the cause of stomach pain, headaches and dizziness with yellow blobs in vision?",
"src": "Patient: Hi, I was wondering if you could help? I ve been unwell this past week constantly feeling sick with stomach pains and headaches, I now have these stomach pains and head aches all day my head feels really heavy, I to dizzy and I keep seeing yellow spots that take up bear enough my hole vision I do wear glasses have done since a toddler but never had anything like this. I ll be going the doctors in the morning just looking for some more insight? Doctor: Hi.Thanks for your query and an elucidate history. Yes, we can always help you if you provide the full and detailed history. Since you are not well for more than a week, have sickness, headache and stomach pains with yellow blobs in vision, I would advise you the following:Get blood tests for CBC, Widal for typhoid, blood culture and sensitivity.; urine and stool tests. Anyways, you are going to the Doctor, get vitals like pulse, BP and temperature, abdomen for liver and spleen checked.,This will give us an idea about the disease you are having .You also need to be seen and examined by an Ophthalmologist.Proper diagnosis will give us the perfect plan for treatment. Till then have bed-rest, plenty of fluids orally."
},
{
"id": 66366,
"tgt": "What to do if having lump, tightness in the upper thigh?",
"src": "Patient: My son is a track runner and he has been having trouble with the upper part of his leg/thigh. It has been hurting for some time now. It feels really tight and feels like it has a lump inside. Is there something in the upper part of this leg that could be torn or maybe just strain? What can he do to make this feel better so he can complete his season without so much pain. He has been icing and heating Doctor: Hi, thanks for sharing your son's health concerns with HCM! If I were his treating Doctor for this case of the movable lump/tightness in the upper thigh, I would come up with three possibilities, these include: 1.\u00a0\u00a0\u00a0\u00a0\u00a0a lipoma or benign fatty tumor or neurofibroma2.\u00a0\u00a0\u00a0\u00a0\u00a0The second possibility is of a benign cyst like ganglion cyst bursa\u00a0\u00a0\u00a0\u00a0\u00a03.\u00a0\u00a0\u00a0\u00a0\u00a0The last possibility is of a lymph node or even could be some normal / developmental nodule like muscular nodule or just kind of muscular sprain!\u00a0\u00a0\u00a0\u00a0\u00a0Overall, it is benign and not to worry about this but you could go for FNAC test for confirmation before consulting a surgeon in your area! He might have rest and continue physiotherapy as you mentioned....Hope this answers your question. If you have additional questions or follow up questions then please do not hesitate in writing to us. I will be happy to answer your questions. Wishing your son good health."
},
{
"id": 171645,
"tgt": "What could cause spreading discoloration below stomach with bedwetting problem?",
"src": "Patient: i have a 8yr old biracial daughter that hass dicoloration it started on her stomach below belly button and had a weird shape to it. then it started to spread to her vaginal area now its spread to inter theighs. please i need answers. im a very scared mother over this. she also has had alot of differnet female problems. she has no sinsation when she has to urinate til last minute. so wwe have lots of accidents. also wets bed every nite. no one is giving me answers. they thought she was constipated. this is not the case. PLEASE WE NEED HELP Doctor: Hi ,may be we can put all symptoms to one cause.She has contipation and nocturia .there is a condition called celiac disease which can start with a rash,that keeps increasing in size .at 8 year of age its unclear what female problems she can have.You have to do some special tests in her blood ,in a specialised centre with doctors from many specialities specially like endocrine,deramtologist and who will surely help .Keeping away from food containing gluten (bread) may also help tiill you get your appointments"
},
{
"id": 72007,
"tgt": "Could lung problems cause vomiting of blood?",
"src": "Patient: My Aunt called my mother this morning, asking her if it was possible to \"Take my husband to the hospital, since he's vomiting blood.\" My mother agreed and was on her way, when my aunt called and told her that he had stopped and didn't wish to go anymore. He's been smoking for over 45 years, and has less than 1/4 a lung left. Does any of this tie in together? Should he see a doctor ASAP? He's 68 years old. Doctor: Thanks for your question on Healthcare Magic.I can understand your concern. Blood in vomiting is known as hemetemesis. It is often confused with hemoptysis (blood in sputum).Since that man is smoker and having lung disease, possibility of hemoptysis is more likely. In any case (hemetemesis or hemoptysis), it is better to visit doctor as soon as possible.If bleeding is severe and continuous, patient can die due to hemorrhagic shock. So better to take him to hospital. Hope I have solved your query. I will be happy to help you further. Wishing good health to that man. Thanks."
},
{
"id": 157340,
"tgt": "Elevated D dimer, lymphadanopathy, large varicose vein, no clots. Thyroidectomy done. History of thyroid cancer. Cause of concern?",
"src": "Patient: Hi I had a slightly elevated d dimer of 245 normal range was up to 235 I have had thyroid cancer but had a thyroidectomy , I have been experiencing some lymphadanopathy I also have MS, i had pain in my leg and was told via doppler I have a rather large deep varicose vein but no clot. Should I be concerned with this result? Doctor: elevated d dimer and enlarged varicose veins are of no concern but your lymphadenopathy requires evaluation based on certain characteristics like size, location, duration of their existence, consistency etc. if you think they are increasing in size, it would be better if you see your doctor and get evaluated. Also important is regular follow up for thyroid malignancy"
},
{
"id": 205320,
"tgt": "How to deal with anxiety and suicidal thoughts?",
"src": "Patient: I have been suffering from anxiety attacks and suicidal thoughts. However, these attacks only really seem to arise when I conversate with my boyfriend, who quickly reminds me that I am an annoyance and an irritation. Those and other negative comments always make my anxiety attacks worse and I don t know how to stop them. This partner makes me feel like it s my fault and that I am making his life more stressful and unbearable and that I shouldn t be sharing these anxiety type thoughts with him especially when I talk about numbing myself with medication. I think I do this to get his attention but it is only backfiring and making me feel extremely guilty and thus increasing my anxiety. I have tried to get out and talk with other friends but I end up feeling guilty and ashamed again because it may be construed as dating or gossiping. I am not sure if I have provided enough information or if this is more a psychological rather than medical issue but I need to talk to someone who will not call me crazy. Doctor: in my opinion you are undeegoing a stressful relationship. as you rightly said you attimes do things to attract the attention of your husband but it goes otherwise and he get more irritated with you . you are trapped in a vicious cycle where you are unable to find a way out. i would recommend you to go to a therapist who can do analysis of your situation more objectively and help you to overcome the situation."
},
{
"id": 162102,
"tgt": "What should be given to an infant for ear infection causing fever?",
"src": "Patient: Hi ...I have a 16 month old with an ear infection, nausea/vomiting, fever (100.9 - 104.9) was seen on the 1/19 by Dr. was prescribed Augmentin 250mg TIB. Tylenol allergy also allergic reaction to flu vaccine. Administering ibuprophen Q4-6 hrs. refusing fluids. Some glanular swelling in odd places of the neck...I m assuming glandular??? I feel the inflammation is to low on the sides of the neck...they are not fixated..feel more cyst like... any suppositories OTC for the nausea/fever? Doctor: Hi, From the history, it seems your child has upper respiratory tract infection. The glandular swelling is probably reactive lymphadenitis, which settles as infection subsides. As you have already started on oral antibiotics, continue and complete the course as advised by your pediatrician. If it is a bacterial infection, it should respond to antibiotics. If viral infection, can give brufen every 8th hourly or meftal if not reponding. Consult your pediatrician before starting this drugs. Tepid sponging with warm water may help little bit. Keep your child well hydrated, by giving plenty of oral fluids. Hope I have answered your query. Let me know if I can assist you further. Regards, Dr. Nirubhan Bharathy, Pediatrician"
},
{
"id": 47525,
"tgt": "Suggest treatment for pain in kidney area radiating to abdomen",
"src": "Patient: Im feeling very sick since i took excedrin extra strengh rapid release caplets and i just found out the y recall the medication along with some others, im having a sharp pain in the right side of my back where is the kidney i guess, the pain travel to the side of my belly and legg, pls i need an asvice, Lisa Doctor: HelloRight sided abdominal pain may be due to many causes like problem in liver or gall bladder,right kidney calculus,musculoskeletal causes etc.You may need clinical correlation and further investigations.Investigation include routine hemogram,liver and renal function test,urine RE/ME,ultrasound of abdomen.Ultrasound of abdomen can exclude many conditions.Proper treatment depend upon findings.Get well soon.Take CareDr.Indu Bhushan"
},
{
"id": 206135,
"tgt": "Suggest remedy to overcome insecurity and negative feelings towards others",
"src": "Patient: Hello Doctor,My brother is 23 yrs old and his behavior is a little different for the past one year. He is very suspicious of the people around him, and he doesn't seem to trust anyone. Not even my mom, dad or me. He constantly feels insecure about himself and he has a negative view of the world. He says that it's hard to survive in a world where people are always trying to put others down. He always thinks that everybody is acting weird around him while it's only him who misunderstands people. We asked him several times to come to a psychologist. But he refuses to go. Can you please tell me how to convince him to get him to a psychologist? Thanks!3 Doctor: DearWe understand your concernsI went through your details. I suggest you not to worry much. From the description given, I see a case of paranoid personality disorder / paranoid schizophrenia. Could be the outcome of possible negative thoughts and worthless feelings. You may need psychiatric intervention instead of psychologist help. Your brother may be hard to convince. But he will definitely be having some good friends, whom he listens. Find them and convince him through them, that he needs treatment.Psychotherapy techniques should suit your requirement. If you require more of my help in this aspect, Please post a direct question to me in this URL. http://goo.gl/aYW2pR. Make sure that you include every minute details possible. I shall prescribe the needed psychotherapy techniques.Hope this answers your query. Available for further clarifications.Good luck."
},
{
"id": 134633,
"tgt": "Suggest remedy for pain in hip joint with broken ribs due to fall",
"src": "Patient: Hello My name is Fran Green.My husband fell off the top of our camper 9 weeks ago .He spent three days in Hosp.They said he had 4 broken ribs collapsed L. lung and a hairline crack in his lower spine. His back is still bothering him real bad but he says the pain is coming from his right side near his hip joint. Doctor: kindly meet a physical therapist and get assessed. a graded gradual exercise plan will be of help. ribs needs attention which coughing and deep breathing. back Needs slow strengthening program. meantime you can see a physical therapist for an exercise prescription."
},
{
"id": 134205,
"tgt": "What causes pain after total knee replacement?",
"src": "Patient: Hello, I had a TKR 6.5 years ago with no problems whatsoever until about a month ago when I stood up and it hurt so much I couldn t put my weight on it. This has happened 4 more times. Always in one very specific point on my knee. Does this sound like glue breakdown or loose prosthesis? Thank you. Susan Doctor: hi,thanks for providing the brief history of you.as you mentioned you have undergone the TKR almost 6.5 years back and now giving you pain at one particular place in knee.what i recommend you is to first take hot water fermentation and relax the muscles first which has got into spasm.post that try continuing the exercises which you have performed in the past, post knee replacement.Also to mention that certain times weakness of muscles leads to pain symptoms as repetitive pressure on the muscular tissue.you can also take one x-ray just to remove the doubt if the prosthesis is dislocated. where as per me chances are less.Also try performing exercises at your own and get required strength in the muscles of the knee, hip and core .If you fine there is no Change then you need to take again a physical therapy session to improve the strength in your muscles. As guidance always give precision.I have seen people complaining me about the pain post TKR but only thing I have come across the reason is the patient have stopped doing any exercise which has lead to again the symptoms.So I Advice them again the physical therapy and intense exercise which have made a 100% difference. they come back to functionality again.I hope to hear from you soon and you will be seen partying again.regards"
},
{
"id": 43557,
"tgt": "Stopped taking ginette for PCOS due to side effects. Taking thyronorm. Will PCOS cause infertility?",
"src": "Patient: I have Polycystic ovaries. So doctor prescribed be Ginette35 tablets. I took that tablets for 3 months. Since the side effects(like breast pain) is more Doctor prescribed me novelon tablet. I took novelon 1 cycle. Also my TSH level is 5.7. so having Thyronorm 25mg. Body pain is more. I feel I am so weak. I am very lean type. height is 163cms, weight is 46kg. Please tell me whethe cyst is dangerous or It will cause infertility? Doctor: Hi, PCOD is a multi-symptom disease that causes irregular or absent ovulation accompanied by multiple follicular cysts of the ovary, obesity, hirsutism, acne, insulin resistance and infertility. It is basically a disturbed pattern of the hypothalamo-pituitary-ovarian axis. It has a very good response to proper management. As your cycles are steadied you may be induced for ovulation with proper monitoring and this would give you good results. You may need Metformin also along with induction; tubal patency also needs to be tested before initiating treatment. Take folic acid. Please be positive as PCOD has a very good response to treatment and follow your doctor's instructions. Good luck."
},
{
"id": 52224,
"tgt": "Is nausea, tiredness, frequent urination and breast tenderness due to being positive on HPT ?",
"src": "Patient: i Had positive hpt nausea tired dizzy frequent urination super breast tenderness with some fluid. spotting lmp dec 25 2011 and u/s showed endometrial linin 1.6 cm plus a huge luteum cyst on one ovary andbilateral multiple cysts in the other plus a very small quantity of fluid in cul de sac.. What is this? Doctor: Hi, You are suffering from a disease that cause multiple cysts in the body. It is a hereditary disease and if you search quite well, you will find others in the liver and kidneys. Hope I have answered your query. Let me know if I can assist you further. Regards, Dr. Salah Saad Shoman, Internal Medicine Specialist"
},
{
"id": 107567,
"tgt": "What causes back pain after having liposuction?",
"src": "Patient: Hi, may I answer your health queries right now ? Please type your query here... Hello , I had tummy and back, liposuction about 2 months ago, I have healed well bit my lower back hurts so much that I wonder why its taking too lo g for it to heal. Thanks Doctor: Hi there.There could be fat necrosis or death of some remaining fat tissue which can cause the pain. Also clots can cause the same. Take pain killers for relief. Apply hot bag/ Consult your Surgeon for a thorough examination and appropriate management."
},
{
"id": 124890,
"tgt": "What causes tingling & slight numbness on right leg?",
"src": "Patient: I am currently experiencing tingling and slight numbness in my right leg for the past two days...I am about 3 weeks into a regiment of potassium 10MEQ ER tabs twice a day and 1 tab of hydroclorothiazide a day for an inner ear hyperacusis problem...I am wondering if the meds are playing a role in the numbness..? Doctor: Hi, It could be due to a side effect of the medicines you are taking for hyperacusis. Kindly Get back to the doctor who prescribed these medicines. Hope I have answered your query. Let me know if I can assist you further. Regards, Dr. Nirmal Chander Gupta, Orthopaedic Surgeon"
},
{
"id": 211735,
"tgt": "Feeling of nerves bursting, crying, confused, depression. Help",
"src": "Patient: Hi my name is Darla i was on my meds for years and my doctor took another job so i couldnt find me another doctor to talk to i need a doctor help me i have so many issues i don't even know when to start but i thought i could get by this year without nothing but it back fire where i dont have my meds or just someone to talk to and things need to have so isues i cry all the time and ,my nerves feels like i am gonna burst i try to kepp it coll as they say but more and more kepp get on top of me like i feel like can breathi just know wht to do or where to find the help i need Doctor: Hello,welcome to Healthcare Magic.From your symptoms of nerves bursting, crying, confused and low mood it appear that you are suffering from chronic depression. This illness need long term treatment under close supervision of psychiatrist. In many cases symptoms recur on stoppage of treatment.It is better to meet psychiatrist and start treatment accordingly, as medicines are prescription medicines. For long term control following simple measures are also helpful: relaxation exercise and non drug treatment like CBT.Wish you good health and all the best.Regards,Dr Ashish Kumar Mittalwww.99doctor.com"
},
{
"id": 81542,
"tgt": "What causes weird feeling in chest radiating to upper back and painful breathing?",
"src": "Patient: I woke up yesterday with this weird feeling in my chest that extends to my upper back. It hurts to take deep breaths but I have absolutely no cough or a wheezing feeling in my lungs. It s almost as if my chest muscles have been strained but nothing comes to mind how that may have occurred. I also have quite the persistent headache extending over the top of my head from ear to ear, Doctor: HIWell come to HCMTwo causes of these symptoms are very likely one is hyperaciditity and second is anxiety and depression, some time these both could be altogether and these need to be treated at once and the best drugs would be \"Tab Omeprazole 40 mg once in day at evening time, Tab Domperidone 30 mg once in day, Tab Chlordiazepoxide once in day, try to keep the stress level very low,everything would be fine soon, take care, have a nice day"
},
{
"id": 140490,
"tgt": "How do steroid injections help treat bulging disc?",
"src": "Patient: My arms & hands have been going numb so my PCP ordered a CT Cervical Spine w/Contrast. Results are At C5-6, mild to moderate cervical spondylotic disc bulging results in mild central canal stenosis and mild to moderate bilateral neuroforaminal narrowing. At C6-7, mild to moderate spondylotic disc bulging and a left paracentral and foraminal disc protrusion. There is remodeling of the left paramedian cord but without overt impingement. Moderate left-sided neuroforaminal narrowing. My PCP is wanting to send me to a pain clinic for injections. How will injections help a bulging disc? What else can be done besides surgery? Doctor: Hi, My assumption is that you're complaining of pain symptoms that your primary doesn't feel he can effectively treat... or you're not responding to the treatment he feels comfortable giving and that's why he's sending you for injections. Personally, I wouldn't go that route as a neurologist because I know there are other things that can still be tried before going to a pain clinic.....if pain is even an issue (which you've not really presented thus far that it is....). I would consider sending you for an EMG/NCV study to more precisely to identify which of those cervical lesions are really the important ones. Just because you see it doesn't mean it's relevant to your symptoms. You may also have something called thoracic outlet syndrome and not cervical radiculopathy although imaging is compelling for at least some degree of cervical pathology causing or majorly contributing to your symptoms. Hope I have answered your query. Let me know if I can assist you further. Regards, Dr. Dariush Saghafi, Neurologist"
},
{
"id": 107789,
"tgt": "Suggest treatment for severe middle back pain",
"src": "Patient: Hello, A couple of nights ago i went to sleep just fine when I woke up my sides middle back and overall midsection was hurting. I thought maybe I slept wrong or something but it hasn t eased up. It s tender to touch and hurts when I twist or turn or even lean back. Doctor: You haven't mentioned if the pain radiates to leg or elsewhere.So it could be vertebral arthritis or intervertebral disc syndrome. But not prolapse.Avoid bending, sleep supine, do not lift weight.May take anti inflammatory/ analgesic tablet 1 sos for 2-3 days, after meals. But don't make it habit.Ayurveda's excellent study on turmeric powder suggests that you additionally take 1/2 tsp in warm milk or water twice a day. This may be contd. It's natural anti inflammatory analgesic. Healer also.If still problem, get x Ray of lumbo sacral spine AP and Lateral view and lab tests done for uric acid, RA, ESR and report back. Hope it helps you."
},
{
"id": 76872,
"tgt": "Why am i getting chest pain?",
"src": "Patient: I have been experiencing a low-grading \"aching\" in my upper chest for months. I don't have any shortness of breath or feelings of fainting - just a constant ache. Moving or breathing deeply doesn't make the problem any worse. I am a 50 year old woman, and have been a smoker for many years. I am using the patch to stop, but the aching hasn't subsided in weeks. I've monitored my blood pressure the last few days and it's within normal ranges and my heart rate is around 70. Of course, my first concern is heart disease because of my smoking. Would it be wise to see a cardiologist? I know many times constant chest pain is caused from stress. Doctor: Hi thanks for asking question...Here first you have to rule out respiratory causes like restrictive lung disease as that of fibrosing lung disease or pulmonary fibrosis by smoking....Second smoking release elastase so alveoli can be damaged that can lead chronic obstructive pulmonary disease.Chest x ray and pulmonary function test useful.....If no respi causes present then rule out cardiac cause by ECG or echo...Avoid stress as that can aggravate the condition.....Take care....Dr.Parth"
},
{
"id": 217137,
"tgt": "Suggest remedy to relieve pain due to fibromyalgia and rheumatoid specs",
"src": "Patient: thank you I am diagnosed with fm and rheumatoid spec twenty years ago in newmarket and dr bykerk Mississauga I am desparetly in need of something for pain I thought I could get along without the med walk in clinic dr sent in blood work to a pain spec in barrie do not knowhow and have not heard anything for a month thanks Helen jarvi Doctor: The basic resion of fibromylgia is stress, anxiety, improper sleep, and disturb lifestyle with mental stress.There are three drugs which can be given for fibromylgia:1. Pregabaline : helps to reduce anxiety and sleep disturbance.2. Cymbalta and 3. Savella: both this are anti depressant. For permanant good relief I suggest my patient to improve their life style have good balance diet, proper sleep and rest, keep positive attitude towards any thing, and exercise regularly.Here for fibromylgia regular exercise like stretching of pain full muscles and progressing towards strengthening the same muscles also help a lot for pain relief. But here the exercise has to be started on a gradual mode.Apart from this, regular body warm up exercise like cycling and walking also helps for fibromylgia patient.Your next question was for treatment on rheumatoid: here understand what happens with patient who are rhematoid positive. There will be auto immune inflamation in body which will damage not only the harmful cells but also the normal body cells and so that patient will have complain of pain and inflammation, swelling etc.Here again the best treatment I suggest my patient is keep your self active. Do different joints and muscles mobility and flexibility exercise it will help a lot to reduce the inflammation and so that the condition will not progress.Here for both fibromylgia as well rheumatoid also there is anti inflammatory diet available. Talk to your dietician to have more idea on it.For detail exercise program. you can again check with your physiotherapist.And for medication do not start on your own but check with your doctor. They will suggest you best since they have seen all your report.Take care."
},
{
"id": 177792,
"tgt": "Suggest treatment for lymph node TB in a child",
"src": "Patient: my 5 years daughter has got lymphnode tb and she is taking macox zh 150 and combutol 200 regularly since last 50 days but suddenly after 50 days she started vomiting and eating any thing . dr. suggested blood test for sgpt and her sgpt result is very high 714.2 . her dr. has discontinued both the previous tablets and suggested new set of medicines 1. sorbiline syrup 2. udiliv 150 tablet 3.neogadine sg syrup 4.serpep 10 tablet 5.livofloxacin tablet .Please suggest should we continue with these medicines , is it safe after a very high sgpt result of 714.2 Doctor: Hi, I'am Dr Suresh K Yadav MD (paediatrics), I had gone through your question and understand your concerns,It is unfortunate that your daughter has adverse reaction to anti tubercular drugs . Although very uncommon but severe adverse reactions due occur after some anti tubercular drugs . In that case some drugs are stopped and other drugs are given for some time before again starting previous drugs . As it is not recommended to stop therapy , it is essential to continue treatment with other drugs for some time before previous medicines are started again. As these medicines are not hepatotoxic , they can be given safely.Hope this answers your question. If you have additional questions or follow up questions then please do not hesitate in writing to us. I will be happy to answer your questionsTake care."
},
{
"id": 5000,
"tgt": "Due for periods, mood swings, sore breasts. History of having unprotected sex and intake of pill 4 days later. Possible pregnancy?",
"src": "Patient: Hi, i got my period on July 16/2013 i had unprotected sex on July 29 with full ejaculation inside and since i was away and could not get the morning after pill it took about 100hrs (4days) for me to take it. Now and I am regular so due anytime on the 12 or 13th of August for my period but i am really tired and moody breasts are sore, Should I take an ealy morning test? when can i do it? and from the sounds of it can i be pregnant? Thanks for your help and advise Doctor: Hi, Thanks for writing to us. The chances of pregnancy have always been there with unprotected sex. The chances of pregnancy are also there in your case because you had your emergency pills after 72hrs. You should consult a gynaecologist for urine, blood test and ultrasonography to rule out pregnancy. You can do home urine pregnancy test after 5-7 days of overdue for period. Good luck. Regards Dr. Richa"
},
{
"id": 87331,
"tgt": "What causes swelling and pain in stomach?",
"src": "Patient: I have had a very swollen stomach for about a week now. It s more uncomfortable than painful, but there are sudden pains that come in my sides and under my breastbone. I ve been to the doctor and he told me it was acid reflux so gave me medication for it. I have been taking it, but the pain has now spread to the middle sides of my back. It may be that I haven t been taking the tablets for long enough, but I just would like an opinion. Doctor: Hi.Thanks for your query.Your history of distension - swollen abdomen with discomfort and sudden pains and now backache is suggestive of an intestinal obstruction with the intestines trying to push up the the things in it causing sudden pains also called as spams. I would advise you the following:X-ray of the abdomen in standing position.Ultrasonography of the abdomen. If there is intestinal obstruction, you have to get admitted for investigations and treatment of keeping you nil orally, intravenous fluids to supplement , investigations of the blood including electrolytes.This is not the case of acid reflux. You may need a surgical intervention if you do not get relief within 3 days. Be prepared for surgery."
},
{
"id": 119176,
"tgt": "Suffering from a disease which causes increase in WBCs and decrease in the RBC levels. Name?",
"src": "Patient: I have a close friend in Germany, he just turned 60, and about 2 years ago he was diagnosed with some sort of blood disease, I cannot remember the name, but it is causing his white cells to increase and red cells to decrease so they are both falling outside the normal limits. They have told him to come in once a year and get checked. And it sounds like there is not really any good treatment available. I would like to know the name of this condition so I can research it, and also any info about prognosis . Thanks in advance, Diana O. Doctor: Hello welcome to HCM, I am Dr das Look, many diseases can increase WBC and reduce RBC. It ranges from leukemia to myelodysplatic syndrome and lymphomas. To come to a definite diagnosis, other parametersshould be checked. Every particular disease condition has other diagnostic criteria. So, without knowing other particulars of this disease, it cannot be commented about the diagnosis. Regards."
},
{
"id": 38259,
"tgt": "Can lung infection and cough be due to mold on walls?",
"src": "Patient: my mothers old house has mold in the wall, I moved back home recently for a few months and have developed a cough that wont go away.i am on antibiotics for a lung infection,but it seems I cant quit coughing! it makes my head constantly ache and im miserable! do you think I was exsposed to black mold? my sisters been sick with phnemonia last month, but still coughing a lot! what do you think? Doctor: hello,there are high chances of you all developing lung symptoms due to it,do clean it.keep the surrounding clean,things will go better"
},
{
"id": 213433,
"tgt": "Hearing music at night, may be from TV",
"src": "Patient: My husband s aunt (90 years old) is experiencing hearing music at night and has said someone was in her duplex for about 20 minutes. She thinks the music is from the TV but we have pretty much eliminated that idea, but she still thinks that is where it is coming from. We are pretty sure no one has been in her duplex. Help Doctor: Hello The information provided is very much limited. Is this was just one incident or she persistently complain about it. Secondly does she hear these voices frequently or just this one time. If these are persistent problem and she is not getting convinced by repeated explanation by you guys than , she may be suffering from psychosis. At this age it may be secondary to some neurological cause so she need a neurological assessment and also psychiatric intervention. Get her to doctors ASAP as it may progress also which can be difficult to manage. Take care"
},
{
"id": 24910,
"tgt": "Suggest dosage of omlezest and fibator inspite of having normal triglyceride levels and BP",
"src": "Patient: I have been taking omlezest 20 and nebi 2.5 for last 2 years as my blood pressure was high. Post taking these medicines the BP is normal . I am also taking Fibator 10 mg for Trigylgerides for last 3 years when my triglycerides were above 450. Since taking Fibator my trigyclerides are in normal range for last 2 years. Also taking 37.5mg Thyronorm for thyroid. Please advise in case I can reduce the dose of Fibator or change the medication for BP and Trigyclerides. My cholestrol levels are within normal. Doctor: Hello, Thanks for using Healthcaremagic. i had gone through all the data you have posted. as you told your blood pressure is under good control you need to continue both medications for blood pressure. kindly continue lifestyle modifications like avoid pickels, low salt, diet rich in fruits and vegetables, regular physical activity along with BP meds. coming to cholesterol, as you said for triglycerides when they are very high we use medication to decrease to normla levels and then you can stop medication for triglycerides but you need to continue for LDL. it would have been helpful for us to decide if you have posted the full lipid profile report. FIBATOR 10 what you were using might be a combination of atorvastatin + fenofibrate of which fenofibrate can be stopped and atorvastatin should be continued 1 tab daily night. common reason for elevated triglycerides are eating oily foods, alcohol, diabetes and hypothyroidism. so thryoid treatment can be continued, decrease the oil food content, quit alcohol so that TG levels may not rise after stopping tablet. continue regular physcial activity.hope this helps..."
},
{
"id": 152868,
"tgt": "Suggest treatment for cancer",
"src": "Patient: My wife had a Stage 2A Type AB thymoma removed. The Surgen is very confident that it was encapulsated and that he removed it all. The radiation oncologist feels that radiation may reduce the chance of reocurrnace. Is radiation in her case the way to go or do the rediation risk make it not the way to go? Doctor: Thanks for your question on Health Care Magic. I can understand your concern. Yes, she should definitely take radiotherapy. Any cancer surgery has risk of spillage of cancer cells at operative site even if it is capsulated tumour. So chances of recurrence at operative site are high due to spillage of cells. To avoid this local sit recurrence, post operative radiotherapy is needed. So get done radiotherapy in her case. Hope I have solved your query. I will be happy to help you further. Wishing good health to your wife. Thanks."
},
{
"id": 63816,
"tgt": "What causes lump under armpit?",
"src": "Patient: hi i am a 16 year old and i have a lump under my armpit. i have had this for about 5-6 years.is it cancer.it doesnt hurt but around periods it feels tender and after periods it feels smaller. i recently had a viral illnes and when i toon ibuprofen then i could hardly feel it. Doctor: Hi, dearI have gone through your question. I can understand your concern.You may have some soft tissue tumour, enlarged lymphnode or axillary tail tissue of breast. You should not worry about cancer. Chances of cancer is minimal at this age. So just be relaxed. If it enlarges then you should go for biopsy or FNAC of that lump. It will give you exact diagnosis. Hope I have answered your question, if you have any doubts then contact me at bit.ly/Drsanghvihardik, I will be happy to answer you.Thanks for using health care magic.Wish you a very good health."
},
{
"id": 72222,
"tgt": "What causes shortness of breath with chest tightness?",
"src": "Patient: I enjoy singing in the choir at church and have experienced slight shortness of breathe near endings, then I have tightness in chest. I have had some heart issues but my cardiologist recently did a cath and said no problems. I then had an endoscopy and dr found nothing. I have had this issue for a while now and still trying to determine what the problem is. Doctor: Thanks for your question on Healthcare Magic.I can understand your concern. First of all no need to worry for heart diseases because your angiography was normal. We should rule out lung diseases (asthma or bronchitis) for your symptoms.So get done clinical examination of respiratory system and PFT (Pulmonary Function Test).If PFT is also normal then no need to worry for lung diseases. Sometimes stress and anxiety can also cause similar symptoms. So avoid stress and tension, be relax and calm. Don't worry, you will be alright. Hope I have solved your query. I will be happy to help you further. Wish you good health. Thanks."
},
{
"id": 184546,
"tgt": "What causes persistent mild burning in lower throat with dry mouth?",
"src": "Patient: I ve had a persistent mild burning in my lower throat, along with dry mouth, and lump-in-the throat feeling. It causes a mild feeling of nausea from time to time. I was diagnosed with GERD; however, I am apprehensive about continuing to take my medication since my husband and I are trying to get pregnant. What should I take? Doctor: Hi,Thanks for posting the query, Take tab oflox 200mg BD tab Nimi bd for 3-5 days.At home take lukewarm saline nad antiseptic mouthwash rinses.Take multivitamin suplements.Take care!"
},
{
"id": 29459,
"tgt": "How can UTI be treated?",
"src": "Patient: Hello I was just looking up some info on UTI's. My sister is in the hospital and they are keeping her overnight to flush her kidneys because the UTI traveled to her kidneys. She has one Kidney because in 2013 she donated one to me I had FB Gomerulonephrits. I am worried about her and that one kidney. How serious is this? Doctor: Hi i do care for your concern. The urinary tract most of the times affects the lower part of urinary system and only in some the infection may ascend upto kidneys. First of all need not panic as this condition most of the time can be completely treated with the most powerful antibiotics available. I would advise To consult your physician and get culture and sensitivity testing done and get antibiotics according to it only. In case of sensitive antibiotic the infection clears within few days. Since she had donated one of her kidney it is advisable to prevent her from further attacks. Maintain adequate hygiene and have antiseptic soap for bathing. You have not mentioned her age, as also check for blood glucose levels. Hope everything gets well soon. Hope i have answered your question, if you have more feel free to ask. Thank you."
},
{
"id": 84780,
"tgt": "Which disease is cured by zifi dry syrup?",
"src": "Patient: please let me know that which disease is cured by zifi 100 Dry syrup. I have observed that 2nd dosage of Hepatites A HAvrix has not been given to my child form 1.5 yrs now he is sufferiing from this disease. will the above mentioned medicine will help. or please suggest what to do now Doctor: Hi,Susceptible bacterial infections. Zifi 100 mg dry syrup contains third generation cephalosporin cefixime which is a broad-spectrum antibiotic commonly prescribed to treat a variety of conditions such as urinary tract infections, typhoid, throat infections and lung infections caused by susceptible bacteria. If your child is suffering from hepatitis A, there is no specific drug treatment for hepatitis A. Managing the symptoms can help your child feel better while recovering Have your child bed rest when symptoms are the worst. Do not give paracetamol to your child without consulting a doctor. Give your child fluids in the form of fruit juices or electrolyte solutions, such as Pedialyte. This helps prevent dehydration. Zifi will not work against viral infections.Hope I have answered your query. Let me know if I can assist you further. Regards, Dr. Mohammed Taher Ali, General & Family Physician"
},
{
"id": 208216,
"tgt": "Suggest remedy for mental health problem",
"src": "Patient: i have reactive hypoglycemia.Now i have observed that whenever my sleep patterns are disturbed lets say i normally sleep at 12am and i extend that to 1am,i cannot sleep very well and am tired next day even if i sleep for 10 hours it takes a couple of days before i get back to normal. If i get excited or am in a stressfull situation it takes me longer to recover. what is going on,help please Doctor: Hi, thanks for the query. Can you kindly specify your age, gender, your current work profile & whether you are in habit of consuming alcohol, tea, coffee, smoking cigarettes etc. during evening hours usually post 8pm? Based on current information, I would say that many people after a certain age (50-60 years) tend to become light sleepers with frequent sleep disturbances with slightest changes in sleeping patterns, habits etc. Hence, to avoid this, we can practice some good sleeping tips known as sleep hygiene. These are 1. Waking & sleeping at a fixed time 2. Avoid over/under sleeping 3. Avoid daytime naps 4. Avoid cigarettes, tobacco, caffeine after 8pm 5.Use bed for sleeping only 6. Dont stay in bed for more than 20 minutes if you are unable to sleep after retiring to bed. Additionally there are drugs like melatonin (available over the counter); ramelteon, agomelatin (prescribed by doctor) which reset your circadian rhythm & help you sleep better. Lastly, I would like you to get back to me with what makes you excited or when do you feel stressed out, how is your mood, energy levels & since when do you find it hard to recover from stressful situations. Good Luck"
},
{
"id": 119952,
"tgt": "What does hypo-echoic lesion with calcification suggest?",
"src": "Patient: Hi! my father age 64 yrs has undergone sonography of neck and report says ill defined hypoechoic lesion measuring 7*7 mm with calcifcation and minmal vasculartiy adjacent the left submandibular gland with clear demarcation not made out. no cervical lymphadenopathy. should we go for ct scan . Doctor: Hi, They are suggestive of old healed tubercular lymphadenitis after initial caseous necrosis at the centre of lymph nodes, which over time gets calcified. They show as hypo-echoic lesion with central calcification. Hope I have answered your question. Let me know if I can assist you further. Regards, Dr. Rohan Shanker Tiwari, Orthopedic Surgeon"
},
{
"id": 45747,
"tgt": "Suggest treatment other than opiates for LPHS",
"src": "Patient: i have suffered with lphs for 20 years the pain is agonising i take large qauntities of opiates but they just ease it slightly rather than take it away i had a kidney denervation operation 2 years ago but the pain came back after 7 months i keep getting fobbed off by my specialists as no one understands the condition is there any other course of treatment other than opiates as i hate taking these drugs but it is all i am being offered i am a 30 year old female in the uk Doctor: Hi, Prolonged use of opioid analgesics is not recommended and it can harm your kidneys. You can try Tramadol which is a safe opioid group and can be used for a longer duration. Hope I have answered your query. Let me know if I can assist you further."
},
{
"id": 173357,
"tgt": "Will fifth disease occur only once in child?",
"src": "Patient: My 11 month old has had a low grade fever and really red cheeks for a week now. He had fifth disease a month ago and I was told they only get that once. Is this true? He also has a rash on his private area and lower tummy that has come up in recent days. What could this be? He doesn't act as if this is affecting him? Doctor: Hello dear ,Fifth disease is caused by parvovirus B19. It is most prevalent at kids,but it can spread among people of any age.Any disease can repeat second time if immunity is low.It looks as fifth disease You just give to her acetaminophen(Tylenol) and wait she will fight off the virus. Keep her hydrated with plenty of waterWishing your baby speedy recovery"
},
{
"id": 183256,
"tgt": "Can abscess in mouth cause bitter taste?",
"src": "Patient: Hi, I have had a bitter taste in my mouth for about five montehs now. I have great hygiene but notticed a bitter taste in my mouth in the top right hand molar. I had it checked and had a an abcess. I had root canal and they found a crack. I had an extraction but the gum tissue is still bitter casuing tons of drainage.I had another crown just replaced witha temporary and now I have no more metal in my mouth. Is this related to my sinuses or is theis tooth related. I have bee to many dentists and natural doctors and have had many different medications. The oral surgeon said my extration sight looked good as well as the bone. Something is not right . Please help amm Doctor: helloo...read thru ur query...accordingly since dentist told socket looked great and other conditions are ok in the oral cavity(he might have checked whole mouth thoroughly)..drainage can be from sinus!!!or can be due to fungal coating on the tongue(if u have noticed it)..but nothing to worry...if coating is present u can follow these steps..after brushing daily u have to do tongue scraping with scraper use chlorhexidine mouthrinse and apply nystatin gel on the tongue..drink lots and lots of water...and see for a week...if it still continues meet a dentist...for sinus..u have to do steam inhalation daily twice in morn and night do breathing excercise twice daily and if possible do excercise for 30mins...follow this regularly ur sinuss problem will be solved..if after 2 wks still it persists meet an ent surgeon..besides all this be cool and stressfree...stressfree mind and happy face can relieve all probs...hope ur benefitted something from this reply.have a healthy day!!"
},
{
"id": 147175,
"tgt": "What are the chances of normal living after a Brain tumour surgery?",
"src": "Patient: My aunts brain surgery was on Wednesday. It was a timor removale, they found many smaller branches of the timor. Since the surgery, she has not been able to move her right leg, what is the chance she will never walk again? The timor was on the top of her head. Doctor: Hi,Thank you for posting your query.I have noted your aunt's symptoms. I noted that she had brain tumor surgery and after that she has developed right leg weakness.The weakness of right leg could be due to the tumor or due to brain injury or bleeding during the surgery.A brain CT scan would help us determine the real cause of her right leg weakness.She would gradually improve with physiotherapy and exercises.I hope my answer helps. Please get back if you have any follow up queries or if you require any additional information.Wishing you good health,Dr Sudhir Kumar MD (Internal Medicine), DM (Neurology)Senior Consultant NeurologistApollo Hospitals, Hyderabad, IndiaClick on this link to ask me a DIRECT QUERY: http://bit.ly/Dr-Sudhir-kumarMy BLOG: http://bestneurodoctor.blogspot.in"
},
{
"id": 18691,
"tgt": "What causes chest pain despite a normal chest X-ray report?",
"src": "Patient: My son is 17 and has experienced chest pain ,what he calls palpitations, when he exerts himself for about a year now. He has seen a cardiologist and had a holter monitor and just had a stress test (which he was unable to reproduce his symptoms because he also has compartment syndrome of BLE and was only able to tolerate 8 min of test. Cardiologist thinks it is NOT cardiac related. A recent chest xray to r/o pneumonia was negative for any abnormalities of the chest, ribs, sternum. Could this be musculskeletal? Where else should we be looking? Doctor: Hello and Welcome to \u2018Ask A Doctor\u2019 service. I have reviewed your query and here is my advice. I need to know more about the symptoms, like precipitating factors, factors that increases pain etc. It can be a gastritis and reflux disease. Do you upper abdominal pain, nausea, bloating, burping, increase in pain on food, sour water feeling in throat or chest burning, if yes it further supports the diagnosis. Secondly does this pain increases on applying pressure over the area, if yes it could be costochondritis and it's a musculoskeletal type of pain and needs prolonged nsaid therapy. Does this pain increases on deep breathing and coughing etc. if yes it a pain arising from lining of lungs and needs nsaids treatment and evaluation. Hope I have answered your query. Let me know if I can assist you further."
},
{
"id": 51132,
"tgt": "History of post PTCA. Ultrasound shows prostatomegaly. Loss of sexual desire, burning sensation while urinating",
"src": "Patient: i am aged 59 years old retired. i have a history of post ptca. recently i was feeling buring sensation while voiding urine and also had to uninate freequently and urgently. i went to doctor. urine culture test was conducted. the result is e.coli species grown in culture. he prescribed some medicines. after medication condition was almost normal. it was 10 days back. after that i was asked for psa test. psa is 6.3 . after that i was asked for untrasound scan of kub test. the result of the test is grade - ii prostatomegaly. size of the prostate size : 4X3.7X4.4cm volume = 35cc urinary bladder is normal. no evidence of calculi. both kidneys are normal in size. the doctor advised me to approach urologist problems at present since one year suddenly i lost 90 % sexual desire. still if am feeling light burning sensation while voiding the urine. please help me Doctor: Hello Thanks for your query.You had burning micturition with increase in frequency and urgency.As per culture report your Dr has prescribed antibiotics and your symptoms have reduced to normal.Are you taking any medicine now for enlarged prostate.These drugs have side effect of decreasing libido.The loss in sexual desire could be related to the age Pl consult some Urologist.who will be able to assess you clinically and prescribe medication.."
},
{
"id": 170760,
"tgt": "Is horlicks a recommended drink for children?",
"src": "Patient: I have 10 month old twin boys & would like to know if I can give them horlicks... They follow a healthy nutrition schedule... it includes mother s milk, formula, soup & rice & oat meal porridge. They seem to have lost weight but are healthy. They weigh approx 7.6 & 7.8 kgs Doctor: Hello and Welcome to \u2018Ask A Doctor\u2019 service.I have reviewed your query and here is my advice. An infant of 10 months weighing 7.6 to 7.8 kilograms is doing excellently well and I congratulate you for it. I as a pediatrician do not suggest using any of the health drinks till 2 years of age as they might pose a solute load on the kidneys. Even for you, my advice will be start using health drinks like what you have quoted, only after 2 years of age. Hope I have answered your query. Let me know if I can assist you further.Regards,Dr. Sumanth Amperayani"
},
{
"id": 25456,
"tgt": "What could pain in chest after taking medical marijuana?",
"src": "Patient: I'm a 43yr old female.I use medical marijuana for cronic pain, arthritis, and scoliosis. I had a hysterectomy 1/5/12. I also had a emergency tooth extraton last week. I'm currently taking 2,400mg of Ibuprofen,15mg Vicoden and 1,200mg of clindamycin. After taking one puff I started feeling chest pains. could this be a reaction? Doctor: Hi Ms.. I read ur question, the description of your pain is unlikely to be reaction but it is like whats called pleurtic pain due to inflammation in the pleural membrane or due to some muscle strain or inflammation of the chst cage muscles , however it is benign conditions you better continue ibuprofen for toltal one week, not advisable for long term use..it is good if you can quit smoking.. Get Well soon."
},
{
"id": 32798,
"tgt": "Should precautions be taken if no redness is seen on tetanus injection site?",
"src": "Patient: Hi, may I answer your health queries right now ? Please type your query here...My wife received a tetanus shot 1 week ago and her initial reactions of redness and swelling and pain in her injection have changed and seem to be improving. Should she take other precautions? Doctor: Hi, Thanks for posting in HCM.It appears from your description that your wife has got an reaction due to taking of injection tetanus toxoid. If you see pain and redness at the site, she can apply ice pack to reduce the inflammation. She can take anti-inflammatory and analgesic like Ibuprofen and Paracetamol combination, twice a day after food. If she is pregnant, she should avoid ibuprofen has to be avoided and only paracetamol needs to be taken. Hope the information provided would be useful."
},
{
"id": 56619,
"tgt": "What causes enlargement of lever?",
"src": "Patient: I have a mildly enlarged liver measuring 15.4 cm with increased per-portal echoes and no focal lesion seen. IHBR: Normal. Lab reports: Total Bilirubin - 0.81, Bilirubin, direct - 0.13, Total Protein - 7.6, Albumin 4.3, Globulin - 3.3, Asparate Aminotranserase (SGOT) 26, Alaine Aminotranferase (SGPT) 53, Alkaline Phosphatase - 77, G-Glutamyl Transferase - 21, Lactate Dehydrogenase - 151, Calcium - 9, TSH - 2. Can you please tell me what may be wrong? Doctor: Hello Welcome to HCM Thanks for posting your query I understand your concerns. I understand your concerns. Except for mild enlargement of the liver, your LFT that is liver function tests looks normal. Mild enlargement is quite common in the middle aged persons due to fatty infiltration of the liver. This is benign condition and it will subside on its own. If you don't have any other symptoms like nausea and vomiting, no need to worry. You are fine. Thank you"
},
{
"id": 42242,
"tgt": "Suggest treatment for infertility",
"src": "Patient: Hi Doc, myself & my husband have been trying to fall pregnant for the past year. Went to the gynae recently & he prescribed 200mg of Utrogestan daily for 10 days as I've been having very irregular cycles. Ive only taken them twice now. I took a ovulation test just this morning & it's positive. Can I fall pregnant while on these tablets? Or should I stop them this cycle as I am ovulating? Doctor: Hi,I read your query and I understand your concerns.Following is my reply:1) You can take these tablets which will help you to conceive.2) No need to stop them.Let me know if you have anymore questions.Regards,Dr. Mahesh Koregol"
},
{
"id": 83031,
"tgt": "Suffering from Lupus, have Dizziness, bloating and heartburn. History of H-pylori. Is it caused by Helicobacter?",
"src": "Patient: Hi I have issues with chronic hives , hairloss , joint pain, dizziness, nausea, bloating & chronic heartburn - initially I tested positive for lupus . I had H-pylori about 4 or 5yrs ago - how lon will it remain positive in blood tests. I ha all my levels retested yesturday as I just had a flare up of hives etc with constant heartburn, bloating, nausea, & I ve vomited & had random diarrohea a few times over the last few wks -the pain has been so bad I almost went to hospital as the chest pain was also radiating into my shoulders & up my throat & I couldn t breathe deeply. Tests results came back as lupus not being active but positive for H-pylori...not sure if this is a positive as it was a blood test & I was treated for this 4-5yrs ago? Could this all be just caused by Helicobacter & not lupus at all???? Which tests should I be now asking for ? Doctor: you should probably undergo endoscopy and test for h pylori in the sample."
},
{
"id": 103095,
"tgt": "Have an allergic reaction. BP seems to be high. Lacking concentration, focus. Breathing problems. Treatment?",
"src": "Patient: Hello. I believe I am experiencing an allergic reaction to mold. I have noticed my blood pressure has been higher than usual, and have been lacking concentration and focus, as well as an increase of temperature (I'm always feeling warm). I have also have been experiencing breathing problems. What should I do? I have visited my doctor last year and have had blood tests, but nothing seemed to abnormal. What should I do? Doctor: Hello thank you for your queryi will advise you to avoid mold which is causing you breathless.you reduce salt intake in your diet and do brisk walking regularly atleast 30 minutes per day for 5 days in a weeki will suggest you to consult physician again and check blood pressure, blood counts etc., and take his advise.i hope you understood me, take care and get well soon."
},
{
"id": 16978,
"tgt": "What could be the cause of rapid heart rate?",
"src": "Patient: hi I hear my heart rate in my right ear., all the time it even wakes me up. I have been on high blood presure meds. I have tryed relaxing the musles. I have gone to a ear doctor we have put in tubes. nothing. went too a nother doctor and found out I have tintes heart rate one. I really need some help it driving me nutssss. Doctor: Hi, Ecotrin (low dose Aspirin) is prescribed to prevent blood clots in the heart vessel to avoid a heart attack. It doesn't influence cholesterol levels. For lowering cholesterol levels specific medications are used called statins. If your husband underwent a stress test and it was positive for ischemia, he should be referred for coronary angiography. It is a specific investigation to image heart vessels and found which one is blocked that causes ischemia, if such a vessel is found, then the doctor opens it. I would advise you to talk to your doctor and discuss further steps and coronary angiography. Hope I have answered your query. Let me know if I can assist you further. Regards, Dr. Lilit Baghdasaryan, Cardiologist"
},
{
"id": 203807,
"tgt": "How to speed up my puberty?",
"src": "Patient: I'm a 15 yer old male, about 5ft 2 in height, so I am small. I have a tiny bit of under arm hair under my right arm, but none under my left. I have public hair around my pen is, and can ejaculate, but it's either clear or cloudy, but always sticky. It is never a thick white substance though. I have a deep voice, but unsure if my voice has broken. Is there anyway I can speed up puberty, either by prescribed pills or anything? Doctor: Hello welcome to heakthcare magicAs owr your request for mediciens to improve growth of secondary sex characteristicsAnf your age is 15 now. Its not a problem with your health which you described in refference of your growth.Growth in evwry body are vary, do not make yourself hurt by compring another persons.No need to take any medicines, but be mature mentally.Purberty will go at its time.Pills or any hormonal thaerapy is not advisable as per its adverse effects.Wait and watch till mature level.Regards,Dr. Nikul PatelAyurveda Sexologist and Sex consultantAhmedabad, India"
},
{
"id": 1466,
"tgt": "Can i get pregnant if having high prolactin levels?",
"src": "Patient: i am 26 years married actively trying to conceive but no luck.my weight is 50kg.5'3\" tall.i had an medical examination which shows my prolactin level is high i.e.38.52 ng/ml.and TSH is normal.can i get pregnant despite of this.my doctor done physical(manual)examination and said everything is good.but i am bit worried with high prolactin .plz help. Doctor: Hi, I think you should start some medicines like cabergoline for high prolactin levels. Discuss with your doctor. It can be treated. Get a repeat test after 1 month. If your periods are regular, you can try naturally for 6 months. Be in contact with your husband every 2 to 3 days after your periods stop. If it doesn't work, then you can take some medicines like clomiphene for the growth of your follicles and track your follicles growth by repeated ultrasound. When follicles reach a size more than 17 to 18 mm, take injection for rupturing the follicles. Be in contact with your husband for 2 to 3 days after injection. Take progesterone for next 2 weeks. Do a urine pregnancy test at home after that. You can try like that for 3 to 6 months. Hope I have answered your question.Regards Dr khushboo"
},
{
"id": 111145,
"tgt": "How to overcome the pain in my lower back?",
"src": "Patient: I have been feeling pain in my lower back since 1 week. Particularly at sacrum level. It started suddenly last week in the morning. I can walk normally in straight position but whenever I am leaning forward to lift anything , I feel pain at the sacrum level. Kindly, suggest me treatment. Thank you. Doctor: Hello,I had gone through the case and found that you first go for MRI of spine to know the cause.Meanwhile take mild painkiller and apply muscle relaxant gel.After the report take proper treatment and physiotherapy.It might be either muscle spasm or slip disc.Hope my answer will be effective for you.Thanks"
},
{
"id": 166091,
"tgt": "Suggest remedy for severe cough, itchy eyes, ear infection and fever in an infant",
"src": "Patient: My 5 month old has been sick with cold and/or allergy like symptoms since 4 weeks with no break, the symptoms my become more minor but have never cleared. They include cough in all form but never wheezing or whooping , itchy eyes which have gotten worst since the weather has gotten warmer and we have started opening windows, she has had a ear infection, runny nose that will become green and thick and then she runs a fever of 100 to 103. We suction her nose all the time along with saleen, use herbal rubs, and humidifiers. This will become clear again and she stops running the fever and the cough becomes more dry but never goes away and the other symptoms will still remain. The clearing up of the symptoms last for about a week and has happened 3 times. She still eats well, as long as long as the fever is not high and can be cranky with short spells of her being in a good mood. I have heard of babies being sick with colds often but 4 straight months this is crazy. Please help! Doctor: Hello.Kindly provide detailed history including family history of allergy.Its probably allergic rhinitis along with spells of acute otitis media and pharyngitis.i would recommend to start amoxil(125mg/5ml. 2.5 ml three times a day during acute phase of fever and discharging ears for 5-7 days)continue some anti allergic medicines like cetrizine 2.5 ml at night and montileukast sachet 4mg at night(both for 3 months).clean the ears meanwhile with cotton wick just from outside,dnt insert it inside the canal.Hope it will help."
},
{
"id": 153516,
"tgt": "What will happen if various types of cancerous cells are introduced into the body?",
"src": "Patient: to my understanding cancer is a mutated cell that rapidly multiplies. theoretically, if various types of cancerous cells were introduced into a body corresponding to the various types of cells would an individual be able to survive, abet cellularly hyperactive? Doctor: Hi, dearI have gone through your question. I can understand your concern. Cancer is multi step process. Only introduction of cancerous cells in to body is not sufficient to produce cancer. It is killed by NK cells of immune system. Hope I have answered your question, if you have doubt then I will be happy to answer. Thanks for using health care magic. Wish you a very good health."
},
{
"id": 75141,
"tgt": "Suggest treatment for chest discomfort and shoulder pain",
"src": "Patient: I am 49 years old and had a heart cath when I was 42 for a branch blockage. Other than that I have been pretty healthy. I have hypothyroidism but it is under control. A few days ago, I began having chest discomfort and left shoulder pain that has not let up. My pressure seems to be up. Should I go to ER? Doctor: Hello!Thank you for asking in HCM. I can understand your concern. According to your history about your heart problems it would be better for you to go to ER.If i was your doctor i will firstly exclude cardiac problems by doing :EKGCardiac enzymes Cardiac ultrasound If these test are negative,then i do not think that this pain is related to the thyroid glands and the next step it would be:A CHEST X Ray to rule out lung diseases .If it comes negative too may this pain can be due to stomach discomfort.The ER doctor will advice you further after he has the above results.I wish you a good health Thank you Dr Jolanda Nikolla Pulmonologist"
},
{
"id": 138815,
"tgt": "What causes warm sensation on top of foot?",
"src": "Patient: I started getting a wam sensation in the top of my left foot. This has been going on for almost a week now. It happens multiple times of day. There is no swelling, redness, or pain and externally it does feels the same temperature as the right foot. I am a 33 year old female. I had hernia and fall bladder surgery 9 weeks ago. I also have arterial tortuosity syndrome. Do you know what this could be? Doctor: HiSensation of warmth without any actual warmth is a sign of neurological involvement. Check for ur sugar level as peripheral neuropathy may show such signs also see if ur an alcoholic as this can cause neuropathy. Also get vitamin D and B 12 level done."
},
{
"id": 146553,
"tgt": "Suggest better treatment for slip disc problem",
"src": "Patient: Hi Doctor, I am suffering from slip disc problem (L4-L5) for 3 months, due to which i have pain in my lower back and right leg. Taking medicines and also 2 epidural injections were given by the doctor. But i have no relief at all. So please suggest me what to do know. Thanks Doctor: First try conservative measures like physiotherapy , wearing lumbosacral corset.avoid bending forward and lifting heavy weight. Take tab pregaline and analgesics. If these measures falils then you may need surgery. Please mention canal diameter to comment better. Hope my advice will help you. Take care. Don't forget to rate me."
},
{
"id": 105541,
"tgt": "Cigratte, marijuana smoker. Had asthma attack. Severe cough with mucus. Health concern. Reason for attack?",
"src": "Patient: I am a 27 114lb 5 7 female ive been smoking cigarettes since i was 13, quite a heavy marijuana smoker in teens but no more, I started using crystal meth when i was 21 only snorting it for the first 6 months i started smoking it quite heavily thru a water bong often with incredibly sugary juices and candies in it for flavor. I quit for 1 year in 2008 relapsed very shortly after yr to date when i relapsed in 2009 using same bong method with juice. last year in march i went to rehab but left in july due to a relapse, my daily intake is rather excessive i can often smoke up to an eight ball (3.5 grams) a day, i started to occassionally use heroin in september of this year I am not physically addicted to it, but i do use it more than i should. around christmas of this year i noticed i was becomming very short of breath and having troubles breathing wheezing very badly. on christmas i suffered my first asthma attack my respiratory health has steadily declined unfortunalty my meth bong usage has not. i have just recently stopped smoking heroin(i was using bongs for that also) and i can only take about one puff of a cigarette without choking literally and coughing till i usually cough up blood. I have been in the hospital now about twice a month since december. my coughs are very severe and often have a sick churning gurgle sound and the mucus that comes up is very yellow and thick thick balls and large curdy thick chunks and resemble long disgusting wormy looking mucus noodles, often m with brown or bloody like color swirls. this last month has been especially bad, my wheezing is constant and walking up 5 stairs will often bring me to the point of asthma attack. im 27 and i8 sound like a 90 yr old woman. I am obviously growing very concerned about my condition and i am now trying to no longer smoke jib on a bong....or really at all. i would like to mention also that i have had more than one set of chest xrays when nothing was said to be the matter.....I guess my question is do you think this is due mainly from bong usage noting however i never encountered a problem EVER in past or is there something else to worry? thank you Doctor: YOU STILL NOT HAVE DAMAGED ANYTHING BUT MAKE SURE THAT YOU STOP ALL THE BAD THINGS NOW BECAUSE THIS IS TIME TO QUIT EVERYTHING THIS IS YOUR WILL POWER WHO CAN STOP YOU START TAKING TABLET MONTELUCAST 10 MGM THRICVE A DAY D DO IT FOR 6 WEEKS SEROFLO INHALER 125 MGM TTWICE OR THRICE A DAY TAB ZINETAC 150 MGM EMPTY STOMACH TAKE 3 GLASSES OF HOT WATER IN THE MORNING AND 3-4 GLASSES OF WARM WATER BEFORE SLEEP"
},
{
"id": 278,
"tgt": "Will duphaston help to conceive in pcos patient?",
"src": "Patient: hi doc im 29 years old.tring to concv from last 2 years have pocs and thyriod.m on 25 mg thyronorm.my thriod level is not increasing.my doc say its very mild(5.25).now my doc is advising me for duphaston on 19day of my periods . my periods are coming on 33day.its normal flow.will duphaston help me to get prego? Doctor: Hello and Welcome to \u2018Ask A Doctor\u2019 service.I have reviewed your query and here is my advice.You should start with clomiphene from day 2. Then get a follicular study. When follicle reaches 18 mm, then HCG will be given for rupture. Then Duphaston to be taken for 14 days after rupture and keep relationship with husbandHope I have answered your query. Let me know if I can assist you further.Regards,Dr. Sheetal Agarwal"
},
{
"id": 130938,
"tgt": "What causes pain in the shoulder and soreness under the clavicle?",
"src": "Patient: Most of my joints have some degree of crunch but today my shoulder is sore, with extra crunch. I fell last night but didn't notice any shoulder pain. It hurts to raise my arm past shoulder level, to rotate the outstretched arm and soreness is condensed in front between my ribs and arm under my clavicle. Doctor: I recommend cataflame intramuscular daily for 1 week you may not need physical therapy if you follow this one week anti inflammatory course Good Luck"
},
{
"id": 149848,
"tgt": "Child with bruising, low platelet count.Diffuse petechial rash neck, trunk, extremities, groin . What would be DD for patient?",
"src": "Patient: 6 year old girl with a sudden onset of easy bruising and rash. with low platelet count (5,000),and an increase in size of platelets. Diffuse petechial rash on her neck, trunk, extremities and groin . Recent UTI (2 weeks ago). What would be the DD for this patient? I think the dx. is ITP, but could it be Drug-induced thronbocytopeniea, also I ghave a DD of leukemia . Thanks...I really just need a third DD. I am a student. Doctor: Hello,Please Don't waste your time in looking for d/d.Investigate and establish the diagnosis.Get a peripheral smear and a bone marrow to get things done in right direction.A hematologist will be of great help.thanks"
},
{
"id": 176783,
"tgt": "What causes swelling around nose and eyes?",
"src": "Patient: My six year old son is swollen around his nose and eyes. He s had headaches for the last couple days. His nose is a bit runny. He s very miserable. Is it more likely a sinus infection or could he have possibly stuck something up his nose? What do I do?? Doctor: HI SEX YR OLD HAD NO SINUS INFECTION he had rhinitis which can cause swelling around neck and eye. U have to give antihistamic and decongetant for running nose. U give him saline nasal drop in him nose which can stop blocking of nosefor headache u give hi peracetamol 15 mg/kg wheneve he had headache"
},
{
"id": 201473,
"tgt": "Is an unusual penis ridge line normal?",
"src": "Patient: The line that runs from my scrotum up to my penis, I don t know what its called but its like a ridge, i hope you know what i mean. when it gets to my penis about halfway up, it twists to the side, and then comes back where it meets the opening in my penis. I m uncircumsized, is that normal? Doctor: HIWell come to HCMI appreciate your concern, this is dividing line that exactly taking place at the time of organogenesis and this is absolutely normal one and this is part of human body in male person, in some subject this could be more prominent, but this is normal one and no need to worry about this, hope this information helps."
},
{
"id": 199568,
"tgt": "Suggest remedy to increase sperm count and motility",
"src": "Patient: Hi, I am married from 6 years now. We are trying to conceive from 4 years. I conceived twice in last two years through ovulation Induction. Both the times it was a missed abortion, one at 5 weeks and the other at 9 weeks. I am a PCOS and taking Metformin from last 1 year after which my periods have become regular from last one year. I have done all my blood works and the harmones are normal. My husband is having low sperm count from the beginning. Is there any way to enhance his count and motility? And also from last 3-4 months he is suffering from lower back and groin pain. We consulted a doctor and after few tests,they said it is due to some infections in Prostate and my husband was prescribed antibiotics for 45 days. From last days 20 he is again complaining with on and off symptoms. Is there a any way we can cure this completely? And is this effecting the fertility? Now how can we boost the chances of conceiving faster? Doctor: Hello Low sperm count and low motility need proper clinical evaluation and investigations.Your husband may need investigations like routine hemogram,random blood sugar,ultrasound of scrotum.It is important to exclude any surgical reasons for low sperm count and motility.Ultrasound and clinical examination is helpful.Hormone assay can be done if needed.He should avoid tight undergarments.He should increase his physical activity and do exercise.Take nutritious diet and antioxidants.Take CareDr.Indu Bhushan"
},
{
"id": 202726,
"tgt": "Constant pain in breast, armpit and above elbow, worsens with certain arm movements. No relief with ibuprofen",
"src": "Patient: Male - 58 y/o. Last 2 days, rather severe right breast dull pain, also felt in arm pit and between elbow and armpit in right arm. Today, a little more severe on top side of right breast. After morning ibuprofen yesterday, faded away around lunchtime, and returned in evening. Has not faded away today, despite more ibuprofen this morning, and seems to be more severe in the breast area. Does hurt more with certain right arm movements. Doctor: HIThank for asking to HCMI really appreciate your concern, this could be muscular pain spasm is likely, if I would be your doctor then I would treat you with the following medicines,1) Tab Acetaminophen three times in day 2) Tab Diclofenac 50 mg sustain release once in day3) Tab Chlorzoxazone three times in dayhope this information helps you have nice day."
},
{
"id": 90401,
"tgt": "What causes pain in abdomen with gas problem?",
"src": "Patient: I have a problem that my lower right hand side of my abdominal.. i always have gas problem.. once i ll touch it beat like a drum.. corresponding to that it put pressure on my penis and bowels.. and sometime it gives me a pain.. like stretch in lower right hand side of my abdominal part.. and sometimes complete lower abdominal is full of gas.. and also give me a trouble in urine passage.. like white fluid, milky water in my urine.. once it passed away.. gas in the lower abdominal part would start releasing out.. please provide me the solution or please referred a doctor who can diagnose and provide me the medication.. Doctor: HelloPain in lower part of abdomen with gas formation on right side may be due to these reasons , these includes.1 Amoebic colitis , as this is a most common cause of such symptoms . Diagnosis can be confirmed by physical examination by a physician , because in this disease usually colon is palpable and there is tenderness in this area. Stool examination shows Entamoeba histolica.2 Right renal colic ( calculus or urinary tract infection is another such cause . Get in blood for C B C , E S R , SERUM creatinine level . Urine examination for culture .3 Tuberculosis of large intestine is another such cause . Get in M R I of this areas .Blood examination for E S R ( if very high > 70-80 mm of 1st hour is suggestive of tuberculosis or any chronic infection long standing).At present deal as I mentioned above. Hope information will help you. Still no relief then consult a Gastro-enterologist and get his opinion."
},
{
"id": 38927,
"tgt": "Suggest treatment for tuberculosis",
"src": "Patient: hi i m 30 yrs old female i m suffering from tb. my symptoms is feverish. i already taken 6 mounth madicine AKT-4 kit. then i leave after blood test. but after 1 or 2 mounths i do mantoux test it is positive what i do? i have no kids, i have problem in consive. on feb 2007 i got marriage. plz help me . Doctor: HelloYou were ( are ) a tubercular patient and took treatment for 6 months . Montox's test is still positive even after 2 months treatment stopped . Now for confirmation of tuberculosis get in these tests.==Sputum examination for A F B , if no sputum then get it through nasogastric tube .==Bronchoscopy , because you stopped the treatment so this is important because Montox's is still positive.==Chest X RAY , particularly see healed calcified & fibrosis . Also confirm whether FRESH lesions present or not present.==Sputum for TB CULTURE and sensitivity test.==Blood test for C B C ( leucocytosis in mild to moderate cases of T B ) E S R ( if > 100 mm highly suggestive that still T B is active).==Protein Purified Derivative (PPD0-Tuberculin Skin Test.The main question is about the lesion of tuberculosis . As far as about pregnancy , even after 7 years of married life , consult an INFERTILITY specialist and get his opinion . But I recommend you consult a CHEST & TUBERCULOSIS specialist and get his treatment .Hope you will understand seriousness about T B treatment ."
},
{
"id": 187033,
"tgt": "Is it possible to have a metallic taste in mouth after heart attack?",
"src": "Patient: Is it possible to have a bad/metallic taste in mouth after a heart attack? Or, is this a sign that another attack may be coming? Asking for a friend who just came home from the hospital and is having trouble eating because everything tastes terrible to her. Doctor: Hello, Welcome Thanks for consulting HCM, I have gone through your query, as you have metallic taste in mouth dont worry ot can be due to medication you are taking or nutritional deficiency . You take one capsule vitamin B complex once daily for 7 days Do mouthwash chlorhexidine twice daily for one week Hope this will help you. Wishing you good health."
},
{
"id": 34802,
"tgt": "Does mono cause exhaustion followed by headaches and fevers?",
"src": "Patient: I had mono about 3 yrs ago. I am 50 now. The mono was really bad and I had to miss a few months of work but was still very fatigue for about 8-9 months. Every time I really exhausted now, I seem to get these slight headaches and fevers. Every time I get sick, it seems to take longer to recover cause I get these fevers and headache, light headedness afterwards/ could this be a long term affect from the mono? thanks, Deb Doctor: Hello dear,Thank you for your contact to health care magic.I read and understand your concern. I am Dr Arun Tank answering your concern.Yes, infectious mononucleosis can occur as chronic infection.Usually the infection of infectious mononucleosis is very benign, it will be cured without any adverse consequences.But in some cases it can be seen as chronic infection this is more common in immunocomprmise states , organ transplantation and cancer chemotherapy. The prognosis for this conditions are very poor.I suspect you are having chronic infection. For the diagnosis purpose you can take the viral antibody test done. Treatment of the chronic condition is very challenging. Only chronic fatigue specialist can help you.I will be happy to answer your further concern on bit.ly/DrArun.Thank you,Dr Arun TankInfectious diseases specialist,HCM."
},
{
"id": 27649,
"tgt": "What causes bleeding and bruising for small injuries?",
"src": "Patient: I am a 76 year old white male. I have some partially blocked arteries and my cardiologist is treating me for this. As part of my daily meds, I take 3 fish oil gel-tabs, a baby aspirin and a 3 garlic gel-tabs daily. My doctor knows this and these supplements seem to help me with my circulation. I am easily prone to bruising under my skin and if I get stuck by a minor tree branch or other things, I sometimes bleed . Would these 3 supplements have a tendency to cause my bruising and bleeding? Doctor: Sir with aspirin many of my patients have a tendency to have spontaneous skin bruises, particularly after 70 years when the skin is a little lax. Apart from cosmetically ugly, these lesion with go in some time with no serious complications. The other two drugs are supplements. Omega acid doesn't inference much here, however garlic tablets have anti platelet action and may aggravate the bleeding tendency, the exact proportion of which is unknown as there are no studies quantifying it's action. However I would like to reassurance you for the same"
},
{
"id": 89741,
"tgt": "Suggest treatment for constant upper abdominal pain",
"src": "Patient: I am 33 yrs old wt 55kgs, height 5'3''.I had Pain in right flant radiant to right side of lower abdomen for the last two months. I got the ultrasound done which reveled bulky Lt Adnexa measuring 4.7X4.0X2.9coms(29cc volume) which as per the doctor is incidental finding. Minimal Free fluid is seen in the pouch of Douglas. I don\u2019t have any pain in Lt side. Also small bilateral renal concretions were found. Doctor have suggested antibiotic Amphy IBL for 10 days,which relieves my pain but now I\u2019v a constant stomach ache in the upper abdomen\u2026.Please suggest because the pains are now really getting onto my head I\u2019m working and my job requires lot of traveling and standing most of the time.TLC 12400 DLC N15L14Urine 10-15 pus cells. Doctor: Hi.Thanks for an elucidate history. The history and the findings and the lab reports are well connected to each other .You had urinary tract infection in the past which I think was half treated. It is possible that you have an obstructing lesion in the right ureter like a stricture or so .Now this looks to be an Infection in the Kidney hence the reports. I would advise the following:First of all give the sample of the urine for culture and sensitivity before starting the antibiotics. Once the sample is given you have to get started on an antibiotic most suitable for UTI like Ofloxacin or so. Once the infection is settled and f your blood urea and creatinine are normal you should go for IVP- intra-venous Pyelography, CT scan of the abdomen.Further management will depend upon the findings. Once you start the treatment, do not stop till you get a complete relief"
},
{
"id": 27389,
"tgt": "Taking Pradaxa, Sotalol and Isosorb for A-fib, is ablation required?",
"src": "Patient: I have a history of heart disease and have had a few bouts of Afib. Been taking Pradaxa, Sotalol and Isosorb. Had an ablation about a year ago and they stopped the Sotalol. Afib returned and I was put back on Sotalol. Lately, I ve had ongoing PVC s and PAC s for weeks. Was given Magnesium but they continue. Doctor is suggesting another ablation but I m not confident. Doctor: Atrial fibrillation had to be controlled, if it causing increase in heart rate it is life threatening. If not controlled with medicines, follow your cardiologist opinion."
},
{
"id": 78638,
"tgt": "Suggest treatment for chest pain while breathing",
"src": "Patient: I m a 46 yo female who went back to boxing after a 4 mouth leave due to travel/work. About 10 minutes into punching the hard bag, I felt my heart beat quickly and hard in my chest. It actually hurt to inhale 1/2 way. Later that night, I had pain in my left collarbone area and across my chest. I slept sitting up for 3 nights and then was only able to sleep on two pillow on my back. This happened 8 days ago. Today I feel also like me. However, I feel tired when I am doing normal activities like walking up stairs, lifting laundry, etc. Normal breathing is ok, but to take a deep breath still hurts slightly on the left side of my chest. Can you tell me what happened? Doctor: Thanks for your question on Health Care Magic. I can understand your concern. Possibility of musculoskeletal pain is more in your case. Your symptoms started only after your boxing practice (exertional). So chances of muscular pain is more. But better to get done ecg and 2d echo to rule out heart diseases because you are having left sided chest discomfort. If ecg and 2d echo are normal than no need to worry for heart diseases.Do following things for better symptomatic relief. 1. Avoid heavyweight lifting and strenuous exercise. Avoid movements causing pain. 2. Start painkiller and muscle relaxant drugs. 3. Apply warm water pad on affected areas. Don't worry, you will be alright. But better to first rule out heart diseases. Hope I have solved your query. Wish you good health. Thanks."
},
{
"id": 92555,
"tgt": "Had hernia repaired. Now paints in left abdomen. Have left hand variocoele. Is it related?",
"src": "Patient: I've seen a gastrointestinal consultant about pains in the lower left abdomen in a similar position to where I had a hernia repaired 20 years ago. The pains are also accompanied by severe wind. The hernia was a congenital inguinal hernia that was accompanied by a hydrocoele. The consultant has carried out a CT scan and a colonoscopy and ruled out a reoccurence of the hernia. He says there is a little evidence of wear to the bowel but nothing serious. However, the pain is there. Both the pain and the wind sometime get less but at other times they are both severe. My family doctor says I also have a left hand variocoele, which may or may not be related, though it too causes quite a bit of discomfort at times. I am occassionally feeling the need to pass water more frequently but this sensation isn't always present. All these symptoms feel very similar to my original hernia and the pain is in a similar place but have had no more than the occassional bit of discomfort since the original surgery. I'm therefore still worried it may be something else. I'm happy to accept I don't have a serious bowel problem but something is not right. I don't know what to do. Doctor: if everything is normal you may have food protein allergyget allergy doctor consultation who can find by blood serum test to which protein you are allergic and after witrhdrawl of protein will be better"
},
{
"id": 193520,
"tgt": "Will testosterone shots increase semen and make me more fertile?",
"src": "Patient: I just started taking testosterone shots onces a week and i don't want any kids a friend of mine said to me that there is another shot i could take to increase semen not that i'm trying to be more fertal i just want the best felling sexually he also said it would build muscle i don't remember the name of this shot i think it started with an (h) can you recommend or do you know what he could be referring to? Doctor: Hi, Yes, I know what your friend is suggesting .testosterone can increase your sexual performance but only to a small extent. It's like watering a plant. If you pour more water to plant will it grow faster or you need to consider testosterone only as medical use, not for muscle building or result. Hope I have answered your query. Let me know if I can assist you further. Take care Regards, Dr S.R.Raveendran, Sexologist"
},
{
"id": 35606,
"tgt": "Is it okay to get tattooed after having staph infection?",
"src": "Patient: is it ok to get tattooed after having a staph infection? in june of 2010 i had a staph infection on my spinal cord..i had surgery they removed the abscess and after a 3 or 4 months of recovery i was free of the infection so i was wondering if it would be safe to get a tattoo? Doctor: Thanks for asking in healthcaremagic forumIn short: avoid tattooExplanation: Tattoos can cause infection by carrying surface organism to deeper tissue. So, it is better to avoid in your case though usage of sterile instruments can reduce/minimize the infection rate. Good luck."
},
{
"id": 1550,
"tgt": "What are the chances of pregnancy in case of missing a contraceptive pill?",
"src": "Patient: Hi im on the pill rigevidon and i've just finished my months worth of pills, im waiting for my period to come but it's not happening! i did miss one pill once around the time i was having sex ( two week period). We did and we didn't use a condom, bacically he was inside me but then pulled out, put a condom on and then carried on. is there a chance i could be pregnant and if so what are the choices from then on? Ellie Doctor: Hi, you can clear your doubt by doing a urine pregnancy test at home. If positive consult a doctor if you don't want to be pregnant. If negative, wait for your periods. It takes around 1 to 2 weeks for periods to come after stopping the pills. Also, you have missed a pill, chances are there for pregnancy. Hope I have answered your question. Regards Dr khushboo"
},
{
"id": 133057,
"tgt": "Why can hands not be bent after elbow surgery?",
"src": "Patient: I had surgery on my right elbow where they took the muscle that goes over the elbow and moved it over to the side and inbeaded the nerve in it. So now I have limited movement an not bend it all the way out or up. And it hurt to bend it up and out or pick up stuff more then 5 lbs. the nerve test comes back ok. but I still can not bend it either way so my question is it that cause of the muscle is bad. or maybe scare tissue. I getting frustrated cause the Dr. will not listen to me all I get it the nerve is ok. Doctor: hithank-you for providing the brief history.As per the history this looks more of the post surgical stiffness. I will advice you to undergo a thorough physical examination post which undergoing physical therapy should help you regain the physical movements.Regards Jay Indravadan Patel"
},
{
"id": 5604,
"tgt": "Taking metformin for PCOS. No periods. Prescribed with hypnoid. Will I be able to conceive?",
"src": "Patient: hi doctor im 24 years old i got married 4 months before .i have pcos so my doctor gave me metroformin 500 2 pills for a day .but i dint get my period then i checked cravindex but its shown negative then my doctor suggested me to take hyponidd 4 days before..still i dint get my period.could you pls tell me can i concieve ??..im worried alot pls help me Doctor: Hello Thanks for your query. PCOS is characterized by irregular periods, delayed periods, scanty menstrual bleeding etc. You need to get yourself thoroughly investigated and treated for PCOS. Women with PCOS often have difficulty in ovulating and hence in conceiving. Weight loss and lifestyle management is the mainstay of treatment for PCOS. You have to maintain a healthy lifestyle, exercise regularly and eat sensibly. Metformin will help you regulate the condition to a certain extent. For conception and ovulation, you might require ovulation inducing agents such as clomiphene citrate. Please consult an Infertility specialist for thorough evaluation. Take care."
},
{
"id": 91459,
"tgt": "What causes swollen right testicle and burning in lower abdomen?",
"src": "Patient: hi i am 33 yr i have this problem 1 month ..my right testical is swelling and lowerabdominal is burning all the area is burning ..i went to a doc 10 days before he take urine and never unswer ,i had back pain 4 months. the doc give antibiotic but i did not use good anough. Doctor: Hi ! Good evening. I am Dr Shareef answering your query. With all probabilities, you have got a urinary tract infection with spread of the infection to the testicles. If I were your doctor, I would go for a urine routine, micrscopic and culture sensitivity test and depending on the report, I would prescribe an antibiotic, which you might have to consume at least for 10-14 days for eradication of the infection. Apart from this, I would advise you a HbsAg and HIV test as a routine. If you are sexually active with multiple partners, then I would advise you to avoid it till you are cured, and then take appropriate precautions for the same.I hope this information would help you in discussing with your family physician/treating doctor in further management of your problem. Please do not hesitate to ask in case of any further doubts.Thanks for choosing health care magic to clear doubts on your health problems. Wishing you an early recovery. Dr Shareef."
},
{
"id": 211870,
"tgt": "Induced coma, in rehab, burning during urination. Have severe sepsis and contagious sepsis. What is going on?",
"src": "Patient: my friend is in rehab after being in an induced coma in ICU for 20 plus days with dx of severe sepsis. went to rehab then had an abscessed ruptured appendix back to hos now again in rehab. ready to be discharged soon. tonight the staff moved her into a private room saying she had a contagious UTI and are administering oral antibiotics. she is experiencing no burning upon urination or discomfort. they did not do a urinalysis either... what could be going on??? she has excellent medical insurance Doctor: Hi,Thanks for asking your query.Your friend has gone through a really tough time and still fighting the sepsis.At times there may some infections remaining in the urinary tract which need to be treated aggressively even if the patient is having no complaints. Such infections are seen more in patients who are comatose for a while and having a urinary catheter inside.These infections must be treated before patient is sent home and that is what the doctor's are doing right now.Hope this helps."
},
{
"id": 32590,
"tgt": "Suggest treatment for chronic vomiting and influenza",
"src": "Patient: My son is six years old. He has woken up twice in the last two weeks vomiting, but feels fine in the morning and doesn t have any other signs of the flu. He has also been having unexplained outburst of crying and in a very odd mood. He is getting plenty of sleep. Im concerned that this could be more than just being a kid. Could it be a medical condition? Doctor: hello,well-come to healthcare magic,i concern your question.may i concern it may having intestinal infection marked by watery diarrhoea,abdominal cramps,nausea or vomiting and some times fever(stomach flue).the most common cause of way to develop viral gastroenteritis in children often called stomach flu is by contaminated food or water.there is no other effective treatment for chronic vomiting for viral gastroenteritis ,so prevention is avoid food and water that contaminated.if u wish to have medication u may consult your family doctor about anti-emetic drugs or proton pump inhibitors.frequent hand wash are best defence in children wish your child a very healthy life.Take care.thank u."
},
{
"id": 31231,
"tgt": "Suggest treatment for fever and cough",
"src": "Patient: Hi, I'm a 27 year old man and early this morning, after only a few hours of sleep, I suddenly woke up feeling unable to breath properly, as if something was blocking my throat. In an effort to breathe I was doing this weird cough-like function, like 10 times, which didn't change the situation. I felt that only minimal or no air at all was allowed in. I had a bottle of water near me so I drank some and after that I was able to breathe better, though I don't know if that's what helped me. However I was somewhat scared since this has never happened to me before, so I went to the ER.There, the doctor did a few basic tests, heard my breathing, checked my throat, took my blood pressure and a blood sample for blood tests, mostly to see the oxygen levels, as well as a chest area x-ray, and asked me the usual questions, like if I've been sick and taking any medication, which are both negative. He also asked me if I felt anything weird still and I said my throat was sore though it may have been just because of the effort it took to breathe earlier.The doctor found everything normal, my blood pressure was surprisingly good too since I usually have a little bit hypertension, so he sent me home and I was frankly relieved, though I didn't feel like going back to sleep, in the end I fell asleep. It has now been 5 hours since the incident, and my throat still feels sore and weird, perhaps more so than previously, so I'm worried the episode will happen again.Other information the doctor asked me, in case you need it too, is if I smoke, which I don't personally though I live with chain smokers so the house is often full of smoke, if I have any known allergies, which I don't, to my knowledge, as I only remember having an episode like 10 years ago but we never figured out what it was that I reacted badly to, with rashes and itch all over my body at the time, and it has never happened since. Anyway, I also feel like I may be coming down with a fever now, though the doctor took my temperature as well and while I didn't ask how it was he didn't mention anything about it.So, what could this be, waking up unable to breathe, and still feeling sore after 5 hours despite the ER doctor finding everythingi n order? Doctor: I have read all your question in detail first I would like to ask you* do you have snoring?* frequent Nasal stuffiness/blockade ?I want to know this also because sudden awakening from sleep and feeling of sob may be due to sleep apnoea as well .and of course your home environment (chain smokers) is also not good.and sore throat with fever may be due to viral or bacterial infectionso you can take some broad spectrum antibiotics, take hot fluids and I would suggest saline gargles as wellI hope my answer will give you satisfactiongood luck"
},
{
"id": 204234,
"tgt": "Does Gianvi cause depression and headache?",
"src": "Patient: Hi - my pharmacy switched my birth control from Loryna to Gianvi. I had been on Loryna for 7 months with no issues at all. I originally was prescribed it for my acne, and it didn t seem to help my acne a ton, but I felt normal, no serious side effects. I took my first Gianvi pill last night and all day today I ve felt depressed, been crying uncontrollably and had a headache. I m not sure if this is because of the change in birth control, but it seems far too coincidental to just ignore. I m wondering what should I do - should I continue to take the Gianvi until I can see my doctor? Or should I discontinue the use of the Gianvi asap? Should I go to my pharmacy to see if I can get back on the Loryna? Not sure what to do. Thanks! Doctor: Hello and Welcome to \u2018Ask A Doctor\u2019 service. I have reviewed your query and here is my advice. Yes, Gianvi is associated with depression and headache. But single use of Gianvi will not cause depression. If you are using it for some time and then the depression symptom occurs it can be related to Gianvi. You need to be shifted on some other contraceptive if the problem persists. Best Wishes."
},
{
"id": 9049,
"tgt": "How can I maintain my skin complexion ?",
"src": "Patient: hiii doc ma skin complexion iz dam fair bt nw itz bcm toooo mch darker i cnt take this ny more Doctor: Hi,Welcome to HCM. Read your problem.You can use sunscreen lotion containing SPF 30 over exposed parts of the body This will avoid skin darkening as well as sun damage to the skin.You can also use basen powder mixed with turmeric to improve your complexion. Take care."
},
{
"id": 38115,
"tgt": "Suggest treatment for a cat bite",
"src": "Patient: Hello, I am currently fostering kittens. One has bitten my index finger today around 6:30 PM where it completely punctured it through the other side. It was bleeding for about 45 minutes and now it has stopped. I cleaned it with peroxide and alcohol then but antibacterial cream on it. Also, I had taken amoxicillin that was left over from another issue a while back not pertaining to a cat bite. My index finger is extremely swollen, hard, red, numb, and painful. The pain is not moving to the other parts of my body. Also I have other scratches on my other hand from the same kitten. I rather not be sent to the ER if it is not infected because I can not pay for my medical bills at this point. I am very very worried since the swelling is not going down. I do not want to lose my finger and I am very worried. Please advise and let me know what actions I should be taking. I am very scared. Doctor: Hello,Welcome to HCM,Rabies is a 100% fatal disease but 100% preventable by proper and adequate treatment. Dog is the known reservoir of rabies virus and can transmit rabies by biting.As you were bit by the cat following which there was bleeding and swelling of the index finger. You need to take Inj TT, active immunization with anti rabies vaccine and passive immunization with equine rabies immunoglobulin (ERIG) around the wound.ERIG is advised as there is bleeding from the site of bite. In addition to this you need a course of antibiotics like Tab Augmentin for five days and NSAIDs like Tab Ibuprofen.Thank you."
},
{
"id": 183038,
"tgt": "Suggest treatment for tooth ache after treated tooth abrasion",
"src": "Patient: My name is Sam and I recently had a tooth ambrasion repaired by my doctor and I am pleased that it is no longer visible and I can no longer place my nail between my tooth and gumline. But... I am still in pain as if I have it. When I press my thumb down it feels like the same pain is there. I am not sure if this is a permanent feeling or what.?? Doctor: HiThanks for writing in.If you experience pain even after filling then it indicates you might require root canal treatment in that tooth.Consult your dentist for further opinion.RegardsDr. Neha Sumra"
},
{
"id": 123261,
"tgt": "How to treat pain due to scoliosis?",
"src": "Patient: I have pain due to scoliosis. At times of pain I take Etoshine MR. Some times I have to take it for upto 5 days or in extreme cases 10 days. Often for just a day or two. Is it ok if I stop with a few tabs? Is there a recommended dosage? Is it an anti- biotic to be taken for a minimum period? Venkatesan Doctor: Hello, For scoliosis pain, I will advise taking a short course of the anti-inflammatory drug. Having a temporary kneading massage will be good. It would have been great if x-ray reports were shared or at least the level at which the scoliosis is formed and the angle of it. I will advise indulging in regular breathing exercises because due to scoliosis the respiratory system may get compromise and make more prone to respiratory problems. Also, doing exercises for the spinal muscle strengthening, correction of scoliosis by posture, activating the muscle which is affected will be helpful. Hope I have answered your query. Let me know if I can assist you further. Take care Regards, Jay Indravadan Patel, Physical Therapist or Physiotherapist"
},
{
"id": 58519,
"tgt": "Elevated WBC, distended abdomen, weightgain, mild fatty liver, bedridden. Suggestions?",
"src": "Patient: My husband is 43 yrs old and has felt bad for close to 3 weeks now. He had elevated WBC about 10 days ago and they thought maybe sinus infxn & put him on Bactrim & prednisone . After a week his wbc was still 13. That may or may not be connected to the rest. He has had a distended abdomen to the point that at night when he s lying down he has a hard time breathing because he says he feels full . He also feels like his gut is swollen after he eats. He has had an almost 20lb weight gain in the past month. After 3 visits to MD and being told liver function tests were normal but cholesterol was high, we went to the ER. They said today his SGPT was 80 ( it was 31 last wed), also said that US & xrays did not show any fluid or anything else. Said they saw a CT from almost a yr ago that showed he had mild fatty liver & suspicious of NASH. No one ever told us that then. Today they said yes it s there but did not recheck to see if it was worse & said as stated above that his SGPT was way high and may need to be rechecked in a week or 2 and maybe see a GI but thats all they said they could do. He feels really rotten, dont have the energy to get out of bed for more than 10 minutes, has been that way for almost 3 weeks, and now feels like he cant even get any help. Any suggestions? Could this potentially become life threatening? Doctor: Hello, Thanks for the query to H.C.M. Forum. There was elevated level of W B C but the level as you mentioned ( 13 000 c mm) is not very high . This level may be due to any mild degree infection. Prednisolone in itself causes fluid retention ,so please mention the the dosage of prednisolone took your husband. Now his sgpt level is high 80 I U ,however this raised level is not an alarming condition and not so specific . As you stated that your husband is bed ridden and feeling so tiredness and even breathlessness, so evaluation of diagnosis is very important . Further you mentioned that ultrasound is normal at normal at this time . Further stated that there is no fluid in abdomen . I would like to say that there is a very serious disorder according to your stated history of illness . In my opinion please consult another physician and get his opinion as this time condition is not good. In early stage if NASH ( NONALCOHOLIC STEATOHEPATITIS) fatigue & weakness are most important symptoms. While further symptoms are weight gain and cirrhosis . So don't delay. Good luck. Dr. HET"
},
{
"id": 189015,
"tgt": "Extremely painful gums on upper left side, bleeding when touched, painful lump on lower jaw bone. Serious?",
"src": "Patient: I have been experiencing gum pain on my upper left side of my mouth. I cannot chew on this side and it is extremely painful to touch. Yesterday, when i touched the gum and it started bleeding, my cheek is a bit swollen and i now have a lump on my lower jaw bone close to where it connects with my neck, that is also painful to the touch. I do not have dental insurance until the 1st of June so i was hoping to hold out a few more days. Should i have this checked out immediately based of the symptoms or can i wait a little while longer? Thank you. Doctor: Hello and welcome to HCM,The presence of painful, tender and bleeding gums suggest inflammation of the gums.Gums can become inflammed in conditions like infection, improper brushing, formation of pockets (as a result of infection/ inflammation), etc.You need to consult your dentist for clinical examination and management.Management comprises of removed of pockets, use of mouth wash, and maintaining good oral hygiene by proper brushing.Presence of lump in lower part of the jaw bone can occur due to many reasons.It can be due to cyst, soft tissue growth, bony growth, etc.You need to consult your dentist for this problem also.A X-ray/ OPG may be required to look for the site and origin of the lesion.In case the swelling is due to some mass, a biopsy may be required.Further management will be guided by the result of the investigations.Thanks and take careDr Shailja P Wahal"
},
{
"id": 59777,
"tgt": "Pain, numbness in lower abdomen, blood red stool, have liver lesion. Treatment?",
"src": "Patient: I am having slight pain in my abdomen and numbness in my lower abdomen and my stool is pure blood red, I called my doctor who said call the ambulance but I am waiting on my boyfriend to arrive here to take me to the ER, in the mean time I need a second opinion??? I supposedly have a 1.8 cm liver lesion- IM BLEEDING AND I HAVE TO PAY FOR AN OPINION? REALLY? NO,. Doctor: Hello! Thank you for the query. Such bleeding can be caused by ruptured hemmorrhoid (most often - the blood bright and not mixed with the stool), colon polyps or colon cancer (blood is mostly bright and mixed with stool). There is a possibility that such blood can come from stomach or small intestine but it is usually dark so the stool gets black. Lesion in liver can be nothing but it ca be also a metastatic tumor. It should be diagnosed with biopsy. If there is a lot of blood in your stool and you feel weak this is urgent and you should have colonoscopy performed as soon as possible. Hope this will help. Regards."
},
{
"id": 159694,
"tgt": "What is the stage of my breast ca ?",
"src": "Patient: What is the stage of my breast ca size--3.8x3x3 cms nodes axillary nodes 13/15 + apical 2/4+ histological grade 3 nucleus comedo pet/ct scan-sternum c6 d12 i5 increased uptake with few osteolytic changes increased uptake in l234 with osteolytic changes nodes free skin free dctal ca x-ray chest normal ct scan reports c6 involvment Doctor: Hi. You have metastatic breast cancer which automatically becomes Stage IV. Now that you have baseline PET scan, follow up PET scan is the best way to monitor response to treatment. Repeat PET scan is best done 8 weeks after chemo and 12 weeks after radiotherapy. Interim PET (done in between chemotherapy) is best performed a few days before the next cycle of chemotherapy"
},
{
"id": 89836,
"tgt": "What is the sharp paain in the lower abdomen along with delayed periods?",
"src": "Patient: Hello, not long ago i felt from my bottom belly button, down to the top of my vagina, a sharp pain go down. Quick, straight, no curves, a long with that besides the sharp pain, after i had a really bad cramp, i have irregular periods and hvent gottwn mine its been 2 weeks, i tested on the 1st week it was late, but got a negative result, im going to test again soon but what could it be? I poop and pee well, and i workout Doctor: welcome to Health care magic.1.Your history suggest that its related to the PID - pelvic inflammatory disease at first place. Pregnancy can not be ruled out, need to get your test done as you already had a missed period.2.For confirmation an ultrasound abdomen pelvis would help to evaluate exact cause in this case. 3.Get an appointment and see your doctor as soon as you can, and get the needful tests done.4.In car of PID a course of antibiotic would help.5.Hope it helps you. Wish you a good health.Anything to ask ? do not hesitate. Thank you."
},
{
"id": 1630,
"tgt": "Can pregnancy happen while on birth control pills?",
"src": "Patient: Hi. I am eighteen years old and a virgin. Recently, my boyfriend and I got close to having sex. We were just messing around, so we didn't think much of it...but we slowly moved to almost having sex. But, we did not use a condom. His penis went in my vagina, but not fully. It was probably less than 10 seconds long that he was in when I decided I did not want anything more to happen. I am on a birth control pill, but I was on antibiotics at the time so now I'm worried that I could be pregnant.. Doctor: Hi, if there was no ejaculation by your boyfriend, then there is a very little chance that you can be pregnant. Also, you are on birth control pills, it also prevents pregnancy. Hope I have answered your question. Regards Dr khushboo"
},
{
"id": 199814,
"tgt": "What could be the reason for having white discharge from penis?",
"src": "Patient: I have small amount of white discarge from penis. Reoccurring problem, this is the third time. Tested for STDS, negative; urinary tract infection, negative. They thought prostate infection. The only thing I can see in common with each time the discharge started was that I drank beer the night previous. Doctor: Thanks for asking in healthcaremagic forum Any discharge from urethra need to be investigated for Sexually transmitted disease/ urinary tract infection including prostatic infection. I dont think there is any relation ship between beer and urethral discharge, may be mere co-incidence. So, please go for a followup with your doctor to rule out prostatic problem. All the best."
},
{
"id": 58238,
"tgt": "Have torn meniscus, damaged ACL, severe arthritis, shoulder, hands, arms sore, unbearable pain, have to wait for knee replacement. What should i do?",
"src": "Patient: Hello, 56 year old female, who after 2 bad falls on the ice last March, found out I have a torn meniscus and damaged ACL , but artritis is so severe, they can only do a total knee replacement , but after having a steroid injection in each knee, and having it only help me for 3 weeks, I have to wait 6 months for a knee replacement, pain is off the charts... I assumed it was osteo arthritis, but lately my hands, fingers, wrists, elbows and up my arms to shoulder are so sore I can hardly get dressed or move them, Ive neve experienced that before. I am on Meloxicam(15MG), have been on Hydrocodone since March this year, and will have to continue it till at least March of next year, and Tylenol, so have a feeling, my liver is pretty toxic... my arm and hand symptoms make ME think this has to be Rhuematoid Arthritis instead, something is going on there, I have a raised lump on back of one hand, the hand I cannot even lift anymore......any suggestions? karen Doctor: HiCheck a blood routine test.If it is a systemic illness like rheumatoid arthritis it will give a high ESR.You can check RA factor and ANA profile also.Other conditions like a high blood cholesterol and unccontrolled BP gives such a picture also.You can take intraarticular injection of hyaluronic acid also.give knee cold compress.it also give some relief.Hope this may help youthanks"
},
{
"id": 196773,
"tgt": "What causes acute fever?",
"src": "Patient: My SGPT is 304, and SGOT 191.2. I was running a temp of 101-102 on/ off since last 4 days, and was taking Ibugesic to lower the temperature. I dont drink at all and not obese either. I only take Thyronorm 50 everyday and the TSH shows 4.46. There are no other symptoms e.g. no yellow eyes, no dark urine, Can you help ?. Doctor: Hi and welcome to Healthcaremagic. Thank you for your query. I am Dr. Rommstein, I understand your concerns and I will try to help you as much as I can.AST (SGOT) and ALT (SGPT) are reasonably sensitive indicators of liver damage or injury from different types of diseases or conditions, and collectively they are termed liver tests or liver blood tests. So the next step is to look for certain liver damage and most common casue is fatty liver disease. It should be confirmed by ultrasound. Other causes are viral hepatitis,cirrhosis, medications, alcohol intake or autoimmune diseases. To verify exact cause, US, bilirubin levels and tumor markers should be additionaly done Then appropriate treatment can be started.I hope I have answered you query. If you have any further questions you can contact us in every time.Kindly regards. Wish you a good health."
},
{
"id": 40023,
"tgt": "Recovery time for chicken pox?",
"src": "Patient: Hi! I'm a 16 year old guy. Doctor confirmed it to be chicken pox. It's been five days now. Most of the blisters have turned black but not yet flattened. I've been applying neem juice on the body and face to reduce itching. There are no blisters in my mouth. Is it okay to eat home made snacks? Also, how many more days will it take for me to heal completely? Pls do reply. Thanks! :) Doctor: Hi, thanks for using HCM in my opinion the rash will disappear in next 1 week but scares will take another couple of weeks to go . Yo can definitely have home made food. No need to apply need juice now as you are already in your recovery phase."
},
{
"id": 31401,
"tgt": "Suggest a remedy for fungal infection on the thighs and scrotum",
"src": "Patient: I have fungal infection according to my research it is Tinea cruris in the inside of my thigh and on my scrotum. Sometime I do get pimples around my infected area. It has been present for over 10 years now. I have been taking terbin and sebifin ointment from time to time when the infection worsens. How do i get rid of this fungal infection permanently? Doctor: Hi thanks for asking question in HCM.Whenever you are infected with fungi apply local antifungal ointment over it that you have mentioned.After treating infection preventive care has to be taken.here is few suggestion.Wear clean cloths and undergarments.Wash cloths in warm water if feasible.Make affected area clean with taking proper bath and keep that area dry.Moisture aggravate fungal infection.Warm water bath helpful.especially salt added war water helpful for bathing.Take healthy nutritional diet with more fruits and vegetables.Garlic, cocconut oil,onion like foos found to be helpful.Olive leaf paste made by water can be applied over affected area,found useful in few study.If severe fungal infection oral antifungal tablet can be taken.My suggestion will surely help you."
},
{
"id": 31977,
"tgt": "What could sinus infection at night suggest?",
"src": "Patient: I've recently had a bad sinus infection - but have been clear of most symptoms for about a 10 days. Beginning 2 days ago, I've been experiencing airway constriction that has led to difficulty sleeping and apnea-like reactions. This came on suddenly and usually only bothers me at night. However, now on day 3 of this, I'm started to get concerned. Doctor: Hi Dear,Welcome to HCM.Understanding your concern. As per your query you have symptoms of sinus infection at night which are pointing towards condition called sinusitis. I would suggest you to take steam 2-3 times a day to relieve congestion, drink plenty of fluid , give hot compresses to face ,do warm saline gargles and if you are allergic to penicillin then you can opt for amoxicillin (under expert advice) . You can also take ibuprofen (if not allergic) or acetaminophen for pain and swelling. For allergy take over the counter antihistamine drug like benadryl. Consult ENT specialist (ear, nose ,throat) for proper examination and treatment . Doctor may order blood test or X-ray. He/ she may prescribe antibiotics course nasal corticosteroids , antihistamines and decongestant (oxymetazolin).Hope your concern has been resolved.Get Well Soon.Best Wishes,Dr. Harry Maheshwari"
},
{
"id": 29366,
"tgt": "Suggest medication for hives apart from Keflex and Doxycycline",
"src": "Patient: I took two days of keflex two weeks ago for a bacterial infection from a splinter. The dr. changed my antibiotic to doxicycline. 4 days later another big rash on my belly. It has been two weeks and I still have hives everyday. How long can this occur? They gave me a steroid but it made my bp go up. Plus, I already have high bp ( treated though) Thank you. Doctor: If you are having rashes two weeks after stopping the medications, then it is unlikely that it was related to them. Is there anything else that you could be having an allergic reaction to? It may also be worthwhile to see a Dermatologist to help diagnose the cause of the rash. I hope it resolves soon."
},
{
"id": 214179,
"tgt": "Suggest natural cure for diabetes",
"src": "Patient: I am currently a boarder line diabetic....I do not want to start pills. I am 57 years old. I have a family history if diabeties. Is there a \"natural\" tea etc that can waylay the pills? Thanks, Linda YYYY@YYYY Currently the doc wants me to lose 10 pounds and increase exercise to 150 min's a week....I currently weigh 147 and I'm 5' 4\" tall. My cholestrol is also high....I currently take meds for that. My blood if 5.6 (test for being a diabetic?) I feel so bad because I really try hard to eat properly etc....any suggestions would be appreciated!!!!! Doctor: Hello and Welcome to \u2018Ask A Doctor\u2019 service. I have reviewed your query and here is my advice. First of all, you need to do laboratory tests to confirm diabetes, which include: fasting blood sugar, random blood sugar, 2 hours postprandial HbA1c. Another important point to consider is that, there is no cure for diabetes. All the treatment methods including life style changes, diet and drugs to lower and control blood sugar AIM to slow the progress and prevent the hazards of diabetes. I hope I have answered your query. Let me know if you have any further questions. Take care."
},
{
"id": 210974,
"tgt": "What causes sudden weight loss with anxiety?",
"src": "Patient: I have been grieving over the loss of my husband two years ago and have tried to cope with the running of the house, my daughter who has alcohol problems and was giving me serious stress a few months ago. My daughter s children have lived with me since they were very young they are now teenagers. I am a smoker and do drink which has been rather excessive at times. I did have difficulty at times trying to swallow but after a few drinks found it easier to eat without thinking about it. Recently I feel like there is a lump in my throat and food does not go down the other week I choked but it was on a Dorito so it has put me off of eating. The gp has requested a test on my throat and I am waiting for the appointment to be brought forward. Went to a and e explained how I was feeling but although blood pressure is soaring and has been for a while they did not keep me in. Went to gp again yesterday she said chest was clear she thought but there is considerable weight loss gp said it has been over 2 years and not sudden could be anxiety she hopes. My younger daughter thinks I have cancer and wont be around for very long she thinks I have ignored symptoms and been selfish but I have not really done this intentionally and thought it was all down to my grief which has been intense. I know that my symptoms can be caused by various types of cancer but is it possible to have these symptoms and lose weight when grieving? Doctor: HiThanks for using healthcare magicThere are number of reasons for sudden weight loss like cancer, overactive thyroid, depression, diabetes, COPD, addison disease, etc. In your case, if you have underline depression with anxiety symptoms, that may be the reason. May be due to decrease food intake, you have sudden weight loss. You can take tablet like mirtazapine that would increase your appetite and weight. It would also help to decrease underline anxiety. Rest discuss with your doctor.Thanks"
},
{
"id": 134651,
"tgt": "Suggest remedy for severe wrist pain in an elderly person",
"src": "Patient: My 85 YO mother is suffering wrist pain and the doctor has asked her to take Aleve 2x2 daily. She continues to suffer because she believes she cannot lay down if she takes it. She lays down 60 % of her waking hours. Should I try and convince her to take it anyway. She also takes Tylenol 3 for other issues. Doctor: Hi thanks for asking question.The aleve is the naproxen containing analgesic...For pain this drug or simple paracetamol can taken..But search for cause of wrist pain should be done...Does she has any history of trauma..??With advancing age because of estrogen secretion decreased bone becomes weak and lead osteoporosis like changes.x ray of local part done...X ray also detect if synovitis or arthritis related changes if present...Calcium tablet can be given to her.If condition advancing if needed bisphosphonate group of drugs also useful...Take care."
},
{
"id": 9965,
"tgt": "Suggest remedy for hair loss",
"src": "Patient: I think that I am losing my hair but I am not sure why or what to do. I am 22 and recently changed my diet to eat healthily and I work out doing a lot of cardio at least 5 times a week. I've always had full, long brown hair about 2 inches past my shoulders at least but it's suddenly shorter and my hair is half the size in some places. I know that I am not eating enough for the amount I exercise which I find very hard to change as I have been obsessed with my weight for a long time and I am finally 3/4 months into consistent exercise. Doctor: Hello, I have gone through your query and would recommend you to get your haemoglobin, serum ferritin levels, serum vitamin B12 and serum vitamin D levels and thyroid function tests done to rule out any internal cause of hair loss. Start applying minoxidil 2 percent lotion on the scalp twice daily and take hair supplements containing biotin once daily. You need to continue this for 4-5 months to see significant results. Hope I have answered your query. Let me know if I can assist you further. Take care Regards, Dr Asmeet Kaur Sawhney, Dermatologist"
},
{
"id": 2813,
"tgt": "Will i conceive naturally while having 12.3 FSH and 1.3 AMH?",
"src": "Patient: My age is 45. My FSH level was high 12.3 and AMH low 1.3. I am advised donor eggs by gynec. I had conceived naturally in 2011 but had to abort for abnormal birth. Is there any chance for me to conceive naturally again even with poor ovarian reserve. Seek your advice thanks Doctor: Hi, Your fsh is high and your amh is low. Bit you can conceive naturally. Chances are there but the thing is that the quality of eggs decrease with the increasing age and that can result in abortions. You can go for IVF with your own egg but chances of abortions will be high. Thus donor eggs are an option for you.Hope I have answered your query. Let me know if I can assist you further. Regards,Dr. Khushboo Priya"
},
{
"id": 96360,
"tgt": "Black spots in stool",
"src": "Patient: I have had dirrhea for many weeks now. Sometimes all water and color, sometimes there is actual stool in them. My question is, recently I've been noticing black spots in the small stools that I pass. I'm not on any special diet, or new meds. I have meds that I take regularly, but have never had a problem with them. I do get discomfort in my lower abdomen, and somtimes near my diaphram. I have lost a significant amount of weight recently, I get dizzy, light headed, my heart will start to hammer for no reason, and my muscles somtimes get weak. Could this all be linked? What could cause all of this? Thank you for your time. A.S. Doctor: if you are having black spot in stools after attack of diaorrhoea it may be due to transfomed blood you must nearby Physician and go for occult blood test in stool if you were not on any iron rich diet"
},
{
"id": 153760,
"tgt": "What is the treatment for Prostate cancer?",
"src": "Patient: My dad is a 72 y/o he was Dx w/ Prostate cancer 2 years ago he was being treated but the cancer spread to his lungs so they started Chemo and Radiation like 3 weeks ago I guess it was too much for him that a day after radiation he had a heart attack in his sleep I took him to the hospital and immediately they started treating him, the Dr.\u2019s found out he had Sepsis due to his catheter. They did not give me hopes for him to wake up from that, they admitted him to ICU and treated him aggressively w/ antibiotics he did not wake up for 9 days so that Dr\u2019s told me that they were going to give him one more week for him to respond finally he woke up two days ago from today. But know his Dr. is telling me that the cancer has spread almost all over his body and there\u2019s nothing else to do that we need to make a decision. I just want to know if there\u2019s a way he can stay a lil bit more longer or if there\u2019s medicine that can help him keep going. But I don\u2019t want him to go yet. He is showing me that he don\u2019t want to go yet. Doctor: Hi, dearI have gone through your question. I can understand your concern. He has prostate cancer that spread all over body. So it is last stage cancer. No treatment will cure the disease. Chemotherapy and radiotherapy work as palliative treatment. But according to my opinion no treatment should be given. Sorry to say but it is the fact. He has only few months left. Consult your doctor and plan accordingly. Hope I have answered your question, if you have doubt then I will be happy to answer. Thanks for using health care magic. Wish you a very good health."
},
{
"id": 181502,
"tgt": "What causes extreme pain in inner cheeks after putting ceramic bridges?",
"src": "Patient: I had two ceramic bridges put in upper and lower. I also had two extractions, one upper and one lower. When the ceramic bridge was put it in, it hurt right away. Now two months later, I have extreme pain in my inner cheeks almost feels like razor blades. It comes and goes throughout the day and makes my eyes water with pain. I went to the original dentist, talked to the oral surgeon, went to a doctor and an allergist. No one can seem to know what it is. You can see it, but it hurts like nerve damage and is getting worse rather than better. Doctor prescribed prednisone and allergy tablets. I don't know where to go next. Doctor: Hi..Welcome to HEALTHCARE MAGIC..I have gone through your query and can understand your concerns..As per your complain the ceramic bridge hurt you when itself shows that the pain is related to teeth only and chances of nerve pain are less.There is a possibility that you have high points over the bridge and it is causing pain..It can also lead to pressure over the jaw joint causing jaw joint dysfunction..Other reason can be that the teeth over bridge is placed have exposed nerves that gets inflamed and irritated leading to pain.Although less chances but chances of nerve induced pain like Trigeminal Neuralgia cannot be ignored.. I would suggest you to first of all consult an Endodontist or an Oral Physician and get a full mouth x ray done to check for teeth as well as jaw joint..In case if there is any abnormality detected it will be treated accordingly..In case if still nothing is found you can consult a Neurophysician and get evaluated and an MRI can help in ruling out for nerve disorders and treated accordingly..As of now if pain is severe you can take Tramadol and fr moderate pain you can take Ibuprofen.Hope this information helps..Thanks and regards.Dr.Honey Nandwani Arora."
},
{
"id": 30987,
"tgt": "Can hepatitis-C infections be contracted form tattoos?",
"src": "Patient: I am a professional firefighter, I know I am high-risk for contracting Hepatitis C. I recently got a tattoo from a pretty reputable place. What are the chances I could have contracted Hep-C from this tattoo? also I recently got the tattoo when should I get tested if I had any concerns? Doctor: Hi, Thanks for posting in HCM. I understand your concern. Hepatitis C virus is transmitted only by blood. Unlike many other blood borne viruses (like HIV), virtually any source of blood or blood products seems to be capable of carrying the virus, even if the source is indirect, like for example - a used razor. This makes hepatitis C far more transmissible than most other blood borne viruses. Hepatitis C virus is most frequently transmitted through large or repeated direct percutaneous exposures to infected blood. Hence, there is a high possibility of contacting infection with Hepatitis C virus by means of getting tattoo as it involves repeated skin pricks. Hope the information provided would be helpful. All the best."
},
{
"id": 149248,
"tgt": "Have conversion disorder. Tremor in hands, painful arms, shoulder tense. Help",
"src": "Patient: I was was told that I had conversion disorder when all of a sudden on day here at home was watching TV and my hands started to tremor and then my arms, it was very painful, we call 911 and the EMS staff had never seen anything like it. It was very strong and I had no control at all. Since this was my first one by the time we got to the hospital they had stopped as quickly as they started. I have had them now maybe twice a month or more. I can tell when I am going to have one as in my left on top of my shoulder it starts to tense up. There is nothing I can do but try to do some movement as making circles with my arms as the issue continues. It will finally stop and it leaves me exhausted. Doctor: This could be Epilepsy and you will need to have EEG done, there you need to see a Neurologist. Moreover you need Brain CT or MRI. regards,"
},
{
"id": 104806,
"tgt": "Heavy mucus, difficulty in eating or drinking until long hacking episodes to eject mucus. Family history of thyroid. Reason?",
"src": "Patient: I am coughing up a lot of mucus over the past several weeks. The mucus hacking episodes last for 15 minutes of gagging to get this stuff out. It doesn t stop for me until I get out the clump of mucus which is sometimes a dark brown, yellow, or red color. The majority of the mucus is a thick and white. It is to the point where I can t eat or drink until the stuff comes out. It makes me dizzy by the end of the episode of hacking. I don t have any food allergies . My dad s side of the family has thyroid, heart, and digestive problems. My grandma and dad had some procedure done to their throats. I have been tested for hypothyroidism many times and I come back clean. The only digestive problem I have had is with constipation , but I had a colonoscopy and everything was fine. I m not pregnant. I haven t done anything for this except gargle warm salt water and mucunex stuff. Nothing is working. I don t smoke or do drugs. I don t know what s wrong with me. Doctor: Hi Montoya, Thanks for writing your query. After reading your post, it seems that you are having bronchitis and lower respiratory tract infection. First of all you should get the following investigations done :- 1. Chest X-ray PA view 2. Complete Blood Count (CBC) 3. Sputum examination (Culture and Senstivity and AFB testing) Yellow and thick mucus can be due to some infection in the respiratory tract. Red colour in the sputum can be due to some leakage of blood from thin blood vessels as a result of severe coughing . You can take bronchodilators like Levosalbutamol along with mucolytics like Bromhexine. Antibiotics, if needed can be started according to the culture/senstivity report. Steam inhalation and chest physiotherapy will also help to cough out mucus easily. You should get yourself examined medically and investigated before starting any medication. I hope this is helpful to you. Thanks."
},
{
"id": 14853,
"tgt": "Suggest medication for skin rashes?",
"src": "Patient: I was operated due ananl fishers .Doctors prescribed me antiboitics (augumentin 625) i.e. amoxicilin and ZANOCIN- OZ. I took this medicine for aproximately 12days.after that skin rashes (alergy) started .i took Allegra-180 but it is only a time reliver.What should i do now? Doctor: Hi,It seems that you developed amoxycillin induced exanthems. After consulting your doctor, you change the antibiotics. Azithromycine or erythromycine may be safe. Take oral steroids in tappering dose for short period. Anti histaminics like levocetirizine may be taken till you feel relief. Local application of steroid with antibiotics might enhance the result. I hope you got my answer.Thanks.Dr. Ilyas Patel MD"
},
{
"id": 124679,
"tgt": "Suggest treatment for knee cap injury",
"src": "Patient: I fell on the corner on a cement block hitting my knee cap. It has a visible lump, bruising where is was hit and then around the edge of the knee cap. It is warm and painful to the touch. There is also pain the radiates down the outside of my leg into my ankle area. There is stiffness in the knee after resting though the pain gets better with resting. It hurts the most when going down stairs and on the way down to sitting. Doctor: Hello, Consult an orthopedician and get evaluated. As of now you can use analgesics/anti-inflammatory combination like aceclofenac/serratiopeptidase for symptomatic relief. Most probably it is a contusion and fracture is unlikely. If symptoms persists you can consult an orthopedician and plan for an X-ray. Hope I have answered your query. Let me know if I can assist you further. Regards, Dr. Shinas Hussain, General & Family Physician"
},
{
"id": 68851,
"tgt": "What causes lump on upper inner thigh?",
"src": "Patient: I have a large lump on my upper inner thigh, close to the groin area. It is about 5inches long, about 3 inches wide and feels like at least 2 inches deep. I found it about 2 days ago. it has become very painfull. I have an Appt to see my Dr. in about 3 days. but i have started carrying a high fever for the last 2 days. Also, suffer from reacurring acute Pancreatitis. (non-alcohol related). could this high fever cause an Pancreatitis flair up. It kinda feels like I getting one. Any advise please. Doctor: welcome to Health care magic.1.The lump what you are seeing / feeling could be lymph nodes.2.Lymph nodes do arise after any infection / inflammation or underlying malignant causes.3.The pancreatilts will give its symptoms with pain.4.If you are my patient i would ask for a ultrasound scan to see the condition of your pancreas status, and lumps what you feel - to confirm about the lymph nodal mass.5.An FNAC will be good idea to find out the reason behind.6.Get a complete investigate you with lab and imaging - as nodes does not develop simply.7. Hope it helps you.Anything to ask ? do not hesitate. Thank you."
},
{
"id": 177888,
"tgt": "What causes ammonia smelling urine in children?",
"src": "Patient: my grandson wet the bed and at 4:00am woke up screaming and his whole stomach area looked like a very bad sunburn. I have also notices a VERY strong ammonia smell (so strong I had to struggle to breath and leave the room) when washing out is clothe diapers. Is there a condition that we should be worried about and what should we put on the burn like area. He is 2 Doctor: Thanks for the query.The description seems incomplete.But for such a case it is advisable that the child must be taken to a doctor ASAP.The doctor may be able to see other signs and get investigations done for diagnosis."
},
{
"id": 174745,
"tgt": "What is the difference between Hirshsprung s disease and Hershberger s disease?",
"src": "Patient: Infant 3 months old. Wasn t keeping her mothers milk down. GI problems from birth. Had a feeding tube surgically placed @ 6 weeks but continued to gain little to no weight or gain & then lose. 9 pounds currently. Otherwise, alert, responds well to others, holds her head up, scoots. She just can t keep weight on & has stomach pain. The GI Dr.s did a colonoscopy to look for the cause. Did a a biopsy waiting on results. They mentioned Hershbergers disease which if she has it they said they would have to do a colostomy. I looked it up but Hirshsprungs disease also sounds a lot like what they talked about as far as the definition. What is the difference in the two disorders? Doctor: HI mam, I could understand your grief u come across. According to the symptoms and history given here, your baby had a problem from birth, probably due to non migration of nerve cells which will help in control of stools and intake of feeds- Hirschprungs disease.It is not related to Hershberger, which causes contractures and weakness of hands, legs etc. Please follow the biopsy reports and advise of your doctor"
},
{
"id": 219683,
"tgt": "Can withdrawal bleed occur even if I am pregnant?",
"src": "Patient: I had got period on 21st November den I missed a month n thought of taking regestrone . started taking it on 13th night twice a day for 3days I.e till 16 morning. I got my withdrawal bleed on 21st January which was light for the first 2days n den normal. but suddenly on 29,30,31 it became very heavy and I finished on 2nd Feb. my withdrawal bleed lasted from 21st Jan to 2nd Feb. can withdrawal bleed occur even if I m pregnant? Doctor: Usually not, withdrawl bleeding occurs only if there is no pregnancy. If you have further questions, you need to visit a gynecologist"
},
{
"id": 147004,
"tgt": "What can cause bouts of dizziness?",
"src": "Patient: Iv'e been having bouts of dizziness for the past couple of weeks, I don't think it is an ear infeaction as there is no pain or discharge, I haven't stood up too quickly as I get it when i'm sat down or walking or lying down, I don't think it's too low sugars as I had a coffee with sugar and an almond croissant about 2 hrs ago. I'm only 29 and not pregnant. Please advise if I should bother my doctor with this. Doctor: Hello dear,The symptoms as mentioned in your post can be attributed to pathology in the Vestibular Apparatus in inner ear (it is associated with maintenance of body posture & balance) most probably Labyrinthitis or Benign Paroxysmal Positional Vertigo. They may not present with any pain or discharge, but repeated attacks of dizziness.Symptomatic relief can be obtained with intake of Vestibular sedatives like Betahistine or Cinnarizine preparations (to be taken only under the guidance of a Physician).If it still persists, then you need to consult your Physician/ ENT Specialist & get a complete clinical examination done.Investigations like estimation of blood pressure, blood sugar levels, serum electrolytes & vestibular function tests will be required to rule out any pathological cause for the symptoms.There is no need to worry, you will be fine.Till then, maintain adequate hydration & proper nutrition status and avoid stress.And also take precautions of getting up from bed slowly & avoid sudden head movements.Wishing you a Good Health.Take care."
},
{
"id": 103796,
"tgt": "Have breathlessness.Took ER test and was diagnosed with chlamydia pneumonia. What is the cure for this problem?",
"src": "Patient: Hello, For about a year and a half now OFF AND ON I have had trouble breathing, especially taking a deep breath in. I went to the doctor to see if I have Asthma, and I do not. about six weeks ago I went straight to the ER because I felt like I was suffocating and my chest was in a lot of pain! I got diagnosed with chlamydia pneumonia. about three days ago I woke up gasping for air, and since then I have not been able to catch my breath, I have been experiencing weird things since then that I don't normaly expirence. Mucous is suck in my throat and I coughed big chunks up all day today, and I could never seem to get it all out of my throat. Randomly throughout the day my heart beat will start beating fast and I can feel it through my chest. I have been feeling light headed a majority of the day and I do not sleep very much at all anymore, no matter when I go to bed. Doctor: FUNGAL INFECTION OR YEAST ALLERGIES BEHAVE LIKE THIS ALSO SO YOU HAVE TO TREAT FUUNGUS TO GET RID OF YOUR PROBLEMMS ADD FORCAN 150 MG ONCE A WK FOR 8 WKADD ANTI ALERGIC SINGULAR 10 MG BD OR TDSSYP ASTHALIN 1 TSF TDSGELUSIL 2 TSF TDSSYP MITS LINCYUS CODEINE 1 TSF NIGHTCONTINUE 4 - 6 WK AVOID ELIMINATE YEAST PRODUCTS FROM DIET COMPLETELY YOU CAN RECOVER FAST"
},
{
"id": 136019,
"tgt": "Suggest treatment for spondylosis",
"src": "Patient: Have had blood tests and xrays and so far the only thing that shows up is spondylosis. Extreme tiredness during the day and falling asleep, and wide awake at night. One leg and foot swelling....hair loss for several years.....aches and pains and like a biting all over the body.....arms and legs hurt and having hard time getting from laying or sitting position to standing and walking. Bottom and tops of feet hurt...RA ruled out... Doctor: hiyou must get checked up by a spine/orthopedic doctor and an MRI spine may be required. treatment with pain medicines, physical therapy to follow after diagnosisbest wishes"
},
{
"id": 33463,
"tgt": "What causes recurrent sinus infections?",
"src": "Patient: I have recurrent sinus infections. I ve had 2 in the last 5 weeks and my doctor prescribed antibiotics and sterioids. I have had 6 -8 sinus infections the last 2 years with 5 bouts of pneumonia. Why do I keep getting sinus infections and pneumonia? I ve had my tonsils out. Doctor: HI, thanks for using healthcare magicIf you have allergic sinusitis then this may be predisposing you to infections of the sinuses.Treatment of sinusitis involves avoiding any known allergen, use of oral antihistamines and topical steroid nasal sprays.Allergy testing can help to specify the allergen.Imaging of the sinuses may also be needed to see if there is any structural change that would cause sinusitis.I hope this helps"
},
{
"id": 99505,
"tgt": "How to treat difficulty in breathing due to dust allergy?",
"src": "Patient: hi i am 26 y, M. sometimes i feel difficulty in breathing specially when exposed to dust etc, following high fat food routine. i can feel wierd sound from my throat in such conditions it feels like heavy load has been put on my chest. can it be asthama. Doctor: Hello,Thank you for asking at HCM.I went through your history and would like to know more about you like what are your nose symptoms (congestion, sneezing, running nose, etc), how frequent are your symptoms, how they subside, any digestion related complaints, etc. This will help me to know more about you and therefore to make more specific suggestions for you.At present, from your provided information, I would like to make suggestions for you as follows:1. I would suggest you spirometry with post-bronchodilator reversibility testing. This test will help to know your lung capacity and also to know whether you have asthma.2. Such symptoms could also be caused by chronic sinusitis and gastroesophageal refllux disease.3. I would also suggest you allergy testing for common air-borne allergens like house dust mites, molds, pollens, insect proteins, animal dander, etc. This will help you identify the substances causing allergies to you as well as to know the measures to avoid them.4. Based on allergy testing, an Allergist-Immunologist may prescribe you allergen specific immunotherapy which works on immune system to gradually improve allergy symptoms over a long period of time.5. In general, please avoid exposure to dusts, smokes and air pollution as much as possible. Please avoid high fat food also.6. To suggest medications, I would need more details of your symptoms.Hope above suggestions will be helpful to you.Should you have any further query, please feel free to ask at HCM.Wish you the best of the health ahead.Thank you & Regards."
},
{
"id": 14026,
"tgt": "Suggest treatment for rashes and bumps on face after shaving",
"src": "Patient: EVENING DOCTOR, whenever i am shaved with razor blade i develop some pumbs and rushes on my skin? what could be the cause and what is the solution? i have tried to apply some detergents and bump patrol but as soon as the shaving is with a razor blade it comes up. i dont see it when shaved with machineali Doctor: Hello and Welcome to \u2018Ask A Doctor\u2019 service. I have reviewed your query and here is my advice.In my opinion, you rahes are due to folliculitis. Do not apply detergents as they can cause irritation of skin. Apply Fusidic acid cream on bumps for a week. In case if you do not get any improvement, do consult your dermatologist.Hope I have answered your query. Let me know if I can assist you further. Regards,Dr. Siva Subramanian."
},
{
"id": 73155,
"tgt": "What are the chances of passing the nicotine test?",
"src": "Patient: Hello, I recently quit smoking about 2 weeks ago, and have a pre-employment screening and nicotine test comming up for a new job next week. I was a moderate smoker (half a pack per day). What are my chances of passing the cotinine (nicotine) test? Doctor: Thanks for your question on Healthcare Magic.I can understand your concern. Nicotine stays in the body for 48 hours.Since you have already quit smoking since 2 weeks (more than 48 hours), you will mostly pass nicotine test without any trouble.Better not to restart smoking as it is very bad for health. Hope I have solved your query. I will be happy to help you further. Wish you good health. Thanks."
},
{
"id": 36027,
"tgt": "Can patient with addison disease be in contact with person receiving MMR?",
"src": "Patient: I have addison disease. My grand -daughter needs to get a MMR and a chicken pox shot . The shot is a live virus. Am i allowed to be in contact with her since this is a live virus? If I am not for how many days would I have to wait to be in contact with her? Doctor: Dear I can understand your concern . Dear I would like to inform you that there is no such evidence that live virus vaccine can cause virus transmission from one person to another. You can live with her always . There is no need to worry about that . Hope this will help you . If you have any other concerns please feel free to message me . Thanks for contacting HCM ."
},
{
"id": 164281,
"tgt": "Suggest medication for diarrhea and vomiting in a 2 year old",
"src": "Patient: I have a toddler (2) who has had diarrhea going on three days now.. at first I thought it was bc her body was fighting off an infection she had a cough and congestion during this time. The cough has passed but the diarrhea is still there\u2026 In fact she has started vomiting late last night and throughout today. Can I treat this with over the counter medication? Doctor: Hello, Ihave read your answer and understand your concerns.Mostly diarrhea is caused by viral pathogens. Involvement of respiratory system of your child also supports viral etiology. Diarrhea usually settles in 10-14 days. You should keep an eye on hydration status of your child. Give her usual foods she eats. But add extra fluids, ORS being the best and avoid colas and juices. For vomiting you can give an antiemetic, 30-45 minutes before meal. Vomiting will also settle with time. Once vomiting settles start zinc syrup. It helps bowel mucosa to repair itself. Do give it for 14 days.Anti diarrheals are not recommended in children.Hope I answered your concerns.Regards"
},
{
"id": 153223,
"tgt": "What is the treatment for metastatic prostate cancer?",
"src": "Patient: Hi, may I answer your health queries right now ? Please type your query here... My husband has metastatic prostate cancer to the bones, he is 88yrs old, and is on chemo medication and 3monthly injections. this was discovered after being treated for severe arthritic pain in the neck and shoulders.what is his prognosis. Thank you. June Doctor: Hi,Thanks for writing in.The treatment for metastatic prostate cancer is by chemotherapy and radiation therapy.Surgery is not possible in a person who has metastatic prostate cancer. This is an advanced disease and cannot be cured completely.For symptomatic relief and to slow down the disease process, radiation therapy might reduce the pain from the cancer and chemotherapy will kill and control the cancer cells. Pain is also important to control and he is also having arthritis. He might require pain shots in the neck and pain relief for cancer symptoms."
},
{
"id": 181750,
"tgt": "Suggest treatment for teeth pushed back by 3cms after wearing twin blocks",
"src": "Patient: Hi I got twin blocks at the start of October and I wore them none stop I went back to the clinic in dec they told me my teeth got pushed back 3cm and after that I haven't realy worn them much i don't know why I've put them back in today my next appointment is at the start of feb any ideas on what will happen Doctor: Thanks for your query, I have gone through your query.Considering your teeth movement, you need to discuss with your orthodontist and take an opinion immediately as early as possible. Because 3cm teeth movement is too much.I hope my answer will help you, take care."
},
{
"id": 199642,
"tgt": "Suggest treatment for erection problems",
"src": "Patient: at one point in my life I was using cocaine and during the middle of sleeping with a girl I felt a popping sensation in my lower abdomen to the right side of my bladder and I lost my erection immediately and it felt as if I had a burst blood vessel. My sex life has never been the same, I did not see a doctor, and now find it hard to get an erection at times. It the vessel a probable cause and what treatment options are out there to restore my sex life Doctor: Hello dear,Thank you for your contact to health care magic.I read and understand your concern. I am Dr Arun Tank answering your concern.If the vessels has been burst it will never get erected.So in my opinion the vessel is intact and you should not worry regarding the possible burst of vessels.But if you think that you have some problem in the penis. You can go for USG examination of the penis. It will tell you the exact problem for non erection.In some case such non erection problem can be solved with the sildenafile citrate tablet. This tablet can be help ful in erection. Please do not consume more than usual dose. Overdose can cause damage to the vascular tissues. Which can cause permanent damage to the tissue.Watching the porn movie before the sex can give good erection. Same way foreplay can be a important part in erection. It give us good joy of sex.I will be happy to answer your further concern on bit.ly/DrArun.Thank you,Dr Arun TankInfectious diseases specialist,HCM."
},
{
"id": 123579,
"tgt": "What causes radiating pain on the knees?",
"src": "Patient: Yesterday night I was travelling by bus for 6hrs. I had severe radiating pain in both both the knees and above. I tried all positions but did not improve. After I came home and rested it became alright. What could the reason be. It has never happened before Doctor: Hello, As the nature of the pain is radiating, it could be due to bad posture there will be a spasm of the muscles of the spine leading to nerve entrapment. As soon as you came home and rested the pain vanishes as the muscle became relaxed. This incident indicates there is a weakness of the muscles which stabilize the spinal column. Doing exercises to stabilize the spine will be helpful to avoid future symptoms. Hope I have answered your query. Let me know if I can assist you further. Take care Regards, Jay Indravadan Patel, Physical Therapist or Physiotherapist"
},
{
"id": 89601,
"tgt": "What causes indent & pain in lower abdomen?",
"src": "Patient: Hi, I gained over 30 pounds and have now lost about 15.. I am noticing a indent in my lower abdomen in the center that won t go away. I has been having abdomen pain and gotten many test but nothing came back and I didn t know if this could be the reason. I have never had children. Doctor: You have not given details of your test reports and age that will be valuable in accessing your conditions. Any if ultrasound has not revealed any thing you should go for ct abdomen. At the same time total harmonal study like thyroid and LH; Fsh; prolactin; Progesterones even testosterone.At same time Renal and gastrointestinal pathology should also be investigate d. Mean while start pantoprazol and domperidon empty stomach. Take bland and fiberous diet.mild laxatives if constipated. Get in touch with good medical institution. All the best."
},
{
"id": 69642,
"tgt": "What causes tender lump in right hand side armpit?",
"src": "Patient: I am an 18 year old male. 6ft tall and mixed race. I have recently, within the past week found a small but tender lump in my right hand side arm pit. I wondered if you could relate whether this was due to somekind of hormonal catalyst. Or something more concerning? Should I be contacting my GP? Doctor: Hi.Thanks for your query and an elucidate history.Of course , contact the GP. Simple test::: you can do yourself.IF the lump is moving freely along with the skin, this is boil or a small sebaceous cyst. IF it is deep seated and not attached to the skin- this is more likely to be a lymph node. Either of the lumps may need a treatment with antibiotics and anti-inflammatory medicines. So visit a GP."
},
{
"id": 174068,
"tgt": "Suggest treatment for severe stomach pain in a child",
"src": "Patient: Hi there, my daughter is aged 3. This morning she has been crying out in pain saying she has tummy ache. Yday she had a chesty cough and blocked nose. I have given her calpol and temp is 34.6 but is sweating a clammy sweat. She is also drifting in and out of sleep. Doctor: HiChildren do experience tummy pain during respiratory tract infection. If the pain is severe and associated with blood in stools then assessment by paediatrician is important to rule out possibility of Intussusception.Take Care"
},
{
"id": 125701,
"tgt": "What are the symptoms of psoriatic arthritis?",
"src": "Patient: what symptoms to look for psoriatic arthritis if you are under treatment for oesteoarthritis and have new skin breakouts that are painful similar to sunburn, blistering or burns on a large area of upper forehead on left side near and above eye that has been painful for 7-10 days but not a usual headache and no bumps or big blisters but it looks like a small rash to smooth area. Doctor: Hello, Psoriatic arthritis is similar to other forms of arthritis but typically involves small joints. Patients also have psoriasis somewhere on their body. Consult an orthopaedician and get evaluated. Hope I have answered your query. Let me know if I can assist you further. Regards, Dr. Shinas Hussain, General & Family Physician"
},
{
"id": 2708,
"tgt": "What could sudden lengthening of menstrual cycle after Duphaston intake indicate?",
"src": "Patient: Hi i am 37 i got marrid six month bck.bt still am nt concv. My mc is normal.last mnth i went to my dr she has given me clofert 100 (5) and hch 100 injection(1) nd duphaston tablet for 15 days then last month i hd my periods on 35 days i was depressed every month i got my mc on 23 , 24 days.i was expecting dat i hv concvd plz advice Doctor: Hi,It can happen sometimes that your periods get delayed after taking Duphaston. I think you should go for an IUI for next 2 to 3 cycles after getting a tubal patency test. Do a semen analysis for your husband. Hope I have answered your query. Let me know if I can assist you further. Regards,Dr. Khushboo Priya"
},
{
"id": 153572,
"tgt": "Suggest treatment for adenoid cystic carcinoma",
"src": "Patient: Hi, One of my friends have been diagnosed with Adenoid Cystic Carcinoma in the year of 2009 in his Nasal cavity and he went for a major surgery and 80% of his Nasal has been removed. No Radiation & No Chemo Given after surgery. Periodic checkup was done on every 6 months. After 3 years on November recurrence was found on his right eye canthus later after biopsy doc confirmed it Adenoid Cystic Carcinoma. We have consulted the major hospitals in India and he was suggested to go for a major surgery which is opening the skull and there is no guarantee for his eye . So we have been gone through most of the alternate treatments like ayurveda, siddha and chemo and nothing has worked for him. Can you please let me know if there is any possibility to cure his tumor without loosing his eye. Regards Paul YYYY@YYYY Doctor: Hi,Thanks for writing in.The cancer might have become locally aggressive and that is the reason the doctors have suggested major surgery. A surgery being possible is a curative approach. I am not sure of the position of his eyes as reports are not available but however if surgery is not done then risk to life persists.If there is risk of loss of vision in one eye then I think it is worth to take a chance with surgical treatment. This time it is suggested to have detailed discussion and also take complete treatment like chemotherapy or radiation therapy if required after the surgery. It is less likely for alternate treatment to work at this stage. I suggest that he does not waste more time in decision making and proceeds with a surgery at a good cancer hospital. Hopefully his eye will be safe. Please do not worry."
},
{
"id": 186269,
"tgt": "Can gum bleeding happen due to the new tooth brush and paste?",
"src": "Patient: hi, today i got a dizzy spell after brushing my teeth, i was using a new tooth brush and a new types of tooth paste if that helps in any way, i was wounding what might have casued it, i brushed my teeth as normal, there was a little bit of gum bleeding but nothing major, but once id finished i suddenly felt very faint as if i i was about to pass out Doctor: Thanks for using Health Care magic.Read your query.Using a new tooth brush and a tooth paste will not cause any bleeding.Bleeding while brushing will be because of gingivitis(gums inflammed) or u may have put more pressure on the gums so it caused a little bleeding.Get your teeth cleaned with the local dentist and use a mouth wash.As for the faint feeling that should not have any relation with the brushing and maybe because of some other issues.If worried or is persistent consult a doctor.Hope this was beneficial and u recover soon.Thanks and regards."
},
{
"id": 141513,
"tgt": "Suggest supplement to cure memory loss problem",
"src": "Patient: Is there a health supplement to improve or slow down short term loss. I am a male, 64 years old.I am presently taking a product from \"Puritans Pride\", which contains \" phosphollipid Complex from Soy Lecithin (standardized to contain 20% Phosphatidylserine.) The dosage is 100 mg, 3 times a day. Thanks Doctor: Hello and Welcome to \u2018Ask A Doctor\u2019 service. I have reviewed your query and here is my advice. There are many supplements in circulation but unfortunately I have to make you present there are few studies proving efficacy for any of them, including the one you mention. One other alternative I would consider would be a supplement containing gingko biloba which is a popular aid for memory problems.I hope to have been of help."
},
{
"id": 135390,
"tgt": "Suggest treatment for pain and pressure in the upper shoulder blade area",
"src": "Patient: My husband is having pain in his left upper shoulder blade area, where if he applies pressure on a certain muscle the pain seems to diminish some. It bothers him when he rotates his arm in a circular motion and many times it s painful to sleep on his left side. This has been going on for three months or better. He s taken Advil and nothing seems to help. Is this something he should see a doctor for and if so, what kind of specialists do you suggest. Doctor: Hi Dear,Welcome to HCM.Understanding your concern. As per your query you have pain and pressure in the upper shoulder blade area. Well there can be many reasons for symptoms you mention in query like poor posture , compressing or pinching a nerve , arthritis , gallbladder diseases or heart attacks . I would suggest you to consult orthopedic surgeon for proper examination . Doctor may order CT scan , MRI or physical examination . Doctor may prescribe muscle relaxant along with anti inflammatory and recommend physical therapy and stretching of trapezius muscle . Doctor may also refer you to gastroenterologist for gallstone or cardiologist for heart problem .For now apply warm compresses , do not sit or work in one pose for long time and take ibuprofen or acetaminophen for pain . Hope your concern has been resolved.Get Well Soon.Best Wishes,Dr. Harry Maheshwari"
},
{
"id": 107883,
"tgt": "Suggest treatment for severe lower back pain",
"src": "Patient: I have some low back pain causing severe pain with ambulation and standing. I have had several epidurals and feel it it is time to get something done however I m very reserved about having any spinal surgery as I have seen over the years more hard done than good. Unfortunately here I am needing something done I m only 55 and was looking at neurosurgeons Dr .Gregory Nazar and Steven Reiss being new to Louisville I have no idea who has a successful surgery rate so I do have many questions. YYYY@YYYY Doctor: Still I would suggest that surgery on spine should be the last resort.Have you exhausted every other alternative . Have you exhausted Ayurveda's 'Agnikarma' treatment without invasion. Have you tried to manage by herbals or its simply giving up?Pl. do not give up. Hope is still there. Pl weighh the pros and cons of spinal surgery.I would not allow my patients for this till other available channels are also exhausted.Hope it guides you even a bit !"
},
{
"id": 120478,
"tgt": "What causes numbness around the tailbone area?",
"src": "Patient: I have numbness around the base of my back (tailbone) area. If I am sitting, and squeeze my muscles around this area, it is very numb and sore. Also when I try to stand from sitting, I get very sore/numb pains in my tailbone area. I don t know if it s linked but have also got plantar fasciitis and very limited mobility in my ankles/feet/toes. Doctor: Hello,I read carefully your query and understand your concern. Numbness\u00a0in the buttocks may be\u00a0caused\u00a0by a medical condition, such as piriformis syndrome or a pinched nerve.\u00a0I suggest using anti inflammatory medications such as Ibuprofen to relieve the symptoms.I also suggest to avoid movements that can trigger the symptoms. If the symptoms continue, you should see a neurologist for further evaluation. Hope my answer was helpful.If you have further queries feel free to contact me again.Kind regards! Dr.Dorina Gurabardhi General &Family Physician"
},
{
"id": 49761,
"tgt": "Kidney infection, taken antibiotic, had cervical cancer, had radical hysterectomy. Suggestion?",
"src": "Patient: Hi wonder if you can help, I have very recently been treated with antibiotics for a kidney infection, which still has not cleared. I have had blood tests. However it has now begun to hurt when I have to go to the bathroom to poo. Two years ago I had cervical cancer , and had a radical hysterectomy . I am a female age 33. Thanks Doctor: HelloThanks for your query.Based on the facts that you have posted it appears that you have undergone radical hysterectomy for cancer of the cervix and now has UTI .Please get your routine urine test and urine culture done to find out the organisms causing this infection and antibiotics to which they are sensitive to.Please take broad spectrum antibiotics along with urinary antiseptics like Nitrofurantoin twice daily and later on switch on to appropriate antibiotics as per culture report. .Get the prescription of medicines from your family Physician.Ensure to drink more water.To keep your urine dilute This will help to control dysuria.Dr.Patil."
},
{
"id": 195509,
"tgt": "What causes spasms in the bladder post masturbation?",
"src": "Patient: Hi Good day doctor How are you? I masturbate mostly once a week but after that i feel a kind of bladder spasm and fill uncomfortable for some days but it disappears itself I am 36 years old and just want to ask some questions? 1 is masturbation really bad or good or nothing? 2 does it make prostate problem ? 3 can it be related to testicle shrinkage? 4 is it related to prostate cancer or it helps not to get it? Thanks Waiting for your answers..... Doctor: Hello and Welcome to \u2018Ask A Doctor\u2019 service. I have reviewed your query and here is my advice. 1- masturbation has both positives and negative effects positive effect is that it will remove the sperms from the body and make room for newer sperms as the process of sperm formation is ever going in men, it is bad in a sense that excessive masturbation does leads to loss of libido and lesser stimulation by natural intercourse 2-in routine masturbation does not causes prostate problem but keep in mind that during ejaculation the prostate also squeezes to release its fluid which contributes in semen so transient urinary obstruction due to prostatic contraction or pain in elvis may occur 3-no masturbation does not cause shrinkage of testes 4-yes there are studies that proves the relation of excessive masturbation with prostatic cancer as excessive masturbation will cause more testosterone to release and this will have effect on prostate which could cause prostatic cancer the proof of this is that the castrated males who were castrated in there early teens never develop prostatic cancer. Hope I have answered your query. Let me know if I can assist you further."
},
{
"id": 133387,
"tgt": "Suggest remedy for lumps & swelling in the ankle",
"src": "Patient: I was hit on the shin about 4 weeks ago mid-way between my ankle and my knee. The spot immediately swelled like an egg under my skin within seconds. I still have a hard painful lump there. Is it scar tissues? I ve started massaging it hoping my body will reabsorb any clot or scar tissue. Doctor: Hello sir/madam.According to your complain it looks like blood accumulation in subcutaneous tissue. It wont get reabsorb with massage.You need xray evaluation for any bony injury,kindly consult an orthopaedician for the same he might also ask for sonography of that swelling and will direct you for the treatment.You might require surgery for that swelling. And things will be fine.Thank you."
},
{
"id": 38085,
"tgt": "What does sweating and lightheadedness after DPT shot with BP - 104/70 indicate?",
"src": "Patient: I am 82 yrs. old. My primary doctor gave me a DTP shot on 6/12/14. On Sat., 6/14/14 I got up and within a few minutes I felt faint. I was sweaty and lightheaded. I took my blood pressure and it was 104 over 70. Is this a reaction from the DPT shot? I felt poorly for 2 days then started to feel more normal. Today is Tues. 6/17/14. My arm at injection point is still bruised and my upper arm is red and itches. Is there anything I should do or should I just give it time to wear off? Doctor: Hello, Thank you for your contact to health care magic. I understand your concern. If I am your doctor I suggest you that either you had allergy or reaction to DPT vaccine. You have to be careful when you go for second dose. Right now you can take a ORS and chlorpheniramine maleate as antihistaminic to relieve from such s light headedness also think for low dose prednisolone for further improvement.I will be happy to answer your further concernYou can contact me. Dr Arun Tank. Infectious disease specialist. Thank you."
},
{
"id": 115168,
"tgt": "Are there any alternative treatments for myelofibrosis?",
"src": "Patient: Hi. Are there any alternative or complementary treatments for myelofibrosis. I am enquiring on behalf of my mother who is currently taking hydroxyurea. Presently her white blood cell count is low and I wanted to find out if it is worthwhile her taking something to boost her immune system as she will be travelling. I was thinking of the herb astragalus and some vitamin C. Can you advise? Thanks Doctor: Hi, dear I have gone through your question. I can understand your concern. Myelofibrosis is terminal illness. Your bone marrow has fibrosis and unable to produce enough cells. Chemotherapy is the treatment. Hydroxyurea is drig of choice. Otjer treatment option is bone marrow transplantation. Consult your doctor and take treatment accordingly. Hope I have answered your question, if you have doubt then I will be happy to answer. Thanks for using health care magic. Wish you a very good health."
},
{
"id": 196916,
"tgt": "What are the side effects of masturbation?",
"src": "Patient: I ,m Saleem. My age is 46 years. In my boyhood i used to hand practice. Due to this my penis shrinked and now the size is onlly 3.5 to 4 inches during erection. And i also feel weakness and cannot do intercourse twice at the same time. Timing is very very short. Please help me can i be normal and recover in sex. Doctor: Hi welcome to the health care magic With advancing age erection capacity decreases.. You are having over masturbation history as per your given details.. It can lead weakness, groin discomfort, social anxiety like problemSo avoid over masturbation Daily yoga and healthy exercise done along with healthy diet If you are having erectile dysfunction and if your physical condition suits than you might prescribed sildenafil like drug for it that is taken 30 minute before sex Take care Consult physician or psychiatrist for examination"
},
{
"id": 124823,
"tgt": "Suggest medications for pain & swelling in neck",
"src": "Patient: I have had continous pain and swelling in my neck for well over 2 years. Nothing seems to help and it is getting worse. my pinky fingers go numb as well as my pinky toes. I did therapy and had to stop due to increase in migranes. I have had injections in my neck and they quit working after awhile. I have had an xray and it swoed a small case of scoliosis about 4 inches under where my problem is. Muscle relaxers do not help. Pain meds help get through the day but is severely worse when meds not taken.What could it be? Doctor: Hello, It could be due to conditions like cervical spondylitis. As a first line management, you can take analgesics like paracetamol or aceclofenac for pain relief. If symptoms persist better to consult a neurologist and plan for an MRI scan. Hope I have answered your query. Let me know if I can assist you further. Regards, Dr. Shinas Hussain, General & Family Physician"
},
{
"id": 168886,
"tgt": "What causes red bump on inner cheek of buttocks?",
"src": "Patient: My 18 month old has a big, hardened, red bump on the inner cheek of her buttocks. At first I thought it was a pimple, but it grew to be a size beyond typical sized pimples and it has clear as well as a dull yellowish puss coming out of it upon a soft touch. Some soft blood comes out as well and it is painful to my baby. Please help m understand what this is? I m so scared! Doctor: Hi,From history it seems that there might be having developing abscess over that site.There might be having diaper rash brought soreness and developed bacterial infection.consult your doctor and get examined.With removal of pus and proper dressing by the doctor is advised for proper and quick healing.Appropriate antibiotic medicine should be given.Ok and take care."
},
{
"id": 56007,
"tgt": "What does these SGPT and SGOT values of liver test indicate?",
"src": "Patient: hi myself gurmeet singh i have problem in liver. my liver is midly large in size 15.6cm in ultrasound scan & my sgot is 76 & sgpt is 124 & i m feeling pain in my chest & right side pain. please do me favour how these all control please suggest me. i m waiting for your reply..... Doctor: Wekcome at HCM i have gone through your query and being your doctor I completely understand your health concerns.. Raised sgpt and sgot indicate liver pathology.. Do u drink alcohol?? Have you done your viral screening like HCV or HBV?? You better consult hepatologist so that exact cause can he ruled out. Fatty liver disease is one differential but other causes first should be excluded... Stay safe Regards Dr saad sultan"
},
{
"id": 152084,
"tgt": "problem with left side of my head",
"src": "Patient: i am 40 years male. i have pulsating left temple and a feeling as something is crawling underskin in left side of the head. got my blood pressure it is 100/65 which is low than it normally is. i cant concentrate with my eyes and feel uneasy and dizzy in my head. sometimes feel numb also in the left side of the head. rest there is no problem with sleeping or eating. i am presently on assignment in congo and suffered from malaria. i took coartem for malaria. however this feeling continued even after the malaria medication. Doctor: Thanks for the query Your bp is on the lower and u r having symptoms suggestive of hypo perfusion. I suggest u withhold your hypertensives after meeting your doctor if u r taking any. If it doesn't improve then meet your ophthalmologist Have a healthy living"
},
{
"id": 215091,
"tgt": "18 years old suffering from constipation and gas problem",
"src": "Patient: hello lam 18 years old and i have problem of constipation and because of constipation i am also suffering with gas problem and this is not my age to be suffer like these problem so i wanna be free from constipation so is there any permanent solution Doctor: Take plenty of fibers in diet Take plenty of fluids Avoid spicy and d fermented food stuffs Eat Fruits like Oranges/papaya and apple Take Isaphgula Preparation like Fabolite 2tsf in 1/2 glass of water at night"
},
{
"id": 58194,
"tgt": "Have Stage IV Cirrhosis, blood work positive. How can it be treated?",
"src": "Patient: I am in Stage IV of Cirrhosis of the Liver/Autoimmune Hepatitis. My blood work came back positive a couple of weeks ago. My Dr is in Raleigh, NC and I just moved to Atlanta, GA a couple of weeks ago. My Dr in Raleigh said it was imperative that I seek a Dr here in Atlanta or neaby. Can you recommend a Dr to me. I am also on Medicaid. Thank you. Doctor: Hi and thanks for the query,It is important to know what came back positive in the blood work up. Was it an infection or was it the presence of absence of viral ;particles? It is important to know what was positive exactly in order to institute an appropriate treatment. Antibiotics are needed in case of infection and antiviral agents, and at times liver transplants in some cases could be offered. More information is required and seek the opinion of your gastroenterologist. Kind regards"
},
{
"id": 102658,
"tgt": "Why have steroid and albuterol inhalers not helped in curing severe breathlessness and unproductive cough?",
"src": "Patient: having trouble breathing when i lay down, feel like i'm drowning, my breathing is ok going in but exhaling has loud crackling bubbling sound that gets worse as i lay there for longer than 5 mins. i have an unproductive cough and when something does come up its almost textured like solid chicken fat and white. I have been using a steroid and arbuterol inhalers, but they are not helping and its getting worse. What can I do to get good air in me? :(sorry unable to pay on disablitiy. thank you for your time anyway Doctor: Hi. When the steroid and albuterol inhalators are not working, there is something else to be looked after. Your inhalation is OK- but exhaling has a sound - indicates there mat be a mechanical factor causing this problem. As is worsening on lying down indicates the same. I am afraid to say, you need to consult a ENT specialist to find out the reason to get it treated ASAP."
},
{
"id": 167056,
"tgt": "What to do for abscesses in child's mouth?",
"src": "Patient: MY 4 YEAR LD SON CONTINUES TO GET ABCESSES IN HIS MOUTH HE ALREADY HAD A SURGERY RESTORATION OF TEETH BECAUSE HE HAD HIS TEETH ROOTING AWAY BUT HE IS CONSTANTLY SPITTING UP WHAT CAN THAT BE? HE ALWAYS RUNNING TO BATHROOM TO SPIT UP AND SAYS HE HAS TO MANY BUBBLES IN HIS MOUTH? Doctor: Hi..Welcome to HCM..As per your complain it seems that your son has a dental abscess due to a deeply decayed tooth and as it is appearing that it has started draining and releasing pus inside the mouth that you son keeps spitting..Due to rotting or decaying of teeth, when the decay progresses below the root tips of the teeth there is pus formation and at times the pus perforates the bone and accumulates below the gums and at times there is an opening formed on the surface leading to pus discharge..I would suggest you to consult an Oral Physician or a Paediatric Dentist and get evaluated and a thorough clinical evaluation and x ray can help in diagnosis..In case if there is a deeply decayed tooth then Root Canal Treatment or Pulpectomy can be done followed by a course of antibiotics and painkillers.At times Incision and Drainage can be advised.Hope this helps..Regards.."
},
{
"id": 134822,
"tgt": "Suggest remedy for persistent cough and joint pain",
"src": "Patient: I am suffering from two month cold cough and joint pain.I have taken som medicine for cough but not yet cure. Some nodal problem is also facing.ocasionaly I feel some chest problem but I have don chest xray.doctor says allergic problem.iam feeling some sound changing also Doctor: Hi,thanks for writing to healthcaremagic.you seem to have allergic coughyou should do the following. 1. Steam inhalation 3-4 times a day 2. warm saline( water+ pinch of salt) gargles 3 times day,especially before going to bed 3.Drink only lukewarm water 4. Take mild analgesic like ibuprofen or acetaaminofen for joint pain 5. start antihistaminics(anti allergic tablets) like levocetrizine. most important of them all is STEAM INHALATION and WARM SALINE GARGLEShope i have answered your query. if any doubts you can get back anytime you want, otherwise please close the discussion and rate the answerThank you"
},
{
"id": 57551,
"tgt": "How can I self help for high BP,shortness of breath & shoulder pain post gallbladder followed by collapsed lung with blood clots?",
"src": "Patient: I had gallbladder surgery Monday and ended up with a collapsed lung and blood clots to the lungs. I am on xaeltto and have been to er today and had another cat scan as I have had bad shoulder pain since last night and also had elevated bp. It is better now but I am a little short on breath and still have pain in my shoulder. My heart rate is high 104. I am not getting much help from er so what can I do for myself? Doctor: hi, Following GB surgery, some complications like collapsed basal segments of right lung and shoulder pain may rarely occur. Pain may increase your blood pressure and heart rate and collapse may be causing breathlessness. I suggest you to start beep breathing exercises which will help not only your breathlessness but also your blood pressure. cut your salt to less than 6 gms per day. Take an anxiolytic like alprazolam and pain drug like brufen for best response. all the best."
},
{
"id": 116328,
"tgt": "Suggest remedy for blood transfusion",
"src": "Patient: My problem is that my blood system is not working right, at the moment, some months in fact causing me to have blood transfusion about every three weeks, iron I now ok but still not working, blood count ranges from 73,83, even after transfusion, so now trying Prednisolone 25mg if this might give the system a kick start. I am 87 years old and have never taken more than a light wine, this has just started this last year after a bowel operation for a cancerous node and a heart pacemaker this has occurred in the last year, prior to that I had een in good health Doctor: HIWell come to HCMYou have not specifically mentioned the clinical diagnosis for which you are in need of blood transfusion neither the value of hemoglobin is given here, there are certain medical conditions in which patients need blood transfusion, if the volume of blood decreases, if certain component of blood deficit, some pathological leaking or oozing of blood, in such condition underlying cause need to be treated, hope this information helps, take care."
},
{
"id": 136180,
"tgt": "What causes severe pain in the hip and buttocks?",
"src": "Patient: last year the doctor found 8 kidney stones and sent me hope., i also have severe hip buttocks pain., i have been dealing with discomfort since then..but the last few days it feels like i have a knife stabbing me in my left side and it s becoming worse and worse Doctor: Hello, I have studied your case,Due to compression of nerve root there can be sensation in hip and butt painI will advise you to do MRI spineBlood work up and EMG-NCV [nerve conduction] study will help. For these symptoms methylcobalamine and neurotropic medication like amitryptiline can be started consulting your doctor.I will advise to check your vit B12 and vit D3 level.Epidural steroid shot can help if no relief with medication.Hope this answers your query. If you have additional questions or follow up queries then please do not hesitate in writing to us. I will be happy to answer your queries. Wishing you good health.Take care."
},
{
"id": 47425,
"tgt": "Is Norflox recommended for kidney stones?",
"src": "Patient: Hey i have kidney stone since last 4weeks and taking the medicine for the same. I tried masterbating in between this period but no ejaculation. Is this because of kidney stone? My list of medicines are: Norflox 400mg Hyocimax tab Urimax 0.4 cap Spiromide tab Doctor: HelloNorflox contains norfloxacin and it is an antibiotic.It is prescribed in many types of infections.Norflox is not recommended for kidney stones.Treatment of renal calculus depend upon many things like composition,position,size of calculus etc.Up to 5.0 mm calculus may pass spontaneously.Drink lot of water and initially you may need syrup like sodium and potassium citrate.Urimax is generally prescribed in enlarged prostate and Hyocimax tab is an antispasmodic prescribed in pain.Get well soon.Take CareDr.Indu Bhushan"
},
{
"id": 199703,
"tgt": "What causes penile pain during an intercourse?",
"src": "Patient: SIR,this is to say, that while doing intercourse with my wife, my foreskin of ,my penis got down and their was lot of pain on my penis due to this.i am not able to do sex with my wife .how will my skin of my penis will get normal.how should i take care of this problem Doctor: HelloYour findings may suggest phimosis.It is a surgical condition in which fore skin can't be retracted over glans penis.I suggest to go for circumcision.It is a minor operation in which foreskin is cut out and skin is tied over coronal sulcus.It is better to get rid of this condition earliest because phimosis may cause surgical emergency if it is stuck over glans penis.It may compromise distal blood supply.So,you should consult surgeon for proper management.Get well soon.Take CareDr.Indu Bhushan"
},
{
"id": 76603,
"tgt": "What causes tightness in chest and throat with difficulty in breathing?",
"src": "Patient: For the past few days I have been having a tightness in my chest and throat, and have trouble breathing. I often have to yawn to be able to take a deep breath. I get dizzy sometimes and feel like passing out. Its getting to the point where I can't sleep, I can't yawn to take a deep breath, and my chest is begining to get sore. I do not have panic attacks. I do not have asthma. My allergies hardly affect me and are very mild. I do not have any other symptoms. Iwas given an inhaler but it does not work and I do not smoke. Doctor: Thanks for your question on Healthcare Magic. I can understand your concern. By your history and description, possibility of stress and anxiety related symptoms is more. Stress and anxiety can cause feeling of chest tightness, chest pressure, inability to take deep breath etc. You are not having allergies and not having smoking habit, so lung related diseases are less likely. So better to consult psychiatrist and get done counselling sessions. Try to identify stressor in your life and start working on it's solution. You may need anxiolytic drugs too. Don't worry, you will be alright. Avoid stress and tension, be relax and calm. Hope I have solved your query. I will be happy to help you further. Wish you good health. Thanks."
},
{
"id": 179189,
"tgt": "What causes hard stools after taking Nan formula milk?",
"src": "Patient: hello my baby is 6 nd half month old.i m from saudi arabia.few days before i have came to saudi arabia from pakistan....my baby was drinking lactogen milk in pakistan but now insaudi arabia lactogen is not available.....Now im giving my baby nan milk.....bt now problem is her stool is very hard and she cries so mch...what should me do now? Doctor: It is a common problem and you dont need to worry. Formula fed children often get constipated. The things you should check is proper preparation of the formula. I scoop of nan should be added to 30 ml of water. If this concentration is not mentioned then there is increased chance of constipation. Also as he has crossed 6 months start to give him semisolid foods like cerelac and home made smashed food. For temporary relief give glycerine suppository and bowel movements will normalise."
},
{
"id": 157919,
"tgt": "Advised to undergo chemotherapy before surgical operation for endometrial cancer. Is it the right process?",
"src": "Patient: hi im cheerymeel from philippines, i just want to ask questions regarding with the condition of my mother who is diagnosed with endometrial cancer probably stage 2, our doctor told us to undergo my mom with chemotherapy before she proceed a surgical operation. is it right to undergo chemotherapy before she proceed with thw surgical operations? Doctor: i am sorry to say that this is not the right treatment protocol for endometrial cancer. Normally, the first step in the treatment of endometrial cancer is surgery. I this we remove the uterus, both tubes, both ovaries, bilateral pelvic nodes and take a number of samples from various parts of the abdomen. We also take washings from the abdomen surface and send it for cytology testing. This is called a staging laparotomy as this not only is part of the treatment, it also gives the exact stage of the cancer. Depending on the final stage that comes after surgery, she may require postoperative radiotherapy. There is nor role of chemotherapy in endometrial cancer except in the last stage for palliation. You can visit nccn.org and see for yourself. I suggest you change your oncologist."
},
{
"id": 144681,
"tgt": "How safe is going for a brain fluid test after a stroke?",
"src": "Patient: 77 yr had stroke in nov2013, left side paralyzed. 2 was ago went to hospital and diagnosed with UTI, pneumonia, MRSA in nose, and possibly had mild stroke in 1 -2. Wk ago which was unnoticible . She does not speak, respond, and closes her eyes most of the time. Doc want to do brain fluid test. What are the pros and cons for brain fluid test, what s the risk? Doctor: Might be your doctor is suspecting infection spreading to brain (meningitis/ encephalitis), for which he planned brain fluid test (CSF examination). CSF examination is usually done by lumbar puncture (putting a long needle in between vertebral bones in lower back).It seems diagnotic test in your patient, to check for infection in brainThere is a small risk of complications of lumbar puncture- infection at local site, spread of infection from puncture site , local hemorrhage, post lumbar puncture headache. But these complications are very uncommon if proper aseptic conditions are maintained and performed by thin needle (21/22 gauze)."
},
{
"id": 93880,
"tgt": "Stomach ache. Had unprotected sex. Taken postinor. Pregnant?",
"src": "Patient: I am still breastfeeding my 1 year old Daughter. I had unprotected sex with my boyfriend on the night of 12th April 2013 and Morning of 13th April 2013, i took Postinor 2 emergency pill both tablets at the same time on the night of 13th April 2013. I am feeling stomach ache and ache in the lower back side. Could i be pregnant or are these just symptoms of the Postinor 2? Doctor: Hi! Possibly you are getting the pain due to smooth muscle contractions initiated by the postinor 2. However, you should also go for a urine prenancy test which may be co existing even before having sex with your boyfriend. In both the cases you have nothing to worry about. In the first case you can to for some anti spasmodics/ anti inflammatory. In the second case, the management depends on your choice. With you an early recovery."
},
{
"id": 44195,
"tgt": "Can you please advise me if my semen analysis is normal?",
"src": "Patient: can you please advise me if my Semen analysis is normal or not? and can i get a baby? volume :4.5 ml coulour :Gray PH :8 liquefaction:15 viscosity:slightly viscous sperm count: 27.2 mill/ml total sperm count: 122.4 mill WBC :0.1 mill/ml agglutination:not seen cellular debris :present Motile:35% sluggish:15 % immotile: 50 % normal shape:79 % thanks, Doctor: Dear customer, As per the reference normal range of semen analysis you fall within the normal sperm count and morphology so you should NOT worry and can bear as a father. thanks"
},
{
"id": 67852,
"tgt": "How can a painful cystic lump in the pubic area be treated?",
"src": "Patient: I have a cystic lump quarter size in my pubic area. I ve had them before but not this bad. I have iced it I ve put heat on it and it still won t drain. Should I go to the er my provider won t be able to see me for a week and I m in pain? I m unable to wear under garment or jeans cause of the pain. Doctor: Hi! Good evening. I am Dr Shareef answering your query.From the history, it seems that you have developed a cellulitis (inflammation of the subcutaneous tissue) around the cystic lump which possibly was a boil.If I were your doctor, apart from the local treatment like cold or hot compress, would also advise you for a broad spectrum antibiotic and an anti inflammatory along with a proton pump inhibitor. I would refer you to a surgical specialist or to the ER only when these measures did not work.I hope this information would help you in discussing with your family physician/treating doctor in further management of your problem. Please do not hesitate to ask in case of any further doubts.Thanks for choosing health care magic to clear doubts on your health problems. I wish you an early recovery. Dr Shareef."
},
{
"id": 146523,
"tgt": "What causes unconsciousness after skull bone surgery?",
"src": "Patient: my wife had a stroke on5th may2010.was operated and skull bone of the right hemisphere removed.she has continued to be unconscious since then.how long will she continue as such? Doctor: I am afraid your wife has suffered a very big stroke, because it's only then that there is the need to remove the bone in order to lower the pressure caused by the swelling. It's the stroke which causes her condition not the surgery itself, without it she probably wouldn't have survived. Unfortunately it seems that 4 years have passed and if your wife is still unresponsive I am afraid that there is little chance of her neurological condition improving. The damage has been very extensive and whatever compensation from the unaffected areas there could have been hoped for at first, it would have taken place during the first year."
},
{
"id": 27812,
"tgt": "Need treatment for chest pain",
"src": "Patient: okay so heres my story. Im 24years of age, weigh 150lbs, workout reguraly, and eat healthy. I also quit smoking 2 years ago due to chest pain, and headaches while i would smoke. I do not do any drugs or drink, because i get very sick. I also have a five year old son. I remember the day i first experienced chest pain. It was about 4years ago, and i was pumping gas, when all of a sudden a sharp stabbing pain came threw my chest, by my heart area. At first I wanted to call an ambulance, but decided to drive my self to ER. Chest xrays, EKG, and blood came back normal, except my potassium level was extremely low. SO they gave me this big horse, see threw pill, and sent me on my way. Meanwhile, something else seemed to be happening in my body. I was having excuriating headaches, that would come on very dull, and then explode like a firecracker near my temples. They sometimes would affect my vision. MY wieght was going from up to down, and it just seemed like i was just a wreck. Finally the one thing doctors could rule out, is that i had a very ugly diseased gall bladder that needed removal. During the examination before surgury, I explained to my doctor, of headaches, chest pain, dizziness, extreme fatigue, nausea and upset stomach and weight gain where among my symptoms. She ordered up a few tests, inlcuding an ECHO, in which it was discovered that I did have a heart murmur with LOW regurgitation(but at that time still my EKGS where normal), and a cyst on my kidney. In all I was able to get my gall bladder removed, and was told to have heart and kidney followups. Well that was 3 years ago. In the between time I was suffering, feeling like im dieing, my EKGS went from normal to Abnormal, and was told only because of age. It was ruled out aniexty was the diagnose. Now, I have in total went to the ER on 7 different occasions, where one time my EKG was showing a slight heart attack, so the doctor says, but blood work was fine. On all occasions my symptoms where shortness of breath, chest pain, and headaches. my blood pressure on this one occasion was very high, in which they took me back right away, and put me on a heart monitor, and all was fine. I actually have walked out the ER 3x times before speaking to the doctor, because i felt like they didnt care, or wasnt listening to my symptoms because of my AGE. Today has been the worst of my pains ever. I was driving then all of a sudden i couldnt breathe, and i got so weak i felt like i couldnt drive. I literally feel like my life is slipping out of me, and im dieing from heart failure. My primary doctor knows my history. but hasnt sent me to see a cardo doctor yet, only a nero doctor. He also prescribed me migrane pills, and zololoft(which he promised that it would take away my chest pain 100%) I have stopped taking all his medicine because it doesnt work. Im utterly confused, and lost at the point. My mother died at 44 of a heart aneurysm, and my grandma(from dads side) is suffering from heart failure right now. The last 3x times i came down with these crazy espisodes(atleast thats what i call them), i felt like i was going to die, but i didnt even bother to go to ER, because how can you diagnose a condition if there not even ordering tests to determine the underlying cause? If anybody could help me understand, please reply to this message Doctor: Hello,I have gone through your history.first of all let me reassure you that since all your ECGs and blood reports were normal ,it is unlikely that you have serious heart trouble.However I need to your detailed echo report .secondly your headache is suggestive of migraine and you must continue its prophylaxis All your episodes of ER visits are likely to be panic attacks .Now onwards if you have such spell please take mouth dissolving 0.5 mg clonazepam.It will make you better quickly.I feel you may be under stress ,you should try to sort out any such social or other issues(if it is) Meditation and yoga will help you.My best wishesDr.Rajesh Teli,MD."
},
{
"id": 26558,
"tgt": "What causes dizziness, nausea and tightness in chest?",
"src": "Patient: My daughter is out of the country. She is 21. She broke her foot. Two days later, Drs. diagnosed (English not first language so possible vague areas) as a side-effect of cipro (she was taking for an unrelated infection), she had heat at the back of her neck, nausea, and what she said felt like \"tighness in her veins\". Yesterday, a couple weeks after this incident, she was watching a film in a movie theatre - heart began to race, dizziness, same tightness. She went to the ER in an ambulance. The EKG showed no abnormality. It was called an arrythmia -- but I had the impression the Dr thought it was \"anxiety\". She was not anxious at the time of the incident. This is of great concern to me and I am 7000 miles away. Any insight? I'd be grateful. Doctor: Hello!Welcome and thank you for asking on HCM!Regarding your concern, I would explain that a comprehensive differential diagnosis between anxiety and cardiac arrhythmia should be done. You should know that even the trauma she has gone through (foot breaking) could be related to anxiety and heart palpitations. I would recommend consulting with the GP for a careful physical examination , a resting ECG and a chest x ray. Some blood lab tests (complete blood count, PCR, sedimentation rate, thyroid hormone levels, cortisol plasma levels, fasting glucose and blood electrolytes) are needed to exclude other possible metabolic causes like chronic anemia, thyroid or adrenal gland dysfunction. An ambulatory 24-48 hours monitoring and a cardiac ultrasound may be necessary if suspicions of a possible cardiac arrhythmia are raised. If all the above tests result normal, the most probable cause of her symptomatology would be just pure anxiety. Hope to have been helpful!Feel free to ask any other questions, whenever you need!Greetings!Dr. Iliri"
},
{
"id": 20763,
"tgt": "What cause swelling in the body with blockage in the arteries?",
"src": "Patient: hi, my name is khalil from tunis and i have Disease name s volkmann It is the result of swelling of the body fat and fat, which cause such materials to the blockage of the arteries carrying of albumin is located swelling of the artery, which leads to the loss of the amount of albumin and Fattaminac Doctor: HelloThanks for posting at HCM. The symptoms you mentioned are hard to understand because of language issues. But I can get a raw idea that you are suffering from blockages in the arteries and swelling of the body. Blockages in arteries causes decreased blood supply to the organs causing disturbances in the normal functioning of these organs. Such blockages when they affect the heart and kidney can lead to fluid overload and accumulation of fluid in legs and face causing swelling. You also mentioned about albumin being low. Albumin is a protein and deficiney of albumin can also lead to swelling all over the body. The treatment for the condition includes diuretics to remove excess water from body and protein supplements with high protein diet. Also finding out the blockages by angiography and if significant blockages are there, treating them by either stenting or bypass can help get rid of the cause of the swelling. Wishing you good health. Regards"
},
{
"id": 179278,
"tgt": "Suggest treatment for cough with mucus in kid",
"src": "Patient: my daughter is 4 years old she is suffering cough with mucus. she is suffering with caugh from last 3 weeks. doctors suggest medicines like kep-x, tusq-x+ but no use so what is the solution for it. in blood test it showing little infection how can she cure well she cannot sleep in the nights until now she is suffering with caugh. Doctor: Hello. I just read through your question. Cough with mucous that has lasted this long may be due to a bacterial infection. In this case, antibiotic is required to resolve the infection. I recommend consulting with your doctor about this possibility."
},
{
"id": 32124,
"tgt": "Why does the hands smell like sulfur on them?",
"src": "Patient: My boyfriend just realized today that his hands have a slight smell of sulfur. He doesn't smell it anywhere else and he has washed his hands repeatedly. I also smell it on his hands. He has not handled anything that should cause this. Is this something we should be concerned about? Doctor: Hi Dear,Welcome to HCM.Understanding your concern. As per your query sulfur like smelling from hands of your boy friend is mainly due to increased sweating which is leading to hyper hydrosis and it will mainly occur when there is more of humidity in climate. It could be due to delusional disorder which you should ignore completely, otherwise you have to visit psychiatrist for the same. Need not to worry. I would suggest him to take bath 2 times a day. He should use antiperspirants and deodorants as well. Concentrate on work and try to ignore it little bit. Hope your concern has been resolved.Get Well Soon.Best Wishes,Dr. Harry Maheshwari"
},
{
"id": 126133,
"tgt": "What can cause pain in the leg upon lying down?",
"src": "Patient: I have what feels like growing pains in my left leg from time to time and it is only in that leg and seems to hurt more when I lay down. Also I don\u2019t think it\u2019s growing pains because I haven\u2019t grown for awhile and it\u2019s only in that leg. What could this be and what can I do about it? Doctor: Hi, Consult a neurologist and get evaluated. A detailed evaluation is required. Hope I have answered your query. Let me know if I can assist you further. Regards, Dr. Shinas Hussain, General & Family Physician"
},
{
"id": 80274,
"tgt": "How to treat breath shortness,chest pressure and back pain?",
"src": "Patient: I am down with shortness of breath, pressure on the chest and back pain. Doctor advised to take Deriphlyn 150 mg twice a day for 15 days and advised me to use Inhaler. Ater one week, I could not find any signs of recovery. could you please help me to know the reason and advise me what should I do Doctor: Hello I share your concernChest pain and shortness breathe at your age might be related to heart or lung disease like vasospatic angina or acute exacerbation of bronchial asthmaso please ask your doctor for EKG and blood investigation for serum cardiac markers , CBC , AEC and allergy testAnother possibility is stress and anxiety disordersAvoid fatty fried spicy and junk foodsay no to smoking and alcoholHave fresh fruits and steamed vegetablesHave plenty of dietary fibersaerobic exercises walking and swimming is recommended for youHope this answers your questionBest wishes"
},
{
"id": 20244,
"tgt": "What precautions should be followed for provocable myocardial ischemia?",
"src": "Patient: recently i have undergone trademill stress test at a hospital. my qge is 49 years. my height is 5 feet 10 inches and weight is 87 kgs. bp is 140/85 at rest . theb abstract of strss test states \"Mildly positive for provocable myocardial ischemia\" what is the significance / what precautions i have to take/? what medicines/ Doctor: I'm not quite sure what to make of that report. However, a stress test is compelling to do something further only if high risk criteria are met. They were not. It is of no value as reassurance that coronary artery disease is not present. You may want to ask your doctor about a heart scan. A low score is reassuring. A high score is an indication that you need to be strict about diet, lifestyle and your future risk of heart attack."
},
{
"id": 127070,
"tgt": "How can pain in the little toe be treated?",
"src": "Patient: I have pain in little toe, if walked on much it turns black, and if I soak it there is a hole inbetween it and the next toe. I was treated by antibotic for 20 days when I went to MD with pain in toe. Still have problem only now it has began to hurt and turn black. What do I do? Doctor: Hi, Possibly you have chronic infection in the toe and on walking there is increased circulation causing discoloration. I suggest you get X-rays, complete blood counts, and ESR to assess the problem. If you can better to consult orthopedic surgeon. Hope I have answered your query. Let me know if I can assist you further. Regards, Dr Gopal Goel"
},
{
"id": 143804,
"tgt": "What to do for the bruising along my back pelvis and spine area?",
"src": "Patient: I was in an accident on March 5th, a semi rear ended me doing about 50 mph. The drs originally only paid attention to my neck and shoulder area, i have tried to explain that my major issue is in my lower back but i feel like I m getting ignored because i don t have insurance. I have bruising as along my back pelvis and spine area, and shooting pains. Going to see a dr tomorrow, What kind of specialist do i need to see? How can i get drs to respect and treat me well? Even with all the meds I m on, I m in consyant pain. I actually just received a form of Medicare, hoping that helps Doctor: Hi ,thanks for your query.You had injury to back at the time of accident .As you are having shooting pains with bruising , I would suspect bony injury to back or traumatic disc prolapse causing nerve root compression.If I would have been your doctor , I would advise Xray and MRI of DL and LS spine to see cause of pain . Advised to consult with neurosurgeon ."
},
{
"id": 55872,
"tgt": "Treatment for hepatomegaly, nonspecific coarsened liver echotexture",
"src": "Patient: These are the results on our son who is 6 yrs old. Could you help me understand. He has also had anzym test and it is normal. EXAM# TYPE/EXAM RESULT 0000 US/US Abdomen-Complete COMPLETE ABDOMINAL ULTRASOUND INDICATION: Hepatomegaly. COMPARISON: Abdominal ultrasound 06/04/2008. FINDINGS: Pancreas: Not well seen, but grossly negative. Liver: Heterogeneous echotexture. No focal mass. At the midclavicular line, sagittal liver length is enlarged, measuring 14.2 cm. Gallbladder: Negative. Gallbladder Wall = 1 mm, normal. Sonographic Murphy Sign: Negative Biliary System: No calculi seen nor abnormal dilatation. At the midclavicular line, the common hepatic duct diameter is 0.6 mm, normal. Right Kidney: Negative; no hydronephrosis or calculus is seen. Length = 7.8 cm, cortical thickness: 11 mm. Left Kidney: Negative; no hydronephrosis or calculus is seen. Length = 7.3 cm, cortical thickness: 11 mm. Spleen: Negative. Size 7.7 mm, normal Aorta: Unremarkable. Size 1.2 cm, normal Inferior Vena Cava: Visualized and unremarkable. No ascites. IMPRESSION: 1. Hepatomegaly. Nonspecific coarsened liver echotexture; however, no focal liver mass is seen. 2. Remainder of the exam is unremarkable. Doctor: HelloCoarse echotexture of liver may indicate liver parenchymal disease.Normally liver is homogeneous in echotexture.Liver is also enlarged in size.It may be due to many causes like hepatitis,fatty liver,metabolic disorder etc.You may need clinical correlation and few investigations.Investigations include routine hemogram,liver function test,random blood sugar,lipid profile,urine RE/ME.Further investigations like viral markers,liver biopsy can be done if needed.You may need to take few medicines after evaluation. Other findings are normal.Get well soon.Take CareDr.Indu Bhushan"
},
{
"id": 111659,
"tgt": "What is the remedy for back pain?",
"src": "Patient: I fell front first onto a vinyl floor and hurt my knees and arm. A few days later I worked a 15 hour shift where I was on my feet the whole time and when I left I had to lift my left leg to get into the car. I was in a lot of pain in my left lower back and from there down my leg to my feet. I could not lay on my left side and the pain in my left lower back was constant. My Dr sent me for xray that showed nothing and then he sent me for a ultrasound steroid into the area between hip and femur. I still experienced the symptoms and the right lower back was still paining. That was about 6 weeks ago and since then it has improved. 4 days ago I reached down to do something and hurt my right side upper back and had pain so severe it felt like I had a broken rib. When I breathed the pain was severe felt like pain of pleurisy I had many years ago. Every time I moved, my back spasmed and I was in severe pain. I could not lay down. The dr at emergency sent me for xray and nothing to explain was the result. I still have the pain and whne I cough it grabs me. Right side below shoulder blade. Doctor: Hello Thanks for writing to us, I have studied your case with diligence.As per your history of trauma there is possibility of rib contusion.Rib contusion is not always visible on X ray .You may need to do MRI or HRCT to see details of injury and to see for any lung puncture.If there is lung injury then there will be increasing pain with every breath.I will advise to consult to orthopaedic surgeon.Medication like muscle relaxant and analgesic will reduce pain along with it use rib support belt.You may consult physiotherapist for pain management. He may start TENS, or ultrasound which is helpful in your case.You may need to take rest for early healing.Hope this answers your query. If you have additional questions or follow up queries then please do not hesitate in writing to us. I will be happy to answer your queries. If you find this answer helpful do not forget to give rating at end of discussion.Wishing you good health.Take careRegards Dr Vaibhav"
},
{
"id": 84099,
"tgt": "What are the side effects of topiramate 150 mg?",
"src": "Patient: hello doctor , m an epileptic paitent from last 3 years n taking topiramate 150 mg daily as per my neurologists prescribed ..... n from last few months m seeing some changes in my behaviour ..... i want to knw tht these changes of my behaviour are due to sideeffects of these medicines ... Doctor: Hi,Topiramate is commonly prescribed to treat various seizure disorders. Its common side effects include confusion, drowsiness, fatigue, lack of concentration, visual disturbances, diarrhea, and anorexia. Other side effects include psychomotor disturbance, impaired memory, speech disturbance, and depression.If you experience behavior changes during the course of treatment you need to stop taking topiramate and ask your doctor for an alternate effective but a safer medication to control the seizures.Hope I have answered your question. Let me know if I can assist you further. Regards, Dr. Mohammed Taher Ali, General & Family Physician"
},
{
"id": 50129,
"tgt": "Suffer with polycystic kidney disease, anurienzymes, fibromyalgia. Would it be advisable to have gastro sleeve?",
"src": "Patient: im seriously concidering having a gastro sleeve i have several health issues that make me feel abit cousious my drs think its a real good idea for me to have this surgery,i have polycystic kiddney diesase and thee anurizumes caused by the pkd . fibromialgia is a cronic problem for me . i feel that i have a positive attitude about the surgery. and also had a serious fall with a tramatic brain ingury last winter. would you have any advice about stomach surgury? Doctor: you have not mentioned your weight and why should you need gastric sleeve surgery.moreover with so many disease conditioned already associted i will not recommend the surgery."
},
{
"id": 53554,
"tgt": "Suggest treatment for end stage liver disease",
"src": "Patient: My husband has esld, we have dealt with this for 4 years now he has terrible mood swings, what sign,s should i look for to know that I need to get him help fast. Also he is diabetic now, has hep c, memory loss at times also barretts eshopahgus, now they say he has a hernia on same side as liver but it would be too dangerous to operate Doctor: Hi and welcome to Healthcaremagic. Thank you for your query. I am Dr. Rommstein, I understand your concerns and I will try to help you as much as I can.This is serious disease and quitting alcohol is the most important thing that should be done. Also, medications which are hematotoxic should be avoided. This is necessary to prevent disease progression which is lethal in most cases. At early stages it can be treated with these measures but but in case of progression, only liver transplantation may help. Diet should be balanced and healthy and get regular exercise. Limit high-carb foods such as bread, grits, rice, potatoes, and corn. And cut down on drinks with lots of sugar like sports drinks and juice. If there is viral hepatitis as udnelying cause then antiviral medications are required.I hope I have answered you query. If you have any further questions you can contact us in every time.Kindly regards. Wish you a good health."
},
{
"id": 201513,
"tgt": "Suggest treatment for bleeding penis rashes",
"src": "Patient: I had a penile rash that looked like the pictures of male thrush or balantis, they both looked similar. My penis got irritated and I started applying moconazole nitrate and it seemed to get a lot better, but today while i was wiping it down before applying miconazole before bed, it bled a little, what should i do? Doctor: It is advisable to consult a dermatologist and get proper diagnosis and treatment. Self treatment is not safe, you should find if it is of fungal origin or bacterial? Also see if you have got this after a sexual intercourse with some one particular or due to usage of any specific condoms? (it can be allergic contact dermatitis due to latex allergy)TC"
},
{
"id": 113534,
"tgt": "Pain in lower back, hips, buttocks. Using meloxicam. Ultrasound shows bulging herniated discs. Pain relief?",
"src": "Patient: Hi , I am 65 years old and I have been suffering from lower back pain for several years. I am on pain medication (meloxicam) which just dulls the pain. I found that I have severe pain in my hips , back and buttocks . My doctor checked for cancer but this was negative. The pain is now constant and walking or sitting is now very painful. I have seen a doctor in the US also who said that I have bulging, herniated disc in the L5 and L4. Please advise of what I can do to relieve my pain without surgery. Thank you. Doctor: Thanks for the query. You have disc prolapse which is causing back pain and leg pain. Initially it is treated with medicine and physiotherapy. In case of persistent pain one need to consider intervention such as pain block or surgery. This is usually decided on the MRI findings. In your case we need to see correlation between MRI findings and clinical symptoms to decide the course of treatment. I hope this information will help you. Thanks."
},
{
"id": 66326,
"tgt": "What causes a lump on the thigh?",
"src": "Patient: Hi, I am a 25 year old female of average weight . Today I noticed a small lump on my inner thigh. It is not movable, hard, and painless and the skin near it has a purple spot. I do not have a hx of anything similar , have never had cancer or an std. Also no drainage comes from this are when pressure is applied . Doctor: Hi, thanks for sharing your health concerns with HCM! Hi, If I were your treating Doctor for this case of an immovable, hard, painless thigh lump, I would come up with only two possibilities like an infected sebaceous cyst or a dermatofibroma-like benign tumor of the skin! However, it could be some lymph node, neurofibroma or histiocytoma also but possibly never cancer!Therefore, nothing to worry about this and try not to get infected there!I suggest you to go for an FNAC test of the lump for confirmation and to relieve your concerns!Hope this answers your question. If you have additional questions or follow up questions then please do not hesitate in writing to us. I will be happy to answer your questions. Wishing you good health."
},
{
"id": 102584,
"tgt": "How to treat reflux symptoms and sporadic diarrhoea in a 8 yr old who has vit D deficiency and allergic to cow's milk protein?",
"src": "Patient: My son is 8 years old and suffers with reflux symptoms and sporadic diarrhoea. we have had blood tests and biopsies taken during an endoscopy which showed to be normal although a mild reflux damage in his stomach was noticed. He is deficient in vitamin D and shows a mild allergy to cows milk protein. We have now implemented a strict milk free diet although there has been no improvement to the symptoms. What else have you come across that could be another avenue to explore as this is affecting his day to day standard of life and we are desperate to help him overcome it. Doctor: Hi, did you consider the possibility of lactose intolerence? if he can not tolerate any milk, it is a possibility. Alternatively, other conditions like celiac sprue, vit A defeciency, other food allergies like soy or bean allergy, viral infections etc should be considered. Though inlikely for a 8 year old child, thyroid and pancreatic diseases are also worth ruling out. Hope this helped."
},
{
"id": 66869,
"tgt": "What causes lump on the inner side of thigh just beneath testicles?",
"src": "Patient: I have a lump beneath my stomach,just above the pubic area and now there s a lump developing on the inner side of my left thigh just beneath the testicles.other then that There have been lumps developing on my thighs since years which automatically burst.but now these Lumps create pain Doctor: not to worry much as this is benign condition like infected sebaceous cyst or skin tags / fibroepithelial polypif really worries you can go for an FNAC test for confirmationall the best!"
},
{
"id": 220763,
"tgt": "Does stenotised cervix cause hindrance to pregnancy?",
"src": "Patient: My fertility doctor said I have a stenotised cervix and I cant get pregnant on my own. I m not a doctor but I can help but doubt what she tells me. I have a regular 25 day period and I m just wondering how does blood get out and on time EVERY month and sperm cant get it??? Doctor: Hallow Dear,Tight cervix is one of the factor of failure of conception. You have to find out how tight is the cervix. Very tight cervix will cause intolerable pain during your menstruation. Your question is very valid that if your menstrual discharge can come out how is it that sperms cannot go in. My dear, during the menstruation, the uterus is contracting and pushing the lining and the blood out when the cervix will open; while there is no such force for the sperms to get pushed inside the uterine cavity. The sperms swim their own way through the cervix towards the egg. However, if too much uterine force is required to open the cervix, it will cause considerable pain in lower abdomen and low back ache. May be the inner lining of the cervix is not conducive for the sperms. This is called as cervical factor which (empirically thought) gets corrected after dilatation of the cervix. To have the clarification on your doubt, you may undergo Ultrasonography in which the diameter of the cervical canal and the inner opening can be measured.I hope this clarifies your doubts. If you want to have further more discussion on this issue, you may ask me Direct Question. Dr. Nishikant Shrotri"
},
{
"id": 154333,
"tgt": "How to treat stage IV lung cancer?",
"src": "Patient: my aunt has stage IV lung cancer with the tumor wrapped around the aorta. She went through Chemo and Radiation from last July to January. The tumor shrunk then but now without any chemo it has grown back pressing on the main artery. She now is on hospice. Would she be a canditate for a new heart? Doctor: Hi, dearI have gone through your question. I can understand your concern. She has lung cancer stage 4. So surgery will not help. Chemotherapy and radiotherapy remains treatment options. But despite of treatment prognosis is not good. Life expectancy is not good. Sorry to say but it is fact. Hope I have answered your question, if you have doubt then I will be happy to answer. Thanks for using health care magic. Wish you a very good health."
},
{
"id": 192231,
"tgt": "What causes arms tremor and making strange noises with the mouth?",
"src": "Patient: high my uncle has these episodes where he will just be setting there and his arms will start to shake and he makes weird noises with his mouth and if you yell at him or shake him he will snap out of it,but he never remembers what happens while he is having these episodes can you help. Doctor: Hello,You are suggested to get your uncle clinically examined by a Neurologist. He should get Nerve conduction tests, nerve biopsy & if required imaging tests (CT, MRI, EEG) done to detect any developing neurological disorder, dystonia, Essential tremor or Parkinson disease, etc. His treatment will depend on the pathology detected & the severity of the symptoms. In the meantime, give him vitamin B6, B12 rich food or supplements. Take care. Hope I have answered your question. Let me know if I can assist you further. Regards, Dr. Nupur K., General & Family Physician"
},
{
"id": 9599,
"tgt": "Dry red skin on knees and face, Oilatum bath formula, Alfacort cream ineffective",
"src": "Patient: hi my 8 monts old son got a very dry skin (red) at the back of his knees and in the face . his dr recommended oilatum bath formula and alfacort cream and yet it doesnt go away. I have swithched to dove soap bar from johnson s and johnsons as well. can you pls help me what to put on his dry spots and do you know any dermatologist I can take him in Doha, Qatar? Thanks. Doctor: hello, welcome to healthcare magic. i will be glad to be of assistance. it will best to apply pimecrolimus cream twice a day on the affected areas. it will be effective. most likely your son has a condition called atopic dermatitis, which causes the skin to get red, itchy and dry. oilatum bath and the alfacort cream can also be continued. i am sorry but i cant recommend a dermatologist in doha. regards. please let me know your queries."
},
{
"id": 213735,
"tgt": "Is there any treatment for stuttering ?",
"src": "Patient: hi i am swapnil hatkar i am 26 yer old. John stuttered from childhood Doctor: hello stuttered can be well treated by homoeopathy but apart from medicine what you need is to develop you self confidence there is nothing in the world which cant be change.if you are knoeing your oroblem then you can search the way.so whatever way of treatment but change your self.you should go for speech therpy. thanks"
},
{
"id": 19144,
"tgt": "Is heart palpitations and a leaking valve with chest pain serious?",
"src": "Patient: i am having pains under my left breast and rib cage, also, I am having pains in my chest. I have just been diagnosed with heart papitations and a leaking valve. I have been told, bc I am not getting dizzy or feeling fatigue, it not anything that I should weary about. Could this be something serious?[ Doctor: Hello!Welcome and thank you for asking on HCM!I passed carefully through your question and would explain that it is important performing a comprehensive differential diagnosis between different possible causes that may lead to this clinical scenario: inflammation or a musculo-skeletal pain, a lung disorder, a cardiac issue, etc.. Coming to this point, I would recommend performing further tests to investigate for the possible underlying causes: - a chest X ray study and pulmonary function tests- complete blood count, PCR, ESR for inflammation- an exercise cardiac stress test to exclude myocardial ischemia- thyroid hormone levels to exclude a thyroid gland dysfunction. You should discuss with your doctor on the above tests. Hope you will find this answer helpful!Kind regards, Dr. Iliri"
},
{
"id": 132059,
"tgt": "What are the uses and side effects of gabapentin?",
"src": "Patient: does gabapentin really help pain in lower back. My husband was put on that last week and so far, nothing! He fell 45 years ago and the five bones to the pelvis were like a spider web and coccyx (tail bone) was crushed. He feel 25 feet to the ground and landed on his tail bone with both legs straight. Then 8 years ago, he was moving a bath tub and he over corrected using a cart and it pushed him backward but he remained on his feet just pushed all the muscles backward. Now at age 72 he can hardly do anything, pain is so bad. He does yard work on his knees and can hardly get up out of a chair. Pain all of the time even when riding in a car? Will this Gabapentin help this pain? other pain meds have not worked. Doctor: dear user, intake of Gabapentin helps reduce nerve pain, but also causes dizziness, drowsiness, nausea, or vision changes, swelling of legs and may also cause tremor.try to avoid it."
},
{
"id": 141662,
"tgt": "What causes sharp pain in centre and lower section of neck?",
"src": "Patient: i have been getting sharp pain in centre and lower section of my neck ... it fades after time is wurse when inhaling and dull on exhaleit takes hours to go away... is much wurse when i try laying down and onsite is sharp and extremewhat might it be? Doctor: Hello and Welcome to \u2018Ask A Doctor\u2019 service.I have reviewed your query and here is my advice.It could be due to Cervical Spondylosis.Please start neck extension exercises for relief from pain.Hope I have answered your query. Let me know if I can assist you further.Regards,Dr. Sudhir Kumar"
},
{
"id": 33122,
"tgt": "Is having brown-tinged urine while having candidiasis normal?",
"src": "Patient: I was diagnosed with candidias and i have been takin medicens and I suspect that I suffer from it because I went on a birth control pill. my first question : Is urinating Brown Urine normal while having candidias? Question 2: Me and partner Had Sex now his penis Burns and has reddish rash on it, Is that serious, Should we use a med.? Doctor: Hi, I have gone through your question and understand your concern. Urinating brown urine means you are taking less water. In candidiasis there is a white discharge from vagina so white fragments may come in urine. Burning and rash on penis may be due to other infection for that you must take proper antibiotics like azithal and for candidiasis take fluconazole or apply candid cream locally. Maintain good hygiene and avoid sex."
},
{
"id": 173255,
"tgt": "Why my 3 year old's heart is paining?",
"src": "Patient: My 3 yr old had an asd closure with an amplatzer occluder 2 months ago. Today she told me her heart hurt. Then a few minutes later told her dad she felt fine. Can she be feeling pain from this or should i be worried and call her cardiologist. He is 4 hrs away from us... Doctor: Hi...I suggest you call on her cardiologist. As it is only 2 months down the line after the procedure, it is better to have a cardiologist have a look at her. Another important thing - do not wait till 4 hours, as of now - consult your local physician and get her vitals checked before you start to the cardiologist so that you can be rest assured.Regards - Dr. Sumanth"
},
{
"id": 30316,
"tgt": "Is acute pain in abdomen cause of positive GBS?",
"src": "Patient: my 20yr.old daughter has just tested positive for GBS. She went to the doctor originally because of acute pain in the abdomen radiating down right leg. 500mg of penicillin had immediate effect on the pain. Dr. says that GBS is not the cause of the pain??? Any ideas??? Doctor: Guillain-Barre syndrome (GBS) is a rare disorder that causes your immune system to attack your peripheral nervous system (PNS). The PNS nerves connect your brain and spinal cord with the rest of your body. Damage to these nerves makes it hard for them to transmit signals. As a result, your muscles have trouble responding to your brain. No one knows what causes the syndrome. Sometimes it is triggered by an infection, surgery, or a vaccination. Antecedent infections are thought to trigger an immune response. Symptoms are preceded by an antecedent event. Respiratory infections are the commonest followed by gastroenteritis as the antecedent cause. The commonest manifestation is limb weakness. In your case abdominal pain might be due to gastrointestinal infection i.e. antecedent event which triggered GBS. So be in contact with your doctor. Intravenous immunoglobulin & plasma exchange (plasmapheresis) are treatment options of GBS. Thanks"
},
{
"id": 135861,
"tgt": "Suggest treatment for severe pain in the armpit",
"src": "Patient: On and off severe pinching to both armpits radiating down rin cage. One side or the other. Never at same time. Relieves itslef within 5 mins. Comes on weak then gets real stong, to the point i cant move arm, then passes with no intervention. No swelling, redness noted. Happens about 1 a day latwly x last 2 weeks. Any ideas? Doctor: hiA physical examination of shoulder joint is required and look up for any deep axillary lymph nodes tenderness.Take tylenol tabs in meantime and consult a GPthanks"
},
{
"id": 223202,
"tgt": "What causes no bleeding in gap week of contraceptive pill?",
"src": "Patient: I have been taking the contraceptive pill for over 2 years and have never missed a gap week. i am on my gap week now and still no bleeding? i was sick and had antibiotics and pain killers for my back during my last 21 days but used condoms as extra protection. should i go to my gp? Doctor: Hello,The company which you are saying suggesting me to rule out pregnancy first so for that reason opt for urine pregnancy test but at the same time chances of it to come positive is very less but still would like to rule it out first, then if its negative it means because of your recent illness your periods have got delayed so start taking multivitamin and iron tablet because many times because of nutritional deficiency also periods dates may get altered.Hope I have answered your query. Let me know if I can assist you further.Regards,Dr. Medhavi Agarwal"
},
{
"id": 40649,
"tgt": "Is Fertigyn effective in the treatment of infertility?",
"src": "Patient: hi doctor on my 16th day after taking hmg 150 iu my egg was 19*21 and doctor asked me to take fertigyn 5000iu . i took yesterday so when do th egg get rupture and doctor has asked me to have contact for next 4 days. so what is chance of getting prgenant Doctor: Hi there,I have understood your concern and and I will suggest you the best possible treatment options.Please do not worry.Have complete faith in your treating doctor.He has rightly suggested you to take injection HCG- Human Chorionic gonadotropin.This helps to release the egg from follicle.Please keep doing USG for follicle study. It will tell you about the exact day of ovulation. Egg is viable for 24 to 36 hours after that. Having natural unprotected sex activity on and around that day will help you to get pregnant.I will suggest you to opt for a healthy diet and regular exercise regimen.Start on Folic acid and vitamin B 12 supplements regularly. This helps to prevent various complications during pregnancy and delivery.I hope this answer helps you.Thanks.Dr. Purshottam Neurgaonkar"
},
{
"id": 62172,
"tgt": "Suggest remedy for lumps on tongue",
"src": "Patient: My tongue has a burst blood vessel that has now become so large that it is the size of maybe a half a golf ball in circumference, which is purple and black in colour with small lumps which is now so bad I can t even go to dentist as the last time i went they actually told me they would not even attempt to do any dental work what so ever and they referred me to the hospital which they tested and come bk as benign (even of a minor amount) due to the fact they do not know if they are disturbing anything underline, my teeth are now at disgusting standard and causing myself alot of stress, embarrassment and anxiety and I need work done to them who or where do u turn for help? I can sed you a picture if required thanks in advance Doctor: Hi Dear,Welcome to HCM.Understanding your concern. As per your query you have lumps on tongue along with poor condition of your teeth which seems a lot going on in your mouth. Burst blood vessel on the tongue as in your case need proper medical evaluation. Lump on the tongue is definitely not a normal part of body. The possible causes can be Benign lump of the muscles, Foreign body granuloma, Cancer and Infection in the cut. For now do warm saline rinses several times a day and maintain proper oral hygiene . Proper diagnosis ascertained only by a clinical examination, FNAC or Excision biopsy. I would advise the excisional biopsy for to get rid of the lump and you get a histo-pathological diagnosis to be certain the diagnosis and start treatment accordingly. I would suggest you to consult oral pathologist for proper diagnosis and treatmentHope your concern has been resolved.Get Well Soon.Best Wishes,Dr. Harry Maheshwari"
},
{
"id": 132577,
"tgt": "Suggest treatment for leg pain",
"src": "Patient: Hi.my name is tara,I suffer from severe major intense leg cramping,spasms,restless legs,tremors,porralises,fire burning pain,bone pain.also my left arm biceps/triceps area the muscle riped in half after a very bad spasm accorde witch started out as a tic...sometimes my legs turn sunburnt red,then around the knees strat turning purple then almost black the pain is horrific..the pain starts at hip level goes to my feet.....the cramp size gets as big as grapefruit sometimes.please help me.what is happening to me?is this possible m.s.its getting worse all the time.im scared ....my feet swell up also..I m not a diabetic just to let you know..this is crippelling me..I m 48/female. Doctor: Hi tara Hope this message finds you in good health.I have gone through your complaints and understand your concern.U seem to have some sort of hormonal,multiple organ disease causing the problems.U should get an detailed blood profile done to find out the exact cause. Such symptoms are common in menopause phase.Nothing to worry about.\u00a0\u00a0\u00a0\u00a0\u00a0I hope your question has been answered.If you have any follow-up queries,feel free to consult me anytime.Thanks,Take care,God bless."
},
{
"id": 186425,
"tgt": "What is that i got small circular in my tongue at tip and bottom?",
"src": "Patient: i have this small circular in my togue at the tip and bottom,it doesnt hurt but made me sick of thinking what it is,the first i saw its just 3 circular red like the tip of a match but not that big now they become more,and noticed that i had it also on the other side of my togue,pls give me an advise Doctor: thanks for your query, i have gone through your query. the the small circular area could be a depapillated tongue or benign migratory glossitis or an ulcer. consult your oral physician to rule out these two. if i am your treating doctor i would have suggested symptomatic treatment like topical anesthetics and analgesics with multivitamins. i hope my answer will help you. take care."
},
{
"id": 122880,
"tgt": "What cause pepperish in my head,eyes and stomach?",
"src": "Patient: why im having pepperish in my head, eyes, stomch? and also seems my head is block. pls Dr. help and answer my question, im in Nigeria now and im a Nigeiran don t know how to pay you. please im pleading to u with God s name help and answer my question. Thank you. Doctor: Hello, This can happen due to any allergy or due to gastritis. You need to consult a physician for a direct examination and treatment. I hope this information has been both informative and helpful for you. Regards, Dr. Praveen Tayal Orthopedic Surgeon"
},
{
"id": 224837,
"tgt": "What is the cause to bleed after sex after 3 negative hpt's?",
"src": "Patient: i am 5 days late and i had 3 negative hpt's 2 days ago.. after having intercourse with my husband i noticed some blood traces but it was very light. i thought i started my period.. but when i woke up this morning no blood at all. up to now still no blood.. what this could be?? Doctor: you are having no pregnancy. how ever husband you discuss and observe proper precaution. There are many safe method for you aswell as your husband. you should undrr stadit follow."
},
{
"id": 41023,
"tgt": "Is masturbation problematic when trying to conceive?",
"src": "Patient: Should my spouse masturbate when trying to have a baby. In Sept we lose 3 babys and my baby Dr it s ok now to try agian. My spouse seems has a problem cum right now . He told me that he has been masturbate when iam not hm and should he stop or does this matter Doctor: HelloMasturbation will not cause problem in conception.Loss of babies are not related to masturbation.It may be due to some other reasons.Your doctor has rightly said that you can try again.As your case is high risk,you need special care and attention.Wish you all the best.Take CareDr.Indu Bhushan"
},
{
"id": 24946,
"tgt": "What causes spikes in bp along with tingling in face, teeth and gums?",
"src": "Patient: I have been having a burning sensation in my face and tingling teeth and gums for 5 days. My blood pressure was normal until yesterday when it went up to 175/89 with a 90 pulse rate. I went to my doctor and the blood pressure was down to 151 over 69 when his nurse took it and then down to 140 over 79 when the doctor took it. I stop taking a new multi vitamin ( take a variety of other vitatmins including fish oil, calcium and vitamin c, etc), thinking the new multi vitamin (it was a new brand for me) might be causing an allergic reaction. Today my bp was 122 over 77 with 84 pulse at 7:30 a.m. and then 117 over 79 with pulse of 90 at 10:30. Then I took it again at 12:30 on a different monitor and it was up to 156 over 85. My face is hot again and tingling is in teeth even though it has been over 24 hours since I took any of the new multi vitamin. I was a little dizzy this morning when the blood pressure readings were low. The doctor thinks I am having anxiety issues over this hot face and tingling which he feels is caused by the allergy to the new multi vitamin and the anxiety is causing the spikes in my bp. I am hopeful that he is right but am worried as the dizziness started this am. Any suggestions you may have for me would be appreciated. Doctor: HiYour blood pressure numbers aren't too worrisome and I think your doctor is correct about the anxiety issue"
},
{
"id": 178028,
"tgt": "What causes severe headaches,dehydration and cold/hot general malaise?",
"src": "Patient: 9 year old boy, healthy, no allergies, NKDA. In the last two months, has become sick, very suddenly at night. No fever, no vomiting (though he thinks he might). Severe headache, hot/cold, general malaise. attributed it to dehydration first few times but I have been strict about water intake as of late. worse night was tonight, he seemed to not be able to get comfortable at all (body aches?) Cool compresses helped, finally asleep. Seems like the flu but no fever and usually wakes up feeling 100%. Any ideas? Doctor: Hi....Fever of few days without any localizing signs could as well a viral illness. Usually rather than fever, what is more important is the activity of the child, in between 2 fever episodes on the same day. If the kid is active and playing around when there is no fever, it is probably viral illness and it doesn't require antibiotics at all. Once viral fever comes it will there for 4-7 days. So do not worry about duration if the kid is active.Hope my answer was helpful for you and if so rate it if you are happy. I wish your son a speedy recovery.Regards - Dr. Sumanth MBBS., DCH., DNB (Paed).,"
},
{
"id": 208408,
"tgt": "What causes tiredness and lack of concentration?",
"src": "Patient: hello dr. i am SOIKAT age 18 years and i am feeling and i am unfit becoz of following points i observed:- 1.i get tired quickly during physical work like playing cricket. 2.while making sex with my partner i cannot satisfy her and get over within 1 min. 3.i cannot concentrate in my work. 4. my weight is 70 kg for last 1 month. Doctor: hi dear,what ever you describes may be due to you have depression. but for depression to diagnose detail history and mental status examination is necessary.is there any other symptoms like sleep disturbance, appetite disturbance, easy irritability, negative thoughts, low mood, loss of interest in work etc.....for satisfaction of female you have to do foreplay to excite her first than do sex and after play for better satisfaction.for detail counselling and treatment consult psychiatrist.not to worry.there are many medication and psychotherapy which help you.Thank you"
},
{
"id": 171979,
"tgt": "What causes painful urination after circumcision?",
"src": "Patient: my grandbaby was circumsised 2 days ago. The gauze like tape has not fallen off,yet. It did not hurt to pee at first, but now it seems to be painful. He trys to hold it until he can no longer hold it,and then he cry, profusely. Is that last piece of guaze restricting him, or is something else going on? Doctor: Hi,Thanks and welcome to healthcare magicYour history suggests that it may be a stitch infection leading pain and consequent holding of urine.Better consult your doctor.He will suggest antibiotics and remove the stitch that has not absorbed or come out.Hope this answer serves your purpose .Please feel free to ask further queries if any.Dr.M.V.Subrahmanyam."
},
{
"id": 68394,
"tgt": "What causes lump on hip above pelvic bone?",
"src": "Patient: I have a lump on my left hip above my pelvic bone. It started out small when i first got it (approximately 6-8 months ago) & it's grown to be larger. It's underneath the skin & painless. You can't feel it unless you apply pressure. It feels like a bone but i know it can't be a bone. What could it be? Doctor: Welcome to health care magic. 1.Lump in the pelvic region and seen prominent on applying pressure and painless - suggest possibilities of inguinal hernia.2.In this case the next step should be to beta an appointment and request for an ultrasound scan after physical examination.3.The ultrasound will detect any kind of lump,assess the nature of the lesion and its content.4.In case of hernia anterior abdominal defect will be measured on cough reflex, and evaluate the contents of the hernia, weather there is any strangulation or not.5.So get appointment and needful investigation and treatment. Good luck.Hope i have answered your query.Any thing to ask do not hesitate. Thank you."
},
{
"id": 164210,
"tgt": "Suggest medication for continuous vomiting,sore back and head ache",
"src": "Patient: My 7 year old son is vomiting non stop - since last night ( 15 hours now), cant keep any of the meds down... also has a fever, complaining of a sore back and head-ache. we were out on a farm yesterday where he cut his foot rather badly too, am worried its some infection perhaps from the cut ( there was animal dung all over the place and he was barefoot) or is this completely irrelevant ? and the two are not linked ? Doctor: Hi...continuous vomiting with head ache is a red flag sign and may be associated with a zoonotic disease due to exposure to animal dung. Vomiting with headache signifies that there could be raised intracranial tension. I suggest you take him to the nearest emergency room immediately and get him evaluated.Regards - Dr. Sumanth"
},
{
"id": 89454,
"tgt": "Suggest treatment for abdominal pain",
"src": "Patient: Hi, the last 2/3 months I hav had 3 boats of the same thing. nmost recently last night and still getting lesser pains now. Im woken by sharp pains just below my stomach which fade then come back stronger and stronger each time until it reaches a point where I start getting really hot and stripping off all clothes. U always end up on the floor in the featal position with my partner lit throwing cold water over me. I do feel nauseaous but am never sick, but then the need for number 2 as the pains continue. Eventually I go and on and of coninue to go for the next 2 hours until it changes from being hard to soft to water. COmpletely wipes me out for the whole day.? Doctor: HI.Thanks for an elucidate history. The most probable cause of such type of pain is Volvulus of the intestine ot an acute intestinal obstruction. The best way to get a right diagnosis is to have an X-ray in the standing position at the time of an attack and can be sufficient simple investigation to prove a point. IF this is not sufficient , you may have to go for CT scan of the abdomen , and all other tests of blood, urine and stool. You may need a corrective surgery once the cause if found."
},
{
"id": 16540,
"tgt": "Suggest treatment for pain in chest",
"src": "Patient: i am a hypertension patient, but may be due to heavy exercise done previously like lifting of weights while lying down, i am getting pain in chest specially during winter months. I have done all tests of heart like angiography,doppler test,TMT test and all test related to heart and that also from good hospitals in india ,like FORTIS moali (punjab) and Alchemist (panchkula). Please do the needful for advising me and prescribing me suitable medicins. Doctor: Hi, Chest pain can be because of a variety of causes, the most important being due to heart for which you have already been evaluated thoroughly. The pain which occurs periodically is generally not Cardiac especially with the type of history you are telling. It looks most likely the pain due to muscle pull or some ligament injury. The treatment is with basic pain killers with muscle relaxant combination (like Tab Diclovis MR/Tab Myoril). You should also not do heavy exercise so at least 6months. I will advise you to consult an orthopaedic Doctor who will tell you the proper tests to be done and do'es and don'ts for the same. Hope I have answered your query. Let me know if I can assist you further. Take care Regards, Dr Sameer Maheshwari, Cardiologist"
},
{
"id": 195093,
"tgt": "What causes penile bleeding after a bowel movement?",
"src": "Patient: My son is bleeding from his penis after a bowel movement with painful burning. No pain during ruination alone though. Being in the ER twice and all urine and blood work are normal. Cr scan and ultrasound of kidney and bladder showed everything is normal as well. He\u2019s only 26 and I\u2019m very concern. He\u2019s overweight and sits for long periods of time , not sure if that matters. Please help! Doctor: Hi, Consult a urologist and get evaluated. Hope I have answered your query. Let me know if I can assist you further. Regards,\u00a0\u00a0\u00a0\u00a0\u00a0 Dr. Shinas Hussain"
},
{
"id": 37179,
"tgt": "Suggest treatment for dog bite",
"src": "Patient: On 17th of October, my 8 years and 10 months old, weight by 26.6 kg son was bitten by street dog (bitten more than 8 people on the same day), two units of Human Immunoglobilin was inject on the wound area by the doctor on that day, and ten antiboitic injection was given for five days (two times a day) which is completed now, and taking rabipur, on a count of 0, 3, 7, 14, 28 (two is remaining on 31st of October and 14th of November), Sir please do suggest me for further care, and what food and things should be avoided? Doctor: Hello,Welcome to HCM.I am Dr Rakesh Sharma answering your query.I can understand your concern.His anti rabies injections schedule is right. If wound has healed, no more care of wound is needed.You can give him every thing to eat.Hope I have answered your query, if any thing else,you are welcome."
},
{
"id": 77427,
"tgt": "What causes weakness and severe cough in a person with asthma?",
"src": "Patient: I am coughing a lot feel very weak when I eat, food doesn t go down easy. I do have asthma but this doesn t seem to be in my lungs, more in my chest, also out of breath alot. Can t seem to walk more than 5 feet with out losing all air. I m almost 67 and am 5ft.4in.tall and weigh 115 lbs. Doctor: Thanks for your question on Health Care Magic. I can understand your concern. In my opinion, you are mostly having acute exacerbation (worsening) of asthma. This is the cause for your constant cough, weakness and breathlessness. So better to consult pulmonologist and get done 1. Clinical examination of respiratory system 2. PFT (Pulmonary Function Test). PFT will tell you about severity of the disease and treatment of asthma is based on severity only. You will need inhaled bronchodilators (formoterol, salmeterol) and inhaled corticosteroid (budesonide or fluticasone). Combination of antihistamine and anti allergic drugs is also beneficial in your case. Don't worry, you will be alright. Hope I have solved your query. I will be happy to help you further. Wish you good health. Thanks."
},
{
"id": 99158,
"tgt": "What caused my mother's anaphylactic reaction?",
"src": "Patient: Dear Dr, My mum had a suspected anphylactic reaction to anaethetic 4 years ago, she then was refferred to an allergy clinic to have further tests in ICU, all test resulta came back negative, mum had to go in for another operation and the proffessor from the allergy clinic advised to premedicate her 24hrs and 4hrs prior with citrizine and prodeslone and the operation ran smoothly. mum went back in to hostiptal recently under a different surgeon and this time was advised againt the professer and only to take the citrizine and prodeslone only 24hrs prior to operation. mum was told to arrive at hospital at 7am to comence surgery by 8am as was first on the list then surgeon arrived and told her she wouldnt be going down until 12-1pm, mum was concerned the pre-meds would be wearing off (hospital staff said 'dont worry we will give you more in surgery'). Mum didnt go down for surgery until '4.45pm'. I was reunited with my mum at 9.10pm, no operation had been started as mum had another anaphylactic reaction!! Please could you tell me if this is neglegance and if this could be the reason my mum has had another anaphylactic reaction due to the pre-meds wearing off and not following the professors guidelines? Thank you in advance. Doctor: Hi I have read all your queries and understand the concerns.If your mother had allergic or anaphylactic reaction to certain drugs then it must be disclosed to the doctor before going for operation.Yes you may be right that the operating surgeon must have to give pre-surgical medications but when if it is been clearly said my patient himself or relatives that the listed drugs are allergic.In spite of this,if surgeon had not operated and if your mother was not given medications or having said they will give during operation could be a case negligence (act of omission).Hope this will help you.Regards,"
},
{
"id": 200528,
"tgt": "Suggest remedy for swelling and inflammation in frenulum",
"src": "Patient: There s a hole in my frenulum which attaches to the glans of my penis. It hurts to have sexual relations with my girlfriend, with swelling and inflammation occurring at the site. Originally there was a tear, which I applied antibiotic ointment to for several weeks. Thinking it had healed I tried having sex today but it still hurt. Any idea what I should do? Doctor: Thanks for asking in healthcaremagic forum If there is till a tear which is not healed/huge tear which does not heal on itself, it needs to be seen by surgeon. Recurrent infection can cause tear and pain over the frenulum. Please visit dermatologist/surgeon for examination and further management. All the best."
},
{
"id": 192681,
"tgt": "How to cure pain in testicles?",
"src": "Patient: I've got Kidney disease and my left testis is double it's size! I'm in so much pain i cant move, i took some pain pills left over from other issues and my head is dizzy can think strait and i faint standing up. what pain pill do you usually give for testicle pain? Doctor: Hello, The symptoms are more likely suggestive of conditions like orchitis( infection of the testes). You can go for an ultrasound scan of the testes to confirm the diagnosis. Nothing much to worry and most cases respond well to antibiotics. Hope I have answered your query. Let me know if I can assist you further. Take care Regards, Dr Shinas Hussain, General & Family Physician"
},
{
"id": 153916,
"tgt": "What are the symptoms and treatment for bone cancer?",
"src": "Patient: I have intermittent bone pain in 5 specific areas. It has been going on for over a year but seems to be getting more intense and more frequent. The bones affected are my right ulna, left humorous, right femur, left foot and pelvic bone. It is stabbing, sometimes throbbing pain that can be very intense and then it just goes away. i had a terrible ear infection recently that took 6 weeks to heal. We finally figured out that the infection likely started in my in my mastoid bone. I was treated for mastoiditis and have no idea if that is related or not. I am a 37 year old female that is otherwise in good heath. I have 5 children and am very active. Two recent blood tests showed elevated bilirubin and blood in my urine but a liver work up showed nothing. That was about 10 months ago.We recently moved to a new state. The bone pain is getting worse and I would like to see a doctor. I am worried I may have bone cancer or something serious. I don t want to go into a new doctor and demand testing, but what is the best way for me to encourage my doctor to rule out anything serious? Doctor: Hi,Thanks for writing in.Bone cancer might involve any bone and rarely also affect multiple bones. The usual signs of bone cancer are swelling which is sudden in onset and rapidly progressive with pain in the region. Some bone cancers can also cause signs of inflammation in the region.Since you have been diagnosed to have mastoiditis therefore it is almost certain that your condition is infective in nature. Mastoiditis occurs due to chronic infection of the middle ear which involves the mastoid bone. Treat with medicines is recommended and in some patients surgical treatment is required and it is not a bone cancer.. Please do not worry."
},
{
"id": 151988,
"tgt": "Treatment for intramedullary astrocytoma of spinalcord",
"src": "Patient: my son is suffering from intramedullary astrocytoma of spinalcord any cure Doctor: Hi cathy, intra medulary can be operated by microscopic surgery butpost operative neurological sequele depends on the size of tumor, site and operating surgeon and method. so consult neurosurgeon for further details. Take care"
},
{
"id": 224842,
"tgt": "Missed birth control pills. Heavy period, tiredness, headaches. Anything to worry about?",
"src": "Patient: Hi. I ve been on my birth control for over six months. A couple weeks ago I accidentally missed a day or two of pills. I started my period then and have continued taking birth control normally and on time. I ve been having a unusually heavy period with no end in sight for two weeks. I m feeling weak and tired with headaches . I m wondering if its anything to worry about or should I see a doctor? I am on a high hormone pill, they upped my dosage six months ago because I was having small periods every weeks. Until now on the new dosage, my periods have been on time. Doctor: Hi. Your intake of pill was quite erratic and this has probably caused irregular and heavy bleeding. Consult your dr. She may like to get a scan done before putting you on a higher dose of hormones this time to control bleeding.Other symptoms are due to heavy blood loss and anaemia. Thanks."
},
{
"id": 118300,
"tgt": "Is there a connection between being thalassemic and taking aspirin?",
"src": "Patient: I have thallasemia minor and have been taking a baby aspirin for about a yr. Many times I would get light headed if stood up quickly. I recently had to stop the aspirin for minor skin surgery. I have not been light headed since stopping the aspirin. Is there a connection I should be aware of being thalasemic and taking aspirin? Doctor: Dear I read your scenario. Its not good to take aspirin for minor headache because there are other drugs available in the market for minor headache like Paracetamol. Aspirin has antiplatelet activity which can causes bleeding and further worse the thalesemic picture. So you should be very careful about aspirin. Hope you willfollow my suggestion and you can ask mmore question regarding this issue otherwise you should go to near by doctor. Thanks for asking."
},
{
"id": 25327,
"tgt": "Suggest remedy for fatigue in a person with heart ailments",
"src": "Patient: My husband had an angioplasty ant 2 weeks back after a minor heart attack and had 2 stents placed . He is 43 years old. He had an angioplasty 6 years back at 37 and had 6 stents placed at that time. He resumed work early this week just 8 days after the procedure however he feels very weak since morning Doctor: Hello!Welcome and thank you for asking on HCM!It is normal to feel week after such a short period after angioplasty and stenting. Nevertheless, I would recommend consulting with his attending physician for a careful physical exam and some tests: - a resting ECG and cardiac ultrasound to examine his cardiac function and structure and exclude possible heart failure- blood lab tests (complete blood count for anemia, PCR, sedimentation rate for inflammation). I would recommend having some rest for a more prolonged time and avoiding straining physical activity during this period. Hope to have been helpful!Best wishes, Dr. Iliri"
},
{
"id": 89523,
"tgt": "What causes pain in lower abdomen?",
"src": "Patient: Hi Dr, I have been having this sharp piercing pain around my lower abdomen just close to my navel, which seems to radiate down to d point around my genital. This comes and goes away but can be felt when ever I sit down and sometimes when I stand. The confusion is I had an appendicetomy when I was barely 17 yes and 17yrs later I begin to have these pains. What could possibly be wrong with me? Doctor: Hi Thanks for asking HCM. What ever symptoms you have written may not be related to your previous operation.But you should get your ultra sound done to confirm the diagnosis.Though till then you should take plenty of oral fluids fiber rich small and frequent fiber rich diet.You can take antacids SOS and add mild laxatives if constipated.Consult your doctor with reports if not benefited."
},
{
"id": 16662,
"tgt": "Suggest treatment for high blood pressure",
"src": "Patient: My mom she is 59yrs old she has BP and diabetes.2 year s ago her sodium went down low and she was unconscious for a day in hospital and recovered fine. but nowadays and especially last 6 months her BP goes to 190 for few min and drops back to 130 that time she feels dizzy,wobbly. Checked with Cardiologist and neurologist they are unable to find anything.Please help. Regards. Doctor: Hi, Your mother should control her blood pressure and blood sugar level as well in their normal levels....she can take amlodipine tab to do so. Hope I have answered your query. Let me know if I can assist you further. Regards, Dr. Salah Saad Shoman, Internal Medicine Specialist"
},
{
"id": 30007,
"tgt": "Can Zimig 250mg intake be repeated over days for treating ringworm infection?",
"src": "Patient: I have taken zimig250mg for one week but ring worm agin appear after few days then i take again one weak after few days it back again then doctor said to take one doze more ihave taken but after few dsys it appears in some extent ..what will i do can i repeat the same Doctor: Hi,Thanks for contacting Healthcaremagic.I have gone through your query,\"zimig\" is the brand name of antifungal medicine terbinafine.Zimig is very effective for ringworm infection,We prescribe once daily or twice daily dose according to the severity of the infection.As you are having recurrence,you can go for twice daily dose.Use a topical antifungal cream along with the oral medicine. Clotrimazole is a commonly used for local application.Try this after having a word with your doctor.Hope it will work.Wishing you all the best.Thank you."
},
{
"id": 190040,
"tgt": "White lump inside the cheek. Could this be due to eating fried food?",
"src": "Patient: HI I have a white bump inside the back of my cheek. It appeared maybe 3 days ago. When I shone light on it it seems white and bumpy. It does not hurt but does irritate. I don t think it is a canker sore because it does not seem like the sores I m used to having (when I accidentally bite my lip). I did eat some fried food a couple of days ago, but am not sure if that is related. Please let me know what I should do. Doctor: Hello there.....Fried foods as such does not cause any problem...when hot foods are served that would have burnt your mucosa...But your presentation is as a white bump...that would have been an accidental finding...that would have been a fibroma that would have been present previously...since that would have grown in size or you would have accidentally seen the bump you would have thought that the swelling was only for the last three days... these kind of swellings wont hurt you except for the discomfort....Clinical evaluation is required before confirming the diagnosis...visit a Oral and Maxillofacial Surgeon for an expert opinion...."
},
{
"id": 95790,
"tgt": "I have a recurring cyst in my belly button",
"src": "Patient: I have a recurring cyst in my belly button. the last time it presented itself was in feb and i was hospitalized and saw a specialist who preceeded to tell me it was too late to remove because it had already burst however if it came back i would need to have my belly button removed. is this true and should i go to the hospital now since i woke up with severe pain again this a.m.? (same pain as before and in same spot so i know its the cyst back. ) Doctor: hi welcome to hcm as for the acute pain,u could take ultraset tablets. and consult your surgeon at the earliest. take care!"
},
{
"id": 202981,
"tgt": "What is the remedy for erection problems?",
"src": "Patient: Iv for the past 3 months bin having problems getting an erection and this is bringing problems in my home.I would struggle getting one which just fades away the moment I get it.can it be tht iv got one testicle or what? This has never happened to me before.iv got a son with my spouse who is 8 months old.pliz help me! Doctor: HelloThanks for your query,based on the facts that you have posted it appears that you have Secondary Erectile Dysfunction.In absence of any major health issue cause of ED is mind related and due to performance .anxiety.Following measure will help you to boost up your confidence and getting good erection.and delay ejaculation.1) Practice regular exercise for 45 minutes followed by meditation for 1/2 an hour in the morning.2) Take high protein diet rich in vegetables and fruits and Vitamin A,C,D,E.and Zinc3)Take anti oxidants like Almonds 5-6 everyday..4) Avoid alcohol and smoking..Taking Sildenafil (Viagra ) on demand will help to get good hard erection .Please consult your family Physician for prescription and guidance as regards timing and dose of the medicine.Do not worry this will get resolved soon.Dr.Patil.to be taken"
},
{
"id": 147160,
"tgt": "Can i take lorazapam instead of Keppra?",
"src": "Patient: My husband had a stroke and a seizure after one year. They currently have him on Keppra, which is causing up to 15 hours of sleep daily and he has lost 15 pounds in about two weeks. He has, in 4 years had 3 seizures , about 2-3 minutes. Can he get off this and maybe take lorazapam instead? Doctor: Hi,Thank you for posting your query.I have noted your husband's symptoms.He seems to be suffering from epilepsy, resulting in seizures.His seizures are fairly well controlled with keppra (levetiracetam), however, he is having a side effect of increased sleepiness with keppra.Lorazepam is not a good alternative, as it has very poor anti-seizure efficacy and this also caused too much sleep.Good alternatives would include sodium valproate and oxcarbazepine tablets.I hope my reply has helped you.I would be pleased to answer, if you have any follow up queries or if you require any further information.\u00a0\u00a0\u00a0\u00a0\u00a0Best wishes,Dr Sudhir Kumar MD (Internal Medicine), DM (Neurology)Senior Consultant NeurologistApollo Hospitals, Hyderabad,For DIRECT QUERY to me: http://bit.ly/Dr-Sudhir-kumar My blog: http://bestneurodoctor.blogspot.com/"
},
{
"id": 34430,
"tgt": "How to treat reoccurring yeast infection?",
"src": "Patient: I keep getting reoccuring yeast infections. I get at least two or three a year. I go to the doctor every time and I get prescribed Diflucan. Is there an oral pill I can take that is over the counter so I don't have to visit the doctor each time? Also, what can I do to prevent the yeast infections? Doctor: Hello dear,Thank you for your contact to health care magic.I read and understand your concern. I am Dr Arun Tank answering your concern.Yeast infection recurs only when you have some condition favouring its growth.I advice you should test for the HIV and diabetes. This are the two most common cause of the recurring yeast infections.Diflucan also available as a oral pills over the counter. But in my advice you should take it under your doctors guidance.Please maintain good local hygiene by frequent cleaning and dressing. You can also apply local ointment of clotrimazole while doing the dressing.Please avoid injury to the affected side otherwise the injury will increase.I will be happy to answer your further concern on bit.ly/DrArun.Thank you,Dr Arun TankInfectious diseases specialist,HCM."
},
{
"id": 2085,
"tgt": "Suggest treatment for irregular menstruation while trying to conceive",
"src": "Patient: Hi doctor, Whenever we try for a baby my periods are getting delayed by 15 more days. In the mean time i am also feeling like pregnant. But after 45 to 50 days i am getting my periods. We are trying for a baby. Wat could be the reason and what i can do to conceive Doctor: Hi I think there can be many reasons for delayed periods. So you should get yourself evaluated first. Do a thyroid profile and prolactin levels and a ultrasound for follicles. Also do a semen analysis of your husband. If everything is fine you can try with medicines and can do repeated ultrasound to see if ovulation is happening or not. If it's normal then you can try for 6 months. If it doesn't work out then you can go for IUI. Hope I have answered your question."
},
{
"id": 213218,
"tgt": "Difficulty falling asleep. Why was I getting restless? Is it anxiety or depression?",
"src": "Patient: hello im 25 yr old female.for the past one month i have diffficulty falling asleep at night.i was getting restless because of that.i started taking lesotanil 3mg half a tablet at night to sleep.it helps at nights but sometimes it doestnt.im still up al night.i dont suffer from anxiety or depression .but sleepless nights causes me depression and i cry for sleep.im shifting country and getting married in a month. Doctor: hi, if you have sleep problem only, then follow sleep hygiene for 1-2 weeks before starting any medicine. 1. don't sleep in day time, even short naps 2. don't take tea or coffee after 4 pm. 3. don't watch TV and don't use computers in evening, although you can listen or read anything. 4. never look at watch after going to bed. 5. do some physical exercise in evening 6. wake up at same time daily, even if you slept very late in night. I am sure practicing these for few days will solve your problem best of luck, sweet dreams"
},
{
"id": 202366,
"tgt": "How to get rid of itching in scrotum and penis skin?",
"src": "Patient: I have problem with my scortum and penis skin and it itches a lot and itching never stops. The skin shed in every two-three days and there is also a very bad smell which at times frustating. It is bound to happen on the day when i miss my shower but anyways it happens at regular intervals and itching continues throughout the day ,the smell stays on the cloths and is permanent , i have been applying candiderma daily once and it is not helping, i am a virgin , the skin becomes dry like our lips and sheds off like scales on our lips when rubbed , the skin devlopes red cut marks sometimes and the thigh skin in contact with scortum also has red patch , initially my skin used to havr burning sensation when soap was applied but i am not having that problem from past 6 months, pls help Doctor: HIThank for asking to HCMI really appreciate your concern looking to the given history here I could say that this could be nonspecific type of itching generally this does not need any treatment just take care for hygiene and the thing would come around no need to worry about this, take it easy, hope this information helps you, take care and have a nice day."
},
{
"id": 216514,
"tgt": "Suggest remedy for severe pain in top layer of skin due to allergy",
"src": "Patient: Most of my adult life I have had sensitivity to environmental allergies, and food allergies, Recently I have had three reactions.. that may deal with food and latex allergies ... I feel the redness and burning are causing more and more intense pain. I take Benadryl immediately after exposure. Is there anything else besides an epi pin I can use to decrease the pain level. The intensity of pain to the top layer of skin is very off the chart pains. Do I need to take a precaution ...often I do not know if food eaten or environment is the culprit. I firmly believe it is due to latex. I want to keep from anaphylaxis... what preventative measures do I need to have in place? Doctor: Hello,I can understand your concern. It seems that you have allergies to more than one substance and you do not exactly know what substances are acting as allergens for you. Thus, firstly, I would recommend you to go for a blood test for allergy. It will show in results what food and environmental substances are acting as allergen for you and so you can be alert in your day to day routine regarding exposure to these substances and avoid yourself from exposure. This is the best precaution you can take.In addition, you can also keep antihistamine such as Loratidine 10 mg (Lorfast) or Cetrizine 5 mg (Levocet) all the time with you so that as soon as the reaction starts, you can take the medicine immediately and control the reaction.Also, if you are allergic to more than one substances that you can be exposed to routinely, you can talk with your physician about taking an antihistamine regularly as hypertension patients take antihypertensives. Always take care to try new medicines and food items.I hope this information helps. Thank you for choosing HealthcareMagic. Take care.Best,Dr. Viraj Shah"
},
{
"id": 211384,
"tgt": "Could the strange, anti social behaviour, lack of interaction, being detached show possible psychosis?",
"src": "Patient: my brother seems to go through a phaze of possible psychosis , he wont eat for days sometimes , and has a mysterious personality as far as being introverted goes , but it now seems he has found christianity. there is nothing wrong with religion, and the fact he got into it but , he has become detached , anti social ,very strange behavior. he is only 20 and he seems as if he losing touch with reality , he wont sleep or anything , he just reads the bible day and night writing notes,he sits outside for hours in the cold repeating chants over and over for hours, then only interacts with family only a little bit. he looks like nothing but and empty vessel...it hurts my heart because me and the rest of my family know the old him and hes no longer there. Doctor: HiWith this history I would have considered in my patient the first possibility of Rapid Bipolar Mood Disorder and would have put him on lithum genetic name 300 bd with monitoring of blood level of drug to remain around o.8 and no tremors in hand and kidney function with in normal lilts your doctor will give you prescription it may long time for him to stabilise A personal examination by a psychistrist may be required for long term treatment and regular follow upWith history does not appear a schizophrenicBipolar Mood disorder Has a good out come on tratment .Be positive but put him on regular treatment Dr Lal Psychiatrist"
},
{
"id": 57965,
"tgt": "How to bring down elevated liver enzymes into normal numbers?",
"src": "Patient: i have elevated liver enzymes since the beginning of Sept. numbers were in the thousands along with a 12 for bilirubin which has since come down. Enzymes are now at 833. I am not feeling as sick as I did but am very concerned about what I can do other than wait for the numbers to come back to normal Doctor: Hi and welcome to HCM,thank you for your query.First you should find the exact cause of elevated liver enzymes which is most commonly fatty liver,heaptitis,alcohol, certain drugs or autoimmune diseases. You should do liver us and some more specific blood tests. Till that you should follow hepatoprotective diet,avoid alcohol and regulate body weight. stop drugs which can cause liver damage.Wish you good health. Regards"
},
{
"id": 8117,
"tgt": "Use of lacto calamine and conception",
"src": "Patient: Hi, I am a married lady planning to start a family soon. I am using lacto calamine aloe lotion for my skin as it has acne . Will this lotion affect my conception? pls reply . My age is 31. Doctor: hi dont worry. lactocalamine is safe in pregnancy and conception. it will not affect ur conception"
},
{
"id": 93716,
"tgt": "Have had irregular heavy periods, on pills, pain in abdomen, had appendix removed. Help?",
"src": "Patient: Hi, I have had irregular heavy periods since they started when I was 14, I am now 30, I had been on the pill for 12 years and they became regular, very light and lasted 5 days, I stopped the pill in an attempt to get pregnant in 2008, they returned irregular and heavy after nothing at all for 9 months, I then finally became pregnant and had my son in dec 2011 via c section, pretty straightforward other than my pelvis was too small for him to be delivered naturally so he needed a little help :) since his birth my periods were regular but extremely heavy for 5 months and then became irregular once again but my question is this ... I have a lot of pain in my right side around that time of the month before and after my periods which resembles a stabbing pain , it is so painful I have ended up in a and e and had my appendix removed 6 wks ago as they thought this was the cause but I came on a week before the pain and I have just finished a period and now the pain has returned again :( they did query fluid on the ovary before removing my appendix which was apparently inflammed, I m confused and fed up , please can you help? Doctor: Hi welcome to Health care magic forum. Thanks for calling H.C.M.Forum. You had irregular periods since minarchy, at 14 to 30 now.Used pills and you was allright. When you stopped pills again they are irregular and heavy, recently had a child. Now as the pain returned the doctors thought it may be appendix and operated for it. Again there is painafter period. Operated doctors said that there is fluid over the ovary and it is inflamed. in the Begening irregular periods may be due to anaemia, latter after delevery it may be due to infection, so accompanied with pain. any how you got operated for appendicitis, the doctors would have removed the ovary of right side when they have a doubt. As the pains have returned you can try medical treatment in guidance of a ginacologist for ovarian problem, and anaemia. Wishing for a quick and complete recovery. Best regards."
},
{
"id": 128636,
"tgt": "What causes twitching sensation in the legs while on Requip?",
"src": "Patient: I was diagnosed with restless leg syndrome several years ago due to my symptons of walking in the bed uncontrollably. My doctor prescribed Requip 3mg at night. About 2 months ago, the leg twitches started occurring every afternoon. The twitches are uncontrollable and they are like seizures between my knees and ankles. I am scheduled for an EMG/NCS procedure of my lower extremities. I understand the procedure, but my question is, how painful is this procedure? I have a very low tolerance for pain. Can they give me anything to relax me? Thank you for taking the time to provide your expert advice. Doctor: emg or ncs is not a very painful procedure. It's done with electrodes. Its a relatively short procedure. procedure is quite painless. Don't be afraid and go through tests. Before you know they will be over"
},
{
"id": 25956,
"tgt": "Can low Vit D or virus cause rapid heart rate in swimmers?",
"src": "Patient: My 16 year old daughter, who is a competitive swimmer, started having a rapid heartbeat while working out. It happened about 4-5 times over the course of 2-3 weeks and now hasn t happened again for 3 weeks. All tests came back normal, except she is low on Vitamin D. Cardiologist wondered if she has/had SVT but she has been unable to capture the heart rate using a portable heart monitor since it usually only happens when she is swimming (she has to get out of pool, get monitor, etc and by that time heart rate seemed to have slowed down). Could this be caused by low Vitamin D? or a virus? She had complained about being really tired too but feels less tired now than she did a few weeks ago. Mono test came back negative too. Doctor: vit d deficiency is not directly associated with increased risk of SVT.there are meta analysis of its supplementation decreasing cardiovascular risk in elderly but many have contrary results.Any myocarditis due to viral etiology can have this presentation but your cardiologist seems to reliably rule out those.she may need holter monitoring to see for episodes and a loop recorder which may capture the rhythm during swimming which may give a better idea."
},
{
"id": 18982,
"tgt": "What does an ejection fraction of 91% indicate?",
"src": "Patient: I just had a stress test and the doctors office said my results were good. I asked for a copy of the report and the report says that my ejection fraction is 91 %. I did some reading and learned the 50 to 70 % is normal. So I guess my 91% IS HIGH. Does that mean that my test results are not good and that I need to see a specialist? Doctor: Hello, Brief Answer: During stress or exercise, your heart ejection fraction increases.Explanation: My opinion is that during vigorous exercise or excessive stress, the heart responds by increasing both heart rate and function. That is meant by ejection fraction. In normal conditions, ejection fraction ranges from 50 to 70 %. Ninety percent ejection fraction may be a little bit overestimated but as long as there\u2019s the test is negative, nothing to worry about. The information provided by you is not sufficient to provide a good opinion. If someone comes to me with this condition I would ask them about hydration status during the stress test.Conclusion: I suggest you check with your physician.Hope I have answered your query. Let me know if I can assist you further.Regards, Dr. Mahmmad Gamal"
},
{
"id": 144579,
"tgt": "What causes nausea and dizziness while lying down?",
"src": "Patient: Hi, I am experiencing a drunkeness feeling and nausea when I lie down in bed at night. Sometimes during the day I have had dizzy spells too but not for a while. When I was younger I used to have blackouts, after speaking to a lady I met today at a hospital visit for something else, we got talking and she told me she had experienced these exact same things, and she only found out by chance after having investigations for something else that she has a brain tumour. My sister has also had a brain tumour. I am worried about this, can you help? Doctor: Nausea or vomitting occuring specially while laying in bed or bending down can be feature of raised intracranial pressure.Few brain tumors are genetically associated, but not all.as you have problems while you lay down in bed, i advise you to undergo MRI Brain and also MR Venogram of brain and fundus examination by opthalmologist to look for papilledema."
},
{
"id": 52773,
"tgt": "What causes severe pain in liver while having NAFL?",
"src": "Patient: I have been diagnosed with NAFL (by CT Scan) and I have had high uric acid w/gout for 3 yrs. I have pain, sometimes severe, in liver area especially after eating. I also take Protonix for GERT. Blood test are normal. I am 25 lbs overweight. Due to pain in stomach have not been exercising. Can not find out what is causing the pain and losing weight has been impossible. What should be done to determine the cause of the pain now? Doctor: Hi there,Given your problems of pain after eating, the most common cause of your pain is likely stones in your gallbladder. The easiest way to diagnose this is to get a right upper quadrant ultrasound to look for stones in your gallbladder. If you do have stones, and you can continue to have pain after you eat, the treatment is to have your gallbladder removed by a general surgeon, which is usually an outpatient procedure. I hope this answers your questions. Please feel free to contact me should you have any further questions."
},
{
"id": 219118,
"tgt": "What causes skin rash during pregnancy?",
"src": "Patient: Hello, my wife who is pregnant 12 weeks tomorrow (First pregnancy and no health concerns) has developed a rash over the last 3 weeks. We initially suspected that she had an allergy to a body cream but it has persisted. It initially began as an oval shapped (almost scratch like) mark on the side of her upper body near the above the love handles and now has burst into a rash all over her abdomen, breasts and a few on the back. We have gone to a dertmatoligist and she has diagnosed Pityriasis Rosea. 2 Questions, I have done research online and only found 1 study showing that it increases the risk of miscarriage and may cause mobility issues or strength issues in the muscles as an infant that can be overcome. Are there any other more concrete studies showing the effects of this and what is your experience with this virus amongst pregnant women? The second question is what are the differences in symptoms of Pityriasis Rosea and PUPPP? Doctor: Hello there, thanks for your query. Your efforts in researching for scientific literature is commendable. As you have rightly found out there are not much studies regarding pityriasis rosea in pregnancy. Those reports suggest possible miscarriage, premature delivery, movement issues in the baby. It is very difficult to draw conclusions from a single study which includes a very limited number of patients and hence the inference cannot be generalised. What one can do in such a circumstance is adopt a wait and watch policy and take adequate precautions to prevent miscarriage and preterm delivery. And undergo a detailed genetic sonogram (anomaly scan) to rule out structural defects in the baby. It usually starts as a single oval patch with well demarcated red boundary and central pink region which over 2 weeks spreads to trunk, back, neck and extremities and they are oval raised lesions which would look like Christmas tree. It spares face, feet and palms. Whereas puppp is seen in the last trimester of pregnancy and seen as papules along lines of striae over abdomen and thighs and it gets intensely itchy. It is most likely related to stretching of the skin. I would like to add that i have given gross features of PR and it should not be used to diagnose the lesion by oneself. Pityriasis rosea should be diagnosed by a trained dermatologist who does job at their best and the management in pregnancy should be a team work comprising of your obstetrician and dermatologist and a good radiologist.Hope this information is helpful to you. Take care."
},
{
"id": 186519,
"tgt": "How can soreness of lips and teeth be treated?",
"src": "Patient: yesterday i noticed my lips along my upper teeth (so inside of my lips) were sore, i didnt think anything of it until today when it started to be really sore (like i had braces and my lips were cut on them or something) when i looked in the mirror they are all red and look as if they might be bleading. it doesnt look like a sore though Doctor: thanks for your query, i have gone through your query, the soreness could be because of the trauma, or it can be an aphthous ulcer which gets aggravated by stress. consult your oral physician and get it ruled out. if i am your treating doctor i would like to put you on topical anesthetics, analgesics and steroids for a week. i hope my answer will help you, take care."
},
{
"id": 110295,
"tgt": "Suggest remedy for pain in lower and middle back",
"src": "Patient: I am also suffering from back pain percolating in middle and lower back. It may be due to prolonged working on computer and wrong posture. Can you suggest me how I can get rid of it and what are the precautions I have to adopt in daily routine of life.with Regards.M.C.Agrawal Doctor: Hi,Welcome to healthcare magic.After going through your query I think your You are suffering from chronic backache.It is related to prolonged working on computer and wrong posture. Treatment of back pain is exercises and analgesics (diclofenac 100mgSR). Sometimes vitamin D deficiency is the cause so get your vitamin D checked .If it is low then vitamin D supplementation(Bon DK 60K weekly with milk) can be taken.Avoid long continuous sitting on computer.Sit in a straight posture.Eat milk, fruits and green leafy vegetables daily. I think your query answered.Welcome to any follow up query."
},
{
"id": 84990,
"tgt": "Does vitiligo lead to patchy skin?",
"src": "Patient: sir i am having vitiligo from last 14 years but it was very minimal....three years back i hv taken homeopathic treatment which made it bad and it increased,,,then i stopped that medication and started some other allopathic but not done seriously nd left that also....now my patches r inc slowly now i m little bit worried ..is this possible to get rid of it...kindly suggest Doctor: Hi, Vitiligo is an autoimmune disease causing white patches on the skin.White patches occur due to the loss of melanin in the skin.There is no complete cure for vitiligo. Do not worry. The progression of the disease can be retarded to a certain extent by local application of immunosuppressant ointment and uv treatment.I would suggest you to continue treatment. Please consult your Dermatologist for appropriate treatment. Reduce consumption of seafood and sour food such as curd, tomato and tamarind. Always use sunscreen before stepping out. Hope I have answered your query. Let me know if I can assist you further. Take care Regards, Dr. Saranya Ramadoss, General & Family Physician"
},
{
"id": 166205,
"tgt": "What causes viral fever in a child?",
"src": "Patient: hello, my son is 7 months old n his fever started last night, i didnt chk his temp cz i thot hez teething so itmyt be dat but thn over da night it was not cuming down so in the morning i gav him calpol n he had slight runny nose and cough, but thn i took him to the docs n she sed its viral n keep him cool take his cloths off n let him cool dwn wid regular temp chk n calpol every 4 hrs, n av to take him 2mrw morning, is ther anythn 2 worry about n how long does viral fever last n wat causes it.... Doctor: viral fever is caused by rhinoviruses, u just need to keep temperature down by giving syp paracetamol 15mg/kg every 6-8 hrly. there is nothing to worry about that, it will subside in 3-4 days, just avoid exertion ."
},
{
"id": 222148,
"tgt": "Can loosing weight hinder pregnancy?",
"src": "Patient: Hello, me and my husband are wanting to get pregnant. I recently stopped taking my birth control medicine. I have no idea how long it is going to take for me to get pregnant. I am also unhappy with my weight at the moment. I was wondering if I start to lose weight while we are trying to get pregnant could this hurt my possible pregnancy in any way? Doctor: Hi,Weight loss before getting pregnant is advisable, as ideal body weight before pregnancy, leads to a uncomplicated pregnancy and delivery.Overweight and obese women are known to have more complications.So, it would be advisable for you to get to your ideal body weight, by diet and exercise. You could get in touch with a dietitian for a good diet plan. Also, you should try for a baby for a year's time, if unsuccessful contact a fertility specialist.Hope this helps.Regards."
},
{
"id": 162801,
"tgt": "How can a rash above the hips along with nose bleeds and vomiting be treated?",
"src": "Patient: My 2 year old daughter has a localized red rash going across the lower 1/4 of back starting just above her hips, at the top of her diaper line. She has started having nose bleeds yesterday about 4 of them, and now she just vomited her diner up about 4 hours after we ate. Any suggestions Doctor: Hello and Welcome to \u2018Ask A Doctor\u2019 service. I have reviewed your query and here is my advice. Skin conditions are best diagnosed only after seeing directly. I suggest you to upload photographs of the same on this website, so that I can guide you scientifically. Please revert back to me with images so that I can guide you better. You can approach me at the following link. Once the page opens there will be an option below my image as \u2013 ASK ME A QUESTION \u2013 click on it. Please find the link below - www.healthcaremagic.com/doctors/dr-sumanth-amperayani/67696 Regards."
},
{
"id": 46699,
"tgt": "How can creatinine levels be reduced?",
"src": "Patient: Hi, may I answer your health queries right now ? Please type your query here... My nephew is 23. His creatine reached 15.8 and his pressure was also very high 195/119. He was put on dialysis three times a week and was given amlopin 5 mg and concor 5mg twice a day. He has been on dialysis for 4 months and his creatine has come down to 8 and for the past month his pressure has been normal without taking pressure tablets. He is a 6 ft tall and weighs 100 kg. His is on a salt free diet and has limited protein intake. He mainly has vegetarian food and a hard boiled egg thrice a week. Can he reduce his dialysis to twice a week? How can he reduce his creatine further? Kindly give me your advice. Doctor: Hello and welcome to HCM.As an Urologist and transplant surgeon,i can understand your anxiety.Your nephew has kidney failure with H.T.and now on maintainence dialysis.He needs to undergo a kidney transplantation or continue with dialysis.As his kidneys have failed,till a transplant is done,a dialysis is only option.The kidney function is reflected in the creatinine level,so it's done regularly.If his dialysis is reduced to twice weekly,his general condition will worsen.If you've any doubts,send all the reports to me,as a direct question.Dr.Matthew J. Mangat."
},
{
"id": 190239,
"tgt": "Gap between the teeth. Treatment options? Need information on the safety of teeth bands",
"src": "Patient: Hi doctor, I am a 23 yrs. old girl. I have wide gaps in my 4 front teeth of the upper jaw and small gaps in the teeth of the lower jaw. Kindly suggest me if I can opt for invisible braces to close my gaps and what s the estimated duration and cost for the same. Also let me know if teeth bands are safe enough to close the teeth gaps. Doctor: Hello Welcome to HCM I would like to tell you that duration of orthodontic treatment depends on the case & the age of patient.The correct time of treatment is teenage period because that time we get good results as that time bone is growing & tooth alignment is easy. Still it takes around 6 months to 2 years depending on the case.For further treatment you need to consult your orthodontist for proper diagnosis & treatment planing. Take Care Regards Dr.Neha"
},
{
"id": 173422,
"tgt": "What causes small brown spots on back?",
"src": "Patient: My 8yearcold daughter overnight developed very small brown spots on her back ! About 3 to 4 small dots ! I am worried what are they ! First I thought they are brown sketch pen marks but they don t go with scrubbing ! She has no pain or itching ! Pls help ! Doctor: Hi...Thank you for consulting in Health Care magic. Skin conditions are best diagnosed only after seeing directly. I suggest you to upload photographs of the same on this website, so that I can guide you scientifically. Hope my answer was helpful for you. I am happy to help any time. Further clarifications and consultations on Health care magic are welcome. If you do not have any clarifications, you can close the discussion and rate the answer. Wish your kid good health.Dr. Sumanth MBBS., DCH., DNB (Paed).,"
},
{
"id": 154233,
"tgt": "What is the treatment for endometrial cancer?",
"src": "Patient: Hi, I had endometrial cancer. A high grade endometroid adenocarcinoma with squamous differerentiation. I had surgery - removal of my uterous, fallopian tubes, ovaries, some lymph nodes, and part of my ometum. I had superficial myometrial invasion as well as cervical stromal invasion- invasion of cervical connective tissue. The maximum invasion was less than a millimeter. My lymph nodes, ovaries, fallopian tubes, peritoneal fluid and omentum were negative for malignancy. I have had a cat scan and mriabout 6-7weeks after surgery which also was clear. I am very reluctant to do chemo (1st try had a reaction right away) and radiation due to the severe side effects. I have completely changed my diet, I am slightly overweight and losing weight now though as I am eating mostly vegetables. I am eating very healthy and am reasonably active. I feel really good - infact better than I have in a few years. Doctor: Hi, dearI have gone through your question. I can understand your concern. You had endometriod carcinoma with squamous differentiation. You also had invasion in myometrium and cervix. Treatment of choice is surgery followed by chemotherapy. You have hone through surgery. Now you should take chemotherapy. Take proper nutritional diet and healthy diet. Youshould also go for ultrasound abdomen every 3 to 6 months to detect early recurrence or spread. Consult your doctor and plan accordingly. Hope I have answered your question, if you have doubt then I will be happy to answer. Thanks for using health care magic. Wish you a very good health."
},
{
"id": 224507,
"tgt": "Can IUD cause delayed periods?",
"src": "Patient: hi,i am 29 years .i have 2 year daughter.last year i hav inserted copper T.and now facing the problem delayed periods. my last date was 29 july and still waiting for it . is this much period is as expected or is the case of failure of copper T.what should i do? Doctor: Hello,Copper T is not a hormone based device hence, it doesnt cause delayed periods. There is either a failure of the device or an unrelated hormone dysfunction. Do a pregnancy test if your period is delayed by more than a week. If its positive, see your doctor immediately. If its negative, your doctor can give you medicine to get your period. Plus the cause for the delay should be investigated."
},
{
"id": 196779,
"tgt": "Why do I have pain in testicles while ejaculating for the second time in a day?",
"src": "Patient: hey i am 17 years old and i sometimes have a inner feeling to produce a sperm. so tell me is there is any effect on my health or any side effects of it. also ejecting the sperm for the second time on the same day, i gets a pain in my testes so is it a negative sign for me?????? Doctor: Hi and welcome to Healthcaremagic. Thank you for your query. I am Dr. Rommstein, I understand your concerns and I will try to help you as much as I can.All these symptoms are suggestive of epididimitis which is inflammation of part of testicle and not uncommon in males. This is usually treated with 10 days of antibiotics, usually ciprofloxacin and there are no permanent consequances. MAsturbation and intercourse should be avoided till pain persist. Some more serious conditions such as tumors or torsion are rare but in doubtful cases you should do Doppler ultrasound or scintigraphy. In this particular case, I don\u2019t think this is required.I hope I have answered you query. If you have any further questions you can contact us in every time.Kindly regards. Wish you a good health."
},
{
"id": 191812,
"tgt": "What does a pain in lower-right flank indicate for a diabetic patient?",
"src": "Patient: have t2 diabetes w/ elevated blood sugar. Over past 4-5 weeks have developed a dull pain in my lower-right back. (Flank). Is this the start of my kidneys startingto fail, or is it a sign of something else? The last blood test I had about a month ago-everything came back within range. Please advise, K Doctor: Hi,Flank pain is a relatively common condition that can have a lot of possible etiologies and a number are commonly seen in urology. Anyway I want to say that this is not a sign of diabetic kidney disease because or diabetic nephropathy do not cause pain. So, don't worry. In diabetic nephropathy the filters of the kidneys, the glomeruli, become damaged. Because of this the kidneys 'leak' abnormal amounts of protein from the blood into the urine. This is the main sign that the kidney is starting to fail. Possible causes of your dull right flank pain can be urinary tract infections like Cistitis or pielonefritis or renal calculi.I suggest to consult an urologist to investigate about your flank pain (urine test, blood test, ultrasound etc) and if you find the cause ,you can have the right treatment.Hope I have clarified your doubts. If you have further queries, feel free and ask. Regards,Dr. Elona Xhardo"
},
{
"id": 144447,
"tgt": "What causes episodes of dizziness and panic attacks?",
"src": "Patient: Hi i am 35 female before 1 month i feel neck and shoulder pain and feel dizzy when get up or lay down i used serc suggested my doctor i felt better but after a week i have panic attack then i consult my psychiatry t i am taking eziday 50mg and cipralex 20mg and lyrica 75 i am feeling better but not very well my head is spinning while lay down and get up i have feeling of fear and panic every morning Doctor: HIWell come to HCMI really appreciate your concern, this is the psychological condition you have called functional condition try to come out of this, be positive, practical, bring back the confidence, no need to fear anything, you can even can do best without the helps of medicine and its needs your guts, keep the oral high change the view of thinking hope this information helps."
},
{
"id": 63282,
"tgt": "Suggest treatment for lump on clitoris",
"src": "Patient: Hi. I am a hispanic woman, age 20, 5 4 and right now(pregnant) I weigh 150 pounds. I am usually about 120 pounds. I am 36 weeks pregnant, and when I woke up this morning I found a lump in my clitoral hood on my right side of the clitoris. It does not hurt, isn t red or colored at all, and I can barely notice it without touching it. It is about the size of a marble, but it is oval. I don t know what to do, and need to know if this is something I should go to the hospital for right away! I have never seen or felt anything like this on my body before. Doctor: hi.it could be a cyst. it is best if you consult with a gynecologist to physically check on it. it is not an emergency, and out-patient consult would do. management will be directed accordingly.hope this helps.good day!!~dr.kaye"
},
{
"id": 202084,
"tgt": "Suggest treatment for the shrinking of pee hole",
"src": "Patient: hello doc...my pee hole is closing and it is driving me out of my mind...i feel like it will close any moment and I won't be able to release the urine...I am frinking out thinking that I will uriniate inside myselfe...I am 47 very healthy..exercise daily(weight lifter) eat well...my food is cooked with olive oil only...dailey cooked...I am 5'8 and 220 pounds of muscle...please help...I was told by friends that could be an infection that could be treated with cramberry juice or cramberry suplement...please reply Doctor: thank you for query.It may be due to phimosis.Please consult surgeon immediately for examination and confirmation.It may require simple procedure like slitting or circumcision.Clean the area with warm water twice daily.Dr Bharatesh"
},
{
"id": 19827,
"tgt": "What causes sudden feeling of squeezing in heart?",
"src": "Patient: My heart just started feeling Squeezed when I spoke to an attorney --because we are having depositions tomorrow about my mom being burned alive in a car from GM--I'm scared and I don't know what to do. I have so much office work to complete tonight before they get here. Doctor: Hi, So sorry to hear that. Sometimes, anxiety and tension do lead to such feelings and it tends to be short term. I would suggest you relax, take a deep breath and concentrate on your work. Once you feel comfortable and/or your work gets over, kindly take an appointment with your GP for a routine check up (EKG, physical examination, etc).However, if the chest pain is still present or is severe, you must visit an ER. Let me know if you have any further query or have other associated symptoms."
},
{
"id": 212693,
"tgt": "Feeling of bug bites. Mental illness?",
"src": "Patient: i have the feeling that bugs are biting me all the time have been to dr ad they state that it is all in my head. although i can feel when i lands on me, which is a very small black thing. looks like a hair after you get your hair cut. they are on my face ears private areas arms. if you can tweeze the fine black like thing out fast it will not grow. they then turn white, after awhile they are brown. they are in my mouth also and have rotted almost all my teeth since june of last year. i have been to dr s 9 times. Doctor: Hi there ~ You seem to be suffering from a form of somatoform illness. Psychoeducation about such illnesses is one way to treat them. You will find more information about your particular kind of illness on the national institute of health's website, an example weblink is posted below: http://www.nlm.nih.gov/medlineplus/ency/article/000955.htm I hope this helps. Take care and have a good day!"
},
{
"id": 84065,
"tgt": "Is there a difference between Stamlo and Stamlo Beta?",
"src": "Patient: Is there a difference between Stamlo and Stamlo Beta ? WHich one should be taken for hypertension. My BP is typically 135/90. I am a Male aged 41 in India. Reasonably active. Sarted Yoga 2 months ago. Also run jog 5km twice a week. (I had a few episodes of fast heart beat about a year ago. Went through the TMT/Stress echo/Halter/ECG etc - all was relatively normal. symptoms went away and have not recurred.) My question is more to do with BP but am just providing a background for you. Thank you. Doctor: Hi,Stamlo contains medicine amlodipine which is a calcium channel blocker commonly prescribed to treat high BP. Whereas stamlo-beta is a combination of amlodipine and a beta receptor blocker atenolol. Atenolol is helpful in controlling the BP as well as irregular or faster heartbeats hence your doctor has prescribed it. Continue taking it as prescribed.Hope I have answered your question. Let me know if I can assist you further. Regards, Dr. Mohammed Taher Ali, General & Family Physician"
},
{
"id": 176776,
"tgt": "What causes hard lump on neck in a 7 year old?",
"src": "Patient: A egg sized lump has suddenly appeared on the left side of my seven year old sons neck. It s firm to the touch, but he gets no pain from it. I ve googled up on it with plans to get him into the doctors on Monday since he feels no discomfort however most of the advise points to cancer causing me to get even more worried. I was just after some advise about wether this is most likely swollen glands or if it could be something more sinister. It s situated in the middle of his neck but leaning more towards his ear rather then his collarbone. It appeared within the space of a hour I would guess and it s not painful or tender, there s no rash or red skin around it. It s just a hard lump that I can get my fingers round. He was I ll five days in the last two weeks could this be a lasting effect of his illness??? Any advise would be appreciated thank you... Doctor: Hello. I just read through your question.it is exceedingly unlikely that the lump you felt was cancerous. Cancerous lumps do not appear that quickly. Additionally the location is very typical of a lymph node reacting to a recent illness, perhaps his recent viral illness. Though I do agree that it should be seen by your doctor, there is truly no need to worry."
},
{
"id": 45295,
"tgt": "I have TB in uterus. what are my chances of concieving in future ?",
"src": "Patient: hi my question is i have tb in uterus what are my chances of concieving in future my gynea has asked me to take akt4 i have started taking med from yesterday i had 2 abortions in past my age is 22 plzz tell me what are my chance of getting pregnent after the tb treatment Hi doctor , my question is this that i had 2 abortions in last 2 yrs before 3 months i wanted to concieve but was not able to my gynea did an endometrial biopsy to check the cause of infetility she found that i have tb in uterus she asked me to take AKT4 and said this treatment will continue for 9 moths plz tell me will i be able to get pregnent after the treatment should I take a second option or these medicines will effect my tubes ? Doctor: Hi, Welcome to Healthcare Magic Forum. Endometrial TB causes Infertility. You will have to AKT 4 tabs as prescribed by your Doctor and go for regular follow up every 15 days, to see how you are responding to the treatment. Don't worry, once the Infection subsides, you will have good chances of conceiving. Eat healthy fiber rich fresh fruits and vegetables and exercise regularly, this will improve your immunity and overall general health. Good Luck."
},
{
"id": 89762,
"tgt": "Suggest treatment for abdominal pain",
"src": "Patient: Hi, I have been having abdominal pain, bloating, pain over my bladder and lower right quadrant pain, nausea without vomiting. Had 2 positive blood and leukocytes found in urine. Had KUB and cat scan done. Both were negative. Still having the pain. What should I do? Doctor: HI.You have pain in bladder and lower right quadrant area with bloating, nausea. IF there is an infection either in the urinary tract and/ or the gastrointestinal tract, the treatment almost remains the same in the initial period. That is : antibiotics and supportive and symptomatic treatment. As KUB and CT scan are normal the mainstay remains the same. Go for the blood tests for indication of the infection by WBC count, urea , creatinine, sugar to rule out diabetes.Urine -routine, culture and sensitivityStool - routine and microscopy.These findings will guide for the further treatment."
},
{
"id": 121540,
"tgt": "Suggest treatment for inflammation in knee with ASO of 390",
"src": "Patient: My daughter who had inflammation in the knee was advised for ASO test in Oct On Oct 30 2011 ASO was 90 After 2 weeks ASO was 370 Today ASO is 390 Last week she had throat infection and was on antibiotics for a week. From October onwards she has completed 2 courses of antibiotics What will be the next treatment for her? (All other tests are normal) Doctor: Hello, It can be an infective arthritis. Antibiotics will be the treatment of choice and most cases respond well the antibiotics. Hope I have answered your query. Let me know if I can assist you further. Take care Regards, Dr Shinas Hussain, General & Family Physician"
},
{
"id": 210573,
"tgt": "Should I change from Citalopram to Xanax for more relaxation?",
"src": "Patient: I am on Citalopram 30mg and have been for 10 years. I have anxiety. I have always been a very active person. I want to have the energy during the day for exercising but want to sleep so I have also turned to wine to go to relax, sleep at night and stay asleep. I am now drinking up to 2 bottles a night for the last 8-10 years. I want to stop drinking on my own and am on my second night of no wine. My question is do you think I need to stay on the citalopram or change to something like Xanax because I m having trouble relaxing and staying asleep at night? I don t know if I have been on the Citalopram too long. Doctor: Hello,Thanks for choosing health care magic for posting your query.I have gone through your question in detail and I can understand what you are going through.Citalopram and xanax are totally different groups of drugs. Citalopram is essential for treatment of anxiety where as xanax can be complimented for further symptomatic relief and sleep disturbances. You may take them together. Hope I am able to answer your concerns.If you have any further query, I would be glad to help you.In future if you wish to contact me directly, you can use the below mentioned link:bit.ly/dr-srikanth-reddy\u00a0\u00a0\u00a0\u00a0\u00a0\u00a0\u00a0\u00a0\u00a0\u00a0\u00a0\u00a0\u00a0\u00a0\u00a0\u00a0\u00a0\u00a0\u00a0\u00a0\u00a0\u00a0\u00a0\u00a0\u00a0\u00a0\u00a0\u00a0\u00a0\u00a0\u00a0\u00a0\u00a0\u00a0\u00a0\u00a0\u00a0\u00a0\u00a0\u00a0"
},
{
"id": 39063,
"tgt": "Suggest treatment for bronchitis",
"src": "Patient: hi my husband keeps having fever for 2 weeks now., It will usually be fine with in 24 hours then come back after 2-5 days accompanied with colds and cough. his temp will be anywhere from 38-39.1. He was hospitalized last week but was just diagnosed with bronchitis. what to do? Doctor: Hello,Welcome to HCM,Bronchitis is a condition in which the bronchial tubes connecting airway to the lungs become inflamed. Various agents like viral infection and bacterial infections may lead to mild inflammation of the bronchial tubes. The patients will have symptoms like edema of the airway, productive cough and breathing difficulty. For patients with bronchitis following measures are advised.1.Bronchodilators -i n the form of inhalers or pills.2.Corticosteroids- Steroids should be given in the form of aerosol sprays. They reduce inflammation and swelling of the lung tissue.3.Antibiotics like Tab Augmentin, 625 mg, in case of bronchitis due to a respiratory infection.4.Oxygen therapy is advised where breathing is very difficult.The aforementioned measures will help to improve your husbands condition.Thank you."
},
{
"id": 46385,
"tgt": "Suggest treatment for kidney stone",
"src": "Patient: good evening sir, i am working in kuwait,i would like to know about the problem persist in kidney of my wife.my wife pregnancy time there was a problem found in kidney.that time doctors told us no treatment needed after delivery we can check it.now after 45days delivery we scan kidney and showing oneside a little stone and other side building stone also..iam fearing that it will be a problem in future,what can i do..? Doctor: Hi and welcome to HCM.Most small stones in patients with relatively mild hydronephrosis can be treated with observation and acetaminophen. More serious cases with intractable pain may require drainage with a stent or percutaneous nephrostomy. Treatment of nephrolithiasis involves emergency management of renal (ureteral) colic, including surgical interventions where indicated, and medical therapy for stone disease. In emergency settings where concern exists about possible renal failure, the focus of treatment should be on correcting dehydration, treating urinary infections, preventing scarring, identifying patients with a solitary functional kidney,.Kindly regards. Wish you a good health."
},
{
"id": 92707,
"tgt": "Stomach and back pain, heard there is a muscle that goes from the stomach to the spine that may cause this pain",
"src": "Patient: My niece has been in and out of the hospital for several months now with severe stomach and back pain. When she has these episodes, she cannot keep any food or liquids down. After a few days, the pain will subside, she'll eat and be right back in the ER. She has been to specialists and spent a lot of time in the hospital. I've heard there is a muscle that goes from the stomach to the spine that may cause this pain. What might that be? Doctor: one of the food proteins is reacting with body protein it stars reacting suddenly may be milk wheat potatoor othermajor food yu takeget allergy doctor advice who may get blood serum tests for specific antibodies for milk wheat potato orother food get the food withdrawn you can be bene fitteted"
},
{
"id": 194500,
"tgt": "Suggest treatment for erection problems",
"src": "Patient: I am 33 years old recently married. My penis is not erecting while having sex. Even if it gets erection it is for very short time and while entering in to my partner's part the erection gets lower. So my sperms are not getting released. Please advice. Doctor: Hello, Lack of erection and slackening of erection during sex are different issues. During sex, if you divert, slackening can happen and there is nothing wrong in it. Hope I have answered your query. Let me know if I can assist you further. Regards, Dr. K. V. Anand, Psychologist"
},
{
"id": 199250,
"tgt": "What causes losing erection during sex and difficulty in getting erection?",
"src": "Patient: HelloI have a lack of libido and don't get erection easily. And I tend to lose it during sex. The other thing is I noticed that I started to lose sensation in my penis. This has been on going for almost a month now. Every time I get erection I feel bloated and big pressure in my stomach that I feel pressing on my abdomen making me lose the erection. I am constipated and bloated all the time. The problem started with a pain in my lower back - which I still have but its subsiding, and pain in my testicles that keeps shifting from one side to another ( I still have). I saw a urologist, he performed ultrasound on my testicles and said everything is alright. He said It's probably a pinched nerve in my back. I am taking 800 mg ibuprofen, and I have black stool. My question is: 1- Is the erection problem related to the pinched nerve in my back because I might have a Pudendal Nerve Entrapment as I read online?2- or is it because I might have gastrointestinal bleeding because of taking 800 mg ibuprofen, which I am suspecting, but no evidence except constipation, bloating , and black stool?3- or it other problems related to the prostate or kidney stones? Doctor: DearWe understand your concernsI went through your details. I think you are \"thinking\" too much. The three possibilities you are mentioning are far possibilities. In the first hand, you just are worried and anxious about your performance. Anxiety is showing in the stomach sensation you are mentioning. I suggest you to consult a psychologist for counseling and related therapies. You should be alright.If you still need my assistance in this regard, please use this link. http://goo.gl/aYW2pR. Please remember to describe the whole problem with full detail.Hope this answers your query. Available for further clarifications.Good luck."
},
{
"id": 100480,
"tgt": "Any suggestion for unable to smell/taste after having sinus infection and bronchitis?",
"src": "Patient: I have not been able to smell or taste for about 4 months after a bad sinus infection and bronchitis. I saw doctor at that time and took antibiotics and steroid shot. I take Allegra D on a regular basis. Several years ago I had surgery on my sinuses. I often have sinus infections and cannot smell or taste for a few weeks, but have never gone this long; also loosing weight. What would you recommend? Doctor: Thanks for your query...after reading the history given by you i think in my personal capacity that you might be having sino-nasal polyps...taking allegra-d contonuously is not the solution.. you should visit an ent doctor and get your nasal endoscopy done and a CT- PNS should be done to establish a diagnosis and accordingly you'll be treated.I hope you got the answer."
},
{
"id": 119999,
"tgt": "How long should i rest after surgery for rotator cuff?",
"src": "Patient: I had shoulder surgery for bone spurs and torn rotator cuff in August. I am in more pain now than before the surgery. I am 53, female and otherwise healthy. I am a postal clerk in a very busy office, constant reaching to put up mail, pushing and pulling very heavy mail cages, lifting bags of parcels approx. 100 lbs, etc etc. How likely am I to be able to return to work and how long before doing so? I have severe shoulder pain, no backward range, can t sleep, actually can t do anythng more than about an hour before having to just rest. I only have 2 more years to be eligible for retirement. Would disability be possible? Doctor: Hello,Recovery for the surgery of rotator cuff can take 4 to 6 months, depending on the size of the tear and other factors. You may have to wear a sling for 4 to 6 weeks after\u00a0surgery. During this period you should rest. Then physical therapy can help you\u00a0regain\u00a0the motion and strength of your\u00a0shoulder.Hope I have answered your question. Let me know if I can assist you further. Regards, Dr. Dorina Gurabardhi, General & Family Physician"
},
{
"id": 25649,
"tgt": "Does Atenolol 50 mg intake cause heartburn?",
"src": "Patient: Does Atenolol 50 mg cause Heartburn. Also when I eat I feel that the food goes down with difficulty and after it has gone down get a sensation as though there is a lump in my throat. I drink water it helps to relieve the discomfort. I would appreciate an explanation Doctor: Thanks for your question on Health Care Magic. I can understand your concern. No, Atenolol does not cause heart burn or lumpy feeling in throat or difficulty in swallowing. In my opinion, you should consult gastroenterologist and get done clinical examination and upper GI (gastrointestinal) scopy to find out the cause for these symptoms because Atenolol is not a cause for these. Hope I have solved your query. I will be happy to help you further. Wish you good health. Thanks."
},
{
"id": 78751,
"tgt": "What causes severe pain in the chest while breathing, dizziness and light headedness?",
"src": "Patient: 23 year old, healthy female.. Last night began having extremely sharp pains in left side of chest, almost closer to the center of my chest. The pain came every time I tried to breathe it and was extremely sharp and painful and I could not catch my breath and felt very dizzy and light headed. Sat down and drank some water.. Tried to breathe in deeply again and the sharp pain was still there and continued every time I took a deep breath. During that time my chest felt extremely tight and almost like I could only breathe in half way and when I tried to take a deep breath I had to really pull in hard and that s when the pain came. I ve been tested for asthma and do not have it. Since those pains started last night the chest tightness and not being able to breathe in fully without the pain have not gone away.. And since I woke up this morning there has been a constant throbbing in that left chest almost center area and it has not gone away and when I touch that are where the throbbing is, it s very sensitive. Any ideas? Thank you. Doctor: Thanks for your question on Health Care Magic. I can understand your concern. It would be advisable for you to get a chest x ray done. If its normal and your lung function test is also normal, then it could be severe GERD. Which needs treatment.Don't worry, you will be alright. Hope I have solved your query. Wish you good health. Thanks"
},
{
"id": 57296,
"tgt": "Swelling in hands, feet, worse on lying down. Taking melodican. Gall bladder removed. Treatment?",
"src": "Patient: Hi I am a 39 yr old female. Only problems I have had recently was gallbladder removed. About 2 months ago. An now for the last month I m having bad swelling in hands an feet . Worse on right side an worse when I lay down. Mornings are horrible been the doc he looks at them an an rights me a script called Melodican. Been takin them for 3 weeks. Pls help me. It hurts so bad.. Thank u Doctor: I think you might mean that your doctor is giving you Meloxicam, an anti-inflammation medicine that treats pain as well as other signs of inflammation like swelling and redness. In any case, swelling in the hands and feet joints that is worse in the morning in a woman of your age is likely to be an inflammatory arthritis like Rheumatoid arthritis or lupus but could also be any number of other diseases - the list is quite long and it can take some skill to sort this out. If the symptoms are no better after three weeks you definitely need to see a doctor. They will ask about additional complaints - heart lungs, abdomen, lymph nodes, rashes, eye problems, etc... and will probably order blood and urine tests. The doctor may also order x-rays of the inflamed joints.This could be a serious illness and this warrants immediate attention by the family doctor or by a specialist rheumatologist."
},
{
"id": 213041,
"tgt": "Pain behind head, pressure, dizziness on lying down, weird feeling in right foot, going up to chest, snoring problem. Treatment?",
"src": "Patient: hi! this is the first time i ask a question to a doctor by internet but anyway.. uhm in the past few days I ve been getting pains on the right side, back of my head. its like a pressure pain its not strong but i feel it there. when i lay down to go to bed i get very dizzy and get strange feelings on my right foot and up to my chest .. its very strange... it has happen 2 times now that when ever i get that feeling i fall asleep but am still awake. I m just snoring really loud but I m not actually sleeping i get back up and am super dizzy. do you know what it can be? i do have a stressfull job i saw a doctor and he gave me 2 types of pills for it and told me its because i bite down when ever i get stressed. Doctor: yes your doctor is right.. your symptoms are due to stress. consult a psychiatrist. do aerobic exercises.. reduce your stress level.."
},
{
"id": 19032,
"tgt": "Suggest medication for blood pressure that is safe during pregnancy",
"src": "Patient: I am a 29 year old female and have high blood pressure.I am concidering getting pregant,I am, wondering if this med is best for me , my doctor says to take the methyldopa 250mg twice a day.Is there a another blood pressure med that I can take just once a day and be safe for when I am pregant? Doctor: Hello,Methyldopa is the best choice because proven that doesn't harm the fetus. I suggest you use it.Hope I have answered your query. Let me know if I can assist you further.Regards,Dr. Anila Skenderi"
},
{
"id": 59720,
"tgt": "Recovered from jaundice. Is it safe to for pregnancy now?",
"src": "Patient: Hi, I m Sahni from India. I suffered with Jaundice in the month of July and it kept me on bed for almost two months. I m feeling all well sine past couple of months now and have resumed my oral workout routine as well. I m 26 years old married man and planning for our first baby. Could yo advise me if it s advisable to go ahead right now or is there any prescribed gap that s must keep while proceeding with our family planning? Shall appreciate your answer, Thanks Doctor: Hi, Welcome to HCM You have suffered jaundice in the month of July but two month is long tome to recover . Jaundice is just symptom which can be due to various causes most common is viral hepatitis. If it was viral hepatitis it is important to know which type A,B,C& E are common one as Hepatitis B&C can cause chronic hepatitis and can be transmitted sexually also so it is not only time gap it is type which is important. if it was B or C then you should consult a Gastroenterologist and test for viral markers is required and he will guide you depending on result. Even vaccine is there for Hepatitis B . If it was A or E and you have completely recovered there should not be any problem in going ahead with family. Hope this information is useful for you. Take care Good Luck."
},
{
"id": 206142,
"tgt": "Suggest treatment for stress and depression",
"src": "Patient: I think I have a medical problem.. my boyfriend just thinks I'm crazy though. Anyways I'm 18 years old, I weigh 104 lbs and I'm 5'1. I had a heart murmur fixed when I was 6 months old, reoccurring Kidney and UTI. But other than that I am pretty healthy. I know that depression runs in my mothers side of the family, but I don't know if that is something that can be a genetic trait. But some days I wake up so depressed I don't want to eat or get out of bed. Other days I feel like a maniac, i feel like i am overly happy for no reason. About 90% of the time I find myself crying with no explanation. I have recently lost quite a bit of weight and yet I still wake up and look in the mirror and think i am fat. I do get migraines pretty frequently upon waking up and I have been seen for those with no diagnosis. I'm wondering if this is even a medical problem and if my craziness and migranes might be coexisting. Thank you for your time. Doctor: DearWe understand your concernsI went through your details. I suggest you not to worry much. Anxiety and Depression is known to create physical fatigue and mental fatigue. You are anxious about something which is giving you these series of symptoms. Migrains could be related to anxiety and stress. Disappointment in unexpected life happenings can cause anxiety and depressive symptoms you are experiencing. Understanding those disappointments may be of help. Researches have shown that medicines alone cannot cure mental disorders. Psychotherapy to have life style changes, Yoga and meditationn to streamline thought process and metabolism can be practiced for easy and permanent cure. Contact a psychologist for further information.If you still need my help, please describe the whole problem in detail and post a direct question to me. I shall definitely help you with psychotherapy techniques to over come your problems.Hope this answers your query. Available for further clarifications.Good luck."
},
{
"id": 6359,
"tgt": "TTC with PCOS",
"src": "Patient: HI THERE .. ITS BEEN A YEAR SINCE I POSTED MY LAST QUESTION HERE.. I DIAGNOSED PCOS THIS YEAR AND I AM SUFFERING WITH HAVING A LONG TIME VAGINAL BLEEDIN (MENSE), I AM NOW TAKING METFORMIN 500MG 3X A DAY, AND WILL BE TAKING BY NEXT WEEK OF DUPHASTON TABLET . I JUST WANT TO ASK IF I COULD BE PREGNANT OF THE MEDICINES IM TAKING .. AND IF THESE CAN HELP ME REGULATE MY CYCLE AND HELP ME TO OVULATE.. PLEASE HELP ME I AM ESPERATE TO HAVE A BABY AGAIN AFTER 2 M/C LAST YEAR.. THANK YOU Doctor: Hi dear, 1)If you miss your periods this time....get a urine pregnancy test done to confirm pregnancy. 2)If you do not conceive in this cycle ..you need to start ovulation inducing drugs like clomiphene citrate from the second day of your cycle because in PCOS there are anovulatory cycles( i.e.No ovulation). 3)Consult your gynaecologist for follicular studies and further management. 4)you can continue taking Metformin. take care"
},
{
"id": 151221,
"tgt": "Numbness in tip of fingers after sleeping on floor. Nerve damage?",
"src": "Patient: After sleeping on the floor, I woke up to find that the tip of my fingers were numb. The numbness and tingling effect progressed trough the tips of my left hand fingers and became worse after doctors tried to give me a drip through my left hand. I pain gradually cept up my lower and upper arms and my muscles became weak. I feel little pain around my neck but the nerve pain had trasferred to the left side, my waist and down to my leg AND FOOT. cAN YOU PLEASE HELP AS i HAVE DIFFICULTY SLEEPING AND FEEL WEAK WHEN I EXERT MYSELF. There are one or two other possibilities for my possible nereve damage : Excessive inhalation of whitewash paint Inhalation of traces of argon gas over a couple of weeks Intestinal problems or diabetes Dejean Johnson Doctor: Dear Johnson, . Abnormal position during sleep can cause compression of neurovascular structures resulting in tingling, discomfort and pain. The symptoms can last for few minutes to days depending on the amount of nerve damage. These phenomena can explain the symptoms you had in the upper limbs, while I am unable to explain the lower limb weakness too. If a definite weakness is documented by your doctor, then you would need to get a CT / MRI scan of the brain and rule out intracranial pathology such as ischemia / demyelination. A specific treatment can be planned based on the results of scan. If no significant / clinically co-relating pathology is found, then we may attribute these symptoms to anxiety and stress. Hence a scan is mandatory. Discuss with your treating doctor. Hope this information suffices. Regards"
},
{
"id": 172,
"tgt": "Can stimulating clitoris after ejaculating on hand cause pregnancy?",
"src": "Patient: The other day my gf pleasured me and I ejaculated all over myself. There was alot on my left hand and my right had none. I went to the bathroom and washed my hands with soap. After my hands were dry she wanted me to pleasure her with my hands. I used my right hand to pleasure inside her and my left to stimulate the clitoris. We thought nothing of it but now we worried that some of my sperm may have gotten in. How high is the chance if her getting pregnant? Her period is in 2 weeks. Doctor: Hello,See sperm need liquid media for transport and sperm dies in contact with solid media. Sperm should travel vagina cervix till tubes, in your case unlikely. Ideal would be getting the cause of backache and treat it permanently rather than taking empirically tablets for pain relief.Hope I have answered your query. Let me know if I can assist you further.Regards,Dr. Sheetal Agarwal"
},
{
"id": 225787,
"tgt": "Had unsafe sex. Been bleeding. Was on microgynon, taking cerazette. What is going on?",
"src": "Patient: I was on microgynon for many years and then after my last period finished last Friday I started taking cerazette on Saturday (the day I would have started my microgynon again). Two problems, I had sex with my boyfriend Saturday night and the condom split so I had to take the morning after pill on Sunday as I know you aren't protected for the first 7 days of taking this pill so already my body feels a bit mucked up.However, I really want to go back onto microgynon (having only been taking cerazette for 4 days) as I've heard so many bad stories about bleeding for long periods of time and I've got the opportunity to go to Dubai for a week at the end of April and now I'm worried that cerazette will muck up my cycle and I could be bleeding when I go away. Can you please advise what it is the best thing to do? I'm 40 years old. Doctor: Hi,As you had intercourse soon after your period, you were still in the safe period and there is no cause to worry; moreover, you have no risk at all as you took the emergency pill. Please remember that the emergency pill contains excess hormone and could cause some amount of breakthrough bleeding and it is likely to disturb your next cycle. In the current situation, it would be advisable not to switch the birth control pill you have already started taking as changing the pill in the middle of a cycle is not going to give you any protection for that cycle and is likely to cause menstrual disturbances. You may take the advice of your consultant for further assistance. Hope this helps. Take care."
},
{
"id": 201564,
"tgt": "What is the medication for sperm count?",
"src": "Patient: Hi, my name is Waseem and i just went for another semen analysis. i used to have only 15% slightly active sperm and the last report showed them all dead. the lab guy said i have 10 - 12 hp of pus cells in my semen. is there a cure and how long would it take me to be able to get her pregnant ? best regards, Waseem Doctor: Good Day and thank you for being with Healthcare Magic!A proper physical examination is needed to assess infertility problems in males. The testis needs to be palpated to check for size and consistency. A rectal examination is also needed to check for prostate cyst. The spermatic cord is also needed to rule out varicocoele which is the most common cause of infertility in males. Urinalysis is also needed to rule out UTI. Once the etiology is identified then the proper medicine and intervention will be prescribed to help you. I hope I have succeeded in providing the information you were looking for. Please feel free to write back to me for any further clarifications at: http://www.HealthcareMagic.com/doctors/dr-manuel-c-see-iv/66014 I would gladly help you. Best wishes."
},
{
"id": 324,
"tgt": "Suggest healthy diet during conception",
"src": "Patient: Hello, I am swetha 26 yrs old my husband and i are planning for a baby i am 4.11 height my weight is 54kg i feel u am obese i have a bulged tummy can i you suggest me what kind of diet should i follow or any exercises before i get pregnant or any sites that u can suggest me plz. Doctor: protein diet...like soybean paneer eggs. ..no carbohydrates. ..no fat....so dat body's carbohydrates n fat will be utilized"
},
{
"id": 141886,
"tgt": "What causes dizziness and severe headache?",
"src": "Patient: Today at work i got really dizzy n sick feeling to my stomach right after i started feeling dizz n night before i had a bad headache n again today fee hours after getting dizzy i have another really bad headache but it really was a weird feeling today when i got dizzy n it lasted for avout two hours today so could this b a sign of sumten serious going on with .me because honestly it made me nervous i havent ever felt that way before i yes i have been under alot of stress lately but im kinda used to alot of stress but im 42 n i work on ladders every day so getting dizzy could indanger my life Doctor: Hello!My name is Dr. Aida and I am glad to attend you on Healthcaremagic!Your symptoms could be related to a viral infection. For this reason, I would recommend having some rest and taking painkillers (acetaminophen or ibuprofen) and an antiacid or PPI (omeprazole, esomeprazole, etc..). If your symptoms persist in the next days, I would recommend consulting with your GP for a physical check up and some blood lab tests (complete blood count, PCR, sedimentation rate, blood electrolytes). Hope you will find this answer helpful!Best regards!"
},
{
"id": 162209,
"tgt": "What causes child to be born with microcephaly?",
"src": "Patient: Sarah, age 14 months, was born with microcephaly, small head (cranium) size and is severely retarded. Chromosome analysis indicates that she lacks a piece of one of her number 13 chromosomes. Her parents come in to have their chromosomes analyzed to see whether one of them may have been a carrier for a translocation that could result in children affected like Sarah or children with other chromosomal, anomalies. Doctor: Hi, I understand your concern but that specific genetic anomaly which you are quoting itself can be a risk factor for microcephaly, please. So it is appropriate that both the parents need to be tested for genetic analysis if either one of them is the carrier to predict the future pregnancy risk. Hope I have answered your query. Let me know if I can assist you further. Regards, Dr. Sumanth Amperayani, Pediatrician, Pulmonology"
},
{
"id": 148512,
"tgt": "Is back surgery required for degenerative disc disease?",
"src": "Patient: I have had severe back pain for many years and my recent MRI report reveled degenerative disc disease and L3- L4- L5 L5-S1 diffuse disc bulging and Spinal Stenosis moderate central spinal canal narrowing at L-4 and L-5 and ligamentum flavum hypertrophy and focal disc herniation, can you explain this to me and would I be a candidate for back surgery? Doctor: Hello Thanks for writing to HCM According to reports you have degenerative disc prolapse of spine in lumbar region.1.You have diffuse disc bulge at L3-L4-L5 and S1 level and herniated disc is causing moderate stenosis of central spinal canal at L4-5 level.This is causing pain in your case.There is also ligamentum flavum hypertrophy and focal disc herniation.Degenerative changes occurs due to aging and development of weakness in annulus fibrosis which allows herniation of disc.Treatment for pain includes physiotherapy or surgery. Effectiveness of both modalities are same.So,physiotherpay should be tried first. It causes strengthening of Para spinal muscles and it prevents further changes. Surgery should be the last option. There are also medicines like capsule pregabalin available for pain relief. Other pain relieving modalities are Ozone ablation,Radiofrequency ablation,percutaneous disectomy etc.Treatment can be planned after consultation with neurosurgeon.You should also modify your life style like don't abruptly bend forward and avoid standing for long time.Don't lift heavy objects.Get well soon. Take Care Dr.Indu Bhushan"
},
{
"id": 100713,
"tgt": "Is esiflo inhaler effective for asthma?",
"src": "Patient: Hi.I m 23 yr old student.I stay in mumbai.I study in bangalore.In bangalore i got weezin problem wen i come back to my native place for vacations I dnt have any of this problem...i consult doctor he discover that I have asthma problem by checkin blood report and x-ray of my chest...he gave me esiflo inhaler...I would like to know this inhaler will cure my asthma and it is addicted?...and is there any side effects?..plz reply me as soon as possible...:( Doctor: Hello.Thank you for asking at HCM.I went through your history and would like to make suggestions for you as follows:1. Inhalers and most of the medicines do not \"cure\" asthma. They control symptoms of allergies & asthma.2. Depending upon your severity and frequency of symptoms, you may need \"controller\" type of inhaler such as Esiflo as your doctor has prescribed you. I would suggest you to take it regularly for at least 2-3 months and then continue/discontinue as per your doctor's guidance only.3. Inhalers do not cause addiction. It is a great myth. Only thing is that as asthma is a chronic disorder, its symptoms return on stopping inhaler. So patients feel they are \"addicted\". It's not an addiction, it's a requirement for the patient.4. Esiflo as two contents: Salmeterol and Fluticasone (inhaled corticosteroid). Side effects of salmeterol are tremors, increased heart beats, feeling of heart beats, ets. Side effects of Fluticasone are throat irritation, change in voice, throat infection with long term use etc.Above side effects can be minimized by proper technique of using inhaler, using a spacer device along with inhaler and rinsing mouth after use of inhaler.5. Were I treating you, I would add regular montelukast in your regular treatment. It will help allergy-asthma symptoms and will reduce your requirement of inhalers.6. I would also suggest you allergy testing which will help you identify the substance causing allergies-asthma to you and also to know measures to avoid them.Based on results, an Allergist-Immunologist may prescribe you allergen specific immunotherapy which works on immune system to reduce allergy symptoms on long term basis.7. In general, I would suggest you to avoid exposure to dusts, smokes and air pollution as much as possible.8. Regular breathing exercises & healthy diet rich in vitamins & minerals (adequate green leafy vegetables, fruits, sprouts, etc) will also help you in a long run by improving your lung capacity & immunity respectively.Hope above suggestions will be helpful to you.Should you have any further query, please feel free to ask at HCM.Wish you the best of the health.Thank you & Regards."
},
{
"id": 41546,
"tgt": "How to get pregnant while suffering from high prolactin level?",
"src": "Patient: Have been trying to be pregnant for a year now, at 6months of that yr, i did hormonal profile to know the problem. It was discovered that i have elevated PROLACTIN 39. Some drugs were administered to me which include PALODEL TAB, folic acid, vit c etc for 4 months again, yet no result. i went for hormonal test again, at this time the PROLACTIN IS 36 and FOLLICLES STIMULATIN HORMONE 21. what should i do or take to bring down this figures so that i can be pregnant on time? thx. Doctor: Hi It seems you are suffering from high prolactin levels and your doctor has prescribes Parodel for the same. Firstly not all patients respond to the same dose of the drug. In my practise I initially prescribe the lowest dose ,check prolactin levels and if they haven't come down I increase the dose. Secondly your FSH levels are elevated too. You haven't mentioned your age which would have been helpful. To know the reason behind your elevated FSH you should get other hormonal tests like LH, testosterone, estradiol done as well.Let me know if you have any other queries.Thanks"
},
{
"id": 96567,
"tgt": "What causes numbness in the head after a head injury?",
"src": "Patient: Hi. I was in an car accident on 1-1-2017, of all days! First time going out. The car tire slipped on a patch of ice and the car spun then slammed into the rails. I was on the passenger side but I hit my head on the window. My head has been feeling strange in that one spot. I was told by my Dr. that I should be okay and the numb feeling will eventually got away within 2 to 3 months. It's been longer than that now. What does this sound like to you? Doctor: Hi, thanks for asking from HCM.I can understand your concern. The numbness could be due to nerve injury or concussion. It will subside on its own. If no relief at all, you should- Take tablet Pregabaline after prescription- take nerve specific vitamin like B12, folic acid, pantothenic acid.Hope it will help. Thanks. Take care."
},
{
"id": 41078,
"tgt": "Suggest treatment for bulky ovary",
"src": "Patient: I am 25 years old girl, my height is 5.4 feet and my weight is 55 kgs. my right ovary was removed when i was 10 years of age due to some complications. my periods have never been regular but this time it has got delayed by more than three months. My left ovary is said to be bulky measuring 34*32*28* mm ( vol- 15cc) and it shows 5-6 follicles of size 4-9 MM. Endometrial stripe is shown triple line and measures 13 mm in total thickness according to the sonography report. I always had dense facial hairs as a result of all this. I want to know I'll ever convieve and is all this irregularity curable. Doctor: Hello Your symptoms and ultrasound findings may suggests polycystic ovarian disease(PCOD).PCOD has typical appearance in USG and it is indicated by increase ovarian size,hyperechoic stroma and necklace appearance of cysts arranged in periphery.PCOD should be confirmed by clinical correlation and biochemical markers like hormonal assay for PCOD.PCOD is a curable disease and mostly it is treated by medicines.Surgical intervention is required in very few cases.Hope I have answered your query.Take Care Dr.Indu Bhushan"
},
{
"id": 98159,
"tgt": "What is the cause of increase in TSH post intake of homeopathic medicines ?",
"src": "Patient: Hi Doc , I have thyoid since 2006 , with allopathy the T3,T4 & TSH were normal also Periods were on time sometime very perfect of 30 days..... My TSH levels have suddenly raised from 6.19 to 12.75, after I opted for a Homeopathy treatment , i have been treated with homeopathy along with allopathic medicines from past 4 months, now i have noticed a DELAY in my periods,since 2 months when I questioned the same to my doctor, she said it will regularise and TSH will be normal... what do i do now.. should I stop the homeopathy and just continue with allopathy... pls help ASAP Doctor: minimum thyroid treatment i.e carbimazole and propylthyouracil to be taken for 18months .......................and no other alternative is required with these allopathic medications...............TSH levels are showing hypothyroid state from hyperthyroid state which was present............indicating excessive supprssion of throid levels and thyroid is one of the important hormone that cause changes in menstrual cycle"
},
{
"id": 210515,
"tgt": "What is the cause of low IQ and what is the solution for it?",
"src": "Patient: My name is Barbara I'm 27 years old, I used to be brilliant through out my childhood but it all changed when I was 13 since then I have been struggle with my academic and it pains me alot that at my age I can't really read or write.I have a very low IQ,what's the cause or solution to my problem?thank u for ur time Doctor: HiThanks for using healthcare magicLow IQ happen due to multiple reason like low birth weight, intra-natal complication, oxygen deficiency immediately after birth, any vitamin deficiency, genetic reason etc. There is no solution of low IQ. Whatever IQ you have, that would stay for rest of life. Better to plan your life according to your capability.Thanks"
},
{
"id": 121726,
"tgt": "How to cure Lymphedema of the right arm?",
"src": "Patient: I developed right arm lymphedema after having a stem cell transplant for leukemia, possibily from central line damage? my right breast is also affected i am worried however, could it be inflamatory breast cnacer? my breast does exibit all of the symptoms listed but many are the same for lymphedema i guess what i am wondering is can the ibc cause right arm lymphedema? Doctor: Hello,Considering your past medical history, it is necessary investigate your breasts too. For this reason, I would recommend performing a mamography and a breast ultrasound. Nevertheless, you should know that arm lymphedema is quite common in this clinical situation. Hope I have answered your query. Let me know if I can assist you further. Regards, Dr. Ilir Sharka, Cardiologist"
},
{
"id": 67239,
"tgt": "What causes painless lump behind each ear?",
"src": "Patient: Hi i have a little lump behind each ear where the lobe connects to my neck. If i squeeze them a think white substance (it doesn t smell) comes out. They don t hurt or anything they are just there and are quite small. I m just curious as to what thry could be? Mary. Doctor: Hi! Good evening. I am Dr Shareef answering your query. Although it needs a personal clinical examination of the lumps, from the history of bilaterality, and whitish substance coming out on sqeezing, these seem to be some kind of a benign cyst like a dermoid cyst or a sebaceous cyst. To be certain, it needs to be examined by a general surgeon who might advise you for either a FNAC (fine needle aspiration cytology) or an excision biopsy for a definite diagnosis, the excision being of therapeutic value as well.Till then, I would advise you not to sqeeze these to avoid any kind of ensuing secondary infection.I hope this information would help you in discussing with your family physician/treating doctor in further management of your problem. Please do not hesitate to ask in case of any further doubts.Thanks for choosing health care magic to clear doubts on your health problems. I wish you an early recovery. Dr Shareef."
},
{
"id": 182162,
"tgt": "What causes painful bleeding mouth with sores?",
"src": "Patient: I am 67 female with a DX of COPD four years ago, also adult onset of allegic asthma. In the last four years I have had chronic thrush and nothing seems to clear it up. I am in mouth pain continuously with sores and bleeding. None of the thrush medicines work. I have now lost my taste. and one sore was so large on my tongue I had to have a biopsy done, it was negitive. HELP I an broke and my husband just had a stroke and last month I had a heart attack.Like everyone else thanks for nothing. Doctor: Hello, Thanks for consulting HCMRead your query as you have painful bleeding mouth with sores dont be worried so much I will suggest you to consult oral sugeon for examination of sore on tongue .Inmeantime you can do warm saline gargle two - three times a day you. You can apply ointment like candid gel on tongue . Use chlorhexidine mouthwash once daily and maintain proper oral hygiene. Hope this will help you."
},
{
"id": 71807,
"tgt": "Suggest treatment for chest pain and phlegm",
"src": "Patient: Hi my name is sreekanth...and i am employed.. recently i got a chest infection which started 1 month before with light coughing... i had to travel a lot and due to that it got worsened. i consulted doctor and did a x-ray and blood test and ol seeme dto be fine.. and i took antibiotics prescribed by the doctor too. but still i am having some problem and mucus is still devoloping and i am having slight chest pain... pls suggest a medication Doctor: Thanks for your question on Healthcare Magic.I can understand your concern. By your history and description, possibility of post infectious bronchitis is more likely. Bronchitis (inflammation of airways) is common after respiratory infections and is known to cause prolonged coughing, chest pain, tightness etc.So you will need inhaled bronchodilators (formoterol or salmeterol) and inhaled corticosteroid (ICS) (budesonide or fluticasone) for bronchitis.Consult pulmonologist and start above treatment.Hope I have solved your query. I will be happy to help you further. Wish you good health. Thanks."
},
{
"id": 126291,
"tgt": "What causes severe pain around upper thigh while standing?",
"src": "Patient: When I stand or walk there is pain in the area where the upper thigh and torso meet, in the bending area, towards the outside more than towards the center of the body. It s intense enough to cause a significant limp. Started when I got out of the car this morning after driving to work. Is this a hip joint situation or nerve? Doctor: Hi, It could be due to musculoskeletal pain. As the first line of management, you can take analgesics like Tramadol for pain relief. If symptoms persist you can consult an orthopedician and get evaluated. Hope I have answered your query. Let me know if I can assist you further. Regards, Dr. Shinas Hussain, General & Family Physician"
},
{
"id": 221481,
"tgt": "What causes low blood pressure and high pulse rate during pregnancy?",
"src": "Patient: I have an appointment with a cardiologist today but wanted to read up on what the problem could be. I have normally low blood pressure (100/60) and my pulse is normally between 60 and 70. I am in the 6th month of pregnancy and my blood pressure drops to 80/40 and my pulse shoots to 130-150 at times. It happens when I am not doing anything strenuous. Just walking around. Any ideas? Doctor: HiDr. Purushottam welcomes you to HCM virtual clinic!Thanks for consulting at my virtual clinic. I have carefully gone through your case, and I think I have understood your concern. I will try to address your medical concerns and would suggest you the best of the available treatment options.Please do not worry.Have plenty of fluids in diet. Have small frequent feeds.Just observe if you are hungry or do not eaten out for long time. Low sugars can also be a reason.I hope my answer helps you.Thanks.Wish you great health."
},
{
"id": 208480,
"tgt": "Can I continue with lustral for anxiety while suffering from stomach cramps and diarrhea?",
"src": "Patient: I am a coeliac (Female 55 years ) and was prescribed lustral a few months ago for anxiety. However I've since suffered from stomach cramps and diarrhea afew times a week initially and for the past week I have had a few episodes every morning. Is lustral the cause of the probelm. I'm good for staying on gluten free diet and I eat well. Doctor: Hi, thanks for the query. Short answer Yes; Lustral (sertraline) is the cause of your abdominal cramps & diarrhoea. Srtraline is a SSRI type of drug which is commenly used for treatment of depression & anxiety. As such it is very safe but it does cause abdominal cramps, bloating, diarrhoea in a few patients. If you are having some relief in anxiety symptoms with Lustral; then you can either wait for 2 weeks for your body to adapt to lustral so that these symptoms will subside (side effects usually within first few weeks of starting medications).Otherwise you can be switched to other SSRI like Escitalopram, paroxetine, fluoxetine, fluvoxamine etc. which are relatively free of abdominal side effects. Consult your doctor for change of drugs. Good Luck"
},
{
"id": 207847,
"tgt": "Suggest medication for anxiety",
"src": "Patient: ok so i am 26 and i have been given everything from lezapro tp all the things they give for anziety i react bad horrible to all of those and i mean i have been given all the new one3s old ones so finally thye put me on xanax also klonopin not at the same time anyway it worked but only the highest dosages the lower dosages did not work doc did not put me on the dose that a doc had given me before like point 5 or 1 mg i need this med long term bc nothing else works im have a new born a 4 yr old i cannot be a guinee pig for them i needto be fine and this is the only thing i can take how do i tell miy doc this bc nost of them thinking drug addiction i have med recors from when i wa on the high dose how do i finda dox ann what do i say Doctor: earWe understand your concernsI went through your details. I suggest you not to worry much. You must understand that anxiety is not a disease but a disorder. There are underlying causes for that. Understanding them deeply can be good.Many researches and researchers confirm that medicines alone cannot cure mental disorders. Life style changes, change in thinking pattern, relaxation etc are as essential as medicines. Psychotherapy can help you changing your lifestyle and thinking patterns. Yoga and meditation help you to streamline your metabolism and neurological balance. Please consult a psychologist for further information.If you require more of my help in this aspect, Please post a direct question to me in this website. Make sure that you include every minute details possible. I shall prescribe the needed psychotherapy techniques which should help you cure your condition further.Hope this answers your query. Available for further clarifications.Good luck."
},
{
"id": 53300,
"tgt": "What causes pain in the right abdomen with abnormal results for LFT?",
"src": "Patient: Hi Doctor I had a big pain up right deodunom , i went emergency my LFT was abnormal and high fever . I hospitalized in hospital DR diagonist and has done Endosonografy then checked and report Gallb has a trace sludge and normal , liver was normal pacrease was a mild chronic my CBD dialeted 10mm and at the end was taperd( ampulla vater is daiverticul) now All LFT is normal alkp is normal amilase lipase are normal billirobin D and T are normal . what are you recommended, I remove Gallbellader , Dr fix Stentsor I wait until Pain come over ,I consume Ursoflor \u00b7300 mg two times per 24 hrs. thaks a lot before M .Jebelli Doctor: Hi and welcome to Healthcaremagic. Thank you for your query. I am Dr. Rommstein, I understand your concerns and I will try to help you as much as I can.LFTs are sensitive indicators of liver damage or injury from different types of diseases or conditions, and collectively they are termed liver tests or liver blood tests. So the next step is to look for certain liver damage and most common casue is fatty liver disease. It should be confirmed by ultrasound. Other causes are viral hepatitis,cirrhosis, medications, alcohol intake or autoimmune diseases. To verify exact cause, US, bilirubin levels and tumor markers should be additionaly done Then appropriate treatment can be started.I hope I have answered you query. If you have any further questions you can contact us in every time.Kindly regards. Wish you a good health."
},
{
"id": 92521,
"tgt": "Sharp pain in lower right side of abdomen while sneezing, coughing, bloating in stomach. Active in sports. What could this be?",
"src": "Patient: I am 17 and am very active in sports. Here lately I have developed a cough that has lasted weeks. Well anyways usually my stomach becomes sore and you can tell my stomach muscles are just sore.. It usually goes away within a few days. BUT.. This time there is a SHARP pain in the mid/lower right side of my abdomen every time I cough,sneeze, apply a slight amount of pressure, breathe in too deeply, or sometimes laugh. Like I said. It hurts to touch and I seem to be a little bloated too! (I don't know if that contributes or not) Doctor: Hi,From history it seems that due to long lasting cough there might be having heavy strain and spasm on abdominal and lower chest muscles giving rise to this problem.Take some analgesic with muscle relaxant medicine to get relief.Apply some muscle relaxant cream on affected part.Ok and take care."
},
{
"id": 178524,
"tgt": "Is the large abdominal bruise a cause for concern?",
"src": "Patient: 8 yr old son has Chronic ITP (platelet count last month was 59,000), and he has a bruise on his lower left side of his abdomen/hip area. It is large (about 2 1/2 in by 1inch). Could this be a cause for concern due to the location or just simply another symptom of his ITP? No injury occurred. Doctor: hi..it is a symptom of ITP. the site of ecchymosis in your child is not a concern. in ITP bleeding occurs spontaneously without trauma.currently there are many new modalities of medical treatment available(eltrombopag and romiplostim, are available to patients with chronic ITP). if not responded since he is 8year old you can go for spleenectomy."
},
{
"id": 202225,
"tgt": "What causes premature ejaculation and erection for only short periods?",
"src": "Patient: mai apani bibi ke sath sax karta hu to hast methun ke karan mera sax kartesamey jaldi ho jata h aur mai aapni wife ka sax ka maja bi nai de pata huiskan mera ling bi jada samay tak tait nai rahta h aur mai aapni bibi ke sathsax bhi nai kar pata hu Doctor: HelloThanks for writing to HCMYour problem may be due to stress or over excitement.You should take time before attempting repeated penetration.You should try to be calm and before penetration spend sufficient time in foreplay.This is very important in intercourse.You should avoid cigarettes completely.You shouldn't take alcohol.If problem persists then you can consult your doctor for proper evaluation.Hope i have answered your query.Take CareDr.Indu Bhushan"
},
{
"id": 209924,
"tgt": "Reason for chest pain feeling?",
"src": "Patient: I take Telma -20 daily, but my bp ranges from 110/75 to 140/100 (particularly after office hours).Many times i feel like i may have chest pain.My lipid profile say total cholestrol 207(167 & 40).Not diabitic, triglycerides around 150, blood urea normal.What should i do to increase my confidence Doctor: HiThanks for using healthcare magicI think, you have underline anxiety. In that case, you should try a low dose antidepressant or benzodiazepine. Better to consult a psychiatrist for proper diagnosis. That would help you to control anxiety symptoms. You can also try relaxation exercise like JPMR or deep breathing exercise. In case, you need further help, you can ask. Thanks"
},
{
"id": 96522,
"tgt": "Is coughing up of blood post a head injury normal while having normal MRI reports?",
"src": "Patient: My boyfriend fell and cracked his head open a few nights ago and he went to the hospital. They did an MRI and x-rays and said everything looked all right. The kept him for two days and released him today. Since he s been home, though, he has been coughing up blood. Is this normal? Doctor: Hi, I really appreciate your concern, what the history of head injury given here, symptoms of cough with blood, is not the normal symptoms and it demands medical attention, patient need to be kept under observation for a couple of days again, repeat MRI or CT study is advisable. Hope I have answered your query. Let me know if I can assist you further. Regards, Dr. Husain, General & Family Physician"
},
{
"id": 167175,
"tgt": "Suggest remedy for sore lump on the back",
"src": "Patient: My one year old son has a small lump on his back, and it started off looking like a skin tag or a mole, then it grew and it developed a white head, but it doesn t have pus or anything in it (we tried squeezing it) and it doesn t seem to discomfort him at all, but its really red, and its hard and we don t know what to do about it Doctor: Hi... by what you say, I feel that it could be a dermoid cyst. The white head which you are seeing on the top could be a secondary infection.I suggest you upload an image of the lesion in this website and get back to us.Regards - Dr. Sumanth"
},
{
"id": 23685,
"tgt": "Can i take 'amlodipine' for high blood pressure?",
"src": "Patient: hi. previously i used amlopipine/valsartan 5mg/160 mg from EXFORCE (novartis). will there be any advantages/disadvantages or complications from changing to amlodipine 10 mg (from eurodrug) ?49 years, male, 1.75m , 78 kg. Asian. Current blood pressure: 135 / 80 Doctor: Hi,Your previous medication contains two medications, Amlodipin 5 mg and Valsartan 160 mg, if you switch it to only Amlodipin 10 mg, it may not be enough to control your blood pressure.Take careCome back if you have any further questions"
},
{
"id": 198097,
"tgt": "Is icsi helpful in asthenospermia ?",
"src": "Patient: My husband is 33ys old. He is having moderate asthenospermia. 2% abnormality are shown for sperm head and neck 1% abnormality for tail. He is also having grade 1 vericocele in right side and grade2 in left side. I had two miscarriages at 20 weeks due to foetal defects. will icsi be helpful for us Doctor: HelloThanks for query . Your husbands semen analysis has revealed Asthenospermia .ICSI is advised in a situation when there is no effective pregnancy as a result of sperm abnormality either low count or less motility .Since you have conceived twice you do not need ICSI to have pregnancy.The miscarriage that you had twice could be due to hormonal imbalance and needs to be checked and evaluated by Gynecologist once you are pregnant and can be treated with medications to prevent miscarriage. Dr.Patil."
},
{
"id": 212130,
"tgt": "Upcoming exams, not feeling too good. On lexapro. Can valdoxan be taken along with it?",
"src": "Patient: I have been taking Lexapro 20mg for about a year and have been doing well but last couple of weeks not feeling too good. I have major exams coming up in May and was wondering if I could take Valdoxan with Lexapro on a short term . I've been on Valdoxan before and had good results but was taken off due to my liver enzymes increasing Doctor: Hi, Thanks for using healthcare magic. I have gone through your history. You might have performance anxiety due to stress related to exam. Lexpro is escitalopram which is very good anti depressant drugs. Valdoxan contains agomelatine which is novel, good anti depressant drugs. But you must consult your psychiatrist before starting valodxan on your own. You may also get benefit by propranolol for performance anxiety if any. You may have prescription for it from your psychiatrist. You may also benefit by doing yoga, meditation or breathing exercise. thanks"
},
{
"id": 196072,
"tgt": "How to improve sperm count?",
"src": "Patient: My husband age is 35. in july 2010 , he had 5 million count of sperm. doctor adviced him to take Hey Forte twice per day. Count increased to 68 million after that it was flucating between 28 million to 68 million during period nov'10 to apr,04. as per doctor's advice, still he continuing the same along with ubi q 300, siphene. But now his sperm count is about 5 million to 11 million for past 3 months. What we have to do to get concieve Doctor: hi.welcome.avoid stress.have adequate sleep.avoid cigarette and alcohol intake.take your medications properly.thank you."
},
{
"id": 205728,
"tgt": "Suggest treatment for schizoaffective disorder",
"src": "Patient: We Have went through about 8 different test and 3 diagnoses. My son is has be sent from therapist to another. After this last test he was diagnosed with schizoaffective Disorder and sent of to another doc that is a C.N.P and she gave use yet another disorder Mood Instability disorder. how do we know were to go from here. the women saw him for on appt. what ? s do we ask Doctor: DearWe understand your concernsI went through your details. Different therapists diagnosing differently? Really a matter of concern. I don't know what to say to you but I can understand your emotions. Let me suggest you to please provide more details here itself so that I shall try to diagnose. There is another alternative. Forget about the diagnosis till now and consult a prominent clinical psychologist after researching on the internet. Proper diagnosis is the key for any cure. If you require more of my help in this aspect, please use this URL. http://goo.gl/aYW2pR. Make sure that you include every minute details possible. Hope this answers your query. Available for further clarifications.Good luck."
},
{
"id": 42116,
"tgt": "What causes severe leg pain after 3rd iui?",
"src": "Patient: Hello doctor,I am 29 years old.I usually have 28 days period cycle.I done my 3rd IUI .Now 18 days got over.I t was done 2 times .I have proper egg rupture every cycle and even my husband has porper sperm count..on day 12 my doctor gave me Injection gonal F75 mg.on day 14 follicle size was Right side:1.9*1.7.same day they gave me Injection Oviterallamp.on day 15 th evening around 7 pm I had severe ovulation pain but my doctor advised to the IUI on day 16 morning around 9 pm.then my second IUI was done on 17th day evening.Now I am 18 days post IUI i have severe leg pain .my period due date is over.Is my leg pain is normal or ?is there any chance that I got pregnant now? Doctor: Hello,Welcome to healthcare Magic.I have gone through your query and Would like to reply in detail as follows:1. There is chance of pregnancy.2. Leg pain has nothing to do with this processI hope your doubt is clarified. Let me know if you wish any other information.Regards,Dr. Mahesh"
},
{
"id": 54009,
"tgt": "Suggest treatment for Hepatitis B",
"src": "Patient: I got diagnosed with hepatitis b today. All the nurse told me was that I am positive for Hepatitis b. After reading up on the disease I don't know what type I have or if I need further treatment. What would you suggest I do because I am very disapointed in the way the nurse informed me of the diagnosis. Doctor: Hi welcome to health care magic... Don't feel shy in diagnosed as hbsag... Your work up needed with... -Serum biliribin -sgpt and sgot estimation This to rule out acute hepatitis.... You have to repeat your hbsag test after six month... If still positive than you are chronic carrier which remain for life time... For such hbsag nothing done actively.. In acute hepatitis b infection 95% approx. Cleared from infection... Take less fried food, fruits more... Regular exercise... Avocados, lemon juice, spinach, Indian gooseberry, onion are good for liver... Take care.... Advise: Consult physician and get your examination done...."
},
{
"id": 92332,
"tgt": "What is the reason for heaviness in the right lower abdomen?",
"src": "Patient: I have been feeling uncomfortable in the right lower part of my abdomen. By uncomfortable I mean like I feel something heavy there. When I sleep if I lay on the side of the uncomfortableness the feeling worsens and my hips hurt.... Is it an appendix thing or are my hips just dislocated from my heavy backpack? No clue! Doctor: Hi. Definitely the appendicitis is one of the reason for this feeling of heaviness in the right lower part of your abdomen. This is on of the most common reasons. IF the walking and moving hip joint is normal, the joint problem is ruled out. If this is of long duration with associated symptoms of bloating, nausea, constipation, i would advise to go for further tests like ultrasonography of abdomen. Less is the discomfort more can be a problem."
},
{
"id": 9614,
"tgt": "Why did my skin turn dry post delivery ?",
"src": "Patient: hi, after giving birth my skin is sooo dry all over my body. tried lotion for weeks and no change, still dry makes me feel 10 years older Doctor: I have the same thing, and my midwife said it's because my estrogen levels are low after the baby is born. Also, I'm breastfeeding, so she said the estrogen levels will not return to normal until I stop breastfeeding. She said to use lots of lotion, stay hydrated, and try adding flaxseed or foods with omega-3's in my diet."
},
{
"id": 170533,
"tgt": "What causes spots on the cheeks?",
"src": "Patient: My 10 month old baby had one spot on his cheek last week that was very red for a day and then got a white center to it and popped about 24 hours after it appeared. I took him to the doctor and they didn t know what caused it, but gave us a prescription antibiotic cream to put on it. Today, my baby woke with another spot on his other cheek and one on his let. Do you think they could be gnat bites? He is outside at daycare for a couple of hours each day. I know that they are NOT mosquito bites. Doctor: Hello there, thanks for using Health Care Magic.From what you ve described, it seems like your baby has furunculosis or a small abscess. It should settle on its own and if it is very painful, causes fever or large, may require an incision to drain it. I would advice you to wait aand atch unless he develops spots around his mouth or under his nose, in which case he will need antibiotics. Another thing to keep in mind is that if he gets furuncles often, he may have a bug called Staph aureus in his nostrils or armpits and will need a decolonization regimen with mupirocin ointment in both nostrils for 7 days, and a chlorhexidine body wash for 10 days. Hope this helps you.Regards,Dr. Vaishnavi"
},
{
"id": 132275,
"tgt": "Suggest treatment for severe pain and red spots on lower back",
"src": "Patient: my husband was climbing down from a roof and hurt his back and for last two days been able too get out the bed. we went to chiropractor this afternoon and hew only made matters worse. I just pulled his shirt to rub Lotion on his lower back and there are two red spots?help please Doctor: HiYou must take him to ER for X-rays and treatment not to miss major injury to spine, fracture or sprainsHe should take hot fomentation, apply muscle relaxant gel and have complete restriction of movement out of bed for acute condition to subside.Avoid chiropractics absolutely..Advil tabs may be given after meals and chloraxazone tabs for pain and muscle relaxation till he's seen by orthopedician/ER doctorsBest wishes"
},
{
"id": 156799,
"tgt": "Which is the best prostate cancer treatment? Does cancer cause swollen ankles?",
"src": "Patient: Hi, I was just diagnosed with Prostate Cancer and plan on choosing treatment. I had an intitial visit consultation with the Radioligist and he noticed that my ankles were swollen. Does that have any thing to do with the Cancer? Also, is it better to get the prostate removed or get radiation treatments? Thanks I am 58 years old Doctor: HIThank for asking to HCMI really appreciate your concern if this is the malignancy prostrate enlargement then better to remove this surgically then rest of the management depends upon the classification of cancer stage, some time it needs chemotherapy some time it dos not need it, hope this information helps you have nice day."
},
{
"id": 138006,
"tgt": "Suggest treatment for weakness in legs",
"src": "Patient: Hello Dr. Sudhir, I would like to introduce myself as Roopa from Bangalore. My mother has been diagnosed with MND in 2012. Symtoms of weakness started since jan2011. Current status- foot drop in both the legs, right leg from hip to toe (full leg) weak cannot lift no strength,\u00a0 left only foot drop she can fold her knee and lift her leg.hands are fine. I would like to take your kind help and know your opinion on this case in terms of betterment for the patient\u2019s condition and IVIG treatment for the same please. I\u2019ve been surfing internet about this medical situation, and have come across \u201cIVIG for MND\u201d. I would like to take your advice in this matter and understand if my mother can undergo IVIg treatment.Your inputs are deeply appreciated.\u00a0 Regards, Roopa Bangalore Doctor: dear patient motor neuron disease is progressive and ireversible disease with no effective cure found till date. intravenous immunoglobulin is found effective in many cases and can be tried. it may lead to adverse antibody reactions which are life threatening and very costly. you need to consult expert neurophysician and get youself examined and opined.."
},
{
"id": 129915,
"tgt": "What could cause swollen and hard cervical lymph nodes with constant low iron levels?",
"src": "Patient: Good evening, I have had low iron levels for the past 112 days (I usually donate blood every 56 days and was denied because my iron levels were to low both times) and one of the cervical lymph nodes on the right side has become very swollen over the past month with no pain... Just hardened area that does not move. Any suggestions Doctor: cervical lymph node enlargement is a significant issue and cause should be evaluated by ultrasound abdomen and CT scan abdomen. . consult ur surgeon without delay and get evaluated first. thank u"
},
{
"id": 183840,
"tgt": "What causes white spots inside cheeks and tooth pain?",
"src": "Patient: Hey, i have been freaking out lately on my tooth pain. well its broken and nothing in there and right under it in the jaw there is something that is painfull and that moves which suddenly showed up. Besides that the inside of my cheeck has white spots now and im freaking out. please help. Doctor: Hi,Thanks for posting the query, I would suggest you to get an x-ray done of the tooth if required a tooth canal treatment can be performed antibiotics analgesics course is required, at home take lukewarm slaine nad natiseptic mouthwash rinses , take multivitamin suplements maintain a good oral hygiene.Take care!"
},
{
"id": 141467,
"tgt": "Suggest treatment for freezing in a Parkinson s patient",
"src": "Patient: Hi my name is Cory. I have a friend who is 44 years of age and was diagnosed with Parkinson s Disease in 2012. He is not wheelchair bound yet. He is getting medications. He is now seeing a neuropsychiatrist. He also suffers from high anxiety and depression on a scale 1-10 his anxiety is a 10. Many times during the day or evening he experiences his body freezing up and not being able to move. This last from anywhere from 45min to 3 hours. Does Parkinson s Disease cause freeze ups for this long? Or is it possible in my friend s case that it may be psychiatric? Thank You Doctor: Hello and welcome to \u2018Ask A Doctor\u2019 service. I have reviewed your query and here is my advice. This does not have the description of a PD patient. Freezing is not that long and is usually related to when the patient is walking in Parkinson's Disease....it's called GAIT FREEZING. Has this person been checked for STIFF PERSON SYNDROME? That is what I would check. If you're asking about whether the person could have a psychiatric condition that causes or contributes to this picture then, the answer is \"yes.\" Psychiatric disease can cause whole or partial body immobility that lasts up to hours. It is referred to as CATATONIA. However, if it is true freezing as in real live boardlike rigidity then, go with STIFF PERSON SYNDROME. If they are simply immobile and unresponsive then, CATATONIA. Please rate this a 5 STAR ENCOUNTER and write me at: www.bit.ly/drdariushsaghafi with more comments/questions. Hope I have answered your query. Let me know if I can assist you further."
},
{
"id": 224723,
"tgt": "On birth control. Sexually active, missed pill, spotting, pregnancy test negative. Pregnant? Should i continue pills?",
"src": "Patient: Hi there, i have been on birth control since 2006 (alesse) i have never had issues with spotting before. The past few months I have been very inconsistent with my pills, and have been sexually active. My partner has not ejaculated in me, but we have also not been safe . When missing the pills, I started doubling up for 3 days until the end of my pills (but was already spotting at this time) I have now started my new pack of pills and am still spotting. is there a possibility that i am pregnant? I have been spotting for about a week now. Should I continue taking my pills as normal, or take a break and let my body reset ? I have taken 2 home pregnancy tests and both came out as negative. Any feed back you could provide me with would be appreciated. Doctor: HelloThanks for writing to us with your health concern.The erratic spotting is because of the irregular intake of birth control pills.What you do right now is finish the pack ( hopefully regularly ).Stopping it out cold will only worsen your spotting.Then take a break for a month, using only condoms.Once the break is over, start a fresh pack only if you intend to sincerely use the pills.Else use another method.Take care."
},
{
"id": 77297,
"tgt": "What causes pain in lungs while deep breathing or coughing?",
"src": "Patient: When I take a deep breath my lungs burn at the top and when I cough it feels like fire at the top of my lungs. My ears itch and my cough is productive. My temp is only 98.8 with tylenol but I have been on antibiotics for 2 weeks and a prednisone pack for 6 days. Doctor: Hi thanks for asking question..Pain while breathing could be by pneumonia like condition.chest x ray useful for it to search for consolidation...If consolidation seen tuberculosis also has to be ruled out by sputum examination...You can have bronchitis that also can lead pain.x ray useful for it.You have not mentioned x ray finding....If you have asthma case then also by allergen bronchospasm can lead pain...auscultation done to heard rhonchi....Meanwhile avoid smoking and CBC done....Take care...."
},
{
"id": 55207,
"tgt": "Is surgery necessary for gall stones and helicobacter pylori?",
"src": "Patient: Hi. I was diagnosed through an ultrasound of multiple gallstones, the biggest measures 16mm, associated with thickened wall denoting calculus cholecyctitis. In my stool test, i am positive for helicobacter pylori. The doctor said i need to undergo surgery. I'm not sure if the surgy is for that h pylori or for that gallstones. What are the risks of the operation? And what is the consequences if i delay the surgical operation. Thanks a lot. Doctor: Hello! Thanks for putting your query in HCM. I am a Gastroenterologist. Surgery was advised for gallstone and cholecystitis that is cholecystectomy that is removal of gallbladderRisks of operation are less l;ike bile duct injury, sepsis, and any risk of operation done under anaethesiaThe consequences of delaying the surgery can be recurrent cholecystitis, empyema of gallbladder that is infection of gallbladderI hope I have answered your query and this will help you . Wish you a good health"
},
{
"id": 30239,
"tgt": "How to treat campylobacter infection?",
"src": "Patient: campylobacter relapse. I am now getting the syptoms of campylobacter for the 4th time. Every 3 weeks its starts I am bad for 10 days, have a good week then am back to square one. The first 2 times I had postive stool tests, the 3rd time given erythrymyacin which I was unable to tolerate after 2 days. Now it seems it is back again.I also have CFSany advice, is this normal to keep relapsing?regards Maxine Doctor: Hello Maxine,I read your message carefully and I understand your medical problem.You are going through campylobacter infection for the fourth time now. You mention positive stool test and intolerance to erythromycin. Azithromycin is the first choice in Campylobacter infection. Erythromycin is a drug of choice because its resistance remains low. You are sensitive to Erythromycin so you can not take it. Other alternatives are ciprofloxacin, tetracycline and clindamycin in adults, but in a combination therapy. Actually, there is a schedule for the treatment of campylobacter involving antibiotics, anti-H2 antagonists (ranitidine, cimetidine, etc) and proton-pump inhibitors (omeprazole, pantoprazole, etc). You are experiencing antibiotic resistance. Campylobacter is a bacteria which is able to develop mechanisms to protect itself from drugs.Combination therapy of antibiotics can be of help in this case based on culture and sensitivity test of the stool. Do not forget about changes in your diet habits and keep a good personal hygiene as it is a cause to foodborne illness. Discuss this further with your doctor. Wish fast recovery. Dr.Albana"
},
{
"id": 160014,
"tgt": "What time it will take to regrow hair after chemotherapy ?",
"src": "Patient: hi ..ihad my chemotharphy..n now i m having herceptin for one year ...i need to ask when my hair will come n how to take care of my skin n wat to eat .....please tell wat should i drink as well Doctor: your hair loss is temporary,it usually grow again within 3 to 6 monrhs after ceassation of chemotherapy.you should eat homemade,healthy & fresh food."
},
{
"id": 57150,
"tgt": "Suggest a diet which is good for the liver",
"src": "Patient: hi,I am 36 years old recently i am on akt4, because of this my SGOT and SGPT have gone up to 71 and 91 respectively. I just want to know what precaustions i should take to get this down. Some home remidies or some diet change. which diet is good for liver. Kindly suggest Doctor: Hello Thanks for writing to HCMIncreased SGPT indicates liver injury.SGOT is non specific finding and it increases in many conditions.Increased SGPT may be due to hepatitis,altered lipid profile,autoimmune causes,medicines etc.You need clinical examination and few other investigations.You need few more investigations like random blood sugar(RBS),renal function test(RFT),complete lipid profile,Liver function test,viral markers,urine RE/ME.You should avoid red meat and high fat milk products.You should avoid fried and junk food and avoid alcohol completely if you take it.Medicines should be prescribed after complete investigations.I suggest my patients to take tablet ursodeoxycholic acid 300 mg twice daily after food for three months.This help in regeneration of liver cells.Get well soon.Hope I have answered your question.Take CareDr.Indu Bhushan"
},
{
"id": 156669,
"tgt": "How to treat red raised bump at the back of the tongue?",
"src": "Patient: i have red raised bumps on the back of my tongue and my tounge is begining to look like a road map. The grooves started down the center and are branching off. The grooves are so deep that strawberrie seeds get buried and stuck. i have scared myself into tounge cancer. (darn internet) My dentist named 2 diseases that he was familar with and neither one fit my symptoms......which are the deep groves in center and now reaching out to the side of my tongue, the red bumps in the back of tongue, everything i eat stings the inside of my mouth. im thinking that my body nees a certain vitamin????? I can not run my tongue over my teeth as it feels like i am tearing my tongue. i also wonder what kind of doctor do i see......ENT?????? this is very uncomfortable ... brushing teeth is the worst. thank you Doctor: hello thanks for consulting at hcm..based on ur symptoms it could be either fissured tongue or geographic tongue,, either way u need a through blood tests,,and apply topical anesthetics for prevention of pain under the prescription of ur dentist,,u can also take a consult with ur internal medicine specialist,, keep ur tongue clean by brushing with a soft brush,,hope it helps,,tc"
},
{
"id": 35662,
"tgt": "Is Pantin-z and Aglozyme effective in treatment of fever and loose motions?",
"src": "Patient: sir, i have been suffering from fever, loose motion, weakness lose of appetite since last 3 days . i had visited doctor yesterday he prescribed the medicine - PANTIN-Z, OFORD-LB,Aglozyme & Coslyte ORS. But still i have little fever and weakness. It my treatment in a right direction please do reply Doctor: Hello dear,Thank you for your contact to health care magic.I read and understand your concern. I am Dr Arun Tank answering your concern.Your treatment is in the right direction. You should continue the treatment.For fever to disappear it require some time, till that time you have to continue the treatment.I also suggest you to do culture sensitivity of your stool. Many a times it may be possible to have resistant organism which is causing you diarrhea difficult to treat.If it is so sensitivity report will tell you the truth, taking treatment according to it can cure you rapidly and effectively.Please adopt good hand washing practice in future it is good tool to combat any infection.I will be happy to answer your further concern on bit.ly/DrArun.Thank you,Dr Arun TankInfectious diseases specialist,HCM."
},
{
"id": 14181,
"tgt": "Suggest treatment for red bumps on the tattooed area",
"src": "Patient: I believe I have an allergy to the blue ink in my new tattoo. I had a cover up done a week ago and now I have red bumps surrounding the tattoo area. Some of the red bumps have white heads to them. It also itches occasionally. What can I do to treat until I see a dermatologist on Tuesday? Doctor: Hi.As per your case history you are having allergic dermatitis.My treatment advice is \u2013 1. Avoid scratching the tattooed part.2. Apply a mild antibiotic plus steroid cream like fusiderm-B cream twice daily on it.3. Take an antihistamine like levocetirizine for 7-10days .4. Other treatment options are oral steroid and hydroxyzine given only after consulting a dermatologist.Thanks.Dr.Harshit Bhachech.MBBS, DDVL."
},
{
"id": 184982,
"tgt": "Will my tongue get smaller and fit into space after having dental implants?",
"src": "Patient: I just had four lower dental implants and temporary bridge. I have no permanent lower teeth, They were all removed. The problem is my tongue is now larger than the teeth space and I keep biting my tongue and lip. Will my tongue eventually get smaller and fit into the space and normal chewing then resume? Doctor: Thanks for your query, i have gone through your query. The tongue position what you are feeling could be because of your previous edentulousness. nothing to worry. the implants should be placed over the alveolar ridge only. it take little time for the tongue to get adjusted. if the problem continues then you consult your implantologist. tongue will not become small or large after placing the implants or bridge. it can only get adapted to the new position. i hope my answer will help you, take care."
},
{
"id": 95477,
"tgt": "I cant even touch my stomach without it hurting,on top of the chest and back pain. What can I do next?",
"src": "Patient: i cant even touch my stomach without it hurting, on top of the chest and back pain . it worsens after i eat anything. Ive tried bland tastless foods. Anything i put in my system hurts. I have been vomiting, sometimes there is blood in my vomit. I have also been very lightheaded and dizzy where I cant stand up straight. They ran CTs and blood work and said everything looked normal, what can I do next? Doctor: DO NO WORRY AT ALL ,GO FOR ENDOSCOPY.YOU ARE PROBABLY SUFFERING FROM BENIGN GASTRIC ULCER DUE TO INFECTION OF HELIOBACTER PYLORI. IF ENDOSCOPY CONFIRMS THE ULCERATION YOU MUST START A FULL COURSE ANTIBIOTIC REGIMEN ALONG WITH ANTI ULCER DRUGS LIKE OMEPRAZOLE.,IN GUIDENCE OF A GASTROENTEROLOGIST.63%( nor normal in your case ,but very low because 80 % are NON-MOTILE) with >12 % exhibiting normal morphology ( normal in your case ) In my opinion , consult an INFERTILITY specialist and get his opinion regarding this semen analysis ."
},
{
"id": 202445,
"tgt": "What does this semen test showing total spermatozoa count suggest?",
"src": "Patient: Hello Doctor According to my Semens test report done today the findings are as below- Quantity- 4.00 ml Reaction- Alkaline Total Spermatozoa- 80 million cc Actively Motile Spermatozea- 60% Sluggishly Motile Spermatozea- 20% Non Motile Spermatozea- 20% Pus Cells- 8 to 10/ HPF is this ok? or need any medication? Doctor: Hello Thanks for writing to HCMYour semen analysis report suggests infection.Infection is indicated by the presence of pus cells in semen. Normally there shouldn't be any pus cells in semen.Infection may be due to prostatitis,UTI etc.You need antibiotics for infection.You need few more investigation like routine hemogram,Semen culture and sensitivity,RBS,Ultrasound of pelvis should be done.Trans rectal sonography of prostate can be done if needed.Proper antibiotics should be prescribed after culture and sensitivity report.Other findings like sperm count and motility is within normal limit.Get well soon.Hope I have answered your question.Take CareDr.Indu Bhushan"
},
{
"id": 104125,
"tgt": "Acute gingivitis, diabetic, allergic to benzocaine, hyperthyroidism, panic attacks. Help?",
"src": "Patient: My temp is 96.7 I hv never really checked my temp but I went to er yesterday i was told i hv acute gingavities. Ima diabetic My daughter in law died do to infection spread to her blood stream I was told to tk my temp observe but dont no exactly what I m lookn 4 I was gvn zitromax 250 mg Im allergic to benzocaine all caine Im 55 yr female 5 3 212 lbs asthmatic, hv hyperthroidism, panic attack, hbp,diabetic, unable to get tooth extracted by reg dentist do to also suffering agoraphobia , Doctor: Hi welcome to Health care magic forum. thanks for calling H.C.M.Forum. You have acute gingivitis, You also have hyperthyroidism, panic attack, Hbp, diabetis, and agoraphobia. you are 55 yr , female, 5.3, 212 lb, You are unable to extract the tooth. You are asked to observe the temperature to find if the infection is spreading to the blood, If that happens, and neglected there is chance of septicaemia and risk increases due to dibetis. I advise you to consult a physician for diagnosis and treatment. You may need to get culture and sensitivity of pus of the oral cavity done besides other routine tests for confirmation of type of infection and sensitive drugs. You should be under control of diabetes, B.P, and hyperthyroidism for prompt cure of gengivitis. Under the supervision of a dentist you can have a relief after the controle of the conditions. Wishing for a quick and comfertable relief of the disease. Best regards."
},
{
"id": 34125,
"tgt": "How long do symptoms of mono last?",
"src": "Patient: My son was diagnosed with mono over 2 months ago. Symptoms constant lethargy hungry but unable to eat any decent amount. These symptoms were an entire two months before he was diagnosed. I feel like he should be better by now, and his symptoms seem atypical: no fever and no vomitting, but constant upset stomach and complaining of heartburn....only 18 years old???? Doctor: Hello.If I understood weel, your son is diagnosied an infectious mononucleosis and symptomes last for 4 months by now. It is a bit too long duration of illnes, since it usually resolves in few weeks.You should contact doctor and do check-up.All the best."
},
{
"id": 108514,
"tgt": "How to treat acute back pain?",
"src": "Patient: Hi ! Doctor...I am suffering from an acute back ache for the last 30yrs or so.Tried occupressure,physiotherapy,medicine occupunchure have taken ayurvedic,homeopathis and other medicines,but for sometime I get relief.But this relief does not last long.Iam of 65yrs,wt 71kg,height is 5.8inches.Xray show cartilage worn out in the lumber region.I go for walks in the morning for about an hour.Basically a vegitarian,but at times relish nonveg as well.Have used gels n sprays various types prescribed by the doctors,but back to the square one.Pls help..Thx YYYY@YYYY Doctor: Please do lumber muscle strengthening exercises.you have to do a epidural discogram.aftr seeing the image you can undergo interventional pain management.currently you can take tab ultracet 1/2 tab tds and tab newbona plus i tab daily"
},
{
"id": 167698,
"tgt": "Can milk allergy cause reflux in babies?",
"src": "Patient: i have a newborn that has reflux and is on a soluble tablet for it i have had to take him off breastmilk after changing my diet several times. he has been on two other formulas and now is on his 3rd.he spits up constantly no weight gain and cries after every bottle. he had fever last week and took him to the dr his lab came back okay she said he could have inflamed intestines. could a milk allergy have caused this? will the soy formula work or neocate formula? other formulas were nutramigen which caused his reflux to act up majorly and the other was allimentum it helped with the reflux just made his digestive system act up.. he is 2 months old and has cried continuously Doctor: the signs and symptoms of cows milk protein allergy includes gastric upset , vomiting, cramps, failure to gain weight and sometimes diarrhoea / constipation or bloody stool .in sever cases of cows milk protein allergy neocate is the gold standard formula because it contains completely hydrolyzed proteins into amino acid which doesn't cause allergies unlike proteins .the only problem is its coast . so another option is to use partially hydrolyzed formula like Nan HA or bebelac HA . it has a reasonable price and effective .soy-based formulas are not recommended because they cross reacts with cow's milk ( about 60 % ) which can cause allergy .I hope this helps"
},
{
"id": 41626,
"tgt": "Suggest treatment for woman with left part of uterus not visible to conceive",
"src": "Patient: My name is Khanuddin.I am living in Pakistan.I am a married man.I and my wife has no kids for which we went to specialist Obs/Gynae who done M.R.C.O.G from London UK.The doctor advised us to HSG.My wife had done two times HSG.In her last HSG was like that \"Only right horn of the uterus and right fallopian tube has been opacified along with the fallopian tube with free flow of the contrast into the peritoneal cavity.The left fallopian tube and left horn of the uterus are not visible in the current examination\".On this she told us that you couples can not be able to be parents.Now what should we do we need Help............................... Doctor: HelloWelcome to health care magic. It is uterine developmental problem. In simple words only half part of uterus is developed , other half not there.If that half of part of uterus is in good size there is little chance of pregnancy.You can try test tube baby With surrogate mother."
},
{
"id": 84221,
"tgt": "What Ecospirin, Evion forte, Solfe and Calcimax tablets do?",
"src": "Patient: Dear sir. I m in 35 weeks of pregnancy. I m taking Ecospirin, Evion forte, Solfe and Calcimax tablets as advised by doctor. May i know the purpose of these tablets. My weight is increasing rapidly. Due to any of the above tablets, the weight will be increased? Doctor: Hi, They are used during pregnancy to prevent some complications. Ecospirin is commonly prescribed to prevent still birth and preeclampsia. Evion forte also helps in preventing preeclampsia and intrauterine growth retardation. Solfe tablet has a combination of iron, folic acid and vitamin B-12 and commonly prescribed to prevent pregnancy associated anemia. Calcimax tablet has a combination of vitamin-D, zinc sulfate, magnesium hydroxide and calcium. It is useful in preventing their deficiencies during pregnancy. These medications are not associated with weight gain. I advise you to consult your doctor for further evaluation and to rule out the possible causes of rapid weight gain. Hope I have answered your query. Let me know if I can assist you further. Take care Regards, Dr. Mohammed Taher Ali"
},
{
"id": 14570,
"tgt": "Suggest treatment for rash on leg which is spreading",
"src": "Patient: I have had a rash on my upper leg/ lower but for a couple of months. It started out small and ha become larger and now is starting to spread. It doesn't have a distinct shape but the edges are a little raised. I already went to my family doctor and he gave me a steroid cream, it helped at first and then it came back! Please help before it gets worse!!!! Doctor: Hello,Thank you for posting on HCM.It seems you are suffering from Tinea corporis, a kind of fungal infection.I would suggest you to consult your dermatologist for proper management of the condition.I usually recommend proper course of oral anti-fungal drugs like Fluconazole or Itraconazole along with anti-fungal cream like luliconazole for local application at bedtime. You can additionally use antifungal dusting powder containing sertaconazole during day time and a soap containing ketoconazole for rinsing of affected areas. Take oral antihistaminics for itching as required.Maintain hygiene over those areas and avoid wearing tight undergarments.Hope your queries are resolved and wish you best of health.Kindly spare some time to rate my answer and drop your valuable review at the following link:https://urldefense.com/v3/__http://doctor.healthcaremagic.com/doctors/dr-hardik-pitroda/67169If__;!!Mih3wA!SBzm6_kI6hCZ58EPH6N_05MFfiPbxWXT0a2TJCdFQObRWm5mV5ur7hW9rW0j8A$ you require any further assistance in future, you can reach me directly through the above mentioned link.Thank youDr Hardik Pitroda"
},
{
"id": 118515,
"tgt": "Have genotype SC, do not have serious health issues, normal child delivery. Should I be alarmed of sickle cell anaemia in future?",
"src": "Patient: I am 31 yrs old and have genotype sc which I never knew until I was 10 yrs because of my Healthy nature,and now for the past 14 yrs I have not had any serious health issue ,I had a normal child delivery n there was never any crisis since then,should I be hopefull that I may not have any serious health issues connected with sickle cell anaemia in future since I have enjoyed quite a healthy life in the past? Doctor: hi theredont worry too much about sickle cell diseasechances are you have one normal gene in your genes and therefore you will be safehowever about kidsyou need to be careful with your sonyou see the male child inherits the x chromosome from motherso you may pass on the gene to sonsince father does not donate x chromosomeyour son might have problemyour daughter may have problems too if your husband has sickle cell trait toothe best means to avoid sickling crisis is good hydration and maintaining oxygen levelshope this has answered youbest of luck"
},
{
"id": 128327,
"tgt": "What causes nerve pain around the wrist region?",
"src": "Patient: I have been having what seems like nerve pain (it radiates) in my left wrist at the base of my thumb. It comes and goes but has lasted for about 5 weeks. I am 68 and otherwise very healthy. I practice yoga regularly but am not aware of having caused any injury to this area. Doctor: dear sir /madami would like to know are your thumb movements painful,i would also like to examine for any local bone tenderness, movements of the thumb and also perform certain manuevers to rule out tendon inflammation,then come to the diagnosis, its very difficult to tell anything with the information given,"
},
{
"id": 93731,
"tgt": "Lower abdominal pain. Diagnosed soft tumor. Ultrasound normal. Should I go for FNAC test?",
"src": "Patient: My mother was complaining pain in lower abdomen . She was rushed to a Local hospital yesterday where she was diagnosed as having a soft tumor and advised for a ultra sound and FNAC test. The US report came normal result.But due to absence of the technician FNAC could not be conducted yesterday, The test would be conducted day after tomorrow. I was told by my friend that, in case the tumor is a non-benign and being aspirated with a needle the tumor shall be spreaded all over the body and the patient would be un-recoverable. My question is should I go for an FNAC test as advised by my Sargon or leave the case for open surgery With regards Jamini Kar Mob : 0000 YYYY@YYYY Doctor: Hi jaminikar, Thanks for visiting HCM. FNAC or fine needle aspiration cytology is a safe procedure using the smallest possible calibre of needle (24 or 26 G), going into tissue, aspirating whatever comes out, preparing the slide and getting it examined by a pathologist. And with in a matter of one day, the nature of the lesion is detected. Very few tumours spread along the track of FNAC. Believe me it is a relatively safe procedure and knowing the nature of the lesion is very important and many a times avoids surgey. Suppose it comes a tubercular lesion, only antitubercular treatment will do. It may be a simple benign cyst. Don't worry go for the procedure.... Best wishes"
},
{
"id": 149218,
"tgt": "Will a person with hydrocephalus have seizure disorder and due to imbalance of electromagnet current in the brain?",
"src": "Patient: For curiousity sake is it usually the case that a person with hydrocephalus have a seizure disorder as well due to imbalance of electromagnet current in the brain? I have had hydrocephalus since I was 3 months old. I am 55. I had my seizures start when I was 17. I take a lot of medication and also have a vagus nerve stimulator. I hope this is enough information for you. i can add to it if need be. Doctor: Hi,Thank you for posting your query.Generally, a person with hydrocephalus does not have seizures. So, we need to look for other causes for your seizures.MRI of the brain (epilepsy protocol) can help us find out the reason for your seizures. Sometimes, no cause can be identified, when it is referred to as idiopathic or cryptogenic seizures.I hope it helps.Please get back if you require any additional information.Best wishes,Dr Sudhir Kumar MD (Internal Medicine), DM (Neurology)Senior Consultant NeurologistApollo Hospitals, Hyderabad,My personal URL on this website: http://bit.ly/Dr-Sudhir-kumar My blog: http://bestneurodoctor.blogspot.com/"
},
{
"id": 219680,
"tgt": "When should I take pregnancy test?",
"src": "Patient: I come off the pill 06/01, to try for a baby and had my first period 11/01 for 4 days. My next period came 44 days later 24/02 and lasted for 4 days also. It is now 11/04 - 46 days after my last period and I have come on yet. I was on microgyon 30 for 6 years. I took a pregnancy test 27/03 and was negative. Is there anything my doctor can give me to bring on my period? or is it too early? Doctor: since your periods are late for 46 days, if you do want to bring them on, Progesterone (provera) 10 mg can be taken for 5 days , so long your pregnancy test is neagtive. it will bring on the periods with in 1 to 2 weeks."
},
{
"id": 134479,
"tgt": "Suggest remedy for swelling in leg after an injury",
"src": "Patient: 4 days ago i injured my knee, had something fall against the left side of my left leg while i had my full weight on the leg. It swelled up and became impossible to walk within an hr so have been using RICE ever since in combination with light massage on the knee area and staying off the leg. There was a great deal of pain the first 2 days and i did not take anything for pain because i m allergic to anti-inflammatory s and do not like taking tylenol because it wouldn t help the main problem. Most of the swelling has gone down and i m able to move about easier but am still not applying any weight to leg. How long should i keep this up before seeking a drs care? Doctor: Hi,if your pain and swelling has come down. Then try to focus on doing some exercises for the knee like static quadriceps , static hamstring, ankle toe movements and straight leg raise. Post that try doing slow weight bearing exercises while taking full support of your hand and stand on one leg. Once you get strength in your leg you will get ease of the symptoms and your muscles will be stronger. If later on doing all these you still fill a little discomfort is there, you can meet up a physical therapist who will help you regain the strength and confidence. I wish you have a speedy recovery."
},
{
"id": 2036,
"tgt": "Suggest treatment for inability to conceive a child",
"src": "Patient: Hello! My husband and I have been trying to have a baby for about 18 months. I've been tested and pronounced ok. He hasn't been tested but I've been impregnated by him before a few years back. He smokes and drinks and Ive recently switched to decaf coffee. Getting a bit discouraged now! Please help Doctor: Hi I think a semen analysis of your husband needs to be done. You can try medicines for ovulation. Talk to your doctor regarding this. Track growth of your eggs by repeated ultrasound. When the time of ovulation comes be in contact with your husband. Then you will need some medicines for supporting the implantation. Hope I have answered your question."
},
{
"id": 91318,
"tgt": "What causes a discomfort/burning in the stomach?",
"src": "Patient: I have just completed triple therapy of flagel ciproxin and Zoton 60mg on Monday. The last two nights i have been wakened early with a discomfort/burning at the head of the stomach radiating towards the left. There today is was tender again at 5pm to which i took a Zoton to help alleviate discomfort. I adhered to taking the medication correctly so as to help eradicate effectively. Is this discomfort secondary to coming off 60mg of Zoton?? Many thanks. Doctor: Hi,After completing the tripple therapy, you need to continue zoton for a month or two to help heal the ulcers created by the infection. That pain is due to unhealed ulcers.RegardsDr. Ashish Verma"
},
{
"id": 179416,
"tgt": "What causes a swollen tongue and low sugar in a newborn child?",
"src": "Patient: Hello, My newborn grand baby is in Tripler Army Medical Center with a swollen tongue and low sugar. What can cause this to happen. We are worried because Army Medical Center s tend to have PA s and student probing around. We would rather her be in a Children s Hospital. Please advise. Doctor: HiThanks for writing to health care magic.Hypothyroidism, infant of diabetic mother, sepsis are the few common causes of low sugarIt needs treatment with IV dextrose.Investigate for causes.Wishing your child good healthRegardsDr Arun"
},
{
"id": 170942,
"tgt": "What is the treatment for celiac disease?",
"src": "Patient: Recently my 4 yr old granddaughter was diagnosed with celiac disease. I ve been reading about celiac disease and the more I read...I began to wonder if a diagnosis of cystic fibrosis (in 1960) could actually have been celiac disease. My husband had a sister who died at 4 yrs old from CF. I know Celiac is hereditary and the symptoms could be for many things. Celiac was not really known in the 60 s, was it? Doctor: Hi, celiac disease and cystic fibrosis are both different diseases. Both sign and symptoms are different. Also, in celiac disease if gluten containing foods like wheat are avoided then child can live a normal life. Take care."
},
{
"id": 211574,
"tgt": "Took nordet as ECP, nausea, dizziness, brown discharge. What does it imply?",
"src": "Patient: Hi, I took nordette as ECP exactly last oct 13, 5hrs right after unprotected sex and 12 hrs after first dose. Last sunday, exactly 7 days since i took the pill, I had brown discharge until today. It s been 3 days already. My next period is due next week. Around 27-31 since I had my last period last sept 29. What does the brown discharge imply? I also experience nausea , diziness, tenderness of breats.. Am I pregnant? I took PT out of anxiousness and it turned out negative. I d appreciate any response. Thank you. Doctor: Hello,NORDETTE is used as ECP in some country while no alternative drug is available. It gives protection more than 95% against unwanted pregnancy when it is taken within first 24hrs with proper dose.Next, you got withdrawal bleeding after 7 days due to effect of the pill and it is quite common & normal phenomena. Other possibility is implantation bleeding in the failure of ECP which can occur 1 week prior to period due date.Early pregnancy test is not significant. You have to wait for coming period and repeat the pregnancy test soon after missed period and after 1 week for confirmation.Based on test result, you must consult with your local doctor for next line management. Till then, you would maintain genital hygiene. Good luck."
},
{
"id": 69370,
"tgt": "What does lump on vagina which oozes yellow fluid indicate?",
"src": "Patient: i have a lump on the left lip of my virgina and it'd really round and big, sometime if i strain it it burst and release yellow creamish fluids. its comes and goes first time i had any of this was a cyst that got operated on and taken out now it comes and goes as at present sitting on a big on now Doctor: Hi,It seems that you might be having infected Bartholin gland cyst.Due to infection there is recurrence.Consult surgeon, if cyst is there, requires complete excision of cyst with sac.Ok and take care."
},
{
"id": 31891,
"tgt": "How to get rid of wheezing?",
"src": "Patient: My daughter is 11 years old & she suffers from veezing quite frequetly for the last 4 years. Initially alopathy medicine was given to her but not cured & then I switched over to homeopathy for quite some time. But whenever she gets veezing it is very severe & only remedy is nebulisation for 2 - 3 times. Is there any permenant cure for this. Am not able to see my daughter suffering from veezing like this & due to this she also misses her classes. Doctor: Hello, and thank you very much for your question. Wheezing that comes and goes such as you describe is usually caused by asthma. In most cases, there is not one single treatment to cure it and make it go away forever. However, many children with asthma get much better as they grow up. If allergies are causing asthma, then allergy shots can be very helpful. For most people with asthma, there are preventive medicines, often given as inhalers so that the medicine gets right into the lungs where it is needed, that help cut down how often and how severe the wheezing is. The hard part is getting a child to use a medicine every day or twice a day for months so that they won't get sick later. They feel fine today - why do they need the medicine? But lots of tests, and my experience in my practice confirm, that taking preventive medicine such as inhaled steroids every day can help prevent the wheezing from occurring as often, and keep it from getting as bad when it does happen.Your doctor may be able to help you develop an Asthma Action Plan, which helps you plan what medicines to give your daughter when she is feeling fine, what to do when she is starting to have some wheezing, or starting to get a cold that is likely to cause wheezing, and then what to do when things get severe.Also - people with asthma should make sure they get their flu vaccine and ask their doctor if they need a pneumonia vaccine.I hope this information has answered your questions. If you have further questions please check back with us. I hope your daughter is doing much better soon!"
},
{
"id": 196506,
"tgt": "What is phimosis?",
"src": "Patient: hi.. i m 25.. till now i was unaware that foreskin shud be retracted.. but as i found on net i tried strecthing.. in a month my foreskin retracted only wen flaccid and only in shower.. and in normal times the foreskin forms a small mass.. and twice i tried to make my penis erect by manually retracting.. i couldnt erect fully cos the foreskin jus behind the glans was like foldings... very tight.. i could pull foreskin fully over the shaft wen flaccid but wen i tried this with erect ..i cud not pull the foreskin over the shaft.. it formed the forskin gathered there... i cant retract the foreskin wen erect (without manually retracting wen flaccid and then making it erect) .. is this phimosis.. i m confused.. is this tight foreskin.. short frenulum and y the foreskin forms a mass wen normal.. ..my question is how i shud continue my strecthing exercise.. ???? Doctor: Dear userWe understand your concernsI went through your details. Phimosis is a congenital narrowing of the opening of the foreskin so that it cannot be retracted. Phimosis is a very common problem. In most of the cases you can clear the problem yourself. Phimosis could cause moderate to severe pain while having vaginal intercourse. Apply oil on your foreskin and on the glans. Move the foreskin forward and back slowly without hurting you much. Because the oil is lubricant, it will be easier than without oil. Do this forward and back movement 20 times several times in a day. Repeat the same every day for around one month. You will get the elasticity and you will be able to retract the foreskin completely. In case if there is no result from the above massage, you can always go for a minor surgical procedure called circumcision. If you require more of my help in this aspect, please use this URL. http://goo.gl/aYW2pR. Make sure that you include every minute details possible. Hope this answers your query. Available for further clarifications.Good luck."
},
{
"id": 183601,
"tgt": "Suggest treatment for bump on back of gums",
"src": "Patient: Good Morning, Last night, after brushing my teeth I thought I still had some food stuck in the back of my mouth. There wasn\u2019t any food left and just a small bump feeling on the back of my gums. I did have chips and salsa as an appetizer for dinner yesterday and it\u2019s very possible the chip slightly nicked my gums. I wake up this morning having the same feeling only to see a black blood clot the size of a small bean. It\u2019s not painful and isn\u2019t all that irritating under the circumstances. Is this something I just wait a few days to go away? Should I use mouthwash a few times a day as well? Could this be something more serious? Doctor: Hi,Thanks for posting the query, Bump on the gums could be because of trauma infection at the site, i would suggest you to get the site examined by the Dentist, take an IOPA x-ray of the region, at home take lukewarm saline rinses and antiseptic mouthwash gargles.Take multivitamin suplements. Take care!"
},
{
"id": 98775,
"tgt": "How to treat vascular dermatitis?",
"src": "Patient: Hi I have been using propygenta for the last year and a half for mole like allergies on various locations. some of the allergies have disappeared but some of them are still there. the dermatologist i visited told me that i have vascular dermititis and that i am allergic to chocolate, peanuts, strawberry, chips, frostings,etc. and that i should never eat them. Ihave discontinued eating those things and i have regularly used propygenta cream but i can see no development in my condition. Please advice me on what i should do. Doctor: hi,thank you for providing the brief description of your history.as mentioned in your history about the determination of your consciousness I am really amazed.well to understand further, you have been trying all things and now you feel there is no improvements.Why don't you try performing breathing exercises, meditation and general exercises. As the combination of three has an effect on the metabolic system and immune system.. What you need now is to reactivate the metabolic system and immune system. By which there will be a good drug absorption and the healing will be optimum. Also, after the symptomatic medication the body needs its own medical help, than can be possible by reactivation of the immune and metabolic systems.I wish you a good health.RegardsJay Indravadan Patel"
},
{
"id": 175804,
"tgt": "What is the treatment for headache and fever?",
"src": "Patient: My 4yr old is complaining of a headache, stomachache, and knee pains and has a fever of 101.4. She was fine all day then all of the sudden she asked to take a nap, which she never does, and when I felt her head she was kind of warm so I took her temperature and she had a fever. Doctor: Hi...Thank you for consulting in Health Care magic.Fever of few days without any localizing signs could as well a viral illness. Usually rather than fever, what is more important is the activity of the child, in between 2 fever episodes on the same day. If the kid is active and playing around when there is no fever, it is probably viral illness and it doesn't require antibiotics at all. Once viral fever comes it will there for 4-7 days. So do not worry about duration if the kid is active.Paracetamol can be given in the dose of 15mg/kg/dose (Maximum ceiling dose 500mg per dose) every 4-6th hourly that too only if fever is more than 100F. I suggest not using combination medicines for fever, especially with Paracetamol.Regards - Dr. Sumanth"
},
{
"id": 180209,
"tgt": "Why 13 year kid s shoulder popping out frequently?",
"src": "Patient: Yes, maybe you can. Ny 13 year old granddaughter has a shoulder that keeps popping out. She is very tall and is a dancer since the age of 3. She has been in physical therapy for months, but no improvement as yet. She will be seeing an upper body extremity pediatric specialist for the second time and I feel she may need surgery but know at her age they don t like to do this. I have heard about Marian syndrome and are worried she could be one that might have this. Her shoulder just seems so fluid and will pop out whenever she lifts her arm or rotates it. She was unable to do her solo in dance and complete her completion programs. I worked in orthopedics for years so know probably too much and it scares me. Please give me your insight. Nancy Doctor: Hi...You are probably right. Definitely this frequent shoulder popping out is abnormal. The possibilities could be -1. Hypermobility syndrome2. Marfans syndrome3. Ehler Danlos syndrome.I suggest you meet an orthopedician with the above tips and get this resolved please.Hope my answer was helpful for you. I am happy to help any time. Further clarifications and consultations on Health care magic are welcome. If you do not have any clarifications, you can close the discussion and rate the answer. Wish your kid good health.Dr. Sumanth MBBS., DCH., DNB (Paed).,"
},
{
"id": 92525,
"tgt": "Pain in abdomen. Ultrasound shows lipomas. What could be done?",
"src": "Patient: Hi I told my MD and a MD that I see here in a hospital. I am in Australia but form the US. I have fibro and renaulds but I am having a lot of pain in my abdomen. My MD ordered an ultrasound and they found between 20 to 30 limpomas. I have them on my arms and legs as well but they dont hurt like the ones inside. They have a lot of trouble with the medical system here and have shut down a lot of spaces in the hospital. I guess what Im saying is This will probably not be followed up. My mother and sister have Dercums.I would like to move home but now I have been told that with the illnesses I have that I would not be able to keep insurance with the new system. So first what can be done for the pain etc and is that true that getting Insurance would be impossible. I also had a mild pontine stroke 2 years ago which basically triggered the fibro which is bad. My life has been ruined. so is there anything that can be done and would moving back to the US be a big mistake? I hope you can read this my typing since the stroke has gone crazy Ive tried to correct what I could see lol. Never mind I just read the pay and get answer. I have no disability so I have no money sorry to waste your time Doctor: Hello there! Thank you for asking. If the lipomas are botherdom then they can be removed surgically. But the trouble is that you had a stroke history and anesthesia would be a risk. SO pros and conses need to be balanced and then a decision will be made what to do and what not to. Regarding your insurance you need to decide it for your ownself. One can not afford itself in US with out insurance. SO there is not much we can do about it from here excerpt wishing you well. But i suggest go where you can get treated. As health is wealthRegardsDr S Khan"
},
{
"id": 189125,
"tgt": "Lump in mouth, near gum, increase in pain, had a root canal. Why does it feel squishy?",
"src": "Patient: I have a small lump under the bottom of my gum on the inside of my mouth which when I wake up in the morning is about the size of a pin head , throughout the day it increases to about the size of half a pea , its not visable and can only feel it with my tongue.The lump is not really there when i wake up in the morning but grows throughout the day then shrinks again, not painless.Had root canal done yesterday and the pain my tooth has gone but the lump is still there - will this disappear over time or will it require any treatment - I have no idea how long it could have been there but the fact that it shrinks and grows throughout the day is a worry - any ideas what else I can do now I have had the root canal treatment - it feels squishy almost like a gland then sometimes it feels hard and is right below the gumline - should I be worried about this ? Doctor: Hi user thanks for posting your query. As per your histroy the swelling or lump may be because of perapical lesion if the lump is buccally(that is inner side of cheek) and if it below the tongue .it would be some small salivary gland or duct infection.anyway to rule out this you may need to take x ray for rct tooth and if it gland problem =saliogram.but with your qurey it most likely sounds like perapical lesion so may have to undergo re root canal and if requrid endoperio surgery or extraction depends on lesion on x ray and tooth present condition. Plesse reply back for further help. Thank you. Dr.leeladevi. dentist bangalore"
},
{
"id": 220573,
"tgt": "What causes raised body temperature during pregnancy?",
"src": "Patient: Hi, I am 20 weeks pregnant and having raised temperature (37 degrees celsius = 98.6 fahrenheit) while my normal temperature (before pregnancy, in the period before the ovulation was around 36.2-36.4 celsius (97.16 - 97.52 fahrenheit). I know that in the 1st trimester teh raised temperature is a good sign of hormones doing their job.... but how long the elevated temperature should stay? Is that temperature normal in week 20? or is it a sign of some infection? Thank you for your answer. Doctor: Hello dear,I understand your concern.In my opinion the raise of body temperature of about 0.1 or 0.2 degrees might not be significant.The normal body temperature is 38.4 degree C or 98.6 degree F .The body temperature is dependant on the basal metabolic rate(BMR).So considering the above values your temperature raise is insignificant.Anything more than 99 degree with any associated symptoms might be significant.So relax.There will be increased basal metabolic rate in pregnancy so it might also cause the raise of body temperature.Take healthy nutritional diet and adequate fluids upto 3 litres per day.Urinary tract infections(UTI) are common in pregnancy so taking adequate fluids will prevent them.Nothing to worry.Hope this helps.Best regards..."
},
{
"id": 226366,
"tgt": "Underwent tubectomy, baby dead after 5 days. What are the chances of good health after reversible tubectomy ?",
"src": "Patient: hi i m from india, we recently got family planning operation tubectomy on 19th Nov 2012, unfortunately after 5 days the baby dead due to some strong reasons. but we not want to go with reversible tubectomy, is this possible ? what are the chances and MAINLY HEALTH ABOUT THE MOTHER AND CHILD after reversible tubectomy. my email id = YYYY@YYYY Doctor: Hi thanks for your question. You had undergone tubectomy and now you want to know chances of conception after recanilization. After tubal recanilization there are 70% chances of becomming pregnant. but that depends upon many factors ,like type of ligation done (there are variou methods of ligating tubes) ,age of mother and many more.It is better to consult gynaecologist and discuss with him/her various aspects of recanilization. Hope this answers your question."
},
{
"id": 223968,
"tgt": "What causes irregular period flows when on birth control pills?",
"src": "Patient: I am on birth control and am sexually active without using a condom. For the lady 3 months I have been getting my period a week early. The first month my period lasted 11 days, the second month my period was really light and lasted 2 days and this month I just started my period a week and a half early. What is wrong? Doctor: Hallow Dear,In fact birth control pills are supposed to and used to regularize the menstrual cycle and control the menstrual bleeding. There are mainly three types of birth control pills: Oestrogen dominant, Progesterone dominant and balanced. It is not clear which pills you are taking. From your history it appears that you are an Oestrogen dominant woman. These women usually have heavier type of bleeding. If such women start consuming Oestrogen dominant pills, their menses would be still heavier. They should take Progesterone dominant pills. Hence, I would advise you to consult your Gynaecologist and get prescription of appropriate contraceptive pills for your phenotype, after thorough clinical examination. Every packet of contraceptive pills have fixed number of pills - either 21 or 28; either way, only 21 pills of them are hormonal pills. After finishing these hormonal pills, within 4-7 days the menses appear. If your periods are starting earlier, may be your original periods were rather longer. If in spite of the contraceptive pills, your periods are becoming irregular, you will have to get investigated for your hormonal profile - particularly Thyroid function. Please report to your Gynaecologist for assessment and management. I hope this helps you. Dr. Nishikant Shrotri"
},
{
"id": 115921,
"tgt": "How can pyruvate kinase deficiency causing hemolytic anemia be treated?",
"src": "Patient: Hi, may I answer your health queries right now ? Please type your query here...my.daugter has pyruvate kinas e deficiency, an inherited enzyme deficiency that causes hemolytic anemia. She has had her spleen removed eliminating the need for blood transfusions (except for during her pregnancy. Are hickiesd Doctor: Hello,I understand your concern.I am Dr. Arun Tank, infectious diseases specialist, answering your query.Pyruvate kinase is enzyme. It is necessary for the integrity of the red blood cell wall.The person who is in the deficient state of enzyme will not sustain the integrity of the cell wall.It is inherited disease and there is no treatment for it. But transfusion can help replenish enzymes and help improve symptoms.Such types of patients should avoid factors which causes red cell lysis.I will be happy to answer your further concern, you can ask me on bit.ly/DrArun. Thank you.Dr Arun TankInfectious diseases specialist."
},
{
"id": 107016,
"tgt": "What does pain in the lower back indicate?",
"src": "Patient: I have some unusual pain in lower back. I think from kidney. Recently I took cholesterol medicine after years of break. I remember that a few years ago I had same problem , same discomfort. I thought is something not right about my kidney. I felt that I am allergic to statin Recently, My urin test showed some blood Shall I be very concern ? Please advice. Thank you very much Doctor: Dear patient statins are known to cause muscle damage in few patients.That may lead to increase protein levels in blood which in turn goes to kidney for excretion and hence may affect your kidney. Chances are few but since your urine test showed blood this needs to be ruled out. I would advise visit to your physician for further examination and investigation like Ultrasound of KUB. meanwhile stop taking statin if it's possible."
},
{
"id": 22592,
"tgt": "What causes lightheadedness and heart flutters?",
"src": "Patient: I am 28 5' 9\" and weigh 180.6 lbs. I have had history of panic attacks and chest pains. been told nothing is wrong w me. For the past year I have had what feels like someone trying to revive me w a defibrillator. I just took my BP and it was 106/75 and I was feeling light headed and I was having heart flutters and i am sitting down is that ok? Doctor: Hi,All these symptoms looks psychological, most probably panic state. Your bp is also fine. You try ignore these problems and try some relaxation techniques like yoga. Also maintain good physical activities. Breathing exercises and traditional breathing retraining involves slow, abdominal breathing exercises. There are many ways of psychotherapy which will be taught to you in person by psychiatrist. Medicines are also available like fluoxetine which you can get prescribed from your psychiatrist. At this point of time, no need to worry.Hope I have answered your query. Let me know if I can assist you further.Regards, Dr. Sagar Makode"
},
{
"id": 83759,
"tgt": "Does cypon and feris syrup has any side effects?",
"src": "Patient: Dear Doctor, My son is 9 yrs 8 mths. and weighing 27.5 kgs. Since he is underweight and frequently prone to cold and cough (due to allergy) the doctor has prescribed Cypon and Feris, 2.5 ml two times a day each. Does these syrup has any side effects ? Thanks. Doctor: Hello,Cyphon syrup may cause nausea, vomiting, dry mouth, dark stools and drowsiness. But don't worry every body may respond differently. Les 5% may get these side effects.Hope I have answered your question. Let me know if I can assist you further. Regards, Dr. Penchila Prasad Kandikattu, Internal Medicine Specialist"
},
{
"id": 160925,
"tgt": "What causes red itchy blister like spots on body?",
"src": "Patient: My son is 22 months years old.He has been having these spots showing up everywhere on his body.They kinda look like blisters and they sometimes turn red from him itching them.Ive tried hyrocortizone cream & aquafor.Do you maybe have any idea what it could be? Doctor: Hello, It could be a folliculitis or impetigo. It will be better to attach a picture in the follow-up query. You can add, topical antibiotics like Neosporin also. Hope I have answered your query. Let me know if I can assist you further. Take care Regards, Dr Shinas Hussain, General & Family Physician"
},
{
"id": 15143,
"tgt": "Developed rashes, redness and blotchiness, no itching, cough. Tried benadryl. Is it an allergic reaction?",
"src": "Patient: My son has been developing a rash that is red and blotchy, but not raised. He says it does not hurt or itch and it goes away using children's benydryl. He also has had congestion and a mild cough lately. The rash is only on areas not covered by clothing. It is also on his ears. It seems aggravated by headphones, is it possible that a brand new pair of headphones is causing an allergic reaction? Doctor: Hi yes there is a possibility of brand new earphones causing the rash but usually allergic rash is itchy.it think in ur case it can be due to sun allergy or milaria due to excessive sweating .even then rule out other viral rashes where itching is minimal.use of benadryl tablet along with vitamin c chewable tablet can cure the condition."
},
{
"id": 116098,
"tgt": "How to normalize RBC count?",
"src": "Patient: hi my urie test show that my RBC is 10-15Hps this blod in my urine is ocurance and my doctor told me that it it prostitis after undergoing a cestoscopy. he gave me antibiotics bud after 40 days taking antiabiotics my urine test show always RBC is 10-15hps. Doctor: Hello,some red blood cells in patients with enlargement of the prostate can be expected. You've mentioned cystoscopy which means that your urologist has already checked your bladder for potential lesions that cause bleeding and I suppose he's checked the kidneys as well. If the kidneys have not been checked, they should. Otherwise you should discuss with the urologist for further testing of the genito-urinary system.I hope I've helped!If you'd like more information, please let me know.Kind Regards!"
},
{
"id": 50097,
"tgt": "Tumors on each of the kidneys. Advised for a stress test. Suggestion?",
"src": "Patient: I am suppose to have surgery but doctor wont clear me unless I have a stress test, he said i have 2 tumors each one on the kidnesy I am afraid, had cat scan done, he wrote I have reviewed with patients results and rec renal u/s to eval lesion. d/w patient rec pelvic Angina of effort (413.9) multiple risk factors including dm, lipids, htn and change in pattern where she now had with stress Doctor: in case of a high risk surgery due to presence of cardiac risk factors, surgeons and anaesthetists need a clearance or approval from physicians or cardiologists before doing surgery.same is a case with you.you can consult a cardiologist as advised and if a stress test is required, a dobutamine stress test can be performed using infusion of a drug named dobutaine to increase heart rate. in this test you don't have to walk on a treadmill.alternatively your cardiologist can examine you and have a woed with the surgeon or anaesthetist.of course you will have to sign a high risk consent showing that you understand the risk posed by cardiac disease or risk factors if present."
},
{
"id": 69503,
"tgt": "Why there is a lump on my chest with offensive smell?",
"src": "Patient: I have a very tiny lump on my chest that sits under the skin. If I touch or squeeze it, it gives off an offensive smell. I have an implant in my right breast as was born with a deformity called Poland Syndrome. Could the smell have any connection to the implant? I have had this for approx 3yrs Doctor: Hi,The smell is likely to indicate an infection. If this is close to the implant then you should see a doctor as there is a risk that the implant may become infected. If you have had this problem for a number of years, this may indicate that a sinus or tract has formed under the skin. This may need to be treated with antibiotics or surgically excised. Regards,Dr K A Pottinger"
},
{
"id": 201795,
"tgt": "Any suggestion for numbness and no growth in penis?",
"src": "Patient: Hi, I m a fourteen year old boy who is going through puberty and haven t has Any penis growth at all since birth, I m not fat and have Every sign of puberty as body hair, facial hair, acne and body oddurs, no muscle growth as I ve noticed myself aswell is my penis numb and the only sexual sensation I have is when i ejaculate. I m suspecting low testosterone levels to be the case bit I want to be sure. It might be because of genetics because my mother is asian although my father has a big penis, yes I ve seen it. when he gets out of showers. I don t Knowles how to confront him about that if low testosterone is why. Please answer /A confused teen Doctor: DearWe understand your concernsI went through your details. I suggest you not to worry much. Please understand these type of negative sexual thoughts can only ruin your mental health. Let me assure you, big penis has nothing to do with increased sexual pleasure or female satisfaction. Those can happen only in porn movies. Even a 2 inch long erect penis can satisfy a woman. Larger penis cannot give more pleasure because, female vagina has pleasure nerve endings within the first two inches. Please don;t over burden yourself with anxious thoughts about sexual maturity and concentrate on your education and career.If you require more of my help in this aspect, Please post a direct question to me in this URL. http://goo.gl/aYW2pR. Make sure that you include every minute details possible. I shall prescribe the needed psychotherapy techniques.Hope this answers your query. Available for further clarifications.Good luck."
},
{
"id": 107696,
"tgt": "What causes intense pressure and popping sound in the lower back?",
"src": "Patient: I am 16 and I have smoked weed nearly 2 years, in the last week Ive been getting intense pressure and popping sounds in the lower back of my head and upper neck.The pressure gets more and more intense as I come off the high. It will get worse when I look at a screen for a while or simply start thinking about it. How can I avoid this pressure, I have a slight addiction to pot physically and if slowing it to less then once a day would help I d really like to know. Doctor: Hello, I have studied your case.Most probable reason for your symptoms could be postural problem as you may be sitting for long time and working on computer/table for long time.When such patient comes to my hospital we usually take x ray spine or if required MRI to see for any nerve compression.Medication like methylcobalamine with muscle relaxant and analgesic will reduce pain; you can take them consulting your treating doctor.Some exercises which can be done after pain has reduced include- Spine extension exercises, Lying on your stomach flat lift leg 6 inches from ground, do it for other leg.Now lift both hand and leg simultaneously, 6 inch off the ground and stayPosition for around 10 breathes. Core stabilizing spine exercises will help.You may consult physiotherapist for further guidance. He may start TENS, or ultrasound which is helpful in your case.I will advise to check your vit B12 and vit D3 level.Hope this answers your query. If you have additional questions or follow up queries then please do not hesitate in writing to us. I will be happy to answer your queries. If you find this answer helpful do not hesitate to rate this answer at end of discussion.Take care."
},
{
"id": 137193,
"tgt": "Suggest treatment for acute pain in the elbow",
"src": "Patient: I have acute pain in my elbow that began 24 hrs ago. It is now at an 8 on 1to 10 pain scale. I can not move it without pain and there has been no trauma . I have taken three doses of advil in the last 24 hrs with very little relief. Doesn t appear to be any swelling Doctor: Hi there.This could be due to possible arthritis. It could be septic, rheumatoid or due to Gout. I advise you to perform cold soaks over the elbow. Consult with your Orthopaedic surgeon as soon as possible."
},
{
"id": 69881,
"tgt": "What does small, hard movable lump on side of jaw indicate?",
"src": "Patient: I have a small, hard moveable lump on the side of my jaw. I went to my doctor 2 months ago and she told me to just keep an eye on it. 2 months are up and it is still here. It doesn't hurt, only slightly occasionally before I eat. It doesn't make my jaw swell or anything. Doctor: HI.With the history you have provided , this looks to be a lymph node. A simple FNAC= fine needle aspiration cytology can confirm as also an ultrasonography o a CT scan is to be done if the FNAC reports shows problematic findings."
},
{
"id": 207893,
"tgt": "Suggest treatment for nervousness while talking to people",
"src": "Patient: My problem is that I get so nervous talking to people and talking in front of people And when I talk sometimes I stutter and I get so embarrassed and blush I start shaking when I have to talk in front of people and my heart starts beating so fast even when I m making a phone call to anyone Doctor: DearWe understand your concernsI went through your details. I suggest you not to worry much. This sort of complaints are very general. Once you get yourself accustomed with the situation, the problem goes away. never associate this problem with anxiety and shy away. Get into that. Specific phobias are always treated with involvement in that phobic area. Do indulge in such activities and ignore the nervousness.If you require more of my help in this aspect, Please post a direct question to me in this website. Make sure that you include every minute details possible. I shall prescribe the needed psychotherapy techniques which should help you cure your condition further.Hope this answers your query. Available for further clarifications.Good luck."
},
{
"id": 92427,
"tgt": "Diagnosed with GERD and Interstitual cystistics in bladder. Suffering from abdominal pain and cramps so hospital visit is required?",
"src": "Patient: Hi! I'm suffering from pain in the abdomen and stomach area. It all started a week ago early a.m.. I was sitting on the toilet when all of a sudden I sneezed very had and sound like I tore or ruptured something in my stomach also forced stool when I didn't feel like I needed to go. My stomach cramps and with pain very bad. Do I need to go to the hospital or see my Dr.? I also have been diagnosed with GERD and Interstitual cystistics in the bladder. I have had diarrhea since this has happened. Doctor: HIThank foe asking to HCMI think no need to go the doctor, the force which exerted due to sneezing might have pulled the abdominal muscle or smooth muscle of gut might be affected just take a little rest this will come around for sure, your other problem of diarrhoea will also be fine and nothing to worry for GERD you might have taken treatment, if not then Omeprazole 40 mg would be the drug of choice and can be taken at evening time for six weeks hope this will help you have nice day."
},
{
"id": 116840,
"tgt": "What causes high esr and swollen lymph nodes?",
"src": "Patient: My mom had very severe anemia Hg 5.8, She was admitted to the hospital, they did a no.of tests on her and found her ESR results were high, but the immunoglobins test was negative. she had her physical examination of the lymph nodes in her chest and abdomen and they were normal, doctor had seen some swollen lymphs in the xray, could she be suffering from lymphoma. her iron is 2. Doctor: Welcome to HCM.Severe anemia itself a cause of high ESR. On the basis of swollen lymph node we can not diagnose lymphoma. Tuberculosis can also cause chronic anemia and high ESR with enlarged lymph nodes. You should not be worry. I suggest you that your mom really need a full work up to confirm diagnosis.You should also mentioned complete blood count report including peripheral blood film with duration of illness.Thanx for query."
},
{
"id": 32166,
"tgt": "Can green eye be due to sinus congestion?",
"src": "Patient: My son is 4 years old and has had congested sinuses with green snot and heavy mouth breathing for a couple days. He now has green eye goo that started in one eye and is now in both. Is this part of the sinus congestion or is it possibly an eye infection? Should he get antibiotics? Doctor: Hi Dear,Welcome to HCM.Understanding your concern. As per your query you have green eye yes it could be due to sinus infection as sinuses are separated by a thin sheet of bone from the eyeball and sometimes infection from the sinuses can easily spread into the eyeball due to incomplete barrier. I would suggest you to put decongestant nose drops like otrivin as it will cure sinusitis and due to which infection will be treated. Visit ENT specialist once and get it examined. You should go for nasal sinus X-ray as well. Do warm saline gargles and take steam inhalation. You should take tablet Levofloxacin as well.Hope your concern has been resolved.Get Well Soon.Best Wishes,Dr. Harry Maheshwari"
},
{
"id": 87539,
"tgt": "Suggest treatment for pain in stomach due to inflamed esophagus",
"src": "Patient: Hello my name is Ruth a couple months ago I had an endoscopy done because I been having stomach pain.. Certain things I eat become painful .. My results came back normal..my biopsy came back that I have inflamed esophagus.. My pain is still there what can I do to make it feel better Doctor: Thank you user for your query.There are may causes for inflamed oesophagus mainly Candida, Herpes, GERD etc.All you have to do is to start taking any antacid to avoid acid production such as heart burn. An antibiotic or anti fungal or any antiviral can also be taken but under physician`s guidance. Try to gargle after eating anything as this reduces inflammation.While treating for oesophagitis you should follow certain stepsAvoid spicy oily, hard foods. Avoid alcohol, and acidic fruits like grapes, oranges, tomatoes. Instead try eating soft mashed food as this will help in healing of your oesophagus. Liquid diets can also be taken.God Bless"
},
{
"id": 25766,
"tgt": "What does heart rate 50-60 bpm indicate?",
"src": "Patient: Hi - my heart rate is typically in the 60-80 range. Lately it has been in the 50 - 60 range. No lifestyle changes. I am on the treadmill or bike 3-4 times a week and work out with light weights 3x week for 1/2 hour at a time. All other health items are ok. I am 67 and my blood pressure has been ok. Weight is normal. I take blood pressure and cholesterol medicine. Any ideas? Doctor: Hello , As I start I would like to reassure you that your pulse rate is perfectly normal , and rather it's best to have pulse rate between 50-60 . The reasons may be 1) normally as we age the pulse decrease because metabolism of our body slows down .2) As you are active , the heart is well trained and hence the pulse would go low as such heart can effectively pump more blood with each heart beat 3) many medication for hypertension particularly beta blocker can decrease the pulse rate . It's just prudent to keep a tab on the pulse rate by doing ecg once in 6 months to make sure that the rhythm is normal . Let me know if any clarification needed and I would be happy to help . Regards Dr Priyank Mody"
},
{
"id": 185036,
"tgt": "What causes pain in the lower right jaw in the wisdom tooth area?",
"src": "Patient: i had a drug induced seizure from an Accidental ultram overdose. since this event i have had the worst pain in my lower right jaw back where my wisdom teeth should be. i am 21 5'11 over all healthy male, could i have done some serious damage to myself during this seizure??? Doctor: Hello thanks for consulting at hcm.the accidental seizure could have resulted in tongue bite--ulcer.since you are having jaw pain it could also be due to impacted teeth..due to wisdom tooth..get an iopa radiograph-to check for type of impaction..plz consult a oral physician to check for any ulceration secindary to biting during seizure... Hope it helps take care..."
},
{
"id": 156020,
"tgt": "What causes spasm in lungs while having breast cancer?",
"src": "Patient: Last night I woke around 4 am because I had what I thought was a spasm in my lungs. It hurt so badly on my right side, it went up to my jaw and actually my gums hurt as did my ears. I thought I had water in my ears they hurt so badly. It lasted approx 1/2 hour and I was scared to death that I was having a heart attack, but the pain was on my sternum and towards the right. I also have breast cancer, been going through chemo (Halaven). Had a oxgen test today - was 100%; had chest x-ray- nothing showed. Do you have an idea of whats going on. Doctor: you probably had an attack of bronchospasm last night. you need to take bronchodilators at that time to get relief. Such attacks are not common either due to cancer or due to chemotherpy. You need to get pulmonary function tests to check lung reserve. Halaven is not know to cause such side effects. You probably had an allergic reaction. re check your diet and environment and list of medications carefully and try to identify the allergen which has precipitated the attack. be careful to avoid exposure to that allergen again"
},
{
"id": 82657,
"tgt": "Does lupus cause low blood count?",
"src": "Patient: MY MOTHER HAS SEVERAL HEALTH ISSUES. DEFIB/PACEMAKER, LUPUS, LOW BLOOD COUNT, AND LAPBAND REMOVAL DUE TO COMPLICATIONS (RIPPED A WHOLE IN HER STOMACH). THE LOW BLOOD COUNT IS OUR CONCERN AS OF NOW. SHE HAS BEEN PASSING OUT AND BEEN TO ER 2 TIMES IN LAST 6 WKS DUE TO HERE FALLING AND INJURYING HERSELF. HER COUNT IS AT ITS LOWEST SO FAR 7.0. 6 DAYS AGO IT WAS 8.5 AT ER THEN 2 HRS LATER DROPPED TO 7.2. CAN LUPUS MAKE YOUR COUNT DROP DRAMATICALLY? Doctor: Yes lupus is known to cause leucopenia , thrombocytopenia and even anemias so definitely its because of that only , you should consult your rheumatologist for the same."
},
{
"id": 62281,
"tgt": "What causes pea size lump under right arm?",
"src": "Patient: I discovered a pea size knot under my right arm, when I press on it, it feels as if the mass may be the size of a grape. I have had cancer twice. At the beginning of this year tests have been very good and the doctors said they need not see me for one year. I am frightened. Doctor: Hi,It seems that there might be having enlarged axillary lymph nodes giving rise knot and and lump.As there is h/o cancer, keep watch on it and if size increased or becomes painful, get it removed and go for biopsy.Ok and take care."
},
{
"id": 115207,
"tgt": "Suggest treatment for hemolytic anemia",
"src": "Patient: HelloI have hymolytic anemia and am taking rutuxin.I added Imunoglobulin[spelling?] to suppress the spline this week.Since that application I have felt weak/have headaches and feelings of nauceus and am jaundiced Is this the reaction to this procidure?Again please for give my spelling!Thank you for your helpHarriet Doctor: Hi,You seem to have autoimmune hemolytic anemia,looking at your treatment profile. Jaundice and fatigue due to anemia are part of the disease itself and will get better once the disease is in control. Headache may be related to disease or treatment. Please see your doctor and get checked including a eye examination. Get well soon!"
},
{
"id": 129786,
"tgt": "Suggest treatment for weak knees",
"src": "Patient: I had a car accident in march and since the accident my left knee gives out every once in awhile it has created many problems like spraining my ankle the newest injury happened friday my knee gave out and i went down on my left knee really hard and in tile needless to say the is a really dark bruise with dark lines coming from it im worried about what it could be Doctor: Hi, there!Its recommended to make MRI scan for Your knee to see if anterior cruciate ligament (ACL) is torn.ACL refers to knee stability when pivoting, bending etc.Surgery will be performed if ACL is torn or physical therapy including thigh muscle strenghteningI hope this helps"
},
{
"id": 190581,
"tgt": "Swollen lymph nodes with canker sores, pus on gums, difficulty in swallowing and chewing. No improvement with amoxicillin, vitamin C. Suggestion?",
"src": "Patient: Hi. I am from the Philippines. It s exactly one week now that my lymph nodes are swelling. Five days ago, crank sores started to built inside my mouth. I have one under my tounge, and then others look like pus in the gums. I have pus on my upper and lower gums. My tounge started to feel painful. My denstist recommended me to take amoxicillin 500 3 times a day and vitamin C .. she recommended also Pyralvex solution. I am also gargling Bactidol. It had been a week now that I am not feeling well. I eat almost nothing for it s painful to swallow and bite. please help. Doctor: hi. as per the information provided by you , i can see that you are having acute periodontitis where there is acute infection in your gums. i would suggest tetracycline instead of penicillin , as tetracycline is more effective in soft tissue infections, the treatment for periodontitis should have been started also. i dont think that your present dentist is capable of solving your problem. she should have started the treatment long back. consult a periodontist. he will guide you to perfect dental health after properly diagnosing and doing periodontal surgery and saving most of your teeth. take care."
},
{
"id": 151560,
"tgt": "Sudden fall on the ground, mild seizure, disorientation, trembling after the episode post a glass of wine. What could be the cause?",
"src": "Patient: my partner was drinking a glass of wine (the first of the night) and started coughing, then dropped his glass, then fell on his back and looked like he was having a seizure. he is not a heavy drinker, he just happened to be drinking wine when this happened. it only lasted a few seconds but he looked dazed and his right arm pulled up and shook slightly Doctor: Hi welcome to healthcare magic presently if your partner is asymptomatic and doing well then there is no cause of major concern. Cut down on his drinking and possible complete abstinence, if he develops one more such symptom then consult a physician. Take Care"
},
{
"id": 36463,
"tgt": "What could clogged breathing issues suggest?",
"src": "Patient: For months now I have had what I call an echo in the head accompanied by what feels like a stuffiness that affects my speech [ability to enunciate] ..I can hear my breathing. It happens daily off and on a few times a day, varies in severity and has not been helped with doctor recommended nasal sprays, and a course of steroids. Any idea what this can be? Doctor: welcome to hcm, the cogged breathing and hearing of your breathing may be due to obstruction in your respitatory tract, or upper respiratory tract infection if it is not cleared by using nasal sprays you might need minimum investigations like chest xray and complete heamogram and you may need some antiobiotics. hope this explains"
},
{
"id": 127443,
"tgt": "Is a podiatrist the right specialist for foot pain?",
"src": "Patient: My husband is having a high level of pain in his right root, right calf, and down the outside of his leg to the foot. He foot has tingling, numbness, and is cold to touch. He has an appointment with a foot Dr tomorrow. Is this the best choice for a dr speciality for his sysmtoms? Doctor: Hello and Welcome to \u2018Ask A Doctor\u2019 service. I have reviewed your query and here is my advice. Podiatrist is more concerned with infected foot as in diabetes. The best possible specialist for your condition is a neurologist because his symptoms are more likely of neurological like numbness and tingling. Hope I have answered your query. Let me know if I can assist you further."
},
{
"id": 91367,
"tgt": "Could abdominal cramps and spotting be due to having cysts in cervix?",
"src": "Patient: Hi, I have been in hospital there other dayn with severe lower abdo cramps, I discharged myself as I wanted to be at home, but today altho the pains are not as strong I have just lost a quite big bit of whity creamy coloured sticky stuff, I actualy felt it come from me, just to try and explain how much was there, I am 19+4 days pregnant, should I be worried. And also had bits of spotting but been told this could be down to having cysts on my cervix? Doctor: hi,welcome to healthcare magic forum.yeah,you may get spotting due to the cervical cyst,but in my opinion as you are pregnant so better get an USG done to know about the condition of uterus,fetus and placenta.there may be some other problem inside also.so get it confirmed as soon as possible and consult your doctor.good luck."
},
{
"id": 116691,
"tgt": "What is the treatment for hemochromatosis?",
"src": "Patient: I was having phlebotomies for eight years due to Hemochromotosis but my feritin has been normal for the past five years and the phlebotomies have stopped. Does this mean I no longer have iron overload? My understanding is that hemochromotosis is forever. Last January my feritin was 38. Doctor: Hello welcome to HCM. I have read your question and understand your concerns. Yes . Normal serum ferritin levels means you have no iron overload. Usually hemochromatosis takes at least 20 years to develop after excessive storage of iron and phlebotomy is the treatment of choice. So, now if you have no cause for iron overload it will again take this much time to redevelop ideally. But as now you are aware you can take precautions to avoid iron overload. Do yearly basis whole iron profile and show it to your haematologist. Now you don't need any phlebotomy and you are cured of hemochromatosis but excessive iron load will damage liver and may cause diabetes. So Go for whole liver profile and blood sugar levels including HBA1c report. Thank you. Have a nice day."
},
{
"id": 23542,
"tgt": "What causes palpitations associated with sleep?",
"src": "Patient: i have palpatations mainley assocciated with my sleep. I have been going to a cardioligist for years, and have had many tests done to no avail. I have worn a 24 Hr halter 2 times with no results I had a spontanious lung collapse 15 years ago followed by a thoracotomy at the time to repair the damage When i am awoken by these palpitations and feel my heartbeat it is normal or perhaps slightly elevated and completley out of rythm with the palpatations . MY cardioligist has been trying for some time to figure these palpatations out Does anybody have any feeling about this Thankyou Doctor: Hello!Welcome and thank you for asking on HCM!Regarding your concern, I would explain that your symptoms could be related to a sleep disorder or sleep apnea. I recommend performing a polysomnography. It can register the brain and heart electrical activity coupled with your lung oxygenation. If sleep apnea is detected, it should be treated properly. Another possibility to take into consideration is gastro-esophageal reflux. I recommend taking some antiacids before sleep. If your situation is improved by an antiacid or PPI (omeprazole, lansoprazole), it would be indicative of gastro-esophageal reflux, which could trigger this symptomatology. You should discuss with your doctor on the above issues. Hope you will find this answer helpful!Kind regards, Dr. Iliri"
},
{
"id": 31018,
"tgt": "Suggest treatment for pain in chest and cold",
"src": "Patient: I've been getting pain in the centre of my chest for sometime now. I've always suffered slight bouts of raynauds in my right hand when cold for years but recently (last week) I had a rather bad case that was White finger on both hands all fingers. I went to docs he prescribed me nifedipine asked about my chest (no lung or heart probs detected for ref) he looked at my hands asked if my finger nails had always been like it. No splits or cracks or problems i see other than i bite them, Then suggested that some blood tests might be needed to look at my autoimmunie system. This has left me very confused Doctor: Dear , This is Dr. S. Tomar from healthcaremagic.com's medical team and I ll be your medical advisor for this question. First of all thank you for sharing your concern with us and showing trust on us.Any chest pain first should be ruled out for any heart related issue and I would suggest you to have ECG , ECHO , chest x ray done to rule out any serious issue . I guess that your physician was more concerned about the clubbed nails rather than white fingers , which is a characterstic finding in some heart and lung diseases which should be ruled out with mentioned further testings . As far as reynaud's phenomenon is concerned so you can start nifedipine .This is my initial response and discussion is open until you have any query or need further clarification.I wish you good health .Thank you for your question and feel free to contact further for any question .sincerely,Dr. Shardendu Tomar"
},
{
"id": 175756,
"tgt": "What causes severe cough in a child?",
"src": "Patient: Hello my daughter is 3 half months. Hes has a terrible cough barking wispy sound. I took her to th3 doctors monday cheast is clear m but it seems to have go worse. Should ibbe worried and do u suggest any medicine witch might help with her cough thanks stacey Doctor: your baby likely to viral bronchiolitis ,which needs only symptomatic treatment if you consult many child specialist you will be confused please consult only or if you doubt consult one more ok"
},
{
"id": 151235,
"tgt": "Slipped, fell. MRI report shows L4-L5 diffuse disc bulge, herniation, facet hypertrophy. Complete cure possible?",
"src": "Patient: Hi, Dr,: I slipped & fall 6 months ago. I just did MRI Sep. My MRI report shows my L4-5 there is a diffuse disc bulge with a small left paracentral/foraminal disc herniation with mild facet hypertrophy . This causes left-sided lateral recess stenosis with compression of the descending left side s1 nerve root . Can this problem completely heal? Doctor: Thank for the query. The first issue to be addressed is if the MRI findings are due to the fall or if that were already there before the fall. Facet hypertrophy may be constitutional or due to degeneration. Disk bulge and herniation may be caused by trauma, but it could be already there - did you ever had any sort of left sciatic pain? True disk herniation usually causes an inflammatory reaction that may lead to its resorption and cure. However, if their parent disk is already dessicated, they won\u00b4t resolve. Facet hypertrophy is a permanent condition due to its origin. Just a comment on your MRI report - it is unlikely that a L4-5 disk herniation might cause S1 nerve root compression. This is because, within the L5 lateral recess, resides the L5 root, not S1 (just a minor mistake) Best wishes,"
},
{
"id": 94155,
"tgt": "Pain in the lower abdomen on the right side. Nausea and fatigue with pelvic pain. No appendicitis. What else?",
"src": "Patient: I have pain in my abdomen on the right side sometims left it swells up and I am unable to lean to the side that is swollen. I become nautious and tired. this happens every month around ovulation and is starting to get worse. It is also accompanied by pelvic and groin pain. I have been to several doctors no appendicitis or kidney infection or gastrointestinal problems. Pain is becoming severly worse to where at times it hurts to move,.. Do you have any idea of what this might be? please help Doctor: Hi welcome to Health care magic forum. Thanks for choosing H.C.M.F. According to your description the pain is there only during the ovulation period and it is going to be worse upto the period (i have assumed) As you said there is no appendicitis or kidney infection, or gastro intestinal problems. So it appears that you are missing gynacologist , The pain appears to be dysmenorrhoea due to pelvic inflamatory disease, or p.c.o.d.etc. So i advise you to consult a gynacologist for diagnosis and treatment. You may need to have trans veginal ultrasound scanning, besides other routine tests for confermation. Wishing for a quick and complete remission form the disease. I welcome to call me for any doubts in this matter. best regards."
},
{
"id": 103568,
"tgt": "Had respiratory lung infection. Why am I short of breath, have cough. Will inhaler help?",
"src": "Patient: hi, im 43 year old female and approx 3 months ago i had a respitory lung infection, which has taken a long time to get better... just recently i have been becoming increasing short of breath after taking coughing fits and talking. i have tried my husbands inhaler and this does note help, but when i take a puff of his steriod inhaler it calms me down. Doctor: these are allergies and inhalers are preventors steroid help but it should be taken restrictlyyou have allergies and antibioyics trigger themstart with montelucast 10 mg bdbronchodilator syp tfdantacid stp tdsdo for 3 wkafter that go to allergy doctor anf get treatment for curing"
},
{
"id": 85757,
"tgt": "Can duramine affects the pregnancy tests?",
"src": "Patient: Hi, I ve been on antidepressant pills for 12-18 months and stopped taking them about 4 weeks ago, and I am taking duromine for a month now. I also had my tubal clamps removed 7 months ago, this was not a reversal it was due to savere abdominal upper and lower pain. I have now in the last 7 days done about 10 pregnancy tests and I have had 4 come back with faint lines and the others negative. So I m really unsure what is going on. I also have had a lot of continuous discharge, sometimes clear sometime with a snit kind of colour. I just went to toilet and had a long string of discharge that I ve had to manually get out. (Yuk yes I m sorry). So I guess I m wondering if there could be a chance that the duramine is having an effect on the home pregnancy tests ? Because I was told that the tubal clips made my tubes dead and I can t and won t fall pregnant. I m a mother of 6 kids and am 32 years old after some answer. Thank you Lisa Doctor: Hello, Duromine would not be expected to affect a pregnancy test. While it would be unlikely to get pregnant after your procedure, it would not be impossible. However, since you are mostly having negative tests, I would suspect you are not pregnant. If you are, the tests should be more consistently positive as time goes on, so I would check again in a week, and if you are still getting negative then, it would reliably rule out pregnancy. Hope I have answered your query. Let me know if I can assist you further. Take care. All the best. Regards, Dr. Aaron Branch, General & Family Physician"
},
{
"id": 165493,
"tgt": "Does low grade fever in children need medical attention?",
"src": "Patient: My 1 year old son is running 38.9 celsius fever since yesterday evening no other symptoms at all except farting with bad smell. Panadol is not taking the fever down except after 2 hours, I tried neurofen this morning and it helped the fever go down in half an hour. Doctor: Hello and Welcome to \u2018Ask A Doctor\u2019 service.I have reviewed your query and here is my advice.If your child is not lethargic, feeding or drinking well, having normal urine output then you may give Paracetamol or Ibuprofen for fever.However, if child is lethargic, drowsy, not feeding, decreased urine output you may need to consult a doctor.Hope I have answered your query. Let me know if I can assist you further.Regards,Dr. Khan Shoeb Mohammad Sher Mohammad"
},
{
"id": 102907,
"tgt": "Child having asthmatic symptoms. Worse during early fall. Diagnosed as non-allergic rhinitis. Taking Flonase. Is it panic/anxiety reaction?",
"src": "Patient: Hi , My 10 year old has been having asthma like symptoms - nasal congestion, shortness of breath, chest tightness, tingling of feet and fingers. His symptoms get worse during weather change from Summer to Fall. He is ok going from Winter to Spring and to Summer. He thought it was a panic attack and went to the ER and they gave him asthma treatment but it did not help. What helps is him relaxing and calming his mind down when we gets the breathlessness. Dr says it is not asthma. Allergist thinks it is non-allergic rhinitis so he is taking flonase. Otherwise he is is just inhaling steam w vicks. It is hard to figure out if it is a panic/anxiety reaction or he has a real physiological issue. Pls advise. Doctor: HIThank for asking to HCM I really appreciate your concern, looking to age of kid panic attack is less likely even if it there then this kind of panic attack induces asthmatic attack which very typical, but the nasal congestion and other upper respiratory elements is impossible, it may be a kind of allergic constitution and need to be treated accordingly, encourage the kid for drinking water give him pure honey before going to bed, it will come around, more aggressive medicine some time flare up the symptoms have nice day."
},
{
"id": 215851,
"tgt": "Suggest substitute of generic drug mallinckrodt for pain",
"src": "Patient: since I am disabled I am forced to take generic meds for my pain. generic does not work the way a brand/real dose would. I have only found 1 generic that remotely close to the brand. I think the name is Mellinrodt (according to pharmacy. It is so difficult to obtain this as drug stores carry the cheepist. You would not believe the remarks that come from the mouth of some of some pharmacist. I thought aarp was a good insurance. Last year my prescriptions cost over 5,000. I take 60mg of ms contin generic, 3 times daily. My breakthrough drug is Nucynta and truthfully it is not a good pain med. It does help me sleep through the pain. Nucynta does not have a generic, therefore I pay close to 500 for just those 2 pain meds. I need to change the Nucynta, I,m tired of being hungry. Any advice?? Doctor: Hello, Mallinckrodt is a pharmaceutical company which manufacture many drugs. Nucynta is a brand name of opioid drug Tapentadol. It\u2019s good and effective drug for chronic pain conditions. You can consider Tramadol, another generic opioid which is very effective for you and is safe for long term use also. Hope I have answered your query. Let me know if I can assist you further."
},
{
"id": 86756,
"tgt": "What is the treatment for lower abdominal pain?",
"src": "Patient: HI i am having pain in lower abdominal portion, when i go for running it become more, if I stop & walk pain get less, i am also feeling like burning sensation on tip of my pinse after urination, kind of burning on tip of male organ. Urining routine has increased but burning sensation is arround after peing. i am 38 year old. i also feel like pressure on my lower abdominal part and on left side pressure toward back of kidney. please need your help, i already had before this problem and i used citralka syrup, pain were gone, but i could feel like pain in urinatory tack for many days. Doctor: Hi.Thanks for your query.You are absolutely right. You problem looks like recurrent urinary tract infections as suggested by your history of : Lower abdominal pain which increases on running, frequency of urination, burning at the tip of penis, lower abdominal pain going to the kidneys and helped with Citralka, the urine alkilinizer. I would advise you the following in details:First of all get an Urine : culture and sensitivity, routine and microscopy. Ultrasonography of the abdomen particularly for urinary system.Intravenous pyelographyCT scan in some cases if cancer is suspected."
},
{
"id": 46902,
"tgt": "What causes severe kidney pain and vomiting?",
"src": "Patient: Hi, may I answer your health queries right now ? Please type your query here...My sister got a renal calculus removed frm ureter thrugh her urinary tract 2 days back .... Doc said he has put a stent der. But she has got severe pain n vomiting n nausea... Wat cud b da reasons.... Or these r da post op complication Doctor: HiIt is possible that your sister has a urinary tract infection following the operation. Occassionally stent itself can cause these symptoms which are occasionally severe like in your sisters case. The other possibility is side effects of the medication given during anaesthesia - however it is rare to get it after two days. Please contact the doctor who did the surgery."
},
{
"id": 186727,
"tgt": "Could the tooth extraction cause pain an tingling in right portion on body?",
"src": "Patient: I had a tooth extracted and now have tingling in my chest which is radiating down my arm and hand. All of this is on the right side....the same side as the extracted tooth. It took a lot of pain medication to numb the area. Could this be the cause of the pain I'm experiencing now? Thank you. Doctor: Hi,Thanks for posting the query, No this could not be related.I would like to tell you that pain, swelling, nausea, fever, dairrhae are some of the complications after tooth extraction, tingling in the right side of the body could be due to other reasons.I would suggest you to get a checkup done by a Physician and get your blood pressure examined.Take care!"
},
{
"id": 151964,
"tgt": "How long can you live with a brain tumour before it becomes cancerous ?",
"src": "Patient: How long can you live with a brain tumor before it becomes cancerous Hi I have a sister with a brain tumor on her motor skills, and her doctor said its a grade 3, and they said its inoperativeable, I would like to know how long can she live with it, and is there something else they can do except chemo and radiation because she already received 6 months of radiation and is currently getting 2 different types of chemo therapy. also how long before it will become cancerous? Doctor: Hi,Mnatali, Thanks for query, Expectancy of life in brain tumour depends upon site of tumour. If it involves vital centres then life span is short as compared to other silent area of brain. Radiation and chemotherapy are the treatment in brain tumour if it is unoperable. Since beginning your sister is having cancerous lesion. Ok and bye."
},
{
"id": 182069,
"tgt": "Suggest treatment for painful infected lipoma inside cheek",
"src": "Patient: I have a lipoma on inside of cheek I also have multiple sores on inside of mouth was seen by oral surgeon and was put on amoxicillian (liquid) and got that filled twice and Peridex rinse and now I have Lidicaine to put on it but pain comes right through.I am getting really depressed over this and these sores have been happening for over a year now.I am diabetic and have low white blood cell count too. Doctor: HelloIs your diabetes in control? As in diabetic, intra-oral sores get routinely infected so you need to bring that under control.Try using betadine gargle for 2 weeks also to heal those sores soon.Rest treatment is already being given by the oral surgeon.Thank you."
},
{
"id": 11507,
"tgt": "How to regain original color of the face skin?",
"src": "Patient: Hie im 20 yrs old..bt due to some circumstances i was living in a vulnerable house fr two yrs with direct sunlight..nw aftr four yrs my body has come to its own colour bt my face is still dark n widout glow...i hav faced lots of mental tensoin also...please i need mah colour back n mah glow...help Doctor: Hello and welcome to healthcaremagicI have gone through your concern and I have understood it.Tanning seems to the problem that is bothering you. I would suggest you to use a sunscreen, regularly.UV A part of the electromagnetic spectrum is mainly responsible for tanning. Therefore, a good sunscreen should have broad spectrum UV A coverage in addition to UV B coverage, in order to be able to prevent tanning and it needs to be applied in sufficient quantity and every 2-3 hourly.A sunscreen with minimum 30 spf would be adequate. It should mention both UV B and UV A coverage. Therefore it is important to select the right sunscreen for desired effectsIt should be applied in the following quantity:--3ml for face and neck(a little more than 1/2 teaspoon)--3ml for each arm(a little more than 1/2 teaspoon).A sunscreen should be applied every 2-3 hours.If I was your treating doctor I would have asked you to use a cream containing one or more of the skin lightening agents e.g kojic acid (kojic cream), glycolic acid (Glyco -A cream), arbutin (Demelan cream) or hydroquinone etc once daily at night to gradually fade away the tan.Hope this helpsRegards"
},
{
"id": 71344,
"tgt": "Suggest possible treatment for COPD",
"src": "Patient: COPD Chronic Obstructive Pulmonary Disease Dear Sir, First of all many thanks for your kind consideration to pay your valuable time in reading my e-mail. Through this communication I would like to bring your kind attention, our most serious problem about my Father (Age 80+), who is suffering from COPD (Chronic Obstructive Pulmonary Disease). My Father had already gone by thorough medical check-up & he is all right in other health aspects. The problem is COPD (Chronic Obstructive Pulmonary Disease), which is perfectly diagnosed & we have all the reports in hand. Now a days my father is living in North part of India (Kotdwara - Uttarakhand State), Near Haridwar/Dehradun. We are looking for the best possible Hospital/Doctor (Pulmonologist) to treat COPD problem of my Father, somewhere in North part of India (Preferably in Delhi, Meerut, Chandigarh, Dehradun, Bijnore), since he can not travel long distance due to his bad health. Also the transportation infrastructure has been affected badly in Uttarakhand, Uttar Pradesh, Delhi\u2026.. Therefore, please let us know your valuable suggestion, in order to provide the best possible treatment to my Father. By considering his age (80+), we prefer to give him treatment without going for any serious surgery. His physical appearance is also too weak. If you are aware about the above, please provide us detailed information about the Hospitals/Doctors including perfect address/location, name of the doctors, telephone number, e-mail etc. Your Favorable answer will be highly appreciated Thanks & Best Regards Bharat Singh Rawat Kotdwara (Near Haridwar/Dehradun Garhwal) District : Pauri Garhwal State : Uttarakhand India Tel. No. : 0091 - 222836 Mobile No : 0091 - 0000 E-Mai : YYYY@YYYY (Note : Please note that, I will be available on above Mobile No. from 01/10/2010, meantime just reply me by return e-mail) Doctor: Hello,For COPD (Chronic obstructive pulmonary disease) treatment, any good MD Pulmonologist can treat your father's COPD but inhaler like MDI FORACORT twice a day, MDI TIOVA 9 microgram 2 Puffs. Both with ZERO STAT VT SPACER can be used.Use oxygen, if SPO2 less than 95, use Bipap, if breathlessness present after inhaler therapy. Go for yearly flu vaccination, staphylococcal vaccines 5 yearly and pulmonary rehabilitation can also be done.Hope I have answered your query. Let me know if I can assist you further.Regards,Dr. Chandra Shekhar"
},
{
"id": 114929,
"tgt": "What causes elevated leukocytes counts?",
"src": "Patient: My grandfather is 95 yr old and from few days he is facing alot of health complecations his leucocyte count is more than 12,000 he has sepsis n is on iv antibiotic, nebulisation is also done.he started eating but from yesterday even milk is coming out from nostril if amount is increased, he is not able to lie on bed though feeling sleepy.please help me I request. Doctor: Counts above 10,000 are considered raised. The most common cause for raised leukocyte counts is infection/ inflammation. In your case, sepsis- its a type of infection is responsible for raised leukocyte counts."
},
{
"id": 199646,
"tgt": "Does masturbation cause loss of memory and concentration?",
"src": "Patient: Does masturbating make you feel this: - looking at something but can't absorb the information from the object, feel like I'm in my own word, trapped in my mind.- lose memory of things I get told at school and I have trouble remembering how to work out equations and remembering meanings if words.I'm 14 and I have being madturbating a lot for 4 years now i think and on average i masturbate 3 times a day, I grind my teeth, my parents say its because if schoole but, I think it's masturbating to much each day. Now it's 2014 now so I set a goal to masturbate for ever but... I have masturbated again 2 to 3 times a day. What I'm mostly concerned about is how I feel like I'm in trapped in my mind and not able to consentrating and absorbing information that feels like my brain is way to slow to process information. And I do think I have anxiety which makes me clench and grind my teeth day and night, I don't wanna tell my parents what is also making feel this way ( masturbating of course). I would really appreciate your help, I have tried for years to stop masturbating but just to addicted to it. :C Doctor: Hi, You should decrease it to once or twice a week. You will consciously have to avoid it and get busy in some outdoor sports activities with friends of your age. This will help you get your mind off the masturbation and you will regain your confidence and memory and concentration also. Results will be very soon I can assure you,If you are unable to then you might require help of a Psychologist or Psychiatrist. But I feel if you try you have the inner strength to give it up. Take care and feel free to contact me through my Profile page on the website.Take care,Dr Rishi, New Delhi, India"
},
{
"id": 152644,
"tgt": "Suggest treatment for cancer that is spreading to the esophagus",
"src": "Patient: My brothers lymphoma has traveled into his esophagus he can no longer eat or drink he has gotten nine treatments of chemo and the cancer had spread through his esophagus . He went to a specialist today and said that they need to wait two weeks to see what the chemo will do this is the ninth and I m afraid something is going to happen to him if no one can help him Doctor: It takes time for the chemo to work. But you have not mentioned what lymphoma it is. lymphomas usually respond fast but aggressive types also recur fast. In cases of recurrence, chemo is unlikely to respond quickly. All treatment is by a protocol. let them complete the protocol and then reevaluate."
},
{
"id": 53356,
"tgt": "What does burning pain behind scapula after gallbladder removal suggest?",
"src": "Patient: I had gallbladder removed 06/28/10. I have been having burning pain behind right scapula since my first gallbladder attack on 05/15/10. about 2 hours ago I brgan having this same burning pain but now it's all across my upper back. I ate a full meal about 30 minutes before the pain began. should I go to hospital? Not sure what's going on. Doctor: Hi,It sounds like postcholecystectomy syndrome which is usually a temporary diagnosis. An organic or functional diagnosis is established in most patients after a complete workup. Once a diagnosis has been established, treatment should proceed as indicated for that diagnosis. Treatment may be either medical or surgical. Patients with irritable bowel syndrome may benefit from the administration of bulking agents, antispasmodics, or sedatives. Surgery is option just for retractable cases.If there is no improvement you should do ERCP procedure for diagnosis and therapy.Wish you a good health.Kindly regards,Dr. Ivan R. Rommstein"
},
{
"id": 52659,
"tgt": "What is liver flushing?",
"src": "Patient: I was recently diagnosed with parasites and have been going through a parasite elimination protocol. I m feeling better and better by the day. I ve read that after getting rid of parasites it is beneficial to do a liver flush. What, if anything, can you tell me about liver flushing? Do you recommend it and, if so, which one do you suggest? I m a little leery of anything that involves fasting, but am open to doing whatever will help me heal. Thanks for your feedback! Doctor: Hello! Welcome to HealthcareMagic! I would suggest you take Ayurvedic medicine which are safe from all this. Tab Agnitundi vati 2 tablets, 3 times a day; it will cure your liver problem completely without any complications. Try it definitely you will get result. Thank you for contacting us. Hope this clarifies your concerns. Take care Regards, Dr. Naser Jani, Ayurveda Specialist"
},
{
"id": 44201,
"tgt": "Best contraceptive method Crisanta is recommended?",
"src": "Patient: Me and my wife are looking for the best contraceptive mnthod for two years and so we were suggested CRISANTA (Cipla) Pills. I want to know whether is there any risk of having Crisnata for a period of TWO years ? Is is possible that after two years on Crisanta, there might be problem in having pregnancy and Effects of Crisanta on Infertility after two years of use ? Thank you so much ... Doctor: Hello, Crisanta is a contraceptive pill,when used properly is over 99% effective. Common side effects such as nausea, dizziness, reduced menstrual flow, headache,breast tenderness are usually mild and short lasting. Any serious side effects like rashes,severe headache,fainting episodes,calf muscle pain, jaundice, breast lumps, to consult your gynecologist immediately. If not taken properly, can lead to contraception failure and pregnancy. It is contraindicated in women with pre-existing cardiovascular disease/liver disease, allergic to medication, in women who have a familial tendency to form blood clots, severe obesity and/or hypercholesterolemia (high cholesterol level), and used with precaution in smokers, in women who are on other medications. Contraceptive pills can be taken for a period of 2 years and conception after stopping the medications should not be a problem. Consider stopping the medication 3 months before planning conception and to start folic acid tablets 5 mg one daily. Period can be irregular for 2-3 months after stopping its use but regularizes itself."
},
{
"id": 175088,
"tgt": "Should I be concerned for a breastfed kid, who has loose stools and green potty?",
"src": "Patient: My 4 months old (premature baby) breast fed son is suffering from loose stools & his potty is green color. Doctor gave him Rinifol one in a day 2.50 ml & Econorm sachet daily for 3-5 days. He told me that its just a stomach infection. Whether its any serious thing? Also my son's head is always warm than his body. Doctor: Thank you for your consulting in Heath care Magic.Color of stool is not normal,it should be yellow and consistency like as milk cream.Your doctor prescribed good treatment,but we don't know what kind of infection he has and has he inflammation in inyerstine. That's why I advice you please do stool analysis for excluding gastroenteritis and stool culture for flora and sensitivity to antibiotics .You should buy sterile container with small spoon inside for stool analysis at the chemist shop ,take from 3 different place from diaper,you should take fresh stool only in the morning and give it in sample collection center in laboratory. Best regards Dr. Svetlana"
},
{
"id": 83027,
"tgt": "How can my daughter get relief from canker sores which is inside her lips near the gums ?",
"src": "Patient: Hi, I am a mother of a 2yr old girl and she is getting recurrent canker sores(usually on the inside of the lips near the gums and on the corners, not sure abt the rest of the mouth she never allows me to have a peek into her mouth).Its been almost a yr now and the same problem persists.I dont know wat to do...how can i relieve her from this pain.No medication has shown any fruitfull results and the same problem comes again within days.She has become so weak now...Although she is very active but she is unable to eat anything the way she used to.Please guide me through this in detail. Doctor: Hi, Thanks for posting the query, Canker sores occurs due to physical or chemical trauma . Visit to a Pedodontist and get clinical evaluation done , look for any sharp tooth causing irritation . Apply dologel oral ointment topically over the affected area. Take vitamin suplements. Maintain a good oral hygiene , clean the mouth with antiseptic solution using handkerchief or cotton this will prevent infection and promote healing . Hope this helps out. Regards.."
},
{
"id": 185409,
"tgt": "How to cure swelling and itching on the left tragus area?",
"src": "Patient: hello. a few days ago i woke up with a small red dot on left tragus that itched like crazy for a couple of days. it was also swollen. the itching and dot have disappeared, still a little swollen and it feels rough. now there is an itchy red dot, not swollen yet, on my right tragus. Doctor: Is this swelling painful?just a red dot which is etchy can be a insect bite or some dermatological problem. Is there any pain in your ear?there kind of swelling n etching do appear around the Ear which may be simply fungal. Try applying antifungal ointment."
},
{
"id": 23255,
"tgt": "Is Doxasosin safe to take for high BP?",
"src": "Patient: hello,I came off amiodarone 8 weeks ago.I was taking this for A.F .I was on normal sinus rhytm at the time and the doctor put me on1.25g of bisoprolol.This made my pulse fall to 48 .She then told me to stop taking it until pulse reached60.I then went into af 2 weeks later and doctor now says take bisoprolol even though pulse is only around 46.I am also taking Doxasosin for high blood pressure.I am concerned as to why my pulse is still low and when will it start to rise. thanks in anticipation david,Scotland Doctor: hiif you are in bnormal sinus rythma pulse rate around 60 to 100 is normal,at nocostt should it be 40/min,brady cardia is a very common side effect of amiodarone and beta blockers , ask you doctor to shift you to calcium channel blockers like verapamil or diltiazen , its a good substitute to brisoprolo with minimal effect on heart rate"
},
{
"id": 6934,
"tgt": "hi i am 31years old and have a 4 year",
"src": "Patient: hi i am 31years old and have a 4 year old baby boy.and 2 miscarriage ,i am trying to have another baby but by hsg i came to know that my one tube is blocked ,please suggest me the right treatment to get pregnant Doctor: Hi, Thanks for query, Your second tube is open so there should not be a problem for pregnancy. go for ovulation study by ultra sound and see that ovum comes out or not. and it comes then see for ovulation and do sex accordingly. ok and bye."
},
{
"id": 11779,
"tgt": "Used betnovate-n cream on face for pigmentation problem. Skin not evenly toned. Any other non-steroidal cream?",
"src": "Patient: hello doctor,im a female. i used to use betnovate-n cream on my face(alomost 3+ yrs) only at night, in minimum quantity for pigmentation problem on my face well it did give out good results but later i got to know that its steroidal cream and i should stop using tht cream. Thank God that it didnt spoil my skin tht badly after i stopped using it..its been 1 and half yr tht i stooped using it however my skin on the face i not very even toned. n i do see very light pigmentation on the face(well its not very dark but its there) could u plz suggest wht cream i sud use? however i have been using kojiglo n yugard cream(suggested by the oliva clinic dermatolagist) on the face every alternate night. plz let me know which the best non-steriodal cream for pigmentation and plz let me know if kojiglo n yugard are good creams. Doctor: hi welcome to hcm. betnovate-n cream over face is not good. its a potent steroid and it has many side effets specialy over the facial skin. acne,thinning of skin,rosacea etc may happen due to it. i have seen many cases who damaged their skin completely due to betnovate use. it should not be stopped suddenly.it should be gradualy tapered. as you have asked about kojiglo and yugard, these are good. you can use them without those side effects. it may take long time to have the effect. dont loose hope. apply regularly under supervision of your dermatologist. dont wory. good luck"
},
{
"id": 137649,
"tgt": "Suggest treatment for swelling in feet",
"src": "Patient: I SERIOUSLY NEED HELP my feet hurt so bad it feels like it s swollen but it s not i can t even walk sometimes from how much it hurts , im always tired i can t even go to college !! i almost can t do anything can t get out of the house cause i always feel fatigue and my feet hurt specially when i first wake up and put it on the floor !!! went to too many doctors but can t seem to find the solution plz help !! Doctor: Hi,Thanks for your query.From your symptoms, it seems that you are suffering from plantar fasciitis. It happens due to inflammation of the plantar fascia.It hurts the most with the first few steps in the morning because the plantar fascia is tight and has not stretched out yet, and throughout the day it loosens up. The best thing you can do is when you wake up in the morning, before getting out of bed and taking a step, give yourself a little foot rub.There are many things you can try to help your foot get better:Give your feet a rest. Cut back on activities that make your feet hurt. Try not to walk or run on hard surfaces.Use shoes with good arch support and a cushioned sole.Do toe stretches, calf stretches and towel stretches several times a day, especially when you first get up in the morning.I do hope that you have found something helpful and I will be glad to answer any further query.Take careYou can consult me again in detail directly through my profile http://www.healthcaremagic.com/doctors/dr-saurabh-gupta/64132"
},
{
"id": 168841,
"tgt": "What causes body tremors in a child?",
"src": "Patient: Hello I have a 23 month old son who weighs 25lbs. He has been having terrible shaking episodes for the last 3 months (so shaky he cant get the spoon in his mouth) This usually only happens when he first wakes up in the moring and again after he wakes from his nap. I took him to the dr and we did blood tests, the results came back with high blood sugar and mono? Our dr doesnt want to do anymore testing and thinks the hsaking is just from the mono. I no better and im starting to feel like no one will listen to me. Also when he was 6 months he had 3 seizures, we had an eeg done and the results came back good. Does this sound like a serious problem or is it normal as my dr put it?? please help! Doctor: Please do vitamin b12 levels in blood to rule out infantile tremors ....and as u described seizures at 6 months of age....does he had fever at that times? Cause fever may cause febrile seizures. ...in which there is nothing to worry...."
},
{
"id": 199055,
"tgt": "What causes weird sensation in testicles?",
"src": "Patient: Hello I'm just enquiring I keep getting a weird sensation in my testicals it's been happening quite regular and has gotten worse over this week I am not sure wot it is an just wondering if you could tell anything.it keeps making me feel i nee to go to the toilet quite often Doctor: Hello I share your concern, i will try to help you out in best possible way.Looking at your description this could be may be due to trauma or infection like epididymorchitis.Another possibility is prostate problem or a Renal stoneI would like further information help you better relevant medical history like metabolic disease or hormonal disorderAny recent exposure to unprotected sex?You should ask your doctor for few investigations to confirm the diagnosis like Urine culture and sensitivity test and an ultrasound to see for prostate enlargementBlood for CBC and liver function test or urine test for culture and an ultrasound are advisable if it is a chronic issueMaintain good hygieneyou should wear scrotal supportUse condom during sex if you are sexually activeHope this answers your question please feel free to ask for more clarificationThanks and regards"
},
{
"id": 144032,
"tgt": "What causes episodes of severe vertigo?",
"src": "Patient: Hi, I had wasp sting last week on the outer ear, and after that couple days i woke up and feel dizzy, spinning in the head ( vertigo) real bad every time, it still going on every time i bend over ...work under the table or lay down flat ( no billow)....i wonder is it coincident or something else Thanks Doctor: HIYou need to be examined. The sting was probably just a coincidence but this is an inner ear problem and you need studies to find out the exact cause"
},
{
"id": 46480,
"tgt": "What causes pain in kidney area after taking Ran-ciproflox?",
"src": "Patient: Hello. I am taking an antibiotic (ran-ciproflox 500MG) for a UTI and just started it last night. When I woke up this morning I felt a pain in my kidney area. I took one of the pills and threw up shortly after. Should I stop taking this medication or should I continue? Doctor: HelloThanks for query .Pain in kidney area with vomiting is not due to Ciprofloxacin that you have taken for UTI but it is most likely to be due to presence of stone either in kidney or ureter and needs to be investigated .Consult qualified Urologist for clinical examination and get following basic tests done to confirm the diagnosis.1) Urine routine and culture. 2) Ultrasound scanning of abdomen and pelvis3) Renal function tests.Further treatment will depend upon result of these tests and final diagnosis more precisely on size and site of the stone and effect of the stone on kidney function .Till then take antibiotic likee Cefotaxime and Nitrofurantoin with anti spasmodic like Cyclopalm twice daily and urine alkalizer like Citralka thrice daily .Ensure to drink more water to keep youself well hydrated .Dr.Patil."
},
{
"id": 150190,
"tgt": "Diagnosed with sciatica. Severe pain. Body looks crooked while walking. Took medications. Suggest",
"src": "Patient: hello my name is alexis and i would like to know if i am able to collect disibility living allowence for sciatica pain? i currently am working in a sandwich shop in which i stand all day and by the end of the night i can barley walk. when i sit too long i can barley stand, it hurts to bend over if i have to pick up something off the ground, and also when i walk my body looks crooked. the pain runs down my left leg and hurts so bad i take the prescribed medications but they dont take away the pain they just make it a little more barable for me to handle. Ive had the pain for over 2 years now and was diagnosed with siatica. Doctor: Hi,Thanks for the queryI understand your concernPlease check with your local neurologist as your symptoms are highly suggestive of neurological problem which gets aggravated by prolonged standing. There are different rules in different states and your local neurologist may be able to help you in this issueBest wishes"
},
{
"id": 192482,
"tgt": "How to get rid of masturbation and improve penile size?",
"src": "Patient: HII DONT HAVE ADEQUATE CONTROL IN MY EJACULATION. I CANT SAISFY MY PARTNER PROPERLY, ALTHOUGH SHE TELLS ME I DO BUT I KNOW THAT I DONT HONESTLY...& I ALSO HAVE REALISED THAT MY PENIS SIZE IS GETTING SHORTER DAY BY DAY...IS IT DUE TO OVER MASTURBATION??? I MASTURBATE 4 TIMES A DAY...PLEASE HELP ME...I REALLY WANT TO GET RID OF THIS & WANT TO SATISFY MY PARTNER... Doctor: Hello, Penis size cannot get shorter unless it shrinks because of old age. You cannot improve the size of penis at any case. Masturbation addiction has to be overcome with addiction therapy. Therefore, I suggest consulting a psychologist for physical examination, diagnosis and treatment. Hope I have answered your question. Let me know if I can assist you further. Regards, Dr. Sameen Bin Naeem, General & Family Physician"
},
{
"id": 94528,
"tgt": "Lower left abdominal discomfort, hurts while coughing. What could it be?",
"src": "Patient: Hello there my name is brittany and I am 23....3 days ago I started having this pain ...well I wouldn t say pain pain more of an annoyance in my lower abdomin I thought the first day maybe it was just gas since that s kinda what it felt like but it has been going on ever since more so on my left side.....it also kinda hurts when I cough Doctor: HI.Lower abdominal pain on left side needs to be evaluated for Colitis/Left Ureteric colic . If the pain is associated with mucus in stools,it might be large intestinal infection.Cough raises intra abdominal pressure,so you are getting pain .You should exclude Inguinal hernia on left side also as the swelling increases on coughing ,but subsides after lying down.All the best."
},
{
"id": 224327,
"tgt": "16 yr old, had sex, not sure whether sperms have entered. Should I take an i-pill for safety ?",
"src": "Patient: hii.. i turned 16 in dis september... n i had sex wid my lover yesterday around 11.30 am. he s not sure if during d tym of sperm release his penis was completely out of my vagina ..n v r afraid if any sperm myt hv stil entered in me. amean v r nt sure if v had protected sex. please guide if i shud take an i- pill for safety Doctor: Hi,When was your last menstrual period? Are your menstrual cycles regular?If you have regular cycles the first week and the last week of your menstrual cycle are safe. If you have had unprotected sex during this period it does not result in pregnancy. So there is no need to take I PILL.Please note that the fluid released before ejaculation also contains sperms and can cause pregnancy. You are not sure whether the penis was completely out of vagina. So there is a definite possibility of pregnancy if you have had sex on days other than safe period. In this case you should take I PILL. The earlier after sex you take the emergency contraception more effective it is ( 95% effective if taken within 24 hours).There is a small possibility of method failure. So if your period is delayed for more than a week get a pregnancy test done. I hope I have answered to your satisfaction."
},
{
"id": 115686,
"tgt": "How to lower MCV in blood?",
"src": "Patient: I had an MCV reading of 100, blood-test done via GP. I need a medical through the DVLA soon - in order to renew my Driving vehicle licence as I was disqualified for a drink-driving offence. Although I have not drank any alcohol at all for nearly 9 weeks now I still consider the MCV of 100 as high? How do I help lower it? Gina. Doctor: Hi, dearI have gone through your question. I can understand your concern. Most common cause of high MCV is vit b12 deficiency. Other causes are folic acid deficiency, alcoholism, liver disease etc. You may have vit b12 deficiency. You should go for estimation of vit b12 level. Then take treatment with vit b12 injection accordingly. You will get benefits. Hope I have answered your question, if you have doubt then I will be happy to answer. Thanks for using health care magic. Wish you a very good health."
},
{
"id": 45697,
"tgt": "What does this abdominal ultrasound test result indicate?",
"src": "Patient: dear sir my ultrasound report shows there is a 7 mm shadowing mobile intraluminal echo seen in GALL BLADDER. what does it mean?. is there any connection with gall bladder cancer? IMPRESSION - cholelithiasis(solitary single calculus in G.B) Doctor: Hi, You are having a gallstone problem. It is not gallbladder cancer. So don't worry about that. You can take Udiliv tablet for that which dissolves cholesterol in bile and so beneficial. If you are asymptomatic than only screening is needed. No further action required. Hope I have answered your query. Let me know if I can assist you further. Take care Regards, Dr Parth Goswami, General & Family Physician"
},
{
"id": 91639,
"tgt": "What is the cause of pain in stomach while laying on back?",
"src": "Patient: For years now, every time I lay on my back or sleep on my back, my stomach hurts a lot. Usually is really bad if I have been on my back all night. And if I try to push my stomach out the pain increases.... If I lay on my side or stomach it doesn't do this , only if I'm on my back. What's going on and what can be done, I'm tired of it and it's very unpleasant feeling. To make the pain go away I just have to get up out of bed and move around. Doctor: Hello dear,The symptoms as mentioned in your post is attributed to accumulation of gas secondary to increased acid production in the stomach.Symptomatic relief can be obtained with intake of Pantoprazole preparation (to be taken twice a day before meals) & antacid gel (to be taken after meals).They prevent formation of excess acid in the stomach & provide symptomatic relief.Anti-spasmodic preparations like Dicyclomine can also be used for symptomatic relief.And it is also needed that you take certain precautions regarding your diet & lifestyle, such as:1. Take soft, plain, non-spicy foods2. Drink lots of water to maintain adequate hydration3. Avoid fast foods & gas producing food products like cabbage, etc.4. Avoid smoking & alcohol.5. Keep away stress, think positive.If symptoms still persist, kindly consult a Gastroenterologist for proper clinical examination.An Ultrasound scan of Abdomen will be helpful.There is no need to worry, you will be fine.Wishing a good health.Take care."
},
{
"id": 135197,
"tgt": "What causes painful small knot like lumps in muscles?",
"src": "Patient: I have painful small knot like lumps in almost every main muscle in my body , my muscles spasm and cramp suddenly after a very short time of doing anything and the weirdest thing of all is the tendons in my forearms (both) ache and are as tight as piano wires. I have had bloods done I have high phosphate, ggt, and eosinophils but the docs are,nt concerned about them .icant put up with this any more whats wrng with me?? Doctor: Hi Dear,Welcome to HCM.Understanding your concern. As per your query you have painful small knot like lumps in muscles. Well there can be many reasons for symptoms you mention in query like muscle tumor or lipoma . I would suggest you to consult general practitioner for proper examination . Doctor will examine physically to check tenderness and consistency . Doctor may take biopsy . Doctor may refer you to oncologist after diagnosis for radiotherapy or other treatment . Doctor may also recommend liposuction for lipoma . For now apply warm compresses on lumps and take ibuprofen or acetaminophen for pain . Hope your concern has been resolved.Get Well Soon.Best Wishes,Dr. Harry Maheshwari"
},
{
"id": 124299,
"tgt": "What causes swelling of hands & shoulders ?",
"src": "Patient: For the past 3 weeks my hands have been swollen and my shoulders and elbows hurt. Now I have knots in my wrists and my feet and ankles are swollen with knots at the joints of my feet. I have an appt with a rheumatologist but they can t see me until Aug. 27. I am always in pain. Any suggestions? Doctor: Hello, Since you don't have any appointment slot now and time is moving on. I would recommend you to take up the physiotherapy sessions which might include the ultrasound therapy and TENS therapy. Also, the inclusion of exercise for maintaining the ROM and joint stability will help improve the muscle strength and overall joint functions. Hope I have answered your query. Let me know if I can assist you further. Regards, Jay Indravadan Patel, Physical Therapist or Physiotherapist"
},
{
"id": 155913,
"tgt": "Does pioglit cause bladder cancer?",
"src": "Patient: I was given Pioglit 15 by a doctor. The sugar level drastically reduced and sometimes went below level.The side effect was that my weight increased up to six kilos within two months. At the same time I listened to TV program and it was said that there is a danger getting cancer in the bladder. Is it true? Doctor: Hi, dear. I have gone through your question. I can understand your concern. pioglitazone can causes weight gain and hypoglycaemia. But bladder cancer is not associated with the pioglitazone. So don't worry about that. Hope I have answered your question, if you have doubt then I will be happy to answer. Thanks for using health care magic. Wish you a very good health."
},
{
"id": 35731,
"tgt": "How to treat ear infection caused fever in a child?",
"src": "Patient: 7 mo old ear infection my daughter was diagnosed with an ear infection in both ears and thrush on Friday 11/30. she was put on amoxicillin and something else for the thrush. she is still running a high temp ...not exactly sure what due to I am out of town with her and do not have a thermometer. cheeks are red and her head is extremely hot...compared to my hand, my cheek, and her body. should I be worried? I gave her Tylenol...now she is sweeting. it is Tuesday at 5am and I am not sure what to do natalie Doctor: Thanks for your question, I am Dr Asanghanwa, I would say your child is suffering from a condition known as acute Otitis media, which is mostly due to bacteria. The most appropriate antibiotic for her is a combination of amoxicilline and clavulanic acid given her age as the germs are more sensitive to this rather than just amoxiciline. I also would advise you get to a pediatrician once you get into town for proper examination and tests. Keep on administering the tylenol thrice daily. Good luck."
},
{
"id": 213026,
"tgt": "Heart beats fast when I smoke. Is it due panic attacks?",
"src": "Patient: Went from no panic attacks for 28 years since I was born to three in the last 2 months. I stopped smoking 9 months ago but have started Hookah Pipe Smoking . It seems to happen when I feel my heart beat when I smoke . Is it a trigger for my panic attack and is it normal to get a higher heart rate 100 to 129 bpm when smoking? Thanks Doctor: Hello and welcome to Healthcare Magic. Thanks for your query. Your symptoms seem to be indicative of panic attacks and smoking can indeed trigger such attacks. It is definitely not normal to have a such a high heart rate when smoking. Please consult a psychiatrist for further evaluation and therapy - both for your panic attacks and for help in quitting smoking. There are effective treatment options - in the form of medication or counselling / psychotherapy which will help you overcome your problems. Wish you all the best. - Dr. Jonas Sundarakumar Consultant Psychiatrist"
},
{
"id": 5240,
"tgt": "PCOD, had miscarriage, taken follicular test. Can i conceive again?",
"src": "Patient: am 37 yr . i have lost my baby for 8 month 23 days during pregnancy i have pod. can i conceive again . now i am taking follicular test and result is my follicle is MSF the day of 12 ND 15. plz ans me Doctor: Hi,The cause of your pregnancy loss is not PCOD as conception is the main problem with PCOD and also early maintenance of the pregnancy, but not a pregnancy loss in the late trimester. You can definitely conceive again if ovulation can be achieved and tubes are patent. A mature follicle measures about 18 mm. just before ovulation and sizes more than that may indicate cysts. Considering your age, I would advise you to see an expert in infertility and get further advice. Take care."
},
{
"id": 88955,
"tgt": "What causes recurring stomach pains?",
"src": "Patient: HI I'm a healthy 27 year old but everyday around the same time (between 1 and 3pm ) for the past 3 weeks I've had stomach pains. I go to the bathroom regularly, I exercise and eat right. What could cause this? I have bad stomach pains now as I am writing this.. Please Help... Doctor: hi welcome to hcm your pain could be related to GERD/GASTRIC ULCER.Take antacids like Tab.Pantoprazole 40mg once daily before breakfast and anti motility drugs like tab.lomotil twice daily for 5days.Avoid alchohol and spicy food,hot liquids.take fruits,buttermilk,water 3-4 litres per day.Get an upper GI Endoscopy by consulting your doctor if pain persists. Thankyou"
},
{
"id": 93549,
"tgt": "Severe stomach pain after having a change in the food. Medication?",
"src": "Patient: doc , i had my stomach aching since last night. the actual pain occurs every hour and then. it really hurts my abdomen. the reason i think might be the korean food that we ate last night since my stomach isn't used with those stuff. i then had my LBM but its controllable unlike the usual. what medicine can you doc recommend me to drink? and what do you think is this condition? thanks in advance . Doctor: Hi and thanks for the query,Its important you recognize already that its due to a change in diet. The first thing might be try to know whether the symptoms you are experiencing are also experienced by others who ate the same food. This could be important to give an idea on food intoxication. If others are not complaining, that it would be worthwhile somewhat avoiding the particular food type in future. Depending on whether you are experiencing only abdominal pain, or is accompanied by diarrheoa or vomoiting the management would be different. If pain is mainly above the navel and moving to the back, this could be indicative of gastritis, and if present, antacids could give relief. If its only abdominal colics, anti spasmodic drugs could help for now.In case symptoms persist for two or more days, you might to consult your primary care physician for reevaluation. For now, antacids and antispasmodics, as advised by your pharmacist could be very useful.Thanks and best regards,Luchuo, MD."
},
{
"id": 161677,
"tgt": "Suggest remedy for high fever, phlegmy cough and runny nose in toddler",
"src": "Patient: My 18 month old had a fever of 102 five days ago. Wet phlegmy cough, runny nose. At night she has some wheezing. Took her to ER friday night, she had no temperature, they gave her an inhaler. This morning, Sunday, she has same symptoms and a fever of 103.8. What to do, if anything???? Doctor: Hi Dear,Understanding your concern. As per your query your toddler have symptoms of high fever, phlegmy cough and runny nose which seems to be due to bronchitis/upper respiratory tract infection. Need not to worry. I would suggest you to visit ENT specialist once and get it examined. You should go for throat swab test and X-ray chest as well start treatment after proper diagnosis. You should get pulmonary function test done to check level of infection. You should do warm saline gargles. You should take steam inhalation. You should take anti allergic medication along with proper course of antibiotic along with Ibuprofen. Do tepid sponging. Hope your concern has been resolved.Best Wishes,Dr. Harry Maheshwari"
},
{
"id": 11785,
"tgt": "Suffering from pigmentation and dark spots, taking Janumet, vitamin suprative and calcium. Solution?",
"src": "Patient: Good evening doc am suffring from pigmentation a dark spot since 2 years have taken no of treatments but no results am 45 yrs old well maintained, lady i do walking, n yoga , am a diabetic, under went histraktamy, i take janumet 50/500mg n amryl 1mg, for diabetese i take vitamins suprative n calcium n vitamin e regularly. Doctor: Hi, Thanks for writing to us. There could be many causes for pigmentation over face and hence the treatment protocol may vary according to underlying cause. You may follow these general recommendations to help with your concern 1) use a Glycolic acid based face wash like melawash. Keep it over face for two to three minutes then wash off. Little bit dryness is expected 2) use a sunscreen with SPF 30 or above, even if staying indoors 3) take vitamin c, beta carotene and antioxidant supplements 4) apply hydroquinone 4 percent cream over dark spot. Withhold use if redness or irritation occurs. These recommendations might help you out but I would still suggest to meet a dermatologist and ascertain cause of pigmentation. If topical creams alone do not help, chemical peels and lasers can be done to reduce pigmentation. Regards"
},
{
"id": 191546,
"tgt": "What causes high fever and excessive breathing while having high blood sugar level?",
"src": "Patient: Husband has belching, retching at times, 102.fever, weakness, pain in thighs, out of breath, what can that be? His blood test only showed sugar was high. Is not doing blood culture to check for infection in blood. Fever yesterday was 106 and last night - heavy, rapid breathing, worse than his normal sleep apnea. What do you suggest? he took xray today - no results yet. Doctor: since he is diabetic and he is having high fever and is sugar level is too high. he might have some urine infection because of increased glucose output in his urine it might cause some urine infection. better to do on urine culture and to check whether any growth is present. taking antibiotics according to that might reduce the fever and infection.thank you."
},
{
"id": 141744,
"tgt": "What causes head spinning while waking up with CABG?",
"src": "Patient: Hi Doctor My father is operated CABG on June 21st, it's close to 2 months now. Since last night he says that when he is waking up, his head is spinning for a couple of mins. Can you explain the reason for this? Is this something we should be worried about? Doctor: Hello,Yes, it is a complaint which needs Immediate Medical attention. Please monitor his vitals, and also do repeat 2D-Echo with Cardiologist Only.He should take the Medications Prescribed for Post-CABG regularly.Hope I have answered your query. Let me know if I can assist you further.Regards,Dr. Mohammed Abdullah"
},
{
"id": 128836,
"tgt": "Suggest surgical procedures for swelling and poor circulation in the legs",
"src": "Patient: My husband is 69 years old and has swelling and poor circulation in the legs. What kind of surgery can be done to help with this condition? Will this be in patient or out-patient surgery? How Long will it take to heal? Or for him to return to normal activity? What are the complications, restrictions and side effect associated with this type of surgery? What precautions should be taken after the surgery? Doctor: Poor circulation to the feet should be improved, if he has ischemic pain of the feet or legs, or a ulcer of the feet. If he has neither, then increasing circulation is not necessary, since an peripheral angioplasty will get blocked again in about 8 months, and a peripheral bypass surgery will get blocked again in about 18 moths or so.Firstly, a peripheral MR angiogarm or CT angiogram is done by the Radiologist. After studying the film, doctor can inform if there are angioplastyable or by passable blocks. You can then on his advise get the revascularisation done."
},
{
"id": 81377,
"tgt": "How to stop coughing caused by choking up on coffee?",
"src": "Patient: My fianc\u00e9 swallowed some coffee but at the same time he breathed in and it caused him to choke, he coughed a lot after but he thinks the coffee has gone into his lungs, he has a slight pain under the bottom part of his ribs of his right side. He keeps coughing now and is wondering how to get the coffee out? Doctor: Thanks for your question on HCM.In my opinion your fianc\u00e9e is having aspiration.Aspiration can cause 1. Inflammation2. Irritation3. Infection in the trachea and lungs.So better to consult pulmonologist and get done chest x ray to rule out infection.Usually coughing stops by itself. But since she is having continuous cough, better to get done clinical examination by pulmonologist.She may need antihistamine and inhaled bronchodilators.If still she is not improving than she may need bronchoscopy and evaluation.So better to consult pulmonologist and discuss all these."
},
{
"id": 177567,
"tgt": "Can PDD and ADHD cause aggressiveness?",
"src": "Patient: My child is very violent and abusive towards me. He is nine and has ADHD and PDD. He threatens me by saying he is going to kill me one day with a knife and punches, kicks and verbally abuses me. What is wrong and what can I do? He already sees a psychiatrist and a therapist and takes Adderral, Paxil and Trazadone. Something is wrong with him and it is getting worse as he gets older. What could it be and what can I do? Doctor: YYour son may have bipolar disorder. The things to consider are: Is there a family history of severe mood disorders? Does your child seem to have dramatic changes in his personality, mode of expressing himself, behavior, facial expression, demeanor during his aggressive rages and then after an episode is over does he seem to feel sorry but not really appreciate the details of what he has done? Does his personality return to normal briefly after a rage? Can you tell just by the look on his face or the way he is speaking that he is going to go into a rage and is it impossible to reason with him when he is in a rage? Does he become extraordinarily strong during a rage episode? Does he appear to be menacing, threatening, frightening, unwilling to back down and unusually cruel in what he says and does? Does he seem, after a rage attack, to be at peace, more normal, kind, apologetic? Does he truly not remember the detail of what he has said or done or does he seem to disregard the seriousness as if he does not believe what he is told he did? Does he have difficulty going to sleep? Does her seem unhappy and discouraged a lot, as though he is unable to help himself feel good? Does he have a lot of anxiety? Are these symptoms more common at home or when he is trying to \u201cshut down his brain\u201d after playing games or otherwise occupying his mind. If these seem to relate to your child, he needs to stop the Adderall, wean the Paxil and start a mood stabilizing medication like Tegretol. Once he has been stabilized on the Tegretol and the episodes have stopped, remaining ADHD symptoms can be safely treated with a stimulant medication. These episodes are seizure like events arising from the amygdala and the use of a stimulant medication makes them more frequent and more severe. So does the use of Paxil. When he starts Tegretol, you will immediately notice a few things. He will be happier, will be able to sleep well, will not have such severe rages. As the body builds a tolerance to Tegretol until the maximal metabolism of it is in place, you may notice backsliding and need to raise the dose repeatedly over a period of a week or two until he saturates the ability of his liver to metabolize it after which this level will remain steady and he will stop having rages. When this happens, you will notice what I call the Tigger stage, where he will be happy and very hyperactive, impulsive. This is the time to start the stimulant medication to fuel the parts of the brain in the frontal area that have not been well fueled because the rage activity has been such a great consumer of fuel. Make sure his doctor is taking all of this seriously as interruption of this process will eventually extinguish it and prevent long term psychiatric disability. Allowing these episodes to continue will cause the development of permanent neutral pathways of responding to life in the manner he does when in a rage and therein lies chronic psychiatric disability. He is young and with proper treatment you can have your child back and assure his good future. Best of luck to you an feel free to contact me again should you run into difficulty or have further questions. Remember, he is the child you love. His symptoms are not him and he is not in control of what he does during episodes of rage."
},
{
"id": 120435,
"tgt": "Is a total knee replacement advisable based on the report?",
"src": "Patient: MARKED GENU ARTHROSIS BILATERALLY WITH SIGNIFICANT REDUCED FEMOROTIBIAL JOINT SPACE ON THE LEFT NEARLY OBLITERATED ON THE RIGHT OSTEOPHYTE FORMATION NOTED ALSO INVOLVING BOTH PATELLA \u2013FEMORAL JOINT SPACE IS PARTIALLY REDUCED MILD GENU LAXUM SEEN R L MODERATE TO SEREVE FEMORO \u2013TIBIAL ARTHROSIS R L THE ABOVE IS THE RADIOLOGIST RECENT REPORT SHOULD SHE HAVE TOTAL KNEE OPERATION AT 75 SHE IS TAKING ADVIL ONE MORNING ONE EVENING Doctor: Hi, As the proper guidance can be given only by seeing X-rays and scanogram however according to report total knee replacement is a good option in your case. Hope I have answered your question. Let me know if I can assist you further. Regards, Dr. Jaideep Gaver, Orthopedic Surgeon"
},
{
"id": 144188,
"tgt": "What causes dizziness and double vision after suffering from viral meningitis?",
"src": "Patient: I am a 50 year old female and have always been in good health. About four weeks ago I was diagnosed with viral meningitis after a CT Scan, MRI (with and without contrast) and a lumbar puncture. I never had a fever or stiff neck - only a headache that would not go away and vomiting on several occasions. I also developed double vision about 10 days after the headache started. The day before the double vision started I had received a clean MRI; however, once my vision was affected (on a Friday afternoon) my primary doctor suggested I may need to go to the ER. The next morning on Saturday I felt worse and experienced more vomiting so I headed to the ER. They admitted me and after 5 days in the neuro ICU and a round of every test they could run, the final diagnosis was (or it seemed like their best guess) was viral meningitis. I ve now been home from the hospital for three weeks and still do not feel well. Additionally, I have seen a neuro-opthalmologist specialist that agrees with the diagnosis. She reviewed the results of the lumbar puncture as well as conducted a round of tests, and she did not find any permanent damage and believes the double vision is post-viral. I still have headaches and feel like I am slightly dizzy and like I am in a bit of a fog. My neurologist says to rest, drink plenty of fluids and be patient; however, I am very depressed and worried that I am never going to feel well again. Also, the double vision is terrible...although the opthalmologist says it is normal for this to take up to several months to go away. Additionally, the neurologist says you are fortunate that you did not suffer a stroke, seizures, memory loss...and that you are walking and talking . While I am greatful that I did not suffer those things, I have not worked in 5 weeks and have no quality of life. The feeling of being in a fog is worse than the headaches. As soon as I get up and start moving around, I feel strange and feel like I should sit down. I am eating and drinking the fluids as the doctor suggested and resting all the time. Because I m not getting any real clear answers from the doctor, I have spent a lot of time reading articles on the Internet and I am very scared now. Most documentation out there says I should have been better in 7-10 days with Viral Meningitis; however, there are numerous stories of people that say their lives have never been the same and they were sick for months (some years)! At this point I don t know what to do. If I go to another Neurologist for a second opinion, I am pretty sure that my insurance will not cover an additional round of the same tests. The current Neurologist is at a loss other than doing a 3rd MRI - and the insurance company has not yet authorized. Doctor: Hi, I am Dr.Bruno. I have read your question and understand your concerns. Let me try to help you Question : What causes dizziness and double vision after suffering from viral meningitis?Answer : Meningitis Causes Scarring (damage) of Sub arachnoid spaces. It is through these spaces blood vessels that carry blood to your brain stem travel Assume the Blood vessels as flexible pipes (or hoses) that carry water. What will happen if some stones are thrown over them. They will compress the flexible pipes. Isn' tit Same way Due to Meningitis, the areas around the blood vessels (Sub Arachnoid Spaces) are damaged. Hence they interfere with Blood Circulation to your brain stemThis causes dizziness and Double Vision Some Times, The Nerves can also be directly involved by the scarring (That is electric lines running along the pipes) Most of these cases will heal completely Anyhow, I would advise you to do another MRI after 4 weeks and compare both Hope you found the answer helpful.If you need any clarification / have doubts / have additional questions / have follow up questions, then please do not hesitate in asking again. I will be happy to answer your questions.Let me know if I can assist you further.Take care."
},
{
"id": 42015,
"tgt": "Does conception require medications given the follicular study results?",
"src": "Patient: i had my folilicular study my endometrium is 9mm.on 11day it was 9mm.n on 16day again it was 9mm.n again on19day it was 9mm only n both right n left ovary follicles are small .my age is 29 yrs n my wieght is 80 n my hieght is 5,6.pls tel me im normal r i need medicines to concieve n i have hypo thyriod .after my 1st delivery i have 3 years child.pls tel what i do. Doctor: Hi, Welcome to HealthcareMagic. You are not ovulating properly. So you need to take ovulation inducing drugs like clomephine . Also your endometrium thickness is less. So you need to take progesterone tablet per vaginal. You need to take thyroxine supplement to treat your hypothroidism. Once hypothroidism is treated and you reach normal thyroid levels with drugs you may start planning for pregnancy. You may discuss this with your gynacologist. All are prescription drugs. Hope I have answered your query. RegardsDr.Deepika Patil"
},
{
"id": 122590,
"tgt": "Suggest treatment for the tail bone pain",
"src": "Patient: I am a 38 year old woman, fairly fit, two children. I have had very painful tailbone pain for several months now and it seems to be getting worse, rather than better. The pain is unusual, it almost is a burning pain and it worsens if I sit or even lay on my back. I am in constant discomfort to pain. Doctor: Hello, For resistant coccydynia[tail bone pain] sacral local block injection may be helpful. Prolotherapy [local injection] may also work for sacral pain. Consult your doctor for an injection. Physiotherapy like ultrasound and interferential therapy will give relief. I will advise checking your vit B12 and vit D3 level. Continue sitz bath and use of ring cushion. Hope I have answered your query. Let me know if I can assist you further. Take care Regards, Dr Ivan R. Rommstein, General Surgeon"
},
{
"id": 148847,
"tgt": "Trembling, shaking arms on coughing, twitching eyelids. On BP, cholesterol medication. Treatment?",
"src": "Patient: I m a 41 year old female 5 8 , sedentary lifestyle and 250lbs. On blood pressure meds and cholesterol with good results. When I cough or raise my arms (or voice) both arms tremble and shake. Sometimes my eyelid twitches too. Never lasts for more than 30 seconds. Slight pain on the right side of my neck. What s causing this nonsense? Doctor: Hi,Thank you for posting your query.These disturbances are due to changes in autonomic, mainly the sympathetic nervous system.On coughing or with other activities as described by you, there is release of sympatho-mimetic amines, which can cause shaking and eyelid twitches.There is no need to worry about them.I hope it helps.Please get back if you require any additional information.Best wishes,Dr Sudhir Kumar MD (Internal Medicine), DM (Neurology)Senior Consultant NeurologistApollo Hospitals, Hyderabad,My personal URL on this website: http://bit.ly/Dr-Sudhir-kumar My blog: http://bestneurodoctor.blogspot.com/"
},
{
"id": 53325,
"tgt": "Suggest treatment for back ache and sweating after gall bladder removal",
"src": "Patient: hi i had key hole surgery and stones and gall bladder out too afther my surgery i was getting very mild tummy pain and so fare now iv had 3 bad attacks includeing back pain and sickness sweating and more i had my surgery october please can you help Doctor: Hi,Symptoms like abdominal pain can occur after gall bladder removal that is called post cholecystectomy syndrome. The cause for that has to be searched for if present like biliary microlithiasis, adhesions etc.For that investigation done with USG and if needed endoscopy or ERCP like procedure. According to cause specific treatment instituted. Meanwhile diet modification done with low fat diet If stones present in biliary canaliculi than ursodeoxycholic acid can be prescribed.Consult surgeon for your examination and subsequent work up accordingly to plan your treatment Hope your concern solved.Hope I have answered your query. Let me know if I can assist you further.Regards,Dr. Parth Goswami"
},
{
"id": 98072,
"tgt": "What medicine should I take for severe sore throat ?",
"src": "Patient: I have severe chronic fatigue and recently developed a bad severe sore throat . I know this is a symptom of cfs . What can i do for it besides drinking salt water and taking motrin. i am 34 female. i also have hot flashes and then get cold. What can i do about the soar throat it feels like im swallowing glass and is very swollen. i also have post nasal drip. Doctor: Hi, Virul infections, bacterial infections, irritants and injuries can cause a sore throat. Sore throat symptoms may be accmpanied. by coughing , sneezing , fever , tonsils , chronic sinusities, step throat and allergies. Try to rest your voice for the day. Avoid straining your throat by talking too much, especially talking lowdly. Avoid fried or cold foods , which can aggravate a sore throat. Sufficiant intake of water helps in washing away infections quickly from your system. My advice is; Gargle with infusion made from HENNA LEAVES heals sore throat as it has anti bacterial Gargle with 2 tablespoons of SESAME OIL three times a day. Apply paint in the throat with GARLIC JUICE + HONEY mix Chew THALISHADI VATAKAM as lozenges. Take SEPTILIN tablet twice daily.."
},
{
"id": 116858,
"tgt": "Suggest remedy for improve hemoglobin levels",
"src": "Patient: im during 8th month pregnacy my hemoglobin level is 9.6.in 2&3rd month my hb level is above 10.3, in 6th month my hb is 9 at that time doctor iron gluose is given.7th month my hb is 10. any problem to baby this condition.pls suggest how to improve hb level Doctor: Hi, dear.I have gone through your question. I can understand your concern. You have iron deficiency anemia. You should take injectable or oral iron therapy. You should take high iron diet like dates, green leafy vegetables, green wheat, jeggary, dryfruits etc. Go for regular Hb estimation. Take treatment accordingly.Hope I have answered your question, if you have any doubts then feel free to ask me. I will be happy to answer. Thanks for using health care magic.Wish you a very good health."
},
{
"id": 70143,
"tgt": "What could be the lump behind neck?",
"src": "Patient: I have gotten my neck pierced at the back, just above my spinal column, it has has been getting cleaned regularly and somehow it has a lump where the piercing is, aswell as a slight amount of pus radiating from it. Could you help me get to the bottom of what this can be ? Doctor: Hi,Thanks for writing to HCM.That is an abscess following infection due to the piercing.You need to shoe it to a surgeon if its incision and drainage under antibiotic cover. Or if it is very small then take a course of antibiotic and burst it yourself after cleaning it with an antiseptic solotion, followed by regular dressing.RegardsDr. Ashish Verma"
},
{
"id": 91546,
"tgt": "Will wine cause upper abdominal pain?",
"src": "Patient: Hi there, i have been having pain in my upper abdomen that feels like indigestion but it is more painful then the other time i have had indigestion. i am also having some back pain. i have taken gaviscon and it helps for a little while but soon comes back. i have had this since i woke up yesterday morning. i had a few glasses of wine the day before yesterday, not sure if thiat may be the cause Doctor: HI. It looks you have a pre-existing acidity problem, got exacerbated due to irritation that the wine causes. Continue gaviscon, Ranitidine , soft bland diet.If the problem persists go for an upper GI endoscopy to see the exact problem."
},
{
"id": 31786,
"tgt": "Does contact on the genitals cause infection on my valvular skin?",
"src": "Patient: Hi, i recently got an infection on my vulvar skin. I visited a gynae and she said that its nothing except a skin disease. she has treated me with ointments and capsules. But i would like to know something from you. I went physical with my partner about 2months ago. Though we didn't had any kind of sexual intercourse but we certainly had a contact on the genitals. I couldn't say this to the doctor as my mom was present over there. I would like to know if this has affected me. Also i have masturbated myself many times. Can this be a problem either? About my infection- there's little bumps with white faces which were small but now have grown up with lot of pain. these eruptions are present on the skin joining the labia majora and the minora. also these are present just at the opening of my vagina. But there is no such irritation while urination but when the urine or water touches those bumps. it itches at times. All i want to know is that the possible causes for this infection and also that whether it is affected by my sexual activities and does my partner play any role in this? Doctor: Hi, Thanks for posting in HCM. I understand your concern. Infection or inflammation of skin of vulva could be caused due to abrasions caused during foreplay by rubbing the genitals or due to friction during masturbation. This generally happens when there is lack of enough lubrication during the act. Based on its severity, it would cause a variety of symptoms, and some of which you are experiencing.Kindly keep the area clean and hygienic by washing with warm water and mild antiseptic solution. You can apply neomycin with hydrocortisone cream during night time and moisturizer in the morning. Wear loose fitting cotton undergarments till the infection subsides. Hope the information provided would be helpful. All the best."
},
{
"id": 78568,
"tgt": "Suggest treatment for persistent dry cough",
"src": "Patient: my mom-in-law is suffering from persistent dry cough from last 5 months.all x rays , broncoscopy,ct scan of the chest have been done and the reports are negative . she is now taking cardiozol-paracodina which gives her temporaray relief.if she stops the cough starts again.kindly give advice Doctor: Thanks for your question on Health Care Magic. I can understand your mother in law's situation and problem. No need to worry for major Pulmonary disease because her CT scan and bronchoscopy are normal. Sometimes small airway disease like bronchitis or bronchiolitis can cause dry cough. These are functional diseases and so can not be picked up by imagining studies or scopy.For this, she needs to get done PFT (Pulmonary Function Test). If it shows obstructive airway defect than her cough is mostly due to bronchitis or bronchiolitis. And then she will improve with inhaled corticosteroid and inhaled bronchodilators. So get done PFT first. Hope I have solved your query. Wishing good health to your mother in law. Thanks."
},
{
"id": 125123,
"tgt": "How can fractured tailbone be identified ?",
"src": "Patient: Hello, I went to sit on the edge of my son s bathtub and didn t sit back far enough. I landed hard on my tailbone on the edge of the tub and heard a crack. I then started to feel hot, nauseous and lightheaded like I was going to pass out. Could I have fractured my tailbone? Doctor: Hi, I'm afraid, yes, you might have fractured your tailbone. I highly recommend to consult with orthopedic specialist as soon as possible. You should run X-ray of tailbone to confirm this. Hope I have answered your query. Let me know if I can assist you further. Regards, Dr. Albana Sejdini, General & Family Physician"
},
{
"id": 59565,
"tgt": "Chronic right sided back pain, radiating to buttocks. Due to gall stones?",
"src": "Patient: just came back from radiology and they told me I have a handful, and then some of gallbladder stone and can t make out what it says on the paper but he wrote down 9mm, I have this chronic pain the right side but in the back down to my butt could the stone project all the way down to the butt cheek, and are 9mm big stone? Doctor: hi looking at the history pain in back radiating to the buttocks is not due to gall stones but because of lumber spine . gall stone finding is incidental . kindly consult orthopedic surgeon get your spine examined ."
},
{
"id": 24024,
"tgt": "What causes dizziness and rapid heart beat?",
"src": "Patient: okay so im 43 and my whole body shakes all over when i start to get up and it feels like some 1 is standing on my chest the world feels like its shaking fast its hard to see my eye sight is becoming poor it feels like there is a fog in my brain and and i feel like im going it faint my heart beats really fast then suddenly slows to like 2 beats light and my heart feels heavy and arms and limbs are really weak some tomes when im out about i get small shake but i got to tell myself not to bend down this has been getting worse since march 2011 Doctor: Hello!Thank you for asking on HCM!Your symptoms could be related to orthostatic hypotension. I recommend consulting with your doctor for a physical exam and some tests: - a Head UP Tilt test for orthostatic intolerance- complete blood count for anemia- fasting glucose and HbA1C for diabetes- blood electrolytes- thyroid hormone levels for possible thyroid dysfunction- cortisol plasma levels for possible adrenal gland dysfunction. You should discuss with your doctor on the above issues. Hope to have been helpful!Kind regards, Dr. Iliri"
},
{
"id": 104135,
"tgt": "allergic bronchitis during April and May every year. Problem since childhood, genetic. Using Ashtalin inhaler. Problem?",
"src": "Patient: I am suffering from allergic bronchitis which generally comes into affect in the month of april and may every year. I am having this problem since childhood and its genetic. I am using ashtalin inhaler 6-8 times in 24 hours to get relief. My bronchitis lasts for around two weeks every year. Is there any problem using the inhaler. Doctor: ok, Mr. Vishesh ,Ihave got your quiry.Idont know your age.So how long you are suffering is an important point.you said that as genetic also.Have you any skin allergy also?it may be associated with it.Now see you are using the inhaler only at times of your attack of bronchitis.But you have to follow a guideline after proper check up by your doctor.some tests will help to determine your lung's status or how it is functioning .some blood tests are also important to determine your allergic status.After a proper check up doctor will advice you medication and the dose has to be adjusted according to your response.Actually in the month of march-April that is the spring time ,many allergic substances are there in the air.So you are getting the problem.But nothing to be worried.Remember every medicine has some adverse effects either little or more.So only taking medication at the time of attack is not right for your case.You have to undergo a continuous health check up by your doctor and to take medicine according to his/her advice."
},
{
"id": 11331,
"tgt": "Suggest treatment for hair loss & dandruff",
"src": "Patient: Sir, i have been shedding hair in past six months during shampooing and 5-10 hairs while combing. . i do not find hair on pillows and nobody in family is bald. i do have dandruff problem, hence shampoo on alternate days,and also a oily scalp. what is this type of hairfall? and is it temporary?Regards. Doctor: Hello. Thanks for writing to us at healthcaremagicYou seem to have scalp seborrheic dermatitis.Dandruff Or Seborrheic Dermatitis is a common cause of increased hair shedding. Your increased hair shedding is most likely due to underlying dandruff.Controlling dandruff effectively would control excessive hair fall too.Various antifungal agents (e.g ketoconazole, sertaconazole, ciclopirox olamine, piroctone olamine) are available either as shampoo or as overnight leave-on lotions, for dandruff.Nizoral shampoo contains ketoconazole. It is available OTC and is the most commonly used antidandruff shampoo. It may alone be sufficient in mild cases.In severe cases, along with the antidandruff shampoo, I usually ask my patients to use a topical steroid lotion (Mometasone furoate Or Clobetasole propionate lotion) once daily at night for a short period of 2 weeks Or I might switch them to an overnight leave-on antidandruff preparation.Seborrheic dermatitis is a steroid responsive condition, therefore, topical steroids are usually effective in this condition.A two weeks course of topical steroids brings about remarkable improvement, which can then be maintained with just an anti-dandruff shampoo, every alternate days.You may visit a dermatologist in your region. Your dermatologist may ask you to use one of the prescription antifungal shampoo e.g Ciclopirox olamine, Piroctone Olamine based shampoo Or choose to prescribe an overnight leave-on antidandruff lotion.Your dermatologist might also choose to prescribe a topical steroid lotion for a short duration of 2-4 weeks.Regards"
},
{
"id": 39589,
"tgt": "Can I take Nexium and Domperidone for H. Pylori and indigestion?",
"src": "Patient: Have been taking Nexium ( esomeprasole) 40mgs once daily for four years along with 10mgs domperidone for my transit issues! Following endoscopy it transpired I was suffering from poor transit and carrying the H Pylori bacteria I was treated for this four years ago. Three weeks ago I started having indigestion issues and one wk ago stopped taking my Nexium! I now feel very sickly and nauseus with no energy feeling extrely lethargic ! I have been on no other meds. Doctor: Dear Friend.Welcome to HCM. I am Dr Anshul Varshney. I have read your query in detail. I understand your concern.H.pyolri requires treatment with a particular regime. If you have taken that, even then it can reoccur.You can take Nexium and Domeperidone for indigestion.Take soft bland diet.This is my best opinion for you based on available details. If you have any further query please.Stay Healthy.Dr Anshul Varshney, MD"
},
{
"id": 23888,
"tgt": "What causes hypertension, chest pain and nausea with overweight?",
"src": "Patient: 37 yr old male, overweight,very high lipids, hypertension, severe chest pain , arm and jaw pain nausea and vomiting for several days--grandfather and grand mother--one on each side have heart disease--went to er cardiac enzymes were normal sent home from the emergency room what next Doctor: Hello,Thanks for writing to Health Care magic, I am Dr , Muhammad Ahmad , I have read your question closely, I understand your concern and will be helping you with your health related problem.After the heart issues were ruled out you seem to be having stomach issues, here is the plan of how you deal with stomach problem.1) Avoid coffee, caffeine, Fizzy drinks , junk food, smoking, alcohol (What ever is relevant).2) Take small multiple meals instead of few larger ones.3) Drink much fluids.4) Use fruits and vegetables more than meat.5) Use fiber diet like fruits vegetables whole grain bread and fiber supplements if needed.6) Stay physically active.7) If you are over weight reduce it.8) After trying out all these measure you can use syrup gaviscon thrice a day this will keep a check at your gut PH.9) You can discuss with you doctor about digestive enzymes they too help with digestion and excessive gas.10) I also recommend similar patients to use Omeprazole 40mg once a day to check stomach acid levels.Next Plan about heart protection is as under.1) Get physically active.2) Reduce you weight. You can join a gym or a fitness club under a certified trainer and you can get healthy weight loss. This will reduce you cholesterol and chances to get heart issues. Will also help with stomach issues.3) Avoid fatty junk food, and smoking if relevant.4) Use a good balance of food, vegetables and fruits have ingredients which are cardio protective.5) Pay regular visits to your doctor to get your progress discussed.I hope this answered your question, If you have more queries I am happy to answer you.Regards.Dr.Muhammad Ahmad"
},
{
"id": 18417,
"tgt": "What does this ECG test result indicate?",
"src": "Patient: Had an ecg yesterday for work physical. Ecg comments say sinus rhythm minor inferior repolarization disturbance, consider ischemia, LA overload or a specific change . Doctor at Industrial clinic said it was fine. Last comment was Ecg without significant abnormalities . Some other values are posted on the Ecg that I don t understand. Doctor: Hello and welcome to \u2018Ask A Doctor\u2019 service. I have reviewed your query and here is my advice. The comments on ECG is always not correct so don't worry it may vary. So please show it to your physician he will examine and interpret with results. I think I answered to your question if you have more questions feel free to ask. Hope I have answered your query. Let me know if I can assist you further."
},
{
"id": 35192,
"tgt": "If no symptoms persists, is Herpes contagious?",
"src": "Patient: i wanna ask if i have no symtops (sores) at all for herpes and my blood test IGM -ve and IGG +ve its an combination test still dont know which type is the +ve ... if i got my type 2 +ve without sores any symptops can i still trannfer it to my partner? Doctor: Hello dear,Thank you for your contact to health care magic.I read and understand your concern. I am Dr Arun Tank answering your concern.No, you are negative for the herpes. You are negative for IgM and positive for IgM this signifies. That you have past infection of herpes. You are not currently infected.As you are negative there are no question of further infection or transfer to partner.But always wearing a condom is good. It will protect you from STDs and also gives contraceptive benefits.I will be happy to answer your further concern on bit.ly/DrArun.Thank you,Dr Arun TankInfectious diseases specialist,HCM"
},
{
"id": 61137,
"tgt": "What does a lump on the rib cage indicate?",
"src": "Patient: I ve had this hard lump pop out below my right rib cage and breast. I ve had it for years. But only happens a couple times a year. I quick stand erect and it subsides within a couple of minutes. The other day it happened twice...and the hard lump stayed for 20 minutes. It would start to subside...and as soon as I moved it would get all hard again. I was so scared. Pretty sure it s a hernia. I did make an appointment with my doctor...but don t go till Tuesday. What can I do till than so it doesn t pop out again? Doctor: Hello dear Warm welcome to Healthcaremagic.comI have evaluated your query in details .* As the lump is reducing in nature there is definite connection with internal structure .* Ultrasound examination will surely prove the details , till then you should avoid weight lifting , strenuous work so that it does not pop out frequent.Wishing you fine recovery .Feel free to ask any further doubts .Regards ."
},
{
"id": 136038,
"tgt": "How to cure pain on the lower right leg after an injury?",
"src": "Patient: Hi, I had tripped and hurt a part of my lower right leg (shin bone area) by hitting it against a pice of concrete two days ago. I was wearing jeans, so there was no direct contact between the skin and concrete, but my skin still ruptured and the skin in the area around the open wound got stuck to my jeans (was sort of scraped off because of the jeans). I had washed the wound and applied antiseptic as soon as I had gotten home, but today the skin area which had scraped off seems to be a yellowish-brownish scab surrounded by reddened skin and the pain in my shin bone surrounding the wound (upto 2 inches above and below the wound) seems to continuously increase. Could there be a serious problem with the wound? Doctor: HiWelcome to healthcaremagicI have gone through your query and understand your concern.There is nothing serious about it if there is no fever. So you can keep area open with antiseptic ointment applied over it. If you have fever then it is cause of concern. It denotes infection and needs urgent care by antibiotics and if needed incision and drainage of pus. Antibiotics such as ibuprofen are given to relieve pain. You can discuss with your doctor about it. Hope your query get answered. If you have any clarification then don't hesitate to write to us. I will be happy to help you.Wishing you a good health.Take care."
},
{
"id": 13239,
"tgt": "What causes sudden rashes around implanted defibrillator site?",
"src": "Patient: I have had an implanted defibrillator since 1998 and had the device replaced successfully in December 2013. Within the last 2 hours, a sudden rash has appeared around the device implant site. The area itches extremely bad and is hot to the touch. I have never experienced any problems in the 16 years of having an implanted defibrillator. Can you please help me? Doctor: Hello, I have gone through your query and it seems to be an allergic reaction. I would recommend you to apply topical steroid like cortisone cream on the affected areas twice daily and take antihistaminic like tablet Cetirizine once or twice daily depending on the intensity of itching. Hope I have answered your query. Let me know if I can assist you further. Take care Regards, Dr Asmeet Kaur Sawhney, Dermatologist"
},
{
"id": 66224,
"tgt": "What does stinging pain radiating from a bump in lower calf suggest?",
"src": "Patient: Thank you. For several years I have experienced stinging pain raidiating from a very small bump in my lower calf. Compression stockings made it worse. An ultrasound has shown that it may be that my leg has cysts. Surgery is scheduled but I don't feel comfortable with the diagnosis. Please tell me what you think. Doctor: Hi,From history it seems that there might be having some vascular insufficiency due to vascular block like clot or deep vein thrombosisi.Color Doppler will be of much helpful to come to conclusion.Consult vascular surgeon and get examined.Ok and take care."
},
{
"id": 224077,
"tgt": "Delay pill cycle. Can I? When else can I take them?",
"src": "Patient: I m getting married in the history of 28-9, but I have a problem I am in this period will be two sessions monthly and I want to delay taking the pill cycle ... My question is when to take them? Whether it helps in the prevention of pregnancy? Knowing my fiance and I do not want to have children during the first year of marriage .... Doctor: Hallow Dear, Birth control pills are very good for delaying the period. These pills have to be started in one a day dose at least 7 days prior to the expected menses. Continue the pills till you are OK to have menses. After discontinuing the pills, within 4-7 days you should get the withdrawal bleeding. In fact, the exact timing of this withdrawal bleeding cannot be predicted; hence better take the pills till you are ready for menses. Considering your requirement of postponing the pregnancy for few years, I would suggest you to start the birth control pills regularly from the day 2 of the next menses. Continue this pills cyclically till you are ready for the pregnancy. It will serve your both the purposes - delaying the period and contraception. I hope this helps you. Dr. Nishikant Shrotri"
},
{
"id": 117449,
"tgt": "What does low rbc & high wbc count with rubella IGG antibody suggest?",
"src": "Patient: During my pregnancy, my lab tests indicate I had high white blood cell count and low rbc count. I also tested high for absolute neutrophils and low for absolute eosinophils as well as having equivocal (leaning toward positive) detection of rubella IGG antibody. Later in the pregnancy, I developed significant swelling of lower extremeties that are getting worse each day and intense itching all over my body. Should I and my doctor be concerned. What do you think? What is your guestimate as to what's wrong with me. Doctor: Hello and welcome to HCM,A raised white blood cell count and absolute neutrophilia suggests infection probably bacterial infection.A low red blood cell count can lead to anemia.Consult your doctor for finding out the cause of infection and to treat anemia if present.Itching all over the body during late pregnancy is common because there is increase in levels of bilirubin which leads to increased itching all over the body.Detection of Ig G against rubella suggests that there was a past infection of rubella but the infection is not active now and no harm will be done to the baby.Swelling during late pregnancy is also normal because gravid or pregnant uterus causes compression of blood vessels of lower extremities.Thanks and take careDr Shailja P Wahal"
},
{
"id": 29851,
"tgt": "Suggest remedies for blister with pus on the thigh above the knee cap",
"src": "Patient: Hello, a blister with yellowish pass formed on my thigh just above the knee cap. At first I had itchiness and I scratched quiete vigorously the next thing a blister with yelowish stuff has formed and is quite painful. Should I puncture it with a sharp need and squeeze out the fluid or what should I do? Doctor: HI, thanks for using healthcare magicIt would be best not to puncture it because the aseptic technique can introduce further infection.At the moment, it is likely that you have an abscess which is a localized collection of pus, though part of treatment is incision and drainage of the abscess, this should done aseptically to prevent worsening.In addition, the abscess cavity is supposed to explored or prodded for a bit to break down any additional pus walls to prevent recollection.It would be best to see your doctor for assessment and treatment.Antibiotics are also neededIf this is difficult at this time then you can consider magnesium suphate paste, this encourages discharge to come out of an abscess.I hope this helps"
},
{
"id": 126617,
"tgt": "What causes stiffness and pain in the finger?",
"src": "Patient: My pointer finger on my left hand stiffens into a trigger position with severe pain. I rub it and try to relax it. It takes several minutes to stop freezing up and will usually cramp back up again within a few minutes. Any ideas on what s causing this? Doctor: Hi, It might be an early sign of arthritis. We have to rule out other causes like sprain and contusion also. As of now apply warm compresses and take analgesics like Acetaminophen or Tramadol for pain. Hope I have answered your query. Let me know if I can assist you further. Regards, Dr. Shinas Hussain, General & Family Physician"
},
{
"id": 6367,
"tgt": "How can I control my weight and get pregnant ?",
"src": "Patient: helo, sir/madam im29 years old having thyriod problems from last 8monthy thyroid is 5.07 and have polycystic ovary i am trying to get pregnant doctor give me medicine(thyronorm 25mcg) for my thyroid and ovary problem and for regular periods.....first thyroid report was 5.07, second report was 4.07, 3rd was2.74 and last was 1.10. its decreasing.but having some weight problem.im feeling helpless to loss weight,i want to get prego what should i do.my diet is normal i dont take butter, ghee, sweets.can i take oberids for weight loss? thank you Doctor: hello iam sorry for the delay in you getting conceived for you hypothyroid status TSH is in the normal range ,so hypothyroidism will not be a problem for you to get conceived. pcos-is one of the difficult situation-firstly you have to loose weight appropriately next you menstruation has to be normalised(if not so) maybe finally you need ovulation induction for further clarification kindly reply thanks dr arun kannan"
},
{
"id": 55263,
"tgt": "How to stabilize a high level of SGOT which is 140?",
"src": "Patient: Hi Doctor, i have done liver test. In the result, SGOT is high as 140 but SGPT is in normal value. let me know whehter there is any problem due to this value. i have undergone treatement for TB 7 years back and for jaundice 12 years back. will it be a reason for this high level Doctor: Hi thanks for asking question.You have not mentioned which type of jaundice do you had in past??You have taken antitb treatment which can lead hepatotoxicity and lead to hepatitis.so drug induce hepatitis could be cause.Just now viral cause for hepatitis has to be ruled put by doing viral marker.HBSag carrier can lead its elevated value .Fatty liver also can lead its elevated value.USG will be helpful for that.Sometimes previous heart attack and muscle injury can lead to elevated liver enzymes.It is better to repeat value after 3 month if cause not clear.If still cause not clear and value increasing then USG and FNAC can be done as further work up.for that consult gastric surgeon. I hope I have solve your query.Regards;Dr.parth"
},
{
"id": 141913,
"tgt": "Is there any treatment for brain atrophy?",
"src": "Patient: 5 month old baby has been diagnosed with brain atrophy, he gets seizures, he has hearing loss. they have him on sterioids...is this ok to give a baby? what does this mean for my son? any treatment? is there a life span for brain atrophy? do you recomend any hospital? Doctor: Hello!My name is Dr. Aida and I am glad to attend you on Healthcaremagic!Your son symptoms seem to be related to serious neurological disorders (epilepsy syndrome).It is OK to give steroids in such clinical condition, as they may help stop seizures and improve his situation. Anyway, it is difficult to judge on his future situation, based on what you refer. Genetic tests may be necessary. You should discuss with his doctor on the above issues and the exact diagnosis. Hope you will find this answer helpful!Best wishes!"
},
{
"id": 15572,
"tgt": "Itchy rash that started on ankles, spread farther up leg, mirrored on either side. Tried topical cream, antihistamine. Solution ?",
"src": "Patient: I have a rash . It started just on my ankles and is severely itchy. After a while it left my legs and moved farther up my leg. It has continued to do this moving in a very systematic pattern along my body. It is mirrored on either side, like currently it is on both of my wrist , the insides of my upper arms, my inner thighs and just around the sides of my back. When it first started it was only one or two bumps, slightly red much like an irritated hair follicle , at a time and they only lasted in one spot for a couple of days before moving. Now they show up in much larger patches 10-15 at a time and last for a weeks or so before they move. This started back in Nov. I have seen my gp for the problem and he just keeps on prescribing me allergy medicine, so far we have done a topical cream and a rx antihistamine, it was immediately after starting the antihistamine that my rash mutated into larger sites with longer breeding time. I need to stop itching, any thoughts!! I have also spent many hours scouring the interenet for any picture that resembles what I have and I absolutely cannot find anything that matches it. The raised bump is the size of a very small pimple and nearly always the same color as my skin or slightly pink after lots of scratching . Doctor: hello there, the description of the rash which comes and goes since november may indicate an allergic component. however an infestation of scabies has to be ruled out. treatment includes an antihistaminic, topical moisturising lotion and if it is scabies a course of permethrin overnight application all over the body below the neck followed by washing off the next day will be enough.take caredr.akhilesh"
},
{
"id": 116925,
"tgt": "When will hemoglobin level normalize after taking blood transfusion?",
"src": "Patient: I had a GI hemorrhage and I was in ICU for 5 days Then I was in the hostipal for another 3 days . Iwas given 3 units of blood & 3 units of iron in my IV. I went to get blood drawn the other day & my hemoglobin level is 10.1. . While in the hostipal tests reveal I have ulcers, diverticulosis & diverticulitis &hemmoriods. How long will it take for my hemoglobin level to come back up to normal? Doctor: Hello, Thnx to contact us. I understand your concern. If I am your doctor I advice you that since you have ulcers, diverticulitis & hemorrhoids all of which are bleeding condition in your intestines it is necessary for you to take treatment till you get all this corrected. your hemoglobin is good right now so at this moment you can start with the oral iron therapy.I will be happy to answer more of your concerns, kindly know me,Wish you a very good health at health care magic. Dr. Arun Tank. Infectious Disease."
},
{
"id": 69977,
"tgt": "What can be the reason of having soft lump on my back bone?",
"src": "Patient: hi. I have a soft lump on the upper right side of my back bone. its in the muscles. I recently had an accident and had a hairline fracture in my right rib. I also get dizzy and vomit. let me know what is this. my email address is YYYY@YYYY Doctor: Hi,This is likely to be a lipoma which is a fatty lump. Most lipomas need no treatment.It will not be causing your dizziness or nausea and vomiting.Regards,Dr K A Pottinger"
},
{
"id": 87381,
"tgt": "What causes pain in the lower abdomen that radiates to pelvis and knees?",
"src": "Patient: I was wanting to know why my lower stomach like the pelvic area . I have been having real bad pain from one side to the other .. It hurts so bad I cant move and it go down into my knees and makes my lower back hurt also . Last about an hour then it goes away . Also had a cat scan done , it picked up something in my cervix .. dr. said it could be a cell that got stuck and it was normal it was about as big as a dime . what do you think .. I told him that I have been hurting , and he said it was fine , but I don t think so . Doctor: Hi.Thanks for your query. The cause of the pain in the pelvis , so severe and radiation to the knee is suggestive of inflammatory process in the pelvis or in the back.I would advise you to undergo an MRI of the spine as there is a possibility of the inflammatory process going on in the spine or a prolapsed disc causing all the pains. Since your CT scan is done and does not show any cancer in the uterus , the pain can not be explained by the local problem in the pelvis."
},
{
"id": 138187,
"tgt": "What causes pain in arm after a dog bite?",
"src": "Patient: I am a vet tech at a small animal hospital. I was scratched during a dog nail trim a week ago. It has almost fully healed but now I m getting sever pain in my arm from the elbow to my hand, on the bottom side (sensitive ski. Area) is there any concern from the scratch or is this something totally different? No swelling and no rash. Doctor: Hi Thank you for contacting us. I will to address your concerns to the best of my ability. The single most important thing to rule out here is an infection. Antibiotics are prescribed for bite wounds to prevent infection in certain situations. For example, if:1.You have been bitten by a cat. All cat bites are usually treated with antibiotics, as they are much more likely to get infected than dog bites.2.The bite wound is on an arm or leg - especially a hand. These sites are particularly prone to nasty infections that can cause severe damage after a dog or cat bite.3.The wound is large, deep or punctured. A puncture wound may not look large but may go deep into the tissues.4.Your injury needed an operation to clean it out, or repair the damage.5.Your resistance to infection is low. For example, if you are on chemotherapy; have no working spleen; have diabetes; have an immune system problem such as AIDS.6.You have an artificial heart valve (and sometimes, if you have an artificial joint).Antibiotics will also be prescribed if your wound has already become infected. It might be infected if:It is getting more painful rather than improving as time goes by.It has become red or swollen.It is oozing.Sometimes the doctor's exam will discover some red lines in the skin of the arm - this is known as lymphangitis - this also needs antibiotics.Best to get this checked by your doctor as soon as possible. I hope this helps. Please message me back if you have any further questions.Best wishes and I hope you get better soon.Adrian Rawlinson MD"
},
{
"id": 73455,
"tgt": "Suggest treatment for tietze syndrome",
"src": "Patient: I have been told I have a severe form of Tietze syndrome.....I have been on morphine and other pain meds for 2 years now and I am getting worse. I dont have lapses of it, I am always in excruciating pain and can barely move...I have been told I have to live with it now as the steriod injections did not work, infact they made me worse....is there any surgery I can have to help? Doctor: Thanks for your question on Healthcare Magic.I can understand your concern.Tietz syndrome is severe form of costochondritis.Till date there is no known cause of this.Similarly till date no treatment is discovered which cure it completely.Yoi can try alternatively ice packs and warm water pad applications on affected areas of chest.Tramadol patch on affected chest area is also beneficial in some cases.Anxiolytic drugs like clonazepam, diazepam are also given to see if it works.Rib belt application should also be tried.No surgery is available for this.Hope I have solved your query. I will be happy to help you further. Wish you good health. Thanks."
},
{
"id": 184066,
"tgt": "What causes blisters on sides of mouth?",
"src": "Patient: Hi, i have on the sides of my mouth these blister type things. they hurt to open my mouth and stuff. i've tried regular lip chap to put on them because its like dry skin but its now been a week. i tried blistex to but it doesnt seem to be helping. do you know what will ge rid of these? Doctor: Thanks for using Health Care Magic.Read your query.The mouth ulcers can be due to stress ,nutritional deficiency ,vitamin deficiency and many other factors.I would advice you to apply mucopain /dentogel on the lesion two to three times a day till the symptoms are subsided.For the pain ,take paracetamol .Do betadine mouth wash gargling.Consult your GP and start with multi-vitamin tablets.Lead a healthy and stress free life style.If the ulcers are persisting ,please consult your oral surgeon.Hope this was helpful.Thanks and regards."
},
{
"id": 71491,
"tgt": "What could a mass on upper lung indicate?",
"src": "Patient: Hi, I am 54 yrs old and have cad/w2 stents that are 12 yrs old . I have a lot of chest pain and also hurting in my upper back. When I move around (walking , doing housework,etc) my ribs feel like they are squeezing together. I have high cholesterol, high blood pressure, and was recently diagnosed with diabetes. I am on meds for all problems. I also had a left heart cath done in Dec. and was told everything looked good. Also Drs discovered a mass on my upper right lung in Dec. I saw a pulmonary Dr , and he told me my symptoms were not from my lungs. What should be the next course of action to figure out what is going on? Doctor: Hello,You should do a chest CT for the mass and for the symptoms better to consult your cardiologist.Hope I have answered your query. Let me know if I can assist you further.Regards, Dr. Jnikolla"
},
{
"id": 8585,
"tgt": "Single mom, wants to enhance breasts, a-cup size. Cost of procedure?",
"src": "Patient: hi my name is xxxxx im a single mum of 4 i was wondering if you no someone that can help me.im looking to have a boob job done my size is a acup a liitle smaller but im on a low income is there any way i can get them done for free it would be great.ive never been bigger then an a cup im from the Doctor: Yes breast augmentation procedures are done to enhance breast size. If cost is a constrain, I would suggest you visit plastic surgery dept. Of a tertiary medical college. The procedure would be cheapest there."
},
{
"id": 86659,
"tgt": "What causes side abdominal pain and blood when wiping with tissue?",
"src": "Patient: Gm. Dr.,I woke up this am with (r)side abdominal pain. I noticed bright redblood on the tissue again. I realized it happened yesterday wo. Pain. I thought it was my menstrual. I am under a heat pad and now questioning what in really going on. W a hx of bleeding cyst and a dx. of fibroids, I am a bit concerned. Doctor: Hi.Thanks for your query.Read and understood our problems. The cause of right sided pain with blood on wiping paper can be Stone in the right ureter. This causes pain in the right side as well as causes blood to appear in the urine. This is not your menstrual bleeding , nor is it relate to the bleeding cysts or fibroids. I would advise you the following:Ultrasonography and X-ray of the abdomen will confirm the diagnosis.Intravenous pyelography to confirm the functioning of the kidney.Consult Urologist for further managment which may be removal by Ureteroscopy or lithotripsy or open surgery. This also helps to rule out other causes like colitis, Ovarian cyst or appendicitis with UTI"
},
{
"id": 135049,
"tgt": "Suggest remedy for muscle tightness and cramps",
"src": "Patient: I have cramps and very tight muscles in my legs. Sometimes I will wake with muscle cramps so bad in my legs that I have to have help to get out of bed to stand up. I am taking Potassium tables but lately everyday I seem to have cramps in my legs most of the day. Doctor: hi,thanks for your querymuscle cramps occur due to imbalance between the strength of your muscle and your activity level.you should drink lots of water everyday( 10-12 glasses atleast).drink coconut water if you caneat bananas regularlyDip your legs in a bucket of hot water( temperature comfortable to you) 2-3 times a day.you can also take vitamin E capsules 2 times a day for a month hope this helpswarm regards"
},
{
"id": 87389,
"tgt": "What causes stomach pain and lower back pain when i stand?",
"src": "Patient: I have been having stomach pains and lower back pain when standing up. Also been nausea and diahea for about 3weeks now. In a lot of pain. Cant do daily activities. Been diagnose with a stomach virus, ovanian cysts and even told i am having bladder contractions. I dont believe it us bladder contractions. Please help me and let me know what i should do because i cant take this pain anymore. Doctor: Hi.Thanks for your query.The causes of pain in abdomen with back ache when you stand have been noted by you in your history itself; stomach virus, ovarian cyst and bladder contractions. Yes, the bladder contractions can be secondary to the intestinal infections you have. This looks to be severe enteritis; colitis giving you the backache. I would advise you the following:Get the blood, urine and stool tests done. CT scan of the abdomen A course of an antibiotic and metronidazole.If the condition is bad, get admitted to the hospital."
},
{
"id": 42062,
"tgt": "Suggest treatment for infertility",
"src": "Patient: hi sir, i m trupti i m suffering from infertility i m age of 31,height:5.3,and 6 year pass of my marriage.and doctors sugges me for ivf but i m not ready for it by mentaly becoz it has not been garented result,and cost also.& i had an opretion for ektopik &remove left side filopin tube befor 5 year.but my right side tube is normal.so now what can i do now for become mom.pls help me sir pls Doctor: trupti do not get upset.sorry to say but ivf will best for you as it is having more success rate.go for it. you can go gujarat as ivf rates are very les in ahemdabad and surat.do not worry ivf is having good chances to get pregnant. hope my answer helped you.take care. do not forget to rate the answer.i wish you will be mother soon."
},
{
"id": 191621,
"tgt": "Suggest ways to manage type 2 diabetes",
"src": "Patient: I m a 63 year old with type 2 diabeties for 7 years now, resently within the past three months i ve gone fr from weighing 165 - 135 lbs and i m struggling to maintain that weight. I want to get back 165. was researching omega 3 oil. The reviews looks good. Your opinion please. Doctor: HelloI have gone through your question and understood your concern.It is true.Supplementing with omega-3 fish oil seems to be a safe and effective option that can help prevent and treat overweight and obesity.These supplements has been shown to be effective for promoting not only weight loss but also fat loss in various trials.Research reveals that lower body levels of omega-3 fatty acids are associated with higher BMI results, meaning that improving your omega-3 levels may reduce the risk of having a weight problem .Omega-3 fatty acids may contribute to improved body composition by increasing metabolism and fat burning potential. They also seem to suppress appetite, which leads to the consumption of fewer calories.But i want to underline that these supplements can not be effective for weight loss if you don't have a nutritionally balanced calorie-restricted diet and regular exercise.These are the key elements if you want to maintain your 132 Ibs.Hope my opinion is helpful.If you have other question feel free ans ask.Best regards."
},
{
"id": 217515,
"tgt": "What causes sudden pain in shoulder,arm,fingers and chest?",
"src": "Patient: my wife had a pin prick pain in her left shoulder and gradually went to the arm and then fingers, she also had chest pain to the tune that she started crying. She also had a vomiting and it all gradually disappeared in about 1 - 1.30 hrs yesterday night. please advise. Doctor: she is having possibility of heart problem so go for check up cbc ; ecg;other test to rule out heart problem."
},
{
"id": 80516,
"tgt": "Suggest treatment for chest infection",
"src": "Patient: im 21 weeks pregnant and have had a virus that ended in a nastey chest infection, i have had horrendous pain in my left rib and whilst coughing last week i felt a pop in my ribs, i now have a pop sensation when ever i breath and am in pain when i wake in the mornings if led on left hand side. lifting my left arm high causes discomfort what have i done Doctor: Hello dear, thanks for your question on HCM. I can understand your situation and problem. Since you are pregnant, chest x ray to confirm lung infection, is contraindicated in your case.Treatment of chest infection is based on isolation of causative organism.So get done sputum culture and sensitivity. Culture is needed to isolate the causative organism and sensitivity will be helpful in guiding appropriate antibiotic treatment. By this, we can avoid haphazard antibiotic use.Antibiotics in pregnancy should be used cautiously as they can damage fetus.So better to get done sputum culture and sensitivity to guide appropriate antibiotic treatment."
},
{
"id": 192461,
"tgt": "Suggest treatment to increase the semen quality and quantity",
"src": "Patient: dear sir my age is 28 year and i get married two years ago .i have semon problem that is total quantity 1.5 mltotal spermatoza count .30 milllion / mlmotile .40%sluggish 25%dead 35%pus cell 14 to 16 HPFRBC 18 to 20 HPFplease advise me the medicine to improve the semon Doctor: Hello,Since there are some pus cells in your semen analysis I suggest to undergo a culture studies as well and an ultrasound as well. It will be better for you to visit your doctor for your proper checkup for possible infection of your sex organs and seek treatment accordingly.Hope I have answered your question. Let me know if I can assist you further. Regards, Dr. Sameen Bin Naeem, General & Family Physician"
},
{
"id": 130619,
"tgt": "Suggest remedy for left side pain near the rib cage and purplespots on ankles",
"src": "Patient: I have left side pain near the rib cage. It feels tender with some pain.The lower left rib cage protudes more than the right lower rib cage. X-rays revealed no rib fractures.Tylenol medication does not help. I also have purple skin spots on the right foot in front of the ankle area.The physician assistant examed the purple spots and said it could be from crossing my ankles. I ve been avoiding crossing the ankles for 2 months. The purple spots have increased. I get shortness of breath when power walking. I had a physical 2 months ago. Also, the lab work (blood/urine) came back normal. Doctor: Hi,From the history what I suppose you have is Cryoglobulinemia disease from the group disease called vasculitis. Though this is a provisional diagnosis, I would like you to once again go for urine and blood test to verify any involvement of kidney and NY raise in WBC count respectively.Also to do a stress test as you feel breathlessness on power walking. Even though the test is very negative previously but there are chances that they are false negative. Hope this helps you in diagnosing your condition and apt treatment. Let me know if I can assist you further.Take care and hope for your quick recovery.Regards,Dr. Jenis Bhalavat"
},
{
"id": 16308,
"tgt": "Red blotchy rashes all over body. Having anaphylaxis to dairy products. No new food introduced",
"src": "Patient: my 12 year old son has red blotchy rashes all over his body, it started as a red rash on his cheeks, then went all over. Its not itchy, he has no temperature and he doesnt feel sick. He is however has anaphylaxis to dairy and egg. No new foods have been introduced, washingliquid hasnt been changed. have given him his antihistamine, but it doesnt seem to be helping. Doctor: Hi, Thanks for your query. By what you describe it is more likely to be urticaria. However it is better that you consult a doctor who will be the best person to guide you. Urticaria is a manifestaion of an allergic reaction to food or medicine. Anti-histamine should help and since it has not worked in your son's case it is all the more to know the exact problem. Wishing your son an early recovery."
},
{
"id": 225299,
"tgt": "What chances of pregnancy do I have after unprotected sex without birth control pills?",
"src": "Patient: Hi. Im on the pill & my period started before I started the inactive pills. I decided to stop taking the pill until my cycle ended & then would start a new pack. My husband & I had unprotected sex when I had finished my cycle but I had not yet started the pill. What ae the chances of a pregnancy? Doctor: NamastheWelcome to Healthcare-MagicGreetings of the dayits unlikely that you could be pregnanct, as you are in safe period that is first ten days of cycle, i suggest you to start using the pills and aalso advise you to use alternate contraception during 10th to 18th day of this cycleIn case you need any further assistance, will be glad to assist you.Wishing you happy and healthy lifeTake CareBest RegardsDr Deepak K KMBBS,MS,MCH"
},
{
"id": 4464,
"tgt": "What does the lighter line in test window mean in a home pregnancy test?",
"src": "Patient: HI,Im 26 days into a cycle, i have worked out my average which is 25 days. I did a HPT this morning and the control window was dark, i forgot to read the test within the 10 mins but the test window was lighter line that the control does this mean i am pregnant? Doctor: Hello! Home pregnancy test should be read within two minutes atleast . you can repeat once more and see. If still in doubt can go for serum b HCG levels which will be confirmatory! Thank u"
},
{
"id": 166647,
"tgt": "What causes frequent fever and cough in a child?",
"src": "Patient: my chld is 5 yrs old with fever for 5 days with antibiotics it does not cure. after 5 days fever goes dowm. but now having stuffed nose and a bad cough. before 3 weeeks he also having fever and it cured by antibiotics. iam little worried of having my child frequent infectionc. get me the answer please? Doctor: Hi,Welcome to HealthcareMagic.Thank you for your question. Infections in toddler age group is common(3yrs-6yrs). I would suggest you to take him to a paediatrician and get a few investigations done to rule to urinary tract infection. Otherwise proper hygiene, good diet including fruits and vegetables should be enough for him to build immunity.I hope this answers your query. Good luck.With best regards,"
},
{
"id": 25259,
"tgt": "Are increased BP and dizziness signs of stroke?",
"src": "Patient: I am 50 years old female. For three weeks my blood pressure is 136/96. I have dizziness when I lying down and when I getting up. I can not continue my exercise regime because I am worried I can get stroke\u2026. I have beet to the doctor and she measured it .This time blood pressure was 140/96, bur she did not pay attention\u2026 Doctor: Dizziness present while lying down and on getting up can mean more than one thing.Postural Hypotension, fall in BP when getting up from lying down position can also cause dizziness. For this a BP measurement in both positions needs to be taken. A fall in systolic BP above 20mmHg and diastolic BP fall of more than 10mm Hg is diagnostic of Postural Hypotension.This needs to be checked.If this is normal, a complete ENT evaluation needs to be done. Dysfunction in your ear canals can also cause Dizziness.Therefore you need a thorough evaluation of all the possible causes.Your BP at this stage is borderline and does not warrant you to be on medication. A healthy lifestyle with regular exercising and reduction of salt in your diet can infact being down your BP. Im sure your physician will recommend the same before starting you on any medication."
},
{
"id": 192444,
"tgt": "Suggest treatment for erectile dysfunction during sex",
"src": "Patient: hi i am 23 years old and i have problems getting an erection when i get intimate with a girl. i can get one on my own time, but when im with a girl my heart starts beating very fast and all i can focus on in my head is that i dont have an erection and she is waiting. what are my options? Doctor: Hello, Erectile dysfunction at your age may not be due to any physical issues and could be due to psychological issues. Therefore, I suggest consulting an urologist for physical examination, diagnosis and treatment. Hope I have answered your question. Let me know if I can assist you further. Regards, Dr. K. V. Anand, Psychologist"
},
{
"id": 105263,
"tgt": "Sneezing and runny nose. Quit smoking. Why restriction in nasal passage? Help",
"src": "Patient: I quit smoking on Jan 1 2012 about 2-3 weeks after I began to sneeze and have runny nose. This has continued until Sept. I have taken numerous medications and used sinus rinse for several days now. I have tried nose sprays which only irritate and make it worse. I ve been to an allergist/ ENT and his diagnosis was not allergic to anything and very little restriction in my nasal passages. I m about to the point I d try smoking again and see if this problem goes away. I cannot keep trying to function with this problem I probably sneeze 100-150 times a day and live with a paper towel in my hand to wipe my nose......can anyone help? Thanks Whit Doctor: SMOKING IS NO MEDICINE YOU STOPED DONT TRY AGAIN AND IT IS COINCIDENCT TO HAVE ALLERGIES ACTUALLY YOU PREVIOUSLY HAD IT OR NOT IT WAS GETTING DEPRESSED BY NICOTINE AND IT CAME TO FULL AFTER STOPPING NICOTINE BUT NOW IT WILL NOT GO EVEN IF U START NICOTINE YOU GET XRAY PNS DONE I GIVE YOU TIP STOP ALL MILK AND DIARY PRODUCTS STRICTLY,STOP MUSTARD OIL AND ITS PRODUCTS GET YOUR BLOOD SERUM TESTED FOR MILK.WHEAT,POTATO,AND CHANA AFTER THAT GO FOR SUBLINGUAL IMMUNOTHERAPY FOR PERMANENT CURE TILL YOU START WITH TABH METHOTREXTAE 2.5 MGM ONCE WEEKLY DOSE TAB MONTAIR FX TWICE A DAY SYP TOSSEX 1/2 TSF AT NIGHT WITH WARM WATER APPLY NEOSPORIN H EYE OPINTMENT IN NOSE IN THE MORNING AND SEA WATER IN THE FORM OF DROPS 2 DROPS AT NIGHT YOU CAN CONTINUE FOR LONG TILL YOU GET SLIT"
},
{
"id": 57374,
"tgt": "What could cause recurred fatty liver after being treated while having diabetes and calcium deficiency?",
"src": "Patient: Hello doctor...i am nazeer. now i am 53 years old.i scanned my liver 2010.that time my report was mild hepatomegaly with diffuse fatty changes.after one year treatment i was normal..that time by doctor suggestion i took essential-L capsules.but now i am not normal.again i have fatty liver,swellingand digestion problem.and the quantity of food decreased.after eating i am getting pain when i take immediate rest on bed.and also some undigested food come to my mouth.i have sugar problem also.i have calcium deficiency.so by doctor suggestion i am taking milk..but if take daily ,after 5 or 6 days liver swelling and pain occurs..after taking that tablet the prolem temporary.i use gold winner oil only. i avoided fatty food items.please give me the solution... Doctor: Hi,Diabetes itself is a cause of fatty liver. Other than that it causes gastroparesis ( slowing down emptying of stomach) which gives feeling of distended abdomen after meals. Both being overweight and gatroparesis lead to gastroesophageal reflux,causing undigested food to come to mouth. I think its high time to check your ALT levels. Drugs commonly used for fatty Liver are Metformin 500mg thrice a day after meals and for diabetic gastoparesis, Metoclopramide before meals. Regards"
},
{
"id": 195183,
"tgt": "How can sexual dysfunction and depression be treated?",
"src": "Patient: I am around 27 year old and i am single(Not married). i am facing some sexual issue i think i have low spoerm count and whenver i try to physically intreact with girl ..my body get full warm. Now i am full of tension and getting depresion day by day. Please help me . Doctor: Hi, You are not having any problem according to mentioned history. Your warmth is because of sympathetic stimulation at the time of sexual arousal. For checking your sperm you can investigate with semen analysis report. For anxiety you can practice yoga and meditation. If any other specific complaint present then mention it. Hope I have answered your query. Let me know if I can assist you further."
},
{
"id": 112659,
"tgt": "Back pain. Taking Aceclophenac fom few days. Blood tests normal. Known TypeII diabetic and hypertension. Advice?",
"src": "Patient: I am 58 years old and under medication for type II diabetes and hypertension. Both are under control. I have been experiencing higher right side back pain for last three weeks for which I took pain killers Aceclophenac 100 mg for around 15 days. The pain is still continuing and recent blood report has revealed normal blood sugar, normal lipid profile, haemogram but elevated SGOT (46) and SGPT (66). I have been taking rabeprazole and domperidone everyday. Please advise. Doctor: Hello.GOT and GPT levels (indicators of destruction of liver cells) are slightly elevated, that's not worrying.If your back pain has not improved after taking Aceclofenac for 15 days, I think it is necessary changing to another painkiller and to associate some other drug.In my opinion Tramadol 50 mg thrice a day + Diazepam 5mg at night can be taken for treating your back pain.Please consult your local doctor for a prescription.Only if your back pain does not improve after 4-6 weeks of taking drugs, an radiography would be needed.I wish you good health.(In case the answer would have been useful please indicate this)"
},
{
"id": 223388,
"tgt": "Can Loestin 24 Fe be used as emergency contraceptive after malfunction with condom?",
"src": "Patient: Hi, I was wondering if I could use Loestrin 24 Fe as an emergency contraceptive. My last period was from last wednesday to last sunday, and last month I have missed quite a bit of doses. However, last night, there was a malfunction with the condom and now I am worried. Plan-B is not an option for me, currently. Doctor: ya..u can use it as emergency contraceptive,take 2 pills repeat after 12 hrs.but this has to be taken within 72 hrs of intercourse..consult your gynecologist as well..."
},
{
"id": 93448,
"tgt": "Pain in the stomach, nausea, severe itching. Cause?",
"src": "Patient: I am a 22 year old female and I have pain throughout my stomach and it helps when I eat sometimes but not all the time. Tums doesn't help either. I have normal Bowel movements and urination but the pain makes me sick to my stomach and makes me want to vomit. I had uncontrollable itching for about 9 months and it stoped about a 3 weeks ago with the help of clobetasol propionate ointment. Usp, 0.05%. I still ich a little sometimes but not as bad It's just one problem after another and I need help! Doctor: Hello! Welcome to Health care magic! The abdominal pain when relieved after food and nausea is in favour of a peptic ulcer. If your food habits were irregular or too spicy, or you have lot of stress, this could be a possibility. There are other possible causes too like chronic hepatitis, colitis, etc.And the itching need not necessarily be related to the abdominal pain. itching can accompany abdominal pain in a hepatitis. It can also be due to an unrelated problem - fungal infections of the skin, dryness, etc. So kindly get evaluated throroughly and see to that you get a liver function test and hepatitis screening to rule out liver pathologies. Hope you get well soon! Take care!"
},
{
"id": 59922,
"tgt": "Suffering from jaundice, dark yellow urine, indigestion, stomach upset. Treatment?",
"src": "Patient: hii i am 24years old and have been diagonised with jaundice 5days ago..my urine color was orange in the begining but now it has tuned dark yellow. my eyes also look yellowish.i had indigestion and related problems in stomach 2weeks before this was diagonised..my question is how long does it take me to recover from this? what are the precautions to be taken? how can i know that am completely free from jaundice? Doctor: Hi, Welcome to Healthcare Magic! There are umpteen number of causes of jaundice. The first step is to have a complete liver function test done. Based on LFT report further tests may have to be done to see whether the jaundice is due to hepatitis or obstruction to bile flow. Only after knowing the cause one can tell when you will recover from jaundice. Viral infection is the most common cause and complete recovery can be expected in 4-8 weeks. If it is due to obstruction ( for eg by a stone), removal of the stone will provide rapid relief. So the first step is to find the cause of jaundice."
},
{
"id": 104004,
"tgt": "Child with severe allergy to peanuts and soy. Any way of outgrowing the allergy?",
"src": "Patient: My 6 year-old daughter is allergic to soy. Tests results: 4.62 kUA/L. She is also severely allergic to peanuts (40.80 kUA/L with one severe reaction so far) and 8 other food allergens. We haven't witnessed any symptoms that we could have linked to soy consumption.- What exactly would you recommend she avoids: food that may contain trace amounts of soy? food that contain soy protein? food that contain soybean oil? What about soy lecithin?- How important is it that she avoids contact with soy to eventually outgrow her allegy to soy?- Would daily, moderate exposure to soy increase chances of aggravating her allergy to soy? and her prognostic for outgrowing her allergy to soy?Many thanks in advance,Matthieu Doctor: Hi Thanks for using Health Care Magic Forum, Soy Induced Anaphylaxis in Child with Asthma and Peanut Allergy Soy is one of the top eight allergens. What you may not know is that a child with a peanut allergy can have an allergic reaction after eating a food that contains soy, according to the Asthma and Allergy Resource Center. How could soy cause an allergic reaction in my child with peanut allergy? In scientific terms, soybean allergens are \u201chomologous\u201d to known peanut allergens and can be recognized by 44% of peanut-allergic patients. What that means is that a child with a peanut allergy can have an allergic reaction and even go into anaphylaxis after eating soy. Because this cross-reactivity is rarely mentioned in the press, many parents are unaware of the potential health risks that soy may present to children with peanut allergies. What are the Signs of a Cross-Reaction to Soy? The child suffer fatal asthma attacks. What Can I Do to Protect My Child with Peanut Allergy? \"If your child is allergic to peanuts, you should consider eliminating soy as well as all peanuts from your child\u2019s diet, even if your child has never reacted poorly to soy in the past. Some sensitive children have \u201chidden\u201d soy allergies that manifest for the first time with a severe \u2013 even fatal \u2013 reaction to even the low levels of \u201chidden\u201d soy commonly found in processed food products. Those at the highest risk suffer from asthma as well as peanut allergy.\u201d What products contain soy? Soy can be found in soy milk, tofu, soy sauce, baby formula and other soy products. Soy is also used in many processed foods, as soy lecithin, soy lectin, soybean oil and other soy derivatives. Soy is commonly used as animal-feed in livestock. It is only within the last nine years that soy has become one of the top eight allergens. Soy has recently been genetically engineered to contain new proteins and potential allergens. Daily moderate exposure to soy increase chances of aggravating her allergy . I Hope I Provided Useful Information. Do ask Furthur Questions to help You get well soon. Regards Dr.J.Kingson John David"
},
{
"id": 37700,
"tgt": "Suggest treatment for h.Pylori infections",
"src": "Patient: I was treated for h pylori and unfortunately thought everything would go back to normal. I drank this past weekend and now I am hurting all over again. My doctor says it is gone but why are the symptoms still here and what can I do to make it better Doctor: Hello,Welcome to HCM and thanks for your query.The treatment for H.Pylori is a triple regimen which include a proton pump inhibitor (PPI) and two antibiotics. If the doctor has given you these and you had taken them regularly, then you probably do not have it. The way to make sure that it is gone is to do a urea breath test after a month of the treatment.If your pain recurred after drinking alcohol, it is probably due to alcohol induced gastritis and not H.Pylori.So my suggestion would be to do a urea breath test and if it is negative, keep off alcohol and take a PPI for 1 month to help the gastritis heal.I am available if you have queries,Wish you good healthDr. Noble Zachariah"
},
{
"id": 75063,
"tgt": "How long will it take to cure cough with medication?",
"src": "Patient: I have been on antibiotics for five days and I am still coughing up and blowing out yellow mucus. The cough takes my breath and I am using Symbicort up to 5 times a day . My doctor keeps telling me to give it 1 to 2 more days. How long should I wait before contacting him again or find another doctor? Doctor: cough is a reflex to help you bring out secretions built up in lungs. it is uncomfortable and painful if it is making you breathless.usually it takes 1- 2 weeks based on immunity of the person to cure oneself of infection.There are cough suppressants to get reliefbenzonatate pearls to take thrice daily for two days and then twice when cough reduced.cough lozenges finger honey lemon flavors, Strepsils also quite helpful.You need to avoid triggers that cause attacks of respiratory infections like dust smoke pollution exposing to infected persons.take precautionary antihistamines to prevent allergens."
},
{
"id": 171873,
"tgt": "What cause fever and rashes on an infant s body?",
"src": "Patient: I have a 17 month old grandson who has what seemed to be a minor cold for the last 2 weeks. He threw up a couple of times in the last 2 days, (mostly bile and water), and had a temperature of slightly more than 100. He went to the pediatrician with a heartrate of about 200. He is now in the hospital where he has had bloodwork that came back normal except for a couple bands high in his wbc count. His hr is now about 140 but when he s upset it spikes to 180. Chest xray appears a normal heart and lungs, but they are saying his lungs may have borderline pneumonia? He also has presented with a rash. Any ideas? Doctor: Your child is suffering from the ill-effects of a lung infection which may have occurred initially as a virus, but got complicated into a bacterial one about the time he had to be put in a hospital. All the symptoms and signs and the investigation results prove that the infection is a rather serious one, so he should be on a path to recovery if the antibiotics they are using on him are working well. The rash could be part of the manifestations of the germs that gave him the infection, or it could be due to the antibiotic that is currently being given to him.Hope this helps!Dr. Taher"
},
{
"id": 62734,
"tgt": "What causes a hard sore lump inside the knee?",
"src": "Patient: i have a bump on the inside of my left knee. it is on the inside of my left knee in the middle sort of ontop of the knee cap. its very painful, and hard? i can t really put pressure through my leg. i have patellar subluxation, but this is different. i was wondering, what do you think it is? Doctor: Hi,Welcome with your query to HCM virtual Clinic.I reviewed all the details of your query.Based on the facts of your query,the cause of the sore knee hard lump seems to be-Painful PrePatellar Bursitis(Housemaids Knee) with accompanied Patellar subluxation /or it could be Synovitis as you had history of knee trauma which led to Patellar Subluxation,formed due to hard hit ,as you don't deny having hard hit.Would suggest USG/Xray knee left/right to fix this ambiguity.As I don't have direct information and as you haven't yet shown to any doctor except your cardiologist,I would suggest to take Second Opinion from your Orthopedic Surgeon/ or General Surgeon.He would investigate /check and treat it accordingly.Hope this reply would help you to plan treatment with your doctors and resolve your worries.Will appreciate writing excellent review and feedback comments to help the needy patient visitors like you at HCM.Welcome any further query and information in this regard.Contact at My HCM Clinic,with direct Premium question for further suggestions if need be.Good Day!!Dr.Savaskar M.N.Senior Surgical SpecialistM.S.Genl-CVTS"
},
{
"id": 213369,
"tgt": "Habit of eating raw rice, feel relaxed and stress free on eating. How can I overcome this habit?",
"src": "Patient: Hello doctor,i have a habit of eating raw rice(uncooked rice) from my childhood. i really dont know how to stop it...i learnt from one of your doctor that its a behavioural disorder... when i eat raw rice i feel relaxed and stressless...when i m stressed or tensed i eat it ...could you please help me come out of this bad habit.... Doctor: Ok u can withdraw it at once..u should decrease this habit slowly slowly And u should start few medicine which may help u too Syp.benefecial 2tsf once in a day tab.shelcal 500mg 1tab once a day"
},
{
"id": 87077,
"tgt": "What causes a painful and swollen tummy?",
"src": "Patient: I have a very swollen tummy, it is painful and noticeable, I look pregnant. I am 51 and Not pregnant as husband had the snip when our 13 year old was born. I had numerous polyps which were causing extreme bleeding during my period and I was checked for menapose hormones end of last year. They said I was not producing the markers. I and the Marlena coil (not sure correct spelling) back end of the year and it has addressed the bleeding. I only have odd spotting till last week. I had slightly more bleeding, needed to wear a pad for a few days, very dark ans stringy. My husband noticed the swelling last week, but I really an see it now, it is painful, I get pain in all positions, sitting standing it is breath taking at times. Lower tummy pain and slight tenderness when pressed. Bleeding has all but stopped. Doctor: hello miss, it seems u r suffering from PCOD. the cysts have grown and may be due to some reason there is pain. you should immediately cnsult a gynaecologst."
},
{
"id": 126619,
"tgt": "What causes painful stiffness in the knuckles and joints?",
"src": "Patient: My ring finger is red, swollen, and feverish at the end to my first joint. I went to dr and she thought it was due to infection around my cuticle. I ve been on antibiotics for 5 days. Not much change. Finger still red, slightly swollen, knuckle is stiff and painful. I ve soaked it in hot water and salts. What should I do? Doctor: Hi, It might be due to cellulitis. Consult an orthopedics and get evaluated. You might require higher end antibiotics for better results. Hope I have answered your query. Let me know if I can assist you further. Regards, Dr. Shinas Hussain, General & Family Physician"
},
{
"id": 210305,
"tgt": "What medicine will calm an elderly diabetic person with cataract?",
"src": "Patient: My father is 65 year old and he is behaving very wierd and beating my mother who is 60 years old with anything and is just fighting with all the childrens and wife and making our life hell, plz prescribe some medicine to calm him. he is diabetic, is having a cataract in both of his eyes, and we want him to get operated but he is not ready for that also. Doctor: Hello,I understand that you are quite distressed about your elderly father's aggressive behaviour. Now, this kind of a sudden aggressive behaviour can be due to a variety of causes - like late onset psychosis, delirium, etc. It is very important that he has a proper evaluation in order to find out the cause of his symptoms. Only if the correct cause is identified, further treatment can be advised.So, I would suggest that you take him to a psychiatrist for a detailed evaluation and further treatment.Regards,Dr. Jonas SundarakumarConsultant Psychiatrist"
},
{
"id": 55113,
"tgt": "How safe is to take apri 21 when having gall stone?",
"src": "Patient: Hello. I am 33 and never took Apri21. I wanted to know if it s ok to take this medication when I just found out that I have gallstone. Also, since I had my son three years ago I been having problem with my period. ..heavy flow, cramps and pain, and It last for 7 days..After that I start feeling that I m ovulating and the discomfort continue until my period start again. Doctor: hi.what is the generic name of Apri21, and is it FDA approved? personally, i am not against but i do not recommend taking in of supplements/medications if they are not FDA approved or hasn't passed the clinical trials recommended for approval. majority of the medications/supplements we are taking in, whether with medical indications or none, will pass thru our liver for metabolism, and kidneys for clearance. this may cause problems, maybe not now, but in the future. it is best if you consult with a doctor for physical examination and clinical evaluation first. diagnostics and management (medical and surgical) will be directed accordingly. for gallstones, definitive treatment is cholecystectomy or surgical removal of the gallbladder. it could be done laparoscopically (goldstandard) or thru open technique. low fat diet is also recommended.hope this helps.good day!!~dr.kaye"
},
{
"id": 12634,
"tgt": "What is the treatment for Psoriasis ?",
"src": "Patient: Which is better for psoriasis , allopathy or homeopathy? Doctor: Homeopathy is crap man. They just make false claims. I have seen many people failing with it. In allopathy there are different modalities like tablets, creams and radiation as well."
},
{
"id": 102362,
"tgt": "What do you suggest for painful headaches with allergies?",
"src": "Patient: Hi I have had painful headaches lately and I think it s caused from my ears always feels like its full of liquid in there I have allergies so I take the nasal spray for that and I m waiting to go see the ear n throat specialist but that s takin a while how can I keep clear from gettin the headaches Doctor: Hello, Welcome to HCM, The allergic manifestations can result in wide range of symptoms from generalized hives to Anasarca. You have to confirm first that whether you are having allergic reaction by testing for absolute eosinophil count (AEC) and then you need to identify the allergen causing these symptoms. The allergen should be identified by skin prick test. After identifying the allergen then we can decide the further mode of treatment. Immunotherapy can be considered is there is a known allergen. Thank you."
},
{
"id": 224581,
"tgt": "Is it safe to take 3 I-pills in the same month?",
"src": "Patient: i had my periods on 2nd sep and it was early coz of the intake of i pill. otherwise it was supposed to be on 7th.. that was the first time i took i pill.. n after 2nd sep, its third time that i have taken i pill coz of unprotected sex in this month.. please guide me what should i do? will ipill cause any severe problem because of takin it 3rd time in a month?? i will not repeat it after this.. i am totally worried..:( please help .... (sneha, 21 years, mumbai). Doctor: Hello Sneha and welcome,I understand the issue. As of now as you have already taken the pills we cannot undo it. But apart from causing irregular periods this must not be an issue to worry about. I must inform you that I pill is an emergency contraception that is used only during accidental emergency intercourse that is unprotected. It is effective only once in one cycle and repeated use will not provide protection against pregnancy as there is no data to support this type of use. So please remember this during future use of I pill and do not worry. Hope this satisfies your query. Thanks for using HCM.\u00a0\u00a0\u00a0\u00a0\u00a0Feel free to ask any more questions that you may have. Dr Madhuri BagdeConsultant Obstetrician and Gynecologist"
},
{
"id": 125292,
"tgt": "What causes pain in foot while sitting?",
"src": "Patient: i find it hard to indian sit. my right foot ( the one underneath whenever i sit this way) hurts. I think the pain is near my ankle. i have an active lifestyle, i play sports, dance and stuff. i remember having a sprain 3 years ago but i forgot if it s my left or right foot but i think i ve had a sprain more than once Doctor: Hello, As first line management, you can take analgesics like Paracetamol or Aceclofenac for pain relief. If symptoms persist you can consult an orthopaedician and plan for an MRI scan. Hope I have answered your query. Let me know if I can assist you further. Take care Regards, Dr Shinas Hussain, General & Family Physician"
},
{
"id": 34873,
"tgt": "How is enterobacter aerogenes usually treated?",
"src": "Patient: What is Enterbacter Aerogenes, as I was diagnois with this in April? It was also discovered that I have food intolerances. My doctor had placed me on antibotics for the Enterobacter Aerogenes but it made me feel like I had respiratory problems so I came off of it right away. Then my cardiologist placed me only olive leaf extract 2 a day, but it has made me constipated, but it really helps my circulation problems that I felt and my sinus problems. Can garlic help with Enterbacter Aerogenes to get rid of it? Doctor: Hello dear,Thank you for your contact to health care magic.I read and understand your concern. I am Dr Arun Tank answering your concern.No, this are enteric bacteria and won't be treated with home remedies. If not treated properly their consequences are very bad.I advice not to take home remedies and other trials. You can instead go for culture and sensitivity of the bacteria responsible.Drugs which is taken after such a reports can cure you not only at earliest but also effectively.Gralic has some natural bacteria killing property but scientifically it is not much significant. So instead you can go for sensitivity testing.In any way good living practice should be adopted. That is good food with high protein, high carbohydrate low fat and low spicy food should be consumed.Exercising everyday for approx half hour early in the morning is also beneficial. Such life's style can clear all your problems.I will be happy to answer your further concern on bit.ly/DrArun.Thank you,Dr Arun TankInfectious diseases specialist,HCM"
},
{
"id": 95473,
"tgt": "Having green stool with black spots",
"src": "Patient: i noticed my stool was in green colour with black spots in it what is the reason, is it anything serious? Doctor: yWgB4r syhixbwxtkpi, [url=http://ubsqtxhhuqku.com/]ubsqtxhhuqku[/url], [link=http://sxniozbaqwkl.com/]sxniozbaqwkl[/link], http://oraskkxfqflb.com/"
},
{
"id": 22300,
"tgt": "Suggest remedy for heart ailments",
"src": "Patient: as per my doctor or echocardiography with dopler report my arotic valve is not properly shut , ihave seen in report in 2008 my herat inject capacity is 70ml, in 2009 it go down 66ml and in 2010 it 55ml . Now i want to know is it oprable stage? if not then when should it necessaries?my age 39years,hight-5'11''weight-68kgs Doctor: Hi,You probably have aortic regurgitation, surgery will be done if you are having symptoms like shortness of breath. Also, surgery decision depends upon your ejection fraction if less than 50%, and dimension of left ventricle at end diastole is more than 50 mm. So under these conditions surgery should be done otherwise 6 monthly to yearly echo should be done.Hope I have answered your query. Let me know if I can assist you further. Regards,Dr. Sagar Makode"
},
{
"id": 43225,
"tgt": "Undergone IVF, ET done using two embryo with grade 1. Negative hcg. Suggested sustan injection. Suggestions?",
"src": "Patient: hello doctor, good morning, recently my wife went in IVF treatment and embroyo transfer done on 15/9 and based on DR. request done Beta hCG on 29/9 which output is -ve. we are surprised becoz ET done using two embroyo with Grade 1. still how this possible, we are very much worried here. and during this period she also suggested to take sustan100mg injection daily.
Doctor: HelloThanks for your query.Based on the facts that you have posted it appears that your wife has undergone IVF treatment and two embryos were transferred.The beta Hcg test done after 10 days have unfortunately turned out to be negative indicating failure of IVF.There are 40-50 % chances of failure of IVF and no one can explain the reasons for failure.One has to accept the reality with the hope for +ve results in next IVF cycle.Dr.Patil."
},
{
"id": 65990,
"tgt": "How can cold sore on crease of mouth and lump under chin be treated?",
"src": "Patient: hi want to know wat i have wrong with me i have a coldsore on my left hand side in the crease of my mouth which is very painful an dry an i also have a lump under my chin or tounge with hurts if i touch it with my fingers or softly touch it its like a swollen gland but im unsure how do i fix this? Doctor: Hi,From history it seems that there might be having Aphthous ulcer producing cold sore at angle of your mouth.Due to not getting rest of the part due to talking,eating etc healing of ulcer delayed.Due to this infection there might be havingenlarged tender sub mandibular lymph nodes.Go fro one antibiotic medicine course for 3-5 days like Amoxicilin.Apply antibiotic cream or gention violet locally.Ok and take care."
},
{
"id": 159095,
"tgt": "Suffering from endometrial cancer. Pain in abdomen and urination short up. Relief from pain and reduce urination frequency?",
"src": "Patient: My mother is suffering from endometrial cancer . Her creatinine in dec2012 was 1 and now on 16feb13 it is 2.07. Could giving her combiflam aggravated her creatinine? She has moderate hydropherosis. In the last 6 days her urination has short up to onc eevery 2 hrs. What could be the cause for the urination. Also her pain in the abdominal AND pelvic region.She is on endace 40mg four times a day since mid dec2012. The spread is to her lungs and spleen(mildly) and contained since its detection in Dec2012. What should I do to relieve her pain and reduce urination frequency. Please advice. Doctor: Hi, Combiflam can not be given in impaired renal function. You should better try opioid group of drugs. Increased frequency of urination may be due to urinary tract infection, or bladder irritation due to local spread of disease, so a urine routine and culture may be helpful then the drugs can be selected accordingly. Consult your oncologist."
},
{
"id": 24495,
"tgt": "What does this angiography report results indicate?",
"src": "Patient: My history is: Antoroseptal MI on 14.02.11, thrombolysed with STK. Angiography (24.02.11) results: LMCA-Nor.; LAD-40% Ostial Segment Stenosis & 90% mid Segment Stenosis; D2-80% Ostial Segment Stenosis; LCx- 30% mid Segment Stenosis. PDA-RCA Ostial Segment Stenosis. Coronary Angioplasty Report (24.02.11): PTCA / Stent (XIENCE Vascular) to LAD + Inj. Faximab with good end results & no residue stenosis or flap.Discharge Summary (26.02.11): Acute Anterior Wall Myocardial Infraction, Double Vessel Disease, PTCA + Stent to LAD, LV Dysfunction (LVFE 45%).Stress Echo (16.06.11) Report: LVFE 50% at rest & 60% at Impost. Trace MR at rest & No MR at Impost. Impression: Negative for reversible Myocardial Ischaemia.Myocardial Perfusion Scintigraphy Report (19.01.12): Test done following i/v Inj. of 110 MBq of TI201 at rest & 444 MBq of Tc99m Tetrofosmin at Bruce. Exercised for 06.51 Min. & attained heart rate of 148 BPM (89% MPHR). Test Reveals: Post stress images show reduced tracer uptake in septal / anteroseptal & apical wall of LV myocardium. Refilling kinetics is seen in above mentioned area at rest thallium images.Impression: Features of SPECT myocardial perfusion scintigrapghic study are showing evidence of exercise induced reversible ischaemia in Septal / anteroseptal and apical wall of LV myocardium.Personal observations: Post angioplasty I used to have throat irritation on exertion. Even during stress echo on 16.06.11 & on 19.01.12 the throat irritation / chocking feeling increased & exercise had to be stooped.Now what do you conclude & suggest future course of action.I shall be highly obliged for your expert consultations / suggestions. Thanking You & RegardsAnil Goel+91 0000 YYYY@YYYY ; YYYY@YYYY Doctor: Dear Anil Goer,On your angiography in 2011, there were significant narrowing of two coronary arteries and some non significant marrowings which couldn't cause any ischemia and complaints. One significant marrowing was treated with stent, but the other one remained. I think they didn't perform stenting of the other artery as the stress test results were good in 2011, but of you have positive stress test results in a year, then there is a need to repeat coronary angiography to evaluate arteries again, there may be two possibilities, either the other narrowings became more and are significant now or it is the D2 from the first angiography causing the problems. In both cases angioplasty (stenting) may be needed.Take careCome back if you have any further questions"
},
{
"id": 176478,
"tgt": "What causes stomach cramps along with sore throat in children?",
"src": "Patient: my little girl will be 6 next weekend, shes complained of stomach cramps for a few weeks an most to nearly everyday, yesterday she had a sore throat an she woke up with a sore neck, an for 2 days her poo is green...her diet is pretty poor I try make her eat her vegies but usually gets out of it but is obcessed with salt an vinger chips? any answer u can think of? Doctor: Hi, the child has gastritis along with an Upper airway infection. Spicy food usually causes gastritis and reflux in children. Sore neck could either be due to gastritis and reflux or due to infection. Give her tab Junior Lanzol 15 mg one tablet once daily empty stomach in morning for next 3 to 5 days. Also de-worm the child with tablet Albendazole 400mg one tablet stat. Avoid spicy and oily food and give her regular food like curd and rice. For sore throat, dont give any antibiotics right now since she just have pain in the throat and not running high fever or any other symptoms. Supplement her with Syrup B complex 5 ml once a day for 15 days.Thanks and regards."
},
{
"id": 120138,
"tgt": "Suggest remedy for numbness in face",
"src": "Patient: My sister woke up with left side of her face numb and the dr. said it was bells palsy, she got steroid shot but then the next day she was so dizzy when she opened her eyes and became weak in the legs. CT done no tumors or stroke. went to ent and said she does not have vertigo or bells palsy. Is on b/p med and b/p is controlled and has no other health issues. Doctor: Hello,Weakness in legs, numb face in a person with high blood pressure is denoting towards a neurologic cause. In such a case, she may need a opinion from a neuro-physician. The advanced investigations like MR Angio are more helpful in such cases.You should not wait. Hope I have answered your question. Let me know if I can assist you further. Regards, Dr. Mukesh Tiwari, Orthopedic Surgeon"
},
{
"id": 28057,
"tgt": "Suggest treatment for final stage of heart failure",
"src": "Patient: My mother is 89 years old. over the past few days they have had to tap fluid off. Each time it is equalt to a litre. Nobody will give me an answer as to where this is going. Are we in the final stages of Heart failure? What can we expect going forward? Doctor: Hi,It is really difficult situation for you.I want to know1) Cause for heart failure in your mother (like heart attack) if you are knowing.2) Any previous records available 3) Any associated conditions like diabetes, blood pressure problem, cholesterol problem, kidney problem. 4) Any symptoms like cough, cold, fever, swelling over body.There might be following causes for fluid collection in chest1) Heart failure.2) Infection of lungs.3) Kidney failure.Following investigations may be helpful in management 1) Blood tests - Haemogram, Kidney function test, liver function tests, BNP, Electrolytes, calcium, magnesium, phosphorus .2) Urine examination for infections, urinary proteins.3) ECG, 2DEcho of heart (to know pumping capacity of heart)4) Test on fluid collected from chest.5) Ultrasound of chest, abdomen and pelvis.6) CT scan of chest.Depending upon cause she may be benefited by -1) Antibiotics.2) Diuretics."
},
{
"id": 113418,
"tgt": "Pain in the lower back rib, above the belly button, fluctuating pain. Dieting for weight loss. What can the pain be due to?",
"src": "Patient: hello- i have had a slight pain on my left lower back rib area and when feeling around i felt a slight bump..proabbly about the size of a pea, and it was very tender...i asked my doctor about it when i went in for my yearly, and she stated it was probably just fatty tissue , and with me trying to lose weight it should go away. that was a year ago... recently i am having more pain in my left side on my love handle area straight over from my belly button...shart pains that also come and go..and i just felt around and felt the same size pea lump that was tender. It is very tender and if i go back, im afraid she will just tell me the same thing and not to worry about it. What do you feel this could be and do you agree with my doctor. Note: I am 23 years old, 215lb. and 5 3 ....and dieting to lose weight. (no kids) Doctor: Hi thank you for sharing your problem here. These painful nodules are quite common and in some people they swell up more when they do strenuous activity. Usually no treatment is given, but in severe painful situations a local injection over the swelling may be given"
},
{
"id": 218675,
"tgt": "What does this pelvic ultrasound test result indicate?",
"src": "Patient: Hi! I m in 31 weeks pregnant and have gone an ultrasound with diagnosed of single, live, intrauterine pregnancy, cephalic 31 4/7 weeks sonar age with good fetal cardiac and motor activity, anterior high lying placenta grade 2 maturity index. Adequate amniotic fluid volume. fetal congenital screening appears grossly normal. My doctor didn t explain this to me, what does anterior high lying placenta grade 2 maturity index means? Doctor: Hi there, I have understood your concern and I will suggest you the best possible treatment options.First of all do not panic.USG report is showing good fetal growth corresponding to the period of pregnancy.Baby is normal in appearance.Bag of water around the baby is having adequate amount of water in it.Placenta is located at its normal location and is not low lying. This rules out placenta previa- a condition that causes bleeding issues.Grade 2 maturity is indicative of ongoing function of the placenta to provide nutrition to baby.So please do not worry about the reports..Please opt for a healthy diet and regular exercise regimen.Include more portions of fruits and vegetables and salads in daily diet.Drink plenty of water in the day.Avoid refined sugars and deep fried foods and bakery products in the diet.Please take Iron, Calcium, protein and vitamin B 12 and Folic acid supplementation on regular basis.Think positive.Meditate regularly.Practice deep breathing.May God bless you with a bundle of Joy.I hope this answer helps you.Thanks.Dr. Purushottam Neurgaonkar"
},
{
"id": 195865,
"tgt": "Suggest treatment for chronic follicular cystitis",
"src": "Patient: Hi.... i just want to ask a question on behalf of my father he is having a problem called chronic follicular cystitis (medical term) and in common language fibrosis in urine bolder,Due to which he has frequency in passing urine increased, burning sensation on passing of urine.He has also operated 3 times once for prostrated and other for chronic follicular cystitis, but no result.We have also gone for ayurvedic treatment taken ayurvedic medicine named as Chandra prabhawati, Shilajit, Guggul etc.Please give us your valuable suggestion for above mention problem.RegardsAnita Doctor: Hello,There is no single treatment option for follicular cystitis. An effective course of treatment for one patient may not work for another. Antibiotics are prescribed if infection accompanies inflammation of the bladder. Drugs used to treat follicular cystitis are some of the same ones used to treat chronic cystitis. These may include bladder coatings, antidepressants, and antihistamines. Antispasmodics and bladder anesthetics may also be used.Surgical options, such as bladder augmentation or cystectomy, are sometimes considered, but only as a last resort after all other options have been exhausted.Hope I have answered your query. Let me know if I can assist you further.Regards,Dr. Penchila Prasad Kandikattu"
},
{
"id": 83503,
"tgt": "Is there any alternate medicine other than epsolin for proper ECG and MRI?",
"src": "Patient: i am taking epsolin er 300 on doctor advise. on completin of three years of of medice and after proper eeg and mri tests the doctor has adjusted the medice as 100 mg of epsolin plain 12 hourly . should i go fo the adjusted medice or there is any other alternate Doctor: Hi, You may continue with the adjusted dose. As long as you do not have any issue with Epsolin-ER you may continue using it after its adjusted dose. Otherwise carbamazepine or levetiracetam can be useful alternatives to control the seizures. Hope I have answered your query. Let me know if I can assist you further. Take care Regards, Dr. Mohammed Taher Ali"
},
{
"id": 24984,
"tgt": "What causes heart pain and murmurs?",
"src": "Patient: I have had bouts of bradycardia for about 1.5 years--no insurance now so I can only go on how I feel and what my bp machine says. In the last 6 months, I had bradycardia--rate 40ish--with EKG and two days later I was in the ER with tachycardia and high bp. I'm on a waiting list for a cardiologist. Now, I went to the doctor a couple days ago and EKG showed enlarged left atrium and he heard a heart murmur, I didn't know to ask what kind at the time. In the last couple weeks, when I have chest pain, I also have a pain in my left neck. I'm 47, 5'2\", 165 lbs., trigs in the high 600's. I have lost 10 lbs. recently and about 35-40 over the last year (trying to). How serious is this? Is my heart getting worse? If so, how and why? What can I do? Doctor: HIWell come to HCMI really appreciate your concern, if this the bradycardia then underlying cause of matter of concern, some time it may be physiological one if does not have any clinical symptoms, it may not be serious condition, if this is the really heart disease then it needs to be differentiated as long as the murmur is concern, hope this information helps"
},
{
"id": 9104,
"tgt": "I want to pierce my nose under local anesthesia can it be done in a hospital ?",
"src": "Patient: i want to pierce my nose under local anesthesia can it be done in a hospital, if so then can u pls suggest a hospital in coimbatore? Doctor: You can just go to any body, tattoo, or piercing shop and they'll pierce it for you there with a piercing gun, which just feels like a little pinch and it goes very fast. It wont hurt very much after there done, just for the first day or so and then it will be fine after that. It will be a little bit pink around the piercing for a few days to a week, but that it normal. I did mine by myself with a needle, which hurt a little bit more because it didn't go as fast as the gun, but it was certainly worth it! saved money, you know? so that's another option you can try. I hope this helps!You are going to look adorable with your piercing by the way, so go for it! :)"
},
{
"id": 92791,
"tgt": "Have upper abdomen pain. Diagnosed with Lax LES , Antral gastritis in endoscopy and fatty lever in ultrasound. Suggestions?",
"src": "Patient: Hi doctor,I have been suffering from upper abdomen pain from last 15 days. I went to a specialist and was diagonised with Lax LES , Antral gastritis in endoscopy and fatty lever in ultrasound. I am prescribed with1. Sompraz d 40 mg - 1 empty stomach2. Ursocol 300 mg -2 during day 3. Evion 400. -1 at bedtime. I am taking this medicines from last 4 days but there is very less relief.Can you please suggest if I need to consult Any other doctor and also what should be the diet. Doctor: Hello, Thanks for the query to H.C.M. Forum. If I were your treating doctor for antral gastritis I would come up with following measures for your treatment. Try raising the head of your bed about 4 inches with blocks. It also might also help to avoid eating or drinking 2 hours before you lie down. To help control the stomach acid one should not drink alcohol or drinks with caffeine in them or eat chocolates or spicy food, greasy foods. As omeprazole provide symptomatic relief , while ursodeoxycholic acid acts over liver and evion is Vitamin E . In such type of patients I usually prescribe rebamipide 100 mg 2-2-2 for 10 days along syrup antacid with oxetacaine. Good luck. Dr. HET"
},
{
"id": 8149,
"tgt": "Can acne occur while being anorexic ?",
"src": "Patient: I d like to know if acne will go away while being anorexic or will it show up Doctor: Anorexia will lower the immunity of the body and in turn all infectious diseases get a chance to spread more."
},
{
"id": 173320,
"tgt": "Can symptoms of herp and strep be seen in 8 month old kids?",
"src": "Patient: Can symptoms of herpes ( 1 and 2) show up in my son 8 months after delivery? He had a severe diaper rash at a week old, thrush at 11 days old and now at 8 months he has these red spots on the roof of his mouth, a creamy white residue and red spots on tongue. Dr said the spots on roof of mouth look like it could be strep but said very rare in children this age. Doctor: Hello dear, yes it is rarely,but happens when baby has low immunity.For diagnosis confirm you should perform mouth culture for flora and sensitivity to antibiotics. Moreover,herpes 6type,EBV,CMV can cause following up symptoms . For confirmation we test babies Ig M, G to herpes 1,2 types,HHV 6type, CMV,EBV.Hope I answered your query.Wish your baby speedy recovery"
},
{
"id": 191372,
"tgt": "What causes breathing difficulty and swollen feet while suffering from diabetes?",
"src": "Patient: I am a diabetic and have been for years but for the past couple of months I have had trouble breathing and my legs are like concrete. They are swollen so are my feet and I have no ankle bones anymore that you can see. My feet are swollen also and it is hard to walk after a few hours. I do work on my feet for 8 hours a day. Doctor: Hello,If this is the swelling of the lower limb, then it could be due to a long-standing position that causes the return of blood difficulty it is the most common reason seen if the swelling subsides on its own in the morning.And if this is not the reason, then it could be the renal disease, cardiopulmonary disease, with diabetes such symptom of lower limb swelling need to be taken seriously, I would suggest to get done the test for renal disease and for cardiopulmonary disease.Hope I have answered your query. Let me know if I can assist you further.Regards,Dr. Akhtarhusain"
},
{
"id": 64136,
"tgt": "What is the lump with a mole on my face?",
"src": "Patient: HI on my right side of my face I have aet lump with a mole on it the lump makes my mole poke out fauther on my face the lump hurts just a tiny bit its about the size of my thumb it appears and goes my mole is black and as round as my small pinky finger Doctor: Hello!Thank you for the query.This seems to be two separate things located in the same area. First is a mole which is a skin lesion. Second, located deeper can be a sebacues cyst - benign lesion filled with whitish masses. As this cyst tends to get infected it can come and go appearing as a red and painful area.As it is close to the mole, I suggest you to consult dermatologist. Both things should be removed and tested.Hope this will help.Regards."
},
{
"id": 34063,
"tgt": "How to cure the condition of gonococal infection?",
"src": "Patient: i have a chronic type of whitish discharge and i try to check what it was, and i found out that i got gonococal infection. then i take treatment for this infection.while i get rid of the gonococal i still got gram negative rod shaped bacteria and staphilococcus, and also yeast infection. i try home remedies as well medicene but i cant get rid of the discharge. pls help me out.am 24 years old 155cm and 53kg weigh. Doctor: Hi,Welcome to HCM.I am glad that you have been treated for gonococcal infection. Based on the details you have mentioned in your query, you are suffering from genital infection which commonly presents with discharge, itching and pain during passage of urine. I suggest you get culture and sensitivity test done for the discharge and take appropriate antibiotics based on the test results.Antibiotics have to be taken for atleast 7-10 days. Strict personal hygiene has to be maintained.As you have a history of sexually transmitted infection like gonorrhea, I suggest you also get a consult at an STI clinic to rule out any other STIs.Thanks."
},
{
"id": 25507,
"tgt": "What causes skipping of heartbeat, chest tightness and lightheadedness?",
"src": "Patient: hi doctor,i have skip beat i guess.sometimes it happens for many days before going away.when i m having it,symptoms like very mild tightness in chest,lightheadedness and fatigue will follow.there would be uncomfortable feeling in chest.i m worried can it cause sudden heart attack?thanks Doctor: Welcome to HCM: Please note that this sounds like Atrial Fibrillation, and it can cause a stroke. Please go and see a doctor and get an EKG and get evaluated. Of course it can lead to a heart attack, but a stroke is major concern. I hope this helps, good luck"
},
{
"id": 34216,
"tgt": "Suggest treatment for virus flu",
"src": "Patient: Hi, i just want advice, i have had flu for the past 2 weeks, i dehidrated and went to the hospital to be \"re-lubed\". after the 2 weeks i had to go back to the docter for a secind check up but he told me that the flu virus entered my blood sysetem. i am currently sweating terribly, its hard to breath and if i just get to sleeping peacefully i break out in a cold sweat 3 times per night and have to change cloting because im am so wet. what can i do? im frustrated. Doctor: Thanks for posting you query to health care magic.If it is a viral flu then it will be recover spontaneously after completion of virus life sapan .but we should not forgate to find out bacterial cause also .So I suggest you some investigation that will help in making diagnosis.1.Blood culture and sensitivity .2.X ray chest to find level of lung inflammation.3.Complete blood count to detect level of defence cells of body and its correlation with infection.presently you should take good nutritious dite , drink plenty of water and take rest .review me after report to suggest you further managment .Hope you would be satisfied with my answer . Feel free to communicate if any query .regards,Dr.Manish PurohitInfectious disease specialist"
},
{
"id": 147144,
"tgt": "What is cerebellar atrophy with no signs of dupal venomus sinusetc and what could be the treatment for patient with stroke history?",
"src": "Patient: Good afternoon doc! My sister had a stroke some 10 yrs ago. Dr. Prescribed anti epilectic drug. 2 days ago she got severe headache in left side of head (back). MRI says changes of Cerebellar atrphy. No signs of dupal venomus sinusetc..what is it can it be treated? Doctor: atrophy is a sign of nerve volume loss and when in the cerebellum can cause problems with coordination and possibly memory. there are no sensory pathways going to the cerebellum and no sensation in it. this MRI finding does not explain the pain in the back of the head. the treatment for atrophy is to reverse or stop the cause. this requires a more extensive conversation with the doctor who ordered the MRI ."
},
{
"id": 48424,
"tgt": "What causes yellow urine with odour accompanied by lower back pain and chills?",
"src": "Patient: Hi Dr. Grief, I have yellow urine with an odor even though I drink alot of water a day. I also have lower back pain which maybe kidneys, but I also have deg, disc in my lower back (difficult to issolate the two). I've also had chills and sweats... but not very often. No pain during urination. Doctor: Hello and welcome to HCM, Lower back ache along with chills can be due to urinary tract infection. Yellow colored urine is not suggestive of urinary tract infection. In case of urinary tract infection, the urine is usually turbid or contains pus. In case there are other urinary symptoms like burning sensation, increased frequency of urination, urinary tract infection is likely. However, you need to get examination of the urine to assess the presence or absence of urinary tract infection. Thanks and take care Dr shailja P Wahal"
},
{
"id": 93498,
"tgt": "Have pain in stomach, bloated, frequent urination, panic attacks and blood in urine. Guidance?",
"src": "Patient: Hi ya!I'm a 31 year old woman!I've been to the doctors and he thinks I've got a stomach bug!I'm not convinced!I've got a pain in my tummy,sometimes under my rib and in my side and back!my tummy is bloated,I'm really tired and have gone of food or when I eat I can get tummy pains!I'm weeing more then usual!when I went to the doctors my urine was quite a dark colour and had a bit of blood in the urine but doctor said that that's normal!I thought urine should be clearer rather then darker!I also suffer with bad panic attacks sothis is making me worse!Could you please help me as I'm worrying!Thanks! Doctor: Calm down and then get urine routine and microscopy examination done and make it clear if there is real blood in urine or is it concentrated and hence dark colored.See your doctor with this and he/she shall be able to investigate further if required and shall help solve your problem."
},
{
"id": 14890,
"tgt": "How can rashes and blisters on vagina and thighs be treated?",
"src": "Patient: Hello doctor, I have a rash on my inner thighs and blisters around my vaginal area that itches and produces puss. It hurts when i try to clean that area in the shower. My boyfriend also told me that he is begiinning to get the same rash on his inner thigh. Wnat can it be? Doctor: Hi,I can understand your concern for rashes and blisters on vagina and thighs. There can be multiple possibilities of the rashes and blisters on vagina and thighs and it requires a close physical examination to diagnose the exact entity of the problem.In my view correlating with the facts you have provided it appears to be fungal infection of the area.I suggest you to meet a dermatologist to for proper therapy. In between you should wash the area properly. If I were your dermatologist then presumptively would have preferred to apply an antifungal cream such as clotrimazole or miconazole. You can take an antihistamine such as loratadine for once a day if itching is present.Do not try to pinch, pop or squeeze the area as it may cause secondary infection of the area.All the best"
},
{
"id": 43779,
"tgt": "Done semen analyis. Can I take Addyzoa?",
"src": "Patient: My semen analysis is as follows. Volume: 2ml, Self Liquefaction: 30 mints, Appearance: greish white, Viscosity: normal, PH: 8.0, Sperm count: Number of spermatozoa per ml: 12 million/ml, Number of sperms per ejaculate: 24 million. Grade: A Rapid Linear Progressive - 20% B Sluggish Linear Progressive - 50% A+B Total Forward Progression - 70% C Non Progressive - 20% D Immotile sperms - 10% Epithelial cells: Nil, Pus Cells: 3-5 Red blood cells: 1-2 Sperm Clumps: Nil Is this report normal, If not can i take this medicine CCQ 25 or Addyzoa. Thank you, With best regards, Ivan. Doctor: Hello, Welcome to HCM, I am Dr. Das Look, the sperm count is lower than normal value. Total motility is absolutely normal. Pus cells are present, that means there is presence of infection. This is not normal. The drugs you are taking is absolutely normal. So, continue the drug. Regards."
},
{
"id": 150000,
"tgt": "Have hemangoma. Have bulging enlarged veins in the forehead, Feels weak and looks pail. Recommendations?",
"src": "Patient: I have a student that may have hemangoma. Doctors are still running tests to determine a definite diagnosis. She has bulging enlarged veins in the forehead. She often has episodes that cause seizure like symptoms. She turns pail, has weak mobility, and then cries. After these episodes that last about 30 seconds, she sleeps. Doctors are concerned with a hit to the veins being fatal. What recommendations would you have for a school environment to keep this child safe. Doctor: Hi,Her symptoms are consistent with seizure phenomenon ( fits) ,she has to be put on antiepileptic drugs.This will prevent falls and hit to the veins (hemangioma)Avoid stress at school, and requires timely tiffins at school.Hope this will help you"
},
{
"id": 194298,
"tgt": "Why is it difficult to keep an erection?",
"src": "Patient: My husband is finding it difficult to keep an erection. He has the urge to pass urine, but it comes out in dribs and drabs. Not painful but annoying and affects his erection. He has been to see the doctor and tested negative to urine infection. What can this be? Doctor: Hi, I can understand your concern for your symptoms, difficult to maintain erection called Erectile Dysfunction. The common causes are 1. Decrease sensation of the penis- can be due to infection, diabetes, spinal injury, recent masturbation 2. Stress due to work or family related, night duties 3. Depression 4. Varicocele 5. Fear of failure - having one episode of ED once may make you feel you will end up with ED 6. Low self-esteem (Chronic masturbation) 7. Homosexuality 8.\u00a0Religious constraints 9. Long term illness \u2013 Cardiac disease, thyroid disease, kidney disease, any other disease with long term medication 10. Negative attitude towards Your partner 11. Hormonal issue \u2013 thyroid issue, low testosterone 12. Long term drug usage 13. Diabetes and Hypertension Ruling out the above reason to find his cause and appropriate treatment. Hope I have answered your query. Let me know if I can assist you further. Regards, Dr. S.R.Raveendran, Sexologist"
},
{
"id": 39537,
"tgt": "Should i be worried about the ear infection during pregnancy?",
"src": "Patient: I just had my ears pierced a few weeks ago and they are infected. I also just found out I was pregnant. Last Tim I found out I was pregnant, I had my belly button pierced and that got infected also, 3 months later I had a miscarriage. So should I take it out now or leave it in? I just can't have this happen twice. Doctor: Hello,Welcome to HCMAs you have ear infection and also as you are pregnant i would suggest you to take out the earrings now and take a course of antibiotics.Pencillins and cephalosporins are safe in pregnancy.So,only these group of antibiotics should be taken during pregnancy.It wont have any bad effects on pregnancy.Hope you are happy with the answer.Thank you"
},
{
"id": 114041,
"tgt": "I have migrating pain in lower back and hip joint. Taking evion still pain remains",
"src": "Patient: I am 42 year male in India suffring since jun 24 , 2011 for migrating pain in lower back + heap joint area xray was normal. I have done 10 sessions of physiotherapy also the pain was reduced by 70%. Now again due to some physical work pain started & When I went to private medical MD doctor he prescribed the medicinecs for 10 days. these medicines are 1. Evion-C 2. PAN 40 3. Myoril plus 4mg . Please help me & suggest me on such type of pain and also on what is use of each medicines prescribed by doctor Doctor: Hello and welcome to HCM. Pain originating in the lumbar region and travelling to hips and lateron to thieghs is called Sciatica and is generally due to nerve compression at the disc level. It is often temporary and when the pain is severe, rest on a hard mattress and anti inflammatory pain killers eases it. Hat bath and general back massages helps to sooth the pain. Avoid lifting heavy objects and bending down to lift things.If severe, an orthopaedic opinion and an MRI scan is necessary. Thanks"
},
{
"id": 31508,
"tgt": "Suggest treatment for typhoid fever and blood in stool",
"src": "Patient: hi,a few months back i was under treatment for typhoid fever and on the 6th day i had a bloody stool...doctors adviced me to go for a colonoscopy and the byopsy reprt said that it's non specific colitis...i am 25 years old. Now i am fine, my stool is perfectly normal. Was it due to the typhoid fever that i developed bloody dyssentry for 1 and a half days. Doctor: Hi thanks for asking question.You have bloody stool in past.It may be due to amebic or shigella dysentry.You have diagnosed non specific colitis and now you have no problem.So nothing to worry about it.This bloody stool may be because of thyphoid ulcer bleed or dysentry.So just now you just do your stool examination for further confirmation.Avoid excess spicy food for few days.Drink more water and shift your diet for few days with fruits more and green leafy vegetables.So just now no worry.If symptoms apprear again then we will decide about to whether start treatment or not.Thanks. I hope i have solved your query."
},
{
"id": 133828,
"tgt": "Suggest remedy for numbness in right thigh and pain in knees",
"src": "Patient: Hi,I have numbness in my right thigh and there is a serious pain in my both knee when i wake up in morning and after taking some rest. I have high blood pressure also. What is the reason for that. I have consulted a local doctor and tested for D25 hydroxy and he said i have deficiency for vitmin d. Now i am taking medicine for Vit D for 10 days but there is no improvment at all. Please suggest Doctor: hithank you for providing the brief history of you.A thorough neuromuscular assessment and musculoskeletal assessment is advised.As you have a numbness in the later aspect of the thigh, this could be related to the pinched nerve in the spine as to assess this a thorough neuromuscular assessment is advised post which if needed an MRI will be guided.Also, for you pain in knee a thorough musculoskeletal assessment is guided post which if needed than an MRI to detect the soft tissue status needs to be viewed.Based on the diagnosis, with simple medication and physical therapy should help you much better.Usually, if vitamin D medicine is taken, it should act in 7 days of time, if it doesnt than need of sunbath is advised. if still the pain and numbness persist than the MRI comes to existence.I see cases with such numbness and such, but the problem I usually confirm from the pinched nerve in the spine and vitamin D is just a deficiency and may not give major symptoms. I advice my patient physical therapy and they respond well.RegardsJay Indravadan Patel"
},
{
"id": 25464,
"tgt": "What causes pain in chest and ribs?",
"src": "Patient: Hi,i ve been having chest,right and left kidney pain,chest pain,throwing up,and now I have two new symptoms a tingley feeling below my right rib cage and a weird hard pulse thumping feeling in my right chest area.. I ve been to the er 7 times last month and all they say is it s from a uti... I don t believe that.. This is to much pain and symptoms for it to be a uti.. Please help me,i don t wanna due,in scared. Thanks. Doctor: Hi, there are many causes for pain in the chest and ribs, and most of these causes is not related to heart disease. The most common cause of chest pain is the pain arising from the chest wall (muscle, bone, tendon, skin) and they do not usually cause death. However, we should always research the most common causes, but that can bring risk of death, such as a heart attack, pulmonary embolism, dissecting aortic aneurysm ...The heart attack has sudden onset, intense chest pain like a bunning pain, accompanied by nausea, vomiting, profuse sweating... Theese makes everyone go the emergency room immediately. The chest pain pulmonary embolism also has sudden onset, is accompanied by severe breathlessness, faintness and impending death, so the discomfort makes patients immediately seek the emergency room. The dissecting aortic aneurysm is a very serious disease and the chest pain is described as the worst pain a human can feel, looking like a spear into the chest, causing the patients also seek the emergency room.For all that, the description of pain you reported does not appear to have the characteristics of these serious diseases. However, as the pain is being repetitive, it is important to look for a good doctor to rule out these serious diseases and arrange treatment.Hope I have answered your question. If you have any further questions I will be happy to help you."
},
{
"id": 203923,
"tgt": "Small dark spots on testicles. What could be the reason?",
"src": "Patient: Well I was using the restroom today, and noticed that I had a couple (6-8) small dark spots on my testicles. I can only remember having a small mole there before, so this caught me by surprise and raised my suspicion as to what they might be. I haven t been sexually active in the last 6 months or so. So I m not sure what it could be Doctor: Hello,Thanks for the query.These spots are plane warts.It can be a sexually transmitted disease.Usually it takes years for the disease to develop.You dont need to worry.It is not a severe condition and can be easily treated with radio frequency ablation.Please meet a dermatologist for exact diagnosis.Let me know if you have any other doubt.you can ask a direct question to me on this forum, following the below link.http://www.healthcaremagic.com/doctors/dr-rahul-kumar/64818Wishing you a good health.Thank you"
},
{
"id": 6193,
"tgt": "Trying to conceive, Pcod. Taking glyciphage, folic acid and deviry. What is the ovulation period?",
"src": "Patient: hi am shilpa anand an iam ttc i have been daignosed with pcod an been pricribed for it my harmones are glysephage ,follic acid an got indused my periods buy deviry tablets thats i took deviry on may 11 an i got my periodss on 28 may then on the second day i took fertyl tablet for 5 days im confused when wil i ovulate please help Doctor: Dear Shilpa Most women ovulate between day 12 - 14 when taking fertyl. Nothing can be said for certain for you as you have PCOD and this is your 1st cycle with fertyl. To find out consult your doctor and get a follicular monitoring done by transvaginal sonography. Once you know the days when you ovulated, they will remain the same in the subsequent cycles on fertyl. Suppose if you ovulate on day 13, you must have intercourse on day 12, 13 and 14. hope for the best."
},
{
"id": 28284,
"tgt": "What causes severe chest pain, shortness of breath, numbness in left hand?",
"src": "Patient: Hello, my wife is 44 years old and has had Rheumatoid Arthritis and Fibromyalgia for over 9 years. Recently she has been having sever pain in her chest and shortness of breath, her doctors have tested her heart and have ruled that out, she has an appointment with a pulmonary specialist tomorrow. However, today she is very weak in her lower extremeties, to the point of not being able to walk, and has been having numbness of her left hand. Doctor: Hello! Thank you for asking on HCM!I understand your concern, and would like to explain that, facing her doctor has ruled out cardiac issues, further investigations should be performed on alternative directions. First to rule out any Rheumatoid arthritis exacerbations. A physical exam with some additional lab tests, like complete blood count, PCR, ESR, RF, X ray films study, etc are necessary for the differential diagnosis. Meanwhile, check her blood pressure and try any anti-inflammatory medications. If the complains persist, you should visit an ER doctor without delay for some preliminary tests.If after all diagnostic work up, nothing new results, then an optimisation of fibromyalgia management should be implemented. Drugs like duloxetine, etc, could be helpful. You need to discuss with a specialist on the field (neurologist) on that issue. Hope to have been helpful to you. Greetings! Dr. Iliri"
},
{
"id": 30558,
"tgt": "Is TB of spinal cord curable?",
"src": "Patient: Hi, may I answer your health queries right now ? Please type your query here...My 35 years old Sister suffered 03 years back by lung TB, for that she took medicine treatment and fully cured but just 02 week back in MRI scanning TB infection found in back spinal cord. Will it be curable as soon as possible?????? Doctor: hi sir, welcome to HCM, i understand ur problem, sir,please just follow the treatment, u will get better results, u have so many positive chances to cure.... thank u"
},
{
"id": 197909,
"tgt": "Suggest treatment for nocturnal emission",
"src": "Patient: Hi sir, I got problem last two months, I daily clean my gland. Even I get white layer on my glands. And itchy sometimes. I am using soap to clean. Sometimes when I sleeping automatically spem will come out fully without knowing me. And I will clean morning. I am fully scaring. Not even before I am clean weekly twice. Nowadays I clean regularly it happened like this. Is this something wrong??? Pls clarify me. My one uncircumcised. Thanks Doctor: Dear, We understand your concernsI went through your details. The whitish layer in your gland is smegma. Smegma is mixture of semen, urine and dust particles. This gets accumulated because of lack of cleanliness. You should clean your gland twice daily.Nocturnal emission is a natural and normal process. Nothing to worry about it. Semen is produced whenever you are sexually excited. Produced semen is then stored within your body. It is a waste product now. Body needs to throw the waste out. If you do not masturbate or engage in sex, body removes the waste through nocturnal emission. I suggest you to masturbate alternate days to stop your nocturnal emission.If you require more of my help in this aspect, please use this URL. http://goo.gl/aYW2pR. Make sure that you include every minute detail possible. Hope this answers your query. Further clarifications are welcome.Good luck. Take care."
},
{
"id": 85189,
"tgt": "Is Galvus a safe medicine?",
"src": "Patient: i m diabetic type 2 aged 26 with sugar around 180-200. previously i was prescribed glyciphage and now shifted to galvus met 50mg half a dose after lunch and the rest after diner. i read galvus met is not recommended in USA, so is ti safe to take it or can you prescribe an alternative? any side effects u came across Doctor: Hi, Yes, it is a safe medicine for the treatment of type 2 diabetes. Galvus (Vildagliptin) belongs to a class of drug known as DDP-4 inhibitors which works by blocking the breakdown of \u2018incretin\u2019 hormones in the body. These hormones are released after a meal and stimulate the pancreas to produce insulin. By increasing levels of incretin hormones in the blood, galvus stimulates the pancreas to produce more insulin when blood glucose levels are high. It is widely used either alone or along with other antidiabetic drugs like metformin in the treatment of type 2 diabetes. The most common side effects of galvus include headache, nasopharyngitis, cough, constipation, dizziness, and increased sweating which gradually wean off over a period of time. The most common side effects of metformin include abdominal cramps, diarrhea, and very rarely a serious condition called 'lactic acidosis'. Every drug has its own side effects but it is not essential that everyone will get that. There are many drugs used for treating diabetes. But there is not a single drug which does not has its known side effects. If you are not comfortable with the side effects of galvus met you have to discuss with your doctor to prescribe an alternate effective but a safer medication for the control of your blood sugar. Hope I have answered your query. Let me know if I can assist you further. Take care Regards, Dr Mohammed Taher Ali, General & Family Physician"
},
{
"id": 19529,
"tgt": "Suggest proper diet chat for high BP",
"src": "Patient: Hello doctor,my mother is facing with high blood pressure problem, we already consulted with doctor and he is advising my mom to start medicines, but actually i want her to control it by taking proper diet because she has not yet started medicines and i heard if she will start, she has to continue it throughout the life. can u please help me out with a proper diet chat. details of my mother are her age- 50,B.P record- 140 by 100 sometime 150 by 90.Thanks Doctor: as far as diet goes ask her to cut on salt ..fried foods ...soft drinks ...ghee etc ..ask her to go for morning or evening walk ...deep breathing and meditation...these are good things but ewually good are allopathic medications ...if a person starts taking he is not hooked ...you have been misled by some ignorant person ..."
},
{
"id": 183269,
"tgt": "Suggest treatment for an infected wisdom tooth",
"src": "Patient: Hi, I got infection around 3 weeks back in skin flap around wisdom tooth and was cured by antibiotic course. Again last week, the skin flap got swollen and started paining. I did gargling with Salt water and applied clove oil. Now , feeling much relieved. Today I have visited Dentist and they suggested to remove the tooth. Later on, in x-ray, they have notice nerve may be touching and explained me, the nerve may get damaged, though chances are very less. But I am feeling scared now. It there any alternate? What you recommend Doctor: Thanks for your query, I have gone through your query.The infected wisdom tooth has to be removed since it is infected, if there was no infection then we could have left it like that only. Get a CBCT done, which shows the exact distance between the root tip and the nerve canal. Later we can plan acordingly and remove it without damaging the nerve canal and nerve. Consult a good oral maxillofacial surgeon and get it done. nothing to worry.I hope my answer will help you, take care."
},
{
"id": 84868,
"tgt": "What are the side effects of Lipikind F tablet?",
"src": "Patient: What are the side effects of LIPIKIND F tablet? I have been consuming the tablet for about 2 months for reducing my cholestrol level. I also have Mild lever enlargement. Does it create any side effect for that problem.? Can i take ayurvedic medicines along with LIPIKIND F? Doctor: Hi,Incomplete data like age, gender, any associated medical issues, alcohol intake, purpose of taking Ayurvedic medicines, etc not given. Lipikind F is commonly used for the treatment of elevated cholesterol and triglycerides. It can cause stomach upset, muscle pain, tenderness, weakness, and liver dysfunction. So, it may cause inflammation of the liver which needs to be monitored carefully.If you develop upper abdominal pain, nausea and jaundice you must report to your doctor for an alternate effective but a safer medicine to reduce your cholesterol level. Since you have not given the exact name of Ayurvedic medicine hence it is difficult to ascertain whether it will be safe or not if taken along with Lipikind-F tablet.Hope I have answered your query. Let me know if I can assist you further. Regards, Dr. Mohammed Taher Ali, General & Family Physician"
},
{
"id": 45548,
"tgt": "What causes pain in the left pelvic region when the USG shows mild splitting in the left kidney?",
"src": "Patient: My ultra sound report shows mild splitting in left side kidney. Otherwise my both kidneys are normal in size. I am suffering from pain in my left pelvis region. I do not have symptoms of burning sensation during urination. My x ray report is also normal. What is causing this pain. Doctor: Hello, It is not related to mild splitting in your kidneys. Possible causes like urinary tract infection must be ruled out. You can go for a urine culture and sensitivity and start antibiotics in case of infection. As of now drink plenty of water to keep yourself hydrated. Hope I have answered your question. Let me know if I can assist you further. Regards, Dr. Shinas Hussain, General & Family Physician"
},
{
"id": 177522,
"tgt": "Suggest treatment for intestinal pinworm in a child",
"src": "Patient: My 6 year old has been diagnosed with pinworm today and has taken the dose of vermox this morning. How long does this take to work please? When will all the worms be dead and out of his system? Been told to repeat the dose (5ml) in 2 weeks time aswellMany thanks Doctor: Hi...usually the drug works in 24-48 hours. Your doctor was right, you need to repeat the dose in 10-14 days time.Hope my answer was helpful for you. Happy to help anytime. Further clarifications and consultations on Health Care Magic are welcome.Dr. Sumanth MBBS., DCH., DNB (Paed)."
},
{
"id": 31155,
"tgt": "Suggest remedy for yeast infection",
"src": "Patient: i am twenty six and for the past year i have been having alot of yeast infections. i have gotten antibiotics and they helped in the past but now nothing is helping anymore. i am tired of visiting the doctors and i cant get an answer i have alot of antifungal creams and pills Doctor: HI, thanks for using healthcare magicWith frequent repeated yeast infections , you may need to consider whether there are any conditions that may be contributing to the problem.This would mean any conditions affecting your immune system that may be predisposing you to repeated infections eg (a)diabetes, (b)use of immune suppressants such as steroids,(c) poor immune system due to conditions such as lymphoma, leukemia- rare cause(d)wearing clothing that keep the vaginal area hot - should use cotton underwear, skirts etc(e)excess weightYou may want to consider seeing your doctor for assessment including physical examination and blood tests (screening for diabetes, complete blood count and ESR)ESR is an inflammatory marker that goes up if there is any reason for inflammation in the body eg infection, trauma, allergies, cancer, autoimmune diseasesI hope this helps"
},
{
"id": 100316,
"tgt": "How safe is usage of cortisone shot for allergies?",
"src": "Patient: Hi, my name is Deborah and I have always suffered with allergies I went to the doctor about three weeks ago and I was given a cortisone shot and some antibiotics. I began to feel fine and now I am sneezing every ten minutes it seems. The only thing that I'm doing is the nasal spay, please help Doctor: Hello Deborah,Thank you for asking at HCM.I went through your history and would like to make suggestions for you as follows:1. Cortisone is a systemic steroid. I would prefer to use it only when syptoms are very severe and other medications are not working. I would not suggest frequent use of cortisone for nose allergies.2. I would like to know which nasal spray you are using - intranasal corticosteroid or other.3. Were I treating you, I would suggest you regular use of intranasal corticosteroid for at least 4 weeks.4. All the medications for nose allergies actually can only \"control\" symptoms but cannot \"cure\" allergy itself.5. Hence I would suggest you allergy testing for common air-borne allergens like house dust mites, common indoor molds, common pollens in your area, insect proteins and pet dander (if you have contact with pets).This will help you identify the substances causing allergies to you as well as to know the measures to avoid them.6. Based on allergy testing, an Allergist-Immunologist may prescribe you allergen specific immunotherapy which works on immune system to gradually improve allergy symptoms over a long period.7. In general, I would suggest you to avoid exposure to dusts, smokes and air pollution as much as possible.8. I would also suggest you healthy diet rich in vitamins & minerals as well as vitamin C for at least 4 weeks. This will improve your immunity.Hope above suggestions will be helpful to you.Should you have any further query, please feel free to ask at HCM.Wish you the best of the health ahead and a very good recovery.Thank you & Regards."
},
{
"id": 190237,
"tgt": "Child having Canker sores in mouth, not able to brush teeth, had cavities filled. Advice?",
"src": "Patient: My 4 year old daughter developed 2 cancor sores in her mouth. I gave her OTC meds to try to lesson the pain. Knowing it takes a few days to heal, she still complains it hurts and will not let us brush her teeth on the side the sores are AT ALL! Though we understand these sores are very painful, we are concerned that her teeth are not clean in that area and that it could cause other more severe issues. This has gone on for about a week now. Called her pediatric dentist and they suggested a cancor rinse but we can t get her to use it. They said that s about all we can do. We are ready to call her pediatrician to have her mouth looked at. She just had 4 cavities filled so she is proned to them. Doctor: Hello, Thanks for your query. I read your problem,first of all i would like to inform you that apthous ulcer take a time period of 10-12 days to heal completely,so nothing to get worried and continue a proper treatment with patience. The treatment options for apthous ulcer are- 1. Vitamin B complex. 2.Topical application of gels like-mucopain,dologel,quqdragel,hexagel etc..... 3.Maintain oral hygiene.etc........ As you said that she if having 4 filled teeth at this age,it is not a good sign,you must be more careful. Please follow following- 1. Maintain oral hygiene. 2.brush twice daily. 3.Regular dental visits after every 6 months. 4.Topical fluoride application 5.Pit & fissure sealants,if needed. 6.Use of fluoride containing tooth paste.etc........ nothing to get worried. Take Care. Dr.Gunjan Gupta"
},
{
"id": 51080,
"tgt": "Done physical test. How much is GFR likely to change with more hydration?",
"src": "Patient: I recently had a physical and all of my labs came back at acceptable levels accept my creatine 1.1 and GFR 55. My doctor wants to retest my kidney function and told me to hydrate well before the test. How much is my GFR likely to change with more hydration? Also, I have severe urge incontinence . When I drink too much water, I have to urinate in large quantities every 20-30 mins. Won t this effect my hydration levels? Doctor: Hi Welcome to HCM. Hydration is a prerequisite for proper kidney function and it is hard to quantify the change. Incontinence is a bladder issue and has go nothing to do with the kidney. Th basic fact is that kidneys need water to keep functioning, irrespective of how often you pass urine. Best regards DR GS"
},
{
"id": 134015,
"tgt": "What is the treatment for lymphatic blockage in left leg?",
"src": "Patient: i have had lymphatic blockage in my left leg since june 2000 due to an accident.i was reading online and came across the use of cabbage poultice&elastic bandages which av bin using for over a week now.its actually reducing the swelling on a gradual note but my skin is peeling&itchy,i don t knw if its right or what other natural methods an i try?thks Doctor: hi,thank you for providing the brief history of you.As you mentioned you had a lymphatic blockage since 2000 due to an accident, the extent of the injury to the blood vessels needs to be estimated by using the ultrasound scanning and also the doppler test. Once we know this, there will be a treatment plan designed specifically for it. Also, for the swelling, performing regular lower limb muscle strengthening is important. As medicine and natural methods are more of symptomatic relief. The focus has to be made to make the muscles stronger as to avoid any space for the fluid to accumulate. Also, exercises will help improve the metabolism and keep the lower limbs at the optimum.I have seen many chronic lymphedema cases, and the exercises have helped them alot for reducing the swelling and also the over all functioning of the lower limbs.RegardsJay Indravadan Patel"
},
{
"id": 37364,
"tgt": "How to treat raspy voice and severe phlegm following a sinus infection?",
"src": "Patient: Hello I ve had a cold/sinus infection for a few days ive been taking dayquil but tonight I took musinex dm. Ever since then my voice is so raspy! Its annoying me beyond belief. Im only 17 and they cut my health insurance for some unknown reason. So I cant really go to the dr. But I just wanted to know if there were anyways to help treat my voice? Its nerve racking. Im coughing up a lot of mucus too. But its all clear. I just wanted to see what you may recommend? Thank you for your time! Doctor: HiWelcome to Health care magic.I went through the history you have provided here. It seems from the history that you are having raspy voice following a sinus infection. This indicates acute laryngitis secondary to sinus infection. This can be treated by antibiotics and anti-inflammatory drugs. Salt water gargling and voice rest will provide symptomtic relief and the condition will improve in 4-7 days and voice will become normal.Hope this is helpful to you.Wishing you a good health and speedy recovery.Thank you for writing to us."
},
{
"id": 188626,
"tgt": "Dry, sand paper like feeling inside the cheeks. Spreading with no pain, swelling, white patches. What could it be?",
"src": "Patient: Hi doctor, Could you, please, tell me what is the reason that inside my cheeks, especially the left side feeling like dry sand paper? I don t smoke , no STDs , don t drink. It seems like it is spreading slowly, but there is no pain, no swolling, no blisters or white patches . Just seems like the wetness of the tissue is gone. Please, help understand what can it be. thank you for possible help... Doctor: Hello,Thanks for choosing hcm forum.Dryness of cheeks may be due to-xerostomia,diabetes,salivary gland disorders,oral lesions like lichen planus,oral submucous fibrosis.I would suggest you to get a thorough clinical evaluation done.Salivary substitutes has to be administered.Take plenty of fluids as well as nutritious diet including fruits and leafy vegetables to keep yourself nourished.Maintain oral hygiene well.Avoid anxiety and stress.Take vitamin becosules as well as iron supplements.Hope this helps."
},
{
"id": 113404,
"tgt": "Sore back after a fall. Chances of concussion?",
"src": "Patient: I fell about 7 weeks ago -- off a ladder about 2-1/2 feet , landed on my rear end, whiplashed back and hit my head on the concrete. I had no symptoms of a concussion at the time, but I had a dent there for about a week and it was sore to the touch. With all this business about Hillary Clinton my husband has been bugging me about it -- is it something I should be concerned about? Doctor: Hi... as of now if you have symptoms like headache, vomiting or seizures then you should be bothered. Since it's already 7 weeks and if you have no symptoms no need to press the panic button. Take care."
},
{
"id": 96000,
"tgt": "Why is my mom finding difficult to release her bowel after one week of undergoing TAHBSO ?",
"src": "Patient: My mom is undergo in TAHBSO after a week she experience in difficulty in releasing her bowel what should I do My mom is undergo in TAHBSO after a week she experience in difficulty in releasing her bowel what should I do Doctor: hi, . this is the commonest complaint after any abdominal surgery.usualy such pts are send with stool softners if your mom in not on any stool softners then give her duphalac syr 30 ml at bed time for 10 days and food with high fibre content eg vegetables,fruits esp dry fruits,wholegrain breaskfast cereals,nuts,beans ,peas etc hope that it will help your mom regards"
},
{
"id": 193668,
"tgt": "Suggest treatment for pre mature ejaculation",
"src": "Patient: hi sir,i'm male 27.i cant hold my ejaculation at my exiting state. very few or no stroke need to ejaculate.but i can watch a movie for a day or half a day with full erection. but i feel very sexy or something then the sudden fall semen. how can i control it,and to increase my stroke to exit a women and how long. thanks in advance Doctor: Hello, The erection induced by watching porn and masturbation or intercourse are different things. If you ejaculate early like without any stroke during intercourse than premature ejaculation could be possible. I suggest you apply topical Lignocaine jelly over penis after erection to prolong ejaculation. You can squeeze your tip of penis with thumb and index finger when you feel imminent ejaculation. You need to learn when to slow down sexual stimulate while performing intercourse. You can take help of psychiatrist or sexologist for counselling. Hope I have answered your query. Let me know if I can assist you further. Take care Regards, Dr Parth Goswami, General & Family Physician"
},
{
"id": 150520,
"tgt": "IIH. Pins on hands and feet, fatigue, headache, pain in knees. On Diamox. Reason?",
"src": "Patient: Hi. I was diagnosed with IIH at the end of 2011. I have been taking Diamox ever since. I am now taking 6, 250mg tablets a day and have been on this strength for 4 months. I have regular pins and needles in my hands and feet as well as fatigue, I also get headaches. Lately I have had terrible pain in my knees and I was wondering if it might be connected to the diamox or IIH? Doctor: Hi IIH is idiopathic intracranial hypertension.You are having regular pins and needles in hands and feet as well as fatigue, and you get headaches. Fatigue can be sideeffect of Diamox. You need to be rexamined in detail by your neurologist for papilledema ,CSF o,pening pressure and ABG to rule out metabolic acidosis. Based on report further decision to be taken and need of thecoperitoneal shunt to be planned"
},
{
"id": 131310,
"tgt": "Suggest remedies for twitching in chest muscle",
"src": "Patient: I am having chest muscle twitches on a reg. basis on the left side of my chest. Had a serious shoulder operation in Feb to repair a rotary cup. No rotary cup present so operation wasn t quite what it should be. I am 75 and my guess is that I have pull a muscle from trying to rehab my left shoulder which is almost impossible. My question is how can I control it somewhat so I can take a nap in the afternoon or go to sleep at night. Doctor: you can start with upper part breathing exercises,that treated your upper part of lungs.these exercises are to be done while sitting and lying on back.also do exercises that specifically involve your pectoral muscles strecing and strengthening like holding hands backwards,shoulder shrugging,"
},
{
"id": 17583,
"tgt": "Can Acitrom timings be changed without any side effects?",
"src": "Patient: acitrom is being taken by me aftr th surgery of aortic valve so now i am experiencing high bp is it due 2 the dosage tht is sunday and thurs 3mg and rest of days 2mg i was operated on 12th jan 2010...as my ramazan fasts r comming near i usually tak th medicin @6 in the evening so can i shift the tim till 7 .30 Doctor: Hi, Yes, you can take Acitrom as per your convenience and there is no specific time it should be taken. But remember not to miss it as it is very important for you. Also, get PT/INR checked at least once per week. Acitrom has no relation to BP and if you are having high BP, you will probably need medicine for high BP too. I will recommend monitoring your bp daily and decrease the salt intake in the diet. Hope I have answered your query. Let me know if I can assist you further. Regards, Dr. Sameer Maheshwari, Cardiologist"
},
{
"id": 174366,
"tgt": "What causes lightheadedness and tremors?",
"src": "Patient: Hello, My daughter is a 16 year non diabetic. Earlier today, she was light headed and started shaking. Her blood sugar was at 91 and she had eaten approximately 1.5 hours before. My husband is a diabetic and thinks her blood sugar should have been higher after a meal. Please advise if she should be examined by a physician. Doctor: Hi,Thank you for asking question on health care magic.Normal blood sugar is 90 to 110 mg/dl.She is not suffering from diabetes.Hence this light headedness may be due to some other cause.Better consult Physician.Hope this answer will serve your purposePlease feel free to ask any more queries if requiredTake careDr.M.V.Subrahmanyam MD;DCHAssociate professor of pediatrics"
},
{
"id": 144752,
"tgt": "Suggest treatment for motor neuron disease",
"src": "Patient: I was diagnosed with motor neuron disease. I cannot walk and my arms and abdominal muscles are weak. I started with ozone treatment and I want to know if it will be worth buying an ozone machine to continue treatment at home? I don t know yet if the treatment works. What is your opinion about ozone treatment? Doctor: I read your question carefully.I wish I could say that there is a proven treatment for motor neuron disease. Unfortunately there is no such treatment with proven benefit until now, at least not by trials. The only approved medication is riluzole, but it only slows progression of the condition, it does not reverse it.So as a professional I can not recommend ozone treatment for now.I wish I could have been of more help. I hope things work out for the best."
},
{
"id": 168218,
"tgt": "Suggest remedy for stretch marks in the groin area post child birth",
"src": "Patient: I had my youngest son January of 2013. Every since the pregnancy I ve had stretch marks all over my groin, obviously after delivery those stretch marks became loose and wrinkly looking, but have always remained there since. However two nights ago i got out of the shower and bam! One of the wrinkles had become swollen but same color as my skin, same texture as my skin. Over night out goes back down but every day it swells back up. I have been faithful, it is not an std, i don t want my boyfriend freaking out, but what s the deal? Ooh and i have been battling a cold for the past two weeks. Doctor: Help thank you for writing us here. Stretch marks are common in pregnancy and also in other disorders of thyroid. It is possible that it's healing slowly. Same colour but little swelling is suggestive of healing. Also weight changes causes such symptoms. Don't worry alright? It would slowly regress as you loose some weight. Do some cardio exercises and you'll also be in good shape and no more stretch marks.Best regards,Dr Gunjan"
},
{
"id": 173424,
"tgt": "What causes diarrhea after swallowing a penny coin?",
"src": "Patient: Hi. My son (7) swallowed a penny on Wednesday night. I brought him to the er to get xray and they said the coin was progressing, watch for the coin, have another xray in 4-5 days. Since then he has had diarrhea so I cannot see anything in the bowl. He has an appointment with his doctor for next Tuesday. He doesn t have any other symptoms. My question is, is the diarrhea something to worry about or are we safe to wait another 4 days for his appointment? Doctor: HI...I don't think this is related to the coin ingestion. It seems your kid is having viral diarrhoea. Once it starts it will take 5-7 days to completely get better. Unless the kid's having low urine output or very dull or excessively sleepy or blood in motion or green bilious vomiting...you need not worry. Regards - Dr. Sumanth"
},
{
"id": 75524,
"tgt": "Can bronchitis cause pain in chest?",
"src": "Patient: well i dont konw if it has to do with bronchtis but tonight while sleeping i woke up to an intense pain right underneath my left breast. it is like an intense pain right underneath. it hurts more when i take deep breaths. i was wondereing if it could be from being sick cuz i do have the flu so whats going on?? im so scared!! Doctor: Thanks for your question on Healthcare Magic. I can understand your concern. Yes, bronchitis can cause chest pain. Bronchitis can cause pleurisy (inflammation of pleura) and this can cause chest pain on breathing. But in my opinion, we should rule out heart diseases because you are having mid night left sided severe chest pain which is more common in heart diseases. So first get done ecg and 2d echo. If both these are normal then no need to worry for heart diseases. Take simple painkiller and anti inflammatory drugs for pleurisy related chest pain. Apply warm water pad on affected areas. This will also gives symptomatic relief.Don't worry, you will be alright but first rule out heart diseases. Hope I have solved your query. I will be happy to help you further. Wish you good health. Thanks."
},
{
"id": 128861,
"tgt": "Suggest treatment for severe pain in the left hand, foot and mid back",
"src": "Patient: Hi, I have SLE Lupus. It has bothered me since age 15. I got diagnosed 2009 and I am 63 now. Recently I have had severe pain in the left hand 1st finger knuckle all the time. Pain left foot between 1st and 2nd toe in which I had a shot last month. Finally, mid back to left hip area that sometimes stops me from walking. I am on plaquenil and muscle relaxer and tramadol. Doctor: i know you are suffering from pain don't worry I will give u some medicine that can cure it1.you can take paracetamol up to 4g/day2.or tab ibuprofen400mg/600mg1-1-1daily or3.tab diclofenac 50mg TDs4.Aceclofenac 200mg be5.Etoricoxib 120mgod6.flugiritine100mgbd. or7.Zeltoprofen 80 mg bid8.tab aceclofenac +paracetamol +thiocholcosidetopical NSAIDs gel may be useful"
},
{
"id": 85828,
"tgt": "What causes severe headaches after taaking Fol123 pills?",
"src": "Patient: i am taking fol123 pills for the past 4 days. Yesterday night onwards I am having severe headache and today vomitting (only once). My doctor prescribed this pill since I had irregular periods and bi-lateral cyst. Advice me the above side effects will be there on consuming this pills Doctor: Hello, Fol123 pills is not known to cause any headache but there may be individual variation. If you don't tolerate the drug, you need to discontinue it and take some other preparation of omega-3 fatty acid, Folic acid & Vitamin B12, if these are really indicated. Hope I have answered your query. Let me know if I can assist you further. Take care Regards, Dr Tushar Kanti Biswas, Internal Medicine Specialist"
},
{
"id": 225283,
"tgt": "Is there any possibility of pregnancy as i missed taking the pill for one night?",
"src": "Patient: I ve been on birth control for over a year and a half. I m good about taking it everyday around the same time. I had sex Thursday, took the pill that night, but then I forgot the next night. I realized and took it the next day at around 12pm. I m worried I m pregnant... Doctor: Hi,Dear.If you took your pill the next day then there is nothing to worry about.But you should take care of taking the pill everyday.Do not worry about pregnancy because long term pill users have a constant amount of hormone in their blood ,so missing a single pill will not affect your contraception.But you are supposed to take the remaining pills regularly.be stressfree and happy. Hope this answers your query. If you have additional questions or follow up queries then please do not hesitate in writing to us. I will be happy to answer your queries. Wishing you good health."
},
{
"id": 36807,
"tgt": "What causes deep sleep after a foot infection?",
"src": "Patient: My father is currently in a hospital and was admitted due to an infection in his foot. They have that under control and clearing up however since Saturday my father has slept 90% of the time and on Sunday a special team was called in to try and wake him up. With out success they took his vitals which were all normal and said he is in a deep sleep and they font know why and I gave tried to find out why with no luck. Can you please tell me what could be causing this to happen and when he foes wake up he says off the wall things he actually thinks he is at home instead of the hospital Doctor: Hello, Welcome to HCMI am Dr Rakesh Sharma answering your query.I can understand your concern.It may be because of following,1.side effects of medications.2.hypoglycemia,3,Infection spreading up to meninges,4.May not be related to present condition, but some other cause of brain involvement.Treating doctors are in better position to explain his condition.You may discuss my points with them.Hope this suggestion will help you.Good Luck."
},
{
"id": 104163,
"tgt": "Indoor, outdoor and food allergic reactions. Biting sensation in body",
"src": "Patient: Hi Doctor, My grandmother has been experiencing indoor/outdoor/food allergic reactions for a while now. At first it was indoor and outdoor, then it spread to her sense of smell, and now she is allergic to all foods and the smells of cooking, paper, etc. She experiences these biting sensations throughout her body(skin) and into her digestive tract( stomach ). She describes it like someone s poking her with a knife. Doctor: THESE ARE FOOD ALLERGIES TYPICALLY IN FOOD ALLERGIES THE STARTING SYMPTOMPS ARE ONLY IN PARTICULARS TIME AND POSITION BUT AFTER SOME TIME OF DISEASE EVERYTHING SEEMS TO REACT THIS MOSTLY HAPPEN IN FOOD MILK OR WHEAT GET SERUM TEST FOR MILK AND WHEAT SPECIFIC ANTIBODIES ELIMINATE FOOD FROM DIET ALL YOUR SYMPTOMPS WILL GO AWAY WITH MINIMUM MEDICINES"
},
{
"id": 151527,
"tgt": "Had stroke. Had hiccups and nausea. Mild swelling in brain. Can neurologist help?",
"src": "Patient: my husband had a stroke 8 days ago-he has been experiencing the hiccups and some nausea , he has had an mri and does have some mild swelling on his brain, could the stroke be causing the hiccups & nausea or possibly some stomache issue? They have checked his gallbladder and said it is fine. They have been giving zofran and reglin for nausea with some releif, now he has instead of actually vomitting been getting short of breath or losing his breath for a few seconds and just says he feels sick-so wondering if you or a neurologist could help??? Doctor: hello some patients do get nauseaand vomiting as well as swallowing difficulties, especially in stroke involving the area of brain called the brainstem. Usually these symptoms take few weeks to subside, but eventually they do... so consult your neurologist and follow the advice. take care"
},
{
"id": 77597,
"tgt": "Suggest treatment for burning in chest radiating to right back and shoulder",
"src": "Patient: For the past few weeks I get intermittent burning in my chest in the mid to right side that also radiates to my right back/ shoulder blade. I've had heartburn rarely before but not this intense. Antacids help but only sort of. No other symptoms. Sometimes it occurs daily and sometimes there are days without it occurring. Doctor: Thanks for your question on Health Care Magic. I can understand your concern. In my opinion, you are mostly having GERD (gastroesophageal reflux disease) and related symptoms. It is due to laxity of gastroesophageal sphincter. Because of this the acid of the stomach tends to come up in the esophagus and cause the symptoms of chest pain. Only antacids are not useful to control this. You need to follow these lifestyle modifications for better symptomatic relief. 1. Avoid stress and tension. 2. Avoid hot and spicy food. 3. Avoid junk food. Avoid large meals, instead take frequent small meals. 4. Go for walk after meals. Quit smoking and alcohol if you have these habits. Don't worry, you will be alright. Hope I have solved your query. I will be happy to help you further. Wish you good health. Thanks."
},
{
"id": 212578,
"tgt": "Stress induced insomnia. Taken vyvanse and benadryl. Side effects?",
"src": "Patient: Hey, I recently took 30mg of vyvanse in a slow release capsule about 5 hours ago. I still feel wide awake and I really want to sleep tonight. I have stress induced insomnia and I am not prescribed any medication for it so I compensate with 125mg of benadryl . While this is not healthy by itself, I plan on taking 75mg of benadryl to counter act the vyvanse so i can sleep tonight. I just want to make sure I am not going to die if I take it or have another horrible side effect. This is a one time thing but I still want to make sure before I just go and take some pills. Doctor: Hi there, thank for asking. Benadryl is not a good medication for sleeping, although it is sold over the counter as sleep aid. Benadryl can change heart rate as well as vyvance can do. The arousal induced by vyvance will maintain and it is useful to take advantage of some relaxation method like meditation as they do not have any side effect. Vyvance is usually taken in the morning in order not to cause insomnia at night. The dose should be increased gradually in order to minimize the side effects and there should be a reason for it taking (it is usually prescribed by doctors for attention deficit disorder, etc). Otherwise, the side effects would be more than benefits. I wish you the best health."
},
{
"id": 82537,
"tgt": "What does PTB minimal undermined activity in a chest X-ray mean?",
"src": "Patient: hi dr. if i have a chest xray finding of ptb minimal undermined activity what does it mean? how many times that i had an xray exam but the result is still the same. what will i do? because it is a major problem that affect when i am going to apply for a job. answer me pls. thank you Doctor: Thanks for your question on HCM. I understand your concern for this.I routinely see many patients with the same query. I follow these protocol.1. CT thorax with contrast. As CT gives more idea about lung parenchyma.2. Bronchoscopy and BAL analysis for AFB.If both of these are normal than you can assure the job authority that you are not suffering from TB.So consult pulmonologist and discuss these things with him."
},
{
"id": 20991,
"tgt": "Does raised cholesterol causes heart block?",
"src": "Patient: Hello. I am a 23 year old male. I haven t worked out in a couple months because I began feeling this pressure in my head when bending over or standing up to fast. I would also get light headed and feel the need to faint when standing up. Anyway, My cholesterol reading were: HDL: 33 TRIG: 45, Total 187. LDL was N/A but I am guessing they are around 140. I am very worried that I have clogged arteries somewhere and this is why I am getting this head pressure. When I took my cholesterol a year ago, it was 209 total, with a 59 HDL and 127 LDL. Triglicerydes were normal. Could this new shift in cholesterol be the reason why I am having these problems? Do I have some clogged arteries? Please help. P.S. And MRI of my head was normal. Doctor: Hellothanks for posting here.High cholesterol for a long. term duration leads to deposition of cholesterol in the arteries of the heart and other organs leading to clogged arteries. However it is a gradual process and going by your age it seems unlikely that you have clogged arteries. These symptoms can be related to high blood pressure, internal ear issues and also in cervical(neck) spine problems. Please check your BP, get your ear checked up for internal ear issues and do a MRI of the cervical spine(if not already done) If all these three are normal, then you need not worry about your symptoms, they may disappear soon. Also your cholesterol levels are not very high for your age to be concerned of. wishing you good health. regards"
},
{
"id": 226356,
"tgt": "Severe shooting pains after having mirena insertion. Is this normal?",
"src": "Patient: Hi Dr Lundi, I am a 22 year old female and recently had the Mirena IUD inserted (3 weeks ago). I have not had a child before and the insertion was very painful. I had consistent cramping for about a week and on and off ever since. Recently though, I have started getting severe shooting pains. Some start in the abdomen and shoot all the way down my cervix and walls. I checked two weeks ago for the strings, which were intact. Should I see a doctor? Thanks so much. Molly Doctor: Hi Thanks for your query, Insertion and removal may be associated with some pain and bleeding. In case of difficult insertion and/or exceptional pain or bleeding during or after insertion, physical examination and ultrasound should be performed immediately to exclude perforation of the uterine corpus or cervix. Severe shooting pain may also be a symptom of a ectopic pregnancy. So i will suggest you to see a doctor soon. Best of luck Hope it helps...."
},
{
"id": 4698,
"tgt": "Diagnosed H pylori positive. History of having unprotected sex. Pregnancy tests negative at 6 weeks. Possible positive test at 12 weeks?",
"src": "Patient: Hi Dr. i had unprotected sex(vaginal) after almost 3weeks i developed all ars symptoms: fatigue, abnominal pain,soar throat, mouth ulcer(only 2 days and disappeared), pimples at lower back then later on bach shoulders, chest, dark knuckles, high BP, dizziness. i was diagonized and found h pylori postive and put under medication- healed. But i was overwhelmed by anxiety and the symptoms continued. at 19 days took p24 ag test-neg. at 24 days elisa test-neg, at 6 weeks 6days took pcr dna and elisa test both came negative. Could there be a possibility of turning positve at 12weeks? Doctor: HIThank for asking to HCMThe best way to confirm the pregnancy is imaging study ( Ultra sonography) this can detects the gestational sac of very early days, for your other problems it seems functional problems acid level could increase in state of stress and this can aggravate the symptoms of reflexes so keep your stress level low, no need to worry it will be okay be safe and take care of yourself bye."
},
{
"id": 191189,
"tgt": "Should I go to the emergency room for this?",
"src": "Patient: http://i36.tinypic.com/rkpngw.jpg Should I go to the emergency room for this? Doctor: Hi, Swollen gums is not an emergency unless it is bleeding profusely. Please visit a Dental Surgeon who will determine the cause of the condition. Swollen gums could be due to Calculus or deposits on teeth, reaction to medication, deficiency of Vitamin C, Bleeding disorders or many other causes. The cause needs to be identified and treated. You may need scaling, use of a gum astringent, a Periodontic Flap Surgery depending upon the condition and cause."
},
{
"id": 116012,
"tgt": "Why is my hemoglobin low inspite of taking iron supplements?",
"src": "Patient: Hi, may I answer your health queries right now ? Please type your query here...hi I have had low hgb results for many years and have been on iron supplements as a result they continue to be low should I be concerned? What may be causing this even with supplementation? Doctor: Hello and welcome to HCM, Low hemoglobin levels despite iron supplements suggests that the low hemoglobin is not due to iron deficiency. Thus, more investigations need to be done to know the cause of anemia or low hemoglobin. The investigations include red cell count, hematocrit, red cell indices, vitamin B 12 and folic acid levels. The probable cause of anemia will be determined by these investigations. Thanks and take care Dr Shailja P Wahal"
},
{
"id": 144155,
"tgt": "What causes persistent seizure attacks for twice in a 63 year old?",
"src": "Patient: Doctor, I am 63 years old and a male someone overweight at 5-7-1/2 and just less the 250lb. I don t have a regular doctor and haven t seen one in quite awhile because I usually feel good. The only doctor I see is my dentist twice a year to be honest with you. Anyway just yesterday I was in my home office and felt something weird going on in my left hand that I could not control before you know it my hand was bent over and it was taking over my arm and pushing towards me finally talking over my body that I could not talk and I believe I passed out. Here I am thinking I might of had a seizure of some sort. This was maybe 11am in the morning being I am retired and was at home. I took a Aspirin just in case after that as I was ok then which seemed like maybe 15 minutes later. I am not sure. Same thing happened when I was out in my barn later yesterday afternoon but I sat back in a chair and was fine like possibly 15 minutes later. During these 2 attacks which I never had before I barely could talk and was having a hard time breathing during it. Since late yesterday afternoon I haven t had any problems and have been taking 1 Aspirin maybe every 4-5 hours. I feel fine today as I do any other day. My wife when she got home from work wanted to take me to emergency but I didn t feel the need to go. Spent a lot of time laying on the couch today resting as not wanting to push anything and I feel find as of this letter. Do you have any idea of what may of gone wrong? Doctor: Hi, I am Dr.Bruno. I have read your question with care and understand your concerns. Let me try to help you Question : Do you have any idea of what may of gone wrong?Answer : Both the episodes you resemble fits. Hence I request you to kindly consult a Neurosurgeon / Neurologist at the earliest Hope you found the answer helpful.If you need any clarification / have doubts / have additional questions / have follow up questions, then please do not hesitate in asking again. I will be happy to answer your questions.Let me know if I can assist you further.Take care."
},
{
"id": 215096,
"tgt": "DARK SPOT ON FACE LIKE A MOLE",
"src": "Patient: HI I M 17 YEARS TEEN GIRL.,I HAVE BIG DARK SPOT SIMILAR TO A MOLE ON MY FACE ...HELP ME....WIT GOOD SOLUTIONS Doctor: hi welcome to healthcare magic forum the dark spots might be due to high melanin synthesis and has many causative factors. That could be melasma, dermatitis so on. The first thing to do when seeking treatment for dark spots on face is to talk to a dermatologist. He can give you a diagnosis for your particular condition. Then you can choose for any treatment. All treatment including homeopathy needs a proper case study then treatment. Thank you"
},
{
"id": 42570,
"tgt": "Does metformin and eltroxin help to treat infertility?",
"src": "Patient: Good Morning Doc,I am suffering from PCOD and thyroid, i am on metformin (gluconorm SR 500 mg) and eltroxin (50mcg) i have been consulting Dr Madhulika Bhattacharya @ Fortis Marthahalli. For the past 9 months. I will soon turn 31 and trying to concieve. Wanted to know your feedback on Dr Madhulika is she good? Should i change my doctor? Please advice. Are the medicines i am on will help me conceive? Doctor: During PCOD there are many hormonal that increase including insulin. Metformin is given to check this insulin under control. If Eltroxin is being given probably you are a hypothyroid. Check your thyroid hormones once in 3 months. Keep you weight under control by doing regular exercise and dont get tensed. May be you can get a follicular check done and time your intercourse for a better results."
},
{
"id": 221987,
"tgt": "What is the best treatment to get pregnant?",
"src": "Patient: I am Divya (25 yrs old). My husband is 28 yrs old. We got married 2 years back. We started trying for a kid a year back but i did not get pregnant. So we took all tests for both me and my husband and everything was normal except I had slightly elevated prolactin (52 ng/ml). I took medicines and everything is normal now (21 ng/ml). I got pregnant 5 months back but we had to abort it as it was not fully formed. Only sac was there. We started trying again 2 months back, but still I am not pregnant. How many more months should we try? I am little worried as it is getting delayed. I want to have a baby soon. Thanks for you answer in advance. Doctor: Hi,Since you did get pregnant 5 months ago within a year of trying to get pregnant, it means that both you and your husband are fine, especially since you have also done the tests as well.It was unfortunate that you lost the pregnancy, but this occurs in 1/6 pregnancies. So, it should not concern you much, as it has happened only once.Since you have been trying again for only two months now, I would advise you to continue trying for another 8-10 months months.85% of couples conceive within a year of trying. Only the remaining 15% need help. And since you have got pregnant before there is a very high chance that you will get pregnant soon.Please do consider folic acid while trying to conceive, this is for normal development of the baby.Hope this helps.Regards."
},
{
"id": 102780,
"tgt": "Could faster heart beat and uncontrollable arm shakes after drinking coffee be a symptom of being allergic to coffee?",
"src": "Patient: My granddaughter has drank coffee but gave her fast heart beat and weird feeling. Today she tried decaf coffee and within 40 mins she had uncontrolled arm shakes.Could she be allergic to coffee or caffeine? She is 18 and in college and wanted to try out coffee.. Doctor: Hi...she could be...if it is causing problems better to avoid it...else the intensity of reaction may escalate..Dr. Ashish Verma"
},
{
"id": 217045,
"tgt": "What causes pain under and around my ribs?",
"src": "Patient: Im having pain under and around my ribs on the right side I dont have a gaulbladder i am having regular bowel movements and passing gas i have vomited the pain is constant with sharp pains frequently that make me unable to stand up all the way i dont know if this warrents a emergency room visit or not Doctor: Hello dear user! and thank you for sharing your concerns with us!I'm sorry for the situation you are in!Pain around and under the ribs may be caused by too many conditions that affect, the abdomen, chest, ribs and muscles etc.It may be caused by, gastritis, peptic ulcers, liver diseases (you say you don't have gallbladder), pancreatitis, cardiac diseases, inflammatory conditions of muscles and ribs, etc.Additional examinations and test are needed to have a better view of this situation.I would recommend you the following tests and examinations:- Chest x-ray- ECG- Abdominal ultrasound- Liver and kidneys function indicators- Complete Blood count, lipids and sugar- Complete urine countAfter you get the results we will be able to have more information of your current health status, and we will find if something is going wrong in your body. After that your primary doctor will give you the proper cure or instructions to pass this condition.Feel free to contact us again for further information and details.I hope you will be fine soon.Take care!DR ERIOL."
},
{
"id": 72336,
"tgt": "Suggest remedy for granulomatous lymphadenopathy with tuberculosis sarcoidosis",
"src": "Patient: Excision biopsy of cervical lymphadenopathy on the right side in 2004 showed confluent epithelioid granulomas. Areas of hyalinization and necrosis also seen. Multinucleite giant cells of Langhans type noted. Z-N staining did not reveal acid fast bacilli despite intensive search. No atypical cells observed. Diagnosis: Granulomatous lymphadenopathy ?tuberculosis ?? sarcoidosis. Bactec culture showed no growth of Mycobacterium species after 6 weeks of incubation. In Feb 2010 granulomatous lymphadenitis on the left side of the neck with FNAC showing epithelioid cells present as single cells or clusters. Occasional multinucleated giant cell is seen. No caseation identified. What could be the diagnosis for this cervical lymphadenopathy? Doctor: Thanks for your question on Healthcare Magic.I can understand your concern. In my opinion, this patient should start anti tubercular drugs according to weight.Tubercular Lymphadenopathy is much more common than sarcoidosis. Sarcoidosis is more common in mediastinal lymphnode. TB is more common in cervical lymphnode. So even if culture is negative, absence of caseation necrosis, we should definitely go for anti tubercular drugs for atleast 6-8 months. Hope I have solved your query. I will be happy to help you further. Wishing good health to your patient. Thanks."
},
{
"id": 190247,
"tgt": "Dark spot on lip. Should I see an oral surgeon?",
"src": "Patient: Hi there. I brought my daughter to see her Dr about a dark spot on the lip that she s had for 7 months. It hasn t got any bigger but it s not going away either. He told me it wasn t anything to worry about but when I brought her to her dentist app today her hygenist almost seemed a little worried and said that they may refer her to an oral surgeon the next time she comes in if it s still there. Should I make an app for her now to see an oral surgeon? What could it be? Doctor: Hello In your question you haven't mentioned the age of your daughter as these spots disappear by them self in growing age( 10-16).Normally they are of not major concern.These spots are basically melanin pigments deposited in skin of lip.Wait for next 6-8 months and notice any change, if there and then see a skin specialist rather than an oral surgeon. Best Dr. Amit k. Gaba"
},
{
"id": 209250,
"tgt": "Advise on overcoming depression from strict parenting",
"src": "Patient: I am having some problems at home.my parents are very strict.I am 17 years old and i'm not even aloud most of the times.I'm only aloud out when i'm at college. I feel really depressed being at home all the time.I cant really enjoy myself at all. My parents and i argue nearly everyday because they won't allow me to go out They don't trust me at all. They treat me like a little child. I don't drink or smoke, i am a really good girl. I don't know why i get treated this way. Please would you help me. Doctor: HIThanks for using healthcare magicI think, you have adjustment issue and in that case, you need help of a psychologist. She would help you to understand your issue to them. The moment, they would realize that, their behavior towards you is not right, you would feel better. Better to consult a psychologist who has good experience in this field. In case, you need further help, you can ask.Thanks"
},
{
"id": 28848,
"tgt": "How can hepatitis B be treated?",
"src": "Patient: Hi Doc i want to ask something my husband did medical examination applying for abroad then the result was he was hepa b reactive and calcific scar due ti that i told him to go to another laboratory to test again, can you tell me how to cure this if he has really had this disease im so sad if this is real. Please reply thank you so much. Doctor: Hello,Unfortunately, there is no cure for hep b.you should have to run the test and see the infectivity and immunity status against it.Hope I have answered your query. Let me know if I can assist you further.Regards,Dr. Bhadresh Lakhani"
},
{
"id": 109354,
"tgt": "Suggest treatment for lower back pain",
"src": "Patient: Hello I am looking for advice, I have been suffering with lower back pain since Monday I went to my gp yesterday as the pain was ecomming severe and I was struggling to walk . She give me tramadol and paracetamol to take for a week to try and control the pain. She could no examine me due to the level of pain I was in but suspected it was muscular. Today the pain is slightly better as I have been taking the medication bu I have noticed that I am struggling to pass urine I literally have to force it out and don t get the feeling that I need to go. Is this common with back pain? Thanks in advance for our help Doctor: Dear patient don't take back pain so casually. You have got back pain with difficult y in passing urine. This is most likely due to disc herniation with nerve roots compression which is serious. Are u suffering from weakness in Lower legs or decreased sensation on your feet? If any of this symptoms are present you immediately need to consult orthopedic surgeon or neurophysician. You gp gave you pain killers which relieved your pain but difficult urination indicates serious condition. I would advice MRI of lumbosacral spine that would confirm your diagnosis and would guide further treatment. Meantime continental taking tramadol plus paracetamol and add methylcobalamin plus pregabalin 75mg at bedtime. Do take proper bed rest. All the best."
},
{
"id": 187743,
"tgt": "What is the cause for a swollen gum even after flossing?",
"src": "Patient: Hi, I been having gum pain for about a week now. I tried everything from flossing, warm water with salt, garlic, and mouthwash.. and i still have pain. Today i woke up and noticed my gum is really swollen over my tooth. My tooth have a small chip in it also i see little white dots on my gum near the tooth.. what should i do? Thanks! Doctor: Hello,This gum pain does not look that it is because of infection from gums, as you have tried a lot of remedies.If tooth is chipped off and you are experiencing pain in the same region , it can be because of infection in the tooth , which has caused an abscess.You should visit a dentist and get an x-ray done, as it will rule out the source of infection and the problem will be treated.Thank youHope it will help you"
},
{
"id": 194168,
"tgt": "What causes itchiness in penis with blisters?",
"src": "Patient: im a 47 year old male and my ball sack is very itchy and has been for a couple weeks. when i sit in a bath they sting and itch with contact of water then gets better. play a lot of sports, and now the ball sack area (skin) is very hard and now penis is itchy from contact with area. today i noticed they itch even more and now a clear liquid type, (like popping a blister wetness) has been noticed. i put hydrocortisone cream on this morning and the washed are and did same this afternoon. anything i should be doing differant? Doctor: Hi, It can be a fungal infection. Sharing a picture will be helpful in proper diagnosing. Application of antifungal will be helpful. Hope I have answered your query. Let me know if I can assist you further. Regards, Dr. S.R.Raveendran, Sexologist"
},
{
"id": 42707,
"tgt": "Is there medication to induce ovulation?",
"src": "Patient: Hello Doctor if anyone of you can help. I had IUI on 23rd December 2011 and then after I get in periods on 5th January 2012 means my IUI not successful. I have again next period on 4th February 2012 and today its my 15th day of period. I have done LH surge test today through first urine in morning but my ovulation test not came positive yet. can you pls suggest what I can do here. Can i take any medicine from 7th or 8th day of period so I can have good ovulation by 14th or 16th day. Please advise. Doctor: Hi,Thanks for writing to HCM .Ovulation inducing drugs like clomephine can be taken to achieve good eggs under guidance of your doctor. This drug should be started any day from 2nd to 4th and5 days from that. If this fails then fsh injections are given . But majority get good eggs by clomephine. Also you should monitor your follicular growth . Clomephine helps multiple follicles to mature and rupture . Thus increasing chances of pregnancy .Hope your tubal patency test is normal which you must have got done before IUI. Hope I have been helpful .Regards Dr.Deepika Patil"
},
{
"id": 35509,
"tgt": "What is the treatment for the serratia bacteria?",
"src": "Patient: I am recovering from a brown recluse spider bite...I have deep wounds and have just been put on Cipro. A culture of the wound showed serratia bacteria. I wonder if I can do anything else to heal it faster...I am also packing the wound with Iodoform ribbon. Doctor: Hi XXXX Thanks for your query at HCM! I am Dr. Sheetal Verma an Infectious Disease Specialist answering to your query! I went through your query and understand your concern. The antibiotic ciprofloxacin is effective for treatment for serratia. Take proper dose under your doctors guidance. Other drugs effective is ceftriaxone and amikacin. As you have deep wounds do dressing with antiseptics like betadine regularly and keep the wound clean. Hope I have answered your question. If you would like some more information I will be happy to provide. It would be kind if you hit a thanks if found helpful. You can also write a review for me. Wish you good health Take care! Wish you best of Health. Dr. Sheetal Verma Infectious Disease Specialist"
},
{
"id": 39892,
"tgt": "What treatment is suggested for dog bite on lips",
"src": "Patient: I got bite my my dog in the lip (not the dogs fault) I needed stitches they used ones that dissolve and as it is healing a have a scab forming on my lip. It is very tender under the scab. Is this normal and if it comes off before the stitches dissolve is that a problem? Doctor: Dog bites on the lips is very serious (due to its proximity to the brain) and post-exposure prophylaxis should be started immediately. You should approach a physician and get the Rabies shot without delay.As far as first-aid measures are concerned: wash your lips with lots of soap water. Do not -mind you, do not - suture (stitch) the wound as this will only allow the virus to reach the peripheral nerves faster.Of course, if the dog is not infected with rabies there is nothing to worry, but, then, it's always better to be safe than sorry."
},
{
"id": 12642,
"tgt": "Is there any permanent cure for psoriasis ?",
"src": "Patient: I am suffering from psoriasis . Is there any permanent cure for psoriasis.Please let me know the doctor name & phone number Doctor: Hi, welcome to health care magic forum, yes why not psoriasis can be completely cured with Homeopathic treatment without any side effects, unlike other treatments where you will be on steroid and topical ointments which have lot of side effects on your body, and will only suppress your skin condition. i am not criticizing any pathy but Homeopathy has very good and best treatment for skin problems. If you really want to see psoriasis cases cured by homeopathy visit us at https://urldefense.com/v3/__http://www.drkackerhomeopathy.com__;!!Mih3wA!SBzm6_kI6hCZ58EPH6N_05MFfiPbxWXT0a2TJCdFQObRWm5mV5ur7hW2n-Z0MQ$ For More Information Mail Us At :- info@drkackerhomeopathy.com or visit us at https://urldefense.com/v3/__http://www.drkackerhomeopathic.com__;!!Mih3wA!SBzm6_kI6hCZ58EPH6N_05MFfiPbxWXT0a2TJCdFQObRWm5mV5ur7hUXvG24rA$"
},
{
"id": 123260,
"tgt": "Suggest treatment to cure bump on arm caused by fall",
"src": "Patient: yesterday i fell on a hard concrete floor while acting out a scene (i was supposed to fall) and accidently fell on my arm and now it has this bump and it hurts when you touch it. its still swelling from last night. theres some blood on there visible like its stuck there or something Doctor: Hello, As this looks to be muscular injury since no mention is about the elbow swelling or disturbed functions of the fingers. So I will advise doing hot water fermentation at the pain site, simple gentle kneading massage, elbow flexion and extension exercises. Hope I have answered your query. Let me know if I can assist you further. Regards, Jay Indravadan Patel, Physical Therapist or Physiotherapist"
},
{
"id": 33983,
"tgt": "Suggest treatment for dog bite in pregnancy",
"src": "Patient: my brother in law wife was biten by their pet dog today morning. only the dogs teeth impression was found. no symtoms of blood wound. she is four month pregnant. when consulted to the gynacologist she was not able to prescribe treatment for this. could u please advice Doctor: HiThanks for posting your query on HCM.I appreciate your concern for the treatment of dog bite.Pet dog's vaccination status not mentioned.Rabies poses a 100% risk for death to pregnant women and a high risk to the fetus. Because of the high mortality associated with untreated rabies virus infection, pregnancy is not considered a contraindication to post exposure (dog bite) prevention of rabies.Moreover, rabies vaccine does not have any adverse effect on fetus, mother-to-be and the course of pregnancy. Hence complete post-exposure treatment should be given depending on the category of the exposure.In view of the evidence based explanation she must receive the anti rabies vaccine as per the following scheduled days 0, 3, 7, 14 and 30 days for the category II bite she had.If not given earlier during pregnancy she must receive a booster dose of Inj Tetanus toxoid.If the pet dog appears to normal on the 10th day of the treatment, then the treatment should be stopped.Hope this advice could be of great help to her in getting the prevention of rabies.A feed back is appreciated."
},
{
"id": 213834,
"tgt": "I suffer from depression I am in therapy and on antidepressants At times",
"src": "Patient: Sometimes I wish I was dead I suffer from depression I am in therapy and on antidepressants At times I find myself cutting myself. I have been cutting off and on since I was 15 and I m 40 now,and still I feel life is not worth living but I cant seem to end my life as much as I want to, so I continue to take it out on myself by cutting so that I can escape my feelings or else to feel something hoping that someday I will be able to do it right Doctor: Hello.welcome.You are having this depression for past 25 years which is a very long time.You must consult a psychiatrist and go for counseling sessions with him along with antidepressant treatment.Along with this treatment you must also think of improving your physical health by doing regular exercises,yoga,meditation and good nutritious diet.Good luck."
},
{
"id": 207871,
"tgt": "Suggest treatment for depression and severe anxiety",
"src": "Patient: I have depression and severe anxiety...I ve been on Wellbutrin 300xr for over a year...w/1mg xr xanax (1 per day) and 1mg reg xanax (1 per day) Stress in our life has gone up tremendously and my anxiety is out of control. I am afraid to ask my dr for a higher dosage or more doses in a day. How do I ask with out her thinking I m after more drugs ...I m looking for relief of anxiety and xanax is what helps me even as tabu as others find it. How do I ask her and how much more do you think I may need? Doctor: HiWelbutrin is bupropion and its NDRI.Usually we do not prefer it for 1st line treatment for depression and anxiety.Xanax is good to handle anxiety.This kind of condition can be treated with SSRI like escitaloprm OR paroxetine.Now a days SSRI is the choice of drug for depression.So need to discuss with your doctor if you have still problem and symptoms.Besides thisDO meditation and exercise.Deep breathing.Proper and regular sleep.Take regular and healthy diet.Consult best psychiatrist nearby and discuss all this and get well soon.Thank you."
},
{
"id": 207086,
"tgt": "How to treat social anxiety disorder?",
"src": "Patient: So I had a report to do on social anxiety disorder, and I ve always had a peoplephobia but I was doing research on SAD I realized that those symptoms are ones that I often experince when being around people, talking to strangers, etc. What are the chances that I have a social anxiety disorder Doctor: DearWe understand your concernsI went through your details. I suggest you not to worry much. In one way everyone has their own anxiety symptoms. Some are anxious to talk to strangers, to talking on stage, to write exams, to be in a high place, to insects, to water and so on. the list is end less. If you categorize by just anxiety symptoms, everyone in the world will come under anxiety disorder. Therefore, to determine whether you have any social anxiety or not, first just ignore the symptoms for around 5 years. Then if you feel, you can consult a psychologist.If you require more of my help in this aspect, Please post a direct question to me in this URL. http://goo.gl/aYW2pR. Make sure that you include every minute details possible. I shall prescribe the needed psychotherapy techniques.Hope this answers your query. Available for further clarifications.Good luck."
},
{
"id": 113365,
"tgt": "Back pain, moving to left side of buttocks, pain on applying pressure. Tried heat packs. Cure?",
"src": "Patient: i have sever back pain, not all the time usually at night or when i rest. it goes down left side of my buttocks and hurts when i put pressure on that side of my body. for example when i put weight on left leg when i walk. is there anything i can take for the pain ? have been using a heat pack. but nothing really helps. Doctor: Hello and welcome, Thanks for your query. When you say your pain is usually at night, is it because you sleep on your back and thus apply pressure on your lower back and/or left leg (mainly till your mid-thigh) ? Does the pain start from her lower back and go down till mid-thigh? If yes, this could most probably be her Sciatic nerve being irritated and is also called Sciatica. Let me list few of the most common causes of sciatica: -Degenerative disc disease: The disc's in every human being undergo a degenerative process with age, and is considered to be normal. When these disc's undergo degeneration they tend to irritate the nerve root present in the lower back, which can result in sciatica. -Sacro-iliac join dysfunction: the lowest end of your spine consists of fused bones called the Sacrum. This bone articulates with the iliac bone (one of the three bones forming your pelvis), thus this joint is called the sacro-iliac joint. Any irritation of this joint can also cause the same pain that would be typical for sciatica. -Piriformis syndrome: There is a muscle present in the buttock region called the Piriformis muscle. The sciatic nerve passes under this muscle, and any irritation of this muscle can also lead the sciatica. -Herniated disc: The Lumbar part of your disc can develop a hernia which is when the inner core of the disc leaks out or herniates. This protrusion can irritate the nerve roots, also resulting in sciatica. --Isthmic spondylolisthesis: In this condition a small fracture allows one vertebral body to slip forward on another. Due to this the sciatic nerve can get pinched and cause the pain. -Lumbar spinal stenosis: Stenosis (narrowing) of the spinal canal can cause the sciatic nerve to get pinched and cause this pain Other causes can be: muscle strain, spinal tumors, pregnancy, scar tissue, infection, direct injury, etc. Please consult your doctor on the same and try to find the cause in order to properly manage the sciatica as different cause require different management techniques. I hope I have succeeded in answering your questions, you could write back to us anytime for further clarifications. Best wishes."
},
{
"id": 210649,
"tgt": "Any suggestion for social problem having no option of feeling on anything?",
"src": "Patient: my husband is completely emotionally just not there, he never has a feeling or opinion on anything. And it's like we are just roommates raising children. I'm a psych major and willing to try anything to get him to at least talk, but he says thats just the way he is and he doesn't expect me to change. I don't know how long I can live like this, we have been married 7 years and I thought he would open up by now but he hasn't. There is definitely a social problem with him because he is like that to everyone, but please help me i'm so lonely in this marriage Doctor: hisorry to hear about how your life is.i would suggest you and your husband should go and see a psychiatrist as its a long duration you are married.there is something between the two of you that is preventing you to open up to you counselling will really help.hope to have answered your querythanking you."
},
{
"id": 187208,
"tgt": "How can a person have blood in urine from jaw infection?",
"src": "Patient: My friend had a root canal that failed & now an unknown pocket under her tooth that turned into an infection. It's been a few days but now she is nauseas & has blood in the urine. No pain like a bladder infection would give though. Why would she have the blood in urine from jaw infection? Doctor: hello thanks for consulting at hcm..it could be isolated finding blood in urine ad may not be related, u take consultation with ur dentist for the root canal, and a nephrologist if she continues to have blood in urine...hope it helps..tc"
},
{
"id": 65067,
"tgt": "What causes lump on the crease of sore arm?",
"src": "Patient: Hi, for 4 weeks now I discovered a boil type lump on the crease of my arm, the swelling has dissapeared but lump is still there, it hurts if touched and also my arm has started to ache and feels as though I have been punched 1000 times Is this just normal effects from boils Thanks Doctor: hI,Thnks for the query.1-In my opinion now the lump is subdued but still active internally.2-From the story from you,the lump is growing slowly and steadily ,due to the infecting germs.3-Thats the reason of soreness of the lump on the crease of the arm, though the swelling has reduced.4-Its now in a acute on Chronic abscess stage, which needs immediate treatment by drainage of the abscess.5-You should approach a nearby surgeon-doctor as soon as possible for the same.If any more questions in this regard,I would love to answer for you.Wellcome."
},
{
"id": 192831,
"tgt": "What causes itchy allergic rashes on the head of penis?",
"src": "Patient: HI, recently i got some rash on the head of my penis. Its very itchy and allergic. At first only one rash appeared and for last 2-3 days another one appeared now. I noticed some white liquid coming out of it now and I am feeling very allergic and scrubbing my penis. I have taken tablet Centrizine Dihydrochloride. Also put some plaster around the rash. Any suggestion please? Thanks. Doctor: Hello, I am concerned about your health issues. This seems to be an infection of the head of your penis called balanitis. It usually occurs in uncircumcised males. You need to see your doctor for the antibiotic prescription and detailed checkup as well if its something else or balanitis. Hope I have answered your query. Let me know if I can assist you further. Take care Regards, Dr SAMEEN BIN NAEEM, General & Family Physician"
},
{
"id": 71825,
"tgt": "What causes constant cough followed by inner stomach pain?",
"src": "Patient: I am having cough for past 15 days and i dint get reduced for any antibiotic and cough syrups and i have taken blood test and other test also for TB diogonise but for me everything is normal but the thing cought dint get reduced and for the past two days i am having severe inner stomach pain what is the reason for this and what is should do now Doctor: Hello This is mostly related to gastric reflux according to the history. Discuss with your doctor Regards Dr.Jolanda"
},
{
"id": 118694,
"tgt": "Child suffering from histiocytosis. Brownish spots spreading on the body. What are these?",
"src": "Patient: My nephew has a disease called histiocytosis he s had it since he was two. He s 11 now and has some long term side effects that they keep an eye on. About a year ago he had a brown freckle-like spot on his back near his lower spine . Since then it has progressed to about ten and they re all around the same size and pretty close together. They are about 6 to 8mm in diameter each! My sister has an appt set up with dermatologist but it s a few months away! Any idea what this can be? Doctor: Hello and welcome to HCM,Histiocytes are macrophage like cells present ubiquitously in all body tissues.Histiocytes play a role in the defense mechanism of the body.They are present in the lymph nodes and cause sinus histiocytosis.Histiocytes are also present in the skin.Proliferation of these cells in the skin can cause skin lesion.The nature of the skin lesions can only be confirmed after skin biopsy.Proliferation of histiocytes in the skin biopsy will confirm the neoplastic process of the histiocytes.Appropriate drug (anti-metabolite) can be instituted after diagnosis of the skin lesions by biopsy.Thanks and take careDr Shailja P Wahal"
},
{
"id": 144974,
"tgt": "Suggest treatment for air embolism in the brain",
"src": "Patient: About a week ago I had a CT scan of my brain due to an auto accident. The scan showed I have an air pocket in my brain. My primary doctor has ordered a second CT scan to be done in about 2 weeks to see if there is any change in the air pocket. What is the usual causes of an air pocket and is having one something serious? Doctor: Hello, thanks for writing to HCM.Your CT scan has showed an air pocket in the brain- also known as pneumocephalus. The most common causes are trauma- as is in your case and neuro-surgery.It occurs when there is a fracture of the skull/para-nasal sinus, and outside air enters the brain cavity. It is not usually serious, unless something called a 'tension pneumocephalus' develops(signs of increased pressure in brain).Otherwise, this is asymptomatic in most patients , and the air will gradually get resorbed.Hope I have answered your query. Do write in if you have any more questions. Wishing you a speedy recovery."
},
{
"id": 53049,
"tgt": "What causes high level of SGPT and SGOT after H pylori infection?",
"src": "Patient: i had H. Pylori infection and had erosive gastritis detected in December, 2010. i took antibiotic as per doctor's advice for eradication of h pylori. but now my recent blood test (in october) shows sgot 40 and sgpt 70, total bilirubin 1.1 and others normal. tests for hepatitis B & C is negative. what may be the probable cause of high sgot, sgpt. kindly also advice for follow up action. Doctor: Hi, I hope you are well, studying the elevation of liver enzymes is very important. Viral infectious causes such as hepatitis B and C should be initially ruled out. In addition, certain metabolic and autoimmune diseases must be pursued intentionally. In your case, the association of drug-induced liver damage should be sought, since your HP eradication regimen probably includes amoxicillin and clarithromycin, which can cause liver enzyme abnormalities. In any case, this is a diagnosis of exclusion, and should be evaluated narrowly over time."
},
{
"id": 41113,
"tgt": "What causes inability to conceive inspite of getting regular periods?",
"src": "Patient: hello doctor,its been since 7 months i am trying to get conceive,me n my husband underwent all tests as prescribed by known gynecologist and our reports were perfect in all the tests done..i was taking \"follinine\" tablets aswell for 3 months but nothing happened.i am getting my cycle regularly ,wht would be the wrong.is it the time of intercourse is wrng time or ani other possibility. Doctor: Hello, in my opinion now you should get an IUI done after ovulation induction and follicular monitoringIn case you have any questions in future you can contact me directly on http://bit.ly/drmanishajain"
},
{
"id": 181271,
"tgt": "What causes headache post wisdom tooth removal?",
"src": "Patient: my son had 6 wisdom teeth removed and his 2 eye teeth exposed - the left one was under a lot of bone - they forgot a tool at their other office and so he woke up during the surgery, he has had headaches since the surgery that are worse with light and sound. today the oral surgeon said that there was some inflamed tissue on the left eye tooth site and is going to flush the area - but doesn't think that is the reason for the headaches, no nausua Doctor: Thanks for contacting healthcare magic. You mention that you have headach after removal of wisdom tooth that may be because of over strain of muscle during treatment. Some portion of tooth remain in bone lead to this type of problems. Take one x ray to recheck any tooth portion still present or not.More amount of bone cutting also lead to swelling and muscle strain.You are not mention which wisdom tooth removed maxilary or mandible.In lower jaw means mandible wisdom tooth removed than chanses of dry socket more.In dry socket warm saline irrigation and idoform paste helpful to prevent infection. But you are not mention a intensity and duration of pain.Consent a oromaxillofacial surgeon for further treatment. I hope my guidance helpful to you.. Take care. Thank you."
},
{
"id": 103762,
"tgt": "Lupus SLE, Fibro, Cervical Dystonia, Chronic-Obstructive Asthma. Taking Cymbalta, Plaquenil generic, Clonazapam. Guide",
"src": "Patient: thank you - was referred to Heart Specialist (from Rheumotologist) had heart palpitations. Heart Dr office did 5 tests on me (4 total hours) Lexiscan, UltraSound, Chemical Stress Test (didn't like this one - the gal administrating it casually called it a 'chemically induced heart attack'...). others ? name. I did not see the Heart Dr... he did not even pop in to check on me. Next day, got a call from office girl with \"You have a normally functioning Heart and a mild leak in heart bicuspid valve\" - I asked was that all... does it show what was causing the palps. (PS no monitor used in the tests) She repeated and said the results will be sent to the Rheum. as he was the one that referred me. Went for OV to him - he said results not in his notes/computer... will have his Nurse follow-up + let me know. Heard nothing for 3 weeks. I typed a letter re-stating above story and took it to Rheum office to give to him... have heard nothing. What should I do... Make another appt to see Rheum (6 month visit not till July-I mentioned that in my letter to him as well... Quandry as to what to do. Your professional opinion is requested please. I am a sincere, retired professional woman 65 with many health issues ie: Lupus SLE, Fibro, Cervical Dystonia, Chronic-Obstructive Asthma... on very few meds. Was using Nebulizer w/ albuteral vials at that time... stopped using them and the major 'thumps' have subsided but still have heart flutters and feel the need to draw deep breathes. On 90 Cymbalta (one 30 + one 60); Plaquenil generic; Clonazapam .05 for sleep; Advair 150/50 once a day; stopped nebulizer.. I appreciate your time. Doctor: you are having all pobem which devlopin this age due to long diseasesyour copd as written snd medicines you are taking might be causing problems written bu u .chronic abstructv asthma can bhve like this to effect like this but no need to add anything for thee resultshave to feel light with almedicines to prevent further pressure on heart to increase its symptopms controll all other symptomps well ith your medicines so that no other syatem is affected in future"
},
{
"id": 99389,
"tgt": "Could weakness and breathing difficulty be related to jaundice?",
"src": "Patient: i was suffering from jaundice and typhoid last month..typhoid was 160 and jaundice billiribin was 1.42..afetr treatment typhoid recover but jaundice hepatitis E bilirubin was 3.85..I am in the treatment and now bilirubin is 1.18..but my agot and sgpt 36 and 110..but I am feeling weakness, shivering of hands and breathing asthma type sometimes in this period..please suggest this weakness and breathing problem is due to jaundice? Doctor: Hello,I am Dr. Shiv, as per the history you have told, your weakness is directly linked to your jaundice part, and breathing problem may or may not be associated with this.i will suggest you to continue treatment for jaundice as advised by your treating physician, and regarding breathing problem i will suggest you to undergo your chest examination by a physician and some tests if needed after physical examination.You should take plenty of liquids, fruit juices, and soft bland diet till your liver reports are back to normal.you can take glucose water also if you are not diabetic.Practice hygiene every time and use purified / boiled water. Hope this was helpful to you.Thanks"
},
{
"id": 61271,
"tgt": "What does a hard lump under the ankle bone indicate?",
"src": "Patient: Hi I have a hard lump underneath the ankle bone near the achillies tendon on the medial side, I got tackled in football and his studs went into the area and it bruised and swelled now the brusing and swelling has gone it's been around 4 weeks there has been a hard bony lump there I have had an x-ray and they confirmed there was no brake or fracture Doctor: Respected user , HiWarm welcome to Healthcaremagic.comI have evaluated your query thoroughly .* This is in relation with post traumatic hematoma formation giving secondary changes in the subcutaneous space .* Needs soft tissue sonography evaluation for the same .Hope this will help you for sure .Regards ."
},
{
"id": 34955,
"tgt": "Suggest treatment for fever, diarrhoea and lung congestion",
"src": "Patient: I am an adult with fever for 6 days- ranging from 100.3 to 102.3. Body aches, lung congestion- phlegm was think and opaque but now very copious and watery. Just began having diarrhea today. Today first day with no fever, but chest/ribs/lungs very sore, wheezing upon exhalation. Exhausted. Take methotrexate and Humira for rheumatoid arthritis. Doctor: Hello dear,Thank you for your contact to health care magic.I read and understand your concern. I am Dr Arun Tank answering your concern.You are having the infection of the chest possibly of pneumonia.For confitmation you should do xray chest PA view along with culture and sensitivity from the sputum coming out.This will guide us possible consolidation site and also the probable organisms and its sensitivity pattern.Taking antibiotics can cure the infection at earliest.Diarrhea you are having seems to arise from the side effects of the drugs of the arthritis. Please do good respiratory exercise like deep breathing and other. This will help you clear your chest rapidly because it will improve the circulation in the chest. The steps of exercise can be taught to you by the physiotherapist in the right way.Please eat natural foods like foods, green vegetables etc. They boost your imunity and you can fight with infection better.I will be happy to answer your further concern on bit.ly/DrArun.Thank you,Dr Arun TankInfectious diseases specialist,HCM."
},
{
"id": 142973,
"tgt": "What causes blisters on the shin after having spinal cord injury?",
"src": "Patient: My husband had a spinal cord injury in Sept. 2013 which left him paralyzed and unable to urinate on his own for one month. He is now walking with a cane and can urinate on his own but still not properly. He is on Flomax to help. He is also on water pills because his legs will swell up. An emg showed nerve damage. My question is his shins are very shiny and they occasionally blister or split open even when his legs do not look swollen. This is a great concern to me. Even though he does not work or walk around like he used to, he is certainly not sitting around all day like he was when he was first injured. Any ideas? Doctor: Hi, Welcome to HealthCareMagic.com I am Dr.J.Mariano Anto Bruno Mascarenhas. I have gone through your query with diligence and would like you to know that I am here to help you.The Blisters can be due to 1. Autonomic Neuropathy2. Vitamin Deficiency3. Involvement of Autonomic Pathways in SCIClinical and Lab Evaluation is needed to Narrow down the diagnosis and start appropriate treatment. Please consult a Neurosurgeon at the earliestHope you found the answer helpful.If you need any clarification / have doubts / have additional questions / have follow up questions, then please do not hesitate in asking again. I will be happy to answer your questions. In the future, for continuity of care, I encourage you to contact me directly in HealthCareMagic at http://bit.ly/askdrbruno Best Wishes for Speedy Recovery Let me know if I can assist you further.Take care."
},
{
"id": 70406,
"tgt": "What is the thick lump filled with pus swells again after it is popped?",
"src": "Patient: large pimple on backside like a pea at first thought it was a lump that had to be cut off, after a few months of observing it and not seeking a doctors advice , I pressed it and a thick paste white thick paste come out and the pea size pimple becomes smaller or empty after sometimes it swells back again fill with this thick puss . what is it? Doctor: HI. This is a classical example of a sebaceous cyst. If this comes again, please consult a Surgeon and get it operated ."
},
{
"id": 18708,
"tgt": "Suggest treatment for recurrent atrial fibrillation",
"src": "Patient: I have severe cardiac disease. CABG 18 years ago. Occluded LAD. 10 stents. Increasing frequency of Afib. Taking 400 mg amiodarone OD. Had ablation 6 weeks ago but Afib came back. What other options? Also occluded subclavian jugular veins on both sides so cant get pacemaker. Doctor: Hello and Welcome to \u2018Ask A Doctor\u2019 service. I have reviewed your query and here is my advice. Its better if you can send me the details like reports of echo and other. I think, in your case it's better not to chase for rhythm control strategy as it's unlikely to be successful in you. It's better to go for rate control strategy with beta blockers, calcium channel blockers and Amiodarone if needed. Hope I have answered your query. Let me know if I can assist you further."
},
{
"id": 113972,
"tgt": "How can the back pain of my wife be cured which she is having after delivery ?",
"src": "Patient: Hi, my wife is having a stiff back from last night and could not sleep for a single minute. She is a mother of 6 months old boy. The pain is so much that she is not able to sit down/bend or even lift the baby to feed him. Request your valuable advise to prescribe some tablet for her to get releif for the pain. Her age is 30 and she is 5 feet tall and her weight is 46 kg. Doctor: she should consult an ortho and go in for an XRAY. did the pain start suddenly or it was after some kind of injury."
},
{
"id": 49269,
"tgt": "What is the cause for flank pain after a hx of kidney stones when on ASA?",
"src": "Patient: I have a hx of kidney stones, was having flank pain yesterday. went to the ED, they did a CT scan, was negative for kidney stone. I continued to have pain, all through the night. It has went from my left flank area to my left chest area. I also have a gastric band. I don t want to go back to the ED if nothing is wrong. But I also don t want to wait until something goes wrong. I know its sharp pains, worse when I breath. I took 324mg of ASA this morning, been extremely fatigue and sleepy but serious pain. What could it be? I have read various scenarios. I am a RN, don t want to over think the situation but I also don t want to injure myself by not seeking help. Doctor: HelloThanks for your query,based on the facts that you have posted it appears that you have undergone treatment for kidney stone very recently and now have pain in flank.Assuming that you have undergone Extra Corporeal Shock Wave Lithotripsy (ESWL)Iand your CT scan does not reveal any residual stones in kidney or ureter the pain that you have is most likely to be due to trivial injury to the kidney by shock waves of ESWL.This is nothing to be worried about .Please get your routine urine test and urine culture done to find out the organisms causing this infection and antibiotics to which they are sensitive to.Please take broad spectrum antibiotics like Cefexine along with urinary antiseptic like Nitrofurantoin twice daily and urine alkaliser thrice daily.Later on switch on to appropriate antibiotics as per culture report.Get the prescription of medicines from your family Physician.Ensure to drink more water.To keep your urine dilute This will help to control dysuria.his will get cured within a week.Dr.Patil."
},
{
"id": 25643,
"tgt": "Suggest treatment for premature ventricular contractions",
"src": "Patient: I am a 40 year old female who has recently been diagnosed with PVC s. I had a 24 hour Holter monitor that showed 10,000 PVC s in that time period. The PVC s go away with exercise. I had become an avid long distance runner and lost 20 pounds in the past two years. I have run 2 half marathon races and was logging about 20 miles a week. I would push myself sprinting at the end going full force. My doctor has prescribed a Beta Blocker to see if the PVC s will lessen. He said I would be a good candidate for ablation if the Beta Blockers don t work. I am not running much now because I ve injured my knee, but I still run some and attend Boot Camp type classes at the gym. Did my running cause the PVC s? Is there anything natural I can take or a special diet I can follow to make them go away? Thank you so much for your advice! Doctor: Thanks for your question on Health Care Magic. I can understand your concern. PVCs (premature Ventricular contraction) are due to conduction defect in heart. It is due to accessory pathway in the heart. It is not caused by exercise (running) is not a cause for PVC. In patients PVCs can be due to stress and tension. So if you are having stressful life then avoid stress and tension, be relax and calm. No natural remedies are available for PVCs. You need to take anti arrhythmia drug like beta blocker to suppress accessory pathway. If beta blocker fails then ablation of that pathway is the only option. Hope I have solved your query. I will be happy to help you further. Wish you good health. Thanks."
},
{
"id": 105692,
"tgt": "Persistent dry cough, pain in right chest cavity, asthmatic trait in family",
"src": "Patient: Am 37 year old man and has been having persistent dry cough and painful right side of chest cavity. Been going on for sometime and several x-rays reveal nothing wrong. Have an asthmatic trait running in the family but never had any asthmatic bout in my life. Does this mean I should get worried about having an asthmatic bout anytime? Doctor: Persisitent dry cough is also a type of presentation of asthma called `cough variant asthma` and the pain could be because of constant coughing. Cough could also be due to a condition called GERD or LPR in which the acid from stomach regurgitates upwards into the throat and causes cough. You should get a PFT, blood for AEC and sputum for eosinophils done. Take Montair 10 mgm, levocetrizine 5 mgm and Pantocid 40 mgm. This will be of help."
},
{
"id": 113859,
"tgt": "I am having severe back pain , kindly advise me",
"src": "Patient: I am having severe back pain , lower pelvic pain , and naseua. I also have lumps all throughout my breasts, and they are very painful. Doctor: Lumps , have you got it examined and diagnosed. It should not be left unchecked as it could be a serious thing like even a cancerous lesion. So you should be careful in such lesions. First get it done."
},
{
"id": 115200,
"tgt": "Does grapes thicken the blood?",
"src": "Patient: My mom who is 86 has been on Warfarin now for 18 years. All of a sudden recently, her blood has been either way too thin or she is at the extreme opposite of being way to thick. I would like to know if grapes are a blood thickener because she craved grapes for about 2 weeks and ate them all day long. Doctor: Hello dear,Thank you for your contact to health care magic.I read and understand your concern. I am Dr Arun Tank answering your concern.No, it's your belief that grapes can thicken your blood.Warfarin is usually given to thin the blood as in your mothers case. If excessive warfarin is taken on the blood than blood can be thinned out.If blood goes much thin than Vitamin K can be taken to revert the effect of the warfarin. Please take the above treatment under your doctors guidance as its requires constant monitoring of INR value of anticoagulant.I will be happy to answer your further concern on bit.ly/DrArun.Thank you,Dr Arun TankInfectious diseases specialist,HCM."
},
{
"id": 71086,
"tgt": "What causes shortness of breath along with pain in the chest and back?",
"src": "Patient: my wife was in the hospital last week for 7 days with a bowel infection i think they called it C-Diff and she had pneumonia in her right lung ..when she was released last friday she said she felt better and was sent home with 2 different antibiotics and pain meds .. but in the last 2 days she having severe shortness of breath and tighting in her chest and back to the point that she cant even complete a sentence without running out of breath ...as well as the pain in her chest and back is increasing .. Doctor: Hello and Welcome to \u2018Ask A Doctor\u2019 service. I have reviewed your query and here is my advice. In my opinion, you should immediately take her to hospital to rule out worsening of pneumonia and bronchitis. She needs urgent chest X-ray, CT thorax and PFT (Pulmonary Function Test). If chest X-ray and CT scan are suggestive of worsening pneumonia then she will need higher antibiotics and Intravenous fluids. If PFT is showing obstructive defect then she will need inhaled bronchodilators (Formoterol or Salmeterol) and inhaled corticosteroid (ICS) (Budesonide or Fluticasone). Don't worry, she will be alright with all these. Hope I have solved your query. I will be happy to help you further. Wishing good health to your wife. Thanks."
},
{
"id": 179828,
"tgt": "Does my child need to attend a special school for children with mild mental retardation?",
"src": "Patient: I have a son aged 6 years 3 months. We have consulted a psychiatrist for IQ testing. Based on the IQ score of 70 my son was diagnosed as Mild Mentally Retardant (MMR) and the doctor advised to join my son in a special school. But we are not sure to accept it. He might be having some difficulties in social behaviour and problem solving, but he is good at languages (writing alphabets), playing, doing skill based activities, playing with toys. Doctor: HIThanks for posting your query to HCM . I definitely agree with you . Mild Mental retardation doesn't require him to be put in special school . But teaching him in normal school requires good dedication from the teachers which may be difficult to get as it puts more work on teachers . Also your child may feel that he is not as smart as his friend and may push him into depression . So these factors also needs to be considered . What I advise is you can teach him at home for 1-2 years and then put him to a normal school ."
},
{
"id": 142625,
"tgt": "What could cause numbness in limbs and head, also have sciatica pain?",
"src": "Patient: I have sciatica pain from a hip reconstruction I had in 2002. Now I'm experiencing weakness when lifting my arms. FAcial numbess, mostly on left side but feel a little on right. Legs are almost numb from knee down, it does travel up my left leg. Feet numb and tingling too. My head is almost kinda numb and I can't think straight. Dizzy on and off. I feel sick to my stomach. Please help.Thank you, Nancy Doctor: Hi, Welcome to HealthCareMagic.com I am Dr.J.Mariano Anto Bruno Mascarenhas. I have gone through your query with diligence and would like you to know that I am here to help you.The history is suggestive of Peripheral Neuropathy You need to visit a Neurologist and undergo Clinical Examination Examination of ReflexesExamination of Sensory System Nerve Conduction Studies After that we can treat and try to cure you at the earliestHope you found the answer helpful.If you need any clarification / have doubts / have additional questions / have follow up questions, then please do not hesitate in asking again. I will be happy to answer your questions. In the future, for continuity of care, I encourage you to contact me directly in HealthCareMagic at http://bit.ly/askdrbruno Best Wishes for Speedy Recovery Let me know if I can assist you further.Take care."
},
{
"id": 84669,
"tgt": "What is the remedy and side effects of udlive 300?",
"src": "Patient: I am asking the following question for my daughter who is approx. 8 years old., weight approx. 23 Kgs. 1) What kind of remedy provide to consume udlive 300 2) Is there any side effect, if so what will be? 3) Is this medicine effect on golbloder stone? Doctor: Hi,Udiliv (ursodeoxycholic acid) is commonly prescribed for the treatment of liver diseases including hepatitis, gall stones, cirrhosis and cystic fibrosis. It helps in dissolution of cholesterol rich gallstones. Generally it is well tolerated however, it may cause diarrhea, indigestion, vomiting, headache, and allergic reactions.Hope I have answered your query. Let me know if I can assist you further. Regards, Dr. Mohammed Taher Ali, General & Family Physician"
},
{
"id": 163792,
"tgt": "Suggest treatment for frequent urine infections in a child",
"src": "Patient: hello my daughter is 4 years old shes had about 3-4 urine infections altogether last one being about a month or two ago. Shes had blood in her urine when she had the last uti, which she got the all clear for after she recovered. In the last 2 days she is beginning to wet the bed again and the symptoms look like they are coming back. There is no pain just running to the toilet alot Iam so worried and dont want to keep giving her antibiotis I dont know what to do. Doctor: Hi... I understand your concern. By what you say this could be cystitis or inflammation of the urinary bladder or it could also be stone in the urinary bladder.I suggest you consult your pediatrician with this tips and worked up further.Regards - Dr. Sumanth Amperayani"
},
{
"id": 135036,
"tgt": "What causes severe pain in thigh area?",
"src": "Patient: Hi my daughter for no reason has server pain in her upper thigh area it happen on Monday and was fine until her leg just went from under her last night and she is 10 I need to no do I bring her to a doctor or is it just a pull muscle she does a lot of irish dancing but didn t happen while she was dancing Doctor: hello healthcare user.. this appears to be a a muscle pull...I would advice you to take rest for some days...apply ice over swoolen area....take antiinflammatory medications for few days...also I would like to advice you to get an Xray done...thank you"
},
{
"id": 210352,
"tgt": "Experienced fast heart beats and shaking body when i was very sad and in stress",
"src": "Patient: Thanks for give this chance to ask my concerns. The other day i was very sad and stress talking with my husband and suddently i start to feel my heart beating very fast and my body shaking very fast as i was freezing..since that i have experience oain on my left side and i think there are the muscles ache...should i do something? Doctor: Hello, It is most likely due to the stress and so, need not worry so much. For your doubts of having some heart ailment, you may get an ECG, this will rule out any related heart condition. and if this is fine the cardiac doubts are unnecessary.Next time try to avoid such conversation and practice deep relaxed breathing technique. you can practice it twice daily and then do it under any stressful situation. this will help you a lot and you will not need meds.Hope this helps you good luck! Dr. Manisha GopalMD psychiatry"
},
{
"id": 149862,
"tgt": "MRI shows spondylosis deformans with disc degenerative disease. Lump near lumbar spine. Treatment?",
"src": "Patient: MRI study of my wife(28yrs) says Evidence of posterocentral and both lateral disc bulge of L4 L5 with mild central canal and bilateral lumber lordosis is lost with osteophytes in lower lumber vertebrae.The Spinal Canal Dimension at disc level in midsagital section in mm is L1L2-13, L2L3-13, L3L4-14, L4L5-11, L5S1-13 Impression-Spondylosis deformans with disc degenerative disease with cauda equina compression and bilateral compressive radiculopathy,there is also a lump near lumber spine with back and leg pain,So please tell me about the treatment and my sexual activity with my wife,thanks Doctor: Hi friend,Welcome to Health Care MagicThere is compression \u2013 that is an indication for surgical relief (decompression)It is a mechanical phenomenon \u2013 needs mechanical intervention / drugs and physiotherapy may give temporary or symptomatic improvement...Earlier the better \u2013 to avoid nerve damageNo restriction on sexual activityTake careWishing speedy recoveryGod blessGood luck"
},
{
"id": 132532,
"tgt": "Need treatment for sprained ankle",
"src": "Patient: My daughter sprained her ankle playing volleyball 3 days ago. x-rays showed no broken bones. the issue im having is the leg, past the knee is visably a couple shades darker than the other leg and is cold (ice) to touch. I took her back to er, they couldn t answer as to why it was that way. she has numbness, tingling, in the knee. and of course there is still a lot of pain. she will not put any weight on it. she is 15. Ive iced it q2hours at night and its been elevated during school hours...HELP! Doctor: Hello. thank you for writing us here.The most common cause could be injury to her common peroneal nerve on the knee joint which causes symptoms of tingling sensation and numbness of her leg.You can apply a bandage so her feet is immobilized and give her diclofenac sodium for pain. Also I would suggest you to see an orthopedician for proper management. she could be told for bed rest and application of a plaster for immobilization of the limb.Best regards,Dr Gunjan"
},
{
"id": 219755,
"tgt": "Will total hysterectomy lead to severe pain in the legs?",
"src": "Patient: If you can answer my health problem you should receive a Nobel Prize. I had a total hysterectomy approx. 10 years ago. I am now 54 years old and have had every diagnosis you can think of. Initially, my feet felt like I was walking on rocks - sent to arthritis specialist. Meds they offered were useless - including Embrel injections. I have had gallons of blood taken which has resulted in nothing. Nerve conduction tests - again nothing. Next a neurologist who came up with nothing but the suggesiton of anti-depressants. The pain is pins and needles non-stop. Wakens me multiple times at night, has kept me all but bed ridden. The pain of walking is not worth it therefore I have lost the enjoyment of my 3 teenagers as I don t do much if anything with them anymore. My husband and I have not had relations in years. Had I known this could be the result I NEVER would have done it - it is almost worse than being dead as there is nothing I can do or take that I know of to stop the 24 hour pain every day. What do you think the problem is? Desperate for help. Doctor: I vascular surgeon to test the circlation in your legs might give some answer. Also check for diabetes. Diabetes can cause neuropathy for leg pains. I do not think Hystrectomy has any relation with this problem. i do not know if ovaries were removed with hystrectomy or not . FSH level might help in determining the menopausal and the ovarian function."
},
{
"id": 60585,
"tgt": "Can lip cracks that doesn t bleed pass HBV ?",
"src": "Patient: i have lip cracks that doesn t bleed..i am 21 years,5'3\".i am digoned with hbv ,can it pass..???? Doctor: Can chapped lips pass it tho?"
},
{
"id": 185129,
"tgt": "What is recovery period after crown preparation before air travel is advised?",
"src": "Patient: Good evening... I am currently experiencing a very sore jaw from dental work... the preparation for new crowns on No19 & No20... lower left... permanent crowns to be installed tomorrow... and a vacation upcoming... involving air travel. Please advise as to the appropriate time line for recovery... before air travel. Thank you. Doctor: thanks for your query, i have gone through your query. you can go ahead with the crowns for your teeth. it will not interfere with you air travel. make sure that the teeth are root canal treated before capping. because in future if the teeth get infected again we have to repeat the crown. i hope my answer will help you. take care."
},
{
"id": 201975,
"tgt": "What could causes a itchy mole to appear on foreskin?",
"src": "Patient: A small brown mole appeared on my foreskin about a month ago, it has caused no irritation up until last week but it feels itchy most of the time, I'm a 22 yr old male and I spend very little time in the sun, never mind nude bathing. I'm very concerned aboutGoing to the doctors. I know you will tell me to go anyway butIs this common? How worried should I be? Many thanks Doctor: Hi, I would not be worried much about a mole on the foreskin specially if it is not causing you any problems. However, I would advise you to keep it under observation for any increase in size, change in color, pain or discharge. If anything like this happens then you should see a Dermatologist. Take care.Dr Rishi, India."
},
{
"id": 152046,
"tgt": "What should I do for learning disabilities in writing or Dysgraphia ?",
"src": "Patient: Since I began writing papers in college, I have had a hard time organizing my thoughts on paper. Ultimately, I make the grade but it takes tremendous effort. I often had to go to the writing center. Thus, school has been harder for me than it should have been. I am now going for an advanced degree in college. I am having the same terrible problem with writing papers. I have begun going to the writing center again. My professors want to have a meeting with me. I am a nervous wreak and think they want me to leave. What should I do? I have not been tested for a ld so I don t have documentation or anything. Thanks for your help. Doctor: Hi welcome to Healthcaremagic Hi.., you need must not worry about your problem, think positive, you needed to be counseling, only counseling will help... medicines or drugs have not much effect for this problem, consult a psychiastry... Hope I have answered your question.. Takecare..."
},
{
"id": 159740,
"tgt": "Any method is there to take food other than food pipe for a patient of GBM?",
"src": "Patient: my mother in-law have GBM stage 4 in brain and she was operated in january 2010 , 2 week ago doctor precriebed her food pipe as she is unable to take anything but she refused it yesterday what will we do her total intake of liquid 500ml per day what will we do Doctor: Well, if she has to be force-fed anyway and if feeding through a food-pipe (nasogastric tube we assume) seems to be very difficult or impossible option, then one may think of 'feeding gastrostomy'. It is a small operation by which a feeding tube is inserted into the stomach directly. Feeding through a gastrostomy tube is much more comfortable for a patient than having a food-pipe through a nose. (Imagine both the situations for yourself, and then you will understand.) Thanks."
},
{
"id": 17663,
"tgt": "What causes abnormal BP when on Exalgo and Valium?",
"src": "Patient: I have been in pain management since 2007 because I need my knees replaced, 5 discs in my back fused. there are a few more thins but those are the main ones. I have been on methadone for over 3 years at 160mg. 4 10mg 4x a day. they just changed my medicine to give my body a break for a few months. they put me on exalgo er at first 3 16mg 1x in the morning. they tapered me off the methadone and tapered me on the exalgo er at the same time in a week. they told me I wouldn t have any withdrawal but I have had bad withdrawal even though I have another drug in my body. my b.p. has been normal and my pulse has to but ever since this my b.p. has been 167over140 and pulse at 120. it has varied the past 2 weeks but it has still been around these levels. Is it because of the withdrawal or from the exalgo? they gave me valium to help my body with it s cravings but it hasn t helped my blood pressure or pulse either. My question is what is causing it and when will it stop?? Doctor: Hello, I passed carefully through your question and would explain that your question and would explain that high blood pressure values could be related to the withdrawal syndrome and increased pain or anxiety. Anyway, I recommend you to closely monitor your blood pressure values in the next week. If high blood pressure values persist, I would recommend starting antihypertensive therapy. Some blood lab tests (complete blood count, kidney and liver function tests, blood electrolytes, etc.) are necessary too. Hope I have answered your query. Let me know if I can assist you further. Take care Regards, Dr Ilir Sharka, Cardiologist"
},
{
"id": 115147,
"tgt": "Could bouts of vertigo be due to subarachnoid hemorrhage?",
"src": "Patient: I had a subarachnoid heamorrahge two years ago which resulted in me having surgery. Two stents and coiling. I made a very good recovery. Since September last year I have been suffering from terrible bouts of vertigo. I am violently sick with it and usually take a few weeks to recover. Can you tell me if you think this might have anything to do with the heammorra.ghe Doctor: Hi, dearI have gone through your question. I can understand your concern.You may still have some subarachnoid hemorrhage. You should go for ct scan brain to rule out any possibility of hemorrhage. Then you should take treatment accordingly.Hope I have answered your question, if you have any doubts then contact me at bit.ly/Drsanghvihardik, I will be happy to answer you.Thanks for using health care magic.Wish you a very good health."
},
{
"id": 132750,
"tgt": "Need treatment for third disc abnormality",
"src": "Patient: my third disc is abnormal would it cause me to walk funny or on the side of my feet? and also would it ever cause me to be in a wheel chair ?when u have a 3rd disc abnormal are you born like that? cause my lower back and some times middway of my back bothers me Doctor: Hi Hope this message finds you in good health.I have gone through your complaints and understand your concern.U seem to have compressed a nerve in the back somewhere that might be causing radiating pain in those regions.Possibility of a lumbar disc prolapse should be kept in mind.I suggest you get an Xray as well as an MRI to find out the exact cause3rd discs generally heal properly if treated apropriately.Physiotherapy,analgesics,anti-inflammatory drugs should help.Nothing to worry about.I hope your question has been answered.If you have any follow-up queries,feel free to consult me anytime.Thanks,Take care,God bless."
},
{
"id": 153148,
"tgt": "What causes ribs and back area pain, breathlessness after a radical hysterectomy?",
"src": "Patient: I am 49. I had a total radical hysterectomy on 21st August because of uterus cancer Ib stage diagnosed. The complications were high temperature, too much liquid drained, pelvic hematoma, left leg edema. I was released home with high temperature, leg edema, hematoma and bad blood test results (CRP 150, white blood cells 12000, high sedimentation level). The doctor said it takes time to heal after such a big operation. Oncology consillium reported no other treatment (either chemotherapy or radio therapy) is necessary. After two days my temp. got higher (38.3 C) and I got pain in my right side starting from the ribs downwards both front and back. The next day the pain moved to the left side on the back. I feel a twinge which prevents me to breath in deeply. The pain can be very severe and wake me up. Now I am at the pulmonology ward. They performed lungs Xray, heart ultrasound scan, legs colour doppler scan, lungs scan and blood test. They say pulmonary embolism is excluded as a possible diagnose but it could be pneumonia. I have been treated with antibiotics for 5 days and this morning I was woke up by pain again. What do you think? Doctor: Good morning. The likely cause for your pain is either pneumonia or musculoskeletal pain. YOu need not to worry as your cancer was early stage and no further treatment was necessary. Pain will go in a few days. be happyregards"
},
{
"id": 195881,
"tgt": "Is long term usage of an expired lubricant a concern?",
"src": "Patient: Hello, I bought 3 (or 3 and a half) years ago a durex play lube and have been using it sometimes with my girlfriend since then. I didn t know it was supposed to be used within 3 months from first opening. We even used it after its expiration date which was approximately one year ago. I also applied it directly on my penis sometimes when I was masturbating. We haven t noticed any skin infection or irritatgion. What could happen to us? Doctor: Hello,Although, you have not any reaction to condom material but it is not advisable to use expired condom. So, avoid expired condom use, it is not at all recommended.Condom has an expiry of 2 to 3 years so buy it after checking the expiry.Hope I have answered your query. Let me know if I can assist you further.Regards,Dr. Parth Goswami"
},
{
"id": 104316,
"tgt": "Persistent dry cough. Doctor prescribed and changed medicines every fortnight. No relief from any of the medicines. Suggestions?",
"src": "Patient: Since end of august 2012, I start dry coughing and last about two months,Dr provided Apo-Amoxi-500mg, then Sept 9 change to Cotridin Syrup,then Oct2 change to Ventoline-HFA-100ca Inhaler , then Oct 7 change to Novo-Lovoflexacin-500mg,then Oct 15 change to Ratio-Cotridin, then oct 25 change to Novo-Lovoflexcin,thenNov 4 change to ventolin-HFA Inhaler,thenNov 8 change to Sandoz-Clarithrong Cin 500mg, then Nov 22 change to Sandoz Clalrithrong 500mg, thenDec19 change to Atorvastin 100mg Tablets,thenJan 3 change to Ciprodex otic suspension, then Jan3 change to Ratio-Cotridin and last Jan7 change to Sandoz- Clarithrong Cin 500 mg. I have taken all these medicines: Dry cough to chest wheezing .But still I have to clear my throat because mucise. It is about four and a half month, still not fully recovered. Please help Doctor: all symptomps of allergy dont use these medicines take montair fx tab bd syp ventrolin 1 tsf tds syp tossex 1 tsf bd add liquid antacid syp tds for 3 week sroflo inhaler sos in emergenct"
},
{
"id": 161985,
"tgt": "What causes vomiting after eating and sleeping for an 1 years old?",
"src": "Patient: For the last few months my daughter (almost one) vomits just after she goes to bed about 50% of the time. Sometimes soon after falling asleep, sometimes immediately after her bottle, sometimes after crying or coughing. My doctor said it is not a concern if it only happens once. But that was a couple of months ago and I m wondering if I should go back. Doctor: Hi, Yes, if it is happening very often then need to be evaluated further. This could be due to an umbilical hernia (?)/ Gastritis. But before going for a further evaluation. You can just try out the following things: 1. Just don't make the baby lie down just after meals/diet/feeding. 2. Short and frequent diet plan to follow strictly. 3. Medication like: Domstal drops to be given. Hope I have answered your query. Let me know if I can assist you further. Regards, Dr. Ajaygupta009, General & Family Physician"
},
{
"id": 60382,
"tgt": "Can smoking be the cause of high SGPT ?",
"src": "Patient: im 24 yrs of age.. my height is 5'7''my sgpt level is 130.. is smoke danger to high sgpt? is smoke is danger of a high sgpt? im 24 yrs of age 5 7 . my blood ,sugar,uric is normal.. my sgpt level is 130. is smoke danger of a high sgpt? Doctor: hello jayare, welcome to HCM there is no direct relationship between smoking and sgpt. it may be due to other causes like viral infection of liver like hepatitis and alcohol consumption. so find out the cause for it by consulting you doctor. smoking definitely bad for all organ system.so try to avoid smoking take care"
},
{
"id": 139854,
"tgt": "What does this CT scan report of spine indicate?",
"src": "Patient: On my C.T. At the level of C3-C4 - Disc bulge with mild impression on the dural sac at C4-C5 Disc bulge with mild impression on the dural sac. Compression on he left nerve root. At c5 -C6 Disc bulge with cenraal impression on he dural sac.The iner-vertebral foramina are narrowed due to the degenerative changes Doctor: Hello,These CT scan findings are compatible with chronic degenerations of the vertebral bones and intervertebral disks. I would recommend taking painkillers and holding a cervical collar during pain exacerbations. If the pain persists, consulting with a spinal surgeon may be needed.Hope I have answered your query. Let me know if I can assist you further. Regards, Dr. Ilir Sharka, Cardiologist"
},
{
"id": 92961,
"tgt": "Nausea, vomiting, diarrhea, stomach and abdominal pain. Flu symptoms?",
"src": "Patient: I have been having nasua and some abdominal pain . I got sick and was out a few days then went back to work. the Diarehha and the vomitting stop but the stomach ache, nasua and abdominal pain have not. My doctor said it was just the stomach flu and would be back to normal. it has been over two weeks and im still getting the same pain. Any suggestions? Doctor: hi, looks like gastroenteritis , as your doctor ve told in simple words, don't worry , u ll b alright. thanks"
},
{
"id": 189092,
"tgt": "Getting blister on gums and blister pops. Suggest",
"src": "Patient: I had a crown put in back in january. ever since, I have been getting a blister on the outside of my gums just under the crowned tooth. they only last for a few days and pain comes in. The blister pops and the pain goes away until the blister comes back a week or two later. This time however, it has been more painful over the weekend than normal. I am going to call the dentist on Monday, but was wondering if there are some possible thoughts on this issue.Thanks, Robert Doctor: Hi,Thanks for asking the query,According to your clinical history i suspect that there can be abscess formation due to food lodgement between the tooth and the crown.I would suggest you to visit the Dentist and get the checkup done take the x-ray done, go for symptomatic treatment of complete mouth scaling and polishing curettage.Apply metrogyl oral ointment topically over the affected area.Take a course of antibiotics and analgesics.Take lukewarm saline and antiseptic mouthwash rinses.Hope this helps out.Regards..."
},
{
"id": 89804,
"tgt": "Is hernia associated with testicle & abdominal pain ?",
"src": "Patient: My right testicle is painful to the touch in a certain area. It feels like it goes into my abdomen, also on the right side. It really doesn't bother me all day, but enough for me to be scared! I've done tons of lifting and such over the past couple of months, which is when I noticed it. It went away when I stopped lifting etc.. however came back recently b/c of the same activity. Does this sound like a possible hernia to you? Thanks! Doctor: hi thanks for askingyes the history is suggestive of inguinal hernia.......you can do ultrasound pelvis and scrotum to confirm it"
},
{
"id": 157677,
"tgt": "Have stage 4 colon cancer, green bile vomit, jaundiced eyes, dark brown urine, burping, swollen abdomen",
"src": "Patient: My father is terminal with stage 4 colon cancer mets to peritonium (spelling); Last 4-5 days condition -green bile vomit (versus the dark coffee brown 2 months back), jaundiced eyes, dark brown urine and output is very less. Lots of burping; Swollen abdomen, hard rock. Cancerous tumor in sigmoid colon also swollen hard rock. Swelling in both legs, and fluid accumulation in the back, lungs area. Fentanyl patch, tramadol being given for pain control; He is not much in pain from the tumor, but the discomfort from ascetic fluid build up and throw up; I am aware that not many days, can you tell us if it is going to be a week or more Doctor: HIThank for asking to HCMI am sorry to hear that, without the clinical examination it is difficult to say any thing, but does it matter to you? the death is certainly sure for every human being later or sooner, what is wrong if some one is staying with us little longer time, at least we can get a chance to serve the ill one, take a good care of your father have great day."
},
{
"id": 96474,
"tgt": "Vomiting in 3 yr old child",
"src": "Patient: Hello, my son is close to 3yrs now. from past 2 days in have seviour close from yesterday he is not eating any thing and when i tired to give him fever medical he vomoited every thing. I am leterally worried . Kindly help me with this regards, Deepa Doctor: We recommend that you don?t give him any medicine before getting him examined by a pediatrician. If there is fever, sponge him with tepid water. Give him plenty of oral fluids in sips. Since the symptoms did not subside for 2 days then it is better examined by a pediatrician."
},
{
"id": 7767,
"tgt": "Which medication should I use to get rid off brown spots on my face ?",
"src": "Patient: hiiiiii gdmrng im 20 years old. i have brown color spots in my face , which has got from pimples . i have applied many natural kitchen items and i also consulted doctor and my friend also suggested me to apply ERYTOP. since 2 monts, im doing all these things. i also changed my soap. these r not going. please suggest me.... it shud soon get rid of my face.. pls pls.... Doctor: Hi...dear user., Thanks for choosing HCM., Brown colour spots on face.....FRECKLES., it could be due to sun exposure...ok.., 1) Morning use SUNSCREEN LOTION...Melaguard SPF 50.., 2) Night times use BAN A TAN...it is curatio product.., 3) Take plenty of vegetables and plenty of fruits ...ok...good luck.,"
},
{
"id": 36380,
"tgt": "What to do for jaundice?",
"src": "Patient: I have suffered from jaundice attack ,i am very fond of eating spicy and oily foods .Is glucon-D can be a good medicine for liver.Besides i am allergic to sulpher and has high tendency towards asthma and tonsilitis and cold.What is the preventive measure I will take.?R u taking any charges for online consultation? Doctor: HelloWelcome to HCMThanks for posting your query in HCM.As you had an episode of jaundice,you should avoid spicy and oily food atleast for a minimum of 4 weeks.This time is needed for the damaged liver to recover.Glucon-D is nothing but a energy drink that provides glucose to the body whenever you are very much strained or unable to take solid foods.It can be taken.It doesnt do any harm to liver.But as far as you can avoid spicy foods.Thank you"
},
{
"id": 50108,
"tgt": "Had a kidney failure. Why is she weak? Has problem with communication?",
"src": "Patient: hi sir I have a serious problem related to my mother. actually my brother got kidney failure before 4 years. from that my mother is started to not to talk not to eat and every time keep silent.And does not take any interest in any thing. now she is very very weak she not able to walk without any support. she is 60 yrs old. we are from varanasi (u.p.). Plz give me any suggestion. what i should do. thanks Ritesh jaiswal YYYY@YYYY Doctor: Hello Ritesh,Your mother is a patient of CKD & now has developed complications in the form of Encephalopathy,malnutrition related cacexia as it appears from your description.She urgently requires to consult a Nephrologist.Meanwhile,you do not give her any fruit,fruit juice or dry fruit.All the best"
},
{
"id": 136074,
"tgt": "Suggest treatment for pain and tingling feeling in legs after injury",
"src": "Patient: Hi, i fell on my hips at hockey today and im getting an ache / kind if tingly feeling in my legs which i cant really tell if its serious or not. Fell once with pads then later once with no pads and hit really hard. Will circulation get messed up or what I feel stupid needing to ask this and im probably way over thinking this Doctor: hishow to an orthopedic doctor or visit ER for an x ray of hip to rule out any crack/fracture.secondly take pain medicines prescribed by doctor and do not do weight bearing till you get clearance from ER about any deep injury.best wishes"
},
{
"id": 9528,
"tgt": "Child. Dry skin patches on face, red cheeks. No relief from Advantan fatty ointment. Suggest",
"src": "Patient: My son (4yo), has had increasing dry skin patches around his mouth, nose, and eyes (specifically from the corner of his eyes and over his eyelids), he has also had red cheeks for the past couple of days.It is autumn where we are, but the weather has been nice enough to spend a little while inside as well as out. He is prone to dermatitis, and currently uses 'Advantan fatty ointment' (contains 1g methylprednisolone aceponate). This has not helped in this case.Not sure if there is something I should remove from his diet, or if we need to see a specialist.Thank you for taking the time to answer (in advance:) Doctor: Hi thanks for writing here.It seems your child has been suffering from atopic dermatitis.It will be better you consult a dermatologist ,but few things you can keep in mind.1.Use a mild soap or body wash,you can ask your dermatologist for a soap or a syndent bar.2.Always apply good amount of emollient like cetaphil moisturising cream immediately after bathing.Use it round the year.3.Use loose cotton clothes covering whole body,avoid wollen clothes.4.Use luke warm water for bathing.5.Do not make steroid ointements as a habit,use them only on and off basis,use only when disease is flared up .6.Give him sedative antihistaminics like atarax 7.See that your house is free from dust particles .Take care."
},
{
"id": 96707,
"tgt": "Can whiplash occur when one is stationary?",
"src": "Patient: i am a bus driver i bwas at work the other day when another bus backed into mine while i was stationary is it possible i could have whiplash as find it hard to move my neck and my left side i have been told you cant whiplash when stationary and if you no the impact was about to happen. Doctor: hellothank you for contacting HCMno in your case whiplash injury is unlikely to occur. you are having muscular spasm due to sudden jerky movement. i would suggest you following things:> take proper rest for 2-3 days> take hot water in rubber bottle and place against your neck and back. it will give soothing effect.> take naproxen 500mg one at night for 3 days. it will relax your muscles and reduce pain.hope this answers your question.wishing you good health"
},
{
"id": 41700,
"tgt": "Suggest treatment for infertility",
"src": "Patient: Hello Doctor, my age is 31 yrs, iwill be 32 by november 2011, my husband is 39 yrs, my weight is 75, my height around 165 cm. am married for 5.5 years without a child , my husband has low sperm count, i have PCOS and fibroid of 4 cm outside uterus, we did ICSI but i did not work. am frustrated and confused, please advise me, i really need to have children of my own. Doctor: Hi welcome to healthcaremagic.I have gone through your question.Both low sperm count and Pcod polycystic ovarian disease should be investigated and treated properly.Low sperm count should be investigated with ultrasound color doppler to rule out varicocele.For pcod consult a gynecologist he/she will advise further.You can go for IUI intrauterine insemination or IVF invitro fertilization if you both are not relieved from these conditions.Hope i answered your question.Would be happy to help you further.Take care."
},
{
"id": 125411,
"tgt": "How to treat foot injury?",
"src": "Patient: Hi. I dropped something heavy on my foot yesterday afternoon and it still hurts. Mild swelling and bruising but I can t walk on my right foot ever since. There is a small bump where I dropped it and a small red dot, if I touch the red dot, unbearable pain sets in. Is it broken/fractured/or? Doctor: Hello, We have to rule out fracture. Consult an orthopaedician and get an X-ray to look for any thin fracture. As of now you can use analgesics/anti inflammatory combination like aceclofenac/serratiopeptidase for symptomatic relief. You can also apply ice packs for pain relief and faster recovery. Hope I have answered your query. Let me know if I can assist you further. Regards, Dr. Shinas Hussain, General & Family Physician"
},
{
"id": 164473,
"tgt": "Suggest medication for hives on a toddler",
"src": "Patient: My daughter is 9 months old, about a day ago a bunch of red dots started to cover her body, like hives , she doesn t have a fever and they don t seem to bother her she hasn t scratched at them at all. Its on her arms, legs, face and around her private area kind of. Is there anything I can give her to make them go away? Doctor: Hi...Thank you for consulting in Health Care magic.By what you quote it should be an urticarial or a simple skin allergy. You can use Hydroxyzine at 1-2mg/kg/dose every 6th to 8th hourly for 7-10 days. Most important thing to be remembered is that it has a propensity to recur (called as second crop) within 10-14 days. If this happens, you can start using the same medicine but I suggest you get the kid evaluated with your paediatrician.Hope my answer was helpful for you. I am happy to help any time. Further clarifications and consultations on Health care magic are welcome. If you do not have any clarifications, you can close the discussion and rate the answer. Wish your kid good health.Dr. Sumanth MBBS., DCH., DNB (Paed).,"
},
{
"id": 203036,
"tgt": "What is the cause for burning sensation of the penis?",
"src": "Patient: this past week ive been having penis pain near the hole while urinating. its a burning sensation and the hole seems red. also after a period of 3 to 6 hours of not touching my penis, there's flint from my boxers that accumulates around the hole. it seems its because of a sticky liquid from the hole. the only sexual thing if done in the past year was a month ago and it was anal with a girl. an other thing is my penis hole appears to be slightly cut if so. please help, id really like to know whats going on. im 19 Doctor: Redness and pain while urinating is suggestive of urinary tract infection. As you had a sex long back before the problem, this infection is unlikely related to that episode. I did not got your concern regarding flint around hole- what is this boxer?However you just need to take antibiotics and drink lots of water to cure your problem."
},
{
"id": 145908,
"tgt": "Suggest treatment for spinal headaches",
"src": "Patient: I had a CT myelogram and spinal tap done about a month ago. I had and still have problems with spinal headaches. They did a blood patch then and it seemed to help great. However, I am still having problems with them. My neurologist is having me try nortriptyline to see if that helps, but I cant get anything done because I am trying to lay flat. With these stupid spinal headaches ever go away? Doctor: Post Lumbar puncture headache is common but usually such headache relieves in less than a week. Persistence of headache may be due to leaking CSF and CSF hypotension. Repeat blood patch , caffeine intake and plenty of fluids may help. Prolonged bed rest may aggravate symptoms. Try to arise slowly from bed after sitting for few minutes first. Hoping you recover fast."
},
{
"id": 130212,
"tgt": "Suggest treatment for severe muscle pain",
"src": "Patient: My entire spine causes sugnfinice pain, my joints feel like achy raw flesh. I hurt 24 7 and ready to give up. I am a burden on my family and Christmas will be much merrier without me, I sit in tears most of the time begging for the strength to end it all, am very dipressed, my doctors just go with the flow they don t get it. If I die I will no longer be in pain-that is my thought process, my wife could be happy with someone who isn t riddled with pain. She deserves better! Doctor: hello,first of all you are not feeling well physically ,it doesnt mean that its the end.its common to feel depressed when you are fighting with the pain.it takes time for doctors to diagnose and understand the illness sometimes.Its the hope and inner positive energy that helps in recovery along with the medical treatment.I would like to know since when you are having the pain in spine and joints?any history of accidental injury?Any family history of same symptoms?Any tests done with specific results?It will be good if you visit one more doctor who is specialist in immunity related disease or a rheumatologist.after your proper examination in person it will be easy to give an opinion as looking at the signs and symptoms.As the symptoms you have said are seen in many illnesses.So do not lose hope,I am sure your family cares for you as much you do.and also I advice you to visit a psychiatrist and a psychotherapist as well for the depression and stress.Few counselling sessions might help you to fight back and feel better.its very common to feel like that when you are not well."
},
{
"id": 198369,
"tgt": "What is the treatment of low sperm count ?",
"src": "Patient: Hi Doctor? I am a man aged 32 years and been married for 4 years without a baby. i did a sperm count test which showed that i have a low sperm count of 3.5 Million and 50% mortality. i was put on oligocare drugs for 60 days and up to now no success. is there any medication that can treat this problem? Thanks Doctor: HelloThanks for query .Based on your statement you have Oligospermia meaning there by that your sperm count is low as compared to required WHO standards .Truly speaking there are no medications that can effectively increase sperm count however following general measure will help to increase sperm count to some extent .1) Practice regular exercise for 45 minutes followed by meditation for 1/2 an hour in the morning.2) Take high protein diet rich in vegetables and fruits and Vitamin A,C,D,E.and Zinc3)Take anti oxidants like Almonds 5-6 everyday..4) Avoid alcohol and smoking,Taking Clomiphene daily for 25 days does help to increase sperm count .Get prescription from your family Physician .Dr.Patil."
},
{
"id": 111861,
"tgt": "Causes for severe back pain due to muscle strain",
"src": "Patient: My daughter (age 25) was recently admitted to hospital due to extreme upper back/left side pain. It had started in the morning after getting out of bed and got suddenly worse PM. She received Oramorph (liquid morphine), a Paracetamol drip and Entinox. These did little to reduce the pain. Later she was given Tramadol. This helped slightly. Blood tests were normal, apart from the D Dimer test which was raised (level 21). Initially though to be a blood clot on her lung, the Consultant said that that was unlikely as an X Ray of her chest was clear and heart trace normal. She was then given Diazepam (to relax her muscles). This seemed to help and later that day she was discharged. She is still in a lot of pain. The medication does help when it kicks in. She had a check-up today. The Doctor said he could detect muscle strain as he side and back muscles were in spasm. Any idea what caused this, what it is, is it likely to return and what about the D Dimer level? Doctor: Hello, Thanks for your query.I suggest you to try back strengthening exercises by extension of the back muscles which can be taught by your Orthopedician or Physiotherapist. This will help to relieve the possible muscle strain which causes the pain.you may be prescribed pain killers and/or muscle relaxants which should give you relief. Physiotherapy may also be advised if necessary. I would suggest you to try swimming as this will help most in back conditions.Avoid weight bearing exercises. Do only mild aerobics which does not put your back to strain. You can discuss about this with your Gym instructor.Eat a diet rich in fresh fruits and green leafy vegetables, nuts, milk, eggs, pulses and cereals. Drink 1-2 litres of water daily. Do brisk walking exercise for 40 minutes daily. Get 6-8 hours of refreshing sleep daily.I do hope that you have found something helpful and I will be glad to answer any further query.Take care"
},
{
"id": 7011,
"tgt": "Why am I not able to conceive inspite of all my reports being normal ?",
"src": "Patient: Hi am 25 years old..trying for kids since 1 year with no luck..from last month 23(my last period may 22nd) onwards am taking B long F and ovacare..i took 5 siphen tablets from my 2nd day of period to 6th day(5 tablets)..i did follicule scan and i came to know i ovulated on 16th day.i tried 12th day to 16th day..now i have 6 days to get period...last time i got period 3 days late..but from yesterday iam getting pain at my inside vagina.it comes and suddenly goes off..i usually get this pain 3 days before my period.but this month i have this pain 7 days before and no discharge..so am going to start period right..?is this is tablets effect i mean starting my period before my period date..last time got late..but this month iam taking tablets too but no luck.really am so sad..my husband semen test is normal..my thyorid test also normal..but am not getting pregnant ..why this is happening to me..?is there any problem in me..?and please help me anyone How many days after ovulation can i expect period? Doctor: Please reply me anyone....please"
},
{
"id": 197058,
"tgt": "What causes a hard pimple on the testis?",
"src": "Patient: i am 40 years male indian, sexually active, started having a small pimple on the left testis. In due course of time, it started feeling hard and started growing. Some times I have sting when in contact with clothes or underwear. What is the significance? How serious is this issue? Doctor: HelloThanks for query .Small pimple on the scroyum that you aave is mostly a Sebaceous Cyst .The scrotal skin is rich in sebaceous glands and hence prone to get more sebaceous cyst due to accumulation of sebum beneath the skin. Normally they fade away without treatment however they need to be treated if increase in size or get infectedConsult qualified General Surgeon for evaluation .You may need to get it excised in Totto under local anesthesia.It is not a serious issue at all .Dr.Patil."
},
{
"id": 63215,
"tgt": "Does lump in armpit and coughing indicate lymphadenopathy?",
"src": "Patient: i m 32 years old.i had a lump in my armpit.went to see a doctor who had me get a breast ultrasound.which showd no mass in my breast.but dignosed lymphadynopathy bilaterally.now she asked ne if i had a tb family history.the answer was neg.but i do get these coughs very frequently.so i have to get a blood cp n esr...can please tell me whats going on n is this lyphadenopathy something to worry about....i m married n have 2 kids ..one is 11 n the other is 4.5 years old.. Doctor: Hi,Dear Thanks for the query to HCM virtual Clinic.I studied your query with available details given in your query.I understood your health concerns.Cause of the armpit lump with Cough-AS your breast on USG study showed normal,the bilateral lymphadenopathy in axilla/armpit,could be due to -Pulmonary TB with general debilityor With Hodgkin's Lymphoma/or NHL.Update more information-of accompanied malaise,fever,joint pains, and any weight loss lately with this cough and armpit lumps.Hence I would advise you to Consult ER Surgeon /Onco-Physician ,who would investigate it by-CBC(Complete Blood Test) with ESR,IGRA (Interferon Gamma Release Test -to find out any evidence of old or current TB infection in the body.X-ray Chest,FNAC Biopsy with HPR study of the armpit lump,would confirm the nature of the lumps and cause of the accompanied Cough with it.So Don't worry till you get the FNAC Biopsy of the armpit lumps, to rule out other causes of such a lump.So consult a ER Surgeon.Don't Self-treat or medicate in your case as the case may go in to further complications.Hope this reply would resolve your query and would help you to plan treatment with your doctor.Hope this would relieve of your health concerns.Welcome for any further query in this regard before final suggestions are given in your query.Wishing you fast recovery..Have a good day.Dr.Savaskar M.N.Senior Surgical Specialist."
},
{
"id": 65382,
"tgt": "Suggest remedy for lumps in knee",
"src": "Patient: My cousin had knee replacement surgery 2 weeks ago. Tonight she discovered a lump behind her knee and she is worried it might be a blood clot. She does take warfarin and wears the stockings that are used to protect her from clots. There is no discoloration or warmth to the lump but she says it hurts like cramping. Doctor: Hi,It could very well be a blood clot. I suggest you see the Surgeon who has operated and let him examine your Cousin to see what is going on. Most likely all will be well and there usually can be blood clots after surgery which resolve spontaneously over a period of time and there should be nothing to worry about. But it would definitely be better to get it checked up.All the best,Take care,Dr Rishi, New Delhi, India."
},
{
"id": 35543,
"tgt": "Suggest diet for hepatitis e infected patient",
"src": "Patient: hi doctor i have been infected by hepatitis e virus and after 10days of hospitalization i returned home yesterday my doctor advised me to take 15 days rest since my biliruben has come down from 8.4 to 7.8 (total) so which juices are good ? and what i need to know when will i get completely healed i mean a month or more ? and which vegetables is good? can i have buttermilk or curd ? Doctor: Thanks for asking in healthcaremagic forum I understand your concern. The treatment for hepatitis E is only symptomatic. Just you need to avoid oily/spicy food completely for atleast 03 months. You can have high carbohydrate food. Some of the examples are a) Rice b) Butter milk c) fruits etc All the best."
},
{
"id": 139888,
"tgt": "Does head injury have a long-term impact on the brain?",
"src": "Patient: When I was 4 yrs old I was knocked in the head accidentally with a softball bat. The location is on the upper left hand part of my forehead, this left a small dent. I am now in my early 20 s and sometimes wonder could this have affected something in the front part of my brain? The dent doesn t seem very deep, I had to have 6 stitches to close the wound. Could this have affected me long-term in any small way? Doctor: Hello,If there was no brain injury ( this is presumed since you had no significant symptoms at the time ), it is very unlikely that this episode affected your brain function. The small dent is probably a partial compression of the bone ( at that age is very soft ). So, don't worry about any serious impact from that episode of head injury.Hope I have answered your query. Let me know if I can assist you further. Regards, Dr. Erion Spaho, Neurologist, Surgical"
},
{
"id": 158162,
"tgt": "Asked to stop Ginet while on treatment for lymphoma, suffer depression, low mood, abnormal vaginal bleeding. Will it go away ?",
"src": "Patient: hi there/ I have just started taking Ginet 84 again after about two years. my Gynecologist told me to stop taking it when i was being treated for non hyodgkind lymphoma. recently i have been suffering from deprestion and low mood especially when pre menstrual. i asked my doctor if i could go back on ginet 84 and now i am. i have had abnormal viginal bleeding for the last week i dont know weather to stop taking it or to wait and see if it goes away. i have been sick with the flu also...twice since i started it on the 30th of july Doctor: Hello, Thanks for the query to H.C.M. Forum. Ginet ,ingredients are cyproterone +estradiol and side effects of these two drugs mental depression. alopecia, hirsutism ( if use for prolong may develops endometrial carcinoma). Similarly cyproterone is not fit for you because it develops following side effects,galacorreha, benign nodules, alterations in hair patterns, fatigue , breathlessness, weight change ( gain or loss). More or less this (ginet ) also develops dependence as seems in your case. Now stop this drug and consult your doctor and get his opinion & treatment . Good luck. Dr. HET"
},
{
"id": 87130,
"tgt": "What causes right abdominal and back pain?",
"src": "Patient: hi my name is heather yesterday i started getting severe pains in the lower right section of my abdomine and also my lower right back it does not go to my side though. It is not my appendix I know that for a fact. please help me it really hurts. Doctor: Hi.Thanks for your query and proper history. The pain in the lower right abdomen and the power right back can be due to many reasons apart from appendicitis. And how it is possible for you to say that it is not appendicitis. Other reasons can be Salpingitis, ovarian problems, PID, colitis and lymph node mass, Urinary tract infection , right ureteric stone and so on. The confirmed diagnosis can be had only after an ultrasonography and / or CT scan of the abdomen. Get this done, get the proper diagnosis and get treated according to the findings ."
},
{
"id": 174518,
"tgt": "What causes diarrhea and body chill?",
"src": "Patient: My child normally feels hot but now has diarrhea (sp?) and actually feels cold to the touch although he isnt complaining of felling cold. He just is normally a little heater so it is weird to feel him being cold and I wondered why that might happen? Doctor: HI having cold peripheries with loose stools has to be taken seriously and needs to be supplement him with plenty of ORS fluids to prevent dehydration if he is tolerating feeds well.Please show to a child specialist immediately"
},
{
"id": 105504,
"tgt": "10 year old having reddish blotches on face, swollen eyes and stomach cramps. Due to allergy?",
"src": "Patient: My 10 y/o daughter started showing red blotches on her face and complaining of mild stomach cramps yesterday. Later in the day, the redness went away, but her face is puffy and skin around her eyes is very swollen. She says her stomach still feels crampy. It appears to be an allergy . I m not sure that she s eaten anything out of the ordinary; we don t believe so. She has been around a lot of outdoor, farming areas the past couple of days, including hay fields, a vineyard and commercial berry farms. Doctor: Hello, Occasionally hives (or urticaria) on the skin (if generalised) can cause abdominal pains due to histamine release in the gut. Obviously, food allergy will cause all the above symptoms you mentioned. Swelling around eyes of lips tends to stay longer than the itchy rash. A full allergy profile, mast cell tryptase level blood test is required."
},
{
"id": 48426,
"tgt": "Is it dangerous to have kidney having pole calculus with back pressure changes?",
"src": "Patient: Rt kidney reveals a 8mm mid pole calculus with gr II BACK PRESSURE CHANGES. Another 5.6 mm approx is seen in the right proximal ureter. Lt kidney also shows a 4.5 mm mid pole calculus, no back pressure changes seen. Is it dangerous. My age is 21 yrs. Doctor: Hello and welcome to HCM, Kidney stones with back pressure changes is a dangerous condition if not treated in time. If the back pressure continues, it can lead to atrophy of the renal parenchyma and thus deterioration of renal functions. Thus, you need to consult your surgeon or nephrologist for management. You also need to get renal functions done to assess the status of renal functions. Thanks and take care Dr Shailja P Wahal"
},
{
"id": 11400,
"tgt": "Could severe hair fall be due to excess protein shake intake?",
"src": "Patient: i have been taking protein shakes excessivly for 5 months now and just within the last 2 months i have noticed my hair has started falling out and is gettting quite thin, my protein shakes are very high in cholesterol could this be the cause? i am 24 Doctor: Hello,Welcome to healthcare magic.I understand from your query that you have been suffering from severe hair fall.Proteins are good for hair and protein shakes which you are consuming should not attribute to hair fall.However, sudden and severe hair fall could be because of excessive weight loss, crash dieting ( consuming mainly shakes and juices with not balanced diet), an illness which you had 6-8 months ago, surgery or a thyroid problem.Please get your hemoglobin, serum ferritin levels and thyroid status checked to rule out underlying problems.You could take a tablet containing 5 mg of biotin alone with cysteine and methionine for 2 months.Have a balanced diet. Hope this helps you.Take care."
},
{
"id": 136874,
"tgt": "Suggest medications for knee and groin pain",
"src": "Patient: my right thigh from my hip to my knee is in great pain the front part of mt thigh from the groin to the top of my knee is raw to the touch what can I take to ease I have been to the E.R 2 times and seen to doctors but none of the medication ease the pain it puts me to sleep for 2-3 hours can you help .Tom Clark Doctor: Hello, I have studied your case. Due to compression of nerve root there is pain in your thigh and knee.I will advise you to do MRI spineFor these symptoms analgesic and neurotropic medication can be started.Till time, avoid lifting weights, Sit with support to back. You can consult physiotherapist for help.Physiotherapy like ultrasound and interferential therapy will give quick relief.I will advise to check your vit B12 and vit D3 level.Hope this answers your query. If you have additional questions or follow up queries then please do not hesitate in writing to us. I will be happy to answer your queries. Wishing you good health.Take care"
},
{
"id": 11813,
"tgt": "White patches developed on cheek. Tested negative for fungus. Safe to use flutivate cream?",
"src": "Patient: Hi... Recently some small white patches developed in my right cheek. However they are not itchy and i tested for fungus under doctors supervision and no fungus was found either. He suggested me to apply flutivate skin cream of GlaxosmithKline.However Its been a week with no particular improvement. he asked me to visit after 2 weeks . should i continue with the cream?? PLS HELP... Doctor: Hi Sourav , Thanks for the query. The condition you are suffering from can be polymorphic light eruption,P.alba etc. As the fungal infection is ruled out you can safely apply Flutivate cream. It is safe steroid and you can use it for 15 days. Improvement is usually slow. Visit your dermatologist after 15 days. Hope this helps. Regards. Dr Sudarshan MD Dermatologist."
},
{
"id": 76452,
"tgt": "What causes tightness in chest with occasional dry cough?",
"src": "Patient: Hello..I am mildly asthmatic and take symbicort twice a day. I am male 5'9 and 11 stone in weight. For the last two weeks I have developed a tight chest sensation with occasional dry cough. Ventolin and/or symbicort do not make any difference. My peak flow reading consistently registers 610+ (as normal) when chest feels tight and this doesn't change after medication. My chest feels restricted, but I can exercise hard without any problem. The doctor has completed an ecg, blood pressure test and listened to my heart/chest - all is clear. I have a feeling that this could be anxiety, possibly a virus or an allergy to pollen. Please advice. N Doctor: Thanks for your question on Healthcare Magic. I can understand your concern. I don't think, your symptoms are due to worsening of asthma because your peak flow reading is consistently high. So no need to worry for viral infection or pollen allergy. So possibility of stress and anxiety related chest tightness is more. So better to consult psychiatrist and get done counselling sessions. Try to identify stressor in your life and start working on it's solution. You may need anxiolytic drugs too. Avoid stress and tension, be relax and calm. Don't worry, you will be alright. Hope I have solved your query. I will be happy to help you further. Wish you good health. Thanks."
},
{
"id": 136767,
"tgt": "Suggest treatment for a sharp stabbing pain in the hip bone",
"src": "Patient: I am a fifteen year old female. For about a week, I have been experiencing a sharp, kind of stabbing pain in my lower left abdomen (directly next to my hipbone) when I walk or do anything that involves moving my torso. I feel it when I extend my leg when taking a step and I end up walking differently to compensate for it. It becomes much more prominent immediately after I eat and if I don t eat for a few hours it s much less noticeable. The pain is very localized and in the exact same spot every time I feel it. I mentioned it to my dad when it started and I didn t talked about it to anyone again until today because I thought that it was going to go away. If anything, it feels worse now than before and I m scared to move or eat (I haven t stopped doing either); It s very uncomfortable and it feels very wrong. It sounds strange, but the best way for me to describe it is that it feels like one of my organs is out of place. I noticed a bit of swelling when I got home today, but there s none now and I don t know if it was real or if my nerves were getting to me (I did check a few times to confirm it, so it was probably legitimate). I only checked for it for the first time today, so I don t know if there has been any for in past week. I tried to ignore it, but I m scared now. I almost contacted my parents at work today so that I could ask them to take me to the hospital because I didn t know what to do. What do I do? What could it be? Doctor: Thank you for using HCM. It could be neurological or abdominal condition which needs clinical examination and if needed further tests like an ultrasound. If it's neurological likely to be sciatica but do you've radiating pain in lower limb? if it's abdominal could be an abscess for which fever would be present or is there a swelling in the groin which could be hernia. Please visit the hospital ER at the earliest so that it can be ascertained and treated."
},
{
"id": 48360,
"tgt": "Suggest remedy for severe pain due to kidney stones, gallstones and bilateral medullary nephrocalicinosis",
"src": "Patient: I do not have health insurance but in June I was hospitalized with an 8 cm kidney stone in the left kidney. They discovered I also have gall stones and bilateral medullary nephrocalicinosis. 2 days ago I started running a low grade fever and have not been able to get it down below 99.5. I am experiencing unbearable pain in my left side that goes all the way to my back. I can not seem to get enough to drink no matter how much water Ive had. It takes a while to urinate. Its to the point I feel almost dizzy just trying to stand. I ve used heating pads and baths and several over the counter pain relievers including Tylenol and naproxen. Doctor: Pain killers and fluids , nil by mouth since intake of food will increase the pain. followed by elective removal of the kidney stone"
},
{
"id": 86283,
"tgt": "Suggest treatment for severe pain and burning sensation in the abdomen",
"src": "Patient: I have been waken almost every night with so much pain and a burning sensation in my abdomen. I do not have acid reflux, but was told I have a hernia...I also have a lot of problems with my colon. I end up having to take a laxative. But I throw up several times when my abdomen feels like it is on fire, and I m usually sore afterwards in my abdomen. Help Doctor: Hi,Looking to the history as you were told having hernia, from symptoms it seems that you might be having strangulated hernia producing severe pain in abdomen, soreness and frequent vomiting.There is another possibility of having sub acute intestinal obstruction.You might require,Ultra sound of abdomen,X-ray abdomen.This will give exact cause of your problem.Consult surgeon as soon as possible.Ok and take care."
},
{
"id": 134863,
"tgt": "Suggest remedy for pain, numbness and tingling sensation in arm",
"src": "Patient: I have issues while doing excersise said in the gym such as back or front squats,bench press, and military press, which cause my arm to get s tingly numb sensation which will fade away after the workout. It gets very painful at some points that I lose stability in the arm. Could you please assist me? Doctor: Usually tingling and numbness in arm is because of nerve compression in neck. but in your case, that might not be the reason. Other cause can be compartment syndrome which is seen during exercises because of tight compartment. It needs clinical evaluation.It may also be some other pathology. Diagnosis is hard because you have not given your details, duration of problems. Plus clinical info is not available."
},
{
"id": 42432,
"tgt": "What are the chance of getting pregnant with weak motile sperms?",
"src": "Patient: Hi Doctor, I recently had my Semen Analysis done, the report showed only 8 weakly motile sperms(8 only not mills),after i started taking medicines clomefine citrate 25mg, multivitamins and other fertility drugs, my wife s HPT showed a faint positive and after few days my wife had a period. Our doctor suggested a very early miscarriage,what are the chances of i becoming a father whether naturally or through ART. Looking forward for your kind advice, Warm Regards, Vijay Doctor: Hi,Welcome to HealthcareMagic .I read your query and understand your concern . I would suggest you to go for IUI . It is intrauterine ingestion of sperms . Here sperms are washed and processed and healthy sperms are placed in uterus . This will increase chances of pregnancy .Here problem of reduced motility is solved to major extent . If iui fails you may go for IVF. Hope I have been helpful .RegardsDr.Deepika Patil"
},
{
"id": 207677,
"tgt": "Suggest treatment for hallucinations",
"src": "Patient: i keep hearing a voice inside my head, i dont know if its my voice or not because ive had it for so long. it calls me by my own name and thats why im not sure. for example if i was standing in a que and theres someone stood infont of me doing nothing wrong at all i get this voice and it tells me to hit the person or pick somethin up and hit em, even though they havent done anything wrong. i picture myself doin this aswell the voice like encourages me and i see myself doing it. i never get hurt back though. i just hurt them. and sometimes it makes me do the things i see, sometimes i cant stop myself. i get really paranoid aswell like i think somethings watching me and im always looking around me in everycorner and behind me to see if somethings there. Doctor: DearWe understand your concernsI went through your details. I suggest you not to worry much. If the symptoms you gave are correct and accurate, then you must be experiencing a mental disorder named parnoid schiophrenia. It is a complicated mental disorder and needs expert psychiatric treatment and proper management. Please visit a psychiatrist in this early stage itself and start the treatment. Please do not delay.If you need more of my services, please post a direct question to me in this website. I am happy to serve your purpose.Hope this answers your query. Available for further clarifications.Good luck."
},
{
"id": 3356,
"tgt": "What are the chances of getting pregnant with precum on penis?",
"src": "Patient: Hi, I have a situation me and my girlfriend kinda had sex twice. I say kinda because it was for short amounts of Time and I wasn t close to orgasming I am just worried about precum and the odds of her being pregnant it was 4 days after her period. Also is there anything she can take to help avoid it (preferably something you can buy when younger then 18) Doctor: Hallow Dear,Precum is pre-ejaculation secretion which is expelled out unknowingly. It does contain some sperms which can cause pregnancy. However, you had this experience 4 days after her period. This day is rather safe as far as the risk of pregnancy is considered. Egg is released 14 days prior to next expected menses. Egg has a life of 24 hours while sperms are active for 3 days. Hence a period of about a week around the day of egg release is considered as fertile window. 4 days after the menses falls quite away from fertile window. Still, I would say, better to err on safer side. Hence, she should take post coital emergency contraceptive pill protection. These pills have to be taken within 72 hours after the unprotected sex. If this period is already over, she may get intrauterine contraceptive device inserted within 5 days of this unprotected sex. This will give her protection from the pregnancy and moreover will give her ongoing contraception for years to come. If 5 days also are over, you cannot do anything now. Just wait for her periods to appear (which mostly she will get). If she misses her period, get pregnancy test done on her urine after one week; earlier the results may be false negative. Alternatively, you may get her blood checked for Beta hCG for diagnosing pregnancy, 10 days after this incidence. If she is pregnant, she can get the pregnancy terminated by medicines safely and effectively up to 9 weeks of gestation. However, this has to be done with Gynaecologists advice and prescription only. And lastly, any step she takes is to be done with her parents consent since she is minor; otherwise it will be an illegal offence. I hope this provides you detailed guidelines.Dr. Nishikant Shrotri"
},
{
"id": 214863,
"tgt": "Had chalazion, after sty on eye. Home remedies?",
"src": "Patient: I've had a Chalazion for about three weeks. It resulted after a sty formed on my left eye. It's not very large (smaller than a pea) and I just started adding heat to it. I'm starting a new job On Monday, and being that I can't take off right away I'm hoping it can be treated without surgery. Are there any home remedies that could help me? Doctor: Hello & thanks for your Q on HCM I recommend applying a warm compress to the cyst up to four times a day until the swelling reduces and finally disappears. Hold the warm compress on the affected area for 10 to 15 minutes at a time to soften the blockage and speed up the draining process. Always wash your hands well before applying the compress and use a fresh clean cloth. Never heat the water for a compress in a microwave as it can become too hot and burn the delicate skin of the eyelids. Instead of tap water, you could use normal saline solution. ( salt water). Other home remedies for eye cysts are Use of guava leaves as a compress. Warm these leaves and place them in a clean dry cloth. Place this cloth over the swelling and leave it on for a while to reduce pain and redness. Boil some acacia leaves in water. Remove the leaves and use this water to soak a towel or clean cloth. Place the warm compress over the eyelids to remove in order to get rid of the chalazion cyst naturally. Apply a mixture of rosewater and honey directly on the eye which has a soothing effect and helps the eye heal at a faster rate. Tea bag eye treatments are effective in treating an eye cyst- the used teabag of chamomile tea should be chilled for a while and then applied to the affected eye for 5 to 10 minutes or so. Hope that it helpful & all the best for your new job"
},
{
"id": 116761,
"tgt": "What causes the blood blockage in the leg?",
"src": "Patient: hi doctor my mother taken 6mg acitrom my mother left leg has blood bolcked doctor check it PTR & INR to take the tablet acitrom 6mg and take to belt.my mother taken belt .she is use daily.side effect is heart pain she told heart pain and sleeping not well any solution doctor pls tell me my e-mail id YYYY@YYYY Doctor: Hi,Thanks for asking.Based on your query, my opinion is as follows.1. Endothelial damage, increased coagulating factors or change in blood movement within the vessel commonly leads onto thrombosis. This thrombus blocks the blood vessel.2. Due to reduced activity of your mother, blood movement would have slowed down resulting in formation of thrombus.3. INR needs to evaluated to maintain at a higher level of 2-2.5 during anti-coagulation therapy. If she has any breathlessness, meet your doctor immediately.Hope it helps.Any further queries, happy to help again."
},
{
"id": 98037,
"tgt": "Diagnosed with bilateral varicocele. Can it be cured with homeopathy?",
"src": "Patient: I am 32 yrs old.my report says there is evidence of bilateral dilated, tortuous, serpigenous veins of the pampiniform plexus on either side which gain prominence in the errect position and post valsalva suggestive of bilateral varicoele with dilatation upto 5.5 mm suggestive of grade 3 . Impression : bilateral varicoele ( grade 3). Can it be cured with homeopathy? Doctor: varicocele is a structural change in the veins of the testes, where they get elongated and tortuous. Hence, while homeopathic medicines can help to reduce the problem. Homeopathy can help depends on the cause and degree of the problem. In some cases surgery may be necessary. Better you should visit a Homeopathic expert for constitutional treatment soon. All that you need to do is to wear a supporter or T-bandage to support your scrotum. Wearing loose cotton trousers will prevent overheating of your testes. Activities like swimming will help to keep your scrotal temperature on the lower side. Avoid bathing with hot water or using steam-baths. it help you to reduce the chance of infertility. \"Varicocele is usually harmless except in cases of infertility\". Hope you recover soon. Dr Rajesh Sarkar. rs787878@gmail.com"
},
{
"id": 204614,
"tgt": "Suggest treatment for alcohol addiction and depression",
"src": "Patient: I am an alcoholic, I work M-F and then stay home in bed all weekend, drinking rum and beer. My 20 year old daughter passed away on May 21,2001 from a stroke caused by a gunshot wound to the neck, bruising her carotid artery. Am not suicidal, and have happy and wonderful memories of my girl, but after 15 years Doctor: Hello,Most important thing is persistent motivation and detoxification. There are various detoxification treatment available with high success rate. Next Steps- You can consult neuropsychiatrist.- Health TipsRemember craving is hard for 30 minutes and if you can overcome that 30 minutes by drinking water or diverting your mind in some other work I think you will be able to control craving. Hope I have answered your query. Let me know if I can assist you further.Regards,Dr. Rohit Kothari"
},
{
"id": 30686,
"tgt": "Suggest treatment for pseudomonas infection and candida",
"src": "Patient: Hi,I got pseudomonas infecction and candida,and after many month to homeopatic treatment and finally using antibiotic(from antibiotic resisten test)I use CLAVAT for a week under my doctor prescripcion but still my test is positive..Please can I get a guide about it??thanks Doctor: Thanks for posting your query to HCM.First I wanted to know from which specimen you have been diagnosed such infection and what are your clinical complaint .Second Clavat is not the drug of choice to treat such infection . Piperacilline-tazobactum is used to treat Pseudomonas ans Azole antifungal like Fluconazole for candida .But the thing is wheather these organism are really causing infection or not as these are common contaminant of clnical specimen .review me after required information .hope this information will be sufficient for your query .rate me if you are satisfied with my answer .feel free to ask."
},
{
"id": 186585,
"tgt": "What causes nausea and dizziness after tooth extraction?",
"src": "Patient: Hi, I had my two upper wisdom teeth extracted 2 days ago and I am experiencing waves of nausea and dizziness. I am off the pain killers right now although the dentist recommended I take them for 8 more days. One side was easy to extract the other side, however was harder and they removed part of a bone and sutured it. I feel much better with the medication but the pain isn't unbearable without it. I constantly feel like i'm going to be sick and I am having night sweats. Is this normal? What can I do? Doctor: Hi,Thanks for posting the query, I would like to tell you that after extraction the healing process starts and there is increase salivation in the mouth leading to nausea and diziness.I would suggest you to take lukewarm saline and antiseptic mouthwash rinses.Take the complete course of antibiotics and analgesics prescibed to yoy also take an antacid twice half hour before meals.Take multivitmain suplements.Maintain a good oral hygiene.Take care!Dr Ammara."
},
{
"id": 23373,
"tgt": "What causes vomitting after stress test?",
"src": "Patient: I recently had an abnormal stress test and another test that showed damaged to the two bottom chambers of my heart. I have noticed that I have been throwing up frequently and feeling very sick lately. What could be the issue? I am now seeing a cardiologist for the heart proboem. Doctor: hia stress test is highly sensitive for cardiac ailmentsif it is abnormal you need a thorough investigation of your heart damaged bottom chambers can mean that there is some motion abnormalitities in that chambera detailed analysis of your printed report is very essential before making any commentbut throwing up and feeling maliase can be because of cardiac conditions like coronary artery disease and congestive cardiac failurea detailed report is to be analysed as it is a matter of high concernthank you"
},
{
"id": 224687,
"tgt": "Could having unprotected sex followed by intake of emergency pill within 72 hours prevent pregnancy?",
"src": "Patient: pregnancy helloi had unprotected sex on wednessday bt did not take an emergency contraceptive pill because da due date fr my periods was saturday(today) but den when i did not get my periods today i got scared and took an ipill.. i took it within 72 hours though. is der any chance that i maybe pregnant? Doctor: HIThank for asking to HCMI really appreciate your concern look no method is hundred per cent give you protection against the pregnancy, even after taking the pill within 72 hours chances would be there for pregnancy, because this agent disturb the hormone but may not prevent the fertilization of ovum for sure, hope this information helps you, take care and have nice day."
},
{
"id": 107144,
"tgt": "How can chronic backache be treated?",
"src": "Patient: I have chronic back pain. aherniated disc(c3-l4 & l5) and facent joint lumbar pain tha facet pain is going to be burnt off on 8-29 9 A TRIAL OF 10HRS WAS DONE A MONTH AGO ) BUT NO ONE IS DOING ANYTHING ABOUT THE DISC PAIN. i know IT IS A HERNIATED DISC FROM READ THE RESULTS ON MY PORTAL & i HAD L4 REMOVED 20 YRS AGO. THE SYMPTOMS ARE THE THE SAME AS 20 YRS AGO.i HAVE A PRIMARY DOCTOR APPOINTMENT TODAY AND NEED TO KNOW WHAT TO SAY/DO TO GET THIS DISC ISSUE SOLVED. THEY ARE DOPING ME UP FOR THE PAIN BUT IT JUST MAKES ME TIRED,PAIN IS STILL THERE. PLEASE HELP! Doctor: Hi, I had gone through your question and understand your concernsWith such presentations in my clinic, I would first reassure my patient telling him that the first line in treating disc herniation problems is conservative treatment then to wait for improvement of back pain and radiculopathy if present .If there's no improvement of the back pain and progressive increase of radicular symptoms in spite of treatment so we proceed to surgery.conservative treatment for your problem include the following:1- Physical rest :Avoid heavy activities,prolonged standing,bending 2-Wear lumbosacral belt 3-Receive Muscle relaxant once daily ,NSAID when needed and pregabalin 150 mg once daily before sleep."
},
{
"id": 34153,
"tgt": "What causes bump on tongue?",
"src": "Patient: Hi - I felt with my tongue that on the inside of the mouth near the lower corners of the lower part of my mouth there are a small bump on each side. Problem is on the left side the bump seems to be a little enlarged even though there is no pain. Could i have been softly biting that bump with my teeth while asleep? Doctor: Hello and thank you for your question.Bumps in the mouth can be from infection, trauma or cancer. If these bumps are enlarging then see a doctor or dentist, especially if you use any form of tobacco. If you think it is just trauma from biting it in your sleep then it should go away in a week or so. If not see a doctor or dentist.Regards"
},
{
"id": 81479,
"tgt": "Suggest treatment for COPD and asthma",
"src": "Patient: I caught a cold or allergy the other day now it is in my chest. I cannot cough anything very well. I have COPD and asthma. I have taken a breathing treatment three times today, as well my regular 2 times per day inhaler. I am still having a hard time breathing . My pulse is 122. Should I give it a couple of days to see if it passes, or go ahead and go in now? Doctor: Thanks for your question on HCM.By your history and description, I think you are having COPD overlap syndrome.I advice you to consult pulmonologist and get done1. Chest x ray2. PFT (pulmonary function test).Uncontrolled disease can be due to untreated or recurrent infection. So chest x ray is needed to rule out infection.PFT is needed to know the severity of the disease. And treatment is based on severity only.Along with inhalers I think you should follow these things too for better symptomatic relief.1. Enroll in pulmonary rehabilitation center where chest physiotherapy and deep breathing exercise are done.2. Get done adult pulmonary vaccination with pneumpcoccal and influenza to prevent future infection.So consult pulmonologist and discuss all these"
},
{
"id": 152229,
"tgt": "Cold numb feeling on the right side of my head",
"src": "Patient: i got a cold numb feeling on the right side of my head it washed over me and went away again what was that a numb cold feeling washed across the right hand side of my head and went away again what was that??? Doctor: that could possibly due to due to some temporary neurological event.It can occur due to stress or some neurological deft.You can wait for sometime may 1-2 weeks and if such an event is repeated within that period than you need to consult your neurophysician for further management.Otherwise you may consult your nearest homeoapath, thanks jignesh"
},
{
"id": 49793,
"tgt": "Dizziness, swelling in legs, high potassium and creatin levels, ice cold and numbb hand and legs. On treatment for bladder infections. Kidney problem?",
"src": "Patient: I would like to know about kidney disease. My doctor has been treating me for bladder infections this past year. He sent me to a bladder specialist who sent me back to my primary care doctor. Now I am waiting to talk to a recommended specialist on the kidneys. The one he recommended cannot see me till the end of Oct.. My primary wants me to be seen within this week. I am dizzy, one leg swells up more than the other. I have high potassium and my creatin levels have been high. So has my B D. Many times it is high or a good number on top but the lower number has been in the 40's, 50's. Last night my hand was ice col and numb. My legs have also been numb up to my calfs. I didn't want to go back to bed because it scared me. I don't know whether to alarm my primary and call him or not. Doctor: 1. your symptoms of high potassium-B.P-hyperkalemia, creatine, points towards CRF but numbness and edema in one leg: one has to rule out DVT by color Doppler.2. avoid high protein diet.3. take carbohydrates and fat as usual.4. avoid potassium ( banana, papaya, cheese,tomato, potato, grapes, coconut water, fruit juices)5. take calcium"
},
{
"id": 31682,
"tgt": "What are risks of getting measles infection after having MMR vaccine?",
"src": "Patient: Hi I am 7 weeks pregnant and my husband has been taken into hospitalo with measles and fluid on the lungs i had my MMR vaccine when i was a child, what is the risk to me and my un born baby. (i also have a 1 year old baby and a 4 year old should i get them checked out?) Doctor: Hi As you are vaccinated no need to worry. Mmr vaccine give lifelong protection. As you wont get infection your baby will be safe. If your childs are vaccinated no need to bother."
},
{
"id": 131892,
"tgt": "Why the entire left arm from shoulder to fingers is swollen with icing no help ?",
"src": "Patient: A couple weeks ago i came home from work only to realize that my entire left arm from shoulder to fingers was swollen i didn t think too much if just elevated it and put an ice pack on it a few times but the swelling has yet to go away and it is two weeks later i think some sort of bug bit or stung me and i know i got a bee sting on the same arm a couple days later which i think made it all worse. I have a really odd looking bruise on my bicep area that is normal looking in one spot and in another has small hairlike or vein bruises, this is not that close to either bite. I went to a local care spot and they put me on steroids to try and reduce swelling for five days and then an antibiotic when i had taken all of that and the swelling still hadn t gone down my last day for the antibiotic is tomorrow and i was supposed to see a big difference by the third dose and i haven t seen that much of a difference. The arm gets very tired easily and rather stiff at times and it was so swollen and numb that i didn t even feel the bee sting me. What should i do? Thanks Doctor: HiSting takes 7 to 10 days to heal under cover of antibiotics, anti inflammatory and anti allergic meds. ,Local dressing with antiseptic oint ment..Do shoulder, elbow, wrist exercises, keep arms in a sling It will settle in few daysThanksBest wishes"
},
{
"id": 133846,
"tgt": "What causes soreness and swelling in abdomen with fracture?",
"src": "Patient: Five days ago I fell and hurt my side. I went to an urgent care two days ago and had a chest and left ribs xrays done. I was told that i had two nondisplaced fractures in my 9th and 10th rib, no phemothorax, etc. I was told to follow up with an orthopedic. I noticed some soreness and swelling in my abdomen. Is this normal? Doctor: hi,thank you for providing the brief history of you.A thorough musculoskeletal assessment is advised.Also due to the fracute of the 9th and 10th ribs there might be swelling and pain in that area. Also the swelling in the abdomen is not connected to the ribs injury until the quadrant of the abdomen is advised. If the swelling is in the same side quadrant where the injury has occurred than swelling is possible. By performing slow and stable breathing exercises the healing of the fractured ribs will occur and also the Strengthening of the access muscles of respiration involved.Since the ribs are non displaced fracture, while performing the breathing exercises you should keep the hand over the injured area to avoid direct strain.Within 4-6 weeks of time, the healing occurs depending upon the metabolism of the body and normalcy can be expected soon.RegardsJay Indravadan Patel"
},
{
"id": 190747,
"tgt": "TMJ erruption, talking aleve, swollen neck. Should i be concerned about carotid artery?",
"src": "Patient: I seem to have a TMJ erruption, since, Wednesday. I ve been talking Aleve two twice aday It seems a little better, but ,I notice a swelling on the left side of my up neck like a bulge right below the ear area. Is that the coratid artery do I need to be concerned right now? Can I wait to see a dentist or Dr. Monday? It s making me very anxious Thx for your help I just took two Aleve.Myabe, I need to put warm compress. Do you think it s something with the heart? I m 65 years old. overweight. Doctor: hello.. first of all dont be anxious at all, its a minor problem.. the swelling of neck may be your inflammed muscle.. As u had taken aleve ur inflammation is reduced.. as per ur description i think its disease of tmj related muscles and is called MPDS.. your clinical and radiographic investigations can properly diagnose the condition. pain in tmj region is commonest by MPDS .. causes include excessive occlusal load on tmj.. Removal of excessive load on tmj, Soft diet, Muscle relaxation by muscle relaxants, NSAIDs gel/ tablet, warm fomentation will resolve the tenderness and swelling in region..Carotid artery is rarely involved in TMJ diseases.. so dont worry.."
},
{
"id": 207397,
"tgt": "What is the treatment for stress and depression?",
"src": "Patient: right now i m just fed of my life i don t know what to do... i just wanna achieve everything in life but without taking any risk....these days i m little bit disturbed..... i want to be an A+ grade student but nable to do that.......i even love a girl but never ever had the courage to walk up to her and ask her......please doc please do help me i m in great need of your help......!!!! Doctor: Hello dear. Your problem is very common of teenage. Dont worry go for some counselling sessions to builtup confidence. Dont be upset try to live stressfree life. Grade doesnt matter to become successful in life. Never loose hope. I never got A grade in my school. But still i am performing better than my A grade collegues. Go front of your girl and say whatever you want to say. Never keep in mind as when time goes it never comes back. You can take help of some antidepressants like escitalopram. So dont worry. Hope my suggestions helpful and thankful to you. Take care. And sont forget to give five stars."
},
{
"id": 173520,
"tgt": "Need medication for viral infection",
"src": "Patient: Hi - 8 year old son has viral chest infection and has been coughing extensively for past 3 days. Last night he added a horrible \"retching\" to this. It's something we've never heard before - seems to come on all of a sudden with this very loud retch. Should this be of concern? Doctor: Hi,Thanks and welcome to healthcare magic.Extensive cough with retching is really a matter of concern.This may be allergic cough,whooping cough and even asthma.Benedryl cough syrup may give relief.Severe cough is controlled by bronchodilators like salbutamol, terbutaline or steroids.Better consult pediatrician for thorough investigation and treatment.Hope this answer serves your purpose.Please feel free to ask further queries if any.Dr.M.V.Subrahmanyam."
},
{
"id": 64101,
"tgt": "Does a recurring lump in the breast need medical attention?",
"src": "Patient: I have a lump on my left breast in the same spot where I previously had a lump removed (benign). I am currently pre-menstrual with tender breasts. Do I need to see the doctor? Or should I wait until after I finish my period to see if the lump is still there? Doctor: HI,Dear,Thanks for the query to HCM.I understood your concerns about the shin lump.In my opinion its -benign lump still?fibroadenoma? fibroadenosiTreatment-a-Take opinion from your ER Surgeon if it remains and grows in size in next 2 mthsb-I would advise Vit E- till then.Hope this would resolve your worry.Hope this would help you to plan the treatment with Your family doctor.Wishing you early recovery.You are always Wellocome for further query to HCM in this regard.Have a Good Day...!!"
},
{
"id": 161342,
"tgt": "When should I be concerned about my child s head injury?",
"src": "Patient: My 8 year old daughter fell from the top bunk into her brother and hit her head on his leaving an indention in her forehead above her left eye She does not show any signs of sleeplessness or vomiting. I put ice to the site Do I need to take her to the Dr? Doctor: Hello, If a similar patient comes to my OPD, I would first inquire about a history of loss of consciousness and vomiting. as you tell she was fine, I hope there is nothing to worry. Just make sure that her daily activities is normal and she is playful. Hope I have answered your query. Let me know if I can assist you further. Take care Regards, Dr Jilu Joseph, General & Family Physician"
},
{
"id": 113941,
"tgt": "Is there any reason for lower backache,abnormal urination and shrinking of penis ?",
"src": "Patient: STIFF LOW BACK PAIN EVERY MORNING SPECIFICALLY WHEN I TAKE Chicken, Fish, or Heavy Food in Dinner, but The pain surprisingly subsides STIFF LOW BACK PAIN EVERY MORNING SPECIFICALLY WHEN I TAKE Chicken, Fish, or Heavy Food in Dinner, but The pain surprisingly subsides/decreases in minutes once I start my exercise workout includes Jogging, Body Building, Muscle Fitness Training,. etc., ABNORMAL URINATION since 2yrs urology reports normal PLEASE HELP? ABNORMAL URINATION since 2yrs urology reports normal PLEASE HELP? Suffering with ABNORMAL URINATION since last 2yrs, Consulted Urologist undergone Tests all reports normal. MY PROBLEM It starts immediately after urination there is a strong urge and uncomfortable condition unable to sit or sleep at least for next 3-4 hrs and strong feeling and urge to urinate more after. To relax I apply strong pressure only by FORCE just few drops maybe up to 3ml to 5ml comes out twice in 3-4 hrs resulting into ease, into a RELAXING state of COMFORT. MYSELF I am a 26 yr old healthy Weight Trainer-COACH into Physical Training, I consulted urologist and undergone extensive tests all are normal and also underwent medications but no use. IMPORTANT Often twice a week my PENIS shrinks right in the middle visible like shrunk, pressed, squeezed right in the middle, both other ends are normal and thicker, THIS HAPPENS twice in a week irregularly.Dear Doctors and if anyone who experienced or know about this via Y-Answers-PLEASE ADVISE and HELP ME, May God Bless you all who wish to HELP ME. Doctor: Hello \u00a0\u00a0\u00a0\u00a0\u00a0it appear you have some infection of trigon area in the urinary bladder.in this last part of urination is painful & infected. so you need culture of urine in three part , first sample ,middle sample & last sample separately.spasm of back muscles can occur due to many reasons ,all are not due to organic cause ,even simple bad posture ,some dietary habits also predispose to morning stiffness.Get frequent investigation to find the reason \u2018Hope I have answered your query, I will be available to answer your follow up queries, \u201cWish you Good Health and trouble free speedy recovery\u201d"
},
{
"id": 34866,
"tgt": "What causes the persistent symptoms of amoebiasis?",
"src": "Patient: Thanks. I live in mexico. Have off and on amoebiasis. I don\u00b4t use metronidazole ( I drink beer daily). I use Quinfamida. 300mg 1 day.My question is can I repete this treatment after , say, a week is symptoms persist? Or, are there any other substances. ( yes, I also use Diodoquin when symptoms appear, tho 20 day useage produces heavy anal itch!)Thanks, and good luck and Good Workbo Doctor: Hello.. welcome to HCM.My first advice for you is strict maintenance of personal hygiene & drink safe water. As per your profile it appears that you may be a chronic carrier.Also it would be great if you stop alcohol for a week and take metronidazole or tinidazole followed by Diloxanide furoate for destruction of cyst in gut.Also consult your doctor and see whether you require any radiological investigation for extraintestinal lesion and Amoeboma in gut.I hope i have answered your concern. Ask for further help if need in future.Wish you a good health.Regards.Dr jigar."
},
{
"id": 209655,
"tgt": "Please suggest a cure for smoking and depression",
"src": "Patient: Hi, I cannot find a Phyc doc, seems either they do not take my insurance or they are not taking new patiences. So my regular doctor is helping me by prescribing me medicines. The problem is I was on prozax not doing much good. So he wants me to quit smoking to we switched the prozax to welbutrin. As to help me quit smoking and something for my depression, it is day 7. Already been back to smoking. And these meds make me feeling horrible. What should I tell my doctor? Doctor: HiThanks for using healthcare magicYou can take wellbutrin. Better to start from the basic minimum dose and increase gradually. That would help to control side effect due to drug. Wellbutrin contains bupropion and that helps to maintain dopamine level, through which, it help to control nicotine craving and withdrawal. You can also try nicotine chewing gums or patch with it. In case, you need further help, you can ask.Thanks"
},
{
"id": 153938,
"tgt": "Does bone cancer cause death?",
"src": "Patient: My husband has bone cancer, PSA has started rising. I would like to have an idea of what to expect from here. What about this type of cancer actually causes death. He is 70 yrs old, very overweight, treated for high blood pressure, high colesterol, Diabetis, has polycistic kidneys, depression Doctor: Hi,Thanks for writing in.It is important to know what type of bonce cancer your husband is having. There are many bone cancers in appearance and types. Some are locally aggressive while others can be highly metastatic and destructive.If bone cancer is detected in stage 1 and 2 then patient survival chance is more. In stage 3 and 4 bone cancer the survival chances are less however some patients might survive with aggressive treatment.Your husband has some co morbid factors like overweight, treated for high blood pressure, high cholesterol, diabetes, has polycystic kidneys, depression and that might play a role in recovery. Please discuss with your doctors concerning the stage and treatment. PSA has no isolated predictive value is bone cancer but if more than 10 ng/ dl then his prostate must also be evaluated and treated for any condition. Please do not worry."
},
{
"id": 112337,
"tgt": "Severe lower back pain for months. Ultrasound shows swollen ureter. History of kidney infections",
"src": "Patient: Hi ....I have been having severe lower back pain for 5 months now, ITS bad when I lie down and try to sleep, been to clinic few times, always say ITS nothing serious, common to have back pain, ITS not when You cant sleep are not getting better and lasts for this Long!! Recently an ultrasound showed that my ureter was swollen!? So IM now booked for a CT scan. I should also mention that I had 2 Kidney infections 4-6 months ago.... Any thoughts? im so tired and in pain and I have a 7 month old.... Doctor: Hi,Thanks for writing in.The swollen ureter might be due to earlier infections or a new infection. The best paramenter to study your kidney function is serum creatine and blood urea. If these two are in normal limits then there is not much to worry. The CT scan will give a better image than ultrasound and tell if there is an obstruction in the course of the ureter.Hope this helps"
},
{
"id": 145715,
"tgt": "What does my brain MRI report indicate?",
"src": "Patient: Hello, I am a 29 year old female who had a brain MRI without contrast done. The results just arrived in the mail and I m not clear about them. Indication: Common migraine Technique: Multiplanar multi-sequential imaging performed through the brain parenchyma, without contrast. Findings: No evidence of acute intrcranial hemorrage, mass effect, midline shaft, or acute/subacute infarcation. The sulci and ventricles are normal in size for age. There are no extra-axail collections. There is a cavum interpostium noted which impresses upon the internal cerebral veins. The flow voids at the skull base are preserved. The orbits are normal. Mild mucoperiosteal thickening within the ethmoids and along the floor of the left maxillary sinus. Impression: 1. No evidence of acute/subacute infracation. 2. Note is made of a cavum velum interpostium, an anatomic variation. Would someone please help me and explain these results in a language that I can understand before worrying myself? Doctor: HelloMRI findings suggests almost normal findings except changes of mild chronic sinusitis in ethmoid and left maxillay sinus(sinuses are air containing spaces in facial bones).Chronic infection is indicated by mucosal thickening of sinuses.You may need treatment for sinusitis.Other wise findings are normal.Brain parenchyma appears normal.There is no e/o infarction,intracranial hemorrhage,collection etc.There is a cavum interpostium,which may be a normal anatomical variation and this findings usually doesn't require any treatment.Hope I have answered your query.RegardsDr.Indu Bhushan"
},
{
"id": 222841,
"tgt": "Suggest treatment for vomiting during the pregnancy",
"src": "Patient: My Wife is pregnent of 6 weeks, she is facing lot of vomiting and sleepness, doctorsuggested to take Doxinate in the evening , Perinorm on in morning and one in evening and vominous in te evening 2. Can we take all tablets? or it is affecting the pregnancy Doctor: Yes, you can safely take these tablets. She should avoid drinking water in empty stomach and also spicy foods. If still vomiting persists, she may need certain tests to determine whether there is any abnormal cause of vomiting."
},
{
"id": 111725,
"tgt": "What are the reasons for back pain, muscle pain, chest pain, bloating and difficulty in breathing?",
"src": "Patient: i got always pain my back, muscle somtimes, chest and bloating all the time, also im hardly to breath properly and my stomach always pain and my head especially my forehead. my nose seems block and sometimes open to much. im cold sometime my mout and lips always very dry. Doctor: Hi, Welcome to Health care magic forum. It appears that you have three problems.one is back pain, second is the gastric pain and bloating. third is the nose block and head ache. The nose block and the head ache may be due to the sinusitis, causing indigestion, bloating due to the drugs, or some other reason. Back ache may be due to infection,of the sinus or some other. I advise you to consult a physician for diagnosis and treatment. Wishing for aquick and complete recovery. Thank you."
},
{
"id": 9083,
"tgt": "How much is the cost of laser operation to remove the hair of face ?",
"src": "Patient: how much is the cost of laser operation to remove the hair of face only.? Doctor: hi it is procedure and not operation. it is done every 1 to 2 month average 7-10 session require full face laser cost from 2000 to 5000 per session depending on density of hair"
},
{
"id": 60744,
"tgt": "What causes a weird red bump on the side of the head?",
"src": "Patient: Hi there I have a wierd bump on the side of my head it has gotten bigger over the years I have not gotten it checked out but now I'm kind of worried as like I said it has gotten bigger over the years and more red and I know it's not a pimple or cyst. I do have a picture if you are able to reply to me e-mail I can send it and maybe you can give me some insight thank you my name is lindsay. Doctor: Hello, Chronic red lump on side of the head can be dermoid lesion or else as per the narration. However, if you can provide a photo picture of the same in follow up conversation, it would be of immense help to comment further in this direction. Hope I have answered your query. Let me know if I can assist you further. Regards, Dr. Bhagyesh V. Patel, General Surgeon"
},
{
"id": 23390,
"tgt": "What causes elevated CPK levels in a type II diabetes?",
"src": "Patient: I have elevated cpk levels as determined by a blood test. My doctor took me of liptor as a result. I have congestive heart failure and type 2 diabetes. Recently, after performing one test shortly after exercising my Rheum Doctor seem not to be concern with cpk coming from the brain or heart. Is there a way to lower cpk level? I weigh 400+ pound so perdiosine (?) is not an option. Doctor: THANK YOU AND PLEASED TO ANSWER YOUcreatine phosphokinase elevations can be found without any health complaints. CPK elevated levels enzyme come from the muscles active work. muscles release CPK that tends to increase with heavy exercise. To make the things clear front of asymptomatic CPK increase, doctor check it again after 1 or 2 weeks exercise rest.Some drug has as side effects a rhabdomyolysis (\"rhabdo\" and \"myo\" for \"rod-shaped muscle\"; \"lysis\" for \"breakdown\") like statins. In obese patients, CPK value is not correlated to obesity checked by BODY MASS INDEX (BMI).Prednisone a corticosteroids drug, is not well for you as you are very obese and you might gain weight with it, and you are suffering from heart diseases, then you have to lose weight. Corticosteroids increase blood glucose, then you are asked to not use corticosteroids. Hypothyroidism often causes weakness, cramps, myalgia, and a mild to moderate serum CPK elevation.Serum CK levels are strongly affected by race, sex, and physical activity.A patient with truly elevated levels should be evaluated for a variety of nonneuromuscular causes, including endocrine disorders, metabolic disturbances, drug effects, and malignancy.If you are diagnosed Asymptomatic, no Treatment Exists and Course is Often BenignMAY THIS OPINION HELP YOUbest wishes"
},
{
"id": 96714,
"tgt": "How should headache after a car crash be treated?",
"src": "Patient: i was in a car crash 3 days ago. nothing was life threatening so i did not have to be transferred to A & E. but i was checked by the local doctor to check my injuries and she only pescribed me with pain killers. since then i have had a on going headache and feel weak. what should i do? Doctor: Thank You for contacting HCM.Pain after head injury can be simple concussion or could lead to sever bleed. It takes around few days to recover fully. I would suggest following:> Take rest.> Avoid sternous activity.> For headache acetaminophen can be taken.> Take a cloth piece and heat over flame so that it becomes warm, then place it over the eye. It will give soothing effect. Black eye is due to blood accumulation under skin.> Avoid drink till she becomes healthy.> Take healthy food including fruits & vegetablesReport to hospital (preferably to neurosurgeon as he is specialized in head trauma cases) if:> Condition remain same after 4-5 days> Any unusual symptom appears> Condition deteriorates.> You loose conscious or has altered state & talk irrelevant things.> There is visual problem.Hope this answers your question. If you have additional questions or follow up questions then please do not hesitate in writing to us.Wishing you good health."
},
{
"id": 94888,
"tgt": "Lower abdominal pain, aggravates during urination and bowel movements, urine test showed pus cells. What could it be?",
"src": "Patient: Hello I had a diagnostic laparoscopy 24 days ago which found no reason for my heavy and painful periods.They also fitted a mirena coil. Since then I have been having lower abdominal/ pelvic pain which increases when I got to the toilet (both urine and bowel) Yesterday I had my urine tested and it showed pus cells so was given three day course of antibiotics and urine was sent for analysis Today I have extreme abdominal pain which was so severe this morning I contemplated calling an ambulance. It did subside but is still there periodically especially when I move or try to open my bowels what could this be? Doctor: Hello. Welcome to hcm. Pain after mirena is most commonly due to perforation and you need to immedietly visit ER and undergo ultrasound scan or laproscopy to confirm and/remove mirena. Delayed will be life threatening. Other reasons might be due to infection,pre existing uterine pathology but emergency causes should be ruled out first and treated. All the best"
},
{
"id": 145820,
"tgt": "What causes pain in shoulder followed by nerve pain in wrist?",
"src": "Patient: Hi! On Thursday I was in a very minor car accident and was totally fine, but a day later my shoulder had a dull pain and felt weak, that has gone away for the most part, but now I feel like something is wrong in my wrist. It feels like nerve pain when you run your fingers over it. Also I have been feeling a some pain, almost like a pulled muscle or headache at the base of my neck. All of these are on the same side. I was wondering if they are related, what could be wrong, and if I need to see a doctor. Doctor: Hi,Thanks for writing in.Sorry to hear about your car accident and hope you are not having much pain and discomfort. Due to sudden movements you might have hurt your neck. This is where the nerves supplying hand and wrist originate. It is usual for a person not to start having more pain a day after a minor traumatic incident. The muscles go into spasm maximum a day after the injury. This is due to release of small amounts of neural transmitters which cause the pain. You are having the pain more on the second day for this reason. This should improve over a week after applying ointments containing pain relievers and muscle relaxants. A X ray of cervical spine might help to confirm that your spine is normal and not a concern."
},
{
"id": 148594,
"tgt": "Have spinal stenosis, noticed rash on lumbar spine area of skin, bumps. How to treat it?",
"src": "Patient: I have Spinal Stenosis.......lately I have noticed a skin rash on the lumbar spine area of my skin......This started happening a couple of months ago........an itch arises and when scratched it s stings and burns......the rash is even with the skin, small bumps .....only appears in the lumbar area....... Doctor: Hi,Thank you for posting your query.First of all, I would like to reassure you that the itchy rash and skin lesions in the lumbar region are not related to the lumbar canal stenosis. They are two separate problems.The skin rash needs to be evaluated by a dermatologist.The most likely cause could be urticaria, and may be related to an allergy. Use of antihistaminic agent such as levocertizine or cetrizine tablets would be helpful.I hope my answer helps. Please get back if you require any additional information.Wishing you good health,Dr Sudhir Kumar MD (Internal Medicine), DM (Neurology)Senior Consultant NeurologistApollo Hospitals, Hyderabad, IndiaClick on this link to ask me a DIRECT QUERY: http://bit.ly/Dr-Sudhir-kumarMy BLOG: http://bestneurodoctor.blogspot.in"
},
{
"id": 64547,
"tgt": "What causes a hard lump over the tooth extraction site?",
"src": "Patient: Hi I had 3 wisdom teeth out 9 days ago. I I got my stitches out on Friday (one week after surgery) and the dentist said everything was healing great. But I ve noticed on my bottom right side there is a hard lump over the extraction site. I got my lower left taken out as well but that one looks normal. I just wanted to know if that lump is anything to worry about or if it s normal. It doesn t really hurt or give me any problems. It s just kind of there. Doctor: Hi..Can understand your concern..As per your complain formation of a lump after the removal of wisdom tooth is nothing serious but it is due to inflammation that occurs as a result of trauma caused during the surgical procedure which is normal in all surgical procedures..The more the injury or manipulation of the tissues the more is the inflammation and more time is taken for the inflammation to resolve..As there must be more trauma incurred to the lower right side so you have more swelling there and not on left side..You need not to worry and as the inflammation will subside and healing will progress the lump will reduce and slowly disappear..Hope this information helps..If you find the answer helpful please write a positive review and click on find this answer helpful as a token of appreciation..Thanks and regards..Dr.Honey Nandwani Arora."
},
{
"id": 50494,
"tgt": "Had an accident. Back problem. Undergoing physiotherapy. Recent Kidney analysis showed scar tissue on kidney. What do I do?",
"src": "Patient: Hi I was involved in a RTA 3 yrs ago when a 44 ton lorry hit me. I have had back problems ever since and currently receiving more physio, particularly on my left side. Recently after a 2nd blood test, my GP has informed me that I am in stage 3 CKD. I have no other explanation for this, i.e. low cholesterol, low BP, I'm 42 and healthy. I also had a scan of my kidneys, liver and gallbladder in Jan 2011 approx 7 mths after my accident and was found to have gallstones. She told me at the time of the scan, that I had scar tissue on my left kidney and that it looked to be recent, she also asked if I'd had any type of infection, of which no, I had not. I never thought to mention my car accident 7 months earlier. My question is, is there a possibility that the damage I have was as a result of the car accident? How would I go about finding this out for definite, would my GP need to refer me to a nephrologist? Thank you in advance. Doctor: HI I guess, your prime question is about scar of kidney, Right, so first of all let me clear it, that merely you stated so, otherwise actual examination of usg plate is required, if it is scar then it could be due to Your RTA, as such there is no obvious kidney related problem then you need not worry at all, you again stated that you are CKD patient for that you need to be in regular follow up with your treating physician, Good bye. (Low BP can not be it self, there must be some obvious reason behind this, clinical examination will reveal it)"
},
{
"id": 146914,
"tgt": "Suffering from Parkinson's disease. Addicted to medication, over dosage. On syndopa plus, pramipex and zolfresh. Need help",
"src": "Patient: sir mi aunt is suffering with Parkinson disease since 6 years and now she is having D.B.S the main problem with her is highly edicted to medicines and she is consuming over dosage even though not required and always thinking about medicines and nothing else. now she is using syndopa plus 1/4th tablet for every 2 hrs in between 6.am-9.pm pramipex 0.5mg thrice a day,zolfresh 5mg at 9.am & 5mg at bed time,mitraz 15mg at bed time. i request you to suggest to control her over thinking about the medicines and ediction to medicines. thnak you.... Doctor: I dont think that she is overdosing/addicted, though that is a possibility.The chance are more that she forgets the medicine she has already taken.Therefore as soon as she feels uncomfortable, she remembers again that she can take a pill and goes to take it.I think the best solution, and a little uncomfortable for you is to stop leaving her medication up to her. You can leave her medicines that she needs to take in a separate box. The morning and evening doses you can give to her yourself. In other words, you start medicating her."
},
{
"id": 151271,
"tgt": "Giddiness, head and neck pain, swollen eyelids, normal blood test. What is wrong?",
"src": "Patient: morning doctor, i am suffering from giddiness and heaviness in head and neck pain on the left side. i feel giddy when i just lie down to sleep or when i turn my head when i am sleeping and when i get off the bed in the morning. eye lids are a bit swollen. my mind has become dull. blood reports and all tests are normal. please let me know whats wrong with me Doctor: Hi Mr Bheemreddy, Thank you for posting your query on Healthcare magic. Based on your symptoms, the most likely cause of your symptoms is benign paroxysmal positional vertigo (BPPV). Please do not get scared looking at this big name, as this can be well treated with medications. BPPV is due to a minor problem in semicircular canals (in ear). The symptoms of giddiness get aggravated with movements of head and neck in this condition. For relief of symptoms, betahistine is helpful. To get fully well, and prevent its recurrence, certain exercises (vestibular rehabilitation exercises) are helpful. This can be learnt under the guidance of a physiotherapist. I hope it helps. Best wishes, Dr Sudhir Kumar MD DM (Neurology) Senior Consultant Neurologist"
},
{
"id": 53233,
"tgt": "Can strenuous exercises contributed to the abrupt rise in my sgpt?",
"src": "Patient: I am 70 years old. Last Dec. 8th my sgpt was 70 only , 72 was the upper normal limit. Last Jan.28th my sgpt went up 122. My ultrasound indicated I have fatty liver. I do a lot of strenuous exercises. Could the strenuous exercises contributed to the abrupt rise in my sgpt or is it caused purely by my fatty liver condition? Doctor: No, It's mainly because of your fatty liver. It needs to be evaluated further with LFT . Considering your age , Blood test including AFP , HBsAg, anti HCV and ferritin should be done"
},
{
"id": 140889,
"tgt": "What could cause burning sensation and pain radiating through the neck, spine, hip and thigh?",
"src": "Patient: I have had a lum lam 2011, 5 cervical fusion with hardware 2013, 2 level fusion thoracic with hardware 2015. Most recent rfa two sides bilateral L2-3 and 3-4, with burning pain down r hip and thigh, followed with 2 facet injections on the rt. I have strange sensations up and down spine with neck pain, vibrations in groin area and spasms down rt leg. what do you think? Doctor: Hi, It\u2019s all related to your spine problem and due to nerve impingement. Consult your neurologist and get an MRI scan for further assessment. You can take analgesics like Gabapentin which is quite effective in neuropathic pain as in this case. Hope I have answered your query. Let me know if I can assist you further. Regards, Dr. Shinas Hussain, General & Family Physician"
},
{
"id": 173197,
"tgt": "Need treatment for rash on the neck with liquid discharge",
"src": "Patient: Hi, my 3 months baby has a rash on her neck and can see there is a white liquid coming out from the same area. I consulted a pediatrician and he recommended Lulifin cream and Zucon powder. The rash has spread all over the foldings on both the sides of her neck. Please suggest if I am using the right drug? Doctor: Hi,Thanks and welcome to healthcare magic.Rash usually occurs in folds of the neck when it is not cleaned properly after bath or after eating or drinking..Because of moisture infection develops.Clean the folds of the neck with a clean dry cloth thoroughly .Apply lotio gention violet over the infected area twice dailyUsually the infection clears within 5-7 days.Hope this reply is OK for you.Please feel free to ask further queries if any.Dr.M.V.Subrahmanyam."
},
{
"id": 84074,
"tgt": "Could body pain, headache and cold be side effects of O2H tablet and Vizylac-dt tablet?",
"src": "Patient: Hi, I had stomach pain and vomiting 3 days befor. My famili doctor prescribed me O2H tablet, ViZylac-DT tablet (1 each twice a day after meal). I got relief from stomach pain. But now I have body pain, headache, cold and cough. Is it because of any side effect? Please advice me. Thanks Doctor: HiAntibiotics and probiotics may not cause the above symptoms.These symptoms can be due to a viral flu.Pain killers ,steam inhalation and betadine or salt water mouth gargling can be done.Antibiotics may be required in case of secondary bacterial infections.Incase of high fever or fever more than 3 days,blood tests are essential to rule out infections.Hope I have answered your query. Let me know if I can assist you further. RegardsDr.Saranya Ramadoss, General and Family Physician"
},
{
"id": 88649,
"tgt": "What causes severe abdominal pain?",
"src": "Patient: My girlfriend is on her period and started yesterday. We were just recently lying around watching a movie and all of a sudden she started complaining about a severe abdominal pain. She turned pail white, had to rush to the bathroom to relieve her bowels. The pain is in the center of her pelvic area. Sharp pain. She says that it isn't like any other cramps she's had. She has dizziness as well. She says that it feels better when she lays on her side. the pain is slightly subsiding but I am worried it could possibly be her appendix. Please advise Doctor: Hello,Apart from appendicitis and a urinary tract infection (UTI), I would be more concerned about an unsuspected ectopic pregnancy in case you both are sexually active. Diagnosing any of this condition would require a physical examination of your abdomen along with some routine blood tests like a complete blood count, a serum HCG test, and an ultrasound of the abdomen at the earliest. Therefore I would advise you to please take her to the emergency room of the nearest hospital for a detailed assessment and appropriate management. Till then you could give her some antispasmodic drug for symptomatic relief.Hope I have answered your query. Let me know if I can assist you further.Regards,Dr. M Y Shareef"
},
{
"id": 182601,
"tgt": "Should there be approval of GYN for the extraction of tooth?",
"src": "Patient: I stopped breast feeding over 1 yr ago, but still have a lump. I went to my dentist to have a tooth removed and he said he would not extract my tooth with out approval from my GYN... I m in the Philippines and I think the Dentist is way behind the modern world.. Is he right? I m afraid to have a biopsy as complications from such surgery happen way to often here. Sincerely, MM Doctor: Thanks for your query, I have gone through your query.It is advisable to get a opinion if you have any family history of breast cancer. If it is a small lump, it goes in favor of benign lesions like lipoma or fibroma. Nothing to be panic, Consult a good oral maxillofacial surgeon and explain your problem, he will examine and give a right suggestion whether to consult a gynecologist or you can go ahead with extraction. No need to go for biopsy, you can get an ultrasound or FNAC done to confirm the diagnosis.I hope my answer will help you, take care."
},
{
"id": 208464,
"tgt": "What to do for a patient with mental problem?",
"src": "Patient: My father is a mental patient since last 40 years. He had got doubt to my mother. Without geniuness he is telling and imaging the matters. He connected 3 to 4 of our relatives and telling bad relation with my mother. Sometime he is very violent. Nearly 2o years back we approached a Psycologist and he suggested a medicine DEplat 75mg, E. When some problems showing give this medicine. Last few years when this tablet given some face reactions appearing i.e. lips are going to one side and all.... so we are afraid and not given that tab. One year back he had a Heart attack - Silent. As per Cardiologist 3 blocks are there. Bye-pass surgery need. But the health condition is not good - Diabetic and Blood pressure - not yet surgery done and continuing Medicine (No circulation problem is shown in the Anchiogram). Medicine given for heart disease is Deplatt 75mg, Ecospring AV 150, Mono Isordil SR 30, Semi Daonil for Diabletic and Betacard AM for BP. Now few days he is very violent and creating problem/quarrel with others. We have not approached a doctor because if he know he will agains create problem. If we give the earlier medicine (mental) any problem will occure? He is 73 year old, Height : 5 8 . Weight : 78 Kg. Kindly advice us.... Doctor: hi dear,there are many medical illness your father have so treatment of psychiatric illness must not affect medical part. as you described your father suspicious over your mother and frequent violent episode suggest that he has psychotic disorder and need treatment.is there any other features associated with it?any sleep disturbance, hearing of voice, abnormality of speech and behavior etc...for proper diagnosis detail history is must. so consult psychiatrist for detail history and treatment.Thank you"
},
{
"id": 35381,
"tgt": "What does herpes virus value 34.9 suggest?",
"src": "Patient: hsv1+2 igg herpes simplex virus observed value was 1.99 . I took alovir tablets 4 everyday for 7 days. repeated the test after 10 days and found it shocking. observed value 34.9 .What does that mean. The value should have gone down.. not up. why did that happen?? what is this rising figure doing to my health? is it serious? Doctor: Hello dear,Thank you for your contact to health care magic.I read and understand your concern. I am Dr Arun Tank answering your concern.The provided value is of Ig G, which is suggestive chronic infection. As you have taken the treatment infection becomes the past so you have developed the immunity against the infection.Immunity against the disease is suggested by raised IgG levels.I advice you to continue the treatment. And allay fear of raising titre of antibody.Please maintain good hygiene till you are taking the drugs, it is equally important in treating infection as taking the drugs.I will be happy to answer your further concern on bit.ly/DrArun.Thank you,Dr Arun TankInfectious diseases specialist,HCM."
},
{
"id": 217696,
"tgt": "Suggest treatment for pain between armpit and waist while coughing",
"src": "Patient: I am female, 52 and have a sharp pain when I move or cough. It's located on my left side in between my arm pit and waist. I had a case of Shingles last June and the pain area is similar but this is much more painful. Could it be the same thing, meaning wait it out? Doctor: Hi there! A sharp pain in the left edge of the chest which worsens on moving or coughing could indeed be due to post-herpetic nerve damage (shingles=herpes) though I'd still recommend a chest xray to look for lung infection or fluid in the chest which might cause similar symptoms. Do write in with your reportsAll the best!"
},
{
"id": 199362,
"tgt": "How to interpret semen test results?",
"src": "Patient: I need help understanding my semen test results. What does this all mean? Colour: creamish , volume: 3.5ml , reaction /ph: alkaline, viscosity: reduce, liquefaction time: 30 min pus cell:1-2/hpf , RBC s: nil /hpf, spermatogenic cells : 1-2/hpf , sperm agglutination: absent active motile: 70%, sluggish motile: 10%, non motile: 20% morphology: abnormal forms- 2% , total 26millions / ml I appreciate any help you can give. God Bless you. Doctor: Hello Your findings suggests almost normal semen analysis.However,you may need culture and sensitivity due to presence of few pus cells.You may need to take antibiotics if there is any growth.Sperm motility is normal.Normally sperm motility should be at least 55 %.Your findings suggests 80 % motile sperms.Sperm count is normal.Normally it should be at least 20 millions/ml.Other findings like semen volume,sperm morphology,liquefaction time etc are also normal.Take CareDr.Indu Bhushan"
},
{
"id": 150825,
"tgt": "Would Kepra, hydrocodone and Xanax cause aneurysm?",
"src": "Patient: my wife erica rittenhouse just died on july 24th 2012. i was hospitalized and i woke up in the morning and found her to be unresponsive and foam was out of her mouth, i ran for help and they called code blue, three days later in icu , she was hooked to a ventilator and according to the doctor she suffered a anuyerism and the pressure was building in her brain causing her death. im deeply saddened by this and torn, and scarred, and i wanted to know is her taking medications such as kepra for seizures, hydrocodone for pain, and xanax for anxiety would cause this in her. Doctor: Hi, Thank you for posting your query. It is unfortunate to note that your wife passed away due to rupture of brain aneurysm and brain hemorrhage. The medications she was taking- keppra (levetiracetam), hydrocodone and xanax- are not related to aneurysm formation. In most cases, aneurysms are present from birth. Best wishes, Dr Sudhir Kumar MD DM (Neurology) Senior Consultant Neurologist"
},
{
"id": 119116,
"tgt": "Elevated WBC, lymph results high. What does it mean?",
"src": "Patient: I am a 50 year old female and recently had a check-up. Every year my WBC is slightly elevated. This time it was 10.6. Also this year, my #Lymph result was 4.0 and the paperwork from doctor indicated that the normal range was .6 -3.4 Do these two results have anything to do with each other, and should I be concerned about the Lymph results? Doctor: Hi, Your WBC count is slightly elevated according to you .What is the reference range written on the report. If it is towards upper side of normal limit with this lymphocyte count then you should not worry as you can have these values after mild viral illness."
},
{
"id": 38042,
"tgt": "Is it necessary to put vaccination because of dog bite?",
"src": "Patient: Hi, My pet dog who is almost 3 months old bite me today. Little bit of bleeding was also there. My dog has already had 3 injections of anti infections injections. Should i take some sort of antibiotic or something? Please advice me I am really worried. Regards, Mohit Raheja Doctor: Hello, Thnx to contact us. I understand your concern. If I am your doctor I advice you that your dog had completed vaccine and no need to worry. But please take tetanus toxoid vaccine, antibiotics for prophylaxis of infection. As dog is under your supervision check for any abnormal behaviour in the dog. immediately report if the dog comes with it.I will be happy to answer more of your concerns, kindly know me,Wish you a very good health at health care magic. Dr. Arun Tank. Infectious Disease."
},
{
"id": 169751,
"tgt": "Suggest remedy for ADHD and anxiety in a child",
"src": "Patient: Hello my child suffers from being able to focus on task and tends to forget school work and takes very long to start his school work, he also suffer a little from anxiety. What type of medicine would be best for him. He is not hyperactive but tends to wonder a bit. Doctor: Dear, Welcome to HCMWe understand your concernsI went through your details. First of all you did not mention the age of your child. At childhood, hyperactivity and fidgety is the basis of ADHD diagnosis. When age progresses, hyperactivity comes down and attention deficit become predominant. Anxiety is also common in childhood which usually is not associated with ADHD.I suggest, instead of asking for medicines here or online, please consult a psychologist in person. Let the psychologist assess the mental health of your child to determine whether the child has ADHD or not. Most of the ADHDs do not need medicines. Household and school training methodologies provide relief from the symptoms. There is no need to worry. Even if the child has ADHD, it is 100% curable.If you require more of my help in this aspect, please use this URL. http://goo.gl/aYW2pR. Make sure that you include every minute detail possible. Hope this answers your query. Further clarifications are welcome.Good luck. Happy New Year. Take care."
},
{
"id": 206555,
"tgt": "What causes nausea, tiredness and watery discharge?",
"src": "Patient: Hello. For the past two weeks I have been experiencing nausea after I eat anything,,it doesn t matter what I eat, im extremely tired, moody. I had my period. I think it was very light almost like spotting for only 1.5 days then it was gone. I have a lot of watery discharge...we are trying to get pregnant could it be that? Doctor: Hello,Possibly your symptoms suggest pregnancy. But it should not be just based on symptoms. You should go for a urine pregnancy test to confirm. Thanks."
},
{
"id": 82037,
"tgt": "Suggest treatment for wheezing",
"src": "Patient: i have many serious medical conditions but this started on Thurs, I started wheezing while breathing. then on Fre I am still wheezing but coughing a deep cough with flem mostly left side I am very achy all over. CHF what shhould I do? Doctor: HelloAs you mentioned that you are having so many serious medical conditions .Now ,you are also having wheezing , phlegm , cough ( deep ) on the left side ( according to you ), all these symptoms may be due to these possibilities , these include;1 Pleural effusion in the left lower lobe of lung . Although you mentioned that this problem is new one but you are also having so many serious medical condition. As I have noted in my many patient that in serious medical condition fluid accumulated in lungs and this is very common. Diagnosis can be confirmed by X ray chest.2 Wheezing may be due to bronchoconstriction added with infection . Physical examination reveal the condition. As sometimes allergy to any drug may produces such symptoms . Even allergy to dust , mite , pollen ,mold may produces such symptoms. 3 Congestive hear failure , also produces such symptoms .There are so many other causes of such symptoms like bronchiectais etc but I recommend you visit a pulmonologist and get his opinion.Good luck."
},
{
"id": 177721,
"tgt": "What could cause hallucinations, severe anger in a child?",
"src": "Patient: A 4 year old male child is hearing voices, he wants them to stop and says things like i want to cut off my head to make the voices stop, he tells his mother that that she is not his mother and he wants her to take him to his mother and that he wants to go home even when he is home. He gets extremely angry and says i am really angry and when he is sorry he says that he doesn t want to be bad/angry that he can t help it. The majority of the time he is a loving happy little boy, full of energy and wants lots of cuddles and kisses. Should the mother take him to a medical doctor or to a child psychologist. Doctor: Hello. i just read through your question. this is very concerning. The child should be seen by a psychologist or psychiatrist. the mother should first make the child's doctor aware and ask for a recommendation. The sooner this is done, the better."
},
{
"id": 37320,
"tgt": "What causes body ache, warmness, high fever and lack of energy?",
"src": "Patient: I came home from work early on Thursday due to body aches and feeling warm and lack of energy to continue my sit down job. Had fever of 100.4 that night...only vomited once a lot. Have slept most of the time since..drinking lots of water and hot tea. Also able to eat toast and scrambled egg. Nothing else even sounds good. Do I still have the flu? Doctor: HI,It seems that you might be having viral fever.Take analgesic like paracetamol or ibuprofen for pain and fever.Take plenty of water.Take light diet.Take rest.Within a day or 2, you will be all right.Ok and take care."
},
{
"id": 170312,
"tgt": "Can scorpion sting cause continuous high fever?",
"src": "Patient: my daughter was stung by a scoprion last week. She has started running a high fever for the last 3 days (unto 104.4). MD said ears, throat, lungs, urine all were ok. No other symptoms. CBC was done, but results not back. Could this be from the scoprion sting? Doctor: Hi.... I do not think that Scorpion sting can cause fever after one week. This could be an unrelated illness and most probably a viral illness. If the kid is well when the fever is down, you need not worried about this. I suggest you give Paracetamol in the appropriate those after checking temperature every four hourly once and that too only if the fever is about hundred degrees Fahrenheit.In scorpion sting rather than fever, heart rate and palpitations and profuse sweating are more significant symptoms. If these appear we need to be really concerned regarding scorpion sting.Regards - Dr. Sumanth"
},
{
"id": 213979,
"tgt": "Depression after back Surgery. Is it a related symptom ?",
"src": "Patient: I had a back fusion 1 month ago. I am not taking pain meds now. I am taking 1 muscle relaxer once a day. At the time of surgery I quit taking St. John s Wort all at once. 2 weeks post op I had an adverse reaction to Tramadol. I am now very moody and easily irriated and somewhat depressed. I have started taking the SJW again. on it for 3 days now. Can all of this be related? I don t know what to do about being so angery. Thanks Doctor: Hi Yes, it COULD be the fact that you stopped taking SJW. This is just an educated guess since the antidepressant compund in SJW has not been isolated so we don't know if that has withdrawal symptoms if stopped suddenly but I think this should be a ok enough guess. It could also be due to unresolved pain symptoms and hence if the pain is chronic and sub clinical you would probably not feel it so severe to go for pain killers but would be so irritating as to make you feel irritable and angry and moody....here I speak both from personal experience of chronic pain and as a professional. I would suggest you check out with your doctor first about interactions of SJW with your current drugs. If you are a female, then we know for sure that it interacts with the pill. There may be other unknown interactions also. Hope this helps."
},
{
"id": 10494,
"tgt": "What to do for severe hair fall?",
"src": "Patient: Hello doctor, Is using arcolane scalp solution effective for dandruff treatment? Facing severe hair fall problem atleast 200 hair per day ( looks like). Please help. Also, can altris gel help to reduce greying hair? I am 31 and my baby is 8 months now. Facing severe hair fall problem. Using follihair too.... Please help. Thanks Doctor: Hello and Welcome to \u2018Ask A Doctor\u2019 service. I have reviewed your query and here is my advice. It seems that your hair fall is due to dandruff. Arcolane shampoo contains Ketoconazole, so it is good to clear dandruff. Continue it on alternate days. Take Biotin and vitamin E cap. Altris 5 solution may be used for graying of hairs. Daily normal hair loss is 150. So do not worry. You will feel better with this treatment. Hope I have answered your query. Let me know if I can assist you further."
},
{
"id": 57636,
"tgt": "What could be the reason for experiencing liver damage symptoms?",
"src": "Patient: Hi, I took Tylenol Cold a couple of different times last year when I was sick and on vacation...but did have some alcoholic beverages while on vacation both times. I have experienced a few of the symptoms mentioned about liver damage. Is there anything other than just avoiding alcohol that I should do? I also take milk thistle daily. I haven t mentioned anything to my family doctor at this point. I look forward to hearing back and getting your expertise on this. Thanks! Tammy Doctor: Dear Friend.Hi , I am Dr Anshul Varshney , I have read your query in detail , I understand your concern.If you have been consuming alcohol in high amounts and taking Tylenol regularly , you are at risk of liver damage.To be sure get the following investiations :1. Liver Function Test2. Ultrasound Abdomen.This would clear the picture f your liver has some effect due to alcohol or tylenol/i would advise you not to consume any of them.This is my personal opinion based on details available here. If you want to discuss your issues further, you may please ask usStay Healthy. Dr Anshul Varshney , MD"
},
{
"id": 108658,
"tgt": "What causes stabbing pain in upper back?",
"src": "Patient: my father is 43 years old, he is experiencing back pains, more upper, and it feels like a something sharp stabbing him when he moves. When he raised his left arm, he felt excruciating pain. I thought of sciatica, but nothing is in pain with his thighs or leg areas. What do you think? Doctor: Hi welcome to HcmIt could be due to gall stones.Get an ultrasound abdomen and chest x ray immediately to rule out the disease.thankyou"
},
{
"id": 220981,
"tgt": "Suggest remedy for severe vomiting during pregnancy",
"src": "Patient: Hi Doctor My girlfriend have 3month pregnant So the problem is that she don t stop Vomiting ,special in the morning Last time she vomit a blood .I try to bought somethings. Will help her so at the shop they told me that sorry we can t sell,go to docter first so I need your help before I go to docter help me docter Doctor: Hi I have gone through your complaints.vomiting during pregnancy is quite common in first 5 months.start Tab Doxinate plus in the morning 10-15 mts before meal.if vomiting are severe take 2 tablets also , nothing to worry.these tablets are safe.if not reducing with this medication go for Tab Vomiset .consult the obstitrician and take the further management .Thank youRegardsVasundhara"
},
{
"id": 169133,
"tgt": "What causes hives on legs and arms in an infant?",
"src": "Patient: my 6 month old woke up with hives on his legs yesterday and it also appeared on his arms during the day. i put desitin on it cause someone told me that might help and alot of the rash went away by this morning but then it started coming back agian today and it gets worse when he gets upset. is this something i should take him to the er for or can i just wait till his doctor appointment tomorrow? Doctor: Hi Dear,Welcome to HCM.Understanding your concern. As per your query you infant have symptoms of hives on legs and arms which occurs when body give response to some external stimuli or some kind of allergen. Introduction of allergen to skin cause release of some chemical which make the skin to as well. They are usually harmless until they cause problem like breathing problems, swallowing problems. Need not to worry. I would suggest you to :-- Give cold compresses to reduce itchiness . give cold compresses several times a day . - To find out the trigger factor which is a complicated thing , But the prevention of hives is the best thing one can do . So try to avoid the trigger factor . Consult dermatologist/pediatrician for proper diagnosis . Doctor may prescribe steroid therapy. Until then continue taking antihistamine drug 'benadryl'Hope your concern has been resolved.Get Well Soon.Best Wishes,Dr. Harry Maheshwari"
},
{
"id": 224911,
"tgt": "Injection to avoid pregnancy for one year, safe to use ?",
"src": "Patient: Hi, i want to now if it is safe to take an injection to avoid pregnancy for over an year Doctor: HI, thanks for using healthcare magicYou may be referring to the depo provera injection. Yes, this can be safely used for a year, some persons use it for longer.Many women who use this form of contraception would notice that they may experience reduced or no menstrual bleeding during this time. There may be some episodes of breakthrough bleeding which is occasional spotting but this tends to resolve as the medication is continued.When you decide you wish to conceive you can discontinue the medications and your cycles would return.I hope this helps"
},
{
"id": 42696,
"tgt": "What should be the normal values for semen analysis?",
"src": "Patient: ok. i have my semen analysis report with me. could you pl interpret it?Quantity: 2 mlColor: greyish-whitePH: 8.0Viscosity: NormalLiquefaction time: at 20 minutesSperm count: 43 million/ mlMotility: actively motile-35 % Non-motile-65%Pus cells: 3/pfRBC: NilEpithelial cells: 2/pfAbnormal forms: 25 % (approx) Doctor: hai welcome to hcm here your semen analysis is normal.inability tu conceive with normal semen could be of fusion defect between egg and sperm.you could try iui as an first option and ivf for failed iui."
},
{
"id": 99519,
"tgt": "What is the treatment for persistent cough?",
"src": "Patient: Hello Dr. Goyal, thank you very much for offering your service. I was born with asthma, it would be mostly seasonal when I was younger. I didn't have to treat it all the time, but when I had the occasional asthma attack it would be pretty serious, almost to the point of hospitalization. As I grew older, the asthma became less and less apparent. Up until about 2 months ago, I had not had any asthma symptoms for several years. Then, I did. It went away for a week, so I thought it was ok. Then, it came back, and it was very scary trying to breathe at night. I went to a doctor and they said I had pneumonia. The antibiotics they gave me didn't seem very effective, and a few days later they also started me on a regiment of steroids. The steroids helped, as did a nebulizer. I was resting and feeling very ill for about a week and a half of that period, then I started to feel better. Well, that only lasted a few days, and now for the past few days it has been getting worse again, necessitating several nebulizer treatments yesterday, and (after much effort and wheezing) coughing up mucus with a strange bittersweet taste (perhaps the albuterol from the inhaler and nebulizer?)I am concerned because the illness has already been exposed to a full cycle of both steroids and antibiotics, and yet it persists. What might a doctor be able to do for me? Does this require immediate medical attention, or might it subside? Whooping cough is going around my area, and I do have a persistent cough. Thank you. Doctor: Hi,This could be acute bronchitis with your asthma as you are having persistent cough with expectoration.It can be diagnosed clinically but you must undergo routine blood investigation with sptum R/ M and C/S,chest x- ray to rule out any other pathology of lungs.You might require long course of antibiotics (amoxyllin with chavulanic acid or cephalosporins) with other supportive treatment like bronchodilators ( steroids,beta agonists,anticholinergics) inhalers or nebulizers with steam inhalation frequently.Consult pulmonologist immediately and take advise.Thanks."
},
{
"id": 153907,
"tgt": "Can cancer be detected through blood test?",
"src": "Patient: hi doc ive been having syptoms which could lead up to cancer im 19 year old male ive been for bloodtests and im afraid my parents are worrying and may of had the bloodtest information if so could a bloodtest tell that im at a late stage or possibly dieing this is my main concern Doctor: Hi, dearI have gone through your question. I can understand your concern. Blood tesy can detect blood cancer. Other cancer is also detected by blood test from tumour marker. But they are not primary test. You may have blood cancer or may not have cancer. Please send me your reports so that I can help you further. Hope I have answered your question, if you have doubt then I will be happy to answer. Thanks for using health care magic. Wish you a very good health."
},
{
"id": 141354,
"tgt": "What causes dizziness and nausea while on Tylenol?",
"src": "Patient: My husband has been taking anywhere from 10-20 Tylenol PM or Advil PM daily for the last 6 months. A few days ago he became extremely dizzy with nausea/vomiting. He went to the ER and they checked him thoroughly (CT scan, EKGs, labwork, etc.) and found nothing wrong. He did not tell them about the PM meds he s been taking. Is it possible these pills were the cause of his dizziness and nausea/vomiting? He has not had them as far as I know for about a week now but still complains of some dizziness though not nearly as bad. Doctor: Hello and Welcome to \u2018Ask A Doctor\u2019 service. I have reviewed your query and here is my advice. Tylenol PM long term consumption may cause drug induced liver injury, also possible that nephrotoxicity and labyrinthine injuries. This dizziness may be due to BPPV or hypotension or hypoglycemia or infection etc. Until examination is done it is difficult to say what it is. Avoid high risk activities, use water vapor inhalation and salt water gargling thrice a day. Use tablet Vertin thrice a day for five days. If symptoms not improved please consult your ENT specialist he will examine and treat you accordingly. Hope I have answered your query. Let me know if I can assist you further."
},
{
"id": 187195,
"tgt": "What could it mean to have white clumps while rinsing after having a tooth extracted?",
"src": "Patient: Its been three days since I had my bottom left back molar out and while I do saline rinses ive noticed white clumps with dark spots sometimes coming out after. The extraction site still looks white underneath where the clumps come out so im not sure if this will cause dry socket. I rinse very gently and im very careful not to get food stuck but still this is happening. Should I be worried or is this normal? Doctor: Hello, thank you for consulting with healthcaremagic. Actually it takes at least 10-15 days for socket to heal properly , as it has been only 3 days after extraction . These white clumps are food debris which get lodged in the socket. So just rinse the mouth after every food and get it cleared.Hope it will help you."
},
{
"id": 121010,
"tgt": "Suggest treatment for sore joint on middle toe",
"src": "Patient: FOR WEEKS I HAVE HAD A VERY SORE JOINT ON MY MIDDLE TOE WHERE IT MEETS THE BALL OF THE FOOT. IF I WEAR MORE SUPPORTING SHOES/BOOTS IT S BETTER FOR WALKING BUT WHEN I TAKE THEM OFF THE PAIN IS UNBEARABLE WHICH REDUCES ME TO TEARS. THE CLOSEST I CAN DESCRIBE, IS WHEN YOUR FINGERS ARE REALLY COLD AND THEY START TO HEAT UP AND THE PAIN SHOTS UP THE ARM IT IS A BIT LIKE THAT BUT A LOT WORSE AND IT CAN LAST FOR AGES AND PAIN KILLERS DO NOT HELP. HELP PLEASE I CAN T TAKE MUCH MORE. cHOP MY TOE OFF. Doctor: Hi, That is a very common inflammation of the toe joint. Change your foot wear. Go for a more softer sole. Soak your feet in a tub filled with warm water for about five minutes. Follow it up immediately dipping your feet in ice cold water for about three minutes. Repeat again, five minutes hot and three minutes cold. If you can visit a nearby physiotherapy center for ultrasonic treatment, it would work wonders. No need to chop your toe off! Hope I have answered your question. Let me know if I can assist you further. Regards, Shruti Bhave, Physical Therapist or Physiotherapist"
},
{
"id": 72463,
"tgt": "What medication is suggested for headache and stomach pains due to inhaling chemicals?",
"src": "Patient: my girlfriend was cleaning bathroom today with a product called super clean , cleaner and degreaser, an automovive product she was spraying the chemical on surfaceses and inhailed vapors... and now several hours later is complaining of head ache and stomach pains ???? you ve got to b kidding me rite i have a medical issue and your fuc ing billing me for and answer !!!!!!!!!!!!! what the hell is this world coming too Doctor: Hello dearWarm welcome to Healthcaremagic.comI have evaluated your query for girlfriend in details .* This is in relation with systemic effects of the chemical .* Guidelines for better outcome - Allow her plenty of liquids .- Prefer soft , light , bland diet .- Intake of proton pump inhibitor as omeprazole ( 20 ) mg with sips of water on emtpy stomach morning and evening before meals , tab. tylenol ( 500 ) mg mor and eve after meals for 2 days will clear the problem totally .- Avoid further exposure to any noxious substances of this or other types .- Deep breathing exercises , YOGA will support in faster outcome .Hope this will help you for sure .Wishing her fine recovery .Feel free to ask any further doubts .Regards ."
},
{
"id": 172128,
"tgt": "Suggest treatment for a necrotic lymph node in a child",
"src": "Patient: My 2 year old may have a necrotic lymph node in the jaw area caused by trauma. He has been on a coures of Antibiotics for the last week after having the infected area drained via a small cut uner the chine. What would the next step in the treatment be? Doctor: Hi dear, I understand your concern. At the first you should consult ENT and treat throat problems with antibiotics-Ceftriaxon 100mg/kg divided for 2,times during 7 days,decide question about tonsillectomia,If it is necessary.Secondly, take Vitamin E, A and cod oil capsule ,it will decrease cholesterol,also use olive oil 1 tablespoon 2 times a day .Eat pomegranate or Apple every day.Treat completely your nephritis, then cholesterol will normalize.Wishing you a good health"
},
{
"id": 136163,
"tgt": "Suggest remedy for rhomboid muscle pains",
"src": "Patient: Hi, i am having rhomboid muscle pains very badly and not able to work at my office...its kind of irritating..i am doing Yoga...but i am not getting right sitting position to cure my pain...when i am in home, i observed pain is not much,...but when i am in office only its getting increased. please help me Doctor: HiWelcome to healthcaremagicI have gone through your query and understand your concern. You are likely to be having muscular pain. Treatment of it is rest and analgesic such as ibuprofen for pain relief. Multivitamins supplements and mineral supplements can be useful. Vitamin B and C is useful in recovery. Vitamin D test and Vitamin B12 test is useful. If found deficiency you can take their supplements. You can discuss with your doctor about it. Hope your query get answered. If you have any clarification then don't hesitate to write to us. I will be happy to help you.Wishing you a good health.Take care."
},
{
"id": 194966,
"tgt": "How can erectile dysfunction be treated despite taking Tentex Royal?",
"src": "Patient: Hi doc, I am suffering with ED since last one month, am taking tentex Royal as per physician advice since last 4 days, but still no any improvement , so plese tell me can this problem be solved & how many time can ED remains, means throughout my whole life or it will be cured in any timeline. Doctor: Hi. I understand your concern.ED has a wide list of pathologies that may cause it, most of them are treatable, so I suggest not to be alarmed by it. Once the ED is on most of the cases won't pass until the cause is treated. I suggest my patients to see a urologist and go for further studies that may show the cause so the treatment is the one that you need.I hope this can help you."
},
{
"id": 176631,
"tgt": "What causes pain in arms and legs after fever?",
"src": "Patient: My daughter had a fever of 101 and a headache and chills on Monday and thru Tuesday. Wednesday she was better and went back to school but her legs were achy. She got small red dots on her legs Thursday night after her bath. This morning she woke up feeling fine but legs still very achy and the red dots have spread from her legs up her whole body to her chest and neck and her lip is slightly swollen. The rash has become about a 5 on the itchy scale. No fever, no other symptoms besides the achy legs and arms? Advice? I think it might be 5th Disease? Doctor: HI...I think she is going through a viral illness and it could even be viral haemorrhagic fever like Dengue too. I suggest you take to the nearest emergency room immediately. If her urine output is reduced, it is a tell-tale sign that she is going into shock. Itching is a sign of dengue again.Regards - Dr. Sumanth"
},
{
"id": 98062,
"tgt": "Bad appetite, low weight, inability to eat spice, on synthetic analogue steroids and ayurvedic medication. Help?",
"src": "Patient: Dear Sir, Greeting! I m a 30 yrs old male (currently a research scholar). I ve a moderately lean built with a body weight of around 64 Kg. More so I m keen to increase my weight to around normal (at least 4-6 Kg extra) but the problem is that I m skeptical about health/ protein supplements and more so I have a bad appetite to gain weight naturally. I also remain chronically constipated often with seldom having to go more than twice in a day (usually once in two days on an average or even less frequently sometimes), vegetarian, cann t eat spice food (probably also taking a toll on me in a hostel life) and cann t eat frequent diets in a day (poorly defined & busy for nothing sort of lifestyle of a PhD scholar). I m interested in gaining general health & weight naturally. Although I m a doctoral student of pharmaceutical science still I don t advocate the use of health/protein supplements as they may still contain synthetic weight gainers including synthetic steroid analogs. Due to lack of a good appetite and general feeling of weakness I sometime feel unable to meet my daily tasks. Further I have started developing a depressive attitude towards life and a lifestyle, which almost lacks adequate physical work. I need your help to suggest me a solution based on principles of ayurveda to gain what I feel lacking. Doctor: 1. any hemoglobin level or chest x-ray been done? 2. any Nutritional disorder? 3. try to remain calm and happy, because body and mind are interlinked, and this is what we call in Ayurveda the definition of 'Health' 4. You can Improve your Appetite by doing little bit of jogging,brisk walk for half an hour, but in the morning. 5. You can consult an Ayurveda Physician about the medicines helping in improving appetite and gaining weight (Kushmandha Avleha, Chaywanprash,Ashwagandha-avleha/arishta, Draksharishta)"
},
{
"id": 131207,
"tgt": "What causes sudden abdominal and joint pain?",
"src": "Patient: hi, i woke this morning with sudden adbominal pain and joint pain. this is the last day of my period and I m 49. before my period last week I had similar joint pain but no nausea. I ve been un able to keep water down an have thrown up three times today with diarrhea. initially i thought the joint pain was from dehydration so i kept drinking but I just threw up. now i have pain on my left side under the rib (spleen?) and exhaustion. do you think this is food poisoning or could it be related to releasing a cyst or something? thanks for your information Doctor: HiIt points to gastritis sort of thing as stomach is situated this side.cause could be from food related infection as you have both diarrhea and vomiting.A physician should be consulted,who may consider giving you anti emetics,anti microbes and advise on hydration and electrolytes balance.if fever and joint pains, can be some systemic viral infection cause also.Your history of illness and check up would help for treatment"
},
{
"id": 148942,
"tgt": "Seizures. Menstrual cycle not proper. Tests on brains all negative. Is it Lupus or due to anemia?",
"src": "Patient: Hello, I am writing you all because I am in desperate need to help my girlfriend. She has been having seizures everyday anywhere between 1-6 times a day no for the past 7 months.She's seen two neurologists and all tests on the brain come back negative. She was tested for epilepsy and those tests came back negative. However very bright or flashing lights can instantly send her into a seizure, as well as loud noises.Her menstrual cycle is very horrible as well. She frequently has blood clots which causes her flow to be abnormally heavy and the pain unbearable. Sometimes when she has her seizure afterwards she can feel a very sharp pain in her head and through her body, her vision seems to get blurry in her left eye, he left side has also gone limp and shown symptoms of a stroke, but that was ruled out as well.She is anemic and has low iron levels. We are not sure what is causing this and I need immediate help because It is gradually becoming worse and I am in fear she may lose her life if help is not given. No doctors will even prescribe her any meds for her seizures so we resort to high grade marijuana which has seemed to help, but not treat the symptoms.I did some research and thought she may have lupus or some type of anemia caused by low hemoglobin levels. She just turned 25 and is a black female if that may make a diagnosis easier. Doctor: Hi,Thank you for posting your query.First of all, we need to make a clear cut diagnosis in order to decide the treatment. At this point, there are two main possibilities- first, epilepsy and the second, non-epileptic (psychogenic) seizures.The diagnosis can be established by doing a long term (24-48 hours) video EEG recording. As she has several seizures per day, it is possible to record her seizures on video and see the EEG pattern during those seizures.After diagnosis, she would either need anti-epileptic seizures or psychological treatment.I hope it helps. Please get back if you require any additional information.Wishing you good health,Dr Sudhir Kumar MD (Internal Medicine), DM (Neurology)Senior Consultant NeurologistApollo Hospitals, Hyderabad, IndiaClick on this link to ask me a DIRECT QUERY: http://bit.ly/Dr-Sudhir-kumarMy BLOG: http://bestneurodoctor.blogspot.in"
},
{
"id": 192151,
"tgt": "What causes pain in penis with delayed ejaculation?",
"src": "Patient: hi sir, i am 23 years male newly married couple.i need good counseling from you.daily i sex with my wife.only one time only i do.next i try i didnt able to do.sperms are not come quickly.less amount of sperms only come.i have pain in my end of penis.some time blood also come.why sir what reason.any big problem sir.both we are not satisfied sir.for this problem, any problem to get a baby sir.i am so afraid........plz give a good suggestion how to cure Doctor: Hi, You are newly married and having too much passion for sex which is normal. Please maintain enough gap between sex like 3-4 days. It will give relaxation to your penile muscles and you will be re-energetic to enjoy sex next time. your amount of sperm will also increase. No problem, to get baby you will have baby soon keep enjoying your sex life. Take care. Hope I have answered your question. Let me know if I can assist you further. Regards, Dr. Pramod Kokare, General & Family Physician"
},
{
"id": 14392,
"tgt": "Suggest remedy for rashes in the mouth",
"src": "Patient: Hi my one year old son has developed a rash around his mouth and nose area, and has spread to his hands and feet. They are red pimple like spots. He has had all his shots, latest ones at 12 months. Could these be the measles? I don't think they can be chicken pox. Can you help? Doctor: Hi,You said that your son has reddish pimple like lesions on face around nose and mouth and spread to other areas. He has taken all vaccinations. With this much history, I assure you that it is not measles. But, most probably it may be impetigo. It is a bacterial infection caused by staphylococci. You consult dermatologist for firm diagnosis.Antibiotics and antiinflammatory drugs in proper dose may be considered. Antibiotic like sisomycine cream may be applied on lesions twice a day and in nostril at night. Give him bath with few drops of dettol containing water. Improve his immunity with nutritious food after taking advice from his paediarician.I hope this would help in managing his skin problem.Thanks.Dr. Ilyas Patel MD"
},
{
"id": 209719,
"tgt": "What is efficient treatment for Manias disease, noises in ear and hearing loss?",
"src": "Patient: i am abdulrahman..from koyilandy...i am undergoing treatment for manias disease since 10yrs..there is no sufficient remedy so far..suffering lot of noises in my ear and loss of hearing also... if i come to you..can u make treatment for a better changes in my sickness.. Doctor: DearWe understand your concernsI went through your details. I suggest you not to worry much. I suggest you should be some more descriptive. The problems are simply mentioned manias, lot of noises in my ear and loss of hearing etc. These simple terms are not enough. Psychiatrists and psychologist treat after thorough examination of whole symptoms. Therefore please be descriptive.I suggest you to call me at night after 8 PM. I am from Ottapalam, Palakkad. Phone 9995407714. We shall work together for a cure.If you require more of my help in this aspect, Please post a direct question to me in this website. Make sure that you include every minute details possible. I shall prescribe some psychotherapy techniques which should help you cure your condition further.Hope this answers your query. Available for further clarifications.Good luck."
},
{
"id": 121450,
"tgt": "Suggest treatment for blisters and rashes on ankles sparine area",
"src": "Patient: I sprained my ankle 7 days ago. The ankle was swelled and bruised over most of my foot. I could not put weight on it for a day, but after that I have been able to walk. Starting yesterday I noticed a little more swelling at the ankle joint and I have small blisters on the ankle. The blisters or maybe a rash itch like mad and burn at times. I have sprained my ankle before but never had the blisters or rash? I have not had x-rays Doctor: Hello,You may be having blisters secondary to a sprained ankle. The blisters usually occur after 48 hours and are usually seen after severe ankle sprain. I would suggest you to get an X-ray of the injured area to rule out bony injury. If on X-ray, there is a bony injury then you may require a plaster after a orthopedic consultation otherwise it can be treated with elevation rest and crepe bandage.Wish you a speedy recovery. Hope I have answered your query. Let me know if I can assist you further. Regards, Dr. Santosh S. Jeevannavar, Orthopedic Surgeon"
},
{
"id": 131040,
"tgt": "What causes lower back/hip pain, throbbing below knee when having controlled BP/cholesterol?",
"src": "Patient: Lower left back and hip pain. Terrible throbbing below my knee in my right leg- especially at night in bed. Almost too easy to lose weight weight after years of trying without success and sometimes, my insides just hurt, especially after eating. My doctor was leaving the practice and didn t seem too concerned at our last check-up. I will be 57 this weekend and weigh 209 [down from 231] last fall. I quit my desk job to work 3 days a week in a busy diner near my home. Blood pressure, and sugar are good. Cholesterol has dropped like a rock and meds are cut in half and hopefully will be cut out at appointment this summer. Just feeling lousy. Should I be more concerned than my doctor and arrange for earlier appointmentwith my new doctor? Doctor: In my opinion you have multiple level lumber spondylosis along with peptic ulcer , you will need a back x ray and endoscopy for your ulcer also do stool test for h.pyloriGood luck"
},
{
"id": 91905,
"tgt": "Is recurring, constant sharp pain in upper abdomen below rib cage concerning?",
"src": "Patient: Hi, Friday night at around 9pm my upper abdomen started havin a sharp pain that kept me awake until 5am when my alarm went off for work. This continued all day Saturday and seemed to get worss Saturday evening. Seemed to have let up by thismorning (Sunday) until I got out of bed. Now it seems to be coming back. It is a sharp pain centered up in my upper abdomen, just below my rib cage. What are your thoughts? Doctor: Hello,With your presenting compliant I would think of two possible conditions.First possibility may be Gastritis. I would advice my patient to avoid oily, spicy food and Caffeinated drinks. Omeprazole at least for 2 weeks is advised.Second possibility may be Pancreatic. Blood test like Serum amylase and Ultrasound of the abdomen is advised."
},
{
"id": 193299,
"tgt": "Will nightfall resume after stopping masturbation?",
"src": "Patient: sir i used to masturbate at an average 3-4 times a week. i am 19. Then it suddenly came to my mind that why not stop doing it for a while and see whether nightfall resumes or not, just to check whether everything is normal or not and to make sure that i did not do it excessively. I asked one of my friends and he said it will never resume. please tell me whether it will resume or not? and if yes then how much time it will take for it to resume? Doctor: Hello,The duration of night fall and time for night fall cannot be predicted if you stop masturbation. Three to four times a week masturbation will not cause any harm. You might have night fall if you stop masturbation because after few days to weeks semen need to be ejaculated from storage genitals and so night fall is obvious.Hope I have answered your query. Let me know if I can assist you further. Regards, Dr. Parth Goswami, General & Family Physician"
},
{
"id": 115938,
"tgt": "Can a blood clot cure itself if blood thinners are taken daily?",
"src": "Patient: I also have have 20 Lovenox injections for the blood clot twice a day, every 12 hours. My INR is only 2.1 and seems to be going the wrong way. What could be causing it to go 2.1 from 2.4 in four days? I am following doctor's orders for hopefully dissolving the blood clot. Any ideas? Also, can a blood clot cure itself if blood thinners are taken daily? Doctor: Hello,I assume you're talking about deep vein thrombosis. The usual treatment starts with a low molecular weight heparin like enoxaparin (lovenox) and continues with a coumarin-drug which raises INR to protective levels. Blood thinners help to stop ongoing thrombosis but they don't affect the already formed clots. This is very important because an expanding clot can detach from the vessel wall and occlude pulmonary vessels causing life threatening conditions. With time, the body will absorb the clot and sometimes the vein can become patent again although not as functional as before the thrombosis.I hope I've helped you understand your condition better!If you'd like more information, please let me know and I'll be glad to answer.Kind Regards!"
},
{
"id": 33082,
"tgt": "What is the treatment for sneezing and sinus infection?",
"src": "Patient: Hi Doctor...I am 28 years woman..I have a problem of sneezing i sneez alot due to little change in weather my right nose is blocked not clear...doctor said its a sinus problem due to which i sneez alot..MY question is we are planning a baby if i got pregnant does it effect my child?? Doctor: Hi Dear,Welcome to HCM.Well the symptoms you mention in query are due to sinusitis ( as you also mention ) . Sinusitis is infection in the sinuses in which linning if the sinus become inflamed and produce mucus . As you mention that you are planning for a child . I would suggest you to first of all consult ENT specialist for proper treatment of sinusitis , because it may not effect your baby directly but as infection will lower your immunity hence also lower the immunity of child and baby become disease prone . Some times the infection spread to blood and through blood it may transfer in to your baby and cause problem . So it is better to go for proper treatment . Doctor may prescribe antibiotics (amoxicillin) for at least 10 days , decongestants and anti histamines .Hope your concern has been resolved.Best wishes,Dr. Harry"
},
{
"id": 185886,
"tgt": "Suggest treatment for swelling around the wisdom tooth",
"src": "Patient: Hi I have three fully grown in wisdom teeth one I had to remove when I was nineteen but the other evening I found it very swollen around the one remaining in my bottom almost like a hard knot it has not gotten worse of better it almost seem like the the tooth is going in a little again can this happen??no pus the bump is hard like swollen gum tooth seems fine what could this be and how long should I wait till I seek help I have no insurance and don't want to unless I have to Doctor: thanks for your query, i have gone through your query. the hard structure over the wisdom tooth could be because of the chronic irritation over the gum by the opposing wisdom tooth. the wisdom tooth can erupt til 21years of age. consult a oral physician and take a radiograph. then if sufficient space is there for the tooth to erupt then get the soft tissue covering removed. if the tooth gets infected then it can produce swelling , pain, limited mouth opening. if space is not there then get the tooth removed. i hope my answer will help you. take care."
},
{
"id": 110986,
"tgt": "What could cause a prolonged back and abdominal pain?",
"src": "Patient: hello i been having real bad stomach pains and back pain for over three years i be sick in my stomach everyday i been going to the doctors for three years then had lab test done and all different test done and still don t no whats wrong with me. Im afraid because i think that something is serious wrong with me. Doctor: Hi, thank you for posting.I have gone through your query and I understand your concerns.Your symptoms are caused by IBS(Irritable bowel syndrome). To treat this disease you should follow these guidelines:1. Eliminate spicy foods and acidic foods from your diet.2. Avoid alcoholic drinks, carbonated drinks, coffee and milk.3. Quit smoking cigarettes(if you do smoke).4. Reduce the amount of fiber in your diet.5. Take proton pump inhibitors medicines such as Nexium.Contact your gastroenterologist.Take Care.Dr. Behar."
},
{
"id": 164546,
"tgt": "What causes vomiting, fever, lethargy and loss of appetite with constipation?",
"src": "Patient: My four 1/2 year old daughter has had occasional vomiting (3x in 5 days) on and off low grade fever, lethargy and loss of appetite for five days. Today she had hard pellet like stool so I insisted she drink a full cup of juice/water and she threw it up almost immediately. Do you have any insight as to what might be the cause or next steps I should take, thanks! Doctor: fever , vomiting, loss of appetite is due to acute bacterial infection, start amoxyclav for 5 days and anti emitics for vomifor 2 days.. ask took take plenty of oral fluids"
},
{
"id": 128782,
"tgt": "What does knee pain indicate despite taking Ibuprofen and wearing compression braces?",
"src": "Patient: what could cause sudden onset of knee pain in a 15 yr old female who has never had problems ? Jusg started hurting first week of band camp back in June. Still hurting 2 months later. She is taking ibuprofen and wearing compression brace and still hurts. Doctor: Hi, I understand your concern. The female patient is young 15 years old. The knee pain could be due to knee trauma. Get her visited by an orthopedic specialist, get an x-ray of the knee done and possibly MRI if nothing is found. We should ensure there is nothing torn there. Hope this helps."
},
{
"id": 42937,
"tgt": "How to increase sperm motility in a 37 years male?",
"src": "Patient: Hi, Doctor, Good Day, I have been married from past 10 years ,my age is 37years and I don\u2019t have a child. My sperm count is 72 million , but motility varies from 0 to 15. What treatment (medicine herbal or English )I should take for improving my motility and what would be the reason behind low motility. Kind suggest. Thanking You in anticipation Doctor: Welcome to HCM!According to WHO study if a person has 15million sperm he is considered to be a normal man earlier it was 20 million. It is good that you have such a high count but motality rate is low.There are so many medicine in homeopathic system of medicines which will increase the sperm count as well as it will increase the motality rate. So consult a good physician.Take care.Dr.Minhas"
},
{
"id": 224617,
"tgt": "On ortho tri- cyclen lo. Feeling pressure in ovary after urination. Feels headaches and fatigue. What to do?",
"src": "Patient: I am on Ortho Tri- Cyclen Lo. I have regular periods each month, ONLY because of the BC due to my PCOS . I am 30, and my next period is due in about 12 days. I noticed a mild/pressure feeling in what seems like my lieft ovary about 3 days ago. It mainly bothers me AFTER urination when I go back to sit down, or sit and lean forward, but then subsides. I can feel it mildly when I walk, or move at all, but it does not really hurt.. just an annoying pressure. Kind of feels like it radiates to my left side of back, but not sure if it is just a feeling.. Other symptoms are regular when I am PMS , Breast tenderness, headaches, fatigue.. Just curious what this could be? Due to have a PAP next week.. thanks! Doctor: Hello. Thanks for writing to us. The symptoms that you are having are mild side effects of the pills that you are taking. usually the symptoms subside on their own in few weeks. I hope this information has been both informative and helpful for you. Regards, Dr. Rakhi Tayal ,drrakhitayal@gmail.com"
},
{
"id": 75285,
"tgt": "Suggest treatment for difficulty in breathing",
"src": "Patient: Hi doc. Right now, my son can't easily breathe when he is laying down. When i asked him to feel if his chest is affected he answered NO. He said that the problem he feels is only from his nose. When I checked it, there is a small passage of airway. I am confused if it is inflammed. I don't know if he is suffering from rhinitis right now. That's why i can't give him medication. I just ask him to lay down and head tilted. As far as his history is concern.. He has an asthma acquired from allergic rhinitis and aggravation of food with MSG, chicken, egg, etc. Hope you could help me give him a remedy. Thank you. Doctor: Hi and thank you for asking. Given the fact.that your son is already diagnosed with alergic asthma his concern now it might be related with the asthma. But for excluding any problem like adenoids better for him te do a test called Nasoscopy. And if it is positive the ENT specialist will remove it surgically.And your son with be better again.Thank you Dr.Jolanda Pulmonologist"
},
{
"id": 25005,
"tgt": "What could cause continuous pain in chest?",
"src": "Patient: I have continuous chest pain which increases to unbearable at night. By 9 am the pain subsides to dull again. Echocardiogram came back clean. chest xray the same. it was found that the right side of my heart was slightly enlarged, No ephsophogus problems found. Bone scan bothing Doctor says it is in my head and told me to wait a few months and he will check again. Doctor: Hello and thank you for using HCM.I carefully read your question and I understand your concern. You should not worry about. I will try to explain you something and give you my opinion. You should know that typical heart pains have some characteristics. They usually are retro-sternal pains like squizzing ,tightness or heaviness. They may irradiate to both arms, neck and jaw. They last from 5 to 20 minutes. Pains that last all night are not typical heart pains. The first and most important thing is to exluce ischemic heart desease.The fact that you already have done echo and its normal its a very good thing. If I was your treating doctor I would recommend a strees test to definitely exclude ischemic heart disease. The fact that you have excluded any skeletic problem is also a good thing.The other thing to exclude is a gastro intestinal problem that sometimes are resposible for this kind of pains. So, I would recommend a fibrogastroscopi.After excluding this we can be shure thats nothing to worry about. Hope I was helpfull. Best regards, Dr.Ervina."
},
{
"id": 90952,
"tgt": "How to treat ulcer and typhoid with abdominal pain?",
"src": "Patient: I'm 48 years, average height of 1.7 and wieght approx. 70kg. I was treated of H-poleri with amocin,amoprazole and the third antibiotic in jan'2011. I still feel abodomenal pain after one year of treatment. In jan 2012 I develop typhod symptoms and the test shows typhoid. I'm now face with ulcer and typhoid. I still have abodomenal pain as I take the typoid drugs. Doctor: Hi.Thanks for your query and an elucidate history.If you have taken a proper treatment for ulcer and typhoid , both will be healed well..Why are you bothered? If you are get confirmed by upper GI endoscopy, blood tests as a routine and for typhoid and ultrasonography for the abdomen. If the tests are normal there is nothing to worry about as there are other causes that can cause the pain.. You have to undergo a proper clinical examination to be done by a Doctor who can catch many of the problems on your proper history and physical examination and can then guide for further investigations and treatment."
},
{
"id": 173631,
"tgt": "What causes twitching of left foot with fever in 2 year old?",
"src": "Patient: My 2 yr old daughter has had a slight cold for about a week today she started running a tempature of 100.8 as the day went on her tempature went up to 101.5 and with Motrin it went down but her left foot twitches for about 30 seconds every 30 minutes or so she doesn t seem to notice when it happens what could be the cause of this and should I be concerned? Doctor: There are different causes of twitching leg-1.Seizures are changes in the brain\u2019s electrical activity. This can cause dramatic, noticeable symptoms or even no symptoms at all. The symptoms of a severe seizure are often widely recognized, including violent shaking and loss of control. However, mild seizures can also be a sign of epilepsy.2.Alkalosis is a condition in which the body fluids have excess base (alkali).3.The hormone disorder hypoparathyroidism may also cause calcium deficiency disease and twitching.4.Calcium deficiency disease was caused by malnutrition (starvation) and malabsorption (when body cannot absorb the vitamins and minerals it needs from the food). I suggest--Consult neurologist-Check level of Calcium in the blood-Give to baby diet,enrich with Calcium:cheese,curd,milk,dahi,sesame seeds,spinach and etc."
},
{
"id": 116544,
"tgt": "What causes sudden anemia?",
"src": "Patient: Last week i had a renal angioplasty done and a heart cath done... On Thursday the renal required a stent, then on Friday morning Heart Cath required a stent.... Something happened after the heart cath... I remember vaguley that i was put back in my room... Then all sudden I was freezing from my body core and sweating like a river... I was immediatwly was rushed back to the Cardia Cath Unit and Doc and his team worked with me 3.5 hours before admitting me to SICU.... Since i have come home.... But a few days later i went back to ER not feeling well at all... Ended up, that they say i am anemic. I have always been fine as far as stuff like that... But now i cant walk 10ft w/o having the symtoms and exhaustion... Would there had been something during the Cardia Cath stuff that would cause me to go anemic so quickly .... Also i had i 6bypass surgery back in September.... Would all this be realated, how, and what do i do to feel better.... So far they just added iron supplements Doctor: Hi,Thanks for asking.Based on your query, my opinion is as follows.1. You had a recent surgery and you cannot expect to be normal soon.2. Even mild reduction below normal, will impact a large way at present as the heart is still adjusting.3. Take the nutritional supplements and get a reticulocyte count done after 15 days. If its increasing continue therapy and follow up with haemoglobin estimation after a month. 4. During cardiac Cath, you would have lost some blood, but that would not be the major cause of anemia.Hope it helps.Any further queries, happy to help again."
},
{
"id": 72071,
"tgt": "What causes shortness of breath, fatigue and discomfort in right rib?",
"src": "Patient: I am 39 I have had shortness of breath fatigue and discomfort under my right rib, i am a smoker from the age of 13, i get a metal like taste in the back of my throat, it feels like my lung is blocked sometimes. I have had two chest x rays. I have coughed up blood, but i dont know if that is caused by coughing hard. I have had blood tests etc, possible CFS but i dont know why it feels heavy in my right rib area, i havent lost weight as such, but i dont have an appetite. I constantly worry about lung cancer, and still smoke (stupid i know)i have a constant fuzzy head, i do have a lot of stress and have for a while, i know i need to stop smoking and reduce stress Doctor: Thanks for your question on Healthcare Magic.I can understand your concern. Since you are active smoker, possibility of bronchitis is more likely. Bronchitis (inflammation of airways) can cause cough, breathlessness, hemoptysis (blood in sputum) and chest pain.So better to consult pulmonologist and get done clinical examination of respiratory system and PFT (Pulmonary Function Test).PFT will not only diagnose bronchitis but it will also tell you about severity of the disease and treatment is based on severity only. You will mostly improve with inhaled bronchodilators (formoterol or salmeterol) and inhaled corticosteroid (ICS) (budesonide or fluticasone).Smoking cessation is must so enroll in deaddiction center. Avoid stress and tension, be relax and calm. Don't worry, you will be alright with all these. Hope I have solved your query. I will be happy to help you further. Wish you good health. Thanks."
},
{
"id": 203665,
"tgt": "Can regular ejaculation by masturbation affect the future in any ways?",
"src": "Patient: hlw doc,i am of 17 and i am ejaculating sperms since i was of 14.now it's being habbit of mine to ejaculate everynight as i can not sleep if i do not ejaculte. After ejaculating i feel tired and sleepy, and if i don't ejaculate i feel that there is so much of energy in me and ican't sleep.so, my question to you is that will it affect me in future?????? Doctor: Hi Masturbation at your age is a normal phenomenon. so you should not be so much worried about yourself and your future. But you should know to control yourself. i suggest you to do some relaxation exercises before going to bed.Hope this answers your query. If you have additional questions or follow up queries then please do not hesitate in writing to us. I will be happy to answer your queries. Wishing you good health."
},
{
"id": 129047,
"tgt": "What causes severe hand cramps?",
"src": "Patient: In the last two weeks have been experiencing fairly severe hand cramps. Have also had leg/foot cramps nightly for several years. What can I do to alleviate? NOTE: My thyroid was irradiated approximately 7 years ago. Don t know whether this has an impact or not. Doctor: This can be because of dehydration or electrolyte imbalance or due to hypocalcemia also as you mentioned your thyroid was irradiatedWatver,get yourself evaluated for the same"
},
{
"id": 197253,
"tgt": "Why am i losing the sperm very soon?",
"src": "Patient: Sir, I am 20 years old and I am single, I have a sexual problem, when here rubs my penis so I lose my sperm very soon and some time in a few seconds, I have left masturbation for long time but when I use to check my desease so it do the same. I have become bore from my life, plz tell me what should I do, give me a good suggestion, Thank you very much. Doctor: Hi ...Provisional Diagnosis : premature ejaculation ...Here are few advise ....This type of problem mainly related to anxiety related or by depression....So try to avoid stress...Be relax during sex and assume different position like female with active role and on top while sex ...Kegel exercise help in strengthen pelvic floor muscles .....Double condom can be wear. ..Masturbation before sex helpful ...You can communicate with partner when to slow down to prevent early ejaculation...When immitent feeling of ejaculation squeeze penis head with index finger and thumb to prevent early ejaculatiin called Johnsons squeeze technique ....Deep breathing exercising helpful ...Local anesthetic ointment can be applied on glans ....If needed fluoxetine can be given ...Discuss all this aspect with your psychiatrist .Hope your concern solved. .."
},
{
"id": 28244,
"tgt": "What causes an extra systolic best?",
"src": "Patient: I am now 66 and since the age of 22 I have had periods of becoming aware of a missed beat sensation. It feels like there is a hiatus - as though my heart has stopped for a moment followed by a stronger than normal beat. If I feel my pulse there is indeed a gap in the rhythm of my heart which corresponds with the feeling of the missed beat . I was told by a cardiologist about 20 years ago that I get an extra systolic beat and then a post extra systolic beat which is the one I feel. At the moment I am getting this a few times a minute. In the past it can go on like this for days or weeks. I always find it mildly disconcerting and wonder if there is a way of encouraging my heart to return to its normal rhythm. I don t get any other symptoms or pain. Doctor: Hello! Thank you for asking on HCM! Regarding your complains, I would explain that when you have an extrasystolic beat, it is followed (after a compensatory pause) by a post-extrasystolic beat that is stronger and disturbing.For a better evaluation of those ectopic beats, I would recommend you to perform an ambulatory 24 to 48 hours ECG monitoring, in order to investigate the nature of those extrasystoles, as well as their quantity.It is also needed to study the heart structure and function by a cardiac ultrasound, coupled with some additional lab tests (complete blood count, thyroid hormone blood levels, chest X ray, serum electrolytes, BUN and creatinine, etc).If you have a structurally normal heart and the extrasystolic arrhythmia is not clinically important (according to Holter monitoring), probably you are not going to get prescribed drugs). If an etiologic factor is detectable, a properly correction would resolve your heart beats issue.Hope to have bee nhelpful to you. greetings! Dr. Iliri"
},
{
"id": 66848,
"tgt": "What causes painful lumps in armpit?",
"src": "Patient: A week ago I discovered multiple very painful lumps under my left armpit. They do not seem to move. Since I read that some can be caused by an infection, I started taking an antibiotic. I have been taking it regularly for a week now. I do not have any symptoms of Mono or any other illness. The lumps have gone down slightly, but are still painful and seem to throb inwardly. Any thoughts?? Causes? Doctor: not to worry much as this is benign condition like infected sebaceous cyst or skin adnexal abscesses! however, could be lymph nodes also!if really worries you can go for an FNAC test for confirmationall the best!"
},
{
"id": 47645,
"tgt": "What causes white sticky mucus in urine while having kidney stone/cyst?",
"src": "Patient: I have had kidney stone and cysts on my kidney and a ovarian cyst and vaginal bacterial infection and still in pain today I wake up and hurt to the touch and when I peed I had a white sticky mucus come out what could it.be I have been like this for.a.month Doctor: Hi, welcome to HCM.looking to your history, it seems that you are suffering from kidney stone with infection.Pus discharge in urine can produce this type of picture.You should check basic urine report with culture, S.creatinine, and sonography or preferably CT scan.Take Gynecologist's opinion for vaginal infection. It can sometime produce recurrent urine infection.Maintain adequate hydratio.You can consult urologist or you can communicate with me also with above reports.I think this would be helpful to you.Best wishes. TC.Dr Jay Patel."
},
{
"id": 182282,
"tgt": "What causes tender lump in socket after removing the wisdom tooth?",
"src": "Patient: I got my wisdom teeth removed a month ago. I ended up getting dry socket and an infection so the doctor gave me more medication. I started to heal and even flushed out the sockets. Not I have a lump along my jawline that is tender to the touch. Is it possible that food got stuck in the socket? Doctor: Thanks for your query, I have gone through your query.The lump over the jaw line can be because of the infection of the bone that is garres osteomyelitis. Nothing to be panic, complete the course of antibiotics particularly amoxicillin and metronidazole combination. keep the socket area clean by rinsing with saline. If it is not reducing in a week, consult a oral physician and take a occlusal radiograph to confirm the diagnosis.I hope my answer will help you, take care."
},
{
"id": 72483,
"tgt": "Are Prednisone and Amoxicillin given for cough due to bronchitis?",
"src": "Patient: After two weeks cough I went to the doctor who said I had bronchitis and he prescribed me Prednisolone and Amoxicilline, what I didnt tell him is that I also got a lower back pain sore lower belly and I pee alot, are these symptoms sometimes related to bronchitis ? Doctor: You might be having a UTI too, as you pee a lot and have a low back pain. Apart from that you have cough too....I think we need to see your urine routine exam for infections there. Back pain in this case has a even more grave implications as it may suggest infection of the kidney itself. In such a case prednisolone would be bad. Please revisit the doctor and tell the full story."
},
{
"id": 138099,
"tgt": "Suggest treatment for pain in leg which is causing difficulty to walk",
"src": "Patient: Over the last 6 weeks my legs have been hurting so much that I can t walk like I did 2 months ago and I can tell that can t go as far as I get 3 weeks ago. And when I say can t wLk to far I really mean very far and I am only 55. I need this fixed NOW it really hurts. Please tell me you can tell me something to make it feel better, Thank You very much Doctor: Hello,Welcome, and thanks for sharing your concern I went through your query, and I feel, you need to make some points clear to us1 Where does it hurt in the leg, exact site, whether it is the joint or the bone.2 Do you have any associated symptoms like fever, redness or rashes on the leg. Only if you can provide us with these answers can we proceed further in your case. I hope my advice would have been useful, in decision making regarding your treatment, still if you have any clarifications or doubts feel free to contact back.Thanks."
},
{
"id": 206320,
"tgt": "Suggest treatment for behavioral changes in a 13 year old boy",
"src": "Patient: my son is 13 years old and has aspergers. he has always been sweet and obedient. we switched schools this year and it has been awful. he threatens to hit me, tells me to shut up , pees on the carpet in his room and in the playroom. He always promises not to do it again but does and they are big stains. he just threatened to slap me. What do I do. I know some of this is puberty but why the urinating? Doctor: Hi dear,thank you for your query.Your son might have some psychiatric issue like adjustment problem in new school or it may be opposition defiant disorder in which child became rebellious to parents.But detail history and mental status examination is required for proper diagnosis.Is there any symptoms like high talk, excess talkativeness, etc..Nee to evaluate in detail so consult psychiatrist as early as possible.Thank you"
},
{
"id": 218585,
"tgt": "Is absence of fetal heartbeats during pregnancy a matter of worry?",
"src": "Patient: actually i have done my first ultrasound of pregnancy in 6 weeks and 6 days for checking the heart beat of baby but doctor said no heartbeat has been formed. N ask me to come again after one week. So i just want to know the status n if there is something to worry? Doctor: Hello and Welcome to \u2018Ask A Doctor\u2019 service. I have reviewed your query and here is my advice. Actually in transabdominal ultrasound by 7 weeks fetal heart beat can be seen. And in transvaginal ultrasound usually by around 6 completed weeks you can see the fetal Heartbeat. But sometimes when you are having wrong dates you may not able to see the fetal Heartbeat according to your estimated gestational age. So, better to wait for one more week as suggested by your radiologist and repeat the scan after one week if you are not able to make out the fetal heart even after one week the possibility of blighted ovum has to be ruled out. Hope I have answered your query. Let me know if I can assist you further."
},
{
"id": 99397,
"tgt": "Suggest treatment for asthma and chest thightness",
"src": "Patient: Hi. My 15 year old has asthma. She is experiencing tightness in her chest, rising fever, and a cough with yellow phlegm. It started today. Should we treat her at home or wait until tomorrow to see if she improves? Didn t realize there was a charge for a response. Never mind. I will follow my instincts. Doctor: Hello,Thank you for asking at HCM.I went through your daughter's history and would like to make suggestions for her as follows:1. As she has fever and yellow phlegm, I would suggest her to start antibiotics like amoxycillin or azithromycin.2. As she is a known asthmatic and has experiencing tightness in chest, I would suggest her to use salbutamol inhaler as rescue inhaler, which will provide her relief.However, if her chest tightness or breathlessness does not improve with rescue inhaler, I would suggest you to get examined by a doctor immediately.3. Please continue regular medications for asthma, if any.4. If she has breathlessness any time, please contact your doctor immediately. Also, contact your doctor immediately if - fever does not respond to paracetamol, very high fever/rising fever, any another new symptom develops, etc.5. For relief of fever, I would suggest her to take paracetamol if temperature is more than 101 F.6. If she has additional nose symptoms like running nose, sneezing, nose congestion, etc, I would add levocetirizine + montelukast to above treatment.7. Please give her rest. Make sure she takes adequate amounts of liquids during day to maintain hydration.8. Please avoid exposure to dusts, smokes and air pollution as much as possible for her.Hope above suggestions will be helpful to her.Should you have any further query, please feel free to ask at HCM.Wish your daughter a very good recovery and best of the health ahead.Thank you & Regards."
},
{
"id": 110972,
"tgt": "What causes severe back pain?",
"src": "Patient: Hi, my boyfriend suffers with severe back pain most days, he is very active and fit and only 27 years old but following many doctors appointments and treatments at the chiropractor nothing has worked. We just want to know what s happening in his spine as the lower aspect feel crunchy and gristly when massaged.... If we knew his condition we could then get proper treatment for him but want to stop his suffering and pain. Thankyou. Laura. Doctor: Hello, Thanks for your query.From description it seemshe might be suffering Mechanical back pain.\u00a0\u00a0\u00a0\u00a0\u00a0It is a common medical problem.Mechanical back pain implies the source of pain is in the spine and/or its supporting structure. The surrounding muscles and ligaments may develop reactive spasm and pain.1. Lie on hard bed2. Taking muscle relaxants in combination with mild pain killers will help .3. Hot water fomentation to back 2-3 times a day and applying a local analgesic gel on the affected area will also help in providing a relief.4. Physiotherapy - Interferential therapy (IFT) to your back will help to relieve the pain5. Avoid driving, leaning forward, sitting for prolonged time or lifting heavy weights. These activities can increase back pain.He can start with swimming and mild aerobic exercises. These do help a lot in the back pain. Do adequate warming up before starting the exercises daily.Taking calcium and vitamin D 3 supplements will also help in early recovery. He can get these prescribed from orthopedician.I do hope that you have found something helpful and I will be glad to answer any further query.Take care"
},
{
"id": 222691,
"tgt": "What causes cramps in groin region during pregnancy?",
"src": "Patient: im in the 6th month of pregnancy and experience very bad catch/cramps in the groin region wenever i get up from a sitting position lasts for about 10-15 mins before i can straighten up and walk straight is that normal and wat shud i do to prevent them. Doctor: Hi, I understand your concern. The cramps/catch in groin while walking/ straightening up .. is mostly due to pressure on nerves in groin by growing fetus .. this in turn leads to muscle pull causing the symptom. This usually disappears when the baby grows.. & occupies abdominal cavity.. to return back as baby' head is fixed in pelvis in 9th month. Avoid sudden change in posture, Take vitamin B12 supplements - Mild yoga exercises done under expert guidance would help ( by making body more stretchable & flexible) Thanks."
},
{
"id": 202254,
"tgt": "What does presence of fructose,pus cells:5-6hpf and epithelial cells:2-3hpf suggest?",
"src": "Patient: I am 26 years old. I had a semen analysis done. The report says,Volume : 3.0 mlColor : Gery WhiteAppearance : CloudyViscosity : NormalReaction : pH-8.0Liquefaction Time : 40 MinsFructose : PresentSperm count : 80 mill/cummActively Motile : 35%Sluggishly Motile : 25%NON Motile / Dead : 40%Abnormal Forms : 10%Pus cells : 5-6/ hpfEpithelial cells : 2-3/ hpfWe are trying for a baby for the last 5 months and have had no success. Am I infertile? Please advise. Doctor: Hello Thanks for writing to HCMYour semen analysis report suggests infection.You are not infertile.Fertility will definitely improve with control of infection.Infection is indicated by the presence of pus cells in semen. Normally there shouldn't be any pus cells in semen.Infection may be due to prostatitis,UTI etc.You need antibiotics for infection.You need few more investigations like routine hemogram,Semen culture and sensitivity,RBS,Ultrasound of pelvis should be done.Trans rectal sonography of prostate can be done if needed.Proper antibiotics should be prescribed after culture and sensitivity report.Other factors like sperm count,motility and morphology are within normal limit.Get well soon.Hope I have answered your question.Take CareDr.Indu Bhushan"
},
{
"id": 78558,
"tgt": "Could I have rheumatoid in my lungs which does not show up on blood work?",
"src": "Patient: I have a lung problem. My Rheumatologist thinks rheumatoid has settled in my lungs. Had rheumatiod for 16 years, but blood test are all negative. Dr. can't tell what it is by X ray. I will see a thorasic surgeon to tlak about a biospy. Could I have rheumatoid in my lungs and it not show up on blood work? Doctor: Thanks for your question on Health Care Magic. I can understand your concern. Yes, it is very common to have Pulmonary manifestations of rheumatoid arthritis (RA) without abnormal blood reports. And to confirm this you need to consult pulmonologist and get done 1. Clinical examination of respiratory system 2. PFT (Pulmonary Function Test). 3. HRCT (high resolution CT) thorax. RA can cause bronchitis, fibrosis, bronchiectesis etc. So PFT is needed for the diagnosis of bronchitis. HRCT thorax is needed to rule out fibrosis and bronchiectesis. So consult pulmonologist and discuss all these. Hope I have solved your query. I will be happy to help you further. Wish you good health. Thanks."
},
{
"id": 17612,
"tgt": "What are normal blood pressure levels?",
"src": "Patient: Hi I have a BP monitor at home . Right now my reading is 130/86. I have a Bicuspic valve. Today I have been put on a low dose of BP medication after my clinic measured my BP at 151/113. This was taken immediately on arrival at the clinic after walking from the car park and a rather stressfull car journey. Am I right to be concerned about the starting medication on this evidence? .. I have recently had my personel BP monitor calybrated ..AGE 50 thank you Doctor: Hello, After going through your medical query I understand your concern and I would like to tell you that considering your age you can be hypertensive but if your BO stays less than 140/90 then you may not require and antihypertensive drugs. Kindly monitor your blood pressure at different times at home and discuss again with your treating physician. Hope I have answered your query. Let me know if I can assist you further. Regards, Dr. Bhanu Partap, Cardiologist"
},
{
"id": 204956,
"tgt": "How can ADHD in a child be treated?",
"src": "Patient: My 6 yr old son has scored for ADHD on the Vanderbelt ? scoring. He has more focus - attention than hyperactivity. He does fidget, play with his pencil, etc at school and home. Which medication is best to help him with concentration/focus? Doctor: HelloIn my opinion, atomoxetine or methylphenidate can help your child. Consult your doctor for right doses for your child. (prescription drug).Thanks"
},
{
"id": 119684,
"tgt": "Suggest remedyfor bruises in buttocks",
"src": "Patient: WORRIED!! I fell down the stairs a week ago. Hurt my upper buttocks area pretty bad. It bruised reall bad after a few days on the spot where I landed. The bruising eventually went down my buttocks further. NOW...I have a large fatty-feeling swollen bump (like the third of a softball) on the upper area where I landed. It feels very strange and is still very sensitive if I sit on it wrong (against back of chair). It, however is not bruised but worries me. I have no insurance or I would have had it checked out. -Poor Rad Tech Student Doctor: Hello, I think you have a haematoma there which needs to be addressed, if it is not getting resolved automatically over time. Apply some hot fomentation or Seritz bath if possible. It should settle down. Make sure you're not having fever that it should not get infected it will need a surgical treatment. Hope I have answered your query. Let me know if I can assist you further. Take care Regards, Dr. Rajesh Hingwe"
},
{
"id": 160871,
"tgt": "Is Asthalin inhaler safe for a child in case of cough and cold?",
"src": "Patient: My son is 1 year 3 months old. He has been catching cough n cold frequenty (about twice every month), sometimes accompanied by fever. This has been happening for the past three months, since the weather has started changing. Up recently he was prescribed flucold n crocin. We saw a new doctor n he prescribed asthalin inhaler one puff thrice a day. Is it safe n necessary? Is asthalin addictive? Would he need to continue using it all the time? Doctor: Hi, Please do not use asthalin in every case of cough and cold. It is used primarily for the treatment of asthma and other asthma like illnesses. that too we use it for short duration and if some specific findings in chest are present. Yes, asthalin use produces side effects. you please stop it and consult a good pediatrician.Hope I have answered your question. Let me know if I can assist you further. Regards, Dr. Deepak, Pediatrician"
},
{
"id": 71152,
"tgt": "What cause random hiccups with squeak?",
"src": "Patient: hi i have like 3 random hiccups at least twice a day. and it also doesn t feel much like a hiccup either. its more of a squeak. its completely unexpected and very loud - its slightly different and sometimes its quite irritating to my diaphragm. is there a medical word for this ? Doctor: Hello and Welcome to \u2018Ask A Doctor\u2019 service. I have reviewed your query and here is my advice. By your history and description, possibility of stress and anxiety related tics is more likely. So better to consult psychiatrist and get done counselling sessions. Try to identify stress inducing factors in your life and start working on it's solution. You may need anxiolytic drugs (Propranolol and Flunarizine combination) too. Don't worry, you will be alright with all these. Avoid stress and tension, be relax and calm. Hope I have solved your query. I will be happy to help you further. Wish you good health! Thanks."
},
{
"id": 178801,
"tgt": "What causes frequent stools in baby?",
"src": "Patient: My 9 month old baby is very active. No fever, cold, crankiness. Feeds on solid from 6th month onwards regularly. But from past 8 days she is passing stools about 4-5 times aday. Her stools are normal sometimes and sometime its loose and creamy. What is the reason for passing stools so many times?? Doctor: Thank you for posting your question.Increased stool frequency may occur normally in a lot of babies and may depend on the type of diet consumed. Also since you mentioned that the stools are mostly of normal consistency, it is most likely to be a normal occurance. However, considering the age, it could also be a mild viral infection of the intestine. In any case, all that needs to be done is keeping the child hydrated by giving plenty of liquids. You may breast feed more frequently, use oral rehydration salts or home made lemon water/rice water etc. with a pitch of salt. Giving curd or any probiotic sachets like econorm may also help.Also keep a watch for any decrease in the urine output, blood in stools or fever. If present, a medical consultation would be needed. Continue with normal diet and feeding. If you have any more doubts, do write back."
},
{
"id": 133631,
"tgt": "Suggest treatment for muscle weakness due to multiple system atrophy?",
"src": "Patient: I have severe muscle weakness due to multiple system atrophy. Previously known as Dry Shagers. Would steroids be a possible treatment for building some muscle strength? I know this not a cure, I am looking for something to help during my downward cycle. Please let me know if you have any questions. Thank You, Loren Doctor: Hi,Thank you for providing the brief history of you.A thorough neuromuscular assessment is advised.As your muscles are weak and do not have much strength, ofcourse steroid can help build the muscle fibres in the muscular system, but understanding that what actually is your muscular strength weakness root cause will help further for the choice of medicinal treatment.Undergoing physical therapy in muscular dystrophy helps majority of patients to regain the strength and physical abilities to ensure the good health.RegardsJay Indravadan Patel"
},
{
"id": 18394,
"tgt": "What causes tingling sensation in the left arm while having migraine and high BP?",
"src": "Patient: My boyfriend has been having an uncomfortable tingling sensation in his left arm a few nights a week for months and it keeps him up at night. We are trying to figure out if it is serious as we do not have health insurance and a trip to the emergency room would be expensive. I am concerned however because he gets migraines often accompanied by high blood pressure when he is stressed out. He is 29 years old and has no prior health conditions. Doctor: Welcome to ' Ask a doctor ' service . I have reviewed your query and here is my answer . Just get his ECG and blood pressure checking and X-ray cervical spine ap and lateral view . These should be normal . Otherwise take any over the counter neurotonic . I hope I have solved your query. Let me know if I can assist you further in this query. With Regards, Dr Varinder Joshi"
},
{
"id": 4782,
"tgt": "Condom broke during sexual intercourse. Chances of pregnancy?",
"src": "Patient: 2 days ago I had sexual intercourse. I've been pooping orange the past two days. I'm not sure of its something I ate. But I've been eating noodles these past days and I'm not sure if it that or an early sign of pregnancy. I had protected sex but condom broke. My boyfriend still don't come in me. And it broke a few seconds after checking if it was fine. I think I'm just paranoid Doctor: Hi,If there was no ejaculation and if you are not around your ovulation period, you need not worry about conception. It could be something that you ate. Please wait for your next cycle and if you miss it, go for a urine pregnancy test 3-7 days later. If you wish for an earlier diagnosis, you can opt for the serum beta-hCG estimation about a week before your next period and not earlier. This test is more sensitive and specific. Hope this helps."
},
{
"id": 38355,
"tgt": "Should Clindamycin for infection be discontinued due to abdomen pain?",
"src": "Patient: Hello my name is Ashley I'm taking clindamycin for infection and my upper abdomin hurts off and on. I have been taking it for 3 days called my doctor and he never replyed they just told me to make an appointment I just need to know if I should stop using the medication until I go to the doctor in the morning Doctor: dear, clindamycin is an antibiotics, it can cause gastric irritation & burning sensation. these are just side effect. for these you donot need to discontinue the antibiotic. On the other hand you should start taking an antacid before meals to avoid the pain. take care."
},
{
"id": 42426,
"tgt": "Can I conceive with PCOD?",
"src": "Patient: I am 28yrs old, ultrasonography report gave impression of multiple tiny peripherally located follicles with central echogenic stroma in both ovaries.. i had my last period on 17th march, nxt april month there was no period.. i am trying to conceive.. can i conceive.. ? how many days are required for treatment of PCOD? Doctor: Hi,Welcome to HealthcareMagic .Yes you can conceive with pcod. Pcod treatment takes around 3 to 6 months.I would suggest you to go for ovulation inducing drugs like clomephine . This will help multiple follicles to mature and rupture . Thus increasing chances of pregnancy .Hope I have been helpful .RegardsDr.Deepika Patil"
},
{
"id": 162390,
"tgt": "What causes stomachache and feet pain in 3 year old?",
"src": "Patient: My 3 year old is complaining of her belly hurting and her feet hurting really bad. She is running a low grade fever also. Plus about 3 weeks ago she was coughing really bad and coughed up brown blood. What should I do? She cry so hard from her feet and tummy hurting she will hyperventalate. Doctor: Hello, As your baby has stomach pain and low-grade fever, so it may be a stomach infection. Kindly give him syp Mefenamic acid, syp Paracetamol, syp Ondansetron as per his weight after pediatrician consultation. Give him hygiene water and food. Keep him hydrated. Hope I have answered your query. Let me know if I can assist you further. Regards, Dr. Shyam B. Kale, General & Family Physician"
},
{
"id": 176622,
"tgt": "Can Sinarest be given along with Paracetamol?",
"src": "Patient: Sir my 5.5y daughter suffering from fever .we have 1.paracetamol sirp ...125 mg 2 amoxicillin tablet. ...500 mg 3.Sinarest.... From last night I given half tablet of amoxicillin and 125mg paracetamol to her but there is no any improvement seen. Plz suggest that can I given sinarest 5 ml with paracetamol Doctor: Syrup Sinarest P itself has Paracetamol in it.So there is no need to give yor child PCM. If the preparation of sinarest that you have , is different from Sinarest P, then it is required to give paracetamol with it. Dont give your child Amoxycillin as of now. It will only cause gastric problems and lead to drug resistance in the bacteria. I suggest , you buy Sinarest P and give her 5 ml thrice a day.Thanks and regards."
},
{
"id": 179401,
"tgt": "Suggest treatment to gain weight and vomiting in new born",
"src": "Patient: My baby is 3 month old...as he was a sick baby when he was born...he is in under weight..I want to gain his weight...and also the main problem is my baby is vomiting milk immediately after eating...I consult to the dr,he said it s normal but he is not gainning weight..as I ve not sufficient breast milk,for that dr told me to give him lactogen..but still he is vomit.. 1.how to stop vomitting? 2.how to gain his weight? 3.what formula of milk should I taken? Doctor: Hair I'm Dr. SubramanianWelcome to HCMThe complaints which you have are often seen in preterm and low birth weight babies.This is not a serious issue unless you take proper care.Adequate burping after each feed is vital.If at all your giving lactogen prepare it in right proportion.Mix one scoup of the powder to 30ml of Luke warm water.In order to increase weight add Simyl MCT oil 1ml to each feed.Kangaroo mother care is important in under weight babies.Put your baby over your breast and abdomen, so that the baby gets your warmth. Do this for 30mins thrice a day.Take care.She will grow well and gain weight."
},
{
"id": 148265,
"tgt": "What could cause chronic heating sensation on the back of the head (right) despite tests showing all clear?",
"src": "Patient: Hi, I m feeling this strange, anoying and strong heat on the right side of the back of my head for 5 or 6 years now. I m 23 now. It starts in the noon or around 1 pm and lasts until I fell asleep. It s really making me miserable and depresive. If I take a nap in the afternoon it disapears after I wake up. Also, as soon as I m not in a closed space, as soon as I m breathing in some fresh air, it disapears. It also stops when I m showering my head with ice cold water for a couple of minutes. I ve been to all kind of doctors, tried everything, but nothing helped. All tests show that nothing is wrong with me. Please help me! Doctor: Hi,Thank you for posting your query.First of all, I would like to reassure you that your symptoms do n to suggest nay serious neurological disease.The most likely cause for your symptoms is vascular headache something similar to migraine.You should be started on preventive medications such as topiramate, divalproex or beta blockers, so that the recurrence of symptoms can be prevented.I am sure MRI brain was done in your case and it was normal.I hope my answer helps. Please get back if you have any follow up queries or if you require any additional information.Wishing you good health,Dr Sudhir Kumar MD (Internal Medicine), DM (Neurology)Senior Consultant NeurologistApollo Hospitals, Hyderabad, IndiaClick on this link to ask me a DIRECT QUERY: http://bit.ly/Dr-Sudhir-kumarMy BLOG: http://bestneurodoctor.blogspot.in"
},
{
"id": 84884,
"tgt": "What are the side effects of clopilet a?",
"src": "Patient: My father with 60 years old suffering with paralysis and heart problem.doctors guided with clopilet a.BUt he is fealing wamting sensation and gastric problem.he dont want to eat any thing.In past also he faced same issue with the tablet.is ther any suggesions. My email id is: Doctor: Hi, The side effects of Clopilet A include: Headaches Dizziness Nausea Diarrhea Black tarry stools Indigestion (dyspepsia, abdominal pain, nosebleeds, increased bleeding tendency, easy bruising and at times intracranial hemorrhage) Hope I have answered your query. Let me know if I can assist you further. Regards, Dr. Dorina Gurabardhi, General & Family Physician"
},
{
"id": 37958,
"tgt": "Suggest remedy for pain,itching and swelling from bee bite",
"src": "Patient: I got stung by a bee (not sure what kind of bee) two days ago. The First day I just experienced pain and swelling on the finger. Second day, it has spread to the neighboring finger and it s getting really itchy; on top of that, my pinky finger where I originally got stung is turning purple. What should I do to stop the pain, itching and heal it as quickly as possible? Doctor: Hello, Thank you for your contact to health care magic. I understand your concern. If I am your doctor I suggest you to start with chlorpheniramine maleate for itching, cefixime tablet for infection, Aceclofenac and serratopeptidase for pain and healing. Maintaining local cleanliness and regular dressing with neosporin powder and betadine will help you further. I will be happy to answer your further concernYou can contact me. Dr Arun Tank. Infectious disease specialist. Thank you."
},
{
"id": 65191,
"tgt": "Need medication for lump on the upper back near spine",
"src": "Patient: Hello, over the last few years I have had a lump on my upper back near spine. It hurts on and off but recently has been causing a lot of pain. Not so much that I cannot lay on it but it hurts to touch. I am going to the doctors tomorrow but would like some idea of what to expect. Laura, 14. Doctor: Hi, dearI have gone through your question. I can understand your concern. You may have lipoma, neurofibroma or dermatofibroma. You should go for fine needle aspiration cytology or biopsy of that lump. It will give you exact diagnosis. Then you should take treatment accordingly. Hope I have answered your question, if you have doubt then I will be happy to answer. Thanks for using health care magic. Wish you a very good health."
},
{
"id": 89177,
"tgt": "What causes pain in the abdomen after an orgasm?",
"src": "Patient: i have pain in my left abdomin after an orgasm. Last for like 20 minutes. What is causing this. I am 27 years old and i have anxiety and a high white blood count. I also get migrains all the time and see a nuerologist and a hemotologist. I also am going for a gi scope tuesday for a lump in the middle of my throat. Doctor: HI.The reason for pain in abdomen after an orgasm can be due to PID, Tubo-ovarian mass or so. Get an ultrasonography; Gynecologist's opinion and internal examination. This may give the internal problem which may need a medical or surgical treatment. Anxiety adds to all the problems you have. It has to be well controlled by self-help / medications. Continue treatment with Neurologist and Hematologist. Go for proper investigations. Let us know the findings of GI scope. Wishing you an early and permanent recovery."
},
{
"id": 87648,
"tgt": "What is the treatment for severe abdominal pain?",
"src": "Patient: I dont know whats wrong with me that I had ve a pain in left abdomen, it happens long times ago and gone, but now it start again, the pain is not severe but whenever I move it feels, and sometime pinch near to the heart, even I findout blood on my stool, I was surprise, thought it may be my pile problem, and I used hadensa pile cream, after that my stool was clear with blood, but still ve sensation pain inside my abdomen, today again I timeout little jelly way of blood in my stool..... I had consult softer and he ask me to do some test like ultrasound for whole abdomen, x- ray, blood test, urine test but the results were all normal, so he prescribe me pantest tab, myzyme syrup as we ll equirex tab, intact I didn t find any relief, please do suggest me. Doctor: Hi. Thanks for your query and an elucidate history.With the history of pain inthe left side of the abdomen with blood and mucus in the stool the probable problem is obviously colitis. That is why the reports you did are normalI would advise you top take a course of Metronidazole, continue the medicines of Equirex, panjtest, myzyme to continue. If there is no relief within 5 days, you need :CT scan of the abdomen _Colonoscopy and biopsyThis will clinch the diagnosis to get you the correct line of treatment. If it is cancer or so you may need a Surgical treatment."
},
{
"id": 190518,
"tgt": "Peripheral giant cell granuloma, recurring cell growth. Taking steroid injections. Had mouth surgery. Reason?",
"src": "Patient: I recently had mouth surgery to remove a benign growth called a perifrial gaint cell granulnoma. Over the span of the last three months I have been receiving streroid injections at the surgery site. After the first injection, it appears the growth has returned. In all I have had three injections. The doctor thinks the gaint cell granulnoma has grown back. I am scheduled for a ct scan to confirm his diagnosis. He also did state it may possibly be build up of calcium from the steroid injections. Is there any hope at this point, regardless if it is a re occurance or calcium build up to avoid surgery? Thanks, Joshua Doctor: Dear Joshua peripheral giant cell granuloma has recurrence rate of about 20%. if at all it has recurred, surgical removal is the only option. during surgical removal if the granuloma is not removed completely then chances of recurrence are more. surgeon this time would take extra care not to leave any bit of tissue which can result in recurrence. take care. Dr.Avinash"
},
{
"id": 126305,
"tgt": "Suggest treatment for swelling in the calves",
"src": "Patient: My daughter age 46 suffers from a leg/calf issue for many years. her calves look swollen. She eats right, drinks plenty water and walks a lot, but the doctors cannot figure it out. Some say: lymphedema lipedema Abnormal distribution of fatty tissue......please help She is only 46, no children. Doctor: Hi, Consult an orthopedician and get a Doppler scan done for further assessment. Hope I have answered your query. Let me know if I can assist you further. Regards, Dr. Shinas Hussain, General & Family Physician"
},
{
"id": 144487,
"tgt": "Suggest remedy for persistent dizziness and strong metallic taste in mouth",
"src": "Patient: For about 3 weeks, and at random times, I get dizzy, have a strong metallic taste in my mouth, feel nauseated and of all things, my teeth will hurt at the same time. As if my jaw line gets tighter. Weird and causes me to stop everything and just lay down. Doctor: Hi welcome to HCM I have gone thru your query regarding dizziness and strong metallic taste in mouth .It is a matter of concern ,Dear , we face health problems , due to our faulty food habits and derailed life style . Lack of essential nutrients - vitamins , minerals , antioxidants fiber in natural form or accumulation of toxins in our blood can create health problems , as, dry mouth, medications, regular spoiled foods, a mild health condition, such as regular constipation , common cold, or a serious condition, such as poisoning or cancer can cause metallic taste in mouth . Deficiency of Iron (anemia with fatigue and paleness) Zinc (anorexia, fatigue, diarrhea) , Copper (fatigue, paleness) ,Vitamin B2, B3, B6 or B12 (bright red tongue, cracked lips in the corners of the mouth, scaly skin, pernicious anemia, weakness) , all can cause dizziness and strong metallic taste in mouth and affects over all health , dental health also . Our health depends upon physical activity level and the diet we take. Strengthening your immune system by Modifying your life style and food habits is the fist step towards your healthy trouble free life , Include essential nutrients - vitamins , minerals , antioxidants fiber in natural form to eradicate toxins and for proper blood circulation , regular physical activity is essential for long lasting healthy life .To get rid of metallic taste treat the cause . Sugar-free mint or eucalyptus chewing gum can help mask the metallic taste. Homeopathic Pilocarpine 30 , 2 drops in 3-4 table spoons of water ,3 hourly 3 times .Consult your dr, if problem persists . She / he may advise examination by a dentist, an ear, nose and throat specialist, gastroenterologist or neurologist needed to determine the cause of a metallic taste .Hope this helps solve your query Take care All the best"
},
{
"id": 100263,
"tgt": "Suggest alternate to antibiotic to cure sinus congestion and breathing problem",
"src": "Patient: Hi, I had been suffering from sinus in the past and taken medicines for the same. This time again I am feeling the same .. blocked nose, difficulty in breathing and phlegm, headache , swelling and pain in eyes . Multiple episodes of sinus results into antibiotic everytime.. Is there any alternate to antibiotic for my problem.. please guide.. Doctor: Hello,Thank you for asking at HCM.I went through your history and would like to make suggestions for you as follows:1. First of all, it is important to know the cause of the sinusitis. Causes can be allergic or mechanical obstruction (like deviated septum of the nose, etc).2. I would suggest you to consult an ENT surgeon who will examine the nose from inside and may identify any such physical cause of sinusitis. He may suggest you investigations like CT scan of sinuses. He may also identify clues to other causes of sinusitis like allergies by examining the nose from inside.3. I usually suggest my such patients allergy testing which helps to identify the substances causing allergies.4. Intranasal corticosteroids, antihistamines like cetirizine, nasal decongestants, oral corticosteroids, etc are drugs other than antibiotics which are used to treat sinusitis. But these can be suggested only after thorough evaluation of your sinuses.5. Simple measures like regular steam inhalation and nose irrigation with saline are also very helpful.6. In general, I would suggest you to take plenty of warm fluids during day and also to avoid exposure to dusts, smokes and air pollution as much as possible.Hope above suggestions will be helpful to you.Should you have any further query, please feel free to ask at HCM.Wish you the best of the health ahead.Thank you & Regards."
},
{
"id": 119770,
"tgt": "How to treat sebaceous cysts?",
"src": "Patient: I have a 5 cm sebaceous cyst (I assume) between my shoulder blades. It has flared up from a small painless lump to red and very painful in last 14 days. Can a sebacious cyst be drained by applying a warm herbal poultice or is it necesary to remove surgically (expensive, no insurance)? Doctor: Hello, It is better to remove it by surgical excision. Natural remedies may not be effective. It a simple procedure and can be done under local anaesthesia. Hope I have answered your query. Let me know if I can assist you further. Take care Regards, Dr. Shinas Hussain"
},
{
"id": 21098,
"tgt": "What causes intense heart beats in a 28 years old?",
"src": "Patient: i have had a condition for the last 13 month now whereas it feels like a drop beat in my chest which happens everyday.but for the past 4 days these beats in my heart have been happening every 10 seconds. i got really scared and called an ambulance. i was on the heart monitors for 5 hours in the hospital and it was picking up my proble3m. the doctor said i am getting extra beats from the bottom of my heart and sent me home because their is nothing they can do for me. they have said nit is harmless but the problem is i am really scared and the beats are so intense. is this normal for a guy my age at 28 to be experiencing this. it is making me stressed out and the fear is terrifying....do u have any advice for me.......please help Doctor: Hello! understand your situation, but you don't have to worry. With what you refer, it does not look a serios problem. The problems with heart beat can have different causes. In your age, the possibility that this comes from organic factors, such as a heart disease, is very low. I think that your situation is transitory. However, if the problem persist for a long time and becomes more frequent and disturbing then I would recommend that you consulte a cardiologist and make some examinations to evaluate your cardiac rhythm and function, such as ECG, echocardiography and a holter rhythm 24 hours. After these test's redults and evaluating the heart rhythm and it's nature, the cardiologist will take a decision to start or not any medication. I hope to have helped you with my answer.I wish you good health!Thank you for using HealthcareMagicYou are welcome to write again if you have other questions.Regards,Dr. P. Cerri"
},
{
"id": 8626,
"tgt": "Disfigured bump on lips looking like cleft palate after stitching up due to accident. Interferes with eating. Guide for rectifying",
"src": "Patient: I was in a horrible car accident on my bicycle, was hit at 30+ miles an hour, flown from the bike and my face scraped the concrete. When I regained consciousness, my lip was cut in half! My lip is the major problem, after having it operated on twice, both doctors did a horrible stitching job (they're not beauticians). My lip was left with a bump on it and is visible when I smile. It looks like I have a cleft lip! I feel disfigured, and am extremely self conscious about it. Sometimes it interferes with eating, and my teeth scrape it. What can I do? Can I go to the emergency room to fix it? Cut off the excess flesh? I could bite it perhaps? These are the insane ideas I've had to live with and contemplate. Could I also be suffering from head trauma? Or accident related trauma? I've been in six car accidents in my life, three on bikes and I am deathly afraid of careless drivers. Doctor: Hello,The problem you are complaining of arises from an improper repair of the muscle inside your lip which is known as orbicularis oris. Revisional surgery entails a complete disruption of the repair, stitching the lip in 3 layers with resection of any mucosa that might occur in your case and with particular attention to repair of the orbicularis. Please visit a plastic surgeon for more details, there is no need to be depressed.Dr. Adhikari"
},
{
"id": 7970,
"tgt": "What can i do to remove the red acne scars ?",
"src": "Patient: hi i am 17 years old i had acne before 2 months now it is all cured but i am left with severe red marks all over my face and there are also some holes formed how can i get rid of this?? i used kojic acid cream for more than a month but of no use please help.... Doctor: Welcome to Healthcare MagicYou seem to be having post-inflammatory scars. This can be relieved by trial with chemical peel. If not working microdermal abrasions, laser therapy may help. Scars may not heal in all instances for which you may require skin graft in severely damaged areas. Consult your Dermatologist for the options available to you."
},
{
"id": 190100,
"tgt": "Abscessed tooth, facial swelling, tooth numbness. Unable to get the tooth removed. Cure for symptoms?",
"src": "Patient: Hi my name is XXXXXX. I am concerned about my mother, she has bad teeth and now has an abscess tooth which was giving her the worst pain. This morning she woke up face is still swollen but her face and tooth is numb now. That tooth has always been abscess and has never become numb so she is very worried. She doesn t have the money to get it pulled or even go see a doctor for that matter. I wanted to ask a professional their opinion on what this could be, so we can take see what else there can be done for her. Thank you for listening and hope to hear from you shortly. Doctor: Hello Welcome to HCM I read your query & would like to tell you that all symptoms suggest that there is abscess formation. It occurs when infection reaches pulp & causes inflammation in periapical region. I would suggest you to get her treated as early as possible after consulting your dentist. It might require RCT or extraction depending on clinical & radiological condition. Also start with medicines prescribed by your dentist. Ask her to do betadine gargles twice a day & avoid doing warm compresses over swelling. Take Care Regards Dr.Neha"
},
{
"id": 176804,
"tgt": "What causes itchy reddish ears and cheeks?",
"src": "Patient: At night my son, 10yr old gets one or both ears and sometimes his cheeks red, hot and itchy. Mostly it is at night but sometimes during the day as well. He has also been complaining of stomach ache due to gas, if it is at all related to this. Please help explain the red and hot ears. Doctor: Hi,from history it seems that he might be having some allergy, either internal or external giving this problem.There might be possibility of having worm infestation giving rise this allergic manifestation over ears and cheek and having stomach ache.Give him one course of anti-worm medicine like Albenmdazole.Give Cetrizine or Benadryl for allergic rashes.Ok and take care."
},
{
"id": 12755,
"tgt": "Suggest treatment for red rashes on inner thigh",
"src": "Patient: I have a flat, red rash - almost looks like a bruise - on my inner left thigh. I first discovered it two days ago. Today it looks like its spreading just a little. It doesn't itch. It isn't painful. If I didn't see the rash, I wouldn't know there was any reason to be concerned. What should I do? Doctor: Hi Dear,Understanding your concern. Red rashes on inner thigh could be due to fungal infection of thighs mainly Tinea, as it localizes on body folds and thigh area.Need not to worry. I would suggest you to start with oral tablets of Terbinafine for 1 week. You should clean the skin with soap and cold water and apply petroleum jelly. Wear loose cotton clothes. You should visit dermatologist once and get it examined and start treatment after proper prescription. Take tablet of cetirizine 10 mg once daily for symptomatic relief from itching.Hope your concern has been resolved.Get Well Soon.Best Wishes,Dr. Harry Maheshwari"
},
{
"id": 135541,
"tgt": "Suggest remedy for sarcome synovial in knee",
"src": "Patient: My grand daughter has a sarcome synovial BEHIND THEW KNEE she was operated but now they want to amputate , I waould like her to see another doctor just to know if another treatment could be given to her. Excuse my \u00c9nglish i am French .Thank you! Doctor: Dear Grandparent.I understand your concern about amputation. I will explain the line of treatment first. Since Synovial sarcoma is an aggressive malignant tumour the current line of treatment is surgery. The type of surgery -that is how far should we operate depends on the site or location of the tumour. As you have mentioned behind the knee, this means that the popliteal fossa is involved. Synovial sarcomas in such locations spread rapidly through the blood and lymph channels to the lungs. Therefore depending on the location most oncosurgeons prefer amputation in order to save the life of the patient. Please remember that synovial sarcoma is very aggressive, grows and spreads fast . I do not have the biopsy report of your grandchild but from the description you have given I presume that after the first operation, the pathological report and surgical findings may how shown involvement of the popliteal fossa and fascial planes. You may request another specialist opinion if you wish but please do not delay the operation. You may also post the details of the histopathology report for further assistance and guidance. You may also refer websites such as Merck for patients or WebMD which explains further details of this disease in simple English. You may use google translate to convert such explanations into French Best regards"
},
{
"id": 158577,
"tgt": "Painless lumps on the neck, fatigue. Had a surgery on the nose. Family history of lymphoma. Worried",
"src": "Patient: Hi i am Heidi a 37 yr old female i have found 2 small lumps on the left side of my neck. They are not painful approx size is about 2cm for each. I have no other symptoms apart from fatigue but that is quite normal for me not a recent symptom. I have recently had ENT surgery on my nose and am on a preventative course of antibiotics so i should not have an infection currently . I have a family history of Lymphoma my Auntie died from this several years ago. Doctor: Hi, The duration of the lump should be mentioned. Initially nothing to worry about. This may be due to infection though infective lymph nodes are generally painful. Anyway continue the antibiotic course if after two weeks the size does not regress then other possibility to be ruled out. Do not think about lymphoma right now because there are other common diagnosis to be ruled out first."
},
{
"id": 185711,
"tgt": "Suggest alternate method to floss between teeth",
"src": "Patient: A few days ago I had some fillings in between my teeth. However, with one tooth, I'm afraid to floss. I always end up tearing the floss with that gap and want to know if I shouldn't floss that gap and should do something else instead if there is an alternative. Doctor: thanks for your query, i have gone through your query. you can floss if there is any catch it means there is some overhanging restoration consult your dentist and get the filling replaced. if the filling is fine then you can use interdental brushes that will help in cleansing the teeth. i hope my answer will help you take care."
},
{
"id": 111911,
"tgt": "Can thigh pain be related to back pain?",
"src": "Patient: I recently had spasms in my lower back that caused pain in my thigh, front side. I took muscle relaxers and the back pain subsided. I now have really bad pain in the front of my thigh that is extremely painful to touch. The pain keeps me awake at night. Is this related to the back pain or something else. Doctor: Hello I have studied your case Due to compresion of nerve root in back there can be radicular pain in your thigh. I will advise to do MRI spine to see for disc bulge. Neurotropic medication can help to reduce symptoms. Physiotherapy will help to get pain relief. Wish you good health Take care."
},
{
"id": 26634,
"tgt": "What causes heart racing, spasm sensation and elevated BP when falling asleep?",
"src": "Patient: When just falling off to sleep, my heart starts racing and pounding, spasm sensation to right of sternum.Arms, hands, legs and feet are hot, BP feels elevated. Left side of head, face feels numb and hot,Arms look redder. Just can't cross over to sleep without this happening. Doctor: Hello. Thank you for your question and welcome to HCM. I understand your concern. Most of the people sense the heart beat as normal and understand this and accept it as a normal, physiological phenomenon. Some people, like yourself, sense this as a pathological phenomenon. I understand that. Every minimal change in perceived \"physiological\" or normal functioning, is interpreted like pathological things. This is a cause for anxiety or fear (situational anxiety) in these people and may or may not be followed with a panic attack due to one questioning him/herself \"am I going to die?\" \"what is wrong with me?\" etc. Every time we assume a lying down position, our heart, that beats by vibrating our chest wall, comes closer to a certain area of the ribcage. If lying on the left side it is so close to the chest wall that can even vibrate the bed you are lying on. All the other symptoms you mentioned are triggered by the anxiety attack you have, because all types of anxieties have one thing in common: flush of adrenalin and noradrenalin in the system, which then produce the symptoms you perceive. Every thing you feel is real, but it is not pathological. Therefore, if the heart rate is regular, every symptom is attributed to the anxiety, which I depicted above. If this is the case, then there is nothing to worry about. But, if this phenomenon affects your quality of sleep, by reducing its hours, then I would recommend a consult with a psychiatrist about the possibility of being put on a trial with sedatives for one to two weeks. I hope I was helpful with my answer. Take care. Kind regards, Dr. Meriton"
},
{
"id": 143709,
"tgt": "What could cause drowsiness, choking and swallowing difficulty, have history of stroke?",
"src": "Patient: Several times lately As I am falling asleep I can t swallow and mouth is very dry. It feel I will choke to death.I get ice chips to add moisture to mouth. This is a scary paralyzingly feeling. It last an hour or two. I had a sleep test last week, a throat X-ray with barium today and will have a scope down throat next week. 2 of these occurrences were by taking a new pill after my normal meds. Once 1/2 tablet of Trazadone and another time I took 2 ibuprophen. I thought an allergy, but had a spell last Sunday nite with no extra meds. I had a stroke last summer. I am bipolar and take my reg meds each nite....no acid in throat feeling, just swallowing . Never know when this will reoccur. Doctor: Hi ,as you are suffering from bipolar disorder,the symptoms like difficulty in swallowing ,choking ,dry mouth are due to depression .I advise you to consult psychiatrist so that proper antidepressants can be started .Thanks"
},
{
"id": 181010,
"tgt": "What causes low saliva and dryness in the mouth?",
"src": "Patient: Even though I drink quite a bit of water throughout the day I've always had a dry mouth, especially upon waking in the morning. However, in the past 2-3 weeks I now feel almost as if my saliva production has shut off - drinking water and swirling it around in my doesn't seem to help for long and if I push my tongue straight out between my teeth and lower lip it feels dry as compared to the rest of the tissue in my mouth. What could be causing this and what can I do to turn it around? I'm not eating anything different and I'm not taking any different medications either. Doctor: Hi..Thanks for the query..Dry mouth or Xerostomia can be due to a number of causes like oral thrush that appears as white coating on tongue, dehydration, diabetes,side effects of medication, mouth breathing habit etc..If it is more after waking then can be due to mouth breathing habit..I would suggest you to consult an Oral Physician and get evaluated so that exact cause can be ruled out..As of now gargle with special mouthwash like Biotene..Chew sugar free chewing gum..Suck lemon drops..If this does not relieve then oral lubricants like Oralgel can help..Hope this helps..Regards.."
},
{
"id": 148993,
"tgt": "Black out, problems with sleeping. Normal CT scan, scheduled for MRI. Epilepsy?",
"src": "Patient: about a month ago i had blackout while driving the car. i felt kind of funny and i think i blacked out for about 60 seconds. thank god i didnt harm anyone or myself. i went to a and e and had ct scan no bleed. they said i had a migraine blackout. i now have to have mri scan and attend migraine clinic. could this be epilepsy. for years i havnt slept well. Doctor: hello madam thanks for the query the discription you hav given dosent fit into seizures or epilepsy...but migrane yes is possibility it will be seen in females most commonly and presents as unilateral throbbing head ache with waterring of eyes and spinning and blackout in varying combination....yoga helps for migrane and life style modification and changing food habits all these play crucial role in the treatment... thank hope i had clarified your query thank you regards"
},
{
"id": 33066,
"tgt": "Are blister like bumps on the buttocks a viral infection?",
"src": "Patient: Hi, my sisters grandson has a cluster of bumps in straight line that looks like blisters on his buttocks, one doctor thought it was shingles another one said it was a virus, neither dr. Seemed concerned. My concern is that they did try very hard to find out what it was, if it s a virus is he contagious, he has siblings and he s in school. Doctor: Hi, Is he recovering with the treatment. Shingles is a viral disese (Varicella Zoster) and it not not infective. It occurs due previous infection of nerves(latent).There is no point to worry about."
},
{
"id": 26992,
"tgt": "Is it safe to continue with the present prescriptions?",
"src": "Patient: My doctor has prescribed Simvastatin 40mg and Amlodipine 10 mg but I have read that a recent study suggests that Simvastatin should be limited to 20mg if taken together with Amlodipine. Is it safe for me to continue with my present prescription.William R Shakespeare Doctor: Hello, I really appreciate your concern. It is scientifically proven that with amlodipine, concentration of simvastatin increases significantly. So it is recommended to reduce the dose of simvastatin to 20 mg with 10 mg amlodipine to avoid side effects of simvastatin like myopathy. Fluvastatin, pravastatin, and rosuvastatin are probably safer alternatives in patients receiving amlodipine. So kindly consult your treating physician and draw his/her attention to this matter to set the doses accordingly.Hope my answer has solved your query. Take Care.Thank you."
},
{
"id": 68990,
"tgt": "Suggest remedy for foul smelling big fat lump below armpit",
"src": "Patient: i am 33 years of age n have a big fat lump just below the armpit. i have got it checked out n have been told it is a fat lump and nothing to worry about. but the odour is terrible. even after a shower, the armpit with the lump still smells....the lump is growing bigger n if i press then it hurts. what is the solution for the smell as it is getting embarrasing now Doctor: Hi,I suspect the original diagnosis is wrong and this is an abscess. I suggest you see a doctor as you probably need antibiotics and if there is no improvement you may need incision and drainage. Regards,Dr K A PottingerMBChB. FRCA"
},
{
"id": 94243,
"tgt": "Stomach pain. Minimul sigmoid colonic diverticulosis. Had MRI, CT scan, virtual colonoscopy. Help?",
"src": "Patient: For past 10 years when I push my lower right side of stomach I get pain only when I press with fingers.I am 53 years male, had mri, ct scan , test for kidney stone , hernia , appenidix and all of them were ruled out . Yet sometimes I get same side lower back pain, and kiddney pain at least that s how it seems. Had also virtual colonoscopy using a siemens sensation 4 multi-slice CT scanner, spiral CT images were obtained of the abdomen her eare findingds The colon is well distended. Evaluation of the colon is limited die to mild diffuse residual colonic fluid and stool. The well distended and evacuated portions of the colon are without stricture, obstruction, constricting lesion, definitive evidence of significant gross focal mass freater than 1 cm, or evidence of diverticulitis. The appendix is normal ther eis ill-defined focul soft tissue present in the cecum adjacent tot he ileocecal volave which liekly represents reidual adherent stool, though a small polyp cannot be entirely excluded. Minimul sigmoid colonic diverticulosis is present Is there a natural way of diet to get rid of these possibilities? Doctor: Hello Thanks for the query. You Report is suggestive of mild sigmoid diverticulosis Diverticulosis of the colon affects nearly one-half of individuals older than age 60 years. Fortunately, only 20% of patients with diverticulosis develop symptomatic disease. Asymptomatic diverticular disease discovered on imaging studies or at the time of colonoscopy is best managed by diet alterations. Patients should be instructed to eat a fiber-enriched diet that includes 30 g of fiber each day. Fiber softens the stool and helps prevent constipation. It also can help decrease pressure in the colon and help prevent flare-ups of diverticulitis. High-fiber foods include: Beans and legumes,Bran, whole wheat bread and whole grain cereals such as oatmeal Brown and wild rice,Fruits such as apples, bananas and pears Vegetables such as broccoli, carrots, corn and squash. Drink at least 8-10 cups of fluid daily. Fluid will help soften your stool. Exercise also promotes bowel movement and helps prevent constipation. The incidence of complicated diverticular disease appears to be increased in people who smoke. Therefore avoid smoking. Symptomatic uncomplicated diverticular disease with confirmation of inflammation and infection within the colon should be treated initially with antibiotics and bowel rest. Patients should remain on a limited diet until their pain resolves. For long-term medical management of uncomplicated diverticular disease, rifaximin (a poorly absorbed broad-spectrum antibiotic), is associated with 30% less frequent recurrent symptoms from uncomplicated diverticular disease. Furthermore, the use of probiotics has been shown to decrease the incidence of recurrent attacks. Hope I have answeres all your queries."
},
{
"id": 45991,
"tgt": "Does pus at the site if nephrostomy tube insertion and other symptoms indicate infection?",
"src": "Patient: my mom had a nephrostomy tube inserted about 2 weeks ago. For the past week she has been feeling very weak, tired, shes had fever, chills, shes very swollen, her urine is red, and last night i noticed that pus is coming out of the insertion site is this a sign of a very serious infection Doctor: Hello,I would like to know why your mother needed a Nephrostomy insertion. Discharge from the site of nephrostomy usually is a sign of some infection in the skin around the tube.It is generally not serious if the nephrostomy is draining clear urine. If the drainage from the nephrostomy is also purulent then it indicates infection in the kidney.As you have mentioned that she has fever and there is blood draining from the urine I would strongly recommend you to visit your treating physician at the earliest.Hope I have answered your query. Let me know if I can assist you further.Regards,Dr. Karthik Rajan"
},
{
"id": 92003,
"tgt": "What causes abdominal pain below the naval?",
"src": "Patient: My 18 year old daughter, who has a high tolerance for pain and does not complain often, is complaining of abdominal pain below the naval. Says it has been going on for at least a year. Says she is always aware of some type of pain there, sometimes mild, but sometimes sharp. Says it is definitely worse when she wears tight low-rise pants. As for food she eats a lot of salad, bananas and peanut butter, raisin bran cereal, granola bars. Doesn't associate the pain with stress. Was diagnosed with an umbilical hernia when she was 2 or 3, seen by a surgeon but told surgery not needed at the time and no issues with it since then. A year ago she saw a nurse practitioner who had a blood test done and recommended Zantac (which she did not take because she didn't feel the pain was in her stomach or esophagas.) Doctor: Hi. Gone through the history. The pain bellow the naval area should be investigated by a proper clinical examination by a Surgeon, who on examination can think of a proper diagnosis and advise further investigations like ultrasonography and so on on the basis of clinical suspicion."
},
{
"id": 201154,
"tgt": "What causes pain in the testicle and abdomen after having sex?",
"src": "Patient: hi doc, having been having strange pain in my testical and abdominal pain. I notice the pain three weeks after having sex with a woman I have been dating.Before that I have not had sex or dated for 2 years.i notice a white ,saliva like discharge.Three weeks later I started having pain in my testical and obdominal. she said she has not had sex in the last 6 months.So where did this pain comes from....Thank you Doctor: Hello..Causes pain in the testicle and abdomen in your case may be:-Infection-Stone in urogenital tractYou should meet urologist as you may require testing in form of ultrasound of whole abdomen with testis and culture-sensitivity of discharge. Then treatment will depend on investigation findings. Till that time drink plenty of waters.Take care.99doctor.com"
},
{
"id": 120077,
"tgt": "Suggest treatment for shooting pain in ankle after fall",
"src": "Patient: I fell off a chair yesterday, and hurt my ankle . Whenever I try to move it, wiggle my toes, put weight on, there is a shooting pain in my ankle, it is not swollen or bruised though . I dont have great insurance and Im pretty sure you can t do much for a sprain but Im not sure what to do. Doctor: Hello, Get the xray done on an urgent basis,const nearby hospital till than apply ice pack and diclofenac gel. Hope I have answered your query. Let me know if I can assist you further. Take care Regards, Dr. Jaideep Gaver"
},
{
"id": 144107,
"tgt": "What causes pain in back when i sneeze or cough?",
"src": "Patient: I have cyst on spinal cord so went to neurologist and he has ordered more mri with contrast, also of my brain. I have had back pain for 3 years, with an electric shock every time I sneeze or cough. now it happens more often, my pain is awful in legs , lower and upper back, bottoms of feet. I have weakness in my legs and arms. ive noticed my heartbeat is off too. its so bad I cant stand for more than 2 hours. The doctor gave me Lyrica but just started taking so cant tell if it helps. I would really appreciate your opinion. Doctor: Hi, I am Dr.Bruno. I have read your question with care and understand your concerns. Let me try to help you starting to tell the basics 1. Brain is surrounded by a Fluid Called Cerebro Spinal Fluid. This Fluid also surrounds the Spinal Cord 2. So Any Change in Pressure inside the skull is reflected around the spinal cord also 3. When We cough or sneeze, the pressure within our abdomen raises4. Because of this increased pressure within the abdomen, the pressure within the veins also raises 5. Hence the pressure within skull also rises 5. This rise is transmitted to the Spinal Cord via the Fluid So When you sneeze or cough, you get a pain in your back since you have a cyst there Hope you found the answer helpful.If you need any clarification / have doubts / have additional questions / have follow up questions, then please do not hesitate in asking again. I will be happy to answer your questions.Let me know if I can assist you further.Take care."
},
{
"id": 100794,
"tgt": "Suggest treatment for asthma and runny nose",
"src": "Patient: R/sir i have been suffered from Asthametic Problem . My nose runs every wheather.It repeats in Every 2 or 3 month contineously from 3 last year. Please let me know about this effective & permanent treatment. My health history is given below:- Age-43 year Hight-5'6\" Approx Weight- 68 Kg Approx Salt taking during-Tritublin or salbutamol,Droxollin etc Doctor: Hello.Thank you for asking at HCM.I went through your history and would like to make suggestions to you as follows:1. Were I treating you, I would prescribe you montelukast regularly in addition to your treatment. It helps in both nose and asthma symptoms.2. For nose symptoms, I would suggest you levocetirizine or cetirizine. For asthma symptoms, I would suggest you to use salbutamol inhaler on as-and-when-needed basis. 3. Personally, I would suggest you allergy testing which will identify the substances causing troubles to you and also to know how to avoid exposure to them.An Allergist-Immunologist may prescribe you allergen specific immunotherapy based on allergy report which works on your immune system to improve your allergy symptoms gradually.4. Please avoid exposure to dusts, smokes and air pollution as much as possible.5. I would also suggest you regular breathing exercise and a healthy diet rich in vitamins & minerals (adequate amounts of green leafy vegetables, fruits, sprouts, etc) which will improve your lung capacity and immunity respectively and will help you in a long run.As long as permanent/long term effective treatment, allergen specific immunotherapy may be effective in allergic asthma. All the other medications used in treatment can only control the symptoms.Hope above suggestions will be helpful to you.Should you have any other query, please feel free to ask at HCM.Wish you the best of the health.Thank you & Regards."
},
{
"id": 83638,
"tgt": "Can Metronidazol taken for bacterial infection be the cause sudden swelling in right eye ?",
"src": "Patient: I have recently started taking Metronidazol for a bacterial infection. Suddenly, the same day I started taking the medicine my right eye has started to swell. At first I thought it was just a stye and I did rub my eyes with a tissue the same day I started taking the medicine. It has been four days and my eye keeps getting worse. I have avoided makeup around my eye and am applying warm, wet compresses regularly. Is this a side effect to the medicine? I haven t had a stye in years and never this swollen before. Doctor: Hello,The symptosm are not likely to be related to the use of Metronidazole.It is more related to an eye infection.An eye infection usually goes away within a week. You may need antibiotic treatment.You may also use anti inflamamtory medication to relieve the inflammation.Hope I have answered your question. Let me know if I can assist you further.Regards,Dr. Dorina Gurabardhi, General & Family Physician"
},
{
"id": 46083,
"tgt": "Suggest kidney dialysis treatment in patients with breathing problems",
"src": "Patient: my grandmother is suffering from breathing issues due to blood clot in lungs. She is on ventilator now. Dr suggested that dialysis would be possible treatment since she have weak kidney. Now they are unable to do dialysis also because her breathing is getting critical. Please suggest can they do dialysis with the help of vent or what is the best treatment? Doctor: Dear User, dialysis need depends on the cause that is giving her breathing problems. Was pulmonary embolism demonstrated by a CAT scan? If pulmonary embolism is demonstrated, I think dialysis can't do much more. On the other hand, if there is pleurical effusion or signs of interstitial congestion, dialysis should be considered to reduce volemia (it is called ultrafiltration). Let us know,"
},
{
"id": 83644,
"tgt": "Is it safe to take aspirins and advil together?",
"src": "Patient: Hi i took 3 bayer asprins over an hour ago for some pain i had for an ear infection. and about 5 or 10 minutes ago i took an advil for sinus congestion. i remembered hearing that you should never mix them but at the time i completely forgot. should i be worried right now? Doctor: Hi,It is not safe to take aspirin and advil together. If taken together these medicines may increase your risk of developing stomach ulcers and bleeding.Contact your doctor immediately if you develop severe abdominal pain, sudden dizziness or light-headedness, nausea, vomiting (especially with blood), loss of appetite, and/or black, tarry stools. Hope I have answered your question. Let me know if I can assist you further. Regards, Dr. Mohammed Taher Ali, General & Family Physician"
},
{
"id": 221678,
"tgt": "Can non penetrative sex cause pregnancy?",
"src": "Patient: my girlfriend and i were messing around during foreplay she gave me a hand job and i kept wiping the precum off with my hand then wiped my hand on a blanket. a little later i touched her vagina skin to skin but never penetrated her (she is a virgin and not on birth control) could she get pregnant? later she gave me another hand job and i cummed when i did i put it in my underwear and stayed away from and then we went and washed our hands after words. now we are worried she is pregnant and she is just a little bit later on her period. HELP! Doctor: DearWe understand your concernsI went through your details. There is no worry about your girl friend getting pregnant if the given information is factual. Sperm needs moist conditions and prerequisite temperature to survive. In room temperature and without moist atmosphere, sperm cells die within some minutes. In your case, while having the hand job, you kept wiping the precum off with your hand then wiped your hand on a blanket. A little later you touched her vagina skin to skin but never penetrated her. In the second instance, you never even came near your girl friends vagina. From the given information, there are no chances that your girl friend can become pregnant. Don't worry.If you require more of my help in this aspect, please use this URL. http://goo.gl/aYW2pR. Make sure that you include every minute detail possible. Hope this answers your query. Further clarifications are welcome.Good luck. Take care."
},
{
"id": 114382,
"tgt": "What causes elevated WBC and neutrophils count?",
"src": "Patient: I seem to always have a high wbc and neutrophils so my dr gave me two rounds of antibiotics amoxicillin and clarithromycin we could not find an infection repeated CBC and my wbc was higher and so was my neutrophils wbc15.8 neutrophils 10 I also had a crp and it came back at 8.8 my dr said maybe this was just going to be normal for me but I\u2019m worried what do you think?? Doctor: Hello and Welcome to \u2018Ask A Doctor\u2019 service. I have reviewed your query and here is my advice. I find it relevant to dig a little deeper and see if you suffer from autoimmune disease or chronic inflammatory disease. A referral to a rheumatologist or a specialist in infectious medicine seems appropriate. Hope I have answered your query. Let me know if I can assist you further."
},
{
"id": 168598,
"tgt": "What causes allergy in 3 and a half week old baby?",
"src": "Patient: Wanting a second opinion. My 3 1/2 week old babygirl started to develop perhaps what looks like baby acne/? breastmilk allergy allover her face, behind her ears, neck and a bit on her chest. Would yousay this would be related to what I am eating/drinking? Doctor: thanks for asking i gone through your question and understand your concern,your 3.5 baby girl started developing some problem on her face,ears, neck and chest.some time baby get these type symptoms ,due to milk or dairy product used by mother . as if i am your treating paediatrician, i would advise your to stop milk and diary product. if skin problem improve when you stop these product, then stop using these products till your baby is 1 year of age and not give dairy milk or dairy product till one year of age. hope if able to answer your query"
},
{
"id": 205681,
"tgt": "Suggest treatment for stress and depression",
"src": "Patient: I was divorced about 6 years ago (no children) My ex-husband left me for another woman after 17 years of marriage (and a few more living together). I went through some very difficult years getting over that and a few other things, including a job loss, the death of a close sibling a year later (from alcoholism), and seeing my elderly mother go into a nursing home. I think I ve been over the divorce and death of my brother for a couple of years now. I don t feel that pain anymore. I ve had some good times, have a good job, I m financially ok and I have a decent life. Lonely, yes but ok. The thing is, ever since the divorce, I haven t felt truly close to anyone - not even good friends or family. I don t feel really welcome or accepted anywhere. I haven t thrown a party in 6 years. I don t call people very much and when I do I have this gut feeling that they don t want to hear from me. I rarely initiate social get-togethers. I accept most invitations (and am grateful for them), I enjoy my friends and have always genuinely liked other people. I just don t feel like I belong anywhere. I would like to understand why I feel this wat and what to do about it. It seems to be part of my wiring now. I really want to find a way to feel like I used to. If i can work this out, I think I will have a lovely life. :-) Thank you L Doctor: dear,the symptoms you happen to show are strongly suggestive of major depression.An urgent psychiatric consultation is advisable.Course of antidepressant with proper psychotherapies can be required.Everything will be ok do it worry.Regards"
},
{
"id": 114087,
"tgt": "How can i cure from TB ?",
"src": "Patient: Dear sir, i am shabnam ansari residing mumbai. i fell in pain my back bone my expert doctors findout that i am a patient of t.b i regularly treatment of t.b. dose of eighteen month.but i am not feeling now better.my esr is also increased 102 and wbc is also incresed so i cannot decides about my disease.pls. guide me a right path thanks, b Doctor: Welcome to Healthcare Magic Good Day ESR and WBC counts are high in TB infection. Are you having any weakness in your legs. The pain may be due to some deformity because of the TB. After early treatment RNTCP treatment could cure the disease. You may be patient and eat protein rich diet like pulses, egs, nuts. You should improve along with physiotherapy, but ONLY after you discuss with your Orthopaedician regarding various options to deal with this problem."
},
{
"id": 30828,
"tgt": "Suggest treatment for swelling and soreness in ear lobe",
"src": "Patient: What should my husband do for earlobe which is slightly swollen and red. There is a crusty scab-like area. A couple of days before there was no redness, swelling or scaliness. Only a very small white headed pimple in same general area. He had applied clearasil type product on the the pimple Doctor: hello there ,other than waiting for the slight inflammation to pass away there is the spirit swab that he can apply on the area for a soothing effect .. an astringent would be better perhaps..i don't foresee the lesion to become purulent so application of Clearasil is fine by me.. wait it out for a couple of days and he should be fine.. hope the suggestion and assurance helps.."
},
{
"id": 62489,
"tgt": "What causes lump in the arm where anesthesia injection was administered?",
"src": "Patient: Hi I am an ardent squash player and I had an operation for a hernia three weeks ago and haven't been able to play since. While I was having the anesthetic for the operation the anesthetist could not put the needle in as she had trouble with the vein and it took several attempts until she was successful. I had a little play of squash today just simply knocking the ball about for a practice and I felt this strange pain in my arm where I had the anesthetic and my arm went red a bit and was painful and swelled a bit. It has gone down now but there is a lump on the vein and a bruise mark like a track mark along the upper inner part of my arm from the inner elbow to about bicep area and then it stops. It is tender and sore to touch and the lump is painful to touch. Can you let me know what it may be please Doctor: Hi,I can understand you problem.Pain at the site of I/V injection is common.It is due to inflammation and known as superficial thrombophlebitis.To reduce discomfort and swelling you can start wearing support stockings, apply a warm compress to the area,took painkillers and try to give some rest to your limb.Thanks"
},
{
"id": 136608,
"tgt": "What causes warm sensation in foot?",
"src": "Patient: I I first had a warm sensation on my feet sometimes last year but after a while it disappeared without any medication. Two months ago it resumed on both feet (upper) and disappeared again but resumed last week on my left foot (mainly lower part) . No pain but I m uncomfortable with it. I m about 37 years old. Thanks. Doctor: Hello, I have studied your case. Probability of swelling and warm sensation can be due to venous congestion which gets relieved by lying on bed supine.I will advise you colour Doppler leg and EMG \u2013NCV[nerve conduction study].For these symptoms antibiotic and neurotropic medication can be started.I will advise to check your vit B12 and vit D3 level.Clinical examination is important to advise further investigation.Hope this answers your query. If you have additional questions or follow up queries then please do not hesitate in writing to us. I will be happy to answer your queries. Wishing you good health.Take care."
},
{
"id": 75505,
"tgt": "What causes difficulty in breathing?",
"src": "Patient: i am facing breathing prob past 2 months . i consult dr shetty coimbatore . i took ecg also . the doctor said he didint find any problem in my ecg report . he gave me 2 months medicin . i had all the medicin . while i am having the medicine ifeel much better . after completing the 2 month .the prob started . kindly help me Doctor: Thanks for your question on Healthcare Magic. I can understand your concern. No need to worry for heart related diseases as your ecg is normal. Possibility of bronchitis is more. So better to consult pulmonologist and get done clinical examination of respiratory system and PFT (Pulmonary Function Test). PFT will not only diagnose bronchitis but it will also tell you about severity of the disease. And treatment is based on severity only. You may need inhaled bronchodilators and inhaled corticosteroid (ICS) on the basis of PFT report.Inhaled treatment is the best treatment for bronchitis. Don't worry, you will be alright with all these. Hope I have solved your query. I will be happy to help you further. Wish you good health. Thanks."
},
{
"id": 9409,
"tgt": "Suggest treatment for exfoliation",
"src": "Patient: An 68 year old female, diagnosed to have gouty arthritis 3 months ago came to the OPD with history of dry scales all over the body since 3 weeks. There was history of fever with chills. She started exfoliating profusely, initially trunk and buttocks, gradually progressed and involved face, back and lower extremities. There was no history of evanescent skin lesions, weakness, joint pains or history of red eyes. There was no history of jaundice vomiting or abdominal pain. Systemic review of symptoms was within normal limits. On examination patient was ill looking. There was no pallor, jaundice, clubbing or lymphadenopathy. Systemic examination was normal. On skin examination, there were dry scales involving trunk, buttocks, face, back and lower extremities involving more than 90% body surface area. Skin was shiny, thickened and erythematous. Scalp skin also showed exfoliation. Hair and mucous membranes were normal.What is the most probable diagnosis on her? Doctor: Hello,I can understand your concern for exfoliation of skin. Kindly check for the exact reason for exfoliation as it can be due to varied reasons. In my patients I combine Petroleum and other oils as emollients and urea as keratolytic cream.Petrolatum and other oils are available over the counter easily. I suggest my patients to apply any of these just after bathing when the skin is still wet. It preserves the moisture in the body and therefore reduces flaking. On applying any of the creams later you should first give compressions of mildly warm water of can take a shower and then apply it. Do not use any strong soap or detergents over skin. Wear simple lose cotton clothes and avoid synthetic clothes.All the best."
},
{
"id": 127482,
"tgt": "What causes pain in the right shoulder?",
"src": "Patient: I have been having a lot of pain in my right shoulder, it even hurt to touch the area where the pain is located. It hurts in the front and on my shoulder blade. It is waking me up at night because of the pain. When I walk I hold it close to my body. I do not hold it down by my side or let it swing the way a person normally does when they walk. Doctor: Hello and Welcome to \u2018Ask A Doctor\u2019 service. I have reviewed your query and here is my advice. There is a possibility of two problems I your case 1. Frozen shoulder 2. Slipped disc at C4-5 level. To differentiate between the two you need good clinical examination and MRI of neck. Start pain killers and muscle relaxants in the meantime. Hope I have answered your query. Let me know if I can assist you further."
},
{
"id": 137593,
"tgt": "Suggest treatment for severe neck and shoulder pain",
"src": "Patient: Hello. I am 60 yrs young and have had a strange pain under my right clavicle about 2 fingers to the right of the dip, center of neck. It hurts more each day and really bad at night, especially when I go to turn over. It wakes me up. If I try to raise my right shoulder, it also causes that area to hurt. I have researched this and not sure what type of doctor treats clavicles. Can you help? Doctor: Hi,Thanks for your query.From the description, it seems you might be suffering from Costochondritis.The causes are not exactly known for this condition, but certain things trigger costochondritis. It is an inflammation of the cartilage that connects a rib to the breastbone (sternum). Costochondritis usually goes away on its own for some weeks,although in some cases it may last for several months or longer. -Try to rest and avoid activities that make your pain worse.-Take painkillers for the pain (Paracetamol,Ibuprofen etc).Visit the doctor. Your doctor will be better able to diagnose costochondritis. I do hope that you have found something helpful and I will be glad to answer any further query.Take care"
},
{
"id": 77550,
"tgt": "What causes chest heaviness and shallow breathing?",
"src": "Patient: I am female 52 years old. I have heavy shallow breathing, weakness, and dizziness once in a while. I also have mild chest pains on occasions. My ecg was within normal limits but I know there's problem in my body that's causing me to have a heavy chest and shallow breathing. Please let me know what could be causing these? Doctor: Thanks for your question on Health Care Magic. I can understand your concern. Shallow breathing, chest heaviness etc are commonly seen in allergic asthma and bronchitis. So better to consult pulmonologist and get done clinical examination of respiratory system, chest x ray and PFT (Pulmonary Function Test). PFT is must for the diagnosis of asthma and bronchitis. It will also tell you about severity of the disease and treatment is based on severity only. You may need inhaled bronchodilators and inhaled corticosteroid (ICS). If PFT is normal then possibility of stress and anxiety related symptoms is more. So if you are having stressful life then consult psychiatrist and get done counselling sessions. Try to identify stressor in your life and start working on its solution. You may need anxiolytic drugs too. Don't worry, you will be alright. Hope I have solved your query. I will be happy to help you further. Wish you good health. Thanks."
},
{
"id": 215804,
"tgt": "What causes constant pain on the lower left side of the abdomen?",
"src": "Patient: I am having a constant pain on my lower left side abdominal. Movement is painful and I have had an issue with having like a blood diarrhea for about three weeks and have an appointment to go get a colonoscopy on the 1st of November. The pain started last night, it is my only new symptom. I also have a history of Bulemia and I am a 23 year old female. I don t even now if this is real or not but I really do need some help/advice.. Doctor: Hi, Detailed evaluation is required to find out the exact cause for the pain. You can consult a physician and get an ultrasound scan done to look for any lesions or any bowel related problems. As of now you can take analgesics like Tramadol or Acetaminophen for pain relief. Hope I have answered your query. Let me know if I can assist you further."
},
{
"id": 139821,
"tgt": "Is feeling dizzy and vomiting signs of vertigo or cerebellar stroke?",
"src": "Patient: Hi. My dad had an angioplasty 3 years ago and still takes his maintenance medications today. He s about 6 ft tall, has a big tummy, around 40-42 inches of waistline. last night he experienced some dizziness while driving, and when he got out of the car started vomiting. He threw up about twice last night and then two more this morning when he woke up. he said he felt dizzy when he opened his eyes as he woke up. We got him to a doctor, they said it was just vertigo. I asked a neurosurgeon about it, and she said it might be cerebellar stroke.What could be your diagnosis for this? Doctor: Hello, His symptoms are suggestive of peripheral vertigo. Anyway, considering his past medical history, I would recommend performing a brain CT scan in order to exclude a possible cerebellar stroke, which may mimic this clinical situation. Hope I have answered your query. Let me know if I can assist you further. Take care Regards, Dr. Ilir Sharka, Cardiologist"
},
{
"id": 209174,
"tgt": "What is the diagnosis for severe mood swings?",
"src": "Patient: i just wanna tell u my sm symptoms for last 2 yrs i m feeling on and off tension like feel like crying feeling of lonliness cant concentrate on my studies and dont feel like doing anything i wanna be haapy but i cant think positively and feel like so lazy and sleepy so wat is diagnosis and how can i prevent these symptoms? Doctor: DearWe understand your concernsI went through your details. I suggest you not to worry much. You may not be depressed. I am sure about that. I certainly think that you are a teenager. Between the ages 15 and 25, the brain become so creative and vibrant. Brain wants to do creative tasks and is vibrant in everything. If you are unable to give tasks which are creative and vibrant, then it gets bored and becomes lazy. That is what you are feeling now.Don't worry. This is the case with almost 90% of teenagers. They are lazy and cannot concentrate on studies. Mainly because their tasks are mechanical according to their brain. Be active in other activities such as games and extra curricular activities to contain your brain.If you require more of my help in this aspect, Please post a direct question to me in this website. Make sure that you include every minute details possible. I shall prescribe some psychotherapy techniques which should help you cure your condition further.Hope this answers your query. Available for further clarifications.Good luck."
},
{
"id": 218077,
"tgt": "Experiencing pain on the right side of the rib cage",
"src": "Patient: Im experiencing pain in my right side under my rib cage it is swollen and i haven't done anything active to have a broken rib and if i put any type of pressure on it or when cough i have a sharp shooting pain. I'm also experienceing nausea and vomiting and a slight fever. Could it be my bladder? Doctor: HelloThanks for the queryIt can be either your gall bladder or liver which could be causing the pain. However local tenderness strongly goes against these, to know for sure you will have to get an X ray chest Anterior-posterior view and an USG abdomen and pelvis. Please visit your GP for local examination and further management. Tab Aceclofenac is a good painkiller in these types of painI hope I was of helpRegards"
},
{
"id": 212844,
"tgt": "Suffer with sickness phobia, hesitation to try new foods. How can I get over the habit?",
"src": "Patient: Hi.. I have a big phobia of sickness.. Being sick.. Others being sick... Seeing sick.. Others saying they for feel good! I know what I am doing but I seem to always convince myself that I m going to be sick so I feel sick and get all panicked. I don t sleep well when this happens, I sleep with at least 4 pillows so that I m upright... I ve stopped trying new foods as I m worried they will make me ill, I don t drink alcohol anyway or if I have say 1 glass I m instantly in the panic stage! I just don t know what to do to control this as its really getting me down and upset! I feel like it controls my life.. I was going to try hypnotherapy but didn t know what was best.. Any advice? Doctor: Hello.... Thanks for your query. Recurrent, intrusive thoughts of being sick can occur in various anxiety disorders including obsessive compulsive disorder (OCD) as well as neurotic disorders like somatoform disorders/ hypochondriasis. Hence you require a thorough psychiatric evaluation for the same prior to initiation of any anti-anxiety or anti-obsessive drugs. Also psychotherapy (eg cognitive behavioural therapy) might be beneficial to you. Hope you found the information useful. Regards Dr Sundar Gnanavel Psychiatrist"
},
{
"id": 25411,
"tgt": "Is there chances for second heart attack?",
"src": "Patient: My husband had a massive stroke 18 mths ago, lost his right side and speech. We have been told that where he was at 6 mths is probably where he will stay. Also what are his chances of suffering another stroke.? He is 72 yrs old, doesn't walk and speech is very poor. I try to understand him but its so hard..People have told me I shouldn't try to care for him, but rather put him in a nursing home. Am I killing myself? Doctor: Hello!Welcome and thank you for asking on HCM!I understand your concern and would explain that the chances of a stroke reoccurrence depend on his cardiovascular risk factors, the cause of his primary stroke and the preventive treatment that he is taking. I would like to have some more information on his current medication and on his past medical history and possible risk factors (smoking, high blood pressure, diabetes, atrial fibrillation). It is also necessary to know the situation of his cervical arteries (possible atherosklerotic plaques) and brain collateral circulation. With the best preventive therapy (anticoagulation or antiplatelet agents, coupled with statins and a better control of risk factors like hypertension and diabetes) the chances of re-occurrence would be up to 30-50%. It is also important to continue rehabilitation, even if 6 months have passed away. This will keep him active, avoid pneumonia and bed sores and also muscular spasticity which is a secondary complication of paralysis after stroke. Hope you will find this answer helpful!Kind regards, Dr. Iliri"
},
{
"id": 25181,
"tgt": "What is the treatment for hypertension and hypothyroidism?",
"src": "Patient: I AM TRISHIT SUR ROY,AGE 28,HIGHT-6'1\",WEIGHT-92 KG.FROM 2005 I AM SUFFERING WITH HYPERTENTION AND FROM LAST 4 YEARS I AM A HYPOTHYROIED PATIENT ALSO WITH SEVERE ANXIETY.I CAN FEEL WHEN MY B.P. INCREASED.MY NECK AND BACK HEAD GET PAI8NFULL THAT TIME.RIGHT NOW I AM TAKING THYROXINE 75 MG,LOSARTON POTASIUM 50 MG WITH MODAFILIN 200 MG AND NEXITO 20,DUZELA 20.HOW CAN I LEAD NORMAL LIFE Doctor: Thanks for your question on Healthcare Magic. I can understand your concern. At your age of 28 years, uncontrolled stress and anxiety are the most likely causes for your hypertension. So we need to first control your stress and anxiety to achieve control of hypertension and good quality of life. You should definitely consult psychiatrist and get done counselling sessions. Try to identify stressor in your life and start working on it's solution. You may need additional anxiolytic drugs too. Don't worry, with proper treatment, you will be alright. Also do regular exercise, strict salt restriction in diet and weight loss (if you are obese) to achieve better results. Hope I have solved your query. I will be happy to help you further. Wish you good health. Thanks."
},
{
"id": 30872,
"tgt": "Suggest treatment for hepatitis",
"src": "Patient: Dear Sir, I am kazi here from Bhilai. i would like to know more information on Hepatitic treatment. I am facing these type problembut yet I am not getting proper treatment on prbolem. can I possible to getyour detail address or contact number? Doctor: thanks for posting your query to health care magic.you should be concern about your disease and consult to gastroenterologist .there are different type of hepatiis with diferent sevirity and different clinical out come . so you should immediately consult to your clinician and take proper managmen after making confirm diagnosis .thanks ."
},
{
"id": 159517,
"tgt": "Taking slimming coffee, fullness of stomach, abdominal cramping, watery stools. Symptoms of colorectal cancer?",
"src": "Patient: hi Im Jer, 21 y/o, female . Im taking slimming coffee(brazilian coffee, known as appetite suppresant). i would say that this is effective for i lost 26 pounds in 5-6 weeks...These past few days, i easily get full even i just ate a little. i experienced abdominal cramping (upper abdomen, just below the rib cage, under the xyphoid process) last week for 4 days that was relieved by heating pad that i put over my abdomen. Today, im experiencing watery stool (light brown), i have taken 3 capsules of imodium but no result.. my questions are: Could this condition im experiencing be possible for colorectal cancer? if so, Could this slimming coffee became the reason why my health is at risk? Doctor: Hello, welcome to HCM The symptoms you described indicates a side effect of the coffee you are taking. It does not allow the fatty materials to get absorbed into our body. So, these fats causes loose watery stool. Stop that coffee. Consult with a gastroenterologist. Do not worry, it is not due to colorectal carcinoma."
},
{
"id": 127578,
"tgt": "What causes muscle ache and numbness in the hands?",
"src": "Patient: I have had carpal tunnel surgery on right hand. Left hand indicated that future surgery might be necessary. Recently I/ve been experiencing pains in both arms, anterior. Like muscles are bruised kind of aching pain. Have had increased tension manifesting in neck and shoulders. And finally, both hands are experiencing numbing episodes while trying to type, chop veggies, dig weeds. Doctor: Hello and Welcome to \u2018Ask A Doctor\u2019 service. I have reviewed your query and here is my advice. Probably you may be suffering from peripheral neuropathy or compressive neuropathy. Until examination is done through nerve conduction it is difficult to say what it is . Use vit b12 tablets daily. Get examined for serum vitamin b12deficincy and folic acid level or serum calcium or magnesium or nerve conduction studies may useful. If symptoms not improved please consult your doctor he will examine and treat you accordingly. Hope I have answered your query. Let me know if I can assist you further."
},
{
"id": 73601,
"tgt": "Suggest remedy for cold with congestion in lungs",
"src": "Patient: Hi, may I answer your health queries right now ? Please type your query here... we have a son who'll be 2years in April. He picked up a cold with congested lungs and a cough that keeps him up at night. What can I do to help. Should he see the doctor? Doctor: Hello dearWarm welcome to Healthcaremagic.comI have evaluated your query thoroughly .* He must see a doctor , along with that precautions at present - Allow him plenty of liquids orally to maintain hydration .- Do not let him go out to excess cold .- Gargles with salted lukewarm water added peppermint oil 3 times a day - Inhalation of vapors of boiled water with blanket over the head 3 times a day 5 minutes each .- Prefer semi reclining position when doing rest .Hope this will help you for sure .Welcome for further assistance .Regards ."
},
{
"id": 168359,
"tgt": "How safe is it to administer Cefprozil for an infant?",
"src": "Patient: My 2 month old son was acting strangely. Sleeping more, eating less, and his urine smelled strong and had a sticky substance in it. I took him to my doctor and upon urine analysis determined it was a urinary tract infection. They prescribed omoxicillin based on the preliminary results, but upon further study of the urine, prescribed cefprozil to clear it up. Is this safe for my little baby? He s only 9 weeks old! Doctor: Hello. I just red through your question.Of course it is unfortunate for any child this age to have a UTI, and no doctor ever likes to prescribe any antibiotic for a baby. But in this case it is warranted. The only potentially adverse effect is some mild abdominal discomfort and diarrhea. Outside of this, you can give the cefprozil confidently."
},
{
"id": 158811,
"tgt": "Itching in between toes, abnormal bowel movements, heavy periods, bruise on arm, nausea. History of ewing saracoma. Advise?",
"src": "Patient: Hello i used to have ewing saracoma, i survived:D.(kicked cancers Ass!) i had radiotherapy done and had chemo about three times. My right leg did not grow ever since i was ten (im 17 now) due to radiotherapy. lately i've been experiencing a lot of symptoms that i feel like my body is trying to tell me something. In my right foot i usualy get itchy around my toes area especially in between my toes, also i've been a bit stressed lately and i find my self farting a lot, my bowl movement isnt very normal, rather its abnormal, i experince different bowel movements every time i go to the toilet, from green, brown to white round stools, i get my period heavier now and a bout a week ago i found a mysterious bruise on my arm and then a experienced a nother one in a different place. Sometimes i feel very nauseous and tired. I have become very lazy and love to sleep. About 5 month ago my right foot (where cancer was) got all swollen and doctors couldnt find out why. After 2 rounds of antibiotics the swelling went away, verrrryyyy slowly. Doctor: Hello; Congratulations for becoming a cancer survivor. Regarding your bowel symptoms ,it is very difficult to relate them to your cancer or the treatment. They are non specific and can be due to so many different non-ancerous causes. Being more tired is common after undergoing radiotherapy and chemotherapy. It usually subside on its own and there is no specific treatment. In the foot where you got radiation treatment ,the lymphatic drainage is altered.So you are likely to get lymphatic swelling called lymphedema.That leg is also more prone to get infection due to it.Probably you had some infection in the leg which got better with antibiotic. Regards"
},
{
"id": 179369,
"tgt": "What causes rash on hands, knees and elbows after fever?",
"src": "Patient: My grandson, 20 months old ran a low grade fever with cough and congestion. Fever is gone but he has developed a rash on his hands, knees and elbows. This iis the second time this has happened. The first time the doctor said it was not hoof and mouth disease. It disappeared on it s own. What is this? Doctor: Hi welcome to HealthCareMagicRashes after fever and cough are mostly due to viral illness itself.Usually they appear after subsiding of fever and resolve in 3-5 days.Mostly create doubt of drug reaction , but are not.Usually child is active and playful.They are to be taken seriously is baby is dull , there is pain abdomen ,vomiting,cold exterimeties, bleeding from any part of body,baby is passing less urine.Hope I answered your questionrevert back if any other querythankd"
},
{
"id": 5417,
"tgt": "Trying to conceive. Taking hucog injection. Any advice?",
"src": "Patient: hello i m shilpa. i am not able to conceive from last three years. my all reports are normal. even Dr. are also surprised that why i m not able to conceive.today is my 14th day. doctor has given me HUCOG 5000 injection at noon. and suggest that not to intercourse taoday. try on 15th day. is this true that we should not try today. i think everyone said that 14th day is best day to intercourse. i had intercousre last night.tell me what should i do now. i m worried.my cycle form last three months is of 26 days.s Doctor: Hello Thank you for your query. You are having unexplained infertility. HUCOG is HCG injection, given to help the mature follicle to rupture. That is, it induces ovulation. Normally, it takes 36 hours after the injection for ovulation to occur. And the next 3 days are ideally best for you to have intercourse. since you had intercourse yesterday, your doctor might have advised to avoid it today and hence improve the sperm count. SOme practitioners believe in daily sexual activity during the fertile phase, and some believe in alternate sexual activity. Normally ovulation occurs on the 14th day, so its considered the best day for intercourse, however if your doctor has given the shot today, you would ovulate tomorrow. So please have faith and go ahead. Take care."
},
{
"id": 215158,
"tgt": "What cause enormous pain the right arm which moves towards neck?",
"src": "Patient: I feel this enormous pain in my right arm . On a scale from 1 to 10 I would say it is 9 to 10 ,It also. feels so heavy that I can hardly lift it up . The pain is now moving towards my neck. Blood work show inflamtion .I have been put on antibiotiques a few times but nothing works . What could be causing this ? Doctor: Hello,I read carefully your query and understand your concern. The symptoms seem to be related to cervical\u00a0radiculopathy or a pinched nerve.It occurs when irritation of the nerves originating in the spinal cord in the\u00a0neck\u00a0causes\u00a0pain\u00a0to radiate down the\u00a0arm.I suggest using anti inflammatory medications such as Acetaminophen to relieve the pain. I also recommend to avoid arm movements that can trigger the pain. Hope my answer was helpful.If you have further queries feel free to contact me again.Kind regards! Dr.Dorina Gurabardhi General &Family Physician"
},
{
"id": 89032,
"tgt": "Does blue black bruise on lower left side of stomach needs attention?",
"src": "Patient: yesterday the lower part of my stomach on the left side started hurting , and almost felt like a burning pain . i didnt think much of it . my stomach has been hurting on and off for about 5 days . i am over weight . bit i just noticed a very large bruise there on my lower left side of stomach . it is purple , blue and black .i do not recall bumping into anything . what could this be , should i got to the doctor or wait a day or two to maybe see if it goes away ? Doctor: U should very cautious about this, at the place u said spleen will be there, if it damage by any injury it can cause like u said, but u should rule out wether it confined to skin because of any superficial trauma or any insect bite..hope I am help ful"
},
{
"id": 207703,
"tgt": "Suggest treatment for traumatic stress disorder with irritability",
"src": "Patient: how do i go about getting a diagnosis for post traumatic stress disorder? its not me its my mother. she has been incorrectly diagnosed about 5 times now. she has been put on about ten tablets a day, and has started getting worse and worse. also its like all her anger she has carried with her for yrs is suddenly coming out. this is a living nightmare, her gp is the one who put her on all these tablets but the neurologist we seen for yet another diagnosis, said its ptsd, she doesnt need to be on all these tablets. what would you advise? Doctor: DearWe understand your concernsI went through your details. I suggest you not to worry much. The given case is beyond the treatment ambit of online concept. If your neurologist says your mother has PTSD, she needs psychiatric treatment as well as rehabilitation therapy. Psychotherapy and meitation and breathing exercises should also do good. Work with your GP and neurologist for good treatment.In case if you need more explanation in this regard, please post a direct query to me in this website. I am happy to explain.Hope this answers your concerns.Good luck."
},
{
"id": 21537,
"tgt": "What causes low resting pulse rate?",
"src": "Patient: Hi, I am a runner who has had a low resting heart rate for the past few years <60. Since the fall I have started having unexplained shortness of breath which goes away for short periods (x1 week) and then comes back. When I started checking my pulse again it haas been droping below 50 (45-48) coudl this be the cause? Doctor: a low resting heart rate in runners and athletes is due to increased vagal tone which is normal. However the cause for shortness of breath needs to be evaluated. i would suggest1) baseline Ecg2) 2D Echocardiogram3) Chest xray"
},
{
"id": 163558,
"tgt": "Is nebulization with Clenil Compositum advisable in children?",
"src": "Patient: Can i nebulize my child 5 year old with clenil compositum and atem together?? I used to nebulize him with simple clenil compositum and normal saline every winter season .. but now his coughing is not going.. he recently started penecillin day before yesterday because his chest infection was not going Doctor: Hello,Short answer: YesDetailed answer: You can use clenil compositum together with atem. Scientific tudies show that when clenil compositum (containing salbutamol) is given together with atem (containing ipratropium bromide), the results are better compared to clenil compositum alone.Therefore, you can give both preparations together, 2-4 times a day because they will not harm your baby but at the contrary, will help him.Also, you should find a solution for the chest infection, by changing the type of antibiotic for example, in close contact with your doctor, because it is a cause of your baby's cough.Hope I have answered your query. Let me know if I can assist you further.Regards,Dr. Ervin To\u00e7i"
},
{
"id": 105468,
"tgt": "Inhaled bleach fumes. Headache and sore throat. Will I be brain damaged?",
"src": "Patient: Is there anything I can do if I accidentally inhaled bleach fumes? I was cleaning some mold in my kids closet and I may have gotten a little carried away. Now I have a head ache and a mild sore throat- not sure if it was the mold or the bleach? I ve gotten out of the room, opened a window and drank 2 large glasses of water. Will I get brain damage , are there any supplements or foods that improve cognitive functioning? Doctor: Hi, thanks for query, Bleach fumes contains sodium hypochlorite. The first exposure to respiratory tract is causes irritation to throat ,Breathlessness coughing etc. The eyes may also become red and start burning, and there may be pain in the throat. In severe cases, bleeding may occur in the throat and mouth. Inhaling too much bleach can also lead to delirium and confusion. The person may forget where he or she is or what he or she has been doing. Headaches are also common, even in the days following the inhalation exposure. If the person has had skin contact with the bleach, skin irritation and burns may also result. Vomiting and stomach pain are also not uncommon. The person should also immediately drink a glass of milk and move to an area with plenty of fresh air. Taking long, deep breaths and exhaling slowly can help clear the lungs of the sodium hypochlorite gas. As you have mild headache with mild sore throat your exposure is not too much so nothing to worry about brain damage. Thanks."
},
{
"id": 150539,
"tgt": "Got paralysis attack due to blood clot in brain. Is diabetic, high bp. Time to recover? Can get cured?",
"src": "Patient: hello m atit frm vadodara my grandpa is of 75 age he jus got a paralysis attack on right side of body due to a small clot in brain ..he has been atmiited in icu since dis sunday. he has daibitties n high blood pressure too..blood clot is not too big he is concious too..how much tym will it take to recover..will he b alright..?? Doctor: Hello, Your grand father is suffering from stroke and\u00a0reduction of, or disruption in, blood flow to the brain is the cause of a stroke. A stroke is usually defined as two types.. Ischemic (caused by a blockage in an artery) Hemorrhagic (caused by a tear in the artery's wall that produces bleeding into or around the brain) The consequences of a stroke, the type of functions affected, and the severity, depend on where in the brain it has occurred and the extent of the damage. Depending on the severity of the symptoms and how much of the body is involved, these impairments can affect the ability to walk, to rise from a chair, to feed oneself, to write or use a computer, to drive, and many other activities. Many stroke survivors recover functional independence after a stroke, but 25% are left with a minor disability and 40% experience moderate-to-severe disabilities.\u00a0 Once your grand father recovers,then he must control all his risk factors like diabetes and hypertension ,high lipids etc,because chances of recurrence of stroke is highest in next few months. Thanks"
},
{
"id": 113065,
"tgt": "Constant pain in lower right back side, swollen and sore. What could it be ?",
"src": "Patient: I have pain in the lower right back and side and it was constant for about 55 days and now it is not as worse as it has been but it is still their and it feels like something is there like swollen and soar and the pain is achy out to the tip of my hip and back infact when the pain is really bad i can press on the lower right back and hip and it hurts. Had blood in the urine but have had an abdominal ultrasound and they say that is fine no kidney stone and also they said the gallbladder and liver are normal. Blood work was done and all was normal he checked a number of things but was mainly interested in my liver and he said the numbers were good. Went to the gyn she did an vaginal ultrasound trying to find out why i had strange looking and thick blood, i had started my period but she wanted to check me to make sure i did not have a female problem and that was from the pain. She said the blood was thick and a little odd looking and it was heavy, so she sent me to have an ultrasound then and they said everything was normal. I have a pap next month. Sometimes the pain will run around into the groin and top of right thigh. Lately at night i have been having cramping in my hips and thighs while sleeping. Also when i had a bowel movement it did not hurt to go but right after a bowel movement maybe 5 to 10 minutes after the pain seemed to be aggrevated and i would start hurting a little worse. I have had my appedix out so we know it is not that. I have had some right side pain off and on since i was in high school which has been since 92 but i have not had it in a couple of years until. it came on me all of a sudden which it has never done before and it was like a cramp under back of my right rib cage and then it moved downward above the hip and then it moved further down like it is in the lower hip and butt area, but it has not gone away and my doctor says he does not know who to send me to or what to do. I did have a very bad urinary tract infection two weeks ago that went untreated for awhile because i don t normally have systems but when they did a urine culture it came back showing a back terrier eccolocilus, this is not the right spelling i know but they said it came from the colon. I was put on 5 days of levequin and the pain was somewhat better and know i still have the pain but it is not as bad as before. Do you have any idea what it could be. This is so frustrating. I have had about 10 to 12 cat scans over the years because of these doctors sending me in not knowing that this is harmful and could cause cancer so now the doctors dont want to do a scat scan which is understandable since i have had so many. What else can be done for me. Doctor: Hello. Thanks for writing to us. Pain in the lower back on the right side could be due to a muscular pain, disc prolapse, a pelvic pathology or a renal calculus. Investigations like MRI scan and ultrasound scan will help in the proper diagnosis. I hope this information has been both informative and helpful for you. Regards, Dr. Praveen Tayal drtayal72@gmail.com"
},
{
"id": 62965,
"tgt": "How to treat lump on my head as i hit myself?",
"src": "Patient: I fell over about a month ago. I knocked myself out and apparently had 2 seizures while I was being carried to my bed. I ended up with a very large lump on my head as I hit a tile floor just above my right eye. About a week after the incident I have been getting pain in my head and a feeling as if something is moving around in my head. I am very worried as it is now a pain that moves from the point I hit around my head in waves. Doctor: hi.it is best if you follow-up with your doctor, preferably a neurologist, and ask for a skull x-ray or head ct-scan just to be sure that there's no lesion (such as hematoma or contusion) in the area. management will be directed accordingly.hope this helps.good day!!~dr.kaye"
},
{
"id": 153773,
"tgt": "Is increase in prostate weight and PSA level sign of cancer?",
"src": "Patient: My grandfater's age is 79 years and he has been having problem relating to urination froma long time. Recently some test has been done and his prostate is found to be 140gm. Also his PSA was 16.....I just wanted to know if this was a symptom of Prostate Cancer or this is just BPH Doctor: Thanks for your question on Health Care Magic. I can understand your concern. Both the conditions, prostate cancer and BPH (benign prostatic hyperplasia) give rise to increase in prostate volume and high PSA (prostate specific antigen). To differentiate between these two conditions, detailed ultrasound report and biopsy is needed. If ultrasound is suggestive of calcification, homogenous prostate than BPH is more likely. But if ultrasound is showing absence of calcification, heterogeneity, invasion of capsule etc then possibility of cancer is more. Biopsy is confirmatory in such cases. So consult doctor and discuss all these to differentiate between cancer and BPH. Hope I have solved your query. Wishing good health to your grand father. Thanks."
},
{
"id": 124365,
"tgt": "What does recurring pain from armpit to rib cage and lump on breast indicate?",
"src": "Patient: I m a 22 year old female. I have a dull pain that comes and goes on my right side from my arm pit to the bottom of my rib cage. I also have a small round lump on my right breast. Just wondering if this could be of any concern. No medical history of anything serious. Doctor: Hello, What I understand by the history that you need to get firstly the lump in the breast examined by the gynecologist. As sometimes there is a physical examination more helpful along with the present verbal history. The mammary glands located in the breast do get lumps and needs an examination of its location. Once having it checked the next treatment plan can be taken up. At times even with the lump, there is not much indicative pathology but ruling out any possible cause will be of great help for peace of mind. Hope I have answered your query. Let me know if I can assist you further. Regards, Jay Indravadan Patel, Physical Therapist or Physiotherapist"
},
{
"id": 128927,
"tgt": "How to treat biting sensation on ankles?",
"src": "Patient: Hi. I am 62. I have a blood disorder. I don t know if I have anything to worry about, but I don t know why my ankles (the last couple of days) have (off and on, but not a lot), felt like something was biting me, but there has been no itching or redness. Doctor: Hello,Thank you for using healthcaremagic.I read you question and understood your concern.I would like to know the name of your blood disorder to give a better opinion.Anyway from the few details that you give I would recommend to check the Diabetes.It gives changing on the sensation in your feet.Dr.Selmani"
},
{
"id": 100791,
"tgt": "Does asthma effect the unborn child?",
"src": "Patient: hello doctor i had a confution .. so i believe that u clear my confution.. i had 1 child ( 5yrs) now i am planning for 2nd..but i am a asthama patient so i take montek lc daily night.( half). If i became pregnent there is any problem to child pls answer me.... Doctor: Hi dear,At the outset, please take a note that asthma might have affect on unborn child ,but chances are minimal.You may be put on bronchodilators or inhalers like Foracort during your pregnancy ,because pregnancy will bring physiological changes in body like you may have difficulty in breathing .So.only Tab .montek-lc will not be effective in pregnancy .I would be glad to know if my advice any helped you.B/RDr.Harpreet kaury"
},
{
"id": 210882,
"tgt": "Time period for hallucination to recover?",
"src": "Patient: My nephew 25 yrs old just diagnosed of schizophrenia, started zyprexa 10mg yesterday. He did not sleep well for a week even with ambien . He slept 14 hours last night. He still felt tired this afternoon & went to sleep 7 hours after getting up. How long do we need to wait to see his symptoms( hallucination) getting better after the med. Any other treatment does he need ? Doctor: Hello,Thanks for choosing health care magic for posting your query.I have gone through your question in detail and I can understand what you are going through.Treatment of psychosis wil require antipsychotc like zyprexa and it generally requires 2 weeks for around 30-40% improvement to occur. Its difficult to predict when the voices will stop but if the medication is effective in your case then in 2-3 weeks they should stop. Hope I am able to answer your concerns.If you have any further query, I would be glad to help you.In future if you wish to contact me directly, you can use the below mentioned link:bit.ly/dr-srikanth-reddy\u00a0\u00a0\u00a0\u00a0\u00a0\u00a0\u00a0\u00a0\u00a0\u00a0\u00a0\u00a0\u00a0\u00a0\u00a0\u00a0\u00a0\u00a0\u00a0\u00a0\u00a0\u00a0\u00a0\u00a0\u00a0\u00a0\u00a0\u00a0\u00a0\u00a0\u00a0\u00a0\u00a0\u00a0\u00a0\u00a0\u00a0\u00a0\u00a0\u00a0"
},
{
"id": 110861,
"tgt": "What causes pain in lower back?",
"src": "Patient: Have pain lower back sometime felt in groan. Fell on concrete on left side mri shows degenerated disease at l4 and 55 left foraminal disc protrusion cqusing foraminal narrowing with facet degenerated chqnges l5- s 1. Disc space narrows at l5- s1 lowered right iliac crest. My lower back hurt! Doctor: Hello, Thanks for your query.You might be suffering from degenerative disc *disease & it occurs due to repetitive overload or stress to the disc & it increases the risk of disc herniation & spinal canal stenosis.For this condition you have to do stretching & strengthening exercises in which you have to stretch tight muscle & strengthen weakened ones which will alleviate stress on facet joint and disc.Meanwhile for pain control take anti-inflammatory drugs like ibuprofen.I do hope that you have found something helpful and I will be glad to answer any further query.Take care"
},
{
"id": 145305,
"tgt": "Suggest treatment for back pain due to moderate bulging disc L-2 through L-5",
"src": "Patient: question regards moderate bulging disc L-2 through L-5. Also denerative disc desease as evidenced via MRI with no contrast. Looking to help with pain management (other than drugs) such as exercise at this time. Am unable to have appointment with specialist until end of Oct- and do not think I can live like this until then. Doctor: Hello!Thank you for the question on HCM!I understand your concern.I would recommend a lumbar brace, and physiotherapy for the pain. You should avoid straining physical activity and staying in sitting position for a long time.It is not a matter of surgery. The non steroid antiinflammatories can help.If the pain still persist I would start Amytriptiline or gabapentine, but these medications ate given only under doctor prescription.Best wishes Dr. Abaz Quka"
},
{
"id": 213061,
"tgt": "Severe stress due to loss of near one, feel vomiting on talking, have anxiety attacks. Any ideas?",
"src": "Patient: I have a friend who is going thru a stressful time. His best friend was in an accident & may die. He has been visiting the hospital & spending time with the friend s parents. He says hes trying to be strong for them but that he is a mess & will go to therapy next week. What im curious about is that he says he cant talk without wanting to vomit . I have had anxiety attacks etc & I have never experienced this nore ever heard of it. Any idea what could cause it? He has told me that he used to be very anal. (His words) he has also said he has OCD . Doctor: Hello and welcome to Healthcare Magic. Thanks for your query. I'm sorry to hear about your friend's difficult situation. I hope his therapy helps him to manage his stress. Now, regarding your question, anxiety can manifest itself as psychological as well as physical symptoms. The physical manifestations of anxiety can vary widely and can literally present with symptoms of any system in the body. Moreover, when the mind is anxious / in a state of arousal, there can be a heightened perception of sensations and stimuli, which would otherwise be ignored by a non-anxious person. So, it does make sense that your friend's nausea sensation could be anxiety -related. Please encourage him to see a psychiatrist for a detailed evaluation and management. Wish you all the best. - Dr. Jonas Sundarakumar Consultant Psychiatrist"
},
{
"id": 46227,
"tgt": "Why is one kidney bigger than the other?",
"src": "Patient: hi my name is andrea and my right kiddny is smaller then the left one. the right one is 9.8 cm and the left one is 12.1 cm. i just want to know if this is something i should worry about and if this coud cause my higy blood pressure? i also have itp could it be because of that? i was an prednison for 1 year. than you for your time. Doctor: Hi and welcome to Healthcaremagic. Thank you for your query. I understand your concerns and I will try to help you as much as I can.No, this is not something that you should worry about since this is normal variation of kidneys size and two kidneys are never the same size. Only if this is larger than 14cm or smaller than 6 cm it may indicate some diosrders. Also it will not affect your blood pressure unless there is some parenchimal kindeys disorder and for that to evaluate, the size itself would not be sufficient. YOu should do additional:1. Doppler ultrasound and blood flow measuremnet2. Blood urea and creatining3. Holter blood pressure measurement.These tests will show if blood pressure is determined by kidneys or heart function and then you ll have more info about how to treat it.I hope I have answered you query. If you have any further questions you can contact us in every time.Kindly regards. Wish you a good health."
},
{
"id": 157975,
"tgt": "Had cyst on ovaries. Scheduled laproscopy. Ovarian and fallopian tube removed. Help",
"src": "Patient: Hi, my name is natasha. i have always had cyst on my ovaries for as long as i can remember my menstrual starting. In oct.2009 i had a torsion surgery , i had my right ovary and fallopian tube removed. The dr. said the fallopian tube was basically crushed. Years go on and about 6 months after that surgery i had noticed that i had the pain once again, i started getting 3 months ultrasounds and everything was fine. just cyst that came and went monthly. however. 5 months later i fell to the floor in pain and ended up at the er. They had told me i had a huge cyst the size of a grapefruit on my left ovary. I was scheduled in for a laproscopy . during the surgery the surgeons had removed the cyst, also scar tissue on the right side and that was on my live. a month after surgery i noticed some pain. I went into the er, and had fluid. during the next three months i noticed the pain of a cyst , last month in aug. I went in the er have a cyst it is complex a month later i go in and its a follic cyst 2cm. I am in great pain, i saw the dr and the dr said nothing to worry about- i am in pain to the point jeans hurt my cyst. i can t lay on my left side or anything. I have a scheduled ultrasound in 6 weeeks. I don t know why i keep getting cyst. 6 months ago i had the surgery. could it be cancer? endo? psoc? should i ask for metiform (sw?) Doctor: pain is very rare in cancer unless there is torsion of ovaries or severe adhesions causing intestinal obstruction.usually cancer presents as a abmdominal mass or distention with free fluid in the abdoman. on ultrasound cancer presents as a complex cyst.as u already had surgery twice, it looks to me more like a adhesions during surgery which can only be corrected with resurgery with adhesionolysis. if you have completed the family? then removal of ovaries is also an option"
},
{
"id": 64138,
"tgt": "What is the growing lump in the arm?",
"src": "Patient: I have a lump in the bend of my left arm that has been there for about 3 years and it keeps getting bigger but the ultrasound said it was nothing but it keeps growing and it makes my arm hurt sometimes or makes my arm tired if I try and use it too much Doctor: Hi am Dr Fahim and I will help you with your problem \u00a0\u00a0\u00a0\u00a0\u00a0I have gone through your question and understood your concerns. It appears to be Lipoma, which is a benign tumor composed of fat. Its treatment is surgical excision under local anesthesia, if its size is big or its bothering the patient. \u00a0\u00a0\u00a0\u00a0\u00a0If such a patient walk into my clinic, I do excision of the lipoma under local anesthesia and prescribe tab Augmentin 625mg 1+1+1 and Paracetamol 500mg 2+2+2 to prevent postoperative infections. Patients are usually better in matter of days. You need to see a general surgeon for treatment of your condition. Wishing you rapid recovery.I hope my answer will help you. Do rate it, if you like it.Regards"
},
{
"id": 57840,
"tgt": "Can congestion, back pain and itching in knees be related to elevated liver enzymes?",
"src": "Patient: 1. 25 year old male 2. liver enzymes elevated ~4x normal limit, discovered in November 3. Pain in back, below shoulder blades and in center, started January (pain then eases 2 days later pain again) 4. congestion (head) 5. itching behind the knees otherwise healthy, does not drink or do drugs. negative for iron build-up, autoimmune disease and hepatitis Doctor: Hi, Welcome to Health care magic forum. It appears that you have hepatitis, may be the hepatitis E, hepatitis B, etc. The pain suggests that there may be the choli cystitis, or stones in the gall bladder. I advise you to consult a gastroenterologist for diagnosis and treatment. You may need to have an M.R.I. for confirmation, Wishing for a quick and complete recovery. Thank you."
},
{
"id": 192551,
"tgt": "Suggest treatment for hard and long nipples after gynecomastia surgery",
"src": "Patient: I am 36yrs old and had Gynecomastia surgery over 12 years ago. This really worked in changing the shape and look of my chest (male breast). But the actual nipples are still very long and typically hard. I don't like to take my shirt off, even though I work out regularly. I refrain from wearing just a t-shirt, especially a white shirt without a tight fitting tank top. I have to wear dark shirts to hide my nipples from sticking out like a sore thumb when dressing. If insurance won't cover nipple reduction surgery, it seems as if I could just cut the nipple part off....I know this is drastic but if done in a clean environment with sterile equipment, couldn't I save myself the money for a plastic surgeon. Doctor: Hello,No, its not wise to remove a part of it at home as it may get complicated with excessive bleeding and post removal infection. Its always wise to let doctors handle such circumstances themselves since performing surgery demands skill and perfection as well.Hope I have answered your question. Let me know if I can assist you further. Regards, Dr. Sameen Bin Naeem, General & Family Physician"
},
{
"id": 41037,
"tgt": "Suggest remedy for infertility problem",
"src": "Patient: hi i am having only sperm count of 48 millions/ cu.mm and quantity of 0.5ml and still my wife is not getting pregenant as docotr has advised all the test to my wife which had been done and result is ok, now doctor says i have low sperm count and quantity she prescribed me siotone for i month doctor pl suggest Male 32 years married 185 cm 90 kg Doctor: Hallow Dear,I empathize your situation. Your sperm count is not bad to label you incapable of fertilization; however, your semen volume is low. Try to increase the volume by including extra proteins in your diet. The other important parameters of the semen examination are missing. The sperm motility, morphology of the sperms, etc. form vital parameters of the semen examination. If you mention that, I may be I a better position to guide you further. You may please ask me direct question and upload your reports for me. Siotone that you have been prescribed is a medicine which is supposed to help in sexual weakness and improving the libido. I have my own doubts how much it will be useful in improving the quantity and quality of your sperms. I would advise you to get your post coital semen examination performed. For this examination, the semen is collected from the vagina after the intercourse. The motility of the sperms is assessed under microscope. If the motility is grossly affected, it suggests that there are antibodies present in the vagina of your wife which are killing the sperms. In such case, intrauterine insemination may help. I hope this will guide you for further steps."
},
{
"id": 193985,
"tgt": "Can I have intercourse with sebaceous cyst on scrotum?",
"src": "Patient: Hi, I had a sebaceous cyst on my scrotum which popped by itself two days ago, most of it is gone but i can still feel a bit of hardness under the skin. Can I have sex tonight with my girlfriend? We use lube when we have sex. I ust dont want the cyst to get infected. Can you tell me what I should do? Thanks. Doctor: Hello, Yes, you can if its sebaceous cyst for sure. Yes, you can use lube. Since its in scrotum no issue. Hope I have answered your query. Let me know if I can assist you further. Take care Regards, Dr S.R.Raveendran, Sexologist"
},
{
"id": 69359,
"tgt": "Suggest treatment for a lump under the feet",
"src": "Patient: Hi Dr. Shankdhar i am 27 year old. Recently there was some problem under my right feet, a small lump which started paining. When i visited doctor he prescribed antibiotics and asked me to take blood test. In my blood test my sugar levels are very high 270. Please advice.. Doctor: This lump can be a corn or abscess or anything like that needs to be examined.Do you have fever?If clinically its difficult to diagnose then FNAC (fine needle aspiration cytology) can be done.Diabetes strictly needs to be under control.Hope this information is useful to you.Thanks. Regards."
},
{
"id": 145773,
"tgt": "Suggest treatment for disc problem",
"src": "Patient: I have been suffering from Disc problem from last 5 years. I works in QC Department As A Analyst in a Pharma company. Somtimes I face severe pain due to overtime or excess works. What should i do , Should i continue my job, or Should i change my work as medical store means siting work . I am a registered pharmacist also. Please advice me . Doctor: Hi,Thanks for writing in.The lower back is made up of an alignment of small bones. These bones have sponge like discs between them and the spinal cord passes along the back of these bones through the spinal canal. You seem to be having one of the discs protruding between the bones and this protrusion is backwards and causing narrowing of the spinal canal. You are having these discs protruding in to the spinal canal at more than one level and causing pinching of nerve roots leading to severe pain in your back and legs.There are patients who get relief by doing physiotherapy and taking neurogenic pain relievers. Injections give relief as you have experienced earlier. However this might nor always happen. If your pain is severe and not responding to medical treatment then the disc protrusion needs to be removed by doing a small surgery at the lower back region. Presently, laser assisted surgical techniques are also available.You can try to feel more comfortable at work by taking small breaks every 30 minutes and not straining your back."
},
{
"id": 210406,
"tgt": "How to treat head pain in an autistic child?",
"src": "Patient: My son is 11 yrs old. He is Autistic with Hypotonia and non verbal. He just recently in the last 2-3 weeks started showing symptom of head pain. We know this because we think out of desperation he starts crying, yelling, screaming and starts to self punch the top of his head and also right below his eyebrows just above his nose with his hand in a fist. We have taken him to the doctor and he prescribed my son ibuprofin and Q-Dryl. It seem to help at first but now it is not helping. He seems to have more episodes and worsening. Just yesterday, we took him to emergency because he got pretty bad. They gave him a CT scan and it came up negative. He now has an MRI scheduled in the next two weeks. There is a long line. Can you offer your thoughts on this? Doctor: HiThanks for using healthcare magicSuch kind of behavioral issues are common in autistic child and i think, you son also has cebebral palsy. It is happening due to underline mood swing and impulsiveness. In that case, i would suggest, oxcarbamazepine or risperidone to control such behavior. Better to consult a psychiatrist and discuss about the treatment.Thanks"
},
{
"id": 3996,
"tgt": "Can light spotting a week before periods affect fertility?",
"src": "Patient: Hi I experience light spotting a week before my period is due since stopping breastfeeding. My temp stays elevated until full flow. Can this affect my fertility as I am trying to conceive and had an early miscaraige last month. Can this spotting have caused it? thx Doctor: HIWell come to HCMActually menses of either any manner scanty flow, heavy flow, spotting, this is nothing to do with ovulation and fertility important is ovulation, production egg, and presence of perm at the time of coitus, for pregnancy, if these are not there this affect the pregnancy, hope this information helps."
},
{
"id": 126345,
"tgt": "What causes knee pain while on Eliquis?",
"src": "Patient: I was diagnosed with three pulmonary embolisms in my right lung. Since taking Eliquis I am having pain behind my right knee and my right leg is very heavy and pain when I walk. I become tired very quickly when I walk down the hall of my apartment building (about 60 yrds.) Is this normal with taking Eliquis 10mil twice a day? Doctor: Hello, You haven't mentioned about your age first of all because that's important while thinking of differential diagnosis. You could have developed another clot which could be responsible for your symptoms. I would suggest you to go for color Doppler. Hope I have answered your query. Let me know if I can assist you further. Take care Regards, Dr Anuj Gupta, Spine Surgeon"
},
{
"id": 66341,
"tgt": "What causes lump inside of lower back?",
"src": "Patient: Hello, I have a small lump under the skin in my lower right back. It cannot be seen, only felt. It is very movable, no pain and is semi hard. It seems to be about 2 inches from my spine and even with my hip bone. I m a 30 year old male Any idea for what it could be? Thanks Doctor: Hi, thanks for sharing your health concerns with HCM ! Well, you mentioned that ' It cannot be seen, only felt. It is very movable, no pain and is semi hard!'...Anyways, If I were your treating Doctor for this case mobile painless lump in an adult male, I would come up with three possibilities, these include: 1.\u00a0\u00a0\u00a0\u00a0\u00a0a lipoma or benign fatty tumor2. a neurofibroma or benign tumor of peripheral nerve2.\u00a0\u00a0\u00a0\u00a0\u00a0a benign cyst like sebaceous cystOverall, it is benign and not to worry about this but you could go for FNAC test for confirmation if still worried!Hope this answers your question. If you have additional questions or follow up questions then please do not hesitate in writing to us. I will be happy to answer your questions. Wishing you good health."
},
{
"id": 160328,
"tgt": "I have lymphoma of some discription but gp is unhelpful",
"src": "Patient: Hi im nigh on sure i have lymphoma of some discription but gp is unhelpful ive been enduringg the following 26wks starting with rash on face around eyes ears chheks, scalp(un bearable)and entire body intensely itchy and painful not responding to any treatments to date,3 large lumps in neck either side painless and growing slowly very hard, 2lumps under arm, multiple unexplained bruises of vast sizes,loose teeth need them all removing hairloss tiredness but cant sleep,and sweating sporadically both day and night am i right Doctor: Of course u r nt rite. u need a clinical examination by a generall surgeon or a Oncologist to know whats ubderlying and to treat it accordingly."
},
{
"id": 36322,
"tgt": "What causes blood spitting, swelling above eye, sinus pain after forehead/sinus injury?",
"src": "Patient: hi. i got hit in the forehead/sinus by the scope of a gun today during backfire.I've been consistently spitting up small amounts of blood for about 5 hours. I have a little bit of swelling above my eye and some pain in my sinuses. The blood seems to be coming from mucus. Should this be a concern? Doctor: Hello,I understand your concern.I am Dr. Arun Tank, infectious diseases specialist, answering your query.In my opinion you should take your xray face done. There is chance of blood and mucus coming from the sinus fracture. X ray will tell you the nature of injury, if its fracture it is the emergency situation and you should visit the near by ER.Fracture should be attended as early as possible. If it remains unattended it can cause further bleeding.You can take tablet aceclofenac and pantoprazole for pain and acidity.Please maintain local hygiene it clears the wound rapidly.I will be happy to answer your further concern, you can ask me on bit.ly/DrArun. Thank you.Dr Arun TankInfectious diseases specialist."
},
{
"id": 71549,
"tgt": "Is there laser treatment for external piles?",
"src": "Patient: Respected Doctor., I am suffering from external piles, and I read an article that there is painless and bloodless surgery availaible through laser treatment.I am resident of Shimoga Karnataka and I want to know hospital which can treat me at the place which will be nearest to Shimoga. Doctor: Hello,For piles, conventional surgery is much better than laser removal. Laser removal has higher chances of recurrence. So better to go for conventional piles surgery for good future outcomes. Hope I have answered your query. Let me know if I can assist you further.Regards,Dr. Kaushal Bhavsar"
},
{
"id": 216418,
"tgt": "What causes pain that radiates from neck base to ear?",
"src": "Patient: Have recently developed a pain that runs from the base of my neck up to the ear area. It catches the nerve and makes it feel like it needs to be adjusted or massaged immediately. If I quickly turn my head into a side by side rotation it will stop. If I push on my shoulder blade area using my index finger I can find the sore spot there. Could it be I need a new pillow? Doctor: Hi,It may be due to some sudden turning of neck that you have done unnoticed. Or due to improper sleeping habit to sides where neck remains unsupported. And at the point of soreness as told there some nerve might be impinged in the sore muscle that why you are getting pain radiating to ear. My suggestion to you is to use cervical collar while travelling. Use of ice to relieve pain and soreness, and use of butterfly pillow for proper positioning of neck while sleeping will help. Don't do any sudden jerky movement of neck, it may aggravate the condition. Hope you find the answer useful. Let me know if I can assist you further.Regards, Dr. Harsh Swarup"
},
{
"id": 151051,
"tgt": "Fluttering or pulsing sensation behind head. History of epilepsy, seizures. Causes?",
"src": "Patient: Occasionally I get this odd fluttering or pulsing sensation in the back left side of my head. I probably wouldn t be so concerned, but I have epilepsy . I m 35 and have not taken medication for it and have not had a seizure since age 6. I also, currently have no health insurance and don t want to go to the doctor and end up getting a bunch of tests if I m overreacting. Doctor: Hi, Any Anti-seizure drugs can be stopped if there is not even single seizure for 3 yrs You did not have seizure from your 6th yr till now. so almost 29 yrs. Fluttering or pulsing sensation you are complaining of may be due to AURA means indication to get epilepsy. No other go You have to consult a neurologist and take EEG and follow his advice Best wishes"
},
{
"id": 96983,
"tgt": "How to cure pain under the rib cage after falling down?",
"src": "Patient: hi, i went ice skating and as i was falling i used my arms to cover my face from getting hurt pushing my abdomen out, but i landed on what felt like my rib cage and i lost my breath, i was straining to breathe, its bruised, but now the pain has traveled underneath my left side of my breast what do i do? Doctor: Hi and thanks you must get a chest xray and show it to a doctor and if you have more pain then take Paracetamol.naproxen, ibuprofen or aspirin."
},
{
"id": 214660,
"tgt": "Suggest natural remedies for constipation",
"src": "Patient: I quite often suffer from occasional constipation (about once a month) but now it's becoming nearly constant. Are there any natural remedies that might help (apart from water and an increased fibre intake)? It's making me tired and grumpy and I'm losing my appetite... Doctor: hello..i suggest the following to prevent the constipation..Cut back on refined food items.Avoid foods that contain white flour like maida, white sugar and other processed foods. In the morning consume one tablespoon of olive oil along with lemon juice. It works best on an empty stomach, so have it before you eat anything else. eat plenty of Beans,Apricots,Whole grain bread, (AMLA)Berries,Broccoli,Plums, pears, and apples and nuts, raisins.Adding condiments like jeera, haldi and ajwain in your food while cooking it is a great way to help digestion.orMix 1 tablespoon of flaxseed oil with 1 glass of orange juice. Drink as needed, but give it time (up to 5 hours) to start working so you don\u2019t overdo it.ordrink one cup of aloe vera juice in the morning.orDrink one glass of prune juice in the morning and one at night to relieve constipation. It should start working within a few hours.orEat a cup of yogurt with breakfast, and then on its own or with snacks throughout the day.regular exercise is also a must and also maintain a regular time to evacuate bowels.hope you find relief"
},
{
"id": 169695,
"tgt": "Suggest treatment for severe fever and frequent urination in a child",
"src": "Patient: HI DR MY BABY IS 20 MONTHS OLD AND SHE IS SUFFERING FROM FEVER AND TODAY IS THE FOURTH DAY THE FIRST 02 DAYS SHE HAD FEVER SO TOO MUCH THIRD NO FEVER BUT THAN IN THE EVENING AGAIN SHE HAD FEVER AND ONLY HER HEAD IS BECOMING HOT AND SHE IS ALSO PASSING URINE MORE MY HAS JUST IT MATAFIN AFTER VERY FOUR HRS Doctor: fever with frequent urine passage means your baby needs urine examination and then only we can start the treatment ok"
},
{
"id": 37695,
"tgt": "Does bite from squirrel cause rabies?",
"src": "Patient: I was bitten by a squirrel while trying to capture it. It s leg was broken and I wanted to help it. I was bit through a sweatshirt and cleaned the wound with peroxide and warm water and put a band aid over it. I had a tetinis shot a year ago and I am 13. What do I do? Could I have rabies? Doctor: Hi,Thank you for your query. I can understand your concerns.Small rodents, such as squirrels, hamsters, guinea pigs, gerbils, chipmunks, rats, and mice, and lagomorphs such as rabbits and hares, are almost never found to be infected with rabies and are not known to transmit rabies to humans.An injection of tetanus toxoid is all that is required.Regards Dr. T.K. Biswas M.D.Mumbai"
},
{
"id": 167780,
"tgt": "What causes nose bleeding?",
"src": "Patient: My 3 year old son has a lot of food allergies and takes zyrtec for them. My concern is the purple rings under his eyes and today he had a nose bleed from one nostril for no apparent reason (he also has had a swollen gland in his neck for 2and half years). Are these common symptoms for allergies? Doctor: all of these symptoms are related to allergy the circles around his eyes are due to allergic rhinosinusitis .the nose bleeding is due to weakening of the mucous membranes of the nose due to constant irritation by the allergen the swollen gland in his neck is mostly an enlarged lymph nodes due to recurrent inflammation of the upper air ways due to allergic reactions .I suggest to at least do a complete blood count and a blood film to make sure of the diagnosis of the neck mass and to exclude low platelets which can cause a nose bleeds ."
},
{
"id": 164919,
"tgt": "Does a baby s yellow skin color need incubation?",
"src": "Patient: doc if the baby had a blood enfection after he admit in 1 week in the hospital then now the blood infection is clear but now the doctor said the baby nid to put in incubator because of the yellow color of the skin i dont know if its true i nid a second opinion. its that posible? Doctor: Hi thanks for the question. I assume that your baby is more then a week old . Most of the babies have yellow discolouration of skin and sclera during the first few days of life . This is called physiological jaundice and it subsides on it's own without any intervention. This is a normal phenomenon as a result of transition from fetal to extrauterine life. You should be aware when jaundice is a cause of concern: 1. If jaunduce appears on first day of life2. If the level of bilirubin is more then 12mg/dl. Bilirubin is the substance that causes the yellow coloration of skin . It's level can be checked with a blood test .3. If the the rise in bilirubin level is more then 5mg/dl/hr. For this your doctor my prescribe repeated test and then plot the levels on a graph to predict the expected rise in bilirubin.4. If 'direct ' bilirubin is more the 2mg/dl or 20% of the total. Bilirubin is of two types physiological jaundice is due to the water insoluble ' indirect' bilirubin so if ' direct component is high it means it's not physiological and should be a cause for concern. The direct and indirect breakage is given in the blood test. 5. If your baby is otherwise unwell e.g is lethargic, is not feeding well, has fever or difficulty breathing then it's not physiological jaundice because in that case your baby will be otherwise totally ok.You have asked if your baby needs incubator care. Incubators are for premature babies who can not maintain their body temperatures as they have arrived earlier and temperature control mechanisms have not yet been established. The incubator provides specific temperature. Incubators are not used for the treatment of any type of jaundice. Phototherapy is used for that. These are specially lights that convert the indirect water insoluable bilirubin into water soluable forms that can then be excreted out through urine . As i have told high levels of direct bilirubin are a cause of concern and these need treatment with a phototherapy unit . Your doctor may prescribe phototherapy if your child has any of the criteria i have mentioned earlier but incubator care is not needed as your baby is not premature and is at least older then a week. If you have any other questions i will be happy to answer."
},
{
"id": 101284,
"tgt": "Is relpax a safer medication to use?",
"src": "Patient: I am allergic to Imetrex it causes a dystonic reaction. The one time I was given it the muscles on my whole right side seized. My question is would relpax be safe for my to use? I have migraines more than 15 days a month, actually most days. I am legally blind with congenital nystagmis and that is what they think causes them. Doctor: HI, thanks for using healthcare magicThough the medications are slightly different in terms of their structure, metabolism and mechanism of action, they belong to the same class of drug.This means that it is possible that you would also have a reaction to the relpax.You and your doctors may have to choose another type of pain killer and may also consider the use of prophylactic medications.Prophylactic medications reduce the frequency of the migraine attacks.I hope this helps"
},
{
"id": 11653,
"tgt": "Bump on face below eye, pigmented, hurt when touch. Is it serious?",
"src": "Patient: I have a small bump on my face below my eye that has a slight dark pigment to it and seems to hurt every time I touch it. The area around it is red, but only now has it started hurting. Before it was only a bump on my face that was slightly pigmented. I've been meaning to go to a dermatologist but I've been pushing it back, is it something serious? Doctor: Hi ! Good evening. I am Dr Shareef, a general surgeon answering your query.If I were your treating doctor, I would have advised you for an excision biopsy of the swelling specially because of the dark pigment to it. Instead of delaying the process, I would suggest you to see your doctor at your earliest convenient opportunity.Thanks for choosing healthcaremagic forum for your health query. Good luck."
},
{
"id": 124387,
"tgt": "Suggest treatment for pain and bruising in legs",
"src": "Patient: Certain areas on my legs hurt really bad and bruised to the touch, sometimes when my 2 year old will lay back on my leg, I jump because it feels painful, it is like maybe 2 to 3 inches above my ankle, like maybe a bruised bone, idk .anyone know, Thanks Michelle. Doctor: Hello, As a first line management, you can take analgesics like paracetamol or aceclofenac for pain relief. If symptoms persist better to consult a general surgeon and plan for a Doppler scan. Hope I have answered your query. Let me know if I can assist you further. Regards, Dr. Shinas Hussain, General & Family Physician"
},
{
"id": 225794,
"tgt": "Irregular bleeding since Implanon inserted, bleeding after intercourse. What is the cause ?",
"src": "Patient: a year and halve ago I had the implanon inserted into my arm. Before that I used contraceptive tablets and my menstrual cycle was normal, but since the implanon I have irregular bleeding or no bleeding at all. About 3 months ago I started to have spot bleeding after intercourse, which was getting worse. So I went to have a pap smear done today and immediately started bleeding as soon as dr tried to do a swab. She also said that the cervix does not look normal. She did send the blood sample away as well as the smear for further testings. the results will take 10 days. Can you please tell me what the main causes for this could be. I am 33 years old, had a baby in 2010 and my last pap smear was a year and halve ago, which was normal. Doctor: hello,condition is definitely unrelated to your implanon. however spotting after intercourse could be due to infection of your reproductive tract, pre-malignant conditions of cervix. be cool, relaxed as both the conditions can be treated. wait for result and get treated. wish you good health"
},
{
"id": 214834,
"tgt": "Severe pain, large bump under the skin on cheek, oozing water like stuff and blood when popped. Any natural home remedy?",
"src": "Patient: Hello I need help please! I have a very large bump under my skin on my left cheek, and it hurts BAD, I've tried popping it but only water-like stuff and blood comes out. There is a core and I can feel it from the inside of my cheek too. It is about the size of two fingertips sidebyside. Please help! I need a natural home remedy due to no insurance. Doctor: Hi and thanks for this query.I am so sorry to hear about this bump. Hope we get to find solutions earlier than later.This sounds like a big pimple or cellulitis. The best course is to refrain fro squeezing or touching it as this may promote infections like cellulitis. Try some pain medications like Ibuprofen and paracetamol. They will offer great pains relief and a reduction in the sie of the swelling. If fever or bump increases in size or becomes softer than before, then please visit the hospital for antibiotic prescription.Also, applying ice packs on this bump will reduces its swelling and painful nature.Hope you ind this helpful and informative. If you need more answers, please feel free to contact us for me. I wish you well.Dr. ditah, MD"
},
{
"id": 84182,
"tgt": "What are the side effects of dexona tablets?",
"src": "Patient: Sir i use to take dexona tablets to gain weight about 2 years, i gained around 15 kgs then i stopped it, after 3 to 4 months i am loosing my weight, but i did not feel any effects while taking the tablets, what should i do shall i continue or stop it pls help me Doctor: Hi Dexona contains a steroid called dexamethasone. Steroids are used to reduce inflammation and are useful in treatment of many chronic diseases such as autoimmune diseases.Weight gain is a side effect of the drug and not an indication.The weight gained with steroids is not a healthy weight gain.It causes abnormal fat deposition behind the neck,abdomen.Muscle mass does not increase.Kindly stop the drug.Identify the cause of weight loss and treat the same.Healthy fat and Protein should be increased in diet inorder gain weight.Hope I have answered your query. Let me know if I can assist you further. RegardsDr.Saranya Ramadoss, General and Family Physician"
},
{
"id": 184085,
"tgt": "What causes numbness in lip and chin while having tooth pain?",
"src": "Patient: My bottom tooth 5th one from the left was hurting for a few days. The pain shot up into my neck and down into my shoulder where my arm was giving out. My jaw began hurting and now the pain is gone On and off but my bottom lip and chin are numb and have been for 3 days now:/ Doctor: helllooo....numbness is due to the infection at the root surface of the tooth ..it is pressing ur nerve to the lower lip..actually nothng much to worry in this..take amoxicillin and metronidazoleand diclofenac ...for 5 days...go to dentist ...and get the root canal done....pain and numbness will be reduced within few days time...and dont wait for long..sometimes it can spread to ur neck n all below the tongue etc...meet dentist and get it done...dont get panic ..be cool and have a nice day!!!"
},
{
"id": 62923,
"tgt": "What causes boil on nape and lump beside it?",
"src": "Patient: I've had bilateral mastectomy, stage 0 breast cancer, 17 lymph nodes from my right and 7 from my left underarm were removed by my surgeon. Now I have a boil on my nape and i feel another lump beside it. I'ts quite bothersome because it's painful. What do I do. Thank you so much. Good day! Doctor: Hi, dearI have gone through your question. I can understand your concern. You may have boil or enlarged lymphnode. Enlarged lymphnode may be due to reactive hyperplasia, tuberculosis or metastatic breast cancer. So it should be tested. Go for examination. If needed go for fine needle aspiration cytology or biopsy of that lump. It will give you exact diagnosis. Then you should take treatment accordingly. Hope I have answered your question, if you have doubt then I will be happy to answer. Thanks for using health care magic. Wish you a very good health."
},
{
"id": 122753,
"tgt": "What causes pain in groin and left leg?",
"src": "Patient: I have pain in my groin and left leg. Sometimes I think it is my hip but now I am convinces it is my left leg. I cannot lift my left leg very far. In the morning it is stiffer than in the evening. I tried ice but that caused unbelievable pain in my leg and hip. What is it? Doctor: Hello, This can be a tight muscle in your hip rotators, so it may need to be examined clinically, however you can try doing the opposite position of pain, I mean you should try to position your leg in the pain-free range or position as long as you can. Otherwise, I would need more details about your age, weight, activity level and positions that aggregate pain. Hope I have answered your query. Let me know if I can assist you further. Take care Regards, Ayman Darrag, Physical Therapist or Physiotherapist"
},
{
"id": 25540,
"tgt": "Why do I have intermittent hypotension?",
"src": "Patient: Why do I have intermittent hypotension. Will be low one minute and high the next. It also gets postural at times. It makes me very weak and muscles hurt. I am on Toradol 25mg. qday and lisenapril 20mg. at night. I have been holding off on meds if to low. I need to mention I have had a pacemaker placed in 2008, heart surgery in 2010 and have had 2 heart attacks, several angioplasties and a stent this March due to a 75 to 80 % block. Cured my s.o.b. Thank You, Sandra Janney. didn t know have to pay for answer Doctor: hello Sandra,I have gone through your query.Thanks for using HCM.i request you to reduce and or omit toradol adn lisinopril if your BP is persistently low.You should also get your ECG,Echo and pacemaker function tested to find any possible clue to such fluctuation BP.My best wishesDr.Rajesh Teli,MD."
},
{
"id": 114968,
"tgt": "What causes low WBC level?",
"src": "Patient: Hello, I have been progressively getting sicker over the past 8 months since a rotator cuff injury where the subscapularis was full of toothpaste like calcium, supraspinatus has a 1 cm transverse tear in the centre & the bursa is full of fluid. 1 week ago I had a radionuclide bone scan show low grade active arthropathy in the glenohumeral joint. I have dizziness, fevers, nausea, pain in my hip and shoulder joints, achey all over at times, extreme tiredness, very foggy, can't focus well, red skin rash every now and then & flu like symptoms. I have been tested for everything under the sun for my low WBC, recently I had a Short Synecthen Test and it came back negative for Addison's, my specialist thinks I may have CRPS & FM but they do not explain the low WBC, do you have any idea what it may be? Thank you :) Doctor: Hi, dearI have gone through your question. I can understand your concern.There are many cause of low WBC count. Most common cause is viral infection. Other causes are malaria, typhoid, drugs, certain inflammation etc.You should not worry much about that. Drink plenty of water and take a course of antibiotics if needed. Just repeat your total WBC count after a week. Continue your medication for rotator cuff injury.Hope I have answered your question, if you have any doubts then contact me at bit.ly/Drsanghvihardik, I will be happy to answer you.Thanks for using health care magic.Wish you a very good health."
},
{
"id": 102927,
"tgt": "Chest congestion and cough. Prescribed inhaler to treat Pleurisy. Earlier had bronchitis. Would inhaler & nebulizer suffice?",
"src": "Patient: I had bronchitis months ago and still had breathing problems, and a cough that would come more at night time. The Dr. I went to twice said it was pleurisy and prescribed me an inhaler, which has worked some, but I am still coughing some at night and feel like there is 'something' I have to cough up but there is not. There is also the stabbing pain on my left side of my chest. Is the inhaler and nebulizer going to clear this up? Doctor: Hello dear,The inhaler and nebulizer will provide symptomatic relief by causing broncho-dilation (dilating the smaller airway passages, relieving the obstruction & increasing airflow to lungs)and reduces the inflammation.But as long as the underlying cause is not treated, the symptoms will recur.Investigations like complete blood count & X ray chest will be helpful.If any infection is detected, then you need to take a course of antibiotics.Take care."
},
{
"id": 209087,
"tgt": "What could cause seven year old boy lean against chair most of the times?",
"src": "Patient: Iam a teacher and one of my seven year old boys 'self pleasures' as his mother puts it. He stands leaning against his chair much of the day. I ask him to sit down, he gets red in the face and says 'sorry', but it is constant. I havenever seen this in 23 years of teaching. Can you help? Doctor: First of all let me congratulate you for being so keen observer. Being a teacher you have noted a problem which can be a mental illness too.leaning on chair as such is not any illness but the reasons behind it or what is the thinking behind leaning and subsequent anger when opposed make me think there may be some hidden anxiety or any illness. Kindly ask his parents to evaluate him near a psychiatrist for his betterment. Also note any other abnormality in his behaviours."
},
{
"id": 146503,
"tgt": "What causes hiccups,headaches,dizziness and blurred vision?",
"src": "Patient: Hi I always get the hiccups , at least twice a day sometimes more, i suffer from headaches often and when i am laying down and get up i always feel dizzy, so dizzy i have to stop and stand still for a few seconds and my vision is completley blurred , that doesnt happen all the time, but it does happen alot Doctor: hihiccups can be related to multiple causes likeneurologically related with cerebrovascular accidents,respiratory related,gastrointestinal related,uraemia or kidney related or lastly psychogenic.all. caused due to the irritation of the diaphragmplease meet your physician to rule out the causestake care"
},
{
"id": 218702,
"tgt": "What is Hepa-Merz concentrate infusion prescribed for?",
"src": "Patient: my doctor advised hepa Merz infusion concentrate with 1000 ml ringer sol. bcz due to morning sickness and nausea i could not eat and whatever i took i just vomit out as i m in my 1st trimester. this drug is for liver, why do i need it in my condition Doctor: Excessive vomiting in early weeks, hyperemesis, may also be due to liver reasons, basically encephalopathy. That's why your doctor must have prescribed. You can also take doxylamine and pyridoxine combination as an alternative. But first get your liver enzymes checked."
},
{
"id": 110815,
"tgt": "What is the treatment for back pain?",
"src": "Patient: Hello Doctor, I have an odd question. I was a U. S. Marine and while in the service I suffered an injury to my lower back. A very long story shortened, I would like to know why sleeping in my recliner alleviates most of the pain I feel compared to when I sleep in my bed. I have tried all kinds of mattresses from expensive to cheap and sleeping on a bed always kills my lower back upon wakening. Can you give me an explanation on why a recliner helps alleviate my back pain and if there could be any issues in the long run (other than a mad wife) lol. Thank you for your time. V/r, Jay Doctor: recliner usually are not ergonomically designed, they are mostly design specific raher than anatomical correct. staying in recliner for long time can casue back pain due to static wrong posture can put all the strain on your back muscle. the pain can be suppressed if you meet a good physical therpaist who works more on movement disorder or sply muscle rehabiliation also you can adjunt with dry needling to enhance the result. but to gain permanent change its better you start your ecercises and get ergonomically desigened recliner"
},
{
"id": 110256,
"tgt": "What causes back ache,abdominal cramping and nausea?",
"src": "Patient: For approx 1.5 hours this afternoon my mid back was aching and my stomach was cramping and I had nausea. I still have all the symptom. But I can deal with them. This issue also took place about a mo.nth ago. But i just assumed it was a gas issue. Can you help me Doctor: Hi,Welcome to healthcare magic.After going through your query I think You are suffering from acute backache. Treatment of it is rest and analgesics . Diclofenac three times a day after meals is effective. Sometimes strong analgesic(such as ultracet three times a day after meals) is required. OMEPRAZOLE 20 MG before meals is taken to prevent acidity caused by analgesics .To further investigate MRI of the region is advised. I think your query answered.Welcome to any follow up query"
},
{
"id": 82665,
"tgt": "Suggest treatment for lupus",
"src": "Patient: I TESTED POSITIVE FOR LUPUS; BORDERLINE POSITIVE ANA TITER 1:40; AND ANA PATTERN SPECKLED. I HAVE JOINT PAIN WITH PSORIASIS LIKE RASH OVER JOINTS THAT COMES AND GOES. HISTORY OF BREAST AND SKIN CANCER. AGE 56, FEMALE. HOW TO INTERPRET MY MOST RECENT TEST RESULTS? AND WHAT NEXT STEPS SHOULD I TAKE. MY DOC DOESN\u2019T SEEM TO KNOW WHAT\u2019S NEXT. Doctor: Titer 1:40 is not significant, However in view of speckled pattern and psoriasis like skin lesions kindly get ANA blot, Anti Ro, Anti La done to rule out sub acute cutaneous lupus (SCLE). Skin Bx will help to differentiate between psoriasis and SCLE."
},
{
"id": 4816,
"tgt": "On implanon, taking trimethoprim, nauseous, stomach pain, negative pregnancy test. Antibiotics affects birth control?",
"src": "Patient: I've been on implanon for around 2 years, and I was taking some antibiotic's called sulfameth/trimethoprim last month, and for the past few days I've been feeling nauseous, tired and having pains in my stomach, though they don't lest very long,. but I took a pregnancy test and it had came up negative. and I was just wondering if it would affect the test. and if that antibiotics would affect the birth control.? Doctor: Hi,Trimethoprim can affect contraception to some extent. I suggest getting a pregnancy test. Please repeat a urine pregnancy test if you see your cycles and have missed them or go for the serum beta-hCG estimation which is more sensitive and specific. You should also get a trans-vaginal sonogram to review the pelvic organs. Please also remember that the antibiotic itself can cause some amount of gastric distress. Please see your doctor for further suggestions. Hope you find this information useful. Take care."
},
{
"id": 170309,
"tgt": "How effective is calpol in treating fever in kids?",
"src": "Patient: Hi.. My daughter is 11 months old and is suffering from fever for the last 2 days. We are giving her 1.25 ml Calpol drops. Fever drops and after taking the medicine and again gets increased after 4-5 hours. Shall we continue Calpol drops? She is having 100.9 now. Doctor: HiWelcome to the HCMI understand your concerns but don't worry. The most likely reason for fever in this age group is self limiting viral infections. It improves within a few days with symptomatic management. Do, continue calpol for fever spikes. If the fever persists for more than three days or there is any worsening than do visit a pediatrician for proper clinical examination and definitive treatment.Take care"
},
{
"id": 204391,
"tgt": "How can depression along with anxiety and blurred vision be treated?",
"src": "Patient: i suffer from depression , anxiety , blurred focus , ups and downs etc my doctor said i have post traumatic disorder +... had a difficult childhood with lots of trauma. my dr. recommended E.M.D.R treatment , i just have a hard time trusting, she only saw me once and had a full report and diagnose i want to hear more opinions. thank you , Miri Doctor: Hello and Welcome to \u2018Ask A Doctor\u2019 service. I have reviewed your query and here is my advice. Antidepressants works well for both anxiety and depression on long term basis so it's better you take antidepressants by a consultant psychiatrist which which makes you feel better. Regarding blurred vision, it's always better to consult an ophthalmologist and take his valuable opinion. Cognitive behaviour therapy is one modality of treatment which makes you feel better by changing your thoughts followed by changes in your behaviour, so taking cognitive behaviour therapy always helps you feel better I hope I have answered your query. Feel free to ask any further questions."
},
{
"id": 39821,
"tgt": "Do i need to put vaccine after bitten by a small puppy?",
"src": "Patient: please help me out ,i have bitten by a small puppy althouth the mother of the puppy is immunized... wat steps to take ??? infact, my injuries is too small .... jst a normal scratch.. but have pain .. when i used to wash it soap and water ,! kindly suggest me sumthing good . my number is 0000 Doctor: Puppies are more notorious than dogs as far as transmission of rabies is concerned.Yours is a Category 1 exposure. You have also done the first-aid - washing the scratch site with soap and water. However, to be on the safer side, you need to take the anti-rabies vaccine. The vaccine is to be taken on days 0, 3, 7, 14 and 90."
},
{
"id": 46701,
"tgt": "Suggest alternate medicine for kidney problems",
"src": "Patient: HII m 38yrs f with crfI hav kidney transplant.4 months agoEven after 4 months my transplanted kidney is not working properlyI m on tacrolimus and averolimus immunosupprerts with 10 mg steroidsUrine output is still not proper, createnin is1.90Kindly suggest alternative medicine Doctor: Hello and welcome to HCM.As an Urologist and transplant surgeon,i can understand your anxiety.After a transplant,it can undergo rejection at any time.That's why you're taking anti-rejection treatment.You need a doppler scan.If the creatinine levels don't return to normal levels,rejection is a possibility.The steroid dose is low,probably because you're having low body weight.You've not written dose of other immuno-suppressants,so can't comment.If you've any doubts,send all reports and medication,as a direct question.Dr.Matthew J. Mangat."
},
{
"id": 142691,
"tgt": "Feeling dizzy quite often",
"src": "Patient: Hi. I'm and 11 year old girl and I get dizzy spells a lot. When I'm sitting down or standing up it will feel like I'm about to pass out. I get them about 2 or 3 times every two days. Is this normal? And I not what might I have? Do I need to go to a doctor? Doctor: Thanks for your question dear first thing out should have blood pressure measurement while sitting and be quit at that position for 10 minute and then again take blood pressure surely there will be difference of br in two reading that's most probably orthostatic hypotention thanks"
},
{
"id": 77316,
"tgt": "What causes sharp, stabbing pain in middle of chest?",
"src": "Patient: i have been having chest pains for about three months now,it starts on the right side goes to the middle and the left side and in my back.I have had a doppler stress test done and ekg and blood work and they said it all turned out good.I still have the pains in my chest what could it be,I get sharp stabing pains in the middle.What could this be? Doctor: Hi and thank you for choosing H C M to post your concern.I read carefully your question and in my opinion your chest pain is a noncardiac chest pain since your cardiac examinations turned out good.Do you have any other complain for example cough ,difficulty breathing?I am supposing no since you don't mention in your question. This make me think that is less possible a chest pain associated with the respiratory systemIs your pain infuenced by eating or does it worse while supine,or increase in intensity with movements of the body?It would help me to have this information to narrow the differential diagnosis of your chest pain.Now my conclusion is that the most possible causes of your chest pain are as follows:Gastroesophageal reflux disease,spasm and gastritis and if this is the cause, your pharmacologic therapy may include antacids such as Ranitidine,protons pomp inhibitor such as omeprazole and lifestyle and dietary modification.Costochondritis is an other possible cause and the treatment include painkillers such as Ibuprofen.Sometimes chest pain can be a manifestation of anxiety disorders.I hope my answer can help in resolving your complain.Kind regard Dr.Dushi"
},
{
"id": 145506,
"tgt": "What causes dizziness?",
"src": "Patient: With all due respect sir/mam, I am 22 years old. I am very sad. From 8 months I am having some dizzy feeling in my head. For 2 months it was just some dizziness. Then I consulted a doctor and he gave me a neck collar. I used it for 2 weeks and found that whenever I move my neck it feels as if neck skin is stretched. And neck feels stiff. And many times when I move my neck a certain cracking sound comes. And one more thing sir. In previous 8 months l have had around 4 episodes of around 5 seconds during night where I got up from bed due to a certain sudden spinning in my head . that is very horrible experience. And I have the feeling that it may always happen again. I have to change my sides during night to averse this spinning attack. Please help me out. Doctor: Hi,Thanks for writing in.Moments of dizziness can happen due to decreased blood flow from heart to brain. When it happens, the blood flowing through the vessels from heart to brain might have a reduced flow. These are by the carotid and vertebral arteries on either sides. This is more significant when you are standing upright and doing any moderate to heavy physical activity. You might require a CT scan or MRI scan of the brain.The best way to analyze if the reduced blood flow to the brain is causing your dizziness is by doing a carotid and vertebral arteries Doppler ultrasound. It will show the flow of blood from the heart to the brain and if it is adequate.Clinical examination of inner ear and vestibular system in the ear is also required.It might help to do a Electronystagmography (ENG) or videonystagmography (VNG) done to record abnormal eye movements while the head is placed in different positions or your balance organs are stimulated with water or air. For this you might have to consult the ENT specialist.Dizziness can also occur due to stress related anxiety or overflow of emotions."
},
{
"id": 9225,
"tgt": "What causes dry patches on skin?",
"src": "Patient: I have an extreme, dry patch of skin on my right ankle that is about 2 inches long. It has recently started to discolor and does not feel anything like skin. It has started to develop on the same spot on the opposite ankle as well. There is not a history of eczema or anything that could result in this... Doctor: Hi,I can understand your concern for xerotic patch on right leg. Now the same disease also started on left leg. Most probably it may be xerotic eczema. However you consult dermatologist for firm diagnosis. You may be atopic.I would like to recommend as below..- tacrolimus 0.1% oint twice a day- antihistaminics- steroid in tappering dose- avoid soap bathI hope this would help you.Thanks..Dr. Ilyas Patel MD"
},
{
"id": 41785,
"tgt": "What are the best infertility treatments?",
"src": "Patient: hello dr,we undergone treatement for infertility since past 2 years.we did iui for 6 cycles.my husb is having less sperm count and i had pcos,i did scopy 4 months back .now dr advised me to go for icsi .at present we stopped treatment .what shall i do now dr Doctor: Hi welcome to healthcaremagic.I have gone through your question.As you had pcod polycystic ovarian disease and husband has low sperm count and failed 6 cycles of iui, i would advise to continue the treatment and go for ICSI intra Cytoplasmic sperm injection or IVF invitro fertilization.Consult a gynecologist and take his/her advise for further treatment.Hope i answered your question.Would be happy to help you further.Take care."
},
{
"id": 132248,
"tgt": "What causes pain from armpit to the wrist?",
"src": "Patient: I have a pain in my right arm that starts under the arm pit down the arm ending just above the wrist. there is also some small bruises that appeared under my skin and I didn t bump into anything. It feels like there is a piano wire in my arm that is pulled tightly. I m wondering if this is something I should be overly concerned with or will it go away on its own Doctor: If you have some small bruises and blisters along a line for a few days - then probably you are suffering from herpes zoster infection.It normally goes on it's own but for pain relief you should consult your doctor"
},
{
"id": 25581,
"tgt": "How to improve ejection fraction rate?",
"src": "Patient: I'm a 33 year old post hrt attack. Im about 5 wks out. I'm an avid weight lifter which I do 4 to 5 days a week. My ef was 35% after the mi. I had 4 stents in my RCA. I also had one placed in my circ. My question is can I get my ef to get better??? Also will I ever be able to lift weights again. My cardiologist said as long as I feel up to it down the road. Then another cardiologist in the office said absolutely not. My history is hyperlipidemia of genetic cause. Doctor: Thanks for your question on Health Care Magic. I can understand your concern. In my opinion, you should should avoid heavyweight lifting and strenuous exercise. This can precipitate heart failure. In your following drugs should be started to improve ejection fraction (EF). 1. Strict control of dyslipidemia. So optimal dose of lipid lowering drugs like atorvastatin, rosuvastatin, finofibrate etc. 2. Low dose beta blocker like metoprolol. It is having cardiac protective role and proven benefit post MI (myocardial ischemia). 3. ACE (angiotensin converting enzyme) inhibitors like enalapril, captopril etc. Same benefit as beta blocker. 4. Digoxin. Increases force of contraction and improves EF. 5. Low dose diuretic to prevent fluid overload. So discuss about starting these drugs with your cardiologist to improve EF. Hope I have solved your query. I will be happy to help you further. Wish you good health. Thanks."
},
{
"id": 218445,
"tgt": "How can sensitivity on the right side of the ribcage during pregnancy be treated?",
"src": "Patient: Hi, During and after pregnancy, I ve had rib sensitivity on the right lower side. Massages in that area hurt so good, that super soar muscle kind. I thought it would right itself over time, but it s been five or six months. Most of the time it doesn t bug me, unless I do a certain motion, or push on it. Anything I can do to treat it? Doctor: Hello and Welcome to \u2018Ask A Doctor\u2019 service. I have reviewed your query and here is my advice.I think your problem cones from gallbladder and is not related to pregnancy but is just a coincidence. You may have colecistitis or gallbladder stones. You have to do an ultrasound of gallbladder to see it and a total blood count and biochemical liver tests.Hope I have answered your query. Let me know if I can assist you further."
},
{
"id": 87079,
"tgt": "What are the symptoms of potential appendicitis?",
"src": "Patient: I had my appendix out in march, beacuse i had pain in my right hand side they wernt sure if it was my appendix or not so they had a look inside but could not find anything else so they took my appendix out now since having the operation i still get pain in my right hand side the doctor told me it will just be scaring but the last few weeks the pain has got alot worse, i feel sick, pain in the right hand side, pain up the side of my right side of my back, high temperature, hurts when i go for a wee just at that side of tummy, going for a wee more than noraml. I have been signed of sick now for three weeks! I have had lots of tests done, eg blood tests, urine samples, ultra soundscans, internal examination but found nothing! any ideas???? I have seen a gynecologist at the hospital and they did an internal and found nothing but now the doctors are sending me back to them Doctor: Hi.Thanks for your query.I can understand the plight you must be going through as you are having pains in spite of the surgery for looking inside, which was normal and they removed the appendix.The symptoms of potential appendicitis are : Pain starts in the upper abdomen and shifts to the right power quadrant, with or without other symptoms of nausea, vomiting, fever , loose motions or constipation; the pain can be mild or very much severe. Your problem looks to be related to IBS: Get remaining tests of CT scan of the abdomen and Colonoscopy to rule out other diseases. If all the investigations are normal , this is a case of IBS- Irritable Bowel Syndrome and you will be fine."
},
{
"id": 35216,
"tgt": "Suggest treatment for severe hip pain after staph infection treatment",
"src": "Patient: I was treated for a staph infection with IV antibodics for 6 weeks. Then I stayed on Cefuroxime for 6 months (500mg twice a day. When I stopped taking the Cefuroxime, the pain in my hip became so bad that the Hydorocodone I was taking could no longer control the pain. I have had several blood test and my infectious desieze doctor assures me that I do not have any remaininig or reoccuring staph infection. How can the cefuroxime have such an enourmous effect on my pain???? Doctor: Hello there,I am dr.milan an infectious disease specialist answering your question,I also agree with your doctor. Because sudden pain after stoping of antibiotic is not the feature related to staph infection. If infection is remaining than you have associated symptoms like fever, bodypain and many more.You have not mention your age here. I think you need to see an orthopedic surgeon. Because in later age our bone became weak and slight straining may lead to so called pathological fracture. You just go through x-ray examination and along with your serum calcium level. According to that calcium supplement can be started.You need to do suggested things.Hope i have given appropriate guidance to you.if you have any query you can consult me anytime.Give me star rating according to your satisfactory level.Thanking you."
},
{
"id": 202126,
"tgt": "Is this method of masturbation harmful?",
"src": "Patient: I am 18years old. I'm play with my penis and rub it against the bed till sperms come out. I'm doing this since last 4 years. Now days I try to control ,Iam able to but not always.Is this harmful? Will this have any effect on my sexual life? Will I be able to become a father? Doctor: Hello dear,First of all, I wish to tell you that Masturbation is common among young males in their teenage and it does not have any bad effects on health unless you get addicted to it.Since you are feeling that masturbation is interfering with your normal functioning, you can try out the following measures:1. Improve your social life by spending more time with friends and relatives.2. Keep your mind active and busy at all times either through curricular or extra-curricular activities.3. Have a diet rich in fruits, vegetables, fish, nuts & honey.4. Avoid fast foods & lipid containing diet.5. Avoid smoking & alcohol.6. Keep away stress, think positive.7. Exercise regularly & practice meditationThese measures will be helpful in improving concentration & builds up the confidence level.Take care."
},
{
"id": 205817,
"tgt": "What causes pain and anger when on Nordette?",
"src": "Patient: The clinic has changed my pill frm oralcon (5mnths) , to nordette( 2) month this, im angry al da tym, weepy, light pain on lower abdominal side( ryt). Spotting,could this b a side effect, on oralcon i did have sum problems but they stoped aftr my 3rd cycle. Doctor: DearWe understand your concernsI went through your details. Most side effects of Nordette are not serious. The most common such effects are nausea, vomiting, bleeding between menstrual periods, weight gain, breast tenderness, and difficulty wearing contact lenses. These side effects, especially nausea and vomiting, may subside within the first three months of use. Anger and irritation are not commonly seen as side effects. You could be facing these problems because of your anxiety towards this drug. Consult your doctor.If you require more of my help in this aspect, please use this URL. http://goo.gl/aYW2pR. Make sure that you include every minute details possible. Hope this answers your query. Available for further clarifications.Good luck."
},
{
"id": 184245,
"tgt": "Can getting a wisdom tooth extracted cause headache?",
"src": "Patient: I have flu like symptoms after having a wisdom tooth extracted. Could this be related to the extraction procedure? My symptoms are....skin on my head is tender to the touch and I have an ear ache along with generally feeling out of it. The strange thing is the discomfort I'm feeling is all on the opposite side of the extracted tooth. The tooth was pulled a week ago and the symptoms began 2 days ago. Doctor: Hello, Thanks for consulting HCM, Read your query, as you hae headache and earache after one week for extraction this can be due to viral infection or it can be due to weakness or to carious tooth present in opposite arch or impacted tooth . Dont be worried so much you should consult physician for these symptoms headache and earache and if you wont get relief then consult dentist for examination of oral cavity. Inmeantime take propr nutritious healthy diet , take proper sleep , dont take stress.Hope this will help you. Wishing you good health.Regards, Dr. Priyanka tiwari"
},
{
"id": 206757,
"tgt": "Suggest remedy to overcome fear of everything",
"src": "Patient: Hello, I have developed a fear of pretty much everything. I am afraid to be alone, I am afraid of every stranger I see. I can't sleep at night because I am worried someone is going to break in, or is watching me through the window. I can't walk alone outside because I'm afraid I'll get attacked. This is affecting my everyday life. Please help. Doctor: Hi.I understand your concern. It is anxiety of being fearful. Phobia is one of the type and it can be social phobia or specific phobia.Fear of being attacked can be categorised in psychotic anxiety. So fear and anxiety of everything require further evaluation to categorised in types or form confirm diagnosis. So consult psychiatrist and get help. It can be treated with psychotherapy. CBT means cognitive behaviour therapy is the choice. Exposure and response prevention/flooding etc can be useful.Medicines like combination of SSRI and benzodiazapine will be useful. Treatment can be possible after proper consultation with psychiatrist. Get well soon. Happy to help you further. Thank you."
},
{
"id": 158580,
"tgt": "Precancerous cells, delayed period. Undergoing treatment. Is that normal?",
"src": "Patient: I had my first smear test in February, I was told I had precancerous cells, I had treatment to have them lazered off. Everything has gone well, through all that I decided to stop taking the pill. I have had my period since stopping the pill and I'm not due again until 1st may, however I have started bleeding again now, with everything that's been going on down there, is this normal after stopping the pill? Doctor: Hi, What for the smear test has done? Precancerous cells means what? The exact report to be produced. Anyway, bleeding after stopping the pill is normal but type nature of bleeding to be looked for. Clinical examination is necessary. Right now nothing to worry. Probably you want to say about the report of pap smear, in this case pap smear report to be repeated after 6 months. Take care."
},
{
"id": 44827,
"tgt": "Is having 5% rapid motile sperms normal ?",
"src": "Patient: Count 80 million per ml MOtility Rapid progressive 5% slow progressive 20% immotile 70 % MOrphology Normal forms 60% Is it good ???? Doctor: hello, welcome to HCM sperm moility to be 50% or greater is good indicator.but you have 20 %. rest of the tWo factors morphology and count is good. take care"
},
{
"id": 82350,
"tgt": "Does the scar appear in the X-ray after recovering from TB?",
"src": "Patient: hello Doctor Goyal... im kim barrera.... From philippines.... i have an minimal pulmonary tubercolosis laST six months.... and this month i done successfully my 6 month medication and my doctor said that i am cured.... my question is.... it is possible that the it have a scar when it undergone to my medical examination to my respective shipping agency? if it have a scar how would it be remove? and what medicine can remove that scar? to have negative result in xray? thank you Doctor: Thanks for your question on HCM.In my opinoon you should not worry about lung scar. Thrre is no treatment for lung scar because it is harmless and it is inactive.In any lung insult whether infection with TB other bacteria or any lung injury, healing occurs by these way.1. Fibrosis2. Calcification3. Bronchiectasis4. Very rarely complete resolution.Fibrosis and calcification are most common. So once you have taken the treatment, lesionis healed and produce fibrosis, which looks like scar.And this scar will remain as it is. No treatment is available since it is dead, inactive tissue. So no need to worry about it."
},
{
"id": 179617,
"tgt": "What is causing the fever and headache for my 9 years old?",
"src": "Patient: my 9 year old has been sick for a week. Last Monday her temp was 103 it has going down today its 100. she gain a cough she says head hurts like if someone is hitting her in the head with a hammer. one min she can be fine the next she cant move,,, I have tried to get her in with her Dr but they have on openings till Friday. What could this be? should I take her to the ER Doctor: Hi High fever with headche for nearly one week I highly recommend you to consult emergency if your regular doctor is not available.thanks"
},
{
"id": 204405,
"tgt": "How to deal with focus issues at work?",
"src": "Patient: I have no inspiration to do work. I tried so many times tryin to get up from my situation. Yeah i read motivational quotes..they inspired me..but it didn t last long. I always get back here again. Lack of motivation. I try so many times but I always failed to do it..now I m desperate. I didn t have the strength to wake up anymore. I didnt trust myself anymore. What could I do? Pliz help I really really feel hopeless Doctor: Hello and Welcome to \u2018Ask A Doctor\u2019 service. I have reviewed your query and here is my advice. Usually external motivation may not work consistently and therefore watching motivating videos or reading stories may not work well. What you need is internal motivation which comes from external motivation. You need to know how to cultivate internal motivation after watching videos or reading books. Motivation therapy should help you.Hope I have answered your query. Let me know if I can assist you further. Regards, Dr. K. V. Anand"
},
{
"id": 153851,
"tgt": "Suggest tests to confirm brain cancer",
"src": "Patient: Hi, I have been acting differently lately and my curious and imagintaive side of me searched up cancer possibilties that I might have. But, to my dismay, I had a match. I think I might have brain cancer. I have most of the symptoms, minor, but still. Doctor: Hi, dearI have gone through your question. I can understand your concern. If there is suspicion of brain cancer then you should go for investigation. MRI brain is the first step. Then if needed stereo tactic biopsy should be done. It will give you exact diagnosis. Then you should take treatment accordingly. Hope I have answered your question, if you have doubt then I will be happy to answer. Thanks for using health care magic. Wish you a very good health."
},
{
"id": 109481,
"tgt": "What is causing bloating of stomach and lower back pain?",
"src": "Patient: my stomach is bloated I was on iron tablets but they ve made I carnt go to the toilet so I stopped taking them.] last few days ago.i feel sick and hungry all the time and I have lower back pain its like sometimes its like symtons of period pain but im not due for another 8 days and I take the pill so I knw. Doctor: HIThanks for posting your query to Healthcaremagic. Low back pain could be due to many reasons like Sitting or sleeping in wrong postures , Spine Problems , Inter vertebral disc problems , Muscular spasms, Kidney stones or kidney infections, or Pelvic Inflammatory diseases. I need to know whether the pain is intermittent or continuous in nature ? Is then any discharge from Vagina ? Any burning or pain in Urine ? Also get an Ultrasound Abdomen pelvis scan , X ray lumbar spine and urine routine test done and revert back . Hope this information was useful to you. Any clarifications feel free to ask ."
},
{
"id": 210486,
"tgt": "I am fuzzy headed after the intake of medicines. Easy way to cure?",
"src": "Patient: I have been on steroids and xanax for over 5 years. I am so fuzzy headed all the time that I can'function. It is not dizzy but fuzzyheaded. I have been to many doctors but none seem to be able to help me. I did go to phychiatrist last week and she cut back one xxanaz but still no better. I take 3 a day lowest dose available . Can anyone please help me? Doctor: DearWe understand your concernsI went through your details. I suggest you not to worry much. The symptom you are describing as fuzzy head is just an imagination or illusion. Let me explain. What will be the condition of a person who gets up at 9 O clock on a sunday morning and has nothing to do? His head will be fuzzy and he will be lazy. The fuzziness is due to the laziness. Being active, cures the laziness. Same is your case. You are being lazy or the anxiety problems makes you lazy and thereby fuzzy headed. Be active for a fortnight or so and you shall see the difference.Please post a direct question to me in this website. Make sure that you include every minute details possible. I shall prescribe some psychotherapy techniques which should help you cure your condition.Hope this answers your query. Available for further clarifications.Good luck."
},
{
"id": 73335,
"tgt": "What causes chest infection, coughing, indigestion with pyrosis?",
"src": "Patient: hi had indigestion fo years and was taking losec but after a realy bad chest infection and severe coughing, istarted with heartburn and ache that goes through to back on left side and a feeling of something moving like when your pregnant but this only happens when i sit down doc gave me lansaprole but dont seem to be helping thanks Doctor: Severe indigestion that leads to lung problems can be serious.There are more advanced treatment options for bad indigestion and reflux that include surgical procedures. You can talk to your gastroenterologist about these options. In the meantime, I would recommend that you sleep with a wedge pillow, with your torso angled up at your hips to help prevent acid from regurgitating into your airways and lungs, which can cause several major problems. If you don't have a wedge pillow, you can use something to elevate the head of the bed (like bricks under 2 corners of the bed)."
},
{
"id": 212725,
"tgt": "Difficulty concentrating, feeling sleepy. What is the problem?",
"src": "Patient: I report to work at 7.30am. am usually alert and productive till 10.00 when i become sleepy, slowed down and cannot concentrate though am not tired physically, i only feel sleepy till around 1.00 when i take lunch, work for an hour then the sleepiness comes again. Please note that i go to bed at 11.00pm and sleep till 5.00. what could be the problem and how can i help it? Doctor: Hi there ~ What you describe to me is a typical circadian rhythm (day night time table) sleep problem. It is quite possible that you feel sleepy after meals as is normally the case. However sleeping late like in your case and waking up early may result in less hours of total sleep time which may result in daytime sleepiness. It is also quite possible that you may be able to undo this by sleeping early and waking up at the same time. You would just need to put behind all those things you were doing closer to bedtime to an earlier time and start winding down early to go to bed. If this, and relaxation while trying to sleep are still not helping, you may need to consult a doctor. I hope this helped. Take care."
},
{
"id": 36592,
"tgt": "Need treatment for fungus infection in groin",
"src": "Patient: fungus infection in groini am having fungus infection in groin ; went to skin specialist who advised me candid b creme at night n zimig creme in morning along w tyza 1od allegra 1 daily; now my medicine dosage is over can i continue creme if so foe how longl Doctor: Tinea cruris is an infection of the groin caused by a fungus which is dermatophyte. It is a superficial skin infection. Treatment with an antifungal cream(Azoles) usually works well. 1.Wash your groin daily. 2.Change underwear daily. 3Check for athlete's foot 4.Do not share towelsTreatment is usually advised for 2 weeks.Take care!"
},
{
"id": 45108,
"tgt": "What is mean rupture in follicular study ?",
"src": "Patient: I am 32 year old trying for baby since last 3 years. the recent ultrasound report confirmed that there is a endometriotic/ haemorrhagic cyst in right ovary . the lesion measures 6.1x5.1x52cm (85cc). there are two opnion of the Doctors i) remove it through laproscopc surgry and then go for pregnency ii) directly go for IVF. which i should prefer I am in dillema! Anju Singh Doctor: Hello anju; welcome to HealthcareMagic If it is an endometriotic/haen=morrhagic cyst it requires treatment.Since you are 32 yrs and can take chance of getting naturally pregnant so you can go for a laparoscopic removal of cyst and if you do not conceive the IVF will be option which will be open even after 2 yrs also.As for the follicular study it is a procedure where sonography is done to find whether the size of the follicle produced by ovary is increasing to the desired size and then the follicle ruptures or bursts releasing the egg which is fertilized by the sperms and it is necessary during fertilization so that you can have pregnancy. I hope i have answered your query. Thanks"
},
{
"id": 204224,
"tgt": "Is the feeling of persistent sickness a symptom of a mental disorder?",
"src": "Patient: my sister is always claiming to be sick with one thing or another. As soon as she solves one problem then another arises the same day. she talks about her health all day long and seems to love talking about it . I cant take it any more. Does she have a mental problem ? She goes to the dr all the time , at least every week. Doctor: Hello and Welcome to \u2018Ask A Doctor\u2019 service. I have reviewed your query and here is my advice. Yes, she seems to be suffering from illness anxiety disorder or hypochondriasis. In it their is a persistent belief in the presence of at least one serious physical illness underlying the presenting symptoms, even after repeated investigations came negative they have persistent preoccupation with presumed deformity; along with this they persistently refusal to accept the advice and reassurance of several different doctors that there is no physical illness or abnormality underlying the symptoms. She needs to be shown to a psychiatric clinic. Hope I have answered your query. Let me know if I can assist you further."
},
{
"id": 4435,
"tgt": "How can nacfil forte help in conception?",
"src": "Patient: Hello Doctor,We are planning for a child for almost 8-9 months and recently my wife was prescribed with Nacfil forte. She is having some small irregularities in her periods, but not consistent. May I please know what could this medicine help in (Nacfil forte)?Thank you. Doctor: Hello. Thanks for writing to us. This is most likely a multivitamin supplement to increase the general well being which will help in releasing a healthy egg. It is not a specific treatment for infertility.I hope this information has been both informative and helpful for you. Regards, Dr. Rakhi Tayal ,drrakhitayal@gmail.com"
},
{
"id": 74915,
"tgt": "What could cause passing out with bright red face and profouse sweating while on pulmonary function test?",
"src": "Patient: I had pulmonary function test this a.m. During the first part of the test I passed out while blowing the air out of my lungs at fast volume. Technician said I turned bright red in the face, began sweating profusely, then started trembling as I passed out. He called my cardiologist, who said repeat the test. He did, with the same result. What possible causes are thre for this, and do I need follow up? I had nuclear stress test two weeks ago, results were normal. Doctor: Respected user, hi I evaluated your query thoroughly.*Relates with functional disability of your lungs to cope up with the test requirements.*May be due to less immunity of lungs to resist certain level of stress to bronchoalveolar system.*I suggest regular deep breathing exercises with YOGA and fresh air walking to rejuvenate your lungs with enough quota of oxygen.*Avoid smoking / alcohol if consuming.Hope this clears your query.Thanks for using Health care magic & reviewing my answer.Wishing you a very happy healthy life ahead.Regards take care."
},
{
"id": 157991,
"tgt": "IBS, cervical cancer, cysts in liver. History of bacterial infections. Treatment for symptoms?",
"src": "Patient: I have been diagnosed with bacterial infections for over 10 years and am wondering if dr. maybe need to look for something else that could be wrong. I was diagnosed with IBS 2 years ago after having colonoscopy and removal of unremarkable polyps and then diagnosed with cervical cancer this March and also have found cysts on my liver (still waiting for consult regarding liver). Doctor: First of all, i would like to know which part of the body are you having bacterial infections in. If it is not a particular site and you have been having infections all over, then you need to rule out two things--Diabetes and HIV infection (both can be ruled out by simple blood tests, blood sugar fasting and post-prandial + serum ELISA for HIV). IBS itself can lead to repeated gastrointestinal infections in which case you might need to be started on medications fro IBS in consult with a Gastroenterologist. You have not mentioned about the treatment you have taken for cervical cancer. If you have taken radiotherapy then that can cause repeated urinary tract infections and rectal infections. Cysts in the liver should not concern you at all as they are benign lesions which dont produce any problems and dont require treatment"
},
{
"id": 219447,
"tgt": "What causes contractions during pregnancy?",
"src": "Patient: hi doctor .. my name is akanksha ..m at my 24th week of pregnancy ..frm last few days was feeling some contractions so doctor asked me to take duvadilan - 10 mg (twice a day) ans susten 200 ..after dat there was relief ..but frm yesterday in site of taking regulat medication i can mild contractions which last for fraction of second. Is this normal ? Doctor: HIWell come to HCMI really appreciate your concern, if this is the symptoms then it need to be brought to investigation and in my opinion better to get done the ultrasonography, you can not take this as granted because of medicines, hope this information helps."
},
{
"id": 17327,
"tgt": "What causes fluctuation in blood pressure with nausea & headache?",
"src": "Patient: hi, i have recently a blood pressure dias 90 - 100 but its not constant sometimes it goes below 90 . am male 50 yrs old , no history of seriuos diseases , but this i feel its due to tension and hot weather . am now in saudi arabia and the weather is hot , i feel nausea and headache and sometime dryness in tongue . do i need a therapy ? Doctor: Hello, Dehydration can be a common cause to your low Blood pressure switch you should keep yourself hydrated and get your general body checkup done which includes CBC, KFT, Thyroid Function test, ECG and a Echocardiography done and consult a physician with reports for further treatment. Hope I have answered your query. Let me know if I can assist you further. Take care Regards, Dr Bhanu Partap, Cardiologist"
},
{
"id": 96388,
"tgt": "I am having hernia. Some times i feel vomiting sensation",
"src": "Patient: i am having hernia.some times i feel omitting senstion .iam taking omaze tablets 20mg in the evening . can i take onother capsule in the morning also? iam having bilateral& umbilical hernia iam 56.iam at present doing share online trading b.p.130/125 Doctor: you can take one more but if oyu are having nausea you should see your surgeon as it can be can be due to the hernia getting stuck...."
},
{
"id": 66957,
"tgt": "Could bump on top of forehead indicate bone cancer?",
"src": "Patient: Hi - I saw my family doctor in October and showed him a small bump at the top of my forehead, where the hairline starts. He ordered an x-ray of my skull which showed density changes suggesting possible metabolic bone disease, and has ordered a bone densitrometry test and extensive bloodwork. He is ordering these as he thinks it could be that the parathyroid gland is overactive, and if that is the case, that would be removed. I asked him if there is any possibility of cancer, and he said possibly, but that he feels it is more related to the parathyroid gland. I asked him if I am diagnosed with cancer, what type of cancer could it be? He told me it would bone cancer. What are your thoughts? Doctor: Hello and welcome to HCM,Overactive parathyroid glands lead to formation of brown tumor in the bones.Brown tumor are formed due to over activity of bone forming cells.'Brown tumor' is a misnomer since it is not a tumor and it is a manifestation of metabolic disturbance caused by hyperparathyroidism.A couple of investigations are required before diagnosis can be made and management can be done.An aspiration cytology of the swelling is required and this investigation can differentiate between a bone tumor and brown tumor caused by hyperthyroidism.Thereafter, blood tests for calcium and parathormone also need to be done to assess the function of parathyroid gland.In case these investigations suggest any abnormality of parathyroid gland, specialized scan for parathyroid will be required to assess whether there is enlargement of all parathyroid glands or enlargement of single parathyroid gland.You need to consult an orthopedician for these investigations and thus further work up.Thanks and take careDr Shailja Puri"
},
{
"id": 45708,
"tgt": "What could nausea and dehydration post kidney dialysis suggest?",
"src": "Patient: A friend of mine was suffering from kidney failure. She went on dialysis and improved so much that the doctor decided to remove her from it for a week as a trial run. She got sick with nausea, diarrhea, and dehydration and is now in the hospital. Can you tell me what is going on? Doctor: Hi, Diarrhea and vomiting may be due to acute gastroenteritis or electrolyte imbalance or dehydration or anemia or septicemia or hyperuricemia etc. Until examination is done it is difficult to say what it is. Use tablets Ondansetron. Use tablet Sporlac. If symptoms not improved please consult your physician/nephrologist and get it done blood urea and serum creatinine, he will examine and treat you accordingly. Hope I have answered your query. Let me know if I can assist you further. Regards, Dr. Penchila Prasad Kandikattu, Internal Medicine Specialist"
},
{
"id": 89368,
"tgt": "Suggest treatment for abdominal pain after laproscopic gall bladder surgery",
"src": "Patient: I just had laparoscopic gallbladder surgery yesterday (about 36 hours ago now) and I m in unbearable pain. The Percocet they gave for pain isn t helping and I m not even taking it. I have no complaints of pain aside from not being able to take a deep breath without pain in my upper right torso area. Is this normal? I ve read it can take years to go away and sometimes it never does go away. Anyways, second question, would I be okay smoking marijuana two days after surgery? I know smoke in general isn t good for the healing process, but would there be any chemical reactions? I m joining the Marine Corps in October and I need pain relief so I can focus on getting back on my feet. Doctor: Hi! Good morning. I am Dr Shareef answering your query.Even though you might have some pain and discomfort in the operated area, it should not be unbearable with the dose of analgesia prescribed. If I were your doctor, I would avoid recommending you to smoke marijuanha and refer you to your surgeon for a review check up for the unbearable pain. Till then, I would consider prescribing a proton pump inhibitor and an antispasmodic drug for you.I hope this information would help you in discussing with your family physician/treating doctor in further management of your problem. Please do not hesitate to ask in case of any further doubts.Thanks for choosing health care magic to clear doubts on your health problems. Wishing you an early recovery. Dr Shareef"
},
{
"id": 225582,
"tgt": "Using two months injection, trying for a child. How to use the birth control tablets?",
"src": "Patient: Good Day, My name is Charlene, 32 year old lady. I've been on the 2months injection eversince the birth of my daughter, she's turning 15 in Oct. I would really want to have a child by next year, God willing. Any advice what to use as the clinic sister I've been visiting said that i have to use the birth control tablets 2 weeks before my injection leaves my body. I want to know on which one is the best one to use? Doctor: Hello. Thanks for writing to us. Since you have already taken an injection for birth control, now it is not possible to reverse its effect. You will have to wait for at least two months before trying for conception.I hope this information has been both informative and helpful for you. Regards, Dr. Rakhi Tayal ,drrakhitayal@gmail.com"
},
{
"id": 172677,
"tgt": "Should i consult a doctor as my baby hit his eyebrow bone on the cabinet?",
"src": "Patient: My 7 month old grandson was sitting on the floor and tried to crawl forward and hit his eyebrown bone on the cabinet. He cried and then shortly after spit up alittle-not vomit but spitup. He calmed down quickly and seems fine. Should he go to the doctor Doctor: Hi dear,I understand your concern, you should observed baby. If baby will not have vomiting, headache, will not cry continuosly,then don' t worry. Apply Pilex to affected area with gentle massage 3-4 times for 5 days"
},
{
"id": 173716,
"tgt": "Can i give calpol,vizylac,atarex,distaclor and tustel for high fever?",
"src": "Patient: Hi my child is 13 months old and is suffering from high fever 103.5 F doctor has advised foll medicines he is suffering from fever since last 4 days kindly suggest wheather the prescription is right or not,Calpol 250VizylacAtarexDistaclor 125Tustel txThanks,Jitendra Jadhav Doctor: Hi... I feel it is too many medicines for a simple viral illness. Fever of few days without any localizing signs could as well a viral illness. Usually rather than fever, what is more important is the activity of the child, in between 2 fever episodes on the same day. If the kid is active and playing around when there is no fever, it is probably viral illness and it doesn't require antibiotics at all. Once viral fever comes it will there for 4-7 days. So do not worry about duration if the kid is active.Paracetamol can be given in the dose of 15mg/kg/dose (maximum ceiling dose of 500mg) every 4-6th hourly that too only if fever is more than 100F. I suggest not using combination medicines for fever, especially with Paracetamol.Hope my answer was helpful for you. I am happy to help any time. Further clarifications and consultations on Health care magic are welcome. If you do not have any clarifications, you can close the discussion and rate the answer. Wish your kid good health.Dr. Sumanth MBBS., DCH., DNB (Paed).,"
},
{
"id": 213954,
"tgt": "What are these symptoms of my dad who is alcoholic feeling drowzy, taking burps and restless ?",
"src": "Patient: Hi, my father is 54 years old and is an alcoholic. He recently was admitted in the hospital for behaving abnormally. He was feeling drowzy, was taking burps, was in half senses and was very restless. LFT,KFT nd other tests are all normal and the ammonia test showed that it is high in comparison to what should be the normal in the body. But he got recovered and came back but it has been three days only and he is again showing all those symptoms. What shall we do and how shall we proceed on with the treatment? please help.. Doctor: Hi Mr/Ms Gupta, It seems that your father is suffering from delirium (acute confusional state). The reasons for delirium in a patient with alcohol use are so varied (both physical and psychological) that he will need a physician and a psychiatrist working together. Has he stopped drinking suddenly? If so that is not good and if he is not being supported by some medication to stop his withdrawal then he is bound to have problems. Please check it up with whoever is treating him. Good luck to you both"
},
{
"id": 79053,
"tgt": "Suggest treatment for a sharp pain in the mid sternum",
"src": "Patient: I m a 20 year old new mother. recently having a weird tight/sharp pain in my chest mid sternum. and the last day or so I ve had a salty taste in my mouth. the past week or 2 I have been sick with cough and congestion. any serious reason for the chest pain? Doctor: HIWell come to HCMI really appreciate your concern, mid sternum chest pain could be due to hyperacidity, salty test in mouth indicates the presensce of pus of blood it might be due to dental infection, cough congestion could be due to allergic condition, and this can be managed with antihistamine, no need to worry about this, hope this information helps, take care and have a nice day."
},
{
"id": 150035,
"tgt": "Had lumbar fusion and foot drop. Started epidurals. Got vertigo after stopping Metatoporal. Suggestions?",
"src": "Patient: I had lumbar fusion 2 weeks later L partial drop foot accompanied with cramps/pain jan 9 2013, Arril April 2013 i have a senomiand cyst 3-4 c. I started with a series of 5 epidurals (after lumbar fusion jan 9. Stopped taking Metatoporal/Xanax) After #2=vertigo for 1 week After #3 everlasting strong vertigo. High BP. I returned to 25 mg Metatoporal and 20 mg Zanax = 2 x day. Vertigo all but left I m ready for #4 of 5 epidural next week. Doctor: Thanks for writing to us. Metaprolol is for blood pressure and fluctuation in blood pressure can cause vertigo. You should continue the medicine. Epidural shots has no direct relationship to vertigo. I did not get the point why you are taking epidural block after fusion. WHat is the cyst never heard of? Thanks."
},
{
"id": 38673,
"tgt": "Is ketoconazole or miconazole effective for fungal infections?",
"src": "Patient: My 7 yr old son has been diagnosed with a fungal infection. It s over 15% of his body and pretty angry. Doc prescribed ketoconazole for small patch on his face when we thought body was infected wounds from falling off his bike.Saw nurse yesterday and she said these are also fungal and prescribed miconazole 2%.Which should I be using please? Doctor: HI, thanks for using healthcare magicBoth of these are anti fungal creams and you can choose to use either one to treat his rash.It can sometimes take 2 to 3 weeks in some persons for improvement to be seen because fungal infections can respond slowly to treatment.It is important to use the cream consistently.If the extensive rash does not respond to topical treatment then he may need an oral antifungal.I hope this helps"
},
{
"id": 21663,
"tgt": "Suggest methods to control blood pressure",
"src": "Patient: Hello sir,my father is in ITBP police,and during his service he face many problem like high blood pressure due to which some heart problem is also there,they take many medicine to control Bp but none are effective,please give me guidance to cure these problems Doctor: Hi ThereI understand your concern for your father and instead of prescribing you something here without examining your father I would like to advice you to take your father to a cardiologist and after proper evaluation start him on medication properly so that he will be benefited.Good luck"
},
{
"id": 102003,
"tgt": "How to treat recurring allergic reactions, medicines offering temporary relief?",
"src": "Patient: Dr., My son have 8 years old. He suffering allergic complaints. & dr. prescribed to take Montek Lc tab for 2 months. He OK for the 2 months. But when stop that tab he fully in allergic again. Is there any problem to continue that tab? any side effects? Doctor: HI, long term continuous use of montelukast can lead to several side effect including gastrointestinal disturbances, dizziness, fever, rash, sleeplessness, weakness cough, nasal congestion, angioedema, arthralgia, cholestasis and other liver problems, bleeding tendencies and few others also. he should go for allergen test to get the list of them and then try to avoid the exposure to them , if respiratory tract is also involved then special precaution should be taken and steroids can be added when required and regular follow up is also required.other drugs can be used depending on the condition and for this you should consult a physician. wish you all the best."
},
{
"id": 145760,
"tgt": "Suggest treatment for plexiform neurofibroma in abdomen",
"src": "Patient: My grandson has plexiform neurofibroma in his abdomen! Will it continue to grow if not treated? If he has the NFi gene, can it be treated with meds? If not what kind of treatment is available. The mass in his abdomen also shows it s wrapped around major arteries! What now? Doctor: hi,Thanks for writing in.Plexiform neurofibromas in strategic locations are to be removed through skilled surgery. It is important to know the size and extension of the tumor and if there are any cancerous areas which are developing within. In that case, you must consult an oncosurgeon and vascular surgeon and know the implications of the tumor causing pressure effects or the rare possibility of the tumor infiltrating the arteries. Medicines are not a treatment option for these tumors. Though the tumors are usually benign and harmless, increase in size with time and malignant transformation cannot be entirely ruled out in your grandson. That might take years to happen but it can be easily prevented by getting treated through surgery at present."
},
{
"id": 72342,
"tgt": "Should the antibiotic for bronchitis be taken to complete the course?",
"src": "Patient: Hi, may I answer your health queries right now ? Please type your query here... \u00a0\u00a0\u00a0\u00a0\u00a0 I had bronchitis. And now I am better, and have a slight and occasional cough. should i still take the antibotic given to me, and completete the course? even if im better? Doctor: Thanks for your question on Healthcare Magic.I can understand your concern. Yes, you should definitely finish the entire course of antibiotics given to you. It is good that you are better now. But you don't finish the antibiotics, infection might relapse and cause recurrent bronchitis. There is always chance of drug (antibiotic) resistance if you don't finish complete antibiotics.So better to finish whole antibiotic course given to you despite of improvement. Hope I have solved your query. I will be happy to help you further. Wish you good health. Thanks."
},
{
"id": 16721,
"tgt": "Suggest remedy for fluctuating BP due to a concussion",
"src": "Patient: I have post concussion syndrom,my blood pressure is raising and lowering from 190/100 in the morning to 160/86 during the day I m on blood pressure meds and it s been increased but I don t seem to stabalize, my concussion happened 3 1/2 weeks ago they did a cat scan of my head and neck,how long can all this go on Doctor: Hello, I would explain that these blood pressure fluctuations are not very common after concussion. For this reason, I would recommend checking thyroid hormone levels for thyroid gland dysfunction and some other blood lab tests (complete blood count, PCR, ESR, kidney and liver function tests). Hope I have answered your query. Let me know if I can assist you further. Regards, Dr. Ilir Sharka, Cardiologist"
},
{
"id": 57278,
"tgt": "Suggest remedy for fatty liver",
"src": "Patient: hello doctor my husband is 35yrs old, he never uses alcohol and smoking but he is having fatty liver.his cholesterol level is HDL-34 and LDL- 171,his bp is 110/70.can you please tell me how he must have got this fatty liver.what are the methods that can be used to overcome this fatty liver?his weight is 68 and height is 175cm Doctor: Hello Thanks for writing to usMost commonly fatty liver is due to obesity and sedentary life.It is also related to alcohol intake.Your husband need need few other investigations like routine hemogram,Random blood sugar(RBS),Liver function test(LFT).He should try to lose your weight if he is overweight.Fatty liver is a reversible condition and it can be reversed by diet and life style changes.He may need some medicines after full investigations.It may take 10-12 months to reverse the findings.He should be active in your daily life. He should avoid red meat and high fat milk products.Cook food in olive or canola oil.He should take limited salt in his diet.He should also avoid fried and junk food.Take lot of fruits and vegetables.He should also increase his physical activity in the form of walking and exercise.Hope i have answered your query.Take Care Dr.Indu Bhushan"
},
{
"id": 155848,
"tgt": "What are the symptoms of postpartum desmoid tumour?",
"src": "Patient: I believe (after an ultrasound for something else yesterday) that I have what is known as a postpartum (baby born 2 years ago) desmoid tumour. I have a specialist appt next week, but am wondering if I should go to A&E earlier as it is starting to be quite uncomforatble. Doctor: Hi and welcome to HCmno,there is no need to hurry with treatmnet since this is benign condition and just be patient and folow your doctors instructions, Wish you good health. Regards"
},
{
"id": 2505,
"tgt": "How to get pregnant?",
"src": "Patient: Hello Sir. I am 28 years old and my wife 27 year old. We get married 2 year ago. Noe we are planning to have a Kid. We are doing sex since last 3-4 months but everytime my wife gets period so what should we do. and what she should eat to give a birth to healthy child.. Doctor: Hi, You both are very young & have been trying since 3-4 mths, let nature take it's time. Child is God's gift , wait for it. Don't burden yourself with investigations / treatment.But if you are very anxious , kindly get investigated.Husband - Semen analysisWife - Tube patency test- HSG / SSGIf all normal , Have sex daily or on alternate days once the periods stops till day 10 of periods( Day 1 is when bleeding starts). By this elder sperms are discarded.Have abstinence of 2-3 days & then have sex daily , any number of time for 4-5 days, which is the time of ovulation.Life of sperm is 3 days & egg is 1 day. By this egg gets to meet healthy young sperms & pregnancy chances are highNo special food for increasing pregnancy chances. Avoid food which make you obese All the best"
},
{
"id": 56739,
"tgt": "What does this ultrasound report of liver indicate?",
"src": "Patient: Hi i am female, my name is seema gupta upadhyay, i did an ultrasound today and the report says, liver enlarged in size (14.0cm. Cranio-caudal) and shows normal echotexture. There is no intra-heptic biliary dilation. A 20\u00d716 mm haemangioma is seen in segment IV of liver. No evidence of any other focal lesion. My gp has has not to worry, but wanted expert opinion. Doctor: HI Welcome to H C M I am Dr B M GUPTA from Delhi answering your question.Your GP is right.-Get Blood for L F T (Liver Function Test)-Take Vaccination against Hepatitis A & B-Advice not to take Herbal Drugs(many a times harm to liver)Hope you are satisfied ,if not inquire again.Thanks"
},
{
"id": 155342,
"tgt": "Suggest treatment for malignant melanoma",
"src": "Patient: I was recently diagnosed with malignant melanoma on my left callve. I am having surgery wednsday and the plastic surgeon might do a skin graft. If I have it done will they admit me into the hospital? My insurance does not cover the machine they want to give me. Doctor: Thanks for your question on HCM. For skin grafting hospital admission is must.And you have to take post operative chemotherapy to prevent local as well as distal spread. Skin grafting needs daily dressing and wound care for atleast 2 weeks.So 1 week hospitalisation is necessary and advised.So better to discuss these with your treating plastic surgeon before the procedure."
},
{
"id": 3603,
"tgt": "Why am I not getting pregnant?",
"src": "Patient: Hello Dr. Hi, my name is Stenicha, my boyfriend and I been trying to have a baby for over two years now... for some reason I don't know if I can't pregnant (I really don't wanna think this is the case) but I would like for you to give me some tips please...I also started taking New Chapter prenatal vitamins yesterday but don't really know all the details for me.... please get back to me.Thank you Doctor: Hallow Dear Stenicha, Motherhood is every woman's natural instinct and you are no exception to it. For pregnancy, mainly following factors are responsible:1. Woman has to release egg. This can be ascertained by different examinations and investigations. Though ovulation detection kit is available for domestic tests, the best way to go for ultrasonography. Daily scan for ovulation monitoring from day 9 onwards till ovulation occurs is very useful. Along with the evidence of ovulation, it gives information about the size of follicle and endometrial thickness at the time of ovulation. Follicles between 18 mm and 21 mm size discharge the healthiest ova. Endometrium between 9 mm and 14 mm is most conducive for implantation of fertilized ovum.2. Patency and normalcy of the Genital tract can be assessed by hysterosalpingography, Laparoscopy and Hysteroscopy. 3. Semen examination to find out the quality and the quantity of sperms and some other factors in the semen. So please to your Gynaecologist and get these investigations done to find out the status of the normalcy of your fertility. Since you are trying sincerely for 2 years for pregnancy, in vain; you need to get these evaluations done. Based on the reports of these investigations, further management can be planned. With these reports, you may ask me Direct question for further guidance. Dr. Nishikant Shrotri"
},
{
"id": 221684,
"tgt": "What causes spotting during pregnancy?",
"src": "Patient: hi i am 12 weeks pregnant.i had 3 drops bleeding yesterday. scan report said ok with my baby.doctor said hormone imbalance gave me an injection n hormone tablet. today i got back pain n slight blood is going alternatly. today also i took hormone injection by docter s prescription. why is it happening . Doctor: It's threatened abortion that means chances of abortion is there. Take full bed rest with prescribed treatment. If still bleeds or amount increases then consult your doctor and go for scan"
},
{
"id": 72752,
"tgt": "Suggest precautions to be taken for flu",
"src": "Patient: I am just recovering from a case of the flu or a reasonable facsimile of the flu. I am supposed to fly to a wedding this Wednesday.........my sis in law just had surgery, my parents are elderly and my niece is 8 weeks pregnant.would it be safe for me to go? Doctor: Thanks for your question on Healthcare Magic.I can understand your concern. Yes, you can definitely fly and attend the marriage ceremony. Flu is caused by influenza virus. It is infectious only in the initial part of disease. Since you are improving in flu, no need to worry spread of virus by you. Drink plenty of fluids orally, do warm water gargles and steam inhalation 4-5 times a day. Don't worry, you will be alright in 1-2 days.Hope I have solved your query. I will be happy to help you further. Wish you good health. Thanks."
},
{
"id": 145314,
"tgt": "Suggest treatment for vestibular schwannoma",
"src": "Patient: Recent MRI to rule out vestibular schwannoma (resulting in unilateral hearing / balance loss) reported \"cerebral white matter T2 signal changes compatible with chronic microvascular ischemic disease\". Am I having mini-strokes? Inner ear and auditory canals listed an normal / unremarkable. How would you define chronic microvascular ischemic disease. (I was a nurse before entering engineering field.) Doctor: Hello. I have been through your question and understand your concern.Microvascular white matter disease is a normal feature found on most of the MRIs after a certain age. There is no treatment to this condition and is not really a stroke or ministroke. These are just perfusion changes in the level of artherioles, not artheries, which are thought to be related to hypertension.You should not be worried. This is a normal MRI.Hope this helps.Wish you the best health"
},
{
"id": 161195,
"tgt": "What causes rhabdomayosarcoma in the testicle of a child?",
"src": "Patient: my son was diagnosed with rhabdomayosarcoma in his right testicle in dec 2008. we was 5yrs old. it was stage 1 and had the testicle removed no other signs anywhere else, he did chemo for 6 months , and has been great since then. dec 10/2010 ct scan showed left hilar node in medinastal , enlargment to 11 mm. what are the chances of this being cancer? he has been cancer free for 18 months since end of chemo. in the last 6 month he has had chicken pox, with secondary infection, streap throat and ear infection as we speak, also has some exema spots on his legs. going to do biopsy but sugery sounds scary because they might not be able to get it with scope and might have to cut him open and spread the ribs. very worried parent !! thank you Doctor: Hello, The cause is unknown. we pray the Almighty that he will be free from neoplasm. hilar node may be due to some infection. Do not worry. he will be alright. Rabdomyosarcoma is rare tumor with out any known risk factor. Happy that you diagnosed it early and treated it. Hope I have answered your query. Let me know if I can assist you further. Take care Regards, Dr Rajmohan, Pediatrician"
},
{
"id": 126724,
"tgt": "How can vibrating sensation and pain in the leg be treated?",
"src": "Patient: My legs vibrate for hours after driving a car for long periods of time in a similar way one might feel vibrations from movement of a car. It\u2019s bothersome. Also I have pain in my right leg and some cramping from excessive driving. I have two hip replacements. Any helpful suggestions? Doctor: Hi, The vibrating sensation in the leg is likely to be related to a nerve related cause. Methylcobalamin supplements and nerve conduction study can help. Hope I have answered your query. Let me know if I can assist you further. Take care Regards, Dr Praveen Tayal, Orthopaedic Surgeon"
},
{
"id": 12527,
"tgt": "Using propysalic ointment for psoriasis. Feeling nausea. Need a cure",
"src": "Patient: dear sir I am suffering from Psoriasis for the past one decade.I am currently only using Propysalic ointment and one mosturizer. I feel nausea when i take methotraxate. I have also tried Homeopathic and ayurvedic medicines but none seems to work. Currently my condition has worsened and the ointment is not able to control the same.Kindly help me Ved Prakash Doctor: Hello Ved Prakash.. Thanks for sharing your query here. Topicals in the form of Propysalic ointment can be used if involvement is up to a certain percentage of body area by Psoriasis. Beyond a specific limit, you will need oral /injectable therapy which unfortunately has the potential if side-effects ,if not monitored closely. For a chronic condition like psoriasis , treatment regimen is modified according to the severity of disease ,which might be different in different times of the year.So, a close follow-up with your dermatologist is important for optimum result and achieving a remission. For the methotrexate that you are using , side effects like nausea and vomiting may be dose-dependent.Therefore, several physicians consider adjusting the dose or modifying the mode of intake to eliminate the problem.The total dose of Methotrexate can be divided & taken over 2 days every week; or you may take it at bed-time ,if taking during the day causes nausea; it can also be take after food ,instead of before . Still if nausea persists , you may consider switching over to injectable Methotrexate once a week . Several other drugs are available for treatment in psoriasis and include, Retinoids, Cyclosporin, PUVA therapy. I would advise you to discuss these with your dermatologist and continue the therapy.Hope the is helpful Take care!"
},
{
"id": 79150,
"tgt": "Feeling shortness of breath after having a quadruple bypass surgery",
"src": "Patient: My 71 year old brother had quadruple bypass surgery about three weeks ago and has his first visit to the cardiologist tomorrow. My brother still has shortness of breath and is on oxygen, he is sleeping in a chair in the rehab afraid to lie in bed because he cannot breathe when lying down. What questions should he be ready to ask his cardiologist in his visit tomorrow. Oh, he is also a diabetic. Doctor: Thanks for your question on Health Care Magic. I can understand your brother's situation and problem. Breathlessness on lying down is not a good sign after bypass surgery. This suggests heart failure due to poor pumping of heart. So he should ask for 2d echo to to determine heart function. About treatment, diuretics and inotropic drugs (this drugs increase heart pumping) are useful in his case. So discuss about starting these drugs in his case. Also ask cardiologist about daily fluid intake and food. Hope I have solved your query. Wishing good health to your brother. Thanks."
},
{
"id": 92543,
"tgt": "Have lower abdomen pain, bloating, radiating to lower back, more urination. What could be this?",
"src": "Patient: I have lower abdomen pain kind of like constant and bloating and it radiates to my lower back and it's constant there or sometimes it'll do the opposite but they usually both hurt at the same time. And for the most part it's not sharp it's more dull or ache. I just hold it when it hurts and the pain slowly just gets worse. And today I've peed more then usually but I drank cranberry juice. Doctor: Hello,The symptoms are suggestive of Urinary tract infection.In this situation I would advise my patient to continue drinking Cranberry juice as it is effective against UTI's.You may need an ultrasound of the abdomen and urine test to rule out UTI.Prescribe antispasmodic for pain and a course of antibiotic for infection.Consult and discuss with your doctor. He will help you with the diagnosis and treatment of this condition."
},
{
"id": 200646,
"tgt": "What is the treatment for chronic prostatis?",
"src": "Patient: I HAVE A CHRONIC PROSTATIS SO FAR NO BACTERIA FOUND IN URINE I AM SUGGESTED TO TAKE DAPOSURE TABLET AND TAMFLO TABLETS, I STARTED TAKING THIS MEDICINE AND GOT RELIEF FROM SYMPTOMS. BUT I STOPPED FEARING SIDE EFFECTS, WILL PROSTA-Q OR Q-UROL WILL HELP TO GET RELIEF FROM SYMPTOMS Doctor: Thanks for asking in healthcaremagic forum I appreciate your concern. Many times you may not find bacteria in urine though you have infection. You have not mentioned your age and associated illness if any. So, consult your urologist regarding supplements(Prosta Q) for prostatitis as medical data regarding this is not valid. Also I would like to advise you not to stop any medications without doctors consultation. ALl the best."
},
{
"id": 19145,
"tgt": "What is causing my chest pains and shortness of breath?",
"src": "Patient: I have been having chest pains, shortness of breath, and a abnormal symptom.. I sometimes \"forget to breath\". My brain will tell me it is time to breath but by the time it gets to that point I start gasping for air. Is there any reason you think this could be gapping? Doctor: Hello!Welcome and thank you for asking on HCM!I understand your concern and would explain that your symptoms do not seem to be related to any cardiac disorders. Anxiety seems to be playing an important role in all this clinical situation. Coming to this point, I would recommend consulting with your attending physician for a careful physical exam and some tests: - a resting ECG and chest X ray study- complete blood count for anemia- thyroid hormone levels for thyroid dysfunction- blood electrolytes. If all these tests result normal, you should consider anxiety as the main cause of your complaints. Hope to have been helpful to you!Wishing good health, Dr. Iliri"
},
{
"id": 41131,
"tgt": "Suggest treatment for infertility problem",
"src": "Patient: hi, i m 26yr old married lady and i want to be a pregnent but i have a problem related to periods. my doctor adviced me to take kimson 35 for 4 months. i have taken it for 4 months. but still my follicle size is not incresing. pls give me solution for that as i want to b e pregenent Doctor: Hello, after stopping krimson the chances of pregnancy are very high.In case still follicle size is not increasing then take FSH injections to increase the size as much as possibleIn case you have any questions in future you can contact me directly on http://bit.ly/drmanishajain"
},
{
"id": 163488,
"tgt": "How to treat Graves disease in a diabetic child?",
"src": "Patient: I just was informed my 15 year old granddaughter who since birth has been a Type 1 diabetic has now been diagnostic with Graves Disease. I use the web for input, but can you point me in the direction I should be using.Thank you. As my granddaughter calls me OPA BILL Doctor: Hello,In Graves disease, you have to start thyroid tablets daily basis, thyroid function will improve. As you told baby having type 1 diabetes from an early age, continue insulin, diet management, regular sugar monitoring. Even diabetes will improve as a treatment for Graves disease started.Hope I have answered your query. Let me know if I can assist you further.Regards, Dr. Sachin Kumar Agarwal"
},
{
"id": 54828,
"tgt": "Is hepatitis B curable after taking Tenofovir Disoproxil Fumarate?",
"src": "Patient: Dear Sir, I m 39yrs old, weight - 62k height- 5.5inch . I have Hep B virus. Recently I checked below HBV DNA - Positive HBV DNA copies / ml - 49450 ( pls. advise the normal range) S. ALT (SCPT) - 27.0 U/L ( Nor. Range- upto42 ) S. Creatinine - 90.0 micron mol/.L ( Nor. Range- 60 ~130 ) AFP (Alpha-Feto-Protein) -2.81ng/ml HBeAg - Negative I dont feel any other problem right now and just start the treatment here in Bangladesh. We can not discuss with doctor more as they are very busy and spend few minutes for a patient. Here one physician advised to take the medicine -- Tenofovir Disoproxil Fumarate 300mg -- One/daily. Doctor advise to take long time only. Is HEP B curable or what is the function of this medicine ? Is there any side affect ? Pls. advise. Pls. advise Doctor: Hi thanks for contacting HCM.This drug you have mentioned prevent viral multiplication when disease is in active stage.You have positive HBV DNA ...Here yes this drug can be taken....But it is not guaranteed that cure will there.Main aim in carrier or chronic hepatitis is to prevent complications..Take low fat diet.Trans fat like cheese, butter , non veg less.Avoid alcohol.Papaiya like fruit good.Grined carrot and spinach leaves juice useful Hope your concern solved...Dr.Parth Goswami"
},
{
"id": 200423,
"tgt": "Suggest medication for warts on penis",
"src": "Patient: Hello sir, I have some warts on my penis since 10-12 yrs, I recently consulted with a dermatologist and he prescribed VDRL test (result-non reactive) and also Azithromycin, Acutret(Isotretinoin), Vitamin (D3), and Fusidic Acid cream however there are not mejor effevt in 15 days. Is it some bacterial infection or any viral infection? Please note that I never ever have any type of sexual activity or any exposure to drugs. I smoke regularly since last 3 years. Your suggestion matters as I do not have a very well qualified doctor here. Thank you and god bless you. Doctor: hello,thank you for your querythis wart problem takes long time to cure..drugs you were prescribed are appropriate..continue those.you can also use podophyllin or imiquimod newerdrugs.now a days cosmetic surgery is also very popular for this wart problem.you consult a dermatologist who is efficient in these special therapies.also i want to tell you that wart is a std.so have protection by barrier method during sex.hope this is helpfulregards."
},
{
"id": 26925,
"tgt": "What is the treatment for high blood pressure?",
"src": "Patient: i havebeen prescribed novarsk, due to the elevated blood pressure. i am 65 and had incidents of bp 160/95 etc.all systems doing Ok, echo heart had came up with a slightly leaking valve.I am frightened taking any heart related Rx, including blood thinners Doctor: Hello. Thank you for your question and welcome to HCM. I understand your concern. High blood pressure (hypertension) is a condition that should be maintained under tight control, \u0131.e. under 140/90 mmHg, because there are a lot of studies that confirm that a long-standing, above these figures, blood pressure produces other organ damages, such as kidneys and eyes. Therefore, \u0131f the f\u0131gures you provided are constant findings, this is a sign that an another blood pressure-controlling drug should be added to your therapy. I would recommend you a beta-blocker, which not only lower the blood pressure, but also reduce the heart rate, thus the cardiac workload is reduced.Combining your age and hypertension, you have at least two known risk factors for coronary artery disease. So, by the body of studies that exist, aspirin should be added to your therapy as a primary prevention (I assume by blood thinners you meant aspirin). The thing with the minimally leaking valve is not to worry about, at least at this moment. It may become an issue after several years, or never. I recommend you to discuss with your cardiologist the possibility of adding these two drugs to your regimen. And, last, I suggest you to run blood levels of total cholesterol, LDL and HDL fractions, and triglycerides. I hope I was helpful with my answer. Best regards. Dr. Meriton"
},
{
"id": 179190,
"tgt": "Suggest treatments for upper abdominal pain,vomiting and diarrhea",
"src": "Patient: My son is 17years old. He has been hurting from the upper part of his abdomen for the last 10-12 days. He can t keep food down and has lost 9 pounds in 12 days. We took him to 2 ERs and they gave him a couple of meds to help rebuild the stomach wall. He continues to throw up and we are just waiting for a GI specialist next week. Any help? Please help? Doctor: He is suffering from gastritis. Avoid to remain in empty stomach for more than 3 hrs, avoid too sour, too hot and too spicy food for next 2 months. Give him Tab Rablet D 1tab daily in empty stomach for 6 weeks. Gradually his symptoms will come down. Also try to reduce if any stress is present in his life... If the symptoms are not resolving after 15 days do a blood test for serum amylase and lipase, and upper GI endoscopy."
},
{
"id": 17695,
"tgt": "Suggest treatment for heart attack",
"src": "Patient: after echo test dr. advise me for angiography and ask when i got heart attack before then i went to hospital where echo and tmt are dione and found positive i am not interested in angiography they refer me metrolol 30mg, ecosprin 100mg, isosorbide mononitrate 20mg,crestor 10mg please advise me ; my heart health and can it be cure by any means Doctor: Hello, After reading your details I would like to tell you that without doing Angiography is like trying to hit a target without seeing it. It\u2019s not possible to heart a heart disease without knowing how severe it is. It\u2019s advisable that you should get an Angiography done first so that disease can be identified properly and then treated accordingly. Hope I have answered your query. Let me know if I can assist you further. Take care Regards, Dr Bhanu Partap, Cardiologist"
},
{
"id": 173935,
"tgt": "How safe and effective is Daytrana patch for reddish passage of urine?",
"src": "Patient: My 7 year old son has been on the Daytrana patch since last Monday. He gets .7mg of Daytrana. He had reddish urine tonight, I thought it might have been blood. The patch was accidentally left on for almost 12 hours today. He told me a few days ago his urine had also been red. Is htis because the patch was not removed promptly after 9 hours? Doctor: Thanks for question.I understand your concern. Color of urine depends from taking food, medicines,flavors. When child takes methylphenidate side effects occur :-Black stool-blood in the urine or stools-blurred vision -convulsions-crusting, dryness, or flaking of the skin-muscle cramps I suggest:1.Give urine analysis.2.Take Charcoal 1 tablet /10kg for absorption of medicine3.Vitamin E 1 capsule once for 1 month.4.Ascorutin 1 tablet once for 1 month, Dicynone 1 tablet twice a day for 2 days for blood in urine. Take care Dr.Svetlana Shrivastva"
},
{
"id": 56443,
"tgt": "Suggest treatment for liver cirrhosis",
"src": "Patient: Hello sir i am saksham jain from indore my mother is cause by liver cihhrosis. It was also having a dermatomaitocytosis. It may be taken a long term steriod like wysolon or deflacart during 14 years please suggest me a good treatment of the liver problem. In the body the fluid will be increase so it may cause also swelling problems . i need your help please suggest me a good treatment. Doctor: Good morning I am sorry to hear about the illhealth of your motherLiver cirrhosis with dermatomyositis indicate that she may be suffering from autoimmune hepatitis. She may need life long steroids +/- azathioprine (immunosuppresant) for her diseaseDepending on the stage, cirrhosis of liver may have multiple manifestations like swelling of the body, bleeding, low protein, abdominal swelling, renal failure, disorientation, poor sleep, jaundice etc and each one of these symptoms needs to be treated separatelySo I would suggest you take her to meet a gastroenterologist for futher evaluation and treatmentHope this would answer your questionWould be happy to answer your further questions Wish you a speedy recovery"
},
{
"id": 38150,
"tgt": "What to expect after getting treatment for dog bite?",
"src": "Patient: Hello doctor,myself Manoj gupta, yesterday one sleeping on stair of one complex and wrongly I stepped on dog.then he bite me on leg by one teeth.I washed it and went to hospital, doctor cleaned it with betadine and given me rabies vaccinesen along with TT .then advised me to wash by bit warm water after reaching home.washing done after an hour. That dog was street dog.so kindly advice. Doctor: Hello, thank you for your contact to health care magic. If I am your doctor I suggesting you that as you have taken a good primary care chances of you getting rabies decreases. Though some 0.01 % chance is there for rabies. You can track the dog for 10 days if he is not missing for 10 days than it has no virus in his body, and so you are complete free from infection.If you have to ask me anything you can contact me. Dr Arun Tank. Infectious disease specialist. Thank you."
},
{
"id": 207818,
"tgt": "Can Prozac be continued for depression symptoms?",
"src": "Patient: I was taking Prozac 20mg a day for 10 weeks i started to feel better, then i stopped without asking my doctor, now after 2 months i started to have some depression symptoms againthe question is can i started taking Prozac again i have talked to my doctor and she prescribed a Prozac but she did not tell me any thing about how is it going back to Prozac she was seem in hurry. is it going to be okay to get back using Prozac again? Doctor: HiI admire you for positive outlook.Dear usually 1st episode of depression require treatment up to 6 to 9 months after complete recovery.Yes you can start prozac again and continue it up to 9 month even after you feel better.Consult in between to your doctor but you have to take regularly for better outcome.Thank you."
},
{
"id": 163704,
"tgt": "What causes itchiness in ear?",
"src": "Patient: Hi, have got a 7 month old bay her ear itches badley, i have taken her to her doc. several times they say theres nothing wrong with her ear, my gut feeling sais otherwise. i've notice when she drinks her bottle\"luke warm\" it itches really bad ,can it be because of her allergies? is there something i can use? Doctor: Hi...by what you say this seems to be an eczema.You can use Atogla lotion for external application thrice a day for 6 weeks.Skin conditions are best diagnosed only after seeing directly. I suggest you to upload photographs of the same on this website, so that I can guide you scientifically. Please revert back to me with images so that I can guide you better.You can approach me at the following link.Once the page opens there will be an option below my image as \u2013 ASK ME A QUESTION \u2013 click on it.Please find the link below -www.healthcaremagic.com/doctors/dr-sumanth-amperayani/67696Regards - Dr. Sumanth"
},
{
"id": 86884,
"tgt": "What causes persistent stabbing pains in the abdomen followed by nausea?",
"src": "Patient: I have been having really sharp stabbing pains in my abdomin but sometimes its just feels like pressure i have had it for a month now and its constant everyday but the pain comes and goes i went to the er finally and all they said was it was pain of unknown etiology i also have nausea at times. what could this possibly be? Doctor: Hi.Thanks for your query.Pain in abdomen for the last one month, which is sharp at sometimes and just a pressure feeling sometimes with nausea and Doctor saying this as of unknown etiology is not acceptable unless these are investigated properly.I would advise you the following in such pain in abdomen:-Tests of blood, urine and stool: particularly for diabetes, liver and thyroid and kidney functions.-Ultrasonography to start with and Contrast enhanced CT scan of the abdomen to confirm or rule out certain problems.-Physical examination by a Surgical Gastroenterologist still has the main place in proper diagnosis hence get a reference for it. -Upper GI Endoscopy and / or Colonoscopy as may be indicated. - In doubtful cases, I would like to do Diagnostic Laparoscopy and sos procedure. A proper diagnosis is possible with the advent of the newer machines and once a diagnosis is done, it is just possible to have a proper treatment."
},
{
"id": 38427,
"tgt": "What are the precautions for ebola virus?",
"src": "Patient: My grandson travels all over the world. I have called CDC in Atlanta to inquire what precautions may be taken against the Ebola virus. I was told that a detergent or bleach base disinfectants, however, they were unable to tell me of specific products. Referred me to MA Dept. of Health which was not helpful at all. Doctor: Hello, Thnx to contact us. If I am your treating doctor I would like to advice you that take care that body fluids, blood and sexual intercourse may spread ebola to you. kindly stay away from such a activity. You can use a bleach solution which is available in your local market. There is no point of using specific products. If you have anything else to ask please contact me. Thanx. Dr. Arun Tank; Infectious Disease Specialist"
},
{
"id": 972,
"tgt": "Does washing vagina after sex lower chances of pregnancy?",
"src": "Patient: Hi, may I answer your health queries right now ? Please type your query here...We are planning a baby for two months..first my wife feels a lot of pain when i try and insert my penis..also i want to kn she cleans her self 15mins after the act. does this also reduce the chances of her getting pregnant Doctor: Washing the vagina (douching) does not reduce the chance of conception."
},
{
"id": 59072,
"tgt": "Lipid profile tests done. Suggestions and treatment?",
"src": "Patient: Sir,I did my lipid profile on 10/05/13 and the results are : S. Triglycerides is 242TOTAL CHOLESTROL 165HDL CHOLESTROL 35LDL CHOLESTROL 81VLDL CHOLESTROL 48LDL / HDL RATIO 2.33TC / HDL RAIO 4.77Shoul I tame any medication for this and if yes which medicine you will recommend which is very effective/safe and no side effects. kindly respond.Ramesh Bahirwani Doctor: Hi,Welcome to HCM,Your Lipid profile shows normal total cholesterol and LDL, the TG(Triglycerides) are slightly elevated as desirable is less than 150 mg/dl,the other abnormality is low HDL which is good cholesterol.Looking to this report you do not require any medicine but take care of diet with adequate amount of fats and carbohydrates reduction and regular exercise with avoiding alcohol and smoking lifestyle changes and repeat test after 3 months.You should understand every investigation is to be correlated to history and clinical examination.It laso depend on your age and other risk factors.Please visit your physician and take proper advice about lifestyle modifications.You can also take up yoga and meditation to reduce level of stress.Please take careGood Luck Dr.Akhilesh Dubey M.D."
},
{
"id": 57075,
"tgt": "What causes stomach pain and bloating?",
"src": "Patient: My dad was diagnosised with end stage cirrhosis about 8 months ago.... He has varices bleeding and has had banding done on those.... he recently has another bout with varices bleeding and was passing it in his stool also... they banded again... now he has some stomach pain and feels bloated... is his disease getting worse... Doctor: Hi and welcome ot HCM. it can be caused by several causes. in most cases this is related to diet so he hould eat less irritative food. also this can be sign of gastritis gerd or hiatal hernia or obd and ibs. to rule these out her should do gastroscopy ad colonsocopy. Wish you good health. Regards"
},
{
"id": 62070,
"tgt": "Suggest treatment for painful hard lump on the labia majora",
"src": "Patient: I have a semi hard lump near my bikini line. It s developed in size and gotten bigger during the past two days, where it makes it very very painful to walk. I think it may be a boil or carbuncle. I ve been placing a warm towel over it to sooth the pain, it helps a little. If I try to walk around it is extremely painful Doctor: hi.based from your description, it could be any of the following lesions: cystic or an abscess formation. other lesions must also be ruled out. it is best if you consult with a doctor, preferably a gynecologist, for clinical evaluation. diagnostics and management (medical and/or surgical) will be directed accordingly.hope this helps.good day!!~dr.kaye"
},
{
"id": 69295,
"tgt": "What is the treatment for lump and pain in the left breast?",
"src": "Patient: Hi chetna ma'am.I am 24 years old, and from past few months i am having pain in my left breast. As such the doctor didn't find any lump but when i assessed i can feel the tender point. first the doctor suggested me primosa for a month but i didn't feel any sigh of relief. than they change it and started natvie fort and doctor told me to have it at least for four months. still the pain is same after having the medicine for 2 months.my height is 5 feet 2 inches and wieght is 63.doctor also told me that i may be suffering from poly cystic ovaries. but i am not finding any symptoms PCO.please suggest me what should i do? Doctor: Hi.Thanks for your query and an elucidate history.The most probable cause of your condition is wither fibro-adenisis or rarely fat necrosis or underlying costo-chondritis. In costo-schondritis the pain can be pinpointed on the chest wall and increases on deep pressure, no lump will be found.On fibroadenosis , the lump can be felt only on pinching and not on palpating by flat of a palm.I would advise you to undergo mammography, ultrasonography, FNAC if needed. This will give guideline for futher management. If the pain is amenable to the usual treatment you can request Doctor to get the area removed and get the histopathology done.IF the lump is small, it will neither distort the breast nor will there be a pain as the lump will go away."
},
{
"id": 66201,
"tgt": "Suggest treatment for lump in stomach",
"src": "Patient: I sometimes feel a lump in my stomach about 2 inches in diameter on the right side about 3 inches below my rib cage...it is not painful but extremely annoying. It is not always there and seems to move to the middle of my stomach. Is this somethin I should worry about? Doctor: HiWelcome to hcmThis must be a lipoma that is fat lump. But rarely it can be hernia so you should see a surgeon for confirmation. Lipoma doesn't require any surgery unless it is too big or bothering you. Hernia always need surgery.Regards"
},
{
"id": 63195,
"tgt": "What causes a large lump on the neck above the thyroid?",
"src": "Patient: my 11 yr old son has a hard large lump in the front of his neck, above the thyroid, below the chin. he says it feels like pressure, a little gagging sometimes, and nauseous due to the pressure. it is obvious when felt. he states it started mid summer. Doctor: it can be an ectopic thyroid gland or a thyroglossal cyst. ectopic thyroid gland is abnormal position of the thyroid gland high in the neck. thyroglossal cyst is a fluid filled swelling along the path of descent of thyroid gland during fetal life which presents later in life even though it is congenital swelling. also it can be a prominent thyroid cartilage considering his age"
},
{
"id": 9693,
"tgt": "Is there any way to remove the stitch mark ?",
"src": "Patient: hi, Am 22 years old and 8 months back i got a surgery done on my nect near the chain line. its very much noticable mark and it has a bulging skin on the stitch area. i wnat to get rid of this. is there any way to look out. Doctor: Dear Jaeshela, Welcome to Healthcaremagic! You are suffering from hypertrophic scar or keloids. There are a number of treatment options available. 1st get it properly diagnosed by doctor who performed the surgery. If it's a simple scar or hypertrophic scar, you can go for scar revision surgery. Fraxel Laser is also a good option. Topical gels like Hexilak or silicon gel/sheets can also be used. For keloids, best results are seen with intra-keloidal injections of triamcinolone. Good luck."
},
{
"id": 124031,
"tgt": "Suggest treatment for hand and neck pain",
"src": "Patient: Hi Doctor, my dad has a hand and neck pain. He cannot raise the hand beyond the shoulder. If strained, by doing gardening, driving the van and stuff the pain becomes intensive. It has been like this for about 6 -7 months. How can this be cured? Thank you, Kitty Doctor: Hello, I think you should start providing the exercises for the upper limb. Like lying down in the mat and trying to do all the 4 movement's of the shoulder like flexion, abduction and extension in positions like supine, side lying and prone. This method is called the gravity eliminated method. Post this you can try with gravity resisted position and post this try with resistance. Change the exercises to standing and later if needed take the help of a physiotherapist who can guide if any shoulder mobilization is needed. Hope I have answered your query. Let me know if I can assist you further. Regards, Jay Indravadan Patel, Physical Therapist or Physiotherapist"
},
{
"id": 202571,
"tgt": "What is the remedy for the tightness in the foreskin that never gets retracted?",
"src": "Patient: Hi, i have a very tight skin which never gets retracted. I have been to a doctor few months ago and he suggested circumcision is the only option for me. I just wanted a second opinion about this. Is it advisable to get circumcised at an age of 25? How much does it cost in India for circumcision procedure? Doctor: HelloThanks for your query,based on the facts that you have posted it appears that you have tight foreskin which prevents retraction of foreskin over glans penis This is called as Phimosis.The only treatment option is to get circumcised ,This is a simple surgery done by qualified General Surgeon done under local anaesthesia as a out patient procedure and offers permanent cure so as to have enjoyable sexual life in future.There is no harm in doing it at the age of 25 years .Dr.Patil.pa"
},
{
"id": 99313,
"tgt": "What causes itchy red spots all over the body after taking hot bath?",
"src": "Patient: hello ive got 2 main problems. one is whenever im taking a hot bath i randomly start to itch and red spots apear all of my body? and my second problem is my testicles have been hurting for a few days now and ive discovered red vains and little red bloodclots? hope you can help me Doctor: Hello,Thank you for asking at HCM.I went through your history and would like to make suggestions for you as follows:1. 1st problem, itching and red spots appearing immediately after exposure to hot water, I would first think of cholinergic urticaria. It is a very benign condition that requires no need to worry. Usually hives (red spots with itching) also appear following heavy exercise, sweating, how water, etc. Some patients develop such symptoms only after warm water. 2. For above, I would suggest you to avoid hot water bath. If it is not avoidable, I would suggest you duration of hot water bath to reduce exposure to hot water. 3. Regular application of calamine lotion will help improve your skin and might reduce symptoms over time.4. Even despite above steps, should you develop itching and redness, I would suggest you to take antihistamine medications such as levocetirizine or cetirizine or fexofenadine on as-and-when-needed basis to control symptoms.5. Regarding the 2nd problem, painful testicles with redness, I would suggest you to consult your family doctor who will examine the genitalia directly to come to conclusion.Hope above suggestions will be helpful to you.Should you have any further query, please feel free to ask at HCM.Wish you the best of the health ahead.Thank you & Regards."
},
{
"id": 223110,
"tgt": "Should I stop taking BC pill if I have no sexual activity?",
"src": "Patient: Hi,, I would like to ask, Im on my 5th day of pill free period now after my 2nd pack of Althea and my last sexual contact was April 25.. and now its May 13th and I dont have my period yet.. I took the pills regularly and at the right time everyday. my concern is I want to stop taking pills since I dont have any sexual contact anymore. Is it okey to stop? Am I pregnant? Doctor: Hello and Welcome to \u2018Ask A Doctor\u2019 service. I have reviewed your query and here is my advice. 1) Yes. Why to take it ? Hope I have answered your query. Let me know if I can assist you further."
},
{
"id": 79438,
"tgt": "Can i get TB again inspite of being treated for it?",
"src": "Patient: Hi, Doc.MINTZ i have a tuberculosis last year and i finish my medication after six months.my question is,There is a tendency that my tuberculosis will comeback again?what should i do indorder not to occur again,im am very busy with my work i think im always over fatigue. Doctor: Thanks for your question on Health Care Magic. I can understand your concern. Yes, there are chances of tuberculosis recurrence in 5-10% cases even after successful completion if treatment. Common causes for TB recurrence are 1. Diabetes or patient is on immunosuppressive drugs like steroids or chemotherapy drugs. 2. HIV infection and AIDS disease. 3. Poor nutritional and immunodeficiency. 4. Smokers. So to avoid recurrence of tuberculosis eat healthy food, maintain good immunity, avoid steroids, quit smoking as soon as possible if you are smoker, regularly check your sugar levels to rule out diabetes, avoid stress and tension. If you follow all these than chances of tuberculosis recurrence are very very less. Hope I have solved your query. I will be happy to help you further. Wish you good health. Thanks."
},
{
"id": 179408,
"tgt": "Suggest treatment for diarrhea and abdominal pain",
"src": "Patient: Respected doc. My daughter is six and a half years, a problem eater, suffered from diarrhea , I gave her entamizole syrup. But now is having greenish stools and complains of abdominal pain. Half an hour ago I gave her fresh apple juice and then she passed very small amount of greenish stool and after that her abdominal pain subsided. She complains of abdominal pain every morning . I have started giving her syrup zinc from today. I gave her syrup entamizole for ten days And the stools were formed. I am not giving her milk these days , kindly help. Doctor: Hi Dear Welcome to the HCM,child has got some stomach infection. Requires stool culture and sensitivity.Report of stool culture will guide you about the exact cause of infection and also the exact drug required for the eradication of the infection.Hope all the symptoms will subside after giving a complete course of drug as guided by the culture.Hope the query is answered.Thanks"
},
{
"id": 49971,
"tgt": "Kidney stone, severe pain, upper abdominal pain. Cause of upper abdominal pain? How long will this continue?",
"src": "Patient: I found out I have a kidney stone and been experiencing vary bad pain for usually about 3 hours a day. The nurse thought it was wierd when I me roomed my upper abdomin also hurts suing this time. I thought maybe my zipfoid process? It gets pretty intense. I had reg discomfort for about 2 weeks before this but now wonder why it hurts so bad when it does and why does my upper an hurt if it shouldn't be? Also how much longer is this going to go on for? Doctor: kidney stones are common these days not to worry there good medical and surgical methods for this..now for your kidney stone i suggest u consume plenty of water where smaller stones tend to pass down with urine..for if your pain worsens you may take effective pain killers like tramadol for once if you find no relief then consult your surgeon who would advice a sonography of abdomen..there are surgical procedures like shock wave lithotrypsy which has given good results.."
},
{
"id": 151924,
"tgt": "Are there any treatment for cerebral palsy ?",
"src": "Patient: sir, I have one child 10 yr old he cannot walk on own legs and suffering from CP do you have any suggestion and treatment for it Doctor: hi well come to hcm. cerebral palsy occurs due to birth asphyxia.except supportive care like physiotherpy speech therapy etc there is no specific treatment known. Thanks"
},
{
"id": 108501,
"tgt": "What causes lower back pain that radiates to stomach?",
"src": "Patient: I have a pain in my lower back which radiates to my stomachI have been to see a physiotherapist she says I have a problem with one of the discs in my back i have a constant ache feels like someone has kicked me in the nuts excuse the way I put it. I am wondering do you think its worth seeing my doctor just to rule out any other problems. I have had this on going for over a week now. Thank you for any sort of info you might be able to give me. Doctor: HelloLow back ache may be due to many reasons like genito-urinary causes,pelvic causes,musculoskeletal reasons etc.You need proper clinical examination and routine investigations.Investigations include routine hemogram,random blood sugar,liver function test,renal function test,urine RE/ME,ultrasound of abdomen,X-ray L/S spine (AP/Lateral view),Thyroid profile,serum calcium and vitamin D level.It may also be due to low calcium level or muscle spasm.MRI of L/S spine can be done after evaluation.It is also necessary to exclude intervertebral disc prolapse in lumbo-scaral region.Proper treatment depend upon findings.Get well soon.Take CareDr.Indu Bhushan"
},
{
"id": 199740,
"tgt": "What causes pain in groin region?",
"src": "Patient: 51 year old male. 6 3 , 220lbs. Have a history of inguinal hernias...had one when I was 6 years old, and then a double hernia laproscopic operation (mesh) when i was 39 years old. Believe I may have a reccurring inguinal hernia. Active weight lifter. Worked out two weeks ago and experienced no problems. About 3-4 hours after working out, felt a dull pain in the upper groin/lower abdomen. No swelling, redness or bumps and not painful to the touch. Went to see a doctor after one week for a hernia exam. Noted that he DID NOT press his fingers into the scrotal skin and against the pubic tubercule to feel for the inguinal ring and a possible hernia sac. When I questioned him, he told me it was not necessary. Informed me I did not have a hernia and it was ok to continue lifting weights. Assessed that I may have torn tissue around were the mesh had been installed. It s now been two weeks since I experienced the pain, and it does not appear to be going away. I feel the pain most went sitting down. Contemplating getting a second opinion/hernia exam. Doctor: Hello I had gone through your question and I can understand your problemLooking at your history this could be hernia or muscle spasmin my opinion you should take a analgesic like ibuprofen and muscle relaxant like chlorzoxazone combination drug after consulting your doctorIf it persists even after that consult another general surgeon for thorough examination for hernia which generally causes constant dragging painHope this helps youthanks for your question"
},
{
"id": 97157,
"tgt": "How to remove sea urchin spines from foot?",
"src": "Patient: Hello, a few days ago when I was on holiday and swimming in the sea I bashed my foot against a rock. Its not derious at all but unfortunately for me there was a sea urchin in that spot. Now I have its spines in my foot and I m struggling to get them out! Is it serious and do you have any tips? Doctor: Hello, listen whenever incidents like sea urchins stings happens, first thing one should worry about is taking out stings, you can try to takeout by hand, if not possible try applying wax and let it dry, later u can remove dried wax so attached stings will come along with it, just make sure all of stings are out. Or else there is a good chance of infection to set in. You can ask for furthur such querries, happy to answer. Thank you."
},
{
"id": 120571,
"tgt": "Suggest treatment for shoulder problems after the surgery",
"src": "Patient: a year ago was kick in upper arm a few days later arm swollen i cannot lift my arm up back i haved surgery for rotator cuff surgery may 2011 then they thought i had frozen shoulder so they took me to surgery july 2011 for mua then i found out sept.2011 that they tour my rotator cuff so will i always have problems with this shoulder Doctor: Hello,Stiffness and pain are common after rotator cuff surgery. Recovery can take\u00a04 to 6 months, depending on the size of the tear and other factors. You may have to wear a sling for 4 to 6 weeks after\u00a0surgery. Pain\u00a0is\u00a0usually managed with medicines. Physical therapy\u00a0can\u00a0help you regain the motion and strength of your\u00a0shoulder.Hope I have answered your question. Let me know if I can assist you further. Regards, Dr. Dorina Gurabardhi, General & Family Physician"
},
{
"id": 46206,
"tgt": "Can diuretic cause kidney stones?",
"src": "Patient: Hello...my husband is 38 and takes medicine for high blood pressure. His mother gives him a diuretic to take and, if he runs out of his own medicine, will give him some of her blood pressure medicine. Right now he is in the hospital for kidney stones...can any of this cause that? Doctor: Hello,As a urologist, I can understand your anxiety. Self-medication is a very risky approach. Diuretics don't cause stones. Taking diuretics without supervision can cause serious side effects. He should drink 3 liters of fluids daily. He should avoid milk and dairy products.For stone treatment following tests are needed after seeing urologist:1. Urine routine and culture2. Blood routine, creatinine, RBS, LFT, calcium, phosphorus, uric acid, and plateletsAccording to reports an expert opinion can be given.Hope I have answered your query. Let me know if I can assist you further.Regards,Dr. Matthew J. Mangat"
},
{
"id": 103739,
"tgt": "Have been diagnosed with costchondritis. Biopsy said seborrheic keratosis. What does this mean?",
"src": "Patient: I am 54yrs old, and have been diagnosed with costchondritis numerous times since my 20 s. I was a letter carrier, and carried a satchel on my shoulder. I also am on a disability for SI Joint Dysfunction. For the past 3 yrs I get these eruptions of bumps, sometimes scaly on my scalp, and now one on my elbow, sores in my mouth, with EXCESSIVE amounts of mucous in the back of my throat, and a feeling as though the palate is swollen or the back of my throat is swollen. ENT said initially I had swollen tonsils, and prescribed an antiobiotic mouthwash which helped. But it keeps coming back. He sent a scope down my throat, and said everything looked clear. 6 yrs ago, during an endoscopy it was found that I had a yeast infection in my esophogus, and prescribed meds to clear it up. I have severe non rheumatoid arthritis since my 20 s in my hands. I ve had one partial rotator cuff tear , and one now has nerve impingement due to spurs. I am a bone spurrer on my hands, as well. I asked my rheumatologist if it could be psoriatic arthritis , since I ve always had arthritic conditions or ligament issues, and now am getting these skin eruptions. He didn t think so. Well, when the one on my elbow went into a bump and got another recurrence on my scalp I asked to get a biopsy. The biopsy diagnosis is seborrheic keratosis. For over 2 yrs, I ve had monthly (it seems) episodes where I feel as though I can t get air. My oxygen levels are always in the 98% range. A pulmonary function test results: Pulmonary Obstructive Disease and told to take an inhaler. (doesn t really seem to help) 4 out of 6 of my bloodworks showed high levels of CO2. Now, recently, I am being woken up, numerous times throughout the night with a feeling of panic, as though I m not getting air. It is VERY painful to sleep, as laying on my ribs causes pain. One night last week I woke up to excruciating stabbing pains in my left lower rib. Normally, this pain is more of a gnawing pain on both sides lower ribs. Sometimes the gnawing pain moves to the center just to the right of my lower rt rib. Because this is becoming worrisome, I saw a dr. in my clinic, since my reg dr. was unavailable, and he is also a DO. He said he could try some myofascial release, and some adjustments. He said I would be sore the following day, but that within 2 days I should know if it helped. He also told me to take 440 mg Naproxen twice daily for inflammation. I also take Tramadol for my SI Joint Pain. Last night, again, was very painful, but not the stabbing pain, and I awoke, feeling like I couldn t get enough air, and like I was run over by a truck. Do you think it is viable to see a pulmonologist at this point. I m slowly moving to an anti inflammatory diet...lots of antioxidants, fruits, veggies, fish and poultry, nuts. I am at my wit s end, any advice. Doctor: you dont have to use animal proteins as they cause the problems and increases inflamationyour is case of animal protein reaction causing inflamation of systemsand harmonal imbalances leading to the problems get blood serum tested for food specific antibodies for these milk wheat egg rice potato nuts and other food you takefood causes these problemslatest study shows psoriasis is continuous irritation by food when you withdraw food components according to blood tests maybe 1-3 you will start improving in 3 wktill then continue sympatomatic rx and be sure of results after the above tests all systems will recover slowly"
},
{
"id": 162177,
"tgt": "Suggest treatment for fever and stomach pain in a child",
"src": "Patient: my son is 3 year old, he had fever, eye pain and tummy ache for 2 days, no vomitting, no diarrhea. all symptoms stoped a half day, and come back. he always cried for eye pain and tummy ache.I took him to see doctor today, the doctor didn t take more deep checks and just said it need to take more time to see what happened. But when he came back home, his fever, eye pain and tummy ache come back. Doctor: Hi, I have gone through your question and I understand your concerns but please get a stool analysis as stomach pain and fever might indicate gastroenteritis. Hope I have answered your query. Let me know if I can assist you further. Regards, Dr. Salah Saad Shoman, Internal Medicine Specialist"
},
{
"id": 160445,
"tgt": "Why is my infant passing stools drop by drop?",
"src": "Patient: 20 months old baby is not passing Stools properly.Passing stools drop by drop whole day.we have used dulcolex suppostiries last week that time stools passed fine, after that again the problem started yesterday we have used dulcolex but no use..he is passing drop by drop.Today also the same till now stools passed 15 times drop by drop.Past three weeks only he is having this problem.Kinldy advise on this. Doctor: Hi, It is due to the feeding habits of your child that affect the stool pass and constancy, so return back to the food you were giving him. Take care. Hope I have answered your question. Let me know if I can assist you further. Regards, Dr. Salah Saad Shoman, Internal Medicine Specialist"
},
{
"id": 136982,
"tgt": "What causes stiffening of right hand?",
"src": "Patient: Hi. I m feeling very odd today same as yesterday too. My right hand is trying to stiffing up on me and I am becoming really nervous. Thinking that I might have a seizure or stroke. I have not checked my temperature. I am at a public place also. My head is hurting a little bit but not too much where I can not stand the pain. the pains are short. What should I do? Doctor: Hello, I have studied your case . Due to compression of nerve root there can be hand pain.I will advise you to MRI cervical spine for better diagnosis.Another possibility of vascular obstruction to arm, for this clinical examination may be required.For these symptoms analgesic and neurotropic medication can be started.Till time, avoid lifting weights. You can consult physiotherapist for help.Physiotherapy like ultrasound and interferential therapy will give quick relief.I will advise to check your vit B12 and vit D3 level.Hope this answers your query. If you have additional questions or follow up queries then please do not hesitate in writing to us. I will be happy to answer your queries. Wishing you good health."
},
{
"id": 163013,
"tgt": "Is elderberry juice safe to consume?",
"src": "Patient: My daughter is giving her children elderberry juice or what ever it called. My 3 year old grandson is complaining of stomach pain and is running low grade fever off and on. This has been going on for about 4 days. Are there any side effects to taking elderberry juice? I believe this is a home made juice. Doctor: Hi, I understand your concern. I think what your grandson is going through (low grade fever off and on; stomach pain) are suggesting acute viral infection. I advise to:- monitor his body temperature- give plenty of fluids especially plain water and tea- stop giving elderberry juice just in case- give probiotics to stabilize intestinal floraIf still no improvement, should consult further with his pediatrician. Wish fast recovery. Dr.Albana"
},
{
"id": 98836,
"tgt": "Can red rash on face with itchy eyes be symptom of hay fever?",
"src": "Patient: My son is 11 years old and has a red papular rash on the left side of his face that extends down onto his neck. It is also on the front upper chest/throat and his upper back. He has a red, itchy area on the roof of his mouth with a couple of white bumps. The back of his ears itch. Itchy watery eyes and sneezing. Are all of these symptoms of hayfever? Doctor: Thanks for your question on Health Care Magic. I can understand your concern. It is unlikely to be hay fever. If this has occurred for the first time , kindly consult a dermatologist as this requires a detailed examination and skin biopsy to come at a diagnosis. please do not neglect.Don't worry, you will be alright. Hope I have solved your query. Wish you good health. Thanks."
},
{
"id": 5882,
"tgt": "High TSH levels. Not able to conceive. Thyronorm suggested. Will this help in getting pregnant?",
"src": "Patient: Hi I hv been trying to conceive for last two yrs all test are normal recently doc reported high tsh of 6.4 and suggested to take 25 mcg thyronorm for a month and go for test . T3 and t4 are normal.parallely tkng medicine m trying as well. Can this medicine hlp also m 31 without ny issue trying desperately for a baby. Jess Doctor: Hi, Thyronorm is not going to help in conceiving. But its very essential medicine which you should take. It will prevent bad effect of high TSH on baby's brain development if you conceive. For conception, you need to consult some infertility specialist who will guide you for faster conception. Wish you good health."
},
{
"id": 104304,
"tgt": "Felt dizzy smelling chemila, continuously felling dizziness. Whats the treatment?",
"src": "Patient: Last night I opened up a new plastic shower curtain in the bathroom that had an extrenely strong chemila smell. I immediately felt dizzy and even though I ventillated the bathroom and the opened wondows in my home the severe dizziness persisted all night long. I am at work today and still feel dizzy but not as bad as last night. Please advise. Thank you. Doctor: allergy to some sustance go exposure and hence dizziness dringk lot of warm water morning evening apply neosporin h eye ointment in nose take anti allergic loratidine 10 mg od add syp gelusil 2 tsf tds do for 5 days"
},
{
"id": 93584,
"tgt": "Discomfort in lower abdomen, increased urination and nausea. Sugest",
"src": "Patient: HI I have a queryI have been suffering with a slight discomfort in my lower abdomen, and increased urination, at first I thought it was just a urine infection or cystitis, but I had no discomfort when passing urine, I have since developed slight nausea, and a bit of an upset stomach, I have recently come off the pill approx. 8 week ago, and my first period since stopping the pill finished 10 days ago, and was wondering if this could just be my body getting used to not being on the pill anymore? or should I go see the doctor in regards to a water infection? Doctor: Hi welcome to Health care magic forum. Thanks for choosing H.C.M.Forum. You had slight discomfert in lower abdomen, and increased urination, nausea, upset stomach, You have stopped the pills 8 weeks ago. and you had period 10 days back. After stopping the pills some people will have such changes, but you should not neglect the pain, it could be urinary tract infection, or may be gastroenteritis. I advise you to consult a doctor for diagnosis and treatment. Wishing for a quick and complete recovery. best regards."
},
{
"id": 195591,
"tgt": "Can candida and haemolytic streptococci in sperm culture cause poor motility?",
"src": "Patient: i did a sperm culture for a patient and he had candida and haemolytic streptoccoci and the isolated. his semen was highly viscous with sperm counts very normal, but poor motilty. could this germs be the major causes for high viscosity and poor motility? Doctor: Hello and Welcome to \u2018Ask A Doctor\u2019 service. I have reviewed your query and here is my advice. I would like to know what were the findings on regular semen analysis and more specifically what was the amount of pus cells in semen analysis. Pyospermia is among the main causes for poor sperm motility. Hope I have answered your query. Let me know if I can assist you further."
},
{
"id": 124825,
"tgt": "Suggest treatment foe knee pain due reduced gap between joints",
"src": "Patient: Salam Sir, i am Muhammad Ejaz, having 34 years of Age. i suffer in Knee Joint Pain Exrays Shows the Gap between Joint is reduce. Pls tell me Have you any medision for increase Gap between knee joints. hopping positive replay YYYY@YYYY Muhammad Ejaz 0333-0000 Doctor: Hello, It can be a part of arthritis. Consult an orthopedician and get evaluated. You might require an arthroscopic repair probably. Hope I have answered your query. Let me know if I can assist you further. Regards, Dr. Shinas Hussain, General & Family Physician"
},
{
"id": 203491,
"tgt": "What is the cause for itching and redness under my penis skin?",
"src": "Patient: I am having itching under my penis skin and redness.I wash it twice a day and it give me relief for 3-4 hours after after then again starts itching.Please suggest what to do,what may be reason.Is it a serious problem or may become serious like penis cancer in future I am very much afraid and also please let me know which doctor to consult in such cases?? Doctor: Hi. Just to ask you few questions in the beginning. Are you diabetic? Are you married / have sex partner? The condition which you described is called balano-posthitis and simply an infection of the area and need a simple remedy. So do not worry. It will NEVER become cancer in future. It needs an antibiotic,, meronidazole, an antifungal tables ( to be taken by both if you have a sexual partner) , no sex till you get OK. Visit a Dermatologist."
},
{
"id": 19918,
"tgt": "What causes pain towards left side of heart?",
"src": "Patient: I have a small pain (lasts 1 sec or so) on my left side near the heart(not heart pain).This happens once a month(sometimes never)Today It happened 6 times during the day.I must tell you I'm overweight and I'm afraid of heart attack...(I'm only 24years old)Whats the problem? Doctor: Hi ThereAfter going through your query I understand your concernI would like to tell you that a heart attack at your age is very unlikely until unless you have a strong cardiac family history and even your symptoms are not suggestive of any heart disease.Its advisable for you to maintain a healthy weight and healthy lifestyle to reduce the risk of heart attack in future.Hopefully this has answered your query.Kind RegarsDr. Navneet Mahajan"
},
{
"id": 176911,
"tgt": "What causes recurrent rashes all over the body in 2 year old?",
"src": "Patient: My two years son is having don t know seboric dermatitis or atopic dermatitis n food allergy also esp with egg protein n dairy dry fruits since march 2014he has severe aggrevation of homeopathy drug but could not get good result he is recurrent rashes all over body even in the scalp also please help me Doctor: hi dear. thanks for your query.From your history it seems your child has skin rash in all over body and scalp and food allergy.form distribution pattern it may be seborrhic dermatitis but because you child also have food allergy atopic dermatitis can not be excluded.i suggest you to consult dermatologist or pediatriation for proper evaluation and treatment because without seeing the type of rash, whethere it cause itching or not, proper diagnosis cannot be made.meanwhile you can use antihistamincs like cetrizine for rash and antidendruf shampoo for scalp. also avoid food which is causing aggrevation of symtoms.regards- dr deepak vaishnav"
},
{
"id": 196945,
"tgt": "What is long rock hard thing hanging from scrotum after hernia surgery?",
"src": "Patient: I had lower inguinal hernia surgery ten days ago. It was very large and was hanging down in my scrotum. Now that i had the surgery the doc told me i would be swollen and sore for a good while. I can handle that much. But now ive noticed an obvious long rock hard and round \"thing\" running down my left side of scrotum. What is this thing? It doesnt hurt, just very uncomfortable! Seems to be right where the hernia once was. Will it go away along with all the other swelling? He did use a mesh plug and mesh to repair with. Could that be the plug hanging down there? Need some answers please Doctor: HelloThanks for query .You have undergone surgery for large inguinal hernia repair 10 back and have noticed hard swelling running across the scrotum This is due to a inflamed spermatic cord resulting from the handling of spermatic cord during surgery while dissecting hernial sac from the cord.This is known and normal complication of hernia surgery which gradually gets resolved over a period of one month .Take antibiotics like Augmentin and anti inflammatory medication like Diclofenac twice daily . Dr.Patil."
},
{
"id": 74794,
"tgt": "Suggest treatment for sarcoidosis of chest",
"src": "Patient: I am a 63 year old male, diagnosed with IC some 20 years ago. 2 years ago I was also diagnosed with Sarcoidosis in my chest which manifested itself as an annoying cough. I have been taking oral steroids to help manage the Sarcoid; however I noticed that my IC at this time improved. More recently I have been diagnosed with Neuro- sarcoidosis, apparently the sarcoid has migrated up through my central nervous system and affected my vision in that I now had double vision! I am currently on 35g of steroid per day and my IC is improving beyond measure. My double vision has also improved and life hasn t been better. \u2022\u00a0\u00a0\u00a0\u00a0\u00a0IC and Sarcoid are auto immune diseases could it be that they are somehow connected. \u2022\u00a0\u00a0\u00a0\u00a0\u00a0and have you heard of steroids making IC improve. Life is good I just wanted to share this with people if it could help them. Doctor: Hey there,I think there is no connection between ic and sarcoidosis.But yes as both are auto immune diseases treatment for both are same.And thanks for sharing with us."
},
{
"id": 136665,
"tgt": "Suggest remedy for swollen painful feet",
"src": "Patient: four days ago my husband had a fever that lasted a couple of days. That has gone away. His feet also started swelling, blistered and now they are hurting. The blistered went away after soaking feet in ebson salts, but they are swollen (cannot see ankle) and they hurt. Your thoughts Doctor: Hi,Thanks for your query.This painful swelling over feet can be due to infection, inflammation, dvt. etc.I advice you to give you rest to the part affected, take anti-inflammatory drug like motrin 1tablet with food as and when required (upto 4 tablets daily) to reduce pain and inflammation and consult your doctor for thorough examination and rule out any infective cause.I do hope that you have found something helpful and I will be glad to answer any further query.Take care"
},
{
"id": 73629,
"tgt": "Is it necessary to take steroids for mycoplasmic pneumonia?",
"src": "Patient: Dear Dr.Mahashur, i have been diagonised with 'MYCOPLASMIC PNEUMONIA' by a Dr in Singapore,(mycoplasmic pneumonia antibody fbc= >1:1280) and am now on a 20-days course of 'avelox' ( moxifloxacin 400 mg) and have also been advised to take steroids for 10 days. I have already gone through a course of antibiotics to treat pneumonia (CLAVICEF)and TB has been eliminated by carrying out a bronchoscopy and lab analysis for TB. I am apprehensive about taking steroids, so pls advise if it is necessary to take steroids? thanks. Doctor: Hello dearWarm welcome to Healthcaremagic.comI have evaluated your query thoroughly .* The need of steroid is determined by the clinician taking into considerations many parameters .* Steroids help to relieve mucosal inflammation of the bronchi , gives faster recovery , so under medical guidance a short course of steroid to be consumed has no harm in it .Hope this will help you for sure .Welcome for any further assistance .Regards ."
},
{
"id": 139756,
"tgt": "What to do if having neckpain due to cervical issues?",
"src": "Patient: Hi Doctor, This is Nemichand jain ,age-62,height-5 10,weight-85 I have cervical problem because of that i have neck pain and My BP also high its 150-104 my physician suggested to consult with any neurologist . could you please suggest what should i do in this problem. Doctor: Hello,I will suggest you to monitor your blood pressure regularly. if it is persistently high even in absence of pain, I will suggest you to start anti hypertensive medication (pain sometimes increases blood pressure). Regarding your cervical spine problem, takes simple analgesics for it like parcetamol and orphanedrine, and start work up for cause of this pain. Go for an X-ray cervical spine, calcium, phosphate and alkaline phosphatase levels and vitamin D levels to look into the cause of this pain. Moreover, take pillow of thickness equal to your shoulder width on one side from ear to tip of shoulder. Limit extreme mobility of neck and head by using cervical collar for at least 3 weeks.Hope I have answered your query. Let me know if I can assist you further. Regards, Dr. Muhammad Faisal Bacha, Internal Medicine Specialist"
},
{
"id": 89597,
"tgt": "Suggest remedy for abdominal pain",
"src": "Patient: I've been having abdominal pain that's just recently sprung up. I have pain in my left shoulder and forearm, right in the middle of the upper quadrants of the abdomen, and what appears to be an indent in the upper right quadrant. I have also been having muscle pain in my legs but that is slight. I've been through many tests and a pulmonologist is leaning towards asthma being my diagnosis, but I'm not sure if that's the cause of the pain. Doctor: HI,Asthma can not cause pain in abdomen or leg. The pain in the upper abdomen can be due to pancreatitis, cholecystitis, gastric ulcer , colitis of transverse colon, The pain in the left shoulder and the forearm can be due to cervical spine problem. and You need the investigations of MRI cervical spine, CT scan of the abdomen as it will show the lower chest too. Consult your Physician as he would coordinate all the varied symptoms and findings and help you to come to a diagnosis and then may refer you accordingly."
},
{
"id": 22000,
"tgt": "Suggest treatment for fluid around heart and lungs",
"src": "Patient: my mum has is a 74 year old smoker who has fluid around the heart and on her lungs. Her feet are swollen and leaking. She has been put on a stronger diaretic but this has been going on for about a month. Her breathing is always bad but she seems to be making grunting noises when she sleeps. Whats the next step, She has an xray booked in about ten days. Doctor: Hi thereYour mum needs to stop smoking and she seems to be in low cardiac output state her heart is most probably not working adequately that causing the full to accumulate. Instead of waiting for an X ray u need to take her to a cardiologist asap and get an Echocardiography done to assess heart function.Good Luck"
},
{
"id": 38784,
"tgt": "How long can typhoid virus exist in the body?",
"src": "Patient: I had been diagonised with Typhoid and dengue 1 month back. I got admitted in hospital and was given 6 bottles of IV fluids and got dischared in 2 days. My platelet count was within limits - 1.5 Lakhs. Later doctor advised me to take 6-8 litres of water a day and asked me to avoid oil and spicy foods. But still I get feverish feeling (my temperature is however normal) , weakness and body pain after 6pm and whole of the night. I am also taking some vitamins as prescribed by Doctor and I feel quite normal during the day. Any suggestions in this regard? Do i still have typhoid virus and how long will this problem be there? Doctor: Hello,Typhoid is a bacterial infection and becomes all right in 10-14 days of proper antibiotic course.Weakness is expected after getting sick from dengue and typhoid infection.Nothing to worry as weakness wold go away gradually.Keep taking vitamins,eat healthy and drink plenty of water.Thanks"
},
{
"id": 92282,
"tgt": "What causes sudden sharp pain in the lower right abdomen which worsens on moving the legs and shifting weight?",
"src": "Patient: i have a sharp pain in my lower right abdominal region. hurts worse when i move my legs or shift my weight, just started earlier today, after i went to the bathroom i couldnt walk it hurt so bad. Doesnt appear to be swollen. im 18 female and 120 lbs 5'3. i am not on my period. Doctor: Hi. the pain in right lower abdomen which increases on movements is suggestive of an inflamed appendix or a twisted ovarian cyst as the commonest of causes. I would advise you to have an urgent sonography of abdomen to ascertain the cause and decide further management."
},
{
"id": 77504,
"tgt": "What does interstitial markings on lungs in my X-ray indicate?",
"src": "Patient: I just received my chest xray report and this was stated at bottom. All else seemed to be normal. What exactly is this and what could it mean? Thank you. IMPRESSION: I just received my chest xray report Prominence of the interstitial markings in both lung fields. Doctor: Hi thanks for asking question.If you have no symptoms at all then it has of less value.But here are few possibilities.1.First interstitial pulmonary edema can lead to such prominent marking. Patient will also complain pink frothy sputum and dyspnea.2.Second if you are working in textile industry or asbestos using industry then chances of pneumoconiosis is present like silicosis, asbestosis etc.can lead to dyspnea with such x ray finding.3.If you have dyspnea, cough , fever , chest pain then interstitial pneumonia like atypical viral or mycoplasma pneumonia can be the cause.4.If you have increasing breathlessness and no specific cause found for that idiopathic pulmonary fibrosis has to be ruled out.To distinguish between restrictive and obstructive disease pulmonary function test is useful.Complete blood examination also helpful.I hope my suggestion will help you."
},
{
"id": 198000,
"tgt": "What causes unilateral nipple discharge in a male?",
"src": "Patient: Hello. I am a 22 year old male. About 9 months ago, maybe more, I noticed in the shower that my left nipple was sticking out farther than my right. I promptly squeezed it and it had a discharge much like a pimple before it was empty. After this, nothing else came out that day, but the nipple has remained larger on the left side, and occasionally (once every few days,) if I squeeze it, it will discharge a tiny bit more. I cannot identify any lumps of any kind to indicate gynecomastia or breast cancer, and am really stumped/concerned as to what else could possibly be causing unilateral nipple discharge in a 22 year old male. Doctor: Dear, We understand your concernsI went through your details. A women can normally have some type of nipple discharge throughout her adult life. For men, nipple discharge can indicate a problem. During hormone imbalances such as adolescence, the male gland can be stimulated. This hormonal stimulation can result in gland growth (Gynecomastia), tenderness, and produce milk and or nipple discharge.If squeezed / traumatized enough, a clear fluid can be express from almost anyone. This is normal but hurting tissues this much is not a good idea.With gland stimulation, the discharge can be milk like or thick like a pimple. Nipple discharge is a concern if:bloody or fluid colored red, pink, or brownlooks like pus especially accompanied by inflammation and or feverone sided (unilateral)happens without squeezing the glandsticky and clear or a cloudy black or brown.I suggest you not to try diagnosing your problem. Please consult a surgeon.If you require more of my help in this aspect, please use this URL. http://goo.gl/aYW2pR. Make sure that you include every minute detail possible. Hope this answers your query. Further clarifications are welcome.Good luck. Happy New Year. Take care."
},
{
"id": 206239,
"tgt": "Suggest treatment for compulsive behavior",
"src": "Patient: i am abdul wahid i am suferring from a mental diease i am facing a compulsive thoughts. sometime one sentence alawaways in my thoughts i think but i donot want to say these compulsive thought always with me some on says that i am suffuring from a madical condition that affact the balance of my thoughts and mind please give me answer Doctor: Hi.I had gone through your query.I understand your concern.Compulsive thought or obsessive thought may indicate OCD.OCD is obsessive and compulsive disorder in which repeated and persistent intrusion of thought occurs and you have to do compulsive behavior like repetition in response to it.Detail Psychological evaluation will give more confirmatory diagnosis.Scales like Y BOCS can help to find area of compulsion and its severity.Evaluation of mood/thought processes are also necessary.It can be treated with anti OCD medicines like fluoxetine and fluoxamine and clomipramine but selection is depend upon type and severity.Exposure and response prevention is also helpful psychotherapy in OCD.So consult psychiatrist and get help.I hope i have answered your query.Thank you.Regards.Dr. vishal Garala."
},
{
"id": 90468,
"tgt": "Suggest treatment for stomach pain",
"src": "Patient: Hi I have been getting strong pains in my stomach which has now contained itself in one area, the right side. The pain then shoots up my chest to my shoulder and neck and is made worse when lying down, breathing deeply or when trying to empty my bowls. Doctor: hello and thank you for asking HCM,I understand your concern. Regarding to your symptoms, it seems like you are having problems with your gallbladder. My suggestion is to visit a gastroenterologist to make abdominal ultrasound and check your liver enzymes. Until than try to avoid fatty and spicy food. I hope this answer was shelpful to you."
},
{
"id": 148322,
"tgt": "What is the treatment for herniated disc in lower back?",
"src": "Patient: ive been to two different pain management doctors for 3 herniated disks in my lower back...and have been threw the shots in the spine on three different occasions, with no success. I have been on all types of medication for this as well with the only success being Percocet to help with relief and management of the pain. I am not in a position to have surgery and take the time off work after it..but they refuse to help me manage my pain that way and keep saying surgery is the next step. what can I do if anything Doctor: Hi,Thank you for posting your query.I have gone through your symptoms, and treatment received so far.Is there any specific reason for refusing the surgery? As per the treatment algorithms, once medications and epidural injections fail, surgery is the most preferred option.So, I would request you to reconsider your decision and opt for surgery. This would ensure a pain-free life.I hope my answer helps. Please get back if you have any follow up queries or if you require any additional information.Wishing you good health,Dr Sudhir Kumar MD (Internal Medicine), DM (Neurology)Senior Consultant NeurologistApollo Hospitals, Hyderabad, IndiaClick on this link to ask me a DIRECT QUERY: http://bit.ly/Dr-Sudhir-kumarMy BLOG: http://bestneurodoctor.blogspot.in"
},
{
"id": 202224,
"tgt": "How to overcome premature ejaculation by excess masturbation?",
"src": "Patient: Hi I have problem of premature ejaculation by excess masturbation, due to excess masturbation I also suffer knee weakness and premature grey hair I m 28 years old and live in Germany. Kindly recommend me cure which is easily available here, I prefer natural cure. Thanks a lot Doctor: Hello Thanks for writing to HCMYour problem is due to excessive sexual thoughts and over excitement.You should avoid excessive masturbation and try to concentrate in your activities.You should avoid provocative literature and videos and try to keep busy yourself in your job.Knee weakness and premature graying of hairs are not related to masturbation.You are wrongly informed.You can consult your orthopedician for kneed problem.You should do exercise and take nutritious food.You will be alright.Hope i have answered your query.Take CareDr.Indu Bhushan"
},
{
"id": 153569,
"tgt": "What is the prognosis of adenocarcinoma with lung, hip and brain metastasis?",
"src": "Patient: My husband has been fighting adenocarcinoma for 3 years and 1 month. His left lung was removed, followed by extensive chemo and radiation. 6 months later cancer in the hip and then 2 brain tumors. Followed up by radiation and tomotheropy. Then spots in his right lung followed by chemo again. In November 3 more non operable brain tumors. Tried radiation, now sent home to die. What is the lenght of this prognosis? It's so sad to watch Doctor: Thanks for your question on Health Care Magic. I can understand your husband's situation and problem. Metastatic lung cancer is having worse prognosis. And prognosis is even worse if brain and opposite lung is also involved by metastasis. Your husband is worsening even after chemotherapy and radiotherapy. This suggest that his tumour is resistant to chemotherapy and radiation. And hence further chemotherapy and radiotherapy are not advisable. Better to keep him at home and discuss end of life issues with him. Give him nutritional and emotional support. Life expectancy in his case is less than 6-8 months. Hope I have solved your query. Wishing good health to your husband. Thanks."
},
{
"id": 115334,
"tgt": "Does fluctuating billirubin levels in blood indicate jaundice?",
"src": "Patient: In a blood test done i got to know that my billirubin was 3.45 on 22nd feb but sgpt and everything else was normal.... i got the treatment done and after a month it was 2. 40 which again has gone up to 3 .31 with alkaline phosphate 396.could you pleas tell me why is it so and is it a type of jaundice ? Doctor: Hi, dearI have gone through your question. I can understand your concern. Your bilirubin is higj. You are suffering from jaundice. You should go for complete liver function test including ultrasound abdomen. You should check your direct and indirect bilirubin. It will give you type of jaundice. Then you should take treatment accordingly. Hope I have answered your question, if you have doubt then I will be happy to answer. Thanks for using health care magic. Wish you a very good health."
},
{
"id": 217067,
"tgt": "What causes Throbbing pain in chest and left Shoulder?",
"src": "Patient: I have a throbbing pain in my chest, near the left side to more specific. And it seems to get worse when I lay down on my back. The pain isn t horrible but I am concerned that it might be a serious problem. I am a 21 year old male. And I occasionally have a throbbing pain in my left shoulder. Should I be worried? Doctor: Kindly consult your doctor and go for cardiac imaging first ...A whole scan MR I sh be done for chest and abdomen"
},
{
"id": 1396,
"tgt": "Is there any possibility of getting pregnant with negative pregnancy test?",
"src": "Patient: False NegativeDear doctor. For the past month, i have been showings signs of pregnancy. Overly tired, food cravings (especially milk), my breasts are tender, the area around my nipples has got darker and there is a white substance coming out of my nipples and i have put on weight in a sense that my stomach or tummy has got rounder, Emotionally i am a ticking time bomb and i dont have the patience i normally would. I am showing most of the signs of pregnancy. So far i have done 3 pregnancy tests, and they have been at least a week a part and all done when i go to the toilet in the morning. 2 were invalid and one was negative. My question is, are false negatives common, is it likely that the tests were wrong? And if i am not pregnant, why am i showing most of the signs of pregnancy and what else could it be? Thank you, Sarah... Doctor: Hi, I think if your periods have delayed and you are having these symptoms, you can go for a bhcg blood test for pregnancy. If negative, get your thyroid profile and prolactin levels done, because white discharge from breast can be due to these problems. Thyroid disorders also lead to excess tiredness. Increased prolactin levels can lead to breast discharge. So get yourself evaluated. Hope I have answered your question. Regards Dr khushboo"
},
{
"id": 45455,
"tgt": "Male infertility!",
"src": "Patient: my sperm count reads 25million/ml,i av not been able to impregnate my wife,could my sperm count be the problem?what can i do to increase the count?her medical test are normal. Doctor: Sperm counts seem to be normal. what about your sperm motility? if that is also normal, then may be your wife would would recquire to go through some tests to ascess her fertility status."
},
{
"id": 27487,
"tgt": "Is bypass surgery the only solution for blockage as shown on angiography done after experiencing heartburn?",
"src": "Patient: i had a feelings of heartburn i consulted cardiologist they have done my angiography In this scan they say i am having 90% blockage in one arteries and 70% on second.............Cardio says i should have bypass surgery but i insist there should be some second way i mean treatment with medicine Doctor: Hello. Thank you for your question and welcome to HCM. I read your query and I understand your concern.By the brief information you provided, I am assuming these blockages are in delicate parts of coronary arteries. Further, I think the main blockage is in the left main coronary artery (LMCA) or in the ostial (entering) part of the left anterior descendent (LAD) artery. From all data that exist and from our clinic's experience, the stent placement in these portions is a very dangerous procedure, both technically and from a result-oriented point of view. Therapy with drugs will only help you calm your symptoms, while the plaque is still stable. But, this blockage will eventually get bigger and totally block the artery, causing a heart attack which, in this position of the artery, is potentially lethal. Therefore, my recommendation is that you should proceed with the coronary artery bypass grafting (CABG). Of course, the choice is only yours.I hope I was helpful and thorough in my answer. Good luck!"
},
{
"id": 225112,
"tgt": "Is it normal to have spotting after intercourse, weeks after IUD insertion?",
"src": "Patient: Hey there ! I had my paragard iud inserted on 18 sept,,2013,, last month,, and i didnt had intercourse till yesterday its like 3 weeks now and today i had it and i noticed little spoting after that ,may i know is this something serious ?? or do my iud got moved or something ,, i am just worried,,, please answer . . Doctor: Hello!Thanks for your query.Postcoital bleeding isnt normal but it can be caused from: infection cervical or endometrial polyps vaginal cancer cervical cancer traumaTo determine aetiology of postcoital bleeding, you need for further evaluation by your gyn .If your recent PAP smear is negative ,despite the low rate of serious pathologies seen in referred postcoital bleeding,is recommended colposcopy.Take care of your self"
},
{
"id": 224910,
"tgt": "Delayed periods after copper-T insertion. Had cold and fever. Does antibiotics delay periods?",
"src": "Patient: hello, i had put copper-t in may of this year that is 5 months back, all these months my period was on date but this month i did have till now, my date on which i was supposes to have my period was on 27th of september 2012, but still i haven t had. a week before that i had fever and cold , so i had to take antibiotics from a doctor, does that medicine had a role in postponing my periods, iam having all the symtoms of having period like cramps leg pain etc. pls let me know wht am i suppose to do? Doctor: HIThank for asking to HCMI really appreciate your concern, antibiotic would not postpone the menses but of course some time the copper T it self time does this but this is nothing to worry at all, this is normal phenomenon, hope this information helps you have good day."
},
{
"id": 155851,
"tgt": "What does neutrophilic infiltrate with bacilli in a pap smear test indicate?",
"src": "Patient: Hi My GF's pap smear test result under Microscopic Description says ''Back ground shround shows red blood cells and moderate to marked neutrophilic inflitrate with bacili. Pls explain what does it mean and if any treatment required or not. She has doen this as part of normall check up. Doctor: Welcome to H.C.M. I ma Dr Krishna Dubey.This is very common finding seen in PAP smear. It suggests Inflammatory smear.It means that you will need full antibiotic course like (Cefixime 200mg twice a day for five days) then repeat PAP smear after antibiotic course.You did not tell your age so i expect that you will tell you age next time.Thanx for your query."
},
{
"id": 212691,
"tgt": "Behavioural changes after dropped out of medical school. No help from psychiatrists. Worried about violence towards wife, baby. Suggestions?",
"src": "Patient: Hi its regarding my elder brothers mental health .hes is a 35 yr okd,married for 10 yrs abd a 1 yr old baby.he does not work, has not finished univerdity rducation.highly intelligent,very shy,very violent has trouble controlling his anger.now I am his younger dis 3 yr younger snd have grown up together. Hes behavior changes took place after he dropped out of medical school after 1 st yr dedpite being a good student.he then camr back and lived with family in isolation.no ineraction with outside world and very suicidal. Hence bomy mum and dad listened to every thing he said or wanted. And covered his behavior wuth family.however some family mrmbers were approached snd came to concil after ehivh he was mire violent.also haf a pschycatris come to examin and prescribef anti depressants of which sone were discretely mixed wiyh foof and coz my mum worrued about sude effects stopped giving them.in addition to this he showed signs od sadism and vulgarity and urge if having sec and dirty thougjts .due to tjis mt parents took him ti ibdia to get narried .byt he had to work in dads shop for 1 year. He marrief a lively girl and settled togethet. And contnued working at the shop fr one mmore year. Aafter whuch he stopped and had been at hime ever since.hus eife had brrn worrief because if his vuolance towards ger byt dedpite that she had a baby and us worrued about vuolence toeards naby.my parents to mske mattets worde spoon feed him by giving money minthly as they are scared uf they dont di so he will get violent.im worried fir my parents and my dis in law n baby pls help Doctor: Hi there ~ I understand that your brother's situation is complicated and it is giving his, you and your family a lot of heartache. It is understandable that you are quite worried about the whole situation. I am unsure of what the current living situation is for your brother. It would be very well to consult a psychiatrist or a qualified psychologist in your brother's case. It seems like he does require both medications and therapy. More needs to be known in terms of his symptoms and this can only be elicited during a direct interview with the patient. I hope this helps. Take care and keep posted."
},
{
"id": 177262,
"tgt": "Suggest treatment for high fever and loose stool",
"src": "Patient: 14 month old Grandson has fever for three days ranging from 99.9 to 100.9. He has had 4 bowel movements a day for two days but today s were softer but not through diaper. Should he see pediatrician tomorrow or wait another day. Eating okay Thank you Carol Doctor: Pl give paracetamol syr for fever. Some probiotic like enterogermina for loose stools. To investigate further for cause of fever like cbc,crp levels,malarial antigen assay and a stool routine with culture sensitivity and a complete urine examination are needed. Try to maintain child's hydration with plenty of oral fluids. Regards"
},
{
"id": 163526,
"tgt": "What causes vomiting with fever and chesty coughing?",
"src": "Patient: My 9 yr old son came home from school saying that he had vomited twice and school nurse took his temp and it was normal so she told him to go back to class. He was tired so he went to take and nap, and now that he is up his temp is 103.5. Should I take him to er or just monitor his fever. He is also coughing, and it sounds very chesty cough. I did give him a tylenol to help with the fever. Doctor: Hello,Fever of a few days without any localizing signs could as well a viral illness. Usually, rather than fever, what is more, important is the activity of the child, in between two fever episodes on the same day. If the kid is active and playing around when there is no fever, it is probably a viral illness, and it doesn't require antibiotics at all. Once the viral fever comes, it will there for 4-7 days. So do not worry about the duration if the kid is active.Paracetamol can be given in the dose of 15mg/kg/dose (maximum ceiling dose of 500mg) every 4-6th hourly that too only if fever is more than 100F. I suggest not using combination medicines for fever, especially with Paracetamol. This type of congested cough is very common in kids, and you can give syrup promethazine for it, 5 ml thrice a day for three days.Hope I have answered your query. Let me know if I can assist you further.Regards, Dr. Sumanth Amperayani"
},
{
"id": 224725,
"tgt": "How long after sex can pills be taken to prevent pregnancy?",
"src": "Patient: i had an unprotected sex with my boy friend on friend evening,saturday and sunday but on monday i took some pills(postinor 2) on monday. which happen last month. i saw my period last month but it was scanty but it lasted just 2-3days. am still expecting it this months does it means am pregnant Doctor: HelloThanks for writing to us with your health concern.Take a pregnancy test to know right away.Take care."
},
{
"id": 188656,
"tgt": "I have done cosmetic teeth surgery & i usually have fever",
"src": "Patient: hi i am 19 year old female. my teeth were crooked and braces did not work out so i underwent for a cosmetic teeth surgery . the doctor did not explain me before that he would root canal my front six teeth and cut them very short. he also sliced my back tooth a bit saying that it will correct my bite. i am very depressed.i have caps on my original six teeth now.but the process was also not done correctly.it led to swelling of gums so i had to go for a gum surgery and now i am going for a re root canal for all the teeths..my complaint is that my eye and jaw hurt a lot.i dont know why. since the time i have done this surgery i usually have fever . its been almost 3 months now..have i ruined my life ? do many people do such cosmetic teeth surgery ? please help..i am very depressed...thankyou Doctor: i think your RCT is not properly done thats why all the problem is there re RCT should be done and about your pain you should take medicine to subside your problem. take cifadroxil 500mg 1 tab twice a day and serratiopeptidase 10mg 1 tab thrice a day and voveron SR 1 tab once a day. it will help in reducing the pain and get the IOPA xray to see any pathology and if needed go for Re RCT."
},
{
"id": 74761,
"tgt": "What is the treatment for Pulmonary edema?",
"src": "Patient: my daughter has fluid in her lungs from birth. she is 9 months old. she is never ill , strong and very playful. she only breaths with a kind of sound coming out from chest cavity when she plays hard and the sound is very faint when she sleeps. i am bothered Doctor: Hey there,Fluids in the lungs can't be common in any situation.Which ultimately get dry and it may affect lungs of your child."
},
{
"id": 98785,
"tgt": "What causes itching and rash on body after taking Lemectial?",
"src": "Patient: Hello my daughter was on lemectial for Bi Polar disorder, she broke out into a severe rash all over her body, the doctor told her to stay off the Limenicatal ?? for two weeks because it is a serious side effect, well, it has been over 2 weeks and she still itches, not as bad, but she still has the discomfort of itching, it looks like tiny bumps all over her body,but being off her med. is causing terrible mood disorder, he put her on seriquil but she's drowsy all the time. Doctor: it's important to understand when the itching and the rash is started. there are many advers drug reaction, some of them not completely understood.if the reaction started after 2-8 the first time she take the pills, may be useful doing some exam (complete blood count, gamma-gt, GOT & GPT ecc)"
},
{
"id": 187549,
"tgt": "What do you suggest for nerve damage in tooth after tooth extraction?",
"src": "Patient: I had a difficult molar extraction on tooth with root canal and crown that decayed. The dentist was very rough and was in there three hours. I have since had extreme pain in gum and jaw area. I have had permanent bridge for over a month, but can not chew without pain on it. The area under at the gum line is painful as well as down in jaw. Went to dentist other day and he says I know have nerve damage in tooth in front of extraction. He wants to drain the nerve in that tooth and then remove a piece of tooth(?) that is coming up from gum under bridge. Is this common - do I need a oral surgeon? Doctor: Hello,Thank you for consulting with HCM.Actually when there is traumatic extraction of tooth as you are mentioning that the dentist took 3 hours to remove it, there are chances to develop dry socket in that area.Dry socket is a condition when blood clot is not formed and bone is exposed which is extremely painful, treatment of dry socket should be done to get the pain relieved. In case of nerve damage you will feel numbness not pain in that area, so it can be a dry socket only.Better you should visit an oral surgeon to get it properly examined and treated as a wrong diagnosis will degrade the condition.Hope it will help you"
},
{
"id": 60298,
"tgt": "Can jaundice cause swelling type feeling in upper left side of the head ?",
"src": "Patient: i have swelling type feeling in upper left side of head and back down right side of head....and backache upper right side of back bone ..i m suffering from jaundice from last 2 months...my hapatitese is neg..pain occur is not continous but effect i feel regularly..what do i do....i had been treated with liv.52 and vinoliv..rantac.d.vitamin b tabs.. i have swelling type feeling in upper left side of head and back down right side of head....and backache upper right side of back bone..i m suffering from jaundice from last 2 months...my hapatitese is neg..pain occur is not continous but effect i feel regularly..what do i do....i had been treated with liv.52 and vinoliv..rantac.d.vitamin b tabs.. Doctor: Hello; welcome to HealthcareMagic The swelling needs to be examined to see if it is any sort of infected lump;cyst or just a palpable lymph node.Since it is not painful it may not be infected but better consult a doctor and get examined.After examination he will prescribe the needed medicines.It does not have any relation with jaundice you had suffered. Thanks"
},
{
"id": 79249,
"tgt": "What causes heaviness in the chest and shortness of breath?",
"src": "Patient: Hi there.I am just writing to ask about these symptoms. Hard to breath. heavy chest. feels like I have the only way to describe it... you know when popcorn kernel gets stuck between your tonsils and the bac of your throat. I cannot seem to bring it up as the passage feels block and is moving up and down when i lie down. I have a sharp pain in the middle of my back whenI breathe. I am quiet tired. have a little headache now and throat is starting to get sore again. and a slight cough developing. Doctor: Thanks for your question on Health Care Magic. I can understand your concern. By your history and description, possibility of viral bronchitis is more. It is due to inflammation in airways because of viral infection. So better to consult pulmonologist and get done clinical examination and PFT (Pulmonary Function Test) for the diagnosis of bronchitis. You need antibiotics, antihistamines, mucolytic and expectorant drugs. Inhaled bronchodilators and inhaled corticosteroid are also needed in bronchitis. Don't worry, you will be alright. Better to consult pulmonologist, first diagnose yourself and then start appropriate treatment. Hope I have solved your query. Wish you good health. Thanks."
},
{
"id": 180516,
"tgt": "Does leukoplakia cause adverse side effects?",
"src": "Patient: Hello doctor,I have diagnosed with leukoplakia last year,inthe month of may.Doctor said it happened due to sharp edge of teeth and he removed that teeth.some dotted like white spots are still there.Is there any adverse effect would happen later?I'm really worried.kindly answer my query.Regards,Bandita Sahu Doctor: Hello and Welcome to \u2018Ask A Doctor\u2019 service. I have reviewed your query and here is my advice. As per your complain it seems that the dotted white spots caused due to sharp edge of the teeth can be due to causes like Frictional Keratosis that is a harmless overproduction of keratin layer over the mucosa as a protective phenomena to prevent damage to mucosa with the friction and chemicals.Other cause can he Leukoplakia that is also over production of Keratin but there is also disturbance in cells known as cell dysplasia.It is a Precancerous lesion and needs medical assistance.I would suggest you to consult an Oral Physician and get evaluated and a clinical evaluation and investigations like Biopsy can be done for diagnosis.In case if it is due to Frictional Keratosis it will get resolved on its own once the tobacco intake is stopped.In case of Leukoplakia you can be advised Multivitamins and anti oxidants like Lycopene for atleast 2 to 3 months.Also the first thing that has to be done is to get the sharp cusps of teeth trimmed so that it does not injure the mucosa further.Hope I have answered your query. Let me know if I can assist you further. Regards, Dr. Honey Arora"
},
{
"id": 76219,
"tgt": "Does having beer at night help to loosen mucus due to COPD?",
"src": "Patient: I am 83 years old have copd which u know creates mucus iI take med to counter it but it's hard to get loose. I have found that a few beers at night helps get the mucus loose so I can get relief. I get lot of static from family about me using beer.what's ur opinion Doctor: Thanks for your question on Healthcare Magic. I can understand your concern. No, beer can not cause loosening of mucus and makes it easier to cough out. In fact beer should be avoided in COPD patients because beer can cause sedation and thus worsens COPD. So don't drink beer. Better to consult pulmonologist and start mucolytic and expectorant drugs like N acetyl cysteine, ambroxol, guafenesine etc. Enroll in pulmonary rehabilitation center where deep breathing exercises and chest physiotherapy are done. All these will help in easy expectoration of your mucus. Don't worry, you will be alright with all these but don't drink beer. Hope I have solved your query. I will be happy to help you further. Wish you good health. Thanks."
},
{
"id": 126403,
"tgt": "Can Oxycodone and Acetaminophen be taken instead of Percocet for knee pain?",
"src": "Patient: I am preparing for knee replacement surgery soon but am confused about pain medicines. I have used Percocet in the past and hated it and had numerous bad reactions and it seems that everything they are suggestion is some form of oxycodone and acetaminophen, is there a difference between vicodin, OxyContin, norco, roxicodone. My thoughts are that maybe it is different amounts of codone in each? Doctor: Hi, Oxycodone belongs to the same opioid group and provides good pain relief. You can take Acetaminophen/Oxycodone combination instead of Percocet. Hope I have answered your query. Let me know if I can assist you further. Regards, Dr. Shinas Hussain, General & Family Physician"
},
{
"id": 202,
"tgt": "Suggest remedy for getting pregnant",
"src": "Patient: Hi. My husband (29) and I (28) are trying to conceive our first child. I am on Clomid to help me ovulate (second round). I am 11 days post ovulation on a 38 day cycle and not due for my period until Friday 13th. Are there any particular pregnancy symptoms that I need to look out for that specifically deal with Clomid users? Doctor: Hello,You should do follicular study after taking Clomiphene to know the exact time of ovulation and keep the relationship around it and IUI, if possible, for early results. Next month do serum TSH and serum prolactin if you don't conceive.Hope I have answered your query. Let me know if I can assist you further.Regards,Dr. Sheetal Agarwal"
},
{
"id": 98417,
"tgt": "What causes chronic bronchitis in a toddler?",
"src": "Patient: My 3 year old child has been getting a cough (bronchitis) for the past 5 months. She gets better for a week or two and then goes back to being sick for 2 to 3 weeks. Doctor said is not asthma yet but seems to be cause by allergies. My question is...would Nido milk be one of her allergens? Or any milk maybe causing her allergies to spiked. I know the flowers are allergens since I seen her reaction to the pollen. Doctor: Hello,Are you taking her to any Refreshment park on a regularly basis? Or is she spending most of her time in Lawn (If present at your home)?Yes, Nido milk might be one of the allergens that may cause Recurrent Bronchitis. One need to get an x-ray done, along with it we need to check her ESR(Erythrocyte Sedimentation Rate), AEC(Absolute Eosinophil Count).Do not give her more of Sweet Content in her Diet. We may need to give some Nebulisation for 3 to 5 days if the Symptoms are severe. Be in Regular follow-up with a Paediatrician.Hope I have answered your query. Let me know if I can assist you further.Regards,Dr. Mohammed Abdullah"
},
{
"id": 21562,
"tgt": "Is it safe to run or walk in between EKGs?",
"src": "Patient: Hi, I am a 24 year old female and I run 25-30 miles per week. I have a normal blood pressure (one teens over seventies usually) and a normal resting heart rate at around 58 bpm. I have no history of cardiac illness, and neither does anyone in my family. Recently, I had an annual physical at my university\u2019s clinic, and I asked to have an EKG ran (no chest pain, just curious what it said). It stated that I was in \u201csinus bradycardia\u201d but that, with the msec, \u201c-Left axis for age- possible anterior fascicular block.\u201d My friend (a nurse practitioner) is wondering if it is artifact, because some of the leads don\u2019t show the block. I plan to have the EKG repeated, but my question is this: is it safe to run/walk/jog in between EKGs? Doctor: Hi There After reading the details I understand your concern and I would like to assure you that you are alright. Any heart rate below 60 bpm is known as BRADYCARDIA, so the EKG machine will mark it as Sinus bradycardia but as you have an athletic running schedule a rate of below 60 is normal for you.Yes that could be an artifact and it's safe for you to go for a run. Wish you good health"
},
{
"id": 197525,
"tgt": "What medication is suggested for itchiness in groin,inner thigh and sides of testicles?",
"src": "Patient: I am 28yrs old. Height 5'8 feet. Weight 90kgs.I have a itch in my groin area,inner thighs and sides of the testicles.As a result of this the above mentioned area becomes red, and there is itching sensation and sometimes its like the skin is coming out.I had atheletes foot before, which is healed now, but the above problem is a reoccuring problem please let me know what to do? Doctor: Hi Dear !! Thanks for your query to HCM .Read and reviewed your query and health concerns. You seems to suffer from -Acute on Chronic Jockies itch/ or Tinea Cruris.With the history of athletes foot, possibility of this being fungal infection is more.Other causes need to be fixed with Consults from Dermatologist.Remedy-Being a chronic case-Tab Diflucan-150 mg x 3 times a day-for 4-6 weeks would be neededBesides this- keep the private parts dry and clean with savlon/dettol solution-Smear private parts with -coconut oil and dettol solution- after bath.Use loose and Cotton Ugies.Avoid Synthetic and wet Ugies.Wash the Ugies, with Savlon and don't use common Ugies-if in large family.Good Food /plenty of fluids and Zincovit to supplement skin health would resolve the chronic groin health issue, only with repeated follow ups with Dermatologist.Hope this reply would help you to resolve the health issues with help of doctors attending on you.If need be, update any health issue 24 x 7 by a direct question to ME, at following HCM link-Dear, if satisfied,Don't forget to close this query with YOUR pleasing feedback comments to rate this reply and service, to boost the morale of incoming Emergency patients like YOU, at HCM services.If you want to update more details and ask more update queries ,You are most Welcome herewith !!Good Day!!Wishing Good Healthy Life in time to come!!Dr.Savaskar M.N.Senior Surgical SpecialistM.S.Genl-CVTS"
},
{
"id": 37482,
"tgt": "Could there be a possibility of infection with iron nails being pierced to foot?",
"src": "Patient: Hello I stood on two nails in one day on the same foot about a week ago and one of them is fine but the other is still very saw and the pain is about two inches around the hole, and the foot that hurts is hotter than may other foot. It s not agony just very annoying as I m a roofer and stood on it all day every day, thought there might be a chance of infection as it probably gets direct work Doctor: Hello,Thank you for your contact to healthcare magic.I understand your health concern, if I am your doctor I suggest you that there is probability of infection. As it has pierced in the foot there is probability of anaerobic infections.I advice you should take metronidazole for this infections. Also take the tetanus toxoid vaccine injection. This will prevent you from getting tetanus.I will be happy to answer all your future concern. Thank you,Dr Arun TankInfectious disease specialist.Wish you a best health at health care magic."
},
{
"id": 207254,
"tgt": "Suggest remedy for repeated acts of misconduct by 13 year old boy",
"src": "Patient: A 13-year-old boy has been forced to come see you by his school's principal for repeated acts of misconduct, including a recent episode in which the patient pulled a knife on one of his teachers. The patient's mother says she has \"given up hope\" and thinks their coming to see you is a \"waste of time,\" as he has been a difficult child since early he was small. She says the boy takes after his father, an abusive alcoholic who left shortly after the patient was born. She reports that the boy frequently skips school and has been arrested three times on charges of stealing, vandalism, and setting fire to a neighbor's dog. The patient smiles and even giggles a few times during your conversation with his mother. The patient seems fairly intelligent and even charming at times, and is alert and oriented. When asked about the knife-pulling incident at school, he becomes sullen and refuses to talk about it, mentioning only that his teacher is \"lucky\" she didn't get stabbed. The boy then refuses to talk any further and simply stares at the wall and fidgets during the rest of the interview, ignoring all remaining questions. Doctor: DearWelcome to HMWe understand your concernsI went through your details. I suggest you not to worry much. I sincerely opine that the child definitely is a victim of lack of care and security during developmental periods. Criminal mentality is clearly visible and he could be troubled with borderline personality disorder. You should assess himm with the help of a clinical psychological using psychometric testing methods for BPD. He may also need psychiatric intervention.If you still need my help, please describe the whole problem in detail and post a direct question to me. I shall definitely help you with psychotherapy techniques to over come your problems.Hope this answers your query. Available for further clarifications.Good luck."
},
{
"id": 94755,
"tgt": "Abdominal pain, nausea, constipation. Pain due to appendicitis?",
"src": "Patient: I have been having lower right abdominal pain for a while now. It comes and goes during the day. I haven t really had a loss of appitite. I feel nauseous but i just burp. I have a little bit if constipation . The pain moves back and forth from my bellybutton to my lower right abdomen . My parents think it is appendicitis . Should I seek medical attention now? I really need to know what is wrong with me because I m tired of hurting. Doctor: Hi Appendicitis is generally accompanied by fever. there are other structures in the area - ovary, tubes, ureter and so on. Are there other symptoms? - bladder or bowel or gynaecological? Appendicitis is often a clinical diagnosis - doctor's clinical examination, corroborated by laboratory Your description does not suggest appendix - yet it is highly recommended to see a doctor. It is better to look than wait! Good luck"
},
{
"id": 163294,
"tgt": "Is Cefprozil ideal to a child suffering from pneumonia?",
"src": "Patient: I accidentally gave my son a double dose of Cefprozil (treating him for pneumonia). He is 16 months old. It has been 5 hours since the double dose occurred and he has vomited twice, each time bringing up food. He is to be taking 3.5mL twice daily. I gave him 7mL this evening by accident. What should I do? Doctor: Hello,I think vomiting is a good thing in this case and help your baby feel better. You may give a glass of warm water as gastric lavage help him vomiting, the next part of medical.Then, you can feed your baby, so nothing to worry about.Hope I have answered your query. Let me know if I can assist you further.Regards,Dr. Elona Dashi"
},
{
"id": 118228,
"tgt": "Have rapid fluctuation in the WBC count in the blood",
"src": "Patient: My relative has been diagnosed with AML type M2. Under this condition, her wbc count was initially 28,000. It went up to 37,000. With initial treatment platelet and mild chemotherapy, the count has come down to 0000. Does this show an improvement of her condition, leading to hopes of cure ? Doctor: hi..WBC count dropping to zero is a good sign of response to treatment. but it doesnt mean that the patient is cured. chemotherapy has to be continued until the bone marrow shows complete response to treatment, also after that maintenance course has to be taken to prevent relapse. if there is no relapse within 2 yrs after stopping the treatment, then you can have hope of cure. meanwhile you should take good care while on chemotherapy, as they are more prone for infections. all the best."
},
{
"id": 77279,
"tgt": "Is pain in neck, aram and chest after lifting heavy boxes concerning?",
"src": "Patient: HI I am 30 yr old women who just start working for ups. I been lifting heavy boxes a lot and today I start to notice chest pain. I never felt this pain so I'm concern. It's in my neck,left arm and left chest. Please help its been going on for about 30 min now. I can breath fine I just never felt pain in my chest. Oh I haven't smoked In over 2 years but also lately I been smoking socially like one cig a week. Doctor: Hi,Dear,Thanks for your query to HCM.Dear I read facts of your query and reviewed it in context to your query health issues.I understood your health concerns and feel Concerned about them.Based on the facts of your query the reply of your query is-Pain in chest and left arm and neck is mostly from heavy work up.Seems to be from Muscle injuries due to heavy work up.Direct history of lifting heavy boxes with young age of yours, reduces the risks With a good lifestyle and care about your health, you need not worry of heart attack.But in the absence of other information,I would suggest you to go for Cardiac Consults.As the facts could be different from what you submitted.So looking in to the importance of heart functions,I would suggest you to rule out ischemia possibility by Consulting and investigating with Cardiologist doctor.It would be prudent to have physicial Check from Consulting Physician before planning further investigations and treatment for it.Other causes need to be ruled out in any case.Do's -suggested for your case-Check with Physician and thereafter with Cardiologist if need beGet ECG done and holtzer to rule out any cardiac ailment.Don't miss to have checks for neck and arm by EMG and Nerve Conduction tests if needed, to rule out any causes of the neck-/and left arm illnesses.Tab-Motrin-400 mg x 3 times a day x 5 days Just don't worry and be patient and co-operate with your doctors,till you verify with your attending doctors.Need to Check with family Physician and Cardiac-specialist if need be, if things don't recover.-Other causes need to be verified with your family GP doctor and team of -Specialist doctors, as the facts may be different what you submitted.-For this Second opinion from PCP-or Gp doctor and if need be with Cardiac specialist-is needed who would do proper investigation after proper physical assessment to resolve your health issues.-for early recovery-please follow above do's which would resolve your complex health issues.Hope this would help you to treat your health issues in the best way possible. Welcome for any further query in this regard.Good Day!! Dr.Savaskar M.N.Senior Surgical SpecialistM.S.Genl-CVTS"
},
{
"id": 124097,
"tgt": "Feeling like punch nerves & sharp shooting pain on hip",
"src": "Patient: I been having like a sharp pain from my right hip shooting down to my knees and when I stand it hurts even more. It feels as if it is a punch nerves. I been putting icy hot n massaging every night I even do so stretch to see if it helps but it been getting worse. I don t know what to do Doctor: Hello, As per the history, it sounds more of the pain which is radiating in nature. This pain can be due to either of the reasons. Piriformis syndrome or Nerve impingement. In Piriformis syndrome, the Piriformis muscle gets a spasm and the nerve get pressure leading to symptoms. Applying hot water or icing over the Piriformis muscle should provide relief. In case if this fails then we should look for other things. In nerve impingement, the never coming out of the lumbar spine from the spinal cord get trapped either by the intervertebral disc or muscles of the spinal column. Applying heat pad or icing should stop the symptoms. If it does stop the symptoms then its nerve impingement in the lumbar region. Taking an MRI will be of good to understand that what is the level of damage to the soft tissues. Looking out for McKenzie maneuver should help centralize the pain. You can check this online. Post which, exercises which includes core stability, spinal extensor muscle strengthening, pelvic floor strengthening, and lower limb muscle strengthening should help to a greater extent. You can even take exercise guidelines from the physical therapist Hope I have answered your query. Let me know if I can assist you further. Regards, Jay Indravadan Patel, Physical Therapist or Physiotherapist"
},
{
"id": 5370,
"tgt": "Delayed periods, increased appetite, frequent urination, breathlessness, cramps, brown discharge. Had unprotected sex. Pregnant?",
"src": "Patient: My period was late for 2 weeks, and I've been craving for foods and my appetite became bigger than before, I having cramp on and off recently, I pee more than usual even without drinking much water, I'm more tired than usual and I have backache sometimes , not much, I feel stuffy sometimes and harder to breathe sometimes. today I realise I have brown discharge. Me and my boyfriend have intercourse before my period , He did wear condom at first, then second round, without peeing to clear off the remaining sperm in his penis, we have another intercourse after that, could that means I'm pregnant now? Doctor: Hi, Thanks for writing us. These symptoms may be due to pregnancy because you have done unprotected sex.brown discharge may be because of implantation bleeding.You should done home pregnancy test. You should consult a gynecologist for urine, blood test and ultrasonography to rule out pregnancy. Good Luck. Take care."
},
{
"id": 175581,
"tgt": "What causes vitiligo on body of a chil?",
"src": "Patient: My 11 year old son has lose of pigment that started on his left arm now is on his chest and neck. He also has a foot fungus problem and has lost some toe nails. I don t know if one has anything to do with the other but the lose of pigment is worrying me Doctor: Hi...Thank you for consulting in Health Care magic. Skin conditions are best diagnosed only after seeing directly. I suggest you to upload photographs of the same on this website, so that I can guide you scientifically. Hope my answer was helpful for you. I am happy to help any time. Further clarifications and consultations on Health care magic are welcome. If you do not have any clarifications, you can close the discussion and rate the answer. Wish your kid good health.Dr. Sumanth MBBS., DCH., DNB (Paed).,"
},
{
"id": 131917,
"tgt": "Suggest treatment for arthirtis",
"src": "Patient: hi....my husband has arthirtis in his knees real bad and he s not strong enough for a knee replacement although the ortho doctor said they could give him a epidural but am not sure if hubby could walk enough. he is also diabetic and takes insulin....lantus and novolg. he has had trouble voiding and when i took him to the doctor his white blood count with was high but the PA didn t know where and gave him ciproflaxin because he was pretty sure it was in his lungs. but anyway i thought maybe he had bladder or kidney infection was the trouble he was having going to the bathroom. urine sample showed no kidney or bladder infection so the PA gave him samples of toviaz to help him void. he took it for a week and was way worse than it was before. the PA has him on (2) 50 mgs pf tramadol every 8 hours...i took him to the er a couple of weeks ago and they told him his white blood count was up a little but still no one knows where the infection is and the er doctor put him on a predisonne regimen. i forgot to mention he also had shingles on his left buttock and on his gentials. that started last october.....what a year it has been...he is afraid if i take him to a bigger hospital they won t find anything wrong. he also lost 50 to 60 pounds in a year....i am very worried about him Doctor: Hi your husband is diabetic and has severe arthritis in the knee joint . He also has an intractable infection with increased white blood counts. he has difficulty in voiding but urine reports are normal. He also had shingles on (R) Buttocks and genitals. In my opinion , with his problems , I would recommend 1. Search for the cause of Elevated WBC Counts- could be lungs , any infection of nails , foot or a small sore in back . 2. For knee pain I would recommend Intra-articular injections injections of Viscous substance ( Viscosupplementation) or PRP Injections. 3. For urinary problem- I would suggest MRI L S Spine and NCV Tests along with urine C/S , S Vit B12 levels, Will proceed according to reports."
},
{
"id": 67370,
"tgt": "What does large, painful lump on outer right thigh indicate?",
"src": "Patient: I found a large lump on my outer right thigh about halfway between my waist and my knee. It is painful to touch. I have been running a good amount and doing squats, but hadn t exercised for a day and a half when it appeared. I have a race this coming weekend (3 mi, 5 mi, 5 mi over 24 hours) and I want to know if it is okay to run on. I iced it and elevated it last night when I found it and have also started taking excedrin. Doctor: Hello and welcome to HCM,Is there any history of trauma to the outer aspect of thigh?Trauma is the most common cause of pain and lump on outer aspect of thigh.Other than trauma, pain can be due to muscular spasms, however, muscular spam will not cause lump.If the pain and swelling is responding to conservative measures like icing and elevation, you can continue that.However, if there is no relief, you will have to consult your primary healthcare provider.Thanks and take careDr Shailja P Wahal"
},
{
"id": 162155,
"tgt": "Suggest treatment for white spot on the tongue of a child",
"src": "Patient: My son 10 month old son has a rare genetic disorder. He is primarily affected by cardiomyopathy and neutropenia. He recently had a really bad case of thrush that spread down to his esophagus and caused him to be hospitalized for a week. Four weeks later he had a small white spot that formed on his tongue. Within two days, the affected area grew to the size of a dime. I took him to his pediatrician and he thought that it looked like some type of scar tissue from a burn or bite. Because of the baby s history he gave me a script for nystatin just in case it is another case of thrush. It s been a week and now the growth has spread to the bottom side of his tongue. Any suggestions? Doctor: Hello, It could be a thrush or canker sore. Consult a paediatrician and get evaluated. Antifunagl creams may be required once the diagnosis is confirmed. Hope I have answered your queries. Let me know if I can assist you further. Take care Regards, Dr Shinas Hussain, General & Family Physician"
},
{
"id": 82874,
"tgt": "What are the signs and symptoms of lupus?",
"src": "Patient: I am trying to work out if an elderly relative has lupus. He recently had a stroke clot and possibly has had another one. his walking has got really bad legs hurt and has recently developed like a dry skin on his scalp. he has high blood pressure on lots of medication Doctor: Hicommon sign and symptoms of lupus are--hair fall, oral ulcer, skin rashesfluids in lung and heart coveringskidney problemheadache or depressionblood component deficiencypositive serological testBut I think stroke clot is not due to lupus, it is due to atherosclerosis only.Regards"
},
{
"id": 54537,
"tgt": "What causes elevated bilirubin levels?",
"src": "Patient: past year we have increased bilirubin,are currently at the top,to have mild temperat.no doctor still did not work analizs.29 years and 2 children.what this might indicate?thanks.a few years ago i had 4 of pneumonija-one year,now everithing is ok,but 3 years! Doctor: Hi thanks for contacting HCM...Here your detail history has to be taken and accordingly investigated....First rule out chronic hemolytic anemia peripheral smear examination and retic count.If needed electrophoresis done to rule out hemoglobinopathy....Second rule our obstructive jaundice like Gall stone and cholestatsis by USG...As you have mild fever , chronic inflammation to liver can be the cause like chronic hepatitis....For that first viral cause ruled out by viral marker study with ELISA....If needed alpha 1 antitrypsin deficiency and Wilson disease like condition ruled out....So liver biopsy needed if cause not clear....Do your liver enzymes estimation also by sgpt and sgot..And yes if you have habit of alcohol drinking then it can elevate liver enzymes.....Meanwhile take low fat diet...Avoid alcohol.Papaiya, apple like fruits taken more...One tsp licorice with honey taken for chronic hepatitis....Hope your concern solved.Take care.Dr.Parth"
},
{
"id": 16108,
"tgt": "Rashes behind ear, pus discharge. Wearing hair in ponytail to prevent irritation. Causes?",
"src": "Patient: My 17yr old daughter has a rash or something behind her ear and it is causing the skin to split and weep. it seems to continue to get worse no matter what we have tried. Some doctors have said that she is allergic to her own hair.. but she has been wearing it in a ponytail to prevent her hair from irriating the area.. It is still getting worse. Not sure what to do. Please help Doctor: Hi,Your daughter might be having intertrigo or seborrheic dermatitis. The confirmed diagnosis can only be delivered after a thorough visual inspection of the lesion. However to offer empirical treatment, you can clean the affected area with warm saline water and apply a combination anti fungal, antibacterial and steroid cream like monovate gm cream twice in a day for a week to see response. Hope that solves your query. Take care."
},
{
"id": 59770,
"tgt": "No proper diagnosis, dementia, fatty liver, ammonia levels and bilirubin high. Other possibilities ?",
"src": "Patient: MY HUSBAND HAS FATTY LIVER DISEASE AND GETS CONFUSED AFTER EATING. WE WERE TOLD THEY THINK HE HAS HEPATIC ENCEPHALOPAGTHY. HE IS A NON DRINKER. WAS ORIGINALLY DIAGNOSED WITH DEMENTIA , NOW THAT THE AMMONIA LEVELS ARE HIGH AND BILLERUBIN UP, THEY SAY ITS MOST LIKELY THE LIVER. EVEN WITH LACTOULOUSE AND RIXFAXIMIN HE IS GETTING WORSE WITH CONFUSION. WHAT OTHER POSSIBILITIES COULD BE GOING ON. Doctor: Hello and welcome to HCM all the findings in your husbands case- increase in levels of ammonia, high bilirubin, dementia, confusion after having food point towards the diagnosis of Hepatic Encephalopathy. This diagnosis is already made. However, now there is no response to lactulose. It is due to the progression of the disease process going on in the liver. Now, the condition has progressed from simple fatty liver to more serious state. You need to get a liver biopsy to know the exact picture. In case of end stage liver disease or liver failure, liver transpant is the next step in management. Thanks and take care Dr Shailja P Wahal"
},
{
"id": 153726,
"tgt": "Why is ovarian lymphoma a contraindication?",
"src": "Patient: Hello I had ovarian lymphoma last year treated with chemotherapy after surgery (it was a primary ovarian lymphoma, very rare presentation) and am in complete remission. My tummy has changed a lot after surgery and I am considering laser lipo to improve shape but do not understand why cancer is a contra indication. Can you explain and say whether it's extra precautionary or whether there are valid reasons. Doctor: Hi,Thanks for writing in.It will help to know the time since you took your last treatment. At times the abdominal wall might be weak and there might be associated relative contraindications to liposuction after cancer. This happens because your general immunity might be low and your body might be recovering from the associated effects of medicines given during your cancer.Many patients might also develop increased abdominal fat due to regular intake of steroids and this gradually disappears with time. Please discuss with your doctor in detail and then know the risks and benefits and any time period to wait for before getting it done. Please do not worry."
},
{
"id": 152209,
"tgt": "Seizure type attacks with unconsciousness",
"src": "Patient: seizure type attacks with and without consciousness, weakness in all limbs and rubber band legs My problem started about 4 days and they have continuously gotten worse each day I am having a severe attack of what ever is going on. This has happened before just never this severe. I started 4 days ago fainting every time I tried to walk even 10 feet or so. I wasn t loosing complete consciousness but I would go into these horrible seizure like attacks that would violently thrust me to the ground and then fling me forwards and backwards. Then I started having extreme weakness in my hands (primarily left) that made it hard to carry anything knowing at any time I could drop it because with no warning I would loose my grip and I had 0 strength to pick it up. Then it moved to my legs where they felt like rubber bands and could barely withstand my weight. They start wobbling and I have no control over them. It is even hard to move them from my bed to the floor without having to physically pick them up and place them on the floor. I am completely freaked out at this point. I also have a ton of other things going but I dont know if they are related or not like aches and pain all over like being bruised with out the bruise. And just an achieness all over my body. I am hoping someone can give me a little guidance as to what is causing all of this. Other symptoms (not sure if they are related or not) are Slight numbness of left leg almost complete short term memory loss blurred vision head aches loss of apetite nausia dry mouth hard to swallow I am pretty sure all of this has been happening on and off for about 10 months. However the ruber band legs has only happened twice. Can someone please help me on this. Thank you Doctor: it is urgent and you should get yourself admitted in a hospital under neurologist's care thorough investigation required to reach a diagnosis you need c t scan /m r i to rule out any condition involving brain do not delay hope you be all right"
},
{
"id": 119338,
"tgt": "Low platelets count, liver spleen enlarged, TORCH, VDRL, BCR, ABL negative",
"src": "Patient: My daughter is having some issues in blood...low platelates count since birth... liver splen enlarged.. 2 times bone marrow done, nothing found...Haematological teats for acute leukamea and BCR/ ABL done...both negative....TORCH, VDRL also negative.kindly help Now she is 1 year old, platelates counts are 2 lacs...but liver and spleen are still enlarged...What should be my next step. Kindly revert at YYYY@YYYY Doctor: Hello, I can understand your concern regarding these negative tests. However, only a careful systematic approach will ensure that nothing is missed out. Presently, peripheral blood and bone marrow cell morphology need to be done, if they've not been done already. The platelets and their bone marrow precursors need to be carefully assessed, so as to understand what exactly is responsible for the low counts. Since you said the count has been so since birth, we may be dealing with a congenital platelet disorder, but the tests you mentioned are insufficient to unravel which one it is. You should see a specialist hematologist, to help you through the process. Best regards."
},
{
"id": 162475,
"tgt": "What should be done for Mebendazole overdose in a child?",
"src": "Patient: Hello, My 13 year old son weighs 170 lbs. He takes generic Biaxin 500 mg and Mebendazole 100 mg twice daily for Lyme, Bartonella, and Babesia, as well as other supplements. Tonight he accidently took his evening dose of the RXs twice, about an hour apart (11:30 p.m. and 12:30 a.m.). I just contacted a pharmacist who said the Biaxin was not an overdose amount, but the Mebendazole (anti parasitic) may be. What do I need to do? Doctor: Hi, I will agree with your pharmacist that the amount of the antibiotic, Biaxin, is not an overdose. The usual dose of Mebendazole is 100mg. Side effects of Mebendazole can be tiredness, hives, chills, and seizures. If your son has not had any of these side effects, I don't think the extra 100 mg will harm him. You need not take him to the emergency room. Hope I have answered your query. Let me know if I can assist you further."
},
{
"id": 217175,
"tgt": "Suggest the type of specialist to consult for Reflex sympathetic dystrophy",
"src": "Patient: I m desperately looking for a highly skilled and experienced doctor in treating RSDthe Charlotte area. I m currently being treated at SOUTHEAST PAIN CARE and we re just not getting anywhere. He s actually an anesthesiologist not pain management specialist. What s the difference anyway between an anesthesiologist and a pain specialist. Doctor: Anesthesiologist will have more idea as well information in terms of giving anesthesia but he may not have idea on how to deal with pain in different condition. He can be useful only for pre operative anesthesia or local anesthesia.Where as pain management docotor will also deal with route cause of pain so it may not come back and patient gets cured faster as well for permanent for possible treatable conditions. Now to answer your question for reflex sympathetic dystrophy I suggest you consult first neuro physician and based on his advise and medication take a view of physiotherapist as well pain management doctor for further improvement.As per my view I believe the major issue with rsd is pain, swelling, and vasomotor dysfunction. And this can be treated well with regular exercise use of hot and cold pack for swelling and reduce inflammation too. As well with regular exercise work out body releases certain hormones to regulate the imbalance within the body and helps to heal the inner body. So also concentrate on curing the cause rather then just managing the symptoms of pain. One last suggestion as a physio and nutrition specialist keep more of omega 3 in your diet. It has anti inflammatory as well hormonal balancing natural property which will be helpful for you. Take care."
},
{
"id": 223647,
"tgt": "How effective is the Mononessa birth control pills?",
"src": "Patient: Hi, I take mononessa birth control. I take it everyday within a two hour window: 1-2. I have been on it for a couple years while having a very active sex life. I have never gotten pregnant with them and I am not worried about it. But should I be since I take it within a two hour time frame? Doctor: Hello dearI understand your concernMononessa birth control pill is very effective in preventing the pregnancy after regular use.It has just 0.5-2% failure rate.Yes you can take it in 2 hour frame time.On irregular use more chance of the side effect.Nothing will make difference in 1-2 hours window.But make habit to take regular by putting the reminder in the mobileIf you will miss tablet then take it as early as possible or on next dayIf you will missed two tabet then take it on next day with same day tablet and use barrier contraceptive like condom for next 7 days.Avoid stress, take healthy diet, do regular exercise and drink plenty of waterHope this may help youContact HCM for further hea;th queryBest regardsDr. Sagar"
},
{
"id": 175620,
"tgt": "What should i do as the medicine accidentally went to my baby's eyes?",
"src": "Patient: my 16 month baby is prescribed maxtra syrup for cold (cntaining phenylephirine hydrochlride and chloropheneramine maleate). While giving her medicine he resisted and accidentaly it went in her eyes. i am worried about its effect. please let me know what should i do Doctor: DearWelcome to HCMWe understand your concernsI went through your details. I don't think there is anything to worry. The tears has the capacity to wash off any foreign substances accidentally falls into the eyes. In the same way, tears would have washed the syrup away. You can wash the child's eyes with clear running water. In case irritation persists, please consult an eye specialist.If you still need my assistance in this regard, please use this link. http://goo.gl/aYW2pR. Please remember to describe the whole problem with full detail.Hope this answers your query. Available for further clarifications.Good luck."
},
{
"id": 123391,
"tgt": "What is the procedure to cure bilateral leg pain?",
"src": "Patient: i had a percutaneous disekomy(stryker) on L3/4 because of posterior disc bulging deforming thecal sac without compromise of neural or foramina. i have radicular L3/4 leg bilateral pain, shin, knee, quads. it is 6 weeks later, pain is same. what procedure do i look for next? thanks Doctor: Hello, I will advise you to take the help of a Physiotherapist who can help with therapeutic ultrasound therapy for reducing inflammation and TENS therapy to reduce pain. Post which exercises to help improve the stability of the spinal column and strength for allowing muscles to hold the spine stable to avoid nerve entrapment. Usually, most cases are receiving guided physiotherapy and are helped better. Research studies show that physiotherapy plays an important role in helping in such conditions. Exercise includes - core stability, spinal muscle strengthening, hip muscle strengthening, and lower limb strengthening exercises. Hope I have answered your query. Let me know if I can assist you further. Take care Regards, Jay Indravadan Patel, Physical Therapist or Physiotherapist"
},
{
"id": 129824,
"tgt": "What causes pain under the ribs?",
"src": "Patient: Pain upper right side under rib. Hurts to take deep breath cannot lay on side. Have had fever up to 101.6 for past 3 days lots of discomfort. Have been told several years ago I had fatty liver. 54 yr old female. Gallbladder removed in 1988. Appendectomy in 1987 Doctor: Hello!In my opinion its sceletal pain, if related to activity/movement.You should see Your primary doctor first"
},
{
"id": 88241,
"tgt": "Suggest cure for sickness in the stomach",
"src": "Patient: have a friend who had a very painful attack inher stomach and brought up turquoise bile and had a hard time to breath she said she is as white as a sheet the pain is something else she is waiting to go for tests but its a long wait, now she is scared to wait. the pain lasts for about an hr. Doctor: Hi.Thanks for your query and concern for your female friend. Read the history and understood the problem.The most probable cause of such a severe attack can be due to one of the following reasons:-The bile coming in vomiting indicates that there can be an intestinal obstruction-Such a paleness occurs only in few things like- Torsion of the ovarian cyst.Rupture of the tubal pregnancy.Rupture of the ovarian follicle with more bleeding or cyst.I would advise her and you the following:Take her urgently to the ER or the nearest hospital, get her admitted.Keep her nil orally as she may need an urgent surgery.Urgent Standing X-ray of the abdomen and ultrasonography of the abdomen.Surgical exploration may be needed, be ready for this and inform her relatives."
},
{
"id": 71373,
"tgt": "How is respiratory failure treated?",
"src": "Patient: My husband had an APR May 16th. There have been no problems with stoma until hospitalized for 1week for respiratory failure. Ostomy care was sub-standard and when i brought him home yesterday, found the stoma partially covered by wafer and the whole inside perimeter of wafer packed with a quarter inch of dried feces, the area around the stoma surrounded with bloody feces. pouch had blood and after i cleaned stoma, I discovered stoma had been bleeding, enough that it flowed down his belly. I applied gentle pressure but it has continued to bleed and form large clots near the stoma opening. Called surgeons office this morning, explained problem and she said doctor will see him tomorrow afternoon. Is this soon enough, or is this a more urgent problme? Doctor: Hello,As you explain the history, I think that this is an urgent situation which needs medical help.Hope I have answered your query. Let me know if I can assist you further.Regards,Dr. Jnikolla"
},
{
"id": 120947,
"tgt": "Suggest remedy for tender ribs with spams",
"src": "Patient: I got kicked on the lower ribs playing football, four days have passed and its very tender I also get terrible spasms when I stretch or make any movement, the worst problem is when I m lying down & then try and get up - spasms are very painful. Thanking you for your co-operation. Andrew Doctor: Hello,The symptoms seem to be related to muscle spasms.I suggest using a muscle relaxant such as Baclofen three times a day. I also suggest using magnesium supplement for muscle relaxation. Hope I have answered your question. Let me know if I can assist you further. Regards, Dr. Dorina Gurabardhi, General & Family Physician"
},
{
"id": 131438,
"tgt": "What causes pain and soreness in both the arms from the shoulder o the elbow?",
"src": "Patient: I have pain and soreness in both arms from the shoulder to the elbow. A doctor gave me enough medicine for about four days, prednisone ,steroids. In the fourth or fifth day the pain seemed to be going away. Next day my pain was back. I was also told to exercise the arms , which I have for a couple of weeks, no change. I have been told that a neurologist would be the best specialist for my problem. Please help. Doctor: HiIt could be frozen shoulder both sides.injection depomedrol infiltration is usually helpful and phys therapy. Exercises help.Take Aleve tab twice a day after meals for 10 days,take up physiotherapy.Inj depomedrol infiltration may be repeated.Frozen shoulder is self limiting and does often takes few weeks to subside.Exercises after hot fomentation and application of muscle relaxant ointments or gels, gives best outcome.even voltaren gel massage helps.Wait and watch..As advised by your doctor, do seek help of a neurologist too"
},
{
"id": 162554,
"tgt": "Suggest an alternative of Nan Excella Pro",
"src": "Patient: My twin babies are feeding lactodex lbw. After 3kg both switched it to nan excellapro. But one baby have heavy loose motion whenever we give it. Kindly suggest another baby food which should follow after lbw.other than nan. Lactodex any other one can you suggest also Doctor: Hello and Welcome to \u2018Ask A Doctor\u2019 service. I have reviewed your query and here is my advice. You can give Lactogen instead of nan excella pro. Stage one can be given up to 6 months. You need to change to stage 2 at 7 months . How to mix : take 30 ml of heated and cooled water and mix 1 scoop of milk powder in to it. Proper mixing is must. You should not add sugar . Hope I have answered your query. Let me know if I can assist you further."
},
{
"id": 165862,
"tgt": "Suggest treatment for vertigo in a child",
"src": "Patient: sir with due respect that my ten years son is suffring from virtigo on his right check just near the ear. we used two medecines mometasone furoate & terbinafine hci cream & placentrex lotion but he don t got any releif. So please give us your right treatment to us. Doctor: Hi....by what you say I feel that this is vitiligo and not vertigo. Skin conditions are best diagnosed only after seeing directly. I suggest you to upload photographs of the same on this website, so that I can guide you scientifically. You can approach me at the following link.Once the page opens there will be an option below my image as \u2013 ASK ME A QUESTION \u2013 click on it.Please find the link below - www.healthcaremagic.com/doctors/dr-sumanth-amperayani/67696Regards - Dr. Sumanth"
},
{
"id": 120974,
"tgt": "Suggest the dosage of Ibuprofen while treating mild arthritis",
"src": "Patient: I suffer from mild arthritis.I take two 200mg ibuprofen (cheapest, supermarket) tablets before two hours of tennis.For maximum effect, how long before playing should I take the pills?i.e how soon do the pills take effect and how long do they last? Thank you for your advice. XXXX Doctor: Hi,For minor pains of arthritis initial dose is 200 mg orally, may increase to 400 mg orally every 4 to 6 hours as needed.You can take the drug half an hour to forty minutes before tennis.The effect of drug can last upto four to six hours.Hope I have answered your query. Let me know if I can assist you further. Regards, Dr. Saranya Ramadoss, General & Family Physician"
},
{
"id": 49209,
"tgt": "Is kidney transplant right treatment for CKD 5, creatinine 18, pulmonary edema and RLS?",
"src": "Patient: HiMy brother is diagnosed with CKD 5 and now undergoing dialysis thrice a week.His current creatinine level is 18 and has great difficulty with pulmonary edema and RLS.We have decided to do a kidney transplant ASAP.My mother is the donor.Will be he OK till then? Doctor: Hi and thank you so much for this query.I am so sorry to hear about what your brother is going through. This is the best treatment for CKD stage 5 with multiple dialysis sessions weekly. It would be the magic pill that can make him feel a lot better and even live without the need of dialysis. I hope it goes on as planned and great!I hope this addresses your query fully. Thank you so much for patronizing our services and please do feel free to ask for follow up clarifications and information if need be. I wish him the best of health and your mom well.Dr. Ditah, MD."
},
{
"id": 34757,
"tgt": "Why do I get huge infection after a small cut?",
"src": "Patient: Recently everytime I get even a small cut or bug bite I've been getting huge infections in these area's. Usually I have to use a needle and pop below them to get a greyish yellow puss out. Recently I've gotten on one my leg and now the puss is coming out a purplish red and the area around it is probably 3 inches in diameter. Just to give a quick personal history my wife and I are swingers and have been with many couples over the last 3 years, also I smoke, am over weight. My daughter was diganosed with Mersa a couple years ago, that she got from wrestling. Any ideas on the possible cause of these infections or what I should ask my family doctor when I go in Doctor: Hello. Welcome to HCM.I am Dr Jigar(ID specialist).Yes you need to see expert doctor.Looking at your profile i am sure that your immune system is weak either due to lifestyle and metabolic defects or it may be due to Other secondary causes like HIV also needs to ruled out as you are promiscuous.For this you need to go for primary blood test like CBC, blood sugar, urine analysis, liver function test. Other tests like HIV,HbsAg,HCV must be done.Please First see your doctor, get the tests done and then proceed. I hope this will help you. Give your valuable feedback.Regards.Dr Jigar."
},
{
"id": 148998,
"tgt": "Child with cerebral palsy, spastic diplegia, vomitting, unpleasant smell while burping, diarrhoea. Advice?",
"src": "Patient: Yes, I have a 3 1/2 year old son with cerebral palsy , spastic diplegia , and it started on Saturday. He was vomiting, but not alot. He continued to seem sick, and vomit maybe every 4-6 hours. He has not puked since yesterday morning, but now every time he burps it smells like rotten eggs or feces. Now he has diarrhea real bad. Is this something that will blow over, or should I take him in to see his doctor? Doctor: Yes definitely, you should see a gastroenterologist. It appears that some intestinal obstruction may be the cause for these symptoms. you should take him today it self"
},
{
"id": 27131,
"tgt": "What causes rapid heart beats?",
"src": "Patient: as of the past few days, i am experiencing rapid heart beat. it was short at first but has increased in duration to about 10 minutes at a time. when it occurs, i am not doing anything strenuous. i am 36 and in relatively good health. is this something i need to worry about? Doctor: Dear- thanks for using our system. Rapid heart beat can be secondary to multiples causes. It is important to find out if the heart rate is more thatn 100 per /min and if it is regular or irregular.The best way to find out is taking your pulse and having an electrocardiogram. Medical conditions as panic attack or anxiety, hyperthyroidism, caffeine intake , are some examples of rapid heart beat. You need to see your doctor for a checkup, electrocardiogram and 24 hr holter monitor trying to identify why type of arrythmia you have. Some valvular conditions as mitral valve prolapse can give you rapid heart beat too.I hope that my advise has been helpfulDR.Sara"
},
{
"id": 89313,
"tgt": "Does swelling in the groin area indicate hernia?",
"src": "Patient: Hi, I'm a 37yr. old female, and I have a bulge in my right groin area. During my annual physical I showed it to my Gynecologist. She said that I have an Inguinal Hernia. She sent me to a Surgeon, but he couldn't really tell if it was or wasn't a hernia. I started having pain and swelling again, so I went to see a different Surgeon. Because the swelling had went down he too could not say for sure that it was or wasn't a hernia. I've noticed that it only swells up during certain times of the month. By the time I make an appointment & go to see the doctor; the swelling has gone down making it hard to identify. I've had this problem for almost 2yrs. Is there anything I can do? Doctor: Hi.The swelling in the groin can be a Lymph node or a Hernia. Hernia is a swelling which is expansible - meaning swelling is increased on straining like during coughing and is Reducible- meaning can be pressed down on pressure. Whereas the lymph node is neither expansible or reducible. The simple test at home and by Your Surgeon can diagnose this easily and ultrasonography can help differentiate between the both. Get operated for two reasons. The disease is gone and one gets the perfect diagnosis."
},
{
"id": 207195,
"tgt": "Is stealing things and screaming sign of personality disorder?",
"src": "Patient: My 7 year old nephew steals and stashes clean and dirty panties bras and bathing suits. He has been doing it for 2years. At first he only stole his mothers. Now he steals them from everyone. Weve even found panties that do not belong to anyone. He usually stashes them quickly in the same room they were in. Sometimes he locks himself in the bathroom urinates on them in the bathroom. Theyve even been found stuffed in flashlights. My sister moved recently and found that he had packed a backpack full of undergarments. Before he started this he would throw fits and scream and even hit and kick strangers. Do you think that this could be an outlet for a personality disorder? Doctor: Hi,Thanks for writing to healthcaremagic.Personality disorder can't be stamped until age of 18 years reached.He might be getting pleasure from doing all these stuff.It might be component of ADHD(Attention Deficiet Hyperactivity Disorder) or may be paraphilias especially fetichism.You must consult child psychiatrist for this because at this age it's more easy to unconditioned his behavioural pattern.Hope I have answered your query,"
},
{
"id": 216951,
"tgt": "What can cause leg pain while sitting and laying?",
"src": "Patient: Thank you....I am experiencing recurring pain radiating from my left thigh all the way through the left side of my lower left leg. Does not affect my hip or lower back and is worse below the knee. Pain is very bad when lying down or sitting but walking around does seem to help lessen the pain. Never mind.....not going to pay for your answer. Will wait until I can see my own physician and let Medicare pay for it! Doctor: Hi there. Thanks for your question.It is not uncommon to have radiating sciatica kind of pain at rest. Pain whcih is present on sitting and lying down, relieved on movements, suggest an extraforaminal kind of disc herniation, whcih is particularly common in people above 50 years of age. It is quite distressing since rest exaggerates the pain. I would suggest you to get an MRI and re confirmation the finding. aonce its confirmed injection shots give considerable relief.If no pain relief with injection, the next step would include surgery. But surgery would surely be the last step in the ladder. Hope this helps. All the best. Regards.Dr.SBK"
},
{
"id": 57905,
"tgt": "How to get rid of the pinching cramp in my lower left side?",
"src": "Patient: I went onto depo 3 months ago and was due last month. But ive skipped my period and have symptoms of nausea in the morning and evening. Fluttering tummy, my breast r tender and my stomach is unsually big. And ive gained weight. Could thus be because ive stopped using depo? Doctor: Hello!Thank you for the query.Your symptoms sound like a pregnancy symptoms. If this is possible, please have pregnancy test at first. Such symptoms can sometimes appear after steroids usage depending if it was muscles or joints injections, or it was in tablets. Sometimes, long steroids intake can cause adrenals insufficiency and give such symptoms.So please consult your doctor and have pregnancy test at first. General blood work, ions levels should be checked. You should also consult endocrynologist.Hope this will help.Regards."
},
{
"id": 5104,
"tgt": "Trying to get pregnant. Got Choriomon 5000 shot. Prescribed Duphaston. What is this process?",
"src": "Patient: hello doctor,i'm trying to get pregnant and my doctor said i ovulate within 3 days . she also gave me a choriomon 5000 shot . i just want to make sure what is it exactly. and she said after i should start taking Dupbaston tablets one per a day for 10 days and then i should be pregnant . can you explain what happening exactly? Doctor: Hi,Thank you for choosing Healthcaremagic. Choriomon 5000 IU is HCG (Human Chorionic Gonadotrophin). Once the ovarian follicle has reached optimal size, injection HCG is given for ovulation. Then after 36 hrs of injection, ideally scan is done if the follicle is ruptured and planned for intercourse/ intrauterine insemination.I hope this information has been both informative and helpful for you. In case of any doubt, I will be available for follow-ups. If you like my answers kindly rate it, and write a review as well. Please do not forget to accept it.Thank you,Wish you good health.Regards,Dr ArifYou can consult me again directly through my profilehttp://www.healthcaremagic.com/doctors/dr-arif-n-khan/65133"
},
{
"id": 23582,
"tgt": "Suggest treatment for low LDL levels with abdominal ectasia",
"src": "Patient: I have taken medication for abnormal cholesterol for a numbe of years. I am now 81 and was diagnosed by CT scan with a small abdominal aectasia and extensive plaque along most of the aorta. I have been taking Crestor 5 mg for a few months. It was increased to 10 mg. Tests yesterday show that LDL had decreased from 70 to 17. What is the treatment? Doctor: Hello ,Small abdominal ectasia can be because of atherosclerosis and both are due to age related wear and tear . As such there is No lower limit for Ldl. Studies have found Ldl even lower than 50 had no side effects and decreased atherosclrotic progression . The guidelines recommend that Ldl should be kept below 70 . I would recommend that you may decrease the dose of Crestor to 5 mg . That should suffice and would give you optimum benefit . More than by all means is not required . Regards Dr . Mody"
},
{
"id": 145365,
"tgt": "What is the best medicine for stroke?",
"src": "Patient: My mom is 67 year old & had a mild stroke 5 month back and under Psychiatric medicine(qutiapine) since then. Also doctor prescribed her ecosprine av 75. on 2nd Jan & on 9th feb she become pulse less revived by blowing air & heart pump. Kindly suggest. Doctor: I read your question and I am sorry about your mother. It is hard to know exactly why she had no pulse, you don't give any description on the circumstances and what other signs and preceding symptoms were noted. I believe an evaluation from a heart doctor with ECG and heart echo are necessary.As for the treatment for the stroke, Ecosprine is necessary, it is beneficial even if heart arrhythmia is at the base of those 2 episodes. Quetiapine on the other hand is not for stroke itself, it is given to control agitated patients who can't be controlled otherwise, so if she has no agitation episodes it is not advised as it has many side effects including cardiac ones.I hope to have been of help."
},
{
"id": 173733,
"tgt": "Suggest treatment for toticolis in a 4 months old child",
"src": "Patient: I have a four month old and she has toticolis. My Doc said it will get better when she can hold her neck up. I also got a specialist to see her and he said its nothing. She will be 5 months soon and uIt hasn't gotten any better. What should I do, please help. Doctor: Hi,Thank you for asking question on health care magic.Torticolles usually present from birth due spasm of the Sternocleidomastoid muscle.Some times there may be a sternmastoid tumor in the muscle responsible for torticolles or wry neck.Physiotherapy may help.Better consult pediatric surgeon soon.Hope this answer will serve your purposePlease feel free to ask any more queries if requiredTake careDr.M.V.Subrahmanyam MD;DCHAssociate professor of pediatrics"
},
{
"id": 172473,
"tgt": "Suggest treatment for constipation of a 7 year old",
"src": "Patient: Hi dear Doctor My little boy of 7 years old is having a long term constipation problem. Though, we have attended many medic centers and seeing different doctors, but the problem still exist. Stomach scan and other examinations have been done for him and all doctors say he has no problem, but he still couldn't dispose before 10 to 15 days. Your advice is highly appreciated Doctor: Constipation is very distressing thing for parents and child. We can treat constipation with following measures1.change dietary habits like stop milk, give leafy vegetables, one fruit a day, give much water 2.give him enema with neotonic enema which remove impacted stools. 3.give syp levoluk /gudlux/duphalac start at 10ml at night if still not have smooth stool u can increase dose accordingly. Do not stop drug in between it should be continued at least a month. 4.give him syp position which contain multivitamin and some amount of laxative. 5.in view of chronic constipation got for thyroid screening. 6.give him toilet training7.watch for anal fissures some time it precipitated conyipation"
},
{
"id": 174613,
"tgt": "Suggest treatment for abnormal behavior in a child",
"src": "Patient: my son just had the same episode for the second time in less then a month...he wakes up and he will be playing and just starts freaking out crying for no reason while this is happening his body go limp he cant stand sit up are anything...his lips and face proceed to go blue and about 2-3 mins he comes out of I and hes fine Doctor: Hi....by what you quote I feel that your kid might be having a night terror or nightmares. It would have been great if his age has been mentioned. It could even be a post traumatic stress disorder. This is what I feel he could be experiencing. I suggest you see clinical pediatric psychologist or pediatrician for this.Regards - Dr. Sumanth"
},
{
"id": 5083,
"tgt": "Trying to conceive. Have delayed periods, only light spotting and have light clear mucus. Advise?",
"src": "Patient: i am now about 3 days late than what i expected to get my cycle. my periods are irregular but i usually have a sense of when i should get them. this morning went to the bathroom and when i wiped i had clear/brown mucus. i thought it might of ment that my period was coming so i put on a pantie liner went to work and only had 3 very light spots in it and have not had any more sense however i am now noticing light clear musus on the paper when i wipe im trying to get pregnant and have been taking vitiamins that are supposted to help since the 3rd of july Doctor: Hello, Thanks for the query to H.C.M. Forum. Since period is very light and just spotting . So please get in home pregnancy test and conform it. Light spot is very common even after pregnancy test positive. Good luck. Dr. HET"
},
{
"id": 223612,
"tgt": "What are the chance of getting pregnant despite of contraceptive tablets?",
"src": "Patient: from: anonymous (kindly dont mention my name in your reply, thanks) hi doctors i am 37 yrs old with very regular menstrual cycle of 26-28 days. me & my bf had an unprotected sex yesterday afternoon. the first day of my last period was last may 7 and yesterday was my 13th day, im fertile right? i took 4 white tablets of nordette pill (its a 21 pill nordette) yesterday at 6pm and took again at 6am 4 tablets after 12hrs. question: will the chances of being pregnant high? or will nordette pill prevent it? when do i expect withdrawal bleeding? is this means that im not pregnant? how will i be so sure? when do i expect my next period? please help. thanks. Doctor: Hallow Dear, You are right; the day of your sexual play was very near to the day of egg release; hence you had sex in the fertile window. In fact to cover this unprotected intercourse, you should have taken some emergency post coital contraceptive pills within 72 hours of the intercourse. However, you had consumed 4 pills of Nordette, which very much is near the dose of post coital pills. There was no necessity to repeat the pills after wards for two times. You are well protected from the pregnancy due to this particular intercourse. However, due to so many pills you have taken, your following menses will be delayed by more than a week or so. You need not be concerned about it. Still if you are stressed with the possibility of the pregnancy, you may perform pregnancy test on your overnight morning first urine sample a week after the missed period; earlier the test may report false negative results.Also due to so many pills, you may experience increased stomach acidity, nausea and vomiting. Some proton pump inhibitor antacid like pantoprazole will give you relief from these symptoms. Henceforth, please make it a point to keep some post coital emergency pills handy for such incidences. I hope you are happy with this advice now. Dr. Nishikant Shrotri"
},
{
"id": 26503,
"tgt": "Is BP of 120/100 very high?",
"src": "Patient: Hi,My mom aged 59 has problem of arthritis and has high level of physical activity. Though she is fit but her BP is 120/100.This is by chance that she got it checked on visist to GP and came to know about it. Is this too high?What precuations should she take? Doctor: Hello,I have gone through your query.Thanks for using HCMAs such reading of 120 upper I.e.systolic is very much normal.But lower diastolic 100 is high.First of all she should reduce her salt intake .If she is on NSAID group of medicine or steroid she should modify medication in consultation with treating doctor .she should avoid resistant training exercises.My best wishesDr.Rajesh Teli MD"
},
{
"id": 125293,
"tgt": "Suggest treatment for L4-5 Concentric disc bulg with posterior central annular tear",
"src": "Patient: I am a 43yr old white male just diagnosed with L4-5 Concentric disc bulg with posterior central annular tear. Moderate facet arthropothy is present. Mild indentation of thecal sac and minimal bilateral foraminal narrowing is present.symptoms - burning middle of lumbar with movement, leg burning ,tingling, unstable, wobbly, fear of falling sensation, groin numbness, sitting on toilet is very painful, sleeping is almost impossible, frequent urination (prostate is ok though)...any suggestions? Can I get out of this w/o surgery? Thank you Doctor: Hello, If you are asymptomatic or symptoms are mild, We can proceed with conservative measures like braces and analgesics. In severe cases, an intervention like steroid injection or surgical correction might be required. Hope I have answered your query. Let me know if I can assist you further. Take care Regards, Dr Shinas Hussain, General & Family Physician"
},
{
"id": 131042,
"tgt": "What causes pulsating pain in the knee having a history of GERD and Gastro paresis?",
"src": "Patient: I am a 44 yr old female with a history of gastroparesis, gerd, metabolic syndrome and pcps...i have had a pulsating on right side (inside) of left knee all day on and off..can see it pulse..doesn t hurt doesn t pulse with the pulse in my neck...doesn t feel like muscle twitching..seems to go away when i move...more noticeable when i am sitting standing or lying down.. Doctor: In my opinion you have a popliteal aneurysm i recommend you do a doplar to be safe Please take it seriouslyGood Luck"
},
{
"id": 151832,
"tgt": "Should I take Depakine for EEG reports showing sharp wave activities in the right of the brain ?",
"src": "Patient: I passed out completely twice , one 4 years ago , and last week so the Doc ordered EEG , it\u2019s showed a sharp waves activities at right side of my brain , and give me Depakine, should I take it or seek a second opinion Doctor: Hi,thanks for query.Your eeg is suggestive of epilectic activity,the drug that is given to you is valporic acid,which is videly used in treatment of epilepsy world vide.You can safely take it as per the advise.Make sure you keep good follow up with your doctor as this drug needs adjustment in dose with your weight and response. wishing you good helath."
},
{
"id": 110793,
"tgt": "How to treat back pain?",
"src": "Patient: I had back surgery in January of this year but the pain is still horrible. My most recent MRI states the left s1 descending nerve root appears to be displaced posteriorly. A non enhancing component is noted in the left lateral recess. The possibility of a small recurrent hernia toon should be considered. What does all that mean? Doctor: Hello, I have studied your case.Your MRI says lateral recess foraminal stenosis which leads to compression of exiting nerve root which later on supply lower limb [leg].Due to compression of this nerve root there is tingling numbness in your leg and pain associated with it.Medication like methylcobalamine with muscle relaxant and analgesic will reduce pain; you can take them consulting your treating doctor.You may consult physiotherapist for further guidance. He may start TENS, or ultrasound which is helpful in your case.I will advise to check your vit B12 and vit D3 level.MRI shows disc compressing on nerve root then surgical decompression is permanent solution.Hope this answers your query. If you have additional questions or follow up queries then please do not hesitate in writing to us. I will be happy to answer your queries. Take care."
},
{
"id": 94571,
"tgt": "Experiencing abdominal pain, back pain and have fever. What could be causing this?
",
"src": "Patient: Hello. I am a 21 year old female. I have been experiencing a sharp stabbing pain in my abdomin that comes and goes. It starts out near my navel and spreads throughout most of my abdomin. It lasts for about 30 seconds and becomes almost unbearable, then goes away, only to return again anywhere from a minute to an hour. I am also experiencing constant pain in my lower back. It s dull and achey. And I am running a fever of 100.5. chilling, and then randomly start sweating. no possibility of pregnancy Doctor: Hello, Welcome to Healthcare Magic. I appreciate your efforts for medical consultation in so much distress. Well, u need to answer few questions first before I could suggest you anything, like about your motion habbits, any urinary tract symptoms, any menstrual irregularities\u2026. And also any associated symptoms like nausea,vomiting and name of medicines if taken any\u2026.. Wish you good health. Regards"
},
{
"id": 96726,
"tgt": "How to treat pain between my thighs as hit myself by hockey ball?",
"src": "Patient: I hurt myself playing ball hockey where i felt alot of pain between my thighs and along my pelvic area. I pushed it a bit but quit playing for awhile after the pain became really severe i could barely walk or sleep after i played but over usually a weeks time the pain would recede. I have seen a doctor a few times. At 1st i was given anti inflammatory pills then a cream then i had an xray that showed nothing. Im about to start a new hockey season but i feel really apprehensive because while i dont feel pain there normally when i cough or certain movements i feel pain like if i move suddenly or if im pushing weight with my legs i feel a sharp twinge but its gone right away. I dont wanna go back to the doctor but i dont want to go thru all that again. Any advice would be greatly appreciated thankyou Doctor: Welcome to hcm, Short answer. Your pain is due to injury inthe muscles with or without hematoma Detailed answer .when a hockey or an object is hit in the muscles it will cause a injury to the muscles. So I advice you to take a good rest for 2 weeks If you have a normal xrayApply ice indirectly.by covering with a towel.Take nsaids for your pain.Hot wax bath helpsIf there is no external wound apply elastic crepe bandage in the whole of your thigh. The bandage must not be very tight.it must allow the blood to pass through.Review after a week. To your doctor.Hope this explains.With regardsDr.Amarnath."
},
{
"id": 151525,
"tgt": "Dizziness, chest pain, increased heart beat, migraine, tingling, cold hands. EKG done. What could this be?",
"src": "Patient: Good evening, I am a 34 year old female who started having dizziness , chest pain , pounding heart, migraine , becoming flush, tingling hands/cold hands about 7 days ago. Menses started 6 days ago, was fairly norma. 5 days ago I sought care at the ER. They did an EKG (it was a 2 second ekg and I am not exaggerating) said I was fine and sent me home. Im still having the same symptoms but the migraine isn t half as bad as it was. I m hydrated. I m 5 5 125. The blood tests they did do came back normal but the BUN came back a tad bit low at 5 and the protein came back a little high at 8.3. My blood pressure was a fine. What could this possibly be? Doctor: most likely you having anxiety disorder and get panic attack. consult nearby psychiatrist. paroxetine 25 mg in night would help in this condition...."
},
{
"id": 191912,
"tgt": "What causes high blood sugar levels?",
"src": "Patient: Hi, my doctor recently said I have stomach ulcers,but I keep having pain towards my back in the stomach region and when I walk. I know its not a kidney infection. Today i just have not felt right and tested my blood sugar and it was 150 and hour after my meal. I am not a diabetic. Could i be having problems with my pancreas? Should i go to the doctor tomorrow? Doctor: Hi,After going through your question I want to clear certain facts-1) Blood sugar level of 150 1 hour after meal can be considered as normal.2) Pain in stomach radiating to back may be due to pancreatic problem.Hence I would like to advise you to consult your doctor.There is a need for clinical examination depending on it there may be need for ultrasound of abdomen and pelvis which help in evaluation of pancreas and kidneys, blood investigations like complete blood count, serum amylase and lipase (pancreatic enzymes), liver and kidney function test or CT scan of abdomen.With your mentioned blood sugar level you cannot be labeled as diabetic, I will advise you to do OGTT (oral glucose tolerance test) to confirm your diabetic status.Hope I have answered your query.If so do vote the answer."
},
{
"id": 107091,
"tgt": "How can severe backache be treated?",
"src": "Patient: I have been having sever lower back pain the last few years. I do have discs that are misaligned and bone spurs and arthritis. I have had 2 epidural cortisone shots, but only get relief for a few weeks. I have read Pravastin can cause back pain and I have been taking it about the same amount of time. Could that be the source of the pain? Doctor: dear patient you have got degenerative disc disease which can lead to back pain. Also pravastin has side effect of muscle pain which may be read in for back pain. you have taken two epidural corticosteroids before. why it was given?I would like to see you Mri report if done. if it is showing significant disc bulging with bony spur formation and arthritisand not relieved with epidural steroids surgical intervention may be needed. please send your mri report . thanks."
},
{
"id": 210264,
"tgt": "Can mental problem cause health problem?",
"src": "Patient: My husband has mental health problems and will go for up to 6-8 weeks without eating properly, he will only eat biscuits and drink only orange juice or milk. He does not wash or go to toilet very often either, what are the health problems he is likely to have. Doctor: HiThanks for using healthcare magicIn his case, the chances of getting nutritional deficiency is very high. Better to get him admitted in a mental hospital and get him treated properly. That would help to avoid physical problem like nutritional deficiency, anemia, weight loss, underline infection due to poor hygiene etc.Thanks"
},
{
"id": 54128,
"tgt": "What does the ultrasound report with enlarged parenchyma indicate?",
"src": "Patient: Hi.... in my us scan report liver is mildly enlarged (17cm in vertical span) parenchyma shows homogenous ecotexture, rest everything is normal. I don't have any other symptoms.... I am 5'2\" and 70 kg, and I have 9 month old baby. I am hypothyroid also taking eltroxin 100, Should I be worried. Doctor: 1) Do you have any symptoms of fever, loss of appetite, jaundice, swelling of limbs? 2) have you done any blood test along with USG? If liver function test is normal then there is nothing to worry. you also need to check lipid levels. 3) does USG has mentioned anything for fatty liver other than normal echotexure? If you have this then you just need diet modification by reducing fat and sugar in diet, doing moderate excercise."
},
{
"id": 108463,
"tgt": "Suggest treatment for back pain",
"src": "Patient: I have suffered with a back pain (L5) for 2 years after and prolapsed disc and had recovered but got worse recently and experienced lower back pain and pain in both legs. After 10 days this escalated into numbness from the waist down and admitted to hospital and diagnosed with Caudia Equina Syndrome and had surgery to remove the compression within about 24 hours. It is now over 3 weeks since the operation and still have little feeling in the right foot, have lost a large percentage of muscle tome in my calves and backs of legs and buttocks and still have no feeling artound my saddle area. It is also very difficult to predict when I need to empty my bowels. Whatg are the chances of a full rcovery in this instance Doctor: Dear you have come out of a crisis due to timely intervention of the treating surgeon. Cauda equina syndrome is a urgent indication for surgery and that so because, it causes compression on nerve supplying our bowel and bladder. now since decompression has been done you will have no further deterioration, but the neural deficit you already developed before surgery takes tie to recover and this time depends form 4 to 6 months and sometimes year.Be patient follow the instructions given to you regarding physiotherapy. and you will get ok with time"
},
{
"id": 32960,
"tgt": "What causes recurring blisters on the toes?",
"src": "Patient: hi I have had a blister on my 3rd toe on my right foot twice now I poped it and it came back in the same spot. I don t wear any shoes that would rub it and cause a blister . It has been there for a while now so last night I figured I would just pop it myself. when I did it was a clear very thick liquid that came out of it. nothing I expected to have I thought it might be a sliver or something but that s all that was there. so I squeezed it then it bleed some . should I be worried about it? Doctor: HiThanks for your query at HCM.I understand your concern and situation.I would suggest that you must get a tetanus shot. Do not manipulate the pustules unless with sterlized needle. If pustules are recurring get blood sugar levels checked. Apply topical antibiotics on the lesion like neomycin. Keep the hygiene of area maintained. If resistant wound get culture sensitivity for aerobic and anaerobic bacteria and take treatment according.Take care.Hope I answered your queryDr Sheetal Verma"
},
{
"id": 223066,
"tgt": "What are the chances of pregnancy?",
"src": "Patient: I have been taking the contraceptive pill for years now, an am now on my 3rd diferent one called Dianette. >When i first started taking this pill I didn't wait for the first day of my period and only later found out that i was technically unprotected that hole month i was having sex, luckily I didn't get pregnant. This month I'm in my 3rd and last week of taking the pill and have currentlt missed 5 pills. I once got told your actual chances of getting pregnant while on the pill even missing them is still extermely low. I need to carry on straight to a new pack without having my 7 days break as i'm going on holiday but this means i wouldnt know if i'd missed a period and may be pregnant. As i never got pregnant in that month before i'm thinking maybe i cant get pregnant. help please :) Doctor: Hi,If you have missed pills in your 3rd week, it is correct that you take the new pack immediately without the pill free week, this will give you adequate protection against pregnancy.It is extremely unlikely that you are pregnant, if you have missed pills only in the 3 rd week .So please be reassured and do not worry.But make sure you take your pills regularly without amiss.Enjoy your holiday.Regards."
},
{
"id": 194607,
"tgt": "What can cause painful and bloated stomach after hernia surgery?",
"src": "Patient: Hai Sir, My name is Anitha.Am writing this for my husband,Rajesh.(who is now 34yrs.wt.70kgs.ht.172 cm)He had a surgery for ingenuinal Hernia last week.Now he is having bloated belly and after having food ,he is experiencing pain in the stomach.Please advice Regards Anitha Doctor: Hi, Post op problems has to be discussed with the surgeon who operated on you. Hope I have answered your query. Let me know if I can assist you further. Take care Regards, Dr B. Radhakrishnan. Nair, OBGYN"
},
{
"id": 71831,
"tgt": "What does gray phlegm while coughing indicate?",
"src": "Patient: , may I answer your health queries right now ? Please type your query here... I've been having a soar throat where it feels like I have a peice of bone stuck on the one side of my throat. I cough,and when I cough I have a grayish green phlegm. I also notice that during the mid day it goes away,at night it starts to come back and when i wake up in the morning it hurts the most. I looked at the back of my throat in the mirror and it looks like i do have swelling of some sort one one side of the back near the tonsil. Any ideas? Doctor: HelloAs you explain the history you should be examined by an ENT specialist for evaluating the throat and the larynx better with a laryngoscope.ReagrdsDr.Jolanda"
},
{
"id": 4519,
"tgt": "Does spotting, cramping & clots on visiting toilet mean period/ pregnancy/ side effects of starting the birth control pills late?",
"src": "Patient: I have been on birth control to regulate my period, but my dr. was a week late to fill my prescription, I had ended up having unprotected sex on December 10 ive been having sore breasts and that's about the only difference I notice in my body, I was suppose to start my period on Jan 2 and I never got it. I took 2 home pregnancy tests and they both came up negative .I woke up this morning Jan 3 and went to the bathroom and notice some blood on my underwear so I put a pad on. The next time I went to the restroom I noticed there was hardly any blood on my pad, but as soon as I would go to the restroom blood would come out . Ive had cramps all day, but no new blood on my pad, until I actually sit on the toilet and go number 1 or two it comes out, and now Its coming out like more chunky but only when I go to the restroom. is this my period, or could I possibly be pregnant? or can it be due to me having to start my birthcontrol pills about a week later then I was suppose to have them ? please help And sorry if its to much information . Please get back to me asap thank you for your time Doctor: Hi, thanks for using HCM.Your period was delayed for only a day and your description of symptoms are suggestive of menstrual bleed. As you are passing clots, could be due to well formed endometrium resulting more bleed. not to worry.Usually urine pregnancy test shows positive results a week after missed period, Blood HCG test is more sensitive in detecting early pregnancy.Use of birth control pill helps to regularize hormonal changes in body, so makes cycles regular. Usually bleeding starts with in a week of taking last active pill.Hope this helps you Regards"
},
{
"id": 46277,
"tgt": "How long is dialysis required in case of diabetes?",
"src": "Patient: Hi, my mother in law,she went to india she has diabets,high blood pressure,,,...one day she went to hospital....dr recomand dyasis..she had 4 i n hospital and now again suggested 4 more ,,can u tell me how long this process goes..she want to come back can she travel.... Doctor: Hello, Dilysis in Diabetes is generally occur when patient developed nephroapyhy. In nephroathy, when developed Chronic kidney disease(CKD). In your description, it's not clear wether patient is suffering from acute Kidney injury(AKI) or CKD.Generally in AKI, patient needs dialysis for short duration of time unless creatinine level get down to normal or stable in medication without any symptoms. But in CKD, patient needs dialysis frequently depends on severity of disease for a longer time or for a life time. In that case, one should be think about transplant option depends on disease and another high risk factor.In your case, if she developed CKD then she will need dialysis frequently for life time and require transplantation if possible. Meanwhile at present case, if doctor said more dialysis she needed then definitely she will go for more dialysis and afterward if general condition becomes good then she can travel with medications. It will be no problem for her.Thank you"
},
{
"id": 159481,
"tgt": "Dark, painful scab on elbow, dark marks, pain on pressing. Cancer symptoms?",
"src": "Patient: Hi, I noticed yesterday a dark (painful) scab on my elbow , I haven t bumped or cut it, it just appeared, it is about the size of a penny piece. That itself isn t the problem, the fact in exactly the same place on the other elblow, it seems the same thing is happening, although it is not as yet a scab, just a dark mark, press it and it hurts.. this is about the size of a small pea. I m 43 and worried about cancer! Doctor: Hi, apparently it looks like some infection or pressure sore. Possibly fungal infection, and cancer is a very very rare possibility. I would suggest you to see skin specialist and clinical examination will be of great value. Take care."
},
{
"id": 66961,
"tgt": "What causes sore lump on palm?",
"src": "Patient: A couple of weeks ago my palm itched like crazy. The next day it was bruised from where I had been scratching and rubbing it to try to relieve the itch. The bruise went away, but now there is a small, pea sized, lump that is sore (still feels like a bruise) and is blue, like the color of the vein below it. Should I be worried? Doctor: Hi, dearI have gone through your question. I can understand your concern. You may have some benign vascular tumor like hemangioma. You should go for examination and if needed go for excision of that lump. Consult your doctor and plan accordingly. Hope I have answered your question, if you have doubt then I will be happy to answer. Thanks for using health care magic. Wish you a very good health."
},
{
"id": 187918,
"tgt": "What is the cause for pain under chin after crowns placed on upper back molars when on ibuprofen?",
"src": "Patient: I have been having pain under chin periodically since spring 2013. Tightness and burning, like inflammation. Had 2crowns placed upper 2 back molars, and started noticing sx's not too long afterwards. Dental x-rays have beenDone and dentist sees no problem. Internist felt maybe was nerve related. Sx's prevalent at night and will Awaken me at times. Ibuprofen does seem to give some relief. Periodically especially in am feel a lump in theback of my throat. After exam md notices no swelling of glands. Any suggestions? Thank you Doctor: hi thanks for your query. the problem you are complaining might be because of high points on the crown getting in contact with the opposing teeth or overzealous crown cutting resulting in exposure of nerve or recession of the gum line resulting in periodontal problem or in general crown is placed over a root canal treated tooth. if there is little amount of pulp tissue left behind this results in pain after crown placement.treatment can be planned by ruling out these conditions.visit endodontist.hope this is helpful."
},
{
"id": 88813,
"tgt": "What causes severe pain on the right side?",
"src": "Patient: Last night I woke up and went to the toilet, I am on my period and when I tried to use the toilet a sharp pain came into my right hand side. The pain was so bad it took my breath away and I become clammy and my vision blurred ever so slightly. I managed to get back to bed and lay until it went away (an hour later) today the pain is now in my lower right side in my back. I first thought it was my appendix but I am not sure as the pain is mostly in my back rather then the front. Doctor: Hi ! Good evening. I am Dr shareef answering your query.From the history and shifting of the pain to your back, without a clinical assessment, the diagnosis goes more in favour of a ureteric colic due to a ureteric stone, and/or a renal pain. If I were your doctor, I would advise you to consume sufficient water along with an anti spasmodic drug, go for a routine/microscopic examination of your urine followed by a culture/sensitivity test if need be and also an ultrasound of whole abdomen to get a clearer picture of the diagnosis. Further treatment would depend on the results of assessment and investigations.I hope this information would help you in discussing with your family physician/treating doctor in further management of your problem. Please do not hesitate to ask in case of any further doubts.Thanks for choosing health care magic to clear doubts on your health problems. I wish you an early recovery. Dr Shareef."
},
{
"id": 175180,
"tgt": "What causes diarrhea in an infant?",
"src": "Patient: hi, my 6 month old has had diarriea for over a week now and is having about 10 dirty nappys a day, they are very smelly and she seems to be in pain while she is doing one, she is on sma stay down formula milk, has she has reflux she has been on this milk about three months now, what could this be? Doctor: Let me assure you that it is normal for a child of this age to pass stool 10 times a day or even after each feed. It is not a cause for concern. Most formula milks are cow milk based and hence difficult to digest. Undigested food matter can be degraded by intestinal bacteria to give foul smelling stools. In case diarrhea actually occurs, it is generally due to a virus called rotavirus and is a self limiting condition. The reflux could be due to inadequate burping. You need to hold the bay upright for about 20 mins after each feed to prevent reflux from occurring."
},
{
"id": 110532,
"tgt": "Experiencing pain at the left back side up to the rib cage",
"src": "Patient: I have a pain in my left side at the back - when lying in bed it feels like a brick that I have to lift to move in the bed - it is a dull pain not sharp. When I get up my left side up to my rib cage and slightly above feels swollen and stiff but the pain subsides Doctor: Hello, I have studied your case. Due to compression of this nerve root there is pain in your ribThere is another possibility of costochondritisI will advise you to do MRI thoracic spine, HRCT [CHEST] and EMG- NCV [nerve conduction study]For these symptoms analgesic and neurotropic medication can be started.Till time, avoid lifting weights, Sit with support to back. You can consult physiotherapist for help.Physiotherapy like ultrasound and interferential therapy will give quick relief.I will advise to check your vit B12 and vit D3 level.Hope this answers your query. If you have additional questions or follow up queries then please do not hesitate in writing to us. I will be happy to answer your queries. Wishing you good health.Take care"
},
{
"id": 75148,
"tgt": "What causes difficulty in breathing after having a hiatal hernia surgery?",
"src": "Patient: I had surgery to correct a hiatal hernia and acid reflux a year ago this past Nov. and ever since, I have difficulty breathing just about every other day. Prior to surgery, it was more frequent. I sleep on a wedge pillow, plus another standard pillow to prevent acid coming up, and I eat by 5:30 to give adequate time for food to digest (this has helped tremendously to the point where I don t need medication regularly, but wonder why it happens every other day and is there anything else I can do? And is it happening because acid comes up to my airway and creates swelling? PS--my pulmonologist prescribed inhalers, but it brought no relief whatsoever. Doctor: Hi, Thanks for your question. I can understand your pain and discomfort. your problem should be evaluated by your doctor to make sure that its not a cpmplication from the surgery you have undergone. Ask your doctor as it could be a gas bloat.A common side effect of the surgery is gas bloat. Air becomes trapped in the stomach, causing pain and discomfort and difficulty in breathing. Chew your food well. Eat small meals. Avoid carbonation (soda) and drinking from a straw. All of these will increase the air in your stomach.I hope I was able to address your query. If you have any further questions, please do not hesitate to write to me. Wishing you all the best. Thanks"
},
{
"id": 161380,
"tgt": "Suggest treatment for child suffering from fever and chills",
"src": "Patient: My 1 year old has had a fever out of the blue since last night, when we gave him Motrin he spit it right back up, this morning the fever was over 103 so again we gave him motrin, that was 5 hrs ago and since then it has came back and now he has the chills, should I ride it out with meds or take him to the ER? Doctor: Hello, If a similar patient comes to my old I would suggest admitting the kid aa his fever is above 100. it is always suggested to admit the child and start on iv medications to bring it down. Once it drops down you can continue with tablets. so kindly tale him to ER. Hope I have answered your query. Let me know if I can assist you further. Take care Regards, Dr Jilu Joseph, General & Family Physician"
},
{
"id": 47798,
"tgt": "Suggest treatment for angiomyolipoma kidney",
"src": "Patient: I was diagnosed with angiomyolipoma on the right kidney. I was informed it s like a wig covering half of my kidney which exceeded 5cm. I ve had 2 CT and multiple ultrasounds in the last 2 years. As there was no pain nor effects on my daily life, I was told to continue monitoring the situation. Lately, I feel pain on the left side of my stomach but I m not sure it s related to kidney angiomyolipoma as it was not sharp pain. The pain comes and goes and it happens more often than before. I have been exercising (running) and I wonder if it s muscle pain or something else. I was told to go immediately to hospital if sharp pain due to hemorrhage. I m very worried now and don t know what to do. Doctor: Hi,Thanks for writing in.Angiomyolipoma is a benign condition which is a tumor containing vascular, smooth muscle and fat components. It is a lump in the kidney and might grow to a large size and cause pain and discomfort.Usually symptomatic angiomyolipomas require surgical treatment if they are causing symptoms. It is exceeding 5 cm and therefore a surgery might be recommended as an acceptable form of treatment. Since you are also participating in physical activities therefore a relation of the pain with the pinching of nerves and muscle irritation cannot be ruled out. The image of the tumor is to be carefully studied and this will tell if the pain is indeed from the tumor you are having or any complication like hemorrhage. You might get ultrasound scans done every 3 months. Please do not worry."
},
{
"id": 134939,
"tgt": "Suggest remedy for tingling sensation and muscle spasms in leg",
"src": "Patient: I have recently been referred to see a rheumatologist however there are no doctors in my area that accept my insurance. I am in a huge amount of pain, with tingling feet, muscle spasms and weakness. I have been to the ER before for these symptoms and I only get temporary relief. I really don t want to go again but I can t take this anymore. I m in pain ALL day EVERY day. Please provide me suggestions on what I can do right now!! Doctor: hii suggest u get an detailed blood investigation done and come to a diagnosis firstthen v can start you on advanced drugsthanks take care"
},
{
"id": 70761,
"tgt": "Is PTB cured when X-ray report shows clearing of the previously noted minimal koch's infiltrates in left upper lobe?",
"src": "Patient: Hi, i undergone medication for 6 months because of Minimal PTB left..and after 6 months i had my x-ray and it says that clearing of the previously noted minimal koch s infiltrated in left upper lobe . Does it mean no scars seen in my chest?am i totally free of TB? Doctor: Hello Welcome to the HealthcareMagic You are in category 1 of tuberculosis. So after completion of AKT-4 kit now you have radiological clearance of lesion. So it suggest improvement and now you can be free from TB. Repeat sputum AFB examination after completion of therapy Hope your concern is solved Take care Regards Dr. Parth Goswami, General & Family Physician"
},
{
"id": 69583,
"tgt": "What medications are suggested for lumps in breast post pregnancy?",
"src": "Patient: My Wife delivered baby on 20th Dec 2011( 7 weeks back ). 15 days Back ,She started getting lumps in the right breast along with Fever and Body Ache. The doctor advised to take Anti Biotic along with paracetamol. The lump continues to be there and pain is still there. What to do ? Doctor: Hi,It seems that you might be having developing breast abscess giving this problem.consult your gynaec and get examined.She might require more strong antibiotic and anti-inflammatory medicines to control pus formation and healing.Remove breast milk with breast pump.Ok and take care."
},
{
"id": 86985,
"tgt": "What causes nausea and headache after an abdominal injury?",
"src": "Patient: I was at the gym yesterday and attempted to do a plank exercise when it felt as though I was stabbed in my abdomen. I ended my training session and went home.The pain was there consistently all night, it hurt to move. Today it has continued as mostly an ache all day when I m not moving, but if I move, it s a sharp stabbing pain. I just ate lunch, but now I feel nauseous, have a headache and it hurts really bad to sneeze. Doctor: Hi.Thanks for your query.Plank exercises involve the abdominal muscles. It looks you has a strenuous session and you have either ruptured some muscles or a blood vessel causing a hematoma. I would advise the following in such a patient:First of all, strict bed rest unless you cant to visit a Doctor.Clinical examination by a Surgeon.High resolution of the abdomen and the abdominal wall to see for the diagnosis. Tell the Radiologist where to see for the findings.Take an anti-inflammatory like Ibuprofen as per the body weight at least 8 hourly.Further treatment as per the diagnosis. Do not try anything at home."
},
{
"id": 168654,
"tgt": "What should be the normal blood sugar level of an infant after eating?",
"src": "Patient: What should blood sugars be in a 3 month old baby an hour after eating? I have diabetes and was on insulin while pregnant but when she was born her blood sugars ranged from 50-80. I just checked it as was curious and it was 132. She just ate 6 ounces of formula an hour ago. Doctor: Hi...this is a normal finding. If the baby is otherwise active do not worry. there is no specific blood sugar levels for any baby at this age as the feeds will be frequent. Regards - Dr. Sumanth"
},
{
"id": 191501,
"tgt": "Is Meloxicam advisable for a heart and diabetic patient?",
"src": "Patient: I am a type 11 Diabetic and have just had a stent implanted in my artery that was 90% blocked. I am experiencing a muscular leg pain from my buttocks through my thigh and down to my calf and shin. My question is haw harmful will and should I try Meloxicam? Doctor: Hi,I can understand your concern with the medicine meloxicam.Meloxicam is just another NSAID and it decreases your pain and inflammation.So far your question is concerned,I will say that it is not contraindicated in diabetes and even in IHD but it should not be taken regularly and if you wish to take it for a longer period of time,you should get your kidney function checked every three months as with any other NSAID.My advice - As you are a diabetic,please check your kidney function every 3 months even if a doctor has prescribed it for a longer period.I hope,My answer is useful.Please do not hesitate to ask a counter question if required.Thanks,Dr.Prabhas Verma"
},
{
"id": 133901,
"tgt": "What relieves the swelling and soreness of the foot?",
"src": "Patient: Hello, I tripped over my daughters toy yrsterday. When I fell my ankle popped and twisted and fell hard on my hip. I iced and put my ankle up. My foot is swollen and sore and my toes are numb feeling. My hip hurts in any position but laying down. Stairs and sitting down and standing up is when the pain is the worst.How long before the pain gets better? Thank you, Crystal Doctor: hi,thank you for providing the brief history of you.A thorough neuromuscular assessment is advised.As you tripped of the toy and then twisted the foot , so the swelling was there and subsided as well. You tried doing ice but it did not work much. Now since the mechanism of injury you did not mention indetail so i can suspect you feel on the floor, for which a thorough neuromuscular assessment is guided.Also, when the hip is hurting and while going up and down the stairs, then you need to consult an ortho who may ask for an x-ray of the hip, ankle and lumbar spine. May be an MRI of lumbar spine as well. Some times the lumbar spine is ignored and the treatment is provided symptomatic for muscular pain.Once the reports are out, simple medication and physical therapy will be advised after a thorough musculoskeletal assessment. 2-3 weeks of time you will be back to normal life.RegardsJay Indravadan Patel"
},
{
"id": 28927,
"tgt": "How can sialadenitis be treated?",
"src": "Patient: My doctor diagnosed me with Sialadenitis In January and now it\u2019s the middle of March and I\u2019ve been drinking a lot of water and massaging the glands but it\u2019s not getting better and I really do not want to pay another medical bill, hinch I don\u2019t have the money for Another medical bill, what should I do? Doctor: Hello,As per your complaint, sialadenitis is an infection of the salivary gland and commonly occurs due to obstruction of the duct of the gland with salivary stone. It can also be due to reducing saliva flow. Hydration and massage is a good choice but for reducing infection. You need to consult an oral physician or an oral surgeon and get evaluated. You need to take a course of antibiotics and along with it. To improve saliva flow chew sugar-free chewing gums and suck lemon drops. Drink plenty of water.Hope I have answered your query. Let me know if I can assist you further.Regards,Dr. Honey Arora"
},
{
"id": 173224,
"tgt": "Suggest treatment for cough, fever and pink color in mouth of a 2 year old",
"src": "Patient: my child(2 1/2 years) having severe and continues cough(full day) and his mouth and tongue looking like pink color, fever also reached 101 & 102. He faced the problem since 10 days. He reduced almost 2.5 kgs in this illness. what is the symptoms is this, what is the permanent remedies for this. Doctor: With regard to your concern I would like to tell that the normal color of the human tongue is pink to red. In some people the tongue tends to be more red than in others but this is not necessarily a discoloration of the tongue.Glossitis is inflammation of the tongue that may arise for any number of reasons. The process of inflammation increases blood flow to the tongue and leads to redness, pain and swelling. The more common causes of glossitis are acute mechanical or chemical injury or infections.Vitamin deficiencies as is seen with vitamin B12, niacin or folic acid may cause a strawberry red tongue.Scarlet fever (scarlatina) is a condition that may sometimes develop in a person with streptococcal pharyngitis (strep throat). It presents with a red rash that covers most of the body along with a strawberry red tongue. Scarlet fever arises as a reaction to the toxin from the streptococci (bacteria) that causes the infection. For these cough have you not given any medicine or showed any doctor ? If you are giving medicine the colour may be due to that also.I would check his blood for CBC and related tests,if you have not done so far. I have outlined the diseases I would think of limited resources I have got. Please review with your doctor and get back please. Regards"
},
{
"id": 78962,
"tgt": "Am I eligible for lung transplantation when having EF rate of 37%?",
"src": "Patient: I suppose to get lung transplantation due to COPD, asthma and emphysema. My pulmonologist - surgeon doctor referred me for MUGA scan test. My Ejection Fraction (EF) rate is 37%. Due to this number I eligible for lung transplantation or not? Thanks.My e-mail: YYYY@YYYY Doctor: Evaluation for lung transplant consisits of package of investigations and based on these results as a whole dicission is taken by the operating surgeon to take a call to go ahead with a surgery."
},
{
"id": 118134,
"tgt": "Am i in danger with the pulse of 108?",
"src": "Patient: my doctor raised my blood pressure medicine to a higher dosage, it was too high iam not exercising and my pulse is 108 has one double beat and one skiped when i check my pulse for the whole minute, am i in danger of a heart attack or something dangerous?e mail is YYYY@YYYY Doctor: It sounds like you may be having a heart arrhythmia, which is abnormal heartbeats. Some types of irregular beats can be dangerous. 108 is also too fast- not dangerous by itself, but could indicate an underlying problem that needs investigation. It is possible the increased blood pressure medicine is causing the problem. You need to talk your doctor right away and have a heart tracing (EKG) done. If you are having any kind of symptoms with it, such as chest pain, shortness of breath, dizziness, nausea, etc, that is an emergency and you need to call for emergency services right away."
},
{
"id": 75809,
"tgt": "How can swollen legs, lung scars and breathlessness be treated?",
"src": "Patient: hi,i have been having shortness of breath for some time now,esp when i start moving,my legs are swollen.It feels like my heart,prev tricusp valve repair.I have had ultra sound,told everything is normal.Have seen chest consultant,small scars in lung,housebound for last week.I am female 56 Doctor: Thanks for your question on Healthcare Magic. I can understand your concern. By your history and description, possibility of chronic bronchitis is more. Chronic bronchitis can cause lung scarring, breathlessness, edema feet etc Chronic bronchitis can damage right side of heart and it will cause 'Cor Pulmonale'. Cor pulmonale causes fluid retention in the body. So you should first consult pulmonologist and get done clinical examination of respiratory system, PFT (pulmonary function test) and 2d echo (for Cor pulmonale). You will need diuretics, inhaled bronchodilator and inhaled corticosteroids (ICS) for treatment. Don't worry, you will be alright with all these. Quit smoking if you are a smoker. Hope I have solved your query. I will be happy to help you further. Wish you good health. Thanks."
},
{
"id": 31484,
"tgt": "What causes tightness in throat, nasal blockage and heaviness in ear/head?",
"src": "Patient: hi! I m jai laungani from Spain, From one year i have much problem in my throat,nose and ear,dostor says dat its fungus ,I m taking treatement but no recovery,,My troat left side is very tight,left side nose is always blocked and left ear inside is too heavy and my head also too heavy...According to u what problem it should be..?wat i should u..plz tell me.. Doctor: Hi Dear,Welcome to HCM.Understanding your concern. As per your query you are having symptoms of tightness in throat, nasal blockage and heaviness in ear/head which could be due to sore throat , which can be caused by bacterial or viral infection. It could be due to Eustachian tube dysfunction which is leading to creation of negative pressure inside ears. Need not to worry. I would suggest you to consult ENT specialist for proper examination . Doctor may order test like viral culture test , throat swab test and blood test to confirm the diagnosis and rule out causes like acid reflux. You should go for otoscopic examination of ear as well and start treatment after proper prescription. Doctor may prescribe antibacterial ( amoxicillin) or anti viral ( acyclovir ) depending upon the diagnosis along with decongestants . For now continue doing gargles and drink one spoon of honey mix with glass of lukewarm water .Hope your concern has been resolved.Get Well Soon.Best Wishes,Dr. Harry Maheshwari"
},
{
"id": 222308,
"tgt": "What causes delay in menstruation other than pregnancy?",
"src": "Patient: I am 21 y.o and took oral contraceptive pills on the 14th of December. i was suppose to have my period last dec15. i am 2 weeks and 3 days delayed for my period. I tried 2 pregnancy test first thing in the morning and it came out negative. Is this accurate? Doctor: HiDr. Purushottam welcomes you to HCM virtual clinic.I have gone through your query. I think I have understood your concern, I will try to suggest you the best possible treatment options.1] Usually if oral contraceptive pills are taken regularly then periods start within 7 days of pill free period.2]As your urine tests are negative even with 2 weeks delay you are mostly not pregnant.3] As of now you have to keep watch and wait till onset of periods. The delay in periods can be due to hormonal causes, hormones that are produced by ovaries.4] I will suggest to get USG done once you are done with periods, to look for ovarian status.I hope my answer helps you.Thanks.Wish you good health."
},
{
"id": 161411,
"tgt": "Suggest remedies for a constant fever in a child",
"src": "Patient: My son has had a temp since yesterday afternoon. The lowest it has been is 101.0 and the highest is 102.8 without adding a degree. The mortin we have been giving him only brings it down a little. We ve tried a bath and that didnt help lower the fever either. Is this somthing we should bring him in for? Doctor: Hello, Fever of few days without any localizing signs could as well a viral illness. Usually rather than fever, what is more important is the activity of the child, in between 2 fever episodes on the same day. If the kid is active and playing around when there is no fever, it is probably viral illness and it doesn't require antibiotics at all. Once viral fever comes it will there for 4-7 days. So do not worry about duration if the kid is active. Paracetamol can be given in the dose of 15mg/kg/dose (maximum ceiling dose of 500mg) every 4-6th hourly that too only if fever is more than 100F. I suggest not using combination medicines for fever, especially with Paracetamol. Hope I have answered your query. Let me know if I can assist you further. Take care Regards, Dr Sumanth Amperayani, Pediatrician, Pulmonology"
},
{
"id": 181374,
"tgt": "What causes tooth pain upon applying slight pressure?",
"src": "Patient: Hi. I don't know what to do. My tooth was once a cavity. I got it filled. Everything was okay. The filling fell out a while ago and there was no pain. But lately it's been hurting. There's this awful taste coming from it. It doesn't hurt when I eat on it but it hurts when I apply pressure to it. Does that mean it needs to be refilled or pulled Doctor: Hi..Welcome to HEALTHCARE MAGIC..I have gone through your query and can understand your concerns..As per your complain pain in tooth that has lost filling along with bad taste from it can be due to deep infection in tooth and aweful taste is due to pus discharge caused due to infection..I would suggest you to consult an Oral Physician and get evaluated and a thorough clinical evaluation and investigations like x ray can help in diagnosis and treatment can be done accordingly..If tooth can be saved then Root Canal treatment can be advised..In case if tooth cannot be saved then extraction can be done..As of now take anti-inflammatory painkillers like Ibuprofen and do antiseptic mouthwash gargles..Hope this information helps..Regards."
},
{
"id": 89250,
"tgt": "What causes bleeding in the umbalicus?",
"src": "Patient: Hi, I'm 28 year old women. Bleeding is occuring through my umbalicus. Sonography results says: An umbilical fistulas tract noted from skin through subcutaneous tissues upto the peritoneal cavity. Peri-fistulous fluid collection noted. Doctor advised me to go for surgery. Please tell me if there is any remedy on this other than operation? Doctor: the fistula has to be removed only surgically. there no other specific treatment to stop bleeding other than removing the underlying cause."
},
{
"id": 83531,
"tgt": "What are the side effect of eldoper dosage?",
"src": "Patient: For diarrhea I used eldoper ( only one capsule) .after that for twodays motion was blocked. Now iihave irritation while urinating and find a little bit of mucous in. My drool. Is it the side effect of that capsule . If not what is the remedy for it. What diet I should follow Doctor: Hi,Constipation can occur as a side effect of Eldoper. Avoid taking the drug. Lactulose syrup 15 ml can be taken at night. It takes 3 to 4 days for the drug to soften the stools. Maintaining healthy fibre diet and adequate hydration in the following days can avoid constipation.Hope I have answered your question. Let me know if I can assist you further. Regards, Dr. Saranya Ramadoss, General & Family Physician"
},
{
"id": 219579,
"tgt": "What are the symptoms of potential pregnancy?",
"src": "Patient: I had my period 3 days late and it was different at first I didnt even know I was on I didnt bleed at first just a blood clot came out. Then I had it like usual. Then the day after it ended for two days I had VERY dark brown discharge. My period looked orangeish. It has been a week since my period and I am still very emotional last night I was watching a comedy and bursted out in to tears for no reason. I have been crazy emotional. I have been making problems out of nothing. Idk whats wronge with me. A few weeks ago I got a really bad migrane and threw up. I was terribly nauseated. I know this sounds stupid but I tried that homemade bleach pregnancy test for fun and it frothed and bubbled. It bubbled at first then it frothed. My question is could I be pregnant? Doctor: Hi.A period can also cause hormonal fluctuations which can all result in such symptoms, although it is more likely during pregnancy. But if at all you experienced a period that lasted more then 3 days, then pregnancy has lesser chances, yet not ruled out.I recommend a home pregnancy test using an early morning urine sample, but it does seem unlikely to me so far, but you never know ma'am.Best wishes."
},
{
"id": 110740,
"tgt": "Suggest remedy for back pain with spine bruises",
"src": "Patient: i have severe back pain .I have blood that has came to the skin on my lower spine permanent bruises on spine and tail bone is deformed into spine at L5S1 early degenerative disk disease at L5S1 and a bulging disk at L5S1. I have osteo perosis rhumetoid artherits as well as empyhsyma asthma and copd. I have 3 fingers that are deformed and don t straighten out all the way on both hands last 3 fingers both hands. I am in severe pain what do you suggest I do ? Doctor: Hello,I have gone through your question in detail and I can understand what you are going through.Its apparant from your description that you have got a degenerative disease of spine. RA has aggravated this dgeneration .If the condition is treatable, l.Otherwise the option that we have are to mitigate the pain. You can start pregaba, a molecule which will reduce you pain.Even amitryptiline is a good medication which reduces such pain to a large extent.CONSULT YOUR DOCTORDo consult a physiotherapist who would like to advise you some back exercises and do , do them regularly.Hope I am able to answer your concerns.If you have any further query, I would be glad to help you.If not, you may close the discussion and if possible you may rate the answer for me, so that I get a good feedback.Wish you good health,"
},
{
"id": 176960,
"tgt": "Suggest treatment for stomach pain and vomiting",
"src": "Patient: Hi my daughter is 9 has just recently had a gastro bug she was sick about 50hrs ago she had one day previous of upset tummy and the same day as vomiting she had an upset tummy. She has been struggling to eat and drink properly since and has been complaining of feeling bloated in her chest area and keeps burping. Today she went for a poo and her stools were a very pale yellow/grey and of normal consistency. About 6 hrs before she went to the toilet she had had ibuprofen for a sprained foot and had eaten it with a small sausage roll. She has drank a bit but not alot today. Should i be concerned? She has a normal temp of 35.9 Doctor: Hi....It seems your kid is having viral diarrhoea. Once it starts it will take 5-7 days to completely get better. Unless the kid's having low urine output or very dull or excessively sleepy or blood in motion or green bilious vomiting...you need not worry. I suggest you use zinc supplements (Z&D drops 1ml once daily for 14 days) & ORS (Each small packet mixed in 200ml of potable water and keep giving sip by sip) as hydration is very important and crucial part of treatment. If there is vomiting you can use Syrup Ondansetron (as prescribed by your paediatrician).Regards - Dr. Sumanth"
},
{
"id": 33040,
"tgt": "Suggest treatment for mushroom poisoning",
"src": "Patient: I m wondering about some raw mushrooms I ate. I ve heard you shouldnt eat a lot of a raw mushrooms. I at a whole cartoon of them! What are some risks. Also I have a fungal infection I m going to see Dr about. Do the mushrooms have any bearing on this? Thx Doctor: In the absence of a definitive identification of the mushroom, all ingestion should be considered serious and treated. Liver and kidneys are the organs affected. Treatment is largely supportive. Drink plenty of water for gut irrigation. You have to use activated charcoal, which plays an important role in limiting absorption of most toxins."
},
{
"id": 76987,
"tgt": "What causes constant chest pain with back pain and stiff neck?",
"src": "Patient: im having chest pain in my left side and then it spreads across my chest. then my back begins to hurt and my neck feels stiff, feels like im going to vommit ,also being choke. but ive been to the doctors and hospital and they did a ekg and they said every thing is fine, so the did blood test urine tes and ct scan and every thing looked normal. no gas built up, NOTHING. but this pain is constint. whats wrong. Doctor: Thanks for your question on Healthcare Magic. I can understand your concern. No need to worry for major heart and lung related diseases because your all reports are normal. Possibility of stress and anxiety related symptoms is more. So better to consult psychiatrist and get done counselling sessions. Try to identify stressor in your life and start working on it's solution. You may need anxiolytic drugs too. Don't worry, with treatment and counselling, you will be alright. Apply warm water pad on affected areas. Avoid movements causing pain. Avoid stress and tension, be relax and calm. Hope I have solved your query. I will be happy to help you further. Wish you good health . Thanks."
},
{
"id": 82409,
"tgt": "Suggest remedy for chest pain",
"src": "Patient: I swalled an alcaselser this morning wrong and now everytime I swallow I get a good amount of chest pain, mostly on my right side of my chest. I m not sure if it s stuck, went down the wrong tube, or if just caused very bad irradiation. It s painful. What should I do? Will it just go away? Doctor: Thanks for your question on HCM.As a rule in chest pAin, we must rule out cardiac cause first. So I advice you to get done ECG first to rule out heart disease.If this is normal than I think it is most probably painkillers associated gastritis only.So try to follow these.1. Avoid hot and spicy food.2. Avoid large meals, take frequent small meals.3. Avoid stress and anxiety. 4. Take proton pump inhibitors. 5. Avoid painkiller use as a routine.6. Go for walk after meals."
},
{
"id": 14950,
"tgt": "Why did my son have red circular rash on back and leg?",
"src": "Patient: My son was playing outside now he has a rash on forhead i gave him a bath and noticed rash on his legs red circular rash kinda white in middle red splotchy spots on his back that spread down his back. Plus he developed a cough any ideas????? Please let me know asap thanks Doctor: Hi,I can understand your concern for the red circular rash on back and leg in your son. There can be multiple possibilities of the problem and without pictures it is practically not possible to guide you for exact cause or diagnosis.I can make a possibility of allergic dermatitis due to sweating or clothes or friction.The only precaution you must follow is not to touch, pinch pop or squeeze any of them other wise it can get infected and can cause complications.You can start application hydrocortisone cream and Benadryl to help him out.Take care."
},
{
"id": 10125,
"tgt": "Suggest remedy for hairfall",
"src": "Patient: hello doctor,I have long hair , since 2009 every year in the month of june to August I face hair loss from roots.I am 26 years old, married and take a good diet and have no stress. More hair fall when I take a Head wash .also my scalp becomes itchy in these months. What do I do? Doctor: Hi. If there is itchy scalp with hairloss then it is most probably due to dandruff as it is in a specific time span. So my suggestion is to apply Ketoconazole and Coaltar shampoo and apply warm coconut oil over the scalp. Wash your hairs every alternate day and drink plenty of water. Tave a Vitamin E capsule daily. Eat plenty of green leafy vegetables in your diet and eat fresh fruits. Avoid using any harsh chemical cosmetics containing hair oil or shampoo. Use herbal products aa much as possible. Hope I have answered your query. Please let me know if I can assist you further. Regards. Dr. Honey Arora, Dentist"
},
{
"id": 196737,
"tgt": "What cause sharp pain in my right nipple?",
"src": "Patient: Hi I'm 18 year old male and within the past 4-5 days I have been experiencing a sharp pain in my right nipple. It's almost feels pulsing and only lasts about 2-3 minutes and happens about 5 times a day. It feels more sensitive and tender than my left. Doctor: Hi welcome to the health care magic Here nipple and breast physical examination needs to be done as per history along with palpation... There might be mastitis like breast inflammation or infection or superficial bacterial skin infection... If no specific cause found than it could be muscular pain For inflammation relief ibuprofen might prescribed for five days If during examination any mass palpable than FNAC like investigation done for that... Take care Consult physician for examination"
},
{
"id": 44061,
"tgt": "Weakness, feverish, cramps, vomiting. What can help me? Why is this happening?",
"src": "Patient: The night before I started my period I began feeling weak and feverish, chills everything. That same night at 3 am I started having cramps so I went to the toilet, I didn t have to poop but the cramps got so bad I was screaming. Then I threw up everywhere and the cramps continued to get worse. I could barely breathe, I m not pregnant nor have I ever been, but it felt like I was giving birth. The worst of it subsided after about half an hour, but the painful cramps continued, I still have them. The strange thing is the weakness. I feel weak everywhere, but especially my fingertips, legs and back. I m 18 years old. I also believe I may be infertile because I ve been having sex nearly every night for the past year and a half ( with the same man, of course ). I think an ultrasound would be a good idea.. Doctor: Do hormonal tsts, HIV,HbsAg,VDRL,sugar level Blood tets along with uterine sonography.Can take Ayurvedic treatment.Wish you all the best."
},
{
"id": 113979,
"tgt": "Why am I experiencing lower back pain after 6 months of my delivery ?",
"src": "Patient: i have back pain now,6 months after delivery.pain is not at the lower back but towards middle where the spinal anesthasia was taken.can anybody give the reason.pain is more when i lift my child. Doctor: Hello. . Spinal aneasthesia is given in lower back only and not in the middle. The pain you are having may be due to a spinal cord problem. Get an X-ray spine lateral view done and consult an orthopaedician for your problem. Dr. Rakhi Tayal"
},
{
"id": 121705,
"tgt": "What causes pain in the rib when lying down?",
"src": "Patient: my right front rib hurts when i lay down too sleep, now there is a spot in the middle of my spine under my bra line that hurts. My doctor took xrays of my rib and didn t find anything, and gave me muscle relaxers. They don t help. What could be the problem. I have been off work for 2 months so i know its not job related. Doctor: Hello, The symptoms seem to be related to a muscle strain. I suggest continuing using muscle relaxant. I suggest using Baclofen three times a day. I also recommend using warm compresses for local applications. Hope I have answered your query. Let me know if I can assist you further. Take care Regards, Dr. Dorina Gurabardhi, General & Family Physician"
},
{
"id": 112790,
"tgt": "Severe back pain, abdominal cramping, altered menstruation. Are these related? Need to get checked?",
"src": "Patient: Hello. I have been having really bad lower back pain for about 4 days now and abdominal cramping on my right side for 2. The last two months of periods have been off a bit being a whole week early and been mostly very light with one heavy day. This past one has been over almost a week and I am still seeing light brown discharge. I don't know if my back pain and the abdominal pain are connected cause we had a serious cleaning day at my house and I may have just used the muscles too much and the abdominal could be ovulation cramps? I just was wondering should I wait to see if the symptoms go away themselves or should I really see a doctor about them? And what does it sound like is wrong? Thank you. :-) Doctor: As it sounds to be a typical backach after a hard day so u should take some pain killer and antispasmotic and do rest for atleast for 3 days ,and if the pain doesn't goes then see ur doctor"
},
{
"id": 187409,
"tgt": "Experiencing excruciating pain in my mouth after teeth extraction",
"src": "Patient: i had 3 teeth out on wednesday morning , ever since the numbness wore ff i have had excruciating pain on my right side of mouth, i had a bottom molar and top molar out but the bottom molar on the oppisite side is healing well. but my right side is in throbbing pain im on anti biotics and have been taking inbuprofen, so far i have had 20 since thursday, i dont know what to do? Doctor: HiWelcome to HCMFirst you should stop having ibuprofen, 20 tablets arr too much. If you have extraction and pain is extreme then there muat have formation of dry socket. So I will advice to get an iopa xray done and visit your dentist for local treatment so that he can put local application on it to relief your pain.Thank you"
},
{
"id": 124027,
"tgt": "How soon can you drive after total knee replacement?",
"src": "Patient: Had my 2nd knee replacement surgery last May 21st. Still undergoing physical therapy until mid- August. At this point I can walk without a cane but for only a short errand daily. How will I know when I can safely drive my car again? I am 74 years old. Doctor: Hello, As you are already into the physiotherapy sessions and have a good improvement post surgery i think it wont take much time for you to get back to ADL. Usually, the muscles take some time longer to get the strength and timely contraction in the aged individuals due to the ageing process which the muscles are undergoing. Trying out hard the physiotherapy and continuing it for some extra time than predicted should help you walk with freedom and confidence. Regaining the mileage of the muscle strength in ageing is little slow but possible still. Follow the exercises advised by the physiotherapist and wait for a few more days, you should be fine soon. Hope I have answered your query. Let me know if I can assist you further. Take care Regards, Jay Indravadan Patel, Physical Therapist or Physiotherapist"
},
{
"id": 193283,
"tgt": "What is the treatment for abrasion in the penis?",
"src": "Patient: 5 days back (that is on Thursday) I made out with this girl and we did not do intercourse but just kissing and sucking. What I am worrying about is that on that day (i.e. on Thursday), I had an abrasion (it was a minor one where a little skin waqs peeled off) on my penis (which was 3 days old (i.e. it happened on Monday night) but was rescratched (on wednesday night). Now on thursday (5 days back) when we were making out she for 30 secs took my penis' head (unprotected) in to her mouth. Am I at risk of HIV? Doctor: Hello, There is a very low risk of HIV being transmitted via oral sex, however risk is increased if anyone has cuts in oral cavity, STI of throat, recent dental work. If this is not the case you are at a very low risk of interacting with HIV. You can apply any moisturizer or petroleum jelly over abrasion of penis as needed. If you see a pus coming out of that abrasion consult your physician immediately. Hope I have answered your query. Let me know if I can assist you further. Regards, Dr. Sameen Bin Naeem, General & Family Physician"
},
{
"id": 21017,
"tgt": "How can heart blockages be treated in the elderly?",
"src": "Patient: my aunty is suffering from heart problem. yesterday we got her ngeogrphy report where it is being seen that she has 2 blockage out of 3 and the 3rd one is already 80% blocked. she is 70 years old and having high blood pressure. please suggest what to do now.. Doctor: Hello, I had gone through your question and understand your concerns.With such type of presentation in my practice, I usually advise doing a procedure called CABG (Coronary artery bypass grafting) as it involves 3 arteries. I advise you discuss this with her cardiologist. He/she will know better what actions to take as he/she knows her full history. Best wishes"
},
{
"id": 168916,
"tgt": "Is moxicip medication safe in infants?",
"src": "Patient: Hi Doctor, My daughter - 2 months old, for past 2 days is getting some yellow thing from her eye..I have been washing her eye with cold water whenever that yellow thing comes out..I have been advised medicine Moxicip..just wanted to check whether its right and safe for 2 months old child.. Doctor: Hi, Can understand your concerns.Moxicip eye drops contain moxifloxacin and it is safe to use in children. Since, your child has yellowish discharge from Eyes Moxicip eye drops should be given accordingly. I hope this will help you. Wishing your child good health. Take care. Regards dr Deepak Patel MD Pediatrics"
},
{
"id": 225715,
"tgt": "Began the pill Microgynon 30, had unprotected sex after it. AM I protected or not ?",
"src": "Patient: Hi, began the pill Microgynon 30 on Tuesday which was the 5th day of my period. My doctor said I would be protected imediatly. So the day after I had unprotected sex, which I know was stupid of me to do, but by then i had taken 2 pills and was under the impression that i was protected. I was on top and i told my boyfriend to pull out but he didn t do it fast enough. I went to the toilet straight after and most of it came out and then there was only a little bit remaining the morning after. The BBC website and NHS website also say that if you take it on the 5th day you are protected unless you have a short ovulation cycle but my periods are irregular usually so i didn t know if i was in that category and many sites seem to be skeptical about it working imediatly on the 5th day . I recently read that if you take 4 Microgynon tablets within 120 hours of having sex and then 4, 12 hours later that is can act as emergency contraception . Being paranoid and panicking about the possibility of getting pregnant I took 4 tablets and i don t know whether to take another 4 in the morning because that would mean only having 7 pills left in my pill pack. And if i do that should i take the 8 pills out of the second pack and then bleed or should i just bleed when i ve finished my first pack. Everyone just told me to take my pills as normal but i couldn t stand the waiting. Please help!! Doctor: Welcome to HCM.First of all you had unprotected sex on 5th day and it is not the fertile period so there are no any chances of pregnancy.The day you started pill is the right day and no need to worry,you have to continue it till complete pack.It will definitely protect you to become conceive.Now wait for next cycle without any worry and if you miss the cycle than go for urine pregnancy test and consult your gynecologist for further guidance."
},
{
"id": 59142,
"tgt": "Yellow urine and eyes due to jaundice. Diagnosed with enlarged liver. High fever. What to do?",
"src": "Patient: hi my Husband is suffering with jaundice , his eyes yellow and urine color is also yellow, we are giving liv 52 and sweet juice, food without oil. he is not taking food but juice glucose , earlier dignosed with enlarge liver but that time jaundice not visible, he suffers fever upto 103 at night , what we should do as we alredy confused about his condition, he is active but not taking food himself Doctor: Hi and welcome to HCM. You should go to hospital. This can be acute cholangitis or some other intraabdominal infection The right cause of jaundice must be found usually this is biliary obstruction or hepatiits but you cant treat it without exact cause. He should do ultrasound and liver enzymes at least . ANd follow hepatoprotective diet,reduce fever too, WIsh you good health."
},
{
"id": 179875,
"tgt": "Suggest good anti-emetic for a child",
"src": "Patient: Hi... My baby of 7 months has 4-5 vomitings since morning, no fever, rash, or dairrhea. even wake-up from sleeping and vomiting. Can you please suggest some good anti-emetic for her. I am MD Pharmacologist by profession and seeks your kind opinion for choosing good anti-emetic. Thanks & Regards, Faheem Doctor: thank you for using healthcare magic.best to use metoclopromide or domperidone syrup for vomiting.sincerely,Mark Rosario MDGeneral pediatric/pediatric pulmonology"
},
{
"id": 27756,
"tgt": "Suggest treatment for hypertension with elevated TSH levels",
"src": "Patient: My mother is suffering from hypertension and her TSH levels are also high. Last week after performing some tests it was found that her creatinine level is 2.1 and mild proteinurea(+). On USG left kidney size 9.64 cm and right kidney is 8.65 cm and there is partial loss of cortico-medullary echoes. Can we plz tell the problem and treatment plan?? Doctor: The creatinine is higher, most probably it is due to hypertension related kidney injury (usg and s crest suggest early stages of chronic kidney diesease) , likewise kidney problems itself will result in higher blood pressure, triggering a vicious cycle. Because the medication he is on is not available, I suggest you to consult a nephrologist or your doctor and review your meds as decreasing proteinuria should be a priority . Also Tsh on higher side suggest s hypothyroidism, which would require supplementation of thyroxine"
},
{
"id": 103772,
"tgt": "Having reocurring allergy patch at bottom of back. Cure?",
"src": "Patient: Hi, I have a reocurring allergy patch that comes up on the bottom of my back every now and then. It 1st came up about a year ago and has come up about 4 times since. It is extremely itchy and lumpy. It seems to weep slightly as it develops tiny scabs but I m not sure if this is due to the scratching . It always comes back in the same place and probably lasts about 2 to 4 weeks. It is around the same size as 2, 2penny coins (uk). Doctor: Hello,Thanks for the query to H.C.M. Forum.You have mentioned only allergy patch from one year and 4 times in a year. This is not a complete history.Please mention, Age,Sex,Previous history of allergy ( as bronchial asthma)Family history,Diabetes, fever,any other member of family infected or not. Report of blood tests as about eiosinophilia or not. So please mention about above things.Good luck. Dr, HET"
},
{
"id": 204598,
"tgt": "How can anxiety and depression be treated?",
"src": "Patient: Gabapentin ( my psch,orders this for me ,and it seems to cover a very large range of things he said it might help with my anxiety I think not.a friend takes it for leg cramping not restless leg,I think he is avoiding xanx which that is what. I ve been taking for yrs,I m trying to go further in college and it makes it very difficult to do so with anxiety, depression,and chronic back pain should I really keep taking it? Doctor: Hello and Welcome to \u2018Ask A Doctor\u2019 service.I have reviewed your query and here is my advice.In my opinion, anxiety, and depression need to be treated with cognitive behavioral therapy and antidepressants. Also, moderate exercise and adequate diet are very beneficial.Hope I have answered your query. Let me know if I can assist you further.Regards,Dr. Gayathri"
},
{
"id": 51230,
"tgt": "High creatinine, low sodium. Funny smell in urine, dark color. Had pneumonia. Should I be concerned?",
"src": "Patient: I am a 28 year old male. Relatively healthy. I was in the hospital recently for pneumonia . They notice that my creatnine levels were high in my blood and low sodium . He mentioned he was worried about my kidneys. They gave me some fluids and sent me out the door. What I forgot to tell him is that I have been having a funny smell to my urine for almost a month. Sort of like incense. Its almost always darker colored urine as well. Unless I drink a TON of water. Should I be concerned Doctor: Hi, Welcome to HCM. You need urine analysis, urine culture, renal function tests with electrolytes and ultrasound scan of the urinary tract. Treatment is based on the results. Regards DR GS"
},
{
"id": 222606,
"tgt": "Suggest tests to confirm pregnancy",
"src": "Patient: Could sperm slip out the bottom of a condom and get you pregnant? i had sex with my girlfriend and i had a condom on and a sticky liquid was at the base of my penis leaking out the condom what are the likely hood she is pregnant and she isnt on the pill Doctor: Hi,If you had sex during safe period, risk of pregnancy is less likely but if sex was during unsafe period there are all chances of having pregnancy as even few sperms entered in vagina might migrate into the uterus might come in contact with ovum and can produce pregnancy.Ok and take care."
},
{
"id": 193807,
"tgt": "Suggest remedy for Ulcer sore on scrotum",
"src": "Patient: How to treat Ulcer sore on scrotum? Father is 88 years old and we just discovered the ulcer soar when he returned home from an extended Hospital stay. He will be seeing a wound care specialist Monday morning but what can we do until then? The sore is in the crease of his scotum where the elastic from his jockey shorts would rub against it so he is now wearing boxer briefs. We think it developed from wearing disposable briefs for an extended time while being confined in the Hospital most of the time sitting in a wheelchair. Doctor: Hello, The scrotal ulcer has to be treated properly because it may lead to life-threatening conditions like fournier\u2019s gangrene. Consult a general surgeon and get evaluated. Thorough wound cleaning and antibiotics are the mainstays of the treatment. Make sure that his blood sugar is well controlled because uncontrolled diabetes worsen the recovery. Hope I have answered your query. Let me know if I can assist you further. Take care Regards, Dr Shinas Hussain, General & Family Physician"
},
{
"id": 110051,
"tgt": "Suggest remedy for pain in the lower back",
"src": "Patient: sir i am 28 years old i suffering from low back pain only in left side from month 10 days back i will take an MRI report says L4-L5 disk was extended and it compressed the nerves , but i am able to walk some distance if cotunious walking pain will come on also standing countinueious pain wii come leg what s u r suggestion pls tell me doctor Doctor: Hi,Welcome to healthcare magic.After going through your query I think your You are suffering from lumbar disc prolapse.There are two treatment options.First option is Rest in position of relief(generally lateral position with both knee bend),NUROKIND GOLD ONCE DAILY and analgesics (Diclofenac three times a day after meals is effective). Sometimes strong analgesic(such as ultracet three times a day after meals) is required. OMEPRAZOLE before meals prevent acidity caused by analgesics . This method needs patience as it take longer treatment times sometimes up to 6 months.Second option is go for surgical operation(Discectomy). You can discuss with your treating surgeon.I think your query answered.Welcome to any follow up query"
},
{
"id": 136157,
"tgt": "Suggest remedies for pain in head after banging it",
"src": "Patient: Hi my son caught his ribs on the left side he reached down and banged his side .He s in pain and it really hurts when he breathes .He s been to see his doctor and he said just to take paracetamol and ibuprofen .Is there anything you could suggest .Thanking you . Doctor: HiWelcome to healthcaremagicI have gone through your query and understand your concern.It is most likely muscular pain. Your doctor is right in prescribing ibuprofen and paracetamol. You can continue with these drugs. In addition to this you can take vitamin B and C for early recovery. Rest will be useful in recovery. You can discuss with your doctor about it. Hope your query get answered. If you have any clarification then don't hesitate to write to us. I will be happy to help you.Wishing you a good health.Take care."
},
{
"id": 200585,
"tgt": "Suggest remedy for low sperm count",
"src": "Patient: hai sir/madam my pennis size is 10cm lenth and 1.5 cm round when in mood.i have a hand pump habbit. i ussually do it for 4 times a week.i am doing it since from 4 years.now iam getting very less quantity of sperms after peek stage.is there any problem in size and quantity of sperms.am i fit for marrued life.iam 23 old. Doctor: Hello,,There will be some cause for low Sperm count like Hormonal Imbalance , testicular problems like Varicocoel. As long as cause of the problem is not known treatment will not be effective . So I advise you to get the following tests done Fasting and Postprandial Blood sugars, Serum TSH, LH, FSH , Prolactin and Scrotal Ultrasound . Depending on what your reports shows treatment shall be advised accordingly. But till that time you can ask your doctor for a prescription of Tablet Paternia which will improve the sperm count . Thank you."
},
{
"id": 102486,
"tgt": "What is the cause for swelling on my neck area and ear lobe with allergies?",
"src": "Patient: pain and slight swelling on my right neck area below my ear lobe. I do have allergies but have never gotten ear infections. it is tender when I open my jaw wide...and almost felt \"locked\" earlier today..only on tht side.I worked out hard today with weights...and it feels like a muscle pull. hurts to touch. Doctor: Hello,Welcome to HCM,As you history of having allergy, it might have exacerbated leading to pain in the neck region and in the ears.Infection in the throat may lead to ear by the tube connecting between throat and the ears.Infection in the ear may be presenting with ear pain.As you are doing gym and muscle building exercise during the illness you will be exhausted soon.Neck exercise may have pulled your neck muscle. You need to undergo physical examination to know what exactly causing pain in the ear and whether it is a symptoms of allergy or not.Thank you."
},
{
"id": 61855,
"tgt": "Suggest treatment for a bruise on the scalp",
"src": "Patient: My 12 year old daughter has been complaining of a bruising pain on her scalp. Although I feel a slight bump, there is no redness or visable bruising. She has been complaining of pain to the touch for 4 days now. Has no knowledge of bumping her head. Doctor: I welcome you to health care magic for your health concernsI am Dr Fahim, General Surgeon and I will be answering your query I have gone through your question and understood your concerns. With available information it appears to be as a result of some trauma. Most important signs in head injury are vomiting, loss of consciousness, ear, nose, throat bleed. If none of the mentioned signs are there in your daughter then I think you need to worry much only some over the counter analgesics will suffice. However if she is sure about to trauma encounters then I advise you get an x-ray skull and see a surgeon to rule out any other rare pathologies. Wishing you and your daughter all the best and rapid recovery.I hope my answer will help you. Do rate my answer if you like it.Regards"
},
{
"id": 198681,
"tgt": "Suggest treatment for loss of libido in men",
"src": "Patient: I am 39 years unmarried male. I am habituated in musturbate in the age of 14 to 18 due to ignorance but after this I stop this but from the age of 25 I saw that during latrine Semen lekeage through urine n I feel dullness and Mind does not work in any works. I can't take silent sleep n night fall is not happening from the age of 30. I feel no sexual desire. Please suggest me some medicine . I am asol in depression. Doctor: DearWe understand your concernsI went through your details. Your lack of sexual libido and erectile dysfunction is due to lack of proper sexual education and knowledge. Masturbation is normal, natural and not at all unhealthy. Everyone masturbate including animals. Drops of semen along with urine is a normal process because semen is a waste product and your body automatically throws it out. Night fall also happens because of the same reason. You cannot control it unless you masturbate or indulge in sex regularly. This sort of negative knowledge led you to be anxious in sex matters. Therefore, I suggest you to consult a psychologist for counseling.If you require more of my help in this aspect, please use this URL. http://goo.gl/aYW2pR. Make sure that you include every minute details possible. Hope this answers your query. Available for further clarifications.Good luck. Take care."
},
{
"id": 104700,
"tgt": "Bloating, diarrhea, psoriasis, IBS. Done colonoscopy. Help?",
"src": "Patient: daughter has had diarrhea and stomch problems 28 days - nd taking ammodium--she has a bad head vol and has been workimg and has lots of stress--she is custodial parent to my 11 yr old grandson -- and travels 45 min to and from work ea day - and is prof artist -- she has bloating -- is divorced over 8 yrs now - and has a large house to take care of -- she saw a dr. yesteerday and said that asap a colonoscopy is needed -- we do not know any more - she lives 200 mi awa - and we are her parents - we have two daughters - Lori Ann is 46 and also has psoriasis - food allergies - smokes and is overweight - and has never had this before - she mentioned IBS--what do you think about a colonoscopy -aren t there other tests - what about the dr getting a dietician to help her plan menus- Doctor: hello, welcome to healthcare magic, if your daughter is having persistent diarrhoea it would be good to do a stool test to rule out any kind of infection, to do h-pylori test to rule out ulcers, side effect of drugs can cause diarrhoea and bloating. if she has bleeding accompanied with diarrhoea then would rush for a colonoscopy. from the brief history you have given me, she might be suffering from ulcers, she has many risk factors pointing to that these include. -stress -obese - smoking - food allergies. would suggest she needs to be investigated to know what is causing the problem would benefit from endoscopy instead yes she will benefit from a dietician for the food she needs to eat. hope to have answered your query wish your daughter a quick recovery and dont be stressed life is short enjoy it to the most. takecare."
},
{
"id": 63025,
"tgt": "What is the treatment for a itchy lump in the breast?",
"src": "Patient: I have a lump in my right breast about an inch under my nipple. I noticed it two weeks ago and it has not changed in shape since then. The lump is not painful and the skin on my breast above the lump is red and the skin is peeling. I m really worried. I am 33 years old and have breast fed three children (stopped a year ago with my youngest.) Doctor: Hi, thanks for your query to my virtual Clinic at HCM.You seem to suffer from ? abscess in breast cyst? Intra ductal Cystic Adenoma of breast, with ? Fibocystic disease.I would suggest you to consult Er Surgeon for Second Opinion as other causes of such breast lumps need to be ruled out by first hand check ups from doctor.Treatment -I would advise USG gided FNAC biopsy of the lump with planned surgical excision there after, by Er Surgeon under antibiotic and ani- inflammatory drugs.Hope this reply satisfies you.Don' t forget to hit thanks and write excellent review comments to help needy visitors at HCM.Wishing fast recovery to you.Dr.Savaskar M.N.M.S.Genl- CVTS.Senior Surgical Specialist."
},
{
"id": 166985,
"tgt": "What causes persistent fever in a 7 year old?",
"src": "Patient: MY SON AGED ABOUT 7,WEIGHT 21 KG. IS SUFFERING FROM FEVER LAST 5 DAYS,FIRST THE Dr. PRESCRIBED AZITHRAL FOR 3 DAYS NOW CHANGED IT TO MACPOD-CV-100-DT.BUT FEVER IS STILL CONTINUING.WHEN BODY TEMPERATURE IS RISING HIS LIPS BECOME MORE RED.KINDLY SUGGEST Doctor: HI!! I would like to know if the child has any other symptom like sore throat, cough, headache, bodyache, burning in urine. also has he been immunised for typhoid fever. any fever lasting more than a week needs blood and urine investigation. ask your doctor to rule out malaria, typhoid, dengue, invective hepatitis,urinary infection."
},
{
"id": 21416,
"tgt": "How can heart rate, high blood pressure with cholesterol be treated?",
"src": "Patient: I'm kinda worried, i have high blood pressure which i control with Diavan, my cholestrol is high which i am taking medication for, right now my blood pressure is 122 over 89 and my heart rate is 108, i normally everyday have a heart rate of 85, i just finished washing my truck and when i was done i didn't feel good at all so i checked my blood pressure and it was 136 over 91 my heart rate was 121. I sat down to relax for a little bet, then i checked my heart rate again and my heart rate went down to 103 and now it went back up to 108 and can't get it down should i be worried please help thank you by the way my name is Patricia Washington Doctor: Hello, Patricia. I understand your worries. It is normal for everybody to have an increased blood pressure after physical activity. Continue taking your medications as prescribed, you will be fine. Warm regards"
},
{
"id": 40320,
"tgt": "Need medication for swelling on body with lump due to a dog bite",
"src": "Patient: I was bitten by a pit bull 2 weeks ago the swelling is gone the puncture wounds are healing fantastically and the bruising is changing colors everyday however the is a large hard lump underneath the affected area is this/normal? Its also on my left outer thigh in the muscle. Thanks for your help. Tiffany Doctor: Hi,Welcome to HCM.Dog bite wounds if tended properly should heal by 2 weeks. More than the wound, rabies vaccination is important in this case. If the dog is vaccinated then you need to get a TT shot and observe the dog. If it is not vaccinated, you need to get antirabies vaccination as soon as possible. Application of antibacterial cream over the wound will heal it faster.Thanks."
},
{
"id": 154648,
"tgt": "Suggest treatment for meta bone cancer",
"src": "Patient: Sir my mother has 72 years old she has got meta bone cancer his origin from lung and already spread different type of body at that moment no treatment started but vomiting not stopped so plz tell me what should I do bcoz I cannot afford chemo or just like radiation. Plz suggest some to stop vomiting and abdominal pain. Doctor: Hi,Thanks for writing in.Cancer is a difficult condition to treat and the treatment changes with course of disease. Sorry to know the progress of cancer in your mother and hope she is not having much discomfort. It is best to start treatment as much possible by consulting a cancer help group in your place. She is most likely having stage 4 cancer and in this condition a patient might survive from 6 months to 2 years. We have to fight cancer till the end.Vomiting can be due to many causes and it is important to take her to a local doctor and get treated by injections to stop vomiting. There are many self help support groups for cancer and these might help you get support for treatment. You can also approach government facilities for treatment options and slow the disease process. Please do not worry."
},
{
"id": 157353,
"tgt": "Suffering from prostate cancer. Bone scan found prostate cancer spread to bones. Will it be cured by bilateral orchiectomy?",
"src": "Patient: sir,my father 78 years old is suffering from prostate cancer since 2009.In 2009,completed IGRT treatment and psa falls to 0. .Now from 2011 his psa rises from 0 to 15.5, then performed bone scan and found that prostate cancer is spread to bones. Then injections Zoladed quarterly,lupride quarterly but now psa rises to 61. Now doctor advices for Bilateral orchidectomy.I am very much worried .My father is heart patient. Whether this operation orchidectomy is necessary for him. Whether cancer will be stopped growing after this operation .Please help and guide me. Thanks Doctor: yes, orchidectomy helps him. Prostrate cancer is driven by masculine harmones and cutting down the production of harmones slows down the progression of disease. Lupride also does the same job, but orchidectomy is simple and more effective. But remember since the disease has already progressed, complete cure is not possible now and we are aiming at best possible survival with good quality of life. he might soon require chemotherapy also"
},
{
"id": 56582,
"tgt": "What does bilirubin direct-0.3 and indirect-1.3 suggest?",
"src": "Patient: Direct 0.3 ( normal .02 and below) Indirect 1.3 ( normal 1.2 and below) Total 1.6?( normal 1.2 and below) All other liver tests came back in normal range. Also all other blood tests came back perfect . Should I be worried? I just had another private test done today Doctor: Hi there,Thanks for using HCM.This mild increase in bilirubin is not significant and may not indicate anything serious. So you need not worry about it.If there is fluctuation of the bilirubin levels with normal liver enzyme levels, then the likely cause is Gilbert Syndrome which again is a harmless condition and does not require any treatment.Sometimes increased destruction of the red blood cells can cause similar picture. Hemoglobin level, a peripheral blood smear picture and a reticulocyte count may be done to see if destruction of red blood cells is the cause.Is this answer helpful?"
},
{
"id": 64690,
"tgt": "Suggest a remedy for the lump on foot",
"src": "Patient: Trying to save my scooter from falling the side foot step rammed into my leg.This happened a week and a half ago. The bruise is barely noticable now. i awoke this morning ,got up and went to walk and it hurt. I noticed when I rubbed my leg there was a knot that had formed. I am puzzeled because I now have a lump where the bruising was.I had blood clots years ago in my lungs years ago so needed to know if this is normal to have a lump form so long after the initial injury. I don't go to the Doctor much because my employer dropped our health insurance coverage.I have to pay cash each time I go and belive me I don't eat nothing but ooodles of noodles till my bill is paid. What do you think I should do Doctor? Doctor: Hi,dear thanks for the query to my HCM virtual online clinic.Its my pleasure to help you as you are without health insurance benefit.My diagnosis -its a -Unresolved Hematoma- which you noticed.As you had lung clotting history,you should test your clotting factors if it worries you , but not needed for this lump.With the treatment this lump would get reduced in another 2 wks time.You should not worry of it but I advise you to Consult a ER Surgeon-on the safer side.Dont worry and act fast.This would solve your worrysome query.Hope this would solve your query.Wellcome to my HCM clinic once again.Have a Good Day."
},
{
"id": 82157,
"tgt": "What causes chest pain,nausea and cough?",
"src": "Patient: I have right sided chest pain....came on quickly a week ago. Was bad at first...I vomited. I have random bouts of nausea...the cp is isn t constant. Yesterday I had little today more. I had the flu type A about two months ago and I have had a bad productive cough until about 5 days ago. Now it is almost gone. I am an RN so I though it might be pleurisy or walking pneumonia....any ideas? I am 44. Have. 3 kids. No other health issues. Also my stools are claycolored and I have very little appetite. Perhaps gallbladder? Doctor: Hi,Welcome to HCM,Coming back to your query, this variable symptoms of Chest pain, nausea, Vomiting and Clay coloured stool with no cough at present it would be worthwhile to investigate for Gall Bladder with Ultrasound of abdomen. If you doesnt have any relationship of pain with respiration and resolved Cough then it is less likely to have Pleurisy or Walking Pneumonia"
},
{
"id": 168374,
"tgt": "What causes fever and headache in a child?",
"src": "Patient: my 8 year old daughter wieghing 60 pounds is running a 101.9 fever after Ibro. just came out of her room laughing saying daddy threw me at the house then started to cry then laugh again. after 3 minutes of this she started to complain her head hurts really bad when asked about the previous behavior of laughing then crying she does not remember it should I take her into ER Doctor: Hello. I just read through your question. Though in most cases this could be due to a harmless virus, her behavior is concerning. I agree that she should be seen immediately."
},
{
"id": 72274,
"tgt": "Suggest treatment for tenderness in lowest rib",
"src": "Patient: I had a pain just under my left ribs on my side, it came and went (almost a spasm or pulsating pain) for about 5 hours a couple of weeks ago. Have had some sharp pains a couple/few times a day since. Now it seems that the lowest rib on left side in back is VERY tender. Doctor: Thanks for your question on Healthcare Magic.I can understand your concern. Possibility of musculoskeletal pain is more likely. So follow these steps for better symptomatic relief in musculoskeletal pain. 1. Apply warm water pad on affected areas of chest.2. Avoid movements causing pain. Avoid sudden jerky movements of chest.3. Avoid heavyweight lifting and strenuous exercise.4. Take painkiller and anti inflammatory drugs.Don't worry, you will be alright with all these. Hope I have solved your query. I will be happy to help you further. Wish you good health. Thanks."
},
{
"id": 109719,
"tgt": "What causes lower back pain and burning sensation during urination?",
"src": "Patient: I'm having bad lower back pain that shoots through the rest of my back with I wanna say burning discomfort, and when I go to urine I have the mist uncomfortable cramps I've never had not to mention but my stomach feels horrible cramping aching burning ......I have ova and a kidney stone I really don't know what to do I go to my doctor and to the E.R and still no relief and I'm also taking Amitiza for ibs .......any Suggestions Doctor: Hello,I had gone through the case and found that it might be due to urinary tract infection or passing out of renal stone to tract.so immediate take painkiller injection and go for sonography of abdomen and urine routine and culture test.After getting the proper diagnosis take treatment.Hope my answer will be effective for you.Thanks"
},
{
"id": 10396,
"tgt": "Suggest remedy for hair fall",
"src": "Patient: hello i am 23 years old girl. my hair has always been falling but for the past few months i have lost almost more than half of my hair. i am so upset and panicked. i have got all my tests done namely, thyroid test, iron test, female hormones test from Germany. everything turned out to be ok. my hair is still madly falling. i am in serious need of help... Doctor: Hello and Welcome to \u2018Ask A Doctor\u2019 service. I have reviewed your query and here is my advice. I have gone through your complaints and would recommend you to start with minoxidil 2 percent lotion on the scalp twice daily and take hair supplement tablets containing biotin, calcium pantothenate. These medicines need to be continued for at least 4-5 months to see significant results. Hope I have answered your query. Let me know if I can assist you further."
},
{
"id": 209868,
"tgt": "How can one overcome anxiety, negative thinking and sleep?",
"src": "Patient: suffer from negative thinking.... anxiety...low self esteem....... fits of rage,,,,, and lash out at my family members. After a fit of anger i feel dazed and unable to focus or sleep . I try light exercises and meditation but is of little use. Can you help ? Doctor: Hi,I can understand what you might be going through. You seem to be having symptoms suggestive of depressive episode. I would have liked to now how long are you having these symptoms from, your age and any comorbid medical disorder you might have been diagnosed with?I would suggest you consult a psychiatrist for proper evaluation. You might benefit with medications like selective serotonin reuptake inhibitors. In addition, relaxation techniques like deep breathing and progressive muscular relaxation will help you maintain your calm.Hope this information was helpful. Best wishes."
},
{
"id": 45596,
"tgt": "Is flushing on the face after a lithotripsy a matter of concern?",
"src": "Patient: Hi, I had a procedure yesterday to break up a kidney stone using sound waves. In the discharge instructions, it mentions calling my doctor if I have skin changes, such as rash, itching or hives . I don t have those, but my face is flushed. I tried to call the doctor, but they close at noon on Fridays. I don t have any other issues, but I didn t know if I should use their answering service to contact the doctor on call. Hate to bother them if it is nothing to be concerned about. I am taking hydrocodone/acetaminaphen for pain and Bactrim antibiotic. I have taken both medications before with no issues. I am not having any problems urinating. Doctor: Hello, It\u2019s all normal and happens in some cases after surgery. You don\u2019t need to worry about anything. Hope I have answered your query. Let me know if I can assist you further. Regards, Dr. Naira Jahangir, OB and GYN Specialist"
},
{
"id": 196395,
"tgt": "What causes lack of libido?",
"src": "Patient: hi doctor my husband is 34 years old ,he doesnot get the urge of having sex on his own... may be the arousal is ok but every time i ve to help him in his arousal , every time he gets angry if i want from my end.. we get physical at an interval of 1 and a half month and so.. i get irritated at times .What i want to know is is it a physical disorder do i need to take him to a doctor for this Doctor: Hi welcome to the health care magic Low libido means decease desire for sexual activity For low libido physical as well as psychological issue are related So by careful history cause, needs to be evaluated Causes for low libido are as follow.... _chronic medical illness _low testosterone _stress, depression, anxiety _certain drugs etc... Check diabetes , thyroid issues as well According to cause specific treatment given If depression or stress associated then medication for that given So with keeping all this in mind consult nearby physician for examination Take care"
},
{
"id": 147932,
"tgt": "Need suggestions to cure vertigo",
"src": "Patient: Hi my name is Liz, i have an 82 year old mother, who mentally great, however she is now in an aged care facility due to lack of mobility, she is now using a walker. She complains of her feeling like she is carying a watermelon around, she feel extremely dizzy the room often turns upside down for her. She is now at the point she doesnt want to leave her room. The doctors here in Tasmania, Australia, just say oh probably old age vertigo. She is so unhappy. One doctor two years ago mentioned to us after a brain scan that it is to do with cerrebullum being cloudy and messages are having trouble getting through to her limbs, other doctors have said they dont think this is the case, but they dont knowmwhat is her problem. She does have high bp and is on meds for thatCheersLiz Doctor: Hello, Thanks for your query.There are a few things that you should follow to prevent vertigo. They are as follows -* Change your position slowly, especially when going from a lying or sitting position to a standing position. When you get out of bed, sit on the side of the bed for a few seconds to gain your orientation and allow your circulatory system to adjust.* When walking, focus on distant objects. Do not look down at your feet. Avoid walking in dark areas or on unstable ground. * Use a cane, walking stick, or walker for support and to give additional pressure and touch (tactile) orientation.* Avoid activities that move the head up and down repetitively.* Try to avoid keeping the head tilted back for long periods of time, for example painting or dusting above your head.* Be cautious when using medications that may cause balance problems as a side effect.I do hope that you have found something helpful and I will be glad to answer any further query.Take care"
},
{
"id": 73356,
"tgt": "What could intermittent chest tightness suggest?",
"src": "Patient: hello sir / madam,i am male 23 years old and overweight with a heavy body . i was doing all good but when i returned from malaysia last week i started to feel light pain and tightness in the middle my chest, which comes in day and goes at night .what can it be ? please help Doctor: Asthma can often cause tightness in the chest. This is something that you can be tested for. Other causes include gallbladder inflammation (cholecystitis) and reflux."
},
{
"id": 160888,
"tgt": "What are the things to look for after headache,high BP and vomiting?",
"src": "Patient: 16 yr old son was air lifted to hospital yesterday with severe headache, bp 170/110, vomiting during basketball practice. bp stabilized, 2 ct scans were good, lumbar puncture clear, sent home in the evening. diagnosis post exertion headache. he is sleeping today due to tiredness. what should i be watching for & how do i know it is safe for him to do anything? Doctor: HiYou need to watch for any fever, vomiting, headache and convulsions.Blood pressure should be measured twice a day. Adequate hydration is essential.In case of any of the above symptoms,visit your physician immediately.Hope that was helpful.Let me know if i can assist you further.RegardsDr.Saranya RamadossGeneral and Family Health Physician"
},
{
"id": 110071,
"tgt": "What is the treatment for sharp back pain?",
"src": "Patient: Hi Doc, whenever I reach out for something , I get a sharp pain in the upper quadrant of my back,,near the mid - spine and below the right shoulder. How can I get rid of this pain? Should I treat with heat or cold or a combination and at what interval and for how long? Doctor: Hi,Welcome to healthcare magic.After going through your query I think you are suffering from chronic backache . Treatment of back pain is exercises and analgesics (diclofenac ). Sometimes vitamin D deficiency is the cause so get her vitamin D checked .If it is low then vitamin D supplementation ( weekly with milk) can be taken.Avoid long continuous standing.Sit in a straight posture. Eat milk and milk product, fruits and green leafy vegetables daily. To further investigate MRI of the region is advised . I think your query answered.Welcome to any follow up query"
},
{
"id": 46201,
"tgt": "Suggest treatment for back pain above the waist",
"src": "Patient: I am single 29 years old male. working in an IT MNC. Since few months I am facing backache all over above the waist when I am awake in morning and I have experienced this on almost all kind of beds. Also, these days I feel kind of pain in bones. History: I had gone through a stone gall bladder surgery where it has been removed through laser therapy. Also many years back, stone was found in kedney which was removed through medicine. (through urine). Please suggest. Doctor: Hi being a sedentary worker, you should walk/ do back exercises/yoga regularly. Also check your calcium and vitamin D levels --if deficient, take supplements---regards"
},
{
"id": 18550,
"tgt": "What causes breathlessness after stent placement for a blocked aortic valve?",
"src": "Patient: Aorta is 80 % blocked .Another valve is 30 % blocked. Stint placed in 30% blockage. Mother is having difficulty breathing and has been intobated. O2 sat going into 80\u2019s T least once a day and she crashes. Why would Dr place stint first , instead of repairing aortic valve Doctor: Hello and Welcome to \u2018Ask A Doctor\u2019 service. I have reviewed your query and here is my advice. After going through your medical details I understand your concern and I would like to tell you that blood supply is very important to heart muscles if you want your should be working in extreme physical stress during surgery. In absence of full detailed medical history of your case I can just tell you that there are many possibilities of why they didn't repair valve as it can be patient isn't fit for surgery, patient was having acute coronary syndrome etc. Kindly speak to your treating Cardiologist as he can tell you in detail about your specific query. Hope I have answered your query. Let me know if I can assist you further."
},
{
"id": 87565,
"tgt": "Suggest remedy for nausea and abdominal pain",
"src": "Patient: I am a 53 year old woman recently movefd to Bulgaria. Before I used to take Livial Tiblorone hrt, unfortunately I am not able to get it here and my doctor has prescrible Kilogest. I have been taking it now for 2 weeks and I still have the side effects of nausea and abdominable pain. Will this go off shortly or should I find an alternative Doctor: Hi! Good afternoon. I am Dr Shareef answering your query.You did not mention for how long have you been on this HRT. If you are already nearing one year, you should get yourself re assessed by your treating doctor for if you would be needing this medicine any more or not. Nausea and abdominal pain could be a side effect of these medicines, which could get gradually relieved. However, if I were your doctor, I would also advise you to get some basic blood tests like a CBC, blood sugar, and an LFT as these drugs are metabolised in the liver. In case of any abnormality I would advise you to consult your doctor for a possible discontinuation of this drug.I hope this information would help you in discussing with your family physician/treating doctor in further management of your problem. Please do not hesitate to ask in case of any further doubts.Thanks for choosing health care magic to clear doubts on your health problems. I wish you an early recovery. Dr Shareef."
},
{
"id": 126332,
"tgt": "What causes stiffness in the neck and soreness from the neck to the shoulder?",
"src": "Patient: I have had a stiff neck. soreness from neck to left shoulder. I just have soreness today but can not turn neck. for instance it is difficult to turn my head at a stop sign to see traffic. I just turn my body a little. when I try to turn my head there is a sometime extreme soreness Doctor: Hello, Consult an orthopaedician and get an MRI neck to look for conditions like cervical spondylitis. As of now you can take analgesics like Aceclofenac or Tramadol for pain relief. Hope I have answered your query. Let me know if I can assist you further. Regards, Dr. Shinas Hussain, General & Family Physician"
},
{
"id": 18738,
"tgt": "Should I be concerned with a blood pressure of 97/64 and pulse rate of 102?",
"src": "Patient: my blood pressure is 97/64 and my pulse rate is 102 should I be concerned? The pressure seems low for me and the pulse rate is definitely high for me. My normal BP range is between 105-120/77-85. My pulse is normally 65-80. I had an episode on Sunday morning where my chest felt tight, my hands started shaking, my heart was beating very hard and fast, I felt like I was going to pass out and when I went to lay down I couldn't move because it felt like my body seized up on me. Is there something I should do? Doctor: Hello and Welcome to \u2018Ask A Doctor\u2019 service. I have reviewed your query and here is my advice. I passed carefully through your question and would explain that your blood pressure values are low and the high heart rate is a normal physiological reaction caused by low blood pressure. Coming to this point, I would recommend having some rest and taking plenty of fluids in order to remain well hydrated. Increasing salt intake in your diet will help avoid low blood pressure values. If the problem persists, I would recommend consulting with your attending physician for a physical exam and some tests: - a resting ECG - complete blood count for anemia - blood electrolytes for any possible imbalance - thyroid hormone levels - kidney and liver function tests. Hope I have answered your query. Let me know if I can assist you further."
},
{
"id": 164873,
"tgt": "Could persistent fever in a 8 year old be a matter of concern?",
"src": "Patient: My eight year old has had a low grade fever for three days. The first day she was very tired and achy, the second day tired and today she has a bit of a cough but says she feels fine but the fever is still there. Is there cause for concern? Should she go back to school tomorrow with a low grade fever? Doctor: Hello. I read your query. Your daughter might be suffering from Viral fever which generally takes 3-5 days to go completely. I would suggest you to giver her antipyretic drug...whichever you are giving. Please don't let her go to school. She will get even more exhausted on doing exertions. Let her take complete rest. She will be fine soon.Give her plenty of fluids. Keep her hydrated.Light food to eat so that it can be easily digested.I hope this helps you. Wish her speedy recovery.Dr. Tripat Mehta"
},
{
"id": 121991,
"tgt": "Am I in danger of brittle bones?",
"src": "Patient: I broke my right arm close to the shoulder holloween weekend, simply by throwing my arm... I have no insurance so I haven t seen a doctor but at ER but find it strange it broke so easily. MRI was recommened and blood was drawn but other then that and a brief follow up, nothing was ever done. What could it be? Doctor: Hello, Such fracture needs further investigation including : - vitamin D and calcium plasma levels - a serum protein imuno electrophoresis in order to investigate for multiple myeloma, etc. Immobilization is needed, considering the fracture. Hope I have answered your query. Let me know if I can assist you further. Take care Regards, Dr Ilir Sharka, Cardiologist"
},
{
"id": 223361,
"tgt": "Can pregnancy happen despite taking Levonorgestrel after having sex?",
"src": "Patient: First day of last period: 2nd Feb. Last day of last period: 7th Feb. Had unprotected sex, ejaculated inside on 13th Feb. Usually my cycle is 28 days. Took a pill with levonorgestrel 28 hours after, on 14th Feb. What are the chances of pregnancy? Should I be concerned? Is there anything else I can do to prevent this? Doctor: hello user,u took emergency contraceptive,so dont worry..wait for your period..levonorgestrol provide effective contraception...thanks.."
},
{
"id": 108562,
"tgt": "How should persistent back, thigh and knee joints pain be treated?",
"src": "Patient: hi doctor i m 27yrs old working as a design engineer. i am suffering from severe back pain and though i consulted an orthopedician here in dubai. he told me its because of some muscle stiffness and needs pysiotherapy. and i undergone pysiotherapy for 1 week and got some cure from pain. again after 1 month pain starts and when i consult the same doctor he is saying its because of IVDP. he given some pain killlers and muscle relaxation medicines. but still i am suffering from pain. now the pain extending to the thighs and knee joints also. also i hve pain in the left shoulder. during my work i m always sitting infrontof computer. but now i used to go for a walk after every 20 minutes. Doctor i want to know is this a serious issue and want to know how to get a permanent recovery from this back pain. waiting for your valuable reply....... KIRAN. Doctor: Yaa if u dont answer the problem now than it might become a permanent problem. Try relaxing after u hav worked for 1 to 2 hrs. Take a break of 15mins. Relaxe ur muscles and start working back."
},
{
"id": 78035,
"tgt": "What causes shortness of breath after popping infected bumps?",
"src": "Patient: My mother 57 years old has had smelly white cheesy filled bumps on her back most of her life. I popped them last night for her tons of this yucky stuff came out and now she is very cold and has shortness of breath do you think the infection coming out could make her physically sick? Doctor: Thanks for your question on Health Care Magic. I can understand your concern. Yes, severe infection (sepsis) is more likely cause for her symptoms. She might be having gluteal abscess. And the stuff you are talking about it pus only. Pus is liquefaction of tissue due to local infection which is uncontrolled. This severe form of infection can also spread to blood. This is known as sepsis. Sepsis is most common cause fir hypotension (septic shock). And this hypotension causes shortness breath, cold clammy skin. So your mother is mostly having sepsis. Better to consult doctor and start antibiotics and surgical debridement of pus. Hope I have solved your query. I will be happy to help you further. Wishing good health to your mother. Thanks."
},
{
"id": 106956,
"tgt": "What does lower backache on the right side indicate?",
"src": "Patient: I keep having sharp pains in the lower right side of the back of my head. Last for about 10-20 sec and occurs about every 30 minutes. I have a metal mitral valve and take blood pressure, warafin and colorestol medicines. In addition, am being treated for a cold. Doctor: hi sir/madam,Thanks for your question on Healthcare Magic.Common Causes of Lower Left Back Pain:-1.Bulging or herniated discs.2.Muscle spasms.3.Pulled back muscle.4.Muscle imbalances.5.Muscle strain, injury or trauma to the back muscles, ligaments or joints.6.Trigger points.7.Stress / Negative emotions.If lower left back pain is particularly severe, it may be an indication of a herniated or ruptured disc, or sciatica.Recommended Treatments for Lower Left Side Back Pain:-1.Muscle Balance Therapy.2.Massage Therapy.3.Trigger Point Therapy.4.Inversion / Decompression Therapy.5.Acupuncture / Acupressure.6.Use a Back Support or Cushion.7Relaxation, Meditation.8.Dietary improvements (ie. reducing foods that cause inflammation like dairy, wheat / gluten, sugar, nightshades).9.Drink more clean, filtered water.Left Side Back Pain Prevention:-1. While walking and sitting, keep a relatively upright posture2. When lifting objects (heavy or light), always lift with the legs-never bend from the waist3 .Exercise; focus on working out the muscles in the core (abdomen and back)4. Daily Stretching. Hope i was helpful.Have a healthy day."
},
{
"id": 206445,
"tgt": "What causes night sweats and mood swings?",
"src": "Patient: Hi, may I answer your health queries right now ? Please type your query here... Im 46 do experience night sweats, difference in my mood swings which hve been apparent for a least a year now. I have also put on a bit of weight which i'm not happy about. I am a single parent and have a v. stressful job and sometimes feel I cant cope. I dont feel excited about much lately and everythng seems like a chore. . I have lost two relatives over the few years and seen a slight deterioration in my mum now Not sure whether to go to my GP for advice or be firm and try and kick start exericse regieme. Its all difficult though as I am on my own. Doctor: HelloNight sweats, mood swings are commonly seen in severe anxiety. In middle aged females mainly in 40-50 years of age pre menopausal syndrome may be the main reason of such symptoms. Females suffer night sweats, hot flushes, mood swings, low mood, crying spells. These symptoms occur due to change in hormonal levels in a female of menopausal age. You have many aggravating factors also like stressful job, loss of close relatives, deterioration of your mothers health etc. You are a single parent which also increases the risk. All these factors are further increasing your depression and anxiety. I would advise you to consult a good Psychiatrist for proper management of these symptoms.- Start exercise daily. Practice yoga and meditation. These things will help in keeping you calm.- Take balanced food and plenty of water.- Decrease the intake of dark chocolates and coffee. Avoid smoking and alcohol.- Medicines like SSRIs and SNRIs can be used in such symptoms. Consult a psychiatrist for proper prescription.Thanks"
},
{
"id": 40372,
"tgt": "What diet should be followed for recovery from typhoid and jaundice?",
"src": "Patient: hello, i am suffering from typhoid and jaundice, felling very weak and full stomach too.. what kind of foods can be taken to cure it quickly my doctor reports say serum bilirubin : a.2mg/dl widal ; 1:160 1:80 1:20 1:20 so please suggest some thing that can cure quickly Doctor: Hello, Welcome to HCM, As you are having increased bilrubin level and raising titers of O and H antigen of typhoid, you should not put extra burden on liver. Liver will be affected in this condition, as liver is considered as factory of the human body, most of the metabolism takes place in liver. As liver is affected, the diet rich in protein and fat should be avoided for atleast 4 weeks following the infection. You can take diet rich in carbohydrates and soft diet, which will help for faster recovery of the damaged liver caused by the infection. Thank you."
},
{
"id": 179671,
"tgt": "What should be the usual weight for 2 1/2 year old girl?",
"src": "Patient: Our adopted little girl who is now 2 1/2 only weighs 20 pounds and still wears a size 9-12 month. I have seen both bio parents and they are not small. She also has a Bio brother born a year to the day after her and we hear he has some medical issues that are similar and they are now doing genetic testing. So far, our daughters doctors don t seem to concerned, but she is our youngest of 8 children and her weight seems for to be really low for her age. Doctor: Hello welcome to health care magic average eight at 2 1/2 years is 26 pounds , yes she in underweight but more important than a single value is growth pattern of baby , if she there is downtrend in percentile , you need to worry and rule out any disease responsible for it. If her growth pattern is OK you need not worry and monitor her weight at 2-3 months intervals.Apart form weight you need to monitor height also.Hope I answered your query."
},
{
"id": 57629,
"tgt": "What could loose stools, blood in vomit while having liver abscess mean?",
"src": "Patient: My boyfriend had liver abscess nearly three years back. His liver is sensitive. He doesn t eat junk food but last night he had loose motions and he suspects little blood in the vomit. He passed a lot of loose stools.Help me know what s the matter and what can be done. He is 21 years old. Doctor: Hi,Welcome to Health care magic forum. As you describe it appears to be the food poisoning, means infection of the foods,they need not be the junk food related. Some times the home foods also get infected due to storage problem. I advise you to consult a doctor for diagnosis and treatment. He may need to have infusion of I.V.fluids for correction of dehydration. I usually prescribe to my patient with such symptoms injections of metaclopramide,cefotaxime,tablets of lactic acid bacilli, ofloxacin,and domperidone, and loperamide. Wishing for a quick and complete recovery. Thank you."
},
{
"id": 43310,
"tgt": "Can I get a baby in one testicle ?",
"src": "Patient: Hello, sir, i am suffering from undiscended testicle, ie left part of the testicle is housed at lower abdoment, right part of the testicle is in normal position(it is in scortum), my qurery is can i get a baby in one testicle... Doctor: Hi,Thanks for writing in.First of all you must get your undescended testis in the abdomen removed surgically. This is because there is a very high chance for cancer formation in undescended testes. Please have yourself examined by a surgeon.Secondly, you can become a father if you have only one testis. However, the sperms produced by you must be normal in number, morphology and motility.Hope this helps."
},
{
"id": 215353,
"tgt": "Suggest remedy for cramping in stomach and back",
"src": "Patient: My husband works in a sawmill and it gets really hot in there in the summertime. Today he got really hot and lost a lot of fluids. He has been cramping all over; especially in his stomach and back. Is there anything that can help with the cramping? I know he is probably dehydrated so he has been drinking a lot of fluids.... Doctor: Hello, As per your history, it may be due to stomach infection. For that, you can take tablet cyclopam after consultation. Take a soft diet. Keep your self hydrated. In case of loose motion or vomiting, you may require antibiotics, complete hemogram, ultrasound abdomen after consultation. Hope I have answered your query. Let me know if I can assist you further. Regards, Dr. Shyam B. Kale, General & Family Physician"
},
{
"id": 5047,
"tgt": "Missed period. Had intercourse. Taken unwanted 72. What is the reason?",
"src": "Patient: Hello Madam/Sir, I am 30 yrs & my wife is 26 yrs. She had her menstruation on 2nd of June\u201913 (with a normal tenure 27-28 days & for 5 days). On 9th June, we had intercourse and though I am not sure yet my wife had taken \u2018Unwanted 72\u2019 pill after 6 hrs, After that she had her menstruation on 20th June for three days (in 19 days gap). We supposed that her next menstruation will be on 2nd of July\u201913 (as it was her normal date).But it was missed. We wait till 21 July\u201913. Till now her menstruation is not occurring. Now we can\u2019t understand the reason. 1). We can\u2019t understand, on 20thJune\u201913 when she had got her menstruation on 19 days gap, whether it is her menstruation or only bleeding as a side effect of taking the pill? 2). Should we wait for 2/3 days more? 3). Should we go for pregnancy test? 4). If it is, what is the convenient time for test? 5). If pregnant, is there any process to terminate this by taking medicine only? 6). If it is, between how many days, we can do it? 7). As we intercourse just after completion of her menstruation cycle, how far it is safe to not be pregnant? Note: - On her menstruation, her physical condition is ok. There is no side effect like, abdomen pain or any thing. (If you need any other information regarding the physical condition of my wife, plz tell me). Doctor: HelloThanks for writing to us.Here are your answers -1. It was mostly the side effect of taking the emergency contraceptive pill.2. The emergency contraceptive generally prepones or postpones your normal periods by a week, or upto 10 days. You can wait for a week.3. It is better to take a pregnancy test.4. Early morning urine samples are the best.5. Yes, medical abortion is possible, but it is to be conducted by a gynecologist.6. Generally, upto 49 days, or 7 weeks of pregnancy.7. Mostly just after menstruation, chances of pregnancy are quite remote.Take care."
},
{
"id": 4181,
"tgt": "Which is the appropriate time for conception?",
"src": "Patient: I have been taking UBiphene table since 7th months but yet i have not conceived , from 2nd day of my period will take UBIPHENE tablet my period was on 5th may i know when my egg will release on which day should we should we have sex pls do let me know. Doctor: Hello and welcome, I understand that you are keen to get pregnant soon but taking siphene and keeping contact is not the best way to do it. Please understand that drugs like siphene have thier own set of side effects. They cause thinning of the uterine lining and hence actually may prevent pregnancy if used for prolonged periods. It is not recomended to use siphene for more than 6 months.So my suggestion is that if you are having problems with conception please visit an infertility specialist and get evaluated for ovulation, open tubes and husabands semen count. Then the proper treatment can be formulated. For women that are having regular periods the ovulation occurs between day 10-20 of the cycle and keeping contact on these days has maximum chance of conception. Hope this satisfies your query. Thanks for using HCM.\u00a0\u00a0\u00a0\u00a0\u00a0Feel free to ask any more questions that you may have. Dr Madhuri BagdeConsultant Obstetrician and Gynecologist"
},
{
"id": 157111,
"tgt": "Taking treatment for prostate cancer, having bleeding from colon. Also having pain due to diverticulitus",
"src": "Patient: Following proton irradiation in Bloomington IN 4 yrs ago for prostate cancer, I m have been experiencing bleeding from my colon. It appears on the toilet paper. It has been examined by colonoscopy, and the gastroenterologist a year ago said that the vessels bleed easily now and it s normal to have a small amt of bleeding. I also have an old colon ulcer which prevented using an argon laser. I also was diagnosed with diverticulitus which has given some pain. But the bleeding has increased lately. I see it the next day after taking my 1hr walk. I am 76, and otherwise in good health. Doctor: HiThanks for your query.Options for radiation induced colorectal bleeding include argon plasma (which is probably not suitable for you), some rectal enemas or oral medicines like tranexemic acid. You can discuss these options with your gastroenterologist. This is usually a difficult condition to treat.Hope this helps.Regards"
},
{
"id": 110969,
"tgt": "What is the treatment for swelling painful back caused by an injury?",
"src": "Patient: Hello, Please, I need help. my name is Luminita Bondoc, I was falling on the stairs last morning and I have pretty bad injuries: my left side is swelling on my back around kidney are. the pain is very sharp, I can move, I tried to find a position in bed, but no success. The pain is acute, sharp and continuos. I did have also minor injuries on my head , bruises on my left hand, but the pain is bearable. Due to fibromyalgia I have o.5 percocet. Does help Please I need help Thank you, Sincerely Luminita bondoc Doctor: Hello, Thanks for your query.From description , it seems you might developed muscular or ligamentous strain over back .If pain is severe ,I would suggest getting this evaluated by an orthopedician for an accurate diagnosis and appropriate management. You may get the MRI of the area affected under his/her guidance.Meanwhile following measures will help with the pain\u2022 Lie on a hard bed.\u2022 Anti -inflammatory drugs like Tablet Motrin 1 tablet as and when required \u2022 Avoid forward bending.\u2022 Avoid strenuous activity & lifting heavy weights.\u2022 Apply diclofenac gel on the affected area. Warm compresses will also helpI do hope that you have found something helpful and I will be glad to answer any further query.Take care"
},
{
"id": 55434,
"tgt": "What causes infection after gallbladder stone removal?",
"src": "Patient: my mother had a gall bladder stone operation on last monday.she had a pipe from the cut end to outside to release the wastage.but day by day the wastage is increasing & neutrofil count has increased.doctor said its an infection.he wants to do endoscopy & ERCP.is it ok?how much serious is my mother s health? Doctor: HiIf The colour of fluid coming out from drain is greenish,then its bile leakage from gallbladder stump which hasnt been tied properly or due infection.If the colour is greyish creamy then its most probably pus due to infection.I would have suggested laparoscopy to look into the the abdomen and fix whatever it isI woud also send fluid for culture and sensitvityIn my opinion you shoud consult a general surgeonThanks"
},
{
"id": 59140,
"tgt": "Pain in lower back, abdomen and near rectum, diverticulitis. Taken antibiotics. Past bladder sling, hysterectomy, diabetes. Advise?",
"src": "Patient: Okay I have a pain near rectum also I have has diverticulitis in the past to. I had a bladder sling in 2009 since then I believe things have gotten worse lower back pain sever loewer abdominal pain and sharp pain on right side while trying to urinate . I've been to drs had scans and the thing they did see was diverticulitis and I was given antibiotics , but as I told them I have trouble having bowel movements some times I strain and notice blood mixed in stool. Though no blood just pouring out. Also to urinate I feel a full bladder and it sometimes feels like its not coming out or I have to adjust my self to get it flowing but they say no urinary infection I feel as if I'm going crazy!! I've had to have full historectomy gallbladder removed just months after and my apendix was robed when I was 20 I was31 when I had my hysterectomy 32 gallbladder and almost 36 for bladder sling . Please so u have any answers or info keep in mind I am diabetic but it has been undercontroll on sliding scale insulin. I'm tired of going to er just to hear they can't find anything and I don't know if not explaining it right or just not telling wut I so know . But it's getting expensive especially not having insurance. Doctor: There can be a number of reasons for lower abdominal pain amongst these are earler operations in the area and adhesions which would not show in and x-rays. However,if you see blood in the stool and CT- scans show diverticulitis, you should have a colonoscopy done to rule out polyps or other colon pathology , which could cause your symptoms."
},
{
"id": 38557,
"tgt": "What are side effects of bactrium ds 800-160?",
"src": "Patient: Hi my doctor prescribed 2 bactrim ds 800-160 twice a day for an ingrown toenail infection. It just seems like a high dosage. I weigh 181 pounds and I am 5 foot 7.I am 22 years old and in good health. Can you tell me if that dosage amount sounds reasonable. Doctor: HI, thanks for using healthcare magicThe dose is appropriate.Bactrim is a combination of sulfamethoxazole and trimethoprim.The strength that you have been given in the double strength. The difference between the double strength and the normal strength is the number of tablets taken.In double strength only one is taken with each dose, in the other strength 2 are taken to make up the strength.The dose is fine.I hope this helps"
},
{
"id": 94396,
"tgt": "History of appendectomy and abdominal scar. Have abdominal pain, UTI, headache, high blood pressure",
"src": "Patient: I had open appendectomy 22 years ago and a long scar on the right lower abdomen. I have been feeling pain from this since. I have had series of urinary tract infection , ultrasounds, both transvaginal and pelvic . The pain can be very discomforting at times. A week ago I had a severe pain after a dance exercise, the pain was so severe that I could not walk, stand, sit or turn in bed. I went to the emergency they had a ct scan and blood work everything was normal and I was given vicodin and muscle relaxers. I still feel the pain constantly I cannot stand straight,the pain is not going away. I have been having headaches consistently with the pain, my blood pressure is high. I went see a GI doctor and he suggested a colonoscopy which will be done nextweek. I cannot bear this pain any longer this month I have not seen my period, I noticed a change in my bowel movement . Also I am feeling a foul smell in my breadth. I am 47 years old this really affecting my work, i read articles about stump, ingunial hernia and would like to go back to my doctor and discuss all these, do you have any treatment regimen for me to discuss with my GI Doc. I want to get back to my normal. Doctor: Hi The pain may be due to Incisional hernia occuring in the scar site. Stump appendicitis will be diagnosed in CT scan. Another possibility is Sub acute intestinal obstruction due to adhesions . Please come back with Colonoscopy report,i will further clarify. Till then take Smooth muscle relaxant like Dicyclomine,a Pre and probiotic. As you have not seen your period,please Consult Gynaecologist also. Wish you good health Regards"
},
{
"id": 152994,
"tgt": "What are the chances of survival after treated bladder cancer?",
"src": "Patient: hi< my father in law has bladder cancer for the second time. The first time he had it removed but never followed through with treatment. Now its back and has been for at least a year maybe more. Its to the point where he screams when he urinates and can hardly get out of bed in the morning. He refuses treatment and drinks, smokes heavily. What am i up against as far as life span for him and suffering? Doctor: Hello dear. From your description it seems an advanced stage of disease. As he is refusing further treatment, his survival will be low somewhat around 4 to 6 months or so. However you can provide him best supportive care, Thanksregards"
},
{
"id": 209142,
"tgt": "Are mood swings symptoms of CHF?",
"src": "Patient: My husband was diagnosed with CHF and sleep apnea last year. He has mood swings and are getting worse by the day. He sleeps a lot, does not do anything all day. He is not on oxygen and is on a blood pressure medicine with a diuretic in it. My question is...are mood swings a symptom of CHF? Also, about 2 months ago, his lower legs became very swollen with a rash that went from his ankle to his mid calf. He was told to go to the ER but he refused. He says that they are better but my kids have found him \"soaking\" his legs in water. What could this be? Doctor: HiThanks for using healthcare magicMood swings are not symptoms of CHF. It is symptoms of depression. May be due to physical complaints, he has underline depression. In that case, he needs psychiatrist consultation and that would help to control these mood swing. Swelling of both limbs could be due to CHF, in that case, he needs physician help. In case, you need further help, you can ask.Thanks"
},
{
"id": 222026,
"tgt": "Is normal delivery possible?",
"src": "Patient: I am 37 years old. My first daughters age is delivered through LSCS due to BP. My LMP is 15.03.2010. during last visit on 29.11.2010 my doctor said my BP is 150/90 asked to be admitted on 03.12.2010 to do LSCS on 04.12.2010. For me Is normal delivery not possible? Please tell me? Doctor: Hi dear, I have gone through your question and understand your concerns.Increased blood pressure is an indication for delivery.Normal delivery is a little less feasible in your case, as cervix is not prepared for normal delivery till now.Your doctor will decide the right mode of delivery for you.Hope you found the answer helpful.Wishing you good health.RegardsDr Deepti Verma"
},
{
"id": 150481,
"tgt": "Taking tegretol for seizure disorder. CT scan showing inflammatory granuloma. What is it?",
"src": "Patient: I am praveen from India, I have a seizure disorder right back 10 years ago, after that doctors advised me to take tegretol 100 for 3 years, I had completed my course and had no compalints for 10 years.But before 2 days I had similar seizure disorder and agin got admitted in hospital, after treating one day they have discharged me and advised to take EPTOIN.Now i want to ask that what to do, to completely cure this disease. I am computer researcher and have to travel a lot alone, if that happened while i am alone then i will be in huge trouble. My CT SCAN shows it's Inflamatory gramuloma and all the rest is normal.Please advise Doctor: Hello Welcome to HCM Mostly in cases of seizure disorder there is no definitive cause can be found. You took treatment form 10 years and remained seizure free for that period then suddenly you again had two episodes. This can happen even with drugs also. You CT is showing inflammatory granuloma. It may be due to granulomatous lesion caused by Neurocysticercosis or may be chronically healed Tubercular granuloma. This may be a cause of seizures. Eptoin is right drug for control of seizures but I would advise you to get yourself checked by a Neurologist. There may be additional need for some medicine for granuloma. Thanks Dr. Seikhoo Bishnoi, MD"
},
{
"id": 131031,
"tgt": "Numbness in hands and legs when I lie down",
"src": "Patient: Hi,When I sit on chair or lie down, I feel numbness in my hands and legs. And I always feel something strange in my body, like jijijiji. Which part of my body is touched with something, it start numbness on that place in a minute. I visited lots of doctors in different countries but they are unable to find the cause. If you please help me ?Thanks,AKM Doctor: HelloI understand your concern.What you describe is called a parasthesia, an odd sensation when a particular area of your body is touched. There are multiple causes of paresthesias, including nuerological, vitamin deficiencies and even psychological.If you have seen many physicians already, I assume you had blood tests to check thyroid, B12 level, calcium level, magnesium level, iron level, as well as kidney and liver fuctions. Some liver and kidney conditions could cause parasthesias. Also some autoimmune diseases cause similar symptoms such as lupus or polymyalgia rheumatica. Next is to get electromyelograms with nerve conducting velocities of your arms or legs. This will help identify any problem with your nerves or muscles. Some types of neuropathies cause parastheias. Do you have any medical conditions or are you on any medications that could cause these symptoms?The causes as you can see are varied.You can also see a pain management specialist who can help with the parasthesisas.I hope you feel better"
},
{
"id": 60762,
"tgt": "What could cause a hard lump on the head that hurts upon pressure?",
"src": "Patient: I have a hard lump on my head that has been there for a while and it\u2019s hurts to press on it and I\u2019ve been throwing up and having really bad headaches I\u2019m really scared about what it might be I think it\u2019s a tumour or something but my nana says it probably isn\u2019t do you have any ideas of what it might be? Doctor: Hello,As per our clinical experience, an extracranial lesion will not give rise to throwing up. So if persistent throwing up is there, an intracranial extension of the tumor should be considered and CT scan of the brain must be considered needed for precise evaluation and management.Hope I have answered your query. Let me know if I can assist you further.Regards, Dr. Bhagyesh V. Patel"
},
{
"id": 3537,
"tgt": "Can i get pregnant after being treated for tuberculosis?",
"src": "Patient: Hi..........myself is Suparna,i am 32 yrs old,my height is 5 ft 3 inches,and weight is 63 kgs.on 30th August 2011 i have completed the course of Tuberculosis,I have taken Akurit 4 from 17th feb 2011 to 18th June 2011,thereafter i have started to take Rcinex 600/300.and it is completed on 30th august 2011 my clinical says that i am totally out of tuberculosis,at present i am taking Montair LC for allergic asthama,my question is in this situation shall i try for baby.if not what is perfect time for planning for baby? Doctor: hi healthcare magic user, if you have taken complete 6month course of AKT please recheck for genital tuberculosis with endometrial biopsy ,if report is negative you can try for baby."
},
{
"id": 189151,
"tgt": "Wisdom tooth, swelling in gums. Used salt water. What is going on?",
"src": "Patient: I have an almost grown wisdom tooth and now my gums are swelled and painful. I am using water and salt to ease the pain, but it comes back. I cant open my mouth fully, it hurts. Its the second day and I am having problems to swallow. Also, its hard to clean the tooth since its way back in my mouth. 6 years ago I had a wisdom tooth who grew under my last tooth which was taken out. Doctor: Hi there, Thanks for writing your query, Wisdom tooth is the last tooth to erupt in the oral cavity and often its eruption is associated with symptoms like pain and discomfort in eating , difficulty in opening mouth , swelling in the jaws and cheek bite. wisdom teeth are often tilted in position and impacted in the bone , gums overlying the tooth usually undergoes inflammation,shows pain and swelling known as pericoronitis . i would suggest you to go for symptomatic treatment at home like gargling with lukewarm saline water or antiseptic mouthwash like chlorhexidine or betadine. You may need to get an OPG radiograph done for you which will show the exact position of the tooth, consult a dentist and get the tooth extracted surgically. continue with the course of antibiotic Augmentin 625mg along with analgesics like aceclofenac two times daily for 3 to 5 days . i hope this helps , take care."
},
{
"id": 191844,
"tgt": "Suggest treatment for hypoglycemia",
"src": "Patient: I have hypoglycemia and am border line diabetic. When I drink I don't drink a lot but I sick for the next two days not being able to keep anything down. Is it due to the bloods sugar or is two day hangovers normal? And is it something worth seeking medical attention ? Doctor: 3 tsp of glucose ororange juiceor honeyor 3tsp of sugar with water is treatment not chocolates........................................................................................................................................................................................................................"
},
{
"id": 155871,
"tgt": "What is the treatment of oesophagus cancer?",
"src": "Patient: this is a sons trauma/fear of loosing his mother due to oesophagus cancer,so says the doctor here, need to know what is this at the ealiest,attaching the reports. with your vast experience could you pl tell me what this report says or may/suggest to any particular disease and the nature of treatment to be done. i want my mother to live longer and its giving me nightmares of loosing her. Doctor: I am sorry I am not able to see any reports as this is not a paid query. But don't worry about your mothers condition. Cancer esophagus has good chances of cure if it is in early stage. If there is no spread to lungs or nodes, it means it is in early stage and chances of survival is high. even with spread to regional nodes, there are good survival rates."
},
{
"id": 99374,
"tgt": "Can nebulisation with Asthalin be taken for asthmatic attack?",
"src": "Patient: nebulisation with asthalin v/s impravent or both together intermittentlyage 66, 5ft 2.5 in ht asthametic since very long. stereoids dependent for many years. but have been free of them with seroflo 250 mg bd for five yrs now. but lately problems resurfacing. Doctor: Hi,During asthmatic attack you should take asthma inhalers which are having rapid action and comparatively safer than any other form of asthma treatment like oral tablets.The medicine inside an inhaler goes straight into the airways causing bronchodilation when you breathe in. This means that you need a much smaller dose than if you were to take the medicine as a tablet or liquid by mouth. The airways and lungs are treated, but little of the medicine gets into the rest of the body leading to maximum dilatation of bonchi and bronchioles and lesser side effects.The advantage of inhalers over other forms of asthma treatment are that they are causing instant relief (bronchodilatation) by relaxing the muscle in the airways.There are very good medicines like steroids, alpha and beta receptor adrenergic agonists, anticholinergics available in market in all formulations like inhalation as a MDI or rotahaler and injectables etc which can be effective within seconds even.Thanks."
},
{
"id": 49039,
"tgt": "What does elevated liver enzymes and elevated protein in urine indicate?",
"src": "Patient: I have Multiple Sclerosis. I recently had a severe UTI, that spread to the kidney. Tests revealed protein in my urine, and blood work showed eleveated liver enzymes. An ultrasound of my abdomen was done but I don t have results. Is it likely there are 2 issues going on, or could one problem cause both abnormal results? Doctor: Hi,Thank you for posting your query.I have noted your symptoms. Urinary tract infection spreading to the kidneys are related. This condition is called pyelonephritis. You would respond to appropriate course of antibiotics.Elevated liver enzymes may not be related, unless we are considering a diagnosis of sepsis, where the infection spreads to blood and to all organs, including liver.In your case, elevated liver enzymes could be a side effect of a medication you are on. Please discuss this with your herpetologist/gastroenterologist.I hope my answer helps. Please get back if you have any follow up queries or if you require any additional information.Wishing you good health,Dr Sudhir Kumar MD (Internal Medicine), DM (Neurology)Senior Consultant NeurologistApollo Hospitals, Hyderabad, IndiaClick on this link to ask me a DIRECT QUERY: http://bit.ly/Dr-Sudhir-kumarMy BLOG: http://bestneurodoctor.blogspot.in"
},
{
"id": 4490,
"tgt": "How to achieve conception when diagnosed with Polycystic ovary syndrome (PCOS), however, have regular menstrual cycles?",
"src": "Patient: im 28 yrs of old trying many times to conceive,im married for 8 months but before i get married, i and my husband trying to conceive. as of now we trying to have baby and i know in GOD'S TIME i believe.. my trans viganal result im PCOS,but the problem is im not irregular if im not mistaken PCOS encounter irreuglar menstraution.. but in my case im not a irregural, every month my period is coming 7 days before end my period.. please suggest me what do i do.. Doctor: Hi,It is not necessary to have irregular periods in PCOD as anovulatory cycles also can be regular and in fact, ovulation is possible to some extent in PCOD. If you have sincerely tried for more than a year, please consult an infertility specialist to get further and proper evaluation. You would need a complete hormonal profile, trans-vaginal sonogram and tubal patency tests apart from a regular exam, blood counts and urinalysis. Your husband also should be evaluated. After a complete investigation, you would receive appropriate treatment. Please maintain a healthy weight and healthy diet. Hope this helps."
},
{
"id": 146577,
"tgt": "What are the symptoms of amyotrophic lateral sclerosis?",
"src": "Patient: we have been challenged to the bucket drop and Even though we are willing to do the ice bucket challenge + make a donation , we would like to know a little more about A.L.S . If I could take some of your time would you mind explaining the symptoms for us ? Doctor: At present there is no definite cure for ALS. There are anecdotal reports if improvement with stem cell therapy. Riluzol is a drug approved by FDA for its treatment but results are not satisfactory. Hope my advice will help you. Take care. Don't forget to rate me."
},
{
"id": 217262,
"tgt": "Suggest remedy to relieve pain due to scoliosis and arthritis",
"src": "Patient: I have scoliosis that I never wore a brace for in my younger years. I am now having mid to lower back pain from that and a bad mattress. I am also having right hip pain, from either bursitis or arthritis which runs in my family. I can t afford physical therapy as I am a single mom. My orthopedist doesn t treat chronic pain so I have been referred to a pain management physician. I ve taken Tramadol which didn t touch the pain, cortisone shot for hip did nothing, orthopedist gave me hydrocodone. What type of meds might pain management physician prescribe? Doctor: I went through your explanation here I suggest good enough pain killer you have all ready tried so now try pain killer along with physical therapy. I realise daily physio you are not able to afford then visit a good experience physio and based on your severity of scoliosis take exercise program. And follow the same for say 15 days and then visit once in 15 days for review. As well do use hot pack regularly on your painful area to manage pain with heat and also do apply any pain relieving ointment post use of hot pack for 15 minuts. Understand one thing scoliosis is a structural problem with spine and it will be affecting the surrounding muscles as well the nerves near by the spine. So till the time you do not try to correct it it will not be over. In very advance stage surgical correction option is also available. You can talk to hour ortho for detail view on this. Hope this was useful to you. Take care"
},
{
"id": 15177,
"tgt": "Spreading rash, itching, redness. Using benydryl spray. Rubella?",
"src": "Patient: I have a moving rash, it started on my neck, a little on my face, my ears & on my scalp on my bottom hairline. It is itchy. It progressed and moved to my chest & entire torso within a few hours but it became a solid red skin discolouration rather than a bumpy or dotted rash. As it moved, it seemed to clear up where it had been previous. I had used spray benydryl as well as benydryl syrup, as I thought it may be hives. But didnt clear up. By bedtime, the redness spread down to thighs and arms. Overnight, awoken by itchy palms and in the morning the redness had moved down to ankles & feet. Now it seems like the rash has started all over again on scalp, face, neck, arm pits & arms... So far. Upon asking around to friends & family, it sounds like it could be mono or rubella. Would you be able to help me? I am a 35 year old female. Doctor: Hi,It seems that you may be having urticaria.There may be some cause or it may be without cause..idiopathic.Stress,weather,infection,worms,some diet,dust,fume anything can cause urticaria.Blood tests may be done to find out some cause.Antihistaminics like fexofenadine or levocetirizine may be taken till itch is relieved.If embarassing , steroid in tappering dose would be helpful.You consult dermatologist for firm diagnosis and treatment.I hope you got my answer.Thanks.Dr. Ilyas Patel MD"
},
{
"id": 60024,
"tgt": "Severe stomachache, blood in stool. Test indicated fatty liver. What is the treatment?",
"src": "Patient: Hi, This is Praveen (32 ys), my recent blood test and LP test gave shocking results, Cholesterol - 244 and SGPT - 127. My Ultra sound test indicates Fatty Liver (17.2 cm), I used to be a Alcoholic, but since 8 months i have socially (i,e so far i would have had 4-5 times). i even stopped smoking since 2months. Very frequently i bleed along with my stool. I get bad stomach ache before, when ever i bleed. I keep on getting pain at the bottom of my rib position towards the right side, ie near the liver. Can you please provide me of what i should do further, and take care offf... Doctor: Hi Praveen, Thanks for your query. All the problems which you develop are due to alcohol and smoking. Stomach ache before bleed in stool is due to gastric or esophageal varices. All your symptoms are suggestive of Alcoholic fatty liver I suggest you to stop Alcohol immediately not even socially. Have a balance diet rich in green leafy vegetables & fruits. Have a complete medical checkup with a Medical Gastroenterologist. Hope this will help you. I will we available for follow up. Wish you a good health. Regards"
},
{
"id": 181247,
"tgt": "Suggest treatment for fish bone stuck in the gums",
"src": "Patient: A fish bone got stuck in my gum a couple of days ago... I was able to remove it but I don t know if I removed all because my gum is swollen and the pain is unbearable. Please help. Is it normal for it to be swollen... How long will it take for the pain to ease. What should I do? Doctor: Hi,As per your query about fish bone which got stuck into the gum and causing pain and swelling due to lodgement of particles of fish bone and proliferation of bacteria and might have a pocket formation there. I suggest you to visit a dentist for a oral examination and its effective treatment like gingival curettage and irrigation with a antiseptic solution for proper healing. For the time being you can take paracetamol tablet for having relief from pain.Hope I have answered your query. Let me know if I can assist you further.Regards, Dr. Jyoti Pandey"
},
{
"id": 205756,
"tgt": "Suggest treatment for behavioural change and depression during night time",
"src": "Patient: my sister having a weird problem (my whole family think someone has cursed or someone who hate her applied black magic on her) that she turns to a different person in a timely manner (worst in night time) with the personality of 1.suspicious anyone around her who tries to explain the reality2.Talk about all about her very past days3. want to revenge some specific people to get a justice4. her eye become widerI want to know it is some sort of neurology related problem what she is suffering from.I really appreciate if you could provide me any peice of information on this regardThanks Doctor: DearWe understand your concernsI went through your details. Though the given symptoms are not enough to diagnose the mental disorder, it seems that your daughter is troubled with schizophrenia or acute depressive disorder. She is showing the symptoms of paranoia and delusions. Immediately consult a psychiatrist and get diagnosis and treatment. Delay can be disastrous. If you still need my assistance in this regard, please use this link. http://goo.gl/aYW2pR. Please remember to describe the whole problem with full detail.Hope this answers your query. Available for further clarifications.Good luck."
},
{
"id": 145479,
"tgt": "Suggest cure for dizziness while getting up or bending down",
"src": "Patient: I fell in the shower heavily landed on my bottom. I was stunt shocked and could not do anything or speak for good five minutes. I managed to crawl and kneel with terrible pain and gradually managed to stand up. This happened about two months ago. One xray did not revealed anything and still waiting for the 2nd rray results. I have been taking codamol and recently experienced dizziness when getting up or bend down to tie laces etc. Please advice me for I would be most grateful Joe Doctor: Hi,Thanks for writing in.Since you had a fall in the shower two months back, it is important to evaluate your problems in detail. The fact that you landed heavily on your bottom and were unreponsive for 5 minutes shows that the fall was of a moderate degree. If you are experiencing a dizziness even after two months means that your complete neurological examination is required. The dizziness that you experience is due to either something in the inner ear or in the brain itself. X ray as such is a less detailed investigation and I recommend that you consult the neurologist and get a CT scan brain or MRI scan brain done to make sure that it is not a serious condition in the brain which has remained since your fall two months back. Once it is sure that you do not have any serious concern in the brain then your inner ear needs to be tested for balance mechanism. This can happen due to labrynthitis or vertigo and medications are available to treat it after confirming the diagnosis."
},
{
"id": 37402,
"tgt": "What causes swelling around stitches on forehead?",
"src": "Patient: Hi I have a friend who had gotten stitches in their forehead a few days ago due to a massive hit to the head, it began to swell last night and gradually getting worse, now causing the eyes to have purple/green bruises. Should this be of concern, they have been taking ibuprofen for the swelling, but it has seemed to worsen. Doctor: Thanks for contacting HCMI am sorry to hear about your friend who had a trauma to his head that required sutures. Today you noticed that the area is swelling and he is getting bruising around his eyes. This is to be expected. The trauma has caused tissue damage to the scalp and there is bleeding underneath the skin. This blood now is seeping down toward his eyes and giving him the bruises. This will take about 1 week to resolve. To help limit the bruising he can use ice compresses and take ibuprofen for the pain. He has nothing to worry about.Hope I answered you question. Please contact us again with your medical questions and concerns."
},
{
"id": 62875,
"tgt": "What causes painful and red lump between anus and vagina?",
"src": "Patient: Hi, i have a lump in between my anus and vagina, it is more near my anus. I recently had a yeast infection and have stared birth control pills. This lump has been there for three days now and it came about after having a bowl movemnet. It is all red and kind of hard, it doesn t have any puss and it hurts when i sit down. it also came the day i started my period. I don t know what this is and was wondering do i have to go to the emergency room or what? Doctor: Hi, dearI have gone through your question. I can understand your concern. You may have some hemorrhoid ( piles ). You should go for per rectal examination. You should drink plenty of water. take high fiber diet and avoid constipation. For pain lovL anesthetic gel like lignocain can be used. If medical treatment doesn't work then surgery is the treatment of choice. Consult your doctor and take treatment accordingly. Hope I have answered your question, if you have doubt then I will be happy to answer. Thanks for using health care magic. Wish you a very good health."
},
{
"id": 98532,
"tgt": "Is Albuterol effective in the treatment of asthma?",
"src": "Patient: I had a sudden asthma attack this eve. I am 78 years old and was enjoying a fire in our fireplace this eve. when I inhaled some saliva that went down the wrong pipe. My airway closed up on me and I could not breathe. After wheezing and struggling, I got a couple puffs of my albuterol and calmed down enough tp drink some water. It was a scary experience for my husband and me. Is my albuterol my go to med for something like this? Now I have sore ribs. Doctor: hi sir/madam,As a ayurvedic doctor would recommend you ayurvedic medicine which are best will give you good results too.please note down these following medicine:-1. Vasarishta :- 15 ml should be mixed with equal quantity of water and taken after meals twice daily.2.Haridra Khand :- One tea spoon of haridra khand should be taken with a cup of warm milk at night before going to bed.3. Talisadi Churna :- One tea spoon of talisadi churna mixed with ginger juice and honey should be taken three times in a day.4.The combination of Sitopaladi Churna+Pravala Pisthi+Rasa Sindur:-Take 2 gm of Sitopaladi Churna, 200 mg of Pravala Pisthi and 100 mg of Rasa Sindur to be taken with ghee three times in a day.These medicine doesnt have any side effect as its made by herbal drugs,you can find these medicine very useful in respiratory problems.And as per query Albuterol is also a effective medicine as per allopathic medicine is concerned if taken in right dose as prescribed.i hope i was helpful for you.have a healthy day."
},
{
"id": 44247,
"tgt": "What is the success rate of conception after first IUI? HSG, USG and prolactin level normal",
"src": "Patient: gd. evening mam, i am nitu, 30 years old, my doctor had suggested me for IUI . HSG , USG , prolactin level and my husband s sperm count is normal.I m afraid that will first time Iui be successful.please suggest me what is the sucess rate of conception after first IUI in my case. last year i had misscarriage, it was my first pregnancy.today i m first day of my period and taking ovacare forte on suggestion of doctor. THANKS Doctor: Hi, Thanks for your query. I have read your query & I understand your concerns. IUI is not hundred percent sure method for fertility. Generally it has success rate ranging from 8 to 25 %. It depends on quality of sperms during IUI, your egss etc.... Continue the medications as per your doctor's advice. Be positive & hope for a positive pregnancy. I wish you all the best. I hope I answered your query. I will be available for any followup queries you have. Regards, Dr.Mahesh Koregol IVF & Infertility Specialist."
},
{
"id": 210930,
"tgt": "What is the right medication for major depressive disorder?",
"src": "Patient: I have a diagnosis for major depressive disorder associated with psychotic features. This condition was discovered during active duty in the army. I just recently became eligible for VA benefits and is waiting to get assigned to a primary care doctor at my local VA hospital. What might be the right medication for this condition? Doctor: Hi,Major depressive disorder with psychotic features is treated with a combination of antidepressants and antipsychotics. Ideally it should be managed by a psychiatrist and I expect that your primary care doctor will refer you to one. While you are waiting for the referral, it will be better if your primary care doctor can start you on an antidepressant like Prozac, Zoloft, Lexapro, Effexor etc. Since these medicines take around 4 to 6 weeks to show their response, the sooner these are started, the better it is. Among antipsychotics, both Zyprexa and Risperdal are good options.Best wishes."
},
{
"id": 25930,
"tgt": "What could cause breath shortness?",
"src": "Patient: I am a 37 year old female. I have no idea what this is and I am really scared! Its been happening for awhile now but only in the last few days has it increased. I have heart palpatations almost constantly. It beats faster normal. Its possible I'm feeling shortness of breath and just today I've noticed a tingling and numbing sensation in my left arm (below elbow) and in my fingers. My hands are also shaky. More the left one than the right. I can't always keep my hands steady especially when I am writing or gripping something. I also have an achy back on the left side near the shoulder blade and some in and out pains below the left breast (top of the rib). Some of these pains have been on the right too but more on the left. I also experience nausea sometimes too. I am sooo scared of what this is and I am very young for this to happen to. I don't feel like this is something I should rush to the ER for. Especially since I haven't blacked out, fainted, or had worse symptoms but I am still very freaked out by this. Don't know what to do or if this is gonna go away or get better or what. If anyone has had the same thing or can ease my mind in any way, I'd appreciate any help, thx! Doctor: Hi, dearI have gone through your question. I can understand your concern. You have palpitations and shortness of breath. You may have some problems in heart or lung. Your back pain and pain in left arm and finger may be due to some neursl involvement. You should go for x ray chest, electrocardiogram and blood pressure measurements. Then you should take treatment accordingly. Don't wait for worst symptoms. Consult your as early as possible and start treatment. Hope I have answered your question, if you have doubt then I will be happy to answer. Thanks for using health care magic. Wish you a very good health."
},
{
"id": 16104,
"tgt": "Painful pus filled red rashes appeared on butt cheek. What could it be?",
"src": "Patient: The tag on my underware iritated my skin and cause a rash sort of redness. Couple days after i felt a bump in the same place (upper left buttcheek thords the crack). Its swollen red and painfull when touched. I tried to squize it and yellowish discharge came out followed by blood . But it is still swollen and i feel like it got a little bigger Doctor: Hi Kossara, Thanks for writing in. The condition you are suffering from can be intertrigo,contact dermatitis,scabies,furuncle etc. The treatment consists of oral and topical antibiotics and topical steroid. A visit to dermatologist will be required for confirmation of diagnosis and treatement. Hope this was helpful. Regards. DrSudarshan Md Dermatologist."
},
{
"id": 88716,
"tgt": "Suggest treatment for bleeding and stomach pain after drinking",
"src": "Patient: since drinking heavily on thursday evening i have had random bleeding and a large amount of stomach pain, i have felt sick to eat and suffered from wind, quite badly, i have PCOS, and wondering if this could be the cause of the random bleeding, or could this be the early signs im pregnant.? Doctor: Hello,Welcome to HCM.I am Dr Rakesh Sharma answering your query.You have not mentioned which orifice you are bleeding from, But as you have mentioned PCOS and Pregnancy, I guess it is vaginal bleeding.Vaginal bleeding may not be due to drinking.If stomach pain and bleeding with vomiting, or bleeding with stools may be because of heavy drinkingIn any case It will be good to consult your physician. Diagnosis of Pregnancy will only be possible after full history or urine test,Mean while drink lot of cold water.You can use antacid as well.Good luck."
},
{
"id": 111939,
"tgt": "Can chronic back pain cause premature ejaculation, stress and lower confidence?",
"src": "Patient: DR chandan i have been with this back problem for a period of 6years and now i feel it is limiting my sexual ability i ejaculate prematurely and i feel stressed and low confidence can these two conditions be related what can i do?could my back pain make me impotent? Doctor: Back pain can cause these symptoms. This condition is known as caudal equina syndrome. In this because of nerve compression patient lost control over urine, stool and sexual impotency. Just get an Mri as early as possible. If it shows disc prolapse than you should contact a spine surgeon."
},
{
"id": 100922,
"tgt": "What causes inflammation on lower lip with swelling in eyes?",
"src": "Patient: Hello I have a problem with my lower lip. Its starting to turn blood red and its starting from the inside. I also am struggling with swollen eyes every morning till 12 midday. I went to the doctor and they gave me allergy pills but its been a month now and no change yet. Your help would be appreciated Doctor: Hello.Thank you for asking at HCM.I went through your history and would like to know more about you like - What is your age? Since when you starter having the symptoms? Do you itch? Do you get these symptoms every day? Do you have any other allergies or any other medical conditions? Do frequently/regularly take any other medications? How the symptoms subside - by themselves or with drugs? etc.From your given history, I would suggest you as follows:1. Were I treating you, I would suggest you montelukast and levocetirizine/cetirizine regularly. 2. I would also review your medical history and medications as some common medicines like analgesics may cause such symptoms.3. Personally I would suggest you allergy testing as well as some tests for kidney function like blood urea, serum creatinine and urine examination. This is because early morning eye swelling that subsides by itself by noon may be due to some abnormality in kidney function. Please consult your doctor for this concern. Hope above suggestions will be helpful to you.Wish you the best of the health.Thank you & Regards."
},
{
"id": 108635,
"tgt": "How to treat back pain due to prolapsed disc?",
"src": "Patient: hi I have had back poblems for 2 yrs now following a car accident i had a prolapse disc but after 6 wks in hospital they said i just needed pain managment as the disc was going back in. Over the last 3 wks the pain as started getting worse again and this week end i have been in terrible pain. I woke this morning not being able to move it was as if my back had lcoked after a while i managed to get moving but the paine is very acute I have a real severe burning sensation in my right leg and i feele very sick Doctor: Dear patient Once a disc has come out of its place it cannot go back in but symptoms due to prolapse can decrease or even disappear with treatment. Since your pain has increased acutely for last three weeks there is possibility of aggravation of prolapse or new prolapse at higher level. I would advise repeat mri of lumbosacral spine to look for 1. Status of previously prolapse disc 2. Any new prolapse. Visit radiology center nearby you and get it done. Meanwhile avoid forward bending and weight lifting. Start tab diclofenac plus thiocolchicoside combination twice a day for pain relief. Visit expert spine surgeon with report."
},
{
"id": 42814,
"tgt": "How does fertomid help in infertility?",
"src": "Patient: I am a 25 , mother of a 6 year old girl. Got married last year and have been trying to concieve but to no luck.. My GP put me on fertomid but i want to know how it works and if it is successfull.How long do I have to take it before it helps. What more can I do..? Doctor: Hi,Thanks for writing to HCM .Fertomid is ovulation inducing agent. It means it helps your follicles to grow properly and mature . It also cause multiple follicles to grow. This increases the chances of conception. I would suggest you to undergo follicular study which will tell you are ovulating properly or not. If your ovulation is proper you can continue min 3 cycles of fertomid.If there is not adequate response to fertomid then you need to add fsh inj with that. But follicular study is must whenever you take fertility tablets. I would suggest you to take hcg inj or leupride after discussing with your doctor when your follicle size is more then 19mm.I wish you concieve with this . But if it fails then you need to get tubal patency test.One more thing I want to suggest is you get your hormone test done if not done yet. Like thyroid profile, serum prolactin, fsh and Lh.Hope I have been helpful .RegardsDr.Deepika Patil"
},
{
"id": 141939,
"tgt": "What causes sponge like ball right on my spine?",
"src": "Patient: I have a small sponge like ball right on my lower spine, that can move slight when i touch it , there is no pain, but I've had various back problems and pain including scoliosis, and a blood patch due to a epidural gone wrong that caused spinal fluid to leak from my brain. Doctor: Hello! Welcome to HealthcareMagic! Your symptoms could be related to different possible causes: a lipoma, a cyst, inflammation, etc.. Coming to this point, I would recommend consulting with your doctor for a physical exam and an ultrasound of the soft tissues in this region. A spine MRI in this region would give more information on the nature of this lesion. Hope you will find this information helpful! Best wishes! Dr. Aida Quka, Neurologist"
},
{
"id": 583,
"tgt": "Does irregular menstruation hinder conception?",
"src": "Patient: hi... i am seeing irregular periods since 5-6 years.... am 24years old now, got married last year and we are planning for a baby .... since three months i dint get my periods till date.i consulted gyno the doc gave me folsafe tab... can i ever concieve? am really worried... Doctor: Hi, Thanks for asking. I understand your concern. Good sperm count in male partner & menstruation with ovulation is primary need for conception. You have irregular periods. Please get your ovulation study done to acertain it. * In case it is present &the semen is normal you can try with unprotected sex around ovulation day ( can be assessed with LHsurge test).. for 6 days ftom the +ve test. *Consult gynecologist if ovulation is absent. You would need investigations &specific treatment after finding the cause.once the ovulation is established... try as explained above. Thanks"
},
{
"id": 179754,
"tgt": "Suggest treatment for rash,sore throat and abdominal pain in a child",
"src": "Patient: My 4 year old daughter developed a faint rash on face and neck which resolved after 2 days. The day the rash developed she had what looked like a mosquito bite (areas was itchy & pustule-like) on her arm. Day 2 she was feeling well. On the third day she developed sore throat, headache, fever, 3 episodes of diarrhea, and vomiting. Went to MD & strep test was neg. Day 4 she was extremely fatigued and no appetite. She would vomit in the morning only day 4, 5, & 6. She continues to complain of sore throat and abd pain. She is extremely weak. Also complaining of leg pain, doesn t want to walk. I have been able to keep her hydrated. Live in Northeast & frequently outside. Live in an area with a lot of ticks. Have not noticed any ticks on her. Doctor: Hi...by what you quote I feel everything is fitting into a viral upper respiratory tract infection with a viral induced rash. Even the body pains also fit into the description well. I think she needs some Paracetamol for pain and fever (if any) relief) and good hydration...She will get better in few days.regards - Dr. Sumanth"
},
{
"id": 180823,
"tgt": "How can severe toothache be treated?",
"src": "Patient: I had a tooth out and after 4 days it is still really aching. There is a white spot with red around it on top of where the tooth came out. I have some azitromicina tablets 500mg. Can I take them for infection if I am allergic to sulfa and tetracycline drugs. Doctor: Hello and Welcome to HEALTH CARE MAGIC, I have gone through your query.The white spot surrounded by redness that you referred to seems to be a purulus which contains pus within. It might be discharging or if not you can take a sterile needle to prick it so as to relieve the pressure within the tooth. This will also lower the intensity of Pain. The purulus suggests that your tooth is infected and most probably there is periapical lesion as well which will be seen via X-Ray. Yea you can take the medicine as the same has no affinity with the drugs you mentioned. Your Tooth might need a Root Canal in this Case as the infection can only subside for some days or week by medicine and it(pain) will recur. You might be needing an X-Ray of the involved tooth to see the extension of infection, so the dentist can see whether the tooth really can be saved with root canal or should be extracted. I suggest you to visit the Dentist and get the tooth treated. Thanks and Regards,Dr Himayun"
},
{
"id": 25762,
"tgt": "Is a pacemaker required to deal with syncope episodes?",
"src": "Patient: I was diagnosed with post-partum cardiomyopathy in May of 2011 with an ejection fracture of 30%.. In January my ejection fracture went back to normal 60% with metoprolol.. Starting in Feb started to feel dizzy and weak, having near syncope episodes. I've been in all kind of holter monitors and none of them really picked up anything besides my heart rate being really high or really low. ( also my blood pressure likes to hang out around the 80's) On the 26th of April,2012 i got an implanted Loop Monitor. I had a near syncope episode on the 28th and just took note of it. Yesterday May 3,2012 I went for a regular scheduled appointed to download the DATA off the monitor and the Tech saw that my heart had stopped twice, one being 11 seconds and the other one 18 seconds long. This happened around the time that I had the near syncope episode. My cardiologist scheduled me for a pacemaker implant that same day. However, right before I got to the hospital my cardiologist called me and told me that he would like to wait on the pacemaker. I dont know why he would want to wait when they have record that my heart had stopped for 18 seconds. It is still puzzling me and it is really frustrating. I am 24 years old, Do you Doc's have any idea of what might have changed his mind? Doctor: Hello ,Now considering that there was a pause for so long , without doubt It's an indication of pacemaker . Now because As you said that the function have returned to normal , Generally there should not be such episodes . One thing that explains is ongoing inflammation of the heart muscle , but its temporary and may settle down by itself . Lastly I would just like to point out , trust me at 24 years of age if we can avoid a pacemaker it's best , because it stays with you for life and requires care . However if still in future there is evidence of such pauses , it would be safer to put a defibrillator. Let me know if any clarification needed . Regards Dr Priyank Mody"
},
{
"id": 174190,
"tgt": "How to cure the large red sore on the roof of the mouth ?",
"src": "Patient: my son is 1 next month. He has a large red sore on the roof of his mouth no white or yellow though. he seems slightly off his food but will eat something if persuaded enough. i'm not sure to the cause of this or whether to see my doctor or allow the red spot to heal on its own Doctor: Hi,Thank you for asking question on health care magic.Probably he injured himself by putting a spoon or accidental injury when you are feeding.Mouth ulcers heal rapidly.Only hot milk or food, spicy food he can not tolerate.Give cold and sweetened food or drink.You may locally apply Zytee liquid or dentogel once or twice daily.If it does not heal within a week you contact your doctor.Hope this answer will serve your purposePlease feel free to ask any more queries if requiredTake careDr.M.V.Subrahmanyam MD;DCHAssociate professor of pediatrics"
},
{
"id": 172180,
"tgt": "What causes warmness in the body of a kid?",
"src": "Patient: My almost 3 year old daughter woke up this morning, hot to touch, but I am getting no fever reading on either thermometer we've tried. She doesn't appear to have any other symptoms and is acting fairly normal....what could be causing her to feel so warm to the touch? Doctor: Hi dear, I understand your concern. But it can be due heat wave,overheating, hot clothes. It is usual situation, children have better blood circulation and their skin is hotDon't worry too much.Wishing you and your family good health"
},
{
"id": 53800,
"tgt": "What causes elevated levels of alanine transaminase in blood test?",
"src": "Patient: Hi, My name is Saurabh.I have gone through complete blood test.In the report i have found that Alanine transaminase is increased.. it should be in between 0 to 37..but it is 52.... what is the reason? and what should i have to do to come in range... please suggest. Doctor: Hi.Thanks for posting query at HCM.NO need to worry.retest liver enzymes after 3 to 6 months.Usually ALT or AST values higher than \"two times the upper normal limit\", is considered abnormal ( in some countries, ALT or AST values of more than 100 are considered abnormal). Value of AST or ALT greater than 85 or above maybe investigated further.hope to answer your concern.regardsDr Tayyab Malik"
},
{
"id": 217994,
"tgt": "What is the remedy for knee pain after an injury?",
"src": "Patient: I hit the dead center of my knee cap about as hard as you could imagine hitting it against the edge of a metal door frame at work. I can barely walk, my knee is swollen and hot to touch and pain is unbearable in my knee when I bend or unbend my leg.What could I have done? Doctor: Dear Friend.. I am Dr Anshul Varshney. I understand your concern.I would advise you following :1. A pain killer with anti inflammatory action like Tab Zynase D twice a day for 5 days.2. Volini gel locally 3-4 times a day3. Hot fomentation.I suppose you will get relief by this.But an Xray / MRI is advisable to see for any fracture / ligament tear.If you have any further query, you may please ask me.Stay Healthy"
},
{
"id": 115674,
"tgt": "Suggest treatment for elevated SGPT levels",
"src": "Patient: hi im jeric 26 yrs old im planing yo go abroad but sadly im pending in my medical result because of my sgpt 120 sgot 54, now im starting go on diet because after 2weeks i need a low sgpt to make me fit to work. please help me, wat should i do? please to decrease my sgpt thankyou Doctor: Hi, dear. I have gone through your question. I can understand your concern. Your sgpt level is really very high. Sgpt is a liver enzyme. Its level increase in any liver damage. It may be due to viral hepatitis, drugs, alcohol or toxins. You should go for complete liver function test. Find out the cause of that and take treatment accordingly. Hope I have answered your question, if you have doubt then I will be happy to answer. Thanks for using health care magic. Wish you a very good health."
},
{
"id": 137306,
"tgt": "Suggest a remedy for painful tibialis of the leg",
"src": "Patient: I am experiencing recent and painful Tibialis anterior cramping during the night. It has been particularly pronounces the last two nights. It occurs in both legs and can last for several minutes. However, there is no swelling or discoloration. During the day there is some very mild tenderness but no pain, fatigue or cramping. I have been taking calcium, and multivitamins,massage my feet and lower legs before bedtime.Any suggestions? Doctor: rest from sport actioties as running, swimming, cycling, walking quickly. try stretching your lower limbs (bend the leg over the thigh bringing your heels next to your buttocks"
},
{
"id": 129054,
"tgt": "What is the treatment for a lump in the arm?",
"src": "Patient: Hi i shut my arm in a door at work and had instant swelling on my forearm double the size, I had an X-ray and found nothing. Now swelling has gone down I can feel a hard lump/ridge just before my wrist and my little finger and ring finger are nunb, tingly and purple. Doctor: Hello,Thank you for using healthcaremagic.I read your question and understood your concern.I think you have injured the ulnar nerve and/or ulnar arthery which caused hematoma (lump) in the forarm and changes in the color of 2 fingers.I suggest you see the vascular surgeon and neurologist for more examinations such as Doppler ultrasound and EMG.Dr. Selmani"
},
{
"id": 118100,
"tgt": "What is the remedy for dizziness with high blood pressure?",
"src": "Patient: Well, occasionally I have a dizzy spell. Nothing major but concerning. I am 5 9 , 175 lbs,, 62 years old and fairly fit and active. It occurs out of the blue and not necessarily from quickly getting up. I do suffer from high blood pressure but take Coversyl (8 mg daily). Doctor: First thing I need to know since when you are taking that medicine for your high BP. Because with coversyl initially you may get sudden dizziness on getting up but it will subsequently get corrected on long term use. Another explanation of your symptoms may be due to some underlying heart block, for which you need to undergo at least an ECG and if required detail cardiac investigations. If I were your treating physician I would have suggested you to consult a cardiologist for further work up. Hope I have answered your queries."
},
{
"id": 15322,
"tgt": "Rashes and blotches on body, itchy. Prescribed plaquenil and prednisone. Done chest X Ray. What is going on?",
"src": "Patient: I have recently (about 2 weeks ago) been prescribed placquinil (two 200 mg. tablets per day). Since then, I have broken out in an all over the body rash with huge blotches. It has spread until there is no part of my body untouched - from toes to scalp. It is itching like CRAZY! My entire torso looks like one \"port wine birthmark\", for lack of a better way to picture it. My primary has done bloodwork, chest x-ray, given me prednisone (two 5 mg. tablets per day to begin; now, 5 days of 30 mg., 5 days of 20 mg. and finally then, back to my usual 10 mg per day. This concurs with my rheumatologist who prescribed the placquenil and also prednisone. Diagnosis: inflamatory arthritis. I will be seeing a dematologist this weeke. Could the ithe plaquenil be the culprit of this horrific body rash? General opinion seems to be that I'm having some sort of drug interaction, but so far no one's mentioned this drug. Thank you! Doctor: Hi, As you are mentioning, you have started plaquenil 2 weeks ago and simultaneously, you have noticed a \"huge\" body rash?So, according to your saying, your body rash has started with plaquenil (new) use. Did you mention this to your primary?Therefore, it is obvious that you have an allergic reaction to such drug. Then intake of prednisone has stopped other problems. So, stop taking plaquenil, and tell your doctor (rheumatologist/dermatologist) how your rash started and change medication. You can take some anti-itching creams to alleviate the itching and take some warm-to-cold bath or applying cold clothes to itching area.Hope it helped!"
},
{
"id": 35045,
"tgt": "Suggest treatment for fungal infection on the toe nail",
"src": "Patient: I'm 40 year old male about 175 pounds and I don't have diabetes or any other health problem that I'm aware of.I've had toenail fungus on right toe for about two years now. About two months ago, I started soaking it bleach/water for about 30 min a day. The appearance has changed dramatically. Now the skin around the nail is red and scabby and about two days ago I can see a clear liquid coming from where the nail and the skin meet. I also have a slight fever. I don't have any money to see a doctor, is there something I can do without seeking medical help, to see if it will go away on it's own. Doctor: Hello,Welcome to HCM,Seems to be a case of fungal infection worsened by secondary bacterial infection leading on to pain and swelling.I would advice to maintain strict control of blood sugar which is the most likely cause for recurrent fungal infectionIf i was your treating doctor i would have asked you to take an oral antibiotic for a week to resolve the secondary bacterial infection followed by application of a topical antifungal cream either terbinaine or clotrimazole twice daily for a month.An oral antifungal is often required in case of recurrent fungal infections. My choice of drug is oral fluconazole 150 mg once/week for 4-6 weeks.An antifungal dusting powder also helps to prevent recurrences once the infection is cured.An oral antihistamine once daily would help you deal with the itching associated with fungal infection.Thank you."
},
{
"id": 93292,
"tgt": "Is it period when I spot pink first and then brown?",
"src": "Patient: Hi I have a problem I need help with. I m not due for my period for another two weeks but two days ago I woke up very early and when I went to the bathroom and wiped it was a little pinkish so I didn t think it was my period. I went back to sleep and when I woke up there was nothing in my panties. I put a pad on just in case and later still no blood . Later on that night it turned to a brown color. It doesn t come on my pad but when I wipe its there. I have little cramps not like a period though. I need to know what this means please help. Doctor: Hi ! You have not mentioned about your age and also if you are sexually active or not. The symptoms what you have described could be due to pelvic inflammatory disease. Also you have to exclude pregnancy in case you are sexually active. It is not possible to confirm these without getting you examined by your gynaecologist. If there is recurrence of your symptoms with any other new development, you should take appointment with her for a clinical examination and further needful. Good luck."
},
{
"id": 158015,
"tgt": "Had total hysterectomy due to endometriosis. Had been getting iron infusions. Severe back and shoulder pain, fatigue, bone pain. Cancer?",
"src": "Patient: I am 47,female, and have had rare health issues, of late. I had a total hysterectomy at 26 due to endometriosis . I have never had a history of bladder or kidney infections but have had 2 in the past year. I have been severely anemic, over the past few years that have required me to have regular iron infusions every few months. I recently found out that my ESR rate is over 125. Cancer has been brought up. Dr. s are trying to get me to MAYO Clinic. I also have severe back/ shoulder pain and am very fatigued, bone pain in my legs and lots of numbness. What do you think ? What type of cancer could it be ? Doctor: Such high values of ESR are known to occur in multiple myeloma (a type of blood cancer) and in some autoimmune diseases (like rheumatoid arthritis). Your symptoms can be explained by both, although they are more in favour of autoimmune diseases. You need to undergo a number of tests to rule out multiple myeloma including serum protein electrophoresis, urine Bence-Jones proteins and bone scan."
},
{
"id": 196106,
"tgt": "What causes the lack of erection within 3 hours?",
"src": "Patient: Hi, I'm Manish. I'm 32 yrs old with 5'10'' height and 70 kgs weight. I use to mastrubate and ejaculate nearly 2-4 times weekly. after ejaculation, I not able to get erection atleast for next 3 hrs. Does I'm having any defiency? Does it require any serious medical attention? Please help me. Doctor: hi.welcome to healthcare magic.i understand your problem.my opinion is that,1.avoid stress.2.have adequate sleep of about 8 hours a day.3.avoid alcohol, cigarette smoking.(if you have these habits)4.take nutritious foods.5.dont masturbate more than twice in a week.6.himcolin gel application before intercourse will help you to get good erection.thank you.hope my answer will help you."
},
{
"id": 91592,
"tgt": "What could cause severe stomach pain after a kidney transplant?",
"src": "Patient: Hi, I have been to hospital 3 times due to severe severe pain and stomach and they said it was a blockage and constipation. They gave me stuff to try, but none worked or is working, I had a kidney transplant and take a lot of meds for that and a fused neck. Anyway. I haven't gone in almost a month and before that just very little. I can't stand the pain anymore. It hurts so bad in low left stomach, side and low back hip area???? What do I do, because doctors and hospital keep giving me stuff to do at home which I've done with no luck? What do I do? Doctor: Hi. In such situations with the history of transplant I would advise you take a second opinion of another Doctor Specialist. I hope you have undergone ultrasonography / CT scan of abdomen to see any finding. As your pain is in left lower stomach, side and low back, i would think about colitis. Colonoscopy may help..."
},
{
"id": 113897,
"tgt": "Can a person having problems with the spine get paralyzed due to excessive workout ?",
"src": "Patient: Sir, MY father used to have problems with his spine and had suffered.Then as he progressed and became better he started joging a lot on a hard surface in a hilly terrain.Now he has started to limp for a year has problems with his right hand as well.It cannot move and seems paralytical.Is is a neurological problem?? Doctor: hi welcome to hcm you need to do an MRI brain to rule out a paralysis problem if the MRI is clear then it might be because of the spine problem which can also be confirmed via MRI first find out the exact cause and then proceed because the treatment differs plus if he has associated urinary problems, problems with his sensation etc it might be a neuro case"
},
{
"id": 206952,
"tgt": "Suggest treatment for nervous disorder problem",
"src": "Patient: Dear sir, Good Evening. Iam 26 years old, suffering from cold for last few years. Presently iam consuming Frisium 10Mg and Eption tablets to avoid nervous disorder. From Childhood I had a problem of Fitz if i intake some cold food items. Now iam alright, but still i feel i have nervous disorder problem. so can you please advise me what kind of treatment i have to take now. Doctor: HiI understand your concern.i do not understand about nervous disorder will you elaborate properly.Eptoin and frisium are the remedy for convulsion or fits. You mean to sat you have fit in spite of taking this medicines then need to consult your treating doctor.There are many other anti convulsion medication which will help you.If you have nervous disorder like nervousness and anxiety then medicines like anxiolytic and benzodiazapine will help.Require proper evaluation.Still have question then feel free to ask.thank you"
},
{
"id": 153501,
"tgt": "What does feces in the vagina of a cancer patient indicate?",
"src": "Patient: In a cancer patient with stage IV ovarian cancer that has recurred in the colon and has matasticized in the pelvic lymph nodes and liver. She has recently begun radiation, which is the last procedure that the doctors plans to try. Today she found feces in the vagina. What might this indicate? How urgent would it be to contact her doctor? Doctor: Hi,Thanks for writing in.Cancer in ovary that has spread to liver is stage 4 disease; it has limited treatment options and if chemotherapy does not show response then radiation therapy is used to reduce the symptoms.Feces in the vagina might be due to adverse effect of cancer causing a rectovaginal fistula. This is an abnormal connection between the colon wall and vagina and leads to feces matter coming out of vagina. Patient might be considered for surgical repair if general condition permits. Please consult her doctor within a day. Please do not worry."
},
{
"id": 58294,
"tgt": "AST, GPT, GGT levels elevated. Have taken many supplements, weight loss, energy pills, vitamin E, done light exercise. Reason?",
"src": "Patient: Dear Doctor , Appreciate your feedback on this medical tests I got, I did the test twice already on a two week interval, but the results seem to be consistently elevated. AST - 49 and 42 respectively GPT - 102 and 104 respectively GGT - 310 and 324 respectively - I am not an alcohol drinker, but maybe twice or trice only in a year when there are occassions - 6 monhts ago my HBsAG is .27 - Hepatitis AIgG (HNIgG) is 10.35 - Hepatitis A HBS Antibodies is 124.54 - though I have a history of taking too many supplements in the last 5-7 years e.g. weight loss and energy pills, some grapeseed supplement and vitamin E, and takes some pain reliver medicines also. - My height is 172 cm and weigh is 88 Kilos. a normal active lifestyle and i normally go to gym to do some light exercises and jogging Thanks for your help Doctor: HIThank for asking to HCMThe over all report is suggestive of liver damage not bad or may not be damage such investigation always need to be correlate with clinical conditions, without any specific reason if you done test then there could be some changes which may not show any clinical sign and symptoms, and does not required any medicine, have nice day."
},
{
"id": 56923,
"tgt": "What to do for fatty liver?",
"src": "Patient: Hiiiiiiiii Doctor. I am Sandeep From Jagraon,Punjab (India) I have Fatty liver. I am getting cure from Dr. Ajit Sood from DMCH Ludhiana. I am not feeling good. I am in too much depression. My age is 24. Can u give me some positive point ?? Doctor: Hi, How are you? My name is Dr Suresh Raghavaiah. I am a Liver and pancreas specialist and I hope to answer your question today. I am so sorry to hear that you are feeling depressed about your condition. But there is no need to be, as it is very easy to treat. Fatty liver can be caused by excessive alcohol intake (Alcoholic steato hepatitis, ASH) or due to excessive fatty food intake and sedentary life style (Non-alcoholic steato hepatitis, NASH). ASH is usually the beginning stages of alcohol induced liver damage and is completely reversible if the patient refrains from alcohol. NASH is being much more commonly diagnosed currently owing to poor dietary habits and sedentary life styles of people. High caloric intake, high fatty food intake and poor exercise regimens lead to deposition of fat goblets in the liver tissue leading to fatty liver (which shows up as increased echo texture on USG). This is also completely reversible in the initial stages if caloric intake is limited and exercise regimens are adhered to. If left un-checked, patients usually become obese, diabetic and in final stages may lead to cirrhosis of the liver which will necessitate a liver transplant.I would suggest a healthy diet and exercise regimen. Although there is no time limit on when to get a repeat scan, I would suggest one, once you have succeeded in losing some weight.Hope this helped and hope you start to feel better. Please do not hesitate to contact me for further details.Have a great day.Dr Suresh Raghavaiah"
},
{
"id": 36381,
"tgt": "Suggest treatment for spider bite",
"src": "Patient: my husband is on bp medication 1 pill a day (hydrocholrothiazide 25mg) also takes naproxen daily....he went to urgent care 2 weeks ago tomorrow for a place on his inside upper thigh....they said it was a spider bite and gave him antibiotic...he took all of it threw bottle away not sure what it was but was 2 pill twice a day ....the area on his leg is gone and looks normal...but for past few mornings when we wake up his pillow is SOAKING wet with sweat (temp in house on 72)... today his left side of neck and head are hurting him... can hardly turn his head...no arm, hand, or shoulder pain though... any ideas I am trying to get him to go to emergency room... but is stubborn but I am very worried ...he is not a complainer at all... also says he \"just doesn't feel right\" but not dizzy or anything loike that?? Doctor: HelloWelcome to HCM,Spider bites are actually rare occurrences and bites from most spiders cause local redness, irritation and pain that usually can be treated at home.These local reactions usually resolve without treatment over a period of 7-10 days.I would suggest you to followWash the site of the spider bite well with soap and water.Apply a cool compress or ice pack over the spider bite location.Over the counter pain relievers may be used to relieve symptoms.Thank you."
},
{
"id": 104034,
"tgt": "Having asthma. Chest x ray showed large spots by the side of chest. What could it be?",
"src": "Patient: My family and I are Americans living in China. My son has asthma . He has been sick since we arrived here. We took him to a big hospital in Beijing who gave us more medication for his asthma. He had a coughing episode and we rushed him to another hospital. They took a chest xray and said he has pneumonia . I am concerned because there were large spots on his x ray, they even looked at his clothes thinking it was metal. I am concerned what they were.We are doing daily IV medication and antibiotics . The spots were by his heart on the right side. Do you know what they could be? Doctor: the uncontrolled asthma COULD LEAD YOU TO CHEST INFECTIONS THE OILS AND OTHER THINGS OF CHINESE MATERIALS MAY NOT BE TOLERATED BY THE CHILD AND IT BECOMES UNCONTROLLED ASTHA EVEN YOU ARE USING MEDICINES FOR LONG YOU HAVE TO SUPRESS INFECTION MAY BE THE ANTIBIOTICS GIVEN IV CONTROL IT AND THE WHITE SPOTS IN CHEST ARE INFECTED AREAS THEN AFTER CONTROLLING INFECTION YOU HAVE TO RETITRATE THE DOSAGES OF ASTHMA MEDICINES MAY WE HANGE OR TITRATE DOSES DEPWENDING ON THE POSION AFTYER CLEARING OF CHES INFECTION NORMALLY IT TAKES 3 WK WITH ANTIBIOTICS TO DISOLVE THE INFECTIVE OPACITY IN LUNGS"
},
{
"id": 156785,
"tgt": "What could cause concurrence of breast cancer and abnormality in PAP smear, also have undergone hysterectomy?",
"src": "Patient: Yes. I had an abnormal pap smear and it states in the pathology report that there is a presence of ASC-US atypical squamous cells-undetermined significance. I have had a hysterectomy and have just been diagnosed with Breast Cancer. Is this just a fluke that this would happen simutanously? Doctor: it is usually coincidence and breast and cervical cancer doesn't occur together that commonly than seen breast with ovary or endometrial cancer as these are hormone related. and cervical cancer is viral infection related."
},
{
"id": 3925,
"tgt": "Does drinking alcohol before ovulation affect chances of pregnancy?",
"src": "Patient: My husband and I are trying to conceive . According to my calendar I still have 15 days till my expected period (I should be ovulating tomorrow)... I do realize that my body probably does not behave like clockwork though so I can not be certain. My question is.. is it okay to still have a few drinks? We are invited to a social occasion and It would be nice.. but if there is a chance that it could harm things I don't want to do that. I heard that embryos don't share off the mother till later.. but also not sure if thats true. Ive heard a lot of conflicting info on this subject. Thank you Doctor: if u want to become pregnent dont drink..it will adversly effect u r health most importently if u get conceived there is so much effect on fetus especially in first 3 months..its not like u can drink after 3 months.babies body parts most start forming in first 3 months it will bbe retarted by drinking alcohole..so who ever is planing for pregnecy has to avoid alcohol..all the best get a good baby..thanq"
},
{
"id": 198406,
"tgt": "Experiencing erection problem during intercourse",
"src": "Patient: hello Doctor.. My age is 27 and I weighs 70 kgs.. I was doing masturbation since 4-5 years. some days back, I tried my first sex, But i felt erection problem.I tried for many times,but i reached to my climax only 2 times for 10..... Is there any problem or issue like ED?My biggest worry is that i was not able to penetrate completely to my partner for long, i was loosing hardness and erection. Please do suggest me something..and help me. Doctor: HelloThanks for query .Your problem of Erectile Dysfunction is due to habitual and frequent masturbation that you been indulged with since many years .In absence of any major health issue the cause of ED at the young age is mind related and due to what is termed as Performance anxiety .Following measure will help you to boost up your confidence and getting good erection and delay ejaculation.1) Practice regular exercise for 45 minutes followed by meditation for 1/2 an hour in the morning.2) Take high protein diet rich in vegetables and fruits and Vitamin A,C,D,E.and Zinc3)Take anti oxidants like Almonds 5-6 everyday..4) Avoid alcohol and smokingTaking Sildenafil (Viagra)as on demand will help to get good hard sustainable erection and to have enjoyable sex.Get the prescription and instruction as regards dose from your family Physician .Dr.Patil"
},
{
"id": 79954,
"tgt": "Suggest treatment for tuberculosis",
"src": "Patient: I AM 74 YEARS OLD. I TESTED POSITIVE ON MMY TINE TEST WHEN I WAS 12 YEARSOL, PRE-BASKETBALL, AND AGE 22 YEARS OLD AT MILITARY PRE-ENLISTMENT PHYSICAL. I REACTED POORLY TO INH THERAPY (MY LIVER PANEL WENT FROM 23 TO 211. I AM NOW ON RIFAMPIN. WHAT SHOULD BE MY TO FIVE CONCERNS? Doctor: For treatment of tuberculosis start anti-tuberculous medicines which are sensitive as early as possible after an expert advice. You have to take some multivitamins like metilda-forte or meganeuron-plus once daily preferably in afternoon along with some protein supplement like protienax powder with milk to improve your immune power. Don't worry much just take your medicines regularly and properly as suggested by your doctor."
},
{
"id": 144933,
"tgt": "What causes pain in temporal region and neck muscles?",
"src": "Patient: Hi I am a diabetic type one and am now treated since last September for anxiety and panic attacks. I take pro clonazepam daily for this. Since now four days I feel a pain spreading from my left temporal region to my teeth, neck muscles, front lobe and behind my eyes. The pain is mostly on the left side of my head and I rate it as a 5 out of 10. It s growing every day though. Today the pain has spread a lot behind my nose and my right temporal region has started to hurt. I am concerned about a possible seizure... Also I don t know if it s important but since four months I take like half and sometimes less the posology of my clonazepam, without any panic I can t control. Since two weeks though I have suffered light but non stop hyperventilating. Doctor: Hello!Thank you for asking on HCM!Your symptoms do not seem to be related to any possible seizures. The troubles with breathing may be caused by anxiety (or a low control of anxiety from an insufficient dose of clonazepam). I would like to explain that there are different disorders, which can cause pain in this region : 1- A temporal-mandibular junction arthritis 2- A temporal arteritis 3- An infection of the soft tissues or the sinusesI recommend you to consult with the GP for a careful physical examination, a routine blood test, inflammation tests, thyroid hormone levels, kidney and liver function tests and blood pressure monitoring. A chest x ray may be necessary to rule out other possible causes of hyperventilation.You may need to discuss with your attending physician on a possible increase in the doses of clonazepam. A brain CT scan may be necessary to exclude a possible sinusitis or a brain lesion. I would also advise you to consult with your dentist, for a better examination of the temporal-mandibular junction. Hope to have been helpful!Greetings! Dr. Iliri"
},
{
"id": 12290,
"tgt": "How can psoriasis be treated without side effects?",
"src": "Patient: My Name is Ravindran from Bangalore. I have been suffering from Psoriasis for the past 10 years.Been taking Ayurveda treatment for more than 7 years which would not work. I am at present taking alopathy (Past 3 Yerars)but it starts again when I discontinue the medicines.And I am worried of the side effects. I'm 48 Years old. Please advice. Doctor: thanks for asking via HCM,Psoriasis is a chronic relapsing and autoimmune disease. You may be having scaly plaques on various parts of the body,scaly lesions on the scalp. Exact cause is not known. Genetic tendency, autoimmunitity,stress,dry wheather...etc may be responsible for precipitation and exacerbation of psoriasis. You consult dermatologist for firm diagnosis and treatment.I usually recommend cap acitretin 10 mg twice a day along with methotraxate 15 mg weekly in three devided doses 12 hourly...for psoriasis.This may be taken for long time till good response. Blood tests may be done to monitor the treatment. For itching, you may take antihistamnics like levocetirizin...Avoid steroid ,as there may be severe flare up after stopping it.you may apply mild steroid cream or calcipotriol oint on the lesions on the psoriatic lesions. She may apply moisturiser to improve the skin texture. you .should avoid soap bath for few weeks..hope this will clear your facts.regards"
},
{
"id": 104886,
"tgt": "Biopsy of the neck lymph nodes showing sarcoidosis. Pulmonary function test showing asthma. No effect with the steroids. Need further tests?",
"src": "Patient: I recently had a biopsy of my neck lymph nodes that showed sarcoidosis . Had a pulmonary function test which definitely shows asthma. Been on steriods for one week, noticing improvement in breathing but still wheezing and need to use inhaler three to four times a day. I still have watery eyes, nasal congestion, and lymph nodes are still swollen. I was wondering if there is any questions that I should be asking my doctor. Should I be getting any other tests? Pulmonary doctor doesn t feel the lymph nodes in my lungs should be biopsied at this time. Doctor: Hi sixguysandagirl, Sarcoidosis being a chronic disease it cannot be cured in days. A week of steroid treatment cannot cure the symptoms of sarcoidosis. Sometimes it wont be cured with steroid treatment alone needing more powerful immunosuppressive drugs but sometimes it may be cured spontaneously also, so continue taking the drugs....Since the diagnosis has been confirmed and pulmonary function tests done no need to take lung biopsy.. If at all you want, you can see complete blood count, electrolytes to look for associated comorbidities and liver function tests to see for liver involvement... With regards, Dr. Parthipan"
},
{
"id": 178222,
"tgt": "Suggest medication for diarrhea & vomiting with foul odour",
"src": "Patient: My son is 11months and I m very concirned, he is teething but I don t think that s the case with the way his body has been acting. About a month ago, me and my 3 sons (ranging from 4yrs, 2yrs and 11months) were sick for an entire day from who knows what. We all went to the ER but all they could guess was a 24hr bug. My son (the 11month old) has had yellowed diarrhea that smells like vommit and continues to pass gas and burp and this has been happening on and off recently he hasn t had any fevers or anything unusual like that but the only time I ve ever even been arround anyone with egg burps was my grandfather and I don t remember why he got them either. Alittle insight would be helpful or some home remedies to make my little guy feel better. Thank you Doctor: HiThanks for writing to us.Diarrhoea is usually viral.I usually recommend ORS and zincEnsure he passes urine atleast 6 times daily.If not improving take him to nearby doctorWishing good healthRegardsDR ARUN"
},
{
"id": 112687,
"tgt": "Have lower back pain. Can hardly walk. Pressure on head, nausea, cold sweats and lost vision for 3 minutes",
"src": "Patient: Hi, I'm a 37 year old female weight 130 height 5'3. I woke up with a bad lower back pain could hardly walk, felt pressure in my head i felt like my head was on fire felt nauseas had cold sweats and lost my vision on both eyes for like 3 minutes, my vision slowly came back. I went to the emergency room got no answers.Thank you Doctor: Hi, it appears to be due to fever, or hypertension, both can cause the above symptoms. So i advise you to consult a physician for diagnosis and treatment. I to my patients with such symptoms prescribe cifixime, analgesic, omeprazole, anddomperidon, if hypertension is there pills for it . Thank you."
},
{
"id": 11162,
"tgt": "Will Folimost tablet and minoxidil solution help in hair growth?",
"src": "Patient: hi! doctor my name is siddharth ,19years old i have been suffering from hair lose from 2 years, i had once taken follihair tablet for 3 months and Minoxidil solutio but now after 1 year again i told to have folimost tablet and minoxidil solution please recomend what i should do.? Doctor: Hello Siddharth,Thank you for posting on HCM.I appreciate your concern regarding hair fall. Hair fall is usually ascribed to multiple factors like diet and nutrition, hormones, stress, cosmetic products etc.From your description, it seems you might be having Male pattern hair loss or Androgenteic alopecia, which is due to male hormone, testosterone. If i were your dermatologist/trichologist, i would like to take through history including family history, history of recent major trauma/illness/stress/medications etc, and recommend some basic investigations like CBC,blood sugar, Thyroid function test.I would suggest you a course of oral tablets containing biotin and other essential vitamins and minerals for minimum 3-6 months. (Follihair or Folimost) Also, would put you on solution containing 10% minoxidil once a day and a hair serum containing peptides for hair growth at night.(Q sera etc). Finateride (Finax) can be used in selected patients after thorough counselling and checking your liver function. Finasteride, inhibits metabolism of testosterone at hair root level. It sometime leads to embarrassing side effects like impotency, decrease in semen volume but which are totally reversible on stopping the drug.I would advise you use of gentle shampoo and conditioner on regular basis and use of coconut oil twice a week.Avoid combing in wet hair and let them dry by wrapping in towel. Avoid blow dryers and hair-color/dyes. Take plenty of fresh fruits and vegetables in your diet and try to de-stress your routine life.Hope your queries are resolved and wish you best of health.Thank youDr Hardik Pitroda"
},
{
"id": 58121,
"tgt": "Hypertension, high sugar level, thyroid and cholesterol after gall bladder surgery. How can I improve?",
"src": "Patient: i had sudden gall bladder removal surgery 2 years back after suffering from it for a month and having jaundice . I had to go for surgery with good faith and thought trouble might be over. But i think unusual real troubles started afterwards . i suddenly developed hyper tension , higher sugar , thyroid and higher cholesterol . I am 29 years old and i did not have these troubles and i was completely healthy before gall bladder removal. i have asked numerous doctors and no one have any clue. they said you need to be treated for metabolic syndrome . Any Doctor in forum can comment that would be great. I think if i get on thyroid treatment all these other symptoms might go away. I was on statin and i had terrible side effects. After having bad experience i had started natural remedies like no sugar diet/less salt , omega 3 acids, olive oil, oats , flex seeds, an hour of gym . after doing all these although my blood pressure is 130/86 and not coming down. I am going for further checkup and i dont have any hopes after not having any results. Any one know how i can improve on this I am taking all steps but no results so really hart broken. Doctor: Hello & welcome to HCM,I had gone through the case, I want to know your family history of diabetes, hypertension , thyroid problem. Do you have any symptoms and sign of these problem right now like headache , obesity, rising of blood sugar etc. I will advice you to do pranayam( ANULOM-BELOM) exercise for 15 minute daily for 1 week and then major your blood pressure, definitely your problem will resolve. If not then you have to consult the doctor for hypertension, who can advice you medicine, correct diet pattern and more exercise for it. Hope so your problem will resolve.Thanks & RegardsDr Soni VermaHomoeopathic Physician & Life style counselor."
},
{
"id": 139077,
"tgt": "What are the episode of severe pain in my shoulder blade?",
"src": "Patient: I have had a bad spasm from ledt shoulder blade across my back the pain is quite bad and preceeded by diarrhea and then afterwards constant wind. It comea on quickoy with no warning Im 31 yrs old and 5 ft 5. It has happened twice now and the pain is worse this time Doctor: Hi, your pain is of utmost concern looking your age. if you have sprained your shoulder, then what you need now is some rest, heat fomentation, and analgesics. if you do not find relief, in that case you will need through examination and a battery of tests to come to a conclusion.I hope that helps."
},
{
"id": 130262,
"tgt": "Have I fractured my pelvis bone?",
"src": "Patient: I fell on the stairs and hurt my pelvis bone. Ten minutes later I vomited ( I ate 40 minutes before falling) It hurts to sit down, to get up from sitting down and it hurts if I try to bend down to get something off of the floor. Did I fracture something? Doctor: hello,Yes there are chances of fracture either to your pelvic bone or to the lower back bone(includes tail bone)If is advisable together a X-ray done and till then avoid painfull movements.Hope this helps youThank you"
},
{
"id": 224806,
"tgt": "Is it safe to take emergency contraceptive pills or regular birth control pills?",
"src": "Patient: I'm currently taking 100mg of topiramate once a day and know I can't take any long-term birth control pills, or at least I'll have to make changes before starting anything. So my question is; is it safe to take the emergency contraceptive pills, or would it have the same effects as taking regular birth control pills? Doctor: Hello and welcome to HCM,\u00a0\u00a0\u00a0\u00a0\u00a0I am Dr Nilajkumar a consultant gynecologist and I will be helping you in your queries. I understand the issue. Emergency contraceptive also contains one of the hormone in regular pills [ the progesterone]. So it is expected that topiramate will decrease the action of pill as it has an effect on liver function. The efficacy of this pill in women on topiramate may be accurate and it may be lower than expected. So it may not provide protection against pregnancy. Hence it is not recommended to use it. It is better to use either condoms or Intrauterine devices like copper T or Multioad. Hope this was helpful. Feel free to ask any further queries and I will be happy to help.Thanks for using HCM.Have a good day.Dr Nilajkumar Bagde\u00a0\u00a0\u00a0\u00a0\u00a0\u00a0\u00a0\u00a0\u00a0\u00a0Consultant Obstetrics and Gynecology"
},
{
"id": 137244,
"tgt": "Suggest remedy for problem in ankle",
"src": "Patient: Hello. I had a leg surgery two months ago. I have a rod placed in my lowed left leg. The screws seem to aggravate my ankle and it hurts really bad. I want it removed. When is the soonest that I can do that since my bones are healing well. I can t do anything because of the pain. Doctor: Hello,I have studied your case and I think there seems to be some infection or extra long screws which are hurting you because usually these implants are not painful. If there is any discharge is coming then it is surely an infection.I would suggest you to follow these steps.1.Get x ray and either share it with me or some other surgeon. If there is enlarge screw then it can be changes. If there is infection then you might need antibiotics for it.2.I would also suggest you to get your blood sugar status because Diabetes can also cause such symptoms.3.If there is every thing normal then screw removal possible after three months after fracture union but it depends on many factors like bone comminution or fracture pattern or type of screw.I hope this answer will be useful for you. Let me know if there is any other followup questions.thanks"
},
{
"id": 144291,
"tgt": "Suggest treatment for dizziness",
"src": "Patient: For the past few days when i get upto do something or move around i get real dizzy like ive been spinning in circles. When i sit down it goes away after a few minutes. Sometimes i feel like im gonna pass out and sometimes like im gonna throw up. No headache jus dizziness. Please help doctors office is closed for the weekend cant see them til monday and dont wanna go to the er unless its an emergency. Thanks for your time. Doctor: HiI am Dr Mittal.I have read your query.I think I can help you.The symptoms are quite suggestive of a orthostatic hypotesion.Try following measures.1. get up slowly from the lying down to sitting position, or from sitting to standing position.2. wiggle your toes before getting up.3. watch your fluid and electrolyte balance.4. Check your bp in lying down position, and then on getting up to standing position, exactly 2 minutes after standing up.Let me know the 2 reports of bp.I have tried to make it as simple as possible.Please feel free to ask a further clarification.Best of luckDr Mittal"
},
{
"id": 169396,
"tgt": "How to treat loose motion and fever in a toddler?",
"src": "Patient: hello sir My 2year bay is suffering form loose motion and fever from this Monday. after stool test they found some infection and given some antibiotic and paracetamol for fever he was good for 1 day/night but again gone for 4-5 times motion yesterday. doc is saying nothing to worry just leave it and just make sure intake is going on and passing urine properly. baby is good and playing as usual but just worry if nything is wrong further as he was for 1 day and again caused motion, any help/suggestion ?? Santosh Doctor: Hi Dear,Welcome to HCM.Understanding your concern. As per your query your child have symptoms of loose motion and fever which seems to be due to acute gastroenteritis which could be due to a virus. It is due to disturbed metabolism and electrolyte imbalance in body. Need not to worry. I would suggest you to start giving oral rehydration fluids sip by sip to maintain the hydration. You should take Probiotics such as lactobacillus. Visit pediatrician once and get it examined. You should give Ibuprofen to child along with proton pump inhibitors as well. You should go for blood tests as well and start treatment after proper examination. I would suggest you to take antiemetic like domperidone and ranitidine for gastritis. Avoid giving any sharp and spicy food.Hope your concern has been resolved.Get Well Soon.Best Wishes,Dr. Harry Maheshwari"
},
{
"id": 29116,
"tgt": "How can toxic shock syndrome be treated?",
"src": "Patient: I am very woried i was on my period last four days later i noticed bleeding with foul smell and thick puss coloured mucus. Today i took a bath giving extra attention down there trying to feel for the source of smell. I felt around deep as i could looking for maybe cuts or bumps. To my horror i felt something weird it didnt hurt to touch it so i pulled it out. It was a black tampon it smelled like death. Should i go to doctor i dont know how long it was up there, i have amoxicillan here can i just take those? Doctor: Hello Dear. Thank you for using Healthcare Magic. I read your query and understand your concern.I think you are heaving not toxic shock . Its symptoms are sudden high fever, low blood pressure (hypotension), vomiting or diarrhea, a rash resembling a sunburn, particularly on your palms and soles, confusion, muscle pain, redness of your eyes, mouth and throat, seizures, headaches. If you develop any of this go to ER. Best regards"
},
{
"id": 192260,
"tgt": "What causes pain during ejaculation with thick semen?",
"src": "Patient: HII'm 25 and just your average guy - but i do smoke.Whenever I have sex and ejaculate I experience pain. My seamen is very thick and almost like jelly and doesn't shoot far which is a bit embarrassing at times. It's quite painful at the time and ruins the whole session. The pain subsides almost instantly but sometimes theres an ache in my anus for up to 40mins. Just wondered if there was anything I could do to make it more liquidy. Doctor: Hi, It may be because of some infection in the prostate gland or in the urethra. You need examination by doctor, an ultrasound and examination of semen to look for infection. Maintain hygiene. Hope I have answered your question. Let me know if I can assist you further. Regards, Dr. Sujoy Dasgupta, OB & GYN Specialist"
},
{
"id": 5570,
"tgt": "Took recogon before IUI. Having mild bleeding. Should I continue with IUI?",
"src": "Patient: Hi, i planned for IUI and for that i took medication - Injection - Recogon-50 units 1 ample(SC) from 6th to 10th day and some other medicine and advised to go for E2 level blood test this Monday (18th day) but today (17th day) having red discharge... Can i continue with IUI or not.... Plz assist, i am very much worried..... thank in advance..... Doctor: Hello Thanks for your query. Firstly, you need to provide me with more details if you want assistance. If IUI had been planned, serial follicular monitoring must have been done. You need to tell me what was the status of follicular development during the stimulation. Also, whether ovulation occurred as expected or not? Simply having a red discharge is not a contraindication to having an IUI. You need to give the details of what has happened so far, and what is planned, for me to opine. Take care."
},
{
"id": 217122,
"tgt": "Suggest remedy to relieve pain due to fractured humerus",
"src": "Patient: at 78yrs old a female fell on concrete patio, and fractured my left shoulder humerous bone, and dislocated my pain has gotten worse have had occapational therapy, and outpatient therapy has progressively gotten worse, take hydrocodone every 6hrs, and still doesn;t ever completely eleviate this pain....what can be done, do not want to take more than prescribed..... angela Doctor: First if all u need to cosult an orthopeditian for fixing the fracture.otherwise it is difficult to manage the pain.take tab etoricoxib 90mg once a day"
},
{
"id": 138605,
"tgt": "Suggest remedy for fluttering on side of rib cage",
"src": "Patient: I have had fluttering on my right side under my rib cage, its as though there is something moving. This has been going on for quite some time and no Dr. can figure it out. I have had ultra sounds, colonoscopy etc. This is becoming very irratating as I have no answers. Can you give me some ideas of what it could be? Doctor: There are various resion for fluttering for one side rib cage. It can be heart issues like murmuring, or vitamin b deficiency and muscle twitching, it can also be due to abdominal organ issues or after having fall on rib side Now here you have already done ultrasound and colonoscopy and I hope the report is normal so you have ask again here. I suggest to do ecg and tredmile test for cardiac health along with vitamin B complex test to know vitamine B level in body. As per the report if there is a issue with vitanin B then have it from suppliment range and if there is issue with heart then take treatment as per the need and suggestion by doctor. Now if all above reports are normal then don't worry I suggest to have good life style and take balance diet rich in calcium and protein. Have proper sleep and do regular exercises.Here I will also suggest to check with ortho or physio with other possibilities like muscular and bone pain. And take treatment and exercises suggested by ortho and physio.I am sure this will be helpful if you follow it properly and do not lift much weight on or with affectes side.Take care"
},
{
"id": 62921,
"tgt": "What causes a lump below the ribs having pain in upper abdomen?",
"src": "Patient: Hi. I m female, 40, had surgery and was treated with radiation for a 2.5 cm ductal carcinoma with micro mets last year through the end of November. I ve started working out again over the last 3 weeks. About one week ago, I noticed pain in my upper abdomen, about an inch below the juncture for my ribs. Sometimes, I just randomly feel pain, and always I feel it if I arch my back a little. This morning I noticed that I have a lump there. It feels like a nodule resting on top of a vein. It also hurts if I lightly rub down on the nodule. I m not sure if I should go to which doctor... I have a lot right now. Doctor: Hi, dearI have gone through your question. I can understand your concern. You have history of ductal carcinoma of breast. You may have soft tissue tumor like lipoma, neurofibroma, dermatofibroma or metastatic breast cancer. You should go for ultrasound examination and if needed go for fine needle aspiration cytology or biopsy of that lump. It will give you exact diagnosis. Then you should take treatment accordingly. Hope I have answered your question, if you have doubt then I will be happy to answer. Thanks for using health care magic. Wish you a very good health."
},
{
"id": 187545,
"tgt": "How can one treat soreness of gums and tooth ache caused by flu and cold?",
"src": "Patient: I have had a cold/flu for the past week and it doesn't seem to be going away. I went to a clinic because I was also getting sore gums and one was infected with white pus in it. The doctor gave me penacillan (sp) but the gums are still really sore. It doesn't feel like a tooth ache, and the gums are red all over. Doctor: thanks for your query,the features what you are giving suggests you might have a periodontal abscess means collection of pus..you need to take radiographs to rule out any decay and other pathologies....you can take doxycycline or metronidazole...take an opinion from dentist before starting the medicine...you have not mentioned about your gender, marietal status (pregnancy)...so please consult your dentist...thank you..."
},
{
"id": 55802,
"tgt": "Suggest treatment for elevated liver enzymes and diabetes",
"src": "Patient: Yes, I am a 54 year old female. I have type 2 diabetes and neuropathy. I also have muscle twitches that we suspect are from a statin. I am off the statin 3 months and it helps but does not stop the twitches. However, today I have had on and off a tingle as if a hair was resting on my nose at the tip. I know this is unusual and I have gotten used to unusual. I have had 14 hrs in one surgery for masthectomy and reconstruction with ovary and fallopian removal. I know all kinds of unusual feeling. I am going to Jamaica tomorrow and want to make sure I am ok before leaving. I have normal blood pressure with no facial drooping. I weigh 163 lbs. I do have high triglcerides and high liver enzymes. (Non alcoholic fatty liver) Doctor: hi,For elevated enzymes when you know that you are having non alcoholic fatty liver that is the cause for elevated enzymes ,The treatment very simple you need to consider proper diet modification like less oily and fatty food as it will help both for your diabetes as well elevated enzymes.Next important thing is proper brisk walking for 20 to 30 mins daily Proper controlling of your sugars .Regular follow up every 3 months with LFT and other diabetes and dyslipedemia profile.You can start using tab udiliv 300 mg twice day with tab Evion once a day,This may help you in normalizing your enzyme levels.Thank you"
},
{
"id": 187469,
"tgt": "Suggest remedy for bad cold followed by numbness in teeth",
"src": "Patient: I had a bad head cold about a week ago and the day that I started feeling myself again, my front teeth went numb. the numbness has moved from my right front tooth to the left and into all the other teeth on that side of my face. no pain ever experienced, just numbness. unsure how to treat. the numbness has been there for over a week now. Doctor: hiwelcome to HCMnumbness in upper front teeth must be because of tooth infection in maxillary which may be causing you cold too or sinus infection.for this i will advice to get an Opg xaray done and visit your dentist for treatment.thank you"
},
{
"id": 76074,
"tgt": "How to get rid of persistent cough?",
"src": "Patient: i was out in the rain all weekend and now i have a persistent cough. But now that i think about it I was using bleach and CLR on sunday afternoon and now i have a persistent cough that is like a sudden coughing fit and i feel like i could cough at any time. do you think it was the chemicals or being out in the rain all day? Doctor: Thanks for your question on Healthcare Magic. I can understand your concern. In my opinion, your cough is mostly due to bleach and CLR use. These chemicals produce Fumes. And when these Fumes go inside airways and lungs, they cause irritation and inflammation. And they inturn cause cough. So while cleaning, you might have inhaled Fumes from these cleaning chemicals which is causing you cough. Usually this kind of cough is self limiting in nature in 1-2 days. So drink plenty of fluids orally and keep your self hydrated. Do warm water gargles 5-6 times a day. Take simple antihistamine like levocetrizine. Don't worry, you will be alright with all these. If you still have cough (after 2-3 days) or develop breathing difficulty then consult doctor. Hope I have solved your query. I will be happy to help you further. Wish you good health. Thanks."
},
{
"id": 10994,
"tgt": "Suggest treatment for excessive hair loss",
"src": "Patient: Hi I'm SUMON PAUL.I am 21 years old & having excessive hair loss from the front two sides of my head & I like to add here that my father had lost his hair before my birth due to typhoid .So is it from him,b'cause I dont have any deseases.what should I do???????? Doctor: My DearWelcome to Healthcaremagic!I have noted your details and it fits a condition called Androgenetic Alopecia.This condition is due to genetic factors and the hair loss is at first confined to both front and sides but later on other areas too get involved.For most of my patients of this condition, I prescribe Minoxidil 5% solution for topical application, one ml each application for several months. results are good but it takes several weeks before you can actually notice them.Please follow the instructions and you should be feeling good after some months.Best wishes."
},
{
"id": 143881,
"tgt": "Suggest remedy for tenderness and persistent throbbing in head",
"src": "Patient: I have been suffering with throbbing in my head for 3+ years,also my head is tender to the touch but not always in the same place also I become red and flushed in my face and neck and also hot to the touch,I have been told I am suffering with chronic daily migraine,but medication I am on is doing nothing and it has steadily got worse but my own doctor still insists that it is migraine. Doctor: ur doctor is correct. most likely u have headche of migraine. Sometimes it may cause redness of face or watering of eyes. Try tab Topiramate as prophylaxis. It has very good efficacy and few toxic effects.Another possibility is Trigeminal autonomic cephalgia. It responds to tab indomethacin. Consult ur neutologist and discuss in detail about headche character, frequency and associated features. it will help to formulate diagnosis and treatment.regards."
},
{
"id": 182978,
"tgt": "Would taking Doxycycl for new partials have any side effects?",
"src": "Patient: I m on Janumet 50/10000. Metoprolol 50mg. Hydrochlorot 12.5mg.Losartan 50mg .My dentist now wants me to take Doxycycl hyc 100mg.Are there any side affects from doing this . I ve only taken penicillan or amoxicillin before, He felt the doxycycl would be better to prevent infection from my new partials. Doctor: Thanks for using Health Care Magic.Read your query.Doxycycline is an effective antibiotic in certain oral conditions. If your dentist has evaluated your oral status and prescribed , go ahead and have it.I would recommend that you give a proper medical history to your dentist.Hope this was useful.Thanks and regards."
},
{
"id": 96457,
"tgt": "Bowel obstruction",
"src": "Patient: I have ulcerative colitis and think I may have a bowel obstruction. I have been constipated and do not use laxatives but was desperate so took full dose of milk of magnesia and still have not gone on 2 days. Cramping and stonach noises present. Cramping in lower right side. Not sure if I should wait it out or see a doctor. Any advice? Doctor: yes you should see a doctor preferably in ER. constipation can cause obstruction and surgery might be required to relieve the same. before it goes to that extent you need to get it checked from the doctor."
},
{
"id": 223942,
"tgt": "Does i pill cause excessive bleeding?",
"src": "Patient: hi sir, i had done sex with my gf on the seventh day 20 mar after her period stoped (14 mar) ..i gave her i pill within 24 hours and after that she had her period on 26 mar and bleeding more than expected and having headache ...sir m i need to be worry plzzz suggest me Doctor: Hi dear..bleeding is one of the commonest effect of I pill along with pain,fever and headache etc..so I advise not to worry but she can follow this points1) Take iron supplements2) Take green leafy vegetables and fresh fruits3) Check her Hb level4) Bed rest for 2 days if possible5) in case pain and fever take Tab. PCM6) If still it continues for more than 3-4 days then take progesterone supplement under the observation of gynaecologistHope this will help youThanks and regardsDr.Sohil Takodara"
},
{
"id": 68205,
"tgt": "Can a lump in the breast cause low glucose levels?",
"src": "Patient: 33 year old male..... Went this morning for pre-opp blood/lab work. I'm having a lump/mass removed from my left breast. The doctor called and told me that my blood glucose level was 42 and I needed to come back in for more blood work. What could be the reason behind the low glucose level? Is it related to the lump in my chest? Non-diabetic. Thanks for your help Doctor: Hi, dear. I have gone through your question. I can understand your concern. Youare suffering from hypoglycemia. Your low sugar level is not related to your breast lump. Therr are some other causes for your hypoglycemia. You should go for investigation. After your sugar level become normal then only you should go for surgery. Hope I have answered your question, if you have doubt then I will be happy to answer. Thanks for using health care magic. Wish you a very good health."
},
{
"id": 57641,
"tgt": "Can Monocef injection given to hepatitis patient cause death?",
"src": "Patient: Hello Sir, My father was suffering from Liver disease.(liver was smaller in size),he was under treatment of DOCTOR since 7-8 months,recovering but on 7 feb .2014 blood was passed out from mouth and pain started..we have called doctor...doctor told for an injection MONOCEF (Ceftriaxone 1gm).We have given that injection within 15 mints he was no more....i have heard that the monocef injection is not given in Hepatic patient.I WANT TO KNOW THE CAUSE OF DEATH IS INJECTION OR NOT. Doctor: Monocef injection is never cause death in any patient. Unless your father has any allergy to ceftriaxone. But if he has received this injection in past, chances are there he died because of liver failure. History which you are giving is also very typical of liver failure, as patient development bloody vomiting, and died due to this."
},
{
"id": 224185,
"tgt": "Have clear discharge after taking Next Choice pill. What does it mean?",
"src": "Patient: I had sex on the 11th(august) around noon and the condom broke. I am pretty sure my partner came inside me. I took Next Choice on the 12th around 5pm. It is now 3am the 13th. I have been experiencing clear discharge since a few hours after i took the pill . Is this ovulating? Don t really know what that means. Am I pregnant? I am really worried, I don t know what this means. My last period started july 21st and ended on the 26th. Not sure when I should be getting my next period because i have only started keeping track since last month. Is it likely that i m pregnant? Help! Doctor: Hello,I have gone through your query and understood your concern. It is difficult to say if you were in the ovulation phase at the time of the intercourse since your menstrual pattern is not available. Yet, you would be amply protected since you took the emergency contraceptive pill. The discharge is likely to be the pill-effect as there is high progesterone content in the emergency contraceptive pill. You can get a blood test for pregnancy not earlier than one week after the intercourse. This is the earliest way in the current scenario to detect conception. Your next natural period may get delayed by a week or two and if you are overdue by ten days, please see an expert for further management. Hope you find this information useful. Take care."
},
{
"id": 141184,
"tgt": "How long should Eption be taken?",
"src": "Patient: Sir, My son have operated his head due to tumour in headache. The tumor was checked and doctor opinion the tumer is simple menengioma. then after my son have taken eption 300 er for morethan five year And the doctor has stoped to take this medicin. but after six month my son facing feet at morning. what to do ? Doctor: Hello, I am afraid I am not entirely sure what do you mean by \"facing feet at morning\", there must be some writing mistake. Do you perhaps mean he is having seizures (fits) in the morning? If that is the case then the medication should certainly be restarted. Even though meningiomas are benign tumors and surgery was successful it may have left a small scar in the brain predisposing to seizures. He should also have a control MRI to make sure there aren't any new tumors (one can have multiple meningiomas). Hope I have answered your query. Let me know if I can assist you further."
},
{
"id": 219243,
"tgt": "What are my chances of getting pregnant?",
"src": "Patient: I took primolut n to delay my menstruation which is supposed to be on 1.3.14. Started 2tabs 2 times a day on 23.2 till 28.2. However I had unprotected sex on the night of 28.2. So after 42hours, i took escapelle to prevent of getting pregnancy. What are the chance? Doctor: Hi.The earlier an emergency contraceptive pills (ECP) is taken the better. But as you took it within the first 48 hours, your chances of preventing conception are about 85 %. This should be good enough to avoid it.Best wishes."
},
{
"id": 59485,
"tgt": "Diagnosed with hepatosplenmegaly, have irregular lifestyle due to professional constraints. Advisable diet?",
"src": "Patient: I have been diagnosed as having hepatosplenmegaly I have no family history of this.But being a pilot I have very irregular life style,eating at odd hours,having large meal ,then go for hours without food and water.I do drink moderately .I had jaundice when I was ten Years old,if at all i thas any implication on this.Please guide metohave correct diet .Is this condition reversible? Doctor: Hello and welcome to HCM. The causes for hepatosplenomegaly are the following: -Infections (hepatitis, infectious mononucleosis, rubella, etc.) -Blood disorders (anemia, thalessaemia, etc.) -Metabolic disorders -Liver diseases -Portal hypertension and many more. Please note that hepatosplenomegaly is just a symptom, and there should be some underlying condition. Please visit your doctor for a more elaborate examination including a complete blood workup. And yes, this condition is reversible, once the cause is treated this gradually subsides. In the meantime, you should avoid all fatty, oily food, and also meat. I hope this information helps. Best wishes."
},
{
"id": 76573,
"tgt": "While having Pectus Excavatum (mild) could occasional pain be cause for concern?",
"src": "Patient: I am 19 years old with absolutely no problems except that I have mild pectus excavatum. I have had twitching under my lower right rib cage sometimes uncomfortable, generally annoying. Today I woke up and it's sometimes painful and I'm not sure if I should be worried. I've seen my doctor about it and she hasn't seemed to find anything wrong when I had scans so I don't know. Doctor: Hi thanks for contacting HCM...Pectus excavatum can be congenital deformity or sometime rickets can lead it...Anyway your issue is regarding disofort at lower rib cage...Here nothing seems serious...Palpate the rib cage costochondral junction ..if prominent then it could be teitz syndrome...If no swelling and tender to touch then it could be costochondritis like inflammation...Ibuprofen like antiinflammatory drug can be useful...Consult pulmonologist for further examination and advise...Take care.Dr.Parth"
},
{
"id": 70588,
"tgt": "How can tightness in the chest and throat and cough be treated?",
"src": "Patient: Hey Doc, Ive been having a tightness in throat below adams apple when i breath at times. ive been coughing up small amount of clear bubbly mucus. I have been having chest discomfort and I feel tightness in throat most when i eat. i am clearing my throat alot as well. i took allegra d for a few day and although it somewhat helped i still have the tightness in throat and post nasal drip Doctor: Hello, As per my clinical experience, the condition with chest tightness and throat discomfort with cough, post nasal drip indicate possible upper respiratory infection with some sort of virus or bacteria most likely. Though primary relief with gargles and off the counter medicines can be obtained, I prefer you to get one time physician check up for upper respiratory tract as supportive antibiotics, anti viral and decongestant medications will help to resolve the issue faster and prevent secondary complications of infection. Hope I have answered your query. Let me know if I can assist you further. Thank you. Take care. Regards, Dr. Bhagyesh V. Patel, General Surgeon"
},
{
"id": 96757,
"tgt": "Can a hit on back of head in 3 years old cause danger?",
"src": "Patient: My 3 year old slipped on our wet bathroom tile and fell hitting the back of his head. He cried and was in pain, but calmed down and said it didn t hurt anymore. I keep thinking of Natasha Richardson and the fall she took while trying to learn to ski. Is there anything I should do right now? Doctor: Hello XXXX,Thank you for posting your question on HCM.If I was your child doctor, I would recommend for you to observe him in the next 24 hours and be very careful if:-- you notice any new behavior in your son, -- if he falls asleep not in his usual time, -- try to do something with him in the next hours after the head hit, even if its sleeping time, to keep him as much as possible on observation for strange/new behaviors that you can not see while he is asleep-- and check him from time to time while he is asleep.Things you should look in his behavior, that should make you take him to the Er department are complaints of headache, dizziness, nausea and vomiting, difficulty with coordination or balance (not being able to catch a ball), feeling, confused and dazed, difficulty concentrating, thinking, or making decisions, trouble remembering things, such as what happened right before or after the injury, slurred speech or saying things that don't make sense, feeling anxious or irritable for no apparent reason, feeling sad or more emotional than usual.Hope this helps,Regards,Dr. Klarida"
},
{
"id": 24606,
"tgt": "What causes sudden atrial fibrillation while sleeping?",
"src": "Patient: My husband is 34 with Af. He was in ICU for 2 days then to main floor for 1 then released. He is under supervision and currently taken Ritmaland PP meds. He was fine for 2 weeks with no fibulation but all the sudden every night this week he is waking up in AF about 3:30 am , It last any where from 30 mins to an hour. What is causing this and why at night when he is sleeping. Doctor: when lying in supine position venous return to heart will increase and work load increase so if heart not working well Means atria get dilated leading to AF to the patient."
},
{
"id": 102178,
"tgt": "Could nausea, rib pain, facial pain and nasal congestion be related to allergies?",
"src": "Patient: 55 y/o female. Approx 6 months ago thought I was biting right cheek, and the dentist thought that was what it looked like. The dentist has a watch on a lower tooth on the right side that may need a new filling. Noticed right nasal congestion off and on. I have a history of allergies and get allergy injections. Past week started having a lot of increased saliva, facial pain and can feel the muscle or bump in front of ear. Not seeing any noticeable facial swelling. Also I've been having nausea, and some occasional right upper pain below the ribs. My right side of my face and neck is a little sore. Doctor: HelloWelcome in H.C.M.I have gone through your query regarding recurrent nasal discharge and obstruction with soreness over right half of face and upper chest. You are a diagnose case of nasal allergy.According to your clinical history it appears as you are having chronic respiratory tract infection invlving both upper and lower respiratory tract. Probably you are having sinusitis with infection of resp system may be bacterial or viral.I would like to tell you to consult your ENT doctor for thorough examination and investigations for making diagnosis that may be accordingly manage.Regards."
},
{
"id": 201331,
"tgt": "What are the side effects of excessive masturbation?",
"src": "Patient: hy doctor. i can control i am addict of mastrubit. . i am of 20 years and have not done real sex up till now. . so please tell me that up till which limit i can mastrubit which will not effect my sexual health after my marriage. . . or i may not face any arectile dysfunction. Doctor: Hi,This habit is common at this age.There are no any bad effect on body or sexual life in future so do not worry.Keep frequency of masturbation in control.Ok and take care."
},
{
"id": 57268,
"tgt": "Is tracheotomy process necessary?",
"src": "Patient: Have a freind who has been hospitalized for over a week with two tubes in mouth for breathing and stomach drainage. Pancreatitis swollen twice its size. The doctors want to do a tracheotomy and the faimily needs to say yes or no. what is this benefit for the paitient? More comfort possibly? Doctor: Yor friend might need prolonged hospital stay and ventilator . The benefits of a tracheostomy are it will help to keep the airway / wind pipe clean of secretions. It will be more comfortable than a tube passed through his mouth. It will also help in assisting his breathing better via a ventilator"
},
{
"id": 103879,
"tgt": "Excessive sneezing, swollen nasal passage. Taking montelukast and fexofinadine. How long to take? Permanent cure for this allergy?",
"src": "Patient: hi doctor, good eveNowadays, I am having number of sneezes everyday. Previously I was not allergic but now every time seasons changes (i.e. in every 3-4 months) I am facing the problem of excessive sneezing and red throat. Nasal pasage also gets swollwn (may be allergic rhinitis).I am taking Montelukast and Fexofinadine per night. How long I have to take these medicines.Is there a permanent cure to this allergic reaction of my body.Please help. Doctor: Hi, The main principle in treating allergic rhinitis is to stay away from the allergen it might be a pollen, dust... Still you need to check your eosinophil count to range your dose. usually you can take for 14 days. And if you are planning to use nasal spray for congestion not advisable to use more than 3 days.wish you a good health."
},
{
"id": 109075,
"tgt": "Suggest treatment for back pain",
"src": "Patient: Dear Dr.,I'm 55 Yrs old, 152 cm tall and 44 Kg and healthy woman.I have problem with pain in my back & calf and numbness from my right leg through my foot, from CT scan said \" The disc spaces are preserved. There is slight anterior slip of L4 on L5At the L3/4 level there is no posterior disc protrusion. The neural foramina and lateral recesses are clear. Minor facet degenerative changes are present.At the L4/5 level is broad-based encroachment on the spinal canal by the prominent posterosuperior border of L5 and by minor broad- basded posterior disc protrusion. There is indentation of the thecal sac. Facet degenerative changes are present and there is some flaval thickening with sideways narrowing of the canal. Canal narrowing is not significant. Compression of the L5 nerve roots may be significant. The neural foramina are clear.At the >5/S1 level there is no posterior disc protrusion. The neural foramina and lateral recesses ar clear. Minor facet degenerative changes are present.Conclusion: Mino L4/5 spondylolisthesis with prominent facet degenerative changes. Compression of the L5 nerve roots may be significant. Minor L4.5 posterior disc protrusion. No significant canal narrowing. Lateral recesses and neural foramina are otherwise clear.I have physiotherapy which help a little some what.Could you please tell me that what shall I do or expected with my back , some times I feel it better after I'd passing wind or go to the toilet which I feel during exercise. Best Regards,Amanda Lou Doctor: Hi,You need core strengthening exercises. Also maintain good posture. Do not lift weights or bend forward. There are various physio modalities to take care of your pain. Hot water fomentation will surely help to sooth your back. Exercises to strengthen your overall back will improve pain."
},
{
"id": 122948,
"tgt": "Suggest treatment for back injury bruise on collar bone",
"src": "Patient: I was in a head on collision Thursday night was taken to.the er in a ambulance. They said I have some back injuries and a mild concussion as well as a bruised collar bone from the seatbelt. Today I woke up w a bruise on my forehead and I have a fever of 100.6 that I cant get it to go away. Why would I have a fever Doctor: Hi, As 100.6 is not a high fever to be considered. We need to check for other symptoms along with it. Usually after the head collision there might be some physiological changes leading to increase in temperature and it's a normal. Healing process. In case other symptoms arise than we need to look out for the blood routine and provide medications as needed. Hope I have answered your query. Let me know if I can assist you further. Regards, Jay Indravadan Patel, Physical Therapist or Physiotherapist"
},
{
"id": 80986,
"tgt": "What causes intermittent severe pain in chest area and chronic indigestion?",
"src": "Patient: I have severe pain at times in my chest area, I have had heart checked, and upper and lower gi and shows nothing. I have had indigestion all my life even from a teenager till 73 years later. My doctor says I have barrett s esophagus disease. Can this cause the chest pain I am experiencing? Doctor: Thanks for your question on HCM.I can understand your situation and problem.Yes, Barrett's esophagus can cause chest pain.It is actually change in esophageal mucosa, due to chronic GERD (gastroesophageal reflux disease). On chronic exposure to acid reflux Esophageal mucosa becomes resistant and gives rise to Barrett's esophagus.So treatment of GERD is needed to treat Barrett's esophagus and thus your chest pain.Try to follow these steps.1. Avoid hot and spicy food.2. Avoid large meals, instead take frequent small meals.3. Avoid stress and tension4. Start proton pump inhibitor.5. Go for walk after meals.6. Keep 2-3 pillows under head in bed to prevent acid reflux.7. Avoid smoking and alcohol."
},
{
"id": 69202,
"tgt": "What causes a hard lump on the top of the lips?",
"src": "Patient: Today I noticed a small hard bump on the roof of my mouth. I have just gotten over a cold and have a cold sore on my lip and also I ate some fried tortilla chips that always tear up my gums could that be it? I know it wasn't there this morning when I brushed my teeth Doctor: Hi,Thanks for posting the query, This could be due to infection from the tooth, any type of trauma, beningn lesion. I would suggest you to get a checkup done, an OPG radiograph of thge lesion is required.Is the lesion painful??Could also be due to allergic reaction.Take care!"
},
{
"id": 23658,
"tgt": "What causes heart fluttering?",
"src": "Patient: I get heart fluttering now and again, had an episode today just a little while ago. I was relaxing and watching TV with my daughter and it started beating fast. I beared down like I was having a baby and it went away. However, the last week or so I've had chest tightness and almost like a bruise soreness in the middle of my chest? What the heck? Doctor: HiThe heart fluttering which you are experiencing may be a SVT (supra ventricular tachycardia ).they are small electrical problems in our heart and are very common aan amenable to treatment.they are due to re entrant circuits.I suggest you to get an EKG done and consult a cardiologist for the same."
},
{
"id": 129884,
"tgt": "How to treat very painful L5 sacralization?",
"src": "Patient: L5 SACRALIZATION. Very painful. Tweaked the other day. What are my options as far as bending and lifting. My boss wants me to do warehouse work, but I m afraid that those days may be over. Problem is, one wrong step and I can hit the floor from seizing up and pain. The sacralization is on both sides. Doctor: Hi...Please don't worry about sacralisation...as it is there from the time you born...so relax...Coming to your pain...I would recommend you to Keep icing your back frequently...to reduce spasm...To do lot of core muscle strengthening workout...To do glute stability to improve spine stability...To stretch your hip flexors and gluteus to reduce forces pulling on spine...Avoid lifting weights for a while..if happen to lift keep your back straight and bend your knees to lift weights ..Do spinal extensions frequently to avoid imbalance to set in....Hope this is helpful for you..Kindly revert back in case you need any further help in this regard.."
},
{
"id": 182892,
"tgt": "What are the advantages of unibody implant and the regular dental implant?",
"src": "Patient: hello, i want to know the advantage of unibody implant and the regular dental implant, which one is better, my front bottom tooth is really small, and I heard unibody is better for me, but when I search for unibody on google, i can t find good information, can you please help me? Doctor: Hello,I generally prefer to allow the implant to heal under the bone and avoid any initial loading. The single stage or unibody advantage will allow you to have a temporary crown placed for the anterior tooth immediately. Ask your dentist for additional information and the particular brand. You want to see some clinical studies for the implant that you are considering. Consider a second opinion with a periodontist. The most important consideration is bone support. You want to do everything possible to have the best surgical outcome for the implant placement. Consider a removeable temporary tooth while the implant is placed and heals as an option. Bicon dental implants use short implants. There are MDI Mini dental implants designed for dentures by 3M. OCO Biomedical and Tischler Dental are some implant brand names to search for more information.thank you for your inquiry."
},
{
"id": 215034,
"tgt": "My right arm is always aching and pain in the chest also. What is the remedy for it ?",
"src": "Patient: its been for the past 1 year that my right arm is always aching, at times even in a sunny weather i have to wear warm cardigans as the pain makes me freeze. The veins through out my thighs and especially in my right arms have become visible and they appear in green color. When the pain comes it gives pressure on the vein through out my right breast and chest causing me so much pain in the chest also. What do you think is the remedy to this health problem, sir! ? Doctor: Hi,thanks for query.Visible green veins on arm in normal ,not a disease.You may be having pain but there is has to be some other reason like bad posture,lack of exercise for your pain.Please visit your doctor and look for any compression of nerve in neck which can give rise to pain in shoulder.Avoid thick pillow if you are using,do regular exercise and visit a doctor.bye."
},
{
"id": 142592,
"tgt": "What causes pain in the lower back radiating to the legs and arms?",
"src": "Patient: Hello I don t understand why I feel like I do I know I have scardosos, I have had 5 neck surgeries over the years, and now I have problems with my lower back pain, which is now going down my legs my arms and hands are weak, I have trouble catching my breath lime today we went and picked strawberries I felt dizzy like I was going to pass out or vomit what is wrong with me Doctor: Hello!Welcome and thank you for asking on HCM!You should know that weakness of your hands is not related to the lumbar spine. In fact it can be explained by cervical spine degenerations or spinal canal stenosis in this levels. Other possible causes to consider would be polyneuropathy, especially considering the fact that this is a common complication of sarcoidosis. Coming to this point, I would recommend performing a cervical spine MRI study and a nerve conduction study for polyneuropathy. You should consult with a neurologist and discuss about these issues. Hope you will find this answer helpful!Wishing all the best, Dr. Aida"
},
{
"id": 134471,
"tgt": "How to treat osteoporosis of the spine and hip?",
"src": "Patient: I have had mitral valve robotic repair dec. 2013. Now they are telling me I have osteoporosis of spine and right hip. I am 66, and otherwise in good health, weigh 115, I am leary of drug side effects, especially with the recent heart surgery. I have a ring keeping the valve working. Dr. assistant, PA, wants to put me on Fosomax. for 2 years -2.9 spine, -2.6 rt. hip Dexa scan, other areas are osteopenia. I have read of serious side effects Can I treat with just vitamins, diet, exercise. Would I be foolish to do that? I am very sensitive to all drugs, fast metabolism. Doctor: hi,as you mentioned your age as 66. let's look into the things which can help you fight this osteoporosis. Having a good vitamin D can be done by regular sun baths. because sun is the only source of natural and unharming vitamin D. for calcium you need to take regular milk. now how does your osteology improve? well doing a weight bearing exercises in a graded pattern will surely be of help as to many it has done. bone mineralization occurs when one takes proper vitamin D through sun, calcium through milk and absrobing of it through exercise. I wish you will be helped and the pain and aches due to osteoporosis will be removed. taking to the issue with the heart, well exercise is the best medicine for most of the problems as it's a natural inducer of metabolism and immune system."
},
{
"id": 222067,
"tgt": "What causes lack of movement of fetus during pregnancy?",
"src": "Patient: I am in my 39th week of (my 4th) pregnancy and I have been feeling very little movement throughout the day today...should I be calling my doctor? I have never made it this far in the past as I have always been induced so I am not sure if this is normal because the baby must be very cramped, or if this is an indication of distress. What should I do? Doctor: Hi,Decreased or absent fetal movements in term pregnancy needs full evaluation immediately. You need to visit a hospital where fetal heart tracing can be done, also an ultrasound of the fetus is advisable to know the amount of amniotic fluid around the baby and the fetal development and size and fetal heart rate.These need to be discussed with a gynecologist to time your delivery depending on the babies condition any other risk factors that you may have.Hope this helps.Regards."
},
{
"id": 126016,
"tgt": "What causes protruding vein and clicking noise on the right elbow?",
"src": "Patient: My right elbow crease has a noticeable vein running through it that I can feel when running my fingers over it. My left arm doesn t have this. The pain is very dull and constant and gets a little worse when I constantly extend my arm. There is also a pop or click that I feel when fully extending that arm but I can t feel what it is. I thought it was distal bicep tendinitis at first but now I m not sure as it s been bothering me for about 8 months now. Doctor: Hi, 1. Vein crossing the elbow is normal and it may not be same on both sides, 2. The pain is usually not because of vein but probably it is tennis elbow of Golfer's elbow. 3 Start painkillers. If no improvement in 10 days then consulting Orthopedic Surgeon is necessary. Hope I have answered your query. Let me know if I can assist you further. Regards, Dr. Gopal Goel, Orthopedic Surgeon"
},
{
"id": 163778,
"tgt": "How can an itchy back, stomach and nappy rash be treated?",
"src": "Patient: My 7month old baby has a red itchy like rash on her back ond stomach, only came in last few days, like a heat rash but is in great form with it, haven t changed food or milk but she has broken her first two teeth, She also has very bad nappy rash, i ve been treating it but is still quite bad. Any idea s Doctor: Hi...Thank you for consulting in Health Care magic. Skin conditions are best diagnosed only after seeing directly. I suggest you to upload photographs of the same on this website, so that I can guide you scientifically.By what you say I feel that your baby may be having a diaper rash. You can use siloderm ointment for external application price a day for a week.Please revert back to me with images so that I can guide you better.You can approach me at the following link.Once the page opens there will be an option below my image as \u2013 ASK ME A QUESTION \u2013 click on it.Please find the link below -www.healthcaremagic.com/doctors/dr-sumanth-amperayani/67696Regards - Dr. Sumanth"
},
{
"id": 107075,
"tgt": "What causes sudden sharp chest pain after taking spinal injections?",
"src": "Patient: I had a lumbar injection today for a L5-S1 herniation. I am a healthy 43 year old female with no chronic issues. During the injection I experienced a sudden onset of sharp chest pain that has continued now after a couple hours. Is this a side effect? I am a registered nurse in the ER and I know that most phycians would tell me to go to the ER but with no cardiac issues I don\u2019t really want to. Thanks Doctor: if the pain Is Mild and is not Continouse which isn't disturbing ur breathing nor is worsening in intensity then Stay at home . and take a tablet of ibuprofen 400 mg or any pain killer if u have in home as a single dose"
},
{
"id": 217445,
"tgt": "What cause sudden pain in the kidney area?",
"src": "Patient: Hi,I am an athletic and healthy 57 year old male. Rather suddenly I am experiencing debilitating pain in my right kidney area (lower back, right side). I have been hiking a few days before, no lifting. I almost cannot walk right now with shooting pain. Any thoughts?Thank you,JP Doctor: Dear JP,Thank you for your question on HCM.considering the fcat that you are an athlete and that you have been hiking from a few days,i would like to consider that you may have a pulled muscle.Take Tab.myoril plus twice a day for two days (after food) and apply a hot compress for relief.you can apply brufen gel for pain relief.hope this information was helpful.regards,Dr.Radhika"
},
{
"id": 178017,
"tgt": "What causes black stool in babies?",
"src": "Patient: I have a daughter who will be 3 in October, and she seems totally healthy except her stool this evening was totally pitch black, and when I dumped it from her potty chair into the toilet, it was like someone had dumped soot instead... her diet is the same, except she ate a ton of strawberries and blueberries this afternoon. Is this something that I should be concerned about? Doctor: Hi...this means she is having mucosal bleeds in the gut and it could be a hemorrhagic disease of the newborn. I suggest you take her to the emergency room immediately, even if she is looking and acting normally.Regards - Dr. Sumanth"
},
{
"id": 84772,
"tgt": "Suggest medication to prevent infection from tampon",
"src": "Patient: I had a tampon in side for about 2 weeks & 4 days I forgot to take it out I happen to douche today & it came right out I feel a little bit better but feel I would like to take something over the counter just in case what can I take. please help I m a little worry.... Doctor: Hello,I will suggest you to keep an eye on being dizzy, feverish or having a nauseating feeling or dizziness if you get any of these or any associated headache you should immediately consult emergency physician. If you can get over the counter penicillin or clindamycin. Hope I have answered your query. Let me know if I can assist you further. Regards, Dr. Muhammad Faisal Bacha, Internal Medicine Specialist"
},
{
"id": 69015,
"tgt": "Suggest remedy for lumps",
"src": "Patient: Hi, I was hit by a line drive in the thigh during a softball game. I have had a serious bruise ( about the whole side of my thigh) for about a week. My leg is sore now, and there is a large, hard lump in the center of the bruise ( at point of impact) What can I do? And should I be concerned? Doctor: Hello!Thank you for the query.Such lump is most likely a blood collection caused by recent injury (hematoma). Injury can also cause veins thrombosis which gives similar symptoms (hard an painful lump along with painful calf).In case of hematoma, it may turn into an abscess. That is why it needs to be drained. Veins thrombosis needs heparine treatment.So in both cases you need to consult a doctor. Please do not wait with it.Hope this will help.Regards."
},
{
"id": 136250,
"tgt": "Suggest treatment for severe pain in the leg and calves",
"src": "Patient: Hi I had a heart attack in may . I had pain in my leg for a few days before the attack . I now have the same pain in the other leg and it has been going on now for 9 days and getting very little sleep it is so bad went to my own gp who said the pain was more likely to be coming from my back and said to go home and see what happens in another week. The pain is in calf and now in the thigh at the back I have tried ibuprofen but nothing is taking the pain away could you please help. Doctor: hiif pain not responding to ibuprofen, then consult orthopeic doctor. Leg cramps may have circulatory cause and a doppler ultrasound test is indicated.Do massage with olive oil, fomentation and take vitamin E capsules daily along with vitamin B-complex.some more medicines are there but I would suggest an thorough evaluation firstbest wishes"
},
{
"id": 8759,
"tgt": "Sunburn, dark skin on face, taking VLCC pigmentation treatment, applied hydro-quinine cream, using lomela cream, black patches. Advice on good cream without side effects",
"src": "Patient: DOCTOR ....THERE WAS DUE TO SUNBURN ,MY FACE IS DARK,I TRIED MANY TREATEMNT FOR THAT ,TAKING VLCC PIGEMENTAION TREATEMENT,AND SOME HYDROQUNINE CREAM ,BUT THERE IS NO RESULT ,FINALLY ONE OF FRIEND TOLD ME THAT APPLY LOMELA CREAM ,AND I APPLIED FROM 4MONTHS ,MY COMPLETE COMPLEXION ARE NOW CHANGED ,ITS PRITY GOOD FACE LIKE BEFORE,THE DARK COMPLEXION IS COMPLETELY GONE ,NOW I ALSO CONTINUE USING LOMELA ,BUT NOW THERE IS SOME BLACK PATCHES ARISING ,SO WHAT I RIGHT NOW TO RETAIN THE FAIR COMPLEXION ,WHETHER ,I WILL LEFT LOMELA ,OR SUGGES ME SOME CREAM WHICH CAN RETAIN MY FAIR COMPLEXION AS NOW ..PLEASE DOCTOR ,PLEASE TELL ME SOME GOOD CREAM WHICH IS NOT HARMFUL FOR SKIN AS WELL AS IT RETAIN EVER FAIR COMPLEXION LIKE NOW ..THANK YOU ..DOCTOR Doctor: Hi...dear user ., Thanks for choosing HCM.., I gone through Ur history.., LOMELA cream contains 2% Hydroquinone.., Intially it will give good respond...for long time use.., will create side effects...like..Dryness, rough , change of texture, and colour of the skin..., So stop the cream immediately.., 1) Morning times use SUNSCREEN LOTION.., which contains SPF 30...eg..SHADOWZ 30(Dr.Reddy's lab), NEUTROGENA 30(NCU)., 2) Night times use Moisturizer cream...CETAPHIL..(galderma) will give good looking skin and use long time safely.., thanQ"
},
{
"id": 179519,
"tgt": "Suggest treatment for recurring boil on scalp",
"src": "Patient: Hi doctor.! My baby is 16 mnths old now.1mnth back she gt boils on her scalp which later filled with pus and cause ictching.with one pediatrician suggestion I used antibiotic syrup and ointment.bt again it recurred in the same area causing much problem.pl help me. Doctor: Hi,Thank you for asking question on health care magic.If recurrent boils are coming,do a culture and sensitivity of the pus from the boils and the appropriate antibiotic suggested by the sensitivity report may be used in consultation with your doctor.Antihistamine like cetzine may be used to relieve itching.Hope this answer will serve your purposePlease feel free to ask any more queries if requiredTake careDr.M.V.Subrahmanyam MD;DCHAssociate professor of pediatrics"
},
{
"id": 139763,
"tgt": "What causes TIA?",
"src": "Patient: my wife is suffering from TIA.All the tests like MR angiogram,EEG,CT can,Dooppler test are normal .She is not diabetic nor blood pressure .ALl her blood analysisi .lipids.tryglecerides ,LDL.HDL are quitre normal.what care to be taken,she got it two times ina span of 3 months w.first time it was for 1 hr second time it was for 30mint. kindly advise her ageis 45 years wt 51 kg ht 4 11 Doctor: Hi, Get a prolonged Holter monitoring done. In young people, it could be cardiac arrhythmia. If needed, ILR (implantable loop recording) can be done. Hope I have answered your query. Let me know if I can assist you further. Regards, Dr. Sudhir Kumar, Neurologist"
},
{
"id": 206362,
"tgt": "What causes irritability and trouble in sleeping?",
"src": "Patient: hi, i am worried i have anxiety. i get irritated and annoyed very easily, always tired/stressed, trouble sleeping and cringe at any simple sound.. mostly chewing. i also have hypothyroidism and I'm afraid this is why i feel this way. i spend most of time in my room because i feel most comfortable there rather than in the living room with my family because they're loud and i can't take the sound of there chewing Doctor: HiI had gone through your query.Stress and anxiety can cause sleep problem like insomnia and irritation.Need to evaluate underlying stress and depression.Hypothyroidism also associated with depression.Antidepressant like escitalopram/sertraline/peroxetine etc can useful to regulate your mood and irritation.Anxiolytic like etizolam/clonazepam will be useful for sleep and anxiety.But you have require proper consultation before taking any medicines.So consult and get better assistance.Still if you have query then feel free to ask me on directly on http://doctor.healthcaremagic.com/doctors/dr-vishal-garala/68046Thank you."
},
{
"id": 91455,
"tgt": "What causes left sided abdominal pain?",
"src": "Patient: i alway have abdominal pain on my left hanside it affect my leg, it make me sweet, feel like throwing up but i cant, going the toilet but the nothing. i have the pain every single day after an hour. please help it is for 7 years now, i go to c the doctor the say there nothing wrong they cant see anything. Doctor: Hi.7 years is a long time to suffer so long...The pain is left sided you say and has also affected the leg on that side and other history noted.I hope by this time you have consulted a Gastroenterologist and undergone minimum of colonoscopy and CT scan of abdomen. If not yet done , please go for immediately. This is necessary as the proper diagnosis is of utmost importance for a proper treatment. I hope you have already must have used anti-spasmodic medication and that for irritable bowel syndrome"
},
{
"id": 182249,
"tgt": "Suggest treatment for swollen and stiff gums after cleaning gums",
"src": "Patient: I have periodontal disease, very bad, bone loss. I went to the dentist last week. We set up a plan to start with deep cleaning of my gums. Since my appointment and before my first cleaning, I have an issue with my back bottom tooth. Last tooth. It became very sensitive and painful. I went this morning and am a bit frazzled since then. My gums feel swollen. So I'm taking advil. When the Advil wears off, the swelling is so bad I almost feel as though I can't open my mouth, I can but feels stiff from swelling. She insisted I'm clenching my teeth. Quite the opposite. Since it's feels odd, I'm not closing my teeth together at all. Not at all, want to make that very clear since she seemed not to believe me. Without doing another x-ray, she wants to give me a large dose of antibiotics to see if it helps. I am a very special case, allergic to most antibiotics. Only one I've been able to take is a sulfa drug called Bactrim. She said, wouldn't work on that type of infection. Said she'd call pharmacist to see what they could give me. Disappointed in no x-ray and that she was questioning my teeth clenching. Advice please Doctor: Gums can become swollen as the result of inflammation due to poor oral hygiene. Since sticky plaque at the base of the teeth is what causes the gums to swell, brushing and flossing your teeth regularly, particularly at bedtime, can help prevent plaque from forming. Proper tooth brushing technique involves moving the brush in small, circular motions. Eating and drinking from dishes, glasses and eating utensils, which have not been properly washed and sterilized also transmit the bacteria that can cause gum disease."
},
{
"id": 39244,
"tgt": "Suggest remedy for sore throat,body pain and fever",
"src": "Patient: hai i am 15 year girl.. i have slight fever and i have whole body ache and from this morning i have pain in my throat and i could see some acne like in my throat inside .... pls pls doctor i am suffering a lot i am writing my exams now and i am suffering Doctor: Hello,Welcome to HCM,The history and symptoms suggests me that you may be having upper respiratory infection, which has led to sore throat,body pain and fever.The symptoms are due to release of mediators of inflammation which is causing all these symptoms.For your symptoms I would suggest you to follow1.Drink plenty of water2.Gargling with chlorhexidene mouth wash3.Oral Bactoclav, 625 mg, twice daily for atleast 5 days.4.Oral NSAIDs to relieve pain and fever.These medication will reduce the symptoms and improve the condition.Thank you."
},
{
"id": 13894,
"tgt": "What causes itchy rashes on the neck and arms?",
"src": "Patient: Hello my name is jacqueline, I had a question I began having this rash on my back on my neck it happened before going to bed I scratched myself and ignored it and went to bed. The next morning I woke up and realised the rash had spread to my ears and my ears got a little swollen and they were so itchy and hot. An hour later it had spread all over my face and its been like this for a week already and its going to my shoulders and my face is fool of the rash. I just want to know whats wrong with me help me please. Doctor: Hi, The rash could be erysipelas which is a bacterial infection of skin. It is usually associated with fever. You can initially apply a topical antibacterial cream. But, ideally, you should visit your Dermatologist immediately, to confirm the diagnosis and to prevent worst outcomes. Hope I have answered your query. Let me know if I can assist you further."
},
{
"id": 135352,
"tgt": "What causes weakness in arms and popping sound in joints?",
"src": "Patient: Hi everyone. I am a Mom of a 4 year old girl and I am 30. Lately I have been experiencing or having a hard time gripping. My arms get tired easily. I am having a hard time to type fast. I ve also started to have this weird sound like my joints (ankles, knees, elbows, wrists, & shoulders) popping with slight movement. My shoulders felt stiff sometimes. My calves felt like I took a long run and they re slightly painful. I also noticed that my great toe nails are slightly brittle. A month ago I had a surgery. I had excision of bartholin cyst. I had anesthesia, the less invasive one as the anesthesiologist put it. I don t know if it has something to do with it, but I feel like it s an important detail to miss. I also had my daughter breast fed for 22 months. I never had issues with my physical activities before until recently. I never had history of sickness. Based on the last x-ray and blood tests I had prior to my surgery a month ago everything was normal. Your opinion will somehow help me and I will truly appreciate it. Sincerely, Youngladynextdoor Doctor: HIThank you for asking HCMI have gone through your query with diligence. Reactive arthritis or post infectious arthritis is a possibility in your case. Blood routine including ESR , C reactive protein and Uric acid will be beneficial to find out the reasons. A swab test and culture from genital area will be helpful to rule out any infectious causes also. Calcium and vitamin D level also should be checked. NSAIDs , physiotherapy and exercises will be helpful to get relief from this. A consultation with a Rheumatologist is well appreciated.Let me know if anything not clear.I am happy to help you.Thanks."
},
{
"id": 181445,
"tgt": "What are the repercussions of wisdom tooth removal?",
"src": "Patient: I have pain in tooth n also in head .then i go to the dentist . He said the x rays says that ur all wisdom tooth touch the tooth from all sides of jaws like left, right , up , right.due to this i have headache all the day .he said that go to the oral surgen and remove all wisdom tooth. After all my question is if i removed all the wisdom tooth then after will be any problem to the mouth or teeth or any other issue like jaws pain etc .tell me the answer. Doctor: Hi..Welcome to HEALTHCARE MAGIC..I have gone through your query and can understand your concerns..As per your complain in case if you have pain related to wisdom teeth there is a possibiliity that they are impacted and there is also a possibility that the gum flap over the wisdom tooth is inflamed leading to Pericoronitis and causing toothache as well as radiating pain to head..As you are advised for extraction of the wisdom teeth, you should go ahead with it but get it extracted only by an experienced Oral Surgeon..As the wisdom teeth are rudimentary and does not participate in chewing, therefore extraction will not cause and difficulty in eating as well as if done properly there are no post extraction complications too..There can be some pain and swelling after extraction that will be controlled with anti inflammatory painkillers and antibiotics will be advised to you to avoid infection.. So my suggestion will be to get extraction done by a trained oral surgeon and follow the instructions that will be given post extraction for avoiding any complications..Hope this information helps..Thanks and regards.Dr.Honey Nandwani Arora."
},
{
"id": 102703,
"tgt": "What are the best possible treatments for asthma ?",
"src": "Patient: Hi, I have had increasing dry mouth, nasal passage and eyes for 3 months. I have had blood tests done and Sjogrens Syndrome was ruled out, only weak ANA was noted. I have asthma and take a preventer for that. I am a 38 year old female. I used to take Amiltryptyline but have stopped since the start of the dryness. Any further suggestions would be helpful? Doctor: Welcome to HCM.Inhalation therapy is best available treatment foe asthma.PEFR,spirometry and chest x-ray is required before starting any treatment for asthma.Long acting/short acting steroid with bronchodilators rotacaps and inhalers are available and used according to severity and investigations.Long course of levocetrizine with montelukast will be helpful to reduce asthmatic episodes.Steam inhalation will be helpful.Do regular breathing exercise.Try to avoid triggering factors.Try to avoid cold,foggy,dusty,smoky exposure.Consult pulmonologist for proper guidance and medical management."
},
{
"id": 171965,
"tgt": "Will Asthma be the reason for having a hoarse voice?",
"src": "Patient: Hi. My 5 year old daughter has had a really hoarse voice since yesterday. We do have a history of seasonal asthma, but no problems this last fall which was great! Now she started to have a really hoarse voice, no fever, complains of bottom front of the neck hurting when she talks but not when she swallows. She also has been followed for what we were told a osteosarcoma on her clavicle. We did miss the last follow up. Now it seems the end of the clavicle nearest to the throat is larger than the other side and she complains its sore when touched. What is going on? Laryngitis or something else? Her pulse ox is reading ok...between 95-99 but or prbpm is going from 50 up to 125 down to 65 and back up again. Let just say its all over the place. Help! Doctor: Asthma is not the reason for her hoarse voice. This could either be laryngitis or involvement of the vocal cords or the nerve that supplies the vocal cords for proper functioning of the cords. This nerve is known as the recurrent laryngeal nerve and it is part of the tenth cranial nerve (the vagus nerve). It descends into the chest in front of the lungs and goes very close to the clavicle's inner end. Take her urgently to her primary physician who may refer her to the correct specialist.Dr. Taher"
},
{
"id": 112158,
"tgt": "What is the cure for lower back pain and stomach pain?",
"src": "Patient: I have a dull ache in my lower back/sacrum area. I keep experiencing tummy pain that feels like my tummy is just about to rumble like I haven't eaten for days. Sometimes it does rumble but I have eaten. Seems to be worse at night but is there all day Doctor: your symptom suggestive of nonspecific back pain.i advised you first try conservative trratment like Rest for 3days, muscle relaxant and physiotherapy -IFT/TENS for 6 week with swimming excercise which help you alot"
},
{
"id": 208327,
"tgt": "Why do I stammer due to fear?",
"src": "Patient: Hello doctor... Im 25yrs old and I have stammering problem. I stammer due to fear. I'm unable to get through any of the interviews due to this problem. I try to keep myself cool while attending the interview and after entering the room, I become nervous and loose my confidence and start stammering. I'm really worried about my problem. Since my childhood till now, I had to face criticism bcoz of this problem. Is there any solution to my problem? Please help me out. Doctor: Hi,Thanks for writing.Stammering during heights of emotion is a frequent occurrence. May be trying to relax in such situations will be helpful. You can try relaxations and yoga. If need be, you might consult a trained psychologist.Hope this helps,Dr A Rao"
},
{
"id": 195237,
"tgt": "How to treat a torn frenulum?",
"src": "Patient: I used to have a bracket on one of my molar teeth with a hook for a rubber band for invisalign treatment. Two years ago when I was performing oral sex on my boyfriend, his frenulum got caguht on the hook. It took me about 30-60 seconds to get it off. He did not break but it tore and was bleeding. He was in a great deal of pain. Based on all of my research online it seemed like it would heal up on its own and be fine. However it is still an issue to this day. He has lots of pain with insertion durring sex and cannot mastrubate more than once in a day or have sex for very long without having alot of pain and discomfort. This has not only caused physical discomfort but it has caused him to be depressed and have low self esteem and less of a will to live he still resents me for it. We rarely have sex anymore. He refuses to see a doctor because he is pretty sure the only option is surgery to either repair the frenulum or have a full on circumcsion. He refuses to have surgery in his most sensitive area and says that the most he would do to treat it is an ointment. I would like to know what the options are before making any assumptions, but I have a feeling he may be right. Are there any options other than surgery? Doctor: Hello and Welcome to \u2018Ask A Doctor\u2019 service. I have reviewed your query and here is my advice. The tear might have fibrosed and causing the pain and discomfort. So it is better to go for circumcision now. Circumcision will cure all his problem. Consult a urologist and go for circumcision. Hope I have answered your query. Let me know if I can assist you further. Regards, Dr. Shinas Hussain"
},
{
"id": 131235,
"tgt": "Having groin and knee pain for over three months. What should I expect from a bone scan?",
"src": "Patient: hi I am a fifty seven year old female I have been having Groin and knee pain for over three months. I am going to have a bone scan and not sure what to expect. can you help me please. I feel like I am going crazy. I have hade an x-ray and blood taken all come back good .I just want someone to tell me what it is please can you help. Doctor: Don\u2018t worry, bone scan is non interventional non painful quick investigation. This important in detecting the presence of abnormal mass, swelling, tissue. You should do it and wait the result with no worru, i hope you are well."
},
{
"id": 37764,
"tgt": "How to treat UTI?",
"src": "Patient: I have treated UTI with different drug for over 2 weeks . the last drug was ornilox and it helped. It was prescribed for only five day. I started taking on the last 2 days of taking taxim-o. This last drug helped and the pain went away the last day i took it but after 3 days the terrible pain came back. Previoulsy to taking any drug a culture and sensitivity was done. It found the common bacteria e. coli in teh urine +++++. Now I am hurting very much. Doctor: HelloWelcome to HCM,Your history and symptoms suggests that you have UTI secondary to Bladder Neck Obstruction.Bladder Neck Obstructions causes incomplete emptying of bladder during micturition resulting in to some amount of urine remaining in bladder .This stagnated urine ids the source of infection and getting UTI.Taking antibiotics will solve your problem temporarily.I would advise you go for endoscopic incision of bladder neck (Bladder Neck Incision) so that your bladder neck will remain wide open and help in emptying bladder completely during every micturition ,and there by prevent frequent episodes of UTI.This is simple surgery you need to consult the urologist for opinion and treatment.Thank you"
},
{
"id": 207933,
"tgt": "Suggest painless method of suicide",
"src": "Patient: I am sleepless from the last month after i broke up with my boyfriend. I am just hating myself and dont want to live anymore. Just feel like different voices always echo in my ears and dont feel like talking to my friends and family. Tell me a painless method of suicide Doctor: Hello and welcome to Healthcare Magic. Thanks for your query.I understand that you are going through a very difficult time and are feeling hopeless and suicidal. From the description of your problem, it appears that you are going through a depressive phase. This is what is making you feel dejected, hopeless and not worth living.Since you are having such severe depressive thoughts and suicidal ideation, I would suggest that you speak to someone immediately about this. You can share your feelings with a close friend or relative or can even call the suicide helpline in your place. Even though things seem very hopeless and negative, I'm sure that with professional help you will be able to overcome this depressive phase soon. There is help available in the form of anti-depressant medication and counselling which can mitigate your symptoms. So, please don't hesitate to seek professional help immediately.Wish you all the best.Regards,Dr. Jonas SundarakumarConsultant Psychiatrist"
},
{
"id": 13078,
"tgt": "Suggest treatment for painful rash on the fold of inner thigh",
"src": "Patient: I have a very sore and painful rash on the fold of my inner thigh near my vagina. I've tried hydrocortisone cream but it burns and it is very unbearable. When I shower and water gets on it It's very sore and very painful. What can I put on what I think is a fungal Infection to ease the pain and help it heal? Doctor: Hello,I read carefully your query and understand your concern. The symptoms seem to be related to a fungal infection. I suggest using Clotrimazole cream for local application. I also suggest using Diflucan for three days.Hope my answer was helpful.If you have further queries feel free to contact me again.Kind regards! Dr.Dorina Gurabardhi General &Family Physician"
},
{
"id": 190301,
"tgt": "Swelling over broken tooth, pressure on pressing the corresponding cheek causing runny nose. Chances of dental abscess?",
"src": "Patient: I have a swollen area over a broken tooth . It is above the top of the tooth where the cheek meets the gum. I can t see it, but can feel it. ThEre is no pain associated with it but some pressure if I press on my cheek right below the cheekbone near my nose. And my nose runs. Sould I be worried about a dental access, or is it more likely related to a sinus infection? My ears have been clogged up for weeks since allergy season started but my noes isn t too stuffy. Doctor: Hello Welcome to HCM I read your problem & would like to tell you that if you have broken teeth & swelling adjacent to that then it can be due to some abscess formation.I would like to ask you that is there any pus discharge from same tooth?? You need to get an IOPAR done for same tooth for getting it properly treated. You might also have problem of sinusitis because of other symptoms like running nose. You need to consult your dentist as well as ENT specialist for same without making any further delay. Take Care Regards Dr.Neha"
},
{
"id": 24812,
"tgt": "What does fainting episodes with heart murmur and high blood pressure indicate?",
"src": "Patient: Hi my daughter has had fainting episodes, and flushing since 12pm yesterday, we saw a g.p last night that is not her practice as she staying with me. He said she had a hart murmer, there was blood in urine, and blood pressure up, also a tempreture, he said to monitor her last night and if she looses conciousnes then call 999. She is very tired and sicky but wont wake easily, her age is 21, and we came back from ireland yesterday. thankyou. Doctor: Hello there This report doesn't look too good.. Your daughter may be suffering from endocarditis a potentially fatal condition if Your G.p has suggested right!!I feel she needs to be hospitalised and monitored and put on heavy antibioticsPlease do let me know of the progress in her caseWishing her a speedy recoveryCheers and all the best"
},
{
"id": 199710,
"tgt": "What could be the reason for having white moles on my penis and pain during urination?",
"src": "Patient: I m having white moles on my upper skin of penis i had sex without using condom 2 months its a kind of sti i took augmentin twice for 10 day aftr that i took azythro mycin now i am having that problem on my penis i guess its sti kindly advice me medicine asap some times i have pain while doing urine Doctor: Hello I can understand your concernLooking at your history this could be related to STI or UTI in my opinion your should consult your GP doctor for examination and a battery of tests like blood for hemoglobin CBC liver and thyroid function test , urine for culture and sensitivity test you may require a course of antibiotics depending upon the clinical findings and lab reportshave plenty of fluids maintain good hygienemultivitamins supplementsHope.this help youPlease fell free to ask for.more clarification I will happily answer you best wishes"
},
{
"id": 53503,
"tgt": "Is Lisinopril safe while having hepatitis c?",
"src": "Patient: i was recently prescribed lisinopril for hypertension 152/98 and i drink at least a 40 oz. of 7.5 malt liquor or more daily. This averages to to be a six pack or more. After reading what the possible side effects are i stopped taking it. i am also hcv+ and would like to know if i continue the medication and drinking if it will be ok on my liver. Doctor: hi.thanks for posting query at hcmadvise:- quit alcohol- continue medication if BP has been repeatedly high in more than 2 visits and if diet/lifestyle modification ( decrease salt intake, NO red meat/ junk food/ regular exercise ) did not help to maintain normal BPwish you good n sound healthregardsDr Tayyab Malik"
},
{
"id": 37123,
"tgt": "What should I do if the surgical incision is opened?",
"src": "Patient: My siter had a lumpectomy and removal of a few lymph nodes had to have 2 incisions under armpit and o Jackson Pratt pump inserted. Well her pump is suction into properly on call doc yesterday said see doc Monday but this morning the Tape came loose due to drainage from incision and the incision is completely open is this ok or do we need to go to er Doctor: HelloThank you for contacting Healthcare magic.After lumpectomy pain killers and antibiotics are given. It is not good sign that incision is completely open. It may increase the chances of infections and discharge which cause fever and may cause the condition severe.You must immediately consult your surgeon for complete closure of both the incisions and continue dressings and antibiotics.Hope all things will be ok.take care..."
},
{
"id": 41795,
"tgt": "Is IUI and laparoscopy needed for uterine TB infection with tube blockage?",
"src": "Patient: I'm 31 yr male & my wife is 28. we have been trying for pregnancy for last 2 yrs. We started treatment for infertility since last one yr. First with tablets, then with follicle study & injection for rupture. But no result. My wife was diagnosed to be having uterine infection of TB for which she is taking AKT tablets for last 7 months. Also during sonosalpingography, it was found that her right F. Tube is blocked. Now our gyanac is suggesting us first undrgo 3 cycles of IUI & then undergo Laparoscopy to clear the problems. Pl. suggest if this is right route. I'm from Mumbai, India - thanks Doctor: Hi welcome to healthcaremagic.I have gone through your question.If i am it treating doctor then i would suggest to do laparoscopy first and to do recanalization of blocked tubes. And if laparoscopy done successful then advise to go for naturally and try this for atleast 6 months.If not success then can go for iui or other methods. Hope i answered your question.Would be happy to help you further.Take care."
},
{
"id": 46633,
"tgt": "Suggest treatment for alport syndrome",
"src": "Patient: My son has Alports syndrome and has developed leg pains and some fatigue in this past 12hrs. Is complaining of pain when urinating and using bowels.He also has hashimotos thyroiditis My son is Male 14ys,60kgs,last bloods were reasonable.Blood pressure norm.Takes ramipril and oroxine. Doctor: Alport syndrome patients may develop kidney problem, eye problem and hearing problem. so consult nephrologist, ophlmologist and ENT surgeon. he may require ACE inhibitor like drugs to retard progression of disease."
},
{
"id": 123237,
"tgt": "What causes pain in the tailbone area?",
"src": "Patient: My daughter has had tailbone pain for a couple of weeks. Had an X-ray and no fracture. Dr. Said to watch for worsening symptoms and to advise if any problems with bowel movements. Last couple of days she has seen a small amount of blood when she wipes. What could this be? Doctor: Hello, Please let me know whether she had a history of injury, sprain, heavy weight lifting. For that, you can take tab. motrin and apply diclofenac gel or spray with heating pads. In case of not getting relief, I would suggest you to MRI back after orthopedic consultation. Hope I have answered your query. Let me know if I can assist you further. Regards, Dr. Shyam B. Kale, General & Family Physician"
},
{
"id": 53230,
"tgt": "What are the symptoms of hepatitis?",
"src": "Patient: I am very much pleased to find you. Actually I have 7 years old daughter with the symptoms of Hepatitis A. I have done Laboratory Tests of Liver profile (LFT)S.TOTAL BILIRUBIN\u2026\u2026\u2026\u20263.7 mg/dlS.DIRECT BILIRUBIN\u2026\u2026\u2026...2.0 mg/dlS.INDIRECT BIILRUBIN\u2026\u2026...1.7 mg/dlGAMMA GT\u2026\u2026\u2026\u2026\u2026\u2026\u2026\u2026226 IO/LSGPT (ALT) \u2026\u2026\u2026\u2026\u2026\u2026\u2026\u2026.782 IO/L RECHECKEDALKALINE PHOSPHATASE\u2026\u2026.628 IO/LHEP. A ANTIBODY (IGM) \u2026.. Reactive (16.48)Cutout rate is\u2026\u2026\u2026\u2026\u2026\u2026.. 1.0 (Test performed by ECLIA)Lymphocytes\u2026\u2026\u2026\u2026\u2026\u2026\u2026.47%Eosinophils \u2026\u2026\u2026\u2026\u2026\u2026\u2026..09%Monocytes \u2026\u2026\u2026\u2026\u2026\u2026\u2026\u202608%Total cell count \u2026\u2026\u2026\u2026\u2026\u2026. 100All other TEST statements are normal.Please can you tell me conditions of above mentions tests?Thanks and regardsGhazanfar AbbasFather of 7 years old daughter. Doctor: Salaam and hi.thanks for posting query.HepA is self limited disease.nothing to worry.hope you also have ruled out HepB infection. No need to test for HepB if your sure that your daughter has been vaccinated against HepB virus.Advise:- LOW fat diet should be followed, AVOID junk food and beverages- decreased oil consumption (oily food)- NO red meat- green vegetables should be ingested daily- use lemon juice (lemonade) once in a day-\"recheck liver enzymes after 12 to 24 weeks\" and/or ultrasound.any further questions are welcomed.http://doctor.healthcaremagic.com/doctors/dr-tayyab-malik/71638hope to answer your concern.wish you good health.regards,Dr Tayyab Malik"
},
{
"id": 140853,
"tgt": "Is there a type of traction system that could help relive this debilitation problem?",
"src": "Patient: Back and leg/foot problems; X ray and MRI reviewed by doctors: Seemingly not too severe damage to T 12 / L 1 in spinal column (due to a mild fall onto her left hip) has approximately 75% interfered with apparent brain access to legs and feet. Result continues to be extreme leg weakness, i.e. 75% +/- debilitated walking after 8 months recuperation and carefully followed rehabilitation exercises. Walking is limited to a wheeled walker and for not too far a distance (100 feet +/-). Before this condition walking was normally good. Debilitation has been diagnosed as \u201cpressure on the spinal cord\u201d or \u201cpinched nerve\u201d. Ongoing Physical Complications: 1.\u00a0\u00a0\u00a0\u00a0\u00a0Right leg experiences consistent and complete \u201cdrop foot\u201d 2.\u00a0\u00a0\u00a0\u00a0\u00a0Numbness in both feet 3.\u00a0\u00a0\u00a0\u00a0\u00a0Occasional (25% of the time) sharp, debilitating pains in upper and lower portions of both legs 4.\u00a0\u00a0\u00a0\u00a0\u00a0Constant weakness through entire leg area Is there some type of traction system that could help relive this debilitation problem - before any back operation may be attempted? Ruth is my mother in law and beloved matriarch to all in our extended family. She is 90 years old, a competition bridge player, always stayed in good physical condition. Other than this current, grievous, physically debilitation condition Ruth seems capable to continue for years once past this hurdle. She continues to live independently and due to this debilitating condition receives any assistance necessary from family and friends. Doctor: Hi, Question: Is there a type of traction system that could help relive this debilitation? Answer: No. This condition Requires Surgery. Hope you found the answer helpful. If you need any clarification /to have doubts / have additional questions / have followed up questions, then please do not hesitate in asking again. I will be happy to answer your questions. Hope I have answered your query. Let me know if I can assist you further. Take care Regards, Dr Bruno Mascarenhas, Neurologist, Surgical"
},
{
"id": 199650,
"tgt": "Why am I not able to reach the orgasm during intercourse different from masturbation?",
"src": "Patient: My spouse were involved in an intimate encounter yesterday (last night). Although it took a while, I was able to achieve an erection however, regardless of the time SHE devoted, or the numerous different ways she attempted, I could not ejaculate and in fact never even experienced the orgasm experience. I/we were not arguing with each other, there was no tension and we had nothing but time to devote to our attempted sexual experience. Would you PLEASE tell me how this could happen to a 55 year old male who most other times has no problem with masturbation or ejaculation? Doctor: HelloThanks for query.Inability to ejaculate even after long foreplay and successful penetration and spending adequate time in intercourse does happen once in a while in every ones life and is nothing to be worried for .Do not give too much importance and think over it and get prepared to enjoy your next sexual activity without having any apprehension and anxiety in your mind.Dr.Patil..."
},
{
"id": 38494,
"tgt": "Is anti rabies necessary after dog bite if its vaccinated?",
"src": "Patient: my mother got very minor bite by dog over two-three layers of cloth(Saree) 3 days back. she enquired about the dog. it is a pet dog & the owner told that it doesn't bite, it just holds.& it is proprly vaccinated.Is it necessary to get anti-rabies vaccination? Can it be started now? Doctor: If you are certain the dog has been vaccinated for rabies, there is no need for your mother to get the anti-rabies vaccination. If it's not certain, it's not too late for her to start the series. Hope this helps."
},
{
"id": 164184,
"tgt": "What causes stomach pain?",
"src": "Patient: My daughter is 5 years old and has been experiencing headaches, coughing, high temperatures of 103 and chills, and lower abdominal pain. She has slept a lot for the past 3 days and woke up feeling a little better this morning. After about 3-4 hours of playing and doing well, she s now crying and complaining that her lower stomache is hurting and her private area. I was thinking that maybe a bladder infection was possible but she s not going to the bathroom frequently. I was also thinking constipation because of the childrens advil she has taken of the past few days to lower her temperature. Any ideas? Doctor: Hi... I understand your concern. By what you say I feel that your kid is recovering from a simple viral illness. The possibilities here are - 1. Pain related to constipation.2. Cystitis or inflammation of the urinary bladder.3. Extremely rare for the pain to occur in that region but gastritis is still a possibility.Regards - Dr. Sumanth"
},
{
"id": 143288,
"tgt": "What does my MRI suggest?",
"src": "Patient: in my lumber spine MRI POSTERIOR and right paracentral protrusion of L4-L5 intervartebral disc causing compression over ventral aspect of dural theca and exiting nerve roots AND POSTERIOR DIFFUSE BULGING OF L5 S1 INTERVERTIBAL DISC A CAUSING MILD COMPRESSION OVER VENTRAL ASPECT OF DURAL THECA ANS EXITING NERVE ROOTWHAT ARE THERE MEANING PLS GUIDE ME AND SUGGSEST FOR TREATMENT Doctor: Hello!I carefully read your MRI report and would explain that these findings indicate two intervertebral disc herniation in two levels : L4-L5 and L5-S1. They both do not cause spinal cord compression but exit the nerve root, which means that they can cause radicular pain in the inferior limbs, by compressing the nerves. I recommend consulting with a neurosurgeon on the possibility of surgery. Holding a lumbar brace could be helpful!I would also recommend avoiding prolonged sitting or standing up position and weight lifting. Hope to have been helpful!Kind regards, Dr. Aida"
},
{
"id": 114785,
"tgt": "Suggest remedy to lower the monocyte level",
"src": "Patient: Hi,thks for ur attn,my recent blood test shows that monocytes are above the normal range. How can I reduce it,and which food to avoid,plz.My bad cholesterol also is a bit high and Hdl a bit low,thus showing average risk of heart disease,thks in adv.sir. Doctor: Hi, dearI have gone through your question. I can understand your concern. Your monocyte count is having gh but it may be due to some viral infection. No need to worry about that. Just drink plenty of water. For cholesterol you should take high fiber diet & low fat diet with high amount of polyunsaturated fatty acids. You can take drugs like atorvastatin or pravastatin to control your cholesterol level. It is prescription based medicine so consult your doctor and take treatment accordingly. Hope I have answered your question, if you have doubt then I will be happy to answer. Thanks for using health care magic. Wish you a very good health."
},
{
"id": 192234,
"tgt": "How can persistent dizziness be treated?",
"src": "Patient: nha,hello my name is noorul haq, i am male , 21 years old, i have taken one illness, one year ago i went to pakistan to take care, because my head was turning, my all skin had ach, all my body was taken pain, finally the doctor said that beside your nose there is water, then the doctor gave me medicine after a mounth i got healthy, now we are in kabul , there is sometime earthquak, now i do not have bodyach, and other problem but only my head is turning. i think , i have taken drag, or cigrette, or others ,i think there is earthquak, but in reality there is nothing. please guid me that which specilist should i take? thanks Doctor: Hello,You probably need to see a general physician first in order to rule out if all these symptoms are due to ear related or brain. It's only possible after your proper checkup physically. Then he will be able to refer you to an ENT specialist or a neurologist.Hope I have answered your question. Let me know if I can assist you further. Regards, Dr. Sameen Bin Naeem, General & Family Physician"
},
{
"id": 125158,
"tgt": "What causes shoulder pain during boxing?",
"src": "Patient: my name is abhinav and iam 19years old 176cm tall and 66kg weight.iam doing boxing by 5 years and my sholder poping out by last year and occours countinuesly during training.not very pain ful but some pain on that time only not falling down from socket but just stuckking from some where Doctor: Hello, As a first line management you can take analgesics like paracetamol or aceclofenac for pain relief. If symptoms persists better to consult an orthopaedician and plan for an MRI scan to rule out ligament or tendon injury. Hope I have answered your query. Let me know if I can assist you further. Regards, Dr. Shinas Hussain, General & Family Physician"
},
{
"id": 4940,
"tgt": "Trying to conceive. Taking Fertomid and Utrogestan. Chances of pregnancy?",
"src": "Patient: Hi,I am 28 years old, weigh 70kgs and have been trying to conceive for over 2 years. I recently went to get myself checked out as I fell pregnant with my son 2 weeks after going off the pill who is now almost 4 years old. The gynie found and removed adhesions that were attached to my fallopian tubes as well as an infection that in my right tube that cause permanent damage to the tube. She has now put me on fertomid as well as utrogestan as I was not ovulation and there seems to be a hormonal imbalance of some sorts. I would like to know what are the chance of me falling pregnant on these meds? Doctor: hi i had gone through your query and understand your concerns. i would come up with two most possibilities to fit your case.these include1.first possibility is,PCOD;PCOD POLYSYSTIC OVARIAN DISEASE is one of the main factor of infertility in females nowadays. Women with PCOD have High levels of male hormones (androgens),An irregular or no menstrual cycle ,Many small cysts (fluid-filled sacs) in their ovaries (although this may not always be the case).due to above reason the OVUM CANT BE RELEASED.confirm with ULTRASOUND SCANNING FOR PELVIS.2.second possibility is,HYPOTHYROIDISM;confirm with THYROID PROFILE..TREATMENT PLAN;i advise you taking Homeopathic medicines gives safe and permanent cure for your complaints of infertility of problems.consult your local homeopathic physician for correct diagnosis of the case and remedy to fit your complaints. or mail me at drravib4u@yahoo.co.inI hope this is helpful for you, thank you"
},
{
"id": 169257,
"tgt": "What could tremors, dizziness after BC pills intake for painful periods indicate?",
"src": "Patient: my 14 year old daughter was born with gastroschisis and corrected with surgery. No problems until this past year. After periods of severe stomach cramping and pain and nausea, she had her gall bladder removed this past February. The surgeon also freed up her right fallopian tube as well as removed adhesions. The surgery corrected most of the abdominal cramping; however, she still has very painful periods which to date, birth control is not providing much relief. While on her period, she experiences painful unrination. We believed that the new birth control was the cause of the headache she has had for 4 weeks; however, her doctor doesn t think so. Dizziness and shaking hands accompany her headaches. Recent CT Scans didn t show any problem with kidney stones. She has had many tests but still isn t feeling any better. The symptoms and list of tests is long. I am tempted to take her to ER just so they will do another round of tests all at once. Doctor: Being born with gastroschisis is a major malformation with lifelong consequence potential. Adhesions are one of the reasons. Other hidden malformations are possible.That is to say, her pains and complains may be long or lifetime, without it meaning incomplete medical management of her troubles.I\u00b4m sure you can find a doctor who can view her problem with a fresh perspective considering all factors, not only physical, and review every system for interactions.An ER will not do that.Best of lucks,Dr. Lino, MD"
},
{
"id": 121802,
"tgt": "Suggest treatment for ankylosing spondylitis",
"src": "Patient: I am a 29-year old female, recently diagnosed with AS (ankylosing spondylitis). I also had spine surgery in July, for 2 thoracic discs that were herniated. I just started Humira last Thursday and weaned off of Prednisone that day as well. In the meantime, over the past 2-3 weeks, I have had a lot of pain in the area where I had my surgery. I am now on a 5-day steroid burst of Prednisone. I took the first dose of 25 mg today, and within a couple of hours, I began feeling like I had the flu...my entire body aches from head to toe. I have had this happen with a Medrol Dose Pack in the past, as well as after receiving an epidural steroid injection. Is this a common reaction from higher doses of Prednisone? It usually only lasts about a day or so, and then I wake up feeling better the next day. Doctor: Hello, At higher doses of prednisolone, these are the common side effects. The symptoms will gradually decrease in next few days. There is no need to worry. Hope I have answered your query. Let me know if I can assist you further. Take care Regards, Dr Praveen Tayal, Orthopaedic Surgeon"
},
{
"id": 40860,
"tgt": "What causes fever while undergoing IUI treatment?",
"src": "Patient: I am ANUSHA aged 31. Married for 4 years. We have nt had intercourse because he had erectile dysfunction. Went for iui. Before iui underwent Lap procedure with fentons. 20 and 21 of this month had my iui. My husband provided good amout of sperm. But today, from morning i feel tired and run a temp of 99.4. Asked to take dolo650. If continues asked to report. Why does fever occur and what are possibilities of my concepton. is there early symptoms. Also aked to lie tomy right which actually hinders my sleep. Pl advise Doctor: Hi Anusha, thanks for the query. Let me help you understand few things. Any external procedure carried out on our body has to be considered seriously and carries a risk of infection. Your fever could be due to any viral or bacterial infection unrelated to the procedure carried out or any mild infection due to embryo transfer which is very unlikely as you don't have any other pelvic symptoms. You have been advised rightly to take paracetamol to keep your temperature low. Do hydrate yourself well and get plenty of rest. Regarding your question about chances of conception, there are no direct studies done in humans how fever affects fertilisation and embryo implantation. The animal studies done shows it possibly affects egg and sperms, fertilisation and embryo implantation. With such equivocal evidence, all that one can do is treat fever effectively. I suggest you to take care of yourself and be hopeful. Good luck."
},
{
"id": 128666,
"tgt": "How can pain and pulsating sensation in arms be treated?",
"src": "Patient: Hi,I went to GYM last week for 3-4 dyas. I had lot of pain in arms as i was starting after 3-4 months. Slowly pain went away but i feel little pumping near elbow on and off in a day. I am observing this from last 3-4 days, i never had this thing before. Anu suggestions? Doctor: don't worry. you must have wanted to become like Arnold in one day resulting in lifting much weight. don't worry rest and doing day to day activities will do wonders"
},
{
"id": 81509,
"tgt": "What is the remedy for flu?",
"src": "Patient: I have had symptom like flu for almost three weeks. It started with very thick sticky phlem from my lungs and severe congestion of nasal sinus . I have coughing constantly. It does not matter what I take the mucus just keeps reproducing. I also have a sore cushy spot on the right side in the roof of my mouth. It feels like a sore or type of ulcer underneath the skin. I had double pneumonia with ARDS about 4 1/2 yrs. ago. I have had the flu and pneumonia vaccine and this is the first time that I have been really sick since then. Doctor: Thanks for your question on HCM.In my opinion you should first consult pulmonologist and get done chest x ray and PFT (pulmonary function test). There are two possibilities in your case.1. Pneumonia2. Bronchitis.I don't think its flu. As viral infection is self limiting and does not last for three weeks.So we need to rule out above mentioned causes first.So consult pulmonologist and discuss all these."
},
{
"id": 199484,
"tgt": "What are the side effects of excessive masturbation?",
"src": "Patient: Hi i am 29. I am badly involved in masturbation . Every day i make promise with self not to do it again,but can't help it. I masturbate 2-4 times in a day. Please help me out. I want to get rid of that.2. I am fat also. I want to reduce my fats. I joined gym but i feel tireness in my legs because of which i couldn't continue . I have acidity problem as well. Please help me in both the cases. Doctor: HelloThanks for query.You have been indulged in frequent masturbation 2-4 times in a day.Though it is harmless, excessive and frequent habitual masturbation does affect quality of erection and delays ejaculation.Please try to reduce the frequency of masturbation by keeping yourself busy in activities like sports,reading ,social work etc so that you will not get free time to masturbate.For acidity problem You can take H2 receptor antagonist like Pantoprazole twice daily Avoid spicy and junk food and include curd,milk in your diet..Taking small frequent meals at regular interval of 4 hours instead of having 2 full meals will help to resolve the problem of acidity.Dr.Patil."
},
{
"id": 172146,
"tgt": "What causes headaches with fever in 8 year old?",
"src": "Patient: my 8yr old son has been having headaches in his frontal lobe since he had a staring seizure 31/2 yrs ago he hasnt hed another since that time but he complains of a headache now and has a fever cold chills but is being his normal self when he was 2 yrs old he fell out of a grocery cart and hit his forehead he was seen by neurologist and had cat scan done and more nothing was found but high electrical activity on the left side of his brain im thinking i should have them check his head agin to be sure nothings there but this fever came out of no where today along with this headache Doctor: I think this fever is separate and not related to the previous seizures and headache...do not panic..wait for fever to subside.if not subsiding after four to five days then you can consult doctor .that is just to rule out bacterial infection."
},
{
"id": 131399,
"tgt": "What is the remedy for the pain in the tail bone extending towards the hamstring area?",
"src": "Patient: Hello I fell on my back side two weeks ago while snowboarding. At first it just felt bruised around my tailbone area then the pain went to the back of my legs (hamstring area). I cant isolate the pain like I could if I had pulled something. It also hurts in the back of my legs when I cough. Should I see my Doctor? Doctor: Yes, you need to see a orthopaedic doctor as you might have a hamstring injury. If there is tear suspected by doctor then you need to confirm this by MRI. You will need rest, Pain killer like Brufen and compression bandage, Some time if tear is big then surgery may be needed."
},
{
"id": 45642,
"tgt": "Can a kidney be donated by the parent to the child after losing a kidney due to grade 5 reflux?",
"src": "Patient: My son has been throwing up and feeling nauseous for a while. When he was 4 he had grade 5 reflux and because of that has only 1 kidney. He does have another small one however it only functions 5 to 7% because it was necrotic for 4 years till we found that he had grade 5 reflux. when he was little (under 4) he would throw up all the time when he ate too much. it seems to get better that he got older but now I am worried. Can I give him one of my kidney s? I am 58 1/2 and I figure he needs it more than I do? Doctor: Hi, Please note that any healthy person can donate kidney provided there is proper cross matching. You should consult a transplant surgeon and he will advise necessary tests. Hope I have answered your query. Let me know if I can assist you further. Regards, Dr. Varinder Joshi General & Family Physician"
},
{
"id": 101989,
"tgt": "Any suggestion for severe dry cough and mild fever?",
"src": "Patient: my son is dry coffing severely. More especially at night times and can not have sound sleeps. Coffing initially since 2 weeks has now aggraved. We have been giving him Vikoril AF syrup but no success. Since yesterday night we have also given Calpol 125 5 ml looking at mild fever. What is your medication suggestion now? Doctor: Hi,Medications can be fixed based on complete evaluation.For now based on your explanation, continue syp with 15 mg per kg along with salt water gargle. If any bacterial infection then antibiotics should be given.If more fever do tepid sponging. Hope this helps you. Regards"
},
{
"id": 81266,
"tgt": "What is meant by hyperinflation in chest?",
"src": "Patient: Hi, My husband was a first responder an 9/11 at the world trade center and has been monitored by Mount Sinai every 6 months. He went for his physical the other day and today we received a notice that he has hyperinflation in his chest xray and needs to follow up with his primary dr. What is hyperinflation and is something to be very concerned about? Doctor: Thanks for your question on HCM.I can understand your situation and problem.Hyperinflation in chest is seen in following conditions.1. In normal person if chest x ray is taken after full inspiration.2. In smokers due to COPD (chronic obstructive pulmonary disease)3. Age related.4. In asthma.And this should be confirmed by CT thorax and PFT (pulmonary function test).The above are possible causes.So if your husband Ia symptomatic with shortness of breath and coughing than better to consult pulmonologist and rule out above causes.If he is asymptotic than no need to worry much.If smoker than ask him to quit smoking as soon as possible."
},
{
"id": 114918,
"tgt": "What does Traces of sar tissue in white blood cells suggest?",
"src": "Patient: Thank you so much. I had blood work done and they say my white cell count is high/normal AND I have had blood in the urine....an ongoing thing for a few years. (Had a series of urology tests, 3 years ago and all they said is traces of sar tissue ....But, the white blood cell count being high, is new. I see a urologist and a hematologist next Tuesday, June 24. Thank you for your reply. Glenda Doctor: Hi, dearI have gone through your question. I can understand your concern.There are many causes of high white blood cell count. Most common cause is infection. You have blood in urine so you may have infection in your kidney or bladder or urinary tract. Please send me report of your complete blood count and urine examination so that I can help you further.Hope I have answered your question, if you have any doubts then contact me at bit.ly/Drsanghvihardik, I will be happy to answer you.Thanks for using health care magic.Wish you a very good health"
},
{
"id": 222903,
"tgt": "What causes urge to urinate and back pain during pregnancy?",
"src": "Patient: I am 36 weeks pregnant and ALWAYS feel like I need to urinate. When I go to the restroom, not much comes out. This accompanied by my constant back pain, which I've have for about a week and a half, and the contant pressure I have against my pubic bone, is causing me concern. Should I be worried? Doctor: Hi,The last few weeks of pregnancy are rather difficult for most women, as the weight of the growing baby on the bladder can cause frequent trips to the wash room.This is absolutely normal though troublesome.The growing uterus and baby also put weight and stretch on the back muscles which are pulled and can cause ache and pain. All these symptoms will subside after delivery .As a precaution just get a urine routine microscopy done to rule out any infection.Hope this helps.Regards."
},
{
"id": 108821,
"tgt": "What causes pain/burning in butt and lower back while sitting?",
"src": "Patient: Hi, may I answer your health queries right now ? Please type your query here... After sitting for 10 minutes, I feel a little bit of needle (or burning) pain on my right butt and low back. Seems the nerve is pressed. But I do not know what the root cause is. The sympton has been for 3 months and started from my over execise (I guess). I am a man, 47 years old. Please help. Thanks. Doctor: Hi,From history it seems that this can be due to pinched nerve pressure.There might be having some degenerative changes in your lower spines giving this problem.Go for x-ray lumbo-sacral region for spondilits.Consult orthopedic surgeon/neuro surgeon and get examined.Go for back extension exercises.Ok and take care."
},
{
"id": 147222,
"tgt": "Suffering from degenerative disc disease, bulging discs in neck, pain from base of skull to shoulder. Back surgery, epidurals done. Treatment?",
"src": "Patient: I have degenerative disc diese in my lower back and a buldging disc in my neck , with constant pain in those areas sometimes pain shots up and down my left side from the base of my skull spreading down my left shoulder to my hip and down to my toes. I have had one back surgery in 2008, as well as, several epidurals. But, nothing seems to help. I take pain medication but that does help either. What should I do? Doctor: HIThank for asking to HCMI really appreciate your concern looking to the history given here I could say that if this is degenerative disc disease then the over all case management is depends upon the severity of the disease most of the cases well responds to conservative line of treatment in my opinion you better discus this with the orthopedic doctor, hope this information helps you, take care and have a nice day."
},
{
"id": 192026,
"tgt": "What is the normal blood sugar level in children?",
"src": "Patient: HiI got blood work done for my kids for blood sugar.my 11 years old's level is 5.8 and my 16 years old has 6.0.The test was done early morning and they hadn't eaten anything after dinner.so it was like fasting whole night.Should I be worried? what are the normal numbers for this age kids? Doctor: Thanks for your question on HCM. No need to worry much for their reports as both are in normal range.In pediatric population, sugar level less than 6 is considered as normal.In other unit , in mg/dl , less than 110 is considered normal."
},
{
"id": 88199,
"tgt": "What causes pain in lower stomach, numbness and tingling on leg?",
"src": "Patient: For about a week I ve been having really sharp pain down my right leg. It s also a numb tingly feeling ontop of the pain. I ve also been having sharp pains in my lower stomach. I dont think its a blood clot because I dont have any discoloration or swelling. Please help.. Doctor: Hi.It looks you are a female patient. A combination of a lower abdominal pain and severe pain down one leg which has developed into numbness is always suggestive of a spinal cord problem.I would advise you an MRI of the lumbar and thoracic area and of the abdomen particularly of the posterior wall to rule out any disease of the spine like tuberculosis or so, Ilio-psoas abscess and so on. Get an examination done by an Orthopedic Surgeon / General Surgeon, get appropriate treatment as per the findings."
},
{
"id": 89226,
"tgt": "What causes stomach ache?",
"src": "Patient: Hi, may I answer your health queries right now ? Please type your query here...my 6 year old daughter has been complaining of stomach aches for that last few days, they have been on and off, but she shortly goes to the washroom for a bowel movement after, is fine then it comes back. It also comes back in the morning when she gets up, or it will wake her up. I don't know what to do. Any suggestions? Doctor: Hi.The commonest problems at the age of 6 years for the complaints you noted are :Appendicitis.Tummy Tonsils meaning inflamed lymph nodes in the abdomen.Worms to mention a few. The best way is get a clinical examination done by a Pediatric Surgeon, X-ray of the standing abdomen and ultrasonography. Also get the stool tests, urine and blood work done. The treatment will depend upon the diagnosis. She can be given a 7 day course of an antibiotic , metronidazole and anti-worm therapy."
},
{
"id": 120481,
"tgt": "Can alcoholism and vasovagal hypo tension cause unconsciousness?",
"src": "Patient: Hi there, my brother who is 36 y/o and heathly has been found losing conscousness and fell down after voiding a couple times. These episodes accompanied consuming some red wines. I quess that these episodes might be related to red wines polyphenol and post voiding vasovagal hypotension. Am I right ? Thanks ! Doctor: Hello,I read carefully your query and understand your concern. Drinking alcohol can cause\u00a0blood vessels to expand,\u00a0causing\u00a0blood pressure to fall and\u00a0syncope\u00a0to occur. This may be triggered by pressure on the carotid arteries, and\u00a0can lead to\u00a0low blood pressure and\u00a0syncope.Hope my answer was helpful.If you have further queries feel free to contact me again.Kind regards! Dr.Dorina Gurabardhi General &Family Physician"
},
{
"id": 147590,
"tgt": "Had seizures. Now stressed. Can I take medicines after a boost? Will I be fine?",
"src": "Patient: i am 45 yr old male, in my life i had seizure complaint in two occasions. the first one was 25yrs back when i felt no rest for consecutive days and second one 2 yrs back when undergone treatment in a hospital for too much official work stress/ tension complaints.now i am taking medicine eption 100mg- 2 tablets/day in night for last 2yrs, recently as per doctor advise i was undergone mediical test for completing 1 yr but the result was not satisfactory infact the level gone up.my doubt is this may becos of alcohol comsumption once in while and medicine will be taken after reaching home from party with in the time gap 1/2 hr - 1 hr 1.now the question is it correct to take medicine after full boost becos doctor advised not to stop medicine even for single day? 2.why the result was not in favour since i am rare case of seizure complaint? 3. now i stopped consuming alhohol for last 6 months and medicine is continued. Will it be helful for next test? 4.give best suggestion. Doctor: Hi,Thank you for posting your query.I have noted your symptoms. You have a mild seizure disorder, as you have had only two episodes of seizures in the past 25 years. I do not understand by \"the test result was not satisfactory\". Please explain what was the test and what was its result? Please upload it here. As far as I am concerned, you are seizure-free for two years while on eptoin (phenytoin), so, it is a good news.I agree that alcohol is not good, so, it is better not to re-start it. Also, keep yourself tension-free and get adequate sleep at night to prevent any recurrence of seizures. I hope my answer helps. Please get back if you have any follow up queries or if you require any additional information.Wishing you good health,Dr Sudhir Kumar MD (Internal Medicine), DM (Neurology)Senior Consultant NeurologistApollo Hospitals, Hyderabad, IndiaClick on this link to ask me a DIRECT QUERY: http://bit.ly/Dr-Sudhir-kumarMy BLOG: http://bestneurodoctor.blogspot.in"
},
{
"id": 97222,
"tgt": "What can I do to lesser my sore rib?",
"src": "Patient: I tripped and fell on an arm of a bamboo chair in my den and as a result I hit my midriff on the right. I now have a sore rib which is tender to touch and a little uncomfortable It is towards my back area and sensitive if I take a really deep breath. Otherwise it is just sore. Will warm showers help? It happened about a half hour ago. Suggestion? Doctor: Hai injury 2 the ribs is very painfil it lasts at least 4 two to three weeksushud take pain killers can go for tab flexon mr two times daily, wid topical application of volini pain jel, pls do deep breathig exercises."
},
{
"id": 153066,
"tgt": "Does cancer cause griping pain in bowels?",
"src": "Patient: i am really worried as i have something wrong with my bowel, i suffer griping pains at the same time of the day every day i cant stand with them,iv tried to vary my diet incase of irritation to foods but nothing has worked, what worries me is that 3 years ago i had a complete gastric bypass with a metre of my intestine removed does this make me more vunerable to cancer? i do have reactions to sugar because that part of intestine has been removed has this caused damage? Doctor: Hello dear. Thanks for question. The cause of your gripping pain as you described is likely due to the dumping syndrome which is seen after removal of stomach and intestines. It has nothing to do with the development of cancer. So you need not be worried for it. You can visit your doctor for suggestion requiring dumping syndrome. Hope that assures you.regards"
},
{
"id": 99990,
"tgt": "Suggest treatment for asthma and stomach aches",
"src": "Patient: My son has been suffering from asthma all week. He's doctor put him on presidone an antibotic and he's using the neublizer as needed which today has been ever 3 hours. He's been in the med for 3 days. Now he's complaing of stomach aches and his pupils are dilated is this a normal side effect? Doctor: Hi, I understand your concern. * The oral antibiotics & prednisone ( if taken on an empty stomach) can lead to increased acidity & pain in upper abdomen.( Avoid giving medicines on an empty stomach/ avoid spicy food/ can give ranitidine along with other medicines -after consulting treating doctor.) - Similarly stress of breathing difficulty/ coughing can result in pain in abdomen.( addition of suitable cough syrup will help) * The dilatation of pupils are not directly associated with the illness- but the physical/psychological stress can cause it. * Treatment of Asthma-in acute stage should be antibiotic, anti allergics with Montelucast , mucolytic cough syrup along bronchodialaters inform of sprays, nebulizers oral drugs. n severe case tappering dose of prednisone can be added. In controlled phase- identification of allergen & desensatization therapy along with bronchodialaters( Spray form) as per need & breathing exercises are to be done."
},
{
"id": 189382,
"tgt": "How to keep tongue away from upper teeth ?",
"src": "Patient: sir, i have a gap in my upper teeth. and my tongure always touches the upper teeth. so please tell me how to keep tongue away from upper teeth. age 22,male Doctor: Hello and Welcome to HCM, Thanks for writing to us. training of proper swallowing pattern and exercises for the lips and tongue. In situations where the habit is severe, a removable appliance may be necessary to block the tongue away from the front teeth. Unfortunately, some are never corrected due to lack of parental support or lack of cooperation from the patient. Hope this helps you. Wishing you good health... Regards."
},
{
"id": 157145,
"tgt": "Why there is a loss of vision after taking chemotherapy for Pancreatic and bile duct cancer?",
"src": "Patient: I had sudden loss of vision in my right eye and sharp pain on the left side of my head with severe vomiting followed can you advise me of why this may be happening. My history is I am currently having chemotherapy treatment after major surgery for pancreatic and bile duct cancer. My platelets are reducing and my neutrophils are practically not existent during recent fbc tests. Doctor: Hi and welcome to HCM. It is possible that such agressive chemotherapy could couse eye complications but other more common causes of eye diseases should be ruled out suh as acute glaucoma, multiple sclerosis, optic nerve neuritis and osme braid disorders. so you should visit ophtalmologist and continue with chemotherapy unless chemo is considered to be the right cause of your problem.Thanks for the query. Wish you good health. Regards"
},
{
"id": 24324,
"tgt": "What is the life of cypher stent post angioplasty?",
"src": "Patient: sir, i am 43 years old men. i had undergone angioplasty with cypher stent in january 2011 for one block. i am having aspirin gold 20mg, finolip 145, losar 50 now. i am doing my regular activities now and doing exercise like walking every day morning one hour. kindly let me know how long the stent validity. ravi Doctor: It's around stent blockage chances 6% per yr... Since some person stent get blocked with in year and some with more than 8-10yrs... Medication sudden stoppage commonly causes stent blockages.... And what about new blockages...if no t taken proper care ...no healthy lifestyle then new blockages can come ...don't worry about stents ...just follow medication and healthy lifestyle...take care"
},
{
"id": 79088,
"tgt": "Suggest treatment for chest congestion and cough",
"src": "Patient: Sir, I am Aditya, age 28 from Delhi.I have been having chest congestion and cough for past 6-7 days. There is no fever. On doctor's advice, have been taking Monocef-O and Montek LC for past 2 days.However, I have a feeling of constant heaviness in my head. It reduces on taking an analgesic and then comes back soon after. What could be the reason for this and what is the remedy?Thanking you,Aditya S. Doctor: Thanks for your question on Health Care Magic. I can understand your concern. By your history and description, possibility of lower respiratory tract infection is more. Chest congestion is mostly due to collection of secretions. And these secretions cause irritation and hence coughing. So you need mucolytic, expectorant and antioxidants drugs like N acetyl cysteine (NAC). It will dissolve the secretions and make them easy to cough out. So your chest congestion and cough can be reduced. It is prescribed medicine, so you need doctor's prescription for this. So consult your doctor and discuss about NAC. Hope I have solved your query. Wish you good health. Thanks"
},
{
"id": 148729,
"tgt": "Lightheaded, spinning feeling and sweating. What could be the reason?",
"src": "Patient: Was at work and feeling fine when I suddenly got a little light headed and then the room started spinning g real bad and so I sat down and then I started sweating to the point it was dripping off my face and left a wet mark on the desk where my arms were resting. After I sat for about 10 minutes it went away. Now I feel light headed Doctor: Hi,Thank you for posting your query.Your symptoms are suggestive of pre-syncope, where the blood supply to the brain is reduced for a short while.The predisposing conditions include exhaustion, tiredness, lack of sleep, hunger, fever, etc.There is no need to worry. You can consult your GP and get a general evaluation done. Low haemoglobin (anemia) should also be excluded.I hope my answer helps. Please get back if you require any additional information.Wishing you good health,Dr Sudhir Kumar MD (Internal Medicine), DM (Neurology)Senior Consultant NeurologistApollo Hospitals, Hyderabad, IndiaClick on this link to ask me a DIRECT QUERY: http://bit.ly/Dr-Sudhir-kumarMy BLOG: http://bestneurodoctor.blogspot.in"
},
{
"id": 56879,
"tgt": "Suggest treatment for elevated SGPT level",
"src": "Patient: I m 24 year old male, I recently gone through health checkup and found that my SGPT level is too high i.e: 139. I dont smoke or drink, however I am suffering with cough and cold since 15 days and taking antibiotic and pain killers for the same. Could you please suggest what can be the reason for the high SGPT level? Height: 5.8, Weight: 64 Doctor: Are you taking medications containing acetaminophen? Anything over 3-4 grams per day can be toxic to the liver. Also, viral hepatitis can cause elevated liver enzymes."
},
{
"id": 54445,
"tgt": "Suggest treatment and diet for liver cirrhosis?",
"src": "Patient: a friend of mine on account of alcohol consumption for along time has fallen victim to liver cirrhosis.it is been reported that it is in the first stage.would you be kind enough to provide the guidance to improve his condition and what food intake he should takethanking you.warm greetings.kiritbhai. Doctor: Hi I can understand your concern....Noted your friend father having liver cirrhosis....See once liver damage and cirrhosis occur it is irreversible condition means you can't revert it back....But with good habits further complication can be prevented or delayed ....Alcohol strickly avoided....High protein diet taken...Low salt for preventing edema....Take one tsp crushed papaiya seed with lemon juice...Beet roots , onion , aloe vera juice , Brussels sprout are some good foods...In early morning grined spinach and carrot juice taken.If portal hypertension present beta blocker might needed....In progressive disease final treatment is liver transplantation...Keeping this in mind consult gastroentetologist.. For physical examination esp...spleen palpation...Avoid alcohol.....Take care.Dr.Parth"
},
{
"id": 206240,
"tgt": "How to treat PTB?",
"src": "Patient: I Sarat Kumar Varma, age 51 years, detected with PTB on 30.6.2010 , have done full course of treatment. Also have been detected with COPD and am on Seroflo 250 inhaler and Duolin inhaler,have been prescribed Potklur syrup.What could be the best treatment for this? Doctor: Welcome to healthcaremagic,For treating any new case of Pulmonary Tuberculosis treatment is given for at least 6 months. For treating a old case of PTB or treating a relapse case of PTB treatment is given for at least 9 months.In your case, if you have taken the full course and if the physician has checked your sputum sample and if it iwas negative, then your treatment is successful and you do not need to do anything else. But after competing your full course if your sputum is positive then you have to take treatment again for 9 months as it takes time to cure this dangerous disease. Your physician may also check if you have a resistant strain of bacteria which is called as drug resistant PTB for which they will give a different treatment called as DOTS plus. Remember DOTS treatment for tuberculosis is provided by the government of India free of cost in government hospitals and you can take treatment from these hospitals.Hope that answers your question.Regards, Dr.Imaad.(if the answer has helped you, kindly indicate this)"
},
{
"id": 157561,
"tgt": "Had cardiolite tests. Had occlusion of right cardial artery. Can radiation be killing?",
"src": "Patient: I have had 5 cardiolite tests over the last 15 years. In fact, I had a total occlusion of the right cardial artery 7 years ago. The only intervention was lipitor . Should I stop these quacks from irradiating me? The WSJ stated 2 weeks ago that one nuclear heart scan produces more radiation exposure than 4 CAT scans. I fear that with great insurance these quacks cannot resist performing expensive tests but I fear the radiation is killing me. I found a web page that states that this expose alone, without including dental x=rays, etc. has increased my chance on getting cancer by 3%. Doctor: Thanks for the query.Itcan be killing. these are certain doses of radioactivity but it is well investigated and there is no some significant risk of cancer development. Of course,it doesnt mean that you can have these tests as musch as you can,but it si much more beneficial to check your heart function and prevent possible complcaiitons than think about radiation.Wish you good health. Feel free to ask. Regards"
},
{
"id": 34698,
"tgt": "What causes fever with shivering and post fever aches?",
"src": "Patient: I was fine all day yesterday but when I went to bed last night, I was shivering even though it was very warm in the room (according to other people in the room). Throughout the night I developed a very high fever and my whole body was really really warm. I got up in the middle of the night to go to the bathroom and get water, and could barely walk because I felt so weak. I couldn't pick up the water pitcher with one hand because my muscles were so weak, and I could barely grip on to anything. I slept for about 12 hours, and now my fever is mostly gone but my entire body still aches. What do you think this is? Doctor: Hi, I am sorry for the health situation you are in. Your symptoms are indicative for possible common viral infection. I advise to:- take more rest- drink more fluids (tea, water, juice, etc.)- have light meals (chicken breast soup or yoghurt)- take OTC pain killer or flu medication to relieve the aches - take vit.C supplement to increase body energy to fight infection.If no improvement within a week or the symptoms will get worse, should consult with your physician for further evaluations. Wish fast recovery!Dr.Albana"
},
{
"id": 199477,
"tgt": "Is all time sperm oozing a problem?",
"src": "Patient: Hi Doctor, I m really concerned because I have sperm come out of me every single day. The thing is, that I don t think about any sexy things and I don t masterbate. I could just be sitting down or doing chores or homework and all of a sudden sperm comes out of no where. I m 17 , a female, still a virgin. Can you please help Doctor: Hi,As you are female so there is no question of sperms coming out.I think you mean to say that there might be having some vaginal discharge while sitting or doing home work.nothing to worry if this discharge is watery and odorless.This is normal vaginal secretion coming out.Only thing to remember is keep your local hygiene clean and healthy.Ok and take care."
},
{
"id": 115767,
"tgt": "Suggest therapy for blood disorder",
"src": "Patient: Hi, may I answer your health queries right now ? Please type your query here...\u00a0\u00a0\u00a0\u00a0\u00a0\u00a0\u00a0\u00a0\u00a0\u00a0\u00a0\u00a0\u00a0\u00a0\u00a0\u00a0\u00a0\u00a0\u00a0\u00a0\u00a0\u00a0\u00a0\u00a0\u00a0doc my sister is suffering from a blood disorder in which her white blood cells are above the normal and I am worried. she says that she had the treatment but the results are the same. is it dangerous Doctor: Hi, dearI have gone through your question. I can understand your concern.Your sister has high WBC count. There are many reasons for high wbc count. Common cause is infection. However leukemia can also leads to high wbc count. You should go for Complete blood count and peripheral smear examination.It will give you clue to the diagnosis. Then take treatment accordingly.Hope I have answered your question, if you have any doubts then contact me at bit.ly/Drsanghvihardik, I will be happy to answer you.Thanks for using health care magic.Wish you a very good health."
},
{
"id": 178862,
"tgt": "Suggest treatment for continuous vomiting and fever",
"src": "Patient: My 8 year old started vomiting and running a fever of 102 on Thursday around noon. Friday evening he still wasn t able to hold fluids and had no appetite and had been vomiting constantly so we brought him to urgent care who sent us to the children s e.r. where they determined he didn t have a surgical abdomen then gave him blood tests which came back clean other than mild dehydration, so they gave him zophron for nausea and fluids via iv. And ibuprofen for fever which we had been giving him alternating w/ acetometophen. The doctor said he has gastroenteritis which seemed like an educated guess on his part he also said that my son would have diarrhea blabla and we might get sick which neither ever happened. also his initial exam was also clean. After we came home and since then he is still sick the last of the vomiting was last night but the fever and fatigue and the pale face and dark circles and sleepiness is still there also he had been complaining of headaches and dizziness but not recently. Does this actually sound like gastroenteritis? And is it still okay to continue giving him ibuprofen and acetometophen now that is day 5 and he is still getting fevers? Doctor: If all the report came clean, then it could be viral infection and associated viral gastritis which is getting worse with the continuous use of paracetamol and ibruprofen. Please dont use ibruprofen, use paracetamol at a correct dose of 15 mgs per kg body weight when temp crosses 100. also give him at least one biscuit before giving paracetamol... start Tab Lanzol 1 tab daily in empty stomach and continue for 15 days . The fever will resolve in 2 days. If you live in dengue endemic area get a Dengue test done."
},
{
"id": 62021,
"tgt": "Suggest treatment for small painful lumps on the forearm",
"src": "Patient: sometimes especially when i do an intense workout or lifting the vien in my forearm gets about 6-7 lumps in it theyre soft lumps and each about half an inch apart if i stop and rest it goes away or if i put my hands over my head it stops is this bad and what is it Doctor: Hi,Dear,Thanks for your query to HCM.Studied your query in full depth of its details.Reviewed it in context of your health concerns.Based On the facts,You seem to suffer from -varicose veins of the forearm under stress only.You may need to treat them by Endovenous laser ablation(EVLT-Endo-Venous-Laser-Treats) of the specific incompetent section.These are for cosmetic purpose only.Lesser for any thrombotic DVT complications-as in leg veins.Consult your surgeon and vascular surgeon after the Doppler dual image study.This reply would help you to plan further treatment soon with your treating doctors.Best of Luck and early recovery.Welcome any further query in this regard,which would be replied in next session.Good Day!!Dr.Savaskar M.N.Senior Surgical SpecialistM.S.Genl-CVTS"
},
{
"id": 10536,
"tgt": "Suggest medication for hair loss and dandruff issue",
"src": "Patient: Hello Sir, I m 32 years old and facing hair loss problem for last 5-6 years. My centre of the of the scalp have badly damaged and having very hair. Now the frontal scalp is also becoming thinning the hair. I dont know there is some dandruff kind infection on the whole scalp. is that causing my hair loss or any other reason behind it? if yes what are the medications should i start? Please help? Doctor: Hello and Welcome to \u2018Ask A Doctor\u2019 service. I have reviewed your query and here is my advice. As per your query you have symptoms of hair loss and dandruff issue which seems to be due to fungal infection and hormonal changes. Sometimes hair fall can also occur due to side effects of certain medicine. Need not to worry. I would suggest you to consult dermatologist for proper examination. Doctor may order skin patch test , blood test and physical examination. Doctor may also prescribe supplements like Vitamin A, Omega-3 and Vitamin E. Doctor may prescribe shampoo like candid TB, Finpecia and Minoxidil spray. You should take Indian goose berry powder (half spoon) with water and use olive oil for massage. Hope I have answered your query. Let me know if I can assist you further."
},
{
"id": 111962,
"tgt": "Are leg problems and a torn disc in the back related to being sporty?",
"src": "Patient: Hi . Our son has very tight archillies and is looking at an operation on both legs. In the meantime we have found out he has a torn disc in his lower back. He is 14 years old yet very sporty. Would this be the cause of his leg problems or would it be a separate issue altogether? Doctor: Disc disease is more common in drivers, weight lifters and horse riders. Repeated jerk promote disc ruptures. if he is already having disc problem he should follow back strengthening physio program. i wold also recommend that he should avoid games in which chances of sudden bending are more,such as hockey."
},
{
"id": 186131,
"tgt": "Can impacted tooth cause dizziness and headaches?",
"src": "Patient: I have been suffering from dizziness and headaches for 2 months. Did every single check, everything is ok. Only thing I have is one impacted tooth on the top right side, it's really big and it's lying on the 7th.Can this be a cause of my problems? I'm going to oral surgery on Monday. Doctor: Thanks for using health care magic.Read your query.Tooth pain can cause referred pain to the head and in turn because of that you may feel fatigue and stressed.Get the tooth removed and continue with the medication given and it will be fine.Hope this was helpful.Thanks and regards."
},
{
"id": 217622,
"tgt": "Suggest medication for joints pain and soreness from vomiting due to food poisoning",
"src": "Patient: I got food poisoning last night and was hospitalized - I feel much better than I did last night - but still terrible. The nausea has passed but I am unable to eat. Is there anything I can do for relief from the aching joints and soreness from vomiting? Doctor: Hi,You will gradually get better over the next few days. In the meantime, take paracetamol and ibuprofen for the joint pains and muscle soreness from vomiting. Drink plenty of water to avoid dehydration. Take a light diet, soups or toast are suitable. Take ibuprofen after eating, not on an empty stomach.I hope you feel better soon.Regards,Dr K A Pottinger"
},
{
"id": 157686,
"tgt": "Had Crohns disease. Taking purinethol. Had numerous bouts with squamous skin cancer",
"src": "Patient: Hi,I have had Crohns disease for fifty years. Over the last fifteen I have been on Purinethol 75 mg. a day.In the last five years I have had numerous bouts with squamous skin cancer. I have been told that the purinethol has exasperated the progress of my skin cancer. It has really helped my crohns but now I have a new problem and feel like I am between a rock and hard place.I am 68 and very health aware but they don't seem to stop cropping up on my arms and legs and hands.Any advice.Thank you YYYY@YYYY Doctor: it be best to continue with the medicine if that is helping you relieve Crohns symptoms, but you need to be under regular supervision and screening for skin cancer because you had it many times"
},
{
"id": 70098,
"tgt": "What could growing, painful, extremely red lump on back with inability to get fluid out indicate?",
"src": "Patient: my husband had a lump on his back for around 10 years..it has recently started growing fast and became very painful and extremely red.he went to the e.r. a few days ago ,they tried to get fluid out, but there was none..he is going to a surgeon tomorrow, but he is nervous because they could not get fluid out and someone he works with told him that was not a good sign..should we be worried.? Doctor: HI. Nothing to worry at all. This looks to be a sebaceous cyst (10 years history - on back) , now got infected so grew very fast, with pain and redness.Just make sure he is not a diabetic. The Surgeon has the option to clean it under anesthesia or to give antibiotic and anti-inflammatory medicines now and go for definitive surgery later. Both things are OK and will be decided by the Surgeon on merits of the case. Just follow his advise"
},
{
"id": 30934,
"tgt": "How to treat chronic sinusitis?",
"src": "Patient: I had unprotected sex with a woman about 3 years ago. About 16 weeks after that I started to have pain in my underarms and in my groin area. Also had night sweats and was very fatigued for about 2-3 weeks. Some days I could not even make it through work it was so bad. I was never hungry and barley ate anything. Then all of a sudden it went away. A few months later I started to get brownish red spots in my mouth that would come and go, and sores in the inside of my noise. Also I started to have chronic sinusitis (I still have that now). Only thing that clears it up for a while is nasal spray, very hot or cold air, when I take long walks, or if I have some kind of mint gum or candy but it only lasts for a while. A year later I started to get canker sores in my mouth and the brownish red ones every once and a while too. I had never had a canker sore in my life. I also started to get very dry skin on my forehead. About 2 months ago I started to have joint pain in my right elbow and it will not go away. I have also had a lot of sinus drainage going down my throat causing sore throats in the last 2.5 years its not funny. Also in the past week maybe 2 I have noticed a lump under my right ear (could be a lymph node).I went and bought a home access HIV test and took it at 28 or 29 weeks after the last unprotected sex and it came back negative. I would have no problem with the results if it was not for the fact that when I called and put my test number in it said thank you for registering this number and that it was now safe to take the test. I had to call back and get an operator and told her what happened and she said they were having problems with the phones and to just give her the number. She is the one that gave me my negative result. Should I have be concerned about the automated system messing up and possibly messing up my result? What else could this be? Doctor: It is better for you to get the VDRL,HIV,HBSAg and the STD profile to get done in authorised lab than these confusing home kits."
},
{
"id": 76920,
"tgt": "Suggest remedy for breathing issues",
"src": "Patient: I quit smoking two months ago. Had gone to Doc yesterday. He advised me Acelast 100 mg caps for 15 days. My basic problem was heaviness in breathing (most probably because of monsoon season in India-the air is quite moist). I also mentioned to Doc that I wanted to increase efficiency of my lungs and also check my cholesterol. Whay is Acelast useful for? Doctor: Hi thanks for contacting HCM...The tablet acelast is bronchodilator drug.So by cold or moist wave if you have inflammatory bronchial swelling then this drug helpful....Here first you have to check for any restrictive or obstructive lung condition by pulmonary function test.It will distinguish between two..Chances of COPD or asthma or fibrotic lung disease will be ruled out.Chest x ray if needed done....Regular exercise and yoga useful for maitaining lung health....Cardiorespi exercise like running , swimming also useful....Take care...You can discuss this with your doc...Dr.Parth"
},
{
"id": 105173,
"tgt": "Suffering with cough and allergy, throat irritation, throat gets blocked immediately after meals, impaired voice. Advice?",
"src": "Patient: Hello sir I am suffering from cough like allergy with some irritation in throat . Every time a small volume of cough block my sound and i have to clear it. After my meal my throat always blocked with cough otherwise it is O.K. It is affecting my voice in early in morning and when i want to speak to anybody. My skin, Nose , hair and smelling sense are very sensitive. Please advice me. Doctor: Hello The combination of throat irritation, cough AND voice getting affected after food (as also early morning) raises the possibility of reflux - stomach contents back-flowing to food pipe and beyond... You have to see your physician for proper evaluation - if it is long (you didn't say how long!), you may need an X-ray, blood count and/or upper GI endoscopy (tube to see your food pipe and stomach) Good luck"
},
{
"id": 107592,
"tgt": "What causes severe lower back pain after being treated for osteoporosis?",
"src": "Patient: I am having a lower back pain. When I get up from my sleep in the morning my back is very stiff. I take 2 tablets panadiene but does not help. I also use a hot pack on my back but it only gives momentary relief. I am also under treatment for osteoporosis. Please advise. Doctor: Thank you for choosing HCMBased on your description, it is likely that osteoporosis can be causing the pain that you are having. I do hope that your Doctor has started you on Vitamin D and Calcium to help prevent the problems and symptoms as far as possible.There are a few more things that you can do to see if they help:Ice the back for 15 mins at a time Then warm the back with a hot pack for 5 minsRepeat this after 1 hourAlso, you can do stretches for the lower back and also massage the area.Follow up after 5 days. All the best. Another therapy is giving local injections to stop the pain but this is a last resort.Hope this helps you"
},
{
"id": 180017,
"tgt": "What is the right age a child should show signs of walking?",
"src": "Patient: My baby is 14Months old and he is not walking yet. I have been teaching and training him since he was 8 months but still he isnt walking, he crawls and walks arounding holding onto furniture he can even stand on his own but he just isnt showing signs of walking. What can I do? Doctor: Hi dear welcome to the HCM, Walking is a important mile stone for the growing babies.Quite variables.depends upon many factors like nutrition,genetic factor,constitutional. enviormental, seasonal, socioeconomical, any chronic disease prenatal or postnatal. For most is Rickets in the current living scenario.Some times walking may start at the age of 15 months.improve his nutrition, add vitamin D, multivitamin etc.Hope he will start walking.thamks"
},
{
"id": 101396,
"tgt": "Suggest treatment for swelling in cheek and lower lip",
"src": "Patient: spontaneous swelling of left cheek and left lower lip, from just below the nose to the chin. This is the 4th incident in 4-5 months. I know of no allergies. My eating habit are very consistent. I took Benedryl. The prior incidents subsided within a day. Doctor: Hello.Thank you for asking at HCM.If you have had no other health issues apart from spontaneous swelling of left cheek & left lower lip which subsides with Benadryl, I would first think of angioedema, which may be allergic or non-allergic.I would advice you to keep a watch on your diet (& drugs, if any) and try to correlate occurrence of swelling within 2-4 hours of any food (or drug). Common food substances can be fresh fruits & vegetables, cheese, fish, packed & preserved foods, etc.If you cannot identify the reason, I would suggest you to consult an Allergist-Immunologist who may suggest you allergy testing and identify the cause.Hope this will be helpful to you.Wish you best of the health.Regards."
},
{
"id": 12410,
"tgt": "Suggest medication for psoriasis",
"src": "Patient: what is medicine for external applications to suppress/ cure psoriasis?age 39height 6 ftsuffering from psoriasis since age of 14. it is on the scalp, elbow, knee and few spots on the back, thigh. currently I am using diprovate cream to suppress scalp. Doctor: Thanks for using healthcaremagic.I understand your query.Topical calcipotriol ointment twice daily or topical tazoratene(Tazorac)0.05% gel once daily can be the other options.If i am your family physician,would suggest combined therapy,based on the type and extent of lesions (like photo therapy+topical agents) which will be more effective in enhancing faster remission/clearance of lesions.I would advise you to discuss this with a dermatologist for above prescription if its appropriate for you.Hope i have answered your query.If any further questions,happy to answer you.Take care."
},
{
"id": 222037,
"tgt": "Does German measles affect pregnancy?",
"src": "Patient: I m over 4 months pregnant, how dangerous is it for my unborn, if I ve been possible very exposed to German measles/or possibly fifth disease? Is this something I need urgent care for/see my OBGYN? Should I be concerned? If I end up having symptoms- how should I respond? Doctor: Welcome to HealthCareMagic. Your query has been assigned to me and I am happy to assist you today for the health issues you raised.German measles or Rubella is very bad for you and the baby. You should consult your OBGYN right away for proper preventative care. You should definitely be concerned and control the situation before you get the disease. Even after revelation of symptoms, you need to consult your OBGYN for the best of you and your baby.Hope it was all you needed the answer for. If you have some more questions or doubts in mind, please let us know to help you. We are all a team of best experts from all over the world to assist patients online. Give your feedback to help us make this service even better.Healthiest Regards!Dr. Sumaira Kousar MD."
},
{
"id": 46300,
"tgt": "Can a defective kidney be treated by dialysis only?",
"src": "Patient: Dear Doctor, This is babu rao from Chennai, My father has high creatinine levels of 9.8, Our doctor is advising us for dialysis,My father is not prepared for it and finally we have do him admitted in hospital for full check up, One of my father's friend recommended Homeopathic treatment but our doctor strictly say NO to it and he says homeopathic treatment will do no good and in turn harm kidney, He said once the kidney is damaged then it can not revive. Please Advise. Doctor: Hello Welcome to HCM. I'm sorry to hear that your father is unwell. 1st thing sir, his age?? Going by most adults getting kidney related diseases that shoot up Creat he may be 65 plus? I suggest you few things. 1. Do not panic.. 2. Get him evaluated completely with Renal function tests3. Get a renal scan done. We can know the kidneys better. 4. Generally if Creat is more than 9 as you say, both kidneys are damaged. 5. If he's too old, see his prostate too. Sometimes simple obstruction causes high Creat. 6. Never ever go for homeopathy. Absolute NO. 7. As you said, normal ppl will spoil their kidneys by taking some homeopathic medications. Imagine same with creat of more than 9!!To summarize, get him evaluated completely, find out the cause for such high creat and no homeopathic. And in case he needs dialysis, better to get a fistula done. Hope this helps you and pray for the recovery of your dad."
},
{
"id": 72048,
"tgt": "What causes recurring fever post TB treatment",
"src": "Patient: Hi Doctor,I was having fever for the 3 weeks, after diagnosis with the help of test ADA and Montex , Doc identitfed it as PUO and started treatment of TB. now after taking AKT4 , fever disappeared but after a 10 days its back again.is fever come during TB treatment , please suggest me... Doctor: HelloYes it might be but do not worry its too early since you have start the treatment.It take sometime to be effectiveRegardsDr.Jolanda"
},
{
"id": 132935,
"tgt": "How to treat partially broken ligaments?",
"src": "Patient: hi i had a ankle injury while playing tennis..i got the treatment from the hospital and they said that i had 3 ligaments partially broken. i was not having any pain in my ankle or leg. now since last 2 weeks i m using crutches for walking..but since yesterday i felt like i can walk slowly by my own..is it ok that i can walk for 5-10 mins? Doctor: Hello,\u00a0Thanks for writing to us,\u00a0I have studied your case with diligence.As per your history and injury pattern there can be persistent tear in ankle ligament leading crunching sound or capsular tightness.I will advise to do MRI ankle for\u00a0soft tissue\u00a0involvement.Till time you can continue physiotherapy exercises and also use supportive\u00a0splint\u00a0or brace.Physiotherapy like ultrasound and TENS will help in early healing.Persistent pain after ankle injury is common and may take 6 months to heal.You can walk with splint support.If there is complete ligament tear then arthroscopic ankle ligament repair can be done or if capsular tightness is severe then release can be done.Hope this answers your query. If you have additional questions or follow up queries then please do not hesitate in writing to us. I will be happy to answer your queries.\u00a0Wishing you good health.Take care."
},
{
"id": 133941,
"tgt": "What causes tremor on foot?",
"src": "Patient: i am experiencing a tremor on the left foot between my ankle bone and bottom of my foot. the tremor has been occurring every 1 to 1 and half minutes. It is like being shocked without the pain. What would cause this. this has been happening for the last 10 hours. Doctor: hi,thank you for providing the brief history of you.A thorough neuromuscular assessment is advised.As mentioned by you, that you face tremor in the feet, I will advice firstly toget a Neuromuscular assessment, by which we come to know what factors to consider and what to rule out. The first line will be MRI of lumbar spine to see any soft tissue pathology, later NCV of the lower limbs, and lastly the MRI of the brain is the Neurologist feels so.usually Tremor of the feet muscles can be due to the pathology of the spine. And very rarely with the brain but still an examination will guide us better.Also, this tremor can be due to any nerve weakness or vitamin deficiency, which can be corrected by medication as well.I will advice further to undergo physical therapy for tremor as it will help to regain the strength and correction of muscular imbalances.regardsJay Indravadan Patel"
},
{
"id": 189640,
"tgt": "Bad breath problem. Can it be because of stomach acid?",
"src": "Patient: Hi, Recently I am suffering from having a bad breath. My dentist told me my teeth are fine and that my bad breath is probably caused by my stomach acid. A chinese doctor/acupuncturist also told me that I have a bad breath, due to my stomach is not working properly. I have noticed I have a bad breath, dry mouth and a white tongue after I have not talked for a while or after enjoyed my meal. Now I am really worried and start to link everything with my stomach, because last year I have had a stomach reflux . Additional information is that I do not smoke any tobaco, have a skinny body and I am only 22. Doctor: Hi, Thanks for asking the query, Bad breath can be because of several reasons, Poor oral hygiene , deposition of plague and calculus, periodontal infections , tongue coating , xerostomia, faulty restorations retaining food and bacteria , oral pathologic conditions , mouth breathing habit , use of certain medications fasting , starvation , foods like garlic , onion Diseases of GIT, GERD ,Nasal infections like rhinitis , Pulmonary infections like bronchitis Systemic conditions like diabetes , dehydration, fever. I would suggest you to get a complete checkup done. Visit to a Dentist , get complete mouth scaling and polishing done . use chlorhexidine mouthwash gargles twice daily . Drink lots of water. Maintain a good oral hygiene . Hope this helps out. Regards...."
},
{
"id": 19669,
"tgt": "What causes extra heart beats?",
"src": "Patient: FOR THE LAST 5 MONTHS I HAVE BEEN FEELING SOMETHING GOING ON WITH MY HEART. MY DOCTOR SAID THAT I HAVE EXTRA HEART BEATS, AND THAT THERE IS NO REASON FOR CONCERN. WHAT CAUSES THIS CHANGE IN A PERSON? ALSO, I HAVE ALWAYS BEEN A VERY NERVOUS PERSON AND LATELY IT HAS BECOME MUCH WORSE. IS THIS A PANIC DISORDER? Doctor: Hello there I have read through your question and what you have is tachycardia that is increased rate of heartbeat. As you said you are a nervous person it may be the cause for your increased heart rate. I would advise you to consult a psychiatrist for your problem and further assessment. I hope that helps."
},
{
"id": 66416,
"tgt": "What causes a lump on the head?",
"src": "Patient: I have had a lump on my head for a couple of years, it formed on my head after a stone wear loaf pan fell out of a cabinet and hit my head, recently I noticed it was a little bigger, it is soft but almost feels like it s filled with fluid or it feels similar to a lipoma, today I as poetically poked it with a needle and it started leaking clear non odorous fluid, it shrunk a bit, as I palpated the area I can feel a hold in my skull which the bump has covered up, it s a hole the size of a fingertip. Is it possible it was CSF leaking and I have a serious problem? Doctor: Hi, thanks for sharing your health concerns with HCM! If I were your treating Doctor for this case of a head lump, I would come up with only one and one possibility that is a sebaceous cyst! Nothing to worry about this and try not to get infected there!Well, this is a fairly common benign disease of the skin/skin appendages and CSF has nothing to do with it; however, in worse case it could be some parasitic cyst also!I suggest you to go for an FNAC test of the lump for confirmation and to relieve your concerns!Hope this answers your question. If you have additional questions or follow up questions then please do not hesitate in writing to us. I will be happy to answer your questions. Wishing you good health."
},
{
"id": 101509,
"tgt": "Suggest treatment for prolonged allergic rynitis and productive cough",
"src": "Patient: Hello docter,my name is rosy alex, 31 years old women,married,my height is 162cm and my weight is 57kg,I have allergic rynities and productive cough, since 10 years,I have consulted with one chest specialist and she priscribed budecort inhaler, for past 6 months i have not had bad cough. I am taking budecort as prescried by my doctor for the past 3 - 4 months. Of late i have noticed dry blood stains along with mucous. I do not have any sysmtoms of breathlessness,,my blood pressure is 120/80mmof hg,no ,irritation of nose,no,,cough,etc. I taking it as i have allergic rihnitis.my chest x_ray is normal ,and my sputum AFB is negative , Doctor: So the condition of your allergic rhinitis first requires Nasal Spray Duonase spray per nostril per day at night bedtime. ensure you lie down on the bed before sparying the Duonase, as most of them do it wrong.your budecort i feel of you have no breathlessness now. then have a spirometry done and if there is no significant change of reversibility, in the report then your specialist will ask you to discontinue the budeusnide inhaler.and regarding your blood in the mucous you should be afraid or worried as these things a happen now and then. if you have coughed very forcibly.any way once again you notice blood in mucous upon coughing, pl contact your specialist and seek a chest x ray and get an opinion from specialist."
},
{
"id": 163982,
"tgt": "What causes child to have pain in heart?",
"src": "Patient: Hi, may I answer your health queries right now ? Please type your query here... My 4 year old son has been complaining about once a week that his heart hurts he is very active and very healthy, I am not sure if I should have this check out. Please advise. Thanks Doctor: Hi.... there are 2 issues in here - 1. A 4 year old cannot say that complaint just like that to seek attention.2. In an active and otherwise well normal 4 year old kid, it is not a very serious complaint.My suggestions for you - 1. Unless the kid is having breathlessness or fast breathing or abdominal distension or swelling of feet and easy fatigability you need not worry.2. No need to panic but get your kid examined by a physician.3. An innocuous simple issue like mitral valve prolapse can cause this sort of symptoms.Regards - Dr. Sumanth"
},
{
"id": 116272,
"tgt": "What causes fluctuating BP and vomiting blood?",
"src": "Patient: my uncle hospitalised due to vomiting of blood.after one day doctors said he is ok now.but after that bleeding occured and its not stopped and its happening intermittently,,and blood pressure is fluctuating(lowBp-96).. may i knw what is his current state?is there anything to worry?what can we do to help him out of this? Doctor: Hello,it depends on the cause of his bleeding. Rebleeding after initial improvement usually involves ulcers or esophageal varices. There are effective treatments for both conditions. A gastroenterologist treats them endoscopically. Hemorrhagic gastritis (another potential cause of upper gastrointestinal bleeding) usually stops even without treatment within a few hours and heals within 24 hours. If you do know which of the above conditions is the cause, I'll be able to give you more specific information.I hope I've helped!Kind Regards!"
},
{
"id": 7417,
"tgt": "Pimples on forehead, pus discharge on squeezing, scars. Suggest creams to reduce scars and pimples?",
"src": "Patient: i am having a lot of problems with pimples , especially on my forehead . i can t help but squeeze them because it hurts a lot if there is pus . i have so many scars on my forehead now. i am currently trying fadeout cream, please give some info about decreasing pimples and getting rid of scars. pimples are not deep, and are small. Doctor: Hi, Thanks for posting on HCM You are most probably suffering from septic or inflammatory acne (i.e pimples superinfected by bacterial infection). In order to deal with this you will need lotions such as salicylic acid or benzoyl peroxide 5% associated with oral antibiotics such as doxycycline or clindamycin (could be applied topically too). You will need to apply them 1-3 times daily for a month. You can also pay a visit to a dermatologist for proper review and medications. Hope this helps"
},
{
"id": 115860,
"tgt": "How long does Orofer need to be continued for HB count?",
"src": "Patient: Hi Am Aparna .unmarried.Am suffering from Factor-2 deficiency.My heamotologist suggested me to take orofer-xt for 6 months as my HB % is down every now and then.Also i had given with steroids for one months and stopped gradully.Doctor said steriods can n't be taken for long time.Now am on orofer -xt for 4months and Pause tablets during m.cycles 3grms per day (trail for two cycles)as am complaining about heavy blood loss during cycles.Am worried that how long i need to take orofer to manage with HB count Doctor: Hi, dearI have gone through your question. I can understand your concern. You should continue orofor tab till your hemoglobin count becomes normal. Then you can stop it according to your doctor's advice. Steroids should not be taken for long time. So you are going on right way. Continue your treatment with pause according to need.Hope I have answered your question, if you have doubt then I will be happy to answer. Thanks for using health care magic. Wish you a very good health."
},
{
"id": 57586,
"tgt": "How to treat depression co-joined with abnormal liver numbers?",
"src": "Patient: Hi. I am 19 yrs. old. I have had lots of constipation recently and really pale stools. I took antibiotics for 6 months. At the end of the summer I lost my appetite and didnt feel right. I went to the doctor and had abnormal liver numbers. I had just stopped the antibiotics. I have be diagnosed with depression since. But, Im still experiencing the digestive issues. Should i go get my numbers checked again. Doctor: hi you are writing that you have used antibiotics, your symptoms seems to be related with bowel candida infection, so you should do fecal exams and after that take proper treatment.with regards to your liver panel , it may be side effect of antidepressant therapy.Talk with your doctor to change the medication. wishing you fast recovery Dr.Klerida"
},
{
"id": 205508,
"tgt": "What is the best medicine for anxiety disorder and depression?",
"src": "Patient: Hi, I'm a 29yrs old and have been on zoloft since I was 18 yrs old. I use to have such a high libido before the meds and enjoyed and thought about having sex everyday. I used to have so much fun. I've recently married my husband March this but we have been together for 11 years. I love him so much but sex doesn't interest me at all. I'm never in the mood, but I want to enjoy one of the best pleasures life again. Another factor is that I'm a stripper in a no touching club. But we met each other at the same club he was my manager and we worked together for 3 years and have lived with each other for years. I was diagnosed with bipolar at age 18/19 as I was admitted to a mental hospital by force as I was very suicidal. I think that being on zoloft at the start helped. Well its kept me somewhat sane. However I'm still experiencing anxiety and depression. I used to love sex and being impulsive and now I just dread sex and I feel numb but I want to, I don't what to do. I have sex with my husband like once every two months. I feel like I don't even know how have sex anymore. Is there a better antidepresent as I feel like I've missed out on best years of my life. I'm a stripper that can't even initiate sex with my own partner. Why? Should I just get off zoloft. I want my happiness back. What can I do to get myself in the zone as I feel like we have never had proper sex. Is there any drugs I can take to bring my libido back. Help please! Doctor: DearWe understand your concernsI went through your details. I suggest you to consult a psychiatrist before stopping or switching psychiatric medicines. Your bipolar may not have cured fully. Your medicine is helping you a lot in that aspect. Being so long in medication must have taken its toll. You need to address your anxiety and depression too. I also suggest you to worry about sex later. For the time being, consult a psychiatrist and start Cognitive Behavior Therapy and pharmacotherapy.If you require more of my help in this aspect, please use this URL. http://goo.gl/aYW2pR. Make sure that you include every minute details possible. Hope this answers your query. Available for further clarifications.Good luck. Take care."
},
{
"id": 85048,
"tgt": "What are the side effects of the medicines uvox,dipproex and ativan?",
"src": "Patient: I m taking uvox, dipproex250and ativan 1mg since 14 months Uvox is continue right from begining and other tablets changes time to timeis and i m well now but i want to know is there any measure side effects by taking these medicine for long time Doctor: Hello, Let me tell you that like other drugs Ativan, uvox and dipproex have also some side effects. But I'd also like to add that most of the side effects of these drugs are harmless such as nausea, dizziness, weight gain, headache, blurred vision to name a few. Hope I have answered your query. Let me know if I can assist you further. Take care Regards, Dr. Prabhash Verma, General & Family Physician"
},
{
"id": 202487,
"tgt": "Stinging, increased sensitivity near anus after swimming with trunks. What could be the cause?",
"src": "Patient: Towards the back of my testicles, close to my anus , it stings and is sensitive. I don t know what I could ve possible done. Today I played basketball and felt fine, but after I went in the pool. About 20 minutes before I got out, my testicles started to ache. What do you think it could possibly be? Was it maybe that the net on my swim trunks were constantly rubbing on the balls? Doctor: Could be allergic reaction to chlorine and sweat in your anus. Use steroid cream and then see if the symptoms are better. Please rate 5 stars."
},
{
"id": 196873,
"tgt": "What do freckle type spots on penis indicate?",
"src": "Patient: Hi. My boy friend has these spots on his penis. They look like freckles. They aren't raised or painful or sensitive to theTouch. But they come and go. Right now he has three, two he has had and one noticed today. Do we have any reason to worry? What could this be? Doctor: HelloThanks for query .Spots on the shaft of the penis that your boyfriend has are mostly whar are called as Fordyce Spots .These are visible sebaceous glands seen on genitals in many individuals and does not signify any disease .Normally they fade away without any treatment and does not need any active intervention.Maintaining a proper personal hygiene by cleaning genital area with warm water helps to prevent from getting these spots .Dr.Patil."
},
{
"id": 172273,
"tgt": "Can i use 'ciplox' eye drops for my child for eye discharge?",
"src": "Patient: my 2 month old son has white discharge from his eyes since birth. Doc said that its because tear ducts are not yet open and asked me to squeeze my milk into the eyes. But the discharge has not stopped and the baby is finding it irritable. The skin has become redish at the corners of the eyes. Another doc says that baby has infection and has given ciplox drops and now novamox drops. Is it safe to use and upto how many days these medicines can be used? Doctor: Hi...Thank you for consulting in Health Care magic.Eye problems and conditions are best diagnosed only after seeing directly. I suggest you to upload photographs of the same on this website, so that I can guide you scientifically.Hope my answer was helpful for you. I am happy to help any time. Further clarifications and consultations on Health care magic are welcome. If you do not have any clarifications, you can close the discussion and rate the answer. Wish your kid good health.Dr. Sumanth MBBS., DCH., DNB (Paed).,"
},
{
"id": 103947,
"tgt": "Skin allergy since childhood, itching transforming into wounds, spots all over body. Treatment?",
"src": "Patient: hello im a girl age 19 and have this problem of skin allergy since childhood mostly due to mosquitoes and environment and sometimes even when i eat more sugary foods which causes inflammation and itching and i have developed a very bad habit of itching over and over so they transform into wounds and while they are healing i peel the brown thing off and so i have millions of spots all over my hands,legs,back mostly but also on thigh and hip but less and my feet are very ugly due to the spots...besides ive noticed even if i dont scratch, lighter marks are always left behind...my spots are very large and very black...what options do i have do ranging from natural to surgical procedures? Doctor: since CHILDHOOD ALLERGY MEANS ALLERGY TO MILK THESE ARE FOOD ALLERGIES CHILD NEEDS MOTHER MILK TILL 2 YEARS AND NO MILK AFTER THAT IF WE ADD MILK AND DIARY MOST CHILDREN START GETTING ALLERGIES WHICH ARE NEVR CONTROLLED BY MEDICINES GET BLOOD SERUM TEST FOR FOOD SPECIFIC ANTIBODIES FOR MILK ALSO TEST FOR WHEAT POTATO CHANA RICE NUTS POTATO TO SEE THEIR ROLE ELIMINATE` PARTICULAR DIET YOU WILL BE RELIEVED AND GO FOR DESENSITISATION FOR THAT FOOD FOR CURE TILL THEN TRY WITH MILK AND DIARY PROTEINS ELIMINATION"
},
{
"id": 124375,
"tgt": "What is the treatment of weak bone and muscles ?",
"src": "Patient: My 15 year old son who was very active physically lately cannot do anything physical. He has muscle spasms and passes out if he does. He becomes nauseas and his legs get numb. He has grown very fast in the past year. His doctor tested him for mono but the tests came back negative. I am worried. What could it be? Doctor: Hello, The weakness that he is feeling can be related to low levels of calcium and vitamin D3. You can get his levels evaluated and start with supplements accordingly. Hope I have answered your query. Let me know if I can assist you further. Regards, Dr. Praveen Tayal, Orthopaedic Surgeon"
},
{
"id": 83435,
"tgt": "What are the side effects of Nexito Plus 05 mg?",
"src": "Patient: I am male 39 yrs old, diabetic, suffering from mood swings & depression due to family conditions & job pressures, I am using Nexito Plus 05 mg from last 2 months after taking this pill I am feeling OK. I want to ask the side effects of this medicine on Liver,kidney and diabetes. How long I can safely use this medicine. Doctor: Hello, Let me tell you that apart from gastro intestinal side effects such as nausea, vomiting,diarrhea, nexito plus may cause several sexual dysfunctions such as low sexual desire,delayed ejaculation and sometimes anorgasmia. It has no specific effect on diabetes as such. It usually takes 4 to 6 weeks for escitalopram, which is one of the ingredients of nexito plus, to work and you can continue it upto 6-9 months although this depends on your treating physician. Hope I have answered your queries. Let me know if I can assist you further. Take care Regards, Dr. Prabhash Verma"
},
{
"id": 131831,
"tgt": "Noticed a spot just below the left rib towards back at the bone density test",
"src": "Patient: I had a bone density test and the technician told me that she saw a spot just below my left rib cage but towards the back. She said that it was not there 2 years ago when I had my last test. I had been hit by a car and fractured my pelvis 8 weeks prior to the test. Could it be something from the accident? Doctor: HiIt's unlikely.the shadow may be an artefact unless the radiologists opine it real.Show to a orthopaedic doctor to assess by examinationIt can be a simple bone cyst.Thanks"
},
{
"id": 194381,
"tgt": "What causes swollen testicle's epididymis?",
"src": "Patient: My left testicle's epididymis felt a bit swollen and sore earlier today. It feels a bit better now. I was wondering if it might be because I've held off from ejaculating while masterbating this week (I've been edging in hopes of saving my semen for a wet dream), or if it might be more serious. Thanks! Doctor: Hi, Yes, you can have pain if you are stopping the release of sperm during ejaculation, which can cause retrograde flow. Yes, it can cause infection and injury to your prostate. It is a very odd or injurious way to get wet dreams. Hope I have answered your query. Let me know if I can assist you further. Regards, Dr. S.R.Raveendran, Sexologist"
},
{
"id": 100697,
"tgt": "What medicine to take as I feel mildly unwell after having oyster?",
"src": "Patient: i had a reaction to eating an oyster. the symptoms were tightening throat, severe anxiety and feeling extremely cold. it took about an hour for the severe symptoms to relax. 12 hours later i still feel mildly unwell, any advice on medicines, anti histamines for example Doctor: Hello,It is probable that you are allergic to Oyster. Your symptoms were related to bronchospasm (narrowing of airways) and in severe cases it can be dangerous. I think you should not take oyster in the future. Other sea foods like shrimps also may produce symptoms and so be careful.If you do not have any wheezing or breathlessness you don't require any medicines. Antihistamines are not indicated for bronchospasm. If you have itchy lesions, you can take an antihitamine like diphenhydramine.Hope you will feel fine soonDr Noble Zachariah"
},
{
"id": 183343,
"tgt": "How can bleeding gums be treated?",
"src": "Patient: Hi there. My 2 year old daughter has bleeding gums around the teeth when brushing. She complains that it hurts to brush behind her teeth. Teeth and gums look healthy. We brush every night and floss too. She's late to teeth, and working on a 4th molar right now. No eye teeth yet...just the 3 molars that recently came in, and the 4th on the way. Doctor: Thanks for your query, I have gone through your query.he bleeding in the oral cavity could be because of the gum infection secondary to deposits. Or it can be because of a erupting tooth that breaks the mucosa and erupt, during this process the gums get inflamed and cause bleeding and pain.Consult a oral physician and get tge teeth cleaned.you can use stolin gum paint to prevent bleeding.I hope my answer will help you, take care."
},
{
"id": 144618,
"tgt": "What causes pain in left temporal region?",
"src": "Patient: I have been having a sharp pain in my Left temporal region for a couple years now. It comes and goes it has been happening more frequently and last night I went to bed with the pain and woke up with it this morning. Applying pressures to the area alleviates some of the pain but once I take pressure off it continues. What could this be due to? Doctor: unilateral intermittent temporal pain has few differential diagnosis 1. migraine2. temporal arteritis3. Some intracranial space occupying lesionbut in temporal arteritis usually it has some scalp tenderness and should not be releived by applying pressure.Still i advise you to get MRI Brain, ESR test done"
},
{
"id": 139419,
"tgt": "Does masturbation affects brain functioning?",
"src": "Patient: hello doctor i am 18 yr old and my height is 5.5 feet and weigh 75 kg. i masturbate 4 times a week and often get headaches after dat. i want to know wheather masturbation reduces brain efficiency or IQ in any way ???? and secondly wheather it stops the body from growing like effecting height Doctor: Hi, There is no scientific evidence to say that masturbation causes 'all those' problems you mentioned in the question. What you are describing is probably orgasm headache which is usually benign (not dangerous one). There is no specific treatment except symptomatic, like paracetamol. If it is really troublesome there are some medications which can be better taken after consultation with a neurologist. My suggestions to you: - Get your blood pressure checked once - Take at least 2 litres of fluid intake daily to improve your general well being - Avoid fast and junk foods, excess sugars, salt - Consume good amount of fruits and vegetables - Do regular exercise at least 30 minutes dailyTake care. Hope I have answered your question. Let me know if I can assist you further. Regards, Dr. Mohammed Ashraf, Internal Medicine Specialist"
},
{
"id": 52121,
"tgt": "strong odor Urine",
"src": "Patient: Hi! I've been having a problem for about a year now! When I go to the bathroom, my urine has a very strong odor, like it's been sitting in the toilet for 2 days. It's not a fishy smell or anything like that, it's just really strong, and the only way to explain it, is the old pee thing. I don't pee as often as I should either. I pee maybe 2 or 3 times a day, tops! I've been drinking only water for the past 2 months now, and that hasn't helped at ll. What could be wrong with me?? Thanks for your time!! Doctor: Urine with strong odour does not seem to be of much concern. Drink plenty of water and fluids. If\u00a0 it is aasociated with other symptoms then you may need a nephrologist opinion."
},
{
"id": 15817,
"tgt": "15 year old with rashes on the arms and legs. Came back from and international trip. Reason?",
"src": "Patient: hello, I am 15 years old and an weird rash has came up on my arms and legs about 2 days agoit went down a bit but I went out today and the rash has came up worse on my arms and moved onto my hands I have recently came back from Italy on a school trip and i'm not sure what the rash is I am having some Piriton for it but its not doing much good, would mean so much if you could help,thank you amellia Doctor: hitwo common things we have to think of are prurigo and sun allergy.how was the sunexpopsure during the trip?currently do the eruptions itch on sun exposure?the answers to above will help us guide you further.in the mean time topical application of mid potent steroids and oral anti histamines can help.why dont u visit a dermat so that you can b guided abt appropriate medicines for the desired period.hope it helps.feel free to ask further.regardsdr amit"
},
{
"id": 53972,
"tgt": "Can our body reject the gastro intestinal stunt placed after gall bladder removal?",
"src": "Patient: Can your body defecate (get rid of) a gastro-intestinal stunt placed here after gall bladder removal? I had some stones stuck in the liver duct and had to be placed a stunt but then after a month later, a regular x-ray DID NOT SHOW THE STUNT INSIDE OF ME ??I am 45 years olf 5'3 weigh -172lbs and am in good health. Doctor: Hi and welcome to Healthcaremagic. Thank you for your query. I am Dr. Rommstein, I understand your concerns and I will try to help you as much as I can.Yes, this can happen and this usually happens after certain period of time since all stents are moveable so this should not surprise you.I hope I have answered you query. If you have any further questions you can contact us in every time.Kindly regards. Wish you a good health.DR. Ivan Rommstein"
},
{
"id": 191494,
"tgt": "Suggest treatment for nausea and weakness while fasting",
"src": "Patient: My wife tried to fast on Sunday, she only missed breakfast and lunch. By 1400 she was weak, nauseous and felt horrible. She felt like she would faint. This is not a new thing for her. I took her blood sugar an hour after she ate crackers , and it was 97. She was always very sick with pregnancy and has the same symptoms when she tries to fast. sometimes she does faint and vomits. She always has to eat something as soon as she wakes up or she feels weak. I know breakfast is important, but she seems to be very sensitive and I m not sure what may be off to cause such and sensitivity. She finally started to feel better around 2030 when she got Gatorade in her. Her fasting glucose was 97 the next morning. Doctor: If she ripitadly suffer to nausea and feel weak. We indiian people have a samart home remedy which destroy those symptoms. Just take small small slice of ginger and put it month and slowly suckeed its juice . Just finished it between 10- 15 min. Or as you want. Its help to improve your digestion ,feel your mind better decrease nausea and produce intrest to food. Safe for pregnant womenYou can repeat it 3-4 mines a day"
},
{
"id": 60094,
"tgt": "Reports showed elevated bilrubin, serum bilrubin, ALT/SGPT, advised liver scan. Is it serious?",
"src": "Patient: Hi Doctor, Below is the report which was given to me when i made a Health check up at my office. Report: BILIRUBIN , TOTAL, SERUM BILIRUBIN, TOTAL H 2.02 0.00 - 1.00 mg/dL ALANINE AMINOTRANSFERASE, SERUM ALANINE AMINOTRANSFERASE (ALT/ SGPT ) H 61 0 - 40 IU/L ASPARTATE AMINOTRANSFERASE, SERUM. When checked with the doctor they asked me to take a scan for the Liver . Is there a serious cause in my liver. can you suggest me. Warm Regards, Pradeep Doctor: Dear Pradeep, The level of Bilirubin and enzymes do indicate that there is some form of inflammation in the liver. It is possible that someone can have Hepatitis due to viral causes and not have severe symptoms and when it begins to heal you may still show some changes in the blood tests. However , other things that could affect the liver include alcohol, even some prescription medications (Even over the counter ), Gallbladder problems etc. Sonogram will show if you have stones in the gallbladder. You do need complete screening for Hepatitis A & B. It does not appear to be serious. Good luck."
},
{
"id": 117370,
"tgt": "What is causing blood counts to drop?",
"src": "Patient: My sister has a blood problem and the doctors are doing test on the veins in the stomach. They are saying they think the bleeding is coming from the tear in a vein in the stomach. Her blood count drops from 10 to 8 in thrtee months therefore she takes 2 pints of blood at that point. This has been going for nearly five years and getting worse. She is a diabetic and has had heart issues. Is is also tested every rwo weeks and sometimes needs a blood shot. Please advise. Looking forward to hearing from you Doctor: Hello and welcome to HCM,Blood loss from any site in the body leads to fall in hemoglobin or anemia.You have mentioned that there is a tear in vein in the stomach.Tear in the veins of stomach can occur due to sudden vomiting, retching or varices.You need to consult a gastroenterologist for management of the condition.Treatment of the cause of bleeding will improve the fall in hemoglobin.Thanks and take careDr Shailja P Wahal"
},
{
"id": 177765,
"tgt": "Can Lbuprofen & Paracetmol Suspension be given for fever?",
"src": "Patient: My daughter is fever since yesterday 1). Her age is one year and seven month old. 2)Her body weight is 9kg. Now I have \u201cLbuprofen & Paracetmol Suspension (FLEXON SUSPENSION)\u201d PLESE I NEED YOUR KIND HELP . Is \u201cLbuprofen & Paracetmol Suspension (FLEXON SUSPENSION)\u201d ok to take for my baby fever ? .Best regards from Sadananda from Greater Noida Doctor: Hi...Fever of one day without any localizing signs could as well a viral illness. Usually rather than fever, what is more important is the activity of the child, in between 2 fever episodes on the same day.If the kid is active and playing around when there is no fever, it is probably viral illness.Paracetamol can be given in the dose of 15mg/kg/dose every 4-6th hourly that too only if fever is more than 100F. I suggest not using combination medicines for fever, especially with Paracetamol.Regards - Sumanth"
},
{
"id": 171966,
"tgt": "Suggest diet plan for kids after loose stool",
"src": "Patient: for chest infection my daughter aged 2.6yrs was given 2 doses of rosephine(750) the doc said not to come if she does have fever for the 3rd dose and to start klacid. 1 day she was fine the 2nd day after she consumed klacid she started vomiting and loose motion side by side. the doc gave her an injection and after that she did not vomit but as soon as she eats something she passes it as loose motion. the doc has prescribed supraxiz 100 for 7 days. since she is still not eating anything, kindly advice doc. he has no strength to walk also kindly advise us which food is best during this time. Doctor: Why is the doctor changing the antibiotic so often? I am really surprised the antibiotic is being revised almost every day or two! Please go to a good paediatrician for review of the case. As far as the issue of diet is concerned, you should allow her to eat whatever she wants to eat, but avoid oily and fatty foods till she is completely okay.Dr. Taher, with best wishes."
},
{
"id": 14075,
"tgt": "Suggest treatment for itchy rash spreading from scalp down the neck and upper back",
"src": "Patient: My daughter started out with an very itchy scalp. I checked for lice numerous times and there is no sign of them . Her scalp continued to itch. She now has a rash spreading from her scalp down her neck and upper back . The rash is particularly bad behind her ears. What could be causing this? Doctor: Hello and Welcome to \u2018Ask A Doctor\u2019 service. I have reviewed your query and here is my advice. She seems to have either scalp seborrheic dermatitis or scalp psoriasis. In that case, if I was the treating dermatologist I would suggest a coal tar and salicylic acid based shampoo for her. I would suggest that she used a topical steroid and salicylic acid based lotion for scalp application. In addition I would also suggest her to take an oral antihistamine e.g Cetrizine tablet for symptomatic relief from itching. These are all prescription medications and I advice you to kindly talk to a dermatologist in your region for the needful. Regards."
},
{
"id": 99592,
"tgt": "Suggest treatment for allergy to metal",
"src": "Patient: Weeping skin. years ago a plastic part of my glasses broke at the ear peice. I endured wearing the glasses with the exposed metal frame despite clearly being allergic to the metal. Probably 3 years. I finally have plastic framed glasses but they seem to irritate this same area. As I speak my skin is weeping uncontrolably Doctor: Hi,It can be due to your allergic to metal or even plastic when they come in direct contact with skin.This weeping of skin might due to vitamin A deficiency.The best way to cure allegy to any substance is to avoid its exposure rather taking seeking for treatment despite having good antiallergic drugs like steroids and antihistamines.You can also take vitamin A supplement once daily and eat vitamin A riched fruits or vegetables.If problem persists then contact your dermatologist first.Thanks"
},
{
"id": 112154,
"tgt": "Is Ibuprofen recommended for sharp shooting pain in back after having had a vasectomy?",
"src": "Patient: Hi,I'm a 28yr old male. I had a vasectomy about 9 months ago without complications. Since the surgery, I have had on and off sharp shooting pains in my testicles that come and go at random times. I am now concerned because I found a lump a little larger than a pea in my right testicle. It is sensitive to the touch. I'm expierincing a sharp stabbing discomfort feeling that feels like a spasm. Now I also have extreme back pain (about 8/10) on the left side. I have taken ibuprofen with no relief. Also, about a week and a half ago I had some urine leakage after urinating. No burning or pain with urination, and it has not happened since. I'm more concerned about the lump on my right testicle with the discomfort and the severe pain in my lower back on the left side. I also have had no recent injuries or traumas. Any information would be great as I can't get into the urologist for another week and the pain seems to be getting worse. Doctor: hi. post vasectomy you should not experience pain,rarely due to neuroma you may get some pain. your left side pain looks unrelated to vasectomy. it may be due to stone in kidney/ureter.you may take buscopan for pain relief till you see a urologist.other alternate is to see a surgeon or emergency physician"
},
{
"id": 115856,
"tgt": "Treatment for viral fever and low HB level",
"src": "Patient: I was suffering from viral fever and my Hb level was 6 .Before signing forced deed infront of notory other people never allowed to get medical treatment . what medical jurisprocedance openion in this case . After signing deed within 30 minmtutes District sergion admitted me in District hospitaland tested for blood and widal test. Doctor: Hi, dearI have gone through your question. I can understand your concern.Low hb level with viral fever need hospitalisation. Your hb is very low. you need hospitalisation and blood test to find out the cause of your severe anemia and then you should take treatment accordingly. If you had fever then widal test and malaria test need to be done to find out the cause. Hope I have answered your question, if you have any doubts then contact me at bit.ly/Drsanghvihardik, I will be happy to answer you.Thanks for using health care magic.Wish you a very good health."
},
{
"id": 78743,
"tgt": "Suggest treatment for severe ribcage pain while breathing",
"src": "Patient: I have been having rib pain on my right side for 4 months. It is better then ever but still hurts when taking a deep breath and is slightly tender to touch. Previously it was very painful when. Lying down or taking a deep breath. It came on after a bad cough, doctor suspected fractured rib but X-ray showed nothing. He said it could still be a fracture. Can it take this long to heal , thanks Doctor: Thanks for your question on Health Care Magic. I can understand your concern. Four months is too long, if it was rib fracture it should have healed by now.If the chest pain is disturbing your routine activity then repeat a chest x ray done to rule out any lung infection or it could be just a muscle pull. Do not lift heavy weights. Also get a ECG done just to rule any cardiac cause.Don't worry, you will be alright. Hope I have solved your query. Wish you good health. Thanks."
},
{
"id": 207059,
"tgt": "Suggest treatment for severe anxiety",
"src": "Patient: I m looking for a psychiatric doctor form a list of doctors provided by me work with Anthem blue cross. I m currently being treated for severe anxiety for the last 4 days. My primary doctor is out of town but prescribed me with 10 mg ambien. I went to the emergency room last night and they prescribed me with Ativan 1mg 3 times a day. I have been taking Effexor for the last 11 years but now my anxiety has been real bad. Doctor: you are taking benzodiazepines like zolpidem and lorazepam. but these drugs are hypnotics. will not help much. its better to take buspiron and fluoxetine. propranolol. these drugs will help you a lot. whenever you feel anxious take tab clonazepam 0.5 mg mouth dissolving named as clonotril. it will help you a lot.hope my answer helped you.do not forget to rate the answer."
},
{
"id": 137554,
"tgt": "Elaborate on calcification of collarbone",
"src": "Patient: I have been diagnosed with a calcification below the collarbone above the 1st rib and will be seeing a Rhuemetologist. What exactly is a calcification? I had rectal cancer in 2001, stage II, lower anterior resection and 6 months chemo. My oncologist is referring me over, should I be concerned, I have discomfort in my chest in that area and have been having low blood pressure with fainting type dizziness. Doctor: Dear patient Calcification is deposition of calcium at abnormal sites where normally it is not found for example in lung. You have hot calcification below clavicle and mostly in lung apex. Since you have been treated for rectal cancer with chemotherapy damage to vascular tissue and lung alveoli might have occurred which leads to abnormal calcification. Onother possibility is calcification of subclavian artery which lies just elow clavicle since you are having dizziness and fainting. You need to consult your oncologist and get I ourself examined. All the best."
},
{
"id": 66046,
"tgt": "Is lump inside and outside nose normal after bcc removal?",
"src": "Patient: I have had bcc removal about 5 weeks ago using flap operation, the scar seems awful lumpy & their is a big lump outside and inside my nose where it was removed. Is this normal & will this lump go away ? I am due to see the plastic surgeon on Tues. Please help as my son gets married 1 Sept this year. Doctor: Hi, dearI have gone through your question. I can understand your concern. You lump inside and outside the nose is not normal. It is pathological. You should go for examination. Hope I have answered your question, if you have doubt then I will be happy to answer. Thanks for using health care magic. Wish you a very good health."
},
{
"id": 218714,
"tgt": "Are R-Cinex and Benadon safe during pregnancy?",
"src": "Patient: I am on r cinex and benadin as I was diagnosed with lymph node tb around my neck. This is my 8th month since I am taking medicine as per doctor I am suppose to take medicine for 9 months so only 1 month is left. I am 7 weeks pregnant and concerned whether taking medicine will harm my baby. So please suggest Doctor: Hi there, I have understood your concern and I will suggest you the best possible treatment options.Please do not panic about the medicines.Tab Benadon is vitamin B 6 preparation and is essential vitamin. It is helpful for pregnancy as well as helps liver to tackle Rifampicin metabolism. Secondly, Rifampicin itself is not known to be harmful for pregnancy as such. It's not known to increase the risk of abortion or fetal anomalies.Benefits of Use of Rifampicin always out weigh the risks with the use of the medicine. Please be in touch with your treating doctor, complete the treatment course rigorously.Opt for a healthy diet and regular exercise regimen.Start on Folic acid and vitamin B 12 supplements.I hope this answer helps you.May God bless you with a bundle of Joy.I hope this answer helps you.Thank.Dr. Purushottam Neurgaonkar"
},
{
"id": 172149,
"tgt": "What causes flesh colored bumps on buttocks?",
"src": "Patient: My 8 year old daughter has had 2 flesh colored small bumps on her right butt cheek for 2-3 years. Originally the pediatrician felt it was caused by a viral infection and it would go away in time. It has not! One of the bumps has changed to a hard white center with red around the outside of the bump. She is complaining that it hurts as well. Please help us figure out what is going on!! Doctor: They could be fungal infection. It is caused due to constant wetting of groin area.u can try topical antifungal ointments. It will take some six eight weeks for that. Keep the nappy area dry. Use only cotton under wears."
},
{
"id": 4745,
"tgt": "Had unsafe sex during ovulation. Taken Plan B. Negative PT. Worry about pregnancy?",
"src": "Patient: Like most guys, I'm freaking out about the possibility of my gf being pregnant. We had sex around 1 AM on 6 Oct. The condum broke and she took plan b 2hours later. We looked at the date of her last period 17 Sept and now I'm freaking out because this means she was most fertile and probably ovulating. What are the chances of her being pregnant? Should I worry? By the way, she took a pregnancy test Sunday and it was negative. Doctor: Hellothanks for your query.The plan B is 95 % effective in preventing a pregnancy when taken within 24 hours, however the rate is less around ovulation.if she skips her period, please go for a pregnancy test.If it is negative, do not worry, as plan b often causes normal cycles to be delayed or preponed by a week or more.Repeat the pregnancy test after a week, and if still negative, consult a gynecologist for further evaluation.Take care"
},
{
"id": 199450,
"tgt": "Suggest treatment for premature ejaculation",
"src": "Patient: hi , i an 25 years of age and i do masturbate sometimes. but now a days i am experiencing premature ejaculation as soon as my girlfriend touch me on my penis . it takes only 5 to 10 sec for me to ejaculate. i have also experienced that my neck starts paining just after i masturbate . please find me a good treatment i will be obliged . Doctor: Dear Patient, Masturbation differs from real sexual intercourse with a female. It is a new experience for you with your female partner,therefore,your very high level of sexual excitement and anxiety can cause your premature ejaculation. This is absolutely normal and you will overcome it when you have more sexual experiences with your partner. Also you can follow ''Masters and Johnson\" guidelines to delay premature ejaculation. It means you or your partner can stimulate you but at the moment of ejaculation you or your partner can stop stimulation by gently squeezing the head of the penis. After a while you can again start the stimulation and stop it just before ejaculation. This will help you to extend your foreplay and delay your ejaculation. Also if the course of your premature ejaculation related to stress or anxiety, you can do breathing exercises to relax yourself. If you are needed SSRI inhibitors like Fluvoxamine to treat anxiety related conditions that has to be prescribed by a physician only. However, I believe by talking to your partner and practicing above mentioned pause-squeeze techniques is the best way to solve this problem without medication. I hope you won't need medication and those techniques will help you to overcome your problem.Good Luck!Dr. Dinesh Weerasinghe MD."
},
{
"id": 197766,
"tgt": "Need medication for Jock itch",
"src": "Patient: I have recurring \"jock itch\" like symptoms and subsequent yeast infections. I've never had this before. What can be causing this? What treatment is available? I've used traditional OTC treatment and even a stronger prescription. In addition I've used vinegar and tea tree oil Doctor: Hi.Welcome to Healthcare Magic.The reasons for repeated yeast infections are:1. Immuno compromised condition.2. Diabetes.3. Thyroid abnormalities.4. Poor personal hygiene.If once when we know the exact cause then we can treat it accordingly.You may be required to use topical anti-fungal cream for 2 weeks along with oral anti-fungal medications.Please try to see your Doctor, so that he can perform blood work and then can plan your treatment accordingly.Hope this helps.If you have any other questions, then i will be happy to answer them.Wish you good health.Regards.Dr. Surender sharma."
},
{
"id": 57726,
"tgt": "What should be the treatment and diet for hepatitis C?",
"src": "Patient: My mom is sufferung from a disease hepatitis c. I need your help and guidance. She has gone through a few laboratry tests. I request you to please go through these reports and tell me the status and guide me about her treatment please. Tell me about her diet as well. Please send me the email id so that I may forward these reports to you. Doctor: HIThank for asking to HCMI really appreciate your concern, lot of antiviral regimes are there that can be used according to the clinical condition of the patient and total blood work done, for the diet she can take routine food, only alcohol need to be stop, hope you can understand this take care of your mother, and have nice day."
},
{
"id": 24760,
"tgt": "What causes shortness of breath,rapid heart beat and pain and burning in heart?",
"src": "Patient: I have had shortness of breath, very rapid heart beat, followed by my heart feeling sore and then after the soreness there is extreme burning and pain in the heart area. This had been happening for 2 weeks. Starting very gradual but now it is happening several times a day. I am a 33 yr old female. I went to er 2 weeks ago at onset and they said anxiety. I went to my family dr. He had and echo done. Results were mild mitral valve reguritation. I also wore a holter monitor. I had periods of where my heart rate was up to 220. My dr. sent me to a cardiologist. He looked at my er report and tols me this was all anxiety. He did not even want to hear my symptoms or the progression of symptoms. Said shortness of breath could be allergies. Askes about weight gain but would not even talk to me about why I was reffered(mitral valve regurgitiation). I just want my life to go back to normal. Please help. Doctor: Thanks for your question on Healthcare Magic. I can understand your concern. No need to worry for heart diseases as your ecg, 2d echo, Holter monitoring and cardiologist consultation are normal. It is true that uncontrolled stress and anxiety can cause similar symptoms. So better to consult psychiatrist and get done counselling sessions. Try to identify stressor in your life and start working on it's solution. You may need anxiolytic drugs too. Don't worry, you will be alright. Avoid stress and tension, be relax and calm. Hope I have solved your query. I will be happy to help you further. Wish you good health. Thanks."
},
{
"id": 178469,
"tgt": "Will ear infection cause pain during a flight?",
"src": "Patient: My son was diagnosed with an ear infection today (Tuesday) and was sent home with amoxicillin. He had one dose this evening before bed. We have a necessary flight on Thursday evening. Will it be okay to travel with him on an airplane or will he be in terrible pain? The Dr. Today said he should be fine but I'm wanting another opinion. Doctor: Hello. I just read through your question.By the time your flight departs, he will have had 48 hours worth of antobiotic. that is usually enough time. He will be fine."
},
{
"id": 41853,
"tgt": "Suggest a remedy for infertility",
"src": "Patient: My son 24 year old with sub fertility, azoospermia (absence of sperm within the semen), normal (male) appearance, unremarkable prior medical history, hormone evaluation (including FSH 90-.03 mIU , LH 13.99 mIU , Estradiol (E2 ) 30pg/ml , Testestron total 3.79ng/ml normal size and appearance of testes, normal ultrasound examination, and AFTER OPERATION OF ONE TESTICLE UNDER GENERAL ANESTHESIA and histopath result there is maturation arrest in spermatogonium stage ,please would like to ask ,there is aim for sperm production or there is treatmentB RALI MOTHER Doctor: Hi welcome to healthcaremagic.I have gone through your question.Had he undergone any operation in past like testes reimplantation?Maturation arrest has not any medical treatment. Although consult urologist he will proper person to guide you further.Hope i answered your question.Take care."
},
{
"id": 166445,
"tgt": "Suggest treatment for enlarged adenoid in 3 year kid",
"src": "Patient: my six yrs old boy has enlarge adenoid, But he doesn't snore at all , he doesn't breath by his mouth or having his mouth open all the time, he can blow his nose without problem. He has never had otitis. But the doc has told me he should do sugery! and he should not go in swimming pool! Im so confused.I was thinking about the fact that he has no symtoms, no pain or anything like that probably if we could wait a bit longer before going to surgery... And why not swimming pool?The doc told me because water getting in his ears can cause chronic otitis!!!!!!pls i need some advice, thanks. Doctor: Ok going to swimming pool increases chances of getting infections more commonly upper respiratory tract infections like tonsillitis , pharyngitis, adenoiditis. Since water in pool is contaminated and during swimming child swallow the water, so it is better to protect the child from swim more catiously if he has adenoids problem. As you said there is no symptoms so please do not go for surgery who ever say to you for this. Unless child is symptomatic like snoring causing disturbance in sleep, breathlessness, frequent severe infections, affecting speech no surgery is indicated. It may regress gradually by the age of 9 or 10. Please get them recheck."
},
{
"id": 212132,
"tgt": "Feeling bubbles in the legs, dizziness. Have stress. Have vitamin B12 deficiency. Is that the reason?",
"src": "Patient: My legs and feet feel like there are little bubbles popping in them all the time. I am 60 years old and my mother has Parkinson's disease and is on Hospice Care for pancreatic cancer. I am under a lot of stress right now. I also lost my brother to chronic liver disease about 4 months ago. Could any of this be related. My legs do not hurt. I am forgetful and get dizzy (vertigo) at times. Just wondering if any of this is related? My hands even shake sometime. I did recently just find out that I have a b-12 deficiency. In now take b-12 twice a day. Should I be concerned? Doctor: Dear I understand your concern. May be you are at your maximum height of stress. You are 60 and you witnessed a death of brother and mother is terminally ill also. Most probably you are realizing that your fate would also be something like that. But is it true? This is the time you should have full faith in god. He created you, nurtured you,developed you etc. and when he wishes, he will take you with him. That is the truth. Truth is always bitter, and now your trying to deny the truth, worry about future, health etc. Forget all. You are now good.But nobody can say what is tomorrow. Why worry about what you are not certain or about what you cannot control? Be happy now. Thats all. Bible says: Live life as if today is your last day in life, and enjoy."
},
{
"id": 223832,
"tgt": "Should I be concerned for bleeding after taking EC?",
"src": "Patient: Hi I had unprotected sex and took an EC the day I was ovulating. However, I experienced lighter than usual bleeding 4 days before my expected period. Normally my cycle is 5 days. I am on the 6th day and its heavy and smells funny with slight cramping occasionally. Should I be concerned? Doctor: Hi you do not have to worry ,as you got your period and the irregularity is usually due to EC as it contains hormones"
},
{
"id": 58832,
"tgt": "Diagnosed with cirrhosis, sleeplessness, ammonia levels abnormal. What can be done?",
"src": "Patient: Hello I have a sister who was diagnosed with cirrhosis, was hospitalized for over 2 months. was doing well and all of a sudden she has gotten out of control, crazy like, not sleeping for days, running around like a crazy person. Know one can say anything without her being defensive. We don't know what to do? Is it her meds? She has had some problems with her ammonia level. How can we help her and what should we do? We are desperate. Can you help me understand what could be wrong and what I should do? Doctor: Hello! Thanks for putting your query in HCM. I am a Gastroenterologist (DM).The symptoms your sister is having, along with increased ammonia levels suggest that she is having hepatic encephalopathy (HE). For treatment of HE you should know the factors that leads to HE in stable patient of cirrhosis like overuse of diuretics like lasix or spironolactone if she is on them, Gastrointestinal bleed, low potassium levels, medications like benzodiazepine, dehydration, low sodium levels, infection, constipation and diarrhoea.Treatment is correction of the precipitating factorI will like to advise that you should visit a gastroenterologist before the thing worsen.I hope I have answered your query and this will help you. Remain in touch"
},
{
"id": 13419,
"tgt": "What causes recurrent itchy blisters on the toes and feet?",
"src": "Patient: Good morning...My name is Heather & I am trying to help someone very near & dear to my heart figure out what might be causing some issues they are experiencing...Hopefully you can be of some assistance...for over a year now, they have had reoccurring episodes (every 2 months or so) of tiny, extremely itchy blister like rashes on the exterior of his heels, also on the outer edges of the soles of his feet but not underneath on the bottom of the soles & between his toes on both feet, on the upper buttocks (above & slightly down into his butt crack) & in his upper groin/pelvic region...he complains that the itch is almost unbearable & when I observed these areas he had scratched them so much that it s caused some scabbing...when popped, the tiny blisters release a clear fluid/pus...we ve tried so many different things such as creams & lotions with no real results...like I said these episodes have been reoccurring for some time now & we have racked our brains trying to figure out what might be causing their on-set but again are dumb-founded...I should inform you that he has been regularly checked for STD s & ALL tests have come back negative...in the past he had tested positive for H-Pyloria but was successfully treated...any help or insight into this bizarre nightmare would be greatly appreciated!...a little more information on him...Black/Hispanic male...50 years old...good health...takes anti-anxiety & acid reflux medications daily...never has been a drinker, drug user or smoker... Doctor: Hi, Your near and dear fellow may be having dermatitis herpetiformis most probably. However, you consult the dermatologist for the confirmation of diagnosis. You may give him Dapsone thrice a day till he gets relief. And you may add cetirizine 10 mg daily at night. Apply antibiotic cream to prevent secondary bacterial infection. Hope I have answered your query. Let me know if I can assist you further. Take care Regards, Dr Ilyas Patel, Dermatologist"
},
{
"id": 174558,
"tgt": "What could severe, sharp pain in abdomen, diarrhea with nausea in a child indicate?",
"src": "Patient: My son is ten years old. Lately he has been complaining of severe sharp stabbing pains in his abdomin. he says that he feels faint and has diarrhea. Once the diarrhea is over, he is fine. He metioned that it usually onloy happens once a day. He says that sometimes he feels nauseated as well. I am perplexed. Doctor: Hi,From history it seems that he might be having gastro-intestinal infection producing stomach pain and loose motions.Give him antispasmodic medicine as needed.Give him one course of Ofloxacilin, metrogyl combination medicine course for 3 days.Give him light diet and plenty of water.Ok snd take care."
},
{
"id": 63491,
"tgt": "What is the small hard lump below my ear lobe that radiates pain?",
"src": "Patient: I have a rice sized, hard bump behind my lower left ear directly behind the lower ear lobe that seemed to appear over the last 4 months and won't go away. It feels it is sometimes sending an infrequent pain down the side of my next that lasts for 2 seconds. Doctor: Hi.Thanks for your query.Read and understood your history of a small rice sized hard lump behind the ear that sometimes cause 2 second sharp pain. The possibilities are:GranumolaNeurofibromaCyst on a small sensory nerveSince it is so small the ultimate treatment would be to get it excised under local anesthesia and send for histopathology examination to get a final diagnosis. Also watch for similar lumps anywhere else in the body."
},
{
"id": 25312,
"tgt": "What causes chest pain?",
"src": "Patient: I was resently told that my ANA was positive again after 12 years, I went to the ER today with chest pains normal BP is 90/60 was 140/80 heart rate going from 66 to 110 when my chest pains were happing the only blood work that was abnormal was my ALT was 65. Gave me Xanax, Morphine and still having chest pains just not as sharp Home now still having chest pains taking lortab and Xanax ...What is wrong Doctor: Hi thanks for asking question.Elevated ALA here can be because of heart problem.Do you have done ECG?You have positive ANA ..so you could have SLE like multisystem disorder ..Rule out it by antids DNA or other suitable method.SLE can lead to endocarditis and so chest pain can occur.Do your ECG to rule out cardiac condition.If still pain increasing and respiratory symptoms present with fever chest x ray done to rule out infection.If you have long term viral infection then also chest pain can occur as constitutional symptom.Wish you good health...Take care..."
},
{
"id": 209075,
"tgt": "Suggest medication for social fobia and obsession",
"src": "Patient: hi sir i'm rana from bangladesh i have social fobia and obsession from my chillhood.i have gone to a doctor,he gave me madicine Nexito 10 and other madecines.i am taking Nexito 10 from 3rd march 2011 to till now.my doctor has gone to abroad...that's why i can't check up by him.so sir what must i do??shalll i cotinious my medicines Doctor: HiYou should continue medication as of now and consult your doctor ASAP. Nexito can be increased up to 20 or even more. How much is improvement so far with nexito. Anxiety disorder need a long term treatment and don't discontinue treatment without consultation.Regards,Dr Arun"
},
{
"id": 97197,
"tgt": "What treatment should I go for stuff neckand and chest pressure?",
"src": "Patient: Dear Sir, During childhood,due to an accident my jaw muscles became stiff.After ignoring it for years,the problem is I have a stiff neckand looks lik my left ear is closed.i feel as if my neck is falling downwards and can feel the presuure on my chest.What treatment should I go for?pl suggest. Yours Faithfully, Upen patel,kolkata Doctor: Hi, This requires regular physiotherapy, muscle relaxation exercise and surgery may be required. Get examined by your doctor once for evaluation. Regards"
},
{
"id": 110776,
"tgt": "What causes recurring back lower back pains and dizziness?",
"src": "Patient: Hi, I have been experiencing on and off lower back pains and when I do have them they are very painful. I also get back pains and lightheaded when I stand for a long period of time. What does this mean?? (Sidenote: I was also diagnosed with very mild scoliosis) Doctor: Hi, thank you for posting!I have gone through your query and I understand your concerns.Your symptoms are caused by neck and lumbar arthritis. Dizziness is caused by neck arthritis.Physical exercises, massage and non steroidal anti inflammatory medications can help you.To confirm the diagnosis you need:1. Lumbar and neck Ct-scan.2. Complete blood count.3. Fibrinogen test.4. Urine test.5. Glucose test.You should consult your neurologist for the diagnosis and treatment.All the best.Dr. Behar."
},
{
"id": 224832,
"tgt": "Sprintec birth control pills, constipation, weak, itchy, no rash, feels bolted,",
"src": "Patient: Yes hi how are u doing today. As for me some what. I have been on sprintec birth control pills for 3months now. The first two months were great no problems but now since i started my 3month i started having constipation and now for like a week i have been itchy all over my body no rash thought, i feel bloted. Is horrible. I have tried stool softners dont work, laxative oills, not anymore, i excirse about 6 days a week and eat alot healthyer. Was the problem now. I have 2 kids no more for me in a long time. Please help. Oh also have feel a bit nauseous sometimes not always. Doctor: HIThank for asking to HCMI appreciate your concern, looking to the history given here this could be allergic condition and nothing to rake take with the contraceptive pill, if I would be your doctor then I would treat this with the following medicine,1) Tab Levocetrizine 10 mg three times in day For the constipation just no need to take any medicine, drink more water eat fibber containing diet this will come around, hope this information helps you have nice day."
},
{
"id": 153215,
"tgt": "Suggest treatment for cancer of food pipe",
"src": "Patient: Good morning Doctor, I live in Melbourne, Australia. I have a query for my uncle who has Cancer of the food pipe and is very serious. He is being treated by Dr Rai in Chennai and is in his hospital. I am afraid from what I hear that he is not getting the required attention. I am thinking of moving him to the Adyar Cancer institute. I am concerned that my uncle is suffering far too much. Can you please suggest what I do in this situation. Thank you for your time Doctor. Doctor: Hi, dearI have gone through your question. I can understand your concern.Your uncle has esophageal cancer. Treatment of esophageal cancer depends on type of cancer, stage of cancer and spread of cancer. In early stage surgery can be done to remove that part. In later stage chemotherapy and radiotherapy are the treatment options. Please send me his all reports so that i can help you further Or consult your oncologist and plan accordingly.Hope I have answered your question, if you have any doubts then contact me at bit.ly/Drsanghvihardik, I will be happy to answer you.Thanks for using health care magic.Wish you a very good health."
},
{
"id": 105335,
"tgt": "Vomiting, dizziness after drinking bubble tea. Allergic reaction?",
"src": "Patient: Do you think I m allergic to soy as the other day I drank bubble tea that was dairy free (not sure if it was soya or not) and I threw up six times and almost fainted, I don t think it was a sickness bug because none of my family caught it, and also we all had some of the bubble tea so there wasn t anything wrong with it Doctor: Hello, I am unsue what was in your bubble tea but checking the list of ingredients will help. Soy allergy is not very common and a soy-free diet is very difficult and almost impossible to continue for more than a few weeks. If you have a problem with drugs such as pain killers (NSAIDs) or aspirin, occasionally salicylates in tea can cause such problems as it gets concentrated. Hives on skin, vomiting after ingestion of foods (within 1 hr usually) is a sign of allergy and needs to be checked in detail. Keep antihistamines (cetirizine 10mg) handy for future but get this checked if it happens again especially if you have underlying asthma. Thanks."
},
{
"id": 50048,
"tgt": "Elevated BUN, serum creatinine after radial nephrectomy. Sign of chronic renal disease?",
"src": "Patient: Hello Sir, I am patient of lymphoma who also underwent radical nephrectomy of right kidney! Recent blood test indicated elevated BUN 60 and Serum creatinine of 3! Had edema of feet,also had high PTH values! BP was elevated! But now after BP medication BP is under control, edema is gone and i have stopped getting up in the night to pass urine! Do you think i have chronic renal disease Doctor: HiThanks for the query.It is difficult to diagnose if you have chronic kidney disease based on the information provided.You need to repeat the blood tests as well as do urine exam and Ultrasound of the kidney and bladder.Based on these tests and a careful analysis of your past history and medications etc.; a nephrologist would be able to comment if you have chronic kidney disease.Hope this helpsGood luck."
},
{
"id": 204093,
"tgt": "How can one overcome complex post-traumatic stress disorder and borderline personality disorder?",
"src": "Patient: Hello, I have been diagnosed with Complex PTSD and BPD traits and have done three modules of the DBT course but am yet to complete the last module. I have seen a few psychologists over many years but am at a complete loss . i just need to talk to a psychologist (but an insightful one whp understand what I need)i FIND THAT IT is very difficult to find doctors who are interested and engaged and really want to help ppl like me as most want to lump us all ito the group of hard baskets and many believe that we are out to just stalk them or do some crazy stuff so many are unwilling to even see ppl with DBT. not many are compassionate as to the circumstances that have brought up thus far. so basically my Q today was wether yoi knew anyone just for one or two sessions for me to talk to about my progress thus far and what my future with this psychological illness holds. or what I can expect in future. Thank you for your assistance. Doctor: Hi, In both, we need to have a combination of pharmacotherapy as well as psychotherapy to have a better outcome. Tab sertraline along with PPI works wonders. Hope I have answered your query. Let me know if I can assist you further. Regards, Dr. Junaid Nabi, Psychiatrist"
},
{
"id": 185678,
"tgt": "How can paresthesia on the roof of mouth be treated?",
"src": "Patient: I have had paresthesia on the roof of my mouth - just on the left side for the past 2 years. Feels like it is burnt. I also have a strange taste of grapes in my mouth constantly for the past two years. I can hardly take it anymore. I have been to dental surgeons, dentists, etc. No one can give me an answer. Doctor: helloi have gone through your query.you have not mentioned your age.have you gone any surgical treament like extraction of a tooth?have you checked your blood sugar level?do you have any habit like intake of alcohol?any recent disease or medications u r taking?Any type of tingling, burning, or numbness is usually a symptom related to a sensory nerve being damaged, diseased, or injured.following medical conditions are some of the possible causes of Mouth tingling/ paresthesias:Diabetes mellitusAlcoholismHypocalcemiaSpinal cord lesionNerve root compression Herpes zosterMultiple sclerosis neuropathygenerally parsthesia resolves on it's own over time.persistent parasthesia can last apx for six months.for persistent paresthesia, it may be possible to attempt some type of surgical repair depending upon the actual cause of parsthesia.as in your case you r having it over 2 years i wud suggest you to visit a oral and maxillofacial surgeon and a physician to find out the actual cause of your parsthesia.Hope this ans helpsRegardsDr. Shesh"
},
{
"id": 70514,
"tgt": "What is the growth in my butt oozing blood and not stopping?",
"src": "Patient: Hi. I felt a growth of some sort in my butt for a few weeks now. During the past few days I've had blood pouring out of my butt, I just went and there's tons of it and it won't stop bleeding. I'm not having stomach aches and I don't know what to do. Do you know what it could possibly be? Doctor: Hi.This can be a fistula in ano, a condition which needs a consultation with Gastro Surgeon.This can be a recurrent affair. Please request your Doctor to go for the tests , particularly the fistulogram for you to predict the future course of the disease and the possible treatment for cure.Do it ASAP"
},
{
"id": 95114,
"tgt": "Appendix surgery, pain in left and right side, pain where surgery was performed",
"src": "Patient: Hi, i have pain both at my left and right sides. I had my appendix removed few years ago but i still feel pain there and on the exact spot at my left. What does these mean? Doctor: Hello. Welcome to HCM You had appendicectomy few years back. Local pain will not persist for so long. But minor abdominal surgeries may cause adhesions and intra-abdominal bands which can cause altered bowel motility and thus lead to vague lower abdominal pain on both sides. Pain exactly opposite to side of appendicectomy needs to be investigated by at least an ultrasound.Make sure you eat healthy with plenty of dietary fibre water intake. If pain persists and is severe you need to visit a gastroenterologist and get investigated at the earliest. Take care."
},
{
"id": 207344,
"tgt": "Suggest treatment for inferiority complex and phobia",
"src": "Patient: Hi, may I answer your health queries right now ? Please type your query here... my sister in law suffered from diffuse cerebral atrophy as per MRI scan report. She is getting afraid while seeing crowd. Seh find difficult to walk normally. While get gown from staircase, she cannot fold her led and just making steps without folding her leg. She has inferiority complex. Doctor: she is having social phobia which is treatable. go to psychiatrist he will rescribe fluoxetine tablet for three months side by side behavioral therapy. she will be fine soon. so don't worry. hope my answer is helpful and thankful to you. take care."
},
{
"id": 108370,
"tgt": "Suggest treatment for severe back pain and neuropathic pain",
"src": "Patient: I started with pain in my back on right side before Christmas. My doc said I needed a massage. I went for a massage and had no relief. Over Christmas, my arm and hand started to tingle and go numb. I went to doc after Christmas and he gave me painkillers. I started physio therapy treatments, and they started dry needling treatments. I am presently going for massage and dry needling, still no relieve. I have since developed a large red patch under my arm. I saw another doc on Saturday who said I am suffering from neuropathic pain and prescribed lyrica. I am starting to suspect that I have shingles. What do you think? Please cancel this query I didn't realize there was a charge for it. Doctor: i understood your concerns no need to worry, just be calm and relax.... Neuropathic pain (also called nerve pain or neuropathy) is very different from pain caused by an underlying injury. While it is not completely understood, it is thought that injury to the sensory or motor nerves in the peripheral nervous system can potentially cause neuropathy. Neuropathic pain could be placed in the chronic pain category but it has a different feel then chronic pain of a musculoskeletal nature. back pain especially neuropathic may be due to compression over sensory nerves, avoid massage and dry needling treatment continue with physiotherapy take ift ultrasound and mild back exercises...."
},
{
"id": 47487,
"tgt": "Suggest medication for recurring fever after stent placement in kidney",
"src": "Patient: My brother is having a DJ stent placed in the kidney for calculi at mid uretor. He has come home. He was prescribed with antibiotic medicines, pain killers and medicine for fever. he even joined office for 2-3 days. Now he is having chills with fever. On advice of doctor he is taking calpol tab . the fever stops with lot of sweating. Is this ok. should he take calpol only if fever comes or is it that he has to take it even if fever is not there...pl advise. Doctor: HelloThanks for query .Your brother has been detected to have stone in the mid ureter and D.J stent has been placed in his renal unit to felicitate stone to pass down easily without any pain .The fever with chills that he has is mostly due to inflammation of ureter (Ureteritis) or primarily due to nfection .He needs to get his urine culture done to trace out the organisms causing this infection and to know the antibiotic sensitivity .Please give him broad spectrum antibiotics like Cefixime along with urinary antiseptic like Nitrofurantoin twice daily and urine alkalizer thrice daily .Later on switch on to appropriate antibiotics as per culture report.Get the prescription of medicines from your family Physician.Ensure he drinks more water .To keep your urine dilute This will help to control dysuria.Do not worry his will get resolved once the stent is remved .Dr.Patil."
},
{
"id": 101453,
"tgt": "What causes recurrent allergies despite medications?",
"src": "Patient: Hello My son is 2 years old and have a allergy problem and doctor start Myteka sachet for one month pls tell me its right because last week he have a infection and dr start Cyclor bt after one week again he have infection and dr started injection for treatment pls reply Doctor: Hello.Thank you for asking at HCM.From your history, I guess that your son has got frequent respiratory complaints, most probably due to allergy.Myteka contains montelukast and it does not have major side effects even after using for a few months in children. Rarely, a child may have nausea, vomiting, headache, etc but they are not seen in most of the patients.I would like to inform you that infections are different from allergies. So if your son is having recurrent allergies, please do not worry. There are good controlled medications for allergic respiratory symptoms.However, if he is having recurrent infections, I would suggest you to consult a pediatrician who may suggest some tests for immunodeficiency. It's an entirely different group of diseases, fortunately far less common than allergic disorders.Hope this will help you.Wish your son fast recovery & best health.Regards."
},
{
"id": 39629,
"tgt": "Will neosporin suffice in case of yellow drainage from an ingrown nail?",
"src": "Patient: Have ingrown toenail.it had yellow drainage a week ago, have been on warm soaks and neosporin since then.the swelling has come down a bit and yday I started applying soframycin. It is getting better but I am not sure if there is an abscess. What must I do. I can't take oral antibiotics as I am breastfeeding Doctor: HI, thanks for using healthcare magicAn abscess is a collection of pus which is definitively treated by a small incision to encourage drainage along with antibiotics.If the swelling and other evidence of infection is improving then you can consider monitoring it for now.If there is no further improvement or the symptoms worsen then you should consider visiting your doctor.There are some oral antibiotics that can be used in breastfeeding if this is what is needed.I hope this helps"
},
{
"id": 16467,
"tgt": "How can I get rid of this pimple like on my head which is itching ?",
"src": "Patient: Am Mr Peteri have brown pimple like on my head itches a lot 2moths now i tried medicated soaps invain can you help Am Mr. Peter,i have brown pimple like on my head itches a lot 2moths now, i tried medicated soaps invain can you help? I do have dizziness when I lie on bed Doctor: hi better if you send snap. apply eumosone cream over it for 10 day,it will work. for itching you can take cetrizine tab at night."
},
{
"id": 169858,
"tgt": "What causes a red pimple at the back of the neck of a 5 week old?",
"src": "Patient: Hi. My baby is 5 weeks old and we just noticed that she has a red pimply red on the top of her back and the back of her neck. She has a little rash, same one as the back, on her chest. We changed over from breast milk to formula milk, just yesterday. Could this be the cause of the rash? And if not, what is the cause of the rash and how can we treat it? Doctor: changing milk has no relation with the pimple. it's called a neonatal pustule quite a common finding in newborns. count the number of pustule she has. if it is more than 10 then she might need to be seen by a practitioner. cleaning it with betadine douches and keeping dry will be enough."
},
{
"id": 128417,
"tgt": "Suggest remedies for hip pain",
"src": "Patient: Hi, I have been having hip pain for few months now. Initially there was pain while sleeping on the side. But now there is pain all time. It also radiates to the thigh and there is tightness in the groin as well. On sitting, there are weird sensations in the thigh and legs. Doctor: Dear patient what is your age? if pain is constant and on one side of the joint likely cause is infective cause or inflammatory pathology. I would like to advise xray of the pelvis with both hips anteroposterior views. please get it done from radiology centre nearby you. Meanwhile start tab diclofenac sodium 50 mg twice a day for pain relief. If xray report is abnormal please consult orthopaedic surgeon nearby your area."
},
{
"id": 176059,
"tgt": "What causes itching and pimples on neck and elbow?",
"src": "Patient: Dear sir, good morning, my son is 11 years Old, he is getting sample pimples in his neck and itching. in elbows also small pimples are there. past one year he is facing this problem, before that his skin was good, we met 4 to 5 dermatologists in bangalore, please give me the solution for my son s problem Doctor: Hi....by what you quote I feel that this is phrynoderma. But skin conditions are best diagnosed and treated after seeing them. So I suggest you upload photos here for me to guide you scientifically.Regards - Dr. Sumanth"
},
{
"id": 117185,
"tgt": "Is bone marrow transplant or blood transfusion advisable for low WBC?",
"src": "Patient: My friend has white blood cell count. What causes this. He is under doctors care. Had 3 options to cure this. Blood transfer, bone marrow transplant or do nothing for awhile. They don t seem to know what is causing the low white blood count. Ellen Kay Dirks bone marrow transplant Blood transfusion or not do anything for awhile. Doctor: HI Thanks for posting your query . WBCs are produced in the bone marrow . Low WBCs could be due to some problem in the bone marrow or problems outside bone marrow which suppress bone marrow to produce WBC. So it is essential to know what is the problem exactly . I would advise you to upload the reports of test done on your friend so that I can advise better. If a Bone marrow biopsy is not done then better get it done . The treatment depends on the cause of problem . Any clarification feel free to ask . Happy to answer. Regards."
},
{
"id": 128032,
"tgt": "How can a pulled muscle in the hip area be treated?",
"src": "Patient: I fell two months ago, (basically did the splits!) didn t hurt much at time but week later I had a pulled muscle in my inner hip/groin area. hurts with movement of getting out of bed or out of car where I have to move my leg sideways. Do I just take it easy and wait for it to heal? Doesn t hurt when walking or standing but when I move just so it really hurts! Doctor: Dear patient It seems you have pulled adductor muscle of thigh. This will heal with time and till that time around 3 to 4 weeks you need to avoid activities which leads to pain like separation of legs apart. Do ice fomentation and take tab Ibuprofen 400 mg thrice a day for initial 5 days. If not helping please consult orthopedic surgeon nearby your area. Take Care Regards, Dr. Jayesh Vaza, Orthopedic Surgeon"
},
{
"id": 5562,
"tgt": "Trying to conceive. No periods, brownish spotting. Pregnancy test negative. Any thoughts?",
"src": "Patient: I just got off birth control last month and had my withdrawl bleed February 24th. My husband and i are trying to concieve. I am now expecting my period but have not gotten it. Yesterday and today i have been experiencing a very light brown discharge . A little light light cramping yesterday but non today. Got anxious 5 days ago and took a pregnancy test ... Negative. Thoughts? Doctor: Hi there ~ I do not think that you are pregnant, especially so if your pregnancy test is negative and you are having brownish spotting. Give yourself 2-3 months after stopping the birth control pill to start having regular periods and then you might have a chance at a normal pregnancy. I hope this helps relieve your anxiety. Take care and have a lovely day !"
},
{
"id": 98670,
"tgt": "How can mosquito-bite allergy, leaving dark spots on the skin, be treated?",
"src": "Patient: Doctor I have very bad problem my two hand &two leg are dark spot look pox type..I have mosquito allergy...My childhood time I have more bad spot...Then am treatment lot of medicine. But not effect ...now my age 31 .But same problem... I can't dress any good and short hand... All of people looking my hand....shame myself....lot of cash am waist in this treatment.::).Plz give any suggestions... Doctor: you have insect bite allergy and left over dark spots in the skin. The problem seems to be simple, you need to apply steroid cream like mometasone,hydroquinone treinoin cream in the night( you get a combo cream) .Treinoin cream is retinoid and you should not get pregnant when you are under this treatment.If worried about it you can use only mometazone cream.The marks should get lightened in 3 months of time n never allow mosquito bite again"
},
{
"id": 52198,
"tgt": "Suggest Ayurveda treatment for liver failure problem",
"src": "Patient: I am 52 year old, 5 feet 6 inch height, 79 kg, was suffering from liver failure problem, got treated for that Punarnava Ayuervedic hospital Udupi, and at Asian institute of Gastroentology Hyderabad, now Iam well, left drinking habit. May be because of heavy drinking and mental stress Iam now suffering from Pyeronies. Please let me know the Ayurvedic treatment for this problem. Doctor: Hello,Ayurveda may not be useful in this condition. Many of the Ayurvedic preparations contains heavy metals as their active content and might further worsen the conditions. Consult a gastroenterologist and he will direct you accordingly.Hope I have answered your query. Let me know if I can assist you further. Regards, Dr. Shinas Hussain, General & Family Physician"
},
{
"id": 48162,
"tgt": "Suggest remedy for kidney pain and unsteady stream while urinating",
"src": "Patient: My daughter was taken to ER with severe pain in the kidney area. While there she was told no stones and no infection. However, the UA RBC test shows 20-30 A. The ultra sound and CAT scan showed moderate right hydronephrosis and visualized proximal ureter is dilated. She also shows multiple phleboliths. They gave her some vicodin and an antibiotic and sent her home. That was last Thursday. She is still in severe pain and when she urinates it is not a steady stream......can you advise? Doctor: HelloThanks for query.Your daughter has severe pain in kidney area and ultrasound scanning and CT revealed hydronephrosis and dilatation of upper ureter .This is suggestive of presence of obstruction of the ureter in middle portion of ureter.In majority of cases it is due to the impacted stone in the ureter .This can be confirmed by doing radiological tests called Intra Venous Pyelography.(IVP).Consult Urologist for evaluation and get the I.V.P done under his supervision .You need to confirm that her creatinine levels are normal before going for I.V.P.Further treatment will be decide by him and will depend upon the results of test ,position of stone and degree of obstruction .Dr.Patil."
},
{
"id": 96737,
"tgt": "What causes pain on the abdomen and thighs after an accident?",
"src": "Patient: I was in a car accident yesterday. My car skidded on ice and hit a skip. I went to hospital and they said I was suffering from shock (had headache and nausea all day) and muscular pain (pain in back and shoulder). Today I've had pains in my abdomen (both sides) all day, along with pain in my lower back (I have had arthritis although no pain recently), pain in the backs of the thighs, hips, down side of leg and knees. Could this be a hernia. Doctor: hai.this pain can be due to the after effects of your injury due to the accidentheadache & nausea can be due to sudden variations in the bloodpressures mostly vasovagal effect.there could have been a blunt trauma & injury to the abdomen & your back,not probably a herniaplease go ahead with an ultrasound abdomen,x-ray lumbosacral spine, to rule out any injurytake care"
},
{
"id": 21041,
"tgt": "Suggest treatment for high BP,headache,blurred vision and nausea",
"src": "Patient: my blood pressure has been running high for the past 3-4 months, it used to run 120/80 average all my life (51yrs) today I checked it twice and the first was 160/101 pulse was 110 (pulse has always ran high all my life around 100) and the second reading was 164/104 pulse 96. The readings were about 2 hours apart. I laid down and slept about an hour and that usually helps. Symptoms that I have on and off are headaches, nausea, blurred vision and light headed or foggy, (sometimes just one or two symptoms and sometimes more)hard to explain. Thought it was just side effects of my meds but then tied it to my blood pressure. This is not a daily occurence but they do seem to happen more often. I am not on any bp medicine Doctor: Dear sirall the symptoms that seem to have are because of the raised blood pressure but the cause behind this must be evaluated and treated at the earliest for which you need to attend a cardiology clinic as soon as possible"
},
{
"id": 126922,
"tgt": "How do you fix a locked knee?",
"src": "Patient: I can t straighten my knee all the way and I cannot bear my entire weight on my knee. Also, the knee catches at the end of my stride and I cannot bear any weight on the heel strike of my stride when walking. The doctor referred me to PT but the first available appointment was 6 weeks out. I am using ibuprofen and using crutches to keep weight off it. What else can I do? Doctor: Hi, Consult an orthopedician and get an MRI scan done to assess the status of your knee. You may require an arthroscopic correction of needed. Hope I have answered your query. Let me know if I can assist you further."
},
{
"id": 93147,
"tgt": "Bruised pubic pain. Bacterial infection? Ultrasound said femoral hernia. Pain due to?",
"src": "Patient: I m a 31 year old female who almost a month ago started to feel like I bruised my pubic bone right side only. About 1.5 weeks ago I notice a lump/ swelling on that exact spot. Swelling spread all up my right abdomen to my hip bone and around my back, again right side only, and swelling is visible to all that view it. It hurts to sit, stand and walk I feel best laying down. First ultrasound said femoral hernia so I went to a specialized hernia hospital where both a surgeon as well as cheif of surgeon said I was misdiagnosed. They told me to go back to my dr and get antibotics as they said it d probably a bacterial infection . Went to dr and said she was not comfortable giving rx when not certain what diagnoses is. She sent me to er where urine and blood work came back normal, (not pregnant). An ultrasound was also completed at the hospital which came back clear, appendix was not visualized. Dr rx anti inflammatory and said I must have pulled my muscles. I m in pain and I am worried it may be something serious that is not being dealt with. Please advise as to what is going on as I want to get back to living my life. Thank you kindly, Christina. Doctor: hi.. swelling in the grion could be due to lymphnode enlargement or hernia.. so take one antibiotics and antiinflammatory if it is due to lymph node it will reduce.. see your physician for this.. femoral hernia should be ruled out early because it may cause obstruction and strangulation.. see your surgeon immediately.. all the very best.."
},
{
"id": 79795,
"tgt": "What causes pain in chest and rib after coughing?",
"src": "Patient: About 3 weeks ago I got a cold or flu of some sort, there was lots of drainage and I think I may have had nodules on the back of my throat. It took about a week for my throat to stop hurting, but a phlegm-y cough has continued this entire time and my nose has stopped being runny just this past week. For over a month my hands have been shaking almost constantly, so about a week before the cough started, but that could possibly be due to anxiety. This last week I've been feeling dizzy and weak, and I've had very low energy ever since the shaking started. I'll also get random, fleeting, but INTENSE pains, in my chest. Related: about a year ago I broke a rib coughing when I had an illness similar to this one, that lasted about a month, but never went to a doctor for neither sickness nor injury until the sickness was gone and the rib was healed months later. Every once in a while I swear I can feel the two pieces popping against each other. Currently, just started about half an hour ago I'm experiencing steady, but not too intense pain where that rib is, and it began after a coughing bout. Doctor: thanks for asking your queryI completely understand your problemsurely cough can cause fractures in ribs, and pneumothorax alsobut if you r having pain in chest now also you should get a chest xray done and consult a pulmonologist who can see the chest xray and guide you best.as root of all this hastle is cough so u need to take some medications for controlling your cough ,and for that cause needs to be sorted out,and for this you should get a sputum pyogenic culture and other blood investigations done .if cough is severe along with yellow phlegm a course or antibiotics such as cefexime should be taken for a week thanks /regardsfeel free to ask more questionsmay gos bless you with good health"
},
{
"id": 47775,
"tgt": "What does this CT scan and sonogram indicate?",
"src": "Patient: My CT scan showed I had hydronephrosis and hydroureter on my right kidney but no stones. My doctor said I might have passed a stone but I didn t feel it. Two days later I had a sonogram of my kidney and it showed no stones and that my kidney went back to normal also my urine test came out cleared too. Can it be caused by something else and can it come and go? Doctor: Hi,Thanks for writing in.It is important to know the symptoms you had which made you consult your doctor. There are occasions when a kidney infection might cause hydronephrosis and hydroureter and this causes symptoms of fever and pain due to infection. Usually urine culture reports are indicative of an infection in these patients.Stones might also cause hydroureter and hydronephrosis after they have passed out and this is also one of the arguements put forth bu your doctor. This is to be suspected if you had severe pain along the sides of your abdomen going to the groin on the right side.Lastly it might be a congenital variation showing the swollen appearance of the kidney and proximal ureter. In this condition the other tests are normal and the patient usually does not have severe symptoms. Patient might be asymptomatic or have mild symptoms. This does not require any specific treatment. Please drink adequate amount of water. Please do not worry."
},
{
"id": 18246,
"tgt": "What causes foamy vomiting while on BP medication?",
"src": "Patient: I have had a cough since December. Sometimes it makes me throw up white foam. I started taking blood pressure medicine in December. I have tried two different kinds. I stopped taking them a week ago and have not noticed any difference in my coughing. My voice is horse and I do not sleep much. No amount of cough syrup or cough drops will make it quit. I sneeze some with the coughing and my eyes will run and my nose will run. Could this just be allergies? Doctor: Hello and Welcome to \u2018Ask A Doctor\u2019 service. I have reviewed your query and here is my advice. I would explain that your symptoms could be related to a sinus infection or allergic reaction. Coming to this point, I recommend consulting with your doctor and performing further tests: -a complete blood count -a chest X ray study and pulmonary function tests -a sinus X ray study -allergy tests. Consulting with an ENT specialist may be needed. Hope I have answered your query. Let me know if I can assist you further."
},
{
"id": 45733,
"tgt": "What does \"faint cortical calcification in kidney\" seen in CT scan mean?",
"src": "Patient: Prior CT examination demonstrated a tiny nonobstructing right renal calcification. Right kidney demonstrates a few tiny nonobstructing calcifications measuring up to approximately 7 mm. Lower pole small cortical cyst is seen measuring up to 11 mm. There is a 9 mm nonshadowing vein nodules associated with the right kidney lateral cortex. Although sonographic appearance suggest possible angiomyolipoma, this is not seen on CT examination. This could represent a faint cortical calcification. This is from a ct in October. What does it mean? I have not heard back. I have another test 2 weeks ago and I will send it to you\u2026.thanks Pam 2 weeks ago obular contour is seen in the inferior aspect of the< right kidney. This could represent a lobulation, although the lesion is not excluded. Please correlate with CT when appropriate. Doctor: Hi, The CT scan result is suggestive of small calculus (stone) that is not obstructing your ureter or draining system of your kidney. Angiomyolipoma is not likely. If you have symptoms like abdominal pain, you can plan for the removal either by ESWL or endoscopy, otherwise no treatment is required. Hope I have answered your query. Let me know if I can assist you further."
},
{
"id": 28858,
"tgt": "How can UTI be treated while on a nephrostomy tube?",
"src": "Patient: what is the preferred antibiotic for UTI in patient with Nephrostomy tubes and 1 atrophied kidney,: Cipro or Sulfamethoxazole tmp ds t ? sorry that the computer messed up please send your question again i was answering it when error must have occurred He is on hospice He has been on Cipro for previous UTI infections. He has been unable to recover from the Whipple Surgery of 9/26 17 He also has prostate cancer. He already has trouble with diarrhea and sometimes vomiting due to the Whipple having his gastro intestinal system in an altered state ( to say the least ). Also, I give him his meds and change nephrostomy bandages and I am highly allergic to Sulfa drugs Doctor: Hello,The combination of Sulfa and TMP DS is the first choice for it if the area in which you are living has not developed resistance to it. Instead of Cipro, use Norfloxacin as the second choice of drug.However, for both, you have to consult a physician as these are prescription only medicines. For diarrhea and vomiting, start Oral rehydration solution (ORS - WHO approved).Hope I have answered your query. Let me know if I can assist you further.Regards,Dr. Hiren Hirpara"
},
{
"id": 71624,
"tgt": "What does this chest X-ray test result for recurrent fever post VSD closure surgery indicate?",
"src": "Patient: Hi ,m poonam .my baby is nw 8 months old.3 mnths back her vsd closure surgery was done by after that also she was getting fever and cough.after 1..5mnths of that surgery doctor came to know there was a vegetation just above the septal valve.they had given antibiotics for that bt baby didn t respond to that so they decided to do surgery again for removal of that ve citation they removed that also .operation was successful. Bt still baby was getting fever and cough continuously .again they started antibiotics therapy after 15 days her condition was improved somehow.all vitals were normal.but one day morning suddenly she became unconscious.doctors kept her in ventilator and doctor r telling her lungs function is not so good .bilateral paracardiac haziiness is present. Her pulmonary pressure is almost 70 mmhg so they can t say she will be fine .so sir I want to know what is ur opinion in this situation Doctor: Hello, Sudden rise in pulmonary artery pressure with the bilateral lung haziness, sudden unconsciousness are more suggestive of pulmonary embolism (PE). It is quite possible that her vegetations might have traveled in pulmonary vessels and cause a pulmonary embolism. So get done CT pulmonary angiography for PE. Hope I have answered your query. Let me know if I can assist you further. Regards,Dr. Kaushal Bhavsar"
},
{
"id": 115115,
"tgt": "Suggest treatment for severe dehydration",
"src": "Patient: I was admitted to the hospital overnight for severe dehydration from my blood pressure pills. I collapsed at work and was taken by ambulance. They did numerous tests and I have followed up with my physician. She took me off the medication. While I am told everything is well, I still feel glittery and nervous although she assures me it won t happen again. I can t shake my uncertainty and am anxious about returning to work with my energy level low. I haven t had an appetite for days although she says that will also return. How long before I feel well again. I had no idea I was so ill. Doctor: Hi! you have to be more conscious about health; dehydration is easily preventable!How does it take place? Well, there are two components in our body/body fluids including blood: solid and liquid (most of it contains water/salt solution!).Whenever this fluid portion is reduced in our body, it is dehydration and causes fall in BP, weakness, irritability, loss of appetite, etc...And this happens in cases of severe sweating, vomiting, loose stool, high urine output, hormonal problems, brain tumors etc...Therefore you need lots of tests if your dehydration is recurrent and plan a unique lifestyle with your physician accordingly.regards,"
},
{
"id": 48274,
"tgt": "Will taking too much of medicines damage kidney?",
"src": "Patient: i am 35 year old, have high blood presure, and was control with medication, creatine level raised from 1.4 to 2.5, doctor have started me with to many medicine, febustat, amlog, lasix, rosuvas, moxivas. taking all these medication, do it will damage the kidney more? Doctor: Considering your history,that sounds a logical prescription.In my opinion there should not be any concern with these medicines as for as kidney is concerned."
},
{
"id": 125358,
"tgt": "Suggest treatment for swollen painful ankle after an injury",
"src": "Patient: Hello! I fell down 2 stairs carrying a box and landed wrong on my foot. Now I have a burning sensation at the outside top and outside of ankle and it has a dull ache and if I move it much a sharp pain. What do u think I have done? YYYY@YYYY Doctor: Hi, Most probably it is due to sprain or contusion. As a first line management you can take analgesics like Paracetamol or Aceclofenac for pain relief. You can also apply ice packs for faster recovery. If symptoms persists better to consult an orthopedics and get evaluated. Hope I have answered your query. Let me know if I can assist you further. Regards, Dr. Shinas Hussain, General & Family Physician"
},
{
"id": 153832,
"tgt": "Treatment for neuroendocrine carcinoma",
"src": "Patient: Hi My father just got diagnosed with the Neuroendocrine Carcinoma with mets into liver. his age is 73, colonoscopy is clear, CT with contrast showed no tumors anywhere else apart from liver, CEA is normal, Hep B and Hep C are also negative, Gamma GT is 100% more than normal, Prostrate is 40G, fasting blood sugar is upper borderline since last two months with no history of diabetes before that, hypertensive since 20 years. had an eye operation for cateract and lense two years ago. Doctor: Thanks for your question on Health Care Magic. I can understand your father's situation and problem. Since he is having liver metastasis, he is having stage 4 neuroendocrine tumour. It is inoperable tumour. So only palliative chemotherapy, radiotherapy, pain management and nutritional support are required. So better to consult oncologist and discuss all these and start appropriate treatment. Hope I have solved your query. Wishing good health to your father. Thanks."
},
{
"id": 211212,
"tgt": "Can Flunil, Clonil and Ciplar be taken for being afraid and phobic for everything?",
"src": "Patient: Hello Doctor,My brother in law was getting afraid of almost everything, a sort of OCD. Few days ago he was he went to Psychiatrist on which doc prescribed Depsert. Later on, he experienced certain side effects and again visited Dr., on that Doc suggested to continue it for a week. After continuing it, he is feeling nice and continuing his work properly. After taking the dose of medicines (doc prescribed for the first time), he again visited doc. This time doc prescribed him with Flunil - 10 mg, Clonil - 25 mg and Ciplar - 10 mg to take for long duration. Could you please suggest me should we continue them as I guess there will be problem due to continuous consumption of Flunil and Clonil? His condition gos like this - He speaks very slowly...seems like he thinks lot before speakingTakes more time for bathing and getting readyAs such he seems very normal but few days ago he told me that he is afraid of almost everythingThinks lot about many thinksNot able to concentrate on studiesKindly suggest what to do and whether to continue with medications prescribed by doc as mentioned above.Thanks and RegardsKailash Doctor: Dear kailash, Lets first discuss about diagnosis as your history is confusing. OCD- Repetitive intrusive thought leading to increased washing, checking, slowness in daily activities .Getting afraid always- might be symptoms of Gen anxiety Dis, Secondary to suspiciousness ( Delusion of persecution)Treament of OCD- clomipramin (clonil ) and fluexetin ( Flunil ). Both are good choices. Flunil can be prescribed upto 60mg and clomipramine-150 mg. If U gradually increase the dose , side effect chance is low. so it's better , you leave it to your psychiatrist regarding diagnosis and dosage of medication, but u focus on symptoms- remitting/relapse, A/E if any to drugs and report to your psychiatrist."
},
{
"id": 173038,
"tgt": "Suggest mild antibiotic to 2 year old with fever and sore throat",
"src": "Patient: antibiotic generationwhich generation antibiotic to be used in 2 years old for fever due to throat infection. please dont give hard antibiotics. also please explain the generation of antibiotics, what is it all about? why and how is it used? does generation of antibiotic has to do some thing with its intensity? which antibiotic is mild, I can use for my baby as I don't want to give him hard medicine and weak his body resistance.please explain in detail and help. Doctor: HiWelcome to the HCM I understand your concerns. Most of the throat infections with fever are caused by viruses which actually do not need any antibiotics at all. These are mostly self limiting illnesses in children and get improved with just symptomatic treatment over a period of 1 week.Sometimes, if they are associated with congestion of throat, enlarged lymph nodes, high grade fever, enlarged and inflamed tonsils, the bacterial infections are the cause. Streptococcus is the most common reason. It needs to be confirmed by a pediatrician, throat swab culture and a complete course of penicillins or other sensitive antibiotics. I usually recommend single injection of Benzathine penicillin, or 10 days penicillin v oral or 5 day course of Azithromycin. Hopefully this will be helpful. For any further questions, you may contact us.Take care"
},
{
"id": 13181,
"tgt": "What causes rash on the left lower leg with bleeding?",
"src": "Patient: Hello Doctor,Rash on back of left lower leg with 3 constant spots that flare up at least once a day and bleedWhen scratched. I have a lot of broken veins and on my feet most of time.Has only appeared in last 4 months and not constant but visible.47 years of age.Diet normal. Blood test? Doctor: Hi, It seems to be dermatitis.I would recommend you to apply topical steroid and antibiotic creams like cortisone cream and neosporin ointment on the affected areas and take antihistaminic like tablet cetirizine depending on the intensity of itching. Hope I have answered your query. Let me know if I can assist you further. Regards, Dr. Asmeet Kaur Sawhney, Dermatologist"
},
{
"id": 224349,
"tgt": "Can an IUD cause pain after intercourse?",
"src": "Patient: Hello, I'm a 24-yr-old female, 5'10\", 170 lbs. I had an IUD incerted 2 yrs ago and recently I've been having discomfort/pain after sex and also experienced rectal pain. This morning it was really strong, but when I had a bowel movement it kind of went away....any advice? thanks in advance... Doctor: HIWell come to HCMYes such discomfort may be due to IUD, and this need to be ruled out, some times this may happened due to infection around the IUD, it is already couple of years gone now better to get the thing out, it is most advisable, hope this information helps, take care."
},
{
"id": 37594,
"tgt": "Can typhoid fever reoccur after 45 years?",
"src": "Patient: I contracted typhoid fever in 1969 and was hospitalized for 10 days. Back in March of this year, I began being sick and I thought I have not felt like this since 1969: headaches, fever fluctuations, diahrrea, abdominal cramps. Is it possible to have a recurring bout of this 45 years later. My lab test show Positive A results for Salmonella Typhi H Type A, Type b, Type C and Typhi O, Type VI. Doctor: welcome to hcm, you might be a long term carrier of the infection it is recommended to have a 28 day course of antibiotics and after that i want you check your stools another time to confirm all your infections have passed have a good hand wash after coming from toilet and avoid handling food items or preparing it until you are free from the infection hope this explainswith regardsDr.Amarnath"
},
{
"id": 134245,
"tgt": "Could the pain still be due to bruised muscles in upper rib cage?",
"src": "Patient: I was in the er over a month ago due to what I was told were bruised muscles in my right upper rib cage they did x rays ... Said it wasn t pluresy... Gave me a few scripts for the discomfort... But, Tuesday am I woke up and it feels bad.... Even worse... Could it still be from supposed bruised muscles? Doctor: hi,thank you for providing the brief history of you.let's being with the pain you are having in your chest area. since you may had a direct impact on the rib bones, it will give pain for some more time . Healing will take it own time as the rib bone is thin chip of bone.also, as x-ray has confirmed no fracture or pleurisy you shouldn't worry much.you should try performing regular breathing exercises - Deep breathing, coastal breathing, diaphragmatic breathing.note - when you do breathing exercises you should place the hand over the area affected as it will give support to the area to avoid any pain.you can repeat the same for as many times you feel to do it in a day.with the grace of God I wish you a good health.regards.for any doubts you can contact me directly.Jay Indravadan Patel"
},
{
"id": 38645,
"tgt": "Should I take tetanus vaccination after leg injury?",
"src": "Patient: I injured my leg a week ago; a 7-8 gash but not deep so I did not get stitches. It appears to be healing, no infection. I did not think to have a tetanus vaccination booster. Am I still in danger of developing tetanus? I believe that my last tetanus shot was about eight years ago. Doctor: HI, thanks for using healthcare magicTetanus immunisation can potentially cover a person for up to 10 years so if your last injection was 8 yrs ago then you would be covered.Additionally since the gash was a week ago, immunisation at this stage would not have been beneficial in terms of preventing tetanus. It is normally recommended within 48 hours if it is needed.I hope this helps"
},
{
"id": 182852,
"tgt": "Suggest treatment for an infection in the gum",
"src": "Patient: Hi. I have a question. On my upper gum near my right side, i have a small swollen spot, its red/purple and every morning when i touch it puss and blood comes out. I use scope and when i do it hurts so much. What is it and what can you can recommend me? Thank you in advance!!! Doctor: Dear User,Thanks for using health care magic.From the available description it appears that there is local infection or gingivitis. It can be treated with antibiotics. Empirically I prescribe Ofloxacin 200 mg and tinidazole 500 mg for five days. Since this is an prescription drug you need to see a dentist in person to get one.'Hope I have answered your query. If you have any further questions I will be happy to help\".Thanks"
},
{
"id": 157321,
"tgt": "Colon cancer. Face red and sweaty, pale lips. Was it breathing issue?",
"src": "Patient: My father died four days ago. Before we took him to ER, he was suffering colon cancer . I know he was in great pain at the time. I want to know why his face was red and sweaty but his nose and upper lips turn white (pale) and cold. Was it a breathing problem or something else? It was so hard to watch him without knowing what to do. - Estel Doctor: Sorry to hear about your fathers demise. Severe pain can cause redness and flushing of face. It is more obvious in fair skinned people. Circulatory disturbances cause pallor of extremities and make them cold. All this part of physiological process. I think you should stop thinking of all this come out of it. Life goes on and it is best to take life as it comes. Take care"
},
{
"id": 97840,
"tgt": "Have spermatorrhoea. What cure do I have? Why has my nervous system weakened?",
"src": "Patient: Respected Sir I am male age 24, i am suffering from spermatorrhoea from last 1 year and i want to cure this at the earliest. i am not doing any bad thing like hand practice from last 3 -4 years and also i have noticed that whenever i am on bike due to jerks always i feel the whole nervous system is vibrating and then i see sticky water like transparent urine thats why my sexual nervous system is also weakened. please suggest me what to do, i have already taken homeopathy medicines, and some of ayurvedic medicines like chandraprabhavati, giloy ghanvati, neo capsules, himalaya cystone, and presently i am taking ashwagandha churna, dabur ashwagandharishta, and mehmadhurvati and purnachandraras, kindly suggest me right medicine to cure weakening of nerves and spermatorrhoea permanently without side effects please, my contact number is 0000 Doctor: Dear Palwal The first thing to note very clearly is that there is no weakening of your nerves. This is more related to your thoughts and mental confidence. Have faith in yourself and forget the disease, you will be cured in a day. If you are unable to kick away this fear then medication is needed. Here trivang is the ideal drug along with some brain tonics or anti anxiety herbs. Musli pak cap. bravobol and trivang may cure."
},
{
"id": 124201,
"tgt": "Is it serious if frequently having lower backpain and thigh joint pain?",
"src": "Patient: I frequently get back pains in the middle and lower part of my spine along with thigh joint pains,I usually travel more in bus and even walk more.I am interested in Defense Services,So,i don t want to get disqualified in my medical tests due to these reasons,Is it a serious trouble or is it curable ? Please help me out Doctor: Hello, As what I see is your lower back muscles aren't strong enough to hold the spine in position and due to this you have pain in the lower back as well the lower limbs. I would advise you to follow the strict exercise program under a guided physical therapist so that you regain the maximum strength in the lower back muscles and also lower limb muscles if you want to join the army services. As their training is much strenuous and demands more output from you than normal individuals. Hope I have answered your query. Let me know if I can assist you further. Regards, Jay Indravadan Patel, Physical Therapist or Physiotherapist"
},
{
"id": 94698,
"tgt": "Stomach fullness, acute stretching in gall bladder. Problem persist after having ganaton total tablet",
"src": "Patient: Hi Doctor, I am 35 Year Old Male. I am suffering from fullness in Stomach . It feels that there is accute stretching in the Gall Bladder part of my Stomach. Yesterday I visited Doctor and he suggested me Ganaton Total tablet. I had taken 2 pills, one yesterday night and one today morning but the problem still persists. What should I do now ? Please advise. Doctor: Clearly your symptoms are not stomach related. Possibilities are Gall Bladder infection, Liver and Pancreatic problems. See a General Surgeon and get a ultrasound abdomen done, at a good centre."
},
{
"id": 172140,
"tgt": "Suggest treatment for tonsil infection in a 3 years old child",
"src": "Patient: HI, my 3 year old son had a tonsil infection due to beta hemolytyc strep, after a week of the infection a found he had and has (after three months after infection) a white spot (not pus) on one of his tonsils and vissible veins on this tonsil. What could this be? Doctor: hello i am dr.vishwa theja MD paediatrics hyderabad strept infection is one of the most common causes of severe sore throat. This bacterial infection will frequently show itself as white spots on the tonsils and throatYou will notice pain when swallowing, or a scratchy feeling in the throat. The lymph nodes in the neck and the tonsils will become swollen as they attempt to rid the body of this infection. High fevers are also common when battling strep throat.TREATMENT : home remedies may include gargling of worm salty water , taking worm liquids without caffeine ,avoid irritants and try to create some comfortable atmosphere by using humidifiers so that body can combat the infection well with medical treatment you can go antibiotics they respond better to oral penicillins also surgical treatment not usually required can be done in case og recurrent tonsillitis or when they cause difficuly in breathing .... FEEL FREE TO ASK ME ANY OTHER QUSTIONS"
},
{
"id": 161515,
"tgt": "Suggest remedy for pain and swelling in testicle",
"src": "Patient: my son is 6 years old, and he just complained of pain in his scrotum. I looked and noticed his right testicle appears very swollen. He said that he fell and hit the area. I dont know if this is just swelling from hitting it on something, or if I need to get him checked out for a hernia or something else ? Doctor: Hello, I feel this is a medical emergency. The possibilities are that it could be a testicular torsion or that it could be epididymitis. Please approach the nearest Emergency Room immediately. Hope I have answered your query. Let me know if I can assist you further. Regards, Dr. Sumanth Amperayani, Pediatrician, Pulmonology"
},
{
"id": 117200,
"tgt": "What causes a bruise on lower abdomen?",
"src": "Patient: I have a bruise on my..... well, about an inch above my pants. My kids noticed it a couple of days ago, and I just figured it s because my pants are always to tight and I slouch most of the time, but I just wanna know what other concerns there might be? It seems to have a texture, which leads me to believe it s from my pants. Stress (I m under a lot), exhaustion, lack of sleep, being overweight, food allergies? Doctor: Hi,Thank you for your query. I can understand your concerns.Purple striae-is commonly seen over abdomen in Cushing's syndrome & excessive steroid treatment due to easy bruisability as a result of loss of perivascular supporting tissue from protein catabolism.Bruising can also occur on lower abdomen with minor trauma like tight pants,if you have underlying bleeding or coagulation disorder. Simple obesity is also associated with the incidence of stretch marks on the abdominal wall giving similar appearance of bruising.Regards Dr. T.K. Biswas M.D.Mumbai"
},
{
"id": 33723,
"tgt": "How can MRSA infection be avoided here?",
"src": "Patient: hi i am 5 months along with a little boy and my husband has 3 kids from his exwife one of his twins got mersa and they will be comming to stay with us for the summer. yes they will be gone befor the baby is born but right back at christmas for a week and i dont want to be mean to her but i do not want my kid getting mersa or my self or any one eles pls help me with ho do i live with a teen that has mersa she is 12 and i am not sure what i can do and what i cant do... Doctor: Hi i did review your concern But if the MRSA infection has been adequately treated there is no need to worry about. MRSA or for any other staphylococcus stays in nose as a carrier in some patients and the best way to avoid transmiission is frequent hand washing with soap and water or alcohol based sanitizers after handling or touching the teen. Keeping yourself clean and eating a healthy diet always will always go long way in preventing an infection of any sort.I hope this helpsLet me know if you have any more questions or concerns.Wish you all the best."
},
{
"id": 87300,
"tgt": "What causes abdominal pain with a normal sonography?",
"src": "Patient: i am male and i have lower right abdominal pain continuesly since last 1 - 1.5 yrs..i drink 3-4 times in a week 90ml - 135ml alchol each time approx..i has visited to doctor 7-8 months ago he gave me some medicines for a week with albansazole .. and then changed for next week.mean while i got sonography done of the abdomen , there where no symptoms of kidney stone or appendix.and doctor told me it could be gastric or muscular pain .. and will get well ..but m still having the pain..can you please suggest what could be the problem.. Doctor: HelloRight sided abdominal pain may be due to many reasons like hepatitis,acidity,musculoskeletal reasons etc.Ultrasound of abdomen may be normal in these cases.You need clinical correlation and investigations like routine hemogram,liver function test,renal function test,urine RE/ME.Upper GI endoscopy can be done if needed.Proper treatment depend upon findings.You should avoid alcohol completely.It may compromise your liver and it may be the reason for abdominal pain.Get well soon.Take CareDr.Indu Bhushan"
},
{
"id": 77210,
"tgt": "Suggest remedy for cough with yellow phlegm",
"src": "Patient: I have been coughing up yellow phlegm for about 3 weeks with no other symptoms other than voice hoarseness during the first week. For the past few days though my phlegm is still yellow but occassionally has a few tiny gray flecks in it. Should I be concerned? And no, I am not a smoker and I live in a smoke-free home out in the country where pollution is low. Doctor: Thanks for your question on Healthcare Magic. I can understand your concern. Yes, you should definitely concern about these symptoms. You are having chronic cough (cough for more than 2 weeks) with yellow phlegm. This suggest possibilities of bronchitis or lung infection more. So better to consult pulmonologist and get done 1. Clinical examination of respiratory system 2. Chest x ray to rule out lung infection. 3. PFT (pulmonary function test) (to rule out bronchitis). You may need antibiotic, inhaled bronchodilator and inhaled corticosteroids (ICS). Don't worry, with proper treatment, you will be alright. Hope I have solved your query. I will be happy to help you further. Wish you good health. Thanks."
},
{
"id": 120477,
"tgt": "What does growth protruding on inside of inner thigh indicate?",
"src": "Patient: On the inside of my right inner thigh, I have a growth protruding out about an inch. All in all, it s the same shape as a clamshell, maybe two inches long and about an inch deep. It does not hurt, it does not itch but the odor leaves something to be desired. All along the edges of it, are white. And it bleeds alot. Doctor: Hello,I read carefully your query and understand your concern. The symptoms can be related to a folliculitis which is an infection of the follicle. I suggest using warm compresses for local application. I also suggest using anti inflammatory medications such as Acetaminophen to relieve the inflammation. I recommend to cover it with an antibiotic cream such as Neosporin cream. Hope my answer was helpful.If you have further queries feel free to contact me again.Kind regards! Dr.Dorina Gurabardhi General &Family Physician"
},
{
"id": 199061,
"tgt": "What causes bleeding from penis?",
"src": "Patient: I am a 44 year old male, 6 -1 ,190#, diagnose with Spondylitis Ankilosante and taking enbrel 50mg twice a week and cataphlan twice a day. I started 10 days ago with two doses of enbrel since the inflammation persists after 3 months with one weekly dose. My actual problem is that since two days ago I wake up in the morning with blood in my underwear. Its like a 2 inch diameter area. I made an urinalysis and it is ok just with 4-6 red blood cells and during the day I can t see blood in my urine when I go to the bathroom.I have an appointment on thursday with the urologist but this has me in a lot of stress. Do you have any idea? Doctor: Hello and .You should not worry about the it, but blood in urine is not normal.You're doing the right thing to get a check-up with a Urologist. If you go with certain test reports, he will be able to start treatment. The tests to be done are urine culture,blood sugar,s.creatinine,PSA and a Ultrasound scan of abdomen.Drink 10-12 glasses liquids daily."
},
{
"id": 108645,
"tgt": "Suggest treatment for degenerative disc disease",
"src": "Patient: My daughter is 15yrs old and was involved in a severe car accident in december 2008...she was a cheerleader in 6th and 7th grade and 9th grade!! now almost 2 yrs later she has extreme back pain and has been diagnosed with degenerative disc disease...is this normal for a 15yr to get this??? Doctor: Post trauma this is possible with spine injury. Get an MRI of spine done .This age is not common for degenerative diseases but trauma can lead to damage of disc resulting in fibrosisof annular ring with prolapse of disc.It certainly requires treatment by spine specialist"
},
{
"id": 206536,
"tgt": "Suggest treatment for agoraphobia",
"src": "Patient: Hey im really worried about my girlfriend, she's really scared of more or less everything, mostly the outside world, she'll lay in bed all day and will find any excuse not to go outside. She won't even work because she's scared of meeting new people and being in big crowds. She has pains in her chest and shakes alot. She wen't to the doctors because at one point she was passing out because of it the doctors took some blood test and said nothing is wrong. Iv'e tried making her see the doctor but she's even scared to do that. The only way she can go outside is with me she say's she feels safe around me. And we can go out in busy places and i notice shes very clingy when we care like wont let go. She can't even leave a window open because she thinks someone can get inside her house. Im getting worried because I can see it get worse and I want her to make more out of her life. Please help me what do i do ! Doctor: Hello She is having symptoms of anxiety like pains in chest, tremors etc. She is not able to move out of home alone. She has fear in crowds and she is very scared with most of people. She is not able to control her anxiety and she is not even getting ready to consult a doctor. See from available history I can tell you that she is definitely having some form of anxiety disorder with symptoms of agoraphobia as well as claustrophobia. I would advise you to consult a Psychiatrist for her detailed treatment. Try to request her to make her ready to visit a Psychiatrist. You can visit psychiatrist on her behalf and then you can ask her to visit. Many medicines like low dose benzodiazepines as clonazepam can be given in her case to relieve symptoms and then when she become a little better, she can visit psychiatrist.Hope this helps you, thanks"
},
{
"id": 131271,
"tgt": "What causes swelling on the left foot and dizziness?",
"src": "Patient: I recently moved to Arkansas and believe i had a bad run in with chiggers, i have about 40+ bites on my left leg, which are about a week old, and about 8 which are a few days old. Im beginning to wonder if the ones on my ankle are chigger bites at all tho because my left foot is extremely swollen and im begining to feel dizzy. Is this normal? Doctor: These are serious symptoms and you should go immediatly to the hospital to take vaccines or antipoisons drugs according to the type of bite. Take cortisone injection and antiinflammatory and analgesics injections."
},
{
"id": 138879,
"tgt": "Suggest medicines for shoulder pain",
"src": "Patient: I have pain in my left shoulder bone almost month ago,I can do my task house chores but I feel less energy and bit hurt so..at this moment I take medicine nidol And famotedine.can u help me some treatment aid like exercise or more medicine please.thanks Doctor: Hi,Thanks for your query.Although it isn't possible to diagnose your condition without an examination and a few tests, your history is suggestive of Adhesive capsulitis(Frozen shoulder) or Calcific tendonitis.These conditions are easily managed by regular physiotherapy ( Range of motion exercices,stretching and strengthening exercises). Over-the-counter pain relievers, such as aspirin and ibuprofen (Advil, Motrin), can help reduce pain and inflammation associated with this condition. If these pain relievers not work then You can take voltarin after prescription of your doctor.I would suggest getting this evaluated by an orthopedician nearby for an accurate diagnosis and appropriate management. Your doctor after diagnosing the condition might refer to you a physical therapist.A physical therapist can teach you stretching exercises to help maintain as much mobility in your shoulder as possibleI do hope that you have found something helpful and I will be glad to answer any further query.Take care"
},
{
"id": 41770,
"tgt": "Suggest remedy for infertility problem",
"src": "Patient: Dear Madam, My names is GP, we have married past 3 yrs before, starting my marriage time i have irragular periods, after getting treatment it was resolved, but still now i could not able to get baby, my self and my husband was taken so may tests final results was found nagative, so please help and support me, am was there in near by chennai only madam, advance thanks for your reply Doctor: Hi welcome to healthcaremagic.I have gone through your question.Your and your husband reports are normal and still not conceiving is \"unexplained infertlity\".I would advise to do try naturally , do intercourse in fertile period on alternate day. Try this for 4/5 cycle.Then consult gynecologist if dont get success, and there are options like IUI (intrauterine insemination ), IVF invitro fertilization.Hope i answered your question.Would be happy to help you further.Take care."
},
{
"id": 208376,
"tgt": "Suggest remedy for tobacco addiction",
"src": "Patient: Dear Doctor i am in the habbit of using this hans chaap khaini which i use to keep in betwwen by lips .i am using this for the past 10 years. I am not able to quit this habbit. now i feel that i have lost some of the skin tissues ,,will it cause a mouth cancer,,,i feel whenever i use this i feel thirst very often that my mouth & throat dries immediately after spitting this..pls help Doctor: Hello, thanks for the query & let me congratulate you for the resolve to quit tobacco. First thing you need to understand is tobacco is one of the EASIEST & one of the MOST DIFFICULT ADDICTION TO GIVE UP AT THE SAME TIME!! Its easy because quitting tobacco does not cause any significant side effect apart from some craving & irritability. Difficult as we tend to continue tobacco thinking lets have one more before we quit. One way of quitting tobacco is decide a quit date (usually 2 weeks from today) & start using nicotine replacement therapy (available as Nicotine 2 & 4mg chewing gums & pastilles) These are park & chew type gums similar to your khanini use & 1 chewing-gum can last 2-3 hours. Gradually replace khaini with these & you shall get used to them. Additionally you can visit a Psychiatrist & get yourself Tablet Bupropion or Amitryptiline which reduce the craving for tobacco along with these chew-gums during this 2 week quit date period. You will also get tobacco quitting counselling help from Psychiatrist. Do get back to me if you need further help. Good Luck"
},
{
"id": 175319,
"tgt": "How to treat high fever in a toddler, antibiotics offering little relief?",
"src": "Patient: hi ! My 2yrs baby is suffering with high fever around 38.4/5 sometimes it reaches to 39. I already giving adol n now antibiotic ceclor too. but ths is happening every now n then i m really worried n now his head also is very hot wat to do ?I m really worried he is also havin nasal congestion. Doctor: Hi...during fever - the heat will be more perceived at the scalp (head) and tummy part because blood supply is more in those part of the skin. 95% of the kids illnesses are viral and do not require antibiotics. Antibiotics are not required in viral illnesses and they will not help too. Viral illness is self limiting. Fever of few days without any localizing signs could as well a viral illness. Usually rather than fever, what is more important is the activity of the child, in between 2 fever episodes on the same day. If the kid is active and playing around when there is no fever, it is probably viral illness and it doesn't require antibiotics at all. Once viral fever comes it will there for 4-7 days. So do not worry about duration if the kid is active.Paracetamol can be given in the dose of 15mg/kg/dose (maximum ceiling dose of 500mg) every 4-6th hourly that too only if fever is more than 100F. I suggest not using combination medicines for fever, especially with Paracetamol.Regards - Dr. Sumanth"
},
{
"id": 99566,
"tgt": "Is nose surgery needed for constant breathing problem and heavy head?",
"src": "Patient: I HAVE BREATHING PROBLEM SINCE LAST 7 YEARS,I RECEIVED BLOW IN MY NOSE 7 YRS AGO,SINCE THEN I CAN'T BREATHE PROPERLY ,SINCE LAST 5 YEARS MY WEIGHT IS CONSTANT ,MY HEAD REMAINS HEAVY,I FIND DIFFICULT TO DIGEST FOOD,I CAN'T SLEEP PROPERLY,COULD NOT STUDY PROPERLY ,I HAVE TO UNDERGO NOSE SURGERY IN UR HOSPITAL ,I WANT TO KNOW WHAT MAY BE THE TOTAL EXPENSE OCCUR FOR NOSE SURGERY N HOW LONG I HAVE TO STAY IN HOSPITAL?MY AGE IS 24 YEARS,HEIGHT 5FIT 10 INCH.WEIGHT 63GS, Doctor: Hi,Welcome to healthcare magic.I had gone through your question.I can understand your concern.Secondary to trauma you could have developed deviated nasal septum. I recommend you the following.1) A through ENT examination.2) Surgical intervention if required.3) Oxymetazoline nasal drops. Dont use this drops for more than 6 days continuously.4) The septal surgery is a day care procedure. The cost and methods wary from place to place.You will be fine.Hope I have answered your question. If you have any questions, I am happy to help.Wish you a speedy recovery and good health.Dr.Paparaja S MBBS, MSConsultant ENT Surgeon."
},
{
"id": 152141,
"tgt": "My son is having Epilepsy attacks. corrective action during attack while at home",
"src": "Patient: My son is having Epilepsy attacks from last 1 and half year. In case of attack, it does not stop at home and we have to rush to Hospital for giving him Diegepam Dose to stop the attack. My Query is For corrective action during attack , what can be done at home ,I mean any substitute of Injection that can be given at home. Thanks Doctor: whats ur sons age .... all u can & shud do is whenever fit occurs let it take its course for that attack... u just turn him sideways or turn him over on his face...preferably on a level surace /ground ....donot try to put anything in mouth for opening it.....let the treatment part b at the discretion of doc"
},
{
"id": 109592,
"tgt": "What causes back pain and left side chest pain?",
"src": "Patient: Hi , I have upper back pain (around shoulder blades) with left side chest pain (also when taking deep breath etc) and both back and chest hurt when using left arm to pull things eg like rowing at the GYM. Just started a couple of weeks ago, just annoying at first but is getting worse. Do you think I have pulled a muscle somewhere? Doctor: Hello. Thanks for writing to us. The pain that you have is likely to be a muscular pain. You need to take rest for a few weeks and do a hot fomentation in the area. I hope this information has been both informative and helpful for you. Regards, Dr. Praveen Tayal . For future query, you can directly approach me through my profile URL http://bit.ly/Dr-Praveen-Tayal drtayal72@gmail.com"
},
{
"id": 21431,
"tgt": "Is it necessary to do angiogram for high BP?",
"src": "Patient: Hi , my husband is 36 years old. He recently went for TMT and ECO test.the ECo test was normal , but the TMT test showed slight variation. he is currently being treated for High BP The doctor has advised for an Angio. Pls advisej is it necessary. His Cholestrol levels are being checked The report is not out yet. Thanks & Regards VM Doctor: Hi There After reading the medical details provided by you of your husband I understand your concern and I would like to tell you that your doctor has advised him to go for Angiography due to the variations in his TMT results not due to high BP.Yes it's important to go for Angiography to make sure there is no blockage developing inside the heart vessels because the variations in the TMT results are suggestive of that.I wish him good health"
},
{
"id": 215254,
"tgt": "Suggest remedy for pain due to sciatica",
"src": "Patient: Hi my name is Tracey and I suffer from sciatica, what can I do to make the pain go away without taking Ibprofen, since I have been taking 1000 to 2000 mg a day for 2 months Im afraid that my stomach is having a reaction because I constantly feel like I am going to throw up Doctor: Hi, Okay, cannot say in your particular case, but generally, that is a high amount of Ibuprofen and has a considerable risk of stomach injury. Sometimes, even fatal ulcers won't have even that much of a warning. I haven't found surgery to work on pain, muscle relaxers, nerve dampeners and of course opiates are likely to work on sciatica pain. Hope I have answered your query. Let me know if I can assist you further. Regards, Dr. Sameen Bin Naeem, General & Family Physician"
},
{
"id": 12207,
"tgt": "A 8 years old child has 3 cm circular white spot on his chest, please suggest me how to cure this ?",
"src": "Patient: A boy child, aged 8 years, has a 3 cm circular white spot on his chest. Is it possible to stop Vitiligo from spreading beyond this area ? What are the chances of success in arresting the spread of this condition ? Doctor: As such treatment of the vitiligo is difficult and the success rate is very low. since there is only one patch that to on chest, which is usually covered area. local steroid creams can be used to restrict the expansion of the lesion. there is home phototherapy using UVB lights that can be tried. if there is a chance, as the skin lesion is small, 3 cm, surgically the white spot can be removed as the spot is on the chest with a minimal scar. my vitiligo spot was 2cms and 3 cms on the knuckles, i got it removed and now even the scar is not visible, i think surgical removal is the right way to treat if the option is possible to prevent the spread!"
},
{
"id": 64382,
"tgt": "Suggest remedy for pain and movable lump in the back of neck",
"src": "Patient: am 5 ft 3 weigh 9 stone in both sides of family there is heart disease and cancer. for about a year now i have had a terrrible pain in the back of my neck right in the middle there is kinda like a movable lump like cartilage or something which is sore my head feel heavy all the time and im tired all the time i have a pretty happy life im a stay at home mum to my three kids but this is getting me down its almost like pressure to back of my head and neck Doctor: HI,Good Evening.Thanks for your query.I studied the query in-depth.I understand your concerns.With whatever facts told in your query,I would try to give most probable causes to help you to reduce your worry.-By your description,its mostly of -Sebaceous cyst on the back of neck with little infection and inflammation-may be by trauma or handling.-I would advise to rule out the Cervical spondylitis and hypertension with Xry Cervical-Spine and physicians opinion, as a cause of the pressure to back of your head and neck.-The infected sebaceous cyst could be a cause for the head and neck pressure.-Excision Biopsy after antibiotic / NSAID cover would relieve you with this movable lump.-You need not worry of the outcome as this-tumour is non-cancerous.-Hope this would help to regain the lost-health fast.-Wishing you early recovery and good health.Wellcome again."
},
{
"id": 131763,
"tgt": "What causes swelling in ankles?",
"src": "Patient: I am a 55yr old female and only recently (6mnths) started getting swollen ankles (no pain) and tinkling in my hands and fingers at different times. I have always had poor circulation and cold hands and very occasionally my one index finger would go numb and tingly (this was rare). I am concerned. Doctor: HiSwelling over both ankles without pain may be non joint problem.It may be due to water retention or other causes.Consult physician who may get serum creatinine, GFR,serum proteins,sodium,pottassium checked for possible causes.Medication can be diuretics or diet modificationsBest wishes"
},
{
"id": 99023,
"tgt": "Can allergy to milk cause mouth ulcer?",
"src": "Patient: i am 43 years old, male and i hav been suffering from mouth ulser and i start avoiding some food and as soon as i stop drinking milk the ulser start to heal now after three month i start to drink milk again and it came back. dose this mean that am allergic to milk, if yes what things to avoide and how i can replace the milk with something equivalent. Doctor: Milk is among one of the food which can cause allergy. With your history it seems that you are allergic to it. But mouth ulcers commonly occur due to acidity in stomach, vitamin B folic acid deficiency or injuries. My advice is you take one antacid (Cap Omeprazole) once daily and VItamin B and Folic acid supplement twice daily for 10 days. This will solve your problem.You can replace milk by fruits juices, banana."
},
{
"id": 45079,
"tgt": "Can I use Primolut n to treat boost my chances of getting pregnant?",
"src": "Patient: Can I use Primolut n to treat boost my chances of getting pregnant?? Doctor: Welcome to Healthcare Magic Priolut-N is only for prolonging or delaying periods. Has nothing to do with pregnancy though you could have increased bleeding after it's withdrawal and possibly have regular cycles later and more chance to conceive."
},
{
"id": 105464,
"tgt": "Runny nose, excessive sneezing, headache, pain in hands, feet. Reason?",
"src": "Patient: Hi For the past few weeks I have been experiencing classic allergy symptoms - runny nose , excessive sneezing , headache and temporary relief in the mornings. I am told that it could be Saharan Dust in the upper atmosphere. In addition to these symptoms I am also experiencing pains in my hands and feet. In the hands it feels like my soft tissue is inflamed and in my feet it feels like my veins are inflamed. Could these also be a result of my allergies. The pains are never always in the same spot. They seems to move up and down. Doctor: Dear Narishar, Greetings, Sorry for replying as this was not posted in Ayurveda section which is my expertise.But i think Ayurveda has a very promising results rather i will say it has cure and no recurrence.The rasayan drugs of Ayurveda has the capacity of building the immunity so resulting in no recurrence of any disease. I have cured many such patients with very good success.Even these Ayurveda medicines have no side effects."
},
{
"id": 85793,
"tgt": "Should I go to the emergency room for dizziness and jaw pain after Tramadol overdose?",
"src": "Patient: I just started new medication called tramadol 50mg and i forgot to take it yesterday so today i decided to take two and now i feel dizzy, lightheaded , numb , my head feels on fire and my jaw hurts to chew and swallow should i go to the emergency room? Doctor: Hello, Your symptoms seem to be related to Tramadol overdose. I suggest to go at the emergency room to take proper treatment. Hope my answer was helpful. If you have further queries feel free to contact me again. Regards, Dr. Dorina Gurabardhi, General & Family Physician"
},
{
"id": 35170,
"tgt": "Can the bumps and rashes on the genitals be due to virus?",
"src": "Patient: Hi I have genital herpes, caught it quite recently, so far only red rash in genital area. I also have small red bumps on some parts of my body. Are they a sympton of the virus as well ? I also have muscle aichs especially in my arm is this part of the virus as well.rgds M Doctor: Hello dear,Thank you for your contact to health care magic.I read and understand your concern. I am Dr Arun Tank answering your concern.Herpes is characterised by burning pain with vesicles on the geneital area. If you do not have same symptoms on the genitalia. I don't think that as per your description its not herpes.There are many reason for the bumps on the skin. Virus infection is one of the reason. But in skin conditions we can't identify based on the descriptions only. Visual examination is utmost important in the diagnosis part.I advice you to visit the skin specialist to diagnose yourself and treatment according to that.You can take acyclovir once your herpes is confirm. Other drugs you should take it is cetrizine, gabapentin, vitamin B complex. Please take the above medication under your doctors guidance.Please maintain good skin hygiene over the infection area this will equally helpful in the treatment.I will be happy to answer your further concern on bit.ly/DrArun.Thank you,Dr Arun TankInfectious diseases specialist,HCM"
},
{
"id": 5670,
"tgt": "Trying to concieve with IVF treatment. Experiencing pregnancy symptoms. Pregnant?",
"src": "Patient: Hi I am 34 and have been trying to concieve just over a year in that time I had a miscarriage so took a break as my cycles went a bit wonky, this cycle I decide to try soy isoflavones cycle days 2-6 a dose of 80mg day2/3 120mg 3/4 and 180mg day 5, I was fine taking them and got a positive opk day 12 (I have a normal 28/29 day cycle now) had positive for 4 days, day 14 I woke really Ill, I was feeling sick before I even got out of bed, that day I was sick, had really bad headache all day, couldn t have bright light on in the evening, stomach was bloated anything touching it felt uncomfortable, had low abdominal, back and hip pain, and maybe it was feeling Ill buy I was very emotional, I didn t eat much but forced a bit down but I can t remember moving my Bowles that day and urine was very dark, had very strong positive on opk, do you think I could of had ohss, I ve not really seen it with soy isoflavones more so with ivf treatment? Doctor: Symptoms are suggestive of ohss,but can't be confirmed until you get a sonography done a few blood tests done,I would advice you to meet your doctor and follow his advice. It might be just stress related also but get a check up done once. Take care."
},
{
"id": 182401,
"tgt": "How can gums darkening and teeth bleeding be treated?",
"src": "Patient: Hi my gums have started to get darker on top and bottom last few mths my teeth last few weeks have bled abit while brushin i did get a tooth out at the top bk of my mouth bout 2 yr ago i was left with a hole in my gum now im getting worried incase there somethin wrong im usin cordisol mouth wash can u help me in any way as what mite be wrong im only 31 Doctor: Bleeding gums are initial signs of gingivitis (Inflammation of the gums). In your case It could be due calculus deposits around the teeth(Tartar). You should see a dentist and get a scaling done. There is no local remedy or medication available over the counter."
},
{
"id": 13202,
"tgt": "What are the rashes on my mother's skin?",
"src": "Patient: my mom has a bad rash it started out looking like misquoto bites but since they have expanded into huge red circles with yellow spots in the middle i took her to the dr and he said he did not know what it was put put her on some kind of steroids but i see no change Doctor: Hi, As per your query, your mother has symptoms of rashes which seems to be that fact that she is being exposed and re-exposed to the allergen and this is the reason that the allergic reaction is persisting. It could be due to contact dermatitis. Need not to worry. I would suggest you take a proper course of antihistamine drug along with a short course of oral corticosteroids after proper prescription. Exfoliate your skin once in a week to remove the dead skin. Drink plenty of water. If the condition doesn't get better in a month then consult dermatologist again for proper diagnosis and to rule out any underlying condition. The doctor may prescribe isotretinoin, steroid ointment or antibiotics course. Hope I have answered your query. Let me know if I can assist you further. Regards, Dr. Harry Maheshwari, Dentist"
},
{
"id": 76944,
"tgt": "What causes high fevers/chills and sweats with back pain and diarrhoea?",
"src": "Patient: I'm a 32yr old female and for the last 2 wks having high fevers, chills and sweats. Both side pain but more on the right that feels like someone is squeezing. Hurts to breath in, also back pain. First was throwing up, then diarrhea, that is now just soft and very thin. Stomach makes sounds. This has never happened in my life. I'm being tested for the Hanta virus, c-diff and some other virus. I've been the the ER twice, but feel like going everyday because of the pain. They think it is just the flu so far. Please advise. Doctor: Hi thanks for contacting HCM...According to your history it seems acute viral flu ...No need for much worry.Symptomatic treatment done....For congestion relief Antihistaminic given.Steam inhalation also useful.Drink more water as you have diarrhea ...Maintain hydration.If stomach pain is there as you have mentioned omeprazole given....Avoud stress .As anxiety induced panicity can lead such presentation....Hope your concern solved.Dr.parth"
},
{
"id": 128441,
"tgt": "How to confirm a rotator cuff tear in the arm?",
"src": "Patient: I am being evalutated on Tuesday to confirm a rotator cuff tear.... My shoulder work, I can t put my arm behind my back or over my head on the left side. The pain is constant and intense. I have had 2 ER visits with pain control, 1 urgent care visit for intial fall and 1 ER visit for xrays. So the night before last the pain was so bad I only had 10 minutes of sleep ... Last night I ended up having the chance after my husband got home from work to go to the ER. They gave me 1mg of Diluadid and 325 mg of Norco to control pain. Today is the first time I have been holding between a 5-7 on the pain scale which normally a 10 plus constant. I have to be premedicated ahead of time for Tuesday s procedure as they are using saline/water due to allergy of contrast.... My question is, one of my prepatory drugs is benedryl... Can I take Benedryl with Norco? I also have an axiety medication that I will be taking as well which is Buspirone.... Are these 3 medications safe to take together the day of the appointment? Doctor: Dear patient Confirmation of rotator cuff can be done by Mri of the shoulder. Mri is very sensitive and specific for diagnosis of rotator cuff tear. All three drugs can be taken together for pain relief. Keep shoulder immobilised in shoulder brace till Mri is done. please consult shoulder surgeon with Mri report."
},
{
"id": 142598,
"tgt": "What is the cause of body tremors?",
"src": "Patient: In the middle, and the very bottom of my ribcage (generally it pulses here when you have hicups) it keeps shaking like crazy every few minutes. It doesnt hurt but the rest of my body is getting the shakes. Could this be from stress or something more serious Doctor: Hello!Thank you for asking on HCM!I understand your concern, and would explain that your symptoms seem to be related to anxiety. As a neurologist, I would exclude any possible neurological disorders. I would just recommend checking your thyroid hormone levels, because a thyroid gland dysfunction, can mimic all this clinical situation. If these tests result normal, I would recommend you to focus on your anxiety. In this regard, Yoga, meditation, a lot of physical activity and a healthy balanced diet would help. Psychotherapy would be the next step to consider. Hope you will find this answer helpful!Wishing all the best, Dr. Aida"
},
{
"id": 2977,
"tgt": "How to increase chances of conception after removing ovarian cysts?",
"src": "Patient: Hi, My wife has endometrial cyst(chocolate) in both the ovaries. It has been removed by the doctors. Though I know that this type of cysts generally have Fallopian tubes blockage problems. In the discharge report, the hospital has only given in writing that the blockage was not present, the reason of given in plain paper , they mention that , the medi-claim will not be settled , if this will be a part of all the reports. After the operation we are regularly in check with doctors and everything seems normal, but she is unable to conceive after 1-1/2 years of effort. The doctors are now suggesting 2 cycles of IUI. I have too much doubt on Fallopian tubes blockage. Please guide me. Doctor: Hello and welcome to \u2018Ask A Doctor\u2019 service. I have reviewed your query and here is my advice. You concerns about tubal blockage can be answered by a diagnostic test called hysterosalpingography which shows any blockages in the tubes. In this procedure dye will be injected in to the uterus and and x ray of pelvis will be taken to see whether drug spills out from the uterus. If it spills out it indicates tubes are patent and if not on the side where there is no spillage, there is a block of tube which will be evident on the x-ray film. If tubes are patent you check for ovulation whether it is occuring or not and growth of the follicles which is called follicular study. If ovulation occurs and tubes are patent chances of conception are good. Hope this is helpful to you. Let me know if I can assist you further.Regards,Dr. Sushma Dubbaka"
},
{
"id": 39334,
"tgt": "What causes relapse of helicobacter pylori infection?",
"src": "Patient: Hello I am extremely worried about my mother has had the Helicobater pylori infection twice, her symptoms finished and she was cured 2 weeks ago( or a month i am unsure after suffering) for 7 months. however this past week she has had similar symptoms to the bacterium (back and stomach pain internally and surrounding) and yesterday started to get very high blood pressure. it was 170/110 yesterday at nigh time and 180/108 today this has cause her to have the shiver, As I said I am very worried and was hoping that you could offer a diagnosis and possible help. Doctor: Dear Friend.Welcome to HCM. I am Dr Anshul Varshney. I understand your concern.H.pylori infection spreads by feco-oral route.It is quite impossible practically to prevent the infection due to poor sanitary conditions.however, high blood pressure has no relation with H.pylori infection.Get it evaluated and treated.This BP is very high so requires proper evaluation.This is my personal opinion for you based on available details. If you have any further query please ask me.Stay HealthyDr Anshul Varshney, MD"
},
{
"id": 30807,
"tgt": "What are the symptoms of potential malaria?",
"src": "Patient: Hi, may I answer your health queries right now ? Please type your query here... Hi, im an African kid living in Nigeria, i had a sore throat four days ago, so i started ampicilin as adviced by my aunt who's a nurse, the next day a serious fever began, and i didn't have access to drugs for about eight hours into the fever, that night my mom began treatment on me for malaria, with some drug called amatem, now my skin is sensitive to touch, every morning when i wake up i have serious aches all over, and my fever comes and goes. i need your help i need to know whats wrong with me. Doctor: Hello and thanks for sharing your concern with us.I would like to tell you that first a definitive diagnosis should be made before starting the treatment . For making a definitive treatment , proper diagnostic tests like CBC , Peripheral picture of blood thin and thick smear both , should be done . Such fever with sore throat could be a viral sickness , bacterial infection and can be any other infection as well which should be first be evaluated through proper testing and clinical examination and then after making a diagnosis , the treatment should be begun .I would recommend you to consult with your PCP STRAIGHT AWAY and don't take medicine yourself . Best Regards,DR.S.TOMAR"
},
{
"id": 63161,
"tgt": "What causes a lump on the knee cap?",
"src": "Patient: I have just discovered a small lump about the same size as a pea on my left knee cap. It is mobile, it goes round in circles about 1cm diameter. It is not hard like a stone but harder than a pea, a bit like a mushroom, but it doesn't squash. thank you Doctor: Hi,Dear Thanks for the query to HCM virtual Clinic.I studied your query with available details given in your query.I understood your health concerns.Cause of the lump on knee cap-Its very difficult to give any cause from the given data which is insufficient.Still with my expertise and experience andFrom the facts given in your query,you seem to have either of the following-Sebaceous Cyst or Epidermoid Cyst /or could be a F.B.fibroid if history of injury in the sand, corroborates it/or Could be Bursitis(house maid's knee)I would advise you to consult ER Surgeon,to rule out various possibilities of such a lump, and to plan for its treatment,which is your main concern.Don't Self-treat or medicate in your case as the case may go in to further complications.Hope this reply would resolve your query and would help you to plan treatment with your doctor.Hope this would relieve of your health concerns.Welcome for any further query in this regard before final suggestions are given in your query.Wishing you fast recovery..Have a good day.Dr.Savaskar M.N.Senior Surgical Specialist."
},
{
"id": 2553,
"tgt": "Are Gulucophase and Dupaston helpful in treatment of female infertility?",
"src": "Patient: Hi,, I am 25 yr and married from 2 yrs, I have pcos, and very irregular periods, and I badly need to conceive baby, my Dr told me to take gulucophase and dupaston for 3 month, does this medicine help me to get pregnant.... And clomid from day 3 to 7 in my periods,,, only one cycle... Please tell me is this help me Doctor: Hello dearI understand your concernPCOS is complex disease and treatment response is variable.It cause hormonal imbalance, menstrual irregularity, insulin resistance and weight gain.All these factors cause female infertility.Both glucopahse, duphaston and clomid will help to become pregnant.Glucopahse: decrease insulin resistance and induce ovulationDupahston: regularize hormone imbalance, support implantation and early pregnancy.Clomid : increase ovulation in 80% case and increase fertility by 50%.After taking clomid go for USG scan and ovarian follicle study to know response of treatment.If ovarian follicle size will be between 18-21 mm then it will release best quality egg and high chance of pregnancy.You should use above treatment for 4-6 months to become pregnant.You can also use HCG injection if ovulation will not occur and ovacare pillAvoid stress, take healthy diet, drink plenty if water and do regular exerciseHope this may help youContact further if follow up neededBest regardsDr. Sagar"
},
{
"id": 208362,
"tgt": "What are the symptoms of schizophrenia?",
"src": "Patient: Do i have schizophrenia I was diagnosed with schizophrenia after being somewhat paranoid in a psychologists office and telling a psychiatrist that i felt as if i was somewhat high after i woke up and kind loopy, she put me on meds for schizophrenia but it got worse, all started when a friend asked me if i was gay when i was on e and i forever thought he was going to tell the world . Now three years after being on meds when i try to come off of them two or three weeks later i will be fine until something substancial sets me into a mode of being crazy Doctor: Hi, thanks for the query. Based on the symptoms you described; its not possible definitely rule in or rule out whether you are suffering from schizophrenia. Can you describe in detail what do you mean by feeling paranoid? As per ICD-10 (a diagnostic system to diagose & classify all diseases designed by WHO); a person is said to have a definitive diagnosis of schizophrenia when he has 1 months of continuous symptoms of 3rd person auditory hallucinations of commenting, discussing variety, delusions of thought insertion, withdrawal, broadcast, thought being spoken aloud (thought echo), bizarre delusions, persistent hallucinations coming from a body part/other modality; apathy, social withdrawal, severe difficulty articulating thoughts properly (formal thought disorder) etc. Some substance intoxication like amphetamine, cannabis etc. also mimic schizophrenia symptoms temporarily. Hence, we need to rule out these things before coming to any conclusions. What medications you are on? are there any side effects? Please revert back to me for a more definitive diagnosis & further management. Good Luck"
},
{
"id": 56640,
"tgt": "Is it normal to have puss from belly button and fever after removing gallbladder?",
"src": "Patient: Hi, I had my gallbladder removed on 5/1/14 through the belly button. I am still in a lot of pain especially on the left side of the belly button and it goes all the way to my back. Puss was coming out of the belly button and I have been taking antibiotic for 2 days. I started having a low grade fever today. Is that normal? Should I go to the ER? Doctor: Dear Friend, greetings from HCM. I understand your concern . i think you underwent laperoscopic cholectstecromy. Pus and fever really indicates infection . you need to take stronger anti biotics please meet your surgeon and get the prescription. Might need culture and sensitivity of the pus. But better you meet your surgeon. And get final opinion from him . Thanks a lot and do contact us for more questions."
},
{
"id": 83841,
"tgt": "Does taking Creatine cause pain in lower back & around kidneys?",
"src": "Patient: Hi, i have been taking creatine mono 5g per day for the last 3 days and i m experencing lower back pain around the kindeys, is the creatine causing this pain? If so should i go to the hospital or book an apt with my doctor. I read online it could be kidney failure is this true? Doctor: Hello, It's important to drink sufficient water while taking creatine. It's usually safe to take 4- 5 g per day. Maybe your back pain is related to unusual or heavy exercise. Signs of kidney failure include reduced urine output, high color urine, etc., usually not back pain. You can observe a few more days, look for the signs I mentioned, drink 2-3 liters of water daily, avoid hot environment, and start gently while working out. If pain persists or becomes worse, surely book an apt. Hope I have answered your query. Let me know if I can assist you further. Take care Regards, Dr. Md Abu Sayeed Mustafa"
},
{
"id": 5523,
"tgt": "Planning to conceive. Concerned about folic acid overdose",
"src": "Patient: Hi,pls help me..i think i ate overdose folic acid = 5mg. me n husband plan to have baby this month but a month ago after my last period,i take folic acid because some said it's good to take earlier before pregnant. Now, my period this month is late. normally cycle is 28 or 29 day.today enter 31 day. i'm curious.why i'm not having my period.am i pregnant? Doctor: Hi, Thanks for query. Your period is delayed,wait for some more time to do pregnancy test at home. You should not worried about folic acid,it is advisable to take 3 month before you are planning to concieve. So don't worry.You had no overdose of folic acid,it is useful in neural development of foetus. GOOD LUCK. Take care."
},
{
"id": 86561,
"tgt": "Suggest treatment for lower abdominal pain",
"src": "Patient: I ve been having constant pain in my lower right side in my abdomen. Sometimes it s shooting pain but mostly ongoing aching. It s very sensitive to touch and hurts to walk, bend or lift. I m a 20 year old woman and wondering if this could be a hernia? I ve also had severe coughing in the past month due to laryngitis and a cold. Doctor: Hi,From history it seems that you might be having Sub acute appendicitis producing this problem as there is tender spot in right lower abdomen and hurt while walking,bending or lifting weight.If you have bulging in inguinal region while coughing or lifting weight then there might be having formation of inguinal hernia.consult surgeon and get examined.Ok and take care."
},
{
"id": 173214,
"tgt": "Is it safe to give antibiotics for cough and cold in infant?",
"src": "Patient: my baby is 8 months old and has a bad cough n cold for past 4 days his doctor has prescribed bricanyl syrups but evn now after 4 days his cold is still there. with no improvement he is only on liquids for last 3 days. now the doc has advised him to take clavan bd dry syrup and omnacortil solution. is it safe to give him styroids and antibiotiche is just 8 months old plus he has not eaten anything solid? Doctor: I understand your fears, Apart from cold he would have had chest infection associated with wheeze, THis could be the reason for starting giving antibiotics and omnacortil. I would also do a Blood test and Chest x ray and also give nebulisation if wheezy bronchitis.There is no harm in giving a short course of steroid in a wheezy attack as this category of drugs are preventers of soft tissue swelling the respiratory tract and is combined with relievers like Terbutaline ( Bricanyl as is done ).As he is taking breastfeed he is not on empty stomach as you fear. Some people may also give Rantac syrup to prevent gastric irritation. YOu can review with your doctor and I wish your son well."
},
{
"id": 200894,
"tgt": "Suggest treatment for enlarged prostate",
"src": "Patient: my father is 88 he is being treated for enlarged prostate with tamsoulin and for fluid retention with furosemide 40mg twice daily he is finding pressure on his stomach as if it is full of gas and he constantly feels like he needs to urinate but only passes small amounts, he is awaiting call to heart failure clinic, he had been in hospital for a week treated for edema and fluid retention, since he came out his brething is better but the other symptoms make him uncomfortable, any suggestions Doctor: Thanks for asking in healthcaremagic forumIn short: Surgery at this age is risky.Explanation: As you said he had oedema and under treatment for that. Most of the symptoms you said are related to enlarged prostate, so please visit a surgeon after taking opinion from a cardiologist."
},
{
"id": 159620,
"tgt": "Cancer detected again after surgery and radiation. Best treatment?",
"src": "Patient: my father have cancer lower alov. detect 1 year ago after detection doctor saying for surgery and we done surgery 21 aug. 2011 and take after 1.5 month radiation 60gy but after 11 month doctor detect again cancer in opposite side of mandible what ,s the best treatment for this , that is showing in only biopsy not in ct scan ? Doctor: dear amith the treatment varies depending upon the cell study report of the biopsy. Usually recurrent disease of the same type is managed by chemotherapy plus radiation if the margins are operable then tumour may be surgically removed but if not then chemotherapy and radiation are only hope"
},
{
"id": 20302,
"tgt": "What causes tiredness,tightness in chest and breathing problems?",
"src": "Patient: I feel tired most of the time and sleep all night and when i get home from work. At work i feel tired and my body feels heavy, my chest feels tight and i breath shallowly. I feel like falling asleep and have strated to get angry alot easilier. My eyes are also very tired and i want to close them for releaf. I also find it hard to smile because my face feels strained most of the time. What could be the cause of these things? Doctor: Abnormal sleep is a common problem. It is often present in people who are overweight, particularly those who snore. If you are very tired in the morning, sleep easily during the day and snore loudly, you may have sleep apnea. Sleep apnea may cause or worsen high blood pressure. Sleep apnea is a common cause of abnormal heart rhythm requiring treatment. You should discuss these problems with your doctor and consider assessment for sleep apnea."
},
{
"id": 208448,
"tgt": "How to improve children's concentration level?",
"src": "Patient: My name is Ashim Nayek residing at Durgapur and I have two children and my elder son aged 17 year who is very absentminded and can not concentrate in his academic studies normally. His patience is very little and erratic in nature. He did very worse in his recent school final exam ( Class X exam.) due to his unmindful nature and he also involve himself in smoking cigarette seperately. He frequently behave very eratic with his perents. So please advice me how I tackle my son in coming days. thaking you, ASHIM NAYEK VILLAGE AND POST.- GOPINATHPUR DURGAPUR - 713211 DIST.- BURDWAN WEST BENGAL MOBILE : 0000. Doctor: DearWe understand your concernsI went through your details. I suggest you not to worry much. Your son is adolescent. Adolescent nature is always like what you described. This happens due to hormonal and body changes. They feel that they are matured and indulge in risk taking behavior. They ignore and behave erratically with parents and elders when they are advised because they never want to listen to advises. But you should not worry. Within some tow or three years, his behavior shall be good. For the time being, be his friend.If you require more of my help in this aspect, Please post a direct question to me in this website. Make sure that you include every minute details possible. I shall prescribe the needed psychotherapy techniques which should help you cure your condition further.Hope this answers your query. Available for further clarifications.Good luck."
},
{
"id": 219007,
"tgt": "Is subchorionic bleeding dangerous during pregnancy?",
"src": "Patient: Sir my wife is in 13 week of pregnancy she is having subchorionic bleed since 7 week of pregnancy and at present it is in a clot form of size 3.5x1.7x2.6 cm taking Tab Progesterone 200 mg, Injection Hydroxyprogesterone 500 mg weekly x 4 weeks taken, Tab Calcium, Tab Folic Acid, Tab Eltroxin 50 mcg is there any problem further Doctor: Hi, subchorionic bleeding is not much of a problem if size is decreasing on repeat scan.She is taking correct medicines. Just tell her to take as much bed rest as possible. One more thing that there is a chance of less liquor with growth retardation of baby, so get the growth scans done after 26 weeks at intervals of 3 to 4 weeks. It will tell you early about that. Hope I have answered your question. If you have any other query I will be happy to help.Regards Dr khushboo"
},
{
"id": 80962,
"tgt": "What is the cause of chest pain?",
"src": "Patient: My 16 year old daughter is having right sided deep chest pain. She describes it as sharp stabbing pain. She states it is behind her breast. Negative trauma, negative cough. 500mg Tylenol given at 9:00PM, tried stretching and applying heat. Nothing I helping to resolve the pain. What could this be? Doctor: Thanks for your question on HCM.I can understand your situation and problem.By your history and description, I think we should rule out pleurisy first in her case.Pleurisy is inflammation of pleura. And it can cause similar kind of pain she is having (sharp , stabbing pain).Possible causes for pleurisy are1. Pneumonia2. Pleural effusion3. Tuberculosis.So get done chest x ray to rule out above mentioned causes.If chest x ray is normal, then no need to worry much for pleurisy.It is musculoskeletal pain mostly.So ask her to take painkiller and muscle relaxant tablet."
},
{
"id": 55994,
"tgt": "Suggest alternative treatment for gall bladder stone",
"src": "Patient: I have Gallbladder stone of 8.2 mm size. Is ti possible to remove the stone without removing the gallbladder itself. I am 50 years old. In case i remove Gallbladder, will there be any problem in future like life style changes, food habits, etc.. Kindly advise me. Doctor: Hello Findings suggests a large gall bladder calculus.Medicines are usually not helpful in gall badder calculus.You should go for definitive treatment of gall bladder calculus.Definitive treatment of gall bladder calculus is surgical removal of gall bladder(cholecystectomy).Gall bladder calculus is prone to infection and obstruction,so it is advised to get rid of calculus early.Get well soon.Take CareDr.Indu Bhushan"
},
{
"id": 175751,
"tgt": "What causes abdominal pain?",
"src": "Patient: my 12yr old daughter has had both mild and severe upper abdominal pain that will radiate lower for 2 weeks now. blood tests show elevated lymphocytes. i am worried and wonder what I need to say to the doctor in order to figure out what is going on with her. Doctor: Hi,From history it seems that she might be having urinary tract infection giving rise abdominal pain radiating back and high lymphocytes.Go for urine check up for pus cells and red blood cells.After report go for treatment accordingly.A course of antibiotic medicine for 5-7 days will give her cure.Give her plenty of water.Keep her local hygiene proper.Ok and take care."
},
{
"id": 126276,
"tgt": "What causes tightness and lump like feeling behind the knee?",
"src": "Patient: in back of my leg behind the knee very tight almost the feeling of an internal lump that inhibits me when im kneeling to sit with my legss bent under me without discomfort walking standing ok but i can feel the tightness i stretch but doesnt really help Doctor: Hi, We have to rule out conditions like bursitis. As the first line management, you can take analgesics like Aceclofenac or Tramadol for pain relief. If symptoms persist you can consult an orthopedician and plan for an MRI scan. Hope I have answered your query. Let me know if I can assist you further. Regards, Dr. Shinas Hussain, General & Family Physician"
},
{
"id": 217302,
"tgt": "Suggest treatment for pain near ribs, back ache and calcium deficiency",
"src": "Patient: Earlier this week (maybe four days ago), a pain started to develop just below my ribs and in my gut region. It felt like that whole region had been twisted. I didn t do anything particularly taxing this week, just working at my office. When I look in the mirror and stretch my arms upwards, the bottoms of the left and right sides of my rib-cage seem to be out of line. There is a dull ache when I m sitting or walking, but it becomes more pronounced when I m lying in bed (for instance, last night it stopped me from getting to sleep). To give you some background, I have occasional back problems, calcium deficiency and I require testosterone suppliments. Any suggestions or advice you can give would be greatly appreciated. Thank you! Doctor: get your vit-d level check and eat calcium medicines too. apply ice pack for 10 mins and a good ointment after that. eat a pain-killer do little stretching exercises like whole body stretch ,knee to chest,lower body rotation by bending your knees and rotate it to both side,do strengthening exercises as well like bridging. and get a doctor consultation also"
},
{
"id": 180522,
"tgt": "Is a hard bump on the roof of the mouth during pregnancy a matter of concern?",
"src": "Patient: I am a 25 year old female I also am 22 weeks pregnant. I was eating something and I brushed my tongue back towards my left side by my molar on the roof of mouth and it's not really close to the molar but it's up by my top wisdom tooth but I had the top wisdom tooth surgically removed. I've never noticed it before and it's not really seeable by my eye is it to be a cause of an alarm? It feels like a hard bump is this something to be concerned about? Doctor: Hello and Welcome to 'Ask A Doctor' service. I have reviewed your query and here is my advice. A hard bump at the area near the upper wisdom tooth that is not painful but apparent with naked eye can be a bony torus that can be an overgrowth of the bone.It appears as a painless bony hard lump on the palate.If it appears to be of bluish or purple color then it can be due to mucous cyst or Mucocele formation.However , both conditions are harmless, so do not worry about iT. For confirmation of exact cause you should consult an Oral Physician and get checked.In case if it is a bony tori and does not cause any problem then it can be left as it is.If it increases in size it can be surgically removed by an Oral Surgeon.If it is a mucocele then doing concentrated salt water rinses can help in its regression.If it does not improve then it has to be surgically removed.Hope I have answered your query. Let me know if I can assist you further."
},
{
"id": 132527,
"tgt": "How to treat sciatica?",
"src": "Patient: My name is Marcie and I m 54 years old. I was told I have sciatica which actually started on my left leg but now is on my right leg. I have been taking pain medication for it. I went to a spine specialist becaus it s not getting any better as a matter of fact it s getting worst. I can t stand or walk for a long time. I get pain and numbness down in my back and legs. My psyciatrist said that my MRI shows arthritis in my spine and my Xray shows a moderate anterolisthesis of L4 and L5 which reduces slightly with extension. There are prominent degenerative changes of the facet joints of the lower lumbar spine. When I read this result it didn t seem so serious. But the pain that I m feeling feels like I need some kind of surgery. Please advise. Doctor: Hello Marcie. thank you for writing us here.Sciatica is because your lumbar vertebrae are compressing the nerve roots causing pain in both your legs.For the treatment you can either go for a steroid therapy which would have several side effects but the results are quite quick.secondly you can go for physiotherapy which would strengthen the muscles and help proper regeneration of bones and would help in treatment the it would take around 10 to 12 months for full recovery.You can see an orthopedic doctor for the treatment. take your time and I highly respect your decision which way you would like to get treated.Best regards,Dr Gunjan"
},
{
"id": 149750,
"tgt": "What are the effects of marijuana given to people with severe dementia ?",
"src": "Patient: Hello, I am currently looking at the effects of mariujana given to people with severe dementia. Mu husband is in a nursing home in high care however he is in severe pain every time he moves or I'd dressed etc. My son talked about the relief the drugs may give my husband but I am dubious of how it would affect him. Do you have any knowledge of t his treatment. Doctor: Hi,Thank you for posting your question here, I will try to answer it to the best of my abilities.Medical marijuana is currently only given to cancer patients. There are theories about its useful effects on dementia and Alzheimer's patients, but it is still being researched and hasnt reached clinical trials yet, as such to give your husband marijuana, is something that may not work out. So I recommend you follow the conventional treatment till the research has progressed and has proven the uses of marijuana.I hope this answered your question."
},
{
"id": 32345,
"tgt": "What does headache, fever and pain in throat suggest?",
"src": "Patient: My daughter has a frontal headache and fever of 102. My dad gave her 160 mg of acetaminophen about 6 o'clock. It has not helped her fever. Her neck is not stiff. Her throat hurts a little, but she described it as achy, not scratchy like a traditional sore throat. Doctor: Hi Dear,Welcome to HCM.Understamding your concern. As per your query your daughter is suffering from headache, fever and pain throat which seems to be infection from streotococcal bacteria leading to strep throat. It could be due to viral infection as well. Need not to worry about it. You should visit pediatrician once and get it examined. You should start medication after lab tests, blood tests and proper diagnosis. Till then give antobiotic along with Ibuprofen / Acetaminophen. Give anti histamine such as Levocetrizine to her. Avoid use of cold liquid substances. Give her steam inhalation. Use mist humidifiers in her room. Avoid spicy, sharp and oily food. Give diet rich in multivitamins. Hope your concern has been resolved.Get Well Soon.Best Wishes,Dr. Harry Maheshwari"
},
{
"id": 70214,
"tgt": "What causes tiny white bumps on the upper lip?",
"src": "Patient: I woke up this morning and realize I had tiny bumps that covered my while upper lip. They re aren t noticeable but I feel like my upper lip is very scaly nothing has really changed about my diet except that ive been drinking more amounts of water and have a tendency to eat ice alot. So if u can please help I greatly appreciate it thanks Doctor: hello thanks for consulting at hcm..it could be fordyce granules,,since it is painless, and it could be If this does not resolve in 10-14 days or becomes increasingly worse, seek the treatment of a oral medicine/general physician. Pagophagia is specifically pica for ice, and it is specific indicator for iron deficiency anemia.dnt forgt t tk opinion f specialist..hope it helps..tc..."
},
{
"id": 31194,
"tgt": "How to treat UTI?",
"src": "Patient: Hi goodmorning. Im 17 yrs old and I just had (or still have) UTI. It started the other day when I was constantly peeing, the urge to pee but nothing's there and then the pain after peeing. I went home immediately, but the pharmacies were closed already so I took medicine the next day. The med that I took was Rifampicin, that was what my grandfather gave me (im living with my grandparents). He also said that it had side effects of red urine. I took 2, one in the morning and one in the afternoon. All of the pain went away but the red urine hasn't, and I'm just confused if its my period (i'm supposed to have my period this week, im a regular) or if its still the side effect of the drug. Please help me? Thank you and God bless. Doctor: Hi dear,It seems that the red urine you are getting is not the side effect of the Rifampicin, but blood cells in urine suggesting UTI. Also, Rifampicin is the medicine given mostly in TB cases, so it might be the coincidence of taking Rifampicin and your immunity boosting up.Anyways, I suggest you take Levoflox 500 mg 2 times a day for 5 days and also take Zinetac 150 mg 2 times a day to cover for acidity due to medicine. I would suggst you check if you habe fever by thermometer. If you have, you can take PCM 500 mg as need arises. If symptoms don't resolve within 2 days, go for blood test for Complete Blood Count (CBC) and urine routine test. Do complete the course of Levoflox if you start it. I hope this solves your query. Wishing you a veru healthy life ahead. Thank you for choosing us to help you. Regards,Dr. Viraj Shah"
},
{
"id": 125047,
"tgt": "What causes leg hurt while playing?",
"src": "Patient: Hello 4 years ago i was hit by a car my tibia and fibula bone broke after a surgery my doctor placed a rod and three screws inside my leg. after 18 months i went back to my doctor and he operated my leg for taking out the rod and screws from my leg. operation was successful but right after one year of removing rod from my leg i started playing football. but it was not good because my leg started hurting really bad and i went to my doctor and so many other doctors they all said nothing is wrong with your leg. even i did x rays nothing was unusual in the x ray as well . but it still hurts a lot please help me sir i don t know what to do. thank you for your time... Sayed Saber Sadat Doctor: Hi, As a first line management you can take analgesics like Paracetamol or Aceclofenac for pain relief. We have to rule out possible causes like peripheral vascular disease and ligament sprain. If symptoms persists better to consult an orthopedician and plan for an MRI scan. Hope I have answered your query. Let me know if I can assist you further. Regards, Dr. Shinas Hussain, General & Family Physician"
},
{
"id": 120597,
"tgt": "What causes abnormal strep in blood test in case of joint pain?",
"src": "Patient: Hi, I had a lot of joint pain that came on suddenly and was most prominate at night. I had it the pain for 3 or 4 days so i went to my local GP. He blood tested me and I have got a call back. The receptionist said my (i think she said) strep was abnormal? What does this mean? I have an app. for tomorrow. Doctor: Hello,I read carefully your query and understand your concern. In the blood test we check for antibodies that your body makes while fighting group A Streptococcus.The antibodies work against a substance made by the bacteria called streptolysin O.Streptococcus can also cause joint pain.In case that the tests are positive the treatment is specific and should be given by your gp.Meanwhile,I suggest using anti inflammatory medications such as Acetaminophen to relieve the joint pain. Hope my answer was helpful.If you have further queries feel free to contact me again.Kind regards! Dr.Dorina Gurabardhi General &Family Physician"
},
{
"id": 18057,
"tgt": "Suggest treatment for high BP and blurred vision",
"src": "Patient: 15 y/o male. 6 4 230#. High blood pressure earlier today 150/95. Was feeling dizzy at basketball practice. Trainer took his BP. Rested, drank water and it lowered to 130/80. Tonight he s complaining of blurred vision and feels like a migrane is coming on. Any concerns? Doctor: Hello! Regarding your concern, I would explain that high blood pressure values could be related to straining physical activity or anxiety. Anyway, I would recommend consulting with his attending physician for a physical exam and some tests: -a resting ECG- complete blood count, PCR, ESR for inflammation- thyroid hormone levels for thyroid gland dysfunction- kidney and liver function tests for possible dysfunction- cortisol and aldosterone for adrenal gland dysfunction. You should discuss with his doctor on the above tests. Meanwhile, it is necessary to closely monitor his blood pressure values during the day. Hope I have answered your query. Let me know if I can assist you further."
},
{
"id": 171343,
"tgt": "Could wound on temple cause problem with eyes?",
"src": "Patient: My 18 month old was running, fell and hit my platform bed on the one inch corner that sticks out right in the temple. Immediately left an indent in the temple, now its puffy there, she is playing now. After she stopped crying she was blinking her eyes and rubbing them. Doctor: Hi,Welcome to Hcm,Sorry to hear about the unfortunate incident. Good thing is that your child is active and the trauma is not a major one. It could have been lot worse. Although from your description it doesn't seem like the trauma can cause any problems to vision. But, any trauma in and around eye, especially in a small child should be very careful about. If you have any doubt, its better to consult an eye specialist because we need to look inside if any internal trauma has been caused. Wish your child a good health. Take care."
},
{
"id": 160265,
"tgt": "BRAIN CANCER SYMPTOMS, PLEASE HELP",
"src": "Patient: My grand mother age is 65 and couple of days backs doctors have clarified that she has breast cancer and adviced to cut one of them firstly would like to know how safe is it and wanted to get her treayed in delhi AIMS how do i go about it Doctor: hi md tauseef khan..risk of surgery is much lesser than cancer spread.in early stages it may even make disease free.so don't think much and leave this issue for doctor."
},
{
"id": 115006,
"tgt": "Suggest suitable medicine for thalassemia",
"src": "Patient: Hallo sir,i am jatin sejpal a major thalassemic from pune on regular blood tranfusion and iron chelation thearpy.my age is 35 years male and unmarraiged.my father has diabetic for last 15 years,I am on cap. kelfer 500 mg taking 4- 3-3 .my hba1c is 8.4.my Bsl fasting is 104 and my Bsl pp is 143.i am presently is on tab.voligbose 0.2 BD.FROM LAST 1 MONTH.pls tell me is this a right medicine.thanks Doctor: Hello,since you have thalassemia there is a strong suspicion that your pancreas has been damaged by chronic iron overload. The chelating therapy surely helps but not always (and not completely).If this is your case, then Voligbose might help with postprandial blood glucose but it won't target the source of your problems which is (presumably) insulin deficiency due to pancreatic destruction. If insulin is really missing you should take it regardless of the blood glucose levels to prevent serious complications like ketoacidosis. Voligbose is usually given to type 2 patients who have an almost adequate insulin production.Your doctor is the right person to investigate (by evaluating your history and laboratory tests) whether your diabetes is type 1 or type 2 and modify your treatment accordingly.I hope I've helped!I'll be glad to ask any further questions you may have. You can contact me again if you'd like!Kind Regards!"
},
{
"id": 127380,
"tgt": "How long does a bruised trachea take to heal?",
"src": "Patient: how long does it take for bruised trachea to heal? my husband and I were experimenting with choke play about 2 weeks ago and bruised my trachea however it is still extremely painful to swallow and feels like my throat dries out three times faster than it used to. Doctor: Hello and Welcome to \u2018Ask A Doctor\u2019 service. I have reviewed your query and here is my advice. Trachea is a sensitive organ and it will take a month to heal. Analgesics will help. Hope I have answered your query. Let me know if I can assist you further."
},
{
"id": 90581,
"tgt": "Suggest remedy for abdominal pain",
"src": "Patient: i first started to have apain on the right side of upper abdomen now the pain is on and off but what worries me most is constant pressure i feel under right rib cage radiating to the back more especially when im sitted,im 1.5meters tall,55kg was treated for tb in 2004,have gastritis problem. Doctor: The following conditions often cause upper right abdominal pain right abdominal pain:AppendicitisCholangitis (bile duct inflammation)DiverticulitisFecal impaction (hardened stool that can't be eliminated)Gallbladder cancerGallstonesGastritis (inflammation of the stomach lining)Hepatitis (liver inflammation) liver abscess , pleurisy, kidney stone (right) etc... There are many causes to rule out . I would suggest a CT abdomen to rule out causes and few blood investigation . Consult your physicians."
},
{
"id": 56642,
"tgt": "How serious could multiple gallbladder stones with pain and swelling be?",
"src": "Patient: My father has multiple stones in his gall bladdr with severe pain.after ultrasound we came to know.there is also swelling on the wall of it.i want to know is it dangerous? Can the gal bladdr burst? Doctor said we will do surgery after 4 to 5 weeks when swelling will end Doctor: HelloYour father is rightly advised for surgery after 4-5 weeks .Findings suggests acute calculus cholecystitis means acute infection of gall bladder due to calculus.Definitive treatment in this condition is surgical removal of gall bladder but it is wise to operate after control of infection.There can be more complications if it is operated at this stage.Surgery can be done laproscopically.Recovery is fast by this procedure.Present condition can be managed by antibiotics and analgesics.Don't worry about this.Get well soon.Take CareDr.Indu Bhushan"
},
{
"id": 1269,
"tgt": "How to increase chances of conception?",
"src": "Patient: Me and my boyfriend have been trying to get pregnant for about a month with no success. I don't know what else to do. I had my period the 21st and it ended 4 days later which is irregular for me. It usually lasts 7 days or so. I need some help as to what to do about this issue. Doctor: Hi, I think if your periods are regular and you and your husband are not having any obvious problem, you can try naturally for 6 months. Be in contact with your husband every 2 to 3 days after your periods stop. If it doesn't work then you can go for evaluation. Hope I have answered your question. Regards Dr khushboo"
},
{
"id": 51955,
"tgt": "Is my urine result normal ?",
"src": "Patient: sir,iam 37 y female.i have recurrent uti.atpresent i take cefxime-5days.my urine result as follow.reaction-alkaline,albumin-faint trace,sugar nil,bilesalt,pigment-negative,puscells-2-3,redcells0-1,epithial cells4-6,bacterial rods nil,mucus threads nil,.is this normal or not sir.iam worried. Doctor: Hello banu ; welcome to HealthcareMagic The report you have posted is normal but it is also important to see that you have no symptoms like before :-frequent urination;fever;burning during urine or any other symptom you had.If there is no symptom and you are feeling better and since the report is also normal you should continue the medicine for the period as recommended by your doctor and you will be fine. Thanks"
},
{
"id": 92273,
"tgt": "What could be the cause for severe pain in left side after endoscopy for ulcer?",
"src": "Patient: friday I had a endoscopy and everything went well...have uclers...Sat and Sun I felt fine but today I have a lotof pain in my left side..very painful..I don't know what could cause it...someone told me might be from the air theyput into my stomach, but this is in my side not my stomach Doctor: Hi...it maybe ureteric colic or an ovarian cyst (if you r a female)..... get a usg done to find out...Dr. Ashish Verma"
},
{
"id": 50414,
"tgt": "CT scan shows multiple stones in right kidney, nausea, pain in the right side of the back, blood in urine. Any procedure to remove all stones?",
"src": "Patient: I recently had a ct scan . I have msk and am a stoneformer. The scan showed a 5mm in midpole of my right kidney and a 2mm in upperpole of right kidney also multiple stones in both but more prominent in right that couldn t be measured accurately. Multiple stones isn t unusual in my case. I am having nausea and the pain in my side and right side of my back(flank) also blood in my urine . Should these stones be causing this pain? I have had more than several stones. Also I heard of a procedure where all stones could be removed. Is this something I should consider and speak to my urologist about considering my situation? Doctor: Hi, Many thanks for the query!You need to do certain basic investigations first, which include-USG (KUB), Urine routine & microscopy, Sr. creatinine, Sr. uric acid.Blood in urine is usually due to Ca-oxalate stones having spikes on their surface causing mechanical trauma to mucosa of urinary tract & bleeding.Pain is due to spasms of the ureter to get rid of the stone by more frequent & powerful contractions to propel it out.Also as you have recurrent stones, do- Metabolic workup of 24 hrsurine sample, Sr. PTH, Sr. Calcium.As per the reports, appropriate diet instructions can be given & if Sr. uric acid is raised, Uric acid synthesis inhibitors like allopurinol can be started.Along with thiazide diuretics, you need to take Potassium Magnesium citrate, also take urine alkanizing agents with your Urologist's opinion.Take symptomatic treatment as per Urologist's opinion till you get your investigations done, viz. Plenty of oral fluids, anti-spasmodics, urinary alkalinizing agents, pain killers.5 mm & 2 mm stones need not be removed surgically, they will come out with above medicines.Wish you a good health.Take care.Regards."
},
{
"id": 146342,
"tgt": "Could pineal cyst be the cause for headaches and imbalance?",
"src": "Patient: In 2010, I had a MRI of the brain with and without contrast and nothing abnormal. July of 2014, I had another MRI of the brain with and without contrast. The result was that I have a pineal cyst 6 mm. My neurologist and family doctor say in 6 months to do another MRI. However, I believe I need to be followed by a brain surgeon because I have some serious symptoms such as, legs becoming like jello after waking up from a nights sleep, so I fall to the floor because I feel nothing in my legs; I have headaches but to get rid of the headache is the struggle, but after a headache, my head is so sore and tender all over; I use to have 20/ 20 vision, know I wear glasses and the eye doctor says it is stigmatism; then i have fibromyalgia and focus, fatigue, steep, and blurry vision occurs daily. I have spinal issues (disc bulges, mild degenerative changes, a tiny posterior annular tear; mild bilateral hypertrophic degenerative facet changes, spinal canal stenosis. I weight 318 pounds, and I had the gastric bypass surgery of 8/2013. I have lost 115 pounds. Everything is worse, the pain and my thinking and thoughts are somewhat hard to get out. I have depression, anxiety, and short term memory loss. I meet with a spinal surgeon this Friday. I meet with a brain surgeon on 18th of aug. I am meeting with the surgeons for 2nd opinions because I need to know what my future goals could or need to be done. I want to live and I want to get better. All this stemmed from a car accident, other than the pineal cyst. My question is how do I decipher what symptoms go with what diagnosis because it appears that the same symptoms can either be from this illness and that illness? I done a lot of pineal cyst research, of what is out there. Doctor: I don't think the pineal cyst is responsible for your diagnosis. It's of a small size and there is no reason for it to cause any symptoms, pineal cysts are found by coincidence in many people. As you yourself said many of your other medical issues can cause overlapping symptoms and justify your complaints, but I don't think the pineal cyst has to with that and I agree with your neurologist's recommendation of leaving it as it is and have a control MRI, perhaps even later than 6 months, say in a year."
},
{
"id": 10411,
"tgt": "Can marijuana cause hair loss?",
"src": "Patient: Has marijuana useage been known to cause significant hair loss in some people or is it the affects on b12 levels that causes this? i need to know bc i have no other medical symptoms and i have noticed significan hair loss since i started smoking but i notice if i lay off of marijuana for a while my hair starts to grow back? Doctor: Hello and Welcome to \u2018Ask A Doctor\u2019 service. I have reviewed your query and here is my advice. Addiction to marijuana can cause nutritional deficiency which I n turn can lead to excessive hair shedding. Hope I have answered your query. Let me know if I can assist you further."
},
{
"id": 103775,
"tgt": "MRI scan reveals c/5 c6 degeneration with foraminal stenosis. Suffer from raynauds syndrome, kidney failure, depression. Suggest",
"src": "Patient: Hi. I had a mri scan in 2012. The results were c/5 c6 degeneration with foraminal stenosis. I saw a orthopedic consultant who didn't really explain my diagnosis. He spoke briefly of a operation but I suffer with anti phosphilid syndrome and raynauds syndrome multiple pes livedo reticularis. Kidney faliure. Asthma. Servere deppression and ptsd. It is my left side that is mainly affected servere pain in my arm and elbow numbness in my hands with tingling my arms hands and legs are weak. I also suffer very bad headaches and hair loss. Any explanations greatly appreciated. Thank you. YYYY@YYYY . Doctor: Hi welcome to Health care magic forum. Thanks for calling H.C.M.Forum. You had MRIscan in 2012, results were c/5, c/6degeneration with foraminal stenosis, orthopedic advised surgery, but you are a patient of phospholipid syndrome,raynouds syndrome, multiplePES, livido reticularis, kidney failure, asthma, severe depression,and PTSD, headache, and hairloss, pain in arm, elbow, numbness and tingling in hands, arms and legs are week. Of course it is better to avoid surgery if possible,in that case you explain the doctor about your medical history. Besides medicines i advise you to avoid potato, other tubers, tomato, egg, chicken, and seafoods, so long you have pains. Wishing for a quick and complete recovery. Best regards."
},
{
"id": 112492,
"tgt": "Severe back and neck problems. Taking oxycodone. suggestions?",
"src": "Patient: I have several back and neck problems. i have been taking oxycodone for years and had one surgery so far and may be looking at another soon. the oxycodone 10 I am currently taking is not working any more, My sister takes roxicodone 15 that help her considerably but my pain doctor says they are really bad for you. Is there that much difference in the two. I know none are good but I don't have any chose anymore. Doctor: Hi and welcome to HCMThere is no much difference between these tow drugs. These are of same group. For your back and neck problem you should do regular physiotherapy to strengthen the muscle and relieve in pain.I do hope that you have found something helpful and informative and I will be glad to answer any further query.Take care."
},
{
"id": 223952,
"tgt": "Is it normal to have brown discharge after installing the implant?",
"src": "Patient: Had te implant inserted on the the last day of my period 26th August have not had my period this month, but on the 20th September I started having brown discharge for four days, it went away for 3-4 days and is now back, yesterday there was blood spotting, Is this normal side effects of the implant ? Doctor: Hi health care magic user Ya its absolutely normal to have brown discharge after implant,Its common symptoms few weeks after INSERTING IMPLANT SO BE RELAX AND DONT WORRY . THE REASON BEHIND THAT IS HORMONAL CHANGES WHICH OCCUR AFTER INSERTING IMPLANT i HOPE MY ANSWER HELP YOUTHANKS"
},
{
"id": 99371,
"tgt": "Suggest remedy for difficulty in breathing",
"src": "Patient: Hello doctor, I stopped smoking 6 months ago, since then I have not visited the club till 2weeks ago when I went twice and stayed for hours in an airtight place covered with smoke, after then I started feeling sick, firstly I had stuffy nose and blocked nose to fever, now I can t breathe properly, I m still awake cos I find it difficult to breathe and can t sleep, what can I do. Doctor: Hi welcome to HCM Dear polution is the main cause of falling sick and difficulty in breathing . By now you must have realised this fact ? Adopt Exercise to Breathe Easier .Weak muscles need more oxygen, so you can become short of breath just inhaling or just moving . Exercise changes that. When your muscles are stronger, daily activities are easier . Yoga & pranayam - Deep Breathing Inhale - Hold - Exhale - Hold , for 2 seconds each , repeat 15 times . Always breath thru nose . Anulom Vilom - close your right nostril with your thumb ,inhale from left nostril , Hold for 2seconds ,Exhale from right nostril ,close your left nostril with your thumb , Hold for 2 seconds and Inhale from right nostril . this Repeat 10 times . Kapalbhatti - Exhale with slow force , only throw out your breath , all doing regularly will further strengthen your mscles . If you haven't been active lately, check with your doctor before starting an exercise .A balanced diet which include all essential nutrients is equally important .Peanut butter can pump up your intake of both calories and protein. Protein is important at every meal for people with breathing problem. Good sources include eggs, lean meat, fish, poultry, legumes, and nuts .Crushed Garlic ,ginger with honey , lemon juice with warm water with a pinch of salt and black pepper Vitamin B6 and Vitamin B12 are very important nutrients to treat asthma decreasing the inflammation in the lungs.Vitamin C Is needed to fight infection, increase the amount of oxygen and reduce inflammation. Lemon juice and amla are the best source of vit C . and also a level spoon of turmeric powder with milk at bed time help boosting immune system . Avoid milk and milk products, substituting other calcium sources.Avoid processed foods carbs , sugar , fried ,fast foods , Tea , coffee ,alcohol and smoking , stress , sleeplessness and Constipation all will produce biochemical and metabolic conditions in your body that will decrease your immunity.So avoiding is definitely the first step in the right direction to detoxing the body, boosting the immune system .Always drink plenty of water to detoxify & keep your respiratory tract secretions more fluid. Medical treatment suggested by your doctor can be complemented using a variety of alternative therapies. Most of these therapies or home remedies focus on strengthening your immune system so that you can overcome your problem.Regular check ups from your proffesoinal are recommended . Hope this helps solves your query .Take care .All the best .Get well soon.Don't hesitate to come back in case of any further query ."
},
{
"id": 58672,
"tgt": "Inflamed liver, vomiting constantly. Normal gallium scan, waiting fro biopsy report. What type of diet can be followed?",
"src": "Patient: my son has been very ill for 9 weeks, has inflamed liver, hep has been ruled out, he has had a galium scan which has shown nothing but the inflamed liver, he has just had a liver biopsy but results will not be available until thursday, he is vomiting near constantly is there anything I can give him to help him settle his stomach Doctor: Hello, Thanks for the query to H.C.M. Forum. Inflammation in liver itself causes vomiting & hyper acidity.Plenty of water ( add salt+sugar) but sip by sip will help. Syrup ( with ingredients oxetacaine ) will provide relief. Stemetil Mouth dissolving tablet + antacid tablets. Banana is also helpful .Still , no relief . Take tow CLOVE boil them in water (100 ml) when this water become tepid sip it like tea and at once relief. Good luck for good scan report. Dr. HET"
},
{
"id": 13950,
"tgt": "What causes ref flat dots on ankles and legs?",
"src": "Patient: Hi, My name is Ashley, and over the past week I have noticed an increasingly large amount of different sized red flat not itchy dots on the tops and ankles of my feet. And slowly migrating up my legs. I have cleaned my lwhol house changed my bedding and still there are more. I was at the beach today and after coming home and showering they have multipled again. They are not itchy or raised up.. What could this be,? Doctor: Hello, I have gone through your complaints and it seems to be pigmented purpuric dermatoses. It is a chronic pigmentation disorder associated with prolonged walking, physical activity, diabetes, obesity and thyroid disorders. Hope I have answered your query. Let me know if I can assist you further."
},
{
"id": 169250,
"tgt": "Suggest remedy for lack of appetite in a child",
"src": "Patient: Hello Dr. My son is 9 years old and he isn t taking food properly.He brings back lunch from School.He says he is not feeling hungry.He is 27kg in weight and 138cm in height.Can you please suggest multivitamin tablets or syrup to catch up the required nutrients. Regards Mom Doctor: Hi Dear,Welcome to HCM.Understanding your concern. As per your query your son have symptoms of lack of appetite. Well the symptoms you mention in query can occur due to many reasons like slow growth of the child, sickness, stress, depression , anorexia nervosa, medications like antibiotics, anemia, intestinal worms and irregular bowel movements. Need not to worry. For now drink plenty of water and avoid fatty and spicy food. You should give child proper rest and diet rich in multivitamins. I would suggest you to consult pediatrician for proper diagnosis. Doctor may order blood test, stool test and physical examination. Doctor may prescribe drug like albendazole , iron tablets and laxatives and do counseling. Hope your concern has been resolved.Get Well Soon.Best Wishes,Dr. Harry Maheshwari"
},
{
"id": 187181,
"tgt": "What are the treatments for painful and swollen gum and cheek?",
"src": "Patient: Hi, I have a swollen gum on the left side next to the wisdom tooth. It's very painful and I have swollen cheek. I have been taking Iburp and using salt water which had helped. I have had this pain for the 3 days which has reduced. Can you suggested what should I do and could be the problem?.I have dentist appointment on the 20 January Doctor: Hello, thank you for consulting with healthcaremagic. This swollen gums can be because of an impacted tooth that might be situated below that gums. Better to get it removed as, it will create problem till it is there.So you should visit your dentist and get it removed. Hope it will help you."
},
{
"id": 102537,
"tgt": "Does dust allery cause Suspicious Apical Densities?",
"src": "Patient: hi doc, my husband just had his xray and the results were Suspicious both apical densities, =lordtodic view apical density he has no issue of having TB or PTB Do you think his allergies from dust pollens etc, might have caused these? Whenever he smelled dust he continually sneeze Doctor: Hello,Welcome to HCM,Most common cause for apical lung densities are Pulmonary tuberculosis and pneumonia leading to consolidation of lung.Pulmonary tuberculosis more commonly involves apical lung fields.In bronchial asthma you will see nothing on x-ray.Asthma is a condition of hypersensitivity of airway tract due to allergy to dust, pollen and mites. Asthma is diagnosed based on the clinical symptoms of the patient.Hope I have answered your doubt.Thank you."
},
{
"id": 184288,
"tgt": "Suggest treatment for swelling and pain in jaw after root canal",
"src": "Patient: Hi, I had a partial root canal on Saturday, in which the dentist left it open for it to drain. It was a previous root canal that had gotten infected. Well, my cheek and jaw weren't swollen before I got there, but now the lower left cheek and jaw are huge, a bit numb, and appear to be growing! Is this normal? He left the tooth open to drain, but I haven't seen any drainage. This is my second day taking Clindamycin. I'm getting worried. Should I go in somewhere now? Thank you in advance for your help!Doug Doctor: Thanks for your query, I have gone through your query.The pain and swelling cheek and neck region could be because of the infection from the same tooth. Following instrumentation the infection would have got aggravated. You can take a course of antibiotics like amoxicillin 500mg and metronidazole 400mg tid for 5 days (if you are not allergic) instead of clindamycin.Consult your endodontist and get the tooth irrigated and cleaned so that puss will drain from the tooth. Do not apply pressure over the swelling. do saline gargling.I hope my answer will help you, take care."
},
{
"id": 223175,
"tgt": "Are stomach pain and urge to vomit side effects of i pill?",
"src": "Patient: hi i am mansi . i am 18 and had sex wid my bf.. after having sex i had takken ipill on the same day. but after taking ipill i am not feeling well . i had a sensation of vomitting nd sickness or pain in my stomach.. do i get pregnant ? or is these are just the side effects ? Doctor: Hello and Welcome to \u2018Ask A Doctor\u2019 service. I have reviewed your query and here is my advice. It can be both.More chances of IPill side effects because you have taken immediately after sex. Frequent IPill usage should be avoided. You can wait for 24 hrs and if no relief consult a gynaecologist. Hope I have answered your query. Let me know if I can assist you further."
},
{
"id": 96797,
"tgt": "Does an injured antihelix require medical attention?",
"src": "Patient: Hi, my son fell and hit his head on the corner of our TV stand. It looks as if he pierced his antihelix or around that area. There is a quarter inch gash and it was bleeding pretty heavily. My husband seems to think that there is nothing that a doctor could do about it but I am worrying slightly more. Should we take him to the Er or can we wait to see his doctor? Doctor: Hello..Please don't panic..your son will be alright,there is nothing much to worry about. The pinna or antihelix is a cartilagenous area and will take its own time to heal but if there is an open wound on it, it should be properly cleaned up and dressing should be done to avoid infection . Please take him to an ENT specialist just to check whether he has not sustained any internal injury to ear drum . Only if he has constant vomiting and vertigo,please take him immediately to emergency .otherwise there is no problem."
},
{
"id": 9201,
"tgt": "Suggest remedy for dry face with swelling",
"src": "Patient: I used a cloth facemask last thursday evening, a day later my face felt very dry, 24 hrs later my face looks swollen and burnt, it was hot to touch, it looks as if I had some sort of chemical burn, I applied some e45 lotion every couple of hours and the swelling went down, its now feels very very dry, and still looks like I have had bad sunburn Doctor: HIWell come to HCMI really appreciate your concern, it could be due to some contact dermatitis and it is better to use \"Diphenhydramine lotion\" if this fails to responds then it is advisable to use the steroid hope this information helps, take care and have a nice day."
},
{
"id": 116543,
"tgt": "What does \"hemoglobin-11.7 and hematocrit-34.7\" suggest?",
"src": "Patient: hello doctor! I worry about the outcome of lab tests that I performed. My hemoglobin is at 11.70 and 34.70 hematocrit, both appear to be low. Eosinophils in addition to 43 (high) Monocyte 49 (high) Band 7 (high) and Neutrophils 1 (low) What could be the diagnosis? Doctor: Welcome to H.C.M.I am Dr Krishna Dubey.Dear friend, your monocyte counts and eosinophil counts are very high. Hemoglobin 11.7 and Hematocrit 34.7 that is slightly low.(Normal Hb value-13.0-18.0 g/dL in adult males and 11.5-16.5 g/dL in adult, non-pregnant females and HCT-normal values : 40-52 in adult males and 36-47 in adult females)You need to properly evaluate. Repeat your complete blood count and peripheral blood film( to see abnormal cells in blood) from other lab.This will us for further management. keep in touch.Thanx for query."
},
{
"id": 161248,
"tgt": "What is the treatment for eczema in a baby?",
"src": "Patient: HI I have a query regarding my 4 year old daughter. She was a bottle-fed baby,suffered eczema when she was an year old. Recently she suffered wheezing in the month of september. These are her medical history. Since nov 4th my daughter had a snoring problem.She snores at sleep.she s fine when awake.I tried steam-bath.I met the doctor and found that she had adenoids but surgery was not recommended.SHe is currently on budecort (spacer) for her whezzing problem. for the snoring she was precribed cetzine and ear wax by an ENT,saying she had developed wax in ear.I find since nov 4th snoring comes a while and goes,now since 3 days she has snoring in the noon naps and she has a caught a cold,but the nasal discharge is zero.she tries hard to blow but nothing comes out.My query is: 1.Is she not able to discharge mucus coz of adenoid. 2.SHe has difficulty since the onset of winter.today she has this blocked nose problem since noon. I am totally confused.Why did the doctor not prescribe me some saline drops for her nose? Can I assist her with one? Doctor: Hi, Atopic dermatititis. Child could be suffering from dry nose allergy eczema. Please give Montair 4mg for 1 month. Hope I have answered your query. Let me know if I can assist you further. Regards, Dr. Prasanna Lakshmi, Pediatrician"
},
{
"id": 167993,
"tgt": "What does this thyroid test report indicate?",
"src": "Patient: Hello Doctor, We have got thyroid test done for our 8 months old son. The report is as follows: FT3 (Free Triiodothryronine) 4.06 pg/ml 1.71 -3.71 FT4 (Free Thyroxine) 1.01 ng/dl 0.70 - 1.48 TSH (THYROID STIMULATING HORMONE) 1.38 uIU/ml 1.0 - 39.0 Does the high level of FT3 alarming? Please suggest Thanks Doctor: HelloI won't get worried as TSH and free T4 levels are within normal limit.T3 is a derivative of T4 and mildly raised T3 does not has any significance.I suggest you to get a repeat test done after after 1-2 months.Regards"
},
{
"id": 201750,
"tgt": "What causes lowering of sustainability during erection?",
"src": "Patient: Dear doctor, The last 3 times during intercourse with my wife I was not able to sustain the erection. I have a very low sperm count (~1 mn/ml) but never before faced the issue of not sustaining the erection. in order to boost sperm count i did take Co Q and another prescription medicine for ~2 months. And all these 3 instances have happened post taking the medicine only. Otherwise, during masturbation I do not face any problem. Any guidance will be a great help. Thanks Doctor: Thanks for contacting HCM.I am sorry to hear that you are having problems with maintaining erections and having low sperm counts. It is possible that if you masturbate too much that your sperm counts can decrease and you overall desire for your wife may decline. I recommend to help increase sperm counts and improve your erections that you decrease masturbating. If this does not help then I recommend you seek a medical attention fro possible low testosterone levels. Hope this answers your question. Please contact us again with your medical concerns and questions"
},
{
"id": 193251,
"tgt": "Can I have sex satisfactorily as my penis is only 6.5 inch long?",
"src": "Patient: Hi, I am 24 years old. I am living with lady in shared room. she is interested in and ready to have sex with me. My penis is 6.5 inch long only. it is fine for a woman? can I enjoy sex with a lady 18 years older than me? Can I fuck her enough? can I control myself till her satsfaction? Doctor: Hello,You are talking about a penis size of 6.5 inches and you are worrying about it whether it will be able to satisfy your partner with it. Please understand, average male penis erect size is 4.5 inches and you have 6.5. You should be happy. Secondly, for your knowledge, even a two inch long erect penis can satisfy women because the pleasure sensors of women vagina are available only up to two inches from their opening.Hope I have answered your query. Let me know if I can assist you further. Regards, Dr. K. V. Anand, Psychologist"
},
{
"id": 224365,
"tgt": "Supposed to start week of inactive sugar pills but accidentally started a new pack of Trivora. Concerned about preventing pregnancy ?",
"src": "Patient: I am on Trivora birth control . Yesterday I was supposed to start my week of inactive sugar pills but i accdientally started a new pack. I am sexually active and am more concerned about preventing pregnancy than period irregularity. Should I just continue with the new pack or should I take the sugar pills as I was supposed to? Doctor: HiThere is no reason to be worried. Continue taking the new pack.Since you started the new pack without taking the inactive pills you will not get your period this week. Now you will get your periods after completing the active pills of the new pack.Taking the new pack directly is definitely going to protect against pregnancy. The only change will be that you will not get mensus this week.I hope I have answered to your satisfaction"
},
{
"id": 213351,
"tgt": "Irate, mentally immature, depressed, unable to control emotions, supporting family from early age. Treatment?",
"src": "Patient: hi doctor, i feel like just to hate myself now a days. reason is now i get very irate and have no control on it and this is affecting my dear ones very much. they think that i am a good person and they forgive me every time and this is a limit now that i even cannot say sorry to them, because this situation had gone much worst now. i think i am mentally immature and i think i don t have ability to handle my relations with my friends and family. please suggest me something so that i should stop all these non sense things. about myself : i am a 25 years old guy who works in a private firm and i support my family with my earnings. i had never been in a relation with any girl in the past, i know i have interest in girls but the age i started working and supporting my family was a bit early and i never distracted my goal to support my family. my father left us when i was 10. and from that day onwards we became very poor, nothing to eat, no roof to hide, and lots and lots of problem. i think to do something good, but it goes opposite. i was in a great depression in past around 3 to 4 years ago and i think i had never came back normal. please suggest me. Doctor: Hi, From your description following are possibilities which need assessemnt and management: -Over expectation from self -Personality issues -Chronic depression -Anxiety -Poor coping skills -Excessive family stress For treatment get personality assessment, detail psychometric assessment, stop substance use (if any) morning walk and healthy diet will be helpful. Sometime effective time management solve several interpersonal issues. Final management can be planned after assessment in your case. I hope this information has been both informative and helpful for you. Wish you Good Health. Regards, Dr. Ashish Mittal www.99doctor.com"
},
{
"id": 95549,
"tgt": "I have constipation for almost 3 months . What do i do?",
"src": "Patient: ive been having problems with constipation, and lack of ability to have \u00a0bowel movements for almost 3 months. ive seen two doctors and nothing seems to be working. Doctor: Hi well come to HCM. For chronic constipation instead of taking purgative should try to improve your life style and eating habit. Take more of green leafy vegetables, fresh fruits, drink plenty of water and regular exercise will help you.Rule out condition like hypothyroidism which can cause constipation. And may get done one course of anti amoebic medicines. Thanks"
},
{
"id": 19179,
"tgt": "Suggest treatment for palpitation, insomnia and lightheadedness",
"src": "Patient: I took naddol (40mg) daily for years for rapid heart rate and hypertension. My insurance company put this drug in a different tier and suggested i take pindolol. My Dr. wrote a prescription for 10 mg twice a day. I am now up to 15 mg twice a day. My blood pressure is OK but my pulse is rapid -- about 90 rpm. I also feel nervous light- hreaded and have trouble sleeping. Should i switch back to nadolol? Doctor: Hello!Welcome and thank you for asking on HCM!I passed carefully through your question and would explain that your symptoms could be related to pindolol adverse effects. Nadolol has a longer half life and can be tolerated better compared to pindolol. Coming to this point, I would first recommend raising the dose of pindolol up to 30mg daily. If this doesn't help, I would recommend discussing with your doctor on the possibility to switch to nadolol again. Hope to have been helpful!Kind regards, Dr. Iliri"
},
{
"id": 149441,
"tgt": "On Torvela for brain AVM. History of having undergone embolisation, gamma knife surgery. Any problem in conceiving?",
"src": "Patient: Hi Sir, My spouse aged 32 yrs old suffering with Brain AVM and underwent twice emobolisation and twice gamma knife surgery. Last gamma knife surgery was done in 2009. From then she is taking Torleva 1000 mg --> 1-0-1 and Torleva 500mg --> 0-1-0 Will there be any side effects beacaus of this medicine and can we plan for kids ? Doctor: Hi and thanks for the query,There is no major problem with conception. However, I am afraid, in this case the pregnancy shall be considered a high risk pregnancy, certainly because of the increased risk of vascular complications you are exposed to, both during pregnancy and at delivery. These include increased risk embolisation especially after delivery. Torvela is no major concern here. You might need to consult an obstetrician/gynecologist who is very available, and collaboration with your treating cardiologist would be additionally very helpful.Thanks and kind regards,Bain LE, MD"
},
{
"id": 140431,
"tgt": "What course of treatment should be done for pseudotumor cerebri and migraine?",
"src": "Patient: Hello, I ve been diagnosed with psuedo tumor cerebri and migraines by my neurologist just as a brief background. I have had a migraine now for three days with auras, dizziness, vomiting, hard time being in light and dealing with sound and I am very easily agitated and an extremely painful itchy rash that has completely taken up one side of my body. I called my dr she prescribed me sumatriptan injection and Tramadol tabs and so far it has done nothing at all and she hasn t said much about the rash, I m not sure what to do Doctor: Hi, If your diagnosis is pseudotumor cerebri or IIH as it now called Idiopathic Intracranial Hypertension then, the treatment of choice would be to first identify possible underlying causes such as obesity, overuse of Vitamin A, or the use of certain medications such as Antiglaucoma, Carbonic Anhydrase Inhibitors, Loop diuretics, Cardiovascular, Corticosteroids, Beta-Blockers, Tricyclic Antidepressants, or Antiepileptics. If any of these medications you should ask your doctor if you could go on a trial without them and see if things improve. Otherwise, the definitive medication of choice would be to start with something such as Diamox or topiramate and then, consider other solutions as needed. Your rash may be a reaction to the Sumatriptan or Tramadol (BTW, I'm not a fan of the Tramadol for treating headaches...not terribly effective and risky in terms of side effects AND high percentage of patients have allergic rashes to the drug). Hope I have answered your query. Let me know if I can assist you further. Regards, Dr. Dariush Saghafi Neurologist"
},
{
"id": 13246,
"tgt": "Suggest treatment for rashes and scars on face",
"src": "Patient: i have scars rashes on face i have oily scan .. and in winter its dry .. i am looking for subscision, co2 laser, microdermabrasion in kolkata is it advisable to do .. or i should opt something like any cream or oitment i dont eat oily or junk food .. i have loads of water whn i am in gym .. awaiting response Doctor: Hi, As per your query you have symptoms of rashes and scars on the face which occur when there are hormonal changes, stress and clogged pores of sebaceous gland filled with sebum, dead skin and bacteria get infected. Need not to worry. I would suggest you to make use of product containing benzoyl peroxide. Drink plenty of water daily and add fresh fruits and vegetables to your diet. Take Multivitamin supplements. Apply Adapalene gel once at night. Also wash your face with mild soap and cold water twice a day and exfoliate your face once a week. If condition doesn't start getting better in a month then consult dermatologist. The doctor may prescribe antibiotics course and can also go for chemical peel containing Glycolic acid. Hope I have answered your query. Let me know if I can assist you further. Regards, Dr. Harry Maheshwari, Dentist"
},
{
"id": 67499,
"tgt": "Suggest treatment to bring down sore lump of knuckle",
"src": "Patient: I play softball and I took a ball to the tip of my pinky and middle finger about 2 weeks ago the middle finger seems to be doing better but the pinky now has a lump at the second knuckle on the inside and is still sore when I move it and hurts when I touch the lump.. what could this be Doctor: Hi ! Good evening. I am Dr Shareef answering your query.From your history, an inflammatory swelling an abscess or a fracture/dislocation would have to be excluded. Each of these condition requires different specialist management, and for the diagnosis, this ought to be examined physically by your family physician/orthopedic surgeon in your area along with some related investigations. Therefore, I would suggest you to take appointment with the concerned doctors in your area for an appropriate management. Till that you could take an anti inflammatory drug along with a proton pump inhibitor drug for a symptomatic relief. I hope this information would help you in discussing with your family physician/treating doctor in further management of your problem. Please do not hesitate to ask in case of any further doubts.Thanks for choosing health care magic to clear doubts on your health problems. I wish you an early recovery. Dr Shareef."
},
{
"id": 4823,
"tgt": "Planning pregnancy, taking medicine for ovary cyst, suggested duphaston, clomiphene citrate. Recommendations?",
"src": "Patient: Hi.... Am 24 yrs old n planing for kids .... Since 3 months am taking medicine for simple right ovary cyst....after that my gyny sugst me to have a duphaston n clomiphene citrate tablets for 3 months course... Is there any chances to get pregnent while using these pills?.... Pls suggest me..... Doctor: Hi,Thank you for choosing Healthcaremagic. Clomiphene citrate is used for ovulation induction, but you need regular follow up scan to know the follicular growth and if required you will be given injection HCG for ovulation to occur. kindly continue taking Duphaston, which is a Progesterone.I hope this information has been both informative and helpful for you. In case of any doubt, I will be available for follow-ups. If you like my answers kindly rate it, and write a review as well. Please do not forget to accept it.Thank you,Wish you good health.Regards,Dr ArifYou can consult me again directly through my profilehttp://www.healthcaremagic.com/doctors/dr-arif-n-khan/65133"
},
{
"id": 145274,
"tgt": "Suggest remedy for vertigo while standing up",
"src": "Patient: I arrived in New Zealand from a long haul flight this morning. I am currently suffering from vertigo while standing up and I would really like to know how I can relieve these symptoms fairly quickly without sleeping (I would like to get in to the time zone routine). Thank you. Doctor: If your vertigo is associated with the room spinning (or a sensation of movement even though you are still), and there are no other symptoms associated with this, like weakness, numbness, difficulty hearing, etc, then a trial of meclizine (start with 12.5 mg by mouth twice a day) may be helpful. A scopolamine patch from your physician may also help. If you are having symptoms of vertigo only when you move your head, I recommend going to see your doctor (or an ear nose throat specialist) to have them perform Dix-Hallpike maneuvers, as you may be suffering from BPPD. (benign paroxysmal positional vertigo). If you are lightheaded when standing up but the room doesn't spin and there is no sensation of abnormal movement, you most likely are suffering from orthostatic hypotension and at this point I would recommend trying to stay well hydrated and you can consider also purchasing some over the counter compression garments for your legs."
},
{
"id": 111908,
"tgt": "What to do for severe backpain due to wear and tear arthritis after pain killers, physiotherapy have not helped?",
"src": "Patient: hi i am a female aged 52 i have servere back pain i have had xrays but they come back normal i have had back pain for 2 years but im just being told i may have wear and tear arthritis and to take pain killers and see physio i have done all this but it dosent help Doctor: Back pain after 50 is more commonly due to low calcium. Are you taking any calcium if not than take vitamin D 3, calcium 1000 mg daily. Apart from that you can wear a belt to support your back. As far as physio is concern it may take some time to show it effect, you can take a second opinion from another physio."
},
{
"id": 103099,
"tgt": "Suffering from allergies, pink eye, no drainage or seeping. Taking eye drops containing polyethylene glycol. Is it safe?",
"src": "Patient: Hello my son is 7 years old. He is suffering from allergies. He was sent to the school nurse for a pink eye. There is no drainage or seeping. I was wondering if it is safe to give him eye drops. I have some advanced relief that contain, Dextran 70 0.1% Polyethylene glycol 400 1% Povidone 1% & Tetrahdrozoline HCI 0.05% Doctor: Hello,If there is no discharge of the eye then it may be due to allergy. In that case steroid eye drops with help in reducing the redness of the eye."
},
{
"id": 159590,
"tgt": "Suffering with prostate cancer, spread through bone, remote chances of cure. Better treatment in India?",
"src": "Patient: Dear sir. My father is suffering from prostate cancer . The cancer cell is spreed through bone. Our doctor say that there is no possibility to cure him.Doctor give him hormone therapy .We have done all necessary test like prostate biopsy , CT scan of whole abdomen, CT scan of chest, bone scan. If you could kindly inform me, is there any better treatment in India? Mesbah-ul-alam Assistant Director Department of Environment Dhaka Bangladesh E-mail: YYYY@YYYY Doctor: Since tumor has spread to bones it is impossible to get it cured.By the way have his bilateral architectonic (Removal of Testis)being done.Since you have done all the test and investigations as per advice of oncologist or urologist.There is nothing much can be done except giving a palliative hormonal therapy that currently being given to him.and ovserve his serum P.S.A level frequently.If these metastasis give him severe pain you will have to give him selective radiotherapy."
},
{
"id": 206200,
"tgt": "recommend help for scrupulosity individuals",
"src": "Patient: My son suffers from severe Scrupulosity for a year. He thinks he is blaspheming the Holy Spirt and feels damned. SSRIs are having no affect. His wife is angry at him and says she doesn t think she loves him anymore he gets no support from her only blame. Has also been getting counseling. Where does he go from here?viduals Doctor: DearWe understand your concernsI went through your details. I suggest you not to worry much. You should know that these problems are not mental diseases, but are mental disorders. Many researches and researchers confirm that medicines alone cannot cure mental disorders. Exercise, Life style changes, change in thinking pattern, relaxation etc are as essential as medicines. Psychotherapy can help you changing your lifestyle and thinking patterns. Yoga and meditation help you to streamline your metabolism and neurological balance. Please consult a psychologist for further information.Psychotherapy techniques should suit your requirement. If you require more of my help in this aspect, Please post a direct question to me in this URL. http://goo.gl/aYW2pR. Make sure that you include every minute details possible. I shall prescribe the needed psychotherapy techniques.Hope this answers your query. Available for further clarifications.Good luck."
},
{
"id": 176895,
"tgt": "Suggest treatment for swelling in face after cavity filling",
"src": "Patient: My daughter age 5 was going to have a cavity filled she got the injection in her gums then didn t go through with the filling. This was done over 4hrs ago and her face is a bit swollen with her lipslightly drooping on the side she has the local done. Doctor: Thanks for your query on health care magicFrom history it seems your child is injected with local anesthetic so that she does not have pain during cavity filling.after local anesthetic injection for dental procedure it is common to have some swelling and drooling of saliva. You need not to worry for this because it will go by itself after few hours.Regards.- dr deepak"
},
{
"id": 16734,
"tgt": "What causes chillness in hands and feet after an angioplasty?",
"src": "Patient: My husband had a diagnostic angiography on Wednesday and an angioplasty on the left iliac artery. He developed a red itchy rash which I assume is from the contrast dye. He has developed flu like symptoms, his hands and feet and cold and very painful and his urine output has decreased and is dark, is this to do with the angioplasty? He is 41 in good general health and a ex smoker. Doctor: Hello There After going through your medical query I understand your concern and I would like to tell you that chilliness is not because angiogram procedure, it is because of some urine infection, so it is recommended for you to get a Urine Analysis done and consult a physician with reports for further treatment. Take plenty of fluids Hopefully this information will guide you properly.Kind Regards Dr Bhanu Partap"
},
{
"id": 18049,
"tgt": "Is gastric surgery recommended while having high BP?",
"src": "Patient: Hello, My name is Phyllis, and I am currently trying to decide on weather or not to have gastric surgery because I m over weight. I m very nervous because I have High Blood pressure and just became a diabetic. Are there any oral medications I can take instead of going through with the surgery that will help me lose the weight that are not harmful to my liver and addictive? Doctor: Hi, I understand your concern and would agree with you on the fact that this procedure can lead to a lot of adverse effects, considering your past medical history. For this reason, I would recommend taking Garcinia Cambogia for a period of three months, and make some modifications to your lifestyle (consulting with a dietitian and performing a lot of physical activity). Hope I have answered your query. Let me know if I can assist you further. Regards, Dr. Iliri"
},
{
"id": 143805,
"tgt": "Report from MRI suggest Superior sagittal sinus thrombosis",
"src": "Patient: Does a white upward (point up) Triangular shaped spot in the bottom section of a MRI/MRA image indicate Superior Sagittal Sinus Thrombosis (SSST)? Some picture I also noticed the same thing on the top of my brain but the point to the triangular shape is pointing downwards. I have been having chronic migraines for 7 years every day practically and am lucky if it feels like it s just a bad headache. The people that did my MRI/MRA gave me a disc with special software of my images and there are 60-70 images of this bright white triangular shape in the Superior Sagittal Sinus of my brain. Note: The software with my images is complicated and at first only shows 26 pictures on it, but then I accessed a option on it by accident and found about 300-400 more pictures in categories and subcategories, so I think it would be easy for someone to miss or skip them. Doctor: Yes it may indicate that you have superior sagital sinus thrombosisAlso its more suitable for your daily headache more than migraine"
},
{
"id": 189784,
"tgt": "Swallowed anesthetic during root canal treatment. Nausea. Chances of complications?",
"src": "Patient: I had a root canal finished today I have no pain but I did swallow a little bit of that anesthesia well I think it was that it tasted very bitter and now Im feelin nauseous! and Im looking online for information and Im seeing that there could be alot of complications with root canals...Im so scared. I want to know if it Is normal to feel nauseous ? Im taking amoxicillin 3 times a day I started on Monday and still taking them Doctor: hello and welcome, Local anesthetics when administered in the blood circulation may produce serious systemic complications. As you have mentioned you have swallowed little amount of the anesthetic solution ,nothing to worry it dissolves in the saliva and its effect is minimized , You may feel nauseous because of the bitter taste thats all dont worry. There is no chance of complication since the amount was small. visit your dentist nad get the root canal treatment completed. take care."
},
{
"id": 3358,
"tgt": "Could i be pregnant even if i use Mirena?",
"src": "Patient: I have the Mirena, had it for 3 years now. Lately I ve been having a lot of pregnancy symptoms. I have clear liquid coming from my boob, tired, moody, nausea, head aches, cramps even after the little couple day cycle that I had. The list goes on. Could I be pregnant? Doctor: Hello, and I hope I can help you today.No birth control method is 100 percent effective, so if you have been having unprotected sex, pregnancy is a possibility.However, if you can feel your strings, which means that the IUD is in place, the chance of your being pregnant is still extremely low.In addition, many of the symptoms we associate with pregnancy like mood changes, breast tenderness, fatigue, spotting and nausea can also be caused by an abnormal menstrual cycle. You may not bleed very much with the Mirena, but you do ovulate and you may be feeling premenstrual hormonal changes which can mimic pregnancy.So to be on the safe side, I suggest you purchase an over-the-counter pregnancy test and perform it according to the enclosed directions. Modern pregnancy tests are extremely accurate, and will become positive within 10 days to 2 weeks after conception.If you are not pregnant, you should see your women's healthcare provider as nipple discharge can be a symptom of a breast problem if you have not been nursing recently.I hope I was able to adequately answer your question today and that my advice was helpful.Best wishes,Dr. Brown"
},
{
"id": 71181,
"tgt": "Suggest treatment for severe cough when diagnosed with sarcoidosis",
"src": "Patient: Hi, my apartment was renovated for 7 months i have inhaled drywall dust. NOW i keep coughing up flem and i was told by others,each time I m speaking. I keep taking breaths. I don t notice I m doing that. And also i have sarcoidosis . Please advise me, as to what i can do. Thank you. By the way I m 67 years old. Doctor: Hello and Welcome to \u2018Ask A Doctor\u2019 service. I have reviewed your query and here is my advice. By your history and description, possibility of bronchitis (inflammation of airways) is more likely. Bronchitis is caused by inhalation of dust. And it is commonly seen with sarcoidosis. So better to consult pulmonologist and get the clinical examination of respiratory system and PFT (Pulmonary Function Test) done. PFT will not only diagnose bronchitis but it will also tell you about the severity of the disease and treatment is based on the severity only. You will mostly improve with inhaled bronchodilators (Formoterol or Salmeterol) and inhaled corticosteroids (ICS) (Budesonide orFluticasone). Don't worry, you will be alright with all these. Hope I have solved your query. I will be happy to help you further. Wish you good health. Thanks."
},
{
"id": 94200,
"tgt": "Have food poisoning, cramps in stomach, diarrhea, vomiting. Help?",
"src": "Patient: I believe I have food poising the other day after lunch I started cramping and then I had diarrhea and was vomitting the rest of the night and the next day. This morning I woke up feeling better, but then the cramps and diarrhea came back. Just recently I went to use the restroom and it was like bloody diarrhea. What do I do? Doctor: Hi. You got dysentry due to contaminated food or water. Please take Antibiotic Ofloxacin and Ornidazole ,Antispasmodic Dicyclomine and Pre and Probiotic. You should take Oral rehydration solution . Nothing to worry. It will come down. Wish you speedy recovery Regards"
},
{
"id": 220660,
"tgt": "What does ultrasound scan finding gestational sac not well defined indicate?",
"src": "Patient: My last period was 3rd of may, though my period was irregular before then, but my Dr. was tracking my ovulation as I was desiring to concieve, eventually i missed my period in June, i went for both urine and blood test and they came out positive, I have no symptoms of pregnancy yet except for sore breast which comes and goes and diahorea, i went for utrasound last week and my Dr. said that the gestation sac wasnt well defined, that i should come back this week, i am a liitle bit worrid cause i dont understand what he really meant Doctor: Hallow Dear,When pregnancy test is positive but still the ultrasonography report comes as 'Gestational sac not well defined', there is a possibility of absent embryo in the sac. This is called as Blighted ovum. You may repeat the scan again after about a week or so. If empty gestational sac is seen, the diagnosis is confirmed. In Blighted ovum, the pregnancies do not continue and grow. If it does not get aborted spontaneously, you may have to get it terminated by medicines. Your Obstetrician will take the decision after repeat scan. I hope this helps you. Dr. Nishikant Shrotri"
},
{
"id": 122933,
"tgt": "Is there option other than surgery for osteoarthritis?",
"src": "Patient: Dear Sir, My wife (44 years) is suffering from osteoarthritis. She is working woman. She can walk but use of stare case is painful. Doctor suggested to go for HTO surgery for both the knee. Kindly advice is there any option for this surgery. Thanks a lot in advance. Doctor: Hello, As she is just 44 I think with the exercise she can again become better and back to normal life. Usually, in Osteoarthritis (OA) of knee there are degenerative changes in the bones and due to which there will be the weakness of the muscles of the knee leading to abnormal gait (walking) pattern and by which the joint compression. Doing exercises like - static hamstring, static Quadriceps, straight leg raise, reverse walking, improving the knee Range of Motion will be beneficial. In my clinical practice of over 12 years, most patients at her age respond well to physiotherapy and exercise under guidance. Hope I have answered your query. Let me know if I can assist you further. Regards, Jay Indravadan Patel, Physical Therapist or Physiotherapist"
},
{
"id": 134893,
"tgt": "Does a small crack in the metatarsal of the bone not require surgery?",
"src": "Patient: A week ago I fell on my left foot at a friends house. I could not put weight on my foot or move it. It hurt on the inside, the outside, on my big toe, and my ankle area. I waited a day to see if it would feel better but it didnt so I went to the orthopedist. They sent me to X-rays and did a weight bearing view and a top view xray. The doctors did not see any obvious fractures but they did see a small crack in my 4th metetarsal. They said it was natural and if didn t have to do with the injury. They decided that I had deeply bruised my foot so they put me in a walking boot and crutches. After a few days on the crutches, I switched to the walking boot and no crutches. Now, 2 weeks later, my foot hurts worse and hasn t improved at all. It hurts with the walking boot and without. My mom refuses to take me back to the doctor but I am becoming worried that it is getting worse. I am icing, elevating, and resting my foot but nothing seems to help. I need some advice about what to do and I was wondering about that crack in my bone that the doctors saw. Please help. Doctor: helloI think doctor may have advised to avoid weight bearing for 4-6 weeks and not to discard crutches to keep the weight off the foot on walk even when walking boot was borne, for which you may consult again with the treating doctor.In my opinion defer weight bearing for 2-3 weeks, keep foot elevated, apply local anti-inflammatory gel like voltaren and just do ankle and toes drill with foot off the ground , exercise for several minutes many times a day, it may settlethanks"
},
{
"id": 195450,
"tgt": "What causes burning sensation after urinating and sex?",
"src": "Patient: I am a male and have burning sensation after urinating and sex. I also have a small bit of discomfort on my lower abdominal and dont have a noticable discharge, but do feel like i am constantly wet. I have test negative for STD's ( particular Chlymedia and Ghonoria). My symptons had started in February, shortly after i had unprotected sex. I had consulted 2 docters that had both indicated that i might be just an irrative bowl and blood test do show that i dont have any infection (red and white blood cell counting). I can def feel that something is not right and dont know how to re-explain it to determine to cause for my concern or for what other possible infection i can test for. What else can cause the indicate symptones and for what other diseases can i ask to be tested for? Doctor: Hello and Welcome to \u2018Ask A Doctor\u2019 service. I have reviewed your query and here is my advice. The most common cause for burning and itching or pain after sex or urination is lower urinary tract infection which includes prostatitis,urethritis or cystitis, i would recommend you to get your urine complete examination done and see if it shows any infection but in most of the cases the burning is due to urethritis which would not show any infection on urine complete. I would start you on antibiotic from quinolone group like levofloxacin for one day along with cranberry extract juice to reduce the burning. Hope I have answered your query. Let me know if I can assist you further."
},
{
"id": 135604,
"tgt": "Suggest treatment for meniscus tear on knee",
"src": "Patient: MRIs done on both knees. medial meniscus tear on one knee and floating cartalidge on the other. i have an orthopedic surgeon appointment soon but I m trying to get general idea of treatment options and time needed to be off work. i will know more after seeing surgeon but i am just trying to get ideas til then. Doctor: Hi Dear,Welcome to HCM.Understanding your concern. As per your query you have meniscus tear on knee . A torn meniscus is one of the most common knee injuries. Any activity that causes you to forcefully twist or rotate your knee, especially when putting the pressure of your full weight on it, can lead to a torn meniscus. As your have already have MRI . Doctor may recommend physical therapy , which will be very helpful in this case , icing and proper rest . Doctor may prescribe analgeics and anti inflammatory . In case od serious injury doctor may recommend surgery . Sometimes possible to repair a torn meniscus . If the tear can't be repaired, the meniscus may be surgically trimmed. Surgery may be done through tiny incisions using an arthroscope. After surgery, you will need to do exercises to optimize knee strength and stability.Hope your concern has been resolved.Get Well Soon.Best Wishes,Dr. Harry Maheshwari"
},
{
"id": 130227,
"tgt": "suffering from pain and bruise in the shin area",
"src": "Patient: About two weeks ago I was in a snowmobile accident. I got thrown and hit my shin just below the knee cap. I did go for xrays and was told I severly bruided the bone....I have a goose egg on my done and that hurt (understandably) but this week my ankle has started to swell a little and the bruising is horrible (dark blood red/ yellow). It is normal for a major injury to cause the bruising to go down the leg? Doctor: Hi i am Dr Ahmed Aly thanks for using healthcaremagic site ,I had gone through your question and understand your concerns .. Sometimes it happens that the edema or the inflammation goes down making swelling but that happens when you stands a lot or bending your knees while sitting thus causing less blood flow back to your heart from your venous flow . I recommend elevating your ankle above your abdomen level as much as possible while taking your anti edematous drugs like alphintern tab 3 times daily before meals , NSAIDs painkillers like advil tab or aspirin , hot massages with topical gels , avoid standing and sitting for a long time , avoid bent knees , will be effective for now if swelling persists i suggest further investigations like Doppler U/S on your venous system of your swelled leg for proper management . Please click and consider a 5 star rating with some positive feedback if the information was helpful. Wish you good health,Any further clarifications feel free to ask."
},
{
"id": 94099,
"tgt": "Drowsy, shaky, depressed, uncomfortable abdomen. Urine dark yellow. Suggestions?",
"src": "Patient: been drowsy,little shakey, little winded and depressed the past month ( mostly because I cznt understand my problem). Very minor abdominal uncomfortableness. Some uncomfortableness in urethra . I checked out ok for uti . And blood work came back normal. No lower back pain . Most of my uncomfortableness is on lower left side right under rib cage. Also my urine is dark yellow and musty. Im 32, 225 lbs. I just srarted a job last nov. Labor intensive. Going to make another dr. App. Doctor: Hi welcome to Health care magic forum. Thanks for calling H.C.M.F. You are drowsy, shaky, winded, uncomfertable in the lower left side under the rib cage. Uncomfertable in urethra. urine is dark yellow colored and musky. You are 32, 225 lb.New job, with intense labour. All these suggest some thing concerned with the urinary tract, or gastric irritation. I advise you to consult a urologist, for diagnosis and treatment. You may have to undergo M.R.I. gastroscopy besides other routine tests for confermation. Due to hard work in the new job it could be psychological as well. I advise you take plenty of water, bland foods with less spices, avoid taking junk foods, and oily foods. Wishing for a quick recovery. Best regards."
},
{
"id": 70833,
"tgt": "How long does it take for pneumonia to be cured?",
"src": "Patient: I was given amoxycillin for a chest infection and did not improve after 48 hours, therefore I went back to the GP who then gave me Clarythmomycin as it resulted in pneumonia. I have been off work for 3 weeks now and feel completely washed out and exhausted. I had a chest x-ray which was clear, but was given an inhaler temporarily as I was out of breath and wheezy. I still have an incredible amount of phlegm which has now changed from brown to sometimes green to clear. Unfortunately, for last 3 days, my trachea is extremely bruised to touch and feel as if someone is pressing my windpipe. Although my tonsils aren t swollen, my ear hurts deep inside. I have had to start taking paracetamol again this week as feel weak, wobbly and feel as if I am going backwards again instead of improving. I am 45 years old. Non-smoker.This is the 3rd time I have had pneumonia in 18 years. I am reasonably fit and healthy, and I don t suffer from colds normally, but whenever I come down with something, I seem to suffer with it. How long does it take to get back to normal? Are the trachea symptoms part of the initial pneumonia symptoms or would I need to see the GP again. I am feeling extremely fed up of generally feeling unwell. I am taking a tonic (Metatone) and drinking and eating healthily. Basically, is it normal to feel this unwell for so long. Doctor: Hi, Yes, it might be that after the pneumonia the recovery time lasts up to one week however I recommend you to do a chest CT to see if the pneumonia has gone completely. Hope I have answered your query. Let me know if I can assist you further. Regards, Dr. Jnikolla, Pulmonologist"
},
{
"id": 21837,
"tgt": "What is the treatment for adenoid problem in a child?",
"src": "Patient: My son is suffering from breath and hearing problem. Dr advised adenoid X-ray. It shows the adenoid problem. Dr told it is around 75% advised one month medicine,repeat X-ray and advised surgery if result is same. We had X-ray , which shows the same result. My question is is it curable with medicine. Doctor: Hello. it sometimes happens that the adenoids get enlarged which affect breathing and hearing by blocking. in such case, we try reducing its size with medicines at first, but if medical therapy has failed, we go for surgery.i believe your doctor has already tried medical therapy but it didn't work hence surgery would be the only option right now.i would like to assure you that it's a very safe and common surgery and I'm sure your doctor will help you with any other questions you might have"
},
{
"id": 144913,
"tgt": "Suggest treatment for protrusion with L5 nerve root displacement",
"src": "Patient: good afternoon, i have had 3 back surgeries last one resulting in a L5-S1 fusion, i am now experienceing left leg numbness from knee down. had an MRI and it shows severe moderate protrusion with L5 nerve root displacement. is this potentially causing nerve damage / is surgery my only option? I see a surgeon next week just preparing myself Doctor: I read your question carefully and I understand your concern.Unfortunately fusion while it may improve the symptoms on that level doesn't help with other levels, it can even accelerate their damage because the reduced mobility in L5-S1 after the fusion adds the strain on L4-L5. Hence it is not to be surprised to have protrusion on L4-L5 with displacement of L5 root. It does justify your symptoms, although to be sure about that an exam is needed (reflexes, muscle weakness, numbness location) to confirm symptoms correspond to that root and not to remains of old problem.As for remedy usually it is attempted with physical therapy and pain killers, local injections can temporarily help as well. If those are ineffective and symptoms persist surgery can be considered again.I hope things work out for the best."
},
{
"id": 157728,
"tgt": "Started hemmorraging, history of breast cancer. Treatment?",
"src": "Patient: Hi,Im 26 years old and had my first child a year ago now,this past july out of the blue I started Hemmorraging really bad,by the time I got to the hostpital I had clots the size of my fist(it was the scariest thing tha has ever happened)they did a bunch of test ,nothing was found,nothing!I cant help to wonder why that happened to me and why nothing was found!My docter just kinda quite giving me test ,basically said it was nothing,its hard to just forget about it!My mother was diagnosed with breast cancer in January (she beat it)but she thinks maybe that has something to do with it!Is there any information anybody can me ,maybe thing wernt looked into enough Doctor: Dear ,you have history of breast cancer, possibly must have undergone adjuant chemotherapy following which you had pregnancy so you must have developed two way amenorrhoea, one chemotherapy induced and second lactational.Basically you are a young lady with active hormonal status. Once the menstrual suppressant factors waned you have started with fresh bleeding.S ince there must not have been a good passage to the menstrual flow it landed into clot formation. After one or two cycles of this clotted menstrual flow you should have normal flow with certain supportive medications to be prescribed by your doctor. I don't know what was your hormonal status after breast surgery. Normally at this age of yours breast cancers are aggressive because of the active hormonal status. You may need to undergo artificial menopause so that there is no recurrence of cancer or or there is no new cancer formation.Please seek the advice of a good clinical Oncologist who should guide you the future best course of action.Truly,Dr. Ticku"
},
{
"id": 127396,
"tgt": "Does a herniated disc cause spasms in the pelvic area?",
"src": "Patient: I have been diagnosed with a herniated disc that is causing spinal cord compression. The neurosurgeon says it is very serious and he s surprised from the mri that I m not in more pain than I m reporting. I m having surgery for this in about three weeks. Today I notice a light vibration or spasm in my pelvic area. I m curious if this could be related to the cord compression. Doctor: Hello and Welcome to \u2018Ask A Doctor\u2019 service. I have reviewed your query and here is my advice. Spasm is not a typical presentation of disc herniation. However, it can be present secondary to the severe pain people experience in disc herniation. In your case though, spasm is highly unlikely to be due to the disc disease. Wishing you a fast recovery after surgery. Hope I have answered your query. Let me know if I can assist you further."
},
{
"id": 11111,
"tgt": "Suggest treatment for hair fall",
"src": "Patient: I have been suffering hair loss for the past five years. I am just 28 and the centre part of my head has almost gone bald. This has become very stressful in life, since I am yet to get married. No medicine works on treating my hair. Can you help me with some remedy Doctor.. Doctor: Hi,After reading history you provided,I can conclude that most probably you may be having androgenetic alopecia. You consult dermatologist for firm diagnosis.I would recommend as follow...- tab finesteride 1 mg daily- minoxidil 10% solution to apply twice a day on bald area- tab biotin 10 mg twice a day- shampoo with ketoconazole lotionContinue treatment till good result.I hope this would help you.Thanks.Dr. Ilyas Patel MD"
},
{
"id": 99508,
"tgt": "Is Nebulizer advisable for coughing and chest tightness in a smoker?",
"src": "Patient: hi my mom is 67 years old and have been struggling with this cough and tight chest for 3 weeks now, she has been to the doctor and been on 2 sets of medication and the cough has not improved,however she is a smoker.i want to know if she can use a nebulizer she is quite healthy only a bit of high blood. Doctor: HI, thanks for using healthcare magicThe nebulizer may help if the cough is related to increased hyperactivity of the airways.This may be the case since the coughing is persistent and associated with chest tightness.Smoking will continue to aggravate the cough and worsen any symptoms present so she may want to consider trying to stop.I hope this helps"
},
{
"id": 163962,
"tgt": "Does being drenched after high fever cure fever?",
"src": "Patient: my son has been running a fever since saturday. (101 plus) and last night i went to check on him and he was drenched. now has a normal temp. could the illness be gone? mind you we have been exposed to people having influenza type a. i was planning on going to doctor today. Doctor: Hi...I understand your concern. Drenching sweats after high fever is quite common. Nothing to worry about it. But as you are concerned the exposure to people with flu symptoms, I will suggest the danger symptoms when you need to worry and seek medical attention.Respiratory distress, fast breathing, excessive cough, diarrhea leading to dehydration, vomiting and decreased fluid or food intake. If these are not there you need not worry.Regards - Dr. Sumanth"
},
{
"id": 49726,
"tgt": "Chronic kidney stones, cystitis, endometriosis. On oxycodone. Trying to wean off. Safest way?",
"src": "Patient: Hi I am dependent on prescription pain medication and very badly would like to quit. I started using pain meds due to chronic kidney stones, interstatil cystitis and endometriosis. I have had to increase my dosage because of tolerance. I use up to 180 mg. Of oxycodone daily. Those are the heavy days. I have been using for about two years now. I would like to get a dr. To prescribe me medication to wean me off these meds at home. I have a psychiatrist i have mentioned this to but he said i needed to go to rehab. I cannot afford rehab and do not have insurance. I have a masters degree in counseling and have learned all the mental health aspects of addiction and what i need to do to stay clean after detoxing. Im just scared to death of withdrawal symptoms as i have felt the onset of them before. Any help would be greatly appreciated. I am a 28 year old female who weighs about 108 lbs and is 5 foot tall. Doctor: Hi,Your psychiatrist perhaps advised you to go to rehab because quitting this high dose of oxycodone abruptly can be expected to result in strong withdrawal symptoms which would be easier to manage if you are under constant medical supervision. If going to rehab is not an option, then rather than quitting this abruptly, try reducing the dose of oxycodone gradually before quitting. Go very slow, not more than 5 mg per week, and even slower if you wish. Keep yourself hydrated. Use over the counter medicines if there are mild withdrawal symptoms. And above all, remember that one may easily die from opioid overdose but not from opioid withdrawal.Hope this helps you. Best wishes."
},
{
"id": 224865,
"tgt": "Butterflies in stomach, breathlessness, always tired, headache. Side effect of removal of IUD?",
"src": "Patient: I ve taken my merina out on October 10. Had my period on October 21. Been daily active. The past 4 days it feels like I have butterflies in my stomach , shortness of breathe, and tired all the times. (plus headaches )There s no pain just irritable. Is this a side effect of the removal of my IUD? I have four children and never have experienced this before Doctor: HIThank for asking to HCM I really appreciate your concern, this is not the side effects of removal of IUD but this seems functional feelings and for that no need to worry try keep the stress level low and stop thinking negatively, hope this information helps you, have nice day."
},
{
"id": 52127,
"tgt": "I am having problem in passing urine, please suggest me the remedy for it",
"src": "Patient: I am having problem in passing urine. I am takaing Urimax 04 mg in the morning and was taking Roliflo in the evening. Now that Roliflo is not available, I was prescribed Trofame XR cappsuslels but these capsules perhaps do not suit me as it seems to block urine free flow. Which medicine I should use in its place Thanks Doctor: Oh are you having Prostate issue. If you are not gettin help frm medicines, then you will need to undergo surgery as the ultimate treatment. But with timely medicines, they say that surgery can be prevented."
},
{
"id": 137926,
"tgt": "Suggest remedy for numb & itchy patches on foot",
"src": "Patient: I have a numb patch on the left side of my right foot under the ankle bone. It is extremely red, and itchy.. Its almost as if its a giant mosquito bite it itches so much.. but its not. Its one of those spots that if I barely scratch it while putting on my shoes it becomes inflamed and itchy. What could it be and how could I make it go away? Doctor: Hi,Thanks for contacting HCM. An area of the skin that is red, patchy, and itchy can indicate a skin fungal infection. Over the counter topical anti-fungal creams can help clear fungal infections, but at times prescription strength may be necessary.I hope this helps answer your question.Thanks."
},
{
"id": 225893,
"tgt": "Got marina coil inserted. Having heavy bleeding. Taken cetezette. Feels itchy and have mood swings Suggest",
"src": "Patient: Hi I was recommended by s gynecologist to have the marina coil fitted. Due to me having non bleeding PCOS I got this fitted 5 month ago been experiencing a lot of bleeding. I also got cetezette to try stop the bleeding but since coil been inserted I been crying at silly things n getting annoyed at nothing . But since been on cetezette as well as the coil my mood s are crazy like jeckel and Hyde. After having mad moment I then start to cry also I know feel my full body itchy. Doctor: Hi,Thank you for posting your question here, I will try to answer it to the best of my abilities.IUD's are known to cause mood swings, it is not something new, so that could be the reason you're feeling the way you are.Contraceptive pills, also cause mood swings and well it seems that both of them working together are really getting to you.As such you should consider having the IUD removed.I hope this answered your question."
},
{
"id": 23681,
"tgt": "Suggest medicines for high BP",
"src": "Patient: hello doc, My mother,55 has been a hypertensive from the past 12-13 yrs.She was started on amlodipine 2.5 and she did fine for 3-4yrs when she started having acute cough.Amlodipine was changed to Nifedipine and the cough subsided.For another 3-4yrs she was doing well when her legs started becoming edematous.Nifedipine was then changed to hydrochlorthiazide and her BP has been perfect until the past 6 months when it shot to 145/90.Inspite of increasing the dose there was no difference so she was kept on telmisartan40mg+hydrochlorthiazide.Even now the BP does not seem to come down and it shows a value of around 145/90.She does not have any other complaints (except for urinary incntinence for which she has not yet started any medication),her heart and kidney profiles hav also come out clear.Can u suggest a gud combination and is the BP at an alarming level? thank you Doctor: Hi,You can increase telmisartan dosage from 40gm to 80 mg along with hydrichlorthiazide.Take care"
},
{
"id": 176716,
"tgt": "Suggest remedy for discomfort around anus",
"src": "Patient: Hello XXXX My son 8 years old, had complained of discomfort around anus. He felt like he needed to pass bowel, but after straining a bit it didn t come. He is constipated. There was tiny swelling just on the lower end of his anus at that time and it was feeling extremely itchy and distracting. After another toilet visit 1h later stool didn t come, but the little swelling from the anus disappeared. Couple of hours later whilst having shower, I observed that area leading from his bottom towards scrotum sac is swollen. Skin is in normal condition no redness and he said he hasn t been injured there. He said it feels strange but not painful. What should be the cause? Doctor: Hi,From history it seems that due to constipation and giving pressure for evacuation of bowel there might behaving soreness in anal region giving this problem.There is another possibility of having worm infestation.Give him light laxative at night for few days.Give her plenty of water.Give him more fiber containing food.Ok and take care."
},
{
"id": 214352,
"tgt": "Suggest natural method to regain lost hair",
"src": "Patient: Sir ,I am vikas,i am 23 years old.from last two year i hair falling and still falling.eactualy i have a very few hair in front.Is there any possibility to again dense neturaly and how much time it will take.suggest some best doctor in north-east resigon. Doctor: Yes it is possible to regain your lost hair only if r dedicated and willing to spend money.Chances are more that you regain your hair because you are young. Search a Good and Honest Ayurvedic physician around your area who practices Authentic Ayurved with Panchakarma.Firstly, take cleaning therapy called Vaman .Then Nasya for 21 days, simultaneously Shirolep for same period.only Then start internal Ayurvedic medicines and Oil applications.I can assure you, if you follow above things, which will tk around 2 months, 3rd month onwards you will see regeneration of few hairs . Continue the treatment along with Rasayan Chikitsa for a year.Regards,Dr. Prasad Parabnavjeevan. ayu@gmail.com"
},
{
"id": 84924,
"tgt": "What causes acne breakout after taking Wysolone?",
"src": "Patient: hi sir!!!! my mother is using wysolone tablets since 4 weeks now she gettng small pimles and achne around her forehead and ears almost around her face n she is also reportning that she is feeling hungry even after taking meals.......plz suggest me a repli Doctor: Hi, This is a common effect of steroids. Lowering/adjusting the dose can be helpful or using an alternative anti-immune drug such as methotrexate but cannot say in your particular case, but generally, there are adjustments. Hope I have answered your query. Let me know if I can assist you further. Regards, Dr. Matt Wachsman, Addiction Medicine Specialist"
},
{
"id": 23650,
"tgt": "Suggest treatment for pain in heart area",
"src": "Patient: I had an EKG done today because I was complaining to my doctor about palpitations / brain fog. The EKG showed that I might have an enlarged heart. I'm currently experiencing some pain localized around my heart. I have a history of partial pneumothorax in my lung on that side. Should I go to the ER? Doctor: Hi Dear XXX! I have gone through your question and I understand your concern . According to what you write and the fact that you have the pain , Yes you should go to the ER. It's necessary to rule out any heart problem the one that I'm concern about we call it angina pectoris . You have an enlarged heart and also palpitations , foggy brain the doctor will do an ECG to find if the pain comes from the heart.After the doctor rule out heart problem , it has to confirm if you have again pneumothorax with a chest X- ray .Meanwhile relax , don't do any physical effort that can worsen the pain.Wish you good health!Thank you for using HCM If you have any other questions please feel free to ask me ."
},
{
"id": 67150,
"tgt": "What causes recurring lumps with mucus in the throat?",
"src": "Patient: Hi, I am 21 years old and recently I got a bump in the inside of my bottom lip but it didn t hurt at all. And then the next day I got a red bump on the back of the right side of my tongue. It was red and it did hurt. It went away the next day and that night another one formed but on the opposite side. I feel like I have a bit of mucus in my throat. The bump feels a bit sore but I can feel it going away. The only way I can think of catching an infection is when last week I went to a party and we were all sharing the same shot glasses? What could this be. I m not sure if my throat is sore. I think my head is just playing games with me or I m feeling stressed. Help! Doctor: Hi.Thanks for your query and an elucidate history.The cause of the lumps in the lip and the back of the throat and mucus look to be due to aphthous like ulceration which are coming and going faster than normal duration of 3 days.You may get this if you have colitis, You had a party of sharing shot glasses last week and such infections do not occur so late. I would advise you the following:Take a course of Metronidazole if you have colitis. Start taking ProbioticPlenty of oral fluids and water frequently , this has a cleansing action too.Consult an ENT Surgeon to get a check-up done to rule out any internal problem."
},
{
"id": 76950,
"tgt": "Can bronchiectasis be related to pneumonia?",
"src": "Patient: I was diagnosed with bronchiectasis four years ago. Since then I have had pneumonia four times. Last week I was exposed to a lot of outdoor smoke and my chest and throat felt raw. I had pain in my ribs and that was made worse by touching. No fever or sweating. Saw my doctor and he xrayed two days ago. Has not said anything but i wonder if it was a bronchial attack or heart involvement. I continue to have a dry cough when outside or at night time. Doctor: Hi welcome to HCM...You had bronchiectasis before 4 year.In that by obstruction and so collection of secretion chances of infection more.So you had developed pneumonia thereafter.Just now after exposure to smoke you can have bronchitis and so on cough...If wheezing and dyspnea present then bronchodilator needed.Otherwise just Antihistaminic taken...Avoid smoking and air pollution like respiratory irritant....For congestion relief eucalyptus lozenges helpful...Echinacea herbal product that resist infection and improve immune system.Take care.If condition worsen then chest x ray needed.Dr.Parth"
},
{
"id": 223304,
"tgt": "What precautions need to be taken after an IUD insertion?",
"src": "Patient: I have been prescribed three Misoprostol tablets to take tonight and tomorrow morning before having an IUD inserted tomorrow. I m supposed to go on vacation (pool and hot tub) in three days. I remember some directions about not taking a bath after taking this prescription the other time (before an invasive sonogram thingy) but don t remember for how long after taking that medication I was at elevated risk of infection. Please advise. Doctor: hello user,Iud is an effective contraceptive..IUD does not provide protection against STD infection..for preventing std infection you need to use condoms..if there is severe pain or severe bleeding you can consult your gynecologist..thanks."
},
{
"id": 173663,
"tgt": "What causes headache, tiredness and stomach pain inspite of having epilepsy?",
"src": "Patient: epilpsy my daughter has been diagnosed wit epilepcy she is 9 yrs old. she seems to be realy tierd and she has a lot of headaces n some times she looks real sick n her stomach hurts n she feels nausues n dosent wanna eat. she gets dizzy n has a fever some times she been to the dr n they gave her antibiotics n these same symptoms keep coming back. she is on her medication for epilepcy. Doctor: HiI completely understand your concern.At the outset I would like to know about the medicines your daughter is taking,since how long and have the drugs been stepped up or down.Anyway it's common for children on anti epileptic drugs to get such symptoms. It's mainly due to lack of adequate sleep as children on and need minimum 8 to 10 hours of peaceful sleep, the lack of which not only causes such symptoms but also can trigger a seizure. The stomach pain can also be a part of her condition because there is also an entity called abdominal epilepsy which can have the same symptoms. An ultrasound scan of the abdomen might be useful to rule out any other pathology. Please consult your doctor again and clarify regarding these. If you have further questions I would be glad to answer. Dr.V.S.Sri Vidhya Chennai"
},
{
"id": 79003,
"tgt": "What causes sternum pain?",
"src": "Patient: Dear Doctor, My wife 29 yr. old having Sternum pain last one year, we made X-ray, ECG, tracemill test. but she found OK. but pain not going, now 2 months she start sneezing frequently. more worse is in the morning and after shower. She had mild headache also. Please advise. Doctor: Thank you for writing to Health Care Magic.I think that you have chest pain. There are a few possible causes of chest pain but the one to be most worried about is related to heart related chest pain. Since the ECG did come back negative, I would recommend that you get a Stress ECG done or a treadmill test done. This will help you get to the cause. If that is negative, I would think that you have a chest or lung infection. We will need to follow up to confirm this.Hope this helps you."
},
{
"id": 131302,
"tgt": "Suggest treatment for mild pain in the shoulders",
"src": "Patient: About a year ago, I started feeling a mild pain when I would shrug my shoulders, rolling my collarbone forward. It is only on the left. I thought the pain was coming from the clavicle itself but now I realize it must be the muscles through my neck and shoulders, too. I didn t address it because I assumed it was from sleeping funny and would resolve on its own. Instead, a year later the pain has gotten worse but still only when I am making a lifting, shrugging motion. It s probably not anything so serious but my dad had thyroid cancer and good old web md listed that as a possibility of this type of discomfort. I will make a doctor s appt but in the meantime would appreciate input. Thanks. Doctor: HiiiIf the symptoms are continuously getting worse then please meet your physician as early as you can. If your physician say that this is just muscular spasm or pain then with your history I can revealed that you nay have many trigger points on your left side collar and neck muscles. The following are the treatment method you can choose:Self Treatment :- Take any tennis ball and then roll the same on your collar muscles with moderate pressure. Repeat 15 - 20 times. After that you may have some soreness that will be relieved within one or two day. It can give you good relief but can't cure completely.Physiotherapy:- There are many techniques are available in Physiotherapy to relieve the same pain like MFR, Myokinetic Release, IASTM and Fascial manipulation. But the best method to cure the same completely is Dry Needling Technique. Please visit only well experienced therapist if you choose Dry Needling as a treatment choice.Thank You"
},
{
"id": 6991,
"tgt": "How long should we have sex to get pregnant ?",
"src": "Patient: hi doctor!good morning.i am 19 Yrs old.My sex is happened on jan 15 about 7 minutes.It was happened after 15 days period.but after sex,9 days before periods came.Again now normal.Can i get pregnancy at any time.Or otherwise if i have sex second time again about 10 minutes can i get pregnancy.To get pregnancy about how much time i have to spend for sex.Please tell me doctor. Doctor: Hi lucky.Welcome to HealthcareMagic forum.usually pregnancy occur around the time of ovulation.this is usually mid way in the cycle.period after sex usually rule out pregnancy but not 100%.duration of sex has nothing to do with pregnancy.it is ejaculation containing sperm is required."
},
{
"id": 107449,
"tgt": "What causes severe pain in the left side of the back?",
"src": "Patient: I am having a lot of pain on my left side of back right under rib consent with shooting pain going down side towards my abdominal could that be my kidney that s where the pain is plus am really fatigue been sleeping night and day for 4 days to where can t hold my eyes open Doctor: Hi,In my opinion your pain maybe due to a renal underlying cause like a stone or a gravel in your ureters or a muscular spasm. I don' t actually think your eye problem is correlated to your back pain. You may ask your physician for some investigations like urine analysis and U/S to exclude any renal pathology . For now I suggest you increase your water intake till your results arrive and avoid high proteins and salts meals. Please consider some positive feedback if the information was helpful. Hope the above information helps you,Any further clarifications feel free to ask.Regards,Dr Ahmed Aly Hassan"
},
{
"id": 211672,
"tgt": "Depression, phobia of exams, difficulty to get along with people, daydreaming, loneliness, anxiety, mood swings. Help",
"src": "Patient: Hello sir, I am summit from haryana. Sir for the past 3 yrs I have depression just for the reasons.. . .1. I am not gud luking2. I have exam fobia3.I am not able to talk in crowd I am lost in my self imaginations and not able to focus in class.The main reason is that I feel myself alone .... And u must b knowing when a person thinks that he is alone he enters in the world of fantasies by those imaginations I do some wrong things. All time I feel romance... and dreams also comes of romance.. and I am not able to talk girls in college.just beciz of depression ,anxiety and all time I feel like I m gonna to cry... .I have much mood changes... and don't like to talk with my fronds. Finally ,I m very physcho ... depressed.... alone ...this is becoz nobody likes me ... I am not that smart .. gud at health...and I need someone who loves me and I need someone to love. Specially girls ... please help me out. Tell me the best medicines ayurvadic.and I am very tensed about my future and my wife....bcoz I thinks that she will only love and take care of me. Doctor: Hi there, thank you for explaining. If you do some sport, it can help to boost your self esteem. Many girls are not very pretty, but when they work on their body, they look attractive. It helps for depression too. I wish you a good health."
},
{
"id": 115085,
"tgt": "Is rise in white blood cell count dangerous?",
"src": "Patient: Its this dangerous my white blood count is 11to 14 for the last 2 months my pulse rate is 94 and blood pressure is 136/82 I ve been running a tempature of 99.5 regularly and I m dizzy and confused sometimes and also I have been having sever stomach pains on my right side and headaches that won t go away and the doctors ruled out pendacidas and bladder infection is their something wrong with me and also my right side of my stomach has a bruise and I m shaking. Should I be worried? I m only 21 years ol Doctor: Hi, dearI have gone through your question. I can understand your concern.Your total wbc count is sligh high. No need to big worry. It is common in bacterial infection. You should go for ultrasound abdomen to search the cause of pain and you should take a course of antibiotic according to cause of infection. Consult your doctor and take treatment accordingly.Hope I have answered your question, if you have any doubts then contact me at bit.ly/Drsanghvihardik, I will be happy to answer you.Thanks for using health care magic.Wish you a very good health."
},
{
"id": 3237,
"tgt": "Suggest remedy for getting pregnant",
"src": "Patient: hello doctor This is sravani.. i am 25 years old. married of 1 & half year.. i have an irregular periods.. and also have un wanted hair under chin, chest etc...i consulted gynacologist.. she said it is PCOD.. and i am planning for pregnancy.. she suggested me OVAFORCE tablet for one month..to use from 3rd day of menstrual cycle.. is it enough to get pregnant sir ? Doctor: Hello dearI understand your concernPCOS is very complex disease and cause hormonal imbalance, insulin resistance, menstrual irregularity and infertility problemJust OVAFORCE does not help as yo have menstrual irregularity.I would suggest to authorized gynecologist and undergo reproductive hormone analysis and USG scan for better management.Following additional treatment will help youMetformin: for maintain glucose homeostasis and weight, induce ovulationProgesterone pills: regular the menstrual cycleTopical eflornithine: for abnormal hairClomiphene citrate can be used to induce ovulation and maintain fertilityDiet: take small frequent meal rather large mealAvoid fatty/spicy/salty foodsAvoid excessive sweetsStart regular exercise for 45-60 minutes in the morning and reduce 10-12% body weight if you are overweight.Hope this may help youBest regardsDr. Sagar"
},
{
"id": 25111,
"tgt": "Suggest remedy for heart ailments",
"src": "Patient: First, I failed the stress test. The Doctor seemed very concerned that my heart rate was so high after 2 minuets on the tredmill. My question is that after doing a cardio exercise for about 1 minuet, my heart rate went to 150 and there was no immediate ache but at some point durning the night, I had a dull ache in the heart. Could there be any other concern? I had the procedure where they went in to check for blockage. There was none. My age is 46 and I am approx 275lbs Doctor: Hi,welcome to HCM..Thanks for writing to us.First of all, you failed in the stress test, that means the TMT test was positive for inducible ischaemia.So there is some concern.There are certain risk factors for the development of coronary artery disease(heart attack),these are:1.waist circumference >40 inches(men)2.triglyceride>150mg /dl3.hdl cholesterol4.BP>130/85 mm of hg5.fasting blood sugar>110 mg/dl If 3 or more risk factors are present in your case. chance of developing CAD is much higher. So,you should first go for the following tests:1.lipid profile2.fasting blood sugar3.complete blood count4.EKG5.Repeat ANGIOPGRAPHY(procedure to check for blockage) if necessary.Hope I have answered your query.Stay fit,good luck."
},
{
"id": 208104,
"tgt": "How to get rid of depression?",
"src": "Patient: After 3 years of my relationship i had a breakup with my boy friend. and now I don t know how to move on in life since last six months I ve been depressed like hell... Can t even concentrate on my work.. I can t even like anyone forget bout love.. dun knw wt to do.. Doctor: Hi,I am sorry to read about your breakup and resulting symptoms that you are facing. You seem to be depressed and if even after 6 months, the symptoms are persisting, you should consider starting treatment for the same.You should seek a psychiatric consultation for the same. Ant-depressants like escitalopram or sertraline will be highly helpful in your case. In addition, you should start exercising daily. It will help you relax and feel better. Additional option of treatment with supportive psychotherapy can also be considered. It will consists of multiple sessions focusing on letting out your emotions. You can discuss with your psychiatrist regarding the same.I do hope that I was able to answer your query. Best wishes."
},
{
"id": 5259,
"tgt": "Trying to conceive. Started bleeding after periods. Taking insuling fasting sugar. Thick endometrium. Any problems in conceiving?",
"src": "Patient: hi, i am a 27 yr old lady.not conceived yet.i am trying these days.but this month after getting periods on my 14th day after periods i found bleeding and its a very irregular flow but i have this till today. i went to doctor and she asked me to take a insuling fasting sugar test. my endometrium is 16mm and thick. do i have any problems for conceiving? Doctor: Hi,Thanks for the query.At the time of ovulation some amount of spotting can be seen because of hormonal fluctuation.As you are having spotting for prolonged periods there could be some hormonal imbalance.Increased endometrial thickness is also suggesting that.So you once get evaluated for hormonal profile.Depending on that treatment can be taken.Once periods become regular you may not have any problem in conceiving.For more details you can ask me through: http://www.healthcaremagic.com/doctors/dr-sree-gouri-sr/63429Take care."
},
{
"id": 133636,
"tgt": "Suggest treatment for the hurt and pain in my knee",
"src": "Patient: Help I am a 27 year old female who is fairly active. While I have always had problems with my knees especially my right knee recently. I ve never really gone to the doctor and had an actual diagnosis that anything is wrong yet though. But I was laying on my stomach on a lawn chair outside and when I stood up to walk inside my knee started hurting. Not like the usual pain it was very foreign and very weird. I looked down at my knee and it looked fine until I went to bend it. When I bend it something protrudes from my knee cap and it looks whitish. The pain isn t as bad now I can walk on it but it is really bothering me to what it could be. Doctor: hi,Thank you for providing the brief history of you.A thorough musculoskeletal assessment is advised with MRI of knee.As the pain in the knee is persisting a thorough assessment is needed. Due to ligament injuries within the knee joint, such types of pain can be expected. On a longer run, this ligament injuries if left untreated leads to early OA knee, that is the degenerative changes in the knee joint. Also, undergoing an MRI of the knee should help understand the soft tissues better and than you have to undergo physical therapy for 2-3 weeks post which you can follow the exercises at home itself. Any prevention in the early stages provides good results.In my clinical experience, early detection helps in early time and late detection cases responds slowly.RegardsJay Indravadan Patel"
},
{
"id": 181496,
"tgt": "Suggest remedy for tooth abscess while on antibiotics",
"src": "Patient: I had a tooth abcess on the left side of my face about a month ago. After taking antibiotics for 4 week, my oral surgeon cut a slit under my chin and put a dain tube in. That cleared up and now the right side of my face is drastically swollen and hot, but no tooth pain. What could be going on? Doctor: Hi..Welcome to HEALTHCARE MAGIC..I have gone through your query and can understand your concerns..As per your complain it seems that there is spread of abscess into the soft tissues leading to swelling and hot appearance is due to inflammation..The drainage is performed but tooth is not treated seems strange as the first thing that must have been done is to treat the tooth with root canal treatment or extraction to remove the source of infection..I would suggest you to consult your treating surgeon or other ora lphysician and get evaluated and if tooth needs treatment it has to be done..As of now you can be given injectable antibiotics and steroids t reduce inflammation and swelling..Once swelling goes not the most probable thing that I thing is that you will need to get the tooth extracted followed by antibiotics and painkillers to relieve infection..Hope this information helps..Thanks and regards.Dr.Honey Nandwani Arora."
},
{
"id": 42668,
"tgt": "Can my husband undergo varicocelectomy for fertility purpose?",
"src": "Patient: Hi, just want to know if my husband can undergo Varicocelectomy for fertility purposes? In his sperm count test, it showed \"O\". But they got sperm inside that was why we did the IVF last year but unfortunate, unsuccessful. Please advise if it is possible for him. Thank you. Noemi Doctor: Hi,Thanks for writing to HCM.Varicocele is not directly related to cause low sperm count in men. It is not a cause for decreased motility.But a person having varicocele with low sperm count surgery for varicocele has shown to improve quality of sperms. I would suggest your husband to go with varicocele sugery . And proceed with IVF.Hope I have been helpful.RegardsDr.Ashish Verma"
},
{
"id": 217954,
"tgt": "Can I continue to use Etofree pain killer tablet 15 days after piles operation?",
"src": "Patient: gud eving sir, i hv used etofree pain killer tablet for last 15 days . because i undergo fissur and piles operation. can i continue the tablet are stopp the tablet pls advise meand also one more doubt surgery was held in 28 sep 2012 but till now i am having pain and bleeding wt can i do sir pls tell me Doctor: Using pain killers can cause many side effects 8n the long run tat after 18 , months post procedure.u can go for stoolsofteners, laxatives, local anaesthetic gels, high fibre diet for easy passage of stools"
},
{
"id": 3256,
"tgt": "What should I do get pregnant after an abortion?",
"src": "Patient: hai DR,i am hjbjty i got married jully2013 and i conceived oct but i did abortion on jan2014 becouse of baby growth s not well.after that i got perriod jan31,mar3 and mar31 then still i didn t get perriod i took scan and blood test my scan report s multiple small follicles in both ovaries blood report TSH-7.54,FT4-.67,FT3-2.97 WHAT CAN I DO GET PREGNANT? WHAT IS THE TREATMET FOR THAT Doctor: Hi.. Thanks for writing.. You need to correct your thyroid levels by taking antithyroid hormones prior to pregnancy.. Multiple small follicles on ovary can be due to PCOD.. For this ovulation induction drug can be used to stimulate ovulation.. Hope I have answered your query. Good day."
},
{
"id": 193874,
"tgt": "What causes sore throat and loose motion after fever?",
"src": "Patient: I had a fever 102 F doctor prescribed me levofloxacin (500mg) and cetirizine dihydrochloride (10mg ) both to be taken after every twelve hours for five days and have completed bed rest. Diagnosed soar throat. I have just taken two dozes and now i have lose motions. What should i do? Doctor asked me to have light food so i am having boiled rice, milk biscuits.age 37,maleheight 5,11, 90kg Doctor: Hello, It may be due to stomach infection with sore throat. For further assessment you may require complete hemogram, stool analysis after physician consultation. Keep your self hydrated with oral rehydration solutions. Continue steam inhalation and Betadine gargle twice a day. Take hygienic food and water. Hope I have answered your query. Let me know if I can assist you further. Regards, Dr. Shyam Kale Family and general physician"
},
{
"id": 112230,
"tgt": "Severe back pain after hurting tailbone. X-ray, MRI all normal. What is causing this pain?",
"src": "Patient: So about 4 years ago, I fell on my tail bone while rollerblading. Went to the my doc and he said that it'll go away with time. It hasn't. 4 years now and nothing has improved. I have two kinds of pain. A sharp one, that usually happens when I'm mensturating or constipated and a dull one, usually when I'm sitting on the floor or on a chair. In addition I that, I have back pain (mostly lower) when I'm standing for too long or when I lift something heavy. I have no idea what is wrong and I have done all kinds of tests, from x-rays to MRI. Almost every possible test. And nothing! I just wanna know what is causing this pain? And what I can do about it? I try not to take medication unless I absolutely need it. Doctor: Hi, When you fell on your tail bone, you have damaged the bones and muscles and nerves passing by. Coincidentally, the nerves and muscles are related to the internal organs like uterus or bowel. So, when you menstruate or strain due to constipation, you are putting more pressure to the affected area, thus, increasing the pain. I'd suggest to - do vaginal swab testing to exclude vaginal infection- urine analysis to rule out urinary tract infection- try to avoid being constipatedIf everything is ok, then, it is most probably due to fall. So, have patience and you everything will be over Can take painkillers (locally or orally) and drink more water.All the best!Dr.Alba"
},
{
"id": 183423,
"tgt": "How can headaches caused by toothache be treated?",
"src": "Patient: hi one of my teeth is really aching to the point where i am waking up with headaches. i had a fiiling repalced in this tooth, then the dentist thought i needed a root canal which i had and now it is hurting again. he is going to tell me too have it extracted, is this right? Doctor: Thanks for using Health Care Magic.Read your query.Was the root canal treatment completely and how long is it since its done?Severe pain associated with the root canal treatment usually suggests presence of underlying infection.I would advice you to visit your dentist and have it checked up.For the pain ,ibuprofen ( if you are not allergic to any medicine) can be taken.Antibiotics like amoxicillin 500mg and Metrolag may be required on prescription.Based on the radio graphic and clinical symptoms ,a retreatment of the tooth or any appropriate treatment needs to be done .Hope this was helpful.Thanks and regards."
},
{
"id": 118142,
"tgt": "What causes a high MCV in blood test?",
"src": "Patient: I had a blood test done and my MCV was 104 one Dr. told me to cut back on alcohol, I had the same test run by another Dr and they asked me how much red meat I ate in my diet, when I told them very little they said that's why I had a high MCV test. A little confusing. Doctor: Mean corpuscular volume or MCV is the average amount of space occupied by each red blood cell. Red blood cells help carry oxygen in the blood.A normal MCV is usually considered to be from 80-100.High MCV can be due to macrocytic anemia ,due to some drugs, liver disease, hypothyroidism, folic acid deficiency, liver problems and thyroid problems.RegardsDr DE"
},
{
"id": 113700,
"tgt": "Sore hip. Cannot walk or run. Will get better?",
"src": "Patient: I have a sore hip that I think I got from tight clothing. I have had it for a few months now and I was hoping it would get better but it s not. It feels like a bruise and hurts if I put pressure on it or streach it, eg. Walking or running. There is no visible mark or bruise. How long will it be before it gets better? Is there anything I can do to help it? Thanks Doctor: Acupressure Therapy;- Do jogging on acupressure plate for 10 minute morning n evening for aweek and report, hope you will get addressed your problem."
},
{
"id": 97280,
"tgt": "Does getting scratched by vaccinated dog require medical attention?",
"src": "Patient: Hi, I got scratched by nails from a friends dog. I got very little blood and little scratch marks which are barely visible after applying betadin. I had taked rabbis injections 3 yrs ago for a dog bite. This friends dog is vaccinated. Is it serious or I dont need to do anything Doctor: A scratch from a properly vaccinated dog does not require an ARV .A anti tetanus vaccine is required though(if not taken for 3 months)"
},
{
"id": 1619,
"tgt": "Can I conceive inspite of having PCOS by having primolut N and Metformin?",
"src": "Patient: hi. i ve pcos, annovulation, LH nand prolactin are now normal,been trying for a baby for 3yrs. i m on primolut N and metformin for the last 4days.i m hoping this clicks. i m 32yrs of age,weigh 64kg.had hysteroscopy done on d 20th of jan 2011-mild cervical adhesion,also have fluid coming from my nipples. Doctor: Hi, primolut N and metformin al9ne will not solve your problem. You have to take medicines for inducing growth of your follicles and track your follicles growth by repeated ultrasound. When follicles reach a size more than 17 to 18 mm, take injection for rupturing the follicles. Be in contact with your husband for next 2 to 3 days. Take progesterone for next 2 weeks. Do a urine pregnancy test at home after that. You can try like that for 3 to 6 months. Hope I have answered your question. Regards Dr khushboo"
},
{
"id": 19146,
"tgt": "Suggest treatment for blockages in the heart",
"src": "Patient: I have been told I need to be catheterized because 3 of my arteries to the heart are blocked. 2 years ago they were borderline, so I suspect that they wiill be putting in a stent. My questions are: Can they do all 3 (if warranted) at the same time? If so, is there any more risks involved? Will I be able to breathe better? I cannot walk 30 feet without feeling out of breath. Doctor: Hello!Welcome and thank you for asking on HCM!I passed carefully through your question and would explain that all these arteries can be treated with endovascular angioplasty at the same time. Regarding your shortness of breath, it depends on your cardiac function. You should know that shortness of breath could be related to heart failure. For this reason, I would like to directly review your cardiac ultrasound reports and NT pro BNP levels, coupled with other blood lab tests. Hope to have been helpful!Kind regards, Dr. Iliri"
},
{
"id": 156804,
"tgt": "Diagnosed with uterine fibroids. Could they turn into cancer?",
"src": "Patient: Hi Dr. I was diagnosed with uterine fibroids couple years ago, could they turn into cancer? I was told they could shrink as I get older, but mine are very big do I need surgery or is there any medication that could help dissolve them, I m very worried. Sometimes my lower abdomen hurts. Thank you Doctor: Hi and wecome to HCM Thanks for the query. Fobroids are benign tumors and extremely rarely it can be cancerous so you dont have to be worried aout that. it can grow and cause symptoms such as pain and pressure in pelvic area and it can affect pregnancy and concieving. in these cases it is recommended to remove it surgically or do hysterectomy. if you are not in fertile age then i recommedn it to since it causes pain in your case.Wish you good health. Regards"
},
{
"id": 40288,
"tgt": "What could cause a itchy bruise on the lower leg?",
"src": "Patient: Hi, i have a itchy bruise on my lower leg for months and it won't go away. sometimes at night my leg hurts to the point that i can't sleep at night and my legs begin to shake. i go to the doctors but all they give me is cream for it and it doesn't work. what can i do about this? Doctor: Hello,Welcome to HCM,Your symptoms and history suggests me that you may be having an eczema of the lower limb ,Insect bite, contusion, varicose vein, contact detnatitis, etc.all these are also possibilities for your symptoms.I would suggest you 1.Ice and compression to the area. 2.Elevating the area will help relieve pressure.3.Massaging the area gently 4.Oral Antihistamines5.Apply Moisturizing Cream.Your condition will improve with these remedies.Thank you."
},
{
"id": 58894,
"tgt": "Pain on the right side. Undergone tests. Increase in parenchymal echogenicity in liver. Suggestions?",
"src": "Patient: I have been having pain on my right side so I visited a clinic in Abu Dhabi (UAE) - am working here - A CBC was done and an ultrasound. The spleen, pancreas, gallbladder, kidneys and bladder are all normal. The liver shows a mild increase parenchymal echogenicity. No focal solid or cystic lesion noted. The cbc found the ALT was 54.7 and 0-50 being normal. The AST was 29.7 and 0-40 is normal. What do you think? Doctor: hi.. you have pain in the right hypochondrium.. mild raised liver enzymes and altered liver ecogenicity in the ultrasound examination.. could be viral hepatitis/early alcoholic liver disease.. get one HBsag and antibodies for Hepatitis A to rule out viral hepatitis...if it is ruled out, alcohol if you are taking should be abstained.. take multivitamins and liver enzymes.. take fresh fruits and vegetables..see your physician for getting liver protectives like silymarine..all the very best.."
},
{
"id": 7273,
"tgt": "I have positive ovulation test and i have sex on the same day. Could i be pregnant?",
"src": "Patient: If i have a positive ovulation test, and i have sex on the same day and day after, but no sex after then, can i still be pregnant . Doctor: Welcome to Healthcare Magic Good Day By Koparday's formula, calculation of fertile period is as follows Your shortest cycle days minus 18 days=1st day of fertile period. The longest cycle days minus 10 days= last day of fertile period. Ex: If your menstrual cycle varies from 26days to 31days cycle, The shortest cycle [26days] minus18days=8th day. The longest cycle [31days] minus 10days=21st day. Thus, 8th to 21st day of each cycle counting from first day of menstrual period is considered as fertile period. Period other than this fertile period in a menstrual cycle is considered as SAFE PERIOD."
},
{
"id": 198388,
"tgt": "What treatment could be used for pimple like piece in my perineum?",
"src": "Patient: hi doctor..i just wanna ask something..i had this small tiny piece looks like a pimples in my perineum..is this sort of STD diseases and what treatment could it be used..sometimes this thing most especially when irritated to my underwear it kinda painful and also when i squeeze it. Doctor: helloThanks for query .A pimple like painful swelling on perineum is due to Staphylococcal infection of hair follicle (Boil) in a stage of induration ,You need to take broad spectrum antibiotic like Cefixime along with anti inflammatory drug like Diclofenac twice daily .Observe proper personal hygiene by washing the area with warm water twice daily .It should get cured within a week .Dr.Patil."
},
{
"id": 5671,
"tgt": "PCOS, HSG done, patent tubes. On medication, regular periods but no egg formation. Will i conceive?",
"src": "Patient: Hello I am a 31 year old centrally obese female with PCOS . I have done HSG and found to have patent tubes . My husband s semen count is also good. I have been on, metformin , folic acid and siphene 50 mg for the last 6 months without any folliculometry done to detect if the dose was okay. Recently I changed my physician and he did a folliculometry on the 13th day of my last cycle only to found out that there was no sign of any egg formation. My periods are regular now (though it was very irregular before taking metformin and folic acid) at around 34 days cycle. My new doctor gave me 45 pills of ovares to take and then then to increase the dose of siphene to 100 mg for five days and go for another folliculometry. I am afraid as to whether I will ever be able to concieve or not.......I am very worried. Please help Doctor: Hi, PCOS responds very well to properly assessed treatment and so, do not worry. There are a few things you can do. Reduce your body weight by at least 5-10% as brings a significant improvement in your hormonal status. Do some regular daily exercise. Avoid fatty and fried foods; instead include more fruits and fresh vegetables in your menu. The dose of Clomiphene can be raised up to 150 mg. per day if lower doses do not induce ovulation. Proper monitoring of the cycle is necessary. Since you have regular periods after Metformin, you are likely to respond to Clomiphene alone. Please follow your doctor's instructions. Good luck."
},
{
"id": 126522,
"tgt": "Are having pain in the hips and left foot sign of arthritis?",
"src": "Patient: I sleep on my left side I have a history of sciatic nerve issue on that side however recently I have been experiencing hip pain and pain in the bottom of my left foot . After sitting or sleeping I cant put my total weight on my left foot. Most of the pain is on the heel area sometimes its in the arch area is this arthritis? Doctor: Hi, It can be a sign of arthritis, we have to rule out other conditions mimicking the same symptoms. As the first line of management, you can take analgesics like Acetaminophen or Tramadol for pain relief. If symptoms are severe better to consult a rheumatologist and get evaluated. Hope I have answered your query. Let me know if I can assist you further. Regards, Dr. Shinas Hussain, General & Family Physician"
},
{
"id": 193856,
"tgt": "How to get rid of sperm leakage?",
"src": "Patient: Hello Sir . I am Rohit ,i am about 18 year my height is 5 foot 5 inch and weight is 44kg . i have lot of problem . (1).my sperm is leaking 4 to 5 times in a weak i have this problem from 1 year . (2). having habit of hand-pump to get enjoy the sex feeling but after sperm leakage i feel very bad and aggressive on yourself .(3)my sperm leake in night. (4)sometime my stool is oily. kindly help me how i solve this problem..have a nice day sir Doctor: Hello, Usually vasectomy won't cause semen leakage. After vasectomy only pre-cum or prostatic secretions may be ejaculated. Yet, semen leakage may be seen in pathological conditions like 1. Nocturnal emissions 2. Medication side effects 3. Prostate problems 4. Nerve injury Please consult your doctor, he will examine and check for it then treat you accordingly. Hope I have answered your query. Let me know if I can assist you further. Take care. Regards, Dr. Ivan R. Rommstein, General Surgeon"
},
{
"id": 24091,
"tgt": "Do i need a blood thinner after getting stents in heart arteries?",
"src": "Patient: my husband just had 3 stents put in his arteries. One in the main heart artery and 2 on the other side of the heart. he doesn't have high coloesterol. does he need to take a blood thinner or is aspirin enough along with the anit platelet medication? he is 66, 330 lbs not on any other meds. he is now on heart diet, already lost 20 lbs. heart is strong, no damage. Doctor: Hello The aspirin and the antiplatelet medications together are fine for him. He doesn't need to take a blood thinner like warfarin in addition to those"
},
{
"id": 117746,
"tgt": "Is penidule recommended for sickle cell anemia?",
"src": "Patient: I have one sister and one young brother who are sick from sickle cell anemia at they have 22 and 24 years respectively,we seek for medical manegement of this disease but we ended by being given penidule is this helpfully for their management of disease symptoms like sickle cell crisis,anemia,and frequent disease.thanks. Doctor: hi, sickle cell anemia is the genetic disease, in that your RBCs become sickle shape in deoxygenated stat. and ended with crisis. treatment of choice is Oxygen in that situation. also you should take folic acid regularly to improve your hb. permenent treatment is bone marrow transplantation only. which is very difficult and costly.you should go with mainatinance therapy and Oxygen as and when required.thanks for using health care magic."
},
{
"id": 182298,
"tgt": "What causes funny taste in the mouth after dental work?",
"src": "Patient: I had some dental work done almost a week ago. they did some graffing on the upper right side for 3 teeth.Took the skin from roof of mouth and stiched over where the gum was receeding. I have a slight funny taste in my mouth and my nose has been running on the right side. The swelling has gone down and just brusing is showing now. Not sure if i should be concerned. I had the left side done last year and don't remember my nose running or having a slight funny taste. Only one stitch has fallen out. go back in one week exactly to get the stitches out. Doctor: Thanks for your query, I have gone through your query.The funny taste or altered taste can be because of the antibiotics taken during the gum surgery, the second cause can be the stitches or sutures can cause altered taste. Nothing to be panic, get the sutures removed and complete the course of antibiotics. Once you stop the antibiotics, the taste sensation will return back to normal. mean while do saline gargling.I hope my answer will help you, take care."
},
{
"id": 12107,
"tgt": "Black spots on skin but it is not itchy",
"src": "Patient: My son is 4 year old. From last 20 days i observed black spots on his hand in open area. Also ther are few black spots on cheeks and just behind ears. The spots appeared as black only, there is no itching and any history for skin disease. Doctor: hi it may be due to allergy from environmental factor mainly sun ,wind ,pollution tell him to wear full cloths apply sun screen lotion twice a day apply eumosone cream over black spot better to consult dermatologist"
},
{
"id": 166332,
"tgt": "Suggest treatment for throat infection with cold and cough in an infant",
"src": "Patient: hi Doctor My son is 2.7 yrs. he had high fever a month back, admitted in hospital for a weeks time. He had acute cinesitis, throat infection. he was all back to normal and active. Now since yesterday, he has got running nose and cough. can u please advise Doctor: Upper respiratory infections are common in children under 5 years of age, so you need not worry. Although if he had a sinusitis, he needs to be checked on this time too to look if he has the same. Avoid all cold foods and use cetrizine and ambroxol for cough. See a paediatrician to resolve the sinus problem."
},
{
"id": 219981,
"tgt": "Could absence of cardiac activity at 5th week be a matter of concern?",
"src": "Patient: Hi, Doc... Today I had to undergo sonography in my 5th week as yesterday i saw blood fr one minute as happen during periods. Although I am fine and have no pain n stomach or anywhere else. My report says:- Well defined round gestational sac is seen within uterine cavity, with good surrounding chorio decidual reaction. Gravid uterus with single intrauterine embryo of 5 weeks 6 days. Yolk sac is seen. No cardiac activity noted at present scan. Cervical length is normal and internal OS is closed. Both asnexae are clear. No e/o free fluid in cul-de-sac. Corpus Luteal cyst is noted in left ovry. Is everything normal with me??? Doctor: Hallow Dear,When you saw some amount of bleeding after missing period, it is but natural for you to get stressed. However, ultrasonography after the incidence of spotting has not revealed any bleeding from the chorionic tissue and the other findings are normal.It is good no note that the gestational sac is well defined and round, meaning there by that the embryo is viable. Even the embryo of about six weeks also is noted. 5+ weeks of pregnancy may be rather early for depicting cardiac activity. You need not infer any conclusion at this stage. The scan report is optimistic. Please repeat the scan after about a week or more. After 6 weeks duration of pregnancy, usually foetal cardiac activity is noted. With the current scan report, I feel you would have cardiac activity noted in scan after one week. Dr. Nishikant Shrotri"
},
{
"id": 10697,
"tgt": "Which ayurvedic hair oil can control hair fall?",
"src": "Patient: Hello sir, i am 26 year old . n i got suddenly hair fall problem and i got very less hair on middle of my scalp so can you please suggest me some good Ayurveda hair oil . where i can control my hair fall . and sir i am using Head-Shoulder shampoo anti-denruff. so is it perfect please revert soon sir. thank you. Doctor: hi.welcome to healthcare magic.i understand your problem.you have alopecia.my opinion is that,1.avoid stress.2.sleep for about 8 hours in a day.3.take nutritious foods and avoid junk foods.4.apply minoxidil lotion twice daily over the scalp.5.take multivitamin tablets and iron tablets once daily in night time.thank you.hope my answer will help you."
},
{
"id": 67159,
"tgt": "Suggest treatment for lump in the butt cheek",
"src": "Patient: I m found a rather large lump on the inside of my right butt cheek. It s very sore and hard to sit straight down without have to slow myself and lean when wanting to sit? I don t like going to the doctors about this so I looked online and it said something about a boil? Please help? Doctor: HiWelcome to hcmYou are having abscess/pus in butts which needs to be drained. It is not a simple boil. Treating yourself according to your net knowlage is very dangerous. Delaying treatment may cause sepsis and very deficult to treat. So show it to the surgeon as early as possible.Regards"
},
{
"id": 103630,
"tgt": "Had an allergic reaction to penicillin long back, severe chest pain now. Am I still allergic, can I be desensitized ?",
"src": "Patient: Hi Dr, I had an allergic reaction to penicillin..a suspension mixed with water when i was 14 some 43 years ago..I got large 1-2 inch diameter lumps like boils all over my mucous mebranes my eylids mouth throat etc and all over my body...causing me to have great difficulty to breath...I had a severe chest infection to start with!How do I find out if I am stil lallergic and can i be desensitised if i am. Doctor: hello there..welcome to healthcaremagic.. well after going through your query its quite clear that you have had a severe allergic reaction to penicillin in the past.. and i want to make you clear that once if a person is diagnosed of having penicillin allergy wil be allergic to penicillin for the rest of his life..so there is no need to find if u r still allergic..there is no way to get it desensitized.. infact if u consume penicillin once more there may be chances of life threatening anaphylaxis reaction.. and there is no need to rely much on only penicillins as there are many new antibiotics are released which are very effective.. so there are alternatives to penicillin which are safe and very effective thatswhy there is no need to worry about it.. just dont forget to tell your doctor that your are allergic to penicillin then they prescribe u an equally effective medicines.. i hope this solves ur query.. thanks and take care.. wish a healthy life ahead..."
},
{
"id": 87873,
"tgt": "What could abdominal cramps during sex suggest?",
"src": "Patient: I started on the combined pill last wednesday and have been taking it regularly every day at the same time. however yesterday and today I have had painful abdominal cramps and it hurts especially during sex. I was wondering if this was a common side effect or if it could be something more serious. any help or information would be appreciated.thanks,Laura Doctor: Hi.Thanks for your query and an elucidate history. IT is not common to have such a problem after taking combined pill, although these being hormones can cause a change in the ovary and uterus , but pain during sex after such tablets is not common.This is called Dyspareunia meaning painful sex or pain during sex. The causes are as follows:PID, Pelvic inflammatory disease/SalpingitisOvarian cyst, torsionPsycho-somaticI would advise in such a patient the following:Ask the Gynecologist to have an internal examination by doing per-vaginal examination, this can elicit many a things.Urgent ultrasonography, per abdomen and per-vaginal . Routine examination of the blood, urine and stool. The final diagnosis hence made will help us to get a proper line of medical or surgical treatment."
},
{
"id": 138746,
"tgt": "What causes numbness in thumb and biceps after taking anti-inflammatories?",
"src": "Patient: I have had a neck/back pain that affects my left arm. It feels like a pinched nerve. When the pain became unbearable, I saw my doctor on Thursday and was put on anti-inflammatory, muscle relaxers, and pain killers (diclofenac sod dr 75 mg, cyclobenzaprine 10 mg, and hydrocodone-acetaminophen 5-325). since I began taking these drugs my neck and back pain have improved, but now my left hand is going numb in my thumb and first two fingers and my arm is weak and numb around bicep area. I have less control of the same arm. In 2010 I had a discectomy and fusion in my C4 and C5 vertebrae and it feels like the same thing is happening again. Doctor: Hello,I have studied your case and I think there can be nerve compression in your neck. I would recommend you to get MRI of the neck to confirm the diagnosis. If there is nerve compression present than that is causing you nerve compression.If it is less than fifty percent than it can be taken care with Medicines and physiotherapy. If it is more than fifty percent than you might need surgical intervention.I hope this answer will be useful for you.Let me know if there is any other followup questions.thanks"
},
{
"id": 102721,
"tgt": "How can I prevent cough that is accompanied with heavy heart and sneezing?",
"src": "Patient: Hey I am 19yr old maleFrom chidhood em having vomiting ProblemLik if I laugh alot I cough then I vomitOr if i run ......Dis happend wen I was 16ys 0ld...Now em having cough problemWen I talk with sum 1 I coughI feel my heart beat fastEvery morning I sneez alot Doctor: HIThank for asking to HCMThis could be allergic condition and might be constricting your airway at the time of laughing and shouting, I would advise you to take the second generation anti histamine for long time \"Terfenadine \" twice a day single tab. for ten 10days it will be come around with this have nice day."
},
{
"id": 100453,
"tgt": "Are breathing difficulty, tiredness and mucus in throat signs of asthma?",
"src": "Patient: what would be the symptoms of ashtma and i am having diffuculty bvreathing i am having tiredness i have mucus in the back of my throat and i am constantly yawning and i was just told by a doctor over the phone that it was not cancer so is it asthma Doctor: HelloDifficulty in breathing , tiredness ,mucus in the back of throat , constantly yawing may be to these reasons , these include:1 Bronchial asthma :Airways hyperresponsiveness that is secondary to inflammatory airway disease that results in recurrent or chronic episodes of wheeze,Shortness of breath ( SOB ), CHEST tightness , &/or cough particularly at night or in the early morning.2 Bronchiectasis , in this usually patient feels as some mucus is plugged in throat . However it is very difficult to say something on the behalf of these symptoms . At present I can say that these are not the symptoms of CANCER.Anemia may be one such cause of these symptoms. Cardiac involvement may be the another reason .In my opinion the best way is to consult a physician and get his opinion because physical examination is important.Hope this will help you."
},
{
"id": 34764,
"tgt": "What is staph? How can it be treated?",
"src": "Patient: My name is Ben, i am 38yrs, 5ft 6inches tall and weighing 85kg.Always having malaria.My problem is this: I went for semen analysis and i was diagonosed of having staph. Aureus in my semen and my sperm count has gone down to 19 million. What do i do for after taking the prescribed antibiotics, the bacteria is still there and for 2 yrs now after our my wedding, my wife has not been able to conceive.I need a solution doctor. Thanks. Doctor: hi,sorry for your problemi dont understadn why you always have malaria..?it is a curable disease if you take proper medication..staph aureas is a kind of bacteria which can be killed by using antibiotics..use the antibiotics prescribed by your doctor..if the problem a change of antibiotic may be needed..donot worry there are a lot of methods to cure infertility..meet a=infertility specialist for further treatment..wish you good luck..if you have any more queries i would be happy to help you.."
},
{
"id": 60782,
"tgt": "What does a lump between the knees and hips indicate?",
"src": "Patient: i have a golfball-sized lump on/in my hamstring half way between knee and hip. Next to it is a depression in the same muscle. The mass seems to be involved in the hamstring muscle itself. I did injure something a few weeks ago which causes deep aching inside the buttock where the muscle attaches to the hip bone structure. Doctor: Hi, Golf-ball sized lump on hamstring, midway between hip and knee with a nearby depression after an injury can be due to one of the following reasons: - Hematoma formation - Abscess or cellulitis - Preexisting lump like a lipoma or so. This can be diagnosed by the following ways: Clinical examination by a General Surgeon gives fairly correct diagnosis. To be confirmed by high resolution ultrasonography or MRI as is needed. FNAC that is fine needle aspiration may be needed in selected cases. All these will give a proper diagnosis and a plan of an appropriate management wither surgical or medical as the need be. Hope I have answered your query. Let me know if I can assist you further. Regards, Dr. T Chandrakant"
},
{
"id": 132969,
"tgt": "Suggest remedy for pain in leg",
"src": "Patient: I broke my femur 2 months ago and everything felt good. I was weening myself off the walker and using a cane. I recently moved to a home that I have to negotiate stairs to get to the bedroom, and only yesterday did my surgical leg started aching and it is very difficult to put weight on it. Any idea why? Doctor: hihope this msg finds u in good health it seems ur bone might have still not United completely.. Climbing upstairs causes severe stress on the fractured bone.. so u shud reduce d amount of weight bearing u do for few mire weeks.. only when ur xray shows good union..u can bear weight completlythanks Take care god bless"
},
{
"id": 79879,
"tgt": "Suggest treatment for fullness in the abdomen, heart burn and joint pain with shortness of breath",
"src": "Patient: I guess I was raised to fear the label of malingerer more than that of recently deceased. Over the span of a few days the following symptoms have been intermittent but persistent: distention and full feeling of upper abdomen, burning in chest (heartburn), mild pain in left jaw, neck, shoulder and arm. Some swelling of face and hands, shortness of breath. Was recently treated for pneumonia but cough and congestion still a problem. I am just wondering at what point does this become emergency? I am a 44 year old female. Doctor: Thanks for your question on Health Care Magic. I can understand your situation and problem. By your history and description, possibility of GERD (gastroesophageal reflux disease) and bronchitis is more. GERD causes micro aspiration and this can aggravate bronchitis and cause breathlessness.Your heart burn, upper abdominal discomfort etc are due to GERD. So proper control of GERD is needed in your case. So avoid stress and tension. Avoid hot and spicy food. Avoid junk food. Start proton pump inhibitors and prokinetic drugs. Go for walk after meals. For bronchitis you should first get done PFT (Pulmonary Function Test). This will also tell you about severity of the disease. And treatment of bronchitis is based on severity only. You may need inhaled bronchodilators and inhaled corticosteroid (ICS). So consult doctor and discuss all these. Hope I have solved your query. Wish you good health. Thanks."
},
{
"id": 94439,
"tgt": "Testicle pain, tightness, lower abdominal pain. Testosterone shots. What can I do?",
"src": "Patient: I am 38 and My right testical started hurting last week and has gotten tight. I try to check after a hot shower but it seems to feel normal just still partly up in me. The left side seems normal. It has been very painful at times and a dull ache all the time. It hurts All the way up into my lower abdomen . I have been taking totestarone shots for a few months now but haven t really had any side affects. I can t get to my urology specialist until the first of march but if its something I should be really worried about I will go to ER. My stomach was hurting really bad and I was doubled over with pain but my stomach got messed up and that went away. The pain in testical and the drawing up started the next day. It s so bad at night I can t lay on my side with my legs closed. Any help would be great full. Doctor: Hi, Welcome to HCM, From your history it seems that you are having acute Epididymo Orchitis giving rise to severe pain on local part and lower abdomen. Consult your doctor urgently and get examined. A course of antibiotic and anti inflammatory medicine will cure your problem. Delay in the treatment might cause big problem in future. Ok and bye."
},
{
"id": 129466,
"tgt": "What is the cause for pain in sore tail bone and rectal bleeding?",
"src": "Patient: My tailbone and lower back is really sore, I have rectal bleeding, and some days constipation or some days narrow stools that are painful to pass. I also passed blood from my anus just when I was sitting in the tub. And I have itchiness in my anus... UGH.. .I am all messages up Doctor: Good day. You need to be seen urgently. Rectal bleeding can be due to piles, but your history with pain and constipation, you need to have a colonoscopy urgently. Please arrange to see a surgeon asap."
},
{
"id": 140981,
"tgt": "Suggest generic form of prestiqe for adult ADHD",
"src": "Patient: Is there going to be a generic for prestiqe available in the near future? My mediation is too expensive and have had to decrease my dosage and am ready to stop altogether in the next week or so. But I have to say its an amazing product and has made a huge difference to my adult A.D.H.D Thanks Doctor: Hello, The generic for Pristiq has been available in the U.S. since March 2017. Some predictions had reported that not until 2022 would a generic be available. However, TEVA Pharmaceuticals was first to the U.S. market with the generic in 2017. Hope I have answered your query. Let me know if I can assist you further. Take care Regards, Dr Dariush Saghafi, Neurologist"
},
{
"id": 90519,
"tgt": "What causes lower abdominal pain and loss of appetite?",
"src": "Patient: I have lower abdominal pain & no appetite. I went to the Dr. and he said something about my white blood cell count & put me on an antibiotic for a UTI (that has not helped at all-I did not feel urgency to go to the bathroom...which I usually get when I have a UTI). Any ideas on what this can be? Doctor: Hi there, If your doctor put you on an antibiotic for UTI, then, your white blood cell count might have been higher than normal range. If your urine analysis has also shown UTI, then, the antibiotic will help improve your symptoms. Furthermore, bowel infection can also be treated with antibiotic. As you may see, there are many of your medical data missing in order to give an appropriate answer. Can you please provide the reports of the lab tests you had? I need to analyze them myself to come with final conclusions. Meanwhile, I highly recommend to get the medications as prescribed by your doctor and drink plenty of water/fluids. Wish fast recovery!Dr.Albana"
},
{
"id": 74297,
"tgt": "Suggest treatment for chest pain while suffering from pleural effusion",
"src": "Patient: hello, I have just spent 4 days in hospital for sharp pain when inhaling. the test showed a small plural effusion, they say from RA. My sed rate was over 100 now down to 55. help me made sense of this and what other test should I have. I had c scan x ray mri Doctor: Thanks for your question on Healthcare Magic. I can understand your concern. Most common cause for small pleural effusion is pneumonia. Other causes are tuberculosis, RA associated pleurisy etc. So to identify the cause, we should definitely aspirate fluid and send it for investigations. So other recommended tests are pleural fluid ADA, pleural fluid protein and sugar, pleural fluid routine and pleural fluid culture. So consult pulmonologist and discuss all these. Hope I have solved your query. I will be happy to help you further. Wish you good health. Thanks."
},
{
"id": 84519,
"tgt": "Is it safe to take cefdinir 300 mg?",
"src": "Patient: why do they give cefdinir 300 mg capsules to people when there are so many side effects,i expierenced awfull effects blisters all over private parts,my mouth looked rotton all the way up to my ear.up set stomach really bad.i had to quiet takeing them. Doctor: Hi,It is not safe to take Cefdinir-300 if you are allergic to this type of antibiotic. Ask your doctor for an effective alternate but a safer antibiotic to eradicate the infection.Hope I have answered your query. Let me know if I can assist you further. Regards, Dr. Mohammed Taher Ali, General & Family Physician"
},
{
"id": 81502,
"tgt": "What causes difficulty breathing post defecation?",
"src": "Patient: On occasion, right after a bowel movement, I have difficulty breathing. I don t have any difficulties with the bowel movement, no excessive straining. Once I m finished my lungs seem constricted and I can t get a full breath of air. Is this something I need to see a doctor about? Doctor: Thanks for your question on HCM.By your history and description I think you are having breathlessness on exertion (defeacation).This can be seen in cardiac and pulmonary causes.So we need to rule out both in your case.So get done1. ECG and 2d echo to rule out cardiac causes like heart failure and coronary artery disease.2. Get done chest x ray and PFT (pulmonary function test) to rule out chronic bronchitis and cor pulmonale.So consult doctor and discuss all these."
},
{
"id": 158618,
"tgt": "Irregular period, painful, had bleeding, tested for abdominal diseases. Is it cancer?",
"src": "Patient: Hi I'm a woman 35 years age, I'm 5 2\" and my weight is 46kg. I've always been thin. Recently I've been getting sharp pains when I wake up and eat my breakfast. It's irregular but now frequent. Today I had a gush of small amount of bleeding but my period ended on Saturday. So its not period. It's not much blood and its brown. I'm going to get it checked but sadly my GP never takes anything I say seriously. She says I'm perfectly healthy. I will insist I get tested for stomach and other abdominal diseases. At this point it would be very helpful if you could direct me as to what I should ask for specifically. I'm obviously thinking \"cancer\". Today I suddenly felt exhausted and slept for half an hour although I could've done with a few hours because it didn't do much for me. Please help me or guide me? Doctor: Hi, You have mentioned about sharp pain but the site of pain is not mentioned. Anyway abdominal pain should be carefully investigated. Bleeding after the end of period should also be investigated. Nothing to worry about cancer right now because there are lot of of other causes. You should be examined by a gynaecologist. After clinical examination he/she may have ordered investigations like ultrasound whole abdomen. Please visit a gynaecologist's office. Take care."
},
{
"id": 155980,
"tgt": "What causes spreading of the cancer after chemo therapy?",
"src": "Patient: my father is having lungs cancer.........he was also a patient of tata memorial mumbai...........his treatment was going in amri dhakuria follwing the schedules of radiation and kemo given from tata memorial..........10 radiation was given and 5 kemo.but now the report is telling that cancer was spreaded babdly.so what we should do now? Doctor: Thanks for your question on HCM. Lung cancer is always spreading in nature. You can slow down the rate of spreading by chemotherapy and radiotherapy but you can not completely stop the spread. The cancer cells are spread by blood so they can affect liver, bones, brain, adrenal , gut etc. So at this moment your father needs palliative care in the form of multivitamins, central pain killers like tramadol, hematanics, cough supressants etc. So better to avoid further chemo and radiotherapy."
},
{
"id": 61027,
"tgt": "Are clear bumps below the penis head are friction blisters?",
"src": "Patient: Hi, I have three very small clear bumps just below the head of my penis. They have no pain, they\u2019ve been around over a week without bursting or changing color. It\u2019s been over a year since sexual activity. I do masterbate, could they possibly be Friction blisters or something more serious? To clarify no pain at all, no bursting, completely clear, approximately 10 days since I\u2019ve noticed them. Thank you! Doctor: Hello dearWarm welcome to Healthcaremagic.comI have evaluated your query in details .* Possible viral infection induced eruptions .* Should seek advise of expert skin specialist for further management guidelines .Wishing you fine recovery .Feel free to ask any further queries .Regards .Dr. Bhagyesh ( MS , FMAS - consultant surgeon )"
},
{
"id": 9933,
"tgt": "How to regrow hair after breast cancer treatment with taxotere?",
"src": "Patient: 57 year old caucasian female, no history of hair loss in family or male-pattern baldness prior to breast cancer treatment with taxotere hair was very thick three years after treatment with taxotere hair is extremely sparse and will not grow beyond soft downy stage help please Doctor: Hello, I would recommend you to apply minoxidil 2 per cent lotion on the affected areas of the scalp and take hair supplements like tablet follihair once daily. You need to continue these for 4-5 months to see significant results. Hope I have answered your query. Let me know if I can assist you further. Take care Regards, Dr Asmeet Kaur Sawhney, Dermatologist"
},
{
"id": 88719,
"tgt": "What causes pain in right lower abdomen?",
"src": "Patient: Hi, may I answer your health queries right now ? Please type your query here...i had an abortion where the fetus was removed at 5months,and iv been having pain on my right lower abdomen since then, i was diagnosed with pid a year ago.could my fallopian tubes be blocked? Doctor: Hi! Good afternoon. I am Dr Shareef answering your query.The pain on the right side of abdomen could be due to a persisting pelvic infection, or could be due to a urinary infection after exclusion of any chances of appendicitis by a physical examination. If I were your doctor, I would have advised you for a urine routine/microscopic/culture sensitivity test, an ultrasound of abdomen and routine blood tests like a CBC and ESR. The status of the tubes could be known by a radiological investigation like an HSG (Hystero salpingogram) by an experienced gynaecologist in association with a radiologist. Till the reports are ready, you could go for an anti inflammatory along with a proton pump inhibitor drug for a symptomatic relief.I hope this information would help you in discussing with your family physician/treating doctor in further management of your problem. Please do not hesitate to ask in case of any further doubts.Thanks for choosing health care magic to clear doubts on your health problems. I wish you an early recovery. Dr Shareef"
},
{
"id": 82788,
"tgt": "Could microvascular ischemic demyelination be due to lupus?",
"src": "Patient: I am a 48 yo white female with a history of epilepsy, SLE, hypothyroidism, SVT, etc. Recently I experienced at severe headache with dizziness, sensation of fainting, and nausea. I have had migraines in the past with neurologic symptoms of partial vision loss. This recent episode was different. My doctor ordered a cerebral MRI and the results show 'chronic microvascular ischemic demyelination.\" Could this be a new manifestation of the lupus? Doctor: Central nervous system (CNS) lupus refers to several different neurological and/or behavioral clinical syndromes in patients with systemic lupus erythematosus (SLE). The neuropsychiatric manifestations of lupus, which are frequent, vary from mild to severe and are often difficult to distinguish from other conditions and etiologies. Any location within the central nervous system (brain and spinal cord) may be affected with a variety of presentations from mild cognitive dysfunction to seizures, stroke or coma.summarizes the major manifestations of CNS lupus.Major manifestations of CNS lupus described below:- Cognitive dysfunction (not thinking clearly, memory deficits etc.)HeadachesSeizureAltered mental alertness (e.g. stupor or coma)Aseptic meningitis (inflammation of the covering of the brain)Stroke (disturbance of the blood supply to different parts of the brain)Peripheral neuropathy (e.g. numbness, tingling, burning of the hands and feet)Movement disordersMyelitis (disruption) of the spinal cord. Visual alternationsAutonomic neuropathy (e.g., flushing reaction or mottled skin) Hope this will helpful to you..."
},
{
"id": 130942,
"tgt": "What is the cure for stomach spasm pain?",
"src": "Patient: I have been having best I can describe as stomach spasm pain. Takes my breathe away so bad. I can move around a little to stretch in different directions and that usually helps then stay still for a while and it goes away. sometimes have them in side and going up to shoulder blade. What is going on? I'm 71 and had colonosphy within last 1 1/2 years. Doctor: you are experiencing symptoms of liver failure , i recommend checking your liver profile as soon as possibleGood Luck"
},
{
"id": 65575,
"tgt": "Suggest treatment for a painless lump on the leg",
"src": "Patient: I have a spot on my side that looks somewhat like a hickey. I ve had it for about 4 months now. It s about 2 inches long and about an inch wide. It doesn t hurt or itch. I have just noticed, just showed up, a spot on my leg. It s only like an inch long and very thin. What could thus be? Thanks! Doctor: Hi, thanks for sharing your health concerns with HCM! Well, If I were your treating Doctor for this case of a painless lump on the leg, I would come up with three possibilities, these include: 1.\u00a0\u00a0\u00a0\u00a0\u00a0a lipoma or benign fatty tumor or neurofibroma2.\u00a0\u00a0\u00a0\u00a0\u00a0The second possibility is of a benign cyst, ganglion or bursa\u00a0\u00a0\u00a0\u00a0\u00a03.\u00a0\u00a0\u00a0\u00a0\u00a0The last possibility is of some normal / developmental nodule like muscular nodule or just a feeling for you!\u00a0\u00a0\u00a0\u00a0\u00a0Overall, it is benign and not to worry about this but you could go for FNAC test for confirmation if still worried!Hope this answers your question. If you have additional questions or follow up questions then please do not hesitate in writing to us. I will be happy to answer your questions. Wishing you good health."
},
{
"id": 132402,
"tgt": "Suggest medicine for knee pain after falling down",
"src": "Patient: Dear doctor, i fell over the other day and landed on all 4s. I grazed my right knee quite badly and a bruise has now developed on my knee around the cut. Since the day after my fall, on standing i get a throbbing in my right knee and it is quite bad and has got worse over the last few days. Its also not too bad on walking, just when standing still. Just want to know if i should go to a doctor or wether it should resolve by itself Doctor: Hello welcome to HCMI am happy to help you outaccroding to your history and description of wound,I feel its not very severe injury.If such patient comes to my clinic then i would examine knee joint for possible grave injury like ligament tear,fracture then according to finding would suggest further investigations like xray and prescribe some analgesic till investigation and educate him about wound dressing to be done at home.such pain may last for 7-15 days but if pain persistthen i would suggest you to consult your nearby doctor for proper evaluation and treatment.Thank youRegards,Dr.Om"
},
{
"id": 80074,
"tgt": "What causes tingling sensation in the chest?",
"src": "Patient: I am a 51 yr. old, 135 lb. female. I have experienced some tightening in my chest when laying in bed, either first thing in the morning or in the evening when I am ready to sleep. It has happened 4 times in the last month. It lasts approx. 30 seconds to a minute. My husband thinks it is because I have been on the elliptical machine for 45 minutes and it is muscle strain. He may be right, but I am a little concerned due to the fact my 46 yrs. old brother had a heart attack and he was in good health. I am in very good health and exercise regularly. At my last physical I did have an EKG (in November) and it was fine. should I be concerned?? Doctor: Dear Mam, symptoms(tightness) you are talking about with strong history of heart disease in the family, i feel you need work up to rule out heart disease as well. EKG is sometimes normal at rest in persons with heart disease. I recommend you to go for TMT(EKG while running on TMT machine) and lipid profile of blood. If these are normal take some pain killer+muscle relexant for 3 days. take care"
},
{
"id": 91921,
"tgt": "Is morphine recommended for abdominal pain after undergoing gastric bypass?",
"src": "Patient: My 130 lb mother was given 4 mg of morphine at 0630 and another 4 mg at 0930. Unbeknownst to us, LTAC has been giving her 4 mg of morphine as needed. Last night she complained of her stomach being on fire She is being fed through an NG tube that goes into her duodenum (she had a gastric bypass). After they reversed morphine overdose with Narcan and dopamine (BP dropped like a stone), she till complained of abdominal pain. Took her for catscan which revealed blood in abdomen and in pancreas. What could cause this? She is in LTAC recovering from lobectomy (she had a trach and they are weaning her from vent which was going quite well until this episode). Doctor: There's no contraindication for giving a gastric bypass patient morphine. However, she might have been given overdose given that-Her BP dropped (morphine can drop BP)-she needed Narcan to reverse Morphine's effectThe BP drop could have been due to Morphine, loss of blood (as blood is seen in and on CT scan)It seems like the Morphine and blood in abdomen are unrelated events.If I were a doctor treating her I'd like to know the following:- when was the bypass surgery- is she currently on any anticoagulantsTake her back to the surgeon who performed her bypass or at least, her lobectomy for evaluation.Hope this helps."
},
{
"id": 206753,
"tgt": "Suggest treatment to cure mental condition",
"src": "Patient: hi my brother is sufring by mentle problem, i treated him by so many dr bt he is not imrov by his dieses.since last ten years he is a civel eng due to the disese all the family members r faceing vry tough time.due to the same problemof mybrother, my father no more now.so the whole responsiblity is on me to care my bro ,kindly suggest me to is there any treatment is posible to my brother.thanks sir reply pls Doctor: DearWe understand your concernsI went through your details. I suggest you not to worry much. From the description give, it is very difficult to diagnose which type o schizophrenia your brother has and the depth and breadth of it. There are good psychiatric treatment available for scizophrenia. You should continue the treatment without fail because continuous treatment can only cure your brother. Don't loose your hope.If you still need my help, please describe the whole problem in detail and post a direct question to me. I shall definitely help you with psychotherapy techniques to over come your problems.Hope this answers your query. Available for further clarifications.Good luck."
},
{
"id": 122205,
"tgt": "What causes sharp shooting pains throughout the body?",
"src": "Patient: I have random sharp shooting pains throughout my body. I am also always very sleepy. even if i sleep in, i feel like i want to stay asleep, and about 3-4 hours after i wake up, i am already fighting to stay awake. I am very forgetful lately, which I ve never been , and its really hard to stay focused. i have to really work hard to follow a conversation, and even then i usually forget half of it.. this is so unlike me. Doctor: Hello, Your symptoms could be related to vitamin D deficiency or a metabolic disorder (anemia, thyroid gland dysfunction, etc.). For this reason, I would recommend performing some blood lab tests (complete blood count, PCR, ESR, thyroid hormone levels, blood electrolytes, vitamin D and vitamin B12 plasma levels). You should discuss with your doctor on the above tests. Hope I have answered your query. Let me know if I can assist you further. Take care Regards, Dr Ilir Sharka, Cardiologist"
},
{
"id": 168355,
"tgt": "Is it normal to have blue bruise on infant s face after hitting?",
"src": "Patient: I have a 2 month old who was accidentily hit in the face with a cell phone 14 days ago. Some days it seems like it s going away and then other times it looks worse. It is blue in the middle with red on the edges that resemble broken spider veins? I bruise easily and she is very pale...is this normal? Doctor: Hi,Welcome To Healthcare Magic.I can understand your concern regarding to your child\u2019s health.This type of bruise is not normal but very common occurs after some blunt injury which breaks some very small and tiny veins, which has happened with your daughter by cell phone.So, for subside bluish bruise with red edges, you should apply the ice on the affected area 5 to 7 times in a day for 4-5 minutes for 5 to 7 days.After doing this, I hope your child will get well sooner.Wish her good health.Take Care, Thank You.From:- Dr. Nupur Shah"
},
{
"id": 184429,
"tgt": "Suggest treatment for halitosis",
"src": "Patient: hello... i am 33yrs old...169cms height...62kgs weight... i am suffering since long from halitosis...is there a cure for it? i have consulted my dentist and my oral hygine is now good...but even after brushing teeth i suffer from bad breath...can you please advise me? Doctor: Thanks for your query, I have gone through your query.The halitosis could be because of the deposits or any pus discharge secondary to tooth or gum infection or any respiratory tract or gastrointestinal infection.Consult a oral physician and get it ruled out.If it is because of gum infection, then get your teeth cleaned and maintain oral hygiene and use mouth washes.If there is tooth or gum infection get it treated with RCT or extraction.If it is respiratory tract or gastrointestinal infection then you have to consult pulmonologist and gastroenterologist. I hope my answer will help you, take care."
},
{
"id": 219306,
"tgt": "Why is fetal pole and yolk sac not visible in my pregnancy?",
"src": "Patient: uterus gravid uterus showing 7 weeks and 2 days sized gestational sac. foetal and yolk sac polex yet not visible in view of an early pragnancy advised re scan after 21 days for gestational viability. inference early pregnancy of about 6 weeks and 5 days means hindi pdf Doctor: Hi, Thanks for asking . I understand your concern. Normally gestational sac is evident by 3to 5 weeks of pregnancy. Yolk sac is seen by 5.5 to 6 weeks of pregnancy & fetal pole is evident by > 6 weeks pregnancy. The inference ( Hindi me matlab Ki ) of your USG isPregnancy is 6 weeks & 5 days pregnancy ( Hindime.. 6 hapte aur 5 din ka garbha hai). * Absence of fetal pole at this stage indicates - Delayed conception, - blighted ovum ( pregnancy without fetus ) - Ectopic pregnancy with pseudo gestational sac in uterus. * In this respect I would suggest USG with vaginal probe, as advised after 21 days / to get a second opinion .. to confirm the findings. Thanks."
},
{
"id": 52483,
"tgt": "Need a good liver specialist who can deal with end stage liver failure",
"src": "Patient: I need to find a good liver specialist. I saw a liver Transplant Surgeon at UCLA but that it a bit far. I also need to find a liver specialist to follow my care. I am in End Stage Liver Failure, have a TIPS plus a multitude of health issues. I am a 55 year old female Doctor: Hello and Welcome to \u2018Ask A Doctor\u2019 service. I have reviewed your query and here is my advice. You can get a online register of doctors and find a nearby doctor. Wishing you good health. Thanks."
},
{
"id": 5563,
"tgt": "Diagnosed with PCOS. Planning for a baby. Took fertyl tablets during periods. No periods now. Pregnant?",
"src": "Patient: I have been diagnosed with PCOS in february..we just planned for a baby from january 2013 and my doctor advised me fertyl tablets during my period(day 3rd to 7th day-5 tablets)..me and my husbnd met from day 13th to 18th And my last period was on february 22nd,,til now i didnt get my period..is this is the sign of pregnancy or delay of my period due to PCOD ,,plz reply to this Doctor: Hi, If you had ovulated while on Clomiphene, and followed the doctor's instructions, then it is most likely that you are pregnant. You can confirm this by taking a home pregnancy test with a fresh early morning sample. If the result is ambiguous and you wish for a better confirmation, you can get the serum beta-hCG measurement and a trans-vaginal sonogram. Please see your doctor for further suggestions. Good luck."
},
{
"id": 190932,
"tgt": "Does biting on the inner cheek cause itching ?",
"src": "Patient: a few days back i consulted two Dentist regarding thickness on my inner cheek after biting over and over. i am damm worried about this. A DR. (MDS) has said nothing to worry about. but i am still feeling certain itching there not always but when i am worried. Although there is no change in in shape and color of it. Is itching normal please answer. Doctor: hello, due to repeated biting of cheek thickening of mucosa occurs which generally is not a thing to worry. Start betadine gargles and try not to bite again on that part. Ask ur dentist to examine if there is a need for a tooth extraction or some sharp tooth cusp has to be smoothened... itching can be there cos of lodgement of microrganisms so maintain oral hygiene. U can also start warm saline gargles 3-4 times a day.. take care.."
},
{
"id": 20684,
"tgt": "What causes pain in the heart area passing to the left arm?",
"src": "Patient: Sir Myself Vijay Kumar From Jawahar Navodaya ViDyalaya Vill. Tewa Kaushambi (U.P.). My Wife Smt. Pallavi (age 21) Suffering from pain at the location of heart and pain is passing in the left arm. We need help and suggestion from you please. YYYY@YYYY Mobile No. 0000. Doctor: Hello Welcome to HCM.Chest pain passing to the left arm is typical of heart problem but in view of her young age the reason to suspect a heart ailment is very unlikely . Please ask her to take antacids tablet Pan 40 mg twice a day before food for two weeks. Also get her vitamin D and Vit B12 levels checked. If they are found to be low, then she should be started on supplements for vit D and vit b12.Also do and ECG to rule out any associated heart problem . If despite all these measures she has still got these symptoms, please do a treadmill test which requires her to walk on a treadmill while her ECG is continuosly being monitored . This will show whether she has any blockages in the heart.Please do the above tests and revert back.Wishing her good health Regards."
},
{
"id": 157567,
"tgt": "Diagnosed with Barrett s esophagus. Will Radio frequency ablation help?",
"src": "Patient: thank you- Ihave been diagnosed with Barrett s esophagus . I have read a lot about Radio frequency ablation And I would like to get this done . I do not have dysplasia according to the pathology report at this time based on A pathology report. I am very nervous about waiting another two years before I can get another endoscopic Procedure done. I could advance to cancer by then. I have read where a lot of people with no dyspasia have been treated and with good results in eliminating Barrett s. my insurance won t pay for RFA unless you have dysplasia. What if I Pay for it myself ? is there any way I can get this RFA Procedure done? Where can I get this procedure? Doctor: Dear Friend ...Welcome to Healthcare for your queries ...I read your query and I would like to help you to get RFA procedure done.You can have good result if done in earliest without dysplasia . I would suggest you to Consult your treating physician and can be done in nearby Hospital without insurance . Do not be nervous as it is curable one . You are not likely to develop dysplasia changes faster therfore do not be worried . You can also visit MGH ( Digestive Health Care )55 Fruit StreetBoston, MA 02114Hours: 8:00 am to 5:00 pm165 Cambridge StreetBoston, MA 02114Hours: 8:00 am to 5:00 pmAsk me if you have futhur quries , I will help you for your queries .Regards Dr.J.Kingson John David"
},
{
"id": 90330,
"tgt": "What does abdominal pain when eased with bowel movement mean?",
"src": "Patient: My almost two year old woke up from her nap Wednesday crying like she was in pain. After 10 minutes she'd quit and did this every few hours. She did finally have BM and that did seem to help. She still now says her tummy hurts and does this like grunting when she's breathing. Doctor: It is most likely colicky pain which happens due to excessive peristalsis of bowel after meals and are eased by bowel movements. Do not worry it will get relieved with time and for severe pain you can give her anti spasmodic medications. Take care"
},
{
"id": 47272,
"tgt": "Is burning throat and salty phlegm is caused by kidney problem?",
"src": "Patient: I have had a burning throat for some time, and salty phlegm. I found that cutting out milk products and wheat helped, but a recent blood test suggested either I was dehydrated or had a kidney problem. I was advised to have a repeat blood test. Could there be a link between the burning throat and a kidney problem?I am 59. Female. Teetotal, non-smoker. 5ft 3ins in height, on no medication. Healthy. Doctor: Hi, dearI have gone through your question. I can understand your concern. You have burning throat and phlegm. It has no relation with kidney disease. You may have some respiratory tract infection. You need to take antibiotics like azithromycin, amoxiclav or levofloxacin for that. It is prescription based medicine so consult your doctor and take treatment accordingly. Take plenty of water. Repeat your complete blood count and serum creatinine level. If your creatinine level comes high then you need to go for complete kidney function test to search the cause. Then you should take treatment accordingly. Hope I have answered your question, if you have doubt then I will be happy to answer. Thanks for using health care magic. Wish you a very good health."
},
{
"id": 181986,
"tgt": "Suggest treatment for tooth infection",
"src": "Patient: I had an infection around a tooth which my dentist extracted on Thursday. I am taking Amoxicillin since then but my lymph node is still swollen and my gum area still appears whitish. The pain has lessened but I just wanted to be sure the infection isn't spreading. Doctor: Hi Removal of the infected tooth and proper antibiotic cover will do the needful to subside the infection . My advice to you is that besides following basic oral hygiene regimen you should also use betadine rinse or warm water saline to fasten the healing process . i preferably advice limci (vitamin c) tablets once for 15 days for improving gum condition and restoring them back to health. The symptoms you have mentioned will subside in a couple of days . take care ."
},
{
"id": 177084,
"tgt": "Suggest treatment for high fever in a child",
"src": "Patient: HI...MY SON AGE 7 YEARS WEIGHT 20 KG...HAVING FEVER LAST EIGHT DAYS. INTIALLY 3 TO 5 DAYS WITH SHIVERRING AND TEMT...WAS..NEAR..104 1ND 105...BUT AFTER START ANTIBIOTICS..CEFIXME...200 MG..FEVER..GRADE..REDUCE...BUT NOT..SUBSIDE..PLEASE SUGGEST..HOW..MUCH.TIME..TAKE..IN..RECOVER Doctor: good morning.. since your child is having high grade fever since eight days i advise you to get his urine examination done to rule out any urinary infection. hat It can be one of the causes of fever with chills.. also get his blood counts done to see for any infective focus.."
},
{
"id": 21911,
"tgt": "What causes heart palpitations and heart racing?",
"src": "Patient: i get intermittent heart palps with heart racing that can last for hours i don,t know if it is related to blood sugar, stress, hormones or real heart disease it is scary i am a 47 y.o. female with elev a1c, lots of stress, elev cholest but normal periods so far. how concerned should i be? Doctor: First of all get your ECG done,and thyroid profile,do you have any other symptoms like chest pain or breathlessness?"
},
{
"id": 166239,
"tgt": "What causes dizziness and headaches?",
"src": "Patient: Ever since I got back from school, I ve been feeling very dizzy and have had multiple headaches. I also feel tired and my stomach hurts. I am 13 years old and have not been doing anything gross or dirty. I am usually very healthy but I have felt like this before 3 months ago. Doctor: hi, this could be due to viral infection or a stomach infection. During viral illness there is whole body ache, fever can be present. In stomach infection, There is pain abdomen, vomiting and feeling of dizziness. You should take soft diet. Take antispasmodic like Tablet buscopan 10 milligram which contains hyosine. take more fluids. An examination by doctor should be done so that we may not miss any important finding. prebiotic like enterogermina ampule should be taken once a day for 3 days to increase intestine immunity. take care."
},
{
"id": 51041,
"tgt": "Frequent urination, occasional pain of abdomen and kidney. UTI or enlarged prostrate?",
"src": "Patient: Hi, I am 22. I started to urinate frequently these few days. I am not sure whether its due to drinking too much beers or masturbation. I don t feel any pain when I urinate but sometimes I feel my left abdominal and left kidney . Is it UTI or enlarge prostate or something else. I am quite worried as I have already seen a doctor and have been on antibiotics for 3 days, but I still feel the urge to urinate. Doctor: Hi, Your symptoms are related to Urinary Tract infection as you are already taking Antibiotics from the last three days. Please perform Urine R/E to see any pus cells and blood cells. If there are that will resolve. Do urine R/E after 02 days after completion of Antibiotics for 05 days. If there is pus cells and blood perform urine C/S 48 hours afetr discontinuation of Antibiotics to check for sensitive ones that can resolve your problem. I hope I am sucessful in solving your query If there are further you can ask me Take care Regards, Dr. Azhar Sattar"
},
{
"id": 163258,
"tgt": "What causes low WBC in a child?",
"src": "Patient: My 9 year old son has been ill with various bugs, infections since October 2010, including d&v, coughs, bad cold, flu and then probably swine flu over Christmas. The doctor did various blood tests to see if there was an underlying reason and they came back today with low white blood cells. She's said she'll retest in 3 weeks. Should we be concerned? Thanks Doctor: Hello,Cause of many infections includes white cells and many causes dec white cells. If it is less than 4 then it's the matter of concern.But the primary infection will be treated sooner. They will get normal in case platelets and HB is also normal. Hope I have answered your query. Let me know if I can assist you further.Regards,Dr. Hina Javed"
},
{
"id": 182230,
"tgt": "Suggest treatment for tooth pain",
"src": "Patient: Hi, I recently had an exterpation of pulp on my top incissor tooth. I am going to have a root canal done on it soon. The dentist said the pain should subside in a day or two and It has now been four days and my tooth is still really painfull. Should I worry? Doctor: Thanks for your query, I have gone through your query.The pain can be because of the remnant infection in the root tip. Nothing to be panic, take a course of antibiotics like amoxicillin and metronidazole for 5 days and analgesics like diclofenac. Later you can get the tooth treated with root canal treatment. Mean while get the root canal irrigated or cleaned with saline or hydrogen peroxide. It may take 5 days for the pain to come down.I hope my answer will help you, take care."
},
{
"id": 28513,
"tgt": "Why does fever persist after recovering from a kidney infection along with fatigue and nausea?",
"src": "Patient: I have had a kidney infection for about a week with a mild fever pain in back right side that goes into my side and right side of stomach. Fatigue some nausea and throwing up and stabbing pain on the right side of my head. its been a week and the fever wont go down past 99.9 when I take meds and when they wear off it goes up to 100.9 Doctor: Hi, Persistent fever due to Kidney infection (Pyelonephritis) despite treatment may be due to antibiotic resistance or TB or fungal infection. Re-investigation for fever along with fresh clinical examination to find any fresh symptoms or sign are needed. Hope I have answered your query. Let me know if I can assist you further. Regards, Dr. Tushar Kanti Biswas, Internal Medicine Specialist"
},
{
"id": 113222,
"tgt": "Backache, bloated stomach, breathlessness. After effects of stress?",
"src": "Patient: Last wednesday I had backache ,bloated stomach hard to catch breath but when I got adjusted on Friday I felt great.About Monday it started again.Seems like everytime I m stressed this week it happens more my back hurts gets tight and burning in my stomach and back off and on like heart burn feels like I cant catch my breath.Our two cars broke down in Jan.and my dad had a heart attack in Jan.I also home school my daughter hardly get outside right now because its cold and we just got our twenty year old into counseling and meds for anger management .I m wondering if I have the after effects of stress. Doctor: Hello. Thanks for writing to us. The symptoms that you are having could be related to stress, anxiety, gastritis, hiatus hernia or a cardiac problem. The best way to differentiate all these is to get yourself properly examined and investigated by your physician. I hope this information has been both informative and helpful for you. Regards, Dr. Praveen Tayal drtayal72@gmail.com"
},
{
"id": 72458,
"tgt": "Suggest treatment for pneumonia",
"src": "Patient: I was diagnosed with double pneumonia 1 week ago having had the symptoms for a week, I have been on heavy penicillin and am left with weakness and a bad cough. My query is that my urine is reddish brown, looks like some blood could be there. Is this part of pneumonia. Thank you for your time. Jean aged 72 and normally healthy except for overweight. Doctor: Hello dear Jean , hiWarm welcome to Healthcaremagic.comI have evaluated your query in details .* The urine color is in relation with side effect of penicillin , not a part of pneumonia .* Has to be addressed immediately to your treating doctor for this issue .Wishing you fine recovery .Welcome for any further guidance .Regards ."
},
{
"id": 132303,
"tgt": "How to treat a swollen and bruised foot post injury?",
"src": "Patient: I walked into a pothole over a week ago and my foot bent completely upward. Over the past week and a half I have had moderate pain on the heel and outside of my foot that becomes severe when I put pressure or wear tight shoes. It feels a little better with elevation and ice. It is pretty swollen, but not incredibly, and there is no bruising, so I have ruled out a break. Any idea what it could be? Doctor: Hi if you have ruled out fracture you must be having sprain.And remember foot and ankle injuries are very notorious even you dont have fracture there are chances of ligament injuries which causes severe pain and swelling and complete recovery is not possible in such cases.Mild pain remains in foot even after recovery.As your injury is fresh you need to give rest to your foot till the swelling and pain subsides."
},
{
"id": 167910,
"tgt": "What causes high TSH level in a child?",
"src": "Patient: My son was diagnosed with hypothyroidism with his newborn screening. He is now 17 months old and has been on synthroid since he was 3 weeks old. He has had labs done every 6 wks with adjustments on meds.. he is now on 44 mcg and labs now reporting high normal T4 Free and high normal TSH. How is this possible? High T4 free would point to hyperthyroidism, and high TSH points to hypothyroidism... Help please. Doctor: the dose of thyroxine is altered mainly on the base of the value of TSH, if it's high that means the dose isn't enough or a Lab error since he doesn't have any symptoms t4 is normal because of the thyroxine he is taking but it doesn't seem to be high enough to suppress the TSH my recommendation is to repeat the tests again and adjust the dose based on the TSH value I hope this helps"
},
{
"id": 160724,
"tgt": "What causes seizures in babies with loss of development?",
"src": "Patient: I have a 3 year old daughter who has seizures at 6 months of age and lost all developmental and cognitive abilites. She is slowly gaining them back at is at this time functionally at a 1 1/2 to 2 year old. We have had mris and catscans done and all is well we have had e.e.g s done and they are improving. She was born with a low sugar level of 24 and otherwise was ok. She has been seizure free for 2 and 1/2 years and was on topamax for 2 years. She is dealing with the side effects of that now. She is currently on keppra 1.5 ml bid as a preventive measure. what could cause a baby to have seizures and then they go away but leave such a bad path. I see her improving daily. We have had tests ran and nothing comes up no cerebral palsy chromosomal or metabolic. Please help. Doctor: Hi,Seizure and loss of already attained milestones occur in a condition called neuro-degenerative disorder. Truly speaking, this is not a single disease but a group of conditions with neuro-regression. Frequently, despite thorough investigations, we won't be able to find out a specific course, as new diseases are being discovered year by year and the list we have now is incomplete and yet to expand.The hypoglycemia she had after delivery could be a reason, but in that case we expect MRI to be abnormal and although milestones can be delayed, loss of already gained ones is unusual. In such cases, what we can offer is symptoms oriented supportive measures like early stimulation to maximise development, seizure control with drugs, maintain proper nutrition, vision and hearing assessment and management. Your doctor will guide through these steps.Hope I have answered your question. Let me know if I can assist you further. Regards, Dr. Muhammed Aslam TK, Pediatrician"
},
{
"id": 11550,
"tgt": "What cause hyperpigmentation on face and its medication?",
"src": "Patient: Hi Doctor, I am facing Hyper pigmentation problem on my face since 5-6 years. Tried lot of medicine but didn't work for me. Somebody suggested me to use Omela. Need your feedback on this. Is Omela cream affective for this problem? Regards, Gaurav Doctor: HIThank for asking to HCMI really appreciate your concern this could be age related problem as such you have not mentioned your age so if you are above 4o then this could be age related problem that what mostly seen, in my opinion this could be best treated with the \"Hydroquinone cream\" some times this agent gives good result, hope this information helps you, have a nice day."
},
{
"id": 61677,
"tgt": "What causes bleeding lump between legs?",
"src": "Patient: Hello doc, I had a mole like something between my legs but due to using razors it became elongated, now when I shave it bleeds along with pus sometimes.& now it has formed its 2elongated heads. Its not hard, but soft and lumpy.Waiting for your help. Doctor: hello,thank you for consulting you have not mentioned the age.most probaably you are having sebaceous cyst which got ruptured during shaving its not life threatening condition.is just a retention cyst. you need not worry.please get it checked with general surgeon near by. thank you"
},
{
"id": 129473,
"tgt": "What type of back brace is beneficial for a compression fracture?",
"src": "Patient: My mom ,92 has a compression fracture in T 12. What type of back brace would benefit here. She has been prescribed a nascott tlso back brace but it appears to rigid and presses heavily in her sternum. Rides up an hits her an her Adam s apple when sitting and makes swallowing difficult. It has a rigid hard plastic shell with Velcro straps and a triangular hard plastic sternum piece. What are your thoughts? She hates the way it feels around her neck and sternum area. Doctor: Hello ,I am Dr. Selmani and I read carefully your question.T 12 compression fracture happens from forces when spine bends forward so the purpose of treatment is to immobilize the spine with a brace in a position not to allow the patient to bend forward which may risk the patient to displace the fracture and cause nerve injuries.Usually the principle of these braces is the \"three point pressure\" like the tool that bends the wire.The brace should have 2 points of contact forward the body, which one is up in the sternum and one is down in the pubic symphysis in lower belly. The third point is on the back in the level of T12 vertebra that is compressed. The 2 points in front push the 3-rd point in the back of the brace to straighten the spine.Usually the brace should not be high until the chin but until the sternum.If it goes until the sternum you should shorten if possible or go to the ortho technician to repair it. Also your mother should not spend much time sitting in the first 3 months as this is a position that increases the forces of the spine to compress more the fracture. Your mother can do some back muscle strengthening exercises , walking and laying down.and some water aerobic or water gym exercises.I hope she will recover quickly."
},
{
"id": 109867,
"tgt": "What causes severe pain on lower right side of back?",
"src": "Patient: I'm 32 male. While jogging recently I had a very acute pain on the lower right side of my back. Should I be worried? I don't know if it's because I haven't exercised in a long while (more than 6 months).I am about 1.8 metres tall, 95kg with no medical history of backaches. Doctor: Hello,Thanks for your query.After going through your query I came to know that you are probably suffering from lumbar disc prolapse. It can be confirmed by MRI scan.Its treatment is rest in position of relief(generally lateral position with both hip and knee bend),neurotopics such as mecobalamin and analgesics (Diclofenac three times a day after meals is effective). Sometimes strong analgesic(such as tramadol three times a day after meals) is required. Omeperazole before meals prevent acidity caused by analgesics .Second possibility is muscular pain. Treatment of it is rest and analgesics for nearly 5 days. You can discuss with your treating Doctor about it.I do hope that you have found something helpful and I will be glad to answer any further query. Take care."
},
{
"id": 170472,
"tgt": "What does urine report as Albumin present 3+ suggest in children?",
"src": "Patient: Hello Doctor, I have more than a year that i have pain in the lest side of my throat especially when i swallow my saliva(no problem with food or drinks) i have a swallen gland under my chin(it comes and goes, did a CT its ok), i get mild ear ache and neck pain (all this is on my left side) i eat and sleep well. i am stressed and worried Doctor: HiThanks for writing to us. The symptoms are of tonsillitis with reactive lymphadenitis.I usually prescribe amoxicillin for a week to resolve this. Urine albumin 3 + is usually seen in nephrotic syndrome. Consult your doctor immediately. He needs further tests and treatment with prednisone. Regards"
},
{
"id": 72989,
"tgt": "What causes chest congestion with breathing difficulties and black phlegm?",
"src": "Patient: My father is suffering from chest congestion for the last 1 week or so, and is having difficulty in breathing as he is an asthamatic. Initially the phlegm spit out was blackish in colour but now, it is of the normal colour. He is unable to sleep in the night properly Doctor: go to investigation of sputum - gram staining bacterial culture sensitivity , and fungal staining and total serum IgE , you will get your answer. other thing take proper inhaler for asthma if it is not controlled then it may turn into ABPA. take tab Abflo sr 200 od tab medrol 16 mg for 5 days after sample taken"
},
{
"id": 113727,
"tgt": "Jogging and after about 4km had severe back pain. Took foxair, no help. Any ideas ?",
"src": "Patient: Hi, I was jogging and after about 4km/ 2.5 miles I started walking and suddenly got severe back pain and could not take deep breaths . Bending over helped a bit and it passed after a couple of minutes. I have asthma but take foxair twice a day which keeps it under controle. I am not fit at all just started running . Any help will be great. Thank you. Doctor: Hi,If you have begun jogging after a long break, then you should prime your body. Don't start directly with 4 km. Start with 1.5 km for a week, and increase your distance slowly over a period of 2-3 weeks. What has happened is that because your muscles are in continuous need of oxygen during such sudden, strenuous exercise, and your body is not able to supply it, it switches to anaerobic mode of energy production (wherein oxygen is not needed). Kind of like taking a detour from the usual path. The problem here is that because of this alternate mode, a lot of lactic acid gets produced in the body as a by-product and gets stored in your muscles. This gives rise to pain, because lactic acid gets separated from your muscles very slowly. Your back muscles have gone into a spasm. Do some hot fomentation, take rest from jogging for 2-3 days or till the pain subsides. Then begin slowly. Don't worry, you'll get back to your 4 km run slowly, albeit, a bit slowly.All the best!dr.neharv@gmail.com"
},
{
"id": 90765,
"tgt": "Cause for severe abdominal pain?",
"src": "Patient: Hi I am a 19 year old female. I have been getting real bad stomach aches lately, my boobs also hurt. I go to use the restroom several times a day, like stomach aches within everytime. Lately I have been getting up in the middle of the night having to use the restroom and sitting on the toilet quite sometime. Idk if it s what I ate the night before or during the day. I had a hot pocket & a pop tart today. My period isn t in another week. It s just not normal to me having these symptoms. I am quite worried that it could be something serious. Doctor: Hi, The symptoms you are having might be related to premenstrual syndrome as your periods are due next week. Feeling to use the restroom several times a day might be related to :- indigestion- food poisoning- viral gastroenteritisI advise to: - take more water- take tasectan for diarrhea- take otc orsIf no improvement, should consult with your doctor!Wish fast recovery!Dr.Klerida"
},
{
"id": 148277,
"tgt": "Can spinal tap cause severe pain in back, neck and shoulder, spasms and headaches?",
"src": "Patient: I'm 24 years old I had a spinal tap done 5 days ago about 10 hours later I started having severe back pain,neck and shoulder pain also while this pain happens I have spasms in my neck shoulders and back I cannot control them also I have the spinal headache these spasms and pain have put me on bed rest I cant do anything im on muscle relaxers pain meds please help me and give me an estimate on how long I will be going through this Doctor: Hi. Welcome to HCM. Spinal tap also called a lumbar puncture, a procedure in which the fluid surrounding the spinal cord (called the cerebrospinal fluid or CSF) is withdrawn through a needle and examined in a lab. Approximately 10% to 20% of people develop a spinal headache (one that worsens when sitting or standing). Avoid strenuous or vigorous exercise for a day or so following the lumbar puncture. If you have a headache, lay down as much as possible and drink plenty of fluids. Headaches usually lasts for 3-4 days. In some cases it might take a long time to subside.I suggest u to consult your treating doctor if its worsening. Dont forget to drink lots of fluids (also consume caffiene products like coffe, tea). Regards."
},
{
"id": 96028,
"tgt": "Can peanut butter cause abdominal pain ?",
"src": "Patient: This is the 5th or 6th time now, if I eat peanut butter after 9pm I have severe abdominal cramping. Just under the breast to the bottom of the rib cage. Taking a hot bath helped at first, Any ideas? Please. Doctor: Hi , welcome to healthcare magic. Your history suggests allergy from peanut butter which irritates the stomach.It can cause allergy,so avoid them. Also look for food labels to check if they are present. Food allergies are with particular types of food one of which is peanut. Wish you good health."
},
{
"id": 108770,
"tgt": "What causes lower back pain?",
"src": "Patient: Hello, I am taking Procaria 30mg and Coeg 6.25 mg. I have been taking them for 2 weeks and I am experiencing lower back pain on my left side. Can this be a side affect of the drugs? Besides high blood pressure I am in good health with no other health conditions. Doctor: depending on age,occupation and physical habits, low backache may have different causes of origin.Left side lower backache may not be from meds you are taking, even then get you Kidney function tests like serum creatinine, urea and urine test done for ruling out side effects.If movement of back ,bending or twisting is originating symptoms of pain and you feel tender points in low back area, then maybe its an isolated backache.You may take ibuprofen 400mg Twice a day for it after meals, do some hot fomentation and avoid strain to back in meantime"
},
{
"id": 55878,
"tgt": "What do the S.G.O.T and S.G.P.T levels suggest?",
"src": "Patient: Hi, I recently had a complete blood panel done for query Hypoglycemia. Fortunately I am not diabetic. My S.G.O.T and S.G.P.T were 15 U/I respectively. Are these healthy amounts for liver tests? Could I still have a fatty liver with these results? 41 year old woman moderately drinks alcohol.Many thanks Doctor: Hello, Welcome to Health Care Magic. I read carefully your query and I understand your concern.I see that your lab test liver enzyme are at their normal range, this mean that your liver hasn\u2019t any damage that mean that you have not fatty liver disease, for be certain that you haven\u2019t you can do a abdominal echo.For avoid the fatty liver you should lose weight, eat a healthy diet, do minimally 30 minutes of walk every day and avoid the alcohol intake. \u00a0\u00a0\u00a0\u00a0\u00a0 Hope this answers your question. If you have additional questions or follow up questions then please do not hesitate in writing to us. I will be happy to answer your questions. Kind regard,Dr .Marsida Janko"
},
{
"id": 111625,
"tgt": "What is the cause of lower back pain while playing cricket and what is the treatment for it?",
"src": "Patient: Hi, I get lower back pain on the left side while bowling(Cricket). It happens sometimes but once it starts it stays for months. Doc said it is the disc which is getting sprained. He turned me in few directions but I dont feel pain. Its only during bowling I get it, that too when I bowl for long time. I do good warm up and warm down thou. May be coz i stress too much during match. But is there a permaent fix for it? I really wanna continue playing. Abhishek Doctor: Hi, Thanks for using HCMIn my opinion all you need to do is give you back a little rest. For immediate relief have some pain killer along with a muscle relaxant and have proper bed rest for 2 weeks. Do not lie down on soft mattress use hard one instead, do not sit for longer durations and do not travel long distance for 2 weeks . You can also apply some good anti inflammatory pain killer gel on your back. If the pain increases do some hot fermentation locally and make a routine of doing some back strengthening exercise daily . Start with 20 mins twice a day to 40 mins twice a day and walk for 30-40 mins a day at least. It will help you build you back muscle and you will be able to continue with you bowling in few weeks but do not over look at the symptoms right now. As it is still early days for you and you can be fine and back in action with these small things. Good luck"
},
{
"id": 114435,
"tgt": "Suggest treatment for low iron levels other than IV therapy",
"src": "Patient: my daughter has recently been diagnosed with celiac disease . her iron levels are very low. The doctors has recommended IV therapy. Unfortunately, she is refusing to get them because of the amount of time involved. She is a college freshman...go figure! Are injections an alternative? Doctor: Hello,Oral iron supplements are useless in presence of celiac disease. In celiac, disease intestine will become malfunction, and so iron won't be absorbed. To replenish iron stores IV route prefer. A gluten-free diet is indicated in celiac disease. After the prolonged gluten-free diet, your intestinal mucosa will be normalized, and you can take oral supplements at that time; but not now.Hope I have answered your query. Let me know if I can assist you further.Regards,Dr. Siddartha"
},
{
"id": 119280,
"tgt": "Chronic ITP, bleeding from nose, gums, purple spots. Ways to increase platelet count?",
"src": "Patient: Hello Doctor. My wife is having Chronic ITP.She has bleeding frm nose , gums, purple spots.Recently we are blessed with a baby girl and she is normal. Now am afraid of my wife . Can u plz suggest me wt cn b done now to increase her platelet count. What are the risk factors and what medicine to be used? Doctor: Hi, welcome to HCM I am Dr Das Basically, in tip, the placements get destroyed rapidly which causes bleeding . There are many modalities of treatments available. First, you have to admit your wife in a hospital for monitoring then arrange for placements from blood bank , consult with doctor for giving steroids and feasible monoclonalantibodies ."
},
{
"id": 148917,
"tgt": "On and off seizures. MRI showed cyst on left side of brain. On seizure medicines. Major surgery to get this removed?",
"src": "Patient: hi my son is 28 yrs old, hes had 2 seizures at work last month and last week in my home he had a seizure then an hour later had another,he spent night in hospital and had an mri which showed a cyst on left side of brain,he was put on seizure medicine but does not have insurance for more testing to be done,will this medicine help the seizures or will he continue to have more seizures on the meds because of cyst?? is it major surgery to have it removed? thank you,deb russell Doctor: Hi, Welcome to Health care magic forum. Your son had seizures, and found a cyst in the brain. Prescribed the seizure medicines, the drugs will control the seizures but cant help for the cyst. The cyst is to be studied, by further investigations and follow ups. If it is growing, may need to be operated. Wish that every thing go well.Thank you."
},
{
"id": 181159,
"tgt": "How can severe toothache be treated?",
"src": "Patient: I have 4 bad teeth on left side. Wisdom teeth top and bottom. Can't sleep for a week and pain bring me to tears. Meds ran out. Tromodol is gone. I do not have job but I can payment plan. Please help o can't take it any longer. Dental appointment is in June Doctor: Hi..Thanks for the query..Well, if you have 4 painful teeth then taking painkillers alone is not a good idea as they will provide only temporary relief and the pain will flare up once the effect of the pain reliever will wean off..In case if you do not have early appointment available with your dentist then you can consult another oral physician or an oral surgeon or dentists at dental colleges to get attended early..Your teeth first of need to be scanned by an IOPAR [X-ray]..Once the cause is ruled out the teeth has to be treated accordingly.If your wisdom teeth are causing problem they can simply be extracted for permanent resolution..Other painful teeth can either be saved with root canal treatment or extracted depending on the condition of teeth..Hope this information helps.."
},
{
"id": 225121,
"tgt": "Can birth control pills avoid pregnancy for years?",
"src": "Patient: Hello Sir I have a query. Can these birth control pills or pregnancy control pills (like MALA-D) hold pregnancy for 1 or 2 years? means if any lady being pregnant can avoid pregnancy for how much time through these pills. Can it be more than1 year???? Doctor: Dear member,Thanks for using healthcare magic. Birth control pills also known as contraceptive pills if used regularly are very effective in preventing pregnancy. If the lady who is consuming the pills is a young lady (To answer your second query regarding lady being pregnant and wants to avoid further pregnancies I assume your query is regarding the use of birth control pill after the lady has delivered her child and dosen't want another baby now. MALA d cannot be used in lactating or mothers who are breast feeding. In such a scenario I would usually recommend other methods.Kindly get back if you have more doubts on this issue.Thanks.Dr Bhagyashree."
},
{
"id": 94109,
"tgt": "Pain in the stomach near belly button. Pain during bowel movements and urination. What is wrong?",
"src": "Patient: I have pains in my stomach right around my belly button and on the sides. It ferls light sharp shooing pains. I keep belching and it hurts when I move. I have had a bowel movement but my stomach hurts when I go also when I urinate. It hurts when I eat as well. This is the second time this has happened to me. Last time it lasted a week and I went to the ER and after a scan they found nothing wrong. Doctor: Hi, thanks for asking in Healthcare Magic. Many conditions can result in pain around umbilicus (belly button). The possible causes are 1. Irritable bowel syndrome. 2. Appendicitis. 3. Gaseous distension. 4. Inflammatory bowel disease. 5. Gastric ulcer. You are probably suffering from pain due to gaseous distension without any organic (intestinal) obstruction. It results in sharp colicky pain aggravated by any movement. It can occur due to eating food like meat, cluster beans, dairy products etc., Stretching of the intestine due to gas will increase the pain due to increased intestinal movements, Most people try to relieve pain by belching. Minimal gas may not be prominent in scan as the intestines normally have gas shadows. Only gas accumulation due to obstruction will be prominent in scan pictures. Simethicone, an inert anti foaming agent can relieve the symptoms. But, if the symptoms persist or if it impairs the ability to perform normal activities then you should consult your treating Doctor. I hope this is useful to you."
},
{
"id": 128146,
"tgt": "What causes knots on the left side of the stomach?",
"src": "Patient: HI I am a 41 year old woman who has high blood pressure that has nots on the left side of stomic. These nots have been growing for a good while, with the past 4 days my side has been hurting so bad that the pain will wake me up from a dead sleep. I can t lay on this side at all, and when I eat it makes it worst or even carry heavy items. I had an obgy dr. push on the upper part of my stomic 15 years ago trying to deliver my daughter during her birth. Doctor: Hello.Thanks for writing to HCMI have studied your case and history.There is possibility of bowel obstruction due to previous surgery or infection possibility below duodenum.I will advise you to investigate like Ultrasonography pelvis and abdomen.If required CT abdomen may be doneTill time start antacids [proton pump inhibitors like] rabeprazole -D.There can be possibility associated gastric infection if you are having fever? Or diarrhoea?Try to maintain body fluid by increasing fluid intake, take easily digestible food.You need to consult your treating doctor for further investigation.Hope this answers your query. If you have additional questions or follow up queries then please do not hesitate in writing to us. I will be happy to answer your queries. Wishing you good health.Take care."
},
{
"id": 25161,
"tgt": "What causes irregular heartbeat with muscle twitches?",
"src": "Patient: Hi, Lately i have had som odd symptoms. I feel my heartrate has been more irregulare lately. Also i have had alot os muscle twitches. They kan be big or small, from one to a prolonged twitching. I have also felt creeping in my skin, usally going from my neck to my head. Sometimes other places. I have also felt inner tremors at night. I usally wake upp i the middle of the night feeling vibrations coming from inside my body. Usally the chest area and armes. My puls feels normal though. I have had these symptoms ( exept for the tremors) before but alot these last weeks. I also hade rashes before but nothing like that lately. I have felt som anxiety about my university applikation and my fathers death this year. I have thought it might be psychological but ime scared it might be something else. I havent seen a doctor about it yet. I saw a doctor about the rashes before and they said it might be som sort of allergic reaction. What could these symptoms be related to do you think and what can i do about it? Doctor: Thanks for your question on Healthcare Magic. I can understand your concern. In my opinion, your all symptoms are mostly due to stress and anxiety. But better to first rule out heart related diseases like arrhythmia (rhythm disturbances in heart) as this can cause similar symptoms. So get done ecg, 2d echo and Holter monitoring (24 hours continuous recording of ecg). If all these these are normal then no need to worry for arrhythmia and other heart diseases. Possibility of stress and anxiety is more in your case. So better to consult psychiatrist and get done counselling sessions. Try to identify stressor in your life and start working on it's solution. You may need anxiolytic and other supportive drugs. Don't worry, you will be alright with all these. Hope I have solved your query. I will be happy to help you further. Wish you good health. Thanks"
},
{
"id": 191983,
"tgt": "Suggest treatment for high blood sugar and headache",
"src": "Patient: i am having severe headaches. My blood sugar level is very high and Diamicron MR (Gliclazide) is recently added to my diabetes meds Metformin and Januvia . I am also taking Bissoprolol and other high blood pressure meds.What could be the cause of this headache. Doctor: HI, thanks for using healthcare magicDiamicron is not associated with headaches normally, this is not a known side effect of this medication.There are different possible causes of headaches such as: migraines, tension headaches, trauma, sinus headaches, lesions in the brain, cluster headaches, side effect of medication.Since your headaches are severe, it would be best to see your doctor for assessment to determine the cause.I hope this helps"
},
{
"id": 192419,
"tgt": "What causes testicular pain while doing heavy work?",
"src": "Patient: My husband has one testicle that seems to have shrunk and he gets pain in that testicle when doing heavy work, weather changes & sometimes at random. This has been for 2-3years now. Any idea what might be going on? We have had 2more kids since this happened so it doesn't seem to have effected procreation abilities. Doctor: Hi,The symptoms that you have described may be seen with the following:- Torsion of the testes- Inguinal hernia- HydroceleIn these, the size of the testes is not usually reduced but one side of the scrotum can appear bigger than the other side.Some symptoms that one may expect are pain when coughing, straining, or lifting heavy objects. There may also be an increase in the size of the scrotal swelling with coughing or lifting heavy objects. The next best step is to visit a doctor for an examination to confirm the cause. Some doctors may also suggest ultrasound to confirm the extent of the swelling.Post this, the usual management is through surgery.Hope I have answered your question. Let me know if I can assist you further. Regards, Dr. Vignan Rachabattuni, General & Family Physician"
},
{
"id": 216458,
"tgt": "What causes pain and soreness in calf?",
"src": "Patient: Hi Doctor I woke up to a terrible calf cramp that was so painful. After a few minutes it subsided and i was left with soreness. now day 2 and it is more sore and stiff. It is worse when I am sitting and then stand and when i walk up stairs.Before all of this let me tell you I have been working out almost daily on a cybex machine for 30mins. I just want to make sure it is not a blood clot.thanks Doctor: dear user, do not worry about it, prolonged sitting and improper posture of sitting will decrease dlow of blood which causes soreness sometimes.better if you take TENS therapy for a week will bring muscles back to normal.try to walk daily make a routine to avoid any pain"
},
{
"id": 103212,
"tgt": "Itchy areas on legs, black bruises. Blood work shows wbc tad off. Allergic to penicillin. Should i take more blood tests?",
"src": "Patient: Random itchy areas on my legs, it's a new area every day. I do not scratch it hard or apply too much pressure but I keep waking up with huge purple and black bruises on my legs. I had blood work done once and it said my wbc was a tad off. Could this have something to do with it? I also do not like meat so I don't get too much iron. Could this affect that as well? The only allergy I have is penicillin, and amoxicillin. (As well as grass and lotion but I have done nothing different in the past 3 months that could cause a reaction from those two) Do you think I should get some up to date blood work as this has been occurring for a month or so? Thanks Becca Doctor: if you have drug allergies you can have other allergies any type of substance can cause allergy at any timeneed to go for blood serum tests for cause of allergy and treat accordinglytill you can use antiallergic tabs and local application of ointments"
},
{
"id": 34718,
"tgt": "Suggest treatment for puffy edema everywhere my body",
"src": "Patient: I have been taking Gleevec 400 mg daily for control of GIST . My last ct scan showed no evidence of disease. In the last few weeks my fluid load has become more problematic. I have puffy edema everywhere I had a 10 pound weight gain in 3 days and was directed by my primary doctor to double my 40 mg Lasix to 80 mg whenever this occurs. My tissues are like sponges that depress and stay that way. I am concerned about the effects this much fluid may have on my heart. Doctor: HiWelcome to HCM!I appreciate your concern for the generalized puffy edema.Accumulation of fluid due to salt and water retention is an important adverse effect of Gleevec chemotherapeutic drug. This side effect is commonly seen during the long treatment with Gleevec , it may be associated with high BP and sometimes the fluid gets collected in the lungs causing difficulty in breathing, etc. Now since the scan is not showing the evidence of the GIST you may please consult your doctor whether to continue with Gleevec for how many weeks or months.Lasix definitely helps in clearing the edema fluid but it also has some side effects on its long run.Take care of your health."
},
{
"id": 87080,
"tgt": "What causes burning pain in abdomen?",
"src": "Patient: My urologist prescribed Tamsulosin for BPH. I took it one night, along with my Tolterodine. I awoke that night in a terrible burning pain that felt like a line of lit gasoline racing about in my lower abdomen. I stopped taking these pills immediately. I am a functioning quadriplegic who has suffered from burning pain for almost 19 years. I had a cystoscopy done to finally see if there was a reason for my chronic pain. Instead, worse burning pain, still, after three days. My opoids can calm the pain, but only temporarily. I am a pro active patient, and want to know how I can rid myself of this lake of fire.(I have just completed a novel about my entire experience, and the health care profession does not seem credible, let alone proficient. Any answers. (Am I corresponding with a machine or real flesh and blood.) Sincerely, Gary Pierluigi Doctor: Hi Gary Pieluigi.Thanks for your query.You are interacting with a real Doctor of flesh and blood and not the machine.The causes of burning pain in the abdomen : in your case can be IBS- irritable bowel syndrome or Neurological in origin as some Sensory nerve would have got compressed or entrapped. I can think of this with the history of functioning quadriplegic with the history of taking Tamsulocin, Tolterudine and then getting pain. But the pain is persistent and the character is like a gasoline flame racing in lower abdomen. Since you have a burning pain for almost 19 years, I would recommend you to undergo the tests of MRI of the whole spine and find the cause and get further treated for ablasion or removal of the discs if protruding and get well."
},
{
"id": 222855,
"tgt": "Does eating pineapple and gingelly after sex affect pregnancy?",
"src": "Patient: sir,my period date is 27th of this month.I sex with my husband on january 14th.May abort it? because I eat pine apples & gingelly s after threedays of sex.And one more thing is just three months before i lost my child before delivery due to high BP & albumin presence.what can i do ? Doctor: there is no literature suggesting that pineapple and gingelly can affect or lead to abortion however be careful about sigh symptoms of abortion"
},
{
"id": 187542,
"tgt": "What is the remedy for painful gums?",
"src": "Patient: my gums start to hurt a few days ago. it is painfull but not swollen. I have an average teeth care, but my sleeping times are all over the place and my eating times are different. I eat very little and sleep very little. I'm 16 years old and I want this so go away as it is very irritating an painfull. PLEASE HELP!!! Doctor: Hello, Thanks for your query.According to your explanation it seems that your having gingival recession. It is a condition characterized by exposure of root of teeth caused by loss of gum tissue and retraction of the gingival margin from the crown of the tooth. It can occur due to many reasons like tooth brush trauma , gingivitis, periodontal inflamation, trauma,tooth crowding, eating disorders. I suggest you to visit dentist to rule out the exact cause. Get the scaling done , maintain oral hygiene , use softer toothbrush,prevent food retention,use antiseptic mouthwash, massage on gingiva with stolin gum paint twice a day. I do hope that you have found something helpful and I will be glad to answer any further query.Take care"
},
{
"id": 64696,
"tgt": "Suggest treatment for a itchy lump on the armpit",
"src": "Patient: I have a growth in my armpit. Ive had it for a year but the past couple of months its grown to about an inch and a half in diameter. It has small veins in it and it was painless until this morning. This morning I got up and noticed it had a head like a pimple on it and I barely touched it and brownish discharge kept coming out, no odor but the discharge is gritty like wet sand. And now its pAinful. Feels like I have needles all in my armpit Doctor: This is hidradenitis suppurativa, infection of sweat glands in the arm pit. Show it to a general surgeon, he will clean it and take a sample for culture and sensitivity and will prescribe antibiotics accordingly."
},
{
"id": 204372,
"tgt": "How can aggressive behaviour while suffering from ADHD be treated?",
"src": "Patient: My son has autism and ADHD he is 13 years old he was hospitalized for aggressive behaviors against me he was medicated on Risperdal but his prolatine level was too high he was tried clonidine and benztropene which had to be removed due to erections my son has become aggressive and recently put on Depakote which has not helped much he touches the teachers body parts and mine also I am fearful for my sons behavior is uncontrollable Doctor: Hello and Welcome to \u2018Ask A Doctor\u2019 service. I have reviewed your query and here is my advice. As Risperidone is problematic, your child can be tried on other low dose anti psychotics, especially in the night as sedation will not be a problem in the day by which his aggression can be controlled. Joining him in special school benefits him a lot...he learns quickly when joined in special schools... Feel free to ask further questions."
},
{
"id": 17376,
"tgt": "What causes swollen ankles, migraine and high BP while taking Lisinospril?",
"src": "Patient: It is the weekend and am concerned about blood pressure. Last 3 readings: 140/88, 133/106, 137/105. Also have pounding headache in back of skull and top of head like a throbbing migraine. Saw my primary in May. All blood tests and EKG normal. Was taking Lisinopril HCTZ 20/12.5. My concern was swollen ankles so Dr. switched me to Lisinopril tabs 20mg & Lasix 40mg. I have not felt well since the switch. I have been treated for HBP about 15 years, am 56 and do smoke. Last 2 days have taken Lisinopril twice a day and 1/2 Lasix. Dr. told me I could do that as long as I monitor myself. Have not had numbers in 100s for a long time. Other medications are Aciphex 20mg, trazodone 100mg, sertraline 150mg and the lisinopril 20mg & lasix. I just do not feel well and need some advice. I will call my dr. on Monday to discuss. Thank you. Doctor: Hello, Swollen ankles can be an adverse effect of trazodone. I suggest you to monitor your blood pressure for somedays . These values of blood pressure are not to worry much about. I would recommend you lisinopril, HCTZ and probably lercanidipine in a low dose. You should see your doctor after monitoring your blood pressure and he will see if a change in medication is needed. Hope I have answered your query. Let me know if I can assist you further. Regards, Dr. Anila Skenderi, General & Family Physician"
},
{
"id": 178924,
"tgt": "What causes no bowel movement in a baby?",
"src": "Patient: my two months old son breast feed normal, passes urine frequently but does not passes stool regularly sometimes once or twice a week and passes gas regularly what can it be ?because he doesn\u00b4t also sleep at night but feed frequently at night too. Doctor: Hi, I had gone through your question and understand your concerns.Passing stool once or twice a week may be normal if there is no abdominal distension, and passed stool is not very hard .Sometimes a thyroid profile may be done done to rule out hypothyroidism.Many baby sleep in day and are more awake in day.Hope this answers your question. If you have additional questions then please do not hesitate in writing to us.Wishing your baby good health.\u2022\u00a0\u00a0\u00a0\u00a0\u00a0DISCLAIMER: - All the information provided here is for information purpose only , it is not a substitute for the advice of a physician after physical examination , it is not intended to replace your relationship with your doctor. This information in no way establishes a doctor-patient relationship. Before acting anything based on this information do consult your doctor. I recommend that online users seek the advice of a physician who can perform an in-person physical examination"
},
{
"id": 201398,
"tgt": "Suggest treatment for erection problems and premature ejaculation",
"src": "Patient: hi doctor ...due to childhood habits now my dick is not properly stand and sperm also come fast..with in ten fifteen shake ...kindly provide me better solution so that i can help my family ............thanks doctor pls help me pls and provide me better solution Doctor: HelloThanks for your query,based on the facts that you have posted it appears that you are facing problem of premature ejaculation.Please note that premature ejaculation is due to anxiety and not due to any organic cause.Following measure will help you to boost up your confidence and getting good erection.and delay ejaculation.1) Practice regular exercise for 45 minutes followed by meditation for 1/2 an hour in the morning.2) Take high protein diet rich in vegetables and fruits and Vitamin A,C,D,E.and Zinc3)Take anti oxidants like Almonds 5-6 everyday..4) Avoid alcohol and smoking..If you are married Taking Sildenafil (Viagra)as on demand will help to get good hard sustainable erection and to have enjoyable sex.Dr.Patil.."
},
{
"id": 142500,
"tgt": "What causes persistent dizziness?",
"src": "Patient: This is a question for my mom. She has been sickly since I was a kid, she was once diagnosed with MS but was told she didnt have it. Now she is very frail looking, her sympotoms are: very weak ( she says her body feels like a weat noodle), heart rythm will beat really fast or slow, she also has fainting spells. She has seen a cardiologists they say her heart is just fine. What do you think just with very little I have told you? Thank you in advance for any info. Doctor: Hello!Welcome on Healthcaremagic!I read carefully your question and understand your concern. Her symptoms could be related to different possible causes: a cardiac arrhythmia, orthostatic hypotension, thyroid dysfunction, vitamin D deficiency or vitamin B12 deficiency, epilepsy seizures, etc.. I would like to directly review her brain MRI. It is also necessary performing further tests: - an EEG - complete blood count for anemia- blood electrolytes- an rhythm Holter monitoring for cardiac arrhythmia- vitamin D and Vitamin B12 plasma levels - a Head Up Tilt test for orthostatic hypotension- fasting glucose and thyroid hormone levels. A new brain MRI may be necessary, especially if it has passed a long time since her last one. You should discuss with her doctor on the above tests. Kind regards, Dr. Aida"
},
{
"id": 211965,
"tgt": "Son is non-verbal autistic man. On Naltrexone. Lower doses were working fine for him. When dose increased has abnormal behavior. Suggestions?",
"src": "Patient: Hello, my son Max 19yrs old non-verbal autistic man is taking Naltrexone started with 50 mg and innow increased to 200 mg advice by his psychaitrist to treat agression behavior . It was working quite well during lower doses. Now, he developed agression behavior like hit the wall and left hole, very confusing, emotional/upset easily without reasons, problem sleep, look depress, nausea and vomiting,press cough, tiredness , lost interest of any kind of activity, requested more sleep...I try to contact his psychaitrist but he 's away...my question : should I reduce the doses ? Thanks for advance Doctor: Hi, Individuals on Naltrexone, that is used to reduce self injurious and autistic behaviors in autism, have reported feeling fatigued, tired, and listless. Some individuals have reported an increase in irritability, restlessness, mental confusion, and slowing following naltrexone administration. If he was maintaining well on lower doses then you can reduce the dose till his psychiatrist returns and then discuss it with him. Do not do it abruptly, it has to be done in small increments. If you do not feel confident to do it yourself then consult another psychiatrist. Hoe this helps, Dr Anjana rao"
},
{
"id": 4107,
"tgt": "Is siphene medication required even if the cycles are normal?",
"src": "Patient: Having Siphene tablets.Hi, I have a normal 28 day cycle and my follicular scannings have showed that i am ovulating normally on either 13th or 14th day. My doctor has still adviced me to take Siphene tablets from 2nd day to 5th day of the cycle. Kindly advice if Siphene medication is required even if the cycles are normal. Thanks,Sushma Doctor: Hi,I read your query and I understand your concerns.Following is my reply:1) Siphene helps to increase egg size.2) Cycle length has nothing to do with this.Let me know if you have anymore questions.Regards,Dr. Mahesh Koregol"
},
{
"id": 197981,
"tgt": "Suggest treatment for epididymitis",
"src": "Patient: Hey I ve been diagnosed with epidymitis about a moth ago.. At first the pain was bad but after a 10 day dose of medicine the pain resolved but came back a few days later. The pain is not like it was at first in fact the its a dull pain that come and go frequently.. Its mainly on my left side but sometimes I can feel a lump on both sides.. I get a slight burn when I urinate.. How can I resolve this issue completely? Doctor: helloThanks for query .You have been diagnosed to have Epididymitis and treated accordingle with antibiotics .Epididymitis is always secondary to UTI hence one needs to get routine urine test and urine culture done to find out the organisms causing this infection and antibiotics to which they are sensitive to.Epididymitis takes long time to get cured hence you will need to take appropriate antibiotics and anti inflammatory medicine like Diclofenac twice daily for 4 weeks .Dr.Patil."
},
{
"id": 156736,
"tgt": "How to be sure about ovarian cancer?",
"src": "Patient: I need to find an obgyn in warner robins who is very good and a doctor who cares. I am hurting where my ovaries are. I have had a partial hysterectomy in 01 and my ovaries are still there. I am 50 years old and I am worriee that I may have ovarian cancer. Is there someone you can recommend? Doctor: Only just pain in ovarian region cannot be sure of cancer. It should be combination of symptom of pain, anorexia, early satiety, constipation- very non specific symptoms, and no single symptom is suggestive of cancer. get CA-125 levels, get USG abdomen, and if anything suspicious is there, doctor can further advise regarding on that."
},
{
"id": 124000,
"tgt": "What causes burning sensation in left side groin near penis?",
"src": "Patient: I have a burning sensation in the left groin side near penis. Sometimes urination relieves the pain. Sometimes, in the past, sit ups have helped to cause this pain to disappear. It s not there all the time. Do you know what it is? And are there treatments you would suggest? Doctor: Hello, With history, this burning sensation can be due to factors like urine infection or degenerative disc disease of the lumbar spine. Having a urine sample test done will help to differentiate whether the burning sensation is due to urine infection or not. Once this is clearer then we can look for MRI of the lumbar spine as due to compression of the bladder over the spinal column may lead to radiating pain and also the MRI will help reveal the soft tissue injury. Once we get the MRI reports we can plan the exercise prescription for the same. Strengthening the core will be helpful in such cases. I will not stress on urine infection more, as there is not much about the complaint of burning while passing urine. But the Stress I would make over the lumbar spine more here. Hope I have answered your query. Let me know if I can assist you further. Take care Regards, Jay Indravadan Patel, Physical Therapist or Physiotherapist"
},
{
"id": 79650,
"tgt": "Suggest treatment for pulmonary embolism in the lung",
"src": "Patient: I HAD A CLOT GO TO MY RIGHT LUNG AND WAS PUT ON LENOVAL INJECTION FOR 7 DAYS AND 5 MG WARFARIN FOR THREE MONTHS,,,,I WAS TAKING 500 MG PHENYTOIN A DAY AND WAS CUT BACK TO 100 MG A DAY..WILL THIS MESS UP MY DILANTIN LEVEL AND PUT ME A THE RISK FOR A SEIZURE...MY LEVEL STAYS AROUND 12 AND LAST SEIZURE WAS IN 2004 Doctor: Thanks for your question on Health Care Magic. I can understand your situation and problem. Phenytoin is having drug interaction with warfarin. So both will interact with each other and decrease their therapeutic levels. And hence desired anti epileptic and anti coagulant effect can not achieved. So possibility of seizure attack and recurrence of Pulmonary embolism is high. So in my opinion, you should continue warfarin. Better to start newer anti epileptic drug. Newer drugs are having less interactions and more effectiveness. So better to consult your epilepsy doctor and discuss about changing phenytoin to avoid drug interactions with warfarin. Hope I have solved your query. I will be happy to help you further. Wish you good health. Thanks."
},
{
"id": 120850,
"tgt": "What does burning sensation on sprained ankle indicate?",
"src": "Patient: I m pretty sure I sprained my right ankle yesterday. I can put weight on it, there is little to no bruising and the swelling has gone down a little but I have a constant burning sensation on the outter side of my ankle going up my calf a little. Is this something I should worry about. I plan on going to work tomorrow. Doctor: Hello,Your symptoms seem to be related to the sprained ankle. I suggest using anti-inflammatory medications such as Ibuprofen to relieve the pain. I recommend to use cold compresses for local application. I suggest to use a splint to prevent further damage of the ankle.Hope I have answered your question. Let me know if I can assist you further. Regards, Dr. Dorina Gurabardhi, General & Family Physician"
},
{
"id": 79045,
"tgt": "Suffering from chest pain for over 2 years",
"src": "Patient: i have chest painfor the last two years.iconsulted a cardiologist and he who after seeing the the same told me that ihave no heart related problem. But I am sorry to say that i am still suffering the pain often which becomes sometimes quite painfull. pl. suggest some remedy. Doctor: You need to undergo a chest X ray to see if there is any problems related to your lungs. And also if you have acidity/heart burn related issues you would require treatment for the same as well."
},
{
"id": 131136,
"tgt": "How to treat a prolonged weak feeling wrists that has increased in it pain?",
"src": "Patient: I have had issues with my wrists feeling weak in the past, like years ago. Now, I suffer from wrist pain, but not only that, it seems my hands are clumbsy. I did experience some tingling here and there. However, since I have not been getting answers from the places I have been going to seek help, I have done research which led me to isometrics. I was doing it faithfully for a while. It seemed to help, however, I still had a good amount of pain but it was less than what I had been experiencing. And then, it seemed that it didn t help at all after about 2 weeks maybe. It was only temporary relief, because then I started feeling the pain travel thru my arms and on into my shoulders. I would wake up in the middle of the night from Pressure/pain across my shoulder blades in the front or sometimes in the same area but on my back side. One doctor, said I had carpal tunnel, but couldn t do further testing because of my inability to pay. So then I was approved for Obamacare. I saw an orthopedic doctor who thought it was not carpal tunnel but possibly tendinitis with a touch of rheumatoid arthritis. However, the very first doctor I saw drew blood to check for rheumatoid and it came back negative. So anyhow, the orthopedic doctor referred me to a hand surgeon. The hand surgeon said he thinks its a problem with a possible pinched nerve. And during my exams it is hard to associate the different feelings in my hands as the doctors are examining me, because I have not been given anything to manage my pain level. So now I am confused and really I just want to get to the nature of the problem so I can have it fixed. Doctor: Only way to know for sure is by nerve conduction studyHowever if pain reaches your shoulder blades then you should suspect cervical lesion not carpal tunnel , i recommend a cervical MRI or a simple x ray can do if you can show it to me Good Luck"
},
{
"id": 165987,
"tgt": "Suggest remedy for pain while passing motion and urine",
"src": "Patient: my 3 year old has pain when passing motion all the time and also the past few weeks he has complained of his wee hurting and last week i noticed his willy was swollen so i put warm water on a sponge to wipe it and all milky puss came out the end and it had a horrible smell. the next day he seemed fine and didnt want to go to the doctors but im concerned as to why he keeps getting problems going to the toilet? he has also got like skin tag spot under his chin which i took him to doctors for she named them but i cant remember she wasnt very helpful....she said they would get worse before they got better and not to pick them but they are not warts and not contagious Doctor: Pain while defecation may b due to anal fissure or passing hard stool. U should contact ur pediatrician for an per rectal examination.Pain and pus dischage while urination may b due to urinary tract infection. constipation is a risk factor for that. U should go for urine microscopy and culture."
},
{
"id": 122018,
"tgt": "Suggest treatment for sprain",
"src": "Patient: My husband got a 2nd degree sprain 3.5 weeks ago. Every since, his parts of his foot and his toes feel like they are asleep. Is this normal? Also, everytime he tries to put partial weight on it, it hurts. How much pain is normal when he tries to walk? Should he continue to try to walk with pain? Doctor: Hello, For sprain to heal soon, rest is very important. Please ask him to rest with his leg elevated, apply ice packs and crepe bandage. If the swelling or pain increases, take over the counter anti-inflammatory medication. Hope I have answered your query. Let me know if I can assist you further. Regards, Dr. Haleema Yezdani, General & Family Physician"
},
{
"id": 190945,
"tgt": "What does purple blood near tooth mean ?",
"src": "Patient: My tooth has been hurting to floss on and off for a bit, when I flossed there today it bled a lot and I the part of the tooth closest to the blood was purple. Even after i washed the blood out its purple. What does that mean? Doctor: Hi, if the tooth is discoloured, there may have been a trauma to the tooth resulting in pulp necrosis or pulp death. The surrounding gums may also be bleeding due to the same reason. The trauma may be mechanical, chemical, exposure to high temperature due to consumption of very hot foods. All these result in pulp and gum trauma. Please apply Mucopain ointment on the gums and consult a dentist to know about the treatment plan for the tooth. You may need root canal treatment with a crown for the tooth. Take care!"
},
{
"id": 138956,
"tgt": "Suggest treatment for stiff neck",
"src": "Patient: I am 45 and hurt myself playing volleyball it started as a stiff neck and turned into muscle pain in my neck upper back and shoulder. The only releif I get is if i keep my shoulder in the correct posture position. I have been taking aleve or tylenol and it is not helping. Any other suggestions ? Doctor: Hi,Thanks for your query.From the description that you have provided, it seems that you have suffered a muscle sprain. You need to provide rest to the neck- preferably use a cervical collar. Avoid massaging the area. Doing a hot fomentation will be helpful. Avoid all overhead activities and do not use a pillow.In case the pain is persistent even after taking a mild pain killer, then you might need stronger prescription medicines like muscle relaxants after consulting your orthopedician.I do hope that you have found something helpful and I will be glad to answer any further query.Take care"
},
{
"id": 22943,
"tgt": "Can these blood pressure medications be stopped?",
"src": "Patient: Hi there, about 5 months ago I went to see a Dr and he prescribed me coversyl 4mg and amlodipine besylate 5mg, because he said I had high blood pressure. I was having a bad anxiety at the time, and since then I've been on these medications. I have never been sick, and I never had any of the side effects he was asking me. But he said you must have you just don't know. Ever since, every time I go and see him my blood pressure seems to go up, and I get really anxious. So now I am looking for a second opinion on what I should do, because he has really scared me, and said it's called the \"silent killer\", so now my anxiety/panic attacks are bad and I've been seeing a mental health worker. My other question is can I stop taking these meds on my own? Doctor: Don't stop medicine. Although high blood pressure is problematic but if it's controlled then there is no problem. You should get one blood pressure apparatus and monitor your blood, initially three or two times a day then once e week. Consume low salt diet and have healthy lifestyle, including regular exercises. If your blood pressure is under control then we can think of tapering blood pressure medication, you will monitor it since high blood pressure is usually asymptomatic. You also get your sugars and lipid profile done. And definitely no need to worry at all."
},
{
"id": 97884,
"tgt": "Patient of Ankolysing Spondylitis, after taking SAZO500 HB rapidy fell down. Is there any alternative for this drug that has no such side effect?",
"src": "Patient: Hi , My Husband is a patient of Ankolysing Spondylitis. After taking SAZO500 his HB rapidy fell down. My Rheumatology consultant has prescribed SAZO. Is there any alternative for this drug that has no such side effect? And I also want to kknow about the proper treatment of Ankylosing Spondylitis in India? Please kindly Help me. My husband is a construction Engineer so he has very hectic work load all the time. Please suggest me. I am a helpless wife. Doctor: ANKYLOSING SPONDOLITIS IS BELIEVED TO BE VARIANT OF RHEUMATOID ARTHRITIS , WHIICH IS AUTO IMMUNE IN NATURE AND CLOSELY LINKED TO GENE HLAB27 . HERE SYMPTOMS ARE NOT CONFINED TO PAIN ONLY BUT DEFORMITY OF SPINE AND RIB CAGE IS MAJOR CONCERN .SO EARLY DIAGNOSIS AND TREATMENT CAN PREVENT DEFORMITY ALL ANTI MITOTIC DRUGS TENDS TO REDUCE HEMOGLOBIN PERCENTAGE , TOTAL RBC COUNT AND LEUKOCYTES COUNTTHERE ARE SO MANY HERBAL DMARDs[ disease modifying anti rheumatic drugs ] AND IT CAN BE COMPLETELY CURED BY HERBAL DRUGS WHICH ACT AS ANTI INFLAMMATORY DRUGS AND DMARDs AS WELL BETTER CONSULT SOME NEAR BY QUALIFIED AYURVEDIC DOCTOR"
},
{
"id": 164943,
"tgt": "What causes constant fever, body aches and burning sensation in the eyes?",
"src": "Patient: my 8 yr old child has a fever since yesterday afternoon when he first complained of a head ache I gave him Ibuprofen and after 3hrs his fever had was at 102 then i proceeded to give him Tylenol and alternate between Tylenol and Ibuprofen every 3 hrs. 6 hrs between each dose of Tylenol and 6 hrs between each dose of Ibuprofen. He has been crying off and on. This morning he complained of body aches that would come and go in different places arms legs big toe it seemed. Now he is saying his eyes burn. I m not sure what to do. Doctor: Thanks for consulting at Healthcare Magic.Fever, headache, burning eyes, body aches etc. probably suggest your child is suffering from viral infection.Give Paracetamol 15mg/kg 6 hourly, Cold sponging, If fever not controlled add Ibuprofen 10mg/kg 8 hourly, Give juices, oral fluids to maintain hydration.Consult pediatrician if no relief."
},
{
"id": 222046,
"tgt": "What causes stomach sickness and bowel movement pain during a pregnancy?",
"src": "Patient: Am I pregnant it hurts to go number 2. Everything I eat makes me feel sick I had sex the past 3 nights and been off my bc for 3 days counting today! I have been pregnant before but lost baby and for some reason I don t remember much of pregnancy eben though it was 12 weeks. Doctor: HelloSorry to hear about abortion.Are you suffering from constipation?Is your stool hard? If yes take plenty of fluids, include fibrous food in your diet.You have not mentioned date of last period it is difficult to say about pregnancy.I hope to hear from you Dr.Mira Butani"
},
{
"id": 75429,
"tgt": "What causes pectus cavernosum?",
"src": "Patient: I have pectus cavernosum, Ive had it for as long as I can remember.Its always bugged me and now that im older im self concious about it.I heard getting implants can make it look less obvious.what are some options for this condition.Im 18 years old around 5 5 and about 110 pounds. Doctor: You may consult a cardiothoracic surgeon or a plastic surgeon for correction of deformity. It is better to have it corrected not only from asthetics point of view but also for any deficiency it may cause in the functioning of lungs."
},
{
"id": 88517,
"tgt": "What causes lower stomach pain after intercourse?",
"src": "Patient: actually me and and my boy friend had a unprotected sex on 6th but he did not insert but rubbed his penis into my vagina,we had it on may 6th and i had my periods on 27th april.i took immediately 2 hoours after on may 6th but am having lower stomach pain now..am i safe plz suggest me Doctor: Hi.Thanks for your query.Read your history and understood the problem.As per the history you have provided, your boy friend rubbed the penis into your vagina but you say there was no insertion. If no insertion, the chances of pelvic infection is remote, rather not possible.The cause of pain in your lower abdomen then can be due to :Enteritis,Any problem related to the Fallopian tubes or Ovary.PID-Pelvic Inflammatory disease if you had sex before.Appendicitis.I would advise you the following:Get blood and urine tests.Ultrasonography of the abdomen and pelvis to get an idea.Examination by a Gynecologist and / or Surgeon to see for the probable cause. Start treatment with an antibiotic and metronidazole, probiotics and supportive and symptomatic treatment under the guidance of the Doctor ."
},
{
"id": 82879,
"tgt": "Suffering from lupus. What does CT scan after panic attack suggest?",
"src": "Patient: Hi my name is Ronnie and I m 39 years old I have lupus. I recently went ion the hospital for a panic attack and they did a ct scan. The result did not show any blockage but it s stated that their where several subcentimeter para tracheal lymph nodes. I don t understand what they mean by that. Please help me understand. Doctor: Dear Madam,You need to tell Ct scan of which all portions was done because you are mentioning findings of ct chest which are not otherwise indicated in panic attack generally . Kindly elaborate.Dr. Shruti"
},
{
"id": 69419,
"tgt": "Suggest remedy for lumps in neck",
"src": "Patient: I have two lumps in my neck, one i small and more firm, while the other one extends a little past my ear and is a bit larger. I also just recently noticed i have a small lump on the opposite side directly under my ear. The (smaller) lump on my neck hurts at times. My sister told me it is swollen lymph nodes but i'm not sure. Doctor: Hello!Thank you for the query.It might be a swollen lymph nodes indeed. Especially if you have had ear or throat infection recently. Due to infection, lymph nodes usually get larger and painful.If you have also enlarged lymph nodes in other areas like armpit or groins, lymphoma should be considered.I suggest you to consult your doctor with this issue. Ultrasound of this lumps should be done. Hope this will help.Regards."
},
{
"id": 188552,
"tgt": "Constant discharge of pus from gums of front tooth, bad odour. Had root canal treatment. Remedy?",
"src": "Patient: I had a root canal treatment about 3 weeks ago after a discovered that pus was coming out right in front of my front tooth. I'm surprised that the pus is still coming out. i went back to the dentist who examined my tooth and the gum. she explained that pus will stop, and that my gum is fine. I'm worried because it's the front tooth that is affected and my mouth smells of bad odour with bad taste. I have just bought listerine for mouth wash in addition to the mouth brush. what is the way out? Doctor: hello and welcome to HCM forum,I would like to inform you that if the root canal treatment has been completed with a permanent filling on the tooth then pus discharge is not expected but if your dentist has placed a temporary medication in your tooth then it might take a few days for the pus discharge to stop and infection to resolve completely.I will advise you to take a second opinion and visit an endodontist.For now, you can use a mouth wash twice/day as well as perform warm saline rinses 3-4 times/day.The dentist might have to prescribe you antibiotics to resolve the infcetion which still persists.I hope this answer was useful,I wish you good health,take care."
},
{
"id": 56666,
"tgt": "Can a failed liver revive after a week?",
"src": "Patient: A friend has been diagnoised with liver failure due to psorisis, no treatment available for his situation. His spouse, who is in denial of the diagnoises, told me today that the doctors are now saying he has a parisite in his stomach and there is a chance of his liver regenerating itself after taking meds to kill the parisite. I question whether a liver that has shut down a week ago can start up again Doctor: Dear Friend, greeting s from HCM. I understand your question ... Liver if it is inflamed ./ swollen .. It can heal and function normally .. But cirrhosis indicates that liver tissue has been completely replaced by fibrous tissue .. Which cannot be replaced by normal tissue ... Liver keeps quite till the end without giving any sympyoms ....You please find out in which stage he is in .? You need a proper complete evaluation by a gastero / hepatobiliary surgeon now and find out the cause if it is a parasite definitely there are chances ... Thank you .... Do contact is for more questions. .."
},
{
"id": 74550,
"tgt": "What causes pain in the chest with discomfort?",
"src": "Patient: I took a pill Friday noon leaving me with the sensation it was still in my throat. Hoping drinking more water would do the trick I drank more, although it felt as though the pill was still stuck there. It is now Sunday & now I have somewhat of a chest pain right in the middle, along with an uncomfortable pain every time I inhale deeply, cough, or laugh really hard. I did some Googling and it appears I may have damaged my esophagus? That or it is infected & filled with liquids? I was just wondering if a doctor's visit was completely necessary or if it would go away on its own? If there are any tips on how to make this horrible feeling go away on its own, please let me know for I don't have the money to be going to doctors at the moment. Another question is how long would it take to get rid of this feeling? Please & thank you... Doctor: Respected user , hiThanks for using Healthcaremagic.comI have evaluated your query thoroughly .* This discomfort relates to esophagitis caused due to the issue .* Certain guidelines for primary relief , if not must see doctor immediately - Drink plenty of liquids in sessions avoid hot drinks , beverages - Soft , light diet at intervals avoid oily , spicy , non veg - More of high fiber content in food - Gargles with apple seed vinegar 2 - 3 times a day - Cap. Prilosec ( 20 ) mg and tab. tylenol ( 500 ) mg 2 times a day after meals .Hope this clears your query .Welcome for further assistance .Regards ."
},
{
"id": 159195,
"tgt": "Have non ulcerated, non mitogenic malignant melanoma. How serious is it?",
"src": "Patient: Husband dx malignant melanoma , Clark s level II; Breslows thickness 0.5mm, non-ulcerated, non-mitogenic, atypical melanocytic proliferation. Shave biopsy , as the derm did not think it was much concern. Radial growth phase is present. Vertical not present, small specimen. Appears to have partial dermal regression. How serious is this, recommended return for re-excision with appropriate surgical margins. Doctor: Hi, Malignant melanoma is notorious for its tendency to recur and high chance of systemic failure. As your case is concerned most likely has not got that high grade of invasion. again total surgical clearance must be done . negative surgical margin you must have to secure. please consult your surgeon."
},
{
"id": 115648,
"tgt": "Suggest treatment for Myelodysplastic syndrome",
"src": "Patient: Hi, may I answer your health queries right now ? Please type your query here...My Question is my brother is suffering from Myelodysplastic syndrome.Rcmd andWBC COUNT falling up to 3/4.we are giving blood transfusion every 15 days however it falls after week. we have done bone narrow test.however report is clear. Please advise.Thanks Doctor: Hi, dearI have gone through your question. I can understand your concern. He has myelodyplastic syndrom. So his count will remain low. Chemotherapy and radiotherapy may be helpful according to type and stage of disease. Stem cell transplantation is also helpful. Consult your doctor and take treatment accordingly. Hope I have answered your question, if you have doubt then I will be happy to answer. Thanks for using health care magic. Wish you a very good health."
},
{
"id": 225511,
"tgt": "Missed period, stomach cramps. Taken contraceptive medication. Had safe sex. What do I expect?",
"src": "Patient: Hi me and my boyfriend had intercourse and came , then cleaned himself and changed condoms and we did it again , later the condom broke so I ammediatly went to wash my self later I took a next choice one dose emergency contraceptive but not quiet sure If I swallowed or it fell it because I was driven while I dranked it with water ,2 days later my period stopped and my stomach has been hurting , but im still scared to get pregnant is it okay if I take a next choice one pill again??within two days please help Doctor: Namaste Welcome to Healthcare-Magic Greetings of the day Ideally emergency contraception is to be taken within 7 2hours of unprotected sex, if you are not sure then there is no harm in taking one more. So go ahead and take it. Take care Regards"
},
{
"id": 43857,
"tgt": "Trying to concieve, irregular periods. Pain in hips and back. Will I get a positive pregnancy test result?",
"src": "Patient: i am trying to conceive,but after marriage my periods goes irregular. and also i have pain in my left side hip to left side back . this pain starts from my mid of cycle and continue for about 2 weeks............... this month i expect positive for pregnant but i start get that pain for about one week. my last period is 13 th october. whether i get positive ???????????. sa Doctor: Hello It is not unusual for women to have period irregularities at some point in their life. Many women complain of this with changes in their life like exams, marriage etc. It should gradually settle down. About the pain, see a gynecologist if the pain continues, although it does not really sound like period pains. If you go overdue, do a pregnancy test to make sure. Best wishes."
},
{
"id": 101787,
"tgt": "Will microdermabration cure allergy in the face?",
"src": "Patient: i have some history of allergic problem on my face, it started 2 years back. after getting itching and irritation i develop eczema on my face and it last for 2-3 days. i got it for 3-4 times till now and after that i got severe tan on my face i heard that glycolic peel is effective and i have another problem like small pores on my face which occured after comedone extraction. Will microdermabration help this or not? Doctor: Hi and thanks for the query,It could offer mild help, but important and remarkable results could come from elsewhere. Associating systemic steroids and anti histaminic drugs could be more helpful. Fighting against infection of existing lesions could also be helpful. The opinion of a dermatologist would be most welcome. kind regards"
},
{
"id": 214318,
"tgt": "How to remove kidney stones naturally?",
"src": "Patient: hai ! my cousin brothers dad is suffering from kidney stone of 19 mm and chronic infract of brain.sme doctor told its a huge sized stone its need surgery but also they told bcz of infract we cant operate it nw ! so i m askng you can it possible to remove the kidney stone of 19 mm naturally by ayurvedic medicine !!! Doctor: , large stones in the kidney can be removed by extracarporal shock wave lithotripsy. using laser the stones can be fragmented and the fragmented stones will pass through the urinary tract.there are ayurvedic medisines such as dystone and cystone it is proclaimed that they will prevent further formation of stones.use of banana trees water is also useful for preventing further formation of stonesbut the best thing i recommend is to do a extracarporal shock wave lithtripsy for him.hope this explains with regardsDr.Amarnath"
},
{
"id": 104801,
"tgt": "High or mild temperature with cold and flu during winter. Medication?",
"src": "Patient: Hi, this is Basith, I had a query in regard to the common cold n flu more particularly is winter season. I allways had a problem with runnig nose n cold due to which I even suffer with a high or mild temp.plz do advise me of one of the best medicine available in the market for the problem. I also wanted to know, is this are the symptoms of cold allergy and if yes what is the best medicines suggested to be taken in advance before the attack. Doctor: Hi, Welcome to HCM, This is a viral infection caused by various virus,like Rhinovirus,Corona Virus,Adeno virus and Influenza. See recovery takes place in 3 to 7 days,treatment is symptomatic like warm saline gargle,nasal decongestant and antitussive for cough. Personal hygiene and and repeated hand washing is best preventive measure. Although a INTERFERON NASAL SPRAY for prophylaxis for Rhinovirus infection. Even addition of high dose of vitamin C is claimed to reduce duration of illness.In adults with good immunity these episodes are much less in numbers /year. Common cold and flu should not be confused with allergic Rhinitis which is due to pollen and dust. Take care Good Luck"
},
{
"id": 213462,
"tgt": "Black out. EEG done, diagnosed JME. Prescribed keppra. Anxiety, loss of concentration. How can it be treated?",
"src": "Patient: hello, when i was 15 close to my 16th birthday i was diagnosed with JME. I had been experiencing episodes in the morning related to sleep and one day i had a big one which cause me to black out and fall down the stairs i ended up in hospital and had an eeg which i was diagnosed with JME. So i was put on keppra i am now 16 and i have been on keppra for a short time but i feel as if i already am having some side effects, i get really down and gloom: where i find myself sitting on my bedroom floor for a while. Ive also notice a change in my temper i get angry alot quicker then i used to at the smallest things. i cry alot and am feeling a lot of anxiety im also struggling with concentration and need sleep all the time! im doing my gcses at present and am really worried about the fact being on this medication is affecting my chances of doing the best i can in my exams. has anyone noticed this to? or know what i can do to limit this? Doctor: Hello Dear Thanks for the questioning in HCM Dear the all symptoms you are explaining above in your question are the side effects of Keppra. Other side effects are Dizziness; drowsiness; irritability; sore throat; tiredness; weakness, memory loss; muscle or neck pain; new or worsening mental, mood, or behavior changes (eg, aggression, agitation, anger, anxiety, apathy, depression, hostility, irritability, panic attacks, restlessness)etc. As you are suffer from JME you have to take this medicine and to get rid of these symptoms you must consult your neurologist for better opinion. Regards Dr. Dinesh"
},
{
"id": 172637,
"tgt": "Which medicine to be taken as my daughter is having high fever?",
"src": "Patient: my 4.5 year daughter is having high fever of 102 FH. She has been taking Septran Antibiotic for the last 2 years related to urinary infection. One of the doctors prescribed Fepanil 250 mg but fever is not subsiding right now. What should be the medicine to be consumed now? Doctor: Pls check with urinary culture to rule out current episode of u t I ...can continue patracetamol pending cultures...if cultures is positive you have to see the doctor for appropriate antibiotic"
},
{
"id": 30049,
"tgt": "What causes green nail discolouration while having Sjogren's syndrome?",
"src": "Patient: I was just diagnosed with Sjogren's Disease. I will be having a lip biopsy on Friday to confirm, but the doctor has already put me on medication. I have gel polish on my natural nails & recently had a green fungus on 2 of my nails. I have never had this with 5 yrs of gel polish. Now one of my nails is lifting from the nail bed on one of my nails. Could this be caused my Sjogren's? I have never had a nail lift like this. Thank You Doctor: HIWell come to HCMI really appreciate your concern, if this is the fungal infection then it is likely that it would affects the nail bed and the lifting of nail bed is not because of Sjogren's disease, still the clinical diagnosis would be the last judge as long as the differentiation is concern, but it is not due to Sjorgren's disease hope this information helps."
},
{
"id": 52888,
"tgt": "What does pancreatic parenchymal abnormality consisting of hyper echoic foci mean?",
"src": "Patient: I need this translated into lamen terms paincreatic duct had an irregular contoured endosonographic appearance and had hyperechoic wallsin the tail of the paincreas and paincreatic parenchymal abnormalities consisting of hyperechoic foci and lobularity were noted in the pancreatic tail Doctor: this is USG findings consist with chronic pancreatitis...we need CT scan for further information on this mass...you may require surgery for it"
},
{
"id": 65104,
"tgt": "What causes hard and painful lump above left clavicle?",
"src": "Patient: I a am 40 year old female who observed a lump above my left collarbone approximately two weeks ago. A recent ultrasound revealed a one inch mass above my left clavicle and another small lump above the large lump. The larger lump is palpable and visible. I am very concerned about these masses. The large mass is hard and painful.. The ultrasound technician informed me that the large mass lies above a blood vessel. I anxiously await my appointment to learn the results of this ultrasound. Can you please share your thoughts? Doctor: Hi, dearI have gone through your question. I can understand your concern. You may have supraclavicul lymphnode or mass. It can be due to reactive hyperplasia, tuberculosis, lymphoma or metastatic carcinoma. You should go for fine needle aspiration cytology or biopsy of that mass. It will give you exact diagnosis. Then you should take treatment accordingly. You should also ho for x ray chest. Consult your doctor and take treatment according to diagnosis. Hope I have answered your question, if you have doubt then I will be happy to answer. Thanks for using health care magic. Wish you a very good health."
},
{
"id": 149718,
"tgt": "Restless Leg Syndrome got worse. Nothing reveled in MRI. Relief with Sifrol. Permanent cure?",
"src": "Patient: I have severe Restless Leg Syndrome which has got worse over the last 12 months. An MRI revealed nothing and my Neurologist prescribed Sifrol which worked like a miracle but only for a few months. I was prescribed Tegretol today however this has made me very agitated and distressed and have been roaming the house for 4 hours tonight. I was also provided with a prescription for Gabapentine (100mg) but am a bit scared to try this in case it has the same effect as the Tegretol. I am so tired and at my wits end. Doctor: Hi,Thank you for posting your query.Pramipexole (Sifrol) and other dopamine agonists (such as ropinirole) are the best drugs for restless legs syndrome. So, my suggestion is to continue them. It may happen sometimes that the effect seems to wear off, but it may become effective again.Gabapentin and pregabalin have more chances of working in your situation, as compared to tegretol. So, you can try gabapentin.Please get back if you require any additional information.Best wishes,Dr Sudhir Kumar MD (Internal Medicine), DM (Neurology)Senior Consultant NeurologistApollo Hospitals, Hyderabad,My personal URL on this website: http://bit.ly/Dr-Sudhir-kumar My email: drsudhirkumar@yahoo.comMy blog: http://bestneurodoctor.blogspot.com/"
},
{
"id": 103477,
"tgt": "Developed breathing congestion. Have allergies. What can help me cope them?",
"src": "Patient: Hi im from Brooklyn, NY and have been going up to Long Valley New Jersey to visit my girlfriend every weekend. At first it was never a problem, until recently ive developed a breathing congestion everytime i go up to visit. I figured it was my allergies or something that I could be allergic to in her house. She lives in the country side of jersey where its mainly farmland, trees, and less air polution unlike NYC. Its hard for me to last up there for more than a couple hours since my breathing becomes very difficult. Almost like im sufferacting. Is there something I could take? I will be going up again soon and wanted to find a solution or at least a temporary one. I appreciate all advice and help I can get, thanks! Doctor: THERE CAN BE ALLERGIES TO POLLENS AROUND YOUR FRIENDS HOUSEYOU CAN TAKE TAB MONTELUCAST 10 MG BD FOR DAYS YOU ARE EXPOSED TO ALLERGENSBRONCHODILATOR SYP BD OR TDSAPPLY NEOMYCIN H EYE OINTMENT IN NOSE BD AND SEA WATER DROPS IN NOSE 2 DROPS BD THESE WILL ACT AS PREVENTORS FOR YOUR ALLERGIES INITATION"
},
{
"id": 224928,
"tgt": "What could be the reason for sharp pains in vagina and bleeding after skipping birth control pills?",
"src": "Patient: I drank with friends one weekend and skipped my birth control for 2 days and the very next day i got my period. A week later my period stopped and i started getting sharp pains in my vagina and I started bleeding again. Did I have a miscarriage then a regular period or a elongated period with sharp pains? Doctor: HI, thanks for using healthcare magicIt is possible that it is an elongated period. Miscarriages are associated abdominal cramping, clots and the passage of ' fleshy products'.If you continue to have periods with prolonged bleeding then you need to consider visiting your doctor for an assessment.This would include physical examination, hormonal level blood tests and imaging of the reproductive organs by ultrasound.i hope this helps"
},
{
"id": 73149,
"tgt": "What is the treatment for chest wall syndrome?",
"src": "Patient: I recently had an EKG with normal results (2 weeks ago). Was told I had chest wall syndrome. The last couple of days, the pain has been worse, feeling like a sore chest muscle. I just took blood pressure at Publix with a reading of 134 over 82- the worst I've ever had. I am 40 and work out regularly. Doctor: Thanks for your question on Healthcare Magic.I can understand your concern. No need to worry for heart diseases as your blood pressure and ecg are normal. Do following things for better symptomatic relief. 1. Avoid movements causing pain. 2. Take painkiller and muscle relaxant drugs like ibuprofen and thiocolchicoside.3. Apply warm water pad on affected areas.4. Avoid heavyweight lifting and strenuous exercise.5. Wear chest belt.You will mostly improve with all these in 1-2 weeks.Hope I have solved your query. I will be happy to help you further. Wish you good health. Thanks."
},
{
"id": 109901,
"tgt": "What causes pain in different sites in back?",
"src": "Patient: Hi, I am 35 years old, female. I have had upper back pain since two weeks ago. It just started one morning from shoulders, then started radiating to lower for few days, now it has reached my tail bone. But I have sharp but very short pain in different parts of my back just jumping from one location to other. I had right and left rib x-ray (because at that time pain was in upper). I am going to see my family doctor. I am ANA positive with last titer 1:160 (15 months ago), Homegenous pattern. I was tested for Lupus as that time with negative results. Do you have any clue for now? Is this can be Ankylosing Spondylitis? is this a potentially life threatning? Doctor: First of all..this is not life threatening. And it doesnt sound like ank spond as well. You must get some physiotherapy and start back muscle strengthening exercises. Get a clinical evaluation done by a spine surgeon if you do not see some relief in 3 to 4 weeks time. Vitami D deficiency is another factor that should be considered in your case."
},
{
"id": 162565,
"tgt": "Is Zifi effective for stomach infection in an infant?",
"src": "Patient: My son is 5.5 month old.. From 2 ND month he is having dark green stools full of mucus 6-7 times in a day and little poo with every urine 12-14 in a day.. Entirogermina and dios-sf doesn t work.. Is it safe to give zifi 100 to him.. Doctor recommended 4 ml twice a day.. Does zifi help Doctor: Hello and Welcome to \u2018Ask A Doctor\u2019 service. I have reviewed your query and here is my advice. Zifi dose in your child would be 50 mg per day in two divided dosage. Yes, it sage but you have to check quantity and child diet also. Hope I have answered your query. Let me know if I can assist you further."
},
{
"id": 201919,
"tgt": "Suggest treatment for erectile dysfunction",
"src": "Patient: I am 28 yeas old. I am into the habbit of Masturbate for the past 12 yrs..... And now i feel that my penis erection is very bad. I am getting married next year. Is it possible to get my penis erection back. If not, i can take action to stop the marriage.... Kindly advice me.... Doctor: Hello dear,First of all remove all myths & misconceptions from your mind about masturbation. In fact, Masturbation is completely normal & doesn't have any bad effect to the body and mind.Regarding the problem of Erectile dysfunction, it is due to decreased blood supply to the penis.And medications like Cialis & Viagra increase blood flow to penis & maintains erection. It can also be improved by having a diet rich in fruits, vegetables, fish, nuts & honey.Avoid stress...practice meditation..this helps in improving concentration & builds up the confidence level. Avoid smoking & alcohol. You can also do oil massage of the penis...it increases blood flow & maintains the erection.Despite these measures, if you experience difficulty, it will be better to rule out other causes like high blood pressure, blood sugar and other local causes that impair blood flow to the genital organ.There is no need to worry.Wish you a Happy Married Life. Take care."
},
{
"id": 176219,
"tgt": "How to breast feed a baby?",
"src": "Patient: my baby is 81/2 month old, because of nurses ignorence and less knowledge baby ddnt accept mother feed, i tried alot bt not succssesful now i really want to start giving him mother milk bt how and is this possible at this age . and when i pump it just shows drops on the nipples...pls suggest.. Doctor: Breast milk is the best food for a baby upto 6 months of age. After that semisolids need to be introduce as breast milk can no longer meet the demands, although breast milk can be continued till 1 year of age or more. But please note that breast milk is produced as per the requirement of the baby. If the baby is hungry, the baby sucks more and therefore more breast milk is produced. Once you introduce external feeds, the baby is no longer hungry and hence breast milk production falls. Breast milk production has already been depressed so long and it is hard to start afresh at this point of time. Still, you may try giving Lactonic granules 1 sachet in a glass of milk twice daily to the mother and see if it makes a difference in breast milk production. It should help at least to some extent."
},
{
"id": 44342,
"tgt": "Trying to conceive. Follicular study done, blood test, pelvic scan normal. Treatment?",
"src": "Patient: I am trying to get pregnant and i dont know whats wrong.my doctor has started follicular study from 3 months but still no progress as my reports of blood test , pelvic scan everything is normal.i had miscarriages 2 times,so know its too frustrstion for me.taking folic acid , multivitamin tablets.my husband reports are also normal,so please suggest what to do . Doctor: Hi, Thanks for your query. I read your query and I understand your concerns. Please see an infertility specialist with details of miscarriage reports to evaluate the reason for miscarriage & pregnancy loss. Try having intercourse during ovulation which is helpful for conceiving. Ovulation days will be told by doctor doing follicular scan. I hope I answered your query. I will be available for any followup queries you have. Regards, Dr.Mahesh Koregol IVF & Infertility Specialist."
},
{
"id": 200897,
"tgt": "What causes pain on one side of penis?",
"src": "Patient: HI today I was in a chair and felt a small pain on the right side of my penis , a little I felt a small crawling on the skin of my uncircumcised penis , what could this be is it something to worry about or is it something should just keep an eye on ? Doctor: Thanks for asking in healthcaremagic forumIn short: If its only one time leave it.Explanation: Sometimes stretching of skin/muscle of inguinal region can cause pain. So, if it only one episode and after that if you are normal then be cool. If its severe and associated with other symptoms then visit a doctor."
},
{
"id": 157302,
"tgt": "Lump on left side of rib cage, tender, painful, cancer ?",
"src": "Patient: Hi, I am a 54 year old woman. I am healthy and thin. I recently had a cbc for some gastro problems. Blood came out normal. Sunday 4/15/12 after my usual workout I took a shower and felt an o obvious lump on the left side of my rib cage. Its tender and painful. I didn t have lump Saturday. No other symptoms so far. Could this be cancer of some sought and what kind of doctor should I see. Thank you for your expeditious response. Doctor: Hi and welcome to HCM,thank you for your query.In most cases this is just benign subcutaneous tumor and cancer is the last thing to think of. If there is any suspect of malugnancy you should do ultrasound with biopsy to rule this out.but as I said this is usually just lipoma,fibroma or muscle injury.Wish you good health. Regards"
},
{
"id": 7155,
"tgt": "Why is Norethisterone tablets I.P 5mg DUB-5 prescribed ?",
"src": "Patient: I had sex during my periods and after 2 days also. After a month i missed my periods. i got for the check up and the result was negative. and doctor prescribe me Norethisterone tablets I.P 5mg DUB-5 what does this medicine do and can i get pregnent?my age is 24 Doctor: Hi Welcome to HCM Read your prob, ovulation occure around 12th to 18th day of period, if you do unprotected sex at that period then there is more chance to get pregnant. you had sex during period n 2days of period, at this time ovulation does not occure. so there is no chance of pregnancy .so dont worry. do as per advise of your doc. this tab helps in regularing your period."
},
{
"id": 130490,
"tgt": "What is the treatment for swelling in the toe?",
"src": "Patient: Squashed my right leg big toe, while playing soccer a few times, redish black strait ions become a little yellow, twisted my ankle 2 weeks back and tied a creap bandage for couple of days, toe was numb on and off when I where shoes, now wAs swollen after a short 2 hrs flight, and numb Doctor: Hi,Thanks for contacting. Do hot and cold therapy ( take a bucket of hot water and one with cold water, dip your leg first in hot water for 3 mins then in cold water for one min. Do it 7-10 times, and keep your feet elevated. Don't keep your legs hanging, as it might increase swelling, and do use crepe bandage . Hope it will help you. Let me know if I can assist you further.Regards,Dr. Harsh Swarup"
},
{
"id": 47833,
"tgt": "Suggest medications for gout",
"src": "Patient: hello i am concerned about my husband he has been having gout attacks alot lately to a point where he can t walk. and i took him to the summit in flagstaff and Dr. thomas told him his kidneys might be failing. i am trying to find a Doctor that will help him. Doctor: HELLO SIR, HERE I AM SUGGESTING YOU FOR GOUT PROBLEM, BETTER YOU USE HOMEOPATHIC TREATMENT. IN HOMEOPATHY IT IS CURABLE AND RELIEF OF SYMPTOMS WITH OUT ANY SIDE EFFECTS. Gout is a disease in which defective metabolism of uric acid causes arthritis, especially in the smaller bones of the feet, deposition of chalk-stones, and episodes of acute pain.Risk possibility by these conditions:Being maleHaving a family history of goutHaving been born with a rare condition that causes high blood uric acid levels.Regular use of aspirin (more than 1 or 2 aspirin a day) or niacinDiuretic medicinesChemotherapy medicines (usually used to treat cancer)Medicines that suppress the immune system, such as cyclosporine, that are used to prevent your body from rejecting an organ transplant.Obesity.Moderate, regular, or heavy use of alcohol, especially beer.A diet rich in meat and seafood, which can be high in purines.Frequent episodes of dehydration.Very low-calorie diets.Diabetes.Kidney (renal) disease.High blood pressure.Lead poisoning."
},
{
"id": 122764,
"tgt": "What causes pulsating feeling in outer calf muscle?",
"src": "Patient: It started when I began using the treadmill (fast walking). My right calf - on the outter side above the ankle flutters. It sometimes feels like it is bubbling or pulsing. What does that mean? It doesn t hurt but feels very odd. Now it happens when I just walk..... Doctor: Hi, Mostly it is nothing to worry about, this can be either muscle contractions, spasms, or arterial or venous pulse due to blood and circulation activation. Hope I have answered your query. Let me know if I can assist you further. Regards, Ayman Darrag, Physical Therapist or Physiotherapist"
},
{
"id": 24278,
"tgt": "What does the CT scan indicate?",
"src": "Patient: Hi! My 70yr old granny has been having shivers on her left had. Its been seen since 1 month. Its not continous and keeps coming now and then. Also, she says she has a weakness around her heart everytime this happens. We got CT scan doen today and it shows 'a tiny oblong hypodensity seen in left parasagittal top parietal location, with ipsilateral ventricle pulling towards it - possibly represents subacute to chronic infarction. In addition multiple tiny, hypodensities seen both gangliocapsular and periventricular locations (left top parietal region more involved), with no mass effect or bleed - could suggest old lacunar infartions.' Rest is shown normal for age. What does this mean? Doctor: Brief Answer: Old infarction areas without any emergency meaning. Should be followed by a cardiologist.Detailed Answer:Hello,Thank you for using HCM. I will try to explain what this CT scan report means.A left parietal, parasagital tiny oblong hypodensity, suggestive of a subacute or chronic infarction represents a small, linear area of lower density on CT, than the rest of the normal brain, due to a previous (old) arterial thrombosis on the top the brain cortex, close to (along) the sagital superior sinus, of the left parietal lobe. Local necrosis around the ischaemic (thrombosed) area is followed by liquefaction of the brain tissue and hypodense appearance on CT. This area, as well as the other multiple tiny hypodense areas in the deeper brain structures (gangliocapsular and periventricular old lacunar infarctions) are associated with ipsilateral ventricle pulling which means dilatation (enlargement) of the lateral ventricle on the affected side. Local volume loss due to infarction and liquefaction, creates local vacuum areas which enable the left lateral ventricle (a cerebrospinal fluid filled cavity) to distend toward the vacuum area. As all these findings do not represent a recent (fresh) infarction, but old ones, there is no reason to seriously worry about. Heart problems, more than a consequence, may be the reason of the above mentioned brain infarctions. That\u2019s why a special care should be carried to prevent further infarctions, such as normal blood pressure maintaining, restricted salt diet and regular cardiologist follow-up. Hope this is helpful.Let me know if you need more clarifications. Otherwise please close and rate the answer.Kind Regards, Dr. Juarda"
},
{
"id": 79098,
"tgt": "Suggest remedy for pain and swelling in ribs near chest, wheezing while lung functioning only 25%",
"src": "Patient: My mom is having some wheezing problem. And recently she went for check-up and for all the tests and doctor said that her lungs are functioning only for 25%. Sometimes she is having pain in the ribs and near the chest, hand. And she got some swelling over there. Is there any remedy for that? Doctor: Thanks for your question on Health Care Magic. I can understand your mother's situation and problem. Lung functioning only 25% means she is having either obstructive or restrictive lung disease. And in both these conditions, lung infection is common. So possibility of chest pain is mostly due to lung infection. So better to consult pulmonologist and get done clinical examination of respiratory system, chest x ray, CT thorax and PFT (Pulmonary Function Test). Chest x ray and CT scan are needed for the diagnosis of lung Infection and restrictive diseases. PFT is needed to rule out obstructive diseases. So better to first diagnose herself and then start appropriate treatment. Hope I have solved your query. Wish you good health. Thanks."
},
{
"id": 209213,
"tgt": "Why am I so bad tempered?",
"src": "Patient: hi im a 16 year old girl i find myself a very angry and down person i have very bad temper which causes me to hit things i also find myself not able to concentrate forget things i just feel like im angry and upset all time and i have no control over my anger is it possible i could be depressed ? Doctor: Depression has varied clinical presentation. The symptoms you have mentioned can occur in depression, if they are quite frequent. A deep routed conflict or stressful life events can cause these symptoms. Personality disorders can also have similar presentations.A detailed evaluation is needed before reaching to a conclusion. Kindly get back to me with more details including .. Age of onset, freqency of symptoms, associated symptoms, comorbid substance use etc.Regards, Dr Arun"
},
{
"id": 221864,
"tgt": "Suggest treatment for cold during pregnancy",
"src": "Patient: I m 36 weeks pregnant and have a really bad cold. I ve been blowing my nose, and one time when I was blowing really hard, my temple had a sharp pain. Pain has pretty much gone away now. I m guessing this isn t a huge cause for concern, but just thought I d ask. Doctor: HiDr. Purushottam welcomes you to HCM virtual clinic.I have gone through your query. I think I have understood your concern, I will try to suggest you the best possible treatment options. 1] First of all do not panic.2] It can be viral fever or simple flue.3] I will suggest you- steam inhalation, warm salt water gargles.4] Tab SINAREST or Tab CALPOL 500 mg can be taken as needed.5] Please ensure plenty of water intake , have hot clear soups, 6] Avoid cold/chilled food items, deep fried foods.7] things will be fine. Let's give time for body to heal.I hope my answer helps you.Thanks.Wish you good health."
},
{
"id": 192727,
"tgt": "Suggest remedy for premature ejaculation",
"src": "Patient: Sir, I suffer from premature ejaculation for the past 8 years. Comparitively I am not fully satisfied with the erection what I had years back. After udergoing unani treatment for the past 1 year, I feel the erection better at times (not always) but I feel the premature ejaculatin getting worse day by day. For ex. while driving 2 wheeler , if I tried to mastubrate without the help of hands, but making use of the vibration of the vehicle alone, then too I get the semen ejaculated. Is musli power extra solve the problem ? Plz advice. I am from Rajsthan Doctor: Hello,There are certain things which you can do to prevent premature ejaculation. You can masturbate 1 or 2 hours prior to have intercourse, this will increase your time of ejaculation. You can also use over the counter sprays and medicated condoms which increases time of ejaculation. You also need to strengthen your pelvic floor muscles as well and zinc supplements also help in some cases.Hope I have answered your query. Let me know if I can assist you further. Regards, Dr. Sameen Bin Naeem, General & Family Physician"
},
{
"id": 117369,
"tgt": "What does this blood test for thyphoid indicate?",
"src": "Patient: Sir Good afternoon sir.My Nephew, having symptom normal fever and throat in the evening for the past one week today blood test carried out result are salm typhi O \u2013 Positive 1:40 , Salm Typhi H- Negativies.para typhi Ah - Negative s.para typhi Bh - Negative Pls suggest whether is a symptom for typhoid if so shall we consult Dr. Doctor: Hello and welcome to HCM,The test that you have performed is Widal test which is done to detect typhoid fever.For diagnosis of typhoid fever antibody titers for Salmonella O and H antigens & Salmonella paratyphi O and H antigens is done.The test is interpreted as reactive when the antibody titers are above 1:80.However, in your test report the titers are only 1:40 and thus negative.Kindly, consult your primary healthcare provider for clinical assessment.Thanks and take careDr Sahilaj P Wahal"
},
{
"id": 160400,
"tgt": "Suggest treatment for stomach pain in a child",
"src": "Patient: My daughter woke up fine this a.m. and suddenly had a stomach ache. After eating she felt better, only to come down with a fever. No other symptoms have shown up yet aside from headache and fatigue. Does this sound like the start of a virus? Thank you! Doctor: Hi,This can suggest either a viral fever, or beginning of an acute gastroenteritis. We need not worry now. Just give her acetaminophen 3-4 times a day (after food) for 3 days. This will control both fever and abdominal pain. In case she develops loose stools, give plenty of fluids like ORS solution in frequent sips after each stools to avoid dehydration. Give ondansetron for any vomiting. If symptoms are not coming down by 3-4 days, she becomes lethargic, or not taking feeds, kindly take her to a pediatrician.Take care. Hope I have answered your question. Let me know if I can assist you further. Regards, Dr. Muhammed Aslam T. K., Pediatrician"
},
{
"id": 186526,
"tgt": "Suggest the cause for hole in the gum",
"src": "Patient: I have a hole in my gum back behind my last molar on the upper right side of my mouth. I have all my wisdom teeth still, my dentist said they didn't need to be removed. I woke up one morning to find this rather large hold in my gum. Just wondering what it is. Doctor: Hello, Welcome Thanks for consulting HCM, I have gone through your query, as you have hole in gums behind last molar dont worry this can be due to periodontal pocket formation , or due to Caries in last molar tooth . Consult dentist and go for treatment if it is carious then go for restoration of tooth if it is periodontal problem then go for Currettage . Hope this will help you."
},
{
"id": 181831,
"tgt": "Suggest treatment for pain in lower molar tooth radiating to rest of mouth",
"src": "Patient: I am having severe pain on the left side of my mouth. The pain starts in my lower molar (second tooth from back), and then spreads to the tooth above it, and then will continue to spread throughout the rest of my teeth and gums on the left side of my mouth and up my jaw to my temple. Both molars where the pain starts have previously had cavities, and the lower tooth has a crown. The pain is excruciating, and the only way I have been able to stop it is to keep cold water or tea in my mouth, once the liquid starts to reach body temperature the pain will start again. I am taking 4 advil at a time now and it does absolutely nothing. I went into my dentist today and she took an x-ray and said she did not see anything wrong with the teeth. This has been going on for three days now and only seems to be getting worse, what should I do? Doctor: You can use a numbing agent like Oragel,Lidocaine ororegano oil, this is a temporary solution for the pain.You said you cavities it might be recurrent caries, depending on the caries, your molars may need Root Canal Treatment,also you can Use listraine \"Make sure it has methyl salicylate in it\", keep it in your mouth for about 1 minute it should ease the pain immediately, you should change you dentist to more experienced one, if you have the X-rays it will help me a lot to diagnose your case."
},
{
"id": 79496,
"tgt": "What causes pain in left side of my chest?",
"src": "Patient: ilifted a box of kitty liter to get the price of it before going to the checkout isle. Now since this i am experiencing pain that comes and goes on the left side of my chest i dont feel any pressure on my heart. i am a 66 year old female., but it is causing alot of anxiety for me now. Doctor: thanks for your question I completely understand your question you need to consult a pulmonologist who can get a chest xray and ecg done to rule out any organic or vital organ diseases like heart and lung .in my opinion it seems like just a muscle injury which ll heal in some days, for now visit a pulmonologist who can prescribe you with some analgesics like paracetamol which ll lower the painno need to panicfeel free to ask more questions thanksmay god bless you with good health"
},
{
"id": 110242,
"tgt": "Suggest treatment for tailbone pain",
"src": "Patient: Hi,I get high tailbone pain with moderate sitting or two-wheeler driving.I started observing this couple of weeks ago and this intensity is getting higher with every passing day and activity.Kindly suggest good doctor to consult in Hyderabad, India.Thank you Doctor: Hello, I have studied your case. Due to microtauma there may be fibrosis formation on sacrum/tail bone.Do hot fomentation over sacrum. It will take weeks to months to subside.Use ring cushion and take sitz bath [sitting in lukewarm water for 10 min]Take analgesic like nucoxia 90 once daily along with tab methylcobalamine 1500 mcg once daily consulting your doctor.If not relieved with all these methods then local injection may be required. You can take this consulting your doctor.You can search doctor near locality on our website.Hope this answers your query. If you have additional questions or follow up queries then please do not hesitate in writing to us. I will be happy to answer your queries. Wishing you good health.Take care"
},
{
"id": 195794,
"tgt": "Is betnovate-n good to remove pimple on penis?",
"src": "Patient: hi.i am 22 years ol.n i have this pimple on ma penis..it doesnt hurt but its been there for a week or so..i havent tried applyin anything yet..cos i thought it was because of the hot weather..the pimple is kinda redish colour.not too big..a kind you get on ur face i suppose..it duznt hurt that much..but i was wondering which ointment would b good.is betnovate N good for this kinda prob ?? Doctor: Hello and Welcome to \u2018Ask A Doctor\u2019 service. I have reviewed your query and here is my advice. Skin problem are difficult to say about without proper examination. However, what appear is that you probably have a carbuncle there which is in fact infection of the hair follicle, occurs commonly on skin. Betnovet is strong steroids and is by no means recommended for such lesion. What I can advise supposing this is a carbuncle is maintain good hygiene and take some topical antibiotics. I prefer Clindamycin or Erythrocin topical. If you have any doubt or the above does not work for you in few days the best thing would be letting your doctor have a look on it and decide. Hope I have answered your query. Let me know if I can assist you further. Regards, Dr. Mahboob Ur Rahman"
},
{
"id": 88364,
"tgt": "Suggest treatment for bruise on neck and abdominal pain",
"src": "Patient: hi ive had a whole bunch of problems. first there was a bruise on my neck right by the throat that would come and go for a couple months but it never really hurt so i didint get it checked out. but in the last four months i have had a sometimes sharp, sometime dull pain from my abdomen all the way down to my ovaries and occasional brownish bruises that i cant account for Doctor: Hi.So this is basically a problem related to the recurrent bruises. I would advise you the following. Get a complete hematological work-up done. CBC, Platelet count, bleeding time, clotting time, prothrombine time, capilarry fragility tests and other as per the opinion of the Hematologist. These problems of bruises inside the organs can be devastating. Do not take anti-inflammatory medicines."
},
{
"id": 56354,
"tgt": "Does crushing Motrin pill for tooth pain affect pancreatitis condition?",
"src": "Patient: i HAVE BEEN DIAGONOIS WITH TYPE 2 DIABETES, DIVERTICULITIS, CHRONIC PANCREATITIS. tHE NUMEROUS MEDICATIONS THAT HAVE BEEN PRECRIBED TO ME APPEARS TO SERVE AS SOME COMFORT; HOWEVER, LATELEY THE MOTRIN 800MG THAT i VE BEEN TAKING FOR MY TOOTH PAINS, AND MIGRANE HEADACHES, i FIND CRUSHING THEM UP GOES DOWN BETTER. QUESTION: WLL THIS METHOD OF CRUSHING THE MOTRIN IRRATE THE PANCREAS/DIVERTICULA? Doctor: Usually motrin or other NSAIDS have no effect on pancreatitis or diverticulitis crushed or uncrushed"
},
{
"id": 6034,
"tgt": "Trying to conceive. Have PCOS and overweight. On ovofar. Pregnancy chances?",
"src": "Patient: Hi doctor..i am 29 yrs female..me and hubby are trying for kids from past 10 months..but everymonth I get my period..I have regular periods my cycle is 32-35 days..I consulted a GYN recently and I was diagnosed with pcos ..so she told me to loose weight as Iam overweight ..and i have no other problems( facial hair and irregular periods) now I lost around 4 kg (lost from 85 to 81 now) and gave me Ovofar 50mg to used from 3- 7th day..can I use them now ..will I be pregnant..please tell me i am very depressed .. Thanks Doctor: Dear customer, The ovofar is a ovulation inducing drug and once you ovulate and have sex during that period you may have the chance of being pregnant. You may have the ovulation detection kit to know the exact date of ovulation and have sex during that time. In pcos the chances of pregnancy are less but not nil so you may proceed as per your doctor's suggestion. Thanks"
},
{
"id": 199497,
"tgt": "What causes pain while masturbating and semen in urine?",
"src": "Patient: i am dare by name, from nigeria, i am 28 years now... i have been masturbating since over 10 years ago... if i go to the toilet, when i pee, i do see sperm rushing out of my penis instead of urine coming out, and also when urinate i do have urine leakage i.e urine don's stop after urination, i do have wet dreams and 1-2 minutes of ejaculations when having sex, am having back pains also and if i hear anything emotional i will start releasing sperm.. sir is there drugs u can prescribe to these symptoms, am a student and i can't come abroad for treatment... YYYY@YYYY my mail Doctor: HelloSemen in urine is a common finding and it may be due to over excited state,straining,inadequate emptying etc.You should try relaxation technique and avoid provocative literature and videos.You should try to be calm and can go for yoga,mediation.You should try to engage yourself in your job.You will be fine with above mentioned measures.If findings persists then you may need psychological counselling.It is also important to evaluate for constipation.Get well soon.Take CareDr.Indu Bhushan"
},
{
"id": 34694,
"tgt": "Could missing the last anti-rabies dose be a cause for concern?",
"src": "Patient: hi i n my 4years old daughter was bitten by dog on 14th May 2011 we got all 4 doses of anti rabies from Balrampur Hospital Lucknow,India ...dog is a pet of my friend and properly vacinated.....dog is still fine no sickness....today is the last day of my n my daughter's vacination of anti rabies but i m not in Lucknow. so i can not go for last vacination ....should i worry for it or what else i can do Doctor: Hello. Welcome to HCM.I am Dr Jigar(ID Specialist).No you dont need to go for last dose.In your case its already long time since dog bite. As the dog was vaccinated and still fine then there is no need of taking further vaccine shots. If dog was not pet and its vaccination status was unknown then i would have told you for completion of course.I hope this will help you. Please give your valuable feedback.Regards.Dr Jigar."
},
{
"id": 45355,
"tgt": "I am trying to get pregnant for the last 3 years, i am having polycystic overy problem",
"src": "Patient: I am trying to get pregnant for the last 3 years, i am having polycystic overy problem hi i am 28 years old.I am trying to get pregnant for the last 3 years, i am having polycystic overy problem,i took so many courses of clomid and Injection Fostimon.i do exercises make my self fit.my all tests are normal.but still i am not able to conceive.my husband reports normal aswell.plz tell what should i do next. Doctor: Dear sanna In herbal medicines we use ashoka arisht and dashmoola arisht. i would recommend cap. ovarin forte 1 bd. have it for atleast one month. also let your husband have vysex forte 2 bd just to ensure conception."
},
{
"id": 29925,
"tgt": "What causes blurred vision with cough and cold?",
"src": "Patient: My mom has been sick since sat with a cold and sinus pressure and cough. Today she woke up and is having a hard time with blurred vision. She says it feels like she s going partially blind. Is this something dehydration would cause it is there a virus going around with these symptoms Doctor: Hello!Thanks for using healthcaremagic.comI read carefully your question and understand your concern. I think you mom have sinusitis. Sinusitis is a common condition in which the lining of the paranasal sinuses becomes inflamed. It is usually caused by a viral infection and usually gets better in two to three weeks.As you have pointed out blurred vision cough and cold are among the common symptom.Blurred vision is caused due pressure around the eyes, cheeks, nose, or forehead that gets worse by stoopingMost sinusitis are caused by a virus. Very rarely are they caused by bacteria.Even if the symptoms appear to be localized in the breasts, they are not always infected. To make a correct diagnosis, the doctor gathers detailed records and performs a physical examination. The doctor may also order more tests, if applicable.These tests may include allergy tests, chest x-rays, computerized axial tomography (with accurate images of sinus cavities), or a sample of nasal secretions or nasal lining.The doctor may also perform an endoscopic examination. In this case a narrow, flexible fiber-optic endoscope is inserted into the nasal cavity through the nostrils, which allows the doctor to see the area where the sinuses and the middle ear drain into the nose easily , Painless, producing minimal discomfort to the patient.Since most cases of sinusitis are caused by a virus, the treatment is aimed at relieving symptoms.Medications include:Oral decongestants (phenylephrine, pseudoephedrine) *Topical decongestants (oxymetazoline, xylometazoline)Saline rinsesMedications for pain (acetaminophen, ibuprofen).Spray decongestants should not be used for more than 3 to 5 days in a row because they can cause \"rebound congestion.\"Steam inhalation and cold vapor humidifiers can also help.Kind regards, Dr. Oscar Martinez"
},
{
"id": 93801,
"tgt": "Unexplained bruising with blood spots, itchy skin, lost appetite. What is the cause?",
"src": "Patient: I have unexplained bruising with blood spots (I think that is what they are called?) along with itchy skin. I ve completely lost my appetite and I m always tired no matter how much sleep I get. Its been going on for about 6 months. I was wondering what could be causing this? I m booked in for blood tests later this week. Doctor: Hi and welcome to HCM. It i shard to say are those 2 symptoms related. Unexplained bruise is usually sign of clotting disorder but it can occur without obvious reasin. Loss of apetite is probbaly something else and you should do complete gastroenterologic and endocrinologic work up because it can be sign of certain diseases. Also if you are female, hormonal imbalance should be considered. WIsh you good health."
},
{
"id": 171189,
"tgt": "Suggest treatment for lice in head",
"src": "Patient: my daughter (age 14) has got so much of head lice. recently she was down with chicken pox and developed head sores adding up to lice growth rapidly. those sores are itchy, sometimes watery, makes the hair ugly and with ba odour. lice growth has multiplied... need a remedy please. Doctor: Hi, for lice treatment you can apply permethrin 1 percent cream to the scalp for 15 minutes and then wash the head. Reapply cream for 3 days and your child's lice problem will be solved. Regards - Dr Deepak Patel, MD Pediatrics"
},
{
"id": 17642,
"tgt": "What does septal myocardial infarction on ECG indicate?",
"src": "Patient: I am looking at a ECG recently taken of a family member. I am only an EMT-B, and do not have sufficient ALS-level training in properly interpreting an ECG. At the top of this copy, i see that it says, Sinus Rhythm, Septal Myocardial Infarction, Possible Left Atrial Enlargement and Abnormal ECG. I wasn t the one who my family member to get this done, and I m upset I wasn t told about it. She is a 78y/o Female. Should I be concerned? If requested, I will scan and upload a copy of the actual ECG print Doctor: Hi, The ECG interpretations which comes printed on the ECG paper are machine interpretation of the ECG and could be wrong. So I would like to suggest you to upload the ECG image so that I can review it and give my opinion accordingly. Septal myocardial infarction means a patient had a heart attack which has affected the interventricular septum. Hope I have answered your query. Let me know if I can assist you further. Regards, Dr. Bhanu Partap, Cardiologist"
},
{
"id": 33482,
"tgt": "Suggest remedy for back pain and blood in urine",
"src": "Patient: ive had a cold the past week,sore throat,swollen glands,cold sore etc but straight after i had a sharp pain in my side that moves around the back i have been examined and everything ok but i have slight blood in urine that you cant see,anything to worry about Doctor: Hello!It might be a infection of your kidneys of urinary tract as a complication of throat infection from streptococcus.You have to do a urine analysis and a throat culture too.Antistreptolisines in blood shows if you have strep infection.If yes you have to take antibiotics.thanx!"
},
{
"id": 37277,
"tgt": "Can a X-ray detect TB?",
"src": "Patient: my FATHER's montux test and serum test is negative but on the basis of x-ray doctor has advised two forecox tab daily befort breakfast. My question is just on the basis of x-ray can we detect tb when other tests are negative. my father age is 69 and he is feeling very week after taking forceox. please guide Doctor: Based on clinical findings and after corroborating with the X ray findings, if the doctor finds evidence of tuberculosis, it can be treated as tuberculosis even if Mantoux test and sputum test are negative. At advanced age like that of your father, the immunity automatically comes down and susceptibility to tuberculosis increases. However, for confirmation, TB Gold Quantiferon test is considered as gold standard for diagnosis and you may opt for that if you have a confusion. Please note that it is a costly investigation."
},
{
"id": 212072,
"tgt": "Depression, irritated, sleep disorders, anxiety, defensive nature. On panex cr, modalert, etizola beta. Advise?",
"src": "Patient: could you please guide through the depression and anxiety problemage is 29, gender - male, have 5 years of work-ex in MNC. Now gap in job from past 1.5 years. Defensive nature, still trying for MNC job only although the knowledge is not enough, getting job nowhere. Not ready for a lower level job. very irritated and sleep disorders. too much anxiety.Currently he has started following medicine:Panex - CR 12.5 mgModalert 100mgEtizola beta .25 mghow should he go furthur...he is facing problem from about past 4 years., not cheerful, always lost in his own thoughts Doctor: Hi,Thanks for using healthcare magic.I have gone through your available history.Panex contains peroxetine which is good drug to control depression with anxiety features.Usual therapeutic dose is 25 to 50 mg per day. He should regularly take the drugs till your consulting doctor advise to have. Anti depressant drugs usually takes time to start action.He will also get benefit by combining psychotherapy or counselling along with drugs.He may also do stress management and relaxation techniques like yoga, meditation for better control over anxiety.Thanks."
},
{
"id": 144252,
"tgt": "What causes tingling sensation on back?",
"src": "Patient: I ve had tingling on my back under my bra hooks for over a year. It comes and go, so I didn t give it much thought. In the past two weeks, my feet have been tingling a good part of the time. Within the last three days, my hands have begun to tingle, also. Today I ve felt a few of the same sensations on my upper lip on the right side. My ankles feel tight and my right hand feels heavy. Any thoughts? Doctor: Hello!Thank you for asking on HCM!I understand your concern and would explain that your symptoms could be related to different causes: - a cervico-thoracic column disorder, leading to nerve and radicular compression. - electrolytes imbalances (calcium, potassium, magnesium)- anxiety could also mimic all this clinical situation- multiple sclerosisI would recommend consulting with a neurologist for a careful physical examination. Depending on the physical exam further tests may be needed: -a brain MRI study in case of suspicions of central neurvous system disorders.- a nerve conduction study in case of suspicion of peripheral nerve disease, including nerve compression and neuropathy.- some blood lab tests (complete blood count, fasting glucose, PCR, sedimentation rate, thyroid hormone levels) would also help exclude other causes. Hope to have been helpful!Best wishes, Dr. Aida"
},
{
"id": 142390,
"tgt": "What causes sudden unresponsiveness, eyes rolling back, left-sided tremors and mouth drooping?",
"src": "Patient: Hello, my husband just had an episode that concerns me. He is resting now but I would like to know what happened. He was talking to me then became unresponsive. His eyes rolled back in his head, the left side of his mouth dropped, lips were white, face was pale, left arm and hand were shaking, he then collapsed to the floor. I held him and he was sweating but his left side was ice cold as was both hands. He eventually came to and didn t remember what happened. He felt weak and tired. I put him to bed and he is now sleeping. What do you think caused this to happen? Doctor: Hello!Welcome on Healthcaremagic!Your husband symptoms are compatible with a partial seizures with secondary generalization. For this reason, I recommend consulting with a neurologist as soon as possible and performing some tests: - a resting ECG- an EEG- a chest X ray study- a brain CT scan- some blood lab tests (complete blood count, fasting glucose, blood electrolytes). Hope you will find this answer helpful!Kind regards, Dr. Aida"
},
{
"id": 25375,
"tgt": "Suggest treatment for high blood pressure",
"src": "Patient: I had CABG 18 years aback. Nowadays when tested I am found to have Supra ventriculare Ectopics and Blood pressure in the range of 160/80. I am taking Cresar AM twice a day, concor 1.25 mg daily. still i find my BP remains to be 160/70 most of the times. Please advice? Doctor: Hello and thank you for using HCM.I carefully read your question and i understand your concern.I will try to explain you something and give you my opinion.As you explain, you had CABG years ago. As you are diagnosed with ischemic heart disease, this means that we should be very careful with hypertension, because high blood pressure values slowly modifies your heart muscle.The ektopic beats have to do with some electrical issues in the heart. Some electrical impulses that generate in a part of the heart different from normal sinusal rhythm cause a premature heart contraction that you feel like a skipped beat, because its not a normal one. There are different factors that might cause this skipped beats like emotional stress, excessive coffee consumption , coca-cola or energy drinks, so you have to avoid them.Pathology like anemia, hyperthyroidism or ischemic heart desease might be other causes. You explain that you still have high bllood pressure values despite your treatment. So, if you have mean values above 140/90 mmHg, than we should definitively correct your treatment.We can do this in two different ways. Raising the dosage of your existing medications or adding another drug. The goal is to have mean blood pressure below 140/90 mmHg. You don't mention the dosage of cresar. Concor can be taken up to 2.5,5 to 10 mg a day. It can better control your extra-sistolic beats and lower your blood pressure.But to decide this is important to know your frequencies and your heart function.So, if I was your treating doctor i will recommend a full check up with blood test analyze, a electrocardioram, echo-cardiography, a pressure and rhythm holter monitoring.After this we can better judge how should we treat hypertension and ektopic beats.Hope I was helpful. Wish you good health.Best regards."
},
{
"id": 152219,
"tgt": "What treatment and exercise is suitable ?",
"src": "Patient: I am in my 30. My MRI report is as follows For C- spine - Early degenerative disc disease mild diffuse annular bulge noted at C4 ? C5 level causing bilateral recess and neural foraminal narrowing. For L - spine - Early degenerative disc disease mild to moderate diffuse annular bulge noted at L4 \u2013 L5 level causing indentation of thecal sac and bilateral recess and neural foraminal narrowing. Now I follow homeopatic treatment. I had went for ayurvedic treatment but got too weak. I want to know what treatment is best, which exercise I can do? Is stationary bike a good choice other than walking as I have shooting pain in lumbar region radiating al over my backside when walk even medium speed. I am overweight.So trying to loss weight also. I want to be fit at least to manage these horrible pain. Please tell what exercise I can do. Doctor: Firstly a proper diagnosis is required to get the relief from ayurveda.As per my experiance the cause may related with large intestine.So that a basti+virechan is required.Also the combination of medicines that works on blood+bones are required.A nadi chikitsa is required for proper diagnosis."
},
{
"id": 148631,
"tgt": "Lost feeling on right side due to TIA. Have a bad disk in back along with sciatic nerve",
"src": "Patient: I have deteriorating disks in my back and they cant operate and a siatic nerve on the right side the disks are L3 L4 L5 and i go to a neurologist .I also have had 2 TIA in OCT and they tell me to go to a heart Dr but why did I loose feeling all on the right side and it tingles and i drop things im weak on the right side and there is nothing they can do i dont understand whyIM sorry I have no income and trying to get on disability, sorry for taking up your time Doctor: HIThank for asking to HCMI really understand your concern, as long as the TIA is concern I would advise you to take care of blood pressure if you have high blood pressure then be on medicine regularly, if you do not have it then measure it regularly, watch for blood glucose level, for the disc problem the best way to deal with it is physiotherapy, and some NSAID this will give good relief to you nothing to worry it will be alright soon have nice day (It would be much better if you would have mentioned your age and gender)"
},
{
"id": 221847,
"tgt": "What are the side effects of Ecosprin-75 in pregnancy?",
"src": "Patient: Hi Doctor, I am in 4th month of pregancy. Due to high BP (80- 130) doctor asked me to take Ecosprin -75. Does any side effects for taking this tablet? In tablet itself its mentioned that Do not take this product during the last three months of pregnacy, it may cause bleeding problem for both mother and baby Plz help me Doctor: Hi,Ecosprin or baby aspirin is given to women who have risk of having smaller babies, it is given to decrease the chances of fetal death in womb. It decreases that risk by about 10%.Also Ecosprin is advised from 12 weeks till delivery and is found to be safe.Most drugs have not been on trial on pregnant women due to ethical reasons but are being used in lots of women in many countries are found to be absolutely safe.Do not worry about bleeding, as these have not been reported and ecosprin is found to be safe and effective.Hope this helps.Regards."
},
{
"id": 88925,
"tgt": "What causes pain in the stomach with spots?",
"src": "Patient: Hi, my daughter has got red spots with a yellow head on her stomach and the top of her legs. She has complained of headache and stomach pain, feeling sick but nothing serious. No temperature. She has had chicken pox before, I think these are different. Doctor: hello,better do get them examined by a dermatologist for the exact diagnosisas they cannot be easily diagnosed without seeing them,,,follow their instructions..things will improve"
},
{
"id": 39665,
"tgt": "Suggest remedy to reduce swelling in feet due to bite",
"src": "Patient: Got my foot bit by something last night and it's swollen the whole foot and almost going to my toes, ect....it looks like blast trades n it's very swollen, don't know what to do, please help....went to ER and the waiting was 4 hrs so I've left but it's getting more more swollen.....Urgh ;( Doctor: Hello,My advise to you is consult your doctor as early as possible.You will need medications and detailed examination of bite site to see extent and type of inflammation.It would be useful, if you can remember bite is from which insect or animal. It will help your doctor to plan your treatment.You will be required to take anti allergic, analgesic, anti inflammatory medications and may be a course of antibiotics.If untreated your pain may increase and if swelling does not subside for a long duration may lead to tissue damage.Hope I have answered your query. If you have any further questions I will be happy to helpThank You,Dr. Narasimha G L"
},
{
"id": 22726,
"tgt": "What causes heartburn and hiccups?",
"src": "Patient: in the last three days i have started burp/hiccupping all day long and when i need to burp but can't its like the burp gets stuck in my chest and it hurts really bad, it feels like i literally have a lump in my throat. the first day i had it, i also had heartburn but now the heart burn is gone but i'm still burping? any suggestions? i'm a 28 yr old female taking 20mg of lexapro and 7.5mg of buspurone a day. thanks in advance!! Doctor: Hello and welcome to 'Ask a Doctor' service. I have reviewed your query and here is my advice. It's a gastritis. Avoid hot, spicy, oily and fatty meals. Have multiple small meals instead of heavy meals. Don't lie down immediately after meals instead have a walk for sometime. avoid smoking and drinking if any. Take Tab Pantop once a day before breakfast.Hope I have answered your query. Let me know if I can assist you further.Regards,Dr. Sagar Makode"
},
{
"id": 103239,
"tgt": "Sneezing, runny nose, teary eyes after using all purpose flour. Have mild gluten allergy. Can there be allergy to raw flour?",
"src": "Patient: I have noticed that on three different occasions when I'm using all purpose white flour to bake or fry I immediately get a reaction whereas I'm sneezing, runny nose, teary eyes etc for 3 days or so. Is there such a thing as being allergic to raw/uncooked flour??? I do have mild gluten/wheat allergies. I have never had this type of reaction to raw/uncooked flour in my life other than these past three occasions. Doctor: Hi, Allergy is a wide scoped entity ,Any thing&every thing in environment will create allergy to somebody. It can be food,clothes,cosmetics, toiletories,insect bites, hairs, dust from any source-bed, house, grain,fumes and so on!The person with allergic tendensy may devolop allergy to newer things in course of time. The expirience of symptoms on exposure to certain suggests the allergy to that thing.OR you can get yourself tested for various allergens . Prevention of exposure to allergen is 1st step in controlling. Anti allergic medicines help to control an episode While& desensatisation treatment is curative Thanks"
},
{
"id": 143379,
"tgt": "What is the treatment for pain in upper vertebral column?",
"src": "Patient: HII have pain in the upper vertebral column, a little below the neck. It radiates down to both the arms along the underarm and shoulder blade. Gets better when i lie down flat on my back. Need to know what it could be . Have a history of a slipped cervical disc and a bulging lumbar disc that have been successfully cured through acupuncture. Doctor: Hello and thanks for using HCM.I have read your question and understand your concerns.It could be degerative discs and spine disease that is causing your symptoms.MRI of cervical and upper thoracic spine is necessary to confirm the diagnosis.In first stages, NSAID drugs use and physical therapy are indicated. Hope you found the answer helpful.Greetings."
},
{
"id": 83155,
"tgt": "Could Glimestar and Loram cause constipation?",
"src": "Patient: my wife is taking glimestar 1m od and loram h2.5 for about three weeks. these drugs have been prescribed to her for first time for bp and blood sugar. She is having severe constipation for over a week. could it be the side effect of any of these drugs? Doctor: Hi,Glimestar or Loran h are not found to cause constipation as a side effect. Possibly altered diet to manage diabetes and probably salt free diet for high blood pressure may be causing less food intake leading to constipation. Addition of plenty of fibers and water in diet would solve your problem. Take care. Hope I have answered your question. Let me know if I can assist you further. Regards, Dr. Vasudha Jayant Athavale, General & Family Physician"
},
{
"id": 199658,
"tgt": "What is the remedy for the yeast infection and small red boils on the penis?",
"src": "Patient: hi sir i m suffering with yeast infaction i m 31male.during 20 days i have take some take medicine till 15days then it become ok after some time small red boils are seem in the penis fore skin.i m taking surfanz-sn tube on it....how time it takes to become ok...plz sir tell me some taking care formula what should i do.i m loosing my hope due to this problum Doctor: HelloThanks for query.You have fungal infection of foreskin .since 3 weeks and have been using topical anti fungal cream for local application .At the out set I would state that fungal infection is very common in persons with Diabetes /Please get your blood sugar levels on empty stomach (Fasting ) and 2 hours after lunch done to rule out diabetes.Take anti fungal medications like Fluconazole twice daily along with topical anti fungal jelly like Daskyl for local application .Ensure to wash your genitals with warm water twice daily.Dr.Patil."
},
{
"id": 135696,
"tgt": "Is there an alternative medicine for sciatica that would not give a headache?",
"src": "Patient: Good Morning Doctor: I was given this med yesterday due to having been diagnosed with sciatica. Not having any particular problems with the med other than they seem rather strong and give me a headache albeit mild. My main concern is the sciatica and whether it can be well managed. I m a 70 year old male in otherwise good condition.Thanks Doctor: Hell. As there is scsciatica.. The medicine that's given for that also relieve headache.. Do find severity and cause.. It might be reduced by surgical procedure... Too. If you are fit and fine."
},
{
"id": 102605,
"tgt": "Is it possible that shortness of breath, chest pain and cough are a result of food stuck in air pipe?",
"src": "Patient: hello name is craig age 59. seams like ever since right before cristmas i was munching on a peanet and i didt chewit up to good bad teath. anyway it went into an air pipe or something. i could here isiling and weasing for a few days. but after that i could not feal it anymore. but now i notice sortness of breath and eveen presure on chest little bit. stop smoking week and half ago. i cough a little bit but not bad. no fleam or ratiling when i couf, so do not no if its cough related or not. blood presure is about 118 0ver 79. if my mechine is working right. i get a kittle bit of a feverish fealing toword nite, but other then that i feal ok i think. any ideas. Doctor: Hello dear,The symptoms as mentioned in your post suggest that you might be having Allergic tracheo-bronchitis.Examination of the throat using a laryngoscope and X ray of the chest will be helpful in clinching the diagnosis.Management includes:1.Antihistamine preparations like Allegra & Cough expectorant syrup for symptomatic relief.2. Montelukast preparations- used as a maintenance therapy.3. A course of antibiotics may be taken to prevent secondary bacterial attack.4. Maintain adequate hydration & take a healthy balance diet.5. Also make sure that you are well protected from cold, dust & other allergens.Wishing you a good health.Take care."
},
{
"id": 55664,
"tgt": "How to reduce SGOT and SGPT enzymes?",
"src": "Patient: How to reduce SGOT and SGPT enzymes , i have gone through the LFT and found SGOT are 109 and SGPT are 140. How to reduce them...what is the diet and what are the medicines as i have less money to consult a doctor in delhi who are charging more than 500 consultation fee as Gastro experts. Doctor: Hi & welcome.I can understand your concern.The raised liver enzymes (SGOT & SGPT) are suggestive inflammation of the liver or gall bladder. Alcohol intake is also responsible for the liver damage.You need certain lifestyle changes to reduce the raised enzymes like1. Stop taking alcohol (if you are alcoholic), avoid smoking2. Avoid fatty meals3. Eat home cooked foods, increase the intake of fruits and vegetables4. Do regular exercise, try to reduce your body weight if you are overweight5. Take Vitamin E containing supplements which are beneficial in reversing the liver injury.6. An abdominal ultrasound is also recommended.Hope this helps you better in getting rid of raised enzymes."
},
{
"id": 55639,
"tgt": "Suggest medication for fatty liver condition",
"src": "Patient: hi.i am sandeep singh .before 2 years ago i found my OT,PT higher near by 90 and 140 in liver function test..then i got treatment from a doctor and he said it is Problem of N.A.F.L.D and he suggested me medicine of udiliv300(Ursodeoxycholic Acid ) i m taking this medicine since two years but i sill have that fatty liver problem .please suggest me .what should i do..... Doctor: Hi, dearI have gone through your question. I can understand your concern. You have slightly high SGOT and SGPT. You have non alcoholic fatty liver disease. You should continue your udiliv. Avoid high fatty diet. Go for regular exercise and weight reduction. Take high protein diet. Avoid alcohol. Life style modification is key in NAFLD. Consult your doctor and plan accordingly. Hope I have answered your question, if you have doubt then I will be happy to answer. Thanks for using health care magic. Wish you a very good health."
},
{
"id": 66276,
"tgt": "What is the treatment for a lump on the jaw?",
"src": "Patient: I have a hard lump on the outside of my lower jaw on the left. I recently went to a dr and was told it was a swollen gland and was put on antibiotics. I am almost done with the antibiotics now and the lump has not gotten any smaller or gone away. should I call my regular dr and have it checked? Doctor: Hi, dearI have gone through your question. I can understand your concern. You may have some salivary gland tumour like pleomorphic adenoma or somd enlarge lymphnode. You should go for fine needle aspiration cytology of lump. It will give you exact diagnosis. Then you should take treatment accordingly.Hope I have answered your question, if you have doubt then I will be happy to answer. Thanks for using health care magic. Wish you a very good health."
},
{
"id": 79070,
"tgt": "Can erythromycin treat bronchitis?",
"src": "Patient: Hi. My 3yr old son has possible bronchitis and is on erythromycin (6mls twice a day). He has had three doses so far. When he got undressed this evening i noticed a rash all over his hips, thighs and upper arms. It is raise and reddish shapes a bit like clouds formations, they are just outlines though. The centre of each marbled shape is not filled in. Is it an allergic reaction to the antibiotic? Thanks for your help. Doctor: Thanks for your question on Health Care Magic. I can understand your concern. Erythromycin is macrolide group of antibiotic. And it can cause allergic skin reaction. So his rashes can be due to erythromycin induced skin allergy. Another possibility is viral rashes. Viral infection also cause similar kind of rashes on skin. He is also having bronchitis and viral infection is most common cause for bronchitis. So viral infection can be the cause for bronchitis and skin rashes. So better to consult dermatologist and get done clinical examination of rashes. Also discuss about possibility of viral infection and erythromycin related rashes.. Hope I have solved your query. Wishing good health to your son. Thanks."
},
{
"id": 188086,
"tgt": "Is difficulty and pain while swallowing after removal of wisdom tooth normal?",
"src": "Patient: Hi I'm 14, and I just got my wisdom teeth out around 10 this morning. An hour ago around 5 my mom tried to give me a pill and I noticed that it hurts when I swallow so I decided not to eat or anything else till tomorrow, but I just wanted to know if that's bad that it hurts when I swallow, it feels like I'm pulling on my stitches, I'm kinda scared. OK well. Hope you can reply. Thankyou. Doctor: Hello, thank you for consulting with healthcaremagic. You are feeling pain and difficulty as the extraction has been done just one hour ago as the removal of wisdom teeth causes slight difficulty as it is the back most teeth, and may be possible some muscles were also exerted, 5hat is medications are prescribed by the doctor. Plz do not take tension, just relax and have the medications timely so that you will be fine. It will take some days to be normal. If it doesn't get relieved within 5 days then you can go and see your doctor. Hope it will help you."
},
{
"id": 57750,
"tgt": "Will consuming antihistamine medicine help deal with the itchy sensation in genitals possibly caused by liver?",
"src": "Patient: I have had this itchy feeling crawelling feeling it started on the outside of my private parts on one side an itchy feeling, I then went to the doctor and she thought it could be excem a and gave me bento ate gel and a body wash, but know the crawelley itchy feeling is all over and keeps me awake at night I have tried taking anti histamine but that does not seem to help, it is there during the day but because I am busy doing things it's not so irritable,do you think it could be my liver , I do like a glass of wine although its low in achol I do have a glass most nights, also I have a cat that sleeps on our bed but on his own blanket I am very fusy about being clean and wash all bedding regularley in a non bio powder as I have always had sensitive skin and hav not changed any washing products please help. Doctor: Hello!Thank you for the query.Liver problem is rather hardly possible. Jaundice can cause itching (I suppose this is what you are thinking of) but usually it affects whole body, not genitals only. With liver problem you should also have yellowish skin, easy bleeding, easy bruising.Your problem is rather caused by fungal infection in genital area. This area has a special, humid and warm environment which makes it prone to fungal infection. I suggest you to consult dermatologist.In a meanwhile please try to wash it regularly, wear loose underwear.Hope this will help.Regards."
},
{
"id": 225633,
"tgt": "Brown bleeding after switching birth control. Negative pregnancy test. Worried",
"src": "Patient: I switched birth control 2 months ago and when it was the day to start my cycle i had the brownish light bleeding that turned reddish throughout the day. I bled that day and then that night it stopped. The next day i had some but not much bleeding, very light. I took a pregnancy test and it came back negative. But since i only had very light bleeding for about 2 days im worried i may be pregnant? Doctor: hi..you are on oral contraceptive pills...if you have taken the pills regularly, the chances of conception is very remote, its about 0.1%...usually the periods that occur with pills are only withdrawl bleed and not the regular menstruation.. and the bleeding will be less when compared to normal cycles..so you need not have to worry..good luck.."
},
{
"id": 84935,
"tgt": "How does fertigyn 5000 help while trying to conceive?",
"src": "Patient: I have pcod!i m getting my ovulation stdy done which shows many eggs some of size 22mm n some 19!!today is twelth day of my cycle.my doctor gave me fertigyn5000so that my egg gets ruptured n we should try to conceive then!!i jst wantd to ask d role of that injection!!thanks Doctor: Hi,Fertigyn 5000 contains Human chorionic gonadotropin (HCG). It is a hormone that supports the normal development of an egg in a woman's ovary, and stimulates the release of the egg during ovulation. It is normally secreted in women for supporting egg development. Injection of the hormone increases the chances of pregnancy by facilitating egg development and ovulation.Hope I have answered your query. Let me know if I can assist you further. Take careRegards,Dr. Saranya Ramadoss, General and Family Physician"
},
{
"id": 86862,
"tgt": "What is the treatment for abdominal discomfort , shortness of breath , thigh and back pain?",
"src": "Patient: hello doctor, I am on metformin for the past two weeks... i have delayed periods of 9 days and i got a negative urine pregnacy test. i have abdominal discomfort and intermittent pain in my thigh , back, lower abdomen..... shortness of breath.. confused ifi am pregnant or not Doctor: Hii appreciate your concernlooking at your history and symptoms it could be a side effect of metforminyou need not to worry please visit your doctor to rule out other possibilities of shortness in breathdeep breathing exercisedaily exercisekeep your blood sugar checkedHope this helps you.have a nice day"
},
{
"id": 104568,
"tgt": "Suffering from cold, cough, sneezing red color. Never taken any treatment before. What to do?",
"src": "Patient: Hello Doctor. Since 3 weeks I am suffering from cold and caugh. Whilw sneezing I noticed that sneeze from one nostil is in red colour. But I don t have any pain or continous flow of sneeze. Usually I don t take treatment for caugh and cold. Every time they by themselves. Actually I don t have any problem with cold but a little with caugh. What should I do Doc suggest me. Doctor: Hello Welcome to health care magic Dear patient, acute rhinosinusitis generally results from allergy or infections most commonly viral infections but may be super added with bacterial or fungal infections and most of the time the episode resolves with in 10 days but may some times extend upto 4 weeks and a blood tinged nasal discharge during sneezing can happen due to forceful blowing and inflammation that may cause rupture of small blood vessels of nasal mucosa but still it will be better if you consult an ENT surgeon who can examine you and can also get some tests like X-RAY PNS or CT scan PNS done to rule out some other cause of bloody nasal discharge besides rhinosinusitis like polyps or tumours Wish you speedy recovery Thanks Dr Arshad"
},
{
"id": 48068,
"tgt": "What does low eGFR levels indicate?",
"src": "Patient: Hello, Dr. Rynne, I ve been a type 2 diabetic since February 1999 and I m being treated for hypertension. Both conditions are presently well-controlled. On September 9 of this year, blood test revealed eGFR of 57, creatine 1.36, calcium 10.8. My physician told me I was mildly anemic (I told her that I had eaten any meat for 9 months) and she said she d test again in 3 months (end of November). She also said she suspected I was dehydrated and she directed me to resume eating meat and to report for biweekly B-12 shots. Blood test results taken during work-related physical on October 3 indicated eGFR of 54, creatine 1.36 (again) and calcium 10.8 (again). Showed second set of results to my physician who told me not to be too concerned and to await follow-up blood test scheduled for end of November. She doesn t believe I have CKD at this point and feels that the totality of my bloodwork doesn t justify this diagnosis. She stated that eGFR values can move up and down given many factors at time of testing. I have read that once eGFR diminishes, it cannot recover...Doctor, is it possible that I don t yet have kidney disease? And is it possible for eGFR to improve? Doctor: Hi, I appreciate your concerns.The risk factors for CKD in your case are diabetes and blood pressure of long duration. If the GFR remains below 60 for a duration of 3 months we label it as CKD stage 2. If you have significant amount of protein in the urine most likely you have kidney dysfunction secondary to diabetes. But there is nothing to worry as we have diagnosed it early and if taken precautions the eGFR can be maintained for a long time. The key to prevent further progression at this point is a good control of blood sugar and blood presure, weight reduction and regular follow up. The anemia is most likely secondary to nutritional deficiency as you are already taking vitamin supplements.I will e pleased to take any queries from you.Tx"
},
{
"id": 141085,
"tgt": "What causes reduced vision, hearing loss and balance issues?",
"src": "Patient: i am a 55 year old male. i was always told that at the age of 3 i had the symptoms of encephalitis as a results of having had the measles. i have always had vision problems and hearing and also balance problems.my questions are will these problems get worse with age .do i have a greater chance of other brain problems like alsheimers or memory loss or stroke. will i go blind or deaf. is the virus still in my brain lying dormant to strike again.can my balance problems be helped.i should also say that hearing aids don t help. they make the voices louder but understanding is still a problem. i was told this may be due to nerve damage on the brain from the initial virus attack. Doctor: Hi, Detailed evaluation is required. Consult a neurologist and plan for an MRI scan to rule out conditions related to brain and an ENT evaluation also is required to rule out inner ear problems. Hope I have answered your query. Let me know if I can assist you further."
},
{
"id": 129319,
"tgt": "What causes painful burning sensation in the knees?",
"src": "Patient: I hae a burning pain in my knee on the inside edge. I think from reading it is a bursitis. I want to know if it is ok to walk and use it or am I supposed to rest it. It seems to improve but have to keep in certain position when lying at nite or even sitting. Doctor: Hello,Bursitis is a possible cause. However, other possible causes may include arthritis/osteoarthritis, mechanical pain, and tendinitis. I will suggest you try the following:\u2022 Take a pain/anti-inflammatory drug like Tylenol\u2022 A local pain gel like voltarene\u2022 Rest well and observeIf pain persists, you see the doctor for examination and other treatment options.Hope I have answered your query. Let me know if I can assist you further.Regards,Dr. Mbuomboh Veranso"
},
{
"id": 141918,
"tgt": "Does it require the scar tissue to grow for the stimulator implant to work?",
"src": "Patient: In Feb 2014 I had the Spinal Cord Stimulator implant. I have had several meetings with the St. Jude representative to program the unit to send go to my ribcage. I am told now that it may take the scar tissue to form properly before it will be able to get to where it is needed. I gave up the opiate pain killers in hopes the unit was going to work. Does this scar tissue growth ring true to you? Doctor: Hello!My name is Dr. Aida and I am glad to attend you on Healthcaremagic!I agree with your doctor on the fact that it is necessary to take the time for the scar tissue to form properly in order to make further re-positioning of the device. You should just try to be more patient. Hope you will find this answer helpful!Best wishes!"
},
{
"id": 6948,
"tgt": "What diet should be taken as I am very slim and planning for pregnancy ?",
"src": "Patient: Need Diet Plan for Too Slim Girl as preparation for her pregnancy .... Need good diet plan for a girl who is looking too slim as her pregnancy preparation.....the girl is looking below the average slim......she is planning for a baby after 7-8 months........advice a good diet plan (mostly vegetarian) with some supplement tablets.....as her pregnancy preparation...... Doctor: hi welcome to healthcare magic forum to be healthy and slim is different if she is healthy and slim than no need for special plan but if she is weak and slim than she need special plan for pregnancy in pregnancy there is high demand of iron and other minerals she need to start tab of iron and folic acid give her salads ,green veg and fruits tomato,carrot ,spinach ,apple are very good for her HB is very important and need to be checkedi hope i answered your question"
},
{
"id": 172513,
"tgt": "What is the cause of mild dilatation of pelvicalyceal system?",
"src": "Patient: Hello Sir,My Baby boy is 13 days old and recent Ultra Sound Scan shows that he has Mild Dilation of pelvicalyceal on left kidney. What are the cause of mild dilatation of pelvicalyceal system on a newborn baby. Is this Dangerous and risky for the baby's life Can it be treated with Anti Biotics? Or surgery is need to cure this permanently. The size is 16mm Doctor: Mild dilation of pelvicalyceal system needs to be followed up regularly with USG.Renal funtion tests are done to check if the kidneys are functioning well.Often this mild dilatation settles on its own .If not surgical correction is possible.It is not dangerous and life threatening if corrected in time"
},
{
"id": 95421,
"tgt": "Why am I having pain in upper left abdomen ?",
"src": "Patient: i have upper left abdomen pain and i have diabetes what do you think is wrong? Doctor: Hi, Thanks for query, This pain has nothing to do with diabetes. If pain on left upper abdomen but having more on back side and associated with urinary symptoms then possibility of having kidney problem is there. You might have some cardiac pain too,go for ECG to rule out coronary. Ok and bye."
},
{
"id": 122204,
"tgt": "What causes sharp pain in hip after horse riding?",
"src": "Patient: Hi, I ride cutting horses and must always ride with my toes out and legs wrapped around horse. Lately, I have been getting a sharp pain in one hip, especially when I move leg forward (walking and getting on horse). Now I have crunching sounds. The pain comes and goes. I am 62 and very healthy (no issues). I make my living on horses, so this worries me. I jog as well. Any suggestions of what it is? Thank you so much! M Doctor: Hello, Your symptoms could be related to a stretched muscle or sciatic nerve inflammation. For this reason, I recommend consulting with your doctor for a physical exam and some tests: - a lumbar spine X-ray study - coxo-femoral joints X-ray study - complete blood count, PCR, ESR for inflammation - vitamin D plasma levels for possible deficiency. You should discuss with your doctor on the above tests. Hope I have answered your query. Let me know if I can assist you further. Take care Regards, Dr Ilir Sharka, Cardiologist"
},
{
"id": 72027,
"tgt": "What causes sharp chest and shoulder pain?",
"src": "Patient: Hi,I have been experiencing (at least 4x a week for the past 3 months) pain in my right upper chest, under my right shoulder blade and radiating down my right arm.The chest pain is usually a stabbing/sharp pain. The shoulder pain feels like a knot with pressure and is achy all the time. My right arm has an achy feeling all the way down to my hand.I do NOT have my gallbladder anymore. Any ideas? Doctor: Hello dearWarm welcome to Healthcaremagic.comI have evaluated your query in details .* This seems in relation with some sort of nerve compression at proximal level either at thoracic outlet or cervical spine level .* Needs relevant MRI evaluation after thorough clinical examination .Hope this will help to clear your doubt .Wishing you fine recovery .Regards ."
},
{
"id": 188603,
"tgt": "Getting wisdom teeth removed, smoking cannabis drugs. Will it cause detrimental health effects?",
"src": "Patient: I am getting my wisdom teeth removed in early/mid August, it is currently early/mid July. I smoke cannabis mildly (about once a week average) and I have heard that if one smokes cannabis 30 days prior to the operation, it can have detrimental health affects. I take no other drugs/alcohol and I have a very health lifestyle. How soon should I refrain from smoking? Doctor: Hello, Thanks for the question. Smoking pure Cannabis not mixed with tobacco has never been directly associated with any pre/post operative complications in dentistry. However, certain combustion (burning) by products of almost all materials can be carcinogenic (cancer causing). Marijuana use is also implicated in Hypotension (reduced blood pressure). You should also bare in mind that Cannabis (Aka. Pot, Marijuana etc) is an illegal drug (narcotic) in almost all countries. If you are using Marijuana medicinally i.e if Medical use is permitted where you live, you should talk to your doctor before discontinuing the drug. Having said that, there is no ascribed safe period. In any case, don't go for you appointment 'high', stop using it at least 1 week before the surgery and abstain for at least 2 week post operative. The act of dragging on your joint/reefer/bong etc will cause negative pressure in your oral cavity that can lead to clot displacement. This can cause a painful condition called 'Dry Socket'. Hope my answer was useful to you."
},
{
"id": 150970,
"tgt": "Child had high fever, excessive sleep, later fits. CT scan showed brain swelling. What could have happened?",
"src": "Patient: HELLO,MY NEPHEW OF 1, 1/2 YEAR GOT FEVER LAST WEEK,IT WAS 103 F, WE TOOK HIM TO DOCTOR,WHO GAVE MEDICINES,ON THE THIRD DAY OF FEVER HE WAS SLEEPING WE TRIED TO WAKE HIM UP BUT HE IS AGAIN SLEEPING,WE IMMEDIATELY TOOK HIM TO HOSPITAL WE THE DOCTORS CONDUCTED CT SCAN,IT SHOWED SOME SWELLING ON THE BRAIN,AGAIN THEY TOOK FLUID FROM HIS BACK BONE THE RESULT OF WHICH WAS NEGATIVE, THEN THEY CONDUCTED MRI ON HIS BRAIN IT SHOWED LOT OF SWELLINGS ON THE RIGHT SIDE,IMMEDIATELY AFTER MRI HE GOT FITS SINCE THEN HE IS UNCONCIOUS THE DOCTORS PUT HIM ON A VENTILATOR SO THAT HE CAN BREADTH EASILY,THE CAUSE OF DAMAGE TO THE BRAIN IS STILL UNKNOWN,TODAY THEY REMOVED VENTILATOR AT ABOUT 1400 HRS AND KEPT HIM UNDER OBSERVATION WITHOUT VENTILATOR.NOW THE DOCTORS CALLED ME AND INFORMED THAT AGAIN HE GOT FITS AND THEY ARE PUTTING HIM ON VENTILTOR. I AM STILL CONFUSED WHAT HAPPENED TO MY NEPHEW.KINDLY ADVICE. Doctor: Hello I have gone through your complaints. I am sorry for your child. Actually your child is having some brain lesions. Brain is the organ which controls all systems. If any lesions are present in brain, it will present to us as fits. While during fits, the child can't breathe and they will go into respiratory arrest. It is the condition where the child won't breathe for minutes together and if it prolongs, the heart also stops. So while during fits your child needs respiratory support which will be provided by a ventilator. So please follow your doctors advice and doings. Thank you. Good day."
},
{
"id": 43417,
"tgt": "Unable to conceive, taking Folic acid pills. Will it have any side effect?",
"src": "Patient: Hi sir thi is xyz n my probs is that my wife and me wnt to have a baby since 7 moths we tried but not getting succed, we went to doc he give some kind of tblets like folic acid and my wife starting taking it, i jst wnt to knw is there any side effect of taking those pills..? Also i would like to knw is there any medicine to make spearms stronger..?? Thank you Doctor: Hello,Folic acid is considered to be a prenatal medication and is harmless. There is no specific medicine to make stronger sperms; medicines are available to increase sperm count but not motility. You may take certain precautions like avoiding hot baths,tight undergarments, trauma to the scrotal region, avoidance of smoking and alcohol. You may consult an infertility specialist for a complete workup and advice. Hope your query is answered. Take care."
},
{
"id": 118217,
"tgt": "What could be reason for swollen cubital lymphnode?",
"src": "Patient: I have a swollen cubital lymphnode and I dont know why. What could have caused it and what can be done to get the swelling out and the pain to go away. It feels like I have a golf ball just above my elbow and on the underside of my arm at about the 5 oclock position with my palm facing down Doctor: May u have some skin lesion on ur arm.if not get a biopsy of this node and let me know.u also verify that its a lymph node and not tennis elbow swelling."
},
{
"id": 26055,
"tgt": "Why do I have heart palpitations with shivers?",
"src": "Patient: I have felt like I've had a few heart palpitations...feeling shaky inside.. little or no appetite ...and a tiny bit of diarrhea. I am currently on a generic Keppra from where I had one seizure months ago....and on lisinopril 10 mg for blood pressure....I do not have actual chest pain. I'm trying to cipher if anxiety or something else Doctor: Hello!Welcome and thank you for asking on HCM!I understand your concern and would explain that your symptoms seem to be related to a viral infection. What is the dose of levetiracetam that you are taking? What is your body temperature? Do you have fevers?I recommend consulting with your GP for a careful physical examination, a resting ECG, an abdominal ultrasound, a stool analysis and some blood lab tests: -complete blood count-PCR, sedimentation rate-blood electrolytes-kidney and liver function tests-blood bacterial culture. You should discuss with your doctor on the above mentioned issues. Hope to have been helpful!Best wishes!Dr. Iliri"
},
{
"id": 184208,
"tgt": "What are the side effects of aleve ?",
"src": "Patient: I have a toothache and in alot of pain. I took 3 aleve today the last two being about 3pm today. I dont see the dentist till tomm at 2:30 pm...can I take more Aleve without the fear of overdosing since that would be limit of 3 today. the first one was at 7am Doctor: Hi,Thanks for posting the query, Toothache indicates infection of tooth, such type of infected tooth can be treated with root canal treatment, i would suggest you to take tab almoxiclav along with tab aceclofenac TDS till you see the Dentist,this will surely subside pain and infection, dont overdose yourself with medications and try to get an apoointmet earlier with your Dentist.Take care!"
},
{
"id": 77428,
"tgt": "Suggest remedy for chest pain due to choking accident",
"src": "Patient: Hi there, I'm a 17/yr old female and yesterday I had a pretty serious choking accident. I was drinking iced tea and my friend made me laugh, I couldn't breath for probably 30 seconds and she was 2 seconds away from doing the heimlich. Afterwards, and still today, my chest hurts if I take deep breaths or sneeze. Breathing normally seems to be alright though. I wanted to know if this is something serious and if I should get checked out by a doctor? Doctor: Thanks for your question on Health Care Magic. I can understand your concern. No need to worry for this kind of chest pain. This is not serious symptom. It will mostly resolve in 1-2 days. So no need to consult doctor. Apply warm water pad on affected areas. Take simple painkiller like paracetamol. Don't worry, you will be alright. If not improving in 1-2 days then consult doctor. Hope I have solved your query. I will be happy to help you further. Wish you good health. Thanks."
},
{
"id": 177357,
"tgt": "What causes intense headaches in a 17 year old?",
"src": "Patient: My 17 year old daughter is having intense headaches that are increasing in frequency. She has had a steady headache for 6 days, and she is also having a stabbing pain occasionally that she describes as an axe being dropped into the middle of head. She says this stabbing pain is new - she has never felt anything like it before. She has experienced a little numbness on left side (but that was a few weeks ago and it was not related to this headache episode). She has been dizzy with this headache. We had blood tests run to see if she was anemic, but she is not. Any ideas? Doctor: Hi...this could be a migraine or a stress headache or a cluster headache. Especially in this age it is quite common. A small question is that was the blood pressure checked when she got into contact with medical personnel. Being dizzy could be a sign of aura of headache. But what I suggest is that such sever headache should definitely be evaluated. Iron deficiency cannot be the sole reason for this.Regards - DR. Sumanth"
},
{
"id": 14129,
"tgt": "Suggest medication prolonged itchy rashes",
"src": "Patient: itchy rash, rash starts as itchy then small blisters and then a red bumpy rash. rash looks like eczema or scabies. has gone on for 4 monthshave tried several lotions and ointments including perscription ointmentsother family members now have the same thing Doctor: Hello and Welcome to \u2018Ask A Doctor\u2019 service. I have reviewed your query and here is my advice. Family history of itchy rashes is very suggestive of scabies. I suggest you to use Permethrin 5% cream, single application, whole body, from neck downwards, kept overnight for 8 hours. This should be applied by each and every affected family member. In addition I suggest you all to take an oral antihistamine e.g Cetrizine tablet for symptomatic relief from itching. Hope I have answered your query. Let me know if I can assist you further."
},
{
"id": 110872,
"tgt": "What causes lower back pain?",
"src": "Patient: So I have back pain on the lower left side... It has moved around to the abdomen too and down my left leg... It's been going on for over a month.. Constant pain, and now I have a bruise forming on my lower left side of my back where the pain started... I am male, 30 years old. Doctor: Hello, Thanks for your query.From description it seems you might be suffering from siatica. Sciatica refers to pain that radiates along the path of the sciatic nerve \u2014 which branches from your lower back through your hips and buttocks and down each leg. Typically, sciatica affects only one side of your body. Although the pain associated with sciatica can be severe, most cases resolve with just conservative treatments in a few weeks.MRI is needed to establish the cause of sciatica, so that appropriate treatment can be prescribed. Sciatica do respond well to bed rest and analgesics (pain killer) and is right mode of treatment. I do hope that you have found something helpful and I will be glad to answer any further query.Take care"
},
{
"id": 176427,
"tgt": "What dosage of Ibugrsic syrup can be given to an infant?",
"src": "Patient: Hello, In what doesge I can give ibugrsic syrup to my 5moths and 16 days old daughter - weight 5.6 kg and 5 year old son - weight 16 kg. We will be 18 hr long flight and both of them are having cold and cough. They have their antibiotic going on for last 7 days now and we are travelling tomorrow. I have used this medicen for them earlier too, just don t rember the dosage. Please advice. Thanks, Manyam Doctor: Hi...the dosage is 10mg/kg per dose thrice a day. The strength of the formulation is 100mg/5ml. 1. For 5.6 kg the dose will be 2.5ml thrice a day.2. For 16kg the dose will be 7.5ml thrice a day.Regards - Dr. Sumanth"
},
{
"id": 56229,
"tgt": "Suggest treatment to remove blockage in gallbladder",
"src": "Patient: yes I visited a dr three yrs ago he told mr I have a blockage do to a bile located with the gallbladder the nurse says I need a lot fiber to slow down the process I do experience some pain I can t eat certain foods as a result how can I remove the blockage without having surgery Doctor: A biliaary diskinesia I guess from what you've wroten, without lithiasis or sludge in your gallblader. I agree with the prescribed by your nurse dietetic changes and there are some cholagogues and choleretics to help you probably before thinking of surgery like Cinnarix i.e."
},
{
"id": 131170,
"tgt": "I have chest pain when i breathe caused by an injury",
"src": "Patient: I have been sick with sinus. But now I have bad chest pain under my arm by my breast. It hurts when I breath and just moving, I also fell off of the soft, in my sleep. So I wasn t sure I hurt a rib falling, or pnuemonia, type thing.i do not have a cough Doctor: that is exactly a problem of a rib fracture or a chest wall contusionthe clincher there is localized pain on a particular area upon deep breathingrecovery may take 2 to 4weeksbut no surgery or bracing necessary for it to heal"
},
{
"id": 12806,
"tgt": "Suggest remedy for skin rashes in infants",
"src": "Patient: Hello Doctor: Am blessed with a Baby Boy on 27- Oct. He is 13 days old.We find some rashes on his face and body. We are concern of his rashes and assuming is this due to the baby soap we are using. Some of them suggested to use Dermadew. What is your suggestion?Thanks Prem Doctor: Hello,Rashes coming on your child body are sometimes normal, however he may be having atopic dermatitis. This skin problem is quite common in babies now a days.I will advise you to stop massage oils. Dermadew baby soap is a better option than dermadew plain. In case rash does not subside, better to consult a dermatologist.Hope I have answered your question. Let me know if I can assist you further. Regards, Dr. R. G. Joshi, Dermatologist"
},
{
"id": 54926,
"tgt": "What does low antibodies to hepatitis b signify?",
"src": "Patient: I have very low antibodies to hep b and I have never gotten the vaccine. Does this mean I have chronic hep b or am I a carrier???? I'am so scared. Everyone explains this differently. Does having very low antibodies,2.8 mIU/mL,mean that I have chronic hep b?? Doctor: Hi thanks for asking question.Your doubt us obvious.But here if you provide full report then i can give still more comment...First you have not vaccinated.So here you have form protective antibody after getting infection.Usually after resolving acute infection protective antibody remain for long time....And here HBV DNA , HBSag result needed...If HBSag also positive then chance of chronic infection ....You can follow up with full report of viral marker to gastroenterologist.Happy to help you further.Take care"
},
{
"id": 148796,
"tgt": "Recurring pleurisy. Will coffee help the inflammation or worsen it ?",
"src": "Patient: I have recurring pleurisy and have for four years now. I am only twenty years old. Will coffee help the inflammation or make it worse? I don't like taking medications and the only things that have helped is coffee and steroids. No matter what I do it keeps coming back and it's never when I have a cold or anything it just happens randomly. Doctor: your steroids are not good for you coffe can increase the inflamationi think you are getting allergy from some proteins may be food or otheri suggest to get allergy doctor advise to find the proteins you are allerguic to and withdrawing can help ypo better than steroids as i havetreated many like this"
},
{
"id": 86055,
"tgt": "What causes dizziness and left sided abdominal pain?",
"src": "Patient: .the last week I've been feeling a rush over my face and tongue, I'm really light headed and dizzy almost passing out. I'm2 or 3 weeks pregnant. What do you think it is? I'm also getting a abdominal pain in my left side almost like a kidney infection. Doctor: Hello and Welcome to \u2018Ask A Doctor\u2019 service. I have reviewed your query and here is my advice. * As far as my clinical experience is concerned, the dizziness and left sided abdomen pain seems to be early hormone changes from the underlying pregnancy. * Kidney infection should be associated with severe loin pain, dysuria with or without fever or other systemic features, which can be easily differentiated from lab tests and clinical findings. Hope I have answered your query. Let me know if I can assist you further."
},
{
"id": 103656,
"tgt": "Small red bumps on back, legs, ingrown hair. Allergic to penicillin. Causes and treatment?",
"src": "Patient: From my upper back down to the back of my legs, I have small red bumps cropping up. I didn t notice until a few days ago since it s only along my back. They re not itchy nor painful at all. They re just there. Some of them look like they re just caused by ingrown hair . I don t think it s an allergy because my diet has stayed the same and I ve also lived in the same place for years and the only allergy i know that I have is to penicillin . Doctor: these are acne piples as rhe acne can attack face or back of skinavoid oily reesy fat foodlot of warm water non toxic soaps shampoo and oils no creamsapply climdamycin gel bdand tretinoic acid gel at night on affected skin switch off lightas it reats with lightebastine 10 mg sos for itchno use of peicillin group of saltsno brufen pcm diet suppliments vitamins calcium as they trogger skin allergy and acne"
},
{
"id": 88058,
"tgt": "What causes sleepiness and stomach cramps with lower level of potassium?",
"src": "Patient: I was just released from the hospital on May 21st. I was admitted because I could not urinate which was brought on all of a sudden. They treated me for low potassium levels of 2.4 and inflamation of the intestines. For the last two days all I want to do is sleep. I feel exhausted and can t stay awake ad I having stomach cramps again. Should I go to the hospital now or wait till morning and get a doctors appt. I don t know if this could be low potassium again. I don t want to over react. Doctor: Hi.Thanks for your query and an elucidate history and understanding of your problem. The over sleepiness and exhaustion can be due to low potassium levels as you have suspected correctly. The cause is the inflammation of the intestines , not allowing the absorption or the loss of potassium into the intestines from the body, what we call a third-space loss. You need to go back to the hospital. and get investigated urgently, particularly the serum electrolytes. and get corrected the balance of electrolytes, get proper supportive treatment and the treatment for your intestinal infection as unless it is corrected the other problems will prevail."
},
{
"id": 176354,
"tgt": "What causes vomiting and Diarrhea in a 30 month old with a sweet pungent smell?",
"src": "Patient: Hi, my daughter is 30 months old. She has vomiting and diarrhoea which both have a strange sweet smell. Vomiting is occasional, diarrhoea frequent and large volumes. I can hear her tummy bubbling and gurgling. She will take some liquids but not lots, she wants to drink milk. She has eaten small amounts of food which doesn t seem to have made it any worse. She has had a mild temperature but nothing serious. I am wondering if she has norovirus? If so should I be giving her any meds or doing anything different? In between episodes she is bouncing round full of energy. Doctor: Hi, diarrhoea till the age of 5 years is a common condition. Keep good hydration of your child by giving her lots of fluids, water and ORS liquid.Give Syrup Zincolife 5 ml once daily for 14 days. Also give her some probiotics like Syrup Bifilac 2 ml twice daily. Af for feeds, give her regular food less oily and less spicy along with lots of curd.There is no role of antibiotics if diarrhoea is transient. If it persists, give her Syrup Oflox-Oz twice daily.Thanks and regards."
},
{
"id": 86744,
"tgt": "What causes abdominal pain with a history of diverticulitis surgery?",
"src": "Patient: hi I just got my ultra sound results.left ovary is seen didn t see rt ovary.where could it have gone?i have had numerous surgerys including diverticulitis surgery,removed a section of sm intestine which endedwith an ileostomy.ilistomy reversed,incesion opened and I had a huge hernia in sm intestinewhich was repaired with mesh.sincei have had rt sider lower abdominal pain. past 2months has gotten worse .pulling which causes pain.feels betterwith rt leg bent toward stomach and a pillow for pressure.sm skinny stools constipation,use mirralax with poor results. surgeon who repaired hernia has no idea why the pain.what happened to my right ovary???any ideas ? Doctor: Hi.Thanks for your query and an elucidate history.Read and understood the history of multiple surgery for Diverticultis , small bowel resection, ileostomy , Incisional Hernia repair, Right ovary not sen on ultrasonography. With so many surgeries it is possible that the ovary is difficult to be seen.For proper evaluation CT scan of the abdomen only can show the real position of the ovary on the right side and associated problems related to bowel, lower abdominal pains, skinny stools and all. The results of the CT and Enteroclysis will help to decide future course of action."
},
{
"id": 220887,
"tgt": "Suggest treatment for diarrhea in 38 weeks of pregnancy",
"src": "Patient: I had 4 loose stools of diarrhea last night that were normal in color. Then today about 8 hours since my last episode of diarrhea I had a light bright green stool - like the color of green tea, soft in texture and medium in size. I have never in my life had a stool this color and I have not eaten anything green today at all and I have not changed my prenantal vitamins. Last night for supper I had cucumbers and yesterday at lunch I had green beans. I did take simethicone 370 mg total last night due to severe bloating and heartburn. I am 38 weeks and two days pregnant. I do not know what to think. Doctor: Hallow Mumma,Green colour stools suggest that you are having some intestinal infection which has resulted in diarrhoea. Now you have two intentions:1. Maintain the hydration: After each loose motion, please consume a glassful of oral rehydration solution like Electral. You should not get dehydrated. Ensure that you are passing good amount of urine. This is a good sign of proper hydration. Also observe your tongue which should be wet. If you pinch the skin on the back of your hand and release it, it should reform again without leaving the fold of skin. This is another test of hydration. Usually within 24 hours the infection gets washed off. 2. Report to the doctor ASAP. If the infection is not under control in spite of good hydration after 24 hours, you may require some antibiotics. Your Obstetrician will assess and decide about antibiotics. Since you have crossed 38 weeks of gestation, even if you go in labour, there is no harm to the baby; however you should land in labour in a healthy condition. Maintenance of hydration is a key factor. I hope this helps you.Dr. Nishikant Shrotri"
},
{
"id": 197557,
"tgt": "What are the bumps in between my thigh and scrotum?",
"src": "Patient: I am 19 years old and I am sexually active. Here recently I have been getting bumps but they are not painful in anyway at all. They occur in the area between my thigh and my scrotum. They come and go from time to time. Everytime I pop one I dont get any scabs or anything or liquid pus, It just real thick like when I pop on my face. It doesnt itch or irrate me at all, it also doesnt get redness around it. It is only in that one area, I dont get lesions or any spot on my penis. What can it be. Doctor: HelloThanks for the queryProbably you have developed genital warts or molluscum contagiousum a viral infection. Your description matches with that. A photograph would have helped me in giving a closer diagnosis. Please meet a dermatologist and get yourself examined.Various treatment modalities like radiofrequency ablator and CO2 laser are available.Let me know if you have any other doubts. Thank you"
},
{
"id": 168764,
"tgt": "What causes rashes and swelling on left side of the body in a child?",
"src": "Patient: Hi, my son had swelling and rashes two days ago, the pedi prscibed stereoids and benidryl to help bring swelling down, and referred us to allergy clinic. I was wondering why his swelling was mainly on the left side, if it s only an allergy, why wasn t it same throroughout? For example, both eyes were solle, but you could barely tell it on the right eye, and the left eye was nearly shut... Same with hands, feet, rashes appeared more severely on left as well Doctor: Hi...Thank you for consulting in Health Care magic.By what you quote it should be an urticarial or a simple skin allergy. You can use Hydroxyzine at 1-2mg/kg/dose every 6th to 8th hourly for 7-10 days. Most important thing to be remembered is that it has a propensity to recur (called as second crop) within 10-14 days. If this happens, you can start using the same medicine but I suggest you get the kid evaluated with your paediatrician.Hope my answer was helpful for you. I am happy to help any time. Further clarifications and consultations on Health care magic are welcome. If you do not have any clarifications, you can close the discussion and rate the answer. Wish your kid good health.Dr. Sumanth MBBS., DCH., DNB (Paed).,"
},
{
"id": 169824,
"tgt": "What causes blood in urine and vaginal burning?",
"src": "Patient: My daughter had a routine check up; she is 10 yrs old. The doctor saw a little blood in her urine, and sent it to the lab on Wednesday, 15 June. Results aren't back; my daughter is scheduled to fly out of town on Saturday, 18 June. She has complained of vaginal burning, but she doesn't complain everyday. Any suggestions? Doctor: Hello, awaiting the urine report, you may please make her sit in a tub of water with potassium permanganate solution, for 5 min 2 times a day.rest has to be evacuated Disclaimer\"This provisional advice provided by me stands subject to the patient undergoing a physical examination and is based entirely on inputs provided by patient/attendants. The patient is advised for physical examination at earliest\""
},
{
"id": 135430,
"tgt": "Suggest treatment for severe knee pain",
"src": "Patient: Hi I m a 53 yr old plumber and I m getting terrible pains in my knees its like somebody s sticking to a knife in them when im climbing stairs and I can t crouch down, and I get sciatica regular in both my legs and its always seems like it starts in my bum, Doctor: dear friendit seems you are developing osteoarthritis of the knees with some amount of tendonitis of the patellar tendons.please get an x ray of the knees done to establish a proper diagnosis, in case the osteo arthritis has set in there are many treatment options available which will be suggested by the orthopedician seeing you depending on the severity of the condition."
},
{
"id": 112839,
"tgt": "Severe back pain, diabetic. Done MRI, disc herniation. Have had radio frequency ablations. Given tramadol, zanaflax. Further?",
"src": "Patient: Severe back pain since 2007 which is worsening every day. Today I am a 8/10 on pain scale with meds. 43 year old female, type 1 Diabetic. My most recent MRi was done 3/15/13 on a 3.0 Tesla Ultra High Filed Skyra Wide Bone MRI. I have had many epdurals in these areas as well as many Radiofrequency AblationsNo narcotic pain medication helps. I have been given dilaudid, morphine, oxycontin, oxycodone, Opana IR, Tramadol, Nortriptyline, Pamelor, Cymbalta, Elevil, Zanaflaz, Flexoril, Amrix, Ibruprphen 800, Lidoderm patches, Flector patches, Trigger point injections, and finally Fentanyl patches which I change every 48 hrs on 100mcg. They work but for constant pain they don't and no medications work for breathrough pain.Considering North American Spine Institute Accurascope all the way in dallas.Disc desiccation at L2-L3, L3-L4 and L4-L5At L-23 level there is a disc space narowing and circumferential disc bulging without central canal or neural foraminal compromise.At the L-3-L4 level there is a circumferential disc bulging with a left forminal disc herniation resulting in narrowing of the left neural formenAt the L4-L5 level there is circumferential disc bulging with a small superimposed central and left sided disc herniation. There is a mass effect upon the descending left L5 nerve rootImpression: Developing disc bulging at L2-L3Stabble L3-L4 left formaminal and L4-5 central and left sided disc herniation Doctor: Your spine surgeon should think now for performing disc removal/laminectomy etc or spinal fusion to very bad areas to relieve your pains and all radiations . this is the only solution as all modalities have been tried on you alreadyjust trust the treating doctor who might be already planning to give you surgeryas ultimate remedy"
},
{
"id": 163499,
"tgt": "Suggest remedy for viral infection along with diarrhea in children",
"src": "Patient: Hi, may I answer your health queries right now ? Please type your query here...My 14 month old has had a viral infection for 4 days and has now been given augmentin due to the change in colour of his nasal secretions. we also keep his temperature down. Is there anything I can give him with the augmentin as he already has diarrhea. He just wants to sleep all the time - he was using voltaren and panado suppositories - which I have now stopped and only using liquid ponstel Doctor: Hello,Cough and cold are viral 95% of the times in children. So antibiotics are not required and if unnecessarily used may contribute to antibiotics resistance. For cold, you can use anti-allergics like cetirizine and for nose block, saline nasal decongestants will do. Paracetamol can be given in the dose of 15mg/kg/dose (max ceiling dose 500mg) every 4-6th hourly, that too only if fever is more than 100F. I suggest not using combination medicines for fever, especially with Paracetamol. For cold, you can use Cetirizine at 0.25mg/kg/dose every 12 hourly for 3 days. For nasal block, plain saline nasal drops will do, every 4-6th hourly to relieve nasal congestion.Hope I have answered your query. Let me know if I can assist you further.Regards, Dr. Sumanth Amperayani"
},
{
"id": 6752,
"tgt": "What is the success rate of ET for blocked tubes as I have thyroid ?",
"src": "Patient: iam rahini kumar 41 yrs 5 5\\ 72kg.i have been adviced to on for ET as i have tube blocks, pls advice me what will be the successful rate in this as i have thyroid. Doctor: Hello Thanks for query Befor going for ET ,please insure your thyroid profile is normal by taking appropriate treatment. Success rate of ET varies from centre to centre from20-40%.For chronic tube bloched at the age of 41 ,ET is the best alternative.So go head. Hope I have answered your question well to your satisfaction. Wish you speedy complete recovery at the earliest."
},
{
"id": 125597,
"tgt": "What caused pain in left side of my neck and shoulder area?",
"src": "Patient: My left side of my neck at the cartarid artery and shoulder area is in pain. I also have recently had driving issues where my body seems to be veering to the left side weaving in and out of lanes? Are these two related and serious enough to be worried? Doctor: Hello, It could be due to conditions like cervical spondylitis. As a first line management, you can take analgesics like Paracetamol or Aceclofenac for pain relief. If symptoms persist better to consult an orthopaedician and plan for an MRI scan. Hope I have answered your query. Let me know if I can assist you further. Take care Regards, Dr Shinas Hussain, General & Family Physician"
},
{
"id": 25399,
"tgt": "What are the chances of survival from triple bypass surgery?",
"src": "Patient: yes please. if you have triple bypass surgery 65 yr old female 9 days after major heart attack and you are still on ventilator tho breathing on your own but also severe body swelling, hands legs arms and cannot keep eyes open what are your chances of survival Doctor: Hi, I understand your concern and I'll try help you. Survival after coronary artery bypass surgery largely depends on the previous state of the patient and the presence or absence of complications during and after surgery. Patients with cardiac function close to normal before surgery, with a good clinical status and who does not suffer complications during surgery or in the postoperative period, the chance of survival reaches 95%.According to what you reported, the patient is not having a good outcome in the postoperative period of cardiac surgery. And the patient seems to be with some organ dysfunction demonstrated by severe swelling and difficulty opening the eyes. Such problems can tert origin for renal dysfunction, cardiac dysfunction and even neurological dysfunction. To get a better idea of what is happening, it would require larger clinical data.Therefore it is difficult to estimate the chance of survival of this patient, however, it is much lower than the ones who have normal clinical outcome after cardiac bypass surgery, regardless of the number of grafts performed.Hope I have answered your question. If you have any further questions I will be happy to help you. Wish you good health."
},
{
"id": 177074,
"tgt": "What are the chances of tapeworm infection transmission from dogs to children?",
"src": "Patient: hi. my dog has just been diagnosed with tapeworms. I have a 4 month old baby. is there a chance my daughter could have been infected with tapeworms? she appears well and is gaining weight and achieving milestones normally. she has had all of her normal vaccinations. Should we treat my daughter speculatively? is there any test that can confirm worm infestation or reliably exclude it? Doctor: cestodes(tapeworm) can be transmitted from animals to humans via feco-oral route. or by eating undercooked or raw meat of pork or beef. Humans are the definitive host and easily infected by animals. Give your child a course of Syp Zentel or Albendazole(200 mg/5ml) 10 ml once daily for 3 days. It will kill any cestodes in her body and will de worm her also if any other parasitic infestation is present. Also during this period child is fond of putting every thing in her mouth. Its advisable to de worm your child with Syp Albendazole every 6 monthly."
},
{
"id": 78787,
"tgt": "Suggest treatment for bronchitis and sinus infection",
"src": "Patient: I was diagnosed with Bronchitus, a double ear infection and sinus infection on the 17th of this month. I had to go back to the doctor for different meds and a nebulizer treatment Friday the 20th and back to another doc on Monday the 23. I have been on a z-pack, steroids, cough syrup with codeine and have had an at home nebulizer for 2 days and am currently taking levaquin. Today I am exhausted and having crazy hot flashes with sweats followed closely by extreme cold. Am I going insane? what can I do to fix this? I m afraid to go back to the doctor, because I am never sick and I am worried they are going to think I am just whinning. Doctor: Thanks for your question on Health Care Magic. I can understand your concern. Get a chest x ray done, if it is normal, no need to worry for lung infection. So better to consult pulmonologist and get done clinical examination of respiratory system and PFT (Pulmonary Function Test). PFT is needed for the diagnosis of bronchitis. It will also tell you about severity of the disease and treatment of bronchitis is based on severity only. You may need inhaled bronchodilators and inhaled corticosteroid (ICS). Don't worry, you will be alright. Hope I have solved your query. Wish you good health. Thanks."
},
{
"id": 180640,
"tgt": "What causes numbness on the tongue after smoking Marijuana?",
"src": "Patient: I rencently took a few hits of dope then after awhile i picked up the needle part of a rig in my backseat. I know it's not mine, also the beedle never poked my skin. Just a few minutes later I puked and my throat became very scratchy and then my tongue went numb.. what caused my toungue to go numb cause it won't go away? Doctor: Hello,Numbing of the tongue after doping can be due to reaction to the drug while it can also be due to tongue irritation due to puking. As the effects of the drug will subside the numbness will get better. You should consult an oral physician and get evaluated. For now, -Drink plenty of water. -Avoid spicy foods and do cool compresses over the tongue. -Take vitamin B complex supplements. Hope I have answered your query. Let me know if I can assist you further.Regards,Dr. Honey Arora"
},
{
"id": 155438,
"tgt": "What to do in level 4 node involvement in malignant melanoma",
"src": "Patient: I have a patient aged 49 years with mucoal primaries. in tongue (Malignant Melanoma). No neck nodes initially N So, Hemiglossectomy done and level 1, 2 and 3 nodes removed biopsy negative. Nodes After one year level 4 node involved. Radical Neck dissection done. What is the future course of action. Doctor: Malignant melanoma of the tongue is a rare and aggressive cancer that has the tendency to come back and spread. Your treatment so far appears to have been correct and scientific. My only concern is to rule out that the cancer has not spread to other parts of the body. For this, a PET scan will be ideal as it will scan the whole body. If it has not spread then there is no further treatment needed except close follow up with an oncosurgeon."
},
{
"id": 1408,
"tgt": "Can I become pregnant without missing the period?",
"src": "Patient: i am 24,female with no medical complications.i take no medicines at present.i have good general health.i have been having abdominal cramp when i ovulate ever since my periods started.i have had it checked by a doctor and she said it is normal.i have seen that from the day of ovulation i have my next periods after 16 days.that is when i ovulate on the 11th day of my cycle i have my next period on the 28th day and when i ovulate on the 15th day then my periods start on the 31st day.in February i had my periods 7th feb but i had my next ovulation on 26th feb that is the 20th day of my cycle.therefore my periods started on the 13th march instead of on 6th march which was the 28th day.however i had a normal to heavy flow like usual with usual cramps for 4 days.i am very worried about this sudden delay.can i be pregnant even after having my period?i have tested with a home pregnancy kit and every time it has been negative.now i am having nipple tenderness(which i often get a week before my periods) and i am expecting my periods on 12th of april since i had my ovulation on 27th march(the ovulation cramp).please help me at the earliest.i am very worried and scared.thank you Doctor: Hi ,How are you doing ?At this young age with no complications, I can think of 2 possibilities.1. Pelvic Inflammatory Disease ( PID ) - infection arising from vagina , after sex. these infection can affect all the structures around uterus - this includes the tubes , ovaries & adnexa. 2. Endometriosis - can occur at any age. Bleeding from the uterus during periods is to the exterior , but a small amount of blood enters into the abdomen, through the tube. This blood forms clots & adhesions around the uterus.Both these conditions around uterus cause increase of blood flow to structures like ovaries , tubes , muscles & ligaments around uterus.So when ovulation approaches, this cause congestion & pain around the lower abdomen, thighs , lower back .Both need evaluation by gynecologist. PID needs treatment with anitbiotics & antifungals, both husband & wifeFor endometriosis- have Combined oral contraceptive pills cyclically will stop ovulation & will stop pain & progress of endometriosis. If you are seriously trying for pregnancy, do have tubal patency evaluated - HSG or SSG or Laparoscopy , do rule out tube block , which can hinder your chance of pregnancy.Hope I have cleared your queryAll the best"
},
{
"id": 84549,
"tgt": "What are the side effects of depo shots?",
"src": "Patient: Had a baby 8 mos ago (natural delivery) had some minor cramps in between cycles since. Just started depo the cramps are much worse now is that normal? I see I have the spotting side effect which is odd, I ve been on the depo before my baby for like ten years never had any problem with it. Doctor: Hi,They may include menstrual changes, fluid retention, weight gain, nausea, vomiting, abdominal pain, indigestion, bloating, dizziness, headache, fatigue, drowsiness, irritability, breast tenderness, acne, hair loss, decreased libido, hot flashes and flashes, mood changes, emotional liability and joints pain.Hope I have answered your query. Let me know if I can assist you further. Regards, Dr. Ajeet Singh, General & Family Physician"
},
{
"id": 29374,
"tgt": "Suggest medications for sinusitis",
"src": "Patient: I have severe pain in the soft palate area. Bright red post nasal drip. Lightheadedness. Weakness. Cold, frozen feeling behind left ear. No fever. No strep, been tested twice. On amoxicillin 500mg 3x daily 10 day course. Just completed day 5. Prednisone 2 days completed of 6 day course. No improvement. Doctor: Hi..Thanks for the query..Post nasal drip, pain in soft palate and cold frozen feeling is appearing due to sinus infection.First of all get a sinus x ray done to confirm the diagnosis.If it is confirmed and amoxicillin did not work then Augmentin [amoxicillin +Clavulanate potassium] can be taken.Other groups of antibiotics like Cephalosporins etc can help..Along with it take anti inflammatory painkillers like Ibuprofen.Use specialised mouthwash like Magic mouthwash to gargles.Do warm saline gargles.Take anti histamines like Levocetrizine and decongestant like Phenylephrine..Do steam inhalation and saline nasal irrigation..Hope this helps..Regards."
},
{
"id": 161271,
"tgt": "What could be the cause for vomiting at night in infants?",
"src": "Patient: My 4 year old daughter has been throwing up every night for the past week. She is fine during the day and plays , laughs and eats but after being put to bed she wakes up out of a dead sleep and instantly vomits a large amount then goes back to sleep as normal. Do you have any clue what Is causing this ? Doctor: Hello, Please check with gastroenterologist. If she is throwing up partially digested food every night it may be due to achalasia cardia or dysmotility disorders..barium swallow to be done to confirm diagnosis. Hope I have answered your query. Let me know if I can assist you further. Take care Regards, Dr Prasanna Lakshmi, Pediatricia"
},
{
"id": 188980,
"tgt": "Had temporary crown placed. Pain in temple. Done root canal treatment. Any thoughts?",
"src": "Patient: Had a temp crown placed, pain in right temple has been excrusiating so I went back twice and they reset my bite by adjusting/grinding crown. Still no help so went to a specialist in endodontics and his final test was to numb the nerve area of the temp crown tooth and my headache went away!! With this info he performed a root canal treatment. Procedure went fine but when I got home and the numbness wore off my headache came back immediately??? Any thought on why this did not fix my issue? Headache pain is terrible! Doctor: Hello,Thanks for posting your query.Headache post-rct usually lasts for 3-4 days.Take complete course of analgesics-antibiotics prescribed by your dentist.Avoid anxiety and take complete rest.Maintain oral hygiene well.Also get your TMJ examined and check for dislocation/tenderness.Take care."
},
{
"id": 87580,
"tgt": "What causes persistent abdominal bloating and pain?",
"src": "Patient: hi. I am constantly bloated and have pain in my stomach. I can only eat very small amounts of food at a time (like a piece of bread) and then I feel very full and sore, the more i try and eat the worse the pain and bloating is. i am losing weight and quite often feeling sick and tired Doctor: HiIf i would be your treating doctor I would suggest you to take pancharista 2-3 tsp twice a day. and suggest you for ultrasound for diagnosis.I think You should change your diet.take light food & avoid outside food.As this is prescription medicine consult to local doctor."
},
{
"id": 150192,
"tgt": "Dizziness spells, nausea and sweating. Reason?",
"src": "Patient: Starting Mar2013 I have experienced about 7 spells, I call them. I all of a sudden get very dizzy have to sit down because it is quite servere . I let it pass for about 15 min by sitting and closing my eyes. Then I break into a big sweat and feel nauseated and tired It happen about every 12 to 17 days. I have had a heart monitor on and since the Dr didn't call me back I think it was OK. My blood pressure seemed to be OK and I had an egg and a blood sugar test and it was normal. The last spell I had was typical but since I have been a bit dizzy and a little nauseated. I can go through a normal day but feel like a have a hangover, to describe the feeling. I can't get into see my Dr for a week and a day. I am very frustrated and frankly wonder what's going on. Thank you for listening. Doctor: Hello,Thanks for the queryPlease note that there are many causes of dizziness which begins with heart problem, neurological problem, and ENT problem and so on. The possibilities ion your case could be migraine. But you need a detailed examination by a local physician to exclude other causes. If the investigations are normal, I feel that you may benefit with anti-migraine medications under supervision. Please get back if you need any additional informationBest wishes"
},
{
"id": 199917,
"tgt": "Suggest treatment for red bumps on penis head and itchy foreskin",
"src": "Patient: Hi i am 24 years old and uncircumcised married for 4 years and only her. We have unprotected sex all the time and the head of my penis was shiny clean not a scar or whatsoever. She recently had a yeast infection but we didn t know and the a few days after i started getting RED DOTS ALL OVER MY HEAD and when i pull back my foreskin it itches alot sometimes plz help as its been almost 2 months now.. Is that bad? Doctor: HiThank you for asking HCM. I have gone through your query. Your problem can be most likely due to yeast infection. As your wife had it you also should be treated. If you were my patient with such case then I would recommend using clotrimazole cream for local application and fluconazole tablet to take orally weekly once for 1 month. Hope this may help you. Let me know if anything not clear. Thanks."
},
{
"id": 47129,
"tgt": "What is the remedy for kidney stone?",
"src": "Patient: I AM SUFFERING FROM KIDNEY STONE PL. ADVISEI am suffering from kidney stone and its 4th time already I have orated 2 times by (PCNL & LITHOTRIPSY)but still stone formation is being in my right kidney. Please advise immediately. It will be very helpful for me and I assure you your helping hand will be always be remembered by me. Regards, MANISH YADAV 0000 Doctor: Hello and welcome to HCMI have read and understood your queryI hope this will be helpfulFrom history provided by you.. You are having recurrent renal stones...there are many causes of recurrent renal stones and includes:Recurrent UTIUrine outlet obstructionHigh excretion of calcium in urineHomocystinuriaHyper uricemia...Surgery is not the exact treatment for me because unless the risk factor is eliminated problem will continue...I will advise you to visit a UROLOGIST..and discuss with him for various investigations including:Urine RE e CULTURE SENSITIVITY24 HOUR URINARY CALIUM AND PHOSPHATESERUM CALCIUM AND URIC ACID LEVELSSERUM PTH LEVELS.And then proceed accordingly.. I hope this was helpfulRegardsDr faeza"
},
{
"id": 165881,
"tgt": "What makes my child fall several times a day?",
"src": "Patient: my daughter age 5 constantly falling several times a day for quite sometime. This was also picked up at a recent speech therapy session in which the speech therapist said it may be a good idea to refer her to occupational health team... Any ideas to move forward Pete from Manchester UK Doctor: Falls in 5years old child who started walking is needed to be attended immediately. Please consult nearest hospital to undergo a neurological and musculoskeletal examination first, to proceed further in the diagnosis.are you and your partner consanguineous? thank you"
},
{
"id": 147006,
"tgt": "Suggest medications for ankylosing spondilitis",
"src": "Patient: I have ankylosing spondilitis & rheumatoid artrithis, I have developed a pain at the base of my skull & pressure in my head, got ct scan & lumbar puncture on thurs in hos as pain was so bad my gp thought i might have a bleed in the brain, but no. Cld this pain be caused by the ankylosing? They gave me meds but not working, I am also just finished simpoini injections & now on Humira, but had to come off them with last mth as due surgery for ovarian cyst. Doctor: Madam, Ankylosing spondylitis and also Rheumatoid arthritis can lead to few conditions in the cranio cervical junction, atlanto axial instability being the commonest. The pathology being close to your cervical spinal cord can have serious repercussions if not investigated and treated early. If not done already, a CT scan of the cervical spine with dynamic x rays of neck of will help to pin point the exact cause. I would suggest you to consult a orthopaedic spine surgeon or a neurosurgeon. Hope this helps."
},
{
"id": 70482,
"tgt": "What is the cure for the painful puss filled lump at the centre of the chest?",
"src": "Patient: Hi Dr. I have a lump in the middle of my chest that is not going away. I ve had them before but this time, a lot of puss came out of the lump and it is now extremely painful (with and without touching). Some blood and clear fluid also came out of it. This is the first time this has happened. Should I be worried? Doctor: Hi. From the location of the disease , this looks to be an abscess gone into a chronic stage for want of proper antibiotic and treatment.The skin in the center of the chest can not be stretched and does not allow healing the way it occurs at other places.May be you need to see a Surgeon and may need a curettage for the disease to be cured. He will also prescribe you antibiotics and other medicines as required."
},
{
"id": 177227,
"tgt": "What causes blood in stool?",
"src": "Patient: My daughter just turned one, her poo is very snotty and sometimes has a little blood in it. Her poo was tested for any infections but they said nothing was wrong. I breastfed her until about 7 months then she was on soy formula, it seemed to be fine after that so i thought it was an allergy with my breastmilk. Now shes on whole milk and it started again. I told her doctor and he acted like its normal. I said should i switch her to soy milk and he said no. Ok so im clueless on whats going on. Please help. Doctor: HI...by what you quote there are two possibilities here - 1. Cows milk protein allergy - uncommon at this age.2. Acute bacillary dysentery - common at this age.My suggestion - Stop all cow's milk products and go lactose free in her diet and in 1 week you see any difference - that means it confirms the diagnosis of cow's milk protein allergy and you need to re-administer cow's milk products after 3 more months and see again.Regards - Dr. Sumanth"
},
{
"id": 109400,
"tgt": "What causes too much sleep and back ache?",
"src": "Patient: Sir, I had unsafe sex with My Girl friend and another day she had the unwanted pregnancy pill, and it s been like one week ago, and she misses her period this month and she is suffering back ache too much even she sleeps too much now. Please tell me what is this? Doctor: Dear caring boyfriend first of all avoid unsafe sex altogether. Your partner's back pain might be due to pelvic infection. She also missed her periods which may be due to pregnancy which is unwanted and she might be getting depressed, which might be the reason for excessive sleep. Take her for visit to expert gynecologist and get her checked for pregnancy test. I would also advice starting antibiotics once pregnancy ruled out. And above all counsel her and avoid unset sex in future."
},
{
"id": 135701,
"tgt": "Suggest treatment for muscle pain and infection due to spider bite",
"src": "Patient: I have been bitten by a spider while at the park.It itched for about a 1/2 hour.. It formed a pimple like bite and then 3 days later I was bed ridden.. all my muscles in my legs calf, thigh, and buttox felt like I had 100 lbs of weight on all my muscles and lower back... that lasted 3 days and I am feeling better .. The bite now is a black circle with pink around it and is sensitive to touch but not unbearable.. I thought I had the flu but then got to thinking this was more than that... any suggestions Doctor: hiThe spider sting is probably sticking at the bite site. show a doctor who may remove it.Anti-allergic, antibiotics and anti-inflammatory medicines are given to combat infection,allergy symptoms, and if need be cortisone is also added if too much toxicity is there. Your dangerous period is over now, but precautionary treatment may be taken if your docotr agreesbest wishes"
},
{
"id": 178771,
"tgt": "What does the fecal exam result with fat globule indicate?",
"src": "Patient: Hi! My baby is one year and seven months. I just would like to ask if fat globule in his fecal exam result says few, is it normal? The color is yellow, the consistency is soft. WBC, RBC, ova and parasite result is non seen. Yeast cells result is few. What does this indicates? Than you. Doctor: Thank you for posting your question.Fat globules in stool examination is a normal finding in stool. It is related to the fat content of the ingested food. This finding may rarely be considered significant in a child with a liver or pancreatic disease. But in the face of a healthy child, this finding can be ignored.Similarly few yeast cells in stool examination could also be an incidental finding with not much significance.Also, any laboratory report needs to be correlated clinically to decide whether it is significant. Thus if your child is healthy and not showing any bothersome symptoms, these changes in the report may be ignored."
},
{
"id": 858,
"tgt": "Are bromergon tablets and ikaclomin tablets effective for getting pregnant?",
"src": "Patient: Please i want to know if i can conceive taking bromergon tablets and ikaclomin tablets. I had a miscarriage 3months ago. my doctor advised that i use 3months to recover. He gave me vitamins E and C tablets, cod liver oil, ferrous sulphate and folic acid all these tablets i have taken for 3months before he recommended the fertility drugs i mentioned in my first paragraph. Are the fertility drugs the right one for me to take. please reply. Nma Doctor: Hi, I think your serum prolactin level might be higher that's why bromergon was given to you. It decreases the prolactin levels. High prolactin levels interfere with ovulation. Ikaclomin is a drug given for ovulation. It increases the chance of pregnancy. You can track your follicles growth by repeated ultrasound and when your follicles is more than 17 to 18 mm, take injection for rupturing the follicles. Be in contact with your husband for 2 to 3 days after injection. Take progesterone for next 2 weeks. Do a urine pregnancy test at home after that. You can try like that for 3 to 6 months. Hope I have answered your question. Regards Dr khushboo"
},
{
"id": 157420,
"tgt": "Had lung infection, on antibiotics, sinus infection, coughing up green phlegm. Could this be a type of cancer?",
"src": "Patient: I had a lung infection eight months ago, had been on antibiotics a sinus infection at the same time and had taken an antibiotic for that also . I am still coughing up green flem hugely . could this be a type of cancer ? I have been back to my doctor but have been told there is no infection . Doctor: Hi and welcome to HCM. Thanks for the query.Cancer is the last thing to think of. It sounds to me more like COPD or emphysema especially if you smoke. Also it can be some other lung disease but cancer is not usually manifested with these symptoms. You can also have some chronic bacterial inflamamtion. so I suggest to do lung ventilation test and more detailed microbiologic testing.Wish you good health. Regards"
},
{
"id": 140231,
"tgt": "Suggest medications for post concussive syndrome",
"src": "Patient: Hi my name is blaine. I have been sufferring from post concussive syndrome for about a year now. Symptoms just seem to not be gettin better. My brain feels like jello and its making my whole body sick. I was taking percacets not knowing that it could cause further damage and have noticed feeling worse because of it...will i ever recover because of taking narcotics. I have completely stopped any intake of harmful substances. Am i stuck like this forever? Doctor: Hello, You did the best thing stopping the drugs or other harmful substances. Recovery from opioids use takes some time (even months) but there should be an improvement. So, I think that you will feel better gradually. Hope I have answered your query. Let me know if I can assist you further. Regards, Dr. Erion Spaho, Neurologist, Surgical"
},
{
"id": 186713,
"tgt": "How to cure bumps at the back of the tongue?",
"src": "Patient: I have a few bumps, polyps(?) on the back of my tongue, out of site but within reach of my finger, and thus feel smaller than a pea. They are not sore, cause no pain, nor any issue other than the annoyance of feeling like I have feed stuck near the back of my mouth that I need to swallow. Any suggestions? Doctor: Hello, Thank you for consulting with HCM.These bumps which are present at the back of tongue can be normal taste buds only, as they are situated at the area where you have mentioned. And even they are not creating any problem.Better you should not worry about them.Hope it will help you."
},
{
"id": 91396,
"tgt": "Why the lower abdomen is heavy?",
"src": "Patient: I have been having dull and sharp pain in my upper right abdomen right under my rib cage but closer to the center. I had this pain before last month but after a few weeks it went away. I ve noticed that today my lower abdomen feels heavy. any suggestion? Doctor: HI. Any other associated symptoms like nausea , vomiting , fever, loose motion or so?This can be due to infection of the liver or gall bladder or colon on that side. You need a high resolution ultrasonography to know what is going on inside and to take antibiotic course. If need be as per reports of the USG , you may need surgery ."
},
{
"id": 57946,
"tgt": "Is Ursocol the permanent solution for fatty liver, and increased SGPT and SGOT levels?",
"src": "Patient: hello,doctor..my doughter is 16 years old. she is fatty liver . weight 64 kg,hight 155cm. current LFT report - sgpt 64 & sgot 73, total bilurubin 2.3 & others within in renge. 6 moths before LFT report - sgpt 90 & sgot 103, total bilurubin 2.3.she has no any illness. medicine continue URSOCOL 450.she is suffring from last 1 year.piyas.maityhow to cure completely..?? Doctor: HelloThanks for the query to H.C.M. Forum.Please note down the indication for which URSOCOL i.e. ursodeoxycholic acid is prescribed.1 Dissolution of cholesterol-rich gallstones .2 primary biliary cirrhosis .3 Prophylaxis of gallstones in patients undergoing rapid weight loss.In my opinion it will not resolve fatty liver .Get in M R I of liver and consult gastroenterologist and find out the exact reason of fatty liver and then proceed.Good luck.Dr. HET"
},
{
"id": 105679,
"tgt": "Persistent welts, constant throughout the week, itching, on arms, torso, neck , face, happens when skin is scratched or tickled. What type of allergy?",
"src": "Patient: My four year old daughter is having persistent welts that come and go frequently throughout the day. For the past week they have been almost constant. They appear suddenly and last for 15-30 minutes. they itch a little. They are on her arms, torso, neck and face. They seem to happen when her skin is scratched or tickled. I have taken her to an allergist . He said it is a problem where the histamines cause swelling at the site of contact that is irritating (such as after a tickling ). Should I take her back to the allergist? He said it is not a problem and she will hopefully grow out of it. Could it be a reaction to stress? heat? pollens? Is ithere an underlying problem I should be searching out? Doctor: Hello. Thanks for writing to us. From your description it seems that your daughter is having an allergic disorder related to some environmental factors. Getting an allergic test done might help in determining the causative agent. Meanwhile applying topical hydrocortisone ointment will help her. I hope this information has been both informative and helpful for you. Regards, Dr. Rakhi Tayal drrakhitayal@gmail.com"
},
{
"id": 24198,
"tgt": "What causes headaches with hyperkinetic movements?",
"src": "Patient: I'm 39 years of age last year I was diagnosed of hypertension when I could not sleep properly , since then I have been place on drug , tenoretic, dopatab , lotrial 5 etc , but in the recent times I have been experience slight headache once in a while, I will be experience loss of life on my legs and strange movements like electric shock , when I'm talking for long period the silivar in my mouth will be ticky , what is the cause and the solution Doctor: Hello!Welcome and thank you for asking on HCM!Regarding your concern, I would recommend performing some tests to investigate for the possible causes: - a resting ECG- a complete blood count for anemia- thyroid hormone levels to rule in/out possible thyroid dysfunction- kidney and liver function tests- blood electrolytes (including calcium, potassium, sodium and chloride to exclude possible imbalance)- a nerve conduction study to examine for possible nerve damage that may cause symptoms in your legsYou should discuss with your doctor on the above tests. Hope you will find this answer helpful!Kind regards, Dr. Iliri"
},
{
"id": 70711,
"tgt": "Suggest treatment for occlusion under the rib cage",
"src": "Patient: I recently saw a vascular surgeon re an abnormally low blood pressure and many times the clinician canot find it at all. This has been going on for about 10 years. After some testing the doctor said I have an occlusion under my rib cage on both sides and have had this for a long time. He did not recommend any treatment because I did not seem to complain about any symptoms (which surprised him)but I have trouble going up and down stairs and using my arms for any length of time ( thought this was because I do not strength train). I am 61 years old and am wondering if I had some type of surgery (not sure what that would entail) but would it help me in going up and down stairs and using my arms. If I was to have surgery what type is it? and how serious. I am very concerned and would like to be able to walk upstairs w/o my legs aching. Any advice or explanation as to what this is? Thank you Doctor: Hello, As you explain the history no I don't think you need surgery and especially if your doctor has not recommended it. Hope I have answered your query. Let me know if I can assist you further. Take care Regards, Dr Jnikolla, Pulmonologist"
},
{
"id": 65025,
"tgt": "Suggest remedy for lumps in head",
"src": "Patient: I fell and struck my head against a sharp corner (wooden ) about four and half hours ago.Now if I bend forward the orange sixed lump on the back of my head burns like fire .I have a mild behind the eyes headache but I have sinusutis which may be responsible for that.I lost my vision for a split secon at the time of the injury but never lost consciousness. When is it necessary if one is has no real deficits to see someone after a hard blow to their head? Doctor: Hi,dear thanks for the query .After indepth study of the query, I feel , there is mild concussion.Only if the -nausea increases/ or vomitting occurs /or the consciousness reduces/ then you may need to have the surgeons expert opinion.Hope you got the anwer ,if not, plz ask query again.Wellcome.Thanks again."
},
{
"id": 147088,
"tgt": "what causes central and obstructive sleep apnea?",
"src": "Patient: My husband has central and obstructive sleep apnea and uses a V-Pap machine. He still thrashes around at night (kicks and punches) and gets headaches and has episodes of confusion. He has had his heart checked...no congestive heart failure and his brain CAT scanned....no reason can be found for the apnea (idiopathic). Is there anything else that can help him? Doctor: His settings may not yet be optimized. You should communicate with the doctor responsible for ordering the settings so they can be readjusted. He may need another sleep study if it's been a while. Central sleep apnea results from some form of damage to the central control mechanism for sleep following stroke, traumatic brain injury, intracranial brain bleeds, infections, hydrocephalus, etc. Obstructive sleep apnea refers to anything within the physical airway that blocks the natural and unimpeded flow of air into or out of the system. Usually these can be easily found and corrected either by surgery, weight loss or other simple maneuvers. Fixing central controls, however, remains extremely challenging."
},
{
"id": 86008,
"tgt": "What does extreme pain on the left side of the lower abdomen indicate?",
"src": "Patient: I came down with extreme side pain this morning. Maybe like a torn muscle, but I wasn't doing any activity.The pain is very sharp and comes when I make certain movements. I have been in bed all day.Sitting is very difficult; I eat standing up instead. Don't know if it's a muscle injury or something else. Location is hard to describe, but it's left side, about belly-button level. And very deep pain, nothing at the surface. Doctor: Hi, As the amount of pain is substantial, acute intra abdomen condition has to be figured out at first instance as calculus in the urinary system, inflamed bowel segment or others. In our clinic, after primary control of pain with injection therapy, I request prescription of CT scan of abdomen and pelvis for final diagnosis. Hope I have answered your query. Let me know if I can assist you further."
},
{
"id": 32155,
"tgt": "Suggest medication for cervical lymph node swelling",
"src": "Patient: I am under Tbc treatment for about 8 months..during my 6th month i had cervical lymph node swelling and it was suspected to be a lymphoma..but after a sectional biopsy it was confirmed to be a healing Tbc in lymph node with absence of mycobacterium bacteria. Although the swelling was decreased considerably, it is showing an on and off symptom of increasing lymph node swelling at the same place.my lungs have almost proved to be clear in my last xray diagnosis. I am still under tbc treatment..does it mean that it is turning into a non resistant tbc..should i worry about it ? Doctor: HI, thanks for using healthcare magicLymph nodes are part of the immune system and respond to the presence of inflammation/infection in the area.It can take 4 to 6 weeks after an infection has resolved for a lymph node to decrease in size.This means that the node may decrease in size so you can monitor for now.I hope this helps"
},
{
"id": 198253,
"tgt": "Yellowish discharge from penis and itchy urethra is some kind of infection?",
"src": "Patient: Hi good day. I have a a concern regarding my penis. I noticed that their is a small yellowish discharge on my underwear and the part of urethra is itchy to touch. Sometimes i feel pain on the shaft. What is this kind of infection? hope you can help me..Thanks Doctor: HelloDischarge and itchy urethra may suggest UTI(urinary tract infection).It may be due to bacterial or fungal cause.You may need proper antibiotics depending upon culture and sensitivity report.You may need clinical correlation and investigations like routine hemogram,random blood sugar,Urine RE/ME,culture and sensitivity of urine and discharge material,ultrasound of KUB region.You may also need alkalizing syrup.Drink lot of water.You may need analgesics if you have pain.Get well soon.Take CareDr.Indu Bhushan"
},
{
"id": 196989,
"tgt": "What does my sperm analysis suggest?",
"src": "Patient: hi , my age is : 28, male , abstinence period 2 wks , volume 4.2 ml , ph 8.3 , consistency normal, color normal liquefaction time 25 minutes , sperm count 280.2 millions/ml , w.b.c 2 - 4 /hpf , r.b.c 4-6 /hpf (morphology of spermatozoa ) : normal form 50 % , pin head 15 % , round head 5 % , double head 5 %, large head 6 % , atypical form 10 % , amorphous head 7 % , double tail 2 % , motility study motile(active) 30 min 2 % , 1 hour 0 % 2 hour 0 % 4 hours 0 % sluggish 30 min 75 %, 1 hour 69 % , 2 hour 62 % , 4 hours 53 % immotile 30 min 23 %, 1 hour 31 % , 2 hour 38 % , 4 hours 47 % please help me and tell me if i have problems , knowing my abstinence period mor than 2 weeks thanks alot Doctor: your semen analysis is totally normal.. the count is above the normal limit which is 15mil /ml.. total ( active and sluggish) motility is above the normal.limit which is 40%.. normal morphology is above normal limit which is 4%"
},
{
"id": 204996,
"tgt": "What causes recurrent body tremors upon waking?",
"src": "Patient: Hello, I have an issue I have been having for over a year now. when I wake up (either in the morning or from a nap), I m very shaky. Not only that, but when I am getting ready in the morning I have tremors where my whole body jerks for a second and I kind of black out a bit. I m not sure if my eyes are closing or what exactly is happening. Sometimes they are just small tremors but other times they are larger. This usually continues throughout the morning for about 45 mins-1 hour, with the tremors getting less severe as it continues. It could be related to anxiety as I often feel very very anxious and nervous about school or work, but I have never been diagnosed with any anxiety disorder so I can t say for sure. Even during the rest of the day, my hands continue to shake even if I m not feeling nervous about anything. Doctor: haiEarly morning tremors could be due to dreams,Substance use withdrawal(eg.alcohol...),thyroid disorders,mere anxiety...ask your family members to videotape of what is actually happeningget a sleep EEG doneconsult a Neurologistif every thing is normal consult a Psychiatristthank you"
},
{
"id": 109405,
"tgt": "What causes lumbar disc displacement?",
"src": "Patient: I have a lumbar disc displacement im taking tramadol, naproxen, and lyrica... Will injections in my back help take the pressure off the nerve the displaced disc is putting on it and with the numbness,and tingling in my left foot. if i try to lift anything the pain is immediate. 3 disc are bulged 2mm 1 disc is bulged 6mm Doctor: To relieve pain initially local injection are not given Rather take intermittent rest(in occasion of extreme pain), paracetamol 1 gm twice or thrice daily, give hot comprssion, and have some postural modifications. If symptoms are controlled in this way nothing is required. We have to give injections if pain relief is not attained in this way."
},
{
"id": 10216,
"tgt": "What are the side effects of Renova?",
"src": "Patient: hi..i was having hairfall from a long time so went to a dermatologist in bangalore,she recommended me to use renovia.i have been using it for around 2 months now.just wanted to know that should i continue using it or it has any side effects if i use it forever? Doctor: Hello and Welcome to \u2018Ask A Doctor\u2019 service. I have reviewed your query and here is my advice.I think you mean to say Renocia hair serum. Renocia is a solution consisting of hair peptides which are believed to promote hair growth. It usually does not produce any side effects. You can use it continuously without any worry.Hope I have answered your query. Let me know if I can assist you further."
},
{
"id": 198959,
"tgt": "What could be the reason for having milky discharge from my penis all the time?",
"src": "Patient: What other problems could I possibly have if I have milky discharge from my penis all the time sometimes heavy sometimes light, also burning at the very end of urination when last of urine is coming out, but have been screened for all STD's and all came back negative? Doctor: DearWe understand your concernsI went through your details. If all the tests about STDs came negative, the answer to this question is simple. That milky discharge, in all possibility, is your semen itself. I would like to know, do you masturbate? or indulge in sex regularly? Your body produces semen whenever you are sexually aroused and it is stored inside your body If you do not indulge in masturbation or sex, your body finds its own way to throw the stored semen, because it is a waste product by then. Usually body throws it through urine or through nocturnal emissions. If you require more of my help in this aspect, please use this URL. http://goo.gl/aYW2pR. Make sure that you include every minute details possible. Hope this answers your query. Available for further clarifications.Good luck."
},
{
"id": 11834,
"tgt": "Armpits black, between thighs also black. Any cure ?",
"src": "Patient: hi sir.. good morning.. i am very disappointed with black in armpits and between thighs. how can i get white skin bet ween these parts. please give me your precious advice and prescription.. thanking you sir.. am eagerly waiting for your reply. my email id YYYY@YYYY and i am from india.. i hope u will give prescription which is available in india Doctor: Hello, Thanks for writing your query to us. Shaving causes hair to grow back thicker and darker, and this dark hair is often visible under transparent, light skin - which is what makes the skin look darker. Also, waxing removes dead skin, while shaving does not do the same. If you shave underarms regularly, you may want to scrub your underarm area with a pumice stone on a daily basis. This should help combat darkness to an extent. Apply lemon juice. Don't wear polyester clothing near those areas. Wash regularly, make sure these areas are dry by applying powder. Try Shower to Shower. Wax these areas regularly. If your inner thighs rub together this also causes it to go dark, so you may need to lose weight on your thighs. Stop using hair removal cream on your underarms and start waxing. Scrub your underarms daily with a pumice stone, after applying soap, while taking a shower. Take care."
},
{
"id": 206627,
"tgt": "Suggest treatment for triple x syndrome and depression",
"src": "Patient: I was born with a disorder called Triple X Syndrome Im a 30yrs old mum of two children. Thro my childhood I liked t play alone sometimes.hard t make friends. I also had 1to 1 thro during school primary and comp school, I m tall with long legs and have a small head my ears right n left was abnormal had sergay on both ears at 5yrs old. I ve got sum kinda depression suffer from panic attacks and anxiety. I suffer with language speach and poor learning skull, reading writing poor on my spelling. But like to know more about triple x and depression? Doctor: HiThanks for using healthcare magicIn depression, you need antidepressant treatment. These drugs would help you to come out of that phase. Better to consult a psychiatrist and get proper treatment. In case, you need further help, you can ask.Thanks"
},
{
"id": 150545,
"tgt": "Identified Plasma Sugar, blood supply to brain got reduced, weakness. What tablets should i prefer to overcome?",
"src": "Patient: Sir, Patient 36 yrs female. Identified Plasma Sugar-311 and the scanning report says that the blood supply to the brain got reduced.She is suffering from weakness. we consulted Physian and he suggested 1) Secrin 2)B-rich tablets. While using these for the past 2 days, she is suffering from dizziness and weakness. Now I called up the doctor and he suggested Glimisave-M2 1/2 tablet morning and Half in the evening. Doctor: hi either your patient is suffering from diabetic ketoacidosis or evolving stroke.....secrin and glimisave are hypoglycemics to decrease sugar...36 years means stroke is less common it has to be DKA due to diabetes in young.........glimisave or secrin causes decrease sugar and can cause weakness and dizziness.....she need to go to hospital .......for treatment of DKA take care"
},
{
"id": 173657,
"tgt": "Suggest treatment for cut on labia minora after an accident",
"src": "Patient: My daughter (10yrs) fell on the beam tonight at gymnastics and has a cut on her labia minora. It has stopped bleeding now but had produced a fair amount of blood into her leotard. It stings and she won t let me clean it yet. Any advice? The cut goes horizontally across one labia minora. It looks deep enough to scar. Doctor: Hi...she definitely needs to be seen by a pediatric surgeon or a plastic surgeon. This will require suturing too. It will be painless and will be done under local anesthesia. Ask her not to worry and get her seen by a pediatric surgeon or a plastic surgeon.Regards - Dr. Sumanth"
},
{
"id": 146526,
"tgt": "What causes numbness and tingling in hand?",
"src": "Patient: Woke up two nights to feeling like my left hand was on fire. Unable to fall back asleep it hurt so bad....took a bunch of ibuprofen and eventually fell back asleep. When I woke up in the morning my hand feels like it had fallen asleep and tingled for quit awhile before going away. What is causing this? Doctor: Hello dear,The symptoms as mentioned in your post can be attributed to:1. Paraspinal spasm & compression of nerve fibers at the Cervico-dorsal region or due to compression of nerve secondary to improper sleeping posture.2. Inadequate hydration status & electrolyte imbalance of the body3. Dietary deficiencies of certain nutrients like Iron, Calcium, Vitamin D, Vitamin B12, Folic acid, etc. which help in blood formation & nerve conduction.4. Peripheral neuropathy caused due to any fluctuation in blood pressure or blood sugar level.Symptomatic relief can be obtained by intake of nervigenic agents (like Vitamin B6, Vitamin B12, Folic acid) as well as multivitamin & multimineral supplements including iron, calcium & Vitamin D needs to be added in your diet.If symptoms still persist, kindly consult a Neurologist & get a complete Neurological examination done.Certain investigations like complete blood count, serum electrolyte estimation, fresh estimation of blood pressure & blood sugar levels & nerve conduction studies will be helpful to rule out any pathological cause for the symptoms.Wishing you a Good Health.Take care."
},
{
"id": 199258,
"tgt": "What causes cluster of bumps on penis?",
"src": "Patient: Small cluster of skin tone bumps on penis about 3-4 cm from the head...hard to the touch but painless unless rubbed against clothing while walking which gives a minor tingling or itching feeling...no redness or blister like bumps with any kind of color Doctor: HelloThanks for query .Based on what you have described I would say that you have what is called as Fordyces spots over shaft of the penis .The penile skin is rich in sebaceous glands and hence prone to get more sebaceous cyst due to accumulation of sebum beneath the skin. Normally they fade away without treatment however they need to be treated if increase in size or get infected. Dr.Patil."
},
{
"id": 31249,
"tgt": "Suggest treatment for hay fever and bilious",
"src": "Patient: i am feeling pretty rough with hayfever and billious too. have tried piriton but cannot use this,several treatments contain chemicals that are harfumful and do not want nasal sprays as they can give nose bleeds please can you advise am wondering.if boots beands would be best.. thank you, Doctor: Hello dear,Hay fever is most commonly due to allergy to pollens.Take the following adviceOral antihistaminic drug: tablet cetrizine 8 mg two times daySteroid nasal spray: fulticazone or beclomethsone when severe symtomsEye drop : azelastin/sodium cromoglycateStay indoor during treatmentAvoid cutting grassKeep window close during travellingTake shower and wash your hand after outingIf symptoms do not disapperTake subcutaneous Immunotherapy at near centre.Hope this may help you.All the bestDr. Sagar"
},
{
"id": 27836,
"tgt": "Suggest treatment for rapid heart rate and high pulse rate",
"src": "Patient: Hello doctor. I am 28 years old. Have a son 7 years old. I started having a constant heart beat of between 112 and 117 beats per minute. At first i thought i was unfit. I bought a trojan exercise bicycle and tried to ride that. My pulse then goes up to 163. I then stopped because i dont know if its dangerous.. should i seek medical help? Or maybe just by Cardio care disprins? Please help? Doctor: Check ur thyroid hormones and get an ECG. Do u feel irregular pulse/heart beat sometimes ? Do u have palpitations? Blood pressure? Revert back & also get examined by physician ."
},
{
"id": 201514,
"tgt": "What causes white and skin colored bumps under the foreskin of the penis?",
"src": "Patient: I m 17 years old, I live in Jamaica and I had unprotected sex some time earlier this week.......this morning while I was urinating I notice some white and some skin colored bumps under the foreskin of my penis(I am uncircumcised) I ve been looking up\u00a0diseases from morning but non of the symptoms match.........its not herpes because there aren t sores and the bumps don t hurt.......so I d like to know if I have anything serious before I start making a fuss and it turns out to be nothing Doctor: You need not make fuss about it. Consult a dermatologist and get diagnosed and treated. It is meaningless to treat you by giving drugs blindly.Also keep in mind, unprotected sex is always dangerous, and should be avoided.TC"
},
{
"id": 107418,
"tgt": "Suggest treatment for neck and back pain",
"src": "Patient: My adult son has had neck and back pain in increasing severity; He keeps getting sent to therapists and pain management clinics with no help; he move to Missouri from LA and feels no one wants to help him in Missouri. Even though he had some records sent from a Dr. in LA showing that he does have a 1mm shift in one of his discs, the doctors in Missouri say they can t see it on their MRI s. He is tired and frustrated and about ready to give up. He is 27, married for a year and trying to go back to school, but it is difficult for him Doctor: Hello, regarding the situation of your son i suggest him to have a X ray to see if there is scoliosis which may cause this pain. after that, he should have some physical therapy for his neck and back, to strengthen the muscles and to correct his posture as possible.Best,Anisa"
},
{
"id": 177853,
"tgt": "Is swimming safe while child has cough, cold and wheezy chest?",
"src": "Patient: I have a (recently turned)2 year old who have a tendency to suffer from coughs and colds and wheezy chests. She has got anther cold, has been coughing, is full of mucus and is clearly not 100%. We have recently started swimming lessons (in quite a cold pool) and are due to go this morning. Not sure if I should take her or not? She wants to go! Doctor: Hi,It is better not to go for swimming lessons for few days till she becomes alright.There are all chances of having more infection.There are chances of spreading infection to others who are swimming as well.Ok and take care."
},
{
"id": 74077,
"tgt": "Suggest treatmemt for emphysema",
"src": "Patient: hi,my fiance has just been diagnosed with emphasima,he 44..smokes heavily,also they found a 5mm lump in his lung,hes to go for yet another ct scan..he wont stop smoking,and is very breathless..a year ago he was at the gym 5 times a weeknow he cant walk up stairs without bein out of breath..is 44 very young to have this disease.thanx Doctor: Thanks for your question on Healthcare Magic. I can understand your concern. Yes, emphysema can be seen at the age of 44 years. Since he is heavy smoker, emphysema is common at young age. He should get done PFT (Pulmonary Function Test) first. PFT is must totally know the severity of emphysema. And treatment is based on severity only. He may need inhaled bronchodilators (formoterol or salmeterol) and inhaled corticosteroid (ICS) (budesonide or fluticasone). Enroll him in smoking cessation clinic. Hope I have solved your query. I will be happy to help you further. Wishing good health to your fiance. Thanks."
},
{
"id": 9966,
"tgt": "Is keraglo forte helpful to stop hair loss and graying?",
"src": "Patient: hi i am a patient of ulcerative colitis and has suffered a relapse so was on steorids due to which i have terrible hair loss somebody has precribed keraglo forte how effective is the tablet to control hair loss and stop greying of hair?i am 33years old Doctor: Hello, This tablet is effective in controlling hair loss but needs to be continued for at least 3-4 months to see any significant results. Hope I have answered your query. Let me know if I can assist you further. Take care Regards, Dr Asmeet Kaur Sawhney, Dermatologist"
},
{
"id": 96130,
"tgt": "I have been very bloated abdominal. Is there any Online doctor for my symptom",
"src": "Patient: Hi, I am a 30 yr old female who has been very bloated in my lower abdomen & I get some tingling feelings in that area as well. About 6 months ago I went to the doctor with these symptoms as well as at the time a VERY heavy menstrual cycle which would last 8-10 days every 2 weeks, and weight gain. My doctor had me get a ultrasound to rule out cyst & fibrods which turned out to be negative & also did some blood work looking at my iron, WBC & thyroid. All these test showed to be fine. She concluded that it was just my hormones & put me on a birthcontrol pill (Loestroine). I have been taking these pills & eating healthy & exercising. While my menstrual cycle has gotten under control last month I had extreme pain with my menstrual & very heavy bleeding . I have noticed the past few days that my bloating is more noticeable, & I have also gained 4 pounds in the past 1 1/2 weeks. I have taken a home pregnancy test & that was negative. What could be going on with me ? I feel like I m pregnant but I know that I m not because of the test. Doctor: hello and welcome to healthcare magic. since you have already got a home pregnancy test done, you could repeat it or get a serum Beta Hcg levels checked as well as get an ultrasound scan done which will confirm about if you are pregnant or not and end your suspicion also. the bloating could be due to indigestion also, consult your gynaecologist. take care."
},
{
"id": 148054,
"tgt": "What is the reason for stiffening fingers and pulsating eyelid?",
"src": "Patient: hi, i am experiencing stiffening of my fingers, at first i ignored it but i noticed it happens more often now. Most of it at night. And the lid of my right eye is often pulsating, it has been more than a week now. What could be the cause of my concerns? Doctor: Hi,Thank you for posting your query.First of all, I would like to reassure you that there is no need to worry about your current symptoms.They are most likely due to over-strain of muscles and fatigue. It would improve with rest.However, if you do not get better in a few more days, then, a brain scan should be done to exclude any ischemia (lack of blood flow to the brain). Also, thyroid profile and vitamin B12 level should be checked.I hope my answer helps. Please get back if you have any follow up queries or if you require any additional information.Wishing you good health,Dr Sudhir Kumar MD (Internal Medicine), DM (Neurology)Senior Consultant NeurologistApollo Hospitals, Hyderabad, IndiaClick on this link to ask me a DIRECT QUERY: http://bit.ly/Dr-Sudhir-kumarMy BLOG: http://bestneurodoctor.blogspot.in"
},
{
"id": 116311,
"tgt": "What causes low WBC count after tooth extraction?",
"src": "Patient: In Dec I had a tooth pulled, 2 1/2 weeks ago I had prep done for a bridge. I have had a sore where the novicain was injected. A week ago doctor called saying my blood test came back with: WBC at 2.9 (range 3.6-9.1) and lymphocytes at 46 (range 24-44). Is there a possible connection? Thank you. Doctor: Welcome to H.C.M. I am Dr Krishna Dubey.My dear firend , there are not any connection with tooth extraction.So no need to worry. It could be due to some viral infection because lymphocytes are also slightly raised.You should repeat your complete blood count with peripheral blood film.This will very helpful to rule out any further query.Thanx."
},
{
"id": 122141,
"tgt": "What causes pain in my hips?",
"src": "Patient: First time ive done this. After about 6 months of kick boxing i started to eperience a pain in my hips or pelvis. Feels like hip flexors. When i try to do side splits it is very soar on my hips. Feels like bone pain but it must be muscle. Also when doing these side splits my body twists to the left, like im throwing a right roundhouse, which leads me to believe ive overtrained my right round house kick compared to my left Roundhouse kick. Doctor: Hello!Welcome to Ask a Doctor service!Your symptoms are suggestive of a stretched muscle. Anyway, I would recommend consulting with your attending physician for a physical exam and performing a lumbar column X ray study in order to investigate for a possible bulging intervertebral disc, which could lead to a pinched nerve (the femoral nerve). Hope to have been helpful!Kind regards, Dr. Ilir Sharka, Cardiologist"
},
{
"id": 168759,
"tgt": "What causes low temperature at night after fever?",
"src": "Patient: my son is 7month and he has had a fever for days up to 101.2 but now his okay but concern about his body temperature taken on his armpit. Temp is between 96-97 during the day and drops at during the night to 94-96. Im concern do i need to take him to the doctor? He s all bundle up at night but his temperture is still low. Doctor: Hi...Thank you for consulting in Health Care magic.Fever of few days without any localizing signs could as well a viral illness. Usually rather than fever, what is more important is the activity of the child, in between 2 fever episodes on the same day. If the kid is active and playing around when there is no fever, it is probably viral illness and it doesn't require antibiotics at all. Once viral fever comes it will there for 4-7 days. So do not worry about duration if the kid is active.Regards - Dr. Sumanth"
},
{
"id": 183439,
"tgt": "What causes toothache after undergoing crown lengthening?",
"src": "Patient: Hi Dr, in dec I did a surgery for crown lengthening and now they did a cutting on my teeth. Since then I have this terrible pains and i thought it was an infection but they told me its not but they did not tell me the cause. Can you please tell me why i have this pains that painkillers cant help.I have flourosis. Doctor: hellooo...read thru ur query...in my opinion i must say that thepain ur feeling is due to exposure of dentin layer of tooth(dentin sensitivity).it happens on cutting the tooth without undergoing rootcanal treatment..exposure of the root during lengthening can cause this sensitivity...but nothing to be worried..after the completion of the procedure it will be reduced...nothing to be worried in this..u can use sensitivity toothpaste....till the procedure gets over...hope ur benefitted something from this reply..have a healthy day!!!"
},
{
"id": 61189,
"tgt": "What does a lump on the toe indicate?",
"src": "Patient: I had a knot on my 2nd toe at the joint below the nail. It looked like it had a center with a round circle on top of the knot. I mashed on it and a clear sac came out filled with clear fluid in it. A round little sac with clear fluid and left a hole in the knot Doctor: Hello dearWarm welcome to Healthcaremagic.comI have evaluated your query in details .* This seems in relation with bursa cyst or sebaceous cyst .* Needs to be confirmed with physical examination or at least a photo pic evaluation as an attachment to give more precise judgement .Wishing you fine recovery .Feel free to ask any further doubts .Regards ."
},
{
"id": 98095,
"tgt": "Has high SGPT. On polybion-L. Homeopathy a better option?",
"src": "Patient: Hello Sir, my son of 16 years had dignosed with SGPT140 on 19th Feb after that we have conducted 2 more blood test sgpt 103 and now 123. I am giving him Polybion-L syrup as per my docotor advice . I am not giving oily and ghee mixed food , my son just take food which is adviced by docotor . What other medicine i can give because he had Gujarat Board exam and he can npt study for long duration. Can i give him Homeopathice medicine. ADRISH RAVAL Doctor: 1. for what purpose the liver function tests were performed?? 2. SGPT alongwith SGOT are present within the liver cells and their presence/release in blood suggests liver damage (irrespective of the cause) 3. moreover SGPT is present in heart cells also,look for any symptoms related to CVS 4. taking anti depressants/antibiotics can also raise SGPT level. 5. weight and lipid profile of your son?"
},
{
"id": 71395,
"tgt": "Suggest remedy for fluid build up in the lungs",
"src": "Patient: My brother had a procedure done to remove fluid from his pericardium back in December. He recently had a another procedure to remove some of the sack and the doctors found mass around his heart and found a tear in one of his arteries which they stitched. He was released last week now he is back in the hospital with fluid in his left lung and the doctors don't know why the fluid is building up and don't know if it's from his heart. Why can't the doctors figure out what's going on? It's been almost 3 days. Doctor: Hello and Welcome to \u2018Ask A Doctor\u2019 service.I have reviewed your query and here is my advice.The possible reason for fluid buildup in lungs is infection in lung or reactionary action to the pericardial procedure.In our clinic, we suggest to take CT scan thorax to evaluate it further precisely and manage perfectly.Hope I have answered your query. Let me know if I can assist you further.Regards,Dr. Bhagyesh V. Patel"
},
{
"id": 163985,
"tgt": "Why does a child become anemic and is regimen of iron treatment normal?",
"src": "Patient: My grandaughter was taken to the doctor beacause she complains about leg pains. They did a blood test and said she was anemic. She was prescribed a regimen of iron for a month and is to return after that. Why does a child become anemic and is this treatment normal? Doctor: Hello. Thanks for asking on healthcaremagic. Childhood is a continuous growing phase. During these phases we need increased nutrients. However these days, children are picky eaters who might not eat a well balanced diet. Hence it is important to give supplements. Anaemia is commonly seen in growing children. Anaemia can be due to iron, folate or vitamin B12 deficiency. It is important to see from a blood picture whether iron, folate or vitamin B12 ia to be given. In case hb does not increase even after such a therapy, then underlying causes like malabsorption, haemolysis, occult blood loss, aplasia or any minor thalassemia trait will be ruled out. Thanks."
},
{
"id": 62799,
"tgt": "Suggest cure for a knot under the armpit",
"src": "Patient: \"i have a concern about 3 months ago i got a knot under my armpit and went to the doctor and he told me it was a boil.... and about a week ago my husband was n the hospital for the same knot under his armpit and this am i woke up with another one... when it poops its purple and has white stuff come in out of it Doctor: hi.it is best if you consult with a doctor, preferably a general surgeon, for medical and physical examination. based from your description, it could be hidradenitis suppurativa. these lesions usually occur on the underarm, especially after shaving or plucking and/or usage of deodorants with strong chemical contents. hair follicles get infected. inflammation and subsequent abscess formation occur. medical (proper antibiotic regimen) and surgical (excision or incision and drainage) management will be directed accordingly.hope this helps.good day!!~dr.kaye"
},
{
"id": 67716,
"tgt": "What causes a lump in the groin area?",
"src": "Patient: just found out I have a lump on my groin area . Its the size of a walnut and this is the first time I noticed by taking a bath . I will be going to the doctor but could it be a herena . I am 68 years old and scared to find out if it is CANCER. just need a peace of mind thank you Doctor: Hi,From history it seems that you might be having direct inguinal hernia.At this age due to lax abdominal and pelvic floor muscle one gets hernia.There might be possibility of having enlarged inguinal lymph node as well./Nothing to worry about cancer.Consult surgeon and get examined.Ok and take care."
},
{
"id": 209096,
"tgt": "Can i start taking 'zoloft' after a gap of few days?",
"src": "Patient: My boyfriend (25 years old) was prescribed Zoloft about two years ago after a tragic accident. About 2 weeks ago he decided to stop taking the med without consulting his MD (even thought he knew he was not suppose to). For the last week he has been lightheaded, constant headaches and nausea. After he told me today about his act of cutting cold turkey, I'm thinking these are side effects from stopping the med. He is wondering if he should take one tonight to stop the issue. However, I'm wondering after two weeks if it would be a good decision to begin the med all of a sudden again. What is your suggestion? Doctor: Hello, It was wrong on his part to stop the drug on.his own; but he can start it again. You have not mentioned the dosage he was taking, if it is more than 25mg dialy, it will be good that he takes 25mg on day 1&3--50mg on day 3,4&5 ---75mg there after..increase of 25mg is recomended each few days apart till he reaches the past efective dose.Manytimes patient can settle on lower dosage after taking it for long time and keeping well on it. You can talk to your doctor about reducing the dosage [instead of stopping it completely]. Wish you good luck.do make him see his doctor in near future to discuss the pending issues.Dr. Manisha GopalMD Neuropsychiatry"
},
{
"id": 128967,
"tgt": "What causes sciatica pain radiating from the right buttock towards the ankle?",
"src": "Patient: I m quite sure I have Sciatica, pain in the right buttock and down to the ankles. I have had this for 1 -1/2 been to PT & Chiropractor - do not feel any better than before. I am in a lot of pain today, at work and walking and standing. Should I have an MRI to see if it could be my Hip???? thx Doctor: Hello,Thank you for using healthcare magic.I read your question and understood your concern.I think you should do MRI of the lower spine , not of the hip. The problem is in the nerves that come out of the spine so you should check that part at the neurology specialist.I wish you quick recovery.Dr. Selmani"
},
{
"id": 185686,
"tgt": "What causes wisdom tooth movement causing pain and stiffness?",
"src": "Patient: My wisdom teeth are coming down on the left side of my mouth and it s making my jaw ache. I also have a stiff neck and mild pain at the back of my head near my neck, and at the temples. Is this all connected? I m currently abroad and would like to wait a month when i m home to get my tooth extracted. Is it ok to wait or could it do permanent damage to my jaw? Doctor: Hello,You may be experiencing a tooth erupting and some discomfort can be expected. Is this the first time you are noticing the teeth? Rinse the area, keep it as clean as possible and take some pain relief, anti-inflammatory medication.Do you think you have been grinding your teeth? This can be a reaction that causes some stiffness and discomfort similar to your symptoms. With tooth movement, this can change your bite and cause grinding.If the teeth have been present and are starting to move, then you must be careful and evaluate the situation for an infection. Are the teeth dark with decay? Is the gum covering part of the tooth and creating a pocket where something got caught? You can develop infections around the tooth (periodontal) or inside the tooth affecting the nerve. Spread of infection is dangerous and I do not recommend that you wait. Simple treatment and medication such as an antibiotic may control the situation untl you arrive home, but would require you seek professional help now. My advice is to see a dentist or oral surgeon specialist and explain you would like to delay treatment such as an extraction until you arrive home.Please answer some of my questions to help with your decision. I am glad to assist you with any additional questions and hope you will feel better soon."
},
{
"id": 65453,
"tgt": "Suggest remedies for hard red lump in the leg after injury",
"src": "Patient: My daughter got hit on her leg during a soccer game and it has a big red bump and it is hard and warm. it has been that way for about a month, it was purple and then it turned yellow, know it is red and hard in that area with a big bump. We have iced it and put warm clothes on there no change. Doctor: Hellohard swelling following a trauma could be due it cellulitis or inflammation of the local tissueif you can upload us a photograph i will.be able to guide you better in my opinion you should take her to general surgeon for examination and an X ray scanHope this answers your questionwaiting for your replyregards"
},
{
"id": 30512,
"tgt": "Suggest treatment for TB",
"src": "Patient: Sir, My brother 58yrs suffering from tb and he was a heavy smoker.His weight is48kg. From last two months he is ill. Initially he was undergone treatment of malaria after test positive. Thereafter,found tb. He was taking Rcenex and ethambulal but no improvement.He refered to another doctor who prescribed him Brophyle100,BRM100,SSC-4,Carmax.After taking the madicine two days he is feeling dissiness and vomiting.Where should I go for treatment.I am from Assam. Doctor: hi sir, welcome to HCG, i understand ur problem, using tb medicines causing nausea, vomit, abdominal discomfort and ur brother is weak person, about treatment, go to near primary health center, they will provide u free medicine, but u have to take good food for strength, hope u satisfy with my answer, thank u"
},
{
"id": 164289,
"tgt": "Does giving Isomil baby formula causes constipation?",
"src": "Patient: hi sir my baby is now 2 months plus now . had lactose intolerance and now using isomil soy form. she gets redness after every vaccnination . is this related to injection or isomil formula. will isomil formula cause constipation problems or can i continue this for six months Doctor: hi, welcome 2 this forum. The problem of constipation is not related to the formula milk that you are giving. Child may sometimes pass stools in 3 to 4 days, this is a normal thing. Take care."
},
{
"id": 53873,
"tgt": "What are the effects after gall bladder removal?",
"src": "Patient: I have dull pain in my left ear (inner and outer). I have acidity reflux problem. My gall bladder has been removed. Some fluid fell from my nose to throat resulting in accumulation of cough like thin in my throat. For this, sometimes I gargle with Tantrum. Two years back when I visited my doctor for this problem, he advised me to take zinetac, vozet and local application of neosporin-H. Due to some non-specific changes reflected in my ECG, Cardiologist has prescribed me to take Metolar-25, Pantodac, Storvas-10 and Ecosporin-75. Doctor: hi.thanks for posting query at HCM.read your question thoroughly and understood your concern.gall bladder primary role is to store \"bile\".when gall bladder is intact, bile is usually secreted after having a meal rich in fat; bile plays a vital role in fat digestion.after gall bladder is removed, bile is continuously being secreted in small intestine and may reflux into stomach. as a result, gastric inflammation may occur causing pain and discomfort.moreover, high fat meal induces more bile secretion further affecting gastric mucosa.advise:NO or minimum fat intake/oily foodNO junk foodNO beveragesingest small mealsOTC proton pump inhibitor (40mg omeprazole, pantoprazole)to combat gatritis ( after discussing with treating physician)hope to answer your concern wish you good and sound health.regardsDr Tayyab Malik"
},
{
"id": 92818,
"tgt": "Stomach pain, nausea, more at night. What could be this?",
"src": "Patient: Hi last night I had bad stomach pains. I was also shaking. I started to have stomach pains on Friday night. Now today it still hurts a little bit. We went to walgreens and asked them what it was. They said it was bad constapation. I do have a history of this. But the pain was very painful. I also had nausea and it hurts more at night what could this be Doctor: hi i had gone through your query and understand your concerns. i would also come up with the possibility of CONSTIPATION is the cause for your sufferings.Constipation;-\u00a0\u00a0\u00a0\u00a0\u00a0Always results in a build-up of dangerous toxins and danger of AUTO INTOXICATION, which is acknowledged to be the main cause of many of today\u2019s degenerative and life-threatening diseases .may result in a constant state of fatigue, sickness ,and lethargy.-\u00a0\u00a0\u00a0\u00a0\u00a0May result in;1. COLITIS;inflammation arising from the decomposition of blocked faecal waste.it may be the cause for your pain2.SPASTIC BOWEL; if the colon muscle is over worked and tense trying to push through faecal matter it will go into a spasm-a chronic tightening of the fibres of the muscles.it may be the cause for your pain3. BALLOONING; when faeces accumulate and stretch the bowel walls into enormous balloon shapes.it may be the cause for your paintreatment plan; i advise you taking Homeopathic medicines gives safe and permanent cure for your complaints consult your local homeopathic physician for correct diagnosis of the case and remedy to fit your complaints.I hope this is helpful for you, thank you"
},
{
"id": 147638,
"tgt": "What causes numbness and tingling across head?",
"src": "Patient: i have suffered from migraines for years and have had some numbness/tingling in a band across the top of my head with the feeling of pressure behind it during the attacks. Recently, I have been having these symptoms without the migraine with it or continuing after the migrain is gone. Any idea what the cause could be. I am not on any medications. Doctor: Hi,Thank you for posting your query.I have noted your symptoms. It is common for sensory symptoms to occur after an episode of migraine.In migraine, there may be fluctuation of blood flow to certain parts of brain, which can lead to band-like sensations after the headache.However, if these are occurring at an increased frequency, then, it may be better to consult a neurologist and get brain MRI scan done.I hope my reply has helped you.I would be pleased to answer, if you have any follow up queries or if you require any further information.\u00a0\u00a0\u00a0\u00a0\u00a0Best wishes,Dr Sudhir Kumar MD (Internal Medicine), DM (Neurology)Senior Consultant NeurologistApollo Hospitals, Hyderabad,For DIRECT QUERY to me: http://bit.ly/Dr-Sudhir-kumar My blog: http://bestneurodoctor.blogspot.com/"
},
{
"id": 77415,
"tgt": "What causes pain in the chest during cough or breathing?",
"src": "Patient: I have been having this pain in my chest right under my right breast.It is a pain that just stays and does not go away...It hurts to cough,,,Laugh,,,Reach or even breath...I did not take a blow to my chest so it makes me wonder if it is a broke rib, how did i brake it? Doctor: Hi thanks for asking question.Here are few possibilities in your case.1.First according to history you have right side pain under breast so it might be by costochondritis like inflammatory cause.In that case ribs are tender to touch.Analgesic or NSAIDs like antiinflammatory drug useful.2.Second your ribs might have osteoporosis like changes by vitamin D or calcium deficiency.So get their level measured in serum and treated accordingly.As you have said secondary to these rib fracture might present, x ray useful for it.3.This could be simple muscloskeletal pain by improper sleep posture or unaccoustmed situation or heavy weight lifting.4.If you have common cold, sore throat then this could be as a part of viral infection.But if fever also present then pneumonia like infection has to be ruled out by chest x ray.Complete blood examination also helpful.5.If along with chest pain breast mass palpable anywhere in breast or redness over any area of breast then breast infection or inflammation might lead pain.So this breast related possibilty also investigated after rulling out above problems.I hope my suggestion will be helpful to you.Dr.Parth"
},
{
"id": 166424,
"tgt": "How can buttock soreness with redness be treated?",
"src": "Patient: Hi, may I answer your health queries right now ? Please type your query here... my little girl has a sore bottom its very red it really bugs her at night time theres no sign of worms ...i think she been ruff when wipeing her bum. We have applied sudocream but no luck shes five Doctor: Dear parent, you should combine an antifungal cream together with sudocream 3 times daily for one week . you should complete using both creams for whole seven days even if the redness and soreness resolved to avoid recurrence"
},
{
"id": 149388,
"tgt": "Limbic system not functioning correctly. No feelings. Diagnosed with Fronto Temporal Dementia. Dad had Alzheimer. Related?",
"src": "Patient: I am wondering why my limbic system does not function correctly. I seem to have no feelings and don't know what happened to them. I do not feel anything at weddings, funerals, sad movies, fear,hurt feelings, sorrow for others, etc. I feel empty and dead inside because I do not exhibit emotions. I have recently been diagnosed with early fronto temporal dementia and wonder if this could have anything to do with it. I am 69 years old and my father had Alzheimers before he died. Is this familial or hereditary? Doctor: ya dementia may present in this manner.some pts having vascular dementia benefit with blood thinners and exerciseMRI angio of brain may be done to look for vascular insufficiency and treated"
},
{
"id": 13609,
"tgt": "Could Propofol be causing full body systemic rash?",
"src": "Patient: I had a bad reaction to propophol which was used during a colonoscopy. I have heard that there is a known side effect of severe skin reactions to propophol in children with egg allergy. I am not allergic to eggs (nor am I a child!) but have been dealing with this full-body, systemic rash since late July and have been working with my dermatologist to clear it up. Have you familiarity with anything like this? Doctor: Hi, I have gone through your complaints and Propofol can cause an allergic skin rash. I would recommend you to take antihistaminic like tablet Cetrizine twice daily and take apply cortisone cream on the affected areas twice daily for few days. Hope I have answered your query. Let me know if I can assist you further."
},
{
"id": 181529,
"tgt": "What causes abscess on the molar gum?",
"src": "Patient: I have a boil on my gum right beside my 12 year molar. It is on the outside part of the jaw. It has not subsided or gone down. some days it even hurts. I have rinsed my mouth with salt water, lain on a heating pad and taken motrin. I have also used oragel. this thing won't go away. what should I do next? Doctor: Hi.Welcome to HEALTHCARE MAGIC..I have gone through your query and can understand your concern..As per your complain a boil over the gum can be due to a number of causes like a gum abscess or a periodontal abscess secondary to gum infection..In case if tooth is decayed it can also be a dental abscess secondary to pus formation below the root tips of the tooth..I would suggest you to consult an Oral Physician and get evaluated and he can examine clinically as well as take an x ray if required..In case of gum abscess scaling followed by gum paint application and taking a course of antibiotics and painkillers will help..In case of dental abscess either root canal treatment or extraction of tooth followed by antibiotics and painkillers will help..Hope this information helps..Thanks and regards.Dr.Honey Nandwani Arora."
},
{
"id": 59891,
"tgt": "Abdominal pain, severe condition. Ultrasound shows echogenic sludge in gall bladder. Treatment?",
"src": "Patient: Sir, I have to much abdomen pain once in a year, from ultra sound result it suggest that echogenic sludge seen in gall bladder lumen-suggestive of micro calculi.please help me it will treat surgical or non surgical. If it is non surgical treatment please what medicine should i take. I am worried about this. thank you. Doctor: hello, welcome to hcm thanks for the query if stones are not symptomatic then surgery can be postponed.if you are getting abdominal pain more frequently then go for an elective surgery medication are available like ursodeoxycholate are helpful.but it take longer period for the stone to dissolve make dietary modification and consult your doctor for best possible treatment hope this information will be helpful to you take care"
},
{
"id": 18030,
"tgt": "Does carotid artery blockage cause dizziness?",
"src": "Patient: If my carotid artery is blocked, would it cause dizziness when I bend over and come back up? I had testing done with lifeline November 2016 and it showed a mild to moderate blockage in the carotid artery. I started about 4 months ago having dizziness when I bend over to pick up something, empty the dishwasher, etc. It is not every day, but some days it is all day and I have to stop and just lie down. It only occurs when I bend over. It has been recommended that I have a heart catheterization but I would like to have my carotid artery checked out first. Could there be a causal relationship between a partially blocked carotid and this type of dizziness. (The room does not spin, it is lightheadedness following bending over.) Doctor: Hi, The cause of dizziness is likely the vestibular system. Carotid artery blockage may cause dizziness because carotid arteries are important to supply oxygen to the brain. If the occlusion or stenosis is severe (more than 70%) it may cause dizziness or vision problems. Up to 60% occlusions usually do not cause any symptoms. But dizziness while bending is more suggestive of the vestibular system (benign paroxysmal positional vertigo) rather than carotid artery stenosis. You should contact your ENT doctor to check for vestibular causes. Hope I have answered your query. Let me know if I can assist you further."
},
{
"id": 35840,
"tgt": "Suggest remedy for staph infection in the montgomery gland",
"src": "Patient: the montgomery gland was taken off lifting grandchild. now it has infection leakage have had mammogram and ultrasound and consulted oncologist biopsy was not necessary and now scab comes off and does not completely heal the gland was swabed and was told it was a staph infection. worried all the time Doctor: Hi, dearI have gone through your question. I can understand your concern. You may have staph infection. No need to worry. Just take antibiotics effective against staph. If you don't get response then go for culture and sensitivity test and tale appropriate antibiotics. You will get benefits. Hope I have answered your question, if you have doubt then I will be happy to answer. Thanks for using health care magic. Wish you a very good health."
},
{
"id": 115079,
"tgt": "Will body's immune system fight against the semen in body?",
"src": "Patient: Hey there!I am a gay, me and my mate have a daily penetration, he cums inside me...my question is, if i have hommeroids and his sperms goes inside my blood, will my immunity system consider semen as a intruder and fight it then,if yes, what about the semen which is in my body(my testicles), will be fought as well?thank you Doctor: Hi! semen is nothing but a protein-rich secretion that must generate immunity in your body if somehow inoculated; actually it is not against semen as a whole but against some protein components of it! the sperms die soon and only digested/residual proteins will initiate some allergic/immunities in your body!there is no communication of blood with semen due to blood-testes barrier and not to worry!Hope this answers your question. If you have additional questions or follow up questions then please do not hesitate in writing to us. I will be happy to answer your questions. all the best!"
},
{
"id": 113707,
"tgt": "Lower back pain, left arm numbness. Taking pan-d, mayopaz. Treatment?",
"src": "Patient: i am suffering from low back pain on the left side near back theigh. i have taken pan-d and mayopaz with vitamin B tablets for 4 days 1 tb each a day. i can stand, move and if i take rest on a not so soft bed, i can even sit with folded legs for more than 2 hours. but i have stopped the drug as i had some relief from the severe pain. but now my left arm is showing numbness and still have a low back pain on the left side. Doctor: It is advised for MRI scan of LS spine .time being you can wear lumbo sacral belt and an take tab tramadol one tab only after dinner for a week."
},
{
"id": 93657,
"tgt": "White blood count low. History of miscarriage, DNC , bacterial infection in uterus. Causes of abdominal pain, weight gain?",
"src": "Patient: Drs said my white blood cell count is very low and I've had abdominal pain and rapid weight gain. What could possibly cause this? (Background: found out I was pregnant in September-gained weight extremely fast and at 6 weeks looked about 6 months pregnant, but had lost the baby. Had DNC and due to unsterile equipment got a terrible bacterial infection in my uterus. Took antibiotics and was told all is ok. But still have pain and keep gaining weight.) Doctor: Hello and welcome to HCM, A low WBC count is a predisposing factor for infections. I guess you have taken appropriate antibiotics for the uterine infection. Persistence of abdominal/ pelvic pain suggests a pelvic inflammatory infection. You need to visit your gynecologist for some investigations to look for evidence of pelvic inflammatory disease. A pelvic ultrasound may be required. An endometrial curettings and/ or biopsy may also be needed. Weight gain is unrelated to the infection. You need to check out your diet and take regular exercise for that. Thanks and take care. Dr Shailja P Wahal"
},
{
"id": 120336,
"tgt": "How to cure pain in shoulder joints,elbow and right knee?",
"src": "Patient: Hi.I am a 48-year old woman. For the past one year I have been having pain in my shoulder joints and elbow and right knee. X-ray of the knee showed mild degeneration of the disc but my shoulder x-rays showed nothing. Lately I am having pain in my right elbow too. I have taken numerous pain killers and medicines prescribed by doctors, tried homeopathy, yoga, etc without any improvement in the pain. I heard about Amway and would like to try this. Is this advisable? What do you recommend? Doctor: Hello, It may age-related arthritis lead to pain in the joints. It may be a muscular origin. Use physiotherapy and yoga may useful. use tablet.artilage twice a day for one week. If symptoms not improved please consult with your doctor he will examine and treat you accordingly. Hope I have answered your query. Let me know if I can assist you further. Take care Regards, Dr. Penchila Prasad Kandikattu"
},
{
"id": 46961,
"tgt": "Suggest treatment for renal hearing loss",
"src": "Patient: rt. renal hearing loss on 27/05/2014, age 43 years lf. hearing in within normal limits dr. issues medicin medrol 16 mg and cognix plus now full time sound ex. lan line telphone distrabu noise sound another ear now mobile usseing heat and maind ring i am mobile phone usesing more than 13 year over Doctor: HIWell come to HCMI really appreciate your concern, you have not mentioned about the finding of test, most of the time such case needs hearing adds of course medicine can be tried but it gives minimal result although it is all depends, better to get done MRI too hope this information helps."
},
{
"id": 56752,
"tgt": "Suggest treatment for fatty liver",
"src": "Patient: Sir/I me 29 year old, during the health check up,found fatty liver with grade 3rd, ldl & triglisrol level of chelostrol is also high,I m alcholic and 180 ml take daily,I have consult to physician, he consult me udiliv 300 two time in a day & rosuvas f10 in night. This medicine is sufficient , it can recover my deases in 4 week.I have started running and zimm also. Doctor: Hello,Welcome to Healthcare Magic.First please avoid alcohol then medicines will work.Consult your psychiatrist for de-addiction of alcohol.It is now very easy and will helpful in prevention and treatment of fatty liver.Your doctor has given you proper medicines.Please continue them and do regular follow ups.You have started regular exercises is very good for fatty liver.Make exercise schedule fix.Minimum 1 hour brisk walking is necessary.Take fresh fruits and green veg more.Also take protein rich diet, beans etc.Strictly avoid spicy and oily diet.Hope these all will helpful to you.Thanks for contacting us.Take care..."
},
{
"id": 135840,
"tgt": "What causes spasms in abdominal and spinal area?",
"src": "Patient: I am a healthy 96-year old woman. I recently had spasms in my abdominal -spinal area. I had a heart 3 months ago but continue to do my chosen work and enjoy my daily life. I had an ekg, a CT skan@blood tests which were all satisfactory. Can you tell me what other subjects I could study to determine the cause of the back spasms. Thank you Doctor: hiback spasm in old age is due to degenerative changes in spine commonly by osteoarthritis.treatment is by physical therapy,heat ,massage and application of relaxyl geland, if, spasm moderate, then chloraxozone tabs.consult a orthopedic doctor and have x ray spinedonethanks"
},
{
"id": 161548,
"tgt": "Suggest treatment for stomach pain",
"src": "Patient: Hi, may I answer your health queries right now ? Please type your query here.. My five year old daughter has been waking up for the past couple of days saying her stomache hurts, but then in a little while it goes away. This has happened before and I brought her t othe doctors, but nothing was said of why why her stomache was hurting. Should I be concerned? Please advise. Thanks. Doctor: Hi, As per your query, your daughter has symptoms of stomach pain which seems to be due to taking a lot of dairy products in the diet , irritable bowel syndrome, not enough water or fiber in your diet.Need not to worry. For now, eat more fruits and vegetables, drink plenty of water, take ibuprofen or acetaminophen for pain. I would suggest you consult a gastroenterologist for proper examination. The doctor may order stool test, endoscopy and ultrasound. The doctor may change your medicine or prescribe laxative, H2 blockers or antibiotics. Hope I have answered your query. Let me know if I can assist you further. Regards, Dr. Harry Maheshwari, Dentist"
},
{
"id": 103792,
"tgt": "Been suffering from allergic rhinitis, have sneezing, throat pain, green sputum, irritation, breathing difficulty. On telekast. Help",
"src": "Patient: Hi doctor, i have been suffering from allergic rhinitis since 10 years. It all starts with sneezing which leads to itching on the upper mouth and in hours time it changes to throat pain. Throat irritation and pain produce greenish sputum and the irritation on the throat quickly changes to severe cough. Severe cough means its as severe as if im may faint after a cough session. During this time the sputum become greenish and breathhing difficulty starts. During this period i feel heavyness, conjestion and chocking in my chest. Some times fever also persist. nowadays it takes more than a months time to become normal. Anti biotics and cough syrups were used to treat. Reacently a doctor prescribed me Telekast F. But that never helped me from the cycle of events repeated. Now adays i am freequently getting infected. Is long term use of Telekast F help? Is telekast F is the proper prescription? Does there other effective medicin which can save me from this deadly allergic cough? Doctor: these medicines are kay but improves when you take only to subside not to cure you can appky neomycin h eye ointment in nose bdsea water 2 drops at night each nose can take codeine linctus at night which is effecitve in this tpe of coughbut you must go for blood serum tests for specific antibodies for milk wheat potato and common food you eat as the history is long and i expect food component in t when you withdraw you will be much better"
},
{
"id": 19267,
"tgt": "Suggest treatment for heart blockage",
"src": "Patient: my mother is 63 years old and she sufered an heart attack on 18-09-10 and she was admitted to the hospital and the doctors did a angio and told that there is 95% blockage and she should do by pass surgery imediately but my mother is very scared of surgery and i want to know about oxymed Doctor: Hello, Yes best treatment for blockages is angioplasty or bypass surgery. Is there a single block of 90 percent or many other blockages. Because, usually one or two blockages are treated with angioplasty only and bypass surgery can be avoided. Medicine should be continued like Aspirin, clopidogrel, statin like atorvastatin, nitroglycerin, Metoprolol etc.Hope this helps you and get back with some more information or any doubts."
},
{
"id": 25841,
"tgt": "Can abnormal Blood Pressure cause vomiting and lost diet?",
"src": "Patient: Is BP reading of 143/63 normal,This is of my mother's who is 57yrs old and has diabetis and hyperthyroid and BP too.From last three days she is not feeling like having food and has vomiting once/twice a day.the last 2 days the BP reading were normal But today morning it was found 143/60 when tested at home. Doctor: Hello , Very high blood pressure can cause vomiting , However the blood pressure reading you have mentioned w should not cause vomiting . The 2 most probable reasons are . Diabetes related . Keto acidosis . If the diabetes is very high or rather if it's staying in control but continuous vomiting with no intake of food and decreased water . Just check ketone in her urine . A strip is available at all medical stores . A bad stomach . If there is history of food at an outside joint , abdominal cramps and some change in stool consistency . I would pursue that you visit you family doctor as it's already 3 days . Regards Dr Priyank Mody"
},
{
"id": 150177,
"tgt": "MRI showing lesions. Need to get checked by a neurologist?",
"src": "Patient: An opthamologist sent me for an MRI of my head because of some visual issues I have been having. I have a lot of other neurological symptoms as well. The MRI showed two small lesions and he told me not to worry about them and we'll just wait and see if my symptoms change. Should this be investigated by a neurologist as well? I feel like it should. Doctor: Hi,Yes since your MRI brain is abnormal ,you have two small lesions, you need to consult neurologist for sameDetail history and clinical examination is required in your case. MRI details to be given for further additional informationRegardsDr Sandhya ManorenjNeurologist Hi tech city Hyderabad"
},
{
"id": 12373,
"tgt": "Suggest treatment for psoriasis",
"src": "Patient: hi dr, i am a feeding mother of 5 mth old baby. I have psoriasis and so i am off any of the systemic medicine. it agravated in last month. Is it safe to take roxithromycin (150 mg) during lactation along with local applications ? My leisions have become red ... Doctor: Hi..,As you said you suffer from psoriasis from last 15 years.I would like to say few things about psoriasis... . Psoriasis is a chronic relapsing and autoimmune disease. There may be having scaly plaques on various parts of the body,scaly lesions on the scalp. Exact cause is not known. Genetic tendency, autoimmunitity,stress,dry wheather...etc may be responsible for precipitation and exacerbation of psoriasis.You consult dermatologist for firm diagnosis and treatment.I usually recommend only local treatment for psoriasis in pregnant and lactating women.For itching, you may take antihistamnics like levocetirizin...Avoid steroid ,as there may be severe flare up after stopping it. Roxithromycine is safe, you may take it if needed.You may apply mild steroid cream or calcipotriol oint on the lesions on the psoriatic lesions. You may apply moisturiser to improve the skin texture. You should avoid soap bath for few weeks..And you should be free of stress and worries as that might worsen psoriasis.you might get good result.I hope you got my answer.Thanks.Dr. Ilyas Patel MD"
},
{
"id": 177456,
"tgt": "What causes white chalky patches of urine?",
"src": "Patient: My 5 yr old boy has missed the toilet when urinating a few times during the night this.month which i hadnt noticed until the next morning and when i noticed that the urine left white chalky droplets around the toilet. He has no complaints of pain or discomfort. No odor to urine. Doctor: Hello Sir/MadamI dont think your son has any major problem.turbid urine can be due to urinary infection but it is unlikely without any other symptom in children.You check his genitals for tight urinary orifice because it can lead to urinary infection.Also do a urine routine examination and microscopy so that any abnormalities can be detected.I'll be happy to clear any further doubts."
},
{
"id": 107585,
"tgt": "Suggest treatment for severe lower back pain",
"src": "Patient: On Saturday afternoon I was lifting a heavy object and heard a pop in my lower back. Was painful right away. After 30 minutes or so I was able to move around fine and was able to do yard work on Sunday. But every time I sit down and get back up, it takes a few minutes to straighten up and for the lumbar back pain to go away. Am a 48 year old male. Doctor: With such a presentation it is likely that you have muscle strain or a mild form of disc rupture. I would suggest you to avoid lifting weights, bending forwards, sitting on the floor and squating atleast for a week or so.You can use ice gel packs and pain killer lotion/ gel to soothen your back. Hope this helps . All the best."
},
{
"id": 77088,
"tgt": "What cause chest pain during intense cardio exercises?",
"src": "Patient: Hi, I'm a 35 yr old female and I started working out 2-3 times a week 9 mths ago. When under intense cardio, I experience one-sided (either right or left) chest pain right over my breast area. I'm not sure if that's the subclavian artery.The pain is so intense that, while the cardio class is going, I put pressure with my hands on the pain, to alleviate it. Why is this happening? Should I worry? Doctor: Thanks for your question on Healthcare Magic. I can understand your concern. Possibility of musculoskeletal pain is more but better to rule out exercise induced ischemia first. So get done ecg, 2d echo and stress test (trade mill test). If all these are normal then no need to worry for major heart related diseases. Sometimes, excessive cardio exercise can cause musculoskeletal chest pain. So avoid excessive cardio exercise. Avoid movements causing pain. Take painkiller like ibuprofen. Apply warm water pad on affected areas. Don't worry, you will be alright with all these. Hope I have solved your query. I will be happy to help you further. Wish you good health. Thanks."
},
{
"id": 31534,
"tgt": "What is the treatment for influenza?",
"src": "Patient: two weeks ago I had a flu shot, I hardly get sick. Last night I felt a little chilly and I had a fever of 100.5 Now I feel as though one side of my gland hurts when I press it. I also have a sore throat. I just can't beleive, I got a flu shot and now I have these symptoms. Doctor: Welcome to HCMI have carefully read and understood you queryI hope you will find this usefulInfluenza is a viral infection that attacks your respiratory system i.e your nose, throat and lungs.Though it may resolve on its own but the symptoms and sometimes complications may be very bothersome.The best is to take plenty of fluids.Take an NSAID like paracetamol or ibuprofen for sore throat that will settle pain and fever.Do salt water gargles everyday.Dont use mouth washes for gargles.Sometimes we do prescribe an anti viral like oseltamivir as oral medication which will cut course of the medication.Get more sleep to help your immune system fight infection.Do see your doctor before starting any antiviral.I will be happy to know if you have found answer to your queryThank youDr Faeza Sherdil"
},
{
"id": 9254,
"tgt": "How to treat dry lips?",
"src": "Patient: I have these really tiny bumps all over my bottom and top lip. They want to look like little heat bumps (they have a little fluid in them that comes out if I press my lips together. My lips are also very swollen. This is the second time this has happen in the last month/6weeks. It doesn t hurt, and it really doesn t itch (the first time a month ago it sort of itched). My lips feel really dry and like scales are on them. I have been taking Benadryl for 2 days now, whiles it hasn t gotten any worst, it hasn t cleared up like the pharmacist said it would. Any idea what this is? Doctor: You seem to have chronic cheilitis. You develop small vesicular lesions on lower and upper lips. I think it may beome crusted and swollen off and on.There may be erythema, dryness,mild scaling, fissuring and crusting. You might have mild pain, burning and itching. There may be some cause. Lip licking might exacerbate the disease. Sunrays exposure, cosmetics,lipstiks tooth paste, dusts...etc might be resposible. Vitamin B complex deficiencies,anaemia or nutritional inadequacy might cause the disease...You consult dermatologist for firm diagnosis and perfect treatment of its cause.Have blood tests to rule out any internal disease or vitamin deficiencies.You may take folic acid 5 mg thrice a day with iron tab. Antibiotics may be needed if there is secondary bacterial infection. Apply antibitic cream with mild steroid like mometasone. You avoid soap,cosmetics and lipstik for good and fast result. Avoid bad habit of lip licking,if it is there. Do not prick the lip skin with your nails.You might get good result after few weeks. I hope this would help you in your treatment.Thanks.Dr. Ilyas Patel MD"
},
{
"id": 54642,
"tgt": "What causes pain on the right side after removing gall bladder?",
"src": "Patient: I had surgery last Sept. I had a blockage in my bileduct...thet put a stent which had to be removed 24 hrs. After because of servers pain..which I was informed it slipped and inflamed my pancreas. This was done by my Gastrolist but my surgeon shortly after admitted me to another hospital and removed my Gallbladder. I ve had serval CT scans and MRIs....telling dr.s I am experiencing pain in my right side..sometimes extremely bad. I was told it could be scar tissue .........right now I hurt so bad....but I am not going to be told its nothing. Please give me some advise Doctor: Thank you for posting your queryneed further evaluation.Differential: Stones in CBD, Hepatitis, Mild pancreatitisdo following:1. Ultrasound to rule out any stones or abnormality in Common Bile Duct2. Liver function Test (liver enymes, if possible HepB & C serology)3. if above two test results are normal then you may check serum amylase and lipase to rule out pancreas pathologyhope to answer your concern.Health professionals aim to diagnose properly and manage patients according to their limited knowledge. Cure is blessed by the ONE who Created us, whose power and knowledge is unlimited .wish you good health.regards,Dr Tayyab Malik"
},
{
"id": 80179,
"tgt": "Why experiencing chest pain even after normal stress test?",
"src": "Patient: Hi I went to the emergency two weeks ago had stress test chest X ray and also they did Cat scan with and without dye. But could not find anything wrong. And still experiencing chest pain and feeling like sore around my chest and left side of my chest? Doctor: Thanks for your question on Health Care Magic. I can understand your concern. Since your extensive cardiac and Pulmonary work up is normal, no need to worry for major disease. In my opinion, you are mostly having stress related chest pain. Stress is the biggest risk factor for GERD (gastroesophageal reflux disease). And this can cause central chest tightness, pressure and burning chest pain. So better to consult psychiatrist and get done counseling sessions. Try to identify stressor in your life and start working on its solution. You may need anxiolytic drugs too. So avoid stress and tension, be relax and calm. Don't worry, you will be alright. Hope I have solved your query. Wish you good health. Thanks."
},
{
"id": 36376,
"tgt": "What causes infected blister under the skin near my coccyx?",
"src": "Patient: There is a scab (bubble scab) on the left side of my lower back close to me tail bone, right above the crack areas at is full of fluid, around te scab under the skin (non visible) feel like it s full of piss or fluid spread out over about 4 inches. It s sore, almost bruise like. The lump had been there for 2-3 years, but the scan has been there for 5 months. What s going on Doctor: HelloWelcome to HCMThanks for posting your query in HCM.Your symptoms are suggestive of pilonidal abscess leading to pilonidal sinus formation.So i would advice you to consult a general surgeon and get physical examination done for confirmation of the diagnosis.It needs drainage and a course of antibiotics.Thank you"
},
{
"id": 24536,
"tgt": "Could the high cholesterol be a matter of concern?",
"src": "Patient: I'nm a 38 year old female and just had my cholesterol levels checked for the first time. My LDS is 124.2 (they said normal is <100), my Non-HDL is 150 (they said that 60-100 is normal). My overall cholesterol is 200. Should I be concerned?? What should I do to lower my levels?? Doctor: Thanks for your question on Healthcare Magic. I can understand your concern. Yes, you should definitely concern about your altered lipids. LDL is the main lipid which accumulate in arteries (specifically Coronary artery) and cause blockage and subsequently heart attack. And risk is even more when you approach menopause. So at this age of 38 years, you should definitely do following things to lower LDL. 1. Avoid fatty and oily food. Take high fiber diet more. 2. Avoid stress and tension, be relax and calm. 3. Do regular aerobic exercise like running, cycling, swimming etc. 4. Quit smoking and Alcohol if you have these habits. 5. Start sunflower oil for cooking. 6. Start low dose of rosuvastatin or atorvastatin (lipid lowering drug). Don't worry, you will be alright with all these. Hope I have solved your query. I will be happy to help you further. Wish you good health. Thanks."
},
{
"id": 160666,
"tgt": "What causes headaches and low grade fever in a child?",
"src": "Patient: Hi my daughter is 6 years old and typically very healthy. lately shes been complaining of headaches often. shes also had a low grade fever on and off for a week or so (around 100) Yesterday she had a nose bleed, and has had 2 others in the past couple of months. I am thinking its just allergies, but just wondering if there is a possibility it could be anything else that should concern me. Doctor: Hi,Headache and fever without any other associated symptoms like cough, throat pain, abdominal pain, dysuria usually indicates viral fever, that just need paracetamol, bed rest and plenty of oral fluids for few days. But since it is there for 1 week, I would prefer getting a blood and urine routine done after examining her. This is to look for any focus of infection that might need antibiotic therapy.Episodic nose bleed is most commonly secondary to nose pricking habit or mucosal dryness. Allergy can be contributing factor for dryness, but usually is associated with frequent sneezing, itching and nasal discharge also. In recurrent cases, I used to get a blood routine done to rule out low platelet count as a cause. Your doctor will guide you.Hope I have answered your question. Let me know if I can assist you further. Regards, Dr. Muhammed Aslam T. K., Pediatrician"
},
{
"id": 84651,
"tgt": "What are the side effects of luprodex injection?",
"src": "Patient: i m 35yrs female with no kids and have been diagnosed anterior wall intramural fibroid 5.5 x 4.5 cms. i want to have babies lateron. at present i get heavy flow with clots during periods. i want to know about luprodex injection usefullness in my case, its dosage , safety and sideeffects. please help me Doctor: Hi,Commonly prescribed hormone dependent tumors like uterine fibroids in the dose of 3.75 mg injection once a month for up to 6 months or 11.25 mg depot every 3 months. Its common side effects include allergic rash, muscle pain, joint pain, photosensitivity, hypotension, and peripheral neuropathy. You must inform your treating doctor if you develop the above listed side effects.Hope I have answered your query. Let me know if I can assist you further. Regards, Dr. Mohammed Taher Ali, General & Family Physician"
},
{
"id": 150204,
"tgt": "Fluttering in leg, painful, cramps in feet. Help",
"src": "Patient: Hi I have been getting fluttering symptoms all down my left leg..it started a few months back, where at first I was just getting a real sharp pain for a few seconds..which would be at random intervals. Now I have constant fluttering feeling in my left leg, the sensation comes and goes throught the day and night. I also suffer with terrible cramp in my feet/toes. Doctor: Hi,Thanks for the queryI understand your concernThe causes of these types of symptoms can be due to either electrolyte imbalance or neurological due to spinal cord or nerve root problem. A clinical examination will clarify the picture in a better way. I feel that you should get examined by a local neurologist who will help you through investigations if needed.Please get back in case you need any additional informationBest wishes"
},
{
"id": 152123,
"tgt": "Is gabapin 100 a safe drug ?",
"src": "Patient: Is gabapin 100 a safe drug Doctor: Its is advisable to continue or discontinue any medication only under medical supervision. No drug is absolutely safe or unsafe as it depends on the nature of illness and the biological constitution of the patient."
},
{
"id": 177189,
"tgt": "What is the suggested diet for 1 year old in case of cold and nose blockage?",
"src": "Patient: My kid is 1yr old now.This is the first time he got infected by cold.He has severe nose blockage and cough with vomiting,sufered to breath and to have tight sleep,Yesterday i went for a check up .Doctors given Nibulisation for two times and prescribed Asthalin 3 times a day and Cetrizine Syrup as a combo only in night.And they said it is sign of wheezing.His weight 13.7 ,worried about his health and please advice a good diet for him for his better health and immune power Doctor: Hi...by what you quote your kid seems to be having viral associated wheeze or a multi triggered wheeze. There is no specific diet but there have been reports of vitamin C and citrus fruits like orange and lemon to protect from recurrent illness and allergy. Usually kids grow out of such wheezing as the age progresses. Nothing to worry about it.Regards - Dr. Sumanth"
},
{
"id": 125145,
"tgt": "Is surgery needed for arthritis in knee with lump under knee cap?",
"src": "Patient: Hi, I have been diagnosed as having arthritis in my right knee. I have had a movable soft lump slightly off centre and a little under the knee cap of my left knee, im concerned this will result in needing to have this one operated on...can you advise? Doctor: Hello, Consult an orthopaedician and plan for an MRI scan. If required you can go for an arthroscopic repair. Hope I have answered your query. Let me know if I can assist you further. Regards, Dr. Shinas Hussain, General & Family Physician"
},
{
"id": 107870,
"tgt": "Suggest treatment for upper back pain and pressure while urinating",
"src": "Patient: I have been having pain and pressure, and the need to urinate for the past two weeks. My upper back 8 beginning to ache. I have taken cranberryills, drank cranberry juoce, and lots of water. It got better but became sinful again. Is 5 here anything I can do? I can t afford to go to the doctor. Doctor: Dear- your symptoms are from urine infection and treatment is antibiotics.You can try pyridine that is overthecounter but if no improvement go to an urgent care clinic for an evaluation. DrSara"
},
{
"id": 140418,
"tgt": "What are the lasting neurological effects of kerosene ingestion?",
"src": "Patient: Hi Doc, My question pertains to ingestion of kerosene as a toddler. What are the lasting physical effects of such a tramatic experience at such a young age? Having ones stomach pumped and placed in ICU for 2 weeks under a oxygen tent. Can a child fully recover and, or what are the lasting neurological effect on the body? I tend to think that the experience effected the normal growth cycle especially the hands. Doctor: Hello, Chronic neurological effects are exactly not known by animal studies. possibility of hypoxia through aspiration related symptoms or convulsions etc. Please consult your physician he will examine and treat you accordingly. Hope I have answered your query. Let me know if I can assist you further. Regards, Dr. Penchila Prasad Kandikattu, Internal Medicine Specialist"
},
{
"id": 6098,
"tgt": "Had fallopian tube blockage cleared. Delayed periods, pain in the back and abdomen, light bleeding. Could this be implantation bleeding?",
"src": "Patient: Hello, back in November I has my fallopian tubes cleared from a blockage and was told that was probably my reason for not becoming pregnant. I was told that everything should be ok and started checking my bbt . I have not had any problems since then and my cycles have been on time every month with a duration of about 29-30 days between them. My last cycle was on June 20 and I began to taking hpt . One came back slightl faint and prompted me to take addtional test in which all came back negative. Well after 10days of a missed cycle, preganancy symptons and lower back pain on my left side. I woke up this morning to light bleeding and very sharp pains in my lower abdomen. Could I be experiencing a miscarriage or could this be implantation bleeding? I have been pregnant before 11 years prior, which also resulted in a miscarriage, but never had symptoms. Doctor: hi, sometimes urine pregnancy test kits can give false positive results that may be reason for your faint positive first test, anyhow now you once consult gynecologist and undergo blood test for pregnancy which will rule out the possibility, if comes positive it is good, if not you can go for physical examination and ultrasound examination to find out the cause for your delayed periods. take care."
},
{
"id": 145494,
"tgt": "What causes tingling hands and numb feet?",
"src": "Patient: Recently I have been experiencing tingling in my hands and now my feet while lying in bed trying to sleep. I had a sharp pain in my left foot last night that woke me suddenly form my sleep. It was quite uncomfortable and painful. I first thought it may of been a pinched nerve but my symptoms subside during the day. any thoughts? Doctor: Hi,Thanks for writing in.It appears that there might be a condition involving the nervous system in your body. The above symptoms like numbness and tingling are associated with conditions of nerves affected in various regions. This can be due to an acute cause or due to chronic problems like neuropathy associated with diabetes or auto immune disorders. History of any other medical conditions and medications that you are taking is important. Vitamin deficiency states can also appear as non specific symptoms as described by you. Please consult a neurologist and get a complete neurological examination done. Investigations like MRI scan brain and spinal cord might also be required for you."
},
{
"id": 142963,
"tgt": "What causes electric shock sensation while using glass face touchscreen appliances?",
"src": "Patient: I get an electric shock sensation almost like if you lick your finger and touch a battery, but only happens while im using my glass face touch screen appliances i have gotten different phones etc.. and still the same sensation, and i dont know if its relevant but ive been noticing my finger are slightly swollen. Do you have any advice on what this could be Doctor: Hi, Welcome to HealthCareMagic.com I am Dr.J.Mariano Anto Bruno Mascarenhas. I have gone through your query with diligence and would like you to know that I am here to help you.Electric shock sensation while using glass face touchscreen appliances are due to Altered Capacitance in your skin. You may consult a Dermatologist in your area for further evaluationHope you found the answer helpful.If you need any clarification / have doubts / have additional questions / have follow up questions, then please do not hesitate in asking again. I will be happy to answer your questions. In the future, for continuity of care, I encourage you to contact me directly in HealthCareMagic at http://bit.ly/askdrbruno Best Wishes for Speedy Recovery Let me know if I can assist you further.Take care."
},
{
"id": 195519,
"tgt": "How to stop masturbation habit?",
"src": "Patient: hello doctors, can you help me?I am 23 years old Boy. I am badly suffering from masturbation habit from last 10 years. Now I want to quit from this habit. But I am not able to get rid of it. Now I am frequently doing it 3 times in a week. If I am not doing it, the sperms comes out in sleep. what should I do? and tell me how to control habits? Doctor: Hello and Welcome to \u2018Ask A Doctor\u2019 service. I have reviewed your query and here is my advice. Masturbation is not a bad habit but leaking of semen some times misguided with urinary tract infection or prostatitis.nocturnal leaking also common in teenagers , but you have to check for retrograde ejaculation or leaking of semen.so don't worry about masturbation it may vary from 1-5 times a day to once in three weeks.so please consult your physician he will examine and treat you accordingly. Hope I have answered your query. Let me know if I can assist you further."
},
{
"id": 116633,
"tgt": "What is the treatment to increase the hemoglobin level?",
"src": "Patient: My son (4) has had three low hemoglobin levels in the the last three months at WIC. 9.4, than 9.6, than 9.3 WE have been giving poly vi soln with iron daily. We have also been giving milk between meals and OJ with meals. He gets a lot of healthy high iron foods like whole wheat bread, broccoli and meat. What else can I do? Do I need to make a visit to the doc? He goes for his yearly in Aug. Doctor: HelloThanks for contacting usYour worry is natural but children have choices of foods so they are not getting proper food from which hemoglobin (HB) increase. HB is made up of iron and folic acid. So give your child richness with iron and folic acid. give him dry fruits, green vegetables and salad. most important, give him lemon or orange juice after meal, will increase iron absorption and will increase HB but have patience because this will take some months but surely help..take care..."
},
{
"id": 195226,
"tgt": "What does this serum testosterone test result indicate?",
"src": "Patient: HormonesTest Name : Serum Testosterone Result : 411.7 MALES (YEAR) Unit ng / dl 0 - 4 10 - 160 4 - 12 10 - 29012 - 16 10 - 56016 - 20 260 - 73020 - 49 270 - 1734> 50 212 - 755 FEMALE (YEAR) Unit ng / dl12 years upto 20 Ovulating 63 120Postmenopausal 49 113 CLINICAL CHEMISTRY Tests Normal Value ResultFSH UNIT:mlU/ml 46.72 FEMALE: Follicular-phase:3.9-10.0 Mid-cycle: 6.6-29.0 Luteal-phase:2.3-8.0 Post-menopause:26.5-139.0 MALE:1.7-11.0 CHILDREN:0.5-3.3LH UNIT:mlU/ml 46.72 FEMALE: Follicular-phase:2.6-12.7 Mid-cycle: 9.4-27.9 Luteal-phase:1.8-16.7 Post-menopause:13.1-50.1 MALE:1.6-7.8 CHILDREN:0.7-3.8 Doctor: Hello and Welcome to \u2018Ask A Doctor\u2019 service. I have reviewed your query and here is my advice. Your a male or female?? Dear your testosterone report is normal and your FSH and LH report also normal just in case your a post menopausal female. But in case if you're a male it may suggestive of primary hypogonadism like high FSH and LH with normal to low testosterone. So please consult your doctor he will examine and treat you accordingly. Hope I have answered your query. Let me know if I can assist you further."
},
{
"id": 182016,
"tgt": "What causes stinging and burning sensation on gums?",
"src": "Patient: This week I began feeling a stinging/burning sensation on my gums. The stinging was felt in the area where one of my top right teeth were removed about 9 years. It feels uncomfortable when I eat or drink. I took a picture of the isnide of my mouth and saw 3 black dents in my gums. Anytime I put my tongue on it it stings. I'm worried because I've never seen this before. Doctor: HiThe condition you have mentioned above is probably your back tooth which was infected and was removed. however professional removal of tooth involves in total removal of the tooth aliong with its roots , which was probably not done in your case ,that explains the three black stumps . Infected root stumps may pose problematic causing pain sensation in the soft tissue around the root stumps.I advice you to go to an oral physician ,who will take a radiograph of that area ruling out any other infection . the root stumps need to be removed completely hope this will be of some help to you , take care ."
},
{
"id": 191512,
"tgt": "How can diabetes in a child be treated?",
"src": "Patient: My 8 year old granddaughter is testing on levels on my glucose machine. Just because she is over weight I have concerns about her getting diabetes. She tested 301 and the 2 mins later she was 133. So you think this is the machine acting up or something I should be concerned about. i checked mine and they seem off too, what should kids her age levels be? Doctor: Get a laboratory test of fasting and post prandial blood sugar done. More so if there's any positive family history of early diabetes. Insulin is the treatment of choice for people with diabetes at this age."
},
{
"id": 78310,
"tgt": "What causes chest pain with fatigue and dizziness?",
"src": "Patient: Hi... Oct 24th 2013, I was admitted to hospital with a fish bone... which resulted in going to surgery to have it removed from the back of my throat ... I was home three hours after the operation but over the next few days suffered shortness of breath, and a cough.. resulting in having to sleep sitting up. I attended my GP who sent me for a chest xray on 22nd Nov and this was compared to the one taken on Oct 24th.. Results on this was inconclusive so I've been sent for a CT scan... I continue to suffer from chest pains from front to back, shortness of breath and tiredness. (I note that acid reflux has been mentioned as I find that I can't drink even a glass of wine or eat anything after 6pm without feeling unwell in the morning)... Today I got up at 8.30am have been busy until 12.30 - have chest pains... extreme tiredness and am dizzy. Would appreciate your thoughts on all written please? Doctor: What does the CT scan report say. Your could have had a aspiration/obstructive pneumonia following fish bone getting stuck in your airways. May be some residual abnormality is still persisting. You need a complete check up so consult your PCP."
},
{
"id": 35609,
"tgt": "How to treat penile yeast infection?",
"src": "Patient: I am a 57 yr old male and I strongly suspect that I have a penile yeast infection (symptoms match pretty closely). My wife is past menapause and we no longer use the sheath as my erection is often dodgy and a condom unfailingly kills it stone dead. I've thoroughly cleaned myself but there is skin discolouration on part of the head of the penis under the foreskin. We have not enjoyed intercourse for some time and tonight I feel able but I do not want to cause my wife a problem. Should I avoid intercourse? Doctor: Thanks for asking in healthcaremagic forumIn short: Its good that you have concern for your wife regarding this.Explanation: Apply oint clotrimazole locally for relief. Then after relief you may go for intercourse. If it does not subside by 3-5 days then visit a doctor for examination. All the best. Be patient for some time."
},
{
"id": 214028,
"tgt": "lip/mouth twitching and trembling",
"src": "Patient: for months now i have been having problems trembling and twitching in the mouth/lip area. i have been suffering for along time with depression/ocd/anxiety/stress in which i have tried many ways to be stress free and was prescribed with fluoxetine 20mg, now i m not sure whether that has been the problem for this but i had to come of it because it seemed to make it worse. now i thought coming off the medication would relieve my problems but it has not. i am still getting the same problems, and it s affecting me socially and even when alone it still a big problem. recently i have been taking atenolol beta-blocker 50mg, but that to has not helped. it would be great help for someone to tell me what is going on, because i am not sure what to do now. i ve tried to diagnose my problems online, either i have some thyroid condition, tremor problems, or phycological issues. it would be great to get some help. Doctor: Hi Your problem looks like \"Oro-facial Dyskinesia\" ...in ordinary words \"mouth-face disordered movements\". Usually antipsychotics i.e medicines used for psychosis cause dyskinesias but sometimes SSRI antidepressants like Fluoxetene MAY cause this problem. Usually the first mode of treatment is stopping medication or decreasing the dose if stopping worsens the depression. We may also try some other medication to try and see if it helps. Unfortunately, for a few percentage of people, the problem is irreversible. BUT IT CAN NOT BE PREDICTED AS TO WHO WILL HAVE IRREVERSIBLE AND WHO WILL HAVE REVERSIBLE PROBLEMS. so we consider all cases as reversible and try our best. Please consult a psychiatrist for treatment. I would honestly like to know why so many patients, already under psychiatrists, don't tell them about their problems but seek help from other sources. I would really like to know so that I can not do what these psychiatrist may be doing wrong, if any."
},
{
"id": 42442,
"tgt": "How does Letrozole and Astaxanthin work?",
"src": "Patient: my age is 26,height is \"5.2\",weight is 55.i m suffering from PCOS and already missed my pregnancy on 2nd month....now my doctor have suggested to take letrozole tablets with astaxanthin capsules(ova shield).so how it works for ?i want to pregnant? Doctor: Hi,Welcome to HealthcareMagic .Letrazole helps in production of multiple healthy ovum so chances of conception increases .Astaxanthin is an antioxidant which helps in improving quality of eggs and also endometrial growth. So you are adviced to continue the tablets.Also I would suggest you to go for IUI . Here healthy sperms are placed in uterus . This will increase chances of pregnancy .Hope I have been helpful .RegardsDr.Deepika Patil"
},
{
"id": 51396,
"tgt": "On dialisis for kidney failure. Able to pass urine and feels hungry. Is it possible to undergo stem theraphy?",
"src": "Patient: Good night Doctor.My mother is on dialisis for kidney failure thrice in a week.I want to know if it is possible for stem cell theraphy for failur of kidney.She is 50 years old.as of now she is able to pass urine and feel hungry.Is it possible for my mother to undergo stem theraphy? Any where in India especially hyderabad.is it possible Doctor: As of now the stem cell therapy is only experimental with not so good results. Also most of the studies are in early CKD and not end stage disease disease. It would be better for you to continue on dialysis or think of a transplant if possible."
},
{
"id": 120051,
"tgt": "What causes large lump on right knee with tightness in glutes?",
"src": "Patient: Hi, I am a a very fit and active lady in my early fifties and over the course of the last two months have a egg sized lump on the outside of my right knee. I have been to my GP and had an x-ray. This confirmed that it was not calcified. However, it is still there and causing discomfort when I walk and or bend. I used to run 4 times a week, but have stopped since the onset of this condition. I am also feeling a tightness in my glutes which tends to want to cause a twisting motion when I walk. Any thoughts on what this is and what can be done about it? Many thanks Doctor: Hello, The lump outside the knee can be either a synovial cyst or lipoma or a sebaceous cyst.These all are soft tissue cysts.I shall advice you to ask for USG of this swelling.If any of the above diagnosis is there then it should be removed by surgery. The feeling of tightness at gluteus region is most probably due to stretching of iliotibial tract at outer aspect of thigh.You may ask for a prescription of muscle relaxant from your doctor for this. Hope I have answered your queries. Let me know if I can assist you further. Take care Regards, Dr. Mukesh Tiwari"
},
{
"id": 147790,
"tgt": "Is Arecept more effective in curing Alzheimer's than Yasnal?",
"src": "Patient: Thanks for your support.My mother in law has taken Yasnal/10mg for almost two years. She has been suffering Alzheimer for 4 years.Now we are receiving advice from our friends who have patient with Alzheimer and have taken Aricept and more satisfied with its effectiveness. Is the quality of Aricept really much better than Yasnal . Should we ask our physician for change of medicine? Is the Aricept newly developed medicine for for Alzheimer? The fact is that Aricept is much more expensive than Yasnal in our country Iran. Thank for your time and your advice. RegardsBijan Doctor: HIThank for asking to HCMI really appreciate your concern, all the medicines used for the Alzheimer disease have got the same action promising result are very rare, because of the pathophysiology of the disease, hope this information helps you have good day."
},
{
"id": 109611,
"tgt": "What does pain and snapping in lower back while coughing mean?",
"src": "Patient: Hi, I have been having a cold with a bad cough. Yesterday I coughed and something in my loser back snapped. It made me scream out in pain. Now I get this same pain when I sit or stand or lay down. What did I feel snap? and what should I do? Thank youMs. A Doctor: Coughing strains the muscles of your body including that of your back. A sudden forceful contraction of that kind could have given you a slipped disc. It goes with the snap along with excruciating pain that you felt. It is a serious condition and may need to be treated urgently. Please visit a hospital without delay. Applying hot compress may give you some relief for the time being."
},
{
"id": 184691,
"tgt": "Could using listerine mouth wash cause numbness in tongue?",
"src": "Patient: My son has used Listerine mouthwash for some time. Recently, after using it, his tongue went numb. He needed to have his wisdom teeth pulled & thought it might be from that. The oral surgeon just pulled his wisdom teeth & said the nerve to the tongue is not associated with the nerve to the wisdom teeth. They are putting him on a steroid. They said Listerine may have caused it. Is that possible? Doctor: Hello, Read your query, as your son had undergone wisdom tooth extraction this paraesthesia of tongue can be due to lingual nerve injury during anaesthesia given or it can be due to use of mouthwash also dont be worried so much you should consult dentist and discuss to take mrdication like one tablet Neurobion for numbness, discontinue your mouthwash . Do warm saline gargle two - three times a day, take proper nutritious healthy diet , green leafy vegetables , take proper sleep of 8 hours. Hope it will help you. If you have further query I will be happy to help. Wishing him good health .Regards, Dr. Priyanka tiwari"
},
{
"id": 63832,
"tgt": "Can a hard lump on forehead after a hit be a fracture?",
"src": "Patient: I had a fall on the pavement May 5th face first;my eye swelled shut and there was a small cut on my eyebrow and no major bump on my forehead but now there is a hard lump on my forehead and I'm wondering if it could be a fracture. age-55,height-5/5,weight-200,medical history-a few surgeries,acid reflux,and not sure of high BP Doctor: Hi, this forehead lump is hematoma organized and would reduce in 3 wks time.If still worried for fracture get CT scan to rule it out.Sp you neeed to worry of it and Consult a Surgeon.Hit thanks.Gd dayy"
},
{
"id": 159556,
"tgt": "Do I have cancer due to use of tobacco? Why is mouth irritated?",
"src": "Patient: Dear Doctor, I was a gutka ( tobacco) user for more than 18 years and left before 3 years . It was good for me earlier and now for more than six months I am not able to open my mouth more than one inch wide. I suspect I am affected by cancer and ind idgestive system also i have so many problems now. My jaws are aching and I feel very irritated often in inside mouth. What is the next step i can do with simple test to findout and what is the treatment to open my mouth immediatly, atleast to give me some confidence. Kindly help me. I am from south of India. Doctor: Hi, Welcome to HCM, I am Dr. Das firstly, the disease you are suffering from is most probably, diffuse sclerosis of oral mucosa. But the disease must have to be confirmed by doing biopsy from oral mucosa. So, you have to go to any dentist or oral surgeon. then go for biopsy test which will confirm what you really have."
},
{
"id": 162848,
"tgt": "Suggest treatment for severe back pain on the left side",
"src": "Patient: My daughter 13 is a gymnast. She has had back pain which has been muscular and worked with a physical therapist. Tonight though, she is describing a sharp stabbing pain on her left side of back. She has had something like this before. Any idea what this is and how to treat? Doctor: Hello and Welcome to \u2018Ask A Doctor\u2019 service. I have reviewed your query and here is my advice. I think pain may be due to strain. You can give tab. Tylenol, Diclofenac gel or ointment locally. Do regular physiotherapy after consultation. Apply ice locally. For severe pain concern with orthopedician for detailed examination. Take care. Hope I have answered your query. Let me know if I can assist you further."
},
{
"id": 112309,
"tgt": "MRI done for lower back pain. MRI shows minimal bilateral neural foraminal narrowing",
"src": "Patient: I had an MRI done after 2months of pain in my lower back. I am a 43 year old female. The MRI results say l5/ s1 right paracentral disc protrusion exerting mass effect on the traversing right s1nerve root. L3 /l4 small right sub articulate / foraminal annular tear / disc protrusion.L4/ l5 broad based posterior disk buldging with small superimposed central disc protrusion mild to moderate central canal stenosis. Minimal bilateral neural foraminal narrowing.A few tiny scattered schmorl's are noted. Doctor: Hi, thank you for your question.According to the scan you should have pain in the back of your right leg and in the calf region.You may also have pain in both legs on walking for short distance.If you have only back pain with no legs symptoms then no surgery needed unless symptoms change.If however your leg symptoms are severe then surgery\u00a0can help.Kind regards"
},
{
"id": 76354,
"tgt": "Suggest treatment for bronchitis",
"src": "Patient: I am a 64 year old slightly obese woman. I have been diagnosed with brochiestis(sp). I have severe bouts three to four times per year. Currently suffering in my second week of gurgling right lung. When I see the pulmonary doctor he doesn't say anything. Can he be missing something. I am drowning and choking on my own juice. I don't even bother to go to the doctor because he adds nothing. Doctor: Thanks for your question on Healthcare Magic. I can understand your concern. Uncontrolled bronchitis can cause right sided heart failure and cor pulmonale (cor p). This can cause pulmonary edema and feeling of chest congestion, gurgling in chest, drowning etc. So in my opinion, you should definitely consult cardiologist and get done 1. Clinical examination 2. Ecg. 3. 2d echo You will need diuretics (Lasix or torsemide) to relieve pulmonary edema. Control of bronchitis is also very important. And for this, you need to start inhaled bronchodilator (formoterol or salmeterol) and inhaled corticosteroids (ICS) (budesonide or fluticasone). Don't worry, you will be alright with all these. Consult cardiologist and discuss all these. Hope I have solved your query. I will be happy to help you further. Wish you good health. Thanks."
},
{
"id": 216863,
"tgt": "What causes pain and soreness in the area of incision in breast?",
"src": "Patient: I had a breast lumpectomy about one year ago. I have started a rigorous rowing regime and experienced pain in the area of the incision (6 o clock in the right breast) that feels like a stress fracture or muscle strain. I have stopped rowing for the last week and the pain has diminished. Is it possible that this pain is related to my surgery one year ago? Doctor: Number of possibilities. these include the scar forming and tugging on the surrounding area. Also nerves are cut and can have pain from it. The type/location/other features of these conditions are quite different as are the treatments."
},
{
"id": 175415,
"tgt": "What causes red pimple on face?",
"src": "Patient: hey doctor my son is 3 and he had a pimpole and it went away so a couple days later it got big and red i took him to the hospital and there was nothing to drain so hes on meds now i been puttin warm water on it he lays in bed all day but there is no head on it. Doctor: you apply any antibacterial ointment 2 times 5 days, especially chloramphenicol 1 % is very good. You can also apply locally iodine"
},
{
"id": 2741,
"tgt": "Could she get pregnant from pre-ejaculatory liquid near vagina?",
"src": "Patient: Hello, so me and my girlfriend had intercourse a couple of days ago but my penis did not go inside her vagina. I was just rubbing my penis against her outer vagina but I didn t have a condom on. Could she get pregnant with pre ejaculation being around her vagina, but not fully inside of it? Doctor: Hi,I guess your concern is about getting pregnant of your girlfriend from pre-ejaculatory liquid near vagina. So my straight forward answer is, YES. There is chances but its VERY VERY LESS to getting her pregnant. Usually chances of getting pregnant is more when ejaculation occurs when penis is inside the vagina. But in your case as you told that your penis did not go inside the vagina and you were just rubbing it against her outer vagina. So there is very very less chance of getting pregnant. So DONT WORRY, RELAX. You also told that u didn't have condom on that time. And there is pre-ejaculation occurs around her vagina. So there is little bit chances having for getting pregnant. For that you do not need to take tension because your girlfriend can use emergency contraceptive pill (i-pill) within 72 hrs of unprotected intercourse. As told by you that you people had intercourse 2 days ago. So there is sufficient time to take emergency contraception. This is usually available your nearby medical store at affordable prices. With this above discussion you can try out my suggestions-1. Don't waste time and get the i-pill as early as possible. Ask your girlfriend to take it. Wait till her next menses. If she gets menses on her expected or delay date, she will not be pregnant. 2. You can do urinary pregnancy test after 5-6 days of her missed period. It is available at affordable prices your nearby medical store. Buy it and ask your girlfriend to do it by herself. if it is negative, she's not pregnant. if it is positive visit to nearby gynecologist for further help.Hope I have answered your query. Let me know if I can assist you further. Regards,Dr Sudha Rani Panagar"
},
{
"id": 10198,
"tgt": "What to do in case of hair loss?",
"src": "Patient: Hi I m from India and female aged 25 years with 5 feet 8 inch height and wt 77 kg. I had a c section last year n had one normal delivery before this regular hair loss happened after three months n stopped by six months, in a market I was attracted by dove hair mask after which I have several hair loss, I had very thick beautiful hair earlier but loosed many now. I can more scalp peeping at me when I c the mirror. Can I use herbal shampoo now n oil ? Also can I use folli hair tablet his it safe I do not want to put on weight. Please advice Me. ThAnks Doctor: Hello and Welcome to \u2018Ask A Doctor\u2019 service. I have reviewed your query and here is my advice. Follihair can cause weight gain if taken long term. You need to be assessed for your problem to rule out the cause for thinning of hair. This includes blood work up, scalp and hair assessment. PRP treatments may help if you have no underlying problem. Get assessed by a dermatologist first and start right treatments. Hope I have answered your query. Let me know if I can assist you further."
},
{
"id": 94262,
"tgt": "Pain in stomach, back pain, swelling. Positive H pylori. Suggest? Medication?",
"src": "Patient: Hello, Over the last 10 or so months i have had this continous pain/ache/niggle left of my stomach . I used to work out alot and my diet was great, but when i started to have these problems i pretty much completely stopped going to the gym and my diet changed also. It first started as a small twitch on my left lower abdomen right over my belt line, it then progressed to unlocalized pain which differs from small area on waist line, left under rib cage and sometimes but not often about 2 inches left of my belly buttom. I also sometimes have a sore upper middle/ left back pain which sort of feels like i need to rub it to make it feel better. I have visisted my local GP over 8 or so times and have seen 4 docters and all of which tell me nothing is wrong and i have been given a 2 month supply of amytriptalyn (not sure of the spelling) which helped but then it came back. I was tested positive for HPylori and have taken 2 courses of this medication and the second was told was the strongest one and should have cleared it. I must admit that throughout the course of this there have been a few weeks where i have not really felt much. I wouldnt say i have any other side affects, i eat regularly, my weight is the same and i generally feel in good health other then feeling nervous and anxious about this all the time. Please help i just want to fix what ever i may have so i can get back to the gym and get fit again as im putting on weight due to my bad eating habits. I look forward to any advice and help. Many Thanks Luis Doctor: kindly get done upper GI endoscopy....biopsy and test for h pylori...if positive take HP Kit available in market...avoid eating outside..eat home cooked hyginic food and avoid drinking water outside.."
},
{
"id": 72597,
"tgt": "What causes painless pressure in diaphragm area while breathing?",
"src": "Patient: i have painless pressure in diaphragm area where my breathing is slightly strained. what gives? All this happened after i drunk alchohol months ago couldn t breathe and was rushed to the er. I had dyspnea then my stomache was always extended but that is getting better. YYYY@YYYY Doctor: Thanks for your question on Healthcare Magic.I can understand your concern. In my opinion, we should definitely rule out obstructive Airway disease (OAD) in your case for your symptoms.So consult pulmonologist and get done clinical examination of respiratory system and PFT (Pulmonary Function Test).If PFT is showing obstructive defect then you will mostly improve with inhaled bronchodilators (formoterol or salmeterol).Don't worry, you will be alright with all these. Avoid stress and tension, be relax and calm. Hope I have solved your query. I will be happy to help you further. Wish you good health. Thanks."
},
{
"id": 159803,
"tgt": "Doubtfull about cancer due to shadow shown in the lung x-ray",
"src": "Patient: My husband is 51 in very good healthnand has never smoked and works out all the time. He had his yearly Physical and had a chest x-ray and it came back showing a shadow on his lung. The Dr. has him going back for an MRI. His father died in his 50 s from cancer that started in his colon . Should I be concerned? Doctor: colon cancer have familial predisposition like breast cancer,. and shadow in lung in 51 year age could be cancer. i suggest that instead of MRI ,HRCT is best option for lung cancer diagnosis. Biopsy must be taken to give exact histology of disease(NSCLC Vs SCLC) either by broncoscopy(if lesion lies central) or CT Guided(if peripheral tumor)."
},
{
"id": 202911,
"tgt": "Gay man, had encounter with an HIV positive, no fluids exchanged, had oral sex. Chances of getting infected?",
"src": "Patient: Hi...I am a gay man and I had an encounter with an HIV positive person 20 days ago. The condom broke during penetration and i was not aware for a few min...I withdrew when I had a doubt and did not ejaculate and there were no fluids exchanged. However I did perform oral sex . Is there any chance of getting infected. When can I take the PCR qualitative test? Doctor: HelloThanks for your query,based on the facts that you have posted it appears that you had incomplete protected sex with a HIV positive person .Though you claim to use condom it can be considered protected sex as your condom was broken down during encounter.In such situation possibility of acquiring HIV can not be ruled out.Please get your screening test for HIV done now and repeat the same after 4 months to be rest assured about the possibility of acquiring the same.Dr.Patil."
},
{
"id": 62640,
"tgt": "What causes a lump in the throat?",
"src": "Patient: my husband has a lump on the inside of his throat. you can see it. it is size of a peanut and has a white spot on it. it doesn't go away and has been there awhile and has gotten bigger. he can feel it when he swallows. we are afraid it is some sort of throat or mouth cancer. went to dr. once bout it and they asked if he had insurance which he doesn't and they just brushed him off like it was no big deal. Doctor: Hi,Welcome to HCM.This could be tonsillitis with white phlegm mucus on it,or could be a lump,which needs Biopsy due to white spot on it.Check with private Surgeon or oral surgeon who would help you by needful check up n treatment.Hope this would help you to resolve your query.Else Contact with a Followup Premium question to ME.Will appreciate your Hitting thanks and writing excellent review comments to help needy patients like you. Welcome with any other further query in this regard.Good Day!!Dr.Savaskar,Senior Surgical SpecialistM.S.Genl-CVTS"
},
{
"id": 92047,
"tgt": "Have severe abdominal pain and diarrhea",
"src": "Patient: I had diarrhoea on monday morning with severe abdominal pain , I didnt have anything to eat and drunk water for days but abdominal pain has never gone , then when I tried to eat ust got diarrhoea again , cant keep nothing down havent been sick at all my pains are below my belly button if I press on my stomach on left hand it squelches and if feels like its bubbling , have back pain , and had bad rib pain b4 , I also take naproxen , morphine , paroxitine paracetamol and a stomach settler for those tablets but thats not moving my stomach pain thank you Doctor: Hi ! Good morning. I am Dr Shareef answering your query.Possibly you had an intestinal infection which was symptomatically relieved with your food restrictions, but the infection still persists as you did not take any medical treatment for the same. If I were your doctor, I would advise you for a stool test for ova and cysts, and a culture sensitivity of the stool if need be, and prescribe some drugs for you depending on your investigation reports. This should relieve you of your present problem. But you have to avoid eating from outside if you do.I hope this information would help you in discussing with your family physician/treating doctor in planning your treatment. Thanks for using the health care magic forum for your query on health. Wishing you an early recovery. Dr Shareef."
},
{
"id": 167098,
"tgt": "What could regular diarrhea with abnormal, foul smelling stool indicate?",
"src": "Patient: Hello my 2 1/2 year old often has diarrhea. His poos go from dark and sticky to sand and grainy to watery with undigested food. Its not uncommun for him to go 10 to 12 times during the week-end. We had him tested for celiac disease and his results came back negatif. We also tried to cut all milk products thinking that maybe he was lactose intolerant and then again the mustard smelling loose stool keeps on coming back. He recently had a grow spurr and is now back in the 50 th percentile and he is struggling to stay in the 10th percentile for weight. he currently weighs 26 pounds. Could you please help us figure out what might be wrong with our son. Thank you kindly Doctor: Hi...Thank you for consulting in Health Care magic. I understand your concern. I have a few questions and will suggest few possibilities.Questions:1. What was the birth weight and what is the current weight?2. How is the attaining of developmental milestones?3. Is the stool oily and floats in the pan?4. Is there any colour change in the hair or skin?5. Is there any history of recurrent serious infections?Possibilities:1. Chronic diarrhoea - ? Fat malabsorption2. Micro Nutrient deficiency3. Immunodeficiency either primary or acquired.Hope my answer was helpful for you. I am happy to help any time. Further clarifications and consultations on Health care magic are welcome. If you do not have any clarifications, you can close the discussion and rate the answer. Wish your kid good health.Regards - Dr. Sumanth MBBS., DCH., DNB (Paed).,"
},
{
"id": 213419,
"tgt": "Depressed, no social life, increased appetite, weight gain, feeling of worthless, hopeless, no self-confidence. Reason, treatment?",
"src": "Patient: Hello im 16 years old boy and i think i have a depression i dont have many friends and basicly i dont have social lifa i feel sad I love to play basketball but recently i have think about quiting . My appetite have increase and since im alone a lot I eat I have gained a 20kg ponds this summer and now im 110kg so i was 90 or even 85 i was in good shape now im fat lonley guy i feel worthless and even if my situation get a bit better when school starts I am afraid that same thing will happen to me next summer what should i do i feel hopeless and my self-confidence i at level 0 Doctor: Hi young man! There is nothing majorly wrong with you.You are basically overeating to overcome depression.Try to go out and mingle with people and make new friends.Indulge yourself in outdoor sports so that you end up making new friends.Try to exercise,meditate,read self-help and other books.I am sure you will overcome your problem if you follow this advise.But even after this you don't find any change in your problem than speak with your family and get a professional help by consulting a psychologist or a counsellor. I am sure you will be positive and overcome this transient problem.I hope this information is useful to you.I wish you all the best and a great future ahead. Dr.Prashanth"
},
{
"id": 134748,
"tgt": "What is the remedy for tingling sensation of feet after a fall?",
"src": "Patient: Hi, I stepped down from my steps outside about 6 weeks ago and rolled my ankle. The pain was severe when it happened, but I had no swelling. I just thought it was a sprain. It does not hurt to walk on now, but it is painful when I drive or bend my foot forward. Also, when my foot is resting my foot throbs and has a tingling sensation. Doctor: hithere may be minimal sprain of ankle ligaments and initial immobilization for 10 days would have been beneficial, anyway, now, see an orthopedic surgeon for advise, i may suggest wearing anklet strapping for 10 days of ankle and advil tabs for 5 days after meals thrice a day, hot fomentation and application of voltaren gel.All this may be undertaken if your doctor agrees and prescribes neccary treatment. An x ray also if need bethanks"
},
{
"id": 79225,
"tgt": "Suggest treatment for pain below the rib cage and chest pain",
"src": "Patient: I am diabetic. I have been having sever pain in my chest and under left ribs. I have seen a doctor. He has ruled out the heart. He has sent me to therapy for a dislocated sternum. It has not helped at all. He gave me pain meds and a neuropathy med. it does not seem to be getting better. This has been going on for 4 to 5 months. What is your thoughts on this. Doctor: Thanks for your question on Health Care Magic. I can understand your concern. Diabetic patients are at increased risk of lung Infection like pneumonia or pleural effusion or tuberculosis. And all these cause rib pain, chest pain etc. So better to get done chest x ray to rule out all these causes. If chest x ray is normal than no need to worry for lung related causes. Musculoskeletal pain is likely cause. So avoid heavyweight lifting and strenuous exercise. Avoid movements causing pain. Apply warm water pad on affected areas. Take good painkiller and muscle relaxant drugs. Also start vitamin b12 and pregabalin as diabetic patients have low vitamin b12 and nerve related pain. Don't worry, you will be alright. Hope I have solved your query. Wish you good health. Thanks."
},
{
"id": 62341,
"tgt": "What is the large lump behind my knee?",
"src": "Patient: I had an arthroscopy of my knee 3-1/2 weeks ago and a meniscal tear shaved off. In the last couple of days I have noted a lump behind my knee on the back side of my leg. It feels about the size of an egg. Could this be an infection, blood clot, Baker s cyst. It is warm to the touch. Doctor: Hi, dearI have gone through your question. I can understand your concern. You have history of meniscal injury. It can be somr hematoma formation or abscess or backer cyst. You should go for ultrasound study and if needed go for fine needle aspiration cytology. It will give you exact diagnosis. Then you should take treatment accordingly. Hope I have answered your question, if you have doubt then I will be happy to answer. Thanks for using health care magic. Wish you a very good health."
},
{
"id": 181554,
"tgt": "What causes pain in the gums even after teeth extraction?",
"src": "Patient: Hi, I just had a teeth extractions done for my bottom front teeth. I have gum disease and had a chornic infection for years. It's nearly a week after the procedure and mu gum is healing fine. However the infection doesnt seem to have changed at all. I am still feeling the same pain in the same area prior to my extraction which my dentist said should disappear as soon as the tooth is out. Can you help? Doctor: Current ratingDentist, Dr. Honey Arora's ResponseHi..Welcome to HEALTHCARE MAGIC..I have gone through your query and can understand your concerns..As per your complain pain after extraction of teeth is not an abnormal finding and if there is pain even when one week is also not over it is completely normal and you need not to worry about it as the pain will gradually subside when the healing will progress..As you had gum infection which is the cause for extraction you should take a course of antibiotics like Augmentin and ani-inflammatory painkillers like Ibuprofen..Do warm saline gargles and gargle with antimicrobial mouthwash like Chlorohexidine mouthwash..It will prevent sprouting of infection and help in faster healing..In case if the pain does not subside even after 8 to 10 days consult an Oral Surgeon and get evaluated..Hope this information helps..Thanks and regards.Dr.Honey Nandwani Arora"
},
{
"id": 52344,
"tgt": "How can sharp pain in the right upper quadrant after gallbladder removal be treated?",
"src": "Patient: Hello I am an 18 year old I\u2019ve had my gallbladder and my appendix removed the passed day I woke up dizzy with this sharp pain in my right upper quadrant with pain in one spot that rotates to my back and I went to the hospital and they did a cat scan and said that there were no stones and then they sent me home less than an hour I got home I had uncontrollable bowels and vomiting and when I drink something it burns in that spot that I am in pain what would u recommend doing I thing it is a kidney infection but no one will listen to me Doctor: Hi, This looks to me like so-called postcholecystectomy syndrome which has all these symptoms and this occurs in about 10 percent of patients who undergo cholecystectomy, usually can be transient, and rarely persistent or lifelong. Symptoms include systemic symptoms such as sweating and heat sense but also dyspepsia, nausea, and vomiting, flatulence, bloating, and diarrhea, Persistent pain in the upper right abdomen. Some individuals may benefit from diet modification, such as a reduced fat diet, following cholecystectomy since it may be more difficult for digestion of fatty foods. Postcholecystectomy syndrome treatment depends on the identified violations that led to it. Typically, the patient is recommended dietary restriction table with fatty foods, enzyme preparations, antispasmodics, sometimes oral ursodeoxycholic acid can alleviate the condition. Hope I have answered your query. Let me know if I can assist you further. Take care Regards, Dr Ivan R. Rommstein, General Surgeon"
},
{
"id": 102410,
"tgt": "What tests can determine which constituent of meats (bacon, ham) cause allergic reaction?",
"src": "Patient: I have an allergy to bacon. When I eat bacon or country ham I get flushed real red skin nausea, lips get numb, extreme pain in my gut, I through up , and diarrhea. My temp drops to 94 , bp goes down. I have passed out. Is there a test that can be done to see if the Nitrates or Nitrites in bacon or other meats causes this. My doctor has prescribed an epi pen. Doctor: Hello.There are specific tests to determine which constituent of meats (bacon, ham) cause allergic reaction.You must go to the allergy specialist and he will make the mentioned tests.Epinephrine is used in severe allergic reactions.In mild allergic reactions, antihistamines as Ebastine 10 mg twice a day can be taken.Please consult your local doctor for a prescription.I wish you good health.(If the answer has helped you, please indicate this)"
},
{
"id": 27580,
"tgt": "Is Stamlobeta intake safe while suffering from BP?",
"src": "Patient: i am taking stamlobeta and do regular exercises and yoga. my bp is always under control.is it safe to continue with stamlobeta or should my tablet be changed to much milder ones. can i do yoga? i am very fit and in fact a cricketer and i am 49 years old now. please guide. Doctor: Ya it's a good drug for your blood pressure, if you say your blood pressure is being controlled, I wouldn't advice to change It, though newer and milder meds are available. Also you should never stop your meds suddenly as it will shoot your pressure to higher limits, which is not good for your heart, brain or kidney. I advice you to continue the drug and titrate the dosage according to blood pressure reading s"
},
{
"id": 40977,
"tgt": "Suggest treatment for infertility due to blocked folipian tubes",
"src": "Patient: Hi I am a 22 year old female, 5'-5\" and weigh 150 pounds. I live in Liberia. In 2008 I found out i was two weeks pregnant and i had a medical abortion using Cytotec. I inserted the tablets myself. I can't remember the dosage but i had heavy bleeding for 10 days before i went to the doctor as it was adviced to do a D&C after. I was given the D&C which was successful (but not a great experience because no anesthetics was used). One week after that my x-ray (HSG) in my follow up appointment showed that both my tubes were blocked. My doctor advised me to \"blow my tubes\" or take antibiotics for the first three days of my periods and i will be okay.But it has been over two years now and no results. I want to get pregnant now and want to know the best option for my tropical African location or will i be barren forever. Also I am curious about the procedure of \"blowing my tubes\" and whether it can really work. Please help me. thank you. Doctor: Blocked fallopian tubes contribute to almost 40% of infertility cases. It\u2019s common that only one of the fallopian tubes is blocked with the other functioning normally. Women with blocked fallopian tubes may just undergo test until she finds herself of infertility since it may be accompanied by only mild symptoms. Fortunately, most of the cases of blocked fallopian tubes are reversible and can be cured by some medications along with some natural remedies.herbal medicines like Fuyan Pill. However, there are also certain natural remedies that can be used to help cure this problem without taking surgical procedures and having medicines. Natural Remedies for Blocked Fallopian Tubes1. Try meditationMeditation can help reduce your stress level which promotes healing and overall good body health in turn.2. Take up yoga3. Quit smoking and drinking4. Consider fertility massage5. Avoid food containing hormones"
},
{
"id": 132128,
"tgt": "Is tingling feeling normal after broken bones?",
"src": "Patient: Hi, Umm i recently gotten hurt at work and sent to the ER. They did xrays no broken bones but they said i have a contusion on the top of my hand where i had it smashed at. It has gotten progressivily worse over the last 48 hrs sending pains down to my elbow and tingling feeling that wont go away. Normal? Doctor: hi i have read your description of injury and now it is causing pain uotp elbow and tingling sensation . your x rays are normal. It is quite possible that you have soft tissue injury at the wrist causing swelling inside the wrist joint. In my clinic I would advise , splintage ( Immobilization of wrist ) to reduce further damage to soft tissues due to movement , prescribe pain killers and rest ( For The affected hand) for 2-3 weeks."
},
{
"id": 51728,
"tgt": "What should I do to strengthen Pubococcygeus muscles ? And suggest me remedy for acidity",
"src": "Patient: Sir, My name is Shaka Phthaka and I am 20 years old.I have weak Pubococcygeus muscles,what should I do to strengthen these muscles?Which diet should I choose to strengthen these muscle and what exercise should I do?I cannot do kegel exercise because it causes me constipation and same stomach gas problem.What should I do to fight with these disease and my stomach has also acidity and what should I do to reduce my acidity?Please answer my question as soon as possible. Doctor: Of course kegel is NOT the good excercise, he/she just said that it causes them problems!"
},
{
"id": 28257,
"tgt": "Need medication for rapid heart beat",
"src": "Patient: My heart just started beating fast now while I was standing up. I sat down, it couldn t stop beating fast. Then it started hurting so bad that I couldn t stand up. It is still hurting right now but less than before. It is my first time experiencing this and I have to leave for a hiking trip for my physical education class. What is wrong with me? I don t feel well at all now. Should I consult my teacher and tell him that I am unable to go to the trip because I am not feeling well at all anymore. I don t know what to do. This is so random. Doctor: Hi,After going through your case I would like to advise you that report everything happened with you to your teacher, and get medical examination done.I want to know-1) Whether you really want to go to trip.2) Any kind of mental tension is there.3) Any heart problem in your family.In your case there are following possibilities-1) Occasional rapid heart beats in a healthy adult in the absence of giddiness, sudden black outs is most of the time normal.2) Anxiety / panic episode though its possibility is less as it is your first episode.3) Cardiac arrhythmia (abnormal electrical activity of heart)If it happened repeatedly then it is better to consult cardiologist and get the following tests done - 1) ECG done during period of fast heart rate it can give more information.2) Holter's monitoring or electrophysiological test for heart.3) 2DEcho (echocardiography of heart)4) Serum electrolytes, renal function test.5) Thyroid function test."
},
{
"id": 115434,
"tgt": "What causes low alkaline phosphatase level in blood?",
"src": "Patient: my husband had colon resection 10 years ago due to tumor. lately having abdominal pain, vomiting, non alcohol drinker. lab test show alkaline phosphatase is low at 49. mch is a tad high at 31.7. glucose high as well. should we be terribly concerned regarding the alkaline phosphatase low level? if so, what concerns? Doctor: Hi. Do not worry for the decreased alkaline phosphatase levels. Decreased are not associated with disease. He should be treated for high glucose. For complaints of vomiting, you should contact your family doctor. As he had history of cancer, he should be regularly under follow up of his treating doctor. Take care"
},
{
"id": 52597,
"tgt": "What does this ALT of 41 indicate?",
"src": "Patient: my husband alt level went from 39 in 1/17 to 41 in 2/17 and 56 in 3/17. Between 2/17 and 3/17 he had 30 days without drinking. Now they have ordered other test and said he is abnormal but stable. Should we worry? One of his newer test ordered Ferritin came out at 444 which is high. Doctor: Hello and Welcome to \u2018Ask A Doctor\u2019 service. I have reviewed your query and here is my advice. There is nothing to much worry in this liver function test. In alcoholic liver diseases the liver enzymes will be in thousands. Ferritin is little bit high and it might be due to iron over load. As of now nothing to worry now. Wishing all the best"
},
{
"id": 214389,
"tgt": "Suggest treatment for thyroid nodule",
"src": "Patient: Yes, Hello, I have had a thyroid nodule for years. How can I treat it naturally ? I started taking Shilajit Fulvic Acid Complex a couple of days ago.. 250 mg a pill. I take 2 a day.. Can you advise me on anything about helping to shrink the nodule ? Doctor: Medicines made from kaachnaar are used to treat thyroid nodules. But thyroid function is essential to be checked. You may be having hypothyroidism or hyperthyroidism in background."
},
{
"id": 125860,
"tgt": "How can constant neck pain be treated?",
"src": "Patient: My new pain Mgmt doctor changed my medication removing me from taking tremodal 50 mgs daily and one flexil muscle relaxer daily to now taking 10/325 Norco (1 every 6 hrs for pain as needed) and the muscle relaxer Lorzone 750 mgs. 1/3 in AM and 2/3s at nite. After taking the new megication for 3 weeks I find that the tremodal gave me more relief. I want to go back to taking tremodal and only take 1/3 of the Lorzone can only be taken in 1:3 Dias ages or even 2/3 dosages. I really want to see if I can do better in dealing with my neck pain with less pain medication. I have constant pain on the left side of my neck. My MRI shows that I have foraminal stenosis and facet degenerative changes on the left side of neck. Doctor: Hello, Consult an orthopedician and plan for an MRI neck to rule out possible spine related problem. Hope I have answered your query. Let me know if I can assist you further. Regards, Dr. Shinas Hussain, General & Family Physician"
},
{
"id": 207473,
"tgt": "How to get rid of depression?",
"src": "Patient: I have had folliculitis directly after I was assaulted and now I am suffering from PTSD and Depression my Dr says that it can be caused by stress and anxiety it seems to flare up when I am more so and I cant get rid of it I am seeking I have tried antibiotics , I have no fungus or infection tested 3 times I am getting desperate of a solution to get rid of it please. I AM GETTING STRESSED AS I CANT GET RID OF IT AND ITS GETTING VERY PAINFUL ... HELP THANKS JULIE Doctor: HelloThanks for using Health Care Magic for posting your query.I have gone through your question in detail and I can understand what you are going through.You need to keep seeking dermatological help. Meanwhile you may spend some time in creative activities such as drawing to keep you out of this stress. Hope this helps,If you have any further query, I would like to help you on the link:bit.ly/dr-srikanth-reddyWish you good health,Kind regardsDr. Srikanth Reddy"
},
{
"id": 160629,
"tgt": "What causes pain in chest?",
"src": "Patient: My 4 year old daughter has had red cheeks today. Around dinner she was complaining that her chest hurt. She says it feels like wood is in there. She does not have a fever and her pulse seems to be normal. What is this, and do I need to take her to the ER? Doctor: Hi, This can suggest a viral prodrome. Sometimes rash and body aches precedes fever. If she is otherwise active and playful, nothing to worry. Just give paracetamol three times a day for 2-3 days. This will help chest pain, and fever incase it appears. You need to take her to doctor if the pain increases, other symptoms adds up or fever persists beyond 3 days. Hope I have answered your query. Let me know if I can assist you further. Take care Regards, Dr. Muhammed Aslam TK"
},
{
"id": 224469,
"tgt": "Is iud causing back pain and stomach cramps?",
"src": "Patient: Hi, I had an emergency c section about 1 year ago. I have an iud, and last month i had my first with normal bleeding. I have had a lot of lower back pain off and on, and stomach cramps. I would have thought I was starting my period again, or pregnant. I took a test and it was negative. Now, I had another period, and it had normal period cramps. My period lasted longer then most. I finished my period about 5 days ago, and now I am having bad back aches and my hips ache and also dull pain in my lower stomach. Do you have any ideas? THANKS!I am 28, I have 2 kids, I weigh about 115 and I'm about 5'4\" Doctor: Hi, thanks for writing..Initial 3 cycles with IUD will cause cramping, slightly more bleeding than normal and back pain. it will reduce and get normal by 3 cycles. But if the pain is persisting, you can get IUD removed and take alternate contraceptive measures. .Hope I have answered your query. Feel free to ask any doubts through http://doctor.healthcaremagic.com/doctors/dr-dalia-muraleedharan/68596. Good day"
},
{
"id": 214086,
"tgt": "I was kicked in the head. I am also bipolar schizophrenic",
"src": "Patient: I was kicked in the head a few times not long ago. My face and forehead have bruises, swelling, and its tender. I am also bipolar schizophrenic. Should I be worried? Doctor: if u have loss of consciosness, forgetfulness, internal nose or ear bleed or forgetfulness then contact neuorologist immediately or else it is fine.. no probs.,."
},
{
"id": 72968,
"tgt": "What could tingling in chest/arms with nausea and warmness suggest?",
"src": "Patient: I am a 25 year old female. I woke up in the middle of the night last night with tingling in my arms and chest, severe nausea and dizziness. My chest felt tight, I felt really hot, and almost as if I couldnt breathe. I do have reactive hypoglycemia, but I never had an episode like that. It lasted only several minutes and I feel fine this morning. I am afraid to go to the doctors because I do not want them to think I am crazy because I am fine now, but I am really afraid of this happening again. What do you think it could be? Doctor: Thanks for your question on Healthcare Magic.I can understand your concern. Possibility of acute gastritis is more likely in your case. No need to worry for cardiac diseases because you are very young. So avoid hot and spicy food. Avoid junk food. Avoid large meals, instead take frequent small meals. Take pantoprazole and levosulperide combination on empty stomach twice daily. Quit smoking and alcohol if you have these habits. Go for walking after meals. Avoid stress and tension, be relax and calm. Keep 2-3 pillows under head in your bed.Avoid stress and tension, be relax and calm. Don't worry, you will be alright with all these. Hope I have solved your query. I will be happy to help you further. Wish you good health. Thanks."
},
{
"id": 156265,
"tgt": "Is breast cancer hereditary?",
"src": "Patient: I am 52, 5 ft tall, Female, in good health, no high BP, no high blood sugers. This past week I have been having sensations in my right breast. Not pain or tingling, just an awareness that my breast is there. Both sisters have had cysts in breast, nothing to it for them. Doctor: Hi and welcome to HCM Thanks for the query. we cant say that it is hereditary. we can say that risk of caner is increased in wolen who had cancer in family and this risk can be calcualted if you do certain genetic testsWish you good health. Regards"
},
{
"id": 220999,
"tgt": "What are the chances of pregnancy through non-penetrative intercourse?",
"src": "Patient: Hello. I have a question. Last week I had unprotected intercourse with my boyfriend, but just at the beginning, he didn t finish... It was 3 days after my menstruation stopped. It wasn t the first time we did that, but he told me to do a pregnancy test. Do you think I should be stressed about it? Doctor: HI, Thanks for the query. I understand your concern. You had sex about 8th to 9th day of your menstrual cycle..which is not likely to be in fertile days in normal 30 days menstrual cycle. so there is less possibility of pregnancy with the sex you are referring to- & you need not stress (unless you have a very short/ regular cycle of 22 to 24 days. ) In case of short cycle, considering possibility of pre ejaculate deposited in vagina .. you have to exclude pregnancy by urine pregnancy test by HPT after 8 days delay in expected date of coming period. Thanks."
},
{
"id": 138597,
"tgt": "What causes burning sensation on bottom of skull?",
"src": "Patient: Bottom left half of my skull and some of left half of neck keeps having waves of burning flushes - either like burning or like plunged in ice water (I can t tell if it feels extremely hot or extremely cold). I have also recently had loose stools today and stomach cramp but this head sensation happened a few days ago before that began too. It sometimes spreads upwards towards the top/left back of my skull but mostly radiating from bottom left near neck. Doctor: Hello,Welcome to the magical world of health care, I went through your query, and in my understanding of your symptoms, these are due to neural causes, I would advice you to see a neurologist at the earliest.I hope my advice would have been useful, in decision making regarding your treatment, still if you have any clarifications or doubts feel free to contact back.I hope a 5 star rating if you feel guided in your treatment,Thanks"
},
{
"id": 43773,
"tgt": "Trying to conceive. Semen analysis shows low motility. Taking testocaps. How to improve motility?",
"src": "Patient: Hello Dr, I am xxxxx and we have been trying for child for last 13 months but my wife didn t get pregnant. We have gone trough all the tested as Doctor suggested, and she is fine but Dr told me to do semen analysis . Before three month the analysis resulted 28 mil/ml sperm and having motility as 15%. And to increase motility Dr suggested me to take testocaps for three month. But recent analysis shows that still there is huge lack of motility. Please suggest any other medicine so that i can improve my sperm motility . Thanks Praveen Doctor: Hello, Welcome to HCM, I am Dr. Das look, your sperm count is normal. But, motility is normal. You need coenzyme Q, vitamin E supplements. That are only helpful in this situation. Consult with an infertility specialist for further management."
},
{
"id": 34512,
"tgt": "What to do for the infection which is spreading throughout the body?",
"src": "Patient: Hello doctor, my mother has an infection and now the infection has spread through her body and this infection was caused by her an ear infection she had earlier on this year. The doctor will only be able to treat her in October 20. She is really tired and weak, please help, we really cannot wait that long because we dont know whats going to happen. Please help me doctor, I really would like my mother to get well as soon as possoble. Thanks. From: Mpho. Doctor: Hi, I understand your concern. General weakness, lac of resistance & probably uncontrolled diabetes is basic cause of getting repeated infections through out body . You need to get your mother investigated to find basic status of her health. Get her through physical examination done by a physician to find probable diagnosis & get it investigated along with- - Complete blood count, Mx. test, - Urine -microscopic & routine , _ fasting & post meal blood sugar test/ glucose tolerance test, - culture & sensitivity teat ( from the exudate/ tissue currently infected) to find the bacteria & the effective drug, Consult an expert physician who will advise her specific treatment according to the reports. These need to be accompanied by general majors like - good hygiene/ cleanliness - light nourishing diet with high proteins, vityamins, minerals, iron- as per doctor's advise. - Rest, mild exercises once she is settled, - POsitive thinking of the patient induced by love, assurance, nursing . I am sure you mother will recover with methodical management as described above. Thanks."
},
{
"id": 214616,
"tgt": "Suggest an home remedy to resume delayed period",
"src": "Patient: hi last month my persiods were start on 18 novemeberbut this month 18 th date went but i didnot get periods this month . i mean its 02 days more.so could you please tel me any hoem made reasons to bring periods soon.pls reply urgent i need ur help pls Doctor: a decoction[kwath ] of black til and black jeera with jaggery [gud] mixed in it should be taken both morning and evening.also eating of ripe papaya will help."
},
{
"id": 51162,
"tgt": "Excruciating pain in the lower back while urinating after controlling it for a long time. Suggestions?",
"src": "Patient: Once in July while flying & just now in my home I held my urine longer than normal, then when I went to release I had excruciating lower left back pain when urinating. The first time it was so bad it sent me to my knees, which was difficult since i was in an airplane laboratory. That first time I prayed for my death instead of healing it was so bad. it may have lasted 5 minutes & then continued a while longer a mild discomfort The next time was just a few minutes ago it was not as bad. What is wrong with me? Whom do I see for a repair? The email address provide is mine since 1998 and is legitimate so please put forth the effort to respond, Thank you, Male 42 years old with possible kidney issue Doctor: Hi. If you have pain in left lower back while passing urine it could be a symptom of a stone in your kidneys or ureter. The best thing to do would be to obtain a urine analysis and an ultrasound of your kidneys. You will also benefit from some pain relief medications which any family doctor would be able to prescribe. Good luck."
},
{
"id": 180942,
"tgt": "Can Flucloxacillin be taken for severe tooth pain post root canal treatment?",
"src": "Patient: i had root canal treatment 2 weeks ago(2 more visits due) i have had mild pain on and off (plus whenever i have a hot or cold drink): but tonight i have started with a throbbing pain and it is very sore.I have some flucloxacillin in the house - should i take these? (all the emergency dental practices are closed) Doctor: Hi..Thanks for the query.It looks like re-infection in your root canal treated tooth..Soreness and pain is due to Infection and pus formation below the root tips of the tooth..Taking Fluconazole can help but first of all an x ray should be done to rule out the exact condition of the tooth.As antibiotics are prescription medications, therefore you should consult an Oral Physician and get evaluated..As of now you can take painkillers to reduce pain.Do cool compresses over the cheek..Hope this helps..Regards.."
},
{
"id": 178578,
"tgt": "What causes ear canal bleeding?",
"src": "Patient: Hello, so i was cleaning my sons ear tonight with wet q-tips and of course there was a lot of build up because he doesnt let me clean them often maybe once. A month or two, but as i was cleaning i happen to see pink spots on the q-tip (blood) immediately stopped! Should i be concerned? Should i take him to the doc? Doctor: Thanks for asking on healthcaremagic. Ear wax is a substance produced by the ear canal and helps in keeping it lubricated. It comes out on its own and you need not clear it periodically unless there are specific problems. Certain drops like Ceruklean can be helpful if required. While cleaning you might have inadvertently caused some injury and it would be better to get it examined by a ENT doctor.Hope that answers your query. Feel free to revert back with further queries if any."
},
{
"id": 191421,
"tgt": "How can diabetes along with excessive weight gain be treated?",
"src": "Patient: I am 69 female Am Type 1 diabetic and have a 25 year history of thyroid nodules. Had 2 negative biopsies and am carefully watched by an endocrinologist. One plus year ago I was diagnosed with ahashimotos and am on Levoxyl Anyway I have multiple issues but would like some suggestions about the best way to help myself with foods for thyroid problems including ridiculous weight gain. I gained 30 lbs. in a very short time.I went from 125 to 155 in only 3 months.It is causing other medical problems and blood sugars which seem to and are out of control. Anyway,I m looking for suggestions and will appreciate your input.Thank you YYYY@YYYY e Doctor: Hello, I would like to tell you since both your disease are autoimmune thus there are lot more chances of you to have more chances of other problems which are autoimmune so in order to reduce your weight you required to follow few things # Take a diet rich in iodine, zinc, and selenium, for example, you can start taking iodized salt and even some multivitamin tablet which is rich in them# You need to do weight loss for that reason considering your age you can do some yoga activity at home itself# In the food you need to avoid the goitrogens like cabbage, kale etcetera.Hope I have answered your query. Let me know if I can assist you further.Regards, Dr. Medhavi Agarwal"
},
{
"id": 130501,
"tgt": "What causes pain and popping noise every time I lift leg up/walking?",
"src": "Patient: My question is : I was diagnosed with pelvic symphasis when i was pregnant but it went away after the baby was born this was also 2 1/2 years ago and i also had two disc logge out of my left side of back.. but my question is every time i lift my right leg up or walking up a hill on right side .. it is pain full and i feel and hear a popping noise? should i get that check or is that normal? thanks desiree Doctor: Hi,No it is not normal. If you had pubic symphysis dysfunction during pregnancy then it generally resolves post delivery but in some cases it doesn't. If would be good if you get it assessed as the pain and popping noise signifies either a hip or pelvis problem.Hope this helps. Let me know if I can assist you further. Regards,Jenis Bhalavat"
},
{
"id": 43971,
"tgt": "Done sperm analysis. How is the potential for pregnancy?",
"src": "Patient: Dear Dr Candan. I would like to ask you you how you read the following values for my semen? How is the potential for using these values of semen to make an egg pregnant : Volume 1.5 Sperm concentration (mill/mL) 38 Rapid linear progressive (A) 00,0 Slow/ Non-linear progressive (B) 09,0 Non-progressive 08,0 Non-motile 83,0 Doctor: Hello and welcome to HCM The analysis for your semen is as follows: Volume is 1.5 ml/ ejaculate- This is low volume as the normal volume per ejaculate should be atleast 2 ml Sperm count is 38 milliom/ml- This is normal sperm counts. The cut off for normal sperm count is atleast 20 million/ ml Sperm motility is less than the normal range. Non-motile sperms are 83 %. Normally atleast 50 % sperms should have rapid progressive forward motion. There is no comment on morphology and presence/ absence of pus cells, RBCs and other nucleated cells. On the basis of this report your sample is deficient in adequate volume and adequate volume. It can lead to infertility i.e. inability to fertilize an egg under physiological conditions. However, I recommend you to get semen analysis repeated after three days of abstinance. If, again the sperm count remains low, there are other techniques for fertilization of the ovum. You can get details of which from a fertility expert. Thanks and take care Dr Shailja P Wahal"
},
{
"id": 52340,
"tgt": "Suggest treatment for varices of the liver",
"src": "Patient: Hi, I have recently been diagnosed with varices of the liver due to alcoholism and Hepatitis C. They said Im in stage 2 and will not refer me to the treatment. Should I contact a specialist and what can I do to get the treatment. I do not want to wait for stage 3 to be refered. I only carry medicaid. Doctor: Hello, You have varices due to cirrhosis of the liver. For varices sclerotherapy or bang ligation are some treatment option. You can Consult hepatobiliary or gastro surgeon for your detailed evaluation and management. Hope I have answered your query. Let me know if I can assist you further. Take care Regards, Dr Parth Goswami, General & Family Physician"
},
{
"id": 196629,
"tgt": "What causes swelling on face?",
"src": "Patient: My husband, 29, went to the doctor today for his face being swollen, to which i assumed was from the cold he s been fighting for a couple of days. He told me the doctor felt his glands and said something along the lines of she had not seen such swollen glands only on stroke patients. His father died late 50s early 60s from a stroke. Makes me concerned, suggestions? Doctor: Hi,The most common causes of swelling over face is angioedema and HYPOPROTEINEMIA. excessive steroid intake can also cause moon face. Cushing syndrome and also present like this. Hope it helps. If you have any other question please do not hesitate to contact us.Regards,Dr. Atishay Bukharia"
},
{
"id": 18096,
"tgt": "Are low blood pressure and heart rate levels after a mild stroke a matter of concern?",
"src": "Patient: my mother is 90 years old and recently had a very mild stroke. We just got her home 2 days ago from the hospital. I just checked her BP and is was 95/65 with a heart rate of 53. Is this cause for alarm. I don t know what s normal for a 90 year old. Doctor: Hello!Welcome on 'Ask a Doctor'!Regarding your concern, I would explain that her actual blood pressure values and heart rate values are a little low, but as long as she does not feel lightheaded or weak, there is nothing to worry about. Increasing salt and water intake can help improve her situation. If she is taking any betablockers or anti-hypertensive drugs, I would recommend trying to reduce the doses. Hope to have been helpful!I remain at your disposal for any further questions whenever you need!Kind regards, Dr. Iliri"
},
{
"id": 104673,
"tgt": "Allergy test center in Bangalore",
"src": "Patient: Hello sir i have a problem of asthaema.. even i have allergy of many things... following problem always remain with me (cold, tears in eyes (due to cold), cough, underweight ) i want to k ow what things i have of allergies. can u suggest me good and cheap allergy testing lab in bangalore... which all the test should i do...... Doctor: Welcome to Healthcare Magic You could be allergic to pollen, dust, smoke and probably certain food items as well. Use handkerchief when passing through these areas. You should consult an ENT Surgeon to evaluate if you have allergic rhinitis. There is good medicine to prevent further attacks. You can try taking Cetrizine at night for relief. Apply nasal decongestant drops to relieve from cold as well. Keep warm. Avoid eating cold things. For eyes, you should see Ophthalmologist and probably certain drops will be prescribed which will prevent the watering. There is no cure for these problems only way is to prevent them. You should go for Allergy prick testing at Manipal Hospital in Bangalore to find out what you are allergic to and they will also help sensitise you to them so that you won;t get severe reaction next time."
},
{
"id": 198391,
"tgt": "Suggest treatment for testicular swelling",
"src": "Patient: Sir i AM a CA Satnam Singh here ........last monday i have done my varicocele surgery on my left side ,.....now there is some swelling in my left testis ........size of my testis is increased....what are other problems which i can faced in these intial days Doctor: HelloThanks for query You have undergone surgery for Varicocele and developed swelling of the testicle .It is common to get testicular swelling after surgery and is due to handling of testicle during surgery(surgical trauma ) and tissue reaction . ..Normally it gets resolved over a 2-3 weeks You need to take broad spectrum antibiotic like Cefixime along with anti inflammatory drug like Diclofenac twice daily .Ensure to rink more water .Using Scrotal support will help to get relief from pain .Avoid sexual activities till it gets cured completely .Dr.Patil."
},
{
"id": 90854,
"tgt": "What causes severe abdominal pain, back pain and green colored urine?",
"src": "Patient: my brother in law has had severe abdominal pain for a week now. he says it feels like he had something explode in his belly. It even hurts into his back causeing back pain. He also has green urine.... We have all told him to go to ER. what could this be? Doctor: Hi and welcome to HCM. Thanks for the query. there are several possible causes but it may indicate biliary or pancreatiy story and most commonly this is caused by biliary stones which could stuck in biliary or pancreatic duct. there are some more serious causes but it should still be evaluated by gastroenterologist.Wish you good health. Regards"
},
{
"id": 28982,
"tgt": "Can Tylenol be taken for fever?",
"src": "Patient: my husband went to doctor today with bad cough & was told he had a virus--at the time he did not have a fever. He got prescription for Promethazine-codeine 6.25-10mg/5ml syp. He has been running a fever all night-latest was 102.5... is it ok to give him Tylenol for the fever? He s drinking lots of water & chicken soup, but I don t like the fever. Doctor: Hello,Tylenol drug can be given every 8 hours for fever. Tylenol should be avoided, if he has previous history of allergy to the medication or liver dysfunction.Hope I have answered your query. Let me know if I can assist you further.Regards,Dr. Lekshmi Rita Venugopal"
},
{
"id": 33129,
"tgt": "What causes high fever with fainting inspite of clear test reports?",
"src": "Patient: 12 yr old super fit girl having high temp 39+ for past five days. Many bloods done, both while having fever and not, chest xray, sugar levels, MRI, urine testing, throat swab, spit swab, nothing at all wrong. So do you have any idea what else could be causing this....in first two days she also had 4 fainting episodes, first one she was out for ten min and not responding to questions for another 5 minutes after waking up. Doctor: HelloAs you mentioned that all tests are normal , so hormonal imbalance is the main cause ( because her age is 12 years and puberty may the reason i.e. hormonal change and this is normal , not a cause of concern ).2nd reason of such cases may be stress or anxiety or tension ( may be of study ) .So in my opinion there is not a cause of concern and don't require any further investigation .Wait for another 4-5 days but if still fever persist then consult a gynecologist & physician and get their opinion.Hope this will be useful for you."
},
{
"id": 135581,
"tgt": "Suggest remedy for swelling in ankle post injury",
"src": "Patient: My wife, 76, turned her ankle getting out of her chair at a park. I plied an ice pack out of our cooler & returned home within 3/4 of an hour & applied ice packs for the past 2 hours .she has an egg size swelling on the outside of her ankle bone with considerable bruising Doctor: HiThanks for your queryAfter going through your query I came to know that you had injury ankle while playing volleyball and fell while spiking the ball. It seems to me soft tissue injury (sprain). X-ray may be done to exclude fracture. Putting ice is OK . You should not use it till it pains. Splint can be used to rest it. Leg should be keep elevated on pillow to reduce swelling. Analgesic such as diclofenac will be of use for pain. You can take vitamin B-complex and C to promote healing. You can discuss with your treating Doctor about it. Hope your query get answered. If you have any further questions then don't hesitate to writing to us . I will be happy to help you.You can also write to me directly on below link:https://www.bit.ly/askdrsudhirorthoWishing you good health.Take care."
},
{
"id": 3124,
"tgt": "How to find date of conception?",
"src": "Patient: Hello, I started my period on June 4th and ended on the 8th and I had sex with one guy that night. On the 11th of June I had sex with another guy but he put on a condom 5 minutes after we started to keep from nutting inside of me. I had sex with the first guy again on June 15th. I later had sex with the same guy that whole next week. I missed my period for July & when I took a pregnancy test on July 6th I was pregnant. I went to the doctor and they said I was approximately 8 weeks pregnant. My due date was March 7th 2015 but when I went to get an ultrasound on August 11th my doctor told me my due date was now March 11th 2015 and I conceived between June 13-15th. So who is the father? I believe I got pregnant on July 18th because of my first due date on March 11th. I just think my baby is growing fast so therefore the CRL measurements of my baby made the doctor change my due date to March 7th. Doctor: Hello, and I hope I can help you today.Unfortunately, ultrasound findings of the dating the pregnancy can be off for up to five days. So the changing of your due date by four days does not really change the possibility of your first partner having been the father of your baby. Furthermore, sperm can remain alive in your genital tract for up to five days, and once you ovulate, the egg remains alive inside you for up to 72 hours. This is the reason that no test can accurately assess the date of conception based on the exact dates that you had intercourse. Women typically ovulate two weeks before their period arrives. If you have a 28 day cycle, and your last period started on June 4, you likely ovulated around June 18. So either of the men you had sex with between the 15th and approximately the 20th could be the father of your baby.I hope that this information was helpful to you and that I was able to adequately answer your question even though it is not possible with medical science to determine the paternity of your pregnancy. I wish you the best of luck with your pregnancy and good health.Best wishes,Dr. Brown"
},
{
"id": 24648,
"tgt": "Can I continue smoking along with High BP medication?",
"src": "Patient: can i still countinue to smoke pot when i just stared a medication for high blood pressure ive never had high blood pressure before but i just got orthoscopic surgery and the high blood pressure stared ive smoked for over 20 years can still smoke or lay off it for awhile Doctor: Hello and thank you for using HCM.I carefully read your question and I understand your concern.I will try to explain you something and give you my opinion.You should know that smoking is considered a cardiovascular risk factor together with hypertension, high cholesterol, diabetes and hereditary history for cardiovascular disease.So, you don't mention your age, but you are diagnosed now whith hypertension.If you continue to smoke you add to your self another risk factor.There are thinks that doesn't depend on you like hypertension, hereditary but you can control smoking, its on your hand.So, of course you should stop smoking becouse doing so you prevent your self from adverse cardiovascular event caused by smoking.I encourage you to do so, becouse this is the best think for you.Hope I was helpfull.Best regards, Dr.Ervina."
},
{
"id": 112651,
"tgt": "Severe back pain, difficulty in sleeping, severe pain while standing. Remedy?",
"src": "Patient: hi i have severe back pain which runs down to my ankle,i had an exray plus m,r,i nothing showed up, i cant sleep at night ihave to walk with crutches go down stairs on my bottom i feel no pain when i am stooped almost to the ground when i stand up straight the pain is excrusiating and a pull from my right leg numb r big toe Doctor: Hi,Thanks for posting your query.From description it seems you might be suffering from siatica. Sciatica refers to pain that radiates along the path of the sciatic nerve \u2014 which branches from your lower back through your hips and buttocks and down each leg. Typically, sciatica affects only one side of your body. Although the pain associated with sciatica can be severe, most cases resolve with just conservative treatments in a few weeks.MRI is needed to establish the cause of sciatica, so that appropriate treatment can be prescribed. Sciatica do respond well to bed rest and analgesics (pain killer) and is right mode of treatment. The definitive treatment depends upon the cause of sciatica. I would suggest you to consult an orthopediac surgeon who may order MRI to determine cause of sciatica (level and degree in case of disc herniation) and may suggest other mode of treatment. After initial few days of bed rest, you needs physiotherapy (exercises).Hope this will helps you. Feel free to ask me if you have any further queries. Wish you good health. Take care.Regards.Dr Saurabh Gupta.Orthopaedic Surgeon."
},
{
"id": 193836,
"tgt": "Suggest treatment for erection problems during intercourse",
"src": "Patient: Hi, may I answer your health queries right now ? Please type your query here... hello sir my age is 27. height is 5,9 , i want some suggession from u i will be v.thankful to u. during sex some time when my attention diverts then my penis not stayed tight. then i feel ashamed infront of my gf. tell me how i can stay my penis tight during sex? plz solve this prob. thanks Doctor: Hi, These include: Physical problems, such as injury to nerves or loss of blood supply to the penis. Other health problems, such as diabetes, high blood pressure, anxiety, and depression. Side effects of certain medicines. Stress. Drinking too much alcohol or smoking. The exam, lab tests, and sometimes mental health tests can help find out the cause of the problem. Treatment is based on cause and includes avoiding tobacco and drugs and limiting alcohol. Also, talking about the issue with your partner, doing sensual exercises, and getting counseling may be beneficial. Finding and then stopping medicines that may be causing the problem. In some cases, you can take a different medicine that doesn't cause erection problems. Taking prescription medicine that can help you get erections. These include pills such as sildenafil (for example, Viagra), tadalafil (for example, Cialis). In most cases, this is sufficient for treatment but you should just be patient and do necessary tests. Hope I have answered your query. Let me know if I can assist you further. Take care Regards, Dr Iven Romic Rommstein, General Surgeon"
},
{
"id": 193987,
"tgt": "What causes protruding belly button?",
"src": "Patient: I am a male, 33 years old with no known health issues. I am in decent shape and not overweight but my belly button has been protruding more and more over the last year or so. I once had an inny but half of it is now an outy. Any idea what could be the reason for this? Doctor: Hello, It can be due to hernia or infection. Doing an ultrasound abdomen help to diagnose the cause. Share a picture for better understanding. Hope I have answered your query. Let me know if I can assist you further. Take care Regards, Dr S.R.Raveendran, Sexologist"
},
{
"id": 69356,
"tgt": "Can lump on testicle cause pain in leg?",
"src": "Patient: i found a marble sized lump on my right testicle 4 weeks after vasectomy. i squeezed it a little to see if it was hard or soft. i now have pain in my right leg, from hip to foot, with a dulled sensation and my leg is cold. should i go to the er, or can a visit to my pcp wait until monday? Doctor: Hi,A lump in you testicle should be investigated without delay. Therefore, I suggest you go to the ER. The pain on your leg and foot is unlikely to be related to the lump. Regards,Dr K A Ppttinger"
},
{
"id": 37780,
"tgt": "Suggest treatment for cough with wheezing",
"src": "Patient: What can I take for cough with wheezing no hx of asthma already taken Delsym does not seem to work cough drops that dont work and cold multi symptoms tabs that dont work had cough about 5 days with no congestion, had painful breathing when first symptoms started like ice going the wrong way for about 3 hours until I took 2 advil. then cough started and cannot seem to aleviate the cough. What else can I take? Doctor: Hello,Welcome to HCM,Your symptoms looks like you have asthma which is mild now. Asthma could be due to many reasons. You have dust allergy or seasonal allergy which requires complete work up and also get a lung function test to confirm asthma. You may have to do allergy test to know if your allergic to something.If you have allergy then you can start immunotherapy before your asthma symptom increases. I would suggest you to consult both pulmonologist and immunologist for asthma and allergy testing. Thank you."
},
{
"id": 213451,
"tgt": "Starting to lose control over age. Is it eye related or mental issue?",
"src": "Patient: Hello, i have a question. My grandmother has started to lose control of herself 2 days ago. I dont really know how to describe it but i think it s not too severe. The problem is she somehow can not open her eyes or not wanting to do so, and they look cloudy too. Aside from that she can t seem find her way around the house anymore which i assume is related to her eyes problem. We are going to get her checked out at a hospital soon when it s not too hot out side, and in the meantime i want to find out as much about this as i can. Doctor: Hi Forum. it appears that she had raised or lowered BP or sugar. she should be taken to hospital as early as possible. there she may have to go for blood sugar c.t scan and other tests for diagnosis. in the meanwhile give her glucose water{10 spoons in one glass of water. she must be shown to a neurologist for diagnosis. if she is able to eat give her semi solid diet. first ask her then give any thing to her. never insist her to take any thing if not in a position to take. thank you."
},
{
"id": 36065,
"tgt": "What is the treatment for streptococcus bacterial infection?",
"src": "Patient: Hi, my mother is not feeling well. She is suffering from lung problem we did sputum test and doctors are telling that it is a streptococcus bacteria. She also has vomiting problem. Doctors are giving perinorm injection and decdan injection. Can you please tell me why they are giving those injections Doctor: Hello,Perinorm is an anti-emetic(it stops vommiting in a patient). Decdan is a steroid. Amongst the things that steroids do is promote healing and repair and also prevent any adhesions that the bacteria may have caused the lung to form leading to difficulty in breathing. The doctors would have also used a round of antibiotics initially to counter the bacteria.I hope this helps otherwise feel free to get back to me.Thank you for your query."
},
{
"id": 88612,
"tgt": "What causes stomach pain and dizziness?",
"src": "Patient: My father is very ill for about 4 weeks. He said it feels as if his stomach is going to explode every time he sits down, he is also very shaky and weak he refuses to see a doctor in the case that it is nothing, and waists a bunch of cash. He also said there is no head pain only slight dizziness can you please hint to any clues that may help? Doctor: Hi.Thanks for your query and concern about your Father who is not willing to go to a Doctor.Read your question and understood the problem.With the history you have provided, your Father is suffering from : Very sick - 4 weeks - feeling of stomach exploding on sitting down - shaky and weak - No head pain but slight dizziness. I can think of the following problem he may be suffering from:Any general disease like Typhoid, Intestinal infection, inflammatory process in the abdomen like liver abscess, the amoebic one is without fever as also Tuberculosis with its complications.Due to persistent problem of 4 weeks He must been very weak, shaky and dizzy. The feeling of exploding abdomen can be due to the factors mentioned above. There are many serious disorders that can present without fever and other symptoms as the body is weak enough not to respond in a normal way. I would advise you the Following:You have to convince him and family members that he is admitted in a Hospital as then only he can be well examined and investigated and treated.All the relevant blood tests, urine and stool tests. Standing X-ray of the abdomen, Ultrasonography of the abdomen and CT scan only if required.Intravenous antibiotics and fluids and surgical intervention if need be,. Wishing him an early diagnosis and proper treatment and cure."
},
{
"id": 108485,
"tgt": "Suggest medication for back problem",
"src": "Patient: Hi I am having trouble getting sleep. I have back problem and remain in pain much of the time. Thats one of the reasons. I also remain streesful a lot. I need some councelling I suppose and mnedication too. Are there any doctors in Noida? Doctor: You understand it clearly emotional unrest can be a cause of problems, and surely it can exaggerate your back ache problem. Regular exercise, back care, and yogic meditation will help you immensely."
},
{
"id": 200291,
"tgt": "What causes bleeding from urethra post intercourse?",
"src": "Patient: Hi when I ve had intercourse the night before then go to toilet in the morning I find that sperm has got stuck in the urethra so obviously I try to flush it out then when it does finally come out a lot of dark blood follows it, I ve had all the usual tests and camera done but came back clear, this is getting very frustrating for me now and hope you can give me some inclination or suggestions that might help to resolve this recurring problem. Thanks david Doctor: Thanks for asking in healthcaremagic forum It is always better to void urine immediately after sexual intercourse to avoid infection. And if not done you need not flush it causing it to bleed. Please visit a doctor for physicial examination if bleeding is persisting. All the best."
},
{
"id": 80994,
"tgt": "What causes shortness of breath along with pressure in head?",
"src": "Patient: I m a 39 year old male who either works out or plays basketball 4 days a week. Over the last 7 days I ve experienced shortness of breath at times but now it s happening more frequently than in the beginning. Now I start to feel head pressure, a little fatigue and I have trouble doing a full yawn. What is wrong with me ? Doctor: Hello dear, thanks for your question on HCM.In my opinion you should consult pulmonologist and get done1. Chest x ray2. PFT (Pulmonary function test)3. ECG.There are few possibilities in your case.1. Lung infection2. Bronchitis3. Cardiac cause.So get done chest x ray to rule out lung infection.PFT is needed to rule out bronchitis.And ECG is needed to rule out cardiac cause.Breathlessness and head pressure is seen in all of the above causes.So first diagnose yourself and then start appropriate treatment.Don't worry, you will be alright."
},
{
"id": 203503,
"tgt": "What could cause pain in lower part of penis after masturbation with urge for frequent urination?",
"src": "Patient: Hi, My age is 35. I am feeling pain in the lower part of my penis (not on the penis heador skin) after masturbation.I also have to frequently go for urination. It has started 3 weeks before.Sometimes without masturbation,i am having urination problem and have to urinate after every 40 to 50 mins.Sometimes i cannot sleep because of this and feel pain in my penis. I started going gym and after that this urination problem started. At the same time whenever i masturbate i feel pain in the lower part of my penis and have to go for urination most prequestly. Doctor: Hi,Welcome to Health care magic forum. It appears to be the urinary tract infection, prostatitis,or an S.T.D. I advise you to consult an urologist for diagnosis and treatment. You may need to have an M.R.I. besides other tests for confirmation. I usually prescribe to my patient with such symptoms ofloxacin, ornidazole, and diclofenac sodium. Wishing for a quick and complete recovery. Thank you."
},
{
"id": 175261,
"tgt": "What is meant by soft tissue injury?",
"src": "Patient: Hi Doc, what s a soft tissue injury? My six month old daughter took a fall 5 days ago, we noticed a bulge on her forehead the next day, despite applying cold compress, antibiotics and vitamin c (recommended by our GP) the swelling is yet to subside, should we be worried? I m considering having an xray done... Doctor: Thank you for consulting in Health care Magic .Your daughter may be have haematoma, which happened after injury of soft tissue-skin, muscles. Bones are normal, if you want to check -better MRA,because less radiation. If you will apply heparine ointment 2-3 times small amount and rubber this place with massage, then it will disappear, you should wait 10-14 days,if bulge(haematoma) will not disappear then you go to neurosurgeon, he will do punction and removing blood from there.Best regardsDr.Svetlana"
},
{
"id": 47847,
"tgt": "Is it safe to take daily medications for nephrotic syndrome?",
"src": "Patient: Hi Doctor, My wife suffering Nephrotic Syndrome and under medication from last 4 years, Daily she is having medicines like Tecrograph 1mg, Immuran 75mg, Cardace 25mg, Wysolone 7.5mg but last two days she had jaundice, her total serum bilirubin is 3.5mg, conjugated 2.9mg/100ml and unconjugated 0.6mg/100ml. Now my question is can she take her daily medicines ? Doctor: I advise her to skip Immuran.She can continue with rest of the drugs.Also mention the complete lft report.Consult a Nephrologit in person as well."
},
{
"id": 105900,
"tgt": "What measures should I adopt for enlarged adenoids with slight wheezing ?",
"src": "Patient: MY 5 yr old daughter is suffering from cogh for past 8 months. Doctor disgoned her in last winter with enlarged adenoids+Slight Wheezing. Treated with Nasal steroids and Budesal Nebulizsation +Montair for few months. medicine didnot improve the condition much..after stopping medcine...she again has frequent cough especially from evening + night...and she is fine in morning+afternoon...something triggers her in evening and she start having continuos cough (evey 2 minutes or so till she sleeps)..pls help..she is suffering...have already seen too many doctors..even tried homeopathy Doctor: Hi, welcome to health care magic forum, well as you have mentioned that you have tried many treatments even Homeopathy, but i will again advice you to go for Homeopathic treatment as it is the only mode of treatment where you can get cure for your child, i do not know why my fellow Homeopathic physician failed to give your daughter cure, but it is not the dead end, you should concern some other Homeopathic physician, as it is safest treatment with no harm full effects, as in case of other treatments, also there will no re occurrence of the disease once after cured. For more info mail us at info@drkackerhomeopathy.com or visit us at www.drkackerhomeopathy.com"
},
{
"id": 22070,
"tgt": "Is left atrial enlargement serious?",
"src": "Patient: I am 52, 5'2\", 128 lbs, normal blood pressure, 10+ year history of mitral valve prolapse without any symptoms/complications except occasional palpatations. Heart check up recently (ECG) said enlarged left atrium & to see someone. I do have a slight heavy pain feeling in my left upper chest and neck area that is new over the past few months and more pronounced the past couple of weeks. I have also been more tired and feel sleepy when I've had a good amount of sleep and am usually on the go constantly. I've scheduled an appointment with my cardiologist but they have nothing for the next couple of weeks.... with the daily chest pain do you think I should see someone sooner - is there usually an urgency to this condition? Doctor: hello, Severity actually depends upon the amount of enlargement, presence of irregular heart rhythm. Usually, mild to moderate enlargement, with normal heart beat's is not a dangerous condition. Also, it must been there since years, and must not have developed overnight, so it appear to be a emergency. However, get one ecg done, if you are finding that your heart beat is irregular or rapid. Some amount of chest pain may be there in mvp. I guess, it's not exertional, also not of severe intensity, and going to left arm. Overall, it doesn't look like a serious issue."
},
{
"id": 106383,
"tgt": "My mother has dysentery associated with asthma",
"src": "Patient: My mother aged 74 years having chronic bronchotice, complained 15 days back that she is having decentry 5-6 times,I took her to a doctor clinic and prescribed some antibiotics . Even after taking she is complaining of decentry alternative days. what might be the problem she is too tired Doctor: Hi.. If the diarrheal episodes are persisting then change of antibiotics with further evaluation may have to be considered.. But first supplement with the fluids that can prevent the dehydration and make her feel better.. You can try ginger tea alternatively fenugrek seeds which also helps in preventing diarrhea apart from the antidiarrheal medications..."
},
{
"id": 103035,
"tgt": "Allergies and sinus, on augmentin. Take flonase, tylenol sinus, sinus polypus removed. Suggestion?",
"src": "Patient: hi. i have very bad allergies and sinus. a couples of weeks ago i was on augementin(antibiotic) for 7 days. i am still suffering really bad. i cant sleep at night b/c i cant breathe through my nose. i take flonase daily and i have been taking tylenol sinus, nothing seems to be helping and i forgot to mention i had sinus polypus removed in jan 2011. can you suggest anything else that i can do before making a doctors appoinment b/c it takes a while to get in with my doctor Doctor: the sinus is complication of allergiesyou need to consult allergy specialist to find out cause of allerfgy by tests and treat according toallergy guidelinestill can take antiallergics and sinus specific antibiotics"
},
{
"id": 185414,
"tgt": "What is the medical term for the white spot in the vascular duct of the tongue?",
"src": "Patient: I just came from the Oral Surgeon and he examined me for a small (no pain) smooth white spot in the vascular ducts on the underside of my tongue. He advised me to just continue to watch it and return in 3 months for reevaluation. I am trying to recall the name that he used to identify this spot. (something like Phibolist) He said it was a calcium deposit similar to a kidney stone. Do you possibly know the name of this stone? Doctor: Sialoliths may be the name mentioned to you. Have you been told that your salivary gland duct is blocked? Calcium deposits under the tongue are not uncommon. Your salivary flow is important to the health of your mouth. Salivary flow reduction can cause dry mouth, bad breath and even an infection in the duct. Wharton's Duct is located under the tongue. You can try to stimulate the flow of saliva by sucking on something sour such as a lemon. increased flow can push out calcium deposits and prevent the need for surgical removal.A Phlebolith, by definition, is a calcification in a vein or blood vessel. Vascular malformations may present with these. Sometimes, although present at birth, these may become prominent or increase later in life. Swelling may become evident. The phleboliths may be seen on an x-ray. Additional studies including ultra-sound and MRI may be required to determine vessel involvement. Hemangiomas are common with swelling, but less common with a phlebolith association in the mouth."
},
{
"id": 220867,
"tgt": "Are loss of appetite, nausea and abdominal pain signs of pregnancy?",
"src": "Patient: Hi I'm 19 years old and my boyfriend and I have had unprotected sex multiple times while I was ovulating . I usually get my period from the 12th to the 17th or so but this month I had it usual time but then also had some spotting on the 19th to the 23rd . But I'm worried that I am pregnant . I've felt kind of nauseas , and not very hungry . And I've also had a weird crampy feeling in my lower stomach . My boyfriend also says I've been having mood swings too . Doctor: HI, Thanks for the query. I understand your concern. Having unprotected sex many times during ovulation/ your pregnancy like symptoms suggest strong chance of conception. This time you had unusual bleeding after menses.this can be due to- - both bleeding & regular menses & the spotting after wards can be hormone deficiency with early pregnancy / Threatend abortion/Irregularity in period out of physical stress or psychological anxiety / fear about conception./ Hormonal imbalance causing menstrual irregularity. The Home pregnancy test after 8 days of the menstrual date/ USG scan after 3 weeks of ovulation can give exact situation.Consulting a gynecologist for physical examination & immediate treatment in case she suspects pregnancy is advised,/Repeat menstrual irregularity needs investigations to find out the cause & specific treatment. Thanks."
},
{
"id": 39222,
"tgt": "What causes itchy bump on hands?",
"src": "Patient: Hi I came back from a concert two days . The off the concert I had no marks until I woke up the next morning I had bumps on my hands . They itch , they still do and it s two days now. The are kinda risen on my skin and redness kinda went down. I was really in the grass at the concert .So I m wondering if it s mosquito bites or bed bug? Doctor: Dear FriendWelcome to Health care magic. I am Dr Anshul Varshney , I will shortly try to help you with my opinion.It seems that you have been suffering from a condition called as urticaria.it is an allergic condition of skin characterised by red itchy bumps.It can be due to recent infection, bite by mosquito/insect, food, seasonal change or any other environmental factor.In such cases , i usually advise my patients to take tablet levocetrizine in night time and fexofenadine in morning for atleast 1-2 weeks.For topical application, mild to moderate potency steroids are advised.Based on the available details, this is my best advice for you. You should discuss with your doctor and share my opinion, I am sure he/she will agree to my opinion. If you wish to discuss the problem further , you may please ask.Stay Healthy."
},
{
"id": 97321,
"tgt": "How to manage severe pain in face after a fall with bleeding?",
"src": "Patient: Girlfriend has fallen and hit her face on the floor, there was a lot of blood but it has stopped bleeding now, she's now saying it hurts a lot. Visibly I cannot see anything on the outside and she says she isn't is feeling dizzy so what's the best course of action? Doctor: Hi, Thanks for writing your query. After reading your post, I would like to advise that you should get her clinically examined and investigated. Facial injury is usually followed by swelling and discolouration around the eyes.Ice compress can be applied over swelling, if present. A CT Scan of head will be helpful in planning further medical management. She should be observed for any symptoms like disorientation, giddiness, vomiting, persistent headache, , nasal bleed or ear bleed. She should be advised proper rest. I hope this is helpful to you. Thanks."
},
{
"id": 129255,
"tgt": "Any suggestion for child suffering from swollen ring/pink toe of right foot?",
"src": "Patient: I noticed Sunday, 2/2/14 that my 7yr old Daughters ring toe and pinky toe on her right foot were swollen. She has fingers for toes, long and skinny and they were both swelled up like sausages. I could not find any bruising, she can bend them, she doesn t remember hitting them on anything, but she does say it hurts when she bends them and when I push on them. I gave her some Motrin hoping that it would reduce the swelling. Monday, they were still swollen, so I tried giving her Benadryl to see if that would reduce the swelling, thinking maybe it s something allergy related. Tuesday, the pinky toe is not swollen anymore, but the ring toe is still fat. Still no bruising, but she still says it hurts when I touch it. I am at a loss on what it could be or why its not going away. I don t know if something bit her, or did she stub it? I don t know what I should do. Do I take her to have it checked out or give it more time? This is now day 4. Doctor: Hi. My opinion is that it should be checked to exclude insect bites, allergy, Fungal infections or even trauma. Even if she does not recall bumping her foot, it is possible she did and sustained a ligament injury, especially as it is sore when touched. Rather let your doctor have a look. Hope this helps."
},
{
"id": 181830,
"tgt": "What causes pain at wisdom tooth extraction site?",
"src": "Patient: Yes I got my wisdom teeth removed about a month ago and they are still healing and I have this thin wire thing sticking out of the healing wounds of my wisdom teeth holes and it hurts every time I touch it with my tongue . Do you think it's a nerve or something? Doctor: Hello and thank you for your question. It could be a nerve. Nerves to the teeth are extremely sensitive to touch and sometimes to cold. You need to see your dentist as soon as possible so that they can correct this problem for you. Good luck and thanks for using HCM."
},
{
"id": 192603,
"tgt": "What causes smell inside penis?",
"src": "Patient: I have a bad smell inside my penis. I got recently married in the month of october. I used to urinate more than 10 times a day, coz of drinking lots of water. I have been noticing this smell from last one month. What could be the reason for this. Please advise. Doctor: Hello, It can be a smegma which is nothin but an infective material collected over the head of penis. It has to be cleaned regularly otherwise it will get infected and lead to foul smelling discharge. As of now clean the affected area with soap and water and apply topical antibiotics like mupirocin. If symptoms persist better to consult a dermatologist and get evaluated. If symptoms persist circumcision may be required. Hope I have answered your query. Let me know if I can assist you further. Take care Regards, Dr. Shinas Hussain, General & Family physician"
},
{
"id": 44288,
"tgt": "Have PCOD, Ovarian follicules, no fluid in cul de sac. Previous abortion with MTP pills. Chances of pregnancy?",
"src": "Patient: hello ,i have a query,regarding pcod diagnosed by tvs,left ovary showing multiple follicles 4-5 mm.lt. & rt ovary=23 by 14 mm both,no fluid in cul de sac,my husband has oligospermia ,diabetes,minor uti also.i concieved two yrs before but took mtp pills.married since 3 years,please guide my probabily of getting pregnant again,waiting for reply.what should i do?i have no mensrual irregularities and no masculine behaviour. Doctor: Hi Welcome to HealthcareMagic PCOS can not be diagnosed only on the basis of TVS. You need to undergo hormonal investigations like LH,Testosterone and DHEAS to reach at diagnosis as you don't have clinical symptoms. 23*14 mm is size of rt ovary or size of a follicle in rt ovary is not clear. If there is anovulation then you may need ovulation induction with clomiphene. If you are overweight then try to lose weight by diet control and exercise. Testing blood sugar levels will give an idea about need to start metformin. As your husband is oligospermic and diabetic,IUI will be better choice compared with timed intercourse. Don't worry there is a good chance of getting pregnant once ovulation is confirmed. Take care"
},
{
"id": 119894,
"tgt": "Suggest treatment for muscle & vein problems",
"src": "Patient: hi doctor, i am suffering pain past two years (times only) and consultant or tho he advised this is L4 & l5 PROBLEM muscle & veins comes outside.. you go to physio therapy like I.F.T, Traction etc.. right now which type of treatment can i follow? pl advise me .. Doctor: Hello, From the symptoms you have provided it appears to be not a muscle or vein problem,this is simply a case lumbar nerve root compression,but your history and complaints are quiet insufficient to reach towards a definite diagonosis. I would like to advise you to go fo a xray of your lumbar spine,flexion and extension views,and if possible get sn mri lumbar spine with screening of whole spine. Hope I have answered your query. Let me know if I can assist you further. Take care Regards, Dr. Rohan Shanker Tiwari"
},
{
"id": 204324,
"tgt": "What are the symptoms of psychopathic personality?",
"src": "Patient: Hi. I am a 23 years old female and I have seen some videos and posts about behaviour of psychopaths and most of the traits apply to me: i have feelings but I will never show them, often I will not feel any empathy towards other people, I do have that dead stare or dead eyes which send chills to people if they look into my eyes, I never feel shame for the impulsive things I did and there is no remorse, I blame sometimes others or the society for the misfortunes that happen to me. Other things that are not really psychopathic is that I could never hurt children and I am very caring towards my relatives and parents. I hate men when they want to dominate and raise their voice, it makes me go wild and attack or on a tricky way and patiently making them think they dominate while I do and at the end I make a fool out of them. I wonder am I a psychopath or kind of or not? Doctor: hi greetings...as per the information provided by you have little odd and eccentric thoughts, but behaviour is within normal limits as per the information provided by you...generally videos or movies will be pictured\u200b to gain the attention of the audience and all that is shown in videos or movies need not be trueit's very delightful to know that you are showing love towards your relatives and kids...if you were to be really psychopathic then you might have behaved abnormally even towards your relatives and kids...spend time with your friends, go for a movie, exercise 2 to 3 times per week, go for a morning walk daily...if you still get this type of abnormal thoughts it's better you write it down in a book as and when you get this thoughts which released you from anxiety and makes you feel better..chronic anxiety might also lead to these type of thoughts...Good luck for your future endeavours...open sold your query and feel free to ask any further thank you...have a nice day"
},
{
"id": 72821,
"tgt": "Suggest treatment for cough , abscess and wheezing",
"src": "Patient: Hi, I've had a cough for around 5 weeks now, causing me to wheeze, i did have a cold too but that went about 1 week ago. I went to my doctors around 2 weeks ago they gave me antibiotics and an inhaler, i havn't finished my course of antibiotics is that why I still have my cough ? Also have a lump under my chin like near my throat, near the gland, and have an abcess. I do smoke, could this be why i have a cough and the lump under my chin ? I've been feeling a little nausea, and going hot and cold sometimes. Doctor: Hello dear , hiWelcome to Healthcaremagic.comI have evaluated your query thoroughly .* This seems in relation with pulmonary tuberculosis most likely with cold abscess neck .* Needs confirmation with basic lab tests , x-ray chest and FNAC or Gene PCR reorting of the neck lesion .Hope this clears your query .Wishing you fine health ahead .Regards dear take care ."
},
{
"id": 94284,
"tgt": "Have abdominal pain. Had endoscopy and other tests. All normal. Recommendations?",
"src": "Patient: Hello, I have been having constant pain in my abdomen for over a year now. I also have sharp pains. I have had an endoscopy, colonoscopy , and a capsule endoscopy which all did not show any problems...can you tell me what I should do now or what I should say now? I feel like my doctor has given up looking for the problem or a solution... Doctor: Hello. Thanks for using this forum on HCM. Can you further describe where exactly on the abdomen do you feel the pain? if there any other associated symptoms? Your age and sex? Regards."
},
{
"id": 216244,
"tgt": "What to do for pain in the legs due to cellulitis?",
"src": "Patient: Hello my name is Melissa.. I am writting on behalf of my sister I am very concerned she has been in so much pain for the past 2 months. She has been in and out the hospital for cellulitis on her leg so they say then another hospital told her she would need to see a dermatologist the area is getting darker and draining. My sister says it started off as a bruise a few years she feels burning and pressure we ve tried soaking in epson salt the hospital prescribed her mupirocin and nothing is helping. Is there any way i can send a picture to you to get a opinion. At least to know we can not afford a doctor as of now.Thank you agin in advance. Doctor: Ok, continuing cellulitiis is very destructive and often causes loss of limb. Several things to consider would be if the germs are ressitant to antibiotics. This is espeically true if there is a yeast infection that is not even treated. Then, reasons why a treated infection does not go away can include having infection deep in . this can include infection of the bone and this requires specialized testing to evaluate ity. THere can also be pockes like boils externally or internally that need to be removed. there can also be problem if the area does not get good blood flow and this generally requires an expert in vascular surgery to be able to evaluate or treat it."
},
{
"id": 8166,
"tgt": "How to get rid of pimples and its scars?",
"src": "Patient: many pimpples are appearing suddenly on my face leaving scars and stain of those many pimples r apearng on face leavng dark scars .wl these scar wil disapeare and how Doctor: what is your age if you are a teenager then it is due to hormonal imbalance.are you having the habit of bitting it with nails then dont do that that will leave scars on your face already you have developed scars so to remove them you can take tab arogyavardhini& magisthadi 2tab 3times for 1mth."
},
{
"id": 172174,
"tgt": "Suggest treatment for headache caused by a minor injury",
"src": "Patient: My son was playing in his room earlier and apparently bounced off a ball and hit the side of his bed. He did not come and tell me about it immediately. However, before bed he complained about it hurting and told me what happened. He can breathe deeply without pain, but says yawning hurts. He also say that squatting and standing back up hurts and that going up and down stairs hurts. What should I do? Doctor: Hi dear, I had gone through your question and understand your concern. You should give him pain killer-Ibuprofen or Combiflam. Apply Thrombob ointment 3 times a day to affected area with slight massage during 5 days .Hope I answered your question. If you have more questions, please contact us ,we are always happy to help"
},
{
"id": 33657,
"tgt": "What causes fluid filled odd shaped bumps on the inside of lips?",
"src": "Patient: i have what looks to be fluid filled odd shaped bumps on the inside of my lips....they do not hurt and they are just kind of annoying... i have been chewing for about 6 months now and since i seen these i got scared so i stopped...please help and tell me what they are Doctor: Hi dear!Welcome and thanks for consulting me. I have carefully worked through your case, and realize your health worries. Being your physician, I assure you not to worry as your health issues may resolve soon.Looks like it's due to repeated chewing and injury due to that. Although without seeing it (in a picture) it's difficult to tell, it may be a secondary infection.Can you please answer some questions??Do you have a fever?Are they really fluid filled, or just look like they are filled with fluid?Are they only in the region of teeth biting or somewhere else in mouth too?? May you get very well soon and have a wonderful time ahead. Please don't forget to give your valuable feedback and five star ranking, so that I may be in a better position to help you in future too. Go to my profile to write a Review regarding your experience with me here at my Virtual clinic. If you feel easy, you may reply my questions with a photograph of the bumps. Go to my profile and click at \"Ask me a question\" directly from my profile for better and prompt assistance.Regards!Your PhysicianDR. HANIFPHILADELPHIA-PA, USA"
},
{
"id": 150169,
"tgt": "Diabetic having squint in the eyes. MRI of brain says lacunar infarcts in basal ganglia, microbleeds in occipital lobe. Advice?",
"src": "Patient: Himy mother recently have squint in her eyes..she is a diabetic patient.according to doctor suggestion we had a MRI of brain.the report says;-acute lacunae infarct in the facial colliculus-Old lacunar infarcts in bilateral basal ganglia internal capsule and thalami, bilateral periventricular white matter and pons.-old microbleeds in left parasitical occipital lobe, bilateral thalami and left basal ganglia.-age related cerebral atrophy with sub cortical and periventricular white matter ischaemic changes. please suggest diagnosis and remedial measures.thanks Doctor: Hi,Based on the details your mother is diabetic and MRI brain revealed multiple infarcts in brain the acute one being in facial colliculus (brainstem) which is producing squint ,with old microbleed, with small vessel disease of brainSuggestions for her are1) control of diabetes strictly2) diabetic diet3) ecosprin 150 mg daily post lunch4) Statins 40 mg bed time5)Anti hypertensive idealy coversyl plus once daily6) citicholine 500 mg twice daily 7) fluids 3 l/ day8) exerciseCheck her cholesterol levels, thyroid functions and kidney function You should consult neurophysican for further management and treatment planRegardsDr Sandhya ManorenjNeurologistHi tech city Hyderabad"
},
{
"id": 112591,
"tgt": "Having back pain. Regularly working at gym. What could it be ?",
"src": "Patient: Hi Doctor.. I am having back pain probably in my sacrum or coccyx region of spinal chord. Day before yesterday morning when i woke up i had this pain. At that time it was not too much just a hinge but now it has increased. Its not a sudden pain it keeps on painin without any sudden rise. I'm working out in GYM for more than 2 months and i haven't gone for a long tour too. Please suggest me some medication. Doctor: Hi, You have got back pain probably in coccyx, or scrotum, gradually increasing. It appears that the pain may be due to stone in the ureter nearer to the bladder, hence refering to the scrotum, it may also be the infection in the lower part of the ureter. I advise you to consult an urologist for diagnosis and treatment. You may need to have U.S.scanning besides other routine tests for confirmation. Thank you."
},
{
"id": 202412,
"tgt": "What causes pain and inflammation in scrotum and pus cell in urine is 160-180 hpf?",
"src": "Patient: I've got a painful scrutum which seemed bulging/inflamed at the center (not the left and right scrutum). A gentle hit of anything would cause some pain in my groin (left side). I had a Urinalysis and the Pus cells in my urine is 160 to 180HPF... what's my condition? thanks Doctor: HelloBased on information provided it appears that you have developed infection of Epididymis (Epididymitis)You need to take broad spectrum antibiotic like Cefexine and anti inflammatory drug like Diclofenac twice daily.Get the prescription of medicines from your family Physician.Ensure to drink more water.To keep your urine dilute This will help to control dysuria.Dr.Patil."
},
{
"id": 102809,
"tgt": "Constant nasal blockage, cough, running nose and sneezing. No help from pulmigen oral tablets. Any remedy?",
"src": "Patient: sir iam suffering from nasal blockage persistent cough running nose sneezing for last 4year ihave taken pulmigen oral tab 10 tab ina month for 3 month and completed in SEP 13 but in mid nov 13 i am having above mention problem again what to do now Doctor: Hi, It seems to me that you are suffering from allergic rhinitis. I would suggest you to take some antiallergics like antihistamines, decongestants or both as needed. Regular use of an intranasal steroid spray regularly may combat your symptoms.There are various trigger which precipitate such symptoms like indoor allergens,nonspecific triggers like smoke, strong perfumes , fumes, rapid changes in temperature and outdoor pollution. Consider avoidance of these situations or triggers if they seem to aggravate symptoms."
},
{
"id": 164939,
"tgt": "What causes cough and wheezing in child?",
"src": "Patient: My 19 Mth old baby has been wheezy and out of sorts for about 5 days. I took her to the doctor who said she was working to breath, she put her on a nebuliser which seemed to help. That night she was coughing and wheezy again so I gave her prednesol for the next three days which seemed to help. Now that she has finished the prednesol it seems to be coming back slowly. I was told by the doctor she had a viral infection. Doctor: cough and wheezing in a child can be due a number of causes but most commonly it is due to acute bronchiolitis ( viral infection of respiratory tract) or bronchial asthma. in case of bronchial asthma it is recurrent where as in bronchiolitis it is not."
},
{
"id": 153530,
"tgt": "What causes sacral chordoma?",
"src": "Patient: My husband has a sacrum chordoma. It has metastisized to the liver, lungs, bone and blood. Hus urine is continually dark, cloudy, foamy, and white flurry things in it. He is on hospice care and they provide him with antibiotics for 14 days. 3 or 4 days after the antibiotics are finished the symptoms repeat. They have not given us a time frame but he is pretty much bed ridden. Are these urine infections a sign his kidneys are failing? Doctor: Hi,Thanks for writing in.Chordomas are cancer cells that arise from rest cells which start to grow in an adult. Chordoma involves the clivus area in skull and sacral region in the lower end of spine. These are two common locations where it might occur. However rarely it might involve the rest of the spine.Since the chordoma has spread to the liver, lungs and bone, therefore he might be having advanced disease. His kidney function is to be monitored closely for any changes in function and also infection which has to be treated urgently. Cancer patients get week due to low immunity and this is the reason infection causing bacteria find it easy to attack various systems in these patients. Kidney function is studied by doing serum creatinine and blood urea levels which also gives the glomerular filtration rate. Please discuss these levels with his doctor.Please do not worry."
},
{
"id": 64795,
"tgt": "What do lumps around the breast indicate?",
"src": "Patient: I am a 58 year old male. 2 days ago I noticed three elongated lumps around my right breast area under the skin. They were about an inch or so long by a half inch wide. I thought it maybe a spider had bit me because they were a little itchy. Today I noticed several pea sized ones on my stomach. What could this be? Doctor: HI,Dear, Thanks for the query from you for the small moveable lump under skin at the site of the hickey.1-I feel concerned that you are serious about the knowing the cause of the lumps around the breast and the abdomen.2- In my opinion dear,the lump under the skin 2 daya back around the rt breast and on the stomach,with itchiness, are mostly due to the spider bits or bedbut bites.4-I would advise you to take cetrizine , with sos NSAID and apply some low steroid ointment locally ,which would take care of it. 5-Hope this would solve your worry about lumps around the breast and stomach. 6-Wellcome to HCM for any more query."
},
{
"id": 152351,
"tgt": "I am suffering with generalized decreased sensation, kindly help me",
"src": "Patient: I am doctor and i have a vague complaint since dec 3 09... on dec 3rd i have taken 2 large pegs (hard) whiskey (teachers) then after 2 days onwards i am suffering with generalized decreased sensation all over the body..i underwent total hemogram. tft etc all r normal... i was prescribed multivitamins. but i didn't find any benefit. moreover since a few days back i am having episodes of palpitations. can u please help me /? Doctor: Only alcohol resulting in such symptom???are you sure that you did not have these symptoms before?? get a neuro workup done..this might be something hidden, brought into light by your alcohol intake."
},
{
"id": 179263,
"tgt": "What causes sticky motions in babies?",
"src": "Patient: Hi! My grandson is 10 months old and lately he s been having very sticky poo, very dificult to wipe it. What could be the cause(s) of that?? Could be some kind of allergy coming from gluten or lactose? He s eating mostly everything, without salt or sugar, and still being breast fed two to three times a day. Doctor: Thanks for posting your query. It would have really been helpful if I could put my hands on her abdomen and examine for myself but that would not be possible. It could indeed be gluten enteropathy but no need to jump to conclusions at the moment. You may try avoiding wheat based products and see if it makes a difference. Giving boiled rice based meals along with vegetables, chicken and pulses, all mashed together, should be fine. No problems in adding salt. Continue breast feeding.I hope that with the above information, you can handle things better. Feel free to revert back in case of further queries if any."
},
{
"id": 222714,
"tgt": "What is the best treatment to get pregnant?",
"src": "Patient: can bromergon help reduce my high prolactin levels and can i get pregnant while taking the tabs. also what are the side effects of the tablets, i had a fibroid op 9 months ago and still having lots of milky secretions from both breast and still cant get pregnant Doctor: Hi dear, I have gone through your question and understand your concerns.Bromocriptine is the drug to reduce the levels of prolactin and it is quite effective in its action.It has side effects like gastric upset, dizziness, nausea.Inability to conceive and milk secretion from both the breasts are suggestive of increased prolactin levels and Bromocriptine is the correct medicine for its treatment.Hope you found the answer helpful.Dr Deepti Verma"
},
{
"id": 73353,
"tgt": "Can history of pleurisy cause chest heaviness?",
"src": "Patient: For the last week or so I have noticed it to be tough to take a nice deep breath. I need to have a good yawn to make it happpen. My chest also feels slightly \"heavy\". I have had pleurisy before and wonder if it is that again. No other symptoms of any kind. Doctor: Yes it can. You may consider having a chest x-ray as a start to evaluate for fluid accumulating in the pleural space that can cause this."
},
{
"id": 146086,
"tgt": "What does this MRI report of spine indicate?",
"src": "Patient: Hi, I recently had an MRI and I just wanted to know what this result mean? \"At the L5-S1 level there is very mild increased signal at the radial fibers, but no clear radial tear.\" The test type was MRI Lumbar spine without IV contrast. History: Eval for annular tear. Doctor: I read your question and I understand your concern.That report indicates that while there are some mild changes in the cartilaginous disc present between your last vertebrae and the sacral bone, there is no tearing and no compression of the nerve root which exits at that level.So since there is no nerve compression and no spinal stenosis (narrowing of spinal canal) or other rarer conditions are mentioned, it means that you need no surgery and must continue conservative treatment with physiotherapy and pain-killers (I'm assuming you have pain as it is the most frequent presentation in lumbar spine conditions). The symptoms should subside gradually overtime.I hope to have been of help."
},
{
"id": 207433,
"tgt": "Suggest remedy for primary complex from other people of same age",
"src": "Patient: Hi Doctor ! I m a 20 yr old guy I ve had primary complex as a baby (came to know frm my mom) I feel i hav less stamina than other guys of my age. When i indulge in a simple exrcise i start breathin excessively May this be because of the Primary complex ??? Doctor: HiI understand your concern.It is not due to primary complex.Relax.Every human being is different from each other.Their body demand and functions are also different.So everyone has different capacity to deal with situations.To increase stamina you have to do workout regularly.Running and cycling for 30 min is very good exercise to increase endurance power of body.Healthy diet with rich in protein will help.Extra protein supplement by milk shake or juice can elevated muscle power.Consult dietician to make proper diet plan.Consult physiotherapy for exercise plan .Confidence is big thing and motivated your self.set your goal and do not compare with any another.Still have a query then feel free to ask.Happy to help you.Thank you."
},
{
"id": 133545,
"tgt": "Suggest treatment for painful and bruised rib cage",
"src": "Patient: I m 43 year old female and I had a ground level fall 12 days ago and injured my ribs on the right side. I went to me gp and she had my take Zipzor, which didn t help. She took X-rays but said it was just bruised. The pain is almost unbearable. It radiates from the front to the back, and also hurts below my rib cage. Is there something that may have gotten missed? The pain seems to be getting worse not better Doctor: Hi,Thank you for providing the brief history of you.As you dont have a fracture this could be related to direct hit on the rib bone. Which will take some time to heal. Also, the muscles must be in trauma and this leads to the intercostal muscular pain Which will also heal slowly. Also taping can be done to restrict rib movements and control pain,Try performing slow costal breathing exercises as it will improve the muscular strength and also reduce the pain and aches.If the pain increases more along with additional symptoms than a CT may be performed, or else i do not think anything should be worried for.RegardsJay Indravadan Patel"
},
{
"id": 58999,
"tgt": "Diagnosed with many gallstones. Had diarrhea, nauseated . Are these symptoms of gallbladder disease?",
"src": "Patient: I have been diagnosed a month ago with many gallstones from a ct scan at er room. Tje scan showed gallbladder not inflamed. I have had diareah many times a day one gallbladder attack and nauseated so much I can't even work. I have no insurance for surgery. Is nausea and chronic diareah normal symptoms for gallbladder disease? Doctor: While nausea,vomiting,pain abdomen,dyspepsia commonly occur with gallstones,diarrhea is not a symptom of cholelithiasis/gall bladder stones.You need to see your PCP/doctor if the diarrhea persists."
},
{
"id": 33936,
"tgt": "How long does it take to cure intertrigo?",
"src": "Patient: I have been diagnsed with Intertrigo which developed due to an outdoor activity in a very hot and humid weather near my groin and inner thigh area. I used Dermacobin and now using Onabet-B cream however seems like the redness is increasing rather than becoming dull. I want to know how long it might take to be cured completely. Doctor: Welcome to healthcare magic,Intertrigo is very common skin rash that appears between folds of skin- armpits, beneath breasts, abdominal zone or genital area, between toes etc.Have in mind that intertrigo sometimes does not come alone but accompanied with bacterial or fungal infection !! So you have to cure them as well ! Treatment for intertrigo is simple if you keep your skin dry and clean it will be gone soon- less than a week !"
},
{
"id": 221861,
"tgt": "Is it normal to measure less than normal in 36th week of pregnancy?",
"src": "Patient: Hi I m kira. I m 23, 5f 2in I weight 68kg. I m 36 weeks preganat and have blood and protein in my wee. I ve had growth problems I don t measure what I should be so just worried about blood in wee. I had a blood test and urine test r weeks ago and came back normal. Hope u can answer my wuestions. Doctor: Hi,Blood in urine is not normal at any stage in pregnancy and should be evaluated further. Moreover you have protein as well and the baby is smaller than expected. All these point towards pre-eclampsia.You should get your blood pressure checked and get a detailed blood tests for liver function and renal function and ultrasound for size of baby and amniotic fluid level. You should also get Doppler ultrasound to see the well being of the baby. A Fetal heart trace will also be required.Also a scan of your kidney is required to rule out anything abnormal.Hope this helps.Regards."
},
{
"id": 88774,
"tgt": "Suggest treatment for left abdominal pain",
"src": "Patient: I have lower left abdominal pain mostly constant now! Have been to Dr.s countless times for this no resolution. Now is to the point not sleeping much. Last clinic visit Dr. say enlarged prostate I disagree, They also did a ct scan did not find anything. previous meds did not set well with me just made me feel worse. Only thing that helps sometimes is laying down. Pretty much sick and tired of feeling poorly this just goes on and on. Doctor: The roots that may trigger this kind of pain in left side are several. This includes urination, causes associated to the lungs and gastrointestinal. When suffering from diseases such as pneumonia, bronchitis or pulmonary edema you may experience this pain in left side that is at first mild to moderate and even severe at times. Left side organs in your body such as, the liver, ureter, kidney, ovary, fallopian tube, adrenal gland and spleen, the descending or the sigmoidal colon and so on;are the main parts of the body that are greatly affected when you undergo abdominal pain in left side.One of the causes of pain also is formation of gas in the stomach which is , majorely caused by constipation ,over eating or eating indigestible things . Home remedy for stomach aches: mix one teaspoon of fresh grated ginger with one teaspoon of lime juice; take immediately after eating., or some natural remedies like ginger , bishop's seeds , baking soda with hot water should be administered to provide a immediate relief. Take Home remedy for stomach aches: mix one teaspoon of grated ginger with one teaspoon of lime juice ,add some black salt and pepper , take immediately , after meals . The muscles and the nerves located on the left side of the abdomen suffer as well.Gulping on indicriminately only medicines or pills will not help cure unless you take care of your lifestyle , food habits and regular exercise yoga and proper rest . A well balanced and nutritious diet including ,fiber ,vitamins ,minerals in natural form fruit ,veges protein , less spiced and moderately salted also include antioxidents - lemon juice with water , Add antioxidents , garlic ,ginger , papaya , coco nut water.Aloe vera juice 25 ml +Amla juice 2 spoon , bitter gourd juice 25 ml ,turmeric powder a level spoon with a cup of hot milk at bed time , low fat dairy products ,sun flower oil , tofu to your diet . All have quality to kill bacteria on mass basis and give us resistance as well as energy to carry on .- Green Tea & Echinacea helps stimulate the immune system .- Drink plenty of water to boost the immune system as well as flush out toxins, Also little regular activity of body and proper blood circulation is beneficial for trouble free life .Walk Exercise , Yoga ,Pranayam ,deep breathindg also help give resistance from health problems giving strength to your immiunity . Avoid fried ,fast foods ,Tea ,coffee ,alcohol ,smoking ,colas ,anger ,worry ,mental tension , Constipation . It\u2019s important to check with your physician before adding herbs to your existing medications,regarding the herbs that are antioxidant ,have no side effects and go with any therapy of treatment .Hope this helps solve your query . Take care All the best Mail at drsuchda@gmail.com if any doubt .Don't hesitate to come back for any further query"
},
{
"id": 20113,
"tgt": "What causes burning in chest with high blood pressure?",
"src": "Patient: I am 21 years old, 260lbs, 5 11. I ve had pnuemonia about 3 times in my life. Other than being overweight which causes me to have high blood pressure on occasion, there is no other health conditions. My lungs are burning and my chest feels heavy for about 3 days now. I cough once in awhile and when I do, I can taste mucous but nothing comes up. I am not feeling any other symptoms. I do not smoke. What could this be? Doctor: Hello, Thanks for posting your query on Healthcare Magic...* Brief Answer: chest pain needs to be checked , blood pressure need to be controlled by weight loss , diet and medication if needed.*Explanation: My opinion is that high blood pressure on many occasions is an indication for control .That happens on many levels. First, be sure that high blood pressure is not secondary to any other diseases as Cushing or renal failure.Then ,Diet with low salt and less fat content is crucial. Exercise and other healthy activities are also a mean to control blood pressure. If all that dose not help, medications are a must .That is usually decided by your doctor according to your condition and comorbidities .Some drugs that are perfect for patients are not necessary good for others.Regular blood pressure monitoring is as important as starting treatment so that we could assess efficacy of the medication we started.I suggest my patients the following : detailed history about chest pain electrocardiogram and some labs as cholesterol and uric acid.The information provided by you is not sufficient to provide a good opinion. If someone comes to me with this condition I would ask them about other comorbidities like diabetes, hyperlipidemia etc *Conclusion: I suggest you consult your physician about your medications.chest pain need to be checked. If you have any further questions I will be happy to help, If not, you can close the discussion and rate the answer. Wish you good health."
},
{
"id": 26919,
"tgt": "What could be the cause of palpitations?",
"src": "Patient: Hi Dr Samuel,My Girlfriend's heart is beating really hard and every time it beats hard, it makes her really tired. What may be the cause? She has no record of any heart diseases, but she have had really little hours of sleep recently.Thank you.Best RegardsKoo Doctor: Hello. Thank you for your question and welcome to HCM. I understand your concern. Well, if this hard heart beat is also associated sometimes with a shortness of breath, combining with the lack of sleep and/or with the usage of energetic drinks or other caffeine-containing beverages, it most probably is an extrasystole (''sk\u0131pped'' heart beat). In young ages and when they are not usual phenomenons within a day, there is nothing to worry about. Stressful moments in life, physical or psychological, can be associated with extrasystoles. If this is a worrisome event to her and to you, I recommend her to be put on a 24-hour Holter rhythm monitor, to determine exactly what are these phenomenons, and if they are present, are there enough episodes a day to warrant treatment (because there is a cut-off line in the number of extrasystoles that can be tolerated). Instead, there is the option of the episode rhythm monitor - a machine who can be turned on and off by user, when he/she experiences the symptom. When the readings are ready, I suggest you to consult a cardiologist or feel free to ask me again. I hope I was helpful. Take care. Best regards, Dr. Meriton"
},
{
"id": 76376,
"tgt": "Suggest treatment for cough with bloody sputum",
"src": "Patient: I am 55,female, hypertensive and I have previous tuberculosis but treated or finished 6 months medications from May 1, 2008 to November 1, 2008. I have twice experienced cough out blood after sexual intercourse. I felt itchy throat first before I cough blood then later I felt slight headache. what will I do to prevent it? Doctor: Thanks for your question on Healthcare Magic. I can understand your concern. You are having hemoptysis (blood in sputum). You are also having previous history of tuberculosis. So in your case we should first search for the cause of hemoptysis because treatment is different in different causes. Following are the common causes for hemoptysis. 1. Recurrence of tuberculosis 2. Post tubercular bronchiectesis 3. Post tubercular bronchitis. So better to consult pulmonologist and get done sputum examination for AFB (acid fast bacilli - bacteria causing tuberculosis), chest x ray and PFT (pulmonary function test) for the diagnosis. Don't worry, you will be alright. First diagnose yourself and then start appropriate treatment. Hope I have solved your query. I will be happy to help you further. Wish you good health. Thanks."
},
{
"id": 41672,
"tgt": "What is the treatment for infertility in men?",
"src": "Patient: my sperm count is near 85 million/ml then active is 50 ,sluggish is 10 ,dead is 40 but still im not marry what my question is if i marry any probelm will come for my future and wil become a father? plz answer me doctore really too much thinking abt that Doctor: Hi welcome to healthcaremagic.I have gone through your question.Patameters you mentioned in semen analysis are normal, although liqufaction time, pus cells are not mentioned in your reports, they are equally important.Otherwise report is normal.Hope i answered your question.Would be happy to help you further.Take care."
},
{
"id": 50864,
"tgt": "Treated for UTI. Blood in urine with back pain. Back pain persisting. Chances of kidney stone?",
"src": "Patient: I was treated for a UTI on March 21. There was blood in my urine and I was experincing some lower back pain . I have now finished antibiorics and the infection seems to have cleared up, but I continue to have the lower back pain. I am drinking plenty of water as well as cranberry juice. Today, I notice a sharp pain in the vaginal area especially when I get up to walk. I don t have an appointment with a urologist until May 8. Could this possibly be a kidney stone(s)? Should I continue the water intake and see if it clears up or get it checked ASAP? Thanks Doctor: Hi thanks for your question. Commonest cause of pain in the back with haematuria (blood with urine) is kidney stones.Urinary tract infection (UTI) infection is second possibility.Consultation with general surgeon is advisible till you get an appointment with urologist.You may need ultrasound genito urinary system and urine examination to confirm or rule out possibility of stone in kidney (or any where in urinary tract) and UTI. Hope this answers your question."
},
{
"id": 90382,
"tgt": "What causes pain under the rib cage and abdominal bloating?",
"src": "Patient: for three months I have had severe pain, starts on right side under rib cage and wraps to the center of my back then center of sternum. I have to throw up in a wrenching motion for 10 to 15 min. then the pain is a little better. I have had bowel movements that range in color and size. My stomach always feels bloated and soar. I am worried. Made a dr appointment but they can not take me till the 20th of this month what should I do? Doctor: Hi ! Good evening. I am Dr Shareef answering your query.From your history, it seems that you have got an inflamed gall bladder possibly due to stones in the gall bladder. However it has to be assessed by a general surgeon. So I would suggest you to take an appointment with a general surgeon in your area, who after a physical examination, might advise you for some routine blood tests like CBC, ESR, LFT,serum amylase and lipase, Blood urea, serum creatinine, serum uric acid, and an ultrasound of abdomen. If confirmed to be a case of cholethiasis, the surgeon might advise you for a surgery. If not, further management would depend on the results of physical assessment and reports of the investigations.I hope this information would help you in discussing with your family physician/treating doctor in further management of your problem. Please do not hesitate to ask in case of any further doubts.Thanks for choosing health care magic to clear doubts on your health problems. Wishing you an early recovery. Dr Shareef."
},
{
"id": 84452,
"tgt": "Does Gemcal Nasal Spray have any side effects?",
"src": "Patient: Hello Doctor, i am a 24 year old Male. i had met with an accident couple of months ago, and had a severe fracture, my right hand radius and alna bone had broken. after the surgery my doctor told me to take GEMCAL NASAL SPRAY, my question is does it have any side effect with my fracture.?? Doctor: Hi,It does not have any side effects involving the fractures. Gemcal nasal spray (Calcitonin) is commonly prescribed to treat osteoporosis in postmenopausal women and also in elderly men to make the bones stronger and lower the risk of fractures. Its common side effects include nasal congestion, rhinitis, diarrhea, and flushing. It is not known to have any harmful effects on the fractured bones.Hope I have answered your query. Let me know if I can assist you further. Regards, Dr. Mohammed Taher Ali, General & Family Physician"
},
{
"id": 120848,
"tgt": "Suggest treatment for continuous twitching below rib cage",
"src": "Patient: continuos twitching, perhaps muscle spasms, that do not follow any particular pattern.. located primarily on the right front side, below the rib cage. sometimes pulsing is fast somrtimes slow. It may disappear up to ten minutes, but then returns. always in samr spot recently, have had other spasms in other parts of body, but all those eventually go away in less then 1 hour..52 yr old male, history 6 yrs diabetes melutis, bypass triple surgery 5 yrs ago. multiple genetic stenosous lamenectomy 12 yrs ago.. blood work A1-6.0, high triglcerides, normal cholesterol, high c3 serum, ana atibody positive over 1200 honogenious pattern, negative for rhuemotological proteins, fatty liver distended , bipsy reveals nash3 no history of alchol abuse... Doctor: Hello,The symptoms seem to be related to a muscle spasm. I suggest using a spasmolytic such as Baclofen three times a day. I also suggest using magnesium supplement for muscle relaxation. Warm compresses can also be helpful.Hope I have answered your question. Let me know if I can assist you further. Regards, Dr. Dorina Gurabardhi, General & Family Physician"
},
{
"id": 81741,
"tgt": "What causes gas like pressure in chest?",
"src": "Patient: Please sir ,am having. Something like gas in my chest region and it makes me uncomfortable and causes a kind of pressure in that region and I have met the doctor he gave me gestid and I don t have an ulcer please what do u think is wrong becos I don t seem to understand dis awful experience Doctor: Hello dear, thanks for your question on HCM. In my opinion you are having GERD ( Gastro Esophageal Reflux Disease ) mostly. But we need to rule out cardiac cause first as many times this can also gives feeling of gas and bloating in chest.So get done ECG and 2D Echo to rule out this. If both are normal than mostly you are having GERD only. It is due to laxity of gastroesophageal sphincter. Because of this the acid of the stomach tends to come up in the esophagus and cause the symptoms. Try to follow these steps for better symptomatic relief. 1. Avoid hot and spicy food. 2. Avoid large meals, instead take frequent small meals. 3. Avoid stress and anxiety. 4. Start proton pump inhibitor. 5. Go for walk after meals. 6. Keep 2 - 3 pillows under head in the bed. 7. Avoid smoking and alcohol if you have. 8. Loose weight if you are obese."
},
{
"id": 116738,
"tgt": "What is the treatment for the loss of 2/3rd of blood?",
"src": "Patient: My husband lost 2 thirds of blood. .Went to hospital and hey did a bone marrow test and 4 days later they still have no clue what is going on. He needs chemo but they can t start it until they know the true cause... Leukemia. However, they don t know what kind and the treatment for it. Help! Doctor: Hello,Thank you for your contact to healthcare magic.I understand your health concern, if I am your doctor. I suggest you that Until you diagnose your husband's disease it is necessary to give a blood to them. I advice a blood pint to your husband other wise the condition will be very lethal.I will be happy to answer all your future concern. Thank you,Dr Arun TankInfectious disease specialist.Wish you a best health at health care magic."
},
{
"id": 2169,
"tgt": "Is bright red color bleeding due to pregnancy?",
"src": "Patient: I have been on the depo shot for about two years now and mostly never have problems. I usually start spotting like a week before I m due and the blood is like brown and thick not like normal period blood. I got my shot in the middle of April and I m not due again till July 3rd. Well I recently started spotting and it s been bright red blood which I ve never had happen before. I was on a ten day course of antibiotics about two weeks ago for pneumonia which I know lessens the effects of birth control and I ve been sexually active is there a chance the bleeding is from pregnancy? I ve also been on antibiotics plenty of times before while on the depo and never have had bright red bleeding before Doctor: hello,bright red blood may not be due to pregnancy, it may be becausebof other conditions of the endometrium. you have to undergona thorough gynecologycal check upbestanisa"
},
{
"id": 149596,
"tgt": "Small clots in brain. Prescribed rosuvas, ecosporin, zolfresh. Side effects of medication?",
"src": "Patient: hi My fiance s dad was just diagnosed with a few small clots in the brain and has been prescribed Rosuvas F10,Ecosporin 150, Zolfresh 10, Neubrobian forte, strovas 10 and Razo D.He is about 75 years old. Could you please let me know what each drug is for and how it will help cure the problem?Also, please do let me know the side effects of each drug so that he can be updated as to what to expect on taking them. Doctor: Hi,Thank you for posting your query.In this list, the main drugs are Ecosprin and Rosuvas. Ecosprin is aspirin, a blood thinner, and is needed lifelong to prevent the recurrence of stroke. It can cause bleeding from gums, urine, motion, etc, which should be looked for.Rosuvas lowers the cholesterol and prevents further brain strokes. it may cause muscle aches and pains.Both Ecosprin and Rosuvas are needed lifelong.other drugs in this list are not for brain stroke or blood clots. Best wishes,Dr Sudhir Kumar MD DM (Neurology)Senior Consultant Neurologist"
},
{
"id": 147149,
"tgt": "What causes itching feeling in brain?",
"src": "Patient: i have a friend and yes it is a friend not myself so i would not know details.....but my friend claims that her brain feels itchy sometimes i told her that she needs an m.r.i. and that she she may have parasites do you have any information please and thank you for your time Doctor: the brain has no nerve endings so it is not possible to feel parasites in your brain. irritation of the brain causes seizures not changes in sensation"
},
{
"id": 2084,
"tgt": "Can I concieve if HSG shows narrow right fallopian tube, irregular mucosa and scanty perioneal spill?",
"src": "Patient: Hi Dok i took an hsg and the results read that ma right fallopian is narrow and its mucosa is irregular.it exhibits scanty perioneal spill . and the other has normal caliber and exhibits normal and free peritoneal spill. Will that mean i cant conceive? Doctor: hi if your one tube is normal you can conceive. There is no need to worry. You can try for 6 months naturally with medicines for growth of your eggs. Icici it doesn't work out then I think you should go for a laparoscopy full a full evaluation. It will definitely tell you regarding your tubal status because sometimes hsg is not accurate. Hope I have answered your question."
},
{
"id": 161450,
"tgt": "Could intake of orange juice cause frequent watery motion in an infant?",
"src": "Patient: my baby girl is 4 mnths 3 weeks old.since 2-3 days we have been giving her 1-2 ounces of orange juice daily one time.it seems like she puts motion more frequently than what she used to before ,also it looks watery.today i dint give her juice. is it loose motion or is it common wen we give juice, pls suggest Doctor: Hi, Bottled orange juice can contain a high concentration of sugar and hence cause loose motion. It's best avoided. Hope I have answered your query. Let me know if I can assist you further. Regards, Dr. Cajetan, Pediatrician"
},
{
"id": 9888,
"tgt": "Can Komark shampoo treat baldness?",
"src": "Patient: I'm 33 old man, getting bold so soon. the water i got is a hard water and found it hard to avoid the boldness. i'm planning to use komark but i need some info about it before i use it. naturally i've some kind of resistance to alergies and like cough etc. what can you tell me Doctor ? Thanks in advance. Doctor: Hi, As per your query, you have symptoms of baldness which seems to be due to environmental factors, genetic factor, improper hygiene, fungal infection and could be due to hormonal changes. Need not to worry. I would suggest you take a diet rich in multivitamins along with contents such as Ferritin, Biotin. Take Indian gooseberries with half a spoon of warm water. You should consult dermatologist once for proper examination. The doctor may order skin patch test, blood test, allergic test and physical examination and may prescribe supplements like Omega-3 minoxidil spray, and Vitamin E. You should start Komark after proper prescription as it could be possible you allergic to any one of the ingredient. Mintop 10 is good to use to get proper results and don't stop in between. Hope I have answered your query. Let me know if I can assist you further. Regards, Dr. Harry Maheshwari, Dentist"
},
{
"id": 53402,
"tgt": "What causes pain liver and kidneys area?",
"src": "Patient: Hi I have pkd of the liver and kidneys and pancreas. 2 days ago i got a sharp pain like a knife being jabbed into my back. It's hurt every since and hurts more when i'm moving around but also when sitting. I'm try to figure out if it's kidney or liver Doctor: Hi,Given your presentation, you are strongly recommended to have a Kidney Ureter Bladder KUB, radiograph examination and urinalysis. Further course of action maybe advised accordingly.Hope I have answered your query. Let me know if I can assist you further. Wish you good health!Regards,Dr. Tayyab Malik"
},
{
"id": 25529,
"tgt": "What do my blood pressure readings indicate?",
"src": "Patient: I am 73 years old woman, my blood pressure today was 122/70. About a month ago, I was 140/90 and was put on a blood pressure pill and a week ago on a small diuretic. Since starting to take the diuretic, my body temperature has been 95.7 to 97.5. Been very weak. My doctor feels I picked up a bug and just coincidence since just starting the diurectic. Should I be concerned over the lower blood pressure, lower body temp., weakness etc.? Doctor: Thanks for your question on Health Care Magic. I can understand your concern. Blood pressure of 122/70 mmhg is not considered as low. It is normal blood pressure. In my opinion, your low temperature and weakness are mostly due to diuretic drug. Diuretic causes excessive urination. So this can cause low sodium level. And this can cause weakness and lower body temperature. So if you are maintaining normal blood pressure without diuretic drug then ask your doctor to stop diuretic drug and continue blood pressure pill only. No need to take diuretic if you are achieving 122/70 mmhg of blood pressure with pressure pill only. You will mostly improve after stopping diuretic drug. So discuss all these with your doctor and plan accordingly. Hope I have solved your query. I will be happy to help you further. Wish you good health. Thanks."
},
{
"id": 205542,
"tgt": "What cause panic attacks after drinking while taking Citalopram?",
"src": "Patient: hi i went for a drink Friday night i am on citalopram i had 4 pints of ale i tried to go home with a friend i was then told had a panic attack and then proceeded to vomit large about three times i got taken to hospital and they said i also had some panic attacks there while they were doing an ecg should i tell my gp Doctor: DearWe understand your concernsI went through your details. This panic attack should be related to your natural reaction to alcohol. These things do happen. You should not venture to take liquor when you are under panic disorder and anxiety disorder treatment. You should tell your ER everything possible. Do not conceal anything. Take care.If you require more of my help in this aspect, please use this URL. http://goo.gl/aYW2pR.Make sure that you include every minute details possible. Hope this answers your query. Available for further clarifications.Good luck."
},
{
"id": 193472,
"tgt": "Suggest treatment for natural emissions due to excessive mastrubation",
"src": "Patient: i have stopped masturbating for over 5 years months and from time to time I suffer from nocturnal emissions, which I have come to know is completely natural. On other days however when i wake up i see dried up sperm on my underwear which was not there previously, what is the solution to this problem Doctor: Hello,I don't know why you stopped normal masturbation. How do you expect your continuously produced semen will be let out. If you need to stop masturbation, you need to have regular sex or master in yoga. Masturbation two to three times a week is normal.Hope I have answered your query. Let me know if I can assist you further.Regards,Dr. S.R.Raveendran"
},
{
"id": 142850,
"tgt": "What does a shadow in frontal lobe on MRI suggest?",
"src": "Patient: my girlfriend recently had an MRI. We were told that she had a shadow in the white frontal lobe. we were also told that on the MRI the technician wrote that she had bascile grangula in the white matter. What exactly is this? We have an appointment on the 25th of this month. Thank you Doctor: Hello,I am Dr Mittal.It is difficult to tell you what it is based on the information given.However, this is probably an infection in the white matter.A shadow means that there is some lesion- infection being a common cause in young.The granular tissue (not bascile grangula) is a type of tissue that develops in brain in certain types of infections.So based on the information, I think there may be some infection.Best of luck, Dr Mittal"
},
{
"id": 161264,
"tgt": "What could low appetite with stomach pain in children suggest?",
"src": "Patient: my daughter is 4 month old and has been having problems lately very unusually fussy, stomach problems, neck sensitivity, eating less, restlessness, not sleeping as musc at night, and screaming and crying alot more than often also very red cheeks... what might be wrong? Doctor: Hello, 4 month old baby will have colic pain in the abdomen. it is common in babies of this age. You can give antispasmodic drops for the baby. she will outgrow the problem. Hope I have answered your query. Let me know if I can assist you further. Take care Regards, Dr Rajmohan, Pediatrician"
},
{
"id": 101500,
"tgt": "Suggest treatment for coughing and mucus",
"src": "Patient: I have really bad allergies and I at least once a day I cough and have to it out mucous a. It takes a Long time to bring it up and it is a very scary for me and others to listen too. I have had ct scans and was seen by a pulmonologist. Wheat can I do to bring the mucous up without chocking. Doctor: hi take mucolite syp twice daily and take inhalation with easybreathe capsulestwice dailythank you"
},
{
"id": 163263,
"tgt": "What does this blood test result indicate?",
"src": "Patient: Hi doctor, my daughter is 8 yrs. She suffering from fever past 8 days. I hv done blood test. All seems negative and normal. But esr is 33/hr normal is 0-20, Rdw-sd is 35.38 while normal range shows 39 to 46, Eosinophils 0.2% where normal is 1 to 6%,Neutrophilis 65.3 where normal is 30 to 60 ...Is the report OK??? Doctor: Hello,Nutrophil count is little higher. Seems like bacterial infection is causing fever. Check for sore throat or ear or throat infection or urine infection. Rule out these causes and start antibiotics accordingly. Hope I have answered your query. Let me know if I can assist you further.Regards,Dr. Hina Javed"
},
{
"id": 144171,
"tgt": "Suggest treatment for dizziness and numbness in the hand",
"src": "Patient: Hi. I am extremely dizzy and i get an pins and needles numbness onthe rightside of my mouth and my Right Hand. I had a stomac bug earlier in the week and I went to my doctor on wednesday. i went again on friday as the dizziness did not go away. but from today i get pins and needles in my face and hands acompanied with the dizziness. Please can you help? Doctor: Hi, I am Dr.Bruno. I have read your question and understand your concerns. Let me try to help you Question : Suggest treatment for dizziness and numbness in the handAnswer : The Exact treatment depends on the cause of dizziness and numbness. This can be a simple neuropathy or some problem in posterior fossa of brain. The latter needs immediate attention. So I suggest you visit a Neurosurgeon at the earliest and rule out this. Then symptoms can be managed easily by proper treatment Hope you found the answer helpful.If you need any clarification / have doubts / have additional questions / have follow up questions, then please do not hesitate in asking again. I will be happy to answer your questions.Let me know if I can assist you further.Take care."
},
{
"id": 9620,
"tgt": "Can dark spots on the leg be treated with aloe vera plant and how much time does it take to fade away ?",
"src": "Patient: how long does aloe vera plant take to fade away the dark spots on leg which is caused due to scratching.... Doctor: hello welcome to health care magic it will take 4 months if you apply daily once.please dont consume it if you are pregnant ,planning to get pregnant,breast feeding. take care payal"
},
{
"id": 114655,
"tgt": "Having tested for mitochondrial disease",
"src": "Patient: My current neuro is having me tested for mitochondrial disease.ive had symptoms for last year they just seem to keep coming. Neuropathy in lower legs was first symptom but it s never gone beyond knee level. Over last three days I can t seem to get legs walking right, no extra pain, just feels like I m having to drag them with me. This has never happened before and I m very worried. I will email my neuro in Monday but it s Saturday and I m freaked out that it hasn t gotten better Doctor: Hi welcome to the health care magic For neuropathy usually diabetes and vitamin b 12 deficiency ruled out first and treated accordingly if needed Your neurophysician might suspecting mitochondrial disease as obvious cause might not be found.... So you can investigate for that using molecular techniques If needed MRI like investigation useful to do further work up According to cause further specific treatment given Take care Consult neurophysician and follow his advise"
},
{
"id": 204836,
"tgt": "What are the symptoms of Alzheimer s disease?",
"src": "Patient: I think I might be getting Alzheimer s disease. Can do my cashier job OK asked my boss no problems there close front office lock up store set alarm etc. Having problem when talking can not find the right word that I want to use. Mixing by words up calling breakfast supper no as soon as I say it correct myself . No problem driving balance checkbook workout 3 to 4 times a week do things with friends. Husband has a lot of health problems in last 3 years had 4 major surgery. Very stressful 4 me. I am 66 years old work part time. Eat pretty healthy do not drink smoke no coffee pop drink water not on any medications. Do I need to see my doctor? Email address janrob4040&gmail.com Doctor: in my opinion symptoms of alzeihmer being memory disturbance which interferes with daily functioning word finding difficulties later on having difficulty understanding speech in some individuals planning , , organising also can be effectedlater on personality changes can be seen difficulty in doing calculationsdifficulties in understanding environment can be seen later on difficulties controlling bladder is common"
},
{
"id": 63797,
"tgt": "What causes bump in vulva while using cream for warts?",
"src": "Patient: I was diagnosed with genital warts/low risk hpv a little over a week ago I started to use the cream prescribed to me and my Vulva has developed a rather large (for the area it s occupying) flesh colored bump on it. Granted the area is red, sore and highly painful. I m unsure if it s from the cream or if it s a wart. All I know is it s very unpleasant. Doctor: Hi.Thanks for your query and an elucidate history.Read and understood your history of getting the enhancement of the lesions of the genital warts. This may not be related to the cream that you are using. Alas, stop the cream immediately; consult your Doctor and get a plan for cauterization of these warts and get a proper treatment of other conditions you have been suffering. It is possible that you are needed to get your partner too checked and get him too treated."
},
{
"id": 25236,
"tgt": "Suggest remedy for tachycardia in a person with high BP",
"src": "Patient: I am 54 year old female, about 30 pounds overweight, perimenopausal, diagnosed with high blood pressure in 2014, take 1 ten milligram lisinopril daily, left side heartbeart pounds in ear for past two years when I lay on left side, both arms go to sleep during the night depending on side that I lay on, left side has bad circulation all the way from shoulder to toes, have scoliosis since childhood, neck gets stiff on left side but is helped with chiropractor, don t know if bone in neck is pressing on nerve on left side or heart issue, ekg is normal and chest xray normal. Doctor: Thanks for your question on Healthcare Magic. I can understand your concern. Since your ecg is normal, no need to worry for tachycardia. Yes, possibility of nerve compression on left side of the neck is more likely in your case. You are also having Scoliosis (vertebral deformity). So this can lead to nerve root compression and subsequent left sided upper limb stiffness, numbness etc. So better to consult orthopedic doctor and get done clinical examination of spine and MRI of spine to rule out vertebral pathology. Hope I have solved your query. I will be happy to help you further. Wish you good health. Thanks."
},
{
"id": 118087,
"tgt": "What is the treatment for triglycerides?",
"src": "Patient: Hi doctor my age is 38 years i am 5.6 Ft with a weight of 80 Kgs recently i went for a TMT test, test findings are poor tolerence and regarding my other medical tests i found my triglycerides are at 258 which are very high compared to normal range ie., 110 etc. Even My VLDL is 51 mg which is than normal what kind od medications and exercises i have to do to control these Doctor: You are overweight. Now your triglycerides are high, so do the vldl. TMT shows exercise intolerance signifying you have a heart problem. The best way to combat all these problem is to modify your dietary habit and increase daily exercise. Avoid junk food, fried foods. Eat green leafy vegetables, one fruit per day. Resttict calories to 1800 kcal/day. Avoid ghee, dalda, butter. Alcohol not more than 40gm per day. Daily exercise of 25 min like jogging and running can help. Though medicines like fenofibrate can control your triglycerides, I would like to see other part of your lipid profile report before advising to take that drug. Wish you good health."
},
{
"id": 2024,
"tgt": "Suggest ways to thicken endometrium",
"src": "Patient: dear Dr i am 39 yrs old n trying to conceive. i went through 2 cycles of Ivf but in vain.some of the embryoes are keept frozen so tihs month i have to go for third cycle. my doc said my endometium growth is weak thats the reason it is not able to hold embreyo.now im on luprid injection.what more i shoud do to make the endometrium strong n will i be able to cpnceive? Doctor: Hi I think you should talk to your doctor regarding this. Medicines are available which can increase the thickness of endometrium.Some ayurvedic medicines are also available. You can try that. But one thing you should understand that everyone's body is different and they respond to medicines in their own way. So don't get stressed too much. Do some endometrium focused yoga. These are also available. You can attend such sessions before transfer."
},
{
"id": 157067,
"tgt": "What is the treatment for vomiting when suffering from melanoma cancer?",
"src": "Patient: my husband has been vomiting since 4pm yesterday afternoon, he cannot keep anythingdown, have been trying to get him to drink the hydralite but it keeps coming back up. my husbandhas melanoma cancer and is currently on a drug trial, I have tried to ring his oncologist to see whatto do, should I take him to the hospital? Doctor: good day! Oncology medicines are very criminally notorious for that. If oncologist is not responding go to ER and get the vomiting stop. It would be a challenge though. Good Luck"
},
{
"id": 131056,
"tgt": "What causes pain in left hand?",
"src": "Patient: Hi I have been experiencing pain in my left hand it feels like there is a bone out or moving about, its a dull ache which can get worse at times it is mostly based on top of my hand there is also a hard lump which I think is the bone it is very painful now that I am trying to type the pain is shooting up towards my wrist and arm the more I use it the more painful it gets what should I do ? Doctor: getting an idea from your pain while tying it is probably the scaphoid bone that is troubling you.can be necrosis if you had a history of fall on your handor else can be just an overuse pain which had lead to muscle spasm.stabilize the joint with a compression bandage and apply a contrast bath i.e apply ice (10 min)& hot packs (10 min alternating 3/4 times) over the pain area 4 times a day.and ray to confirm the diagnosis is advisablehope you find this helpful"
},
{
"id": 52696,
"tgt": "What causes an elevation in the liver enzymes while having fatty liver?",
"src": "Patient: I believe I am having a slight Pancreatitis attack. I have always had a fatty liver, but lately liver levels are really high, I believe I am indulging too much.I have a swollen stomach, slight pain in liver area, more pain in pancreas. in fact I feel some pain in my left side and back. Pain is not as bad as child birth as I hear some people say it should be.Is it possibly still pancreatitus? Doctor: Hi and welcome to Healthcaremagic. Thank you for your query. I am Dr. Rommstein, I understand your concerns and I will try to help you as much as I can.Yes it is possible.In most cases it can be regulated by medications which include insuline supplements and enzymes for digestion Also alcohol is forbidden in these cases and you should eat less fatty food, less sugar and more vegetables and boiled food. Also, avoid citotoxic medications. Eat a balanced and healthy diet and get regular exercise. Limit high-carb foods such as bread, grits, rice, potatoes, and corn. And cut down on drinks with lots of sugar like sports drinks and juice. This should lead to improvement. You should repeat your liver markers in 3-4 weeks.If there is no improvement then surgical therapy may be considered and include drainage procedures or pancreas resection. This is last option and you should consult your doctor about this.I hope I have answered you query. If you have any further questions you can contact us in every time.Kindly regards. Wish you a good health.I hope I have answered you query. If you have any further questions you can contact us in every time."
},
{
"id": 216455,
"tgt": "How to treat trigeminal neuralgia?",
"src": "Patient: Yeah I ve got triggamenneuralgia n if I don t snort a pill my entire right side of face n head hurts so bad that all I do is cry, my eye seeks like golf ball b runs infection down it n my ear feels like it s gonna explode n my head hurts from front to back, my nose Hurst my face my teeth feel like I wanna pull them out.what do I do Doctor: Hi..Welcome to HEALTHCARE MAGIC..I have gone through your query and can understand your concern...As per your complain and presentation of symptoms the first thing that you have to do is to consult a NEUROPHYSICIAN and get evaluated for the exact cause of the pain as in case of Trigeminal Neuralgia as many a times due to any large artery compression over the trigeminal nerve can lead to severe pain along the course of Trigeminal nerve and until the compression is not removed pain recurs when medication is not taken..In such cases surgical decompression of the nerve from the artery by a procedure known as MICROVASCULAR DECOMPRESSION can help in relieving the nerve pressure and can resolve pain..Hope this information helps..Thanks and regards..Dr.Honey Nandwani Arora.."
},
{
"id": 2703,
"tgt": "What does this ultrasound of uterus indicate?",
"src": "Patient: Hello everyone, Im 40 yrs.old with 2 kids 16yrs.old &15yrs.old.we trying for 3rd for 3yrs.but not success.we went to gyne for pilvic ultra sound.findings that i have 13.6 endometrial thickness with few nabothian follicles and free belateral adnexa..do i have chance to get pregnant?endometrial can solve by pap smear?thank you very much...i hope u can help me Doctor: Hi,I wanted to know regarding your follicles status. Endometrial thickness alone can not tell everything. You can conceive that is for sure but you might need some medicines for that.Hope I have answered your query. Let me know if I can assist you further.Regards,Dr. Khushboo Priya"
},
{
"id": 113071,
"tgt": "Pain in lower back, heard a loud noise, severe pain, no apparent bruise. Cure?",
"src": "Patient: Hi, i sneezed this morning and my lower right back started to hurt very badly. It felt like something got injured on the inside of my body. I heard a loud noise from my lower right back, i checked to see if there was a bruise there but there was not any sign of one. I have very severe pain and I m thinking of going to a hospital if i do not get better. Anything you can help me with doctor? Doctor: Hi, Thanks for posting your query It seems you have developed muscle strain over lower back following sneezing, due sudden jerk and pressure. To reduce pain and for better healing- Do hot fomatation at site Apply musle relaxant gel 2-3 times a day Take antiinflammatory drug like ibuprofen to reduce pain and inflammation. Avoid forward bending and heavy weight lifting. And rest well. Hope this will helps you Regards Dr Saurabh Gupta"
},
{
"id": 105446,
"tgt": "Have tobacco addiction and psoriasis. Family history of asthma and cancer. Advice?",
"src": "Patient: My husband aged 40 is eating 3 to 4 packets of gutka everyday mostly at night,since last 15 or more years. His weight is 80kgs & height is 5.5ft. His father has asthma , & is now ailing from foodpipe cance. He himself has psorasis. Would like to know about the concerning health hazards. He drinks around 3 pegs of whisky thrice a week. Please guide. Doctor: There are many health hazard that your husband is facing because of his personal habits. Most common in his case is cancer of mouth, tongue, throat and food pipe. If he is swallowing the tobacco, he may also be at risk for cancer of other organs. He will have poor oral hygiene which may lead to many dental problems beside the foul breath.Beside this, he may develop high BP, heart problems, bastritis, excess acid secretion and digestive problems. He is drinking in moderation which may not be very harmful but adverse effects of excess alcohol intake are liver damage, high BP and heart problems, gastritis, pancreatitis, effects on the brain and psychiatric problems. These are just to name a few and the list is quite long."
},
{
"id": 90298,
"tgt": "What causes abdominal pain and cramping ?",
"src": "Patient: I have had lower abdominal pain for a week. Feels like I have to go to the bathroom, some cramping also. Pain is dull. I have had regular bowel movements, but the pain is there. My appetite is less than normal but when I am hungry seems like the pain is more pronounced. Doctor: Hi.Thanks for your query.Your history>>lower abdominal pain for a week- increases on empty stomach- feeling like going to bathroom- This can be due to enteritis, that is inflammation of the intestines of mild nature not causing other symptoms to creep up.A course of an antibiotic suitable for intestinal infection like cefixime will be diagnostic if you are getting a relief and therapeutic if it is helping to get a cure with a course of 5 to 7 days.If no relief - you may please go for blood and stool investigations, ultrasonography of the abdomen to see the cause"
},
{
"id": 182568,
"tgt": "Suggest treatment for gum disease",
"src": "Patient: our oral health is connected to many other health conditions beyond your mouth. Sometimes the first sign of a disease shows up in your mouth. In other cases, infections in your mouth, such as gum disease, can cause problems in other areas of your body. Learn more about this intimate connection between oral health and overall health. Doctor: Thanks for your query, I have gone through your query.Yes, you are right, mouth is the mirror of the body. Like that the systemic diseases can manifest in the gums. For example, bleeding gums can indicate any bleeding disorders, multiple periodontal abscess can be seen in diabetes or periodontitis can be considered as the sixth cause of diabetes. Like this so many diseases can manifest in oral cavity and indicate a systemic disease.Regarding the treatment of gum infection, i would suggest you too get a scaling and root planing done. Followed by this, maintain oral hygiene, follow proper brushing habits and use mouth wash.I hope my answer will help you, take care."
},
{
"id": 168965,
"tgt": "What causes chest pain with pressure?",
"src": "Patient: I have an 8 year old daughter that has been complaining of chest pain for the last few months. She is 80 pounds. She is very anxious about everything. She is resting when she complains about this. She says someone is squeezing her chest and says that her heart is beaping really fast. I took her pulse rate and it was 101. But she has also has said it before and her pulse was at 68. She has had a murmur but they said it was innocent. Is this cause for concern? Doctor: one can feel his/her own heart beat only when it's above 150 and as long as the murmurs have been innocent as confirmed to you, there is nothing to worry about."
},
{
"id": 14839,
"tgt": "What to do if having rash between breasts spreading to the back and is itchy?",
"src": "Patient: Hi I have a reddish brown circular rash that originally started between my breasts and now it is spreading down my back the best way to describe the rash is some parts look like pure circles and other circles have merged to make several blobbs from top of my back across shoulder blades and is itchy when I'm warm Doctor: Hello,Thanks for the query,You might have developed fungal infection.The area under and around breast is very moist and is prone to develop these infections.Treatment can be with oral antifungals as well as with topical antifungals.Various antifungal powders like candid are availabe.These can help reduce the moisture as well as control the infection.Please dont use any steroid, it can worsen your conditionPlease meet a dermatologist for exact diagnosis.Let me know if you have any other doubt.you can ask a direct question to me on this forum, following the below link.https://urldefense.com/v3/__http://www.healthcaremagic.com/doctors/dr-rahul-kumar/64818Wishing__;!!Mih3wA!SBzm6_kI6hCZ58EPH6N_05MFfiPbxWXT0a2TJCdFQObRWm5mV5ur7hUOMa8clQ$ you good health,Thank you"
},
{
"id": 21653,
"tgt": "Is it normal to feel discomfort after putting stents?",
"src": "Patient: my husband has 2 stents put in on sat he now has 8 in all. After this procdure he hasnt felt well says he feels run down and just doesnt feel right. Hes never had this happen after all the other stents is this normal or something to be concerned about? Doctor: Hi ThereNo it's not normal to have chest discomfort after putting stents. I would like to suggest you to consult your cardiologist for this as the stents needs to be checked whether they are alright or they have developed some kind of obstruction inside. An ECG needs to be done and probably an Angiogram also to confirm everything is fine with stents.I wish him good luck and health"
},
{
"id": 212349,
"tgt": "Experiencing stress attacks, sleeplessness, trembling, weakness, anxiety, adrenaline surge. Done US. What can be done?",
"src": "Patient: Hi, I am 45y/o mum and I have been experiencing on and off waking in the morning around 3-4am and instantly going into a stress attack. I tremble and I clinch my jaws together also I am yawning constantly. I feel weak when I get out of bed because I feel the need to go to the toilet. I have had my heart checked out with a ECG also a Ultrasound and all is good, but this was because I was having elevated heart rate. They diagnosed me with Anxiety, but what I would like to know is when your heart races with an panic attack, is there something I can do to help me recover quicker and do to help my heart cope with the adrenaline surge. I ask because sometimes I get a aching in my shoulder blade and uncomfortable in my chest initially but then goes down to feeling sick and hurting at the top of my rib cage in the upper stomach area?Thank you Doctor: hello from ur complaints it seems like u are having anxiety symproms along with sleep problems as u are yawning and fatigued. these complaints alomg with early morning sleep disturbance may suggest depressive signs. its better to consult a psychiatrist to rule out depression. for the anxiety symptoms the options are antianxiety medications or exercise will help. thank u."
},
{
"id": 86227,
"tgt": "Suggest remedy for severe abdominal pain that aggravates while coughing or sneezing",
"src": "Patient: I have a pulling, dull, ache-y sensation on the lower right side of my abdomen. It is agitated when I sneeze, cough or try and pass gas or even when I simply am getting out of my vehicle. My GYNO said that my reproductive organs and surrounding areas look fine based on a recent MRI and vaginal ultrasounds. I am 32 year old woman. I do not have any bulging in the area in question, however, perhaps a hernia should not be ruled out? Help! Doctor: Hi. Thanks for your query and elucidate history.Whatever the ultrasound or MRI say, it is the clinical presentation that masters. Since you have rebound pain on movements as explained, there is an inflammatory process hat is going on. Consult a General Surgeon for clinical evaluation, investigations and proper treatment .Diagnostic Laparoscopy can give the diagnosis and elucidate treatment."
},
{
"id": 115044,
"tgt": "Suggest treatment for moderate anemia",
"src": "Patient: I am a 43 yo female diagnosed with moderate anemia due to heavy menstrual cycles due to fibroids and endometriosis. The finding of anemia were accidental on a check up for a new job. Initial H/H wad 9.7/32 with RBC 4.73 and normal WBC. I had labs done again by PCP which showed H/H of 8.8/28.9 wbc 13.9 nd rbc 4.80. Was seen in ER on 1/6/14 for pelvic pain wbc were 14.9, H/H 9.3/30, rbc normal. Er physician said labs results can fluctuate some because of different labs. Started taking iron supplements in Jan, got my period on 1/15, bled heavily for first 2-3 days nd then tapered off but bleed for 7 days. After that I knew my h/h dropped because i felt lousy. Cont with iron nd good diet. Had lab work done again on 2/14/14 and h/h was 9.2/30.7 rbc 4.78 and wbc is at 14.6 with neutrophils and monocytes slightly elevated but my lymphocytes are 5000 but overall absolute percents are within normal range. No fever, chills, slight night sweats but I have been having them since I turned 41. I have had a cold off and on for about two months because we keep circulating it in my household. My pcp referred me to hematologist and I am worried as I am to have a totsl hysterectomy. Why would I need to see a hematologist? Is there a concern for cancer? Doctor: Anemia itself not a disease but it is a symptoms of disease. You should find out cause. Blood transfusion and other medicatiob to increase your hb count would not be longer helpful.As u mention you have fibroid and endometriosis so you should take treatment for this. For current relief you may take iron suppliment."
},
{
"id": 226349,
"tgt": "Unprotected sex, forgot to take marvelon, took high dose later. Any side effects?",
"src": "Patient: hi I ve been taking marvelon 21 for more than a year now. yesterday I forgot to take the pill and had unprotected sex where my partner ejaculated inside of me. I had four pills (including the missed one) left on my packet. being paranoid I took three four pills this morning and am about to take the next dose from a new package as emergency contraceptives. may I know how soon will I get my periods and how to start my new pack after? also of any side effects . thank u Doctor: Hello I appreciate your query. Firstly, after one missed pill, you should take two on the next day. And use barrier method as a back up method, while finishing your pack for the rest of the month. However, in your panic, you have already taken too many pills, excessive dose in fact. I am surprised you didnt have any nausea / side effects. You have already taken sufficient dose to act as emergency contraception also. Please do not take any more pills all together Finish the rest of this month's pills normally ( one daily ), and use barrier ( condoms ) method for the rest of your cycle. Wait for your next period, as consuming more pills mindlessly will not help you now. Also, with such a high dose of pills, you could have your period earlier than usual. If you miss your period, perform a pregnancy test at the earliest, and opt for medical abortion, if you do not want a pregnancy. Take care."
},
{
"id": 26193,
"tgt": "Suggest remedy for high blood pressure",
"src": "Patient: I am diabetic pataint,I am looking for doctor near my home near gariahat, I also little moare about doctorrecently doctor recognise I have diabetic II type I am also suffering high blood pressure today I am feeling dizzy why I do not know what shoul I do? Doctor: there are multiple reasons for dizziness you did not give any info you could be having1. low sugar2. dehydration 3. neuropathy and postural hypotension 4. cardiac arrhythmia 5. obesity6. poor conditioning t/t depends on your condition"
},
{
"id": 212088,
"tgt": "Mood disorders, negative thoughts, uneasiness. Vitamin b12 140, tsh level 6. Thyroxine taken. Advise?",
"src": "Patient: IAM HAVING MOOD DISORDERS FOR PAST TWO MONTHS .MY VITAMIN B12 IS 140 BELOW NORMAL AND MY TSH LEVEL WAS 6 BEFORE TWO MONTHS BUT IT CAME BACK TO NORMAL WHEN I STARTED TAKING THYROXINE 25MCG.THEN IN BETWEEN I STOPPED TAKING THROXINE BUT THEN ALSO IAM HAVING MOOD SWINGS AND SLEEP DISORDERS. I AM HAVING NEGATIVE THOUGHTS THROUGH OUT THE DAY AND IAM FEELING UNEASY. THIS IS DUE TO WHAT? LOW B12 OR HYPOTHYROID ?SHALL I CONSULT PSYCHIATRICT ? SO PLEASE HELP ME TO COME OUT OF THIS.I AM ALSO DOING WALKING AND YOGA REGULARLY. Doctor: Dear We understand our concernsYour problem was due to thyroid over secretion.Thyroxine clears the problem for the time being.Your thyroid gland is supposed to be producing thyroxine.It is not doing its duty and therefore you had to take it as supplement.Stopping it will naturally bring back the symptoms.Because your thyroid is still in its old state.Listen to the advises of your physician and don't take decisions yourself.Wish you good recovery."
},
{
"id": 106502,
"tgt": "What can be done for pain in the lower back after an accident?",
"src": "Patient: Doctor, I am 45 Years woman, Yesterday I fallen on road by bike accident, I have no wounds in my body but my lower back is pain more. I went Doctor, Doctor proscribed aceclo-SR tablet one, I taken but no recovered Pain is much more in morning, no swelling pain is very I con't walk. Doctor what can I do? Which food, which exercise which medicine? I take Doctor: Hello, The pain in the lower back can be due to a back muscle strain. This needs proper bed rest, hot compress and oral muscle relaxants for improvement. Hope I have answered your query. Let me know if I can assist you further. Regards, Dr. Praveen Tayal, Orthopaedic Surgeon"
},
{
"id": 125648,
"tgt": "What causes severe abdomen/lower back/leg/pelvic bone pain when having fibroid/high calcium in blood?",
"src": "Patient: HI, I am a bit concerned over some symptoms and test results I have had. Three years ago I had an abnormal pap. After a cone biopsy it turned out I had precancerous cells. Fast forward three years later I am experiencing excrutiating pain in my abdominal area with pain on my lower back and shooting down my leg. At times the pain is so severe it feels as though there is a knife stabbing me right on my pelvic bone. I went for a D&C Biopsy and I am waiting for the results. During my medical clearance the doctor informed me of high level of calcium in my blood which has me nervous as I do not understand what that means. I am worried that my results from the biopsy may come back as uterine cancer. I do have a 2.5 inch fibroid that was found during the sonogram. Should I be concerned? Doctor: Hi, The current pain you have described could be due to lumbar disc disease. But high levels of calcium are significant. I suggest MRI of L-S Spine. Start mild painkillers in the meantime and wait for biopsy report. Hope I have answered your query. Let me know if I can assist you further. Take care Regards, Dr Gopal Goel, Orthopaedic Surgeon"
},
{
"id": 94677,
"tgt": "Abdominal pain. Had miscarriage. Spotting after starting birth control pills",
"src": "Patient: I have lower abdominal pain that hurts more the longer I stand. I had this while I was pregnant. I thought it was constipation (it still could be). I had my baby at 20 weeks and we lost her. That was over 7 months ago. The pain went away a few months after giving birth but is back for about a month now. I also had a 4 week long period and got on bc pills on 11/9. I am still spotting since I started the bc. Doctor: Hi thanks for your question Bleeding in between periods is called as spotting or breakthrough bleeding.This can occur with any birth control pill, especially during the first few months of use. What causes breakthrough bleeding with birth control is not always clear. It may simply take time for your body to adjust to the hormones in the pill or for your uterus to transition to a thinner lining ( endometrium) Hope this answer your question"
},
{
"id": 5468,
"tgt": "Trying to conceive, cyst observed, given oosure plus, had endometriotic cyst removed, was on pills. Medicine remove cyst?",
"src": "Patient: Hi, I am 33 yrs old. I had endometriotic ovarian cyst on left side& got it removed 2 yrs back.Since then I was asked by the doctor to take contraceptives to avoid recurrence of cyst. But From last 4 months we are trying again for pregnancy & stopped taking contraceptives pills. from last 3 months I am on folical study.This month a 14mm cyst was observed on rt side.Doctor has asked to take oosure plus tablets for 30 days & stopped the folical study for this month.I want to know whether oosure plus tablets will stop or remove the cyst ? In what way these tablets will help me ? Doctor: Hello thanks for your query. Oosure contains DHEA , which is given to patients with diminished ovarian reserve. this means that the capacity of your ovaries to produce mature eggs is subnormal. This is due to endometriotic cysts which replace normal ovarian tissue. This tablet is supposed to boost ovarian reserve, although its role is not very significant. You have failed to ovulate in this cycle, and most probably it is due to subnormal ovarian response as normal ovarian tissue is replaced with endometriotic cysts. These tablets help somewhat in boosting ovarian function. However, you need to have complete testing like AMH, AFC ( antral follicular count ), day 2 FSH, laparoscopy to assess tubal function and ovaries etc. You might require ART ( assisted reproductive techniques ) to get pregnant. Take care."
},
{
"id": 152488,
"tgt": "What causes swelling and discoloration in the lower legs while suffering from cancer?",
"src": "Patient: Yes, my mother recently came home from the hospital. She is fighting a return of her cancer and she was outside working in her garden. Her eyes got red and her face swelled up turned dark and started peeling. She had to go in the hospital. The doctor said it was a side affect from the medication she was on that caused her face to swell because she was in the sun. Once she came home from the hospital, her lower legs and feet started to swell. They are a dark bluish color. I believe that the blood is not circulating back up. The nurse wrapped her legs. Do you know what could be the cause of this? Doctor: Hello and Welcome to \u2018Ask A Doctor\u2019 service. I have reviewed your query and here is my advice. The cancer patients may have a lot of complication due to the disease. If the legs are like you had prescribed then she may need ECHO Doppler. Hope I have answered your query. Let me know if I can assist you further."
},
{
"id": 186712,
"tgt": "Has the tooth infection gone to the bone?",
"src": "Patient: Abscessed Tooth Gone Too Far?First, I don't have dental insurance. second, here is my storyA little over a month ago, I Started having pain in my teeth. I took ibuprofen, And ignored it. now the pain is not so bad, but the swelling is. I did a round of antibiotics, they did not help. now I feel dizzy confused, and am having heart palpitations. getting a fever. feeling unwell in general. has the infection gone to the bone? should I go to the emergency room? Doctor: helo thanks for consulting at hcm..it is cellulitis which has led to swelling and u may need an higher antibiotics and analgesics,,u r having fever hence dehydration possibly,,kindly do an iopa radiograph and incision and drainage may be requd since ur medicines r not working,,plz consult asap ur dentist for i and drainage,,hope it helps,,tc.."
},
{
"id": 110038,
"tgt": "What causes back pain?",
"src": "Patient: i am a 45 year old man, 6'2 210 pounds. over the past couple months i have been having back pain in the middle of my back. i have been to a chiropracter 3 times but don't get any relief. for the past few days I have been having some tightness in my chest, a little to the right side, minor shortness of breath. I consider myself pretty fit and active. 4 years ago I got to feeling bad and I thought I may be having heart problems ( have GERD) and went to the ER but they said no, just GERD flaring up. I had a treadmill stress test to verify that diagnosis. I dont have any family history available. Am I worrying to much? Doctor: Hi,Welcome to healthcare magic.After going through your query I think your You are suffering from chronic backache.Treatment of back pain is exercises and analgesics (diclofenac SR). Sometimes strong analgesic(such as ultracet three times a day after meals) is required.Consulting with treating doctor is advised. Avoid long continuous standing.Sit in a straight posture.Eat milk, fruits and green leafy vegetables daily.You need to done MRI TO RULE OUT other causes.For chest pain you should consult physician to rule out heart related and lung related causes. I think your query answered."
},
{
"id": 150492,
"tgt": "MRI report showing disc desiccation noted on L4 L5 levels, paracentral disc protrusion. Worried",
"src": "Patient: My MRI report states that Disc Desiccation noted at L4 -L5 and L5 and S1 Levels. and Central and bilateral Paracentral Disc Protrusion noted at L4 -L5 and L5 and S1 Levels causing thecal compression with narrowing of bilateral lateral recess /neural foramina and impingement over bilateral exiting nerve roots at L4 -L5 and right exiting nerve root at L5 and S! level.pls help me to understand my MRI report. i m very worried about my position .please tell me the medication so it ca be completely cured. Doctor: Mri shows nerve root pressure due to disc protrusion, You should be having root pains as well as pain radiating to legs .Brace for lower back, avoid forard bending and lifting of weight,take anti-inlammatory/analgesics,do back strengthening exercise. If pain increases over a period of time interfering with routine activity and unbearable, you may have to undergo laminectomy/discetomy surgery. Seems it is tolerable now so you may remain on conservative treatment till otherwise."
},
{
"id": 109638,
"tgt": "Suggest treatment for back pain",
"src": "Patient: hi, i walk all day and when i sit, i felt my back is in pain. when i lie down in my bed, i cant get up. it feels like im paralyze. after that my legs is in pain and i feel dizzy sometimes and i feel like vommiting. i am afraid that it may lead to kidney failure, i have the result of my urinalysis.. for color:yellow, glucose:negative, reaction 6, transparency:slightly turbid, albumin: negative; specific gravity:1.01...pus cells:0-1/HPF;amorphous substance urate phospahte: rare/LPF.. thank you and GOD bless Doctor: Hello....no sir,your problem is not related to your kidneys ....(1) you r suffering from P.I.V.D i.e prolapse intervertebral disc in the region of lumbo-sacral joint (2) driving and staying in the same posture for long like standing or sitting causes MUSCLE SPASM in the lower back region which causes pain n discomfort (3) 1st of all get a M.R.I of lumbosacral spine to see how many discs r involved (4) do exercises to strengthen your lower back muscles,any good physiotherapist can guide you to do which exercises n which 1 to avoid (5) use a LUMBAR BELT while driving and while sitting for long hours (6) dont stay in a single posture for long hours and keep changing your postures (7) start TABLET NUCOXIA-MR 2 times a day and CAPSULE MEGANEURON 1500 mcg once daily...take these medicines for 4 weeks....send me a scanned copy of ur MRI report....follow all what i told u n trust my words,u will be completely free of ur agonizing pain n discomfort....regards....Dr Rahul Rathee"
},
{
"id": 167807,
"tgt": "What causes crying without tears in infants?",
"src": "Patient: My 10 Month old son has never cried tears but is not dehydrated and his eyes are not dry or red. I took him to an eye specialist and he said it had nothing to do with his tear ducts but it could have something to do with his glands. They might be itchy because he rubs them alot but i always just thought that was a sign of being tired. I really need reassurance that this is normal and that there isn t anything wrong. Thank you. Doctor: some children don't she'd tears unless they are in extreme condition or pain . the most common cause of having no tears when crying but having a wet eye is nasolacrimal duct obstruction in which your ophthalmologist excluded , and it's treated with saline drops along with massage near the nasal bone from the inside .if there is no tears at all or marked decrease in tears it's called alacrima, which can be isolated ( isolated congenital alacrima ) , which is the bilateral abscence or being very small tear glands , which can be diagnosed by orbital CT scan or by a neostigmin test which is done at ophthalmology clinic , but this is very uncommon I hope this helps"
},
{
"id": 118777,
"tgt": "80 years. Frequent low BP after taking bike ride, eating breakfast and drinking 1 qt plus. Concerning?",
"src": "Patient: I am 80 yrs old. At 0600 this am my BP was 147/92 pulse 55. I went on a 55 miles bike ride ,avg 11 mph. Stoped for a good bkfast , drank 1 qt plus. BP 15 min after ride 89/55 pulse 77. Three hrs later BP 124/79 pulse 72. This situation has been happening even with shorter rides(but more intense) Temp today, sun & 78F. I'm a bit worried. Doctor: Hello, Thanks for the query to H.C.M. Forum. The Blood pressure is not too high , but the cause of concern is bradycardia ( decreased pulse rate 55 / minute). 2nd time after breakfast B p was low . In my opinion this is not related to bike riding , but may be due to some heart ailment . 55 pulse / minute is an indication of cardiac failure so please consult a cardiologist and get his opinion. Good luck. Dr. HET"
},
{
"id": 141947,
"tgt": "How to overcome the anklosis spondylitis?",
"src": "Patient: sir, i m suffering from joint pain on whole body,specialy in my neck. recently doctor dignosed me as an anklosis spondylities. right now i can not move my neck. i want to know that, is it cureable or not by an accuepresser treatment. kindly suggest me what to do. Doctor: Hello!My name is Dr. Aida and I am glad to attend you on Healthcaremagic!This is a serious medical condition, which may be treated with several treatments to slow the progression of the disorder and improve the situation. You should consult with a rheumatologist on this issues. Painkillers and acupressure may help relieve the pain for a short period of time, but they are not the main therapy for this disorder. Unfortunately the disorder is chronic and progressive and there is no definitive cure. Hope you will find this information helpful!Best wishes!"
},
{
"id": 80863,
"tgt": "Is pain in armpit,chest and mid back symptoms of lung cancer?",
"src": "Patient: I had acdf surgery to c5/6/7 in October. The pain is far worse than before I had it done. Pain is now under my armpit and in chest and misd back. Had facet joint injections in January but they haven't helped. Could this be a lung cancer. I keep reading about pancoast tumours and now I AM VERY WORRIED Doctor: I feel your symptoms are related to nerve compression related to your cervical spine.Are you suffering from cough, breathlessness, loss of appetite or a weight loss?? If you are worried about lung cancer then an Xray will answer your doubts."
},
{
"id": 81118,
"tgt": "What could cause cough?",
"src": "Patient: I have had a cough for about a year and here recently I see that I cough up blood in the mucuse...it s not a lot but enough to make me wonder. I worked at crystal sugar and was subject to breathing lime dust among othere dusts. Could this possibly be linked? Doctor: Thanks for your question on HCM.I can understand your problem and situation.In my opinion you should consult pulmonologist and get done1. Chest x ray2. PFT (pulmonary function test) to rule out chronic lung infection and chronic bronchitis.Yes, you are right about crystal sugar and lime dust.Chronic exposure of both of these can cause pulmonary diseases.And blood in sputum (hemoptysis) is something, you should worry. It suggest worsening of underlying lung disease.So better to first diagnose yourself and then start appropriate treatment."
},
{
"id": 157284,
"tgt": "What is the treatment for retroperitoneal fibrosis and blocked urethra after having a stent placed?",
"src": "Patient: my husband has retro peritoneal fibrosis. his rite uretha is blocked and a stent was placed. he has a history of colorectal cancer 16 years ago. A biopsy was performed and was negative for malignacy. His oncologist suggested he see a pain management doctor. Do you think he she see a nephrologist as well Doctor: Hi...there is no cure for retroperitoneal fibrosis... management is purely palliative... you can see both of them...they will do their best to help..Dr. Ashish Verma"
},
{
"id": 101538,
"tgt": "What causes stabbing pain all over the body and tingling in hands?",
"src": "Patient: I have been to numerous doctors regarding pin and needles sensation all over my body, primarily in my whole back and upper torso, sometimes I feel my feet burning and tingling in my hands. My sugar is normal, thyroid is normal, no MS, what can be causing this problem Doctor: Hi and thanks for the query,You might need a careful evaluation to clearly distinguish itches from pain or pin like sensations. It is of interest that diabetes has been excluded. Blood filarial worms and Vitamin deficiencies should be checked first. Regards"
},
{
"id": 136222,
"tgt": "What causes pain in ribs below the breast bone on the right side?",
"src": "Patient: Hello,I am Okediji, Victor Mishael by name. I will like to make a complain about untraceable pains I use to experience by the right hand side of my ribs below the breast bone,right on one of the costal cartilage,within the right atrium arena,against the tommy wall, and it also use to extend to the back below my scapula. The pain always occur within my skin layer,and I use to feel it like when someone is having burns.The skin around this spot sometimes swell up, and i treated it before i left my home country to study in America. When I came to USA, I went to my School Health center, they did some test and X-ray. but they couldn t trace anything. but, I am not comfortable with it because the pain is still occurring. Secondly, I don t really understand the nature of my penis.Early in the morning or at night it will look appreciable. but, when ever i am taking my bath or shower, it will become shrink, and at time it won t. It use to be very small and time when ever it shrinks. I will like you to give me possible solution to my health problem.I am a martial arts athlete, and my health status is really discouraging me. In conclusion,I have my health issureance certificate as an international student. Doctor: hiI may suggest you consult a physician and possibly,ultrasound of liver, gall bladder may be done to rule out problems there.Take Pantaprazole tabs and do fomentation of chest area and apply local pain cream after fomentation.you can write back to me at my email suniliskumar5@aol.com through HCM.com.An orthopedic consultation may also be needed.best wishes"
},
{
"id": 54254,
"tgt": "What would an increasing bilirubin count imply?",
"src": "Patient: Hello doctor, I have had jaundice for around 1.5 weeks now. After taking medicine, the SGPT count has gone down (~2000 to ~ 1000 to ~ 600) but the reports have Total Bilirubin count increasing (4.1 to 5.4 to 8.5). What would this imply? I am 19, 63kgs. Doctor: Hi, dearI have gone through your question. I can understand your concern. You have jaundice with very high SGPT. You may have some viral hepatitis. It is common that in viral hepatitis initially there is increased in SGPT level. Later on there is increased in bilirubin level. And there is early fall in SGPT level. Your bilirubin level will decreased within few days. So don't worry about that. Continue your treatment accordingly. Hope I have answered your question, if you have doubt then I will be happy to answer. Thanks for using health care magic. Wish you a very good health."
},
{
"id": 66282,
"tgt": "What is the treatment for lump or lymph nodule on head for 4 months old baby?",
"src": "Patient: When my son was 4 months old we noticed a small lump like a blister on the back of head. I took him to the Doctor who said it was a lymph node and not to worry. It went down but has now at 6 months come back we have also noticed it is weeping. Should we b worried? Doctor: Hi, thanks for writing to us!In the present case of recurrent lump on the back of head of your son, if I were your consulting physician, I would also just say you, 'nothing to worry!'As per your description, it disappeared some times back and now it re-appeared; therefore it is simply an enlarged lymph node due to infection in local regions like scalp, ear, throat, neck etc...!However, sometimes it could be converted into an abscess or may harbor tuberculosis and needs treatment...Take caution not to get him injured there as this might invite infection.Please do an FNAC test for confirmation and plan line of treatment if needed at all!Hope you got your answer. Please feel free to write to us if any more queries.Wishing your son good health!"
},
{
"id": 92247,
"tgt": "What is the treatment for endometriosis leading to painful menstruation and bowel movement?",
"src": "Patient: Can you please take 15 minutes out of your time to talk to me for free. I will be thorough and QUICK. I do not have health insurance, i am 28 yrs old working as a Nanny. Ive had endometriosis for many years and saving up for the procedure. My clock is ticking and i am freaking. My horrible pains are not only during menstruation but now having them before and after. Especially during bowel movements which is really starting to worry me and the pain is very bad. YYYY@YYYY Doctor: HIThank for asking to HCMThis type of pain does not need any treatment and this will be fine without the treatment, just keep your stress level low then you will definitely feel good medicine will create some other problem take care and have nice day."
},
{
"id": 203492,
"tgt": "What is the remedy for difficulty in erection during intercourse?",
"src": "Patient: I am 41 years old married person. I do not have any intercoursein 8 years married life. My penis have normal erection but when Itry to intercourse, the hardness of the penis is fall or sometimes not.I use Penegra 50 sometimes but result is same. Sometimes I could notconcentrate in sex. Lubrication of my partner is not enough. Doctor: Hello dear,Regarding the problem of Erectile dysfunction, it is due to decreased blood supply to the penis.Though medications like Cialis & Viagra increase blood flow to penis & maintains erection, it can be improved by having a diet rich in fruits, vegetables, fish, nuts & honey.Connections between erectile dysfunction & excess use of porn has been reported in many studies.For those affected, recovery takes place by 6-12 weeks & mainly involves avoiding the causative factor, that is watching porn.Avoid stress...practice meditation..this helps in improving concentration & builds up the confidence level.Avoid smoking & alcohol.Despite these measures, if you experience difficulty, it will be better to rule out other causes like high blood pressure, blood sugar and other local causes that impair blood flow to the genital organ.One more thing...avoid consuming any over the counter medication which claim to increase the size of penis or improve performance...it may be harmful.Wishing you a Healthy Life.Take care."
},
{
"id": 175960,
"tgt": "Why infant is having bloody diarrhea, red spots on face and nasal congestion?",
"src": "Patient: Hi good morning My baby is 2 months old and 10 days ago he started green bloody dierria after we checked his stool , result was good no any problem with his poop , the Doktor told us stop breast feed and give him soy formula first we gave him 12 hours pedialyte and we start soy formula next 2 days he was ok but Doktor told us now you can start breast feed again when I start breast feed he got red spots on his face like egzama and he was very irradiated looks like his face itching, he is caughing like a dog barking he has nasal congestion and yesterday he vomited 3 times but very very much and we gave him pedialyte again he got fishy smells poop Doctor: Hi...I went through the history points you have provided - I was expecting that there should have been a history of cow's milk and formula feed (this too contains cow's milk protein) given to the baby. I feel that your baby is having cow's milk protein allergy. Unusually babies grow out of this sort of allergy by 1 year of age. My suggestions for you - 1. Mother should go off cow's milk protein completely. This means that you should avoid consumption of anything and everything related to cow's milk - like - milk/ curds/ ghee/ butter milk/ chocolates/ biscuits/ ice creams etc. Even while buying commercial food products, you need to see the ingredients and if they contain milk - do not consume them.2. Feed you baby only exclusive breast feeds till 6 months of age and then start rice based feeds. 3. If this is followed scrupulously - the baby will stop having diarrhoea in another 3-4 days and then start gaining weight too.4. If you feel your feeds alone are inadequate for the baby, then Zerolac is the only option. I request you to keep me posted about the recovery of the baby and follow of the case.You can approach me at the following link. Please find the link below - www.healthcaremagic.com/doctors/dr-sumanth-amperayani/67696Regards \u2013 Dr. Sumanth"
},
{
"id": 48853,
"tgt": "How to interpret renal tract ultrasound result?",
"src": "Patient: Hi Doc, I'm 66yrs.old,and just picked up my Renal Tract Ultrasound result. The conclusion is :- 1. Mild prostatomegaly and 2. Mild post void residual 36cc. Can you please advice whats this all about, as I can't see my doctor till late next week ?? Is this a serious condition? Many thanks in advance. Tony from Doununder. Doctor: hai,As your ultrasound result indicates prostate gland ( gland which lies on either side of the urethra) enlargement. it is common in every men aged above 60 years.Mild post void residual 36 cc indicates retention of urine in your bladder after urination.prostate gland enlargement (prostatomegaly) will cause urinary retention and urinary incontinence.this is not a serious condition to address immediately.Thank youhope i answered your query"
},
{
"id": 82009,
"tgt": "What is the treatment for severe chest pain?",
"src": "Patient: I sometimes have horrible chest pains on theleft side of my chest under my breast. It is exscrutiating pain. It hurts so even worse when I try to take a deep breath. Tonight it lasted for about an hour but I still can t take the deep breath that i need to take. What is this? Doctor: HIWell come to HCMYour age and gender are missing here these was needed here for right advise, now this could be muscular pain but as such this is on left side so better to get done the EKG test to rule out the cardiac disease, else this could be treated with any NSAID drugs and the best option would be Tab Diclofenac 50 mg once in day, hope this information helps, have a nice day."
},
{
"id": 189174,
"tgt": "Have teeth with cavities and weak gums and sensitive. Can go for clip treatment? Can worsen gum?",
"src": "Patient: i am 35 years old and planto put clip for my teeth as it looks bad. I already have more teeth with cavities and my gums too weak as i get pain oftenly as well as sensitive. Can i go for clip treatment as my doctor told its one year process only. Will it pain too much and how many days the pain will last. Will it worsen my gums further.I am in a panic. Please do advise Doctor: Hello there , Thanks for writing your query, Orthodontic wiring or clip treatment is better to be done on the sound teeth with good gum support. So before going for orthodontic treatment i would suggest you to go for complete oral proplyaxis and treatment rendering good oral health. cavities in the teeth can be filled with restorative materials. Weak and sensitive gums indicate gingival infection. It needs to be treated by thorough professional deep scaling of gums and root planing of the teeth to remove all the irritants . This will help in resolving the symptoms followed by a course of antibiotic and analgesics. i would suggest you to go for symptomatic treatment at home like gargling with lukewarm saline water or antiseptic mouthwash like chlorhexidine or betadine. i hope this helps , take care."
},
{
"id": 90471,
"tgt": "Why is my abdomen distended and bloated?",
"src": "Patient: I had an hsg 3 days ago. The cramping was tough during the procedure but very mild once it was over. However, the last 2 days I have been terribly bloated, to the point I can't sleep and my abdomen is so distended. It looks like I swallowed a basketball. I am so uncomfortable. Any thoughts? Doctor: Hi.Thanks for your query.HSG- (hysterosalpingography) is a procedure in which the dye is pushed into the uterus through a canula put into the uterus.The possible cause for distension can be a reaction to the dye or perforation of the uterus and trauma to the intestines. -You need to visit ER/ get admitted -Be nil orally - do not take anything orally.-get investigated by :Standing X-ray abdomen, ultrasonography, Ct scan.-Follow the advise of the Doctors. The treatment will depend upon the findings on investigation."
},
{
"id": 83492,
"tgt": "Does Utrogestan intake lead to negative pregnancy result?",
"src": "Patient: Hi There, please could you help me, i have taken 5 pregancy tests all were positive, i then went to the dr they took a blood test but came back inconclusive, he said to me because it was still very early, the day after i started bleedig it wasnt a flow but it was more then spotting, i went back they said it was difficult to say, they gave me utrogestan. i took another test and it came back negative (hpt) is it possible that the utrogestan played a part in the negative result? or did i miscarry? Doctor: Hi,Utrogestan contains female hormone progesterone which is essential for maintaining pregnancy in the early trimester. There is no evidence that progesterone either in oral or vaginal route can cause miscarriage.Hence you could be experiencing a miscarriage due to other complications of pregnancy.Hope I have answered your question. Let me know if I can assist you further. Regards, Dr. Saranya Ramadoss, General & Family Physician"
},
{
"id": 37680,
"tgt": "What are the side effects of spider bites?",
"src": "Patient: I know someone who had gotten 11 spider bites a few years ago. Someone I know was bitten 11 times by spiders she has multiple physical problems since then. R. arthritis is only one. She has also been taken by ambulance for other problems. She was healthy before the bites. Doctor: Hello,Welcome to HCM,Only a few spiders are dangerous to humans. Most presumed spider bites are actually bites from other bugs.For your symptoms, I would suggest you to clean the site of the spider bite well with soap and water. Apply a cool compress over the spider bite location. If the bite is on an extremity, elevate it.Aspirin or acetaminophen and antihistamines may be used to relieve minor signs and symptoms in adults. Talk to your doctor if you have concerns.Thank you."
},
{
"id": 90359,
"tgt": "Is tiredness common post peritonitis?",
"src": "Patient: I had a ruptured appendix and peritonitis two months ago. I have never gotten over feeling so tired from the surgery. Do you think it is related? I ended up with a peritoneal abscess and then C Diff. I can t do anything for long periods of time without wanting to lie down. I am used to being very active. Doctor: Hi.Thanks for your query.History about perforated appendix and peritonitis 2 months noted. I do not think, it should be related to your tiredness after such a long time. It looks that you have not taken enough care after the surgery 2 months before. I would advise to undergo tests as follows: Blood tests for anemia, infection - WBC , ultrasonography to see any remnant problem in the abdomen."
},
{
"id": 29574,
"tgt": "How can boil like sores on the shin and foot be cured?",
"src": "Patient: I was in the hospital for 7 days last week being treated for COPD and a- fib, being released on Friday. On Monday I noticed a few small boil like soars on my shin and one on my foot. They appear to be pus- filled. I worry it could be MRSA. Can my doctor treat this in his office today? Doctor: Hello,Yes, you should definitely see your doctor today. MRSA can usually be treated in the office.Regards"
},
{
"id": 160785,
"tgt": "Suggest remedy for cough, cold, vomiting and loose motions in infant",
"src": "Patient: Hi, may I answer your health queries right now ? Please type your query Hi, My son is now 9months..he is having cough, cold (running nose) vomiting due to cough (I guess) and loose motions I went to see doctor he prescribed medications but no improvement. we used medication for 3 days. please advise as I am greaatly worried Doctor: Hi,If he is not having fever now, I think you need not worry, it is probably a viral respiratory infection which will get better in 5-7 days. I am not aware of the medications you already got. For such cases, I used to give antihistamine like levocetirizine and broncholdilators like salbutamol to help cold and cough.If not getting better in 3 more days, or if the child is lethargic and not taking feeds as usual, kindly get back to your doctor as we may need addition or changing of antibiotics and nebulization. Give ORS solution or salted rice water after each episode of diarrhea.Hope I have answered your question. Let me know if I can assist you further. Regards, Dr. Muhammed Aslam TK, Pediatrician"
},
{
"id": 115568,
"tgt": "Should I be concerned about my dropping blood RBC count?",
"src": "Patient: Hello, I received a physical and a CBC but was not ill. My blood counts were low (red, platelets, neutrophils, white) and was advised to stop drinking alcohol. I consistently had 2 drinks per night, every night for about 6 years. Six months later, my blood counts all increased after quitting alcohol (red form 4.01 to 5.22, platelets from 110 to 242, hemoglobin, hematocrit, neutrophils from 1.5 to 2.0), but my overall white cells went from 3.1 to only 3.9. Many things I read online associate low blood counts potentially to hiv. I have not been involved in risky behavior for many years. Should I be concerned? Thank you. Doctor: HiThanks for your queryBased on your query, my opinion is1. RBCs and platelet count is within normal limits, WBC count is on the lower side of normal.2. You should not be concerned about these values as long as you are asymptomatic.3. Low counts need not necessarily mean HIV associated. There are many more causes for low counts, commonly when the marrow is suppressed by infectious or hereditary cause.4. Since your counts are back to normal, di not both much.Hope this helpsRegards"
},
{
"id": 86923,
"tgt": "What causes burning sensation in the lower abdomen?",
"src": "Patient: Hello. I have had a burning sensation in my lower abdomen for around 7 weeks now (Also some weakness) all bloods were checked and Thyroid - all fine. I have a history of reflux, around 5 weeks ago I ran out of omeprezole and was taking one zantic tablet per day instead. This is when my symptoms became worse, I experienced black stools and slight constipation and then loose stools alternating. Also started to experience some difficulty swallowing (Not liquid or solids - just general swallowing). At times I had breathlessness and a heavy sensation in my chest,as well as some days where I was so weak I could hardly get out of bed. At this stage I sent in stool samples and had all bloods done again - all fine. My doctor told me to try taking more omeprezole (20mg twice a day). This was 2 weeks ago and although the chest pain has improved and the bowel movements are better - I still have the mild burning in the lower abdomen, the uncomfortable swallowing and at times a sort of tense neck with a tight feeling on the right side. The worst thing is that I still have periods of awful weakness. I'm beginning to think I'm going mad. I am becoming very panicky and very depressed at times. I was completely fit and healthy before this - it has all been rather sudden, my only ever complaint was acid reflux. I am 42 years old. I have no coughing or blood coming from anywhere. It's the weak ness that scares me the most. I have an OGD next week and am terrified as I think only something like cancer can cause this kind of weakness. Now I'm starting to loose sleep with the worry.Thanks for listeningJulie Doctor: Hi Julie.Thanks for your query.Read and understood your history related to the burning pain in lower abdomen as well as the Zantac and Omeprazole consumption.Since all the tests you say are normal in spite of the black stools, something is looking contradictory.I would advise you to undergo OGD and also insist on Colonoscopy and CT scan of the abdomen as there is a chance diagnosis of cancer as you are suspecting. But there is a more chance that there may just be a non-cancerous problemlike an infection or so.Get the investigations done. Take a course of an antibiotic, metronidazole, probiotic.Continue Zantac and Omeprazole too as their role of action are different. Get the treatment as per the reports of the tests and you will be fine."
},
{
"id": 140383,
"tgt": "What causes tremors after head injury?",
"src": "Patient: Hi! So, my friend this last fall got a concussion during Volleyball season because while we were putting things away after practice she got hit in the head. Ever since, she shakes constantly. She has what people call a tremor in her hands and sometimes other parts of her body. It only stops when she touches something. Shes worried shes messed up in her head. I think it s something else. Care to shed some light for me please? Doctor: Hello, Tremors after a head injury are indicative of TBI, traumatic brain injury or Neurological problems developing. She needs to get physically examined, get done EEG, MRI, CT, Neurocognitive tests & Nerve conduction tests, nerve biopsy. Get assessed & evaluated. Her treatment will depend upon the pathology detected. Damage to the mid brain may cause Parkinson disease. Closed head injuries cause tremors. She may be treated with medications, but if a response is not good she may require surgery. Hope I have answered your query. Let me know if I can assist you further. Regards, Dr. Nupur K, General & Family Physician"
},
{
"id": 216627,
"tgt": "Suggest treatment for pain in upper right quadrant,shoulder and groin",
"src": "Patient: I have had upper right quadrant pain for 4 weeks along with shoulder and right groin. Had a HIDA scan that came back normal. Ultrasound only showed one stone that was 3 millimeters. The dr. Doesn t want to really do anything what should I ask my doctor to get the relief I m looking for? Doctor: hi,thank you for providing the brief history of you.The Pain in the right upper quadrant can be due to the gall bladder stone as per your history which I am understanding.For the gall bladder stones most of the times it comes out with medication and incase if the problems become big than the excision of gall bladder is planner.The pain in the groin can be a reffered pain due to abdominal pathology and the shoudler pain I feel it's muscular.Also, you can request your doctor for further tests and understanding since the pain shouldn't become unbearable were the patients patience is broken.Regards Jay Indravadan Patel"
},
{
"id": 65504,
"tgt": "What does a movable painless lump on collar bone indicate?",
"src": "Patient: I have had a semi-soft movable painless lump (long lump not like a pea or marble) on my left collar bone for about 6 years now. It was diagnosed as a lipoma a few years ago. I was told don't worry about it. In the past year I have had a few flair ups on the left side with pain in the joint area and forearm. The forearm near my wrist is also swollen. I had it checked out about 6 months ago and the doctor said the swelling is probably cause by me sleeping, leaning, carrying a purse and seat belt etc.. on that side. Today while at a new doctor he felt it and said I should have an ultrasound because it might be CANCER! I was horrified! The area is slightly painful but the lump its self does not hurt. He then said wait a few days and if it stops hurting forgot about the ultrasound.??? I would think if it was cancer or I had cancer i would have some other symptoms aside from some swelling and inflammation/possible arthritis issues on that side by now? I am 41 - female no history of cancer at all in my family. I am obese and have been for over 10 years (after my son died) aside from the obesity I am in good health. I am now terrified that i have cancer!!! YOU SHOULD STATE THAT YOU CHARGE A FEE BEFORE ALLOWING QUESTIONS! : ( THANKS FOR NOTHING! Doctor: Hi, dearI have gone through your question. I can understand your concern. Chance of canver in your lump is very rare. You don't have recent change in size, weight loss , fever etc. So your history doesn't indicate cancer. Most probably you have lipoma or some benign soft tissue tumor. You should go for fine needle aspiration cytology with ultrasound to search exact cause. It will be lipoma most likely. Still for your mental peace you should go for fine needle aspiration cytology or biopsy once. Don't worry about that. Just be relaxed. Hope I have answered your question, if you have doubt then I will be happy to answer. Thanks for using health care magic. Wish you a very good health."
},
{
"id": 67849,
"tgt": "What causes a lump on the head?",
"src": "Patient: i have a lump on my head that hurts i have been to the doctors and he said it was nothing but the lump has got bigger and more sore my mum died 2 years ago with some thing that happened in her head i dont no what to do as this scares me could it be the same thing Doctor: Hi! Good evening. I am Dr Shareef answering your query.The doctor who examined you might have commented so as it might have been a benign swelling. If I were your doctor and I found a swelling/lump in your head, I would advise you for a FNAC (fine needle aspiration cytology) to know the nearest diagnosis, and treat it accordingly. However, I would also advise you not to be scared of such benign lesions. Further management would depend on the physical examination and related investigation reports. Till then you could go for an anti inflammatory drug and a proton pump inhibitor drug for a symptomatic relief.I hope this information would help you in discussing with your family physician/treating doctor in further management of your problem. Please do not hesitate to ask in case of any further doubts.Thanks for choosing health care magic to clear doubts on your health problems. I wish you an early recovery. Dr Shareef."
},
{
"id": 112739,
"tgt": "Back pain after C-section and injection through spinal bone. History of brain surgery, TB, pneumonia. Opinion?",
"src": "Patient: Good morn, I am from Bougainville PNG my name is Lydia Karol. I have had c section and was injected through my spinal bone cartaligue and last month have been operated at Brisbane hospital with brain surgery(Third Ventriculostomy for the symptomatic obstructive hydrocephalus. I have a past history of TB and pneumonia. I am wondering why I have been have back pain since after my lastest operation. Doctor: HI, I AM DR.TIWARI.USUALLY DURING C-SECTION THEY GIVE SPINAL ANAESTHESIA WITH A VERY THIN BORE NEEDLE SO IT IS RARE TO CAUSE BACKACHE.CONSULT ORTHOPAED.TO ASCERTAIN THE CAUSE OF BACKACHE."
},
{
"id": 61185,
"tgt": "What does a lump in the lower abdomen on the left side indicate?",
"src": "Patient: Two years ago I had an ablation for atrial fibrillation. The operation was successful but I was left with a lump in my left(lower)abdomen that caused extremely painful attacks that lasted between 1 and 2 hrs. and continued for a year and a half about every 10 days. That petered out eventually and I no longer have painful attacks but still have the lump that sometimes becomes sensitive. I was prescribed hyoscyamine for the pain but never took them. I m still having bowel problems. Doctor: Hello dearWarm welcome to Healthcaremagic.comI have evaluated your query thoroughly .* There are so many different reasons which can present as lump in the lower left abdomen as - hernia - lipoma - sebaceous cyst - others .* Needs clinical examination and ultrasound evaluation of the same for the final diagnosis and deciding the line of action for further management .Hope this clears your doubt .Wishing you fine recovery .Welcome for any further assistance .Regards dear take care ."
},
{
"id": 62438,
"tgt": "What causes a hand growth with a lump on irritation?",
"src": "Patient: I have this thing on my hand where when I scratch it or if it gets scratched like by a paper cut, or just by someone else after a little while a bump comes on that spot and leaves after a couple of minutes. What is that and What can I do to prevent it? Doctor: Hi.Thanks for your query.Noted your history of getting bumps on the hand on scratching, or by a paper cut and it disappears after a couple of minutes. The probable reasons are:Exaggerated response of the body to a stimulus and this looks physiological as the bumps disappear within few minutes.Another reasons can be very sensitive histamine response of the body which the body takes care of immediately.Another possibility is that there was exposure to some allergens in the past which have caused this sort of hypersensitive response. I would advise you to undergo the tests of blood, urine and stool.Get a clinical evaluation, examination and investigations as may be decided by your Doctor.Special tests may be needed. Get a course of an antihistamine, steroids, dewormin and as may be suggested by your Doctor. Avoid any such things that can cause this phenomenon."
},
{
"id": 21879,
"tgt": "How to prevent my father's congestive heart failure?",
"src": "Patient: My dad is 84 yrs. old and on peritoneal dialysis. He's also had 5 bypasses. His pulse rate has been 33-35 and he has low blood pressure. His kidney dr. has taken him off all of his heart meds now and he appears to be having congestive heart failure again- has a bad cough. I'm very concerned. Any help I can get would be much appreciated. Thanks. Doctor: Hi There After going through the medical histrory provided by you of your father i would like to suggest that at the age of 85yrs, after going through bypass surgery being on peritoneal dialysis with such a low heart rate and low BP i would recommend you to take him to a multispeciality hospital as he is in a need of ICU care. He require a HOLISTIC approach of treatment and cardiac medications need to be started back with dose titrations. ICU care is recommended for him according to the condition described by you. Good Luck"
},
{
"id": 2632,
"tgt": "Can I conceive after having Diane 35 to regularize period due to PCOS?",
"src": "Patient: Hi i am sudha.i am 26yrs old.i am married for 6 months.i have 44 days of amenorrhoea & preg card negative..Before marriage my hormone levels are normal.my USG-abdomen &thyroid profile normal.my Gynaec doc diag me as pcos due irregular periods &hirsutism .i had Diane 35 for 4 cycles.now my cycles are 30-32 days.i have started meprate 5mg tds for 3 days.i am eager to get conceive. help me. Doctor: Hi,I understand your concern. Once your period becomes regular then it will increase chance of pregnancy. You can become pregnant after stopping diane 35, as it acts as contraception. You should ask following medicine to conceive:Stop diane and do regular sex for 2-3 cycle. Progesterone in second half of period: it supports implantation and early pregnancy. If pregnancy will not occur then following drugs will be useful. Metformin: to induce ovulation and decrease insulin resistance. Clomifene citrate: if there is problem in maturation of follicle. Treatment should be taken under advise of gynecologist and regular monitoring by USG scan is needed. Do regular sex during fertile phase of period. Avoid stress, take healthy diet and do regular exercise to maintain weight according to BMI.Hope this may help you. Contact further if follow up needed. Best regards,Dr. Sagar"
},
{
"id": 101180,
"tgt": "Should i be worried about the mild flu and coughing fits? I'm diagnosed with asthma",
"src": "Patient: in the past 2 days i have gotten a mild flu and developed a non-wheezy cough. At times, i can keep coughing to the point of wanting to throw up (10-20sec coughing fits). I am an asthmatic and have been taking my nebulizer (ventolin and atrovent) once every 4 hours. i have had pneumonia in the past. have i any reason to be worried, do i need to go to the hospital or should i be ok? Doctor: Hi, looking at your history since you already have lung condition(asthma) and past history of pneumonia, I suggest you to consult a pulmonologist."
},
{
"id": 106108,
"tgt": "High Eosinophil Count. How to proceed ?",
"src": "Patient: Hello, My mother (age 53) has very high Eosinophil count (900). This has resulted in breathing difficulty and/or cold/cough/ congestion in the past. She s usually prescribed some meds that reduce the count to about 650 after which she feels ok, but the root cause of the high Eosinophil count needs to be handled. As a side note, she is allergic to gluten (she sufferes severe stomach pain when she consumes anything that has gluten). Please advise on how to proceed, so there is a more permanent solution to this health issue. Thank you! Doctor: Same Eosinophil Count Problem I also had (~1150). I use to take EOFIL FORTE\u00ae - Diethylcarbamazine (DEC) for continuous one which helped me to maintain for Eosinophil Count for next two months. But it has never given me permanent cure. As doctor use to say there is no treatment of allergy in Allopath, only it can be controlled. Allopath (antibiotics) will not give permanent cure but we can improve our immune (resistance) to fight with any infection and can get permanent cure. But permanent cure is possible believe me. I had very serious problem and I am cured now with in two months. I am engineer and working in pharmaceutical company as Manager. My intension behind this write up is to give me self satisfaction by providing and helping people to get permanent cure. If you need more information so mail me for more details you can reach me on \u201cbharat80_shah@yahoo.com\u201d I will give you details of certified Nutritionist who cured me."
},
{
"id": 180752,
"tgt": "What does a blister on the tongue indicate?",
"src": "Patient: I have a blister in the middle of my tongue, low grade fever, about 3 days in no I can hardly swallow. Do not see any white patches just extremely red. No Tonsils, Neck is extremely sore on Nodes down into my chest where it feels heavy or tight when I try to drink something or take a deep breath. What could this be? Doctor: Hello,As per your query, there are blisters on your tongue while suffering from fever which seems mainly due to canker sore and due to viral infection. Usually, canker sore goes away on its own without any treatment in few days.I would suggest you to do warm saline rinses, start taking multivitamin supplement specially containing vitamin B12 and folic acid . If condition persists for longer period of time, then consult oral pathologist for proper evaluation.Hope I have answered your query. Let me know if I can assist you further.Regards,Dr. Harry Maheshwari"
},
{
"id": 81566,
"tgt": "Do toxins from oil spills cause TB?",
"src": "Patient: My wife passed two years ago. She had TB and was hospitalized for 6 weeks after which she had conjestive heart disease. She had an attack and drowned in our tube.My son and I found her. Tjhe TB was believed to be caused by the toxins from the oil spill. We lived on the beach and cleaned up the tar balls. She had a compermized imune system any way. But she was getting stronger each da, until the TB. In your opnion is it more likely the three are inter-related. Doctor: Thanks for your question on HCM. Tuberculosis is an infectious disease caused by infection with bacteria.Bacteria responsible for this is Mycobacterium Tuberculosis. It is spread by inhalation of infected droplets.So it is not caused by oil spills.And yes, you are right about low immunity. Not all infected, develop disease. Only those who have low immunity develop disease. So low immunity can be the cause for tuberculosis, but not the oil spills."
},
{
"id": 78735,
"tgt": "Is Pepcid 20 mg the right medicine for chest pain?",
"src": "Patient: Hi. I have been having a constant pain in my chest for the last 2 months. I finally went to the ER and they gave me prescriptions of pepsid 20mg. And Famotidine for nausea. I have no hearyburn nor nausea,pain in my chest is still there,and at least once a day the pain will strike hard then go away slowly for minutes. Doctor reffered me to see a gasterologists but i have no pain in my abdominal area. My digestive system is good. Doctor: Thanks for your question on Health Care Magic. I can understand your concern. Ideally ecg and 2d echo should be donedone first in your case. If both are normal than no need to worry for heart diseases. GERD (gastroesophageal reflux disease) can cause similar kind of chest pain. GERD is due to laxity of gastroesophageal sphincter. Because of this the acid of the stomach tends to come up in the esophagus and cause central chest pain and nausea. You are taking proton pump inhibitors (pepcid). But along with drugs you need to follow certain below mentioned lifestyle modifications for better symptomatic relief. Avoid stress and tension. Avoid hot and spicy food. Avoid junk food. Avoid large meals, instead take frequent small meals. Quit smoking and alcohol if you have these habits. Go for walk after meals. Keep 2-3 pillows under head in the bed to prevent reflux. Loose weight if you are obese. Don't worry, you will be alright with all these. Hope I have solved your query. Wish you good health. Thanks."
},
{
"id": 74405,
"tgt": "Are ventricular eptopics related with back and chest pain?",
"src": "Patient: I had an upper back injury with pain that radiated around the chest on the left and then later it moved to radiate around to the right. Although this pain has gone i still have deep aches in the area of injury, and then developed left ventricular eptopics are they related? Doctor: Thanks for your question on Healthcare Magic. I can understand your concern. Your pain at injury site is mostly related to nerve damage at injury site. This can not cause ventricular ectopics. So you should consult cardiologist and get done ecg, 2d echo, stress test and Holter monitoring (24 hours continuous recording of Ecg) to identify the cause of ventricular ectopics. These investigations will also guide about treatment of ectopics. So consult cardiologist and discuss all these. Hope I have solved your query. I will be happy to help you further. Wish you good health. Thanks."
},
{
"id": 34113,
"tgt": "Is mold causing headache and sinus problems?",
"src": "Patient: Hi, I have been having mold problems in my house, I thought I got rid of it last year but it came back. Since that time last year I have had bad allergies, headaches, and sinus problems and trouble breathing. Now I have had diarrhea for the last four days with pain in my abdomen, and am wondering if it is mold sickness. Can I wait to schedule a doctor's appointment or is it something that needs to be treated right now? I am 25 years old, 5'9, 170 lbs, I have had a history with allergy and stomach problems. Doctor: Hello dear,Thank you for your contact to health care magic.I read and understand your concern. I am Dr Arun Tank answering your concern.Yes, mold can cause all the problems that you have mentioned above.You should visit the doctor and get the antifungal medication as well as anti allergic medication. Fluconazole, cetrizine, prednisolone and salbutamol can be helpful in the treatment. Please take the above drug under your doctors guidance.Mold should not be present in the house otherwise it can cause multiple problems which can vary from the small health issue to the large health problem.Engineering method should be adopted this can be helpful in clearing the mold from the house. Wet area should ne sealed so that no mold arises out of it.Please maintian good hygiene as this can be helpful in treating and precenting this infections.I will be happy to answer your further concern on bit.ly/DrArun.Thank you,Dr Arun TankInfectious diseases specialist,HCM."
},
{
"id": 56046,
"tgt": "Is there any problem for increased alanine aminotransferase?",
"src": "Patient: what if my alanine aminotransferase is 46 and my albumin is 4.1 and my alkaline phosphatase is 87 and aspartate aminotransferase is 26 and bilirubin is .07, it looks as if the only high result is the Alanine Aminotransferase, should I be really worried? Doctor: Hi there,Thanks for posting in HCM.Yes only the ALT is raised. But even it is not significantly raised.I would consider it significant only if it is more than 2 times the upper limit of normal.In your case I would not worry at all when all the other numbers too are within normal limits.I hope that answers your query."
},
{
"id": 117079,
"tgt": "What causes elevated ALT level when suffering from haemochromatosis?",
"src": "Patient: I am diagnosed with haemochromatosis. My last blood test shows a alt figure of 87. My previous result was 35 2 months ago. My ferritin is slightly low at 26 iron is 11.3 and tibc of 19. My doctor said all results were satisfactory. Should i be concerned about my alt or wait for my next bloods tests in 2 months time. Doctor: Hi, dear. I have gone through your question. I can understand your concern. But liver involvement is very common in hemochromatosis. So slightly high alt is expected. No need to worry about that. Just be relaxed. Take your treatment regularly. Hope I have answered your question, if you have doubt then I will be happy to answer. Thanks for using health care magic. Wish you a very good health."
},
{
"id": 117430,
"tgt": "What is your opinion if Rubella IgG and Cytomegalovirus IgG levels are elevated?",
"src": "Patient: hi during my first pregnancy at ( weeks foetus heart beat stop and i have to remove it ; now i go through TORCH test in which Rubella IgG is 191.37 & Cytomegalovirus IgG is 10.51 which is high but IgM is normal. My doctor said that no treatment is needed but im confused. Doctor: Hello, Thnx to contact us. If I am your treating doctor I would like to advice you that Ig G is the antibody of immunity, that antibody developed as a result of the immunity towards the disease. So in your case you developed Ig G to both the mentioned viruses. in contarary to IgG IgM is the antibody developed when infection is ongoing, that is for killing the virus, that is negative in your case so do not worry for the lab results. If you have anything else to ask please contact me. Thanx. Dr. Arun Tank"
},
{
"id": 161023,
"tgt": "Suggest treatment for child suffering from fever, headache and throat pain",
"src": "Patient: Hi my 4 year old son had a temp of 102 this morning but said he felt fine. Then it went to 99 after giving him childrens tylenol and he started playing and said he was fine but he said his throut and his head hurts which he never complained about before ever. And now his temp is at 100 and he s covered with a blanket which he hates laying with cause now he said he s cold which he s never cold Doctor: Hi, By what you say your son should be suffering from viral illness. Whatever you are describing is very common in viral illness and I suggest you keep him well hydrated and alternatively give Paracetamol and Ibuprofen. Unless he's having respiratory distress or decreased urine output or lethargic while there is no fever or red rash all over the body you need not worry. Hope I have answered your query. Let me know if I can assist you further. Take care Regards, Dr. Sumanth Amperayani, Pediatrician, Pulmonology"
},
{
"id": 225446,
"tgt": "Due for periods next week. On birth control pills. Irregular intake of pills last week. Suggest",
"src": "Patient: Today, I went to take my birth control pills as normal. I was sitting at work, and it was a bit of a crazy day, and I remember at 12:51 thinking that I will take my birth control pills. However, I couldn't remember if I actually took it or not, and I put my birth control pills in my bag. Then at 1:00pm I went to take my birth control pills, and the Tuesday one was missing, but I didn't realize this until I put it in my mouth. I immediately took it out and put it back in the packet. I know this seems so stupid, but I can't remember if I took the Tuesday one today or if I accidentally took it yesterday. I didn't continue with the Wednesday one. What do I do?? I've been on the same birth control for three years with no problems, and this is my last week before I get my period next week. Any suggestions?? Doctor: HelloThanks for your query.If you have missed a pill, you can take two pills immediately the enxt day.If all of them are active pills ( not placebo ones ) - then it does not matter if you take the one meant for Tuesday or the one meant for Wednesday.Continue with the rest of the pills as normal.If you are still in doubt, you can use barrier protection for the rest of the month, and begin your new pack when your period starts.You could still be in the peri ovulatory phase, hence this precaution is recommended.All the bestTake care."
},
{
"id": 683,
"tgt": "Suggest whether i am able to conceive naturally or not",
"src": "Patient: hi doctor \"hello dri m 27 years old.last year in 2010 Oct i conceived but in Dec 2010 it got miscarriage.since last 4 month i have been trying then doctor suggested me to get HSG the report of HSG showing bilateral tubal patency but also showing pertublar adhesion and sight bleeding.and later on i got Mycobacterium Torch pannel in That IgM is showing 1.49 u/ml.pls tell me can it be completely cured by ATT and after being treated will i be able to conceive naturally.plz doctor answered me i m so worried. Doctor: yes any form of tuberculosis can be perfectly cured by ATT provided medicines are taken properly. once genital TB is cured chances of conceiving are more likely. All the best !"
},
{
"id": 60482,
"tgt": "Suffering from chronic liver disease ?",
"src": "Patient: my father is having chronic liver disease........age 58now the sgot gets elevated upto 80 sgpt 55 bilirubin 2.0.......is anything serious he s also having diabetes ...... Doctor: Welcome to Healthcare MagicSGOT and SGPT are not elevated significantly and can be taken as normal. Bilirubin is slightly raised, you can wait for it to come down. He needs to stay away from smoking and most importantly alcohol otherwise he can develop cirrhosis and vomit blood and lot of severe problems. Get his other levels tested as well like ALP, Serum albumin, Prothrombin time for better picture. Encourage plenty of fresh fruits and vegetables in his diet, avoid junk food. Let him drink enough water for the day. Exercise by brisk walking for 40 minutes a day to keep fit and prevent unnecessary illnesses."
},
{
"id": 192543,
"tgt": "What is the treatment for erectile dysfunction?",
"src": "Patient: Hi Doctor, Im a healthy 23 year old male, I workout out everyday, dont drink or do drugs, however I smoke cigarettes. My question to you is, Why do I not have such a strong libito like other guys my age and why is it when I ejaculate I cant erect for another 25-30 minutes? Doctor: Hello,You need to know that the normal ejaculation time is 5.5 minutes. More than that is good for you. Well, you can perform good by strengthening your pelvic floor muscles and eating healthy.Hope I have answered your question. Let me know if I can assist you further. Regards, Dr. Sameen Bin Naeem, General & Family Physician"
},
{
"id": 102523,
"tgt": "Allergic to acyclovir causing blisters in skin, throat fiery with ears broken into hairline. Antihistamine not working",
"src": "Patient: I have an allergy to acyclovir there is blistering and skin feels on fire dwellings on throat area ear broken out up into hair line Got antihistamine but doesn t seem to be working can t touch skin really painful I am sensitive to all medicines is there anything I can do to relief pain and burning Doctor: Hello,Welcome to HCM,If antihistaminics are not controlling the rash and blister caused by acyclovir. Some individuals may be allergic to acyclovir.I would suggest you1.Stop taking the drug2.You can apply cold packs on the lesions3.Moisturizing Creams like calamine lotion4.Oral steroids for 4-5 days.If your symptoms are not improving you can consult your doctor for further suggestions.Hope you find the answer in it.Thank you."
},
{
"id": 122788,
"tgt": "Will I be able to run after recovering from a broken tibia and a fibula?",
"src": "Patient: Welcome to Ask a Doctor . Please type your question here...I broke my tibia and fibula playing soccer 12 months ago. I got a rod and screws put into my leg. It took my bones 10 months to finally heal, I had to use a bone stimulator. Now that they re healed, my doctors said that I could go back to playing soccer but I m still experiencing a lot of pain in my leg while running, to the point where I can t run anymore. Doctor: Hi, You have broken both the major bones of the leg. Almost complete recovery is possible. But keep in mind that it will take a long duration. Long term physiotherapy is also required. Consult a psychiatrist and he will direct you accordingly. Hope I have answered your query. Let me know if I can assist you further. Regards, Dr. Shinas Hussain, General & Family Physician"
},
{
"id": 61528,
"tgt": "Suggest remedies for painful, hard lump under the skin on the vagina",
"src": "Patient: I have a hard golf ball size bump under skin on the bottom of my vagina. It s so swallowen that it s covering the hole to my vagina. It hurts so bad. I ve been sitting in hot baths. Putting heating pads on it. It s to the point that the skin is so raw I get this sharp burning sensation every time I move. It s killing me to walk. Please help me Doctor: Hi and welcome to Healthcaremagic. Thank you for your query. I understand your concerns and I will try to help you as much as I can.This is very likely to be an inflamed Bartholini cyst and in this case heating pads wont be efficient. YOu should see a gynecologist since this abscess should be surgically incised and drained. Otherwise it may just grow and cause even more intensive pain.Also antibiotic therapy may be required so you should not delay doctors visit anymore. Bartholini cyst is a common condition and it is best to solve it once for ever.I hope I have answered you query. If you have any further questions you can contact us in every time.Kindly regards. Wish you a good health."
},
{
"id": 169348,
"tgt": "What causes brown crusts on scalp?",
"src": "Patient: I recently noticed brown crusts on my 3 year olds scalp...but it worried me greatly. I ve examined his scalp for lice but didn t really see any and he goes to daycare which is why I m nervous about lice. (Though I ve never seen an actual lice infestation in my life so I don t really know what to look for) I did gave him a bath and rubbed his scalp and brushed it...and all seemed to came off.. anything I should do at this point? what could be the cause of those brown crusts? Doctor: brown crusts of scalp can be due to sborrhoec with fungal infection ketoconazole shampoo along with antihistamines could help."
},
{
"id": 60825,
"tgt": "What does a lump in the brain indicate?",
"src": "Patient: I have found A LUMP IN MY BRAIN WITH ABNORMAL SYMPTOMS. i AM 56 YEARS OLD ANS AM EXPERIENCING UNUSUAL SYMPTOMS i HAVE NEVER HAD BEFORE. mY QUESTIONS ARE: WHAT KIND OF SYMPTOMS COME WITH SUCH A DIAGNOSIS? cAN YOU GIVE ME SOME GENERAL IDEAS? THANK YOU RICK THURSTON YYYY iDAHO Doctor: Hello and Welcome to \u2018Ask A Doctor\u2019 service. I have reviewed your query and here is my advice. * There can be various types of lump in the brain as benign, malignant tumor or else. * I would like to further assist with all details of symptoms and reports of MRI or else as an attachment in follow up conversation. Hope I have answered your query. Let me know if I can assist you further."
},
{
"id": 61516,
"tgt": "Can a hard lump on the forehead cause headaches and dizziness?",
"src": "Patient: i have a hard lump on my forehead at the left hand side just on the bone above my eye it can be quite tender to touch the doctor has had a quick look and said possibly a cyst growing on the bone? I have since been having headaches and slightly dizzy could this be related? Doctor: Hi thanks for your question.The lump which you are having on your forehead,diagnosed by doctor to be a cyst growing on bone can cause headache and dizziness.Increased pressure inside the cyst or cyst compressing underlying bone can give rise to symptoms you are havingHope this answers your question."
},
{
"id": 51994,
"tgt": "What are the treatments for Renal Parenchymal Disease?",
"src": "Patient: Respected Doctor, What is Renal Parenchymal Disease? What is the treatment for it? Doctor: Dear Joy,thanks for query.Renal parenchymal diseases is structural damage to kidney.there are reasons for this.The treatment is the treatment and good control of that particular disease it self.Some diseases like diabetes for example leads to this.So what you need to look for the reason and take treatment accordingly."
},
{
"id": 86282,
"tgt": "Suggest treatment for right side abdominal pain",
"src": "Patient: I found out I had a tubal pregnancy Sept 6 2015 and went in for surgery that night. About 2 weeks after, my incision in my belly button was oozing yellow/brown liquid. I went to the doctor and was put on antibiotics. It cleared up shortly after.Now, I am having stomach pain on the right side. Kinda under the incision to the right. Should I return again? No fever or discharge.Just chills, pain, and tenderness upon palpation. Doctor: Hello! Thanks for putting your query in HCM. I am a Gastroenterologist. There may be infection at the incision site. So I will suggest you to visit your doctor who may advise you for complete haemogram, and ultrasound abdomen to see for any infection and collection of pus. I hope I have answered your query and this will help you . Wish you a good health"
},
{
"id": 7619,
"tgt": "Bad acne since teenage, taking tab dianette, no results, side effects such as excess hair growth, weight gain, prescribed tetrylsal. Which medicine should I take?",
"src": "Patient: Hi i was wondering whether you could help me. I have had bad acne since my teenage years i was put on Dianette at the age of 14 up to around 26. i then tried various pills cilest, Ovranette, Loestrin 30, Femodene , Marvelon all of which have difference side effects they did clear up my skin. I then decide to have a break from the pill for around 6months but then my acne came back. I then went back onto Dianette as this was the best pill for me with minimal side effects. I have now been taking this for 4-5months and my acne is back on my forehead, chest & back. I went to the Doctors today & he now has told me to come off Dianette & now try Tetrlysal for 48 days. Would you say this is the best option or to continue with Dianette as this has been success in the past. i have also noticed excess hair grow aswell & a little weight gain. I go to the gym 4-5 days a week & i eat healthy. Doctor: Hello thanks for posting your query Acne is a worldwide common problem of younger age group. The treatment depends on many factors like staging of acne, frequent recurrences, family history, tolerance of medications, side effects of drugs etc. Your doctor must be right in deciding that you may have to stop dianne at this stage. antibiotics like tetracycline, doxycycline, minocycline, azithromycin, clindamycin can all be given with lesser side effects. Of course antibiotics will reduce only one causative factor for acne ie. bacterial colonization You felt better because dianne is an anti androgen and can stop oil secretion from sebaceous glands. so, the best thing to do is listen to your doctor's advice. It is difficult for me to tell you which is best as it depends on number of factors. I hope you understand my point Have a good day Prasad PVS"
},
{
"id": 105449,
"tgt": "Frequent cold and runny nose. Allergic to dust and smoke. Feeling drowsy and sleepy. Taking Metosartan and Bestor",
"src": "Patient: hi, doc, i catch cold easily and get running nose and nose blocks regularly, i am allergic to dust smoke etc. Currently i am having nose block and drowsiness in my head, feeling sleepy through out the day, its so irritating i cant even work , i need some medicine to get rid of this, i am having METOSARTAN 50 for BP and BESTOR-5for cholestrol and REVITAL vitamin tablets daily, Doc kindly suggest me some effective medicine to get rid of sleepiness{ drowsiness} Regards, Doctor: Hello\u00a0\u00a0\u00a0\u00a0\u00a0 Welcome and thanks for your question I would be pleased to help you with your Question I would suggest you to meet an ENT doctor . They will ask for certain investigations. An Antibiotic or a Short steroid course will help reduce your allergy or an infection around. Since it is an online site , I am sorry it is not legal for me to prescribe you any medication . Thanks and Take care"
},
{
"id": 179991,
"tgt": "Can antibiotic be given for 3 year old for fever and cough?",
"src": "Patient: Hi my 3 year old son has high fever like 102-103 since past 2 days. I am giving him paracetamol to keep the fever down. He is coughing and also vomiting. Now doc has written cefixime trihydrate for him. A little apprehensive for antibiotic.. please suggest Doctor: HelloThanks for writing in HCMI had gone through your query and understood your concerns.Yes antibiotic cefixime is safe i also suggest it in my clinic for cough and feverHope this guides you. If you have additional questions or follow up queries then please do not hesitate in writing to us. I will be happy to answer your queries. If you do not have any clarifications, you can please close the discussion. Thank you.Wishing you good health"
},
{
"id": 98694,
"tgt": "Suggest remedy for non-productive cough on exertion",
"src": "Patient: I ve had a dry cough for about a week or two now. It only appears when I go up stairs or do an activity that requires heavy breathing or laying down. It s more like a tickle in my throat, but it s really annoying and frustrating when I m trying to sleep and my cough doesn t make me feel better Doctor: Breathlessness on exertion could be a cardiac problem. Do an ecg and treadmill test. Check your heart's compliance or heavy work. If all normal,then we can come to airway. Work increase the oxygen need and if you are previously asthmatic, your lungs may struggle and may cause airway constriction. Check whether you have wheeze or not."
},
{
"id": 6854,
"tgt": "When can I take a pregnancy test to confirm if I am pregnant ?",
"src": "Patient: I'm worried I may be pregnant but think it's too early to take an hpt. Around the 8th of last month I had unprotected sex. I'm never irregular and usually start my period on the 12th but it ended up being four days late this time, which was strange to me. I thought nothing more of it until last Tuesday, Sept. 28th, I began experiencing nausea set off by a heightened sense of smell. I only vomited once but since that day the nausea and dizziness has not subsided AT ALL, along with headaches and an abnormal amount of leukorrhea (odorless) coming out. I know some women can get their period or bleed while they are pregnant, but when would be a good time for me to take a test and get the most accurate result? Doctor: Welcome to HCM. You can get accurate result by urine pregnancy test but it is idealy sesitive after about 8th day of missing cycle."
},
{
"id": 44007,
"tgt": "Done semen analysis. Can I have a baby and can morphology be improved?",
"src": "Patient: Hi my wife and I are trying for a baby for six months now. I recently did a semen analysis and the results are below Volume-3.0ml, Viscosity-Moderate, Non-Specific Agglutination 10%, Motility 40%, Progressive-20%, immotile-60%, Sperm Count-60 x 10^6/ml, Abnormal forms-80%. After these results i was asked to do a second SA and the results are below Volume-2.0ml, Viscosity-Low,Liquefaction-Complete, Non-Specific Agglutination 10%, Motility 80%, Progressive-78%, immotile-20%, Sperm Count-8 x 10^6/ml, Abnormal forms-90%.70% morphologically abnormal forms are associated with decreased fertility . So now my GP has asked me to consult a endocrinologist . I want to know if I can have have my own baby and can the morphology be improved? Please let me know your advise is highly appreciated. Doctor: Hi, According to WHO 2010 criteria, any (normal) morphology above 4% is normal. If your semen analysis was done according to WHO 2010 guidelines, its normal analysis & no need to worry. Just ask your laboratory, what guidelines they have followed in counting morphology."
},
{
"id": 133520,
"tgt": "Suggest treatment for osteoporosis",
"src": "Patient: Hi I am new to this! my doc has put me on Zomta for osteoporosis the anti cancer daily med. Gave me.I have had I guess they call it a round and he is giving me three more one a month. The one every six month for 3years. I was receiving chropratic adjustment for the last 6 months,I have decided ti stop them because the pain in my back was incredible.I guess I want to know if I should ask my doc for Meds other then Aspren buyer back and body.(I have been taking it for about six months. How painful is just the Zometa for most people do they need stronger M reds It was bad for 4or 5 days. I wrote you a book! Doctor: hi,thank-you for providing the brief history of you.A thorough neuromuscular assessment is advised.As you have a chronic back pain, for which you were undergoing chiropractor, well chiropractor helps to unlock the facet joint and the relief occurs. Also, medicines are symptomatic and works well with physical therapy. Exercise is a must for this back ache and research supports the best evidence of its success as the functional ability is maintained well. Also, with exercises you can improve the drug absorption, improved metabolism, improved immune system, improved bone strength and muscle strength so recurrent pain is avoided.For osteoporosis patients coming to my clinic we balance the combination of medication and physical therapy which provided immense changes.RegardsJay Indravadan Patel"
},
{
"id": 90793,
"tgt": "Suggest remedy for burning abdomen",
"src": "Patient: My abdomen feels like its on fire. Its an intense burning. I have been eating ice chips for 2 days. The only solid food I have eaten is 3 chicken nuggets and some soda crackers. I don't have a real appetite and I do not have the senation of needing to puke or have a bm. Doctor: HiAcid peptic disease is the most common cause of gastric burn.You need dietary modifications and drug treatment.Dietary modifications include avoidance of spices, alcohol, smoked/fried food , coffee, chocolate etc.If not improved with dietary modifications, go for drugs.Drugs include antacid, proton pump inhibitors, H2 blockers.Consult your doctor if you want to start drug treatment.Thank you"
},
{
"id": 224175,
"tgt": "Implanon implanted in left arm, pain, swelling, small lump. Is it due to Implanon?",
"src": "Patient: I have Implanon birth control in my left arm . Two days ago i started having a pain in my left arm and when I extended my arm i had pain from my shoulder to my elbow . The next day the pain had moved to my armpit where it now looks slightly swollen and I can feel a small lump and now when I try to left my arm at the shoulder i have a strong pain. could it be caused by my implanon? Doctor: HiDr. Purushottam welcomes you to HCM virtual clinic!Thanks for consulting at my virtual clinic. I have carefully gone through your case, and I think I have understood your concern. I will try to address your medical concerns and would suggest you the best of the available treatment options.Although it is most unlikely because of IMPLANON, you need to get it confirmed by clinical check up.If there is pain, swelling, redness at the site of implanon insertion, and also painful , nodular swelling in the same arm pit it needs treatment.I will suggest a course of antibiotic AMOXY -CLAV 625mg twice a day for 5 days, and Tab CALPOL 650 mg as needed for pain.I hope my answer helps you.Thanks.Wish you great health.Dr Purushottam"
},
{
"id": 72905,
"tgt": "Suggest remedy for severe acid reflux with difficulty breathing",
"src": "Patient: My heart rate is normally 95 - 100 bpm. Lately it has been running around 65 to 85 bpm. I have had severe acid reflux. In fact all I have to do is burp or lay down and stomach contents, be it acid or food, come up into my mouth. Tonight around 4 am, I woke up and its hard to breath. I keep getting acid coming up into my mouth and my heart rate is around 65 bpm. Even though I'm scared it is not really increasing. I dont know if its related but my knee is shooting pain. Doctor: Hello dear , hiWelcome to Healthcaremagic.comI have evaluated your query thoroughly .* Usually acid reflux is not directly related to heart rate alterations unless associated with stress , anxiety underlying .* Guidelines for better recovery- Onetime physician consultation with EKG check up .- Upper GI endoscopy examination to evaluate for the underlying ulcer , GERD , Hiatus hernia or other conditions .- Drink plenty of liquids .- Prefer soft , light diet at intervals .- Avoid oily , spicy , non veg , hot beverages , junk foods , dairy fat .- Avoid over eating , prolong fasting .- Do regular walking in fresh air , exercises , YOGA- Keep stomach empty by 25 % level of satiety .- Avoid going to bed for 2 hours after major meals .- Avoid stress, anxiety .- Manage sound , regular sleep of 8 hours .- No smoking or alcohol if using .- Keep the head end of the bed elevated for 30 degrees when doing rest .- Regular intake of proton pump inhibitor as omeprazole or pantoprazole on empty stomach .Hope this will help you for sure .Wishing you fine health ahead .Regards dear take care ."
},
{
"id": 105017,
"tgt": "Sore throat, pus filled pimple above tonsils, headache due to working in AC. Can I take Azithral?",
"src": "Patient: i have sore throat ....one small pimple of puse inside throat above tonsils.....i have allergy of tonsils since childhood...i am allergic to cold , dust, and some tartaric fruits and curd.Also with throat infection right now my head is paining and have cold and cough due to AC/ and cold climate as on work i cant stop AC for other so need to work in same condition.Should i go for azithral500mg. Doctor: Hello, Azithromycin is a good choice of drug and 500mg od (or 250mg twice daily) for 5 days is sufficient. However, you must remember that a one-off course of antibiotics may not do any harm but if you need antibiotics again, do consult your doctor to see if this is really required. Allergies can feel like a constant flu infection, and testing is recommended. You may already have had tests but skin prick testing or specific IgE blood test for house dust mite, animal dander, molds (alternaria, aspergillus, cladosporium etc) can be difficult to avoid and treat unless you know that these are also problems. Rhinosinusitis or just rhinitis resolves on long acting antihistamines and nasal steroid spray like nasonex taken for 6-8weeks. Thanks."
},
{
"id": 90419,
"tgt": "Feeling tightness on the right abdomen",
"src": "Patient: My husband is having a dull tightness in his right abdomen. He describes it as a feeling of being full after eating small meals and a feeling that his stomach is balled up in one area. He has been very gassy and sometimes a pain or an uncomfortable feeling is there and stays there for a while until he lays down. He states that when he lifts his arms up there is a feeling like it is being pulled because of how tight it is. He has had blood work, stool samples, x-rays, liver enzyme test, pretty much everything except for a CT or MRI and everything has came back normal but he is still having the same feeling. We do not have a clue as to what else could be causing this. Doctor: Hi.Thanks for your query.Just to recapitulate: dull tightness in right abdomen - full after eating small meals - balled up in one area- gassy- being pulled because of how tight it is.-everything except for a CT or MRI- I would advise to start with a simple investigation like: Plain X-ray of the whole abdomen in the standing position- this may show air-fluid shadow if intestinal obstruction.Yes, CT will definitely help the diagnosis.In the mean-time he should stop all the foods and beverages that you both have observed to have increased the problem. The treatment will depend upon the diagnosis and can be medical or surgical.There is a possibility of an inflammatory process going on in abdomen with added intestinal obstruction."
},
{
"id": 144869,
"tgt": "How can a neck nerve issue with finger numbness be treated?",
"src": "Patient: Hi, I have nerve issue on my neck and I have to keep it all the time warm,and my two finger of my left hand is nom my doctor say when your hole hand gots nom you do the surgery but the pain is very knowing I take norco 10 mg tree a day but still is pain there and because not to get educated to norco I would not increase the norco which it helps,what other choose do I have to deal with the pain ,Thank you Doctor: Thank you for asking Healthcare majic. My name is Dr Ehsan Ullah & I have gone through your query.you should do proper digital x-ray cervical spine AP & Lateral view so see the spine alignment,bone..Also do MRI cervical spine to see soft tissues and prolapsed inter-vertebral disc which might cause pain radiating to fingers and numbness.then get visit to neurosurgeon for further intervention and complete solution. Hope this may help you. Let me know if anything not clear. Thanks."
},
{
"id": 18265,
"tgt": "How can high blood pressure be treated?",
"src": "Patient: I am 60 year old female that flew in from London 6 weeks ago to look after my mother who is in hospital since 29 April. I\u2019m experiencing high blood pressure for several weeks and 2 days ago I had a wavy vision in my peripheral view in my left eye, then I woke yesterday with a pain in my left temple behind my eye, I took ibuprofen and used ice pack, which helped alleviate it, but it has persisted during the night again and it keeps waking me up,. My ears are ringing and my BO is 150/110 with 50 beats at 5:15am. I usually have low blood pressure and suffer from vertigo BPV..... I am trying to rest but can\u2019t sleep and frightened if I can have a stroke. I have no medical coverage here if I go to A& E , but afraid to fly home in this condition.... what should. Do? Doctor: Hello and Welcome to \u2018Ask A Doctor\u2019 service. I have reviewed your query and here is my advice. I passed carefully through your question and would explain that your symptoms are not typical of a stroke. But, it is important to exclude possible temporal arteritis which can mimic this clinical situation. For this reason, I recommend consulting with your GP for a physical exam and some tests: - a resting ECG - complete blood count, PCR, ESR for inflammation - blood electrolytes for any possible imbalances - thyroid hormone levels for thyroid gland dysfunction - a brain MRI study. You should discuss with your doctor on the above tests. Hope I have answered your query. Let me know if I can assist you further."
},
{
"id": 44304,
"tgt": "What is the success rate of IVF when I have adhesion and reduced womb size ?",
"src": "Patient: i am 38 yrs old, having conception problem i was told i have adhesion which i went for a surgery of hysteroscopy under anastesia last month i was equally told my womb shape has reduced in shape. i was placed on medication using progynova 2mg for 3week of my cycle and T.mapeed 10mg for 1 week, can this increase my chance of conception and what is the success rate if i decide to go for ivf Doctor: Hello. Thanks for writing to us. Since you had adhesions around the uterus, the chances of conception are low as fibrosis affects the smooth transfer of eggs from ovaries to the uterus. IVF is a good option for you. The tablets you have been prescribed are to regularize the periods so that ovulation can be induced. I hope this information has been both informative and helpful for you. Regards, Dr. Rakhi Tayal drrakhitayal@gmail.com"
},
{
"id": 223777,
"tgt": "What causes foul smelling yellowish urine post Mirena IUD insertion?",
"src": "Patient: Hi i am 19 years old. I got mirena about 2 months ago. And about 2 weeks i started to notice that my urine color started to look dark yellow and has a really bad smell. I do not have pain when i urinate. I don t do any kind of drugs what so ever. What could it be? Doctor: Hello,I have gone through your query and understood the concern. From the description you have given, it appears as though you are passing concentrated urine and hence the smell and color. To clarify, you can get a simple urinalysis. This presentation cannot be related to the insertion of the Mirena. The intake of several foods and also mineral and vitamin supplements can give specific color and smell to the urine. You have to increase the intake of water per day to at least 2 liters. If you notice improvement, the cause is obvious. If not, you should check with a specialist for further management. Hope this helps."
},
{
"id": 41199,
"tgt": "Is laproscopy effective in treatment of unexplained infertility?",
"src": "Patient: Hi doc ! I am suffering from unexplained infertility . After trying for 5 yrs i got 2 spontaneous pregnancies with miscarriage 2 yrs back . Normal hormone profile & normal HSG . My dr says to do laproscopy , but i feel scared . Can you guide me shall i do the procedure or ivf will be a better option . I am 34 Doctor: Hello, in my opinion there is no need of laparoscopy and rather you can try IUI for 3 cycles at least. In case still no success then opt for IVF.In case you have any questions in future you can contact me directly on http://bit.ly/drmanishajain"
},
{
"id": 170487,
"tgt": "What causes itching and rashes on palm and fingers of a 4 year old?",
"src": "Patient: Our four year old daughter was itching her palms and fingers through the night. She seemed really bothered by how much they itched. This morning I looked at them closer and there are red spots, like a rash on her palms. I just don t know what the rash is, what it could be from, if it s an allergic reaction to something and how to treat it. She says nothing else is itching. Doctor: Hi, rash on palms with history of 1 day could be due to insect bite. In insect bite, there is itching and redness at the region of insect bite. In my opinion, you should give hydroxizine syrup 5 ml PO three times a day for 3 days. I advice you to revert back with some good pictures of the skin lesions to confirm the diagnosis. Take care."
},
{
"id": 6715,
"tgt": "Are there any chances of pregnancy based on the report ?",
"src": "Patient: The report says anteverted non gravid uterus was enlarged at 16 cm (l)* 8 cm (ap)* 12 cmW. a 7cm(l) *6cn(ap) myometrial leiomyomas noted at the right anterior uterus. I am 41 years and do not have children,. is a hyterectomy a likely outcome of sx? I am about 190 lbs. I have no other health concerns except occasional anemia ,and fibroglandular stroma Doctor: welcome to healthcaremagic please talk to your gynecologist as you are having no issue and want itconservative surgery to remove the fiboid only if possible must have been considered by your gynecologist once the utrine problem is solved conyinuous treatment for infertilit to be continued till result is obtained your partner should also be examined and tests done to take yreatment if required"
},
{
"id": 178639,
"tgt": "Suggest treatment for loose motion in a child",
"src": "Patient: Hello..my kid is 3 months old...from the birth he is doing loose stool...and from last one month doing alternate days and same loose stool like water...we have consulted with dr. She asked to give darolac sachet for 3 days...but no results...still same loose stool....kindly suggest the remedy... Doctor: Dear sir/madam,Thank you for posting your query at healthcaremagic.comFor many children it is very common, which you need not worry. It could be because of lactose intolerance(ie unable to digest certain carbohydrates in milk). In that case will have to change to other milk(lactose free).I advice you to get the stool examine repeatedly, 1. for bacteria 2. for lactose intolerance. which will give us the better view on current situation and help us in setting further management protocol.Hope my answer give you better understanding.Your thank you note will be very much appreciated!With best wishes,Dr. Vishwanath Patil"
},
{
"id": 20221,
"tgt": "How to wean off Flecainide Acetate?",
"src": "Patient: I am 64 years of age. Height = 5ft 10in Weight = 125lbs. I was first diagnosed with lone atrial fibrillation in 1996 and prescribed a dosage of Flecainide Acetate at 50 mg twice daily. My episodes of Atrial Fibrillation were extremely rare and usually short in duration. The Flecainide did not prevent them entirely. Recently I changed my GP who examined both me and my complete medical history. He was extremely surprised that I had been continuously prescribed the medication without review and advised me to stop taking the medication immediately as he did not feel my condition justified continued use. This was one moth ago and I have suffered no symptoms. I am also very health conscious and eat a very healthy vegetarian diet. I am non smoking and do not drink alchohol (apart from a glass of wine with some meals). I run between 30 - 40 miles a week (have done since my teenage years) and also brisk walk with my wife several times a week. What do you advise about stopping taking the Flecainide Acetate? Doctor: Lone atrial fibrillation has two concerns, symptoms and the risk of stroke. You report no risk factors for stroke but this is an important part of the conversation with your doctor on a yearly basis. Limiting symptoms often requires nothing. Atrial fibrillation will not go away. It will reoccur, usually in brief episodes. Episodes may occur once or twice a year at first. Over time, episodes will get longer and come more frequently. As they do, medicine is used to abort or prevent them. For someone like you in whom they remain rare, keeping medicine with you in case symptoms occur is the current preference. It is called pill in the pocket. Flecainide is actually the preferred drug. The dose is different for pill in the pocket as opposed to daily dosing. The original recommendation was not a bad one. Flecainide has no cumulative harm and is effective. You may want to revisit the issue with the prescribing doctor and ask about pill in the pocket."
},
{
"id": 142612,
"tgt": "What is the treatment for jerking movements in head?",
"src": "Patient: my father is 88 years old & has started having a jerking in his neck. He says it starts in his shoulder & runs up his neck & makes his head jerk, he takes several medications & one is Risperdone 1 mg daily. This started 3 days ago is he said it is not painful. Doctor: Hi, Welcome to HealthCareMagic.com I am Dr.J.Mariano Anto Bruno Mascarenhas. I have gone through your query with diligence and would like you to know that I am here to help you.This condition requires clinical Examination. Please consult a neurosurgeon or Neurologist at the earliestHope you found the answer helpful.If you need any clarification / have doubts / have additional questions / have follow up questions, then please do not hesitate in asking again. I will be happy to answer your questions. In the future, for continuity of care, I encourage you to contact me directly in HealthCareMagic at http://bit.ly/askdrbruno Best Wishes for Speedy Recovery Let me know if I can assist you further.Take care."
},
{
"id": 212470,
"tgt": "Hypothyroid taking 50mg thyroxine, hairloss, pricking sensation in eye. Can it be Candida?",
"src": "Patient: hi there, i m 43, 5ft 7, 9st 7... I am hypothyroid taking 50mg thyroxine . I have had hairloss now for 21 months, also i have developed an annoying pricking sensation in the eye ... any ideas on the hair and eye? I m wondering can it be Candida? I am on iron tablets and Vit D .... still struggle with tiredness and depressive tendencies .... ? Doctor: Hello, welcome to Healthcare Magic. It is difficult to comment about cause of hair loss and eye irritation in your case. But one thing is clear from your description that your weight is very less. Malnutrition can lead to hair loss, dry eyes and depressive symptoms. In your case, it is better to get investigated for low weight. Same time start healthy diet and weight monitoring. Wish you all the best. Regards, Dr Ashish Kumar Mittal www.99doctor.com"
},
{
"id": 97775,
"tgt": "Any replacement medicine for cytra k crystals powder to prevent kidney stones?",
"src": "Patient: My husband has been taking Cytra K Crystals Powder Packets to prevent kidney stones for a number of years. Right now we are having difficulty purchasing them at the drug store (the store says the manufacturer is not producing them at this time). What can we use to replace these crystals (they have helped prevent stones for the past 5 years). Doctor: Hi,Welcome to Health care magic forum. The product cytra K need not be used continuously for longer period, it should be used for some time when you have the symptoms of the stone, or urinary tract infection. As an alternative you can have disodium hydrogen citrate, or some aurvedic preparations like neeri, etc. but never for prolonged use and should be used with the doctors advise. So try to stop it completely and you can use if necesity arises when your doctor prescribes. Wishing for a quick and complete recovery."
},
{
"id": 133351,
"tgt": "How to treat jaw pain?",
"src": "Patient: I FEEL SWOLLEN & THERE IS PAIN JUST UNDER MY RIGHT JAW & DOWN ABOVE MY COLLAR BONE IS PAINFUL & I FEEL A LUMP. I WAS RESEARCHING THE INTERNET TO SEE WHAT IT COULD BE TO KNOW WHAT TYPE OF DOCTOR I SHOULD SEE. FROM WHAT I SAW IT LOOKS LIKE IT COULD BE THE RIGHT LYMPHATIC VEIN DUCT. DO I START WITH MY GENERAL DOCTOR? Doctor: Hello there,Swelling that you have described appear to be lymph node of neck.You need a consultation of ENT surgeon and general surgeon both for the evaluation of that swelling.i hope the best for you.thank you."
},
{
"id": 211556,
"tgt": "Unable to trust boyfriend, consuming life and mind, physically ill,lost weight, stomach ulcers. Help?",
"src": "Patient: lostmy boyfriend said he quit maturbating...i dont believe it and its consuming my life...my mind...schooling....i have trust issues with men...i dont understand y its consuming my life and mind...ive been physically ill over this for a while...lost weight stomache ulcers...etc...i think he does it when he gets mad at me....help Doctor: Hi relax. You can't control him and should not even try to think whether he masturbates or not. His health won't get affected with masturbation. You should think more of your health, eat healthy, spend time with your family members. Engage in exercises and physical work. This will help you to relax and also your well being. Think for your future after 10 years and focus on your education and health. Hope this helps you. Wish you good health.regards"
},
{
"id": 220664,
"tgt": "What causes soft stool and abdominal pain after taking medications to stop contractions?",
"src": "Patient: I am 28 weeks and 3 days and just went into preterm labor almost a week ago. (They gave me medicine to stop contractions) For about the past week and a half my stool has been really extremely soft (almost diarrhea but not quite) My stomach has been hurting, and every time The color has been a lot lighter than normal. I also feel like I ve had to go a lot more than normal. Could this be a sign of anything? Doctor: Hi, Dr Purushottam Neurgaonkar here. I Think I have understood your concern. As such there is no relation between your tummy upset and medicine for preterm labour. Actually preterm labour starts because of intestinal upset. I will suggest you Tab Pan D Sr once a day for 5 days. Tab Flagyl 400 mg three times a day for 5 days and Tab Cyclopam as needed for pain in abdomen. Also use of Cap Vibact DS once a day for ten days will be of help. Include curds, yoghurt and buttermilk in dirt. Eat fresh food, fruits with pulp and drink plenty of water. I hope this answer helps you. Thanks Dr Purushottam Neurgaonkar"
},
{
"id": 161739,
"tgt": "Can Dulcoflax be given frequently for constipation in an infant?",
"src": "Patient: Hi, My lo is 55days old. And he doesn t pass his stool for 5days n then my grandma uses external remedy of inserting in anus for passing the stool. Then again from next day he will not pass his stool.. Its been continuing for past 2 weeks.. My lo is exclusively only on mother s feed... Let me know whether it is safe to use Dulcoflax or any other external remedy for babies so frequently? Doctor: Hi and welcome to Healthcaremagic. Thank you for your query. I understand your concerns and I will try to help you as much as I can.treating constipation in such small babies ahould be managed by simple changes to your child\u2019s diet and potty training can help treat constipation. You may notice a difference within a few days. Sometimes it takes a few weeks before their symptoms improve. Make changes to your child\u2019s diet Give your baby extra water between their normal feeds if they haven\u2019t started to eat solid food yet. If you\u2019re using formula milk, don\u2019t add more water to the mixture. Try gently moving your baby\u2019s legs in a bicycling motion or carefully massaging their tummy to help stimulate their bowels. Give older children plenty of fluids and encourage them to eat fruit. Chop or puree it if it\u2019s easier for them to eat. The best fruits for constipation include apples, grapes, pears and strawberries. Don\u2019t force your child to eat as this can make mealtimes stressfulI hope I have answered you query. If you have any further questions you can contact us in every time.Kindly regards. Wish you a good health."
},
{
"id": 183056,
"tgt": "Suggest treatment for swollen gum on broken tooth",
"src": "Patient: I have a broken tooth. Gum above tooth is swollen feels like a knot in it, Also swelling and feels like a knot on my cheek right besides nose . Took a round of clindycin swelling went down but then it stated swelling again same way. Called dentist. He put me on augmentin . Been on it for a day and half swelling not improved but not any worse.Do you think I need a strong antibodic . Have appt Tuesday to remove but still have infection in it Doctor: Hello,Thanks for consulting HCMRead your query as you have broken tooth now you have swelling in gums this swelling in gums is due to decayed tooth with periapical abscess so dnt worry you take medicine antibiotic you are taking in addition you can take medicine analgesis antiinflmmatory like zerodol sp or diclomol sp . Do warm saline gargle two - three times a day and consult dentist for removal of tooth .Hope this will help you."
},
{
"id": 156548,
"tgt": "What are the symptoms of cancer?",
"src": "Patient: Hi, I keep having calf pains and I noticed a changing mole there. I had a biospey and the lab said the results are still pending. I was told if it was longer then 72 hours that means something is growing. Now should I just assume it's a type of cancer? Doctor: Histopathological reports of biopsy specimen do take time as the specimen has to be fixed in paraffin before examination which normally takes 3 days. Delays can be due to many reasons, so relax and pray for a good report."
},
{
"id": 97262,
"tgt": "What can cause a feeling of low pressure in the ear after a hit on the head?",
"src": "Patient: A friend of mine asked me to look this up for her. She hit her head on Sunday. On Monday, she began to feel pressure in her ears. She describes it as being similar to what one feels on an airplane. There is no hearing loss; she is not in pain. But she wonders whether she should see a physician. Thank you! Doctor: YES SHOULD AS THE AREA IN THE HEAD REPRESENTING EARS MAY HAVE HEART INFLAMED OR BRUISED TO CAUSE PRESSURE IN THE EARNEED TO SEE DOCTOR FOR CT SCAN HEAD ALONG WITH INTERNAL EARS AND LOBES OF BRAIN REPRESENTING EARS AND TREAT ACCORDING TO RESULTS"
},
{
"id": 176841,
"tgt": "Are Gentamicin eye drops safe for children?",
"src": "Patient: 7 year old with pseudomonas aeruginosa from penile swab. Told their is no cream for it. Eye drops to be applied of ciprofloxacin. Is this safe as it is not recommended for systemic use in children. Is gentamicin eye drops safer or any other alternative? Doctor: Hi,There is no problem applying antibiotic eye drops either Gentamicin or Ciprofloxacilin.It is topical application and there is not much absorption in the blood.Ok and take care."
},
{
"id": 126985,
"tgt": "Is swelling in the ankles and foot a matter of concern?",
"src": "Patient: I have swelling in one foot and ankle. It happens mostly when it is hot outside, like it is in Texas, but sometimes when it is not hot, but rarely. Is this a medical problem that must of concern? I thought this site was for some immediate answers, not to have to pay for the privilege. You are just a scam and not legit!!!! Doctor: Hi, Ankle edema or swelling can happen due to many causes including long standing hypertension or it can also happen after trauma which will lead to contusion and later lead to ankle swelling. As of now, consult a physician and get evaluated. Hope I have answered your query. Let me know if I can assist you further."
},
{
"id": 53184,
"tgt": "Suggest remedy for hives during Gilbert's syndrome",
"src": "Patient: Hi, I was diagnosed with Gilbert's syndrome 15 years ago (I'm 25 now). Over the last 4 years this disease showed almost no signs. The only problem that bothers me are hives. 2 years ago they appeared on my back and shoulders all of a sudden. Since then they appear every 4-6 months on my shoulders, neck, face and back. My billirubin level is fine (I've been taking Milk Thistle Extract for months), I feel great. When they appear, they stay for 4-5 weeks. They are transparent with liquid content inside and are easily removed. What can I do about hives? Thanks. Doctor: hiThere is no need treatment for Gilbert's syndrome. it is a benign condition that doesn't cause damage to your liver.with regards to hives you should disccus it with a dermatologist.Dr.Klerida"
},
{
"id": 164931,
"tgt": "What causes stomach pain with headaches?",
"src": "Patient: Hello. My 5 year old daughter complains of stomach pain nearly every day. She says it is right over or on the top of her belly button. Her teacher says she naps 3-4 times per day and complains also of headaches. She seems to have a normal appetite, but the stomach pain has not gone on for a few weeks. Not sure if it s time to see her pediatrician Doctor: if there is no any other complaints like fever, bowel disturbances, urinary symptoms etc associated with stomach pain and if your child has stomach pain which subsides by itself within few minutes and does require any medication for same then it is likely functional (no significant internal abnormality) pain. u should see a pediatrician who will examine ur child's abdomen and can tell u if he needs any investigations or not. the other two symptoms does look related to stomach pain. those two symptoms headache and increased sleep during day time may be due to improper sleep at night. u should see a pediatrician as it will help decrease ur anxiety."
},
{
"id": 145420,
"tgt": "Suggest treatment for cervical radiculopathy and neck pain",
"src": "Patient: I FELL BACK IN FEBRUARY AND WAS DIAGNOSED WITH C6 AND C7 CERVICAL RADICULOPATHY. I VE BEEN OK SINCE, UNTIL NOW. I M CURRENTLY HAVING NECK PAIN THAT GOES ACROSS MY SHOULDER AND INTO MY RIGHT ARM. THE PAIN TENDS TO INCREASE WHEN I INHALE. COULD THIS BE FROM MY FALL? Doctor: You may use a cervical collar for support of neck, do isometric neck exercises under a physiotherapist. take dolo-neurobion One tab BD for 10 days. Do hot fomentation also"
},
{
"id": 145931,
"tgt": "How to manage multiple sclerosis?",
"src": "Patient: I have had M.S. for almost 30 years. After this long, family just seems to ignore it. Sometimes I want to scream it out, but that would seem too in your face. I don't want them to feel sorry for me, I just want them to understand that it just doesn't go away. Doctor: I read your question carefully however you don't say anything about your disease course which can be different from one patient to the other. You don't say anything about your medication either.Generally speaking MS is managed by treatment for relapses with short courses of steroids and disease modifying therapy which traditionally has meant interferons or glatiramer acetate, while now there are many newer options like fingolimod, natalizumab etc.As for the family understanding there is no medicine for that, you could try to talk about it with them, take them with you when you go for check-ups so they can hear what your doctors have to say and realize what you go through.I hope to have been of help."
},
{
"id": 40175,
"tgt": "Do I need to continue remaining two doses of rabipur vaccine?",
"src": "Patient: My dog was sick for few days and while feeding accidently the teeth had a contact on my open wound.I am not sure of completing the full rabies vaccine schedule to my pet. So next day i took Tetanus and started Rabipur PCEC rabies vaccine. I have completed the first 3 doses i.e 0,3 and 7 day. My dog is healthy now and its been 12 days since the accident. So my question is do i need to continue the rest of my 2 doses or is it ok to stop it now. Doctor: HelloWelcome to HCM,As you are bit by a partially immunized dog and taken three dose of antirabies vaccine on days 0, 3 and 7 along with tetanus injection.As the dog is healthy after 12 days of bite, it suggest that dog is healthy and has not develoed any symptoms of rabies. I would suggest you to take one more dose on day 21/28 , so that you can convert your post ecposure prophylaxis into pre exposure prophylaxis. Thank you."
},
{
"id": 169559,
"tgt": "What causes cheat and throat pains and fever in kid?",
"src": "Patient: My daughter is almost 7 yrs. old. On Friday and Saturday, she had a fever, on and off. Yesterday and today (for over 36 hours) no fever. Now she has been complaining that her chest and throat hurt - and says her chest pain is worse when she lies down? I m taking her to the doctor as soon as their lunch ends at 1:45, but was wondering what this could possibly be? Is it serious or perhaps just a pulled muscle? Doctor: Hi Dear,Welcome to HCM.Understanding your concern. As per your query your daughter have symptoms of chest and throat pains and fever which is due to bacterial/ viral infection which is leading to respiratory tract infection. I would suggest you to visit pediatrician first once and get it examined and start treatment after proper diagnosis. Get blood test done to detect level of infection. Give full rest to your child with sound sleep. You should do warm saline gargles 3 to 4 times a day. Take ibuprofen one tablet twice daily. Throat lozenges one every 4 hours is helpful. Drink lukewarm water throughout the day as it can provide relief. Give soft foods to child to avoid irritation to the throat. You should take steam inhalation. Apply mist humidifiers in room. You can give Meftal P to child for fever. Hope your concern has been resolved.Get Well Soon.Best Wishes,Dr. Harry Maheshwari"
},
{
"id": 219159,
"tgt": "Sore breasts, negative pregnancy test, metal taste on tongue. How common is having periods during pregnancy?",
"src": "Patient: My breasts have been sore all over, especially my nipples ,for the past two weeks. My husband and I were scared that I may be pregnant, so i took a test. It was negative, however, my period missed away and my breasts seem to be getting bigger and hurt worse everyday. I have gained 7 pounds in the last month and feel very exhausted for no reason.(April 2nd is my last mensural circle and we took the test on may 11th).Nowdays sometime i feel like a metal taste on tongue ..and sometimes feeling tired.. Should I take another pregnancy test? I know that some women still have their period during pregnancy. How common is that? I do not have health insurance right now so going to the doctor is out of the question. Any help would be appreciated. Thanks. Doctor: hello,.Your LMP was 2nd April and did pregnancy test on 11th which was also negative.There is complete absence of period during pregnancy. If bleeding is present, then possibility of abortion is there.Do a USG lower abdomen to rule out underlying pathology or conception.There are several reason of missed or delayed period like thyroid problem, psychological upset, stress, over wt or extreme loss, eating disorder, ovarian cyst, hormonal imbalance etc.Do some hormone tests like thyroid profile and LH/FSH.Then consult with your doctor with all test reports and take advice accordingly.Take healthy diet, control body wt, avoid stress and need proper sleep.Also wear tight breast support to relief from breast problems.Be well and practice safe sex to avoid unwanted pregnancy."
},
{
"id": 167868,
"tgt": "Is small head size of fetus sign of downs syndrome?",
"src": "Patient: I have just had a 23 week sccan and the baby s head measured small in the bottom 5 percentile at 198.6mm. The BPD was 55.4, AC 182.8 and FL 38.1. The tummy is the only part that is showing on track, although the head measurment is the most worrying. The baby presented in the breech position and my doppler results were RI 0.58 and my RI 0.77 and the cerebral artery was PI 1.26. Its all very worrying. I have a 0.83 percent chance that my baby has downs. Is this confirmation of this fact with the head and femur measurement? Could it be another disorder or is it restricted growth? I am being seen for another scan in 3 weeks which seems so long away. Thank you Doctor: diagnosis of down syndrome prenataly includes the measurement of something called Nuchal translucency it's Measured around the 20th week of gestation , serum levels of inhibin, B-HCG , alpha feto protein , and estradiol are very helpful microcephaly (small sized head ) alone is not diagnostic of down syndrome , it's associated with a lot of disorders which is best diagnosed postnataly. I hope your child has nothing of these disorders I hope this answers your question"
},
{
"id": 32941,
"tgt": "What is the treatment for high fever and dry cough?",
"src": "Patient: My wife age of 25 yrs. She is suffering with following diseases: 1.) Since last 15 days she is suffering with fever of 102 to 103 regularly. 2.) Also, since last 15 days she is having dry cough but before 15 days that she was having wet cough (not dry). 3.) Also, as per chest sonography report there is right pleural effusion (approx 600ml) Doctor: Hello,It seems that your wife has lung infection which could be bacterial or tuberculosis. I would suggest getting a pleural tap done, and examined for gram staining, Ziehl-Neelsen (ZN) staining, and cultural sensitivity for aerobic and anaerobic bacteria and tuberculosis infection. I believe you require taking treatment according. In meanwhile you must take antibiotics after prescription by your physician. Eat a protein-rich healthy and balanced diet with multivitamins and calcium supplements.Hope I have answered your query. Let me know if I can assist you further.Regards,Dr. Sheetal Verma"
},
{
"id": 117265,
"tgt": "Should i start taking 'hydrozyrurea' for polycythemia vera?",
"src": "Patient: My current hemotologist tells me he suspects I may be entering the \"spent phase\" of polycythermia vera. I was diagnosed 3 years ago with PV and ET and have been taking anagrelide and aggrenox since then. My counts are mainly normal now (e.g. hematocrit at 42 and platelets at 330k) except that my Hg is low at 12.9. I am thinking that it may be time to d/c the anagrelide and change to hydrozyrurea. I am also looking for ways to boost the longetivity of my bone marrow. Doctor: Hi,Thanks for asking.Based on your clinical history and query, my opinion is as follows:1. All the counts are near normal limits. There should be no reason to discontinue anagrelide.2. Changing to hydroxyurea could cause rebound increase. Hydroxyurea is known to increase risk for leukemias and need not be started at present.3. Dose reduction can be discussed with your doctor to boost longetivity of bone marrow.Hope it helps.Any further queries, happy to help again."
},
{
"id": 183928,
"tgt": "What causes bleeding in tooth after a root canal treatment?",
"src": "Patient: I'm a 17 year old female going to college in the fall about 1 1/2 hours away. I've been dealing with root resorption in my right front tooth for about a year now, and have had a root canal done. When that was done my tooth was grey and after putting a bleach pellet in it to whiten it up, it sent severe nerve pain to my mouth and surrounding teeth. The other thing we did when that did not effectively work was my dentist gave me a veneer on my front tooth which still has not effectively masked the grey. I went to him yesterday for a consult and we have decided on putting a full crown to attempt to cover it yet again. That's honestly the least of my worries at the moment though because I took a nap just an hour ago and when I woke up and looked in the mirror I had blood surrounding my front tooth. My dentist and endodontist have already told me I won't be able to keep my tooth forever but they don't know when the loss will be - whether in 1 year or 20. Is this a bad sign? (He didn't even touch my tooth yesterday during the consult) and the blood was only on that one tooth. Doctor: Dear Madam, I understand your problem.you must have undergone some trauma to your front teeth several years before root canal treatment was done.Thats the reason for the resorption of the teeth and colour change.your dentist has attempted saving the tooth since its not mobile and its savable.Now the bleeding might be because of the gum problem surrounding the tooth and the veneer .They would have attracted food and plaque which resulted in inflammation of gums and in turn bleeding.Just clean under the gums around that tooth with a dentist and take antibiotics.Nothing to worry about this bleeding and the lifespan of your tooth.Hope your query is answered.For further queries pleaseDont hesitate to contact back.Thank you."
},
{
"id": 207521,
"tgt": "Suggest treatment for Parkinson s disease",
"src": "Patient: I am considering taking a supplement called Accel. It has 50 mg of ubiquinol in liquid form plus vitamin E (tocotrinols) in a gel tablet. It is marketed online by Dr. Al Sears via a company called PRIMAL FORCE. (not a trust inspiring name!) I have Parkinson s Disease and when I asked my nuerologist he said a recently completed clinical trial did not bear out the theory that CoQ10 helped with Parkinson s symptoms. In spite of this disappointing news, there seem to be other benefits that might make me feel better i.e. energy, cardio-vascular health, and lower cholesterol, etc. I would be interested in your opinion. Edith Dzubay Doctor: hi..EDITH,, thanks for writing..i have understood your concerns..the parkinson's disease is a neurodegenerative disease, in which the dopamine containing neurons in the basal ganglia region of the brain gradually degenerate over time..there are hypothesis about the oxidative stress as a etiology of the disease..so further research proposed the possible role of antioxidant medications in prevention and treatment of parkinsons disease.variety of agents have been identified and many trial have been done, but none have shown any promising results. so there use is still a debate.. our clinical experience also have negative opinion..for the parkinsons disease in the initial stage when the symptoms are mild, at that time their use can be beneficial for delaying the degeneration but at severe stage they are of no benefit..if there are documented benefits over other systems of the body then you can give it a try..[most of the studies shows that benefits are unreliable]dont expect that it will help you in your parkinsons..i hope this information will be helpful to you..best of luck..wish you a good health.."
},
{
"id": 177840,
"tgt": "Why is my infant constipated?",
"src": "Patient: My 2 month old baby goes 2 weeks without pooping, I have tried prune juice, water, apple juice, tummy massage, but nothing worked. To make her poop, I have to use a medicine dropper to insert warm water into her rectum, the water will come out with fesses. And the stools are soft and yellow. I don t want her to get dependent to that method. Please help me, what should I do. Doctor: Hi Dear welcome to the HCM,Whenever it goes a week with passing the stool one has to use glycerine rectal suppository consequently for three days.Some honey mixed with water to be tried.But history is of soft stool even after one week of stay then the child to be investigated.( especially for thyroid, hersprung disease etc)Please contact your pediatrician for further workupHope the query is answered,thanks"
},
{
"id": 150371,
"tgt": "Brain rush sensation, lost consciousness, sleeplessness, vertigo. Was on antidepressants. Will I be okay?",
"src": "Patient: hello - i hope you can help me with a weird symptom that i've recently developed. it happens at night, and the only way i can describe it is \"brain fizz\" or \"brain rush\". it's a sensation of my brain flickering on and off. it was pretty scary the first couple times it happened - i thought i was losing consciousness, but i didn't. i was afraid to go to sleep for a couple of nights tho! there seems to be a connection to what i'm eating. nuts, caffeine, chocolate and cinnamon can all cause vertigo, and i wonder if this is somehow related. i did some food diary-ing, and there was a common denominator of some cookies i was eating, and thought there may be almond extract in them, but the bakery swears there isn't. it could be gluten too, i suppose. i recently gave it up and feel soooo much better, in my head. less foggy ,more energy ,more coherent, etc. have you ever heard of something like this?i did find some reference to what sounded like this, from people coming off of antidepressants. but i don't take any. i would love to hear your thoughts!i am female 47 years old. thanks! Doctor: Hello, welcome to Healthcare Magic. You have following symptoms: scary feeling (anxiety), tingling over head, health worries and poor sleep. Your symptoms are related to anxiety. Anxiety disorders can manifest not only with psychological symptoms but with a variety of physical symptoms also like: Tingling, numbness, hot and cold sensations, palpitation, chest heaviness, choking sensation, poor sleep, tremors (shakiness) etc. It is better to get evaluated by psychiatrist and start treatment accordingly. For long term control of anxiety following measures are helpful: relaxation exercise, medicines like SSRIs and non drug treatment like CBT. In food avoid or take minimally caffeine containing items which increases anxiety like: Tea, coffee and cold drinks. Same time do some relaxation exercise like daily brisk walk, yoga and breathing exercise, which will help you in fast recovery. Wish you good health and all the best. Regards, Dr Ashish Kumar Mittal www.99doctor.com"
},
{
"id": 49935,
"tgt": "Mild hydro nephrosis shown in renal CT. Diagnosed with kidney disease due to lupus. Had urinary tract infection. Treatment?",
"src": "Patient: Hi my name is Jayme I recently had a renal ct that shows mild hydro nephrosis and it said course echotexture of the kidneys. 11.1 right and 11.4 left! I've been diagnosed w kidney disease in the past due to Lupus, however the nephrologist I recently seen said blood work is fine and \"Im healthier then he is\"! I'm so confused! I had a uti for the past 2yrs straight and frequently have infections and nephritis by scans! I know drs don't like to go against another, but in general what does this mean and do I see a diff dr? Help!! Doctor: HI, thanks for using healthcare magicKidney disease would be diagnosed by imaging and blood tests. The blood tests that are normally done to assess kidney function are creatinine, urea and also calculation of the glomerular filtration rate.Though the CT scan demonstrated mild nephrosis which is degeneration of the tubules in the kidneys the blood tests may have shown that there is no evidence of impaired function of the kidneys.This means that though there is damage so far the kidneys may still be able to function appropriately.I hope this helps"
},
{
"id": 27163,
"tgt": "Does palpitation, itchy eyes, headache, forehead/chest pain indicate hashimoto s?",
"src": "Patient: Hi my tsh is also 0,016 and ft3 ,ft4 in range, but my doctor said i have hoshimoto. I have some symtoms like palpitation, itchy eyes,headache or actually forehead pain,sharp pain in my chest:( its really hashimoto? i spend to much money for my health :/ i forgot - is one important thing- i use to take Roaccutane for acne,and after finished my treatment i start have all these problems...is is thyroid problem..or something more serious? Doctor: Hello, generally hoshimoto present as hypothyroidism, which means your thyroid is working on lower side, but here the Tsh should be high ) normal Cleveland in most lab - till 5. Now your Tsh is very low, it means you have hyperthyroidism from any cause, which fits with your symptoms of palpitations And other s. Some tests to confirm hoshimoto is s anti tpo, anti thyroglobulin and anti microsomal antibodies. Any one of these three are positive in most of the cases of hashimoto . Roaccutane can cause autoimmune hypothyroidism, ie. Hashimoto. You should stop it immediately if proven as the cause. Also some symptoms may be due to roaccutane. Hopefully I could bring some insight. For any future assistance, you may upload tests and clarify @ Http://doctor.healthcaremagic.com/doctors/dr-priyank-mody/70273. Regards Dr Mody"
},
{
"id": 159458,
"tgt": "Persisting diarrhea, mucus in stool. Stool test suggested. Chances of cancer?",
"src": "Patient: Hi, my stomach is upset since 3 weeks... 1st i had loose motion for 1 day then 4 to 5 days motion was not going, then everything was normal, than again little plain started...5 days back i again visited the doctor he asked for stool test and he prescribed nutroin-b plus and pancron 40..today instead of motion mucus is going... can u help me with this.... i am afraid regarding the mucus as i heard the symtom of cancer Doctor: Hi pillair, Make You clear that You have no CANCER. Symptoms you got are of Diarrhoea , Possibility of Dysentery too. You have to take ORS - Oral Rehydration Solution & make it in Water & Drink as much as possible to avoid dehydration. Diarrhoea also be treated by anti-microbial ask Ur Doctor to prescribe. Take rest & Avoid Spicy Foods. Get Well Soon . - Regards, DR JIGNESH ."
},
{
"id": 143770,
"tgt": "How to determine Glasgow Coma scale?",
"src": "Patient: I am researching the Glasgow coma scale. After my son s TBI, he was asked questions like his age ( which he responded wrong to on 2-3 occasions), the day of week, where he was(city) etc. However, when I see info for GCS, it seems that there are different parameters for developing a score? Doctor: Hi ,your son is not responding correctly to questions verbal response is V4.GCS score is calculated based on Eye response ,Motor response and verbal response.Total score is 15/15.It is for assessing the level of conciousness. Thanks"
},
{
"id": 134572,
"tgt": "What causes swollen thighs and ankles with large blistets on legs?",
"src": "Patient: I am being treated by my family physician and have had very swollen thighs and ankles for 2 years. I presently have had two boutes of large blisters on both legs which have been draining a clear fluid. Swabs have been taken which have turned up negative for bacteria. I have had an dopler and arterial scan, seen a heart specialist to no avail. I was put on a course of dyretics which cause my kidneys to fail and spent ten days in hospital to kick start the operation.l At the present time my uric acid and creatine levels appear normal once again but they are following them closely. No one seems to know what the problem is. Do you? signed Barb Doctor: I wish you can take some advice from a physical therapist for your swelling and cardiac endurance training under his or her guidance. it will surely help you avoid diuretics and reduce your swelling as well. kindly meet a vascular surgeon and a dermatologist also. I needed you can take an advice from a diabetologist as well."
},
{
"id": 9637,
"tgt": "Can Benadryl be taken for allergy who has a bladder infection?",
"src": "Patient: My wife went to the ER on Sunday night and had a bladder infection, so the prescribed ferrous sulfate 325mg. Last night, we had shrimp for diner and she had an allergic reaction and her whole body itches. This morning she bought some benadryl for the allergy but the ingredients say it contains calcium and ferrous says not to consume calcium. Any recommendations? Doctor: Hello Juan.idea is not to take iron and calcium together.they can always be taken in different times of the day."
},
{
"id": 193593,
"tgt": "Suggest remedy for removal of stretch marks like darkness in penis",
"src": "Patient: hello. i am a 16 year old male, and i have noticed that my penis has become much darker. also, what appears to be stretchmarks have appeared on the shaft. how can i remove these marks, or atleast make them less visible? and about the skin tone colour, any suggestions? Doctor: Hello, There are no medicines available for whitening the penis skin. The stretch marks could be due to dryness. Talk to your skin specialist. Hope I have answered your query. Let me know if I can assist you further. Take care Regards, Dr K. V. Anand, Psychologist"
},
{
"id": 145799,
"tgt": "What causes numbness in thumb after wrist fracture?",
"src": "Patient: Hi I fell fully forward onto my hands/wrists about 10 weeks ago. Xray shows no fracture of my right wrist (the left is fine) but it it painfull to use, especially in a twisting action ie. open a lid of a jar etc. 4 weeks ago the tip of my thumb (outer aspect only) went numb suddenly and is still numb now. My GP said it will just go away by itself but i am worried it might be nerve damage? Do I need to see a physio? Doctor: Hi,Thanks for writing in.The hand and wrist is supplied by radial and ulnar nerves which originate from nerves in the neck. There are possibilities of the nerves supplying your hand getting pinched some where in their path causing pain and distress. If you never had a fracture then there is no significant injury to bones. However the injury to nerves cannot be ruled out without proper clinical examination. Also when doing a twisting action, there is complex movement at the wrist joint and that can cause pain because of ligaments getting stretched. It is best you see a neurologist and then check with your physiotherapist. Seeing a neurologist is more important because the tip of your thumb remains numb even after 4 weeks. A test like electromyography and nerve muscle conduction might be required after clinical assessment."
},
{
"id": 85775,
"tgt": "What are the side effects of Betacap and Tryptomer?",
"src": "Patient: Hello Doc, I am suffering from chronic migrane from past approx 10-12 yrs. I am taking betacap tr 80 once in a day and tryptomer 10mg. but some times i used to miss the tryptomer at night. is there any side effects of these medicines. can you tell me the best way to cure the migrane.? Doctor: Hi, Tryptomer (Amitriptyline) can cause various side effects. Some of them are: CV: Orthostatic hypotension hypertension tachycardia palpitations arrhythmias ECG changes CNS: Confusion hallucinations disturbed concentration decreased memory delusions nervousness restlessness agitation panic insomnia nightmares mania exacerbation of psychosis drowsiness dizziness weakness emotional lability numbness tremors extrapyramidal symptoms (eg, pseudoparkinsonism, movement disorders, akathisia) seizures DERM: Rash pruritus photosensitivity reaction dry skin acne itching etc. The prophylactic treatment of migraine include Propranolol (Betacap ) and Amitriptyline (Tryptomer). The probability of success with any one of the anti-migraine drugs is 50\u201375%. Many patients are managed adequately with low-dose Amitriptyline, Propranolol. Many patients are able to discontinue medication and experience fewer and milder attacks for long periods, suggesting that these drugs may alter the natural history of migraine. Hope my answer was helpful. If you have further queries feel free to contact me again. Regards, Dr. Tushar Kanti Biswas, Internal Medicine Specialist"
},
{
"id": 137186,
"tgt": "What causes sudden spasm in mid back and stomach area under bra line?",
"src": "Patient: i have had two occurrences this week with what feels like a sudden muscle spasm in my mid back....under the bra line. It stays for about 5 minutes then goes away. The last one however was accompanied with a spasm in my mid stomach area on the right side directly under my bra line. Doctor: Hi there.This could be due to a pinched nerve at the back or due to bad posture at work or sleep. I advise you to follow ergonomics, apply hot water bottle to that painful area, take pain killer and muscle relaxant."
},
{
"id": 160575,
"tgt": "Suggest medication for painful lump on right side of the neck",
"src": "Patient: Hi, my I just gave my 5year old son a haircut a week ago with a razor, by weekend he had developed a lump on his right side of a neck I m really woried it doesn t seem to be painful when I touch it though, but he had one he was born with close to that one and it never grow Doctor: Hi,Can you mention the exact site?Small firm swelling in neck, that freely moves under the skin is probably a lymph node. If it is small (diameter less than 1cm) and it is not painful or tender to touch, we need not worry now. We can evaluate further if it becomes painful or progressively increase in size.Hope I have answered your question. Let me know if I can assist you further. Regards, Dr. Muhammed Aslam T. K., Pediatrician"
},
{
"id": 200509,
"tgt": "Suggest treatment for pimple on head of penis",
"src": "Patient: Hi sir I am ajay from mumbai I wnt to ask this that in my pennis of top I got small small pimple.and I m doing hand job before 7yrs and some thing white water fell from penis. Sir plz give me right answer that I can do something or I start medicine or any plzz DR. HELP Doctor: Thanks for asking in healthcaremagic forum It is normal to get white discharge(semen) after masturbation and sex. YOu need not worry about this. Pimple like lesion may be due to rough handling with dryness. Consult dermatologist for help. All the best."
},
{
"id": 2436,
"tgt": "Is pregnancy possible after ipill and usage of meprate 10mg?",
"src": "Patient: I had last periods on 16 nov And on 21 nov I had unprotected sex with my boy friend nod taken i pill on same day but now on 16 dec I didn't get my due periods still 25 dec nod I also taken tab meprate 10 mg for 5 days one in a day and also done pregnancy test but it is negative so there is chance to have pregnancy? Doctor: if you did sex on 5 th day 99 percent you will not get pregnant because you did sex in safe period pnly"
},
{
"id": 217797,
"tgt": "Suggest treatment for chronic pain in joints",
"src": "Patient: I have been suffering with chronic aches and pain in my joints i have degenerative disc in my cervical neck and back, i am chronically tired and always cold, it is worse in the morning and after not moving for a while. my stomach also is upset and i am all nausea. i have recently gone off taking cymbalta. can you help me. Doctor: Hai.looks ur in chronic neuropathic pain, with degenerative disc problem &arthritis. Pain is normally more in the morning. U hav to get up with sloe warming up exercises. For ur arthritic pains u can start with immunomodulators likediacerin added with glucosamine for 6 to 8 weeks.U can start with pregablin or gabapentin added with methylcobalamin daily once and see the wonderful change.these are safe on the kidneys also.Opiods &neurolytic blocks are of the last choice.Teke care , good luck"
},
{
"id": 91924,
"tgt": "What could be the reason for having stomach pain after eating raw garlic cloves?",
"src": "Patient: Hi Doctor, I got severe stomach pain suddenly after earting raw garlic cloves. This happened on day before yesterday night. In the next morning, i felt better and took cyclopam tablet in the morning and afternoon.After that I felt better. but the pain is not completely disappeared. I am getting mild stomach pain still for a short time and appeared after some minutes again.please help me.Thanks,Rajesh Doctor: HI, thanks for using healthcare magicThe gastrointestinal tract may still be irritated by the use of the garlic cloves. It can take 36 to 72 hours for resolution to occur.The use of probiotics may help to relieve any irritation present. These can be taken on their own or in foods such as yogurt.I hope this helps"
},
{
"id": 154562,
"tgt": "Suggest remedy for reducing excess fluid in body after prostate radiation treatments",
"src": "Patient: Good evening, I had prostate radiation treatments for 44 days from December, 2013 through February, 2014. My PSA test three months later showed no trace of cancer cells remaining. However, I continue to have excess fluids in my legs, ankles, feet, thighs, etc. I also had a six-month injection of Lupron in October of 2014 prior to beginning radiation treatment. Is there anything I can do to reduce the excess fluid in my body? Thank you for the courtesy of a reply. David Doctor: Hi,Thanks for writing in.Collection of fluid following radiation treatment is seen after radiation treatment in many patients. Collection of fluid in the soft tissues of legs is known as pedal edema and can be seen in people having problems with heart or lungs or those with excessive fluid retention following cancer treatment. This happens due to the collection of water in the extra cellular compartment in soft tissues below the skin.It will help to consult your oncologist and then get a prescription of water pills. These will allow the extra water to be diverted to the vascular compartment and then excreted through the kidneys. This is a temporary phase and completely treatable. Please do not worry."
},
{
"id": 45827,
"tgt": "Suggest remedy for kidney problem",
"src": "Patient: sir i am a patient of Apollo hospitals of chennai. my name is batenur rahman. my id is 0000. i am a kidney transplant patient. i am getting abscess. i got a report of my pus culture. how to fax in nephrology unit of apollo hospitals. there are diferent fax numbers Doctor: Hello and Welcome to \u2018Ask A Doctor\u2019 service. I have reviewed your query and here is my advice. You can contact the public relation department number and directly mail to the department. They will forward the report to your treating doctor. Hope I have answered your query. Let me know if I can assist you further. Regards,\u00a0\u00a0\u00a0\u00a0\u00a0 Dr. Shinas Hussain"
},
{
"id": 108305,
"tgt": "What causes lower back pain radiating to left kidney with nausea?",
"src": "Patient: Yesterday woke with lower back pain. As the morning progressed, the pain moved over to the left kidney and it feels like I have been kicked in the area. Deep seated bruised type of pain. Yesterday afternoon, took 2 nurofen and again last night. Also having bouts of nausea (no vomiting) and sweating Doctor: Hi,From history it seems that you might be having renal colic giving this problem.You have not mentioned about burning urination or frequency of urine.Go for routine urine check up.After report go for treatment accordingly.Meanwhile take Meftal spas as needed.Take plenty of water.Ok and take care."
},
{
"id": 192768,
"tgt": "What could cause sudden decrease in quantity of sperms during ejaculation?",
"src": "Patient: Hi, I am a 21 year old male. The last week I have noticed a sudden massive decrease in my sperm count, to the point of where I am only ejaculating 3 or 4 drops, and they are not dense at all. I am healthy for the most part, am an occasional smoker and don't drink..I am worried and would like to know of possible reasons and solutions? Doctor: Hi, I can understand your concern for your symptoms, It can be due to dehydration and vitamin d deficiency. Take adequate amount of water as you can. Hope I have answered your query. Let me know if I can assist you further. Take care Regards, Dr S.R.Raveendran, Sexologist"
},
{
"id": 157930,
"tgt": "Prostrate cancer. BHP PSA, gleason score done. Removal of prostrate required?",
"src": "Patient: my partner diagnosed with prostate cancer . We in position of making up minds which treatment to choose. BHP PSA is 4.23, gleason score 3+3, no perineural invasion, no lymphatic invasion, no invasion into fat, right side prostate 4 cores positve for cancer of 6 submitted, left side prostate 3 cores positive of 6 submitted. Age, just turned 60, no history of this in family, active sex life currently, have macroscopic results if you require then. Removal of prostate now seems rather radical, what would you recommend? Doctor: One thing that you have not mentioned is whether you have gotten a CT scan or MRI of the abdomen and a bone scan. The MRI/CT will tell us whether the disease has spread out of the prostate or not and the bone scan will tell us if it has spread to the bones.A chest x-ray should also have been done to rule out spread to the lungs. If the disease is confined to the prostate and has not spread anywhere then a removal of the prostate by a cancer surgeon is advisable as the chances of cure with this surgery near 90%. However, that will put an end to his sex life as you already know. If he is absolutely adamant not to undergo surgery, then the only option available is radiotherapy plus hormone therapy. These have their own set of side effects and may decrease the libido. However, it will not make him irreversibly impotent. The downside is that the cure rates will fall and there is a chance that the disease may actually progress and spread to other organs. So, in a nutshell i will advise you to motivate him for the surgery else he runs the risk of putting an early end to his life. In case the disease has already spread to other organs, then the only option is hormone therapy."
},
{
"id": 149423,
"tgt": "Have been taking TOLREVA. Eye under pressure, frequent mood swings, nausea. Reason?",
"src": "Patient: HELLO, i have been taking TOLREVA 500 mg , twice a day, and it has been 2 year and 4 months now, i have been visiting my Neurologist on timely basis during this period, i didn t encounter with an seizure episode during this period of treatment, but i do sometimes feel quiet heavy pressure near my left eye , frequent Mood swings, and nausea what could be reasons behind this, and how they can be over comed or treated Doctor: Hi,Thank you for posting your query.Torleva (levetiracetam, Torrent pharma) is a good anti epileptic drug to control seizures. It has a very good safety profile. However, in some people, it can cause behavioural problems and mood swings. If these are minor, you may continue to use torleva. However, if your mood swings are severe and affecting your daily life, then, you should consider stopping it and replacing torleva with another seizure medication.Please discuss these issues with your neurologist during the next visit.I hope it helps. Best wishes,Dr Sudhir Kumar MD DM (Neurology)Senior Consultant NeurologistApollo Hospitals, Hyderabad"
},
{
"id": 183345,
"tgt": "Does daily fluoride treatment cause mouth ulcers and spontaneous bleeding?",
"src": "Patient: I have been using a fluoride treatment to keep my teeth safe after radiation treatment last year after my redicalo neck dissection. will this daily fluoride treatment eventually cause ulcers to form on the roof of my mouth and spontaneous bleeding? The blood is bright red and after a few rinses starts to calm down. Usually occurs during brushing. White male 56 years old 180lbs. Thank you Doctor: I dont think so. Fluorides does not have these effects. This may be for your radiation therapy. Or this may be some othe bleeding disorder. Find one dentist and consult with your radiologist also"
},
{
"id": 119957,
"tgt": "How to treat the pain in cervical spondylosis?",
"src": "Patient: i am 29 yrs old unmarried female.. I have cervical spondylysis.. Since few months i am getting severe pain in 1. Area between left hip bone and thigh.. Its so severe as if i think its getting dislocated.. I cant even change from one side to another when i am sleeping.. This happens everyday..i have to take support and get up in the morning.. Had done scaninig and mri but doctors didnt find ne abnormality.. Medicine give me relief for time being.. 2. Since few months new problem have started.. If i sit for long my heels start paining and i cant even touch my feet down or walk.. This happens atleast 5 - 6 times a day. Please help.. My height is 5.1 and weight is 55 kgs Doctor: Hi, Treatment is multifaceted ranging from simple physiotherapy and posture correction to surgical decompression and fixation. Initially, conservative treatment is give for a period of 6 weeks. Only if this is not effective or there is rapid progression of symptoms, we have to switch to surgical management. Hope I have answered your question. Let me know if I can assist you further. Regards, Dr. Rohan Shanker Tiwari, Orthopedic Surgeon"
},
{
"id": 13294,
"tgt": "What causes skin rash on the back?",
"src": "Patient: I have this rash on my back which all the doctors are saying its from my heating pad. I haven t use the pad in 4 days and marks are still there. I have had Lyme disease 4 times since 97. We live in a deer protected area and surrounded by 5o + acres of protected woods. last time was 6-12-14 where my blood test indicated that I should take further testing as it was almost at the positive range. Can this be borriles, or western blot. Im sorry I do not have the money to proceed Doctor: Hi,It may be erythema ab igne. Consult the dermatologist for the perfect diagnosis and proper treatment. It may be due to long term exposure to heat. So, avoid the contact with hot pad. Apply alovera containing cream. Take tab vitamin C 1000 mg daily for the long time.Hope this helps.Thanks.Dr. Ilyas Patel,Dermatologist"
},
{
"id": 32270,
"tgt": "What could inflamed and sore vulva with painful urination suggest?",
"src": "Patient: Hi. I am 39 years old. For the last year i have had a very sore, itchy and inflammed vulva. It sometimes hurts when i urinate and it passes the vulva. I have recently had menopause tests as my periods have become very sketchy and irregular. None of the canestan creams are workind. i generally apply them evert two hours. I have only had one partner in twenty years so i don't think it can be anything too serious. Have you any idea's as it is becoming increasingly more painful. Doctor: the symptoms are suggestive of UTI due to vulvo-vaginitis. UTIs are common in females of your age due to short urethra. i suggest you to get a urine r/m test and also culture sensitivity of the urine and start antibiotics accordingly"
},
{
"id": 139179,
"tgt": "What causes burning sensation in the thighs and calves?",
"src": "Patient: Just recently had a cardiac ablation done...they entered both sides of my groin area. Now I am having problems with pain in that area to the point where sometimes I am not able to walk. I am also developing severe cramping with a burning and stinging sensation in my thighs, calves and down the inner leg. I am also suffering pain across my lower back. My pelvis feels like it is about to drop out. Is this something that I need to seek immediate medical attention for. Doctor: HiWelcome to healthcaremagicI have gone through your query and understand your concern.You can take analgesic such as ibuprofen for pain relief. Vitamin B and C helpful in recovery. It doesn't seem to be medical emergency but there is no harm in consulting Doctor. You can discuss with your doctor about it. Hope your query get answered. If you have any clarification then don't hesitate to write to us. I will be happy to help you.Wishing you a good health.Take care."
},
{
"id": 23515,
"tgt": "Suggest treatment for rapid heart beat and breathlessness",
"src": "Patient: i am 36 yrs. old. height5.5 feet, weight 70kgs.my heart beat is very fast, at times it also skips, i am left breathless, i am using oils containing poly unsaturated fats for cooking etc.. can the use of pure olive oil for direct consumption and regular use of a blended oil containing olive & rice bran in the ratio 30:70 help me overcome this health problem. Doctor: very fast heart beat often termed as palpitations by us in medical terminologypalpitations can be of two typenormal(sinus tachycardia)and abnormal(arrythmias)so firstly you have to rule out what is your type of palpitationadv get a holter attached it keeps recording of 24hours and will tell if there are any abnormal beatsas per as your query using three oils in alteration would help you with your lipid profile and will prevent any future blockage but as of now palpitations is a different entity and oils are not going to help in palpitationsthank you"
},
{
"id": 98113,
"tgt": "Hair turned rough after taking homeo medication for hair fall. Reason and alternate medication?",
"src": "Patient: I am suffering from h hair loss problem, I was prescribed following homeopathic medicines. a) Acid phos 30 b) Lyco 200 c) Selenium 30 d) Wiesbeden After taking medicine, I felt that my hair are getting rough day by day, lyk a broom, is it sign of improvement or anything else??? Then I discussed with dr. and then I was prescribed R-89, I want to know, which one should I take? i) All above 4 only ii) R-89 only iii) All above 4 & R-89, both Doctor: hi how is your hair fall problem ? homeopathic medicine cannot give any worse effect .now i m suggesting you that you should go to good homeopathic physician for proper reperterisation of case.he will search out any of one medicine from these.otherwise you can mail me a menses history & any other chronic disease history,medicine history.after that seeing i can help you.you should take rich protein & vitamins diet with these medicine. now you donot take selenium 30. homeopathic medicine is a individualize medicine not like allopathic medicine. every fever patient can take paracetamol while in homeopathic all fever patient could cured by different medicine . Hope I have answered your question. Please let me know if any further clarification is required on anything else. Thank you dr_neerajgupta@hotmail.com"
},
{
"id": 19941,
"tgt": "Do Coreg and Lasix cause muscle pain in the legs?",
"src": "Patient: Background: Father with various heart problems (blockages, attacks) over 20 years. He was in car accident recently, very banged up, had heart another attack after accident. Was released to go home, with new meds, including coreg and lasix -- now complains of extreme fatigue and muscle pain in his legs. Is there any way to tell if that s just pain from the accident bruising or if he s having side effects to the lasix? When he came home a few days ago he (slowly carefully) walked up 2 flights of stairs to reach his apartment. Since then he s been getting progressively weaker, but his bp is 137/69 so I don t think that s the problem. Worried about muscle pain/fatigue being side effect to the meds. Doctor: it can cause cramps due to salt depletion. if this disturbs sleep- quinidine tablets may be helpful"
},
{
"id": 22549,
"tgt": "What causes numb legs in a heart stroke patient?",
"src": "Patient: My mom has had a mild stroke, is intake in rehab, no one seems to be addressing the fact that when she is in bed her legs feel like they have bricks on them. She is fearing paralysis yet she does go for 2 hours of therapy a day for dropped foot syndrome. This was an active, driving woman of 84 before the stroke that they THINK happened even though there is no indication on her MRI as yet. Doctor: Hi,Both foot drop and brick feeling occurs due to neuropathy, that is due to damage to nerves. Now we should evaluate the cause for this. Is she diabetic, if not tested then get the sugars tested. Get vit b12, and thyroid levels done. If any backache is there, then x-ray and if required MRI spine is to be done. In general, she should be on tab neurobion forte once a day which is a nerve vitamin.Hope I have answered your query. Let me know if I can assist you further.Regards,Dr. Sagar Makode"
},
{
"id": 10185,
"tgt": "Can Radiance-H cause painful facial pimples with pus?",
"src": "Patient: Hi Dr. My query is a bit related to skin and hair! I had lost my job, and had been in grt depression for the past three months. hnc i had a very bad hair fall and the dr. advised Radiance H. I have been using it one caps a day, but i have got lot of big painful pimples on my face, that has puss. Is it any way related to Radiance H? plz advise, thnx Doctor: Hello and Welcome to \u2018Ask A Doctor\u2019 service. I have reviewed your query and here is my advice. Radiance H is a biotin and multivitamin and mineral supplements so there are fewer chances of it causing pimples on face. So once consult your Doctor and get evaluated to rule out the exact cause of the pimples. For now you can start applying Muricoprin ointment over it to reduce infection. Also do cool compresses over it. Also a course of antibiotics like Minocycline can be taken for reducing the symptoms. Use Salicylic acid facewash. Apply Adapalene gel at night. Hope I have answered your query. Let me know if I can assist you further. Regards, Dr. Honey Arora"
},
{
"id": 131321,
"tgt": "Could crunching feeling in tailbone withno problem detected in radiographs be displaced ligament?",
"src": "Patient: I fell on the ice 3 weeks ago, I thought I fractured my tailbone; but I had radiographs taken and the Dr. Said it isn t broken. I frequently feel a crunch; I m beginning to wonder if it could be a displaced ligament. When he palpated the area before taking the radiographs he said it seemed to be more in the sacral region, but the radiographs didn t show any problems with my sacrum either. What do you think the displacing feeling could be? I say crunch just because I don t know how to explain it. Doctor: Hi I have read the description . You frequently feel a crunch after falling on Ice and the x rays do not show any fractures. In my opinion it is quite possible that you may have injured the soft tissues and that leads to swelling in the region. This swelling ( not ligament injury alone) may give rise to feeling of crunches. I suggest you start doing spinal extension exercises on a regular basis and hopefully this alone may suffice."
},
{
"id": 190282,
"tgt": "Wisdom tooth infection, locked jaw, difficulty in drinking, talking, yawning. Suggestions?",
"src": "Patient: I have had a terrible wisdom tooth infection which antibiotics have cleared i am in a little pain but have what I can only discribe as a terrible locked jaw - this has lasted for 4 days and I am drinking through a straw yawning is agony and talking too much just causes it to lock more tightly I am now getting panicky as I need to open my mouth desperately also for my job it is necessary for me to be talking constantly and this will be inpossible HELP Doctor: hi hello,difficulty in opening mouth may be due to \u2022 Massage with an ice pack to reduce pain and bleeding \u2022 Try to relax and avoid strenuous physical activity. \u2022 Eat soft food and gradually move on to solid food \u2022 Straw can increase the pressure inside the mouth and can cause bleeding \u2022 Gently rinse your mouth several times a day to reduce swelling and relieve pain \u2022 Do not smoke for 24 hours after surgery. Sucking motion can dislodge blood clots causing dry socket. \u2022 Brush teeth and tongue carefully. spasm caused by infection of oral cavity most commonly seen after tooth extraction.you need to consult ent doctor for further evaluation."
},
{
"id": 135015,
"tgt": "What causes upper arm aches along with swollen wrists and fingers?",
"src": "Patient: I have aches in my upper arm on both arms swollen wrists and fingers swollen with a bulge on the outside of each wrist fingers are stiff and sore. knuckles ache Worse in the morning I can not even lift my arms over my head in the morning. movement gets a bit better as day goes on but constant pain and ache and I am scared Doctor: Hi Dear,Welcome to HCM.Understanding your concern. As per your query you have upper arm aches along with swollen wrists and fingers. Well there can be many reasons for symptoms you mention in query like sudden impacts ,arthritis , rheumatoid arthritis , carpal tunnel syndrome , gout or ganglion cysts. I would suggest you to consult orthopedic surgeon for proper examination . Doctor may order CT scan , MRI along with physical examination , urine test . Doctor may recommend surgical removal or may prescribe antibiotics , allopurinol along with anti inflammatory . Doctor may also recommend physical therapy . For now apply warm compresses and take ibuprofen or acetaminophen for pain . Do not lift heavy object with arm and give proper rest to your arm . Doctor may fix the abnormality by plaster , splint or bandage in case of fracture .Hope your concern has been resolved.Get Well Soon.Best Wishes,Dr. Harry Maheshwari"
},
{
"id": 32948,
"tgt": "What causes sharp pain in one side of the body while recovering from pneumonia?",
"src": "Patient: Age 51, height 5' and weight 160. Strong healthy medical condition, but have experienced bouts with bronchitis ( usually annually) during past 3 - 4 years. First time with pneumonia . . .Day 6 on leviquin (500 mg) but have sharp pain on right side and back of right side. any ideas. Doctor: HiThanks for your query at HCM.Yes if you are having pneumonia then pain can occur. In most cases of pneumonia will cause pleuritic pain or painful breathing due tov nerves irritation by pleura. The pain will resolve with proper treatment depending upon cause of pneumonia whether viral, bacterial or tuberculosis.Get treatment done problem will resolve.Take careDr Sheetal Verma"
},
{
"id": 25701,
"tgt": "Suggest treatment for mild cardiomegaly seen on an x-ray",
"src": "Patient: Respected Doctor 6 months back I had a X/ray that showed Mild Cardio megaly 4months back I visited aCardiologist. HeDiagnosed me with Echo Cardio graph. It Shows my Heart chambers and valves are normal. Ihave no symptoms of any please tell my condition Doctor: Thanks for your question on Health Care Magic. I can understand your concern. Since your 2d echo report is normal, no need to worry for mild cardiomegaly on chest x ray. In my opinion, you should get done blood pressure monitoring because undiagnosed hypertension can also cause mild cardiomegaly on chest x ray. If your blood pressure is also normal then no need to worry for this x ray finding because mild cardiomegaly can be just a subjective finding of radiologist. Hope I have solved your query. I will be happy to help you further. Wish you good health. Thanks."
},
{
"id": 16138,
"tgt": "Rash near nose, blotchy skin on eating something hot. Using steroidal cream. Permanent cure?",
"src": "Patient: Since I moved to XXXXX over a year ago,I ve noticed a rash next to my nose which comes and goes. I saw a doctor Who gave me steroidal cream which seam to help,but for only a short time.ive varied my diets to see if there is a Comin denominator ( none) I stopped drinking hot coffee and gone to frappichinos and it seemed to go away, this morning I had hot coffee and my nose area started to tighten up and get blotchy and for dinner I had hot chili and that set it off , it got really tight and blotchy -- any ideas ? other then that I am in fantastic shape and I eat Healthy Doctor: Hi there Jeneric. Your clinical observations are very astute if i may say so & would help any dermatologist pinpoint the diagnosis. I may be wrong as this is only an online portal & for best results, you need to visit your dermatologist on the ground; But, your condition seems to be what we call Acne Rosacea or plain Rosacea. This will need visual confirmation & as such, feel free to upload clear digital photos on this site. Do ensure you use a sunscreen from dawn till dusk, avoid hot & spicy foods eg hot tea etc & UV light in general. Do try to use a topical antibiotic cream called metronidazole evenings & it just might get rid of your problem. Good Luck! Dr Praveen Rodrigues MD"
},
{
"id": 201939,
"tgt": "Does repressing the urge to masturbate solve emotional issues?",
"src": "Patient: I used to masturbate about once a week. Now because I am Catholic I try not to do it at all. To do this I think I am repressing the urge. I have read that frequent maturbation indicates some unresolved emotional issues. If I have such issues I don't think repression will every solve them. I am 59 yrs old, never been married. Do you think I should seek counseling? Doctor: Hi,Thanks for writing in.Masturbation is a safe solo sex behavior. There is nothing wrong in masturbating once a week at your age. There need not be any unresolved emotional issues. Masturbation helps in de stressing yourself. You do not need counselling. If you feel that you will be relaxed after masturbation then please do it without ant guilt."
},
{
"id": 180105,
"tgt": "What causes bruise on rib cage of a baby?",
"src": "Patient: Hello my 5 months old has a little bruise on a right side of rib cage. I have noticed hard lump under it. And it s hot. She doesn t look sick, no different symptoms and I m sure she hasn t been traumatized. What is it possibly can be? Should I take her to pediatrician ? Doctor: 5 MONTHS OLD CHILD CAN TURN AROUND IN BED . THEY CAN GET BRUISE BY FALL ALSO. THE HARD LUMP ON THE RIGHT SIDE OF RIB CAGE COULD BE LIVER AS IT IS SITUATED IN THE RIGHT SIDE BELOW THE RIBS. ALTHOUGH 5 MONTHS OLD CAN HAVE SMALL LIVER WHICH CAN BE FELT BELOW THE RIBS IN THE RIGHT BUT ITS NOT HOT NORMALY. I SUGGEST YOU TO VISIT A PEDIATRICIAN SOON"
},
{
"id": 214808,
"tgt": "What are the home remedies for recurring hard spots on groin area?",
"src": "Patient: I ve been getting these really hard spots in the groin area, which I believe are cysts. I have been getting these for the last ten months or so and usually at least one really severe painful angry red cyst per month. I ve been using magnoplasm to draw it and then drain it myself. Are there any other remedies to prevent this from recurring over and over again. I ve been forced to take days off from work frequently, have quit the gym and swimming is simply out of the question. Doctor: Hi,From history it seems that you might be having ingrown hair follicle infection giving rise red hard spots.there might be having enlarge inguinal lymph nodes as well.Go for one antibiotic medicine course for 3-5 days.Apply antibiotic cream locally.Keep local hygiene proper.Ok and take care."
},
{
"id": 50449,
"tgt": "Have pain in abdomen and lumbar. Taking lortabs. Had large interstine removed, hysterectomy. What to do?",
"src": "Patient: I've had severe lower abdomen pain for a while now accompanied with lumbar pain. I don't know what is wrong. I have had this pain for long time and have been going to pain clinic. I take lortabs 7.5-500. Recently they haven't helped. I can't sleep bc of the pain. I had my large intestine removed in 2006. It was a dead organ. I had hysterectomy, December 2012 leaving only left ovary, bc I uterine fibroids and endemetriosis. I thought this would end the pain. After a few months the pain returned with a vengeance. I don't know what is going on. I just had a lumbar x-ray on lumbar and after results where sent to doctor he ordered a blood test on kidneys bc he wants to up my dose! Please, any information or advice would be helpful at this point!Thank you Courtney Age 35 Doctor: Hello.You must wait the results of the tests that your doctor has requested you to find the cause of pain(kidney pathology,backbone....)The important is to know the cause of pain for better treatment.If is not possible to find the cause of pain, in Pain Clinic or your doctor can prescribe powerful painkillers more than what you are taking now.I wish you good health"
},
{
"id": 107446,
"tgt": "Suggest medicine for severe lower back pain, cold and cough",
"src": "Patient: I have severe lower back pain for almost 2 weeks. I had a bad cold and cough. While coughing, the lower part of my back seized and has not gone away. I ve been taking Advil, which helps a bit but it never goes away. Also, it hurts when I sit and not comfortable in bed either. What can I do to alleviate the pain? Massage, exercise? Doctor: Hi,For my patients, I recommend hot foments twice daily, advil, physiotherapy, massaging your back with topical gels, muscle relaxants, spinal manipulation stretches, opioids will help you sleep, gabapentin and steroid injections for more sever cases . minor endoscopic or even surgical procedures are needed if the pain is chronic and persistent.For your case its only a mild case that may subsides simultaneously or with some rest so I don't want you to be worried. Please consider a 5 star rating with some positive feedback if the information was helpful. Hope the above information helps you, Any further clarifications feel free to ask.Regards,Dr. Ahmed Aly Hassan"
},
{
"id": 112065,
"tgt": "What should I do cure back pain that arises on standing straight?",
"src": "Patient: Hi my name is Christian im 17 and i think i have a back problem i can't Stand up straight im always walking with my head down . I look at my back in the mirror and one side looks bigger than the other when i try to stand up straight my back hurts and i can't hold my back straight for so long . Doctor: Hi,From history it seems that you might be having Kyphosis-koliosys.Go for x-ray thoracic cage AP and lateral view.Consult orthopedic surgeon and get examined.Ok and take care."
},
{
"id": 189864,
"tgt": "How long will permanent tooth wait to move up if a baby tooth is above it?",
"src": "Patient: How long can a permanent tooth wait to move up if a baby tooth is above it. I have a baby tooth that has moved up in my gum and the permanent tooth is behind it. It s been in there for at least a few years, i m working on getting it out now, if I pull it out, will the permanent tooth move up?!? I m worried. Please help. Doctor: hi and welcome , Primary teeth or milk teeth usually are replaced by permanent teeth at the age of 12 to 13 years of age. In some cases , primary teeth may retain for longer time and get inpacted in the jaw bone making the eruption of permanent successor difficult. Due to OVERRETAINED milk teeth , permanent teeth may get impacted in the jaws or they may erupt in different directions than usual due to lack of space. i suggest you to consult a dentist and get an OPG radiograph done which will show the position of both the teeth accordingly get the milk tooth extracted. after extraction permanent teeth comes in alignment by tongue pressure or you may have to consult a orthodontist for moving the tooth. i hope this helps, take care."
},
{
"id": 181366,
"tgt": "What causes pain in gums while chewing food post extraction of teeth?",
"src": "Patient: I haven't ate in three weeks because the pain of my wisdom tooth was so bad i couldn't eat but I got five teeth taken out on june 14nd when I chew it's sore but no pain or puss in my gums so I know I don't have a blood clot and only being surviving on super soft food and soups but my stool is pure luqid is this normal or is there something wrong with me? Doctor: Hi..Welcome to HEALTHCARE MAGIC..I have gone through your query and can understand your concerns..As per your complain if you had severe pain in wisdom tooth it can be probably due to infection in gum flap that surrounds the wisdom tooth leading to Pericoronitis..As you are on a soft diet which might be lacking an appropriate quantity of fibre therefore you are having loose stools with plenty of fluids..If you still have some infection and pain present consult an Oral Physician and get evaluated and you can be advised Antibiotics and painkillers for reducing infection..For improving bowels take more of green leafy vegetables and salads in diet and do moderate workout..You can also take fibre supplements like Psyllium..Take Yoghurt in diet..Hope this helps..Regards."
},
{
"id": 35714,
"tgt": "What is the small red ring on my leg?",
"src": "Patient: hi, i was just laying out side in the sun when i looked down at my leg and realized there is a small red ring on my leg, no bigger then a centermeter and it has a white center, it is slightly raised. i dont see a bite mark and im concerned as to what it is. Doctor: Hello dear,Thank you for your contact to health care magic.I read and understand your concern. I am Dr Arun Tank answering your concern.It is nothing to worry about, it is the common reaction of the sun light because of longer duration of exposure.I suggest you to apply common sunscreen lotion available at homeYou can also take cetrizine and prednisolone tablet under your doctors guidance. This will clear up the reaction immediately.Please in future apply sunscreen lotion on the exposing part before you go on the sun. I will be happy to answer your further concern on bit.ly/DrArun.Thank you,Dr Arun TankInfectious diseases specialist,HCM."
},
{
"id": 4294,
"tgt": "After D&C, when can I start trying to conceive again?",
"src": "Patient: I had a d&c on Feb 12th as the ultrasound showed no heart beat at 10 weeks of pregnancy and the baby measured 6 weeks. I had an usg earlier at 6 weeks 5 days, then also there was no cardiac activity. My homocysteine level is elevated (around 16 instead of maximum 13 units). I have been having biopreg-f 10D since d&C, but before that also I used to have folvite since June. When should I start trying to conceive? Doctor: Hi, Thank for writing to us. You should start trying for next baby at least after 6months to one year. You should start taking folvite tablet before when you are trying for next baby. Good luck. Regard Dr.Richa"
},
{
"id": 64788,
"tgt": "What does a hard bump on thigh indicate?",
"src": "Patient: i have a bump on my left thigh, feels hard under the skin like I hit it... have a little bit of bruising. I'm 30, 5.6 ft, healthy, noticed it a few days after I did hot yoga, not sure what exactly its from. recently had a bay (3 months). had a blood test done - the hormone that should be in the blood if the muscle is torn wasnt found, had an ultrasound of the thigh tissue - normal. Doctor: HI,Dear, Thanks for the query from you for your painless hard lump under the skin in lt. thigh.1-I feel concerned that you are serious to know where from this hard lt thigh lump- occured after the hot yoga or may have occured when at the bay 3 mth back.?2-In my opinion this hard lump had a bruising meaning thereby it occured from a blunt trauma during hot yoga or trauma when at bay 3 mths back-which is the remote possiblity as its bruised , and most probably appears to be due to trauma during hot youga as it appeared after few days after you did hot yoga.3-In presence of the normal hormonal levels for the muscle injury, it appears to be hit 3 mths back -leading to hard -organising hematoma,which got bruised after few days of hot yoga.4-USG study / Muscle biopsy studie,would fix its origin and cause of development.I would advise to have surgeons opinion from your ER.3-I would advise FNAC to fix the reason ans hence would refer the case to ER Surgeon. 5-Hope this would solve your worry of the hard lump on the thigh.6-Wellcome to HCM for any more query."
},
{
"id": 58086,
"tgt": "Having low energy, nausea, bloating after gastro flu. Frequent diarrhea after gallbladder removal. Cure?",
"src": "Patient: About 10 days ago I had a gastro flu . Since then, I have low energy, nausea, bloating . I had my gallbladder removed 8 years ago and since then I frequently have diaheria. Should I be further investigating my symptoms? I have visited the dr in the past and I am always told to just eat healthy. I am very conscientious about what I eat. It feels like tummy troubles are an ongoing concern. Not sure what to do or ask of a dr on my next visit. Thank you. Doctor: Hi,Thanks for using Healthcaremagic,You have suffered recently a episode of Gastroenteritis, your symptoms of diarrhoea, low energy nausea and bloating if do not improve with healthy eating and treatment.You should take fluid and electrolytes to avoid dehydration and supplementation of pre and probiotics also supplementation of zinc.Usually gall bladder removal doesn't cause any digestive issues,but if symptoms are troubling you please visit Gastroenterologist >Kindly get tests for malabsorption absorption and Pancreas like serum amylase and serum Lipase and other tests like CT scan of abdomen, and colonoscopy with rectal biopsy to rule out various GIT causes.If everything is fine then functional disorder called irritable bowel syndrome should be considered. Your doctor knows all this.Hope this information helps.Take care.Good Luck.Dr.Akhilesh Dubey M.D."
},
{
"id": 207864,
"tgt": "How to get rid of depression and loneliness?",
"src": "Patient: i think i may be depressed, I have no appetite, I'm extremely lonely as I have no friends, i either barely sleep or sleep much too long, i feel like im about to cry most of the time. i have started missing college and am doing terribly in it. im really lonely and i just dont know what to do anymore, as i have tried extremely hard to make friends but they all seem to find someone else better. Doctor: In my opinion, based on symptoms you report, you may be suffering from Depression. It is a clinical condition that can be cured. You should see a psychiatrist. He/She will evaluate your history; Check if you have any other medical condition that can cause depression; may order a few tests. He/She will suggest medicines or psychotherapy or combination of the two depending on the evaluation.Please remember, Depression is a treatable medical condition.I hope this answer has helped you."
},
{
"id": 163295,
"tgt": "Is it dangerous to give 2 doses of Ibuprofen to baby?",
"src": "Patient: I accidentally gave my 8 month old ibuprofen two hours early. She is 17 lbs and I gave her 1.25 ml only four hours a part instead of 6. She has an ear infection and RSV and I thought I was giving her Tylenol the second time but instead gave her two doses of ibprofen. She fell asleep. Should I be worried? Doctor: Hello,Nothing to worry this is just one time and nothing will happen. Your baby has fallen asleep not because of ibuprofen so calm down.Hope I have answered your query. Let me know if I can assist you further.Regards,Dr. Elona Dashi"
},
{
"id": 158215,
"tgt": "Child has crusty brown specks on head. What could this be?",
"src": "Patient: Hi our 2 year old toddler has scab like crusty brown specs on top of his head in a area near the front top of head. some of the spots look like round freckles on the scalp . and one area has a clump of brownish crust. we washed his head and its still all there, some of it combed out alittle and some are clumpy and some are still all over his scalp. What could cause this? Doctor: Hello and welcome to HCM,The crusty brown scabs are suggestive of seborrhea, eczema, dermatitis, or scaly dermatosis.You need to consult a dermatologist for clinical examination, relevant investigations (especially biopsy) and thus further management.Seborrhea is a common ailment seen in infants and toddler.It commonly affects the scalp and flexures of arms and legs.this includes the region of the elbow and region behind the knee joint.Anti-fungal treatment usually suffices and the disease usually clears away as the child grows.However, definitive diagnosis can only be made after clinical examination and investigations.Thanks and take careDr Shailja P Wahal"
},
{
"id": 153009,
"tgt": "Suggest treatment for ovarian cancer",
"src": "Patient: I have been overly tired, constipated, and very bloated for awhile now. I was first scared for pregnancy, but took a test today which came out negative. i then found results online for ovarian cancer, i have had previous problems with my ovaries, and have also had abnormal amount of discharge on top of the previous symptoms. should i worry about cancer? Doctor: Hello dear. I have gone through your details. These symptoms may or may not be due( most likely not) to ovarian cancer and in my opinion you should not worry undue. You can get yourself checked by means of a simple sonography. Thanks"
},
{
"id": 48255,
"tgt": "Are there any foods i can consume to build up my immune system,and any advice to battle the BK virus with rejection?",
"src": "Patient: I am a kidney transplant patient and having issues with the BK virus. I have experienced rejection at first and was treated with steroid regimen about 3 different occasions. Then the BK virus showed high counts in my lab results. My nephrologist tried to adjust my immune suppressant medications, but my BK readings were still high. My doctor told me he can treat for rejection and he can treat for the BK virus, but he does not know how to treat for both occurring at the same time. Any suggestions? I have had my new kidney for 4 1/2 years now. In this period of time, my creatnine level has gone from 1.0 to 3.7.Are there any foods I can consume to build up my immune system, and any advice to battle the BK virus with rejection? I am a 56 yr old male and also have diabetes and high blood pressure, my blood sugar is under control and blood pressure readings are normal. Doctor: HiThanks for your query.If you have both rejection and a BK Virus infection then the treatment can be tricky. Some patients in a similar situation have shown benefits with Iv Immunoglobulins (IVIG).I would also recommend a kidney biopsy if its not already done.Unfortunately there is no way to treat BK virus by making modifications in your diet.Hope this helpsGood luck"
},
{
"id": 150608,
"tgt": "Suffering from multiple tuberculomas of brain, mass on neck. On AKT, solonex. Sufficient?",
"src": "Patient: My wife is suffering from multiple tuberculomas of brain. She is on AKT for the past 5 months, but the lesions are remained the same. Few days back she developed a mass in neck . Doctors have taken biopsy and confirmed that it is tubertculoma and now they have kept if for culture. It will take 8 weeks for report to come. It seems that present treatment is not working , because lesions are same in size. She is taking Rcin 450, Solonex 300, Combutol 800, PZA 750 and glevo 750. So please tell me that if these medicines don t work then what other medicines can we start ? Doctors are telling me to wait for the culture report, but meanwhile if it increases then what ? Doctor: Hi, Thank you for posting your query. Some cases of TB may be drug-resistant, that is, they do not respond to the conventional anti-TB drugs. In these cases, second line anti TB drugs are required. These medications may include cycloserine, ofloxacin, streptomycin injections, etc. Please follow the advice of your doctor and ensure that your wife takes the medicines on a regular basis. Please get back if you require any additional information. Best wishes, Dr Sudhir Kumar MD (Internal Medicine), DM (Neurology) Senior Consultant Neurologist Apollo Hospitals, Hyderabad, My personal URL on this website: http://bit.ly/Dr-Sudhir-kumar My email: drsudhirkumar@yahoo.com"
},
{
"id": 148097,
"tgt": "What causes numbness and pain in legs when suffering from fluid retention in spine?",
"src": "Patient: Woman age 41. Since thanksgiving 2014. Has pain and numbness in both legs and numbness in left arm. Had MRI of brain looking for MS' but was negative. Has had MRI of upper back and found some fluid on the spine. Next week is having MRI of entire spine to look for more fluid with results of MRI to be read by a neurosurgeon anticipating putting a shunt in her spine. I might add that she can no longer drive or work a full day at her desk job. Can no longer shop...walking from her bed to bathroom and back is exhausting...energy is drained. Any other ideas? Thank you so much for any help. Marcia wilson Doctor: Hi, welcome to our site. I am Dr Saumya Mittal.Read your query. That is a very significant question and i appreciate your problem. I will try my best to answer your queryWhat happens is that the nervous system is usually an enclosed organ system, for its protection, and it is quite snugly fit. so there is not much space in the region around the nervous system.Now when there is something extra in this space, like fluid or mass, the space remains limited and the substance in the limited space increases. I hope you are with me in this so far.Now when the contents increase and the space remains limited the pressure will increase. Imagine a box full of water bottles. If you try to stuff in one more bottle, the possibility is that the added bottle will pressurize and possibly damage the existing bottles in the box. This is what is happening. The idea is to put a shunt. A shunt is a tube between the brain, and usually the stomach cavity. Now with this shunt in place, the fluid keeps draining and so the pressure does not increase.I would like to know the exact report of the mriI hope this helps you. Inform the reports mentioned above so i can be of help further. Best of luck.I have given you the answer to the maximum considering the information provided. The results of the tests could further enhance my answer to you.Please do understand that some details could be extracted from a detailed history and examination.Looking forward to your return query with the details asked so that I can help you further.(If the answer has helped you, please indicate this)"
},
{
"id": 189011,
"tgt": "Wisdom teeth grown half way. Repeated infections. Swollen gums, pain, pus, bleeding. Suggestions?",
"src": "Patient: Hello, My wisdom teeth have been grown in half way for years now. I have gotten minor infections before but two days ago, my gums got swollen, which is what normally happens. It has been a lot more painful and swollen. Yesterday I noticed it began to bleed and pus on its own. I got home from work and put some gauze around it. Today, I noticed that there id a very dark red spot were the blood and puss was coming out. I am making an appointment but is this something I should be really worried about? Doctor: hello and welcome to HCM forum,I would like to inform you that the swelling of the gums if from inability to maintain proper oral hygiene as the food gets impacted within the soft tissue which covers the wisdom tooth.I will advise you to see your dentist soon,until then perform warm saline rinses (3-4 times/day).you can take ibuprofen as a painkiller after meals.not more than 3 tablets in a day , every 8 hrs.do not worry you will be fine, let your dentist decide on the treatment.i hope i answered your query.i wish you good health.take care."
},
{
"id": 44900,
"tgt": "What should be the time gap after collecting the sperm and the IUI treatment ?",
"src": "Patient: we have done iui last month, it failed, I want to know what should be the time gap after collecting the sperm and the iui treatment, does the external atmosphere effect the quality of the sperm Doctor: Hi,thanks for query.Timining of collection sperm is not important.Even cases when husband is not available on particular days frozen sperm can also be used.The sample should processed in same premises only if possible.After collection of semen the sperm sepetation normally takes around 1-2 hours,after the IUI can be done.The husband should observe at least 3 days of abstinence before giving sample for good results. wishing you good health."
},
{
"id": 83957,
"tgt": "What are the side effects of neurabol tablet?",
"src": "Patient: Sir,I am doing gymming from the past 5years.I want to have a gud physique,so i am working hard on it.I have worked hard but not have been able to get the desired physique.Lately,my gym trainer asked me to take neurabol tablets(2mg).I just wanted to know the negative and positive impacts of this tablet.And should i reallly take them?I dont want to spoil myself.Please help me.. Doctor: Hi,Neurabol is an anabolic steroid that is now banned in many countries. These drugs on continuous use can cause acne,low sperm count, liver and kidney damage and breast development in males. Kindly avoid using the drug.Hope I have answered your question. Let me know if I can assist you further. Regards, Dr. Saranya Ramadoss, General and Family Physician"
},
{
"id": 224623,
"tgt": "On birth control. Missing periods. Pregnancy test negative. Nose started bleeding. Is it due to birth control?",
"src": "Patient: My girlfriend has been on birth control for the past 3 months. She hasn t had her period in 40+ days but has had 3 home pregnancy tests show up negative at the 10 days late, 20 days late, and 40 days late marks accordingly. She s had 2 random nosebleeds in the past 2 days but no signs of pregnancy. Could it be due to the birth control? Doctor: Hello. Thanks for writing to us. The random nose bleeds can be due to the hormonal pills also. But it is best to rule out the local causes like dry nose or infections after a proper examination.I hope this information has been both informative and helpful for you. Regards, Dr. Rakhi Tayal ,drrakhitayal@gmail.com"
},
{
"id": 211643,
"tgt": "Feeling things crawling in body, feels at night when body is relaxed. Reason?",
"src": "Patient: I have things crawling in my body, up and down my spin, back of my throat, in my nose, in my testicles and my anus!!! It feels like spiders and sometimes they are relentless, what are they????? Is it alL in my head..... It feels real and they are definitely getting stronger! Can you help or suggest a tonic............ They havE made my life a living hell for nearly Two years ....... I'm at my witts end, they only come out at night, when my body is relaxed and I sometimes don't get any sleep...... Can you help please? Doctor: Hi, it appears to be due to the indigestion, or gastro oesophageal reflux disease. The problem in the anus and testicles might be due to the worms in the intestines. I to my patient with such symptoms prescribe ofloxacin, omeprazole, domperidone and albendazole. Thank you."
},
{
"id": 2639,
"tgt": "Suggest precautions to be taken to get pregnant",
"src": "Patient: i had a miscarriage last nov.2011 and that was my first baby and after that iam trying to conceive from past 2 months but not successful and moreover since iam trying,my period flow is less than usual and my cycle decrease from 28 to 26.is this normal?? Doctor: Hi,I guess your concern is about conceiving and decrease in menses cycle, right?Coming straight to getting conceived. You have not shared the information about your age. Was it a late marriage?Women of age 18-40 years are of reproductive age, where chances of becoming pregnant are more. Women of above 50 years, chances of pregnancy are nil, and 42-45 years, its transitional zone, chances are not as high as reproductive age nor as low as above 50 years. I would like to focus on term fertile period. It starts from 10-16th day of menses cycle. Here sperm fertilises ovum and women get pregnant, chances of getting pregnant are high at this time, so its called fertile period. I suggest you and your husband to have sex/coitus during fertile period (scientific reason I have told above) for 3-4 months. If there is no improvement with your menses or above suggestion not working you have to visit nearby gynecologist for further help. If you are in hurry and your age in transition zone, then I strongly recommend you not to waste time and visit a nearby gynecologist or infertility specialist for further.Hope I have answered your query. Let me know if I can assist you further.Regards,Dr. Sudha Rani Panagar"
},
{
"id": 78177,
"tgt": "Suggest medication for chronic cough",
"src": "Patient: I have had several EGD's. no problems. This one a week ago caused more throat soreness and now I have a chronic cough and I can't get my GI doc to take me seriously....he says it is irritation and biopsies that have caused the irritation? I have never had this before? Doctor: Hi. I can understand your concern. It must be a viral infection try stem inhalation and warm salt water gargling. Also drink lots of fluids.Don't worry, you will be alright. Hope I have solved your query. Wish you good health. Thanks."
},
{
"id": 125748,
"tgt": "Can a hip fracture be confirmed without an X-ray report?",
"src": "Patient: I would like to know if it s even possible to break your knee, or some other part within the knee. Also, is there any way to confirm whether you have a hip fracture, without an x-ray. I have been diagnosed with moderately advanced stage of osmosis. I had osteopenia very early on. Since I had a partial hysterectomy, my OB put me on Fosamax. Several months after I started taking it, it caused internal bleeding in me, It was so bad they couldn t get a BP on me, Rushed to the ER by an EMT from the doctor s office, 5 pints of blood were pumped into me. So, couldn t take any of the pills or shots to counteract it or slow it down. Next kicker was breast cancer - really can t go on anything now, So, with all of the above, most of the info you probably didn t need to know, I fell last night in my garage. It is a painted cement floor, I was rushing through with a lot of grocery bags hanging very heavily on one arm, my flip flop slid on the cement and couldn t catch myself because the grocery bags were so heavy, their weight is was caused me to fall on my right side. I really slammed the side of my knee, which have been a problem for me since I was a teenager. Doctor: Hello, Consult an orthopaedic and plan for an MRI scan. Hope I have answered your query. Let me know if I can assist you further. Take care Regards, D. Shinas Hussain, General & Family Physician"
},
{
"id": 35588,
"tgt": "What are the symptoms of herpes simplex?",
"src": "Patient: Girlfriend came over with herpes simplex virus 1. Cold sore on lower left part of lip. Had Abreva covers, swore by them. Put it on to cover up the coldsore and I made out with her. Later on it fell off so I didn t want to kiss her anymore, but could help but to kiss her on the cheek. When I did, a few times she brushed up against my cheek and neck with the cold sore. What are the chances I have herpes simplex virus, and if I did contract the virus from the interaction, how long until I know whether or not I have it? How long would it take for a cold sore to appear, or this infamous tingle I hear about, etc. Doctor: Hello dear,Thank you for your contact to health care magic.I read and understand your concern.Herpes infection occurs only in a person having weak immune system. So if you have strong immune system than you never going to have herpes as such.Though she has bursted at neck and cheek blister you are exposed to the lips for surely but you mostly won't suffer with shingles.Please maintain good hygiene it can prevent future infection. You can take prophylactic acyclovir treatment under your doctors guidance.Please eat healthy diet like fruits, dry fruits and green leafy vegetable.I am Dr Arun Tank answering your concern.I will be happy to answer your further concern on bit.ly/DrArun.Thank you,Dr Arun TankInfectious diseases specialist,HCM."
},
{
"id": 83844,
"tgt": "What are the side effects of zapiz .25 and szetalo 10mg?",
"src": "Patient: Hi I am 29 year old male weight 92 today my doctor refer me two tablets zapiz.25 and s zetalo 10mg my symptoms is feeling heavyness and numbness in my left hand and left chest even I am a sinus paitence and my treatment is going on with ENT surgeon I have done brain MRI and thoyride free T3 free T4 and THS also ECG all my tests was clear. But I was still not satisfied with then I did sinus CT sacn which showed that my left side maxillary sinus is block Dr adviced will control with medicine no need surgery. I ask my doctor about my heaviness and numbness in left hand he said it is physiological complaint. Then I decided to meet a good physicastrist after knowing my symptoms he refer me those tablets mention above please adviced me is there any side effect with those tabs . Doctor: Hello, Nausea, dry mouth, trouble sleeping,difficulty in concentrating, constipation, tiredness, drowsiness, dizziness,delayed ejaculation and increased sweating may occur. If any of these effects persist or worsen, tell your doctor promptly. Hope I have answered your query. Let me know if I can assist you further. Take care Regards, Dr. AJEET SINGH"
},
{
"id": 176552,
"tgt": "Could painful eyes due to TV light, low fever, ear ache, red tonsils causing painful swallowing mean menagitis?",
"src": "Patient: Hello yea my son is 3 and woke up complaining of TV light hurt his eyes, temperature 38.5 and had ear hurting early today he also has red tonsils and hurts to swallow he has had Nurofen and lights on now and his still gas temp of 38.3 seems of but Offliously worried of menagitis coz of light so your advice would be great :) thank you. Doctor: Hi...you are right in guessing that TV light hurting could be photophobia which is a sign of meningitis. But it will be consistent and along with it there will be fever, incessant vomiting and altered sensorium, neck stiffness etc. It could as well be simple viral illness too. Usually rather than fever, what is more important is the activity of the child, in between 2 fever episodes on the same day. If the kid is active and playing around when there is no fever, it is probably viral illness.But I suggest you get him evaluated by your physician and then you can be rest assured.Regards - Dr. Sumanth"
},
{
"id": 54364,
"tgt": "Which tests would confirm whether hepatitis is cured or not?",
"src": "Patient: Hi there, im 31 years male, weighing,75, height5;11. 4 years back i had to under go a nazal surgery, pre-operation tests confirmed hcv, but after further tests doc decided to go ahead with surgery, latter i got six months liver treatment but didn,t go for post-treatment tests to make sure that whether therapy was successful or not, although i feel perfectly alright now but i want make sure whether im hcv free or not, plz guide me where to start ? Or what tests i should go for first? Doctor: Hi welcome to HCM....After infection with hepatitis c infection, chance of getting Chronic infection is very high....There is no particular antidote for hepatitis c ...if antiglobulin like antibody given at the time of exposure within 72 hour then it might help , otherwise it is your immunity that play role in clearing infection...Now six month had gone ...So to confirm its negativity perform ELISA or PCR like molecular method.....If it comes positive then regular follow up to gastroenterologist done....Meanwhile avoid alcohol drinking and take low fat diet....Take care.Dr.Parth"
},
{
"id": 118900,
"tgt": "Breast cancer treated, developed blood blister under eye. Are they sign of low platelets?",
"src": "Patient: My sister has just been treated for breast cancer ,& has finished her chemotherapy recently. She is now taking Tamoxifen ,but suddenly developed A large blood blister beneath her eye for no apparent reason. Her cancer doctor told her to contact her G.P whom she also told that she had been having stomach pain during her treatment,but thought that it was treatment induced. The GP is now giving her extra blood tests. A friend whose father died recently of cancer,said blood blisters are a sign of a problem with platelets(?) Should the cancer doctor see her immediately,rather than waiting 3 months& packing her off to her GP? Doctor: hithanks for your queryi can feel your concernslow platelet count usually manifest as bleeding from different sites, petechiae and bruising. I advise you to have her blood test (routein blood complete examination CBC ) and if platelet count is low then see her cancer doctorwishing her a speedy recoveryi am available for your helpregardsdr.imran"
},
{
"id": 116189,
"tgt": "What causes beta thalassemia trait?",
"src": "Patient: My brother was diagnosed with beta thalassemia trait. He is very tired all the time. Hemalogist said to see his primary care for that. Says it is different issue. have read that having this can make you feel tired He has had this all his life He is now 55. He is 5\"9' and weighs about 150 Doctor: Hi, dearI have gone through your question. I can understand your concern.Thalassemia trait is caused by heriditory factor. Defect in bete chain production. His mothe ror father have this defect and it is transffered to the child. He may have low hemoglobin but able to lead normal life. Hope I have answered your question, if you have any doubts then contact me at bit.ly/Drsanghvihardik, I will be happy to answer you.Thanks for using health care magic.Wish you a very good health."
},
{
"id": 65133,
"tgt": "What causes lump on my cervix?",
"src": "Patient: Hello. I just had a regular pap smear administered the other day and the nurse said she found a lump on my cervix. I am sure it is a cervical cyst because I am positive I had one once before. I have never had an abnormal pap smear in the past, but would the presence of a cyst during this exam result in an \"abnormal\" pap smear result? Doctor: Thanks for your question on Health Care Magic. I can understand your concern. Yes, lump in cervix is always abnormal. It does not mean that it is malignant lesion. Following are the possible causes for lump in cervix. 1. Cervical polyp 2. Cervical cyst 3. Granulomatous lesions (infectious). 4. Malignant nodule. Etc. The most common cause is cervical polyp. It is benign condition but can cause malignant transformation in latter part of life. So better to consult gynecologist and get done removal of cervical lump. Also send it for histopathological examination to rule out malignant lesion. Hope I have solved your query. I will be happy to help you further. Wish you good health. Thanks."
},
{
"id": 36650,
"tgt": "While undergoing chest infection treatment, will the scheduled BA surgery be postponed?",
"src": "Patient: Hi... I'm due for BA surgery next Friday.... I have come down with a severe chest infection. I'm on antibiotics and I feel it's clearing... I have fevers and everything really bad last Thursday and Friday, but I'm wondering if this clears before Friday would they still choose to postpone? Doctor: Thanks for your query at HCM!Yes it is likely that they may postpone the surgery for your breast augmentation.I suggest first get treated for your infection.It's good you are responding to antibiotics, so there is no need to change them.Monitor your fever constantly.You can get tested for blood culture & sensitivity(aerobic/anaerobic) if fever persists.Get you sputum tested for gram stain & culture & sensitivity(aerobic/anaerobic) if fever persists.Get your complete haemogram done with chest X-ray.Investigations will help to reach etiological diagnosis but most of such cases are due to viruses (Rhinovirus, influenza, parainfluenza, etc) which are self limiting.You will have to take antibiotics to prevent secondary infections, bronchodilators, cough suppresants, anti-inflammatory and analgaesics for which you can visit your general practitioner or a pulmonologist.1. Drink plenty of fluids.2. Get plenty of rest to help your body recover.3. If you smoke, stop.Take care!"
},
{
"id": 117655,
"tgt": "Is an MCV of 79 a problem in marriage?",
"src": "Patient: My MCV is 79.1 (and I know that it is below the range 80) I would really appreciate if you could help me what is the problem and for the marriage is there any problem with this result? 29 years old, 179 cm height, 103 Kg weight, no special medical problem, I'm giving blood every 4 or 5 months, I drink tea often (4-5 glasses each day) Doctor: hi, your mcv value is almost near normal. MCV is the mean corpuscle volume. it indicate the average size of RBCs. so low mcv means that you have microcytic rbcs. it is seen in iron deficiency anemia, thalessamia minor etc. but you said that you are donating blood every 4 to 6 months that means your hb level is also normal and you dont have any abnormality. so just be relaxed, dont worry.its not a problem. and it will not creat any problem after marriage.thanks for using health care magic."
},
{
"id": 20120,
"tgt": "What could fever post ASD surgery suggest?",
"src": "Patient: My sister son had a ASD surgery last week. He was in the hospital for days and after that they discharged him. After coming home he was ok for 3 days and after that he got fever. Now he is in the hospital for 2 days and still having fever. What could be the reason ? Doctor: Welcome and thank you for asking on HCM!I have gone through you question carefully. Postoperative fever ( usually within the first 2 weeks) is due to infection and needs to be treated very aggressively. Blood culture will need to be done.Hope you will find this answer helpful!Wishing all the best, Dr. D Sunil Reddy"
},
{
"id": 135462,
"tgt": "Suggest treatment for swollen and painful foot",
"src": "Patient: I have a Great dane who 2 days ago had a swollen foot,,,No puncture wounds no obvious wound. Thought maybe spider or bee sting. Soaked foot in warm water and Epsom salt, swelling went down but last night dog chewed a hudge hole on the top part of his foot. No tissue to suture together. On Antibiotics but wondering what I can do to get it to granulate in? Worried about infection?? Doctor: Hi Dear,Welcome to HCM.Understanding your concern. As per your query you have swollen and painful foot. Well there can be many reasons for symptoms you mention in query like venous insufficiency injury , organ failure like particularly in the heart, liver, or kidney , gout , diabetic foot or lymphedema . I would suggest you to take antibiotics as per prescriptions regularly , take vitamin supplement for quick recovery and keep the side clean and dry . Clean the site with betadine . If condition doesn't get better then consult doctor again for proper examination . Doctor may prescribe stronger antibiotics along with anti inflammatory . Hope your concern has been resolved.Get Well Soon.Best Wishes,Dr. Harry Maheshwari"
},
{
"id": 131462,
"tgt": "Suggest treatment for joint pain",
"src": "Patient: My mother is 62 years old and pure vegetarian.she also does not take milk.she suffers from muscle and joint pain.she also has spondylittis issue.she is an early riser ( wakes up 0400 hrs every day)and fasts on an average 3 days in a week.her diet is also very less.what can i do to help her with her strict regime and religious beliefs Doctor: In my opinion you mother needs calcium supplment i recommend osteocare tablets just after the main meal for 6 month also spondylitis pain improve with activities like walking and jogging you can also try NSAIDs like Alphinturn if pain increaseGood Luck"
},
{
"id": 213368,
"tgt": "Obsessive compulsive disorder, keeping house clean always, everything has a place, anxiety. Treatment?",
"src": "Patient: Hello, I am concered that I may have OCD or another obessive disorder. I have the need to clean and have a clean house all the time. Everything in my house has to have a spot and if something is misplaced I get really anxious and have to put it back in the correct spot immediently. my house is always clean spotless. I get anxious alot and randomly start moving things around. Doctor: Hello, Your symptoms do suggest a tendency for obsessions and compulsions, but to label it OCD you need to be evaluated by a psychiatrist. There are many other things which need to be ascertained to make this diagnosis. There are good medications available for treatment and you must visit the doctor for evaluation. They will talk to you in more detail, assess the level of anxiety in you and accordingly will prescribe you a treatment plan. Good luck."
},
{
"id": 24063,
"tgt": "Is it necessary to do CT angiogram with normal echo and stress test?",
"src": "Patient: Hello Sir, I am Nikhil Gandhi, age 49 years, Male from mumbai. My HDL chol. is 30.3 and LDL chol. is 110.3. My 2d echo is normal and stress test of 7.15mts. is also okay. my cardiologist has suggested to do CTangio of heart. Is it necessary? He has prescribed me Ecosprin AV 75mg. also. Pl. advise. Thank you. Doctor: Hello!Welcome and thank you for asking on HCM!I carefully passed through your question and would explain that it is important correlating your tests with your clinical symptomatology. If you have typical chest pain which raises high suspicions of coronary artery disease, or changes in your cardiac enzymes, or wellknown coronary risk factors (smoking, hypertension, diabetes, obesity, etc.) further tests like coronary angio CT scan or coronary angiography may be needed. Otherwise, considering your normal cardiac tests, no further tests are needed. Hope you will find this answer helpful!Kind regards, Dr. Iliri"
},
{
"id": 148187,
"tgt": "What is the treatment for painful lumps all over my body while suffering from ankylosing spondylitis?",
"src": "Patient: my husband has ankolising spondylitis diog 2000- crones disease diog 2013, just diagnoses with sciatica last Monday 2014 , on humira for AS, prednisone for latest bout of crones, pain control tramadol, morphine, hydrocodone . He is breaking out in painful lumps all over his body , knee, head, foot , neck upper leg , temp 38.5 , bp 167/91 . bp has been low last few days Doctor: Hi,Thank you for posting your query regarding your husband.It is unfortunate that your husband is suffering from multiple medical problems. He needs to be examined to find out the reason for multiple painful lumps. As he is on steroids, the most likely cause could be infections such as furunculosis.Please meet a dermatologist or an internal medicine specialist for confirmation of diagnosis.There is no need to worry, as it can be well treated with medications.I hope my answer helps. Please get back if you have any follow up queries or if you require any additional information.Wishing you good health,Dr Sudhir Kumar MD (Internal Medicine), DM (Neurology)Senior Consultant NeurologistApollo Hospitals, Hyderabad, IndiaClick on this link to ask me a DIRECT QUERY: http://bit.ly/Dr-Sudhir-kumarMy BLOG: http://bestneurodoctor.blogspot.in"
},
{
"id": 5257,
"tgt": "Taken duramin, late in period, want to conceive",
"src": "Patient: I really want to fall pregnant but I m talking duramine I am 10 days latwith my period but I m not pregnant . How long is duramine in my system after I stop taking it please Doctor: Hi,The half-life of the drug is about 25 hours. So, you can expect the drug to stay in your body for a day or two after you stop it. It is better to avoid pregnancy so long as you are on the drug. If you are regular with your periods, then you should test for pregnancy since you have a delayed period ( as you just said that you are not pregnant but did not give the reason). Hope your query is answered. Take care."
},
{
"id": 47147,
"tgt": "Suggest treatment for kidney stones",
"src": "Patient: hello. i was recently diagnosed with kidney stones. I was given drugs for it but i have a questionabout one particular one. It s called Ideos. Its composed of calcium carbonate and vitamin D. My question is should i be taking calcium supplements for kidney stones? Doctor: Hi, dearI have gone through your question. I can understand your concern. As such calcium supplements are not required for kidney stones. But if you have kidney disease or dysfunction then calcium supplements are required. Treatment of stone depends on size and location of stone. Stone less than 3mm need no treatment. Larger stone required surgery. Consult your doctor and take treatment accordingly. Hope I have answered your question, if you have doubt then I will be happy to answer. Thanks for using health care magic. Wish you a very good health."
},
{
"id": 79299,
"tgt": "What causes dull pain in left side of chest?",
"src": "Patient: I have a dull pain on the left side of my chest. It almost feels like it needs to be stretched. I am currently on asthma medication, and had a full cardiology workup (EKG and stress test) done about a year ago. Everything came back as normal. The pain does not get worse with exercise. What could it be? Doctor: Hi,Welcome to HCM,If your EKG and stress test is normal and if you still have dull ache than it can be either muscular or pleural pain. would be worth while to check chest X Ray and if that's normal then mostly it would be muscular."
},
{
"id": 102548,
"tgt": "Is there any long lasting side effect from benadryl?",
"src": "Patient: my husband and I take a Benadryl each night before going to bed. We both have some allergies but I seem to be able to sleep much better and it s the only med I take for allergies. my husband sometimes takes another allergy med in the morning - starts with a Z - can t remember the name of it. Are there long lasting side affects from this? or is it realtively safe to use it once before bedtime? thanks Doctor: Hello,It is my pleasure to answer your query,For any drug there will be a side effects.But advantage of the drug will be more when compared with the disadvantage.Similarly this drug too has some side effect on long term usage they may be Sedation, sleepiness, dizziness, disturbed coordination, Epigastric distress, anorexia, nausea, vomiting, thickening of bronchial secretions, tightness of chest and throat.These are some of the most common side effects with Benadryl.You can take this medicine on consultation with your treating doctor.Thank you.Hope you got the answer for your query."
},
{
"id": 37054,
"tgt": "What could cause rashes,swelling and itchiness on legs & hands?",
"src": "Patient: All of a sudden today I got a rash. My legs got itchy and red and blotchy. It went away shortly then came back later. Also my hands and feet got really red and almost swollen and itchy with hive like bumps. It went away also then came back and went away. What is going on?! Doctor: HIGreetings from Dr.Divakara.PThanks for posting your query. You seem to be having an allergic reaction to something. That something could be due to food, drinks , medications or cosmetics . Just check whether there was anything usual in your food or drink intake or are you taking any medications recently or changed your cosmetics recently ? Kindly let me know these details . You can use calamine lotions 2 times a day and if problem persists ask your doctor for a prescription for Anti-histaminic drugs , that will give you good relief. Ragards."
},
{
"id": 139971,
"tgt": "How can concussion after suffering injury be treated?",
"src": "Patient: Last night I fainted and apparently fell strait as a board and hit the back of my head pretty hard on some concrete. I have a little trouble remebering what happened within 5 minutes before fainting. I woke up shortly after and felt fine for the most part. Today I woke up with a headache and somewhat of a pain behind my eyes and have almost no appetite. im guessing I could have a concussion but I m more worried as to what could of caused the fainting? I had my heart checked by a cardiologist just a few months ago from a sports related colapse but they assured me everything was ok. I am just wondering if this is something I should worry about and go strait to the hospital or is it more of a concussion-like injury and needs to just be waited out a week or two? Doctor: Hello, I passed carefully through your question and would explain that it is necessary for performing further tests in order to investigate for the possible cause underlying fainting: - an ambulatory 24-48 hours ECG monitoring in order to investigate for possible cardiac arrhythmia - an EEG in order to exclude possible seizures - blood electrolytes - fasting glucose for possible hypoglycemia. The headache and pain behind the eyes are quite normal after the head trauma, and I am sure that your situation will improve in the next days. Hope I have answered your query. Let me know if I can assist you further. Take care Regards, Dr Ilir Sharka, Cardiologist"
},
{
"id": 213992,
"tgt": "How can i rid from shivering problem ?",
"src": "Patient: hello sir iam a student of 28. and iam suffering with shivering can any body help me? Doctor: Hello. It is not very clear to us from your description what exactly you mean by 'shivering'. If you have shivering because of any physical condition such as fever, you need to see a physician for diagnosis and treatment. Shivering could also mean tremors, muscle twitches or jerks. If so, you might need to consult a physician, neurologist or a psychiatrist depending upon the kind of tremors or other movements. Please let us know more about your condition in more detail so that you can get a better answer. It will also help if you could describe the duration of your problems. Best Regards Abhijeet Deshmukh, MD"
},
{
"id": 172943,
"tgt": "How to confirm if the baby is intolerant to cow's milk?",
"src": "Patient: My 23 month old has been gassy since switching to cow milk. She is teething and threw up curdled milk. Could she be intolerant to cows milk? Or would it have affected her right away after switching? Could teething cause the vomiting? There is no fever Doctor: HiWelcome to the HCM The symptoms in your child seems to be more of lactose intolerance rather than cow milk allergy. Lactose intolerance is triggered by the lactose sugar in cow\u2019s milk. In people with lactose intolerance, the digestive system can\u2019t fully digest this milk sugar, because it doesn\u2019t make enough of the lactase enzyme. So instead of being digested and absorbed, the lactose stays in the gut and feeds the gut bacteria, which release acids and gases that cause the symptoms of lactose intolerance. Sometimes even high carbohydrate diet may lead to similar symptoms.As most of the cases of lactose intolerance are temperory, I would recommend you to give your child a well balanced diet without milk and excess carbohydrates for the time being. You may gradually introduce milk after a week.Also, probiotics such as enterogermina will be useful.Hopefully this will be helpful. I will be happy to answer any further questions.Take care"
},
{
"id": 40693,
"tgt": "What does the following follicular study report suggest?",
"src": "Patient: My follicle size in 10th days 27 into 17.8 mm and it was decreasing on 12th day 25 into 16 with irregular walls on 15th day 24 into 13.6 mm with irregular walls and endometrium thickness is 10 mm if HCG injection is taken means if it is possible to conceive please answer me Doctor: Hi, I think there can be a chance to get pregnant. Take progesterone like susten or duphaston for 2 weeks. Get a pregnancy test done after that. If positive, consult your doctor. If negative, go for a baseline scan on day 2 of the period. You can try ovulation induction with timed intercourse for conceiving. Talk to your doctor regarding this. Hope it helps."
},
{
"id": 214878,
"tgt": "Suffered insect bites, dark spots on legs due to excessive scratching. Home remedies for treatment?",
"src": "Patient: hye doctor, here my leg has many dark acne because once i went to a forest the insect bite me then it was soo itchy so i scrap it then it became dark so itz hard to remove it n now doctor please help me to remove it coz my bufday is around the corner and i have to wear a gown please doctor help me???? i want it to be a natural steps i dont want to use any product Doctor: Hello, Thanks for your query. It is common to get itch and inflammation after any insect bite, afterwards your skin may show pigmentation or dark spot. Frequent scratching can result in further inflammation or infection so try to avoid it to prevent further damage.Some insect bites can cause skin allergies and irritations. You can use over the counter gels to reduce inflammation and irritation. Disinfectants and ointments for sever itching can be used. Vitamin E Oil can be applied over inflamed area. Rub the flesh of tomato, lemon, or papaya on the bite marks twice a day until the dark spots disappear. For prevention always use an insect repellent when you are at risk of insect bite. Hope it helps. Take Care!"
},
{
"id": 88870,
"tgt": "What causes severe abdominal pain?",
"src": "Patient: I have been having strong abdominal pains for over a week now. Last night I went to the ER, they did blood & urine tests which all came back normal. Honestly, I feel pregnant. I've been feeling really strong movements in my stomach most of the day, and I'm not sure if I'm late on my period yet. I haven't gained any weight according to the scale, but my stomach looks like it got bigger too. I've taken 4 pregnancy tests on my own, that came back negative. And had negative results at the ER. Could I still be pregnant? If not, what is this caused from? Doctor: Hi. This shows that you are not really pregnant. This may be due to a condition called - Pseudocyesis : Means there is no actual pregnancy but the patient gets all the signs and symptoms. This is confirmed by a negative ultrasonography. He patient who has the extreme desire of having Motherhood again develops this sort of a problem. No one really knows the reason and the exact mechanism by which this happens. Consult your Senior Gynecologist who has seen such more cases for counselling and further management."
},
{
"id": 2531,
"tgt": "Why is pregnancy not happening after miscarriages?",
"src": "Patient: i suffered a misscarriage back last june and since then me and my husand have been trying for a baby since then. I didnt have a period for two months last november and december but iv had one since then on 24th december.so im wondering what is wrong my husband is fine . I know i can concieve as i was pregnant last year Doctor: Yes you can conceive beause you had miscarriage do thyroid profile also do torch test and what is your blood group?"
},
{
"id": 94298,
"tgt": "Lower abdominal pain. Done urine test. History of colorectal CA. Done chemotherapy. Meaning?",
"src": "Patient: Hi! My mother has on and off lower abdominal pain . She is 77 years old. Her urine test shows an RBC result of 10-12/hpf. What does this mean? She s taking maintenance medicines for her heart (vastarel and concore) and other food supplements. She also had a history of colorectal CA in 2005, had 6 sessions of chemotherapy . Follow up whole abdomen CT Scan after one year showed that she s clear. Doctor: hello chersmateo. Welcome to HCM. its hematuria. the causes may be many such as: kidney damage, tumors which erode the urinary tract anywhere along its length, kidney trauma, urinary stones, renal infarcts, acute tubular necrosis, upper and lower urinary tract infections, nephrotoxins, and physical stress. Also this is usually asscoiated with urinary infection, such pain, burning sensation while urinating and often need to urinate are typical signs of it. So she needt to do urine analysis and uro ultrasound. Till that she can dring some of teas made for ureteral stones and lot of fluids. Hope I have answered your query. If you have any further questions I will be happy to help"
},
{
"id": 22193,
"tgt": "Should i be concerned about the rise in blood pressure up to 198/63?",
"src": "Patient: My blood pressure went up to 198/83 last night and I didn't feel well. I took my evening meds and it went down to 184/63, Should I have called my doctor? I am 57 years old, 5'1\", 250 lbs and have had HBP for approx 16 years. I was in the hospital last year when it reached 191 and I felt extremely sick. Doctor: yes high BP is a cause for concern and so high is definitely be treated. if u are on drugs just increase or change or add other one.check for the cause if possible. usual cause for Young HBP are related with kidney like renal artery stenosis,CKD or Coarctation of aorta ,Hyperthyroidism and many more."
},
{
"id": 129491,
"tgt": "What is the treatment for pain in shoulder and lower back?",
"src": "Patient: Everytime I m laying down I can t find the right position to sleep because my shoulders are always hurting they feel tired / weak. It s the back of the shoulder the right and left side of the spine. Also if I lay on my back my lower back begins to hurt like pain. And when I bend down to pick something up when I come back up the pain comes back in my lower back. Doctor: Hi...Your pain in the shoulder and back could be due to spasm in the muscles and soreness...It is common to get soreness if your activity level increased suddenly or due to bad posture..I suggest you to..Ice up your low back and shoulder frequently...stretch your shoulder.. low back... upper back... frequently... to ease your pain off..Engage in strength and stability training for your upper and lower back..Like retraction exercises for your upper back and extension based exercises for your low back...core retraining is necessary...flexibility exercises in hamstring gluteus... quads...to be done regularly .I suggest you to sleep with neck support and a pillow under your knee....Hope this is helpful for you...Kindly revert back in case you need any further help in this regard..."
},
{
"id": 52699,
"tgt": "Suggest treatment for enlarged spleen",
"src": "Patient: I was told last friday that my spleen is enlarged. Since friday, it has become very painful. It feel a lot of pressure on my left side under my ribs. My Dr. (pulmonary) wants to see me in 3 weeks to re-evaluate. They found the spleen after a ct scan was taken of my lungs. Lungs are normal. Doctor: Hi and welcome to Healthcaremagic. Thank you for your query. I am Dr. Rommstein, I understand your concerns and I will try to help you as much as I can.Enlarged spleen is called splenomegaly and there are various causes possibly leading to this. In first case these are infections such as mononucleosis Parasitic infections, such as toxoplasmosis or Bacterial infections. It means that any infection in your body may lead to spleen enlargement. Second most common causes include autoimmune diseases such as hrombocytopenia. If this is ruled out by specific tests then some other causes may be considered and some are more or less serious such as Leukemia Lymphoma, or someInflammatory diseases such as sarcoidosis, lupus, and rheumatoid arthritis. In every case, infectious causes are treated with antibiotics and other causes may require surgical removal of spleen.I hope I have answered you query. If you have any further questions you can contact us in every time.Kindly regards. Wish you a good health."
},
{
"id": 217679,
"tgt": "What is causing pain in neck?",
"src": "Patient: neck painfemale,23. my neck has been hurting for the past three weeks . I can't hardly move it and I have to turn myself all around to see things. I have tried every pain medicine over the counter, I have also tried ice and heat. I bought a memory foam pillow and that doesn't seem to help either. I have rheumatoid arthritis and I wasn't sure if it was that causing it. I also have hypothyroidism. so didn't know if there is anything else for me to do? Doctor: This type of pain usually is a case of cervical spondylitis. You may get in touch with a physio & learn to do cervical exercises. In the beginning you may take a medicine combination of pain killer & muscle relaxant"
},
{
"id": 219421,
"tgt": "What is the treatment for cystic ovaries as shown on sonography during pregnancy?",
"src": "Patient: My wife was pregnent, in her sonoghraphy report following pint is came , pl inform me the meaning of this and waht is the inmact what to be to do for this any impact to baby. There is cystic mass with smooth walls measuring 6.7X5.6 cms. with thick hyperrhoic moving echoes within, seen adjiacent to right ovary, could be due to haemorrhagic ovarian cyst or cystic adnexal mass. regards, Rakesh YYYY@YYYY Doctor: Hi.The treatment currently is nothing, allow pregnancy to progress as it is and only once she is done with pregnancy, we have to observe her menstrual cycle. Only if there is an irregularity will she require any form of medical intervention, if her cycles are regular, then no need to treat it at all.Best wishes."
},
{
"id": 64215,
"tgt": "Suggest cause and remedy for lump under the right side jaw",
"src": "Patient: hi im 16 years old about 5\"7 and weigh around 150. i know i shouldnt, but i dipped some tabacco 2 days ago and a white sore formed in between my gum and teeth..what im more worried about though is that today i noticed an average size lump underneath my right side jaw. do you know what this could be? Doctor: Hi,Dear don't worry.I understood yur health concerns.-You suffer from acute sub mandibular lymphadenitis due to oral infected tobacco ulcer you had.-Dont worry for that lump to be Cancerous lump, as its infective lump.Tratment- I would treat it as follows -- tb NSAID x5 days-th antibiotics from Er doctor or GP.x5 days.- bland liquid diet for 1 WK- no hot tea or drinks.- Zytee gel locally 2 times a day. This would resolve the lump and oral ulcers in 1.5 wks time.Hope this attends your query.-Wellcome to HCM for any more help you need."
},
{
"id": 93777,
"tgt": "High abdominal pains. Took nexium. Immunologist suggests that my liver enzymes are repeatedly coming back high. What are the findings and required treatment?",
"src": "Patient: I am a 40 year old female that has been experiencing high abdominal pains regularly for the past 14 years...since the birth of my first child. It began occasionally where the pains would present early in the morning around 4 and last for approximately 5-10 minutes. I would have them for a week or so and then they would go away for a few weeks. After several times I went to my doctor and was prescribed nexium. This did not resolve the issue. After a few years of the pains occuring more often I had an ultrasound and my gall bladder was removed. This pains intensified and began to occur much more often. I've been told IBS, and sluggish colon and so on. My concern is that recently...and discovered by an allergist and now an immunologist that my liver enzymes are repeatedly coming back high...could the pain have been from the liver all this time? Also, the pain begins as a hunger pain and rapidly turns into intense pain that begins in the high front area but ends in the back.....so what is happening to the body when it experiences a hunger pain? And the pain only occurs now at 5-6 in the morning and again by 9-10 if I dont eat a breakfast. Doctor: Hi welcome to Health care magic forum. Thanks for choosing H.C.M.Forum. You have got abdominal pain, since 14 years starting at 4 am lasts for 10 minutes. Occuring for few days and a gap of some days. Started after the birth of your 1 st child. You have used nexium, got gall bladder removed, but pain is increasing and of hunger with more frequency. It also thought as I.B.S.,and increased liver enzymes. At first your pain could be due to reflux of urine into kidney due to retension, and relieved by voiding urine. Liver enzymes increase, hunger type of pains, may be due secondary to gallbladder removal. (seen in some people only.) I advise you to avoid spices, junk foods, and oily foods to avoid irritation to the stomach, and kidney. Wishing for a quick and complete recovery. Best wishes."
},
{
"id": 204745,
"tgt": "Is Lamotrigine the right medicine for depression?",
"src": "Patient: I am taking iamotrigine.200mg. Dr said I am bipolar. At the age of 72 my family tell me no way maybe old they feel that way only thing as I am getting old I am much more forgetful tried and no interested in anything all my life I took drugs like or pro-active went to all kinds of Dr found me to have a site and depressed do I need this drug or should I talk to my Dr more. Sorry for spelling Doctor: Hello! Welcome to HealthcareMagic! In my opinion, it is necessary to do cognitive testing. Check B12 levels, Foliate, thyroid and make sure lipid tests are normal. Continue Lamotrigine at this dose. Hope this clarifies your concerns. Thank you for the query. Take care Regards, Dr. Gayathri, Psychiatrist"
},
{
"id": 119128,
"tgt": "History of TTP. Serum light chain test normal. B2 microglobulin normal. Likelihood of turning into MM?",
"src": "Patient: Hi and thank you. I m a 44 year old man who had TTP in May of 2007. I recovered from this (thanks G-d) after many, many months of plasmapherisis, rituximab and predinsone. I ve not had a relapse. In October of 2012, my SPEP showed a very, very faint monoclonal protein which is MGUS. We repeated the SPEP on March 1st and it was still labled very, very faint monoclonal protein . We did a free serum light chain test that showed my kappa and Lambda in normal range as well as the kappa/lambda ratio . My B2-Microglobulin was also normal 1,949 . My calcium is normal 9.7, CBC, liver and kidney functions are all perfectly settled in the normal ranges. What s the likelihood in your opinion of this turning into MM or someother type of plasma cell dyscricia? Thanks so much. Doctor: Thanks for query Yes we can predict your 10 year risk of MM with reasonable certainty. Of the tests you got done only free serum light chain test will be of some value in prediction.Depending on your country of residence and availability of tests and your own insurance/ financial status we can devise some tests. I would personally recommend flow cytometry for activated plasma cells (aPL) but it is available in tertiary institutes only. You can contact me for detailed discussion Hope you find it helpful"
},
{
"id": 113374,
"tgt": "Back pain, sciatica. Taking ayurvedic medication, coughing, shivering. Suggestions?",
"src": "Patient: Good evening Dr. I am xxxxx, i am suffering back pain. One ayurvedic Doctor told me that this is sciatica problem. what is dis ? and what home remedies or any simple medication. and also i am suffering cough problem and shivering after taking these medicine ( for backpain and cough ) what can i do? pls. give me some advise. Thank you Doctor: Hi ! Thanks for asking! If you lay down on your back and raise your leg, if you feel pain this could be sciatica, it is related to sciatic nerve. if you rest more by laying down will relieve. For permanent treatment do consult orthopedics. Now for cough take oral antibiotics and cough syrup, if shivering due to fever take paracetamol.Take care !"
},
{
"id": 210430,
"tgt": "Does Xanax used for anxiety damages the brain and Ecstasy abuse?",
"src": "Patient: Well to stay on the topic of xanex, or any benzo to to help with my drug induced anxiety. I am afraid I have damaged my brain from heavy LSD and ecstasy abuse. What I can't wrap my head around is any time I tried a stimulant I had a normal reaction, but after a few times my anxiety would be so severe that I could walk out if my house and wouldn't recognize the environment. Where to go, I appeared as if I was mentally challenged. Now add 5 mg to using meth I could have a political debate with you. Otherwise I would hide in my room only hearing my heart pounding and whole body shaking, Lastly, I would forget anything I recently learned after a binge, till I downed boos +benzos. Any explanation? And working on healing my brain. Doctor: Hi,You seem to have developed an anxiety disorder which tends to persist even after you have given up Ecstasy and LSD. Both alcohol and benzodiazepines act on GABA receptors which have a role in anxiety and so make you feel better initially by cutting down anxiety. But when the blood levels fall, anxiety returns. The best treatment for you would be to start a SSRI antidepressant like Paxil or Zoloft or any similar medicine. It will cure the anxiety without the risk of becoming dependent again.Best wishes."
},
{
"id": 67486,
"tgt": "What causes burning sensation and lump below ganglion cyst scar?",
"src": "Patient: Hi I have had a ganglion cyst removed this spring and where the scar is now is really hurting and there is a lump starting to form under the incision. Not only is there a lump but I have a burning sensation under the skin in this spots as well. Should I get this looked at? Doctor: Hi,Recurrence of ganglion is common so there might be having re appearance of ganglion.Consult your doctor and get examined.Ok and take care."
},
{
"id": 28742,
"tgt": "What causes chronic cough while suffering from laryngitis?",
"src": "Patient: My husband has had CHF since 2000. His heart is working at 25% and he also has COPD.He recently learned that his heart is larger than past scans have shown. He also has a damaged nerve that has caused laryngitis. He has a horrible cough that gets worse when he lays down. Could this be caused by his enlarged heart? He has had a pulmonary function test and found that it has not gotten worse since the last test over a year ago. Doctor: Hello and Welcome to \u2018Ask A Doctor\u2019 service. I have reviewed your query and here is my advice. Yeah, surely that symptom you describe is called orthopnea it occurs due to in lying down pre-load is increased and heart can't pump it out and that lead to pulmonary plethora which irritate bronchi and cause cough. Cardiologist opinion should be taken. Hope I have answered your query. Let me know if I can assist you further."
},
{
"id": 82180,
"tgt": "What does the x-ray report mean?",
"src": "Patient: Bronchovascular accentutation is seem bilaterally..... what does it mean???? In today s X-ray reports of my father which says 2 points namely: 1. Bronchovascular accentutation is seem bilaterally 2. Pleural thickening is seem at apices Plz help & advise us on the above mentioned points Doctor: Thanks gor your quetion on HCM.In my opinion no need to worry with this x ray report.As accentuated bronchovascular markings on chest x ray in lower zone is normal finding.Blood follows the gravity and so more bloodsupply in lower part of lungs than upper part. So lower part appears more white and accentuated. Blood vessels with bronchi thus becomes more prominent in lower part and termed as accentuated bronchovascular markings.About uper lobe pleural thickening, it is seen in old age and in old TB or pneumonia lesion.Since the pleura is thickened it suggest old fibrotic lesion.So no ned to worry about this x ray report."
},
{
"id": 135627,
"tgt": "Which among white potato and yogurt can cause osteoarthritis symptoms?",
"src": "Patient: I have OA. I have never had what s referred to as a flare up until the past couple days. I limit nightshade plants but enjoy a plain white potato about 5 times a week....consistently. I also eat yogurt every day. Are either of these the culprit Thanks Doctor: Hi,Welcome to healthcare magic. After going through your query I think You are suffering from osteoarthritis. Osteoarthritis is not related to eating of yogurt and potato . It is a degenerative disease related to aging and somewhat to heredity.You can discuss with your treating doctor.I think your query answered.Welcome to any follow up query."
},
{
"id": 13733,
"tgt": "Are red rashes a side effect of niaspan?",
"src": "Patient: I have a red rash on my chest just below my neck. It is bright red, kind of like when I use to take Niaspan and would have some red flushing. It started about three weeks ago. They changed my cholesterol a few months ago to Zocor. Could I be having a reaction to the meds. I had been taking Vytorin so I don t think it s the Zocor but I don t know. Doctor: Hi, I think the rash is a side effect of medication an allergic reaction to Zocor. Maybe, you should stop using it or change for another brand. Take any antihistamines such Benadryl or Cetirizine for rash and itching. Hope I have answered your query. Let me know if I can assist you further."
},
{
"id": 215244,
"tgt": "What causes pain all over the body and tiredness and restlessness?",
"src": "Patient: Hi I m a 32 year old male. I ve been feeling lots of pain lately all over my body. I ve been feeling very tired and restless. It s hard for me to get off the bed and I feel really stiff all the time. I have a 6.8mm herniated disk on my lower back. I also have arthritis on my back. I popped my fingers the other day and they stayed hurting for awhile all the joints on my fingers hurt my knees and lots of joints. My arms and legs fall asleep a lot or get numb. I get hot flashes all the time I feel depressed and I often get agitated really fast do you know what might be wrong with me Doctor: Hello, The symptoms are typical of fibromyalgia or chronic fatigue syndrome. If symptoms persist, it is better to consult a physician and get evaluated. Opiod analgesics like tramadol or hydrocodone will be useful. Hope I have answered your query. Let me know if I can assist you further. Take care Regards, Dr Shinas Hussain, General & Family Physician"
},
{
"id": 20183,
"tgt": "What causes dizzy spells while on BP medication?",
"src": "Patient: Hello im a sixty year old male on medication for high blood pressure and high colesterol both are under control and ive been on the same dose for 2 years my problem is for the last 5 days i have been having dizzy spells getting out of bed and getting out of a chair and everything starts spinning round and i have to sit down for a few minutes what do you think it can be Doctor: Hello!Welcome on HCM!I passed carefully through your question and would explain that your symptoms could be related to an inner ear disorder or peripheral vertigo. For this reason, I would recommend consulting with an ENT specialist and performing labyrinthine tests. Further tests may be needed ( a cervical spine X ray study for cervicarthrosis, a complete blood count for anemia, PCR and ESR for inflammation). Meanwhile, I recommend having some rest and taking plenty of water. A close monitoring of your blood pressure is also necessary to exclude low blood pressure or orthostatic hypotension. Hope you will find this answer helpful!Wishing all the best, Dr. Iliri"
},
{
"id": 206013,
"tgt": "Suggest treatment for eye irritation and tinnitus",
"src": "Patient: Hi my doctor seems to blame everythink on anxiety but for the last 3 months I have struggled with a Lot of problems and they don t seem to be bothered I ve had a headache and eye irritation and light sensitive allso I can see a number of black spots which are very annoying my eyes just feel as if they are burning also I have tinnitus in my ears but for the last 5 years now I have struggled to breath which they have never found an answer for which they also blame on anxiety. I am 22 years old but feel old misserable and im very depressed as I ve not been able to enjoy myself for years just like my friends. Doctor: DearWe understand your concernsI went through your details. I suggest you not to worry much. Please understand, If your doctor opined that your symptoms are due to anxiety, plese try to accept. If you do not believe your current doctor, please consult another one after you satisfy about the credentials. Then act according to his advises. Do understand, anxiety is known to produce weird symptoms which has no physiological reason. Those symptoms tend to be just obsession. Anxiety is a mental disorder. Researches proves that medicines alone cannot cure anxiety disorder. Along with medicine you should practice psychotherapy techniques to streamline your life style and meditation and yoga techniques to calm your mind, body, streamline your metabolism and thinking style. Please consult a psychologist.If you still need my help, please describe the whole problem in detail and post a direct question to me. I shall definitely help you with psychotherapy techniques to over come your problems.Hope this answers your query. Available for further clarifications.Good luck."
},
{
"id": 154754,
"tgt": "What does this ultrasound report and CAT scan indicate?",
"src": "Patient: I am wondering if I need to contact an Oncologist for a hypoechoic lesion (11x6mm anterior mid body) from an ultrasound test. Done 5/30/14 I was sent for a cat scan which showed low attenuation lesion in the pancreas measuring 7mm. Done 6/11/14. Thank you. Joann Doctor: Hi,Thanks for writing in.A hypoechoic lesion in the mid part of pancreas measuring 11 x 6 mm is usually a fluid filled structure. CT scan is required in such cases and you have got it done. The CT scan shows a 7 mm low attenuation lesion in the same place. A low attenuation lesion again is most likely to be a cyst. It is important to know if this lesion is showing enhancement. There is also a technical parameter called HU value in CT scan. If the HU value is less than 20 then it is confirmed to be a fluid containing cystic structure. It is important to have at least one consultation with the oncologist and discuss the reports in detail. Repeat ultrasound scan might be required in 6 months. This is to make certain that it is a benign cystic lesion and not a serious concern. Please do not worry."
},
{
"id": 164368,
"tgt": "What do these ESR and WBC test results indicate after giving antibiotics?",
"src": "Patient: Hello.I toke my 1 year daughter to the doctor today because she has an abses under her arm which she had it in different places in her body for the last 4 months but this time it came with a fevor. The doctor give her an antibiotic and request for CBC and ESR test.WBC is 17.3. And ESR us 20.What do you recommend. And we just start the antibiotics today Doctor: Hi, welcome to HCM. Can understand your concerns. This finding is suggestive of infection. Based on symptoms, antibiotics needs to be started. Take care."
},
{
"id": 158303,
"tgt": "Had a bone marrow done. Cannot have a heart transplant due to numerous stroke. What are the names of cancer in bone marrow?",
"src": "Patient: what are the 2 names of cancer you find in the bonemarrow my husband had a bone marrow done 10 years ago and his heart dr at shands hospital said it could take 10 years to show my husband is already terminal with his heart he has a 1/4 of his heart left in the front is not a can not have a transplant due to numerous stroke to theback of his neck Doctor: Hello and welcome to HCM,I am sorry but your query is not very clear.I will try to answer to bast of my understanding of your problem.There are a number of cancers which can metastatize to the bone marrow.Lung cancer, and prostate cancer are the common cancers in men which metastasizes to bone marrow.Breast cancer and lung cancer are the commonest among the females.Did your husband undergo bone marrow transplantation?Bone marrow transplantation is done for primary bone cancers like multiple myeloma, and leukemias.The most dreaded complication of bone marrow transplantation is rejection which can be hyperacute, acute or chronic.Chronic rejection occurs after many years of transplantation and can affect many organs of the body.You have mentioned that the functional capacity of the heart is left toonly one fourth.This implies that there is some disease process which is affecting the functioning of the heart.You need to consult a cardiologist for the knowing the disease affecting the heart and its management.Cardiac compromise is a contraindication to many surgeries.A good cardiovascular functioning is required to withstand any surgery.Patient can go into caediac failure if the cardiac functions are compromised.Thanks and take careDr Shailja P Wahal"
},
{
"id": 201910,
"tgt": "What does the ultrasound regarding the scrotum indicates and whats the treatment needed?",
"src": "Patient: im age-34, height-165 cm , weight-65 kg male and an ultrasound scan (doppler) report of my scrotum says few dilated (2.3 mms) anechoic serpiginous structers noted in right spermatic cord showing colour flow indicative of reflux on Valsalva maneouvre suggestive of varicoceles. what treatments can i do? Doctor: Hi,Thanks for writing in.A varicocele is an enlargement of the veins within the scrotum. Most varicoceles develop over time. Fortunately, most varicoceles are easy to diagnose and many don't need treatment. It need to be treated surgically only if your varicocele causes pain, testicular atrophy or infertility, or if you are considering assisted reproductive techniques, you may want to undergo varicocele repair."
},
{
"id": 60061,
"tgt": "Removal of gall bladder. Can the wound dressings be changed?",
"src": "Patient: hi, I had my gall bladder removed on tues 26 june and was discharged on thurs 28 june. I did not received any advice on the removal or changing of my wound dressings. I have read that these need to be changed/removed afte 48 hours. I have my post op appointment on thurs 5th july but do not want to wait until then. Please can you advise if i need to change them or leave them. They were waterproof - was the only advice I received. Kind regards Doctor: Hi Sharon, July 5th is a long way to go. Please go to the nearest physician or nurse practitioner and get it dressed. Though it is waterproof, it may not stay till 5th July. It is also not advisable to leave it undressed for long time. Hope this helps. Get well soon. Regards,"
},
{
"id": 112771,
"tgt": "Dizziness, light headache, lower back pains, tiredness, burning sensation/tingling left arm, ear ache. Test reports due. Recommendations?",
"src": "Patient: I m experience dizziness, light headed, lower back pains, tiredness, burning sensation in left arm along with tingling in my left hand, also the other day I had gotten a earache that lasted for an hour. I am going to have blood work done and a mri on the 31st been in out of the er due to this I was told that it can be numerous of things this is a off and on thing that has been happening for the last week Doctor: CHANCES OF ACUTE SINUSITIS OVER CHRONIC SINUSITISEARS ARE AFFECTED LIKE TIS IN SINUS ONDITIONS AND ALL OTHER SYMPTOMPS ARE EXPLIANED ON THIS DISEASES YOU MIGHT HAVE OFF AND ON COLDS AND ALLERGIES LEADING TO THISSTART METROGYL 200 MG BD FOR 10 DAYS TO CURE SINUSES FASTANTI ALLERGICS MONTAIR FX BD FOR 2 WKAPLY NEOMYCI H EYE OINTMENT IN NOSE BD'SEA WATER 2 DROPS AT NIGHT EACH NOSE TO CLEAR BACK F THROAT AND EARSDICLO SOS AS PAIL KILLER OF EAR AND ARMSDIZINESS AND NUMBNESS WILL GO AS SINUS INFECTION DECREASES"
},
{
"id": 215053,
"tgt": "Should i take the rabies vaccine if i got bitten by a immunized dog ?",
"src": "Patient: last 3 days, my dog s teeth scraped the skin of my foot for about 1-2 inches and it bled. i don t know if my dog is vaccinated but he took many injections and a 2-week antibiotic for some reasons. the veterinarian said that my dog is clean, should i worry? and this happened 3 days ago. i need help. Doctor: hi u need to take anti rabies vaccine as early as possible. because u dont know status of ur dog antibiotic has no role in this as its viral infection. u must take anti rabies"
},
{
"id": 43227,
"tgt": "Trying to conceive. Menstrual cycle getting shorter. Am I not fertile?",
"src": "Patient: I am a 25 year old female. I am concerned about my cycle. They are getting shorter and shorter. I have never been pregnant and am worried I am not fertile. I went to a clinlc about 2 years ago, they checked everything out and swabbed me and the Doc said it looked normal. Im thinking about going to a Planned Parenthood to have another look. Ive been trying to concieve for about 3 years now. My period lasts about 48 hours, it use to go for about 3-6 days. Is there anything I should be concerned about? Thank You! Doctor: Hi,Thanks for your query. I read your query and I understand your concerns. Following is my reply:1) It is not related to decreased fertility.2) Once in a while it is normal to happen this way and there is nothing to worry.3) Please get following tests done on day 2 of your cycle: FSH, LH, AMH, Antral follicle count scan. These will reveal ovarian reserve and allay your fears.I hope I answered your query. Let me know if you have any followup queries.Regards,Dr. Mahesh KoregolIVF & Infertility Specialist"
},
{
"id": 217752,
"tgt": "Suggest treatment for pain under the shoulder blade",
"src": "Patient: I was diagnosed with thoracic outlet syndrome and scholiosis a year ago. I went through physical thereapy and felt better afterwards. I continue to stretch my back neck and legs almost daily. Recently my back under my left shoulder blade has become very tight and sore. My back and neck feel exhausted like they never get a break. I cant move my head forward without a lot of pain under my shoulder blade. Im a 30 year old male, a little overweight, but active: I walk my dog daily and play drums often. Is there anything I can do to keep this from getting worse? Doctor: JYou must continue your physiotherapy and on case it does not get relieved add a regular pain killer. If it still doesn work you may need nerve blocks for which you shall visit a pain specialist in your area."
},
{
"id": 109518,
"tgt": "What causes pain and inflammation in the lower back?",
"src": "Patient: I have severe burning and pain to the lower back, with numbness and pins abd needles running down my left leg. When sitting or standing I experience excrutiating pain and a burning sensation to both thighs. The pain radiates down my leg causing cramping in the calf muscles and three toes. At times I experience this on my right side as well. MRI results : shows L4 disc buldge with a super imposed right paracentral moderat sized disc protrusion, causing moderate central canal stenosis and significant narrowing of the right lateral recess. With compression of the right L4 nerve root in the lateral recess. There is mild facet osteoarthritis and thickening of the ligamaentum flavum. Also moderate narrowing of the right neural foramina and moderate to severe narrowing of the left neural foramina at L4/L5. Doctor: Hello, Thanks for your query.** It seems you are suffering from degenerative disc disease & it occurs due to repetitive overload or stress to the disc & it increases the risk of disc herniation & spinal canal stenosis.The main treatment of such pain is bed rest along with pain killers and muscle relaxants. Muscle relaxants can help with your symptoms if used in appropriate dosage in combination with a potent analgesic.You can get the appropriate drugs prescribed from your Orthopedician after examination.Meanwhile you can follow these measures:- If the pain is severe, you need bed rest till the pain resolves. - Get some analgesics prescribed and apply analgesic spray or ointments.- While resting, keep a pillow under your knees if it doesn't bother you.- Avoid lifting heavy objects.I do hope that you have found something helpful and I will be glad to answer any further query.Take care"
},
{
"id": 126851,
"tgt": "What causes numbness in the toes?",
"src": "Patient: Hello, My name is Melissa and I am experiencing numbness in 3 of my toes on one foot and whats weird is that I actually have feeling on the top of my toes. Not sure if this is an issue regarding shoes or the way I am walking. I have pink color in my toes. Not like it is turning color or anything like that. I am a chef and spend all day on feet running around. I am wearing Dansko shoes but my toe numbness is more irritating than anything. I have had this issue now for a a few months. Wearing heals is out of the question because part of my foot becomes numb if I wear them too long. Oh well, don t really need to wear heals anyways. Not sure if this is a circulation issue or something else. Doctor: Hi, It might be due to neuropathic causes like uncontrolled diabetes. You can consult a neurologist and get evaluated. Hope I have answered your query. Let me know if I can assist you further. Regards, Dr. Shinas Hussain"
},
{
"id": 185328,
"tgt": "What causes bleeding in the gums and raised lump on the tongue?",
"src": "Patient: My mother was brushing her tongue a couple of days ago and it began to bleed. Upon closer inspection, she noticed a raised red patch on the center of her tongue that looks like a piece of dough. She says it doesnt hurt and has no effect on her eating. What could this be? Doctor: Hello, thank you for consulting with healthcaremagic. This patch can be just a fibroma which might have occured because of trauma from something, or it can be just a tastebud of tongue.Better once you visit a good dentist and get the patch properly examined, as right not there is no pain in it, but it might create any problem in future.Hope it will help you."
},
{
"id": 37286,
"tgt": "Suggest remedy for itchy, burning, swollen area between toes with oily secretion",
"src": "Patient: i have an oily secretion from between my toes, it burns and there is swelling rubbed raw etc this happens about once a year i treat with a cortisone like cream some perhaps at about 2% prescribed by may doctor.This time while treating my feet the itching spread to the top of the foot and the skin looks dried out wavy and wrinkled with red spots this is new and i am concerned any remedy doc? Doctor: Hi,It seems that you might be having Scabies infection in between toes and now there is secondary bacterial infection.Clean the part with antiseptic lotion and apply antibiotic cream.If require go for one antibiotic medicine course for 3-5 days.Afterwards consult your doctor and get examined for having scabies.Ok and take care."
},
{
"id": 35538,
"tgt": "Suggest remedy for cough",
"src": "Patient: hello i have a bad cough and saw my internal medicine doctor today, she checked my breathing a freaked out cause of how bad i sounded, i then coughed and cleared my chest out and she checked again and i sounded great. The only problem is she gave me advair.... \"doesnt work immediately she said\" but i will wake up from sleeping and cough myself into a really bad pressure headache that will cause my ears to ring and cause my vision to darken off. what can i do before going to bed, or right after i wake up to break up the mucus that builds up in my chest that causes me to cough for so long. Doctor: Hi, thanks for sharing your health concerns with HCM! If I were your treating Doctor for this case of productive and tiring coughs, I would come up with three advises, these include: 1.\u00a0\u00a0\u00a0\u00a0\u00a0get a chest X-ray and sputum test preferably before starting any medicines!2.\u00a0\u00a0\u00a0\u00a0\u00a0take ambroxol/acetyl-cystein containing cough syrups ! \u00a0\u00a0\u00a0\u00a0\u00a03.\u00a0\u00a0\u00a0\u00a0\u00a0add amoxycillin+clavulanic acid combination tablets BD or azithromycin 500 mg once daily for 5 days in addition to your present medicines but preferably after asking a chest physician!Hope this answers your question. If you have additional questions or follow up questions then please do not hesitate in writing to us. I will be happy to answer your questions. Wishing you good health."
},
{
"id": 224471,
"tgt": "What could cause delay in periods after taking an Ipill?",
"src": "Patient: i get my regular date on 1 dec then on 7th cec we had sex without precaution than i took ipill than date came on 22th dec but now i did not get my regular date which should come on 25th dec as regular period but now its over to one month from today but i tested my pregnancy by pregtes kit that was negative but now im very tense about my date when it will come. please tell Doctor: Hi,thanks for writing..I understand your concern.. I pill has a high dose of hormones. so the common side effects with it is delayed periods and intermenstrual spotting. It will get normal after this cycle. you can wait for a week . If still no periods, repeat preg test and if negative take progesterone for 5 days to get periods. hope i have answered your query. Good day."
},
{
"id": 24976,
"tgt": "Is there a possibility of stroke when diagnosed with Ischemic focus and blood vessel disease?",
"src": "Patient: I have recently been diagnosed with ishemic focus and blood vessels disease in the right anterioor putman.... I had 2 recent episodes that I;m now seeing a nuerologist and have already had 2 mri 's with findings of blood vessel disease and ischemic focus. I was told I had a stroke. is this possibly true. Doctor: Yes an evidence of ischemia on MRI is a suggestion of prior stroke . As it would have been a minor event you wouldn't have noticed and it may have passed silently. However medical treatment to prevent future episode which can be major is strongly suggested and an evaluation for the cause of same should be done depending on your risk factors and clinical evaluation. Regards Dr Priyank Mody"
},
{
"id": 125311,
"tgt": "What causes red spots on leg after an injury to patella?",
"src": "Patient: I fell on my knee and injured my patella 2 mos ago...after that I developed red spots on the same leg and also what looked like a prickly heat rash. The rash spread to my upper chest and neck. Treated with Prednisone. Now I still have the rash on my upper legs, still itchy. What could this be? Doctor: Hi, It could be a pertachiae and does not require any treatment. If lesion persist better to consult an orthopaedician and get evaluated. Hope I have answered your query. Let me know if I can assist you further. Regards, Dr. Shinas Hussain, General & Family Physician"
},
{
"id": 194193,
"tgt": "Suggest a remedy for nightfall",
"src": "Patient: Hi, Now I am 29 and my height 5.5'' and weight is 67. My problem is last 10 years night fall has been continuing (some time once in a week or 7 to 8 times in a month) , besides that some times I do masturbate (2 to 3 times in a week). So is there any problem in future ? Or tell me how to get red of this night fall and masturbate. Doctor: Hello, It can be due to increased arousal and less outlet. You can decrease arousal by yoga, exercise, particular sports. You can let out your sex arousal by having sex and increasing masturbation. Hope I have answered your query. Let me know if I can assist you further. Take care Regards, Dr S.R.Raveendran, Sexologist"
},
{
"id": 193188,
"tgt": "How to quit frequent masturbation?",
"src": "Patient: Hi, i'am sfiso and i'm 19years this year and iam having masterbation problems and i think this started when i was 14 and since than i have been masterbating frequently and i do this if i see more ladys wearing cloth that show parts of they bodies and how can i get help because i think it will affect my university life Doctor: Hi, Do physical games, exercise and yoga to control and divert your mindset. If you can't control then consult for psychiatric counselling. I Hope I have answered your query. If you have further doubts , I would be happy to help you. Happy day.Regards, Dr. S. R. Raveendran, Sexologist"
},
{
"id": 52778,
"tgt": "Can taking Spiraclatone be safe for fluid build up in stomach due to Sarcoidosis?",
"src": "Patient: I'm a liver patient with sarcoidosis and I keep getting fluid build up in my stomach. I would like to know how can this problem be solved. I'm currently taking spiraclatone and bumex as fluid pills but it is on removing fluid elsewhere but my stomach. Doctor: Hi there,Unfortunately, first line treatment for ascites (fluid in your abdomen) is exactly what you are taking - spironolactone (aldactone) and lasix or bumex. I would also recommend incorporating a low sodium diet. The other options for treatment are repeated paracentesis - removal of the fluid with a needle, however this is temporary and will likely be need to be repeated as the fluid tends to build up.Another option to decrease the amount of fluid in your abdomen is a TIPS procedure - a trans jugular intrahepatic portal shunt - where a shunt (connection) is made between the portal vein and the hepatic veins. This decompresses the portal system and ultimately causes less buildup of fluid in your liver, but a lot of your blood bypasses your liver and this can cause worsening mental status due to worsening encephalopathy. Risks of TIPS is also infection and bleeding, and the connection can clot off.I hope this answers your questions. Please feel free to contact me should you have any further questions."
},
{
"id": 119595,
"tgt": "Which tests should i undergo for swelling above collarbone on both sides?",
"src": "Patient: I have swelling above collarbone on both sides, gell like (very soft & squishy). I have an x-ray which showed nothing. My doctor is doing nothing. What tests should I ask for? Thyroid, Lymphoma, blood work? I am extremely tired all the time and cannot stay awake. Doctor: Hello,I would explain that your symptoms could be suggestive of thyroid gland dysfunction. For this reason, I recommend performing a thyroid gland ultrasound (this test can also help examine the soft tissues in the neck), complete blood count and inflammation tests and thyroid hormone levels. You should discuss with your doctor on the above tests.Take care. Hope I have answered your question. Let me know if I can assist you further. Regards, Dr. Ilir Sharka, Cardiologist"
},
{
"id": 11627,
"tgt": "What causes orange pigmentation after surgically operated for right femur",
"src": "Patient: It has been a week since I had surgery on my right femur. I am non weight abetting but I m getting around pretty well using my walker. I started getting out of be and noticed that the top of my right foot had an orange film on it and my lower leg was a bit discolored too. It wiped off with soapy water, but I haven t come into contact with anything orange, and it just appeared out of no where. Any idea what it was from? Will it come back? Doctor: Hi,I can understand your concern for orange pigmentation on surgically operated right femur.DO not worry it is a quite common phenomenon due to application of betadine and povidine iodine application over the affected foot during operation. The area is completely sterilized with a betadine and povidine iodine solution.When this is falling off from the superficial skin then it gives appearance of orange pigmentation.You can apply a emollient cream to moisturize the leg completely so that this tanned appearance disappears completely.Take care."
},
{
"id": 34914,
"tgt": "What causes rashes around the site of incision?",
"src": "Patient: I had my gallbladder removed 4 weeks ago, one week ago I got a rash on my chest (unclear what it was) then I got a slightly different looking rash around the site of the incision where the gallbladder was removed. I went to a NP and she thought the rash around my incision site wasn trelated to my surgery, but was possibly candidae (sp?). I but anti itch medication on the chest rash and antifungal cream on the stomach rash, and took Benadryl at night for the itch. The chest rash is going away, but the rash around the incision site isn t. Also tonight there was a pimple on the incision that opened up when I washed it and had pus (fairly white). I am concerned that this may be an indication of infection or staph? Doctor: Hi, I have gone through your medical history carefully and understood your concerns. The rash you got around the site of incision is suggestive for possible infection as long as pus was coming out. I highly suggest to consult with the surgeon who did the surgery and get the incision evaluated by him/her. You might need oral antibiotics or clearing of the incision. Hope it was of help!"
},
{
"id": 155106,
"tgt": "Can chemotherapy cause epigastric burning pain?",
"src": "Patient: I'm receiving chemo. 2 days ago I got a burning pain in the eigastric area and thought it was gas. The pain has gotten more severe. Now I can't stand up straight or lie on my left side. The pain seems to be localized in 2 areas, just under my left rib nearly in between my ribs and just below my waist and to the right. The lower pain is in the approimate area where I was told it was pain from ovulation. The pain radiates to my back when it is most severe. Antacids have not helped. The only thing that helps is a heating pad. With the pad the pain is tolerable when I don't move. Without the heating pad it is difficult to tolerate any position. Doctor: Hi, dearI have gone through your question. I can understand your concern. It may be due to acidity. You should take Proton pump inhibitors like omeprazol or pentoprazol. If pain is not releived by this drugs then you should go for ultrasound abdomen to search some other cause and take treatment accordingly. Hope I have answered your question, if you have doubt then I will be happy to answer. Thanks for using health care magic. Wish you a very good health."
},
{
"id": 108773,
"tgt": "What is causing my lower back pain with acid reflux?",
"src": "Patient: i am having lower back pain, pain is more when i walk more or keep standing for a long time, early morning i cant eat because my stomach feels bad, i get burning sensation to chest when i eat spicy food, did an MRI its normal, did CT it shows normal, did Xray its normal, one doctor suggested it could be gastric issue, another said i better check my kidney for infection... pls giv me some advice Doctor: Hi Welcome to healthcaremagic I have gone through your query and understand your concern. You are having two issues. One is backache.It can be due to deficiency of vitamin D.You can get your vitamin vitamin D level. If it is found low you can take supplements of vitamin D3. Analgesic such as ibuprofen can be taken for pain relief.Second issue is acidity problem which is causing heart burn you can take Omeprazole before meals for it. You can discuss with your doctor about it. Hope your query get answered. If you have any clarification then don't hesitate to write to us. I will be happy to help you.Wishing you a good health.Take care."
},
{
"id": 222540,
"tgt": "Is delay in menstruation a sign of pregnancy?",
"src": "Patient: hai , iam 26 yr old. my last menstrual period was on sep 14th and my avg menstrual period is 37 days..i wil always get correct 1 week delay ,so my expecting day of menstruation is oct 21st. i used to have premestruation pains like tender brest 1 week before period but now am not having any symptoms , so i tested home pregnancy kit on oct 21st but it shows negative..could i be pregnant ? if not why am not having pre menstrual pains?please help me in figuring out. Doctor: HI, I understand your concern. 21st October,( the expected day of period) was earlier to test by urine pregnancy test. Please repeat test now ( after minimum 8 days delay in periods).. it may turn +ve to confirm pregnancy. In case its -ve, the delay is likely due to hormonal imbalance.. which is to be investigated by hormonal assay, USG abdomen & treated according to cause. Consult a gynecologist please. Thanks."
},
{
"id": 76983,
"tgt": "Can pneumonia occur due to cysts in lymph node ?",
"src": "Patient: My son has been diagnosed with pneumonia and has been on antibiotics for 6 days now with no noticeable improvement. I just noticed he has two hard pea sized bumps on the back of both arms. Initially I thought they were auxiliary lymph nodes but his physician has said they are too low on the arm and are not nodes but cysts. Is this cause for concern? Is this relevant to the pneumonia? Doctor: Hi thanks for contacting HCM...Pneumonia caused when immunity low..It might be hospital acquired also.Vitamin A deficiency and environment pollution also contribute....The cyst you have described is not related to pneumonia ...It could be simple benign cyst...It can be removed...If needed FNAC can be taken from it ...Complete antibiotic for pneumonia....Fruits taken more...Take care.Dr.Parth"
},
{
"id": 156250,
"tgt": "Suggest treatment for bone cancer and jaundice",
"src": "Patient: I am 50 year old female....for past 4 year i am suffering from bone cancer....now since from 1 month i am suffering jaundice in level reaches 32b.....and it effected to lever.....currently taking treatment in kidwai cancer hospital.....give us your advice for which hospital is best for this disease to cure....RegardsAnusuya Doctor: HI ANUSUYA MADAM.THANKS FOR YOUR QUERY.THE TREATMENT PROTOCOLS FOR BONE CANCERS HAS IMPROVED .YOU HAVE NOT MENTIONED WHAT KIND OR TREATMENT YOU RECEIVED IN THE PAST 4 YEARS IE SURGICAL OR MEDICAL/CHEMOTHERAPY?BUT NONE THE LESS YOUR BONECANCER SEEM TO HAVE SHOWN SIGNS OF DISSEMINATION/METASTASIS,IN VIEW OF ELEVATED LIVER ENZYMES.GENERALLY ITS NOT CONSIDERED A GOOD SIGN.BUT NONE THE LESS SOLITARY LIVER METASTATIC LEISONS CAN BE MANAGED METICULOUSLY....TREATMENT OPTIONS FOR SUCH LEISONS INCLUDE SURGICAL RESECTION,CHEMOEMBOLISATION.CONSULT YOUR ONCOLOGIST FOR EXPERT OPINION.THANK YOU"
},
{
"id": 104631,
"tgt": "Suffering from Asthma since birth. Taking Asthalin Rotacaps. Can I Take 2 tablets per day?",
"src": "Patient: dear doctor i am taking asthalin rotacaps upt15 capusules a day previously it was inhaler my question is is this better than 2 tabs a day of asthalin 25 mg.i am 50 years old and since birth suffering from asthmawith more in monthsof april may june.can i try steriode which i stopped 25 years ago,i tried taking combihale 200fb 2 inhalations daily and very few of asthalin should i continue Doctor: Hi, Thanks for posting your query. Asthalin contains salbutamol which is a short acting bronchodilator and its action persist for only for 1-1 and 1/2 hours. You should consult with internal medicine specialist/ chest physician and should go for thorough check up. You should go for complete blood count, absolute eosinophil count, chest x ray and erythrocyte sedimentation rate. You should take combihale containing budesonide, a low potent steroid for a short period. You should take long acting beta agonist such as formoterol or albuterol for persistent long effect. You should also take antiallergic levecetrizine along with montelukast. Take care, Dr. Mayank Bhargava"
},
{
"id": 90939,
"tgt": "Suggest treatment for IBS",
"src": "Patient: i have a prblm related to my stomach. doctors diagnosed it as IBS. I have un formed stools. i can see un digested vegetables in my stool. i feel tireness and lack of enthusiasm. doctors are of the opinion that it is because of tension. i refuse to believe that i have tension of that sort. of course i get tensed and that happens even from my school days. i am now 38 years old and this started suddently on one day before 8-9 years. i am afraid to take medicines because of side effects and fear of habit forming. at last now i decided to get treated and the doctor prescribed the following medcines: Happi D (morning), enterogermina (2wice daily) and deanxit. can i get a permanant cure for this and live without medcines? i dont have any stomach pains,vomiting etc. i dont feel like going to toilet every now and then. in the morning i need to go 2-3 times. thats all. Doctor: Hi.Thanks for your query and an elucidate history.If it is just going to toilet 3 times in the morning , nothing much to worry. But you say there are undigested vegetables and tiredness , lack of enthusiasm, you need a treatment. The present treatment you mentioned is one of the best regimens, you can slowly withdraw then in doses and duration. Well, if your Doctors have diagnosed this as a case of an IBS , there is no specific treatment. Yet to add ::You can add creon or pancreatic supplements with food / avoid strictly all the foods and beverages that these cause or enhance such problems. Instant decision as to have an absolutely stress-free life is a must and changing the lifestyle can alone help you better than the medicines,"
},
{
"id": 162891,
"tgt": "Are Macalvit and Hovite L drops safe in a newborn baby?",
"src": "Patient: Hi, my baby is 1 month old. Pediatrician has suggested her macalvit 2.5ml twice a day and hovite L drops 0.5ml once a day for 3 months. Her birth weight was 2.9kg and after completing 1 month weight is 3.25kg. Please suggest is it safe for her to give these two drops? Ultra d3 drops are regular for twice a day. Doctor: Hello and Welcome to \u2018Ask A Doctor\u2019 service. I have reviewed your query and here is my advice. The drops are safe. But if baby is on breastfeeds then no need to give anything other than Ultra D3 drops. Vaccinate your baby as per schedule advised by your pediatrician. Hope I have answered your query. Let me know if I can assist you further."
},
{
"id": 174087,
"tgt": "Can I give half valtrex when child suffering from cold sores?",
"src": "Patient: Hi. Yes just a quick question. I have a 10 year old son he is about 95 lbs. I take valtrex for cold sores. My sun also has breakouts of cold sores. I take 1 gram when i have a cold sore. Would it be safe for me to give my so I a half one when he has a break out? Doctor: HiWelcome to HCMCold sores generally clear up without treatment. Treatment is needed if:-You have a weakened immune system-The cold sores don't heal within two weeks-Symptoms are severe-You have frequent recurrences of cold sores-You experience irritation in your eyesValtrex is not an appropriate choice for cold sores at 10 years of age. It would be better if you give him Acyclovir 200 mg 5 times a day for 7-10 days.These drugs help in shortening the course of illness but do not completely cure the infection.For further questions, do contact us.Dr. Archana Verma"
},
{
"id": 109251,
"tgt": "Suggest remedy for back pain during pregnancy?",
"src": "Patient: Hi I am 4 months pregnant. I am taking doxinate regularly, still I vomit every alternate day. Sometimes three or four times in morning. Could you please suggest any other wayout for the same. I have back pain and pelvic area pain from last one week. Is there any wayout to prevent this pain or to reduce its intensity. Doctor: Welcome to healthcaremagic,Getting some back pain is fairly common during pregnancy due to loss of calcium and increased strain on your back muscle due to growing baby. Use a pillow to support your back while seated and use it in between your legs to support them while sleeping. Use a hot water back to give heat to your back, this will reduce the pain. Take calcium tablets 500 mg per day for 2 months as you will need extra calcium during pregnancy which will make your bones stronger. Your vomiting will gradually improve on its own from 5th month onwards so just take doxinate till then.Hope that answers your question,Regards, Dr.Imaad."
},
{
"id": 153688,
"tgt": "Is the surgical removal of uterus advisable in case of cancer?",
"src": "Patient: Hi, may I answer your health queries right now ? Please type your query here... pls could you advise me to remove uterus in case of cancer in kolkata to have surgery without risk, the patient is in bangladesh how to go through this procedure, duration, best hospital, cost ect Doctor: Hi, dearI have gone through your question. I can understand your concern. In case of cancer it is advisable to remove the uterus. Treatment depends on type and stage of cancer. In early stage surgery is helpful. In later stage chemotherapy and radiotherapy is the treatment of choice. Please send me details of your cancer so that I can help you further. Hope I have answered your question, if you have doubt then I will be happy to answer. Thanks for using health care magic. Wish you a very good health."
},
{
"id": 147604,
"tgt": "What to do if having purple and blue discoloration, tingling and numbness in legs, cold feet, lower backpain?",
"src": "Patient: Few days of purple and blue discolration in legs with tingling and numbness. Started to get worse heavy feeling in legs while walking. Feet are cold. Lower back pain coming and going over this year. This morning seemed to have similar feelings in my hands. 34 yr old female. Over the summer I had dark stools with blood, it came and went at times. Doctor: Hi,i think your problem not coming from your spine and your purple and blue discoloration in legs with tingling and numbness is because of vascular claudication. which means impaired blood circulation in you legs which increased in severity during walking.you required some of tests like-colour doppler of legsmri of spineblood sugar and CBC.Thanks"
},
{
"id": 198591,
"tgt": "Is slightly darker genital area with distinguishable line on inner thigh normal?",
"src": "Patient: Hi. I am a 24 year old male person. I did always have a slightly darker genital area but I have recently discovered that this darkness ends with a distinguishable line on my inner thighs, rather than a smooth fade out. I would like to ask if this could have to do with anything abnormal. Thank you Doctor: DearWe understand your concernsI went through your details. There are many reasons to the slightly darker genital area with distinguishable line on inner thigh. You could know those reasons better. Use of tight under wears, excess sweating of the genital area, exposure to light or sunlight etc are the prominent reasons. Please do not be anxious about it. If you require more of my help in this aspect, please use this URL. http://goo.gl/aYW2pR. Make sure that you include every minute details possible. Hope this answers your query. Available for further clarifications.Good luck. Take care."
},
{
"id": 68936,
"tgt": "What causes painful pea sized lump in the spermatic cord?",
"src": "Patient: Hi I've had some pain and swelling in my right testicle and I went to the doctor the other day and he ruled out many major problems but just today I found a pea sized lump in my spermatic cord... I figure that's the reason for the pain and swelling but what should I do about this lump and what could it be? Doctor: The pea sized lump n the spermatic cord can be due to a lipoma or a cyst of the spermatic cord. The best way to get a proper diagnosis is ultrasonography and more important is an examination by the Urologist. It is advisable to get this removed for 2 reasons. You get the disease out of the body and Secondly to get the histo-pathological diagnosis."
},
{
"id": 221239,
"tgt": "What are the symptoms of pregnancy?",
"src": "Patient: Hi, i had unprotected sex (due to the condoms not fitting and falling off before we realised) on the 01/01/11 at 1am in the morning and since no chemists were open, i had to wait till 3pm on 02/01/11 to get the MAP. 5 days later, the first condom fell out while I was peeing (we couldn t find it so we used another and then that kept coming off) So then 1 1/2 weeks later I had menstrul bleeding (kind of like a period) and it lasted 4 days, wasn t painful. It is now the 02/02/11 and I have had a few tickle pains, nausea but no vomiting, no discharge, hungry cravings (i wanted prawn chips lol) i am drinking 3 ltrs of water a day! I can t stop drinking and I actually hav lost a few kgs. My feet are swelling and I am tired and feel dizziness! I have done 3 pregnancy tests already (1 week part from each other) and they are all negative? So why am i feeling so weird? I did go to my doctor and he has asked me to come back again in 1 month (He did a test and it was negative too). He said nothing serious but that I might have a low count of hcg and pregnancy will be harder to pick up on? Also I am anemic so blood test was out of the question cause I d pass out and he said that if I am pregnant it would be a little risky (he knowing me so well lol) Doctor: HiDr. Purushottam welcomes you to HCM virtual clinic!Thanks for consulting at my virtual clinic. I have carefully gone through your case, and I think I have understood your concern. I will try to address your medical concerns and would suggest you the best of the available treatment options.You had your periods, and urine test is also negative, this rules out pregnancy.The food cravings and other symptoms that you are getting can be various vitamin and mineral deficiencies or Simply premenstrual syndrome- PMS.Tab B LONG F once at night for 3 months will be of help.I will suggest indulging in a healthy diet and regular exercise regime.Include plenty of fruits, salads, vegetables in the diet. Have adequate water intake , say 8 \u2013 10 glasses per day. Avoid deep fried foods, bakery products, and refined sugars.Also,have FOLIC ACID, B12, Iron supplements, it will take care of your anemia.I hope my answer helps you.Thanks.Wish you great health."
},
{
"id": 84605,
"tgt": "Will perinorm syrup controls both vomitting and loose motion?",
"src": "Patient: Kindly let me know if perinorm syrup controls both vomitting and loose motion? B.P. Nirantar YYYY@YYYY Doctor: Hello,Perinorm syrup is an antiemetic which is used to treat nausea and vomiting. It is not used to treat loose motion so I suggest an over-the-counter medication to treat the diarrhea. I recommend Imodium. I also suggest to take a probiotic and eat a a light diet during this period.Hope I have answered your query. Let me know if I can assist you further. Regards, Dr. Dorina Gurabardhi, General & Family Physician"
},
{
"id": 218869,
"tgt": "What causes blister like bumps on the upper lip during pregnancy?",
"src": "Patient: I keep getting this bump on my upper lip. Its not a fever blister it doesn't hurt or burst with pus, it raises and goes back down. I can't figure it out, I'm pregnant with my first child and would like to know if this is a possible pregnancy symptom? Doctor: blister lik bump on d upper lip can be due to hormonal imbalance...dirt accumulated causing white head or black head etc..nothing to worry.clean face twice daily...if not subsiding consultation from dermatologist.."
},
{
"id": 218898,
"tgt": "Is pregnancy possible despite taking Unwanted 72?",
"src": "Patient: Hi. yestersady I had unprotected sex with my girlfriend n she took unwanted 72 within 5 hours at that night bleeding was started but today there is no bleeding at all. Is there any chances of getting pregnent. She is also dealing with pcos. Please advice. Doctor: No need to worry if already emergency contraception is taken. Emergency contraception is very effective in preventinhg pregnancy.However it us advisable to use condom if you are not keen on planning a child."
},
{
"id": 220542,
"tgt": "What causes delay in periods?",
"src": "Patient: hi me and my 16 year old girlfriend have been having sex and everytime we have been protected but her period is late for instence her cycle last month was 40 somethings day and 30 something the cycle before that why could this be happening? Doctor: HiDr. Purushottam welcomes you to HCM virtual clinic!Thanks for consulting at my virtual clinic. I have carefully gone through your case, and I think I have understood your concern. I will try to address your medical concerns and would suggest you the best of the available treatment options.If her cycles are irregular, then I will suggest to get morning's first sample of urine tested for pregnancy. If it is negative, you need not worry.Even if it is positive, you can opt for termination with tablets safely till 9 weeks of pregnancy, under expert medical supervision.I hope my answer helps you.Thanks.Wish you great health.Dr Purushottam"
},
{
"id": 215706,
"tgt": "Is CBD effective in the treatment of pain due to arthritis?",
"src": "Patient: I have arthritis and a back issue and was wondering if cbd might work and help with the pain. I have been reading reports on how it helped so many people with there pain. I am a 84 year man who is recovering from a bowel blockage in which they removed about 3 feet of my small intestine out and don t have a bag on. I had to have emergency surgery in November of 2017. Doctor: Hello, There is very little active ingredient in the CBD oil that is legally available. There is other cannabis edibles for which the pain effect is probably there, but difficult to evaluate. Hope I have answered your query. Let me know if I can assist you further. Regards, Dr. Matt Wachsman, Addiction Medicine Specialist"
},
{
"id": 144641,
"tgt": "Suggest treatment for head injury concussion and nose bleed",
"src": "Patient: I was hit in the head hard enough to cause a concussion and had to have staples about 8 yrs ago .. I have noticed for the last cpl months clear liquid com3s from my nose after bending over .. I also had problems with my ear on the same side I was hit .. this fluid is like water it is not sticky or has any color to it .. it will come out like if you had a nose bleed . Doctor: clear liquid coming from nise while bending, which is not sticky and has no color may be CSF coming out from your nose. Its called CSF Rhinorrhea and it can happen because of head injury. You need to consult Neurosurgeon. If its CSF Rhinorrhea, it can be cured by surgery at base of your skull."
},
{
"id": 12962,
"tgt": "What causes red dry patches spots on my arms and on scrotum?",
"src": "Patient: Hi I have itchy red dry patches and spots on my arms and I have one single patch on my scrotum they have been there for about a month, I have had the same thing about a year ago but only on my arms and was diagnosed with pityriasis rosea was wondering if this could be the same thing Doctor: Hi,Most probably you may be having contact allergic dermatitis. Consult the Dermatologist for the perfect diagnosis and proper treatment. As there is itching, allergy is more likely possibility then pityriasis rosea. Apply steroid cream and take oral steroid in tapering dose and antihistaminic under observation of doctor.Hope this helps.Dr.Ilyas Patel, Dermatologist"
},
{
"id": 186140,
"tgt": "Suggest treatment for swelling and puss coming from gums",
"src": "Patient: Hi, I am 21 weeks pregnant and have one very deep pocket on my front tooth, I think it is pyorrhea. I am going in for another deep clean on Friday but i'm worried my tooth is going to fall out. The rest of my teeth are perfect the dentist said and it is probably due to my tongue piercing I had in my teens. What can I do to get rid of the puss coming out of my gums and get the swelling down? It's ruining my life.ThanksCharlene Doctor: hello thanks for consulting at hcm..it is due to periodontal problem causing abscess,,resulting n puss frm gums,,u could also do massage of gums with the pulp of ur fingers,,apart frm deep cleaning..dont worry it will improve with deep cleaning and finger massage,, do salt water gargle thrice daily,,hope it helps,,tc"
},
{
"id": 78752,
"tgt": "How to treat breathing problem?",
"src": "Patient: sir my child 16 years old. He studying intermediate in hostel. he doesnot take food properly. one month back apendisitice surgery. one day he fell down in college campus. breething problem effected. he says iam in a problem i am going to dead. so please give suggestion what can i do. Doctor: Thanks for your question on Health Care Magic. I can understand your concern. Probably your child is getting adjusted to hostel and is also missing home. Try to talk to him about it. As well if his breathing trouble increases kindly consult a doctor for the same, as he will require a detailed examination.Don't worry, you will be alright. Hope I have solved your query. Wish you good health. Thanks."
},
{
"id": 73276,
"tgt": "What causes constant chest pain?",
"src": "Patient: i felt barable chest pain left side since long. i have done my heart ecg which is normal, and doctor tell me it is very normal. then again i asked and insisted to do all tests. then my tmt & eco test done. both are normal. my chlostrol is border line high and homostistine hi. so doctor put me on homo check 1 daily & atorva statin 10mg one tab daily. but still barable chest pain is there. what may be the reason? is any chance of heart problem, or it may possible, that some time it malfunction? i dont have problem in doing heavy work like, running or weight lifting... Doctor: Hello dear , hiWelcome to Healthcaremagic.comI have evaluated your query in depth .* This seems in relation with psychosomatic manifestation of underlying stress and anxiety , rather than heart problem .* Needs proper management with meditations , deep breathing exercises , YOGA , concentration techniques .Hope this will clear your doubt .Regards ."
},
{
"id": 16595,
"tgt": "Is drinking safe after a big heart attack and clot in leg?",
"src": "Patient: After a big heart attack i am waiting for a another op to put 2 more stents in. Whist in hospital i got a clot in my right leg, extremely painful. I used to go out once a week to chat to mates and have a jar. I have had Fibromyalgia for many years. I am now getting stir crazy staring at 4 walls. A mate is prepared to give me a door to door lift. So far i have been seen by hospital and 3 different doctors, of of my questions was how much Guinness i can have on this one night I don t drink on any other days. So far i have had 3 different answers and i am getting fed up seeing a different doctor each time. I am housebound. Doctor: Hello, Alcohol is a myocardial toxin and generally, patients are advised to stop drinking alcohol post-myocardial infarction. However several prospectively performed cohort studies have demonstrated that drinkers of moderate amounts of ethanol are 40% to 70% less likely to manifest coronary artery disease or ischemic stroke when compared with nondrinkers or heavy consumers. Moderate alcohol intake is associated with a reduced risk of myocardial infarction (MI). Moderate consumption exerts several beneficial effects, including the following: (1) an increase in the serum concentrations of HDL cholesterol and apolipoprotein A-I; (2) inhibition of platelet aggregation; (3) a decreased serum fibrinogen concentration; (4) increased antioxidant activity (from the phenolic compounds and flavonoids contained in red wine); (5) anti-inflammatory effects (with lower concentrations of white blood cells and C-reactive protein); and (6) improved fibrinolysis resulting from increased concentrations of endogenous tissue plasminogen activator and a concomitant decrease in endogenous plasminogen activator inhibitor activity. One pint of Guinness a day is considered moderate. These are the pros and cons. Rest is up to you. Hope I have answered your query. Let me know if I can assist you further. Take care Regards, Dr Tushar Kanti Biswas, Internal Medicine Specialist"
},
{
"id": 42963,
"tgt": "How to improve sperm motility?",
"src": "Patient: My sperm analysis shows as detailed below: volume- 2ml, viscousity - Slightly, active - 35%, Sluggish - 5%, dead - 60%, Head defects. - 5%, Midpiece defects -10%, Tail defects - 5%. Wgat do I do to improve the motility and reduce the percentages of dead sperms to the barest acceptable medical minimum standard Doctor: Hello i think your dead sperms are 60 percent which are responsible for infertility in most cases.Foremost thing you have to do is strictly lookout for your diet.There is protein deficiency in you whoch leads to dead spermsEat protein diet in any form.Eat spring onionsDrink lots of citrus juicesDo morning walkDo power yogaI can help you with a spiritual mantra also if you are interested."
},
{
"id": 166085,
"tgt": "What causes fever due to throat problem?",
"src": "Patient: Hi.Dr. My child(4 and half year old) having throat problem from last 6 month and due to this he was suffering with fever and some time doctor they are recommending antibiotic (Amoxoline) etc and now i deceided to consult with ENT specilist and he recommended for operation for sometink behind of ear,so i just need help from you.can you explain me is it necessary or not and also its advantages and disadvantages. Doctor: Hello!i would like to have a complete history of your child including the duration of each episode and the frequency of these febrile episodes in a year.If the episodes makes him skip the school and he is febrile with high grade fever for more than 3 times a year,he needs tonsillectomy i.e, removal of tonsills that are basically defensive in cases of infection but gets hyper-active in certain individuals,that needs removal.Every surgical procedure has some complications but as such there are no major complications if performed by an expert person with trained staff.Its a day time procedure and patient is usually doscharged after 24 hrs with complete recovery seen in 1 week."
},
{
"id": 203472,
"tgt": "Could dark red spots on penis that slowly turned to softer colour indicate an STD?",
"src": "Patient: Hi Doc I had unprotected sex two weeks ago .It was not for long but it happened.I'm so stressed I think I'm losing my mind.I don't have any sores ,but i keep on thinking and looking at my penis.This morning after siting and watching tv I wend to the loo and noticed two red marks on my penis head,dark red but after a few min it wend from dark red to softer red color.Do you think its a STD or maby it was form sitting in a bad position . Doctor: HelloI think having had unprotected sex has put you under a lot of stress. The red marks seem to be due to posture or some allergy. Going to toilet repeatedly can be due to anxiety. It doesn't seem to be STD.dr saatiish jhuntrraa"
},
{
"id": 83814,
"tgt": "Suggest alternative of Rcinex,Mycobutol and Combutol for treatment of TB",
"src": "Patient: hi i am suffering for plural infection & water formation in my lung,after diagnosis that is TB. now since 4 months i am suffering from side effects of medicines like rcinex,r-cin ,mycobutol,combutol to lever.I have very high sgpt 300 ,these medicines doesn t match with my lever,so pls suggest me. Doctor: Hi,Advised second line anti-TB medications. Based on the history you seem to have developed liver toxicity apparently caused by the first line anti-TB medications. I suggest you to stop taking these medications and switch over to combination of second line anti-TB medications such as cycloserine, ethionamide, capreomycin, clarithyromycin, or fluoroquinolones.Hope I have answered your question. Let me know if I can assist you further. Regards, Dr. Mohammed Taher Ali, General & Family Physician"
},
{
"id": 24320,
"tgt": "Is it to be concerned about the heart beat suddenly raising?",
"src": "Patient: I was sitting here at the computer and all of a sudden my heart was beating really fast. I got a head rush an then my head slowly turned back to normal. I tried to do some breathing to see if it help. I could see my chest moving really fast and then I felt my heartbeat go back to normal. I just ate lunch not sure what caused this. It happened one time at work about two weeks ago also. Doctor: Thanks for your question on Healthcare Magic. I can understand your concern. In my opinion, we should rule out arrhythmia (rhythm disturbances in heart) in your case. Arrhythmia can cause rapid heart rate, chest movements, head flush etc and if not treated promptly, arrhythmia can be life threatening. So better to consult cardiologist and get done 1. Ecg 2. 2d echo 3. Holter monitoring (24 hours continuous recording of ecg). You may need anti arrhythmia drug on the basis of these reports. Don't worry, you will be alright. But first diagnose yourself and then start appropriate treatment. Hope I have solved your query. I will be happy to help you further. Wish you good health. Thanks."
},
{
"id": 178743,
"tgt": "Suggest treatment for indigestion due to acid reflux",
"src": "Patient: My four month old had a pyloromyotomy at 1 1/2 months. She still suffered from significaht acid refluxwhoch she takes zantac. She eats for ounces of enfamil gentlease every 2 to 3 hours. For the last 3 to 4 weeks I have noticed changes she started spitting up more now its a few times with every bottle and its curdled Also her bowel movements have gotten progressivly abnormal. Normally they are dark green with no curds. Then every othe bowelmovement she had looked a little like it did before her surgey. A week later most of them were abmormal . A couple days ago I changed one and if it was neon green with what looked like black blood clots in it. I had it tested was not blood. Now shenis going a lot more frequently and its bright or pale yellow and curdled. She started teethining already but her fussiness seems a little extreme for that. I have two older girls. Also she has crazy bad gas. I need some insight as to whats going on with her. Thank you Doctor: Your child may be demonstrating a protein intolerance and she should be examined by her pediatrician for this. She may need an elemental (predigested) formula."
},
{
"id": 202971,
"tgt": "How to treat chronic night fall?",
"src": "Patient: I am suffering from nightfall since last six years, i have taken medicine(homeopathy) but unfortunately it doesn't work out. Now a days without any dreams my sperms are coming out and i am suffering from both mentally and phsycaly. Please suggest me something which will help me from this severe diseases. Doctor: Hi, .First of all, it is not at all a severe disease. It is a quite normal phenomena.You had not mentioned your age, your marital status and other types of sexual activities of present and past. These information will be of use for a better guidance. If you abstain from sexual activities or not having any sexual activities night fall is a common thing to be expected. As any other organ of your body, your reproductive system also continues to function and produce sperms. When the stores get filled, it is natural that it has to overflow or has to come out. Semen is just a secretion like your saliva. Nothing like a precious one on not be lost. If you indulged in sex it will go otherwise. Nothing wrong in night fall and don't give any importance to it like passing urine. Dr S.Murugan"
},
{
"id": 48203,
"tgt": "What causes dark urine after 3 years of kidney transplant?",
"src": "Patient: Hi I am three years post kidney transplant and have bad toncilitus so my gp gave me penicillin but it has made sick and diarrhoea. Now my urine is dark and I m having pain in New kidney. Is this normal and just to extra hydrate myself or should I go to my renal team. Thank you Mel Doctor: HelloThanks for query.Dark urine with kidney transplant needs to be taken seriously .It could be due to either 1) Stone in transplanted kidney 2) Infection of the kidney (Pyelonephritis )Please consult your Nephrologist for evaluation and get Renal Function test done .Further treatment will depend upon the results of these tests .Dr.Patil."
},
{
"id": 10414,
"tgt": "Suggest remedy for hair fall",
"src": "Patient: Hi, may I answer your health queries right now ? Please type your query here... hi i am 24 i had very volumised hair before but i lost lot of hair n it become v thin now since i used to try shampoos now i wanna stick to 1 shampoo and i m using some special oil to my hair so suggst me best shampoo n i wanna know whether this shampoo has any side effects how to get back my old volumised beautiful hair Doctor: Hello and Welcome to \u2018Ask A Doctor\u2019 service. I have reviewed your query and here is my advice. You seem to have androgenetic alopecia. I suggest you to apply a topical minoxidil 5 %solution, 1ml, twice daily. In addition I suggest you to take an oral finasteride tab once a day for long. Both of these are approved remedies for androgenetic alopecia in males. Hope I have answered your query. Let me know if I can assist you further."
},
{
"id": 14819,
"tgt": "Is there a skin rash or virus that looks like bug bites?",
"src": "Patient: I have been told i am getting spider bites on my abdomin while i sleep. I have had my house treated tented even bought a new mattress and it is till happening. I am covered in scars and have previously had mrsa because of the problem. Is there a skin rash or virus that looks like bug bites? Doctor: HIThank for asking to HCMI really appreciate your concern, in fact the skin lesion is art of looking for the doctors and without looking to the skin rash clinically it may not give the exact clinical diagnosis here just little assumption can be made from the history given, and I could say that this could be \"Herpes\" lesion because in layman term this is known as spider bite, better to get it confirmed and for that you need to see the dermatologist, take care and have nice day."
},
{
"id": 78892,
"tgt": "Suggest treatment for severe pain in the arm and chest",
"src": "Patient: Plz plz plz help me and very very very upset..I have pains in left hand and arm and chest pain..am 47 years old..family history heart attack s...am overweight..my blood pressure is high.heart spray does not work..that I work was given...am very stressful..when I go to weight training.. the pain comes..or if I have arguments with the wife... Doctor: HI,Welcome with your query to HCM.Based on the facts of your query,after its review,In my opinion You should go to ER for treating your Angina with acute IHD,which needs immediate treatment-By the time reach ER-Call GP and take following from your home kit-Take Depin 10 mg Tab Aspirin 325 mgm/Chewable 2 statTake Deep Breath till your reach ERTab Antihypertensive-if your are on any,to reduce the accompanying high BP attack.Hope this reply resolves your worry with your queries.Will appreciate writing your feedback review comments,to help the needy patients like you at HCM.Good Day!!Dr.Savaskar,Senior Surgical SpecialistM.S.Genl-CVTS"
},
{
"id": 95431,
"tgt": "Bloating after first meal. Do i need to worry ?",
"src": "Patient: Hi, i m 16 years old and i m kinda stress out because I ve been bloating the past few days. When i wake up in the morning its perfectly fine, but when i eat first thing in a day. my stomach s just begins to bloat and it continuous till the end of the day. I ve been drinking powdered weight gain milks because i m too thin, is that what it cause? Should i be worried about whats happening to me? Please help me. Doctor: Hello.Welcome.milk can cause bloating.You must take the services of a nutritionist for weight gain.He will outline the daily requirement and chart the type and amount of food required by you.Good luck."
},
{
"id": 74561,
"tgt": "What is the radiating pain in my chest along with severe burping?",
"src": "Patient: Hi Doctor, I concerns I have been having pain in my chest area for a couple months now, ( it's come and goes) it started on the left side then to the middle and right now it seam like it on the left plus all I want to do is burp even after drink water, seams like when I bend over I feel a little dull pain on the left side, and by lie down on the bed you can hear the rumbling sounds in my stomach, Hope you can help me with this Doctor: Respected user , hiThanks for using Healthcaremagic.comI have evaluated your query thoroughly .* Symptoms are consistent with reflux symptoms which may be due to - reflux oesphagitis - hiatus hernia - gall bladder dyspepsia - acute gastritis or duodenitis - gastric or duodenal ulcer - others .* Need thorough clinical examination , ultrasound of abdomen pelvis , x-ray chest and upper GI endoscopy for final diagnosis and management .Hope to clarify your query .Welcome for further assistance .Thanks for using Healthcaremagic.com & giving opportunity to assistWishing fine recovery . Regards dear ."
},
{
"id": 92486,
"tgt": "Pain in right side of abdomen, swelling and recurring cramps. Is this related to cancer?",
"src": "Patient: Male 46 been having pains in right side abdomen just above hip where my liver is, slightly swollen, pain is like intense cramp on and off over last few months now I have round pink purplish bruise appeared on spot where pain is coming from dont remember knocking it to cause bruise - what could be the cause I'm scared it may be cancer Doctor: HI Thank for asking to HCMI can understand your worry but for sure it is not the cancer, I advise you to first try some antispasmodic preparation \"Dicyclomine\" with Acetaminophen will give good result if it persistence there then it need to be examined and investigated, but right now no need to have some ideas, have ice day."
},
{
"id": 31059,
"tgt": "Suggest remedy for sleep apnea",
"src": "Patient: My 4 year daughter has been congested since Sept 2010. She is now going through sleep apnea. I have tried saline nasal sprays, humidifiers, vick rub. The doctor's keep telling me it's viral they have given her a puffer and antibiotics, but blocked nose has never left her and it's been 5 MONTHS! please help. Doctor: Hi dear, Sleep apnea doesn't go away with nasal sprays, humidifiers or antibiotics, children with sleep apnea might get cold more than others, perhaps that's why doctors suggested giving her puffer or nasal spray just to reduce cold symptoms.Sleep apnea is more of an underlying condition by which muscles at the back of the throat tend to relax making it difficult for the air to pass in and out of the lungs, children with this condition have loud snoring while asleep, sometimes breathing is disturbed (or you can hear like a chocking sound while sleeping). The first line of treatment sleep apnea is surgery by removing tonsils and adenoids to ease airway passage. The best thing i would suggest to do now, is to go to an ENT specialist to evaluate your daughters situation and severity of her condition, the ENT will then decide if she needs surgery or not according to her situation. Since she is just 4 years old and from the way i read your question i believe her situation is mild and perhaps she doesn't need surgery now.Wish you and your daughter all the best."
},
{
"id": 11427,
"tgt": "How to treat chronic hair loss?",
"src": "Patient: hi.. i'm Mr A, my question is i'm suffering from cronic hair loss, i do use unipexil, phytotricovital but its increasing its hair loss more and its been weeks i did not even wash my hair from fear of hair loss..plz do advice.. i'm 27 , measuring 1 mt 72 and 68 kgs Doctor: Hi Mr A,Thanks for posting here.You may be suffering from male pattern baldness, which is usually hereditory.You can use minoxidil 5% solution 1 ml twice daily and take tablets like biotin suppliments and finasteride 1mg. You need to meet a dermatologist for further treatment. newer therapies like PRP therapy and stemcell therapy are other treatment options.If you have an advanced baldness hair transplantation is the choice.Take care.Hope it was useful to you.Take care.Regards.Dr shilpa bhat DermatologistSubodha skin hair and cosmetic clinicBangalore"
},
{
"id": 2547,
"tgt": "Can i get pregnant with a prolactin level of 28.99?",
"src": "Patient: Fsh 8.78 mlu/ml Lh 38.89 mlu/ml s.Estradiol 32.78 pg/ml T4 6.88 ng/dl TSH 2.78 Mlu/ml S.Prolactin 28.99ng/ml this is my hormone report on second day of cycle i am trying to conceive since 15 months can u please explain if there is some problem or what should i do my gynecologist didnt explained whats the problem please help can i conceive naturally my husbands sperm count says normozospermia Doctor: hello...thanks for trusting the healthcare magic doctors for ur health related queries.I guess ur concern is about..with ur hormonal reports whether u can get pregnant..right.I have gone through ur hormonal reports as u send me. ur FSH,LH,ESTRADIOL are within normal range. but ur prolactin is slightly high. this high prolactin effects the menses, n menses are usually irregular. once menses are irregular, there is chances of getting pregnancy is decreases. but u didn't give info about your menses. so it is difficult for me to comment on this.second thing..u told me that ur husband having normospermia, that's a good thing. if u r having regular menses with ur husband having normospermia..chances will be DOUBLE.with the above discussion u can try out my suggestions-1. if u have regular menses with this high prolactin level,then nothing to worry. if having irregular menses then visit to nearby endocrinologist. he/she can help u.2. if u had gone through all the investigations related to getting pregnancy n all r normal, then u have HIGHEST CHANCES OF GETTING PREGNANCY.if not the feel free to ask ur treating doctor for doing all investigations.2. if u r in hurry then visit nearby infertility specialist.he/she can help u.I hope the above information is useful, informative and helpful for u.regards-Dr sudha rani panagar( i can understand ur tense-full situation, DON'T WORRY..U HAVE CHANCES OF GETTING PREGNANT. I pray the God that u will be blessed with a cute n healthy baby...hopefully TWINS)"
},
{
"id": 10539,
"tgt": "Suggest remedy to stop hair loss",
"src": "Patient: sir i am suffering from hair loss .i have oily scalp plz suggest me best shamppo......due too hard shampoo my hair are so weak i m using himalya shampoo but i m not satisfy ......i m using a tablet v b 7 forte.... that is good for me or not plz help me Doctor: Hello,I do not know, you are male or female. If you are male, then it may be androgenetic alopecia, and if you are female, then it may be seborrheic dermatitis, along with Telogen Effluvium. Consult the dermatologist for the perfect diagnosis and proper treatment.However, you may continue Biotin containing capsules and may take Finasteride drug if you are male.Apply mild steroid containing lotion on the scalp. Do shampoo with ketoconazole-containing shampoo on alternate days. Avoid stress and anxiety and never forget to tell about your gender and age, whenever you ask a question.Hope I have answered your query. Let me know if I can assist you further.Regards,Dr. Ilyas Patel"
},
{
"id": 99722,
"tgt": "Suggest medication for breathing problem",
"src": "Patient: Hi, may I answer your health queries right now ? Please type your query here...SIR I AM SUFFERING FROM BREATHING PROBLEM.....I WASPREVIOUSLY TAKING BRONKOPLUS FOR THIS.BUT NOW ASTHIS MEDICINE DOESNT COME ..SO CAN U PLZ HELP ME OUT WITH SOME ANOTHER MEDICINE Doctor: Hi,If u are having breathing difficulty regularly since long time and u were taking some medicine which is no longer available then u can change your medication bcuz there are very good brands of bronchodilators and antiallergic medicines available in market but only after physician's consultation. So Don't worry change that and u will be alright."
},
{
"id": 82348,
"tgt": "What causes breathing difficulty and pain in heart?",
"src": "Patient: I recently took my prescribed medication, but now I have a very horrid feeling in my chest. It hurts to breathe and it s causing me to worry about whether or not my heart can withstand this pain. Could someone please help me figure out what might be wrong? Please and thank you Doctor: Thanks for your question on HCM.In my opinion you should first relax and be calm.I advice you steps to proceed.1. Get done ECG to rule out cardiac cause for your chest discomfort.2. If this is normal than get done chest x ray and PFT (Pulmonary Function Test) to rule out pulmonary cause for your chest discomfort and breathing problem.If both are normal than get done CBC and S.TSH to rule out anaemia and thyroid causes.If everything above is normal than you must have anxiety associated symptoms. So be relax and keep calm. Avoid stress and anxiety."
},
{
"id": 23483,
"tgt": "What causes blood pressure?",
"src": "Patient: I am a 52 year old male. Have had heart ultrasound and stress test. Am healthy except for hereditary blood pressure that i have taken medication for for last 15 years. The doctor says the muscle in the middle of my heart is slightly thicker than normal and they want a better look. I am booked for a heart MRI on Nov 22 in Red Deer and a kidney angiogram Dec 12 in Calgary. Do you feel this is necessary and the proper way to go . Thank you Doctor: Hey,There are several causes of high bp.Considering some important and common causes1) Abnormal kidney function or blockage in the artery of kidney2) Abnormal thyroid function3) Most common at your age - Essential Hypertension, which does not have a cause & has a genetic predisposition. 4) Abnormal secretion from a gland of our body which is Adrenal or Suprarenal gland.The evaluation for all these needs some blood test like Thyroid Function test, kidney function test, test for adrenal gland and testing for blockage in the kidney artery which can be done through renal angiography or indirectly through renal Doppler. Renal angiography is considered better.Secondly increase in the muscle of the heart ( LVH- left vventricular hypertrophy) is a result of increased BP for a long time.Consider reducing salt in diet, stop smoking if u are doing so, do regular exercise & take medicine regularly.ThanksDr Sameer MaheshwariCardiologist, Fortis Hospital"
},
{
"id": 2899,
"tgt": "Can i get pregnant?",
"src": "Patient: I had an ectopic pregnancy in 2007 due to my tubes being tied. while removing, my doctor also removed 3.6 tube on the left and 2.6 on the right. I am considering have another child if possible. would reversable surgery work for me, with this amount taking. Doctor: Hi,Reversible surgery doesn't actually help much. It's success rate is not encouraging. I suggest IVF treatment.Hope I have answered your query. Let me know if I can assist you further.Regards,Dr. Salah Saad Shoman"
},
{
"id": 16718,
"tgt": "Can I increase the dosage of Olmesartan to treat a sudden rise in blood pressure?",
"src": "Patient: Hi, I am 82 years old and have always been in good health. My blood pressure was always on the low side. I was surprised when suddenly it was high. My doctor prescribed 0lmesartin 20mg. My blood pressure is still high. can I temporarily increase the dosage until I see my doctor? Doctor: Hello There After going through your medical query I understand your concern and I would like to tell you that if your blood pressure is more than 150-90 continuously then you can increase the dose but is recommended for you to monitor your blood pressure for 1 week.Hopefully this information will guide you properly.Kind Regards Dr Bhanu Partap"
},
{
"id": 83751,
"tgt": "When will the liver levels return to normal after taking an antibiotic?",
"src": "Patient: My Rx antibiotic says that it can raise your liver AST and ALT but I need to get some bloodwork done for my doctor. I want my blood tests to show my true normal levels. How long does it take for your liver levels to return to normal after taking an antibiotic? Doctor: Hello, You have to remember one thing elevated liver enzymes like SGOT and SGPT may elevated in infections ,drugs or toxic causes etc. Antibiotics may not reduce it unless the elevated liver enzymes due to septicemia or liver abscess. So dear first mentioned the condition what your suffering from or post your blood test report the will answer accordingly. Hope I have answered your query. Let me know if I can assist you further. Take care Regards, Dr. Penchila Prasad Kandikattu"
},
{
"id": 18582,
"tgt": "What causes elevated diastolic pressure despite taking Losartan?",
"src": "Patient: My blood pressure has been sky high 180/110 range. I take losartan 25 mg a day, recently increased to BID. Also diltiazem XR 120 daily, HCTZ 25 mg daily, 81 mg baby aspirin. I have had 2 episodes of a fib in the past 2 years. I can t tolerate lisinopril or metoprolol. Why is my diastolic so high? I am so afraid of having a stroke. Doctor: Hello and Welcome to \u2018Ask A Doctor\u2019 service. I have reviewed your query and here is my advice. After going through your medical details I understand your concern and I would like to tell you that your diastolic and systolic both pressures are high which needs proper management. You are already taking anti-hypertensive shut may require to increase the dose and for atrial fibrillation you need special blood thinner instead of aspirin. Kindly consult a cardiologist personally and discuss to start Debigatron or Warfarin and ask him if you need dose modification for high blood pressure. Hopefully this information will answer your query. Kind Regards, Dr. Bhanu Partap"
},
{
"id": 19830,
"tgt": "Is a blood pressure level of 145 over 90 cause for concern?",
"src": "Patient: Hi, may I answer your health queries right now ? Please type your i am diabetic and on the normal i hav low blood pressure and fast paulse now i have a blood pressure of 145 over 90 and my husband keeps stressing out he doent think i will have a heart attack Doctor: Hi ThereAfter going through your query I understand your concern.I would like to tell you that blood pressure more than 140/90 cones under hypertension and that needs to be treated either with medication or lifestyle modification. But being diabetic you have to be more careful if your BP is going above the normal range. It's recommended for you to check your BP when you are sitting calmly at least for 15 min, if still your BP comes high then consult with your Endocrinologist/Physician for further management.Hope this will HelpKind Rgards"
},
{
"id": 146744,
"tgt": "What are the chances of coming out of coma after opening eyes?",
"src": "Patient: If a male 73 years old had open heart surgery for the second time and surgery was fine but developed fluid in lungs several times then toxins in blood now last Friday was flat lined and brought back after 15 - 20 mins but is in a coma but b/p is normal, opened eyes twice, breathing on own what are chances of coming out of the coma? Doctor: there are many types of coma and whether or not someone regains consciousness depends on the cause. if your family member suffered low oxygen levels during the flat line then the prognosis is poor. this is something you should discuss with the doctors treating him"
},
{
"id": 124247,
"tgt": "What is the red itchy rash below the bump of my knee?",
"src": "Patient: I fell hard on my leg, just beneath my knee. This happened just over two weeks ago. I had a lump and bruising around the bump. Coloring is gone, but the area is still sore. A new symptom is a red itchy rash below the bump. What would cause it and should I be concerned? Doctor: Hello, Since you mentioned you had a fall this could be just a muscle and skin tissue injury and nothing major. Using hot water fermentation and gentle massage will help to heal the bruises and soreness. Itching is because there is a tissue healing in the skin level so don't worry for it. You can apply some Vaseline or any moisturizer which will help you feel relief. You should be fine and there is no major injury incurred. Hope I have answered your query. Let me know if I can assist you further. Regards, Jay Indravadan Patel, Physical Therapist or Physiotherapist"
},
{
"id": 166362,
"tgt": "What causes bumps on back?",
"src": "Patient: My 5 yr old Grandson has been getting red blind spots (bumps) on his back, sometimes on his shoulder and odd ones on other areas. Nobody seems ableto identify them They fade to leave a litlle bruise like light brown stain, and have been appearing over the past few weeks. Doctor: Hi...Thank you for consulting in Health Care magic. Skin conditions are best diagnosed only after seeing directly. I suggest you to upload photographs of the same on this website, so that I can guide you scientifically. Hope my answer was helpful for you. I am happy to help any time. Further clarifications and consultations on Health care magic are welcome. If you do not have any clarifications, you can close the discussion and rate the answer. Wish your kid good health.Dr. Sumanth MBBS., DCH., DNB (Paed).,"
},
{
"id": 32284,
"tgt": "Could cold and sore throat with redness around the mosquito bite be dengue?",
"src": "Patient: hellomy name is sam , im living in malaysia , yesterday morning I attacked by a mosquito and i dont know its dengi mosquito or not. today morning when i woke up i was cold , Sore throat , and the place that moswuito attacked is red like a circle i want to know is it dengi mosquito or not Doctor: Hi, Thanks for posting in HCM. Dengue is a viral fever transmitted by mosquitoes. It is usually a high grade fever associated with nausea, vomiting, tiredness and hemorrhagic tendencies. What you are experiencing coldness and sore throat is not due to dengue but might be due to prodromal symptoms of bacterial infection. Also redness at the mosquito site bite cannot tell weather it is dengue or not and could have happened due to local allergic reaction.Well, if you need to confirm weather you are infected with dengue or not, kindly get the following investigations done after a week: 1. Complete blood count with peripheral smear highlighting platelet count. 2. Dengue test for antigen and antibodies. If it doesn't indicate anything related to dengue, then your doubt is cleared. For sore throat, kindly gargle with warm salt water twice daily and take decongestant syrup twice daily. Hope the information provided would be clear. All the best."
},
{
"id": 82182,
"tgt": "What causes twitching in upper chest with back pain?",
"src": "Patient: just feeling twitchy in my upper body today. back, upper chest area. not hurting but weird feeling. had some before in back with a back/neck problem in February. I have a broken toe so using walking boots and crutches and staying on the couch all day, and I feel them more today just laying around. Doctor: Thanks for your question on HCM.It seems muscular cause only but need to rule out heart causes first.So get done ECG to rule out cardiac cause.If this is normal than your symptoms are mostly due to muscular pain.So follow these for better symptomatic relief.1. Avoid strenuous exercise and heavy weight lifting.2. Avoid bad postures in sleep.3. Take painkillers and muscle relaxant. 4. Apply warm water pad to affected site."
},
{
"id": 193706,
"tgt": "What is causing my husbands sexual dysfunction?",
"src": "Patient: my husband needs a colostomy reversal and hernia repair but has had sexual dysfunction problems that could be related to his original sigmoid cancer 5 years ago - yes he should have had this operation at least 4 years ago but we had to sell our home and various other problems arose. He was on life support for a while and before this was a normally functioning man of his age Doctor: Hello, Yes, it can be related to cancer. Long term diseases can cause loss of libido. Hope I have answered your query. Let me know if I can assist you further. Take care Regards, Dr S.R.Raveendran, Sexologist"
},
{
"id": 137763,
"tgt": "Suggest remedy for problem with left leg while walking",
"src": "Patient: Hello Doctor, My father is 71 year old. He has diabetic (152 mg) patient but under control. Past 7 days he said he having problem with left leg while walking...mean could not able give any presser to left leg if they give he is unbalance....last night doctor he had very heavy cold and cough not able to sleep......again morning we check sugar it s now 475 mg randomly...please advice what could be symptoms Doctor: As per your explanation I understand there are two issues. First is left leg imbalance and not able to give weight in it. Second is high blood sugar.For sugar patient it is common when ever there is some thing wring in body along with inflammation the first thing is sugar level goes up. But it can also be normally with out any cause so first thing you need to do is maintain his diet and blood sugar level through insulin or which ever way possible. If needed do visit your diabetologist for further treatment.Next you have mention he is not able to balance or give weight on one of his leg. But you have not mention what exactly happens when he gives weight on leg. Is it due to pain or loss of control. Let's assume if its pain.Now pain in leg can be due to some injury where he might have hit his leg some were and he Don't remember or it can be due to reduce blood circulation also. Here I suggest patient need to visit a orthopedic doctor and go ahead with proper examination and evaluation and plan treatment as per that.If there is too much pain at leg then patient can use hot pack as well apply a crep bandage to stabilise the soft tissue. This will help for getting some pain relief but remember this will not cure the pain. So do visit doctor as early as possible. If its loss of control then also you need to evaluate it properly why its happening. It may be possible due to age also. But further evaluation as well conformation can be done only on examination so I suggest to check with doctor again for this.I am sure this will be useful for you if you follow the given advice properly. Take care."
},
{
"id": 151816,
"tgt": "What diet should a person having lateral medullary syndrome follow ?",
"src": "Patient: What is the nutritional guidelines for lateral medullary syndrome.Any particulrs restriction in it? Doctor: hello welcome to hcm lateral medullary syndrome is a caused by blockage of blood circulation in the one of the brain vessels supplying medulla of brainstem. so nutritional guidelines will not apply here. if the patient has difficulty swallowing due to the stroke he would be suggested accordingly about tube feeding. to prevent such strokes it is essential to see hypertension, diabetes, lipid problems exist. they would need attnetion when it comes to nutrition and not lateral medullary syndrome per se. hope this is helpful."
},
{
"id": 87635,
"tgt": "How can lower abdominal pain with constipation be treated?",
"src": "Patient: hi im having lower abdominal pain since 2 months on and off. Sept was worse i had extreme pain and did not get my periods in october i got them 2 times my last period got over on the 2nd of nov. n im getting the pains again on my lower right abdominal and also problems while passing stool suddenly what should i do? Doctor: helloI think you should go for ultrasound pelvis to rule out the cause.After confirming diagnosis i suggest you to start treatment.Tab Evecare 1 tab twice a day ( for irregular mensis)Triphala powder 1-2 tsp at bed time ( for constipation)As these medicines are not given without prescription consult to local doctor."
},
{
"id": 9851,
"tgt": "Suggest treatment for hair fall problem",
"src": "Patient: hello doctor! i m undergoing severe hair fall problem.......i was in U.S. for last 3 yrs.i was using olive oil and pantene shampoo.everything was fine.......but now in india(bbsr,orissa) i was using the same oil........it doesn't workout......so i changed to parachute coconut oil and boutique shampoo......my hair is severely falling.....can i get suggestions.......i m 23 yrs old......... Doctor: Hi Dear,Understanding your concern. As per your query you have symptoms of hair fall problem which seems to be due to change in environment, climatic conditions, fungal infection, hormonal fluctuations, and could be due to nutritional deficiency. Need not to worry. I would suggest you to maintain proper hygiene of your scalp. Wash with neutral shampoo and conditioner with lesser chemicals in it. Consult dermatologist for proper examination and go for required testing done. Doctor may also prescribe supplements containing biotin, Acetylcysteine and can give Minoxidil spray as well. Hope your concern has been resolved. Best Wishes,Dr. Harry Maheshwari"
},
{
"id": 156079,
"tgt": "Why is blood transfusion necessary for a cancer patient?",
"src": "Patient: Hi. My Mom is a cancer patient and Dr advised her chemotherapy where they are going to give her Gemcite/Carboplatin combination. One of the Dr said that she may need blood transfusion. I need to know why. I am a postdoc fellow in Immunology and can understand medical terminology. Please help me and guide. Doctor: Hi,Gemcite/ Carboplatin combination, commonly used for lung and ovarian cancer, causes myelosupression i.e., a decrease in WBC, RBC and platelet levels. If the symptoms become severe, the patient might need a blood transfusion or decrease in dose of the drugs being given/ both."
},
{
"id": 134835,
"tgt": "What causes heart palpitations with pricking sensation in arm?",
"src": "Patient: Hi I have been suffering from heart palpitations and sometimes pins and needles in my left arm. It s def not anxiety but it is making me very tired! I do breathing exercises but it does not improve and often I wake up in the night very panicked. The only thing that has changed in my lifestyle is I had an.implannon implant removed and started the combined pill. Could this be the cause? Thanks Doctor: Hi dear ,Welcome to HCM,Understanding you concern. As per your query your wife have pain in left side ribs, finger tips and shoulder . Well the symptoms you mention in query are pointing towards a condition like angina . Such symptoms can also occur due to other reasons also like acid reflux and peripheral neuropathy .I would suggest you to consult cardiologist without any delay to rule out more serious condition, because it can lead to heart failure and rule out causes like stress and anxiety , muscle spasm and nerve dysfunction . Doctor may order certain test like ECG , troponin t , echo and examine with stethoscope . Doctor may prescribe treatment for restoring your ventricular rate. Beta blockers, like Atenolol, Carvedilol, Propranolol along with isosorbid nitrate . Calcium channel blockers, such as Dilitiazem and Verapamil . For now take proper rest and do not exert your body . Doctor may also refer you to orthopedic surgeon for muscle strain . Doctor may also refer you to endocrinologist for thyroid problem . Hope your concern has been resolved.Get Well Soon.Best Wishes,Dr. Harry Maheshwari"
},
{
"id": 189885,
"tgt": "Can implanon cause oral lesions? Outbreaks in the in the mouth. Allergic reaction?",
"src": "Patient: Can implanon cause oral lesions? I started to have out breaks in my mouth a couple months after I received the implanon. I have gone to the doctor and the dentist . Both were very unclear abou what it could be. The doctor ran some tests and they came back negative and the dentist said he has never seen anything like this before and took pictures but still has yet to get back to me. He seems to think its some sort pf allergic reaction . The implanon is the only thing I can think of that would cause this. I haven t changed my diet or anything like that. Doctor: HI and welcome , IMPLANON is the implant for birth control . So you need to consult a concerned doctor like a gynaec and OBS or a birth control consultant who can guide you in a proper way. As far as oral lesions are concerned , its outbreak can be allergic , or stress related or may be due to deficiency or vitamins , immunocompromised conditions , medications or oral infections bacterial or viral . This can be confirmed only on clinical examination so please visit a dentist and get it evaluated. Meantime you can use antiseptic mouth wash like chlorhexidine for mouth rinsing , maintain good oral hygiene , apply DENTOGEL an antiseptic anlagesic oral ointement topically pver the lesion will give relief. i hope this helps , take care."
},
{
"id": 17860,
"tgt": "What can cause thoracic spinal stenosis and moderate disc protrusion?",
"src": "Patient: I ve been diagnosed with mild thoracic spinal stenosis.Moderate disc protrusion that compresses the ventral spinal cord. This was due to an accident at work.I fell on my back while trying to get over a 4.5 ft. fence. I m currently trying to receive compensation for the injury. My employers want me to prove that this was the cause of my condition. Is there anything that you could offer that would help me prove that this indeed was the causes of my condition. Doctor: Hi, I understand your concern and would explain that there is nothing to prove that your condition was due to the accident, besides a previous spine CT scan or MRI in order to compare them. Anyway, usually traumas are associated to other fractures in the vertebral bones. You should discuss with your doctor on the above issues. Hope I have answered your query. Let me know if I can assist you further. Regards, Dr. Ilir Sharka, Cardiologist"
},
{
"id": 49205,
"tgt": "What does mild pelvicalyceal fullness of RH kidney and LH kidney shows mild hydronephrosis mean?",
"src": "Patient: I had undergone a abdomino-pelvic sonography, it reads that I have stones measuring 6-7mm in both kidneys... however it also reads mild pelvicalyceal fullness of RH Kidney and LH Kidney shows mild hydronephrosis. What does it mean? and can the stones be dissolved? The stones are of calcium oxilate? Doctor: HelloThanks for your query,based on the facts that you have posted it appears that you have no obstructing stones of a size of 6-7 mm in both the kidneys.Fullness of the Rt kidney is not significant Hydronephrosis of Lt kidney suggests obstruction to flow of urine from kidney into bladder and is most likely to be due to presence of impacted stone in left ureter which need to be evaluated further by dong radiological test called IVP.There is no medicine that can dissolve the stones.They will either pass down over a period of time through ureter into bladder and urinary passage or may need endoscopic fragmentation in case you get repeated episodes of pain,vomiting and fever ..Ensure to repeat your scanning every month .Ensure to drink more water .Dr.Patil."
},
{
"id": 19068,
"tgt": "Is rapid heart rate while exercising a concern?",
"src": "Patient: I am an active 21 year old. I regularly go to the gym regularly and have a physically demanding job. I eat well, carefully controlling my portions and nutrition. I drink plenty of water. I don t smoke, or use recreational drugs, and I drink only very occasionally. I use caffeine very rarely as I have a low tolerance for it. I don t use caffeine at all on days I work out, as I find caffeine makes me too jittery to be productive. I have a BMI of 23.4. I am in the best shape I have ever been. My concern is this: I have a resting heart rate of a rather boring 62 bpm on average. However when exercising to well within my tolerance (I can talk, but not sing), my heart rate climbs to around 220 bpm. Is this a cause for concern? I show no other symptoms. Doctor: Hello,They are just related to a normal physiological reaction, to the physical activity of emotional stress, by the activation of the sympathetic nervous system. So, there is nothing to worry about it.Hope I have answered your query. Let me know if I can assist you further.Regards,Dr. Ilir Sharka"
},
{
"id": 180852,
"tgt": "What is the dosage of Norco for a dental implantation?",
"src": "Patient: My dentist left me a message that when he pulled 4 teeth he gave me Norco 5.25 or 5.325. What is the correct terminology? I wanted to get the same script and ask my periodontist to call it in as I am having a dental implant next Friday. I was going to stop in his office in the morning and I told him I would leave a message on his answering machine tonight and get the script in the morning. Both the dentist and the periodontist are in different areas. Doctor: Hi.Thanks for the query..Narco is a painkiller combination containing hydrocodone 5 mg and acetaminophen 325 mg.Show the correct combination is Narco 5 / 325.But as it is a prescription medication you should get it prescribed by your dentist or the periodontist and take it as advised..It can be taken before and after the implant procedure to avoid pain after the treatment...Hope this helps you..Regards."
},
{
"id": 136295,
"tgt": "Suggest treatment for swollen and bruised knuckle",
"src": "Patient: i injured my hand 2 days ago. my middle finger and ring finger got jammed. now my middle knuckle is swollen and bruised. my hand feel tingly and swollen and numb. when i do anything envolving my hand. my hand starts to hurt and cramp. Do you think i might of fracture something? what should i do to treat it? Doctor: yes injury leads to fracture so better get x ray of hand done after meeting an orthopaedician and rule out fracture first later every other things can be managed easily"
},
{
"id": 135202,
"tgt": "Suggest treatment for swelling on temporal bone",
"src": "Patient: My three year old son has swelling over the temporal bone just over the L ear. The swollen area is very hard and doesn t seem to cause him any pain. It was also very red this morning which became white again when I gave him some Benadryl. However, the swollen area did not decrease down. Doctor: Hi Dear,Welcome to HCM.Understanding your concern. As per your query you have swelling on temporal bone. Well there can be many reasons for symptoms you mention in query like aneurysmal bone cyst , mastoiditis , folicullitis or inflammation of parotid gland . I would suggest you to consult general surgeon for proper examination . Doctor will check tenderness and consistency along with ultrasound and take sample for lab test . Doctor may prescribe antibiotics along with anti inflammatory or may recommend surgical excision under local anesthesia . For now take ibuprofen or acetaminophen for pain and apply warm compresses . Hope your concern has been resolved.Get Well Soon.Best Wishes,Dr. Harry Maheshwari"
},
{
"id": 176249,
"tgt": "What is the treatment for loose motions?",
"src": "Patient: my 15 month old baby is having loose motion since 2 days..she has done it about 3-4 times a day and yesterday night she vomitted twice though since morning she hasnt vomitted...Her food intake has also reduced since this afternoon and she is having slight temp since morning....doc has suggested sporlac and zinconia syrup..kindly advice is its gastroenteritis as i think her stomach pains when we touch her belly point Doctor: Hi....It seems your kid is having viral diarrhoea. Once it starts it will take 5-7 days to completely get better. Unless the kid's having low urine output or very dull or excessively sleepy or blood in motion or green bilious vomiting...you need not worry. Stomach pain is very common - do not worry. It is intermittent and will settle down.I suggest you use zinc supplements (Zinconia syrup 5ml once daily for 14 days) & ORS (Each small packet mixed in 200ml of potable water and keep giving sip by sip) as hydration is very important and crucial part of treatment. If there is vomiting you can use Syrup Ondansetron as prescribed by your pediatrician.Regards - Dr. Sumanth"
},
{
"id": 106101,
"tgt": "Had pnuemonia with asthma. I have been coughing up a thick/chunky sometimes milky white from lungs",
"src": "Patient: I have asthma and had pnuemonia recently and for at least 2 months have been coughing up a thick/chucky sometimes yellow/brown and sometimes milky white from my lungs. Does this indicate I may still have pneumonia and should I seek medical attention? Doctor: Welcome to Healthcare MagicGood DayIt is quite possible it is the mucous plugs seen in Asthma. The brown colour may be due to associated infection leading to bronchitis. You need to get the sputum examines and get a Chest X-ray as well. Continue taking your bronchodilators as advised."
},
{
"id": 494,
"tgt": "How to get pregnant after having a miscarriage?",
"src": "Patient: i m suja,my age is 33.now its 10yrs after marriage but i didn't have baby til now so i want your better suggestion for dat.and i m having medicines from 7yrs but i don't have good results.my every test results are normal.once i get pregnant but after 2mnths it started bleeding n i lost dat before 6yrs.my husband test results are normal. Doctor: hello..thanks for trusting the health care magic doctors for ur health related queries.i can understand ur condition but don't u can get pregnant.as u told that urs n ur husband reports are normal... right. so there is more chances of getting pregnant. u haven't give information about it medication so that i couldn't comment on this.with the above discussion u can try out my suggestions-1. relax.. don't worry. u can get pregnant.2. do coitus as more as possible during fertile period ( it is from 10-16th day of cycle). because during this period there is high chances of getting pregnancy.3. visit to nearby infertility specialist. he/she can help you.good luck.regards-dr sudha rani panagar"
},
{
"id": 159926,
"tgt": "Does intake of methamphetamine cause any type of cancer ?",
"src": "Patient: simple question...does methamphetamine cause any type cancer? Doctor: Hello. Thanks for choosing HealthcareMagic forum. Methamphetamine does not induce any type of cancer but it only produces hallucinations and mental high. Slowly more and more doses are needed to reach the same mental level and the person becomes addicted to the mental high. Dr. Rakhi Tayal"
},
{
"id": 165719,
"tgt": "What causes raise in the body temperature despite giving Fepanil?",
"src": "Patient: Hi, my baby is 1 year old.i gave fepanil for him last week as per advise of doctor ,he was teething too and was having cough after that fever went,but now he is fussy and i can feel that he is more warmer than usual so i gave him fepanil again now he is playing should i be worried? Doctor: HI...please do not give Fepanil without checking the temperature. Feeling warmer doesn't mean anything when the kid is otherwise active and playing around.Paracetamol can be given in the dose of 15mg/kg/dose (maximum ceiling dose of 500mg) every 4-6th hourly that too only if fever is more than 100F. I suggest not using combination medicines for fever, especially with Paracetamol.Regards - Dr. Sumanth"
},
{
"id": 165964,
"tgt": "Suggest remedy for fever and pain in legs of a 9 year old",
"src": "Patient: My 9yr old daughter has been running a 101-104 temp for 2 weeks now. She went to the doc and was told she may have a respitory infection and given breathing treatments. She is still running a fever 3 days later and complaining of her legs hurting. What else can I do Doctor: Hello dear.I would like to have the complete prescription of the doctor.If she hasnt been given antibiotics for fever and respiratory tract infection,i think you should consult again.Fever also causes body aches so when fever and infection will settle,it will also settle on its own.Give her antipyretics (must be advised by your doctor)Paracetamol tablet 250 mg,one tablet 4-6 hourly to relieve fever.Also you can try home remedies like gargles with warm water having 1/2 a teaspoon of salt in one glass,twice daily.Give her 1 teaspoon of honey in a cup of luke warm water once daily to drink.Avoid cold drinks,icecreams,ketchup and candies.Hope it will help.Wishing your child good health.Stay blessed :-)"
},
{
"id": 10927,
"tgt": "Suggest treatment for hair loss",
"src": "Patient: i am experiencing hairloss on one side of the scalp and also there is some itching , i understand that might be totally genetic , but i just want to make sure if its genetic , otherwise i'd be loosing my precious hairs just because of ignorance , what kind of tests i need to go through and what kind of medications i need ? Doctor: Hello. Loss of hair on one side of the scalp and localized to one well defined patch or multiple patches would suggest to be ALopecia Areata. It is an autoimmune condition which could be stress related and genetic and many other reason. I would like to do for my patient some blood test which include Serum Vit D3 and Serum Vit B12 and Serum Ferritin and Thyroid profile. Next you should definitely get it checked from your doctor to confirm the diagnosis. If its a localized then Intralesional Corticosteriod are given by a dermatologist once a month for 4 to 5 injections.Hope this helpsBestDr Batul Patelwww.drbatul.com"
},
{
"id": 107597,
"tgt": "Suggest treatment for back pain",
"src": "Patient: Hello, I have an important question, I have been out of high school for 2 years now, the beginning of wrestling season my senior year I slipped a disk in my back, I was prescribed blue vicodin at first but after about 3 month I was switched to hydromorphone hc 4mg, and then 8 mg a few months after that, I have a high opioid tolerance which is weird to me because I m 5 6 140 lbs. A little over 2 months ago I lost my insurance because I moved out of my fathers home, I am an athsmatic as well. As having. Reoccuring ringworm and now I cannot find a way to get my proair inhaler and hydromorphone and lamasil, I have not been able to find a job because of my back pain and I m using my little brothers child support to pay rent, I m struggling with my life and the pain is too much to ignore, I would like some advice as to finding the right doctor and as to what I need to do to attain my medication, friends have told me about doctors over the internet and phone but I do not know where to start. Please and thank you. Joshua Doctor: For lower back pain you can take some pain killer like ultracet-plus or zerodol-sp only when the pain is worse enough or disturbing your daily routine along with some calcium supplements like shalcal-d or toscal-gem once daily. Take some multivitamin like metilda-af or neurobian-plus once daily also. Do some physiotherapy exercises after an expert opinion of physiotherapist. I will also advice you to sleep on a plain surface."
},
{
"id": 84596,
"tgt": "Can painful and swollen knees be the side effect of taking Thyronorm?",
"src": "Patient: i started taking thyronome 12.5 mg for my raised TSH level about 10days back. My legs below my knees feel swollen and pain. Even feet are swollen from arouond ankles. Can this be due some side effect of the drug? I tend to get allergies from medicines quite often. XXXX 57 yrs. Doctor: Hi,Suggestive of deep vein thrombosis or cellulitis. Based on the given history you seem to be suffering from deep vein thrombosis until proved other wise. It is unlikely to be due to thyronorm. It is due to clot formation in the deep vein of the legs, the clot may get dislodged and can cause harmful effects like stroke or heart problem. I advise you to consult your treating doctor immediately for further clinical assessment and treatment.Hope I have answered your query. Let me know if I can assist you further. Regards, Dr. Mohammed Taher Ali, General & Family Physician"
},
{
"id": 141456,
"tgt": "Suggest treatment for gradual loss of short term memory",
"src": "Patient: For the past 5-6 years I have had a gradual lost of short term memory. When I go to the grocery store, I have to take a shopping list. Never had that problem before. When I tell a friend something, I sometimes forget what. I remember that I told him but forget what. It s very annoying to look up an address on a map and then when I drive to the vicinity, I ve forgotten the address. I m 86 so is this normal? My lady friend is also 86 and she remembers most of her things and even remembers most of mine. But she claims she sometimes has a similar problem. But in her case, even if she forgets the answer to a question, after talking for a few minutes, she suddenly remembers. Sometimes I ll remember over night but mostly not. I have to find some key item to trigger my memory. Doctor: Hello and Welcome to \u2018Ask A Doctor\u2019 service. I have reviewed your query and here is my advice. I advise you (and your lady friend) get appointments for a full battery of tests including serum, urine, and likely brain imaging studies. It sounds as if you could be suffering from the early stages of some type of cognitive dysfunction such as Alzheimer's dementia, however, this can only be diagnosed after making sure that you have no reversible causes for mental in capacities such as thyroid or other glandular abnormalities, Vitamin B12, Vit. D, electrolyte, or infectious types of disturbances. Seek out a neurologist who is interested in performing a comprehensive set of studies and analyses before coming up with a diagnosis. Don't just accept taking some pill, nothing better than exercising the brain to gain prowess and strength in the neurons and electrical circuitry. Hope I have answered your query. Let me know if I can assist you further."
},
{
"id": 89149,
"tgt": "What causes pain in the lower abdomen?",
"src": "Patient: My son is 14 having lower left side pain about inch away from belly button and hurts to pull up from bed cough sneezing pains are very sharp just was wondering what it could be hurt before but went away like a yr ago and now its back he has been taking tylenol more then he ever has Doctor: HI.Pain in the abdomen on the left side of the belly button which increases on pulling from bed, coughing and sneezing can be due to indirect inguinal hernia or torsion of testis. Get this clinically examine by a Surgeon and get operated to get a cure."
},
{
"id": 168573,
"tgt": "What causes severe acidity in an infant?",
"src": "Patient: my baby is very gassy and cries often due to gas. she also spits up often and refuses to feed for more than 5 minutes at a time. she is a breastfed baby. is there anything i can do to help alleviate these issues? Monitor my diet perhaps. I am worried she may be to sensitive to my breastmilk. Doctor: as per me u need not worry..... this things are common in newborn.... would advice to give the baby saline nasal drops in both the nostrils as it opens the nose and causes to drink milk easily..and secondly give good burping to remove air and for 10 to 15 mins.......u need not alter ur diet madam.....it doesn't effect ur baby at all....regards"
},
{
"id": 212263,
"tgt": "Developmentally disabled, over weight, paranoia, agitation, short term memory loss. On antipsychotics. Help?",
"src": "Patient: 29 yo developmentally disabled adopted daughter, (Hx. precocious puberty age 6, MRI negative, depolupron age 7 - 12), overweight but no other medical problems . Had sudden onset of psychosis (paranoid, aggitated and short term memory loss) lasting 7 days - 4 days were pretty bad, 3 days with improvement each day. Started her mensural period two days after onset of psychotic episode. ED gave her antipsychiotics and Ativan.... only gave her meds for 3 days and she is on nothing now and back to normal mental state. No other paranoia or episodes of pychosis previously. Took her for medical check up - labs pending. MRI of brain w/o and w/ contrast shows empty sella - otherwise negative. Birth mother paranoid schizophrenic - so somewhat worried about that possibility - but would seem like late onset? Now the Q: Can hormonal imbalances cause such a drastic event (very sudden onset of psychosis)? Given that it started a few days before her period and the MRI shows empty sella, does this possible match up for a hormonal imbalance to trigger psychosis? Doctor: Hi, Thanks for using health care magic. I have gone through your summary of history. Psychological disturbances are usually present during peri menstrual period. it may range of irritation to psychosis. so the answer to your question is yes that hormonal imbalance can cause these type of symptoms. but you should consult psychiatrist for detailed evaluation and treatment."
},
{
"id": 69976,
"tgt": "What does a pea shaped lump in groin area indicate?",
"src": "Patient: hi i have a small pea shaped, lump under my skin at the top of my inner thigh near my testicles. Its hard and only movable if i pull on my skin but it bled when i squeezed it (its painless) im not sure what it is but do i need to see a doctor or leave it for a few weeks? Doctor: Hi,This is likely to be a small sebaceous cyst that needs no treatment. I suggest you do not squeeze it as it may become infected.Regards,Dr K A Pottinger"
},
{
"id": 49518,
"tgt": "Having pain in kidney after taking Avas and Vogs. Are these related?",
"src": "Patient: dear sir, I am shefali and taking last two days Avas-5(1 tab.) with break fast and in the night having 1 tab. vOGS-M 0.2 but last 1 day I have in pain in kidney, so you suggest me what problem? I am also hypothyroid(TSH) 14.02 and fasting 199. Doctor: HiThanks for your queryThe use of these pills is unlikely to cause pain in the kidney.If you have persistent loin pain I would recommend a USG of the abdomen and Urine R/M. These will tell us if you have any kidney problem.Hope this helpsGHood luck."
},
{
"id": 219889,
"tgt": "What causes vomiting at night during pregnancy?",
"src": "Patient: Hello Doctor,Recently have completed my first trimester of pregnancy but still suffering from vomiting only during night time.At midnight 12 or 12:30 ,ll vomit .And head ll be bit heavy. I have been advised to take Doxinate tablet only when necessary.Is it safe. And pls suggest me good diet and also what to eat at midnight after vomit.It ll be a great help.Thankew in advance! Doctor: Hello dear,I understand your concern.In my opinion the vomitings might persist upto 5 months in a few pregnant woman.Take doxinate. It is safe and it is the first line of drug in treating vomitings during pregnancy.Its good that the vomitings are controlled by medicines.There is nothing specific regarding vomiting at midnight.But avoid spicy and oily diet .Take bland diet at night.The vomiting can also be due to gastritis.So if there is heartburn antacids also might be helpful.Take adequate fluids upto 3 litres per day,coconut water to prevent dehydration caused due to vomitings.Nothing to worry as such.They will subside spontaneously around 5 months mostly.Hope this helps.Best regards...."
},
{
"id": 145269,
"tgt": "What causes headache and twitching in body?",
"src": "Patient: My sister has been very ill for over 5 months now. She had a baby on January 28th this year. And after maybe one month later, she started getting sick. She s had so many symptoms that it s been hard to diagnose. She said it all started after she drank some coffee/cappuccino from Bad Ass Coffee in Albuquerque that was sitting in her car all day. She started by having constant headaches and a pulling sensation in her head and mouth. She started having trouble sleeping. She keeps taking sleeping pills, not only to sleep. But also to try blocking out her pain. Her family has told her not to take sleeping pills. But she won t listen. She s been very weak, and says that she feels like her body is turning into stone. She can t even tell when she s hungry, or when she has to go to the bathroom. She s also been losing weight from not eating much. She s also been having a burning feeling in her stomach, and says it feels like her stomach is being squeezed. She keeps sending me text messages saying she s dying, and telling everyone she loves them. Her husband and 2 older kids (8 and 15) are at their wits end. She s been to several doctors, had multiple blood work and MRI s, and everything keeps turning out negative. Nothing is showing any problems. I can t remember if she s had a CT scan? What can POSSIBLY be wrong? She s scared to death, and she s been an emotional wreck. A recent doctor gave her some sort of antacids etc., but apparently they haven t helped. There s definitely some sort of neurological problem as well, because she always gets twitches and jerking all over her. She s always saying that it feels like something is pulling her down. Our whole family is at a loss for ideas, and we re all very worried. It s nearing 6 months now. It don t look like our family will be celebrating the holiday season this year. Doctor: Hello ! I read your question and understand your concern. In my opinion the symptoms you describe may be related to postpartum depression . It is a very common disorder affecting women after delivery. The repeated analysis and MRI are another argument in favor of this diagnosis. This is not a neurological problem, but a psychiatric one. I would recommend you to bring her to a psychiatrist , to examine here and to choose the best antidepressant for her. I am sure she will get better after starting an antidepressant. She has anxiety too, and here twitches in the body and the gastric pain are related to that. An antidepressant in effective against anxiety too. Hope to have been helpful! Best wishes, Dr. Abaz Quka"
},
{
"id": 144337,
"tgt": "Suggest treatment for back pain after repeated injuries",
"src": "Patient: I believe when I was a small child, that when my friend pulled me feet first off of the gymnasiam stage and falling down on the hard floor that I started this problem. Then about 11 years ago, I dropped out of the bus with a 12 pack of cans on my hip and I think I may have done more damage. Anyway, recently, I slipped and turned awkwardly and really am paying for this now. I am 60 years old and some days are just not easy to sit in an upright position...other than this dilemma, I am in pretty good health and expect to live a long time. Doctor: hi,thank you for providing the brief history of you.since you suspect you had many falls in the past and the recent incident is bothering you more than I will say, kindly get an MRI of the whole spine done once which will help to come closer to the actual injury and any old injuries if troubling will also be visiblebased on that the treatment of choice can be made. also you can do hot water fermentation and relax the spinal muscles.you can meet a spine specialist and get checked as I mentioned an MRI will also give a good input. if things are all normal you can discuss with the spine specialist about undergoing physical therapy session. The physical therapist will help you in the form of regaining strength in the muscles which will help to improve your functionality again.with the grace of God I wish you a long healthy life."
},
{
"id": 124337,
"tgt": "Which treatment is best for hand pain?",
"src": "Patient: I was at a water park yesterday and instead of following a speed slide all the way down, I ended up flying off the slide where the side plateaued. I slammed down with the back of my arms hitting the sides of the slide, instantly making my hands and fingers tingling and somewhat numb. I wake up today and my palms are burning with pain. What can I do to better the situation? Doctor: Hello, As what I understand by the history is that you had a fall and the initial symptom was a pain, tingling and burning sensation. After a day the pain is restricted to hand region only. This might be due to the nerve entrapment anywhere in its course of bodily movement. Having a physical examination should provide help. Usually, in such cases, the initial part of treatment starts from medicine to rest and then exercise in later stages. Please don't ignore falls and meet up a physician if possible. Hope I have answered your query. Let me know if I can assist you further. Regards, Jay Indravadan Patel, Physical Therapist or Physiotherapist"
},
{
"id": 42612,
"tgt": "Suggest treatment for infertility",
"src": "Patient: Hi,I'm Sanjib from Ranchi,India.I'm 40 years.My wife who is a school teacher ages 32.We r both married since 8 years but there is no issue yet,We visited several doctors including IUI carried out thrice but no result.I think the problem is with me,as I loose complete erection once I insert inside her vagina.and the cum drops at the mouth of her vagina.Could u help out please. Doctor: yes i can help you in this issue. Follow the below mentioned advise:Food substances:follow healthy diet plan includes nuts (almond,cashew etc) steamed food in your daily food intake.avoid junk ,fried foods.take a pomegranate daily and add more vegetables and fruits daily. - gives antioxidant property.do walk for 3 to 5 kilometers daily.Avoid stress and have a sound sleep.oil bath with sesaem oil once in a weekAyurvedic Medication:apply Urad dal oil or tentex forte ointment (Himalaya drug company) in your penis and have a gentle massage- that will increase the blood flow to the penis muscle .Herbs used for infertility purpose are as follows\u00a0\u00a0\u00a0\u00a0\u00a0Kapikacchu (Mucuna Pruriens -, which has been found to increase sperm concentration and motility.\u00a0\u00a0\u00a0\u00a0\u00a0Gokshura - which raises testosterone levels.\u00a0\u00a0\u00a0\u00a0\u00a0Ashwagandha , which enhances spermatogenesis via a presumed testosterone-like effect.\u2022\u00a0\u00a0\u00a0\u00a0\u00a0Shatavari (Asparagus racemosus Willd.), which enhances fertility by reducing oxidative stress.Hope my answer fullfill your query.Thank You"
},
{
"id": 132057,
"tgt": "Cause for the knots in the foot and the severe pain?",
"src": "Patient: I have had two surgeries on my left foot, the first was to put 4 screws and a plate due to foot pain and he also cleaned out my joints. the second surgery was to add some balls in my foot due to the joint being worn out, he took the 4 screws and plate out due to the screws being broke. I have two knots in the bottom of my foot and my foot is hurting constantly. I am waiting for a MRI that should have been weeks ago but the doctor wanted to wait. what could this be Doctor: There is a muscle on the inside arch of your foot that runs from the inside of the heel bone to the inside of your big toe. This muscle pulls your big toe in--toward the midline of your body. If you are sure that the bump is in a muscle, the abductor hallucis muscle is the most likely candidate. It could have been injured and now wants to cramp. If the bump you see is bluish in color it could be a collection of blood from the injury--a hematoma. Massage will help the body reduce this. Another possibility is an injury to the navicular bone in the same general area. This would feel harder than the muscle and will not go away with time or massage. Finally, and least likely if the bump is truly on the inside of the arch and not on the bottom of the arch, is a condition called plantar fibromatosis. This would nothing to do with a muscle or bone but rather a growth in the plantar fascia. This will also not respond to massage and time."
},
{
"id": 57405,
"tgt": "Veins prominent in chest and abdomen, high ammonia in liver, difficulty breathing, bronchitis",
"src": "Patient: Why are my veins showing through my skin in my abdomin, chest? The veins are very dark and transparent. I have just been diagnosed with having a very high level of ammonia in my liver , blood stream. I am having an exteme hard time breathing. Last Wednesday I went to the Dr and I also have Bronchitis ,, I just finished a perscription of the Z-Pac. I m in a lot of pain in my mid section, and can not breathe . Doctor: Dear Friend.Welcome to HCM. I am Dr Anshul Varshney. I have read your query in detail. I understand your concern.High ammonia and Visible veins can be a sign of Chronic Liver Disease.i advise you following:1. Liver function test2. USG Abdomen3. PT/INR4. Endoscopy5. Viral markers.This is my best advice for you with the available details, if you have any further query, please ask us.Stay Healthy"
},
{
"id": 213916,
"tgt": "I was active but now i feel non-energetic and tired. Why is it ?",
"src": "Patient: For the last few months I m so tired I cant sleep enough. i have no energy, I used to work out all the time and loved being active now if im not at school or work all i do is sleep or lay around. I keep prrocrastinating even simple things like cleaning and doing homework. In school i cant focus at all. Im a 20 year old female Doctor: Thank you for choosing health care magic! I could imagine the phase what you are going through.It is a kind of tough transition from a teenager to full adults as person will have to deal with personal and professional tasks. Do not stress out at this time. If you feel like you are not able to cope up with both the things at time do not feel shy to take your parents help. Discuss with them about your difficulties and discuss solutions along with them. Start learning multi tasking which is very important as everything is important in life."
},
{
"id": 20218,
"tgt": "What causes protrusion of rib bone, heart palpitations, insomnia along with heart pressure?",
"src": "Patient: Hi Doctor I have been experiencing the following symptoms for \u00b1 1year: -protruding rib bone, which seems to have gotten gradually worse -palpitations/ inconsistent heart rate -insomnia/ fatigue -dull headache (left-side only) -feeling of pressure over the heart-area -slight dizziness -numbness on left-side of body -tingly -sensation in left hand & left shoulder blade -sensation of tightness over left eye -dull burning sensation over heart-area It can occur during the day, but is most common at night when I m in bed. Please can you advise whether I require medical advice or is this just a common anxiety attack? Thank you, Petula Jackson Doctor: The reassuring aspect of what you describe is that it has been present for over one year. Taken together, I would be suspicious that the need is for healthy lifestyle and regular physical activity. Any time that you ask if medical advice is needed, the answer is yes. A simple examination and reassurance are very important to peace of mind even if nothing is found. When you go, the questions for each problem are, when does it start, how long does it last and what makes it better or worse. By asking yourself each question, your question as to whether the sensation is worrisome may be answered."
},
{
"id": 7324,
"tgt": "I didn't get my period from last 51 days. Can regestrone tablets help me ?",
"src": "Patient: Hi, I didnt get my periods for past 51 daysmy gynec prescribed regestrone tablets for 3 days3 tabs per dayits already 11 days i stopped using these tabletsso when can i get my periods Hi,I didnt get my periods for past 51 days.my gynec prescribed regestrone tablets for 3 days(3 tabs per day).its already 11 days i stopped using these tablets.so when can i expect my periods? also im trying to conceive Doctor: Hello; welcome to HealthcareMagic If you completed a course of regesterone and did not get periods even after 11 days it is better to get re-examined by your Gynaec and if required get investigated to find the cause of irregularity of periods like hormonal imbalance;ovarian cyst;or any other cause.So please consult your Gynaec immediately. Thanks"
},
{
"id": 21551,
"tgt": "What causes low pulse rate?",
"src": "Patient: Hi, I recently had my blod presse tken and the reading was 113 over 60. My pulse rate was also measured at 47 bpm. I am a 40 yr old male in reasonabe shape and fitness. I was told that whilst my blod presure reading was fine, my pulsae rate was too low. Should I be concerned and if so, what should I do about it ? Thank you... Doctor: Hi,Welcome.This is Dr Sameer.Low heart rate could be normal sinus Bradycardia or it could be more serious heart block. Sinus Bradycardia is very common in athletes. You need ECG for diagnosis.If it is not caught on ECG, than we put 24hr ECG monitoring called Holter Monitoring.Treatment will be as per the report of above investigation.Thanks & take care"
},
{
"id": 71150,
"tgt": "What causes weakness and nose bleed after having flu?",
"src": "Patient: Hi there. I was feeling really run down for a few weeks and then I had a flu (proper flu). Was hoping that would be the end of it, but two weeks after the bout of flu I still feel really exhausted - so haven t been back to the gym. I kinda feel like only half my heart and lungs are working - ie quite weak. My voice also sounds tired. Feel worn out and depressed and head feels like it could explode (although don t have headache as such). Have had a runny nose the past couple of days and noticed blood in my mucus - and have just had a nose bleed. Any advice? Doctor: Hello and Welcome to \u2018Ask A Doctor\u2019 service. I have reviewed your query and here is my advice. Upper respiratory tract viral infection (flu) can cause bronchitis (inflammation of airways) and pneumonia (lung infection). And these can cause hemoptysis (blood in sputum). So get done chest X-ray and PFT (Pulmonary Function Test). Chest X-ray is needed to rule out lung infection. PFT is must for the diagnosis of bronchitis. You may need antibiotic and inhaled bronchodilators based on the reports. Don't worry, you will be alright with all these. Hope I have solved your query. I will be happy to help you further. Wish you good health! Thanks."
},
{
"id": 128098,
"tgt": "Suggest treatment for tunnel vision, fatigue and twitching of muscles",
"src": "Patient: About a month ago, out of nowhere, I ve started experiencing confusion, lack of concentration, mental fog, vision issues ( Halos, 3d imaging, tunnel vision), fatigue, trouble sleeping, falling feeling, twitching inside, trouble walking, anxiety and overall malaise ! What is wrong with me! Doctor: You have to consult psychiatrist once. Symptoms are not exactly fitting into any orthopedic disorder."
},
{
"id": 226066,
"tgt": "Had unprotected sex while missed pills, breast muscles sore and tender, tiredness. What could it be ?",
"src": "Patient: okay, my daughter missed 3 pills in her last month pill pack. so, she just stopped and waited for her period to start. she did have a menstrual cycle, restarted a pill pack. she did have unprotected sex during a two week period. so she is now on her next pack for a couple of weeks. has gained 10lbs, is nauseated and does have emesis when she took one drink of an alcoholic beverage. which in the past has never bothered her. breast feel like \"sore muscles\" some tenderness also. she is extremely tired. I was wondering if the birth control pills are masking the hormone to detect pregnancy and that is why she has had so many negative results. she has never been over 115 lbs since high school. Also, would a lab draw prove to be more accurated than the urince specimen. thank you. Doctor: hi,thanx for query.As your daughter did not take her pills properly and she had irregular menses ,ispite of that she can be pregnant.plz get doctors advice and get b hcg test done to confirm pregnancy.a full pelvic scan by agynaecologist and blood work up is required.it might be case of ectopic pregnancy too.plz get help byee."
},
{
"id": 211461,
"tgt": "Suffering from Bipolar. Taking medications Tegretol, seroquel, lithium and lamictal. Will the medications affect the sexual experience?",
"src": "Patient: Hi. I am a 33 year old female who has bipolar. I take a few medications to keep my mood stable which include tegretol, seroquel xr, seroquel, lithium and lamictal. I have a good relationship with my psychiatrist and am responsible about my health. I had been on trileptal which did well for a few years but then I went high and needed something stronger. My doctor put me on tegretol and it has helped with my moods. However for months I wasn t able to experience an orgasm. Then I felt almost over sensitised and sore in the outer genital area and still orgasms really weren t and aren t to this day possible. Might this change? Is there any herb that might help? I have tried many meds over the years and it is best that I stay on Tegretol as others I have tried (apart from the ones I am on now) had side effects that were intolerable or were in some way not a fit. If anyone could shed any light on this it would be great. Thank you for your time and care. Sorcha Doctor: Hi,Tegretol has only rarely been known to cause anorgasmia. In my opinion, you should first get your blood tested for serum prolactin levels to rule out hyperprolactinemia which is common in people on psychotropic medicines and causes inhibited orgasms. If you are found to have hyperprolactinemia, then the more likely causes would be antipsychotics like Seroquel. It can be treated with medicines and would not be a problem if you do not wish to change your treatment regimen. If there is no hyperprolactinemia, you will have to try reducing the dose of your medicines one at a time to identify the offending agent. Once you are sure that this is due to Tegretol or any other agent, you can consider changing it.Hope this helps you. Best wishes."
},
{
"id": 24002,
"tgt": "What happens when the stent in the back of heart cannot be reopened?",
"src": "Patient: My husband has had open heart surgery about 6 yrs ago and the doctor was only able to do 2 bypasses even though he wanted to do 4. He used the artery from his arm for the bypass instead of the leg where the vein is usually taken and used for a bypass surgery. The bypass colapsed in the back of the heart where one of the bypasses was done, the other bypass was an artery connected at the front from his lung. He has now had 5 stents in the back part of the heart where the bypass colasped and was told he was not a candidate for open heart again. What will happen when they can't reopen the stent in the back of his heart. Is there anything that can be done? Doctor: If the stent becomes occluded then he will have a heart attack. Heart attacks are caused by ischemia to the heart muscle. The goal is to prevent the ischemia to the heart muscle. You need to optimize your medical treatment now before the ischemia becomes worse. He should take aspirin daily and all of the medications that have been prescribed by the cardiologist and or cardiovascular surgeon. he should also exercise daily and eat a diet that is high and fruits, vegetables, whole grains, and omega fatty acids. He should avoid red meats and foods that are high in cholesterol. You should also try to avoid fatty foods. He should not eat any fast food at all. He should also avoid alcohol and tobacco."
},
{
"id": 34150,
"tgt": "What causes discoloration,swelling and pain in fingers?",
"src": "Patient: hello,i need a medical assistance.I am 30 years old and i have a problem in my fingerswhich gets swollen and turns into black color specially at the joint area near the nail of my hand fingers. i got acute pain and i need an assistance online and ready to make visit to mumbai if required. Doctor: Hello and thank you for your question.It does sound like you need to see a doctor. You could have a form of arthritis causing your joints to swell.The swelling could cause the discoloration of the skin. It is important to find out the cause of this swelling since it could be serious and there may be a treatment available to you. Please try to see a doctor as soon as possible for exam and blood tests.Regards,Dr. Robinson"
},
{
"id": 171109,
"tgt": "Is it to be concerned about abnormal sound when child cries?",
"src": "Patient: My baby is 7 1/2 months old teething. . . never cries and is crying sounds weezy I believe chills on infant ibprofen few wet dipairs/BM yesterday drank 3ozes formula and 1/2 cup friut 3 hours ago. . .should i be doing more, should i take him in? dehydration? Ahh new mom! Doctor: Hi,Welcome to Hcm,I understand your concern. You seem to be doing very well for a new mom. Infants of this age can sometimes have abnormal sound while crying. Can be due to some secretions in the throat and sometimes mild nose block . But these things usually recovers without any problem and without much treatment. So as long as baby is active, feeding well and passing urine and stool well, dont worry and enjoy your motherhood. Tale care."
},
{
"id": 92532,
"tgt": "Had three C sections and having abdominal pain. Anything serious?",
"src": "Patient: i have had abdominal left lower pain for years i have had 3csetions have had test donr best thing i was told scar tissue an fybroid cissed however it feels like im ripping tearing on daily basis i cant function normal i cant sleep an am in constint pain its worse during an after my period however i have it daily with or without my period no over counter meds help an doctors are reluctant to help me with pain managment i suffer an i feel they do not beleive me on the level of pain i am in an the seriousiness of my pain condition i cant suffer no more i am lost Doctor: hi i had gone through your query and understand your concerns. i would come up with the possibility of DIVERTICULITIS may be the cause for your sufferings.-inflammation of balloon-like protuberance due to excess Gas pressure in intestines.often caused by additives in food,or low blood sugar.diet to follow;-high fibre diet which produces large volume of faeces requiring wider diameter of colon,and gas. -refined food food lead to hard stool.softer stools do not strain the gut.x-ray with barium enema may confirm the diagnosis. I hope this is helpful for you, thank you"
},
{
"id": 94914,
"tgt": "Had severe pain, took out appendix, no relief, doctor said damage to ovaries, took prostap inj., loss of apetite, no periods, passing clots. Normal?",
"src": "Patient: im 24 yrs old, my pain started a year ago and the hospital took my appendix out thinking it was them causing my pain but it turned out they where fine and i have continued to have major pain on right side in stomach/ groin area and sometimes upside the womb. i had a camera through my belly to find out what it was and they said they have found that i have some torn damage to my tubes to overies and that they are stickey and sticking to my pelvic area, ive just had my first prostap injection and have been taking HRTs nearly 2 weeks ago but it stopped my period stright away but a week lateri got servire pains and started bleeding alot and then it happened the next day but i continues to blled for few days alot of blood few clots and there was tissueing in there too. and to top it off i am finding i cant eat and when i do it comes out of either end and sometimes i fain but im not a small girl im a 14 and have always loved my food. is any of this normal??? Doctor: Hi, . I read through the details with diligence. I am sorry for what is happening. It is unfortunate that you were misdiagnosed and had to undergo a surgery. I am unsure why you are on prostap and HRT - ?endometrosis perhaps. If it's the reason, then probably the pain and the bleeding episodes might be related to this pelvic condition. Any way since the bleeding restarted, I would suggest reporting back to your gynecologist. He/she can restart birth control pills to control the bleed and subject you to further tests to locate the actual cause. Vomiting and loose stool following a meal may not be related to pelvic problem. Perhaps consulting a gastroenterologist and getting GI endoscopies can provide us with clues as to the cause. In brief, I am unable to relate your symptoms to a single clinical entity. I suspect that you could have a pelvic as well as gastrointestinal pathology. Hormone assessment such as thyroid disturbance also needs to be assessed. Hope this answers your query. Regards"
},
{
"id": 105350,
"tgt": "Shortness of breath and tightness in throat like when crying. Constricted airway? VCD?",
"src": "Patient: I have tightness in my throat , as if when you are crying how tight it feels and feel short of breath and wheezing sometimes. I began suddenly and the episodes come on suddenly. I am being treated for allergies and allergy related asthma . But from what I am reading since the inhalers don t help the problem could I be dealing with VCD or a constricted airway? Doctor: if your allergies are not controlled it might be one of the food allergen milk,egg,wheat,potato,chana you get your blodd serum tested for particular antibodies against these food items in the mean time stop all milk and diary products start using montelucast 10 mgm tds get your results and start low dose immunotherapy now possible for food products also your problem will be surely solved"
},
{
"id": 5005,
"tgt": "Took postinor after unprotected sex. Took second pill late. Can I get pregnant?",
"src": "Patient: Hello,Just to be sure...last week Monday I had unprotected sex and I took Postinor 2 tablets. However, instead of taking the second pill 12 hours after, i took it 16 hours after. Will this cause me to be pregnant. I could wait for my monthly period which should be next week...but I just want to have an idea of the possible results. Doctor: Hello,Thanks for posting your query on health care magic.Postinor is 99% effective if taken within 24 hrs. Since you have received second dose after 16hrs, there is nothing to worry.Chances of you being pregnant is very minimum.i.e less than 1 %.So you need not worry. I suggest you to practice safe sex to prevent STD.Hope this helps.regards,Dr Nilofer"
},
{
"id": 128613,
"tgt": "How to treat the itchy and the swollen ankles?",
"src": "Patient: Hi, over the christmas period i have experienced itchiness all over my body, not too bad but just annoying. But today in this heat and being 112kg in weight both my ankles have swollen significantly. A few weeks ago i had my annual blood tests and noticed my ferratin level is 244 H. My husband and i have been married 8 yrs, and he has hep C. Could i be having liver problems? I am a registered nurse and maybe just being paranoid. Thanks Maxine. Doctor: YAP IF YOU ARE HAVING UNPROTECTED INTERCOURSE YOU MIGHT DEVELOP HEP C. BUT USUALLY IN 8 YEARS HEP C DON'T CAUSE LIVER DAMAGE.GET YOURSELF TESTED FOR HEP C.REGARDING ANKLE PROBLEM , DEAR YOU ARE 112 KG AND BEING A NURSE YOU STAND FOR LONG TIME.REDUCE WEIGHT AND DON'T STAND FOR PROLONG DURATION.GET WELL SOON."
},
{
"id": 200515,
"tgt": "Can masturbating without foreskin involvement cause any problem?",
"src": "Patient: I am 23 years old. Normally I used to masturbate without my foreskin rolling back. Now when my foreskin rolled back for the first time and exposed the penis, which erected further after that,.. i was feeling little inconvenient to masturbate. Is there any problem that i masturbated all these years without foreskin rolling back? what should i do? Doctor: Thanks for asking in healthcaremagic forum You can masturbate as you wish according to your fantasy. But to do not forget to pull back prepuce(foreskin) after the act. And wash glans and foreskin with water to avoid irritation/infection. All the best."
},
{
"id": 205953,
"tgt": "What are the side effect of Lithosun and clofranil?",
"src": "Patient: Dear Doctor,After consultation with a psychiatrist, I was prescribed Lithosun SR 400mg (once a day at night after dinner), Clofranil 25mg (once a day at night after dinner), Setra 100mg (once a day after breakfast),Encorate 200mg (Twice a day after breakfast in morning and after dinner at night).Duration since I have been under these medications is 1 year plus.MY QUESTION:What are the long term side effects of these medicines (if any)?Can I get addicted to any of these drugs?May I ever discontinue taking these drugs?My symptoms before medication:Uncontrollable rageExtreme Remorse after the factSuicide attempt (once) (I tried to OD on my diabetic meds, yes I am diabetic)Destruction of family property when angryDestruction of my own property when angryBad dreams (negative things like wanting to kill the reasons for my anger)My doctor did not explain to me, minutely all the questions I asked of him. Could you please tell me what these medications are used to treat, exactly? Could you, based on the information I have provided, tell me what is my illness? Doctor: Hello thanks for asking from HCMYou have been prescribed Lithium carbonate, Clomipramine, Sertraline and Sodium Valproate. From the symptoms you have mentioned it appears that you have bipolar affective disorder but detailed evaluation is needed to make final diagnosis. Now coming straight to your questions-- You are on two mood stabiliser drugs. Lithium is associated with tremors, dizziness, nausea, ataxia etc side effects. On chronic use if used under prescription it is a safe drug. Sodium valproate may cause nausea, anorexia, vomiting, dizziness, hepatic toxicity etc side effects. Clomipramine is a TCA drug and it is associated with sedation, weight gain, blurring of vision, constipation etc side effects. Over all it may raise BP on long term use.Sertraline is SSRI and is associated with nausea, diarrhoea, vomiting, sedation, weight gain etc side effects.- No you will not get addicted with this drug. - Yes you can discontinue these drugs after proper consultation of your doctor by gradually tapering the doses of drug.Thanks, hope this helps you."
},
{
"id": 210892,
"tgt": "How to cope up with marital problems along with child s failure at school?",
"src": "Patient: I have severe marital problems and my daughter (12 years) is failing at school and believes that I hate her. I am so stressed, depressed, can not sleep at all for days. max time I sleep in a day is 1-1/2 hours. I am so frustrated, can not find a job, feel like a failure and hate it when a new day starts. Miserable woman Doctor: Hello,Thanks for choosing health care magic for posting your query.I have gone through your question in detail and I can understand what you are going through.Well it is quite clear that you are having a depressive episode. Depression can occur with regular stress and it can also occur biologically without any stress as well. You need proper treatment for the same. There are various effective therapies for depression like fluoxetine, venlafaxine, bupropion, mirtazapine or escitalopram and they are all very effective and mostly safe. You may also require some benzodiazepines like clonazepam or lorazepam for a short period like 2-3 weeks to reduce the anxiety and help with your sleep. If you are averse to the drug therapies then there is also an option of cognitive behavioural therapy. This therapy is taken by either a psychologist or a psychiatrist in which the therapist identifies impaired cognitions (Thoughts) and try to correct them. They are 15-20 min weekly session and there should be around 8-10 sessions to help you with your illness.Marital problems can be taken care of by marital therapy.Hope I am able to answer your concerns.If you have any further query, I would be glad to help you.In future if you wish to contact me directly, you can use the below mentioned link:bit.ly/dr-srikanth-reddy\u00a0\u00a0\u00a0\u00a0\u00a0\u00a0\u00a0\u00a0\u00a0\u00a0\u00a0\u00a0\u00a0\u00a0\u00a0\u00a0\u00a0\u00a0\u00a0\u00a0\u00a0\u00a0\u00a0\u00a0\u00a0\u00a0\u00a0\u00a0\u00a0\u00a0\u00a0\u00a0\u00a0\u00a0\u00a0\u00a0\u00a0\u00a0\u00a0\u00a0"
},
{
"id": 96758,
"tgt": "What first aid can be given to a toddler after drinking hand sanitizer?",
"src": "Patient: our 2 year old grandson drank a little germx hand sanitizer. do we need to take him to the ER, or maybe give him some milk? not sure how much was in the bottle so he may have just had a little taste of it (best case) or as much as 1/2 ounce (worst case) Doctor: Hello, Thank you for posting your concern on HCM,The reaction will vary from the components of the hand sanitizer and the amount that was drunken. In general hand sanitizers have a big % of alcohol, so in children drinking it may lead to alcohol poisoning, which requires medical emergency intervention.I can't know for sure how much time has passed since the time of swallowing the gel, but if it has happened within last hours, I would recommend, to take the child to an ER, so the doctors would keep him under observation, in the case of the worst scenario.Hope this helps,Regards,Dr. Klarida"
},
{
"id": 101420,
"tgt": "How to cure shortness of breath while doing heavy duty work?",
"src": "Patient: When lifting heavy items or climbing stairs, I get short of breath. I am a 37 year old female. This started a year ago. At the onset of this problem, I had test ran on my heart and everything came back fine. The allergy doctor tested my lung function and everything was fine. What could be the problem? Doctor: Hello.Thank you for asking at HCM.Feeling short of breath with lifting heavy weight or climbing stairs may be due to many reasons.As your cardiac evaluation and lung function are normal, I would suggest you to get blood hemoglobin done. If it is low, you could take iron tablets along with folic acid & vitamin B12 tablets for a few months. Also if you are not accustomed to lifting heavy weights or climbing stairs since long and if you have started these activities recently, that may also be the cause. Even obesity/overweight may be the cause. In these cases, I would suggest you to consult a good physio trainer who will advice you regarding diet and exercise.Hope this will be helpful to you.Wish you best of the health.Regards."
},
{
"id": 13290,
"tgt": "Suggest treatment for raised bumps on back,feet and hands",
"src": "Patient: Hi my question is i a rash on my bottom of my back and on the sides of my feet and the Palms of my hands. They are litte raised bumps which are not going away. But are getting better but ha s been over four weeks. I have been given steriod lotions. That is working but very slow. I am not sure what i can do. Can you give me some advice. Thanks Brenda Doctor: Hi Dear,Understanding your concern. As per your query you have symptoms of raised bumps on back,feet and hands which seems to be due to bacterial infection leading to folliculitis and it could be due to allergic reaction. Need not to worry. You should wash area with neutrogena oil free. Apply Adapalene gel once at night. Take antibiotic like azithromycin 500 mg. If condition doesn't improve in a month then consult dermatologist for proper evaluation. Doctor may prescribe hydroquinone cream and retinoids. Wash these areas with mild soap and cold water twice a day , exfoliate your skin once a week , apply aloe vera gel and apply azelaic acid containing cream twice a day. Hope your concern has been resolved.Best Wishes,Dr. Harry Maheshwari"
},
{
"id": 206330,
"tgt": "Suggest therapy for severe vertigo & panic attacks",
"src": "Patient: I've had severe Vertigo for about 2mths straight now,& have never even herd of Vertigo in my life.I've always got car sick & have a fear of fair rides.I've been to the ER,prob about 8-10 times in those 2mths,I've been haveing panic attacks from this as well.The ER did do a catscan & found no tumors or anything,I have no idea what to do,because on top of all of this I have postpartum depression from giving birth 1mth before all of the Vertigo symptoms,& also I have a 1yr old daughter to take care of,this is hell for me right now...& the meds \"Meclizine\"seem to be working less & less,I'm haveing a hard time eating & preventing theses panic attacks that I've also never had before,ugh,what do you reccomend? Doctor: DearWe understand your concernsI went through your details. I suggest you not to worry much. Panic attacks always happen due to underlying anxiety. Therefore you must address the underlying anxiety first. Please understand anxiety and related problems. Once you understand your anxiety, you will be able to keep yourself away from panic creating situations. Therapy always starts with avoiding such situations along with coping techniques when forced to be in such situations. Start slowly, progress gradually, attain neatly.In your case, the vertigo could be anxiety oriented. That is the reason why your ER could not find any problems with your CAT Scan. Believe your ER and live life. If you sus pect, get to another professional neurologist and get your tests done. Once confirmed NAD, then stop worrying about it and consult a psychologist for a therapy.Psychotherapy techniques should suit your requirement. If you require more of my help in this aspect, Please post a direct question to me in this URL. http://goo.gl/aYW2pR. Make sure that you include every minute details possible. I shall prescribe the needed psychotherapy techniques.Hope this answers your query. Available for further clarifications.Good luck."
},
{
"id": 25275,
"tgt": "Why my mother's head is reeling while lying down?",
"src": "Patient: my mother is 60 yr now. She regularly takes medicine for blood pressure. Suddenly since yesterday night, while sleeping, she had a severe head-reeling problm...as if the bed is going up-side down...When she is sitting upright, its ok... but the moment she is lying down, she is having severe type of head-reeling Doctor: Hello and thank you for using HCM.I carefully read your question and I understand your concern.You shoud not worry I'll try to explain you something and give you my opinion. A hypertensive crisis can give symtoms like headache, blurred vision, dizziness, nausea. A person might experience a kind of head reeling but it is not related with head position.In my opinion this symptoms like head reeling, vertigo, strongly related to head position or while lying down have to do with inner ear problem. In inner ear patologyes the patient experience this head reeling, like the house or bad comes over or up side down. They are strongly related with head movement and position becouse in that moment the inner ear takes imupses and controls our eculibre. So, my opinion is that your mother should consult an otorin and a neurologist doctor. There are some simple objective examinations to do and evaluate this problem.But you should not worry.If it is what I suppose there are medications to control this symtoms.Hope I was helpful.Wish your mother good health.Best regards."
},
{
"id": 94409,
"tgt": "Sharp lower abdominal pains, while sneezing also. Reason for pain?",
"src": "Patient: I being having really bad lower abdomal pains and when I sneeze I have being experiencing rather sharp pains in the lower left side.sometimes during the day I also experience sharp pains in my lower abdoman they are really painful and just shot threw my lower abdoman.I really don t now what to make of this.I m female and 20 years old Doctor: Hi. Well during the sneeze intraabdominal pressure is raised and if you have any organ enlargement it can cause pain. Usually it is liver, spleen or uterus. Also stomach disorders such as hiatal hernia can cause such symptoms. Dont be worried , this is nothing serious.It sounds to me like gynaecological story. You should just do lab tests and abdominal+pelvic ultrasound to rule this out. Wish you good health."
},
{
"id": 129448,
"tgt": "What causes dizziness and loss of balance after having humerus and left arm fracture?",
"src": "Patient: I was in a A A 8-2-2016 and suffered a bad neck whiplash? and then I fell out of my s.u.v. 10-3-2016 and broke my humerous, left arm, but I had this enormous bump on the back of my head the size of a goose egg., the hospital ignored this, I have horrible dizzy spells, my head feels like i am swimming and when I get up I almost lose my balance, and am very careful with this NEW thing.,Do I have vestibular? Doctor: Hello, I am Dr. Selmani and read carefully your question.I think that when you felt the second time and had the bump on the back of your had , you had a head trauma that should had been investigated more with imaging , such as Xray, CT scans or MRI of the Head and neck.You may have an hematoma inside your skull that compresses your brain and causes you lost of balance , dizziness in spelling etc. The part of the brain responsible for these functions is exactly in the back part of the head where you had the bump.This signs that you have are not related with the humerus fracture but only with head trauma.I suggest you to see a neurosurgeon or emergency medicine doctor for work up.Hope this information will help."
},
{
"id": 79306,
"tgt": "Is it safe to have one drink during a chest cough?",
"src": "Patient: I am taking hydromet syrup for chest cough. I have a wedding to attend, and would like to have one drink in celebration. How long does it take for the syrup to get out of my system so that having a drink won't make me sick? I took my last does before bed last night around 9:00 p.m. Doctor: Thanks for your question on Health Care Magic. I can understand your concern. Better to avoid drinking alcohol while you are sick or on treatment. Alcohol causes immunosuppression and so chances of infection is high. It also interacts with drugs and so therapeutic effects can be decreased. Hydromet syrup will stay in the body for 16-18 hours. So better to avoid alcohol in your case. Hope I have solved your query. I will be happy to help you further. Wish you good health. Thanks."
},
{
"id": 76561,
"tgt": "Is it to be concerned about the cough along with breathlessness and chest pain?",
"src": "Patient: I have a deep wet coughing fits that lead to shortness of breath and chest pain about once every 20 minutes. I am coughing up rusty green phlegm and can feel a lot congestion in my upper chest. When I breath normally with my mouth open I can hear and wheezing between my collar bone and my sternum. I've been running a mild fever for about two days now. Since this morning I've had shoulder blade pain on my left side and visible veins running across my chest and both shoulders. I have very mild nasal congestion and no soar throat. I quite smoking cigarettes 4 weeks ago. I'm concerned about the visible veins. My fever hasn't been running high enough I don't think for that to be the reason for it. Any ideas? Should I see a physician? Doctor: Hi thanks for contacting HCM...Here according to history you have lower respiratory tract infection like bronchiolitis....Second possibility is about bronchopneumonia.....So here first chest x ray done.....to rule out cause...Sputum examination also done for gram stain and afb stain...Mostly it is bacterial infection....Take cefixime antibiotic for 5 days....More hot water as they help in expectoration to coughed out.....Excess fatty food and spicy food avoided as they aggravate phlegm...If needed decongestent like antihistaminic drug taken....Hope your concern solved...You can consult pulmonologist for auscultation...Take care.Dr.Parth"
},
{
"id": 47894,
"tgt": "What is the treatment for kidney pain?",
"src": "Patient: I have pain that feels like a fist pushing and turning my left kidney. I did have 2 stones removed surgically from the left kidney 7 years ago, but the pain isn t shooting down, it s just the horrible knotting burning feeling. Could a stone be caught in it? Doctor: hello, yes it could be a stone which body is trying to push down but its either big to move down or it is stuck. Its not necessary that there will be shooting pain or radiating pain downwards. It can be just a dull aching pain.As you already have history of having renal stones, it might be a possibility that you have got it again. I would suggest you to get repeat USG scan of abdomen done. And also after this treatment consult a dietitian for recurrent occurring renal stones. Thank you"
},
{
"id": 52618,
"tgt": "Is my ESR normal at 75?",
"src": "Patient: my esr is 75 , i am a female of 55 years old. is my esr normal?hi, my baby boy is 1 & 1/2 months old . presently plenty of pus cell detected in its tool n baby used to continue 4 to 8 times bowel movements from last few days . please suggest what is the problem with my baby and wht precautions and medicine should be given to my baby now ... Doctor: Hello and Welcome to \u2018Ask A Doctor\u2019 service.I have reviewed your query and here is my advice.ESR raised in any inflammation or infective or malignant condition. Here enterocolitis seems more likely from bacterial infection as pus cells present in stool.Stool culture should be done to choose appropriate antibiotic and to find causative organism.In infant and young children otherwise viral gut infection common as compared to bacteria. Consult a pediatrician for the examination.Hope I have answered your query. Let me know if I can assist you further.Regards,Dr. Parth Goswami"
},
{
"id": 75035,
"tgt": "What causes severe shivering at night without fever or cold?",
"src": "Patient: DEAR DOCTOR...IM A 30 YEAR OLD MALE...SOMETIMES I SUFFER FROM SEVERE SHIVERING IN THE MIDDLE OF NIGHT SLEEP FOR NO APPARENT REASON...NO FEVER, NOT FEELING COLD, ITS UNCONTROLLABLE AND LASTS FOR A FEW MINUTES...THE FIRST TIME IT HAPPENED WAS ABOUT 2 YEARS AGO AFTER I CAME BACK FROM MY DENTIST FOR A REMOVAL OF A TOOTH ROUTE WHICH HAD CAUSED ME UNBEARABLE PAIN, SO HE GAVE ME 2 SHOTS OF ANESTHESIA TO STOP THE PAIN IMMEDIATELY...MY MOUTH STAYED NUMBED FOR ALMOST 5 HOURS!! THEN I SHIVERED SEVERELY IN THE MIDDLE OF MY SLEEP....3 MONTHS AGO IT HAPPENED AGAIN, THE WEATHER WAS REAL COLD BUT I DIDNT SUFFER OF ANY DISORDERS EXCEPT FOR SOME \"COME AND GO\" PAIN IN MY CHEST...YESTERDAY I SHIVERED AGAIN IN THE SAME WAY.. I DO HAVE A COLD BUT IT WASNT RELATED TO FEVER....THANKS IN ADVANCE FOR HELPING!!.. Doctor: Hello,In the first case after removal of the tooth root, it was obviously related to pain. In next instances, it`s related to your internal disturbances not external environment at all. There is imbalance going between your soul, subconscious mind and physical body, which is very difficult to understand and many don`t agree with this also, but it`s true. Follow certain steps regularly at home for some period, you will be absolutely fine without any medications, I am confident for it.-Avoid stress or anger in any minor or major issues.- Have regular sound sleep of eight hours by default.-Avoid to watch horror movies or thrilling news before sleeping or avoidtexting or checking emails at least one to two hours before sleep, which is of vital concern in today`s digital era.-Avoid smoking or alcohol or drug abuse if consuming.-Do regular deep breathing and concentration exercises with YOGA for thirty minutes daily which has a tremendous impact on your case particularly.-In occasional cases of a post-traumatic syndrome of past mental trauma, one may need to consult psychiatrist also. But I wish you will not require.Hope I have answered your query. Let me know if I can assist you further.Regards, Dr. Bhagyesh V. Patel"
},
{
"id": 54293,
"tgt": "Is liver palpitations normal post gall bladder removal?",
"src": "Patient: Hi, I had my gall bladder removed 8 days ago and my recovery has gone well so far but today my liver feels enlarged, it is not causing me pain and is only a little tender on palpitation, do you think is just part of the swelling from my surgery or should I get it checked out? Doctor: Hello! Thanks for putting your query in HCM. I am a Gastroenterologist. yes this is not due to sequale after cholecystectomy. You must visit a doctor for evaluationI hope I have answered your query and this will help you . Wish you a good health"
},
{
"id": 139319,
"tgt": "What causes swelling and bruising on shin?",
"src": "Patient: My left leg below my knee is swollen and red with a sheen. My right leg has none of the symptoms. I am 64 305 lbs, diabetes 2, an receiving physical therapy for IT band syndrome on my right side. I noticed the soreness yesterday and I ran a fever yesterday. This evening it the area has gotten bigger. I did take motrin and aleve and tylenol yesterday. I did receive electrode therapy on my right leg during a PT session yesterday. Should I go to urgent care this evening. Doctor: Dear Sir/MadamI have gone through your query and read your symptoms.In my opinion, you seem to have infection, and with your diabetes status this becomes even more of a concern. get blood investigations done and take antibiotics after consultation with your doctor.I hope that answers your query. If your want any more clarification, contact me back.Dr Narender Saini"
},
{
"id": 146007,
"tgt": "What causes dizziness, blurred vision and headache?",
"src": "Patient: While screaming and cheering on our football team my husband felt a pop in his head. He immediately became dizzy and with blurred vision along with hearing only muffled sounds. He got a headache right after that and hasn t felt good since. What should we do? Does he need to go to ER? Doctor: HIWell come to HCMI really appreciate your concern, two possibilities are there one is this could be due to screaming because this caused the lowering of blood pressure, (Hypotension) because of manure, second, cerebral and cardiac condition, for this get done the EKG and CT brain, hope this information helps, take care."
},
{
"id": 139469,
"tgt": "How to treat brain aneurysm in the initial stage?",
"src": "Patient: Hi, Can you please give me some advice. My mother recently passed away as a result of a brain aneurysm and as a result my sister and I asked our respective GPs to have us scanned. My mothers was detected yet untreated for 2 years. I have been advised after a CT scan that I also have one. I have been advised to wait for weeks for an appointment. Doctor: Hello and welcome to \" Ask a Doctor\" service.I have read your query and here is my advice.Controlling blood pressure is essential.So, if you maintain normal blood pressure, risks are minimal.Hope this helps.Feel free to ask if you have further questions."
},
{
"id": 119325,
"tgt": "Blood in stools. Blood test done, shows low hemoglobin. Causes and cure?",
"src": "Patient: i have a problem of blood coming in my stools and a recent blood test done came out that my hemoglobin had gone down to 7.5. what is the reason and what is the cure other than operation Doctor: Hi Mark, I understand your concern. In your case, I see that the chances of having an intestinal worm infestation is more, like round worm, trichuris or ameobiasis infestation. Taking a course of deworming medicines may help you a lot. You can try taking Tablet. Mebendazole for 3 days. It is a prescription drug. Consult your treating doctor regarding the dosage of the drug for you. Low Haemoglobin value may not improve unless these worms are treated. Get a stool routine done to confirm the diagnosis. Other causes can be ruled out by doing a colonoscopy. Do not neglect this symptom and get evaluated early to prevent complications. Meanwhile, consume lot of green leafy vegetables and other food rich in iron like fresh fruits, dates, jaggery. Drink plenty of water. Wish you a speedy recovery. Regards,"
},
{
"id": 225365,
"tgt": "Weight gain and frequent urinary tract infections after getting depo shots. How to treat this?",
"src": "Patient: Hi im savannah. Im 16 years old junior in hs. 5\"4 152pounds:,,( i started depo Feb2013 i was 115 when i started depo. So i gained around 35 pounds. Not only thay but i get uti's every other week yeast infections and bacterial vaginotis. I changed my diet, and exercise alot and no weight loss. Im so depressed and emotional i wanna loose weight and it wont go away:,,,( my doctor said i need ti try harder Doctor: Hi Savannah, You are too young to get depo shots. However, I strongly suggest to consult OG and to be accurately informed over its side effects. Next, weight gain and depression are caused by depo shots. With regards to frequent urinary tract and vaginal infections, I'd suggest to- do urine culture and sensitivity for choosing the best antibiotic treatment- do not be sexually active when on treatment (vaginally or orally)- if you have only one partner, please get him treated as well- if you have more than one partner, please use safety measures (condoms)- do ultrasound of abdomen to rule out kidney or uterus problems.- drink more water- keep good hygiene of your genitalia and anus (wash your vagina first then, anal opening). - always consult OG and/or physician for further evaluation.All the best!Dr.Alba"
},
{
"id": 121907,
"tgt": "What causes severe pain under rib cage during coughing or sneezing?",
"src": "Patient: I have had a tearing pain..under right ribcage when coughing or sneezing! It hurts so bad that I yell out in pain. I can move certain ways, but, not forward. It feels like an abscess when I cough or sneeze. I have SLE and Fibromyalgia never had this symptom before. Doctor: Hello,Your symptoms seem to be related to a muscle strain which can be related to the cough. I suggest using a cough suppressant to relieve the cough. I also suggest using anti inflammatory medications such as Acetaminophen to relieve the pain. Warm compresses can also be very helpful.Hope I have answered your query. Let me know if I can assist you further. Regards, Dr. Dorina Gurabardhi, General & Family Physician"
},
{
"id": 133655,
"tgt": "What causes lost of balance,pain in abdomen and breathlessness?",
"src": "Patient: Hello I need some answers. I m 19 and fit, and I was doing handstands with my friends on the grass and I lost balance and completely bent in half, my back made a major crack sound and I couldn t breathe for a few seconds. Now I have tightness and pain in my back as well as my abdomen and it s hard for me to breath. Should I go to the hospital or just put some muscle cream on help! Doctor: hi,thank you for providing the brief history of you.A thorough neuromuscular assessment is advised along with MRI of lumbar spine.As you have a fall and you completely bend over which made a crack sound I advice you to undergo an MRI of the lumbar spine to see the soft tissue pathology. Also, as you have symptoms like diffiwij breathing, abdominal pain and back ache, MRI should reveal something.Since to understand here is, that the muscles of respiration like the diaphragm and the abdominal muscles are the supportive structure for the connecting the upper and the lower body along with spine. Any injury occuring to the spine will directly affect the respiratory system. Also when one breathes there is a mobility in the abdominal muscles and also the spinal column. So that will be the reason you are facing with breathing issue and pain in the abdomen and back.As post MRI we can come to closer look to the matter and plan the further management. As treatment may include - physical therapy as it is successful in majority of patients .Also, in my clinical practice patients respond well to physical therapy and recovery is achieved at the earliest.RegardsJay Indravadan Patel"
},
{
"id": 19216,
"tgt": "Why do I feel like my heart is being squeezed by someone?",
"src": "Patient: Over the last couple of weeks my heart feels like someone is squeezing it, I then get goose bumps and sweats, and sick to my stomach.. last week it happened at nite and was extremely painful, I am a 39 yr old male and have high cholesterol, any help thanks Doctor: Hello, Most probably an acidity, gastritis and reflux disease. Do you also have upper abdominal pain, nausea, bloating, burping, increase in pain on food, sour water feeling in throat or chest burning, these are the other symptoms. Still there is no harm in getting ECG and stress done to rule out cardiac cause. You should avoid fatty, oily and spicy diet. Have some walk after having food instead of taking rest. Have multiple small meals instead of heavy meals. Have regular sleep habits and avoid stress. Avoid smoking and alcohol if any. You can get prescribed tab Pan DSR 40 mg or Esomiprazole Domperidone combination before breakfast once a day for 2 weeks. If not adequately relieved, then you should add Gelusil two teaspoon three times a day for a week. Hope this helps you Get back if you have any doubts Regards Dr. Sagar Makode, Cardiologist"
},
{
"id": 70838,
"tgt": "Suggest treatment for breathing problems",
"src": "Patient: Hello Doctor, my son 8.5 yrs old suffering from breathing problem from last 7.5 yrs. He had undergone adenoids and tonsils operation 2 yrs back. Still he raises his head every now & then for gasping. He can not stand straight even for 0.5 minute , he needs to bend his neck in backward direction to reduce congection in his neck. He always complaint of having pain in the neck near throat. I have shown this to many doctors, MRI scanning of his brain , barium swallow tests is done, neurological test and sight tests are also done All tests and reports are normal. My son is living normal life even having this big health problem and he never complains about it. We have stopped showing this to the doctors since it was not getting diagnosed by anyone! But now a days frequency of his raising head for gasping has been drastically increased. He is in 3rd std. now. Though is bright student at school , he losses concentarion because of this health problem. Please help Regards, Kishor Sonawane 0000 Doctor: Hi, As you explain the history probably if I were his doctor in my opinion asthma or bronchitis should be excluded as a potential cause of his breathlessness. Please discuss with his doctor for this. Hope I have answered your query. Let me know if I can assist you further. Regards, Dr. Jnikolla, Pulmonologist"
},
{
"id": 9136,
"tgt": "What remedy is there for pimples and black marks on my face ?",
"src": "Patient: i m 26 years old female i use to get lot of pimples on my face .. after a week when pimples goes its leaves a black mark on my face.. mine is very fair and oily skin this black marks and pimples looks very horrible.. for the past one week i m applying sandal and turmeric paste and leaving it for whole night.. please suggest me some remedy for permanent effect Doctor: Hi!, Welcome to HealthcareMagic forum, pimples have become a life style disease & a disease of metropolitan cities...we hardly see them in villages.But can also be because of hormonal disturbances in our body,esp if it increases 2 days prior to periods.Never use a moisturizing cream on face.Avoid using oily soaps too.Use specific anti acne soap like acnelak or dermadew acne soap twice a day.Use nadoxin cream over pimples in day time and adaferin at night.This might help you a lot. Now a days there are very good tablets available for pimples but should be taken under dermatologists supervision only.They can be either azithromycin or isotretinoin tab . Tab azithromycin (500mg) once daily for 3 days(take this tab 1 hr before food).Hormonal pimples can be treated with oral contraceptive pills .They are very effective and safe.All these needs blood test before taking them. For marks use retino A (0.025%) cream at night only on the spot for 2 hrs then wash off . Don\u00e2\u0080\u0099t forget to use a water based sunscreen like UVAVO daily full face so as to protect yourself from direct sunlight which ca increase your dark marks Along with this u need to take 10-12 glasses of water per day & a good sleep. .A chemical peel can also help reduce the marks .Get it done from an expert dermatologist only. regards, Dr.Chawda"
},
{
"id": 8317,
"tgt": "How can facial darkening be treated to achieve whiteness?",
"src": "Patient: Hello sir.. i ****** aged 22 years old, at child i am white but came 2 age 16 years ,then my face turn to black color..What is the permanant solution for turn my face into white.. Is there any 100% result oriented creams ,which doesnt show any side effects.. I consult a dermatologist but i am nt satisfied bcoz,if we r stoping the using creams,tab..then automaticaly turm my face into original color.. Technology improved so lot,bt no one founded the good fairness cream, which doesnt show any side effects.. Is there any ayurvedic product creams for whitening face.. I heard about meldera cream ..is it best or not..? Doctor: Hello, Thank you for posting on HCM.Complexion of any person is has two two components; constitutive, one which is predetermined by genetics and racial factors and usually cannot be changed with any treatment. The other component is due to external agents like environmental exposure, sunlight, use of any cosmetics etc and is amenable to treatment.You can get a series of chemical peels, which actually remove the upper dead layers of skin and stimulate regeneration of collagen and thus newer layers of skin. Many options are available in chemical peels depending on the requirement and skin type. (Glycolic peel, retinol peel, cocktail peel etc)Also, chemical peeling can be preceded by a simple procedure called microderamabrasion or commonly known as skin polishing to augment the results.Also, at home you can try using cream containing kojic acid, tetrahydrocurcumin, arbutin etc for several months (Melaglow cream) for additional benefit and maintenance.Always use broad spectrum sunscreen before going out in sun.Use a plain moisturiser like cetaphil DAM on daily basis for healthy and hydrated skin.Take plenty of fresh fruits and vegetables in your daily diet.Hope this will take care of your issue.Thank youDr Hardik Pitroda"
},
{
"id": 139079,
"tgt": "What is the dent and tenderness in head after an injury?",
"src": "Patient: My 2 year old fell and hit heard head on upper side of forehead it caused a little swelling and dent ever since that day I put a lot of ice on that area it was about a week ago will that area heal and the dent go away? when she looks up I can still see the dent and a tender bump Doctor: Hi, your concern is the dent, which usually form when the hematoma from the center dissolves first, in all circumstances the rest will also go. but watch if the child shows any symptoms as unconsiousness, vomiting or fits, in that case ER consultation is a must.Happy healing, and give my love to the child."
},
{
"id": 211316,
"tgt": "What is the treatment for no sex drive, bad memory, light headedness and reduction in emotions?",
"src": "Patient: promethazine25 ,citalopram 40mg ,metoprolol tartate 50mg ,accupril 10mg , methylphenidate ,perphenazine/amitrip 2mg//25mg , dilaudid 8 mg drug interaction concerns? potential side affects? to include:::??:: NO energy! terrible memory! light headedness {enough to have to go to my knees so I dont fall ... which i gave gone down HARD on a few occasions} NO sex drive, significant reduction in emotions of any kind {the spectrum} can you adress any of my ????s Doctor: HIThank for asking to HCMYou are taking multiple drugs then it is very much likely that certain drugs among them really reduces the libido and will also bring the emotional stress too, I would advise you to have word with the doctor, good luck."
},
{
"id": 186338,
"tgt": "What is the cause and solution for hard white stuff on teeth?",
"src": "Patient: Hi around my front bottom 4 teeth is a lot of hard white stuff! I'm assuming it's plaque.. I've tried removing it by brushing a few times a day and flossing a lot but it doesn't seem to be shifting! It's not so much Painfull but very uncomfortable and I'm worried my teeth are going to fall out? I'm on the nhs and get free dental treatment but my dentist Will only offer me a scrape and clean for 70 pound what I can't afford? What can I do? Doctor: Hello, Welcome Thanks for consulting HCM, I have gone through your query, as you have white stuff in front teeth , it can be dental plaque dont worry go for Oral prophylaxis if possible. Do brushing twice daily , Do chlorhexidine mouthwash 2 daily .Use tongue cleaner to clean your tongue .Hope this will help you. Wishing you good heath ."
},
{
"id": 132424,
"tgt": "What could MRI showing bucket handle tear of medial meniscus, tricompartmental chondromalacia of patella suggest?",
"src": "Patient: An MRI discovered a bucket-handle tear involvoing the body and posterior horn of the medial meniscus, and tricompartmental chondromalacia most severe involving the median ridge and lateral facet of the patella. The lateral meniscus and cruciate collateral ligaments are intact. What are my options? Doctor: Your options depends on the problem you are experiencing right now. when was the injury to your knee? is there any swelling, locking episodes, instability that you are having right now? You may end up requiring arthroscopic surgery for the same if you have problem that is persisting."
},
{
"id": 186308,
"tgt": "What does tiny black spot on bottom gum indicate?",
"src": "Patient: I just had a tooth pulled last week, upper. It was added onto my upper partial. Now I noticed I have a tiny black spot on my bottom gum. Never had it before. Is it because of my dental work recently or could it be a form of oral cancer? I do have a history of skin cancer. Thanks, Diane Doctor: Hello, Thank you for consulting with HCM.This black spot which is present on the bottom gum can be because of this upper partial denture traumatizing the lower gum.Better that once you should visit your dentist again and get your partial denture corrected.Hope it will help you."
},
{
"id": 172520,
"tgt": "How to treat loose motion in a 4 months old child?",
"src": "Patient: Hi my daughter is 4 months old. Since last 10days she is having loose motions. We've done stool test 5days back & prescribed by a doctor for Zifi100 Dry Syrup. Since last 4days we've fed her with milk food and medicine (Zifi100) as advised by doctor howevr no improvement is observed. Today when i checked Zifi100 on web i realised that it is best remedies for few other disease but not for gastrointestinal infections. Doctor has advised Norflox100 DT as an alternative now, but when i searched this on internet i found that it is not recommended safe for infants. ..the comments on stool test report says she's NUMEROUS INCLUMPS IN MUCUS....Please advise best medicines....QUERY FROM INDIA.. Doctor: Zifi works well for gastroenteritis when caused by bacterial infection.If the infection is viral which is the most common cause then Zifi itself can cause loose stools. In addition this baby is most likely having transient lactose intolerance due to viral diarrhoea.Please stop the antibiotic and keep the baby on strict breast feeding. Get stool check done for reducing substances,and if strongly positive,can change milk to a lactose free formula temporarily."
},
{
"id": 62005,
"tgt": "What causes lump on testicle?",
"src": "Patient: When I was around 12 I unjured a testicle and developed a lump attaced to my testicle. I am now 56. I have a dull pain in my scrotum from time to time but believe this is due to my vascectomy. How often should I get this lump checked out? Doctor: Hi,Dear,Welcome with your query to HCM.Studied your query in full depth of its details.Reviewed it in context of your health concerns.Based On the facts, You mostly seem to suffer from-cyst of the epididymis testicle/ or it could be resolved hematoma post-trauma to the testes in tunic vaginalis./ or could be hematoma with funiculitis.USG study would fix the diagnosis from other cause of such a lump on testicle after trauma.Vasectomy related lump could also be there.For your age recheck every year to check for the seminoma of testes. IF suspected -USG Of the testes would fix the cause.This reply would reduce your scare, hope so.This reply would help you to plan further treatment soon with your treating doctors.Best of Luck and early recovery.Welcome any further query in this regard,which would be replied in next session.Good Day!!Dr.Savaskar M.N.Senior Surgical SpecialistM.S.Genl-CVTS"
},
{
"id": 17423,
"tgt": "What causes thumping heart beat with the feeling of blood rush in head?",
"src": "Patient: I can feel my blood pressure rising and it feels like all the blood is rushing to my head. I can t even hear because the pounding is so loud. I feel sick about 25% of the day because of this. I m also having some bloody discharge and cramping. Is that related? Should I get BP medicine? Doctor: Hello, I understand your concern and would explain that your symptoms could be related to anxiety. Anyway, I recommend you to closely monitor blood pressure values in the next days (in sitting position after 10 minutes of total relax) and take note of those values. If your blood pressure values are above the normal ranges (125/85mmHg) most of the time, I would recommend starting antihypertensive therapy. I would also recommend checking thyroid hormone levels in order to exclude possible thyroid gland dysfunction. Hope I have answered your query. Let me know if I can assist you further. Regards, Dr. Ilir Sharka, Cardiologist"
},
{
"id": 158265,
"tgt": "suffering with colon CA stage4, metastasis reached bladder, had chemo cycles, has blood in urine. Cause?",
"src": "Patient: My brother with Colon CA stage 4, recently found out that metastasis has reached his bladder . He has had 4 chemo cycles, the last cycle included target chemo on bladder. It has been 2 weeks since then. He still urinates with blood but this morning he urinated with bright red blood dripping after urination. Is this part of the manifestations of his bladder metastasis? Or could it be something else? Doctor: Hello,Thanks for your query.Passing blood in the urine may be due to many causes. But mostly it might be due to bladder tumors and kidney tumors. sometimes due to stone disease and also after chemotherapy (cystitis). You had already mentioned that your brother is having colon cancer infiltrating into bladder. So now his problem of blood in the urine is mostly due to this infiltration. The chances of new bladder tumor to occur is rare but still may occur. you can get an ultrasound of the abdomen to see if there are any other lesions in the bladder apart from this infiltration. If there are any other lesions then they an be further worked up.Good luck."
},
{
"id": 191587,
"tgt": "Does scarred pancreas have any impact on the life span of a person?",
"src": "Patient: I was diagnosed with having a scarred pancreas a few weeks ago. I was diagnosed with Type 2 diabetes in 2004. It was well under control until just recently. I started taking insulin on May 17th, currently 50 units per day. The diabetes is now back under control. I am a male, 67 years old. I understand that If the diabetes continues to be under control, I will likely have a normal life span. My question is whether or not the pancreas disease will impact my life span? Thanks Doctor: Yes a scarred pancreas means you will always require insulin. But if you keep Diabetes under control, you will have a normal life span--regards"
},
{
"id": 195387,
"tgt": "What causes lumps at the base of the penis?",
"src": "Patient: On the bottom side of my penis, I believe the tube that lets men urine/release sperm.. there are a few bumps on the sides of the outlining of that \"tube\". I have looked at them closely before.. and I don't believe they have grown any. I also looked up pictures of genital warts and Those are ruled out. What could this be? Doctor: Hello and Welcome to \u2018Ask A Doctor\u2019 service. I have reviewed your query and here is my advice. Dear bumps at the base of the penis may be a folliculitis or boils or eczema or sebaceous cysts or Herpes etc. Until examination is done it is difficult to say what it is. Don't worry just please consult your dermatologist he will examine and treat you accordingly. Hope I have answered your query. Let me know if I can assist you further."
},
{
"id": 41392,
"tgt": "Suggest treatment for infertility problem",
"src": "Patient: Hello doctor, I am 26 years old. Its has been 1.5 years of our marriage. Am trying to conceive since last 5 months, but the result is negative. I never took contraceptive pills. My husband used condom for 1 year and then from last 5 months we are planning.This month i had my Pelvic scan done as per doctors advice. PCOD has been detected. She has asked me to take susten 200 mg from 14th day of my last period start date and continue till next 12 days. And Siphene 50mg from 2nd day of my periods for 5 days. I am confused, is the treatment going in right direction? Doctor: Hi,Welcome to HealthcareMagic .Yes your treatment is going properly. When PCOD is diagnosed usually siphene is given to promote ovulation. This helps in ovulation and hence conception.Susten is given to maintain initial pregnancy if you conceive with siphene.If this fails for 2 to 3 cycles then IUI may be started . It is Intrauterine ingestion of sperms. This further increases chances of pregnancy . Hope I have answered your query.RegardsDr Deepika Patil"
},
{
"id": 36917,
"tgt": "Suggest treatment for tuberculosis",
"src": "Patient: hi doctor i am R.S mehta my wife is ivf patient dr wil found tb pcr detected but she uses medcin AKT 4 BEVON capsules .aprox ten weeks but again test result tb pcr detected.so whate i will do now can you give me advice dr advice me again use tow month medicen .AKT 3 ,VEVON AND BENADON 40MG this is right or no please .advice me also i want know whate is pcr thank you Doctor: Hi and thank you so much for this query.I am so sorry to hear about this TB that your wife has been battling with. I will like to know if she is feeling better since beginning treatment of TB? If she has not been feeling better then she should be evaluated for possible multidrug resistance TB and treated accordingly. If she has been feeling better, I suggest you continue the treatment foe up to 6months minimum following the recommended treatment plan. PCR is too sensitive and would clearly not be negative shortly after starting treatment and that's why this treatment goes on for at least 6months.I hope this helps. I wish you well. Thank you so much for using our services and feel free to ask for more information and clarifications if need be."
},
{
"id": 77159,
"tgt": "Suggest remedy for sinus infection& bronchitis",
"src": "Patient: Doctor prescribed me clarithromycin er 500mg two tablets twice per day. I was diagnosed with a sinus infection and brochitus. I've been on the medicine for 3 days. Now my ear hurts when I bend over. Can I take anything with the medicine the prescribed to help with that. If so what? Doctor: Hi thanks for asking question.Noted here you are having sinusitis plus bronchitis.You have reffered ear pain...You can take steam inhalation with eucaltptus for proper drainage of sinus.Salt water gargle can done.Excess spicy and fatty food avoided.Take cetrizine daily....Fruits and green leafy veg taken more.Take rest....Hope your concern solved....Dr.Parth"
},
{
"id": 148529,
"tgt": "Cough, weakness, tingling hands. Could this be an exacerbation of multiple sclerosis?",
"src": "Patient: I am getting over bronchitis & still have the awful cough left behind. It s been 2 weeks of that. Now that I ve finished my ABX I feel awful. Weak. Spasms. Tingling hands and coordination is off. I have insomnia also. I take oxycodone for the pain for my multiple sclerosis . My question is... Could this be an exsterbation of my MS because I m getting over being sick. I normally don t sleep. But now all I want to do is sleep. Is this an exsterbation? What can I do to make myself feel better also. Diagnosed is 2009. Symptoms of MS since 2001. I have 5 braib lesions. my nuero is dr jeff bradley an MS specialist that also has MS andvsime days he is great ither days he uses his electric wheelchair. Please help. 32 yr old female. 2 kids. Daughter is 5 son is 3. I do have great support. My lifelong partner of 9 years now and my mom & dad. But I also want to know are there programs I can attend or online groups because my family does help me... But I believe it will be better for me to have some people in my life that truly understand what I m going thru. Thank you, Ryen E Doctor: Dear I am sorry for the symptoms you are now experiencing.Anyway those symptoms are not related to exsterbitacioning of MS.MS treatment cause immunodeficiency because that's the purpose of treatment so the patient is more fragile to viroses or infections.Wish you all the best dont hesitate to contact for further inquiry"
},
{
"id": 224268,
"tgt": "Is it normal to have numb feeling in leg while taking birth control?",
"src": "Patient: i have been on birth control for about a year now, recently i changed bc because of my mood swings. lately before going to bed i will move around you know. and for the last three months my left leg will have a numb feeling to it, so of course i move it around. it only take a few moment for it to go back to normal. i was wondering if this is normal or i should talk to my doctor? Doctor: Hi dear and thanks for your query.As you may know the main side effect by the use of birth control is deep vein trombosis ,almost in predisposed women .Hence while you have one year now ,on birth control and have numb feeling on your left leg ( is a sign of deep vein trombosis),switch of birth control and contact with your follow -up doctor.All the best"
},
{
"id": 218037,
"tgt": "What is the numbness in my shins with pain?",
"src": "Patient: For a few years, I have experienced increasing numbness in my toes and legs, meaning the numbness is creeping upward. Now my shins are numb and for the first time, I have pain. It feels as if someone smacked my shins with a two-by-four. At times. My walking is clumsy and after sitting or driving, I have real difficulty walking. I often favor my right leg. I am afraid I will lose the ability to walk. I am not diabetic, always been healthy. I am 71, widow, three children, weight, 145, retired English teacher. I have recently had blood work, and everything is okay. I can t spell it, but cholesterol is fine. I go to the gym for aerobics, as in Silver Sneakers, when I have the chance. I take my dog for a walk around the lake, one and one third miles, when it is half-way nice outside. Please help. Doctor: It appears that you have \"peripheral neuropathy\"- this is a condition affecting the nerves that can present as numbness and sometimes as pain. Here are my recommendations1. Diabetes is a cause of peripheral neuropathy. Since you do not have diabetes, other possibilities need to be examined including deficiency of other vitamins like vitamin B12. Other less common conditions include rheumatologic (i.e. immune system related) conditions. The diagnosis of B12 deficiency can be made with a blood test while that of rheumatologic conditions requires examination by a doctor and blood tests and Xrays. 2. Sometimes specialized tests like \"nerve conduction studies\" and \"electromyograms\" may be needed to determine the cause of these symptoms. These tests are usually ordered and interpreted by a Neurologist3. I would recommend that you see a neurologist as these tests are specialized tests that would need to be ordered and interpreted by a physician. Please let me know if that was helpful. Thanks!"
},
{
"id": 184936,
"tgt": "Suggest medication for swelling and irritation on tongue",
"src": "Patient: hi i am 18 female 5'9'' and weigh 200 punds . my tongue hurts when i apply pressure to it and some tiny taste buds ar swollen. the taste buds on the back of my tongue are also swollen and i am having trouble swallowing and feel like there is mucus in my throat Doctor: Its due to infection of taste buds..use tandum mouth wash and vit. B complex tablet for 2 Weeks.if more pain..take diclo 50 Mg tb. Later go for a check up by a dentist"
},
{
"id": 95973,
"tgt": "I got mucus in the diaper of my 2 months old baby",
"src": "Patient: my baby is 2 months old and is consipated... He farted while I changed his diaper and it was just clear mucus.. Is that okay... I m stressed out about it.. Doctor: Hi, Your baby is quit O.K. But you not maintion how many times in a day he do so. Flatus may be the cause of mucus discharge."
},
{
"id": 6325,
"tgt": "Trying to conceive, both tubes open, no endometrosis or fibroids, regular cycles, clomid and hcg, seman analysis report",
"src": "Patient: me and my husband are trying to concieve for past 2 years with no success. hsg report says that both my tubes are open and i am not suffering from endometrosis or fibroids . my cycles are regular 29-33 days. My doctor put me on clomid and hcg last month. my husbands seman analysis report is 78 million sperms, 40 % live and 12 % progressive. what should we do? Doctor: Hi Jollu. Thanks for the query. Since you appear not to have any problem with your reproductive system as of now, your husband's sperm appears a bit weak. Normally more than 30% live with motility and 25% should have progressive motility. He needs to eat plenty of fresh fruits and green leafy vegetables, stop smoking and drinking alcohol. Also taking tablet Paternia once a day will help. There may be some hormonal disturbance which is the reason you will be on hCG treatment. This will bring back required reproductive hormones for pregnancy to occur. Clomid will increase chances of fertilization and pregnancy as well. Kindly follow up with your Doctor and continue with the treatment. Wishing you good luck and good health. Regards,"
},
{
"id": 171776,
"tgt": "Advice required for a week long cough, rashes on body and reduced appetite",
"src": "Patient: hello my 8 year old son has had a cough for about a week which i had to treat with his inhalers he also complained of headaches vomitted once and has npw come out in a rash on his face back and legs...he suffers from chronic lung disease due to being born at 27 weeks..hes drinking lots of fluids and has a small appetite... Doctor: Hi... by what you quote I feel that this could be a viral illness which is presenting itself like this Rash. This cough and cold could also be because of the same viral infection. I suggest you to continue the inhaler which you are currently using. I also suggest you to get the kid examined by your pediatrician. I am telling this keeping in mind your kids chronic lung disease.Regards - Dr. Sumanth"
},
{
"id": 120614,
"tgt": "What is the prognosis for a C5 fracture?",
"src": "Patient: What is the prognosis of someone with a C5 fracture? The fracture was bad, C5 was in pieces and later C4-C7 were fused. I understand that SCI s are all very different, but I am wondering if this person could ever walk again? What about vent-dependency? Doctor: Hello, Please correct me if i am wrong in understanding your situation.It looks that affected person is on supported ventilation and was operated for comminnuted fracture C5 vertebrae. It also looks that he had severe injury and compression or disruption of continuity of spinal cord. Our respiratory centers have supply from C3 TO C6 cord segments..If the present situation is due to edema of cord then situation will perfectly resolve with time but if there is complete disruption of cord then situation goes towards the dependency on mechanical ventilation more. Although medical science is developing day by day so you should be optimistic for future.In my practice i have seen some patients improving upto a good level.At present he/she needs supportive treatment with good physiotherapy. Hope I have answered your query. Let me know if I can assist you further. Take care Regards, Dr. Mukesh Tiwari"
},
{
"id": 107600,
"tgt": "Suggest treatment for severe lower back pain",
"src": "Patient: Hello, I woke up this morning with two bruises on my lower back one on each side. I have no idea where they came from and it is very painful. My husband had to help me out of bed because it hurts to move too much. It even hurts to sit.. its most comfortable to lay down on my side. I havent fallen or been hit at all recently. Is this something i should be concerned about dealing with tonight or can it wait until morning? This pain gets unreal. I do have some disks messed up in my back. The report says noncompressive disk bulging t11-t12 and t12-l1 . minimal but broad based disk protrusion l5-s1 this is in close proximity to s1 nerve roots bilaterally without definite deviation or compression. Help? Doctor: Hello, I have studied your case.As you\u2019re MRI says L 5 S 1 disc then Due to compression of this nerve root there can be lower back pain associated with it leading to walking difficulty.Medication like methylcobalamine with neurotropic like pregabalin will reduce pain; you can take them consulting your treating doctor.You may consult physiotherapist for further guidance. He may start TENS, or ultrasound which is helpful in your case.I will advise to check your vit B12 and vit D3 level.MRI shows disc compressing on nerve root then surgical decompression is permanent solution. You can send your MRI films or report. So that I can help you better.Hope this answers your query. If you have additional questions or follow up queries then please do not hesitate in writing to us. I will be happy to answer your queries. Take care."
},
{
"id": 151740,
"tgt": "What is the cause of persistent headaches and retrobulbar pain with numbness and loss of memory ?",
"src": "Patient: i hav been suffering from severe persistent headaches for last six months and retro-bulbar pain. my eye sight is negative 2.5....... there r no refractive errors........ i have felt some loss in my memory .......... my feelings and sensations all hav become dead...........i remain lost all the times........... Doctor: Hi,welcome to HCM, Consult Neuro physician and get your problem evaluated. You may require CTScan,fundoscopy and through neurological examination. After all reports go for treatment accordingly. Ok and bye."
},
{
"id": 31990,
"tgt": "Suggest treatment for herpes infection",
"src": "Patient: mam i am suffering from herpis infection.Got all my test done and the results were positive.the infection remain under control till i have 4oomg dose of accuvir tab but for long run as soon as ishift to 200mg it start reocurring.Though i have changed my doctor but it didn't helped.Suggest me sumthing. Doctor: Hi..Welcome to HEALTHCARE MAGIC..I have gone through your query and can understand your concerns..As per your complain Herpes is a recurring infection and recurring episodes commonly occurs in individuals who have low immunity states like any debilitating disease, stress, diabetes etc..A long term course of antiviral medication can help in prevention of recurring episodes and along with it you should also take immunity boosters like Vitamin C supplements, antioxidants, zinc and lysine supplements..You should also avoid intimate contact with other individuals who are suffering from Herpes although may not have any erupting lesions presently.Hope this information helps..Thanks and regards.Dr.Honey Nandwani Arora."
},
{
"id": 61830,
"tgt": "Suggest remedy for lump on shoulder blades",
"src": "Patient: My little girl has low mucssel tone,and I have found a fatty lump on her upper back in between shoulder blades she loses balance a lot and she can go very pal and weak towards the end of the day she wakes up crying like something is causing her discomfort in my eyes she seems to be getting worse Doctor: hi.it is best if you consult with a doctor, preferably a general surgeon, for medical and physical examination. based from your description, it could be a lipoma (fatty deposit). these lesions can occur anywhere in the body and have the tendency to recur. other types of lesions must also be ruled-out. diagnostics (such as x-ray or ct-scan) and management (medical and surgical) will be directed accordingly.hope this helps.good day!!~dr.kaye"
},
{
"id": 3903,
"tgt": "How many cycles of fertyl tablets to be taken to conceive?",
"src": "Patient: hi doctor ...my periods are regular...my doc adv me to take fertyl100mg...but i got my periods..how many cycles of fertyl tablets to be taken to conceive?my period is normal on 1st day..but on day 2 its so light.my doc adv me to repeat fertyl tablets this month also..plz advise me... Doctor: Hi dear and thanks for your query.Recommendations are that you can stay on Clomid for a maximum of six months.But you should consult with your gynecologist or fertility specialist to do another examination before you continue with fertyl.All the best"
},
{
"id": 108242,
"tgt": "What causes severe pain and stiffness in the back?",
"src": "Patient: Hi Doctor, My name is nirav and i'm suffering severe pain and stiffness in the left side of lower back. The pain is like anything for me very painfull but, its there on the one side i.e. on left side of the back. because of that daily i got acidity related stomach problems and this is i'm suffering from last 6 years. Please sugges me what should i do as all reports related to blood and joints are proper. Doctor: DO U HAVE PROB OF CONSTIPATION... U POST ALL THE INVESTIGATION U UNDERWENT..... U HAVE WRITTEN NOTHING FOR A DISEASE U R SUFFERING FROM 6 YRS...AS U SAY.."
},
{
"id": 156433,
"tgt": "What is the treatment for thyroid cancer?",
"src": "Patient: Hi, my daugher age 47 had thyroid cancer and everything removed in Feb. 2013. Went yesterday and had a follow up ultra sound, and they found a nodule on left side. They are calling this Metastatic Disease, so they will do a needle biospy next Wednesday. I am so worried, for she has 3 little boys that need her so. Two of her boys are special needs, so they need there mommy. Praying that this is going to be all good. Thank you for your time, Kathleen Doctor: You have not mentioned the type of thyroid cancer. Well differentiated papillary, follicular or anaplastic or medullary?Where is the nodule located on relapse now. Thyroid bed or lungs etc?Was the earlier surgery a total thyroidectomy? Was it followed by radioactive iodine ablation therapy?To give you a clear answer I need to know the details."
},
{
"id": 167114,
"tgt": "Suggest treatment for elevated liver enzymes in a child",
"src": "Patient: I am writing this for my 19 months old grant daughter who undergone a surgary for Bilary Atresia on 21st Jan 2010. She was undrer medication UDCA 25mg syrup besides vitamin suppliments. In Nov 2010 doctors discontinued all the medicines because the blood test report was good. Bilurubin level was very normal. Now her SGOTlevel is 146, SGPT 217 and Alkaline 464 which are three times more than what was on 17th Feb 2011. Bilirubin and other blood results are normal. Kindly advise what we have to do. Thanks. M.O.Cheriyan Doctor: HI...nothing to worry about this if the bilirubin and other parameters are normal. More important the blood tests is the kids general appearance, growth and development. If this is normal, nothing to worry about this.Regards - Dr. Sumanth"
},
{
"id": 44585,
"tgt": "How many days should I take COQ, M-Fertyl, Siotone to increase my sperm count ?",
"src": "Patient: I am using Tablets: COQ+M-Fertyl+Siotone to increase the sperm count as advised by doctor, wanna to know how much is this beneficial and in how many days it take to increase the count Doctor: Monthly progress checkup needed"
},
{
"id": 85488,
"tgt": "Is it safe take cital syrup for urinary infection after unprotected sex?",
"src": "Patient: gudevening!! i get urine infection everytime i have unprotected sex... to cure it i take cital syrup!! i want to know how can i prevent these frequent urinary infections even after havin unprotected sex??? And how much safte it is to take cital syrup on a frequent basis??? Doctor: Hello, I suggest having protected intercourse. I also suggest doing a urine test and a urine culture to check the bacteria that is causing the infection and the antibiotic that can treat this bacteria. In this way you can take proper treatment. Hope I have answered your query. Let me know if I can assist you further. Take care Regards, Dr Dorina Gurabardhi, General & Family Physician"
},
{
"id": 78001,
"tgt": "Suggest treatment for left sided chest pain",
"src": "Patient: Hi i am a 31 year old female my height is 5 4\" and my weight is 158. I have had severe left sided chest pain for going on almost six years now and i have tried it all. I have had a full heart work up, countless x-rays, acupuncture, physical therapy, narcotic therapy, intercostal nerve block, radio frequency ablation, trial spinal cord stimulator and many other things but nothing has helped my symptoms or situation. i also have some other medical issues i have GERD which i am on meds for, i have autonomic disorder, and had superior mensentary artery syndrome which i had surgery to correct. My symptoms get worse when i lay down or when i swallow or eat anything especially when it is cold. I have had two menomatries done to see if i had espohical spasms but i dont. I just want my life back and anything you could suggest would be helpful because my doctors and advanced pain management locally dont know what else to do. thank you Doctor: Thanks for your question on Health Care Magic. I can understand your concern. In my opinion, your left sided chest pain is mostly due to GERD (gastroesophageal reflux disease) and this GERD is worsen by your stress and anxiety. So better to avoid stress and tension, be relax and calm. Along with drugs, you need to follow these steps for better symptomatic relief in GERD. 1. Avoid stress and tension. 2. Avoid hot and spicy food. Avoid junk food. 3. Avoid large meals, instead take frequent small meals. 4. Go for walk after meals. 5. Keep 2 - 3 pillows under head in the bed to prevent reflux. 6. Quit smoking and alcohol if you have these habits. 7. Loose weight if you are obese. Don't worry , you will be alright. Hope I have solved your query. I will be happy to help you further. Wish you good health. Thanks."
},
{
"id": 180297,
"tgt": "Suggest some nutritious food for baby",
"src": "Patient: Hello, My daughter is 9 months old . She doesn t eat rice. I give her veg/fruit puree and curd with breastmilk. can you suggest some foods what I can give her. She s active but weight 15 lbs. can she have ragi java... egg... food what we eat.... thanks. Doctor: Hello, Your 9 month old daughter should gradually be started on all food items that your family eats. Introduce one food item every 5-7 days. You can give her raagi, sooji, mashed potatoes, mashed banana, khichdi, paratha, chapati besides the pur\u00e9ed fruits and vegetables you are giving her. Regarding egg, give her only the yellow portion till she is an year old. Hope that helps.Regards,Preeti"
},
{
"id": 52511,
"tgt": "Suggest treatment for elevated liver enzymes",
"src": "Patient: Hi, I did a liver function test and found out that the values for SGOT/AST=126 on a scale of up to 40U/L.37c, SGPT/GPT=143 and GGT =129 I had a negative hepatitis B and C result respectively and was placed on livolin for 60 days.Can You please advice me when to go for another Liver function test to see if there is any improvement. I stopped drinking for about six years ago .I equally did a scan and was told by the doctor its a fatty liver condition. Doctor: Hello and welcome to ask a doctor service. I reviewed your query and here is my answer.As your doctor said you have a fatty liver disease, so treating that will bring your tests back to normal. for that you have to avoid fried, roasted and fatty meals. Have a daily exercise for at least 15 minutes. You can repeat your these test after a month to see for a change.hope i answered your question. Feel free to ask if you need further assistance. Thanks."
},
{
"id": 76309,
"tgt": "What does mild nodular hazy densities infiltrates on left upper lobe suggest from x-ray?",
"src": "Patient: hi i am 25 yrs old,5'5,and 54 kilos,.last april we had our anual pa, unfortunately my result turned out that suspiciuos lung density have seen in my xray so they required me to have an apicolordotic view. it turned out to be mild nodular hazy densities infiltrates my left upper lobe and ruled out to be ptb,,ptb class 3 as the doctor said. there is no history in our family with that sickness, but in my job i have a co employee who suffered from it,,my question doc is?how contagious i am at this point in time,how high is the risk that i can pass the bacteria?or what are the factors that can affect or to be consider so that i will be able to pass this. Doctor: Thanks for your question on Healthcare Magic. I can understand your concern. I have gone through the x ray report you have mentioned. This report is suggestive of left upper lobe infection. Most common cause for upper lobe infection is tuberculosis and pneumonia. So better to get done sputum microscopic examination to confirm this infection. This kind of infection spreads through coughing. Cough produces droplets and they can infect others. So better to wear mask when coughing. First 2-3 weeks, person is contagious through coughing. So better to start treatment as soon as possible. After 2-3 weeks you will not contagious to others. Hope I have solved your query. I will be happy to help you further. Wish you good health. Thanks."
},
{
"id": 125161,
"tgt": "How can shoulder injury be treated?",
"src": "Patient: I fell on my shoulder and heard a crunch. That was a week and a half ago. Now I have a bruise on my outer bicep as well as my inner elbow both on my left arm. I am also having trouble sleeping because my arm hurts. But the pain goes away when I am sitting or standing. But immediately comes back when I lay down. Doctor: Hello, Most probably it will be a contusion. As of now you can use analgesics/anti inflammatory combination like aceclofenac/seratiopeptdase for symptomatic relief. It is better to consult an orthopaedician and plan for an MRI scan to rule out ligament or tendon injury. Hope I have answered your query. Let me know if I can assist you further. Regards, Dr. Shinas Hussain, General & Family Physician"
},
{
"id": 71589,
"tgt": "Suggest remedies for recurrent congested cough",
"src": "Patient: Hi my daughter is 8 months old and has laryngomalacia. However, for the last two months she has been congested with cough. Appears to be better for a few days then it starts all over again. We use humidifier and bulb syring constanly. It affects sleep and her feeding at times. Any other suggestions? Thanks, Frustrated mom. Doctor: Hello,As you explain the history in this situation probably a bronchoscopy might help her get better with the secretions. Discuss with her doctor for this.Hope I have answered your query. Let me know if I can assist you further.Regards,Dr. Jnikolla"
},
{
"id": 156364,
"tgt": "Could snake like shiny stool with hard cover dark bean things attached to it be due to recurred colon cancer spread to pelvis, intestines and abdomen?",
"src": "Patient: I had a permanent colostomy due to colon cancer which has now return and spread in pelvis, intestines and abdomen. question. recently having strange snake like shiney stool and hard cover dark bean thingys attached to it. what is this and can it cause me a rash due to my colostomy wafer/bag. I feel like I m having an allergic reaction. and not just to this but to this cancer. Doctor: Since you are having history of colon cancer i would suggest u do colonoscopy test.Also do CT scan of whole abdomen and blood for CEA test.Faecal for occult blood test is also an option.RegardsDR DE"
},
{
"id": 28324,
"tgt": "Feeling unconscious and dizzy, is it due to heart vessel blockage?",
"src": "Patient: my father aged 82 yrs had angioplasty 1 yr back due to triple vessel disease. since last 6 months, he is feeling dizziness and 2 or 3 occasions he even felt down due to this and for 2-3 seconds he was unconscious also. Has this got anything with the heart vessel blockage or this is only vestibular problem? Please recommend any specific department we should visit. Pranab Sarkar, Bokaro, Jharkhand, INDIA Doctor: Hi Mr. Pranab Sarkar,After going through your father's case, I want to know about 1) Presence of diabetes, high blood pressure problem, cholesterol problem if any along with heart problem.2) Current medication going on. 3) Habits like smoking, alcohol. 4) Any recent illness like loose motions, vomiting, excessive sweating, high grade fever.5) Any ear problem (pain, decreased hearing, tinnitus- abnormal sound in ear), throat problem.6) Are symptoms increases after sudden neck movements. I would like to suggest following test for your father-1) ECG electrocardiogram preferably during episode. 2) Holter's monitoring, electrophysiological study of heart. 3) 2DEcho (echocardiography) 4) Blood sugar level during episode, Complete blood count, hemoglobin, kidney function test, thyroid function test. 5) Blood pressure check in supine position and In standing position to know postural hypotension. 6) If all these tests are normal then there may be need of MRI brain with angiography and carotid artery doppler. In your Father's case there may be possibility of1) Cardiac arrhythmia (abnormal electrical activity of heart) - it may be a caused by decreased blood supply to heart (vessel blockage) or metabolic abnormalities (abnormalities of kidney or liver function)2) Postural hypotension.3) Problems related to blood supply of brain. 4) Hypoglycemic episodes (low blood sugar level) more common in patients with diabetes on treatment.If symptoms increases with neck movement or sudden change position (posture) and there is no postural drop in blood pressure on clinical examination in that case postural vertigo is most likely in that case you should consult ENT specialist, otherwise you should consult your cardiologist, though neurological evaluation by neurologist may also required."
},
{
"id": 208415,
"tgt": "Suggest treatment to stop negative thinking",
"src": "Patient: Dear doctor, My name is Mariana. I'm 22 years old. Height : 1.45 metres. Weight : 39.5 kilograms. I haven't go to see any docters. My problem is in my mind. I always think negatif thing. If I do my job (everything such as eat, drink, work, etc) I always try to think positif thing. In the case I can't think positif thing, I will redo again my job, again and again till I can do my job while think the positif thing. As example : I'm printing a document by my computer, while I print it, I think about people which I very hate, so I will reprint the document again. And for the first document I will bring it to rubbish. I have another example, when I wanna to do something job like want to sit, want to write, in the same time I have to think about artist/actor I like. If I can't think about them, I will redo again my job until I can sit and write while I think about the artist/actor. Sometimes I do this without conscious. This make me headache. I also always wash my hand. Maybe I day I can wash my hand more than 30 times. This is very disturb me. And in the morning, when I wake up, my head fulled of negatif things. I think this is not the real me. 15 years ago I never do this things in my live. Could you pls help me to solve my problem? Doctor: Dear Mariana,I went through your query in detail and can understand your distress with your symptoms. You seem to be suffering from Obsessive Compulsive Disorder (OCD) which is causing your symptoms. The repeated thought of having to think of a positive thing or about a person you like while doing a task is the obsession and doing the task again and again till you do so is the resultant compulsion. In addition, you also have obsession of contamination which is resulting in washing and cleaning repeatedly.You should seek a psychiatric consultation for initiating treatment. Treatment with medications like fluvoxamine are effective for OCD. In addition, you can also opt for treatment with cognitive behavior therapy in form of exposure and response prevention. It will consist of sessions in which graded exposure to your obsessions will be made which will help in reducing the occurrence of obsessions.I do hope that i was able to answer your query. Best wishes for speedy recovery."
},
{
"id": 130518,
"tgt": "Is swelling and bruise at an ankle due to a sprain?",
"src": "Patient: I rolled my ankle yesterday morning playing basketball it wasn t from a really high jumped, but it was quick and unexpected. The first day I couldn t really put walking pressure on it today I can, but only on the balls of my feet. It s swelling has gone down a lot, but is still very noticeable I can feel and wiggle my toes and my ankle has a slight burning sensation scale of 1-10 (4) Lower than yesterdays (8) and a pea size bruise in the middle where my chin meets the ankle. What do you think I did to it is it a normal high ankle sprain as it seems or is it more? Doctor: Hi,Since you have a history of trauma and complaining of pain since a long time after the incident there is a chance that you may have mild sprain of your ankle ligaments which commonly causes such pain.* The best way to treat it would be to give rest to the leg for few days and apply an elastic ankle support while walking. this would be sufficient for a mild grade of shear. but if the problem persists for long and pain too increases you need to get thorough examination, as delaying would lead to aggravating the problem.* If the pain is mild get an analgesic like paracetamol prescribed or use counter irritant gels and take rest.* Do not massage rigorously as it causes more damage, report to a doctor if severity increases. Do not worry now that you got an idea of your condition and consulted a doctor you shall be alright soon. Take care.Hope I have answered your query. Let me know if I can assist you further. Best regards,Dr.Suhas Chauhan"
},
{
"id": 206053,
"tgt": "Suggest treatment for sleep apnea",
"src": "Patient: i WAS DIAGNOSED WITH SLEEP APNEA IN LATE OCTOBER. I HAVE BEEN USING MY CPAP MACHINE SINCE THAT TIME. I HAVE HAD A COUGH AND BACK PAIN BETWEEN MY SHOULDER BLADES SINCE. MY HUSBAND AND I GOT A NEW MATTRESS BUT THAT WAS NOT THE PROBLEM. IS THIS A COMMON PROBLEM WITH CPAP MACHINE? Doctor: HelloYes it is a common problem and the pressure needs to be adjusted. Meet your physician.RegardsDr. Srikanth"
},
{
"id": 188405,
"tgt": "Three year old child fractured tooth across the nerve. Necrosis of tooth. Treatment?",
"src": "Patient: My three year old son fractured his tooth across the nerve about a year ago, which lead to necrosis of the tooth. I have realized lately he has been chewing excessively on his clothing and blankets. His gums looks healthy, but I am very concerned there can be an internal issue with his tooth. I took him to an oral specialist about 8 months ago and they stated there was no infection and they would leave the tooth. Doctor: Hello,Thanks for posting on HCM,Your son might not necessarily be suffering from a tooth problem (as most of the time will be painful and rather make him avoid contact with hard objects) but could be suffering from a neurological or psychosocial problem. There are several diseases conditions that could affects muscle and cause them to perform involuntary movements thus excessive chewing. In chewing this is most likely going to be due to a behavioral problem and you can guide him into stopping. You will need to see a pediatrician to assist you with your child.Hope this helps"
},
{
"id": 130375,
"tgt": "Suggest medication for bruise of arm and elbow",
"src": "Patient: I fell over a week ago and badly bruised my arm and elbow. The bruises are faded now but it hurts to try and straiten my arm completely. I can t get arm all the way straight and there is tension right above my elbow bone. I am 38 yrs old and weigh 222 and am in good shape. Do I need to get an MRI or will the issue go away. It has been over 7 days already. Doctor: To go directly with MRI is not a good thought. First get yourself examined by some orthopaedic surgeon. If he feels like, first investigation is always a simple xray, else the injury may also be managed with simple medications and no investigations."
},
{
"id": 98759,
"tgt": "What causes lip lumps with itching and dryness after ingestion?",
"src": "Patient: Hi. I recently went to a buffet and ate crabs with butter. Later that night my top lip was itching, and the next day I had tiny bumps on my lip. That later turned into dryness, so my top lil has been dry. (I ve never had any problems in the past when I ve eaten crabs.) Yesterday, I was eating chips and my lip started to itch again. Today my top lip is dry and looks like I have a carpet burn. It s also slightly bigger than the bottom. I ve tried Vaseline and lip gloss which soothes the dryness but you can still see that it is dry. I don t know what is wrong. Doctor: Hello,I can understand your concern. The symptoms you have described indicate that you have allergy to some substance. It might be something else that you have had during dinner or from the environment that may have caused the reaction. Sometimes it also happens that body develops allergy to a substance that you have been safely exposed to before. I advise you to take antihistamine such as Levocetrizine 5 mg (Levocet) or Loratidine 10 mg (Lorfast) once a day for about 5-7 days. If it is an allergic reaction, it should come under control with this medication. In addition, if you develop this reaction repeatedly in future, it is necessary to find out what substance is acting as allergen to you and then avoid exposure to it. In that case, you should go for a blood test for allergy. However, if the reaction does not come under control or increases in severity, you should visit a dentist or physician to get the lip examined physically and further investigations and treatment.I hope this answer helps you. Thank you for choosing HealthcareMagic. I wish you feel better soon.Best,Dr. Viraj Shah"
},
{
"id": 54594,
"tgt": "Can prolonged Respiridal intake cause elevated SGPT levels? Can Essentiale reduce it?",
"src": "Patient: Hi. I did SGPT test and it turned out to be around 120 which is almost tripple the normal value. I had been taking respiridal consta 37.5 mg for 1.5years and then switched to respiridal 2mg and gradually to 1 mg for 0.5 years. I stopped the medication a week before I had the blood test. I assume that it is the cause of my high SGPT. I saw a doctor and I was was prescribed Essentiale 300 mg 1 pill 3 times a day with Cobal 500 mg one pill a day. From your experience do you think it might go down to normal in a week? PS: I haven't been drinking alcohol for 2 years, and I will be watching my diet for this week according to some advice given on the internet. Doctor: thank you for posting your query.increased SGPT reflects hepatitis.most likely , it might be due to respiridal.however, you may get a viral serology done for Hep B and Hep C .if other causes of hepatitis are ruled out, then the enzymes would return to normal values in around 4 weeks (more or less). so you may recheck your enzyme after 3to4 weeks.abstinence from alcohol is adviced as you are already observing.wish you good health.further queries are welcomed.Health professionals aim to diagnose properly and manage patients according to their limited knowledge. Cure is blessed by the ONE who Created us, whose power and knowledge is unlimited .wish you good health.regards,Dr Tayyab Malik"
},
{
"id": 110578,
"tgt": "How to get rid of lower back and hip pain?",
"src": "Patient: whenever I lie on my back through the night I can't get up. I have to used my hands and upper body to pull myself over onto my side to then fall to the floor, before I can slowly stand up. The pain in my lower back and hips is very sore. when I am up and about it is better, if I lie on my side in bed with a pillow between my legs its okay too but I can never stay that way.. I always end up waking on my back. having more and more sleepless nights now due to the pain getting worse. and I'm only 33. Any ideas what this could be? Doctor: HiThank you for asking HCMI have gone through your query.Your problem is most likely osteochondrosis or trauma to spine if you had it.A physical examination with an Orthopedician will give clue for that and a X ray followed by MRI might be needed to confirm it.For such conditions i treat with muscle relaxants like thiocolchicoside and NSAIDs like Aceclofenac.I also advise muscle strengthening exercises and Physiotherapy along with this.In case of no improvement a surgical correction can be considered.Hope this may help you.Let me know if you have any further query."
},
{
"id": 161297,
"tgt": "What is the treatment for mucus in stool of a baby?",
"src": "Patient: my 7 months old baby was passing mucoid stool for last 15 days..at first i started homeopathy medicine(as its has no side effect) but it doesnt work.So i consult with a pediatrician.He adviced me to give him ciprofloxacin.But on the 3 day of that medication baby started pass blood in his stool.Then dr pescribed me to stop cipro& start cefixim&metronidazole.Now its 3rd day.still baby has blood&mucus in is stool...frequency is more than before..what should i do now? Doctor: Hello, Please go to hospital and get admitted. Child needs urgent ultrasound abdomen to see colitis or intussusception.of infection needs IV antibiotics for 5 days Hope I have answered your query. Let me know if I can assist you further. Take care Regards, Dr Prasanna Lakshmi, Pediatrician"
},
{
"id": 119588,
"tgt": "How to strengthen weak legs while having curvature of spine?",
"src": "Patient: My mother who is 80 years old has very weak legs. This has been going on for some now and it s just getting worst, to the point she can t walk. Her Dr. has been telling her to do exercises with her legs and try to walk more. Mom sits all day in her chair and has for years now. Is there any kind of test maybe that she should have done that might give a reason to why her legs are going out on here. Mom has curvature of the spine real bad and takes pain medicine for that along with anxiety pills. Mom weights in at about 110 pounds. Doctor: Hi, As per your history, it seems like an age related weakness and curvature of spine. Lack of interest is the principle factor responsible for this inactivity. Apart from protein, vitamin and calcium supplements you need to ignite will in your mother. Give her a walker to ambulate if she is not able to do it without aid. She need to do physiotherapy exercises to maintain the power and range of motion of the muscles. Take care. Hope I have answered your question. Let me know if I can assist you further. Regards, Dr. Rohan Shanker Tiwari, Orthopedic Surgeon"
},
{
"id": 99645,
"tgt": "Are allergic medications advisable with running low grade fever?",
"src": "Patient: Hi. I have been running a low grade fever for months now. I am pretty much allergic to everything under the sun, and I haven't really taken allergy medication consistently. I have gone to the Doctor before about my symptoms, got a throat cultre and blood tests, but they all came back negative. should I start taking more allergy medicine consistently? Doctor: Hi, if u suffer from low grade fever since 1 month then there is no use of taking antiallergic drugs thinking that it will subside fever despite fever may be bcyz if allergy.First stop taking unnecessary antiallergic drugs,it is possible that there may be chances of drug induced fever,usually it comes of low grade like 99 to 99.5 as u said u have all routine investigations came normal.Leave your medicines and fever would subside by itself"
},
{
"id": 46785,
"tgt": "Suggest treatment for kidney shrinkage with high triglycerides and swollen pancreas",
"src": "Patient: Hi, I have a question about kidney issues. Brief background: all my life, I ve had chronic stones; 4 years ago, 1 kidney failed; I was just hospitalized at the end of Oct/ beginning of Nov for extremely high triglycerides, very swollen pancreas, & erratic heartbeat. At that point, it was discovered that my other kidney had started to shrink. How bad does this situation sound? I m worried because I still have children at home to care for. Also, I have very little access to medical care Doctor: Hello and welcome to HCM.As an Urologist,i can understand your concern.You must write age and sex.By now you must've been referred to a nephrologist,for kidney failure.A solitary functioning kidney with shrinkage,is a sign of failing function.Check the following :1. urine routine,culture,and ACR; 2. blood routine,creatinine,RBS,uric acid,LFT,platelets,lipase and electrolytes. Repeat lipid profiles,ECG and echo scans as advised.3. ultrasound scan-KUB,with residual urine (TRUS,and PSA,if male ).If you've any doubts,send a direct question to me,with all reports.Dr.Matthew J. Mangat."
},
{
"id": 93427,
"tgt": "Acidity and noise in stomach. Taking herbal churna, noise gone, upper part of stomach bulged. Suggestions?",
"src": "Patient: I have acid and gas in the stomach for about an year. The doctors gave me some western tablets but could not cure it. So I got some Indian herbal churna which has did a good job. The noise in my stomach has almost gone but I am still taking the churna daily. The upper part of my stomach bulged, appears the aesophagus is ab normal. Please advise. Doctor: this is due to some food like milk wheat potato and other foods you take normally any food can cause this type of effects at any time at any age there can be gastritis gerd and things you wroteget fod tests go for blood serum test for food specific antibodis for these and other foods you suspect and go for elimination diet to get excellent results of cure from ll symptomps you wrote"
},
{
"id": 118642,
"tgt": "Have Hepatitis B positive. Blood test shows HBe negative, antibody positive. Viral load normal. On LFT. Treatment?",
"src": "Patient: Dear Sir, I am 34 yr old guy recently reported Hepatitis B positive . Subsequently further blood test conducted and reported HBe Ag is negaive ,HBe antibody positive.Viral load of 1580 IU/ML with normal LFT. Doctor recommended no treatment but frequently LFT (every month). What is the status of my disease, am I active or chronic hepatitis patient ?/ Treatment guideline ?? Doctor: HIThank for asking to HCMHepatitis (viral) either of type, the particles of virus remains inside the body, but that does not manifest, no treatment needed for this even if you are in acute phase of disease, Immune system should be very much strong to fight against the infection, no need to worry, have nice day."
},
{
"id": 32844,
"tgt": "What does rubella positive mean?",
"src": "Patient: hello doctor...i'm 25 years old...i got married on june 2010.missing my periods in 3 months..today i went to the hospital for check up... doctor pragnency&rubella test but no pragnencerubella is positive...what does this means?doctor did not tell me any think about this?is this is a serious problema or this normal?why do they test rubella.1 month affected by tourch infection& idoes a 1gG blood test with the result 3.37&1gM blood test result 0f 0.37 which is high so improve i have ?slove my query...i waiting for the reply Doctor: Hi, Thanks for posting in HCM.Rubella is also called German measles. It is caused by a virus that is spread from person to person when an infected person coughs or sneezes. Rubella is also spread by direct contact with the nose or throat secretions of an infected person. If a woman gets rubella during pregnancy, particularly during the first three months, her baby is at risk of having serious birth defects. Hope the information provided would be useful."
},
{
"id": 199584,
"tgt": "Any suggestion for penis not standing properly?",
"src": "Patient: My penis is not standing properly, i am 27 years old age man, married from last 2 years..last 2 years i was not facing any issue.i was enjoying vry much. but from last 2 weeks i am facing issue with my penis is not standing.....pls advise me any tablet for yoga or excersie... Doctor: Hi, dearI have gone through your question. I can understand your concern.If you have some illness or chronic disease like hypertension or diabetes then you should take treatment of that first. Any illness can leads to difficulties in erection. Drugs like sildenafil is useful to get erection but it is very toxic drugs and has many side effects so it should be taken on written prescription only. Consult your doctor and take treatment accordingly.Hope I have answered your question, if you have any doubts then contact me at bit.ly/Drsanghvihardik, I will be happy to answer you.Thanks for using health care magic.Wish you a very good health."
},
{
"id": 95543,
"tgt": "What is the cure of peptic disorder ?",
"src": "Patient: Hi, my symptoms are get motion 2,3 or 4 times a day. Morning its almost twice. At first it seems to be a normal motion. But second one seems to like diareahe. Then 3 or 4 time its in afternoon or evening seems to be again normal. Im suffering this everyday it effects my work, distrats me & cudnt concentrate onwork. When i discussed with some doctors they said its peptic order disorder. But can someone tell if it is so, how it could be cured. Hope its not ulcer because there is no bleeding or something. Doctor: hello and wellcome to healthcare magic ,advice to take antimotility drugs to decrease this ,names would be nonethical to tell ,have x-ray of intestines with contrast so it would show the defects ,take also advice from gastroenterologist ,take care"
},
{
"id": 5331,
"tgt": "Trying to conceive, LD pills to delay periods, grey slimy piece seen",
"src": "Patient: hi , i am 35 year old i have passed 2 years infection but it is 5 years that it is cured and not even any symptoms. and i have tried to become pregnant for 4 years but nothing happened.this month i used LD pills to delay my period and now i'm on my period and experienced 2 white to grey slimy pieces . i'm really worry and no reach to doctor ... please answer me soon Doctor: Hi,Thanks for the query.The pieces you mentioned could be bits of endometrium.As you taken tablets to delay the periods, that might have led to overgrowth of the endometrium.And now as the withdrawal bleeding started, possibly the endometrial bits are also coming along with blood.So no need to worry.Possibly the bleeding will end as your usual period.For more details you can ask me through: http://www.healthcaremagic.com/doctors/dr-sree-gouri-sr/63429Take care."
},
{
"id": 20108,
"tgt": "What is the natural diet to reduce high BP?",
"src": "Patient: Looking to drop down from 10mgs of ramipril to 5mgs then hopefully ween off them totally. Since being prescribed the ramipril for my blood pressure, i have lost 2st 4lbs on a controlled diet. My bp is on average now 120/80 using the 10mgs, but hoping the weight loss may be a help to get off medication. Doctor: HelloYes it can be done if you follow a strict diet and exercise plan.Your bp is well under control as of now. Start doing daily exercise for 45 mins a day for 5 days a week. Cut the amount of salt intake in your diet.Eat good amount of fresh fruits and vegetables daily.Stop smoking completely if you do.Reduce the amount of caffiene containing drinks like coffeeTake adequate sleep and drink good amount of water.Follow these rotinely and your bp will be under better control.Take Care"
},
{
"id": 71025,
"tgt": "Is it normal to experience chest pain and cough after undergoing the treatment for Pneumonia?",
"src": "Patient: i am a 72 yr old man diagnosed with Pneumonia a week ago .. discharged from hospital ast Tuesday and although feeling much better still have a pain in my left chest and prolonged periods of a bubbling feeling which seems to subside after much coughing and bringing up phlegm .....I have one more days antibiotics and tablets .. please can you tell me if this is normal ...thank you Doctor: Hello and Welcome to \u2018Ask A Doctor\u2019 service. I have reviewed your query and here is my advice. Yes, these kind of symptoms and feeling are normal during recovery phase of pneumonia. It's only 7 days of pneumonia. In pneumonia, usually complete recovery is seen after 2-3 weeks. So you will be alright after 1-2 weeks more. So no need to worry for these symptoms, they will definitely subside after 1-2 weeks. If you develop fever or worsening cough or chest pain even after 1-2 weeks then consult doctor and get done fresh chest x ray. Hope I have answered your query. Let me know if I can assist you further."
},
{
"id": 165902,
"tgt": "What is causing hives on my 2 years old?",
"src": "Patient: My 2 year old has had a rash / hives in her diaper area front and back chronically, but has been worsening over the last 6 weeks. She does have a dairy and egg white allergy, but it has been eliminated from her diet all together although she has had 2 very minor exposures. Now the rash is visible red raised welt-like hives that flare and we treat with Triamcinolone that was prescribed by her pediatrician. Should we be looking for another cause of the hives? Her diapers or pull ups? She has no other symptoms except she get s a small patch below her mouth and on her cheek at the same time. Doctor: Hi...Thank you for consulting in Health Care magic. Skin conditions are best diagnosed only after seeing directly. I suggest you to upload photographs of the same on this website, so that I can guide you scientifically. Hope my answer was helpful for you. I am happy to help any time. Further clarifications and consultations on Health care magic are welcome. If you do not have any clarifications, you can close the discussion and rate the answer. Wish your kid good health.Dr. Sumanth MBBS., DCH., DNB (Paed).,"
},
{
"id": 196848,
"tgt": "Does cloudy greyish semen indicate infection?",
"src": "Patient: My lab report has a prostate count of 10 which has risen 1 in 2000. The doctor wanted to takebiopsy sample by putting his finger & a needle up my butt which I said no. He said the count indicates some sort of infection which could indicat cancer. I said I wanted to go to the internet first & try to find a natural to cure whatever it is. In doing that I found that more regular ejaculation tends to clear infectious material from the area & eating yellow, green & red peppers & iodized salt etc would help for lycopene. I d already been doing most of that. My semen I had noticed over the years seems to be clouded with a grayish color to it which I would think is a sign of som infection. What is your experience with the grayish semen etc. Doctor: dear Sir, this change of color means the prescence of infection.. you need antibiotics to clear the infection, mostly doxycycline will help.I stronglly recommend that you take the biopsy.. natural ways may clear the infectious material from the urinary tract but they wont be able to clear the infection inside the prostate.. also prostatic cancer is easily curable in early stages and devastating in the late stages.. so the earlier the better."
},
{
"id": 4598,
"tgt": "Am I more likely to get pregnant after having had an endometrial ablation?",
"src": "Patient: I just had an endometrial ablation yesterday. I am 47 and have been married for 3 years. We do not use any form of birth control, so after the first 2 years I assumed that I could not get pregnant. Am I at a higher risk of becoming pregnant now that I have had the ablation don? Doctor: Hi,Thank you for choosing Healthcaremagic.Considering your age and ablation its unlikely you will become pregannt again, but you wait for your periods to resume, sometimes after this ablation you might go for permanent amenorrhoea. If not you can get a copper T inserted.Please ask if you have more questions. If you are satisfied, please make sure that your ACCEPT my answer so that I receive credit. Good luck!!"
},
{
"id": 78907,
"tgt": "What causes chest congestion after being on hydrogen peroxide IV?",
"src": "Patient: I had my first of 6 hydrogen peroxide IV's a few days ago. The following day I developed a congested cough, bringing up lots of phlem, which I am still doing. I then came down with flu like symptoms & fever of 102.4. My hiatial hernia also began to hurt me, even though I hardly had much to eat prior. Is this all a part of the detox? Doctor: Firstly intravenous hydrogen peroxide is one such alternative therapy which has not been proved to scientifically effective . It only acts a placebo if you have seen people improving. It is a risky procedure and carries issues with it. Do consult a doctor and get yourself evaluated for infection in your lungs."
},
{
"id": 150326,
"tgt": "Low back pain radiating to right leg. MRI showed multilevel dehydrated disc disease. Is this neurological problem or orthopedic?",
"src": "Patient: MRI showed \"multilevel dehydrated disc disease causing central canal and lateral recess stenosis with most prominent changes ate L4-5\"I am 58 years old and I contacted the medical specialist because I was having low back ache radiating to right leg. Please advise me which specialist I should approach for? I mean neurologist or orthopedic? Doctor: Hi,You may approach a neurosurgeon since it involved the nerve of the spine.If the neurological complications are there then a neurosurgeon should be consulted first.Though if the neurosurgeon is not available then consultation from the orthopaedician may also be done.Sometimes a team approach involving both is done."
},
{
"id": 203631,
"tgt": "Any suggestion for having hole in penis and can pull skin back and see tunnel under penis head?",
"src": "Patient: I have a hole in the side of my penis and I can pull skin back and see a tunel under my penis head the ends at the underside of penis where there is a hole there. I feel like I could put something through the whole its on the left side of my penis and it goes from a little over halfway around my head the the middle where my urethra is. It looks like a passage and it has been bugging me emotionally for awhile Doctor: The condition described by you is probably congenital abnormality known as hupospadias. The condition is present since birth and may not come to notice if the abnormal opening is just near glans of the penis. One thing that puzzles me that you did not mention about your urinary stream. Urinary stream is generally affected in hypospadias.In my view present yourself to examination of your opening, to a surgeon so that he can assess the problem and can guide you as per requirement.Thanks"
},
{
"id": 162858,
"tgt": "What causes red spots on the body after an anti-rabies shot while being G6PD deficient?",
"src": "Patient: hi good morning here in Philippines , i have a question my son playing with puppy but suddenly he bite on leg and then i already consult in the doctor but he said that on Monday schedule but i worried because on October 18 night he bite but the doctor said on Monday so we wait 5 days and then he presciption Anti-Biotic and then after 5 days he inject my son but after minutes my son have lots of red spot all over his body I m panic . my question it is safe for my son inject anti-rabies but i forgot to tell the doctor that he having G6PD deficiency ? Doctor: Hello and Welcome to \u2018Ask A Doctor\u2019 service. I have reviewed your query and here is my advice. Some persons get allergic reactions to various drugs and your child developed reaction to anti rabies. But there is no relation to G6PD deficiency. I hope I have solved your query. Let me know if I can help you further."
},
{
"id": 169758,
"tgt": "What causes constant burping in a child?",
"src": "Patient: For about a week now my 5 year old daughter has been burping constantly. She sounds like a boy twice her age when she does it. She doesn t say it is causing her pain and she isn t eating anything out of the ordinary. It worries me a bit because it just started out of the blue. Can you help? (She is about 48 inches tall and 46 lbs. She is also white, if that makes a difference.) Doctor: It is just due to dietary habits..kindly reduce milk and milk based products and snacks items sir...this will lead to excessive burping as it causes more gas formation sir..so kindly reduce these items and give colacid syrup 2.5 ml three times a day for 3 days"
},
{
"id": 10327,
"tgt": "Suggest remedy for hair fall post typhoid fever",
"src": "Patient: Dr, My friend has suffered from Typhoid in August 2010. Now she had recovered but since two month she is suffering from severe itching on whole body and hair loss. Also, she is having gyanic problem after this. Her liquid discharge is regular and mensus period is delayed. what and where to go for the remidy? Doctor: Hello and Welcome to \u2018Ask A Doctor\u2019 service. I have reviewed your query and here is my advice. If your friend is facing some problems with delayed periods for the first time, then I would recommend first doing a pregnancy test. If that is negative then one can wait for the periods to arise. If they don't come then you must visit a gynaec. Hair loss is multifactorial: diet/lifestyle/weather/hormonal/products, etc. to name a few. For the itching it is important to rule out the cause for itching: food/clothing/any product/insect bite. I would suggest you get her reviewed by the concerned doctor. Hope I have answered your query. Let me know if I can assist you further. Regards, Dr. Smruti Pevekar"
},
{
"id": 107018,
"tgt": "Suggest treatment for severe back pain and constipation",
"src": "Patient: Went to Dr. 1/30 for severe back pain and couldn t had trouble with bm. He gave me muscle relaxer that made me more constipated. Don t remember last time I had a bm except a little sometime last week. Called office yesterday and said not to use suppositories but to use Miralax. This is 2nd day and still haven t gone. Also broke out with shingles day after office visit for which he gave me anti-virus med. Am miserable and don t find any rectal Dr s in area. What can I do? Doctor: hi sir/madam,With the help of ayurveda its easily treatable.Low back ache may be a pre-monitory (prodromal) symptom in fistula in ano, sciatica, gouty arthritis, hernia, emaciation disorder etc. Pain is confined to lower part of the spine (back bone) especially lumbar region or lumbo-sacral area (rarely sacro-iliac region also). If it is secondary, earlier history of fall or injury may be associated. Rarely, radiating pain may be complained by the patient towards lower limbs. But it is quite common in low back aches if the defect is in the discs between vertebrae (back bones). Often the movements of lumbo-sacral region like flexion and rotation are hampered either partially or completely.Low back pain may be found in mild form in case of anemia, sciatica, rheumatoid arthritis, hemorrhoids, urinary calculi, uterine disorders etc also.Line of treatment as per Ayurveda:-The general principles of treatment of vata dosha are adopted in case of katishoola (low back pain). It includes various measures to suit its varied clinical entities, stages and associated complaints.1. Snehana (oleation) \u2013 by sneha dhara (pouring oil), abhyanga (oil massage), avagaha (tub bath with oil or oleaginous medicaments), kati basti (retaining medicaments on the back) etc.2. Swedana (sudation) \u2013avagaha sweda (sweating treatment with tub bath), pizichil (kayaseka), nadi sweda (sudation through a tubular device \u2013 local sudation), panda sweda (sudation through medicated paste or powder) etc.3. Mridu Samshodhana -mild purgation.4. Basti (medicated enema) like eranda basti, vaitarana basti, pippalyadianuvasana basti etc.Formulations indicated in Low back ache as per Ayurveda:-1. Dashamoola kwatha.2.Maharasnadi kashaya.3. Rasnaerandadi kashaya.4. Sahacharadi kashaya.5. Gandharvahastadi kashaya.6. Trayodashanga guggulu.And for local application these oils will help:-1. Ksheerabala tail.2. Mahanarayana tail.3. Dhanwantaram tail.4. Maha narayan tail.Avoid these for better results:-1.Bitter, astringent and pungent food2.Cold water3.Fear4.Exhaustion5.Standing6.Driving7.Cold food and beverages etc. Next we will discuss about your Constipation.Inability to completely evacuate the bowels or passing very hard stools is known as Constipation or Vibandh in Ayurveda. This prevalent problem is due to an incorrect lifestyle and poor eating patterns. Although constipation is often looked upon as a common problem, if not treated or if treatment is delayed, it can lead to further problems such as fissures, fistulas, piles, lack of appetite and indigestion.Symptoms:-1. Indigestion (dyspepsia).2. Abdominal pain.3. Nausea.4. Flatulence.5. Pain at the anus while defecating.6. Heaviness of body.7. Lack of appetite.8. Acid eructation (belching).Diet & Lifestyle Advice to follow:-1. Increase intake of fiber, especially fruits and cooked vegetables.2. An apple or banana each day is helpful. Bananas should be ripe (bright yellow.3. Drink 7-8 glasses of water each day, preferably lukewarm in winters and room -temperature water in summers.4. Have a mid-morning soup, preferably prepared with spinach and tomato.5. Take daily morning and evening walks for at least 20-30 minutes each.6. Eat whole grains and nuts.7. Avoid refined foods like white flour, bread, pasta, pizza, white rice, etc.8. High -protein foods like cheese, red meat, and soybeans can cause constipation, so these must be taken with a bowl of salad and lots of liquids.9. Meat is constipating and should be avoided.10.Disturbing psychological factors should be removed, and meals should be taken in a relaxed and calm atmosphere.11. The food should be warm and freshly -prepared, as cold and stale food slows down the digestive power.Home Remedies for constipation:-1. Take one table spoon of castor oil at bed time until you get relief.2. Develop a habit of drinking water in the morning. Start with 2 glasses of water, and increase intake up to 6 - 7 glasses. It is more beneficial if this water can be stored in a copper container overnight.3. Soak 3-4 dried figs after washing them overnight in water. Eat them first thing in the morning, and also drink the water in which they were soaked. They should also be taken in the evening. Try this treatment for 3-4 weeks.4. Apply 3 to 4 drops of lukewarm castor oil over the navel at night in kids. It will help in clearing the bowel movements in kids (of age 1- 3 years) in the morning.5. Apply lukewarm sesame oil on the navel during taking bath and put towel (which is dipped in warm water) on the lower abdomen.Ayurvedic medicines to be taken are:-1. Draksharishta:- To be taken with equal quantity of water in 10ml dosage after meals.2. Sukhavirechanavati:- 1-0-1 with hot water at after food.3. Triphala choorna:- 5g to 10g. To be taken with hot water daily at night before bed.4. Pravalapanchamrutha Bhasma:- One tablet twice a day with buttermilk.These medicines will give best results.Hope i was helpful.Stay healthy,Stay safe."
},
{
"id": 169851,
"tgt": "Suggest treatment for cold, cough and breathing problem of a 7 year old",
"src": "Patient: Hi, My son is 7 yrs old and standard 2.. he is always having cold & cough with nose flow. he is having major sleep problem, he makes sound while slepping last week I met a child specialist and was told that he struggles for oxygen.. most likely he is having adenoid whats that ? he is restless and am getting complaints from his teachers too.. whats the remedy? Doctor: from your description, it looks like adenoids leading to obstructive sleep apnoea. common in this age group. check your kids bodyass index too. this happens due to enlarged adenoids( a tonsil like gland) obstructing the airway. this causes sleep disturbances at night time ( snoring) also excessive day time sleepiness. he will need removal of the adenoid gland following which his symptoms will improve. non surgical measures may not be much helpful because of his sleep disturbances"
},
{
"id": 180857,
"tgt": "How can temporomandibular joint dysfunction be treated?",
"src": "Patient: what if I have been falling into flower bed when dead heading flowers & have fallen to my right side TMJ in jaw on right side,have seen 3 different doctors for this problem. Also getting worse feel shaky & jittery--thought I had diabetes. Canyou tell me what I have? Doctor: Hi..Thanks for the query..It looks like you have injured your jaw joint as you said that you have fallen on your right TMJ..It can cause jaw joint dislocation, damage to the soft cushion that is present between the two bony ends of the jaw joint, fractures etc..Feeling shaky or jittery can be due to anxiety or panic attacks however it can also be related to other causes for which you need to get thoroughly examined..For jaw joint related problem it would be better for you to consult an Oral Surgeon and get evaluated..A Panoromic x ray should be done for ruling out the exact cause..For now you can take painkillers lf there is pain and along with it take soft diet and avoid hard foods.Avoid excessive mouth opening like during yawning, laughing etc..Hope this helps..Regards."
},
{
"id": 54257,
"tgt": "Is it possible that lbm is triggered by hepatitis b?",
"src": "Patient: I was diagnosed with hep b 3 years ago. With the medicine ( baraclude ) that my doctor prescribed my liver tests became normal . Last night I started to have lbm and I can't think of anything that I ate that would cause the lbm. Is it possible that the lbm is triggered by my hep b? Doctor: Hi thanks for contacting HCM....You have now normal liver function after diagnosed with hepatitis b infection 3 year ago ...But you can be carrier ...So confirm your negative infection by viral marker study by ELISA.Anyway here your irregular bowel movement could be by eating outside food or excess fatty meal or excess spicy food.It could be from gastroenteritis like infection also by virus or bacteria.If diarrhea with mucus or blood present or fever start antibiotic....If muscular spasm more dicyclomine can taken.Small meal with frequent interval taken.Avoid alcohol , smoking etc.If problem more USG done .Take care.Consult gastroentetologist if not improving.Dr.Parth"
},
{
"id": 45097,
"tgt": "What sholud be the best treatment for IUI ?",
"src": "Patient: F/Age-30,Wt-50, I have gone for intra uterine insemination by donor semen because my husband has azoospermia.Testes biopsy-no spermatogenesis. What sholud be the next best treatment..?Hormones are necessory..? What type of care now ...?I have no other medical history . Doctor: Hi Welcome to HealthcareMagic. IUI helps many women to conceive successfully.The main task is to maintain the pregnancy till delivery.For this purpose based on Ultrasound findings , hormone levels it would be necessary that you will have to take hormonal supplements to support the pregnancy. With possible chances of miscarriages , you should attend your medical check ups regularly and avoid strenuous work , take good rest and nutritious food along with folate supplements. If you need more information feel free to mail me.. Take care"
},
{
"id": 104698,
"tgt": "Child got swelling in eyes after eating followed by upper lip swelling and numbness. Had taken orange juice, on aspirin. Allergy?",
"src": "Patient: My 7 year old son eyes swollen up after eating breakfast of Bacon and Eggs and orange Juice. He was also given 2 baby asprin to maybe help with a leg crap he s had for a couple of days. (Think he pulled a muscle). After 1.5 hours his eyes started to get better. At supper, hours later his upper lip swollen some and he said it felt numb. He also had orange juice for supper. He is breathing fine. No other symptoms. Could this be an allergic reaction to asprin or orange juice? Doctor: Hi The aspirin, egg and fruit juice are all potential allergens in people with allergy and asthma. For a child of 7 you must avoid aspirin. paracetamol is a better choice. It can also cause gastric irritation. It is very likely that your son may have variant type of asthma who manifest allergy to aspirin and fruits. Fruits can cause rash with itchiness around the mouth and around the eyes. Aspirin and polyps of the nose with asthma occurs in a group of asthmatics. They should avoid other drugs like brufen and diclofenac which are called NSAID. or non-steroidal anti inflammatory. Subsequent exposures to these allergens can make the symptoms worse, so take care. Skin allergy test is not very specific, but there are doctors who strongly believe in it. I suggest that you avoid these for the time being and as he grows older you can check him out for asthma with a pulmonologist. You need to educate your son about his allergy and communicate with his school's nurse and friends not to offer him fruit juice. it is good to keep an epipen at home and also at school which is a life saving drug in case of emergencies. You need to learn how to use it when needed. Hope this helps."
},
{
"id": 161216,
"tgt": "What is the treatment for high fever in a child?",
"src": "Patient: I work at a daycare center and a 11 month old boy came in and he felt really warm he was burning up. I took his tempature and it was 103.0 degree fever. The parents said its cause he was teething and I wanted to know if thats possible or if it could be something else. Doctor: Hello, Teething doesn't cause that high temperature. Kids have less amount of blood compared to adults. It is because of that reason fever in children with such high-temperature warrants immediate consultation with a pediatrician. This kid needs an immediate appointment with the pediatrician. Hope I have answered your query. Let me know if I can assist you further. Take care Regards, Dr Jasmine Jagan, General & Family Physician"
},
{
"id": 165546,
"tgt": "What causes skin rash and runny nose in a child?",
"src": "Patient: My 3 year old son had a basic allergen profile blood test. It showed low allergy to milk and undetermined allergy to egg whites and wheat. Doc will not refer him to an allergist for further testing. He just wants us to try an elimination diet for a few weeks. His only symptoms that we noticed are a rash (keratosis pilaris) and a runny nose. The allergen profile only tests for a few things. Should we insist on seeing an allergist? Doctor: Hello,If symptoms of allergy like a rash or runny nose improve after eliminating egg or milk, then probably you don't need to go for further testing. More than food other allergens like pollens, dust, mites, artificial flavors in candies, chocolates, juices, etcetera, can cause allergy. If symptoms are not improving, you need to go to an immunologist. In the meantime, you may have antihistaminic medicine for symptom relief.Hope I have answered your query. Let me know if I can assist you further. Regards, Dr. Khan Shoeb Mohammad Sher Mohammad"
},
{
"id": 194785,
"tgt": "What causes pain in my scrotum?",
"src": "Patient: i was kicked in the balls 5 days ago. no pain at all till about 4 days later. slightly uncomfortable but no discolour or visible bruising. balls arent really tender. sort of just a dull ache. ive done the reflex test and my scrotum is still reacting. testes both seem to be laying in the same direction. Is this small pain just part of getting over the trauma or could it be torsion? Doctor: Hi, It may be just trauma to the scrotum affecting less to the testis. So, do not worry. It is not torsion of testis which is a very painful condition. You may consult the surgeon for the perfect diagnosis and proper treatment. Investigations like sonography of the scrotum may be done. Analgesic like Brufen might reduce the pain. Hope I have answered your query. Let me know if I can assist you further. Regards, Dr. Ilyas Patel, Dermatologist"
},
{
"id": 49003,
"tgt": "Can stent removal from kidney cause any problems?",
"src": "Patient: I m to have a stent remove from my kidney in the next two weeks. When it was inserted the doctor wasn t sure just what was causing the blockage, but put in a stent anyway. Is this normal? And what can I expect when I have it removed? It has caused me a lot of pain and bleeding. Doctor: Yup, thats quite common in urinary obstruction when no obvious cause is found and people land up having stent in the ureter. Having said that, it is found to deal with many subtle and/or temporary obstructions including cell debris, small kidney stones and so on. Thus it protects the kidney from permanent damage. But it cant be permanent and once basic problem is solved and obstruction relieved, we can get rid of the stent at any later date before it itself becomes a nidus for infection!!!"
},
{
"id": 17540,
"tgt": "What causes bruising arm and numb fingers while on warifin for heart failure?",
"src": "Patient: my husband has congestive heart failure. He takes warifin, cumadin.....and some other medications to keep his condition in control. He is only 40 yrs. old. We recently noticed his elbow getting very swollen and it looked very bruised. I took him to the hospital... they stuck a needle in his elbow to draw out what they thought was puss....ended up being blood. Right at this very moment they have admitted him due to the fact that the brusing has spread and further up through his arm. he is also telling me that two of his fingers are numb.... what is going on? Please help me Doctor: Hello, Consult a cardiologist and get evaluated. Warfarin is a blood thinner and thus cause bleeding. You have to make necessary dose adjustments to prevent bleeding. Hope I have answered your query. Let me know if I can assist you further. Regards, Dr. Shinas Hussain, General & Family Physician"
},
{
"id": 195042,
"tgt": "What causes pain in the left underarm while suffering from gynecomastia?",
"src": "Patient: Hi I am a male 54 years of age with good health. I was just diagnosed with Gynecomastia in my left breast, I have had an ultrasound and 3d imaging of both breasts and there is no signs of cancer. I also have a bit of a painful underarm on the left arm. The left breast is also painful to the touch and there is a mass under the nipple. I was also examined by my physician for swelling of lymph nodes and none were found. Blood work was done with nothing out of the ordinary. My concern is the under arm pain which seems to me increasing. Doctor: Hi, It can be a referred pain from the breast only. You can go for an ultrasound scan. As of now take analgesics like Aceclofenac or Acetaminophen for pain relief. Hope I have answered your query. Let me know if I can assist you further."
},
{
"id": 68624,
"tgt": "What causes bumps and soreness?",
"src": "Patient: my son is 18 and just showed me a 3 bumps (looks like possible bites with a little redness around them) on his lower left side and I noticed on his lower back as well. He said his whole left side up towards his underarm is really sore like someone punched him. While he was touching the area he noticed a bb-sized hard bump under the skin maybe 3 inches down and to the left of his left underarm. Doctor: welcome to Health care magic.1.It can be bacterial infection - with collection / pus and pain on touch, ( carbuncle / furuncle ).2.Post traumatic cause also cause similar infective symptoms. Next possible cause is lymph nodal enlargement.3.If you are my patient i would have examined and ask for a ultrasound lump to assess the nature of the lump, its source, and extensions.4.Depending on the findings the treatment will be simple course of antibiotic treatment to small incision and drainage.5.mean while as a home remedy keep lumpy the area hygienic , clean with antiseptic liquid and do not let him scratch the area as it will complicate.6.Get an appointment and let get examined and treat accordingly.Hope it helps you. Wish you a good health.Anything to ask ? do not hesitate. Thank you."
},
{
"id": 176245,
"tgt": "What causes pain around the naval?",
"src": "Patient: My son is 11 years old and he is having sharp stinging pains around his navel, He had also had the same kind of pain last year and they did a xray, and MRI in his stomach showing up nothing. The pain does not cause him to lose sleep he said it does not hurt while he sleeps but he said it is like a wasp is stinging his stomach what could be causing this Doctor: Hi...by what you quote this could be chronic pain abdomen. Most of the times we do not find a reason for it even after extensive investigations. I have a few questions and suggestions for you - Questions-1. How many months is he having the problem?2. Is there any family history of gall or kidney stones?3. Did he have urinary tract infection in the past?4. Anybody else in the family is having a similar problem?5. Does he have constipation?Suggestions-1. Non-specidifc mesenteric lymphadenopathy is likely in this age group.2. Serious problem cannot go on for months together without manifesting with complications.Kindly get back with answers to above questions. You can approach me at the following link. Please find the link below - www.healthcaremagic.com/doctors/dr-sumanth-amperayani/67696"
},
{
"id": 38220,
"tgt": "How to treat swollen lymph nodes in my neck?",
"src": "Patient: I have 7 swollen lymph nodes in my neck/jaw area and 5 swollen lymph nodes in my groin area. Is this a matter that I should be extremely concerned about? The Lymph Nodes are somewhat hard in my neck and rather immovable based on my judgement. My concern is that I have this many swollen lymph nodes. I thought it was only normal to have a couple swollen during infections? Please help. Thank you. Doctor: Thanks for contacting HCM with your health concernsI am sorry to hear that you have several large lymph nodes in your neck and groin areas. These are not normal findings. I recommend that you seek medical attention to determine the cause of the enlarged nodes. Mononucleosis may cause enlarged lymph nodes as well as other infections. But there could also be some blood born diseases like lymphoma that cause enlarged lymph nodes.Please proceed to your doctor for additional testing and evaluationHope this answers your question. Please contact us again with your questions and concerns"
},
{
"id": 207484,
"tgt": "What causes sudden behavioral changes in a person?",
"src": "Patient: HI , My dad is 74 .His behaviour has changed significantly .He used to be a very good person with sound behaviour .But now he is very very stingy .. cares for no one.. often lies and never admits that he lied . he has become cunning .His habits have changed too ..he is always sleeping .. He is very unhygenenic .. always complains that he has the biggest illness which he claims to be constipation and there is no cure for it He is very mean in his words and very calculative in words and money he hates my mom what is wrong with him ? why does he behave so cunningly .why has he become so selfish? Doctor: HelloThanks for using Health Care Magic for posting your query.I have gone through your question in detail and I can understand what you are going through.There is a definate change in your fathers personlaity. The most common cause is onset of cognitive disorders like dementia. This leads to such personality changes. The other possibility could be a mass lesion inside the brain. Considering his age, it would be wise to get a CT scan brain done.Hope this helps,If you have any further query, I would like to help you on the link:bit.ly/dr-srikanth-reddyWish you good health,Kind regardsDr. Srikanth Reddy"
},
{
"id": 174451,
"tgt": "Suggest treatment for viral infection in a 1 years old child",
"src": "Patient: Hello doctor My kid is one year old.N she is suffering from some viral infection .Frequent fever n loose motions are going on since last one week. There were some drops of blood too in the stool. Every time she passes stool, there is some blood in the stoo Doctor: hi YOUR KID IS HAVING DYSENTERY, A BACTERIAL INFECTION SINCE PASSING BLOOD IN THE STOOLS. NEEDS TO BE GIVEN ANTIBIOTICS AND PLEASE GIVE PLENTY OF ors fluids 50ml/stool to prevent dehydration.Needs to give Zinc syrup.Please contact child specialist now.Hope it answers your query and ready to help you further"
},
{
"id": 183943,
"tgt": "What causes soreness in gums with swollen lymph node?",
"src": "Patient: My husband went to doctor because of fever and swollen lymph node and the doc said that it was post nasal drip and gave him amoxicillin. My husband was also concerned about his sore gums but the doc that he had never seen said it was gingivitis. We called out dentist and he confirmed that my husband has never had gingivitis. Should we be concerned about any possible problem with this? Doctor: Hello,Thanks for consulting HCMRead your query, as your husband had history of fever , swollen lymph node sorness in gums can be due to viral fever related it can be Viral stomatitis causing inflammation of gingiva or desquamative gingivitis . Dont worry I will suggest you to consult dentist and go for visual examination of gingiva in Meantime you can apply ointment Acivir for soreness and you can take medicine like Acivir tablet by consulting with your dentist , do warm saline rinses , you can take vitamin B supplements with medication .Hope this will help you. Wishing you good health"
},
{
"id": 100860,
"tgt": "How can anaphylaxis shock be avoided while taking medications?",
"src": "Patient: HI IAM DR NIDHI AGARWAL (OBS &GYANE) I DIDN T HAVE ANY HISTORY OF ALLERGY .2 YRS BACK I SUDDENLY HAD SEVERE ANAPHYLACTIC SHOCK WITH CEFAUROXIME(CEFTUM) AND AGAIN AFTER 2 MTH WITH ONDEM NOW IAM AFRAID TO TAKE ANY MED.WHOM TO CONSULT.I AM RESIDING IN DELHI Doctor: Hello Dr Nidhi Agarwal,Thank you for asking at HCM.I went through your history and would like to make following suggestions to you:1. Both cefuroxime and ondansetron are known to cause severe allergic reactions like anaphylaxis.2. It is also possible that not the drug itself, but preservatives, excipients, coloring agents, etc used in manufacturing the drug can also cause anaphylaxis.3. There are not very reliable tests available to definitely diagnose or rule out drug allergies except in case of penicillin allergies. Hence, once a patient had an anaphylaxis to a particular drug, the best way it to avoid it.4. I would suggest you to consul an Allergist-Immunologist in Delhi who will ask your history in great detail to come to certain conclusion.5. There are many allergists in Delhi. You can consult the one who is experienced and also the one who is closer to your place. VP Chest Institute in Delhi has a department that specially deals with Allergy-Immunology and has many renowned allergists. Hope they may e helpful to you.Should you have any other concern, please feel free to ask at HCM.Wish you the best of the health.Thank you & Regards."
},
{
"id": 42379,
"tgt": "What is the treatment for infertility?",
"src": "Patient: HAI DOCTOR GOOD MORNING I AM 40 YEARS OLD , MARRIED IN 10 YEARS BEFORE, STILL NOW BABY IS NOT COMING. I WILL CONSULT THE DR, BUT THE COUNT IS VERY LOW (19 MIL) I WILL TAKE TOO MUCH MEDICINE, NO GOOD RESULT, WHAT I CAN DO SIR, MY WIFE AGE 30 YEARS OLD. GIVE ME THE ONE GOOD WAY. Doctor: hai,as your count 19 million/ml is nearing normal.as if you got factors like active motile sperm count,morphology within normal then chance of getting a baby is more unless your wife didn't have any abnormality regarding reproductive system. taking medicine without ruling out the condition will not lead to proper solution.ultrasound will help in finding out the structural abnormality in the reproductive system,i suggest you to rule out he reason behind your infertility with help of infertility specialist.thank youhope i answered your query"
},
{
"id": 111904,
"tgt": "Can Naproxen be taken for the lower back pain?",
"src": "Patient: I am having low back pain which is radiating to the left side of my neck....went to Urgent Care last week they gave me trigger point injections.7....now on Bupropion 100mg in AM ..Diazepam 5 mg 3x a day ...Can I take Naproxen to help the pain? Hot pacs and cold all day. Nothing helps? Yoga hurt me worse today. Doctor: Back pain with leg pain will most probably is due to disease prolapse. Treatment which has you are receiving will be effective in long term. I would recommend you to start gabapentin and methylcobal also. Physiotherapy will be helpful only under supervision."
},
{
"id": 91772,
"tgt": "What is the remedy for the severe stomach pain due to stomach hernia?",
"src": "Patient: I am having severe stomach pain. it could be pancreatics or my large stomach hernia. went to emergency room Saturday. The doctor said I did not have pancreatics but my primary doctor said that he did not see results in report that he received today. I feel full like I have eaten a large meal when I only had dry toast and cup of pudding. I feel very sick. should I go back to the er or spend time visiting my primary. I need relief from this pain. Doctor: Hi. Your posting additional symptoms would have helped me to give you a better idea about diagnosis. Viz: nausea / vomiting/ abdominal distention / constipation or loose motions/ fever/ fainting or collapsing / sweating/ and so on.If these symptoms are not there , it is less probably due to pancreatitis. Large stomach hernia can cause this type of pain if the opening ( neck) is small and there is entrapment of intestines. If one goes for an X-ray in standing lateral position , one can see intestine being trapped. Such a simple test. Ultrasonography and serum lipase and amylase enzymes increase in pancreatitis.Well there still can be another reasons , which can be diagnosed on a good clinical examination and your proper history.Let me know more ."
},
{
"id": 102788,
"tgt": "Feel nauseous and pain in the arm, have been taking allergy shots for some time. Do you have any suggestions?",
"src": "Patient: Hello, I have been getting allergy shots for two months now. I am still in the build up phase. Everything has been fine up until yesterday. I got my normal shots and was fine all day, last night where I got one of my shots (I get three total) my arm was in SEVERE pain. It was excruciating. I did not sleep at all. I took a bath and the pain is bearable now but still HURTS. Now I am beginning to feel nauseous and overall just not well. I called the allergist, she said there is nothing they can do except lower the dose the next time. Do you have any suggestions as to what I should do in the meantime to alleviate my symptoms? Please help, I am miserable. Thank you, Kristen Doctor: HIThank for asking to HCMIf you are facing such problem then it is better to reduce the dose or either skip it or else your symptoms need to be investigated for deferential etiology this may nit be due to the shots you are taking but could be something else, the possibility of this has to rule out, have nice day."
},
{
"id": 109063,
"tgt": "Suggest treatment for severe back pain",
"src": "Patient: I am a 32 year old woman who has suffered with back pain since being pregnant with my daughter. The pain briefly went away after her delivery but came back just as bad when she was 6 months. She is now 3 and I am currently 10 weeks pregnant. The pain is already so bad I can not walk without limping or holding onto furniture to get around the house.I see a chiropractor to relieve the pain but it always returns soon after. I am desperate to stop the pain. Should I have an x ray to check everything is ok as it obviously isn't? Doctor: HelloI would suggest you to go for MRI rather than xray. X-rays can cause harm to fetus but MRI is safe in pregnancy.Apply topical painkiller ointment over back to relieve pain.Consult a physiotherapist.Thanks"
},
{
"id": 199944,
"tgt": "How to treat in Astheno Apermis with poor fertilizing capacity?",
"src": "Patient: Hi i m mohan age 26 years married 1year 6 month my wife did not pregnent so i consulted to one mbbs dgo docter she told me go and give the seman test last thuesday today i got the seman test report on that COUNT -NORMAL MORPHOLOGY - NORMAL(86 percent) MOTILITY-POOR (GR 3 and 4 :19 and 00:19 percent) SPERM DNA - NORMAL (81 PERCENT ) ASTHENO SPERMIA WITH POOR FERTILYSING CAPACITY THIS IS IN MY REPORT so what would be done in future for me whats t next step for me what i want to do for astheno spernia treatment can i get a baby tis my question pls tell t answers doctors Doctor: HelloThanks for query.Based on your semen analysis report I would state that you have Asthanospermoa meaning there by that quantity of grade one sperm which is essential for effective fertilization of female egg is zero percent .In such a situation there are no chances of that you will be able to impregnate your wife.Unfortunately there are no medications that can increase sperm motility.Please consult qualified Infertility specialist to explore possibility of pregnancy with the help of newly assisted reproductive techniques like ICSI or IVF.Dr.Patil."
},
{
"id": 96677,
"tgt": "How to treat swelling in injured finger ?",
"src": "Patient: Hi, may I answer your health queries right now ? Yes, my daughter injured her finger and it was swollen around the finger nail. It went down some but now it has swollen again and a little farther back. Is there anything I can do at home? Doctor: Hi ! Good morning. I am Dr Shareef answering your query.If I were your doctor, I would advise your daughter to keep her injured hand elevated to reduce the swelling due to inflammation. I would also advise her for an anti inflammatory drug along with a proton pump inhibitor. With this, both the pain and swelling would get reduced. However, if the swelling and pain increases in intensity, I would refer you to a orthopedic surgeon to rule out a fracture of the bone in the region and for further management.I hope this information would help you in discussing with your family physician/treating doctor in further management of your problem. Please do not hesitate to ask in case of any further doubts.Thanks for choosing health care magic to clear doubts on your health problems. I wish you an early recovery. Dr Shareef."
},
{
"id": 15443,
"tgt": "Rash on face, burning sensation on lower body. Ate large quantity of tomatoes. Cause of rash?",
"src": "Patient: I have recently noticed a rash on my face and also a burning sensation on my lower body which I know is not a bladder infection because it is on the outside of the body. I have been eating a huge amount of fresh garden tomatoes lately and am wondering if the acid in the tomatoes could be causing my problems. It's is the only thing I can pinpoint that may be causing the red rash on my face and also on my bottom. Please let me know is this is possibly the cause. Doctor: Hello.tomato contains citric acid .when eaten in huge amounts it causes rash and burning sensation.use antihistamine.antibiotic.pantaprozole and calamine lotion"
},
{
"id": 62502,
"tgt": "What could the abdominal warty lesion have been?",
"src": "Patient: Hi, Dr: I am Dr.Bassel , about 8 months ago a warty like lesion in the lower abdomen area , about 8 cm above the penis root , that has a congested pedunculated root , I have no sexual activity , except my wife which is a doctor also and do not have any lesion or sexual relations , I excised it with electorcautatry , 2 weeks ago but its site is still unhealed and erythmatous skin arround it , what do you think is the diagnosis? Doctor: HI,Welcome to HCM.Cause of Warty lesion on lower abdomen-Based on the facts of your query,you seems to suffer from-Pedunculated HPV wart.-A good Hygiene of the private parts with dettol sponging-keep the area dry.-As you are worried for non-healing since 2 wks of electrocuatery of the wart,I would suggest wide excision of the margins and Wedge biopsy of the edge of the Wart for Histopath study,if you haven't cauterized with wide margins.-Also be on High Protein Bland diet / with Citrus fruits for fast healing-Apply Silvadene cream locally to control infection and expedite the healing,which could be tried to begin with and for maintenance of fast healing of the wound.This could also would give you fast healing of the cauterized wound.Hope this would help you to resolve the healing issue by silvadene cream to start with and if no relief after 1 weeks time,you can opt for wedge biopsy with HPR study with wide excision by a Surgeon.Welcome for any further query in this regardWill appreciate writing your feedback review comments,to help the needy patients like you at HCM.Good Day!!\u00a0\u00a0\u00a0\u00a0\u00a0Dr.Savaskar,Senior Surgical SpecialistM.S.Genl-CVTS"
},
{
"id": 165569,
"tgt": "How can baby's eyes be protected during gallbladder screening?",
"src": "Patient: Hi, may I answer your health queries right now ? Please type your query here...there is a problem in the screening of my new born baby, regarding to his gallbladder, the doctor said the eyes of my baby should be affected if it could not be prevented, what should i do? Doctor: hicertain therapeutic procegure like photherapy reduce bilirubin but it causes damage to eye.so during these type of procedure ,a simple bandage are appiled on the both the eye to prevent the rays to enter into the eye."
},
{
"id": 104570,
"tgt": "Child with frequent attacks of cold and cough, asthmatic breathing. Will Montair LC help to overcome it?",
"src": "Patient: Hello doctor, my son has completed 5 yrs..........he is suffering from frequent attacks of cold and cough which will end up with asthmatic breathing if left without medications . He is having these complaints since 4 yrs. We give him cetzene and theoasthaline syrup whenever he gets attack and these medications give him temporary relief.......Now we heard about Montair LC kid tablet and would like to know more about it........whether this will help him, any side effects, how long he should take, etc etc..........It will be a great help for us if u give an authentic reply as early as possible. Doctor: Bincy, probably your child is having bronchial asthma. The best medication for your child will be inhaler with spacer. They contain reliever medication for temporary relief and preventer medication which will reduce the inflammation in the air-pipes of your child. The quantity of drug in these are very low as it is given directly to lungs Montair Lc tablet contains levocetrizine - a purer form of cetrizine( present in cetzene) and monteleukast. Monteleukast can prevent asthmatic inflammation but whether it will be sufficient for your child can be decided by examining your child. Consult nearby peadiatrician or pulmonologist. Monteleukast is generally tolerated with minimal side effects. The duration of therapy will depend on response to treatment . Dont be under the impression that giving the tablet for a specific period guarantees a permanent cure."
},
{
"id": 201704,
"tgt": "What causes tight foreskin with red dots on it?",
"src": "Patient: Hi doctor I ve noticed little red dots on my foreskin . it does nt hurt when I urinate.it s been the second time now in a month.just hoping and praying it s not herpes. my foreskin is kind of tight when I have an erection. The tip of my penis touched a dirty toilet when I sat down. Doctor: it is not herpes. as herpes lesions are so much painful.foreskin is tight at the time of erection normally.red dots on your skin are glands and smegma.so do not worry as you are having everything normal.penis skin is protected by skin and secretions. so single touch with toilet seat can not cause any type of infection.hope my answer helped you.take care."
},
{
"id": 155985,
"tgt": "What is the drug for second line chemotherapy?",
"src": "Patient: hello doctor, Erpr negative and herceptin negative patient of IDC with node \u2013 ve, received first line chemotherapy now having multipal secondaries in lungs leaver , meadiastinum and bones with multi centric disease- please suggest me drug of choice for second line chemotherapy. Doctor: If its been more than 6 months since your last chemo the first line chemo can be reused.Based on what you have recieved initially the second line is decided.If your tolerance is poor a single agent therapy using gemcitabine or cyclophos or doxo.If tolerance is good a combination like cyclo doxo 5fu can be used.Since you have bone mets monthly zoledronic or denosumab helps.Usually with extensive disease single agent therapy along with denosumab or zoledronic is preferred.The decision varies from patient to patient."
},
{
"id": 143688,
"tgt": "How to solve the problem of trouble thinking ?",
"src": "Patient: I am having trouble thinking. I hear the question being asked of me, but I can t process the request. I can t find the words to reply, and today I can t even think through the problem. I m staring an email message at work and can t process the information presented. Doctor: Hello. You may have attention deficit disorder. Your doctor can screen for this and it can be treated with medications like Adderall or Ritalin"
},
{
"id": 149871,
"tgt": "Have pain in the shoulders, chest, back, neck, hanks and arms. Had a spine surgery after an accident. What should be done now",
"src": "Patient: I am a 38 year old woman and I was in a car accident in 2011, which led to compression burst fractures of T-4 and T-5. I had surgery a year later they put in rods, screws, a cage, and done a spinal fusion from T-2 through T-8. While testing before the surgery they found I also have osteoporosis of the spine, stenosis at multiple levels, degenerative disk disease and kyphosis. Anyway during surgery they found the spine wasn't straight and that it was worse then the MRI and X-rays showed. The Surgeon told my husband and family that he couldn't get the spine completely straight and the hardware could never be removed. I have been having lots of pain in the shoulders, chest, back, neck, hands and arms. At times my hands and arms feel like they are asleep and or I could be holding something in my hand and just drop it like my hand isn't even there. I can get cut doing dishes from a knife or burn my hands and not even feel it! When lifting my arms up or reaching something in my back, well the best way I can put it is, it feels as if something is pulling, snapping, and popping. I have been on Norco 7.5mg for 2 years now at first they were helpful but now they don't help at all. As of three weeks ago the doc decided to take the pain meds and told me to take Ibuprofen and over the counter Tylenol. Which I am having to take 1600mg of ibuprofen every 6 hours and 1500mg of Tylenol every 4 hours and that just puts me to sleep it does nothing for the pain. I think the surgeon believes it should all be better and refuses to listen. My back burns,pulls, feels like I am being stabbed from the inside out, and just plain hurts all the time, so I went back to my surgeon whom without any new x-rays or anything, in fact the last tests he done was 3 months after my surgery and they havent even tried sending me to physical Therapy. This same doctor now tells me they need to go in and take the hardware out. I reminded him of his previous statement about the hardware havin to stay in and he replays it will be fine. My questions are is this anywhere normal, what could happen if they remove the hardware? Will i be in worse shape then I already am? Should I even consider doing the surgery to have it removed? I am very confused and in so much pain that it effects everything in my life. Anyone with any Ideas? Anything Please! Doctor: Thanks for writing to us.I understand that you had thoracic spinal fusion for fracture vertebra. If possible please upload the xrays pre and post surgery to know more about the spine. Spine implant usually not removed unless there is a complication related to it. First of all we need to know what is the pain generator? Whats the situation of the implant? If your spine is fused then implant removal can be done provided it needs to establish that thats the pain generator.Please upload the xrays and also get a fresh ct scan of spine. Thanks."
},
{
"id": 29104,
"tgt": "What causes HPV infection?",
"src": "Patient: my son s girlfriend of 5 years just had a procedure for hpv. she says that she must have got it from him. she has been married 3 times. could she have gotten it from one of her 3 husbands and it is just coming out now? myself and my husband think she will do and say anything to hang onto him. tonight she says she must have got it from him because she has been with him for 5 years and now she may have hpv. Doctor: hiyou should get your son tested as well . better to test and check the results than assuming who has got what.Regards"
},
{
"id": 112623,
"tgt": "Back pain from left leg, abdominal pain, body selling. X-ray shows loss of normal lordosis and muscular spasm. Taking medicines. Complications?",
"src": "Patient: Hi, I am 26yr I have been having severe back/waist pain radiating to my left leg,abdominal pain and occasional generalized body swelling.my x-ray reveals I have loss of normal lordosis and muscular spasm.I was on physiotherapy for two week which really helped me a lot but presently my Dr have asked me to run some test which include! Rheumatoid factor, antinuclear antibodies and anti lah antibodies .my Dr also gave me some drugs which are Tizanidine 2mg, Pregabalin 75mg,soft gelatin capsule 1000mg and Diazepam 5mg because I can hardly sleep. Please how long will I reman like this and what are the complications and what do I do because I have presently stopped my physiotherapy. Thanks. Iruoghene. Doctor: Hello,Welcome to Healthcaremagic.From your query i understood that you are worried about the side effects of your drugs.Tizanidine - is given to relax your back muscles.it can cause nervousness, tiredness, painful urination, constipation etc..Pregabalin- is given for neuropathic pain.it can cause dizziness, dryness of mouth, weakness etcDiazepam- for sleepit can cause drowsiness, tiredness, loss of balance, confusion, depressionUsually muscle spasm relaxes in week or 10 days.Continue physiotherapy to relieve pain. have heat pad for 15 to 20 minutes every 4 hour.Hope this helps.Take care.Regards,Dr.Manjeth"
},
{
"id": 48139,
"tgt": "Will Renal cell carcinoma be effectively be treated by Embolization?",
"src": "Patient: How large can a localized* renal cell carcinoma be and still be effectively treated via embolization. *There has been no renal vein or tumor membrane invasion, and no lymph node involvement. The tumor was discovered after sudden onset of back pain, nausea, vomiting, fever. Blood tests were done with a-symptomatic results. CT Scan revealed a very large tumor that has totally involved the kidney. Surgery is scheduled within two days. Doctor: helloThanks for query.Only treatment option for large Renal Cell Carcinoma involving whole kidney is Radical Nephrectomy.Renal Cell Carcinoma of even a small size can not be treated by embolization.Go ahead and get surgery done .Dr.Patil."
},
{
"id": 93331,
"tgt": "Have hiatus hernia, painful, have dizziness, weakness, difficulty swallowing. What is it?",
"src": "Patient: Hello, I have a hiatus hernia, I am 22 years old and I have had it for 6 years, recently it has been getting really painful, I have been to the doctors and they are finally looking into surgery, right now I am in a lot of pain and feeling quite sick and dizzy, I feel like food or something is stuck low down and swallowing is harder than normal. Any advise please?Thanks, Christine Doctor: Hi ! The doctor has rightly advised you for surgery as you possibly have not got much relief with conservative management. However, for the time being to relieve the discomfort, your doctor may prescribe you a prokinetic drug along with a proton pump inhibitor, and some dietary restrictions which you have to adhere to.Wishing you an early recovery."
},
{
"id": 129647,
"tgt": "Is arthroplasty recommended to treat a broken little toe?",
"src": "Patient: I am considering arthroplasty on my right pinky toe. It has been broken several times and of course has arthritis also. The pain is constant and the toe is thick, has turned to the outside and it makes it difficult to wear anything other than wide/soft shoes. I walk a great at least 3 miles a day, play golf and am active. Is it likely that surgery will correct it enough that I will be able to wear normal shoes and resume physical activities? Doctor: Hi there. It looks like due to multiple fractures sustained by your previous toes previously it has not united into its proper position. You can opt for various corrective surgeries around the little toe at the level of the metatarsal bone or at the level of the joint to help correct the alignment and help you with wearing a shoe, and improve your walking. Arthroplasty involves removal of bone and will limit any future bony procedures that may be needed in case. The type of procedure required is mainly based on how your x ray appears and where exactly is the deformity. In order to reduce your pain, consider wearing footwear with soft soles and certain microcellulised rubber(MCR) footwear which has the property of reducing the excess pressure over the prominent bbony region. Make your decision regarding going ahead with any surgery based on your discussion with your doctor."
},
{
"id": 214254,
"tgt": "Suggest medications for injuries on body due to an accident",
"src": "Patient: Dear Sir/Madam,I got my both legs injured in some accident with a heavy vehicle, its too heavy that my legs are not responding properly. I'm unable to walk properly, so I go to doctor and in his prescription he gave me a capsule named \"tricobal OD\", But It's Non-Vegetarian one, Can Someone suggest me a Vegetarian Solution? Doctor: hello,there are usually 3 alternatives .As u have been prescribed to take B12 via a drug, you can have injectable preparations available. U can ask for that to your doc.and as u r vegetarian, i will suggest you to take care of your diet and add following1) Dairy products..milk, paneer, cheese etc.2) Fortified breakfast foods with B123) Capsules that do not have gelatin cover but have cellulose cover."
},
{
"id": 39751,
"tgt": "Suggest treatment for dog bite on a hand",
"src": "Patient: hello sir my mothers hand was accidentally rubbed my pet dog tooth. no injury is visible. it cause burning sensation. my dog is 4 months vaccinated. is it necessary to consult doctor. the area of hand is thoroughly cleaned with soap and dettol. please give suggestion. Thanks in advance Doctor: Hello,Welcome to HCM,Rabies is a disease which is also transmitted by bite of the monkey, it is a 100% fatal disease but it is 100% preventable by proper and adequate treatment.As your mother was accidentally bit by your vaccinated pet puppy which has led to burning sensation at the bite site with out any open wound and the bleeding, according to WHO categorizes it has been categorized into Cat II.Cat II requires active immunization with antirabies vaccine on days 0,3,7,14 and 28, if the dog is healthy after the third dose she can discontinue the remaining two doses of vaccine in addition to this she should be given a shot of tetanus injection too.The aforementioned treatment will protect you against developing the disease rabies.Thank you."
},
{
"id": 155120,
"tgt": "What is possibility of cancer in case of homogeneous endometrium thickening?",
"src": "Patient: I just had an ultrasound done because I have long heavy periods and between period bleeding. They found me to be endometrium thickened 18mm it appears homogeneous. What are my chances this is cancer. I have been bleeding excessively for 4 yes with it progressing the worst for the last two years. Doctor: HiChances of cancer is 50 percent.I would advise you to do a biopsy test.Ct scan of whole abdomen needs to be done.RegardsDr de"
},
{
"id": 33538,
"tgt": "Suggest treatment for fever , diarrhea and vomiting",
"src": "Patient: My son is 14. On Saturday, he woke up with flu symptoms, throwing up, diarrhea and a 101.8 temperature. He was fine Friday went he went to bed (after eating Mexican food. I did not eat the same thing). By Sunday afternoon he had a low grade temp and most of the vomiting was over. By Monday night he was stating his stomach hurt. It is still hurting today. I m not sure if he got food poisoning Friday night, or if he truly had the flu. And not sure if we should go the doctor or not. (I just started a new job and am currently without health insurance until May 1st) Doctor: Hi,Welcome to health care magic,All this symptoms are indicative of food poisoning.Food poisoning is nothing but acute gastroenteritis due to toxins of contaminated food from infections like bacteria, virus,parasite etc.Symptoms usually starts after 12 to 24 hours of eating,like nausea,vomiting,abdominal pain,loose motions,fever,weakness,fatigue, headache,abdominal cramps due to water and electrolyte imbalance.As per your symptoms, you have this gastroenteritis and will require antibiotics(ofloxacin with ornidazole), antispasmodics, antiemetics,probiotics and other supportive treatment as per your symptoms.So,consult physician for proper treatment.Regards,"
},
{
"id": 208282,
"tgt": "What causes frequent attacks of anxiety in a 31 year old?",
"src": "Patient: I am 31 yrs mail n having anxiety problem since long time, at times its gets so bad that i can barely speak. The tough time is whn in public i have to tell my name and stuff. At times I am very confident that i forget that I had anxiety. Pls help me,,,,Gautam Doctor: DearWe understand your concernsI went through your details. I suggest you not to worry much. From the description you gave, I am to understand that you have some sort of adjustment problems and apprehension about speaking in public. This may not amount to classic case of anxiety. But can be categorized as general anxiety. Still need proper diagnosis. Therefore do not jump guns. At times you are very confident, and that is what counts. Gain confidence from those positive incidents and those experiences helps you to be bold.If you require more of my help in this aspect, Please post a direct question to me in this website. Make sure that you include every minute details possible. I shall prescribe the needed psychotherapy techniques which should help you cure your condition further.Hope this answers your query. Available for further clarifications.Good luck."
},
{
"id": 47241,
"tgt": "What does this abdominal ultrasound for kidney indicate?",
"src": "Patient: Abdominal ultrasound for kidney observed lobulated uterus that should be examined via pelvic ultrasound. Primary care DR said if I have heavy bleeding, they would order it. I don t have that, and don t know what other symptoms would warrant concern. I am menopausal/post menopausal (last period GT 9 months ago), 55 years old, relatively good health. No history of fibroids or anything else related to pelvic organs. Should I be concerned? What should I be watching for? Should I push for the pelvic ultrasound regardless of any symptoms? Doctor: Hi, dearI have gone through your question. I can understand your concern. You have lobulated uterus. But you don't have any bleeding or fibroid so nothing to worry about that. If you have post menopausal bleeding then go for pelvic ultrasound and endometrial biopsy. Otherwise if you don't have any symptoms then nothing to worry. Just be relaxed. Hope I have answered your question, if you have doubt then I will be happy to answer. Thanks for using health care magic. Wish you a very good health."
},
{
"id": 114357,
"tgt": "What causes elevated WBC count and swelling under the chin?",
"src": "Patient: My white blood count is up some I have a cyst in my sinus cavity on the left side and swelling around my coller bone on the left side and swelling under my chin on the left side with cold burn feeling and sweating a lot on my neck and forehead I have had X-ray cr scan blood work what could it be Doctor: Hello and Welcome to \u2018Ask A Doctor\u2019 service. I have reviewed your query and here is my advice. High white blood count can be due to infection, stress, bone marrow disease/immune disorder or some type of cancer or even medication with corticosteroids. Since you are already diagnosed with sinus, infection may be a cause of the swelling. You might be having swollen lymph nodes. Consult an ENT specialist, get all investigations & tests done. Your treatment will depends on the reports of the investigations done. Hope I have answered your query. Let me know if I can assist you further."
},
{
"id": 133660,
"tgt": "Suggest treatment for lumbar compression fracture",
"src": "Patient: Hi, my name is Tonya. My 85 year old uncle lives with me. Last week he took a fall and was cleared in an ER. He has a lumbar compression fracture and has been taking 5mg oxycodone twice daily and limited 325mg acetaminophen once daily. He has several comorbidities which include diabetes, chronic kidney disease, and multiple cardiac issues. He is on 3mg warfarin daily, 0.125mg digoxin every other day. There are a few other meds along with vitamins that he takes. The long and short is he has not had a bowel movement in 6 days. He normally evacuates his bowel 2-3 times daily. What can I give him besides his 250mg daily dose of Colace? I am extremely cautious in what I give him because both bactrim and acyclovir have shut his kidneys down. We have had to do extended hospital stays with one time dialysis sessions to get him straightened out. Thank you for your assistance. Doctor: Hi,thank you for providing the brief history of your uncle.A thorough clinical neuromuscular assessment is advised with MRI of lumbar spine. Since your uncle is 85 years old, and had a fall, he needs a proper neuromuscular assessment, as this fall may disrupt his functional ability. Also, by understanding the proper neurological signs and symptoms one can come to conclusion of what to do.Usually, if there is absence of neurological signs and symptoms, then with simple physical therapy the patient responds well. As there is a compression fracture, taking proper bed rest is of due importance.RegardsJay Indravadan Patel"
},
{
"id": 3000,
"tgt": "Is there any chances for getting pregnant without ejaculation?",
"src": "Patient: My boyfriend and I had sex last night and for the first few minutes he didn t wear a condom. He put one on and proceeded, then ejaculated. The condom didn t break. My concern is those first few minutes. I took an ovulation test and I am ovulating. Is it pointless to buy plan b? Doctor: Hello and Welcome to \u2018Ask A Doctor\u2019 service. I have reviewed your query and here is my advice. I think there is no use of buying plan B, as the period during which he was unprotected is the duration during which there was no expulsion of semen or sperm at all. This in my opinion does not warrant any cause for concern.Hope I have answered your query. Let me know if I can assist you further.Best wishes,Dr. Shoaib Khan"
},
{
"id": 70903,
"tgt": "What causes pain and swelling in the lungs?",
"src": "Patient: Hellow my name is adan mohamed from mandera kenya My right lung is swollen i have taken x-ray the lungs are fine but there is something in the lung and its swollen and it also causes sharp pain assist me you could also reach me in my email address YYYY@YYYY namaste Doctor: Hello and Welcome to \u2018Ask A Doctor\u2019 service. I have reviewed your query and here is my advice. First of all no need to worry for lung diseases as your chest x ray is normal. Possibility of musculoskeletal pain is more likely. So follow these steps for better symptomatic relief. 1. Apply warm water pad on affected areas of chest. 2. Take painkiller and muscle relaxant drugs like ibuprofen and thiocolchicoside. 3. Avoid heavyweight lifting and strenuous exercise. Avoid sudden jerky movements of chest. 4. Sit with proper back support. Don't worry, you will be alright with all these in 1-2 days. Hope I have answered your query. Let me know if I can assist you further."
},
{
"id": 212053,
"tgt": "Ativan and oleanz prescribed after having stress. Disturbed by negative thoughts. How to help?",
"src": "Patient: Hello, 3 months ago my friend had a showdown at work and as a result of a lot of stress. I took him to a psychologist who recommended him Ativan and Oleanz Plus. He is 35, male and works in a industry requiring long hours. since that episode it has been 3 months and he has these flare ups. He stopped the meds within a week, and is trying to engage himself socially more. but negative thoughts seems to be hounding him again and he refuses to see a counsellor. His thoughts are random, like he is being spied on, or people are out there to get him, or he doesnt \"know what is gioing on\" and thinks others know more about him and not telling him that. How can i get him to confront this. Doctor: Hi,Thanks for using healthcare magic.I have gone through your available history.Your friend seems to have developed schizophreniform disorder or adjustment disorder with psychotic features.He require medication as well as counselling. If he does not agree to consult psychiatrist, you can give some medications like olanzapine or risperidone out of knowledge of the patient. You may have prescription from your local doctor for the same.Thanks."
},
{
"id": 133869,
"tgt": "Is joint pain due to rosuvas?",
"src": "Patient: Hi, I am 70 and taking Rosuvas 10 since 4 months. Recently I noticed joint pain in the knee more while beginning to walk after sitting for a few hours. Could this joint pain be due to Roruvas 10. Instead of taking these daily, if taken alternate days while monitoring LDL help? Doctor: hi,thank you for providing the brief history of you.A thorough musculoskeletal assessment is advised .As your age is 70, the symptoms of knee pain are ageing related issues. Also, the medicine has its own pharmacokinetics which can possible disturb the metabolism & the physiological reaction at most times are not documented. you can continue the medicine for the prescribed number of days and to check if the pain in the knee joint is because of the medicine, you can stop for 3 days and see if the joint pain comes down, if the joint pain persists then the medicine has no side effects and you can continue.Also, perform regular general body exercises to improve the metabolism, immune system and strength. Body needs medicine to control the abnormal symptoms, but in ageing you need exercises to keep the ADL at optimum.In my clinical experience, I deal with 99% of geriatrics and with regular exercises they have improved metabolism, improved immune system, improved drug absorption etc.I advice you to perform regular exercises to fight the ageing process.RegardsJay Indravadan Patel"
},
{
"id": 36118,
"tgt": "Is tiredness, hearing and sight problem due to herpes virus?",
"src": "Patient: I was diagnosed with herpes zoster virus around my eye back in December since then I have noticed my hearing and eyesight on that side becoming worse I have also had irregular periods feel the cold a lot more than others but seem to have trouble regulating my body temperature as it keeps creeping up to 38.6. I constantly feel tired however I have been told it is because of the herpes virus I am currently taking antiviral medication carbomazipine and naproxen all for pain in my lymph nodes however there is no swelling present. Do you think this is all related to the herpes virus I had 4 months ago or something new altogether? Doctor: Thanks for asking i healthcaremagic forumIn short: you may be suffering from Post herpetic neuralgiaExplanation: As you said herpes virus can involve ear and eye causing hearing loss and corneal erosion. But it has to be confirmed by examination by a doctor. Herpes can cause pain post infection for many months to years in the distribution of the sensory nerve. I think your doctor has prescribed you carbamezepine and naproxen for that only. Go to your doctor and get yourself investigated for any new problem which may be causing tiredness and rise in temperature. All the best."
},
{
"id": 135993,
"tgt": "Suggest remedies for swelling and fluid deposition in bursae",
"src": "Patient: I injured the bersae in my elbo on a weight machine at the gym\u2026my bersae is inflamed, but not infected, yet fluid filled and aggravating. It s been two weeks or more. Will the fluid dissipate on its own or should I be more proactive and get it checked out and fluid drawn? Doctor: hiEither way , it can be treated with aspiration or excision if it refils after aspiration or pains again.suitable coverage with antibiotics may be needed. try to rest elbow in a sling.best wishes"
},
{
"id": 179395,
"tgt": "What causes red, swollen knees with horse fly bite?",
"src": "Patient: hello I brought my 4yr old to the nature center and I had gotten a horse fly bite I noticed a bite behind his knee there is a red dot with swelling and redness across the back of his knee how can I tell if it is an allergic reaction or if it is something I should worry about and also what can I do to treat the affected area safely Doctor: Hello. I just read through your question. Generally, any reaction to a bug bite, big or small, can be considered an allergic reaction. Redness and swelling to the area of a bite is expected. Cool compresses to the area typically help. They resolve on their own after a few days."
},
{
"id": 200904,
"tgt": "What do red spots on penis indicate?",
"src": "Patient: Hello Sir, My penis top point has red marks and it comes for few days and then goes. i also have a problem that when i involve sexually with my girl, the 1st trip i cum within 15min and then for 2nd trip i sometimes fail to satisfy her because my penis doesnt erect properly. kindly suggest me a rapid solution for the same. Thanks Dhruv Doctor: Thanks for asking in healthcaremagic forumIn short: Maybe due to friction/erosionExplanation: If its a ulcer/swelling/growth then visit a doctor. Otherwise it may be because of friction. Give time between the episodes of intercourse, your body and organs require time to recover and again to act."
},
{
"id": 123346,
"tgt": "What causes pain on shoulders, neck and wrist?",
"src": "Patient: i started getting pains about four weeks ago in my shoulders every morning.pains are getting worse waking me upthrough the night also pains in my neck and sometimes in my right wrist.can feel pain all through the day but much worse in the night and when first get up in the morning Doctor: Hello, It sounds like a Cervical Radiculopathy. During this, there is a change in the spinal column normal anatomical position and a disc Degenerative changes leading to nerve root entrapment. Need an MRI to confirm the diagnosis. Using cervical soft collar will be helpful to stabilize the spinal column. Hot water fermentation will help reduce muscle inflammation. Static neck exercise, inter-scapular muscle strengthening exercises, shoulder stabilization exercises and upper limb strengthening exercises are advised. Hope I have answered your query. Let me know if I can assist you further. Regards, Jay Indravadan Patel, Physical Therapist or Physiotherapist"
},
{
"id": 93517,
"tgt": "Pain above the belly button. Polyp in the bladder. Are these related?",
"src": "Patient: I'm having pain about one or so inches above the belly button and the worst is just slightly to the right. I'm aware I have a palyp in my bladder (so far hasn't grown in 3 years), but pain isn't in same spot/doesn't feel the same. I don't feel any lumps but pressure hurts more. I consume only about a tablespoon of dairy each day (creamer). I also try to be careful otherwise w/diet.Any ideas of what it could be and what to do for it? Doctor: Hello,Lower abdominal pain for long time.Of late the intensity of the pain increases and the area of spread is more.You are a known as of polyp of the urinary bladder.Re Examine the polyp for any increase in size, pressure effects, ulceration and infection.If the polyp appears to be same like before than other causes has to be looked for."
},
{
"id": 28814,
"tgt": "How can a hematoma at the incision site be treated?",
"src": "Patient: My husband recently had right leg 4 1/2 hour vascular surgery. About 2 weeks later he developed a hematoma to the left of the incision, which they confirmed with an ultra sound, they put him on antibiotics at home and complete bed rest, it got worse and they put him in the hospital on iv antibiotics for 5 days, he came home and has been home about 4 days on antibiotics and it seem to be getting worse, he doesn't get back to the doctor until March 8 and his hemoglobin is 7.9 and potassium is 4.1, can you give me any more suggestions, he has been on bed rest now since he got back home but I am worried. Charlene Doctor: Hello and Welcome to \u2018Ask A Doctor\u2019 service.I have reviewed your query and here is my advice.As per my clinical experience, the condition is not under control with medicines and rest.He requires surgery of Hematoma in form of Incision and Drainage without delay, which will cause further problems.Hope I have answered your query. Let me know if I can assist you further.Regards,Dr. Purva Patel"
},
{
"id": 26551,
"tgt": "What causes fast and heavy heart beat ?",
"src": "Patient: I am a 12 year old male. As early as 7years I find that my heart beat so heavy and fast some times. It feels very uncomfortable it some times force me to cough to feel better. It been happening more often as I get older. Like every 2 months. It last for about 3o secs. Can you please help. Doctor: Hello and thank you for using HCM. I carefully read your question and I understand your concern.Don't worry, it is nothing serious.There are different reasons why someone experiences fast and heavy heart beats. It might be a simple sinus tachycardia. This means that your heart beats over 100 beats for minute. We expect that your maximal heart rate must be 220 minus your age.It might be a different rhythm issue.It might be what we call supra ventricular tachycardia. You might feel that your heart begins running fast and hard suddenly for some seconds and it goes away. This tachycardia s might happen because of simple emotional stress, anemia, hyperthyroidism, a congenital heart anomaly ect.it is quite known in cardiology that Jung ages experience this kind of palpitations and they are nothing serious.So, my opinion as a cardiologist is that you should do an electrocardiogram, an echo-cardiograph and a holter monitoring just be be sure.Mean wile avoid caffeine products because they rise heart frequenciesHope I was helpful. Best regards."
},
{
"id": 12033,
"tgt": "Why do i have red dandruff like spots on nose ?",
"src": "Patient: hi........i complain from red spots and some things like dandruff on my nose..is there a treatment? male-60 years old complain from red spots on my nose .i used alot of creams but the problem once i stop them it come back again,,so is there a treatment? Doctor: hello tell me please it is itching or not.its is type of allergy.i advice you to go for skin prick test.because there are lot of reason for being a red spot in skin thanks"
},
{
"id": 44145,
"tgt": "Pus cells in semen, reduced sperm count, motility, erectile dysfunction. On antibiotics. Infertility treatment?",
"src": "Patient: Dear doctor, I am 33 years old married man I have two baby girls but unfortunately due to numerous puss cells in my semen my sperm count reduced to 35 million/cmm and motility reduced to 20%.thats why i am unable to fertile my wife and also facing problem in erection of penis. I have already completed a full course of antibiotic but still not shown any improvement. Plz guide me for further treatment as i am residing in xxxxx Doctor: Hi, Welcome to HCM, I am Dr. Das First of all, you are not primariliy infertile, because you have two baby girl. So, there is secondary infertility. Secondly, you did not give the full report. It is important. 35 million / cumm is not reduced sperm count(normal >20 million/cumm as per WHO.) After completing the antibiotic, did you repeat the test? If not, then do it. If then also pus cell persist then antibiotic should be changed."
},
{
"id": 220367,
"tgt": "Suggest remedy for severe cough in pregnancy",
"src": "Patient: hi im Seane 28 yrs old and MAY31 is my due date. I had a severe dry cough and asthma so my doctor advice me to have a medicine DOXOFYLLINE ANSIMAR . My question is what will happen to the fetus at this 9 month stage now that i am having a terrible cough? Doctor: this teriible cough may be due to respiratory infection may be due to acceleted hypertension or something else..you should consult your physician before taking any decision"
},
{
"id": 76738,
"tgt": "What causes pain in left side of chest and throat infection?",
"src": "Patient: hi..i am havin pain on the left side of the chest and on the back,belo now..below armpits for one month now..i did conssult a doctor who gave me antibiotics for 10 days..i get slight fever sometimes..i had a throat infection the doctor said..can the pain be coz of the cold season? Doctor: Hi thanks for contacting HCM...Yes you could have respiratory tract infection viral or bacterial....It could be simple sore throat or bronchitis if cough associated....In such infection muscular pain can occur as constitutional symptom..Along with antibiotic symptomatically management done....Drink more water.Hot water useful rather then cold one Avoid excess fatty and fried food....Avoid smoking....Dust exposure kept minimum...If condition increasing with fever then auscultation for crepitation done with chest x ray to rule out pneumonia like condition.One tsp ginger juice with honey taken twice daily for few days.....Hope your concern solved....Keeping this in mind consult physician for examination...Take care.Dr.Parth"
},
{
"id": 155429,
"tgt": "Will the rectum cancer spread to others?",
"src": "Patient: Hello, I have no knowledge of medical terms. Hence I will put the matter as what I have understood from the doctors Here are the details:- 1)\u00a0\u00a0\u00a0\u00a0\u00a0My father,age-68 years old, was facing problem of blood with stool. So I did colonoscopy which showed that he has Rectum Cancer & it is just 3~5 centimeters from the anus. 2)\u00a0\u00a0\u00a0\u00a0\u00a0Meanwhile, found the cancer is WILD TYPE & not localized & some nodes(which are doubtful) are seen in the liver, lungs as well in the prostates. 3)\u00a0\u00a0\u00a0\u00a0\u00a0So doctors advised to check him for the targeted(Neo-adjuvant) therapy & sent the biopsy to the laboratory & found that he is suitable for ERBITUX-600mg doses. 4)\u00a0\u00a0\u00a0\u00a0\u00a0Now total 4 chemos (FOLFOX with ERBITUX) is done & patient\u2019s health seems to be well responding. 5)\u00a0\u00a0\u00a0\u00a0\u00a0Again doctors are advising to have the Sonography. 6)\u00a0\u00a0\u00a0\u00a0\u00a0Doctors saying is that he will try to kill the doubtful nodes, will shrink the tumor which is present in the rectum & it will be easy to operate on it. My queries are below:- 1)\u00a0\u00a0\u00a0\u00a0\u00a0Are we on right track 2)\u00a0\u00a0\u00a0\u00a0\u00a0Will this therapy help to recover fully at this time though it may be a stage-4 cancer. 3)\u00a0\u00a0\u00a0\u00a0\u00a0Whether the cancer can spread to the others as my father lives with me & I have a small child. Will it be in danger or will all of us be in danger\u2026? 4)\u00a0\u00a0\u00a0\u00a0\u00a0Will just sonography help to come to conclusion to see whether patient is responding or not\u2026 5)\u00a0\u00a0\u00a0\u00a0\u00a0Depending on the above scenario, can you generally comment on the life span of such patients\u2026 Doctor: First of all, your father's treatment has been standard and correct. This is the standard therapy for stage 4 rectal cancer. The chances of cure in stage 4 rectal cancer with full treatment is 25%, i.e. 25% patients will fully recover while rest will not. Cancer is not a contagious disease and does not spread from one to another by touch or contact. Touching him, playing with him, eating in the same plate does not put you at risk. Sonography is not at all enough to detect the response to the chemotherapy. You need to get a contrast enhanced CT scan of the abdomen and pelvis. As to the life span, it will depend upon the response to chemotherapy. If he is a good responder, and all of his disease is subsequently removed by surgery, then he has a 1 in 4 chance of living his normal life. Else, his lifespan will be 6-12 months."
},
{
"id": 193595,
"tgt": "What is the procedure of cutting skin off penis called?",
"src": "Patient: my question for my son 13 yrs old , I want to take him for penis mature before they entry to high school. I came from small island and before we go to high school we all go to the doctor, the doctor cut the little skin of the penis , approx 1 week and go every day to the beach to make it health, how to you call that and please advise Doctor: Hello, The procedure of cutting penis skin is called circumcision. You can consult any surgeon to do the procedure. Hope I have answered your query. Let me know if I can assist you further. Take care Regards, Dr K. V. Anand, Psychologist"
},
{
"id": 67371,
"tgt": "Suggest treatment for a lump in the breast",
"src": "Patient: I have a lump in my breast, I felt it and I let it slide. Today I noticed that the lump is now purple and dark and it is sore. I have read that if you re about to start your period you may get random lumps due to hormones. I am not sure what this is. I am 19 years old. Doctor: Hi young girl.This is very true that one gets changes in breasts as per the hormonal changes, one can have. In your case , this looks to be a fibrocystic disease and nothing to worry about cancer at this age and stage of life. Consult a General surgeon , just in case , to rule out impending abscess or so as the fibrocystic disease can not change to purple and dark- if so you have to take antibiotics and anti-inflammatory medicines."
},
{
"id": 88544,
"tgt": "What causes excruciating pain on the right side of the stomach?",
"src": "Patient: Hello.For the past 4-5 days, the right side of my stomach has hurt. It's not excruciating pain but it hurts. Made worse when I press on it. I have also had an on and off low grade fever (around 99.5 degrees) and nausea. Also, the right side of my stomach feels harder than the left side. Doctor: Based on your description, you seem to be suffering from gastritis. If that is the case, you would need to take omeprazole capsule once daily before breakfast for one and a half months. You also need to take small but frequent meals. However, the final diagnosis can be made only after proper examination and I strongly insist that you visit a doctor and get yourself examined.The low grade on and off fever needs to be investigated and is probably unrelated to your abdominal pain. You would need to get it investigated to rule out conditions like tuberculosis which could be a possible cause."
},
{
"id": 44448,
"tgt": "Trying to conceive. Signs of PCOS. Semen analysis shows less motility and few dead sperms. Any medication?",
"src": "Patient: Dear Doctor, i am female 28yrs, me and my husband are desperately trying for a baby for last 9 months , we will be going to celebrate our 3rd anniversary this July. I consulted a gynaecologist after examination i come to know that i have sign of pcos my height is 5.2 and weight was 58kg in feb after exercise and healthy fat free diet now i am 53kg. Doctor prescribed me clomifene , Acetylcysteine tablets with Astaxanthin for one month and asked my husband to do Semen analysis his reports shows the following results: Volume : 3.0ml viscosity: Normal sperm count : 120,000,000ml viability :70% Motility after 1 hr Active motility : 40% sluggish motility : 30% dead sperm : 30% Motility after 2hrs active : 30% sluggish : 30% dead sperm:40% Microscopic exam: Agglutination : +2 RBC : 1-2 WBC : 1-2 Epithelial cells: nil immature germ cell : - parasites : nil spermatogenic cells : 1-3 abnormal forms : 20 % i am on my 11th day of periods as my ovulation days are near kindly suggest me what to do as i want to conceive in this two months , plz do prescribe any medicines if my husband require.we are currently in middle east. Thanking you in anticipation. Doctor: Hi If your periods are regular then you must be ovulating in 14 days prior to your next menses. That means if you expect your menses on 30th of a particular month then you will be ovulating on 16th of that month. Your ovum must be viable for atleast 24 to 72 hrs. So during this period you can do serial trans vaginal ultra sound to monitor ovum release and its maturation. Your husband' s semen analysis report is relatively normal. Both of you now dont need any medicines. You need perfect relaxation, rest, nutritious food and keep your mind out of tension. Having sex at the time of ovulation and upto 3 days after ovulation is very important... Wish you both all the best.. Dr. Mohammed Ibrahim Surgical Oncologist mdibrahimdr@yahoo.co.in"
},
{
"id": 148405,
"tgt": "Hair loss, wrist weakness. Taking sertraline and thyroxine. Normal?",
"src": "Patient: I was told by my hairdresser I had lost quarter to half of my hair since may. I had blood tests done which have came back iron deficient & prescribed iron tablets. I have also been experiencing wrist weakness in both wrists & wondered if this is normal. I also take sertraline for post-natal depression & thyroxine 100mcg, TS3 results not in yet but slightly low on TS4. I had my thyroid removed at age 16yrs. Sadly I cannot pay for this, I ll just go back to my GP on NHS Thank you for your time Doctor: Welcome to HCM!I have read the question carefully and from my clinical experince I can say that almost all problems - hair fall, wrist weakness, iron deficiency, and depression , are related to thyroid deficiency in your case.It is well established fact that hypo thyroidism, I.e., high TSH, and low T 3 & T4 levels, are related to hair fall, weight gain, increase lipid, lethargy, low mood, weakness, calcium loss, and low iron. the wrist problem or any bony problem. can occur in you as parathyroid gland are also removed when thyroid is removed. Get calcium & phosphorus levels done, along with TSH regularly.I think you need to increase the thyroxine supplement and use more of iodised foods , like- salt. Avoid vegetables like- cabbage, ect, as they increase the problem further.you have been taking iron supplement, along with that you may get help by eating jaggery, more of green leafy vegetables, citrus fruits, nuts, to increase formation of iron and blood. Iton is absorbed more when taken with citrus fruits or water, in empty stomach, never with milk.You may start calcium suppleamt for wrist weakness. Continue with antidepressant, as this will help you.For your hair, you may start a good multivitamin, vitamin A, and most important is thyroid supplement.So, meet your doctor and get adequate thyroid medicines for your self, this is the most important drug for all your problems.Hope the reply is useful for youget well soon!Dr. Manisha Gopal"
},
{
"id": 115851,
"tgt": "What causes high bilirubin level in blood?",
"src": "Patient: I have had my blood tested twice in the last 2 months and it is coming back unsatisfactory on the Bilirubin being high. I went to the doctors initially because of itching skin all over , this has been ongoing for a few months now- the doctors response is to tell me to get my blood checked again in 6months time thats it - should I be concerned, should i be asking for more to be checked at this time? Doctor: Hi, dearI have gone through your question. I can understand your concern.You may have some liver disease. Itching is common with high bilirubin level. You should go for complete liver function test once. Then you should take treatment accordingly.Hope I have answered your question, if you have any doubts then contact me at bit.ly/Drsanghvihardik, I will be happy to answer you.Thanks for using health care magic.Wish you a very good health."
},
{
"id": 218933,
"tgt": "Is pregnancy possible when the uterus is small?",
"src": "Patient: i am 27 years old and will complete 2 years of marriage in august. i am planning for a family since a year,but results are negative. my menses are irregular and have to take medicine fo most of the time to get menses. i go for a checkup and doctor says utreus is small. and today i go for checkup again same result. i am worried,i would like to know whether i will be able to conceive or not. please help me out Doctor: Hi, I have gone through your question and understand your concerns.Irregular periods can be due to hormonal disturbances like hypothyroidism, hyperprolactemia or polycystic ovarian disease. These three are also the commonest cause for infertility.I would suggest you to consult an infertility specialist to get properly examined, investigated including hormonal profile and ultrasound and get appropriate treatment.Hope you found the answer helpful.Wishing you good health.Dr Deepti Verma"
},
{
"id": 164588,
"tgt": "What causes patches of bumps in child?",
"src": "Patient: Hi, I have noticed small round patches of small bumps on my 1 year old. They are not red and don t seem to itch her. I first saw some on her shoulders a few weeks back and put some lotion on it and it seemed to clear up. Then, just the other day, I noticed two more on the bottom of her back towards her side. I don t know if it s related, but I had ringworm under my breast a few weeks ago. I bought the cream and it cleared right up. It seems like I noticed her spots after I got the ringworm. But it may have been just because I was trying to be more aware and to make sure she didn t develope ringworm also. So she could have had them before and I just didn t notice them. Well, tonight, I noticed I have the exact same looking thing on my arm. Like I said, it s a small, round patch of tiny bumps. They re not red, they don t itch. The bumps don t look like pimples and they re not filled with puss, just tiny little bumps. Any ideas?? Thanks in advance! Doctor: Hi...Thank you for consulting in Health Care magic. Skin conditions are best diagnosed only after seeing directly. I suggest you to upload photographs of the same on this website, so that I can guide you scientifically.Hope my answer was helpful for you. I am happy to help any time. Further clarifications and consultations on Health care magic are welcome. If you do not have any clarifications, you can close the discussion and rate the answer. Wish your kid good health.Regards - Dr. Sumanth MBBS., DCH., DNB (Paed).,"
},
{
"id": 70410,
"tgt": "What could be the reason for having a lump in chest?",
"src": "Patient: Hi, i had a lump in chest like a spot but wouldnt go away, eventually i thought this was a cyst as i had a cyst removed on my back a while ago.Anyway i looked lastnight and it seemed to have a white head so squeezed and squeezed and out came a long line of stringy stuff that smelt what is it? Doctor: Hi,Thanks for writing to HCM.That was most probably a sebaceous cyst, which you have drained partially.Now you need to see a surgeon to get it completely excised or else it will reoccur or may even get infected.Dr. Ashish Verma"
},
{
"id": 219535,
"tgt": "Does delayed periods indicates pregnancy?",
"src": "Patient: hi.i had intercourse and on the very next day i had my periods. so i dint take any contraceptive medication. i had period the next month too. but this month my periods have been delayed by a week. i took pregnancy test twice but it was negative. are there chances that i might be pregnant and if i am , i am not ready for baby so how can i get rid of it? Doctor: Hi.You had unprotected sexual intercourse almost 2 months ago and had your period twice? And since then you did not have unprotected sexual intercourse again? Yes ma'am, pregnancy can clearly be ruled out. Speak to your doctor if you do not have your period in another few days or so.Best wishes."
},
{
"id": 78173,
"tgt": "Suggest treatment for sore throat and chest congestion",
"src": "Patient: I don t know if I have the common cold or not. last week I started with a bad sore throat. It hurt to swallow. then the next day congestion started and still going on for 3 days now. I have also been feeling tired but am resting. also ive been taking Nyquil. it might be my allergies. idk Doctor: Hi. I can understand your concern. It must be a viral infection try stem inhalation and warm salt water gargling. Also drink lots of fluids. Take fruits rich in vitamin c like mausambi, orange etc which will help you come over the infection fasterDon't worry, you will be alright. Hope I have solved your query. Wish you good health. Thanks."
},
{
"id": 59632,
"tgt": "Suffering from liver cirrhosis. Reached Grade3. Taking laxative. No motions, no control over bladder. Hospitalize?",
"src": "Patient: my father in law has liver cirrhosis . he is currently based in patna, bihar. his condition is gradually deteriorating with the doctors saying that he has reached grade 3. wbc count is 3500; haemoglobin- 6.9;plateletes- 1.2 lacs; urea-normal;sodium-134;potassium-3.8;chloride-normal;bilirubin-2.8;calcium-0.7. he is taking hepamerz capsules with lactyhep as the laxative. though has not had any motions since 2-3 days with no control over his bladder . what do we do now? do we need to bring him to delhi or can his treatment be done in patna? do we need to get him hospitalized? Doctor: Hello, paramita.chatterjee, Sorry to hear about your father-in-law's diagnosis. I am not sure what the underlying cause of his cirrhosis was. Here in Western world, mostly alcohol is responsible for majority of cases. Other are caused by Hepatitis Type-B or Type-C . We are seeing a lot more Hep_C here and fortunately there is a lot of advance regarding early diagnosis and treatment and even cure for this. Your father-in-law's blood work actually does not look very serious. With a Bilirubin level 2.8 mg/dl it is mildly elevated. His Hb, level, white cell count are low but his platelet count is still good. If he does not have a very distended abdomen or markedly swollen legs, then he probably does not have severe \"Portal Hypertension\" which could cause enlarged veins in the esophagus and stomach and lead to massive bleeding and liver coma. From the blood picture it does not appear to be that case. He still may be having slow blood loss if he has esophgeal varices (Enlarged veins) from irritation. He needs to have his stool checked for hidden blood and if he has black stools, he needs endoscopy to look inside the esophagus and stomach. If he has any signs of active bleeding, he needs to be hospitalized and may require blood transfusion and treatment for those veins. This is done by specialists. If your town does not have experts then New Delhi a university hospital may be a good choice. Hepamerz is L-Ornithine-1-Aspartate helpful to heal the liver. I am not sure if Lactyhep syrup has lactulose which is a laxative and helps to get rid of ammonia build up in the blood that leads to liver coma. If you father-in-law has any signs of altered mental status like feeling sleepy after eating high protein foods, that may be an indication for \"Hepatic Encephalopathy\" in different stages. Neomycin and Lactulose and more recently Rifaximin are helpful in treatment. You can discuss all of these with his doctor. I wish him well."
},
{
"id": 198979,
"tgt": "What causes pain in penis after ejaculation?",
"src": "Patient: I had this infection around my penis a few weeks where little sores developed around the penis head. i sought for medical attention from a hospital and the infection stopped. now i have pains every time i release sperm. could the infection still be there or its something else. Doctor: Hello Thai can be related to spread of infection to prostate or genitourinary tractI would advise you for battery of tests like urine culture and an ultrasoundYou may require a course of antibiotics under supervision of your doctorHave plenty of fluids and maintain good hygieneBest wishes"
},
{
"id": 192770,
"tgt": "Suggest remedy for erectile dysfunction, premature ejaculation, thin semen and nightfall",
"src": "Patient: i 28 yr old n doing musterbation since very young age and in beginning frequency was also very higher then i faced the problem of erectional dysfunction, premature ejacuation during musterbation as i hven't done sex so far and problem of thin semen now my semen is less in quantity also and facing the problem of nightfall also i just ejacuate by watching any erotic scenes .most of time my penis doesnt erect while watching erotic scenes plz help me tell me exercise and certain practices which i should follow preferrebly yoga coz i dont want to take medicine for it plz help me i m about to get married Doctor: Hello,It may be due to performance anxiety. For further assessment you may require testosterone level, lipid profile, electrocardiogram after physician consultation. Take balanced diet and proper rest. Avoid smoking and alcohol. Till then you can take tablet sildenafil one hour before sexual act Hope I have answered your query. Let me know if I can assist you further. Regards, Dr Shyam Kale Family and general physician"
},
{
"id": 104008,
"tgt": "Recurrent sores in throat, fever, dry mouth and allergies. Suggest",
"src": "Patient: Hi I have a recurrent sores at the back of my throut always on the right side I also get fevers with it. It's just stated in the last year my moulth is really dry too. I do not smoke or drink and generally in good health. I suffer from allergies through out the year and have hay fever but I don't think that has anything to do with these sores? Please can you help me. Doctor: Hi and thanks for the query, It is very important to get a proper clinical evaluation. This shall be important in distinguishing a poorly treated infection or resistance of the the causative organism to the used anti biotics. this shall be gotten through institution of a recommended treatment protocol for an appropriate duration of time and followed up by your treating physician. Doing a throat swad, culture and antibiotic sensitivity test could be important in diagnosing the appropriate germ and treating it porper. In case its an allergic condition, or it becomes so recurrent, your ENt specialist might vdecide to remove the tonsils. This could be necessary in prevent repeated infections, which could be a strong risk factor for Rheumatic fever (mainly joing and heart complications). I suggest you book an appointment with your ENT specialist for a proper evaluation and management. thanks and best regards, Luchuo, MD."
},
{
"id": 135520,
"tgt": "Are the reactions to the stitches post surgery normal?",
"src": "Patient: Hi I had surgery 1 week ago to put pins and plates into my ankle. After I broke it in 3 places. I m not getting a burning sensation where my stitches are on the right side of my right ankle. The left side feels tight. Is this normal or should I go back to the hospital as I have a back slab on to protect my ankle from moving. So I cant see the stitches. Doctor: dear friendyou must go back to the hospital and have the stitches looked at at times a few of them may have developed infection and may need removal to drain the pus from there."
},
{
"id": 182874,
"tgt": "Suggest medication for severe toothache",
"src": "Patient: Dear Sir I have been experiencing severe toothache for the last three days. On consulting a dentist, I was informed that there is an infection on the tooth and root canal is required. For the time being, I was prescribed Azimax 500 mg(once a day) and Ibugesic plus(twice a day). In spite of taking this medicine for two days, I still experience unbearable pain.Now my dentist has advised keterol dt twice a day. Pse advise Doctor: Hello,Thanks for consulting HCMRead your query as you have severe toothache dont worry treatment of your pain is root canal go for treatment as early as possible , in meantime you take medicine keterol dt dissolve in water that will give you instant releif . Do warm saline gargle two - three times a day. Consult your dentist and go for treatment. Hope this will help you."
},
{
"id": 109669,
"tgt": "Suggest treatment for back pain",
"src": "Patient: have 100 with chills for last two hours, starting saturday extreme pain on right side of neck going down right side of back, also fullness in spleen area (have slightly enlarged spleen_...I have severe erosive gastritis and cirrhosis...do i need to go to ER yet? Doctor: Hi welcome to health care magic. After going through your query my opinions is that you are suffering from disc prolapse/ cervical spondylitis. Have an MRI of cervical spine done. Avoid lifting heavy weight, pillow. start Hales regime physiotherapy for neck. if pain is very severe than wear soft cervical collor and start physiotherapy as soon as pain resolve. Take analgesics, muscle relaxants, hot fomentation. Visit orthopedic surgeon. for your problem of cirrosis and spleen visit gastroentrologist. Hope your query get solved. Welcome further queries"
},
{
"id": 6805,
"tgt": "Why have I missed my period ene though my pregnancy test shows negative ?",
"src": "Patient: hi i had a period on december 26th 2010 and its now febuary 2nd 2011, i had no period since, i have stomach cramps headache feel mega sick and tired i had loweback pain and weird cravings for foods,i done a test to day and its negative as usual. i dont use any birth control and i had problems in the past with irreglar periods but for last 6 months my periods been normal 5 days, but has been light browny colour for a long time Doctor: i adivce you to visit a doctor.. they a can guide you on a regular dose of medicines. there are also many ayurvedic medicines available in the market as well."
},
{
"id": 206420,
"tgt": "Suggest medication to improve my concentration",
"src": "Patient: I am a19 year old female. I am currently on zoloft 100 mg for anxiety and depression(for 1 year). I was prescrbied wellbutrin to offset the sexual side effects of the zoloft(about 6 months ago). I still have been having trouble concentrating and focusing on my school work. In school, it is difficult to focus for the duration of the class, I am usually day dreaming and simple assignments take me much longer to complete than it should. Outside of school, I have difficulty making simple decisions. It takes me a while to choose what I would like for dinner, minutes to decide which cereal I want or what flavor yogurt. Is there any medication that could help wth my attentiveness and ability to focus? Yet, can still interact with the medication I am currently on? Doctor: DearWe understand your concernsI went through your details. I suggest you not to worry much. There are no medication to improve your concentration. You are suppose to refer to concentration and attention as two. Attention span for any adult is maximum 30 minutes. Sometimes, a person can be attentive to an interesting thing for more than 30 minutes, such as movies or games. Concentration is attention plus understanding. For this too, span os below 30 minutes. If you are able to make your studies interesting and based on variable principle, your concentration level can be increased. Concentration comes only when you are interested in what you do. So please be passionate towards what you are doing or do only those things which you are interested in. It is also possible to improve your concentration level with the help of yoga, meditation and breathing exercises.Psychotherapy techniques should suit your requirement. If you require more of my help in this aspect, Please post a direct question to me in this URL. http://goo.gl/aYW2pR. Make sure that you include every minute details possible. I shall prescribe the needed psychotherapy techniques.Hope this answers your query. Available for further clarifications.Good luck."
},
{
"id": 92343,
"tgt": "How to overcome the pulsating pain in my abdomen after playing soccer?",
"src": "Patient: I get this pulsating pain in my abdomen after I play soccer. I played soccer all my life, and I never felt pain like this. I went in for a ultra sound but they said everything was normal. It pulsates at the height of the belly button on the front side of the abdomen. It goes side to side. From the right or left side to the other side and it switches back and forth. I end up on the ground rolling side to side. Its a real sharp pain. It last for about 30 minutes. Real bad pain Doctor: Hi,Welcome to healthcare Magic!short duration recurrent pain in the athletic and outdoor active people may be due to acid peptic disorder or small ureteric stones or simply due to gut infection leading to excessive gas formation causing spasms.You should avoid fried foods and junk foods.Get a urine routine and culture examination done and also KUB ultrasound to detect any small stone or gravel that can be missed in routine abdomen ultrasound.Also get a stool examination and culture.Review to a local physician and manage accordingly.I hope the advise would be informative and useful for you.Take Care!"
},
{
"id": 124957,
"tgt": "Suggest treatment for swollen shinbone",
"src": "Patient: I hit my shin on something a couple days ago and it is getting worded and worse I can t even walk on it and from half way down my shin to my ankle is swollen and red, I was able to squeeze it and pus came out any ideas! When I bumped my shin it didn t brake the skin Doctor: Hello, It could be a contusion or hematoma. As of now, you can use analgesics/anti-inflammatory combination like aceclofenac/serratiopeptidase for symptomatic relief. You can also apply ice packs for faster recovery. If symptoms persist you can consult an orthopedician and get evaluated. Hope I have answered your query. Let me know if I can assist you further. Regards, Dr. Shinas Hussain, General & Family Physician"
},
{
"id": 105261,
"tgt": "Chest discomfort, bronchial oedema and heaviness on suprasternal notch area. Reason?",
"src": "Patient: 20y male with h/o smoking for past 2 years an average of roughly about 6 a day, started experiencing chest discomfort mainly seemed like bronchial oedema and heaviness around the suprasternal notch area, went to a chest medicine doctor and had my spirometery and FeNO done which was roughly about 44ppb and spirometery showed the mef25-75 i.e the small vessels were effected..which showed improvement on asthalin inhalation .. doctor has prescribed formoterol+ budesonide rotacaps medication, also had an allergen sensitivity skin prick test done which showed im sensitive to spider web dust, dust mite and cockroach i have the following queries 1. what is the most probably diagnosis? (asthma/bronchitis/other inflamatory condition) 2. can the medication be taken SOS and does it have an dependancy or rebound phenomenon esp down regulation of the b2 receptor in my bronchi? 3. now if it is asthma or setting of COPD changes, there is remodelling of the bronchial system, will this drug retard the the process and restore it back to normal? 4. how long am i supposed to stay on medication? also due to rotacap medication is it likely that thepowder may irritate the air passage and cause a bronchoconstriction instead of dilatation? 5. could u suggest some exercises or processes to improve my lung function? and also to reduce the bronchial oedema over a longer duration? thank you for your time :) Doctor: at your age the smoking cannot harm in 2 years it is some other allergy exagerated by smoking these allergies are comon for milk,wheat,egg ,potato,chana,rice you get blood serum test specific antibodies for these foods in the meantime take singular 10 mgm bd tab allegra 120 mgm bd syp ventrolin 1/2 tsf bd syo tossex 1/2 tsf night applu neospotin h eye onitment in nose sea water 2 drops at night in eacxh nose 3 week post after tests"
},
{
"id": 82442,
"tgt": "What causes swelling in the middle of the chest?",
"src": "Patient: I have an area of swelling that has caused me some concern. It is located in the middle of my chest directly under my collarbone. There is more swelling on the right side than the left. There is a little tenderness, particularly if I inhale deeply. I was wondering what it could possibly be and if I should be concerned about it? Doctor: Thanks for your question on HCM. In my opinion you should get done ultrasound examination of neck region.As swelling in the area , you are mentioning can be due to 1. Lymphnodes2. Thyroid3. Bone swelling etc.So ultrasound will diagnose these.In my opinion it is lymphnode only. As cervical lymphnodes are commonly increased in infection.So get done ultrasound neck."
},
{
"id": 45307,
"tgt": "I was given super fertyl for day 3 to 8 followed by hcg injection. What are the chances pregnancy ?",
"src": "Patient: hi, I was given super fertyl for day 3 to 8 followed by hcg injection on day 13 and duphaston from day 14 to 28. What are my chances of pregnancy? Doctor: Hello You should also be having a ovulation scan along with your treatment.If you are ovulating well,then chances of pregnancy are very good.It all depends on your follicular growth.However,it may take time,so dont lose heart if you dont concieve the first cycle."
},
{
"id": 86290,
"tgt": "What causes persistent abdominal pain?",
"src": "Patient: Hello, I m a 20 year old pretty healthy female, I m having right abdominal pain. My urine is foggy but it isn t painful. It s been hurting all day, as if I haven t used the bathroom in hours but I have. My period was two weeks ago, so I don t think that s it. Doctor: Hello! Thanks for putting your query in HCM. I am a Gastroenterologist (DM). Abdominal pain on right side may be due to many causes related to liver, gallbladder, billiary tract, stomach or intestine so I will suggest you to visit a Gastroenterologist for evaluation and management.I hope I have answered your query and this will help you. If you have any further query I will be happy to answer that too. Remain in touch and get-well soon."
},
{
"id": 221901,
"tgt": "What causes stoppage in menstruation?",
"src": "Patient: Hello, I have not had a period since November 6th 2010. Its now January 12th 2011 and still no period. I am exhibiting pregnancy symptoms: nausea, vomiting, changes in breast, fatigue, and dizziness. A little over a week ago I tested negative using a hpt. I definently suspect pregnancy. With my youngest child, I tested negative and almost 2wks later I got a positive and at the time I found out I was over 2mnths pregnant. Please help. Thx. Doctor: Hi dear, I have gone through your question and understand your concerns.Delay in the periods can be due to pregnancy, stress or hormonal disturbances like hypothyroidism.If the urine pregnancy test is negative, then pregnancy is ruled out.However, you can do a serum beta hCG levels to definitely confirm whether there is pregnancy or not.Hope you found the answer helpful.Wishing you good health.Dr Deepti Verma"
},
{
"id": 170044,
"tgt": "Suggest treatment for fissures on foreskin of penis in kid",
"src": "Patient: Hello. My 3 year old son has developed fissures on the foreskin of his penis. He did have phimosis as diagnosed by his pediatrict urologist however, we were prescribed betaderm 0.1% and since using it, his foreskin has loosened enough to expose the glands. All was great for the past 6 months. Yesterday evening, our 3 year old son came to us and showed us that there were fissures on the outside of his foreskin and it is too painful to pull the foreskin back. Is it ok to apply the betaderm to this location? Doctor: HiThank you for asking HCM. I have gone through your query. Decreasing in moisturization can be the reason for the fissures. Avoid soaps and irritants to wash the area and instead you can use saline or warm water. You can also apply moisturisers based on liquid paraffin. Applying betaderm ( betamethasone) will be helpful to reverse the phimosis again. If there is no improvement then approach your doctor. Hope this may help you. Let me know if anything not clear. Thanks."
},
{
"id": 52829,
"tgt": "What causes the grabbing, squeezing pain in the gall bladder?",
"src": "Patient: I have a grabbing, squeezing pain in the gallbladder area that makes you feel like you are going to throw up & pass out all at the same time for about 30-60 seconds and then it is a burning sensation off & on. Had a Lapchole in 2011, what could this be ? Doctor: Hi and welcome to Healthcaremagic. Thank you for your query. I am Dr. Rommstein, I understand your concerns and I will try to help you as much as I can.Your symptoms may be suggestive of gallbladder stones so you should do at least ultrasound or CT scan to evaluate it more accurately. This is caused by stone in bile duct and is very common condition. Stones are see more commonly in obese people and those with diabetes. You need to change your dietary habits first. Need to avoid fried food, carbonated drinks, coffee, alcohol and spicy food. You should eat more milk products, vegetables and boiled food. If there is no improvement on lifestyle or dietary changes, then surgery is recommended. Other causes include pancreatitis or pleuordinia and should be considered if gallbaldder stones are ruled out.I hope I have answered you query. If you have any further questions you can contact us in every time.Kindly regards. Wish you a good health."
},
{
"id": 54144,
"tgt": "Suggest treatment to reduce alkaline phosphatase levels in blood",
"src": "Patient: I just had a routine blood test and my Alkaline Phosphate came back a little high, 261 (out of the range of 25-150) but my AST & ALT are low normal? I have IBS but other than that completely healthy. What can I do to improve this number or is this just normal for me? Doctor: HelloI see no reason to worry about this.The elevation is minor and since the other liver enzymes are normal, there's no indication of a liver problem.Were I your doctor, I would just repeat the tests in about three months"
},
{
"id": 189533,
"tgt": "11 year old lost baby tooth. Flesh hanging near the permanent tooth. Is this serious?",
"src": "Patient: My daughter who is 11 has just lost her baby tooth ( first one next to the molar tooth). There is something that looks like a piece of flesh on one side of her permanent tooth, which is fully out. First, I thought it might have been a piece of food, tried to gently remove it, but she said it was painful, and I did not want to touch it any more. I phoned my dentist , and he will try to make an appointment for tomorrow. Is this something serious/dangerous, something I should be worried about. It is 15 min. past midnight where I am, but I cannot sleep, I am so worried.Sleepless and concerned mom. Doctor: Dear friend. Thanks for sharing your concern. Need not worry at all.you can ask her to do mild saline rinse before she goes to bed. at this age you can expect erupting,her permanent premolars.this premolars come in position of previous baby molars. therefore the bulge and piece of flesh could be due to that. there is nothing to worry here at all,but since you have decided to visit your dentist you can do so and get an routine oral checkup done. hope i have answered your query. thanks. take care."
},
{
"id": 51681,
"tgt": "Can any online doctor tell me the process of kidney transplantation ?",
"src": "Patient: hi i live in pakistan n we need imidiate kidney transplant of my husband.doctors rare saying for dylsis but my husband is not agree for this tell me the process of transplantation and how we come to india as soon as posible my husband age is 36yrs and swelling on his foot and continously motion.his crytinia 13.9 Doctor: please tell me the process and what is the cost of transplantation"
},
{
"id": 46620,
"tgt": "What causes kidney pain and change in smell of urine",
"src": "Patient: pain in left kidney since last night..lower back kind of throbbing pain Also for the last few months change in smell of unrine..not all the time but alot of times is strong smelling and I have a bad taste in my mouth when I do. Does any of this sound familiar? Doctor: Probably you are suffering from urinary infection where you may get loin pain foul smelling urine. You should go for complete urine examination and urine culture for further management. All the best. if you have any questions feel free to contact me."
},
{
"id": 171010,
"tgt": "What causes cough with blood in mucus?",
"src": "Patient: Hello my son is 4 Yr old and was down with chest infection which caused him to hv high fever....fever has subsided but cough and cold is still there I have been giving him a concoction of tulsi,Ginger,pepper and honey 3 times a day and not any medicine....should I continue the same and also today while he was sneezing to remove mucus from nose I saw little blood...pls advise... Doctor: HiWelcome to the HCM I understand your concerns but don't worry. It takes 2-3 weeks for cough to completely improve after chest infections. I would recommend you to start him on antihistamine such as Chlorpheniramine or fexofenadine for cough and cold. Continue the herbs you are giving. Also, use nasal saline drops to prevent crusting and bleeds while blowing nose. Avoid nose picking by your child. Use room humidifier while sleeping if the weather is dry.Hopefully this will help him. I would be happy to help you in any further questions.Take care"
},
{
"id": 114546,
"tgt": "Is Haemoglobin level and WBC count according to blood test the cause of viral fever?",
"src": "Patient: Dear Doctor I am writing this from Dubai, UAE. This is for my Wife aged 29 years . One month back ( 19 october, 2010) she was affected with Viral fever and we consulted a Gynae Doctor ( as we were trying for a second baby) . The Lab results are as below : Haemoglobin \u2013 12.8 G/DL ; WBC count \u2013 10200 Cells/U; Neutrophils \u2013 80%; Lymphocytes \u2013 15%; Eosinophils \u2013 02 %; Monocytes \u2013 03 %; Basophils - -% ; ESR \u2013 40MM. The Gynae Doctor prescribed her NEOMOL CF tablets and Finallerg 10 mg tablets . After taking this for one week her fever was cured but her ears got blocked . We again went to the Gynae Doctor and Doctor adviced her to do some exercise ( like close the nose and push air oustside from mouth) so that blocked ears will open and also to inhale vapour from a nebulizer) and prescribed her NEOCLAV1 g. Still after a week it was not cured and she was suffering with a Buzzing sound from her left ear. Then we consulted an ENT Doctor and the Doctor checked her ears and prescribed her BETASONE (corticosteroid), Clarinaise ( Loratadine 5 mg, Pseudoephedrine Sulphate 120 mg) tablets . After one week the buzzing sound was little bit cured but she was feeling heaviness on her ears ( like pressure coming inside to her left ear ) . As this was not getting well we consulted another ENT Specialist doctor and doctor checked her thoroughly after inserting some equipment in her both ears . The test results are : Left Ear: Peak YA 0.4 mmho ; GR (width) 100 dapa; TPP \u00a0\u00a0\u00a0\u00a0\u00a0 -10; Vea(+200) 0.8cc Right Ear Peak YA 0.4 mmho ; GR (width) 95 dapa; TPP \u00a0\u00a0\u00a0\u00a0\u00a0 -20; Vea(+200) 0.8cc Then Doctor referred for a Audio Test and mentioned that she cannot hear the minor sounds 50% in her left ear ( I will forward you the audio test after scanning if required) . Then the Doctor prescribed her the following medicines : BETASERC 16 mg 60\u2019 S \u2013 10days ( 1-0-1); FIXIT 300 mg CAPS 14\u2019s \u2013 7 Days ( 1-0-1) TRENTAL 400 mg TAB 20\u2019- 14 days ( 1-0-1) , LOVIR 800 TAB 50\u2019s \u2013 7 days ( 1-1-1-1-1) GUPISONE 5 Mg TAB 20\u2019s (4-4-4 for 4 days); (3-3-3 for 4 days); (2-2-2 for 4 days) ; (2-0-2 for 3 days) ; (1-1-1 for 3 days) ; ( 1-0-1 for 3 days ) ; ( 1-0-0 for 3 days) Now she is taking only Gupisone as the other medicines which were prescribed already finished the dosages. After one week she discontinued the TRENTAL tablets as she was infected in her left hand a reddish allergy and itchiness with pain . The doctors said that it may be bacuse of Trental . Still the buzzing sound and pressure in her ears and pain around the left ear persists. Our query is this curable after taking these medicines as we are afraid that she will put on more weight after taking these tablets . Also how long it will take to cure this type of disease and whether this tablets will affect her body to conceive a second baby . Are the medicines prescribed are correct ones and in your opinion should we travel to India so that a more cost effective treatment can be taken. Should you need any further details please email me or send me a sms and I will call back. Regards JOSHY VARGHESE YYYY@YYYY +0000 Doctor: Hi Joshy, It is not a blood related problem. Kindly post this question in ENT section. With regards, Dr. Girish Kamat"
},
{
"id": 172859,
"tgt": "What does submucus cleft palate with high thyroid and calcium levels indicate?",
"src": "Patient: my daughter is going to turn 8months old on saturday, we have just had some test results for her finding out she has high thyroid and high calcium . on top of this she has a submucus cleft palate. what do these all mean? i would really appreciate some answers as the hospital did not give answers just results. thank you Doctor: Cleft palate is congenital abnormality which require surgical correction. High thyroid means high tsh or high t4.if high tsh is there then it is hypothyroidism and if high t4is there then it is hyperthyroidism. Hypothyroidism is more dangerous. Both require treatment. Actually I want investigations for proper ans. Thanks"
},
{
"id": 108085,
"tgt": "How to cure abdominal discomfort and bloating of stomach after herniated disc surgery?",
"src": "Patient: I am 50 years old and had emergency herniated disc operation 6 weeks ago ( went through my neck) ever since I have had stomach bloating, feeling full all the time, abdominal discomfort above my belly button and short of breath. I had upper and lower scope last week which was normal. The GI wants me to see gynecologist ( I just went off the birth control pill in December and didn t have a cycle until a week and half after my surgery and then again 19 days later. I saw my gynecologist in March and he did a menopause test and I was right at the border. Doctor: Dear Sir/MadamI have gone through your query and read your symptoms.In my opinion, the surgery for the disc cannot lead to your abdominal complains, normally such things can have physical causes, which have been ruled out by by your gastro, moreover your gynae also has no such cause to put it on for your symptoms, to me it feels this could be due drugs, or psychological causes, are there any drugs you are using now, any type, then the second aspect you need to look at is your diet and exercise programme, diet with excess of fats, and proteins can cause such symptoms, and lack of physical activity is another reason, now try this, for next 1 week, avoid all fatty and sweet cooked foods, take more of salads, and fruits, less of salt pepper non veg, and see the difference.I hope that answers your query. If you want any more clarification, contact me back.ThanksDr Narender Saini"
},
{
"id": 1737,
"tgt": "What does \"evidence of dominant follicle in ovary\" on follicular scan mean?",
"src": "Patient: I had undergone serial follicular scan test on 8th day after my periods Below is the result Right ovary - 36 x 23 mm Left Ovary - 29 x 25 mm ET (mm) - 5.9 mm Right ovary (mm) - evidence of a dominant follicle measuring 16 x 16 mm Left ovary (mm) - no dominant follicle POD - free could you tel me whether the resullt is normal? Doctor: Hi, I think you have a normal scan. Dominant follicle means that the follicle which is largest in size and it releases an egg when it reaches a size more than 17 to 18 mm. Egg gets fertilised by sperm and pregnancg happen. So you have got one dominant follicle which is normal. You can be in contact with your husband every 2 to 3 days if you want to get pregnant. Hope I have answered your question. If you have any other query I will be happy to help.Regards Dr khushboo"
},
{
"id": 136314,
"tgt": "Suggest remedy for bruise on forehead due to bumping",
"src": "Patient: I bumped my forehead yesterday and today it feels very bruised to touch but no bruise showing . I also don t feel good at all but I m not sure if I m just over tired as husband is away on work trip and I ve been running around for the past week after my one year old Doctor: Please get done x ray of skull and ct brain if u have feeling of vomiting or if u had history of loss of consciousness aafter injury"
},
{
"id": 45021,
"tgt": "SUFFERING FORM CHLAMYDIA BUT TRYING TO CONCEIVE",
"src": "Patient: I am 31 years old, my daughter is 14 months old and I have been trying to conceive #2 for 4 months. My period returned about 4 months ago, but I breastfed my daughter until December 18, 2010. My pap test showed possible chlamydia infection which was not confirmed by a vaginal culture ( Elisa method). My current ob/gyn said that I need to get a PCR to be 100% sure and that a chlamydia infection could block the fallopian tubes . Is this possible to happen in 14 months? I have had two very easy conceptions in the past and have never had chlamydia infection, plus it is not possible to have contracted it in the past 14 months. Thank you for your answer. Doctor: Hi welcome to Healthcaremagic hi..., after the first pregnancy there is needed of gaping for next pregnancy at least of 2 year.., this is to improve the health status of the mother and good health of next baby.., hence better you need to improve the health and then get cured of the chlamydial disaese and then get conceived... the infection will lead to abortions and congential abnormalities,, you need to get cured completely before getting pregnant... Hope I have answered your question.. Takecare..."
},
{
"id": 57105,
"tgt": "What do the Liver function test, SGPT and SGOT levels, suggest?",
"src": "Patient: my sgot is 41 IU/L AND SGPT 29 IU/L IS I T NORMAL? LIVER FUNCTION TEST TEST DESCRIPTION REFERENCE RANGE FINDINGS SGOT 0-37 IU/L 41 IU/L SGPT 0-40 IU/L 29 IU/L Doctor: Hi,How are you? Your liver tests are totally fine and there is no need for concern,Please let me know if you need any more clarifications. Have a great day"
},
{
"id": 127268,
"tgt": "What causes tenderness in the knees and groin area?",
"src": "Patient: A patient complains of persistent tenderness in the right knee and groin area. The tenderness started a few weeks after he injured the big toe of his right foot , which became red, swollen, warm, and painful. He says the toe improved without medical attention. The knee and groin tenderness remain. What could the physician identify as the cause of the patient\u2019s knee and groin pain? What is a possible treatment? AND WHAT TYPE OF X-RAYS Doctor: Hello and Welcome to \u2018Ask A Doctor\u2019 service. I have reviewed your query and here is my advice. Can be related to an infection in the area or osteoarthritic changes. Direct examination can help. Hope I have answered your query. Let me know if I can assist you further. Regards, Dr. Praveen Tayal"
},
{
"id": 102097,
"tgt": "How to treat chronic cold,sneezing and breathing difficulty?",
"src": "Patient: Hello doctor, I am 25 years old suffering from cold, regular sneezing, hard breath, and there is a whistling sound in my chest from last 2 months when the weather started changing.There is wet cough and I have to spit it after every few minutes.my doctor recommended me brozedex syrup and L montus AB.whenever I stop taking the medicine it comes back.How can I get rid of this condition? As it comes whenever the season changes. Doctor: these medicines are fine but can help to control it but not cureas these are allergies allergy tests can help you find allergenseliminating food allergens and treating aero allergens with immunotherapy can help you in my experience"
},
{
"id": 131629,
"tgt": "What causes pain and soreness between hip bones and bottom ribs?",
"src": "Patient: I am sore/sensitive internally on both sides between my hip bones and bottom ribs...? Had a uti a couple of weeks ago but had it with medication and am fine now as far as those symptoms.. I m 24 healthy (aside from hypo thyroid).. Also last night I had sharp pain right behind my last rib on the left side.. Almost inside rib cage and tightness behind my left breast in the chest muscle.. No fever or I ll feeling just tenderness. Doctor: possibilities are musculoskeletal pain..intercoastal muscles strain or overuse.cold...other wise distended abdomen and gases can cause pain in this area."
},
{
"id": 154471,
"tgt": "Are the scabs on the cancer patient contagious?",
"src": "Patient: I just found out I am expecting & my oldest has begun to hang out with a friend whom her mother has cancer of the bladder he is not sure what treatment she is undergoing however when I met her I was overwhelmed of how she is covered in what looks like scabs all over her body. Is this a reaction to her treatment should I be concerned him going over to there house & what germs he can bring home? Doctor: Hi,I understand your concern. No cancer can spread by direct person to person contact. So he is not going to bring any germs to your home which can cause cancer. I cant comment about her scabs because we have to look at the lesion before we comment upon.Dont worry.. Cancer is not a transmissible disease."
},
{
"id": 19812,
"tgt": "Suggest medication for elevated BP level and hypertension",
"src": "Patient: My Blood pressure is often 125/89 today its jumped to 138/100 should I tell my doctor she can finally write the rx for hypertension BTW I'm 43 and female and take no medications but have just gotten over a cold but am still coughing could that raise my pressure? Doctor: HIWell come to HCMI really appreciate your concern, I always advise my all the patient to go to Doctor for reading blood pressure, if you are going to read it on your own then it may show error and most of the time it comes elevated. If you do not have any clinical symptoms then it is nothing to worry, looking to blood pressure reading is given here it may not fall in hypertensive category, still you can see the physician hope this information helps."
},
{
"id": 125822,
"tgt": "What causes numbness in the thumb and blurred vision?",
"src": "Patient: My son, who is 26 yrs. old has developed numbness in his thumb and forefinger as well as pain in the upper part of the same arm. MRI shows no cervical issues or signs of MS and he is now being referred to another Neurologist. He also complains of blurred vision at times. What else can cause these issues? Doctor: Hi, The numbness in the thumb can be related to mild nerve root compression or irritation. the blurred vision is not likely to be related to this. Hope I have answered your query. Let me know if I can assist you further. Regards, Dr. Praveen Tayal, Orthopedic Surgeon"
},
{
"id": 182199,
"tgt": "Is Droxyl-250 Dt effective for teeth cavity with swollen cheeks?",
"src": "Patient: Hello Doctor ! My son is 7 years old now. And he is suffering from teeth cavity pain from last few days. But when he wakeup today we found his cheek has been swelled. Then we consultant to the Doctor and he prescribed Droxyl-250 DT thrice a day. Is it perfect or not ? Doctor: Hello,Medication should reduce swelling and should be taken as prescribed to be effective. What was the diagnosis? Additional dental treatment should be required and must be cometed to stop this condition from reoccurring. Decay should be treated in children's teeth and regular exams help diagnosis small areas of decay before a more serious condition occurs. Retaining your child's teeth helps maintain room for the second set of teeth to erupt. Infected children's teeth require extractions or root canal treatment.Thank you for your inquiry."
},
{
"id": 202187,
"tgt": "What causes tingling feeling under the head of the penis?",
"src": "Patient: For the past week I've had a constant tingling feeling under the head of my penis. The area has been very sensitive lately. The feeling is most noticeable when the area is touched, or when I urinate or ejaculate.It's not painful but I'm concerned it could be something much worse. Doctor: Hello. Above symptoms could be seen in case of excessive sexually active in the last period of time may cause tenderness and discomfort, if it is so give a break and see if the symptoms subside, pain in urination you have said in your question can cause due to less intake of fluids, please keep your hydration properly.this should help to resolve your problems. If symptoms persists you should see a doctor for local examination. Hope it helps you. Thank you."
},
{
"id": 76937,
"tgt": "What causes chronic cough and back ache with haziness in chest?",
"src": "Patient: HI, I'm 23 yrs. old, female. last August 7, I experienced throat itchiness, productive cough and shortness of breath and when I sought consult to my doctor, she prescribed me to take azithromycin x 3 days, salbutamol+guaifenesin x 5 days and cetirizine x 7 days. everything were resolved except for the cough so I was advised to get a chest x-ray. The report said that there is minimal haziness in the paracardiac area which could be pneumonitis or overlapping of vascular markings. the doctor did not giveme anymore antibiotics and said that it would resolve eventually but until now there is occassional cough and midback pain. What does that mean? Doctor: Hi thanks for contacting HCM...Noted in x ray you are having mild opacity now.So yes it could be consolidation or opacity by exudate filled alveoli....So it could be pneumonia like infection or bronchiolitis that resolved by antibiotic.This opacity will take time to clear completely....No need for antibiotic use.If dry cough troublesome then codein or dextromethorphan syrup can be taken...Take care.Dr.parth"
},
{
"id": 150663,
"tgt": "Lightheadedness when drinking water. Reason?",
"src": "Patient: i get lightheaded when i drink water and it doesn t happen with any other drink. It is only momentary while I m drinking/ swallowing and not breathing . As soon as I take a breath it goes away. I m just curious because I can drink any other liquid the same way and it doesn t have this effect. It s always happened to me. Doctor: Thanks for your query , Lightheadedness usually is not a cause for concern unless it is severe, does not go away, or occurs with other symptoms such as an irregular heartbeat or fainting. Lightheadedness can lead to falls and other injuries. Protect yourself from injury if you feel lightheaded: Lie down for a minute or two. This will allow more blood to flow to your brain. After lying down, sit up slowly and remain sitting for 1 to 2 minutes before slowly standing up. Rest. It is not unusual to develop lightheadedness during some viral illnesses, such as a cold or the flu. Resting will help prevent attacks of lightheadedness. Do not drive a motor vehicle, operate equipment, or climb on a ladder while you are dizzy. Do not use substances that can affect your circulation, including caffeine, tobacco, alcohol, and illegal drugs. Do not get dehydrated, which can cause or increase lightheadedness, when you have an illness that causes diarrhea, vomiting, or a fever. Drink more fluids, especially water. Other fluids are also helpful, such as fruit juice mixed to half-strength with water, rehydration drinks, weak tea with sugar, clear broth, and gelatin dessert. If you have another medical condition, such as kidney disease or heart disease, that limits the amount of fluids you are allowed to have, do not drink more than this amount without first talking to your doctor. If you have vertigo: Do not lie flat on your back. Prop yourself up slightly to relieve the spinning sensation. Move slowly to avoid the risk of falling. Regards , Dr.Sharmila"
},
{
"id": 156811,
"tgt": "Myelodysplastic syndrome, taking cyclosporine. Ultrasound showed perpancreatic lymph node. What can this be?",
"src": "Patient: I am 66 years old I have MDS and have been on cyclosporine for four years my blood levels are normal right now but an ultrasound I had the other day picked up a1 centimeter perpancreatic lymph node what can this mean Doctor: Hello.I hope you are in good health. The possibilities for this node could be either1 Normal finding ....nodes are present in abdomen2infection in abdomen......check if you are having any fever episode 3 Disease of nodes also called lymphomaYou need to check.Usually size > 1.5 _2 can be considered for this.Biopsythanks and take care"
},
{
"id": 105508,
"tgt": "Runny nose, itchy eyes, sore throat, breathlessness. Is it due to allergy?",
"src": "Patient: Hi, I moved to the city for the first time in Sept. 11 (have lived in a very rural area before), and since spring began this year, I ve started to have symptoms that may be from allergies - runny nose , itchy eyes, occasionally a sore throat , even though I never showed signs of having allergies in the countryside. These symptoms have been fairly mild, so I haven t taken any medication. But today I ve been having shortness of breath for no apparent reason, and the allergy level in my city is quite high today. So I m wondering if shortness of breath can be linked to allergies. If so, what s the best course of action from here? Doctor: you symptoms sound like SAR or seasonal allergic rhinitis and allergic bronchitis. in thiscase, please get tested for allergies esp if any pollens. use of nasal sprays and inhalers during the allergic season will keep your problem under control"
},
{
"id": 40936,
"tgt": "How does artificial insemination work?",
"src": "Patient: I m 34 yrs, male married fo 6 yrs. I was administered chemotheraphy at the age of 13- 15 (Leukemia- Non Hodgkins). Now my seimens tests show zero sperm count. I would like to know about the process and cost of artificial insemination and local reliable sperm banks Doctor: Hi thanks for asking question.You have zero sperm count.So your wife has to be conceived through donated sperms.In this sperm artificially introduced into cervix or uterus of female that is called intracervical or intrauterine insemination.In vitro fertilisation is also one of the method.These procedure is done at various andrology laboratory or specialised gynec clinic or test tube baby centre.This procedure is costly.The charges applied for investigation, for procedures and for drugs.You have not mentioned your city.In metro city you can easily get such clinics and you can ask for total cost of procedure.According to institute cost is different.I hope my suggestion will help you."
},
{
"id": 176522,
"tgt": "Suggest treatment for stomach ache and vomiting when eating fish",
"src": "Patient: My daughter is four years old. She vomits every time I give her fish. I have kept her off fish for a year or so then try to reintroduce it in her diet. I tried again this evening and she had rather intense stomach pain & then vomiting. She has however eaten fish fingers at school just once without feeling ill. Should I give up or can she outgrow this?. Is this an allergy? Doctor: Hello. I just read through your question.What you are describing seems like an allergy. I don't recommend giving up. Have her checked first by a pediatric allergist who may decide to have her tested."
},
{
"id": 55707,
"tgt": "Suggest treatment for fatty liver",
"src": "Patient: Recently we went for full body check up. My husband diagnosis by grade 2 (10.2) likely to be fatty liver. His sgpt level is 105. Pls suggest wether there is any major lever problem? What we will do now? He never take alcohol or any major medicine but he eat junk or outside food regularly. Pls help us by giving u r advice. Regards Doctor: HI thank you for posting your query in HCM. My opinion is that you should not much worried about this as the fatty liver is a reversible condition. This can be achieved by simple lifestyle modifications for weight loss.I would advise the patient to follow these instructions given below. Try losing 1 or 2 pounds per week not more than that, as it would lead to health problems. Go for cardiovascular exercises like brisk walking, running, swimming for 45min a day, 6 times a week. Strength training 4times a week.Avoid alcohol , quit from smoking.Avoid fatty foods, junk foods, deep fried foods, simple sugars, pizzas, chocolates, ice creams, soft drinks. Have a balanced and healthy diet that should include complex carbohydrates like brown rice, oats, brown bread. High protein foods like egg white, pulses, beans, sprouts, soya beans, milk, cottage cheese. Low fat foods. Use olive oil. Have 4 to 5 servings of green leafy vegetables along with all mixed vegetables. 5 to 6 servings of all fruits except banana, mango, chickoo.Drinky 3 litres of water per day. keep your blood sugars under control,if the patient has diabetes.Get regular check ups for cholesterol and triglycerides from your doctor. Hope i have answered your question, if you have further questions i will be happy to help."
},
{
"id": 205264,
"tgt": "What causes a sudden disoriented feeling?",
"src": "Patient: Felt very odd today. A bit like an out of body experience, time was moving and I didn t really remember it the time passing. People said I sounded foggy. Fell asleep and feel much better now. This happened years ago and after being checked out thouroughly, they found nothing, Doctor: Hello,The experience you have mentioned is known as derealization where one feels something changes in the environment, everything around appears strange and changed, time feels like moving slowly and we feel that we are outside of our body and observe ourselves. Sometimes, this can be due to excess of stress, or it can be due to extreme fatigue or when we are about to fall asleep. If you don\u2019t get these episodes frequently, then nothing to worry. If you feel a lot of stress or feel unusual things happening around you most of the time then you need to consult a psychiatrist.Hope I have answered your query. Let me know if I can assist you further.Regards,Dr. Prashant Gohil"
},
{
"id": 23302,
"tgt": "What causes PVC s condition?",
"src": "Patient: I have been diagnosed with pvc s. Normal labs, normal ekg, normal echo, even had stress test, all normal. Lately they have gotten worse, especially at night. my cardiologist doesnt think i need anything but he said if it bothers me, he prescribed bisoprolol 2.5 mg. I m only 33 so i really dont want to start heart meds but the pvcs really scare me which only makes them worse. My concern is this is a beta blocker which lowers blood pressure, and my blood pressure stays around 90/50 so i dont want my blood pressure to drop to low, Doctor: Hello,PVCs may be observed in many people without any abnormality. If they are less than 1000 a day, they are not dangerous and there is no need for any treatment, unless the patient is very symptomatic. For such cases bisoprolol is quite helpful. If start with low dose it usually doesn't cause low blood pressure. If your PVCs are not much, my advice will be not to take anything. Just remember that they are not dangerous at all, if you stop worrying, they may become even less. In such cases Mg supplements may be helpful as well, they usually make our nervous system more stable and reduce the amount of PVCs.Take careCome back if you have any further questions"
},
{
"id": 52943,
"tgt": "What causes hepatomegaly with fatty and renal parenchymal changes?",
"src": "Patient: My mother is 60 years old. Recently she went for ultrasound as she was feeling symptoms of prostrate enlargement. The ultrasound revealed that she has Hepatomegaly++ with mild fatty and Renal parenchymal changes and also slightly echogenic mass lesion measuring 8.3*7.1 cms in the urinary bladder The doc advised for cystoscopy/CT scan for further evaluation. she never had blood in her urine etc. Her Uterus is absent. Doctor: Hi and welcome to Healthcaremagic. I understand your concerns and I will try to help you as much as I can.There are many causes of hepatomegaly or so called liver enlargement and treatment depends on exact cause. if this is caused by diet, obesity and diabetes then dietary and lifestyle changes will be beneficial in every case. Alcohol and viral hepatitis are most common causes and in this case medical evaluation is required.You can also get alcoholic liver disease from drinking lots of alcohol. It can even show up after a short period of heavy drinking, other causes are obesity and hepatitis B and C.If you have alcoholic liver disease and you are a heavy drinker, quitting is the most important thing you can do. Eat a balanced and healthy diet and get regular exercise. Limit high-carb foods such as bread, grits, rice, potatoes, and corn. And cut down on drinks with lots of sugar like sports drinks and juice.I hope I have answered you query. If you have any further questions you can contact us in every time.Kindly regards. Wish you a good health."
},
{
"id": 185523,
"tgt": "Suggest remedy for pain near jaw tooth",
"src": "Patient: Hi. for two days a jaw tooth was cracked, it was not painful as I can eat and drink. yesterday it was warm slight pain, now the tooth next to this cracked tooth is slightly red. I think it has an abscess. wanted to extract it but.... I have a tumor on my left temporal lobe and may have surgery within a week or 2. Doctor: Hello, Thank you for consulting with HCM.Yes it looks like your tooth was infected a few days back and now there is abscess in the apex of the tooth, it is not necessary that you have to go for removal of the tooth, you can go for root canal treatment of the tooth as well, as it can also reduce the infection and you can start with the treatment before the surgery for tumor. Hope it will help you."
},
{
"id": 166623,
"tgt": "Suggest treatment for constipation in kid",
"src": "Patient: Hello my son is 3 years of age and was always constipated as a baby Doctors placed him on Miralax treatment in his bottle as a baby. And on Pedi-Lax fiber gummies after he has not taken treatment for 2 months now but I have noticed that he has a fart disorder he frequently farts and poop comes along with the fart... it is on the soft side and his stools are formed. Doctor: Hello I have gone through your question and understand your concern.Constipation can cause stomach cramps, reduced appetite and irritability.Constipation can usually be controlled with good bowel habits and medicines.Diet is less important in treatment of constipation in children than it is in adults.Giving your child adequate fibre in their diet might help prevent constipation in some children who have a natural tendency. To add more fibre to your child's diet, you can give your child: At least two servings of fruit each day. Fruits with the peel left on, such as plums, prunes, raisins, apricots and peaches, have a lot of fibre.At least three servings of vegetables each day.Cereals that are less processed, such as bran cereals, shredded wheat, whole grain cereals or oatmeal. Avoid refined cereals, such as corn flakes and rice bubbles. Potty train your child, Children should learn to respond to their body's urge to poo. It is important for constipated children to develop the habit of sitting on the toilet regularly."
},
{
"id": 99827,
"tgt": "Suggest treatment for fever, coughing and bleeding in eye",
"src": "Patient: I had high fever,eye bleeding at night,slimy cought,one stage I couldn't even talk,and the sniffles,and my fever won't disappear even I took two weeks Burana 400mg. But the temperature started go down,the point is,I still have fever,and it wont go away. Doctor: Hi, as per your symptoms, tab.ibuprofen won't be able to help alone, bleeding from eye may be due to various reasons like severe cough,hypertension, roughly rubbing of eyes,blood thinners drugs.As u are having high grade of fever u should first get investigated simply CBC mainly for total count if there any bacterial infection and chest x- ray.the most probable cause can be upper respiratory tract infection clinically but u can start with Tab.levofloxacin 500 mg once a day,Tab.paracetamol 650 twice a day,Tab.levocetrizine 5 mg twice a day after your total leukocyte count.nothing to do with bleeding from eye ,as fever will reduce it will subside automatically."
},
{
"id": 158040,
"tgt": "Diagnosed endometrial cancer, total hysterectomy scheduled. Lost both parents to cancer. Expectation after surgical removal?",
"src": "Patient: Hi...First, thank you for being here. I just learned 4 days ago that I have endometrial cancer, and I will be scheduled for a total hysterectomy. Having lost both parents to cancer, needless to say, I'm quite frightened. I've had a C-section (20 years ago). What can I expect for surgical removal and what may follow? Doctor: welcome to HCM!prognosis in endometrial cancer depends upon the type of cancer.second,how deep it is extended in the myometrium.whether peritoneum and lymph nodes invoved.consult your gynaec.wish u all the best!"
},
{
"id": 163865,
"tgt": "Suggest laxatives for a toddler apart from laxoped kid powder",
"src": "Patient: hello dr, my son is 2.4 years he his not passing motion properly, he his facing very difficulty while passing 4 to 5 days once..it is very hard.our doctor suggested laxoped kid powder for 10 days but he dosn t drink. for this situation any other surup can i give him..please suggest me with regrads, sowmya Doctor: In my opinion you can also give EVA Q syrup child pack. 2teaspoon bed time you can give . Along with this add an probiotic like ENTEROGERMINA SUSPENSION 5ML ONCE A DAY."
},
{
"id": 15725,
"tgt": "Rashes on the skin after using a hair dye. Bumps on the hair line. What to do?",
"src": "Patient: I recently broke out in a rash. I had my hair colored and also used color oops to correct it after. I have never broken out in a rash before from coloring my hair, not to my noticing. I now have a rash of tiny skin-toned bumps on my neck, red tiny bumps along my hairline, it has creeped to my shin and on my face. what do I do? Doctor: Hello,Thanks for the query.you have developed an irritant contact dermatitis to dye.Apply a mild steroid lotion, this will help reduce the irritation.you can also use a mild steroid shampoo like sebowash.Let me know if you have any other doubt.you can ask a direct question to me on this forum, following the below link.https://urldefense.com/v3/__http://www.healthcaremagic.com/doctors/dr-rahul-kumar/64818please__;!!Mih3wA!SBzm6_kI6hCZ58EPH6N_05MFfiPbxWXT0a2TJCdFQObRWm5mV5ur7hWGlnVDJA$ meet a dermatologist for final diagnosisThank you"
},
{
"id": 9106,
"tgt": "Why is there change in the skin color of my body , face and hands ?",
"src": "Patient: Why do I have a very darker shade colour on my face and hands than any other part of my body Doctor: hi face,neck and hands are expose part. they expose more to sun light,dust,wind,dirt,pollution as compare to other part of body. so they r more darker then other part most common cause is ultra violet radiation of sun. use sunscreen with spf more then 25 twice a day. wear gloves or full sleeve cloths"
},
{
"id": 155842,
"tgt": "Suggest treatment for cervical cancer",
"src": "Patient: my name is smriti rani roy. i m 48 years. the followings are done with me...Date:27/07/2010Cytology report:type of cytology-cervical scrap(pap).Specimen type:conventional Pap smear.Interpretation:High Grade squamous lesion,in favour squamous cell carcinomia.\u00a0\u00a0\u00a0\u00a0\u00a0Date:7/08/2010Biopsy report: \u00a0\u00a0\u00a0\u00a0\u00a0Specimen:tissue from cervixDiagnosis: Invasive squamous cell carcinomia well differentiated grade \u2013I.Date:12/08/2010Radical Hysterectomy with bilateral pelvic lymph adenectomy done.Hystopathology report:Nature of specimen:1.\u00a0\u00a0\u00a0\u00a0\u00a0uterus with both adnexa2.\u00a0\u00a0\u00a0\u00a0\u00a0right side lymph node3.\u00a0\u00a0\u00a0\u00a0\u00a0left side lymph nodeinterpretation: sections show uterine cervix.it present a squamous cell carcinomia.the carcinomia is moderately differentiated. No vascular or lymphatic invasion is noted.The vaginal cuff ,internal OS ,uterine wall,one sided sdnexa,other sided adnexa,both sided parametrium and both pelvic lymph nodes are free of tumour.DX: uterus(resected): Squamous cell carcinomia , grade \u2013II.what to do now?? plz help me..... Doctor: Hi Welcome to HCMI have gone thru your qurery regarding treatment for cancer with heavy heart . It is a matter of great concern . After so many years of treatment ,you are still on the coross road , at a loss to know which way to go . I, as a Naturopath Homeopath and Magnato therapist ,want to help you to remove your agony a little at least .Magnitude of the survival advantage from improved nutrition, can even be greater than the magnitude of the treatment effects being targeted in current clinical drug trials .As you are in doldrums regarding your treatment ,You can't sit back , see helplessly . I wiould like to suggest you to modify lifestyle to fight cancer is one of the most effective treatment options in my view . Many have been benefitted by These therapies which slow the growth of bacterias and kill on mass bases .Pls follow the below regimen full of antioxidant and has no side effects and goes with any therapy Our health depends mostly upon 'what & how & when we eat, and our life style .Avoid processed foods, sugar , fried ,fast foods Tea , coffee ,alcohol smoking all will produce biochemical and metabolic conditions in your body that will decrease your immunity, so avoiding is definitely the first step in the right direction.Detoxing the body, boosting the immune system and changing the diet are important elements of treatment for cancer . vitamin C and goat\u2019s milk can help treat the cancer.One of the important strategies to achieve healthy body is a diet rich in fresh, raw whole foods .More of Fiber, protein vitamin C and goat\u2019s milk , fruit, egg fish ,fish oil for Omega 3, green leafy veges . quality, organic, biodynamic, locally-grown food will naturally increase immunity .To keep metabolism on right path increase strength of immune system , take all supplements full of antioxidant serve as antibiotics .Raw garlik gingercoconut water ,Lemon juice with water twice before meals , Turmeric powder , level spoon with with a cup of hot milk with B/F & at bed time , Buttermilk with lunchAloe vera juice 25 + amla juice 2 spoons ml twice a dayPhysical activity in order to renew healthy cell and rebuild itself, so you should make exercise a lifelong commitment, but never overdo.Do 30 mins walk/ exercise - from head to toe ,yoga ,pranayam - deep breatihing , Kapalbhatti / Laughing aloud . proper rest , meditation & positive thinking , to detoxify mind & body & your system , to accelerate the process of recovery . Contipation is the mother of all illnesses so is mental stress /worry /anger The condition is REGULARITYI further suggest you not to worry and take action fast ,Above regimen will surely gradualy , help you to lead happy worry free healthy life ahead Magnets & drinking Water, prepared on North pole magnets also plays as a potent tool in treating cancer and killing bacteria on mass basis.It\u2019s important to check with your physician before adding herbs to your existing medications,regarding the herbs that are antioxidant ,have no side effects and go with any therapy of treatment .HOPE THIS HELPS SOLVE YOUR QUERY Take care All the best . If any doubt mail at drsuchda@gmail.comDont hesitate for futher query if anyWish you early recovery"
},
{
"id": 109364,
"tgt": "How can back pain be treated?",
"src": "Patient: I started losartin 50mg one table per day i am on holiday in malta my gp is in england i have the most awful painful back pain i have never ever had back pain before and have not injured my back what should i do it is ruining my holiday i cannot sllep or rest can i take anything like diclofenac/voltarol Doctor: Hellowelcome to hcmtake this treatment\u25cf INJECTION THIOCHOLCHICOSIDE intramuscularly twice daily for 3 days\u25cf TABLET IBUPROFEN three times a day\u25cf TABLET CHLORZOXAZONE three times a dayafter u return back home get a MRI OF LUMBO-SACRAL SPINE n send me scanned copy of its reportregardsdr rahul"
},
{
"id": 80786,
"tgt": "Suggest treatment for wheezing",
"src": "Patient: sirmy son is suffering from wheezing whenever we are in Chennai. we reside in delhi. he does not getany problem when he is in delhi. but morning sneezing is always there. now he is suffering from wheezing. can u pls suggest which specialist has to beconsulted for this Doctor: Thanks for your question on HCM. I can understand your situation and problem. In my opinion you should consult pulmonologist and get done PFT ( pulmonary function test ) to rule out allergic bronchitis. Your son is wheezing only in chennai, this means he is having allergic phenomenon due to atmosphere or allergen in chennai. So we need to rule out allergic bronchitis in his case.He may need antihistamine, anti allergic and inhaled bronchodilators , whenever he is in chennai. All these drugs will help him to avoid allergy and thus wheezing. So better to consult pulmonologist and discuss all these."
},
{
"id": 99896,
"tgt": "What are the side-effects of Defcort given for skin allergy?",
"src": "Patient: Hello Doctor.. i had an hair line crack in forefoot and a Doctor prescribed me an ointment Relaxyl DA Thermo gel after 1 and half month... but now i am having an skin allergy because of use of it.Now Doctor prescribed me defcort 6 for 1 week( three times per day). Might be skin allergic will be gone.. is there any side effect of it as it is steroid.. if yes, then what kind of precaution should i need to take. Doctor: Hello,Thank you for asking at HCM.I went through your history and would like to make suggestions for you as follows:1. Defcort (Deflazacort) is a corticosteroid. For 1 week use, as mentioned my you, common side effect would be gastritis, stomach upset, vomiting, nausea, etc. You can avoid them by taking it after food, if necessary, you can also take antacids simultaneously.2. If you use corticosteroid for prolonged period, possible side effects are numerous, such as hypertension, weight gain, diabetes, cataract, glaucoma, osteoporosis, mental disturbances, skin thinning, etc etc. Also, if you take them for prolonged periods, you should not discontinue immediately; they need to be gradually decreased under guidance of physician.3. However, for 1 week course, you can stop as suggested by your physician. Hope above information will be helpful to you.Should you have any further query, please feel free to ask at HCM.Wish you the best of the health ahead.Thank you & Regards."
},
{
"id": 65997,
"tgt": "How to treat lump on palate?",
"src": "Patient: Five days I started feeling pain (not so intense) in the area of my hard palate when eating. Tomorrow I noticed a small red lump painful to tongue pressure. The lump is positoned on the posterior left area of hard palate, near the central line of the palate. It is as big as rice. The first two days it was red and now it is starting to have normal colour as the surface arround. But it is still there and I am worried. Since my present anxiety I cannot estaimate if it has changed its size. I only have an impression that it does not grow. Doctor: Hi, dearI have gone through your question. I can understand your concern. You may have some benign lesion or malignant lump. You should go for biopsy of that lump. It will give you exact diagnosis. Then you should take treatment accordingly. Hope I have answered your question, if you have doubt then I will be happy to answer. Thanks for using health care magic. Wish you a very good health."
},
{
"id": 3867,
"tgt": "Suggest treatment to get pregnant",
"src": "Patient: Pregnancy I had a miscarrage in july 2011 & ive been trying to get pregnant ever since and it seems its not working my periods use to be regular and now there not ive had a miscarrage once before and had 3 boys after my first mc what can i do to get my period regular and what could happen if i start taking clomid without being supervised by an ob i really really want to have another baby.... Doctor: HIWell come to HCMFirst of all underlying cause of miscarriage need to be ruled out and treated accordingly, pregnancy is more natural phenomenon than the physiological one apart from the treatment you should keep trying, do not give the hope, compatibility is matter, hope this information helps, have a good luck."
},
{
"id": 163330,
"tgt": "What causes body rashes, loose motions after measles vaccination?",
"src": "Patient: Hello Doctor, My Son Completed 9 Months we had taken mesels Vaccine after a weekdays we found some rash on his body we consult a doctor she said those were mosquito bites but there is no change after 3 days also. 2. He having motions frequently we are not able find any growth. pls advise us Doctor: Hello,This is not because of measles vaccine, rash will subside only give Jardin D or Rigix syrup half teaspoon (tsf) twice a day.You can give a shot of Avil and solu cortef from some hospital in paeds dose if reaction is severe.Check out any unhygienic conditions for diarrhea give ORS (Oral rehydration solution) in sips, syrup Zinc 1 tsf daily for 10 days. Once the condition settled, then no need of antibiotic.Hope I have answered your query. Let me know if I can assist you further.Regards,Dr. Hina Javed"
},
{
"id": 138673,
"tgt": "What causes swollen cheeks?",
"src": "Patient: I woke up this morning and my right cheek is swollen, i have a red mark and it hurts to touch like a big pimple. But when i bend my head down the swollen cheek feels heavy. Was wondering if it is an ingrown hair or a food allergy to something i ate last night and a pimple just happened to be there. Doctor: HiThis is infected and you will need antibiotics for that. Wash it clean with dettol solution and take a short course of antibiotics and analgesic. It will correct."
},
{
"id": 166621,
"tgt": "Suggest treatment for ALD",
"src": "Patient: Dear doctors ... what would be your advice for a 6 years old boy with ALD? Would you recommend a bone marrow transplantation (BMT) ? What kind of diet should be given to that boy? Considering the weak condition of the boy, would you let him to have a blood test monthly (6 bottles of 15cc blood) ? Thanks in advance. Doctor: Dear parent, I understand your concerns but currently there are 2lines of treatment 1) Lorenzo oil this is not a defentive treatment it only tend to improve the symptoms in mildly affected children2) stem cell transplantation , this is a defenetive treatment that is recommended for young children who has the disease , it can be done using umbilical cord blood or bone marrow. the new bone marrow will have a functioning ALD protein. I recommend the transplant. also no problem in doing the tests if necessary . he should avoid fatty foods"
},
{
"id": 22317,
"tgt": "What causes frequent heart flutters, fatigue and dizziness?",
"src": "Patient: I am a 40 year old female, 5ft 2\" and was about 150 lbs but i lost weight and haven't weighted myself after the weight loss. I have had throughout my life heart flutters or palpatations. Somestimes i cough when when it happens. For the past three weeks or so i have been having them regularly and almost all the time and i cough everytime it happens. I believe i got fatigued twice this week. I had to rest in order to continue what i was doing. Also I have done about three ecg's and the result was always nothing is wrong. I also eperienced a sort of dizzyness or unbalance like my brain or head just shifted from one side to the next. This is different to the dizzyness which I suffer and doctors medicate me with a b- complex with iron tablet and stemicil or stugeron forte for the dizzyness. Doctor: Hello, This can be arrhythmia like psvt or atrial fibrillation, where your heart rate suddenly goes up and bp falls. These are episodic so ecg if done during the episode will be abnormal otherwise normal. You will need evaluation like thyroid test and holter monitoring. You should also get one hemoglobin levels done. So visit nearby cardiologist for these investigation. Also, to abort such episodes you should try to cough hard. Till the time your evaluation is complete, you can ask your local doctor for tab ciplar LA 20 mg once a day. If none of the above is successful then last option is of electrophysiological studies after confirming the diagnosis on holter or ecg which is invasive test which can diagnose and treat the arrhythmia permanently in most of the cases.Hope I have answered your query. Let me know if I can assist you further. Regards,Dr. Sagar Makode"
},
{
"id": 200623,
"tgt": "Does meditation cure acute Depreciation and anxiety?",
"src": "Patient: Hello! Doctor, I ve been undergoing Psychiatric treatment since 2007 for the diagnosed Acute Depreciation & Anxiety. I m being treated with Ventab DXT 25, Macorate CR200, Wacalert 50 & Zapiz 1.00 whenever I feel more anxiety or seem not able to control my thoughts. Actually, this started in 1998 from Exam Phobia & was treated for same. Since then I ve been in & out on medication from time to time till 2007 from where there s consistency in medication & follow ups. Sutre, the treatment has helped me to stabilize over the period. I ve got married, have a small daughter, completed post graduation, MBA & also did certification in SAP FI. The doctor has been of great help to me. He was of opinion that I ve lesser coping capacity. And, that s not entirely wrong. His major concern has been my changing the job & settling in it. He s slowly started reducing power of drugs with powers mentioned above. But, it seems I ve developed dependency on these medicines. And, I want to stand on my own with my thought controlling me rather than medicines. In this blog, I read in answer to one patient, reference to Meditation, Self Motivation. Meditation or Yoga is okay for me since I ve learned it scientifically. I want to know about self motivation. Is there any specific resource available? How can I access it? Will I ever be able to overcome these feelings? or they ve become a part of my personality? I wish to live a life free of medication & complete self reliance. Able to communicate with myself, Be social, & grow in my career as well as financially. Cause, then only I ll not just be able to prove to myself but others as well. Humbly request you to reply as early as possible. Doctor: DearWe understand your concernsI went through your details. Self motivation can be achieved through self hypnosis or Cognitive Behavior Therapy. You are doing well with your progress in life and treatment of your mental disorder. If you are trained in meditation and yoga, you are well off. You can do meditation to have positive metabolism effect and also teach your subconscious mind to control and divert your thoughts in accordance with what you want. Please understand you are doing well and that is the first step of self motivation. I will help you in the requested matters.If you require more of my help in this aspect, please use this URL. http://goo.gl/aYW2pR. Make sure that you include every minute details possible. Hope this answers your query. Available for further clarifications.Good luck."
},
{
"id": 196928,
"tgt": "What are the symptoms of guillain\u2013barr\u00e9 syndrome?",
"src": "Patient: my husband 61 contracred Guillon barre 3months ago, he was given immnglobian and is making good progress but for his bladder he still has the catheter in even though he wants to pee when they take it out he has to go every 30mins or so but a very small amount so catheter has to go back in.Does this show signs it is coming back? We live in Portugal and tey don't know even saying maybe it will never?? He is able to walk unaided and even able to drive now, but still not much stamina is there something to help that return?many thanksSusan Doctor: HelloThanks for query .Your husband has Guillain Barre Syndrome and was catheterized with self retaining indwelling catheter to drain urine from his bladder .However he is unable to pass urine of his own on removing catheter .This could be GB syndrome due to involvement of nerves However patients with GB Syndrome do recover completely within few months .Looking at his age to be 61 years the other common cause of difficulty in passing urine is Enlarged Prostate and needs to be ruled out.I would suggest you to qualified Urologist for clinical and digital rectal examination and get following basic tests done to confirm the diagnosis.1) Urine routine and culture. 2) Ultrasound scanning of abdomen and pelvis3) Serum PSA.Further treatment will depend upon result of these tests and final diagnosis.Dr.Patil."
},
{
"id": 115666,
"tgt": "What causes random dizzy spells?",
"src": "Patient: I've went to the hospital to get blood work done because of dizzy spells and everything came back normal. They don't really have any time where it's mild or close to a complete back out, it just randomly comes and goes. I eat normally too, and changing my eating habits have not changed the dizzy spells. What could it be? Doctor: Hi, dearI have gone through your question. I can understand your concern. You may have anemia or hypoglycaemia. These are the most common cause of dizzy spell. You should investigate for blood sugar and hemoglobin. Consult your doctor and take treatment accordingly. Hope I have answered your question, if you have doubt then I will be happy to answer. Thanks for using health care magic. Wish you a very good health. Don't forget to click thank you."
},
{
"id": 180414,
"tgt": "What does a purple rash around the uvula indicate?",
"src": "Patient: Hello. It recently felt like something was stuck in the back of my mouth. I looked in my mouth with a flashlight and noticed bruising or a purpleish-black mark on each side of my Uvula. My throat feels better and do not have the sensation of something stuck in my mouth but it is still the same color. Doctor: Hi Dear, As per your query you have symptoms of purple rash around the uvula which is because of viral infection which sometimes spread in the tissues around the tonsils and produce this coloration. Need not to worry about it. I would suggest you to do warm saline gargles. Drink hot milk with half spoon of turmeric powder. Apply Lignocaine containing gel if it is painful. Take Advil or Tylenol for pain and swelling. Consult ENT specialist for proper examination. Doctor may order blood test or throat swab test.Doctor may prescribe antibiotics course for at least 10 days along with anti allergic drug depending upon the diagnosis. Take diet rich in vitamin C. Maintain proper oral hygiene. Hope I have answered your query. Let me know if I can assist you further."
},
{
"id": 7051,
"tgt": "I have irregular periods and I am on medication. My husband is on anti depression drugs. Can I get pregnant ?",
"src": "Patient: hello doctor ,am priya 34yrold, & got married in Nov19th 2010, got ma 1st period on 23rd nov 10 2nd period on jan 26th 11 ,3rd 5thmar 11, 4th april 11,& 5th on may 5th 11 and 6th on may 23rd 11 and doctor suggested me to take carnisure500mg for 20dys .... will it harm in getting pregnant doctor had prescribd 1st with fertly super100mg ,clomid progestrone , levtol 2.5mg& now carnisure ....and my husband is taking tablets for depression as follows Parkin ,Azur , Stalopam, Telma-H, Amlip-5, Matilda,Glizard... Doctor: welcome to healthcaremagic yes u can very much concieve..buy a LH kit to know exactly your ovulation period and have intercourse on that day..your medications wont cause u problems..once u concieve the medications can be stopped..try to stop antidepressants gradually"
},
{
"id": 77642,
"tgt": "What causes heart palpitations and shallow breathing",
"src": "Patient: My 19 year old daughter has been struggling with heart palpatations for 2 days, feeling also as if she's not getting enough oxygen to her lungs (shallow breathing) and pins and needles feelings in her hands. She looks drained and without any energy. Doctor looked her over very quickly yesterday and said she's suffering from anxiety and should breath into paper bag. This has not helped. Doctor: Hello and welcome to HCMI have read your queryI hope this will helpFrom the history provided by you YES your daughter is suffering from anxiety.. This is a typical pic of such a patient with PANIC ATTACK or GENERALIZED ANXIETY DISORDER..I will suggest you to not worry much..advise her to breath in paper bag at time of attack. If symptoms persists she should be given ALPRAZOLAM .5MG OR TAB FLUOXITINE 20MG.but after yourGP's advise.. She needs a bit counselling and medications if symptoms are not improving...I hope she will recover soon.RegardsDr faeza"
},
{
"id": 113969,
"tgt": "Is ozone discotemy the best treatment for disc degeneration disease ?",
"src": "Patient: wheather ozone discotemy is best treatment for disc degeneration disease ? i am suffering from severe back and radiating pain which coming from my left buttock to foot . 6 mnths before i take mri scan , results shows L5 S1 disc degenerative disease . with advice of ortho dr i take bed rest , traction ,pyisotheraphy , still nw i am feeling same . dr adviced me for surgey but i am not prefer surgey . recently i heard that ozone discotemy how is it. wheather it is economy . how many % result is there. in india how many places are there . which is the best one sajeesh kerala Doctor: Welcome to HCM. Still you have pain with all routine management than go for ozon therapy it will definately help you.better you consult good orthopaedician"
},
{
"id": 76302,
"tgt": "What causes vibration sensation in left side of the sternum?",
"src": "Patient: No one can diagnose this. When I lean back in a chair or lie on back I get a weird vibration in left side of sternum at the end of a deep breath during inhalation. All CT scans of chest and abdomin are normal.Symptoms worse after drinking alcohol. I also have a dry cough.Any ideas? Symptoms keep me from sleeping at night. Symptoms are worse when I lay on left side. Symptoms begin around a minute after laying down or leaning back. They go away after I take a deep breath and return a minute later. Very nervous about this. Doctor: Thanks for your question on Healthcare Magic. I can understand your concern. By your history and description, possibility of GERD (gastroesophageal reflux disease) more. GERD is due to laxity of gastroesophageal sphincter. Because of this the acid of the stomach tends to come up in the esophagus and cause similar symptoms. Alcohol, stress etc are risk factors for GERD and both are present in your case. Reflux is more in supine position. This is the reason why you got more problems in lying down in night. So follow these steps for better symptomatic relief in GERD. 1. Avoid stress and tension. 2. Avoid hot and spicy food. Avoid junk food. 3. Avoid large meals, instead take frequent small meals. 4. Take pantoprazole tablet on empty stomach. 5. Keep 2-3 pillows under head in the bed to prevent reflux. 6. Quit alcohol and smoking if you are smoker. Don't worry, you will be alright with all these. Hope I have solved your query. I will be happy to help you further. Wish you good health. Thanks."
},
{
"id": 51956,
"tgt": "Can low grade fever be a symptoms of having kidney stone ?",
"src": "Patient: my mom 79 was having low grade fever every 2/3 weeks, she was diagnosed for kidney stones and left kidney was25% of its size and had two stones and one stone in the right kidney ureter . all stones were of the size of 1cm to 1.5 cm. the right kidney stone was taken out and left kidney a stent was used. this was done on 30th April. the stent is still there. still low grade fever is thee and ESR is 120 and creatnine is 1.9. her tests are coming negative. TB test is also negative. she had taken TB medication in 2003. Cant understand where is the infection. do you think removing the stent used for left kidney will remove the low grade fever. Her hbaic is 6.1, she takes med for thyroid, cholestrol, BP Doctor: Hi,thanks for query.After any surgical procedure especially in diabetics this is not uncommon.Presence of foreign body like stent could also be the reason.In case where there is some stagnation of urine because of stone it can lead to fever also.So there is no need to worry,keep regular follow up and she will be fine.bye."
},
{
"id": 218446,
"tgt": "What causes brown vaginal discharge during pregnancy?",
"src": "Patient: hello. I m 25, I am 8 week pregnant with my second child tonight I noticed brown discharge in my underwear, and nothing when I wiped. now 4 hrs later i went pee and when I wiped thre was pinkish brown on the toiletpaper but no blood clots or pain. I will be calling my doctors office in the morning just wondering if this is something normal as my daughter is now 6, my last pregnancy I do not remember this happening! thankyou aleisha brasch Doctor: Hello and Welcome to \u2018Ask A Doctor\u2019 service. I have reviewed your query and here is my advice. First of all do not panic. Your symptoms can be due to some retro placental bleeding or bleeding from cervical erosion. Please get obstetric USG done and gynecology check up done including per speculum and per vaginal examination. This will help to know about the cause of your brown discharge. Take rest. Seek prescription support from your treating doctor for tablet natural micronized progesterone. Hope I have answered your query. Let me know if I can assist you further."
},
{
"id": 14268,
"tgt": "Suggest remedy for scabies",
"src": "Patient: my daughter 7 months old is being treated for scabies .she has itchy red rashes on her face, the doctor prescribed hydrocortisone acetate cream bp 1% w/w for it but its already more than 24 hrs and no signs of the rashes clearing. please tell me what i should do for the rashes. Doctor: Hello,Your daughter may be having allergic dermatitis on face. Kindly consult the dermatologist for the perfect diagnosis and proper treatment. Hydroxyzine syrup may be given in proper dose. Apply Calamine lotion on face. Avoid soap bath.Hope I have answered your query. Let me know if I can assist you further.Regards,Dr. Ilyas Patel"
},
{
"id": 69786,
"tgt": "What could small pea size lump in mouth indicate?",
"src": "Patient: HI I HAVE A SMALL PEA SIZE LUMP IN MY MOUTH BETWEEN BOTTOM GUM AND CHEEK ITS BEEN THERE FOR ABOUT 18 MONTHS AND SEEMS LIKE IT IS STARTED TO GET SLIGHTLY BIGGER HAD A EXRAY IT IS NOT A SCIST OR ABSESS NEED TO SEE A CON SULTANT WAITING FOR A APOINTMENT WHAT DO YOU THINK IT IS? Doctor: Hi.Lump in cheek line can be a cyst only. IF you are young no need to worry. A simple small surgery is sufficient to get OK.If you are elderly a chance of cancer to be borne in mind. Get this diagnosed and excised. I hope the teeth nearby are normal . . ."
},
{
"id": 157575,
"tgt": "Fluid retention in left leg, swelling started after biopsy on lymph node. Had non hodgkins lymphoma, having booster treatment",
"src": "Patient: j have a left leg retaining fluid with a very painful heel..burning and sharp pains. what can be the cause and what can I do? I had non hodgkins lymphoma three years ago and I am still having booster treatments evert three months. the swelling first started after I had a biopsy on on the lympb node in the liower back can dr Mathew Mintz reply Doctor: Dear,There must be the recurrence of the disease or must be residual disease after the initial treatment for non hodgkins lymphoma. The swelling of the leg is because of the pressure on the lymphatic vessels by the tumour.If the chemotherapy has not relieved it, did your doctors advise local radiotherapy in the inguinal area. You can discuss it with your doctor and he will be the right person to tell you whether you could be benefited with it. Residual disease after chemotherapy is most of the times taken care with Radiotherapy.Truly,Dr. J. Ticku"
},
{
"id": 58171,
"tgt": "On jaundice treatment, on complete fat free diet, bilirubin levels reduced. Continue fat free diet? Reason for increase in Indirect bilirubin?",
"src": "Patient: Hi sir I am on Jaundice Treatment from 7 Jul 2012. On 07 jul My bilirubin levels were as follows: Total 7.4 Direct 5.2 Indirect 2.2 I was on complete fat free diet for a month. On 07 Aug,My bilirubin levels were as follows: Total 1.04 Direct 0.62 Indirect 0.42 Again I was on fat free diet for 10 days and today (17 Aug),My bilirubin levels were as follows: Total 0.95 Direct 0.31 Indirect 0.64 Should I continue fat free diet?What may be reasons for Increase in Indirect Bilirubin? Doctor: Hello, Your laboratory reports seem to be consistent with diagnosis of viral hepatitis. In a normal person the fraction of indirect bilirubin is more than that of direct bilirubin. In hepatitis the bilirubin increases, with a greater increase in direct bilirubin. Now as you are recovering the total and direct component of bilirubin will decrease. Your last report is absolutely normal. The indirect bilirubin has increased very marginally as compared to the previous report but is within normal range. You need not worry about it. You can have a normal diet now. Hope that helps.Regards,Dr. Preeti"
},
{
"id": 181775,
"tgt": "Suggest treatment for tooth abscess",
"src": "Patient: I have a tooth abscess and made an appointment tomorrow morning to drain it, but it just burst. i'm still going to see my dentist tomorrow morning, but am a smoker and really would like one. I know this is probably a stupid question, but how long do I have to wait before I can have a cigarette? Doctor: Hello, Thank you for consulting with HCM.Actually smoking can worsen the condition till a proper drainage of abscess is done through proper opening of tooth made by dentist.Better you should discontinue it till you visit your dentist.Hope it will help you."
},
{
"id": 146250,
"tgt": "What does this MRI report of spine indicate?",
"src": "Patient: Hi there, I have had a CT scan & a MRI my conclusion is mild secondary canal stenosis L4/5 with indentation of thecal sac, bilateral traversing and exiting nerve roots by a disc bulge. Indentation of the left exiting nerve root L5/S1. I am on plenty of pain relief at the moment & am wondering do I need to see a specialist about this & what does this mean for me. Doctor: Hello dear,I understand your concerns.Secondary canal stenosis means the spinal canal(the canal through which spinal cord passes in the spine)has been narrowed down secondary to some pathology of spine(most common pathology to cause this is degeneration of spine with age).When there is canal stenosis,it may compress some important nerve fibre tracts passing in spinal cord resulting in various symptoms like weakness in limbs,loss of sensations in limbs,urinary retention or urge incontinence.Similar degenerative process may compress upon the nerves which exits from the each vertebrae at different levels to pass into limbs and trunk again resulting symptoms of pain,numbness and tingling in limbs along with weakness,sensory loss,and bladder symptoms like loss of bladder sensations.Yes,you need to visit a neurologist to have an expert advise regarding this.I hope it was of help to you."
},
{
"id": 40203,
"tgt": "What could be the cause for a sinus infection?",
"src": "Patient: Monday I received a shot for allergies, Thursday my nose, and cheeks stole up and turned bright red. Friday is was givin a shot of ammoxacillion. Saturday My eyes have almost swollen shut and the glands in my throat are also swollen, went back to the doctor and received a shot of Benadryl and pain medicine for my head which is now non stop pounding.is this an allergic reaction or a sinus infection? Doctor: Hello,Welcome to HCM,The symptoms what you are having looks like a severe bacterial infection causing upper respiratory tract infection. The symptoms are severe and as it is not coming down by the treatment, he mau require parental antibiotics for which admission to the hospital is required.The antibiotics like I V Bactoclav along with other medicines like antipyretic and antihistamines helps to control the symptoms. Thank you."
},
{
"id": 28860,
"tgt": "How serious is an E.coli infection?",
"src": "Patient: My grandson has been diagnosed with Ecoli, how serious is this? He has been having severe cramps after eating for about 5 weeks and has been in and out of Childrens' Hospital during that time, they just kept sending him home saying he was impacted. Doctor: Hello,My answer is as follows:Escherichia coli (E. coli) is a bacteria that colonizes human intestine. They can cause diarrhea or intestine related diseases but also urinary, pulmonary and other infections (more rarely).Among the 6 types of E.coli, one is of particular importance: Enterohemorrhagic E. coli. In children, this E. coli sub-type might produce hemorrhagic colitis, hemolytic-uremic syndrome (anemia, acute kidney failure, dangerous lowering of blood platelets).Hemorrhagic colitis manifests with bloody diarrhea following non-bloody diarrhea and abdominal pain. Signs and symptoms are dedicated to potent toxins released by this type of E.coli.In this particular disease, the use of antibiotics is contraindicated because this increases the risk of hemolytic-uremic syndrome, which is a medical emergency. The use of antibiotics has been shown to not impact the course of illness or prevent future sequelae (permanent effects of infection).Therefore, the treatment of the disease is done by offering excellent supportive care, according to signs and symptoms.This is why your doctors ask you to visit often the hospital: due to the need for careful assessment and treatment according to the state of the child.Fortunately, in most cases the infection is self-limited and will be successfully managed by the medical team.Hope I have answered your query. Let me know if I can assist you further.Regards,Dr. Ervin To\u00e7i"
},
{
"id": 150456,
"tgt": "I have gotten dizzy for a few seconds. Take premarian and Valsartan, HCTZ. What does it indicate?",
"src": "Patient: A few minutes ago I stood up to answer the phone and became very dizzy and then was not able to make sense to my son over the phone for a few seconds (45) and had to tell him I would call back. I am in relatively good health and 67 years old. I am 15 pounds over weight. I take premarian (20 yrs) and Valsartan/HCTZ 160/12.5mg for only 6 months. I took a mild fluid pill ( trimmean sp) before for 25 years. Iam fine now. I have gotten dizzy for a few seconds several times in the last six months. What is wrong??? Doctor: Hi friend, Welcome to Health Care Magic The first possibility is postural may be postural hypo-tension.. Check your BP - lying AND standing... It can be from anaemia \u2013 what is your haemoglobin? Check your electrolytes - especially potassium... Has an ECG been done? Silent ischemia or arrhythmia should be evaluated... The body's balancing mechanism is in the inner ear. An ENT (Ear Nose Throat) specialist is the one to see - especially in the presence of tinnitus. They have special tests to confirm or exclude the contribution of the ear... If there are no clues, you have to be followed up by a neurologist. Take care Wishing speedy recovery God bless Good luck"
},
{
"id": 71479,
"tgt": "What causes dry coughing with history of pleurisy?",
"src": "Patient: I have a dry cough, seem to cough more than others around me, but it's not constant, also have chest pain on left side which seems to be isolated, sometimes feels better when pressure is applied or if I'm laying on my right side. Have had these symptoms for several weeks - no fever. In the past have had bouts with pleurisy. Am not a smoker, but grew up with smoking parents. 50 yrs/female. Doctor: Hello,A pleural thickening leads to this type of pain and dry Cough, do a spirometry followed by DLCO (Diffusing capacity of lungs for carbon monoxide). We will definitely get a clue.Compare previous chest x-ray with a fresh one for any new lesion something.Hope I have answered your query. Let me know if I can assist you further.Regards,Dr. Vishal More"
},
{
"id": 93208,
"tgt": "Having lower abdominal pain. Missing periods. Pregnancy test negative. Chances of pregnancy?",
"src": "Patient: Hello right now I am having strong cramping pain in my lower abdomen . My period is 2 week late and i did a pregnancy test a week ago and it said negative. This pa\u00edn started 2 days ago and i am so scared Because it is very painful i am pregnant or what is going on with me. It happened to me last time when i was pregnant but then i suffered a miscarriage so id know if it is going to happen again??? Doctor: Hello,In case your periods are not regular, please wait a while because the pregnancy test is negative.Its most likely that you will get your periods soon.In case you dont get them even after 2 weeks \"late\" as you have said, please repeat your pregnancy test again.If your pain is very severe, please consult a gynecologist and get ultrasound scan of your abdomen done.Hope this helps."
},
{
"id": 68859,
"tgt": "What causes a white lump on the tonsil after a cold?",
"src": "Patient: i had cold the other week for a week or so and notice a white lump on my tonsil i went to doctor and took a course of penicilling which i found hard to take 4 a day on an empty stomach only managed 3 a day i have finished the course but still have the lump i some times also cougth up little smelly white lumps is this throught cancer Doctor: Hi.Cancer of the tonsil does not develop within a few days. IT would help if you know whether you had any problem in the past. This is the infection of the tonsil most probably called a ''follicular '' tonsilitis. Consult an ENT Surgeon , he would give you another antibiotic and anti-inflammatory medicines - continue treatment for 2 more weeks , if this resolve be happy and forget about the tonsil.If this is suspected by your Doctor he will go for tonsillectomy... will solve the problem for ever,"
},
{
"id": 134019,
"tgt": "What causes severe shoulder pain along with rashes on wrist and forearms?",
"src": "Patient: Extreme pain that radiates from my shoulder down to my left arm and results in a rash. The rash is visible at the point where the pain is in my sholder to many points down my arm. Rash is at the wrist, forearm, and elbow areas. Feels like paper cuts, burns, and is on fire with any type of touching. Doctor: hi,thank-you for providing the brief history of you.As mentioned the pain is in the shoulder than a thoroguh neuromuscular and skin assessment is recommended.Also, if the nature of the pain is radiating in nature, it could be related to any nerve pathology. Since nerve carries both the functions - motor and sensory as well. Also an MRI post the Neuromuscular assessment of the cervical spine us recommend. An NCV test will help further to check the course of the nerve. On a later stages the skin rashes needs assessment as the skin rashes and the radiating pain has no connection as per my understanding. Also, once coming close to the root cause the next part can be planned.For rashes, it is about the dry skin, and may be a reaction to the medicine if taken any or may be due to any bite of an insect. which might need an dermatologist attention.RegardsJay Indravadan Patel"
},
{
"id": 173701,
"tgt": "Can i give nutrolin-B and lysonate for fever and cough in my child?",
"src": "Patient: hi my daughter is 3 year old and she is having viral fever. Doctor gave only two medicines, Nutrolin B syrup and Lysonate and told to give calpole 120mg whenever she is having fever. Now fever has gone, but cough remains. I would like to know whether medication is right. fever started5 days back Doctor: Hi...by what you quote I feel that she is recovering form viral illness and it is very common in kids to have cough while the viral illness is settling. Cough is a precursor that the kid is recovering from viral illness. As the fever has remitted now the cough may remain for another 2-3 days.Fever of few days without any localizing signs could as well a viral illness. Usually rather than fever, what is more important is the activity of the child, in between 2 fever episodes on the same day. If the kid is active and playing around when there is no fever, it is probably viral illness and it doesn't require antibiotics at all. Once viral fever comes it will there for 4-7 days. So do not worry about duration if the kid is active.Regards - Dr. Sumanth"
},
{
"id": 79560,
"tgt": "Any suggestion for varying temperature, ESR count 110, calciferous deposits in lungs?",
"src": "Patient: my father is 73+. He has been having a varying temperature associated with body pain for 20 days. the doctor suspects it to be rat fever and has prescribed medicines. but his esr is 110 which is a worry. his lung x ray shows calciferous deposits. he quit smoking 30 years back. your advice please. Doctor: thanks for your question i completely understand your question the calcified opacities on the chest xray are old healed lesions.you should continue the treatment as told by your physicianpulmonologist can guide you by conductiong certain investigations like spirometry and sputum for culture and ll start you on appropriate treatment thanks feel free to ask more questionsmay god bless your family with good health"
},
{
"id": 144433,
"tgt": "Suggest treatment for vertigo",
"src": "Patient: Hello im 27 have two kids, a few weeks ago I stared getting dizzy with no explanation dr said vertigo I get super tired. Now im bruising on the palms of my hand, my knees start hurting, tingly sensation on arms, on left side of abdomen like the ovarie I get a pain that gets my back. Soon I have a dr appt since I got an abnormal pap. What is going on with me? Doctor: Hi dear ,As per your query you are suffering from vertigo . The symptoms you mention in query occur due to wide range of reasons like migraines , head injury , certain medications , ear infection , condition like meniere's disease and due to cervical spondylosis . So i would suggest you to consult ENT specialist to get examination with the help of tests like rotatory chair test , prong test , examination with otoscope . Doctor may prescribe antiemetic drug along with anti inflammatory , antibiotics and recommend vestibular therapy .For now i would like to suggest you some home remedies drink green tea with ginger thrice a day and drink lots of fluids like juices and water . Avoid caffeine product .Hope your concern has been resolved.Get Well Soon.Best Wishes,Dr. Harry Maheshwari"
},
{
"id": 2205,
"tgt": "Can i get pregnant while suffering from hypothyroidism?",
"src": "Patient: Good Morning Dr, I m married for 6years. And was on planning for these many years using Condoms as a protective measure . I never had mala d pills, Copper T insertion.. Actually I detected with thyroid last 3years back and is on Eltroxin 50mg medicine. Ma levels are normal. Ma question to u is , Is it difficult to get conceived with dis problem? Bcoz I m planning for baby now after reducing 10kg weight. Ma age is 30 n ma hubby age is 37. So I m bit worried about ma pregnancy. Still we haven t started in a natural way. Pls help and kindly revert back. Thank you Dr. Doctor: Hi.The thyroid profile is generally well controlled with medication and proper follow up, along with lifestyle modifications, a good diet and weight loss. So once your thyroid enzymes are within normal range, there should be no hinderance in conception or during your pregnancy.Best of luck."
},
{
"id": 181234,
"tgt": "What causes severe tooth ache in a 4 year old boy?",
"src": "Patient: I have a 4yr old complaining of tooth ache in lower (non 1st teeth) he started teething late at around 8-9 mtgs. He says he was biting a rubber and hurt. Doesn't want to eat anything, not even marshmallow because says it hurts. I checked gum for change in color but nothing appears out of ordinary. Can u help? Thanks! Doctor: Hi,As per your complain it seems that severe toothache after biting on rubber can be due to either deep tooth decay or fractured tooth. For ruling out the exact cause you should consult an Oral Physician or Pedodontist and get evaluated and a thorough clinical evaluation and investigation like x ray can help in diagnosis and treatment can be done accordingly.He can be advised Pulpectomy procedure followed by antibiotics and painkillers for relief. As of now you can give him Tylenol or Ibuprofen for pain relief..Give him soft diet.Hope this helps. Let me know if I can assist you further. Regards,Dr. Honey Arora"
},
{
"id": 172863,
"tgt": "What causes loose motions and skin rashes",
"src": "Patient: my baby is hospitalised for cold cough and having creps...after hospitalisation he got loose motion and had 15 motion in a one day and now having rashes...Also used rashfree cream but its not affecting and dr. told to make area open. Please guide me what should i do and is it a hospital acquired infection..?? Doctor: hi,yeah, it is a hospital acquired infection,may be concurrently infected by virus.for rashes you have to apply lotion n yes u can apply rashfree cream.though antibiotic is started u can add vit A and some multivitamin.and for motion use lactobacilli containing medicine.thanks"
},
{
"id": 29457,
"tgt": "What causes vomiting after eating food?",
"src": "Patient: Hi I have had mono for about two weeks now and most symptoms have slowly started to dissapear and now it seems like I can t have certain foods without just throwing them up and when I poop there is like this yellow remininse left on the tissue any help will be gladly appreciated Doctor: HelloWelcome to Health care magicYou are probably getting vomiting due to gastritis as a side effect to antibiotics.Take omeprazole domperidone combination once a day for 5 days.Take probiotics for growth of normal gut flora.Avoid hot spicy food or alcohol.Maintain adequate hydration.The side effect of antibiotics will fade as they are completed now.Hope I answered your query.Regards"
},
{
"id": 158552,
"tgt": "Have frequent bleeding, had trachelectomy for cervical cancer, biopsy done. Delivered, breast feeding. Cancer?",
"src": "Patient: Hello. I was diagnosed with cervical cancer in 2010 (stage 1B). I had a radical trachelectomy, and all my biopsies since have been clear. I was lucky enough to have no troubles conceiving after the surgery, and had my daughter in March 2012. I am still breastfeeding. My period returned at the start of February 2013, and I had another period at the start of March, but have been bleeding every week or two since then. My next cancercare appt is not scheduled until July, but I am concerned that the frequent periods of bleeding I am having could be indicative of a return of the cancer? Or is it normal for my period to be much more frequent after breastfeeding? Doctor: Hi, What was the report of the pelvic lymph nodes? Stage IB has got two subtypes IB1 and IB2 . Manage protocol differs in these two subtypes. More definite information is needed. Anyway, what are the follow up protocol your doctors have advised? Bleeding in a treated case of cervical cancer should always be carefully evaluated. Clinical examination is mandatory and PAP smear is necessary. You should also mention the HPV status. Consult your oncologist."
},
{
"id": 147503,
"tgt": "Wokeup with a ague, felt cold, eyes and frontal head pain, light headed. What can be the diagnosis?",
"src": "Patient: This morning I woke up with what appeared to be ague. I felt very cold and I live in a Tropical Climate Where it is hot and humid all the time. I had a mask of pain across my eyes and frontal part of my head. Also I was light headed. The apparent ague happened to a lesser degree up to 10:45 am. There is no muscle of joint pain . At this moment I feel good I am walking around instead of being flat on my back in Bed. I have not been sick in bed like that since I was a small Child. What is your diagnosis? Will it come back Worse? I am a 41 year Old Male and seldom very Ill. Doctor: Hi,These could be the symptoms of sinusitis. I suggest you take paracetamol and ibuprofen for the pain. Drink plenty of water. Rest if possible. Try nasal decongestant sprays or tablets and steam inhalations. Regards,Dr K A Pottinger,MBChB. FRCA"
},
{
"id": 57914,
"tgt": "Is enlarged liver with hep C grade 1 stage 1 concerning?",
"src": "Patient: I have hep. C, grade 1 stage 1. I am 42 years old, male. I do not drink or take drugs. I have not undergone treatment for hep c yet. I recently had an abdominal ultrasound and my liver was measured at 17 cm. I have an appointment to see my gastro dr. How worried should I be about the enlarged liver and how can it be treated? Doctor: HIThank for asking to HCMAs long as the infection is concern particularly the viral infection liver gets enlarge bit, but this will come to normal with the time, and this is just nothing to worry, I personally think that this is good sing that your liver is increase in size and not shrink in state of infection it means it fighting against the infection, take care have nice day."
},
{
"id": 182447,
"tgt": "Could gum swelling with hardness be due to flu?",
"src": "Patient: Hi, I just notice that my left bottom gum is swollen. I used my hand to feel it and it'd really hard. I had an operation slightly above the gum area where my 2 back teethe were removed due to cyst infection 15 years ago. Right now, I am still recovering from flu and cough. Can this be a side symptom of my flu? I'm just worried if my cyst reoccur again. Should I be worried? Doctor: It may be a recurrence of cystic infection but since you have been asymptomatic for 15 years after its treatment chances are lesser for that, you may rather suspect an infection due to some other adjacent tooth, it is not due to cold and flue. To be sure i would suggest you to get it examined at the earliest and also get a radiograph of that region to be sure of what it is rather that delaying it and causing it to increase further."
},
{
"id": 63844,
"tgt": "What causes bumps on the body?",
"src": "Patient: I have four bumps total on my body, under the skin. One on each arm on the top/side, in the same location on each arm; one one right arm near elbow; and one on the middle of my left leg facing inwards. Recently the bumps on my arms have become more swollen and red. The only trauma incurred has been an injection in each arm of phenergan about three months ago. These bumps showed up about two months ago. No other symptoms. Doctor: Hi,Dear,Thanks for the query to HCM.your concern is why painless lump with Phenergan shots 3 mths ago.This lump is of the Fibrosed organised tissue reaction to the phenergan shots preservative chemical.Hope this would reply your query to your satisfaction.Welcome for any further query to HCM in this regard.With Regards,"
},
{
"id": 113243,
"tgt": "Had very painful back spasms. Should I see a specialist?",
"src": "Patient: I had two horrible back spasms - one a sharp pain like a dagger going from one side of my lower back to the other side and the next one 2 days later like a crack right in the middle of my lower back then 3 days later my right hip radiated excruciating pain at the top of my right thigh with any type of leg movement, with no radiating down my leg. Should I let time take its course and let it heal or do you feel it necessary to see a specialist? It has been 2 weeks since the onset but is getting better. Doctor: Hi Thanks for the query Your symptoms seems to be reflecting sciatica due to nerve compression. Such conditions should not be ignored and you must consult a neurologist for proper management as with time the disease progress and neurological deficits will develop which will land you in surgery. So, its better to get it treated now at its early stage Thanks Regards"
},
{
"id": 130041,
"tgt": "What could sharp pain in right butt with any movement indicate?",
"src": "Patient: Hi, my name is jeanette and I've been having a pain in my right butt ,it hurts when I'm laying down or when I stand up and sit down and when I lay in bed I have to do it slow cuz it hurts like a sharp pain.even when I try. Picking my little brother up. Why does it hurt its been 1 week and half. Doctor: Hi,Its due to compression of nerve in lumbosacral region.Get xray of lumbosacral spine in AP and Lateral view done and sand me the report.Till then have tab.aceclofenac100mg +thiocolchicoside4mg twice a day by prescription of family doctor.I hope it helps.Thanks,D.CHANDER MOHAN SINGH."
},
{
"id": 69371,
"tgt": "What causes sore lump on vagina?",
"src": "Patient: On the left side of my virgina I have a quarter size lump that sometimes itch and be sore. I had this lump for a month. Sometimes it doesn't bother me at all. On the right side of my virgina I have a smal it hy bump. It went away and came back. What is this? Doctor: Hi,It seems that you might be having yeast infection in vagina and now there is super-added bacterial infection.Apply triple acting cream locally.Go for one antibiotic medicine course for 3-5 days.Keep local hygiene proper.Ok and take care."
},
{
"id": 18270,
"tgt": "Are elevated blood pressure levels a matter of concern?",
"src": "Patient: I ve been taking amlodipine 5mg daily for 4 months now due to blood pressure. My blood pressure reading is usually 120/80. Lately, my blood pressure reading went up to 140/90 or 130/90. That reading was for 2 days. Should I be worried about these blood pressure readings? Doctor: Hello and Welcome to \u2018Ask A Doctor\u2019 service. I have reviewed your query and here is my advice. 140/90 is not a matter of concern. Continue Amlodipine and avoid salty foods. Check BP again after two weeks and if the blood pressure recordings are persistently over 140 range, additional drug may be needed. Hope I have answered your query. Let me know if I can assist you further."
},
{
"id": 210091,
"tgt": "Why do I get annoyed and have suicidal tendencies?",
"src": "Patient: Hi m uday m aged about 21 I am getting irritated while talking with others and I scold them very badly and I will suffer after words and I willbe sad after scolding them an I will try to suicide ...I won't scold any one wantedly..why I do like this I don't understand??? Doctor: HiThanks for using healthcare magicI think, you have either underline mood disorder or borderline personality. Some time due to underline personality or mood disorder, patients get frequent mood swings and suicide ideas. In that case, you should consult a psychiatrist and get yourself diagnosed. Treatment in your case depends upon diagnosis. In case, you need our help you can ask.Thanks"
},
{
"id": 131097,
"tgt": "What causes pain in right upper quadrant?",
"src": "Patient: I have had an ultrasound, cat scan, and hida scan all normal I m still having pain in my right upper quadrant what could it be and also I had my hida scan done this morning but they only did one injection of the radioactive tracer and then made me drink ensure was it done right?? They never did a cck injection is that accurate?? Doctor: This is accuratly done of scan. Swallowing of of radioactive fluid which absorbed by veins to be showen in hida scan after certain time of swallowing."
},
{
"id": 81990,
"tgt": "What is the dull pain in my left chest?",
"src": "Patient: I am 45 years old and have a dull pain in my left chest from my back all the way thru my breast. I had a mammogram to rule out any breast issue and it came back clear. I have shortness of breath but not all the time. The pain was more frequent when I slowed down in the evenings, but in the last week or so it is there almost all the time. It woke me up last night and the night before. I don t have any heart disease in my family. I don t know what direction to go in and it is starting to be a problem. Doctor: Thanks for your question on HCM. I can understand your situation and problem. Since it is left sided chest pain, we need to rule out cardiac cause first.Your other symptoms like breathlessness and low energy in evenings with night time awakening can be seen in heart disease. So please get done1. ECG 2. 2D ECHO3. Stress test.If everything is normal than no need to worry much. You may have anxiety associated muscular pain. So avoid stress and anxiety. Be relax and calm. Take good painkiller and muscle relaxant."
},
{
"id": 68001,
"tgt": "Suggest remedy for dark purple lump near the opening of anus",
"src": "Patient: Yesterday I didn t feel any discomfort, but today my anus was bothering me. I believed it to be an ingrown hair because I m a relatively hairy person, but after I got out of the shower I took a look and there was a large dark purple lump near the opening of my anus. I d prefer not to go see a doctor unless necessary. Can you tell me what it is and possibly how to treat it? Doctor: Hi, dear. I have gone through your question. I can understand your concern. You may have some hemorrhoid. Or you may have some pilonidal sinus. You should go for examination. Consult your doctor and go for examination. Then plan surgery accordingly. Hope I have answered your question, if you have doubt then I will be happy to answer. Thanks for using health care magic. Wish you a very good health."
},
{
"id": 224335,
"tgt": "Can oralcon contraceptive be used as emergency contraception?",
"src": "Patient: i am 24 years old with a 5years son, i am currently on paragard IUC for three months so i want to know if i can use a morning after pill if something goes wrong during sex. i also want to know if oralcon contraceptive can be used as emergency contraception and how. thank you Doctor: Hi,Paragard is a very effective contraceptive method. It is more than 99% effective in preventing pregnancy. Therefore there is no need to use morning after pill after sex. I could not figure out why you thought of using morning after pill while you are already on a very effective method of contraception.Oralcon contraceptive can be used as emergency contraception. You need to take 4 tablers of Oralcon at the earliest after unprotected sex and 4 tablets again after 12 hours. I have written this just for your knowledge.I hope I have answered to your satisfaction."
},
{
"id": 182001,
"tgt": "Suggest treatment for bump on gums with infected wisdom tooth",
"src": "Patient: I currently have an infected wisdom tooth that I am on antibiotics for. I saw my dentist today. I'm getting all four of them out on Friday, but by the one that is Infected there is a fluid filled bump on my gum. The entire area is in a TON of pain, and I've been using a salt water rinse after I eat anything. Would I be relieved of some pain if this thing burst? Doctor: Thanks for your query, I have gone through your query.The pain is because of the pus secondary to the gum infection over the wisdom tooth, Once it is drained, the pain will come down. If i am your treating doctor, i would have suggested you to take a course of antibiotics particularly, amoxicillin and metronidazole combination. Along with this, you can take an analgesic like diclofenac. Get the tooth removed after completing the course of antibiotics.I hope my answer will help you, take care."
},
{
"id": 103260,
"tgt": "How can breathing problem be treated permanently ?",
"src": "Patient: Hello Doctor My Husband has suffer due to breath problem any time suddenly his breath has been stop an if he take one capsule in pump that time he get relax is it any forever solution? please let me know Doctor: Hi, Substances that cause breathing problem are known as triggers. Avoidance of triggers is the No. 1 way to control allergies and asthma. An accurate diagnosis is important before you can treat and prevent breathing problems. Each of us is different. The specific medication and treatment program that works for a family member or friend may not be the correct one for your problem."
},
{
"id": 225540,
"tgt": "Taking birth control, strep throat, vomiting, forgot a pill, unprotected sex, bleeding. Chances of pregnancy?",
"src": "Patient: Hi, this is a last resort but I need to know what my chances are. I'm on yaz birth control. Started a new pack exactly 13 days ago and took them religiously up until this pack. about two days after starting this pack I got strep throat and was put on amoxicillin. I was vomiting at night time (a couple of times) where I don't know if the pill had a chance to fully sink into my body. I went home for seven days, and about 3 days after I went home, I forgot a pill but doubled up the next night. and on day seven of being home I had unprotected sex with my boyfriend where he ejaculated in me. I went back to my apartment (this being the 29th when I went back) my boyfriend and I continued to have unprotected sex repeatedly over 3 day span from the 29th to the 1st. in which over those days I forgot two pills!! Which i threw out and started fresh. I've been bleeding for the past two days (obviously my body doesn't know what's up) but Im stressing and I need to know the chances of pregnancy\\ chances of having ovulated. Thanks!! Doctor: Hi,Thanks for the query. The possibility of protection from pregnancy by the hormonal pills is less in your case, as :- You had vomitings after intake of pill- Antibiotic usage-History of missed pillsSo you better use some additional method of contraception till completion of this pack. And the bleeding you are having could be withdrawal bleeding due to missing the pills.Possibly it will subside soon. Wait till your expected date of periods. If you miss the periods, go for blood test for pregnancy once.If the test comes negative, you can continue your next pack as per the schedule."
},
{
"id": 140217,
"tgt": "Can any online doctor help me with the following results?",
"src": "Patient: The MRI report of my lumbar spine has this conclusion, please explain where to from here?rineural fat signal is preserved. IMPRESSION: Multi factorial central spinal stenosis at L4/5 with associated bilateral lateral recess stenosis. Focal central and marginally left paracentral disc protrusion at L5/S1 resulting in minimal narrowing at left lateral recess but no nerve root displacement or entrapment. Considerable facet degenerative hypertrophy at the L4/5 and L5/S1 levels. Doctor: Hi, Your MRI report is indicating that you have some common radiographic signs of what we would generically refer to as degenerative disc disease or degenerative arthritic changes likely as a result of OSTEOARTHRITIS which is commonly seen in people who do heavy manual labor or do repetitive lifting all day long. You likely have low back pain symptoms. With this report, I would encourage you to speak to your doctor about getting started in a PHYSICAL THERAPY program that encourages movement and stretching exercises (even though it will likely be a little uncomfortable) since these will be very good for you in the long run. From what you've presented I do not see you as needing any type of surgery at this point. I also am not a provocative fan of \"back injections\" since their effects are highly variable and of relatively short duration in the patients that I see in my office and hospital practices. Other conservative forms of treating your back condition would be medications, Ultrasound/diathermy sessions, TENS unit, Acupuncture or if that's not your cup of tea, \"dry needling\" and lumbar traction. I would not encourage you to do epidurals, facet injections, vagal stimulators, or surgical procedures at this juncture but would very much promote exercise, good diet, weight loss and conditioning to strengthen core and back muscles, stop smoking (if you do), and maintain learn about keeping good posture while sitting, standing, and even walking. Hope I have answered your query. Let me know if I can assist you further. Regards, Dr. Dariush Saghafi, Neurologist"
},
{
"id": 147270,
"tgt": "Coma after accident, twitches eyes, no change in brain activity. Is there a chance for him to awaken?",
"src": "Patient: hi! my ? is my brother had a horrible accident thrown from a charoit been in a coma for 5 weeks no change his brain activity is at 3. no movement twitches once in awhile his eyes open but no movement in the eye balls at all he has a trachia in his neck for breathing is the twitching a good thing and his eyes opening is this a good thing he has severe brain trauma fractured skull , fractured nose fractured jaw IS THERE A CHANCE FOR HIM TO AWAKEN? Doctor: Hello, I'm very sorry to hear about your brother's accident. Brain injuries are devastating, especially when they leave a loved one in a coma. I'm a bit confused by your saying his brain activity is at \"3\". Sometimes people mean that the Glasgow Coma Score is a 3, which means that there is no movement, no eye movement, and no attempt to speak. It represents very severe brain injury. Random eye opening and muscle twitching in the setting of massive brain injury are never good. They can mean anything from seizures to random discharges of a permenantly damaged brain. While 5 weeks is not a \"long\" time in terms of brain injury (especially in young patients), what you describe does not sound good. While there may be a chance for your brother to \"awaken\" it is likely very very small. If you are asking the possibility that he will ever be \"normal\", that I'm sorry to say is not a possibility. It is not my intention to remove your hope of a miracle, but I want you to know where your brother stands in relation to others with similar injuries. My prayers are with you and your family."
},
{
"id": 10672,
"tgt": "Suggest treatment for hair loss",
"src": "Patient: hi,i am getting bald especially on top of my head,i am a Biothera client rightnow n its already 10 months , i dont see any changes in fact i'v been noticing that i think its getting wores ritenow i don't know what to do, if i will stop going there or wait for the result? and is there any side effects of a Biothera? thank you!! by the way i am 68 years old. Doctor: hi there.1. your concern is androgenic alopecia.2. it's genetic and hormonal dependant.3. we can control the hair loss and can make the remanining hair better.4. you can try rogaine 5%, 6 sprays once daily bed time.5. have adgaine capsules, daily one after breakfast.6. wash your scalp only with drinking water and prefer protein rich diet.this will help you in making the hair thick and better.."
},
{
"id": 162608,
"tgt": "What causes body rashes after recovering from viral meningitis?",
"src": "Patient: .daughter was 7 weeks when she was diagnosed with viral meningitis . She was unexposed and it came on after 104 degree fever . Every since then she has suffered with unexplained rashes that often get treated as hand foot mouth. However, these rashes become infected and eventually turn into impetigo for no explained reason. She often gets hot and flares in what I would describe as exzema but its almost more like hives. Her lymph nodes on the back of her neck have been huge and concerning since she was an infant. At times its also the ones in her neck and under her arms but our pediatrician re assures us this is normal. Lately she has been going cross eyed when trying to focus or shell daze off but she seems to laugh it off after some joking from her older sibling. She has seemed to hit most mile stones on target but we recently had a friend lose their daughter to HLH and some of the things in her past medical history identify a little too well for comfort for me. I would just like some answers if possible before I go into full on Mommy Freak Out mode. Thanks in advance . Doctor: Hello and Welcome to \u2018Ask A Doctor\u2019 service. I have reviewed your query and here is my advice. I am sorry for your kid's symptoms. But from what you say I feel that your kid should be evaluated for immunodeficiency and needs for up for a probable seizure disorder. I suggest you get in touch with your pediatrician regarding this. Best regards Dr Sumanth Amperayani"
},
{
"id": 106262,
"tgt": "Why is it hard for me to breathe when excited ?",
"src": "Patient: Sometimes when I get excited if I m kissing or so I suddenly experience a rapid increase of breath and it becomes hard to breath. Doctor: get your spirometry done. you will know the answer."
},
{
"id": 207139,
"tgt": "Suggest remedy for addiction to masturbation",
"src": "Patient: sir i am a male and i am addicted to masterbating tell me what should i test to make that \"liquid\" which is wasted in masterbating sir due to this habbit my right side of a chest is outside and left side is inside i am very ashamed sir please tell me the food to make money in my bones please help me my email address is YYYY@YYYY and phone number is 0000. Doctor: Maturbation is natural. Almost every male would have done. So first thing is accept it is natural and normal act. if you are doing it again and again, many times a day, it can make you feel exhausted. Natural sex is always better. to over come masturbation addiction, whenever you are masturbating, be very slow, bring your attention towards your breath. By doing this gradually you can come out addiction. Surya Namaskara & Pranayama is also helpful.Masturbation has nothing to do with your chest size."
},
{
"id": 49016,
"tgt": "Is is normal to have leg, muscle pain with kidney stones?",
"src": "Patient: im pretty sure i have kidney stones again i had them 12 yrs ago in my other kidney i am a female 38 and i have 4 kids i have high uric acid crystals in urine and low uric levels in blood it is starting to hurt down my right leg all the way to my knee is that normal with kidney stones Doctor: Yes it could be a sign of recurrent recurrent kidney stone which can be either in the kidney itself or more likely in the urine drainage path at some level."
},
{
"id": 105926,
"tgt": "How does calamine lotion help in allergy ?",
"src": "Patient: I take KFC Chicken pieces thatday onwards i am suffering with some allargy.I applied cocounut oil then it somewhat better. I taken suggestion from our doctor .he has given the cetrizine tab & calamine lotion .wht is the use of calamine lotion? Doctor: doctor has prescribed medicine very rightly,as you have got allergy to chicken and produce some reaction to your skin. cetrizine gives systemic effect through blood and use of calamine lotion gives shoodhing and cooling effect to your part,affected. it prevents soreness of the part"
},
{
"id": 145053,
"tgt": "Suggest treatment for desiccation at L4-L5 and L5-S1 discs",
"src": "Patient: there is mild exaggerated lumar lordotic curve small aterior marginal osteophytes noted there is early dessication at L4/5 AND L5/S1 discs At L4/5 level, there is a mild annular bulging of the disc causing flattened anterior thecal sac with very minimal bilateral lateral recess narrowing without any significant neural foraminal stenosis. At L5/S1 level, there is posterocentral tear with mild annular bulging causing flattened anterior thecal sac with mild bilateral lateral recess narrowing without any significant naural foraminal stenosis. Rest of the vertebrae and intervertebral discs are normal in morphology, heights and signal intensity. Early changes of facet arthropathy is seen at L4/5 and L5/S1 levels. The spinal cord and rest of the nerve roots are normal in structure and exhibits normal pattern. No soft tissues abnormality is seen. Doctor: Hello dear Your Mri report is suggesting early and minimal lumbar spondylosis. With this report you need not to worry. You have not mentioned your symptoms. As far as backpain is concerned you can get benefit with physiotherapy, avoiding stooping forward, lifting heavy objects, squatting. Treatment can be taken in the form of neuro modulator for pain like pregabalin, amitryptiline or gabapentin. Local pain reliever ointment can also be used. You will recover early if you follow the advice. You may visit a neurologist. Hope you found the answer helpful. Wishing you good health Dr Neeraj Kumar Neurologist"
},
{
"id": 87669,
"tgt": "Suggest treatment for stomach ailment caused by chewing gutkha",
"src": "Patient: i am 25, male, i have acquired the stomach problem by chewing gutkha. i think it washed my stomach's internal layer. now i have quit the gutkha but my problem still exits. i have been suffering from loose motions for last one and half months. even i took so many medicines but not benefited. i cant eat anything hot and spicy. i need to go to toilet immediately as i eat anything like that. being a student i have been feeling so much depressed and sad because of this desease. i live away from my family, therefore its very painful fore me to fight with this desease. please help me, i promise you not to chew gutkha or anything like that. Doctor: Hi,Chewing Gutkha does cause gastritis and hyper acidity but problem right now you are having might be due to chronic gastro-ontestinal infection.Go for one course of ofloxacilin, tinidazole combination medicine for 5 days.Afterwards take Omeprazole for few days.It is good thing that you stopped chewing Gutkha and now do not start again.Avoid fried and junk food.Ok and take care."
},
{
"id": 170366,
"tgt": "Is a broken teeth in child a cause for concern?",
"src": "Patient: Hi, my daughter is 9 yrs old and the top half of her tooth came off...what remains is a piece of her tooth & her gum, but i looks as if it s her gum in between 2 teeths. I ve never seen this before. It s not hurting her. Is this something we should be worried about? Doctor: hi..according to your observations from your daughter's mouth.. the top half of the tooth hs came off...so the remaining piece of her tooth is the ROOT portion of that broken tooth..which is present in between 2 teeth.You should visit your dentist.. for the removal of that remained ROOT portion of that tooth.."
},
{
"id": 65491,
"tgt": "What does lumps on back of neck indicate?",
"src": "Patient: Hello, I found two small lumps on the right side of the back of my neck about a week ago. I had flu/cold like symptoms. I thought one was a zit so i tried to pop it, but it just made it red and sore. Over the last week they have gotten considerably smaller and I feel like I'm shaking off the cold. Any idea what it may be? Doctor: hi dear thanks for the query on HCMhistory suggests a folliculitis. you need to clean the area regularly with warm water with 2 teaspoon of savlon. apply ointment mupirocin or any antibacterial ointment.thank youhope the query is answered"
},
{
"id": 60933,
"tgt": "What does a lump under the jaw on the right side indicate?",
"src": "Patient: I have a lump under my right jaw line. It feels about the size of a marble. Should I use a heating pad or ice pack for the swelling? It is approx. an inch and a half on the right side of my chin along the jaw line. And now my right ear is starting to get achy. I am 54 and have never even had an ear ache. Doctor: Hello,The most possible reason here is salivary gland swelling from a viral infection which gives an earache too. In our clinic, according to the clinical severity, I suggest proper antibiotic as of cephalosporin group and anti-inflammatory medicines to recover early. Ice application will help better against a heating pad.Hope I have answered your query. Let me know if I can assist you further. Regards, Dr. Bhagyesh V. Patel"
},
{
"id": 173205,
"tgt": "Suggest remedy for over dose of Amoxicillin in 8 months old",
"src": "Patient: My daughter had an ear infection and was prescribed with 3 ml of amoxicillin twice a day. We accidently gave her 3 teaspoons 2 times as we misread the medicine bottle. I am very worried of an overdose. Could you please provide me some guidance? She is 8 months and weights 19.54 lbs Doctor: I share your concern. Dose of amoxycillin is [Dose: 20-30 mg/kg/day PO divided q12h; Max: 30 mg/kg/day; Info: dose, duration vary by infection type/severity[>3 mo]Dose: 25-45 mg/kg/day PO divided q12h; Max: 875 mg/dose; Alt: 20-40 mg/kg/day PO divided q8h; Info: dose, duration vary by infection type/severity. YOu have not mentioned the strength of amoxycillin in the bottle. I think it is 125 mg/ 5 ML . In total YOu may have given 125 x 6 = in total which is well in range.as dosage varies with the conditions. Anyway you can ask your doctor, I hope you are convinced. Regards."
},
{
"id": 185184,
"tgt": "Suggest treatment for mouth ulcer",
"src": "Patient: Hi! My father aged 75 yrs has been suffuring from mouth ulcer (mainly in gum) since last 1 and half years. Sometimes he gets relief but it is temporary. He used to take vit-B complex suppliment sometimes.on advise of local dentist some of his teeth were removed but in vain.. Please advise. Doctor: Thanks for using Health Care Magic.Read your query.First I wish your father good health.Relating to the mouth ulcers,this can be due to chronic vitamin deficiency,stress factors and also in consideration of the age.I would advice you to ask him continue with the muti-vitamins the doctor has prescribed for him.*For the ulcers ,you can apply Mucogel/Mucopain gel on it for a while till the ulcer heals.*Advice him to do betadine mouth wash gargling.*Avoid spicy food and stress should be reduced.*Add all nutrients in his food and check for any digestive problem which will also aggravate them.Hope this was useful.Thanks and regards."
},
{
"id": 4513,
"tgt": "Does ovare, fertibex and foli23 tablets have any side effects and will this help in conceiving early?",
"src": "Patient: Hi, i m 29 yrs lady. me & my husband want baby early we try to concive but my period is not regular. Gynaec doctor suggest me Tablet ovare & fertibex three times in day. & FOL123 tablet at night. this is ok for concive early. This tablets are having any side effect? plz reply. Doctor: Hi,Thanks for the query. Ovacare and fertibex contains multi vitamins and minerals. They help in improving the general health and there by in regularization of menstrual cycles.Folic acid is essential for the neurological development of the baby. So it should be started two to three months before pregnancy. In the next menstrual cycle, you track your ovulation and plan intercourse around that period. This will increase the possibility of pregnancy. If you are not getting regular ovulation, better to go for ovulation induction with your doctor's advice. Then track the ovulation with follicular study and plan intercourse around ovulation period. Take care."
},
{
"id": 186918,
"tgt": "What is the treatment for a bump on the gum?",
"src": "Patient: I have a bump on my gum, that only appeared after I nicked my gum with a hard tooth brush. It filled with blood for two days and would come out and fill right back up. But just right now puss came out of it, and then just a lot of blood. I have no pain whatsoever and I have very healthy teeth. Doctor: Hello, Thank you for consulting with HCM.Actually it looks that you have traumatized your gums with toothbrush and now an abscess has been formed.You have to visit a dentist as he will examine the lesion and clear the area with irrigation and scaling.Hope it will help you."
},
{
"id": 99040,
"tgt": "What is the treatment for a repeated eye allergy?",
"src": "Patient: hiii so i had a classmate before who has a sore eyes(??), well im not sure if that s a sore eyes but she went home because her eyes is so red and she said that s so itchy and she came back after 1 week, well she s my seatmate and i miss her so much and all but that s why maybe im infected by her sore eyes because she s my seatmate. the day after that my eyes got so itchy and became so red and all. so i just let that but a month later but my eyes are still like that, i became so worried so i went to the doctor and he said i got an eye allergy so he gave me an eye drop for that and i used it everyday and it helps but we went out of the town for a vacation and i brought it there so i left it there somewhere so when i got back, my eyes were not so itchy na and now it became so itchy again well my right eye only though so i rubbed it and rinse it with water so when i look into the mirror i notice something. i notice a jelly like something below my eye and i am so worried about it. what would i must do? Doctor: No need to worry. Constant exposure to particular atmosphere that doesn't suit your eyes has caused allergy.Better you use naturals for eye wash . if u get Ayurveda Triphala powder, soak it over, sieve in the morning, rinse eyes with it. Totally harmless, safe.Hope it helps you."
},
{
"id": 121973,
"tgt": "Suggest treatment for Spondylysis",
"src": "Patient: Age - 59, height - 5 ft 4 inches, weight - 83kg, gender - female, i havw thyroid & spondylysis problem. Around 4 months back i had fallen on road & my entire body weight was on my right hand. Since then i have had continous medications as per orthopedic docotor recommendations. X-ray is normal & there is no fracture. But my pain has just increased with every passing day & is now getting unbearable. Doctor: Hi, Do take tablet Ultracet. If pain is unbearable, take injections in nearby hospital to get rid of pain as soon as possible. Hope I have answered your query. Let me know if I can assist you further. Regards, Dr. Parkavi Gunasekaran, General & Family Physician"
},
{
"id": 127745,
"tgt": "Is severe pain on the toes a sign of Morton s neuroma?",
"src": "Patient: I am having pretty bad pain in outer three toes of left foot when I walk or stand for long periods...It gets so bad I have to get off my feet until it passes.....I love to hike and am planning a trip soon..I m worried I ll have to stop too often....is this morton s neuroma or could there be something else....I feel if I put a cylinder under the toes it might help the pain but i ve found no orthodics Doctor: Hello,What I can infer from your complaints is that you may be having symptoms associated with foot strain or Morton\u2019s neuroma.Morton\u2019s neuroma occurs when small nerves in the foot are compressed in between adjacent toes which occur when you wear tight footwear.I suggest you change your footwear to medicated soft footwear for a few days and then see how it goes.Hope I have answered your query. Let me know if I can assist you further.Regards,Dr. Santosh S Jeevannavar"
},
{
"id": 3728,
"tgt": "Is there a chance of pregnancy from pre-ejaculate?",
"src": "Patient: My boyfriend and I had sex tonight. It was the first time for both of us and while it was happening, the condom kept slipping off and getting stuck half way in my vagina. It scared me at first and he assured me that it was going to be okay. So it slipped off quite a few times and we re-adjusted it every time it came off. He did not ejaculate in me because he wanted to make sure none of it would get in me. But I m afraid some of the pre-cum may have gotten inside me. And I m expecting my period within the next few days. Is there a chance I could be pregnant? Also, could I get pregnant from us rubbing our genitals on each others? Doctor: Hi, I understand your concern. Rubbing genitals externally can not cause pregnancy. Pregnancy is possible only when sprm enters vagina. Even precum has sufficient number of sprms to impregnate a woman. So though less , but there is chance of conception due to precum/ some semen escaping in vagina while you tried to adjust it. Even psychological stress can delay a normal period Blood HCG testing can be earliest diagnostic aid . Please remember to use proper contracetive before having sex hence forth. Thanks."
},
{
"id": 51363,
"tgt": "30 year old having polycystic ovary syndrome and IgA nephropathy. Had a cesarean delivery due to preclampsia. High protein in urine. Taking coversyl",
"src": "Patient: Hello i am a 30 year old female with Polycystic ovarian syndrome and kidney disease which is called iga nephropathy .I have a healthy 11 month old delivered at 37 weeks via c-section due to preclampisa.I would love to get pregnant again but wonder if it is safe for my kidneys.I currently have high amounts of protein in my urine and i take coversyl to help control it.I have around 50-60% kidney function and creatinne level stays pretty much the same at around 100.I had to take clomid to conceive the first time and wonder if i will need it again.My cycle is very irregular and i haven t had a period since Feb except for the 9 days of light spotting i am experiencing now. Doctor: You need to quantify the proteinuria in your urine. If it is less than 1gm in 24 hours you can go ahead with the pregnancy as your creatinine is also with in reasonable range. But you should understand this comes at a slight increased risk of prematurity of baby, preeclampsia and worsening of renal functions. You should also stop coversyl once you conceive and have a close follow up with the nephrologist through out your pregnancy"
},
{
"id": 139659,
"tgt": "Suggest management for Ramsay Hunt syndrome",
"src": "Patient: My husband who is 34 years old and 178 lbs, and 5 11 was just informed he has Ramsay Hunt syndrome. We do not how what type he has and are on an antiviral medication. He has been told 4 other times it was Bells Palsy but it was not. He also has headaches, and many other symptoms such as insomnia, headaches, ear aches, etc.. We have done MRI s, CT Scans, and tons of blood work to come up with this solution but are still very confused on whats next. The neurologist, ENT, and general doctor are not fimilar with this syndrome. What do we need to do next? They have told us he has an infection in his sinuses behind the skull, but do not want to do a simple surgery on drilling that out is they way it was described to us. We have no clue why and what we need to do to be proactive. We really would like to prevent Type 1 if it s not to late. Please tell us what we need to do next. It s very difficult listening to so many doctors and not being proactive to help with all these symptoms. Doctor: Hi, First of all,To treat Ramsay Hunt syndrome,you should take acyclovir 800 mg five times per day for 7 days and prednisolone 20 mg three times per day after meals for 5 days,then you should decrease the dose of prednisolone by 15 mg every 4 days to stop it.Second,you should do some investigations such as:CBC and fasting blood sugar. The surgical operation may not help you. Other infections can be treated with antibiotic. Hope I have answered your query. Let me know if I can assist you further. Regards, Dr. Mustafa, Neurologist"
},
{
"id": 33668,
"tgt": "Suggest treatment for blepharitis",
"src": "Patient: I have blepharitis and my upper and lower eyelid margins are quite thick and have been for about two years now. My doctor told me to use warm compresses followed by a lid scrub. I use the scrubs at night and have been for about two years now. I make a solution with a cup of water and two drops of baby shampoo and then I dip a cotton round into the solution and squeeze it out just a little so it isn t dripping wet. I then wipe from the inner corner to the outer corner about six times and then rinse the lids off in the shower. My doctor told me to express the glands after the compress but nothing comes out. In the morning I use the warm compress but no scrubs. When I use the compress in the morning the eyes look very good and the lid margins look much better but after about a half hour the eyelid margins look thick againl Doctor: Hi,As this infection of eye lids is since long time it indicates that there might be having some chronic infection on scalp like Dandruff causing this problem.Go for treatment of dandruff if it is found.Apply and rub eye antibiotic cream on eyelids.Do eye wash daily with plane water.Ok and take care."
},
{
"id": 146913,
"tgt": "What causes lightheadedness while sitting and standing?",
"src": "Patient: My husband, 58 yrs old, woke up this morning with ringing in his ears and after a few hours was lightheaded and is now unable to sit or stand without getting dizzy. The ear thing has gone away, but not the lightheadedness. Does he need to be seen urgently? no signs of cold or flu. He has a family history of stroke and heart disease and is on medication for cholesterol, not overweight, no smoking, and physically in good shape. Doctor: hii think your husband needs to be seen urgently by a neurologistthe chances are that he is having a strokethe symptoms of stroke usually come suddenlythey may not always be typical- the paralysis is not a constant feature of all strokes.vertigo, giddiness, dizziness may be a part of what is known as a PCA stroke. There are many varieties of PCA stroke.This is of course the most drastic condition that needs to be seen as soon as possible. Stroke can be best seen if seen by a doctor within 3-4 hours. There are other possible diagnosis, but the treatment for them can come slowly.as stated, stroke has to be treated in first 3-4 hours for best effectSo please get to a stroke unit hospital as soon as possiblebest of luckDr Mittal"
},
{
"id": 49827,
"tgt": "Suffering from Kidney issues, pseudomonas aeruginosa infection of bladder, scabies, folliculitis. Treated with elimite. Suggest",
"src": "Patient: Hello Doctor!I am suffering from Kidney from last 10 years and I am on regular CAPD.I have rashes on my back and areas around spine. Tried scabies treatment of Elimite (my nephro consultation) but did not work. I have had many a times Pseudomonas Aeroguinosa infection of the bladder many times. I think it is folliculitis problem because all the symptoms are same as mentioned on various websites.First there is a small tiny and very itchy bump and after few days it develops pain grows bigger and after that it dries.There are many tiny itchy bumps developing on my back and thighs thus causing a lot of itchyness. Please help me. Doctor: HIThank for choosing HCMChronic renal problem later or sooner causes some kind of skin manifestation, in certain cases, you need to see the renal function test, if turned abnormal then correct it, you can have words with your nephrologist, for the scabies \" Gama benzine lotion would be very nice apply this all over the body at night for three days, wash your linen in hot water, treat all the garment with chlorohexadine solution, you will get the nice result, have good day."
},
{
"id": 33707,
"tgt": "Suggest precautions to be taken after strep backterial exposure",
"src": "Patient: could streptooccocus bacteria remain in jaccuzzi jets and infect another person? the renter that was here before me, had strep. she said it was misdiagnosed and in a strange way went to her kidney causing life threatening disease which she is still fighting. she said she tried to cure it with herbs and acupunture,etc. she used a lot of essential oils in her jacuzzi bath. when I moved in the jacuzzi jets were ejecting very dark black stuff. I clean and tried to disinfect it many times and ran the jets thru until they ejected no more. Also I used vinegar to clean them. After I thought they were clean I used the tub a few times and once more stuff came out. Could the strep that had infected her somehow infected me? does the strep bacteria remain in and on surfaces even after cleaning? could it enter me? age 60, height 5 1 , 128lbs. hereditary high cholesterol. Doctor: The type of strep complication the renter had would not necessarily occur in you, even if you got strep. Certain types of strep can cause kidney injury in people who are susceptible, but it is a particular reaction of the body in only some people. That being said, all sorts of bacteria and other types of infectious organisms can reside in hot tubs and jacuzzi jets and cause infections. A very thorough cleaning by people specialized in this may be necessary. If stuff is still coming out after you've tried to disinfect them several times, I would suggest that this be looked into. In the meantime it would be best to avoid using the tub or at least the jacuzzi jets until this is settled. If you are not having any symptoms or lesions on the skin, you are probably okay for now. Hope this answers your question."
},
{
"id": 124942,
"tgt": "Suggest remedy for painful bruise inside knee after fall",
"src": "Patient: I had a seizure over a week ago now. i fell hard on it. after i woke back up i could not walk. there is still a little bit of bruising on the inside of my right knee.but the pain is killing me when i walk bend it and even keeping me up all night. what is wrong? Doctor: Hi, You have probably sustained a fracture around the knee or damaged your ligaments on the medial side of the knee and inside of the knee. Use a Knee brace, painkillers and get X rays ASAP. Only after X-rays further course of treatment can be decided. Hope I have answered your query. Let me know if I can assist you further. Regards, Dr. Gopal Goel, Orthopaedic Surgeon"
},
{
"id": 10760,
"tgt": "Suggest treatment for hair loss",
"src": "Patient: i am suffering from hairloss for past 1year and worried of getting bald in near future.i consulted several dermatologists but not satisfied with results.Which doctor should i consult to treat my hair and scalp problems? Suggest me a good doctor inorder to get better results. Doctor: Hi,I do not know your gender and where you stay.You seem to have telogen effluvium. Kindly consult the dermatologist nearby and have proper diagnosis and treatment.I would suggest ...- treatment of underlying disease- biotin tablet- minoxidil solution 5%- herbal shampooHave long term treatment with positive attitude.I hope this would help you.Thanks.Dr.Ilyas Patel MD"
},
{
"id": 75987,
"tgt": "Suggest treatment for abnormal breathing pattern",
"src": "Patient: well my Mom having unusual breathing pattern like been alternated by a short period of apnea then goes back to normal but slow breathing ... besides that makes her to wake up in middle of her sleep with dryness of upper airways .. thus was that what to be called Cheyne stoke ??? and if so what are your recommendations? Doctor: Thanks for your question on Healthcare Magic. I can understand your concern. They way you are describing your mother's breathing, possibility of chyne strokes breathing is more. This is known as central breathing. It is commonly seen with brain diseases and sleep apnea. So your mom there should definitely consult sleep specialist first and get done detailed sleep study. She will also need detailed neurological examination and MRI brain to rule out brain related diseases. If she is obese then she has too loose weight and stay fit and healthy. CPAP (continuous positive Airway pressure) machine will cure this breathing if it is due to sleep apnea. So first diagnose her and them start appropriate treatment. Hope I have solved your query. I will be happy to help you further. Wishing good health to your mother. Thanks."
},
{
"id": 178418,
"tgt": "What causes swollen ankle and pain from foot to knee in a child?",
"src": "Patient: My 10 year old. Has sudden onset of ankle swelling, with pain shot ting into his knees . Sometimes they are so swollen that they are not to touch. No injury has occurd. This is has been going on for about 3 weeks now. They hurt to touch and he complains of shooting strapping pain from foot to knee. Doctor: hi...your history pointing towards following probable diagnosis1. Acute rheumatic fever - most likely2. rheumatoid arthritisconsult your near by pediatrician for further workup. he needs some blood investigations.if you are happy with my suggestion kindly rate me.regards - Dr.Surendra.H.S"
},
{
"id": 10757,
"tgt": "Suggest treatments for hairfall",
"src": "Patient: Hi doctor,I had a severe hairfall bfore 6 months,so i had Xtraglo[Biotin,L-Methionine,L-Cysteine tablets] per nite for 3 months as suggested by a dermatologist,hairfall s almost controlled,so i stopped having tablets,bt nw im facing again,shall i start taking tablets or give any other suggestion plse.. Doctor: Hi,It may be telogen effluvium.Kindly consult the dermatologist for the perfect diagnosis and proper treatment.After thorough examination and investigations,he might give proper treatment. Blood tests for thyroid function,vitamin B12,Hb..etc may be done.You may restart biotin containing capsules along with vitamin E, which may be continued for long time. Apply mild steroid lotion on the scalp. Cleanse the scalp with mild herbal shampoo. Oiling the hairs with almond oil might improve the texture of hair shaft.I hope this would help.Thanks.Dr.Ilyas Patel MD"
},
{
"id": 82286,
"tgt": "How to improve the xiphoid process?",
"src": "Patient: My xiphoid process is painful, its more painful when I touch the center of my xiphoid process and move it left to right, Are you supposed to feel your xiphoid process? Like a straight bone or lump it feels like but its very tender/painful to touch almost feels like its inflamed, I have had this pain for 1/2 weeks and it comes and goes but tonight I have had this pain for nearly 2 hours. I am asthmatic and I also take omeprazole once a day.Thanks Doctor: Thanks for your question on HCM.Xiphoid process is superficial only in all individuals. And no need to move it forcefully.Its function is not like this. Its main function is to provide attachments to the lower ribs and intercostal muscles. I also gives attachments to abdominal muscles.So you may have inflammation in xiphoid end of muscles. Better to avoid forceful movement of xiphoid.Take good painkillers and muscle relaxant. Avoid painful movement of ribs and abdomen and avoid bad postures in sleep.Apply warm water pad to the affected area."
},
{
"id": 161975,
"tgt": "What causes thickened bladder wall and migraine with history of celiac disease?",
"src": "Patient: My daughter has many issues with several doctors following her. I am wondering if the issues could all be related to one issue? She was diagnosed with Celiac disease not by blood test but because of fat malabsorption. She also has overactive bladder with a thickened bladder wall and she taking Ditropan. She has recently begun with what seems to be migraine headaches and possibly a few episodes of abdominal migraines. She was hospitalized for 2 days and other things were ruled out. Most importantly she is 7 1/2 and only weighs 34 pounds even though she has been strictly gluten free for 3 years. Doctor: Hi, By what you say I feel that this might be voiding dysfunction of bladder bowel dysfunction. She needs bladder training exercises. Hope I have answered your query. Let me know if I can assist you further. Take care Regards, Dr Sumanth Amperayani, Pediatrician, Pulmonology"
},
{
"id": 16667,
"tgt": "What causes dizziness in a quadriplegic male with a pacer?",
"src": "Patient: Husband is a quadripelegic, and he also has a pacer, Lately he has been experiencing some dizziness after I transfer him into his bed on the way up to a sitting up position. Today he was having a real hard time going up in the bed, his head; and it took about 3 x s before he was ok. He would not let me take his blood pressure. Cheri Doctor: Hi, This is possibly postural hypotension, you should make him check his blood pressure, its the main cause of this condition. Hope I have answered your query. Let me know if I can assist you further. Regards, Dr. Salah Saad Shoman, Internal Medicine Specialist"
},
{
"id": 96966,
"tgt": "Why my son is having fever and stomach problems post mountain bike crash?",
"src": "Patient: my son crashed today on his mountain bike and it appears that he only has bad road rash on his back, shoulder, elbow and leg but he now is warm and feels sick to his stomach. We have washed out the wounds and will dress them tonight but the fever and bad stomach is concerning us. He doesn t think he hit his head but is not sure. Doctor: A schematic examination is important always to rule out blunt injury abdomen.Fall from bike by complicated mechanisms always requires attention and an emergency ultrasound abdomen and pelvis is essential to rule out blunt injury abdomen which might explain his pain abdomen.If results are inconclusive on ultrasound a contrast ct abdomen is required.Severe pain might activate sympathetic response which could be the cause of fever. So treat pain symptoms will subside. Tepid sponging can be done sos"
},
{
"id": 202935,
"tgt": "What can cause pain in thighs,legs,testis,penis?",
"src": "Patient: Hi Sir, I am masterbating from past 4 years.I am getting pain after ejaculation in thighs,legs,testis,penis and i am unable to walk much distance.I am getting this pain continously.Is there any medicines or tonics that will increase power in my thighs,legs and testis.Will this cause effect in future.Kindly adviseThanks Doctor: Thank you for your question.I understand your concern. The symptoms you are experiencing after masturbation are quite common in most of the people.Immediately after ejaculation, all the muscles get relaxed and give the symptom of weakness. Taking 15 mins rest after masturbation is quite enough remedy for you.If he weakness is too severe, you should have to take healthy food like leafy vegetables and try using Tab B complex once daily.Hope the answer helps. Thank you."
},
{
"id": 16478,
"tgt": "What is the cause for bruise or skin discoloration on my chin ?",
"src": "Patient: i noticed dime sized dark skin discoloration on my chin after a shower today. it tapers off on one side Im not sure if it was there when i woke up. What might of caused this and should i be worried? could it be ringworm? It is raised a little bit from the surrounding skin and is sensitive to the touch but does not hurt. I think it might just be a bruise but i dont know. Doctor: Hi, Thanks for query, As it appeared all of a sudden ringworm can not be there. You might have something bite causing locallized allergy, It may be bruise due to some scratch mark. Take antihistamine and apply moisturizing cream. Ok and bye."
},
{
"id": 56871,
"tgt": "Treatment for mild hepatomegaly and minimal ascites",
"src": "Patient: HI THANKS VERY MUCH YES I AM TYPING MY DAUGHTER 14Y, FOR LAST 30 DAYS WITH INTERMITTENT FEVER GOING UPTO 102 F. SEEN DOCTORS TAKEN ANTI BIOTICS, RULED OUT tb BY SKIN TEST. USED 3 DAYS MALERIA COUSRE, IN CAS IF IT IS MALERIA.ON 14TH AGAIN UT SCAN X-RAY TAKEN. IN X-RAY THEY REPORT AS \" PROMINENT LEFT HILUM NODAL PATHOLOGY\" IN SCAN - MILD HEPATOMEGALY & MINIMAL ASCITIES KINDLY ADVISE Doctor: No need to worry about the mild hepatomegaly and asites..these came becuse of sum infection in body..no need to use drugs for this..u hav to think 4r hilar pathology..it maybe tb..go for sputum test for tb..its imp to rule out tb..skin test wont help much..hope i am helpful..hoilar pathology may be sum other diseases of lung aldo..1st rule out tb"
},
{
"id": 102941,
"tgt": "Diagnosed for asthma. Taking seritide, windel plus and budicord along with Courtson. Long term effects of steroids?",
"src": "Patient: hello doctor,my father is 82 years old.last 15 years he diagnosed for asthma.he was a smoker.he did non smoke last 30 years.now he take seritide inhaler 2 puff daily,onbreeze and nebulized with windel plus and budicord,frulac 40mg tablet daily. but this medicine is not working.daily he is attack by asthma.but if he take tablet cortson 10mg (steroid) he become get better.now every day he take cortson.isn't it harmfull for his health?how many days he continue this steroid? Doctor: it is harmful you cannot continue itt you can change inhalers or increase the dose plus antiallergic medicinesi advice to consult again for retitration of dosages rather than taking cortisone"
},
{
"id": 122389,
"tgt": "How to treat mobile growth on inner thigh?",
"src": "Patient: I just got a sudden mobile growth about the size of a golf ball withing the last two days on my inner thigh...there is no specific pain associated with the growth. Yesterday when I noticed the growth by mid day it appeared to have dissipated i felt some type of muscle spasm had occured. Today it reappeared somewhat larger and somewhat throbing... What are your views? Doctor: Hello, It may be lipoma. For further assessment you may require ultrasound after surgeon consultation. As it is asymptomatic, no need to worry. If it is problematic or big in size then you may require surgical intervention after surgeon consultation. Keep your self hydrated and take balanced diet. Hope I have answered your query. Let me know if I can assist you further. Regards, Dr Shyam Kale, Family and general physician"
},
{
"id": 20894,
"tgt": "What causes blood in urine and irregular heart beat after knee replacement surgery?",
"src": "Patient: My Dad had a knee replacement a week ago, since then is has had blood in the urine, urinating on himself, he cannot controll it, and now has an inirregular heart beat. Today his nurse came to the house and told him he has a temp. Mom has not called his surgeron and I am concerned. Doctor: Your father may have a urinary tract infection = UTI especially since he has a fever. This infection may cause a problem with his heart. He would need an antibiotic for this - so you should call his doctor.Also your father may be on blood thinners - pills or injections after the knee replacement - this also may cause blood in the urine. (but would not cause him urinating on himself). His doctor needs to know about this as well."
},
{
"id": 131024,
"tgt": "What does supraspinatus calcific tendinitis mean in MRI of shoulder?",
"src": "Patient: I had a MRI of the left shoulder and I received the following impression: supraspinatus calcific tendinitis. Noted are several axillary lymph nodes which are subcentimeter in short axis dimension. Are the lymph nodes a result of the tendinitis? My GYN says it has nothing to do with my beast and the Orthopedic refer me to the GYN. I m in a lot of pain and don t know whether to worry about the lymph nodes. Thx Doctor: Hii read your problem,Calcific tendinitis means a disorder characterised by deposition of hydroxyapatite or crystalline calcium phosphate on any tendon of the body,but most commonly in the tendons of the rotator cuff(shoulder) causing pain nd inflammation.the condition is related to and may cause adhesive capsulitis or frozen shoulder."
},
{
"id": 219456,
"tgt": "What could heartburn with sickness post pregnancy suggest?",
"src": "Patient: hi i have a 4 year old child and ever since i fell pregnant with him i had really bad sickness but it carried on after having him i still felt sick and i used to heave at things but then its started to get worse the last two months i carnt keep things down get heartburn i have done a pregnancy test but it says im not pregnant i went the doctors and they said it was a virus but i had tablets and they have done nothing thanks Doctor: It will be better that you consult a gastrienterologist for your symptoms. He or she should be able to answer your questions and will be able to provide treatment ."
},
{
"id": 143942,
"tgt": "Suggest an alternative medication for back pain apart from Cortisone shot",
"src": "Patient: I have been diagnosed with spinal stenosis and L5-S1 Spondolithesis after having an MRI of Spine. Dr. wants to preform a series of Epidural Injections. He said there was a 50% chance of improvement in pain as best result and worse case scenario could be paralysis or death! I don t feel comfortable with these odds. Of note I did have a Cortisone shot in my arm for a different injury and I felt very sick and nausea after for about a month. My arm did not improve. What other recommendations might you have for treating my back injury without injection? Thank you and I await your answer! Sincerely, Eva Doctor: Hello and thanks for using HCM.I have read your question and understand your concerns.You can try physical therapy ( traction lumbosacral exercises ) and NSAID drugs that also may improve your pain.Epidural injections may have some risks, but death it is a little exaggerated I think.However, definitive treatment about spondylolysthesis is spine stabilization surgery.I think you should get evaluated by a Neurosurgeon or a spine Surgeon in order to get a correct understanding of your condition and treatment.Hope you found the answer helpful.Greetings."
},
{
"id": 112348,
"tgt": "Lower back pain, leg pain. Taking Tylenol and Advil. Used heat and ice. Suggestions?",
"src": "Patient: My husband is really suffering with severe lower back pain and pain radiating down the front of his leg..he has been taking tylenol 3 s for a week with Advil q8 hours....what would be better? ...he has a prescription for Celebrex but stopped taking it when he started the Advil......he has tried heat...ice....tried doing the back excercises but he is no better. He is never sick and never misses a day of work but this week he missed 4 days. Please , any suggestions would be appreciated. Doctor: Hi,From history it appears that your husbund might be having some degenerative changes in his lower spines giving rise to this problem.Go for x-ray lumbo-sacral region.Consult orthopedic surgeon and get examined.Physiotherapy will give better relief.Do back extension exercise.Ok and take care."
},
{
"id": 126358,
"tgt": "What causes persistent right upper quadrant pain?",
"src": "Patient: Constant RUQ pain, right under rib cage. Cramp-like or burning pain. Most noticable when sitting, standing. Relief from direct pressure on the area. No pain when lying in bed. No other symptoms. CBC came back normal. Male, 50 s, obese otherwise healthy. Thanks for your input. Doctor: Hello, It could be due to conditions like costochondritis. As a first line management you can take analgesics like Acetaminophen or Diclofenac for pain relief. If symptoms persist you can consult a physician and get evaluated. Hope I have answered your query. Let me know if I can assist you further. Regards, Dr. Shinas Hussain, General & Family Physician"
},
{
"id": 3319,
"tgt": "Should i take contraceptive pill to prevent pregnancy?",
"src": "Patient: I am 19 years old and I indulged in unprotected sex about 53 hours back. The guy pulled out before ejaculating but we are still unsure of the circumstances. I am a case of PCOD which is why menstruation cycle keeps changing and I have been taking medication for that since a year and I have also been taking medication for my skin allergies regularly,homeopathic and allopathic. Is there any way I can know for sure whether I m pregnant or not,also is it advisable for me to take a contraceptive pill like I Pill before 72 hours are completed ? Doctor: Hi there,My opinion after reading your question is that:1) After having unprotected intercourse, it is better to take an I-pill within 72 hours. This is to be done even if there was no ejaculation because pre-ejaculate also contains sperms.2) I do not know what treatment you are taking for PCOD, but if you are taking oral contraceptive pills like Yasmin or Yamini or Mala D or Duoluton-L, then you do not need to worry about taking I-pill. This can be determined by how you are taking the medicines. If you are taking 3 weeks of medicine followed by one week break, then they are most probably oral contraceptive pills.3) There is no way right now to find out if you are pregnant so soon, you will have to wait atleast for two weeks and then get a blood test called serum beta HCG. If you miss periods, you can get a urine pregnancy test done.4) So to sum it all, I think you should take an I-pill as soon as possible. Then you can wait for periods. If you get them, nothing to worry. If you do not, then get a urine pregnancy test done.I hope this was helpful.Regards,Dr. Shikha Shah"
},
{
"id": 18411,
"tgt": "Suggest treatment for cough and fluttering sensation in the throat",
"src": "Patient: I have had high blood pressure in the past - I take no medication at all. I have a cough caused by a flutter in my throat, feels like my heart is fluttery, wake up in the middle of the night with anxiety, have some trouble swallowing at times and don t have much of an appetite. I also have vitilago Doctor: Hello and Welcome to \u2018Ask A Doctor\u2019 service. I have reviewed your query and here is my advice. I would explain that your symptoms could be related to possible cardiac arrhythmia. A metabolic disorder (thyroid gland dysfunction, chronic anemia, etc.) or a lung disorder can not be excluded either. Coming to this point, I would recommend consulting with your attending physician for a physical exam and some tests: -a resting ECG and a cardiac ultrasound- a chest X ray study and pulmonary function tests- blood electrolytes- complete blood count, PCR, ESR for inflammation- thyroid hormone levels for thyroid gland dysfunction. Hope I have answered your query. Let me know if I can assist you further."
},
{
"id": 145346,
"tgt": "Could severe headache and neck pain be due to spondylotic spur disc change?",
"src": "Patient: I have had a severe head ach since last sunday with neck pain and fever of 1100 to 100.7 and I start taken it down ice and cold drinks. Was told I have anterior spondylotic spur plus disc change, mild intervertabral disc space loss in height at C4-C5 and C5-C6 levels.. Straightening of lordosis, degenitive disc disease, And also had an interbody fusin on L4-L5 and S1 5 years ago. should I go to hospital ? Doctor: Hello ! Thank you for the question ! I understand your concern. The symptoms you have may be related to meningitis . I would like to know if you have nausea, vomiting or any other symptoms . The cervical spine degenerations do not cause fever. Another possible diagnosis would be a discitis in the cervical spine ( and infection of the disc), but headache is not typical in this situation . You need more than cold drinks to pass through this situation. I would recommend you to go to the hospital and have a full blood work and a neurological examination. You may need also a brain CT scan and a lumbar puncture to rule out meningitis. Hope to have been helpful. Thank you for using HCM!Best wishes , Dr. Abaz Quka"
},
{
"id": 188385,
"tgt": "Movable lump beneath central bottom teeth. What is it?",
"src": "Patient: Hi, i am an 18 year old. I have noticed a lump beneath the skin of the floor of mouth right beneath the central bottom teeth. It seems like around 0.5cm and it is completely movable. I seem to have another on the right side but it seems smaller. It has been there for quite long i think, probably more than half a year but in still worried! Doctor: Hello,Thanks for writing to us.Movable lump may have arised due to-periodontal infection or deep caries of associated tooth.I would advice you to get a thorough clinical evaluation done.Pus if present has to be drained and curretted.Meanwhile,maintain oral hygiene well.Antibiotics along with analgesics has to be administered.Take care."
},
{
"id": 92417,
"tgt": "What is the cure for recurring abdominal pain and acidity?",
"src": "Patient: Hi! my mother is suffering from upper abdomenal pain and acidity for 5 months. I have done blood test, barrium sellow x-ray, sonography, urinal tests etc. reports are normal. But the pain, acidity are not gone. Now the chest also being pain. Doctors had sugestioned to take rabaprozal, domperidone, librax tab, ulgel syp, happi-d tabs, pentakind, and other tabs . But results are Zero. please advice for her getting well soon. Doctor: Hicontinue with drug but you go with blunt drug for some time .Try to change your food take careDr Lal Psychiatrist"
},
{
"id": 49701,
"tgt": "On omeprazole, ranitidine, pepto bismol. On dialysis. Lost kidney to cancer. Worry?",
"src": "Patient: my stomach has been bothering me lately my primary DR. has me on omeprazole 20mg Iv I have been taking that for years. I seen Him yesterday And he Told me to take 300mgs. of Ranitidine at bed time and pepto bismol 15 mins. before meals.I am on dialysis 3 times a week And lost a kidney to cancer last year do i have anything to wory about with thenew meds. Doctor: Hi. You are suffering from acid peptic disease. As your doctor has suggested you can take those medications. Since you are already on dialysis and your kidneys have been removed, these drugs will be removed from the body during dialysis. These medications are quite safe. Omeprazole, rantac, bismuth are all excreted principally by kidney s and now dialysis will do that job. They are not harmful to liver. However consult your nephrologist."
},
{
"id": 221473,
"tgt": "What could flakes in urine in pregnant woman suggest?",
"src": "Patient: I am 39 weeks pregnant, and here recently I ve noticed skin like flakes in my urine. I know that I ve contracted 2 UTIs & 1 Bladder infection. Also my son s on my cervix. My doctor hasn t checked to see if I was dilated or thinning.. What should I do? Doctor: HiDr. Purushottam welcomes you to HCM virtual clinic!Thanks for consulting at my virtual clinic. I have carefully gone through your case, and I think I have understood your concern. I will try to address your medical concerns and would suggest you the best of the available treatment options.With your history of UTI, the flakes can be shedded cells of the lining of urinary track.I will suggest to have water intake of 8 to 10 glasses per day.Include citrus fruits like sweet lime, orange , lime water in daily intake.I will suggest to have check on your blood pressure and get examined by doctor to see the changes if you are entering labor.May God bless you with Bundle of JOY.I hope my answer helps you.Thanks.Wish you great health."
},
{
"id": 130385,
"tgt": "What causes numbness and pain in both arms caused during car wreck?",
"src": "Patient: hi, Im 44 I was in a bad car wreak broke my neck in 3 places crush c1 broke 4&5 also 3rd degree dislocation of my right shoulder I,ve been in a lot pain had numbness both arms at times can be very painful . I had my neck fused,helped a little with numbness but a 2-3 times a wk same thing been 2 diff. nur. surg. they tell me things are healed and seems just to brush me away! im done sorry Doctor: Hi i am Dr Ahmed Aly thanks for using healthcaremagic site ,I had gone through your question and understand your concerns .. In my opinion spinal injuries are very dangerous and any kind of nervous or neurovascular bundle compression or herniation takes alot of time to heal i t may take from 6 to 12 months . i dont wont you to worry if your nur.surgeons say that every thing is healed then its only a matter of time and patience with some hot massages , physiotherapy , muscle relaxants , painkillers , even yoga and i am sure you will get better such injuries in cervical spines may be fatal so you passed it i think you will pass your healing time time to .Please click THANK YOU and consider a 5 star rating with some positive feedback if the information was helpful. Hope the above information helps you,Any further clarifications feel free to ask."
},
{
"id": 89234,
"tgt": "Is Cerazette causing abdominal pain?",
"src": "Patient: Hi Dr,Im 32 & I've had 2 boys.....me & my partner really don't won't anymore children....due to SPD throughout last pregnancy.....so ive been taking Cerazette for over month now, what I have noticed is Everytime I have take one I get really bad stomach cramps.....is this going to be a on going pain??? Or shall I go & see my dr so I can get something different again??Thx Marcy-Lou Doctor: Dear Ms Lou,Cerazette sometimes causes lower abdominal pain.and it almost recurrs everytime you start taking this.to get relief you must switch to some different stuff.in that case a doctors advice is always helpful.because shifting to different stuff helps,provided you have a provision for regular follow-up by a clinician.also you can take help of barrier method like male/female condom."
},
{
"id": 54526,
"tgt": "Suggest methods to maintain normal ALT levels",
"src": "Patient: dear doctor,my name is Jandill billones 26 years of age, 5'7 in height 78kls. i have a problem in ALT. what should i do to lower my ALT count. My last ALT result is 190 IU/L then a doctor give me a medicine to take for 3 times a day for 1 week. URSOFALK ursodeoxycholic acid. after one week i reapet the test thin the result is higher from 190 to 140 iU/L. i was worried because the doctor said to me to continue the medicine ursofalk for 2 weeks. please help me doc?salamat Doctor: Hi thanks for contacting HCM..Noted you have elevated alt.So first find out cause for its elevation.When hepatocytes damaged, enzyme liberated In blood and so elevated level....You have prescribed ursodeoxycholic acid ..It will help in dissolving cholesterol in bile.So you might have fatty liver or gall stone.USG , liver enzymes study, viral marker study will be helpful to investigate ......For improvemet take low fat diet....Refined food avoided.Fruits taken more like papaiya , apple etc.Green leafy salad more.Avoid alcohol..Non veg , junk foods avoided.If obese loose weight by exercise...If you have hepatitis then one tsp licorice with hobey can be taken.....Take care....Follow up to gastroenterologist after completing urso deoxycholic acid for 2 week.Dr.Parth"
},
{
"id": 114765,
"tgt": "What is the treatment for sjogren s syndrome and blood clotting disorder?",
"src": "Patient: I had my right leg amputated 5 weeks ago I have Lupus, Sjogren s Syndrome, and a blood clotting disorder for which I take Coumadin. Tonight just noticed the right side of the amputated leg is swelled up, just doesn t feel right, it is very hard, feels warm on the skin but feels like it may be cold inside. Doctor: Hi, dearI have gone through your question. I can understand your concern.You have sjogren syndrome and lupus. So you take blood thinner like warferin and maintain your INR between two to three. Steroids are also useful in sjogren syndrome but you should consult your doctor and take on prescription. Hope I have answered your question, if you have any doubts then contact me at bit.ly/Drsanghvihardik, I will be happy to answer you.Thanks for using health care magic.Wish you a very good health."
},
{
"id": 108157,
"tgt": "Suggest treatment for severe neck and back pain",
"src": "Patient: I had shoulder surgery and received a nerve block to help with the pain directly after the surgery. there were some complications during administration of the block. Now I am having severe pain in my neck, upper and middle back. The surgery was 2 days ago, What is or could be wrong? Doctor: Hello, I have studied your case.There can be nerve irritation due to post surgery injury.I t can be due to compression of nerve root or entrapment of nerve.You can take vitamin B12 for reducing symptoms.Vitamin B12 is important in the maintenance of health and normal functioning of the nerve tissue and deficiency of it can lead to numbness. You can increase B12 level by taking meat and fish, or you can take Inj methylcobalamine 1500 mcg If not relieved then I will advise you to MRI spine and EMG/NCV [nerve conduction study for better diagnosis.For these symptoms neurotropic [Pregabalin] medication can be started consulting your doctor.I will advise to check your vit B12 and vit D3 level.I think there is not much to worry about this.Hope this answers your query. If you have additional questions or follow up queries then please do not hesitate in writing to us. I will be happy to answer your queries. Wishing you good health.Take care."
},
{
"id": 69584,
"tgt": "Suggest treatment for lump under right eye while having thyroid problem",
"src": "Patient: I have a lump under my right eye. I have thyroid problem since the past 15 years. In the beginning I thought it must be some water retention because of thyroid. Over the years it has become very prominent and now it looks like a lump and I feel very conscious about, especially when I smile. I take elthroxin 50mg for my thyroid problem. Is there any solution for this problem. Doctor: Hi,it seems that you might be having sebaceous cyst under the eye.If lump is under the eye lid, it can be fibrozed infected stye.consult your doctor and get examined.Ok and take care."
},
{
"id": 218922,
"tgt": "What does this pelvic ultrasound test result during pregnancy indicate?",
"src": "Patient: Hello doctor.Im sangeetha kotha of 33 yr oldand have 5 yrs old girl baby and had 2 miscarriages in nov 2015 and june 2016 at around 10 and 11 weeks.... now im carring .. its 11wk and had Nt scan done which shows the thickness of 3 to 5mm... can u pls tell me the risk of abnormalities... Doctor: nt scan of less Dan 3.5 mm is normal..nt scan is a screening test for down syndrome...u can combine nt scan with blood test called combined test ....increased Bhcg and decrease pappa is seen in downs syndrome"
},
{
"id": 68572,
"tgt": "Suggest remedy for the lump on the shin with redness around it",
"src": "Patient: My brother in law hit his shin on a concrete step. A huge knot popped up. We put ice on it and it was down the next day. There is a sore about the size of a quarter where he hit it. We have treated the sore with medi-honey, and there is no fever. However the entire shin from the site about 12 inches down is red. No fever. He is diabetic with congestive heart failure. Doctor: Welcome to Health care magic.1.The symptoms seems like an infective features.2.With underlying medical conditions like diabetes and cardiac pathology, i would recommend to see the GP and let it get examined. 3.As in diabetics possibilities of infection are more.4.Muscle relaxant, and a course of antibiotic is what he needs according to me.5.But one needs to examine the area only then come to conclusion.Hope it helps you. Wish you a good health.Anything to ask ? do not hesitate. Thank you."
},
{
"id": 225845,
"tgt": "On birth control. Can antibiotic eye drops lower effectiveness of pill?",
"src": "Patient: Hi,I am currently on birth control but I \"feel\" pregnant. I have had 3 pregnancies before with one child currently so I am familiar with the feeling. My question is if I was taking antibiotic eye drops, could this have lowered the effectiveness of the pill at all? Also I am pretty good about taking it the same time every day but may be about 4 hours off sometimes. Thanks! Doctor: Hi, Thanks for using HCM.Antibiotic eye drops wont affect your birth control but few antibiotics when taken orally will affect. Since you are having symptoms this may be due to other reason. Get done one UPT done if you miss your normal period. Otherwise dont worry. There are also some food and other drugs that can affect effectiveness of pills. Consult your doctor for examination once to rule out pregnancy and management.Hope I answered your question. Feel free to ask me if you have any further queries. Wish you good health. Take care.RegardsDr. Lohit"
},
{
"id": 28042,
"tgt": "Experiencing trouble in breathing, going up stairs and have heart palpitations. What kind of a doctor do I see?",
"src": "Patient: I had surgery on my carodid artery in 86. . I am having trouble with my breathilng going up stairs trouble talking,, heart palpitations, Dr at St Josephs Toronto found that I have a bow in my vocal cord and after the surgery in 86 my husband was told that I may not be able to talk when I woke up. I did have trouble talking at the time and my voice has been raspy off and on. Now I have a very low paulse and have a lot of trouble with my voice. What kind of a doctor do I see with this problem Sheila Doctor: Get done 2D echo to check if ur heart is pumping fine. And for voice, visit ENT surgeon. And if any further queries, DONT hesitate to revert back."
},
{
"id": 149969,
"tgt": "Long term chronic Alcoholic, suffers Dementia, severe panic disorder. Does PET scan show dementia in brain ?",
"src": "Patient: My husband is a long term chronic Alcoholic 30 yrs. and I feel very strong that he has some sort of dementia and can not get any of his Dr.'s to be on the same page.I try to give the Dr.' straight facts to his behaviors along with the non logical thinking but he denies all when we get to the hosp or Dr. Office. He is also suffering from severe panic disorder, He is in and out of the Hosp. with the same complaints. I know this panic disorder makes you ill physicaly as well. Would a Pet scan show dementia in the brain? Doctor: Hello,Alcohol dementia and Panic disorders may be due to Excessive drinking over a period of years. The process of acquiring knowledge and understanding through thought, experience, and the senses (cognitive skills), memory and learning are affected because of this excessive alcohol.Positron emission tomography (PET) is useful in certain diffuse brain diseases such as those causing various types of dementias including Alcohol dementiaWith best wishes"
},
{
"id": 108099,
"tgt": "What causes fever,back pain and green bowels?",
"src": "Patient: I AM 43 years old 5 4 and weight s 240 . I have periperal neuropathy x 10 years and rheumatoid arthritis x2 years. I have not done anything different in the past few days but I have had a temp of 99.9 or below for 3 days with back pain. I am now having florescent green bowel movements. I hurt under my ribs and back. I have been a nurse for 20 years , this is a new green. I am now on disabilty due to neuropathy. Doctor: Dear Sir/MadamI have gone through your query and read your symptoms.In my opinion, your bowel color is indicative of billiary pathology, the gall bladder would either be filled with stones or there could be any growth in the bladder. I feel you should discuss your concerns with your doctor and together get out with a solution to your problems.I hope that answers your query. If you want any more clarification, contact me back."
},
{
"id": 173768,
"tgt": "Any suggestion for child suffering from trichotillomania?",
"src": "Patient: My daughter is 6 years old n has been twirling n pulling her hair n has broken lot many hair . The doc recently told me it s trichotillomania. How do I help her with this. She is loved so much n well taken care of has no scared feeling of any person. Behaves normal n laughs n plays well. Pls help Doctor: Hi,Thank you for asking question on health care magic.Trichtillomania is seen in children with psychotic disorders.Better you consult psychiatrist.Clomipramine,Fluexetel and Acetyle cystiene are the medicines tried with some success.If the problem starts before your baby is 5 years old,it is self limited and no treatment is necessary.Hope this answer will serve your purposePlease feel free to ask any more queries if requiredTake careDr.M.V.Subrahmanyam MD;DCHAssociate professor of pediatrics"
},
{
"id": 125805,
"tgt": "What causes pain in the hips and left leg after a C-section delivery?",
"src": "Patient: Hi!. I had a c section delivery. Now it s 4 month 20day has finished.I have pain in my hip to whole left side leg. I can t put pressure on my left leg.Sometimes I can t walk and I can t stand quickly. Even I sneezing or cough I can t bare the pain what is the reason of this pain and what should I do? Doctor: Hi, By your history and description, the possibility of sciatica is more likely. It is an inflammation of a sciatic nerve. You might be having left sided sciatica which is causing all the symptoms. It is common after pelvic surgeries like caesarean section. Physiotherapy in the form of stretching, ultrasound, exercise and warm water pad application is beneficial. So consult a physiotherapist and discuss all these. Hope I have answered your query. Let me know if I can assist you further. Regards, Dr. Kaushal Bhavsar, Pulmonologist"
},
{
"id": 150410,
"tgt": "Has seizure, experiencing nose bleeds frequently. What could this be?",
"src": "Patient: Daughter is 13 years old and about 6 months ago we returned from a holiday in Spain, all of us came home with a bug but my daughter appeared fine until she had her first absence seizure a week later. She had a few more with a number of weeks/months. She has been on anti-seizure medication but recently had another bad seizure and took 20 minutes to come around. she has now experienced a number of nose bleeds over the past few days - what could this be Doctor: Hi, Thank you for posting your query. Seizures would get well controlled with anti-epileptic medications, if used at a good dose and on regular basis. If she is on sodium valproate, then, the nose bleeds may be linked, as this drug can cause thrombocytopenia (low platelet count) resulting in increased chances of bleeding. Otherwise, her nose bleeds may not be connected to seizures. Please get back if you require any additional information. Best wishes, Dr Sudhir Kumar MD (Internal Medicine), DM (Neurology) Senior Consultant Neurologist Apollo Hospitals, Hyderabad, My personal URL on this website: http://bit.ly/Dr-Sudhir-kumar My email: drsudhirkumar@yahoo.com"
},
{
"id": 114503,
"tgt": "How can severe anemia be treated?",
"src": "Patient: Hi, I am g6pd deficient and I didn t know ascobic acid was a red flag for me. I was taking ascobic acid 1000mg once a day and a few days down the line I started experiencing severe anaemia. I am also currently in Africa and I am still not feeling so strong after taking iron pills 2 times daily and one a day for women tablets daily. Though it s better than before but I still have the symptoms. What should I do? I have already gone to the Hospital and all labs prove that my haemoglobin level is not so bad. Doctor: HI. I am glad that we connected. Ascorbic acid is also known as vitamin C. It is not a complete contraindication for anemia or G6PD deficiency. It is possible that one may develop some symptoms because of the oxidative stress on the body. Usually, G6PD deficiency is treated by avoiding medications that can cause increased oxidative stress. The rest of the treatment is managed by maintaing the iron levels, reducing exercise that can cause low oxygen levels, and avoiding excessive stress.Some doctors find using folic acid useful as well. Some cases may require blood transfusion.I hope this works out for you! All the best."
},
{
"id": 67632,
"tgt": "What is the lump underarm and tingling sensation in the arm?",
"src": "Patient: I have a lump in my arm pit and my arm has a tingling feeling almost like it is going to sleep. I need to know what kind of doctor to see. i have brought this up with my general doctor but they say it is nothing. if it is nothing then why is my arm tingling. it seems to be getting worse. Doctor: Hi ! Good evening. I am Dr Shareef answering your query.Whatever the lump might be from, the tingling sensation / numbness in your arm could be due to its pressure on the brachial plexus (nerves of arm and hand). If I were your family physician, I would refer you to a general surgeon, who after a local and general examination might consider a FNAC (fine needle aspiration cytology) to arrive at the origin of the lesion.I hope this information would help you in discussing with your family physician/treating doctor in further management of your problem. Please do not hesitate to ask in case of any further doubts.Thanks for choosing health care magic to clear doubts on your health problems. I wish you an early recovery. Dr Shareef."
},
{
"id": 150426,
"tgt": "Had 8 back surgeries, dorsal spinal implant, taking Fentanyl, Norco for pain control. Any better medicines ?",
"src": "Patient: Hello, I have had 8 back surgeries, (3 laminectomies with fusions or should I say non-union fusions. I have a dorsal spinal implant I am on my second set of steffie plates. I have been on Fentanyl 100mcg/hr q 48 hrs and I also tale 5 Norco 10/325 1 tab 5 times a day as needed for breakthrough pain. Do you think I would have better control with a pain pump. I have gain a tremendous amount of weight due to continuing pain and inability to exercise. I still have not gotten my pain under control and my current Dr. Says I am just about maxed out in the amounts of pain meds I am taking? Thank you. Doctor: Hi, Thank you for posting your query. It is unfortunate that you have not had significant pain relief despite multiple surgeries and high dose medications. You would benefit from a pain pump. Please get back if you require any additional information. Best wishes, Dr Sudhir Kumar MD (Internal Medicine), DM (Neurology) Senior Consultant Neurologist Apollo Hospitals, Hyderabad, My personal URL on this website: http://bit.ly/Dr-Sudhir-kumar My email: drsudhirkumar@yahoo.com"
},
{
"id": 68694,
"tgt": "Suggest remedies for recurrent painful lump in the armpit",
"src": "Patient: my 13 yr old daughter has a lump in her armpit. It seems to be the size of a fingerprint and she says it hurts. I am not interested in going to my pediatrician. What type of doctor can I take her to. She says this has come and gone before a few times. But this is the first I've heard of it. Doctor: welcome to Health care magic.1.If you are not happy to see paediatrician then most suitable doctor will be surgeon.2.It seems like infective aetiology (bacterial).3.Generally depending upon the size the treatment depends upon - starts from antibiotic treatment to incision and drainage.Hope it helps you. Wish you a good health.Anything to ask ? do not hesitate. Thank you."
},
{
"id": 150144,
"tgt": "Suffer from back pain. MRI report showing lumbar spondylosis, annular disc bulge and disc herniation. Explain?",
"src": "Patient: Hi Doctor, I got an MRI report last week because of my Back pain. They have mentioned Impression as 1. Early Lumbar Spondylosis 2. Posterior Annular Disc Bulge at L4/L5 Impinging upon Bilateral Neural Foramina. No Compression of Exiting Nerve Roots. 3. Left Paracentral and Foraminal Disc Herniation at L5/S1 causing Left Foraminal Stenosis with Compression of Exiting Nerve Root. Please guide Doctor: hallo dear friend ,thanks for writing to HCMi have studied your casewhat is your age? this is you first episode or repeatedly having same problem is there any weakness in your lower limb or upper limbyou need methylcobalamine , muscle relaxant with analgesictake rest , , hot or cold fomentation avoid lifting weights sit with taking support to your back continue spine extension exersizes physiotherapy - SWD ,IFT WILL HELP if pain doesnt relieve then you need spinal decompression surgery for lumbar vertebrae ,for further query contact drvaibhavg@yahoo.com"
},
{
"id": 68094,
"tgt": "What is the treatment for a painful lump in the groin area?",
"src": "Patient: My husband is 34 and has had a lump in the crease of his right leg and groin area for the past few months. It changes size from time to time and can be painful too sometimes. But today he has not checked it lately and when he streteched while laying down he felt a sharp pain down there and low and behold he was bleeding and saw a b b shaped hole where the blood was coming from. The lump was sore and the area was soft to touch. What is this lump and what would cause it to burst a hole like that and bleed? Mind u this area has no hair so he has had no need to shave there and we both know its not a boil or ingrown hair. Doctor: Hi, dear. I have gone through your question. I can understand your concern. You may have some inguinal lymphnode enlargement. It may be due to tuberculosis with supraadded abscess formation. Or it may be some other inflamed lesion. You should go for fine needle aspiration cytology and culture of that material. Then you should take treatment accordingly. Hope I have answered your question, if you have doubt then I will be happy to answer. Thanks for using health care magic. Wish you a very good health."
},
{
"id": 50222,
"tgt": "Diffuse parenchymal disease of kidneys, creatinine high, foamy urine, BP patient. Treatment?",
"src": "Patient: my husband had a kub ultrasound recently and itxshowed diffuse parenchymal disease of kidneys. His creatinine also showed higher than normal results. he does not present any symptoms except a foamy urine and he has hypertension for the past ten years. his daily intake of water is beyond average about 4 liters a day. what do we do next? please help Doctor: Hi and welcome to HCM. Kindly see a nephrologist asap if you have not done already. Depending on the level of creatinine, kidney size and urine tests your husband will most likely need a kidney biopsy to find out the cause of his problem(s). The nephrologist will make an overall assessment and guide you about the amount of permissible fluid intake as well."
},
{
"id": 25528,
"tgt": "Suggest remedy for hypertension, cold sweats and panic attacks",
"src": "Patient: High blood pressure 173/116 and cold sweats, just another panic attack? Had a complete work up two weeks ago, with nuclear stress test. Returned to ER a week later because BP was still high even after doubling bp meds. Treated with ativan and released. Started with the drippping cold sweats about two hours ago. Dawned on me a few minutes ago to check BP again. It was high earlier and I took 1.5 ativan. I really dont want to go back to the er to be told the same thing. Doctor: Thanks for your question on Health Care Magic. I can understand your concern. Stress, anxiety and panic attacks are the biggest risk factors for uncontrolled hypertension. So we should first treat your panic attacks to achieve blood pressure control. For this, better to consult psychiatrist and get done counselling sessions. Try to identify stressor in your life and start working on its solution. You may need anxiolytic drugs too. Along with drugs, counselling plays very important role in control of panic attacks.Once you anxiety and panic attacks are controlled, your blood pressure will also reduce. Continue blood pressure drugs and consult psychiatrist for panic attacks. Don't worry, you will be alright. Avoid stress and tension, be relax and calm. Hope I have solved your query. I will be happy to help you further. Wish you good health. Thanks."
},
{
"id": 56947,
"tgt": "Suggest treatment for high sugar level with bile duct dialation",
"src": "Patient: I'm 25 years old and in the past 4 months after a acne medication treatment I find out that my blood sugar is high . My doctor send me for and abdominal ultra sound and he found a 9MM common bile duct dilatation .i was told that's is really weird that someone my age will have such thing. I was advised by my doctor to go for an MRI to see the abdomen better . My concern is how can this be taken Care of and what are the procedure to get it removed if any and what action should be taken because at this point my doctor is not really telling me much and I'm really scared . Doctor: Hello,Dilatation of the CBD suggests obstruction in the distal end of thCBD (common bile duct) this could because several different causes like present since birth (choledochal cyst)stones. Pancreatic disease, nodules/tumors or rarely a normal finding. I would take this step by step. First get the MRI with MRCP done .This will help to identify the cause of the dialted CBD. Once this is done further line of management can be planned.Hope this helpsDo get back in case you have further queriesRegardsDr Samir Patil"
},
{
"id": 73506,
"tgt": "What causes pain in chest and jaw?",
"src": "Patient: my chest&jaw were in alot of pain `i think i hav angina but am i too young for this `i have been smoking since i was 9` im 33 now` had preclampsia when i was pregnant`2 years ago` should i go to hospital or call dr tomorrow~ chest& jaw do not hurt anymore `they both hurt for about 7min about 2o min ago Doctor: Respected user , HiWarm welcome to Healthcaremagic.comI have evaluated your query thoroughly .* This seems more in relation with musculo skeletal issue than cardiac case , still one check up with physician is mandatory to rule out any underlying abnormality .* Primary relief with tab. ibuprofen( 400 ) mg , later if pain persist can rush to hospital or else tomorrow get doctor consultation .Hope this will help you for sure .Regards ."
},
{
"id": 58835,
"tgt": "Is Bhui amla useful in treating hepatitis B? How can I get livomyn and nircocil?",
"src": "Patient: i had thatbhui amla is proving it's effcacy in hepatitis B and liv 52 hb or sy. livomyn should be given along with nirocil for better resultsplease how can i get this drugs and how is the cost of the drugs and how long can i take this drugs then i will get well.please how can i get the website of this company and their phone number so i can contact them,i wish to hear back from youregards Davis Doctor: hello,,nirocil,bhui amla,liv 52 are all supposed to be ayurvedic herbal hepato protective aith multiple advantages..but do not heal hepatitis b (which is a viral disease)which you can get in any ayurvedic pharmacies..along with these drugs you should follow drs treatment and advice,stay healthy,healthy life style,absolutely no alcohol.avod hepatotoxic drugs,control sugar levels and bp,avoid obesity,,take care"
},
{
"id": 91408,
"tgt": "What is the cause of flank pain?",
"src": "Patient: If there is a cost for this service I will decline. My doctor is away this week due to the holiday period and he already has ordered a repeat chest x-ray after I complete antibiotics. I have had a \"tweaking sensation\" r flank and just below ribs anteriorly intermittent for about a couple of months. Ultra sound of gall bladder and liver are o.k. and checked about 3 weeks ago. Last week I came down with clinical symptoms of pneumonia. The x-ray revealed a new linear opacity right lung base. I had multiple nodules in lung years ago. I am also a 17 year survivor of ovarian and fallopian tube cancers, and tested positive for mutation of BRCA2. My son who was 22 3 years ago had stage 4 Hodgkins and he also has the gene mutation as did several family member who also have died from cancer quite young. So obviously I am a bit concerned. Doctor: HI. I think you have to confirm the diagnosis by a CT scan and if necessary FNAC if you still have any positive findings as per your history and family history."
},
{
"id": 68966,
"tgt": "Could lump below nipple be due to cancer?",
"src": "Patient: hi, im 24, i have just had a baby 10 days ago, i am not breastfeeding, and my breast milk is starting to dry up a lot as my breasts are beginning to get softer and softer, i noticed a small lump just below my nipple, i am worried about this, could it be cancer? Doctor: With the history you give its unlikely to be cancer. Is ot warm, red or tender to touch. If yes it could be a lump due to the collected milk. Bodys way of telling you to breast feed. If any of the symptoms I mentioned are there then it could be developing into an abscess and you will need to take antibiotics. Take care."
},
{
"id": 123939,
"tgt": "Is fracture of t6 vertebrae a serious concern?",
"src": "Patient: I fractured my t6 vertebrae april 22 emerge drs didnt notice it and sent me home two weeks later my dr called me and sent me for more xrays and i am now waiting for an appt that isnt until nov 7 Ive had no treatment for this since my fall should i be concerned? Doctor: Hello, Fortunately, it appears that you have no neurological deficit. Kindly go to a spine center for proper evaluation & treatment. Hope I have answered your query. Let me know if I can assist you further. Take care Regards, Dr Nirmal Chander Gupta, Orthopaedic Surgeon"
},
{
"id": 21612,
"tgt": "How to treat pulmonary hypertension and congestive heart failure?",
"src": "Patient: I have CHF and PHT , diagnosed in april. Medical accepted me then canceled me then reaccepted me just to cancel me again. Now I stopped go to doctor ,cant work and one my meds costs $1700.00 a month so I cant afford my med. so I know I m going to die soon if I cant get this fixed. What can I do? My name is Eric Laughlin and I m only 41 years old Doctor: Hi There I understand your point and your worries and I have an idea for this why don't you come to India for your treatment as it's cheap here with quality maintained, discuss this option with your cardiologist there and think over it. I can give you the names of two very good cardiac centers in Bangalore you can search on Internet regarding these hospitals and make up your mind. 1. Shri Jayadeva Institute of Cardiology, Bangalore 2. Narayana Institute of cardiac sciences, Bangalore.Both are tertiary Cardiac centers.I wish you Long and Healthy Life.Good luck Dr Bhanu Partap"
},
{
"id": 130914,
"tgt": "Is warm pack or ice pack better in bringing down the swelling near the area of surgery?",
"src": "Patient: Hi I wanted to know if a warm rag or a ice pack is good I just had surgery a few days ago next to the wound it is swollen and I think infected I won't see my doctor for a few days he did give me meds for infections but I want to know if warm or cold will be better for the swallowing and help the meds with the infection thank you Doctor: Hi,Ice packs are used if injury or surgery has done 2 days back.After 2 days only hot packs help.Apply locally hot packs.Thanks,Dr. CHANDER MOHAN SINGH."
},
{
"id": 113753,
"tgt": "Back pain, burning sensation, bloating, on drinking coffee, eating icecream. CT for kidney scan shows no stones, hard small stools. Treatment?",
"src": "Patient: I am a 46 year old woman experiencing right side/back pain/burning off and on for a couple of months. I feel pressure; like I have to expel gas, or I belch and feel rumbling in my gut. I think the association is with ice cream and coffee; but don t experience it all the time when I have them. Just notice when I do feel it, it is after I have had ice cream or coffee. Had coffee on Sunday and wow... Immediate burning and gas. the discomfort feels deep in my gut, lower right side/ back...Thought it was a kidney stone but CT without contrast and US were negative. I believe the pain briefly lessens when I drink water and go to the bathroom. During episodes, feels worse just before having a stool. Stools can be pebbles or large and formed; never diarrhea ... Have an appt with GYN and GI doctor in a couple of weeks. Previously diagnosed with Celiacs and doing pretty good not having anything sneak by. Worse at times when I lie down, was told I strained my back but muscle relaxants didin t work. stretching sometimes helps. Doctor: a few things remain unanswered, like do u have back pain along with the hard stools or otherwise also. because this could b because of celiac disease per se. back strain needs to be evaluated - at least an MRI is required of the lumbosacral spine. YOu might be suffering from Prolapsed intervertebral disc. the bloating and distention of abdomen could be , again due to celiac disease or gastritis may also cause these problems. You shall take Ranitidine 150 mg once in the morning and once in the evening to begin with. never eat icecream after a heavy meal, and always take non chocolate cookies with your coffee. You can get in touch again after you have your MRI done."
},
{
"id": 146875,
"tgt": "What could cause sudden dizziness and pressure in head at night?",
"src": "Patient: wake up in the middle of the night then all of a sudden spinning n dizziness also like pressure I m assuming, I shut my eyes wen it does this to me happens one or twice a month I really would like to know how to control this .. I have this after I have dreams... Doctor: Hello dear,The symptoms as mentioned in your post can be attributed to pathology in the Vestibular Apparatus in inner ear (it is associated with maintenance of body posture & balance) most probably Labyrinthitis or Benign Paroxysmal Positional Vertigo.Symptomatic relief can be obtained with intake of Vestibular sedatives like Betahistine or Cinnarizine preparations (to be taken only under the guidance of a Physician).If it still persists, then you need to consult your Physician/ ENT Specialist & get a complete clinical examination done.Investigations like estimation of blood pressure, blood sugar levels, serum electrolytes & vestibular function tests will be required to rule out any pathological cause for the symptoms.There is no need to worry, you will be fine.Till then, maintain adequate hydration & proper nutrition status and avoid stress.And also take precautions of getting up from bed slowly & avoid sudden head movements.Wishing you a Good Health.Take care."
},
{
"id": 156813,
"tgt": "Should I consult an Oncologist for the Tumor on the colon removed?",
"src": "Patient: My husband just had part of his colon removed,he was informed on that part of his colon there was a tumor that they said was pretty aggressive. They said they did not see any more signs of a tumor anywhere else. But they are going to send him to an oncologist. Im wondering what we can expect. Thankyou. Doctor: Hello'Welcome to HCM.Depending on stage of colon cancer pateint needs further treatment. Treatment can include chemotherapy ,newer molecular targeted therapy or combination.Your Husband is being referred to oncologist for assessing disease status and need of further therapy.Thanks"
},
{
"id": 126909,
"tgt": "What can cause pain and discomfort in the legs while walking?",
"src": "Patient: Hi, was putting on a pair of jeans and, as I stepped in to them , my left leg seemed to get pulled. The inside of my leg aches as well as the back (by the bending joint) and I feel some pain and discomfort when walking. I put on a compressed bandage and that makes it feel better . Any ideas? Doctor: Hi, It may be a ligament sprain or minor contusion. You can apply ice packs for symptomatic relief and take analgesics like Acetaminophen or Tramadol also. Generally, the symptoms will settle in a couple of days. If symptoms persist, better to consult an orthopedician and get evaluated. Hope I have answered your query. Let me know if I can assist you further."
},
{
"id": 30541,
"tgt": "What causes scarlet & glandular fever?",
"src": "Patient: Hi, may I answer your health queries right now ? Please type your query here...In my late teens I suffered from scarlet fever and glandular fever in quite short succession of each other, I was quite ill with both of them. Now I'm in my late 20's and have finally been diagnosed with a hearing problem which my specialist thinks was probably caused by this. I've never thought about any long term side effects before, but I'm aware that whenever I get a cold or the flu I get really ill with them and it takes me a lot longer than most to recover whenever I'm ill with anything. Could this also be related to having these 2 fevers when I was younger? Or just a coincidence? Doctor: hiwelcome to HCMI read your history n can understand your concern. scarlet fever is streptococcal bacterial infection and gandular fever is a viral flue. both the disease takes its own time to resolve, it depends on each individual immune system. and your hearing problem is not because of these two.but whenever u get cold, there wil be slight decrease in hearing because of E tube blockage. Dont worry. better meet ENT doctor.thank youtake care"
},
{
"id": 134037,
"tgt": "What could cause locking of hip?",
"src": "Patient: I was in the shower today and bent down for my shampoo when i straightened up my Hip got locked in place and i couldn t move it for 15 mins in this time it was Extremely sore i also have AVN in my Hip im not in the mood to go in for a Xray it will take too long i rather get your opinion frist,,, Doctor: hi,thank you for providing the brief history of you. As you mentioned about the locking of hip and also you have AVN of hip, this might be related to the issue you are facing now. I will advice you to undergo a physical therapy session which will help you to maintain the hip mobility and also the strengthening of the muscles, as in AVN the muscles get weak and this kind of locking occurs to safeguard the hip joint and its a neurological protective mechanism call of the brain. In my personal practice we have seen cases with AVN and with physical therapy they get the maximum outcome scores.RegardsJay Indravadan Patel"
},
{
"id": 169252,
"tgt": "How to treat discomfort due to indigestion?",
"src": "Patient: My 1 year old woke up crying didnt eat anything different today...he tries to fall asleep and then I can tell either he is trying to digest something and he starts crying/sometimes screeming and can t get comfortable...maybe he swallowed something I am unaware of. How long could this digestion take? Should I be giving him something or take him in to ER? Doctor: Hi Dear,Welcome to HCM.Understanding your concern. As per your query your child have symptoms of indigestion which is because child is suffering from viral infection which is leading to imbalance in body fluids. It could be due to abnormal abdominal movements. Need not to worry. I would suggest you to visit pediatrician once and get complete examination done. You should go for blood and barium swallow tests and start treatment after complete examination. Give child plenty of fluids and oral rehydrating solution as well. Avoid taking large meals at a time and take small and frequent meals. If fever increases do tepid sponging as well with cold water. You should give multivitamin tablets. You should give antipyretics for fever relief such as calpol,paracetamol or adol suppositories acetaminophen. Hope your concern has been resolved.Get Well Soon.Best Wishes,Dr. Harry Maheshwari"
},
{
"id": 213538,
"tgt": "Student, facing lack of concentration in studies, sports, hobbies. Root cause?",
"src": "Patient: hello sir/madam, I am a 16 year old student from india. i have an problem. i have lost my concentartion during past month and due to this i also cannot concentrate on my studies and other activities such as playing, listning , watching movies etc. please just tell me the rootcause of the problem.. this problem occured with me suddenly i shall be very thankfull to you. Doctor: Hi Deepak, Welcome to Heathcare Magic and thanks for your question... From your brief description, I understand that you have been suddenly having diffifulty in concentration and feeling disinterested in your normally enjoyable activities. Since you have mentioned that it has been sudden, it is important to know if there have been any recent changes in your personal circumstances or social environment or if you are in the midst of a stressful situation. Excessive stress and difficulty in coping is a common reason to feel disinterested and detached from studies and other pleasurable things. There is also a possibility that you may be going through a depression, in case you have other symptoms like negative thoughts, feeling low, reduced energy, poor sleep / appetite, etc. My advice to you would be to talk to and share your feelings with someone close to you, especially if there is something bothering you. In such situations, it is also important to try not to isolate yourself or spend a lot of time in solitude, as it can worsen your symptoms. If you find that your symptoms are severe or worsening or affecting your day-to-day functioning, then you need to seek professional help, probably from a psychiatrist. All the best. - Dr.Jonas Sundarakumar Consultant Psychiatrist"
},
{
"id": 159401,
"tgt": "Boil on nipple, red and painful. Is it breast cancer?",
"src": "Patient: hi, i am 24 year old,and i married 6 months back, now i got boil on my nipple , it s red in color, and it s very painful, and it is spreading,i consult doctor she gave medicine, due to of medicine there is no pain but still the boil is like that only, color also vanishes, i am thinking that it may be breast cancer please give your suggestions. Doctor: Hello, welcome to HCM, I am Dr. Das Look, your description indicates presence of acute inflammation. It is not a cancer. You should finish the medicine and after that consult with your treating Doctor. Regards."
},
{
"id": 153250,
"tgt": "What are the symptoms of stage four liver cancer?",
"src": "Patient: I am having some problems with my taste and smell ,I don t have any. I s been two weeks since my last chemo treatment, I have had four treatments up until now. I have stage four liver cancer and I am taking promethazine for nausae,andgabapentin for nurothopy. Should I be concerned? Doctor: Hi,Thanks for writing in.Stage 4 cancer of liver is advanced disease. It might cause mild symptoms in some patients and others might have severe symptoms.Usually the condition might have a large mass or multiple masses in the liver causing a hard to palpate mass in the abdomen. The mass causes compression of vascular structure including the inferior vena cava and this might lead to blood clots in the vein goind to the heart. The veins of the kidneys might also show clots.Few people develop ascites and fluid collection in the abdomen and this is due to the inability of the liver to function normally. The side effects of chemotherapy medicines also have an influence and this might cause neuropathy and nausea with dizziness and pain. Please discuss pain relief with your doctor. Please do not worry."
},
{
"id": 203441,
"tgt": "What does white crust like formation on penis foreskin indicate?",
"src": "Patient: hi i have got white crust like formation on my penis foreskin, any idea how to treat it? it looks like it started with red pathches on my glans & then i was cleaning it & putting cocunut oil. then yesterday night i cleaned my penis with very cold water before going to bed & when i got up in the morning i could see the white crust now. please advice. Doctor: HIThank for asking to HCMThis could be a formation of smegma this is some what generally forms to a male person who have not circumcised, you have to gently clean it with cotton swab no need to apply any thing just clean it with plain water, take care and have nice day."
},
{
"id": 3698,
"tgt": "Could i be pregnant without direct intercourse?",
"src": "Patient: I feel silly for asking this, but my boyfriend and i were messing around and he came onto my hand and part of his stomach, he cleaned it all off of his stomach, i dont know if any got on his hands, but i assume it was dry, he touched my clitoris so badically his hands only got near my vagina, could i be in any way, pregnant? I am on birth control and never skipped a day Doctor: hello,i can understand your concern.donot worry, there is almost no chance of you getting pregnant.you are on birth control pills, this gives you protection during unprotected sex. this is the most important part which you have followed correctly. the failure rate is very minimal in this.as you have mentioned you havent skipped a single day of the tablet, you are well protected.secondly your boyfriend didnt have a direct intercourse also, the chances of his hands containing sperms when he ejaculated on your hand and this entering your vagina while touching is also very rare.rest assured the chances of you getting pregnant is almost none.hope i have answered your concerns.feel free to ask further queries.thanks for choosing HCM"
},
{
"id": 216503,
"tgt": "Suggest treatment for pain in legs",
"src": "Patient: I have a had a lot of leg pain. Standing at work right now I have a coninuous dull ache in my legs. I do stand on cement. running from my knee down. according to the mri I have a partial torn minisucus and partianly torn tendon. Can these issues contribute pain in other parts of the leg. I also get cramps in my legs while sleeping and I am very healthy. Mike Doctor: Thanks for contacting healthcare magic. You have problem of partial meniscus torn can treated by surgically and nonsurgicaly. Take a rest ....Ice can apply over this area...Physiotherapy is helpful .....In partial meniscectomy torn part is remove by surgery....Treatment is depend on injury site. ...If Meniscus torn at outer layer side that can heal easily after surgery. If torn at inner side of meniscus that has lack of blood supply so delay healing.Consent a physician for further diagnosis. I hope my guidance is helpful to you. Take care. Thanks."
},
{
"id": 14413,
"tgt": "What causes weird rash on the leg?",
"src": "Patient: Hi,My 5 year old daughter has a small weird looking rash like area on her leg. It has irregular shape (almost star like) it is brown on the edges and lighter in the inside area. It is not raised. It appeared as a bright red rash 3 weeks ago. It turned brown in a few days, but it is not going away at all. Thank you Doctor: Hi, Welcome to HCM,The diagnosis is tinea corporis, commonly known as ringworm.Many small rings collide and form a star shaped lesion.Characteristic of tinea is peripheral Papulo vesicular lesion and scales with central clearing.Apply clotrimazole or miconazole cream (OTC cream) twice daily.You can send clinical photograph to us for confirmation of diagnosis.RegardsDr Shahid Hassan MD(Dermatology)"
},
{
"id": 171784,
"tgt": "What causes constipation in an infant?",
"src": "Patient: sir my baby is in 22 days in starting time he was very well in deliver motion but last week he went two days late and three days late so please say the reason and problem and how to cure this problem and this problem only for this time or else in future please say Doctor: Hi, can very well understand your concern. Infants passing motion once in three days to 7-8 times a day is considered to be a normal. If baby is accepting feeds well, no vomiting, no excessive crying then not to worry about motion. Feed the baby every two to three hourly. Burp the baby well by holding the baby in upright position. Baby's having mixed feeding like breastfeeding and top feeding may have digestion issue. Feeding mother should drink plenty of water. Hope this helps you."
},
{
"id": 221221,
"tgt": "What can cause excess enlargement of breasts?",
"src": "Patient: I am currently just over 22 weeks pregnant and my breasts have grown over 12 cup sizes already. I used to be a 32d and am now well over a 32k. I have been seen by the breast specialists at the hospital who have told me that my breast are very engorged, with blocked ducts and valves as well as inflammation of the breast tissue however there solution is that I just get on with it. I am absolutely beside myself, I can t sleep the pain is unbearable and the radiologist thinks I have a rare condition caused by pregnancy (which he couldn t give me a name for) and that I should just learn to love my breasts. He also said they probably wouldn t ever return to their original size. I have looked on the net and have now convinced myself I have gigantomastia which is distressing me greatly as some of the pictures on the net are horrendous. I have gained a stone in weight and yet my bump at the moment is only small. I am having trouble functioning and am desperate for help. Doctor: Hello, and I hope I can help you today. I am sorry that your breasts have developed such a painful condition. You are correct that you may have breast hypertrophy, which is abnormal growth of the breasts, due to the hormones of pregnancy. Gigantomastia describes breasts that increase in size by 2.5kg or greater. Unfortunatley, there are no effective treatments for breast hypertrophy, especially during pregnancy. Hormonal medications really do not work, and elective surgery is not recommended until at least a year after the baby is born, as sometimes the size of the breast does go down on its own. In the meantime, ice packs and supportive, stretch bras, even when you sleep, can help with the discomfort. Limiting your overall weight gain will also help slow down breast growth, as a significant portion of breast tissue is fat. Applying lots of lotion and emollients to the skin will help reduce stretch marks. I am sorry that there is no immediate treatment for your condition, but your breasts may slow in growth as the pregnancy progresses. I hope I was able to adequately answer your question today and that my advice was helpful. Best wishes for the rest of the pregnancy,Dr. Brown"
},
{
"id": 197217,
"tgt": "What causes scrotal ulcer?",
"src": "Patient: hi, i found a scrotal ulcer and it is small and painless. it has stung a little, thats why i noticed it. i am afraid that it could be herpes, but it seems improbable. are there other reasons why i might have something there. something common? Ken Doctor: HelloThanks for query .A sudden appearance of small ulcer over scrotal sac could be due to burst open boil or a sebaceous cyst and does not signify any major issue .Take antibiotic like Doxicycline and Diclofenac twice daily with topical antibiotic ointment like Betadine for 5 days.Ensure to was your genital with warm water twice daily.It should heal up within a week . Dr.Patil."
},
{
"id": 136148,
"tgt": "What causes random sharp pain in the fingers?",
"src": "Patient: I m getting a sharp pain that runs along my left ring finger. It started last night and kept me from sleeping.It has continued at random times today returning every so often.There are no stings or bite marks so I m assuming it s internal, maybe nerve? Any thoughts? Thanks,Eddy. Doctor: Hello Eddy, i am sorry to hear about the discomfort you are suffering from. the sharp pain from which you are suffering is probably due to irritation of a digital nerve. you should definitely visit a hand surgeon if the symptoms remain the same as there maybe a neuroma or compression of ulnar or median nerve. in case of compression they will perform a decompression surgery or in case of neuroma they will excise it under local anesthesia."
},
{
"id": 106426,
"tgt": "How to cure Urticaria?",
"src": "Patient: Urticaria on her neck and arms and nothing ever seems to make it go away and stay away My poor sister has urticaria on her neck and arms and nothing ever seems to make it go away and stay away. Steroids work short term. Has anyone had any success with something different? She is only 25 and it really bothers her. Many thanks Doctor: Avoid any obvious triggers or exacerbating factors and try these simple measures: 1) Keep the skin cool, avoid getting hot from exercise and take lukewarm baths. Resist the temptation to rub the itchy skin and apply copious amounts of moisturizing creams to reduce dryness and itch. Avoid alcoholic drinks and foods containing additives. 2) Apply 1% menthol in aqueous cream to soothe the affected skin. 3) Avoid all aspirin or codeine containing medication (including ibuprofen, diclofenac and mefenamic acid). Only use paracetamol as a painkiller. 4) Carefully read Side Effects list of any other medication you are currently taking and avoid medicines that are known to trigger urticaria or angioedema. The mainstay of treatment is high dose ANTIHISTAMINE medication which may be necessary for prolonged periods (in excess of 6 weeks). Double conventional doses of antihistamines such as Cetirizine, Loratidine may be needed for symptom relief. Occasionally \"stomach-ulcer treatment\" medication is added. New leukotriene receptor antagonists used for asthma have produced some symptom relief in chronic urticaria.. Short courses of steroids may be necessary to settle more severe symptoms. Cortisone should not be used for prolonged periods of time as they may lead to stunted growth in children and osteoporosis in adults. Specialist investigations in Chronic Urticaria may be necessary to exclude other underlying illnesses or to confirm the presence of ?auto-antibodies? in the blood stream. The Autologous Serum Skin Test (ASST) is a good indicator of Auto-antibodies. Highly specialised treatments may include Immunosuppressive drugs (Cyclosporine, Methotrexate, and Azathioprine), Androgens and Warfarin. Colchicine is useful in treating Urticarial Vasculitis. If available, Immunoglobulin injections, Plasmaphoresis or the use of novel anti-IgE monoclonal antibodies such as Omalizumab may be necessary. The future holds many exciting new therapeutic modalities including DNA Plasmid Vaccines"
},
{
"id": 96093,
"tgt": "Stone declared in urine test",
"src": "Patient: My brothers urine test clears a sotone in stocmach. In X-ray it was not displyed.dr. said its a stone of small size. i am in village not have a good hospital nearby. how it can be cured? Is operation necessary Doctor: Please help. Wheather it can be cured by medicine or I have to take my brother to hospital for operation. \u00a0 Thanks Gautam"
},
{
"id": 168290,
"tgt": "Will swelling on nose of a child after an injury subside after some time?",
"src": "Patient: Hi, my 4 year grand daughter hit her nose on a piece of furniture and it started bleeding. We got the bleeding stopped and are putting ice on her nose as it looked a little swallow. She is not complaining of a headache or anything just that her nose hurts. She is sleepy but she did have a hard day today. Is there anything we should look for or should she be okay. Doctor: Hello! Thank you for writing us here. You can giver the little baby aspirin by only when she have lots of pain but apart from that, putting ice would help l. The swelling would subside on its own. But please do take care that you don't use too much ice or else her might get sick. The swelling would go away in a week so don't worry.Best regards,Dr Gunjan"
},
{
"id": 200427,
"tgt": "Is there any problem for doing masturbation with grade 3 variococele?",
"src": "Patient: Hi is it good to masterbate during grade 3 variocoecele. They said I got this due tb attack. Kindly let me know what can be done in this case I am undergoing treatment and also I have masterbated during this time. But after taking medicine the pain has stopped. Let me know masterbation is good at this time or not and is it curable through medicine Doctor: Hello dear,Thank you for your contact to health care magic.I read and understand your concern. I am Dr Arun Tank answering your concern.You can masturbate even though you are having the varicocele.Varicocele arises because of some other reason. So taking treatment for cause can treat you for varicocele.Varicocele has not been affected by the masturabation because it has no relation to it.Masturbation is very common in today's life. If you won't performed than you may have sweet dreams.I will be happy to answer your further concern on bit.ly/DrArun.Thank you,Dr Arun TankInfectious diseases specialist,HCM"
},
{
"id": 72130,
"tgt": "What causes a fluttering feeling in the upper part of my chest?",
"src": "Patient: Hi, recently I have been having a strange fluttery feeling in my upper right chest and upper arm area. It only occurs when i laugh usually. It lasts a second and goes away. It almost feels like that specific are of an artery is expanding or swelling momentarily. Its not painful but feels strange. I have been smoking a pack of cigarettes every two-three days for about two or three months. Doctor: Thanks for your question on Healthcare Magic.I can understand your concern. Since you are active smoker, we should definitely rule out heart diseases (arrhythmia - rhythm disturbances in heart) for your symptoms.So get done ecg, 2d echo, stress test and Holter monitoring (24 hours continuous recording of Ecg).If all these are normal then no need to worry for heart diseases.Sometimes undiagnosed stress and anxiety can also cause similar symptoms. So avoid stress and tension, be relax and calm. Quit smoking as soon as possible because it is not good habit.Hope I have solved your query. I will be happy to help you further. Wish you good health. Thanks."
},
{
"id": 214421,
"tgt": "My Face swells and my right head pain. What can this be?",
"src": "Patient: two years back, i felt a shock like feeling in my right face when i was sleeping, i woke up and looked at my right face it was swollen, and became numb, and little burning effect around my right mouth which last for 2 to 3 weeks then it decreased little. and there was a sharp pain in my right head . I consulted a doctor and took a scan of my head. there was nothing shown in my scan report. For the last one year the pain radiated from head to shoulder and through out the right body. Again i have taken a scan, found nothing. The Dr told me that it is a psychological problem, presently i m taking maxgalin 75 mg and dezella 20 mg. But still i hope that it is not a psychological problem. waiting for your reply. Doctor: , short answer : hopefully you are suffering from trigeminal neuralgia.check for other conditions such as abcessed toothdetailed answer: trigeminal neuralgia occurs mainly due to the blood vessels pressing your trigeminal nerve root it is best diagnosed by MRI scani am not sure what scan has been done before if MRI is not done i advice to take a MRI i think it has to be diagnosed before treated.hope this explainswith regardsDr.Amarnath"
},
{
"id": 149599,
"tgt": "Feet numbness, burning sensation during exercising. Possible remedies?",
"src": "Patient: Hi Doc, I need to if I should see my local GP. I have started back exercising in the past month and I seem to have a recurring problem with my feet as they become numb and give a slight burning sensation during and after exercising. I mostly walk/run and I do cycle a lot, it's got to the point especially for when I run that it's uncomfortable and annoying. What maybe the symptoms and possible remedies. Looking forward to your answer. Thanks. Doctor: Hi,Thank you for posting your query.Your symptoms suggest peripheral neuropathy, where the nerves in the feet do not function normally. In addition, one shoul also exclude lumbar canal stenosis, where the lumbar nerves in the lower back get compressed due to slipped disc.Please consult your GP for evaluation.Best wishes,Dr Sudhir Kumar Md DM (Neurology)Senior Consultant Neurologist"
},
{
"id": 115097,
"tgt": "What is the treatment for sjogren s syndrome?",
"src": "Patient: Hi I am trying to find out which Ionic Minerals (Trace Minerals)are reputable in the Market? The ones that I ma viewing is form Alpha Health Support. I want to try this out for my Sojgren s Symptoms as there is no cure for it. Also I would like to suggest this for for my Nephew who suffers from Colon Cancer Doctor: Hi, dearI have gone through your question. I can understand your concern. This minerals will not help you.Sjogren's syndrome is a systemic disorder due to autoimmunity. Treatment of choice is steroids. You should take steroids according to your doctor's advice. For colon cancer treatment of choice is surgery and chemotherapy. Consult your doctor and plan accordingly. Hope I have answered your question, if you have doubt then I will be happy to answer. Thanks for using health care magic. Wish you a very good health."
},
{
"id": 96129,
"tgt": "Why am I having severe stomach pain with white discharge on my penis ?",
"src": "Patient: I m a male that s having sharp stomach pains a day into having these stomach pains I realized a slight white discharge on my penis. Its day two with the pains and they come and go what could it be and my bowel movements have been less as usual.? Doctor: thanks for choosing health care majic.Abdominal pain with discharge from penis indicates some infection in urinary tract.please get your self examined by your doctor and he may advised some tests like urine and ultrasound.take light diet and drink more of liquids."
},
{
"id": 206866,
"tgt": "Suggest ways to overcome low self confidence",
"src": "Patient: I m 15 and I m a freshman in high school .my sister is 17 and she s a senior everyday people tell me your sisters pretty or she has a nice body etc. this really upsets me because I feel ugly and that no one likes me. All the freshman boys always tell me how pretty she is and I don t know what to say. Also my confidents is low and idk what to do anymore. I also hate how they Check her out right in front of me please help me. Doctor: Thanks for your query. Firstly, you have to be happy with the way you look, you have to tell yourself that if someone don't like you then that is there problem and not yours. once you are happy and satisfied with your looks and you feel happy about that, what others say about you wouldn't matter to you. Also, you need to set a limit with your friends what they talk to you about your sister, you need to ignore them if they don't adhere to the limits that you set. If they crosses a limit in relation to their behavior towards your sister then you need to approach appropriate authorities."
},
{
"id": 129739,
"tgt": "Is recurring cyst in middle finger oozing clear, thick liquid when lanced serious?",
"src": "Patient: I have a cyst on my middle finger, first joint, 6mm in size. In the past two months I have lanced it twice where a clear thick liquid was released. It has returned again, I'm wondering if seeking medical attention and having it removed would keep it from returning. Doctor: Hello,Cysts like this will continue to recur until the capsule is removed. A surgical removal could keep it from recurring.Regards"
},
{
"id": 191135,
"tgt": "I have had a slight infection above one of the back teeth. Some thick liquid discharge is there",
"src": "Patient: Hello, I have had a slight infection above one of the back teeth. Eventually white and sometimes thick liquid leaked out. I had assumed it was the infection coming out. This has stopped but now it almost appears like bone coming through the gum. It is about 1/8 and jagged. What is this? a hardened part of the infection? Doctor: Hi Arthur, An X-ray is needed to make a proper diagnosis. Once a diagnosis is established only then treatment can be initiated. Please consult your dental surgeon at your earliest. Thanks & Take care"
},
{
"id": 167252,
"tgt": "What causes aggressive and temperamental behavior in a child?",
"src": "Patient: hi my daughter has been showing some really dab signs lately she started having issues at age 3 when she showed signs of hurting animals and openly admits she did it she is now 12 and shows a lot of anger towards people she is a compulsive lier and imagine things and then almost makes her self belive them in fact she does believe them she is always ceating problems with my partners going out of her way to stuff things up she has been to a doctor and they said she does not have add but there is serously and issue can you tell me more Doctor: these are symptoms of aggresive and antisocial behaviour. these are due to genetic and enviromental factors. u should go for a chromosomal analysis of ur baby. extra x chromosome in genome may lead to difficulty like thease. frontal lobe lesion in brain may also cause this. so u should seek help from pediatric neurologist"
},
{
"id": 31345,
"tgt": "Suggest treatment for pus formation in foot",
"src": "Patient: Good evening!Two weeks ago I got stung by scissors in my finger foot.It has pus.I went to the doctor and he recommended to take antibiotics(Cefalexina)-5 days(I took for 10 days),local aplication with Betadina,and 2 creams: Fucicort and Baneocin.The finger is better ow but still with pus and hurts when i tuch it.Could you please recommend me a method of treatment of pus execept surgical removal of pus? Thank you in advance! Doctor: Hi thanks for asking question.You are taking correct treatment for pus lesion.With the time infection has to be subsided.If pus increasing along with systemic sign like fever and headache then incision and drainage should be done.It is done under local anesthesia .so don't worry about that.Meanwhile for fast healing take nutritional diet with more fruits and green leafy vegetables.I hope my suggestion will help you."
},
{
"id": 151719,
"tgt": "What treatment should one take for hemifacial Spasm ?",
"src": "Patient: Sir, I have facial problem. It is diagnosed as hemifacial spasm . Is t paralysis? Your treatment with medicine or massage therapy? Pls reply to follow. Varadharajan R, madurai. Doctor: Hello Varadharajan, Hemifacial Spasm involves involuntary contractions of muscles of one side of face.It usually begins around the eyes and can later involve other muscles supplied by the facial nerve. Its usually seen on one side only although it can occur on both sides of face. These movements cannot be voluntarily suppressed like in a condition called Tics and can continue even in sleep.It is thought to be due to blood vessel pressing on the facial nerve although it can be due to other causes also. It is not paralysis like you fear. Effective treatment is taking Botilinum toxin injections or doing microvascular decompression surgery if there is pressure on the facial nerve as earlier mentioned. It is possible to maintain reasonable control over the movements with treatment. Hope this helps!"
},
{
"id": 185321,
"tgt": "Suggest treatment for pain in capped tooth and gum",
"src": "Patient: I had a tooth for which rootcanal was done and fixed a cap. Now after 4 years i am getting pain from the caped tooth and the gum is also painful.doctor advised me to take moxikind cv625 with hifenac for 3 days.i continued the treatment for 5 days.but still the problem continues.I am using gel for the gums. but still the problem recurring can you pl.advise Doctor: Hello, thank you for consulting with healthcaremagic. If you are having pain in the root canal treated tooth, it means, now again some infection is developing below the tooth, which is causing this problem. You have to visit your dentist and get an x - ray of the tooth, which will show the area of infection and then treatment will be done accordingly, after then only you will get relief.Hope it will help you."
},
{
"id": 174409,
"tgt": "What causes constipation and fever in babies?",
"src": "Patient: My baby girl started 6th month on dec 30th 2014. The same day she passed hard stool like pellets and got fever. Next day onwards passes stool 5-6 times a day little watery and pellets like mixed. Today is the 6th day and today passed 6 times and it was dark green, watery and like bubbles. Please advice Doctor: Hi,Thank you for asking question on health care magic.It looks like infective diarrhea.Antidiarrheal like orndazole +ofloxacin along with sporolac sachets gives relief.Paracetamol and tepid sponging gives relief from fever.Colimex -DF drops help relieving straining while passing stools or in between.Hope this answer will serve your purposePlease feel free to ask any more queries if requiredTake careDr.M.V.Subrahmanyam MD;DCHAssociate professor of pediatrics"
},
{
"id": 169790,
"tgt": "Can an infant die suddenly due to heart ailment?",
"src": "Patient: our baby was born at the 2nd july and died at 3rd july,we only saw him for about 5 minutes after he was born and they took him away because doctors say that he dont have enough air and they will bring him back after he is fine,but before we get to see him again they told us that the baby is not fine,we were going to the nursery where they treated him before we reach the nursery one nurse came to us and told us the baby was dead because of some heart problems,can you please give me details of how he died Doctor: Hello, My Heartfelt sympathies with you. There can be number of reasons that a newborn baby can have such incidence.Since you were told that the cause is a heart ailment, I will only concentrate on that, since otherwise its a big list of causesTo make you understand in simple terms, you have to understand that the heart system that works inside the body of a baby inside womb and outside womb is strikingly different. BASICALLY THE PATH AND VALVES INVOLVED IN THE CIRCULATION are altered post birth. There are few heart conditions where in baby cannot do this transition of intra uterine to extra uterine heart cycle. This are usually serious heart problems which are basically structural deformities, and many a times may not be visualized in ante natal investigations too.Hope this helps youRegards Dr Poonam Sambhaji"
},
{
"id": 154731,
"tgt": "What are the symptoms of cervical cancer?",
"src": "Patient: hi,i had pep smear test,and the result is \"smears are cellular,adequate showing predominantly superficial and intermediate cells in a background of heavy inflammatory infiltrate and lactobacilli.benign endocervical cell clusters are seen.no dyskaryotic cell seen.is this mean i have cervical cancer? Doctor: Hello dear, thanks for your question on HCM. The report you have mentioned is not suggestive of cervical cancer.It is having high inflammatory cells. This suggest chronic infection due to bacteria.Report clearly mentioned about absence of dyskeratotic cells. These cells are seen in cancer. You are not having these cells, so no need to worry for cervical cancer. Better to consult gynecologist and start treatment for chronic infection."
},
{
"id": 135732,
"tgt": "What causes tightness in right arm ?",
"src": "Patient: Hi, I have had a bump on my neck right near my hair line for a about 2 or 3 years now. I m not sure if i should get it checked out. But recently my right arm/hand has been feeling tight in the mid afternoon, I have scabs on my arms from scratching, feel like i ve been breaking out a lot. I m not sure what to ask my doctor? Doctor: hiYou may consult a physician for checking of lump.he shall tell you what is the cause andorigin and may ask for tests as required with x ray etc.You have to give history how it arose, and whether it is slowly growing or stationary, any associated constitutional symptoms like fever,mailaise or appetite loss, any tingling and numbness and about skin rashes.the doctor will help youthanks"
},
{
"id": 102238,
"tgt": "What do you suggest for allergic reaction to bactrim?",
"src": "Patient: I have had an allergic reaction to Bactrim itchy red rash. How long does it take to leave the bloodstream I finished the medication more than two weeks ago. The rash is very light looking but the itch is constant. I have taken Benedryl over the counter hasn t really helped. Should I go back to the physician? Doctor: Hello,Welcome to HCM,The bactrim is a combination of Trimethoprim and Sulfamethoxazole, it is known to cause itching in the person who are sensitive to products of sulphur group pf drugs.This drug is eliminated from the body in 2-3 days, the half life of this drug is 8-10 hours and this drug is completely eliminated from the body by 48-72 hours.As you have developed generalized hives for this drug you need to discontinue this drug and this drug is completely excreted from the body by 2 days after stopping the drugs.For present symptoms you need to take oral antihistamines to control the symptoms.Thank you."
},
{
"id": 142792,
"tgt": "Suggest treatment for dysgraphia",
"src": "Patient: Over the past 9 months, I ve noticed an increased level of difficulty in writing. I have to concentrate like one would who s in kindergarten just learning to write for the first time. My letters are awkward and shaky. I m a 48 yo female with a history of high blood pressure. It has been controlled by medicine and I haven t had a high reading by my doctor in years. What could this be? Doctor: hithanks for using HCM.these symptom can be due to small infarct in brain ( can be caused by high blood pressure). Please get MRI brain done and consult a neurologist as soon as possible. thanks."
},
{
"id": 108391,
"tgt": "Suggest remedy to relieve back pain",
"src": "Patient: Hi, I am 31 year old female. Since last 6 months I am having following issue:1. Pain in right knee. Consulted an orthopaedician who on examination and x-ray diagnosed some problem in patella and advised for few exercises & rest plus given medicine (Diacerein tablet). This is showing improvement. What is your advice on this?2. Also, since last 1 month having severe back pain with unbearable pain. On X-ray of back, doctor said that backbone is very week. Have started taking vitamin and calcium tablets and injections. And also going for physiotherapy from last 10 days but there seems to be absolute no sign of improvement. Kindly advise what might be the issue and any alternative treatment because the pain is not coming down. Thank You.Regards,Rachana Aggarwal Doctor: Dear Rachana ji,your knee problem is called as patello femoral arthritis, and at your stage taking exercise is more important than Diacerein(This drug has found to have no good effect on arthritis) Take Paracetamol 500mg if pain is more otherwise knee physio, walking, no squating no cross leg sitting will relieve your symptoms.Now at 31 years, it does not fit that you have such severe osteoporosis that it has started causing severe pain, let me tell you its fracture due to osteoporosis that causes pain not the osteoporosis, and since you say your pain is severe, I would recommend you to consult a spine specialist, may be you are having other problems which needs correction."
},
{
"id": 87602,
"tgt": "Suggest treatment for abdominal pain",
"src": "Patient: ok so i had protected sex within the past couple weeks, however i got my period, a little early. For the first like 4 days it was very very light then about 2 or 3 days it was a little heavier but still light. Also, the past couple days, when I wake up in the morning, I have a complete food aversion and a middle ground between a stomach ache and nasea. Today, I also noticed some very liquidy discharge that was clear and had no smell. Is it likely I'm pregnant? Doctor: Hello and thank you for your question. It is possible that you are pregnant so if you miss a period please check a pregnancy test. It is a little early to be feeling pregnancy symptoms however so you may also consider getting checked for a sexually transmitted disease. If you have abdominal pain and a positive pregnancy test you should go immediately to the Emergency Department to make sure that you do not have an ectopic pregnancy (pregnancy in tubes) which can be an emergency."
},
{
"id": 14659,
"tgt": "Suggest remedy for rashes",
"src": "Patient: We just returned from a trip to a hot springs national park where we swam in the local plunge. today my husband notice a rash on the tops of his toes. He said it is tender to the touch but not itchy. It looks red, swollen, and hot. is this something that we should be concerned about? Doctor: HIWell come to HCMSuch rash well response to steroid preparation and the best option would be Mometasone cream better to have oral medication too and the option is Tab Levocetrizine 10 mg three times in day, hope this helps."
},
{
"id": 125612,
"tgt": "How to treat severe pain in elbow,groin?",
"src": "Patient: My husband, came to me about 3 hours ago and said he was in horrible pain. He described pain in his elbow, and extremely painful groin area on the same side of body. He has Rheumatoid Arthritis and is on quite a few medications. He does have a radiculopathy in his cervical spine and has the elbow and shoulder pain before. This week, he had a bout with severe shoulder pain on the opposite side, radiating around his entire left shoulder area, almost causing the shoulder to feel locked. We saw our rheumatologist s partner this week who upped his prednisone to 20mg per day for 10 days. She suggested that perhaps we will want to get xrays if he has more symptoms. I know about his elbow pain caused from the ridiculopathy,as it has before. I just am very concerned about this unbearable pain in the groin. Doctor: Hello, As a first line management, you can take analgesics like Paracetamol or Aceclofenac for pain relief. If symptoms persist better to consult a physician and get evaluated. An MRI scan is required for further assessment. Hope I have answered your query. Let me know if I can assist you further. Take care Regards, Dr Shinas Hussain, General & Family Physician"
},
{
"id": 211492,
"tgt": "Suffering from depression. Goes violent sometimes. On qutipin. Possible to overcome from this?",
"src": "Patient: Hello Doctors,My daughter, age 26, is suffering from \"depression\" for the past 6 years. She goes violent sometimes. She was taking sulpitac at a reduced dose of 50 mg recently. However, she was having delayed periods (more than 50 days) and this often deteriorated her condition. Her blood report confirmed that her periods problem was due to an excessive level of hormon created by sulpitac. So, her doctor prescribed alternative medicine \"Arip\",but, it did not have any effect on her. After starting this medicine, her behavior deteriorated with violence. Now she is taking another medicine \"Qutipin 100\". Now her symptoms are mostly under control, but, at times she shows temporary behavioral problems. Doctor, can she overcome from this illness? Can she lead a married life? She had undergone a surgery for her \"VSD\" when she was 4 months old. Thereafter she perfectly in good health, except this behavioral problem.Awaiting your reply, thanking you. Menon Doctor: HIThank for asking to HCMI really understand your worry, take it easy every thing will be fine, the anti depressants and the menses bleeding are nothing to do with each other, I advise you to just forget about the menses, right now it is not the big deal our main goal is to pull her out from the depression and anxiety, you ca do it, you first take her under your confidence, have good relation with her if you do not have such relation then try to form such relation that she could consider you as her best friend, try to spend more time with her, share the talk with her, listen her properly, do not try to avoid her or oppose her, she is in need of mortal support and homely environment that only you can provide it, you can discus this with her psychiatrist, have nice day."
},
{
"id": 160133,
"tgt": "Liver Cancer \"Static\" with a large 14cm tumor",
"src": "Patient: Hi There...My Father has a multiple tumors within the liver . 1 being 14cm large and 5 others in the general area.Have yet to confirm via biopsy, the CT is scheduled. Blood and X-rays indicate that it has not spread to the Lungs or colon. He is a 66 year old Asian Male that has Hep B (believe it was onset from childhood). A Cancer Agency Dr. looked at the X-rays and said this might not be operable...the confusing part is that they have yet to do a Biopsy to confirm Cancer and based his observations on an X-ray rather than a CT. If this is truly not operable, are there any other options and hope for recovery. Are there newer Chemotherapy or Laser Removal technology that can perhaps help? Doctor: Hello, Thanks for posting your query. Let his other investigations be done. In-operability of the tumor indicates usually more severity of the condition. He might have to have chemotherapy which would be palliative. But wait for the reports and based on it the treatment would be planned. Hope I have answered your query. Wishing you good health. Regards,"
},
{
"id": 25760,
"tgt": "Why does my blood pressure shoot up when having fear about going to office?",
"src": "Patient: i am a working men, working in a mnc from last 20 years. from last few days donot know why i have a fear of going to office, my blood pressure shoots up to 160/ 100 with a thought of going to office. normally my blood pressure is at 130/80. i am unable to understand the problem. my family doctor suggested to take 2 Librium tablets a day... but i think its not working ,,,as still after taking it for 2 -3 day the problem persist. plz suggest Doctor: Hello , any form of anxiety ( in your case going to the office ) will push in hormones like cortisol and more importantly adrenaline and many other in the blood stream . These hormone will result in constriction of the blood vessels and making your heart pump more vigorously pushing your blood pressure UP. By all means you will have to sort out the reasons why you have suddenly started hating your work . Remember stress will only harm your health and nothing else. Medication like librium act as anti anxiety pills and will help you tide over temporary crisis by calming your nerves . However as I said you yourself have the answer to all your problem. Avoid stress , avoid blood pressure. Regards Dr Priyank Mody"
},
{
"id": 20716,
"tgt": "Is it safe to drive and do cardio workout after a heart attack?",
"src": "Patient: Hi, may I answer your health queries right now ? Please type your query here...I had aheart attack December 30th. Angiogram last Wednesday which was fine. Doctor told me I could go back to work and didn`t need cardio rehab ( I`m a fitness instructor ) I`ve since been told the opposite by a cardio nurse, don`t know who I should be listening to. Also one says I can drive then the other says I can`t. Doctor: HIWell come to HCMI really appreciate your concern, if the cardiac related test are unremarkable and you do not have any symptoms then it is nothing to worry, and ideally you have to follow the instruction given by your cardiologist no any one else beside the cardiologist because cardiologist can only take care of your disease hope this information helps."
},
{
"id": 93011,
"tgt": "Stomach pain, loss of appetite, bloating, hard to swallow solid foods. Normal chest x ray. Any ideas?",
"src": "Patient: Hi I am having really bad head an tummy pain along with that I have lost my appetite. When I eat I start to get really full an bloated it also feels like it is really hard to swallow solid foods it feel like a big knot in my throat! I have loss at least 35lbs do to this an a lot of hair!! My primary said there is nothing wrong with me and I'm fine.my insurance will not pay for MRI which is what I think I need? I also had X-rays done on my chest because I had chest lumps that are hard like bone but again my DR said everything was fine! Should I go get opinions from different dr or what could it be?? Doctor: Hi, it appears that you have gastric hyper acidity, and the head ache may be due to gastric or may be due to anxiety. I advise you to consult a physician for diagnosis and treatment. I to my patient with such symptoms prescrobe omeprazole, domperidone, antacid gel, and pain killer like acyclphenic . Thank you."
},
{
"id": 190037,
"tgt": "Persisting white mouth sore. Need remedy",
"src": "Patient: I have a mouth soar that i bit quiet hard, to the point where the skin is dropping down, and i keep biting it, i think i bit it while i was sleeping and im not sure what to do, ive been washing my mouth out with salt for the past two days, and taking pain reveler, its been working i beileve but now im freaking out because the soar is turning white Doctor: If Its turning white than take it on priority to get proper evaluation done. Symtomatic relief using painkiller is to be restricted."
},
{
"id": 179943,
"tgt": "How will i know if the medicine is curing baby's UTI?",
"src": "Patient: good day, i got a 4 mo and 2 wks baby. her urinalysis shows pus cells of 13-15. her fever is almost 6 days and up to now the fever is still on off ranging temp of 37.9 to 38.5, and she got temp of 40.2 once. Her pedia gave me already antibiotic Cefaclor and its her 3rd day taking med. im wondring, how will i know if the med is curing my baby's UTI? what other causes of uti aside from hygiene? Thanks! Doctor: Thanks for putting up your query at HealthCareMagic. I understand the cause of your concern. Our urinary tract under normal circumstances contains certain bacteria and there is nothing unusual about it. When this number increases beyond a certain limit, the situation is known as urinary tract infection (UTI) and although the body sends its white blood cells to fight, they cannot do so adequately. The dead white blood cells are known as pus cells, what has been detected in urine. If the urine is cultured, the sensitivity of these bacteria to antibiotics can be determined. If the bacteria are sensitive to the antibiotic that has been prescribed, the fever comes down. It may take a few days time and there is nothing to worry. If the fever comes down, you can assume that the antibiotics are working and the UTI is getting cured.Since the bacteria that cause UTI are already present in the urinary tract, all you have to do is to keep their number in check and this can be done by maintaining proper hygeine.I hope that helps. Feel free to revert back with further queries if any."
},
{
"id": 146133,
"tgt": "What does the MRI scan report indicate?",
"src": "Patient: I am a 57 y.o. woman with some prehypertension, slightly elevated cholesterol, no diabetes. I had a severe bout of depression and anxiety recently, which I am being treated for, but I have since been feeling more cognitively slow, and have some tremors in my hands at times, and some muscular twitches in various parts of my body. I had an MRI to rule out MS (which my mother had), and they found microvascular ischemic changes bilaterally in my brain. The neurologist said this amount was pretty average for my age. I also recently had a cardiac calcium score done - it is 1. Should I be concerned about this MRI? At the height of my depression, I was in bed for 3 days with severe headaches (I also have cervical spine degeneration) but I actually felt like I had had a mini-stroke. Has my brain had little mini-strokes, thus the ischemic changes? Doctor: Hello. I read your query and I understand your concern.Technically speaking those ischemic changes are a part of the stroke spectrum, meaning it is a case of an occluded vessel not supplying enough blood. However it is the case of very small vessels and what your neurologist meant is that a certain amount of these changes of small vessels happens in many people over the course of years and are not responsible for your symptoms, can be found in many other people your age without any symptom at all. If these changes are markedly widespread it could be a cause for cognitive changes, I can't evaluate that without the images, but since he has told you they are average for your age I suppose he has considered that.So you shouldn't be concerned too much, although you should pay attention to blood pressure and high cholesterol to lower your future risk of stroke (a recommendation which would be given even without that MRI).I hope to have been of help."
},
{
"id": 168397,
"tgt": "Suggest treatment for accidental ingestion of aspirin by a child",
"src": "Patient: My neighbour s 3-year-old, Mick, got the aspirin bottle and ate half of its contents. The baby-sitter didn t know what to do and called me for help. I get the baby-sitter to call for the ambulance and they tell him to give Mick a glass of water while they come round. What should I do while I wait? Doctor: HiEnsure plenty of water ingestion. A gastric lavage and charcoal ingestion will be useful once emergency team arrives."
},
{
"id": 40946,
"tgt": "Is HCG shot necessary even after follicles are mature?",
"src": "Patient: yep...i am 30 yrs old..trying to conceive since the past 3 months. but me and my husband are away from eachother for the work purposes. i have consulted a gync doc and she prescribed me with the fertility drugs and advised a follicular study. i had done the study , the follicles are mature and do i still have to go for the hcg shots eventhough my follicles are mature bcos i know that even is our follicles are mature it does not mean that i am ovulating? can u please suggest ur views on my case? thanks Doctor: Dear Patient, as per the case study of yours :your Doctor has given you fertility drug .in such condition ,the very crucial point is that you & your husband must be together ,in order to conceive ,additionally ,I advise you to carry out HCG shot which will also be benefical.I hope the information will be helpful Thank you very muchDr.Shubhangi TaideyThane( Mumbai) India"
},
{
"id": 112900,
"tgt": "Back pain hurting through rectum. Sonogram resulted bowel gas restricted pancreas view. Is this OK?",
"src": "Patient: I have been having very low back pain, hurts all the way through my rectum. This has been going on for 8 months. I have had a very early ct scan of the abdomen, a colonoscopy a few months later. An MRI of the lumbar spine a few months ago. I had a sonogram of the abdomen and a pelvic sonogram 10 days ago. The report of the sonograms said I had bowel gas that constricted vew of the pancreas, could not see either ovary and partial blockage of my uterus. The doctor has order both sonograms for tomorrow morning. How can I have a successful sonogram and is this the right thing to do? Doctor: These days color doppler sonography is done for female reproductive organs, uterus, ovary, fallopian tubes.This assessment is necessary to rule out any pelvic pathology which may be transmitting pain in lower back .MRI of spine should have given clue to spinal disease if any,, probably it is negative thats why your doc. might be looking to pelvic pathology."
},
{
"id": 220427,
"tgt": "What are the chances of getting pregnant while taking Microgynon?",
"src": "Patient: Hi, I have been on microgynon 30 for a number of years. Have just got back from Kenya and have been taking malarone. I was on my pill break and due to come on Friday, am regular as clockwork and have still not come on. Due to start taking pill again tonight. Did not have sex in the 2 weeks I was away as bf not with me. Came back tues night and had sex with condom but it split. Have a done a pragnancy test which was negative. Do I take my pill as normal tonight or wait for period. Also what are chances of being pregnant? Thanks, Vikki Doctor: Hello,I have gone through the query and understood your concern. Any antibiotic or antibacterial agent can affect the performance of an oral contraceptive pill and cause failures. This may be applicable to the anti-malarial preparations also. Yet, I do not think you could be pregnant since you had the intercourse after your expected period. Home pregnancy tests read well when performed 3-7 days after a missed period and not earlier. You should get a blood test done to exclude/confirm pregnancy before you decide whether to continue with the pill. Hope this helps."
},
{
"id": 66162,
"tgt": "Suggest treatment for lump on collarbone",
"src": "Patient: my husband has a lump on his left collarbone and said that his hand was tingling. He said he found it this morning and it has already gotten a little bigger. He said the back of his neck has been hurting for the last few days by the botton of his head. I am worried. Doctor: Hi, thanks for writing to HCM.Well, If I were your husband's treating Doctor for this case of a neck lump on collarbone, I would come up with three possibilities, these include: 1.\u00a0\u00a0\u00a0\u00a0\u00a0lymph nodes due to some chronic disorder like tuberculosis, lymphoma or chronic throat / neck infection or even malignant diseases in elderly people!2.\u00a0\u00a0\u00a0\u00a0\u00a0The second possibility is of benign and simple conditions like lipoma or neurofibroma; there is nothing to worry about these!\u00a0\u00a0\u00a0\u00a0\u00a03.\u00a0\u00a0\u00a0\u00a0\u00a0The last possibility is of sebaceous cyst or some other cysts like branchial cyst or lymphatic cysts! Nothing to worry about these also!Therefore, the treatment only depends upon the diagnosis and generally medicines are sufficient in majority of cases!I suggest you to go for an FNAC /biopsy and ultrasound of the lumps for confirmation and to relieve your concerns!Hope this answers your question. If you have additional questions or follow up questions then please do not hesitate in writing to us. I will be happy to answer your questions. Wishing your husband good health."
},
{
"id": 36218,
"tgt": "What causes swollen lymph nodes?",
"src": "Patient: do tonsils disappear on their own ? I have a swollen left gland in the jaw area. And a canker sore in my bottom inside mouth for a week now. I looked at the throat tonight and my tonsils are not there, throat red but not swollen. I did have very large tonsils so you could see them . Doctor: A canker sore can be triggered due to high stress, long periods of acid reflux disease or just some tooth edge trauma. If you have a fever with a swollen lymph node then it is an indication for active infection. Otherwise sometimes a swollen lymph node is just the body fighting off some insect bite on the area of drainage. If your tonsils are painful or you have difficulty swallowing then you should see a doctor who would examine you and take appropriate action like putting you on antibiotics. Otherwise you can wait it out and let the tonsils recede on its own. Thank you for your query"
},
{
"id": 105635,
"tgt": "Child having recurring cough, normalizes with medicines, diagnosed as asthmatic bronchitis, taken injection monobact, augpen. Suggestion?",
"src": "Patient: My 18 month old son have problem of cough regularly and for that we visits several times to doctors. First time in the month of March only one day in night he finds difficulty in breathing , he crying & irritable that time, after visit to the doctor & giving nebulizer (Steam) for two times in a day he feels ok and play & eat well. After one month, around 15 days before he feels likes same problem and that time the treatment is same. Next time he hospitalize for 4 days for the same problem. After checking the entire test report Dr. said that he having the problem of asthmatic bronchitis . After hospitalization and continue with medicine he feels good for next 4-5 days but again in one night he feels the same problem. Please help & guide me. is it really an asthma problem ? I attach the medical reports that taken before hospitalization and the treatment details. Treatment during hospitalization: inj. monobact 375mg -- twice a day inj. augpen 300mg ---- twice a day Kindly give your valuable suggestion. Doctor: From your description, it seems your son is having asthma problem. He should be nebulised with levolin and budecort 2 to 3 times a day. He can also be given a short course of oral steroids like omnacortil 2.5 to-5ml once a day for 3 to 5 days. Give him some antiallergic in the form of cough syrup like coscopin paediatric or phenergen. You have already given him adequate antibiotics and no need for further antibiotics. If symptoms still persist, get a xray chest and routine blood tests done and consult your paediatrician to rule out any other disease."
},
{
"id": 39085,
"tgt": "Is dengue recovered patient's thoughts and intelligence be affected?",
"src": "Patient: Hi doctor online! Please let me know if the dengue recovered patient's thoughts and intelligence be affected? I have a colleague who had just recovered from the virus but she dramatically change her attitude ... from silent type to talkative. Me and my co-workers are really wonder if it has something to do with her sickness?... Thanks a lot and more power - Tess Doctor: Hi,Welcome to HCM.Dengue presents with a combination of rash and fever and gets complicated when it presents as dengue hemorrhagic shock where patient has severe bleeding episodes, low platelet count and disseminated intravascular coagulation. There is no affect on the thoughts as such. Your colleague might just be happy that she has recovered from her illness.Thanks"
},
{
"id": 182946,
"tgt": "What could cause hard lump inside jaws and pain in joints?",
"src": "Patient: i have an enlarged uvula - touching my tongue - has been hard to swallow a few times, have a hard lump on the inside of my jaw - which has just started causing pain, i've been having pain my joints / bones: legs mostly - but also arms and hands at times - all of these things started around the same time - can you please give me your opinion? do all these things sound like they are caused by the same thing - or could they be unrelated - either way - do you have any answers? i am a little worried! thank you Doctor: Thanks for your query, I have gone through your query.The hard lump can be because of the infection in the root tip of a tooth like periapical abscess or cyst, Because of the lump the uvula might have been pushed down.Consult a oral physician and get a radiograph done to rule out the infection. If there is a tooth infection get the tooth treated with RCT or extraction.If i am your treating doctor, I would have suggested you to take a course of antibiotics like amoxicillin 500mg and metronidazole 400mg tid for 5 days (if you are not allergic). The joint pain, bony pain are all not related to this, consult a general physician for these problems.I hope my answer will help you, take care."
},
{
"id": 184314,
"tgt": "What causes the growth in wisdom tooth extraction site?",
"src": "Patient: I had my wisdom teeth out 5 weeks ago, they havent healed well and my surgeon put me on penicillin on monday, he stated that although he could not see an infection, things dont seem right. I now have this bloody growth at one of the extraction sites. I am going to book an appointment asap with my surgeon, but what could this be? Doctor: Hello, Thanks for consulting HCM, Read your query, as you have undergone extraction of wisdom tooth see what happend is healing of wisdom tooth socket takes time if it is surgical extraction or traumatic extraction dont be worried so much this growth can be the clot formation on the extraction site or it can be due to deposition of food debris in clot it look like growth at extraction site. Consult your dentist and follow post operative instructions and take proper course of antibiotics , take proper rest , take soft diet .Hope this will help you. Wishing you good health.Regards, Dr. Priyanka tiwari"
},
{
"id": 222881,
"tgt": "What causes yellowish colored skin after miscarriage?",
"src": "Patient: Hi I recently had a miscarriage and I have noticed in the morning my skin has a yellow look as do the corner of the whites of my eyes, my appetite is greatly increased and I feel tierd. Do you think I should go for a blood test from my GP as my partner and I want to start trying for another baby? Clare Doctor: Hi,Yellowish discoloring of skin, sclera and urine indicate jaundice . you should get fully evaluated by checking for bilirubin in your blood also check for viral hepatitis and take treatment accordingly. Your GP can guide you with the blood test, but you may also need an opinion from a hepatologist depending on findings of the blood results.Get these done done before embarking on the pregnancy.Hope this helps.Regards"
},
{
"id": 40285,
"tgt": "What should i do for the yeast infection which is not curing?",
"src": "Patient: hi, I have an yeast infection that won't go away. I already took 3 diflucan piils, 1 to 2 weeks in between of each other, its a little better, but I still have it. I cannot take over the counter products because I am allergic to them. , what should I do Doctor: Hello,Welcome to HCM,Yeast infection can stay for longer period of time based on the location of the infection. If there is a favorable environment it can sustain for months to years.Depending upon the site of infection, the duration of treatment varies. For genital infections you need to take treatment for atleast 4 weeks, whereas for fungal infection of hair and nails, you need to take treatment for months.As you have not mentioned the site of infection, this information will help you to know your condition and to take appropriate treatment.Thank you."
},
{
"id": 203436,
"tgt": "Is yellow discharge behind foreskin smegma?",
"src": "Patient: Woke up this morning with yellow discharge from behind my foreskin. Could this be smegma? It wasn t clumpy it was more of a liquid. My foreskin is also a little sore now as well. I am very sexually active with my girlfriend and I also master bate a few times per day. Doctor: DearWe understand your concernsI went through your details. I suggest you not to worry much. The yellow stuff you are mentioning could be a mixture of urine, semen, dust, microbes and may be some vaginal fluid. If there is no other symptoms like itching or burning, there is nothing to worry.Masturbation is not wrong. But if you masturbate few time daily and also indulge in sexual activities, my I ask, Don't you have anything else to do? In that case, you are prone to have anxiety and apprehension about sexual matters and sex organs which might lead you to a disaster. Reduce masturbation to once a day. Don't have unprotected sex. Concentrate on studies and career. Keep sex aside and enjoy it within a limit.For sexological and psychological counseling visit http://psychocure.webs.com/Hope this answers your query. Available for further clarifications.Good luck."
},
{
"id": 14006,
"tgt": "Is antihistamine for red bumps and burning sensation on thighs sufficient?",
"src": "Patient: Hi my 10 year old son got a raised red bump on his thigh with burning sensation which then turned dark in colour, the following day he got a few more (same thing). He now has about 10 of these marks all on his right thigh. I have given him antihistamine from the first day. Doctor: Hello and Welcome to \u2018Ask A Doctor\u2019 service. I have reviewed your query and here is my advice. Antihistamines will provide symptomatic relief and itching will be subsided. It would be better if you attach a picture in the followup query. You can apply soft steroids like Mometasone for faster recovery. If the symptoms persist for long, better you have to consult a dermatologist and get evaluated. Wishing you good health. Hope I have answered your query. Let me know if I can assist you further. Regards, Dr. Shinas Hussain"
},
{
"id": 137524,
"tgt": "Suggest remedy for waist and leg pain",
"src": "Patient: Respected Sir, My father suffering from weist pain, leg pain. Doctor decited spinal cord operation must. Now operation completed but pain don't out. He is shouting for pain. doctor prescribed akt-4 tablet will be take quantiniue one year. now what we do? Doctor: Dear Patient,Since your father has been started on AKT4, it means that he is suffering from tuberculosis of the spine, the surgical treatment of decompression has already been done, as your history suggest, but the medical treatment to cure the disease takes 12 to 14 months to complete, if routine pain medication are not giving relief, i would recommend you to see a pain medicine specialist, and under his guidance control pain, let the ATT continue, but please see that the dosing given is weight controlled, as inadequate dose, and multiresistant bacteria is the main cause of treatment failure in TB.I hope this helps you, feel free to contact for any other informationThanks."
},
{
"id": 145767,
"tgt": "Can MRI show clear picture of multiple back and spine problems?",
"src": "Patient: In 2007 I had a lumbar fusion L4-L5 with donor bone not my own and hardware with a laminectomy at L4 I had a MVA in 2005 and fractured my spine and spondylolisthesis. I have been having so many problems with my back and throughout my whole body ever since. I was treated with PT, narcotics, injections, exercises, OTC medicines. I starting detoxing off all pain medications and prescriptions June 27, 2014. I could feel my body getting worse. I am having burning up and down my spine that radiated every where. My lungs were on fire and chest pain with inflammation. I had fluid building up through out my body, basal temp is 96.5. I have to constantly battle to get it up to 97.5 and above to keep my kidneys working. I have continual spasms of all my organs, eyes, throat, back and chest, legs. My spine feels like it is on fire. All labs come back normal, including T3 and T4 and TSH. My tongue is huge, my face swells at will, I have stiffness in my legs and back. I have increasing low back and leg weakness. My legs go completely numb, tingling, cold from the knees down. I think its the hardware or the donor bone. I have multiple chemical sensitivity to alcohol, alcohol by-products or medical products that alcohol is used to process it. Also any medications or medical implants that may be enteric coated. I think those little bone chip things might have been! It feels like my body is attacking itself. I had an appt with an allergist/immunologist oct 1, 2014. He didn t do any tests just took my medical information and promised to contact me. No return call. I called him twice. I afraid I am going to lose my ability to walk before I can get help. The I have to go to the ER for temporary paralysis, hypertensive crisis, and allergic for shortness of breath due to the severity of the spasms. My eye doctor put me on Prednisone 20 mg for the inflammation in my eyes and fluid build up, another doctor gave me Clonidine 0.1mg to manage the hypertensive episodes from the continual spasms. Medications that would normally be used to treat spasms my body reacts with increased spasms and hypertension with syncope. Crazy. This has been going on since July 24, 2014 after all the narcotics and prescription drugs were out of my system. Will an MRI give better detail what s going on in my back? When you see the image of the hardware on the CT Scan part of the metal has a bent gap and crooked and is glowing white. I know it s a lot of stuff to hear. I am living with it. I am 98% home-bound. My legs are too weak to control. I can t drive any more. Doctor: Hi,Thanks for writing in.MRI is an investigation of choice to study the spinal cord, intervertebral discs and nerves. It is important that you confirm that the metal hardware is MRI compatible. This is because the MRI machine is a huge magnet and if the hardware is not MRI compatible then you might have serious injury during MRI scan. This happens when there is ferromegnetic properties in the metal which will attract the hardware. This might cause the hardware to get pulled from its fixed location and cause injury to surrounding structures. It is also possible that you have a MRI compatible hardware and if this holds good then there is no need to worry for any problems in MRI scan. Then your MRI imaging will be safe and the causes for your pain and discomfort can be evaluated in more detail than a CT scan."
},
{
"id": 57594,
"tgt": "Will blood thinner be a best option before a surgery on gall bladder?",
"src": "Patient: I had drug iluting stent placed in september 2013 now my gallbladder is by hidascan functioning at only 5% talked to surgen and he wanted to use a bridge (blood thinner) for five days and then do surgery my heart dr. says no can't stop asprin and plavix he will find me a dr to do surgery your thoughts on this matter thanks Doctor: Hi and welcome to HCM. Thanks for the query.youshould not use blood thinners 5 das prior to surgery and there is no compromsiing with this. to low prothrombine time may have bad effect on sugery and may cause difficulties in hemostasis so consult your surgeon once again about this. WIsh you good health. Regards"
},
{
"id": 71059,
"tgt": "What causes chest pain upon sneezing?",
"src": "Patient: I was hit in the chest over 9 months ago. Couldn t hardly walk the day after. Went to the er after about two months. Said muscle pull from chest wall, 6 mts to heal, very painful. So during the next 6 mts the pain and swelling went away completely for about two months then came back like it had just happened. Went to er again, couldn t tell me anything new but miraculously the pain and swelling went away almost the next day. That was oct 30, 2016. Now its Feb 3 and I m headed back to hospital in the morning. Back like it was never gone. Excruciating pain when I sneeze or try to lay down or try to stand up. And sometimes I m not so sure that it s not my heart. What the heck is goin on with me? Doctor: Hello and Welcome to \u2018Ask A Doctor\u2019 service. I have reviewed your query and here is my advice. There is a possibility of rib fracture at the site of trauma. This can give you pain on cough . Delayed healing can cause painful healing. Hope I have answered your query. Let me know if I can assist you further."
},
{
"id": 212044,
"tgt": "Mood swings, suicidal thoughts, anger issues, low confidence levels. Do I have depression?",
"src": "Patient: I think I have depression but i'm not completely sure of it. I'll have these moods where I feel like some kind of darkness just hits me. I space out a lot and just think horrible thoughts. Ways that I could kill myself if it ever came to it. I always think of what life would be like without me here. I annoy myself too easily. I feel like I have anger issues a lot because when I get mad I seriously feel like starting a fight and feel very violent. I used to cut and still occasionally do from time to time. I wake up every morning about 5 and then my day is off and I get upset too easily. I feel as if a lot of people have a hate toward me and my self-confidence is very low. Doctor: Hi,Thanks for using healthcare magic.I have gone through your available history.Your symptoms are suggestive that you are having depression. You should consult psychiatrist for detailed evaluation and management.Doctor will rule out physical causes like thyroid dysfunction or vitamine B12 deficiency. You may get benefit by antidepressant drugs like escitalopram or sertraline and counselling.You should also do stress management and relaxation therapy like breathing exercise for better control over depression.Thanks."
},
{
"id": 9545,
"tgt": "Dry, chapped lips especially around the edges. Taking Vitamin B and A tablets. Suggestions?",
"src": "Patient: Dear Dr xxxxx, my lips are dry for last ONE month and sometime I see cracks on my lips especially at the mouth edges. I have been using Vaseline for last 1 month regularly but I do not think it is improving as such. For last 1 week I have been taking Vitamin-B and Vitamin-A tablets (twice a day). Can you suggest me some medicines. Doctor: Hello dear. Thanks for the query. It is true that dry chapped lips specially at the corners is usually caused by vitamin B complex deficiency. However the response to treatment with vitamins is often poor. Usually a topical medication is required. Please apply oint mupirocine daily in morning and oint tess in night for 10 days. Along with this also keep taking vitamins. You will get good relief. Thank you"
},
{
"id": 187868,
"tgt": "Can painful gums and restricted jaw movements during wisdom tooth eruption indicate an abscess or infection?",
"src": "Patient: My wisdom tooth on my right side has been coming in for some time now. No problems. All of a sudden my cheek/gums started to hurt. Now it seems I might have an abscess..? And my jaw won't open all the way... maybe infection? I dont know what to do.. go to dr or what. I have no insurance... any advise would help alot!! Doctor: Hi,I have gone through your query.Eruption of wisdom tooth cause severe pain and inflammation.As it comes out, it tears the overlying gum tissue, resulting in inflammation with pain and restricted jaw movement. There are chances that it might get infected.I advise you gargle 3-4 times daily with chlorhexidine solution and take NSAIDs like ibuprofen.If still you have severe pain, i advise you to visit a Dentist. He may look for any sign of infection and if not, overlying gum will be cauterized with acid to relieve pain.Do consider a positive feedback as a credit to my work. Let me know if you have any further questions."
},
{
"id": 176726,
"tgt": "Suggest treatment for loose motions and rashes in groin area",
"src": "Patient: Hi my 6 month old has a bad diaper rash, looks like a brush burn kind of very bright red, kind of pussy. The last couple days he s had cereal and applesauce and peas and maybe bananas. Solid food twice a day formula maybe 4 times a day. He s on similac sensitive advance. He s had it about 3 or 4 days now. I just stopped using wipes and I ve been putting but paste on for the last day. He has no fever. And I ve been trying to give a bath after every couple poops. He s been going at least 6 times a day. What do I do? He hasn t had any breast milk lately because I stopped. Though he had his last bottle today. Doctor: Hi...It seems your kid is having viral diarrhoea. Once it starts it will take 5-7 days to completely get better. Unless the kid's having low urine output or very dull or excessively sleepy or blood in motion or green bilious vomiting...you need not worry. I suggest you use zinc supplements (Z&D drops 1ml once daily for 14 days) & ORS (Each small packet mixed in 200ml of potable water and keep giving sip by sip) as hydration is very important and crucial part of treatment. If there is vomiting you can use Syrup Ondansetron (as prescribed by your paediatrician).Regarding diet - You can use cerelac...any flavour will do. Avoid fruit juices as they might aggravate diarrhea. You can give zinc supplements & ORS apart from normal vegetarian porridges & soups.Regards - Dr. Sumanth"
},
{
"id": 38689,
"tgt": "What is the use of calvepen and chloromycetin inspite of having chicken pox?",
"src": "Patient: I have chicken pox . The spots came out this morning I am 40 years old. My little boy got them Christmas morning. I have a v sore throat and covered in spots even in my eyelids. The secretary at the doctors said not to come to the surgery as I would infect people. Medicine was left at the Doctors front desk for me to be collected by a friend. It is called Calvepen 666mg and the other one is Chloromycetin. i would like to know what they are for. Is one of them an antihestimine for the itch. If not what should i take that I can take for the itch something I can take over the counter. How long do you think i should stay out of work roughly Thank You Aideen Doctor: HiThank you for asking HCMI have gone through your query.The medicines which are given for you are antibiotics Calvepen which is a penicillin and Chloromycetin a chloremphenicol. They are given usually for sore throat due to bacterial infection.If you having chickenpox sore throat is due to that and antibiotic will give no help.For chicken pox if it is beginning i give my patient antiviral drugs like acyclovir or famcyclovir. I also give them antihistamine like cetirizine for itching.For soothing over the counter calamine lotion can be applied or the green dye which if available in your pharmacy.Hope this may help you.Let me know if you have any further query."
},
{
"id": 202321,
"tgt": "What is the hanging piece of skin under the penis?",
"src": "Patient: Hello i am 17 and i have an slight problem i cant stop masterbation and there is a little peace of skin under my penis,when i pull down the skin it hurts very bad,and i am also very scared to talk about it to my dad can you plz give me tips about it? Doctor: skin tagyou can get it surgically removedor stop touching it and it will healPlease rate 5 stars. I strive to provide the best answer to your questions."
},
{
"id": 56056,
"tgt": "Is the high range of SGPT a matter of concern?",
"src": "Patient: Hello, i have a question to ask, back in sept of last year i had some blood work done and was told that my liver enzymes were very high. Saw the test results and it showed that my sgpt (alt) level was 94. I stopped drinking since then, went from 245lbs to 233lbs and excerise on a regular bases. So i went tues to have blood work done before my follow up appointment and my Sgpt (alt) levels are now at 75. Is this a good thing that they have come down? and am i still at high risk even after changing my habits? Doctor: HelloFindings indicate that SGPT level is still increased.Increased SGPT may indicate liver injury.It may be due to many reasons like hepatitis,alcohol intake,altered lipid profile,medicines,auto immune causes etc.It may be related to alcohol intake in your case.You may need few more investigations like routine hemogram,random blood sugar,lipid profile,ultrasound of abdomen.I suggest tablet ursodeoxycholic acid 300 mg twice daily for three months to my patients.It helps in regeneration of liver cells.It is good that you have quit alcohol intake and reduced your weight.You should continue exercise.Get well soon.Take CareDr.Indu Bhushan"
},
{
"id": 140982,
"tgt": "Suggest treatment for klebsiella infection in spine",
"src": "Patient: yes Good Afternoon. My little brother has Klebsiella in his spine. He had three vertabrate hat were shaddered and they buildt a cage to hold his back together. It started to come apart in 2010 and they were going to replace it. But decided to just fix it fuse it. Now he has full blown Klebsiella he needs to have it removed and back surgery. No Drs want to do it. Told him get life in order. Are their any physicians that would do the work. Doctor: Hello, It sounds as if the surgeon considers either the infection or the surgery complex with a very high risk of either worsening of the Klebsiella or for some other reason worsening of your brother's status by trying to remove the cage or the previously fused vertebrae. You may try to contact an ACADEMIC MEDICAL CENTER in your area if you haven't already since many times they have superior technology or a wider knowledge base because of research that can handle tougher more complex cases than community-based physicians. Hope I have answered your query. Let me know if I can assist you further. Take care Regards, Dr Dariush Saghafi, Neurologist"
},
{
"id": 222441,
"tgt": "Suggest medication for constipation during pregnancy",
"src": "Patient: your health queries right now ? Please type your query here...During my pregnancy I got constipation. Because of constipation now I got cut on my asshole mouth. There is infection. I feel pain. What I do? Which medicine I use or what kind of doctor I visit. Doctor: Hallow Dear,The hard stools due to constipation has caused an anal fissure which has got infected, as you mention. First step should be to get relief from the anal fissure and get it healed. You may apply local applications like Practosedyl with finger to the legion after every motion and even otherwise also. This will help healing the wound and relieving you from the pain. Simultaneously, you have to get your constipation treated. Initially, try liquid paraffin at bedtime which will soften the stools. However, it should not be continued for long lest the risk of fat soluble vitamin deficiency. Get your stools examined for any evidence of Amoebiasis. If detected, you will have to undergo treatment for it. Metronidazole is an effective medicine for it. Consume lot of fruits, particularly citrus fruits like oranges in your diet. Avoid meat and eggs. These are constipating food articles. Regular use of Isubgul granules at bedtime with lot of warm water will help you get rid of constipation. Iron tablets taken during pregnancy are invariably cause of constipation. If you switch over to time released spansule form of Iron, it will not cause any constipation. I hope this helps you to find relief."
},
{
"id": 200159,
"tgt": "Reason for foreskin turning red?",
"src": "Patient: I an using Canestan cream to help on my penis and foreskin ,it help to pull back the foreskin,it was working well after using it for three days,the I use Baking soda to clean once only and I find that within two days after using the baking soda just once the foreskin got red to the tip and swollen a bit,i continue to use the Canestan cream for two days,I do see a small reduction in the red top foreskin,should I continue to use the Canestan cream or should I use some other cream. Doctor: Thanks for asking in healthcaremagic forum Penile and scrotal skin is very sensitive so, please do not use baking soda for cleaning. Warm water is enough for cleaning and canestan cream is an antifungal and you need not use it if you donot have fungal infection. The swelling would have been due to irritation of skin by baking soda. Please visit a doctor if not relieved. All the best."
},
{
"id": 149688,
"tgt": "Suffer Addison disease, Dr believe it's caused by depromedral injections, can't take Prednisone. Any alternative medicine ?",
"src": "Patient: I have Addison disease . Some of the Endo doc s I have seen believe the Addison s or Addrenal failfure (last Cort test 0 AM...0 PM) was caused from the depromedral injections I get in my spine about every 90 days. I can not take Hydrodcort, dexamythesone, prednisone . My doctor and I have even tried depromedral (may be spelled incorrectly) muscles injections. I have seen 4 Endo docs all tell me the same thing. There are only these three drugs for this disease. For 5 years I have suffered from takning the drugs and may possible die without them. I have searched, asked, prayed. I truly have no life. The engery level is low a shower is almost impossible. I went from a high entergetic person, owned my own business, worked out almost every day, my life revolved around giving and helping people. Now I can t help myself. If you know another drug (which is doubtful) do you know of some kind for experimental drug. I will try anything that might help. This is not someone playing a joke to see what will happen. My story is very true. If you can help in anyway please do so, but I have already tried the herbal treatments too. Thank you for taking the time to read this. AAAAA Doctor: Hello. Thanks for writing to us. Since you already have a hormonal imbalance, it is best not to take any oral or systemic steroids. The other alternative is local steroid injections at the site of pain to reduce the inflammation with mild pain killers.I hope this information has been both informative and helpful for you. You can consult me again directly through my profile URL http://bit.ly/Dr-Praveen-Tayal Regards, Dr. Praveen Tayaldrtayal72@gmail.com"
},
{
"id": 128570,
"tgt": "Is it possible to have diarrhea after a hit in the head?",
"src": "Patient: I hit my head after a night out 5 days ago, I was drinking alcohol but I do remember the night, but not hitting my head. I woke up with pain on the left side of my head, specifically above my temple. It hurt to eat on the left side of my mouth for the next 2 days. Yesterday, when I woke up I blew my nose and blood came out (although not a lot) and I have been feeling a bit sick, although I m not sure if this is due to worrying about it. A small lump did appear above my temple about 2 days after hitting my head, and my head is no longer painful to touch, but it doesn t feel odd and slightly heavy, although it isn t a headache. I have now had a bit of diarrhoea today and I m wondering if that is connected and whether I should be concerned. Thanks. Doctor: Hi,,in my opinion there is no link between the head hit and the diarrhea ,as having a GIT disturbance in form of diarrhea has nothing to do with the head injury,head injuries if dangerous can cause nausea or vomiting however this diarrhea could be due to eating something contaminated or drinking too much."
},
{
"id": 111161,
"tgt": "What kind of disorder does pain in lower back while bending forward suggest?",
"src": "Patient: I lifted a little weight by bending forward a few days back. Since then i have been suffering from pain in extreme lower back portion especially while bending forward. What is this disease called?Is it fully curable?How long will it take?What precautions should I take. I have been prescribed NUROKIND-G, , ETROBAX 120 AND MYROLILL. for 10 days? Doctor: Hi,It seems that due to heavy weight lifting you might be having strain and spasm of back muscles giving this problem.The medicine prescribed by your doctor is perfect and continue them.Apply some muscle relaxant cream locally.Ok and take care."
},
{
"id": 67126,
"tgt": "Suggest remedy for small lumps on my elbows",
"src": "Patient: I have some small lumps on my elbows. I have had them for a while now however they are increasing. The don t hurt or cause me any pain but they do itch sometimes but not all the time. My right elbow is a lot worse than my left. I am beginning to be very paranoid about revealing my arms in public which is quite a big problem as I am only 25. Doctor: Hi! Good evening. I am Dr Shareef answering your query.Although it needs to be examined clinically to know the exact nature of the lumps, from your history, these seem to be either olecranon bursa (a benign cyst on the elbow due to chronic friction), or could be part of multiple lipomatosis, again a benign subcutaneous swellings mostly causing cosmetic problem as in your case. If I were your doctor, I would advise you to consult a general surgeon, who might advise you for a FNAC (fine needle aspiration cytology) of the lump. Further management would depend on the reports which could be an excision biopsy to be very sure of the cytological diagnosis by FNAC.I hope this information would help you in discussing with your family physician/treating doctor in further management of your problem. Please do not hesitate to ask in case of any further doubts.Thanks for choosing health care magic to clear doubts on your health problems. I wish you an early recovery. Dr Shareef."
},
{
"id": 156954,
"tgt": "What causes pain around breast, nipple area, thick solid pus coming out with bloody discharge?",
"src": "Patient: Hi...I had to have my milk ducts removed due to calcification and discharge 16 years ago after my twins birth. My surgeon also biop the nipple tissue, neg. HUGE cancer risks on maternal father & mothers side. Everything from esophageal,bone,brain,prostate etc. No feeling sensation in that breast for 16 years. Over last month pain around breast,nipple-area and today thick almost solid like string pus came out with bloody discharge when examining my breast. My Hemotologist checked out site and ordered an ultrasound and followup with my surgeon for next week. Could this just be an underlying infection or possibly a pre-curser to ca. I do have scaring tissue but a lot of this came out. My Hemo Dr said he could feel something when he pressed around the area I could feel the pain. any help sugg? Doctor: Since you said pus came out along with blood, infection is a strong possibility. On palpation, underlying collection of pus or dilated ducts can be felt. but it is always better to investigate further and that is why you are requested for further tests. family history increases the risk but it doesnt mean you will definitely get cancer."
},
{
"id": 99735,
"tgt": "What to do for asthma bronchitis symptoms?",
"src": "Patient: I went to immediate med last Friday, the medicine for my bronchitis is not working. I CALLED MY DOCTORS OFFICE AND CAN T EVEN GET A CALL BACK. Went to the emergency room and waited 4 hours and never got seen so I left. Called my doctors office today and she won t be back until Monday, in the meantime how am I ppose to treat the asthma bronchitis symtons. Doctor: Hi, if u have diagnosed correctly that asthmatic bronchitis and u are suffering severe symptoms then one should not wait for that doctor who will come after few days,see its not big deal to treat asthmatic bronchitis as many good medicines available in market in different formulations,u go to another pulmonary physician and get prescribed, take it on a urgent basis but don't bother.u will be alright."
},
{
"id": 114315,
"tgt": "Can protein S deficiency be treated without taking blood thinning medication?",
"src": "Patient: I m 24 now, 23 when it all started, I was a smoker at the time & had been for a few years. I had a miscarriage before i started on the depo shot. I had one shot & didnt like it so i switched to the pill after about 3 months of being on the pill I ended up in the ER with a PE in my right lung they weren t sure what has caused it im not the healthiest I m over weight and don t eat the best. But im not lazy, 2 weeks before this I had been given a promotion to be a Store Manager. So i was super stressed out but working 70 hours a week, & not just sitting at my desk so they ruled that wasn t the cause. So they assumed it was a mix of smoking & birth control & stress ,tho they still thought i was too young. So they started me on blood thinners kept me for 4 days & released me to be on Xarlto for 6 months. With follow up appts & to a specialist who had a bunch of blood test ran. Turned out I have a protein S deficiency & the pills triggered it. Now I m almost to my 6 month mark & my doctor told me that in order to continue any form of birth control I d have to stay on the Xarlto. My bf can t do condoms & we tried the pull & pray method ie the miscarriage. I m not sure yet if I want kids so I don t just want to completely cut that option out just yet & I m def not ready for one now & if or when I m pregnant I have to be on the blood thinners throughout the pregnancy. So either i stay on both pills or just stop having sex completely.... My main concern is staying on the blood thinners for a long period of time, it s been affecting my menstrual cycle the past 2 months & I still feel run down & tired alot of the time which i know is part of the clot. I m just wondering if I have another option in this. I don t mind the pill for the most part expect I m cold 24/7 now. Can u give me some advice plz? Doctor: Hello and Welcome to \u2018Ask A Doctor\u2019 service. I have reviewed your query and here is my advice. It is necessary to be on blood thinning medication for treating protein S deficiency. Not only pregnancy but it may cause many other life threatening complications like stroke, MI, bleeding disorders, DVT, VTE, etc. It is always better to be on the drug. Also, check your homocysteine levels. Thank you."
},
{
"id": 143608,
"tgt": "Could the loss of short term memory be a sign of vasospasm?",
"src": "Patient: My mom had a bad headache last thursday. She had a cat scan on Wednesday and they found a significant brain bleed on the right front side. She is awaiting a bed at a hospital with a neurosurgon, but in the mean time she is in icu. She has no stroke symptoms other than loss of short term memory. In the last 12 hours, that is becoming sigificantly more noticeable. The staff are great but when I mention that, they just say it is probably because she is tired. I read up o vasospasm. Could it possible me a sign of that? Doctor: Hi,your mother had severe headache due to right frontal bleed.Firstly she should be seen by neurosurgeon and treatment decision should be made whether conservative or surgical treatment should be done.Loss of short term memory in this case is caused more likely by increase in bleed and surrounding oedema.She should be done repeat CT head .Thanks"
},
{
"id": 180543,
"tgt": "Is medical advise required for a locked jaw?",
"src": "Patient: First I must say I am under a lot of stress --- putting my mother in a nursing home and dealing with other family matters today while getting her room set up in the nursing home my jaw moved out of aliment an \"locked\" it was only for about 1 to 2 minutes. Is this something I should see my primary about Doctor: Hello and Welcome to \u2018Ask A Doctor\u2019 service.I have reviewed your query and here is my advice.You had a jaw joint dislocation that has led to locked jaw or trismus. Many times the ligaments around the jaw joint become weak and make the jaw joint to dislocate easily. The jaw joint is reduced to its appropriate place by some manipulation, but then it becomes prone to frequent dislocations. If you are in severe stress and tend to grind or clench your teeth then you are more prone to jaw joint dislocation. So in case if this problem happens again you should consult an Oral Surgeon and get checked. A panoramic X-ray can be done to rule out the exact cause of the dislocation and treatment can be advised accordingly. You can be advised mouth guards to be worn at night, physiotherapy and stress relieving exercises and meditation for relieving the symptoms. Hope I have answered your query. Let me know if I can assist you further.Regards, Dr. Honey Arora"
},
{
"id": 91331,
"tgt": "What causes discomfort feeling on upper left abdomen?",
"src": "Patient: Hi i have been having this discomfort feeling on my upper left sbdomen right by the rib cage.. It feels relieved when i burp but it comes back again. Dont really know what helps it but its been going on and off for months. Doc said not spleen related.. Anything enlarged? Doctor: Hello!Thank you for the querySpleen does usually does not give any pain.Your symptoms seems to be caused by stomach issues like peptic ulcer, stomach inflammation or hiatal hernia. It is more possible if you have also chest burning, nausea. The symptoms are usually associated with foods intake. If you drink lots of coffee or have fatty foods often, this can trigger your symptoms.Please remember that such located pain can be also a symptom of heart issue. Heart pain usually radiates to the left arm or jaw.I suggest you to visit your doctor, have blood work and abdominal ultrasound at first. If this tests will be negative, gastroscopy should be next.Hope this will help.Regards."
},
{
"id": 161092,
"tgt": "Suggest treatment for child suffering from achondroplasia, constipation and small rectum",
"src": "Patient: Hi, i have a 14mth old baby girl who has achondroplasia and she keeps getting constipated, it has got to the point where she is straining to go toilet, there is a little blood when she does go and it is hard, she arches her back and i have helped her to go and i also noticed her rectrum seems to be smaller, pls help. Doctor: Hi, Suggestions: 1. Natural methods are the best to relieve constipation. 2. Constipation is a risk factor for UTI 3. Maximum milk consumption per day should not exceed 300-400ml 4. Minimum 3-4 cups of fruits and vegetables to be consumed per day. Papaya is the best natural laxative. 5. Toilet training - that is - sitting in Indian type of lavatory daily at the same time will help a lot. 6. 1 liter of water per day. Hope I have answered your query. Let me know if I can assist you further. Take care Regards, Dr Sumanth Amperayani, Pediatrician, Pulmonology"
},
{
"id": 195375,
"tgt": "What causes red spots at the base of penis shaft?",
"src": "Patient: Less than 2 days after having unprotected sex without climax (still poor judgement), I noticed some red spots at base of shaft. Didn't notice anything else but maybe didn't examine better. Later that night I masturbated, and afterwards noticed some random red dots on the head of my penis, still wonder if they were there before masturbating, I didn't notice them. Very small, don't seem clustered. Not Raised, no itching, nothing noticeable at all. I just can see them. Doctor: Hello and Welcome to \u2018Ask A Doctor\u2019 service. I have reviewed your query and here is my advice. Red spots on the penis may be seen in pressure induced allergy patients. After vigorous masturbation it lead to release of histamine locally lead to induration. Nothing to worry, apply less pressure. Use antihistamine before masturbation. If symptoms not improved please consult your doctor he will examine and treat you accordingly. Hope I have answered your query. Let me know if I can assist you further. Regards, Dr. Penchila Prasad Kandikattu"
},
{
"id": 30262,
"tgt": "Suggest medication for flu causing diarrhea and nausea",
"src": "Patient: back in november I had come down with flu like symptoms, diahrreah, nausea, vomiting and fatigue! Ihad been working and relapsed in december! I went to see my doctor and he had put me on 10 500mg pills of levaquin and nexium and 2 weeks of rest! he had drawn more blood and my wbc was at 15.4 and my vitain d levels down to 16! he sent me to a hematologist and the hematologist said it was propably a viral infection and he couldn,t do anything and it would propably take me another month to recover and my wbc should be back down to normal! He drew 2 vials of blood! A week later I had to see my doctor again and my wbc count was still at 14.3 nd my nuerophils were at 12.8! My doctor drew a vial of blood and my wbc was at 13,000, i asked after a month and a half of resting why it had taken so long to drop and he said the cells were reacting to something! 2 small nodules were found in my right lung, one 5mm and one 2.5 mm! I saw 2 respiratory specialists and they said they were to small to cause an illness and were to small to do anything with! My mother had one 5mm nodule in her lung and some years later it turned malignant! I still get fatigued and at times feel like I am coming down with a cold! What could I have caught and could more have been done? Doctor: HiThanks posting on HCMViral infections may give you the symptoms you describe.However, viral infections may not have any particular treatment and symptomatic treatment/rest and healthy diet will do.The small nodules may pose no problem yet but should not however be neglected.It could just be reactional lymph nodes if mediastinal.A past history of lung cancer in your mother predisposes you to developing it but its still too early to conclude as other arguments will be needed to diagnose.A CT scan or MRI may help.I will suggest you get closer to your lung specialist for regular follow-up.Hope my answer will help youBest regards"
},
{
"id": 93412,
"tgt": "Severe endometriosis, secondary hysterectomy & appendix removal. Under no medication. Treatment?",
"src": "Patient: Hi there, Can't sleep till I get some answers. Thank you for your help I'm advance. Lost my health insurance & I'm rather concerned. Hx.:-Hpv -Severe endometriosis & appendix removal with no relief then secondary hysterectomy 3yrs ago. Some endo left on cecum due to -risks. Discovery of intersistal cystitis during surgery. -6 mo. Later aquired human parvo virus B19-Positive ANA test but was inconclusive for lupus.-Since all of the above I have continued to have IBS, migraines, daily generalized pain, joint pain( difficulty making a fist), low back pain (si area), extreme fatigue( unable to work/ some days even leave the couch) , becoming depressed w/chronic pain & my 2 girls do not have their Mother playing & keeping up with life. So here's the big scare... ***** last 3 weeks I began to feel the very familiar pelvic heaviness, continuous twisting/clenching with zero relief from pain meds. This was my daily life with endo so I'm very aware of the feeling. *****2 days ago my vulva/ upper area only, began to intensely itch. No odor or discharge. Woke up 2 mornings in a row with intense groin & hip pain that is deep & numbing. I had enough & started to inspect the vaginal area. >>I discovered a small, round, flat ( not oozing or raised) white \"ulcer\" looking lesion. There are 2 VERY small ones under the the large one. They do not hurt superficially but I feel pain under the lesion. No pain during urination even though it is in the direct path of urination. *same sexual partner for 2 yrs. last sexual contact 10 days ago. Examined partner & nothing visually noticed. I do have a photo if you need. Again, thank you! Doctor: all these are harmone related problems which are increasing time to time and this fits into milk theory as the human and animal proteins are not competible there is reaction of human harmones with bovine armones of animal female milkif you really want reliefwithdraw all milk and diary proteins completely from diet even soaps shampoo creams containing milkyou ecpect improvement in 3 wk to 3 monthsget blood serum tested for food specific antibodies for milk wheat potato chana rice nuts eggand other food you take dailywithdraw foodsit may be 1-3\\but start milk withdrawl immediately gey sympatomatic rx till thenif you can give photo to sumankumardr@hotmail.com i can suggest some sympatomatic rx for that"
},
{
"id": 11967,
"tgt": "How to get rid of the dark patches on my forehead, cheeks and hands ?",
"src": "Patient: i am 38 years ,78 kg.i am suffering from irregular periods since teenage.i use to have periods in15 days interval.i took novelon tablet for years.but now for 3 years i am not taking any medicines.now my periods r almost regular but i am having very painful periods and clots comes out.alternate months the bleeding is heavy.now the problem is -at times its irregular with pain and clots and am having dark patches on my forehead, cheeks and hands.please help me get rid of these patches. Doctor: Hi,Soma, Thanks for query, As you have irregular period since long and having dark patches on the body. Go for Thyroid test in hypothyroid this type of complain is common. Go for ultra sound . After reports go for treatment accordingly. Ok and bye."
},
{
"id": 111082,
"tgt": "Suggest remedy for back pain and tiredness",
"src": "Patient: hi i am mohan i am suffering from the following probs 1.lower back pain.I started feel this pain from 10nth std.I had taken MRI Scanning and doctor said no probs and he gave me some vitamin tablets.2. somtimes pain on middle of my back. 3.i am getting tiered offten.my weight is - 55 and my height is - 5ft I am taking any medicines,so plz advice.thanks and regards,mohan Doctor: hi ...tell me when you are having more pain . are you IT person ? or working long time in front of computer? or you have wrong sitting posture. so i would suggest you to do some exercise. lie on flat bed as such your chin touch your bed and your back face your ceiling . then slowly take breath and lift your head and strech your arm too full but dont lift your leg and take your head as much back you can hold that position for 2 min then again lie down in rest position .do 3 to 4times .....and see for the change"
},
{
"id": 43434,
"tgt": "Spillage of contrast seen in right tube. Left tube blocked, shows hydrosalpinx. Chances of pregnancy?",
"src": "Patient: I have a question about infertility . the report says that their is a small spillage of contrast seen on the right side. Left tube is blocked at the fimbrial end and shows Hydrosalpinx . the patient report also SPECIMEN SOURCE: MENSTRUAL BLOOD MYCROBACTERIAL DNA PCR: DETECTED GENUS: MICOBACTERIUM SPECIES: MICOBACTERIUM TUBERCULOSIS COMPLEX CAN THE PATIENT GET PREGNENT? Doctor: Hi,Thanks for your query. I read your query and I understand your concerns. Following is my reply:1) You have to take anti tuberculosis medicines for 4 to 6 months and then test again to see if you are cured of TB.2) You may have to go for IVF later as your tube seems to be badly damaged due to TB. 3) Please start anti TB treatment soon I hope I answered your query. I will be available for any follow up query you have.Regards,Dr. Mahesh KoregolIVF & Infertility Specialist"
},
{
"id": 211570,
"tgt": "Insomnia, memory loss, muscle twitches. MRI shows sinus blockage. Taken antibiotics. No periods",
"src": "Patient: I m a 20 year old female and 3 months ago after smoking a hookah for 3 nights in a row I developed insomnia , memory loss , muscle twitches, cognitive decline much like brain fog where I cant think straight or quickly, depression , anxiety and visual disturbances. Two weeks later I had an MRI done which came back normal except for total blockage in my sinuses, I was given steroids and antibiotics but my condition did not improve. I have had a an abdominal and pelvic CT scan and everything was normal. My blood work was normal except for elevated white blood count, elevated proteins , low vitamin D . This past summer I was diagnosed with gastroparesis , non alcoholic fatty liver disease and polycystic ovary disease, I have not had a period in 5 years. I recently had an EEG done by a neurologist and it was abnormal, he said I had diffused slowing and spikes in the background. He thinks I may have had an acute case of bacterial meningitis due to my severe sinusitis and these are the after effects. His final diagnosis was metabolic encephalopathy, he scheduled a follow up sleep deprived EEG to rule out a seizure disorder. I m afraid it something worse like Creutzefeld Jacob Disease. I have no family history of neurological illness. I still have the sinusitis and am taking estradiol and progesterone. Doctor: In addition to the advise by neurologist, in my opinion, you need to consult an endocrinologist who will estimate your hormonal profile like thyroid level, estrogen, progesterone FSH etcAlso, important to consult psychiatristContinue taking vit D till it comes to normal level or as advised by doctorCreutzefild jacob disease, to me seems less likely"
},
{
"id": 115731,
"tgt": "How should a low count with tiredness and weakness be treated?",
"src": "Patient: i have been to see my gp and bad a blood test he explained i had a low count on the chemical side to my imune system so i was sent to see a specialist he has done bone marow sample and a ct scan but i am getting very tired and low and very week i have been back to see my gp but could only see younger female dr who looked up on how to sleep on the internet and printed it off for me it is not the answer i need Doctor: Hi, dearI have gone through your question. I can understand your concern.You may have low rbc count or wbc count or pancytopenia. You should go for PS examination and bone marorow examination. If you have low hemoglobin count then take iron and VIt b12 temporarily till your bone marro report done. then take treatment according to cause.Hope I have answered your question, if you have any doubts then contact me at bit.ly/Drsanghvihardik, I will be happy to answer you.Thanks for using health care magic.Wish you a very good health."
},
{
"id": 9538,
"tgt": "Welts on the back of the thigh with itching. Remedy?",
"src": "Patient: My husband has a large callus like lots of stripes or welts on the back of his thigh from years of his military airborne uniform leg strap rubbing. I would like to know how to help relieve the rough skin, mild occasional itching and have his leg eventually feel more normal and soft and flesh like. I can t find pictures online to liken his issue to get further advice. THANKYOU Doctor: Hi From the description it seems your husband is having persistent friction induced lichen simplex chronicus. The same can be treated by moisturising the area many times with a urea or lactic acid based moisturising cream along with application of steroid salicylic acid ointment. If the lesions are too itchy, antihistamine like cetirizine may be taken. Hope that helps. Take care"
},
{
"id": 129901,
"tgt": "Suggest treatment for leg pain after a miscarriage",
"src": "Patient: i had miscarriage like from a week ago..starting bleeding like period on 27 for 6 days.. I have now constant leg pain..my knees, back of my leg and my calves...even when i stretch or bend my legs , i wake up in the morning with that pain and no improvement at all, please advise Doctor: it might be a deep vein thrombosis ..get an venous doppler of full leg and consult a surgeon . . don't ignore ur symptoms act fast"
},
{
"id": 13008,
"tgt": "What causes a rash,peeling skin and swollen face after taking amoxicillin?",
"src": "Patient: sinus problem was given amoxcillian for 21 days 500mil after completing treatment about 48 hours later broke in a rash swollen face hot to the touch and lips that were chapped and skin peeled and hives extremely itchy. Is in possible to have a delayed reaction to antiobotic Doctor: Hello, I have gone through your query and it seems to be a drug reaction to amoxicillin. I would recommend you to consult a doctor and get it evaluated. Hope I have answered your query. Let me know if I can assist you further. Take care Regards, Dr Asmeet Kaur Sawhney, Dermatologist"
},
{
"id": 128835,
"tgt": "What does sharp pain in the heel indicate?",
"src": "Patient: I have a pain on/in my left heal. It started when I had a long hard day on my feet. Since then it comes and goes. Usually when I stand first thing in the morning it is a sharp pain that after some hours on my feet it dulls somewhat. I initially thought I brused my heal, but it gets no better and thankfully it isn t getting worse. Doctor: In all probability you are having plantar fasciitis of the left foot. That is the reason of your these symptoms. One important treatment is the use of very soft insoles, pressure distributing insoled foot wear.An example is the sports shoes like Nike, Adidas, and the like. You can get such insole inserts of EVA, Plastazote, Silicon, Poron from special foot wear institutes, and please wear these when you are on your feet. Never use hard insoled foot wear or walk bare foot.The other aspect is to gradually stretch the plantar fascia, which is in a contracted state to is normal length in about 4 weeks, when pain will disappear. There usually three plantar fascia stretching exercises which can be got from the internet, or a Podiatric surgeon can tell you, should be regularly done morning and evening for 6 to 8 weeks at least."
},
{
"id": 181763,
"tgt": "What causes bitter chemical taste in the mouth after root canal?",
"src": "Patient: I have a bitter chemical taste in my mouth after the first stage of a root canal filling. Is this normal? Is it due to the chemicals used to clean the tooth or not all of the infection was removed and it is leaking into my mouth? The taste isn't removed with eating, drinking or mouthwash. Doctor: Hello.Read through your query and accordingly I must say that it is just usual to have change in taste after the rootcanal trearment.Its due to the medicines used to clean the tooth and oral cavity.It will be reduced gradually within few days after treatment.You should drink lots of water and also do chlorhexidine mouthrinse so that it will be tasting little pleasant.Hope you are benefitted something from this reply.Have a healthy day!!!"
},
{
"id": 8385,
"tgt": "How can sagging breasts with darkened nipples be treated?",
"src": "Patient: hi ,,im 25 yrs old girl,,i hv saggaing breasts with darker nipples also i hv hormonal prblm i hv PCOS since 4 yrs. so i wnt to kno about any cream or oinment which makes my nipples color pink or lite then darker color. please suggest me any halmless cream for my nipples,, and also suggest me some medicine for tightning / firming my breasts. Doctor: Hi,Sorry to say that sagging of breasts and firming is not possible by medicines. As far as the skin darkening problem is concerned it is an offshoot of your PCOS problem.You will have to visit your gynaecologist and start PCOS treatment which will help with the skin darkening. And once that is started you can meet a dermatologist or plastic surgeon for some skin lightening creams for the dark areas. But without treatment of your PCOS the darkening will come backAll the bestDr.Surindherhttp://www.cosmesisindia.com"
},
{
"id": 163779,
"tgt": "How to increase weight of a 7 months old?",
"src": "Patient: Hi. My baby is 7 months old and weighs 4.4kgs only. Her birth weight was 2.9kgs. She is active & playful. We have done full blood test, CT scan, Eco-cardiogram & stool tests for her. All reports are normal. Please let me know whom i can help her gain weight. Doctor: Hello!The weight of a normal child of that age should ideally be 7-8 kg. That's almost 3 kgs of weight she is lacking.If all the reports are normal as mentioned, there's some suggestions you can follow.1) Make sure she eats properly and the amount of breast milk she takes is adequate (Yes breast milk should be given along with baby foods) 2) The food which you're giving her is healthy and easily digestible. Ceralac is good for children.3) Little children are sometimes notorious and may not eat their food properly so make sure she eats adequately well.4) Few children may develop abnormal habit of eating things like mud or chalks (this is called pica) so make sure she's not consuming such things.I hope these suggestions would be helpful for the little girl.God bless you and your daughter.Best regards,Dr Gunjan Jain"
},
{
"id": 79279,
"tgt": "Suggest treatment for cough caused by antibiotics",
"src": "Patient: I had a tooth filled sept 24 it abcessed I developed a cough shortly after was put on antibiotics had to have tooth pulled last wk but still had chronic cough ..I'm very asthmatic nothing is working for cough a d now metallic taste aftertaste when I cough Doctor: Thanks for your question on Health Care Magic. I can understand your concern. You were having infected tooth and this can cause aspiration of pus in the lungs and development of lung Infection. So we should get done chest x ray to rule out lung infection as a cause for your coughing. Your metallic taste is due to antibiotics you are taking. If chest x ray is normal than no need to worry much for cough. Take antihistamine and anti inflammatory drugs for cough suppression. Hope I have solved your query. Wish you good health. Thanks."
},
{
"id": 212076,
"tgt": "16 year old is often sick, depressed, hardly eats. Any ideas ?",
"src": "Patient: My 16 year old son is often sick, has spent most of the holidays asleep, has few friends and we wonder if has depression. Last Nov he injured his knee playing basketball. This required a total knee reconstruction and a year off sport. He used to play a lot of basketball and rugby. He has hardly been eating also which is why I think he is know down with a cold. Doctor: Hello Welcome to health care magic I can understand your situation. I appreciate your effort to seek online support. As per your details, he is possibly having underlying anxiety/depression. For this consult a psychiatrist for detailed evaluation. Following treatment strategies would be helpful- 1) Antidepressants/Antianxiety agents like SSRIs (escitalopram, sertarline, paroxetine etc) 2) Antianxiety agents like clonazepam, lorazepam, alprazolam etc3) Psychotherapy/ Counselling sessions4) Relaxation techniques like yoga, meditation, deep breathing etc Hope it helps Good luck Regards Dr. Gourav Monga Consultant Psychiatrist"
},
{
"id": 105486,
"tgt": "Red pin prick dots on arms, slightly raised, itchy. Due to my pets?",
"src": "Patient: yes thk u... I have had these pin prick red dots that are in clusters that have been on my arms front op of arm front for 6months... Sometimes they itch very slightly... sometimes thy appear very slightly raised.... and they have increaesed very slightly down onto the edge of my hands ????? They also sometimes appear slightly more red than at other times ??? I have ngever had any skin problems before. I have a cat and a small dog. Doctor: Hi..as per history u given its allergic related to pets, better to avoid contacts with pets. Pet hair is not an allergen. People with pet allergies are allergic to proteins in pet saliva, urine or dander, dead skin that is shed. Nonprescription (over-the-counter) remedies include anti-itch creams containing camphor, menthol, pramoxine (Itch-X, Sarna Sensitive), or diphenhydramine (Benadryl); antihistamines like diphenhydramine, chlorpheniramine (Chlor-Trimeton), or loratadine (Claritin, Claritin RediTabs, Alavert); and cetirizine (Zyrtec); moisturizing lotions. \"Avoid AVOID CONTACT WITH PETS\" Regards. Dr.Syed"
},
{
"id": 21030,
"tgt": "What causes sudden rise in BP of a 58 year old?",
"src": "Patient: My sister (white, 58 years old, no children) has high bllod pressure and takes BP medication (lowest amount). She takes her BP at home in the morning. Today it was 150+/90+ and her husband took her to the hospital and they are keeping her over night. She also had an odd sensation in her breast area (probably some kind of mild chest pain) Her EKG showed no sign of an MI; they ran some tests and said that her tryptophan (?) levels were normal too but they could change in ~ 6-8 hours so they want her to stay overnight, to monitor her and to take blood levels again. Thoughts on what could be causing this? I am no alarmist but I am somewhat worried. Doctor: the blood test ordered was a troponin. it was to check for heart damage, which would be caused by a heart attack. you sisters symptoms are not completely consistent with a heart attack, so it is unlikely the cause of her symptoms, but it is good to rule it out. if her blood pressure is regularly this high, it should be lowered to prevent an increased risk of heart attack and stroke over the long term. the funny sensation could have many causes, muscular or anxiety related potentially"
},
{
"id": 161381,
"tgt": "What causes swollen and red forearm in a child?",
"src": "Patient: My 8 month-old daughter has a swollen and red right forearm. We noticed it about 1 and a half weeks ago but thought she may have strained a muscle doing row-row the boat with an over-enthusiastic 3 year-old. It seemed to improve but today it looks worse again. Apart from that, she is fine in herself, the arm doesn t seem painful to touch and she is putting weight on it when she crawls, etc. What could this be? Thank you. Doctor: Hi, If a similar patient comes to me, I would first check whether she is in pain and the area is tender. Whether the child is playful and not irritated. As you have mentioned it is completely normal, I feel there is nothing to be tensed about. She might have had an insect bite or hit her hand somewhere. Mostly it may fade away by itself. If within a week it doesn't resolve or if it is increasing, kindly consult your doctor. Hope I have cleared your question. Let me know if I can assist you further. Hope your daughter recovers fast. Regards, Dr. Jilu Joseph, General & Family Physician"
},
{
"id": 132623,
"tgt": "What caused pain in the joint of my middle finger?",
"src": "Patient: Hi. Three days ago the joint closest to the end of my middle finger became very very painful. I don t remember hurting it all, but it feels injured. The pain is not dull but very sharp like a knife pain in the joint, whenever I bend that joint. I ve put medical tape on it to keep me from bending it and that definitely helps. However, I don t notice much of an improvement yet and am wondering what could have caused this? I don t notice any redness or swelling to speak of either. Doctor: Hi Hope this message finds you in good health.I have gone through your complaints and understand your concern.u seem to have strained a muscle tendon or soft tissue in that region unknowingly.rest,analgesics,hot fomentation shud help.Nothing to worry about.\u00a0\u00a0\u00a0\u00a0\u00a0I hope your question has been answered.If you have any follow-up queries,feel free to consult me anytime.Thanks,Take care,God bless."
},
{
"id": 8494,
"tgt": "How to get fair skin?",
"src": "Patient: hi doctor, my face, neck and arms(exposed parts) are darker than rest of my unexposed part of the body which are as fair as paper white. i used to play a lot during my childhood under the harsh sun and my parents didn't care about the damage made by the sun. in my family i am the only one who has dark complexion on my face and i feel total odd about that. when i was at class 4, i stayed in hostel for 1 year and during that period, i turned out really fair and i think that was due to that i didn't get exposed to outer atmosphere.Later on after leaving the hostel, my face get started darkening.n Now my question is that can i reverse the colour of my skin and make it like that of my body. i am 21 years old .My height is 5'7\" and my weight is 69 kg. plz help i feel really odd.... Doctor: Hello,Thank you for posting on HCM.The darkening of skin of exposed areas is due to sun tanning.Generally, in my patients of sun tanning I would advise then to get series of microdermabrasion followed by chemical peeling. Microdermabrasion is a procedure that removes the superficial layers of skin and it is done with a help of machine. Then after a week or same day itself I would carry out another procedure called chemical peeling. Chemical peeling is again a similar procedure where superficial layers of skin is removed but here the effect is more profound and it stimulates new skin generation in much better way thus giving you more youthful and radiant skin.Several options are available in chemical peels like glycolic acid, salicyclic acid, TCA but my personal favorite is retinol peel or also known as Yellow peel.The rationale of combining these procedure is to get better and long lasting effects.I am sure 6-8 sessions will bring out a pretty gratifying results.At home the results can be maintained with antioxidant tablets once daily in morning(It can be safely taken for months together) and night application of cream containing mixture of glycolic acid , arbutin and Kojic acid. In morning you can apply a good quality sunscreen with high SPF.Use a good quality facewash like Cetaphil cleansing lotion.Take plently of fresh fruits and vegetables in your diet and indulge into regular exercise for healthy skin and body.Hope this was useful.Let me know if I can be of any more assistance.Thank you and take care,Dr Hardik PitrodaM.D Dermatology"
},
{
"id": 90580,
"tgt": "Suggest remedy for abdominal distensions",
"src": "Patient: Sir I had abdominal distension and fullness sincce5 days,intermitant episodes of indigestion and malaise since 1 mth, wt loss about 10 kgs in 4 mths,I was thinking it 2 be a indigestion entity,but, yesterday I had a bout of severe lower abdominal pain lasting for half a minute or so,after this I had a check ultrasound which showed, moderate ascitis ileal and omental thickening,lymp nodes with loss of hilum and caseation largest ms 4 cms. Now today - am getting intermitant bouts of crampy pain lasting for about 15 second,I had 3-4 such bouts in entite day 2 day,I have send blood for routine investigations ,pls advice me regarding any specific diagnostic tests to be done for adequte diagnosis before I consult u personally,whether a therapeutic tap is imediately indicated and what should !e my plan of action? Thanking u - mr.p.gharat Doctor: Ascites could be of varied reasons if its so minimal less than 50 ml tapping is not necessary is its more diagnostic tapping can be done. Lymph node enlargement can be tubercular mesentric adenitis. I would suggest a ct abdomen to rule out few causes. Its better to consult tour physician at the earliest"
},
{
"id": 22884,
"tgt": "Can increased blood pressure be due to the medications?",
"src": "Patient: My mother aged 70 is on medication for BP. She is being administered the following medicines: cardace 5mg twice a day tonact 20 at bedtime loprin 75 once daily shelcal 500 once daily Around five days back two medicines have been added deecee forte & Etova ER 400 BP in morning is giving increased reading in the morning 140/105, 140/96, 140/98 for the last three days one medicine was added two days ago for BP (asomex 2.5 mg) Would like to know why the pressure is increasing in the morning. is it a side effect of any of the drogs mentioned above ? Doctor: hi, Etova 400 can sometimes may affect blood pressure control. Anyway it a pain killer drug and should be consumed on whenever needed basis, however this issue can be discussed with local physician. other medicine seems fine.Anyway he should have low salt and low fat diet along with regular exercises, which will also help."
},
{
"id": 31340,
"tgt": "Suggest medication for an infection in the stomach",
"src": "Patient: Hello, good, the first thing to ask for excuses him since I am speaking to him across a translator since I am A SPANISH contact with you because I am slightly worried since I have enough indications of small quantity has consumed gas of the burners approximately 6 months ago, nowadays I am not badly in general, algun small, prick in the zone of the bladder or in the mouth of the stomach, but very slightly and during a little time, some time ago if me dolia but not tape-worm either vehicle or means to go to the hospital in this moment and the following day I was perfectly and I was not, wise either that I tape-worm this substance inside. Nowadays I have a small sapullido in the low zone of the gut, and one in the neck, and the hunger like that it is mas long, thank you very much in advance and wait for reputtings, One Doctor: Hi.I would first recommend a deworming course of either albendazole (400 mg orally, once; followed by another tablet in 2 weeks time) or mebendazole, which ever is available in your region. The next thing I would recommend is a course of antibiotics. I too apologise as the translator was not effective enough and I could not understand your query clearly enough.In antibiotics, I would prefer the use of ofloxacin 200 mg oral tablet, to be taken twice daily for the next 10 days. You can also use either ranitidine or omeprazole tablets 15 minutes before each meal for 10 days.Best wishes."
},
{
"id": 176130,
"tgt": "Suggest treatment for cough and cold in a child",
"src": "Patient: Hi, My son is 5 years old. He frequently (Once in every month) has cough and cold and every time doctor advise him to take antibiotics like Phenix with other cough and cold serups. Last week we came to know that with cough and cold he has also developed the uring infection and according to the culture report of the urine most of the bacteria found in the urine are resistant to the antibiotics. Our doctor has advised us to go for another urine culture test before starting the cure for the same. Please advise me what is best for my son in this situation. Doctor: Fever and cough are very common ailments in children. Most often it is caused by certain viruses. Some children are more prone to catching cold but the propensity comes down with age. The treatment of fever and cough is use of plain paracetamol to reduce temperature and use of some antiallergic like cetirizine. Antibiotics are generally not required as they are ineffective against viruses. It should recover on its own in 5-7 days. Steam inhalation should be helpful in diluting the cough.Urinary tract infection (UTI) on the other hand is a primarily bacterial infection. Please note that mere presence of bacteria in urine needs to be greater than 10^5 (100000) per cubic mm for the diagnosis of UTI. If that is not the case but still bacteria are present, a second urine culture is sometimes advised. If the sensitivity report shows resistance to most antibiotics, surely there should be at least a few higher antibiotics like colistin that are still sensitive. If the sensitivity of those have not been checked, a second culture could have been advised. The best treatment for your son would be to administer proper antibiotics as per your doctor's judgement. Please rely on it."
},
{
"id": 198019,
"tgt": "What could cause itching on scrotum, allergy and fungal infections ruled out?",
"src": "Patient: i am 62 yrs old. married and had no sex with other women for more than 25 yrs.I had itching in my scrotum for 2 months now. the doctor allergologist gave oral and topical meds but to no avail. i had lab scraping of scrotum for fungal infection check but it was negative. there are no rashes either. what is the diagnosis? what should be the treatment doc? Doctor: Hi,Thanks for writing in.If you have itching in the scrotum and allergy and fungal infections have been ruled out then chronic skin conditions need to be evaluated. Some people have dryness of skin in elderly age and this can manifest as scaling and itching in the cold weather. It can be a hypersensitive dermatitis to any soap or clothing which has gone unnoticed. Any chronic bacterial infection in the area is also to be investigated.Please apply steroid containing ointments over the itching areas and watch for relief. It is important to consult a dermatologist and get clinically examined. Please keep your genitals dry and avoid wearing tight clothes to prevent sweat formation. Please do not worry."
},
{
"id": 118494,
"tgt": "Have varicose veins and recently noticed a hard and dark lump in lower leg. What could this be?",
"src": "Patient: Hi I have noticed a hard lump on inside of my lower right leg which appeared a few days ago - it feels about the size of a pea! It looks very darkish in colour. I do suffer with varicose veins and have had a two ops many years ago stripping the veins! I have no pain with the lump and no tenderness. Doctor: Hi,I can understand your concern for the recently noticed a hard and dark lump in lower leg and varicose veins.After reading your query it appears to be swelling related to varicose veins only. There is swelling of vein and formation of lump on chronic standing or due to more pressure in the veins. The vein can swell up and gives a lumpy appearance.Do not pinch, pop or scratch or even touch the area unnecessarily as it can cause aggravation of the rash. Do not even try to see the area and touch it again an again in anxiety.I further suggest you to meet a surgeon at the earliest possible so that it should be managed best.Take care."
},
{
"id": 141346,
"tgt": "Suggest treatment for weakness in the legs and headache",
"src": "Patient: I am having a lot of weakness in my legs and dizziness/headache etc. I have a retinal occlusion that has been bothering me and being treated for several months with no success. It doesn t seem right. How do you tell if there is something else I should be looking at? Doctor: Hello and Welcome to \u2018Ask A Doctor\u2019 service. I have reviewed your query and here is my advice. I would explain that your symptoms could be related to vertebro-basilary insufficiency or a metabolic disorder. For this reason, I recommend performing a Doppler ultrasound of the carotid arteries and some further tests: -complete blood count for anemia -blood electrolytes for any imbalances -kidney and liver function tests -fasting glucose. A brain Angio MRI may be needed to investigate for possible vertebro-basillary insufficiency. Hope I have answered your query. Let me know if I can assist you further."
},
{
"id": 188733,
"tgt": "Want lower molar, front lower teeth replacement. Is there any non-invasive, non-surgical method of replacement?",
"src": "Patient: i have my I lower molar of both side and front lower teeth except canines missing since 1 year when i got them extracted due to severe caries. i showed to may nearby dentist for the replacement but either they suggested me for the fixed or implant treatment. but i dont want any invasive procedure. when think of removable, its very cumbersome to adapt and adjust that initially. me confused, is there any other option available apart from this becoz i really need immediate replacement that too non invasive and non surgical. plz help Doctor: Hi The number of missing teeth are more than what would be advised for a fixed partial denture. ideally implants would be the best solution for your problem. with newer techniques the implants are placed witha simple punch cut in which only the spots where implants have to be placed is punched out. Extraction on the other hand is a much bigger and traumatic procedure than this. Still if you do not want implans then you have only 2 options: flexible dentures and cast partial dentures. Flexible dentures are easy to adjust but cast partials are more stable and better for the health of the remaining teeth in the long run. Regards"
},
{
"id": 52384,
"tgt": "What treatment is available for an irregular low-attention liver lesion?",
"src": "Patient: My CT Scan shows a 4.4 X 3.8 cm irregular low-attention liver lesion. I just finished chemo for colon cancer and this is suspected metastatic disease to liver. (also showed multiple nodules up to 11mm in lung) What treatment options would be available. I see my ONC in a few days but just trying to get some info in mind. Doctor: Welcome to ' Ask a doctor ' service .I have reviewed your query and here is my answer .You need chemotherapy as there are metastases .With regards dr varinder joshi ."
},
{
"id": 140673,
"tgt": "What causes dizziness, soreness inside lips and tiredness?",
"src": "Patient: Hi, I have an under active thyroid and take 100 mg daily. Presently I am getting dizzy spell light headed, thirsty, soreness around inside of lips.... feling tired alot also even though getting decent sleep... have tree children 6 4 and 14 months. wondering should i be concerned or will symptoms ease. thanks Imelda Doctor: Hello, If it's been awhile since you've had your thyroid checked then, I would recommend not only a TSH but also an FT4 to see if the amount of replacement you are taking is truly adequate for your needs. I would also look at a HbA1C and fasting blood sugars to screen for possible prediabetes or diabetes itself. Hope I have answered your query. Let me know if I can assist you further. Take care Regards, Dr Dariush Saghafi, Neurologist"
},
{
"id": 12155,
"tgt": "How to cure white dots which have formed on my son's body and they are spreading on his entire body ?",
"src": "Patient: Hi Dr. Krishna..Requesting you to please advise me how to cure white dots which have formed on my son s body and they are spreading on his entire body.He is 5 years old.This problem started to happend three years ago.We have shown to various doctors and that time there were only little dots ,so we igonred it .But now they are increasing rapidly .We have shown to a doctor recently from Max health care and hbis biopsy test was conducted.He prescribed some creams to aply on them but nothing helped.Now we are woorried about him.PLease find the microscopic examination of the skin biopsy of my son below: Hi Dr. ..Requesting you to please advise me how to cure white dots which have formed on my son s body and they are spreading on his entire body.He is 5 years old.This problem started to happend three years ago.We have shown to various doctors and that time there were only little dots ,so we igonred it .But now they are increasing rapidly .We have shown to a doctor recently from Max health care and hbis biopsy test was conducted.He prescribed some creams to aply on them but nothing helped.Now we are woorried about him.PLease find the microscopic examination of the skin biopsy of my son below: 1.Sections show epidermis and dermis 2.There is mild orthokeratotic hyperkeratosis .Epidermal maturation sequence is preserved.Melanocytes are adequate in number and distribution in the basal layer 3.Pigmentary incontinence with scattered melanophages present in superficial dermis,along with mild perivascular ,superficial dermal lymphocytic infiltrate noted.The opinion as per the examiner : suggestive of post inflammatory pigmentary alteration. I would be extremly grateful if you could find out a sloution to this problem. Thanks Doctor: Thanks for the query Post inflammatory pigmentations are common in tropical countries. It has no definitive cure. Try MDA and peels. In time it will start healing on its own. Avoid mosquito bites, ant bites, etc Have a nice living"
},
{
"id": 65243,
"tgt": "What should I do for reddish bump in between my eyebrows?",
"src": "Patient: I have this hard reddish bump in between my eyebrows and I though it was going to be a white head but how it formed but it s not. A whole bunch of clear liquid comes out when I squeeze it and then just refills itself, it seems, and it wont go away. It hurts to make facial expressions and when I touch the spot. Doctor: Thanks for your question on HCM. By your history and description, in my opinion you are having either 1. Infected boil 2. Infected sebaceous cyst. Skin infection is common on face due to constant exposure from environment and if someone is having oily skin then possibility of this is even more. Pain on touch, fluid, redness etc suggest infection and inflammation. So you need to remove it surgically for complete relief. Avoid too much moisture and keep proper hygiene of face. Wash face thoroughly 4-5 times a day. Don't worry, you will be alright."
},
{
"id": 69765,
"tgt": "What causes hard lump on either side of each ankle?",
"src": "Patient: I have a hard lump that feels like bone forming bilaterally over each ankle, on the lateral side about an inch and a half (maybe 2 inches) above the ankle bone itself. It is the exact location where the top of my hockey skates meet my skin, and amongst any rotation of the foot, presses into my skin and bone. At first just a bruise, the lumps have formed and I wonder if it is bone or scar tissue formed as a result of constant aggravation (I play hockey on average 3 times a week) or something more serious. Should I be concerned? Doctor: Hi.Nothing serious.This is reaction o the skin to protect against the irritation as a normal reaction.Adjust the rubbing skates. Apply emolients to keep this skin soft. This should go away.Remember irritatiopn of any sort will casue this to recur fast.Take multivitamins- helps in rejuvenating the skin."
},
{
"id": 108043,
"tgt": "Suggest treatment for lower back pain",
"src": "Patient: I've had this pain in my left side, more toward my back. I first felt it right before I started my period, so I assumed it was either a cramp or a cyst as I've had both before. It's been about a week since I've had my period and I still have that pain. It's gotten to the point where it hurts to put weight on my left foot because I feel that stabbing pain in my side. I'm not sure if I should get this checked out or just wait it out, but it doesn't seem to be getting any better on its own. Doctor: I have gone through your question and Appreciate your concern. U have pain in lower back and it is progressive And not relieved.I think it's better to address health Problems at initial level before They harm You. U may consult orthopaedic spine specialist.General measures to improve back pain are Avoid bending forward and lifting heavy Weights. Hot fomentation thrice a day. Muscle relaxants and analgesics will help. Bed rest and lie on hard flat bed.Local Analgesic gel for local application. Muscle strengthening exercises will help you In long term. Thanks. U can write me back for any query."
},
{
"id": 83893,
"tgt": "How to wean off from the usage of endocet?",
"src": "Patient: i am presently taking endocet and would like to wean myself off of them even though I am in pain all of the time from a previous motorcycle accident. I cannot get out of bed without taking my prescribed dose which is 2. then 2 more at dinner then 1before bed can you suggest anything Doctor: Hello, You can gradually taper the dose and stop the pill. Abrupt stoppage can result in unwanted complications. Hope I have answered your query. Let me know if I can assist you further. Take care Regards, Dr. Shinas Hussain"
},
{
"id": 67807,
"tgt": "Suggest remedy for lump at the place of implant",
"src": "Patient: Hi,I ve had the implant now for almost 2 months and the area often feels bruised and itchy. It s gotten a little inflamed with a rash on two occasions and tonight I can see a clear lump at the top and bottom of where the implant is located.Is this normal healing? Doctor: Hi,Thanks for asking.Based on your query, my opinion is as follows.1. I think you are having an implant rejection or allergic reaction due to implant rupture.2. Itchy bruising lesions at the site of implant suggest possible allergy.3. Meet your doctor to see if the implant has ruptured. It could get infected and become serious condition. Meet your doctor at the earliest to evaluate and possible removal of implant. Hope it helps.Any further queries, happy to help again."
},
{
"id": 83483,
"tgt": "Does taking epilim chrono affect the sperm count?",
"src": "Patient: im currently taking 1500mg of epilim chrono before bedtime ,ive just started taking 25mg of zinc per day and omega 3+6+9 to boost my sperm count the two questions are 1does epilim chrono effect sperm count and also can i take the pills ive just mentioned while i take epilim chrono ? Doctor: Hello, Epilim is an antiepileptic drug and it has got no impact on sperm count. You can take zinc and omega supplements along with epilim as it will not interact with each other. If the sperm count is persistently low, condition like varicocele must be ruled out. An ultrasound scrotum with Doppler is required to make a diagnosis. Hope I have answered your query. Let me know if I can assist you further. Take care Regards, Dr. Shinas Hussain"
},
{
"id": 100250,
"tgt": "Does wheezing and shortness of breath increase during periods?",
"src": "Patient: I have asthma which over the past six months has been getting harder to keep in check. Have bouts of wheezing and shortness of breath on and off and am just starting to notice that it occurs more when I have my period. Is that anything significant or just coincidence? Doctor: Hello,Thanks for asking your query on HCM.Your observation that asthma gets worse before periods is correct. Just before and during your period, progesterone and estrogen levels decrease. In you these hormone changes may worsen asthma. The relationship between hormones and asthma is complex, varies between individuals and isn't fully understood.Asthma requires two type of medicinesOne is a controller or preventer, usually in the form of inhaled steroids and the other is a reliever which is a short acting beta 2 stimulant, as an inhalant. If the steroid inhaler is not enough to prevent attacks, then a long acting Beta agonist (LABA) is added. You could check with your doctor or contact me with your current medications and I would be able to suggest how you can reduce the attacks.Take CareDr. Noble Zachariah"
},
{
"id": 17175,
"tgt": "Suggest treatment for rapid heart rate",
"src": "Patient: I have a heavy heart beat. I am 21 years old, male, moderately active (3 times a week 1 hour physical activity), no smoking, seldom drink. When I lie down I can feel my heart beat and I can on occasion feel my body move as a result of the heart beat. Does this sound normal or not? Doctor: Hello, I would like to tell that heartbeat is ever changing phenomenon depending on your physical activity, stress and physiological changes. Your symptoms are totally normal but anxiety. It\u2019s recommended for you to do some meditation or take some anxiolytics. Hope I have answered your query. Let me know if I can assist you further. Regards, Dr. Bhanu Partap, Cardiologist"
},
{
"id": 186903,
"tgt": "How to treat ulcers on the inner bottom of lip & throat ?",
"src": "Patient: Hi, I have about 6 mouth ulcers 2 on the inner of my bottom lip, 1 on my upper inner lip, and 3 in the back of my throat, I have a had ulcers over the past few years but only 1 at a time and are very small, the ones I have now are all about the size of a pee, any info on what it may be Thanks, Nick Doctor: Hi. Welcome to Healthcaremagic. I read your query. Oral ulceration in mouth can be due to several reasons, most common being Apthous ulceration (cause unknown). Other reasons can ce trauma, allergy, medications, radiation exposure, underlying disease etc.If you have history of one of those, I suggest you to consult your physician for treatment of underlying cause.In case of apthous ulcers, the cause is not known. They can be single or multiple ulcers ranging from pun point to larger ones. The small ones heal in 7-10 days while larger one may take upto 40 days to heal.Apthous ulcer may reoccur in 1-4 months or longer.Treatment for these are symptomatic unless underlying cause is known.Avoid stress and anxiety. Take proper rest and sleep. Take multivitamin and mineral tablet one per day. Maintain good oral hygiene with brushing twice a day and use mouthwash. Take healthy diet.Use Lidocaine gel for ease of pain. Apply it before eating also as it will reduce burning. Avoid spicy and acidic food and beverages.If ulcers enlarge or change shape, contact your physician for further tests.Hope the answer helps you. Thank you."
},
{
"id": 185388,
"tgt": "Can dental abscess cause severe pain in gums and headaches?",
"src": "Patient: Hi, for the past couple months, my first premolar on my left side, has been giving me a lot of pain. I get headaches every now and then, the gum around that tooth is very itchy and sometimes I experience pain where my tooth and gum line meet. I don't see a cavity there. Could this be a tooth abscess? Doctor: Thanks for using Health Care Magic.Read your query.Pain in the tooth can cause referred headaches.Your tooth may be caried or there can be periodontal abscess.I would advice you to visit your local dentist and get it checked followed by a radiograph by which u will get a clear picture of your issue. For the pain you can take Ibuprofen Tab ( if you are not allergic to any medication). Do warm saline gargling.Thanks and warm regards."
},
{
"id": 18100,
"tgt": "Can Amlodipine be taken prior to a venous reflux repair surgery?",
"src": "Patient: I am a 58 year old heart patient with a repaired mitral valve. I have a diffibrilator/pace maker installed in 2013 after collapsing. This year I had a procedure to repair veinous reflux. I have recently experienced numbness in my feet. My cardiologist has prescribed amlodipine besylate and gabapentin. After reading the possible side effects I am concerned about taking these meds. I am a working professional and cannot afford to be dysfunctional at work. Doctor: Hello and Welcome to \u2018Ask A Doctor\u2019 service. I have reviewed your query and here is my advice. You can take Amlodipine before the procedure. Hope I have answered your query. Let me know if I can assist you further."
},
{
"id": 134968,
"tgt": "Suggest remedy for tightness and stiffness in legs",
"src": "Patient: Hi, I have been experiencing tightness and stiffness in my legs. I do regular exercise by means of walking my dog and have recently found this quite difficult. I feel my legs getting really tight almost as if they are going to burst. They also feel really stiff as if I cant open them properly. Could this be related to my heart? Doctor: Hi Dear,Welcome to HCM.Understanding your concern. As per your query you have tightness and stiffness in legs. Well there can be many reasons for symptoms you mention in query like nerve compression , diabetes , neuropathy or multiple Sclerosis . If condition doesn't get well then consult orthopedic surgeon for proper examination. Doctor may order blood test or RH factor , CT scan or MRI along with physical examination to confirm the diagnosis . Doctor may prescribe methotrexate , anti inflammatory , gabapentin long with nerve supplement and recommend physical therapy . I would suggest you to do light stretching exercises along with gentle massage , apply warm compresses on entire leg and take muscle relaxants like orthodox MR .Hope your concern has been resolved.Get Well Soon.Best Wishes,Dr. Harry Maheshwari"
},
{
"id": 71680,
"tgt": "Is HRCT test recommended for pneumonia?",
"src": "Patient: Hello Doctor, My mother is an asthamatic and diabetic and has been suffering from pneumonis and being treated an MAdam Mohan MAlviya hospital in Malviya Nagar Delhi. The doctors there have recomended for a HRCT test done for her chest. Cld u recomend the hospital where we shld go for this test? Doctor: Thanks for your question on Healthcare Magic.I can understand your concern. HRCT thorax is done at radiology center. So ask your doctor about near by CT scan centers.Usually CT scan is not required for diagnosis of pneumonia.But in some cases if diagnosis is not confirmed or patient is not improving or we suspect TB or ILD (interstitial lung disease) then HRCT thorax is advised.Hope I have solved your query. I will be happy to help you further. Wishing good health to your mother. Thanks."
},
{
"id": 48335,
"tgt": "Is it to be concerned about the kidney stone and bladder infection?",
"src": "Patient: My husband has kidney stones and a bladder infection he has taken all the antibiotics and flowmax and he still does not feel like he passed the stones and he has no appetite he has an doctors appt soon but what I want to know is this a more serious sign of something else when we went to the hospital they checked his blood and kidneys and every thing was nomsl what do you think? Doctor: First nothing to be worry about as the kidney functions r normal. Smaller stones can pass even without pain in many patients. If you were my patient then I'll suggest u to maintain hydration adequately so that u will void approximately 1.5 - 2 lt of urine and examine ur urine for microscopy and cytology."
},
{
"id": 92888,
"tgt": "Abdominal burning, pain on the right side below rib cage. Mostly at night. Suggestions?",
"src": "Patient: hi there, I have been suffering from abdominal burning and pain mainly on my right side ,just around the base of my rib cage.for about 5 weeks.Pain does travel through my whole upper abdominal area , sometimes little sharp pains.Most of the discomfort is at night and once i sleep the pain has gone in the morning, I have not lost weight and still feel hungry.Any suggestions/? i have been taking lalso alo vera juice and slippery elm. Doctor: Hello,The most common cause for pain in that area is due to Gastritis(Inflammation of the stomach).Proton pump inhibitors will help in reducing the pain and inflammation.You may need an ultrasound of the abdomen and upper GI scope to diagnosis the condition.Consult your doctor for more help."
},
{
"id": 37723,
"tgt": "Suggest treatment for purple colored itchy skin after a bee bite",
"src": "Patient: Got stung by a wasp on my left arm near my wrist and it swlled the first 3 days then became itchy after the first day it had widen out over the days and was hot to the touch. Put benadryl cream on it now its finally stopped itching but now where the reaction happened on my arm is turning kinda purpleish. Is this bad? Also got stung again on bottom of my foot test now itches bad but cream won t help. Thanks Doctor: Hello,Welcome to HCM,Bee stings most often trigger allergies. However, most people are not allergic to insect stings and may mistake a normal sting reaction for an allergic reaction. Symptoms of a severe bee sting allergy may include one or more of the following Difficulty breathing,Hives that appear as a red, itchy rash and spread to areas beyond the sting,Swelling of the face, throat, or mouth tissueWheezing or difficulty swallowing,Restlessness and anxiety.Severe allergic sting reactions are treated with adrenaline administered by a doctor. Usually, this injection will stop the development of a more severe allergic reaction.Intravenous fluids, oxygen and other treatments are also necessary. People who have had previous allergic reactions to bee sting must seek immediate medical attention.Thank you"
},
{
"id": 132543,
"tgt": "What causes numbness on the leg after dog bite?",
"src": "Patient: I was bitten on my leg by a dog who broke skin about 6 hours ago. I cleaned it and everything but now as I take the bandage off to let it air out I touched the part where the dog bit me and I can t feel anything. Is that normal to touch the part where it was bit and not feel it at all? Doctor: Hi Hope this message finds you in good health.I have gone through your complaints and understand your concern.Generally the severity of the symptoms depends on the wound size,how extensive is the wound or how deep the dogs canines might have pierced.The area in which u have been bitten has tiny nerves around that are responsible for sensations being intact.If during the bite,any of these nerves has been damaged, it causes numbness,as if the part isnt there.These are normal findings in such cases.what we do at our clinic is do proper clening of the wound,apply cream and start on antibiotic tablets.if the numbness is severe,get a ncv study done and start on pregaba tablets..eventually after 2 -3 weeks,the nerves start healing on their own and the numbness subsides completely.only in rare cases,the numbness remains.Nothing to worry about.Do get back to me after ur mri reports are ready.\u00a0\u00a0\u00a0\u00a0\u00a0I hope your question has been answered.If you have any follow-up queries,feel free to consult me anytime.Thanks,Take care,God bless."
},
{
"id": 149671,
"tgt": "Numbness in limbs, confusion, disorientation. What do symptoms suggest?",
"src": "Patient: I saw a neurologist today initially for chronic facial pain but 3 weeks ago developed tingly, buzzing, prickly, numb sensations in my legs, feet, arms and hands. Its 24/7 and getting worse. My legs also have a deep ache that burns at times, they feel heavy and I have decreased strength. I also have experienced confusion and feeling disoriented. He diagnosed me with atypical facial pain, which I figured he would even though I'm convinced its atypical TN. Anyways, he brushed off all my other symptoms as the anxiety and stress of being a new mom but ordered a brain MRI with MS protocol but not a spinal MRI. I feel really defeated and not taken seriously. I am just wondering what these symptoms could suggest? Btw my labs are perfect - not deficient and anything and everything is stellar. Thanks in advance. Doctor: Hi also include a spinal screening with the MRI scan. They will do without a significant increase in cost. I think no organic cause can quantify all your symptoms. I do think it is an anxiety/stress related.Review with the reports. You can also send the reports to my mail also neurosurgeoncp@gmail.com"
},
{
"id": 13962,
"tgt": "What causes itchy red rashes on the chest and abdomen?",
"src": "Patient: hello since this morning i noticed a red rash on my chest and abdominal parts of my body now it s over midnight and my legs and feet are having the same problem and it burns and itch a lot can t sleep because of this and haven t take any medication i wonder what s going on ? never happens before Doctor: Hello and Welcome to \u2018Ask A Doctor\u2019 service. I have reviewed your query and here is my advice. From your explanation rash on chest and abdomen that is spreading to legs and feet seems to be most probably due to allergic reaction leading to Dermatitis. As it is spreading you should consult an Emergency room and get evaluated. You can be advised anti allergic medicine like Fexofenadine or Levocetrizine and also a steroid shot can be given to reduce the inflammation and rash.. Also doing cool compresses over the skin, applying Caladryl lotion and steroid ointment like Betamethasone can help in Improvement. Also if allergy is only ruled out then an Allergy test can be done for confirmation of the exact cause of the allergy.. Hope I have answered your query. Let me know if I can assist you further. Regards, Dr. Honey Arora"
},
{
"id": 82521,
"tgt": "Suggest treatment for cough and sleep disorder",
"src": "Patient: I HAVE BEEN COUGHING UNCONTROLLABLY AT NIGHT AND AT TIMES DURING THE DY, IT CHOCKS ME AND I CANNOT SLEEP BECAUSE EVERY TIME I lay down I start coughing again what can I do to stop this. I happens at work and other places and people think I am sick but I m not. what can I do? Doctor: Thanks for your question on HCM. Coughing especially in night can be seen in many causes but most commonly seen in 1. Cardiac diseases like left ventricular failure2. Asthma3. Lung infection.So I advice you to consult pulmonologist first and get done chest x ray and PFT (Pulmonary Function Test) to rule out asthma and lung infection. Also get done ECG and 2D ECHO to rule out cardiac failure and other valvular heart diseases.Start antihistamines and get done these tests."
},
{
"id": 202794,
"tgt": "What is the cure for the red, dry and cracked foreskin and has been diagnosed for Type I Diabetes?",
"src": "Patient: Hi yes, I have an extremely red, dry and cracked foreskin. It won t stretch over the tip of my penis and I find little white sorta goo under the foreskin when I do peel it back. Have just been diagnosed with type 1 diabetes had these problems since I started showing symptoms for my diabetes but I don t think it is. Please help!! Thank you Cody Bingham Doctor: WHAT U ARE DESCRIBING IS A COMMOMN COMPLICATION, DIABETIC PATIENTS SUFFER FROM.IT IS BECAUSE OF REPEATED INFECTION OF FORESKIN AND GLANS PENIS CALLED AS BALANOPOSTHITIS.THE LONG TERM EFFECTS OF THIS LEADS TO CONTRACTURE OF FORESKIN RESULTING IN INABILITY TO RETRACT IT BACK CALLED PHIMOSIS.U ARE ADVISED TO UNDERGO CIRCUMSCISION BY A SURGEON OTHERWISE U WILL BE GETTING REPEATED INFECTIONS, PAINFUL ERECTIONS."
},
{
"id": 175900,
"tgt": "How should feet and toes whiteness be treated?",
"src": "Patient: My baby girl is 3 1/2 months old and for the past while I have noticed that her feet and toes turn white sometimes. It looks as if they completely lose blood flow, they turn as white as can be, completely colourless. What could this be? I asked my doctor and he was unsure of what it could be and just asked me to bring in a photo of her feet. Doctor: Hypothermia causes whiteness of peripheries due vasoconstriction causing poor blood supply to that area. This can be prevented ant treated by preventing and treating hypothermia.Use mittens and gloves to cover hands and feets.Also breast feeding may prevent hypothermia"
},
{
"id": 128056,
"tgt": "What causes a bruise like swelling on the right side of the shin and ankle?",
"src": "Patient: Hi My right swells up on the shin and ankle red like bruising and it s sensitive to touch. It also get lumps in it. I did fall down the stairs a few years ago. It usually last for a few days to about a week then goes away and comes back over and over again. The doctor has no idea. Doctor: Hi,I have studied your case,There can be skin discoloration/bruise due to blunt injury on calf, and there can be hematoma near ankle.As x ray is normal indicates soft tissue injury due to acceleration and blunt trauma.I can not compare swelling, but if swelling is there, you need to take rest and use splint or compressive crepe bandage to reduce swelling..On ankle you can apply thrombhophobe ointment.Watch for increase in redness, infection.Take precaution to avoid infection around hematoma.Do not use hot fomentation or massage.If required you may need to do blood investigation for prognosis and recovery.Hope this answers your query. If you have additional questions or follow up queries then please do not hesitate in writing to us. I will be happy to answer your queries. Wishing you good health.Take care"
},
{
"id": 102657,
"tgt": "Feeling of lumps in lower throat and cough after several antibiotics and steroids post pneumonia",
"src": "Patient: Had pnuemonia after several rounds of antibiotics and steroids my breathing is fine but I ve had a feeling of a lump in my lower throat and a cough for several weeks after went to doctor and said my throat felt fine and said it was possibly due to allergies/post nasal. Doctor: Hello dear,The symptoms as mentioned in your post suggest that you might be having Allergic tracheo-bronchitis.Examination of the throat using a laryngoscope and X ray of the chest will be helpful in clinching the diagnosis.Management includes:1.Antihistamine preparations like Allegra & Cough expectorant syrup for symptomatic relief.2. Montelukast preparations- used as a maintenance therapy to relieve symptoms of seasonal allergies.3. A course of antibiotics may be taken to prevent secondary bacterial attack.4. Maintain adequate hydration & take a healthy balance diet.5. Also make sure that you are well protected from cold, dust & other allergens.Wishing you a good health.Take care."
},
{
"id": 29521,
"tgt": "Suggest treatment for sinus infection, headache and teeth pain",
"src": "Patient: My husband has sinusitis he has green mucus, coughing headaches, and teeth hurt: he gets this once a year and usually has to have three rounds of antibiotics Any suggestions Doctor: HI, thanks for using healthcare magicBased on your description he may need another course of antibiotics. In addition to the antibiotics, he may also need an antihistamine/decongestant combination as well as a topical steroid nasal spray as well.eg of topical nasal sprays- flonase, nasonex, nasocortAnti histamine/decongestants would be over the counter. He may to take these long term to prevent or reduce the chance of repeated infectionsI hope this helps"
},
{
"id": 92128,
"tgt": "Should I be worried for having abdominal atherosclerotic disease without aneurysm?",
"src": "Patient: had a ct scan with contrast it showed Abdominal Atherosclerotic disease present but without aneurysm, is this something to be worried about ? it was done on august 8 and since the I have had severe abdominal pain on two different occasions. I have ovarian cancer stage 3 . just finished up chemo 3 weeks ago Doctor: HI Obviously the reason can be stage 3 ovarian cancer and not he atherosclerosis . Please do not worry about it. It can be just an incidental finding ( not of clinical importance at the moment)."
},
{
"id": 28912,
"tgt": "How can fungal infection on the toe nail be treated?",
"src": "Patient: I am looking for Charparral (dried) to help with toenail fungus. Have found a Toenail fungus protocol on the internet. Use six tbsp dried chaparral to one quart of boilling whiskey.. Reduce and simmer for 20 minutes. Remove and steep for 8 hours. Soak your feet in this solution for 20 minutes a day, Repeat the procedure for two weeks.Have had laser treatment on both feet and did not help. Costly $400 for one treatment. Have used a natural mixed with Mother of Vinegar, but does not help.According to the info I have seen on the internet chapparral is dangerous to take internal, but want tosoak my feet in this. Would this be safe? Doctor: Hello,Fungal infection can only be treated with anti-fungal agent beside this it cannot be given try with anything else, such experiment you have mentioned here may prove dangerous or some time flair up disease.If this is the fungal infection of the nail then it best can be managed with Miconazole ointment and oral anti-fungal medication like Terbinafine drug three times in day for 10 days, sometime two molecules can be added if single molecules fails to response.But it is all depends upon the clinical condition, why not to consult the dermatologist for best line of treatment and proper diagnosis.Hope I have answered your query. Let me know if I can assist you further.Regards,Dr. Akhtarhussain"
},
{
"id": 83558,
"tgt": "What is the usage of Acitrom tablet?",
"src": "Patient: Why Medicine Acitrom is given to patients . Can you please let me know, as the medicine was prescribed to my mother had suffered from blood clots in her brain by the neurologists here in India in Dec. 2009. She is continuously taking one tablet daily. Can you please answer my question. Doctor: Hi,Acitrom is an anti-clotting (blood thinner) drug which is commonly prescribed in the treatment and prevention of blood clots in the blood vessels thus to help in prevention of stroke or heart disease.Hope I have answered your question. Let me know if I can assist you further. Regards, Dr. Mohammed Taher Ali, General & Family Physician"
},
{
"id": 96171,
"tgt": "I am suffering from intestine disorder from last four years. Now a days I am suffering from headache, indigations & burning rectum",
"src": "Patient: I am suffering from intestine disorder from last four years I am taking IBS drugs like Rabeparazole and I am also taking thynorm, 25 mg before breakfast, Please suggest me what to do Now a days I am suffering from headache occasionally, undigested food material in stool, some times burning rectum Doctor: Dear Hamid welcome to healthcaremagic Ayurveda has described IBS as Grahani . and it has mention successful treatment for IBS related symptoms a. Powder of four sours (Vrukshamla, Amlavetasa, Dadima and Badara) 640 gm, trikatu 120 gm (Pippali, Shunthi, Black Pepper), and five salts 160gm added with sugar 320gm should be used with vegetables, pulses, cooked cereals etc. It is efficacious in colic, indigestion, anorexia etc. b. Panchakola is to be added in cereal soup, Radish soups with black pepper are recommended as diet. c. Buttermilk, sour gruel, and arishta should be taken as post meal drink. d. Buttermilk is specifically advised in this disease due to its appetizing nature, astringency and lightness in all ways. This aids recovery of disease. So take in profuse quantity. e. One passing undigested diarrheic stools due to deficient Agni should take adequate f. Dose of ghrita (ghee) mixed with appetizing drugs, as they are excellent stimulants of weak fire. g. Ayurveda advise \u2018Parpati Kalpa\u2019 in this disorder, which is very efficient for treating this disorder, but strict dietary regimens one has to follow during this therapy. h. Various Asavas / Arishta\u2019s, Medicated ghee & appetizer drug formulations are advised Please consult your nearest Ayurvedic physicians for details Thanks"
},
{
"id": 118469,
"tgt": "Can i smoke marijuana while recovering from DVT?",
"src": "Patient: I was diagnosed with a pulmonary embolism from post surgery about a year ago. Proceeded with treatment. About 2 months ago, I was diagnosed with a dvt. Currently on warfarin. Chest is clear but there is still residual clotting in the leg that is clearing up. Can I smoke marijuana? Doctor: Hi, Welcome to Health care magic forum. Any smoke, or any tobacco can effect the lungs, fatally. You are not cured completely of the deep vein thrombosis. Now it is not advisible to think of the marijuana, especially with the permission of the doctor. D.V.T. is also a result of the smoking , atherosclerosis, and thrombosis. If you have it now, the thrombus can be in the heart, or brain.Sorry it is not threatening, it is a fact. Take more of green leafy vegetables, pulses, sprouts,and protein rich foods to be healthy. Wishing for a quick and complete recovery. Thank you."
},
{
"id": 49884,
"tgt": "Have chronic renal kidney failure. Kidney pain, taking Core. Reason for pain?",
"src": "Patient: I have chronic renal kidney failure. I also have IGAN. I was given Diovan which made me very ill. My kidney doctor never did ANY bloodwork on me. Told me I was in need of seeking a oxycontin Dr. I was very upset as the Divan had caused me to Just about lose my mind. After NOT taking the Divan, I noticed I felt lots better. My anxiety was going away. I fired my Dr. Now I am having sharp stabbing pains in my left kidney. I am also taking core. I have never felt such pain. I have been off Divan and on Core for 3 months. Why am I having such pain. Doctor: you need proper investigations.u should go for ultrasound KUB,X ray KUB,to rule out kidney stones.also needed is urine routine test to rule out infection in kidney"
},
{
"id": 104815,
"tgt": "Suffered stroke, seizures. Taking eptoin, stopped it due to allergy, taking wysolone. Safe to take?",
"src": "Patient: hello doctor, my mother had a stroke last feb n later few months ago she had a seizure and strtd using eptoin .later cz of allergy doctors stopped dat and instead gave levipil plus wysolone for allergy.. today again she s havin skin allergy a little bit as earlier...so should she take wysolone now?? doctor hd asked her 2 decreas the dose n she was nw takin half a pill wysolone alternativly. so if she take a tablet today will dat be okay?/??? plzzz plz do reply. Doctor: Hello, I am not sure what 'allergic reaction' your mother to the eptoin (phenytoin) requiring the drug to be stopped and starting prednisolone for the rash. Phenytoin can cause a severe drug induced systemic reaction called Drug rash, Eosinophilia, Systemic Symptoms (DRESS for short) where there is fever, big glands, skin peeling off and liver problems. Other drugs implicated in DRESS should ten be avoided. I am certain your mother did not have this...but maybe just a rash that can be itchy and known as urticaria. Prednisolone is occasionally used a last resort but antihistamines are more than effective in treating urticaria. The alternative antiepileptic is a good choice, and suggest you discuss with your Neurologist in tapering and stopping the steroids after 5 days if the diagnosis was not DRESS. I hope this was helpful. Best Wishes."
},
{
"id": 201854,
"tgt": "What causes blood in semen?",
"src": "Patient: Hi DR:I have this problem there is blood in my siemens I don't know what to do.before is was little brown and jelly in the siemens now look all brownish and all like jello texture in there.me and my wife try so hard to have a baby, can this cause a problem???and what kind of treatment I should receive inorder help.I have seen DR few week ago they test my blood and urine every thing is normal but last week i started to have the blood in my siemens again I am worried...please let me know soon thank you Doctor: Hi,It may be due to trauma during sexual intercourse or masturbation. However, a semen analysis test should be done to check whether it is blood or not.Thanks!"
},
{
"id": 155829,
"tgt": "What does cancerous cells in liver post liver transplant indicate?",
"src": "Patient: RECEIVED LIVING DONAR LIVER FROM MY DAUGHTER 7/15/2014 HAD LIVER CANCER TUMOR ON LIVER WAS 1.8 cm DID TRANSPLANT ALL WENT WELL HAD FIRST SCAN 10/22/2014.. DR CALLED ME SAID SCAN SHOWED 2 LEISONS 1cm EACH WHAT COULD THIS BE? AFTER TRANSPLANT SURGEON TOLD ME THE CANCER HAD JUST STARTED TO PENETRATE THE CELLS IN THE LIVER. COULD THIS BE THE CANCER COMING BACK? THEY PUT A STENT IN MY HEART 2 WEEKS AFTER TRANSPLANT DR SAYS T TO DANGEROUS TO DO BIOPSY ON PLAVIX BUT IF NECESSARY THEY WILL. Doctor: Hi Welcome to HCM I have gone thru your query regarding cells found post liver transplant . It is metter of great concern . In fact, everyone has cancer cells forming all the time. But normally their strong immune system .kills the cancer cells fast enough that a person is never \u201cdiagnosed\u201d with cancer. But when the immune system becomes weak, the cancer cells can grow out of control , the disease overcomes . Dear, Let your treating doctors do their job and you do your job of to strengthen your immune system by taking care of your life style Side by side . You are lucky that cancer has just started , I would like to suggest you not to worry and modify lifestyle to fight cancer , is one of the most effective treatment options in my view .As your treatment is going on , You can't sit back , see helplessly .Magnitude of the survival advantage from improved nutrition, can even be greater than the magnitude of the treatment effects being targeted in current clinical drug trials .Treatment with some supplements and alternative therapies being full of anti bacterial and antioxidants to improve quality of life , help increase immunity and accelarate the process of recovery . Our health depends mostly upon 'what & when & how we eat, and our life style One of the important strategies to achieve healthy body is a diet rich in fresh, raw whole foods .More of Fiber, fruit, egg fish ,fish oil for Omega 3, green leafy veges . quality, organic food, will naturally increase immunity .To keep metabolism on right path to increase the strength of immune system , take all supplements full of antioxidant serve as antibiotics . 1. Take mixture of extract of Raw garlic , ginger ,coconut water , lemon juice ,mooringa tree bark and honey in equal quantity ,dose is 25 ml with same amount of water half hor before meals . for 4 days and after that without water for 21 days . It is a great antioxidant and antibiotic ,Without any side effect2. Turmeric powder , level spoon with with a cup of hot goat's milk with B/F & at bed time , 3.Buttermilk with lunch4. Aloe vera juice 25 ml twice a day +2 spoon Amla juice5. Bitter gourd juice 25-30 ml 6. Add lot of water to your regimen for flushing out toxins from your body.Antioxidants maximizes natural minerals so that your body has the raw materials it needs , to do what it was designed to do. To kill bacteria on mass basis . 7. Physical activity in order to renew healthy cell and rebuild itself, so you should make exercise a lifelong commitment.Do 30 mins walk/ exercise - from head to toe ,yoga ,pranayam - deep breatihing , Kapalbhatti / Laughing aloud . proper rest , meditation & positive thinking , to detoxify your system to accelerate the process of recovery . .The condition is REGULARITYContipation is the mother of all illnesses so is mental stress /worry /anger I further suggest you not to worry and take action fast ,Above regimen will surely gradualy , help you to lead happy worry free healthy life ahead Apply north pole Magnets to hands morning & feet in evening & drinking Water,30-35 ml every 3 hours , prepared on north pole magnets also plays as a potent tool in treating cancer and killing bacteria on mass basis.Take Homeopathic Carcinocin 200 every fortnight / 3 doses .HOPE THIS HELPS SOLVE YOUR QUERY Take care All the best Wish you early recovery . If any doubt mail at drsuchda@gmail.comDont hesitate for futher query if any"
},
{
"id": 113336,
"tgt": "Got back injury. Getting pain in back and abdomen. Passing blood in urine. What's wrong?",
"src": "Patient: my son is 17 years old. he fell and hit his back a few months ago and was passing some blood in his urine . the doc recommended a urine test and an ultra sound , both of which came back ok but he was having pain in his lower back and abdomen . he was given pain killers and anti-biotics. the helped for a while but occasionally he would still experience some pain in his back and lower abdomine. today he experienced severe pain in his thigh while passing his urine and had to kneel down as his legs were shacking badly. can you say what is wrong? Doctor: Hi, Welcome to the forum. It might also depend on other symptoms now or before. Have you had bladder infection symptoms in the last week or two? And blood in urine. If any of these are true, it is likely a urinary tract infection in your kidneys and you should get in contact with your doctor right away. You can also get over the counter test strips for urinary tract infections. I hope this is helpful to you. You can consult me again directly through my profile. Regards."
},
{
"id": 165818,
"tgt": "Suggest remedies for recurrent fever with Hemoglobin 9 in a child",
"src": "Patient: hello sir/mam I am Amitava from Agartala(Tripura) . i am come to CMC on 19/04/2010 with my child named master Abhigyan dasgupta ,hospital ID-663619D & consult with Dr. Maya Thomas. & get very good responce from her. & my baby is not get any convaltion till now. But now a days he is suffer with repet fever 101-103. he is continue P250-6 ml & Epilex 15 ml a day. he also carring hemoglobin as 9. what you suggest right now. My id is YYYY@YYYY Doctor: Dear parents, you need to get hemoglobin electrophoresis ti exclude the presence of Mediterranean fever.. this is a disease that causes attacks of hemolysis and fever.,"
},
{
"id": 211823,
"tgt": "Need a note from doctor suggesting time away from work, stressed out. Prescribed ativan. Help",
"src": "Patient: I have been suspended from work and feel stressed out and have been trying to find a doctor to give me a note for time away from work but so far this has made me feel worse. How can I get the help I need. I do not need more presrciptions but I do need time away from work. What can I do to get this break? I have been to the health clinic, I was told that the dr does not know me so he can not give note for work but gave me ativan, outpatient department dr says I was med seeking and gave me ativan, family dr is on vacation. What can I do? Doctor: We understand your concernsThis normally happens. Doctors don't give certificates to people whom they don't know or are not treating. This is because there were incidents where such certificates were used illegally. Go to a general physician and take treatment and prescription and after two days again go and ask for certificate. He should help.Good luck."
},
{
"id": 186465,
"tgt": "What causes pain in gums and pus formation after tooth extraction?",
"src": "Patient: I had a 2nd upper molar pulled on friday. It is still sore around the gums and it looks like it has green pus. Is this normal. i have been taking ibuprophen and it is still just a little pain no throbing. I also had a liver clot. Does this have any affect? Or am I healing normal. Doctor: Hello, Welcome Thanks for consulting HCM, I have gone through your query, as you have undergone extraction now you have discharge it can be due to infection in socket . Consult dentist and go for Radiographic investigations done to evaluate the diagnosis . Do betadine gargle twice daily . Hope this will help you."
},
{
"id": 208136,
"tgt": "Suggest treatment for depression",
"src": "Patient: helloi dnt knw how 2 start......n wht 2 say..only thng i cn say is am nt fne my mental condition is nt goood. feel lke crying in a louder voice.untolerable anger feel lke 2 finish of my self.my mind is nt stable my decions r nt stable......feelin vry alone.prblms is i dnt knw whether my prblm is exactly a prblm r nt......nt exactly financial n nt lve... i knw i cn handle myself bt am still in a depression........cn u plz help me out..i dnt feel lke sitting wid frnds n going out il b fne til afternooon i dnt knw wht happens 2 me whn d evening strts.pz dr help me out Doctor: HiThanks for choosing healthcaremagic.I have read your query carefully and understood that you are having symptoms like low/depressed mood, crying spells, anger & irritability, feeling lonely, not able to enjoy previuosly pleasurable activities like going out with friends.I would like to let you know that your condition is strongly suggestive of depressive episode. In my opinion your condition needs detailed assessment.I would advice you to visit psychiatrist near you.In depression , patient tries to search for the reason for his condition. Although patient doesn't acknowledges that its a biological illness. Hope I have answered your query, let me know if you need any further information.RegardsDR. ASHUTOSH SINGH"
},
{
"id": 127143,
"tgt": "What causes pain in the left arm and neck?",
"src": "Patient: I m 57 years old.always active. Woke up about a month ago with pain.My left arm and neck hurt,like I slept wrong. I don t sleep on my left side..I have a knot on top of my left shoulder, a good size. I ve tried different things.nothing is working. Last two days the pain is worse,a and I have numbness in my fingertips.I ve always had scrapes and bruses.Nothing like this.I do have my doctor..Your insight would be greatly appreciated. Thank you. Doctor: Hello, You symptoms are pointing towards possibility of nerve root compression pain. We call it radicular pain. Nerve root compression in cervical spine is usually due to disc herniation which leads to shooting pain In entire limb. If not treated early tingling and numbness starts. Diagnosis needs to be confirmed by doing MRI of the cervical spine. MRI is very sensitive and specific for diagnosis of spine pathology. Meanwhile take pillow if small height while sleeping, Avoid jerky movements of neck and weight lifting. Start tab Pregalin x 75 mg one at bedtime to reduce pain. Visit to neurophysician for detailed clinical Examination is recommended. Hope I have answered your query. Let me know if I can assist you further."
},
{
"id": 126828,
"tgt": "How can pain and swelling in the ankle be treated?",
"src": "Patient: I have noticed swollen in my ankles and lower leg area over the past two weeks, it\u2019s getting worse. I feel I have injuriedbthe tendon on the outside of my left ankle and the Achilles\u2019 tendon on my right ankle, what can I do to help cure my pain and swelling? Doctor: Hi, It can be a sprain or contusion. As a first line of management you can apply ice packs and take analgesics like acetaminophen tramadol for painrelief. Generally the symptoms will settle by itself in couple of days. If pain and swelling persist, you can consult an orthopaedician and get evaluated. Hope I have answered your query. Let me know if I can assist you further."
},
{
"id": 7127,
"tgt": "Why i am not getting pregnant even though mine and my husband's reports are normal",
"src": "Patient: My age is 25 and weignt is 60 heignt 5.2 and female.i did mtp three years back later i didnt get the pregnacy. i am using medicine from two years.i did many test like tube test, thyroid test, harmone test and every montth i am going with trans veginal scan to see egg development.every test is having normal.only problem is some mildly polysists are existng.and uterus size is also normal.and my husband is also havng a semen count of 45 million. i am very much eager to have child as early as possible. please suggest me. Doctor: Hello Welcome to healthcare magic. You neednot worry, since all your tests are normal .if you are having regular treatment and check ups, you just need to have patience and avoid stress.Stress is also a major factor in delaying pregnancy.you are only 25 , so age is not a problem.If you had mtp by DILATATION AND CURRETTAGE then just make sure you get your endometrial biopsy done to check if uterine wall is normal.Please be positive and avoid anxiety to have early results. All the best."
},
{
"id": 30276,
"tgt": "What causes bronchitis infection to occur frequently?",
"src": "Patient: I am a 16 year old female. I have been sick now for 5 months. I have reoccurring bronchitis infection that has been going on for 5 months. About 2 months ago, I was diagnosed with a psuedotumor from the medicine I was on, Tetracycline. After I was off of the medicine, that went away, but I was still getting headaches. They ordered an MRI or my head. I have a sinus infection and cysts. I also went to the allergist about 3 weeks ago. She wanted to do another blood test. During the 5 months, I have had 4 blood tests, but because I am allergic to Penicillin, Sulfa, and Tetracycline, they put me on prednisone, a flovent inhaler, albuteral, and a 6, 5, 4, 3, 2, 1 pack. They are all steroids. The allergist said that the steroids made the blood tests come back clear. She ordered an immune system test and a CBC. I am also having severe bone pain in my legs and it\u00c3\u00a2??s not shin splints, that\u00c3\u00a2??s what my PT says. I also expierence some bruising on my body when I wake up and I don\u00c3\u00a2??t know where it comes from. Also some lower abdominal pain like stabbing feeling. Can you tell me what you think I have?? Or if im on the right track with the doctors?? Doctor: You are on the right track, continue what you are doing. All the symptoms that you are currently having are due to the steroids, which are very powerful medications that will hopefully make you feel much better in the future. Continue to take them as prescribed. Bronchitis is an inflammatory disease of the lungs which is generally caused by an infection or a chemical irritant. Cough is usually the only symptom which can last for many months. In children and teenagers that get bronchitis, they should be screened for genetic and anatomic anomalies of the respiratory tract, immune systems, and for chronic asthma and allergies. Viruses such as RSV are very common at this time of the year. I would recommend frequency hand washing, update vaccinations, avoid smoking and secondhand smoking, control asthma and allergies if you have either of them, cough drops, double you fluid intake, continue the steroids, rest, treat acid reflux if you have it, tylenol for pain and headaches, bendaryl for sinus congestion, and the medications prescribed as above. You should also have an albuterol inhaler or nebulizer that you use daily."
},
{
"id": 194747,
"tgt": "Can a vasectomy cause weight gain, bloating and loss of appetite?",
"src": "Patient: I had a vasectomy on 20 July 2108, I feel like Now I\u2019m always bloated, my appetite is less and I get full very easily. Before I had a big appetite. I could go to bed with a full stomach and wake up with a flat empty stomach, hungry. But after this vasectomy surgery it\u2019s changed I feel full all the time even when I wake up in the morning I don\u2019t even want to eat. So can a vasectomy make a male gain weight. Thanks XXX 30 July 2018 Doctor: Hi, A vasectomy will never cause any of the problems you have mentioned. Get a checkup with your physician for other causes. Hope I have answered your query. Let me know if I can assist you further. Regards, Dr. B. Radhakrishnan. Nair, OBGYN"
},
{
"id": 86928,
"tgt": "What could brown discharge and abdominal pain post the birth control pills suggest?",
"src": "Patient: i have back pain, brown discharge, and sometimes abdominal pain come an go 4a few minutes, i am just after startin my birth control after a months break from it, i started on the first day of my period, witch started 12 days ago, i ahd my period an now am having brown discharge since it ended..what wrong with me? Doctor: HelloWelcome to HCMThis is a common side effect of this pills dont worry about it.You can take other birth control methods after consulting your doctor.For now you can have tranexamic acid and mefenamic combination.Hope this is helpful..Regards."
},
{
"id": 44756,
"tgt": "Can azoospermia be due to bad habit of rubbing of testicles ?",
"src": "Patient: due to bad habit rubing of testicles may also lead to azoospermia and such a case has been done with me azoospermia may be due to bad habit rubing of testicles Doctor: hi,thanks for query.I am sorry dear but I do not agree with you.There are many causes of azoospemia and this is never reported.Trauma,viral infection,local conditions of testis like varicocele etc,can lead to this.Please dispel this myth from your mind and talk to a urologist for more advise if you are suffering from this. wishing you good health."
},
{
"id": 144778,
"tgt": "What causes neck pain and tingling sensation in arms?",
"src": "Patient: about two weeks ago i was berry pickin and fell down a hill...same night i slept bad and woke up very sore in my shoulder blade. thought i had slept wrong. well its been two weeks and my symptoms have changed. the pain moved from my shoulder to the center ofthe top of my spine up my neck...now for the last couple days my arm and underneath and starting to pull on my chest i ve had musle spasems so feirce that my arm moves and jerks uncontrolbly. the pain and tenderness is almost unbaralbe. i have numbness and tingleing threw out my arm and fingers. it feels better with pressure in the spots and im not sure if ive just pulled a musle or if i have blockage of some kind Doctor: HelloTingling sensation in arms may be due to nerve involvement in cervical spine.As you had history of trauma,it is important to screen cervical spine.You may need cervical spine MRI.MRI can detect disc prolapse,nerve involvement etc.Proper treatment depend upon findings.You need proper evaluation by a orthopedician.Get well soon.Take CareDr.Indu Bhushan"
},
{
"id": 173949,
"tgt": "How to know if baby is allergic to food?",
"src": "Patient: My daughter often gets a red itchy rash around her mouth after eating certain foods. We have tried to find the link between these foods but the ingredients are different in all of them. How can we find out what she is sensitive to? She also has a peanut allergy and gets eczema. Doctor: Hi,You have to find out the food gives allergy.Go for trial and error method and you will find out any special food giving allergy like you find allergic to pea nut.After finding allergy giving foos, avoid giving.Ok and take care."
},
{
"id": 190686,
"tgt": "Blood in gum. Could it be a lack of a certain vitamin ?",
"src": "Patient: I have noticed that my 19 year daughter is waking up in the morning (only morning) with blood on her front teeth, her gums do not hurt her or bleed when she brushes or floss, we do not have dental insurance as of yet. Could it be a lack of a certain vitamin, this started about a month ago but it is also not every night it kind of comes and go. thank you Doctor: hi.the most common cause of gum bleeding is accumulation of plaque and calculus around the tooth.these deposits leads to gingival inflammation, which may bleed sometimes.visit ur dentist and go for oral prophylaxis.gum swelling and bleeding may be seen due to vitamin c deficiency.after oral prophylaxis, take vitamin c tablets.wish u good health"
},
{
"id": 11192,
"tgt": "Suggest remedy to stop hair loss",
"src": "Patient: I am 26 years old...before 7 months i had done straightning into my hair..after then only my hair fall started...after washing my hair its falling much..doctor consult me to use tugain 5 which i am using daily night before going to bed since 2 months..but still there is no improvement.i am very worried ..so plz help.. Doctor: Hello. Thank you for writing to us at healthcaremagicStraightening / hair relaxers can make hair shaft weaker and thus lead to hair fall, specially if hair is subjected to repeated straightening, however, it is unlikely to last for 7 months after the procedure.Hair fall is commonly due to either telogen effluvium Or androgenetic alopecia.Telogen is resting phase of hair follicle; once hair enters telogen it is going to be shed subsequently over the next 2-3 months.Causes of telogen are dietary deficiency/ deficient Iron stores; thyroid abnormalities; preceding stress e.g medical illness like dengue, jaundice, surgery etc; Or after delivery in females.After hair is shed, hair follicle again re-enters anagen / growth phase.Androgenetic alopecia, on the other hand is due to shortening of anagen phase of hair growth cycle producing progressively fewer and finer anagen hairs/ miniaturization.The product that you have been using contains topical minoxidil.Minoxidil is an excellent hair growth promoter and it stimulates transition of hair follicle from telogen to anagen as well as prolongs anagen.Therefore it is an excellent remedy against hair fall, however, the results may be slow and only noticeable after 3-6 months of regular use.Therefore, I suggest you to continue with minoxidil for the time being.You may also add an Oral Biotin supplement once daily. Biotin acts as an adjuvant to hair growth.Regards"
},
{
"id": 194077,
"tgt": "What causes discoloration on scrotum?",
"src": "Patient: Hi, am from africa and black am 30 and 1.88m, 65kg, i notice discoloration on my scrutum(redish) it hitches at times and its spreding already taken over the scrutum coming up 2 the penis, i meet a doctor some time he gave me antifungal drugs and cream i don't think its working pls helpy Doctor: Hi, If you had unprotected sex, need to think of Sexually transmitted infection. If not it can be a fungal infection. Using a local antifungal with steroid can help you. If the fungal infection is chronic need long duration application of antifungal. Hope I have answered your query. Let me know if I can assist you further. Regards, Dr. S.R.Raveendran, Sexologist"
},
{
"id": 93300,
"tgt": "History of removal of appendix. Will this relapse with sexual activity?",
"src": "Patient: Good Morning,I am a 28 years old man, I had my appendix removed precisely on the 14th of may, and I got discharged from the hospital a week later,I felt normal with just very little pains! A week later I had sex,though not serious one.. My question is hope the one I had will not hunt me and can I continue having sex? This is because I'm scared of relapse! Thank you Doctor: Hello,You must have had a laproscopic appendicectomy I guess.As you have said, the pains have reduced significantly.Appendix once removed does not recurr again.So you can have peace in mind that the same disease will not relapse again.But please keep in mind that having sex is a strenous physical activity and may not be all that good until 3 months after surgery are over.Please re confirm with your doctor.Hope this helps."
},
{
"id": 102282,
"tgt": "Why do I suffer from asthma or bronchitis only in summer?",
"src": "Patient: Hi Doctor, From past 3 years i have issue like Asthma or Bronchitis..My question is, Why do i get this problem in summer season.. Only in summer season..As per doctor advise i done medical test and started using duolin and telekast D...i would like know wheather its Asthma or Allergic bronchitis Doctor: Hi,There are two reasons of Asthma: Infection or Allergy.Most probably your reason- allergy. Particularly in spring and summer seasons are blooming various plants and their pollen could be a reason for your condition.In such case very difficulty to separate Asthma from Allergic bronchitis, but the treatment are the same anyway. I can recommend to make a skin allergic test, it will help your doctor to prescribe you the best treatment.Take care and wishing you a good health.Always yours."
},
{
"id": 212858,
"tgt": "Pinching sensation in chest, have palpitations due to panic attacks, have anxiety neurosis. Normal life possible?",
"src": "Patient: Hi Doctors, I usually get pinching senses in my left chest frequently from past 1.5 years. Around 1.5 years ago (june 2011), I got into Panic due to palpitations ans was very much anxious due to sudden palpitation attack ( no specific reason for palpitation at that time, as I got it when I was relaxing having a cup of coffee with friends). I got many ECG s done in a 1 month time and all advised to undergo TMT and ECHO tests. I went through TMT and ECHO and the reports are normal. I even consulted Physchologists and they named it as Anxiety Neurosis. I feel something is wrong with my left chest as I feel some pain and pinching senses quite often and also get palpitations frequently. I even got a chest X-Ray and it is also normal as per doctors. Few symptoms I get are :- palpitations, due to that fearness of death, uncomfortable, breathlessness, suffocation , sweating (sometimes) and lose my confidence. Due to all these problems, I get continuous thoughts about my death. Sometimes, reading articles about death make me to lose confidence and make me anxious and all these symptoms recur. Kindly advise me whats wrong with me. What I need to do to lead a normal life. Around 2 years back I lost my mother whom I loved very much. I got married recently, But still I am unable to come out of these problems. Looking forward for your great advises and suggestions. Thanks and Regards, Srinath Doctor: Welcome to Healthcare Magic, I can assure your condition are treatable and you can live normal life. Your symptoms are very suggestive for panic attacks, anxiety disorder. You no need worry about , because your condition start recently , possible you had a shock when your mother passed away. Try to concentrate your attention toward your family , avoid any information, movies, stories, etc. with negative and pessimistic subject, start anxiolytic therapy, your Psychiatrist may prescribe you anxiolytics drugs, alternative you may consult Yoga and Ayurveda Specialist for second opinion. Be always optimistic with clear target in the life. I wish you good luck and speedy recovery. Hope this helps and I have answered your query. If you have additional queries don\u2019t hesitate to ask. Best Regards, Dr.Mihail"
},
{
"id": 100091,
"tgt": "Suggest remedy for face skin allergy",
"src": "Patient: hiiiii doctor,I am ritika,22 years old girl, From the past 1 months i am suffering from face skin allergy. it feels like ma face is burning. What is the reason for this. Earlier( 2 months back) i went for health check up and doctor advice me to follow TB test . I followed that and it was positive. I am taking medicine for TB. May be allergy is due to these medicines or is there any other reason. Plz help me out sir/madam Doctor: HI, thanks for using healthcare magicIt is possible that the reaction is due to your medication but they are not listed above so it would not be possible to check that at this moment.You can speak to your doctor or pharmacist to see if your symptoms are possible side effects.If it is an allergic reaction and not a side effect, then the use of an oral anti histamine would be helpful.These are avaiable over the counter.I hope this helps"
},
{
"id": 177510,
"tgt": "What causes chest pain and lump on breast of a child?",
"src": "Patient: My 5 year old son has been complaining of chest pains when he breathes deep breaths and I found a hard boney lump on his left breast, when I touch it he does not complain that it hurts but it is obviously a lump. He has not been hurt in the chest area but the lump is worrisome, what could it be? Doctor: Hello. I just read through your question. The lump on his left breast could be a lymph node. these typically swell in the presence of infection. They are usually not worrisome. The more concerning thing is the chest with breathing. This could indicate a mild infection in the lungs. if so, it can be treated with antibiotic. I recommend consulting with your doctor so that he can be examined, the correct diagnosis made, and the appropriate treatment plan implemented."
},
{
"id": 2092,
"tgt": "How soon can i get pregnant after AKT-4 treatment for TB?",
"src": "Patient: hI MYSELF IS XYZ I M SUFFERING FROM TB AND MY DOCTOR HAS SUGGESTED ME FOR AKT - 4 FOR 3 MONTH WILL I BE ABLE TO CONCEIVED AFTER THAT , I HAVE ALSO GONE THROUFH THE LAPROSCOPY MY ALL RESULTS WERE READY TO SET BUT ONLY TB HAS OCCURED IN MY CASE... HOW SOON CAN I GET PREGNANT Doctor: Hi you can get pregnant after completing the course of your treatment. If everything was fine you can try naturally for 3 months. If it doesn't work then you can go for some medicines which will help you in getting pregnant. Do a semen analysis of your husband. Hope I have answered your question."
},
{
"id": 212706,
"tgt": "Scoliosis, depression, on norco, lexapro. Suggestions?",
"src": "Patient: i have moderate to severe neck and upper and middle back pain . i see a physical therapist 2x a week and take norco10/325 BID i was diagnosed with scoliosis after seeing a chiropractor aprox 5 months ago. the only time i have been hospitalized was in aug 2012 for childbirth , I am 29 years old and i take lexapro for depression due to the stress of having a newborn, a mother who I take care of who is terminal becasue of ALS and i am in the middle of a divorce Doctor: Hi there ~ I am unsure of what you are asking for in terms of suggestions here. However, I understand that you are going through a lot. Having depression can be stressful. Physical problems like the neck pain and upper/middle back pain that you have may be adding to your stressors, along with the stress of divorce. It may help to increase the dose of lexapro, however you will need to see a qualified psychiatrist for the same. Norco, though helpful with pain may alter your mood. It would be best to seek the advice of your psychiatrist who knows of your complete medical history. I hope this helped. Take care."
},
{
"id": 184403,
"tgt": "Suggest remedy for tender black spot on the surface layer of tongue",
"src": "Patient: Hello gd morning my wife she had black spot on the surface layer of tongue for almost 8 - 9 months treatment done but no cure. when she eat food it pain and that black portion has become little thin as compare to other part of tongue. Size of black spot is 1 cm round. please suggest me what to do? Doctor: Hello, Thanks for consulting HCM, Read your query, as your wife had tender black spot on tongue this can be due to poor oral hygiene there is deposition of food , drug induced , depapillation of papillae , median romboid glositis , geographic tongue . I will suggest you to consult oral surgeon and go for proper examination of oral cavity and tongue and rule out the cause of discoloration of tongue , if needed go for investigation Cytology of blackish portion of tongue . In meantime maintain proper oral hygiene , use tongue scraper to clean your tongue , use mouthwash regularly. Hope this will help youWishing you good health.Regards, Dr. Priyanka tiwari"
},
{
"id": 8197,
"tgt": "My friend has pimples, how to get rid of it ?",
"src": "Patient: hi, my friend has pimples, she used toothpaste on them, but when she woke up in the morning, she has blood on the face , and she has scar what can she do now ? Doctor: i think u have that problem be bold to face any probs........"
},
{
"id": 35569,
"tgt": "What causes pain on the wound while applying placentrex gel?",
"src": "Patient: Good Evening Doctor, I'm Srini from india. Recently i met in an accident and i have wound in legs (Both left and right). I had TT injection and doctor gave some antibiotic and placentrex gel. I'm using for three days. When i clean the wound with water added and apply the placentrex gel, it hurts a little. Does it usually happen or should i check my doctor. Doctor: Hello and Welcome to \u2018Ask A Doctor\u2019 service.I have reviewed your query and here is my advice.With placentrex gel, the burning sensation is normal, nothing to worry.Try to avoid going outdoors with wound uncovered.You can wait for 5 days and report back to doctor for followup.Hope I have answered your query. Let me know if I can assist you further.Regards,Dr. Sathish Chandra M R"
},
{
"id": 107461,
"tgt": "Suggest treatment for lower back pain",
"src": "Patient: I ve suffered lower back pain for 2 years .I ve been to doctor .physio but not any great results .I find it hard to stand for any length of time and it tires easily at work and by end of shift I struggle .also getting up from bed or even turning at night can be very painful Doctor: Hi,Take homoepathic treatment Nux Vom 200 4 pills 2 times Calc fluor 30x 4pills 3 times, Mag phosphoricum 30x 4 pills 3 times for 3 days and revert back for further treatment."
},
{
"id": 131440,
"tgt": "How to overcome paget s disease?",
"src": "Patient: My son is 40 years old and is having a sericous bone problem, He s having growth growing in his joints. His right wrist was cleaned twice in surgery and then the third time they removed the bone in his wrist. Now he has a growth in is elbow on the right side again. No one will tell him what it is or how or why it s happening? They just what to do surgery and leave it at that..Because he doesn t want surgery he wants to know what it is and how to fix it. He believes he has Paget s desease? is there any way you can help us..He has Delaware Physician Care insurance. He has several growth in his body and one is in his rib cage? He s having sever pain from all this and the Dr. won t give him even tylonal for the pain..Something is really wrong with the Doctors here in Delaware...They lost their passion for the patient and greed for the green. I m sorry but I don t have the money right now. I can t do anything til the first. I lost my husband a year ago this month and it s been hard..I m sorry for wasting your time.. Doctor: HiSome people suffer multiple exostosis for no specific reason known yet.All exostosis needn't be removed unless hurting,painful,overgrowing,interfering with movements and not responsive to pain meds,anti inflammatory drugs.Get serum calcium and vitaminD3 levels tested in blood.,if some results not in normal range may be supplemented.Also get parathyroid tests done, parathormone levels in blood at times are correlated with bone growth issues.hypercalcemia due to this is treated with biphosphonates like fosamax plus D once a weekSee a endocrinologist also"
},
{
"id": 82440,
"tgt": "Can habit of chewing tobacco cause shortness of breath and chest pain?",
"src": "Patient: I ve been chewing tobacco for about 15 years. In the last couple of years I have started to have shortness of breath problems and some chest pain..The heart doctor says my heart is strong with no problems..can the chewing be my problem after all this time Doctor: Thanks for your question on HCM. In my opinion you should get done ECG and TMT (Trade Mill Test) again. And consult cardiologist to rule out cardiac cause.If this is normal than tobacco chewing can cause bronchitis. So get done chest x ray and PFT (Pulmonary Function Test).And better to avoid tobacco as tobacco contains nicotine which is harmful for heart and lungs and can cause chest pain and shortness of breath.So quit tobacco as soon as possible."
},
{
"id": 197874,
"tgt": "Any suggestion for having erection problem, small penis?",
"src": "Patient: I hav penis problem especially erection one since last year. Owing to reasons better known to you my penis had been de-shaped and reduced to minimum size. Initially I felt slight problem in passing urine while at the later stage there was no pain. A one year has to passed and now there is some improvement in my penis size but still it is not in its orgininal size. Please advice. Zaheer Shehzad Doctor: Dear, We understand your concernsI went through your details. You have not mentioned the size of your penis. The average of length of penis is 4.5 inches and the average girth is 1.5 inches. You will be able to satisfy any woman across the world with a penis size of 2 inch in erect condition. Woman vagina contains pleasure nerve endings only upto 2 inches from the vaginal opening. Erectile dysfunction (ED) or impotence is sexual dysfunction characterized by the inability to develop or maintain an erection of the penis during sexual activity. This happens due to organic as well as psychological causes. You should rule out organic causes first. Please consult an urologist for investigation.If you require more of my help in this aspect, please use this URL. http://goo.gl/aYW2pR. Make sure that you include every minute detail possible. Hope this answers your query. Further clarifications are welcome.Good luck. Take care."
},
{
"id": 138300,
"tgt": "What causes weakness in right arm compared to the left?",
"src": "Patient: One in a while during the day but not every day for a few minutes I get what feels like my right arm is slightly less strong than my left. I work out almost every day with no problem. It just feels like there is very slightly less control but I can do everything . What do you think? I an 62. Have had stress tests,and all things heart related and all come out normal. What do you think? Doctor: Hello, I have studied your case. . Weakness in one arm with pain can be caused by either cervical spondylitis or rotator cuff tear.I would suggest you to meet a physician and confirm this diagnosis. You might need to take MRI of the neck and shoulder to confirm the diagnosis.I hope this answer will be useful for you. Let me know if there is any other followup questions. thanks for using healthcare magic."
},
{
"id": 178599,
"tgt": "Suggest treatment for deafness in child",
"src": "Patient: Respected Sir, I have a six year s daughter s baby Purva Mittal. Unfortunately she has not been listen since birth as a normal child. Her hearing loss above 95 db (approx). She has hear only loudly sound due to this she has not been develop her speech. She has been regularly used from 3 years hearing Aid & speech therapy, but the result is not satisfactory. Please advice me if any medicine or treatment of her deafness be cure. Thanks & regards. Ravi Mittal. E-mail address - YYYY@YYYY Mobile No. 0000. Doctor: sir,thank u fr contacting us.1. hearing loss in your child is >95 db(appx) which amounts to severe to profound sensorineural hearing loss2. hearing aids & speech therapy have to be started at the earliest for such children which would help in the normal development of speech as the child grows,and here as u say it has not helped much ,which means she needs the next line of treatment i.e cognitive implants which will help in this case3.Get to see an ent doctor at the earliest regarding implants and collaborate with a speech and hearing specialist as well,so that both can work together dont get disheartened,it will be fine...just get to a ent doctor soon without delay,as any further delay can delay speech development in the childregards,dr.sowmya"
},
{
"id": 68953,
"tgt": "What causes oval shaped soft movable lump above right side rib?",
"src": "Patient: Ive noticed a oval shape soft movable lump above right side rib area. This was noticed a couple of days . I was diagnosised with melonoma six years ago and have had a right femur mole removed then and they were able to get it all. Have had followups and no recurrence. Also' recently i have noticed discomfort between my shoulder blades. Should i be concerned? Doctor: I DON'T THINK SO. THIS MAY BE A COLD ABSCESS DUE TO TUBERCULOSIS .THERE IS A REMOTE POSSIBILITY OF THIS BEING A METASTASIS OF MELANOMA ALSO. THEREFORE IT IS ADVISABLE TO GO BACK TO YOUR SURGEON WHO HAD DIAGNOSED MELANOMA . HE MAY BE WELL ABLE TO TREAT YOUR TUBERCULOSIS IN CSE IT WERE SO.THANK YOU."
},
{
"id": 108341,
"tgt": "What causes pain in back with tenderness?",
"src": "Patient: I have been experiencing back pain along with tenderness on my right side. I was told I had several gall stones. Most recently I noticed about six different bruiseds close to my belly button and don't know where they came from. I am 62 and female should I be asking the doctor about this latest development. Doctor: Bruises are to be of concern if you have gall stones, it would be worthwhile to consult your gastro surgeon on this early. Hope you heal yourself soon best of all"
},
{
"id": 31671,
"tgt": "What causes persistent fever while on 'luvox'?",
"src": "Patient: Hi, I've been running a low grade fever 99.3-101.00 for over a month now. I have hay fever, take luvox 200 mg daily, but am in generally good health. Should I see my doctor about the fever or could it be a medication side effect or a side effect of the hay fever? Doctor: Hi Dear,Welcome to HCM.Understanding your concern. As per your query you have persistent fever while on 'luvox\" which is due to viral infection leading to respiratory tract infection. It could be due to exposure to allergens on frequent basis. Need not to worry. I would suggest you to visit general physician once and get it examined and start treatment after proper diagnosis. You should take full rest to and have sound sleep. You should do warm saline gargles 3 to 4 times a day. Take ibuprofen one tablet twice daily. Drink lukewarm water throughout the day as it can provide relief. You should take soft foods. Apply mist humidifiers in room. Get blood test done to detect level of infection. You can take Meftal P to for fever. Hope your concern has been resolved.Get Well Soon.Best Wishes,Dr. Harry Maheshwari"
},
{
"id": 61310,
"tgt": "What do lumps under the arch of the foot indicate?",
"src": "Patient: yes sir , i am having problems bearing down and put full weight on it. the area is the arch of my foot. it feels like their is a rock in my shoe . after checkin the shoe nothin. had one of my youngns look at it and she felt 2-3 little hard knots under the skin. found multiple more different parts of both feet. i am experiencing a tough flair-up of rheumatiod arthritis. first in a few years and more intense and new areas of my body are also being effected all areas that have been damaged in someway . so not sure if the two are connected.. well thank you for your time Hill,Christopher T. (Chris) Doctor: Hi Chris Thanks for using Healthcaremagic.comI have evaluated your query thoroughly .* These are most probably callus formations over the sole part .* However clinical confirmation with the physical examination or a photo pic of high resolution would be of immense help to confirm and guide further precisely .Wishing you fine recovery from the same .Regards ."
},
{
"id": 137929,
"tgt": "What causes ribcage area pain radiating to the back and shoulder?",
"src": "Patient: I have had right side pain for quite some time. I went to the OBGYN and she did an exam and found nothing wrong. I went to another doctor and he told me I had to get more fiber. I have been eating 20 grams daily and drinking a lot of water. I had pain since then. I had two ultrasounds and the first ruled out appendicitis and found my gallbladder wall was thickening. The second found I have fibroid tumors/ovarian cysts. I have had pain since then. This pain is on my right side but higher than my pelvic pain. This pain is in my right rib area and it radiates to my lower and middle back and under my shoulder blade. I am not sure what this is. Should I go to a doctor about this? Doctor: Hello,Welcome, and thanks for sharing your concern I went through your query, and I feel, you need to understand that the character of pain you describe could arise only by internal abdominal viscera pathology.I would recommend you to get your CT abdomen with pelvis with contrast done and that is a very sensitive investigation to rule out any anatomic abnormality. moreover getting some enzyme tests can also help delineate the problem. See a gastroenterologist for this purpose.I hope my advice would have been useful, in decision making regarding your treatment, still if you have any clarifications or doubts feel free to contact back.Thanks."
},
{
"id": 65007,
"tgt": "What is the hard mass in the front of neck?",
"src": "Patient: I am 44 years old, 5ft8 inches, 75 kgs, and healthy. I noticed this strange, almost gristly feeling in my neck. When I feel it I can push this hardness from side to side around the front of my throat and it makes a kind of grinding noise against my throat. It's very hard to explain, is not painful. But I have never noticed it before... is it something I should be concerned about? Doctor: Hi,dear,thanks for the query.AFter indepth study of your query,In my opinion, the tumour appears to be thyroid nodule-? needs to be confirmed by a surgical opinion and FNAC repor.The problem is alarming and needs to be investigated and attended at the earliest.So act fast and be concerned about it seriously.Thnks.Hope your concern is sorted out to your satisfaction.Wellcome again."
},
{
"id": 136191,
"tgt": "Suggest treatment for severe forearm pain and hand pain",
"src": "Patient: Hi Dr. Rynne, About a couple hours ago, I noticed that my right hand was not as strong as it normally is. And a few minutes ago I tried snapping my finger and realized that it caused a slight pain in the tendons about a quarter to midway down my forearm, and also felt the same pain when I shook my father s hand. When I woke up this morning I was fine, but this happened after I was moving some heavy furniture. Could it be possible that I have strained/pulled a tendon? Doctor: Hello, I have studied your case.Due to compression of this nerve root there can be hand weakness.I will advise you to MRI cervical spine for better diagnosis.It can be due to various causes like sudden jerk to spine, due to facet arthritis, degenerative.For these symptoms analgesic and neurotropic medication can be started.Till time, avoid lifting weights, Sit with support to back. You can consult physiotherapist for help.Physiotherapy like ultrasound and interferential therapy will give quick relief.I will advise to check your vit B12 and vit D3 level.Local injection may help if pain remains persistent.Hope this answers your query. If you have additional questions or follow up queries then please do not hesitate in writing to us. I will be happy to answer your queries. If you are satisfied with answer do not hesitate to rate this answer at end of discussion. Wishing you good health.Take care."
},
{
"id": 196119,
"tgt": "Suggest remedy for swollen nipple",
"src": "Patient: my right nipple has a swollen knot on the bottom half. considering i am a man of 35 and have had pierced nipples for 12 years without problems i am concerned. it is painful yet has no discharge or anything, just a hard painful knot that developed in a couple of days. Doctor: hiii.welcome to healthcare magic.my opinion is that, you have inflammation in nipples.1.dont scratch the lesion.2.maintain good hygiene around the nipple.3.apply antibiotic cream like fusiderm ointment twice daily on the area for 7 days.4.take analgesics like tablet ibuprofen twice daily for 7 days to control pain.thank you.hope my answer would have helped you."
},
{
"id": 51725,
"tgt": "Transplantation patient-pus formation on the upper right side of hip",
"src": "Patient: transplantation patient-pus formation on the upper right side of the hip area i am from hyderabad INDIA..my mother was on dialysis because of the kidney shrinkage when she was 36 and had undergone kidney transplantation in 2004 when she was 40....she has now become diabetic after her transplantation.....she also has blood pressure since she was 34.....everything is fine....but after a year of the transplant there has been pus formation above the transplanted area....on the right hip side..at first when it was formed the doctors removed it surgically...the area is not healed yet n the pus oozes out from it even today and there has been 3 ,4 other points in the same area where the pus has made its way to come out....n every month it happens, any point of that area swells due to the pus and after a lot of pain it bursts n the pus oozes out in a day or two...its been 6 years since then that it has not stopped....all the reports are clear....the doctors have seen the ct-scan and the mri reports also....but are unable to figure out the cause...they say that it might be from the native kidney....but are still not sure that it can be fully cured even if the native kidney is removed....she has taken many injections for that and is still taking antibiotics...but it is still the same.....we clean and bandage it and dress it every alternate day with betadine....the pus only seems to be more every day....and there is no sign of it becoming less or stopping....and 1 more thing...during her dialysis her stomach would swell and because of that she had to drain out the water also....and also her menstural cycle is very irregular ....once in a year and sometimes it never occurs..can this be the cause of the pus not stopping....or is it something else.....please reply... Doctor: If the discharge you are mentioning is of infective origin, then it should have responded to antibiotics. I would suggest you to get a pus culuture and sensitivity test done to know the status of the infection if any. That should help her. And for menstrual problems, it could be the added stress causing it and does not have specific relation as such."
},
{
"id": 92108,
"tgt": "What can help persistent pain in stomach and back?",
"src": "Patient: I AM HAVING PAIN TOWARDS THE TOP OF MY STOMACH IT IS ABOUT THE CENTER OF MY BODY UNDER MY RIGHT BREAST. THE PAIN ALSO HURTS IN MY BACK . THIS PAIN HAS BEEN GOING ON NOW FOR THREE DAYS. DO YOU HAVE ANY IDEA WHAT THIS MIGHT BE?I AM A 64 YEAR OLD WOMAN AND MY NAME IS ZELLA. Doctor: Hi. Pain in the abdomen at a site you have mentioned with referral to the back can be due to inflammation of liver or gall bladder and surrounding structures. A good sonography can help the diagnosis. Are there any associated symptoms like fever/ vomiting ?"
},
{
"id": 169008,
"tgt": "What causes swollen, painful, brown spot on penis?",
"src": "Patient: hi my2 year old son came out sayin he penis hurt.i ask to show me were it hurt he pointed to a brown small mark on he penis,wich appeared about a month ago but i thought it was a birthmark,so i pulled bk he full skin and wen i got to the brown mark he cryed and puss came out and its abit swollen on the side of the brown mark Doctor: Hi...Thank you for consulting in Health Care magic. Skin conditions are best diagnosed only after seeing directly. I suggest you to upload photographs of the same on this website, so that I can guide you scientifically. Hope my answer was helpful for you. I am happy to help any time. Further clarifications and consultations on Health care magic are welcome. If you do not have any clarifications, you can close the discussion and rate the answer. Wish your kid good health.Dr. Sumanth MBBS., DCH., DNB (Paed).,"
},
{
"id": 126698,
"tgt": "What could cause a pulling sensation behind the calf after a workout?",
"src": "Patient: Anytime I workout either hiking, on treadmill or strength workout, I get a pull from the back of my calf and also pain which renders me unable to workout. I continuously stretch and it does not help. more like I have the strength to go n but my legs fail me. what could this be. I feel it more where the ankle muscle connects with my calf. Doctor: Hi, It might be a simple muscle spasm. You can take analgesics like Acetaminophen or Diclofenac for pain relief. Generally, it will settle in a couple of days. If persists, you can consult an orthopedician and get evaluated. Hope I have answered your query. Let me know if I can assist you further. Regards, Dr. Shinas Hussain, General & Family Physician"
},
{
"id": 87415,
"tgt": "What causes persistent pain and inflammation in the abdomen?",
"src": "Patient: I have stopped taking Phentermine around last July I was having real bad steady stomach pain i had an upper GI done the inside of my stomach is really red the GI Dr asked what have i been taking i said Phentermine that is it he said no aspirin or anything with aspirin i said no he prescribed me Prilosec it really didnt help i went to the Emergency room several times they gave me a Rx for carafate since then i went to another Dr (family Dr) he told me first thing to try is take more carafate and prilosec it has helped since then i havent had any pain and i have been able to eat the foods that would normally irritate my stomach and make it hurt when the pain happened it would go for hrs and on a scale from 1-10 it was a 20 Doctor: Hi,Looking to the history it seems that you are having severe gastritis giving this problem and having red inflamed stomach mucosa.Take pentaprazole daily for some time.Change your food habit.Take light diet like milk, rice, buttermilk etc.Avoid fried, chillies and junk food.Avoid stress.Avoid taking any pain reliever medicines.Ok and take care."
},
{
"id": 164689,
"tgt": "What causes acute knee pain with inflammation and infection?",
"src": "Patient: My 9 year old daughter has had knee pain for 4 weeks not. Her CRP and ESR were high: CRP=1.5, ESR=35. Indicating inflamation or infection according to her Dr. This knee pain began suddenly about 6 months ago. Was gone after a about 2 months and recently returned and has been present for 3 weeks now. She wakes up in pain and stays in pain all day and worse at bedtime. Her Dr. is baffled. X-rays of her knee and hips have been done with no indication of the cause. She is scheduled to have another bone scan done, which was already done 5 months ago and it was clear. She has a very high threshold for pain, due to many past bone fractures from sports injuries so I don t feel she is exagerating or making it up. She is depressed from being in pain 24/7, I give her 2 xtra strength tylenol at a time. She is tall for her age and also slightly over weight all though she has not gained any weight in a year according to her Dr. Do you have any suggestions to what might be causing this. This happened all of a sudden. No injury either time, she woke up with the pain. Doctor: HiThere can be many possibilities which need to be investigated. Overweight, previous injuries, further examination and investigation to see if any other organ is inflamed will give an insight . She has this thing going on chronically so you get opinion of a rheumatologist."
},
{
"id": 142090,
"tgt": "Why vision, hearing, face and neck feeling lost with high functioning autism and epilepsy?",
"src": "Patient: my daughter is 22 years old with high functioning autism, tourtettes, and epilepsy. She had an MRI, CT and EEG's in the past 2 weeks for increased seizures with no explanation. She has since then lost vision , hearing, feeling in face and neck, and has slurred speech (right side). Her vision is worsening in left eye. Neurology is not moving fast enough to find out what is going on and she is getting worse. I don't know where to take her. She has a swallow study today because she chokes on liquid. I feel I should make my own appointment today at a local ophthalmologist and maybe an ENT? Doctor: Hello!Welcome on Healthcaremagic!I would recommend performing a brain MRI to check for possible brain changes and an EMG to check for possible myopathy (considering the fact that she can no swallow). Consulting with an ophthalmologist is also necessary to check for possible eye or retinal disorders. There are many genetic syndromes which can lead to this clinical situation. Consulting with a geneticist could be helpful too. Hope you will find this answer helpful!Kind regards, Dr. Aida"
},
{
"id": 49420,
"tgt": "Does kidney failure correlate to emotional swings?",
"src": "Patient: Hello Dr.m - My father-in-law is 96 yrs old and his kidneys are failing him. He has opted to not do dialysis at his age and simply live out his days as best possible. My question is what are we in for? We know that death is the end result but we are confused as to what to expect between now and then. He is coherent, not bed ridden, but he is extreme;y agitated and anxious--much more so than usual. Is this an emotional response to the situation or a a physical reaction? And what can we expect as the situation worsens? We desperately need info as the doctor tells us to go to WWW.WWWW.WW and leaves it at that. So here we are. Can you help? Thank you kindly, Bari Dailey Doctor: Hi, Welcome to Health care magic forum. When your father in law is denying the dialysis, the end products of metabolism are accumulating in his body, and cause the different pains, and mood changes etc. To deny the dialysis may also be due to this condition, ask him to face the dialysis once and observe the difference, and if not felt happy he can stop having it.Because he will be comfortable after the dialysis, and he may be convinced. Give him more of green leafy vegetables, pulses, sprouts,and protein rich foods to make him comfortable. Wishing for a quick and complete recovery. Thank you."
},
{
"id": 40270,
"tgt": "Should i consult a doctor as my son is having a cut on his finger which is swollen and filled with pus?",
"src": "Patient: My three year old son has a sore/cut on his left little finger at the side of the knuckle. Is is very swollen and was full of pus which has now popped and is open. I have cleaned it and put on a plaster I was just wondering how serious it was, whether there is likely to be something in it that requires hospital or whether a trip to the GP would be sufficient. Doctor: nothing to worry, give some time it will heal by 7 days. apply antiseptic cream thrice daily, make sure he does not hurt the same site again"
},
{
"id": 210134,
"tgt": "Is it possible to see small worms come out of your head?",
"src": "Patient: Is it possible to see small worms come out of your head? My sister has shaved her head and wears a stocking on her head before she wears her wig. She says she can see and hear them Bakersfield California think she is crazy and keeps giving her pills for illusions Doctor: DearWe understand your concernsI went through your details. I suggest you not to worry much. It is possible to see small worms coming out of a persons head even though actually it is not the case. This type of behavior originates from psychoses. Illusions and hallucinations are normally their companion. Because of numerous such illusions, they settle to one illusion which they think they can overcome with their own trick. And they live with it. Treatment with the help of psychotropic drugs by a psychiatrist is the remedy. The treatment can also be augmented with psychotherapy techniques like Cognitive Behavioral Therapy (CBT).For more psychotherapy techniques, You can post a direct question to me in this website. Include every detail as much as you can. I shall prescribe some psychotherapy techniques to control your condition. I am sure that the techniques should be a success.Hope this answers your query. Available for further clarifications.Good luck."
},
{
"id": 182075,
"tgt": "Suggest treatment for tooth infection and swelling",
"src": "Patient: About 20 days back i had tooth infection which as a result my right cheek along with affected tooth of upper side got swelled the dentist treated with some antibiotic medicines and i got recovered. but for the past 2 days or so i am feeling facial pain. so what do you think it is sinus problem or still tooth infection is there. please note that i dont have pain in the tooth and now dentist has also diagnosed that there is no infection in the tooth. kindly inform/advise what is that pain. Moreover my throat is also sensitive and a thin phlegm needs to be swallowed very frequently for the past many months. so please advise me. Thanks Rizwan Ahmad Doctor: Hello,Thanks for consulting HCMRead your query this pain can be due to sinus related also I will suggest you to consult Ent surgeon for examination of throat and if needed go for investigations like PNS view . Inmeantime do warm saline rinses two - three times a day amd you can take medicine like Antiallegic tablets by consulting with your local doctor .Hope this will help you."
},
{
"id": 4129,
"tgt": "Will endometrium size of 16 mm help in getting pregnant?",
"src": "Patient: hi doctor,I am alphonsa 31 years old with height of 5.7 and my weight is 60 kg.my marriage was on 11.05.2009.i am just started my treatment for infertility.Doctor said my endometrium size is 16 mm in the 14th day of my period and the rest is fine.I would like to know ,if endometrium is 16 mm i will be able to concieve? Doctor: Hi,I read your query and I understand your concerns.Following is my reply:1) Endometrium is very good.2) Please get semen analysis for husband and HSG for yourselfLet me know if you have anymore questions.Regards,Dr. Mahesh Koregol"
},
{
"id": 80165,
"tgt": "How to treat cold with wheezing?",
"src": "Patient: My son showed up at the house and could hardly breathe. Has a president cough that comes and goes. Wheezing really bad. Coughing up green plhem lately he said. Has no spleen and was born with Hyland membrane disease thirty two years ago. Has inhaler was wondering if you think he has asthma. Doctor: HIWell come to HCMI really appreciate your concern, this could be allergic condition may with secondary infection, condition can be managed in two way one is get done the blood test to confirm the infection second antibiotic can be administered just on tentative basis and best drug would be \"Tab Azithromycin 500 mg once in day for three days, and of course antihistamine would be prime drug for this, and best would be \"Tab Loratadine with Pseudoephedrine twice in day, hope this helps, take care."
},
{
"id": 122159,
"tgt": "How can pain and bruising caused by fracture be healed?",
"src": "Patient: I was in a car accident three months ago with a open dislocated ankle. I hit both my knees on the dash board. They were scraped and bruised but no fracture. It still hurts to kneel on my left knee. It hurts on the medial side of my patrols. Any thoughts on what my pain is? Doctor: Hello, I would explain that your symptoms could be related to a meniscus rupture (inside the joint). For this reason, I recommend consulting with an orthopedist and performing a knee joint MRI in order to examine better your joint ligaments and meniscus. Hope I have answered your query. Let me know if I can assist you further. Regards, Dr. Ilir Sharka, Cardiologist"
},
{
"id": 35205,
"tgt": "What causes pain and itching in hands after flu?",
"src": "Patient: I am recovering from the flu. I was very sore all over, feverish and bed ridden for 2 days. Today I'm eating solid foods and feeling no pain, except in my hands! It started with my fingertips, and now my whole hands are itchy and really painfully sore. Doctor: HIWell come to HCMI really appreciate your concern, if this is flue then this couple be self limiting condition may not need any treatment, you need to be in bed for couple of days, drink more water, and stop worrying about this, this would be fine soon, hope information helps, take care."
},
{
"id": 126644,
"tgt": "What is recommended apart from amino acid supplements and increased protein intake for pain in the legs while sitting?",
"src": "Patient: I exercise quite a bit. Run 4 days a week, 4-9 miles each time. Teach bootcamp classes, body sculpt classes, HIIT classes. Lotsa pylometrics. I drink 80 oz of water a day. no sodas. Take Advocare products such as rehydrate, spark, catalyst. My legs ache when I am sitting in the car or at a desk. I ve recently added more amino acid supplements to my drinks. I drink protein shakes, consume protein with meals and protein bars for snacks. Anything else you would recommend? Doctor: Hello and Welcome to \u2018Ask A Doctor\u2019 service. I have reviewed your query and here is my advice. This may be caused by three things. Neuropathy or ischemic changes in legs or celulitis. Cellulitis which is inflammation of subcutaneous tissues and may develop into diabetic foot with redness or pain in legs. Neuropathy may also be caused by diabetes but also other metabolic causes may be present or injury of spine. It is presented with burning pain without skin changes. Last, ischemic changes happens due to poor circulation in legs. If this is not your case then these symptoms are indicating circulatory problems in legs which are common in this age and there may be present atherosclerosis that obstructs blood vessels and causes pain and blisters to form. You should do Doppler ultrasound of the leg or CT angiography to rule out such obstructive changes and then it can be treated properly by medications or by surgical and radiological procedures. Hope I have answered your query. Let me know if I can assist you further."
},
{
"id": 158057,
"tgt": "Have melanoma on 11th vertebra. Side effects and prognosis of radiation?",
"src": "Patient: My step-father has melanoma on his 11th vertebra. He is 85 years old. He will be having radiation on it next week & I want to know about side effects & prognosis , etc. He also has a golf ball sized melanoma on his lung , which is inoperable & they are not doing radiation on that for some reason. I want to know what his quality of life will be & how long does he have & how bad will he get towards the end. Thank you. Janice Doctor: Your stepfather has spine metastasis from melanoma which is an uncommon event, it can pose a complex management problem. Pain is the most common presenting symptom. The radiographic diagnosis is generally made easily by plain radiographs, computed tomography or magnetic resonance imaging, with the most frequent finding being a destructive lesion. Bone scan gave false-negative results 15% of the time. The median survival is about 4 months.It is concluded that melanoma metastatic to the spine represents a late event in the evolution of this illness. Palliation should be the goal of treatment, but symptom management should be individualized, bearing in mind the short anticipated survival of these patients.Regards and take care"
},
{
"id": 151350,
"tgt": "Unilateral pain from forehead, ear and moving to neck, nerve stretching. CT scan, MRI normal. Treatment?",
"src": "Patient: Hey, I am XXXX and my age is 19years. I have got a neurological problem. I had regular unilateral pain nd it starts from sides of my forehead and continues till ear and then to neck as if a nerve is getting stretched!! I don t know I have consulted atleast 5 doctors and hv gone under ct scan , mri ,mri venogram bt results were positive bt then also pain is like 20 to 22 hrs a day !! Could you please help me with this . Is there anything serious let me know Doctor: Hello, By 'results were positive' probably you meant that MRI was normal. If that is the case then it is not a serious thing. There is a possibility that the nerve supplying the area of face is getting compressed and because of that you are having these symptoms. There are many medications available for such pain. Instead of visiting many doctor you should stick to one neurologist and let him adjust the medications till you get best relief. Good luck."
},
{
"id": 203584,
"tgt": "Is there a difference between normal ejaculation and masturbation and does muscles become weak for a while after ejaculation?",
"src": "Patient: hi . are there any differences between normal Ejaculation or a masturbation ? Ejaculation make me tired and I can't workout and for example lift ( as a sport ) . I think it has effect on muscles . how much I need after an Ejaculation before I start workout and go to gym ?? Doctor: DearWe understand your concernsI went through your details. I suggest you not to worry much. Masturbation is not a skin. Masturbation is not unhealthy until you overdo it. Masturbation is natural and safe and even animals do masturbate. It is quite normal after masturbation, the penis musles become flaccid or loose. Penis erection happens due to rushing of blood to th penis capillaries and pressurised availability. After masturbation, the blood drains and due to the effort involved, it is common tobe feel tired. This tiredness is almost equivalent to a 500 metres jogging. Required rest period is around 30 minutes.But because of the mental stress involved, it is better to masturbate once a day before sleep. It keeps your mind relaxed. For psychological and sexological counseling visit http://psychocure.webs.com/Hope this answers your query. Avilable for further clarifications.Goo luck."
},
{
"id": 103078,
"tgt": "Bronchitis, basic level asthma. Prescribed levelin inhaler, maxiflo inhaler. Possible steroid contained?",
"src": "Patient: Hello sir this is Pushpal Ganguly from Kolkata and am suffering from bronchitis & basic level Asma. I have been prescribed by my doctor Levelin inhaler of the 200 metered doses (levosalbutamol Inhaler 50 mcg and Maxiflo inhaler 250. Since it is the 1st time I will be using inhaler so please tell me whether these inhalers contain steroids. As one of my relative told me that. Please guide me sir in this regard Doctor: hello dearthanks for your query at HCMlevelin contains levosalbutamol it is a bronchodilator not a steroidmaxiflo contain fluticasone it is a steroid , but these steroids are locally acting on respiratory system not causing any adverse effects on other system.they are very safe to use , and you can start on them regards"
},
{
"id": 92504,
"tgt": "Have belly button pain and pulse like sensation. Is it a result of lifting heavy things?",
"src": "Patient: I do some heavy lifting at work and think I may have overdone it . I have had pain above and behind my belly button all week when I bend over or lift things up, and now have noticed a heartbeat-like pulse when I press down on the area . I have to throw freight tonight and worry I'm gonna really mess myself up. Any thoughts ? Doctor: Hi and welcome to HCM,thank you for your query.Yes. it can be umbilical hernia and this is the first thing to think of.however it can be caused by some other subcutaneous lesions or some intraabdominal patology so you should do at least ultrasound to verify is there something wrong. Such pain may be felt in stomach or gallbladder issues.Wish you good health. Regards"
},
{
"id": 160151,
"tgt": "Lung crystallization on CT scan, what does this mean ?",
"src": "Patient: I had a CT scan of my abdomen and pelvis. i was told today they seen crystallization at the lung base. I have to wait to see my regular doctor . what does this mean ? Doctor: Welcome to Healthcare Magic I believe the word is consolidation. If Yes, it means there is lung infection of pneumonia at the lower part of the lungs and will require probably antibiotic treatment after few more tests."
},
{
"id": 69349,
"tgt": "Could lumps at back of head and neck be due to having the habit of picking at head when stressed?",
"src": "Patient: Hi, when i get anxious and stressed i pick at my head quite badly.. I have noticed a couple of lumps at the back of my head and neck--would they most likely be swollen lymph nodes? Also, lately I feel very tired and unwell..is that just because i am stressed and have a lot on--or could all the pickin gof my head be making me feel unwell? Doctor: Hello!Thank you for the query.Picking at your head can cause some inflammation and in that way stimulate lymph nodes enlargement. However as you have tiredness, other possible reasons of enlarged lymph nodes should be also considered. Especially if you also have weight loss and night sweating.I suggest you to consult your doctor, have physical examination and ultrasound of this lumps. In case of enlarged lymph nodes biopsy might be necessary.Hope this will help.Regards."
},
{
"id": 194876,
"tgt": "What causes scar like spots on head of penis?",
"src": "Patient: Hi Dr., Around 10 yes back, I happened to be in bed with a friend. She and I had our limitation s till oral and no further. One night, I just happened to find out a little late when I felt that my penis head was inside her for the first time and that too without protection. I somehow didn t want to go any further and was out of her in seconds. Next surprise for me was that she was having her periods. A couple of days later I happened to notice 2 spots on the head. They were not red , not shiny in fact seemed just like a scar about quarter inch wide each. The color you can say was of the skin gone a little dark, which was the only thing to make me notice it. Ever since, no pain no itch, no size difference, in fact it is just there harmless. For the last couple of days, the scars got a little shinny, the skin got somewhat sensitive as I feel some irritation when it rubs against a cloth, still no pain. I got worried and got VDRL done just in case, result came negative. Please guide me as to what it is, which has been there for 10 yrs unnoticed, and now is irritatating me against cloth rubbing. Will wait Thanking you in advance Kamran Doctor: Hi, It can be due to eczema. As the first line of management, you can apply topical steroid like Betamethasone for symptomatic relief. If symptoms persist better to consult a dermatologist and get evaluated. Hope I have answered your query. Let me know if I can assist you further. Regards, Dr. Shinas Hussain, General & Family Physician"
},
{
"id": 24602,
"tgt": "Is increased heart rate and blood pressure after breaking shoulder normal?",
"src": "Patient: Broke my shoulder-- therapy both my hands turned blue. Checked out next day heart rate 120 and blood pressure high. Had ct scan done okay. Still have blue hands at times and today my legs feel heavy and tired. What should I do and what is my problem. I m67 and very active. Doctor: you may have hypertension before it seems but diagnosed now I think no need to worry just keep checking bp levels frequently if they are raised For minimum 3situation and consult general physician and take advice"
},
{
"id": 156576,
"tgt": "Suffering from lung cancer from past 6 months",
"src": "Patient: HI MY HUSBAND HAS LUNG CANCER AND 6 MONTHS AGO WAS DIAGNOSED WITH PULMANARY FIBROSIS WE ARE HAVEING BONE AND BRAIN SCAN THIS WEEK. I HAVE BEEN USEING FRANKINCENSE AND OTHER ESSENTIAL OILS HE SEEMS TO BE SLEEPING ALOT IS THIS NORMAL. also WOULD LIKE TO KNOW IF THERE IS ANYTHING ELSE I CAN DO Doctor: As he is suffering from lung cancer you need to find the stage of disease.Biopsy from the space occupying lesion in chest is a must along with EGFR and ALK mutation test.Also do CT scan of upper abdomen,Bone Scan and MRI brain.Commonest sites of lung cancer spread are liver,adrenals and bones.RegardsDR DE"
},
{
"id": 214779,
"tgt": "What are the home remedies for the night sweats, irregular periods and panic attacks?",
"src": "Patient: Hi, I have been dealing with night sweats since my third child. It has been about 3 years since I have noticed now it has been nights and morning sweats. I am only 36 lol but in the past I have dealt with irregular periods. Cycts... im moody an i have panic attacks that i have never felt before. Is there anything natural that i Can take because my doc says birth control and my body does not do well on that Doctor: **1. Irregular periods, night sweats are common in the mid 40s as perimenopause approaches and is also associated with panic attacks [anxiety/or felling if anxiousness], and is collectively called as ASD [acute stress disorder]2. prior to taking treatment for the same, check with gynecologist/attending physician regarding hormonal profile for irregular periods/mood swings: mainly estrogen/progesterone balance and thyroid profile, ultrasonogoraphy for PCOD/PCOS [since cysts were there], likewise night sweats can also be due to some underlying medical condition.3. avoid caffeine & alcohol, excess physical \u2013mental exertion, sun exposure, corrylium, smoking, errhine therapy, betel leaves, gruel, soup, day time sleeping, garlic, ginger, onion, spicy-heavy- junk food, mustard oil, pickles.4. Take: mint, coriander, tomato, cumin, plenty of water, vitamins and mineral supplements, apple, banana,grapes,dates5. You can practice: bhujangasana,halasana, paschimottasana, sarvangasana, shalabhasana, shavasana and vakrasana and pranayamas like bhastrika and kapalbhati.6. Lemon juice mixed with sugar in a glass of water helps release stress removing hormones from the body and bring mind at peace/rest.PS. drink plenty of water/fluids to remain well hydrated.. Apple, almond, banana, broccoli, cantaloupe, melon, carrot, cashew nut, corn, cauliflower, grapes, guava, lemon, mango, orange, pineapple, soybean, spinach have anti oxidant and detoxifying properties so take them in plenty."
},
{
"id": 25219,
"tgt": "Will the bp medicines cause higher level of triglycerides?",
"src": "Patient: I am 57 yrs old and have been taking nebicard sm daily to control my BP. My cholesterol level is 233 and Triglylcerides is 211. I am not taking any other medicines. Kindly let me know if the triglycerides levels go up because of the BP medicine and if so what other medicine can I take as a substitute? Doctor: Hi & Welcome.Nebicard SM contains a beta blocker (Nebivolol) & a calcium channel blocker Amlodipine commonly prescribed to treat high BP.Increased cholesterol & triglycerides appear to have been caused by Nebivolol. Beta blockers like Nebivolol can affect your cholesterol & triglyceride levels causing a slight increase in their levels. These changes often are temporary. You shouldn't abruptly stop taking a beta blocker because doing so could increase your risk of a heart attack or other heart problems.Consult your treating doctor for an alternate but a safer medication like Losartan to lower your high BP. Hope this will help."
},
{
"id": 62911,
"tgt": "Suggest treatment for small red lumps on the breast",
"src": "Patient: Hi my both my breast have had some light pain on and off in the chest area. I now have more pain in the right breast when lifting my arm. I keep checking foe lumps, redness, and changes but thankfully nothing but I have this pain why and should I be going to the doctor? Doctor: HI,Welcome to HCM with your query.Based on the facts of your query,you seem to have Fibroadenosis due to hormonal imbalance and seems to be of familial /heriditary nature.Its a benign condition.Your query does not indicate any lump,as quoted in your heading of the query.This appears to be moderation mistake and would neglect it.So don't worry of your breast painful areas on right side,as Cancerous,as its a hormonal effect before menses.Treatment-Vit E x 3 months/Anti-inflammatory Drugs Sos under guidance from your doctor would resolve this pain.Hope this would help you to plan treatment with your doctor.Will appreciate hitting thanks and writing review comments to benefit needy patient visitors at HCM like you.Good day.Dr.Savaskar M.N.Senior Surgical Specialist.M.S.Genl-CVTS."
},
{
"id": 160499,
"tgt": "What causes lump between nipples in infant?",
"src": "Patient: hi, i have a 10 week old and the has a lump about the size of a quarter in between his nipple and the base of his neck. went to the doctor yesterday and she said its a little high for it to be a breast bud. it is very warm to my touch but it doesnt seem to hurt him. tomorrow is an mri/ct scan but is there any thing this sounds like to you. im terrified. thanks -leslie Doctor: Hi,From your description, this seems to be an accessory nipple. This is a common minor abnormality seen in some babies. Here an additional nipple or breast tissue (which is small and less developed than the actual ones) develops from embryonic remnants. If confirmed, this need no specific treatment. Your doctor will discuss you in detailHope I have answered your question. Let me know if I can assist you further. Regards, Dr. Muhammed Aslam T. K., Pediatrician"
},
{
"id": 28564,
"tgt": "How can one manage blood drainage from a wound on the elbow?",
"src": "Patient: I have a wound from pulling a skin tag from dry skin on my elbow. It has recently been leaking a LOT of serous fluid. This morning when I changed the bandage, a lot of bright red blood started running out of it. It bled for at least 30 minutes. Noticed that most of the elbow is very red. My dermatologist appointment is not until this coming Wednesday. What can I do for it now? Doctor: Hello and Welcome to \u2018Ask A Doctor\u2019 service. I have reviewed your query and here is my advice. It looks like the wound site is infected and there is infection and inflammation leading to drainage of tissue fluids and blood due to inflammation. So my suggestion is to consult an Emergency room and get evaluated. An antiseptic dressing can be done reduce infection and initiate healing of wound Also a course of antibiotics like Ciprofloxacin can be taken orally. Hope I have answered your query. Let me know if I can assist you further. Regards, Dr. Honey Arora"
},
{
"id": 89686,
"tgt": "What causes stomach pain while eating and drinking?",
"src": "Patient: helloHello. I would like to ask... is that everything time I eat or don't eat, my stomach hurts. it's like someone is kicking me from inside and my stomach feels hard like a rock. I am not pregnant. So what could it be? Your opinion? And Ive been dizzy. for the past 2, weeks now... Standing,laying down... it all makes me dizzy... Would that have to do with anything of my brain?? Doctor: Hello and welcome to HCM,Your history suggests that you are suffering from two ailments.First is dizziness and second is feeling hard in the stomach region.Dizziness alone can be due to some ear problem.If dizziness is associated with tiredness, weakness and easy fatiguibility, you probably need to get your complete hemogram.Anemia presents with the above mentioned symptoms.For the stomach problem i.e. feeling hard, you need to consult a gastroenterologist.An upper GI endoscopy needs to be done to look for any lesion in the upper GIT.Management can be planned once diagnosis is made.Thanks and take careDr Shailja P Wahal"
},
{
"id": 26399,
"tgt": "Is it correct to have blood pressure checked after eating?",
"src": "Patient: I took my blood pressure shortly after eating. It read 148 over 73 with a pulse of 63. I am a 51 year old female. Is it not good to have your blood pressure read after eating and is this a high reading? I haven't had my reading in a year or so because usually it reads 118 over 78. Doctor: Hello!Thank you for asking on HCM!Regarding your concern, I would explain that these values of blood pressure are above the normal ranges and may be indicative for possible arterial hypertension. It is not common to have high blood pressure after eating. I would recommend consulting with your GP for a careful physical examination, a resting ECG, a chest x ray study and some blood lab tests : - complete blood count- fasting glucose- kidney and liver function tests- blood electrolytes- thyroid hormone levels- cortisol plasma levels, etc. to exclude other possible secondary causes related to high blood pressure. You should closely monitor your blood pressure different times during the day (in sitting position, 15 minutes after total relax) and refer these values to your doctor. Some lifestyle modifications are helpful to reduce blood pressure: - a balanced diet (low salt and caffeine intake)- a lot of physical activity (walking 4 km per day is recommended)- lose weight if your are overweight. Hope to have been helpful!Best regards, Dr. Iliri"
},
{
"id": 161097,
"tgt": "Suggest treatment for hyperactive disorder in a child",
"src": "Patient: Our son age 7 years, studying in class 2 is our only child. He is extremely playful, try to play even in the class. His teacher gets often annoyed at his this act. At home too, he generally wants to play, avoid studying, passing time in talking about immaterial things. He dances well, and curious as well but doesnot hesitate to break discipline and disobey his elders. Teachers also suggested us to go for child psychlogists. Kindly go through the case and suggest a doctor in the vicinity. Thanks, Sahay. Doctor: Hello, It may be a case of attention deficit hyperactive disorder. This is fully curable with behavior therapy. You can go to any child psychologist for this. Hope I have answered your query. Let me know if I can assist you further. Regards, Dr. Loveleen Sharma, General & Family Physician"
},
{
"id": 182184,
"tgt": "Suggest treatment for swollen gums",
"src": "Patient: Hi, my gums are swollen for the past week i took amoxicillin 500mg and naproxen every 8 hrs already for 5 days and it improves a bit but still the swelling has not gone. I only have the swelling in one of my tooth which is covered by tartar/plaque. Now, when i press on my gums with my tounge blood is coming out. Doctor: Hello,Thanks for consulting HCMRead your query as you have swollen gum with bleeding dont worry this can be due to deposition of plaque and calculus at subgingival region of gums . I will suggest you to consult oral surgeon and go for scaling and root planning . Inmeantime do warm saline rinses 2 - 3 times a day.Hope this will help you."
},
{
"id": 74411,
"tgt": "What causes left plank pain and difficulty in breathing?",
"src": "Patient: Right sided flank pain, hurts to breathe, sit,stand, well to do anything. No frequency, or burning during urination, currently menstruating so I can t tell if there is blood in urine. Kind of feels like a muscle spasm but not sure with the location of pain? Doctor: Thanks for your question on Healthcare Magic. I can understand your concern. Right sided lower chest pain on breathing is commonly seen with lung diseases like pleural effusion and pleurisy. So first get done chest x ray. If chest x ray is normal then no need to worry for lung diseases. Musculoskeletal pain can cause similar kind of pain. So apply warm water pad on affected areas. Avoid movements causing pain. Avoid heavyweight lifting and strenuous exercise. Take painkiller and muscle relaxant drugs. Don't worry, you will be alright with all these but first rule out lung diseases. Hope I have solved your query. I will be happy to help you further. Wish you good health. Thanks."
},
{
"id": 69381,
"tgt": "What causes a lump behind the ear?",
"src": "Patient: My fiance found a lump behind her right here. she says she has also experienced drainage recently. She frequently gets ear infections but hasnt had one in the last 3 months. I worry more than she does about these things and would greatly appreciate an answer. thank you. Doctor: Hello and welcome to HCM,A history of ear infection and associated lump behind the ear suggests a reactive lymph node enlargement after the ear infection.However, the reactive process has to be confirmed by an aspiration cytology.In aspiration cytology, the cells are aspirated from the swelling by a needle.The cells are spread on glass slide and stained with appropriate stains.The origin and nature of the swelling can be determined by studying these cells.In case there is reactive lymphadenitis, no treatment is required but the ear has to examined for any evidence of infection.The lymph node will regress as the ear infection is released.Thanks and take careDr Shailja P Wahal"
},
{
"id": 46042,
"tgt": "Why is EKG advised in case of kidney stones?",
"src": "Patient: my husband had a kidney stone that would not pass. His doctor went in and put in a stent. They said they could not see any stones with sonagram or xray. Now they want to go in and break up the stones and want to test him with an EKG why does he need and EKG. And since the pain has been gone what are they breaking up? Doctor: Most surgeons and anesthesiologists want to get baseline heart function tests before giving anesthetics for procedures. It is relatively routine."
},
{
"id": 201996,
"tgt": "What is the treatment for enlarged prostate?",
"src": "Patient: My age is 27yrs. My full abdominal USG report says that \"Prostate is mildly enlarged in size. Echotexture appears within normal limits. No focal alteration of its echogenecity could be detectable. Its measure: 39.7mm x 35.0mm x 44.6mm. Approximate weight could be around = 32.46gms. Is any special care or medical treatment needed? Doctor: Hi Are you having any symptoms like urine hesitancy or urine dribbling or pain during urination. If you donot have any complaints and its just a ultrasound report . I would advice if you were my patient you don't need any treatment. Patients with mild symptoms or moderate-to-severe symptoms of benign prostatic hyperplasia (BPH) who are not bothered by their symptoms and are not experiencing complications of BPH should be managed with a strategy of watchful waiting. In these situations, medical therapy is not likely to improve their symptoms and/or quality of life . In addition, the risks of treatment may outweigh any benefits. Patients managed expectantly with watchful waiting are usually re-examined annually.I hope I have clarified your doubt.Wish you a good health."
},
{
"id": 136393,
"tgt": "What causes a large lump on the wrist?",
"src": "Patient: Hi, my husband came home from work with a very large bump on the back of his left wrist on the side the radius is on. He says there is a little pain with it. It looks swollen but not sure why. The swelling is about 3 inches long and 2 inches wide. What do you think it could be? Doctor: Hi,I have studied your caseThere can be bony contusion.As x ray may be essential to rule out fracture.If swelling is there, you need to take rest and use splint or compressive crepe bandage to reduce swelling.Watch for increase in redness, infection.Take precaution to avoid infection around hematoma.Do not use hot fomentation or massage.If required you may need to do blood investigation for prognosis and recovery.X ray may be required after clinical examination.Hope this answers your query. If you have additional questions or follow up queries then please do not hesitate in writing to us. I will be happy to answer your queries. Wishing you good health.Take care."
},
{
"id": 149707,
"tgt": "Shaky, burning feeling in head, lightheaded, panic disorder. On Paxil. Normal BP, pulse 70, blood sugar 86. Suggest",
"src": "Patient: i have been feeling shaky but my bleed presure is 124/70 my pulse is 70 and my blood sugar is 86. sometimes i feel light headed and sometimes my head feels like it is on fire. i have panic disorder but does not feel like that. i have not been having problems with that have been on paxil for 15 years. someone said maybe thyroids. i hope you can help it what i should be looking for the doctor to do Doctor: Hello,You are 100% alright as per investigations. The problem you have may be due to tension or depression. If you are happy with Paxil - continue it.If you feel that you have hypothyroidism - investigate it. Your symptoms may not be due to hypothyroidism.If you are not happy with Paxil - consult your Dr and ask for a change of drug.Best wishes"
},
{
"id": 82697,
"tgt": "What causes chest pain, fever, stomach pain with appetite and weight loss?",
"src": "Patient: Hi I have systemic lupus and have recently returned from a trip to Haiti . I was hospitalised for two days with general malaise and left leg pain plus severe right lower abdo pain . I have been cleared of DVT and peritonitis and have been released on 25 mg prednisone but in the last two day I have developed chest pain and low grade fevers as well as generalised stomache pain . I have lost 5 kilos in the last week and have no appetite Doctor: Hi,Thank you for your query. I can understand your concerns.Fever is common in SLE (Systemic Lupus Erythematosus), either from secondary infection or the disease per s e.Fever and weight loss accompany active disease . Fever & chest pain combination point towards the possibility of Pleuritis+/-pneumonitis. Generalised stomache pain, in addition may occur fromvasculitis of intestine,not uncommomn. Since you are already on steroid,you have to take antacid like Pantoprazole 20-40 mg orally daily.Regards Dr. T.K. Biswas M.D.Mumbai"
},
{
"id": 108307,
"tgt": "Suggest treatment for lower back pain",
"src": "Patient: I have been experimenting lower pain simular to craps related to P.M.S. the pain has been in my lower back as well. I m having trouble going to the bathroom. I wake up with exstem headaches and a great deal of confusion through out the day. I find myself very moody as well. I know that I am not pregnant since I have had a historcetomy aproxmently 6 months ago. A few of the women who live in the same apartment complex as me is having the same issues. The hospital says it a U.T.I pump them up with pain medication and send them home. Im thinking it lead positive due to the children and mother who live are me are having the same issues however I may be far off. Im hopefully you can ease my mind and give me a better understanding of what is going on with my body. Doctor: Here u have told you have there is something called as pms. As per me the full form of pms is pre menstrual syndrome. Another thing you have told is you have already gone for hysterectomy in that case pms should not be possible with you. Now the next thing you are assuming is urinary tract infection. Lett me tell you urinary tract infection will have some more symptoms like burning micturation as well lower abdominal pain and fever. Kindly check do you have any symptoms like this if not then again it cannot be uti. There are different causes due to which person may have back pain some of them are back muscle weakness, spinal cord problem, disc problem, kidney stone etc, to rule out any of this you need to undergo for mri of lumbar spine as well ultrasound of abdomen based on the report you can visit specialist for detailed treatment discussion. Here there will be two ways of treating you the first way is medical managementhere if there is no major issue in your medical report then doctor will suggest you for certain medications along with physiotherapy treatment and back strengthening exercises this is called as conservative management for back pain. Now if there is good enough damage at your spinal level visible in mri report then here the orthopaedic doctor will suggest you for surgical correction. I suggest you can also use hot pack at home for some more relaxing effect on your soft tissues which will release your pain up to some extent.For UTI do not worry and visit your physician and take a complete course of medication as per his suggestions and you will be fine soon. take care."
},
{
"id": 157537,
"tgt": "Have COPD, lymph nodes in lungs, itching in back, red patches, had prostate cancer. Done CAT scan. What does this indicate?",
"src": "Patient: (July 2013 to present date, Oct 10, 2013): Started first with breathing problems. I have copd. Had a catscan and a petscan. Have enlarged lympth nodes in both lungs (many). Waiting for another cat scan in November to make a comparison to see if any are getting larger. Pulmonologist said he can t say for sure if I have cancer and can t say for sure that I do not. Have run a temperature as high as 104.4% and low grade to normal(back and forth) since then. and I have some swelling in my left testicle. Had swelling and a hard lump behind my knee about a week ago and knee was very warm and nurse thought it might be a blood clot but turned out to be a Bakers cyst. Also have red blotches all over my body. Started by itching on my back that turned into a oblong type circle that then kept getting larger. That is lighter in color now but legs are brighter red patches. Also still have redness on parts of my back and both arms. Have had three heart attacks and have 3 stints. Also had prostate cancer and did radiation treatments in 2005. Have had two antibiotic prescriptions since July 10,2013 from Pulmonary doctor. Allergy doctor said I have an unidentified infection in my blood. He gave me a prescription for Ketoconazole for the red blotches that were itching very bad. This made it worse and oval type circle on my back got bigger and more bright red in color. That is finally almost gone but legs are worse. Had blood work done on Monday, Oct 1,2013 on order from the allergist doctor. No report on results yet. Have primary care doctor appt. coming up on the 23rd of October. Also have type II diabetes. Legs are very weak. Doctor: hello,I propose you do a blood test, with all elements (RBC,WBC, HG, HCT etc). In general in infections WBC are alterated and if all is normal you should consider making a biopsy of one of your lymphonodes to see its contest."
},
{
"id": 25853,
"tgt": "How severe is a 4th heart attack with blood thickness 69.3 in patients with Polycythemia?",
"src": "Patient: I was diagnosed with Polycythemia in 2005 but no one ever told me about it until I had a 4th heart attack in 2010 and they said I should have been under care - my blood thickness was 69.3 - with blood thinning it is now at 43.8 How bad has this mess hurt? Doctor: hello,I have gone through your query.Thanks for using HCM.I need to know exact level of your Hemoglobin to guide you properly.You should consult a hematologist and under his supervision you must remove blood from your body.This is called phlebotomy.You must maintain your Hb below 18 mg%my best wishesDr.Rajesh Teli,MD."
},
{
"id": 79274,
"tgt": "Why am I having severe pain in my esophagus while coughing?",
"src": "Patient: having severe pain in the esophagus area between my breasts when I cough...slowly calms down but when I cough again the pain in almost unbearable...don't know if this is a start of a cold...my husband has one. I do have gerd and take dexilant daily but have never felt this before. I am 58. Doctor: Thanks for your question on Health Care Magic. I can understand your situation and problem. Possibility of worsening of GERD (gastroesophageal reflux disease) is more in your case. GERD is due to laxity of gastroesophageal sphincter. Because of this the acid of the stomach tends to come up in the esophagus and cause the symptoms of Central burning chest pain. Along with drugs, GERD needs following lifestyle modifications for better symptomatic relief. 1. Avoid stress and tension. Avoid hot and spicy food. 2. Avoid large meals, instead take frequent small meals. 3. Go for walk after meals. Loose weight if you are obese. 4. Keep 2-3 pillows under head in the bed to prevent reflux. Avoid smoking and alcohol if you have these habits. You will mostly improve with all these. If not improving than get done chest x ray to rule out lung infection. Hope I have solved your query. Wish you good health. Thanks."
},
{
"id": 56763,
"tgt": "What causes pain near liver?",
"src": "Patient: I Recently, had Tonsillitus, due to a severe infection from strep throat in correlation with mononucleosis that i have, i have had my tonsils drained of puss, and was given a heavy flush of anti-biotics to counter the infection, the swelling has gone down but recently, i have noticed hard like bumps built up on the inside of my lips. There is no visual to show that these bumps are present. Doctor: Hello,It may be due to some secondary infection or due to high doses of antibiotics.please consult your doctor for that.Hope this will help you.take care..."
},
{
"id": 196214,
"tgt": "What causes soreness in testicles?",
"src": "Patient: Hi. Im a 35 year old male. I have had sore testicles for the last 3 days. they are sore all over and its also sore on either side of the penis. I have some amphtamines over the last few days as well. have lifted anything heavy. One minute they were ok and the next sore and gradually painful till i had to sit down. Doctor: i think you are having fungal infection on the genital area.kindly apply clotrimazole ointment twice a day for 1 week."
},
{
"id": 204458,
"tgt": "Does drug addiction cause schizophrenia?",
"src": "Patient: If a person who seemed to be normal in their youth but then was a drug addict who used crack for 25 years be a victim of paranoid schizophrenia? I have a room mate who thinks he is autistic, but there is no medical history on him due to him being adopted and nothing known about his biological parents. He has paranoia like I have never seen in anybody, and although I do not see him as becoming violent towards me, I am still concerned. I mean 25 years of constantly using crack seems like it can trigger schizophrenia. Can you help? Doctor: Hello and Welcome to \u2018Ask A Doctor\u2019 service. I have reviewed your query and here is my advice. People who abuse drugs sometimes develop symptoms that closely resemble the symptoms of schizophrenia, and in some cases, drug abuse may actually set the stage for the onset of the disorder. In people who already have schizophrenia, drug use/abuse can worsen the illness\u2019s effects in several different ways. Talk to psychiatrist for a clear picture. Hope I have answered your query. Let me know if I can assist you further. Regards."
},
{
"id": 23083,
"tgt": "What causes palpitations and pain in the right quadrant?",
"src": "Patient: Hi I m a 35 yr old female, 5 7 and weigh 128. I have been a vegetarian my whole life, cholesterol 140, BP usually 110/70. I do have problems with heart palpitations, but my question is about right quadrant pain that I ve been having for about 3 yrs. It radiates to back, it s a gnawing, chronic pain. I feel nauseous, and just generally ill. I ve had barium swallow, endoscopy, hida scan, 2 ultrasounds- all normal. A few weeks ago my primary ordered a CT scan. He called and said my gallbladder is good, but my right kidney shows possible stone, so he referred me to the urologist, who said my right kidney looks swollen and possible stone in ureter. Sent me straight back over to hospital for a urn CT scan (lots of radiation), but he said he felt it was necessary. Nurse called me with results- right kidney is fine, cyst on left kidney, cyst on right ovary, what they thought was a stone is calcifications in ureter. Then she casually mentions there was some mild fatty deposits on liver. I freaked. She said oh everyone says that, but it s no big deal. Followed ip with primary, and I was grilling him about this fatty liver thing. He said, it s mild and wouldn t be the cause of my pain. So I m back at square one, only now I m freaked thinking I have a fat on my liver. First ultrasound I had done the tech said, beautiful liver ....second one I had done just a month before CT scan, and everything looked great. Are Ct scans better at detecting those types of things? Both scans were with and without contrast. I m a wine drinker, but just couple times a week, or sometimes it s twice a month. I do drink more than the moderate 2 glasses, but I don t know anyone who does that. Bilirubin- 0.4, ALT- 15, AST- 16 Blood work normal. Sorry so long, but how serious is mild fatty infiltration of the liver?? How long will it take to repair if I abstain from alcohol? And can the mild fatty liver cause this pain and sick feeling? I still have no answers, but I would like feedback on the liver results and if cysts on kidneys are normal. My doctor just prescribed Zoloft and sent me on my way.....which isn t helping anything,so frustrated. Doctor: fatty liver is common problem and treatment include low fat, sugar diet and regular exercise. you should avoid alcohol. It may not resolve completely but it's not a matter of worry. As fat deposits in the belly, similarly fat deposits in liver.It should not lead to pain and this illness.Now about your pain, is there any relationship with meal, any relieving or aggravating factors, any associated symptoms."
},
{
"id": 152401,
"tgt": "How frequently should a colonoscopy be conducted for precancerous adenoma?",
"src": "Patient: Hi. I am a 56 year old female. I was diagnosed with Stage 1, Grade 3 Ductal Carcinoma in 2015. I had a double mastectomy and chemo. Earlier this year I had a colonoscopy and a precancerous adenoma was found. I was told that I wouldn t need another colonoscopy was needed for 5 years. Since my oncologist told me they couldn t be certain that they got all the cancer, this naturally concerns me. Not that I want to do another colonoscopy but I do want to make sure I catch anything early, as I did with the breast cancer. So my question is why is it medically reasonable to wait so long? Thanks for your time. Doctor: Hello, If I were in your place, would have also been worried. Usually, breast cancer and colon cancer are two different cancers without any association between the two. In general, a person has diagnosed with cancer, the risk of him or her developing second cancer is slightly higher compared to normal individuals. Usually, colonoscopy is recommended every ten years, in your case, it has been recommended, I presume, for this very reason, to detect early when it is still curable. Secondly, usually any cancer takes a certain time to grow, for a precancerous adenoma to become cancer, it will take time. However, it depends upon the type of adenoma, since there are different types ranging from benign with minimal risk of developing into cancer to dangerous ones which have the highest probability in turning into cancer. In order to comment upon your risk, would like to know the details of the precancerous adenoma, whether it is single or multiple, shape (sessile or pedunculated), etc. Whether to wait or not, depends upon the type, I am guessing the adenoma found is one of the mild types and hence the advice of repeat test after 5 years. Hope I have answered your query. Let me know if I can assist you further. Take care Regards, Dr Jatin Bhatia, Radiation Oncologist"
},
{
"id": 101326,
"tgt": "What is the cause for severe cough and breathlessness?",
"src": "Patient: I was diagnosed with COPD 5 years back. Since February 2012 I have had severe cough and breathlessness. I had spirometry done 10 days back and was diagnosed having uncontrolled asthma. Now I have nasal blockage and am constantly breathing through my nose, which is causing severe discomfort. YYYY@YYYY Doctor: Dear you need to follow a strict life style along with proper medication, No smoking absolutely, take good nutritional diet to boost your immune system, do regular exercise in any form either yoga or walking. Your exercise schedule must include breathing exercises . Consult your physician and follow the treatment accordingly as you have an acute episode at present. Infection must be ruled out and any bullous lung disease must also be ruled out to exclude need for any surgical intervention. So get all the investigations done, its treatable you can be completely cured of your illness..............tc care"
},
{
"id": 82709,
"tgt": "What are the symptoms of lupus?",
"src": "Patient: Hi, I m very afraid I have lupus, I have known something to be wrong for 2 years now but can not find anything through doctors. I have acepted that i am lazy and feel pain too easily but now i have only just discovered lupus and my symptoms are so similar I am really worried. Can you help? Thank you! Doctor: If two or more body systems are involved, then we have to think about lupus a simple ANA test will help"
},
{
"id": 135016,
"tgt": "Suggest treatment for sharp pain in the neck , groin and leg",
"src": "Patient: I have a sharp pain in my right side of my neck and in my legs that comes on and can be relieved with advil. With the pain I also feel feverish but I have no temperature. I get chills when the pain comes and the I get flush when the pain begins to subside until the point where it goes away because the advil is working. I have been taking advil for these symptoms for about a week now and cant seem to shake it. Occasionally I feel a sharp pain in my groin area and in my legs as well during the time when it flares up. Doctor: Hi Dear,Welcome to HCM.Understanding your concern. As per your query you have sharp pain in the neck , groin and leg. Well the symptoms you mention in query can occur due to may reasons like nerve compression , disc problem , compressed fracture or cervical spondylosis. I would suggest you to take complete bed rest , otherwise condition may get worse, apply warm compresses to leg back and neck , do not lift heavy object and take ibuprofen or acetaminopehn for pain . Consult orthopedic surgeon for proper examination . Doctor may order CT scan , MRI along with nerve conduction test . Doctor may recommend physical therapy along with particular stretching exercises , prescribe anti inflammatory , muscle relaxant , vitamin supplement for quick recovery . Hope your concern has been resolved.Get Well Soon.Best Wishes,Dr. Harry Maheshwari"
},
{
"id": 107794,
"tgt": "Suggest treatment for cough , lower back pain and headache",
"src": "Patient: I m unsure if I should see a dr. I have been feeling bad for about 1 week. At first I thought just a cold, as days passed not getting any better. Symptoms are congestion, right eye pressure, mild headache when I cough, lower back pain. Clear nasal fluid when I blow my nose, however coughing up is yellow, no sore throat, on and off nausea. Doctor: Dear- thanks for using our service and I understand your concern. It looks that you are having an infection, due to the symptoms, color of the phlegm and congestion. You have been suffering from this problem for one week, which goes more with a bacterial than viral infection. It is advisable to go to your primary care doctor for a physical examination and antibiotic treatment. The reason that you need antibiotic is because of the duration of your symptoms and the color of your phlegm.You might need a CXR too.I hope that my advise has been helpful. Dr.Sara"
},
{
"id": 201474,
"tgt": "What could be the reason for ejaculating red semen during masturbation?",
"src": "Patient: Hello docs n experts, Since yesterday I m ejaculating red semen. I noticed it yesterday while masturbating, even today I seen same thing when I was trying to confirm was it a normal one day case or a serious issue. Now I think it is a serious problem for me. Please help me. Doctor: Hi, thank you for using health care magic.I can understand your concern.Red colored semen could be of sign of infection / injury to prostate.There are other possible causes like injury to urethra, infection to epididymal gland etc.Do you feel pain or burning while urination , if yes then urinary tract infection can be also the cause.Get help of doctor as this issue can not be taken lightly.You may need to under go certain test like urine test, ultra sonogram of pelvis probably. Soon the cause will be find out better chances of recovery.Hope this will help you.Regards."
},
{
"id": 197514,
"tgt": "What do small bumps on glans penis indicate?",
"src": "Patient: Hello. I have noticed for awhile now small bumps on theglans. Barely noticeble unless erect, but when erect skinlooks cracked and large amount of bumps present. I am in good health and have been with the same woman for 11 years. So i am doubting any std. I do have fordyce spots on shaft, scrotum and lips. Any help would be great! Doctor: HIThese sound like what are called pearly penile papules--small, benign cysts and nothing worrisome.They can be removed with a cold CO2 laser by a dermatologist"
},
{
"id": 185945,
"tgt": "What causes small white like patches in my gums?",
"src": "Patient: Hello there! A couple of weeks ago I noticed some small white like patches on my gums on my upper right side of my mouth above my teeth. They don't hurt at all,but they have been there for some time now and im becoming worried as to wheather its something I should see my physician about. About a month ago I went to the doctor for a sore throat and ear infection and was put on 2 different antibiotics. I was on antibiotics for a total of 15 days. Not sure if this was there prior to the antibiotics, but it dosen't appear to be gone yet. What do you think it is and should I worry? Doctor: You did not mention about your general health and any tobacco usage.This can be due to tobacco usage and stress factors.Go for a tobacco free,non alcoholic,stress free lifestyle.Take vitamin supplements.Maintain good oral hygiene.You need not worry till there is a pain/burning sensation or if you see it increasing in size.Regards."
},
{
"id": 161530,
"tgt": "What does pus cells in stool signify?",
"src": "Patient: my baby is 10 months old. she has diahrea from last one week , she is teething as well. i have got her stool tested and she has 10-20 pus cells. her culture etst reprot is stil awaited .. what does this range of pus cells signify should i woprry. she is otherwise active Doctor: Hello, Nothing to worry - this is a normal report. As long as the kid is active - not to worry at all. Unless the kid's having low urine output or very dull or excessively sleepy or blood in motion or green bilious vomiting. You need not worry. Hope I have answered your query. Let me know if I can assist you further. Take care Regards, Dr Sumanth Amperayani, Pediatrician, Pulmonology"
},
{
"id": 33017,
"tgt": "What causes blotchy spots on tongue?",
"src": "Patient: I am a 42 year old woman, for the past 6 months, i have had periodic episodes of a sore blotchy tongue. i eat a pretty good diet, work full time and have 3 children, so a busy life. I have always put it down to being run down, but am otherwise pretty healthy. Should I be concerned? Doctor: Hi dear ,There could be two possible causes for the symptoms ou mention in query that is blotchy spots on tongue .- First it could be geographic tongue which is a harmless condition affecting the surface of your tongue . It is supposed to related to psoriasis a chronic skin condition .- Secondly it could be due to acid reflux disease . If there is symptoms like heart burn then it will be gastrointestinal reflux disease , if there is no such symptoms then it will be a silent reflux disease .I would suggest to consult gastroenterologist for proper diagnosis . Doctor may prescribe medicine like ranitidine or omeprazole for acid reflux disease and corticosteroid or antihistamine therapy for geographic tongue .Hope your concern has been resolved.Get Well Soon.Best Wishes,Dr. Harry Maheshwari"
},
{
"id": 51128,
"tgt": "Have RBBB, diabetes, hypertension. Had kidney failure, anemic. Taking Procrit. Blockage due to previous conditions?",
"src": "Patient: Hello. Mom suffers from a RBBB , right bundle branch blockage....she also suffers from diabetes and hypertension . A few years ago (6) her kidneys failed and well fluids entered her lungs and consequently, she is anemic to the point she must be injected with Procrit every other week. Could her blockage be a result of well frankly all of the above? Susan Doctor: Hi, Welcome to HCM Your mother is having Hypertension and Diabetes long standing( Duration of disease) can cause renal failure and when kidney fail there is Anemia because of hormone deficiency called Erthropoitin. That is why she requires Procrit is to be given to treat Anemia, This is good drug to treat anemia due to kidney failure other wise you need to give blood . We should keep her blood pressure controlled by medications and also keep her blood sugar also with in desirable range. Kidney failure may not be reversible but can always slow it's progression and prepare for Hemodialysis in case kidney failure worsens. Take care of diet fluid restriction,low protein,low Potassium diet as advised by your doctor. Take care of her. Good Luck"
},
{
"id": 9099,
"tgt": "How to remove unwanted hair without any side effect?",
"src": "Patient: hi doctor! i m 22 years old and i want to remove unwanted hair . can you please tell me how to remove unwanted hair without any side effect? Doctor: Hello, which part of the body do you have these unwanted hair. are they coarse or fine. Coarse hair; face- laser, armpits- laser, hand'legs- waxing or creams Fine hair; face- eflornithine cream also get yourself checked for PCOS"
},
{
"id": 24134,
"tgt": "Will artery blockage recur post carotid endarterectomy?",
"src": "Patient: I had left carotid artery surgery three months ago, yesterday when I was at a doctors visit she listened to both sides of my neck and told me that she heard swishing a bruit in my neck. My question is would it be possible for me to have another blockage after just having surgery 29 June 2015. Doctor: Hi Welcome to HCM.I can understand your concerns.Yes there is chance of restenosis.Post your further queries if any Thank you."
},
{
"id": 105778,
"tgt": "Violent sneezing and pain during swallowing",
"src": "Patient: I had a few violent sneezes about 3 days ago as I suffer from allergies and I always sneeze every morning when I get up and they are continuous like one right after another and so on for almost 5 sometimes 10 minutes. Anyways, after it happened 3 days ago, I got this really sharp pain in the left side of my neck near my Adam s Apple and have also lost a bit of my voice. Also, when I swallow it hurts and when I sneeze I get a really sharp pain in that same spot. I am also only a 33 yr. old male and an occasional smoker (2 to 3 cigarettes/day). Should I go and see my family physician about this problem or is there a way that I can treat it myself. Doctor: Hi there. Thanks for the query. Since you have severe sneezing, you may have damaged your throat lining a bit due to the violent sneezing. Since this occurs in the early morning, it is quite typical is Allergic rhinitis. Allergy is mainly due to cold temperature, pollen, certain food, drugs etc. The only way to avoid such attacks is to prevent the allergens as mentioned. Hence if you get in early morning it means you may also be allergic to cool temperature of early morning. Keep warm throughout the day and nigh with proper clothing and blankets respectively. Kinldy stop smoking otherwise you have higher chance for getting infection and delayed recovery. Kindly avoid hot fluids and hard food. Keep well hydrated. Eat a balanced diet. You can take anti-histamine like Cetrizine at night after Dinner which is anti-allergic medication and causes drowsiness also. Do salt water gargling to relieve yourself of the pain and throat swelling for 3-4 times a day. Wishing speedy recovery. Regards,"
},
{
"id": 138033,
"tgt": "Should the medicines be discontinued before a bone surgery?",
"src": "Patient: Hi, My friend is having surgery Wednesday to fix a problem between L3 and L4. The doctor said this was the only solution. My question they did not tell her discontinue any medications that she is on sucn as asprin etc? I am having a knee replacement on the July 16th and I had to get off all my medication 2 weeks prior to that date? What would be the difference?? Thank you. Doctor: Hello,Welcome, and thanks for sharing your concern I went through your query, and I feel, you are intelligent that you noted this fact, if your friend is on asprin or any other blood thinning agents, than stopping them before surgery is a must, spine surgery is even more dangerous than the simple replacement surgery, so ask her to talk to her doctor regarding this concern, and follow the advise given.I hope my advice would have been useful, in decision making regarding your treatment, still if you have any clarifications or doubts feel free to contact back.Thanks."
},
{
"id": 68234,
"tgt": "How to get rid of lump in my shin?",
"src": "Patient: hi i injured both my shins just over 4 weeks ago i have very bad bruising and it was painful when i walked, the bruising has gone now but its still painful and i have a lump on my shin and wghat feels like some dents what could this be and should i be concerned? Doctor: Welcome to health care magic. 1.The possible cause of painful lump in an previously injured site - the possible cause infective collection.2.In this case the it needs to be examined and an ultrasound to assess any vascular involvement and blood supply and nature of lump, its extensions.3.The treatment in this case will be a small incision and drainage followed by a course of antibiotics and symptomatic treatment.4.As a remedy do not squeeze or press, scratch the lump as it can further complicate, clean the area with antiseptic liquid.5.Sugget to get in touch with your GP, get examined and treated accordingly. Good luck.Hope i have answered your query,any thing to ask do not hesitate to ask.http://doctor.healthcaremagic.com/doctors/dr-ganesh/62888"
},
{
"id": 66146,
"tgt": "Suggest treatment for painful sore lump on left inner thigh",
"src": "Patient: Hi there, about a month ago i noticed a little lump on my left inner thigh. It wasnt visible on the surface but i could feel a hard pea shaped lump under the skin, not painful at all though. It had been about a month and a half since then and all of a sudden the lump has gotten much much bigger (about the size of a nickel) and i can now see the lump pushing out on the surface of the skin(though still under the skin) without having to feel it to see its there. The past 3 days it has been really painful ,red and sore and i can barely touch it because it hurts so much. Is there any way to find out what this is and how to treat it? Doctor: Hello and welcome to HCM,A painful sore lump on the inner thigh can be due to an infectious process or due to some other painful lesion like adnexal tumor.Infectious processes include boil and folliculitis.However, the possibility of boil or folliculitis is low.The lesion is probably due to an adnexal tumor.Some of adnexal tumors are quite painful.A clinical assessment and excision biopsy of the lesion will be required.Aspiration cytology will not be feasible since aspiration requires insertion of a needle into the lump and it will be quite painful.Thanks and take careDr Shailja Puri"
},
{
"id": 186972,
"tgt": "What could cause yellowish saliva formation ?",
"src": "Patient: I woke up the other morning,went to brush my teeth and noticed my tounge it appears to be alot more saliva than usual.it feels funny,but I have spit alot when I looked it had a funny color some what white a slight yellow to it.can you help me with this Doctor: Hello, Welcome Thanks for consulting HCM, I have gone through your query, as you have yellowish saliva dont worry this happens when there is problem of poor oral hygiene or poor digestion also. You do proper brushing twice daily. Use tongue cleaner to clean your tongue daily. Do warm saline gargle two - three times a day Use chlorhexidine mouthwash twice daily. Take one capsule vitamin B complex once daily for 7 days Consult dentist and go for Scaling and root planning. If then your problem continues then consult physician for examination and complete haemogram if necessary. Hope this will help you."
},
{
"id": 210858,
"tgt": "Can valium be taken instead of xanax to treat anxiety and panic disorder?",
"src": "Patient: I have Ptsd,panic disorder an anxiety an have been treated for it for 15 years.because OF my limited Insurance benefits an money, i need my dr. To change xanax to valium because im having bad side effects from xanax but need to take a benzo. For condition. Im about to go to mental hospital if i cant get rx changed. Doctor: Hello!Thanks for your query.Valium and xanax are benzodiazepines that help to calm anxiety, as well as treating other disorders.As you have side effects from the use of xanax you should consult with a doctor for dosage of valium because xanax has a much shorter half life than valium. xanax has about a 4-6 hour half life where valium has about a 12 hour half life. This means xanax will stop working sooner than valium, when doses are proportional, and will require you to take more pills/day which contributes to tolerance/addiction. take care of your self"
},
{
"id": 222546,
"tgt": "Could drinking of amniotic fluid by the baby lead to critical condition?",
"src": "Patient: 26 going to fnsh 5.2 64kg last 5 months before oct25th my delivery is done by doctor it is the opearation case my child has druncked more water so he is at the time critical condition in near the operation no pediatrician als o lastly in critical stage they gave my child to husband family than we have admitted at amercian hospital ink bad they have lot of action did every thingfailed my child is no more it is male child 4 kg weight pls help me what i have do to how to get again preganant Doctor: Hello dear,I understand your concern.In my opinion the aspiration of meconium stained liquor leads to lung infection leading to serious condition.It will not recur in future pregnancy.There appears over weight in relation to height.Concentrate on weight reduction and then plan for pregnancy.Because sometimes overweight is associated with complications like diabetes during pregnancy.Regular intercourse increases the pregnancy chances.Avoid stress regarding pregnancy.Ad stress causes hormonal imbalance leading to delayed fertility.Best regards..."
},
{
"id": 34093,
"tgt": "Suggest treatment for fungal infection in the vagina",
"src": "Patient: Hi, may I answer your health queries right now ? Please type your query here... M 17 years old. 1.6m long. M on fungal infection medication.i recently had sex and i was in great pain then we stopped after few hours i started experiencing pain in ma womb nd ma discharge is abnormal very thick n smells bad and my vagina is dry nd itchy.m wondaring what is wrong with me? Doctor: Hello to healthcare magic! Is your fungi infection proven by microbiology test? If so you can take antimycotic drug for example Fungolon ( Fluconazol) 150 mg once! And then for prophylaxis 150 mg every third day (1,4,7) after that 150mg once a week for 6 months while there is still risk of infection! You can take in addition Gyno-pevaryl glomules ( intravaginal) you put the glomule before going to sleep for three following days!"
},
{
"id": 224398,
"tgt": "What pills should i take for preventing pregnancy?",
"src": "Patient: I had unprotected sex last night. I track my period and my calendar says that today is my last fertile day. One other time this happened, I took regular birth control pills in different doses as an emergency contraceptive. I need to know what pills I need and how much to take. It's an emergency. Doctor: Welcome to healthcaremagic,As you had unprotected sex during your fertile period, there are chances that you many get pregnancy. Hence contraception is required. You need to take an emergency contraceptive pill such as i pill or unwanted 72, single dose within 72 hours of unprotected sex. These are very effective if it is taken within 12 hours. The sooner you take the better, It is 95% effective within 24 hours of unprotected sex, 85% between 25 \u2013 48 hours and 58% if taken between 49 \u2013 72 hours. You need to watch for your periods and if you miss your periods, immediately do a urine pregnancy test.Hope that answers your question.Regards, Dr.Imaad."
},
{
"id": 32069,
"tgt": "Why is it required to insert a scope again?",
"src": "Patient: I had a scope inserted into my stomach through my mouth a few weeks ago, I was diagnosed with a hernia and something wrong with my oesophagus. I was prescribed medication and have been fine, no more sickness. However I have just received a letter advising me they want to insert the scope again. What could this mean? Doctor: Thanks for posting your query to health care magic.I didnt understand why your clinician want to do endoscopic examination again . As if they had done examination once no need to repeat the test .So first you confirm for what purpose they want to repeat the test . sometimes it can be repeat to take some biopsy material if not taken during first time . However there are different type of hernia and require surgical managment .review afer complete information for repeating the test then I would suggest what to do .Feel free for any further query.Regards,Dr.Manish PurohitInfectious disease specialist"
},
{
"id": 172378,
"tgt": "How to treat for the pain in chest followed by a fall?",
"src": "Patient: Hi, my 6 years old son got hit on right side of his back and when he fell his hand hit in the center of his chest. said when he got hit the pain went from his side to his heart and felt his heart stop. can anything cause pain from being hit in the side/back area to radiate to the heart? Doctor: Hi,Thanks and welcome to healthcare magic.The pain he is experiencing may be external muscle or bone pain due to injury.No external pain due to fall radiate to the heart.You need not worry about.If the pain does not subside in a couple of days with analgesics better consult your doctor.Hope this reply is OK for you.PLease feel free to ask further queries if any.Dr.M.V.Subrahmanyam."
},
{
"id": 206814,
"tgt": "What could cause weird behavior during breast cancer?",
"src": "Patient: My 50yr old daughter has breast cancer brain mestasteses. she is now telling other family members of hatred of me and how bad a mother I was. Family is horrified by the things she is saying to them. Is this as a result of the pressure and is it quite a normal side effect? Doctor: DearWe understand your concernsI went through your details. I suggest you not to worry much. The description gave by you may not be enough for a proper diagnosis. But it is always possible when people become stressed and engrossed in negative thoughts and are hapless, they find peace of mind in blaming others, especially the near ones. They blame others for bringing them such a hapless episode. This is one sort of minds defense mechanism. Slowly her attitude towards her illness will change. Might take some time.If you require more of my help in this aspect, Please post a direct question to me in this URL. http://goo.gl/aYW2pR. Make sure that you include every minute details possible. I shall prescribe the needed psychotherapy techniques.Hope this answers your query. Available for further clarifications.Good luck."
},
{
"id": 39159,
"tgt": "Is Prednisone safe for treatment of toxoplasmosis?",
"src": "Patient: I am suffering from toxoplasmosis that is affecting my eyes, Doctor prescribed me Prednisone 50 mg for 5 days, then 40 mg for 5 days, 30 mg for 5 days and 20 mg for days and finally 10 mg for 5 days.Is it safe or is there any other way to get rid of this disease. Doctor: Hello,Welcome to HCM,Ocular toxoplasmosis is the commonest cause of posterior uveitis. It usually affects children and young people in age group of 25-45 years.It is characterised by recurrences that can ultimately lead to significant visual loss. It is transmitted to humans by accidental ingestion of the egg form in cat faecal matter which may contaminate fruit and vegetables, ingestion of the cyst form in undercooked or raw meat and vertical transmission to the fetus during maternal primary infection.The most commonly used combinations to treat toxoplasmosis are clindamycin and corticosteroids and pyrimethamine, sulfadiazine and corticosteroids. The steroid should always be used in conjunction with antibiotics.Thank you."
},
{
"id": 129040,
"tgt": "Suggest remedies to relieve severe knee pain",
"src": "Patient: Hello doctor Could u please help me out I have had really bad knee pain for 2 years now I had to quit all my sports (cross country Camogie hockey and swimming) in November because it was too painful to keep going It was gotten really bad recently and it s even sore to walk now and I have gotten an x-ray MRI ultrasound and bloods but nothing has shown up could u please help what else cud be wrong ? Doctor: I suggest you to do knee exercises which includes both quadriceps and hamstrings strengthening exercises"
},
{
"id": 141873,
"tgt": "Suggest treatment for tactile sensitivity",
"src": "Patient: Hello. I m a 62 y/o female. I have always been overly sensitive to certain sounds (sometimes to the point of rage), I tend to have heightened hearing, and for several years now I have been sensitive to people innocently patting my shoulder, rubbing my arm and back. I have never felt any discomfort with my significant other (we ve been together off and mostly on for the better part of 18 yrs, and have been actually living together for 3 yrs), until recently. I flinch (especially when sleeping) and pull away at times automatically. It hurts, and I jump! This of course causes frustration for him, and frustration/depression for me because it takes me so very long to unwind enough to welcome his touch. Once we get going , all is wonderful. I have taken antidepressants for about 10 years, I have degenerative disk disease and COPD. Is there something you would suggest? Doctor: Hello!My name is Dr. Aida and I am glad to attend you on Healthcaremagic!Your symptoms could be related to a sleep disorder. I recommend performing a polysomnogram to investigate for this possible disorder. I would also like to know your actual antidepressants. Switching to another antidepressant, which can help with sleep disorders (mirtazapine, trazodone) may be necessary. Hope you will find this answer helpful!I remain at your disposal for any further questions whenever you need!Kind regards!"
},
{
"id": 104375,
"tgt": "Suffering from tonsillitis, prescribed Avamys spray. Which is the best medicine for allergy and tonsillitis?",
"src": "Patient: hi doc.! i want to ask what is the better medicine for allergy? the doctor recomend me to take aerius tablet and he gave me also avamys spray but its xpensive so i buy rino clenil spray as a substitute for it. and i also have hese tonsilitis , and i want to know what is the best medicine for my allergy nd tonsilitis. thanx and i hope you can help me Doctor: the best medicine is montair fx (montelucast and fexofenadine combination you can take once a day and add syp tossex 1/2 tsf bd Betdine for gargles"
},
{
"id": 159655,
"tgt": "Lump in the neck, advised x-ray to detect cancer. What could it be?",
"src": "Patient: My sister neck has a lump on it,she went to Dr. and he her go o hospital to get x-ray they told her it was maybe cancer or and absess(she had popped an absessed tooth a few days before)and she would have to wait for someone to look at it and sent her home. They told her if her neck goes back down she should get back to hospital ASAP because that means it busted and if she makes it to hospital.They should have kept her.What could ot be? Doctor: Hi, Thanks for query, As she had popped an absessed tooth few days before the chances of lump in the neck is Lymphadenitis.This is an infection of lymphnode. The chances of malignancy is not there as it discharges pus. She needs antibiotics & check up to rule out it's tuberculos nature. Thanks."
},
{
"id": 160950,
"tgt": "Suggest treatment for black colored tongue in a child",
"src": "Patient: Hi, Im very curious as to why my 8 month old baby has a black tongue, I just noticed it a couple days ago, and im pretty sure I think i seen it once when she was a bit younger. It s not all black, but it s towards the back of her tongues, it almost looks as if she sucked on some black candy.... Im almost certain that she didnt eat anything from the floor either! She has been playing with my oldest childs toys... so maybe from that?? I took a wash cloth and lightly rubbed it on her tongue and it is still there, this will be the 3rd day??? She also has somthing called Layrngomalacia.. I dont know if that would have anything to do with it?? Thanks! Doctor: Hello, Sometimes it can happen in any antibiotic usage or certain drug usage. Please see if the kid has been on antibiotic called linezolid.Hope I have answered your query. Let me know if I can assist you further. Regards, Dr. Sumanth Amperayani, Pediatrician, Pulmonology"
},
{
"id": 128631,
"tgt": "What causes severe neck pain?",
"src": "Patient: I STARTED TAKING THE GENERIC VERSION OF LIPOTOR (AVASTORSTATIN) ABOUT 1 MONTH AGO AFTER HEART ATTACK, AND RECENTLY HAVE BEEN HAVING NECK PAIN (MUSCLE PAIN) BURNING SENSATION IN RIGHT ARM, ALMOST AS THOUGH I STRAINED A TENDON LIFTIONG WEIGHTS AND I HAD A CHARLIE HORSE IN MY RIGHT CALF MUSCLE FOR THE FIRST TIME IN MY LIFE (I M 50). THE HOSPITALIST PUT ME ON 80MG AVASTORSTATIN ONE TABLET A DAY IN ADDITION TO EFFIENT, MOTOPOROL AND LISINIPRIL. I STOPPED TAKING AVASTORSTATING LAST NIGHT UNTIL I CAN TALK TO CARDIOLOGIST. ANY ADVISE WOULD BE HELPFUL Doctor: Dear patient you are suffering from acute cervical radiculopathy due to nerve compression at the cervical disc. Atorvastatin is not related to this. this may have happened during weight lifting. I would advise to avoid any further weight lifting. Start tab pregalin x 75 mg two times a day plus tab medrol 4mg twice a day. This wl help I'm reducing inflammatory pain. If not relieved with this medicine , visit to neurophysician is recommended."
},
{
"id": 45248,
"tgt": "Which hospital is good for infertility problem and maternity in bangalore",
"src": "Patient: i would like to know which hospital is good for infertility problem as well as for maternity Doctor: For the gynaecological care and infertility the best hospital is Dr ramesh hospital in bangalore located in 59th cross,4th block, rajajinagar. it is a good hospital we got our first baby after 9 years of our marriage, we tried at many centers, but could not succed. with in 6 months of treatment from this hospital we got the baby. we are thankful to that hospital and i recommend you to visit the hospital for your infertility problems."
},
{
"id": 105404,
"tgt": "Nose pours and sneeze whenever I run. Tried various histamines, no help. Treatment ?",
"src": "Patient: When I go for a run my nose pours for days and days after and I sneeze non stop. I don t think my body is able to regulate its temperature. I have tried anti histamine and various methods without success. The only thing that seems to help if by putting an ice pack on my head. I did the race for life this year for the first time. I ran 5K but then had to take a week off work as my symptoms were so bad, its enough to make you give up exercise, can you help ? I have never been good at regulating my temperature could this be causing my runny nose etc. My symptoms start the minute I leave the gym and get in the car to drive home. My nose drips into my dinner, over my paperwork, down my clothes, its so fast I cannot catch the drips (like a nose bleed ), its very embarrassing and the sneezing is so severe and can be 3-6 on the trot that I end up pulling muscle, lose my voice and have been known to wet myself. This cannot be right. PLEASE HELP. Doctor: You seem to be suffering from allergic rhinitis where symptoms are at it`s worst. I suggest you take montair lc one tab a day and flixonase nasal spray one squirt per nostril twice or thrice a day. Avoid extreme changes in temp, smoke and dust if you can. If symptoms still persists you can go in for allergy testing and do the levels of IgE antibodies and go in for shots of omalizumab."
},
{
"id": 128351,
"tgt": "What causes shoulder pain radiating towards chest?",
"src": "Patient: Started to experience moderate shoulder pain yesterday at the very top of the shoulder. Pain has now moved several inches to the front part of the shoulder / upper part of the chest. Does not respond to heat or cold treatments. No other symptoms present. Doctor: Dear sir /madam I would like to know do you have any palpitations or breathlessness, and how is the nature of pain. I would also like to examine your shoulder joint to look for local tenderness, and also check your shoulder movements, and then decide what to be done"
},
{
"id": 135615,
"tgt": "Suggest treatment for shoulder pain , numbness in the arm and dizziness",
"src": "Patient: HI; about a month ago I began feeling numbness and tingling in my arm and thumb; since well before that I ve had severe neck/shoulder pain especially when tilting head back. I went for an MRI last week but have to wait until Friday to review it with the dr. About 3 days ago I started with a new symptom - pretty severe dizziness when I lean forward or to the side or back. Do I need to go to the ER? Or can I wait until Friday to review my MRI... Doctor: Hi Dear,Welcome to HCM.Understanding your concern. As per your query you have houlder pain , numbness in the arm and dizziness . Well there can be many reasons for symptoms you mention in query , but in your case it seems to me that the symptoms are probably related to heart condition . So i would suggest you to consult cardiologist once also . Doctor may order ECG, TMT , troponin t test and complete blood profile . Doctor may prescribe blood thinners , calcium channel blockers along with diuretics . Some times doctor may order angiography or refer you to orthopedic surgeon in case there is no cardiac problem . Hope your concern has been resolved.Get Well Soon.Best Wishes,Dr. Harry Maheshwari"
},
{
"id": 182691,
"tgt": "Can Erythromycin and Metronidazole be taken together for tooth infection?",
"src": "Patient: Hi doctor, Can I take erythromycin and metronidazole together? I have a very severe tooth infection and have been prescribed metronidazole and a couple of days later erythromycin and advised to continue with both courses. However I have started to suffer with nausea and vomiting, is this due to the 2 sets of antibiotics? Doctor: Thanks for your query, I have gone through your query.The combination of these two antibiotic works well against the tooth infection.The reason for the vomiting and nausea can be because of the gastric irritation caused by these drugs particularly metronidazole.Nothing to be panic, You can take an antacid like ranitidine 150mg BID and Domperidone BID before food and later take the antibiotics after food. If the vomiting does not stop after these medicines, then immediately consult a general physician. Consume lot of liquid.I hope my answer will help you, take care."
},
{
"id": 117379,
"tgt": "What can provide relief from burning sensation in legs due to autoimmune hemolytic anemia?",
"src": "Patient: Hey my wife is feeling a burning sensation in her legs. Warm auto immune hemolytic anemia. She has been fine for a few years until Saturday when she decided to go to the hospital. She recieved 3 blood transfusions over Saturday night till this morning.....any ideas? Doctor: Hello and welcome to HCM,Repeated blood transfusions are associated with iron overload.Iron overload accumulates in different organs of the body like heart, pancreas, skin etc.Iron accumulation in skin causes brownish discoloration of skin and can also cause itching.Thus, I would suggest you to get an iron profile- serum iron and ferritin.In case iron overload is present, iron chelation therapy may have to be started in which excess iron is removed from body using medication.Thanks and take careDr Shailja P Wahal"
},
{
"id": 187077,
"tgt": "Severe pain in teeth while descending on the airplane and took Zyrtec prior to traveling",
"src": "Patient: My 14 year old son started having severe pain in his teeth when we started descending on the airplane this afternoon. He spit out a small amount of mucus and felt a litter better. He had taken a Zyrtec prior to traveling. The pain calmed down for about an hour or 2 but is now hurting him again. Had teeth checked and X-rays a month or 2 back so I doubt it is a cavity. Doctor: hello thanks for consulting at hcm..it could Barodontalgia, commonly known as tooth squeeze (aerodontalgia), is a pain in tooth caused by a change in atmospheric pressure. The pain usually ceases at ground level. The most common victims are SCUBA divers because in deep dives pressures can increase by several atmospheres and military pilots because of rapid changes. Barodontalgia is a symptom of dental disease, Indeed, most of the common oral pathologies have been reported as possible sources: dental caries, defective tooth restoration, pulpitis, pulp necrosis, apical periodontitis, periodontal pockets, impacted teeth, and mucous retention cysts.hence plz consult a dentist since it could be teeth problm..hope it helps,tc"
},
{
"id": 28950,
"tgt": "How can rabies be prevented?",
"src": "Patient: Hello doctor, a week ago on a lake I a hedgehog a piece of food touched or sniffed a piece of food with its nose or mouth, I ate that food accidentally. Now, I'm worried about rabies exposure. I cannot access vaccination center for the next few weeks, I feel I'm getting a flu, my throat hurts. Thank you Doctor: Hello,Rabies is primarily transmitted from the rabid animal\u2019s saliva when it bites or scratches someone. Licks to wounds or grazed and broken skin or to the lining of the mouth and nose can also transmit the disease. It can be prevented by taking five doses of anti-rabies vaccine after exposure. I've gone through your history to tell you that there's no need to worry at all as food even contaminated with the saliva of your pet hedgehog the virus shall not survive in the gastrointestinal tract. So, there's no need to get the vaccination. You may consult your general physician for a sore throat that you seem to be suffering from. Hope I have answered your query. Let me know if I can assist you further.Regards,Dr. Mohammed Taher Ali"
},
{
"id": 86644,
"tgt": "Suggest treatment for sharp lower abdominal pain",
"src": "Patient: Well for about a week and a half or so my nipples have stayed hard. And very sensitive. I m on my period now but this is not a symptom I usually experience when on my period or before. Also I have had little Auden sharp pains in my abdomen since my period has started and that s also unusual. Doctor: Hi.Thanks for your query.Hard and sensitive nipples indicate Hormonal imbalance. As is the Auden sharp pains in the abdomen since your periods have started.The possibility is having PCOD or some ovarian tumors.I would advise you the following :Hormonal assays in the blood of Estrogen, progesterone, FSH, LH, Thyroid function tests. testosterone.Blood, urine and stool routine tests.Ultrasonography of the abdomen.Consult a Gynecologist for opinion and further management."
},
{
"id": 151367,
"tgt": "Pain and burning in the ribs and on the back. Diagnosed with spondylosis. Tingling and pain in the shoulder. Need help",
"src": "Patient: My right side where my ribs are it always is in pain. If I try to massage the area then I get this overwhelming itching, burning and painful feeling internally. The pain goes down through my back to the buttocks on right side only. This is very painful. I ve had the same pain for over 20 years. I was diagnosed with Spondelosis and have have to fusion one on the neck c5 & c7. When I tell my doctor about this pain he seems to blow it off and refers back to the neck. Now after some sometime as passed 12 yrs after surgery I have expirenced my neck to begin to have pain again and now some tingling under my left shoulder blade and a painful area about the 5-6inches under my left breast . The pains on my right side have increased and I can t get any relief. Please help. Doctor: Hello, Your pain which has persisted for so long must have been seen and evaluated by doctors and in such case it may be difficult to arrive at a specific cause of pain. What I would suggest is to try different medications for such pain in consultation with a neurologist and try to achieve pain relief instead of a diagnosis. I am sure that you can get some benefit with medications. Good luck."
},
{
"id": 44964,
"tgt": "How To Increase Sperm Count?",
"src": "Patient: How To Increase Sperm Count? What Medicine Used for Sperm Count Increase? Doctor: Hello.welcome.There are medicines available which increase the sperm count like clomiphen.But they should be used under the guidance of Doctor.good luck."
},
{
"id": 17359,
"tgt": "Suggest treatment for dizziness and high BP",
"src": "Patient: Hi I am a 33 year old with dizziness, ha s and HTN over the last week. A am a RN and took my BP with a automatic cuff 174/106 after expereriencing dizziness and HA and episode of nausea. I immediately called clinic and was seen but BP had gone down to 136/102. They did an EKG, UA a/HCG and labs-all of which were normal. I am healthy, 33 y/o, work out 4-5 days a week, 5 9, 147 lbs and only take BCP s. My mom has HTN but is also overweight. I am not stressed. They thought I had BBBV or a migraine. But my HA was not that bad (it was no migraine). Why would my BP be elevated with BPPV? I saw a PA who did not seem to think it was a biggie having my BP that high. My BP s since then have gone done to 140 s/upper 90 s. I know in the past my BP is usually high when I get seen because I am really sick or in pain. For all I know I could have been having elevated BP for some time now......any ideas. Dizziness is still there a little (not as bad) was able to work out, slight HA, BP 145/98. Thanks concerned RN Doctor: Hello, As what I understand from the history is that the dizziness and nausea feeling is due to vertigo. As both are inter related and are due HMF. The nausea feeling can be triggered by Chemoreceptor trigger zone due to vertigo. Since you mentioned you Blood pressure was high on a digital monitor at home, was it same way measured digitally at the physician centre? If not than did you follow the correct way of measuring the blood pressure? like taking rest in lying position for 30 minutes before taking the blood pressure? also was this measurement of blood pressure happened in sitting position as soon as you reached the physician centre? which arm was showing signs of high blood Pressure the right or the left? Many things are unanswered in the history. but as you mentioned about your age as in the early 30s and you are a good physical workout kind of person , ignore this high blood pressure every now and then as it may be fluctuating in the time of the day. If we have to confirm you are having a high blood pressure than we need to follow the basic principle and also have to check you on a sphygmomanometer along with Echocardiogram and Electrocardiogram with some blood routine. Also, there should be some clinical symptoms before confirming you are having hypertension. As what I feel is your blood Pressure is not the concern and the concern is the BPPV. You need to meet up a good neuro physiotherapist so an exercise prescription is provided which should help you. In my clinical practice of 12 years I have been coming across such cases of vertigo and that is dealt with proper assessment and examination. Most have been having on their early 30s lowered the blood Pressure and also the symptoms of vertigo with a precise exercise prescription and vertigo medication. I wish you good luck and request you to stay calm and follow the guidelines of exercise. You should do well. Hope I have answered your query. Let me know if I can assist you further. Regards, Jay Indravadan Patel, Physical Therapist or Physiotherapist"
},
{
"id": 79359,
"tgt": "Suggest treatment for pulmonary embolism",
"src": "Patient: I was just diagnosed with pulmonary embolism. I had a CT of chest with contrast and the radiology reports states mild cardiomegaly aortic calcifications without aneurysm and nondilated main pulmonary artery. I am on blood thinner for this with a repeat CT in 6 months. My question is that I have had shortness of breath and get an acid reflux pain when I walk and have to stop. I can count to 6 and the pain goes away. Why do I get this pain in my chest and shortness of breath. Does my diagnosis in the CT scan tell you why? Doctor: thanks for asking your questionchest pain in case , along wid pulm embolism can be due to - peptic ulcer-embolism itself-pulmonary infarct you need to visit your Pulmonologist to disscuss the possibilitieswith time and drugs a s the embolisms lol get dissolved , pain lol go awaythanksfeel free to ask more questions"
},
{
"id": 16495,
"tgt": "Why is the itching and rashes still there even after applying cream for peptic ulcer and piles ?",
"src": "Patient: hi i m 43 yrs old female suffering in angina , peptic ulcer , piles ,now i have blister on my upper lip i m using a cream which was suggested my doctor & she said it is herpes but still i have itching&rashes on my lip line its so painful, kindly suggest any fast reliefing remedy for me. thanks Doctor: hi welcome to HCM.it is caused by herpes virus,it is called herpes labialis.it wil go on itself in 2 to 3weeks,but in future these wil reactivate again in severe form.now you use docosanol topical cream the lesion wil go ok."
},
{
"id": 92908,
"tgt": "Dull ache in lower left abdomen, ultrasound shows no cysts. Could this be colon cancer ?",
"src": "Patient: ok so about a month ago I started getting a dull aching in my lower left abdomen, that was pretty constant. The I went out of town on vacation and the pain got worse. So I went to emergency and the did an ultrasound to check for ovarian cysts and It was negative. the pain went back down to its normal dull aching and I returned home from my vacation. Still having the normal dull aching on lower left side ( which also does radiate to middle and back). Well today I woke up and I had blood on the toilet paper after having a bowel movement and happened 2 other times today. So I went back to emerg and the did blood work, x-rays and urine and everything came back negative. The doctor is sending me for a scope of my bowels but I have to wait a couple weeks first. I have been researching my symptoms and colon cancer keeps popping up so now I am beyond scared that is what I have. I should mention there was also a little mucous in my stool as well. I am 34 years old and mother of 3...could I really have colon caner or what other problems could it be from the symptoms I have described??? Doctor: Hello and welcome to HCM,Blood and mucus in stools can occur in number of conditions.Cancer of the bowel is one of the possibilities.However, other conditions like inflammatory bowel disease, irritable bowel syndrome, ulcerations, polyps can also cause these symptoms.Colonoscopy of the large bowel will help to visualize the bowel and identify any lesion.A biopsy of the lesion will identify the pathological change.Malignancy is accompanied by weight loss, decrease in appetite, and weakness.Loss of blood in stools causes fall in hemoglobin and thus anemia.A complete hemogram also needs to be done.Thanks and take careDr Shailja P Wahal"
},
{
"id": 171602,
"tgt": "What causes a knot at the base of the neck along with pain?",
"src": "Patient: My 3 year old has been complaining of neck pain. Last night she tried to look up at an airplane and winched in pain and held her neck. It s appears to only be the right side. We felt a small lump or knot at the base of her neck on the right side. Could it just be a muscle knot like adults get? Doctor: Hi,Welcome to Hcm,It seems like your child has torticollis that means muscle spasm of the neck which causes such severe pain. The knot feel can be muscle or even a lymph node which is very common in this age group. I would advice you to ask her to rest her neck and not to twist and turn a lot. Hot compresses Can be applied over the painful site. If too much severe pain , a pain killer cream would help immensely. Hope she gets well soon. Take care."
},
{
"id": 198252,
"tgt": "Can betnovate cream be used for streching penis foreskin hole big?",
"src": "Patient: hello doctor,i am 21 years old...i have phimosis and the foreskine is totally covered my penis head..i got the cream called betnovate (betamethasone valerate cream Bp)...can i use this for to stretch my foreskin hole big...thank you in advance doctor... Doctor: DearWe understand your concernsI went through your details. You don't have to use any ointment or cream so that you will be helped to pull back your foreskin. The foreskin is elastic. You can use some oil like coconut oil or olive oil to lubricate the skin and slowly pull it back and release forth. Daily do this procedure for 5 minutes for around one month. You will be amazed how easily the procedure works and you will be able to pull back the foreskin completely.If you still need my assistance in this regard, please use this link. http://goo.gl/aYW2pR. Please remember to describe the whole problem with full detail.Hope this answers your query. Available for further clarifications.Good luck. Take care."
},
{
"id": 211019,
"tgt": "What causes paranoid personality disorder with night traumas?",
"src": "Patient: i am paranoide, that everyone is talking about me, i see glowing sometimes and feel strange, my thoughts are blured and racing, i suffer terrible night traumers due to life traumers, i have many thoughts of why am i on this planet, i have withdrawn every type of social communication. Doctor: Hello,Thanks for choosing health care magic for posting your query.I have gone through your question in detail and I can understand what you are going through.The symptoms that you are stating suggest that you could be having a depressive episode. Patients of depression do have symptoms such as the ones that you have described. You will need medications such as fluoxetine or escitalopram or sertraline for the management of depression. If you are averse to taking drugs then there is an option of cognitive behavioural therapy (CBT). This therapy is taken by either a psychologist or a psychiatrist in which the therapist identifies impaired cognitions (Thoughts) and try to correct them. They are 15-20 min weekly session and there should be around 8-10 sessions to help you with your illness.Hope I am able to answer your concerns.If you have any further query, I would be glad to help you.In future if you wish to contact me directly, you can use the below mentioned link:bit.ly/dr-srikanth-reddy"
},
{
"id": 220568,
"tgt": "What causes pink discharge during periods?",
"src": "Patient: I ma scared to death...I am 42 years old, and yesterday after a sexual intercourese I noticed light pink discharge from my vagina. It looks like mucus but colored into light pinkish color. It is my ovulation time as well. I am concerned even more since today I went to the bathroom and wiping my self w/toilet paper I noticed the same kind of discharge again. Could it be cancer? I scheduled my dr's app. for next Tuesday, Doctor: Hi there,,Post coital bleeding could mean a lot of things, not necessarily cancer, but it is a symptoms that cannot be ignored and needs to be evaluated.You will need to see the doctor who will examine your cervix and vagina to see if there is any local cause for it such a cervical erosion, cervical polyp or cervicitis. If your Cervical cytology/PAP smear is due get that taken as well.If any infection is seen , antibiotics should be taken for the same.You will also need to de a pelvic ultrasound to see the lining of the womb and if there is any thickening or not. A thickened lining can cause bleeding in mid cycle.Cervical cancer can be ruled out with cytology and if any suspicion is seen a colposcopy is advised.If the lining of the womb is thickened we will need to send a small biopsy of it to the laboratory for evaluation, all this can be done at the gyanecologist you visit her/him.Do not worry.Hope this helps.Regards."
},
{
"id": 8972,
"tgt": "Skin is too dark only on my face",
"src": "Patient: heelo sir, m 28 yr old, my skin is too dark only on my face , not in whole body so i had consulted a doc who suggest me to take some kind of isotroin 20, acutret and some creams as skinlite/metasone plus, from 1.5 yrs i hv taken that but last 6 months i leaved them, due to which my skin get effected as too dark and oily,. plz suggest me what can i do for that plzzz...... i think these tabs are effecting to my skin, plz tell me also, what are side effect of that. some time i feel too depressed too, i had taken regularily these medicine but from from sometimes i had stopped this by which i m facing these problem. m telling u what tabs i had taking as prescribed by cosmatologist doctor Isotron 20, Acutret cream, metasone plus cream, skinlite cream, Hyclene cream Plz do needfull doc thanxx Doctor: hi, for having a glowing skin you should have lots of water.There are many herbs which will help you to have fair complexion.take good amount of fruits and take care that you are not consipated. Apply powder of Manjishta+Lodhra with honey,and apply as face pack. you will get good result"
},
{
"id": 42945,
"tgt": "Is there any relation of Primolut treatment to infertility?",
"src": "Patient: Hello, im 40 yrs old I was diagnosed of having cervical polyps. Last feb. 25 I got my period when ivicited my ob gyne dr last march 2 she adviced me to take primolut10mg bd for 21 days because she will post me for polypectomy & hesteroscopy and it was done last march 9. Now im still taking primolut as what she adviced. I dont know why im taking this pill for this long period. All I know is to stop the bleeding. Please help me understand the treatment of primolut to my case. By the way im married for 10yrs still trying to to have a baby. We tried infertility treatment before. Is there any relation of primolut treatment to infertility? Thank you very much &more power!!! Doctor: Dear patient! I understand your concern. Primolut is basically meant to regulate your cycles and to thin out the endometrium (so as to facilitate the procedure) and so that you dont get spotting or bleeding following the procedure. And you should stop it after 21 days.As far as infertility is concerned, primolut regulates cycles (which facilitates pregnancy). Hope that answers your question. Awaiting your precious feedback."
},
{
"id": 174792,
"tgt": "What could it be if child is having intermittent cough inspite of taking medication?",
"src": "Patient: 18 month old boy has a recurring dry cough, mainly at night. It will last 3-4 days then loosen and disappear only to return about 5-7 days later. I have tried everything including vaporizers, humidifiers, vaporub, glycerin and honey etc. What could this be?! Doctor: Hi Dear Welcome to the HCMRecurrent respiratory tract infections are quite common, But only 6 to 12 episodes a year are ok, not more than that.Inadequately or /partially treated case may many times present like this.Such children to be treated/investigated with the consultation of pediatrician. All the prescribed drugs to be given with recommended doses and duration according to the weight of the baby.Hope the query is answered.Thanks"
},
{
"id": 115661,
"tgt": "What causes elevated monocytes in blood while having anemia?",
"src": "Patient: Hi, I'm 18 years old and suddnely Have been diagnosed as an anaemic after doing some blood tests but at the same time my monocytes was a little bit high. So I want to know is it high because I'm anaemic or there is another underlying issue I need to know and thanks. Doctor: Hi, dear. I have gone through your question. I can understand your concern. Monocyte count is high in various conditions like infectious mononucleosis, viral infection, malaria etc. No need to worry about that. Slight increase in monocytes count is seen without any disease. You should go for peripheral smear examination and anemia profile and start treatment accordingly. Hope I have answered your question, if you have doubt then I will be happy to answer. Thanks for using health care magic. Wish you a very good health."
},
{
"id": 95757,
"tgt": "Stomach pain after taking morning after pill & had normal period 3 days later",
"src": "Patient: stomach pain after taking morning after pill had normal period 3 days later Hey well I m 16, was fooling around with my boyfriend, well lets just say sperm may or come in contact with me, well within 24 hours I took the morning after pill (levonelle) well had my period the next week 3 days late lasted 7 days like normal but now am having stomach pain every now and then like an ache, don t know what it is so if you can help in anyway much thanks Doctor: Hello; welcome to HealthcareMagic It is normal to get periods with in a week after taking the morning pill so you had in 3 days.As for the pain in abdomen you have it needs to be examined to see what sort of pain you have like spasmodic pain;cramping in lower abdomen or a dull aching pain and accordingly the causes will be different.I guess since you got periods the pain should not be related to the periods or pills.Please consult a doctor in person for examination and treatment. Thanks"
},
{
"id": 48271,
"tgt": "What does this result of abdomen indicate?",
"src": "Patient: I had a ct scan of abdomen pelvis w/contrast as pain in stomach. Report says I have fatty infiltration of the liver, Pneumobilia is identified in the left lobe. Extrahepatic bile ducts are dilated, likely reflecting the prior cholecystectomy. What does this all mean? Could this cause a lot of bloating? Doctor: hello,welcome to HCMpneumobilia means air in the billiary tree.it occurs when there is entry of air from enteric system to billiary system.causes are-recent instrumentation (ERCP),gall stone,billiary enteric anastomosis,pancreatitis,BE fistula,etc.extrahepatic billiary duct dialated means the hepatic duct and CBD are dialated.cause is distal obstruction.many of this causes are associated with bloating discomfort.i shall advise you to consult a gi surgeon with your reports.hope this will help you.regards."
},
{
"id": 132099,
"tgt": "Will Neuracel relieve me from leg aches, when I try to walk?",
"src": "Patient: My legs ache when I try to walk. Sometimes they are stiff. I am 74, diabetic 2. My doctor told me I had Neuropathy. I found this medicine, Neuracel on the computer to buy for pain. Would you know what ingredients are in the medicine? Thank you YYYY@YYYY Doctor: Hi you have diabetic Neuropathy in both legs and causes pain and stiffness. You have specifically asked about Neuracel an OTC product. Since this product does not contain any allopathic formulation , no doctor could comment on the authencity. I personally do not recommend any product , where i do not know the effects and side effects- Pharmacology of the product."
},
{
"id": 91610,
"tgt": "What can cause lower abdomen pain for a person having 2.5 cm ovarian mesenteric cyst?",
"src": "Patient: hey, thanks for doing what you do for us. had recent er trip with lower right abd pain. ct scan found 2.5 cm ovarian mesenteric cyst. however, I had complete hysterectomy 14 years ago. sent home with pain meds and nausea meds. seems as no big deal. something doesnt feel right. what should I do? Doctor: HIThis is a cyst after complete hysterectomy. So better be excised on toto and subject to histopathology.You get rid of the disease and get a real diagnosis too. Also get the visualization of intra-abdominal parts for any other possible reason for pain which are missed even on CT scan."
},
{
"id": 57732,
"tgt": "Suggest medication for liver pain",
"src": "Patient: I have CHB(chronic Hepatitis B), the viral load is below 20IU/mL, but in the liver function test Total Bilirubin is normally above 33\u03bcmol/L(19.3mg/L), Direct Bilirubin above 8\u03bcmol/L(4.7mg/L), Indirect Bilirubin above 28\u03bcmol/L(16.3mg/L), there is usually ache around the right liver, liver ultrasonic check result is normal, I have taken enticavir for almost 3 years, I just want to relief the liver pain, how could I manage? Doctor: HIThank for asking to HCMI can understand your question, taking any pain killer for liver pain is not advisable because analgesic is not good for ill liver the analgesic could flair up the condition, you have to manage it conservatively without the painkiller, and this will go away on its own, do not worry about it, take care and have nice day."
},
{
"id": 48227,
"tgt": "What causes pain in the urinary bladder and kidney area?",
"src": "Patient: I am 29 years old, I I had 3mm stone, however before sometime i have done KUB USG, it has shown the stone is no longer in my kidney. But from last one week I m having pain in back lower part and the same pain I can feel in my urinary blader left side. Please advise, medicine and causes. Doctor: HelloPain in lower back may be due to many causes like renal or ureteric calculus,musculoskeletal causes,infection etc.You need proper clinical examination and investigations.investigations include routine hemogram,random blood sugar,urine RE/ME,renal function test,ultrasound of abdomen.Ultrasound of abdomen can exclude many possibilities.X-ray of L/S spine and CT scan can be done if needed.Proper treatment depend upon findings.Get well soon.Take CareDr.Indu Bhushan"
},
{
"id": 196157,
"tgt": "Suggest treatment for testicular varicocele",
"src": "Patient: Hi I had vericocles in my teaticles confirmed with ultrasound but since it was not that painful the eurologist I consulted did not prescribe any medication. However it has now reappeared and is very discomforting. Tell can it be treated with medicine ? Doctor: hello and regards.varicocele means testicular veins enlargement.it may affect your fertility.kindly consult your urologist.the treatment is varicoceletomy meaning surgical removal of the dilated veins.he will guide you in proper way.hope my answer would have helped you."
},
{
"id": 41881,
"tgt": "How to improve sperm count?",
"src": "Patient: The last count was 35million. I have pus cells in my sperms. I was placed on anti-biotics and recently went for a check up and it went down to 5million. My wife and I have been trying for over 6years. She is on clomid. What can we do again please help Doctor: Hi welcome to healthcaremagic.I have gone through your question.You mention sperm count 35 million. Is it per ml then is normal. Normal value 20 million/ml.Now is 5million is absolutely below normal, significantly low. I would advise you to go for ultrasound of scrotum if not done to rule out varicocele, infection of testes, vascularity. And consult urologist if report is abnormal.Hope i have answered your question.Would be happy to help you further.Take care."
},
{
"id": 157330,
"tgt": "Liver cancer, weight loss, gas. High calorie vegetarian food?",
"src": "Patient: My relative is suffering from Liver cancer and is undergoing Chemo for about 3 months. He has lost about 18 lbs in 3 months and a constant challenge is to provide him foods that are high in calories to give him energy and low in sugar (for cancer) that is vegetarian as well. Also as the problematic area is liver, it has been very difficult due to gas. We have been many small meals, but not making much dent in weight management . Please help suggest high calorie vegetarian foods that would be easy on liver and not harmful for cancer. Thanks. Doctor: Understand that no diet is harmful for cancer. i dont understand why low sugar diet is being given when he is not diabetic. Sugar has nothing to do with cancer, it is restricted only in diabetics. If he not a diabetic best think would be to give plenty of home made fruit juices with milk and glucose added to it. Glucose is very good for liver and helps in rejuvenate. Avoid fatty foods and oil as they are difficult to digest. Porridge with jaggery and milk shakes help him a lot"
},
{
"id": 3902,
"tgt": "Will taking HCG 5000 help in getting pregnant?",
"src": "Patient: Hi, i went through afollicle study last month and the follicles were normal and 20mm follicle got ruptured on day 15...but i did not concieve....i am again undergoing a follicle study..today is day 13 and follicle in left ovary is 19.3mm....i also got a PCT done this morning and result was normal per the gynae...however,she asked me tp take HCG 5000....how would this help? Doctor: Hi, thanks for writing..HCG helps in rupture of the ovum.. Rupture will take place in 36 hrs after the injection.. This is the time you should have intercourse or get an IUI done.. In the post ovulatory phse take progesterone for 10 days for luteal phase support.. HCG has two roles. One is ovulation and second is to support the pregnancy.. Hope I have answered your query.. Good day.."
},
{
"id": 89335,
"tgt": "What causes swelling of the lower abdomen?",
"src": "Patient: I am 47 yr old type II diabetic. I had a heart attack at 43 at which time i had a stent put in. I have had lower back pain for the last 2 days and for the last hour my lower left abdomen is swelling...at first i thought appendix but if i am not mistaken that is on the right....swelling is painful ...bout 7 atm Doctor: HI Thanks for posting your query . You didn't mentioned whether you are male or female as diagnosis can change with sex . Swelling on one side of abdomen is always a matter of concern . Left side swelling could be due to some mass or cyst in the left abdominal quadrant . That too an associated lower back pain points towards problem in your left kidney . Also a possibility of pus collection is there . Is your bowel movements normal ? If not passing bowels partial intestinal obstruction is also a possibility . If you are a female then Ovarian Cyst , Ovarian mass or Pelvic Inflammatory disease . Ok sorry if I am being too technical . Just do these things : 1. Get Ultrasound Abdomen Pelvis Scan 2. Get Urine Routine test Revert back with these reports for further discussion . Any clarifications feel free to ask . Regards."
},
{
"id": 180739,
"tgt": "What do small bumps on the tongue indicate?",
"src": "Patient: I have noticed a number of growths towards the rear of my tongue, but on the side. They are the same colour as my tongue, but protrude quite a bit. I think they may be foliate papillae, but I am not sure. I have them on both sides. Further to this, I have a swollen right tonsil, but no pain Doctor: Hello,As per your explanation, it looks like you have swollen circumvallate papillae. Circumvallate papillae are present on the back one-third of the tongue and are arranged in a typical \"V\" shaped pattern.They are large papillae so slightly raised appearance is normal but in case if they are red and inflamed and swollen they are not normal and can occur due to a number of common causes like burning with hot foods and liquids, inflammation caused due to chemical injury like using concentrated solutions of mouthwash, allergic reaction to medicines, acid reflux, oral infections like thrush, throat infection, postnasal drip etcetera.You should consult an oral physician and get evaluated and treatment of the underlying cause can relieve. As of now, you can do warm saline gargles, soda bicarb gargles, suck ice pops and do cold water rinses. You can also avoid hot and spicy food and take a soft diet. Drink plenty of water and maintain good oral hygiene.Hope I have answered your query. Let me know if I can assist you further.Regards,Dr. Honey Arora"
},
{
"id": 2867,
"tgt": "Is pregnancy possible after unprotected intercourse?",
"src": "Patient: Hi Doctor, I got married on june 15th. I was on contraceptive pills (yasmin) till june 15th. I had unprotected sex on june 16th to 18th.After my periods from 18th.I started with the pill from 26th June. I got my periods on 20th of July. And in july on my 6th day of my period i had unprotected sex. Are there chances of pregnancy? Please suggest. Thanks, Rina Doctor: Hi,Yes there is a chance of getting pregnant as ovulation can occur early in some cycles and you could be ovulating at that time.Hope I have answered your query. Let me know if I can assist you further. Regards,Dr. Salah Saad Shoman"
},
{
"id": 113628,
"tgt": "Have constantly had nausea and heartburn. Why do I get dizzy theses days?",
"src": "Patient: hi, I am a 16 year old, I have constantly felt nausea , heartburn and indigestion whenever I ate. Sometimes i cannot even sleep at night. Along with that I have had some upper abdominal pain (stomach area) as well as pain in the right side of my back towards the upper part of my abdomen. I have been to a nephrologist and have had full tests for my kidney, liver, gall bladder , as well as urine and blood tests, showing negative results. I really haven t been feeling well at all and I have started to feel dizzy these days. Any suggestions? Doctor: I suggest you see a gastroenterologist to perform endoscopy of your Gastro intestinal tract.It WOULD ALSO BE GOOD TO HAVE A GASTRIC MOTILITY STUDY TO KNOW ABOUT YOUR BOWEL MOVEMENTS.In addition you may be required to undergo more tests to rule out malabsorption syndrome too.hope this has helped youget well soon"
},
{
"id": 16116,
"tgt": "What can i do to remove pimples from my face ?",
"src": "Patient: my face on many pimpals this pimpals removed m face what are you doing? Doctor: many pimpals remove in my face wich madician take"
},
{
"id": 206549,
"tgt": "Suggest treatment for insomnia and depression",
"src": "Patient: hi ,i think my contraceptive pill is causing severe anxiety. i started taking it about 4 months ago when i was struggling with severe depression and anxiety before and during my period, but since being on the pill it has got a whole lot worse. i have anxiety attacks every day, suffer from insomnia and feel paranoid and anxious about everything. its putting a real strain on my relationship with my boyfriend and my family. I do have the copper coil, which I've had for just over a year now, and was planning to have it removed and continue with the pill but i don't know if either of them are making me worse. i also have a history of kidney problems, having one kidney removed in 2012. any kind of advice or help would be greatly appreciated!thanks sophie Doctor: Hello welcome to Health Care MagicCommonly used contraceptive pills may aggravate depressive symptoms, and can also cause mood swings and precipitation of panic attacks. In susceptible individuals or in individuals with pre existing depression risk is high. Insomnia may be a part of anxiety or may be due to hormonal changes. If you are having serious anxiety attacks consider consulting a psychiatrist. Medicines like SSRIs and SNRIs are commonly used. Even a lot of medicines cause improvement in menstrual symptoms also. To improve your relationship with boyfriend and family you can consult a psychologist for relationship counselling.Hope this helps you, thanks"
},
{
"id": 80184,
"tgt": "Suggest remedy for itchy dry cough",
"src": "Patient: Hi doctor Iam having severe cough since 10days..it is dry cough n more frequent at night..itching is present at throat but there is no fever...I have taken syrup rapitus fr 4days n taken tab.azithromycin fr 3 days and preaently on ambrodil syrup...but still cough s irritating..the more I take breath, more will b da cough. Doctor: Thanks for your question on Health Care Magic. I can understand your concern. In my opinion we should rule out bronchitis in your case. You are not improving with drugs, this suggest possibility of bronchitis. And bronchitis is very common after upper respiratory tract infection (URTI). So get done PFT (Pulmonary Function Test) for the diagnosis of bronchitis. It will also tell you about severity of the disease and treatment of bronchitis is based on severity only. You may need inhaled bronchodilators and inhaled corticosteroid with antihistamine drugs. Hope I have solved your query. I will be happy to help you further. Wish you good health. Thanks."
},
{
"id": 139862,
"tgt": "What causes parkinson s diseases ?",
"src": "Patient: I have a Nephew 53 years old with Parkinsons, he s on vet. benefits, and goes to a VA Dr. My sister is getteing afraid of him, because he will go into fits of anger, throw things speak in foul language, than the next day not remember what he has done... it is the medications causing this? Doctor: Hi,It would have been useful if you had mentioned what the medications actually were and their dosage. Generally speaking while medication for Parkinson's may cause some changes in behavior, it is not typical to the extent you describe. Other causes should be considered like depression or other Parkinson's disease like syndromes with more prominent behavior changes. Hope I have answered your query. Let me know if I can assist you further. Regards, Dr. Olsi Taka, Neurologist"
},
{
"id": 56022,
"tgt": "What can cause spots on the liver?",
"src": "Patient: Hi, I am 42 and just past august, i was feeling sick, i seen my doctor next day and he said i had to go hospital right away for blood trazfussion. I was enemic also and i had bleeding varcies too. I am now at stage 4 of cirhosis my liver and now i am waiting for my MRI To happen in August 21st, they spotted three spots on my liver, what can they be? and should i be getting worried! thanks Doctor: Hi,Thanks for writing in to hcm.I understand your problem and will try my best to help you out.In a patient with liver cirrhosis, with liver spots mean that there are either:1. Regenerative nodules : regeneration of liver cells taking place2. Dysplastic nodules in the liver showing abnormal cells.Please dont be worried. Let the doctors carry out an MRI and correlate with the clinical findings. You need to start taking medications as prescribed by the hepatologist and also need to follow a liver healthy diet.Bleeding varices is not a good sign of liver condition hence kindly take medications regularly to prevent further complications.Completely cut down on alcohol and smoking.Hope this answered your query.Get well soon.Dr.Riyanka"
},
{
"id": 58767,
"tgt": "Have multiple comorbidities with compartment syndrome. Swelling due to water retention. Suggest",
"src": "Patient: My name is Tamara. I am a 32 year old white female with class V membraneous lupus nephritis. I also have multiple comorbidities including compartment syndrome in my lower extremeties with gross edema. Recently, I have had boughts of severe swelling that results in massive water retention, to the extent of 40 pounds gained in less than a month. While kidney function is fine during this increase and I can eventually pull off the fluid with crazy diuretic therapy, after the output, it seems that all of my muscles from mid calf to ankle are now hardened and very tender to touch. I have waited several weeks in the hopes that the tissue would return to normal, but no avail. What could have caused this, what condition is this called, and is there any way to reverse this as it is very painful (I doubt there is in my experience *I am an RN who's specialty is Renal Transplant/Nephrology* ). Any information regarding this would be very much appreciated so that I can continue on this side. That you! Tamara. Doctor: Hi Thank you for choosing HCM, Look mam you have pretty tough condition in the sense of medical ground,, in my opinion such case needs team work,, this could be chronic nephritis, I personally ask lots of investigations to come on line of treatment and to know the prognosis, without this, it is not possible to come on any decision, you batter have words with your treating Doctor,, meanwhile cut down the salt,,take care of your self, Have nice day"
},
{
"id": 56466,
"tgt": "Suggest treatment for jaundice and weight loss",
"src": "Patient: Hi, My age is 27, Female, in the month of July I got high fever which got cured by taking alopathic medicine, but after 2 days i got rashes on my skin after 2 more days my skin and eyes got yellowish.Doctors diaginiased Jaundice, as i we thought it was due to reaction of ALopathic Medicine, we switched to Homeopathy. Now am weak and lost weight plus also not putting on weight. But Now on 25.102014, LFT report is Bilurubin-1.15 mg SGOT-152 SGPT-142. Please suggest what to do now. Doctor: Hi, dearI have gone through your question. I can understand your concern. Your AST and ALT levels are high. So you have some liver damage. It may be due to some viral infection, alcohol, drugs or toxins. You should go for hepatitis panel and ultrasound abdomen. Search for the cause of liver damage and take treatment accordingly. Hope I have answered your question, if you have doubt then I will be happy to answer. Thanks for using health care magic. Wish you a very good health."
},
{
"id": 202298,
"tgt": "Is it safe to take Amoxiclav for epididymitis?",
"src": "Patient: I have been diagnosed with epididymitis by my GP and been put on co-amoxiclav 625mg three times a day for 7 days. I'm 3 days in and the pain seems to be getting worse. I've read elsewhere online that co-amoxiclav isn't the right treatment in cases of epididymitis in men under 35 as an STI such as chlamydia is usually the cause. However other websites state that amoxicillin-based antibiotics such as co-amoxiclav can be used to treat chlamydia successfully in pregnant women. Should I contact my GP to alter my medication or can I expect the co-amoxiclav to have an eventual effect? Doctor: HelloThanks for your query,based on the facts that you have posted it appears that you have Epididymitis and have been prescribed Amoxiclav.thrice daily.Please continue taking he same antibiotic along with anti inflammatory medication like Diclofenac twice daily.It takes 10 days to get the results and may have to take antibiotics for 3-4 weeks Dr.Patil."
},
{
"id": 98338,
"tgt": "What causes shortness of breath and anxiety?",
"src": "Patient: I am asthmatic and take flovent and a rescue inhaler when I exercise. My flovent is usually fine and does the job however, I am noticing I am having a hard time taking a deep breath which also is increasing anxiety. I had a lung function test about 4 months ago and everything was good. can asthma get worse this quickly? Doctor: Hello. Welcome to healthcare magic. Asthma is unfortunately a disease with a waxing and waning course. You will have good days and bad one, and this can get affected by a number of factors like the amount of dust or pollen you may be exposed to on a particular day. But, i believe that your breathlessness is being brought about more from your anxiety itself. You could try some breathing exercises and calming techniques when you start feeling breathless to rule out anxiety as a cause of the breathlessness"
},
{
"id": 11954,
"tgt": "How to gain 10 kgs with in a month ?",
"src": "Patient: i want to have very chubby cheeks and fat hands. what should i do ? I have just one month to complete my studies and join my It job feel im 2 thin suggest somke measures to become fatter in two weeks plz and to remain fatter Doctor: hi, welcome 2 health care magic include high calory foods in your menu eg ;ice creams,sweets,lessy,shakes etc but it is not a healthy way"
},
{
"id": 145619,
"tgt": "Suggest treatment for vertigo and epileptic seizures",
"src": "Patient: HI MY NAME IS RODNEY G BANKS A 49 YEARS OLD BLACK MAN WHO HAVE BEEN SUFFERING WITH VIRTIGO EPILEPTIC SEISURES..FOR ALMOST 10 YEARS NOW AND CANT SEEM TO FIND ANY DOCTORS AFTER 4 YEARS WHO CAN HELP ME GET BETTER... HAVE NOT WORK SINCE 2010 BECAUSE OF THIS VERY SICKING DISEASE... THAT NO SEEM TO BE ABLE TO HELP ME...BECAUSE OF MY PHISICAL BODY SIZE I FEEL THEY THINK I AM PLAYING SOME TYPE OF GAME ABOUT MY ILLNESS BECAUSE OF MY BODY LOOK HEALTHY.AND THEIRFORE ALL DOORS OF ASSISTANCE HAVE BEEN CLOSED ON ME STILL SUFFERING ....ABOUT TO LOSE MY MARRIAGE BECAUSE OF THIS DISEASE...CANT WORK OR PROVIDE FOR MY FAMILY....FOR THE LAST 4 YEARS VERY HIGHT LEVEL OF ANXIETY AND DISCOMFORT... NEED HELP...682-234-7695 Doctor: Hello !I understand your concern.Yours is a very rare and specific condition.I would like to know more about the treatments you have tried before without results. Have you had a Braun MRI?This disorder can be caused by a structural lesion of the brain . If you were my patient I would recommend you a brain MRI and EEG to find out if there is any brain lesion.I would recommend you treatment with Oxcarbamazepine which has given good results in cases like yours. It is very helpful in vertigo too.Other drugs you may use are Carbamazepine, levetiracetam etc.If no results from therapy you may need brain surgery, if there is a lesion in MRI.Hope to have been helpful!Best wishes Dr. Abaz"
},
{
"id": 127930,
"tgt": "Is surgery recommended for a deep tissue hematoma post accident?",
"src": "Patient: In August, my friend was involved in an auto accident. She sustained a deep tissue hematoma that failed to resolve. Doctors have tried draining, but this has not reduced the area. She was informed of a possible need for surgery, and that this syndrome could be life-threatening if not treated. Is there a syndrome that describes this condition? Doctor: HiHope this message finds you in good health. I have gone through your message and understand your concern. such kind of unresolved hematomas can lead to cellulitis and pressure symptoms on the surrounding soft tissue structures. there is no such syndrome to describe this kind of unresolved hematomas.CT guided aspiration and drainage is recommended follwed by histopathology study of the lesion.I hope your query has been answered. Feel free to consult me anytime.Thanks. Take care. God bless"
},
{
"id": 168366,
"tgt": "What causes severe fatigue, nausea and drowsiness in children?",
"src": "Patient: The past week and a half my 12 yr old son has been extremely tired after school - to the point where some days he will sleep right away. He had an espisode about a week ago at night with funny feeling head, instant drowsiness and nausea and last night his legs felt very heavy after walking up the stairs and he was nauseated. Not sure what to think.. Doctor: Hello. I just read through your question.Most often these things are caused by a virus which ultimately passes on its own. Rest and hydration are usually all we can do. One other diagnosis to consider is Mononucleosis. I recommend consulting with your doctor and discussing this possibility."
},
{
"id": 206688,
"tgt": "Suggest remedy for panic attacks and anxiety",
"src": "Patient: Hi I am a 27 year old male. I have and have had panic attacks for close to two years now. I take sertraline 50 mg once daily, and clonazepam 0.5 mg twice daily. But I recently ran out of my medication, I have no insurance, no transportation, very low income, and I feel lost and have no idea what my next step is. Doctor: Dear young man,First of all let me assure you, it is treatable and curable with the effective support of a Psychiatrist, Psychologist and your unconditional effort.In this platform, you need to be bit more explicit, then only the professionals available can effectively support you.Whatever you mentioned here are the classic symptoms of anxiety. To put it in right way, anxiety is generated from the animal instinct of \"FIGHT or FLIGHT\". That is when we see, hear, feel, or think of something dangerous, we feel anxious either to handle it(FIGHT) or to get out of it(FLIGHT). An ultimate decision is taken based on the situation in hand+ the past experiences you had confronted. Without anxiety to a certain level, you will not be able to adjust with the life or to perform well in any sphere of life. Be it in an appearing of exam, reaching on time for an interview, catching the bus or train for a journey etc. If you are not anxious, you will be missing out many opportunities. Even while crossing a road on a busy street, you should have an anxiety to keep you away from the speeding vehicles. For a person to be ambitious, anxiety is the good stimulator.In the normal course of life, we eat something and forget what we had in the square meal. The digestive system has certain juices(Chemicals) which process the food in the stomach and whatever needed is kept and remaining waste is disposed. In the same fashion, we have some chemicals (You need not know the scientific names of it) in the brain which help us to manifest appropriate emotions in any given situation.Fear or anxiety are all such emotions. You can see, you are happy in a friend's wedding ceremony or in achieving something. In the same way, you feel sad, when you are attending a funeral. It will be awkward if you feel sad in a happy situation or feel happy in a situation warranting gloom. This is the way the chemical in the brain balances and functions.Like a balance, when both the sides of it are maintaining the same weight, you are satisfied to buy something. The same is the case with all living beings generally.The situations of life are such that they always try to test your potential. It is like teaching you everything and making you ready for the final test by giving you periodical, slip or surprise tests.A basic understanding of the situations is very much needed in this point of time.As I mentioned earlier, the chemical balancing in the brain can be looked after by a Psychiatrist by the medicines. The medicines you are taking should be able to help you in balancing the chemicals.Consult the Psychiatrist and explain your concerns to him. He may increase the dosage at his discretion.The services of the Psychologist will be beneficial in letting you know of your personality traits positive or negative by taking a simple yet comprehensive peronality and anxiety tests. By this you can come to know what type of personality you are and what are the changes you need to incorporate in your personality to be a fully functional person. The anxiety can be easily reduced by having an understanding and a modification in the behavioural changes as counselled by the Psychologist based on your personality. Be ready for a change and consult the mental health professionals in town for a comprehensive treatment plan. It is always cost effective. You need not incur huge expenditure for this.Get well soon,Dr Rajesh Thottinal KalamRehabilitation Psychologist from India"
},
{
"id": 143027,
"tgt": "What is the cause of tingling?",
"src": "Patient: Hi,what does it mean if I have High Limphosites,Lymph absolute high and Basophyl Absolute is also high.Aa well as S-CI 109,S-Ureum 7.5......I have extreme pain and swelling in left upper arm with needles and pins from shoulder to fingers your get rid of the disease and 2> you get the histopathological diagnosis- makes you tension-free..All the best , I can understand when a granddaughter is ill ..."
},
{
"id": 114477,
"tgt": "Suggest treatment for reactive hypoglycemia",
"src": "Patient: My blood pressure is 133/ 58. I am on beta blocker, been eating n exercising for a couple weeks n lost 10 pounds. I also have reactive hypoglycemia as well. Seems like the two are coordinated. Should I wean down again as I m losing weight? *previously lost 33 pounds n went down 50 mg of betablockers* Doctor: HiFor the majority of people, reactive hypoglycemia usually doesn't require medical treatment. It may help, however, to pay attention to the timing and composition of your meals:Eat a well-balanced diet, including lean and nonmeat sources of protein, and high-fiber foods, including whole grains, fruits and vegetables.Avoid sugary foods, especially on an empty stomach.Be sure to eat food if you're consuming alcohol, and avoid using sugary soft drinks as mixers.Eat several small meals and snacks throughout the day, no more than three hours apart during the waking hours.Most people will try to find out what dietary changes are helpful for them to minimize the symptoms. For some, particularly those who have had stomach surgery (gastric bypass or surgery for the management of ulcer disease), further evaluation by a doctor may be warranted, but dietary changes are still recommended and very important.It's also important to include physical activity in your daily routine. Your doctor can help decide what's right for you."
},
{
"id": 116468,
"tgt": "What does WBC-6.9,RBC-3.71 and hemoglobin test level-106 g/dL?",
"src": "Patient: Hello, I'm a female teacher, 40 years old. I just got my hemoglobin test result which is 106 g/dL, hematocrit is 0.32, white blood cell 6.9, red blood cell is 3.71. I'm experiencing dizzines and I'm having difficulty in concentration. What type of anemia am I suffering? Doctor: Hi,Thanks for asking.Based on your query, my opinion is as follows.1. You are having anemia. This is causing dizziness and fatigue resulting in difficulty in concentration.2. Complete red cell indices or peripheral smear is necessary to identify the morphological type and the cause of anemia. Based on RBC count and hemoglobin level it appears to be macrocytic anemia.3. Take Vit B12 and folic acid supplements and also increase in diet through fish, egg, pulses and green leafy vegetables. It should improve soon. A reticulocyte count test after 15 days can show if its improving. Hope it helps.Any further queries, happy to help again."
},
{
"id": 73258,
"tgt": "Suggest remedy for persistent cough",
"src": "Patient: My mum has been using Ramipril for a long time. She has been coughing for most of this time. About a month ago she had the Flu injection and is blamimg the cough on that. I have told her to go back to her doctor. This is also advice given by the pharmacist at chemist. Doctor: Hello dear , hiWelcome to Healthcaremagic.comI have evaluated your query for your mother .* The persistent cough is in relation to the side effect of Ramipril , she has to consult her doctor and get the molecule changed to control the issue .Hope this will help you for sure .Regards ."
},
{
"id": 59184,
"tgt": "Have pain in arm, back and abdomen. Burning sensation. X-ray shows gall bladder stones. Guidance?",
"src": "Patient: following are the symptoms which I am having as of now.. 1. Little pain in right abdomen middle and lower Portion 2. Right hand under arm very less pain. 3. Upper back pain and burning sensation 4. Some gaseous things floating in my right abdomen and feels kind of not normal situation 4. Feels to do peeing very urgently but not very frequently based on the water intake. 5. My stomach makes lot of sound ... ( not gaseous outcome .. only inside sound) Recently went through ultrasonic test, blood/ urine test , X-ray about gallstones as I had little pain in upper right abdomen last week of March 2013,,, and found negetive .. Some acidic medicines i am taking it .. Please guide me what kind of symptoms is this.. what diagnosis I should carried out.. Thanks Doctor: Hi Gallbladder stones are better diagnosed by Ultrasound abdomen.Only 10% of gall stones are seen in x ray.As they are negative,gall stones are excluded. Pain in the abdomen may be due to gastritis.Please take Rabeprazole tablet along with itopride. Pain in other regions may be musculoskeletal in nature.Please take muscle relaxant tablets. If the stomach pain still persists,get an upper GI endoscopy Regards"
},
{
"id": 165824,
"tgt": "Suggest treatment for constipation in a child",
"src": "Patient: Hi, my son is 14yrs old and for the past 3yrs his underwear are always saturated with poop stains..i have took him 2 his doctor and they took xrays of his stomach and they diagnosed him as being full of stool , they gave him Miralax, which doesnt help, i took him back just recently and they told us 2 make pudding packs which consist of apple sauce and konsyl mixed in pineapple juice...thats not working...Please Help Doctor: Dear parent , you can use lactulose syrup 3 times daily to treat constipation.. in the meanwhile, you should make sure that he drinks enough fluids daily (at least 8 cups daily with one of them should be orange juice) . also you should make sure he is eating high fiber diet with increased portions of green vegetables and bran"
},
{
"id": 213563,
"tgt": "Not getting sleep, taking tryptomer and eliwel, feels drowsy, unable to concentrate, not stressed or depressed",
"src": "Patient: Hello Doctor!! I am 29 year old male having 180cms height & 74Kgs weight. My problem is that I am not able to get sleep at night. This condition is prevailing since last 1 year or so and I had been given Tryptomer 25Mg & Eliwel 10Mg tablets by doctors, initially a few days these medicines helped a bit but after that the condition was same. I feel really drowsy at day times & unable to concentartae properly on work. What could be the reason. There is no depression or stress related factor for me. Doctor: Hello. Thanks for writing to us. Lack of sleep might be related to a subclinical anxiety. You need to do certain lifestyle modifications like hitting the bed at same time everyday, etc. You can take melatonin supplements and poppy seed milk at night for a better response. I hope this information has been both informative and helpful for you. Regards, Dr. Praveen Tayal drtayal72@gmail.com"
},
{
"id": 191938,
"tgt": "Suggest treatment to control blood sugar and cholesterol",
"src": "Patient: I have a total cholesterol of 223 and I am a woman age 55. I exercise about 3-4 times a week for 30-45 minutes. I know I need to cut down on sugar but besides that not sure what to do? I only eat chicken and fish but I love baked goods. I do not eat butter, I eat smart Balance or Earth Balance. I have eaten an avacado almost everyday for the past two months. My ldl is 131. How can I lower that? I also drink a glass of red wine each night. I drink a lot of coffee-2-3 cups per day Doctor: Try with balanced diet and regular 30-40min daily exercise ( eg.brisk walking) & then check lipid profile again. If it is still deranged, then we can start medicines but I m sure diet plus exercise will help. And check blood sugar level once and get examined ur blood pressure level."
},
{
"id": 6376,
"tgt": "Is there any pregnancy chances after having ipill between two unprotected intercourse ?",
"src": "Patient: hello.. i had unprotected sex last night. i had an ipill after that. after six hours gap we had unprotected intercourse again. i wanna ask will d pill taken between both d intercourses work n i wud not get pregnant?? or will i have to take another i pill. m really worried. Doctor: Hello.Welcome to HCM forum.I pill has effect for 72 hrs and so even if you did unprotected sex after 6 hrs of taking I pill,you don't have to take another I pill after another unprotected sex.Thanks and good luck."
},
{
"id": 36390,
"tgt": "Does an insect bite cause chills, weakness and low grade fever?",
"src": "Patient: my mom had chills weakness , low grade fever 99.6 for 2 days , body soreness , itchy neck, swollen itchy hands - dr prescribed doxycycline as he suspected possible tick bite.. but hands are very swollen and itching has responded to benedryl 25 mg , but not the swelling does it appear to be a bug bite of some kine Doctor: Hello,I understand your concern.I am Dr. Arun Tank, infectious diseases specialist, answering your query.Yes it is possible with Lyme's disease and some other tick borne infection.The prescribed medicine is right and cover all this infection.I advice you to continue the drug.Also take precaution as to prevent future tick bite.Please use insecticide it will kill all remaining ticks and so future infection.I will be happy to answer your further concern, you can ask me on bit.ly/DrArun. Thank you.Dr Arun TankInfectious diseases specialist."
},
{
"id": 154974,
"tgt": "Does ovarian cyst, constipation and elevated wbc indicate cancer?",
"src": "Patient: Like i went to the emergency room a couple months ago for extreme 12 out of 10 abdomen pains, they said it was a ovarian syst and constipation. I dont know if thats the case though. I had X-ray s blood tests and scans done they said the white blood cell count was high but it went back down... Any chances of cancer? O.o Doctor: Hi,It would be more informative if you provide me with details like age exact reports any other symptoms like weight loss. EtcWith the information you have provided..Ovarian cyst is a benign condition..WBC count is raised in many Conditions like infection etc..(exact count is missing in rhe question)Pain from ovarian cyst is common..donot worry..if there were any chances of cancer doctors there would have advised further investigations like a pet scan or biopsy..Just follow your doctors orders and rest assured..Wish you a good health.."
},
{
"id": 115349,
"tgt": "What causes reduction in platelet count in blood?",
"src": "Patient: My father is 70 yrs old and a resident of Navi Mumbai. He didn\u2019t have a BP or Sugar problem. One month back suddenly early in the morning he couldn\u2019t get up, his body became heavy. He was not able to lift his body. We called up a doctor and she told his BP is 250 and he had a slight stroke and his left side was num and suggested a BP medicine, which we gave him immediately. She suggested us to take him to the hospital immediately. We shifted him to hospital where they did his city scan. The report says :- Focal hypodense ares in right frontal periventricular and corona radiate is probably suggestive of acute non hemorrhagic infarcts in view of clinical history. Mild generalized atrophy with periventricular ischemic changes. No evidence of bleed or intracranial space occupying lesion.Suggest: MRI of the brain for further evaluation.MRI has been done. While in the blood test report it was observed that the platelates count was reduced to 36000. Even the bone marrow test was done, the reports were normal. The blood test was done every alternative days in the hospital and before discharging his platelets count was 65000. The doctors team consisted of Hematologists and Neurologist. The doctors couldn\u2019t found the reason behind dropping platelets and after 15 days he was discharged from the hospital. Two days back we did his blood test and found that his platelets count dropped to 45000. We will be highly obliged if you kindly tell us what may be the reason for this and suggest further treatment for this? Email me on YYYY@YYYY Doctor: Hi, thanks for sharing your father's health concerns with HCM! If I were his treating Doctor for this case of chronic thrombocytopenia, I would suggest three possibilities in different stages: 1. reduced production: less number of megakaryocytes or inactive megakaryocytes in the bone marrow2. increased destruction like clotting here and there or splenic problems3. increased demand like medicines or virus or blood disorders consuming plateletsTherefore, assessment of bone marrow and drug review are necessary for confirmation/detection of the cause and to relieve your concerns!Hope this answers your question. If you have additional questions or follow up questions then please do not hesitate in writing to us. I will be happy to answer your questions. Wishing your father good health."
},
{
"id": 192982,
"tgt": "What causes testicular pain and groin pain?",
"src": "Patient: I am 23 years old. I started having testicular pain around Christmas '10 and on 12/31 went to the doctor and was diagnosed with a UTI and epididymitis. I took 10 days of cipro and got a lot better, but after a few days my symptoms began to come back. I called my doctor who gave me 5 more days of Cipro. I then had an unrelated surgery (gynecomastia) and took a week of Keflex and Percocet. When I started coming off the percocet I could feel the pain in my groin very acutely. My GP gave me 10 days of levaquin which helped but didn't seem to help as much as the first 10 days of Cipro. I have contacted a urologist but my appointment is not until the 25. Is it safe for me to wait that long? Should I try to find a different urologist or go to the emergency room in the meantime? Doctor: Hello,Conditions like epididymoorchitis must be ruled out. As of now you can use analgesics / anti-inflammatory combination like aceclofenac / seratiopeptdase for symptomatic relief. A short course of antibiotics must be considered. You can go for a scrotal Doppler to make a diagnosis.Hope I have answered your query. Let me know if I can assist you further. Regards, Dr. Shinas Hussain, General & Family Physician"
},
{
"id": 174277,
"tgt": "Is surgery required for a male child due to crying while passing urine?",
"src": "Patient: hi sir my kid is 16months from morning is crying while passing urine (boy). evening i took him to hospital doctor says he need surgery, but suddenly this many days he never had a problem. he is passing the urine fine but crying. plz advice us. ashok india Doctor: Hi,Thank you for asking question on health care magic.First of all consult urologist for thorough investigation and prompt diagnosis.Male children may have congenital webs in the posterior urethra.It may turn out to be simple urinary tract infection also.Surgery may or may not be required after final diagnosis. Hope this answer will serve your purposePlease feel free to ask any more queries if requiredTake careDr.M.V.Subrahmanyam MD;DCHAssociate professor of pediatrics"
},
{
"id": 32574,
"tgt": "Suggest permanent treatment for Lyme disease",
"src": "Patient: I was diagnosed with Lyme s Disease today after several years of going through many other tests and treatments. Now what? I have experienced fatigue and some of the other symptoms for 8-10 years. Do physicians recognize this and can it be successfully treated? Doctor: Hello,Welcome to health care magic,I think You may suffer from a chronic Lyme disease.Some patients after treatment with antibiotics have symptoms that last months to years. This condition is called Post-treatment Lyme disease syndrome (chronic Lyme disease) and can include fatigue, joint and muscle pain. The reason of chronic Lyme disease is unknown.In my opinion You should contact Your primary care doctor and let him or her check you. Treatment for chronic Lyme disease is often focused on reducing pain and discomfort cause by symptoms like:low-dose antidepressants,sleeping pills,exercise,stress reduction.Hope I have answered your query. Regards,"
},
{
"id": 83101,
"tgt": "Anal rash, blood in stools. ANA positive, blood work shows lupus. History of pseudopheochromocytoma. Treatment?",
"src": "Patient: My daughter has a red rash like anal ring around her anys. There was blood on her toilet paper after she wiped There was blood in the stool and it burns her just laying down. She is ANA positive but no one currently can figure out what she has She has been tested for just about everything but blood work never shows a positive lupus or other rare ones. We are off to mayo in mid July but she can go out in the sun from 10-4 she can t handle heat and has been diagnosed with pseudopheochromocytoma ? What can I do to help her she is only 5! Doctor: Hello Dimokry At the outset I can assure that you are not alone. There are several dedicated doctors who would like to help you. When we do not know what is exactly wrong with our child we need to think from 7 angles 1. What is the problem? 2. Why it would have come? Hereditary/ infection / allergy 3. What are the tests required? Painful and painless tests. Scans. We have done several investigations. Do they need to be repeated again and again 4. What are the drugs child require? Should they have to taken daily like diabetic drugs or as and when required, like acetaminophen? 5. What are special diet the child will require? 6. Apart from drugs and diet what are precaution one has to take at home? 7. Lastly, how will be her future? Should we do anything to-day to safeguard her future. Now let us one at a time. ANA positive does not mean anything serious. It just suggests your child MAY require further investigations. If you have done them, be reassured. Regarding ? pseudopheochromocytoma: you have not mentioned about Documented hypertension Documented tachycardia ( fast hear rate ) Abdomen scan ruling out adrenal tumours Importantly response to adrenergic blocking drugs This would be helpful. ?pseudopheochromocytoma is not diagnosed just like that. please add more details. Regarding blood in the stools can occur in several conditions. Anus redness is can happen in several conditions ranging from milk allergy to dust allergy (atopic disorders). Was there any frank bleeding from rectum? Has she had any rashes in her skin. there is another way to proceed. Check her weight ( atleast 16 to 18 kg) Height ( 99 to 105 cms) Head ( more than 47cm ) Chest ( more than the head ) Mid arm circumference ( 15 cms) If her growth is in this range means there could be nothing serious. Regarding exhaustion, what was her hemoglobin level. That could explain. Give her small frequent fluid diet. Be with her. Parents warmth can do wonders. Regularly post the progress. Before going to mayo, try to document her course in the last 8 weeks. Problems, duration, drugs given, response to drugs Regarding the tests you have done classify them as 1. Basic tests 2. Biochemical tests 3. Imaging studies. If you have any problem, post in the public forum.we will happy to help you. with warm regards arulalan"
},
{
"id": 175487,
"tgt": "How to treat an eczema in a 5 month old baby?",
"src": "Patient: I took my 5 month old baby to the doctors today for a follow up appointment concerning eczema. I have recently noticed bumps on his neck (at the base of his skull). The Pedi. said this is due to the eczema. I have been trying to research the correlation but I am not coming up with anything. My question to you is what is the correlation? Doctor: Hi...eczema while irritating the skin will cause some irritation in the immune system and this is the reason for the bumps which you are noticing. Actually they are lymph nodes which are enlarged. Nothing to worry about. Regards - Dr. Sumanth"
},
{
"id": 74231,
"tgt": "Suggest medication for breathlessness and cough while having asthma",
"src": "Patient: I suffer with asthma but it s usually fine and I virtually never use my pump but with the winter months olive had to use it a lot. I used to smoke but have quite for sometime now. I have woken from my sleep a few times now with an incredible shortness of breathe and coughing. I feel like there is stuff stuck in my lungs and usually once I ve had my preventer inhaler I am able to clear a small amount. It feels like my lungs have filled up with something and I m really concerned as I ve been a heavy smoker and I work around some materials hazardous to lungs. (Types of dust, asbestos etc) Doctor: Thanks for your question on Healthcare Magic. I can understand your concern. In my opinion, you should take preventer inhaler regularly. What you are facing right now is acute exacerbation of asthma. If you use preventer drugs regularly, acute attacks can be stopped. So better to consult pulmonologist and get done PFT(Pulmonary Function Test). PFT will tell you how severe is your asthma. Based on severity, you will need inhaled bronchodilators (formoterol or salmeterol) and inhaled corticosteroid (ICS) (budesonide or fluticasone). Oral mucolytic and expectorant, N acetyl cystine (NAC) is also recommended in your case. So consult pulmonologist and take your preventer inhaler regularly. Hope I have solved your query. I will be happy to help you further. Wish you good health. Thanks."
},
{
"id": 161581,
"tgt": "What causes infection on big toes in kids?",
"src": "Patient: My baby started with an infection on one big toe, I even thought it was a ingrown toe nail but it s not. Her toe is just swollen and red and the skin is coming off, just ugly. Now, her other big toe is starting to do the same thing. What could it be and what can I do? Doctor: Hello, The main causes of the toe infection are ingrown toenails. If it is not ingrown nail then it can be an infection from a wound or an insect bite. He may need to use antibiotics orally such amoxicillin. Also, he can apply ICTIOL ointment over it. Hope I have answered your query. Let me know if I can assist you further. Regards, Dr. Olgeta Xhufka, General & Family Physician"
},
{
"id": 113677,
"tgt": "Back pain, low bone density. Taking paracetamol. How can the back pain issue be solved?",
"src": "Patient: 32 years old man, i have heavy pain in my back,i consult so many doctors and i also ate so many tablets but no use still the pain is continue so what solution to me..pls help me doctor? i want your advice must... [i was test in my bone density that s level is low and i follow PARACETARNOL 450mg every day 6 tablets] Doctor: It is advised to get done MRI scan of the region of back(you have not mentioned). Take tab tramadol everyday 1 after dinner till you get scan done. For osteoporosis, you take alandronate tab 10 mg one tab every week."
},
{
"id": 115884,
"tgt": "What causes the lowering of the WBC levels?",
"src": "Patient: My mom is in icu. She is intebated, She is 85 and weighs approx 205. She has COPD Shes been in the hospital for 4 weeks this visit. The reason we took her to the hospital is she had a fever. Her white blood cells were very low and the doctors told us her immune system is comprimised. So she went to reverse isolation in the hospital where the dr's tried to locate the infection and gave her iv antibiotics ,and meds to raise her white blood cell count. The infection that is somewhere in her body as yet the doctors can't locate it, She is still and has been on IV anitbiotics for many weeks. Her fever went down and her white blood cell count improved. So she was moved to another part of the hospital. Then after a few days she began having trouble breathing and her heart rate rose dangerously high so they intebated her. They also told us she was Septic/sepsis. So again to ICU. Today will be the 12th day and they are going todo a trach today. Also her kidney functions arent good. Now I'm to learn her arms are weeping. So I'm thinking CHF. My sister-inlaw believes she'll pull out of it. I'm not so sure. any words of wisdom would be appreciated. Doctor: hopefully she is getting antibiotics according to Blood culture and sensitivity...Also i want to add...pt. with sepsis (?BP?URINE OUTPUT ???) having AKI with swelling...intubated twice with age of 83..mortality rates are very high...IF her BP allow Dialysis can be a option..but every decision will be made only by doctor on duty...Also her TLC is not of big concern now.."
},
{
"id": 31531,
"tgt": "Suggest treatment for bacterial vaginosis",
"src": "Patient: hi i have suffered with bv ever since i had an abortion earlier this year. i have been treated for it and it went away but now i am on my period and i think i can smell it coming back! me and my partner did have sex while i was on my period could this be the cause and have i got bv again? Doctor: Hi thanks for asking question.According to history given , false smell related to vaginosis.Yes.bacterial vaginosis can spread through sexual route and according to history here possibilities of vaginosis present.Take metronidazole or tinnidazole 1 bd for 7 day.Alternately vaginal cream or gel containing clindamycin can be applied for 5 day.Avoid stress.bacterial vaginosis mostly lead to no any serious side effect.so don't worry.Thanks.I hope I have solved your query."
},
{
"id": 204522,
"tgt": "How can severe anxiety be treated?",
"src": "Patient: My daughter (19, college freshman) takes Adderrall during the day. But she is also extremely anxious and agitated, like a basket case all the time. The psychiatrist prescribed Zoloft. My daughter took it the other evening, and did not sleep all night and could hardly function the next day because she was so tired. Does this mean that Zoloft has the opposite effect on her? It does not relax her? What are other possible solutions? Doctor: Hello and Welcome to \u2018Ask A Doctor\u2019 service. I have reviewed your query and here is my advice. Zoloft contains Sertraline, which is a antidepressant. It can be used for a number of psychiatric illnesses. Zoloft can cause insomnia in some patient where it acts as a activating agent. Even Adderall is a activating agent and causes insomnia. You can shift Zoloft in day time and observe if it helps; otherwise there are medication available which can be used for short duration to counteract the side effects of drugs. Hope I have answered your query. Let me know if I can assist you further."
},
{
"id": 120675,
"tgt": "What causes pop and swelling under the left collar bone?",
"src": "Patient: Friday evening I was in the kitchen, not really doing anything, and I felt like a pop under my left colar bone about dead center of the bone. My daughter said that it has stayed slightly swollen half way up the neck all weekend. Lifting a cup of coffee to drink it is very makes it very sore.Shoulder blade opposite side is sore also. Neck a little hard to turn. Doctor: Hello,I read carefully your query and understand your concern. Sometimes moving your shoulder can trigger a clicking sound or a\u00a0popping\u00a0sensation near where the joint connects at the top of your arm.That\u00a0popping\u00a0feeling is called crepitus. In some cases, there's a sharp pain or warmth that comes along with a popping\u00a0shoulder.I suggest using anti inflammatory medications such as Acetaminophen to relieve the pain.I also suggest using cold compresses to reduce the inflammation. Hope my answer was helpful.If you have further queries feel free to contact me again.Kind regards! Dr.Dorina Gurabardhi General &Family Physician"
},
{
"id": 130802,
"tgt": "Can bowel cancer be the only reason for a pain in the tailbone?",
"src": "Patient: In my 20s, I had been pushed very hard on to the wooden part of the arm (sofa) and had landed on my bottom. I had immediately after had experienced great pain, which I knew was my tail bone. I then went to the hospital due the pain and was unable to walk very well but was told at the Hospital that it was possible bruising and was given ibrophen tablets, eventually got better after a few weeks with plenty of rest and ibrophen tablets. However, Many years later in 1998. I had given birth to my son and had experiensed the same pain in the same area (coccyx ) and heard a poping sound in my lower back and felt pain afterwards. I was yet again in a lot of pain. I had found it difficult to look after my son. I had home help within my home; once a day two nurses had inserted pain killers up my bottom end due to the wicked pain I had experienced. I also had physiotherapy but can't remember how many times a week this was done at my home; Eventually this had also got better but unfortunately felt the same pain in my back and the tail bone, when I had my periods. In 2009 had gone for an examination for bowel cancer. Before I had gone to the hospital for the examination was sent some very powerful laxatives, to empty my bowels. I had then yet again experienced the same servere pain in my tailbone but this time the pain has never gone away since. I have had MRI scans but the results came back as clear but still convinced that i have injuried my tailbone and lower back. It's difficult to keep still in the MRI scanes for a long length of time and wounded if this effected the results. Doctor: Hi i am Dr Ahmed Aly thanks for using helthcaremagic site , It does have a few important jobs. It helps to stabilize you when you sit, and many tendons, muscles, and ligaments run through the area.I dont think that could affect the results because MRI scans are less affected by minor moves and if so the MRI radiologist or your physician would have noticed it at once The pain should go away in a few weeks, or sometimes months. You can try over-the-counter nonsteroidal anti-inflammatory drugs (NSAIDs) to relieve discomfort until your tailbone heals. These drugs include ibuprofen (Motrin, Advil) or naproxen (Naprosyn). Acetaminophen (Tylenol) also can relieve pain. For more severe pain, your doctor can inject a local anesthetic, nerve block, or steroid medicine into the area. Some people get a combination of anesthetic and steroid injections. You can also take an antidepressant or anti-seizure medicine by mouth to ease the pain. Be sure to discuss your treatment options with your doctor. To ease discomfort, sit on a heating pad or ice pack, or go for a massage. The way you sit also matters. Poor posture can put too much pressure on your coccyx. Sit with your back against the chair and your feet flat on the floor to take the weight off your tailbone. Lean forward when you go to sit down. You can also sit on a special donut-shaped pillow or wedge-shaped cushion to relieve pressure on that sensitive area.A physical therapist can show you exercises to strengthen the muscles that support your tailbone. These include your stomach muscles and pelvic floor. You can also try a technique called coccygeal manipulation. This is when a doctor inserts a gloved finger in your rectum and moves the tailbone back and forth to shift it back into position.Most of the time, these treatments will relieve your pain until your tailbone heals. If no treatment has worked, your doctor might recommend surgery as a last resort to remove part of or the entire coccyx. This procedure is called a coccygectomy. Surgery doesn\u2019t always work right away. It can take time before the pain goes away. In some cases, it doesn\u2019t work at all. Surgery can also carry risks, like infection. It\u2019s a decision that you need to make very carefully with your doctor.Start with home pain-relief measures like NSAIDs, heat, and massage. If your tailbone still hurts, check in with your doctor, who can help you find a treatment that works for you Hope the above information helps you,Thanks for writing into healthcaremagic. .healthcaremagic site doctors will be ready to assist you whenever needed ."
},
{
"id": 45124,
"tgt": "Bromergon 2.5mg and ova-mit tablets are the right prescription for polycystic ovaries ?",
"src": "Patient: Hi,i have polycystic ovaries,and my doctor prescibed bromergon 2.5mg tablets and ova-mit tablets for me,will this help in making me ovulate and make my periods regular,i really want to know,i really want to get pregnant soonest,thanks. Doctor: hello, welcome to HCM thanks for the query your doctor has given you right medications. continue to take them simultaneously try to reduce your weight by diet modification and regular exercise. weight reduction will decrease insulin resistance and chances of pregnancy will increase wish you a good health and take care so continue it ."
},
{
"id": 181613,
"tgt": "Is this normal of having dangling piece of gum after tooth removal?",
"src": "Patient: I had tooth 14 pulled yesterday afternoon and am getting an implant. The oral surgeon did not use stitches instead he put something to preserve the bone. Today I noticed a dangling piece of gum it is soft and gel like. I called the office and said that is normal and to bite on gauze to get it back in place. But everytime i drink or rinse with salt water it comes back out. Is this normal will it shrink in size? Doctor: I think you need not worry..it may be some gelfoam like material if the dentist has used that to preserve the bone.do not get bothered about it..use betadine mouthwash and warm saline garles fo keep the hygiene clean in that area..sometimes it can be a fibrous clot formation also which tends to dissolve wifh time"
},
{
"id": 195172,
"tgt": "Does a slipped disc cause pain and numbness in the penis and scrotum?",
"src": "Patient: Hello doc. I am 26 year old guy, I have detected with L5S1 slip disc, there is a pain in my groin and numbness in penis and scrotum, I am a bit worried about it, i want your consultation whether this is one of the symptoms of slip disc, and how this will get cured.? Please advice me Thanks Doctor: Hi, Slip disc at L5 S! will not cause severe numbness in the penis and scrotum. I suggest you get it checked by a neurologist and evaluate the nerve root compression. If severe compression surgery can solve your problem. Hope I have answered your query. Let me know if I can assist you further."
},
{
"id": 33800,
"tgt": "What causes muscle and joint pains and swollen tonsil?",
"src": "Patient: My age is 40 yrs, 4.9ft. I started treatment 10 weeks ago for abnormal BP with 1 tab Hydrochlorothiazide and2 tab Methyldopa. Then 3 days later the Dr introduce Bromazepam 1.5mg. The night i took the drugs, I develop flu like symptom of sore throat and swollen thronsil, muscle and joint pains, high fever, slight coug h and tiny looking rash all over my body except my head and neck region. I became scare that my girlfriend must have infect me with HIV. Two week ago she did a handjob on my penis and it really hurt me, but didn't see wound. I then had protected sex with her but the condom could not reach an open boil i had at the boundary between my penis and soctrum. Is this allergic reaction or has she infected me with HIV, because i presume she is HIV positive with all the symptoms I see in her. Doctor please I have not been my self ever since. It like I am beginning to get insane. Just 2 weeks ago I had an open sore on my penis. Please help me. Doctor: Thank you for your contact to Health Care Magic, i strongly recommend you to visit AIDS centre, give AIDS test and start HIV prophylaxis, you don't be upset it can happen with anybody, but prophylaxis is very effective, you just start to drink medicines,which HIV specialist or infectionist will prescribe you. I think you have usual viral infection. I suggest-1. Paracetamol, allegra.2. Rince gurgle with chlorhexidine.3.Hot drinks.4.Lozenges-strepsils 1 tablets 4 times or sualin5.Avoid sour cold drinks.6.Don't take risk more You know some people are not sensitive to HIV, but better do prophylaxis. Take care! Regards Dr.Sagar Gaurav Shrivastva"
},
{
"id": 149078,
"tgt": "Have herniated disc compressing S1 nerve root. Likelihood of surgical intervention? Have leg weakness, numbness",
"src": "Patient: What is the likelihood of surgical intervention for a L5-S1 herniated disc compressing the S1 nerve root in the lateral recess and obliterating the fat below the exiting L5> This has caused significant left leg/foot weakness and inpaired/decreased dorsiflexion and foot inversion, pain, impaired ambulation and numbness/tingling in left leg/foot and burning pain in inguinal area.This motor impairment and pain have not significantly responded to 5 day Prednisone tx. 60 mp PO daily. Doctor: SURGICAL INTERVENTION IS A MUST AS PER UR HISTORY AS THERE IS ALREADY WEAKNESS CORRELATED WITH THE LEVEL OF THE COMPRESSION."
},
{
"id": 49713,
"tgt": "Done dialysis for renal failure. Had abnormal CPK. Due to mcardles disease?",
"src": "Patient: My 19 year old daughter was hospitalized with cpk 350,000 resulting in renal failure in which she required dialysis 3 times. Her kidneys began to function again. We are awaiting to see a specialist in a neuromuscular group. Awaiting muscle biopsy for diagnosis. She recently after 2 mo from onset of it all has cpk of 298 and normal kidney function test results . In diseases such as mcardles disease does the cpk level fluctuate or does it normally remain elevated. Doctor: Hi there,Thank you for your question.Raised CPK can have many causes, these include injury to the muscles, high fever, inflammation of the muscles called as myositis, McArdle's disease also causes high CPK levels.In any condition the levels of CPK can fluctuate and since she was dialysed so the CPK must have been removed from the blood via dialysis so that could be the reason why it is not raised anymore.Such a high CPK does require further testing such as a muscle biopsy etc to determine exaclty whats causing it and then to treat the condition accordingly.Kindest Regards"
},
{
"id": 50275,
"tgt": "Distended urinary bladder without stones, kidney dilated. Diagnosed with mild pelvocaliectasia. Treatment?",
"src": "Patient: The right kidney measure about 9.8cm x 5cm with a cortical thickness of 1cm exhibiting slight dilatation of the pelvocalyceal system. No mass nor stones noted. The left kidney measure about 10.5cm x 5cm with a cortical thinkness of 1cm exhibiting intact corticomedullary junctions and dense central echo complexes. No mass, stones nor hydronephrotic changes seen. Cortices are normal in thickness and echogenicity. Urinary bladder is physicalogically distended without stones. The wall is not thickened Postvoid scan shows minimal residual urine of about 22cc. IMPRESSION Mild pelvocaliectasia right. Doctor: I do not feel this to be a significant finding.However , i would like you to get a follow up ultrasound both with full/post-void urinary bladder to rule out any PUJ anomaly."
},
{
"id": 15568,
"tgt": "Bumpy red rash on abdominal area, back after starting HCG diet, spreading on and under breasts. Should I stop the diet?",
"src": "Patient: After I started the HCG diet, at the third day I started getting bumpy, red rash on my abdominal area, then the lower and middle of my back, few days after that the rash spread under my breasts and there are even little red dots in random places on my breasts. Now the back of my neck and groin region. The rash are very red in color but no pain at all, sometimes itchy, most of the time just does look good because they are very red.Should I stop the HCG diet?I do not have any history of psoriasis, allergic or any other rashes in the past. Thank you. Doctor: HI.STOP THE HCG DIET FOR TWO DAYS AND OBSERVE.IF THE RASH DOESNT DISAPPEAR IT CAN BE DUE TO OTHER REASON.TAKA A ANTIHISTAMINE DAILY"
},
{
"id": 126623,
"tgt": "What causes knee pain despite a normal X-ray report?",
"src": "Patient: I have a pain on my left knee, i have done x-ray and the results show notting wrong with the knee but the hospital said i need to see an orthopedic surgeon. but if i sit down for too long it swells, at nite my temperature rises and i have loss of appetite. Please what next can i do Doctor: Hi, It might be due to some ligament or tendon related problem which may not be seen in X-ray. You can consult an orthopedician and plan for an MRI scan which is a better imaging modality and it is superior to X-ray. As of now you can take analgesics like Acetaminophen or Tramadol for pain. Hope I have answered your query. Let me know if I can assist you further. Regards, Dr. Shinas Hussain, General & Family Physician"
},
{
"id": 79250,
"tgt": "Can i take medicine for treating cough and wheezing?",
"src": "Patient: hi... i take telekast l on and off as suggested by my doctor as i easily suffer from cough and wheezing. whenever i start having these symptoms, i start taking the medicine, then stop after it gets better, then start again when the symptom starts. is there any harm in the way i am taking the medicine? i had done asthma test and was negative Doctor: thanks for your questionmonteleukast and levocetrizine should be taken as prescribed by your physicianyou can consult a Pulmonologist , he ll guide you bestif you are taking it intermittently I don't think it's going to do any harmtheoretically it should be taken as per course and regularly and some studies says that it can be taken up for months regularly without any side effectsthanksfeel free to ask more questions"
},
{
"id": 179196,
"tgt": "What causes a bump near the anus in a child?",
"src": "Patient: Hi my 6 month old has a bump on her bottom close to her butt hole (sorry). Its not colored it looks almost like a mole but is not dark brown. Its normal skin color. I just noticed it while putting butt paste on her a little bit ago. Should I be concerned? It doesn t seem to bother her when I touch it Doctor: Its anal wart. You dont need to bother about it. It will not cause any problem. Just notice if its increasing in size or number. If ever it changes in colour or become painful consult pediatrician."
},
{
"id": 160271,
"tgt": "Need Information of treatments for cancer",
"src": "Patient: Hi. What is the process of detoxification of the body in a cancer patient after Chemo / Radiology? Doctor: Meet: Ty Bollinger at http://cancertruth.net/ His advice in his book on the above website helped me overcome colon cancer grade 3. Cancer is a world which has not been fully reseached yet. Cancer affects the body the mind and spirit, all three need attention when dealing with cancer. From the date of me being told I had grade 3 colon cancer (during an operation to remove my painful appendix) to the date of the latest MRI, six months later, my doctor read the MRI report to me. YOU ARE CLEAN, so sign of metastases in your body, from head to toe! I refused Chemo having read Ty's book. Order the book, should you so wish, email Ty, he helped me in my darkest hour. Suffering cancer entails deep emotions and getting through one's darkest hour is not easy. The problem with most cases is that the first sign of being afflicted with cancer takes place too late when tumours have already formed with damage. Early detection is a key element and keeping ones immunse system strong by a healthy vegetarian diet, lots of fruit and veggies, etc. and reducing stress, goes a long way to retaining one's health. Egon Kramer email: koisan12@aol.com"
},
{
"id": 123952,
"tgt": "What causes sore, tight & tired legs?",
"src": "Patient: Yes, thank you, for about the last week or so, when i walk, my legs feel sore, tight & tired. I m always very active and exercise but I did stop working out about 2 mos. ago due to my schedule but I still exercise just not at the gym. Also last weekend I got really sick at the lake with motion sickness and as of today I still feel a bit queasy. My major concern are my legs, they just feel tight, like I need to stretch them out before I walk. Yesterday I mowed my lawn and I had to literally push myself to complete the front lawn. This has never happened before, i m about 15 lbs overweight but this has never been a problem for me before, plus I do have a bit of vertigo. . Doctor: Hello, Do some gentle stretching before your walk/movements. Try to reduce Protein, Carbs and Fats intake, so that you don't gain weight. Check Vitamin D & Serum Uric Acid levels. Hope I have answered your query. Let me know if I can assist you further. Take care Regards, Dr Saurabh Singh Rajan, Sports Medicine Specialist"
},
{
"id": 107093,
"tgt": "What causes recurrent lower back pain?",
"src": "Patient: Hi I have had this lower back pain for a while now. I went to a back surgeon about year ago and he said I wish I had your back thus he saw no problems. However I did go through physical therapy and excersies and pain went away. But it is back. I thought it might be a sidiac nerve but I only have back pain no leg pain. Doctor: dear patient you have not mentioned your age. reasons for back pain a vary depending on age of patient. if you are young age and having pain for a year only in back most probable reason is mechanical which means lower back instability or muscle weakness. is tour pain relieved after rest? if yes then It suggests mechanical problem. please wear lumbo sacral belt for 4 weeks and take tablet zerodol SP twice a day for 7 days. I would also advise back muscle strengthen exercises."
},
{
"id": 177874,
"tgt": "Which stage of Nan pro can be given to 6 month old infant?",
"src": "Patient: hello doctor, i am breastfeeding my baby he completed 6 months. i have already introduced supplementary foods for him.Now i planning to add nan pro . Which one should i give nanpro 1 or 2. i am already feeding him two meals per day(potato, apple carrot , rice cereals , banana) and inbetween breast milk. so if i am adding this nan pro to him then how many times and how much should i feed. pls help doctor) Doctor: NAN Pro 2 is intended for children beyond 6 months of age as in your case. But rather than giving such formula milk, breast milk is the best alternative. The frequency of feeds should be 6-8 times a day, be it formula feed, breast milk or semi solids. The amount should be a few ounces initially and gradually increased to 8-10 ounces per feed or as the child tolerates. I would suggest you to continue to breast feed the baby upto at least 1 year of age along with the supplemental feeds that you have introduced. Gradually increase the frequency of the semi solids to 6-8 times a day. Cow milk is optional and not essential in case breast milk is inadequate."
},
{
"id": 99943,
"tgt": "Why am i feeling weak after taking allergy tablets?",
"src": "Patient: I have been suffering throat and ear itching for last two weeks and the doctor gave me allergi tab and after take that tab i faced many problem my heart beat got faster and feeling cold after 4 days my health is got down and i m feeling very weakness and depressed what shoud i do? Doctor: Hello. I just read through your question.The first thing to do is stop the medication. Then, I recommend consulting with your doctor so an alternative treatment place for your allergy can be discussed"
},
{
"id": 223333,
"tgt": "Does IUD cause profuse bleeding?",
"src": "Patient: I am 3 months postpartum from twins and had an iud (Mirena) placed at 6 weeks i have not stopped bleeding since the birth and the bleeding is very different from pre pregnancy menses its mucousy discharge and blood is always a different color and it never stops i am very frustrated and ready to give up on the iud. Doctor: hello user,yes Iud can cause profuse bleeding.you can consult your gynecologist and choose an alternative contraception,if its bothering you much.thanks"
},
{
"id": 182610,
"tgt": "How to treat calculus in teeth?",
"src": "Patient: Im 23 years old and my bottom set of teeth seem to be rotting around the gumline and I m not sure if I have calculus. Today I had one tooth rott off at the bottom around the gumline. I brush my teeth regularly about 2-3 times a day so i dont understand why this is happening to me. I need to find a Dentist in my area that will help me out because my health plan doesn t cover dental and honestly I m living paycheck to paycheck as it is but willing to make payments. In the meantime though I m afraid to eat because I don t want anything stuck in there where it rotted. Doctor: Thanks for your query, I have gone through your query.The deposits on the teeth can be calculus, or it can be decayed tooth also. Nothing to be panic, consult a oral physician and get the teeth cleaned.The most common causes for food lodgement even after brushing teeth can be your improper technique, mal aligned teeth, gap between teeth. So rule out all these things and get scaling and root planing done. Avoid in between eating, sweets before going to bed. Rinse your mouth soon after eating anything, you can use dental floss. Regarding your financial aspect, go to a educational dental institute and get it done, there they will do free of cost or with nominal price.I hope my answer will help you, take care."
},
{
"id": 26846,
"tgt": "How long should i take BP medicines?",
"src": "Patient: I have high blood pressure (since being on 300mg. effexor). I suggested to the doctor that perhaps I should go on bet.a blockers as I do have problems with anxiety. Could I have started on half a tablet to see if that was sufficient? How long does it take to come off of them? Doctor: Hello!Thank you for asking on HCM!Your high blood pressure may be related to effexor side effects. This is a very common side effect of effexor treatment. I would recommend consulting with your attending physician to consider the possibility to switch into another antidepressant drug. If this is not possible, an antihypertensive therapy like a betablocker or an ACE inhibitor is necessary to control your blood pressure.Before starting an antihypertensive therapy you should perform some tests like :a resting ECG, chest x ray, a complete blood count, kidney and liver function tests, blood electrolytes, thyroid hormones to exclude other possible causes of high blood pressure. Anti hypertensive therapy should be continued as long as you are taking effexor and no other causes of high blood pressure are found in the above exams. Hope to have been helpful!Best wishes, Dr. Iliri"
},
{
"id": 64328,
"tgt": "Suggest treatment lumps on ear",
"src": "Patient: Hi I'm 28 years old 5 foot 7 inches tall weigh about 125 and am female I have a little bit of acne nothing horrible tho and I have a red lump that is about the size of an almond in front of my left ear (kind of between the top of my jawbone & ear). It is a little mushy feeling when i push on it and kind of rolls when i push up or down on it. I poked it w/ an alcohol cleaned needle, blood and this oil like stuff comes out when I sqeezed it. It isn't really painful. What is this & should I be concerned? Doctor: Hi I am Fahim and I will help you with your problem I have gone through your question. I appears to be a sebaceous cyst or lipoma. Both can be left as such if they are not very big and not bothering you much. Surgical excision is the treatment, in which the lump is removed and skin closed the non-absorb-able sutures. The wound heals in about 5 days time and stitches removed on 5th day. You need to see a general surgeon, who after clinical examination will make a diagnosis and then will guide you appropriately.I hope my answer will help you. Do rate it, if you like it.Regards"
},
{
"id": 164988,
"tgt": "What causes redness and swelling on penile head in a child?",
"src": "Patient: My 16 month old son is not circumsized. When he was born I asked his pediatrician how to care for his penis. He told me never to retract his skin and just wash. When he was four months old I took him to the ER because he had a fever for days that was not going down. They inserted a catheter and accussed me of not taking care of his penis. His foreskin had never been retracted until that point because of the advice I had received. They showed me the head of his penis and it had like a white build up. So they instructed me to pull his foreskin back and clean it every time he has a bath. His penis always seems to bother him . Tonight he has been crying for hours, but I couldn t figure out what was wrong. I finally checked his penis and the head looks really red and the opening looks swollen. My husband is now at the ER with him. But, I feel very skeptical as to what they might do (I feel that they have caused the problem with his penis 1. because of inserting a catheter 2. because of retracting his foreskin at 4 mo old). Please help. Doctor: Thanks for asking.The fault neither lies in you nor your doctors.In my opinion it's the uncircumcised penis which is the culprit.Studies ha proven that uncircumcised penis can lead to recurrent infections.So I would advise a therapeutic circumcision once this infection of penis is over.Good luck."
},
{
"id": 121678,
"tgt": "Should an orthpaedic surgeon be able to diagnose labral tear with bicep rupture during arthoscopic surgery?",
"src": "Patient: Should an orthpaedic surgeon be able to see there is a labral tear with bicep rupture while he is doing arthoscopic surgery. Was having subcromial decompression, but pain never let up after surgery, had surgery six months (exactly) from first surgery. New doctor found SLAP II tear, had surgery and pain has cleared within three months. Doctor: Hello,Labral tear can be easily seen during arthroscopy. An MRI can also detect the same lesion.Hope I have answered your query. Let me know if I can assist you further. Regards, Dr. Shinas Hussain, General & Family Physician"
},
{
"id": 6027,
"tgt": "Have PCOD. Taking dupheston, folinex, bigomet. Negative urine pregnancy test. What are the ways to get pregnant?",
"src": "Patient: hello dr, i hv pcod . & had starte taking dupheston from 27 of may & stopped on 6 of june. I m continuously taking folinex, & bigomet sr 500 mg daily. we have intercourse only for 3 days during my fertility period (as suggested by our dr.) according to my dr. I should have period on 10 of june. but now it is 18 .2 urine test conducted at home are negative & bhcg test report is .6 .It means that i m not pregnant. we wnt child & triying fr it. I hv never used any contraceptive . kindly tell me that what may be the reasons for this delayed period after taking dupheston. please... Doctor: Hello. Thanks for writing to us. The cause of the delayed is the polycystic ovaries. If there is no bleeding even in the absence of pregnancy then you need to take progesterones for a withdrawal bleeding. Then start with ovulation inducers in next cycle. I hope this information has been both informative and helpful for you. Regards, Dr. Rakhi Tayal drrakhitayal@gmail.com"
},
{
"id": 125374,
"tgt": "How to treat shoulder blade pain?",
"src": "Patient: I was getting ready to get out of bed this am,when beginning to stretch,a very intence sharp pain in the lower right shoulder blade grabing type,also felt like I could not breathe ,took a muscle relaxer,and have been applying ice still hurts to high heaven what else can I do Doctor: Hello, As a first line management, you can take analgesics like paracetamol or aceclofenac for pain relief. If symptoms persist better to consult an orthopaedician and plan for an MRI scan to look for ligament or tendon involvement. Hope I have answered your query. Let me know if I can assist you further. Regards, Dr. Shinas Hussain, General & Family Physician"
},
{
"id": 35763,
"tgt": "How to treat UTI?",
"src": "Patient: I was just wondering, I have been on Loestrin 24 FE for about two almost three years now, unfortunately the other day I developed a UTI and was put on antibiotics. I know it effects my pills, but to what extent and if anything arises would it be problematic to attempt intercourse? Doctor: Thanks for your query at HCM!I am Infectious Disease Specialist! I went through your query!Loestrin 24 Fe you are taking for contraception to prevent pregnancy. It must be stopped if major kidney problems.Yes antibiotics do effect the efficacy of your pills so as to pose a threat of pregnancy.You may use back up birth control like condoms to protect your partner from infection as well as to avoid pregnancy till you are on antibiotics. Happy to take more queries! You can also write a review for me. If you would like some more information, I will be happy to provide. You can take a follow-up query.Take care!Dr. Sheetal VermaInfectious Disease Specialist"
},
{
"id": 215732,
"tgt": "What causes severe pain in the right hip and leg despite taking Tramadol?",
"src": "Patient: I am away from my current home having heavy pain in my right hip and leg. I don t want to go to the ER and I live in Kansas City but I am in Denver with relatives. I have surmised that it is sciatica. I applied heat and it didn t help. I think I need an anti inflammatory drug. I take tramadol regularly but it is not cutting it. What is an OC anti inflamatory? I am a female, 68yurs old and weigh 136. Doctor: Hi, I'm afraid you are having an acute sciatica pain episode. And yes, Tramadol alone can't help you. As you cannot go to the ER, then, I advise to: - apply an analgesic cream on your lower back and on right hip and leg joints - take over the counter Paracetamol alternating with Ibuprofen or Ketoprofen - some sciatic exercise can help you a lot to manage the pain - I think heat won't help but make inflammation worse, should apply cold compresses or Glucosamine packs (ask at your local drug store) - take Glucosamine and vitamin B complex supplement. Hope I have answered your query. Let me know if I can assist you further. Regards, Dr. Albana Sejdini, General & Family Physician"
},
{
"id": 204226,
"tgt": "How can severe anger be managed?",
"src": "Patient: Mother gets angry for small reason or she thinks something went wrong and against her. Episode last for few days she bad mouth ppl,thinks her son is against her, gets angry, doesn t it, gets angry I try to calm her every minute but she does not listen. After few days when everything gets end and she becomes normal she thinks she has not said or behave wrongly. It hurts as I, son looks after her and takes care of her every minute ask her to stay calm n all ppl around her are good but she doesn t listen and starts complain again. If something goes against her will she gets panic , not sure what to do . She does not even eat properly which I borrow from hotel or prepare by person whom she gets angry Doctor: Hello and Welcome to \u2018Ask A Doctor\u2019 service. I have reviewed your query and here is my advice. These symptoms are suggestive of personality issues. With time it starts affecting social and working life. She might be impulsive and do things which she even don't mean to. These symptoms can be taken care off by psychological therapy and medications. Every illness brings stress to family. Do not worry once she will be fine she will not behave like this. Best Wishes. Hope I have answered your query. Let me know if I can assist you further."
},
{
"id": 8802,
"tgt": "Can I use melacare cream for under eye dark circles during pregnancy ?",
"src": "Patient: dear sir i used to apply melacare cream on my face to get rid of the pigments under my eyes. now am pregnant for two months. am afraid now there may have any side effects in this cream. please give your suggestions. with regards fatema india Doctor: Hi..dear.., Thanks for choosing HCM., Don't worry about topical applications.., It won't give any problem.., But it will enter systemic...(In to the blood..only 1%)., eventhough , it is very minimal and can not give.., desired side effects..ok..., So avoid oral intake..., don't tense...relax...ok...good luck..,"
},
{
"id": 195741,
"tgt": "Suggest treatment for pain in the scrotum",
"src": "Patient: My husband is having a lot of pain to his scrotum, and the left side is extremely swollen, it feels like there is a cyst or something on the testicle on left side, the right side is actually causing more pain than the left but I can t tell that there is anything wrong with it. Doctor: Hello and Welcome to \u2018Ask A Doctor\u2019 service. I have reviewed your query and here is my advice. It may be epididymo-orchitis. Kindly consult the surgeon and get investigated properly and then perfect treatment may be given. Antibiotic and anti inflammatory drugs may be given for few weeks. Hope this helps."
},
{
"id": 136226,
"tgt": "Suggest shot to reduce swelling in stomach, feet and leg",
"src": "Patient: HI, my name is Maria. I have been despirately looking for a drug that can be giving by way of a shot. I have a lot of stomach swelling as well as feet and leg swelling at times. Can you let me know if there is a shot out there that can aleviate this proble? Doctor: hicause of selling of abdomen,legs has to be found first, if it is oedema (ascitis and water retention)liver, kidney function tests have to be done , so consulrt a physician. Yes injectible dieuretics are there, but given only on prescription if indicated as per signs and symptoms.I would advise complete evaluation firstthanksbest wishes"
},
{
"id": 4436,
"tgt": "Can taking some vitamins helps in conceiving faster?",
"src": "Patient: My husband and I have been trying to get pregnant for a year now and it doesn't seem to want to work in our favour. We had a miscarriage in march of 2012 and have had trouble conceiving ever since. Is there some vitamins or something I can take that will help us Doctor: Hello. Thanks for writing to us. Yes. Taking a multivitamin supplement with essential amino acids, anti oxidants, vitamin B 12, folic adcid, zinc, etc can help in the release of a healthy egg from your ovaries.I hope this information has been both informative and helpful for you. Regards, Dr. Rakhi Tayal ,drrakhitayal@gmail.com"
},
{
"id": 101382,
"tgt": "What causes nose to drip like faucet and sneezing after anaesthesia?",
"src": "Patient: I had a procedure done on Friday where a general anesthesia was administered along with oxygen mask, and oxygen tubing on the side of my mouth. When I came through, my nose was dripping like a f aucet, and started sneezing. I have already gone through 7 boxes of tissue. Help! Please:( Doctor: Hello.Thank you for asking at HCM.If you have not had any allergy problem in past, most probably it may be local irritation to nose by oxygen that caused running nose & sneezing. There is nothing serious and you need not to worry. It will subside by itself, I would also suggest you to take a levocetirizine (or any other antihistamine drug), nasal irrigation with saline nasal spray & to take a deep and relaxing breaths. This will help a lot.However, if you have been allergic previously, there are many possible allergens in a hospital like latex, some medications, etc. In that case I would suggest you to take a combination of montelukast & levocetirizine.Hope this will be helpful to you.Wish you a quick recovery & best of the health.Regards."
},
{
"id": 150876,
"tgt": "MRI shows diffuse cerebral atrophy. Doppler scan shows calcified plaque. Precautions needed?",
"src": "Patient: Respected Doctor, My father age is about 68yrs. He is suffering from neurological problem.we took MRI&NECT DOPPLER scan In MRI scan shows that : 1-Lacunar Infarct/Prominent Virchow Robin Space In Left Lentiform Nucleus . 2-Glotic areas In Left Temporao-Occipital, occipital Region& Right Posterior Region Parietal Region. 3-Diffuse Cerebral Atrophy . In Doppler it shows that: 1-Circumferential calcified intial plaque in right internal carotid artery proximal to bifurcation causing about 55% narrowing. 2-Small calcified plaque in left carotid bulb and left internal carotid artery. sir, please tell me the precautions for my father. Doctor: Thanks for posting this question in HCM I have noted the findings of various investigations. Please note that all these interpretation of the scans results are done in the context of a solid clinical data. I wish you had mentioned about the exact neurological problem your father had like stroke, visual problem etc. Apart from this you should also have mentioned if he is a diabetic or he has hypertension and cholesterol related issues or not and o on. The investigations suggest that probably had strokes in various areas of brain. The Doppler shows cholesterol deposits. He needs blood thinners like Aspirin and/or clopidogrel to prevent it from happening again. His blood pressure and cholesterol should be checked and he will have to be on medicines for the same. Hope this helps"
},
{
"id": 123006,
"tgt": "What could cause problem in bending the wrist?",
"src": "Patient: I am unable to lay my hand flat (as if to do a push up on the floor). My wrist will not bend. This is a recent issue. I broke my wrist twice approx. 35 years ago and have had no problems since. Is this arthritis or some type of carpal tunnel issue? Can debridement be done if it is arthritic? Doctor: Hello, The inability to lay the hand flat is likely to be due to arthritic changes in the wrist joint. Regular physiotherapy exercises can help in decreasing the joint restriction. Hope I have answered your query. Let me know if I can assist you further. Regards, Dr. Praveen Tayal, Orthopaedic Surgeon"
},
{
"id": 29940,
"tgt": "Suggest remedy for persistent itching and burning sensation in anus and vagina",
"src": "Patient: I am a 65 yr old female with Type 2 Diabetes. I am having dreadful burning and itching in the anus and urinary area. I have been using Vaseline, diaper ointment and hydrocortisone ointment 1% last night. Each only offering temporary relief. The itching seems less but the burning is ongoing when I sit. Your advice would be very much appreciated. I have just finished a course of antibiotics for bronchitis - not sure if that matters. Thank you in advance NEVER MIND - I DIDN'T KNOW THIS NEEDED PAYMENT - I WILL GO SEE MY OWN DOCTOR.SORRY TO TROUBLE YOU Doctor: Hello!I have been through your question.Related your concern and based on your history (use of antibiotics and diabetes type 2) I think this is vaginal and anal yeast(candidiasis) infection caused usually by candida albicans.You should know that you have 2 conditions, use of antibiotics and diabetes, that may predispose women to developing vaginal and anal yeast infections. Anal yeast infection can often be caused by yeast in the digestive system or vagina that has migrated to the anus. Usually itching and burning are the most common signs of a yeast infection and can spread from the vaginal opening to the labia and anus as you have. What I suggest for you first is stop using vaseline, diaper ointments and hydrocortisone and second meet a gynecologist, to do a direct sample test for candidiasis and to start antifungal cream treatment (clotrimazole or miconazole) as soon as possible, use probiotics, eat yogurt and wear loose pants and cotton underwear .I hope my answer help you.I wish you a quick recovery."
},
{
"id": 179629,
"tgt": "Suggest treatment for high blood sugar in a child",
"src": "Patient: my patient is 7 years old with weight of 18 kg kc od DM since two years. recently admitted with DKA that ha\\s reslved but still he had high blood sugar levels pre and post meals currently taking 12 units lentus and 4 - 7 units novorapid with meals what should we do Doctor: Hello I need to know sugar level of your baby before further guiding you .You need to consult a paediatric endocrinologist / endocrinologist to guide you doses of insulin to control sugar levels.Thanks"
},
{
"id": 58731,
"tgt": "Have elevation liver function which was diagnosed after a blood test earlier, till continuing. What does this mean?",
"src": "Patient: Hi Dr Had a blood test a few month ago for life insurance the result came out good but they say that i have Elevated liver function , then i waited a few month and took the test again and they say that is the same as last time no change from the last one so what is that mean Elevated liver function? what do i need to do to take care of it . Doctor: hi,welcome to HCMliver function tests include(1).liver enzymes(ast,alt and alkaline phosphatase)which have specific range and function(2)bilirubin levels (total,conjugated and unconjugated which reflects pre hepatic i.e.blood lysis.hepatic-liver damage or post hepatic _obstruction to bile )depending on which we diagnose jaundice and types if positive..(3)proteins (albumin ,globulin and ratio )which reflects synthetic function of liver...liver impairment is caused by various factors (toxins,infection,obstruction,alcohol,tumours,portal hypertension etc..)the normal levels may vary from person to person..do you have any symotoms like pain abdomen,yellowish dark urine,yellowish conjuctiva,itching etcwe should know which of these values are elevated and by how much...for healthy liver avoid alcohol,smoking,control diabetes,hypertension,get hepatitis vaccinated,eat healthy balanced diet,avoid infections...consult a gastroenterogist or physician and follow their advice..stay fit bye"
},
{
"id": 138603,
"tgt": "What causes pain in lower back in hip area?",
"src": "Patient: For the last 5 days I ve had Stabbing pain in my lower back on the left side, basically my hip area in the back. I can t sit straight it hurts worst then labor pain. The only relief is laying in bed. I m wondering if I should see my MD Monday or go to an ER or Immediate care center. Please help me. Doctor: HelloI share your concernLooking at your history this could be acute lumbago PIVD (disc problem) or muscle pullIn my opinion you should take anti inflammatory drug like ibuprofen with muscle relaxant like chlorzoxazone or tizanidineUse hard bed to sleepStretching exercise may help youAvoid sudden bending forward or lifting heavy weight Of it persists or get worsens than you should go for an MRI LS spine for a confirmatory diagnosis.I hope you will find my response relevant to your question Please feel free to ask for more clarification I will gladly answer youwish you good healthtake care"
},
{
"id": 138293,
"tgt": "What causes excruciating and throbbing pain in ankle?",
"src": "Patient: I have excruciating and throbbing pain in my ankle. It s worse when I wear a certain pair of sandals. They are not heels, just a slight angle lift. I changed the shoes to flats but still had to walk for work all day. The apartment the top of my ankle that wraps around and the sides are throbbing and soar. 10 out of 10. Not swollen. Doctor: Dear Sir/MadamI have gone through your query and read your symptoms.In my opinion, you need to change your shoe wear to a more comfortable, type with well padded sole, i would also recommend you to take some time out and walk bare foot on soft sand, so that the foot develops natural action. and gets exercised.I hope that answers your query. If you want any more clarification, contact me back."
},
{
"id": 79528,
"tgt": "What is the treatment for pneumothorax in lower lung?",
"src": "Patient: My father went to see his doctor yesterday & after x-rays was done he told him that he has a very bad pneumothorax in his right lower lung. My father had TB in 2010 which he contracted from someone working with him. He completed the TB course & was given a clean bill of health. The doctor however suspects that this pneomothorax was caused by the TB.What do we do know?? He says it won't help to use a tube to drain the air that's leaking. We don't know what to do from here. Their medical aid expired because the medical aid company was declared bankrupt. How serious is this & what is the next step in diagnoses & treatment? Doctor: Thanks for your question on Health Care Magic. I can understand your father's situation and problem. He needs urgent treatment because pneumothorax is life threatening emergency and with our treatment patient can die within a day. Treatment options are intercostal drainage (ICD) and video assisted Thoracoscopic Surgery (VATS). VATS is costly procedure while ICD insertion I cheap and cost effective procedure. Possibility of recurrence of tuberculosis and complicated with pneumothorax is high in his case. So better to get done ICD insertion, don't think of cost at present, life is more precious than money. Without treatment he may die within 1 day. Hope I have solved your query. I will be happy to help you further. Wishing good health to your father. Thanks."
},
{
"id": 90837,
"tgt": "What leads to stomach upset?",
"src": "Patient: I ve been having this bad pain in my stomach for over a week now. I can t hardly eat. it feels like my food is still sitting in my stomach and hasn t digested. no matter how little I eat it still hurts. I have volmited a few times and it doesn t make the pain go away. I haven t really ate much at all picked at some dried fruit and a few crackers over the past few days. the hurting is just under my left rib area. just not sure if this a bad thing or not. It almost feels like my food is trying to push back up in my chest area. No indigestion or heartburn, Doctor: Hi ! Good morning. I am Dr Shareef answering your query.At the outset I would advise you not to eat from outside if you do. Outside food could give rise to intestinal infections or might produce some kind of sensitivity reaction to a particular component of food examples being lactose intolerance and gluten sensitivity which are under diagnosed everywhere. Once this factor is removed, I would advise you to go for bland diet without any fat, and least spices to avoid gastric irritation. I would also prescribe you with a proton pump inhibitor, a probiotic and a prokinetic drug. If all these are not helpful, I would not hesitate to refer you to a gastro enterologist for a possible endoscopic evaluation of your gut.I hope this information would help you in discussing with your family physician/treating doctor in further management of your problem. Please do not hesitate to ask in case of any further doubts.Thanks for choosing health care magic to clear doubts on your health problems. Wishing you an early recovery. Dr Shareef."
},
{
"id": 9458,
"tgt": "Suggest medication for dark and dryskin",
"src": "Patient: hi i am 37 years old female.I am fair girl. I have very dry skin. Moisturizer is one of my basic necessity like water, air.... as soon as i take bath, i should moisturize my skin otherwise after 3-4 hours, my skin peels off (like snake skin). My skin is very rough too. But now i have blackheads on my nose, pigments around the nose, it causing my skin discolor around my nose. it is very embarrassing !!! Doctor: Hi,Thanks for writing to us.For your dry skin use a moisturiser that contains urea and lactic acid. As far as pigmentation is concerned, using azelaic acid or kojic acid cream would help. Please consult your dermatologist or you may also use premium features of this website for a more detailed discussion about your skin concern.Take care"
},
{
"id": 50039,
"tgt": "On Wellbutrin, Toviaz to control bladder, suggested to go back to Cepralax. Is it ok with past renal issues to take it ?",
"src": "Patient: I am on Wellbutrin 100mg daily and I take a new drug toviaz to control my bladder. My wife and some friends keep telling me to switch back to cepralax15mg instead of Wellbutrin because I had kidney renal failure in the past which is why I have bladder problem. I just check and found others on internet confirming ppl with past renal issues should b careful what liver and kidneys take in Doctor: HIThank for choosing HCMFor the bladder control if you are taking Toviaz then it is alright, but if you are taking Cepralax for the same purpose then it wont work in this way because it is a anti depression agent, so in my opinion you better consult the Urologist and discus this with him and follow the advise, Have nice Day."
},
{
"id": 172671,
"tgt": "Will it be safe to take ORNOF tablet for stomach pain?",
"src": "Patient: My son is complaining stomach pain for the last 4/5 days after attending a Birthday Party. Doctor advised him to take ORNOF Tablets (One Tablet in the Morning & One Tablet in the Evening). Is it the right medicine ? Is there any side effects ?Sushanta Purkayastha Doctor: Stomach pain not a indication for order. But if he has cramping abdominal pain with loose stool that require ornof. It has not major side effect."
},
{
"id": 135742,
"tgt": "Suggest treatment for stomach bloating and swollen foot",
"src": "Patient: I have a friend who has been having chronic stomach problems for the past week or so Also her leg and foot have swollen twice in that week. What can be the problem? I have already asked about her diet and she s barely eating now because she has no appetite. Cancer is a heavy thought of mine, but, I don t want to scare her with misinformation. Doctor: swelling of foot and leg caused by kidney , liver or heart problem . Stomach bloating(? Ascitis) plus leg swelling could be a liver problem. Better to consult local doctor and get examined for proper diagnosis and further management."
},
{
"id": 48881,
"tgt": "Suggest remedy for kidney problem",
"src": "Patient: hi this is siva my grandfather has some problem in kidney our family doctor said test PSA and the result is 4.64 then the doctor said trus guided biopsy presently we are in vijayawada which doctor can do this test.can i know famous doctor for nephrologist in vijayawada city Doctor: Hello! It is not clear from your question whether the doctor suspects a problem in the kidney or the prostate gland. PSA is done to evaluate prostate status. Your grandpa's PSA is 4.64 which is minimally elevated probably due to mild inflammation in the prostate. I would advise you to get an ultrasound examination of the abdomen and then meet a urologist. All the best!"
},
{
"id": 53116,
"tgt": "Does drinking alcohol damages the liver?",
"src": "Patient: I am a 36 year old Female, I am about 5ft 2\", and weigh about 125-135 lbs...I had my physical, all was well with labs ect..just a little elevated pottassium...My question...sometimes I drink a half a bottle to a bottle of red wine a night (750ML) I've been doing this for aprx 8 years...I don't believe I have a problem, however, am I damaging my liver or health? Doctor: Hello , thank you for your question . I have been asked many times this question . If you were my pafient i would have told you that the effect of drinking alcohol and liver damage are different for every person . It depends on an enzyme , that is commonly in lower levels in women . In most cases , if no liver deasses are present , you can process with no health effect about 20-30 grams alcohol .In conclusion you can drink as much as 150-200 ml red wine without health side effect . In fact there are many cardiology statements that says glass of wine every day have health benefits . Hope i have helped you .Best regards !"
},
{
"id": 87427,
"tgt": "What causes abdominal pain with a normal test CT scan?",
"src": "Patient: I have been experienceing severe abdominal pain for about 3 weeks. It started off as occasional but is not pretty persistant. I was in the ER last week and they did a CT scan of my abdomen and pelvis and nothing showed up. They did lab work, and a pelvic exam and still nothing. All they found was strep throat. The abdomen pain started mid-upper abdomen, but then radiated to the right flank and then lower right abdomen. Now the pain is covering almost my whole abdomen. The pain has now become constant and can be quite severe. I have had 2 pregnacies in the past, and also a tummy-tuck surgery 4 years ago. I am a 29 year old female. I go in for a abdominal ultra sound tomorrow. The pain does not seem to be associated with anything I eat or drink. Doctor: Hi.Thanks for your query and an elucidate history.The cause of the pain in the abdomen with normal CT scan be due to entero-colitis or the infection related to the intestines. Since you already have a strp throat the infection is swallowed and directly affects the intestines to cause the problems that you have. This may or may not be associated with loose stools or so. I would advise you to take a course of an antibiotic and metronidazole along with probiotic and supportive and symptomatic medicines. The 2 pregnancies in the past and a Tummy Tuck surgery may not be associated with the present problem. Proper diagnosis and treatment will alleviate your problems ."
},
{
"id": 153452,
"tgt": "Are constant cough and back pain signs of esophageal cancer?",
"src": "Patient: Hello doctor(s),Im a 22 year old Male - Ive been coughing for about a year, and recently started having gagging episodes 10-15 times a day. Ive been given Proton Pump Inhibitors. For the past few days, Ive been experiencing a strange back pain when I swallow food. No Dysphagia though, just Odynophagia. So far its been found that I do NOT have H pylori, or ulcers, or gallstones etc. But we still havent done an endoscopy. Im being told to wait 4 weeks to get that done. My concern is, do my symptoms resemble esophegal cancer? And is 4 weeks a lot of time for this possible cancer to develop? Im so anxious I havent been to work in a week and its destroying my mind. ANy advice would help so much .... Doctor: Hi,Thanks for writing in.A constant cough and back pain in a young male is not likely to be a cancer. Such acute symptoms are more likely to be due to infection in the throat. Please continue taking proton pump inhibitors. If the odynophagia increases then you should go back to your doctor and get examined for an oral examination and probably a neck CT scan to know if there is any abscess which is evolving. If there is significant infection then treatment is with a short course of antibiotics.Endoscopy is done when the infection has been treated and the inflammation is reduced and is safe to do after 4 weeks in your case. In my opinion, the first possibility is an acute pharyngitis or infection of the upper esophagus. Please do not worry."
},
{
"id": 80240,
"tgt": "What causes breathing difficuty?",
"src": "Patient: A little over two months ago I took a pulmonary function test and when I saw my primary physician he referred me to a lung specialist. When I saw the specialist, he told me that my chest x-ray was clear and from the pulmonary test it says I have a hard time taking a deep breath. Also, He told me that it may due to I gained some weight and He schedule for me to have a lung scan because he told me wanted to see my lungs. i am a non-smoker and ever since I broke my nose years ago, I been having problems breathing through my nose. I have been seen by my ENT and he tolk me that I have to have nasal surgery so I can breathe. My question can my nose be causing my problems? Doctor: Hi welcome to hcmI understand your query and concern.Your symptoms are suggestive of consolidation secondary to pneumonia.I advise you to get a chest x ray PA view,Arterial blood gas analysis,complete blood picture,lipid profile immediately to confirm the diagnosis.Monitor your blood pressure,heart rate,oxygen saturation immediately.Drugs like cefixim 500 mg twice daily,tab.grillinctus,pantop .Avoid smoking and alcohol.I also Exercise daily for 30 min.Consult your pulmonologist for expert management and follow up.Hope I have answered your query at the moment.Post your further queries if any,Thank you"
},
{
"id": 63317,
"tgt": "Could the chain of lumps on neck be due to mineral deposits?",
"src": "Patient: hi i am 15 and i have had a chain of painless lumps on the right side of my neck for about 3 years and i am now starting to worry not because they hurt or are getting bigger but they just haven't gone away i have been looking it up online and it says it can be mineral deposits or something else and i do drink quite a bit of coffee do you think it could be caffeine deposits or swollen lymph nodes or something i think i am going to tell my mom tomorrow to take me to the doctor tomorrow. What do you think i should do. Doctor: hi.you need to consult with a physician for physical examination and review of systems.next probable step is a neck ultrasound or ct-scan to evaluate those lumps on your neck.hope this helps.good day!~dr. kaye"
},
{
"id": 44625,
"tgt": "How can a person suffering from asthenospermia increase his sperm count ?",
"src": "Patient: i am 33 yrs old ht.167cm, wt 70 kg asthnoospermia spermcount 48 mil/ml, motile-40%,nonmotile 25%, dead-35% married 3 yrs no issue. how can i improve the spermcouont and the % of active motile? Doctor: Hi Welcome to HCM Read your prob, you have to rule out the cause of azoospermia,get blood test for hormons. you should consult with urogenital surgeon n start proper treatment ,do not worry this may be curable. as your sperm count is 48 which is near normal n this is called oligospermia not azoospermia."
},
{
"id": 52839,
"tgt": "What causes dark colored urine with yellow eyes?",
"src": "Patient: hi, i have been experencing yellow eyes and my urine is tea color dark. The docor did an ultrasound on my liver and he said it was enlarged and my antibodies in my blood is high. And when you push in on my liver it is really tender. My ast and alt is up in my blood test. Gallstones run in my family also. what do you think Doctor: Hi and welcome to Healthcaremagic. Thank you for your query. I am Dr. Rommstein, I understand your concerns and I will try to help you as much as I canWell, these are typical signs of jaundice and treatment depends on exact cause.Jaundice is usually sign of certain liver disease and high bilirubin value. Causes of jaundice vary from non-serious to potentially fatal. Levels of bilirubin in blood are normally below 1.0 mg/dL (17 \u00b5mol/L) and levels over 2-3 mg/dL (34-51 \u00b5mol/L) typically results in jaundice. High bilirubin may be due to liver diseases such as cirrhosis or hepatitis, infections, medications, orblockage of the bile duct In the developed world the cause is more often blockage of the bile duct or medications. Blockage of the bile duct may occur due to gallstones, cancer, or pancreatitis. Medical imaging such as ultrasound is useful for detecting bile duct blockage.Treatment of jaundice is typically determined by the underlying cause. If a bile duct blockage is present surgery is typically required, otherwise management is medical.Medical management may involve treating infectious causes and stopping medication that could be contributing. I hope I have answered you query. If you have any further questions you can contact us in every time.Kindly regards. Wish you a good health."
},
{
"id": 20769,
"tgt": "What causes pressure headaches, heart palpitations, chest pain and syncope?",
"src": "Patient: Ive had pressure headaches and heart palpitations for a few months now..ive had two episodes of feeling faint during this time...my chest hurts off and on everyday and flutters all the time...i get sharp pains in my head now and then and have colds sensations in my chest from time to time...i haven't felt right since this all started..i was put on zoloft, my doctor thought it was anxiety. Ive been to the ER 3 times. Ive had an ultrasound, chest xray, ct scan w/ contrast (head) Doctor: Hello!Welcome and thank you for asking on HCM!Regarding your concern, I would explain that your symptoms could be related to a cardiac arrhythmia. This would not explain the sharp headaches, which could be just ice pick headache, which is a benign type of headache. Anxiety could also play an important role in all this clinical scenario. In such case, you should know that it may take up to two weeks for the effects of Zoloft to be seen. Anyway, I would recommend peforming some further tests to exclude a possible cardiac arrhythmia: - an ambulatory 24-48 hours ECG monitoring to examine your heart rhythm trends for a prolonged time- thyroid hormone levels to exclude a possible thyroid dysfunction. Hope to have been helpful!Wishing all the best, Dr. Iliri"
},
{
"id": 217052,
"tgt": "What causes pain in back of head?",
"src": "Patient: I have been having moderate headaches for quite some time - pain mainly in the back of the head - high on the left side. Is it worth an MRI? Pain meds seem to help but the pain often radiates to other parts of the head. I do have a swelling on the left side of my neck which my physician said was not serious. Doctor: Hi, I have gone through your medical history carefully and can say as following: You are saying to have moderate headaches, manly in the back of the head and high on the left side. Meanwhile, swelling on the left side of your neck is present. If I was your caring doctor would advise to run X-ray of neck first to evaluate the swelling. Most probably spondyloarthrosis is suspected especially if such pain is subsided after taking pain medications, however, need further diagnostic tests to run. For the moment, I advise to continue taking pain medications and do not apply anything locally until the real nature of the swelling in the neck is totally elucidated. Hope it was of help!Dr.Albana"
},
{
"id": 77763,
"tgt": "Having noticed back pain while coughing",
"src": "Patient: Fell on corner of table the other day. ( Hit lower right side of back ) It s pretty bruised, but for two days the pail was tolarable. However this morning I coughed and something in my back popped and now I m in horrible pain. My wife telling me to go to hospital, but I figure there s not much they can do for me. Doctor: Hi. I can understand your concern. It would be advisable for you to get a chest x ray done.Possibility of musculoskeletal pain is more. Ibuprofen is anti inflammatory. You will definitely improve with combination of painkiller and muscle relaxant drugs. Also apply warm water pad on affected areas. Avoid movements causing pain. Don't worry, you will be alright in 1-2 weeks. Hope I have solved your query. Wish you good health. Thanks."
},
{
"id": 161496,
"tgt": "Suggest medication for fever",
"src": "Patient: hi.my daughter is 19 months.her fever is running around 100.7 to 99.00.Today is the 6th days.i gave her tylenol when her fever was above 98.7.what is the temperature of fever to be condider as fever for her?and what can i do now?shall i keep giving her tylenol when the temperature is above 98.7 or 100.4?when will the fever be back norma? thank. Doctor: Hi, This could be a simple viral illness too.Above 100F needs to be considered as fever.Just keep alternating Tylenol and Ibuprofen please.Fever of few days without any localizing signs could as well a viral illness. Usually, rather than fever, what is more, important is the activity of the child, in between 2 fever episodes on the same day. If the kid is active and playing around when there is no fever, it is probably a viral illness and it doesn't require antibiotics at all. Once the viral fever comes it will there for 4-7 days. So do not worry about the duration if the kid is active. Hope I have answered your query. Let me know if I can assist you further. Regards, Dr. Sumanth Amperayani, Pediatrician, Pulmonology"
},
{
"id": 40859,
"tgt": "Will taking Reactine while going through a fertility treatment affect the hormones ?",
"src": "Patient: hi I am going through a fertility treatment with a natural path, but my allergies are really bad, I have been taking Reactine to help me with the symptoms. I stop taking it because I am afraid it may interfere with my treatment. I am wondering if taking reactine, will afect me in my hormones or something else. please help, my allergies are horrible. thank you Rossie Doctor: Dear Rossie,Hi & Welcome to HCM.There's no clinically significant drug interactions between Reactine and the hormones prescribed in the treatment of infertility. Reactine does not interfere with the natural path or other such treatments neither it is going to affect your hormones. Consult a physician for the treatment of allergy after proper lab tests."
},
{
"id": 9876,
"tgt": "What is the treatment for severe hair loss?",
"src": "Patient: I'm 26, My hair has been thining for the last year and half esp in the front and the sides. Now I started witnessing some facial hair as well which I didnt have before. I went to a dermatologits for my hair loss and she did a scalp biopsy, but I'm still waiting for the results. If it's a hormonal problem, should see a GYN/B? Doctor: Hi Dear,Understanding your concern. As per your query, you have symptoms of severe hair loss which seems to be due to nutritional deficiencies, systemic conditions, compromised immunity of the body, genetic factor, fungal infection and hormonal changes.Need not to worry. I would suggest you maintain complete hygiene and take biotin supplements. You should consult a dermatologist for proper examination. The doctor may order skin patch test, blood test and physical examination. Wait for biopsy results and start treatment accordingly. Keep your scalp dry. Take Indian gooseberry powder with water once a day and use olive for massage.Hope your concern has been resolved.Best Wishes,Dr. Harry"
},
{
"id": 60721,
"tgt": "What are the symptoms of Hepatitis?",
"src": "Patient: My friend told me that I might have viral hepatitis. I am a 23 year old. What are the symptoms and what should I do now? Doctor: Fatigue, Nausea and vomiting, Abdominal pain or discomfort, especially in the area of your liver on your right side beneath your lower ribs, Loss of appetite, Low-grade fever, Dark urine, Muscle pain, Yellowing of your skin and the whites of your eyes(jaundice) are the most common symptoms. If you do have any of the above mentioned symptoms it is better for you to go to a doctor for a physical examination and get a few blood tests and a urine test done. The doctors will advice you on what tests are needed after a physical examination."
},
{
"id": 181321,
"tgt": "What causes pain in right upper teeth?",
"src": "Patient: I woke up Saturday morning with my right upper tooth hurting. I assumed it was a toothache during day on Saturday and Sunday it has got worse. Now I can not touch 3 of my teeth with finger or food and it hurts up in to my cheat. Do I go to ER or call emergency number for dentist Doctor: Hi Dear,Understanding your concern. As per your query you have symptoms of pain in right upper teeth which seems to be due to exposure to nerve root endings due to severe attrition, stress, malocclusion and underlying systemic pathology.Need not to worry. I would suggest you to visit oral and maxillofacial surgeon once and get complete examination done. You should go for clinical evaluation and intraoperative periapical radiograph and start treatment accordingly. You should go for root canal treatment and composite resin restoration. Avoid intake of cold substances and hot liquids as well. Maintain hygiene of oral cavity and avoid sticky food.Hope your concern has been resolved.Get Well Soon.Best Wishes,Dr. Harry Maheshwari"
},
{
"id": 75655,
"tgt": "Is it safe to take MOntecl LC for wheezing?",
"src": "Patient: I'm 28 year old.having a baby girl of 1.5 year old. planning to have one baby boy. but now I'm having wheezing so taking Montecl LC for 1 month. when can I conceive? is the medicine effect in body will affect the new born. was having sympotms of asthma. so any precautoion can take? and for getting a baby boy ovulation date will help? my periods are almost regular..only date changes but it's regular. kindly advice. Doctor: Thanks for your question on Healthcare Magic. I can understand your concern. By your history and description, possibility of asthma is more. And best treatment for asthma is inhaled treatment. Montek LC is having levocetrizine and Montelukast. These are additional drugs for asthma but not main drugs. It is better to control your asthma before conceiving because uncontrolled asthma can cause maternal and feotal complications. So first consult pulmonologist and get done clinical examination of respiratory system and PFT (pulmonary function test). PFT will not only diagnose asthma but it will also tell you about severity of the disease and treatment is based on severity only. You will mostly improve with inhaled bronchodilator (formoterol or salmeterol) and inhaled corticosteroids (ICS) (budesonide or fluticasone). These drugs are safe in pregnancy and have no effect on ovulation or periods. These are also safe for developing baby. So don't worry. First control your asthma and then think of pregnancy. Hope I have solved your query. I will be happy to help you further. Wish you good health. Thanks."
},
{
"id": 218727,
"tgt": "Do pain and spotting indicate complete termination of pregnancy?",
"src": "Patient: Dear sir, i would like to ask one doubt that i took full kit (1 mifipriston and after 40 hours 4 misopristol by Doctor advice in doctor clinic) from LMP (7 weeks 5 days) , now 6 hours were gone. now i felt pain and as well as gets slightly bleeding occurs. is it means that abortion are terminate? Doctor: Hi, Thanks for asking. If you have followed doctor's advise about Mifepristone &Mioprostol pills...& pain &bleeding has started,that inducates starting the process of abortion. Normally complete emptying of uterus with blood,clots,fetal parts & membranes follow within 6 to 24 hours of the Misoprostol pills. You should wait until then for expulsion of products of conception( till 24 hours of the pills.) -Complete emptying of uterus reduces pain &bleeding . An intwrnal chevk up by treating doctor confirms complete abortion. - In case there is no complete expulsion... consult your doctor for need of further treatment. Thanks."
},
{
"id": 105741,
"tgt": "Allergic reaction to cephalexin, itching, red welps, hand swelling",
"src": "Patient: I had a allergic reaction to cephalexin 500mg i only took the pills one day and now its a week later and im still itching like crazy and now im breaking out in these red welps some are like knots that are very sore and my wrist up into the palm of my hand are very red a swollen not to mention very tender and sore Doctor: you might be allergy to the cephalosporin group take anti allergic medicine alday / cetriz 10 mg daily and take hepamerz tab 1twice daily for 3 days supportive medicine for apply caladryl lotion on itching area"
},
{
"id": 184754,
"tgt": "Suggest remedy to stop bleeding in the upper molars caused while flossing",
"src": "Patient: Five hours ago, my husband (age 51) had flossed out a piece of meat between two upper molars which cause some bleeding. He just woke up and found 3-4 cm blood clot had formed and he pulled it off and now it wont stop bleeding. He has rinsed with mouthwash and salt water, still it bled. As I am typing now, he said it feels like it is clotting again. He had a stroke on November 3, 2012 and is on blood pressure medication. Should he go to the ER? . Doctor: There may be slight bleeding on flossing but if the tissue is already inflamed then there will be more bleeding. Hypertensive people have a tendency for more bleeding. Is he on any other medication for thinning blood. Let the clot be there. Clot is formed to stop bleeding and if you keep on removing that clot bleeding will continue. I dont think you need to worry. Take care"
},
{
"id": 164804,
"tgt": "Suggest treatment for nose bleed in kids",
"src": "Patient: Hi, may I answer your health queries right now ? Please typ. My daughter is 3 has suffered nose bleeds 5x plus a week for the last year. We saw an ent consultant today and he s booking her into hospital to be given general anasthetic as both blood vessels are enlarged in both nosterls and to have bloods takenim worried this could be more serious than he was letting on? Could you help with some info thanx. Doctor: Hi... by what you say I feel that he is expecting that there could be a hemangioma in the nasal cavity which is causing profuse and frequent bleeds like this. Probably he wants to explore the nasal cavity and confirm the same. That is the reason why he wants this exploration to be done under general anaesthesia. But as a pediatrician I would also like to rule out bleeding disorders and platelet related disorders also. I would investigate with prothrombin time, activated thromboplastin time, and platelet count.Regards - Dr. Sumanth"
},
{
"id": 27626,
"tgt": "How to treat heart valve problem?",
"src": "Patient: hi my mom is 81 years old and has two bad heart valves the aortic and the valve in the left side is also really bad she has afib and scar tissue on her lungs how long can u live like that her doctors wont operate because she wont live threw the operation Doctor: HiThank you for trusting and asking HCMI read your query and understand your concern Unfortunately at such ages it so difficult to resist big surgeries like heart surgery It is recommended that conservative treatment is the best option Cardiologist depending on the severity of valvular dysfunction should decide the dosage of diuretics, inotrops and anticoagulation Patient treated in the right way may stay safe for a long time Hope my answer will help you Stay healthyRegardsDr DrilonaCardiologist"
},
{
"id": 110791,
"tgt": "Suggest treatment for left sided back pain",
"src": "Patient: Went to the emergecy room saturday had a cat scan, have back pain that is onleft side and goes around to the front..was perscibed a muscle relaxer and antibiotic for urinary trac infection (blood in urine) cat scan showed nothin wrong with kidneys, pancreas or liver. Pain has not eased.. Doctor: Hello, I have studied your case. Your present complain suggest disc pain leading to back pain.Due to compression of this nerve root in spine there is pain.I will advise you to do MRI spineFor these symptoms analgesic and neurotropic[pregabalin] medication can be started.Till time, avoid lifting weights, Sit with support to back. You can consult physiotherapist for help.Physiotherapy like ultrasound and interferential therapy will give quick relief.I will advise to check your vit B12 and vit D3 level.Till time you can continue your medication.Hope this answers your query. If you have additional questions or follow up queries then please do not hesitate in writing to us. I will be happy to answer your queries. Wishing you good health.Take care."
},
{
"id": 26686,
"tgt": "Could high BP be due to nervousness?",
"src": "Patient: I had a mammmogram today and right after the appt I used an electronic BP machine. My BP was 130/81 and pulse was 104. My BP is typically 110-120/60. I'm 45 yrs old. 5'3\" & weigh 115 lbs. Could the results been high because I was nervous or because my arms are so small in the machine?? Doctor: Hello. Thank you for your question and welcome to HCM. I understand your concern. First, good luck with your mammogram. Blood pressure is a highly variable component of the human functioning system. It can go up and down again within few seconds. These extreme changes can be detected especially when one is having fear, a lot of psychological stress, physical activity, or hormonal and neurohumoral substance flushes in the system. So, basically, fear is a type of anxiety, and all anxieties cause the adrenalin levels to spike, and the rest - dizziness, high heart rate, high blood pressure, shortness of breath - are all symptoms of its effects on almost all organ systems in the organism. My opinion is that this is the case, where the fear of a result from an important medical test, also results in higher than average blood pressure figures and heart rate figures. Furthermore, my opinion is that there is nothing to worry about, and this is only physiological high blood pressure, not a pathological or abnormal one. Take care. I hope this helped. Regards, Dr. Meriton"
},
{
"id": 41685,
"tgt": "What does my semen analysis report indicate?",
"src": "Patient: Good night doctor, i am concerned about my semen analysis results and i would like your help deciphering what it means.volume: 2.5Viscosity: NpH: 8.0Motility: 80Rapid Progressive: 30Sluggish: 50Immotile: 20Concentration: 60.0Normal morphology: 80 Doctor: HelloRead this and at end I will explain.Volume 2.5 ( normal 2-6 ml )Viscosity normal pH 8.0 ( normal )Motility 80% ( normal 100 % but in your case sufficient for conception because normal FERTILITY is associated with >64% ,rapid , sluggish are speed of sperm in your semen).Concentration 60 ( means normal because within one hour semen liquefied ).Normal morphology 80 ( normal FERTILITY is associated with >13 % exhibiting normal morphology ).You don't have any abnormality in your semen for normal FERTILITY.Good luck."
},
{
"id": 179578,
"tgt": "Suggest treatment for body inflammation in kids",
"src": "Patient: Hi my daughters rugae looks a bit inflamed. I remember mine feeling similar as a child for periods of time. Im wondering if this is normal and due to something, maybe just growth as shes able to eat fine etc or whether it could be anything serious. I will take her to the drs or dentist anyway but was just curious to see what you thought. Kind Regards Doctor: HiThanks for writing to health care magic.The rash could be atopic dermatitis or vitamin deficiencyThis needs constant application of moisturising oinments daily and good intake if fruits and vegetables.Try avoiding dairy products to see if that helps.Wishing your child good healthRegardsDr Arun"
},
{
"id": 147550,
"tgt": "Typhoid positive, numbness. Prolonged typhoid? Diet plan?",
"src": "Patient: Hi, I have typhoid positive third time now. first one in oct 2011, second in nov end 2011 and now may 2012. i have numbness in my head, back and arms since dec 2011 at least 10 times a day and the duration is few seconds without any vision loss . for this i am taking betacap tr 60 once a day and gabapin 100 thrice a day since two month now. All my tests related to numbness like MRI head, Holter monitor , echo, EEG and blood tests are negative/ normal. Doctor said there is no obvious cause of numbness. kindly guide can numbness caused due to typhoid also? How can i confirm if it is prolonged typhoid? what diet plan or exercise should i follow for the same? Doctor: These symptoms donot look like of Typhoid.Perhaps you have just gone for widal test.This may be positive in a large no. of healthy individuals,it has no significance.The symptoms seem like of depression & in my opinion you need anti depressive treatment."
},
{
"id": 52950,
"tgt": "What causes strong urine odour in a person with liver fibrosis?",
"src": "Patient: Since I had a mesh hernia op, and after ejaculation, my wee smells strongly of fish. It last for about two days then stops. I also have fibrosis of the liver caused by tainted blood and Hep C infection diagnosed 5-years ago and following anti viral course I am viral free. I am 70 years of age. Doctor: Hi, I had gone through your question and understand your concerns.In most cases this is sign of uroinfection and liver fibrosis may contribute to increased risk of uroinfection. Also, dark urine may be sign of more severe liver disorder such as cirrhosis or liver failure. IF this continues you should do urine analysis, urinoculture and kidneys ultrasound to rule these main causes out. You should drink raspberry tea and plenty of water till you do tests. Hep C should not be related to your urine symptoms but as I said it may contribute indirectly as a cause of liver failure. Hope this answers your question. If you have additional questions or follow up questions then please do not hesitate in writing to us. I will be happy to answer your questions. Wishing you good health."
},
{
"id": 154387,
"tgt": "What is \"Parafalcine Meniginoma\"",
"src": "Patient: Last year in March during a MRI Scan my doctor saw something that looked like a cyst and thereforereferred me to Neurologist. I went to the Neurologist and he said there is nothing to worry about, it isnot even a cyst just some kind of hardening of the skin and he thinks this is there from birth on and nothingunusual but he would like me to return after a year in order to have another scan just to make sure it is notgrowing. And if it has not got bigger he wants me to have another MRI after two years and after that it isnot necessary to monitor this any longer. Now, my MRI scan has been arranged for tomorrow and on the referral form that he has written out for the Imaging Centre it says \"Parafalcine Meniginoma\" found. As I looked this up on the internet today, it says slow growing tumor. I am worried now and confused. Did my doctor play this down in order not to worry me or is there reason to get worried? Doctor: Hi,Thanks for writing in.A meningioma is a slow growing tumor of the covering of brain. This is to be regularly monitored on MRI and if there are symptoms then it is surgically removed. Since it is parafalcine in location therefore it is not in an important area in the brain and will not cause any serious symptoms.Regular MRI scans help to know if a meningioma has grown enough to cause symptoms and if it requires removal. It will help to know your symptoms in detail to know if the meningioma is symptomatic in your case.Your doctor wants to to keep calm and take things easy. There is no reason to worry if the lesion is small in size and not causing any symptoms. Sometimes it is also difficult to make a confirmatory diagnosis on MRI scan especially when lesions are very small and parafalcine meningioma might only be a probable diagnosis. Please do not worry."
},
{
"id": 160353,
"tgt": "Traetment for Aplastic Anaemia or any alternative therapy",
"src": "Patient: Do you know of any good traetment for Aplastic Anaemia ? Patient has undergone ATG Treatment recently, wish to know the response. Is it a form of cancer ? what about prognosis ? treatment? alternative therapy ? Doctor: aplastic anemia is a leukemia of bonemarro bone marrow is centrally located in the bone it forms all the blood cells like red blood cells, defensive white blood cells and platelets etc. prognosis is improving now a days whith lot of supportive therapy given usually the prognosis is given in terms of five year survival rate in medical jargons it is around 80% sometimes the patients recover spontaneously however they need the suportive care till even in immunosuppressive therapy like ATG there are chances of recurrence and supportive care continues even then The definitive treatment is said to be bonemarrow transplantation which has 90% success when given from a matched donor"
},
{
"id": 66685,
"tgt": "How to get rid off the sore lump on the upper back thigh?",
"src": "Patient: Sore amorphous lump more than an inch deep on the upper back thigh. Been there over a week--as far as I know. No sign of anything on the surface: ingrown hair, boil, etc. Obviously something to have checked out, as it seems to hurt more today than a week ago. Doctor: Hi, dearI have gone through your question. I can understand your concern. You may have some benign or malignant soft tissue tumor or some cyst. You should go for fine needle aspiration cytology. It will give you exact idea regarding cause. Then you should take treatment accordingly. Most of the soft tissue tumor need excision only. Consult your doctor and take treatment accordingly. Hope I have answered your question, if you have doubt then I will be happy to answer. Thanks for using health care magic. Wish you a very good health."
},
{
"id": 122143,
"tgt": "What causes swelling in stomach,legs and feet after treated for pneumonia?",
"src": "Patient: Doctor my Father s age is 72 yrs old he is an Heart, Diabetic, and Blood pressure paitent. 2 and half week ago he was admitted in hospital for the treatment of pneumonia. Now he is fine with that. But now his stomach, legs, feet and anckle has swelled. and b cos of that he want able to sleep in night properly. Doctor: Hello, We should consult with a cardiologist as soon as possible to see if his heart problems have been aggravated. His kidneys should be checked too. Should see a cardiologist as soon as possible. Hope I have answered your query. Let me know if I can assist you further. Take care Regards, Dr Albana Sejdini, General & Family Physician"
},
{
"id": 167733,
"tgt": "What causes circular patches on tongue?",
"src": "Patient: My two year old has these circular patches on her tounge they almost look as though a scrape, she has one big one on back of her tounge and a little one close to the tip of her tounge, their the color of her tounge, just wondering if its anything serious please help. Doctor: Hello,I can understand your concern. Are the patches painful? If they are painful, then there are chances that your daughter is suffering from viral or bacterial infection. In that case, antivirals or antibiotics should be given after consultation with a physician. If the patches are not painful, then it is possible that the papillae of the tongue that give rough texture to the tongue are located in such pattern. In that case, there is nothing to worry about. It could also be due to vitamin B complex deficiency that can be treated by oral supplements.However, I would advise you to visit a dentist once and let him/her clinically examine the tongue so that any abnormality can be ruled out and treated. I hope this information helps you. Thank you for choosing HealthcareMagic. Take care.Best,Dr. Viraj Shah"
},
{
"id": 166557,
"tgt": "What causes cough with greenish stool in an infant?",
"src": "Patient: My 9 month old has been sick lately - coughing,runny/stuffy nose and just seems to be getting well. She is still drinking her formula but barely any solids. For the past 2 days her poop has been greeny/yellowish colour, slimy and has been smelling a strong vinegar smell. What could be causing this? Doctor: In my experience, the poop can be variable once a child is recovering from an illness. I usually don't get too concerned. The most important thing is that she is drinking and acting okay. The appetite for solids will come back in time. Just make sure she continues to drink and is urinating and acting okay. If you have more concerns, you can take a sample and have her doctor take a look. But most likely it is not really significant.Wishing that she continues to get well!"
},
{
"id": 166912,
"tgt": "What needs to be done if a child accidentally swallows clotrimazole cream?",
"src": "Patient: My 20 month old(she is about 25 lbs, and about 30 inches tall) got into the bathroom, climbed up on the counter and I found her playing with Athlete s foot (Clotrimazole) cream. She had it on her hands, stomach, and some on her face, not sure if she swallowed any or got it in her mouth for that matter. Was just wondering if I should take her to ER? says get medical help if swallowed. Doctor: Hello and Welcome to \u2018Ask A Doctor\u2019 service. I have reviewed your query and my advice is not to worry. This is unlikely to cause any problems even if ingested in small amounts. Hope I have answered your query. Let me know if I can assist you further.Regards,Dr. Archana Verma"
},
{
"id": 162947,
"tgt": "What causes pain in vagina of a toddler?",
"src": "Patient: i have a 2 year old daughter who just started complaining that her private area hurts she said mommy pee pee i thought maybe she had to pee so i put her on the toilet and she cried and never peed i put her diaper on got her ready for bed and she cried again her diaper was a little wet but not much i wiped her private down and put a clean diaper on her private is clear no redness..can you tell me what it might be Doctor: Hello and Welcome to \u2018Ask A Doctor\u2019 service. I have reviewed your query and here is my advice. I am a pediatrician and happy to have the chance to help you. If you see no vaginal discharge and no foreign body(like a piece of toilet paper), then there is a possibility she has a urinary tract infection. Does she have a fever? Has she ever had a urinary tract infection before? I suggest that if she has to pee during the night, you take her clothes off and place her into a tub of warm water(only, no soap or bubble bath) and encourage her to pee. If she continues to cry, then you must take her to the doctor's office in the morning. Before you leave for the doctor's get her to drink around 6-8 oz of liquids so that she will urine in her bladder at the doctor's. I hope this helps and your daughter gets well. Please return if we can assist you again. Arnold Zedd, MD, FAAP"
},
{
"id": 33689,
"tgt": "Why am I experiencing weakness after severe cold recovery?",
"src": "Patient: I am experiencing weakness after a severe cold or flu recovery. I get adequate rest and have an excellent vegan diet. I normally exercise daily but find it too difficult now. I have had a recent physical and blood tests. All Normal levels. I am taking added vitamin and mineral supplements but still experiencing extreme lethargy and lack of energy for workouts. Any advice will be appreciated. Thanks. Doctor: Thank you for following up. I suggest-1.Diet riched proteins(milk,jogurts, dahi,curd,protein powder,egg every alternative day,soya beens, green peas,rajma), olive oil 1 tbs 3 times a day for 1 month.2.Roobis and tulsi tea for boosting your immunity, tulsi juice.3.Fresh fruit juices. 4.Ginseng Wishing you a good health Dr.Sagar Gaurav Shrivastva"
},
{
"id": 41386,
"tgt": "Are Ubiphene and Addyzoa given for low sperm count?",
"src": "Patient: Hi My husband is 28 years old. He has low sperm count problem. His count now is 21 millions. The doctor prescribes us to eat q-gold tablet and testovitforte. Another doctor prescribes ubiphene and adizoa.I need which is the best tablet from the above mentioned we have to continue to increase the sperm count quickly. Doctor: Hi Wecome to HCM, Your husband sperm count is with in fertile range according to new WHO standards. Hope his motility also with in normal range. I recommend Q- gold, and Ubiphene. Please take Q-gold Twice daily. Ubiphene 25 mg Night for 25 days in a Month for 3 months. Taking Adizoa and Testovitforte also not harm."
},
{
"id": 127937,
"tgt": "What causes soreness in the left wrist?",
"src": "Patient: My question is, my left wrist is sore but not real painful, has been for a few days. My fingers are fine, no pain or soreness in them. Did I possibly just hurt my wrist somehow? I also have no swelling. I wonder if this is a symptom of carpel tunnel or just something else? The most is that the soreness is on the top of the wrist, not underneath! Doctor: Hello, I have studied your case. As per your history there is possibility of TFCC/ligament injury to wrist.It may subside with time.Pain indicates inflammatory process or nerve compression.I will advise you to do MRI and Ultrasound of wrist.Till time you can start analgesic and crepe bandage support.Hope this answers your query. If you have additional questions or follow up queries then please do not hesitate in writing to us. I will be happy to answer your queries. Wishing you good health.Take care."
},
{
"id": 115846,
"tgt": "Suggest treatments for anemia",
"src": "Patient: respected sir i am looking for the treatment for anemia cause i have a habit of masturbation due to over performing it i gets very weak i am doing this since 10years of age now i am 27 years old could not able to do thing just normal person do thats why i am finding a right doctorwho know my problem and give full treatment and make my life and career keep going .my mobile 0000 .type of problem. anemia blood deficiency .requirements. anemia Doctor: Hi,Thanks for asking.Based on your query, my opinion is as follows.1. Need hemoglobin levels and complete hemogram to identify specific deficiency.2. If MCV is high, possible vit B12 deficiency.3. Low MCV would suggest iron deficiency. Correction of hemoglobin to at least 15g% is necessary. Followup with reticulocyte count after 15 days of starting treatment would show increase in reticulocyte percentage. Start immediately on vit B12 and iron therapy.4. If masturbation is affecting routine life, then you will require counseling.Hope it helps.Any further queries, happy to help again."
},
{
"id": 81924,
"tgt": "What causes shortness of breath and coughing up blood?",
"src": "Patient: 4 days ago I got a very sore throat. I went to the doctors the next day and got a amoxicillin prescription. It has not been doing anything for me. This morning I had tons of trouble breathing, and coughing up blood. I went to the hospital right after. The doctor was in the room under a minute and said my vital signs are okay. He did not look at my throat and didnt really check my breathing. He gave no time for me to breathe in and out. Then said You will live Im still very worried because Im having shortness of breath. Is there anything I can do at home to help? Also should I go out to a different hospital since the medication is not working and because he did not check properly? Doctor: Thanks for your question on HCM. In my opinion you should definitely consult other doctor , if possible pulmonologist. As you may have lower respiratory tract infection ( LRTI ). The reasons for these in your case are1. You are not improving with the current medicines. 2. LRTI is very common after upper Respiratory tract infection ( URTI ); if URTI not treated properly. 3. Coughing out blood ( haemoptysis ) and shortness of breath, both are seen in LRTI. So get done chest x ray to rule out LRTI. You may also need sputum culture to guide the antibiotic therapy."
},
{
"id": 57775,
"tgt": "Does sludgy material in the gall bladder cause severe abdominal pain that radiates to the back?",
"src": "Patient: Over several years i have had occasional severe upper abdominal pain starting in the center of the abdomen and radiating to the back then to the side. i feel sick to my stomach but do not vomit. i have had an MRI, multiple blood tests and a lower GI that have only shown a Sludgy material in the gallbladder. I have tried to stay Gluten free and it seems to help but not all the time. Doctor: Hi. The sludgy material can cause spasms if the bile conducting ducts or secondary infection in the pancreatic-biliary system. This can cause the pain you explained. Although your MRI shows to b e normal , pancreatitis has to be ruled out by blood test ."
},
{
"id": 23918,
"tgt": "Is it safe to continue with yoga for hypertension and BP?",
"src": "Patient: I am 65 years old. Taking care of my health by free hand exercises and Asan (Indian Yoga) since my boyhood. Doing Shirsason ( Head below and legs upone Indian yoga). But I am a hi-per tension patient and my B.P. is quite normal bt taking medicine, 1 Amlodopine, 1, Zestril, 5 mg and no health related major problem. May I continue my usual present daily life or any other suggestion. Please let me know and oblige. Doctor: Hi,If your blood pressure is well controlled with your medications and you have no complaints during your exercises, then you can continue your activities.Take careCome back if you have any further questions"
},
{
"id": 157224,
"tgt": "Is there anything that can be done to reduce the shortness of breath and dizziness which are the side effects of Enoxaparin?",
"src": "Patient: My husband has been on enoxaparin for four years (for DVT) and has a few of the side effects listed. They are shortness of breath, dizziness (when getting up from chair or bed), fatigue, weakness, and heaviness in legs. He also went through chemo and radiation in 2010-11 (and is now cancer free!) Is there anything that can be done to improve his overall well-being. Doctor: Hi, Welcome to Health care magic forum. Your husband is using enoxaparin, since 4 years, and now you are facing short ness of breath, postural hypo tension,fatigue, weakness, and heavyness in lungs. I advise you to consult your doctor foe these side effects. He may change the drug, or may minimise the dose. Take more of green leafy vegetables, pulses, sprouts,and protein rich foods for maintenance of good health. Wishing for a quick and complete recovery. Thank you."
},
{
"id": 95278,
"tgt": "Why is my nipple hard with pain ?",
"src": "Patient: an 11 year old one nipple is hard and they are both in pain its been happening for awhile now and if i kock it,it feels like a pin is going through it please help!!!!!!!!!!!! Doctor: Hello. Thanks for choosing HealthcareMagic forum. In a growing breast of an eleven year old,it is common to have slight pain or hardness. this all happens due to the hormonal changes taking place in the body at that time.There is nothing to worry. Just wait and watch. Gradually all these symptoms will disappear. Dr. Rakhi Tayal drrakhitayal@gmail.com"
},
{
"id": 710,
"tgt": "Can I get pregnant while suffering from PCOD?",
"src": "Patient: Hi Doc,i am married for 5 months now and recently got diagnosed with Bilateral Polycystic Ovaries.i had irregular periods in the past.In the ultra sound report:My uterus is in normal size measure approx 59.0 X 35.8 X 45.5 mmThe endometrial thickness is 12.9 mm,hyperechoicBoth ovaries are bulky.The right ovary measure 41.5 X 33.5 X 20.5 mm and 14.9 cc vol.The left ovary measures 36.6 X 32.8 X 21.4 mm and 13.4 cc vol.i am metformin 500 twice daily.will i ever become pregnant? Doctor: Hello,Thank You for trusting HealthcareMagicChances of conceiving with appropriate management of Polycystic Ovarian syndrome (PCOS) is high.Maintaining a healthy weight by eating healthy add regular exercising is very important.Continue metformin as prescribe, as metformin regulates the insulin level and promote ovulation.There are also other medications like clomiphene,HCG injections and other treatments available if metformin alone is ineffective.Hope I was able to help youPlease address further questionsRegards,Dr.Lekshmi"
},
{
"id": 117653,
"tgt": "Does a low platelet count pose a threat of some disease?",
"src": "Patient: I just had an annual physical and my blood work came back normal except for a low platelet count. I do not have any unusual bruising or bleeding, and my doctor wants to repeat the test. Should I be concerned about any serious illnesses or could there have been an error in the test its self? Doctor: Hi, its all depends on how much low your platelets count. Because slightly low count doesn't need concerns. Even simple viral infection can leads to slightly low platelet count. However if you have very low count thrn it must need concern. it may suggest some underlying pathology.which may be asymptotic. So you should go for repeat test and thrn consult your doctor accordingly.Thanks for using health care magic."
},
{
"id": 226092,
"tgt": "Had copper coil fitted. Bleeding, cramps, feeling movement. Taking Paracetamol. Normal?",
"src": "Patient: Hello, I just finished my period on Sunday- 2 days ago I had copper coil fitted today and bleeding started again as if its my second day of period , I also feel sick i have cramps and have bedn taking paracetamol i also feel as if something is moving inside my uterus. Is this all normal? Ps I had 2 pregnancies and 1 baby - he is now 7 months old- this was a forcep delivery. Please help, I look forward to your reply Doctor: Hi.Thanks for asking in Healthcare Magic.Copper T is generally inserted during on soon after menstruation. Spotting and increase in flow is common and experienced by 2 to 10% of women after insertion of copper T. It is felt as a foreign body by the uterus and it increases the uterine contraction and that is the reason for your pain and bleeding. The acceptance rate of copper T in women varies and so do not worry. Feel for the thread to be sure that it is in place. Paracetamol may help but if pain is very severe and unbearable and if bleeding is heavy, or if the thread is not in place, then you should consult your gynecologist. X-Ray or ultrasound will help to rule out complications like perforation or expulsion.I hope this is useful to you."
},
{
"id": 62592,
"tgt": "What could painful breast with lumps and swelling indicate?",
"src": "Patient: i started taking cerazette about 6 weeks ago and i have this week developed slightly painful breasts that feel hard and a little lumpy. I was also bitten on the breast by a mosquito a few days ago. Most of the redness and swelling has gone but there now feels like there is a lump there, under the site of the bite. Could this be caused by either the bite or the pill? or could it be something more serious? Doctor: Hi, dearI have gone through your question. I can understand your concern.You have breast lump with pain. It can be due to fibroadenoma, fibroadenosis or some other mass. Chances of lump due to mosquito bite is rare. You should go for mammography. If needed go for fine needle aspiration cytology or biopsy of that lump. It will give you exact diagnosis. Then you should take treatment accordinglyHope I have answered your question, if you have any doubts then contact me at bit.ly/Drsanghvihardik, I will be happy to answer you.Thanks for using health care magic.Wish you a very good health."
},
{
"id": 69476,
"tgt": "What is the painful sensitive lump above my navel?",
"src": "Patient: I have a hard lump above my navel that is very sensitive, sometimes painful. A surgeon recently examinded me but said I didn't need surgery. I am constantly aware of this pain. The same surgeon repaired a ventral hernia on the side of my navel, and it was the same type of pain. Please advise me on this. Thank you very much. Doctor: Hello and welcome to HCM,Presence of hard painful lump on the naval suggests some underlying pathology.A past history of hernial repair raised the possibility of recurrence of hernia or some complication of the wound.A clinical examination and relevant investigation especially ultrasound of the umbilicus is required to look for any abnormality of the wound.In case of recurrence of hernia, repair may be required.You can consult another surgeon if you are not satisfied with current surgeon.Thanks and take careDr Shailja P Wahal"
},
{
"id": 23635,
"tgt": "What causes faster heart beat and breathlessness?",
"src": "Patient: my mother is 60 years gets breathless whilst sleeping and sitting. She feels her heart is beating very fast and also feels her stomach is not normal she feels she is as if her stomach keeps jumping inside her. strange symptom she is asthmatic takes her inhalers. has had all tests including lung, blood diabetes and ashtma review all is normal please advice? Doctor: Hi, being breathlessness on lying down is not a good symptom. it can happen in many condition, predominantly in lung and heart conditions.I suggest your mother to have a consult a cardiologist and a pulmonologist regarding the same.as she's asthmatic, it may be more of a serious lung problem and hence get in touch with a pulmonologist first.also get an ecg and 2 d echo done."
},
{
"id": 223907,
"tgt": "What causes brownish spotting and sore breasts after taking Postinor?",
"src": "Patient: I had unprotected sex on the 12th of july,3 days after my last period, i took postinor 2, 1st pill less than an hour after intercourse and the 2nd pill 12hours later. I saw some type of brownish blood on the 19th when i wiped with a tissue, 7days after taking the pill. It lasted for 2days and my breasts started hurting and they still do. Should i be worried?..please help.thank you Doctor: Hi Hope this message finds u in good health.I have gone through ur msg and understand your concern.it may be due to some sort of hormonal imbalances or variations in the body,though there may be other reasons as wellNothing to worry about, You should eventually get back to normal.Take multi vitamin and calcium supplements Do consult a gynecologist if symptoms worsenGet back to me for any FOLLOW UP QUERIES anytime.Regards,Dr Mahaveer Patil...(MBBS,MS,Mch)"
},
{
"id": 131050,
"tgt": "What does altered morphology with trucation, loss of substance superior, anterior left acetabular labrum mean?",
"src": "Patient: I had a left hip labral repair three years ago. I have a lot of pain just had another arthogram. What does altered morphology with truncation and loss of substance superior and anterior left acetabular labrum compatable with post surgical change mean? Doctor: It means in easy terms that you are prone to osteoarthritis of hip joint look for signs on the side or anterior aspect of your hipGood luck"
},
{
"id": 42480,
"tgt": "Suggest remedy to treat infertility problem",
"src": "Patient: hi iam 26 we r plan to have baby but my peroid is going very less i consult doctor they told after period egg not growing and given me bigomet sr 1000 mg for 6 month now i finished 6 month. last month (august 25 th my period is going very high after period 11 day to 14 day we hv sex but no any result . i dontknow what can i do.....plz give me solution..... Doctor: HelloI want to tell you about few things about scanty menses and heavy menses ( before bigomet as you were a case of scanty menses, after treatment there is heavy menses ) i.e. irregular periods present . The treatment you received were for Poly Cystic Ovarian Disease ( pcod) a very common disease in young female ( child bearing age) . You have sex in ovulation days ( 11-14 days ) ,still no pregnancy . So in my opinion first of all get in ultrasound of both ovaries . Get in blood examination for F SH/LH ratio.Also get in blood for androgen level . Get in follicles study and ovulation date and maturation and rupture of follicle ( dominant ) . So consult a gynecologist or infertility specialist and get opinion.Hope you will understand all things mentioned . If you want to know anything else regarding this , you can ask.Good luck."
},
{
"id": 186110,
"tgt": "Need medication for inflamed small pallet inside the mouth on the roof",
"src": "Patient: i have an inflamed small pallet inside my mouth on the roof i believe along with a piece of hanging skin infront of my uvala it dosent hurt but more so annoying when i eat/breath ive had it for maybe 2 weeks at most i was wondering what this is. It is also white in some spots when looked at with a flash light Doctor: it may be caused due to trauma. if the inflamation is more then you can take a course of antibiotic like ciprofloxacin 500mg bd for 5days.avoid eating very hot food ,better to take soft food .rinse your mouth thrice with a good mouthwash .and for the piece which is hanging its better to get it checked with a dentist if it persists.hope this will help."
},
{
"id": 165349,
"tgt": "What causes mucous in the stool and fever after taking cow s milk?",
"src": "Patient: Hi. My 10 monts old son has slight fever (38,5 C rektal temp) and today his stool was very sour and with a little bit of mucous. He has no other symptomes (no pain or runny nose or earache - nothing). The same thing happened 3 weeks ago and we did blood and urine tests and they were negative on infections. The pediatircian concluded that it must have been a virus because it passed 4 days later with no medication (except for the fever). He ate the whole time (smaller portions) and be breastfed more often (expecially at night). As he didn t have other symptoms I suspected teething but no teeth came out. I m afraid that the same thing is happenin - and suspect cow milk! -though ne had no rash! Three weeks ago he ate 5 spoons of youghurt for the first time and yesterday I added 3 spoons of boiled milk into his lunch. He loves to eat everything that a 10 month old is supposed to eat, he drinks formula milk and is breastfed. Could it be a reaction to a bit higher level of lactose (since formula contains lactic acids)? Doctor: Hi...I went through the history points you have provided -I was expecting that there should have been a history of cow's milk and formula feed (this too contains cow's milk protein) given to the baby. I feel that your baby is having cow's milk protein allergy. Unusually babies grow out of this sort of allergy by 1 year of age.My suggestions for you -1. Mother should go off cow's milk protein completely. This means that you should avoid consumption of anything and everything related to cow's milk - like - milk/ curds/ ghee/ butter milk/ chocolates/ biscuits/ ice creams etc. Even while buying commercial food products, you need to see the ingredients and if they contain milk - do not consume them.2. Feed you baby only exclusive breast feeds till 6 months of age and then start rice based feeds.3. If this is followed scrupulously - the baby will stop having diarrhoea in another 3-4 days and then start gaining weight too.4. If you feel your feeds alone are inadequate for the baby, then Zerolac is the only option till 6 months age.I request you to keep me posted about the recovery of the baby and follow of the case.Regards \u2013 Dr. Sumanth"
},
{
"id": 128281,
"tgt": "Will draining a sprained femur relieve pressure for a ninety year old?",
"src": "Patient: My mom had a fall and sprained her femur badly she is 90 years old. Went to hospital and she was admitted but they will not drain and they are afraid of bacteria, telling the family it will take 6-8 weeks in hospital. I thought draining it would relieve some of the pressure/ Doctor: i would like to know more history , is she able to walk,does she have any fever, does she have thigh swelling,i would like to examine for bony tenderness and look for swelling of the thigh and then decide.based on your description it is difficult to diagnose the condition."
},
{
"id": 43256,
"tgt": "Trying to conceive. Normal results. Taking clomid. Cause of infertility?",
"src": "Patient: hi,I am 28 yrs old and tried to conceive naturally for approx. 6 months(but no luck)..my periods have always been regular with typical 28-29 days cycle. His semen analysis results are perfect too..last month visited a gyno. she put me on clomid 50mg, started medicine on cycle day 3(i.e 22nd Aug'13)for 5 days..ultrasound was done on 31st Aug and 1st Sept. these were d results:day 11th follicle size on right ovary: 17, endo: 5.8day 12th follicle size: 20.8, endo: 7.2this was followed by hcg trigger shot on day 12. experienced tremendous abdominal pain on right side after 24hrs.we even tried on given dates.. but got periods today 17th Sept.why didn't I conceive when every thing seemed soo perfect ? why was I put on clomid without performing all other infertility tests? Doctor: helloplease do not worry and think positively. you will conceive . just have patience and the correct tests and treatment will help you conceive. it is very difficult to give you any advice before looking at all your reports in detail. either you can go back to your gynaecologist and understand the complete thing or you can consult an infertility specialist/gynaecologist at another clinic. u can come to my clinic for advise as well."
},
{
"id": 34364,
"tgt": "Suggest treatment for hepatitis c",
"src": "Patient: My brother 50 year old genotype 1 Hepatitis C probably for about 45 years due to blood transfusion at age 5 for heart surgery. Now has 1+ protein in urine doc says from Hepatitis C should he be treated --looking at Harvoni Thank you in advance for your help. Marian Doctor: Hello,Welcome to HCM,Medical literature shows that antiviral medicines are used to treat long-term (chronic) hepatitis C. These medicines can help prevent the hepatitis C virus from damaging your liver. Sometimes treatment doesn't permanently lower the amount of virus in your blood. But some studies have shown that treatment may still reduce scarring in your liver, which can lower your chances of developing cirrhosis and liver cancer. Further medicines to treat hepatitis C don't work for everyone. Chronic hepatitis C infection is cured or controlled in about half of the people who are treated with a combination of peginterferon and ribavirin. I suggest you discuss this with a gastroenterologist (specialist in liver diseases) and then your doctor will decide what is good for you depending on further testing. Thank you."
},
{
"id": 8970,
"tgt": "Hair removal treatment for children",
"src": "Patient: Hi I want to know is there any hair removal treatment for children Doctor: Usually not advised for children as it is growing age. Or else the same adult treatments can be used but not followd practically as such."
},
{
"id": 32891,
"tgt": "Suggest remedy for itchy and stinging anal region",
"src": "Patient: In the month of Jan 2011 i got severe loose motion due to infection, I consulted a local doctor they had given me medicine for that results stool got very hard for 2 days. And now i have noticed that i got fissure problem, i examined myself and found some tear mark on anal channel and that i have itching, stinting and minor burning pain from last 20 days Please help to get rid of this problem and also please suggest what type of food i can take. And what time period it will take to recover.Devender Roy Thanks Doctor: Hello Due to very hard stool ,there develop laceration and you got tear in the anal canal as well anus opening . This laceration is the main cause of itching and burning sensation .When such case visit I advise my patient to apply lignocaine 2% on effected part . One hour after this apply a combination of ofloxacin+ ornidazole twice in day for 10 -15 days . Take high fiber diet and also take some laxative ( stool softer ) .Still ,no relief within 15 days due to this treatment then consult a surgeon and get his opinion."
},
{
"id": 158900,
"tgt": "Gum-line recession, sore throat, sores in mouth. Is this cancer?",
"src": "Patient: Hi. I am 23 years old. I dipped moist snuff chewing tobacco for almost exactly one year, with frequent use toward the end. I noticed slight gumline recession so I decided to quit. immediately since I quit I have had a very sore throat and a few sores pop up in my mouth. is this normal or could it be cancer? I never had any sores or pain until I quit. Doctor: Hi, Thanks for asking the query, Chewing of tobboco have various hazards on health, it has carcinogenic potential, it causes fibrosis of the oral tissues. So i would suggest you to discontinue the habit. Other than this sore throat and mouth can be vitamin deficiency, anaemia, ulceration, physical and chemical trauma. Direct examination of the condition by Dentist will help in the diagnosis of the condition. Take warm saline gargles, use antiseptic mouthwash gargles twice daily. Take multivitamin tablets. Go for symptomatic treatment of complete mouth scaling and polishing. Hope this helps out. Regards.."
},
{
"id": 129804,
"tgt": "What do multiple bruises on the left leg suggest?",
"src": "Patient: Im 30 years old female in good health. I exersize about 5 times a week. Since yesterday I suddenly have 3 bruises on my left leg. I have problems with anxiety so I worry very quickly. The bruises could come from bumping against my desk at work(this happens sometimes), but 3 seems a bit much. i have no other symptomes. Doctor: Hello!Its possible to get haematoma, when varicose veins blow due to hard exercises, weight lifting and squats.Try to decrease Your sports activities.Heparin based gel can help to remove haematomas faster"
},
{
"id": 133646,
"tgt": "What causes pain on the right side of the body?",
"src": "Patient: I m having severe pain on my right side and sometime it hurts so bad, its like I can t move. I have to like stretch my body out and hold it like or rub real hard on the spot to get it to release the pain. It s sometimes feel like a knot there sometimes and I can t afford go to a doctor or pay for one on line. sorry I have no funds Doctor: hi,thank you for providing the brief history of you.A thorough musculoskeletal assessment is advised.As you feel on the right side of the body pains, it could be related to either a systemic issue or may be neuropathic issue.Getting a thorough neuromuscular assessment is advised post which some simple medication can help further in the recovery.Also, as the body ages, there leads to abundant symptoms which are most of the times due to slow metabolism. Performing regular exercises like general body conditioning should help improve the physical and functional status of your body and get rid of pain and aches.In my clinical practice Certain patient doesn't have any in particular diagnosis but still have this kind of symptoms. I recommend them general body conditioning exercises which has given a majority of difference in their physical and functional status.RegardsJay Indravadan Patel"
},
{
"id": 163643,
"tgt": "What could red blotchy rashes on face and swollen eyes in an infant mean?",
"src": "Patient: Hi, My 4 month old baby has broke out in a red blotchy rash on her face this morning (she had it before but it went after a while) & swollen eyes. The weather has been hotter than usual today but had her with minimal clothing all day. It seems to be itching her & irratating. She normally has red bloches all over her bod but thats due to fair skin. She has also has 4 bouts of diarrhea since yesterday. Although she is taking her feed we. Im weaning her at the moment but havent given her anythign new she could of reacted to. Any advise would be great. Thanks Doctor: Hello,Skin conditions are best diagnosed only after seeing directly. I suggest you upload photographs of the same on this website so that I can guide you scientifically. Hope I have answered your query. Let me know if I can assist you further.Regards,Dr. Sumanth Amperayani"
},
{
"id": 203929,
"tgt": "Irritation, burning sensation, discoloration on penis head after intercourse. What could be the reason?",
"src": "Patient: Hi Dr. Grief, I m a 53 year old white male who had intercourse with a new partner 4 days ago. We used a condom which I have not done in years. The condom came off at some point towards the end of our engagment. I began, within a few minutes, to notice an irritation on the head of my penis and a slight burning sensation. There is some light discoloration in the area as well. I m obviously concerned but logically can t connect a fairly rapid onset with an std . The condom was non- latex (not an animal skin fourex type) so I m thinking (wishfully so) that there is some sort of demitological reaction to the condom. Am I just kidding myself? Doctor: Hi...it is definitely an allergic reaction... either to the condom or to the vaginal fluids of your partner..."
},
{
"id": 168246,
"tgt": "What causes vomitting after food consumption in kids?",
"src": "Patient: Hi...I have a 7 year old son, who over the last two weeks, will sometimes vomit after dinner. He will complain that his tummy is full and it looks to be very bloated and hard. He is completely fine during the day as far as his eating and being able to keep the food down, it is only after dinner. Sometimes the vomiting will last for hours after the first episode. I am very worried that there could be something seriously wrong with him. Any feedback you could give me would be greatly appreciated! Also, plan on scheduling an appt. with his pediatrician today. Doctor: It's due to gastro esophageal reflux and can be treated by giving proton pump inhibitor like lansoprazole 15mg 1 tab daily once for 2 weeks."
},
{
"id": 101092,
"tgt": "Suggest treatment for allergies and asthma",
"src": "Patient: My 8 month old has been mildly conjested for approximately 3 weeks now. It began at the same time as pollen season and when she abuptly was stated on formula from breast milk. There was no fever or breathing issues associated with this and eats ans poos normally. I do hear some wheezing occasionally; howver, one cough and it is clear. She does see her Dr regularly but is not scheduled to be back for another 4 weeks, My thoughts are more allergies than asthma. While I would never deny medical treatmeny my concern is they would treat the allergies with steroids which does concern me. Advice? Doctor: Hello.Thank you for asking at HCM.I went through your daughter's history and understand your concern.I would like to ask a few question to her like - Does she have nose or eye symptoms of allergies? Does she have someone with allergies in family? What type of formula is she receiving? etc.However, from your history, I would like to make following suggestions to her:1. Were I treating her, I would give her montelukast for 1-2 weeks edpending upon response.2. If she has any nose or eye symptoms, I would add an antihistamine drug like hydroxyzine.3. If in spite of above treatment she has wheezing, I would suggest her nebulization with salbutamol after get her examined by a pediatrician. 4. From your history, personally I would like to suggest her to check for food allergies like milk, soy, peanut (depending upon contents of her formula). If allergy testing is positive, that particular food should be avoided.5. I can understand your concern for use of steroids but they are usually not used unless her wheezing is severe or she has repeated episodes of wheezing.6. Please try to avoid exposure to dusts and smokes as much as possible.Hope above suggestions will be helpful to your daughter.Should you have any further query, please feel free to ask at HCM.Thank you & Regards."
},
{
"id": 58925,
"tgt": "On carvidol and remeprazole for liver disease. Do not want to take medication. What to do?",
"src": "Patient: Hi, my grandmother is sick in africa. She has a liver disease and is taking this kind of medication 1.(Essentiale Forte 300mg) 2.Carvidol 3.Remeprazole 20 omeprazole 4.Ginsavit 5.Fusid furosemide 40mgbut, know she does not want to continue the mediation what is the best thing to do?I don't have a credit card can you give us your account number and we will deposit the money. Doctor: Hi!. Thanks for your query. I am sorry to state that she cannot be withdrawn from any of these drugs. Take care."
},
{
"id": 182640,
"tgt": "Suggest remedy for a little flap of gum appearing on the wisdom tooth",
"src": "Patient: hi my wisdom teeth recently just screw in and in there is a little flap of gum on top of it is but its starting to bother me and I'm just wondering what it is it there is no problem or whatever because I brush my teeth regularly and I just want to know if there's a problem Doctor: HelloThe flap is nothing but operculum.as the Tooth come out it will go away but sometimes due to partial eruption it stays over the tooth and causes pericoronitis.In that case you need to get it excised.I would advice you to just wait for some time for the complete eruption of the wisdom tooth.Hope this answer helps.RegardsDr. Shesh"
},
{
"id": 62603,
"tgt": "Suggest treatment for a lump on the breast",
"src": "Patient: hello, am 31 years old a mother of 2 i delivered my 2nd a month ago i am breast feeding plus bottle feeding my baby so am not giving him my breast regularly .. i had breast implant a year ago ( almost ) i felt a lump about 1 cm in my upper inner side of the left breast only .. on exam it is a bit painfull .. shall i worry ? and is there any way to know wether it is a milk cyst or not ? Doctor: hi.it is best if you consult with a doctor, preferably a breast or a general surgeon, for clinical evaluation. based from your description, it could be mastitis or a breast inflammation secondary to breast feeding. it could also be due to a cyst, a fibroma or a fibroadenoma which are all benign breast lesions. other tumor lesions such as malignancy must also be ruled-out. diagnostics such as breast ultrasound and a possible tissue biopsy may also be recommended. management (medical and/or surgical) will be directed accordingly.hope this helps.good day!!~dr.kaye"
},
{
"id": 183527,
"tgt": "How can nerve pain caused during a tooth extraction be treated?",
"src": "Patient: while numbing my mouth for tooth extraction, the dentist hit the nerve 3 times. I experienced 3 electrical shocks. This occured at 12 pm today. My pain medication has worn off and my mouth hurts bad at the back of the mouth at the bottom jaw where the numbing injection sites where the dentist hit my nerve- help Doctor: Thanks for using Health Care Magic.Read your query.For the pain you can take Ibuprofen ( if you are not allergic to any medicine ).Do salt water gargling.Relax and the pain should settle down gradually.If the pain increases or there is swelling or other symptoms please visit your dentist office and have it evaluated.Follow the post extraction instructions .Gently massage the area externally and application of cold compress intermittently will provide relief.Hope this was helpful.Thanks and regards."
},
{
"id": 8537,
"tgt": "How can I become fair?",
"src": "Patient: Dr i am 17 yrs old teenage girl wheatish completion, what cream would you recommend me to become fair? I tried all home remedies but nothing helped me. I used fair and lovely but nothing worked... I have some scars on my face also want to get rid of that... Please help me Doctor: Hello, Thank you for posting on HCM.Complexion of any person is has two two components; constitutive, one which is predetermined by genetics and racial factors and usually cannot be changed with any treatment. The other component is due to external agents like environmental exposure, sunlight, use of any cosmetics etc and is amenable to treatment.You can get a series of chemical peels, which actually remove the upper dead layers of skin and stimulate regeneration of collagen and thus newer layers of skin. Many options are available in chemical peels depending on the requirement and skin type. (Glycolic peel, retinol peel, cocktail peel etc)Also, chemical peeling can be preceded by a simple procedure called microderamabrasion or commonly known as skin polishing to augment the results.Also, at home you can try using cream containing kojic acid, tetrahydrocurcumin, arbutin etc for several months (Melaglow cream) for additional benefit and maintenance.Always use broad spectrum sunscreen before going out in sun.Use a plain moisturiser like cetaphil DAM on daily basis for healthy and hydrated skin.Take plenty of fresh fruits and vegetables in your daily diet.Hope this will take care of your issue.Thank youDr Hardik Pitroda"
},
{
"id": 38170,
"tgt": "Suggest remedy for frequent yeast infections",
"src": "Patient: I have been getting frequent yeast infections after my chemo ended after a full hysterectomy due to ovarian cancer. It is the weekend, so I will have to reach my doctor on Monday. What can a person do for frequent yeast infections that are treated and then come back? Doctor: Hello,Welcome to HCM,As you are having a symptoms suggestive of fungal infection and it is more common in the genitals, as they are the normal comensals of this area which will be kept under control by helpful organisms.Whenever there is a imbalance in the environment it will produces its symptoms.I would suggest you to follow below mentioned measures.1.Keep the are clean and dry2.Topical application of anti fungal cream like Ketcanozole cream3.Oral single dose of Tab Diflucan, 150 mg.4.Oral anti histamines.Thank you."
},
{
"id": 190105,
"tgt": "Lump between the gum and lower lip, painful, having tooth sensitivity. Referred to endontic specialist. Suggestions?",
"src": "Patient: I have a bump on my lower jaw line between my gum and lip...whole right side of my face hurts and I have tooth sensitivity in one tooth near the lump and the upper gums hurt as well. been to dentist who took xrays and said he saw nothing but still referred me to an endontic specialist for further eval..I can feel the lump with my finger and it hurts...feels squishy too Doctor: hiii what you are saying sounds like a carious tooth which is associated to abscess. the lesion is known as periapical abscess it is complication of carious tooth. it requires root canal treatment of the affected tooth....it is advised that you consult a dentist or endodontist-to be precise-for the needfull it could also be mucocele, it is mainly due to trauma to the ducts of accessory salivary glands present at lower lips.....it is bulish in color & would be soft when you press it (palpation)....does this lump disappears automatically after meals....f yes then it s definitely mucocele. consult dentist for physical examination of the lesion & for needful treatment."
},
{
"id": 205092,
"tgt": "What causes olfactory hallucination?",
"src": "Patient: I smell paint, but there is no source... Why do I smell this paint-like smell? I can smell paint where ever I go. I haven t been painting recently, nor been near any paint projects. The thing is, it is always strong and no matter where I go, it stays the same, so I think it s on me. I ve checked for paint and washed myself, but there are no signs of it. It would help if someone could explain this chemical smell. Doctor: Dear userWe understand your concernsI went through your details. Olfactory halucinations or 'Phantosmia' is the medical term for an imaginary odour (phantom smell). Most phantom smells go away in time and are not caused by anything serious. But, for the patient it could cause distress and anxiety.Causes include nasal infection, nasal polyps, migraine, dental problems, exposure to certain chemicals etc. You can talk to your doctor regarding this. But there is no need to be panic.If you require more of my help in this aspect, please use this URL. http://yyyyy.gl/yyyyyyyy. Make sure that you include every minute details possible. Hope this answers your query. Available for further clarifications.Good luck."
},
{
"id": 80530,
"tgt": "What is the cause of tightness in the chest?",
"src": "Patient: Hi I'm Victoria I'm a 19 year old femal. I've been getting a lot of stress. I think I might have some problems with heart or I'm thinking it might just be anxiety but I might be wrong. So when I lay down on my bed or on the couch on the side for long or just for a little while and start moving to anther position to be comfortable there's a lot of tightness or pressure its very sore not painful just extreamly sore. I do experience numbness or soreness on my arms and jaw sorenss and I feel like there's something a lil more on the left side of my throat and my stomach is also sore Doctor: Hello dear, thanks for your question on HCM. I can understand your situation and problem. In my opinion your symptoms are mostly due to anxiety. But better to rule out cardiac cause first. So get done ecg. If ecg is normal then no need to worry for cardiac cause. Anxiety appears more in your case due to stress.Better to consult psychiatrist and get done counselling sessions. Try to identify stressor in your life and start working on its solution. Avoid stress and tension. Be relax and calm. You may need anxiolytic drugs too. Don't worry, you will be alright."
},
{
"id": 77740,
"tgt": "What causes severe chest pain while coughing?",
"src": "Patient: I'm a 30 year old male. I sneezed about 4 days ago and felt a pain in my upper left side, under my arm pit. The pain is still there and never got better. It bother me to lift my arm., and it really hurts when i cough. Last night it started to bruise and this morning got worse. Any suggestions? Doctor: Thanks for your question on Health Care Magic. I can understand your concern. In my opinion, you are mostly having musculoskeletal pain because coughing can cause straining of intercostal muscles and if coughing is vigorous, this can cause chest pain. Another possibility is lower respiratory tract infection (LRTI). LRTI can cause pleurisy (inflammation of pleura). And this can cause chest pain on coughing and sneezing. So first get done chest x ray to rule out LRTI. If chest x ray is normal then no need to worry for LRTI. Musculoskeletal pain is more likely in your case. So avoid movements causing pain. Avoid heavyweight lifting and strenuous exercise. Apply warm water pad on affected areas. Take painkiller and muscle relaxant drugs. Don't worry, you will be alright. But first get done chest x ray to rule LRTI. Hope I have solved your query. I will be happy to help you further. Wish you good health. Thanks."
},
{
"id": 120053,
"tgt": "What causes pain in the legs and knees causing difficulty in walking?",
"src": "Patient: Hi I am a 51 yr male in Chennai India. About 2-3 yrs ago I started having pain in the legs and knees and had difficulty walking. Went through the normal mri etc believing could be age related arthiritis or rheumatism. However all tests are clear and do not indicate same. Now I can barely walk - doctors say muscle deterioration - but cannot figure out the reason - all normal tests done again this week and all clear - have also been undergoing strenous physiotherapy for last 6 months no relief - any suggestions? I feel this could be linked to some kind of infection as I also have a dermatological problem for last 15 years - I get these big boils/ulcers on the foot (fungal/bacterial cause)which then burst or are punctured by a doctor with a needle - and typically antibiotics prescribed - the recurrence of these has increased over past 1-2 years - i feel the two problems could be linked - any suggestions on treatments? any recommendations on good dermatologists in chennai for this kind of problem and/or ortho specialists? PS: I am not diabetic. Since last 2 years am having medication for hypertension (telesartan) and was for a little high cholesterol (atocor). Have stopped anti-cholesterol medication as the phsio felt the muscle weakness and stiffness could be a side-effect of the statins Doctor: Hello, Skin problem ulcers over foot with joint pain in a female should rule out Systemic Lupus Erythematous condition. And also need to think about various autoimmune disorders. Knee joint pain could be due to synovitis, ice pack applications and non steroidal anti inflammatory drugs will give good relief. Hope I have answered your query. Let me know if I can assist you further. Take care Regards, Dr. Krishnakanth. K"
},
{
"id": 6816,
"tgt": "Is it possible for me to conceive during rest of the month other than my fertile days ?",
"src": "Patient: Hi doc. Im 26yrs old married woman, have regular cycle of 30 days. We only take precaution during my fertile days (10 days) and rest of the days we have uprotected sex. Since last one year we have been practicing this method in order to prevent pregnancy . But is it any matter of concern that Im not able to conceive in rest of the month excpet my fertile days because most of the women conceive any time they have unprotected sex? What can be my chances of being fertile or getting pregnant in a month if I have a regular cycle? Doctor: Hi Naina, Welcome to Healthcare Magic Forum, The contraceptive method ur following is safe period method, since ur cycles are regular the unprotected sex during safe period will not make u pregnant, it is the same in other women also whose cycles are regular. But its not always safe some times there are chances of becoming pregnant, better to have unprotected sex for first 7 days of the cycle. If everything is fine u can become pregnant if u have unprotected sex in fertile period in first month itself. Hope I have answered ur question, Regards"
},
{
"id": 77196,
"tgt": "Why are my stomach and lungs hurting?",
"src": "Patient: my sons and i have been sick for about a week. 7 days ago i took an ambulance to the hospitol 3 days in a row i could of swore i had a tear in my stomach then my lungs started hurting now its my back lungs im anxious im waking up with horrible fears somethings wrong i dont feel sick but i never been so scared. please help Doctor: Hi thanks for asking question...Here two possibilities present...First you can have viral infection....It can lead some form of chest pain and muscular pain...Symptomatic treatment done..But here main diagnosis seems is the stress.By excess anxiety panic attack can come and it can lead such symptoms.By stress gastritis or ulcer can be there.Omeprazole taken for it....If still symptoms worsen chest x ray and USG adome done for further work up.Hope your concern solved...Take care...."
},
{
"id": 163795,
"tgt": "How can fever with vomiting and diarrhea be treated?",
"src": "Patient: Sir, my son 6yrs having fever from 27th and we started paracetemol p 250 and Taxim O from 29th, now my son fever is 98-99 sometimes. He is now having vomiting tendency and gone to toilet 6 times today. His urine and previous stool was quiet Ok. Sir please advice what to do, will I stop giving Taxim-O and wait for the best. Plz advice Doctor: Hi...Thank you for consulting in Health Care magic.UNNECESSARY USAGE OF ANTIBIOTICS LIKE TAXIM-O MAY CAUSE ANTIBIOTIC ASSOCIATED DIARRHOEA.Fever of few days without any localizing signs could as well a viral illness. Usually rather than fever, what is more important is the activity of the child, in between 2 fever episodes on the same day. If the kid is active and playing around when there is no fever, it is probably viral illness and it doesn't require antibiotics at all. Once viral fever comes it will there for 4-7 days. So do not worry about duration if the kid is active.Regards - Dr. Sumanth Amperayani"
},
{
"id": 119405,
"tgt": "Suggest remedy for tendonitis",
"src": "Patient: I have a fat nobular inside my upper left arm, just below the shoulder. I fear it is irritating my nerves, as my whole arm is weak and in a lot of pain. Secondly, I don t know if there s a connection but I got my hand slammed in a car door almost nine months ago - I was diagnosed as having Tendonitis, then a minor ulner. What to do? Doctor: Hello,It is important consulting with a neurologist for a physical exam and performing nerve conduction tests in order to examine your nerves. If this test shows normal no pinched nerves, you should consider tendinitis as a possible cause.Take care. Hope I have answered your question. Let me know if I can assist you further. Regards, Dr. Ilir Sharka, Cardiologist"
},
{
"id": 117411,
"tgt": "How to treat an increasing spleen 14.5 to 16.1?",
"src": "Patient: my spleen is increasing 14.5 to 16.1 since last two years. before three years i was suffered with harpies roster. my alkaline phosphate is 186. no any tomour is seen in USG AND CT scan. the white blood test WBC 3500.. some time itching tendencies over abdomen. on right side of abdominal (MIDDLE) varicous vein are seen and swelling like coin at the side. i get cough and cold easily. please digonest and let me know which test is required. Shambhu nath singh age 57 years. Doctor: Hi, I have gone through your question. I can understand your concern. your spleen size is increasing. now there are many causes for that. but as you have no tumour according to your usg report. you have prominent vein on abdomen. so you may have portal hypertension due to any reason. Which leads to increase the spleen size. Thare are many sites for portocaval anastomosis. One of which is around umbilicus. So you need to consult physician. Investigate for that and take treatment accordingly. hope i have answered your question, if you have doubt then I will be happy to answer. Thanks for using health care magic. wish you a very good health."
},
{
"id": 57424,
"tgt": "High bilirubin level. Diagnosed with NASH. LFT, liver biopsy, Hepatitis, CT scan all normal",
"src": "Patient: Ihave all report are LFT, Liver Biopsy coppar pepar,Hep a,b,c,e, Endoscopy , CT Scan ,Normal But Last Five Yeras Bilirubin Is veryhigh 3, 2.5, 4, 3.2 but no any problem I consult Gestroentrology His Daignos NAHS. i have weight 5 kg at time Please Help and your any special Advise Or Treatment ,Futhur in any Problem? Regards Sachin M.0000 Doctor: Hi thank you asking HCM NASH is non alcoholic steato hepatitis. If you have high bilirubin and high liver enzymes it may be related with NASH . Treatment of NASH is diet reached in fibers, increase physical activity and medical treatment for high cholesterol or TG.wishing you all the best dr.Klerida"
},
{
"id": 170063,
"tgt": "How to cure itchiness on the buttocks area?",
"src": "Patient: hi doc, i have an 8yr old daughter and she have this skin problem in her butt, she was seen by her pedia and was given a cream cutivate . It s like eczema and my daughter says it s so itchy sometimes. There are times that it s ok but again after 1 or 2 days my daughter compains again. We went to a dermatologist she was given a soap and a lotion but still no improvement happen. It is like a chicken skin but itchy. Please help Doctor: Hi,It seems that she might have worm infestation, worms coming out on anal region producing itching.Go for stool test for ova and cyst.After report go for anti-worm treatment.Apply triple action cream locally.Ok and take care."
},
{
"id": 30533,
"tgt": "Suggest remedy for lung infection post an open heart surgery",
"src": "Patient: Sir, My 15 months old daughter had a open heart surgery. By the grace of the god, operation finished successfully.. Every thing was going on well. But Today doctor said that her condition is critical due to infection in lungs, she is still on Ventilator, her BP & Platelets are down. We are trying our best What to do..? Please advice Doctor: hi, I read your daughter's history and i can understand your concern.as your daughter is too small, they wont be having good enough immunity to fight against all diseases. heart surgery is the major surgery and because of hospital admission, chances of hospital acuired infection will be more and can lead to sepsis. sepsis may be the reason for it.thank you take care"
},
{
"id": 22770,
"tgt": "What causes chest pain when suffering from low BP?",
"src": "Patient: I am 25 year old female smoker. I have a low blood pressure (i've been told) but suddenly this morning i started experiencing a sharp pain in my chest. I guess it was behind my breasts, but i felt and ache and it's not gone away (been over an hour). What is this?? Doctor: Hias you are a smoker the pain you are experiencing most like whats medically termed as angina,Angina is chest pain or discomfort caused when your heart muscle doesn't get enough oxygen-rich blood. It may feel like pressure or squeezing in your chest. The discomfort also can occur in your shoulders, arms, neck, jaw, or back.It is a symptom of an underlying heart problem, usually coronary heart disease (CHD)i would suggest youEKG (Electrocardiogram)Stress Testing2D echoBlood Testsall these test will rule out the blood supply to the heartChest X-Rays"
},
{
"id": 203991,
"tgt": "Will frequent masturbation affect my health and future sex life?",
"src": "Patient: hi doctor, i m 26 years male.. i m doing daily hand job is it bad for my health and or effect my future sex with partner.. sometimes after handjob i feel headache. also my mentality feels vulgar mostly is this will make any mental caution for me. many thank. Doctor: hi masterbation is a natural process in teenage people.i think it does not effect the health and future life but the anxiety associated may effect ur health.even then do not masterbate more than 3 times a day.masterbation is only indicate of production of sexual hormones in male."
},
{
"id": 158705,
"tgt": "Pain in mouth. Hole with white patches beside. Could it be oral cancer?",
"src": "Patient: Hi. Recently i felt pain in e right side of my mouth. Initially i thought it was ulcers that caused the pain because i saw white spots. After 1 week with medication it has not recover. I went to look at the mirror again with a torchlight & realised that it is a hole in the mouth with white patches beside it. Could it be oral cancer? Please advise. Doctor: Hello Estene, Thanks for posting, Well, thinking of cancer will depend more on your age ( anything is possible when it comes to the human body) as a more advanced age will more likely be cancerous or precancerous lesions. From your description, I do not think it is cancer-related lesion and I suggest you get yourself examined by a physician who can look at it closer and decide what to do. It could be fungi (candidiasis) and may just require antifungal medication like fluconazole. If you are in doubt, just consult a doctor. Hope this helps"
},
{
"id": 131907,
"tgt": "Is it advisable to undergo surgery for fracture after Methamphetamine intake?",
"src": "Patient: Hello,At 7.45pm on Thursday 20th march 2014 I had methamphetamine (crack pipe). I have badly dislocated my finger during netball & Have been in hospital all nite & morning. they have now told me I need to go to another hospital @ 9am friday 21st march 2014 which may have them needing to operate on the fracture. What do I do? I am feeling absolutely sick in my stomach with fear. Can they undergo the surgery giving it would of only been 13 hrs of last having it? Doctor: Hi you are worried about a second surgery in your finger within 13 hrs. In my opinion to get the best results it is better to undergo operation as early as possible. If 2nd operation is required , I am sure they would do it after some delay. Metamphetamine effect would have worn out by then."
},
{
"id": 156647,
"tgt": "Is there any possibility that I may have contracted cancer from the use of a repulsive humidifier?",
"src": "Patient: Hi my name is Mike--I was just diagnosed with tonsil cancer-I have been using a humidifier that has been in my bedroom for probably !0 yrs without canging the filter----it is covered with mold etc-Is there any possibility that I may have contracted cancer from the use of the repulsive humidifier? Doctor: No, there is no such data to support tonsillar cancer causation secondary to use of mould infested filter in humidifier.Tonsil cancer association is more with cigarette smoking and alcohol use."
},
{
"id": 113682,
"tgt": "Back pain laying flat down, lower adominal pain, headaches and nausea. Reasons ?",
"src": "Patient: Hi, Ive been having back pains laying flat down, and lower adominal pains, headaches almost everyday and a nauses feeling in my throat .. I had sex july 22, Started having bleeding which felt like my period, Auguest 4th, was very light, lasted about 3-4 days, In July i got my period around the 14th, I have no idea what s going on??Please help! Doctor: Magnetotherapy treatment;- For your back pain u can place south pole of med. power magnet for fifteen on the pain place.and drink magnetic water thrice a day and report."
},
{
"id": 21050,
"tgt": "What causes low blood pressure?",
"src": "Patient: I have brought my daughter to the er for a migraine. They gave her compazine and benedryl. Cat scan and blood work are clear. Her blood pressure has dropped severly. Running 90/33 and 101/29. The dr. Isn't concerned about the bp but 29 seems awfully low doesn't it? Doctor: Hello.Ma'am it's natural on your part to be concerned but please don't focus on numbers.How old is your child? if she is not comfortable then this should be investigated and treated but in case she is feeling alright then there's no need to treat the numbers."
},
{
"id": 1792,
"tgt": "How can i get pregnant while suffering from irregular periods?",
"src": "Patient: i m having irregular periods and i m suffering from pcod. my age is 21. i m using glyciphase tablets from 5 months. now iam getting periods for every 30 to 35 days. i m using fertyl 50 from the 5th day of my period from 3 months, but i m not pregnant. pls tell me when will i ovulate and which day me and my husband should intercourse. almost everyday we are in sex but i am not getting pregnancy. please help me Doctor: Hi, you should be in contact with your husband every 2 to 3 days, not everyday. You can track your follicles growth by repeated ultrasound and when your follicles is more than 17 mm, take injection for rupturing the follicles. It will make sure that you ovulate. Take progesterone for next 2 weeks. Do a urine pregnancy test at home after that. Hope I have answered your question. If you have any other query I will be happy to help.Regards Dr khushboo"
},
{
"id": 59914,
"tgt": "Negative liver tests, taking ayurvedic treatment for thyroid. Liver problems?",
"src": "Patient: My husband had his regular blood test done yesterday. Strangely, his liver tests turned out to be negative. The data is as below: LIVER PROFILE BILIRUBIN , TOTAL 0.6 mg/dL ALANINE AMINOTRANSFERASE ( SGPT ) H 122 U/L ASPARTATE AMINOTRANSFERASE( SGOT ) H 64 U/L He also is undergoing ayurvedic treatment for thyroid. Can you please suggest the possible liver related problems? Or if he needs to undergo any further tests? Thanks and Regards. Doctor: Hi, Welcome to HCM, I am Dr. Das The liver profile show increased SGOT and SGPT which is mainly caused by hepatocellular insult by any drug or virus. take Liv 52 2 tab thrice daily for one month."
},
{
"id": 139040,
"tgt": "Suggest treatment for severe hand pain",
"src": "Patient: I had trapeziectomy on my right hand in 2006 and on my left hand in 2008. My hands were doing fine, but few years ago, my both hand started hurting. Now they hurt too much. Is there any operation to take care these pain? I am 74 year old woman. Thanks. Doctor: Hi, I value your concern regarding the symptoms. I have gone through your symptoms, and in my opinion there certainly is methods to get you rid of pain may be at cost of some loss of mobility, but for that you need to get your X rays done and send us so that we can decide the cause of your pain first. the treatment then followsHope this answers your question. If you have additional questions or follow up questions then please do not hesitate in writing to us. I will be happy to answer your questions. Wishing you good health.Special note- Any medication prescribed needs to be taken after consultation with your personal doctor only."
},
{
"id": 178691,
"tgt": "What causes dark colored face and light colored body in a child?",
"src": "Patient: Hello, I just wanted to know about my daughter s eventual complexion. She is just 2.5 weeks old and her entire body is fair except her face, which is slightly tan. Her mother is fair and I myself being a father medium light color. Please help answer if you could? Doctor: Thank you for the concern.Congratulations for becoming a proud father of a beautiful daughter.Today is daughters day and you are blessedAt this time your only concern shall be her health, development, feeding habits and timely immunization.As far as color is concerned she will keep changing every day, week, month and years to come. When she will need education, physical activity lots of love and encouragement irrespective of her color.Concentrate your energies in providing this and developing her in to a beautiful person at heart, this will make her most beautiful and loving and caring daughter in the world."
},
{
"id": 168446,
"tgt": "Is it normal to pass stool 8 times a day in 6 month baby?",
"src": "Patient: Hello, I would like to know normal process of 6 month old baby s stool cycle...My daughter is passing stool almost 6 to 8 times in a day...Starting the day with the perfect cleaning, even after every meal - feeding, she is passing stool a little amount, but almost 6 to 8 times... Is this cycle is ok...Or do I need to have a check with doctor for her... Thanks in advance for your feedback. Regards Ayyappan.V Doctor: Hi, the average stool frequency at this age is 4-5 times per day, however in a breastfeeding baby 8 to 10 times passing stool is not uncommon. More important is the nature of the stool, whether too hard or watery. You can compare stool consistency by matching with Bristol stool chart( search in google). If the stool consistency is not normal you should visit your doctor.Moreover good appetite and gaining normal weight ( at 6 months her weight should be double her birth weight) is also an indicator of normal digestion and absorption. At this age you should take advice from your pediatrician regarding starting of complementary feeding. Thanks"
},
{
"id": 10775,
"tgt": "What causes hair loss along with receding hair line?",
"src": "Patient: I am a 33 year old female with hair loss along the receding hair line of the scalp. In the past i have used hair color. Could this be a contributing factor, or should i maybe have blood work for thyroid tests? I also have anxiety and it has increased severity with my age progression. Doctor: HiThank you for asking HCMI have gone through your query. If you provide some more information then it will be more helpful. How is the pattern of hair loss? Is it more on crown and frontal area? Do you have increased facial hair growth?If yes then it can be most likely female pattern hair loss. In such case using minoxidil solution 2 % for long term use on scalp and capixyl gel will be helpful. You can also use ketoconazole and zinc pyrithione shampoo twice weekly. It is better to rule out hypothyroidism and anaemia by doing blood test - thyroid profile and haemoglobin levels respectively.Hope this may help you. Let me know if anything not clear.Thanks."
},
{
"id": 123471,
"tgt": "What causes recurring swelling in feet?",
"src": "Patient: I am curious why my feet get swollen every now and again, though i do regular exercise, am not overweight and it happens like 4 times a week. They are often okay when i wake up in the morning. I work from home and try to keep my feet elevated and move around regularly. Doctor: Hello, I think you should get the scanning for the kidney and also the heart done. Due to reduced venous return, the swelling can be there. There is even the possibility of the kidney malfunction leading to swelling. Until we check the physiological status of the heart and kidney we cannot give you an accurate cause for it. As with respect to swelling, there are multiple factors associated like heart conditions, kidney conditions, diabetes, etc. Until we have some clinical platform and lab reports it will not be fair on our part to make you panic. Get things examined by the laboratory so we can assist you better. Hope I have answered your query. Let me know if I can assist you further. Regards, Jay Indravadan Patel, Physical Therapist or Physiotherapist"
},
{
"id": 134480,
"tgt": "Suggest remedy for severe tailbone pain",
"src": "Patient: I recently did something stupid with my grandkids. I took a ride down the stairs and my tailbone has been killing me. The last few days it has started really hurting in a in a v shape on both hips,lower back. Is anything I can do for the pain until I call my doctor next week Doctor: As you alluded to, you appear to have bruised your tailbone, or \"coccyx.\" The first thing to do is to apply ice, in this case by sitting on an ice pack, 20 minutes on, 20 minutes off, as often as you can. This will reduce inflammation, and will help numb the pain. Additionally, topical anti-inflammatory creams, such as Aspercreme, will help with pain. Finally, Naproxen/Alleve by mouth twice per day with food can help greatly with inflammation and pain. You will also need to find a good cushion or pillow to sit on for cushioning."
},
{
"id": 195290,
"tgt": "What causes penile itching after having protected sex?",
"src": "Patient: I had a sex with unknown women before 9 years, but i 've used condom. After that incidence I am having pennis itching and some times mild pain.I've checked with doctor, who has examined me for several tests and confirmed that I don't have any problem. I was given anti fungal treatment. But still I am having problem since the last nine years.But what I've seen is after this incidence my wife and kids are gone weak and very frequently they are becoming sick.I am confused that they are getting sick due to my problem. Can it happen so?Pls.advise. Doctor: Hello and Welcome to \u2018Ask A Doctor\u2019 service. I have reviewed your query and here is my advice. As per your history, that itching may be due to allergy to condoms. There is no relation of getting your wife and child becomes weak and this. Kindly use latex free condoms if possible after dermatologist consultation. Take care. Hope I have answered your query. Let me know if I can assist you further. Regards, Dr. Shyam B. Kale"
},
{
"id": 44503,
"tgt": "Undergone 2 cycles of IUI, treatment going on. Reason for infertility - cystic and bulky ovaries?",
"src": "Patient: hi i m 26 yrs old and have completed 6 years for marriage...........................have done a lot of medications ....................in 6 years...................also done 2 cycles of IUI ......................now also my treatment is going on.........................my follicular study is going on..........................today is my 12th day And the size of my Right Ovary is 18*20 & left Ovary is 16*16 & today my doctor will be giving me one injection to rupture the egg.................so that he can do my 3rd cycle of IUI..........................when the egg should get Rupture?????.............one more thing that my both the ovaries r cystic & bulky so m i having the problem of infertility b coz of cystic & bulky ovary or some other reason.....................pl.suggest me something Doctor: the eggs should get ruptured usually on the 14th day of menstrual cycle. yes polycystic ovaries will lead to infertility. dont worry continue with your treatment."
},
{
"id": 206218,
"tgt": "Suggest treatment for depression",
"src": "Patient: I'm 23 young girl who just feels down a lot no matter what anyone does for me I just take it out on them...I dnt feel good about myself at all always tryin to change my apperience...I lost my father when I was 7 and my mum has a learning disability .. It was difficult grown up but I had a great family support ..I'm an only child so I had to take on a big responsibility with my mam... I've always done everything for here but she doesn't get me at all getting older I realised how different we where she would lie so much about money and always put men in her life before me. I know that she does try her best but we just dnt get along any more.. I was in a relationship for 4 year and I was cheated on and lied to he just made me feel like i wasn't good enough made me feel horrible about myself and I'd cry every day and got so paranoid ...I became angry and would lash out and take it out on every one...eventually I got out of that relationship met the fella of my dreams who has dne everything he can for me ...but I still can't be happy I'm still down I've lied over stupid things which are big to him and dnt treat him nice at all...I always complain I always fight with him I just feel like I can never be happy with me life I'm just lost.. my relationship with my mam has gotting so bad I feel like she hates me and talk to me I put her dwn I put my boyfriend down I don't want to be this person anymore I know who I am or wat I want in life but I'm angry feel depressed and most days I'd feel like they would all be happy if I wasn't here...dnt know wen I became this angry person who hates herself hates life and treats the people she loves so badly I know it's not right I've been unhappy for awhile ...just dnt know what to do or how to come out of this black whole I'm buried in plzzzz help Doctor: DearWe understand your concernsI went through your details. I suggest you not to worry much. The symptoms you provide here do correspond with depression. But you should know that even disappointment in life circumstances can have depressive effects. These are mental disorders and not diseases. These are mental conditions rather, manifested itself out of wrong perceptions and stimuli. Numerous researchers and researches emphasise againn and again on the fact that mental disorders cannot be cured only with the help of psychiatric medicines. You need to alter your life style so as to suit the requirement, change your thinking patters and maintain positive body metabolism. Psychotherapy techniques, yoga, meditation etc should help you achieve these requirements. A trained psychologist / psychotherapist can help you in this aspect. If you still need my help, please describe the whole problem in detail and post a direct question to me. I shall definitely help you with psychotherapy techniques to over come your problems.Hope this answers your query. Available for further clarifications.Good luck."
},
{
"id": 39213,
"tgt": "Suggest remedy for infection on wound",
"src": "Patient: sir/mam i fell on the road(street) while running on the friday past.and thus developed a gash i washed the wound with anti microbial liquid and applied some antiseptic cream however since yesterday there has been some microbial growth(i am guessing) on the wound, it is yellowish and all over the wound.is there some danger Doctor: Hi,If I was your treating physician I would prescribe you a course of Antibiotics (like amoxicillin thrice a day) for 5 days.Also I think you might need to get wound debridement (cleaning) and dressing done fresh everyday by a qualified health care provider for a couple of days depending on the size of the wound, your age and general health.I am guessing you are young and male so 5 days would be sufficient.And the next time you fall on the road don't forget to get a Tetanus Shot.If you follow as advised the danger is minimal"
},
{
"id": 112313,
"tgt": "Back pain, sore leg, feels going toilet again and again in night, hard to sleep. Symptoms combined or different injuries?",
"src": "Patient: Hi, ive had a lower back pain since about six months ago it switches sides and is a shooting pain feeling somedays it hurts more than others and sometimes it hurts when i bend or even sit, adding to that now i have a sore legg /hip which is also a shooting pain when i put it up to the side or kick it up and i can feel it going over anothr bone i also pee alot especially during night in the space of 15 minuits i can be in the bathroom 7 to 8 times i always feel like i need when there is barely anything there it is hard to get to sleep. What could this be ?are these symptomes combined or different injurys? Doctor: I think you get examined by a spine doctor/orthopedist. Shooting pains down the lower limb along with back pain is often due to disc problems. and might require an MRI to see the fault. In meantime take hard bed, avoid forward bends,and avoid lifting weighty things, do back fomentation and lie in comfortable posture which eases off pain."
},
{
"id": 84743,
"tgt": "What is causing uneasiness in abdomen after taking MT pills in 38 days pregnancy?",
"src": "Patient: Madam my wife has taken MT pill today at 8 am in the morning due to 38 days pregnancy her date of period was 2nd and she missed the period, but after taking MT pill she is not having the period(bleeding) till now. she is suffering from headache and un easy feeling in upper abdoment can you suggest something? Doctor: Hello, Possible causes like retained prof conception must be rule out. Consult a gynecologist and plan for an ultrasound scan. If there are retained products, it must be evacuated manually. Hope I have answered your query. Let me know if I can assist you further. Take care Regards, Dr Shinas Hussain, General & Family Physician"
},
{
"id": 75099,
"tgt": "What causes swelling of neck with vomiting?",
"src": "Patient: my mother is suffereing with sweeling on her neck doctor tested and told that its a tuberculosis ,and they reffered daily two tablets of Forecox,after taking tha two tablets she is vomiting two/three times daily from past two days. please help in this regard what to be done?? shes is suffering with diabetes also from past 3 years. Doctor: hi. .ATT side effect is very common .. first ask your doctor regarding dose of ATT drugs ...take one pantopronozole 40 mg tablet before food (30mints) take ATT after 2 hr of food in early morning .. now pt have to stop ATT and do liver function test .."
},
{
"id": 83980,
"tgt": "What are the side effects of zotreem plus?",
"src": "Patient: what are the side effects of zetroueem plus; slimming capsule; i have just started using it 3 days ago but i have very fast heart beats in addition to low blood pressure....does it have any side effects? and if so please mention it as i need to lose about 25 KG. thanks a lot Doctor: Hello,Diet pills should always be taken in low doses & short term use is the best. You should exercise regularly, do walking, stretching, avoid trans fat, oily, spicy junk food, refined flour, salty and sugary food items. Take more of whole cereals, vegetable protein, fruits and veggies. Take small meals & adequate sleep.Some diet pills cause heart palpitations & rapid breathing as they contain caffeine, guarana or amphetamine based ingredients. Diet pills reduce your appetite and increase your metabolism & this may trigger tachycardia if you have history of some heart ailment.Low blood pressure may be caused by dehydration, Stay well hydrated. Avoid heavy work & take rest between work. Diet pills reduce fat absorption in the body thus interfering with normal metabolism & causing symptoms like dehydration, bloating, nausea, anxiety, etc.You are suggested to report your symptoms to your treating doctor so that the doses you are taking can be monitored(tapered) or medicine replaced by some other pills that suits you better. Get a heart check up done to stay on the safer side. Get lipid function tests, EKG, stress tests done.Hope I have answered your question. Let me know if I can assist you further. Regards, Dr. Nupur K, General & Family Physician"
},
{
"id": 57567,
"tgt": "Should i be worried of pain on right side above waist and dark colored urine after gall bladder removal?",
"src": "Patient: Hi Dr, I have surgeory about 10 days ago to remove my gallbladder all seems ok with the normal pain, gas ,,etc,, I have been drinking water to flash the sedative from my system but today i establishied two thing, pain on my right side just above my wasite and urine is dark? why I am scared Doctor: If I was your surgeon, I'll be thinking:would be helpful to know: are you male/female? age? fevers? Are you jaundiced (yellowish skin/eyes)?So I'm thinking:- you're probably on pain medication? so you may be constipated if you're not having stools (pain meds cause constipation)- Dark urine more like dark yellowish may be due to high bilirubin which can be cause by a leakage along the biliary ductal system or injury. If the pain & dark urine persists, definitely go back and see your surgeon.Hope this helps-"
},
{
"id": 53156,
"tgt": "How to cure grown septated anechoic mass in liver?",
"src": "Patient: my liver is with homogenous parenchymal echogenecity. a septated anechoic mass is seen at segment 8 of the right hepatic lobe measuring 3.1 cm in diameter. what shall i do in order for me to stop this from growing or to get rid of it? thanks and GOD bless. Doctor: Hi welcome to the health care magic For the liver mass, you can be investigated with triphasic CT scan and AFP estimation for exclude malignancy... If mass is suspicious and doesn't look Benign than biopsy needed for histopathology examination... According to cause detected further specific management planned Consult surgeon for examination and further work up accordingly Take care Hope your concern solved"
},
{
"id": 216590,
"tgt": "What causes severe pain in right upper breast area?",
"src": "Patient: I have severe pain under my right upper breast area and the drs cant find anything really wrong with me I have had several test and there normal I was prescribed a pain med but it isn t helping I am in a lot of pain its hard to move and difficult breathing hard, Doctor: the normal investigation have ruled out possibility of any medical condition involving organ.This warrants suspicion regarding a psychological pain disorder.I suggest consulting a psychiatrist is advisable."
},
{
"id": 112926,
"tgt": "Have chronic back pain. Took MRI scan. Have pinching pain and on medication. What is the remedy?",
"src": "Patient: i had a chronic back pain. I had a recent mri scan and below are the results..posterior annular tear at l5-s1 intervertebral disc.l5-s1 posterocentral subligamentous disc extrusion indenting anterior thecal sac with minimal caudal migration.relative spinal comprimise at l5-s1.my dr has asked me to take a complete bed rest for 3 weeks and suggested few pain relievers. i ahave a pinching pain withno swelling at the back....Could you f\\guys pls share your opinions/suggestionsin this regrd..thanks in advance.. Doctor: Hello and Welcome to HCM, Thanks for writing to us.Your MRI report suggestive of nerve root compression by extruded disc at the level of L5-S1.For this condition you have to do stretching & strengthening exercises in which you have to stretch tight muscle & strengthen weakened ones which will alleviate stress on facet joint and disc.There are several treatment modalities with medicines, physiotherapy and with injections in the back called epidural injections and root/facet blocks.Hope this helps you.Wishing you good health...Regards. Dr Saurabh Gupta.Orthopaedic Surgeon."
},
{
"id": 132825,
"tgt": "How to treat pain in calf muscle?",
"src": "Patient: very bad pain in calf muscle was working nights on Tuesday and the pain just started, did not feel as though I pulled or jerked it, still very sore and its now Monday, usually I can walk 3 to 4 miles each day but cannot now had veins on both legs done last year Doctor: Hello,\u00a0I have studied your case.\u00a0 Calf Muscle cramps\u00a0can be reduced byMassaging the cramped muscle with your hands or oilDrinking plenty of fluids to avoid dehydrationStretching\u00a0your leg muscles or riding a stationary cycle.Taking\u00a0diet\u00a0rich in calcium and potassium or oral\u00a0supplements\u00a0of the sameCheck your vit B12 and Vit D 3 levels.Till time, avoid lifting weights, You can consult\u00a0physiotherapist\u00a0for help.Physiotherapy like ultrasound and interferential therapy will give quick relief.If no relief then you can consult your doctor.Hope this answers your query. If you have additional questions or follow up queries then please do not hesitate in writing to us. I will be happy to answer your queries.\u00a0Wishing you good health.Take care"
},
{
"id": 222548,
"tgt": "What causes light bleeding and abdominal pressure during pregnancy?",
"src": "Patient: Hi, may I answer your health qu Hi, I am in my 6th week of pregnancy and last nigth when i got home from work i had a a bleed , not enough to fill a liner , but it as dark blood, i had no pain , just a small amount of pressure in my lower abdomen? bleed had stopped and now brown blood when i wipe? Doctor: Hello dear,I understand your concern.In my opinion bleeding along with abdominal discomfort or pain needs to be investigated.The causes like any thyroid abnormalities,progesterone insufficiency,any abnormal pregnancy like miscarriage or ectopic pregnancy,any cervical bleeding due to erosion,varicosities,any excess physical strain should be ruled out.Avoid physical strain and intercourse.Take strict bed rest.Ultrasound helps in knowing the status of pregnancy.Best regards..."
},
{
"id": 126949,
"tgt": "What causes numbness and swelling in the feet?",
"src": "Patient: My foot fell asleep ..I didn\u2019t not realize and when I stepped down my ankle rolled, I heard several crinkling crunching sounds.it bruised and swelled a bit. I can move it . I wrapped with ace bandage Took Advil,and iced. I can Wally on it but some pain Doctor: Hi, You seem to have allowed continuous pressure on a prominent nerve at the back of the thigh, probably by sitting on a hard edge or sitting with the knee bent for a long period of time.This has caused the foot to go asleep. The crunching noises you heard most probably indicate ligament strain or tear.This can also mean a fracture of the ankle, if you have weak bones.Your first aid treatment is fine.However if its not improving in 3 days, then you will need an X-ray to check for a fracture. Hope I have answered your query. Let me know if I can assist you further."
},
{
"id": 91020,
"tgt": "Suggest treatment for chronic pancreatitis with calcification in the CBD",
"src": "Patient: Dear Doctor, My father in law got an ultrasound done recently as he had acute pain in his abdomen and back, which said that he had chronic pancreatitis with calcification in the CBD, he also has a problem of passing frequent abnormal stools since the last two days, which has somewhat been relieved since yesterday as i gave him three doses of creon 25000 units, please suggest what should we do now. Doctor: HI. A proper history, clinical examination, investigations as per these findings alone can lead to a proper diagnosis . This is most important to lead to a proper management/ treatment.You have to consult a Gastroenterologist who can perform ERCP- remove the stones and pass a stent...This is therapeutic and helps a lot. If he is taking alcohol, he has to stop ..Continue creon with food. He also has to avoid all foods and beverages , which he must have observed that they increase the problems."
},
{
"id": 201235,
"tgt": "What causes azoospermia ?",
"src": "Patient: Hello Doctor, I am 30 yrs old and detected with azoospermia, I do not know the root cause yet, however, i never had any problem with intercourse, erection and ejaculation. I have strong sexdrive, so on the days i do not do sex with my wife I masterbate. I do sometime feel pain in my right ball (testis) after sex or masterbation (may be 2 times a month) from past 2 yrs. I am also hypothyroid taking does of 50 since last year. However, i do not know when thyroid problem started. I am not sure that above information is how much resourcefull but if possible can u suggest me what kind azoospermia i might be having and can it be curable? Also i have semen test only once and As the sample collection was in room at test center and i was not comfertable, so would it a chance that i didnot ejaculate sperm with semen. Also my semen repert says. viscosity high and liquification: incomple.... pls help with suggestions. Thanks Deep Doctor: Hi,As you got complete absence of sperms in the semen, there might be having few possibilities,There might be having atrophy of testicles might be due to injury in testicles or h/o mumps in the past.There might be having blockade of tubes carrying sperms.Consult urologist and get examined.Ultra sound and biopsy might required.Ok and take care."
},
{
"id": 122542,
"tgt": "Please suggest treatment for knee injury",
"src": "Patient: I had a fall 5 weeks ago & my knee cut open due to force of the fall. The wound is healing well though I probably should have had stitches & would be healed completely by now. I am not feeling much pain but still some stiffness when I sit for long periods. My knee cap is deformed looking though with a big bump on the tip of knee cap. Should I be concerned about any long term effects? Doctor: Hello, As per your history, it may be due to blunt trauma. For further assessment you may require MRI knee joint after orthopedician consultation. For pain you can take tablet acetaminophen. Heating pads also helps. You may require aspiration in case of swelling.Use knee cap. Use support while walking Hope I have answered your query. Let me know if I can assist you further. Regards, Dr. Shyam B. Kale, General & Family Physician"
},
{
"id": 159897,
"tgt": "Is having jaundice for a prolonged time,vomitting and insomnia symtoms of stomach cancer ?",
"src": "Patient: My mother is suffering from Jaundish for last three months. She is under tratment of one MD and now her billirubin comes down from 12 to 8. But till now she caould not take food properly as she vomits after taking food & being sick day by day. She is suffering from insomnia also for more than six months. She sleeps maximum 3 hours at night. The doctor suspecting for stomach cancer . We want another openion from other doctors. I reside in Kolkata, India. Please suggest me how to continue the treatment. Doctor: Hi, Thanks for query, Consult Gastro enterologist for expert opinion. He will go for both the problem,jaundice and suspected stomach cancer. Ok and bye."
},
{
"id": 217770,
"tgt": "What could cause the severe pain below the knee?",
"src": "Patient: I am 71 yr old male, still very active. My left leg below the knee along the boneand before the ankle has been giving me a sort of electric sharp pain . This starts and stops fequently. the skin is very sensetive but no discolor.This condition started out very mildly , but has become much worse. This morning I woke up with the pain ans it keeps reacurring every few min. Do you have any idea what can be the cause. Hopefully yours Ronald Filera. email YYYY@YYYY . Tel 845-215-5007. Cell 914-403 7630. Doctor: Respected sir, nice description but sad to say ots difficult to pin point this type of pain without examining the patient. There could be a lot of reasons for this. 1. Osteoarthritis of the proximal and distal joint.2. Varicose veins3. Arterial insufficiency.4. Diabetic neuropathy.So, i sincerely suggest you to kindly consult your nearest geriatric physician."
},
{
"id": 8657,
"tgt": "Anteoderma, started from small spot on chest and is getting bigger. Cure for symptoms?",
"src": "Patient: hello My name is Luiza ,i,m 23 years old and profesional fotomodel I born in Europe ,now living in Canada .I have problem my skin ,i think is anetoderma .Please let my know if is any cures ,or surgical treatment to stop this disease . When i was 12 years old start from small spot on my chest ,but now is biger and biger . Please answer me ,im very ,very scary. Please Thank You Luiza from Calgary Doctor: Hi Luiza, Thanks for writing to us. Anetoderma is an elastolytic disorder where elastin of dermal skin tissue is destroyed leading to laxity and outpouching of skin. Though medical treatment is available none is supposed be very effective. For smaller lesions surgical excision can be tried. I would advise you to consult a dermatologist so that the diagnosis can be confirmed first as your self diagnosis might be wrong! Hope that helps. Take care"
},
{
"id": 126798,
"tgt": "What causes severe pain below the buttocks?",
"src": "Patient: Hello I ve been having severe pain in what I thought was my hips but the pain is really in the area right below my buttocks. It feels like my hamstring are about to snap. Very tight. The pain is less in the morning and intensifies as the day goes on. I do have a curves right knee that I ve been told someday when I m older ILL NEED A KNEE REPLACEMENT. MY knee is achey but nothing like the hips or hip flexor area. When I sit on heat the pain subsides somewhat but still there. I have difficulty bending down and getting back up because it feels like a muscle or hamstring is so tight. I m 58 years old. Used to be in great shape but this pain stops me from doing anything but walking and that only happens for a half hour before pain sets in. I feel like my hips could be out do place but not sure where to turn. I do have a Dr appt with my GP on May 3. Just looking for some insight as to what is happening to me. Doctor: Hi, The pain below the buttocks can be related to hip arthritis, a muscle spasm or a referred nerve pain due to mild disc bulge. Hope I have answered your query. Let me know if I can assist you further. Regards, Dr Praveen Tayal, Orthopaedic Surgeon"
},
{
"id": 97858,
"tgt": "Have high LDL. Prescribed zocor but taking natural supplements. Which medicine will be effective?",
"src": "Patient: HELLO, my doctor just told me my ldl is over 200, he prescribed zocor. After reading up on statins I am very fearful, I have been doing natural things such as resversatrol,lecithin,milk thistle and garlic/nacin supplements for a few weeks, the test was done a week ago, but i was not taking the mention supplements regularly. I am now. I do 'feel' when my chlorlestral goes up (funny symptoms) so I was not surprised that it was high, i didn't expect that high. Question; can I just continue the supplements or must I take the medicine he prescribed. I also had been eating pretty badly for a couple of months stressed/lonelines, but have done some diet changes and am excercising more. Doctor: hi, Lipoprotein is a combination of proteins and lipids (fat). By this combination fat moves in the blood to reach the cell membrane to have its action. So Lipoprotein acts as a carrier of Fat and other enzmes also. There are different verities of Lipoproteins like LDL,HDL,VLDL and Triglycerides. Out which if High density Lipoprotein (HDL) is high in percentage it is good. If others are high it is not good. Zocor is a statin group of tab used to reduce the bad lipoprotetns and enhances good lipoprotein. Supplementary therapy results not known, where as Zocor is proved drug. Better to continue zocor, associated with diet control and exercises. Zocor will not be cause for stress or ? loss of appetite. Regarding loneliness - change of life style will be helpful Best wishes"
},
{
"id": 4994,
"tgt": "Sleepy, increased appetite, mood swings, missed periods. History of having unprotected sex. Drug user. Possible pregnancy?",
"src": "Patient: I was recently having unprotected sex and i havnt had a period the whole month of july, i am a drug user but i normally get my periods, my drug of choice is meth so it should keep me up and not to hungry but even when i have used i would still get very sleepy and eat a lot, i get mood swings, and my stomach feels really bubbly, could i be pregnant? Doctor: Hi Dear,With history of unprotected sex and missed periods, there are all posibilities that you may be pregnant.You can very well confirm it by doing a urine pregnancy test or a B HCG test"
},
{
"id": 25804,
"tgt": "What does the BP of about 150/100 suggest?",
"src": "Patient: had a heart attack august 2010. on plaxic, crestor, ecotrin metotoprolol. this week my blood pressure has been 150/100 consistently. going to my cardiologist friday. not feeling well. a lot of pressure in my head. should i go to the doctor earlier??? Doctor: hello,I have gone through your query.Thanks for using HCM.your BP is on higher side I suggest you to start Tab.Ramipril 1.25 mg for its control.You need regular follow and cardiac check up,though you need not rush for BP of 150/100.my best wishesDr.Rajesh Teli,MD."
},
{
"id": 65280,
"tgt": "What causes lump on arm?",
"src": "Patient: Hi!I have a large painless lump, about the size of a 50 cent piece, on the bend of my right arm, sitting right at the end of the forearm just below where one would usually extract blood from. I did not notice it until last week and cannot accurately say how long it's been there. The lump is hard and shaped like one half of a very small egg. Doctor: Thanks for your question on HCM. Following are the common causes for lump at elbow joint area. 1. Dermoid 2. Lipoma 3. Fibroid 4. Tenosynovitis (inflammation of tendon sheath) So better to consult doctor and get done 1. Clinical examination of swelling 2. X ray of elbow joint 3. Ultrasound examination of swelling. X ray is needed for exact location of swelling and whether associated with elbow joint or not. Ultrasound is needed to determine type of swelling and it's content. If it affects elbow joint movement then surgical removal is necessary."
},
{
"id": 582,
"tgt": "Is there any chance of pregnancy with fluid from the penis?",
"src": "Patient: see, day before yesterday iwas with my girlfriend. then after some time my hand was on my penis. then my hand had some fluid on it. then I cleaned my hand with sofa instead of washing it. then after some time(around 15 mins) I started fingered my girlfriend. is their any chance of being her pregnant? Doctor: Hi, I think you can't be pregnant by fingering only. So, don't worry about it. Pregnancy only happen if there is direct contact of semen with vagina."
},
{
"id": 211028,
"tgt": "If someone speaks, i feel that they are talking about me. What could be the reason for this behavior?",
"src": "Patient: hello doctor..I want to know about psycology problem about some imaginations but tat r not true... for example if someone(not known before) is speaking my relative is thought that they are speaking about me.everybody want to spoil my life like that so many unusual thinking she have.we considered some psychiatrist and continuously she is taking tablets for so many years .but there is no change in her health.her mental health is decreases day by day.she is not believing no one .what we have to do? what may be the reason to behave like this?and how we can find the source of this problem? Doctor: HiThanks for using healthcare magicI think, she has delusion of reference that means she has false, fix and firm thought that unknown people are talking about her and with that way they are harming her. That is the kind of psychotic symptoms. In case she has only that symptoms for more than six months, diagnosis would be persistant delusional disorder. For that she needs continue dose of antipsychotic. Improvement depends upon kind of antipsychotic she is on. In case you need further help, you can connect us.Thanks"
},
{
"id": 67369,
"tgt": "What is the lump in between my vagina and inner thigh?",
"src": "Patient: I have a lump about the size of a dime and its oval shaped. Its right between my vagina and my inner thigh. There is no head on it and it isn t hard. It doesn t hurt unless I mess with it and its red. I don t know what to do because I don t have insurance to go to the doctor. How do I get rid of it ? What could it be ? Doctor: Hello and welcome to HCM,A soft lump between vagina and inner thigh could be due to enlargement of inguinal lymph node.A clinical assessment of the lesion is necessary for treatment.Thus, you will have to consult your doctor.In addition, an aspiration cytology of the lump is also required.Aspiration cytology is an office procedure wherein cells are aspirated using a needle, cells are spread on a glass slide and stained wit appropriate stain.The cells are examined under microscope.This investigation determine the site and nature of the lesion.Depending on the type of lesion, appropriate management is planned.Enlargement of inguinal lymph nodes is common after genital tract or urinary tract infection / inflammation.Thanks and take careDr Shailja P Wahal"
},
{
"id": 94585,
"tgt": "Abdominal pain. History of jaundice",
"src": "Patient: Hello Doctor, This morning I got up with a mild pulsating pain towards the right hand side (closer to the center) of my lower abdomen . This is continuing even now, 4 hrs after I got up. This is out of the blue and I was perfectly alright till I went to bed y day. One thing that s worth stating is that I had dinner party y day which went late (went to be at 1.30 am) and I had a few drinks as well. My total sleep time was around 5 hrs. I am physically fit, with an hr in the guys 3-4 days a week. I had jaundice close to 3 yrs ago and has fully recovered. Doctor: Hello Suraj Thank you for getting in touch with us. As I see, you have been having severe pain towards the right side of your lower abdomen. We would like you to know that it could be due to various causes like appendicitis, renal stone, colic etc. If you had fever along with pain, chances of having appendicitis were quite high. Therefore, we would recommend you to get in touch with your Doctor as soon as possible and get yourself fully examined. As you have not mentioned the cause of the jaundice you had 3 years back, it would not be possible for us to comment if it is in anyway related to this. Hope this helps Regards Dr. Gagandeep Dhillon"
},
{
"id": 10340,
"tgt": "Suggest treatments for hair loss",
"src": "Patient: I am having severe hair fall for the past 3-4 years... have tried several homeopathy medicines, but to no avail.. i take head bath regularly.... take good care of my health and food. i am left with few hair, please suggest how to keep atleast them... also, suggest a good dermotologist Bangalore Doctor: Hello and Welcome to \u2018Ask A Doctor\u2019 service. I have reviewed your query and here is my advice. I suggest you to take a few lab tests like thyroid function test, serum ferritin levels and complete blood count. I suggest you to apply minoxidil 5% solution, 1ml, twice daily. In addition I suggest you to take an oral biotin supplement. Response is slow and it may take 3-6 months for a noticeable improvement. Hope I have answered your query. Let me know if I can assist you further."
},
{
"id": 2409,
"tgt": "What to do if trying to concieve and having PCOD and irregular periods, light bleeding?",
"src": "Patient: Hi I have a pcod problem my dr has suggested me obimet 500mg nd fol123. I was having regular periods since last 3 months without any treatment but this time it almost 22 days delay. Have done upt but it came negative. Period started but with very less bleeding. I have trying for a baby since more than a year now. Pls suggest. Doctor: Hello dearI understand your concernI would suggest to consult gynecologist and undergo reproductive hormone analysis, USG scan, ovarian follicle study for better management.You can use progesterone pill or krimson 35 to correct hormonal imbalance and regularize period.Once period will become regular and you have patent fallopian tube then high chance of pregnancy.You can ask following medicine to conceive:Contine obimetTablet clomifene, tablet ovacare, injection HCGAvoid stress, take healthy diet, drink plenty of water, do regular exercise and maintain weight according to normal BMI.Hope this may help youContact further if follow up neededBest regardsDr. Sagar"
},
{
"id": 70580,
"tgt": "What causes lump right below inner knee and bruises on thighs?",
"src": "Patient: I know that I have a cyst behind my kneecap as I've had tests done before so my right knee is often a little sore, however this time it's more sore than it's ever been I can't bend it all the way or straighten it without pain, and it feels like there's a lump right below my inner knee where my calf connects, also I'm now noticing some bruises forming on the inside of my thigh at the top of my knee that are very dark blue with darker purple spots. What could be wrong and is it serious? Doctor: HIThank for asking to HCMI really appreciate your concern, this could be popliteal cyst at least can be said what the history given here, if this is popliteal cyst then you have to remove this and for that better to consult the surgeon, hope this information helps you take care and have good day."
},
{
"id": 196481,
"tgt": "Can injury to spinal cord cause ejaculation problem?",
"src": "Patient: I m a 24 yrs old male. Height 5'8\" & wt. 55 kgs Four years back i suffered Spinal Cord Injury & bcame paraplegic but now able to walk with the help of crutches. Has bladder & Bowel sensation. has proper Erection but no ejaculation. Can stay on be effective? How long should I take them. Presently i take no medicine & exercise regularly as told by phsiotherapist. Email ; YYYY@YYYY Doctor: Good day and thank you for being with healthcare magic!Yes ejaculation could be affected by spinal injuries and it is hard to say when it will come back. But since you have erections and sensation then there is a good chance it will come back soon. Just continue therapy sessions. I hope I have answered your question satisfactorily and please consider a 5 Star rating for my answer. Thank you."
},
{
"id": 16736,
"tgt": "Are Metoprolol and Triamterene prescribed for hypertension?",
"src": "Patient: My GP doctor prescribed these meds for high blood pressure range average 153/78. I am healthy 75 years old. Your opinion on safety of all of these...Olmesartan 20mg,,amlodiline besylate 10mg,metoprolol tartrate 50mg,triamterene 37.5/HCTZ 25mg. I am concerned about tak g all these. Some are new prescriptions. Doctor: Hello!, Would agree with you on the fact that considering your blood pressure values, there is no need to start with a lot of drugs. For this reason, I would recommend starting just with Metoprolol and Olmesartan and see if your blood pressure will normalize after a week. If after a week your blood pressure values are still high, I would recommend adding amlodipine 5mg to your therapy and increase the dose up to 10 mg after a week, if high blood pressure values still persist. Hope I have answered your query. Let me know if I can assist you further. Take care Regards, Dr Ilir Sharka, Cardiologist"
},
{
"id": 223792,
"tgt": "What are the side effects of having birth control pills?",
"src": "Patient: I took the birth control pills that make you have your period a week early. And a little less than a month later now I m spotting all the time. And it s not time for my period. Is this a side effect of messing up my Birth control or could it be something else? Doctor: hello dear.yes ,its due to side effect.birth control pills has side effects like spotting,irregular periods,heavy bleeding,mood swings,weight gain,pain & cramps,headache,visual changes,nausea.....but don't worry,it can't affect your birth control or any other harm.so relaxed.hope this will help you.best regards.Dr.sagar"
},
{
"id": 47208,
"tgt": "Suggest treatment for kidney dysfunction",
"src": "Patient: My wife is going under Dialysis for the past 5 months , her both the kedneys not working .She is going under treatment at Fortis , Vasant kunj , Delhi.Her blood group is o+, We don t have any blood relation who is having o Blood group.She is just 36 year old , having one daughter. Kindly advice how i can get her Kidney Transplant.If any donor is there then also pls let me know. I m reachable at 0000 , 0000. Doctor: HelloThanks for query .Your wife who is 36 years old has Chronic Renal Failure and is currently getting dialysed .She has been advised Renal Transplant .However there is no matching kidney donor in near relations .There are centers in India like Nadiad in Gujrath ,Mumbai who implement cross kidney donor program where any one of your relative has to donate kidney and they will manage to get matching kidney from the relative of other patient so that both the patients are benefited .Dr.Patil."
},
{
"id": 93741,
"tgt": "Abdominal pain, bloating sensation, hb 11.1, esr 32, wbc 9.89. Family history of cancer. Advise?",
"src": "Patient: Hello Doctor, I'm a women of age 32. I suddenly had stomach pain recently and feel that my stomach is blotted. my food intake is reduced. Doctor suggested a blood test and in the result it is mentioned hb is 11.1 and esr is 32 and wbc is 9.89. I'm in a rural area nt able to go for consultation to the city for past 2 weeks after result. I lost my mom in colon cancer at the age of 52 before 2 years. I'm getting worried now as my friend told me that I have to take it seriously. Please guide. Doctor: Hi and welcome to HCM. You should take it serious but you shouldnt be concerned. You are young and any cancer isnt very possible at your age.There are various conditions which cause stomach pain and bloating. usually this is gasrtitis, GERD or IBD. It should be primary treated by changing dietary habbits,eat fruit,green and avoid fats, alcohol and sugar. Also gastroscopy should be done to see degree of gastritis. Anaemia is common at young woman and usually menstrual bleeding and lack of iron cause it. It can be sign of bleeding from bowel wall or tumor but it isnt the first thing to think of. If you have pain while defecating or opstipation then you should concerned and do colonosopy. If not you should take iron supplements and change dietary habbits. Wish you good health."
},
{
"id": 182701,
"tgt": "How to rectify irregular crowned teeth?",
"src": "Patient: Hi,im having irregular crowded teeth from my childhood.Can you please tell me the procedures involved in the treatment.Can this be treated by applying braces or are there some other treatments also involved? I also have irregular gums.Please suggest.Thanks. Doctor: Thanks for your query, I have gone through your query.The mal aligned teeth can be corrected with the help of braces or orthodontic treatment. You need to consult a orthodontist and take an opinion. After that they will take measurement(impressions) of you upper and lower teeth, You need to get radiographs like OPG and lateral cephalograph. After going through these and analysing, we can give a exact treatment plan, like how much time it takes, whether we have to remove any teeth or not or do you need any surgeries. So consult a orthodontist and take an opinion.I hope my answer will help you, take care."
},
{
"id": 183320,
"tgt": "What causes bump in lower gum?",
"src": "Patient: I felt a bump on the inside of my lower gum just below the last tooth. I scraped it with a dental pick and a hard white lump was removed and the area now feels like an ulcer. Upon further probing I realized that the tissue seems perforated to the bone. I could feel bone. I have been swishing with warm salt water. Should I see the dentist ASAP? Also, I have a long history of Bruxism Doctor: Thanks for your query, I have gone through your query.The bump on the gum can be a periapical or periodontal abscess which on scraping has ruptured and exposed the underlying bone. Consult a oral physician and get it ruled out. If there is tooth infection get the tooth treated with RCT or extraction. If there is gum infection, then get your teeth cleaned once. Do not traumatise the tissues with tooth pick or pin, the mucosa or tissue will get infected. Mean while you can take a course of antibiotics like amoxicillin 500mg and metronidazole 400mg tid for 5 days (if you are not allergic). I hope my answer will help you, take care."
},
{
"id": 106460,
"tgt": "Is polio vaccine a contra indication during dexamethasone therapy ?",
"src": "Patient: My son 5years age withB. asthma I would like to ask about polio vaccine is it contra indication during dexamethasone therapy .(we have campaine this days) Doctor: No this can be given to all children. In case of more concerns, get the baby examined by the pediatrician, and if his asthma is under control, the vaccine can be given i guess."
},
{
"id": 132709,
"tgt": "Suggest treatment for painful finger",
"src": "Patient: Hi I think I broke my finger 9 months ago. Didn t go a&e didn t want to waist there time. Didn t do much with it carried on as usual but it still hurts can t Knock it on anything it s really painful goes red ok rest of time. Unless I tap anything on it it s where my finger bends sometimes struggle picking things up with it. Not been doctors because I didn t want to waist there time. From samantha Doctor: Hi SamanthaHope this message finds you in good health.I have gone through your complaints and understand your concern.U seem to have a soft tissue swelling or a ligament inflammation that might be causing the pain.U should get an Xray done to find out the exact cause.Hot fomentation,analgesics,anti-inflammatory drugs should help. A shot of local steroid injection should be useful.Nothing to worry about.\u00a0\u00a0\u00a0\u00a0\u00a0I hope your question has been answered.If you have any follow-up queries,feel free to consult me anytime.Thanks,Take care,God bless."
},
{
"id": 38976,
"tgt": "Suggest solution for dog bite",
"src": "Patient: Hi.I had a spech dog.he was suffering form pulmonary desease from last one month.10 days back he bite my mom.His vaccination is due on next april.I vaccinate him regular besis.last friday he died.has my mom suppose to take anti rabbies injection or not?please help me out. Doctor: Hello,Welcome to HCM,Dog is a known reservoir of rabies virus and it has got potential to transmit rabies which is 100% fatal but it is 100% preventable by proper and adequate treatment.As the dog has died after some time after biting your mother, according to WHO categorization this is categorized into Cat III, Has dog has died I don't want to take any risk and I would suggest you to follow these measures1.Active immunization with anti rabies vaccine on days 0,3,7,14 and 28.2. Passive immunization with anti rabies serum around the wound.3.Oral antibiotics like Tab Augmentin, 625 mg, twice daily for five days.The aforementioned measures should be followed to prevent rabies.Thank you."
},
{
"id": 151633,
"tgt": "Child absolutely normal, had seizures as an infant due to incorrect dosage, EEG reports normal, dosage of Tegrital tapered and finally stopped. Was treatment appropriate?",
"src": "Patient: when my baby was 5 month old he got seizurs coz of wrong dose of other medicin, he was on tegrital of all most eight month,though after starting this medicin his seizurs never occer, after eight month his eeg reports were normal and doctors told us to temper down his medidin.he medicin tegrital was stoped . noe he is 3year and 4months old and prefectly allright . has we done right by stoping his medicin Doctor: Hi, . Seizure in neonates under 5 years of age is usually benign condition and not worrisome. It is good to know that there are no more seizures after tegrital use. Now that there are no seizures for more than 6 months and EEG is normal, medication can be stopped as it is now done. A few clinicians though believe in continuing it for a year, however as your child is young and chronic use is associated with side effects, I agree with your doctor and it is the right decision. Do not worry. He should be fine. Hope this answers your query. Regards"
},
{
"id": 33661,
"tgt": "Suggest treatment for fungal patches in groin, leg, arm",
"src": "Patient: i have fungal patches in my groin, leg and arm. i have been using dakatrin for about 2 months now and the rash dosnt seem to be going at all. i have been refered to a dermatologist but that could take ages. the rashes seem to be getting bigger even when using the cream. the doctor said that the rash in my groin is ringworm and the rash on my leg he is not sure about, they both look the same to. can you reccomend anything else i could try? thank you Doctor: Hi,Before applying your medication, Wash the affected area with soap and water, and dry thoroughly.consult your doctor and Apply antifungal cream like Terbinafine hydrochloride 1% cream beyond the border of the rash. Maintain hygiene and wash your clothes and bed linen in hot water and dry under the sunlight.Regards,Dr.Riyanka"
},
{
"id": 107286,
"tgt": "How can severe upper back pain be treated?",
"src": "Patient: Hi I've got very sore upper back pain feels like it's in my ribs really hurts to sneeze cough and breathing deep i've been to my local GP and he kinda fobbed me off and gave me some exercises to do which are not helping at all it seems to come on every year at winter time a lot worse.can you please advise me what to do painkillers don't seem to work either kind regards Brent Doctor: Hello,Since the painkillers do not work and you have had this pain for long period of time I would suggest you to do an MRI of spine and consult that with a neurologist specialist. The solution might be surgery if you have advanced discal hernia.Hope I have answered your query. Let me know if I can assist you further.Regards,Dr. Edvin Selmani"
},
{
"id": 123256,
"tgt": "What causes sharp pains in the groin area?",
"src": "Patient: if i stand all day i get real sharp pains in my groin area also i was numb on my left side from head to toe took three and half weeks to go away had a mri it look like ms but i only had one spot but i was very fagitue and had to sit alot not sure whats going on Doctor: Hello, Usually, MRI is the report which suggests the possible tissue pathologies and we need to correlate that with the clinical symptoms to confirm that it is MS or not. Even a lumbar puncture may be advised to rule the possible CSF infection. As there is a pain in the groin area and numbness in the toe I think this should be related to the degenerative changes in the spinal column leading to Radicular symptoms With proper diagnosis and then medicine for the cause can help symptomatic relief. If it's a spinal column issue or even the MS, general exercise to focus on strengthening the muscles of the spinal column along with the upper limb and lower limb will help reduce your symptoms. Hope I have answered your query. Let me know if I can assist you further. Regards, Jay Indravadan Patel, Physical Therapist or Physiotherapist"
},
{
"id": 153412,
"tgt": "Suggest treatment for bone cancer",
"src": "Patient: i'm wondering about the survival rate for bone cancer once someone has already \"survived\" breast cancer. does it depend on whether it's the original cancer returning? Is it possible for it to be a whole new/different cancer unrelated to the breast cancer? Doctor: Hi,Thanks for writing in.Cancer is the uncontrolled growth of abnormal cells involving any organ system. There are cases where a cancer is treated many months to years back and appears again at another location. In other words it is a metastasis of the original cancer after the patient is treated for the primary. The cells that are found in the new location show characteristics of a cancer spread from else where and the pathologist is able to make a diagnosis.Few patients have a completely new type of cancer which is not related to any previous cancer. The biopsy from the new cancer are entirely different and show features of a primary cancer. It is important to get detailed investigation done and do pathological confirmation. Please do not worry."
},
{
"id": 20338,
"tgt": "What causes high blood pressure and arrhythmia?",
"src": "Patient: I am asthmatic, I feel like my bronchial tubes are tight, I have a little indigestion after taking a nexium, my diastolic numbers are higher than usual, and I am having some skips with heartbeat. Not sure if I m a little congested In my bronchial tubes, or perhaps it s a little anxiety. YYYY@YYYY Doctor: Dear user, you should see a cardiologist and a pulmonologist, they can do the necessary check-up and prescribe a therapy"
},
{
"id": 14081,
"tgt": "How to treat rashes in the wrist area?",
"src": "Patient: I ve just recently (over the last 2 days) noticed rash-like areas appearing on both hands. They are on the same place on both hands; below the lowest thumb joint and above the wrist. If you hold your hands out in front of you palm down the 2 rash-like areas will be facing each other. By appearance, they almost look like something between poison oak and athletes foot. Overall the area is red, with tiny, hard bumps. I haven t been working in the yard, so my exposure to any poison oak/ivy has been non-existant. No itching, no oozing. I just need to know how to treat. Doctor: Hello and Welcome to \u2018Ask A Doctor\u2019 service. I have reviewed your query and here is my advice. The rash can be due to allergy, contact dermatitis or scabies. Wash your hands with warm water and do not use harsh chemical soap. Take non-sedating antihistamine (Levocetirizine), depending upon the severity of the rash. It can be contagious so be careful. Use Ascabiol emulsion, put it locally on the areas where you have the rash, leave it there for some time and then wash it off. The rash should go away if it is scabies, else repeat the same after few days. Take care of your hygiene and laundry. Do not scratch, take care. Hope I have answered your query. Let me know if I can assist you further. Regards, Dr. Nupur K."
},
{
"id": 41348,
"tgt": "What should intercourse be done to get pregnant after taking choriomon?",
"src": "Patient: hi I m 43 years old got married 4 years back now planning for the kids from 1 year but failed now doctor gave me clomid 50 & choriomon 5000 I. u......she told start after u get your period....can u tel me after choriomon when I have to do sex that can help to get pregnant Doctor: Human chorionic gonadotropin brand Choriomon hCG is used primarily in fertility medicine for female. In females it is an ovary stimulant very effective in activating egg release and optimizing ovulation."
},
{
"id": 119729,
"tgt": "Can a lump in the knee cap cause pain?",
"src": "Patient: Hi, I am a regular runner, but lately have had pain in my left knee. Having felt the knee there is a small lump on the knee cap itself, does this mean anything? Or just paranoia?! I was taking Terbinafine for a nail infection and had read that sore joints could be side-effect. Doctor: Hi, Swelling over the knee along with pain suggests bursitis, pain in prepatellar bursitis, (type of bursitis in which there is swelling over knee) occurs while doing flexion or on putting direct pressure over the knee cap. You need to get an MRI done to reach a diagnosis. In the meantime, start doing icing over the swelling,and take some anti-inflammatory medicine, along with prevention of any type of pressure over knee cap. Hope I have answered your question. Let me know if I can assist you further. Regards, Dr. Rohan Shanker Tiwari, Orthopedic Surgeon"
},
{
"id": 103203,
"tgt": "Lump near temple, had wheezing, redness and raised upwards. Express concern?",
"src": "Patient: Hi there, I was sitting outside on my balcony yesterday afternoon and felt something an inch below my right temple exactly in line with the top of my ear. I felt a pea size lump that felt a bit tender but it didn't appear to be red. I went to lay down for a nap about an hour later and I noticed that occasionally when I was breathing out, I was weazing but it wasn't consistent. It is now 5:25 am and I have been tossing and turning in bed the last few hours and that occasional weazing keeps occurring but it's not consistent with every breath. the \"bump\" now shows some redness and feels deep under the skin but also seems to have raised upwards. I have not been able to sleep because I feel like I am a bit light headed. Should I be concerned? 27 / Male Doctor: Hi..Thanks for the query..The lump you are seeing may be an enlarged lymph node which is commonly present in any infections..You might be having an upper respiratory tract infection..You can consult general physician who can advise for antibiotics and anti allergics like ceterizine..Take warm water for drinking and do salt water gargling for relief..Hope this information is helpful.. Take care..."
},
{
"id": 136837,
"tgt": "Suggest a remedy for tailbone pain",
"src": "Patient: I have had tailbone pain for well over 12 months. I have seen a physio, I have taken anti inflammatory tablets, I sit on a cushioned ring. The last week or so I have been in so much pain that now it throbs when I'm laying in bed and standing also (not just sitting). It also hurts just standing. I work in a classroom so I am up and down off chairs during the day and in the evenings I'm also on computer lots as I'm completing university also. Any more help would be appreciated. Doctor: Hello, I have studied your case. For resistant coccydynia[tail bone pain] sacral local block injection may be helpful. Prolotherapy [local injection] may also work for sacral pain.Consult your doctor for injection.Physiotherapy like ultrasound and interferential therapy will give relief.I will advise to check your vit B12 and vit D3 level.Continue sitz bath and use of ring cushion.Hope this answers your query. If you have additional questions or follow up queries then please do not hesitate in writing to us. I will be happy to answer your queries. Wishing you good health.Take care."
},
{
"id": 74394,
"tgt": "Suggest treatment for presence of blood clot in the lung",
"src": "Patient: Hi I m a 54 year old woman who just found out I have a blood clot on my left lung, it s a small one and they gave me Zarelto to be taking twice a day with food. Sometimes I m not hungry and I have to take it with food. What food should I take with it,and us it okay to take it with fruits????. Doctor: Thanks for your question on Healthcare Magic. I can understand your concern. Yes, want take xeralto with fruits, no need to worry for this. You can eat all kind of fruit items. No restrictions because xeralto is not affected by food. So you can take any food item with xeralto. Hope I have solved your query. I will be happy to help you further. Wish you good health. Thanks."
},
{
"id": 46789,
"tgt": "Suggest treatment for nephrotic syndrome",
"src": "Patient: AGED 60 YRS, diabetic for last 23 yrs.,HBP FOR 8 YRS ,now diagnosed for being nephrotic, leaking albumen 2.5, total protein 6.2, protein 24 hrs.- 4.5, Creatinine -1.9, uric acid- 7.3, GFR-Lt 55.94%,29.41 ml/min.;Rt 44.06%,23.17ml/min; BUN 22, BUN/CRT ratio-12.22. bp-160/100, fasting sugar 139, pp-230.Taking no insulin,only oral medication for sugar & bp.taking. should I survive for long ? Should I take ALOE JUICE? Doctor: Hello,Thanks for choosing HCM. In my opinion the problem you are facing is diabetic nephropathy in which you will have protenuria which is common but if you have more than 5grams of 24hr protein then you need a biopsy which need to be done for knowing exact cause. About your medication if your dose of medication for bp and sugars is not sufficient then you need to change the drugs tab.galvas 50mg bd os better that ta .metformin in kidney patients. Please check for diabetic retinopathy changes in eye also . Alovera juice in allopathy medicine we dont exactly know whether it is worth taking or not.Take care.RegardsDr.Alekhya"
},
{
"id": 119682,
"tgt": "Suggest remedy for wrist pain",
"src": "Patient: My left wrist hurts when I rotate it, and pain goes up my outer foream. my pinkie, ring and middle fingers fall asleep. 10 plus years ago a ganglien cyst was removed from the inside of that wrist but it doesnt appear to be back. Ibuprofen and tylenol does not help, as does a wrist brace. I didnt hit it or do anything to fracture it Doctor: Hello, Looks like some neuropathy you are suffering from. Get some tests done. But before that, I would like to know if you have diabetes? 1.Blood tests: CBC, ESR, URIC ACID, VIT B12, VIT D3 2. X ray of cervical spine & wrist AP & lateral 3.EMG/NCV Left upper limb Review with reports. Till then continue wearing splint. Hope I have answered your query. Let me know if I can assist you further. Take care Regards, Dr. Rajesh Hingwe"
},
{
"id": 173489,
"tgt": "Does hand foot and mouth virus cause red soles on feet in a baby?",
"src": "Patient: Hello I have a 2 year old that has red soles on her feet, she also has a few red bumps on her fingers. No fever and is acting fine. I was wondering if she has hand foot and mouth and should I first call the pediatrician and second keep her away from others?Thank you Doctor: Hi,Thanks and welcome to healthcare magic.It is not possible to say whether it is hand foot and mouth disease or not without actually seeing the baby.It is wise to consult pediatrician immediately to get to the correct diagnosis and proper treatment .Meanwhile you may keep the child in isolation .Hope this reply is OK for youPlease feel free to ask further queries if any.Dr.M.V.Subrahmanyam."
},
{
"id": 5499,
"tgt": "Trying to be pregnant. On medication. Suffer from vomiting, headache, tender breasts, frequent urination",
"src": "Patient: HalloI am trying to get pregnant, we went to the doctor and the doctor prescribed pills to help me fall pregnant, we had unprotected sex 2 weeks before my period was due, my period came a day late and the first day was basically nothing, then the second and third day was normal and then basically nothing again, i also have tender breasts, nausea vomiting backache and I urinate more than normalPlease help Doctor: Hi, Thanks for query. You should consult a doctor and get your urine test,blood test and USG. done. The symptoms is because of pregnancy because you have sex around your ovulation time. These symptoms is because of less period,so first you consult than take any medicine. take care."
},
{
"id": 172620,
"tgt": "Why my 3 year old is limping while standing?",
"src": "Patient: my 3 year old just got sent home from school after landing funny on her foot. She was crying but is now ok and happy sat down but if she gets up she limps and crys.There is no swelling and she can move her toes and puts weight on her foot while on the chair without complaining. Doctor: Pls rule out any underlying fractures if there is pain or swelling or redness...otherwise it could be some ligament strain which will get cure on its own by giving rest to the leg.."
},
{
"id": 13972,
"tgt": "Suggest treatment for itchy and red rashes on arm",
"src": "Patient: Hello There are some rashes in my arm folding place near biceps. It might becoz I play in outdoors and from sweating and in sun. Now it has very tiny bubbles and redness with itching. What best cream I must apply so that it can cure. Before I remember I apply once cream clomate, but the problem repeat when I don t apply cream. Doctor: Hello and Welcome to \u2018Ask A Doctor\u2019 service. I have reviewed your query and here is my advice. I would recommend you to apply Mometasone cream on the affected areas twice daily and take antihistaminic like tablet Levocetrizine once daily at bedtime. Also apply Clotrimazole dusting powder on the affected areas 2-3 times in a day. Hope I have answered your query. Let me know if I can assist you further."
},
{
"id": 108667,
"tgt": "Can lower back and abdominal pain with fever indicate pelvic inflammatory disease?",
"src": "Patient: I recently went to the hospital for a high fever with back and lower abdominal pain. The doctor believes I may have Pelvic Inflammatory Disease. He gave me two injections, one for pain and one of antibiotics. In addition he gave me a single dose of antibiotic pills. The fever is gone, the back pain is gone. But I still have lower abdominal pain. What does this mean? What should I do? Doctor: Hi,It seems that you are having some infection in your pelvic region giving pain and fever.As medicinr worked it indictes having infection.Continue with antibiotic and anti spasmodic medicine.After complete control of infection you will be alright completely.Take plenty of water.Take light diet.Ok and take care."
},
{
"id": 94658,
"tgt": "Frequent urination, bleeding in urine, abdominal pain during urination. What could this be?",
"src": "Patient: Just recently I have been going to the bathroom alot. And when I do it s only trickling and just today I started to bleed but only when I pee. I haven t had my period since September 14. I haven t had any sickness but I do sleep alot. Everytime I ve thought I was pregnant I wasn t so I don t know what it is. Also it can t be a uti I have had one before. But I have abdominal pain when I pee. And when I get right back up I feel uncomfortable so I try and pee more and it just trickles out. HELP Doctor: Hello! Thank you for the query. Such symptoms can be caused by urinary tract infection, urinary tract stones or pregnancy. Abdominal pain can appear due to uti. Moreover, back pain can appear. It is advisable for you to have full blood work, urine analysis, pregnancy test and abdominal ultrasound. If the uti is present, antibiotics are necessary. You should also have your blood sugar checked as diabetes can cause recurring utis. Hope this will help. Regards."
},
{
"id": 181203,
"tgt": "What causes blood clot at the tooth extraction site?",
"src": "Patient: Hi, I had a tooth extraction 7 days ago and I noticed the blood clot has been falling piece by piece. I have a dentist appt tomorrow but I was wondering if I should be considered about developing a dry socket or is this process part of the healing process? Doctor: Hi..Thanks for the query..I have gone through your query and can understand your concern..As per your complain in case if the blood clot is loose and falling in pieces there is a possibility that you are loosing the healing tissue and in case if it is completely lost you may definitely develop dry socket..I would suggest you to consult your treating dentist and get evaluated and a thorough clinical evaluation is diagnostic..In case if the clot is lost considerably he can put a zinc oxide eugenol pack in the socket as it will act as sedative, prevent pain and initiate healing..As of now you should avoid gargle vigorously..Take a soft diet..Avoid sucking anything with a straw..Smoking if you do, shall also be avoided..Hope this information helps..Thanks and regards.Dr.Honey Nandwani Arora."
},
{
"id": 219916,
"tgt": "Suggest possible tests to confirm pregnancy",
"src": "Patient: HI sir/madam,my wife's age is 27yrs and her last mensus date was jan1 2014,on feb 5 we conducted home pregnancy test and there appeared a very thin line on the positive side,we confused that it is really positive or not,on feb 8 we consulted doctor and she repeated the test there appeared again a very thin line,doctor said that there must be a chance and hope for the best,and said that to wait for 10 days and consult her.doctor gave medicines for 10 days i.e ferrous-xt tablets and arg 9 powder,we are in a great confusion that it is positive or not,kindly guide me... Doctor: Hallow Dear,The first test you performed was rather too early, hence confusing results. Ideally, the pregnancy tests on urine should be performed 8-10 days after missed period on overnight early morning first urine sample for reliable results. Done earlier, the tests may be false negative. However, there is another test - Beta hCG test which can give reliable results even few days before missing period. However, even after 8 days you have got the faint positive line; i.e. weak positive test, it suggests that the hormone hCG is low. This happens commonly in following conditions:1. Test performed too early - may be delayed conception. 2. When the baby has become non-viable. This is known as missed abortion. Then hCG gradually starts declining and hence week positive test.3. Ectopic pregnancy: In this condition the baby is implanted somewhere outside the uterine cavity, most commonly in the tube. It is a very serious condition. Subsequently tube ruptures and causes alarming bleeding. It needs blood transfusion and surgical intervention. The best option for differential diagnosis is ultrasonography. In normal pregnancy, intrauterine sac will be seen with foetal cardiac activity.In non-viable baby (missed abortion), the foetal heart will be missing and sac will be irregular. In such cases timely termination of pregnancy is indicated lest the serious complications like threatening bleeding due to clotting defects (DIC), infection, kidney failure, etc. If the uterine cavity is empty in spite of (weak) positive test, it indicates that the implantation is somewhere else; i.e. ectopic pregnancy. Laparoscopy will help to confirm the diagnosis. If it is unruptured ectopic pregnancy, it can be managed by laparoscopy only. Please report to your Gynaecologist and undergo ultrasonography for correct diagnosis and managing the pregnancy appropriately. I am sure this helps you. Dr. Nishikant Shrotri"
},
{
"id": 140708,
"tgt": "Suggest treatment for upper back pain and numbness in hand",
"src": "Patient: Hello, I have an upper back issue near the rohmboid and scapula (right side). When it flares up I darn near have to take the afternoon off of activities with my kids. I m a single dad so I pretty much refuse to do that. I trudge through my day at work, my right hand goes numb. It is beyond frustrating, I first felt it roughly 1 1/2 years ago, while working extending with the arm repetitive motion doing janitorial.It has progressed worse and worse. I work fast repetitive motion all day long. I recently had an MRI of upper extremity focus on rohmboid right shoulder I got the Mri and supposedly it came back normal. They can feel inflamation, that s why 1 doctor wrote an mri for likely tear . Now the mri says normal my PA says I m fine. ? Doctor: Hello, If you are performing repetitive activities with your arm/shoulder and you feel the pain or sensation of discomfort/swelling in the mid to upper back region where the scapula and rhomboid muscle are located then, you could have a nerve entrapment syndrome of a twig of the dorsal scapular nerve that supplies that region. I recommend seeing a neurologist and describing your symptoms. This should prompt an examination of the back and the affected area to see if you have anything that we refer to as myofascial pain. In your area of complaints, swelling, WINGING SCAPULA, or any other physical evidence of trauma or positional entrapment of even blood vessels in the area that can occur from sitting too long in one position, sleeping on the affected side without moving enough, etc. An EMG/NCV study could demonstrate abnormalities of nerves and muscles in this region as well. Also, I would recommend either a CT or PLAIN X-RAYS of the thoracic spine and right shoulder girdle in both flexion and extension positions (something not able to be done on an MR scanner) which can demonstrate other causes of this sort of pain such as arthritic degeneration, scoliosis, other skeletal abnormalities irritating structures in that region. Hope I have answered your query. Let me know if I can assist you further. Take care Regards, Dr Dariush Saghafi, Neurologist"
},
{
"id": 212566,
"tgt": "Hearing voice in head advising to do things, if not bad things will happen. How to overcome?",
"src": "Patient: hi there i have recently started hearing a voice in my head that tells me to do things either to myself or to others and if i do not comply bad things will happen wich makes me scared to venture outside this happens all the time and i dont know what to do i have considered killing myself a few times please help thank you. Doctor: Hello sterky thanks for question First of all ask yourself whether the voices you are hearing in your head is your own voice or its coming from outside? then whether others are able to hear this voice? or they are not understanding your problem? if these voices are vague sounds or well formed voice? are you able to control these voices by closing your ears? are you able to sleep properly? usually hallucinations are one of important symptom in some psychotic disorders which resembles symptomatology you are describing.. and they are usually distressing to affecting individual, and usually ask to do derogatory things. Second possibility in such situation is in obsessive compulsive spectrum, person is compelled to do a particular thing otherwise some bad will happen. Its more or less similar to magical thinking. Dear friend in both of the situation you should consult a good psychiatrist. And don't worry treatment is possible in both cases. Try to interact with family members and discuss with them regarding the problem. thanks and hope you are satisfied. take care kindly rate"
},
{
"id": 100675,
"tgt": "Can allergy occur due to rise in hb?",
"src": "Patient: my mom went for some allergy tests monday and the doctor sent her a blood test also they told her the results would take up to 10 days to come back then the following day her gp phoned and asked her go and pick a perscription for fexofenadine hydrochloride but they didnt tell her what they were for. When she went and picked the perscription up the pharmasist told her they where to do with hemoglobin. could you please tell me if it because her hemoglobin level is to high? Doctor: HI, thanks for using healthcare magicFexofenadine hydrochloride is an anti histamine. You may know it as allegra.It is to help with her allergy symptoms that she is experiencing. It is not related to hemoglobin levels.She should speak to her doctor regarding any concerns she may have with her medications.I hope this helps"
},
{
"id": 129762,
"tgt": "What causes toe pain?",
"src": "Patient: Hi,In between my 4th and 5th toe hurt so bad. At first it was just very itchy for few days and become very painful all around it and between. I cannot walk or wear any shoes on my right foot. The only I get some comfort is when it up on something. The pain become worst when i wear shoes. Please help. Thank you. Doctor: Hi, there!I think You have problem with Your foot shape.To diagnose and confirm it, You need to make podometry.Suppinators will help to cure this condition, if diagnosis confirms"
},
{
"id": 126578,
"tgt": "What causes arm pain after taking BP readings?",
"src": "Patient: I took my blood pressure ar home and it really hurt my are like when you give blood and they tie a rubber strip around your are to take blood. This was about 5months ago. I am just asking now as I had sever medical events since then but it still hurts. Feels like a nerve pain and hurts when I sleep on that arm. Any suggestions? Doctor: Hello, The pain may not be related to BP recording and can b due to some other causes. As a first line of management you can take analgesics like acetaminophen or Tramadol for pain relief. If pain persists, better to consult a neurologist for any possible neuropathic pain. Hope I have answered your query. Let me know if I can assist you further. Regards, Dr. Shinas Hussain, General & Family Physician"
},
{
"id": 2498,
"tgt": "What could be the reason for fluttering feeling in lower stomach?",
"src": "Patient: So my question is referred to pregnancy, so I started my period last month so I was told I wasn't pregnant. I've been having the feeling that I need to throw up but I don't end up throwin up . This month I started my period it lasted 2 days , but then started 2days later but real light like when you finish . Its been a week since my period and I've been getting a fluttering feeling on the lower part of my stomach for the past 2 days should I be worried? Doctor: Hi, Such mild bleeding could occur in early pregnancy also. Do get your urine tested for pregnancy. Further confirmation with Serum Beta HCG.If the results come negative you need to rule out other causes of periods irregularities. Get Thyroid profile/ Serum Prolactin.Fluttering feeling could be an impending period !!All the best"
},
{
"id": 157915,
"tgt": "Lump on neck near jaw line. Had been treated for breast cancer. Can it be cancer?",
"src": "Patient: I had breast cancer 2 years ago, one lymph node was positive. I had lumpectomy , axillary clearance, 8 rounds of chemo and 30 radio. I now have a lump on my neck, on the same side as my cancer, about an inch in diameter, its just below my ear but not exactly on my jawline, I noticed it a few weeks ago, but I m not sure exactly how long ago. Doctor: The lump that you describe is possibly a lymph node in the upper part of the neck. Lymph nodes are present in all parts of our body and get enlarged due to infection or cancer in the draining area. Now, the area that you have described is not part of the normal draining pattern of breast cancer. Had it been in the lower part of the neck i would have been more worried. Another possibility is that it may be an enlargement of the parotid gland ( a gland that produces saliva and is present in the neighbourhood of the ear). It can be enlarged in a variety of conditions including cancer. In short, you need to see your oncologist to ascertain which of the two it is. He will have to get a needle test done to decide upon its nature (FNAC). Meanwhile you can start yourself on Tab Ciprofloxacin 500mg twice daily for 5 days to take care of an infective etiology."
},
{
"id": 225653,
"tgt": "Trying to conceive. Cramps, acne. Sperm count on the borderline. Taking duphaston. Suggest?",
"src": "Patient: Hi I am 31 yrs old and trying for pergnancy for past 1 yr but still not. My husband s sperm count is on the burder as per the doc and my doc suggested me to take Duphaston tab for next 3 month. My periods is very regular and last up to 3 days. My weight is 57 kg and height is 5.2 ft. I grow some achne over my face during period and left lower stomach cramp sometime during period I had done all my blood test done and all came fine as per doc and no thyroed and I am perfectly healthy. But I am a working lady having lots of work pressue. Please suggest me to get pegnent soon. Doctor: Hellothanks for your query.Please consult an Infertility specialist first.have basic hormonal tests done.Have a cycle of follicular monitoring to check if your ovulation is alright.Your husband should consult an Andrologist, and have evaluation to know the cause of abnormal semen analysis.Sometimes, if the semen count does not improve, IUI ( intra uterine insemination ) or ICSI ( intra cytoplasmic sperm injection ) might be required.Also, the patency of fallopian tubes also should be checked.take care."
},
{
"id": 86021,
"tgt": "What causes abdominal cramps after consuming prune juice?",
"src": "Patient: HI, My husband most likely has the same GI virus that I have and was diagnosed with yesterday. One of the many annoying symptoms is that it is hard to keep food down, and any drink too. While I was at the doc s office yesterday he drank a glass of prune juiice and has very bad cramps and been i n the bathroom a lot. He also has an appt with a kidney doc on Monday due to his histoy of illnesses and the pain meds he has taken for several years. We are worried abot how his kidneys might be effected. Lots of pain has been the norm for a long time but now he just want s to crawl up in a ball most of the time. His upper and lower GI test were good - all ok! Why so much pain after the prune juice? Seemes silly to ask this but he is really hurting! Doctor: Hello and Welcome to \u2018Ask A Doctor\u2019 service. I have reviewed your query and here is my advice. The presentation of narrated symptoms suggest that there is possible acute gastritis or duodenitis caused by the prune juice which may be from its concentration effect over the gut flora mechanics, which may be compromised from underlying infection. I recommend to visit an ER for immediate relief with injection of proton pump inhibitor, anti spasmodic and other supportive care. Hope I have answered your query. Let me know if I can assist you further. Regards, Dr. Bhagyesh V. Patel"
},
{
"id": 220053,
"tgt": "What causes lower abdominal and back pain with bleeding in pregnancy?",
"src": "Patient: hello I am 28 years old and pregnant for the sixth time (one abortion 3 years ago, and 4 living children). I am experiencing cramping in the lower abdomen and lower back and even bleeding with passing of clots. What should I do. I have been to the hospital twice for this issue. The first time I was told baby was not doing good and to come back when I came back they said everything was good. I did not experience back pain the first time but now I am. Doctor: Hello dear,I understand your concern.In my opinion the pain abdomen with bleeding with or without backache first gives a suspicion of miscarriage.But as the ultrasound is normal it doesn't go in favour of miscarriage.So need not worry because there might be some other cause for bleeding like:-1)Hormonal abnormalities like thyroid abnormalities,progesterone insufficiency.2)Bleeding from cervix etc.So the above causes should be ruled out by getting done thyroid profile and other necessary investigations if needed.Avoid physical stress and intercourse.Take prenatals and progesterone hormone supplements.Don't worry as many cases which present with bleeding in pregnancy continue till term and also have good outcome.So relax.Take iron rich diet and supplements to combat anaemia caused due to bleeding.Hope this helps.Best regards..."
},
{
"id": 37593,
"tgt": "What is the medication for PTB?",
"src": "Patient: Hi doctor, I just want to know what I can furthet do to ease my fear. I m on my 6th days taking quadtab.in my 1st 2 days I ve got fever the whole night, and I went back to my pulmonary doctor the next day, asking for her opinion.she told me to take tylenol 650mg every 5 hrs for fever.I felt very uncomfortable taking this quadtab still light fever goes on evrytime I take this treatment.is their any other pills aside from quadtab, in treatment of ptb? Thank you. Doctor: Hi,Welcome to HCM.I understand your concern about recurring fever but it is one of the symptoms of tuberculosis. Quadtab is a combination of four drugs which are first line drugs used in treatment of tuberculosis. These include rifampicin, isoniazid, ethambutol and pyrazinamide. These are drugs of choice to be given as intensive therapy.There are other drugs which are used as second line drugs only when first line drugs are not effective. This is followed because patients develop resistance to TB treatment very easily. I suggest you continue the treatment as suggested by your treating doctor and the fever will reduce within 2-3 days.Also please complete the full course of treatment.Thanks."
},
{
"id": 189748,
"tgt": "Had coronary stent put in, taking antibiotics, had tattoo. Is anything wrong?",
"src": "Patient: Hello. I had a coronary stent put in 18 months ago. I am on plavix, metoporal, hydrchloriazide, crestor,. I normally take antibiodics before a dental session. I had a tattoo done 7 days ago and didnt even think about the antibiodics. Have I placed myself in danger? Should I contact my heart doctor and get antibiodiotics now Doctor: Hello and welcome, As per the details you have produced,its not clear which part of teeth you got the tattoo done. If the tattoo involves within the enamel of teeth,nothing to worry,as the chance of infection is very less. If the tattoo is placed too deep,you should take antibiotic cover as it can cause infective endocarditis. Please do mention about your medical history whenever you are getting any treatment done. I would suggest you to visit your dentist soon and get details regarding this. Please do maintain good oral hygiene. Brush your teeth twice daily. As the chance of gingivitis is severe in your case,please lead a healthy life style. take care."
},
{
"id": 94510,
"tgt": "Severe vomiting with stomach pain. What is causing it?",
"src": "Patient: Hello! Might you tell me what you guess my problem is? I vomited8 times over a period of maybe 7 hours and I was unable to keep even the slightest smount of water in me. Additionally i had serve stomack pains and could only lay on the back woth upper body slightly up. Aswell, i felt like some sort of stone in the stomack - always soon after vomiting and dringing a slight amount pf water Doctor: Hi, thanks for using healthcare magic It is likely that you have gastroenteritis which is usually caused by viral infection. It is less likely caused by a bacteria, in those cases there is usually fever and blood in the stool. You should drink small amounts, if stomach lining is irritated then it would be difficult to keep in alot at one time. Drink a small amount, wait at least 5 minutes before taking another small amount. You should use the oral rehydration fluids which would replace the different electrolytes you are losing. Medication normally used for reflux such as pepcid, omeprazole and others in those drug families may help with the irritation and pain that your are experiencing. In addition medication like gravol may help with the vomiting. If it progresses and is associated with significant discomfort then you should consider visiting your doctor for an assessment. Hope this helps and you feel better soon"
},
{
"id": 73767,
"tgt": "What could acid reflux with heaviness in lungs suggest?",
"src": "Patient: I WAS ALREADY DIAGNOSSED WITH ACID REFLUX. I HAVE BEEN ON DEXILENT FOR 3 MONTHS NOW AND I FEEL TOTALLY BETTER. MY PROBLEM IS THAT MY LUNGS DON'T FEEL HEALTHY. THEY FEEL HEAVY. WHEN I TAKE A DEEP BREATH OUT I FEEL LIKE I WANT TO COUGH. MY THROAT HURTS ME WHEN I TOUCH IT ON THE OUTSIDE. IT IS VERY TENDER. I WENT TO AN ENT AND HE SAID THAT EVERYTHING CHECKED OUT FINE, WHAT ELSE CAN BE WRONG WITH MY LUNGS THAT CAN GO WRONG. THANK YOU Doctor: Thanks for your question on Healthcare Magic.I can understand your concern.You are had acid reflux. Chronic acid reflux can cause bronchitis. Bronchitis can be the cause for Heaviness in lungs.So consult pulmonologist and get done clinical examination of respiratory system and PFT (Pulmonary Function Test).If PFT is normal then no need to worry for bronchitis or other lung disease.Hope I have solved your query. I will be happy to help you further. Wish you good health. Thanks."
},
{
"id": 225546,
"tgt": "Headache, high temperature, dizziness, feeling thirsty. History of irregular and continuous bleeding after using contraceptive implant. Opinion?",
"src": "Patient: Hello. I am a 22 year old female. 10 months ago I started using the contraceptive implant. It is great only bleeding is irregular. Currently the bleeding has lasted for a whole month. In the last 3 days I have been feeling unwell. On the first day I woke up in the morning with a headache. As the day went on I developed a high temperature (not sure of exact temp as I had no thermometer but I know it was high). Took paracetamol and rested and this just about got rid of the headache and lowered the temp. Day 2 I had a mild headache and felt a bit dizzy at times. I am feeling the same today as I was yesterday. I seem to be quite thirsty as well and at certain intervals during the day I will lose my appetite I can sometimes feel nauseous, though this only last for a short while during the day. Overall I have a good appetite and eat enough every day. Have you any idea what might be causing this? Appreciate any answers thanks Doctor: Hi Dear,you are suffering from two unrelated problems.Firstly, excessive bleeding due to implant which should be taken care of. In absence of any treatment you will become anemic and develop weakness. Consult your gynae as it can be managed with medicines. However, if no improvement, the implant has to be removed and you can go for other birth control measure.Secondly, the fever which you are having may be seasonal and has to be investigated properly. Consult a physician and get investigated for all possible causes of fever including dengue, malaria and typhoid"
},
{
"id": 180312,
"tgt": "Can bilirubin increase with days in child?",
"src": "Patient: At 18 days old my daughter had bilirubin level - 335. Is it possible, that she had higher level (f.e. 400) at day 10? 335 level: can it go down by itself or only some treatment could help? Which level is dangerous or the brain cells? She eats well, gains weight well. Just not too much active (may be it is her personality). She is 2 months old, her level is still high: 146. Is it dangerous? Doctor: Hello, Your daughter had jaundice in the newborn period and her bilirubin is still high at 2 months of age. This is definitely not normal and she needs to be evaluated in detail for the cause of persistent jaundice. There are many causes of persistent jaundice and it is difficult to ascertain the cause based on the history you have provided. I advice you to take your child to a paediatrician for a detailed check up, as some of the conditions causing jaundice at this age need urgent treatment. Wishing you good luck.Regards,Preeti"
},
{
"id": 124189,
"tgt": "Suggest treatment for whiplash",
"src": "Patient: we were just in a car accident and we suffer from a pretty bad whiplash we were given novalgin from sanofi aventis and it doesnt seem to be strong enough to take the pain away! we were told that 3 a day should be enoungh but I would like to know if we can take more Doctor: Hello, As there was an accident there is always a chance of Whiplash Injury. This injury occurs due to sudden thrust to the head and leading to an impression of damage to the spinal cord at the cervical region. I will advise the first foremost thing to do is taking up an MRI. because that will provide more light to the injury by revealing the internal structures of the cervical spine and spinal cord. Its good that you have not developed any major injury. because in whiplash injury things can go otherwise. Thank god that you are all fine and there is only pain. Once MRI is done you will be able to notice the extent of soft tissue involved in whiplash injury. Based on the soft tissue injury, painkillers along with physical therapy will be advised. As with physical therapy like Ultrasound therapy and TENS therapy the pain will reduce and muscle spasm will be reduced too leading to less pain. Any muscles which have gone wasting during the resting phase may need to regain some strength. Please use the cervical collar for time being to avoid more compression of the soft tissue. Hope I have answered your query. Let me know if I can assist you further. Take care Regards, Jay Indravadan Patel, Physical Therapist or Physiotherapist"
},
{
"id": 112928,
"tgt": "Pain in lower back, cramps on left thigh. X ray showed l5 problem. Suggestions?",
"src": "Patient: HELLO DR. I have a pain in my lower back and cramps like on my left thigh some times when i sit i feel better some years ago my x-rays showed that i have an L5 problem so i try to control my weight and go walking i am 47 yrs old and this month i have also missed my menses . my weight is 75 kgs and height is 5'1 . would appreciate your kind response in this matter Doctor: Hello and Welcome to HCM, Thanks for writing to us. The symptoms that you are having are likely to be due to a nerve root compression at the level of lumbar spine(L5). You need to get an MRI scan of the area done and consult a orthopaedician for proper diagnosis and treatment. Hope this helps you. Wishing you good health... Regards. Dr Saurabh Gupta. Orthopaedic Surgeon."
},
{
"id": 114606,
"tgt": "Suggest treatment for pernicious anemia",
"src": "Patient: My husband has pernicious anemia and has had for over 30 years the doctor who diagnose this prescribed two cc every three weeks, our new doctor is questioning this... we had labs done today how do we determine how much B12 he should be receiving? Thank you Doctor: Hi, dearI have gone through your question.I can understand your concern.He has perciniciou anemia. So he need vitamin B12 injection. Because he can not absorb vitamin B12 in dietary source. It can be checked by vitamin B12 level in blood.Consult your doctor and plan accordingly.Hope I have answered your question.If you have any doubts then feel free to ask me. I will be happy to answer.Thanks for using health care magic.Wish you a very good health."
},
{
"id": 94586,
"tgt": "Excessive alcohol intake, chills, nausea, burning sensation in genitals. Normal symptoms?",
"src": "Patient: had too much to drink last night and started having chills during night that lasted off/on into the day today. it has been about 20 hrs now since last drink and still have intermittent chills and nausea . however i just started to get genital burning in the last couple hrs. no pain w/ urinating, no fever . i am wondering if chills are normal w/ hangovers or if uti is possible Doctor: Dear sir, chills are not usual signs of hangover. It can be sign of abstinence crisis. But did u take some other substances except alcohol? Or you took tom much coffee or energy drinks? Secondly this can be urinary infection no matter u have fever or not. You need to do complete blood tests and abdominal and urinary US, also microbiology tests(STD and urinoculture) . And stop drinking of course. Wish you good health"
},
{
"id": 106886,
"tgt": "How can lower backache during pregnancy be treated?",
"src": "Patient: I am 30 weeks pregnant and I have been having terrible lower back pain and really bad pain on my lower right side of my stomach and for the past week I have been loosing what look to be my mucus plug..it hurts my back to do anything expically standing up or walkind and my stomach seems to hurt more when I sit down I have been looking it up and getting to many different answers as to what it could be and my ob is not answering plz help Doctor: you need to take calcium tablets along with mahanaryana taila for application for this type case cap asthi shrinkhala is good"
},
{
"id": 190206,
"tgt": "Gums separating from tooth. Oozing blood and pus, swollen",
"src": "Patient: Behind my lower right tooth(my last tooth) it appears my gum is separating from my tooth. It started off as feeling if something was stuck like a popcorn kernel. I looked at it today with a tooth pick and when I poked it, blood and pus came out. That relieved some discomfort but it still feels swollen. I can like lift up my gums and sort of see what is under there. Doctor: Hello. Thanks for posting your query. You are suffering from periodontal disease. You can go for scaling, root planing, Topical antibiotics are generally the treatment of choice. They can include antibiotic mouth rinses or insertion of threads and gels containing antibiotics in the space between your teeth and gums or into pockets after deep cleaning. Surgical procedure include Flap surgery (pocket reduction surgery), Soft tissue grafts, Bone grafting, Guided tissue regeneration, Enamel matrix derivative application. Wishing you good health. Regards."
},
{
"id": 85369,
"tgt": "What should be the dosage of LIV 52?",
"src": "Patient: I am 50 years old, i have been taking medicines frequently for past 2 years (a course of antibiotics for every alternate months due to amoebiasis, gastric ulcer, multiple fungal swellings) so shall i take LIV52. prescribe its dosage and the time period? Doctor: Hello,Liv - 52 is a herbal liver supplement and if you want to continue for a longer period then you should take it one tablet with breakfast and one tablet with your dinner. You can continue it for 3-6 months.Hope I have answered your query. Let me know if I can assist you further. Regards, Dr. Prabhash Verma, General & Family Physician"
},
{
"id": 65205,
"tgt": "What does boil with pus under right arm for 15 years indicate?",
"src": "Patient: I am 36 year old female. I have a boil under my right arm for almost 15 years now. I take homeopathy medicine. Once doctors have taken contents from inside by actually operating it, without removing the whole thing and found that it does not contain any cancerours cells or anything dangerous. they left it untreated. The boil when active gives puss discharge and pain also. homeopathy helps to subside the pain and stop the pus. but gain after sometime this happens and agin i take homeopathy medicine. This doctor tells me that this is sinus or fistulla and it is not harmful. This will not be cured and will remain. I am 5ft 4 inches and 62 kgs. can anybody suggest what this is i am having? Doctor: Thanks for your question on HCM. I can understand your situation and problem. You are having chronic skin infection with discharging sinus. It is not seen commonly. But common in diabetic patients and patients with compromised immune system. So better to first get done sugar level and immunoglobulin levels to rule out these conditions. Also get done ultrasound examination of local part. You need complete surgical resection of infected area and antibiotic therapy for at least 1 month. Avoid too much moisture and keep proper hygiene of axilla. Don't worry, you will be alright."
},
{
"id": 95883,
"tgt": "Am I having abdominal pain due to gas problem ?",
"src": "Patient: I have severe pain in my abdomen. Everytime I feel like I want to go and sit inside the rest room. Is it due to Gas problem ? Doctor: welcome to healthcaremagic could u pls elaborate the symptoms..where exactly is the pain? upper part of the abdomen or the lower part? improve your diet habits..eat lots of fruits and vegetables..drink about 3-4 litres of water every day..also start taking tablet pantocid-DSR 1-0-0 half an hour before food for about one week.."
},
{
"id": 186281,
"tgt": "What causes small pink colored bump inside the mouth?",
"src": "Patient: A few months ago i noticed a small pencil eraser size type bump inside my mouth. I keep biting that area of my mouth. Ive noticed it hasnt gone away an it has a black dot on the side of it. I have a doctors appointment soon but im worried. I do smoke, unfortunately. By the way, the bump is pink in color except for that spot. It is a little sore because it rubs up against my teeth or bitting it accidently. Doctor: Hello, Thank you for consulting with HCM.The bump which is caused by biting on the cheek is called as traumatic ulcer , These bumps heal only after you get the cause removed, either get the tooth which is causing trauma removed or the sharp cusp of tooth should be grinded.Better you visit a dentist and get complete treatment done.Hope it will help you."
},
{
"id": 187044,
"tgt": "What is the cause and treatment for the lump inside my mouth?",
"src": "Patient: Hi. I woke one day and it seems like there's a lump inside of my left cheek and it feels like everytime I try to close my mouth, my teeth seems to almost bite it. It also has some kind of a white vein in it. It kind of hurts but not so much. The vein might just be the effect of constantly being bitten by my teeth everything I try to close my mouth. I don't know. This is scaring me real bad. Doctor: Hello and welcome.Thanks for sharing your concern.Please do not get worried,it appears that you have got infected teeth in your mouth.This has resulted in periapical infection.Therefore please see your dentist and get the clinical examination done.A small x ray is required to see the underlying pathology.Hope it helps.Thanks.Take care."
},
{
"id": 61578,
"tgt": "Can a painless lump on the forehead lead to hours of dizziness?",
"src": "Patient: Hi I m visiting my mother who is 69 years old and outta no were a lump just appeared on the right side of her forehead which she says doesn t hurt also she says been dizzy for the last 3 or 4 hours she says she never hit her head what might this be thank you Doctor: Hi, what she has is an allergy to something she ate. The dizziness is also due to that. Give her an acid suppressant and she will be fine--regards"
},
{
"id": 195317,
"tgt": "How long will VDM-KIT and Augmentin take to reduce the pain in urethra?",
"src": "Patient: Hi. I'm a male and have pain in my urethra near the tip of the penis. Had unprotected sex a week ago. Have seen a doctor and they did a urine test since there was almost insignificant discharge. I was told I have a bacterial infection and have been given the following medicines: VDM-KIT and Augmentine 625mg one tab twice a day for 5 days. Is this correct? How long can expect the symptoms to disappear and get back to normal? James Doctor: Hello and Welcome to \u2018Ask A Doctor\u2019 service. I have reviewed your query and here is my advice. The kit contain higher doses of drugs they may be single dose after post exposure is sufficient. In case of chronic Infection need 2-7 days course in once a day for five days. Urethritis may be due to urinary tract infection also avoid spicy food and alcohol avoid smoking and caffeine drinks. Drink plenty of water, use urine alkalizer, if symptoms not improved please consult your doctor he will examine and treat you accordingly. Hope I have answered your query. Let me know if I can assist you further."
},
{
"id": 58062,
"tgt": "What could be the cause for having gallbladder problem?",
"src": "Patient: Hi i recently found out that i may have problems with my gallbladder (in June?) and I am scheduled for an endoscopy/gastroscopy in about a week. I had two more attacks since I got back from the ER in june and just had another attack last night. I ate chips last night and avocados so maybe that was the trigger. I was just wondering, if having a gallbladder problem makes me crave fatty foods??? Doctor: all bladder can be due to chronic infections of gall bladder or by deposition of stones by excess calciumgall bladder stores bile to digest fat and as there is blockage the fats cannot be digested causing bladder problemsneed to have fat free non oily non reesy foods plentyof water low fat high fiber low protein diet wiuth digestive enzymes and antacids"
},
{
"id": 24419,
"tgt": "What causes fluctuating BP throughout the day?",
"src": "Patient: I'm 56 yr old female weighing 240 & have started eating healthly & do light water aerobics & indoor short power walks. My b/p has never been really bad, but doc put me on 10 mg of Lisenipril because it was sometimes elevated. For the last few days, I have woke up a little dizzy when taking b/p it is running 116/77,108/65,115/66, 107/69 this is check all thru the day. Yesterday I just took 1/2 Lis. and it still was lower than normal.? Doctor: your blood pressure readings are in normal range. so it is unlikely it has caused for diziness. diziness may be due to sudden drop in blood pressure when you stand from from bed in early morning which is called postural hypotension. so dont stand at once from bed. change your posture gradually from supine to standding. check blood presssure regularly from your family physician. get his advice regarding lisprinol dose according to blood presssure monitering results."
},
{
"id": 126030,
"tgt": "What causes muscle cramps and adrenal fatigue?",
"src": "Patient: I have had adrenal fatigue, and I m experiencing muscle cramping. I take magnesium, should I be supplementing with potassium also. I ve also been told a couple times recently when I ve had some blood work done and your analysis, I m slightly dehydrated. I drink plenty of water. Doctor: Hi, There are many causes of muscle cramps and fatigue. Most common causes are like infection or electrolyte imbalance or myositis or anxiety or anemia etc. Until examination is done it is difficult to say what it is. Get it done serum electrolytes, serum creatinine, serum CPK levels. If symptoms not improved please consult your doctor he will examine and treat you accordingly. Hope I have answered your query. Let me know if I can assist you further. Regards, Dr. Penchila Prasad Kandikattu, Internal Medicine Specialist"
},
{
"id": 9314,
"tgt": "What causes skin itching and shedding with dryness?",
"src": "Patient: I itch constantly. It always feels like there are little microscopic things crawling around or me or something. I have dry skin, and my skin sheds off when i m in the bath tub... Literally. I can run my hand down my arm and the skin just rolls up into a ball and i have to wash it off... I have also what appears to be pigmintation blotchs on my arms, neck, chest, and stomach that turn red when I get hot. This can actually be scrapped off with a nail in a matter of seconds but then it leaves a red blotch where the patch once was. What in the world is wrong with my skin. Doctor: Hello, I can understand your concern for skin itching and shedding with dryness.You have not mentioned your exact medical history including your age so that I could have figured out the most probable cause of skin itching and shedding with dryness.In my observation it could be due to due to contact dermatitis due to rubbing and contact with an allergen of day to day life. I suggest you to not to use any cosmetics or related thing on face. DO not wear any synthetic garments touching the neck area or any woolen garments in contact with neck and back.I suggest my patients to apply a cream containing mild steroid such as fluticasone over the affected area. I also suggest my patients to take antihistamine like Levo-cetrizine.Hope the information helps."
},
{
"id": 153087,
"tgt": "How can urethra burning sensations, urinary frequency with less voiding be treated?",
"src": "Patient: I have constant burning in urethra. I had T1G3 grade papillary carcinoma of bladder. I had undrgone 2 TUR Bt, and BCG treatment. Now I have constant pian in urethra and burning sensation. Pass little urine, but constant ureg to urinate. Doctor told it is UTI, tried several antibiotics based on culture, but no effect. Doctor: Hello dear. The usual causes of burning sensation and increase urge in such cases are urinary infection, BCG reaction and reappearance of tumor. You should discuss these options with your doctor. You may also be required to undergo test for these conditions.Thanks"
},
{
"id": 48261,
"tgt": "Do i need a kidney biopsy for a decreasing kidney function?",
"src": "Patient: I am a 54 year old female and had a severe nephritis infection when I was 6. Since then I have had 3 more serious bouts of nephritis. 3 other members of my family also have blood in the urine, but I occasionally get protein in the urine too. My kidney function has decreased in the last three years and my doctor is now concerned. Having seen a specialist it has been suggested I have a kidney biopsy. However, I am extremely weary of such an invasive procedure, especially in the light of the fact that I am actually very healthy in general. In the end it is my decision, but I am finding this a very difficult one to make. Any help will be gratefully accepted. Doctor: Hi,Thanks for writing in.The kidneys are organs which filter out harmful and toxic substances from the blood In this way the urine is formed. This happens through a complicated process in the kidneys and involve stages like glomerular filtration. This is a micro system that causes blood components to flow through small and the main filtration occurs at basement membrane in the glomerulus. When there is proteins seen in the urine in higher quantities than normal then a problem in the basement membrane must be suspected. This also might happen in those who have had nephritis earlier in their life.Your kidney functions might be above normal and it is important to evaluate the excretion of proteins by doing a biopsy. It is done using ultrasound guidance and considered safe and less painful and done under local anesthesia. The biopsy will show any problems in the kidney glomerulus and basement membrane and guide your treatment. Please do not worry."
},
{
"id": 122518,
"tgt": "What causes pain in forearm & muscles?",
"src": "Patient: my left hand starting hurting about an hour ago...near my thumb...now the pain is going up in my forearm...and slowly to the upper arm.... hurts very bad. nothing else seems to hurt...but i can t sleep...as the pain is hurting so bad. the muscles just feel so weak...like my arm is about to fall off. Doctor: Hello, I would explain that your symptoms could be related to a pinched nerve. For this reason, I recommend taking ibuprofen for the pain and having some rest. If your situation does not improve, going to the ER may be needed, for a physical exam and some blood lab tests (a cervical spine X-ray study, complete blood count, PCR, ESR for inflammation). Hope I have answered your query. Let me know if I can assist you further. Regards, Dr. Ilir Sharka, Cardiologist"
},
{
"id": 187317,
"tgt": "What is the remedy for the dryness and burning in the mouth?",
"src": "Patient: I used antiseptic mouth and throat rinse help reduce discomfort of my ulcer that was in my mouth. Now my ulcer is completely healed but my mouth feels like it s raw and the roof of the mouth feels as it s been burnt by acid. Will this go away if I avoid acidic food? I m going to see a dentist on Tuesday next week p.s I stop using the mouth wash Doctor: Hello, thank you for with consulting with HCM.As your ulcers are healed now, then you can discontinue with the mouthwash , as sometimes the components of mouthwashes causes discomfort and metallic taste in mouth.If after discontinuing the mouthwash also you experience burning sensation then you have to visit dentist for complete examination of oral cavity.Hope it will help you."
},
{
"id": 139614,
"tgt": "Is thickening of cover of brain a cause for concern?",
"src": "Patient: I had a CT scan of my brain yesterday secondary to some dizziness that has been occuring for a month or so. My MD called and said I need to have an MRI in 6 months as a follow-up to thickening of the cover of my brain . I heard her say meningioma but she assured me I did not have a brain tumor but was referring to the meninges covering the brain. She assured me I was OK but I am still concerned. Should I be? Doctor: Hello and welcome to \"Ask a Doctor \" service.I have read your query.Thickened brain coverings ( meninges ) may indicate inflammation.Meningioma is easily assessed with imaging, so, I don't think it's your case.Don't worry much more than necessary, follow up with your Doctor.Hope this helps.Let me know if I can assist you further."
},
{
"id": 111211,
"tgt": "How to treat for the back pain?",
"src": "Patient: Hi, I am a guy 21 years old 6 feet, 75 kgs i am having problem with my back. Its aching badly.It started 7-8 days ago but it was mild then, it increased when i asked my mom to massage it yesterday, please if any suggestion so as to decrease the pain i'm having... Doctor: Hello,I had gone through the case and found that it might be muscle spasm or slip disc.So take mild painkiller and apply muscle relaxant gel.Do not bend forward and avoid to lift heavy weight.If pain increases then go for MRI and after that go for physiotherapy.Hope my answer will be effective for you.Thanks"
},
{
"id": 40431,
"tgt": "Can infection cause stomach upset?",
"src": "Patient: I was just diagnosed wth strep... I was told that if looked as though I may also have perialtisilitus also... I have to see how I feel in the morning... If it s no better, go to ER... My stomach feels like its trying to get upset... Is that caused from infection? Doctor: Hello,Welcome to HCM,Infection in the throat if it is not treated properly the infective agents can disseminate to other parts of the body.If the proper antibiotics is not taken definitely it can systemic infection affecting other systems also.If you swallow the infective agent definitely it has got all the possibility that it will causes the symptoms of the affected system.I would suggest you to take appropriate antibiotic and control spread of infection to other parts of the body.Thank you."
},
{
"id": 107743,
"tgt": "Suggest treatment for severe back pain",
"src": "Patient: I AM DIAGNOSE WITH DIAGHRAGMATIC WEAKNESS AT LONG ISLAND JEWISH THE DOCTORS CLAIMS IT WAS LIKELY DUE MY DIABETES I CANNOT LIE ON MY BACK BUT THEY DID LOTS OF TESTS AFTER TWO WEEKS I WAS DISCHARGE THAT WAS 4/1/2110 SINCE THAT TIME I HAVE BECOME WORST CANNOT WALK ONE BLOCK WITHOUT STOPPING FOR BREATHTWO OR THREE TIMES ALTHOUGH I WALK VERY SLOW I CANNOT LIFT ANY THING MORE THAN FOUR POUNDS ISLEEP SITTING DOWN THANK YOU PL HELP YYYY@YYYY Doctor: First of all you have not mentioned your age which is very important in case of back pain. For treatment take some pain killer preferably biozobid-plus or osteonac-th along with some calcium supplements like toscal-gem or shalcal-d once daily. Take some multivitamins like metilda-forte or meganeuron-plus once daily. You can do some physiotherapy exercises after an expert opinion of physiotherapy. I will also advice you to sleep on the plain surface."
},
{
"id": 147423,
"tgt": "What causes hand tremors?",
"src": "Patient: My 17 son has hand tremors that we just noticed. They are worse when extending his arms, not so bad if he is holding something. He has a great uncle who had parkinsons. Is parkinsons genetic? I will make an appointment for him tomorrow, but it seems very odd for an otherwise very healthy kid. Doctor: Hi,Thank you for posting your query.I have noted your son's symptoms. It is unusual to have symptoms of Parkinson's disease (PD) at a young age of 17. Also, the other features of PD such as slowness of activities and rigidity of arms/legs are not there, so, I do not think that your son has PD.PD can be genetic in about 10% of cases.Neurological examination would sort out the diagnosis.I hope my answer helps. Please get back if you have any follow up queries or if you require any additional information.Wishing you good health,Dr Sudhir Kumar MD (Internal Medicine), DM (Neurology)Senior Consultant NeurologistApollo Hospitals, Hyderabad, IndiaClick on this link to ask me a DIRECT QUERY: http://bit.ly/Dr-Sudhir-kumarMy BLOG: http://bestneurodoctor.blogspot.in"
},
{
"id": 130617,
"tgt": "What causes pain in hip and shoulders?",
"src": "Patient: iam having pain in my right hip, pain in front, same leg at the joint in front. knee pains, shoulder pain,dr. said I have a begign tumor in right knew and lots of arthritis, now it seems to be everywhere,even some in leg below my knee to the outside area, I also suffer from RLS/// is there any answer? Doctor: Hi,What you have described there are two possible reasons for your joint pains. First, it can be due to arthritis (most probably rheumatoid arthritis). For this you need to take medication prescribed by orthopedic based on blood test II. Second possibility is that your benign tumor is no more benign (has become malignant) and has spread to the adjoining areas like hip joint and your ankles, the possibility of it straight to the shoulder is rare.To rule out these two possibilities you need to consult your nearby orthopedician or a rheumatologist. Till then you need to take rest and to reduce pain you can take Ibuprofen for the time being but this will not cure your disease. This is just a temporary pain reliever.Hope this helps you. Let me know if I can assist you further.Regards,Dr. Jenis Bhalavat"
},
{
"id": 199788,
"tgt": "What is the sensation of irritation on the tip of my penis?",
"src": "Patient: Hey i have a sensation of a irritation like thing on the tip of the penis head and sensation of pie every time i consulted with a urologist and after examination it reveals that i have renal calculi... does this symptoms appear or very if i have renal calculi ? Doctor: HiThank you for asking HCM. I have gone through your query. Uric acid crystals or small stone particles can give such irritation symptoms on tip of penis. Taking more water, potassium citrate in water, barely water or cranberry juice will be helpful in alkalizing urine and minimising such symptoms. If still symptoms persist consult your urologist for urinary antispasmodic like flavoxate. Hope this may help you. Let me know if anything not clear. Thanks."
},
{
"id": 60728,
"tgt": "What could be the cause for a small bump under my armpit?",
"src": "Patient: Hi there goodnight! About 2 months ago I got a small lump under my armpit. At first, it was painful. It would burn at times. But after a week or so, the pain stopped and the lump grew but not big, you could feel and tell the difference though. To this day I still have it. It doesnt hurt. Doesn t move around or anything like that. Do you know what it could be? Doctor: Hello, It could be either an enlarged lymph node or a healed furuncle. I would recommend you to consult a doctor and get it evaluated. Hope I have answered your query. Let me know if I can assist you further. Take care Regards, Dr Asmeet Kaur Sawhney, Dermatologist"
},
{
"id": 206848,
"tgt": "Suggest remedy for insomnia & loss of appetite",
"src": "Patient: Good morning Doctor, myself Mrs Sindhu working as lecturer. For the past 2 months i'm not getting proper sleep, not interested in having food. Having alot of mental stress. Not able to concentrate on anything, feeling alot tired, sometimes becoming unconcious. Can you please suggest any remedy for this. Doctor: DearWe understand your concernsI went through your details. I suggest you not to worry much. From the description a proper diagnosis may not be possible. But it is almost sure that you are overly stressed. All these symptoms may be related to stress and related anxiety. You are tired, stressed. The best method to come out of this situation is to take rest for a few days and if possible go for a vacation. Enjoy the vacation and you will be alright. Regarding unconsciousness, if it is true, you must visit a physician and get yourself checked.If you require more of my help in this aspect, Please post a direct question to me in this URL. http://goo.gl/aYW2pR. Make sure that you include every minute details possible. I shall prescribe the needed psychotherapy techniques.Hope this answers your query. Available for further clarifications.Good luck."
},
{
"id": 175585,
"tgt": "Suggest treatment for cough and fever in a child",
"src": "Patient: My daughter is coming in and out of fever for one week now. Whenever she takes tylenol or advil the temperature drops and sometimes come back after a few hours and sometimes it doesnt.The symptoms are moderate to high fever, and an occational caughing or sneezing. Doctor: HelloFever if persisting for more than 4-5 days with same or increasing severity,should be investigated for other causes as viral fever usually starts decreasing after 4-5 days.Although in bacterial infection child looks more sick(lethargy,decreased feeding,respiratory difficulty etc)but it should be rule out if fever is not responding. Urinary tract infection may also cause fever(there may or may not be urinary problems).Make a temperature chart to know the exact pattern of fever.I would suggest you to consult a pediatrician to rule out other causes of fever, as it is unusual for a viral fever to persist for such a long duration.Regards"
},
{
"id": 78180,
"tgt": "What causes labored breathing with dry cough?",
"src": "Patient: for three days my wife has felt like breathing has been difficult in an asthmatic kind of way. Labored breathing with a dry cough. She just had a yearly checkup and was fine. no other health issues and she keeps herself fit. No alcohol or tobacco.any insights? Doctor: Hi. I can understand your concern. It would be advisable for you to get a chest x ray done. If it is normal then get a PFT done which will tell if she ahs asthma or not.Don't worry, you will be alright. Hope I have solved your query. Wish you good health. Thanks."
},
{
"id": 189838,
"tgt": "3 years old have sore palate with high temperature and loss of appetite. Sign of ulcer?",
"src": "Patient: My 3 year old child has told me the roof of her mouth is very sore and she has had a high temp. Also has loss of appetite but that may be due to her not wanting to eat due to the soreness. I have had a look in her mouth and can see no obvious signs of redness or ulcers or any kind of soreness. This came on quite suddenly yesterday. What could this be? Doctor: Hi! Soreness in the roof of mouth (palatal region) or ulcers can be due to multiple reasons, the most prominent being an upset stoamch or a previous course of certain antibiotics. The child may also feel soreness due to intake of hot or spicy food. The best thing would be to get it clinicaly assessed by a dentist nearby. Also, it is possible that the child is unable to localise the soreness which may be towards the throat or tonsil region as the rise in temperature could be due to some throat infection as well which might be posing difficulty for the kid to eat. I hope this information would be of some use to you. Warm regards!"
},
{
"id": 99208,
"tgt": "Suggest treatment for severe asthma",
"src": "Patient: I am a 33 year old female. I have severe asthma. I have been having massive attacks since late December and hospitalized 3 times for exaserpation. I have been to the ER about 5 times, most recently, on Monday, sent by my PCP. I am on a back-up tank of oxygen that my Mom happened to have on hand right now. My nebulizer isn't helping. My chest is burning in my sternum area and I am wheezing. My family can't continue to take time off to take me to the ER and my sister is in labor at a hospital about 30 minutes away. Can you please advise? Thank you! Doctor: Dear, You have severe persistant asthma not responding to nebulization. well you have not written what drugs you are taking in nebulization. Steroid nebulization(beclomethasone/budecort) if you are not taking , will definately help. You need oral steroids to control your symptoms as well . this is a prescrption drug."
},
{
"id": 53737,
"tgt": "What causes deranged liver function test after taking Cortisone injections?",
"src": "Patient: Hi, I have taken an injection of cortisone to treat anaphylaxis from antibiotic and after one month I had the same attck from another antibiotic then I mad liver function teste after one month from the second injection and I found iT High Is it from this 2 injections? Doctor: Hi and welcome to Healthcaremagic. Thank you for your query. I am Dr. Rommstein, I understand your concerns and I will try to help you as much as I can.Steroid injections are known to cause such liver changes and abnormalities in liver markers so this is not uncommon. In most case there is no serious damage and after few weeks markers should stabilize. If there is still high liver function tests then ultrasound should be done.I hope I have answered you query. If you have any further questions you can contact us in every time."
},
{
"id": 126283,
"tgt": "Suggest treatment for stiffness in the wrist despite having physical therapy",
"src": "Patient: i had a ganglion cyst removed October 31, 2017 (seven weeks ago). I still cannot bend my wrist downward. doctor says its stiff and put me in therapy. after 2nd PT visit and home exercises each day, it doesn t seem any more loose than before. I have some numbness on the top of my hand and incision still sore. Doctor: Hi, It can commonly happen in a wrist surgery. Generally, it will settle by itself over time. If symptoms persist you can consult an orthopedician and get an MRI scan done. Hope I have answered your query. Let me know if I can assist you further. Regards, Dr. Shinas Hussain, General & Family Physician"
},
{
"id": 158601,
"tgt": "History of pelvic pain, postcoital spotting, high risk HPV positive, severe dysplasia. Done pap test. What do I expect?",
"src": "Patient: Im a 30 y/o with a recent 6 month history of pelvic pain and postcoital spotting. i followed up with my current OB/GYN in March, pap revealed ASCUS with high risk HPV positive, colpo later the next week showing moderate dysplasia; CIN 2, LEEP procedure preformed May 6th; of the 4 specs removed, all revealed severe dysplasia CIN 3 with dysplasia present in all 4 margins incuding endocervical margins. My Dr is recommending repeat pap in 6 months. I have diagnosed polycyctic ovaries, which have also become a bother again, 2 beautiful children (plans for no more-thus my husband's vasectomy 4 yrs ago) and a maternal grandmother who's deceased post Stage 4 cervical cancer. Is this a reasonable wait or should I seek a second opinion? Thank You!!! Doctor: Hi, I can understand your concern. Nothing to worry right now. CIN does not mean invasive carcinoma. There is a dilemma regarding management at this stage of disease. Preferred mode is follow up after 6 months with repeat pap smear. More radical approach like hysterectomy may be required but no universal consensus. So you may better opt for 6 monthly follow up approach . This is an accepted way to follow up."
},
{
"id": 105487,
"tgt": "Depiwhite cream caused redness and irriation when applied to dark circles. Is it a reaction?",
"src": "Patient: HELLO SIR, DOCTOR HAD SUGGESTED ME YOUR CREAM FOR USING IT ON DARK CIRCLES i.e(depiwhite under eye gel) BUT DUE TO CERTAIN MISTAKE CHEMIST HAD GIVEN ME DEPIWHITE CREAM..I HAD APPILIED IT ON MY DARK CIRCLES FOR DAYS REGULARLY WHICH HAD CAUSED A REACTION (REDNESS ON DARK CIRCLES,IRRITATION )AND WHEN I HAD SHOWN IT TO MY DOCTOR HE SAID TO USE DEPIWHITE UNDER EYE GEL .. BUT THERE ARE SOME FOLLOWING QUERIES WHICH ARE AS FOLLOWS 1 IS IT A REACTION ? 2 IF IT IS A REACTION HOW MUCH TIME IT WILL TAKE IT TO OVERCOME ? 3 FROM WHEN TO APPLY DEPIWHITE UNDER EYE GEL AFTER REACTION IS TREATED OR I CAN APPLY IT ON THAT REDNESS PART OF DARK CIRCLES. Doctor: Hello, It appears you may have a chemical hypersensitivity to one of the excipients of the cream and it is important that you see a dermatologist with interest in chemical sensitivity to get a patch test done to exclude this. You can apply a local antihistamine cream or take an antihistamine tablet for a few days to treat the present reaction. It normally takes a few days (less than a week typically) to overcome this reaction."
},
{
"id": 67845,
"tgt": "Cause for lump on the back of the leg?",
"src": "Patient: I have huge lumps that go from the back of my legs all the way around the front of my leg on both legs. The Dr said they are water cysts and I didn't getvany other information at this time. I was wondering if you knew where i could read a article? Or if you had any info? Doctor: Hi, dear. I have gone through your question. I can understand your concern. You may have some benign cyst or ganglion cyst. You need surgical excision of that lump. Consult your doctor and plan accordingly. No need to big worry. Hope I have answered your question, if you have doubt then I will be happy to answer. Thanks for using health care magic. Wish you a very good health."
},
{
"id": 25707,
"tgt": "Suggest treatment for celiac artery blockage",
"src": "Patient: Hi, I am a 33 year old female and I recently had an angiogram done in which showed my celiac artery was all the way blocked. This test was done due to constant abdominal pain, sternum pain, and weight loss. They will not do a bypass surgery given the risk and my age; therefore, can I live with a blocked celiac artery? Doctor: Thanks for your question on Health Care Magic. I can understand your concern. No, you can not live longer with blocked celiac artery. You need to treat it first. If not going to treat you will develop bowel ischemia, obstruction, infection etc. And all these are life threatening complications. So you need to treat yourself as soon as possible. Removal of blocked segment of celiac artery and anastomosis with venous graft is treatment of choice for this. For this part of your calf vein is used. And success rate of this bypass graft is very high. So you should consult vascular surgeon and discuss all these. Don't worry, with proper treatment, you will be alright. Hope I have solved your query. I will be happy to help you further. Wish you good health. Thanks."
},
{
"id": 14801,
"tgt": "Why am I getting red rashes all over my body?",
"src": "Patient: Two days ago I noticed a rash (flat, barely red, not very itchy) I took benedrl, put hydrocortisone crem on it and in the morning most was gone. Before bed it was in new places and more. Did the same as night before now almost gone again. I m 55 and in good health excersize . What is this Doctor: Hello and welcome to healthcaremagicI would keep a possibility of Acute Urticaria. It can present as itching and skin welts/swellings all over the body.Urticaria OR hives can occur due to various causes like(to name a few):--Infections(bacterial , viral, fungal) --Drugs(Pain killers/ antibiotics etc) --Certain foods and food additives/preservatives --Physical Urticaria: Cold temperature, Heat, Pressure(Prolonged Standing, Sitting), Dermaographism etc. --Cholinergic Urticaria: in response to strong emotions, exercise, spicy food etc --Autoimmune disorders like SLE, RA etc. --A few cases have no underlying cause and are known as idiopathic urticaria. I would advice you to get few basic lab tests like Hemogram, stool for ova/cyst, Urine routine, Chest X-Ray.An oral antihistamine like Fexofenadine (Allegra 180 mg tab) OR Loratadine (Alaspan 10mg tablet) once or twice daily for a week would be suitable. They are non-sedating and wont affect your alertness level. A topical soothing lotion like calamine would help you symptomatically. take care regards"
},
{
"id": 99068,
"tgt": "What causes muscle pain , fatigue and allergy?",
"src": "Patient: I am a 45 yr old female. I have been experiencing a wide range of symptoms wich continue to get increasingly worse over the coursevof the past 3 months. They are as follows: severe fatigue, vision impairment, muscle aches and tenderness, joint discomfort, severe allergy like symptoms, stabbing mortins neuroma like foit pain, eye twitching and general nervous/anxious feeling. Any ideas whatvcould be causing all these symptims to present atvthe same time and continue tobworson Doctor: HIWell come to HCMI really appreciate your concern, such symptoms may not be due to any disease but this could be due to menopause, you have control your thoughts, try to come out of this just be positive and practical, try some yoga, meditation, relaxation therapy, hope this information helps, take care and have a nice day."
},
{
"id": 48613,
"tgt": "What does this CT scan of abdomen and pelvic area indicate?",
"src": "Patient: Hello my first question, I had a CT Scan of my abdomen and pelvic done and it showed a 3 mm spot on my liver....should I worry? My second question is..I have pain in my left side in my back around my kidney...burning pain? It comes and goes...it is not continuous.. That was the main reason I had a CT scan done. They said everything looked normal on the CT Scan other than the 3 mm spot on my liver. They cannot understand the pain I have in my left side back. is it my kidney? Thank you. Doctor: Hi,Thanks for writing in.In my opinion a 3 mm spot in your liver need not be a concern right now. A 3 mm spot is too small to characterize by CT scan and this needs to be followed up probably in 6 months or a year. A description of the liver spot might not be accurate enough to label it as a concern.During CT scan abdomen, the kidneys are assessed and I do not think there is any problem with your kidneys which can be causing the left back pain. If the abdominal organs are normal then focus should be on the bones and muscles at the back.Another cause of burning pain not visualized on CT scan abdomen is gastritis. This pain usually intensifies after long hours of fasting and following intake of spicy foods. You might discuss this with your doctor."
},
{
"id": 159893,
"tgt": "Can I get any cancer because I masturbated near a computer and mobile phone ?",
"src": "Patient: I was using a Wireless WiFi modem and after working on the computer for about half an hour I masturbated with the computer close to my face and with the modem connected. I also had a mobile phone near me. Can I get any cancer because I did like this? Doctor: No. You will not get cancer by a single mobile phone exposure. Though of late many studies suggest the bad effects of mobile radiations in the long term- in causing cancer. So, take precautions that anyone would with your mobile."
},
{
"id": 140309,
"tgt": "I venous angioma and seizures are interrelated?",
"src": "Patient: My daughter has a venous angioma and is experiencing possible focal seizures, she also has postural tacicardia syndrome and narclepsy do you think this is all tied together. Her hands begin shaking unable to write feels like she cannot walk & feels really funny. After she is very tired and nausiated. Doctor: Hello, Focal seizures may be linked to venous angioma of the brain, however, EEG study is necessary to confirm this. Narcolepsy and postural tachycardia syndrome, on the other hand, are not related to venous angioma. Hope I have answered your query. Let me know if I can assist you further. Regards, Dr. Erion Spaho, Neurologist, Surgical"
},
{
"id": 70323,
"tgt": "Suggest treatment for painful lumps under both armpits and groin area",
"src": "Patient: I am terrified!!! I regularly have several lumps under both armpits and groin area. These lumps come and go, and are sometimes painful. I regularly wake in the night soaked in sweat. Also recently I have started noticing lower back pain. The pain is concentrated on both the lower right and left side. It is a dull, aching pain that seems to worsen at night and while I'm sitting. I should also mention that overall I just don't feel myself. Are these all signs of something serious I should look further into? Doctor: Hello!Thank you for the query.Yes, this symptoms can indicate serious issue and you should not wait with it any longer. The lumps you have under your armpits and groins are lymph nodes. Along with mentioned symptoms you may have a Lymphoma or chronic infection. Night sweats, weakness, weigh loss are quite characteristic for Lymphoma symptoms.I suggest you to consult hematologist as soon as possible. Blood work and lymph nodes biopsy should be done.Hope this will help.Regards."
},
{
"id": 155830,
"tgt": "Does high blood sugar level increase the growth of cancers?",
"src": "Patient: I would like to know if having high blood sugar can complicate or speed up the growth of cancers? specifically, Non hodgkin lymphoma, stage 4 matastasized to lung and kidney. completed 5 out of 8 recommended chemo treatments and had no sign of cancer on pet scan as of march 2014. but patient has had type 1 diabetes for most of his life, is 45 years old and ever since the chemo has not been able to keep blood sugar stable and within target range. Average blood sugar is in the high 200 s and sometimes as high as 400. Considering that glucose injected into patients prior to a pet scan to reflect the areas of the body that have cancer cells, does having high blood sugar in general happen to feed the cancerous cells or make them grow faster? Doctor: Hi Welcome to HCMI Have gone thru your query regarding increase in growth of cancers while having high blood sugar . Dear though there is no such study has been reported But meanwhile, there's no reason not to get blood sugar under control .Doing so may help prevent diabetes and heart disease, besides possibly making cancer less likely .That will be like killing 2 birds w3ith 1 stone .Keeping blood sugar levels within the normal range \"may reduce cancer risk,\" write the researchers, who included Par Stattin, MD, PhD, of Sweden's Umea University Hospital. Illness or stress can trigger high blood sugars because hormones produced to combat illness or stress can also cause your blood sugar to rise. Same is the case with cancer . Constant stress ,illness ,worry , anger . lack of nutritious food & lack of exercise do cause cancer ,The study doesn't prove that high blood sugar levels cause cancer or that normal blood sugar levels prevent it. And A freindly suggestion is if take care of life style and regular food habits - nutritious food containing all essential nutrients and supplements full of antioxidents -and regular ,walk , exercise ,yoga ,pranayam -deep breething proper rest , sound sleep , meditation and positive atitude towards life . and avoidance of fried , fast foods , tea ,coffee , alcohol , smoking , mental stress worry , anger and constipatin , all factors can help incease strength of your immune system , help resistance from dieseases (may it be diabetes ,heart , or cancer or any big or small) , paving way for happy ,healthy, diseasefree lifeHope .this helps yousolve your query Take care .All the best Don't hesitate to get back for further query ."
},
{
"id": 42166,
"tgt": "What are the chances of pregnancy having low follicular size?",
"src": "Patient: i am 30 years i am maried ,3years no child.i got my follicle ultasound report on 11th day at right ovary 1 small,left ovary 2small,endometrium 6.2mm.i take injection endogen hp75(d3-d7)gynogen hp75(d8-d9)persinal hp150(d10-d11) but no goood result.so that good suggesion me. Doctor: go for the ivf techniques they will help you a lot. i t can be solve easily. so dont worry go for it. i wish you will be mother sonn."
},
{
"id": 133535,
"tgt": "What causes pain in right leg causing difficulty in climbing stairs?",
"src": "Patient: sir, i m 58 yrs old. weight75 kgs and height5 feet. i m suffering from severe pain right leg. not able to sleep when turns leftside when the right crosses. not able to climb steps , buses, trains.taking nucart-oa tabs and seacod caps. kindly advise Doctor: You may suffering from some type of neurogenic prb. So need to consult to neurophysiotherapist at home you can do stretching exerciseuse fearm bedding before going to bet take hot water bath on lower limbor use ice pack on pain site."
},
{
"id": 214292,
"tgt": "Suggest home remedy to stop loose motions in infant",
"src": "Patient: my son is having loose motion after eating some strong food containing butter biscuits with milk. he is 13 months of age. what are the home remedies?stools are sticky no blood stains not wattery,free flowing instead if semi solid. frequency 8 times past 6 hours... he z been given ORS presently. kindly advice. Doctor: Please give him probiotics like Enterogermina twice daily.Continue ORS. Watch for signs of dehydration. If signs present/ drowsy/ no urine output, take him to the doctor right away.ORS, bread toast, diluted curd/ buttermilk, tender coconut water / rice gruel can be given."
},
{
"id": 94104,
"tgt": "Pricking sensation in abdomen, back. Numbness on left side. Cannot sit for long time. Fatigued. Solution?",
"src": "Patient: I get a pins and needle type feeling in my abdomen and back this occurs more often in the last year. I can t sit very long with out my legs n feet falling asleep. And I have experienced numbness to my left side. I m so fatigued in the afternoon n no matter how much sleep I get. Even taking Adderall for ADHD doesn t keep me awake. I m really wanting to know what s going on. Doctor: Hi welcome to Health care magic forum. Thanks for choosing H.C.M.F. Actually you have two diseases, A.D.H.D you are facing since long, so the thing you are suffering now is entirely different, and should be treated now. it appears to be the renal colic due to urinary tract infection or uro lithiasis. I advise you to consult an urol ogist for diagnosis and treatment. You may need to have M.R.I.of abdomen besides other routine tests for confermation. wishing for a complete recovery. Best regards."
},
{
"id": 206019,
"tgt": "How is pachygyria treated?",
"src": "Patient: I am an LPN that is taking care of my little cousin who is a 3yr old m and he was diagnosed with pachygyria and in february developed seizures. They re not grandmal, he stares, then rolls his eyes in the back of his head, stiffens arms flexed and legs straight. Of course, as you know he has low muscle tone and I work with him by doing ROM, putting wrist supports and DAFOS on him for atleast 2hrs a day, and also put him in a stander, which he hates, but I m willing to try anything to improve his quality of life. I just really would like his parents to get a little sleep. He s got an appt. w/ a sleep specialist on friday and a sleep EEG in October. I just wanted to know if you had any suggestions. I d appreciate any tips. Thank You, Dee-Dee Doctor: Hello thanks for asking from Health Care MagicPachygyria is a congenital disorder with malformation in brain. It usually manifest as seizures, mental retardation, developmental delay. Your cousin is 3 years old and he is having sudden abnormal movements of extremities with rolling of eye balls and no loss of consciousness. Most likely these are partial seizures. He is on DAFOS for foot deformities. One thing that need immediate attention is his seizure. Repeated seizures may cause damage to brain neurones which will further potentiate the mental retardation. So control of seizures should be first priority. He has schedule of EEG and after making diagnosis he should be given anti-epileptic drugs. Drugs like Sodium Valproate, Carbamazepine etc are broad spectrum drugs and can be given to him with proper consultation of his doctor. Second if possible get his CT scan done to find out extent of brain damage. For sleep you can consult his sleep specialist. Provide him good care to prevent progression of disability.Thanks, hope this helps you."
},
{
"id": 77174,
"tgt": "How to treat III -defined reticulonodular densities on left base?",
"src": "Patient: i done a chest xray today and the findings areIII- defined reticulonodular densities are seen on the left basethe rest of the lungs are clearpulmonary vessels are within normal limitsthe heart is normal in size and configuration diaphragm and bony thoarx are intactIMPRESSIONCONSIDERED BASAL PNEUMONITISmy question is this can i cure this? can this lead to scarring of my lungs? do i have the scar already? what medicine i can take? Doctor: Thanks for your question on Healthcare Magic. I can understand your concern. Yes, this is curable. Chest x ray picture is suggestive of pneumonia (lung infection). And it is mostly due to bacteria. So antibiotic drugs are needed to treat lung infection. With proper treatment, pneumonia can be treated fully without scarring of lungs. So better to consult pulmonologist and get done clinical examination of respiratory system. You will need higher antibiotic like levofloxacin or Azithromycin or other. Along with antibiotics, other supportive drugs like mucolytic, expectorant, antihistamine and anti inflammatory drugs are needed for faster recovery. Don't worry, with appropriate treatment, you will be alright. So consult pulmonologist and discuss all these. Hope I have solved your query. I will be happy to help you further. Wish you good health. Thanks."
},
{
"id": 68236,
"tgt": "What causes redness,swelling and warmth in my calf muscle?",
"src": "Patient: I have an area on the upper part of my left calf that is red, swollen and warm to the touch. I can feel a lump and it hurts when I touch the lump. I don't see any bites or broken skin. I had vein procedures 3 months ago. Could it be a clot? I have no pain in my leg other than that specific area. Doctor: Welcome to health care magic. 1.The history and symptoms suggest possible cause could be post operative infective changes at the site or adjacent to that area.2.And next possible cause recurrence of the venous pathology which you she operated for. 3.In this case after examination - an ultrasound examination will be the investigation of choice.4.The ultrasound will exactly identify the nature of the lump, its source and extensions, in case of vascular channels identification of case, presence of lump and flow will be detected.5.Sugget to visit your GP and request for an ultrasound and doppler study for the limb - the treatment will depending on the findings. Good luck.Hope i have answered your query,any thing to ask do not hesitate to ask.http://doctor.healthcaremagic.com/doctors/dr-ganesh/62888"
},
{
"id": 124582,
"tgt": "Suggest treatment for ankle sprain",
"src": "Patient: i sprained my ankle and i went to the doctors friday and got an ankle brace the thing is some people say it takes a few months before your really healed is that true. i mean i only sprained it 2 weeks ago and just got it looked at friday when it was still hurting. Doctor: Hello, Take absolute bed rest for couple of days. As of now you can use analgesics/anti-inflammatory combination like aceclofenac/serratiopeptidase for symptomatic relief. You can apply icepacks for faster recovery. Generally the symptoms will settle in couple of days. If symptoms persists better to consult an orthopedician and plan for an MRI scan. Hope I have answered your query. Let me know if I can assist you further. Regards, Dr. Shinas Hussain, General & Family Physician"
},
{
"id": 43694,
"tgt": "Trying to conceive. Taking Bigomet, Fertomid, and Grapex. What can be done?",
"src": "Patient: Hi Madam, I got married in July 2011 and we have been trying for a kid since last 1.5 years. I did all the test prescribed by my doctor but we haven t got any positive response from any of the doctor. Following are the tests and the results which we have done so far: 1. Follicular Monitoring - Egg got ruptured on 14th day. - everything was normal but unable to conceive. 2. Semen Analysis of husband is done - Spirm count was normal - 60 million/ML 3. Prolactin Test Result - 20.07 ng/ml 4. Insulin fasting - 13.8uIU/mL 5. Insulin PP - 62.93 uIU/mL (test done after 90 min.) 6. Glucose fasting - 87.6 mg/dL 7. Glucose PP - 95 mg/dL 8. TB-Gold - Positive (1.86) Next test which I need to do is for Moutax Test which I will be getting it done tomorrow. Medicines which were prescribed to me are as following: Bigomet - 500mg, B-Long F, Fertomid - 50mg, Grapex, Folsafe We have been trying for 1.5 years and no results so far. Kindly suggest what can be done now. Doctor: Hello, Im glad to answer your question. Since TB-Gold is positive, you are likely to have active tubercular infection. Next step would be to get a HSG done to know exact site and extent of tubal blockage. Fallopian tube is most common site of damage in genital tuberculosis. The Mantoux ( tuberculin) skin test is unreliable. It tests merely for the presence of immunity against TB . Resection and reanastomosis surgeries are advised If there is isthmal block , then hysteroscopic directed procedure is done. Meanwhile continue taking antitubercular drugs. Eat healthy, Exercise moderatly, Take folic acid tablets. Dont stress yourself since treatment of tuberculosis in early stages definitely increases your chances of conception. Good luck. Hope i have succeeded in answering your questions. You could write to me anytime for further clarifications. regards, Dr Nilofer"
},
{
"id": 62115,
"tgt": "What causes bleeding followed by brown discharge and nausea after sex?",
"src": "Patient: Ok so idk if it could be stress or somethig else but on Feb 9th my mom went to jail because she got into a big fight with my grandparents trying to defend me..Feb 10 I was really stressed out trying to pay the bond and trying to get my mom out, I was emotionally destroyed..Feb 11, after I had sex I wiped, had light pink blood. Feb 12, lots of brown blood, had to put on a pad, had lots of cramps. Feb 13, a lil bit of brown blood, rarely any cramps. Feb 14: I went on a one day cruise for valentines day, had very little brown blood, I threw up on the boat, I guess I was sea sick, now I m home and still feel naseous..is this normal? Stress? Pregnancy? P.S: i m not sopposed to get my period in another 2 weeks... i have been on birth control for almost a year now but me & my bf don t use condoms.. during the days that i was stressed i missed a pill or two.. Doctor: Hi,Dear,Welcome to HCM. Based on the facts of your query,You seem to suffer from-yes,this nausea and brown discharge could be from hormonal imbalance with more stress hormones causing gastritis and the brown discharge after the sex could only be due to the withdrawn estrogen secretions due to severe stress you had lately.Possibility of pregnancy due to missed pills with unprotected sex, after recent sex,as a cause of this nausea and brown discharge-is very very less.Hope this reply would help you to plan further treatment with your doctors.Contact with a Followup Premium question to ME. Will appreciate your Hitting thanks and writing excellent review comments to help needy patients like you. suggestsGood Day!! Dr.Savaskar, Senior Surgical Specialist M.S.Genl-CVTS"
},
{
"id": 129000,
"tgt": "What causes posterior knee pain radiating towards calf?",
"src": "Patient: a few days ago I started having a pain in the back of my knee, now more in upper calf.. hurts more to go down the stairs them up and my knee cap feels like it needs to crack . I have been sitting at my desk all day and it hurts worse and my calf is swollen now and hard to the touch . Pain more in upper calf Doctor: Hello,Thank you for using Healthcaremagic.I read your question and understood your concern.This looks like condromalatia of the patello-femoral joint with efusion in the knee and Baker cyst.You need to do a knee MRI and then see a orthopedic doctor as different stages of the disease have different treatment.I wish you quick recovery.Dr. Selmani"
},
{
"id": 164647,
"tgt": "What causes shunted growth in children?",
"src": "Patient: Ralph has always been different from his classmates. When he was in fourth grade, he still could not add or subtract numbers larger than 10. He had trouble reading and spelling the simplest words. His memory was poor. All of the kids constantly teased him and called him names. Finally his parents sent him to a special school. When he got to seventh grade, he had to take gym. After the class all of the boys had to take a shower. Once again the teasing and the tormenting started for poor Ralph. The other boys told him he looked like a baby and that he would never grow up to be a real man! When he got home, his mother tried to comfort him. Things just continued to get worse. Ralph kept gaining weight and the nickname \u201cTubby\u201d was given to him by some cruel classmates. Still another problem was that Ralph was at least a foot shorter than other boys his age. What was Ralph\u2019s problem? Doctor: Hello, welcome to HCM there are many reasons for stunted growth like chronic malnutrition, ceoliac disease, chronic infections, chronic liver disease, heart disease etcthank u"
},
{
"id": 199046,
"tgt": "How to cure hydrocele condition?",
"src": "Patient: HI I am vivek 24 m frm india..i had hydrocele in childhood and my parents treated it with homeopathy treatment..and after dat it remained fixed in size (my left testicle is little big then right)..do i need to worry?..what can i do?..as frm last 2-3 yrs.. i m also facing a issue of frequent urination.. Doctor: HelloThanks for query.Hydrocele is an accumulation f serous fluid between two layers of Tunica Vaginalis which results from subclinical infection .Ideal treatment for hydrocele if moderate or large is surgery .Consult qualified General Surgeon for assessment and further treatment if required he may advise you surgery.Dr.Patil."
},
{
"id": 111265,
"tgt": "How to regain strength in back after compression fracture?",
"src": "Patient: I had a compression fracture ( D12) 4 weeks ago. No nurological problems like paralysis. what should I do for rehabilitation to regain the strength of my bac k ? I am 60 years old hypertensive and have post parandial sugar level in blood above 200 Doctor: 4weeks is not enough for vertebral fracture to unite completely. It is risky to start with exercises. U can start with execise for back strengthning once ur fracture unites as seen radiologically. U may need to wait further for at least 2-4 weeks And i recommend u to start exercise under guidance only."
},
{
"id": 77731,
"tgt": "What causes dry cough and mild incontinence?",
"src": "Patient: Just recovered from a virus that caused extreme fatigue and mostly affected my lungs (breathing, coughing). Now over that I have a dry, hacky cough that makes it extremely hard to sleep. I even wake up wanting to vomit and am experiencing some mild incontinence for the last 5 days. Doctor: Thanks for your question on Health Care Magic. I can understand your concern. In my opinion, you are mostly having post infectious bronchitis. It is common after viral upper respiratory tract infection. Bronchitis causes worsening office cough, breathlessness especially at night. And due to this, abdominal pressure rises and this causes incontinence. So better to consult pulmonologist and get done clinical examination of respiratory system and PFT (Pulmonary Function Test). PFT will also tell you about severity of the disease and treatment of bronchitis is based on severity only. You may need inhaled bronchodilators and inhaled corticosteroid (ICS). Don't worry, you will be alright. Hope I have solved your query. I will be happy to help you further. Wish you good health. Thanks."
},
{
"id": 67639,
"tgt": "How to get rid of a lump under the skin after a dog bite?",
"src": "Patient: I was bitten by a friends dog about 10 days ago on the under side of my upper arm. There is a large contusion approx 6 to 7 inches long and about 4 inches wide. It did break the skin and there is a golf ball size lump under the skin. It is still very painful and I cannot stand anything touching it. Doctor: Hi,It seems that you might be having big hematoma following dog bite.Now there might be having infection developed.Go for one antibiotic and anti inflammatory medicine course for 3-5 days.give rest to the part.Go for ARV vaccination.Ok and take care."
},
{
"id": 220073,
"tgt": "Did the Clome tablets or sperms cause this pregnancy?",
"src": "Patient: Hi, My wife has taken clome tablet for 2 months 5 tablet in a month during menstrual cycle and now she is pregnant. We have intercourse twice or thrice during this 2 months. I want to know whether she has been pregnant by just taking tablet or my sperms too contributed to pregnancy. Doctor: HiWelcome.One can not become pregnant only with any tablets. You need sperm. Hope this helps.Take care"
},
{
"id": 108443,
"tgt": "Suggest remedy for back, neck & shoulder pains",
"src": "Patient: I have had pain in my upper back and neck and shoulders for now 5 weeks. I am having trouble sleeping as pain is all the time. I have swollen nodes at base of skull and sides of neck, have been to doctor was given pain killers and muscle relaxers. This is not helping - very wooreid Doctor: hiWith swollen nodes, I would advise a complete blood count,ESR, amntoux test if in tuberculosis endemic area, Liver function tests, examination of abdomen for any spleenic or liver enlargement plus examination of throat,cervical spine and x rays of chest,cervical spine.These base test may give contributary informations for the causes and treatment may be directed specifically depending on results.As of now,pain medicines only.FNAC (fine needle aspiration cytology) of node if persistent swollen nodes or increasing in size.You have to keep a watch whether they are growing,and tender and whether they are mobile or fixed. All this your physician would do if he considers and agreesbest wishesregards,"
},
{
"id": 30335,
"tgt": "Suggest medication to recover from staff infection after leg amputation",
"src": "Patient: my name is joey. in 2006 i lost my leg after a injury,it had got a staff infection. i have had to go in the hospital several times since my amputation. i just got out of the hospital 2 days ago because the infection returned and had made a couple of soars on my stump. im now out of the hospital and i think the infection is gone but not really sure. my leg looks better but is still very painfull. what do you recomend as far as what i can put on my leg to make it feel better? also i have been told that once you have ever had a staff infection it never completely leaves a persons body is this true? my age is 49 i am 6 feet tall, i weigh 262 i have high cholestrol,diabetic,heart rate problem,blood pressure problem,bad nerves,i take effexior 150xr,and a couple other nerve medications,copd,and i still smoke. do you recomend some type of ointment to put on my leg to help the healing process? Doctor: Greetings,Welcome to Healthcaremagic.Staphylococcal infection is the most common infection related to skin and it also usually infect wounds after surgeries. The first medication which is given in staph infection is combination of amoxicillin + clavulanic acid . This medication covers the infection very well . But many times this infection becomes resistance to this antibiotic. In that case there are other medications . But a pus sample is tested in order to know which antibiotic is effective in such cases. That way we can give the exact effective drug and hence have higher chances of recovery. As far as you wound is concerned , it is important to know if you wound seems to be infected on inspection . Or do you have fever . If so then you need treatment and wound care would also be needed . IF you dont have these problems then there is nothing to worry about. As long as your blood sugars are continuously well controlled your infection will be also well controlled. Smoking in addition to diabetes rings alarm bells , So i recommend strongly to quit smoking now before its too late. Ointment can be given like polyfax ointment , But has limited role if infection is wide spread. I hope the answer was helpful. In case you need any further assistance feel free to contact in writing.Regards Dr Talal Hussain"
},
{
"id": 31277,
"tgt": "What causes recurring swelling and soreness on skin after bitten by a mosquito?",
"src": "Patient: Hi there I was bitten about 2 months ago by possibly mosquito and it got badly infected. It s lower part of spine. Was given fucidin cream 0.5% and it took swelling down but still left me with small red marks. It s suddenly swelled up again and very sore and looks infected. Why is this? What can I do? Doctor: Hi,It seems that there might be having secondary bacterial infection following something bite.In this case only local application is not enough.You might require antihistamine like Benadryl or cetrizine and a course of antibiotic medicine to clear infection.Consult your doctor and get examined.Apply triple action cream on affected area.Ok and take care."
},
{
"id": 53632,
"tgt": "Are tiredness, feeling sleepy and headaches due to liver laceration?",
"src": "Patient: I feel sleepy all the time, bad head aches lazy, get tired quickly, minor heart pains, I use to have anemia pretty.bad, and I recently was in a car accident in which I got.a liver laseration, would it be that I am anemic again?.or.is.it from the liver laseration? Doctor: Hello,1) Since when you had these symptoms?2) When accident occurred? Did you undergo any surgery after accident?3) Did liver laceration was managed without surgery? What was last USG abdomen report?4) Generally, if liver laceration is managed without surgery then patient must be bed ridden for 3-4 weeks and allowed to start work if USG abdomen does not show any collection of blood in abdomen or new evidence of bleeding.5) Your symptoms correlates with anemia. Chronic blood loss is also responsible for development of anemia. Thus it is very important to check status of liver laceration and hemoglobin level at present to decide further management."
},
{
"id": 193450,
"tgt": "Suggest side effect for miraquele pills",
"src": "Patient: Dear Madam,I m 29 years old, My marraige has 6 years later, recently my wife doing Laproscopy, but result is normal, and i take Miraquele 100mg daily so how does effect in my body my sperm count is 56 million and motility 40%, so when will i making father pls help me. Doctor: Hello, The mentioned tablet help in increasing sperm motility. Kindly mention the percentage of your actively motile sperm. The active motile sperm percentage is more important as compare to total motility. Kindly revert with all reports to comment further. Hope I have answered your query. Let me know if I can assist you further. Regards, Dr. Parth Goswami, General & Family Physician"
},
{
"id": 18848,
"tgt": "Suggest treatment for high BP post a kidney stone removal surgery",
"src": "Patient: Hello I am 26 year old Indian male,I had a 17mm kidney stone removed from my right ureter 3 months ago,the kidney showed severe hydronephrosis prior to the op,but the real problem is my blood pressure,its in the range of 140 over 90 ,I had several tests done but all came out normal,please guide me, didnt have such high Bp before . Doctor: Hello and Welcome to \u2018Ask A Doctor\u2019 service.I have reviewed your query and here is my advice.It is likely due to kidney damage due to stone. Is your renal function test normal? You should have a healthy lifestyle like avoiding fatty, oily and high-calorie diet.Have a low salt diet and monitor blood pressure regularly thrice a day for one week then once or twice a week.If Blood pressure is persistently more than 140/90mmhg, then you should be on medicines for it like Tablet Amlodipine once a day.You should do regular exercises like brisk walking, jogging according to your capacity for at least 30 minutes a day and 5 days a week.Eat Lots of green leafy vegetables, fruits, and fish once or twice a week, avoid meat.Avoid smoking and alcohol if any. There should not be an abdominal fat deposition or obesity.Hope I have answered your query. Let me know if I can assist you further.Regards,Dr. Sagar Makode"
},
{
"id": 181849,
"tgt": "Can pain near maxillary 2nd molar be sinus problem?",
"src": "Patient: I have had pain near maxillary 2nd molar on left side. Xray showed a large circular space, like an abcess superimposed over the sinus. I have been in bad pain, taking advil 5 times a day. I had o go to soft diet and had horrible headache, TMJ soreness, pain if I touched my max sinus. I wanted the tooth out, since I had a previous rootcanal and crown. I am a dental hygienist. Today the pain is gone with the tooth and I ate more and no advil for 6 hours. The sinus is sore to touch and TMJ still has limited opening. I use a cpap and my left nostril has been dripping for days. I wonder if this is a sinus problem, not tooth. Looks that way now. How can you do a differiental diagnosis?Thanks,Donna T Doctor: Hello,Thanks for consulting HCMRead your query as you have pain near 2nd molar this pain can be due to infection in tooth , it can be due to carious tooth formation of periapical absces or due to infection at apex of root causing inflamation of floor of maxillary sinus .For this I will suggest you to consult oral surgeon and go for investigation Iopa Xray or waters view radiograph for examination of sinus .Hope this will help you."
},
{
"id": 38262,
"tgt": "Suggest remedy for sinus infections like runny nose and sneezing",
"src": "Patient: Hello I am staying in Bangalore for the last 10 Years.From the past 2 years i am suffering from running nose and frequent sneezing in the morning. Strangely if i enter an air conditioned room the condition improves. I travel outside of Bangalore for work and as i have seen the sneezing and running nose is considerably quite less outside of Bangalore. In Bangalore the condition is worse during mornings(running nose and constant sneezing for about 20 mints). I have tried taking cetrizine tablets sometime when the situation becomes intolerable. But the tablets effect dont last long(may be a day or two).Kindly advice how i can cure this. Doctor: Hello, thank you for your contact to healthcaremagic. If I am your doctor I suspect a allergy of something in you. Allergens can only be found out by you. Until you won't find out allergen your condition will not cured completely. Cetrizine will help you only for some time. If you have to ask me anything you can contact me. Dr Arun Tank. Infectious disease specialist. Thank you."
},
{
"id": 27444,
"tgt": "Why does my doctor want to change medicine from sotolol to amiodarone?",
"src": "Patient: My cardiologist wants to move me from Sotolol 80mg twice a day---to 400 mg Amiodarone twice a day---have had debibulator/pacemaker for 4 years----has recenty \"shocked med-due to AFIB--- SEEMS TO BE MANY POSSIBLE SIDE EFFECTS TO aMIODARONE?? SCARES ME 77 YR OLD PATIENT (4 BYPASSES IN 1980, 3 MORE IN 1996) Doctor: Hi,If you have inappropriate shocks while on Sotalol, it means, that it's effect is not sufficient. You are right, Amiodaron has many side effects, but usually when it is taken for a long time. You can start Amiodaron, and maybe after some time your doctor can change Amiodaron, when you are in more stable condition.Hope I could help youWishing you good healthIn case of further questions don't hesitate to askRegards,"
},
{
"id": 191317,
"tgt": "Can Glycomet-GP 1 be taken instead of Glycomet-GP 2 for the treatment of diabetes?",
"src": "Patient: Hello This is syed here.Age : 31yrs I am a type 2 diabetic patient , as of now my sugar levels are 93 Before breakfast and after it is 114.I am taking tablet GLYCOMET GP2. taking only in night time before food.Can i decrease the tablet to GLYCOMET GP1.Please suggest. Doctor: Hello and Welcome to \u2018Ask A Doctor\u2019 service. I have reviewed your query and here is my advice. Glycomet 2 is a combination of two drugs, which seem to have effectively lowered your blood glucose levels to an acceptable level. Glycomet 1 is only one drug, Metformin, not the combination drug which is sometimes considered to be more effective at lowering blood glucose than Glycomet 1. Is there some reason you wish to switch since you are already well controlled? My advice to you if you wish to go to Glycomet 1 is to try it while testing your blood sugar and managing a consistent carbohydrate/calorie count in your diet to see if you get the same control from the Metformin (Glycomet 1) alone. If the same control is achieved then you will be able to stay on Glycomet 1 only without the combination Glycomet 2 treatment, but the switch must be monitored for effectiveness. I hope I have answered your query."
},
{
"id": 209515,
"tgt": "Suggest remedy for chronic akathisia",
"src": "Patient: I have chronic akathisa, which I have had for about 2 years (diagnosed by UCLA movement disorder clinic). I have been on multiple medications (narcotics from failed back fusion--cold turkey withdraw 3 years ago, reglan for motility issues related to narcotics, but cease after 3 months about 3 years ago, neurontin and lyrica for back pain nerve damage, ceased 1 1/2 year ago, prozac for depression due to back pain and stomach pain, but cease 1 1/2 years ago). I have tried valium/klonopin but causes GI motility problems and so cease 6 months ago, and tried beta blocker, but it made me feel very very sick. I now exercise, meditate, yoga, hot baths. But I am still absolutely coming out of my skin and have anger issues which blackens my soul. Yesterday tried passion flower extract, but it not help and not like reaction of feel like I was rushing. I doubt there is anything else, but open to suggestions. Thanks. Doctor: HIThanks for using healthcare magicIn akathisia, you can try central anticholinergic drugs and benzodiazepine. You have already tried beta blocker, so there is no need to take them. Alone with these drugs, you can try vitamin E or clonidine etc. In your case, getting response with these drugs is difficult. Commonly drugs in akathisia are beta blocker and central anticholinergic drugs. Try them and in case, you need further help, you can ask.Thanks"
},
{
"id": 106569,
"tgt": "How can a pulled lower back muscle be treated?",
"src": "Patient: I\u2019m 61 years old stage 4 kidney cancer and rheumatoid arthritis. I take oral chemo 30 mg prednisone, morphine 3 x daily and oxycodine as needed . I thought I pulled a muscle in my lower back 10 days ago. Pain meds are not helping. Added muscle relaxer still no relief. Ice heat and some limited stretching. Have lost bladder control. Feels like I\u2019m crushing my lower back bones when I stand. Planning to see my GP tomorrow. I don\u2019t know if I can take the pain for 3-4 more weeks. Any suggestions? Thank you in advance for your time. Doctor: Hello and Welcome to \u2018Ask A Doctor\u2019 service. I have reviewed your query and here is my advice. A pulled muscle can be treated with oral muscle relaxants and rest. Local analgesic gel can also help. Hope I have answered your query. Let me know if I can assist you further."
},
{
"id": 69538,
"tgt": "What causes small lump below the knee after injury?",
"src": "Patient: Hi, i found a small lump, size of a dime, below my knee, near my Anterior Tibialis. it moves around and doens t hurt. I had broken my foot, was in a cast for 2 months and also developed a large fatty tissue mass on the side of my knee, but that has decreased in size. that one was sore, but it was rubbing against the cast Doctor: Hi.Thanks for your query and an elucidate history.This looks to be again a collection of tissue like fat. I would advise to get ultrasonography and FNAC just to confirm what it is. You can wait and watch. It may regress. If it does not regress or increases in size, redness or develops some symptoms , you can get an opinion of a Doctor and may be get this excised ."
},
{
"id": 168396,
"tgt": "What causes a protrusion between shoulder blades of a child?",
"src": "Patient: My daughter has an unusual protrusion between her shoulder blades. We just noticed it this past week though I can t imagine it just appeared. She is ten years old, about 100 pounds and 5feet tall. Any thoughts? I am very worried and plan to call her doctor for an appt. in the morning. Thank you. Doctor: HiThere can be several reasons for such protrusions or swellings. It depends upon the exact site, consistency, margins, extent, relationships to skin and associated systemic symptoms. So, a clinical evaluation will be useful. Most likely reason can be an abscess, vertebral spine, cyst etc."
},
{
"id": 127484,
"tgt": "What causes pain and swelling in the wrist?",
"src": "Patient: Hi. I ve been experiencing pain in my right wrist for last 7 days. Had a slight bruise, very minor swelling (that s gone now.) Pain is worse when twisting wrist & using thumb to swipe on my cell phone. Heat/cold doesn t help. Wrapping in ace bandage seems to help. Doesn t really hurts when not using wrist. A little painful to the touch on the wrist joint. Not aware of any injury or trauma. Doctor: Hello and Welcome to \u2018Ask A Doctor\u2019 service. I have reviewed your query and here is my advice. How old are you ?? since when you're suffering from above problem??Any history of trauma?? you may be suffering from muscle spasm or wrist injury. Until examination is done it is difficult to say what it is. Take rest ,use pain medication. If symptoms not improved please consult your doctor he will examine and treat you accordingly. Hope I have answered your query. Let me know if I can assist you further."
},
{
"id": 101400,
"tgt": "What causes runny nose, sneezing after colonoscopy?",
"src": "Patient: hi, I had a colonoscopy today and work up from the anesthesia sneezing and developed a runny nose...feels like I just caught an instant cold. I have great before the procedure. The nurse told me that the oxygen was the cause. ( I immediately needed a napkin to wipe my nose before I could sit up. Doctor: Hello.Thank you for asking at HCM.If you have not had any chronic allergy problem like allergic rhinitis, I would agree that oxygen is a possibility.Sometimes oxygen works as irritant for nasal mucosa and this irritation causes immediate sneezing and running nose. Mostly it is self-limiting. There is nothing serious in that. You need not to worry at all.If you have an allergic condition like allergic rhinitis in past, there are many possible allergens in a hospital environment like latex, drugs, etc. In that case taking medications like levocetirizine & montelukast helps.Hope this will be helpful to you.Wish you best of the health.Regards."
},
{
"id": 177061,
"tgt": "Is vandomycin safe for treatment of sepsis in a child?",
"src": "Patient: hi. my niece (5 yrs old) has a sepsis now and according to her mother she became resistant to other iv antibiotics and the doctors will give vandomycin as last resort to treat her sepsis. is this safe? she has history of dermatomyositis. she s in the hospital now because it had cause her to have severe infection and undergone surgery. Doctor: vancomycin is safe to use in children. It is a very powerful antibiotic used in cases of MRSA, and can be given i.v. only. Child will require it for 10 to 14 days depending on the Blood cultures report and at a dose of 15mg/kg/day in 4 divided doses. Vancomycin can cause nephrotoxicity, so a base line KFT should be done and urine output monitoring done daily."
},
{
"id": 72551,
"tgt": "Suggest treatment for severe cough and cold",
"src": "Patient: Not even two weeks ago, I was just getting over a terrible cough. Well, now I am sick again. I cough consistantly, about 70% of the time it's been me coughing so hard and so frequently it makes breathing hard. On top of that, when I end up coughing like that, my chest and stomach hurt, and occasionally I hyperventalate. I am also extremeley congested. With my congestion, comes dizzyness,light-headed, and I am very tired, but I can't seem to get more then five hours of sleep. I don't have a thermometer, so I don't know my temperature, but I feel very hot and have been in the occasional cold sweat. I am only 17 years old, and generally healthy, I am not over weight, and I consume on average 6 bottles of water a day. I excersize and eat healthy. Doctor: Thanks for your question on Healthcare Magic.I can understand your concern. By your history and description, we should definitely rule out bronchitis in your case. So consult pulmonologist and get done clinical examination of respiratory system and PFT (Pulmonary Function Test).If PFT is normal then no need to worry much.In teenage, undiagnosed stress and anxiety can also cause similar symptoms. So avoid stress and tension, be relax and calm. Don't worry, you will be alright. Hope I have solved your query. I will be happy to help you further. Wish you good health. Thanks."
},
{
"id": 64331,
"tgt": "What causes lump near tail bone and butt cheek?",
"src": "Patient: hi, im 22, 5 8 male and a few weeks ago i beleive i found this pimple like lump near my tail bone on my butt cheek, it doesnt hurt at all and i wouldnt know it was there unless i felt for it but i have no idea what it is and it doesnt seem to go away, its decently sized , thanks Doctor: I welcome you to healthcare magic for your health concernsI am Dr Fahim and I will looking into your problem \u00a0\u00a0\u00a0\u00a0\u00a0I have gone through your question and understood your concerns. There is lump near your tail bone and its asymptomatic. It appears to be pilonidal cyst, at present nothing is required except observation. There is chances of it to get infected, them it will results in pilonidal abscess. Timely drainage of this abscess in important, otherwise it will result in pilonidal sinus which will require a surgical excision of the sinus tract under spinal anesthesia. At the moment observe this lump and keep the area dry and clean, if it get painful consult general surgeon for incision and drainage.I hope my answer will help. Do rate if you like it.Regards"
},
{
"id": 23311,
"tgt": "What causes tightness in heart area while suffering from hypertension?",
"src": "Patient: Over the past couple of months, I have noticed some slight pain and tightness in my heart area. I have lost 25lbs over the past year and 3 months but am still very overweight (5'1\" and 180 lbs.) I try hard to eat healthy and exercise (walking) as much as I can (about 15-20 min. several times a week). I am borderline high blood pressure that I take meds for. I am borderline diabetic that I also take meds for. Should I be concerned that something bad is going on. BTW, I also have a family history of heart issues: high bp, heart attacks, bypasses, etc. Doctor: hi and pleased to answer youYou have a very high body mass index (34) while standards are less than 25. Moreover, you are suffering from both, hypertension and diabetes.What I can tell you is that your blood is sweet (diabetes), sugar behaves like a strong oxidizing potential of cholesterol particles, making your cholesterol harmful from a quality point of view.So you risk tremendously to see your coronary vascular axes (cardiac muscle nutrition) limit the nutritional flow to the heart muscle and therefore you will have to undergo chest pain following this irrigation deficiency especially during any activities.So, you not only need to decrease your weight but you must absolutely stabilize your diabetes and hypertension.In the present context of the clinical symptoms that you feel, it seems to me also very urgent to make a thorough consultation with your cardiologist and to highlight the damage present at the coronary level in order to establish an adequate therapeutic regimen.best wishes"
},
{
"id": 20078,
"tgt": "Discomfort in chest is causing me anxiety should I seek medical attention?",
"src": "Patient: I had blood work done and all came back within normal range....I had gallbladder removed in June.....still have some discomfort in chest sometimes =....worried about this and maybe my heart....have had aniexty set in....if 3 doctors wasnt concerned should I be Doctor: HelloDevelopment of chest discomfort post surgery should be evaluated. Its advisable to get one ECG and Echocardiography done to make make this discomfort is not due to any heart related issues.Eat healthy, exercise daily and dont smoke if you do.Take care"
},
{
"id": 68773,
"tgt": "How to get rid of lump on the neck?",
"src": "Patient: Hi, I just notice a lump, maybe the size of a dime on my 5 year old sons neck. It is on the right side of his neck near the back, mid neck. It is hard and moves when I touch it. He says it doesn't hurt and can only feel it when I touch it. I am scared and plan on taking him to the doctor. I noticed it when he moved his head and stretched his neck, it protruded a bit. Should I be overly worried like I am? Thanks, Para-noid Doctor: welcome to Health care magic.1.You don't have to worry about it. But needs attention and evaluation.2.The way you are explaining it seems for me like, brachial cleft cyst ( its disorder by birth - developmental abnormalities), Lymph nodal mass ( after any infection / inflammation)3.Only thing you have to do is, get an appointment with your doctor and get an ultrasound neck and soft tissue swelling done.4.The ultrasound will help in reaching to an appropriate diagnosis, by knowing where it is arising from and what it contains and how it is involving adjecent structures.Hope it helps you. Wish you a good health.Anything to ask ? do not hesitate. Thank you."
},
{
"id": 100157,
"tgt": "Suggest treatment for allergic asthma",
"src": "Patient: Am having allegic asthama since my childhood,I am serious about my health taking precautions regarding dos n donts. Still 1ce or twice m having breathing pblm am using rotahaler . Some doctors prefer its good for asthama but some asks not to use it please suggest me what to do??? I am confused Doctor: Hello,Thank you for asking at HCM.I went through your history and would like to make suggestions for you as follows:1. Were I treating you, first of all I would like to know more about your symptoms in detail. Depending upon frequency of your symptoms and severity of your symptoms only, I would be able to say whether you need inhalers and what type of inhalers. (There are two types of inhalers - rescue type to be used for symptoms relief and controller type for regular use)2. I would at least suggest you regular montelukast which would help in allergic asthma and may reduce your requirement of inhalers. 3. In general, inhalers are safer than oral tablets as they act directly into lungs and hence dose required is small. Enuring right type of inhaler with correct technique also is important to minimize the dose of inhalers.4. I would also suggest you allergy testing which will help you identify the substances causing allergies to you as well as to know how to avoid their exposure. Based on allergy testing report, an Allergist-Immunologist may prescribe you allergen specific immunotherapy which works on your immune system to improve your allergy symptoms for long term.5. In addition to above, I would also suggest you to have a regular exercise schedule and a healthy diet rich in vitamins, minerals and antioxidants with improves your immunity gradually.6. Regular breathing exercises, Pranayama and Yoga also will gradually improve your lung capacity.To summarize regarding your main query for inhalers, I would like to say that inhalers are better than orally taken drugs, and they need to be taken if they are necessary for you. Please follow your doctor's advice regarding inhaler use. Please learn proper technique of using inhaler and avoid the substances which are worsening your allergies/asthma.Hope above suggestions will be helpful to you.Should you have any further query, please feel free to ask at HCM.Wish you the best of the health ahead.Thank you & Regards."
},
{
"id": 106878,
"tgt": "How can backache be treated?",
"src": "Patient: Hi, i was lifting stuff yesterday... and when i went to bed last night my back was hurting, when i woke up this morning it was worse i was all most crying in pain, i have tired pain relief, rest and hot packs but nothing seems to make it better or worse what can i do ? Doctor: Hello! Welcome to HealthcareMagic! Take tab Simhanada gugglu 2 tablets, 3 times day along with Mahanaryana taila or oil apply. It will cure your problem. Thank you for contacting us. Hope this clarifies your concerns. Take care Regards, Dr. Naser Jani, Ayurveda Specialist"
},
{
"id": 74447,
"tgt": "Suggest treatment for sinus infection, bronchitis and pneumonia",
"src": "Patient: I went to the doctor yesterday and shesaid I had a sinus infection,bronchitis, and I was starting to getPneumonia. She gave me a shot ofantibiotics and steroids. She also wroteme a prescription for Bactrim DS andcough syrup. The cough syrup isn'tworking, I'm up all night coughing, Ihaven't slept for more than 30 min sinceSaturday. I feel worse than I didyesterday. What should I do? Any Advice?Thanks! Doctor: Thanks for your question on Healthcare Magic. I can understand your concern. In my opinion, you need higher antibiotic,inhaled bronchodilators and inhaled corticosteroid (ICS) for your current symptoms. Constant coughing at night can be due to worsening of pneumonia or bronchitis. So consult pulmonologist and get done chest x ray and PFT (Pulmonary Function Test). Chest x ray is for pneumonia and PFT is for bronchitis. Don't worry, you will be alright with all these. Hope I have solved your query. I will be happy to help you further. Wish you good health. Thanks."
},
{
"id": 30238,
"tgt": "Suggest remedy for sarcoidosis",
"src": "Patient: Sir, after reading patients blogs about symptoms in details I guess I have Sarcoidosis..(What type Cardiac/Lung) dont know..Still Sir just wanted to know the way out further from you...Or you could suggest me some doctors in Delhi who are specialist in thiss... Doctor: sarcoidosis is multisystemic disease(afecting number of organs )predominantly affecting lungs and surrounding lymph nodes .it amy also affect eyes (uveitis) skin liver,kidney,heart and other tissue u would get following symptoms shortness of breath fever chest pain fatigue,arthralgiayour doctor would examine you physically by stethoscope to check breat sounds.blood test will show significant increase in ESR.chest x ray would show findings of sarcoidosis.pulmonary function test would be done .there are some other disorders which have similiar symptoms to sarcoidosishistiocytosis lymphoma tuberculosis these conditions would be ruled out by your doctor.i would suggest you following-go for blood test and chest x rayyou can take oral tab prednisolone after confirmation of diagnosistake voveron sr 100 tab for painconsult ophthalmologist if eye symptoms appear.specifically for sarcoidosis you should see the doctgor who is MD pulmonary/chest medicine or MD medicine.thankyou and takecare"
},
{
"id": 73385,
"tgt": "Is Levalbuterol inhalation safe to be taken instead of Albuterol?",
"src": "Patient: I have been prescribed Levalbuterol inhaler because I had a reaction to Albuterol. I had high BP spikes and find out that it conflicted with one BP med I am taking. Can you tell me if this Levalbuterol will do the same.I don t want a monthly subscription! Doctor: It is better and safer than albuterol and hence it is in trend now. Generally these drugs don't interact with BP medicines much"
},
{
"id": 190936,
"tgt": "I have a puss like taste in my mouth. Any idea on what this would be ?",
"src": "Patient: I keep getting this taste in my mouth like puss is leaking in my throat I just feel the need to spit all the time because I dont know if it s puss or not. It s been going on for maybe 4 days now and to keep this taste away I would have to either eat something like food or throw a piece of gum in my mouth. I remember when I had a bad wisdom tooth I had puss coming out, but that tooth was removed. Do you have any idea on what this would be? Doctor: 5 reasons : 1. pyorrhea>> in which calculus is deposited on teeth and gums and this leads to periodontal abscess and pus and bad breath(most common) 2. tonsils 3. chronic common cold and blocked sinus 4. uncleaned tongue leads to bad breath 5. Constipation leads to bad breath"
},
{
"id": 13806,
"tgt": "Suggest medication for skin infection",
"src": "Patient: I started itching a few days ago. I used Canesten because i thought it was normal thrush, but the itching reduced in intensity only slightly, so I used 500mg Canesten pessaries, but it didn t subside. So i used Gyno-Daktarin pessaries and ended up with a light orange discharge this morning. What exactly am i dealing with here? Doctor: Hi, Most probably this is not a fungal infection since antimycotics didn't work. Do a vaginal smear to check the type of infection. May be you need to use antibiotics. Hope I have answered your query. Let me know if I can assist you further. Regards,\u00a0\u00a0\u00a0\u00a0\u00a0 Dr. Olgeta Xhufka"
},
{
"id": 18492,
"tgt": "Are heart murmur, dry mouth and chills symptoms of endocarditis?",
"src": "Patient: I am trying to rule out endocarditis I have a stent in my penis and it has been there for 13 days I am going back tomorrow To have more stones removed. In November I went to my cardiologist and everything was fine and may I return and I now have a heart murmur 3 days ago I had skin cancer removed on my right forearm and I continue to have night sweats chills shivering extremely dry mouth that is cottonmouth in November I went to my cardiologist and everything was fine and may I return and I now have a heart murmur 3 days ago I had skin cancer removed on my right forearm and I continue to have night sweats chills shivering extremely dry mouth that is Kottonmouth Let me add that my identical twin brother at age 60 and we are 73 he had a severe massive stroke five days after having a catheter removed from him for kidney stones. I don\u2019t think I am paranoid I am in the behavioral health field and I have a private practice so I would like to think that I\u2019m somewhat practical any advise thanks Doctor: Hello and Welcome to \u2018Ask A Doctor\u2019 service. I have reviewed your query and here is my advice. The best way to rule out endocarditis is 2-D echocardiography supported by positive blood culture.Fever (if any) with shivering is minor criterion. Hope I have answered your query. Let me know if I can assist you further."
},
{
"id": 124186,
"tgt": "What causes black hair growth over the fracture site?",
"src": "Patient: Hello. I fractured my distal radius in February. It was not pinned and has not healed. The end of the bone is out of alignment by at least 15 deg. I am waiting to see a specialist in September. My query is, why is there black hair growing over the fracture site? I am a 55 year old female and very fair skinned. Doctor: Hello, The growth of black hair over the fracture site is not related to the fracture. This is a skin related change. Do consult a dermatologist for examination and proper treatment. Hope I have answered your query. Let me know if I can assist you further. Take care Regards, Dr Praveen Tayal, Orthopaedic Surgeon"
},
{
"id": 62918,
"tgt": "How to get rid of lump on the gum of babies?",
"src": "Patient: My 5 year old daughter has a huge lump on her top left gum and next to it is a smaller lump looks like a little bit of bone. She recently had antibiotic treatment for the tooth in question - 2 weeks ago and has been fine since. Then noticed this tonight... The tooth in question had been previously filled and when i had a second opinion the dentist said it looks like they didnt get all the bacteria out before they filled the tooth causing the absess. Surely the antibiotics would have cleared up the infection or is this lump still from the infection before? Doctor: Dear, Welcome to HCM.Understanding your concern. As per your query your daughter has lump on gum after filling in same tooth . Well the lump is basically a abscess that formed after infection has spread to the tip of the root and cause swelling there . Sometimes infection remain even after removal of infected tooth part it goes upto root tip and form pus there . I would suggest you to consult dentist again for proper examination and treatment . Doctor may order x-ray and decide the treatment option . Doctor can consider refilling , root canal treatment if infection already affected the pulp , or extraction if there is damage to tooth along with bone loss . Doctor may also prescribe antibiotics for at least a week . For now doe warm saline rinses several times a day . Hope your concern has been resolved.Get Well Soon.Best Wishes,Dr. Harry Maheshwari"
},
{
"id": 38585,
"tgt": "What is the treatment for influenza and sore throat?",
"src": "Patient: a friend is having flu, had a fever last night.. he is taking mefenamic.cefalexin.. local drug brand. he is complaining, sore throat, tonsils, congested runny nose.. it has been two days.. should a doctor be seen.. or complete bed rest and proper hydration will keep it at bay until it goes away.. Doctor: Hiwelcome to HealthCareMagicYour friend needs to be examined by a doctor to make a diagnosis and to rule out any serious problem.For healthy young adults flu is self limiting illness of 5-7 days , unless complicated.Things to watch are extreme dullness,respiratory distess , decrease urination, very poor oral acceptance , any rashes on body , cold exetremities.Tonsils infection and pain in throat needs to be examined to rule out streptococcal infection.Give him plenty of oral fluids and a good diet for rapid recovery.Hope I answered your queryRevert back if any other querythanks"
},
{
"id": 46220,
"tgt": "How to treat kidney stones without surgery?",
"src": "Patient: Dear sir,the patient is suffering diabetics problem since last 10 years, two years back we came to know that a stone is in left side kidney,In bhubaneswar kalinga hospital we admit him and stone remove by surgery. again this year we came to know that one stone 16mm size is in right side kidney, due to diabetics doctor is not ready to surgery again because last time it took much time to recovery from it.doctor suggest now to take the patient vellore to break the stone with high sophistcated machine which is not available here at bhubaneswar,Please suggest me is it possible to break the stone without surgery , it is not visible by x ray but only pointout trough ultrasound report.due to blockage of urine doctor fix a small plastic pipes for smooth urine which controll the level of creatinine. please suggest me what i have to do now Doctor: Better get Surgeon expert opinion. Renal functions must be watched closely. With control of sugar surgery should be no issue.."
},
{
"id": 56278,
"tgt": "Suggest treatment to lower the SGPT and bilirubin value",
"src": "Patient: Hi, i am 32 years old male, i done my complete blood test 1 month ago an my sgpt was 72 and belrrubin was 1.9 uric acid was 4.1. then i start eating boiled meal and fruits rice etc but no oil or fatty food. then i go through blood test after a 10 days sgpt was reduced to 56 and belrrubin was still 1.9. for next two weeks i stuck with the same diet but started eating more pulses and then i had attack of gout which was very painful and i had pain killer injection. i didn t took any medication and with the recent blood test my sgpt is 77 belrrubin is 2.4 and uric acid 5.1. please suggest me how to lower my sgpt and bilrrubin many thanks Doctor: Welcome to Health care magic. Thank you for posting your question.I can understand your concern.I can understand that you are very anxious about your health.Jaundice can be due to many causes like Hepatitis , gall stone disease(acute cholecystitis) , pancreatitis - to name a few. Mere dietary adjustments often cannot cure it.It would have been great had I examine you on my own.You need to undergo a few tests to find out the exact cause.I would suggest you to - Avoid oily, fatty food- Drink plenty of fluid- Do not smoke.Stop alcohol- Have a BLAND DIET .- Get an abdominal ultrasound, Complete blood count , Liver function test, platelets, urea, creatinine , electrolytes done - This will help in the diagnosisVisit a gastro-enterologist . He or she can examine you and correlate clinically.Do not worry.Hope this answers your question. If you have additional questions or follow up questions then please do not hesitate in writing to us. I will be happy to answer your questions. Wishing you good health."
},
{
"id": 159150,
"tgt": "Diagnosed with squamous cell carcinoma. Took radiation treatments. Cancer free now. Suggestions?",
"src": "Patient: Hello doctor -- I was diagnosed in March 2012 with Squamous Cell Carsinoma. I was alerted to a mass on the lower left side of my neck....a Lymph Node. I am 64 years old, Caucasion, retired Nurse. I caught it early. There is no sign of the origion.....and by the time it took them to diagnose me a month had lapsed and the mass had expanded from 3.4 cm. to 6 cm. My diagnosis was Stage 4. With the guidence of FLORIDA CANCER SPECIALOIST s Oncologist Dr. Robert Whorf MD -- I was told that he could help me -- with three cyles of Chemo -- 3 weeks apart. This was followed by 33 Radiaion Treatments. I had a feeding tube installed in August. My blood work was not good by Aug. PLATLETS were 50. Hospiliizations required. On 12.12.12 I had a PET Scan which says I am cancer free now. During this time I could not swallow and was on Osmilite and Jevity. ( I lost 60 ponds (the upside of this whole ordeal.) Oct., Nov. & Dec. were dreadful -- but I am in recovery now and feeling better. Ihave discovered a new product made by SUNKIST called SUNKIST NATURALS -- and the Pina Colada is awesome! Complete Protein, FREE of FAT, CHLOSTERAL & GLUTEN . I am alive and feel very lucky. Doctor: You can continue with this protein powder. You can take steam inhalation 2-3 times in daytime & before going to bed at night, to prevent dry throat & cough during sleep. Try frequent non-spicy, non-bitter diets,"
},
{
"id": 82224,
"tgt": "Suggest remedy for irregular heart beat & chest pain",
"src": "Patient: I m facing irregular heart beat and pain in both front and backside of the chest during stress,fear and anxiety.I m facing this prblm since1998 when i suffered frm depression.My present age is 33 years. My name is Md Rahmatullah from darbhanga. Plz rply Doctor: Thanks for your question on HCM.In my opinion you should first rule out arrhythmia for your irregular heart beats.So better to get done holter monitoring. Holter monitoring is continuous(24hrs) recording of ECG. So we can find out what ECG changes are there when you have irregular heart beats.If it is normal than get done CBC and S.TSH to rule out anaemia and thyroid diseases as both of these can cause rapid heart rate.If these are also normal than possibility of anxiety is high. So avoid stress and anxiety. Keep calm and be relax. Consult psychiatrist for counselling sessions. You may need anxiolytic drugs."
},
{
"id": 58991,
"tgt": "Ultrasound report shows mild diffuse wall thickening of gallbladder, tiny, echogenic focu present with reverberation artifact. Explain",
"src": "Patient: My sister's ultrasound report states the following for gall bladder. \"Normal size, shape and position, showing mild diffuse wall thickening. Multiple tiny echogenic non-mobile focu are present within its wall with comet tail reverberation artifact.\" What are the possibilities of gall bladder stone and does this require removal by surgery? Please advice Doctor: Hi ! You have not mentioned the reason for doing an ultrasound for your sister. Did she have any abdominal pain, or belching or any hyper-acidity. Immaterial of whether she had these symptoms, the ultrasound report could mean that she has got multiple stones, which could become impacted anywhere in the biliary tract and cause more problem. I would suggest you see a gastrointestinal surgeon/general surgeon, who may clinically examine your sister, and may also advise for a gall bladder removal surgery, which is done well by the minimally invasive surgery.I wish your sister an early recovery."
},
{
"id": 87349,
"tgt": "What causes abdominal pain with blood in the saliva?",
"src": "Patient: hello doctors, Since a week, i am feeling pain in upper stomach,two days in the moring when clean my nose i found light colour blood in saliva. yesterday moring when spit i found the colour of saliva is brown dark. I went to doctor but forgot to say this blood incident to doctor and doctor suggested omeprazole capsules.Please let me know is there any serious problem. Doctor: Hi.Thanks for your query.The pain in the upper abdomen can be due to gastritis or stomach cancer. You need to get the upper GI Endoscopy done. The blood from the nose and the oral cavity and saliva can be due to local problem of infection or a mass or so. Get an ENT Opinion and examination to see what the problem is and get treated accordingly."
},
{
"id": 137468,
"tgt": "Suggest medication for pain in the left iliac region",
"src": "Patient: hi. i m 31yrs female. i hv pain in my left iliac region which is dull n heaviness like n severe in evening. it is since abt 4-5months n nw it is localised .i hd LSCS 5yrs back n hd wound infection also. my usg n CT r normal except for minimal free fluid. kind adv diagnosis n treatment. Doctor: You must go for some exercise like pelvic briding .lower trunk rotation abdominam setting and stretch your hamstrings ,iliopsoas and glutes.Do iceing 10 min 4 time a day along with any calcium plus vitamin d tablets"
},
{
"id": 43137,
"tgt": "What is the reason for cramps, pink discharge, no pain after taking IVFC injection?",
"src": "Patient: hi there , im 6 weeks pregnant and since yesterday morning i started having pinkish discharge on wiping with no pain i did na hcg level it was 1280 . today it was repeated and became 1450. they couldnt find a sac on transvaginal u/s intra or extrauterine . the bleeding has become less today . i had cramps in the morning through the afternoon but they r resolving . after taking IVFC injection Doctor: Hi, Welcome to Health care magic forum. In such a case it is difficult to determine the cause, It may be an inevitable abortion in the very early days of pregnancy, and presented at this date, H.C.G. may be positive for some more days, and the cramps could be due to the small mass of the sac, to be expelled, or may be due to infection. Ultra sound will be the final finding to rely upon.Still your doctor will be the better judge. Wishing for a better health. Thank you."
},
{
"id": 3343,
"tgt": "What are the chances of pregnancy if periods are delayed?",
"src": "Patient: i had my periods on 26-4-2014 to 1-5-2014 , after my periods i had sex with my hubby i stil did not get my periods mean while i m taking ovacet and evaserve tablet a/c to my doctor advice , so i just want to knw wheater m pragnent or not wt really happing in me Doctor: Hallow Dear, You have not mentioned how many days after the menstruation or on which days of the cycle you had unprotected sex. Usually, 14 days prior to the next expected menses the egg is released (ovulation). A period of about one week around this day is a fertile period; though variations are known. With a history of unprotected sex in child bearing age, if the periods are delayed, pregnancy should be suspected unless and until proved otherwise. To find the evidence of pregnancy status, you have following options: 1. Perform pregnancy test on overnight morning first urine sample a week after the missed period; earlier the test may report false negative results. 2. Alternatively you may opt for Beta hCG test on your blood anytime after 10 days of the last unprotected sex. This should provide you clue to resolve your dilemma. Dr. Nishikant Shrotri"
},
{
"id": 186362,
"tgt": "What causes unbearable pain after root canal treatment?",
"src": "Patient: I had root canal treatmentent filling last Tuesday and yesterday went back to have a permanent filling in the root canal. It took the dentist an age to do it and it was quite painful. Since then the pain is unbearable worse than before the treatment. Advice please Doctor: Thanks for posting your query to HCM.After going through your history, high point on the permanent restoration seems to be possible cause of pain.Please visit a dentist and get the high point reduced.Hope my answer will help you."
},
{
"id": 33293,
"tgt": "How to treat typhoid?",
"src": "Patient: hey, iam 31 year old man, i was suffered with typhoid when i was 14 as u know in that case person got high fever for long time... i was on bed nearly 2 months fighting wid it and took antibiotics. Before this happening, i was fair coloured i saw the difference in the past pictures ...my facial color got burnt and hands upto the armpits. my body color is fair inside after the neck... I feel really depressing when i go out and want to take shirt off while gym , or beach.. naturally iam not dark coloured. i have used home made mixtures , bleaches, didnt expose much in sun light but nothing worked... all my siblings are of fair color and my mom n dad. doc please help me out. this is my first time in my life that iam sharing this wid someone. need a help Doctor: Hello,Its nothing to do with your typhoid fever. You can use fairness cream containing vitamin D. No need to worry. The color of the skin is natural, and no drastic change can be made with medical treatment.Hope I have answered your query. Let me know if I can assist you further.Regards,Dr. Hardik Sanghvi"
},
{
"id": 214160,
"tgt": "both of them had a sexual intercourse once after their engagement but now they refused",
"src": "Patient: both of them had a sexual intercourse once after their engagement,from then he refused to marry her The patient is my younger brother who got married 6 months ago,both of them had a sexual intercourse once after their engagement,from then he refused to marry her and somehow the marriage got over. now still he is not beliving her .now he has applied for the divorse also on his own.but all our otherfamily members are not finding any fault at the girl.could you please advise Hypnotist availiable in chennai whom we can meet and take the treatment or otherwise please advise Doctor: This is a somewhat complex situation May I suggest for you to consult a clinical psychologist or counselor who works with relationships near your place of residence."
},
{
"id": 57951,
"tgt": "Historic high WBC count with latest count of 14.2 and Absolute SEG of 10.41 with normal range of 2.0 to 6.9. Gallbladder and liver tests are normal.Can this cause right shoulder blade pain that radiates to the neck?",
"src": "Patient: Hi, I have had right shoulder blade pain for the past 5 years. The pain radiates to my right side and up my neck. Tests on gallbladder and liver have all come back normal. My WBC has been high for years even after repeated antibiotics. Last CBC shows WBC of 14.2 and Absolute SEG is 10.41 with normal range of 2.0 to 6.9. All other levels are within normal ranges. What could this mean? Doctor: HIThank for asking to HCMChronic condition of infection could be the possible cause, if you do have a shoulder blade pain then the bursitis could be the infection, right now this can be treated on tentative base, and for this the best drug would be \"Doxycycline 100 mg just one tab. in day for 10 days, this will give the good results, have nice day."
},
{
"id": 67563,
"tgt": "Cause and cure for sore lumps between legs",
"src": "Patient: Hi my names Ashley I have recently found a lump under my my legs inbetween my bum and my balls I noticed it about 4 weeks ago when I sneeze it a little bit off puss comes out and some times a bit of blood and is sore it has got bigger and is about the size of a marble and I have also had the runs on and off for about 5 weeks and fell sick and find I have to force myself to eat do you know what this is please Ashley Doctor: Hi Ashley! Good evening. I am Dr Shareef answering your query.Even though this needs to be examined clinically to reach at a diagnosis, from your history it might be a boil/abscess in a pre existing cyst, which got inflamed and infected. This is an infection which could have caused rise in your body temperature (fever) leading to loss of appetite. If I were your doctor, I would have advised you a dressing of the would after cleaning, a broad spectrum antibiotic, and an anti inflammatory drug for a symptomatic relief. In case it was an abscess, I would open it up under GA followed by a broad spectrum antibiotic for a complete healing.I hope this information would help you in discussing with your family physician/treating doctor in further management of your problem. Please do not hesitate to ask in case of any further doubts.Thanks for choosing health care magic to clear doubts on your health problems. I wish you an early recovery. Dr Shareef."
},
{
"id": 15054,
"tgt": "Taking daphne after delivery. Suffering from skin rashes in arms and legs. Should I change medicine?",
"src": "Patient: Hello. I ve been taking Daphne 6 weeks after I gave birth. In the first 5 months, my skin was soft and no rashes came out. However, just recently, for almost 2 weeks now, I started two suffer from skin rashes, specifically my arms and legs. I can t understand why is this happening. I was planning to buy another brand (Diane-35) yet my friend told me that according to her OB, before changing pills of another brand, one must undergo pamsper (which I m afraid to go through). Can you advice me on this? Thanks. Doctor: Hello,Thanks for the query,The chance of this rash being drug rash is very low as you have been using the drug since 5 months.You might have developed a viral exanthem.According to me there is no need to stop or change drug.Even then if you want to change, you can do that without much worry.Brands can be easily changed.Just notice that the drug content should be same.Please meet a dermatologist for exact diagnosis.Let me know if you have any other doubt.you can ask a direct question to me on this forum, following the below link.https://urldefense.com/v3/__http://www.healthcaremagic.com/doctors/dr-rahul-kumar/64818Wishing__;!!Mih3wA!SBzm6_kI6hCZ58EPH6N_05MFfiPbxWXT0a2TJCdFQObRWm5mV5ur7hUOMa8clQ$ you a good health.Thank you"
},
{
"id": 183489,
"tgt": "Suggest treatment for sore and swollen gums",
"src": "Patient: I am 62 and have worn dentures since I was 15. the last time I had them replaced was 25 years ago. this is because I had very bad experiences when I was young and also I suffer very badly from claustrophobia. I know I need new teeth and my gums at the bottom of my mouth are very sore and a bit swollen, I think because I have worn these teeth for far too long. I know I need to see someone but I am so scared, should I see a dentist or doctor because I'm pretty sure I have an infection. I have a doctor on the NHS but as I said I haven't seen a dentist for about 25 years ago and I am scared and don't know which way to turn. Please can you help me? Doctor: Thanks for using Health Care Magic.Read your query.Wearing a denture for 25 years without replacement is not advisable as it can lead to infection and tissue destruction.I would advice you to visit your local dentist at the earliest and have them replaced.Check for any infection in the oral cavity and get them treated.Being scared of the dentist is not a solution ,keep your fear aside and get the denture replaced.Do salt water gargling.Hope this was helpful.Thanks and regards."
},
{
"id": 190006,
"tgt": "Gum injury, hole formed in the middle of the superior labial frenulum. Chances of bacterial infection?",
"src": "Patient: hello? hey, i accidentally slid my gum down a wood and the wood slid up from the front of my teeth up to my gum towards the superior labial frenulum. Now, there s a hold in the middle of the superior labial frenulum. Will that cause any medical trouble? I m more concerned with a potential of infection from accumulation of bacteria might occur. Doctor: Hello and welcome to HCM This kind of injury with a foreign object requires prompt treatment to prevent infection. You need to consult a surgeon for clinical examination of the wound. If it is an open wound primary closure under local anaesthesia is needed followed by course of antibiotics and analgesics. Since the wound is in the oral mucosa, it will heal up fast. So don't delay in visiting the surgeon. Thanks and take care Dr Shailja P Wahal"
},
{
"id": 14319,
"tgt": "Does the skin rash need further treatment?",
"src": "Patient: My arm touched a wet rock at the edge of a stream and started burning and itching. I looked at it and there were little fuzzy hairs that must have rubbed off the rock onto my arm. Over the course of a couple of hours the fuzzy hairs were replaced by a red bumpy rash. What is this? Should I be worried? Was there some unseen organism that caused this? Doctor: Hello and welcome to HCMYou seem to have developed an allergic reaction to moss/lichen or some similar substance present on the rock. The rash developed as your skin reacted to it.I would advise you to clean the area with soap. Do not rub or scratch the area. If the rash has stopped spreading or is reducing on its own, you can apply a soothing menthol or calamine lotion to calm the itching. If the rash is still active and irritating, apply a mometasone cream over the area for 3-5 days till rash goes away.Also take an antihistamine tablet once daily such as fexofenadine, levocet for 5 days.This should bring the allergy under control.In case you notice any blistering, discharge, swelling or fever or persisting rash, do consult a dermatologist who may prescribe steroids or antibiotics as required. Also if you have any breathing difficulty, wheezing, dizziness etc, visit the ER and get a cortisone shot.Hope this helps."
},
{
"id": 128528,
"tgt": "Suggest treatment for severe knee pain with swelling",
"src": "Patient: my wife has swelling below the right knee. she has had off and on knee pain that ihas been helped by wearing a knee brace. The swelling in only one leg below the knee is worse at bedtime and better in the morning she has had gout two months ago in here big toe on same leg her job will not alloe any sitdown time and she stands at a cash register for aleast four straight hours withou any sitt down time Doctor: hi sir/madam,it can be treated easily by the help of ayurveda.Line of treatment as per Ayurveda:-The general principles of treatment of vata dosha are adopted in case of katishoola (low back pain). It includes various measures to suit its varied clinical entities, stages and associated complaints.1. Snehana (oleation) \u2013 by sneha dhara (pouring oil), abhyanga (oil massage), avagaha (tub bath with oil or oleaginous medicaments), kati basti (retaining medicaments on the back) etc.2. Swedana (sudation) \u2013avagaha sweda (sweating treatment with tub bath), pizichil (kayaseka), nadi sweda (sudation through a tubular device \u2013 local sudation), panda sweda (sudation through medicated paste or powder) etc.3. Mridu Samshodhana -mild purgation.4. Basti (medicated enema) like eranda basti, vaitarana basti, pippalyadianuvasana basti etc.Formulations indicated in Low back ache as per Ayurveda:-1. Dashamoola kwatha.2.Maharasnadi kashaya.3. Rasnaerandadi kashaya.4. Sahacharadi kashaya.5. Gandharvahastadi kashaya.6. Trayodashanga guggulu.And for local application these oils will help:-1. Ksheerabala tail.2. Mahanarayana tail.3. Dhanwantaram tail.4. Maha narayan tail.Avoid these for better results:-1.Bitter, astringent and pungent food2.Cold water3.Fear4.Exhaustion5.Standing6.Driving7.Cold food and beverages etc.Hope i was helpful.Have a healthy day."
},
{
"id": 207578,
"tgt": "Suggest treatment for OCD behavior and suicidal thoughts",
"src": "Patient: I started masterbate since i was 4 and now i m totally disturbed due to my OCD behavior and depression , loss of libibo,premature emission,excessive night discharge,fatigue, loss of energy and memmory, lack of concentration ,sleeplessness, irritating behavior ,getting discharge by touching a girl or penis ,obsession of watchin porn, lack of interest in daily life and suicidal tendency. I mgoing to marry under the parents pressure but i m not ready and thinking of suicide. I hope you will send the advice Doctor: HiI understand your concern and your emotional disturbances.You have enumerated a lots of symptoms that are symptoms of depression, OCD and psycho sexual complain.Frankly speaking you need proper consultation for evaluation.because we have to need to reach proper diagnosis.proper address to sexual problem can relieve your obsession and depression.It is only possible by getting brief history.It can be treated with drug as well as psychotherapy.Counseling is the key to reduce stress.So consult psychiatrist and get help.Still have a query then feel free to ask.Thank you."
},
{
"id": 21809,
"tgt": "Does high BP cause numbness in temple area?",
"src": "Patient: I recently got treated with very high hypertension. It seems to be umnder control now but bp is still slightly high. When my BP went up i got dizzy. Now I have a numbness in my left temple and periodically my left arm. I have had blood tests and all is ok. I did a heart sonar ECG and 24 hour ecg all is normal. I am 55 and overweight (lost 7 kgs in the last 6 weeks 20 to go) I am 55 year old male. Any advice would be gratefull Doctor: Dear Sir,You take blood pressure medications and you lose weight, you are on the right way. Also cut the amount of sodium if you don't do it, and it is very important to regulate your sleeping hours, working hours and rest. If you continue to lose weight and meanwhile have blood pressure under control for a longer period, ten your dizziness will disappear, most probably numbness in temple area will disappear too, if not see a neurologist to look for another cause for the numbness.Take careCome back if you have any further questions"
},
{
"id": 118197,
"tgt": "Suggest treatment for rash that follows a blood vein on my arm",
"src": "Patient: I have an itchy rash that follows a blood vein on my arm. Same arm that had blood drawn from a few days earlier starting at the spot where the needle was inserted. Coincidence to poison ivy? Not sure. I was in the woods a day after blood was drawn Doctor: Hi and thanks for the query,it might be difficult to suggest a treatment without seeing the rash. This is so because treatment should be guided by the severity and type of rash. Very easy to diagnose clinically. Images could help. However, I suggest you see your primary care doctor."
},
{
"id": 15910,
"tgt": "Rashes on Forehead, extremely itchy with tingling sensation and swelling. What this could be?",
"src": "Patient: Hi there, Every summer since I was a baby, I ve gotten this sort or rash on the forehead near the hairline. It starts out with a tingling feeling and then gets extremely itchy. Once it gets to the itchy phase, little bumps will start to form and then eventually cause my forehead and sometimes entire face to swell . After the itchy phase, the area will turn to scabs. This only happens once a year (usually after the first major exposure to the sun) and then lasts for about 1 week from start to finish. The doctor doesn t know what it could be either. Do you have any ideas? Doctor: hi,, You seem to have a rare photodermatosis ie hydroa vacciniforme Hydroa vacciniforme is a skin condition, a very rare, chronic photodermatitis with onset in childhood. Mostly affects children aged 3-15 years. It is more commonly seen in females than in males. The cause of hydroa vacciniforme is usually unknown. Long wavelength ultraviolet radiation (UVA) is most often implicated. Sun-exposed areas are affected most and include the face( forehead, cheek) and ears. In most cases signs and symptoms usually start to occur about 30 minutes to 2 hours after sun exposure and present as: Mild burning, itching or stinging Development of tense, swollen bumps (papules) and blisters (vesicles); there may be facial swelling These turn into dimpled, pit-like papules with black scabs (necrosis) on a red and inflamed base Lesions heal to form pale depressed scars The rash usually first appears in spring with recurrences throughout the summer months. Some patients may have other symptoms, including mild inflammation of the eyes and swelling of face In most patients the condition disappears by the time they reach adolescence, although the scars are permanent. Patients with hydroa vacciniforme must take measures to avoid sun exposure by following sun protection strategies. This includes applying high SPF sunscreens with UVA blocking agents and wearing protective clothing. This may prevent or reduce the number of recurrences of the condition. Oral medication may be useful in reducing outbreaks but does not reliably prevent hydroa vacciniforme lesions. I would advise you to search for the above condition on internet,,if you feel that you also develop similar lesions , then you might consult a dermatologist for proper assessment.. Hope it helps Dr Geetika Paul"
},
{
"id": 68757,
"tgt": "Suggest treatment for lymph nodes behind my ear",
"src": "Patient: My lymph nodes behind my left ear have been swollen for a couple of weeks, I saw my dr he said I had a mild fever and most likly an infection of some sort. I finished an antibiotic, my lymph node is still swollen, my ear hurts a bit, now im feeling dizzy, nauseous and my vision seems blurred, what could be wrong? Doctor: Hi,Your symptoms may indicate you have developed an ear infection. I suggest you see your doctor for assessment as you may need a further course of antibiotics. In the meantime, paracetamol and ibuprofen will help the pain and fever. Regards,Dr K A Pottinger"
},
{
"id": 213228,
"tgt": "Engaged, love my partner immensely, unable to express myself as a result hurt him. Serious illness?",
"src": "Patient: We are recently engaged. We do have conversations daily for long hours. Initially i was able to express myself. But these days i dont know why, i couldn t even say I LOVE YOU to him, even when he is insisting to tell. I m hurting him. Frankly i love him a lot.. But what to do... is it a serious illness?? Pls suggest.... Doctor: HI , THIS IS NOT A ILLNESS BY ANY CHANCE, SEE ALL THESE THINGS WHICH IS HAPPENING TO YOU IS NEW, YOU ARE BASICALLY A RESERVED GIRL I THINK. YOU MIGHT NOT HAVE HAD THESE KIND OF CONVERSATIONS BEFORE. DEFINETLY UR FIANCE WILL FEEL BAD. WHEN HE ASKS YOU TO TELL I LOVE YOU , YOU TELL HIM THAT YOU WILL TELL ALL THESE THINGS AFTER MARRIEGE . TAKE CARE BYE..."
},
{
"id": 191838,
"tgt": "What causes arm numbness in a diabetic?",
"src": "Patient: I am a 36 year juvenile diabetic who has been diabetic for 25 years. I am in good health except my right arm falls asleep every night for the last year. Diabetic dr said it wasn t diabetes related because it would be in both arms. What else could it be? Doctor: Hi...Welcome to HEALTHCARE MAGIC...I have gone through your query and can understand your concerns...Numbness in hands is an extremely common and bothersome symptom. It could result from pressure on nerves when your arm is crooked under your head as you fall asleep. Usually painless and soon relieved by removing the pressure that caused it. In many cases, however, numbness in the hands can be severe, episodic, or chronic. It also can accompany other symptoms, such as tingling, pain, itching and muscle wasting. In such cases, numbness may be a sign of nerve damage, which can result from traumatic injuries or repetitive stress injuries, bacterial or viral infections, toxic exposures, and systemic diseases such as diabetes. Such nerve damage is known as neuropathy it affects nerves distant from the brain and spinal cord, often in the hands and feet.Diabetes is one of the most common causes of peripheral neuropathy, accounting for about 30% of cases.If numbness is persistent in your hands it\u2019s important to seek prompt medical evaluation. The earlier the underlying cause of your numbness is identified and brought under control, the less likely you are to suffer potentially lifelong consequences.Hope this information helps...Thanks & Regards,Dr.Shiwani"
},
{
"id": 73414,
"tgt": "What causes chest and lower back pain with shortness of breath?",
"src": "Patient: started last night. Under alot of stress right now. pain in chest, shortness of breath, slight numbess in left arm. Lower back pain and a sharp pain in right shoulder blade region. This morning, when I ate when i swallowed there was pain in chest area? Doctor: Thanks for your question on Healthcare Magic.I can understand your concern.By your history and description, possibility of stress and anxiety related symptoms is more likely. But better to get done ecg and 2d echo to rule out heart diseases. If both these are normal then no need to worry for heart diseases.Consult psychiatrist and get done counselling sessions.Try to identify stressor in your life and start working on its solution.You may need anxiolytic drugs too.Don't worry, you will be alright. Avoid stress and tension, be relax and calm.Hope I have solved your query. I will be happy to help you further. Wish you good health. Thanks."
},
{
"id": 78245,
"tgt": "What could cause pulmonary fibrosis?",
"src": "Patient: I was diagnosed with pulmonary fibrosis 2 years ago after a lung biposy... i was on prednisone for a couple of months.. but i stopped.. from now on im only using an inhaler (symbicort) but every morning as soon as i open my eyes, my cough starts... and it keeps going for more less 20 minutes.. no matter what i do it wont stop.. after those 20 minutes of coughing im just exhausted... i was wondering if there is anything that could help with my cough and the sputum Doctor: Hi,Dear,Welcome with your query to HCM.Studied your query in full depth of its details.Reviewed it in context of your health concerns.Based On the facts, You mostly seem to suffer from-Overnight Lung Congestion and would need Cardio-respiratory assessment.Remedy Suggest -for your case Bronchial Care-with Positional Chest Tappings would reduce the complications associated with these morning cough spells.Restriction of Fluids after 4 pm.Reduce on Drinks after 4 pm.Reduce on Salt in your diet.Use Inhaler infrequently.Practice Deep Breathing x 3 times a day x 10-15 mts sessionsAs I dont have access to cause of your Lung Fibrosis,Second Opinion from your Chest PhysicianOther causes need to be ruled out and treated if present accordingly from Second opinion from Chest Physician and Chest Surgeon.Hope that ,This reply would help you to plan further treatment soon with your treating doctors.Best of Luck and early recovery.Welcome any further query in this regard,which would be replied in next session.Good Day!!Dr.Savaskar M.N.Senior Surgical Specialist"
},
{
"id": 190753,
"tgt": "Facial palsy attack, stiffness near chin, eyes shrink, discomfort in the teeth",
"src": "Patient: Hello Doctor, I sufferred a facial palsy attack 2 yrs back, though looks recovered yet i feel a sort of a stiffness near the chin and the eyes shrink when i look at the other persons face and talk. i feel a dis comfrot in the teeth too. did a dental check up and they said nothing noticiably found as a fault inn the jaws............ Doctor: hi welcome to h.c.m.forum. as you said there is posibility of some fraction of the numbness. according to your statement it is 2% some people will have upto 25%. you need to use neurotropic injections. altermate days forten days. multi vitamin tablets for long time. jucy fruits frequently. thank you."
},
{
"id": 145235,
"tgt": "Suggest treatment for cerebral edema",
"src": "Patient: had horrible brain swelling about 3 months ago could not move neck and the pain went from left graduall to the right waited a month to visits to emergency room no test just mri because of past tbi felt death and buzzing went to neighbors allergy doctor gave me levofloxacin started disappating in 24 hours tokk for two weeks. side effect always have had to clear throat since 12yr old 42 now seem to clear itwhile taking came back. the question is my nodes on either side of my occipital pit have never been fully recovered and in constant pain should i get tested for tb or west nile or what fungal infection bacterial or what Doctor: I read your question carefully more than once but unfortunately I am afraid it was really difficult to understand what really happened to you.You speak of treating cerebral edema, which can have many different causes (stroke, tumor, infection) which must be identified and treated accordingly. Then you say you waited for a month to visit the emergency room which makes the cerebral edema a little questionable because it is a very serious finding with the patient often in a coma. Afterwards you say you had no tests but a MRI, which actually is the best test to detect edema, but tell us nothing of its findings. You close it out telling us to have taken treatment by allergy doctor and got better.So I as a doctor started out thinking you had a dramatic life threatening condition like encephalitis and ended up thinking it looked like the more benign sinusitis or rhinitis.So please try to ask your question again. Try to avoid fancy medical terms like cerebral edema which might not be correct, and tell us your symptoms in plain simple words (headache, vomiting, nose congestion and discharge, fever etc) using as many details as possible to describe their onset, character and frequency. Include also your age, previous conditions and results of tests like MRI.Me or my colleagues would love to help you but you have to help us to evaluate your case properly."
},
{
"id": 103254,
"tgt": "Taking Levaquin and Albuterol for recurrent chronic asthmatic bronchitis. Reason?",
"src": "Patient: why do i get chronic asthmatic bronchitis every time i get even the slightest little cold? I always end up at urgent care and need a prescription of levaquin an inhaler and albuterol for my nebulizer. my cough will last sometimes for 5 or six weeks. i cant be near my grandchildren at all if they have even the slightest cold i end up so sick. this happens to me several times a year. could i have something more serious than bronchitis? Doctor: I don't know your country of origin and / or some herbal preparation are available or not? you can take care of your cold and asthma by increasing your immunity. cap. Giloy and cap. Tulsi can be taken twice a day. home remedies- black pepper-2, cardamom-2 , long pepper 1..dry ginger, a small slice- make a coarse powder. boil it with a cup of water till 3/4 remains. filter and add sweetener as per your taste. sip it like tea two times a day."
},
{
"id": 26450,
"tgt": "What is the dosage of AMTAS for high BP?",
"src": "Patient: my mother aged 55 years has recently diagonsed high BP last 155/90 mm to 195/95 mm. ddcter has prescribed AMTAS 5 mg. She tooke it 12 hrs before now she get it checked the reading now 168/91mm. kindly suggest shd we give her 5 mg more now or follow docter instruction and wait till 24 hrs Doctor: Hello!Welcome and thank you for asking on HCM!Regarding your concern, I would explain that it is necessary a period of up to one week for the antihypertensive therapy to show its effect (when it is started or when changes are made). So, I agree with her doctor that you should wait before making some changes to this therapy. From the other hand, this is a really small dose, but it is started in this way to avoid possible adverse effects related to amtas intake. So if her blood pressure is still above 130/90 mm Hg after a couple of days you should discuss with your doctor the possibility of a dose increase or adding another antihypertensive therapy. Hope to have been helpful!Best regards, Dr. Iliri"
},
{
"id": 106760,
"tgt": "What does pain in the lower back indicate?",
"src": "Patient: I have been having pain in my lower back for several days and nights. Did not fall or bang it on anything. could it be kidneys? Both sides. Hurts to lie down, get up,always. thanks Did not realize there was a charge for this. Please disregard. Doctor: Hello and Welcome to 'Ask A Doctor' service. I have reviewed your query and here is my advice. Sir kindly get your self examined and get ultrasound abdomen done also MRI spine to rule out of you have any disc prolapse. Without examination or investigation it's difficult for us advice for few problems. Hope you understand the constraints of online consultation. I hope this info Helps You Sort out your problems and if you feel its helpful kindly give me user thank vote happy health! Hope I have answered your query. Let me know if I can assist you further."
},
{
"id": 146796,
"tgt": "Had spinal fusion. Can I lift weights and play sports?",
"src": "Patient: how are you doing i had spinal fusion l5 s1 rods screws and cage ( i wish i never rushed into it i personally didnt think i needed it well not right away any how my doctor kinda rushed me inot it i should never have listened) my question to you with it fully healed 2 years out now how strong is it, what kind of abuse can it with stand and what can i do and no do. Can i lift weights play sports etc im a 25 year old very healthy male other wise and very active well once was i am just so scared to re injury it i just dont know how strong something like this heals every time i ask a doctor i never get a straight answer or any answer really. I am also a iron worker i build sky scrapers just like you see on tv. So thats it and what else can i do to make it strong like lifting weights will that help right now i have no pain hardly at all and ever i just want to get back to my normal life without worrying what i can and cant do please help me with your best educated answer!!! you can reach me at YYYY@YYYY my name is ryan thank you very much!! Doctor: Hi Ryan. If it has been 2 years and if ur not having low back pain, it is likely that there is fusion between L5 and S1 which is the main aim of inserting screws and placing a cage. An x ray of your lumbosacral spine will confirm the fusion. If there is still doubt in xray ..then a CT scan will surely confirm the fusion. Once you confirm that it has fused, then there is no need to worry...U can bend, lift weight and lead a normal life."
},
{
"id": 43103,
"tgt": "What can I do for the vomitting feeling after taking ofloxacin along with Novelon for the IVF?",
"src": "Patient: hello sir , i m 28 years old my tubes are blocked,, i get the treatment of laphastreo,, but doctor says my tubes r not healthy it cant get open,, so i m gng for ivf,,, my first cycle get fail nw i m again gng for 2nd cycle in January,,, my doctor had asked me to take novelon,, i also getting some infection so doctor asked me to take Ornidazole & ofloxacin tablet nd cx-3(something),, i m getting vomiting from yesterday,,, wat should i do now Doctor: This is a side effect of the drugs. Try taking the pills after a meal. If this doesn't help then try tab Diphenhyramine or tab dromperidone 15 min before taking the tablets. This will surely solve your problem"
},
{
"id": 74980,
"tgt": "Should I be worried about fast heart rate and body trembling after breast reduction?",
"src": "Patient: Hello, I'm three days post op from a breast reduction. I'm experiencing pounding heart with a fast rate and whole body trembling. Earlier I had a fever which now is gone. I just can't stop shaking or calm my pulse. I'm on no medication. Should I be worried? Doctor: Hey there,I think you should worry a little.Get done chest X-ray and EKG to rule out lungs and heart related primary cause.Get consulted from good physician."
},
{
"id": 108725,
"tgt": "What causes sever pain on the upper middle and back while suffering from viral meningitis?",
"src": "Patient: Hi I am a male 32 years old, 183cm (6ft) 115Kg, I suffered from viral meningitis (swelling of the brain) and was prescribed 3 different stong pain medications, 1 Panadine Forte, 2 Tramadol 3 Endone, I was administeded a fair amount of drugs during the 2 days I was at hospital and then sent home with the above mentioned drugs to manage the pain, which lasted 2 to 3 weeks. I am better now but awake each and every morning with severe and extremly painful upper to mid back pain, it feels as though an elephant has sat on my chest all night and the pain is deep in my back not my chest or front. Could liver damage be causing this pain? Doctor: Thank you for the question. If there is no accompanying fever with back pain then it is not due to infection.Lie in hard bed, do hot fomentation, take tramadol 100mg BD and chloraxazone 500mg BD for 3-5 days.Get in touch with an orthopedist.It is unrelatedto liver pathology,however, to remove doubt get LFT,serum creatine,urea,K,Na test done"
},
{
"id": 171194,
"tgt": "What to do with baby's bcg?",
"src": "Patient: My baby is 3 months old, 15.8lbs. I accidentally pricked my baby's bcg because I stupidly thought it was a boil or an infected insect bite, so I tried to squeeze all the pus and blood inside. What would happen to my baby? Would the bcg be ineffective? What should I do? Doctor: Hi, BCG will not be ineffective. When a bcg vaccine is given, a papule develops at the site at 2-3 weeks, which gradually heals leaving a scar. You need not to do anything. Regards - Dr Deepak Patel, MD Pediatrics"
},
{
"id": 111347,
"tgt": "Suggest treatment for severe back pain",
"src": "Patient: my mother is having serve back pain and her stomach is also getting fat like a balloon, my doctor have earlier prescribed Ultracet and pan-d, but my mother is not getting relief from that. Now one my my known doctor told me to give her zule forte. Can I give both of these together or suggest me some good remedy for her Doctor: You must get to the root cause of pain. And the abdomen increasing in size is a concern. She must have testing, beginning with an x-ray to begin with, not only to look at the lumbar spine, but to look at the bowels and seek a reason for the bloating. But for acute pain relief, ultracet is for pain, and zulu forte is a muscle relaxant, and you can give them together as long as she is not driving or operating any other type of machinery."
},
{
"id": 16669,
"tgt": "What causes tightness in the chest?",
"src": "Patient: OMG...Just where do I begin...my array of problems started 4 days before Thanksgiving of 2010...My Dentist gave me some antibiotics to take before he pulled my tooth...and all was well...til one morning I woke up scratching like I was entering the world record for how many times can one person scratch per minute...believe me I would have won, hands down...lol..being 49 years young...lol...lol..I have never experinced HIVES ever before....yep this was my first ride....I have to say I hated it lol...not fun at all...I phoned a friend which told me to take a oatmeal bath, I thought to myself....that my dear friend has lost her mind, an oatmeal bath???...hmmm it was not what I expected to hear...lol...well, to make a long story short...we are now in March of 2011 and I have turned a year younger....lol...and I still battling hives like its nobody s business...I am now having chest tightness, but I don t have shortness of breath...its only when I stretch my arms apart...that I will feel the tightness...I feel jittery most of the days and night, sometimes I feel my heartbeat all over my body...and sometimes my heart is at the derby..lol, yepppp its off to the races it goes, I must say I don t like the race, but I can deal with it... I think lol...within a year I had to pull three tooth all of them are at the back of my mouth, each time the Doc tell s me that its because I m losing bone so therefore there is nothing else for the tooth to hold on to...hmmm...I am fatigued sometimes, and sometimes I feel like I can take on the world...although rare, but I do have superman moments...right now as I m typing...I am experiencing pain on my right arm, and back shoulder pain...my legs are often swollen, but I think this is due to the fact that I am sitting on a desk for more than 12 hours a day...my hands are often tight, sometime I feel like if I squeeze them anymore than I already am, they will pop like water balloon.....so Doc s any Doc s...can someone ease my mind, as to what I m going through...any help and advice will be well respected and followed. Yes I am desperate for some answers... Doctor: Hello, In your case, it could be related to Drug-induced allergy, idiopathic allergy or something else. Therefore, I suggest you go through a series of tests (RBT, KFT, LFTs, Routine and microscopic urine test, ECG, ABD USG). Hope I have answered your query. Let me know if I can assist you further. Take care Regards, Dr Sandeep Kumar Karn, Internist / Hospitalist"
},
{
"id": 64149,
"tgt": "What causes a lump under the eye?",
"src": "Patient: hi, my 10month old infant had fever and flu last week. and about three days ago, I noticed a small bump at the outer corner of her left eye. there isn t any redness and doesn t seem to bother her. we have been using the eyedrop prescribed by doctor for 3 days but the bump doesnt go away. what could be the possible reasons and what should we do? Doctor: Hi,Dear ,Good Evening.Thanks for you query to HCM.This is Dr.Savaskar from India helping your out of the health query. I studied your query in-depth and understood your health concerns.-In My opinion the Cause of the lump under the eye, in referrence to the facts of your query--is-Chalazion-which is the infection of the Meibobian Gland at the outer corner of the eyelid and usually goes after warm compresses and anti-inflammatory treatment in few weeks time.-It could be -Stye -infection of the hairfollicle of the outer eye on lt side.Treatment-is Antibiotics-eye drops /anti-inflammatory drops / hot compresses by hot sponges for both the conditions.-Oral Syrups could be added if the response is slow-which may be a case in your babies case.Hope this would help you to resolve the issue with your Eye specialist doctor.Wellcome for furher queries in this issue to HCM.Wishing a fast recovery for your baby.Have a Good day.RegradsDr.Savaskar M.N.M.S.Genl-CVTS"
},
{
"id": 199655,
"tgt": "What causes dry skin and cuts on the penis causing burning sensation?",
"src": "Patient: hi doctor. 2 days back i realised iam having burnig sensation on my penis foreskin. when i came home from office and checked it was a small cut on the left side foreskin and it was reddish circle on the foreskin. so i immediately applied candid - B and then today when i checked the red circle is not there and the cut is also not visible but i see a whice dry skin circle instead of the red circle. what could it be? Doctor: HelloThanks for query.Cracks and reddish foreskin is mostly suggests infection of Foreskin (Posthatitis).Check your blood sugar levels to rule out diabetes as this is one of the first presenting symptom of Diabetes.Take broad spectrum antibiotic like Augmentin twice daily with topical anti fungal cream Candid cream for local application .Ensure to wash your genitals with warm water twice daily.It should get cured within a week.Dr.Patil."
},
{
"id": 205108,
"tgt": "Does bipolar disorder cause acidity and digestive issues?",
"src": "Patient: i know a person thats bipopolar and she has stomach problems for months and she has had many tests which show excess acid in stomach, most tests show nothing..my question is : is it possible its all in her head from her disorder????sometimes she acts hiper and likes attention from others....i think she doesn t like to be a lone....i also think she likes attention and gets that from saying she doesn t feel well. Doctor: Hi and thanks for question.bipolar dis. itself doesn't causes hyperacidity or stomach discomfert. some times it's happen with drug used in bipoar dis. second u write that he had some attention seeking behavior which is not occur in bipolar dis. so kindly consult psychiatrist for detailed diagnosis.thsnks"
},
{
"id": 159066,
"tgt": "Voice becoming hoarse. Why? No cold. Cancer chances?",
"src": "Patient: recently almost every day,usually later in the day,my voice becomes very hoarse and scratchy.Im not sure why this is happening.I have not had a change of address or any colds or problems with ventilation in my home.A friend of mine was concerened ,they had cancer discovered when a Dr. noticed his voice was so ruff and hoarse Doctor: Hi, I am Dr Mbong. The likelihood of your voice hoarseness being of cancerous origin is likely if you smoke, take lots of alcoholic spirits, have a family history of throat cancer, strain a lot with your voice or do oral sex (some strains of HPV virus which can be contracted from oral sex) can cause throat cancer. Otherwise if you use your voice a lot (at work or somewhere else) especially lately, then you can have voice hoarseness without it being a cancer. Another cause may be the onset of a palsy of the nerve that innervates the voice box. It could also be onset of symptoms of a voice box muscle involvement of a myaesthenia gravis (or some other related muscle weakness disorder) which often worsens with time of the day. Please see a physician for thorough examination and proper work-up in order to determine the cause. Courage and wish you the best, Dr Mbong"
},
{
"id": 48604,
"tgt": "Is it safe to plan pregnancy if husband has polycystic kidney disease?",
"src": "Patient: My husband has polycystic kidney disease (the dominant type) I am curious if we could have a child with the recessive type. I know the genetic mutation is on two different genes. So would it be possible for our child to have the recessive type which is more severe or would they only be able to inherit the dominant type? Doctor: Hi,Thanks for writing in.Autosomal dominant PKD is the most common inherited disorder of the kidneys. The phrase \"autosomal dominant\" means that if one parent has the disease, there is a 50 percent chance that the disease gene will pass to a child.Autosomal recessive PKD is caused by a mutation in the autosomal recessive PKD gene, called PKHD1. Other genes for the disease might exist but have not yet been discovered by scientists. It is not possible for autosomal dominant polycystic kidney disease father to have autosomal recessive polycystic child.So your child might have risk of autosomal dominant but never autosomal recessive polycystic kidney disease."
},
{
"id": 168443,
"tgt": "What causes continuous cough and cold in children?",
"src": "Patient: hi my daughter is 3yrs10mnths old she is suffering from cough &cold past one yr now doct recommend planokuf,levolin syrup for her for 5 days &blood testso please tell me is this a symptom of asthma or what kind of disease is it .if we medicine cold &cough attack her or if we continue the medicine then she is fine .please tell me is it serious.after these medicines she will be fine or not Doctor: Hi..don't be worried. The term 'asthma' is not used in children before 6 years. This is known as recurrent wheezing, and very few children with recurrent wheezing will develop asthma in their school going age. We use an index known as 'Asthma predictive Index' to predict the possibility of developing future asthma in children of 3 yrs age. Cold and cough is common at her age group. You need to known whether your child is wheezing or not ( whistling like breathing sound) if yes you have to maintain a dairy where you have to keep record of frequency, duration and precipitating factors of the wheezing episodes. As far as levolin is concerned use only at your pediatricians advice. It does'nt cause dependence but has many side effects like palpititons, tremors etc"
},
{
"id": 54386,
"tgt": "What is the remedy for fatty liver?",
"src": "Patient: Good day doctor am having fatty liver for the last one year or so as per the scan but doctor didn't prescribe any medicine. Now i can feel the enlargement over abdomin slightly. I am not a alchoholic but takes 1 to 2 pegs once a week. I am not feeling any pain but I have irritation very rarely. What are the best remedies i can go for, Kindly advice. Doctor: Hi thanks for contacting HCM...Usually fatty liver lead no serious complication ....Still it can be treated by taking life style measures....Take low fat diet...Fruits taken more....Avoid alcohol drinking....Use less oil in cooking...Sunflower or olive oil like good oil can be used in less quantity...Regular 30 minute active exercise done...Green leafy veg helpful like ....-bitter gourd -arugula -Chicory-spinach etc....Beet roots , carrots, lemon juice , Indian gooseberry are good for liver.Non veg fried food less.Refined food avoided.As far as drug concerned udiliv can be taken..Take care....It will take few month for resolve fatty liver....Dr.Parth"
},
{
"id": 219460,
"tgt": "How long does it take to get pregnant post removal of implanon?",
"src": "Patient: I m 18 years old and i have a 1 1/2 year old. i got the implanon removed on may 12 2010. after having it in for 14 months. the whole time i had it in i never had a period, even now after it is removed i still havent had a period? we are ttc so i need help. what is the average amount of time after removal of implanon it takes to get pregnant? my doctor told me 3-6 months but i see and hear alot of women who get pregnant the 1st or 2nd month after having it removed, me and my husband have been having unprotected sex since it was removed and I ve test about every two weeks since removed but test are neg. please help!!! Doctor: Implanon does cause irregular periods or skipped periods while it is in. The time to get pregnant will depend on the return of your periods . You need to consult a gynecologist about not having periods now after removal of the implanon. 6 months is usual time it should take but main thing is the period should return and you should ahve ovulation which will help conception."
},
{
"id": 89242,
"tgt": "Suggest treatment for pain in the scrotum",
"src": "Patient: sir iam from india my age is 23.iam doing hand jobs for the past 15 years and also before 8 months i got an accident on my both side of scrotum and there is very pain and seems something broken ,and in my penis nerves can see clearly on the penis and i have very pain .so i kindly request u to provide me the details of my problems. Doctor: Hi.It looks that your had fractured the Penis while masturbating. This has given swelling on the penis with enlarged veins and that has spread to the scrotum . I would advise you to take an urgent consultation of a General Surgeon or Urologist to see for the probable diagnosis,Get an MRI of the penis to see the reason. MRI is very good option and the treatment will depend upon this diagnosis."
},
{
"id": 172198,
"tgt": "What is the treatment of belly pain for children ?",
"src": "Patient: sir/madam,my 3 years old baby girl is suffering mild or moderate belly pain, pale stool with some solide and some lequied stool. her previous case history one month ago she got belley pain and vomiting 4 to 5 times, then my dr. prescribe lansoprazol 15mg, and emiset to stop vomiting. then she got loose motion so dr. first prescribed livofloxacin and then ofloxacin. problem was cure for that period, now during 3 to 4 days she again got belley pain, then i given her the medicin neopeptine as i think it was for indigestion, but i found mild or moderate belly pain, pale stool with some solide and some lequied part. what should i do now. Doctor: The first thing to do when the same symptoms are repeated is to get her investigated. Do check her out with a stool test, an ultrasound of her abdomen and a complete blood count initially. Take her to the pediatrician with the results of these tests. Or go to the doctor beforehand and ensure that they order those tests."
},
{
"id": 28275,
"tgt": "Suggest treatment for fluctuating high BP and diabetes",
"src": "Patient: Sir, my wife, 67, taking seloken 25xl and cilacar 5. Last one month, half of tablets taken from each one. Remaining time, took cilacar 5- 1/2 taken with 4 intervals based on sys pressure while reaching 160. But my doctor advice is to take medicine in the morning 1tab seloken 25xl and evening one tab cilacar 5 tab. It is all carried out in 40 days. On 3.10.14 she took half tab each seloken 25 xl and cilacar 5 in the morning. After 4hours, we noticed the BP is 202 sys, 79dia and pulse 66. My self, husband, went to Dr and informed the situation. Dr advised to take as per priscription - seloken 25 -1 tab& cilacar-5 one tab and evening one cilacar. Today, 5.10.2014, she took as per Dr s advice. intially decresed for 2hrs from 179 to137 sys, 80 to 65 dia and pulse 71 to 68. But, after that pressure increased to 190-71-77 within 1.30hrs. Immediately we went hospital.Today, OP treament is not available. Before that, to avoid any further increase, I took cilacar 5 one tab while going to hospital. After 3/4hr we measured Bp. It is 166-64-79 and further after one hour Bp is reduced to 134-60-70. she is diabetic patient more than 15yrs and 10yrs for Bp. Kindly give your good advice to relieve from this suffering. Now diabetic under control. Doctor: Hi welcome to HCM.I understand your query and concern.High blood pressure should be treated thoroughly and should be kept under control or else will lead to complications like Dilated cardiomyopathy and ischemia to the brain.Best treatment for elevated blood pressure is through drugs like beta blockers and Ace inhibitors.Before that adequate management and work up is needed.I advise you to have a baseline 2 dimensional echocardiography,ECG and lipid profile to assess the basic cardiac reserve of your heart.Restrict the intake of salt to less than 6g/day.Regular physical exercise in the form of brisk walk for 20 min a day for 5 days a week.Drugs like antihypertensives and antiarrhythmics will help.Reduce the intake of fatty and fried food.One pomegranate a day will help to keep your heart at good condition. Stress management is most vital in your case.Also monitor your blood sugars regularly along with renal function tests.Continue with your present medication without fail.So Consult a Cardiologist for dose revision of your present drugs and further expert management.Post your further queries if any.Thank you."
},
{
"id": 157759,
"tgt": "Not able to have food. Undergone surgery. Stomach infection found in the lining of stomach walls",
"src": "Patient: My farther is very duck and has not been able to eat any foods for a very long time a feeding tube was implanted into his stomach after killing a canxerouse growth at the top of his stomach into the esphiges recently went for a surgery to remove that part of his stomach And a stomach infection was found in the lineing of his stomach walls and now he still has a very hard time eating any food sand comains he's always cold Doctor: Hi there,Thanks for your query. I fully understand your father's agony.The feeding tube is most likely the cause of infection in stomach and intestine. It is usually due to bacteria called H.pylori.A 10-14 days' course of anti-H.pylori (consisting of clarithromycin, tinidazole and anti-acid drug) will lead to significant improvement. Also, addition of Probiotics and digestive enzymes will add to the improvement.I wish him speedy recovery.Dr. Rakesh Karanwal"
},
{
"id": 207147,
"tgt": "Suggest management of lack of confidence and focus",
"src": "Patient: Sir actually i m suffering from a mental problem in which i want to do lots of things but can t focus on single one. I think too much about my problems of family friends. I always doubt on the behaviour of my acquiantances. I some times have good feelin and sometimes bad feelling for them. It makes my confidence week and i can t find exactly what i want to do. This thing also led to tension in my mind regarding my friends and families. Sir i badly need your help i also want to meet u. Please help me.with that. Doctor: DearWe understand your concernsI went through your details. I suggest you not to worry much. Dear, you must understand that YOU are YOU. You cannot become somebody else, and no one can make you some one else. Your behavior is yours. You should not worry about the behavior of others because nobody worries about your behavior. This happens mainly because everybody are busy with themselves. That is the key. You need just a counseling, that's all..If you require more of my help in this aspect, Please post a direct question to me in this URL. http://goo.gl/aYW2pR. Make sure that you include every minute details possible. I shall prescribe the needed psychotherapy techniques.Hope this answers your query. Available for further clarifications.Good luck."
},
{
"id": 175503,
"tgt": "Is the mucus discharge related to the coughing?",
"src": "Patient: Hello, We have a three year old daughter. She's had a persistent cough and congestion the last two days. This morning she woke up to go to the bathroom and said she had to go \"poo poo\". My wife took her in the bathroom and she had a clear, mucus discharge in her underwear. She complained of a little stomach pain afterwards. Her cough has triggered her to gag twice and vomit. She had a regular BM yesterday and no diarrhea. Could the mucus discharge be related to the coughing? Doctor: Hello. I just read through your question.What you describe is very common in this age group. The mucous is typically swallowed and is ultimately seen in the bowel movements. If you notice the symptoms worsening over the next 3-4 days, or a fever develops, I recommend consulting with your doctor."
},
{
"id": 186289,
"tgt": "What to do if wisdom tooth can't come out?",
"src": "Patient: I have a small jaw and hence my wisdom tooth can't come out. Sometimes, I feel pain around my left lower jaw and left side of neck. Is it the 3rd molar pain? As soon as I take Voveran (once), the pain goes. Anything serious?? I also do saline gargles and wash my teeth twice a day Doctor: thanks for your query, i have gone through your query, the pain in the jaw could be because of the wisdom tooth. take radiographs after consulting oral physician and if the space is not there for the tooth to erupt get it removed. take a course of antibiotics and analgesics. do saline gargling. i hope my answer will help you. take care."
},
{
"id": 182893,
"tgt": "How can inwards growing teeth with excessive saliva be treated?",
"src": "Patient: Dear Doctor, I have broken front teeth which got rebalanced recently by a good dentist after they got maltreated by a bad dentist. Aside from that I also have inwards growing teeth, because I have been playing trumpet for a while. The crookedness is causing changes in the way I eat and I cannot put my teeth together anymore. The teeth do not touch correctly anymore and they are now also affecting the balance in my mouth; sometimes my mouth begins watering for no reason and I feel off balance on occasion and nauseous. I read about a solution called invisalign, but there was one dentist from Harvard who had a really big mouth towards me; he refused to treat me unless he got to pull three front teeth etc. He also claimed that I would never be able to handle corrections on my teeth because he had a client who could not adapt his embouchure to an overbite correction and he compared me to that client. He was a pathetic man, claiming invisalign was nothing but a piece of plastic. I simply wish to look- and feel better and am very well trained for resetted embouchures, because my teeth broke more than once, using the Berklee schillinger and zero pressure method. I have also had very good teachers and can adapt to many different situations. But I am looking for a biological solution and not enhancements because I know that the human being is already in balance with itsself, it is supposed to be like that. The human being is supposed to be at peace and balance with itsself and within the comfort zone; doctors can help bring the human being into the comfort zone. Doctor: thanq for your query.i can understand that every human being wants to be at peace and balance.but in your case this may not be possible. moreover you have not mentioned your age and gender. as your previous doctor suggested if you want treatment to be done, you have get your teeth removed because your teeth have grown inwards very much that mere uprighting them may not be sufficient to get the required space to move other teeth. the entire procedure will take atleast 1.5-2 years if u keep your doctor's appointments and allow him to do his job. the beginning to the treatment may be painful and u will have to restrict your diet. the dentist from Harvard you were referring to must have explained you all these. so you better tune your mind to all these issues.all the best!!!!"
},
{
"id": 32074,
"tgt": "What is the treatment for runny nose and sore throat?",
"src": "Patient: I am 27 weeks pregnant. I had a runny nose and sore throat last Friday. The sore throat went away, but my nose completely clogged for 5 days. I took a benadryl but it is still clogged. I used Afrin once, but it is still clogged to the point where I haven't tasted food for 5 days and I can barely get any mucous out when I blow it. Doctor: Hi..Welcome to HEALTHCARE MAGIC..I have gone through your query and can understand your concerns..As per your complain it seems that you have upper respiratory tract infection also known as Common cold that is viral infection and as you are pregnant you can take only limited medications.Also viral infection is self limiting and resolves within a day..You can take antibiotics like Amoxicillin to prevent secondary bacterial infection.Take tylenol for pain..Do warm saline gargles, steam inhalations, saline nasal irrigations and use a humidifier in your room to breathe moist air and it will soothe the nose and throat..You can take plenty of warm fluids and take sufficient rest..Do warm compresses over the chest and around the nose..avoid chilled and spicy foods..Hope this information helps..Thanks and regards.Dr. Honey Nandwani Arora."
},
{
"id": 208875,
"tgt": "Is it advisable to take atenolol to help nerves?",
"src": "Patient: I was perscribed atenolol 25mg for nerves because I have to give a speech in from of 500 people tonight, and I am wondering if I can take one now (cause I m already freaking out a little) its 1:30 here in california, Then take one again later tonight at about 7? Doctor: Hello,Thanks for choosing health care magic for posting your query.I have gone through your question in detail and I can understand what you are going through.Atenolol will not help in relieving anxiety. ITs an anti hypertensive. Hope I am able to answer your concerns.If you have any further query, I would be glad to help you.In future if you wish to contact me directly, you can use the below mentioned link:bit.ly/dr-srikanth-reddy\u00a0\u00a0\u00a0\u00a0\u00a0\u00a0\u00a0\u00a0\u00a0\u00a0\u00a0\u00a0\u00a0\u00a0\u00a0\u00a0\u00a0\u00a0\u00a0\u00a0\u00a0\u00a0\u00a0\u00a0\u00a0\u00a0\u00a0\u00a0\u00a0\u00a0\u00a0\u00a0\u00a0\u00a0\u00a0\u00a0\u00a0\u00a0\u00a0\u00a0"
},
{
"id": 183115,
"tgt": "What causes bleeding in mouth?",
"src": "Patient: i am 45 year old man,dont have any other problem not taking any medication, i have bleedeing in mouth since,mid december, its worse at morning, i already being to dentist and GP,my gums are good as my dentist opinion, i feel my mouth dry at early morning,dose that couse bleedig? my blood tests are normal, ony had B12 deficiency,i have seen my GP three times,he couldnt make any diagnosis,can you please help me what i shoud do next?sanjay Doctor: Hello,Thanks for using HCM.In many of the patients with bleeding problem there is deficiency of vitamin C that leads to fragile blood vessels and bleeding. This is more common when there is improper food habits.If you are a tobacco chewer, there may be benign or malignant lesion that may not be visible due to small size. Ask your dentist for the coagulation profile which includes bleeding time, clotting time and INR. If there is abnormality in that he will be able to correct problem.In case that is already done see specialist ent surgeon. He will examine the posterior part of oral cavity as there may be pathology in that region.Thanks and regards"
},
{
"id": 62656,
"tgt": "What causes a lump at the back of the ear?",
"src": "Patient: I have a lump on the back of my left ear. I ve had since puberty (so 8 years now) and I ve always been curious as to what it was. I ve poked it with a disinfected needle and no pus or anything has come out, it seemed to slide through with minimal pain at all. Doctor: Hi Dear, Welcome to HCM.Understanding your concern. As per your query lump at back of ear with minimal pain could be because mastoiditis, sebaceous cysts, throart infection and it could be due to lymphadenopathy. Need not to worry as it is mainly harmless. I would suggest you to visit ENT speacialist once and get it examined. After proper inspection you can go for medicinal as well as surgical treatment. Donot chew very hard from that side if it is painful.Hope your concern has been resolved.Get Well Soon.Best Wishes,Dr. Harry Maheshwari"
},
{
"id": 163429,
"tgt": "How often is it normal for a toddler to urinate in a day?",
"src": "Patient: Hi - My son is 3 yrs old and is just learning to potty train. He has been peeing in the kids potty for past couple of days now. I m just curious how often is too often for him to pee....this evening, for example, he has had to go pee 3-4 x in the past hour....he does a little each time. Is this normal? How often is too often? I m not sure if he s just playing around or really has to go but he sayys he has to go each tme and does a little each time.... Doctor: Hello,Passing small amount of urine at frequent interval may be seen in urinary tract infection. It is also seen in constipation where stool get impacted.First, ensure adequate liquid intake, look for penile hygiene, if it persist for longer, consider for urine examination.Hope I have answered your query. Let me know if I can assist you further.Regards,Dr. Sachin Kumar Agarwal"
},
{
"id": 219922,
"tgt": "Suggest dosage of Prontogest for spotting during pregnancy",
"src": "Patient: I'm 30 weeks and i have a pain like period pain, and also I have spotting from the beginning of my pregnancy. I'm now taking gynipral and the Dr told me to rest and take prontogest pessaries or injections. Is this right and what is the suitable dose for prontogest and when should I stop it?? also is it harmful for the baby Doctor: HI, Thanks for the query. I understand your concern. POntogest pessary 200mg to 400mg daily once a day is indicated in case of risk of preterm delivery due to progesterone deficiency. ( Dose to be adjusted as per need by treating doctor . It is usually given till the baby is viable ( > 37 weeks ) both developmentally/ physically. & the withdrawn about 1 week before planned delivery.( again to be decided & advised by treating doctor.. according to individual conditions of the patient ) Pontogest contains natural hormone Progesterone.. so there is no harm to baby. Thanks."
},
{
"id": 146525,
"tgt": "What causes small bump on lower back side of head?",
"src": "Patient: Yesterday I had noticed a small bump on the lower back side of my head.It was small, about the size of a dime if not smaller. In about eight hours time it had grown.to about the size of two quarters. It is firm and does not move, but it painful to touch.Eight years ago I had a brain tumor removed from my right frontal lobe of my brain. Is this bump cause for concern, or is it likely a cyst? Doctor: Hi, I had gone through your question and understand your concerns. The bump you noticed enlarged in few hours, probably is caused by local infection of hair roots, or skin glands. However I suggest you to contact your local Doctor to check for the infection signs. If your Doctor is not convinced about the nature of the bump, then you should have brain imaging study. Hope this answers your question. If you have additional questions or follow up questions then please do not hesitate in writing to us. I will be happy to answer your questions."
},
{
"id": 217311,
"tgt": "What could stinging pain in nipples suggest?",
"src": "Patient: I am a 68 year old male for past three months I have a stinging painlike sensation when pushing or squeezing area adjacent to left nipple. My doctor, an internist, told me not to worry about it, but I am still concerned about it. When I take a warm bath and put warm to hot water over it with a wash cloth the problem subsides. The left breast feels firmer than the right breast but it is same size. What could this be? Doctor: Hi, Thanks for choosing healthcare magic. According to your complains it looks like an infection around the neeple. It may be just an infected sebaceous cyst also. Consult a physician and take a course of painkillers and antibiotics. Usually it should subside. If not , then you will have to undergo further investigations for a proper diagnosis . So my advise is dont delay and get a correct treatment at the earliest as your age fits in high risk category patients.Pl. feel free to ask if you have any other doubts. Thanks and take care."
},
{
"id": 130656,
"tgt": "Suggest remedy for numbness in hands and feet",
"src": "Patient: I have just signed up for DSHS, awaiting my insurance coverage with Ameria group, and am looking for a physician in the mean time. I am having difficulty finding a physician. Right now I am experiencing numbness in my hands and feet. Thank you for your reply Doctor: Hi,Numbness is a lack of sensation in some particular area of the body. There are many causes of numbness of legs and feet like nerve compression, vitamins and minerals deficiency, peripheral neuropathy, radiculopathy, PVD, Neuralgia, diabetic neuropathy, spinal stenosis, slipped disk, circulatory disorder, osteomalacia, leprosy, cervical and lumbar spondylosis, intracerebral hemorrhage, stroke, brain aneurysm, multiple sclerosis, anemia, hypoparathyroidism, malignant hypertension and so on. For the relief you can take warm compress which will to increase blood circulation and will relax the muscles and nerves.You can do massage to get rid of numbness. Do regular exercises and yoga to improve the mobility. Take one table spoon epsom salt in a tub filled with warm water and put your hand and feet in it for 10 to 15 mins.Hope I have answered your query. Let me know if I can assist you further.Regards, Dr. Ankur"
},
{
"id": 221262,
"tgt": "Is it still possible to be pregnant when told I had miscarried?",
"src": "Patient: Hi,this will sound a very weird question but is it possible to still be pregnant when told i had miscarried? when i was 7 weeks pregnant i had early scan because of previous miscarriage. Scan showed good strong healthy heartbeat showin at just 6 days behind what i thought i was. 2 weeks later another scan showed that fetus had vanished. Gestational sac was there and had still been growing to the dates i should have been. I passed nwhat i believed to be the sac 2 weeks later. A follow up scan showed empty uterus but still showed a very small bleed still. Antibiotics were prescribed to prevent infection and i bleed for a few more days. I have since had 3 regular periods excatly 28 days cycle. However the last 3 or 4 weeks i have noticed that my stomach has slightly swollen and i regularly am feeling movements in my stomach. I am scared to see my gp because i feel he may think i m paranoid or displaying signs of a phantom pregnancy! please help! my partner thinks its all in my head as he saw alongside with me the scan that showed my uterus was empty and he was there when i was told that the fetus had vanished! Doctor: Hallow Dear,Vanishing of foetal heart beats was confirmed by ultrasonography meaning thereby that you had missed abortion. Later on you aborted completely as evident from the ultrasonography. So continuation of this pregnancy is out of consideration. Your belly has grown and you are feeling some weird movements in the abdomen. This is most likely due to over eating due to which your belly has increased in size and you have gaseous problems. Gurgling movements of gases in the intestines is perceived often as foetal movements. I would advise you to take some carminative mixture, plain soda to expel the gases. If needed, you may take some digestive enzyme preparations. Worms also can cause such symptoms. You may take deworming medicine. This should give you relief. Pregnancy starts showing visibly by increase in the size of the abdomen after 16 weeks. So this increase in the belly size cannot be correlated to pregnancy. However, I do not wish to insult your feelings and perception. I am sure you are honest in describing your symptoms. Therefore, to confirm or rule out your suspicion, please perform urine pregnancy test on your morning first urine sample. This will clarify the dilemma. Even ultrasonography can give you direct visual evidence of the pregnancy status. I am sure, this should help you. Dr. Nishikant Shrotri"
},
{
"id": 192678,
"tgt": "Does urticaria cause low sperm count?",
"src": "Patient: Hi doctor, My husband is suffering from pressure urticaria. He is having some medicines for treating it. He feels that those medicines are reducing his semen. We are newly married and I want to get pregnant soon. I just want to know if pressure urticaria has this side effet or not? Doctor: Hello, Urticaria does not affect the sperm count or quality of the sperms. If your sperm count is persistently low, other causes like varicocele must be ruled out. Consult a urologist and get evaluated. Hope I have answered your query. Let me know if I can assist you further. Take care Regards, Dr Shinas Hussain, General & Family Physician"
},
{
"id": 209072,
"tgt": "How to improve concentration?",
"src": "Patient: Dear sir,I am a CA student, as my exams are approaching , i should be studying now since it require lots of study,i know its very important for me to study, but i dnt study , my brain ask me to study but i prefer to spend time on facebook or other activities,i run away from books ,when i start studying, my mind gets diverted or i feel sleepy, but what should i do to study and spend my time in studies,i understand the importance of time , and i know i am wasting it,my exams r just 22 days by now, i dnt want to get fail, i knw the course and i know my capabilities, i know i can clear it if i study atleast 8 hours a day,which generally require lot more , but i know i can do that,i'm like \" ass on fire and go higher\"...means i will be tensed a day before exam, and will study before a day, that will not be sufficient to pass.what should i do?????kindly help me out with this Doctor: HiYou don't have to do anything big to fight the problems you are facing. Small small things here and there will do the jobMake small slots of study with short breaks in between.Make Facebook and other things as a gift for studying well (contingency management), like if you have studied for a certain period you can use it for few minutes, otherwise don't allow yourself to use it. Here you have to be strict with yourself. Some amount of discipline is a must and there is no other way.Try to find study partners. Keep discussing with them whatever you have studied. That will keep you going and you'll be motivated to study more.Be innovative. You can change the place where you studyTake some break in between and do the things you want yo do.. Like an evening with friends.Regards,Dr Atun"
},
{
"id": 139807,
"tgt": "What causes shoulder blade/chest/arm pain in a scoliosis patient?",
"src": "Patient: I am 20 years old i have scoliosis and also had a bulged disk at the bottom of my back, i suffer from quite a bit of pain but recently i have had a pain in my right shoulder blade when i breathe in, i am also getting pains in my chest when i breathe, my right arm is painful to us and my neck is very stiff and sore on that side.....what have i done? Doctor: Hello, Since you suffer from scoliosis, from time to time the muscles that support your spine may be fatigued. Physical therapy and stretching exercises may help with your symptoms. See a Spine Specialist for your issues. Hope I have answered your query. Let me know if I can assist you further. Take care Regards, Dr. Erion Spaho, Neurologist, Surgical"
},
{
"id": 8201,
"tgt": "I HAVE ACNE PROBLEM, HOW TO GET RID OF IT ?",
"src": "Patient: GOOD AFTERNOON SIR, MY AGE IS 18 AND FROM 3 TO 4 YEARS THERE IS NO IMPROVEMENT OF PIMPLES AND SPOTS ON MY FACE AFTER TAKING A LOT OF MEDICINES.HOW CAN I GET PERMANENT RID OF THEM,BY WHICH METHOD AND IT;S ESTIMATED COST Doctor: u need to send a pic of ur lesions on dr.resham@gmail.com for a proper diagnosis and treatment and details of what treatment u have taken in the past and ur cosmetic use."
},
{
"id": 104814,
"tgt": "Swelling and redness on left side of face above lips, spreading to eyes. Allergic reaction?",
"src": "Patient: hi.. my daughter woke up yesterday with a small amount of swelling and redness on the left side of her face above her lip. today,the swelling has spread so that from under her eye to above her lip is all swollen and red. the swollen area is harder to the touch than the rest of the skin on her face. could this possibly be some kind of allergic reaction or an infection from her teeth or gums...? Doctor: Hi welcome to HCM. Your descriptions are more suggestive of an acute allergic reaction. Since it is progressing and because you have not mentioned the age of your daughter, give her cold compresses and take her to the nearest emergency consultation at the earliest, for further evaluation and management. Take care :)."
},
{
"id": 197652,
"tgt": "What could cause severe abdominal, groin and back itch after Hernia surgery?",
"src": "Patient: My husband had a ventral hernia surgery with scar tissue removal 3.5 weeks ago. He is feeling fine but has excruciating itch still all over his abdomen, groin and lower back. Benedryl works but not good for the daytime b/c it makes him sleepy. He finished a prednisone pack 1.5 weeks ago. Doctor: Hi,Dear,Thanks for your query to HCM.Dear I read facts of your query and reviewed it in context to your health issues submitted in your query.I understood your health concerns and feel Concerned about them.Based on the facts of your query the reply of your query is-Your current health issues seem to be due to -Adhesive dressing allergy if its still there or the after effect of allergy from adhesive dressings.Excruciating itch over abdomen / groin / lower back also could be aggravation of the jock's itch / tinea cruris fungal infection/ or bacterial subacute infection and needs to be ruled out.This is therefore more alarming condition and situation needs to be dealt by urgent Consultation with attending Surgeon and dermatologist doctor .Therefore Cause of these complaints -like above needs to be ruled out.Other causes-like discussed above , need to be verified.For this you need to consult attending Surgeon and Dermatologist.Thus You should consult attending Surgeon and Dermatologist.This would resolve the complaints of your's.Constant follow-up with local family or specialistattending Surgeon and Dermatologist for long term would give you long remissions of such complaints in time to come,which would depend on meticulous follow-up treatment of your health concerns.Just don't worry and be patient and co-operate with your doctors,till you verify with your attending doctors.For early recovery-please follow above do's which would resolve your complex health issues.Hope this would help you to treat your health issues in the best way possible. Welcome for any further query in this regard to following link-http://doctor.healthcaremagic.com/Funnel?page=askDoctorDirectly&docId=70229Wishing Good Healthy Life!!Dr.Savaskar M.N.Senior Surgical SpecialistM.S.Genl-CVTS"
},
{
"id": 197001,
"tgt": "Is it safe to masturbate?",
"src": "Patient: Hi doctor I m 18yr old.i saw some bad movies.when I see a girls see so sexy.then I feel very badly.sorry to say tis when I m mood I try to pump my penis.then the sperm taken out. So plz give any reason to stop it. I feel at sexlife after I m married life. Doctor: masturbation is ethically wrong, it makes u weak and drain the protein content of ur body.. protein is the building block of ur body so eventually u will be lean.. it creates nervousness and neurological problems.. it is the main cause of erectile dysfunction.. it addicts u, the more u do it the more it is hard to control.. it creates drowsiness, u will be tired and need to sleep after masturbating.. it causes stress andvstrain in your mind and soul..it creates depression and makes own feel bad on his own.. it also leads to a noxious problem that u cannot see any girl without having a single though:sex.. it is the main reason for speedy release of sperm during intercourse so it will create dissatisfaction to you and to your wife.. it reduces sperm count.. it leads to loss of memory and loss of interestvof real sex.."
},
{
"id": 139186,
"tgt": "Suggest treatment for severe hip pain",
"src": "Patient: I have had extreme pain in both hips for several months now. The onset of this pain was sort of sudden. Now I am having some pains shooting in my shoulder joints and in my knees. i can tell that this is progressing rather quickly. My doctor is treating me for bursitis. I was on naproxin for a couple of weeks. It helped a little. I had to stop taking it due to side effects. The pain was back immediately. My question is \"Do you think that I might need to seek out a specialist?\" \"Is this something attacking my skeletal system?\" i am a white 69 year old female. My fear is winding up in a wheelchair. Doctor: Dear Madam, I value your concern, and the symptoms you describe with involvement of both hips knees shoulders, cannot be just bursitis, My main concern in your case would be to rule out inflammatory arthritis, and that can be done by few blood tests, for that you will have to consult an Rheumatologist. Do not worry, as your pains could be functional, but to be sure we need investigations, hope you value my concern, and get investigated.All the best, let god heal you."
},
{
"id": 70213,
"tgt": "What is the treatment for a lump on the thigh?",
"src": "Patient: i have big red lump on my thigh which is paining immensly would like know what condition is this..it started two weeks back..a small lump was formed..it is hard..not soft..i had scratched it accidently..but there was no pus ..now it has become bigger but still not signs of pus..just a large hard red lump.. Doctor: Hi ! Good morning. I am Dr Shareef answering your query.From the history and your self examination finding of the hard consistency of the swelling, I would suggest you to get yourself examined by your family physician/general surgeon for a possibly biopsy of the lesion for a histo-pathological examination. Further management would depend on the investigation reports. Till that you could go for an anti inflammatory drug for the pain in the lesion.I hope this information would help you in discussing with your family physician/treating doctor in further management of your problem. Please do not hesitate to ask in case of any further doubts.Thanks for choosing health care magic to clear doubts on your health problems. Wishing you an early recovery. Dr Shareef."
},
{
"id": 12705,
"tgt": "What could a rash near buttocks with pain and foul odor indicate?",
"src": "Patient: Hello i have this rash on the top of by bottom crack it is crusty and very painfull the inside crack is all red and goes to my growing area. i have tried to make a appointment with my dermatologist he is on vacation for a month. this rash has a bad smell to it. any idea what i can do about this. ? Doctor: Hello,The symptoms seem to be related to intertrigo. Intertrigo is a\u00a0rash\u00a0that forms in\u00a0the\u00a0folds of\u00a0the\u00a0skin. When skin rubs against skin, it\u00a0causes\u00a0 creates a warm, moist environment that is ideal for fungal and bacterial growth. I suggest using an anti-fungal cream for local application. I also suggest to mamin a good hygiene.Hope I have answered your question. Let me know if I can assist you further. Regards, Dr. Dorina Gurabardhi, General & Family Physician"
},
{
"id": 149469,
"tgt": "Occasional numbness in legs, white spots discovered in spine. Is it due to lumber puncture?",
"src": "Patient: My daughter has experienced,on occasions, numbness in her legs,this has all started in the last 6 months.. As a result she had an MRI scan . Her brain was clear ,however, on her 5th & 6th platelets in her spine they have discovered white spots. The consultant has asked for a lumber puncture and a further MRI scan in 3 months. Obviously she is very concerned. Is this something that she should consider trying to have done urgently - possibly explore having the work done privately. We certainly do not want to find that her condition irretrievably worsens due to the delay. because of the delay. Doctor: HiThank you for your question.Numbness in the legs may be caused by external compression on the nerve or internal pathology of the nerve itself. It is not clear from your description what the white spots are. If you can upload report/images of the MRI that would be helpful. Electromyography and Nerve Conduction Velocity (EMG/NCV) test can find out the exact location of nerve problem. Management as well as need to repeat the MRI in 3 months depends on what the problem is. Please consult a neurologist for clinical as well as MRI evaluation.Best wishes."
},
{
"id": 34,
"tgt": "Suggest remedy for getting pregnant",
"src": "Patient: hi my name is anvita age 25 weight 90 kgs. i had a fistula surgery in july 2010.doctor had asked me not to conceive for 6 months.we have beeen trying since october.i had a regular 28 days cycle.but from october its varying from 29-34 days.due to which i did a transvaginal scan and everything is normal.my gynaec has asked me 2 be on letrozole frm 3rd to 7th day of my periods.i want to know any other thing that i should do. Doctor: do serum tsh and serum prolactin den do hsg on day 8th den do follicular study after taking letroze den till follicle size increase to 18 mm den rupture den iui fr early results. .."
},
{
"id": 206193,
"tgt": "Suggest treatment for depression",
"src": "Patient: I have a male family member who is no longer a minor and so his mental health diagnosis is not available to the family. He is probably schizoaffective (hearing voices and what looks clearly like depression) + anger. He has been self cutting but recently found out had been cutting girlfriends also. Was on meds (don t know which) and has gone off. How dangerous is he (to himself and others)? Is there any recourse for hospitalization from the family if there is a psychotic episode later? The girls were convinced not to go to police, but I m wondering if it is better to have a criminal record showing violence for the future? Doctor: DearWe understand your concernsI went through your details. I suggest you not to worry much. Schizo active personality or typical schizophrenia can be cured if detected early and treated properly. For your family member, proper diagnosis and treatment is necessary. For diagnosis, he needs to be taken to a psychiatrist. People with Schizo active personality or typical schizophrenia may not necessarily have criminal ideation in the future.Psychotherapy techniques should suit your requirement. If you require more of my help in this aspect, Please post a direct question to me in this URL. http://goo.gl/aYW2pR. Make sure that you include every minute details possible. I shall prescribe the needed psychotherapy techniques.Hope this answers your query. Available for further clarifications.Good luck."
},
{
"id": 119835,
"tgt": "Suggest remedy for cramps in navel",
"src": "Patient: Sharp cramps in navel I was told by the E.R. Recently that I had an infection in my navel. I dont think that I was accurately diagnosed. Ive been getting sharp pains shooting through my navel off and on for years. The navel area swells and turns black when it is inflamed. Then when it heals , it gets itchy and the blackness slowly goes away. This happens to me every month and lasts for approx 1-2 Weeks. I think I have an umbillical hernia. And if this is true, can having a recurring umbilical hernia affect fertility? Doctor: Hello,I read carefully your query and understand your concern. The umbilical hernia should be confirmed through an abdominal ultrasound. The umbilical hernia does not affect fertility. Hope my answer was helpful.If you have further queries feel free to contact me again.Kind regards! Dr.Dorina Gurabardhi General &Family Physician"
},
{
"id": 198707,
"tgt": "What could cause prominent penile vein post erection?",
"src": "Patient: Hi ! I have a vein on my penis which starts from the shaft from right and goes across from the middle to the left side of my penis.Before i didn't had this vein it is very prominent during erection.Is there anyway to get rid of this vein ,it's very much bothering me.What could be the cause of appearance of this vein ?? Doctor: DearWe understand your concernsI went through your details. Some men have small veins related to the skin and some have large ones. Their prominence varies from individual to individual and will change with an erection. Your description sounds to be that of a normal penis. On a normal penis veins as you describe during erection are normal. Please don't worry and don't be anxious.If you require more of my help in this aspect, please use this URL. http://goo.gl/aYW2pR. Make sure that you include every minute details possible. Hope this answers your query. Available for further clarifications.Good luck. Take care."
},
{
"id": 28308,
"tgt": "How dangerous is it for your heart rate to go high?",
"src": "Patient: My daughter has Teckacardia (Think that s how its spelt). A week ago she started with a bad tooth (Being pulled tomorrow) and at the same time started on Lexapro . She HATES medication of any kind! Within in 24hrs of all this she was rushed to ER. She awoke at 3am with a heart rate at 190! This has since happened 3x s in the last few days. She stopped the Lexapro right away. Doctor offered her Perc s but she won t take pain meds. Is it possible the heart rate is being caused by all the pain? How dangerous is it for your heart rate to go that high? Doctor: HI, thanks for using healthcare magicTachycardia is an increase in heart rate.Pain can cause an increase heart rate but it would not go as high as 190 beats/minute.It is dangerous for the heart rate to go this high because it stresses the heart and can also affect the ability of the heart to supply blood to the organs.I hope this helps"
},
{
"id": 132006,
"tgt": "Suggest treatment to cure knee injury",
"src": "Patient: Hello, I have a knee injury for the past 3 years now and never got professional help. I got it playing soccer. To this day I am in pain constantly, I can t stand for long, run or do anything at all because immediately after I get more pain. I had an X ray and other doctors said my bone is okay, Its my right knee and the pain is on the inside. Doctor: The first step is to determine the cause of pain. Since we have excluded bone injury by X ray, the next step should be assessment of the cartilage and ligaments. The best diagnostic tool is knee MRI.Once the injury is localized, you can discuss the possible treatment plans with your orthopedics doctor. Thank you for choosing HCM.Regards."
},
{
"id": 180302,
"tgt": "Suggest treatment for nose bleeding when sneezed in kids",
"src": "Patient: My grand daughter is 5 and when she sneezed really hard this morning, her nose started bleeding badly. She felt she needed to blow her nose and when she did a dime-sized thing came out with the blood. The bleeding has stopped now but I am very concerned about what is was. Doctor: Hello, Bleeding from nose is common in children so please do not worry. Your granddaughter must have injured her nose while picking at her nose leading to a clot formation. When she sneezed the clot could have dislodged leading to nose bleeding. This is the commonest cause of nose bleeding. Consult an ENT specialist if she has frequent and recurrent episodes of nose bleeding. Hope that helps.Regards,Preeti"
},
{
"id": 121662,
"tgt": "Why is my wrist sore?",
"src": "Patient: A few weeks ago I got spiked I got rushed in hospital and was put in a drip it s bin 2wks now and my wrist became very sore I had a lump it s now moved further down put it very sore and burns and now it s on the crease of my wrist so it s restricting e bending my hand Doctor: Hello, The sore wrist can be due to tendonitis or if the IV line was put in the area then it can be due to thrombophlebitis. Hope I have answered your query. Let me know if I can assist you further. Take care Regards, Dr Praveen Tayal, Orthopaedic Surgeon"
},
{
"id": 42829,
"tgt": "What should be follicle size on 9th day of follicular study cycle?",
"src": "Patient: Hi sir, I am 26 years old and suffering from PCODs. At present i am on medication: metformin 500mg twice a day and folic acid. I am going through follicular study. On 9th day of my cycle, following are the ovaries measurents:Right ovary: 3.23*2.58*1.71cm Volume 7.45ccLeft ovary: 3.08*3.20*1.89 cm Volume 9.77ccRight ovary shows 10-12 follicles 2-7 mm in sizeLeft ovary shows 12-14 follicles 2-6 mm in size.Please let me know what should be the follicle size on 9th day of cycle? Doctor: HiThanks for writing to HCM .There is no rule that the day of cycle should have that sized follicle. But on 9th day of cycle one ovary or both should have a dominant follicle of more then 10mm which will increase in size at rate of 1-2mm/day till it reaches 20-22mm and rupture. I suggest you to continue your follicular study on day 11 and also 13th and further to see the progress of the follicle does it rupture or regress.Some may have delayed ovulation like on 16th or 17th day . After your course of metformin you can take ovulation inducing drugs to increase chance of conceiving. Hope I have been helpful .RegardsDr.Deepika Patil"
},
{
"id": 98449,
"tgt": "What causes an itchy rash while on Amox-Clav and virtussin syrup?",
"src": "Patient: HELLO DOCTOR. I AM PRESENTLY TAKING TWO MEDICATIONS. AMOX-CLAV 875MG EVERY 12 HOURS, AND VIRTUSSION AC SYRUP 10MG WITH CODINE EVERY 4 HOURS. I HAVE BEEN ON THIS REGIMEN SINCE 7/15. I HAVE BEGUN TO NOTICE A RASH APPEARING AND ITCHING. WHAT DO YOU SUGEST. DAVID NOCKELS, ST. LOUIS, MO. E-MAIL ADDRESS IS YYYY@YYYY Doctor: HelloIt might be an allergy reaction due to the treatment.Discuss with your treating doctor.RegardsDr.Jolanda"
},
{
"id": 72978,
"tgt": "Suggest medication for chest medication, cough and laryngitis",
"src": "Patient: Sometimes during the fall around the time school begins, I get congested and then get laryngitis. After that I begin getting congestion in my chest and cough up yellow to green mucus a little. I develop a small cough but none of this is terrible. I do not run a temperature. What would you suggest I take for these symptoms. Doctor: Hello dear , hiWelcome to Healthcaremagic.comI have evaluated your query thoroughly .* Guidelines for your current issue- Maintain hydration with plenty of liquids .- Inhale vapors of boiled water added tincture benzoin with blanket covered over the head for 5 minutes each 2 times a day .- Avoid exposure to cold .- Deep breathing exercises , YOGA will help tremendously .- Avoid smoking if using .- Tessalon pearls for symptomatic relief .- Gargles with salted lukewarm water added peppermint oil 3 times a day .Wishing you fine recovery .Regards ."
},
{
"id": 86421,
"tgt": "Suggest treatment for sharp lower abdominal pain",
"src": "Patient: Every few months I have a sharp pain in my LLQ of my abdomen. It comes on suddenly and at full force, then after about 30 seconds, fades almost as quickly. The pain is so bad that i cant breath, scream, cry, or move. The pain is sharp, sudden, twingy and fluttery. Sometimes it happens when i'm changing how i'm sitting, it has also happend when wiping myself after using the restroom Doctor: Hi! Good morning. I am Dr Shareef answering your query. If I were your doctor, after a clinical assessment of your abdomen, I would advise you with an ultrasound of abdomen in addition to the routine tests like a complete blood count, blood sugar, urine routine and microscopic examination, and a stool test. Till the reports arrive, I would advise you with an anti spasmodic and a proton pump inhibitor drug for a symptomatic relief. Further management would depend on the reports of the investigations.I hope this information would help you in discussing with your family physician/treating doctor in further management of your problem. Please do not hesitate to ask in case of any further doubts.Thanks for choosing health care magic to clear doubts on your health problems. I wish you an early recovery. Dr Shareef."
},
{
"id": 2219,
"tgt": "Is it appropriate to test the pregnancy now?",
"src": "Patient: Hello, I am a married woman & we are planning for baby. I got my last periods on 2nd November 2012. And we had committed for a baby on 01st December 2012. Still I have not get my periods. So shall I do pregnancy test now? Or else I should wait for some more days? Doctor: Hi,Welcome to Healthcare Magic. I am Dr Ramadevi Wani. I will be answering your concerns now.Do you have regular periods every 28-30 days? If so, then you can do the pregnancy test now. Generally speaking, you can do the pregnancy test on the day of your scheduled periods. Sometimes the urine pregnancy test may not pick up the pregnancy. In that case repeat the test after 4-5 days if you have still not started the periods by then.I hope this is helpful.Best wishes,Dr Rama"
},
{
"id": 104827,
"tgt": "Asthmatic, have bad allergic reactions, symptoms recur post treatment, sputum mucopurulent. What could be wrong?",
"src": "Patient: i have had late diagnosed asthma 30 yrs old - recently i have been having very bad allergic reactions - couching consistaly , runny nose , producing sticky clear sputum . raised eosinophils 2.5 x 10*9L going on for a year. 3 years ago i had polyps removed from my nose, had very bad sinusitis and produced green nasal discharge. I use flixonase. i have been treated for a fungal lung infection twice. symptoms re-occur after about 10 days when treatment complete.i am constantly wheezing, unable to walk up one flight of stairs 14 steps without having to rest and use ventolin inhaler. My peak flow has always been good - swam competitavly until 20yrs and15 and 800 meter runner, was 500ml now 300 and one episode where 150. o2 stauration 95% sometime slightly lower. have recently needed to use inhalers x 4 times 2 puffs ventolin and symbacort. i am worrried that somthing else may be wrong. My sputum was mucopurulent when tested.thankyou for any insight i will be seing my dr in 2 weeks y Doctor: Hello, Thank you for writing to us at Healthcare Magic. The rased eosinopihil count, constant rhinitis (runny nose), mucopurulent discharge does suggest an allergic rhinosinusitis. A fungal rhinosinusitis needs to be excluded. I am certain you had allergy testing of some sort, either specific IgE blood test or skin prick tests. If you have not had any allergy testing, it is important that the triggers are identified so avoidance, optimal medical treatment and or immunotherapy will help control the allergy. It is also important that immune deficiency (antibody deficiency) is excluded with serum immunoglobulins, functional antibody levels. A sweat test may be indicated depending on chest findings (i.e., if there is lung scarring or bronchiectasis) then yes, the test is indicated. Your asthma needs to be optimally treated with combination inhalers. Please consult the online Asthma Control test (ACT) to see your current status. Best Wishes."
},
{
"id": 60548,
"tgt": "What is glutamyltransferase ?",
"src": "Patient: Hi I had some lab work done. My glutamyltransferase came back high. What is glutamyltransferase. What problems can this cause? Doctor: Welcome to Healthcare Magic Good Day GGT or Gamma Glutamyl Transferrase is an enzyme present insidde the liver cell and is usually released following damage to the cells. It is known to rise in a variety of conditions, the commonest one being after a bout of alcohol/chronic alcohol intake. You need to quit drinking if you are, and quit smoking as well as these two habits are very dangerous. Have you done your other Liver enzyme tests. You will need to consult with your Doctor regarding various other possible causes if you haven't drunk alcohol."
},
{
"id": 75960,
"tgt": "What causes breathing difficulty ?",
"src": "Patient: I M 3I YRS OLD MAN,WORKING IN A PERFUME INDUSTRY,YERTERDAY I HAVE DONE MY BLOOD TEST,BECOZ I HAVE SOME HEAVYNESS&BREATHING DIFFICULTY,ECG ALSO DONE,ITS NORMAL.I M VRY MUCH WORRIED ABT MY SGPT LEVEL,ITS 98U/L,COLESTROL;192MG/DL,TRIGLYCERIDES88MG/DL,HDL;52MG/DL,LDL-122,CREATININE-0.8.I M NOT USING SMOKING, OR ALCHOHOL.BUT REGULARLY USING COLAS. Doctor: Thanks for your question on Healthcare Magic. I can understand your concern. Nothing to worry for your blood reports and slightly raised SGPT. In my opinion, your breathing problem is mostly due to exposure to perfumes at your work place. Perfume when inhaled can cause irritation and inflammation of the airways. Since you are working there, you are exposed to perfumes heavily. So chances of perfume induced bronchitis is more in your case. So better to consult pulmonologist and get done clinical examination of respiratory system and PFT (pulmonary function test). PFT will not only diagnose bronchitis but it will also tell you about severity of the disease. Treatment of bronchitis is based on severity only. You will mostly improve with inhaled bronchodilator and inhaled corticosteroids (ICS).. Wear face mask during your work to prevent perfume inhalation. Hope I have solved your query. I will be happy to help you further. Wish you good health. Thanks."
},
{
"id": 154705,
"tgt": "What care should be given to cancer patient after chemotherapy?",
"src": "Patient: My relative has developed blood clots in his shoulders. He is still recouperating from radiation and chemo for cancer (now in remission) done last winter. He is also t6aking Lovonox shots daily. What should we be doing or looking for regarding home care? Doctor: Hello dear, thanks for your question on HCM. I can understand your situation and problem. Patients on chemotherapy are immunocompromised. So they are prone for opportunistic infections. So follow these home care to avoid infections. 1. Drink only boiled water.2. Ask patient to always wear mask.3. Eat only freshly prepared, home made food.4. Avoid junk food.5. Avoid too many relatives in home. 6. Start multivitamin, anti oxidant tablets for strengthening of immunity. Also start protein powder."
},
{
"id": 137205,
"tgt": "Suggest treatment for severe knee pain",
"src": "Patient: I need some advice... I seem to have hurt my leg and I can t work out what i ve done or how and it s starting to worry me. On Friday night I brushed my right leg against my left, not hard and had a sharp pain on the inside of my left knee... I touched it through my jeans and it felt like a knuckle sized lump had popped up and it was insanely painful. It felt like I had maybe been bitten. I went to the bathroom to check it out and could see it wasnt a bite, it wasn t red like a bit or sting and there was no entry point, just a very angry protruding vein... I hobbled about for about another hour before I got home and took off my jeans and a little purple bruise had started to form but the lump had dissapeared... The next day a large purple bruise developed and is now the size of half a fist i guess? it doesnt hurt more than a bruise do you think ive done something like a vein has ruptured or something? Do I need to go to the doctors... Doctor: Hi there. It is possible that it is bleeding under the skin. It should resolve in 7 days. If you feel the patch of bleeding is getting larger, you may have a bleeding disorder and need to consult your Doctor. Just ice theke and keep it elevated. Take pain killers."
},
{
"id": 214263,
"tgt": "Recommend home remedy for fibroids on uterus wall",
"src": "Patient: whats homeo remedy for fibroids in anterior and posterior wall of uterus causing contours deformity the largest fibroid measure 11.5*8.6*11.3 cm. edometrial canal is displaced anteriorly by the fibroids and measures 8.8mm. enlargeda A/V uterus measuring 14.8 *9.5*14.2 cm(volume:1050ml) cervix and vagina are clear.right ovary 4.60*2.03*1.93cm(volume:5.24ml)left ovary 4.90*1.84*2.37 cm(volume:11.20ml)both the ovary show normal parenchymal echotexture.no adnexel mass/cyst or evidence of fluid in the cul-de-sac.name:Mrs rrrr age:32 Doctor: Fibroids, also called leiomyomas, are benign muscular overgrowths in the wall of uterus. It has already grown to a certain size when it has been detected. The only option is to go for surgical removal. There are no effective home remedies or homeopathic medicines that would be helpful in this regard. If you do not get them removed, you might suffer from menstrual irregularities. If that is not your problem, you may wait since fibroids only rarely turn cancerous (malignant)."
},
{
"id": 109386,
"tgt": "Suggest treatment for severe back pain after Laproscopic Cholecystectomy",
"src": "Patient: I am a chronic pancreatitis patient . recently i have been diagnosed in Vadamalayan Hospital, Madurai. The Final Diagnosis is: Porcelain gall Bladder with GBDS St ERCP cleared of stones. Stented with Chronic Alcoholic Pancreatitis. Laproscopic Cholecystectomy and ERCP done. I am having severe back pain. Kindly prescribe suitable medication for me. My email address is YYYY@YYYY - B.karunakara chandrasekaran. I am 54 years old. Doctor: HiIf i would be your treating doctor I would advise you to take tablet Acefor p or tablet dan p 1 tablet thrice a day to reduce pain.This medicine is not given without prescription so consult to any local Doctor."
},
{
"id": 146436,
"tgt": "What causes fainting frequently?",
"src": "Patient: I passed out 2 times this morning - first time I found myself on the floor from the toilet, no warning. I took a shower with assistane so I wouldn t fall, and sat down in a chair at our table. I passed out again with my head landing on the table. Do I need immediate med check to check for medical condition that may have led to this? Doctor: Hi, I had gone through your question and understand your concerns. Passing out or faint has two major causes: 1. Syncope caused by hypotension or low brain blood perfusion.2. Seizures or abnormal electrical brain discharge. In both cases I suggest you to consult your primary care Doctor and to discus examinations needed to diagnose the cause. Tgese include regular periodic measures of your blood pressure, EEG ( electroencephalogram) to check out seizure activity and blood glucose lab test to check for hypoglycemia ( low blood sugar). Treatment depends on above mentioned examinations results. Hope this answers your question. If you have additional questions or follow up questions then please do not hesitate in writing to us. I will be happy to answer your questions."
},
{
"id": 135644,
"tgt": "How long does broken humerus bone take time to heal?",
"src": "Patient: hi , metal bar fell on my upper arm, broke my humerus bone, I some scratches on my skin under arm, my doctor said there was a blow inside, so my veins, nerves, muscles, soft tissue was damaged. he claims my skin died as it is black right now, and I need skin graft. I believe skin can heal, what do you think? Doctor: Pitch black dark skin patch with no sensations is definitely a dead patch of skin.So it needs to go.the resultant raw area after taking out tge dead patch needs to be covered for sure.whethe a skin graft or something else your treating doctor would be able to advice depending upon the bone and other soft tissues extent of exposure."
},
{
"id": 97718,
"tgt": "Does shifting from alopathic to Ayurveda for BP medication cause muscle tightness, neck pain ?",
"src": "Patient: Dear sir, Am a anxiety pateint with high BP, But bp was never in my life from last to lat year i got this bp attack and started medicin for only 6 months and moved to ayurved..doc said stopped all medicines and start ayurveda...then from july 13 to till jan 14..i was not on any bp medication...from but suffered from muscles tightnss on back of neck, tightness in head...sometimes palpitation...few days back bangalore doc asked me take amlong half tablet. but it didint work as the bp was 180/110...sometimes it was 200/110 yesterday banglr hospital doc. gives amlong 5mg 1 tablet along with injection and bp comes down and one physican said..its all coz. of anxiety.he has given me tazloc-ct 40, thiospas, petrilmd 0.5previous 2 yrs back i was on deanxit 10mg...coz. delhi doc said its Fibromyalgianow am only on tazloc-ct 40, thiospas, petrilmd 0.5 ...bp is not the cause its happens coz. of anxiety.....what to do..i fed up by all these things Doctor: Hello & Welcome to HCM,I had gone through the case and found that these symptoms might be due to some other reason also like Cervical spondilitis.Headache also associated with Hypertension, sleeplessness and some time with anxiety.You do not worry, first go for proper investigation like -Regular monitoring of blood pressure both at morning and during headache and anxiety.- X-ray of cervical vertebrae to rule out cervical spondilitis.Whatever the cause , take medicine if having Blood pressure otherwise do some kind of particular exercises like1- Deep breathing alternate nostril breathing for 15 minute daily, It will reduce your anxiety and blood pressure both.2- Meditation for 20 minute daily,, it will reduce anxiety and improve the sleep.3- Do some cervical exercise and avoid pillow during sleep.All these help you definitely.Hope my advise will be effective for you.Thanks,Dr.Soni VermaHomoeopathic Physician ,fitness trainer"
},
{
"id": 102676,
"tgt": "Is it normal for asthma patients to have high red blood cell count?",
"src": "Patient: I am 53 years old and have had asthma my entire life. I was explained yesterday that I have a high amount of red blood cells and they asked me if I spend a lot of time in the mountains. Is it normal for asthmatics to have high red blood cell count because of the breathing ? Doctor: Hello dear,High red blood cell count in asthmatics can be explained by the following reason:1. In asthmatics, there is obstruction to the smaller airways of the respiratory tract.2. Due to this, there decreased availability of oxygen to the tissues.3. Depletion of Oxygen in quantity or molecular concentration results in a state of Hypoxia.4. This hypoxia is sensed by few specialized cells in the Kidney & they produce a hormone called Erythropoietin.5. Erythropoietin in turn acts on the bone marrow & stimulates the production of Red Blood Cells.Thus, asthmatics tend to have a high red blood cell count.Hope I have been able to answer your query.Wishing you a good health.Take care."
},
{
"id": 173638,
"tgt": "Can a child have diaper rash all over the body?",
"src": "Patient: can a child have diaper rash all over his body - i am a caregiver and a child i take care of has a severe diaper rash, a rash on his cheeks, top of his hands and feet (not on the soles) and the mother is telling me that her doctor says it is diaper rash which i don t believe....could this happen? Doctor: generally diaper rash occur at diaper area. most common cause of generalized rash is allergic disease of skin ."
},
{
"id": 93691,
"tgt": "Stomach bug, stomach flu and gas. What is the reason?",
"src": "Patient: My husband and I both had some sort of stomach bug (It probably is not Norwalk, as the one person who I came into contact with at work tested negative for it...I am a nurse). But, my question is about the gas related to the stomach flu. I seem to be feeling much better today (after about 36 hours of feeling horrible), but the amount of air in my intestines is crazy. It's the only real thing that has got me still feeling a bit \"off.\" Is there anything I can do to reduce the gassy feeling after a stomach virus, or do I just need to let it run it's course? Doctor: Hi ! As both you and your husband have got gastro intestinal problems together, it seems that infection you got has a common source, most likely some food from outside which you both might have consumed. Possibly you have not taken any medicinal treatment for the same nor you have consulted any doctor till now. Once the infection persists in the stomach, it may give rise to all the symptoms you have mentioned about. I will advise you go for a stool test and if necessary , a culture sensitivity of your stool, consult your family physician, and take appropriate antibiotic covering both aerobic and anaerobic organisms. Amebiasis has to be specially thought of. You can also add some kind of probiotic with anti flatulant drug, and a proton pump inhibitor. Hope this will solve your problem. Wishing you an early recovery."
},
{
"id": 93576,
"tgt": "Have stomach pain and diarrhea. And the pain is not continuous also. What can be done?",
"src": "Patient: For the past 3 days I have been suffering from short stomach pains that last between 3 to 5 minutes at 15 minute intervals. This results in me having diarrhoea during each pain. What is Happening to me? I don't think it is food poisoning as I have had that many times before and that turns out to be one long stomach ache but for up to 24 hrs max. Can you help? Doctor: Hello dear,The symptoms as mentioned in your post can be due to Gastroenteritis.Management includes:1. Maintain adequate hydration by intake of fluids & ORS.2. Taking soft, plain, non spicy diet.3. Anti spasmodic for symptomatic relief.4. If bacterial infection is suspected, then antibiotics need to be taken under guidance of a Doctor.Since it has already been 3 days since you are having the symptoms, kindly consult your General Physician & start treatment accordingly.Wishing you a speedy recovery.Thanks & take care."
},
{
"id": 225873,
"tgt": "Wife had Copper-T as suggested by doctor. Has irregular periods. Is this normal?",
"src": "Patient: my wife's age is 29yrs, Doctor suggested to adopt Copper - T, we accepted and he adopted on 28-05-13, but after that is adopted, my wife is facing many problems. like periods not happening at the right time and from 28-05-13 bleeding is stay continued. Please help me doctor. Please tell me after adopting coper t can bleeding was stay continued. Doctor: Hi, Thanks for writing to us. You should consult a gynaecologist for your proper examination and see the proper position of copper-T. These all symptoms may be because of copper-T because of hormonal changes but if she is unable to tolerate you should consult a gynaecologist for removal of copper-T. Good luck. Take care."
},
{
"id": 136961,
"tgt": "Suggest treatments for pain in hip after a hip replacement",
"src": "Patient: I have had a hip replacement, Wright Mfg is the maker of this implant. I have been told the implant is too large for my thigh bone and thus I have constant pain in my thigh and also the hip,. I have to exercise twice a day but still suffer from pain and stiffness. Doctor: Hello, I have studied your case. You may need to consider revision hip replacement if pain is unbearable.Anterior hip replacement surgery gives good results.Rather than choose good quality of implant material which will fit perfectly.Physiotherapist will help you better to learn more exercises in proper protocol.Diet rich in calcium and protein will help for recovery.Hope this answers your query. If you have additional questions or follow up queries then please do not hesitate in writing to us. I will be happy to answer your queries. Wishing you good health.Take care."
},
{
"id": 51473,
"tgt": "Had kidney transplant, weight loss, shortness of breath, lower back pain. Cause?",
"src": "Patient: Hi my name is Jose Carancio and I suffer from kidney disease. I had my last kidney transplant in 2001. I currently have been having lots of problems with my health for example I have lost a lot of weight and have been experiencing shortness of breath. I also have been getting this sharp pain in my lower back and when my back doesn t hurt i get the same pain in the front groin-ish area. i have gone to the emergency room and they have told me that s its not an ulcer and its not related to my heart but they wont tell me whats wrong with me. I don t know if they know and don t want to tell me or they don t know for sure. if you have any insight what I may have I would be happy to get some insight even if the insight is bad. thank-you and i wish you could help me. Doctor: Hello. Thanks for writing to us. Pain in the lower back going towards groin may be related to a back problem. An MRI of the spine will help in detecting any possible pathology. Such pains usually subside on taking a combination of pain killer and a muscle relaxant. You can consult your orthopedician for an examination. I hope this information has been both informative and helpful for you. Regards, Dr. Praveen Tayal drtayal72@gmail.com"
},
{
"id": 158474,
"tgt": "Dry throat, throat block sensation. Abnormal Vitamin B12 level, vitamin D level. Stiff neck or cancer?",
"src": "Patient: Hello sir Can i get your help i am having a feeling of something stuck in my throat from last 5 months and my throat is dry as well have visited doctor quite many times but every yhing seems normal besides the vitamin B12 level and vitamin D level and have been taking them from quite some time. Please let me know what can be the reasons for the same. Also, please let me know if this is some thing serious and if these are the symptoms of throat cancer then wouldnt it had been worse in 4 months.. Also i have a stiff neck as well please reply ASAP Doctor: Hello, Thanks for the query to H.C.M. Forum. From last 5 months , you are feeling , that something is stuck in your throat. 2nd symptom is dryness in throat. You yourself answered that if it would have cancer then wouldn't it had been worse.Your answer is correct. It is not cancer. You haven't mentioned your age and gender as these two are very important. Two possibilities are there,1 Hypertension , if your blood pressure is very high particularly diastolic ( lower B P ) than there are chances that you feel something is stuck in throat. 2 diabetes, dryness of throat is very common but in diabetes Hypertension may be associated . 3 This may be due to hyper acidity chronic . Consult a physician and get in his opinion and blood examination as well. Hope for the best. Dr. HET"
},
{
"id": 1905,
"tgt": "Is spotting for most of the menstruation inhibiting conception?",
"src": "Patient: my periods had always been ontime or two days earlier all these years.now i am 36 years old.since last two years approximately,it is usually 4-6 days earlier,and it is just scanty black spottings on the first and second day,and the really red discharge starts on the 4th day only.it is really heavy just for that day.finishes with black or dark brown discharges again until 7th day.i'm trying to conceive this last two years and i'm finding it difficult to calculate my first day of period inorder to calculate my ovulation.i doubt if i seriously ovulate?all this worry me so much.i am already on prenatal vitamins and follicum.i had been prescribed duphaston and glucophage as well.nothing seems to work.kindly help.need your expert advice.wanting to become pregnent very badly.i'm already a mother of two girls,12 and 7 respectively.what could be wrong,or what should i do,please suggest. Doctor: Hi, I think there can be some hormonal problems. So get yourself evaluated first. It can also happen due to infections. Get a thyroid profile and prolactin levels and a ultrasound for your uterus and ovaries done. If everything is fine , you can try some medicines for ovulation and can track your ovulation. Be in contact with your husband every 2 to 3 days after your periods stop. You can try like that for 3 cycles."
},
{
"id": 100564,
"tgt": "Suggest treatment for runny nose, sore throat and chest congestion",
"src": "Patient: IHAD ALLERGY LAST WEDNESDAY FIRST I HAD RUNNING NOSE SOUR THROAT AND CHEST CONGESTION i HAD TAKEN LEVOFLOX 500, WIKORYL FOR FIVE DAYS AND BRO ZEDEX BUT STILL I FEEL CHEST UNCLEAR DO I HAVE ASTHAMA WHAT SHOULD I TAKE ? WHEN I BREATH FULLY I FEEL LIKE COUGHING Doctor: Hello.Thank you for asking at HCM.I went through your history and would like to make suggestions for you as follows:1. Were I treating you, I would suggest you montelukast, levocetirizine and salbutamol for 5-7 days.2. I would suggest you to have a rest and take plenty of fluids during the day which will help you to recover early.3. I would suggest you to complete the course of the antibiotic (Levoflox).4. If you are experiencing the chest symptoms for the first time, I would suggest you not to worry at present about asthma. But if you have repeated chest symptoms/wheezing/breathing difficulties, I would think of asthma.Hope above suggestions will be helpful to you.Should you have any further query, please feel free to ask at HCM.Wish you the best of the health and a great recovery.Thank you & Regards."
},
{
"id": 59960,
"tgt": "Removal of gall bladder, nicked bile duct, sepsis, stent in the duct, abdominal pain, high white blood cell count, vomitting blood. Suggestions?",
"src": "Patient: Hi, I had my gallbladder removed in 2003 and complications from the surgery, a nicked bileduct caused me to be very ill and I had to be treated for sepsis in the hospital for a few weeks and I had a stent put in the duct until it healed. For the past few months I have had fevers that come and go along with chills and upper right abdominal pain . The pain got so bad a few weeks ago I went to the ER, they say me right away because of elevated HR, fever and upper GI pain. They did an X-ray , ultrasound and blood tests which all came back okay except I had an elevated white blood cell count. The pain stopped suddenly, the fever and my HR went down and the docs could not figure out why though they said it may be bile backing up causing acute infections due to little stones still present in the ducts or scar tissue in the ducts. I saw my regular doctor and she did not order any tests barely looked at me and tried to send me home with pain meds. I told her I do not want the pain meds I want to knoe what is causing this pain. No explanation, no more tests. Today I got very ill again, I had a migraine and tried to take (2) 81 mg. aspirins and my stomach started hurting very bad. I could taste blood in my vomit and at first vomit was very dark brown then stayed dark orange. I could taste blood every time I vomited. I do not know what to do now? Doctor: i need to know is your cbd stent out . get an MRCP done then we can get to know about the status of bile ducts and then proceed hope this helps Dr.Virk"
},
{
"id": 224384,
"tgt": "UTI infection. Taking Ortho Tricycline, prescribed marcobid. Will marcobid make birth control less effective?",
"src": "Patient: Hello, I am taking OrthoTricycline birth control pills. I have been prescribed the antibiotic Marcobid for a UTI . Is this one of the antibiotics that makes birth control less effective? Do you need a back-up method of birth control while taking it? If so, how long after you finishing using Macrobid do you continue needing a back-up method? Doctor: Hi dear and thanks for your query.In the past, women using hormonal contraception such as the pill or patch would be advised to use an extra method of contraception (eg condoms) while taking an antibiotic like this one and for seven days after finishing the course. However, this advice has now changed. You no longer need to use an extra method of contraception with the pill, patch or vaginal ring while you take a course of antibiotics. This change in advice comes because to date there is no evidence to prove that antibiotics (other than rifampicin or rifabutin) affect these contraceptives.All the best"
},
{
"id": 69368,
"tgt": "What causes large lump on lower right of scrotum?",
"src": "Patient: I have a large lump(2\" in diameter) on the lower right of my scrotum. The lump moves with the skin and I have had it for about 3 weeks now. At first it was very hard but in the last few days I've noticed the top of the lump getting softer. I've been on an antibiotic for about 5 days now for an ear infection. Since it has started to soften it has become a little painful but not terrible. Any Ideas? Doctor: Hi,From history it seems that you might be having acute Epididimitis giving this problem.As you are on antibiotics, infection started resolving.Consult urologist and get examined.Ok and take care."
},
{
"id": 33840,
"tgt": "Suggest treatment for sinuses infection and overweight",
"src": "Patient: I am recovering from a pretty severe attack of sphincter of oddi dysfunction which began Thursday night. It is the first time this has happened to me since I had my gallbladder removed 34 years ago. I am 63, female, 5'5\",about 30 lbs overweight. I usually watch my fat intake, but I overdid it this past week and I am paying a painful price for it. My almost life-long allergies are acting up because of the humid weather in my locale. Can I take Claritin while I'm still recovering from an attack of SOD? My sinuses are really bothering me. I feel as if my body will be back to normal within another 24-48 hrs. re the SOD. I am living on a clear liquid, low-fat diet right now. I don't want to upset my progress. Doctor: Thanks for posting you query to health care magic.As I understand you are suffering from sinus inflammation as per your opinion it is allergic in nature .I would like to suggest you that once you should be examined by ENT surgeon to rule out cause of this inlammation as some times secondary infection sets in and produces sever probelem.Clatrin is a suppressive treatment option for allergic sinusitis but it will not cure your probelem .SOD do not affect with Clatrin so you can take this medicine .For your SOD you should also contact to Gastric surgeon he will treat you accordingly .Hope you would be satisfied with my answer . Feel free to communicate if any query .regards,Dr.Manish PurohitInfectious disease specialist"
},
{
"id": 123036,
"tgt": "What causes a feeling of ice water flowing in leg bone?",
"src": "Patient: Hello, What is the general cause of a sensation of ice water flowing in the leg bone. Generally, either leg bone but not both at the same time. My husband may have necrosis in his hip (one has already been replaced), a cyst on his outside of his right knee (probably a future knee replacement) and arthritis in his spine. He is only 50 years old. Just wondering if the ice water sensation is related to one of these conditions or could it be something else? Thank you. Doctor: Hi, Ice water sensation in leg bones is not related to any hip necrosis. It is generally due to neurological in nature - whether pressure or irritation in the nerves. I suggest check up by a neurologist. Hope I have answered your query. Let me know if I can assist you further. Thank you. Regards, Dr. Gopal Goel Orthopedic Surgeon"
},
{
"id": 102783,
"tgt": "What is the treatment for mild asthma and severe cough?",
"src": "Patient: My 18 yr old son has mild asthma but has a deep 'barking' cough and complains of not being able to clear his throat. He now has a rattle in his lower front of chest on left hand side when breathing - is there anything he could do tonight to make him more comfortable Doctor: Hi, From history it seems that your son might be having some lower respiratory tract infection precipitating asthma attach.Give him one antibiotic to cure infection.Give him broncho dilator medicine for ease breathing.Give him cough suppressant syrup for dry cough.Ok and take care."
},
{
"id": 188459,
"tgt": "Molar tooth got chipped twice due to metal implant and buffed out. Underlying cause?",
"src": "Patient: A new pfm bottom full bridge molar tooth got chipped due to the top metal implant just showing through the temporary top bridge, and was buffed out, then a couple weeks later that same bottom molar chipped again, can anyone be sure the structural integrity of that botttom molar is sound? Actually hasnt it now been compromised due to the repeated pecking of the top metal several hundred if not thousand times when some of the highest pressures are created? Even if a potential weak spot is not obvious, can anyone truely know how much damage was done to that molar? I wouldnt want a metal beaten ceramic tile installed on my floor, would you? Doctor: hello thanks for your concern the metal implant is putting pressure on your pfm bridge .you need to get your bridge replaced.. the implant should crowned with porcelain so that it does not harm the opposite crown. the structural integrity of molar will remain sound unless the tooth preparation was done ideally.. even if the molar becomes weak there is a option to place post in the distal root of the molar to make it stronger enough to hold the bridge and pressure as well.. please visit your dentist for further treatment... hope it helps"
},
{
"id": 36461,
"tgt": "Could itching, swelling and discharge be due to yeast infection?",
"src": "Patient: is this a yeast infection?i have a cream cheese like discharge, it is very thick but its not like cottage cheese. i am very itchy down there and irritated and swollen. but there is no foul or strong odor. i am using monistat 3 and when i took the first dosage i felt relief but the next day some itchinh was back. Doctor: Hi,Your problem of having itching, cheesy, thick discharge is definitely due to yeast infection you have.continue with medicine.Apply anti-fungal cream locally.Keep local hygiene clean, dry and airy.Ok and take care."
},
{
"id": 189372,
"tgt": "Have pain, swelling and blisters in mouth. Unable to eat. Should I take antibiotics?",
"src": "Patient: Hello My mouth hurts and I have a couple of blisters in my mouth and it is also swollen on some places including my gum. I can't eat or brush my teeth because it hurts so much. I also went to a doctor who told me to eat antibiotics and when I went to another doctor he told me I shouldnt eat antibiotics. What should I do? Doctor: Hi, Thanks for asking the query, Blisters in the mouth can be due to various reasons it can be due to ulcerations, allergic reactions, bacterial and viral infections. I would suggest you to approach a Dentist for clinical examination and evaluation which helps in exact diagnosis and treatment. Apply metrogyl oral ointment topically over the affected area, go for symptomatic treatment at home by maintaining good oral hygiene to prevent superinfection, use antiseptic mouthrinse , topical analgesic, antioxidant suplements. Hope this helps out. Regards"
},
{
"id": 99753,
"tgt": "How to cure tightness and heaviness in chest due to adult onset asthma?",
"src": "Patient: Hi, I am a 54 year old female with adult onset asthma. On August 8th, I returned to work as a preschool teacher, and after 2 hours of teaching I was experiencing shortness of breath, heaviness and tightness in my chest and wheezing. My small airways were blocked quiet a bit. Since then, I've been on breathing treatments every 4 hours, 7 day steroid pack, singular, zyrtec, and advair. After 11 days, I am still experiencing the very tightness and heaviness in my chest and practically no relief from medicines. I did have an ekg to rule out heart. Could it be anything else? Do you think I should have any other tests? The doctor insists that I am going to get better. Any advise would be greatly appreciated. Thank you! Doctor: See if u are having bronchial asthma diagnosed correctly then u will require life long MDI inhaler or rotahaler and that will be prescribed by pulmonary physician as a maintenance treatment and not the 7 or 10 days course of steroid or sympathomimetic brochodilators.I haven't seen any patient who has cured completely from b.asthma from allopathic treatment. See u accept your problem and consult the above doctor rather than going for different different tests."
},
{
"id": 203527,
"tgt": "What should be done for bumps on penis?",
"src": "Patient: i have 2 shiny bumps on head of penis. one very small and the other is slightly bigger. it is also very uncomfertable at times as well. Pluss after sex the bumps get worse, i got an s.t.d. tests and h.i.v. aids test and they all came back negative. what should i do? Doctor: Hi,As your test for STD is negative then you might be having some bacterial infection on the part.Apply antibiotic cream locally.Go for one antibiotic medicine course for 3-5 days.Ok and take care."
},
{
"id": 35968,
"tgt": "Why is my HCV infection not healing inspite of using pegasys with ribavirin therapy?",
"src": "Patient: helloi m a lady of 32 years having hcv infection from last one year.my hieght is 5,8 and my wieght is 61.i have used pegasys with ribavirin theraphy for 6 months but virus is still present.at the end virus count goes down to 2184.but now its 16 lakhs raised.i want to concieve a baby now bcoz i m having only one baby.my latest test shows ALT-75 and viral load is 16 lakhs.so wht should i do now and wht would u suggest.plz reply me. Doctor: Hi You have not mentioned the genotype of HCV infection u have. Treatment guidelines of hepatitis C have been changed recently.Most important thing in HCV treatment is sustained virologic response [SVR], which is persistent absence of HCV RNA in serum 6 months or more after completing antiviral treatment. As you taken course for 6 month, you must have infection with HCV genotype of 2 or 3. According to latest guidelines, the recommendations for patients with prior PEG Interferon/Ribavirin treatment failure:Genotype 2: Sofosbuvir with Ribavirin for 12 weeks (an alternative is Sofosbuvir with PEG Interferon/Ribavirin for 12 weeks).Genotype 3: Sofosbuvir with Ribavirin for 24 weeks (an alternative is Sofosbuvir with PEG Interferon/Ribavirin for 12 weeks). You should consult Gastroenterologist. It is important to know you that Ribavirin is teratogenic.Take care."
},
{
"id": 129968,
"tgt": "Suggest hereditary pattern of Brachydactyly",
"src": "Patient: I have Brachydactyly... but my parents dont have... My right hand is really short... well it is shorter then my left hand. But I can do everything with it. My question is.. why do I have it if my parents dont have it ? And If I want children will they get it too and will they get the same type? because there are a lot of typs.. so if I have type A2 will they get type A2 too for sure ? pleeeas i really really need some answer thants =) Doctor: Hi i am Dr Ahmed Aly thanks for using HealthcareMagic site ,I had gone through your question and understand your concerns .. In my opinion even when both parents have no seen genes they could have sons or daughters with brachydactyl , all genes consists of two parts sometimes parents have this kind of genes but do not show their properties this means that they have that gene in a weak form or medically called recessive genes which will only be seen in their offspring i think that is your case . I do not want you to feel bad but that does not mean that your children will have that gene for sure , actually you could have perfectly normal offspring without any kind of abnormality , it is just said that the percentage of having it is higher than that of your patients . If you have that gene and your wife does not this means your children are less liable to have it unless your genes are strong enough to be shown or dominant . I hope you have good , healthy children and do not worry you having type A2 brachydactyl does not mean that all of your offspring will have it . Please click and consider a 5 star rating with some positive feedback if the information was helpful. Wish you good health,Any further clarifications feel free to ask."
},
{
"id": 207260,
"tgt": "Suggest treatment to overcome fear",
"src": "Patient: Hello Doctor, I get angry very soon these days...I am developing a very negative attitude towards people and life...I only look at negative aspect of every situation...Most of the times i am scared of everything happenning around me...I have a lot of fear interacting with people...Pls help... Doctor: You are having social phobia which is treatable. In my opinion you can take tab fluoxetine and clonazepam. So dont worry. Hope my suggestions helpful and thankful to you. Take care. Don't forget to give ratings."
},
{
"id": 71783,
"tgt": "Suggest treatment for chest pain and breathing trouble",
"src": "Patient: hi a few week ago i went to my gp with pains in my chest and trouble breathing which they treated as a chest infection and give me antibiotics then a week later as i finished my course of tablets i started to get pain in my lower stomach which i went back to my gp and she said i had a water infection so put me on another course of antibiotics then on friday this week as i fisheeed that course i got an infection in my leg which i went to the out of hours surgery and they have put me on two lots of antibiotics again. so i was wondering do you think all this has any thing in commen and do i need to go and see my gp again as i am not feeling to good in my self at the moment and keep getting pains back in my stomach? thanks mark Doctor: Thanks for your question on Healthcare Magic.I can understand your concern. In my opinion, we should definitely rule out bronchitis in your case. So better to consult pulmonologist and get done clinical examination of respiratory system and PFT (Pulmonary Function Test).PFT will not only diagnose bronchitis but it will also tell you about severity of the disease and treatment is based on severity only. You will mostly improve with inhaled bronchodilators (formoterol or salmeterol) and inhaled corticosteroid (ICS) (budesonide or fluticasone).Don't worry, you will be alright with all these. Hope I have solved your query. I will be happy to help you further. Wish you good health. Thanks."
},
{
"id": 26152,
"tgt": "What are the symptoms of heart disease?",
"src": "Patient: i am 24 year old female, 5'2, 172 lb, medications i took today were ibprophin and zertec. my heart rate now is 115bpm , i have been experiencing headaches for past three day, yesterday my pulse was 124. i am not on any other medications i am eating healthy, recently lost 8 lb. should i be concerned about my heart? Doctor: hello, I have gone through your query.Thanks for using HCM. Your symptoms are suggestive of hyperthyroidism i.e. hyperfunctioning of thyroid gland. Please get your serum s.T3,T4and TSH ,a blood test of thyroid function,done.once you get the report and confirm that you have hyperthyroidism you should conslut a physician for treatment My best wishes Dr.Rajesh Teli,MD."
},
{
"id": 128693,
"tgt": "What causes severe pain in shoulder after injury? Suggest treatment",
"src": "Patient: right shoulder problemHello Doctor. I injured my shoulder two days ago while wrestling with my cousin. I was in a head lock underneath him when he decided to trip me and bring me to the ground. When we fell, all of his body weight landed on my right shoulder and I instantly felt a swelling sensation. I was still able to movw my arm afterwards but as the shouldwr cooled down it made it impossible for me to move it without assistance. Aboit a year ago I was playing football and threw a ball against the wind. When I did that I felt a small pop in the middle/center parr of my shoulder. I did see a therapist for it and had it fixed. This pain is similar but more intense. My shoulder is not poped out but my rotation is very limited. I have to assist my right arm even when I am laying down. Whether its putting my arm on my stomach or above me it still needs assistance. Whenever I feel I want to move my arm anywhere, I feel I have to battle a tweak inside my shoulder. Would this be a rotater cuff injury or just some swelling that isn't allowing my arm to move yet? I fell like something needs to pop back into place but only slightly. I'm told that that feeling is because of the swelling. It also hurts to get up and lay down. I did not ice right away but I did apply salon pas and also tiger balm. Do you have any advice for me? Thank you for your time and help. Doctor: Dear patient There is much possibility of rotator cuff tear or bankart lesion, both soft tissue injury may occur during sports like wrestling. To confirm diagnosis First xray of involved shoulder with clavicle anteroposterior view should be done. Secondly Mri of the involved shoulder also needs to be done to confirm rotator cuff injury. Please get it done from radiology centre nearby you. immobilize your arm in shoulder immobiliser brace for 3 weeks. avoid overhead lifting of shoulder.If Mri report is abnormal please consult expert orthopaedic surgeon nearby you."
},
{
"id": 101775,
"tgt": "Can hair dye cause allergic reactions?",
"src": "Patient: I used \"just for men\" beard coloring and i had a bad reaction to it. I have bumps that are irritated and discharge a fluid that crusts when it dries. I've used the product for years and I have no clue why I all of a sudden have a reaction to it What can I do to stop my skin from being irritated? Doctor: hello,thanks for your query, yes, hair dye can cause allergy due to its ammonium components even after several years. . bumps discharging fluids sounds it may go for secondary infection. it requires oral antibiotics.please visit dermatalogist for evaluation and treatment.all the best.take care."
},
{
"id": 149690,
"tgt": "Tremors, extreme fatigue, weakness, chills down spine on exhalation. Test results normal. What could be going on?",
"src": "Patient: Tremors, extreme fatigue/weakness. Pins n needles traveled from feet up to entire leg. Turned to pain after 2010 r-chop. Left side weakness. Chills down spine upon exhalation . Syncope 4x (hot/humid) Neuro theory was paraneoplastic. Tests not supporting. Told not parkinsons, nor are tremors essential. Normal blood, thyroid, hypoglycemia tests. Normal emg, tilt, mri (did not include neck) Neuro in 1989 told me pd or ms. Drs of internal medicine n onc have said no to ms. What might be going on? Rheum diag fibro 2011, neuros say not fibro (I agree). Doctor: Hello. Thanks for writing to us. The feeling of fatigue and tiredness on exhalation is likely to be due to a circulatory problem like hypotension or an electrolyte imbalance. A detailed examination and investigations are needed to find out the cause.I hope this information has been both informative and helpful for you. You can consult me again directly through my profile URL http://bit.ly/Dr-Praveen-Tayal Regards, Dr. Praveen Tayaldrtayal72@gmail.com"
},
{
"id": 41304,
"tgt": "When can i test to get an accurate result after cavitating morulas transfer?",
"src": "Patient: I had two cavitating morulas transferred 8 days ago. This is my 4th fresh transfer. My first resulted in a successful twin birth. I am currently 40 years old and have always transferred blasts before this. Right now I feel as though my period is about to start and am concerned. In my last 5 failed cycles I have experienced huge bloating which I do not have now. I am due to have my beta test in 3 days but want to try a home test. Would tomorrow (day 9 post transfer) be too early to see an accurate result? Doctor: Yes, it would be too early for a home test. As you must be aware, the urine home tests evaluate the rise in hormone called as HCG. Foe these tests to become positive, you have to wait for about 8-10 days after the missed period. However, there are some beta HCG testing home kits. You can try one of these; but that too 3 days after the missed period."
},
{
"id": 162838,
"tgt": "What is the prognosis for rectal cancer in a 5 year old?",
"src": "Patient: my son was diagnosed with rectal cancer on august 19 last year . he has had 5 weeks of radiation and then surgery to remove tumor and get the bag for about 5 months. he will now need chemo pills like before ,plus intravenus every 3 weeks. eventually, md will reverse the bag and he will have normal bowel movements from rectum. i want to know if he will live long after all this, as he still smokes and refuses to stop.he is 53 years old. Doctor: Hello and Welcome to \u2018Ask A Doctor\u2019 service. I have reviewed your query and here is my advice.The prognosis depend on staging of tumor. It depends on how much it has been spread outside rectum. If metastatisis occurred outside rectum or in liver than prognosis is poor. Complete prescribed chemotherapy and hope for the best. So provide me reports that has been done to give more comment Take care Hope this will help you"
},
{
"id": 184707,
"tgt": "How to treat a painful lump after teeth extraction?",
"src": "Patient: I had 4 wisdom teeth extraction surgary and the doc dis not give antiboitc so in third day the sweilng got so much larger and there is like a soft pump in my left cheek and pain whine i touch it from the outside and strang terrible taste what shoul i do ? Severe swelling to surgery Doctor: Thanks for using Health Care Magic.Read your query.Pain and increased swelling after few days of extraction is due to the non healing of the socket . I would advice* You will need a course of antibiotics which can include amoxicillin thrice daily for 5 days and tab metrolag along with ibuprofen for pain ( if you are not allergic to any medicines).Please visit dentist and get a prescription for this medicines .* Use saline water gargling and avoid vigorous gargling .*Do not use hot bag from the cheek surface ,use cold compress moist for the swelling.*Evaluate the status with your dentist and if needed get the abscess drained and that will later induce the healing.*Maintain a good oral hygiene.Hope this was helpful.Thanks and regards."
},
{
"id": 78402,
"tgt": "What to do as i am suffering from breathing difficulties during high speed fab blow?",
"src": "Patient: In April first week, I was suffered from h1N1 viral fever and got cured by 2nd week of April. However, last one month, I am noticing thick dense phlegm / mucus noticed. I am also some time suffering from breathing difficulties during high speed fab blow. What to do now ? Doctor: Hi. I can understand your concern. Sometimes viral infections leave residual damage to the lungs causing symptoms.So better to consult pulmonologist and get done clinical examination of respiratory system and PFT (Pulmonary Function Test). It will also tell you about severity of the disease and treatment is based on severity only. You may need inhaled bronchodilators and inhaled corticosteroid (ICS)for your cough and phlegmDon't worry, you will be alright. Hope I have solved your query. Wish you good health. Thanks."
},
{
"id": 155058,
"tgt": "What are chances of cortical cyst in abdomen being cancerous?",
"src": "Patient: My mother was diagnosed with lt breast cancer in 2003 after whoch the affected breast along with 11 lymph nodes were removed(although only 1 lymph node was affected). She also underwent 6 cycles of chemo. Now agter 8 yrs in her ultra soind report of her abdomen, a 16X16 mm cortical cyst has been seen in the upper pole of her right kidney. What are the chances of the cyst beong cancerous?Also what are indications and tests required to rule out malignancy? Doctor: Cortical cysts in the kidney are a normal finding that can be found in anybody. They are not cancerous and unrelated to your mother's breast cancer. You should stop worrying about it if it is not causing any symptoms. Just keep it under observation by dong an ultrasound every 3 months."
},
{
"id": 124107,
"tgt": "What causes pins and needles sensation from head to hip?",
"src": "Patient: In the past two weeks, moments before I start sweating, I get extreme pins and needles. It starts in my head and sometimes travels down to my hips. The moment I start sweating or cool down, the sensation disappears. I am positive it isn t an allergy. It also started to happen very recently and I haven t made any lifestyle changes. Thank you in advance for your advice! Doctor: Hello, The pins and needles sensations that you are having can be related to low levels of vitamin B12 or due to an electrolyte imbalance. Taking electrolyte rich fluids and starting with proper supplements will help. Hope I have answered your query. Let me know if I can assist you further. Take care Regards, Dr Praveen Tayal, Orthopaedic Surgeon"
},
{
"id": 152996,
"tgt": "What are the symptoms of potential cancer?",
"src": "Patient: I am a 5o yrs old man. Since I have a history of cancer in my family members [my mother died of Gall bladder cancer at the age of 65 yrs and now my elder brother (55 Yrs) is having stomach cancer], I was advised endoscopy. The endoscopy report dated 7.10.10 suggests in the impression three things \u2013 (a) tongue like projection seen proximal to Z line in esophagus, Sample taken and sent for biopsy to rule out Barret\u2019s Esophagus, (b) mild hiatal hernia, and (c) GERD. But I don\u2019t have symptom of stomach pain/ acidity at present. Two years back (first time) and six months back (second time) I had acute stomach pain. USG done (in Feb\u201910, June\u201910 and Aug\u201910) indicated gall stone, for which I took ursodil 300 mg twice daily, along with pantocid, for 4 months, without any improvement. Now I have stopped medication for past one month and made up my mind for surgical removal of gall bladder. At present I am taking medication for cholesterol ( Atorva 10 mg). I don\u2019t have any symptom of acidity or heart burning. Sometimes, occasionally, I have acidity after reach food. In 1998, I had hear burn and hiccup problems. After endoscopy, I was diagnosed as having distal esophagitis (there was multiple superficial ulcerations in the lower 1/3rd of esophagus. It was treated and I was OK within 1.5 months of medication. I am worried about the findings of the current endoscopy. Please guide me how to go about it. Can the tongue like projections be cured by medication? The local doctor has prescribed Nexpro tablet 40 mg once daily, which I have not started yet. I want to take a second opinion before starting the medicine for long period. The local doctor has also advised for repeat endoscopy after 3 months. I want to know whether, the tongue like projection in the esophagus can be cured by medication. Please guide me as to how I should handle my current health issues relating to my endoscopy report. Doctor: Hello dear. I have gone through your details. However, it is very difficult to comment on so called tongue like projection without seeing the endoscopic films. The final result will of course depend on the biopsy reports. So you need not to be worry and wait for the report. Thanksregards"
},
{
"id": 134216,
"tgt": "Are there any tests to see what causes arthritis?",
"src": "Patient: what good does an arhtiris panel do when you already know you have it wouldn t it be better to see what is causing the arthritis ie, mycoplasma test and vitamins that are known to be depleted the test may be useful for the doctor but itdoesn t tell where the inflammation is and most likely when i am in remission i cannot go and get a test so it always looks like i am doing badly in my medical records and no one seems interested in what is causing it Doctor: hi,thank you for providing the brief history of your case.as you mentioned you have arthirits - this means there is a degenerative change in the human body which leads to arthritis.since you haven't mentioned whether it is osteoarthritis or rheumatoid arthritis I will explain the both in detail which are the most common ones.osteoarthritis - the terminology it self indicates that the bone is involvedrheumatoid arthritis - means the pain keeps moving from one joint to another.Now coming to explain you what causes arthritis - Arthitis is the degenerative changes happening in the human body. this degenerative changes are called the wear and tear of the cells.example is - when you buy a new shoe and use it for 2-3 years and later your shoe will have the wear and tear. You cannot expect the shoe to be the same after 2-3 years of constant use.As I said, degenerative change is a natural phenomena which cannot be revered and also there is no particular test we need to confirm the arthritis.Also, to help you better during arthritis there are medical treatment directed towards the symptoms and its relief. As medicine acts as a supportive system for the symptoms to lower, as this will allow the body to heal of its own. Also to mention that the body is the medicine of its own and this is also a normal physiological phenomena.Next coming to what helps in arthritis is - along with medicine there is need to strengthen the muscles around the joint involved so that the normal joint alignment can be achieved. Any changes in the normal anatomy of the joint occurs there is a pain. Now pain is a protective mechanism of the physiological phenomena of the body.also along with medicine and physical therapy most of the patients recover.if you have more queries I can answer so you can ask me directly.With the grace of God I Wish you a speedy recoveryregards"
},
{
"id": 153447,
"tgt": "Can chemo and radiation be helpful for prostate cancer?",
"src": "Patient: My dad's Psa jumped from 580 to 1100 in a month. He has prostate cancer and it is now in his rib cage and pelvis. He is on lukrin and has already had one round of chemo. He was told by the oncologist that his only hope is a different chemo which does not cause hair loss. he was also told that without this chemo he would only live 3 months. he is exceptionally tired....Can we have hope in this last chemo and would radioation work? Doctor: Hi, dearI have gone through your question. I can understand your concern.Your father has prostate cancer and it is spread in rib cage and pelvis. It is stage 4 cancer. Surgery will not helpful at this stage. Only treatment options are chemotherapy and radiotherapy. He can use different chemotherapy regimen. But ultimate prognosis is poor. life expectancy is not good. Consult your doctor and plan accordingly.Hope I have answered your question, if you have any doubts then contact me at bit.ly/Drsanghvihardik, I will be happy to answer you.Thanks for using health care magic.Wish you a very good health."
},
{
"id": 163021,
"tgt": "What causes a black spot post circumcision?",
"src": "Patient: My newborn was circumcised and on one of the stitches a black spot began to appear. At first it just seemed as one of the stitches but with time it began to grow. it is irregular. My baby is 5 months now and the spot stopped growing 2 months ago. also, the stitch close to it hasn t been reabsorbed. Doctor: Hi...Thank you for consulting in Health Care magic. Skin conditions are best diagnosed only after seeing directly. I suggest you to upload photographs of the same on this website, so that I can guide you scientifically. Please revert back to me with images so that I can guide you better.You can approach me at the following link.Once the page opens there will be an option below my image as \u2013 ASK ME A QUESTION \u2013 click on it.Please find the link below -www.healthcaremagic.com/doctors/dr-sumanth-amperayani/67696Regards - Dr. Sumanth"
},
{
"id": 149686,
"tgt": "Have firbomyalgia, pherial neruropthay , sciatic, osterproses, stenosis. Positive for ANA test. Reason?",
"src": "Patient: i need a doctor who has experince with bertoloti's syndrome i also will need a nerve block done at st. charles in bend oregon i also have firbomyalgia, neruropthay pherial and sciatic iam positibve for ana test. i have osterproest, stenosis. my L5 /SI joint which are fused since birth is choking my spinal cord. im barely can walk at times i have hi inflamation and have no money and have been desnidedfor social seccruty 3 times and the judge discrimenated aginaisnt me he said vverthing was my fault because i m over weight Doctor: Hello Thanks for writing to Healthcare magic,I have studied your case,As you are having severe spinal stenosis i may suggest for operative decompression.epidural injections can be helpfulcheck your vit B12 and vit d3 level .Physiotherapy may help you for longer period.Hope I have answered your query. Let me know if I can assist you further.Take care"
},
{
"id": 29875,
"tgt": "What causes persistent running nose?",
"src": "Patient: I have had a running nose for a week now . When it first happen I though it was a cold so I started to take DayQuil but after a couple days I quit and figured it wasn t a cold . I have been taking a nasal spray to stop my sneezing other then that I haven t took anything else the running nose is all the effect I am having and no I am not allergic to anything Doctor: thanks for posting your query to hcm.as I understand it could be because of some kind of allergy .nasal spray is not the treatment of disease it gives only symptomatic relief. you should consult to a ENT surgeon for examination and proper managment .hope it will help you ."
},
{
"id": 149790,
"tgt": "Have fibromyalgia, seizures. Taking Keppra for pain and hot flashes. Suggestion?",
"src": "Patient: I read the question and comments from NanCcan about having pain waves followed by hot flashes. I have the exact same thing since July of 2010. I have fibromyalgia and just recently have had 2 seizures. I am on Keppra which has reduced the pain waves but not all nor reduced the intensity of the pain. I get them all day and they wake me up at night. I can t get my doctor to believe me. I lost my commercial driving job due to the seizures so I now have no income or insurance and my doctor keeps telling SSD that I m not disabled so he is making it impossible to get assistance. anyway. does anyone have a diagnosis for these pain waves or a better medical term? Something I just realized I forgot to add is that I was on Depo-Provera shots for 11 (eleven) years and stopped in 2007 when I had a hysterectomy. Doctor: Hello. Thanks for writing to us. The pain waves that you are feeling are a part of fibromyalgia. If the pain waves are debilitating then you need to consult a pain specialist for the proper management.I hope this information has been both informative and helpful for you. Regards, Dr. Praveen Tayal ,drtayal72@gmail.com"
},
{
"id": 189149,
"tgt": "15 year old with hot flashes and early periods after having wisdom tooth removed. Reason?",
"src": "Patient: i just had my wisdom teeth removed the Tuesday before last, never have i ever had hot flashes, i'm only 15 years old. i know my body went through trauma, and it probably stressed it out a bit. because i started my period the day after i got it out completely off schedule, like i said, i assume its from where my body was stressed. but, whats causing the hot flashes? withdrawl from the pain medicine? or just hormones? Doctor: Hi, Thanks for asking query, Wisdom tooth extraction is a complicated procedure and is often associated with pain , fever, swelling , difficulty in mouth opening. Early periods and hot flushes your having can because of stress your taking. I suggest you to take rest ,avoid stress, drink plenty of water, maintain oral hygiene, prevent food retention, take vitamin supplement, use anti septic mouth wash. Hope this will help you."
},
{
"id": 153874,
"tgt": "What is the prognosis for stage 4 esophageal cancer?",
"src": "Patient: My husband has stage 4 esophageal cancer that has spread to a lot of his liver, spread also to the lymph nodes, bloodstream and 3 spots on the lungs. He is taking palliative chemotherapy only. How long has he got left, We want to know so that we can get our affairs in order. Doctor: Chemo may be given (possibly along with the targeted drug trastuzumab if the cancer is HER2 positive) to try to help patients feel better and live longer, but the benefit of giving chemo is not clear. Radiation therapy or other treatments may be used to help with pain or trouble swallowing.For cancers that started at the gastroesophageal (GE) junction, treatment with the targeted drug ramucirumab (Cyramza) may be an option at some point. It can be given by itself or combined with chemo. so in 4 th stage only give in palliative treatment .if you r interested in Homoeopathy so in homeopathy medicine r working in good result for this type of cases."
},
{
"id": 118400,
"tgt": "Is my heart healthy with cholestrol 128 and blood pressure 122/80?",
"src": "Patient: I have done heart health checkup. Below is my complete report details for your supportI am 30 yrs old unmarried male having 77Kg wt and 172 cm ht.1) my cholesterol level is 1282) fasting sugar is 90 and pp is 1103) tsh level is 2.3 4) chest x ray pa is clear and normal5) ecg show sinus tachycardia6) echocardiogram show concentric lvh with Gr. I diastolic dysfunction.7) blood pressure is 122/808) anxiety, stess level is high9) total veg, non smoker, non drinkerIs my heart is healthly, how much I worry about my heart. I feel chest pain with palpitation when I got excite or sad or happy etc... Doctor: Hi, welcome to our site. I am Dr Saumya Mittal.Read your query. That is a very significant question and i appreciate your problem. I will try my best to answer your queryYour reports are normal to a large extent. However your echocardiography does suggest lvh or left ventricular hypertrophy. This usually happens when your heart is pumping blood against a resistance to the flow of blood. It is usually a feature of hypertension, but you have mentioned a normal blood pressure.However you have also indicates that you have a type A personality- anxiety and stress. This may be causing an undue load on the heart.I would seriously suggest that you meet a local doc to prescribe you a medicine from the group of ACE-I or ARBs. And do try relaxation techniques.To really make sure all is fine with the heart, you may want to try a stress test- treadmill/dobutamine.I hope this helps you. Inform the reports mentioned above so i can be of help further. Best of luck.I have given you the answer to the maximum considering the information provided. The results of the tests could further enhance my answer to you.Please do understand that some details could be extracted from a detailed history and examination.Looking forward to your return query with the details asked so that I can help you further.(If the answer has helped you, please indicate this)"
},
{
"id": 174032,
"tgt": "How to treat cough and wheezing in a 5 years old child?",
"src": "Patient: Doctor, I have kid 5 years old and he is getting cough and wheesing when ever he is having cold items. I am using treatment from doctor flowhale and asthalin inhealer daily two times. I need advise for prevention, please help me. Thanks & Regards, Srinivasarao Doctor: HelloThe symptoms you have mentioned are suggestive of allergic bronchitis/asthma.The mainstay of management in this condition is to avoid the precipitating factor.So I would suggest you to avoid any cold items to prevent the exacerbation.If symptoms occurs more than one time per week(day symptom) or more than one per month(night symptom) then continuous long term controller treatment is needed.If symptoms are not so frequent short duration treatment(steroid like flohale and bronchodilator like asthalin) is all that needed.So I would suggest you to follow the advice of your doctor and avoid the precipitating factor.Regards"
},
{
"id": 139553,
"tgt": "What causes dizzy spells and metallic taste in mouth?",
"src": "Patient: I just had a dizzy spell, with a metallic taste in my mouth and suddenly felt very hot...could have been sick. I had a good breakfast but an interrupted nights sleep. Should i be worried Age 37, weight 13.5 stone, good general health - dont really get colds etc Doctor: Hello,I think that you should not be worried more than necessary for these symptoms. A viral infection may cause such issues. If symptoms persist or get worse you should see your Doctor.Hope I have answered your question. Let me know if I can assist you further. Regards, Dr. Erion Spaho, Neurologist, Surgical"
},
{
"id": 82966,
"tgt": "Boils on groin and in arm pits. Suffering with SLE Lupus,Chronic Fibromyalgia and osteorithis. Treatment?",
"src": "Patient: Hello Dr.,I continue to get boils in my groin area,sometimes i have 3 or 4 the same time.Two weeks ago i had one under my armpit.The one under armpit only last 2days..Today i discovered one coming in my head. I am very upset at this point.I always keep myself bathed and dry.I use tea tree oil soap.Also i use tea tree oil on a cotton ball and put on boils and all around the areas where the boils are. It heals them pretty fast. The problem is they continue to come back.Would you know what is causing this? I also suffer with SLE Lupus,Chronic Fibromyalgia and osteorithis.I need your help please!! Thank YOU in Advance. Doctor: Hi. Recurrent boils may be a marker of an underlying immunosuppressed state or diabetes. The fact that you have SLE makes me think that you must be on glucocorticoids, and some other immunosuppressant drugs to control your disease, which would make you susceptible to infections and also diabetes. So I would advise the following in my patient:1) If your SLE is under control, your physician/rheumatologist may reduce the drug doses as per need to control the infection. 2) look for diabetes by checking you fasting blood glucose and post prandial blood glucose levels; if high this needs to be controlled3) control the present infection with an adequate course of antibiotics like amoxicillin/clavulinic acid (augmentin for example), draining of the pus collection if required, and dressing.4) if you have nasal carriage of staphylococcus aureus, this makes you predisposed to recurrent boils, and this needs to be eradicated. With these measures, the infection can be controlled.Thank you."
},
{
"id": 101904,
"tgt": "What are the remedies for asthma?",
"src": "Patient: Hi, i am having breathing problem(veecing problem) from last 6 month. i had treatment in allopathy through pulmonologist doctor but still it is not get cured. i have been taking medicine twice a day (budecort 200 rotacap). can you assist me. is this correct medicine. Doctor: Hallo . Thank you for asking this question .I will do my best to help you . You have to take an anti allergy medicine at night and use rotacap that contain budecort with bronchodilator . Once get totally relieved from symptoms then reduce the doses . Hope I have answered your query . For further questions please contact ."
},
{
"id": 171137,
"tgt": "Suggest remedy to prevent H1N1",
"src": "Patient: My son is 20 mth old. He is presently in kerala, India. there is slight emergence if H1N1 cases, plus recently he has been visiting hospital to meet his grandmother who is admitted. does he need to get vaccinated (prophylactically). If so , which one and the dosage n frequency Doctor: Hi, welcome to HCM. Since you are visiting a hospital and winter is coming, i sincerely advice you to give flu vaccine to the child. For first time vaccination of flu, the vaccine is given twice at an interval of 2 months. There after a single dose of vaccine is required every year. Vaccines of flu come by the name of influvac or vaxigrip. I hope this has helped you. Take care. Regards - Dr Deepak Patel, MD Pediatrics"
},
{
"id": 39509,
"tgt": "Is co-trimoxazole bactrim advisable for cough?",
"src": "Patient: doc, i have a cough for 2 weeks already. I've been to hospital for check-up/consultation. the doctor prescribe CLARITHROMYCIN CLARIGET good for 1week. I took this medicine. but I'm not cure. now my question is can co-trimoxazole bactrim forte is advisable for my cough? ty.. jpot Doctor: Hello,As you are having cough for last one week gor which your doctor has prescribed clarthromycin and symptoms has come down for some time but your symptoms are not cured.The viral infection will take some time to reduce your symptoms and as you asked for the use of cotrimoxazole is an excellent drug which helps you toreduce the symptoms if you are resistant to clarthromycin. I would suggest you to wait for few more days before you shift to this drug. Drink plenty of fluid and take antihistamine in addition to this.Thank you."
},
{
"id": 115278,
"tgt": "Is low platelet count with family history of Prothombin factor 2 concerning?",
"src": "Patient: I got my CBC results yesterday. Other than my platelets my DR. said everything was perfect! My WBC & RBC were perfect. My platelets were @ 95 down from 115 2 years ago. I recently found out that many in my family have Prothombin factor 2. When I told her this she said it maybe a genteic issue. She didn't seem to concerned and told me she would see me in 6 months for my BP. I don't show any signs of bruising easy or bleeding, so should I be that concerned if whe isn't? Doctor: Hi,Thanks for asking.Based on your query, my opinion is as follows.1. With RBC and WBC being normal, your normal platelet count maybe in the same range.2. Platelets below 50000 is mildly worrisome. Only, if it is less than 20000, then I would worry. Until then, especially with 95000, well it is really good.3. Platelet is involved in bleeding while prothrombin factor 2 is involved in clotting. Both are not related much. Each would not affect the other.4. As, you have no other signs of symptoms, not to worry. Do regular followup with your doctor as advised.Hope it helps.Any further queries, happy to help again."
},
{
"id": 201130,
"tgt": "What causes itching sensation and discoloration of skin on scrotal area?",
"src": "Patient: Sir I had fungal infection on my penis shaft months ago. And had been treated well but I am facing a new problem now. My scrotum sack itches badly and have a colour change to a little bit blackish colour and a leather like skin. I am afraid is it a untreatable problem. please help. Doctor: Hi,Thanks for writing in.Its possible that you have another fungal infection which is now in the scrotum regions. Fungal infections affect moist areas where the skin remains moist and allows the fungus to grow. It is possible that the fungal infection in your penis shaft is treated but some of the fungi have now started growing in the scrotum. This causes itching and discoloration. Please trim your pubic hairs short and dry your genital area completely after bath. Wear cotton inner clothes and wash your genital region with warm water twice daily. Apply antifungal ointment to the area of itching and skin discoloration twice daily. You can use the same ointment which you have taken earlier. Avoid scratching the area as this will spread the infection. The itching should stop 3 days after starting application of anti fungal ointment and treating the condition might take a month if treatment is taken regularly. Skin will return to normal colour in few months."
},
{
"id": 48866,
"tgt": "Suggest treatment for kidney stones and sperm leakage",
"src": "Patient: hi doc ..i had kidney stones nd i diagnosed it but now may b stone in penis coz itssiritates me nd now i think sperm came wen i gotta peee or even sometime my underwear is filled with sperm like sip sip sperm mean some some sperm came every second wat i hv to do plzz suggent me something Doctor: HI THANKS FOR POSTING YOUR QUERY ON HCM;KIDNEY STONES CAN BE MANAGED WITH PROCEDURES LIKE ESWL-EXTRACORPOREAL SHOCK WAVE LITHOTRIPSY.SURGICAL TECHINIQUES INCLUDE PCNL-PERCUTANEOUS SHOCK WAVE LITHOTRIPSY.SPERM LEAKAGE CAN BE MANAGED WITH URETHROPLASTY.CONSULT AN EXPERT UROLOGIST FOR FURTHER HELPTHANK YOU,TAKE CARE"
},
{
"id": 141172,
"tgt": "Does a brain aneurysm cause temporary memory loss?",
"src": "Patient: hello, my cousin who is 48 just had an aneurysm they had to put in a valve to drain the fluid from the brain. He is doing fine but has short term memory loss. He doesnt seem to remember what happened earlier that day is this normal and is it temporary or could it be a permanent thing Doctor: Hello, The effects of anesthesia from any type of surgical procedure are well known and especially in the domain of loss of short term memory so if the surgery was recent (past week or so) then, it is perfectly reasonable to attribute this to the effects of anesthesia or other medication given for the procedure which will likely get better with time. If it is longer ago than that then, one must ask the question of whether or not your cousin ever had a bleed from that aneurysm? Did it bleed at all during the procedure to install the drain? If so, then, this could be the explanation as well but in that case effects of memory difficulties may linger much longer. Hope I have answered your query. Let me know if I can assist you further."
},
{
"id": 146211,
"tgt": "Is surgery possible for herniated disc and disc degeneration?",
"src": "Patient: I am 43 have herniated disk at L5 and disc degeneration through out spine. Slight pinched nerve at L4-L5. I have had injections, therapy, and medicines. No help. Surgeon says it isn t severe enough. Now pain is in tail bone, buttocks and runs down legs. Sharp, shooting , stabbing and burning sensation. What are my options? Is there a new surgery that is supposed to help this? Doctor: Hello dear,Please accept my concerns for you.In a degenerative spine disease, with disc herniation and nerve root compression,the general principle of treatment is that,till the time your symptoms become so disabling as if to make you functionally dependent on others for your daily routine tasks and the symptoms are not responding to conservative treatment(as medicines and injections in your case),then surgery is the option left for you,but till that time arrives it is usually avoided.You should take a neurosurgeon's or spine surgeon's opinion for it.I hope it is of help to you."
},
{
"id": 7402,
"tgt": "Need effective medication for acne",
"src": "Patient: hello sir.i am 23 year old college girl.and i m suffering from acne problem from since 6 month.from last 2 months i am taking homeopathic medicine and from last 3 days using TOPI BERBERIS cream at night and day.but its not helping out.suggest me something which can work out faster.recently i m using saslic facewash(by cipla) and olay age defense cream.thank you.please suggest something effective. Doctor: Hi, Acne treatment depends upon the severity and type of lesions. The treatment is given in the form of oral medicines( azithromycin/ doxycycline/ minocycline and/or isotretinoin) along with topicals( clindamycin, benzoyl peroxide, adapalene tretinoin) etc. As you said you want faster results, that can only be delivered after a thorough skin examination so that a personalised skin treatment can be suggested. However, you may follow these basic suggestions to prevent and treat acne 1) continue with saslic face wash. Use it twice daily 2) you any continue with Olay cream, if it suits your skin well 3) get your blackheads and whiteheads removed regularly and apply retino a cream in comedone prone areas 4) treat your active acne lesions with application of clindamycin gel and retino a cream. Depending on your acne grade severity, oral therapy might be required, for the same consult a dermatologist nearby. You may also discuss the option of chemical peels with your doctor as peels give good results in short time and also take care of pigmentation and scarring. Hope that answers your query. Take care."
},
{
"id": 76225,
"tgt": "Can certain foods cause severe chest pain?",
"src": "Patient: I was just reading up on why I get severe chest pains after eating certain foods. Have had cardiac catherization heart is okay, take tricor and lipitor, and have had 2 leg stents. Yesterday I had corn on the cob with butter and it seemed to be really bad pain on left side Doctor: HelloDepending on the exact location of the pain this could be gastric acid reflux disorder, a peptic ulcer, or diverticulitisYou need to see a gastroenterologist for an upper endoscopy and a screen for an H pylori stomach infection"
},
{
"id": 3797,
"tgt": "How to get pregnant?",
"src": "Patient: hello please help me i am thirtey years and have daughter from year and half i am trying and my husband to have another baby we went to the doctor he do ultrasound found mild pco but he says it is normal he do hsg and find patent tubes semen anaylsis is normal so he give me clomid and hmg and hcg injection but pregancy does not occur another doctor give me tamxiofen 20mg as once tablet from second day of period for 2 months but pregancy does not occur my prolactin level is 15 fSH level is 5.6 and LH level is 5.6 what can I do next Doctor: HiYou have not mentioned about the day 21 serum progesterone level ( suggests about ovulation)It is reassuring to know that rest of your baseline subfertility investigations are normal.What is your BMI? Do you have regular cycles? Usually women with PCO have irregular cycles as they do not have regular ovulation. I suggest that you take clomid/tamoxifen +Inj HCG for six cycles along with ultrasound follicular study. Ultrasound examination helps us to make sure that ovulation is occurring. If conception does not occur with this treatment then I would discuss with you two options of management. Either to start induction of ovulation with hormones or laparoscopy and ovarian diathermy for polycystic ovaries.Pregnancy rates between laparoscopic ovarian diathermy after 6 to 12 months follow up and six cycle of ovulation induction with gonadotropins are the same. Laparoscopic ovarian diathermy carries the risk of surgery. However, the rate of multiple pregnancies is lower in women who conceive after ovarian diathermy.Take one tablet of folic acid daily.To increases your chances of pregnancy, maintain normal BMI, quit smoking ( if you are a smoker) and have intercourse atleast three times in a week. For women with PCO maintaining normal BMI extremely important. Even reduction of just 5-10% of body weight corrects the hormonal imbalance and improves chances of conception.I hope I have answered to your satisfaction.If you have any further queries feel free to contact me through health care magic"
},
{
"id": 187476,
"tgt": "Could the raw feeling on the roof of my mouth and tongue caused by wisdom tooth extraction?",
"src": "Patient: The roof of my mouth and tongue feel raw. And it is a real discomfort to eat anything. I didnt have this start happening until after I had a wisdom tooth pulled. Is this an effect of that or something completely unrelated?? If so,what?? And what is the cure for this?? Doctor: Hello, Welcome Thanks for consulting HCM, I have gone through your query, as you have mentioned that you have undergone wisdom tooth extraction now you feel raw feeling on roof of mouth and tongue , you do is warm saline gargle two - three times a day Take medication one tablet Vitamin B complex once daily for a week Bad taste on tongue and mouth can be due to antibiotic course you have taken in extractions If you wont get relief then again consult dentist and get Oral examination done Hope this will help you. Regards Dr. Priyanka tiwari"
},
{
"id": 131294,
"tgt": "What causes burning, pain from butt cheek to top of foot?",
"src": "Patient: I been having really bad buring,pains. Starting at left butt cheek, going down to the top of foot. I can only walk few feet then becomes unbarable. Sleep few hours, at times. Sitting an hour at each time .Just not find that one spot for releave. Also all of left leg feels nomb Been to chiropractors 6 visits no releave .Got a M R I waiting to see orthopedic my iliopsoas or sciatica just nervous Doctor: get done serum vitamin B12 level first....put your MRI scan if possible.....eat green leafy vegetables"
},
{
"id": 146857,
"tgt": "Is cerebral atrophy a dangerous disease?",
"src": "Patient: Good Day, I just got from the hospital. I was told that my son was diagnose with cerebral atrophy. I need to know what are the effects of this condition on my 10 month old son. He weighs 12 kgs and is on the 95th percentile in height for his age. He also had 3 attacks of seizures last month. Doctor: HIWell come to HCMIn fact the cerebral atrophy is not the disease but this could be congenital condition and may be related with genetic disorders, in such case the complete case study may need to come on any conclusion this is just brief opinion given here on the basis is cerebral atrophy, some this comes around with the times as the child grow old, hope this helps."
},
{
"id": 131345,
"tgt": "Having noticed severe rash and swelling on face",
"src": "Patient: Hopefully. Had a bone marrow biopsy approx. 3 months ago because of low white blood count. Since then, I have developed severe rash and swelling on my face, ears and other parts of my body. ( fortunately not my genital areas ). Also have sores on my arms and legs. ( The biopsy was negative I might add ). Overall, I have not been feeling well these past 3 months. Could the biopsy be the cause? Doctor: In my opinion to be able to blame it on the biopsy you need to have fever as well , if not then you need to see a dermatologist Good Luck"
},
{
"id": 191225,
"tgt": "Can an additional dosage of Glipizide be taken to manage high blood sugar levels?",
"src": "Patient: I received a Cortisone shot in my knee a few days ago. It has elevated my sugar. I am a type 2 diabetic. My sugar reading are exceeding 300. I take Glipizide xl 5 mg once daily. Is it ok to take additional doses of Glipizide to bring my sugar levels down for a couple days. I was told at time it could drive my sugar up for up to 72 hours. Tomorrow will be 72 hours. Doctor: Was the blood sugar done before that Injection? 300 mg blood sugar does not appear to be a transient rise of blood sugar.No harm in taking Glipizide 1 bd for 2 days,but get your HbA1c done. If it is any thing above 7.5 you will need to be treated on a long term basis."
},
{
"id": 82245,
"tgt": "Why does a spot film of the mid-lower lung field is suggested for further evaluation?",
"src": "Patient: hi. i ve done my chest Pa xray and here s the result: -No previous studies for comparison. -Minimal suspicious opacities are seen in the upper lung fields and in the left mid-lower lung field. -The rest of the lung fields are unremarkable. -The heart is not enlarged. -The trachea is midline. -The costophrenic angles and hemidiaphragms are intact. -The osseous structures and soft tissues are unremarkable. Remarks: An apicolordotic view and a spot film of the left mid-lower lung field are suggested for further evaluation. What does it mean? thanks. Doctor: Thanks for your question on HCM.In noral chest x ray PA view, the apical part of lung is overlapped (hidden) by bony structures like 1st two ribs and clavicle.So the lesion present on apex is not visualised clearly.So we need apicolordotic view.in this view x ray is taken in such a way that bony structures are not going to obstruct the apex. So apex will be clearly visualised and lesion can be identified. And spot film means taking x ray only of interested region. In your case It is left mid and lower part. So in simple words it is the magnifying image of only left mid and lower part. This will give magnified view and so better reporting can be done."
},
{
"id": 25813,
"tgt": "Is it necessary to go for a TMT test?What does the test report indicate?",
"src": "Patient: Sir, My father expired just after angioplasty on 7.1.12 , since than I am feeling pressure on my left side -near under arm portion. Lupid profile conducted. Tests report normal except uric acid 7.00 . My doctor prescribed TMT . should I go for it or not. (reports Tyroid TT3 135.36 ng/dl, TT 4 10.00 g/dl, TSH 0.47, Boood Gulucose fasting 74, urine glu. nil. Cholesterol total 145 , HDL 44, LDL 77, VLDL 24, Triglycerides 120, Total cholesterol/HDL ratio 3.2, LDL/HDL ratio 1.7, Billirubin .. total B. 1.0, Direct B. 0.4 Indirect Bil. 0.6, Alkaline Phosphatase ALP 102 , Aspartate Aminotransferase SGOT 35 , ALT/SGPT 45, GGT 30 , Total protein 7.00,Alb. 4.5 , Globulin 2.5 , A/G Ratio 1.9, , Blood urea Nitrogen BUN 14.0, creatinine 0.7, Uric Acid 7.0, Sodium (Na) 142 , Potassium 4.8 , Chloride 102 , ESR 11 . CBC HB. 15.7, Total WBC Count 5000, Neutrophil 50, Lymphocytes 40 , Monocytes 06 , eosinpolhil 0.4, RBC 4.50 , HCT Haematocrit 41.6 , MCH 34.8 , MCHC 37.7., MCV 92.5 , RDW CV 13.9 , Platelet count 250 , Urine test .. colour Pale yellow , appearance clear , specific gravity 1.015, Peaction 6.0 proteins NIL , Glucose NIl , Bile salts Negative , Bile pigments Neg, Nitrites Neg, Boood Neg, Ketones Absent , urobillinogen Normal , PUS WBC cells 1.2, Urin RBC NIl U. Epithelial cells NILL, Crystals NILL casts NIL , others Nil...... Sir, please advise me . I am hypertensive , taking .. Domtac, OD , covance D , OD , regularly for the last about 10 yrs., started recentely Ecosprine 75 mg. , tonect EZ OD ... Due advise in the light of above is awaited. Thanks .. Dalip Kr. Chhabra 0000t Doctor: Hello Mr Chabbra, Firstly I am sorry for your dad's loss . However that put a positive family history of coronary disease. Other risk factor is hypertension. All the other reports which you have mentioned are perfectly withen normal limits and you shouldn't worry about the same , even uric acid at 7 , no treatment is indicated at present . Now about TMT, the symptoms you describe are not characteristic for angina , however because of the risk factors which I have mentioned and if you are obese , smoke or have angina stressful life you fall into intermediate risk for coronary event in future , so for you TMT is a good test to rule out underlying cardiac issues. In TMT they make you run on a treadmill and monitor your blood pressure vitals and see ecg changes . Also any symptoms during exercise is unmasked . So in short they make your heart exercise and check whether everything is ok than . Hence if your TMT is fine you may rule out any critical blocks indirectly. So yes TMT would be a good test to rule out heart as the cause of the chest discomfort you feel . Hoping I could help . Regards Dr Priyank Mody"
},
{
"id": 187243,
"tgt": "What treatment is suggested for abscess on gums due to temporary crown placed?",
"src": "Patient: I had a temporary crown placed on Monday (lower back molar) I developed a blister on the gum behind the tooth 2 days later. The blister is now higher than the tooth. White, no pus. Some swelling. Not a lot of pain. Could it be from an injection site or should I be concerned about an abscess? Doctor: Hello, Welcome Thanks for consulting HCM, I have gone through your query, as you have swelling on gum it can be due to deposition of food so there can Periodontal pocket formation or due to formation of Periodontal Abscess also . Go to dentist and go for IOPA if necessary , then go for treatment Scaling and curretage . Do warm saline gargle two - three times a day. Hope this will help you. Wishing you good health."
},
{
"id": 174999,
"tgt": "What causes stomach ache and fever in children?",
"src": "Patient: My duaghter, age 5 got out of the hotel pool saying she didn t feel good. When she went to bed she said her tummy hurt and she had lots of gas and began running a fever. We are giving her Tylenol to reduce the fever and put her in a wa bath for awhile. She said her tummy feels better but still had a fever. I don t know how high since I don t have a thermometer with me at the hotel. Right now I have her laying in bed with a cool towel on her back while she is covered up to try and lower temp. Any other suggestions b Doctor: Thank you for following up. In view of above , I believe she has infection in tummy after swallowing water in pool. I appropriate your attention to her condition. It can be also viral infection with intestinal syndrome . You can try to give her niftas(nitrofurantoin) 1 tablet 3 times 7days, econorm(saccharomyces) 1 paket 2 times 3 days. If her pain will not decrease, then start to give antibiotic-cefixime in the dose of 4mg/kg/dose 2 times during 5 days Best regards Dr.Svetlana Shrivastva"
},
{
"id": 51400,
"tgt": "One kidney, diabetes, have problems with knee, taking hydro-codon. will it affect the kidney function?",
"src": "Patient: my husband has only on kidney due to the removal of the other due to cancer in the kidney, he is having problems with his knee and he also has diabeties He was given meloxiam with his Kidney doctor lists not to take. He also has Hydrocodon I want to know if Hydrocodeon will affect his kidney function since it is in the mid 20 s at present. I can t get not get in contact with the kidney doctor. A shot given to him in the knee spiked up his sugar level very high so that is out. Doctor: Hydrocodone does not affect the kidneys directly but it's metabolites accumulate in patient with kidney failure causing adverse reactions. So you need to use it cautiously with proper monitoring."
},
{
"id": 210192,
"tgt": "Suggest medication for manic episode",
"src": "Patient: My son has bipolar 1. He is currently being treated for a manic episode, at home, and he normally take Lithium and Seroquel. His doctor has recently increased his Lithium from 1200mg/d to 1500mg/d (his level was only .6 prior to his mania), and increased his Seroquel to 800mgxr and 200mg short-acting p r n, up to BID, and added Epival 750mg BID. he has had severe episodes of vomiting in the last 2 days, with resulting severe petechiae , and facial bruising. Could this be due to drug toxicity, and does he need to go to ER? Doctor: HiThanks for using healthcare magicYes, severe petechiae is due to drug over dose or toxicity. Sometime, due to vomiting, severe lithium level increases and it lead to blood cell damage. That may be the reason in your son. Better to go to emergency as early as possible and get his serum lithium level done. He needs immediate care.Thanks"
},
{
"id": 53101,
"tgt": "What causes pain at the bottom of rib cage and elevated liver enzyme levels?",
"src": "Patient: Hello. I recently went to the doctor with moderate to severe pain at the bottom of my rib cage. My doctor ran some blood tests and found that my liver enzymes were twice the normal amount (I m not sure of the actual number, I am waiting to here back from the nurse) and my d-dimer was 550 or 5.5, I m not sure which it is (I had a double pulmonary embolism in 2009). I called the doctor to get my ultrasound report and the nurse told me that there was nothing new (I have 3 cysts somewhere, I am assuming on my gallbladder, but she did not say) and asked me if the stomach medicine the doctor gave me was helping. I told her no and she said she was going to call back the doctor. I am really worried that she will put me off like she has before and this pain will not go away. I am a 38-year old female who has hypothyroidism, fibromyalgia, and tachycardia that appears to cause my hypertension. I take Lopressor, levothroxine, and Lortab. What should I do and should I be worried? Doctor: Hello , thank you for your question ! Unfortunately i find it difficult to give you the best answer , because of the lack of information . You said your d-dimers are 5.5 or 550 mg/l . It is normal your d-dimers to be up to 0.5 mg/l . D-dimers are suggestive of trombosis and if you were my patient i would be really concerned about you ,if they are 5.5 . Pain in the upper part of the abdomen is suggestive of in first place of cholecystitis (infection of the gallblader with or without gallstones ) Ussually it produces nausea and vomiting . It can also be gastric ulcer or gastritis . If you have a hystory of gallblader infection , it is common to have infection of the pancreas , due to the connected lymp system . I strongly suggest you to see a doctor ( preferably Gastroenterologist ) for the final diagnose . You should undergo ultrasonography , blood tests and probably fibrogastroscopy . If you are not planning to see a doctor soon , you could start diet 1a (google it ) and take spasmolitics when in pain Hope that ould help you .Best regards!"
},
{
"id": 14842,
"tgt": "What causes itching rashes on the fingers after having Salmon fish?",
"src": "Patient: Hello Sir. I had Salmon fish for twice and after a day or two, rashes formed on my fingers and the fingers swelled. what could be the reason? the rashes formed a rough layer of skin and at times itchy. Please advice if I need to consult a doctor. Thank you Doctor: it is allergy to fishan allergy to protein can occur immediately or 48 hrs after use anti allergic tab apply anti allergic ointment over areaif very uncomfortable short course of steroids will help reduce swelling immediatelyeliminate that fish in future"
},
{
"id": 52138,
"tgt": "Colon xray other then with enima",
"src": "Patient: Is there another way to have colon xray other then with enima ? Doctor: Hi.. Depending on what the doctor are looking for other alternative methods for diagnosing a colon problems are colonoscopy, CT or MRI scans.."
},
{
"id": 85698,
"tgt": "How to stop taking nexitoforte?",
"src": "Patient: i am taking nexitoforte half a tablet daily for past few years i want to get rid of it now .please help .i iwas taking this medicine aLONG WITH TELECALM plus .i have been treated by a neurologist as i used to get a feeling that all the people are after my life and could not sleep Doctor: Hello, Nexito is a neuro drug and it has to be tapered slowly. Sudden stopping of drug may lead to unwanted complications. Consult a neurologist and he will direct you accordingly. Hope I have answered your query. Let me know if I can assist you further. Take care Regards, Dr Shinas Hussain, General & Family Physician"
},
{
"id": 14298,
"tgt": "What causes itchy rashes all over body that spread on scratching?",
"src": "Patient: I ve been breaking out in rashes over various parts of my body over the past four weeks. The rash is itchy and seems to spread if I scratch it. I have dry skin and had been taking extra vitamin D which I stopped taking about 8 days ago. I still have the rash and while it may get better at times, it gets worse at times too. I am a 55 year old female in good health but I do have sensitive skin. I use all perfume free products. Doctor: Hi, sometimes rash can be a histaminic reaction to something we eat. A course of anti allergic medication like Fexofenadine plus Omeprazole for a week will help--regards"
},
{
"id": 34026,
"tgt": "What causes numbness in the tongue and mouth ulcers?",
"src": "Patient: 3 months ago I was treated for an oral yeast infection after taking antibiotics. I took a pill for 7 days and on the 7th day I finally didn t have any ulcers or pain in my mouth. A few days later it started coming back. I then was put on a liquid medication that didn t do anything. I was very frustrated and different people told me I should take sovereign silver. I sprayed it in my mouth 3 times a day for weeks. I finally got tired of putting up with the symptoms and called my doctor again. He put me on the pill form again for 10 days. It did nothing. I still have a feeling of a film in my mouth, my tongue is numbish, it s whitish with red spots on the back, food doesn t taste right, and when I eat salty foods I get ulcers in my mouth. The ulcers haven t been as bad the last month. I had called my doctor and told him it never went away and he recommended an oral surgeon. Do you have any idea what s wrong? Doctor: Hello ,Thanks for consulting hcm Read your query numbness in tongue with feeling of film in mouth and mouth ulcers this can be due to fungal infection , deficiency to vitamin B , depapillation of papilla or due to stress induced . For this I will suggest you to consult oral surgeon for clinical examination of oral cavity and to get proper diagnosis . Inmeantime maintain proper oral hygiene and you can apply ointment on ulcer like Mucopain or candid gel by consulting with local doctor .Hope this will help you ."
},
{
"id": 207890,
"tgt": "What is the treatment for depression?",
"src": "Patient: hi..I am a resident of Haridwar, Uttranchal and wish to get my aunt treated for depression. Her medication has been going on since past 6 months but recently she has started 'trembling' a lot...her condition has severely deteriorated in last 10 days. she cannt even walk properly and tumbles often....its sad to see her in this state as she led an active life working as a teacher for 32 yrs. presently, she is 73. can you give me the contact no. of Dr. Vineet Gupta, Psychiatrist from Dehradun. Doctor: DearWe understand your concernsI went through your details. I suggest you not to worry much. You must understand the present condition do need psychiatric treatment. Dr. Vineet Gupta129, Adarsh Nagar, Near Hotel Surbhi, Dehradun Gpo, Dehradun - 248001 Mob: 9634432589, 9259297679If you require more of my help in this aspect, Please post a direct question to me in this website. Make sure that you include every minute details possible. I shall prescribe the needed psychotherapy techniques which should help you cure your condition further.Hope this answers your query. Available for further clarifications.Good luck."
},
{
"id": 166230,
"tgt": "What causes large welts on back?",
"src": "Patient: I was going to give my 4 year old son a bath...so took off his shirt to see 2 large welts on his back. At first, it look like someone hit him...but after looking more closely...they look like bites of some sort. They are quite large though...nothing like I have seen. I did have to pick him up from pre-school today with a stomache ache...could they be related? Doctor: hi, most likely these are insect bite. These are not related to stomach ache. insect bite results in itching, fever, and secondary infection. you can give anti allergic syrup containing hydroxyzine like syrup atarax to the child so that itcching can be relieved. an examination by doctor is necessary so that we may not miss any important finding. If there is secondary infections and antibiotic needs to be started. Take care."
},
{
"id": 189545,
"tgt": "Filling done for tooth and new crown, shooting pain in tooth. Relief from pain?",
"src": "Patient: Hello, last week My dentist did some work on my teeth . I ve got 2 fillings and new crown .But since that time I am waking up every night in pain . the pain shoots straight up to my ear. I don t know what to do now. And right now Iam on my 2 week vacation overseas and certainly don want to visit dentist in foreign country. Doctor: Hi, Thanks for asking the query, Pain in the filled tooth can be because of two reasons, There could be dental caries left beneath the restored tooth, when it reaches the pulp tissue causes inflammation of pulp known as pulpitis. Or there could be presence of high points if you feel pain at the time of chewing or other tooth movements. I would suggest you to visit a dentist and get an x-ray done, accordingly replace the filling with the Root canal treatment. Take tab combiflam 500mg twice for pain relief. Hope this helps out. Regards."
},
{
"id": 163860,
"tgt": "What causes swollen neck lymph nodes in a child?",
"src": "Patient: my son is 5 years old and has swollen lymphnodes in is neck and all are less then a centimeter and are soft and movable. My doctor has asked to get a chest xray and really has no definete reason or cause for them. He has told me that it should be nothing to worry about but i cannot help but worry but i do trust what my doctor says! My son does not complain of any pain and is a healthy spunky 5 year old boy, he does need some dental work done just waiting to get in to see dentist so i just wondered could this maybe have somthing to do with these swollen lymphnodes or just any answers from you at all, thank you Doctor: Hello. Thanks for asking on Healthcaremagic. I am a pathologist and deal with swollen lymph nodes in my patients daily. From your description it seems that the lymph nodes are small less than 1cm and soft and mobile. Usually most common causes are reactive lymphadenitis. In this case any infection in the area of drainage of the lymph nodes will lead to antigen being trapped in the nodes causing an immune response leading to enlargement. Usually it is not of any concern and subsides on its own. Causes could be as simple as dental caries, tonsillitis or pharyngitis, ear infections etc. Sometimes cause of reactive lymphadenitis is not found. Other causes could be a serious chest infection which is ruled out by x Ray in your case. Other rare causes could be a lymphoma or cancer in which lymph nodes are larger and harder in size and not mobile. All these diagnoses can be easily made with a fine needle aspiration cytology test in which a pathologist inserts a needle into the swelling and obtains few cells to be stained and seen under a microscope. However children might find the procedure scary and a bit painful and representative material might not come in small swellings in children. So I suggest you to wait for some time and if it doesn't resolve or grows in size then do a fine needle procedure. Thanks."
},
{
"id": 112721,
"tgt": "Neuropathy till feet, and screw from L2 to S1. History of failed lower back surgeries. Pain?",
"src": "Patient: Hello, my name is JoAnn Endress. I am 59 years old. I had 2 failed lower back surgeries in 1999 and 2000. I have neuropathy all of the way down to my feet. I have been falling a lot the past few weeks. I have rods and screws from L2 to S1. I have been seeing the same Pain Managemt Dr. for the past 10 years, I have been good and faithful. He has sent some records on D.V.D. form and old x-rays. I am running out of my pain meds. My pain has been getting worse. Doctor: Hi, JoannEndress, You are 59 years old, You had 2 surgeries on lower back, and both failed,had neuropathy, you have rods and screws from l2 to s1, you are on pain medication. You must be knoing that it is a must to bear the pain,you can try to minimise the pain.I to my patients prescribe gabapentine, methyl cobalamine, along with pain killers. Avoid potato, other tubers, tomato, egg, chicken, and sea foods. Practice meditation, minimal yoga, reading books, and hearing music. Thank you."
},
{
"id": 39201,
"tgt": "Suggest treatment for high fever",
"src": "Patient: Good day! I have a problem that is almost destroying me. I have high temperature for more than 1 week now and it doesn't seem to stop no matter what I do. I tried pills, i've been to hospital, pill through anus but the fever still comes back in a few hours - 37,6 usually. I can't even sleep because of the heat. In that first day I remember I masturbated and immediately after I started to have the fever that is bothering me even today. I had no temperature before I masturbated. Am I infected ? What should I do ? Could it be the urinary system ? Please answer me as I don't know what it is or what should I do. Doctor: Hello,you shouldn't be so worried since you were examined in the hospital. If you had something serious, I'm sure the colleague who examined you, would have told you so. You can take acetaminophen for the discomfort. Do not take it only when you have the fever but on a regular basis (every 4-6 hours) for as long as you feel sick. You haven't given enough data to suggest a possible diagnosis. Most low grade fever cases (like yours) are attributed to viral infections. Fortunately masturbation does not cause fever or our planet would had been a really hot place!I also suggest you consult your doctor for the fever in case you haven't seen signs of improvement already. Since you mentioned no other specific symptoms or signs suggesting any diagnosis would be risky.I hope you get well!"
},
{
"id": 133833,
"tgt": "What causes tingling in leg?",
"src": "Patient: I m having issues with my right leg. Pain in my right thigh when I stand and tingleing in my right shin below the knee. Doc did full body scan and found a benigh tumor on my right thigh. He couldn t figure out what tingleing was from, (it feelslike a thousand ants are on my lower leg.) Any thoughts? Doctor: hi,thank-you for providing the brief history of you.A thorough neuromuscular assessment is advised.As you are having tingling sensation in the leg, an MRI of the lumbar spine should be of sight and and NCV test too. Due to abnormal pressure on the nerve in the lumbar spine or anywhere in the course of the nerve the symptoms of numbness, tingling, pain etc is noticed.Once the nerve is been assessed the treatment will be directed towards the same with medication and physical therapy.regardsJay Indravadan Patel"
},
{
"id": 95046,
"tgt": "Pain on the lower side of the stomach post appendicitis operation, had complicated appendectomy with part of intestine excised. Which medicines can I take?",
"src": "Patient: My brother is 20 years old, he has got his appendicides operation done years ago, it was very complicated since it had already ruptured for five days then. A small part of the intestines was removed since they had been severely infected. Ever since he has been complaining of pains in the stomach . My question to you is.. What medicines should he take right away since its pain right now, the pain is felt in the lower side of the stomach, the portion what we call the abs. Doctor: Hi Its not advised to take a painkiller for abdominal pain without consulting a doctor. You should take your brother to a surgeon so that cause of pain can be diagnosed"
},
{
"id": 118979,
"tgt": "Low blood levels, weakness, short term memory loss, borderline diabetic, weak left side. Any ideas?",
"src": "Patient: hi iam 46 female been in hospital too weeks ago low blood levels they hospital was treating me for poss stroke had mri scan came back said no stroke iam very weak i have no strength but sleep i have short term memory loss butm my blood levels are up and down they said iam border line die bets iam on sick leave at the moment i dont feel well at all my blood level went down to 2.7 they could not give me a reason i still have a weak leftside of my body i dont know what wrong iam usuall very active person Doctor: Welcome to HCM.Complete body profile investigation is required to reach the perfect diagnosis.Otherwise short time memory,weakness and borderline diabetes suggest Vitamin deficiency,particularly vitamin B 12.Start intra muscular vitamin B 12 injection.Your diabetes should be diagnosed properly by investigation FBS,PPBS, and GTT.Eat good nutritious,high fiber diet,start routine exercise and yoga.consult your doctor for better medical management."
},
{
"id": 150519,
"tgt": "Taken meningitis vaccine, any side effects?",
"src": "Patient: Because of recent events in the news I got a meningitis vaccine today. I last got one 13 years ago when I was 18 and headed to college--MPSV4. The shot I got today was MCV4, produced after 2005.Question(s): Did this shot create some additional immunity? Did it increase my chance of contracting a neurological side effect such as Guillain\u2013Barr\u00e9 syndrome? Would it cause some slight arm throbbing in the forearm/below the location of the shot?Thanks! Doctor: Hi and thanks for the query, The meningitis vaccine you take certainly created some degree of immunity, against the commonest germ that causes bacteria in adults. However, it does not exclude the fact you could still get meningitis from other strains of bacteria. The frequency of getting meningitis also depends on the individual s immune system. Lack of a chronic disease that reduces body s immunity renders possibility of infectious meningitis lower. Guillain Barre syndrome is a rare condition, seen in less than 1/100.000 population, and is generally associated to gastrointestinal infections due to some bacteria like Campylobacter jejuni. This ascending neurologic disease has been described with some vaccines, but in this case, your possibility of getting this syndrome is exceptionally very very low. However, you can watch out for signs like weakness of tingling at the levels of the limbs, in case you experience any of these, do not hesitate to contact your primary care physician. The pain or swelling after a vaccine shot might take up to a few days to get back to normal depending on the individual in question. More severe symptoms might be observed in some people.In this case, analgesics to reduce pain or fever if presence, in addition to Non steroidal anti inflammatory drugs are usually sufficient. thanks and best regards, Luchuo, MD."
},
{
"id": 160658,
"tgt": "What causes coughing,sneezing and flat red circles on tongue?",
"src": "Patient: My son has been sick for 3 days now, I took him to the er yesterday for coughing and sneezing and while we was there I noticed a flat red circle on his tongue. The doctor said it was a viral infection but now there is around 7 on his tongue. What could this be and is it contagious? Doctor: Hi,Many viral upper respiratory infections are associated with oral lesions, and need not be worried about. Yes, they are contagious, and others, especially children in the household can get infected. Most of these are benign and will recover in 3-5 days, and no need or use of isolating them (infectivity will start before symptoms) from others.If the fever is not coming down by 3rd day, or if there are painful oral blisters turning into ulcers, kindly review with your doctor to rule out herpes stomatitis, that may need anti-viral treatment. Meanwhile continue paracetamol and give plenty of oral fluids.Hope I have answered your question. Let me know if I can assist you further. Regards, Dr. Muhammed Aslam T. K., Pediatrician"
},
{
"id": 125242,
"tgt": "What cause bilarteral bruises on my daughters knee?",
"src": "Patient: my 13 year old daughter has bilarteral bruises on her that came out of know where she doesnst remember hiting her knees, but she is a competative swimmer and that is the only activity she is doing at this time, she has no swelling ,redness or pain in the area should I take her to ER. Doctor: Hello, Bilateral bruises on the knees without any history of injury can be due to low platelet count or a bleeding disorder. A few blood tests are needed to rule out these causes. Hope I have answered your query. Let me know if I can assist you further. Regards, Dr. Praveen Tayal, Orthopaedic Surgeon"
},
{
"id": 210726,
"tgt": "What is the cure for my anxiety disorder?",
"src": "Patient: I have anxiety disorder.i feel to commit suicide. I have no hold in my life. I have two months baby. But, I don't feel any rush of love towards her. I don't like to do any work other than surfing. I almost hate everybody who are living with me. At present, I am doing my masters as part time in computer science. I felt panic about exams and also I can't study. Doctor: HiThank you for asking HCMYour problem is most likely a mood disorder.A counselling with psycologist is essential for this.Breathing exercise for relaxation will also benefit you.A proper follow up with psychiatrist is also needed.With regular medication and counselling you will be better.Wishing your smile to return faster.Let me know if you have any further query."
},
{
"id": 97822,
"tgt": "Blood test shows low blood count. Suggest natural treatment?",
"src": "Patient: Hi, I just went to my doctor and ordered a blood test, the results according to my Dr. tells me that I have low blood cells count, I do not have cancer (not that I know), not HIV or any other infection or illness at this time; also have a low HDL (good) cholesterol (26) How do I naturally treat these issues - vitamins, diet, herbs, supplements, etc.) Please advise! My email: YYYY@YYYY Doctor: Hi! Welcome to HCM! Your doctor has told you that you have low blood counts. Low blood counts can have many causes, including vitamin deficiencies, bleeding, and rare bone marrow diseases . If your blood tests are showing low blood cell counts then it is essential to see which of your blood cells are Low whether it is low red blood cell count, Low white blood cell count, Low platelet count or lowering of all 3 cell lines that is pancytopenia. Each of these cell type have a different function in body and their lowering and amount of lowering both are very important to find out the cause. As for low HDL Strategies for raising HDL-C levels can include various lifestyle and drug therapies, which should be tailored to individual patients. Among lifestyle interventions for raising HDL-C levels, aerobic exercise is probably the most important.. Moderate daily alcohol consumption raises HDL-C levels by 5% to 10%. can take vitamin supplements especially having Niacin. hope this will help you!"
},
{
"id": 188851,
"tgt": "Painful jaw line",
"src": "Patient: I have pain on my jaw line which is directly in line with a cavity I have in my tooth do you think is it related ? Doctor: Dear friend.Thanks for sharing your concern.Definitely, it is related.The pain due to tooth decay can be experienced in the jaw too.very deep cavities might cause abscess and spread in the alveolar bone causing periapical granuloma there.Therfore please do not ignore and get it examined and treated too.Hope it helps.Thanks.Take care."
},
{
"id": 216029,
"tgt": "How to relieve continuous pain in thumb?",
"src": "Patient: hi, been having pain in the thumb for last two weeks,took ibuprufen tabs for five days and applied diclo plus methyl saly acid ointment. nomhelp.started eaucalyptus oil massage plus hot fomentation. some relief. two weeks past.bany suggestion for a quickervrelief...sks Doctor: Hello and Welcome to \u2018Ask A Doctor\u2019 service. I have reviewed your query and here is my advice. As per your history, as you already takng medications and apply gel also,,, so if there is trauma then do x ray thumb and consult with orthopedician. you can take tab. aceclofenac plus chloraxizone after consultation. Take care. Hope I have answered your query. Let me know if I can assist you further. Regards, Dr. Shyam B. Kale"
},
{
"id": 19419,
"tgt": "Diagnosed with cardiomyapathy and renal cyst",
"src": "Patient: I ve been diagnosed w/cardiomyapathy..I am 47 year old female. I noticed heatflushes on my entire body..it s seems to happen when i feel my heart beating fast...even if I am sitting down. Also, I don t have total control over my bladder. I wet the bed twice this week and when I wake up, I have to urniate every 15 mins. The past few weeks I ve been experience extreme back pain. First in the lower back, now on my kneck and shoulder, the pain radiates down to my arm. I do have boldging discs in my neck and dealt with the discomfort, but this is different sever sharp pain. Also, had a few kidney infections in the recent past and on my right kidney a Catscan showed renel cyst that enlarged since last cat-scan. Please help with what ever knowledge you may know about this. I try to pay for your answer, but I don t have a credit card, can you still help? Doctor: welcome to hcm...ok i can help you...as you say that since last CAT scan the cyst has increased in size ..if it simple cyst as per CAT report it should be decompressed by needle and fluid should be examined .."
},
{
"id": 31699,
"tgt": "Can I get herpes simplex if sexual partner is tested negative?",
"src": "Patient: hey i was tested for herpes simplex 1 & 2 they came back positive firstly and then my second test showed negative for 1 & 2 but my lips are now peeling should i take another test to confirm and please note my sexual partner has tested negative for herpes and i have been with him 1 year now Doctor: Hi..Welcome to HEALTHCARE MAGIC..I have gone through your query and can understand your concern..As per your complain in case if you are tested negative for herpes in your second test and your husband is also clear ad does not have an infection then your chances of infection seems less until you are not exposed to infection with some other infected individual or infected things like glasses towels etc..The peeling of the lips can be due to chapped lips or can also be due to a condition known as Exfoliative Chelitis..I would however suggest you to consult a Dermatologist and get evaluated and he can advise you tests like repetition of herpes virus tests like antigen antibody reaction, viral culture while testing for exfoliatve chelitis that can be due to bacterial or fungal infection..In case of herpes you will be advised oral as well as topical antiviral medication like Acyclovir, Valacyclovir etc..In case of Exfoliative chelitis application of petroleum jelly and also Tacrolimus and Marigold ointments are found effective..Drinking plenty of water and in case of bacterial or fungal infection treating the same can help..Hope this information helps..Thanks and regards.Dr.Honey Nandwani Arora.."
},
{
"id": 93944,
"tgt": "Lower backache, cramps, stomach pain, pressure sensation, frequent urination. Cause?",
"src": "Patient: Hello I keep getting lower backache an cramps have started in my stomach about belly button level an they shoot down to my pelvis area I feel extreme pressure on the arch of my back which feels like it spreads upwards an just lately when I urinate they become more painful allso I cant pass wind as it puts toi much pressure on tge front of my stomach do you know what could be causing this please thankyou for your time. Doctor: Hello, thanks for posting your query to H.C.M. C. During urination if pain is there , it might due to urinary tract infection ( UTI). Stomach pain is also but it may happen a refer pain from U T I. Desire of defecation usually associated with U T I . So consult a doctor and get in U/S of whole abdomen to rule out any disease in effected area. Urine and blood examination will also highlight the disease. Good luck Dr. HET"
},
{
"id": 24652,
"tgt": "Pain in sternum common post bypass surgery?",
"src": "Patient: Had a triple bypass 6 months ago. I seem to be doing fine. I am affected by weather badly and still have a lot of movement and pain in my sternum when the weather changes. I also have a lot of pain in the upper portion of my right chest when I cough is this normal for 6 months. Doctor: Hello and thank you for using HCM.I carefully read your question and I understand your concern.I will try to explain you something and give you my opinion.After bypass surgery your sternum experience changes.It is an oppened bone that needs its time to close properly.Having pains during whether changes might be normal.Usaly this pains of the sternum needs 3-4 months to resolve, but this depends on different people.To be shure that this is nothing to worry about, if I was the treating doctor I would recommend a thorax rx to closely evaluate the sternum and sceletic situation.Only after this we can be shure that this pains are normal and will slowly disappear.Hope I was helpfull.Bestregards,Dr.Ervina"
},
{
"id": 221407,
"tgt": "What causes severe pain in ribs during 14th week of pregnancy?",
"src": "Patient: 36 pregnant, 14th week, first child. 6ft 51kg As my hips seemed to have expanded an ribs seemed to have moved, my question is: is it medically possible that a older women who has had her bones set since going though puberty, can experience pain to the extreeme levels from the shifting of ribs. If so what treatment, exercise or medication could assit thw bone in the shifting?. Cheers Doctor: hi, I understand your concern. * In pregnancy the pregnancy hormones cause softening of ligaments .. so pelvic & chest bones tend to expand due to development of fetus, overstretching of abdomen/ pelvis. This causes pain. - In case of ribs.. the abdominal stretching is excessive due to gasses/ indigestion/ constipation. in 9th month. - Similarly bony weakness due to calcium & vitamin D deficiency would increase the pain. * Though complete relief from pain will be after delivery of baby, Some of the abdominal distension can be reduced to get some relief by- - Taking small & multiple food servings, - taking sufficient fibers in diet, - drink lot of water ( 10-12 glasses ) in a day. Continue exercise in form of walk for 20-30 minutes in a day. This will facilitate food digestion/ excessive gas formation & bowel movements & offer some relief from pain. - Consult your doctor for need of calcium / vitamin D supplement. Thanks."
},
{
"id": 20901,
"tgt": "Does wearing magnet close to heart affect functioning of heart?",
"src": "Patient: Hello Doc, I wear an O-shaped magnet around my neck as a good luck charm, it is a particularly strong magnet as it can pull metals in from about 5 cm away. I hang it about a hand's length down from my where my heart is. People say the magnet can affect my heart beat/performance so I stopped wearing it. Do you know if it's true that magnets can affect the heart in this way and cause harm?Many thanks, Magnet-o Doctor: Hello,Magnet has no significant effect on normal heart, however if one has any device put in like pacemaker then it may be dangerous. Hope this helps you."
},
{
"id": 27287,
"tgt": "How can i increase my result?",
"src": "Patient: my first 'echo' gave me a result of 60,, which my doctor told me was good. However 3 weeks later it is 50-55. My doctor said this is still normal. (i have undergone chemo/radiotherapy for cancer) I have another echo in 7 weeks time. How can i increase my result please? Doctor: hello, I have gone through your query .Thanks for using HCM.Basically normal EF is above 55%.so figure of 55 or 60 is considered normal.In your case our goal is to look for side effect of chemo on EF.I think next assessment after few weeks will give you right information.if it is around 55 ,nothing to worry.My best wishesDr.Rajesh Teli,MD."
},
{
"id": 113318,
"tgt": "Pain in lower back and thigh. MRI shows herniation, annular tear. Surgery required?",
"src": "Patient: Hi , I am having pain in my lower back and lower thigh. I did MRI for lower area and the report says I have broad based herniation at L5-S1 with posterocentral annular tear causing mild impingement on thecal sac and bilateral adjacent nerve roots My doctor (physician) has asked me to concern a surgeon but I am afraid to go.. Do I have to go for surgery ?? How severe is this thing ? what precautions should I take and what could be the reason for this ? Doctor: Hi thanls for your question. Your having intervertebral disc herniation at L5 -S1 level with posterocentral annular tear causing mild impingment on thecal sac and bilateral adajacent nerve root.Most probably this condition could be due to mild trauma or improper postions while doing differnt jobs involving your back.Since herniation of disc is associated with annular tear and impingment on thecal sac and nerves on both side,this condition needs medical attentation,if left untreated can lead to further damage and even can cause paralysys of lower limbs.I would suggest you to consult an orthopediac surgeon or a neurosurgeon who may be able to tell you,this condition needs surgery or not.Mean while I Would suggest you to take proper bed rest,avoid all sort of jerks to your back and adopt proper position while doing minor jobs involving your back. Hope this answers your question."
},
{
"id": 132991,
"tgt": "Why do the arm and hand feel weak, tingly and sore after got electrocuted?",
"src": "Patient: My left hand just touched something on the back of my cable box and I began to get electrocuted (my whole hand and arm tingling and pins sensation). I didn t feel it anywhere else in my body beyond that. It lasted about 3 seconds before I could pull it away. my hand and arm feel weak and tingly now and a bit sore. Should I be concerned that I did damage? What should I do now? Doctor: hihope this msg finds u in good health.There doesnt seem to be any problem as this a routine complaint after getting electrocuted.its due to sensitivity of nervesu will b fine in about 3 weeksthanks. .god bless"
},
{
"id": 44847,
"tgt": "How to diagnose a blocked fallopian tube ? What is the best cure with surgery and without surgery ?",
"src": "Patient: Halo, doctor . How to know whether your fallobia tube blocked? If confirm that already blocked that what surgery need to do ? Or any suggestion without surgery ? Doctor: Hi ms.lee chan; welcome to HealthcareMagic To know that fallopian tubes of a female is blocked there are 2 methods:- 1-To do a HSG(Hysterosalphingography) that is to insert a dye through a canula in the uterus and when the dye is pushed X-ray is taken to see if the dye is coming out of the tubes ;it is done in a X-ray dept of a pathology. 2-To do a Laparoscopy from above that is abdomen and from vagina again a canula is inserted and dye is pushed and it can be directly seen by a doctor whether the dye is coming out of the tubes These test prove whether the tubes are open or closed and most of the time the tubes get opened when the dye is pushed if there is a simple block in it .For details regarding the treatment with or without surgery please discuss with your doctor. Thanks"
},
{
"id": 137146,
"tgt": "Suggest treatment for muscle spasms and knots in the abdomen",
"src": "Patient: I have been getting a muscle spasm/cramp/knot in my upper right abdomen for quite some time. It happens almost daily and not sure what the cause would be. It feels like it is being twisted into a tight knot and I have to lay flat in order for it to go away. It goes away after a few minutes. Any advice would be greatly appreciated. Thank you! Doctor: Hi there.You could be having muscle spasm due to bad posture at work and lying down. I suggest you apply hot water bottle to the affected area. Follow ergonomics at work and adopt good posture. Take Tab. Diclofenac 75mg + Thiocolchicoside twice a day after meal for 7 days for relief."
},
{
"id": 139485,
"tgt": "Suggest nutrilite product after analyzing MRI report of spine",
"src": "Patient: I am 32 years man, suffering from cervical spondylosis since 3 years.MRI report of my spine is : (1)POSTERIOR & BILATERAL UNCONVERTEBRAL(LEFT IS MORE THAN RIGHT) AT C6-C7 LEVEL INDENTING THE CORD & NARROWING LEFT NEURAL FORAMINAE WITH FOCAL SIGNAL ALTERATION AT C6-C7 LEVEL WITHIN THE CORD-CONSISTENT WITH COMPRESSIVE MYELOPATHY CHANGES.(2) POSTERIOR & BILATERAL UNCONVERTEBRAL OSTEOPHYTES AT C5-C6 LEVEL INDENTING THE CORD & NAROWING BILATERAL NEURAL FORAMINAE. I am not interested to surgery & also dont want to take alopathy medicines. Can you please suggest me any Nutrilite product which will be helpfull? Doctor: Hi, The report shows compression of spinal cord in the neck region. Nurtilite product will not be useful for this condition. Treatment needs medicines, cervical (neck) collar and physiotherapy. In some cases, surgery may also be needed. Please consult a Neurologist for detailed evaluation and advice. Hope I have answered your question. Let me know if I can assist you further. Regards, Dr. Sudhir Kumar, Neurologist"
},
{
"id": 200545,
"tgt": "What causes frequent spotting of blood?",
"src": "Patient: Hello, I found blood spots on my boxers 3 days ago. They have been happening repeatedly for a few days now. The penis in particular smells. Is this a temporary to permanent problem. Furthermore At which point shall i be worried?......I am walking fine, body temperature is fine. Doctor: Thanks for asking in healthcaremagic forum Spotting/bleeding is normal only in females not males. So, please visit a doctor for examination and further management you might have got infection over there. All the best."
},
{
"id": 46895,
"tgt": "What causes headache after dialysis?",
"src": "Patient: I am Kidney patient CRF since last two years. Presently undergoing dialysis twice a week. I am lady of age 47. My weight is 41 kg. BP remains below 140. It does not increase during interval of Dialysis.Urine output more than 1000ml per day and input is 1500ml. I feel hungry and take normal food. My question is whether I need to undergo dialysis twice a week or once a week. Because after dialysis I feel headache for next 12 hours. Doctor: Hello and welcome to HCM.As an Urologist,i can understand your concern.In CRF,maintainence of kidney function is,twice or thrice weekly dialysis.As your body weight is low,twice weekly dialysis is sufficient.Your blood routine,creatinine,electrolytes and B.P. should remain normal.After dialysis,headache is commonly due to anemia or electrolyte imbalance.Dehydration is another cause for headache.Maintain the intake-output chart.If you've any doubts,you may send the reports as a direct question.Dr.Matthew J. Mangat."
},
{
"id": 105745,
"tgt": "Allergy on head, lozisoft",
"src": "Patient: my daughter(5 months old) having some allergy on head.for that doctor suggest LOZISOFT.when ever apply that one.she have immediate cold .is it safe for my baby please tell me Doctor: Hello, Welcome to Healthcare Magic forum. Allergy only on head (I assume scalp) is not a very common condition.I assume that the doctor has provided you with a diagnosis like Atopic dermatitis.Infants having scaly, itchy lesions in scalp can also have a seborrhic dermatitis or some infection like fungal or bacterial. Lozisoft lotion is just a moisturiser containing liquid paraffin and aloevera and is safe for infants. It is usually given for dry skin conditions or atopic dermatitis. However if your baby is not responding well to it , she can possibly have an allergy to any of the ingredients or fragrance used in the lotion.Also if her scalp lesion is not resolving then please consult your doctor again for reviewing the diagnosis and providing an alternative medication. There are many similar emollients or moisturisers available of different brands like aquasoft, calosoft, nmf_e, etc Your doctor may prescribe an alternative for solving your problem. Wishing your daughter a speedy recovery. Regards,"
},
{
"id": 191708,
"tgt": "What causes persistent swelling in the left foot while on Insulin?",
"src": "Patient: hi,I got some swelling on my left foot..and it is almost a month now. iam diabetic on insulin.also take blood pressure tablets.the swelling goes for a while but a little and again it starts to swell.the pain is on top of the foot at times when I keep the foot down iget a pain on top but not always near the calf muscle kind of a cramping pain at times .I have neuropathy as well and some times the nerve pain comes near the ankle on the same foot sometimes looks like its pain in the bone.i am thonkimh every day it will go away.my questions are, my GP said my pulse are good,but I did not tell him about the pain ,he said if it becomes red you must see us.however it does not interfere with my sleep is it kind of a stres fracture,is it a clot or just the ligaments is it diabetic related Doctor: Hello,The swelling on left foot, do you recollect how it started? Is there any pain and difficulty in walking? Is there any change of shape in the foot? Have been a regular walker or doing any exercise causing constant impact? Is there any color change noticed on the skin? How is the sensation in the foot and sole? I need to know answers to all these questions. So my suggestion is: 1) Keep your blood glucose in strict control (FBGHope I have answered your query. Let me know if I can assist you further.Regards,Dr. Prabhakar Laxman Jathar"
},
{
"id": 64738,
"tgt": "Suggest treatment for a large lump on the head of a child",
"src": "Patient: My daughter hit her head two weeks ago and had three staples put in. Last week she had them removed. She just hit her head on a worsen chair in the exact same spot. There is now a huge knot under the scab. Is this just a second hematoma or should I be worried? Doctor: Hi,Dear,Thanks for the query to HCM .I studied your query in depth.I feel concerned about it.In my opinion your dagugher has-Post-Traumatic -Hematoma / or may be just a Boney Contusion with traumatic-induartion.I would treat it with -Tb-NSAIDs / if no reduction in 24-48 hrs. would advise Surgeons consultation and Xray Skull -who would advise- Antibiotics if fever occurs with cellulitis increases-suggesting-infection. Hope you would solve your query.Please contact again and I would love to attend your queries promptly.Have a good time."
},
{
"id": 42410,
"tgt": "What causes amenorrhea?",
"src": "Patient: I am 32 year old. 162 cm hight and 65 kg weight. i had surgical removal of a cyst in fron of my thyroid about 8 years ago. My problem now is that i did not get my period for about 8 months! i was under stress a i started my postgraduate studies 10 months ago and i am not sure if this could be a factor for my case. I am worry that i had problem in my thyroid or even something worse related to the missed period! what would be the worst of not getting the period that long? Doctor: hai, First don't get worried and stress out Because stress may also influence your menstrual cycle.As you got a history of thyroid cyst kindly go for a thyroid profile test because amenorrohea is common in of thyroid problem. as in certain condition premature menopause and certain drugs will also cause amenorrohea.Kindly visit a gynaecologist with your thyroid profile report for further treatment and management.Thank youhope i answered your query"
},
{
"id": 214894,
"tgt": "Breast wart, painless. Home remedy?",
"src": "Patient: i my 25 yrs old girl my query is i have a wart on my breast and its there since past 9 years i never felt any pain in it but is there any home remedy so that i can get rid of it Doctor: No"
},
{
"id": 44416,
"tgt": "Undergoing follicular study, low result for endometrial thickness, high value of follicle size, nil left ovary. Pregnancy chances?",
"src": "Patient: dr iam undergoing a follicular study test on the 11th day my follicle RT ovary is 15/15mm 15/17mm LT OVARY IS NIL ENDOMETRICAL THICKNESS IS 5MM and on 12th day it is 16/16 mm 18/22mm LT ovary nil ENDO thickness is 7 mm are they normal plz guide me iam 31 yrs old and completed 1 yr of marriage i want 2 have a baby plz guide me Doctor: Hi Welcome to HealthcareMagic Yes follicle in right ovary is good and matured. Endometrial thickness is also normal. Endo thickness above 7 mm is good for pregnancy to occur. Frequent ultrasound monitoring will be required to document ovulation and when it occurs you can make sexual contact. Period of 12 hrs after ovulation is most fertile. Take care."
},
{
"id": 117288,
"tgt": "What causes high level of potassium in the blood?",
"src": "Patient: My family doctor called and told me that my blood work came back and that my potassium level was high and that she wants me to go to the hospital to have blood taken at the lab there cause she will get the test back faster. What can make my potassium high and what can i do to make it go low. Doctor: Hello, Thnx to contact us. If I am your treating doctor I would like to advice you that having high pottasium is termed as hyperkalemia. There are many causes of hyperkalemia like: drug, addisons disease. excessive intake, redistribution of electrolyte, diabetic ketoacidosis. Because of lack of history in your question I am not able to answer the exact cause of your disease. In treatment part you should not take pottasium sparing drug, avoid food rich in pottasium, use resins which absorbs the pottasium from circulation, in severe cases there is requirement of dialysis. If you have anything else to ask please contact me. Thanx. Dr. Arun Tank"
},
{
"id": 84581,
"tgt": "Does skin-lite have side effects?",
"src": "Patient: Hi Doctor, I have been using SKIN-LITE for the past 4 months. And the result is really awesome. But please tell me if i could continue to use it?...Or if I stop using, will there be any side-effects ? Is it advisable to use SKIN-LITE continuously or not? Please advice Doctor: Hi,It is not advisable to use Skin-lite continuously without consulting a dermatologist. Skin-lite cream contains a potent steroid, prolonged use of which can result in thinning of the skin, telangectiasias (red spot on the skin), abnormal facial hair growth and pimples.Hope I have answered your query. Let me know if I can assist you further. Regards, Dr. Mohammed Taher Ali, General & Family Physician"
},
{
"id": 44353,
"tgt": "Enlarged right ovary, cysts seen in left ovary, retroverted uterus. Will this lead to infertility?",
"src": "Patient: I am rXXXXX 24 years old my right overy enlarged size it measures 5.47cm*4.20cm an echo filled cyst (1.8cm*1.4cm)is seen in it few other small cyst are seen. it is adherent to uteres.in my left overy few small cyst are seen . uterus is retroverted,measures 8.01cm*3.77cm.clearendometrial echo is identified no sol is noted pls tell what happen in my future days not something worng i am not married Doctor: Hi, Thanks for your query. The cyst is not of concern. It may be just a simple cyst which will disappear in next cycle. Small cysts are nothing but follicles (eggs) which are normal. Hence I suggest you enjoy life without thinking about anything. You are in perfect reproductive health. Regards, Dr. Mahesh Koregol. IVF & Infertility specialist."
},
{
"id": 83959,
"tgt": "What are the uses and side effects of diarich?",
"src": "Patient: MY wife is suffring from dibitic from 14 years she is 51 years old , she is a non insulin takes morning and evening one tablet, her sugar levels are fasting 134 . after food and after 1 1/2 hours 200. can she take this diarich, is there any side effects of this powder. by taking what is the benifite Doctor: Hi,Diarich is a high protein sucrose free and lactose free dietary food supplement is generally well tolerated by most of its users however it may cause flatulence, bloating, diarrhea, and skin allergy. Yes, it may be taken as a nutritional supplement.Hope I have answered your question. Let me know if I can assist you further. Regards, Dr. Mohammed Taher Ali, General & Family Physician"
},
{
"id": 26507,
"tgt": "Suggest treatment for fatigue and high blood pressure",
"src": "Patient: I am Gowri 46 years old female. Did TMT Max BP 180/80. Max HR 166bpm 95% of max predicted 174 bpm. Exercise time 7.01. Reason for termination : Fatigue. Kindly advice. Also my husband Venkataraman did TMT. Max HR 168 bpm 100% of max predicted 167 bpm. Max BP 150/8 Maximum workload 10.1 METS. Total Exercise time 8.00 mts. Reason for Termination : Max HR attained. Kindly advice. Doctor: Dear Gowri,Both you have good results.Nothing to worry, just follow healthy lifestyle.Wishing you good healthTake care"
},
{
"id": 221192,
"tgt": "Is it safe to take Teva-Amoxicillin during pregnancy?",
"src": "Patient: Im 8 weeks pregnant and i have had a really bad cough for 2 weeks. I recently went to a clinic and was prescribed teva-amoxicillian (250 mg) 4 tablets/day for 10 days. Im nervous about taking it, is it safe for my pregnancy this early? Doctor: Hello,Amoxicillin and its different forms have been categorized under category B by FDA. Animal studies have shown no teratogenecity while there are no well documented human studies. The medication may be used if benefits outweigh the risks. Please remember that a severe cough is equally bad for an early pregnancy due to the constant pressure exerted on the abdomen and pelvis due to the strain apart from the infection per se. If I were to treat you, I would suggest the medication. Please get further suggestions from your consultant. Hope this helps."
},
{
"id": 116430,
"tgt": "What is the treatment for rheumatism and high uric acid level?",
"src": "Patient: Hi I am a female recently my blood test resulted in W and also my uric acid level is high. Bleeding at the time of menopause is less, I put on weight recently to 58 kg from 49. My skin is itching near the anus is it due to this higher uric acid? Please advice for all these problem? Doctor: Hi, dearI have gone through your question. I can understand your concern. Your uric acid level is high. You should take low protein diet and xyloric acid. Anti inflammatory drug and steroids help in rhumatism. Consult your doctor and take treatment accordingly. Thanks for using health care magic. Wish you a very good health."
},
{
"id": 72121,
"tgt": "What causes pounding in chest and stomach areas?",
"src": "Patient: About 7 months ago I took an antibiotic call Zithromax for chest congestion. That cleared up fine. About a week or so after taking the medication I started getting a pounding in my chest and stomach area. I went to my regular doctor and had all kinds of tests (heart, xrays, blook, chest etc) and she said everything was fine. Well I still get the pounding. It does not interfer with my life, as I play golf, run my treadmill and sleep fine. I went to a chiropractor and it seems to lessen the instances of the pounding. When I do get it, sometimes I take one aspirin, or put the heating pad on my back, or do some stretches and it seems to go away in a while. Any suggestions as to what could be causing this. My regular doctor is not helping me at all. Doctor: Thanks for your question on Healthcare Magic.I can understand your concern. First of all no need to worry for heart diseases as your catdiac reports are normal.Your pounding sensations are improving by putting heating pad and taking aspirin. Aspirin is a good painkiller drug. So possibility of musculoskeletal pain is more likely. So follow these steps for better symptomatic relief. 1. Avoid movements causing pain. 2. Avoid heavyweight lifting and strenuous exercise.3. Apply warm water pad on affected areas of chest.4. Take simple painkiller like paracetamol or ibuprofen.5. Avoid stress and tension, be relax and calm.Don't worry, you will be alright with all these. Hope I have solved your query. I will be happy to help you further. Wish you good health. Thanks."
},
{
"id": 26567,
"tgt": "What causes pressure in the heart area?",
"src": "Patient: Burping to relieve mild pressure in heart area, like there is something there. Breathing fine. On high blood pressure mess, vasotec and triameterene. I have ASD, having echo Doppler tomorrow. Is this angina? I'm female 50, generally good health n fit. Doctor: Hello!Welcome and thank you for asking on HCM!I understand your concern and would like to explain that your symptoms are not related to any cardiac disorder (including angina). The symptoms you refer, may be related to a gastric disorder, like gastritis or dyspepsia. Anxiety could also mimic a similar symptomatology (due to phenomena of air swallowing related to anxiety).I recommend performing a fibrogastroscopy if the problem persists or is associated to heart burn.Meanwhile I would recommend not to worry about it and having a healthy diet, avoid spicy food and take plenty of water. Hope to have been helpful!Best regards, Dr. Iliri"
},
{
"id": 165240,
"tgt": "Is having intermittent chest pains common in children?",
"src": "Patient: MY DAUGHTER IS 12 YEARS OLD, SHE HAS CHEST PAINS PRETTY OFTEN. I VE TAKEN HER TO THE EMERGENCY ROOM AT THE CHILDREN HOSPITAL. WHILE THERE THEY PERFORMED AN X-RAY OF HER HEART FROM DIFFERENT ANGLES. THEY SAID EVERYTHING LOOKED FINE. THEY SAID IT IS COMMON FOR CHILDREN TO GET HIS PAIN AND TO TAKE ADVIL WHEN IT OCCURS. TODAY MY DAUGHTER SAID THE PAIN LASTED FOR ABOUT TWO HOURS. IS THIS STILL FALLING UNDER THE CATAGORY OF NORMAL FOR ALOT OF KIDS? Doctor: Hi...Thank you for consulting in Health Care magic.The symptoms you quote might fit into chest pain which needs evaluation. But chest pain if it's serious won't last like this for 1 hour without any complications. Keeping in mind the age and gender of the kid, I feel you should consider hyperventilation and other conversion reaction like symptoms. But I reiterate the fact that this is a diagnosis of exclusion after ruling out all other physical diagnoses.I am considering this diagnosis as the symptoms you quote do not fit perfectly into any known clinical syndrome. I suggest you see your paediatrician.Hope my answer was helpful for you. I am happy to help any time. Further clarifications and consultations on Health care magic are welcome. If you do not have any clarifications, you can close the discussion and rate the answer. Wish your kid good health.Regards - Dr. Sumanth MBBS., DCH., DNB (Paed).,"
},
{
"id": 48919,
"tgt": "What cause headaches,hives,itching and constant urination?",
"src": "Patient: This summer I had a 14x7 kidney stone removed. One month after that I formed three more kidney stones, labs and blood work r all normal. Now one of my kidney stones is 1cm and I have 5 more. They can't figure out why. I have headaches, hives, itch, urinate constently, tired, and I take flow Max to urinate. Doctor: It looks like you need a detailed workup. This should be ideally done after all your kidney stones are removed. The workup should have a test for parathyroid hormone if there is elevation if your blood calcium levels and some patients will benefit from getting a full metabolic work like a 24hour urine estimation for calcium, oxalate, citrate, creayine, uric acid and others. Once we have an idea what is the cause, the actual treatment for prevention of stones can be done."
},
{
"id": 3912,
"tgt": "What causes difficulty in conceiving?",
"src": "Patient: Hello, Its 3 an a half yrs since i am married at first we had no plans for a kid. but from last 4 months we are seriously planning for a kid but all negative results. We have undergone all the possible tests. I would like to know when could i get conceived. Doctor: HAI WELCOME TO HCMFIRST CHECK YOUR DAY 2 FSH,LH,TSH,PROLACTIN LEVELS AND TEST YOUR TUBAL PATENCY WITH SALINE HYSTEROSALPINGOGRAm.semen analysis of your husband should be done.if everything is normal ovulation induction with IUI will give you positive pregnancy.regards dr.vanithadevi"
},
{
"id": 205244,
"tgt": "What causes frequent mood swing?",
"src": "Patient: Hello. My husband was in the military retired after 20 years with 100% disability. He does function very well physically . He s been retired a little over 10 years and his mental health is concerning me very much. He does go to V.A. for his physical care such as high blood pressure ect. The last few years I have noticed and also at a rapid rate he is no longer the same person as his mood is constantly changing for the worst. W have been married 16 years and I no longer know him or do I know what to expect. He is angry most of the time for no reason I am aware of , at times I see him outdoors standing around just staring as if he s in another world. He makes very bad choices about almost everything also about our finances and to the point of loosing our home. Before he got like this we made our decisions and discussed everything together as husband and wife should with no problem. He has become a habitual liar I m sorry but this is the best way I know how to refer to it. It s as if he doesn t remember so he makes stories up to justify his actions. but he is in denial about having a mental problem and will not tell his doctors! There are times he gets so angry over what reason I don t know I fear he will snap and kill someone or me. He was always in control and didn t get angry was understanding , compassionate , caring , loving and fun to be around. He no longer is sociable , doesn t want to be around me or anyone and no one wants to be around him because of his behavior and or attitude. It s so heart breaking and I have no idea or what I can do to help. I am considering calling his P.C. at V.A. but I am afraid of his reaction if he found out .He was always a controlling man in the past but wasn t a problem with myself .He is 65 years old . He was 33 years old when he joined the Army. I don t know if that has anything to do with his condition or not but we need help. I would greatly appreciate any advice. Thank you for you time and I am looking forward to hearing from you. Respectfully, Shirley Matlock Doctor: Hi, i think your husband have depression because of his physical disability.the main reason is he had work in army where they teach that nothing is impossible, in real world because of his physical disability he didn't do some work which was not accepted by him, so he was depressed regarding his illness. start tab.Vantab dxt(50) 1-0-0 for 15 days."
},
{
"id": 168445,
"tgt": "Suggest treatment for a sore leg",
"src": "Patient: my three year old has had a sore leg for 2 days now...she has been limping and it seems to be her knee that is the problem. She can bend it, it is not swollen or bruised and she isn t in any sort of pain to make her cry, she just limps and seems to have trouble putting weight on that leg. Not sure if it s something we should go to the doctor for or not? Doctor: Hi. limping at this age definitely needs evaluation as a child of 2 years can't explain her pain or weakness. There can be multiple reasons for limping in a 2 years old child starting from local injury to rhumatological disorders to joint infections to neurological problems also. So I think it will be a wise decision to visit a pediatric rhumatologist/ orthopedician. Thanks"
},
{
"id": 219764,
"tgt": "How to cure sharp pain in lower stomach during urination?",
"src": "Patient: Hi Dr, I'm 22 weeks pregnant and for about 18 hours I've been feeling queasy in my stomach (nauseous) but I can't throw up. During my entire pregnancy I haven't had any morning sickness or nothing. Also, I woke up yesterday afternoon having to urinate and when I did there was a sharp pain in my lower right stomach. What might be wrong with me? Doctor: Hello dear,I understand your concern.In my opinion the nausea might be due to gastritis.The gastritis is routinely seen in pregnancy especially between 13-29 weeks.Take antacids.Avoid spicy and oily diet.Coming to pain in the right side of abdomen while urination,a urinary tract infection(UTI) should be ruled out.Take adequate fluids upto 3 litres per day to prevent UTI.If not subsiding kindly get a complete urine examination done to rule out UTI.Nothing to worry.It will subside.Hope this helps.Best regards....."
},
{
"id": 194623,
"tgt": "What causes white spots on the tip of the penis?",
"src": "Patient: Hi, I m a 24 yrs old male from chenna and workin in IT company. I have found some white spots on the tip and top layer of skin of my penis. No problems with that as of now. Please let me know whom to consult or do i need to go to the doctor??? Thanks Doctor: Hello, Do those white spots itch, burn or pain? Have you indulged any unprotected sexual activity in the near past? Please provide the image of the affected part. You can also consult a doctor in person. Hope I have answered your query. Let me know if I can assist you further. Regards, Dr. K. V. Anand, Psychologist"
},
{
"id": 211960,
"tgt": "Brain fog, depression, trouble concentrating, lightheadedness, confusion. Due to diet?",
"src": "Patient: my cholesterol has been under 150 for the last 4 yr, and I have a lot of brain fog, depression, trouble concentrating, lightheadedness, confusion, etc, even when sugar levels are 90 -120. I eat meat, eggs, fish am only 32. Any connection? no meds. Do vitamins or mineral supplements raise or lower cholesterol? when my total cholesterol was under 100 it was horrible! Its better now with a level of 144, the highest its been for four yr. How do you get it up if that number is a problem? blood pressure is 100-120 over 60-80. Doctor: Hello, Thanks for the query to H.C.M. Forum. There are so many diseases ,which are responsible for low lipids and so many disease which are due to low lipids. The diagnoses of lipids and apo C deficiency are established enzymatically by assaying trigyceride lipolytic activity in post heparin plasma.So please upload complete lipid profile reports . Without serum trigycerides reports it is not possible to diagnose the disease. Good luck. Dr. HET"
},
{
"id": 44276,
"tgt": "Trying to conceive. Prescribed Clomid and Duphaston. Should I take medication?",
"src": "Patient: hello dr.i am reshma 29yr old.i want my second issue.i am trying to conecive from last 8 month.my husband report is normal.i didnt have any test.my dr suggested me to take clomid and duphstain for three months.my periods are just delys for4 days from last 3 months.should i take the medicine or go for hormonal test first? Doctor: Hello. Thanks for writing to us. Since you have tried taking clomiphene and duphaston for last three months without any success, you need to consult your gynecologist who may consider increasing the dose of clomiphene along with follicular studies. I hope this information has been both informative and helpful for you. Regards, Dr. Rakhi Tayal drrakhitayal@gmail.com"
},
{
"id": 53121,
"tgt": "Suggest treatment for liver cirrhosis",
"src": "Patient: what is treatment for liver cirrhosis? hello Dr. i am a 31yr old male,ht-5.4,wt-80,at the age of 17 i was affected by jaiundce & recovered.as of now i was affected by typoid &hepatitis B,from past 4weeks i m havng symtoms of ascities&vomiting.INR value is 2.6&bilerubin -20 Doctor: Hi welcome to HMCTwo things can be done in your case. 1. Treatment of Hep B and see how much you can recover.2. Evaluation for Liver transplantation.Kindly followup with complete Liver function test and other investigation reports and detailed description of your symptoms for detailed analysis."
},
{
"id": 144876,
"tgt": "What causes pain around T12 and L5 region?",
"src": "Patient: Hi, 20 year old white male. Hurt back in work approx 6 months ago. Pain on specific movements since. Dull ache around T12 after sitting for more than 5 minutes. Achie pain around L5 region on sitting and left shin pins and needles after standing back up. Pain in L5 region in extension also. Single incident of bed wetting after particularly sore day in work. Bladder function returned to normal next day. Any advice greatly appreciated Doctor: Thank you for asking Healthcare majic. My name is Dr Ehsan Ullah & I have gone through your query.kindly get digital x-ray lumbo-sacral spine AP and Lateral view..u can also go for MRI lumbo-sacral spine and visit neurosurgeon for prolapsed disc or traumatic injury to spine at lumbar area..Meanwhile you can take good pain killers like brexin 30mg twice a day,Gabapentin 150mg twice a day...early visit to neurosurgeon with x-rays and MRI will be helpful to you.Hope this may help you. Let me know if anything not clear. Thanks."
},
{
"id": 88507,
"tgt": "What causes lower abdominal cramps,sore nipples and headaches?",
"src": "Patient: i had my ovulation the 27th of march 2011. on the 30th i started to feel some cramps on my lower belly first left side that right side. my nipples are sore, having some headache and i'm having it warm. i'm 37 years old. my obgyn prescribe me metformin and clomid. do you have an answer for me please? can i be pregnant ? Doctor: Hi! Good evening. I am Dr Shareef answering your query. If you are sexually active, yes, it could be a pregnancy. To confirm or rule this out, I would advise you for a serum HCG test, and an ultrasound of abdomen if need be to rule out other pelvic pathology. Further management would depend on the reports of these investigations.I hope this information would help you in discussing with your family physician/treating doctor in further management of your problem. Please do not hesitate to ask in case of any further doubts.Thanks for choosing health care magic to clear doubts on your health problems. I wish you an early recovery. Dr Shareef."
},
{
"id": 24095,
"tgt": "What causes a sudden rise in the heart beat?",
"src": "Patient: I ve recently become concerned about my heart. I m a 21 year old, 5 8, 175 pound woman and consider myself decently healthy. I eat well when I can although I do make poor food choices. I am also not very active as I work at a call centre and sit at a desk all day. I don t drink as much water as I should. I have also been smoking marijuana consistently for 6 years, mostly through a glass bong with at least 1 perculator. Within the last 2 years I have been smoking something called poppers , which is a layer of tobacco from a cigarette with a layer of marijuana smoked through a bong. It creates a strong headrush and intense feelings of relaxation, causing me to want to lay myself down as if it was overwhelming my body like heroin (I have never done heroin but I have seen people react that way to it before) I know it is very bad for me because the carcinogens from smoke is obviously bad, especially cigarette tabacco that s mixed with marijuana and only filtered through water, but I want to know what it s doing to my organs and body functions in the long run. The problem is that I don t think there have been many studies on poppers because it is a relatively new and still underground method of smoking marijuana so its hard to find reliable information. Recently my heart has been occasionally feeling strange. I will be relaxed either sitting or standing and breathing and I will breathe out slowly, then feel my heart almost lump up and beat in my throat, it scares me and I take a deep breath in to counteract it, and for a second I almost feel like its going to stop beating. Then its all good again. What s happening to my organs and why is my heart doing this? Thank you for taking the time to read this and reply! Doctor: Hello , The heart beats you are feeling is because of the ill effects of poppers on your heart as the tobacco would make the heart pump stronger and faster giving you the symptoms you have described. As you have truly said that the effects op poppers haven't been studied however all the ill effects of tobacco applies here ranging from hypertension , stroke , heart issues , lung damage to various cancers . Also the effect is compounded by weed . It's not wise to use poppers as the sudden surge in body metabolic is not good . Regards"
},
{
"id": 102170,
"tgt": "Is it normal to get exhausted during allergy flare ups?",
"src": "Patient: I had raw almonds last night, and I have never had a problem with them, however, I noticed my lips started to tingle and my throat began to get sore. I noticed it an took benedryl (since I do have other food allergies, I new the symptoms). I then slept for the entire next day...as in 24 hours. Every time I have an allergy flair up, I am usually exhausted. Is that normal for someone to experience? Doctor: Hello,Welcome to HCM,The allergic reaction means when you are exposed to the triggering factor like dust, mite, pollen and food to which your allergic, there will be release of mediators of allergy like histamines which will produce all the symptoms of allergic reaction.As you are known case of allergy these are the routine phenomenon to occur in your body, when these symptoms occurs it will make you exhausted and tired. If you test your blood for absolute eosinophil count during the allergic reaction there will be increase in the count.The medicine like benedryl will help you to control the symptoms but it is not going to prevent further occurrence of allergic symptoms.So I would suggest you to undergo skin prick test for the most prevalent allergens and identify the allergen causing all these symptoms and then you can avoid the allergen or you can take immunotherapy.Thank you."
},
{
"id": 86696,
"tgt": "What causes abdominal pain and stripped stomach lining?",
"src": "Patient: Hi, several weeks ago I had bad abdominal pain, I had been through some stress and my doctor said as a result the lining of my stomach had been stripped away. I took nexium for 6 days. I have had dirrohea a couple of times in the last two weeks but yesterday got really bad pains again the same as initially when he said about the lining on my stomach. I also had 4 bowel movements yesterday and when I wiped there was some blood, not in the stool just on the paper. Is this in relation to the same problem or could it be something else? Doctor: Hi! Good afternoon. I am Dr Shareef answering your query. If I were your doctor, I would advise you to stop eating from outside if you do. Out side food might cause intestinal infections specially amebiasis in some regions which might give rise to gaseous distension and pain abdomen on the right side due to collitis. For this I would advise you for a routine stool test for ova and cyst, and treat it if positive. Please carry on taking Nexium as before. In case some kind of infection comes out in the stool test, then further appropriate management could be done.I hope this information would help you in discussing with your family physician/treating doctor in further management of your problem. Please do not hesitate to ask in case of any further doubts.Thanks for choosing health care magic to clear doubts on your health problems. I wish you an early recovery. Dr Shareef."
},
{
"id": 188342,
"tgt": "Child diagnosed with swollen gums, fever. Prescribed Advil, Ibuprofen, Septipan. Not eating food. Normal occurrence?",
"src": "Patient: Hi Dr Grief, my 3 year old daughter has been diagnosed with swollen gums as the cause of her fever. She was given Advil Ibuprofen suspension and Septipan oral suspension. Last night however she has 103.64 temp. This morning however, her fever was down to 99.5. She has not been eating her food [soup] since this morning. Is this a normal occurence? Thank you. Doctor: HIThank for asking to HCMWhen child have fever say above 10 degree F, child may not be remained playful, and the appetite can be lost, till it get alright, still we can encourage the kid for eating, offer the most liked food, keep the patient well hydrated, if it is viral fever then it may take, 4 days to week for getting alright, meanwhile you can continue the prescribed medicine, if it does not got alright then the blood work up would be necessary, for further management, and diagnosis, take care of your daughter, have good day."
},
{
"id": 195735,
"tgt": "How to overcome the habit of masturbation?",
"src": "Patient: Im age is 19 I m relising more sprem day by day over 3 years so now I feeling very lazzi and sometimes. My hands are not supporting with me on that time my talk whre different. What precautions. Should. I take to stop tho and getting weekless body functions Doctor: Hello and Welcome to \u2018Ask A Doctor\u2019 service. I have reviewed your query and here is my advice. As such masturbation safe practice and not leading any major health problem. But excess masturbation in few patient can lead groin discomfort and weakness. To overcome this habit try yoga and meditation. Regular exercise will be beneficial. Make short term and long term goals. Try not to be alone in room. Meet with friends and relatives. Focus on study in this young age. Avoid porn films and magazines. Hope I have answered your query. Let me know if I can assist you further."
},
{
"id": 62167,
"tgt": "What causes sore lump on jawline?",
"src": "Patient: I have a small hard lump on my right jawline about the size of a pea. I had a dentist xray it and they are sending me to a oral surgion...it s not black on the xrays but I am getting worried because it s getting more sore every day. Specially at night when I sleep....throbs. I am worried it s cancer Doctor: Hi Dear,Welcome to HCM.Understanding your concern. As per your query you have sore lump on jawline which could be due to blocked duct of salivary gland. It could be due to infected lymph node . Lymph nodes get infected due to bacterial or viral infection. It could be due to dental abscess . When infection or pus get accumulated at the root tip. You should maintain dental hygiene by brushing and flossing. I would suggest you to consult dentist for proper examination. Doctor may order X-ray and blood test to confirm the diagnosis. Doctor may prescribe antibiotics like amoxicillin for at least 10 days or perform minor surgery to remove salivary gland .Hope your concern has been resolved.Get Well Soon.Best Wishes,Dr. Harry Maheshwari"
},
{
"id": 11536,
"tgt": "Suggest treatment for tanned skin and acne scars",
"src": "Patient: Hello doctor..! Is it good to go on salycilic peels and yellow peels? And my doctor suggesting me to go with glutathione capsules with vitamin c supplements or glutathione injections as another option.My main complaints.- I lost my fairness.. im really tanned and im browm now. I ve acne scars and pores in my face. Can u pleas help me on this! Doctor: Hi,Thanks for writing to us.Whatever treatment suggestions your doctor has offered are satisfactory. However personally I do not advocate use of glutathione. My take on your skin concern would be treat your uneven skin tone, acne scars and pores. Please do not go with skin lightening treatment as results achieved would be temporary and skin color per se cannot be changed.Salicylic acid and retinol( yellow peels) would help you to reduce tanning, oiliness,acne marks, and pores( to some extent). If you have too many scars, go with microdermabrasion or dermarollers. Use a good quality sunscreen to maintain results. Do not go with glutathione injections etc.Take care"
},
{
"id": 94136,
"tgt": "Had a laparoscopic appendectomy. Tightness in the navel with a hard spot. History of ovarian cyst. Pain persisting. What to do?",
"src": "Patient: I had an emergency laparoscopic appendectomy 5 weeks ago. A few weeks later I felt a tightness on the left side of my navel and also felt a small hard spot there. I assumed it was scar tissue. I had a prior laparoscopic surgery in 1993 for an ovarian cyst . The last week the pain has increased and is always present. Now it s around the entire navel. It s not excruciating pain as was the appendicitis , but it s very tender and similar to pain right after surgery. I have no fever or chills and no redness or leakage at incision site. The pain worsens if I m more active - bending, walking, urinating, or if anything presses against it. Thank you for your time. Doctor: Hi Thanks for writing in I would suggest you go for an ultrasound as there are chances that either omentum or intestine is herniating through the port incision Hope this helps Do write back in case of concern Wishing you a speedy recovery"
},
{
"id": 170428,
"tgt": "Suggest treatment for fever and cold in a child",
"src": "Patient: my daughter 2yr 9month was suffering from fever ..twice the fever came to be 102, so we gave crocin syrup but the temp only fall down to 100.4 only. then after sometime i cghnaged to Meftal - p and the temp fall down to 98.4 and since then it has been 10hrs or so the temp didnt came up....Please suggest did i took the right decesion. She was also having cold, so i have gave her Honeytus sysrup and had applied Vicks on her chest and throat, back. I am finding her that she is fine today. (Please note - Meftal - P since given has been 10hrs till now, and no fever) Doctor: Hi,You are on right line of treatment.It seems that she might be having viral fever.Continue giving Crocin or Meftal-p syrup as and when required for fever.But important is to give her plenty of water and liquids and keep her well hydrated so that there will be enough perspiration to bring fever down after giving medicine for fever.Ok and take care."
},
{
"id": 165757,
"tgt": "What causes yellow mucus discharge from nose?",
"src": "Patient: Hi, may I answer your health queries right now ? Please type your query here...I have a 10 month old and for the past three weeks when she has a thick yellow nasal discharge. Mostly in the morning on waking. No fever, cough or chest involvement. Any suggestions? Doctor: Hi...Thank you for consulting in Health Care magic.Cough and cold are viral 95% of the times in children. For cold you can use anti-allergics like cetirizine and for nose block, saline nasal decongestants will do. Paracetamol can be given in the dose of 15mg/kg/dose (max ceiling dose 500mg) every 4-6th hourly, that too only if fever is more than 100F. I suggest not using combination medicines for fever, especially with Paracetamol.For cold you can use Cetrizine at 0.25mg/kg/dose every 12 hourly for 3 days.For nasal block, plain saline nasal drops will do, every 4-6th hourly to relive nasal congestion.Hope my answer was helpful for you. I am happy to help any time. Further clarifications and consultations on Health care magic are welcome. If you do not have any clarifications, you can close the discussion and rate the answer. Wish your kid good health.Dr. Sumanth MBBS., DCH., DNB (Paed).,"
},
{
"id": 200971,
"tgt": "What does white penile discharge indicate?",
"src": "Patient: My boyfriend recently started having white penial discharge. Was treated over a week ago for most std at emergency room a week later its still occuring. We havent been sexually active in over two weeks an neither of us have sex with someone else this happened before an he was treated for stds though testing was negative then as we are still waiting on these results. What else could it be ? Doctor: Thanks for asking in healthcaremagic forumIn short: Penile discharge indicates infectionExplanation: Discharge from penis suggest infection. Usually STD's can recur if under treated or reinfection from the partner. So, it is better that both of you undergo investigation and treatment together for better results. Hope this will help you."
},
{
"id": 151001,
"tgt": "Feeling dizzy, getting headache and unable to speak properly. Required treatment?",
"src": "Patient: Hi, my bf is 25 years old; this afternoon while working he started to get a really bad headache , and dizziness when he tried to talk he got really confused and he couldn t remember any of the words he was about to say. Right now he has to talkm really slow so that he doesn t get all confused again, he understands everything he is told but he can t form sentences easily. Thank you. Amy Matthews Doctor: Hello, I am a neurologist and i thank you for posting this question in HCM Let me tell you that any particular medical problem in medicine can be due to multiple causes and treatment and outcome depends upon the cause. Further in neuromedicine, other things which can help a physician get the right diagnosis are the sequential evolution of the symptoms and the detailed neurological assessment by a neurologist. All these things will hep to get the correct diagnosis. Back to your question, you have listed some symptoms, based upon these i think that you need an urgent neurological examination to see if you have neurological deficits. The second step will be to get a scan done and treat it accordingly rather than simple assumption. Hope this will help you"
},
{
"id": 119384,
"tgt": "Where can clexane injections be taken apart from the stomach region ?",
"src": "Patient: Hey I am a 20 year old female, I have tombosis and I am using clexane. Is there anywhere else except my tummy where I can inject it?? Doctor: Dear Anon, You can take these injections on your outer thigh or outer mid-arm subcutaneously. The effects of the injection does not become less. Take care"
},
{
"id": 94696,
"tgt": "Metallic taste in mouth, fatigue, nausea, abdominal pain, dizziness, loss of focus, lack of sleep. Protein in urine. Suggestions?",
"src": "Patient: metallic taste in mouth at night, fatigue , nausea, abdominal pain , dizziness, loss of focus, poor sleep pattern. work in high stress environment, tested pos for protein in urine (still have darkish coloring). 34 year old female taking mononessa. irregular periods with out mononessa or similar meds. also pain in upper right quadrant of torso. has been diagnosed with sludge in gall bladder, and pain around belt line. liver and kidney function tests normal, thyroid test normal. Doctor: What is your blood pressure?"
},
{
"id": 128128,
"tgt": "What causes left arm pain, sickness and vomiting?",
"src": "Patient: Hi I have pain in my left arm that comes n goes once or twice a day. It happens at anytime, while I m sitting or doing something, even now I was feeling sick n vomitted, after a few minutes I had sharp pain in my left arm. The pain is enough that it makes me grab my arm when it happens.... Can u tell me why? Doctor: Hello, I have studied your case.Due to compression of nerve root there can be tingling numbness in your arm and fingers pain associated with it.I will advise you to MRI cervical spine for better diagnosis.It can be due to various causes like sudden jerk to spine, due to facet arthritis, degenerative.For these symptoms analgesic and neurotropic medication can be started.Till time, avoid lifting weights, Sit with support to back. You can consult physiotherapist for help.Physiotherapy like ultrasound and interferential therapy will give quick relief.I will advise to check your vit B12 and vit D3 level.Local injection may help if pain remains persistent.Hope this answers your query. If you have additional questions or follow up queries then please do not hesitate in writing to us. I will be happy to answer your queries. If you are satisfied with answer do not hesitate to rate this answer at end of discussion. Wishing you good health.Take care."
},
{
"id": 111258,
"tgt": "What is causing stiffness in back?",
"src": "Patient: Hi, I work in a software company and my job requires me to sit a lot and my posture is very bad which i am trying to improve,however i make sure i do lot of physical activity like aerobics etc,however i have stopped from 2 months. I am 172cm tall and my weight is 68kg. I used to go to gym before and just went for a month.Suddenly I started feeling stiffness and uneasiness/tiredness in my back almost 1 and half months back and it still continues.Due to this i have stopped all my exercises and now days i just walk,that also slow walk. I dont know whether this feeling of stiffness is same thing what we call back pain? I love to go to aerobics and gym. But because of this i have to stop everything. Sir is this stiffness which i feel same as back pain? And is it curable and if yes how much time does it takes? Can i go out for exercise in normal way or i have to take precaution for life long thing. I m very worried about my back as if i have back issues in this age ,then i dont know how my life will go. Also 1 more point sir, This pain or stiffness increases as the day goes by, i mean in morning when i get up,i dont have any pain or stiffness,but as day goes by and i work ,it increases . Should i go for MRI scan? and also can i do gym and is it possible that this pain is because of gym exercises. Doctor: Hello,I had gone through the case and found that you did not mention the exact location like upper back or lower back.So go for MRI of whole spine and Vitamin D3 test.After that take proper treatment and Vitamin D3 once a week.It might be cervical or lumbar spondylitis.According to exact cause do exercise.Hope my answer will be effective for you.Thanks"
},
{
"id": 220389,
"tgt": "What causes stabbing pains in the abdomen with back ache?",
"src": "Patient: i had an ectopic pregnancy resolved by laparoscopic salpingectomy on the 16th march. we have been having unprotected sexual intercourse since 26th april. is it possible i could have another ectopic as tonight i am suffering from the same stabbing pains in my side and abdomen and aching in my back. it is just the same as before but is it possible to be pregnant again so soon and get symptoms so soon. i havent had a period since the ectopic. Doctor: Hello dear,I understand your concern.In my opinion the chances of ectopic pregnancy might be less.The recurrence of ectopic in furthur pregnancy is 10-20%.The ectopic pregnancy usually presents with pain abdomen with or without spotting.The urine pregnancy test helps in diagnosing the condition.It is usually weakly positive in case of ectopic pregnancy.And ultrasound confirms the diagnosis .If the urine pregnancy test is negative and ultrasound is normal then there can not be ectopic.So relax and avoid stress.I would like you to consult a doctor and get the investigations done.Nothing to worry.If there is no pregnancy giving a gap of 4-6 months and planning for next pregnancy would be a good option.Hope this helps.Best regards..."
},
{
"id": 223133,
"tgt": "Is it a matter of concern if the Nexplanon implant in the arm has moved by 2cm?",
"src": "Patient: Yesterday I got my nexplanon insertet, and today when I removed the bandage around the arm, I notices that the implant has moved around 2 cm up from the insertion point. I can t seem to feel the upper end of the implant, but can see and feel the lower end. Should I be worried about it going in my arm? I can feel probably 3 out of the 4 cm length of the implant. It hurts a bit, but I think it s just from the regular bruising. Not sure if I should check this out or just let it heal (unless the pain doesn t go away). Doctor: Hello and Welcome to \u2018Ask A Doctor\u2019 service. I have reviewed your query and here is my advice. If the Nexplanon has moved from its position, then there is a need to get it examined by your doctor to confirm that it is in normal position. Hope you found the answer helpful. Regards Dr Deepti Verma"
},
{
"id": 216438,
"tgt": "Suggest remedy for pain after bursting of Bartholin cyst",
"src": "Patient: 4 days ago I noticed I had a bartholin cyst it burst two days ago and is still draining which is normal for me. I ve gotten these often since I was about 17 (now 22). I went about my normal routine but it seems to be a but more painful then normal I m on antibiotics but I was wondering if antiboitic ointment would hurt or help if I put it on? Doctor: you can use antibiotic ointment pervaginally...it will be helpful....you have not mentioned which antibiotic you are taking...if you are not having antibiotics then take Tab Levofloxacin 750mg ODointment clindamycin for per vaginal applicationtake Sitz bath (sit in a bath tub containing warm water )for around 15 min dailynd don't have sexual intercourse until it heal"
},
{
"id": 111785,
"tgt": "How to treat back pain in my upper back due to c-section childbirth?",
"src": "Patient: I have had 3 repeated c sections after the 2nd one I have been having really back pain in my upper back on the left side...an after my 3rd it has become worse I cant stand nor sit for long periods of time an I can hold my baby wih the left side w/out it hurting...what is this? Doctor: Hello,I have gone through your query and understand your concern.You are experiencing one of the common problem of back pain after Cesarean section.One thing that you should know is that there are many back pain remedies that available to help you with any back pain that you might experience! Back pain is honestly one of the worse things in the world to experience for any amount of time and all you want to do is get rid of it. Back pain after Cesarean can be caused by the epidural that was in your spine during the surgery.Back pain after Cesarean can also caused by a muscle imbalance of your abdominal muscles during pregnancy being stretched to their limit and our back muscles being constantly tight to compensate for our weak abdominal muscles. A muscle imbalance such as this can throw out spine and pelvis out of whack, thus causing constant back pain.Alternating hot and cold remedies can also be helpful. This is something simple that you can do right in the comfort of your own home. For example, lay on a heating pad for no more than an hour - and usually half an hour will do - and then replace it with an ice pack.One of the greatest things in the world for back pain as well is going to be hot baths. Just throwing in some Magnesium sulfate salt and soaking in the tub for a while will definitely help you a great deal. And finally, Mild exercise and Physiotherapy is great for back pain after Cesarean. Physiotherapy strengthens abdominal muscles while stretching the back muscles. Physiotherapy achieves this with a series of gentle exercises. Thus, making it the perfect solution for back pain after Cesarean.Hope this helps.Please write back for further queries.Wishing you good health."
},
{
"id": 119482,
"tgt": "What is the painless,recurrent bruise on my upper thigh?",
"src": "Patient: I have a fairly large bruise on my upper right thigh that continues to show up for no real reason. It doesn t hurt at all while the bruise is there, however it does hurt a day or two prior to the bruise showing on the skin. I am not sure why the bruise keeps coming back to the same spot & it seems to be bigger this time then the last time. It takes about a week for it to go away & usually will not come back for weeks & then for no reason just comes back. Doctor: Hello, Such painless bruises generally occur due to:-Clotting disorder in blood-Injury-Some medications like aspirin.I shall advise you first to check about any thing from which you are unknowingly getting injured and then check for any medicine side effects. Take care. Hope I have answered your question. Let me know if I can assist you further. Regards, Dr. Mukesh Tiwari, Orthopedic Surgeon"
},
{
"id": 118111,
"tgt": "How to increase hemoglobin levels?",
"src": "Patient: Dear Dr. Robert My mother is 65 yrs with diabetes and hypertension for more than 20 yrs. SHe has had a history of Gastric bleeding n ulcers wherein she had to undergo bloo wd transfusion and only then she recovered. She had taken a 3 day course of piroxicam 0.d along with rabeprazole 20 mg. Her hemoglobinas 12 last month and yesterday she had a vomitted once. we immediately checked her Hemo and found it o be at 6.9. Our family physician prescribed Rabeprazole 20 twice and SUCRAFIL-O gel: sucralfate 500 mg, oxetacaine 10 mg/5 mL t.i.d What should we do to get her hemo dropping further and what should she take to increase the back to normalcy. please suggest. Doctor: Thanks for contacting HCM. Your mother's bleeding ulcers have to be treated because if not treated properly the bleeding will continue and her hemoglobin will continue to fall. So first priority is to treat her ulcers as most of medications for ulcer interfere with iron supplements. But if the haemoglobin continues to fall she can be given intramuscular iron injections or iron through i.v. drip. Regards"
},
{
"id": 2689,
"tgt": "Will the medication increase the chances of conception?",
"src": "Patient: I am currently taking metformin, I started taken it November 1, 2013. I was taken 2 pills a day. On 3/12/14, my doctor started me taken 2 pills twice a day. I take for PSOS, will taking this medicine possibly increase my chances of conception. I have 1 tubal pregnancy and suffered 2 miscarriages. Doctor: Hi,I understand your concern. You have PCOS and history of miscarriage. I would suggest to consult gynecologist and undergo USG scan, ovarian follicle study and take proper treatment under advise. - Metformin: It will help in induction of ovulation and decrease insulin resistance. - You can ask following medicine also to increase fertility chance: Progesterone pill in second half, Tablet Clomiphene citrate from day 3rd to 8th. Tablet Ovacre. Avoid stress, take healthy diet (avoid excessive sweet, take small frequent meal rather than large meal), do regular exercise and reduce 10-12% body weight, drink plenty of water and maintain proper pelvic hygiene.Hope this may help you. Contact further if follow up needed.Best regards,Dr. Sagar"
},
{
"id": 145934,
"tgt": "What causes tingling and numbness on fingers after wrapping hair band on wrist?",
"src": "Patient: I went to sleep with a hair band on my wrist. I actually wore it all day yesterday and overnight. When I woke up this morning my hand was tingling and numb like it was asleep and the hair band was very tightly indented into my wrist. I removed it and still 12 hours later my thumb, index finger, and middle finger, as well as half of my hand and wrist are still partially numb and tingly. the indent from the hair band is gone, but a red welt is still visible most of the way around my wrist... Will the feeling come back to my hand and wrist? Doctor: According to your narration, it seems to be due to compression effect of band. Blood supply and neural supply may be impaired due to this compression. Persistence of numbness in thumb, ring and middle finger points to median nerve compression at carpal tunnel. In my opinion this may be temporary and will recover on its own if no swelling at wrist present and no distal skin changes in hand. If symptoms persists you may have to undergo nerve conduction study for carpal tunnel syndrome. At present you may take Aceclofenac plus paracetamol and methylcobalamin for 4-5 days with rest of involved limb."
},
{
"id": 24851,
"tgt": "What causes head pressure, fatigue, vomiting and palpitations?",
"src": "Patient: Hi, for a month now I've been having head pressure in my head. The pressure feels like when you lie upside down or a hand stand. It been constant and hasn't gone a way. Also very fatigue, short of breath.Iv had a the found a chat on my occipital horn of the right lateral. The neuro said it was fine. I have also been vomiting in the mornings,feeling heart paps. Or racing, my blood pressure has been dropping off and on, my heart also feels weak. What could cause this? Could it be lack of blood to the heart cause pressur? Could it be heart failure? What could it maybe be? Doctor: Thanks for your question on Healthcare Magic. I can understand your concern. Yes, by your history and description, possibility of heart related diseases like heart failure or arrhythmia (rhythm disturbances in heart). So better to consult cardiologist and get done 1. Blood pressure monitoring 2. Ecg 3. 2d echo 4. Holter monitoring (24 hours continuous recording of ecg). If all these are normal then no need to worry for heart diseases. Sometimes, uncontrolled stress and anxiety can also cause similar symptoms. So consult psychiatrist and get done counselling sessions. Try to identify stressor in your life and start working on it's solution. You may need anxiolytic drugs too. Don't worry, you will be alright. Avoid stress and tension, be relax and calm. Hope I have solved your query. I will be happy to help you further. Wish you good health. Thanks."
},
{
"id": 60086,
"tgt": "Suffering from jaundice past 5 years.Admitted twice. .Is there any permanent solution for jaundice?",
"src": "Patient: Goodmorning doctor my name is arun. I am from mangalore.my decease is im suffering from jaundice past 5 years. I had admitted twice becauce of maleria and jaundice .Yesterday i tested my jaundice the report says total bilirubin is 2 and reference range is 1.5 .IS there any permanent solution for jaundice.plz suggest me Doctor: Hi Arun, The Total bilirubin being 2 is not of a concern for me. I think you should not go about the reference range for yourself and go perturbed. An adult with two time history of malaria and jaundice within 5 yrs in a tropical country like India where malaria and hepatitis A are highly prevalent is just normal. Just take all the measure to prevent mosquito bite and contaminated food and water. Regards."
},
{
"id": 117624,
"tgt": "How long does itching last in obstructive jaundice?",
"src": "Patient: hi..i have been suffering from jaundice for 2 weeks..my bilirubin level last week was 14..this week it went upto 23.6..doc told me it was an obstructive jaundice..which is also causing severe itching in whole body..i have had obstruction removed through ERCP..but my itching is not gone yet?how much time will it take to go away? Doctor: Hi,Thanks for asking.Based on your clinical history and query, my opinion is as follows:1. Itching is due to increased bile acids in the body, which is deposited in skin. This is due to obstructive jaundice.2. Once the obstruction cause is removed, it will come down and it should reduce within a week.3. You can additionally take bile acid chelating agents.Hope it helps.Any further queries, happy to help again."
},
{
"id": 225674,
"tgt": "Had unprotected sex, had bleeding. On alesse, missed pill. Why did it happen?",
"src": "Patient: I'm currently on alesse, today is Friday I missed taking the pill last Saturday and Sunday (first week of the pack, I start my packs on Mondays). I took two pills Monday and two Tuesday. I did bleed for a day or so. I've had unprotected sex multiple times during the week and twice where he finished in me. When is the earliest I can get a pregnancy test? And should I be worried even if I get a regular withdrawal bleeding? Doctor: Hellothanks for your query.The earliest you can take a pregnancy test is when you miss your normal period, the one you had before starting the current pack, count 30 days from then, and if you do not have your period by then, take a pregnancy test.even if you get a regular withdrawal bleed, pregnancy is not ruled out.Take care."
},
{
"id": 80958,
"tgt": "What are the side effects of taking r-cinex600,combutol 1000,benadon and pyzmia for TB?",
"src": "Patient: I have TB in lungs, taking r-cinex600, combutol 1000, benadon, pyzmia 750 (2 tablets per day) . It causes skin itching; tea color urination after 5-6 hrs.Plz suggest me about whether medicines are ok or not. And how to overcome side effect-itching, urination.Gautam Kumar PrasadAge-30 yrsWeight-61 kgBokaro.India. Doctor: Hi,Tea coloured urine after taking anti tbg medications is perfectly ok and is due to your r cinex tablet. As regards the itching, you can visit your doctor who may evaluate it and may precribe an antihistaminic medication for the same after evaluation. Itching is a common side effect of anti tb medicines.RegardsDr. MishraConsultant Pulmonologist"
},
{
"id": 110078,
"tgt": "Suggest treatment for dizziness and back pain after car accident",
"src": "Patient: Hi. I was in a car accident a year and half ago, hit from behind. Since then have had bloated stomach and back pain on and off. I have had many tests done but no one seems to be able to find out what is wrong. Recently I have had bad bloating again and pain on right side of my back that goes from top to middle of my back, dizzyness and am pale. Doctor: Hi,Welcome to healthcare magic.After going through your query I think you are suffering from chronic backache. Along with this you also has anxiety component causing bloating and dizziness.Treatment of back pain is exercises and analgesics (diclofenac ). For anxiety component alprazolam 0.25 mg at bed time is useful. You can consult your doctor about this .Sometimes vitamin D deficiency is the cause so get her vitamin D checked .If it is low then vitamin D supplementation ( weekly with milk) can be taken.Avoid long continuous standing.Sit in a straight posture. Eat milk and milk product, fruits and green leafy vegetables daily. To further investigate MRI of the region is advised . I think your query answered.Welcome to any follow up query"
},
{
"id": 194512,
"tgt": "Suggest treatment for obstructive azoospermia",
"src": "Patient: hi i am from nepal, i am 32 of male and suffering from obstructive azoospermia. i did IVF Tesa but in nepal but no result. one of the doctor suggested me to go to sufdar jung hospital in delhi and meet urinary surgeon. can a urinary surgeon do surgery and make it better Doctor: Hello, There are some surgical procedures or reconstructive methods which can treat your obstructive azoospermia. Hope I have answered your query. Let me know if I can assist you further. Regards, Dr. K. V. Anand, Psychologist"
},
{
"id": 50278,
"tgt": "Remedies to reduce urea and creatinine levels in the body?",
"src": "Patient: hI DOCTOR, my query is how can we reduce urea and creatinine in our body , createnine is 1.76 and urea is 43 in my brother,s blood report and as doctors the size of right kidney is 7.71 X 4.77 cm and the size of left kidney is 8.54 X 4.90 cm . what will be your suggestions . as per our doctors , disease is recoverable . Doctor: Although you have not mentioned your brother's age,I assume that he is more than 15 years.If that is the case then kidneys have alredy shrunk & urea & creatinine may not be completely recoverable.There are some dugs which may help you to reduce urea & creatinine but before that you should reduce salt & protein intake from your diet.You may start Cap Cudo Forte 1OD & Tab Sodabicarb for this"
},
{
"id": 157365,
"tgt": "Is hysterectomy the best option to remove fibroid tumors and to relieve the heavy bleeding ? Keeping the cervix cause cervical cancer?",
"src": "Patient: Hello I have 4 fibroid tumors, rather large in size, one in fact the size of a small fetus. In talking with my Dr. a hysterectomy was my best option both to remove the tumors and to relieve the heavy bleeding I've been going thru the past year. My uterus and 1 ovary will be removed. Will keeping my cervix put me at greater risk for cervical cancer? Doctor: HIThank for asking to HCMNo you are under the wrong impression or may be wrongly informed that is not the condition for cancer of cervix at all, just take it easy and forget about this, nothing will happen as you are thinking, every thing fine have nice day."
},
{
"id": 106648,
"tgt": "What causes pain in the hips and lower back after a cervical fusion surgery?",
"src": "Patient: I had a lumbar fusion about five years ago which included disks 4 5&6! Recently I have had a cervical fusion of 3_4_5 ? My left hip and lower back is feeling very painful again I am using ice and heat antiinflammatory medicine and a tens unit ! My neurosurgery Dr had me go for an injection into the SI joint,. I am in quite a bit of pain most days.Now... The surgeon sent me to physical therapy . After the evaluation....They were not comfortable doing very much until I have an x-ray of the low back area.they are concerned that I am having the same problems with my lower back fusion ! What is your thought on this? Doctor: Hello and Welcome to \u2018Ask A Doctor\u2019 service. I have reviewed your query and here is my advice. Thanks for writing to us.The back pain can be related to a muscle spasm or inactivity. I hope this information has been helpful for you. Regards, Dr. Praveen Tayal"
},
{
"id": 41222,
"tgt": "How safe is ovum freezing?",
"src": "Patient: hi ,i am going to be 36 yrs old women. Due to bad luck my marriage didnt work out and i am seprated. Now i am little insecure in regards to baby . I dont know when i will get remarrige and settled. I want to know ovam freezing is safe and how much does it cost in indian currency . I live in Bhopal mp Doctor: Hello, ovum freezing safe and can cost upto 1.5-2 lacs depending on the centre of freezing and number of eggs retreived and the duration for which freezing will be needed.In case you have any questions in future you can contact me directly on http://bit.ly/drmanishajain"
},
{
"id": 19025,
"tgt": "What can cause pinches in heart?",
"src": "Patient: All of you screen is not showing up on the right side. Any way I have been experiencing what feels like pinches in my heart. Some are enough to make me yell a little. Also the night before a started with this, one of my neck veins was very painful and tender I could not lie on that side Doctor: Hello,Please get examined as soon as possible. Get ECG and chest x-ray as soon as. And also get the neck veins examined as there may be inflammation or clot. Also, get checked for bp and blood sugar level.Hope I have answered your query. Let me know if I can assist you further.Regards, Dr. Varinder Joshi"
},
{
"id": 161190,
"tgt": "Suggest remedy for infection in eye",
"src": "Patient: my son is 2 and was diagnosed with an eye virus. i cannot remember the name of it. the dr said it was like pinkeye but worse. the infection started as a small red blood shot spot on his eye and 24 hrs later the entire eye was blood red then 24 hrs later the other eye was completely red. no white of the eye can be seen now. we continue to use the eye drops he perscribed . i am just wanting to know the name of the virus for research. thank you Doctor: Hi, It is difficult to guess. There are 100s of viruses. But the most common ones are - adenovirus. Other viruses that can be responsible for conjunctival infection include herpes simplex virus (HSV), varicella-zoster virus (VZV), picornavirus (enterovirus 70, Coxsackie A24), poxvirus (molluscum contagiosum, vaccinia), and human immunodeficiency virus (HIV). Hope I have answered your query. Let me know if I can assist you further. Take care Regards, Dr Sumanth Amperayani, Pediatrician, Pulmonology"
},
{
"id": 174346,
"tgt": "What is the cause of rash on the leg?",
"src": "Patient: My 5 month old has a slight rash under her skin on the front of her left leg, it's been there for 4 days now, she has been fine in her self but has woke up today with a cold. The rash hasn't got any worse but I'm worried that it could be something wrong now that she has a cold Doctor: For rash on the leg you can give your daughter some antibiotics like syrup cefadroxyl or syrup cefalexin along with syrup cetrizine or pheniramine as it will take care of rash as well as cold. You can use colp-g or betnovate cream for local application. Don't worry she will be fine soon."
},
{
"id": 43532,
"tgt": "History of miscarriage. Have ovarian cyst. Trying to get pregnant. What is wrong?",
"src": "Patient: I have a cyst on my left ovary. Had a missed abort in week 13 almost 2 years ago. Since then i try to get pregnant. My cycle is regular, and my ovulation tests re positive in midcycle. I am 37 years old. I don't know what to do. Doctors visit 1 month ago said everything ok, but could take the cyst out by surgery. The cyst is 3.5 cm big. What do you think. What could be wrong with me? My husbands spers are average after test. Doctor: Hi, After the age of 37 years, fertility declines rapidly. So you should go for treatment with high chance of conception. If your tubes are fine, you can go for superovulation with IUI, which is less costly & gives decent pregnancy rates of around 15-20% per cycle if everything is fine. Regarding cyst in ovary, if it is simple clear cyst & diagnosed recently, you can wait for 1-2 months for spontaneous resolution. If the cyst is endometriotic OR if its persistent since last 2-3 months, you can go for its removal through laparoscopy. Wish you good health."
},
{
"id": 181472,
"tgt": "Suggest treatment for heightened sensitivity in gums after removing a wisdom tooth?",
"src": "Patient: Can a dry socket from a wisdom tooth extraction that i had done 8 months ago be repaired because i feel that it is more sensitive to heat and spicy foods back there... and also breathing in and having it dry out affects it... just want to know if im screwed or if there are stuff they do to fix that Doctor: Hi..Welcome to HEALTHCARE MAGIC..I have gone through your query and can understand your concerns..As per your complain in case of you have got the extraction of the wisdom tooth 8 months back, there is a very little probability that you have a dry socket and this is because Dry Socket is a very painful condition and if it would have been the cause it would have been very painful, rotten and fatal by now..If there is some sensitivity present in that area there is a possibility that there is any left out root piece in the extraction is or there is also a possibility that the tooth adjacent to the extraction area is getting decayed..Consult a Dentist and get an x ray done to rule out the exact cause and get treated accordingly..As of now you can take anti-inflammatory painkillers like Ibuprofen..Apply numbing gel in that area..Gargle with warm saline solution..Hope this information helps..Thanks and regards.Dr.Honey Nandwani Arora."
},
{
"id": 108133,
"tgt": "Suggest treatment for back pain",
"src": "Patient: i was having pain in the back between waist and hip on the left side for about 2 weeks. this morning while geetting up from a bent position i had severe pain and could not stand erect. After massaging with a cream and lying down on bed for about an hour my pain reduced to a tolerable level. The pain is on the left side of the spine. What could it be? Doctor: Hello, I had gone through the case and found that it might be muscle spasms or slip disc.So take mild pain killer tablet Ultracet twice a day, apply muscle relaxant and avoid to lift the heavy weight.Also go for xray of lower spin to rule out the slip disc. If pain increases then consult with orthopedic doctor.Regards"
},
{
"id": 129793,
"tgt": "What causes dizziness, involuntary hand twitches and muscle fasciculations?",
"src": "Patient: I am a healthy 31 yr old female. For the past few weeks I have been feeling dizzy, like vertigo, and my hands sometimes twitch involuntarily. Also I have other little muscle fasculations all over my body off and on throughout the day. Should I be concerned about MS? Doctor: Hello!If such a patient comes to me in my clinic I would not rule out MS first, instead I would like to check for the following in the order of priority.There are many factors causing fasciculations etc.Visit Your primary doctor first"
},
{
"id": 925,
"tgt": "What precautions should be taken for conception in mid thirties?",
"src": "Patient: Dear Doctor,I am arunjit banerjee from kolkata...i am 34+ and my wife is 34 also. We are planning for her pregnancy by next year May.. We are a little worried as age is slightly on the wrong side as she came to know from her relatives...if that is so what is the precautions that we should take ...and how can we possibly take precautions against risks...thank you Doctor: Hi, I think you should go for a first trimester screening which consists of some blood tests. Also do a nuchal translucency scan between 11 to 14 weeks. These can rule out any congenital abnormality in the child. Due to increased age, chances of congenital abnormality increases in the baby. So, also do a second trimester screening and a level 2 scan at 18 to 20 weeks. Check sugar levels and blood pressure at regular intervals during pregnancy. Hope I have answered your question.Regards Dr khushboo"
},
{
"id": 947,
"tgt": "What are the chances of conceiving with 1.36cms follicle?",
"src": "Patient: Hi doctor i have pcos and got PCOD profile done which is in range. after that I got HSG done on 9th day of menses which was normal and 10th day i started with follicular study yesterday was my 13th day and my follicle measured 1.36cms. What are the chances of conceiving.By Anonymous Doctor: Hi, I think you should repeat a scan after 3 days. If follicle grows more than 17 mm , then take injection for rupturing the follicles. Be in contact with your husband for 2 to 3 days after injection. Take progesterone for next 2 weeks. Do a urine pregnancy test at home after that. There is a chance of pregnancy if you get a follicle more than 17 mm. Hope I have answered your question. Regards Dr khushboo"
},
{
"id": 178580,
"tgt": "Is swallowing Mylanta and Benadryl dangerous for children?",
"src": "Patient: My 13 month old has HFMD. I was told by the dr that we have to wait it out. Tonight he was beyond miserable due to his mouth sores. I have never heard him cry like this. In desperation I tried the magic mouthwash; mixing 1/2 tsp of mylanta and 1/2 tsp of children s Benadryl. In an attempt to get him to swish it around he swallowed it...is he going to be ok? Should I take him to urgent care? I am worried. Doctor: Thanks for posting your query at HealthCareMagic. Hand, foot and mouth disease is a a common and highly contagious viral infection caused by Coxsackievirus A16 or Enterovirus 71 (EV-71) and is characterised by painful mouth sores. So it is natural for your child to cry like hell.There is no harm if the child swallows Mylanta and you need not worry about it. All that can happen is that the child may feel sleepy for sometime.Currently, there is no specific curative treatment for hand, foot and mouth disease. Disease management typically focuses on achieving symptomatic relief. Pain from the sores may be eased with the use of analgesic medications. Infection is typically mild and lasts approximately 1 week, but may occasionally run a longer course. Fever reducers like paracetamol and lukewarm baths can help decrease body temperature. It is a self containing disease and corrects on its own but with time.Hope that I have been able to clarify the matter. Feel free to revert back in case of further queries if any."
},
{
"id": 94682,
"tgt": "Have abdominal pain after meals. History of hypertension. Need immediate care?",
"src": "Patient: BP of 151/101 with upper left abdominal pain . 47yo male 220lbs 5 8 . Abdominal CT from 1 year ago is clear of any issues with Kidneys, Galbladder and Pacreus. GI doc says he has no idea. Ordered a EGD for 1 month out with follow up in 3 months. Pain in daily starting Dull/achy to sharp and stabbing especially after meals. Should I take him to the ER? Doctor: Hello and welcome. Thanks for your query. The one symptom of pain after meals, is most commonly associated with peptic ulcer disease (PUD). Which is an ulcer found either in the duodenum (which is the first part of the intestine, that is connected to the stomach) or the stomach itself. The main differential factor is the duration or initiation of pain after a meal. In the presence of an ulcer in the stomach the pain usually starts 15-20 minutes after a meal, which is how much time the food takes to reach the stomach and irritate the ulcer. In the case of a duodenal ulcer, the pain starts around 2 hours after a meal, which is how much time the food takes to reach the duodenum after consumption. Other findings of a gastric ulcer are: -Vomiting helps in reducing the symptoms -Appetite is usually normal -Patient usually experiences loss of weight -In a complicated or bleeding stomach ulcer, we find traces of blood in the vomitus, and rarely black or dark brownish colored stools -More common in patients belonging to lower socio-economic status (lower middle class most common) Other findings of a duodenal ulcer are: - Higher socio-economic status patients -Normal or increased appetite -Normal or increased weight -Darkish brown stool is more common that bloody vomitus I hope these details at least help you in ruling out an ulcer. An endoscopy would also help in visualizing the ulcer, size and location, and can also take a biopsy (tissue sample) at the same time. Please discuss the details with your doctor. For any further clarifications, you can always write back to us. Wishing you good health."
},
{
"id": 152020,
"tgt": "I suffer from diffuse posterior prolapse of the same discs. what do you suggest ?",
"src": "Patient: I did 2 surgeries in lumber discs L3-4 and L4-5 in 1997 and 2009. Now I suffer from diffuse posterior prolapse of the same discs. Is surgery for the third time recommended or what do you suggest ? thank you. Doctor: wel come, as you are already undergone two surgeriesof lamber.the opinion of third sugery will be merit able.out sider advise may cause some complication for you.GOD BLESS YOU ALL THE PERFECT HEALTH. DR.MOHSIN MADNI madni66@saudia.com"
},
{
"id": 101903,
"tgt": "How to treat discomfort in throat and coughing after using bleach?",
"src": "Patient: Today My mom and I were cleaning. We used bleach and ammonia. The smell was very strong. It made us cough. My mom & I both have dry throats now and she puked. We are still coughing and are throats are still dry and uncomfortable What can we do about this? will it go away Doctor: Hi, thanks for using healthcare magicInhalation of bleach or ammonia could result in varied symptoms such as those that you experiencing. The mixture of both would result in more symptoms.The side effects can vary from mild to severe. In some cases of inhalation there may be injury to the lungs and other aspects of respiratory tract. It may be best for you to see your doctor for an assessment.I hope this helps"
},
{
"id": 176231,
"tgt": "What causes pain in abdomen, low appetite and painful urination in a child?",
"src": "Patient: Hello, my 6 year old daughter threw up all of friday and some Saturday, hasn t eaten much since Friday and has slept a lot the last few days. Today, Sunday she hasn t thrown up but is complaining of major stomach cramps when she sits up or walks and is having pain while she pees? Doctor: Hi,From history it seems that she might be having acute urinary tract infection producing renal colic and pain while urination.Give her antispasmodic medicine like Meftal spas for pain.Go for routine and microscopic urine check up.After report go for treatment accordingly.Give her plenty of water.Ok and take care."
},
{
"id": 173934,
"tgt": "Suffering from Oppositional defiance disorder",
"src": "Patient: Hi my 9 year old son has been on Risperdal for the last two years with a continuous increase in dosage to try and help him deal with his ODD (Oppositional Defiance Disorder). He has had constant weight gain, severe behaviour causing harm to himself, and those around him. We are in Canada and it is to my understanding that this drug should not be used in children any longer. I need to find out how I can get him taken off of this medication without causing him some more harm. Thanks Doctor: I think you can try to stop giving medicines, but replace it to sedative-Ashwagandha or other for 1-2 months. For further treatment you consult your doctor. Dr.Svetlana Shrivastva"
},
{
"id": 107208,
"tgt": "What causes pain in the lower back after removal of Mirena IUD?",
"src": "Patient: I had my mirena removed about 8 months ago and have had severe lower back pain every month during my cycle monthly. It started only lasting 2-3 days and now it lasts 3 weeks and I rarely have time with no back pain. Is this a side effect of removing it? Doctor: The detrimental effects of the Mirena IUD to users are many, with the worst being migration of the device, puncturing the uterine wall and damaging organs within the body cavity such as the liver and intestines. Other possible side effects are ectopic pregnancy, fatigue, depression, mood swings, painful cramps and many more. Many women who are opting to have the Mirena IUD removed to avoid complications are experiencing many of the symptoms that they sought to avoid. These symptoms also occur with women who had the device removed after the prescribed duration of five years. These detrimental effects, known as the Mirena crash, or Mirena IUD crash, can occur for weeks or even months after removal of the device. In a great number of women, the symptoms were severe enough to negatively impact their lives.The Mirena IUD is a T shaped device that is implanted in the uterus, containing a reservoir of synthetic progesterone which is slowly released. This synthetic product, Levonorgestral, causes a thickening of the mucous membrane inhibiting the travel of sperm, preventing pregnancy.Mirena Crash after RemovalSo, what is the Mirena crash? The Mirena crash is the name given to the symptoms of pain and discomfort that are prompted when the birth-control device is removed and the flow of synthetic hormones stop. It can be weeks or longer before the body realizes it needs to produce its own progesterone because it has become \u201clazy\u201d, relying on the synthetic provision. This causes hormone imbalance or estrogen dominance. This same period of time reflects the Mirena crash duration. In many women this imbalance causes severe symptoms similar to those associated with the usage of the device. In some women, Mirena crash symptoms appear recurrently before each period, lasting for months. Some other women suffer for extended periods of time both physically and emotionally at any time, day or night. One sufferer described her feelings as being \u201csmothered with severe despair\u201d at night.How long does the Mirena crash last?Some women have reported the crash only lasting a few days, while others report weeks or months. Medically speaking, there is no way to tell. This is because it all depends on how long your body takes to return to a normal hormonal balance. The sooner this happens, the sooner the symptoms will dissipate.Mirena Crash SymptomsMirena crash symptoms include:Runaway emotions such as anxiety, sadness, anger and depression. Severe cases have produced suicidal thoughts.Frequent mood swings that can strain relationships and effect work performanceLethargyPhysical symptoms such as muscle fatigue, cough, sore throat and even fever which are often misdiagnosedNausea, vomiting and stomach painBreast tendernessDecreased sex drive and painful sex."
},
{
"id": 86058,
"tgt": "What do cramping sharp pain and tenderness in and around the belly button area indicate?",
"src": "Patient: HI I have been experiencing pain behind my belly button awhile now. It s not always there but every once in awhile. It s like a cramping sharip pain. Also feels tender in the cramping location. Today I felt another sharp cramping pain under my right rib. Haven t experienced that before or at least not recently. What could it be Doctor: Hello and Welcome to \u2018Ask A Doctor\u2019 service. I have reviewed your query and here is my advice. * As per my clinical experience, the sharp pain with cramps around belly area may be due to early appendicitis or other issues. * I recommend to undertake lab tests with complete blood counts and CT scan of the abdomen & pelvis for best evaluation of the cause and manage accordingly. Hope I have answered your query. Let me know if I can assist you further."
},
{
"id": 212782,
"tgt": "Been dizzy, lasts a few hours, unable to do any task, ears and face gets red. Help?",
"src": "Patient: I have major anxiety and fear about beig dizzy. I have een dizzy a few times in my life and now it seems to be back. I get dizzy when I move my head down but it s not bppv because it only lasts like a few hours n it doesn t last days. Everyday I wake up worrying I I will e dizzy and I can t do anything without thinking about. Could. The worrying about being dizzy actually cause me to be dizzy? My fa e and ears get rely got and red too sometimes. Doctor: HI I have gone through your mail and as you said that you had faced few episodes of being dizzy in your past and type of tendency you are again suspecting. As you said that you are not having B.P.P.V.You have asked weather just thinking about dizzy may cause dizzy? For your this question I would like to say YES, because psychological factor are also included in the list of causes of vertigo.But as for as diagnosis is concern we should rule out other causes of vertigo and if no pathological cause could be detected than only one may think about psychological vertigo.You are advice to go for these basic investigations like- (1) E.N.G., (2) C.T.scan head with comments on C.P.angle, (3) Audiometery - pure tone & impedance. After this you should go for physical examination by Physician, E.N.T.specialist, Ophthalmologist and lastly Cardiologist and clinical co-relation of recmonded investigations to find out cause of your dizziness."
},
{
"id": 202207,
"tgt": "How to overcome the problem of drops of urine even after finishing?",
"src": "Patient: hii am 30 years old, height 6 ft, weight 80 kg. sometimes when i finish passing urine, i get few drops of urine. As i am muslim and we need to be neat and clean all the time. i dont want that a single drop of urine will come out after i finish passing urine. Doctor: HIThank for asking to HCMI really appreciate your concern looking to the history given here I would like to say you that if you are a male person then the chances of prostrate enlargement is likely and for that you need to confirm this, as such you have not mentioned your gender here, so if your are female person then urinary tract infection and other gynecological problem need to be differentiate, hope this information helps you, take care and have a nice day."
},
{
"id": 175317,
"tgt": "What could recurring small red bump on leg with white center oozing blood in a child indicate?",
"src": "Patient: Hi, my son 8 years old. Two weeks before on his left leg small red thing came,then some white thing we saw in the middle. One day it came out with blood.thenwe put bandade. That is ok. Then on the same leg 4spot came again. Why it is happening? Any thing wrong? I am really worried. Doctor: It seems like as strepto-staphilococcal infection, like as furunculosis. The cause of furunculosis is bacteria which spread from existing infections to nearby hair follicles. Many other causes of furunculosis have been suggested, therefore, such as insanitary personal habits or surroundings, anemia, hypoproteinemia, low general or local resistance to the infection, hyperglycemia, low metabolic rate . Reflecting this uncertainty of causation, next therapeutic measures will recommend: don't eat too much sugar, wash legs with antibacterial soap-for example, dettol. If you will have reccurent infection then apply chloramphenikol 1% on the red bumps for resolution and antibacterial effects.Best regardsDr.Svetlana"
},
{
"id": 170118,
"tgt": "Suggest remedy for lactose intolerance in an infant",
"src": "Patient: My baby is 10 month old weighs 8 kg (3.25kg at birth) heighs 74cm. i find she is allergic to milk.. whenever i give low fat milk she has green poo. once i stop giving milk her poo is normal . As my brestfed is low (only one time in a day) please give me some solution..Thanks in advance Doctor: Hi, Welcome to HCM. I have read your question in detail and I know that you are very concerned about your child but don't worry. By what you quote it seems that your child probably has cow milk allergy due to lactose intolerance. You should give lactose free milk like zerolac, Similac isomil, Similac sensitive or nusobee. I hope this will help you. Wishing your child good health. Take care. Regards Dr Deepak Patel MD Pediatrics"
},
{
"id": 76114,
"tgt": "What causes tietze syndrome?",
"src": "Patient: I have been diagnosed with having Tietze syndrome and had it for over 15 months and it is making me feel very depressed as i am usually a very active person and it is also affecting my work i have had to go to part time hours and i am at the end of my tether, apart from the anti inflammatory`s is there anything out there that can make me feel human again and lead a normal life as i also feel that it is now beating me mentally Doctor: Thanks for your question on Healthcare Magic. I can understand your concern. Tietz syndrome is costochondritis and it is 100% treatable. So follow these steps for better symptomatic relief in Tietz syndrome. 1. Avoid stress and tension, be relax and calm. 2. Avoid bad postures in sleep. 3. Avoid heavyweight lifting and strenuous exercise. 4. Avoid movements causing pain. 5. Wear chest belt or rib belt to avoid movements of affected areas. 6. Take combination of painkiller and anti inflammatory drugs. Don't worry, you will be alright with all these. And don't feel depressed. You will definitely improve with all these. Hope I have solved your query. I will be happy to help you further. Wish you good health. Thanks"
},
{
"id": 2610,
"tgt": "Could I be pregnant due to delayed period?",
"src": "Patient: On june 1st my partner and I had sex and he ejaculated in a condom. He took it off after and there were no holes. He tried not to get semen on his fingers but then he lightly wiped his fingers on a tissue. He wiped the front of my underwear for about 10 min. Then fingered me after. Later when I realized this mistake we purchased plan B and I took that in under 20 hours. June 1st was a day before my 3 most fertile days. I later had heavy bleeding and cramping on June 18th. 2 days later than when I expected my period to come. Making this a 34 day cycle. Could I be pregnant ? Its been almost 5 weeks since the incident and I am having nausea and breast pains when my period is 2 weeks away Doctor: Hi,I understand your concerns but after having heavy bleeding it is unlikely to be pregnant. If you are still worried then you can have a pregnancy test just for the ease of mind.Hope I have answered your query. Let me know if I can assist you further. Regards,Dr. Salah Saad Shoman"
},
{
"id": 83382,
"tgt": "Will the birth control pill taken during pregnancy cause infertility problems?",
"src": "Patient: my mother was on the birth conrtol pill when i was concieved, and did not know that she was pregnat with me for a few months, so of course, she continued taking the birth control pill while pregnat with me. can the medication from the pill have gone inside my embryo and made me infertile? Doctor: Hi,They are unlikely to cause fertility problems. Experimental studies in animals have shown evidence of birth defects or other malformations in the off springs following the exposure of hormonal contraceptives during their pregnancy. However, in human beings such studies have failed to support such harmful effects in the new born babies following intake of oral contraceptives pills by the mothers.Hope I have answered your question. Let me know if I can assist you further.Regards,Dr. Mohammed Taher Ali, General & Family Physician"
},
{
"id": 148591,
"tgt": "Diabetic, weak legs, limping. Hereditary motor sensory neuropathy. How much will treatment cost?",
"src": "Patient: my son is His 32 years old He is diabetic His condition started some three years ago when we noticed that his limping he has been to various doctors and had test done no one here can pin point exactly what is the problem month ago he went for scans and the neurologist thinks its Hereditary Motor Sensory Neuropathy his legs are very weak and he really battles to walk he always needs support for eg when shopping he needs the trolley to push for support if his walking he uses the walking stick. Very sad as his a young man and we as parents really cannot see him going through this difficulty. How long will his treatment last I will accompany him to India and if you can shed some light on the cost anticipated for a case like this one just approximate. Do you have accommodation for me while his doing the treatment in your hospital? Look forward to hearing from you soon. YYYY@YYYY Doctor: Hi,Thank you for posting your query.It is unfortunate that the young man has developed neuropathy, and it is causing disability to him.If it is hereditary motor sensory neuropathy (HMSN), then, no treatment is possible, as this condition is genetic in nature and can not be cured.However, we need to confirm the diagnosis and also try to find lout if it is something else, such as vasculiits, CIDP (inflammation to the nerves), etc. In this case, treatment would be possible.I hope my answer helps. Please get back if you require any additional information.Wishing you good health,Dr Sudhir Kumar MD (Internal Medicine), DM (Neurology)Senior Consultant NeurologistApollo Hospitals, Hyderabad, IndiaClick on this link to ask me a DIRECT QUERY: http://bit.ly/Dr-Sudhir-kumarMy BLOG: http://bestneurodoctor.blogspot.in"
},
{
"id": 203548,
"tgt": "What is the cause for sores on the head of my penis when on a topical cream?",
"src": "Patient: Hello, I went to my Dr. because I was worried that I had contracted genital herpes. He did a test and I did not test positive for the HSV2 virus. I have sores that look sort of like a herpes outbreak on the head of my penis that is not going away. He gave me a topical cream thinking it might be a fungal infection and I have been using it for over a week now and it has not changed at all. They have scabbed over and the scabs come off and they are constantly oozing a clear liquid Doctor: Hello,Thanks for choosing health care magic for posting your query.I have gone through your question in detail and I can understand what you are going through.If the serological test for herpes has come negative, then you may not worry much. You may repeat it as well. But if it again comes negative then you have to leave your worries as the test is quite sensitive and specific. It might be a fugal infection. You may upload some pictures of the lesion with your question which can give us a better picture of your illness. Hope I am able to answer your concerns.If you have any further query, I would be glad to help you.In future if you wish to contact me directly, or want any further details about this same issue you can use the below mentioned link:bit.ly/dr-srikanth-reddy"
},
{
"id": 108240,
"tgt": "Does tendinitis cause lower back pain?",
"src": "Patient: I have an hyperflex in my right leg (was evaluated by physio and doctor) brain scan were normal, I have a iliopsoas tendinitis, I have not been able to walk normally for 6 months. Pain is often in my very lower back. Is it just tendinitis causing this? Doctor: WHEATHER UR LOW BACK PAIN IS RADIATING TO BACK OF THIGH??? IF SO U NEED MRI SCAN OF LUMBO SACRAL SPINE"
},
{
"id": 66037,
"tgt": "Need medication for large lump & black head at the back",
"src": "Patient: m,y husband had a blackhead on his back, it was large we managed to get it out, but over a few weeks it has come up in a large lump, there is no more in it , just a large lump,this seems to be getting bigger, it is not painful, it is located on the muscle to the side of the spine. Doctor: Hi looking at your history it could be a sebaceous cyst or lipomaif black punctum is there then it is just a sebaceous cyst or if lump slips under the examining finger than this could be lipoma please visit your doctor both the conditions can be removed surgically no need to worry Hope I have answered your questionthanks for using HCMgood luck"
},
{
"id": 43549,
"tgt": "Semen test. Am I normal or have infertility issue?",
"src": "Patient: Sir, My semen analysis report is as follows. Quantity - 2.0 ml, Colour- Perly white, Reaction- Alkaline, Total Sperm Count- 106 million / CCM, Motility- 8t0% after 1hr, 70% after 2hr, 60% after 3hr, 30% after 5hr, epithelial cel- 1-2 / phf, Pus Cell- a few, Normal- 90, Abnormal- 10. Whether the aforesaid report is normal or has any infertility issue? I am waiting for your kind reply. Regards Abhijit Ranjan Doctor: hello welcome to health care magic i have read your reports volume count and motility is normal except that, presence of pus cells and alkaline reaction indicates some mild infection,.consult local physician and take appropriate antibiotics,,don't get delay otherwise it may lead to infertility.... for this reports these are normal no infertility problem.. i will tell you some precautions,,please follow that 1.avoid bathing with too hot water(bath with luke warm water) 2.avoid wearing tight pants like jeans,,wear only loose pants 3.avoid smoking all these things decreases the testicular function.. hope you will understand my suggestion thank you Dr.siddartha"
},
{
"id": 31381,
"tgt": "How to check for undulant fever?",
"src": "Patient: I had undulant fever in 1953, have had severe fatigue, joint pain etc. on & off for several years, very severe this past year. I was diagnosed with fibromyalgia 9 i really don't think I have that) I had a dream and was directed to tell my Dr that I had Undulant fever as a child, I did and now she really thinks I am nuts. I have read some things about cronic undulanr fever. So the question is this..could I have this after 57 years? Is there a blood test for this? Doctor: Hi thanks for asking question You want to know about how to diagnose undulent fever.It can present with fever, myalgia, arthritis , headache etc.Blood will reveal anemia and leukopenia mainly.gram stain reveal gram negative coccobacilli.Serological test is helpful like ELISA.Blood or bone marrow culture helpful for diagnosis.If liver is involved liver enzymes like AST elevated.Liver biopsy can be helpful.I hope I have solve your query regarding diagnosis of brucellosis"
},
{
"id": 223142,
"tgt": "Should I be concerned for spotting while being on birth control?",
"src": "Patient: Hi i am in the middle of my birth control pack and have had spotting not a full period but heavy spotting i have been on the same birth control for 4 years now.....should i be concerned nothing like this has ever happend....i am having problems at my job could stree be a factor?? Doctor: Hello and Welcome to \u2018Ask A Doctor\u2019 service. I have reviewed your query and here is my advice. It is not a matter of concern and it might be due to some hormonal effect caused by contraceptive pills. No treatment is required and in case repeated spots better to go for an ultrasound scan and get evaluated. Hope I have answered your query. Let me know if I can assist you further."
},
{
"id": 183159,
"tgt": "Is it safe to leave piece of tooth in the mouth?",
"src": "Patient: I am a 55 year old male.I had a broken tooth which became loose.The dentist removed most but could not get it all as remaing piece was close to bone and tight.He said I would have to see an oral surgeon to complete the job.My question is if I elect to leave remaining piece attatched will this cause any health issues? Doctor: Thank you for your query.Leaving a piece of tooth inside the socket is a tricky prospect and it all depends upon the call of the treating Dentist to leave it like that or call an oral surgeon to remove it.If you leave it like that then there are chances that it may get infected and cause problems in near future. Hence, always advisable to take it all out."
},
{
"id": 29865,
"tgt": "Suggest remedies for cough that persists despite medication",
"src": "Patient: hello i can not seem to get this coughing to go aay after over a week, i have been trying EVERYTHIN to try to get rid of the coughing: i have been taking cough syrup, cold medicine, cough drops, eating chicken noodle soup, drinking warm tea, and i have even went to my doctor and started an antibiotic... i just DO NOT know what to do anymore, why can i not get rid of this horrible cough and what should i try to do next?? thank you soooo much!! :) :) Doctor: thanks for posting your query to hcm.some times cough, takes time to resolve completely could be because of improper treatment, misdiagnosis or due to some unknown allergic factor .I suggest you to undergo sputum microscopic examination and culture and sensitivity examination . It will guide you about nature of infection . Tuberculosis is also a common cause of chronic cough that can takes time and require proper treatment . you should consult a respiratory and chest physician for complete evaluation of case .hope it will help you ."
},
{
"id": 188366,
"tgt": "Dental pain,excess bleeding,yet to undergo root canal treatment,consumes xarelto from long term. Recommendation?",
"src": "Patient: Hi, my 78 yr old mother takes xareltto for long term AF. She currently has severe dental pain. Its sunday and she has appt for root canal treatment tomorrow. She hasnt taken her xarelto since friday, as the last dental appointment resulted in axcess bleeding . What is a safe strong pain killer for her to take now until her appt. She has some endone tablets left over from a previous occasion some time ago, but is itsafefor her to have them? We are in Australia. Thanks Laurie. Doctor: Hello Laurie.Thanks for sharing your query.I have gone through your concern and it seems that the tooth which is paining and advised for root canal is carious one and the treatment is started to remove the the pulp so bleeding occured in last appointment. bleeding in exess can be because of underlying infection or the medicine xareltto also causes unusual bleeding from wound. I would advice you to consult her physician before stopping with the xareltto medicine and taking pain killer. As pain killers like Endone can increase the bleeding.Take Care.Thank you."
},
{
"id": 60880,
"tgt": "What causes a hard lump on the side of the leg after an injury?",
"src": "Patient: I tripped and fell today and banged the side of my leg really hard on the sidewalk now a big knot (lump) has formed on the side of my thigh. It hurts when touched and I was just wondering what causes a knot like that to form? Is there anything I can do to help it go down? Doctor: Hello and Welcome to \u2018Ask A Doctor\u2019 service.I have reviewed your query and here is my advice.The hard lump after injury suggests underlying hematoma formation.Though primary home relief with ice application can be obtained, I suggest you to consult your doctor and get prescription of antibiotic and anti-inflammatory drugs for best results.I hope I have answered your query. Let me know if you have any further questions. Regards,Dr. Bhadresh Lakhani"
},
{
"id": 144088,
"tgt": "What causes bleeding in the brain?",
"src": "Patient: hello, pt has bleeding on the brain can t drill to relieve pressure they say. like in a coma state but he has tears in his eyes running down his face when talk to him but doesn t move , feels his hand squeese your finger once in a while not a strong grip. nurses says its not a anurisom not a heart attack tests show. what could it possible be? i am 5 hours from the pt to go see him. i work in a medical field and i am checking things out on the computer. pt is on a breathing machine at this time also. Doctor: Hi, I am Dr.Bruno. I have read your question with care and understand your concerns. Let me try to help you Question : What causes bleeding in the brain?Bleeding in Brain occurs more commonly due to 1. Injury to Head 2. Problem in Blood Vessels. This can be congenital problems like Arterio Venous Malformations or acquired problems like Atherosclerosis.3. High Blood PressureSome times it can be a combination of various factorsHope you found the answer helpful.If you need any clarification / have doubts / have additional questions / have follow up questions, then please do not hesitate in asking again. I will be happy to answer your questions.Let me know if I can assist you further.Take care."
},
{
"id": 145584,
"tgt": "What causes tiredness and tingling sensation in legs and arms?",
"src": "Patient: I am experiencing some off balance issues, and some tingling in my legs arms and face, and a bit tired, I am seeing a physic therapist for a posture problem and he has me doing some excerise for it, but I did have this balance issue once 2 months ago, went to clinic they did blood work nothing came back and it went away, now it is back, should I be really concerned Doctor: Vertiginous syndromes are more often caused by ear pathologies. Since you had this problems during or after physical exercise, where the head maybe has moved abruptly you should consider an ENT visit. Regarding the tingling sensation and tiredness it can be connected with different issues, like for example diet or physical activity changes, if you're smoking or not, different ages, etc. You may check the cell blood count, blood electrolytes, muscles and hepatic enzymes at first."
},
{
"id": 149217,
"tgt": "Diagnosed with benign neoplasm of pituitary gland and craniopharyngeal duct. Have headache and memory loss. Guide?",
"src": "Patient: Hi, I have been diagnosed with a benign neoplasm of the pituitary gland and craniopharyngeal duct..The size of the tumor is 1.4 cm. I'm afraid the result will be surgery. What do you think? I meet with a Opthomologist tomorrow to have a vision screening done b/c my vision is blurred. I have extreme headaches and so much memory loss. I will be seeing a neurosurgeon as well ASAP. I'm scared. Doctor: Hi,Thank you for posting your query.Yes, your guess is correct. Most likely, your neurosurgeon would advise you for a surgery. However, there is no need to worry, as the tumor of pituitary is most often benign (such as pituitary adenoma or craniopharyngioma). Once they are operated, you would improve for good. The most likely cause of your headaches and memory loss is the tumor.Also, in most cases, the pituitary surgery can be done through nose, without opening the skull. I hope it helps.Please get back if you require any additional information.Best wishes,Dr Sudhir Kumar MD (Internal Medicine), DM (Neurology)Senior Consultant NeurologistApollo Hospitals, Hyderabad,My personal URL on this website: http://bit.ly/Dr-Sudhir-kumar My blog: http://bestneurodoctor.blogspot.com/"
},
{
"id": 121125,
"tgt": "How long will it take for strained calf muscle to heal?",
"src": "Patient: I strained the outside of my calf muscle, I was out for a month and the doctor just gave me the green light to start running and playing basketball again. After my first fun/basketball practice, my ankle hurt. Now I can run on it without limping. Can you tell me how long this will hurt and how to reduce the time? I want to start playing basketball again. Doctor: Hello,I read carefully your query and understand your concern. The length of time needed for\u00a0healing\u00a0a\u00a0calf strain\u00a0depends on the severity of the\u00a0injury. A typical grade I\u00a0calf strain\u00a0will\u00a0heal\u00a0in seven to 10 days, a grade II\u00a0injury\u00a0within about four to six weeks, and a grade III\u00a0calf strain\u00a0within about three months.I suggest to rest to reduce the recovery time.Hope my answer was helpful.If you have further queries feel free to contact me again.Kind regards! Dr.Dorina Gurabardhi General &Family Physician"
},
{
"id": 128691,
"tgt": "What causes left-sided ankle and foot pain after a shoulder injury?",
"src": "Patient: About a year ago I slipped getting out of my commercial truck from the top step. About 18 inches down my left heel hit the bottom step which flipped me sideways. I landed on my left side with my shoulder level dropping a total of about 8 feet. The most immediate pain was my left ankle and foot. I ended up with a football sized bruise on my hip a week or so later. It took more time to realize that I may have done something to my neck. When I cough or sneeze hard my spine pops hard near the most protruding vertebrae just above shoulder level. Is it possibly fractured or more likely just out of alignment? Doctor: I understand your concernafter falling down most probably you get ankle injury which is twisting injury its either high ankle sprain or low ankle sprain,the difference between them are in high ankle sprain there may be huge swelling need more time to heel may reach 6 month it needs rest, ice pack, splint then physiotherapy, while low ankle sprain is simpler need 2-4 week rest with splint and pain killer.I hope you got it."
},
{
"id": 116755,
"tgt": "What is the suggested medication for pneumonia?",
"src": "Patient: I have been taking levoquen for pnemonia got pnemonia when I aspherated in egd test and acid went to lung, while I was in hospital heart went to 124 went to follow up and my family physician said give it till I over this cause it was 114 in his office, should it be ok to wait? weight- 177 height- 5 6. Doctor: Hello,Thank you for your contact to healthcare magic.I understand your health concern, if I am your doctor I suggest you that take the medication as your doctor has suggested you. You should think for doing culture and sensitivity from the sputum. Once you have report take treatment accordingly.I will be happy to answer all your future concern. Thank you,Dr Arun TankInfectious disease specialist.Wish you a best health at health care magic."
},
{
"id": 3688,
"tgt": "How to get pregnant?",
"src": "Patient: Hi Dr, I,m 27 yrs old,trying past 1 year for conceive,my periods are irregular past 6 months i'm taking metformin, i have no pcod and my overies are normal when scanned..my thyroid report also normal..my ht is 5'2\" and wt is 58 kgs..and i'm doing normal regular wrk...my last period on sep 26 till now i didn't got my periods...normally my cycle is 35-40 days...pls help...thank u Doctor: HiDr. Purushottam welcomes you to HCM virtual clinic.I have gone through your query. I think I have understood your concern, I will try to suggest you the best possible treatment options.It is good to hear that reports done by you are normal. If you have missed your periods, get UPT done.If it is negative I will suggest you-1] Husband Semen analysis2] Tubal Patency test like HYSTEROSALPINGOGRAPHY, within 10 day after your next periods.3] Your USG with special attention to volume of both ovaries.4] Day 2 of cycle- get blood tests for FSH,LH, PROLACTIN,AMH.5] Iab FERTYL 50 mg for 5 days from day 2 of the cycle with ovulation study from day 10 of the cycle to know the day of ovulation and having sexual activity from day 10 to 20 at least on alternate days.6] If you are on METFORMIN, It is advisable to indulge in healthy diet and regular exercise regimen. This will help you more than medicines.I hope my answer helps you.Thanks."
},
{
"id": 50439,
"tgt": "Having low GFR, kidney flushing, frequent urination. On thyroid medication. had moderate to chronic kidney disease. Reason?",
"src": "Patient: My GFR was as low as 50 and should never be below 60. With symptoms such as kidney flushing 24/7 and peeing 45 times a day and falling asleep my Doctor was not concerned but he upped my thyroid meds to much and told me my body would adjust in 6-8 weeks. I asked for my blood work the last two years and found I had moderate to chronic kidney disease.I do not no the cause have seen a specialist a couple of times. He let me know we have a kidney dialysis in our hospital.And there is medication I can take. Is there no other recourse? I drank chinese herbs for 1 1/2 years it just stablized it and stopped the symptoms and did not fall asleep. I have since found something I can take daily all natural and never felt better.Do not know how long it will last. But for now it is great. Thankyou just want to know the cause. Doctor: Hi. According to the information that you provided, you are correct in mentioning that you have moderate chronic kidney disease i.e, CKD 3 ( GFR= 30-89). Well at this level of GFR dialysis is not initiated, but preparation like AV fistula formation can be considered if the rate of fall of GFR is very steep. There are many causes of chronic kidney disease as I will mention but Herb Nephropathy seems likely in your case, according to the history you provided. Among the notable causes are Diabetes, Hypertension, Chronic Glomerulonephritis, Obstruction of the bladder or urethra or both of the ureters, Autosomal dominant Polycystic Kidney Disease, Chronic Pyelonephritis due to reflux nephropathy, Interstitial nephritis ( One of the cause is Balkan or herb nephropathy)and some other rare inherited diseases.Regads"
},
{
"id": 200637,
"tgt": "What causes discharge of yellowish solids during ejaculation?",
"src": "Patient: Hello Dr. Recently, i have been noticing yellow solids coming out with my semen. Is it an indication to anything critical or bad? To add, I am 39 years old, married, and I do actually live a normal and healthy sex life! I do have a photo of a sample of that yellow solid that appeared in sperm. Thanks, Omar Doctor: Thanks for asking in healthcaremagic forum Small black/greyish threads are occassionally found in semen. You might seen coagulated semen. Please send the pic of it for further suggestions. All the best. You can go for semen analysis if you are concerned."
},
{
"id": 156014,
"tgt": "What relieves the muscle spasms after a chemotherapy?",
"src": "Patient: My sister is undergoing chemotherapy for stage 4 lung cancer. She is on pain killers for the involvement in her bones. Her main problem has been nausea for which her doctor has perscribed anti-nausea meds as well as H2 blockers. She feels that her problem is more muscle spasms than true nausea. Any ideas? Doctor: HIWell come to HCMAs long as the chemotherapy and side effect is concern this comes around even without the medicine depends how much patient can bear this, still for the muscular spasm the best drug would be Tab Chlorzoxazone and this can be taken as per requirement, take care."
},
{
"id": 4521,
"tgt": "Are there there any chance to get pregnant with first follicular study?",
"src": "Patient: Hi DoctorToday my CD 12th i went for follicular study. my doctor told me that today they are in correct size as required.. they told me to come on day after tomorrow i.e 14th of my period. she told me that she will give injection.. meanwhile she told me that today i.e 12th day don't have sex, but tommorow have sex i.e 13th day and informed me to come on 14th i.e.day after tomorrow... is that the right time to have sex? you told me that after injection, it will take 2 days for ovulation, but we are starting on 13th day, i.e before taking injection for follicle rupture.. is it clicks... first time i went for follicular study... i don't have any previous knowledge about this.. i have ectopic on two years back and left tube was removed.. now she told me that rhight ovary follicles was developed and try to conceive.. my right tube was good.. suggest me how to conceive in the first cycle study itself? is there any chance to get pregnent with first follicular study? Doctor: Hi,Thanks for the query. Usually after giving injection, ovum can get released between 24 to 48 hours. The doctor might have told you to plan intercourse on 13th day because:- Ovulation can occur before injection also, as the follicle reached sufficient size. - Sperms can survive for 48 to 72 hours. So, better to plan the intercourse on 13th to 16th days. This will increase the possibility of pregnancy. The possibility of having pregnancy in one follicular study ranges from 25% to 50%.So, hopefully you may get pregnancy this time if all other factors are normal. Start taking folic acid supplementation from nowonwards. Take care."
},
{
"id": 190838,
"tgt": "What is the remedy for lower palate pain in one with torus palatinus ?",
"src": "Patient: I have torus palatinus now I get pain in lower part of roof of my mouth and I cannot tolerate hot or cold food or drinks it is very painful what do you suggest? Doctor: Hello, Since how long you are suffering from the torus palatinus. Was that by birth? Also since how long it started paining. Have you earlier got any consultation with some specialist and had any Radiographs/X-ray done for that. If not then do get the radiographs done by advice of some General dentist/Maxillofacial surgeon in your locality. My advice to you is to get it surgically removed as early as possible as this is the only cure for the cause. You might be need a palatal obturator if the size is big. Hope this helps. With Regards, Dr. Himanshu Govil Gurgaon"
},
{
"id": 28834,
"tgt": "How can recurrent infection in the vagina be treated?",
"src": "Patient: Hi i am 36 yrs old, Since 1 and half month i hv had recurring episode of infection, symptoms were severe nodryness and due to which i had redness and itching which progressed to white cheese discharge,my gynaec gv me AFc kit for 1 month for once a week for four weeks, but it recurred on the last week , again visited twice with prescribed for canditral once a day for 6 months second day of period and lubic gel for dryness, but i am still getting severe dryness and does not seem to subside , which is affecting me, though after the initial treatment i dont hv any cheese curdled discharge now.pls help Doctor: Hello,The management of recurrent vaginal infection is consisting of multiple levels. Take care of local parts with gentian violet sitz bath, aloe vera gel application, and avoid vaginal sex.The systemic approach includes culture sensitive antibiotics, anti-histamine as per the requirements, a balanced, nutritious diet to boost up the immunity, and maintenance of hydration levels.Hope I have answered your query. Let me know if I can assist you further.Regards,Dr. Purva Patel"
},
{
"id": 199769,
"tgt": "How to increase the sperm motility?",
"src": "Patient: Hello Doctor, Good day. I have gone through sperms test.in the result it is found voulue is heigh but less mortality.may i know the reson for this and how can i get this iaaue resolved..is it a serous issue? I dont have a habit of smoking and drinking...pls advie Doctor: Hi, dearI have gone through your question. I can understand your concern. You have normal count and volume. Your motility is low. Normally there should be more then 50% actively motile sperm. Low motility is called as asthenozoospermia. It could be some infection. You should take a course of antibiotics. Consult your doctor and take treatment accordingly. Hope I have answered your question, if you have doubt then I will be happy to answer. Thanks for using health care magic. Wish you a very good health."
},
{
"id": 211766,
"tgt": "Have PCOS. On mephrate. No periods yet. Am stressed personally. Treatment for stress and tension?",
"src": "Patient: Hi doc, I have pcos and i am 30, my doc recommended me taking mephrate its been more than 10 days but still i havnt ha my periods also i have lot of stress related to personal and office life...please can you suggest i always have negative thoughts because of this. What can i do to releive myself from stress and other tension. Doctor: Hi,You can expect a withdrawal bleed 3-7 days after completion of treatment with hormones. For the specific treatment of PCOD, there is a protocol of investigations to be done and treatment is planned according to the specific problem detected. These include a complete hormonal assay, glucose tolerance test, DHEAS levels, tubal patency if trying to conceive, trans-vaginal sonogram to view the uterus and ovaries. Maintenance of a healthy body weight, regular exercise and a healthy diet go a long way in the treatment. Meditation can help ease out the stress; please take the support of a psychologist if needed. Take care."
},
{
"id": 38856,
"tgt": "What causes coughing with dry lips and scalp after a viral flu?",
"src": "Patient: I have recently been sick with a virus that has lingered for a couple of weeks. Mostly gone, but a weird cough, and now I am extremely dry; especially my lips scalp, my feet, and certain are of my body. I am drinking lots of water, and getting plenty of rest. What is going on? Doctor: Hello,Welcome to HCM,The history and symptoms are suggestive of viral infection and most of the viral infections are self limiting and it will subside by itself. If the symptoms are not improving, you need to undergo lab investigations like CBC, routine urine examination and blood culture.For your symptoms I would suggest you to follow1.Take adequate rest.2.Drink plenty of water.3.Tab Paracetamol, three times a day for 5 days.4.Eat food rich in proteins to boost up the immunity. The symptoms will improve with these measures, if it is not improving you may need antibiotics.Thank you."
},
{
"id": 137856,
"tgt": "What causes ankle pain and swelling?",
"src": "Patient: I have pain in my left leg starting at the ankle. The pain is especially there at night. I had ankle surgery 7 years ago on the ankle. I still have screws in the footy. The pain seems to originate where the screws are located. Could I have developed a blood clot there? there is a continuous swelling. Doctor: Dear patient Past history of surgery on ankle with implant in situ and pain now may be due to recent infection or screw breakage. Since swelling is also there infection is more likely. I would like to advise Xray of involved ankle anteroposterior and lateral views to confirm diagnosis. Visit radiology center nearby you and get it done. If Xray is showing changes of osteomyelitis you need to consult expert orthopaedic surgeon nearby you and get yourself examined. Complete blood counts also needs to be done and of there is increased total leukocyte count it also corroborate infection. All the best.."
},
{
"id": 179591,
"tgt": "How can a colicky baby with gasping be treated?",
"src": "Patient: have a 6week old granddaughter she seems colicky she was breast fed for about 4 weeks then to formula similac then to a soy formula and now nutrimigen been on it for 7 days still quite cranky and she does the little squeaky noise like gasping quite frequently any suggestions? her older sister had to go on nutrimegen also but in a week she was great Doctor: HiThanks for writing to health care magic.Gastica drops can be given at 5 drops thrice a day .Colic pain may last for few months before settling.Ensure proper burping after each feedWishing your child good healthRegardsDr Arun"
},
{
"id": 157015,
"tgt": "What is life expectancy when suffering from stage 4 lung cancer?",
"src": "Patient: Hi, I have a friend who has recently been diagnosed with Stage 4 lung cancer. Doctors said he has 2 tumors but only one is cancerous. Before doctors discovered the lung cancer, he had pneumonia. He is going to be getting treatments for three months, how bad is the cancer? what could his life expectancy be? I hope you can help me because I am not familiar with all this. Thank you Doctor: Average survival in stage 4 lung cancer would be 6months to one year according to clinical studies. but in practise we see patients living longer that than. life expectany also depends on how bad the disease is, histology, treatment plan, drugs used, dosages, mode of administration etc..."
},
{
"id": 207080,
"tgt": "What causes slight delusional state while on Quitipin?",
"src": "Patient: dear doctor i have been on a slight delusional state twice. once on 2003 and then in 2006 while withdrawing from an anti-psychotic. its been 5 years now and i have been on quitipin 100mg (quetiapine) . ive been to 3 psychiatrists and now i have a counsellor . none of them have been able to tell me what i have as in ive not been termed anything. so i just wanted to know if it serious or not. Doctor: Hello, May i ask you to please describe what you really mean by delusional state and what do feel, behave in that state?Antipsychotic withdrawl itself does not cause delusional state. But yes premature stoppage or inadequate dosing for a mental disorder can cause person to return back to previous state of mental illnes.Psychotic/delusional disorder usually require higher doses of quitipin (more than 100mg).Thanks."
},
{
"id": 1776,
"tgt": "What causes unruptured follicles even after having HCG injections?",
"src": "Patient: Hi, I m married since from 4 yrs. Once i got aborted due to water got discharged during 5th month i.e 16th Jan 2010. My husband has infertility he is under treatment and it is normal. Since from march 2010 i m trying to get pregnant but it has nt happened. Till July 2010 follicle ws rupturing bt later till today it is nt rupturing even after giving HCG injection. Kindly tell me the reason Doctor: Hello and Welcome to \u2018Ask A Doctor\u2019 service. I have reviewed your query and here is my advice.It happens due to an inaccurate dose of HCG injection or it can be due to some follicles problem known as LUFS. So you can try with a higher dose of HCG. Ask your doctor regarding this. This can happen by chance also. So it will not happen repeatedly. Hope I have answered your query. Let me know if I can assist you further.Regards,Dr. Khushboo Priya"
},
{
"id": 40702,
"tgt": "How can infertility be treated?",
"src": "Patient: I am 30 year old woman, married for 8 years and have a baby of 6 years old. I had cyst on my right ovary and I conceived for the first time after ovarian cystectomy. After 2 and half year of my first child I conceived for the second time but I had miscarriage on 3rd week since 2 years I am planning to have a second baby Doctor performing many test like HSG thyroid prolactin amh etc. all seems normal but I fail to conceive, every time I have to take medicines to get my period recently I am diagnosed with hsv cmv and rubella having value rubella igg 2.34 hsv igg 1.77 cmv igg 4.32 I am undergoing treatment for above but I am so much concerned about conceiving can you please help Doctor: Hi I think you can go for ovulation induction with timed intercourse for conceiving for 3 to 6 cycles if everything is fine. If it doesn't work, you can go for IUI for 3 cycles. Talk to your doctor regarding this. Hope it helps."
},
{
"id": 173570,
"tgt": "What is the treatment of fever vomiting and loose motion in children ?",
"src": "Patient: my son 1.5 yrs old having fever from last night. Did vomiting and had loose motion in the middle of the night and the fever receded. But from morning loose motion with yellowish sticky material. after light food (over boiled rice) 4 times motion in an hour. What medicine can be administered at this stage. Doctor: HiWelcome to the HCMIt seems to me that your child has an episode of viral gastroenteritis. Don't worry, it can be managed by the following measures:1. Keep him well hydrated by regular feeds and oral rehydration therapy such as lemonade, soups or butter milk. If the fluid loss is excessive, than hydrate with oral rehydration solution available OTC after proper constitution.2. You may give him Syp. Emeset or Domperidone for vomiting episodes.3. Also, start him on Syp. ZnD or Zinconia for next 14 days. It contains Zinc which is good for early recovery.4. In case of any fever episodes, you may give him acetaminophen syrup.In case his oral intake drops significantly and he looks dehydrated then do contact your pediatrician.Hopefully this will help you. In case of any further questions, you may contact me."
},
{
"id": 121120,
"tgt": "Could shoulder pain, tightness and neck pain be due to head hit?",
"src": "Patient: Could arm, shoulder, and slight neck pain be caused from a hit to the back of my head? I hit the bottom back of my head fairly hard roughly nine days ago. I had symptoms of lack of focus, headaches, anxiety.. Have started to feel pain go through my arm- a weird tingle tired pain mostly forearm and tricep and shoulder soreness.. gets worse at night. Bottom of head hurts, not exactly the neck but the neck is a little tight.. I am 20 year old female in good health condition.. Had one other concussion two years ago.. not sure if this was a concussion but had similar symptoms. Most of this pain is in the right shoulder; it is extremely tight. I can feel the left shoulder getting slightly tighter. Doctor: Hello,I read carefully your query and understand your concern. Your symptoms seem to be related to the head injury the The shoulder and neck pain it is probably to a muscular strain.I suggest using anti inflammatory medications such as Ibuprofen to relieve the inflammation and pain. I also suggest using cold compresses for local application. The symptoms should relieve in a few days. Hope my answer was helpful.If you have further queries feel free to contact me again.Kind regards! Dr.Dorina Gurabardhi General &Family Physician"
},
{
"id": 896,
"tgt": "Can pregnancy happen after having non penetrative sex?",
"src": "Patient: the other night me and my girlfriend were messing around which i truly regret. Her pants came off and mine was out. I never ejaculated, but there was a small amount of pre-ejaculation on the tip. I never touched her vagina with my penis and im pretty sure there i never touched it with my fingers but if i happend to have just a small amount on my hand when i put that inside of her, what are the chances of her being pregnant? Doctor: Hi, I think there is a little chance of pregnancy. She can take some emergency contraceptive if possible. If her periods get delayed, do a urine pregnancy test. If positive consult a doctor if you don't want pregnancy. If negative wait for your periods. Regards Dr khushboo"
},
{
"id": 37696,
"tgt": "Suggest remedy for painful bumps on clit area",
"src": "Patient: I have a painful bump only when touched, bump in my clit area, I was recently tested for stds and this bump just recently appeared. I am taking prescription for a spider bite, but the bump appeared before I started taking them. no discoloration no discharge just a small flesh colored bump. I ve been with the same man for the whole time. what is it? is it something that I should worry about? Doctor: Hi3,Welcome to HCM.A single painful bump on the genital area suggests bacterial infection. I would suggest to keep the area clean and dry and apply over the counter antibacterial cream over the bump three times daily. It should reduce within 3 days. If it increases in size or cluster of bumps appear then you need to get the bump examined by a GP nearby you. No other medications required.Thanks."
},
{
"id": 95742,
"tgt": "My lungs hurt really bad",
"src": "Patient: yes hello i drank some water today and it felt like it went down the wrong pipe because it starting to hurt really bad and now the left side of my chest hurts when i inhale i do not know if it went to the lung or heart how can i make this pain go away, and am i going to be ok or should i go see a Doctor? Doctor: Hi welcome to Healthcaremagic Hi, Dennis do not get worried, even if a small drop of water enters trachea there will be cough and water will be thrown out... since you does not have such symptoms this shows water is not entered trachea, it is in esophagus only... if there is breathlessness, severe chest pain, then you can visit doctor, otherwise not necessary... Hope I have answered your question.. Takecare..."
},
{
"id": 196176,
"tgt": "Suggest treatment to reduce masturbation habit",
"src": "Patient: hi,im a 19 year old guy,stuck up with masturbation for last 5 years...but for last two years i ve been doing it thrice a day...please suggest me any drug with no side effects or eating habits to reduce it to thice a week........i do it when i see a shirtless guy of age 12 to 25.... Doctor: avoid watching porn.try to mingle more with your friends and avoid being alone.have a stable mind to control masturbation.you can get it by doing meditation and yoga.have a healthy foods."
},
{
"id": 177922,
"tgt": "Suggest remedies for cement consumption in a child",
"src": "Patient: Hi Doctor, My sixteen months old daughter eaten cement from the wall last day. Now its more than 24 hours but she has not gone for potty. But she is behaving normal and drinking milk happily. But I m very afraid because her stomach is not clean yet. Should we take her to a doctor for checkup or we should do some some remedies to clean her stomach so she can go for potty. Kindly Pls advice. Doctor: If she is not having any problem, no need to do any thing...just wait for 1 more day she will pass the stool . If she eats and drinks well no need to worry..."
},
{
"id": 21372,
"tgt": "What are the symptoms of heart attack?",
"src": "Patient: I have had bad tinnitus for a months. Also headaches daily. My right ear hurts inside. I'm on doxosozin and ramipril for high bp. I had a brain haemorrage years ago and am worried. Also, a few weeks ago I had an excruciating pain rise up my back which got tighter and tighter. It finally eased off. Could this have been a mild heart attack? Doctor: Hi ThereHeart attack presenting with a back pain is very rare scenario. In your case it may or may not be an heart attack. to confirm this i would like to advise you to consult a cardiologist personally and get an EKG and a Echocardiogram done. As hypertension is a risk factor for coronary artery disease so its better to get evaluated properly.i wish you good health"
},
{
"id": 91794,
"tgt": "Will Ibuprofen suffice to treat muscular pain in upper abdomen?",
"src": "Patient: My friend is having some muscle pain on her right upper abdomen and it is still bothering her even though she has been resting, putting heat/ice on it, and taking ibuprofen. Her pain level range is 1-3. Is there anything you can recommend that she does to help with whatever she did? Doctor: Hi,Thanks for writing to HCM.Muscle pain if not getting relieved by ibuprofen, then you need to add a muscle relaxant to it. Ask you pharmacist of a painkiller and muscle relaxant combination and if this is recurrent, then she need vitamin supplementation also.Dr. Ashish Verma"
},
{
"id": 198181,
"tgt": "What causes swelling in penis after masturbation?",
"src": "Patient: Hi, my problem is after mastubaration i find that my pennis has swollen, there isnt any pain . it lasts for 15-30 min. whats the problems. it is happening only some times but not all the times.. i observed that it is bcz of hard mastubaration is it true? or is there any problem else with me? Doctor: DearWe understand your concernsI went through your details. The hard masturbation may have caused blockage to the lymphatic channels in the skin which is leading to this swelling. This prevents the normal fluid that goes back toward your body from making it to the body and leads to the swelling. Vigorous masturbation can cause this. As long as the swelling seems to be improving, I think you should be okay. As you said, it happens whenever you masturbates hard, the problem could be summarized as above. I would recommend that you try to put some warm compresses to the penis to try to get the lymphatic channels to open up to let the swelling go down. If it continues to get worse or does not go away in the next 2 to 3 days, then you should see a urologist as soon as possible. Hope this helps.If you require more of my help in this aspect, please use this URL. http://goo.gl/aYW2pR. Make sure that you include every minute detail possible. Hope this answers your query. Further clarifications are welcome.Good luck. Take care."
},
{
"id": 207149,
"tgt": "Is there a psychoactive medication related to anger management?",
"src": "Patient: Is their a psychoactive medication that has shown to be effective in the treatment of physical & verbal outbursts related to underlying anger? I believe I have read something recently about a disorder known as Explosive Personality Disorder & was inquiring in relation to that Dx. Doctor: DearWe understand your concernsI went through your details. I suggest you not to worry much. Anger is an emotion. Controlling emotion in any way is dangerous to your mental as well as physical health. Even drugs do control your anger. What you need to understand is, you should never try to control anger, but should alter the reasons behind the anger. Know your anger to get rid of it.If you require more of my help in this aspect, Please post a direct question to me in this URL. http://goo.gl/aYW2pR. Make sure that you include every minute details possible. I shall prescribe the needed psychotherapy techniques.Hope this answers your query. Available for further clarifications.Good luck."
},
{
"id": 56266,
"tgt": "What causes high bilirubin levels?",
"src": "Patient: I have high level of total BILIRRUBIN TOTAL-2.8* BILIRRUBINA DIRECTA \u00a0\u00a0\u00a0\u00a0\u00a00.3* BILIRRUBINA INDIRECTA 2.5* and GLOBULINA 3.61* else evrything is in range sgot /sgpt . SGOT\u00a0\u00a0\u00a0\u00a0\u00a026.7\u00a0\u00a0\u00a0\u00a0\u00a0 SGPT 30.1\u00a0\u00a0\u00a0\u00a0\u00a0U/L\u00a0\u00a0\u00a0\u00a0\u00a0 FOSFATASA ALCALINA\u00a0\u00a0\u00a0\u00a0\u00a0238\u00a0\u00a0\u00a0\u00a0\u00a0 PROTEINAS TOTALES\u00a0\u00a0\u00a0\u00a0\u00a07.68\u00a0\u00a0\u00a0\u00a0\u00a0 ALBUMINA\u00a0\u00a0\u00a0\u00a0\u00a04.07\u00a0\u00a0\u00a0\u00a0\u00a0 GLOBULINA\u00a0\u00a0\u00a0\u00a0\u00a03.61*\u00a0\u00a0\u00a0\u00a0\u00a0 RELACION A/G\u00a0\u00a0\u00a0\u00a0\u00a01.12 Doctor: Hi there,Thanks for using HCM.I went through your post.Your bilirubin is only mildly increased. The indirect component is more than the direct component. The liver enzymes are within normal limits. The A/G ratio is normal.There are only two possibilities. The increased bilirubin could be due to either Gilbert syndrome or hemolysis.You can get reticulocyte count done to know which one of these two is the cause.Regards."
},
{
"id": 17718,
"tgt": "What causes low heart rates?",
"src": "Patient: I have had a problem with my heart beat rate changing from laying down at about 70 to 80 bpm, then sitting 90 to 120 or so, then to standing around 146 to 165 and my monitor says EE for error if higher at times. I have been sick with many other symptoms, blurry vision off and on, no appetite, Major Depressive Disorder, Migraines. While preparing medical records I have for an attorney for social security, I noted one doctor had a CT scan taken in 2005 and it said I have a posterior fossa arachnoid cyst, but he didn t think it was anything important. I did some research and found several articles that lead me to believe this may very well be what is causing my problems.... I had also be reviewing cases of Neural Mediated Hypotension which I seem to have quite a few of the same symptoms as some of the writers. Could this cyst being over the nerve area leaving the brain cause me to have Neural Mediated Hypotension? Doctor: Hello, I passed carefully through your question and would explain that your symptoms seem to be related to postural intolerance. But the arachnoid cyst in not related to your symptoms. For this reason, I recommend performing a Head-Up Tilt test and checking also thyroid hormone levels and HbA1C for possible diabetes. Hope I have answered your query. Let me know if I can assist you further. Regards, Dr. Ilir Sharka, Cardiologist"
},
{
"id": 162638,
"tgt": "How much weight should a baby put on a week?",
"src": "Patient: I am a social worker and do adoption work. I have a client and am concerned baby is not gaining enough weight. Below is info re: baby. Should I be concerned? Baby born weighing 7.8 on 9-7-17 weighed 7.6 on 9-13 weighed 7.6 again on 9-21 weighed 7.13 on 10-5 weighed 8.2 on 10-11 They tell me baby is sleeping 8 hrs at night and dr told them not to wake her to eat. Doctor: Hello and Welcome to \u2018Ask A Doctor\u2019 service. I have reviewed your query and here is my advice. Baby of 4 months ideally should have a weight of 6.5kg. As your baby have 8.2 kg it is normal. normally baby born above 5 kg is a rare case. your baby is 7.8 kg at birth. As far as the baby is drinking milk ,passing urine and stools and is playful nothing to worry. Babies have to sleep min 8 hours and it is quite normal. but you can wake up your kid in 2 hours and feed. It will help to improve weight. Hope I have answered your query. Let me know if I can assist you further."
},
{
"id": 82995,
"tgt": "Bumps, chronic headaches, flashing lights triggers anxiety. Anxiety and depression. History of lupus in the family. Could it be lupus?",
"src": "Patient: Since I was a kid I've had joint pain that has been getting worse... I get chronic headaches and I'm very sensitive to light. Flashing lights trigger my anxiety but I've never had a seizure. I'm tired all the time sometimes caffeine doesn't even help. I've had anxiety and depression since I was a little girl. when i was a child i would go to theme parks all the time and when I would wear shorts I would get a red blotchy rash type deal all over my legs, but not when I wore pants... The hear makes me vomit and it gets difficult for me to breathe. Yesterday I broke out with bumps all over my neck it has now spread to my chest shoulders upper back and up to my chin. Lupus runs in my family and as more time goes on I am thinking my symptoms may be from lupus. My cousin who has a more severe case of it is convinced I have it. I'm looking for a second opinion. Doctor: Hi,Going by your symptoms and the strong family history of Lupus, I would also be highly suspicious of lupus in you. Please get yourself screened for the same. Most useful tests would be the ANA and the anti-dsDNA antibody tests.I wish you good health."
},
{
"id": 203919,
"tgt": "What is the treatment for tingling penis after unprotected sexual contact?",
"src": "Patient: A few months ago i was on a trip to Montreal and went to a erotic massage parlor. Might have been the biggest mistake of my life now that I look at it. im so depressed about this any way. the woman gave me a hand job and nothing more. It was a fully naked one so her naked body was on my body but not in a super sexual way but more sensual. she rubbed her breast on my but her vagina never touched my skin from what i remembered. After my session I had a tingle on my penis that wudnt go away. I figure it was due to the warmed oil that she used. A month or so later I had a few little bumps show up near the shaft of my penis. I was scared and went to the clinic immediately. The doctor instantly told me that this was Herpes and I would have it for life and i could infect a woman and destroy her ability to have kids and that I needed to take some pills for a week and then buy more later on. I left the clinic feeling like committing suicide because of this doctor . It felt like he was reading from a play book or something. Any way the bumps turned to scab and went away. Also my anus began to hurt at times as well. the tingly feeling in my penis never went away.. it stayed there for a few weeks and might have went down a bit but always came back. I began changing my eating habits and did a full body cleanse. I did not take the medicine that the doctor ordered me too. I just didn t feel comfortable doing so after his horror story. after a month and a half everything went away and the tingle went away for a bit. but wud sometimes happen again after i masturbated or had sex with a condom. i went back to eating the same and smoking and not working out as i did. and after maybe 2 months now I have the bumps again. I wish i could say that this is just sum fungus from my lotion or from something else. I still haven t got a official test which I will be getting next week. can you help me to understand why i have the tingle constantly and is that something normal with herpes cases. Also why is it that right after the massage i had this tingle. i m really depressed and I am somewhat of a well known figure in my community so i dont know what to do with my life at the moment. Doctor: HiItching or tingling on genitals or around anus is a symptom of herpes.There is no cure for genital herpes. But the symptoms can be lessened and prevented with treatment. Treatment can also reduce the risk of infecting others.Antiviral drugs like acyclovir is used for treatment.You can get a remission by treatment. When symptoms recur, they usually come on during times of emotional stress or illness. That's because, during these times, your body's immune system may be less able to suppress the virus and keep it from becoming active. There are also things you can do to avoid passing the virus to other parts of your body, as well as to other people. Take these steps:Don't kiss when you or your partner has cold sores.Avoid oral sex when either partner has oral or genital sores.Don't have genital or anal contact when any sores are present.Wash your hands with soap and water after touching infected areas.Don't wet your contact lenses with saliva. There are things you can do to relieve the discomfort and severity of the symptoms during an outbreak. Home treatments include:Take painkillers such as aspirin, acetaminophen, or ibuprofen.Bathe sore areas with a warm salt-water solution twice a day (1/2 teaspoon salt with 1/2 pint warm water).Let air circulate around the sores by wearing loose-fitting clothes.Put an ice pack on the affected area. Wrap the ice pack in a towel or piece of cloth.Get plenty of rest.hope this may help youthanks"
},
{
"id": 34794,
"tgt": "Is it normal to feel sudden chills?",
"src": "Patient: I am a 49 yr old, relatively healthy, Asian female who started walking for exercise about 3 weeks ago. However, every day for the past week, right around the same time (about 130 or 230 pm) I start getting the chills. When I take my temperature, it is usually around 99.5 to 99.9. I don't have any aches or pains or infections that I know of. I have been taking Tylenol for the fever, but the next day this happens all over again. I have lost about 6 lbs since I started walking. Doctor: Hi,Thank you for your query. I can understand your concerns.Rise of temperature with chills almost at the same time regularly for 1 week points towards underlying infection.However lack of other constitutional symptoms like aches or pains suggest indolent nature of illness. In Asian countries,one need to rule out TB. Other possibility is lymphoma. You need physical examination and baseline investigation like complete blood count,peripheral smear for malaria parasites ,Urine for routine analysis and ESR (fasting).Regards Dr. T.K. Biswas M.D.Mumbai"
},
{
"id": 148954,
"tgt": "Had craniotomy, meningioma in falcine area. Chronic pain behind eye, acute sinusitis. On penicillin, keppra. Anything serious?",
"src": "Patient: Thanks for responding, I had a craniotomy 6 weeks ago, meningioma in the falcine area. Chronic pain behind right eye, was just seen by gp and have acute sinusitis. On penicillin as well as Keppra, I feel a lot of pressure on the top of my head and left ear area as the sinus infection is breaking up, is this a dangerous situation, given the recent craniotomy? Doctor: HI Thank for asking to HCM You have recent history of craniotomy and right now have symptom which quite related with your surgery If I would be your docotor then I would certainly ask for ct imaging to rule out the possibility of this if your gp is suspecting it as sinusitis and put you on antibiotic then you finish this course if it does not go away then will be required investigation take care of your self and be in touch with HCM have good day."
},
{
"id": 8546,
"tgt": "How to treat facial skin tan?",
"src": "Patient: Dear sir I m Sandhya, 20 yrs old. I had a fair complexion. But i tanned a lot after attending swimming classes during my 7th grade summer holidays. I ve got back only dusky complexion and it exists till now. Will i be able to regain my fair complexion? If so how by natural ways? Plz reply me to this mail Id Doctor: Hello and welcome to healthcaremagicFirst and foremost, you should use a sunscreen regularly to protect you skin from further tanning. Apart from that practicing sun protective behavior like carrying an umbrella, avoiding outdoor activity between 10:00 AM and 3:00 PM etc go a long way in preventing further tanning.UV A part of the electromagnetic spectrum is mainly responsible for tanning. Therefore, a good sunscreen should have broad spectrum UV A coverage in addition to UV B coverage, in order to be able to prevent tanning and it needs to be applied in sufficient quantity and every 2-3 hourly.A sunscreen with minimum 30 SPF would be adequate. It should mention both UV B and UV A coverage. Therefore it is important to select the right sunscreen for desired effectsIt should be applied in the following quantity:--3ml for face and neck(a little more than 1/2 teaspoon)--3ml for each arm(a little more than 1/2 teaspoon).A sunscreen should be applied every 2-3 hours.A skin lightening cream containing one or more of the skin lightening ingredients e.g kojic acid, glycolic acid, Arbutin, Magnesium ascorbyl phosphate, Liquorice etc once or twice daily, can be used to gradually fade away the tan.You could also undergo a few sessions of skin lightening peels (Yellow peels/Retinol CT peel) OR Glycolic acid Peels, from a dermatologist to improve the complexion.regards"
},
{
"id": 79627,
"tgt": "Is bad dry cough and pain under ribcage concerning?",
"src": "Patient: had a bad dry cough for seven days last three days have pain under my right rib cage, pain all the time but very bad when I cough or deep breath have trouble sleeping as hurts continuous on right side, should I go to the hospital to get it checked out. Doctor: Thanks for your question on Health Care Magic. I can understand your situation and problem. Yes, you should definitely consult doctor and get done chest x ray to rule out lung infection and pleural effusion. You are having continuous coughing and right sided chest pain, worsening by coughing. So possibility of pleuritic pain is more. And it is seen commonly with pneumonia and pleural effusion.So chest x ray is needed to rule out these possible causes. If chest x ray is normal then possibility of musculoskeletal pain is more. So start antihistamine drug for cough suppression. Start painkiller and muscle relaxant drugs. Avoid heavyweight lifting and strenuous exercise. Avoid movements causing pain. Apply warm water pad on affected site. Don't worry, you will be alright. Hope I have solved your query. Wish you good health. Thanks."
},
{
"id": 84353,
"tgt": "What are side effects of taking rinifol and nutrolin b plus together?",
"src": "Patient: My local doctor has prescribed Rinifol as well as Nutrolin B plus capsules both 1 each morning and night after meals. would like to know if i have to take both as one of my friend said that both are same. Is there any side effects if both consumed together. I have to take these capsules for a month. Please suggest Doctor: Hi,Age, gender, purpose of taking these medications, etc not mentioned. Rinfol and Nutrolin-B plus contains same active ingredients of probiotic and multivitamins and commonly prescribed in the treatment of mouth ulcers, diarrhea, antibiotic associated diarrhea, and irritable bowel syndrome. Either of these may be used alone. Their common side effects include headache, allergic reactions, nausea, abdominal distension and bloating.Hope I have answered your question. Let me know if I can assist you further. Regards, Dr. Mohammed Taher Ali, General & Family Physician"
},
{
"id": 24610,
"tgt": "What causes BP to increase in the evenings while relaxing?",
"src": "Patient: In the morning, my bp is around 120-130 over 78 ish. In the evening, it can be around 140 over 93, even in the 150's some evenings. I take Lisinopril 50 mg in the morning. What causes my blood pressure to rise in the evenings when I am relaxed, rather than in the morning when I'm getting ready for work? Should I try my medicine at a different time, perhaps at lunch? Will exercise or stretching in the evening help lower it? Doctor: you consult ur physician and inform this may be the dose of drug or action it decreases by evening but it should be taken In morning only if bp tablet night taken means it will cause night hypotension"
},
{
"id": 5517,
"tgt": "Tubes blocked. Overweight. Need surgery to get pregnant?",
"src": "Patient: hey i am 38 years old. my tubes are blocked and i mean both, i don't want surgery to be done to me. I am 250 pounds and i am wondering if the weight has anything to do with my tubes being closed. i just had a baby 2009, and want to have another one but my doctor keeps telling my i cant unless i take a surgery p.s i know its too much information but i really could use your helpthanx Doctor: Hello, Thanks for your query. There is no relation between tubal block and obesity. Tubal blockage could be due to adhesions because of pelvic inflammatory disease or previous surgeries. For RECANALIZATION you should undergo surgery. Laparoscopic recanalization could be risky because of your weight. All the best. Hope you get pregnant fast. I hope this information has been both informative and helpful for you. In case of any doubt, I will be available for follow ups. Wish you good health. Regards, Dr Arif"
},
{
"id": 147176,
"tgt": "Does taking aricept helps to treat dementia with history of ulcer?",
"src": "Patient: my 94 year old mother was just prescribed Aricept. I don t think she should be on it. she doesn t have alzheimers but is showing signs of dementia short term memory is going and a little paranoid. she has had ulcers long time ago and has a very acidic stomach. Is this a medication she should be on? Doctor: Hi,Thank you for posting your query.I have noted your mother's symptoms.I agree with you that we can not be sure that she has Alzheimer's disease (AD), and she needs detailed evaluation. Evaluation would include CT scan of brain, thyroid profile, serum vitamin B12 level and neuropsychological testing.If it is confirmed as AD, then, it would be useful to give her aricept tablets.Acidity is not a contraindication for taking aricept (donepezil) tablets.I hope my answer helps. Please get back if you have any follow up queries or if you require any additional information.Wishing you good health,Dr Sudhir Kumar MD (Internal Medicine), DM (Neurology)Senior Consultant NeurologistApollo Hospitals, Hyderabad, IndiaClick on this link to ask me a DIRECT QUERY: http://bit.ly/Dr-Sudhir-kumarMy BLOG: http://bestneurodoctor.blogspot.in"
},
{
"id": 60838,
"tgt": "What causes a lump on the neck along with fatigue and dizziness?",
"src": "Patient: My english is not my native language, but I ll try to explain. Female 27. I got this weird long lump on my neck in the right side, it pulses and seems it have grown. I feel tierd all the time and I have noticed few days ago my pupils are not the same size. I feel also weak for some reazon, I am usually strong women, but even carrying groseries from store is hard. Stepping up the stairs is exausting. Dizzyness and light headed. I got chekt by doctor, but he could not say whats wrong. He saw my pupils are little diffrent, but reacts to light fine. He seemed not to take me seriously. Also im anxious about my health Doctor: Hello and Welcome to \u2018Ask A Doctor\u2019 service. I have reviewed your query and here is my advice. The lump on the neck mandates possible lymph node enlargement or others. The fatigue and dizziness narrate low hemoglobin, deficiency of vitamins, or other elements. I recommend to consult in follow up with a photo picture of the lump as an attachment for best possible evaluation. Hope I have answered your query. Let me know if I can assist you further. Regards, Dr. Bhagyesh V. Patel"
},
{
"id": 127959,
"tgt": "What causes sudden pain in the lower back, abdomen and legs?",
"src": "Patient: I was cleNing the house and it was very hot so I was perspiring. I took a shower and was feeling fine. I was laying on the bed doing puzzles with my son and when I got up I felt a pain in my upper back (left thoracic) and began to get cramps in my lower back, lower abdomen and legs (menstrual like)My heart seems to be racing as well. Should I be concerned? Doctor: HelloThank you for trusting HCMDear it may be due due spasm of back muscle while standing.Take rest for few days and use cold and hot compression. Take tablet aceclo- MR twice a day for five days.If symptoms not improved please consult your orthopedician he will examine and treat you accordingly.Take care."
},
{
"id": 124492,
"tgt": "Does flying increase blood clots of injuries?",
"src": "Patient: hi, I fractured my ankle in both tibia and fibula yesterday.The orthopedic dr reduced the fractures and piut on a soft cast, I need to fly home in 4 more days which the dr said would be ok. He wants me to visit my home orthopedic Dr on return to home. I may need to have surgery so my home Dr will make that decision, Is my risk of blood clots increased by flying? Is there anyway to lessen that risk? It will be a 3 hour flight.. Doctor: Hi, 3 hr flight will anyway won't be a risk factor. So don't worry. Keep moving your toes actively and try to keep your limb above your heart level. Hope I have answered your query. Let me know if I can assist you further. Regards, Dr. Anuj Gupta, Spine Surgeon"
},
{
"id": 48674,
"tgt": "What is the suggestion for dialysis in diabetic patient?",
"src": "Patient: dear sir, i m from nepal. my father of age 58 is a dibetic patient since long ago. from last month he have swelling in his both legs. he also take blood pressure and heart medicine because here doctors said that his hearts also swell due to fluid in kidney. his creatinine level is 6.5 Doctors said that he soon go to dialysis and cud not transplant kidney due to his age please suggest what to do Doctor: Maintain correct blood sugar levels required during dialysis period, it was seen by dialysis doctor, no need to worry..it can be handled easily.dialysis is must for him now. .58 years is not a cause to avoid transplant. .we r doing transplants to 65 years also..transplant can relieve s all the problems if u can affort..go for dialysis with out tension. .hope i a helpful"
},
{
"id": 184045,
"tgt": "What causes swelling on the middle tooth and brown stuff on it?",
"src": "Patient: i had oral surgery,the dentist thought i had sists behind my front teeth.i had the skin pulled back and 3 stitches put in.i had this done last monday ,the swelling on 2 our going down but the middle one has not and has brown stuff on it .should i go back to the dentist Doctor: Hello, Thank you for consulting with HCM.After surgery the swelling takes at-least 5 days in reducing and even pain is also present. It reduces after taking proper medications.ould waiThis brown stuff can be the granulation tissue of socket as it is healing phase. So better you should wait for 5- 7 days and still if the swelling is present then go to the dentist for follow up.Hope it will help you."
},
{
"id": 197059,
"tgt": "Is masturbation harmful?",
"src": "Patient: I have read multiple articles and websites about masturbation seeking an answer regarding whether or not it is harmful, and if so in what ways. Unfortunately, some said it's harmless some harmful and listed several things such as hair loss, weakened eye site etc...I would like to know from a medical and professional point of view, which is it? Is masturbation harmful? If so how? What is a good masturbation frequency? I am 22 years old, never been sexually active yet, I weight 155 lbs and I am 5'9''. Doctor: HelloThanks for query .Based on the facts that you have been indulged in masturbation and now worried about so called anticipated problems in future discussed by general population . First of all there is myth in a mind of common man that excessive masturbation is the reason of all sexual problems or various general health problems But I would like to state that it has been discussed in scientific forums all over the world and proved scientifically that masturbation does not have any negative effect on any organ or system in the body .Masturbation is safest way to achieve sexual satisfaction without a female partner .One can safely masturbate twice a week .Dr.Patil."
},
{
"id": 103461,
"tgt": "Small red, itchy bumps from face to legs. Having dog at home. Any ideas?",
"src": "Patient: I have small red very itchy bumps with a small white center (before they are itched and popped) from my face to my legs. Not many on my back but mostly on my arms face neck chest and stomach. I haven't eaten anything different or used any different products.i do have dogs but didn't see any fleas or bed bugs. A doctor i saw today thought it may be a cleaning product reaction or something IM allergic to that rubbed off of the dogs. What do u think? Doctor: hi -the skin rash you have is mostly allergic in origin .It becomes important to break the allergy cycle .while tackling such condition. any chlorpfenaraminne maliateor levocetrizine preparations along with betmethasone tabs(if needed) &ointments are usually priscribed. The next stage is to recognise &avoid sourse of allergy. Here allergy detection tests&desensatisation thereafter if needed. -The anathere cause for the such a rash is Scabies infection. Here the Rashes are more in skin folds, seen in family members/friends because it spreds by skin contact.THey are rarely on face. That's why I have kept is as a 2nd possibility.You need to consult a skin specialist incase of doubt. Hope the advice given is helpful to you. Thank you."
},
{
"id": 194770,
"tgt": "What causes thickening of foreskin and bleeding?",
"src": "Patient: I am 59 year old male. My foreskin is getting thicker, harder and I can't control where my urine goes. It goes everywhere. I have also found blood spots in my underwear a couple of times, months apart. I had a urinalysis and blood was found. I had a couple of heart attacks last year, but my heart is doing great! I had 2 stints put in. I take plavix, metropolol, simvistatin, lisinopril, and an 85mg aspirin daily. I will be seeing a urinologist in a month, but I am worried. Would like to have some idea what may be going on. I am overweight. I have arthritis. I don't drink or smoke and have never used drugs. I do use tylenol 3 and percoset for pain of the arthritis. I am not very active due to the arthritis pain. I have a degenerated disk in my low back, along with arthritis there. I have no pain urinating or in my kidneys. I have no difficulty urinating, except for the control of where it goes because of the foreskin. I have no problems with erections. Can you give me a little insight into what may be some of the things that may cause this bleeding? Thanks. Doctor: Hello, It may be due to conditions like chordae or phimosis. Consult a urologist and get evaluated. Most probably you will require a circumcision. Hope I have answered your query. Let me know if I can assist you further. Regards, Dr. Shinas Hussain, General & Family Physician"
},
{
"id": 50339,
"tgt": "Have thyroid disease and kidney insufficiency. Taken Methamazole. Elevated liver enzymes after PTU. Any ideas?",
"src": "Patient: I am a 58 year old female with thyroid disease and recently diagnosed kidney insufficiency. I did a six-month course of methamazole which treated the hyperthyroidism however my GFR dropped to 44 while on the drug. They stopped the methamazole and my GFR came up to 54. They started PTU for the hyperthyroidism and now my GFR is back down to 44, and my liver enzymes are elevated. An ultrasound shows bi-lateral hydronephrosis. My GP is now sending me to a urology surgeon for evaluation. Any ideas on what is going on? Doctor: Hi, the drugs you are using are showing hepatotoxicity, and renal toxicity, as you are recovering after stopping the drugs, after consulting the nephrologist you better stop the drugs, or change to some other alternative for hyperthyroidism. Thank you."
},
{
"id": 72016,
"tgt": "What causes frequent chest congestion?",
"src": "Patient: I've been getting serious chest congestion that last up to 7 days too two weeks every now and then. About a year ago I got bad chest congestion that lasted up to two weeks. It got better and I went on with my daily tasks, about 4 months later I got terrible chest congestion again, I got better within two weeks again. So a year later I got chest congestion again, it lasted one week. About a month later (A few days ago) I got bad chest congestion again, it gets to the point where I can't walk I'm so out of breath and I'm worried that it might be more then just chest congestion, I do have severe allergies. And it get's bad sometimes when the mold is high. I'm wondering do you think I might have COPD or any other serious condition?. I'm young and I don't smoke. My family doesn't have a history of diseases so I'm at a loss. Doctor: Thanks for your question on Healthcare Magic.I can understand your concern. Since you are having severe allergies, possibility of asthma or allergic bronchitis is more likely for your symptoms. So consult pulmonologist and get done clinical examination of respiratory system and PFT (Pulmonary Function Test).PFT will not only diagnose asthma but it will also tell you about severity of the disease and treatment is based on severity only. You may need inhaled bronchodilators (formoterol or salmeterol) and inhaled corticosteroid (ICS) (budesonide or fluticasone).N acetyl cystine (NAC) is also beneficial to relieve congestion.Hope I have solved your query. I will be happy to help you further. Wish you good health. Thanks."
},
{
"id": 101146,
"tgt": "What is the best treatment to get rid of dust allergy?",
"src": "Patient: i m allergic to dust.. i hv been consuming L montus since january of this year as prescribed by d doctor, its very helpful fr my problem ( continuos sneezings, difficulty in breathing which is very painful n tiring).. but if i miss d tablet fr evn a day , m back to my same allergic problems.. m tired n irritated by this whole thing.. kindly help me out here if anybody listening to this.. Doctor: Hello dear,Montus L (pharmacologically Montelukast+ Levocetirizine) is helpful in managing the condition of Allergic rhinitis.Levocetirizine is an anti histamine, which acts as an anti allergic in controlling the acute episode.Montelukast acts as a maintenance therapy to relieve symptoms of allergic rhinitis.Both taken together provides protection from acute episodes + maintains a symptom free period.If you do not consume the medications, their effect reduces & you get prone to acute allergic attacks.This condition is related to the hyper-sensitivity of your immune response.It can only be managed symptomatically.Complete cure will need complete shut down of your immune mechanism, which will make you prone for severe life-threatening secondary infections.So, kindly continue the medications as prescribed by your treating Doctor.Wishing you a good health.Take care."
},
{
"id": 158060,
"tgt": "Smoker. Noticed red veins in mouth. Feel irritated. Have dental problem. What could be the problem?",
"src": "Patient: I am a smoker (about 9 years), when I shine a light in my mouth, I can see all the little red veins in my mouth. My palate-pharyngeal arches are very red, like they are irritated, but I cant tell if it is all the veins or something like erythroplakia. I have numerous dental problems as I haven't been to the dentist in several years. Its almost as if the tissue inside my mouth is transparent.....what is the likely problem. I'm worried I'm developing mouth cancer or something, but I don't have any growths or sores. Doctor: Hi,As you are a smoker, having this type of problem, never neglect.Consult oral surgeon and get examined.Let examination report be normal but rule out Sub-mucous fibrosis or Leucoplekia.Stop smoking.Take vitamin E and B.complex supplements.Ok and take care."
},
{
"id": 156316,
"tgt": "What are the survival rates in type 4 stomach cancer patient?",
"src": "Patient: My Sister was just diagnoised with type 4 stomach cancer.It has spread through all her organs..She had very few symptoms.She went in when they discovered a tumor the size of a lemon and were going to remove her stomach and spleen and when they opened her up there were not expecting what they found..They had done a pet scan and it did not show all of this cancer,They have suggested chemo after they do radiation but i want to know with it being so far advanced how long even with chemo will she have..Our family is in total shock..She is 67 but looks like she is in her late fifties..Please can you help me with some answers..Sincerly..Marylynn Doctor: Hi. Its very sad to hear about your sister's health. Stomach cancer is notorious for producing sub centimeter peritoneal metastasis (peritoneum is the inner lining of abdomen and the coverings of stomach and intestines). Sensitivity of PET scan is low for detecting sub centimeter metastasis and so a diagnostic laproscopy (putting a camera into the abdominal cavity) to rule out peritoneal deposits before radical surgery is recommended.Stage 4 stomach cancer has poor prognosis with a 5 year survival rate of less than 5% and a median survival of 9 to 12 months.Palliative chemotherapy after radiation therapy will continue till disease progression or till the time patient can tolerate chemotherapy.I hope your questions have been answered. Thankyou for choosing Healthcare Magic."
},
{
"id": 5020,
"tgt": "Trying to conceive. Have light spotting, lower back pain, stomach pain, delayed period. Am I Pregnant?",
"src": "Patient: Hallo,My husband and I are trying to conceive. It is 5months now and still nothing. In the beginning my period were irregular, but the last 3 months it has been stable on 32 days. This month I am on day 38 and still no period. I have done a HPT on day 33 but it was negative. Yesterday I had light spotting but only on toiletpaper and still nothing has happened. Usually I have very bad low back pain as well as cramps on my stomach. I only have a slight pain on the right side of my back and only when I sit. I don't have any other symptoms, is it possible that I'm pregnant? Doctor: Hello,To confirm pregnancy, you need to undergo one blood test for beta-hCG. Above features may be associated with premenstrual symptoms in the absence of pregnancy.You need to do unprotected sex around ovulation time to maximize the chance of pregnancy. You can use ovulation prediction kit (LH kit) to predict your ovulation time in mid cycle.For natural conception, both of you have to try for minimum one year. If you fail in this time, you need to undergo several investigation including husband semen analysis to rule out underlying etiology. You can consult with infertility specialist regarding this.Good luck."
},
{
"id": 17220,
"tgt": "Suggest treatment for high BP, chest heaviness and sore arms",
"src": "Patient: i have regular high blood pressure with meds felt like passing out 164/50 pulse 50, then it will go to 80/50, have been feeling very ill, chest heaviness and a very bad new sorew all over arms,i have had a hospital stay for bad staph from a surgery is it coming back.some fever low grade,heart or chest feels weird Doctor: Hello, I suggest you go to the hospital and do some routine blood tests, a chest X-ray, an ECG, a cardiac ultrasound. Hope I have answered your query. Let me know if I can assist you further. Take care Regards, Dr Anila Skenderi, General & Family Physician"
},
{
"id": 160446,
"tgt": "Could fever and watery stools in a baby be due to amoebiasis?",
"src": "Patient: my baby has been having on and off fever for 5 days until now and keeps pooping all the time. Hes poop is always watery. Got him tested for fecalysis, the results were Redcells:0-3hpf, pus cells:5-7hpf no ova nor parasite seen,fat globules:rare. I called my doctor to tell him the result and he said it might be amoebiasis and needs to be tested again.i needed a secnd opinion, pls help me on this thanks Doctor: Hi, You may try to repeat the stool analysis again and you may give your child streptoquin or antinal syrup to make the stool solid. Take care. Hope I have answered your question. Let me know if I can assist you further. Regards, Dr. Salah Saad Shoman, Internal Medicine Specialist"
},
{
"id": 69374,
"tgt": "What causes painful red lump which occurs on the body?",
"src": "Patient: I'm a healthy 28 year old male with no history of any lump issues. For slightly over a month now I've had a red lump under my arm. That alone isn't the alarming thing. What is getting to me is that it seems to drift around. It will be a raised red lump one day, then seem deflated, then gone. A few days later, it will have shifted locations and be raised again. it's not very large, about the diameter of a pencil eraser and raise only about the height of a regular small zit. But it does hurt, and it seems to leave a trail when it moves not unlike a stretch mark. I'm really concerned, and have not seen a dermatologist. Doctor: Hi.Thanks for your query.Well, this looks to be hidradenitis, a condition of the skin of the armpit. One gets the exact symptoms you have. Take a course of an antibiotic. Long-term doxycyclin helps a lot. You need to clean the area with Povidone iodine 4 times a day.IT is most likely to get healed up with this. Occasional patient needs Surgery to excise the affected skin area. You may not need it."
},
{
"id": 132501,
"tgt": "What causes sudden weakness and lightheaded?",
"src": "Patient: Two days ago I was sitting in my chiropractic waiting room. Suddenly I felt very weak and I could not move my arms. Both of them felt very heavy. I felt light headed. After my adjustment, I felt weak and light headed. It took all I had to drive myself home. I have not had any symptoms since then. Doctor: Hi Hope this message finds you in good health.I have gone through your complaints and understand your concern.I generally see many cases like this at my clinic.such symptoms are caused by hypoglycemia or gastritis or a generalised weakness.if it happens again,get a blood investigation done.You should eventually get back to normal in coming few days.Nothing to worry about.\u00a0\u00a0\u00a0\u00a0\u00a0I hope your question has been answered.If you have any follow-up queries,feel free to consult me anytime.Thanks,Take care,God bless."
},
{
"id": 31140,
"tgt": "Should anti rabies injection be taken after injury from dog?",
"src": "Patient: I got injury of index finger by a stray puppy. skin is intact as no breakge is seen, but blood is collected underneath the skin. dere ws no ozzing blood frm site of injury. Do i need to take anti rabies injection? Puppy had many sibilings n dey died due to some insect bites 2-3 months back. He is the only one survived. he is healthy but nt vaccinated. . Doctor: Hi & Welcome.You seem to have Category II of exposure.Therefore you need to take FOUR doses of anti rabies vaccine on days 0,3,7,14 & 28. You may stop taking the treatment if animal remains healthy throughout an observation period of 10 days.Hope this answers your query."
},
{
"id": 80054,
"tgt": "Is there any problem for having headache and chest discomforts after inhaling the bleach smell?",
"src": "Patient: Hi, my friend mixed harpic bleach with normal bleach in a bucket as we were cleaning our flat about 3 hours ago, after mixing the bleach there was an instant extremely strong smell that made us cough for a while, I then emptied the bleach down the sink and we stayed in the room for about 3 hours without realising it is dangerous. I have a very strong burning sensation in my nose and am extremely tired with a headache, and my friend just has a pain in her chest when she breathes deeply. Is there anything to worry about? Doctor: Thanks for your question on Health Care Magic. I can understand your's and your friend's situation and problem. Both of you are having these symptoms due to inhalation of bleach fumes. These fumes can cause inflammatory reaction in nasal and pharyngeal mucosa. It can also cause acute bronchitis due to inflammation in airways. Usually these symptoms are self limiting with avoidance of fumes and inhalation of fresh air. But if symptoms persist for 1-2 days then consult doctor because both of you may need antihistamine, anti inflammatory drugs with inhaled bronchodilators and inhaled corticosteroid (ICS). Since all these drugs are prescribed medicines, you need doctor 's prescription for them. Hope I have solved your query. Wish both of you good health. Thanks."
},
{
"id": 54683,
"tgt": "Suggest treatment for abdominal discomfort, bloating, elevated cholesterol and liver enzymes",
"src": "Patient: My daughter has been experiencing abdominal discomfort and bloating for the past 2 months. She has been on Accutane 40mg a day for 3 months 2 weeks. The bloating has become very uncomfortable and embarassing to her as she is an athlete and not used to having this problem. She was 105 lbs when she started Accutane and has gained 15 lbs. Sometimes she looks puffy to me. Her liver enzymes and cholesterol are elevated . Should she stop Accutane based on these IBS symptoms? Her skin looks beautiful, the doctor wanted to double her dose this month and I got angry with him because he didn t seem concerned about the IBS sympoms. What should we do? Doctor: Hello! Thanks for putting your query in HCM. I am a Gastroenterologist. I appreciate your concerns regarding your daughter. These IBS symptoms are not due to accutane.For bloating I will advise you:1. Avoid food that increase gas like beans, pulses, cabbage, peas2. Avid food that increase acid productions like tea, coffee, citrus and spicy and fatty food3. See if milk make your symptoms worse then please avoid it4. Take capsule containing simethicone\u00a0\u00a0\u00a0\u00a0\u00a05. Also take a proton pump inhibitor like rabeprazole along with a prokinetic agent like domepridoneI hope I have answered you query. Wish you a good health"
},
{
"id": 83080,
"tgt": "Nodules reappeared after lupus vulgaris treatment, had taken daily doses of anti-TB medicines, advised restarting medicines. Suggestion?",
"src": "Patient: 4 year back, i develop cutaneous maculopapular lesion 5 cm below the back of knee. size was 7cm long and 1 cm width. for 2 years i was treated for dermatitis , psoriasis and fungal infection . then i develop pleural effusion . aspirate was negative pcr for m. tuberculosis. i received fixed dose daily regime of anti tuberculosis for 2 + 6and half month. lesion improves. biopsy done . histopathology shows healed scar of lupus vulgaris. but after that some nodule appears. my doc. says restart antitubercular drug. what to do please help me???? Doctor: it may be the same thing. a rebiopsy may be indicated"
},
{
"id": 45302,
"tgt": "Have bilateral polystic ovaries. Could i be pregnant ?",
"src": "Patient: i had a ultrasound done an the report stated that my uterus is anteverted , both ovaries shows small follicles in the periphery, the right ovarian is 9.1cc and the left is 10.2cc , no free fluid was found in the pouch of douglas. the impression is bilateral polystic ovaries. i would like to find out if i ll be able to get pregnant and steps needed to do so.i m 29yrs old Doctor: Hi Welcome to HealthcareMagic Yes you ll be able to get pregnant. Polycystic ovaries are common in women. you must find out if you have only polycystic ovaries or polycystic ovarian syndrome? Though ovarian vol. suggests PCOS. To confirm, you need to get few hormonal tests done, then the further treatment can be decided. The treatment of polycystic ovaries/syndrome depends on many factors like age, if pregnancy is desired at present or not, weight, presence of insulin resistance. To increase your chances of getting pregnant you should loose some weight, your doctor may put you on anti diabetic drug metformin, Clomiphene and letrozole are drugs used for Ovulation induction. Consult a gynecologist who ll guide you to step by step treatment. take care"
},
{
"id": 186744,
"tgt": "What causes sudden change in mouth or tongue taste buds?",
"src": "Patient: dear sir i have a problem some in mean time suddenly my mouth or may be in toung taste buds are change and ,i fell like a vomiting and at in morning at the time of tooth brush i have vomit nad my stomach acid come out side some time they have bitter taste ...... Doctor: Hello, Welcome Thanks for consulting HCM, I have gone through your query, as you have bitter taste and feeling like vomiting for this I can say it can be due to digestion problem due to that depapillation of papillae is there causing bitter taste. You can do is take one capsule Vitamin B complex once daily for one week . Do warm saline gargle two - three times a day Consult physician for digestion problem that will help you more in treatment.Hope this will help you."
},
{
"id": 145155,
"tgt": "What causes dizziness and blurriness in eyes after a hit on the head?",
"src": "Patient: My 5 year old son was running and hit the top right of his head on the door there is a bump on his head the size of a half dollar put ice for the last 2 hours swelling has gone down but now he says he sees spots all different colors should I be concerned and take him into the ER to be seen he is fine otherwise not dizziness vomiting Doctor: Immediately rush him to hospital, there might be a internal bleed which compressing the artery supplying nerves /area of vision.hence the visual changes.if there is associated vomiting and headache and any signs of weakness of any body parts or seizures,then bleed in head is confirmed. Rush him to nearby neurosurgeon, he will take an ct scan/MRI and admit him.all cases of head trauma are to be kept in observation for 48 hours even if patient is healthy as bleed can start in any of these hours.Don't panic most bleeds are treated by medicines alone without need of surgery.we have got excellent treatment if treated within time without having any complications and prevent damage."
},
{
"id": 204966,
"tgt": "How can depression along with anxiety and insomnia be treated?",
"src": "Patient: I am struggling with depression that got worse after open heart surgery and an emergency bowel surgery 6 days later to save my life. I am seeing psychiatrist and taking meds to help with anxiety and sleep disorder. At my appointment Friday my doctor to me she could not help me since efforts have not improved me. What do I do now that she has told my physical condition is all in my head. Doctor: HelloCan you please give the details of medications which you are taking?In general, I would suggest you to practice relaxation exercises like deep breathing and meditation.Hope to hear from you soon.Thanks"
},
{
"id": 205275,
"tgt": "Suggest medication for depression while on NSAIDs",
"src": "Patient: I am currently taking NSAID's for chronic back and neck pain with the occasional vicoden when it gets very intense. What antidepressant would be best for my depression? I've read SNRI's are better as SSRI's effects are diminished with NSAID and pain meds. Doctor: Hi Sir/MadamWelcome to Health Care MagicI can certainly understand your concern about the combined effect of NSAIDs and SSRIs , as you said it may reduce the effectiveness because of some cytokine receptor occupation by both the drugs, so there will be less receptors available for SSRIs But I can suggest you can start on desvenlafaxine which is an SNRI , as due to the combined effect of norepinephrine and serotonin, you can get good results...But it is advisable not to stop this medicationBefore 6-9 months , even if you feel better as you may get relapse of the symptoms...If you stop the course incompletely, hope this information would help youPlease post your further questions if any, I will be always available to answer themThank youPost your further questions if any, I will available to take up your follow up questionsThank you"
},
{
"id": 114200,
"tgt": "I got a back injury but it is not swelling. Should i go to doctor?",
"src": "Patient: I just fell and landed on my back across the top stair of the deck. It hurts really bad, do I need an x ray or just ice it and see what happens? It was about waist height and lower back hip level 2 spots. The muscle that goes from shoulder blade towards lower back hurts when I touch it at any point. My arms and legs are moving fine. Not numb or anything. Doctor: hello take arnica 200 two doses for 2 days one dose in the morning and second dose at night..its a homeoapthic medicine so safe,sure n faster results.. if u r able to move wit no seveare increasing pain then x ray is not required. ice it n do not strain it .. godbless.. drvigsclinic@gmail.com"
},
{
"id": 9689,
"tgt": "What are the causes for waxy skin in a diabetic patient ?",
"src": "Patient: hi Iam huda may you tell me causes of waxy skin in diabetes patients? Doctor: Hi, In diabetes due to metabolic disorder skin loses its luster and due to fat mobilization skin becomes thin and shining So skin is thin and shining it looks waxy."
},
{
"id": 49856,
"tgt": "Inflamed liver. High glucose, potassium high. What is going on?",
"src": "Patient: I have given blood to my doctor three times since Feb. 2012 for inflammed liver . The 2nd blood test 6 weeks from the first in Feb., my test results came back with glucose high. So I gave blood 6 weeks from the second test stating salts, sugar, liver, proteins , kidneys normal. Potassium is high, so I went Monday 7-23-2012 and gave blood again awaiting for results. What is going on? I have had no alcohol since 7-10-12, stopped taking omega Q, Tylanol of any kind, stopped smoking 8-8-08! Doctor: Hello sir/madam,You results are suggestive of probable diabetes.Hyperkalemia needs evaluation . Kindly meet a physician with FBS, PPBS, ABG, Urine spot potassium ,repeat LFT, S creatinine. You are most welcome to get back to me in case of further queries."
},
{
"id": 23108,
"tgt": "What causes a small aneurysm in the place of stent?",
"src": "Patient: My age is 40years, weight is 87 Kgs. I have undergone angioplasty in Jan 2011 due to acute MI . Two medicine eluted overlaping stents were placed in Right Coronary Artery. Due to some weakness of muscle or due to stenting, a small anurysm came into existence at the place of stent. Doctors advised to wait as to whether it remains static or grows. But I am worried and want to take some herbal medicine/s so that anurysm is cured / removed and artery comes to its normal shape. Advise some herbal medicine having no chances of any side effects. Doctor: sorry but there is no herbal medicine to cure aneurism ,aneurism means a defect in wall of artery arising out of wrong placement of stent or weak endothelial musclature around artery!you have to keep your blood pressure in normal range in order to prevent rupture of aneurism!A angogram repeat should be done to see the nature and extent of aneurism.As off now there is no herbal medicine socumented to cure aneurism.thank you!"
},
{
"id": 200228,
"tgt": "What causes white spots on penis?",
"src": "Patient: Hi my name is Harry I would like to know well I have this little white spots on my penis that came on there for about six months now I went to see a doctor and he told me that its normal not to be messing with it because I would always put hydrogen peroxide on it when I first saw a couple of little white spots and I would also scratch it and more little white spot would come out it seems to not keep growing or getting grosser since I have left it alone I also would poke and take out the little white spots wich I also stopped doing just leaving it alone am I ok? Doctor: Thanks for asking in healthcaremagic forum Do not use hydrogen peroxide as it can cause sloughing and irritation. White spots are fordyce's spots, which are visible sebaceous glands and are harmless.Please consult dermatologist if it is of cosmetic concern only. All the best."
},
{
"id": 188378,
"tgt": "Pockets inside the bottom lip after using smokeless tobacco, difficult to speak after saliva build up. Advice?",
"src": "Patient: I have used smokeless tobacco for several years off and on and have formed pockets along the inside of my bottom lip. Since I ve realized this I ve quit! Is there any way to reverse the pockets or at least the size? Saliva tends to build-up in these pockets and I have to consciously control the flow as I speak to people. I ve also noticed a slight issue with pronouncing words with R s, F s, W s and the like. Doctor: Hi,Thanks for asking the query,According to your clinical symptoms i suppose that you have developed localised periodontitis in your lower front teeth region.Its is very nice to hear that you have quit with the habit.Go for complete mouth scaling and polishing and curettage.Take lukewarm saline and antiseptic mouthwash rinses.Maintain a good oral hygiene brush twice daily with a soft bristle toothbrush.Take a course of multivitamin suplements.Hope you fins this as helpful,Regards..."
},
{
"id": 54110,
"tgt": "How to treat for stones in gall bladder?",
"src": "Patient: hi, is spasmo cibalgin or pancreozyme similar to xenical or alli. age:53, wt 120, in pretty good health except for a lot of Gall BLadder stones. has been xenical or alli for the past few years, only when i eat fatty/oily food. I take around 4-6 a month except on vacation. My mom's doctor suggested Pancreozyme or Spasmocibalgin instead. thanks for advice Doctor: Hi welcome to HCM...If gall stones are asymptomatic than no active action required. ....Here if abdominal pain or any other complaint there than its removal is necessary by surgery....You can take udiliv tablet for that ....Clinical follow up necessary....And if pain or complicated gall stone surgical removal can be from laparoscopy or open method.....Avoid fatty foods in excess. ...Heavy meal restricted ....Take care ....Advise : gastroenterologist consultation...."
},
{
"id": 114739,
"tgt": "Is it okay to drink green tea when my blood pressure is low?",
"src": "Patient: Hi,My Blood pressure is 80/40 which is considered to be low - I have headache - and doctor said its because of low BP.My query is - Can I drink Green Tea - m cautious bcz of low BP? does that make my BP lower? Will that cause any problem with electrolyte level in blood? Doctor: Hi, dearI have gone through your question. I can understand your concern. Your blood pressure is too low. Your headache is due to low blood pressure. You should drink plenty of water with salt and sugar. Drinking of green tea is okay. There is no harm with that. Hope I have answered your question, if you have doubt then I will be happy to answer. Thanks for using health care magic. Wish you a very good health."
},
{
"id": 88654,
"tgt": "What causes pain in right abdomen below the rib cage?",
"src": "Patient: hoping someone can help, i am suffering from right sided abdominal pain just below my ribcage for the last 4 days. there is a constant aching there but does increase to sharp pains every now and then, these do not seem to coincide with any activities. 10 years ago i contracted a severe case of tick bite fever whilst living in south africa, and 'heard' there may be long term effect of this, that are agrivated by stress, that may cause problems, likewise i returned from holiday in a malaria zone 6 weeks ago, however i followed protocol with my anti-malarials. any ideas? Doctor: HI.The pain in the right side of the abdomen below the rib cage, for 4 days, which is constant aching but with periodic ups and downs is suggestive of Cholecystis meaning inflamation of the gallbladder. Other probable causes can be abscess of the liver or ulcers in the duodenum and so on. This may not be related to the tic bites you had. Malaria causing pain in abdomen is called algid malaria and the pain is all over and very severe. I would suggest you to undergo an ultrasonography first and then the other blood tests including liver function tests, CBC to know the infection and all relevant blood tests.Start the treatment with antibiotics and as per the findings on investigations ."
},
{
"id": 160645,
"tgt": "How to get rid of earwax buildup in a child?",
"src": "Patient: hi! my son is 4 yrs old. he is having ear problem for almost 2 yrs. actually what happen to his eye is that, so many wax are coming from his two ears. we have consult several doctors. but there is no improvement. still i have to clean his ears every day. can you please give me an advice for this. Doctor: Hello, Wax dissolving drops like waxolytes may be effective. In severe cases manual removal may be required. Consult an ENT specialist and he will direct you accordingly. Hope I have answered your query. Let me know if I can assist you further. Take care Regards, Dr. Shinas Hussain"
},
{
"id": 147740,
"tgt": "Loss of memory post operation, brain cells died due to excess pain and trauma. Options to regain memory?",
"src": "Patient: my husband was gone through a operation in which he lost his memory due to brain cells which died due to excess pain and trauma is there anyway anything which could bring the memory back ? the operation was regarding slip disk .. is it true that with 2 slip disks he wont be able to walk ? hes going to undergo a 2nd operation soon and am very scared doctors say he needs immunity but if i wont be able to give him immunity will the operation be successful? please advice its urgent.. Doctor: you have asked 3 questions here-1)the memory can be brought back but the success depends on the amount of injury to the nerve cells.but as you have mentioned that the nerve cells have \"died\",i find it very difficult practically to say that the memory would come back if the information provided by you is what actually is scenario.i find the information inadequate and also the language used by a non medico may present a complete different picture,so i would suggest you to enclose the reports along with.2)you haven't mentioned at which level disc prolapse has occured.also it depends on the severity of disc prolapse.seeing your anxiety i would like to assure you that its very unlikely that due to disc prolapse the person remains on chair whole life.in fact rest is no more taken into account while treating disc prolapse.he is undergoing operation indicates that the prolapse was of higher degree which couldn't be managed with physiotherapy.but the purpose of operation is to put the disc back on space.BUT POST OPERATION DON'T EXPECT THE PERSON TO HAVE A LIFE EXACTLY THE WAY IT WAS EARLIER THOUGH FEW CASES HAVE BEEN REPORTED OTHERWISE.I HIGHLY RECOMMEND LOWER BACK STRENGTHENING EXERCISES POST 6 MONTHS OF OPERATION.that would take care a lot.3)operation is a stressful condition post which the immune power of body decreases to alarming extent.the immune level can be checked by seeing the blood picture.just make sure to add lots of beta carotene(vit.A),vit.c,zinc,vit.K,B vitamins,vit.D and calcium,and if possible L-glutamine and arginine.AVOID VIT.E AND HERBAL SUPPLEMENTS AS THEY MAY CAUSE HEMATOMA FORMATION.start atleast 10 days prior to surgery.surgical nutrition is important but simple.:)"
},
{
"id": 121455,
"tgt": "What does a knot on the palm below the thumb indicate?",
"src": "Patient: I have a knot on the palm side of my left hand (which i use to write with) that is right below my thumb. I have read the symptoms of carpal tunnel but i didnt see anything about it including knots. Is this something that should be seen by a doctor? My hand throbs in my thumb and wrist. Doctor: Hello,You may have a condition called TRIGGER FINGER which is a small nodule below your thumb. It's commonly seen in people with overuse of hand and fingers. It's nothing to worry about. I would suggest you to visit your orthopedic doctor or hand specialist to have it evaluated.Wish you a speedy recovery. Hope I have answered your query. Let me know if I can assist you further.Regards,Dr. Santosh S. Jeevannavar, Orthopedic Surgeon"
},
{
"id": 130899,
"tgt": "What is the remedy for severe spasms?",
"src": "Patient: I had a lower back injury in 1997. The pain is so severe & not only in that area, but all over. I had a ruptured disc & I was too overweight to have surg. Over a yr. later, I had a gastric bypass & lost 160 lbs. My left leg became totally numb, back side of lower part was always on fire, couldn t stand sheets touching me. Left foot numb & severe spasms. I was told - I have neuropathy of my foot. I had been told that once they got that piece off of my nerve, I would get my feeling back. I never did, they apologized, but said we waited too long & nerve damage was permanent. I have some bulging discs & ruptured disc, spinal stenosis, arthritis throughout spine & pelvic area, & my bottom hurts really bad too, left leg stays numb anytime I stand or walk. It has for about a yr. now become like concrete. Burning, tingling also. I was told I have muscle spasticity? I can t bear to get out of bed, I force myself. At night, my leg & toes twist, all different directions & behind my left knee. (My knee pain has been very painful since I had 2 surg. The 2nd one I had a total knee replacement. A ligament? sticks out real far behind my left knee & I can t straighten my leg or bend it when I m in bed. I can t turn over or get up. It feels like it s breaking, ever since knee surg. I scream out & fight tears. Makes me out of breath, pain so bad. The surgeon told me there was nothing wrong with my knee. x-rays didn t show anything. The other surg. told me the same thing. But, the Dr. that sent me to this surgeon showed x-rays that showed bone on bone. He couldn t believe first surg. said everything was fine. Now after 2nd surg. it s still severe & it constantly pops. I had 2 surg on right knee in 2005 & 2006. I didn t even have therapy - I did the exercises on my own. I don t have pain in that knee. I never had the problems I m having now. The Dr. said it s because I m older now. (58) when I had surg. last in May. One pain management Dr.( had many injections, tens unit, meds, PT, Pain Patches) nothing works. He wanted to put a stimulator in my spine & all the way up to cervical area. (lower back L5-S1) is all that workers comp would cover. But, they refused & Dr. said he couldn t do anything more. My upper spine would have to be covered by Medicare. I have severe head aches, can t turn my head & bone spurs on cervical spine. I am on perm. disability. I went to another Dr. (Orthopedic surgeon) & he had injection put into L5 nerve. If it helped me, he would know where to do surgery. I never felt pain as bad as this. I was trying to jump off the table. I screamed numb me. He said I did. He had to take a couple of breaks. He said he couldn t even get all the meds in there, that it was so jammed up, he had to move needle around to get in. The other Dr. said he went directly into the nerve for it to hurt like that. I was still in pain, so only 1 wk. later, another injection & it didn t hurt. The nurse that called to ck. on me, was upset & said I wasn t fully numbed. This Dr. wanted me to have major back surg. It did feel better after 2nd inject. Now, I don t know which way to go. If I have surg. I want to small incisions. I hear good & bad about the stimulator & having surgery. It s tough when I have so much pain & now lately, my fingers are getting twisted & real stiff, & once in a while, looks like a lump. Doctor: In my opinion best remedy for sever spasm is a hot bath for 40 minalso try standing on your heels with ankle dorsi flextion , extended knees while leaning forwards for 30 secounds , this should give you immediate relief Good Luck"
},
{
"id": 81193,
"tgt": "Is painful chest after being kicked on chest followed by stopped breathing for a minute concerning?",
"src": "Patient: hello im a teenage girl i was kicked at his hardest in a chest by a male wearing office shoes. the heel hit into my chest and caused me to fall to the floor and stop breathing for 1 to 2 minutes. my chest has been painful and very achey. however there is no bruising. should i be worried? Doctor: Thanks for your query. A blunt trauma to the chest may result in fracture of ribs, contusion of lungs, leakage of air from the breached lung. You must get a chest xray done. If everything is normal a course of analgesic will help you. Best wishes"
},
{
"id": 118154,
"tgt": "Suggest treatment for pneumonia?",
"src": "Patient: Been treated for pneumonia on fourth round ofAntibiotic. Have low white count and X-ray shows no improvement Today started prednisone and have been fever free for 12 hrs. Is this typicalOf diagnosis of pneumonia? 69 y/o fem with no med hx or health hx Doctor: Treating Bacterial PneumoniaAntibiotics are used to treat this type of pneumonia. Antibiotics should be taken as directed. If antibiotics are ceased before treatment is complete, the pneumonia may return. Most people will improve after one to three days of treatment.Treating Viral PneumoniaAntibiotics are useless if a virus is the cause of pneumonia. However, antiviral drugs can help treat the condition. Symptoms usually improve within one to three weeks.RegardsDR DE"
},
{
"id": 200342,
"tgt": "Does excessive masturbation lead to delayed ejaculation?",
"src": "Patient: Hi, i am twenty three year old guy. i masturbate 2 - 3 times a day since its considered the moste safest practice of sex , but its not affecting my social life in any manner. Yet does it put me at risk in having delayed ejaculation. i have read many articles about it but none could give me a satisfactory answer Doctor: Hello dear,Thank you for your contact to health care magic.I read and understand your concern. I am Dr Arun Tank answering your concern.No, it won't delayed your ejaculation.But it's possible that your amount of ejaculation might decreases.Yes its safest practice to do. You can do it very sefely.There is no harm in doing it you can do it as many long days as you can do.I will be happy to answer your further concern on bit.ly/DrArun.Thank you,Dr Arun TankInfectious diseases specialist,HCM"
},
{
"id": 138173,
"tgt": "Can De Quervain s tenosynovitis leave purple patches on skin after steroid shots?",
"src": "Patient: I had repetitive De Quervians tenosynitis and had the steroid jab in my wrist. Whilst the pain has gone from my wrist, 2 months down the line I still have a purple-y patch of skin that blisters in the sun and is constantly dry. Can you tell me if this is normal thanks Doctor: Hi injection of steroid in the sheath in Dequervians can sometimes get injected in the subcutaneous tissue causing discoloration, these are usual known complication of the drug, and since you are pain free now, just forget about it now, it will recover its pigmentation at a later date."
},
{
"id": 15078,
"tgt": "Rash around eyes, mouth, ears. Suffering from toothache, sore throat, lightheadedness. Caused by wisdom tooth or infection?",
"src": "Patient: About 4 days ago a rash broke out on my face, mainly around my eyes, mouth, and ears. I also have been suffering a toothache which is at its worst today. Im not sure if the two are related or if its a wisdom tooth coming combining with some sort of food allergy. The only different food i consumed prior was a mango about 6 days ago. I once had an allergic reaction to a raw mango i ate in Hawaii 7 years ago however that rash went away after about two days. its now day 5 and the rash has not gotten any better at all. I don't have insurance so im not sure if its worth going to a doctor to just be told to take benadryl and swish salt water. Could this be serious? Could i possibly have gotten some sort of infection from my tooth and then it spread into a rash. Im also experiencing a sore throat and lightheadedness. Doctor: HIThank for asking to HCMIt is not infection neither related with your tooth problem but it may be the hypersensitivity reaction to some elements may food, right now our aim is to rid out of this reaction and for that you need to take the second generation antihistamine and the levocetrizine would be a choice you need to take this tab, 10 mg three time in day will come around and if does not go away with this then steroid would be needed for that discus it with your doctor have nice day."
},
{
"id": 194942,
"tgt": "How can weak erection leading to erectile dysfunction be treated?",
"src": "Patient: Hi, My issue is i am not able to penetrate inside my girlfriend while having sex. My penis stands but for a very small duration. Moreover it is not that hard. I even beleive the size of penis is less. Please advice what should i do. I m 27 years of age Doctor: Hello dearI understand your concernErectile dysfunction could be due to medical causes like diabetes, hypertension, low testosterone, problem in back nerve, obesity, smoking and alcohol use (Complete absence of the morning erection suggests medical cause)Psychological cause: excessive stress, fatigue, performance anxiety, anxiety due to failure in the past and inability to satisfy the partner.I would suggest to go for complete health check up, fating blood sugar, ESR, serum testosterone level, CRP, USG of pelvis and scrotumIf all the tests will come normal then problem mostly due to psychological cause.For that you require sex counselling and consultation with psychiatric.Meanwhile , following advises will help you:Do regular exercise like walking, yoga, kegel exercise, breathing exercises.During sex, you have to give proper time for foreplay before going for penetration. Foreplay should continue even during the intercourse also. Take high protein diet, drink plenty of water.Avoid sex and masturbation when you are feeling of tired.Take proper rest and sleep of at least 7-8 hours.If problem persists then Medications likeTetrafol plus: It can help in erection and should be taken after complete physical evaluation as mentioned above. It will give promising results with daily intake up to 2 monthsDrug like Cialis can be useful too.Hope this may help youContact HCM for further health queryBest regardsDr. Sagar"
},
{
"id": 33422,
"tgt": "How to cure yeast infection?",
"src": "Patient: hi,im a 21 yr old female and i was diagnosed with a severe yeast infection 2 weeks ago after being on amoxicillin for 10 days,i was directed by my gyno to take 1 pill of diflucan every 3 days for a week,this only got rid of the infection for a while and then it came back after my period passed,then i decided to use an OTC treatment of Clotrimazole for 3 nights, After using clotrimazole for 3 nights,is it normal for it to ooze out gradually?i cant really tell if the infection is gone because most of the cream is coming out during the day,when will it be out of my system entirely and i can become sexually active again? Doctor: HiThank you for asking HCM. I have gone through your query. You can get a swab test to find out whether you still have yeast infection. It will be gone with the therapy you had. But can recur if you have dysbacteriosis. In such case clindamycin or metronidazole combined with clotrimazole as vaginal suppositories will be helpful. You can also use intimate wash regularly to normalize vaginal bacterial flora and PH which will prevent you from getting such infections. Hope this may help you. Let me know if anything not clear. Thanks."
},
{
"id": 170180,
"tgt": "Why is my baby not drinking buffalo milk?",
"src": "Patient: hello,my 7 month old baby was on lactogen 1 till six months ,after 6 months doctor said to give her toned buffalo milk. the problem is that when i give her buffalo milk even for one single day ,from the next day she does not drink milk at all,nither sweetened buffalo milk nor shakes and not even formula feed,right now i m using lactogen 2. Doctor: Hi...sometimes there wont be an reason why some babies do not like the taste of the milk. The best way to feed the baby will be to feed what the baby likes and palatability is very important. I suggest you start rice based feeds now along with Lactogen 2 as the baby is more than 6 months old. This will make the bay put on weight better.Regards - Dr. Sumanth"
},
{
"id": 27615,
"tgt": "What is the substitute for this medication?",
"src": "Patient: sir igot angioplasty on29-11-12 and taking clopidogrel bisulphate and istartedgetting itching and rashes onbody .i am allergic to sulpha drugs . is it due to this medicine or some other .what is the substitute for clopidogrel bisulphate? YYYY@YYYY Doctor: Sir as one year has already passed post angioplasty, a single antiplatlet aspirin 75mg would be enough and you don't need clopidogrel (as per standard America / European guidelines) . Still if you prefer clopidogrel this a clopidogrel carboxylate preparation is available, however I think the the reaction might be due to clopidogrel compound itself rather than its salt."
},
{
"id": 74158,
"tgt": "Suggest treatment for pneumonia",
"src": "Patient: hello doc.Im Gigi 51 yrs old previously i have ptb but the doctor told me that it was cured already 7 yrs ago every time i go to his clinic he told me i have a pneumonia after en x-ray result even though i told him that i spit w/ blood last monday i go to his clinic agin for a medical check up and he prescribed me cefuruxime kefstar i bought 9 pcs only and switch to another brand of cefuruxime Axetil after 4 days i spit mucus w/ blood as for now im suffering severe backpain what should i do thank you so much doc Doctor: Hello Gigi !Coughing up blood with sputum (henoptysis) is a red flag sign .. meaning to say it definitely needs Medical attention . Considering you have had TB in the past there are many possibilities for your present episodes of hemoptysis. It could me a bacterial infection or relapse of TB or post TB bronchiectasis (abnormal permanent dilation of your airways at few places which happens when they get damaged due to your past TB). Take a course of antibiotic as prescribed by your physician ,Consult with your physician and discuss about taking drugs like tranexemic acid which reduce the bleeding tendency , cough suppressants . while sleeping, Lie on the side which was previously affected by TB and be on complete bed rest . Once your hemoptysis gets controlled go for sputum examination to check for TB bacilli , chest x ray and if needed HRCT thorax (to pick up bronchiectasis which can sometimes be not evident on x ray). Since u mentioned severe back pain , vascular abnormalities concerning the descending aorta (the major blood vessel carrying blood from the heart)needs to be also considered Treatment is to be decided based on the cause for your hemoptysis.Do review with your physician and get the necessary investigations done."
},
{
"id": 56236,
"tgt": "How to treat cirrhosis of the liver?",
"src": "Patient: Hello My husband has cirrhosis of the liver from excessive alcohol consumption. What are the stages and life expectancies? He is in the hospital right now for sinus surgery, but staying longer due to white blood cells are low and slight pneumonia. They took 5 1/2 liters of fluid out of his abdomen. His legs and feet swell up all the time, he can t eat but broths and ensure. thank you Doctor: Hi,Thanks for posting your question in HCM.I am Dr.Siva and I am pleased to assist you.Your husband is having cirrhosis due to alcohol consumption. It is an irreversible condition.Initially it remains compensated without any symptoms. It may later become decompensated causing many complications. Some of the common complications are bleeding in the upper gastrointestinal tract due to engorged blood vessels ( varices), accumulation of fluid in the abdomen and legs, poor functioning of the kidneys and brain. Later the liver may even turn cancerous.The life expectancy depends of the severity of the disease. A scoring system called CTP score is done to assess the severity of the disease. Class C is the most severe form with poor outcome.I hope that answers your question.Regards,Dr.Siva"
},
{
"id": 72769,
"tgt": "What causes fever in case of respiratory infection?",
"src": "Patient: My daughter is taking antibiotics for a respratory infection, she has been on the medication for 3 days. She also suffers from Asthma. This AM she was coughing non stop even after inhalers so gave her a dose of a steroid called Celestene o,o5% orally, later she spiked a fever, am wondering if it could have been a reaction to that as yesterday she seemed like she might be improving...? Doctor: Thanks for your question on Healthcare Magic.I can understand your concern. No, fever spike is not due to steroid reaction. Since she is having lung infection, fever spike suggests worsening of underlying lung infection. So better to consult her doctor and get done chest x ray. She may need higher antibiotic and anti pyretic drug. Hope I have solved your query. I will be happy to help you further. Wishing good health to your daughter. Thanks."
},
{
"id": 29151,
"tgt": "What causes warm and tingling sensation in the underarms?",
"src": "Patient: Both of my armpits are warm with a slight tingling feeling. I had swollen lymph nodes and glands in neck with a sore throat. on OCT 1. Took Amox/K Clav 875 125 mg for 10 days. neck swelling went down and throat is clear per the PA. Now, why tingling and warm arm pits? Doctor: do u have diabetes .get blood sugar level checked.i think you have peripheral neuropathy.take neurokind lc 1 tablet daily for 1 month.and calicum 1 tablet daily for 1 month"
},
{
"id": 151047,
"tgt": "Two discs removed from lower back. Having pain. Suspecting degenerative disc disease. Cause?",
"src": "Patient: I had two disks removed from my lower back due t a car accident in 2009. Its been ten months since the surgery and I have been having progressively more pain. Now I m beig told I have a bone spur and another herniated disk below the two removed prior. Doctor said it looks like thespine is trying to fuse itself. There has been talk of degenerative disk disease ect... I had a perfectly healthy back before y accident.... wat does all this mean and why is it hapening? Doctor: Hello, Our body has 33 vertebrae each joined with other through the disc and every vertebrae taking equal load. when some of the discs are removed, the vertebrae above and below the removed vertebrae takes the load of those fused and they get wear and tear early. the new bone formation is the result of excessive motion hence wear and tear and may result in pain. your problem can be relieved in two ways 1. do as your doctor is advising. 2. read and discuss with your doctor about surgically fusing the degenerated vertebrae, removing bony spurs so that there will be no excessive motion, no wear and tear and hence no pain. take care."
},
{
"id": 6570,
"tgt": "How precautionary measures can a woman having low acids take before getting pregnant ?",
"src": "Patient: my future husband is about 56 yrs old and i was going 38 and we r planning to have a baby. and if i have a low acids, what should i do for preparing pregnancy? Doctor: Welcome to Healthcare Magic Good Day When you plan your pregnancy it is ideal to start taking Folic acid tablets 7 days before conception. Avoid drugs like isotretinoin, alcohol, benzodiazepines like anti-anxiety and anti-depressant drugs, warfarin. Are you on any medication Eat diet rich in fresh fruits and vegetables, green leafy vegetables, proteins, carbs to some extent. After getting your pregnancy test, make sure to go regularly for the Antenatal checkups."
},
{
"id": 47074,
"tgt": "Suggest treatment for water retention in body",
"src": "Patient: Recently I have been trying to lose a few pounds. It started off with a reasonable calorie deficit and moderate exercise and losing a healthy two pounds/week. I got to my goal weight and slowly slowed down on all the swimming, running, and aerobics. It turns out I was not consuming enough calories to be healthy and I seem to be retaining a ton of water due to edema. I think I had anerexia athletica (unknowingly) for about 3 months. But I now only do 30 mins of exercise and eat a bit more normally now. I still have massive fluid retention and don t know how to eliminate it properly. When I weigh myself in the morning I am 105lbs at 5 foot 4. But I think I also am holding some water as well. What should I do? Doctor: Thanks for asking this question. There are many cause for edema in body parts.In case of liver cirrhosis edema can occurs.Most common cause for cirrhosis is alcohol drinking.In nephrotic syndrom most common bilateral edema present.Intera-abdominal mass can also present in nephrotic syndrom.It diagnose by urine test , in urine protein is more than 2 mg present.Nephrotic syndrom treated by taking law salt.ACE inhibitor drugs are also use in this condition.Some venous disorder also edema present.In lyphatic disorder edema can happen.Take law salt diet.Diuretic can use in edema case.Take care.Thanks."
},
{
"id": 213743,
"tgt": "How can I continue my treatment for Cymbalta as I am unable to get prescription for the same ?",
"src": "Patient: I recently moved to New Mexico from California and have not been able to reach my physician in California to obtain a refill Rx for Cymbalta. Both my pharmacy and I have left him numerous messages. I have not been able to establish myself with a physician here yet due to snafu s related to insurance, but I do have an appointment scheduled for April. The problem is, I am going through major Cymbalta withdrawl and can t get a prescription! I have taken this medication for many years. My medical group does have an urgent care clinic , but I was already told by phone that they would NOT give me a Rx for Cymbalta as that must be ordered by whatever doctor I establish myself with, but again, that is not until April. What about continuity of care issues? What the heck can I do? Any ideas? Doctor: Hi, Welcome to Healthcare Magic, \"Cymbalta\" is a \"S\" category medicine in most countries of the world, i.e. it can only be sold by a real Pharmacist upon production of Really real Prescription and the ins and out have to be submitted to FDA on daily basis, hence until you have proper prescription no one will sell you, i am sorry, but this is as things stand currently my advise would be to go get a proper prescription. Regards."
},
{
"id": 81099,
"tgt": "Could pain in upper chest and upper back indicate bronchitis?",
"src": "Patient: I ve been sick for a couple of days now... A couple of days ago, I woke up with a temperature of 100.6 and been having a bad cough... Since last night early this morning, everytime I cough, laugh, sneeze, blow my nose, and sometimes talk, I have pain in my upper chest and upper back... Could it be bronchitis? Doctor: Thanks for your question on HCM.I can understand your situation and problem.By your history and description, it seems pleurisy more.Pleurisy is inflammation of pleura. And chest pain on coughing is characteristic feature of pleurisy.Bronchitis will give wheezing, shortness of breath more, along with cough.So I advice you to consult pulmonologist and get done1. Chest x ray2. PFT (pulmonary function test).Chest x ray is needed to rule out possible causes for pleurisy (pneumonia, tuberculosis etc).PFT is needed to rule out bronchitis.So better to first diagnose yourself and then start appropriate treatment."
},
{
"id": 155574,
"tgt": "Does my gastric biopsy report indicate cancer?",
"src": "Patient: Please answer the following question Histopathology report 1-Stomach antral(gastric lesion) biopsies An area showing features of reactive gastropathy noted ( see comment) Thase gastric biopsies show areas with prominent foveolar hyperplasia In view of the above statement do I have cancer Doctor: Thanks for your question on HCM. In my opinion you should not worry at all for gastric malignancy. Your report suggest hyperplasia. Which is not suggestive of cancer.Dysplasia is disorganized growth of cells and it is suggestive of malignancy. Hyperplasia is not indicator of cancer. So no need to worry much. You are not having cancer."
},
{
"id": 98875,
"tgt": "Usually during winters noticed swelling of body parts like elbow,hip & wrist area",
"src": "Patient: My father In law has this intermittent weird swelling and the area Becomes super sensitive and painful. It might be in his foot for a week and go away then reappear later in his elbow or wrist. Then days later his lip may swell. It comes then goes and affects different parts of the body. This seems to happen in the winter months much more so than in the summer months. He has been to many doctors and no one seems to have a clue what it is. Allergy , immune, nuero, circulatory...where do you start??? Doctor: In such conditions, Ayurveda approach to treat Vata Pitta vitiation by way of herbals have been often highly successful by raising general resistance in body agaist such allergens as well as curing the conditions.To my own such patients, I advise to start Turmeric and Neem leaves powder immediately and add Manjishtha herbal capsules for controlling the problem. It's harmless and safe.For further guidance you may seek consideration through direct question.Hope it helps your father in law too - it will."
},
{
"id": 198970,
"tgt": "How to overcome masturbation addiction?",
"src": "Patient: Hi doctOr ! I have been masturbating since i was 16 and now iam 19 so at the start i just did it for fun and it has become a addiction for me ... And now i feel like ive lost my energy and enthusiasm in doing something and also i feel like my eye sight has reduced .... Will too much masturbation bring problem ? Doctor: HelloToo much indulgence into masterbation in harmful to the health I would advise you to follow these guidelinesTry to avoid any kind of.stimulant like pornographic material, movie or visual stimulation Concentrate on your study Play some gamesTry to be with family or Friends, try to avoid to be alone.Develop good hobbies like reading or playing and enjoy itYou should take multivitamins and good food supplements to restore energy and vitalityHope this helps you Please feel free to ask for follow up questionwish you good health"
},
{
"id": 68775,
"tgt": "Normal to have painful lump on injection spot after sprained ankle?",
"src": "Patient: About 4 months ago I went to the foot doctor with a sprained ankle. He shot my foot twice with an \"herbal\" injection. This month I developed a lump where the in injection was. It feels like a bone mass and it's the size of a marble. It hurts really bad to the touch and also hurts when I move my foot. Does this sound normal? Should I get a second opinion? Doctor: Hi,This is not normal after the treatment you describe. The problem could be related to the injection or due to the underlying problem. There may be infection. I suggest you see your foot doctor for further assessment. Regards,Dr K A Pottinger"
},
{
"id": 27357,
"tgt": "What causes heart palpitations after heavy drinking?",
"src": "Patient: I am a 36 year old male. I get heart palps after a night of heavy drinking. I've had EKG's done, an event moniter, ultrasounds & a stress test done. All tests were fine. Nothing wrong with my heart. Should I really be worried about this? Is there any way to stop them? I drink Vodka & Diet Coke. Doctor: Hello. Thank you for your question. Welcome to HCM. I understand your concern.During occasional or binge drinking, depending on how \"trained\" is the organism, in the human body happen a lot of problema and disbalances. An important disbalance is that of body fluids and electrolytes, which tend to produce also skipped heart beats (extrasystoles), at times even true paroxysmal episodes of supraventricular arrhythmias, particularly atrial fibrillation. Tha mainstay here is that you quit drinking alcohol or, at least, restrict it to a maximum of 30 g ethanol a day (which means one beer or one shot of heavy alcohol, such as vodka, a day). In my opinion this symptom should subside, if you undertake this lifestyle changes. However, if symptoms persist, feel free to ask again or consult a local cardiologist.I hope I was helpful with my answer. Take care."
},
{
"id": 1590,
"tgt": "What precautions should i take to get pregnant?",
"src": "Patient: i had an ectopic preganancy and now i am taking treatment like my follicules havescan in 11 th day and what is the next step i am continuing in taking folxt tablet ,bigamet so that i should concieve . doctor also given me the fertiniod for an ovvulation.plz suggest me that what precaution should i take to get preganant. Doctor: Hi, I think you can have contact with your husband every 2 to 3 days. When follicles reach a size more than 17 to 18 mm, take injection for rupturing the follicles. Take progesterone for next 2 weeks. Do a urine pregnancy test at home after that. As soon as you get pregnant, go for a scan within 1 week to confirm the site of pregnancy. Recurrence of ectopic is 4 percent. Hope it helps. Regards Dr khushboo"
},
{
"id": 15781,
"tgt": "Itchy scalp, forehead, ears and knuckles. Hair loss. High blood pressure. Cause?",
"src": "Patient: Hi I have a Ichy scalp, forehead, ears and brows! The scalp is not lice! I have a acute rash on forehead and brows! Ears are just itchy with slight redness! Accompany with all of that my forearm(very hairy) itch along with it! My knuckles seam to have a slight rash as well! O and if I put water in my hair it itches more! I have not researched it too much but I kinda worried here because I'm kinda losing a lil bit of my hair on head and brow. All of this goes away but comes back mainly when my skin is dry and my blood pressure is high! Doctor: Hi, It seems that you are having seborrhiec dermatitis.Seborrhoeic dermatitis is a common, harmless, scaling rash affecting the face, scalp and other areas.Seborrhoeic dermatitis is common within the eyebrows, on the edges of the eyelids (blepharitis), inside and behind the ears and in the creases beside the nose. It can result in pale pink round or ring shaped patches on the hairline.Seborrhoeic dermatitis is believed to be an inflammatory reaction related to a proliferation of a normal skin inhabitant, a yeast called Malassezia (formerly known as Pityrosporum ovale)Seborrhoeic dermatitis in adults may be very persistent. However, it can generally be kept under control with regular use of antifungal agents and intermittent applications of topical steroids.Medicated shampoos containing ketoconazole, ciclopirox, selenium sulfide, zinc pyrithione, coal tar, and salicylic acid, used twice weekly for at least a month and if necessary, indefinitely.Steroid scalp applications reduce itching, and should be applied daily for a few days every so often.Hydrocortisone cream can also be used, applied up to twice daily for 1 or 2 weeks. Occasionally a more potent topical steroid may be prescribed.Topical calcineurin inhibitors such as pimecrolimus cream or tacrolimus ointment may also be useful.Avoid stress and worries.Avoid oily food.Take proper sleep.I hope you may be benefitted., Still, you consult dermatologist.Regards,Dr. Ilyas Patel MD"
},
{
"id": 52671,
"tgt": "What are the ill effects of consuming excessive alcohol?",
"src": "Patient: I AM 37 YEARS OLD AND HAVE BEEN TAKING 4 7.5 HYDROCHONES A DAY FOR THE LAST 14 MONTHS WHICH ARE PRESCRIBED. THEY GIVE ME ENERGY AND ALLEVIATE THE Pain. I ALSO CONSUME 6-8 BEERS A NIGHT WHICH I KNOW I SHOULD NOT DO BUT TRIED TOSTOP LAST WEEK FOR 4 DAY AND HAD NIGHT SWEATS EVERY NIGHT. AM I PLAYING WITH FIRE? I DONT WANT ANYTHING TO HAPPEN TO ME BECAUSE I HAVE A4 YEAR OLD SON AND I NEED THE MEDS FOR BACK PAIN AND DONT WANT TO GO THRU WITHDRAWELS. I ALSO HAD GASTRIC BYPASS 3 YEARS AGO AND LOST 130 POUNDS DO THESE MEDS AND ALCOHOL HAVE ANY WORSE EFFECTS ON MY BODY BECAUSE OF THE SURGERY THANKS FOR YOUR INFO Doctor: Hello, It would be better if you quit beer because already you had surgery . You can take pain killers but only when you have unbearable pain."
},
{
"id": 37586,
"tgt": "Suggest treatment for upper respiratory tract infection",
"src": "Patient: I was diagnosed with an upper respiratory infection on Thursday, Sept. 11. I ve taken Zithromax daily, received a Decadron shot at doctor s office, and take prescribed cough meds at night. My cough is not as productive as it was earlier in the week, AND my temperature has started to spike in the evenings. Should I return to my physician? Doctor: Hello,Welcome to HCM,As you describe,it seems like you are suffering from an upper respiratory tract infection. Little laziness is common with any infection in our body.It can be well treated with antibiotics and anti-histaminics for a few days. For my patients with similar symptoms,I usually give antibiotics like amoxicillin+clavulonate(if not allergic to), along with paracetamol and anti-histaminics like levocetirezine for 5-7 days.Also avoid dusty/cold atmosphere. Avoid cold drinks.Do warm saline gargles 3-4 times a day.Giving adequate rest to body and having a healthy diet with fruits and veges will speed up recovery.Thank you."
},
{
"id": 173550,
"tgt": "What causes small lump with pink color and tender to touch on head?",
"src": "Patient: Hi... my 11 year old daughter complained of her head hurting today near the crown of her head. I felt and looked at her scalp and noticed a small lump on her scalp. It is a little pink and tender to the touch. Any thoughts on what it could be? It was bothering her a good deal tonight just when moving her head up and down. Doctor: Hello. I just read through your question.This could the result of a small trauma, like a small bump to the head or it could be a small abscess. These sometimes happen when there is a small, sometimes unnoticeable, break in the skin and the area becomes infected. I recommend consulting with your doctor so the correct diagnosis can be made and the appropriate treatment plan implemented."
},
{
"id": 69813,
"tgt": "What could be the lump on the ankle after hitting it?",
"src": "Patient: hi Doctor, i am a 18 year old boy weight about 10 and a half stone. i was playing football a few weeks ago when i got kicked to the ankle. at the time i could walk properly on it because it was bruised with a small lump. it has been a couple of months the lump is still there. the lump is situated on my right foot on my ankle below the right side of the ankle socket about 1 inch below it. At the moment i can run and walk on it but it i press down on it, it is painful. should i go get an x ray and do you reckon it could be factured Doctor: Hi, welcome to healthcare magic forum. I understand your concern.In my opinion it looks like haematoma formation has taken place in the ankle region(lump) and there may be chances of hair line fracture. So x-ray will be better idea followed by removal of haematoma by a surgeon. No need to worry. It is curable. Thanks"
},
{
"id": 128377,
"tgt": "Suggest home remedy for wrist and joint discomfort",
"src": "Patient: For about a month off and on, I have been having mild discomfort in my wrist, mainly on the thumb side joint. Now this week, I have been having a shooting pain and some numbness up my arm to my elbow. Are there any exercise or home remedies I can try to alleviate the discomfort.fm Doctor: HiDo hot fomentation and apply anti inflammatory gel twice a day.Take Motrin twice after meals.wear a wrist band for support.avoid working the wrist for 3 days it will help"
},
{
"id": 73499,
"tgt": "Suggest remedy to relieve chronic cough with phlegm",
"src": "Patient: I have had a chronic cough with phlegm for over a year now. I barely get any sleep, and I sometimes cough so hard I throw up. I have been on nasonex now for 6 weeks and my cough has actually gotten worse. I have been tested for allergies and have none. My doctor says it's post nasal drip. How else can it be treated? Doctor: Hello dearWarm welcome to Healthcaremagic.comI have evaluated your query thoroughly .* Chronic cough management requires thorough diagnosis with necessary clinical signs evaluation , x-ray chest , lab tests , CT thorax .* Suggestions at present - Gargles with salted lukewarm water added peppermint oil 3 times a day- Regular walking in fresh air , deep breathing exercise , YOGA .- Avoid smoking if doing .- Avoid exposure to dust / pollen containing environment if required use triple layered face mask liberally .Hope this will help you for sure .Regards ."
},
{
"id": 196319,
"tgt": "What causes frequent urination?",
"src": "Patient: 28 years old, 165 lbs, no medical problems or conditions. My penis feels like it's leaking but nothing is coming out. I've noticed it when I inhale and exhale. I have no discharge. I have been married to the same woman for 5 years and neither of us have intercourse outside of our marriage. I'm in no pain or discomfort at all. I'm not urinating irregularly either. It feels like a little buzzing, almost like it feels after I've ejaculated and there's a little left in there. It's been constant for two days. Any help is greatly appreciated. Doctor: HiGREETINGS You mean dribbling of urine or semen after intercourse.? At times genital infections like prostatitis or epididymoorchitis can present like this.Hence i suggest you to get a physical examination with urologist .Hope my answer helps you. Regards"
},
{
"id": 222534,
"tgt": "What causes fever and brown vaginal discharge during pregnancy?",
"src": "Patient: Hi, I think I am 5 weeks. I have been running a low grade fever for 5 days and everytime I urinate, there is light brown discharge with like very small grey and brown flakes of tissue in it. I called my doc but he said it would be too early to see anything now and to just go to the ER if I started bleeding bright red and cramping. Is this normal? Doctor: HI, I can understand your concern. * In 5 weeks pregnancy it's possible to conform by urine pregnancy test/ USG abdomen. Some times bleeding is possible with fever i early pregnancy -this is due to hormonal deficiency caused by stress *To avoid probable pregnancy loss- Get you pregnancy confirmed/ excluded with help of investigations & a gynecologist's consultation. If pregnancy is +ve, you would need hormonal supplement (for the bleeding you have)along with fever medicines.. Take bed rest/ light nutritious diet. Visit ER if there is bleeding in spite of the care taken. Thanks."
},
{
"id": 147623,
"tgt": "What causes throbbing pain in lower back and spine intensified on breathing?",
"src": "Patient: Hi, I am a 36 year old female, mother of 3 children, and I've been a roofer most of my life, until about a year ago. So back pain has always been an ongoing problem, but nothing major. All of a sudden I am having throbbing pain in my lower back, along the spine, intensified when I breath. What should I do.? What could this be? Doctor: Hi,Thank you for posting your query.I have noted your symptoms. It most probably suggests a pinched nerve in the lower back.This can cause lower back pain, which can increase with deep breathing or coughing or bending down.The diagnosis can be confirmed by doing MRI lumbo-sacral spine and the pain can be relieved by taking pregabalin capsules.I hope my reply has helped you.I would be pleased to answer, if you have any follow up queries or if you require any further information.\u00a0\u00a0\u00a0\u00a0\u00a0Best wishes,Dr Sudhir Kumar MD (Internal Medicine), DM (Neurology)Senior Consultant NeurologistApollo Hospitals, Hyderabad,For DIRECT QUERY to me: http://bit.ly/Dr-Sudhir-kumar My blog: http://bestneurodoctor.blogspot.com/"
},
{
"id": 149776,
"tgt": "Diagnosed with severe viral meningitis, episodes of epilepcies. On induced coma. No rashes, staring just for 2-5 minutes before sleeping for hours. Severity?",
"src": "Patient: Hi my family member,26 year old female has been diagnosed with severe viral meningitis. Experiencing epilepcies and today being placed in induced coma after 8 days in icu in a south korean hospital. We are shattered and helpless being in australia and rest of family in south africa. What is the severity of this illness? She has not been responding well at all. She can hear and recognise her partner (before she was induced today) . No rash on her body. When doctors woke her (she would be awake 2-5 minutes and fall asleep again for hours) she would just stare infront of her. What will happen now? Doctor: Hi and welcome to Healthcare magic.Thank you for the query.This is serious condition and agrresive treatment is necessary. But most people recover completely and there are no consequances. There is no rash in viral meningitis and treatment is usually only supportive. If she has seizures and sleep disorders it could indicate encephalitis. In every case prognosis is hard to predict but as I said , most cases are benign.Wish you good health. Regards"
},
{
"id": 80114,
"tgt": "What is the cause of right sided chest pain?",
"src": "Patient: I have been having right sided chest pain for the past three months every time I eat or drink. I have had a chest x-ray which was normal and have seen my GP who has advised me to try an anti-inflammatory gel and also gave me a months supply of Lansoprazole. Neither of these have helped. Please can you advise what this may be? Doctor: Thanks for your question on Health Care Magic. I can understand your concern. Since your chest x ray is normal, no need to worry for Pulmonary causes for your chest pain. In my opinion, you are mostly having stress induced GERD (gastroesophageal reflux disease). You need to follow these lifestyle modifications along with lansoprazole for better symptomatic relief. 1. Avoid stress and tension. 2. Avoid hot and spicy food. Avoid junk food. Avoid large meals, instead take frequent small meals. 3. Go for walk after meals. 4. Keep 2-3 pillows under head in the bed to prevent reflux. 5. Loose weight if you are obese. Avoid smoking and alcohol if you are having these habits. Don't worry, you will be alright. Hope I have solved your query. Wish you good health. Thanks."
},
{
"id": 213726,
"tgt": "Do I have a psychological problem ? Which psychiatrist or behavioral therapist can help me in my case ?",
"src": "Patient: I am a 27 year-old female. I used to poop or pee in home-made diapers for both fun and curosity when I was a teen, but I quit it many years ago because I thought it was a messy as well as morally bad behavior. After that, I occasionally dreamed of this act suffering all the time I was asleep. My problem is that the rate of these dreams has noticeably increased within last 2 years. In addition, I have found that I have a great desire to watch videos on the internet in which people are peeing or pooping in their pants or diapers. This stimulates me to the extent that I want to piss myself and I struggle a lot to control my urine. Since I haven t had any sexual experience, I can t realize whether this is sexual or fun or an addiction. 1) Would you please tell me how to recognize the type of my behavior among the three afformentioned ones, i.e. sexual, fun, addiction? Am I an AB or a DL or none? 2) Is this a psychologic problem? How can I cure it anyway? 3) Does this have anything to do with my desire to act like boys when I was a child and a teenage and I still lack some of female skills such as behaving seducingly? Doctor: Welcome to Healthcare Magic Good Day You probably have a prediliction to an unnatural sexual fantasy. It could be related to the terms Watersports and Skat. Not that I am hinting on this but psychological influences are implicated in such situations. There are a few articles based on the Freud's theories of the infantile mind, which may interest you. Have a read when you like, just for interest. Nothing to be worried about unless these thoughts trouble you. Nothing to worry about your 'seduction' skills, as and when you enter a relationship it should manifest appropriately. Every person is different and not all need to come across as seductively female. You may speak to a Psychiatrist on this regard to help you with coping with the situation and also engaging you in tried and tested behavior therapies which will definitely improve the situation."
},
{
"id": 129115,
"tgt": "What causes stiffness in the shoulders after recurrent seizure attack?",
"src": "Patient: A graduate student here from Iran has had 2 seizures in the past 3 weeks. with the first one his right shoulder froze up and with the second seizure his left shoulder also froze up Both seizures happened while he was sleeping and his roomates took him to emergency but nothing was done. the college doctor saw him and told him to find a neurologist. How urgent do you think this is. He is thinking of flying home for treatment. Doctor: Hello,Thank you for using Healthcaremagic.I read your question and understood your concern.Your friend may have had shoulder dislocations in both shoulders from seizures so he needs to do first Xrays of the shoulders and then evaluation from orthopedic specialistDr. Selmani"
},
{
"id": 42746,
"tgt": "Why does the follicle do not grow to the same size every cycle?",
"src": "Patient: Hi, My name is Chaitra, 30 years old. Trying to conceive, my doctor asked me to do follicular study, Last month dominant follicle grew upto 20mm in size and ruptured on day15 but this month foliicle did not grow beyond 14mm, What could be the reason for this. My doctor told it normally happens like this in 1 or 2 cycles in a year and not to worry. Please advise if this is right. Thank you. Doctor: Hi,Thanks for writing to HCM .Ya it happens normally in some cycles. It is due to stress which you take . Let it be any tension it will impair your hormone levels and follicle won't grow. Normal development of follicle depends on many factors like your hormone levels, stress, obesity, high surrounding temperature etc. So don't worry and try to avoid things which are possible. If you are desperate for child I would suggest you to go for ovulation inducing drugs like clomephine . This will help multiple follicles to mature and rupture . Thus increasing chances of pregnancy .Hope I have been helpful .RegardsDr.Deepika Patil"
},
{
"id": 143159,
"tgt": "How to treat multiple disc bulges in the neck and back, Sciatica and numbness in arms and legs?",
"src": "Patient: hello dr,I have multiple disc bulges in my neck and lower back worste at c4 and l5 to si being severe.I have sciatica in both legs numbness both arms and hands.. I have weird looking rash on upper chest and stops abruptly above breast area .ribs are tender to touch with worste pain from armpit area and down to lower ribs.any idea about rash?had it for cpl years along with ribs and chest pain.seen few Drs and nobody has a clue. Doctor: Hello!Thank you for asking on HCM!Your symptoms could be related to skin sensitivity and the levels of sensibility related to the pinched nerves. The fact that the rash has ends at a sharp levels, indicates a relationship with the damaged vaso-motor mechanisms and reflexes of this region of the skin, probably related to the damaged nerves or spinal cord innervating this region of the skin. It can also be caused by skin hypersensitivity to the contact with the clothes in this level, because the damaged spinal cord. I recommend performing a cervico-thoracic spine MRI study to examine for possible spinal cord compression in this region. Hope you will find this answer helpful!Best wishes, Dr. Aida"
},
{
"id": 211569,
"tgt": "Having visual disturbances. Done with CT scan for tumour. Taking Lopressor . Getting hallucination. Side effect?",
"src": "Patient: My sister has been having visual disturbances x3 years and has been examined by many specialists. She recently had a head CT to R/O any tumors, etc.. with normal findings. I checked her meds and found out a s/e for Lopressor which causes hallucination, both auditory & visual. How you every had a patient on Lopressor that has experienced this type of problem? Doctor: Hi,While none of my patients have reported hallucinations with metoprolol (Lopressor), hallucinations with metoprolol are well known and have been described in literature. You can read the case series by Goldner (http://www.ncbi.nlm.nih.gov/pmc/articles/PMC3295654/pdf/1752-1947-6-65.pdf) published last year in a reputed journal for more information. If, on reading this, you feel that your sister's symptoms are also similar, please request her doctor to consider changing metoprolol to another agent and see if her visual disturbance improves. If it does, then it proves that metoprolol was the culprit.I hope this is of help to you. Best wishes."
},
{
"id": 190878,
"tgt": "What treatment should I take for my gums problem ?",
"src": "Patient: Ever since I was little, my gums bled all the time. When I brushed, rinsed, or got hit. The dentists always say that I need to brush better, but the the thing is I spend 30 mins brushing, flossing, and rinsing all the time. They have gotten better over the years. I m 14 and I m wondering if this is normal for such a young age. The redness used to hurt and had isiility, but not that much anymore. However, it still bleeds at random. Today I ha a Dentist appointment and they were saying that my gums are inflamed, extremely. They didn t say Gingiitis, but I m unsure. I have braces so it s kind of difficult to floss, but I try. Please tell me what I should do. Doctor: Hi, since your age is 14, you are at the stage of puberty. at this stage the even little infection will be aggravated. so i suggest go to dentist, take proper scaling sittings, if it is infected you may need antibiotics also. take proper care of brushing twice a day, learn from your dentist the method of brushing for proper care. as you have braces please dont use floss as it may entangle in your brackets and harm your wiring treatment(ortho) process. Thanks."
},
{
"id": 163082,
"tgt": "Suggest remedy to get rid of rash all over body on child",
"src": "Patient: For the past 8 days my 3 year old gets rash/ hives that appear and disappear all over body 8days ago the hospital gave me a steroid medication and it was for 5 days it has since finished still the rash has not gone away cleritan helps but the rash does not disapear Doctor: Hello and Welcome to \u2018Ask A Doctor\u2019 service.I have reviewed your query and here is my advice.The causes of rash all over the body are viral fever, allergic reactions or any other skin disorders. So to get rid of the rash I will suggest you apply calamine lotion all over the body and give syrup chlorpheniramine as per his body weight. Consult with the pediatrician. Hope I have answered your query. Let me know if I can assist you further.Regards, Dr. Shyam B. Kale"
},
{
"id": 184970,
"tgt": "How to cure bumps and pain on the upper gums ?",
"src": "Patient: My son is 7 years old and has had a bump on his upper gums (under his lip, right above his tooth) for the last 3-4 months. He has fairly healthy teeth and goes to the dentist annually. It does not cause him pain, however, I am concerned. What is it and how can I get rid of it? Doctor: Thanks for using HealthCare Magic.Read your query.The bulging of the gums can be due to few reasons for your son.If the front teeth of his not erupted , it can be due to this (pre eruption bulge).As the tooth starts erupting in the mouth , it will reduce.Second , it can be due to a abscess formation if any of the mentioned tooth is decayed or traumatised .If there is decay please get it checked with the local dentist.If it still persists , get it evaluated.Hope this was useful.Thanks and regards."
},
{
"id": 37980,
"tgt": "How long does an hernia repair infection take to heal?",
"src": "Patient: I had an umbilical hernia repaired a week ago and it is no infected but is leaking fluid. My dr said I should clean it with hdrogen peroxide and pack it with gauze. how long will this take to resolve. Would it have been netter of Dr did not reconstruct naval? Doctor: Hello, Thank you for your contact to health care magic. I understand your concern. If I am your doctor I suggest you that it should not be more than week with discharge. If it is more than a week than you consider as lake of surgical skills in doctor. Till that time keep cleaning and washing with Hydrogen Peroxide, and betadine.I will be happy to answer your further concernYou can contact me. Dr Arun Tank. Infectious disease specialist. Thank you."
},
{
"id": 105498,
"tgt": "Numbness and asthma on the area of bug bites, numbness spreading to legs that have turned splotchy, purple with visible veins. Worrisome?",
"src": "Patient: About two days ago i was bit by a bug of some sort three times on the back of my leg. After the bite i have had mild asthmah which has carried on till today. Last night i woke up with the back of my leg numb in generalized area of the bites. Later on in the night the numbing worked its way down my leg. Today i still have the numbing and asthmah but now my legs are splotchy red/purple and veiny. Should i be concerned what could this be caused by? Doctor: Hi, Thanks for using the HCM. Thanks for giving good history.The bite has caused hypersensitivity reaction so you are having asthma like symptoms. Please tell me are you asthma patient. For all the symptom better to visit your doctor for the management. You must be started with anti histaminic drug for hypersensitivity and also if required corticosteroid and depending on the condition of the bite wound you can be put on antibiotic and antiinflammatory drug. So better consult your physician. Hope I have answered your query. If you have any further questions I will be happy to help. Thanks get well soon...."
},
{
"id": 51776,
"tgt": "Is it a side effect that i got fever after taking niftran ?",
"src": "Patient: one of the urologist advised me to take niftran 100mg as i had urine infection. but after consuming that tablet, i got high fever . is it side effect? should i continue or stop eating tat capsule. Doctor: Hello, welcome to HCM. Niftran is an antibiotic used for urinary tract infection. You need not worry about fever as it is one the few documented side effect of niftran. Apart from fever it is known to cause nausea, vomiting and rashes. You should continue taking it and also paracetamol tablet twice a day till fever is present. Wish you good health."
},
{
"id": 18021,
"tgt": "Are fluctuating blood pressure levels a matter of concern?",
"src": "Patient: My blood pressure can be slightly high in early morning, but if I work in the yard or the garden I get really tired and have to take rest breaks frequently. When I come in the house I feel as if I am dragging. My lowest reading was 87 over 41. Even doing little things like washing dishing or sweeping the floor it will drop to around 115 over 65. Is this something I should be concerned about? Doctor: Hi, If it is creating the doubt for blood pressure disease then it has to be checked with physician if you go on measuring with your own so it will go on fluctuating every time you read it and it is happening due to over consciousness. If you do not have any illness then it is advisable not to measure it like that only, if the blood pressure fluctuating in normal range so it may not cause in symptoms and it can not be low on its own. Hope I have answered your query. Let me know if I can assist you further. Regards,\u00a0\u00a0\u00a0\u00a0\u00a0 Dr. Akhtar Husain"
},
{
"id": 138627,
"tgt": "What is the treatment for weakening of legs?",
"src": "Patient: my legs are very week walking I loose my balance this star about 1o years ago I go for terapy every year feels better for one or two weeks but then returnes I had a hearth transplant 8 years ago take about 13 pilulas at day little by little my mussuels are getthin weekear what shoul I do Doctor: HiWelcome to healthcaremagicI have gone through your query and understand your concern. I have gone through your query and understand your concern.You are advised to get your vitamin D test.If found low you you can take vitamin D3 supplements.EvionLC and calcium supplements may be useful. For pain relief you can take analgesic such as ibuprofen for pain relief.You can discuss with your doctor about it. Hope your query get answered. If you have any clarification then don't hesitate to write to us. I will be happy to help you.Wishing you a good health.Take care."
},
{
"id": 35994,
"tgt": "What causes fever with sore throat?",
"src": "Patient: I'm a healthy adult. I've had very low fever with fatigue and mild sore throat for the last three weeks. My temperature has been fluctuating between 99.8 to 100.2. For the last two days my temperature is closer to 97.5 and I've been feeling very tired and feverish, still with a mild sore throat. My blood test results will take another week or so to come back. Any advice on the potential cause and what I should do?Thanks Doctor: Thank you user for your queryProbably you are suffering from pharyngitis which is commonly viral in origin.On a safer side you should get the culture of throat swab for group b streptococci, so that you may not develop rheumatic heart disease.Till then you can start doing warm saline gargle, or by chlorhexidine mouth wash.For fever any analgesic tablet can be taken.You can also consult and ear nose throat specialist, if your symptoms won't subside.Do get back after your blood reports.God Bless."
},
{
"id": 174659,
"tgt": "What could cause red dot around the groin area?",
"src": "Patient: Hi, a 2 year old toddler I know has developed small red dots around the groin area, legs and bum, it gives off a fishy smellwhen he has urinated however he seems to be in no pain and doesn t itch it.. One DR suggested too much juice can anyone help with an answer please? Doctor: HI as rash are not associated with any fever/pain/ itching no need to worry.But, fishy smell of urine may indicate a Urine infection in small kids, so please get a simple urine test done and if s/o infection to treat with antimicrobials pending culture report. give plenty of fluids and food rich in fruits and veggies"
},
{
"id": 189443,
"tgt": "Have black colour discoloration all over inside mouth, gums turning white, tongue lost sense, gaps increasing between teeth. Reason?",
"src": "Patient: I have black colour discoloration all over the inside area of my mouth and my gums also appear to be turning white in colour and gaps increase between the teeth,imy tongue also has lost the capacity to sense the taste, whatever I eat tastes to be just bland. please let me know whether its a serious problem and can it be cured Doctor: Hello, Thanks for your query. Details provided by you are not sufficient,you did not mention whether you are taking prolonged medicine or you have any medical problem and since how many days you have the same? 1.Black discoloration inside the mouth may be because of prolonged use of any medication,hydrogen peroxide containing mouthwash etc........ 2.White discoloration of your gums may be following possibilities- a.if white discoloration is limited to edges of gums,this is because of food debris and deposits. b.if white discoloration is spread to whole gum and cover the cheek region,it could be- oral thrush/fungal infection/leukoplakia So i will suggest you please visit an oral surgeon as soon as possible and get the complete oral examination done and take a proper treatment after diagnosis.and maintain good oral hygiene also. Take Care. Dr.Gunjan Gupta"
},
{
"id": 58472,
"tgt": "Bloating, pain in upper right side, fatty liver. CT scan done. Suggestions?",
"src": "Patient: I had a CT scan a few months ago because of lots of pain and bloating in my upper right side, it showed a moderately enlarged diffusely infiltrated fatty liver . My liver tests are still normal however the pain is worse clear through to my back and I feel like my side is so full that it will bust open soon. My Doctor says there isn t much I can do and will probably end up on a transplant list in a year. Any suggestions? Doctor: Hi and welcome to HCM. Thanks for the query.This is absurd to say that fatty liver should be treated with transplantation. Only cirrhosis and terminal liver disease require transplatation and fatty liver is most benign liver condition which is primarly treated by dietary and lifestyle habits so you need to follow hepatic diet,avoid alcohol,regulate sugar and lipids in blood and lose weight. If there is no improvement you need some medicines. if there is sign of cirrhosis you need liver biopsy to find the exact cause. Wish you good health."
},
{
"id": 97943,
"tgt": "Child having severe constipation, has stools with blood. Taken piclin, prescribed Y-lax tablet. Advisable for long term?",
"src": "Patient: Dear Sir, My daughter (03 year old) is suffering from constipation problem.Without laxative stool is not possible & bloody stool happens.Earlier she was using piclin ( Sodium pico sulphate) 5 ml dose.After that with consultation with a homeopath doctor she is using Y-lax tablet (2 tab thrice). Kindly suggest about the long use of y-lax. Siddharth Doctor: 1. is her Thyroid profile normal, and is constipation and blood in stool due to Piles or else (was P/R examination carried out? 2. blood can be due to over straining or constipation can be due to fear of blood, so consult with your health care provider. 3. does her laxative contains any Aluminium? 4. any Physical inactivity? 5. . attempt for bowel movements 15-20 minutes after breakfast when gastro colic reflex is strongest. . give her diet rich in fiber: beans,baked potato, peas,brocooli, corn, apple, oranges, peaches, wheat,isaphgol,and cereals. . give her luke warm water to drink with 5 cashew nuts and 5 raisins boiled in it, and as constipation leads to urinary tract infection, make sure she is taking at least 8 glasses of water everyday. . Ayurveda Decocotion \"Abhyarishta\" is good remedy both for constipation and blood in stool."
},
{
"id": 147685,
"tgt": "Any suggestion for MRI disc bulge seen at mildly narrowing bilateral neural recess ?",
"src": "Patient: hello sir good evening i am suffering from back pain since years and months back i done MRI scan also the problem is mild that is disc bulge seen at - mildly narrowing bilateral neural recess. now i am taking antibiotic tab since months but no use, the pain is like that only and i left the job because of this problem. so what i want to do? which medicine i have to take? Doctor: Short answer: Stop antibioticsDetail answer: I read your complaints.Please stop antibiotics first.Start to do back exercises with the help of a physiotherapist.No bending, lifting heavy objects.Avoid sitting work and standing work for more than an hour.Take good sleep.. dont sleep late . Use a hard bed. and sleep in side position not on back.Try to learn Dhanur Asan for back do it four times a day.Take pain relievers when in severe pain.If there is tingling in the legs or loss of power, you must go to a neuro surgeon or neuro physician asap.VITAMIN CALLED METHYCOBOL WILL HELP . please meet your local doctor for prescription and dosageVITAMIN e will help a lot. also calcium and vitamin d will help a lot."
},
{
"id": 97966,
"tgt": "What treatment do you suggest for my hair loss as I've androgenic alopecia and no allopathic treatment has helped ?",
"src": "Patient: Sir, I have taken the allopathy treatment for hair fall which includes Finax tablet and hair 4 u for controlling hair for almost three months. And that hasn t helped and i am having androgenic alopecia . And he is saying tat now there is no treatment available in allopathy. So I have turned to the homeopathical treatment.Is it good way? And are there any treatments available for me in allopathy? And i also have a problem of seborrheic dermatitis . Now its gone so should i use the shampoo for the rest of my life for cleaning of the scalp. Doctor: 1.Family history of hair loss? 2. Multiple cross sectional studies have found androgenic alopecia homologous to PCOS in female. 3. biopsy is/may be needed to exclude other causes of hair loss,whereas histology will/may demonstrate perifollicular fibrosis. 4. is your testosterone and DHT level checked? 5. treatment modalities can be from hair transplantation to latest Low Level Laser Therapy (LLLT) 6. If your health care provider finds any hormonal imbalance then FDA approved Propecia finasteride is the drug/treatment of choice. 7. Take diet rich in Vit.C, sulphur,iron,zinc,Vitamin B complex and essential fatty acids,Avoid junk, processed foods, take well balanced green leafy vegetables,fruits,protein,salad. 8. Yogic Postures like Vajrasana and Pawanmuktaasan are helpful. 9. use a mild shampoo once in a week, but don't apply oil to your hairs or use tight caps/hats."
},
{
"id": 115288,
"tgt": "Suggest remedy for elevated GGT levels in blood",
"src": "Patient: hi i am 36, last year in september 2013, my ggt was below 45, i was drinking 2-3 bottles of 650ml of beer daily for 5 years now, but was doing sport, in october i quit sport due to a disk problem, but keep drinking and this year the ggt level rise up to 618, i am anxious and afraid of dying, could you please advise how to lower the ggt level the quickest possible, thanks Doctor: Hi!Welcome to HCM!Elevated levels of gamma-glutamyl transferase (GGT) alone or with elevated liver enzymes are highly indicative of liver disease with or without cirrhosis. Since you have been a heavy drinker for the last 5 years or so this is reflected in the form of suspected liver injury giving rise of elevated levels of GGT.Over time, your GGT level will fall from whatever level it was at when you stopped drinking alcohol to within the normal range. This can take several weeks to more than a month. Abstaining from alcohol will decrease your chances of further damaging your liver and should allow your liver function to improve.My dear take care of your precious health, avoid drinking alcohol as time again and again the scientists across the globe have proved that it virtually damages almost all the organs of the body in short period of time.Take care of your health.Best regards!"
},
{
"id": 110080,
"tgt": "Can back pain be a result of sciatica?",
"src": "Patient: My question is what type of doctor to see: I have low back pain and sciatica pain down the right leg, then at the same time, both my big toes are numb and pretty bad pain in the second toe on the right foot. I have had this sciatic pain down the left side too, like last week. This week it is in the right side. Now, who do I need to see, an orthopaedic surgeon or neurosurgeon? Doctor: Dear Sir/ Madam,Thank you for posting your query at healthcaremagic.comI suggest you to visit a superspecility hospital, where doctors work in team. You can go to either of one specialty. At first they would adivice to get investigation done like MRI, X-ray or CT Scan.Investigation will help them to understand the severity or worsening nerve damage, or a serious underlying problem.Hope this answer your query.Please feel free to revert back in case you need any further information.With best wishesTehzib"
},
{
"id": 109677,
"tgt": "Suggest treatment for sudden weight gain and lower back pain",
"src": "Patient: I'm 68 in good health but in the last 8 months I have gained 30 pounds even tho I excires and try to eat right I have been have lower back pain on my right side and my right hand ( my little finger) is sore maybe like arthrits even tho I have never had any problems . I just had blood work but it isn't back yet the weight gain is my biggest problem I think Doctor: Your problems could well be caused due to rapid gain in weight. The cause of such rapid weight gain needs to be evaluated. For weight gain to occur, the amount of energy taken in as food needs to be greater than the total amount of energy spent. So, needless to say that you need to cut down on the food intake. Take high fibre diet. Exercise regularly with regular morning walk. You may also opt for electrical muscle stimulation therapy that is available at many slimming centres. Get your thyroid profile checked (if not done already) as hypothyroidsm could also be a cause of rapid weight gain."
},
{
"id": 212050,
"tgt": "Continuous fever with fluctuation in temperature, chills, fatigue, loss of appetite, stomach upset, very low sex drive. History of surgery for deviated septum. All tests normal. Help",
"src": "Patient: My boyfriend is a 38 year old male who has had a fever between 99 and 104 for the past five weeks. All blood tests, xrays, ct's, urine tests, and cultures etc have come back normal. He gets very shaky and has chills and tremors when his fever starts. He is usually healthy, but has had a history of sinus problems and had surgery on nose and sinuses 2-3 years ago to fix a deviated septum. He has been getting weaker daily as his fever persists, huge lack of energy and tires quickly, has had a loss of appetite since the fever began and has lost 20 lbs or more from eating so little. He's been going to the doctor and hospital for weeks, but the tests show nothing. His mom has sarcoidosis (sp?), an autoimmune disease I believe. He has occasional cough (not bad and stops quickly but not usual for him) and has had an stomach upset lately (bloating, heartburn, constipation), and he no longer wakes up sneezing 50 times every morning, as he has EVERY morning since his sinus surgery. Not sure it's at all relevant, but he's had a very low sex drive for 15 years, which to me is extremely unusual. After his late teens, he has had little interest in sex and often goes months without interest. While single from 20-33 he went years without being sexually active at all. He is the total opposite of any man I know this way, just placing a very low importance on anything related to sex, although he can be aroused and perform normally when he does occasionally feel obligated to force himself to get \"in the mood\". Since the fever began, We have both done tons of reading online about possible diagnoses, but it seems to cause more questions and concerns, and no one can offer any answers and he can't his doctors to refer him to any specific type of specialist or anything, to figure out what could be wrong. Where can he turn next for help? His family doctor and hospital doctors have no idea, and keep telling him to come back every couple days if fever persists, but it has and he is desperate for help. Has missed a LOT of work from this and has not gone one day without a fever since it began. Doctor: Hi there ~After reading your history about your boyfriend, I believe that he has a chronic disease. He needs to be checked out for a sexually transmitted disease profile as well as run a panel for autoimmune diseases. A visit with a psychiatrist will be necessary to rule out bipolar disorder. I wonder if an infectious disease specialist can also help along with an allergist/immunologist. Please take all his previous investigations with you for these appointments. I hope this helps.Take care and have a lovely day!"
},
{
"id": 172010,
"tgt": "Should i meet doctor for red rash on fore arms, cheeks and legs?",
"src": "Patient: I have a 7 year old son with a red rash on his fore arms, checks, bottom of legs, and a little on neck. The rash is swollen and hot to touch. We gave him Benadryl this morning and no change. He has had his mmr vaccine. Should I take him to the clinic? Doctor: Hi.... Sometimes MMR vaccine can cause this type of house minimal measles like Rash after vaccination but you need not worry about this as such immediately. But a physician needs to be consulted because we need to ascertain whether this is a MEasly Rash or not. So I suggest you to see a physician though it is not an emergency.Regards - Dr. Sumanth"
},
{
"id": 117037,
"tgt": "What to do for thrombocetpenia?",
"src": "Patient: i need to speak with a hematologist. i have thrombocetpenia and want to know what i should consider taking for it. i have already been on a round of predisone and it did nothing to correct my platelet count. there seems to be lots of different drugs but they all seem to be dexamthasone which i understand is very expensive. how do i get it covered by insurance or hospital care. i have a medicare advantage plan Doctor: Hi, dear. I have gone through your question. I can understand your concern. You may have ITP. It is idiopathic thrombocytopenic purpura. In that immune mediated destruction of platelets occur. So your platelets is destroyed and thats why you have low platelet count. You should take immunosuppressive drugs like steroids to control that. Hope I have answered your question, if you have doubt then I will be happy to answer. Thanks for using health care magic. Wish you a very good health."
},
{
"id": 120044,
"tgt": "How to cure pain in ribs and back?",
"src": "Patient: .My daughter complained tonight she felt funny and her ribs hurt.she has been feeling strange since thursday..However,tonight her back is hurting too.she has been cold but no fever.she is 16...Im wondering if she is getting sick?she hasnt slowed down any except for tonight.coughing strange a couple of times... Doctor: Hello, Our daughter is feeling associated symptoms of viral infection. These will go away within few days. Meanwhile, you may give her symptomatic treatment for her fever and cold. Keep her well hydrated. If she is having dry cough then she may do warm saline gargles. An analgesic anti-pyretic preparation like acetaminophen will help her. Hope I have answered your question. Let me know if I can assist you further. Regards, Dr. Mukesh Tiwari, Orthopedic Surgeon"
},
{
"id": 211505,
"tgt": "Depression, hair loss, loss of appetite, twitching in eyes, headache. Taking sleeping pills",
"src": "Patient: I suffer with depression, i spend most of the time in the house doing my routine which ive been set in for months. The only time i go out is to go doctors etc. i have hair loss and it is brittle, i am not sleeping well, i have been put on sleeping tablets but they are not working i seem to twitch constantly or get aggrivated unintentionally. i have lost my appetite not much weight loss ive lost a few pounds at most in past few weeks, my left eye keeps twitching, and i suffer with headaches and migranes, they have been more constant for the past few months. I put ot down to stress at first, but i am now unsure as this is not like me at all Doctor: Hi, Forum,I can understand how you might be feeling after such suffering.Considering all your suffering mentioned here there are high chances that you might be having psychosis. (- Kind of illness happens due to certain Neurochemical changes in brain) This can be improved after proper-continuous-prolonged medications.If u not consulted to psychiatrist yet or in recent time then please consult to him as early as possible,he will definitely help to come out of this.Hope this helps .You can post any further queries if you have."
},
{
"id": 18979,
"tgt": "What could blood pressure of 98/55 and an angiogram of 50% blockage indicate?",
"src": "Patient: My husband s blood pressure usually is 136/80 give or take. Last night I noticed his hands were cold and arms clammy the back of his neck was very sweaty. took his blood pressure and it was 98/55 any thoughts. he recently had angiogram and 50% blockage but nothing recommended as yet. Doctor: Hello,Brief Answer: Your husband has symptoms and signs of low blood pressure that needs treatment.Explanation: My opinion is that low blood pressure accompanied by symptoms and signs like sweating and dizziness is important to deal with. First measures to increase blood pressure to normal readings is essential. Then investigations to determine the cause is important. Angiogram with a coronary artery disease may be a cause of low blood pressure if there are chest pain symptoms. Fifty percent of blockage is a general term that needs more details. I suggest my patients the following; good hydration, revise treatment and doses and modify accordingly.Conclusion: I suggest you check with your physician.Hope I have answered your query. Let me know if I can assist you further.Regards, Dr. Mahmmad Gamal"
},
{
"id": 189573,
"tgt": "Getting treated for TMJ. Wearing a splint to prevent teeth grinding. Need a solution",
"src": "Patient: Hello, I started out being treated for TMJ mainly just wearing a splint to keep from grinding teeth ., over a year. Now I find I have jaw deterioation on the right side caused by arthritis . I am constantly aware of this because the teeth collide on that side almost like my jaw has dropped causing speech problems & stress. I have seen 3 oral surgeons & other doctors but they seem to think I don t have a problem & won t treat me. The last Doc was an orthodontist & really showed what was wrong but could not help. What kind of doctor do I need? I m so disgusted w the drs. I have seen & please don t suggest I see another TMJ Dr. Thank you & please help. Doctor: Hello Robinson, Thanks for posting your query. As per the details provided,you were wearing splint to prevent grinding of teeth. This has lead into jaw deterioration due to imbalanced occlusion or biting. I would suggest you to use either side of the jaws equally/simultaneously,without applying pressure on a single side. If required,the splint has to be removed. You can also try jaw movement exercises. avoid too much of wide mouth opening. hope this helps."
},
{
"id": 141527,
"tgt": "What causes dizziness, lightheadedness and lack of interest?",
"src": "Patient: Hi doctor,I am manu from indiaMy problem is very wierd. Every morning i cum to offic i feel sleepy dizzy and lightheaded. Feels the same always. Dont feel so interested in anything, cant concentrate on anything. All my colleagues are so fresh active and charming but I feel so low.Please help me Doctor: Hello and Welcome to \u2018Ask A Doctor\u2019 service. I have reviewed your query and here is my advice. You are not having good quality sleep. The breakfast should also include some caffeine like tea or coffee. Try not to eat too much in breakfast. Include more fruits in diet. Use of multivitamins will be helpful. Hope I have answered your query. Let me know if I can assist you further."
},
{
"id": 98471,
"tgt": "Suggest an effective alternative for Symbicort",
"src": "Patient: I was taking symbicourt for my bad chest allergies ,but my insurance BC/BS stopped allowing it. I have since developed a lot of fluid on my lungs and I cough quite a lot. Think the fluid is caused by severe allergies that the symbicourt helped. I can t afford the $350.00 full cost of the drug so I just have to suffer. Is there a generac drug that will help these severe allergy symptoms? That the insurance will cover? Doctor: hi sir/madam,Thanks for your question on Healthcare Magic.Other alternatives are:-1. Advair (fluticasone / salmeterol):-Take it daily and consistently for the best results. It is not for on-demand treatment of wheezing or asthma attacks.Rinse your mouth with water after each inhalation to avoid fungal infections of the mouth.2. Breo Ellipta (fluticasone / vilanterol):-Breo Ellipta (fluticasone / vilanterol) contains an ingredient, which may cause yeast to grow in your mouth and throat.Not appropriate for those allergic to milk since it contains milk protein.Other types of combination inhaler for COPD may work better to reduce the frequency of COPD attacks.Consult your doctor for further treatment.hope this was helpful.have a healthy day."
},
{
"id": 39378,
"tgt": "Suggest remedy for constant headache, stomach pain and body ache",
"src": "Patient: My son is having a constant headache, stomach pain, body ache, especially in his arms, bottom of his feet, legs, and neck, since last 10days. He has no fever, has good motion, no urination problem, and can eat everything. His doctor says he may be having stomach flu. She ordered all blood work, which was all normal and suggested hydrating him well. Over the counter pain medication, such as children;s Advil has not worked, nor has hot packs. I wanted to know how long it will last as it is not getting better since last 10 days and if there is something else we should be looking at, or should we just wait patiently for the flu to ride out it's own course? Thank you, Anuradha kamath Doctor: Thanks for contacting HCM. Dear Anuradha Kamath as all your son's tests are normal you should ask your doctor to evaluate for Juvenile Rhematoid arthrits and if that is also ruled out then it is probably not a physical problem. Children some time react this way to some thing bothering them like a family squabble or most likely someone tormenting them for example a bully in school. It would be good idea to en-quire at school and meet his class teacher and classmates to get to the root of problem. Also shower him with love and affection and get him talking about his problems, he may trust you and tell you. Regards"
},
{
"id": 56140,
"tgt": "What causes tumour my liver?",
"src": "Patient: Hello I ve been having discomfort and pressure under my right rib cage. I went to a gastro physician. They detected a tumor in my liver. They said it looked benign. But I need to go get a MRI. I have been having constant tingling in my feet. Could the tingling symptom be of a benign tumor or something more serious. Not sure if I should go to the e.r. Doctor: Hello. I doubt if your discomfort and pressure undr your rib cage was caused by your benign tumor and still more the feeling of \"tingling inyour feet\", so I don't see a serious symptom of going to the E.R. I'd suggest to keep calm while waiting for the MRI to be done."
},
{
"id": 217026,
"tgt": "What to do for the pain caused by osteo arthritis?",
"src": "Patient: I have extreme Osteo Arthritis for last 5 yrs I have been on Methidone but was changed to Fentanyl patch 25mcg every 3 days plus hydromorphone all the above cause bad side effects and not helpin g with pain been on patch since Sept 9 what do you recommend I do? Doctor: You have not mentioned your age which is very important in case of arthritis which is quite common in elderly people above 60 years of age. As per your history I will advice you to take some pain killer like ultracet or biozobid-plus only when pain is worse enough or disturbing your routine work. Apart from this you can take some calcium supplements like shalcal or toscal-gem once daily preferably in afternoon along with some multivitamins like metilda-forte or meganeuron-plus. You can do some physiotherapy exercises after an expert opinion of physiotherapist."
},
{
"id": 68156,
"tgt": "What causes large lump in the middle of the neck with headaches and tiredness?",
"src": "Patient: My daughter has a large lump in the middle of her neck right over her larinex. Her primary care doctor took her blood and told her that her SED rate is elevated. her ear is bothering her as well as getting headaches and she is always very tired. any ideas? Doctor: Hi, dear. I have gone through your question. I can understand your concern. You have lump in neck with high sediment rate. So most probably you have tuberculosis. You should go for fine needle aspiration cytology to diagnose tuberculosis. Then take treatment accordingly. Hope I have answered your question, if you have doubt then I will be happy to answer. Thanks for using health care magic. Wish you a very good health."
},
{
"id": 191525,
"tgt": "How can low blood sugar in a diabetic be managed immediately?",
"src": "Patient: Yes, please help me. My friend just called me from Elko, Nevada. She is driving westbound on interstate 80 with her boyfriend Mike who is diabetic. He is slurring his words and confused and irate. I think his sugar must be dangerously low. They don t have a glucose tester with them. I am trying to find a hospital near where she is driving to call ahead of time. What can I tell her to do for now? Doctor: ask them to take some sweat dish or better they can take some table sugar dissolved in simple water or milk"
},
{
"id": 57785,
"tgt": "Could incidental gall bladder polyp be the reason of having dull pain on right side of abdomen?",
"src": "Patient: Hellow Dr. I have been a drinker for some time, and now after I drink I get a dull pain in the right side of my abdomen under the ribs. I have had an ultrasound and my liver looked ok. However, it said that I had an incidental gallbladder polyp. Could this be the cause of the pain? Doctor: YES, DULL PAIN ON RT SIDE AS YOU ARE EXPERIENCING CAN BE DUE TO SO CALLED \"INCIDENTAL GB POLYP\". AS SUCH GB POLYPS LESS THAN THERR IN NUMBERS ARE A RISK FACTOR TOWARDS GALLBLADDER CARCINOMA. HENCE YOU ARE ADVISED GALLBLADDER SURGERY- CHOLECYSTECTOMY AT AN EARLY DATE."
},
{
"id": 193672,
"tgt": "What causes enlargement of liver and prostrate?",
"src": "Patient: i am 75 years old,wt reduced from 69 to 67kg ....having low temp. 99-100F in the evening since 3rd week of aug. 2010 due to overtiredness.Also suffering from abdomen problems and my esr level is recorded on 5.6.10--23 and on 4.10.10--42,,,,,15.9.10 ---14,,,,,,18.10.10-----28,,,,,29.10.10----15,,,,,11.11.10-----73,,,,,,,31.1.11-------73,,,,,23.4.11------107..........Since 22.4.11 my temp. in the afternoon and in the evening is b/w 100 and 101F...My associates other diseases are....hypertension,sugar,and large prostet ,mild increase in liver ,abdomen problem as stated above....plz advice me.... Doctor: Hi, You may be suffering from infection. Doing a urine complete can help find urine infection. If fever is more than 15 days can check blood culture. Share more details regarding your symptoms. Hope I have answered your query. Let me know if I can assist you further. Take care Regards, Dr S.R.Raveendran, Sexologist"
},
{
"id": 165202,
"tgt": "What could cause frequent thirst in a 9 years sold child?",
"src": "Patient: 9 year old female, original concern @ 4-5 years old: Always very thirsty & Behavior = to low blood sugar, blood sugar levels continue to test perfect. Acute Pain which bounces around her body, originally started around the rib cage (both sides) and upper abdomon. ANA + (Aug 4th 2011) shogrens is high, Augest 9-12 72hr. EEG showed slowing brain waves and she was diagnosed with Complex Partials - Two Neurologist state her seizures look like complex tics. Though her seizures have displayed as absance seizures as well... seizure episodes followed by beligerance. Estrogen Level is at 24. She suffers from stomach pain and nausia, pain in mussle and joint areas, confusion, missed information, exhaustion, heat and cold intollerance, loose stool and pain with stool elimination (no blood in stool). 85% in height growth - healthy weight. Adrenal system is perfect as well, brain MRI (1.5 imaging) show no tumors in her brain - No MRI done on spine yet. External sonogram showed perfect growth size for uterine development and bone growth x-ray was perfect. Acute pain also affects her head - headache type symptoms. Oh... periodic phantom smells - visual disterbances causing things to look as if they have a green haze. Periodic night sweats which will occure in succession 2 to 3 nights in a row. Vaginal discharge noticed in her underware seems to appear when she has excessive pain issues -no menstral blood as of yet. She tired of being tired and feeling miserable and she wants to go back to school - currently hospital homebound. Seizures happen predominately in the morning and evening. Seizues have also looked as if she had mental retardation. Seizues last 3 to 20 seconds and come in clusters. She is taking Keppra 1 & 1/2 500 MG tablet in the Morning and 1 500MG tablet in the evening. No success with Ozcarbazapine. No Success with Actaminaphine for pain. Keppra seemed to make the pain go away - but it is coming back again, pain may just not have been in a flareup state. Doctor: I think you must do thyroid function test and hepatic enzyme with serum ammonia may help you for diagnosis and treatment"
},
{
"id": 185698,
"tgt": "How to treat impacted wisdom tooth?",
"src": "Patient: Hi in 22 years old and i have all 4 of my wisdom teeth impacted. i was told i will have to use a general anesthetic... my question is i am anemic and i was worried that the surgery would cause me to bleed out. should i have the surgery done in the chair or at the hospital? Doctor: HiThanks for consulting health care magic. Extraction of wisdom teeth is not mandatory for all the cases.Since your dentist has told you to go for extraction just for anaemic reasons we dont go for extraction under GA.Usually extractions under GA are done for the persons who are allergic to local anaesthetics, handicaped, mentally retarded, in some pediatric patients and some extreme unexceptional situations.I beleive you dont have any of these and you are young too.For any surgical procedure on must have minimum Hb% is 9 you can acheive this by some heamatic drugs or syrups along with good nutritious food you can see the difference in a week in your blood.Hence i suggest you to find out the correct reasons for performing under GA. If you are willing to go extraction of all the teeth i mean 4 wisdom teeth at a time,or you have any discomfort with those teeth then yes it has to be done under GA.otherwise if they are interfering with the other teeth then go for extraction of two on the same side at o nce and remaining two on the other side at once that is better hardly it may it make 1 month to finish it. Kindly know the reasons and proceed further."
},
{
"id": 131730,
"tgt": "What could cause the lower tissues around the toe to swell?",
"src": "Patient: In February I suffered a 5th metatarsal base fracture (tuberosity type). The cast is off & I walk & weight bare. However the main problem is the lower part of the foot - Tissue around the toes swells if I try to bend the foot; there is still not proper movement in the foot (I does not bend fully). Will this lack of movement be permanent & what is wrong with the ligaments near the toes? Doctor: swelling is due to long time of disuse naturally u swelling will go away with daily activitys try running and jogging ligaments will losen up as well"
},
{
"id": 9478,
"tgt": "How to treat dry skin on buttocks and in between thighs?",
"src": "Patient: i am 20yrs oldheight 5'5ftweight 55kgsi m suffering from dry skin on my bums and in between thighsi have tried many ways to get rid of iti think it maybe because of my excess sweat and less proportion of water intakewhat is the solution to my disease..??please reply soon Doctor: Hi,Thanks for writing to us.A well defined dry patch of skin over gluteal area or in between thighs could be due to fungal infection ( tinea cruris). The same can only be confirmed by a clinical examination. If its tinea infection, it will respond to antifungal creams like clotrimazole or terbinafine cream. I would advise that you meet a dermatologist nearby.Take care"
},
{
"id": 24332,
"tgt": "Can blue heart tablet be stopped after having a normal heart ultrasound?",
"src": "Patient: my father was told he needed to take a blue heart tablet 3 years ago after having a ultrasound without seeing a specialist he has been very sick lately with fluid on his legs and feet and was told it may be his heart failing so they gave him another ultrasound ,he was told his heart was great nothing wrong with it very strong so does this mean he shouldnt now be on this heart tablet or that he shouldnt have been given it in the first place Doctor: hello ... some heart failure are temporary and some c are permanent ... so it depends on which type heart failure is there... if have all reports can consult with reported... but let specialist decide.. do not stop cardiac drugs with out consultation to cardiologist.. take care..."
},
{
"id": 188940,
"tgt": "Had wisdom teeth removed. Jaw is swollen and painful, uncomfortable to sleep , eat or even move. Advise?",
"src": "Patient: HelloI had my 4 wisdom teeth removed 6 days ago. And my jaw seems to be swelling more and more all the time. It's Very uncomfortable to sleep , eat or even to move around. The pain seems to be getting worse as time passes. Is this an infection or just normal healing and swelling ? Jaw and cheek are swollen as big as a tennis ball Doctor: Hello Welcome to HCM I would like to tell you that wisdom teeth extraction is not a simple extraction.Its somewhat complicated surgical procedure & there are many post extraction effects that are seen like:- -prolonged numbness due to local anaesthesia effect -pain -swelling -difficulty in opening mouth -fever -malaise You don't need to worry for any of these symptoms. Take some rest & start with amoxiclav-625 twice a day along with zerodol-SP twice a day.Also take some multivitamin & antacid like rabeprazole 20 mg if acidity persists.Take all this for 3 days. Along with that start warm saline rinses 24 hrs after extraction.This will help in healing. Consult your dentist if problem persist. Take Care Regards Dr.Neha"
},
{
"id": 221841,
"tgt": "Is it safe to do IVF treatment in 47 years?",
"src": "Patient: iam 47 year old women. still child less.itried ivf 6times.Two times conceived but aborted.Lastyear with donar embryo i conceived .But aborted after 25 days.again i want to take donor embryo ivf.Is it advisable at this age and suggest good hospital at chennai. Doctor: Hallow Dear,For successful conception, three factors are essential - A healthy matured egg, healthy motile sperm and a good lining of the womb in which the fertilized egg can get implanted and grow. You had successful fertilization. However, if there are chromosomal defects in the baby which are not compatible to continue the pregnancy, it will abort. However, repeating the same faults suggests that the egg donor and/or sperms could be having such a fault that even after the successful fertilization, the pregnancy cannot continue. You better take opinion of Foetal medicine specialist. Was the thickness of uterine lining (Endometrium) monitored at the time of embryo transfer? At least 10 mm thick endometrium is required for healthy implantation. Moreover, after the implantation is successful, enough hormonal back up, particularly Progesterone is needed. The levels of Progesterone can be measured in the blood and monitored accordingly. If there is Progesterone deficiency, it can be reinforced by external Progesterone. At this age, it is very likely that your endometrium may not be conducive to continue the pregnancy. At this age, I would suggest you adoption of a needy baby. I hope this gives you some direction before you attempt your next IVF. Dr. Nishikant Shrotri"
},
{
"id": 18062,
"tgt": "What causes heart fluttering and headache while driving?",
"src": "Patient: Hello I m a 57 yr old female, and I experienced fluttering in my chest with the feeling was getting a migraine while driving into the city this morning. The migraine did not occur, but a headache followed but not to the degree of a migraine. See someone?? Doctor: Hello! I would explain that your symptoms could be related to possible cardiac arrhythmia. For this reason, I would recommend consulting with your attending physician for a physical exam and some tests: - a resting ECG- complete blood count for anemia- blood electrolyte for possible imbalances- thyroid hormone levels for thyroid gland dysfunction. An ambulatory 24-48 hours ECG monitoring may be needed to investigate for possible cardiac arrhythmia. Hope I have answered your query. Let me know if I can assist you further."
},
{
"id": 139743,
"tgt": "What is the treatment and precautions to be followed for the fits?",
"src": "Patient: hi i am balaji, 25 years old and 172 cms height and 54kg weight. i fell down first time in december 2010 due to fits, and then i was prescribed with medicine eptoin 100 mg for 3 months, one per day at morning. after using 2 months again fell down even i taken it. but i drunk day before that day. then i prescribed with epsilon 300mg. i used 2 months, then again i used 100mg for 3 months and i stopped. 9 months later recently i fell down and doctor now given me zeptol 300 for testing of 10 days. can u pl tell me is it match to me? if yes pl explain the things what i should not. Doctor: Hello, The most important thing is never ever skip the drugs. Avoid high-risk activities like driving, swimming or handling heavy objects. An MRI scan may be taken to rule out any possible focal lesion in brain. Consult a neurologist and he will direct you accordingly. Hope I have answered your query. Let me know if I can assist you further. Take care Regards, Dr Shinas Hussain, General & Family Physician"
},
{
"id": 168080,
"tgt": "Suggest nutritional diet for child",
"src": "Patient: My son is 4yrs old his haemoglobin is 11.3, racked cell volume is 35.5, MCV is 73.3, MCH is 23.4, MCHC is 31.9, Total WBC count is 10900, neutrophils is 39.1 and Lymphocytes is 48.6 Please suggest correct food to give him, he has cold and cough on and off. He is also very aggresive by nature. Doctor: Hi, Thanks for writing to Healthcare magic You may please go through the below mentioned article of mine, which in short gives you nutritional needs of a child. Hope you find it useful.In case any more query, I am happy to answer.Else rate this answer and close the discussion.Below is the link. \"Are You Giving Your Child the Nutrition He Needs?\" by Dr. Poonam.M. Sambhaji - https://www.yyyyyy.com/yyyyy/are-yyy-yyyy-your-child-the-yyyyyyy-he-yyyyy-20610/yyy"
},
{
"id": 31880,
"tgt": "Can oral intercourse cause HSV-1?",
"src": "Patient: I have a cold sore. Last night without even thinking about it i gave my boyfriend head. Now my coldsore is practically gone. It doesnt itch burn or anything anymore and has kind of scabbed over. Its on the side of my mouth and not really touching my lips. Im very upset and i think i might give him herpes. I have hsv1 not hsv2. He doesnt have either. I dont understand how hsv1 could turn into hsv2 though. Would he simply just contract hsv1? Thankyou for your time. This means everything to me. Doctor: HiThank you for asking HCMI have gone through your query. HSV 1 can spread to genital area by giving oral sex. But it is not as severe as HSV2. So nothing much to worry. By taking antiviral drugs like valacyclovir the percentage chances of spreading this to your partner will decrease. As you have given oral sex with lesions on chances of spreading to him is more. HSV 1 and 2 are different and there is no transformation from one to other.Hope this may help you. Let me know if anything not clear.Thanks."
},
{
"id": 215285,
"tgt": "Suggest remedy for bruised foot",
"src": "Patient: i stuck my foot in a shoe and it felt like I was walking on a stone. SO, I took the shoe off. It hurt on the ball of my foot near my outer side. I could feel a hard knot on the ball. I sat don and my third toe hurt extremely bad when I touched it. The next morning, the bottom and top of my foot was bruised. The bruising is spreading across the top of my foot. There is still a hard knot on the ball. Doctor: Hello, A bruise is a kind of soft tissue injury and will settle by itself. No definite treatment is required. You can apply icepacks for faster recovery. If symptoms persist better to consult an orthopedic and plan for an X-ray to rule out bone involvement. Hope I have answered your query. Let me know if I can assist you further. Take care Regards, Dr Shinas Hussain, General & Family Physician"
},
{
"id": 11595,
"tgt": "Which cream should I use to treat facial pigmentations?",
"src": "Patient: for facial pigmentation i am using skinlite but skin color darkens after i stop using it.any other safer alternative without steroidal ingredients.is it safe to use nrutrogena 50 spf while going out as medicated sun screens leave white grease like on my skin. Doctor: Hi,I can understand your concern for skin color. This can be due to post inflammatory pigmentation after the sun damage and sun tanning of the area.Please do not use skinlite as it contains steroid which can cause side effects of steroid abuse.If I were your dermatologist then for dark pigmentation I would have suggested you to use azelaic acid cream or Kojic acid cream or glycolic acid cream or licorice extract or arbutine creams at night time in guidance of dermatologist.The most important part is to use a sunscreen regularly in open environment and on exposure to sun.Take a proper guidance and follow up with a good dermatologist so that all the problem can be dealt best for future time.Protect yourself form direct sunlight.Take care"
},
{
"id": 44679,
"tgt": "Why am I unable to conceive after a series of vaginal infections?",
"src": "Patient: i am 39yrs old, 1.52, 85 . had 3kids bw the ages 22 to 29 without problem; at the age of 33 had a 4mths pregnancy that never developed. for the past two yrs i have been having permenant infections (clamdia etc) febuary this year i had a surgery for an infection at the mouth of the uterus; Since 33, i have not been able to become pregnant , want is the reason Doctor: hi welcome to hcm.see because clamidiya infection you having difficulty in pregnancy.as you said you gone through surgery you ask your doc from how much time i take precution after his advice you plan for child.dont lose hope.thanksdr.dharadhara.shah84@yahoo.in"
},
{
"id": 94071,
"tgt": "Have bloated belly and pain. CT scan shows colon wall is thickened. Suggestions?",
"src": "Patient: I recently went to my doctor due to some cramping in my lower abdomen, mostly on the left side & because I noticed I was passing large white chunks along with my stool (about grape-size). My doctor put me on 2 antibiotics to treat possible diverticulitis or colitis & set up a CT scan . The CT scan showed no Diverticulitis, but did show that my colon wall is thickened, so I m being sent for a colonoscopy next week. In the meantime I m trying to figure out what s going on with my body.....I don t know if it was the antibiotics, which did upset my stomach, but the past few days my belly is so bloated & distended I look as if I m 5 months pregnant, the lower left pain is constantly aching followed by short stabbing pains, my stomach always feels full so it s very hard to eat much. My doctor said I may have Crohns or UC, will either of these diseases show up on the colonoscopy? And is there anything I can do before then to reduce the bloating/fullness? Doctor: Hi welcome to Health care magic forum. Thanks for calling H.C.M.F. As you describe you are going to have colonoscopy, and possible diagnosis could be chrons disease, or ulcerative colitis. You want some remission in the mean time . I advise you to avoide spices, junk foods, and oily foods to prevent irritation of the gut, and hence motility. Take bland foods as far as possible, in small amounts at a time and more times. Take more of water. You can take antacid liquids for soothing the gut. Wishing for a quick recovery . Best regards."
},
{
"id": 79383,
"tgt": "Suggest treatment for severe weight gain and shortness of breath",
"src": "Patient: My daughter is 34. She has a pretty good diet, vegetarian, but has been gaining weight over the past year and now weighs 200 and is 4 11 . She has been having trouble sleeping the past several months. She wakes up feeling like she is shutting down and can t breathe, and is afraid to go back to sleep. Is there more information you need in order to give me an idea what type of dr. she should go to for testing? Thanks. Doctor: Thanks for your question on Health Care Magic. I can understand your daughter's situation and problem. By your history and description, she is mostly having obesity hypoventilation syndrome (OHS). It is seen in patients with morbid obesity. Due to morbid obesity, when this patient lie down, excess fat tissue in neck compresses trachea and cause sudden stoppage of air and hence patient arouse and gasp fir air. So you should definitely consult sleep specialist and get done sleep study for her. Sleep study is must for the diagnosis of OHS. For her weight gain, better to get done thyroid function test, sugar levels and lipid profile as alteration in these can cause weight gain. So consult sleep specialist and discuss all these. Hope I have solved your query. Wishing good health to your daughter. Thanks."
},
{
"id": 115588,
"tgt": "What causes low WBC count with red dots on the arms?",
"src": "Patient: Over the last month my white blood cell count has been tested twice and they have been 129 and 123 respectively. I've been told that is low. I also have small red dots on my forearms. I read it could be a vitamin D deficiency so I have started taking vitamins again. I read it could also be a blood disorder. What tests could be run to verify what it is? Doctor: Hi,Thanks for asking.Based on your query, my opinion is as follows.1. If WBC count is low along with red dot on arms, which suggests of possible thrombocytopenia, possible vit B12 deficiency needs to be thought of.2. It could be possible pancytopenia. Get a complete hemogram for confirmation. Differential count with peripheral will evaluate any blood disorder.3. Vit B12 assay can help in confirmation of deficiency. Also associated macrocytic anemia, will be seen. 4. Other tests that would be needed are bleeding time, prothrombin time and partial thromboplastin time and bone marrow biopsy may also be necessary. Based on test results, further opinion possible.Hope it helps.Any further queries, happy to help again."
},
{
"id": 130826,
"tgt": "What causes sharp pain in the ankles and the deep pain in the knee is interfering with walking?",
"src": "Patient: Hello, I have had a recent CT scan with the radiologist statement as idiopathic Fahr s or Hypoparathyroidism. I have an appointment with a neurologist on the 1st. I have been diagnosed with ADHD and Anxiety disorder. I have had a neuro psych baseline prior to the thought of Fahr s since I thought I had early onset Alzhiemer s because of my poor memory, the psychologist said NO to that. I was found to be in menopause, 49 years old and had 0 estrogen. After that was taken care of I felt much better. However, I still feel bad and have a myriad of symptoms but would like to know of other symptoms besides the common ones that I ve read about. I quite frustrated! I was also diagnosed a year ago with a rare genetic skin disease which has not come up since and my dermatologist thought the biopsy was wrong and I might have a autoimmune disease like Lupus. SO, my question is a statement as well, I would like to know if their are more detailed daily symptoms. For example, is there a sharp nail like pain being hammered into you ankles. Or, how does the walking interference feel, is it because of a deep knee pain? Much more facts about the main symptoms and how things might feel. I appreciate any assistance. Doctor: In my opinion First if pain in ankle is in a specific spot in the back of your ankle + pain in the nack of your knee then its just mechanical pain that can be fixed by wearing a foot arch and slightly leaning forwards while walking , second if pain is in a wide area then it could be from lumber disc lesion )From the way you wrote your question your mind and thinking are clear , you should take female hormones and check your thyroid levels , magnesium hormone imbalance can cause your foggy memory Good Luck"
},
{
"id": 139359,
"tgt": "How safe is taking Dexa-tresortil for arthritis in knee?",
"src": "Patient: Hello my name is joseph . I have arthritist in my knees and the specialist prescribedcto me Celebrax , and gave me two injections , one in every knee ( monovisc ) but it did not work and still have pain . Is it good to stop it nd take the dexa - tresortil . I live in Canada Doctor: Hi, the treatment of arthritis by monovisc has long being abandoned and the american association of Orthopadics do not recommend it anymore. the drug you plan to take may be either steroid or analgesic, and both do not modify the disease process, I would need an X ray of your knee to comment what stage of arthritis you have and medications can only be effective in early stages. surgery and physio remain the mainstay or treatment"
},
{
"id": 141298,
"tgt": "What causes delay in regaining memory despite taking Levothyroxine post surgery?",
"src": "Patient: My daughter is taking 100mcg Levothyroxine every day and her TSH is 0.12 but her memory power is still not back to her pre-surgery days. Her doctor had suggested that he wants to lower her dosage of Levothyroxine. Is there any problem with her medication as she wants to regain her fully memory? Doctor: Hello and welcome to \u2018Ask A Doctor\u2019 service. I have reviewed your query and here is my advice. You should get her vitamin B 12 levels checked too.It's deficiency may lead to memory impairment. Hope I have answered your query. Let me know if I can assist you further."
},
{
"id": 193793,
"tgt": "What causes pain in testicles and blood in urine?",
"src": "Patient: My husband has noticed dark blood spots in his underwear for the past week a few months ago he was having pain in his testicles. Now there is no pain but there is blood and what looks like a bunch of veins very close to the skin on his testicles. Please help. Doctor: Hello, Your husband need to rule out epididymorchitis like infective condition. I suggest you investigate with semen culture to check for an infection inside seminal vesicle or prostate. You should also get investigated with ultrasonography of testis to rule out testis inflammation. Your husband needs to be given an antibiotic course and anti-inflammatory drug for that. Hope I have answered your query. Let me know if I can assist you further. Take care Regards, Dr Parth Goswami, General & Family Physician"
},
{
"id": 145462,
"tgt": "What causes severe pain in the neck?",
"src": "Patient: I was involved in a wreck on Friday, but because I'm an idiot I chose not to go to the ER haha. I was thrown from the backseat and ended up sitting on the dashboard. I cannot move my head much because my neck hurts still (it does feel a little better though). Everything else is starting to feel fine already, but should I be concerned about the neck? MRI necessary? Doctor: Hello, Welcome to HCM. I understand your concern. It is possible that you injured your neck significantly. It is better to see a traumatologist or orthopaedic, have yourself examined and the doctor would advise you what tests to have. MRI may be one of them. Take care"
},
{
"id": 94345,
"tgt": "Intense abdominal pain, dark, tarry stool, Done MRA. What is the cause?",
"src": "Patient: I keep seeing call your doctor immediately when it pertains to symptoms I ve been having. Yet we cannot find a positive result on any tests. Today marks the sixth month of intense abdominal pain (left side: middle to low). It is more intense on many days than appendicitis (had that 19 years ago). I ve had scopes (endo and colon ); CT scans, SB series (with barium/ Xray ), and most recently an MRA . While they find no cause, I am literally starving as it is the only reliable way to abate the pain. The dark, tarry stool symptom elicits the call your doctor immediately response. Doctor: hi there, you seem to be suffering from peptic ulcer disease and your treatment lies in taking a course of antacids along with a very regularised and punctual diet with low fat and spices. You will also need to take a course of antibiotics to take care of H.Pylori infection. Get your stool examined for occult blood and that if positive could indicate ulcer bleed and explains the black tarry stools. Endoscopy may be repeated. Do visit a gastroenterologist and let him evalute you for athe same. Get your hemoglobin levels also checked as constant loss of even small quantities of blood can lead to iron deficiency anemai and you may need iron supplimentation.Take care."
},
{
"id": 54580,
"tgt": "Suggest treatment for deranged liver function test",
"src": "Patient: Hi mate , my name is Carey and i am 27 years old , approx 180cm and weigh 88kg . I am Male .I have to pass a liver function test to get my drivers licence back , but i keep failing , i havnt consumed alchahol in 2 weeks and i still cant pass , is there anything i can do to pass ???? Doctor: Thank you for posting your query. increase in the level of liver enzymes reflect an INJURY to the Liver. medically, the condition is know as \"Hepatitis\".Lab results reveal MILD hepatitis. You need to test for HepB & C serology too.if all causes of hepatitis are ruled out , then - abstinence from \"Alcohol and drugs\" - LOW or fat free diet should be followed- Moreover, less activity should be carried out (no sports). - use lemon juice (lemonade) once in a day-recheck liver enzymes after 2 to 4 weeks.wish you good health.further queries are welcomed.Health professionals aim to diagnose properly and manage patients according to their limited knowledge. Cure is blessed by the ONE who Created us, whose power and knowledge is unlimited .wish you good health.regards,Dr Tayyab Malik"
},
{
"id": 106129,
"tgt": "Suffering from Asthma for the past 8 years and put on Seretide inhaler 4 times",
"src": "Patient: My niece of 8 years has Asthma . She was put on Seretide inhaler as well Asthavent 4 times . I was under the impression that Asthavent was a short term relieve to sudden attacks. Doctor: hi welcome to healthcaremagic Asthavent's half life is 1.6 hour so u need to take for 4 times or as u required,it's use for short term becouse of its short half life and its quick mechanism of action. thank you"
},
{
"id": 32709,
"tgt": "Can electric shock to body cause swelling in arm and cellulitis?",
"src": "Patient: recived shock to hand and arm from welder was moving ground cable for other before I could rehook cable other began to weld could not let go current ran in hand and out shoulder have swelling in arm somtimes resulting in cellulitus what can I do and is all this because of shock Doctor: Hello Welcome to Health Care Magic.you need to consult the doctor for correct treatment. you may need regular dressing and antibiotics. if required painkillers too. so better you go to general surgeon. Hope this information helps you.Best wishes."
},
{
"id": 177916,
"tgt": "What does watery eyes and runny nose with high temperature indicate?",
"src": "Patient: I have a 2 yr old that has watery eyes a slights runny nose lethargic, hes not eating or barely drinking heavily breathing and high temps of up to 39.6.. just wondering if you could tell me whats wrong or wether i should take him to emergency department thanx. Doctor: Hi,Definitely, a 2 year old with all these problems should be shown to the doctor immediately. No doubts!Dr Uma"
},
{
"id": 223958,
"tgt": "Should i take nordette inspite of having no ejaculation?",
"src": "Patient: Hi. Had intercourse 3 days before my supposed period. I am regular. Condom broke but no ejaculation happened. I am aware that precum can get me pregnant. Question: do i still need to use nordette as ecp? Knowing that no ejaculation happened and I am not on my fertile window? What is the best thing to do? Doctor: HiDr. Purushottam welcomes you to HCM virtual clinic.I have gone through your query. I think I have understood your concern, I will try to suggest you the best possible treatment options.1)First of all do not panic.2)With history of regular periods, even unprotected sex act on 3 days before expected periods comes in safe and non fertile period of the cycle.So there should not be concern about getting pregnancy , even if semen spillage was there or breaking of condom.3) You need not use any emergency contraceptive pill. I will just suggest you to wait for onset of your expected periods. You will get your periods in due course.I hope my answer helps you.Thanks.Wish you good health.With regardsDr Purushottam Neurgaonkar"
},
{
"id": 52178,
"tgt": "What are the symptoms of a liver disease?",
"src": "Patient: a positive titer of 1 1280 with mildly elevated Alk phos (159 U/L) and GGT (116 U/L=4 x normal) but no clinical signs of liver disease, but with diffuse polyarthralgias and fatigue...likely to be PBCor an autoimmune disorder? or just a false positive test Doctor: Hi, Based on your symptoms, there can be chances that you have some infiltrative disease of liver. You must undergo liver biopsy to confirm the diagnosis. Hope I have answered your question. Let me know if I can assist you further. Regards, Dr. Parvinder Singh, Gastroenterologist, Surgical"
},
{
"id": 159699,
"tgt": "Small child, painless swelling behind ears. Taking monocef o with ibugesic, not reducing. Tumor or lymphoma symptoms?",
"src": "Patient: my daughter is 2 ad half yrs old.she has a painless swelling behind her leff ear from few weeks.I m giving her monocef o in combination with ibugesic as per my doctor prescription.other says it is reducing but i am not satisfied.sir i just wanted to know that it is not any kind of tomor or lymphoma symptoms.kindly ans. me i m very worried. Doctor: If it is reducing with drugs given, it is probably due to infection. To know the real cause F.N.A.C. test is required."
},
{
"id": 72296,
"tgt": "What is the reason to experience discomfort in right arm, sweating and chest pressure while having a pacemaker?",
"src": "Patient: I have had a pacemaker for 2 years. I know the pacemaker is working. Just had a check up. Today I have had sweater, uncomfortable feeling my right arm, and chest pressure.... i know go to the ER but I have had thesy symstoms before and they never find any wrong... Oh. I am female, 51. Doctor: Thanks for your question on Healthcare Magic.I can understand your concern. Since you have already checked in emergency room regarding pacemaker and everything was fine, no need to worry for heart related diseases for your symptoms. You are having Menopausal age. During this age, many women show hormonal imbalance and symptoms like chest tightness, sweating, unexplained arm and leg cramps etc. So better to consult gynecologist and get done hormone levels. You may need hormone replacement therapy (HRT) to regulate hormones.Don't worry, you will be alright with all these. Hope I have solved your query. I will be happy to help you further. Wish you good health. Thanks."
},
{
"id": 29750,
"tgt": "Can Brom/PSE/DM Cough Syrup be taken instead of Promethazine DM for flu?",
"src": "Patient: The doctor said I have the flu. I am taking Promethazine dm and it is actually making me very sleepy and weak. I actually feel worse after I take it. I have Brom/pse/dm syruo mor from about 2 months ago when I had bronchitis and it did not treat me the way Promethazine does. Can I take the Brom/pse/dm instead of the promethazine. Doctor: HI, thanks for using healthcare magicPromethazine can cause extreme drowsiness so it understandable that it is causing weakness and drowsiness for you.You can use the alternative medication, it has a decongestant, (pse), mucolytic (break up the mucus) and should also suppress the cough as well.If you can tolerate this medication then you can use it to help with your symptoms.I hope this helps"
},
{
"id": 66697,
"tgt": "What causes a painful lump on the leg?",
"src": "Patient: hi i was laying in my bed when i felt a sharp pain in my leg i guessed because i was pressing my toe on my leg but I thought it shouldnt hurt that bad. So i sat up and looked at the area of my leg (upper ankle, lower calf). I was should to see a faint white discoloration on my leg it was very small a transparent but when I went to feel it there was some sort of lump beneath the skin. It extended on all sides a little past the discoloration. After looking at the bump I remembered a quick sharp pain maybe an hour or two before which I realized could have been some sort of bug bite. So I went online and looked at slide shows of many bug bites but none matched mine. It might be that the bite isnt apparent enough as many of the sites said it could take 4 to 5 hours for the bite to be truly visible. So I was wandering if you could help me. When I touch the lump I do feel pain but its not severe or really that bad. It hurts the worst when I press on the bone maybe less than a quarter inch away from it. That is what made me sit up and examine it in the first place. I dont want to go the ER if it isnt an emergency as Im supposed to be going on a trip tomorrow. If you could help in advance that would be great. Doctor: This one is a bursa or coud be some benign tumor like condition like neurofibroma! However it could be some abnormal growth too!Please go for an FNAC test to relieve your tension.All the best and regards,"
},
{
"id": 168919,
"tgt": "Suggest medication for viral infection",
"src": "Patient: my son fell down the stairs in my house last may 2, brought him to doctor everything looks fine after 3 weeks he got high fever i brought him again to his doctor he has viral infection my worry if this is side effect of fallen to stair that s why he got sick pls advise. Doctor: Hi, Can understand your concerns. Fall from stairs cannot lead to viral infection as viral infection is caused by a virus. I suggest you that you should not relate both of these things. Viral fever usually remain for 2 days. I hope this will help you. Take care."
},
{
"id": 1712,
"tgt": "What will be the effect of Progynova to the baby if i get pregnant?",
"src": "Patient: Hi. I m trying to get pregnant and I have just received my Pregnyl injection last March 24 which was the day before yesterday. I took Progynova last night as prescribed by my doctor. However, I am a bit worried about it because I read in the leaflet that came with the medicine that Progynova should not be taken by pregnant women. The doctor said I m fertile on the 25, 26, 27th and 28th of this month so I should try to get pregnant by then. I m also going to be taking Progynova on those days. What if I get pregnant? What will be the effect of Progynova to the baby? Thanks so much! Doctor: Hi there, I have understood your concern. I will suggest you the best possible treatment options. 1. First of all do not panic. Please fully trust in your treating doctor .2. You might be aware that organ formation of the baby starts 28 days after the conception. Your doctor must have given the progynova tablets for 15 days from ovulation. Progynova tablet is a estrogen hormone tablet. This is to provide good endometrial lining for the implantation of the embryo. It's well before organ formation. So please do not worry. I hope this answer helps you. Thanks. May God bless you with a bundle of joy. Dr. Purushottam Neurgaonkar."
},
{
"id": 32836,
"tgt": "What causes body pain along with swollen eyes and stomach cramps while suffering from viral infection?",
"src": "Patient: Hi, for the pat three weeks I've had what my doctors think is a virus but I'm not sure about that anymore. At first I had a really sore throat ( no strep), this eventually stopped. Then a runny nose and then a bad cough, eventually the runny nose stopped and the cough is slowly fading. I've also had a lot of night sweating. Just recently though Ive had random Sharp pains through out my body and to top it off my left eye is swelling and my stomach is cramping alot. Doctor: Welcome to HCM.I understand your concern all the complains you have mentioned here indicate that is is definately an infection that you are suffering from but to confirm it you should undergo a blood test haemogram as it can both differentiate between viral and bacterial infection.Also crampy abdominal pain with night sweating indicates that you could also be having stomach infection now so do a stool routine test and along with both reports take opinion of general physician.No need to worry.Medications can cure infections.Hope i answered your query.Thanks and regards."
},
{
"id": 176521,
"tgt": "What causes vomiting and stomach ache in child?",
"src": "Patient: My 6 YO grandaughter woke around four w vomiting and pain. We took her to urgent care center and they said she had an air bubble in stomach, gave her zofran and were told to give motrin prn they determined this by a normal x-ray. Is this dangerous and what could be cause? Doctor: Hello. I just read through your question.Wh you are describing is not dangerous. Another diagnosis to consider, however, is a stomach virus. This commonly causes vomiting in childhood. Making sure she is well hydrated is the most important thing to do. As long as her hydration status remains normal, she will be fine, and there is nothing to be overly concerned about."
},
{
"id": 59323,
"tgt": "Have abdominal pain and fatigue. Have fatty liver. Should I worry?",
"src": "Patient: 3 months before i diagnesd as fatty liver , from that day i felt lower right abdomen pain and terrible fatigue .. i stoped drinking .. and smoking . i took ultrasound scan which resulted bright echotexture fatty liver. i am 36 years old and wt 60 , there is no other notable symptoms i have,, i just wanted to know its fattly liver or something worse than that ,, i wold like to know fatigue due to fatty liver lasts as long as 3 months in a life of no alcochol or smoke Doctor: Hello, jayapen, Fatty liver can be associated with high lipid levels ,diabetes and overweight. If your blood sugar and lipid levels are normal , then the condition is not too bad. Also , your liver blood tests especially enzyme levels , if high that may be more serious. In most cases fatty liver could be reversed with a low fat diet, staying away from anything that can damage the liver such as alcohol and drugs including over the counter or prescription drugs. I would recommend that you start taking Vit-E 800 i.u. daily and Vit-C 1000 mg. daily. High protein diet , fresh fruits and vegetables, nuts are all good to improve liver. Hope you can discuss these with your doctor and take appropriate action. I wish you well."
},
{
"id": 71681,
"tgt": "Suggest treatment for having shadows in between lungs",
"src": "Patient: hi, this is geetha, my is in icu for the past 20 days, he developed breathing problem and was admitted to the hosptial. further he had devoloped urea level high and creating level as 1 and being done dialysis 2 times and above he had developed some shadows in between lungs. can u pl explain what treatment wud be the best. my mail id is YYYY@YYYY Doctor: Thanks for your question on Healthcare Magic.I can understand your concern. Mediastium is the area lies between lungs. Mediastium carries heart, aorta, vena cava, lymphnodes etc. So abnormality of any of these structures can appear as shadow between lungs.So get done CT thorax. CT thorax is must to identify mediastinal pathology.So discuss about CT thorax with her doctor.Hope I have solved your query. I will be happy to help you further. Wishing good health to your sister. Thanks."
},
{
"id": 225671,
"tgt": "Forgot taking birth control pill, withdrawal bleeding lasted only 3 days. Chances of pregnancy? When can i get accurate results?",
"src": "Patient: I'm on the pill but forgot to take the pill on the saturday before my 7 day break starting on the Thursday. Usually in this circumstance I would back to back packs but I totally forgot to and then my partner and I had sex without a condom 2 days later on the Saturday. I then got my withdrawal bleed on the Sunday but it only last 3 days which is unusual. I am now worried I could be pregnant. This all happened two weeks ago so if I did a pregnancy test, would I get an accurate result? Doctor: Hi thanks for asking on HCM.As you already had withdrawal bleeding, chances of pregnancy are very less(almost negligible). To be sure you can do pregnancy test now( better to go for serum beta HCG test, blood test) and it will clear your doubts.As it is already 2 weeks back, this test can be done now for accurate results.Good LuckHope this information is helpful and informative.If you have further queries you can directly reach out to me on the following URL:http://doctor.healthcaremagic.com/doctors/dr-deepti-goyal/65111"
},
{
"id": 110281,
"tgt": "What causes back pain?",
"src": "Patient: Just fed up with my moving back pain that comes and goes it s not kidney stones I have had it checked out it comes about every 6to 8 months and lasts about 3 days and night but very painful full I have taken pain killers today which are strong 30mg but not helped it Doctor: Hi,Welcome to healthcare magic.After going through your query I think your You are suffering from acute backache.Treatment of this is exercises and analgesics (diclofenac 50mg three times a day after meals). Sometimes vitamin D deficiency is the cause so get your vitamin D checked .If it is low then vitamin D supplementation can be taken. I think your query answered.Welcome to any follow up query"
},
{
"id": 199615,
"tgt": "What causes redness around the penis?",
"src": "Patient: Hello I'm an uncircumcised male and have had no sexual encounters. I have a ring of red surrounding the pee hole it appears to be connected to the frenulum however I may be mistaken. Also perhaps unrelated I cannot pull my foreskin all the way over the head of my penis, I've tried stretching but to little to no help. I'd appreciate some info thanks Doctor: HIWell come to HCMI really appreciate your concern, looking to the symptom given here this could be fungal infection and condition can be best treated with 'Miconazole cream\" apply this three times in day, such lesion need to be treated from grass rooted level because chances of reoccurrence is very likely, hope this information helps, take care."
},
{
"id": 185244,
"tgt": "Suggest treatment for gum disease",
"src": "Patient: hello during the past three weeks i have been waking up with qiuet a large amount of blood in my mouth i was treated a week ago by my dentist who said i had gum disease i have been flossing cleaning as told but it still is happening iim wondeing if this could be anything else as it only happens when i wake up , i get a little blood when i floss but nothing like in the morning thanks steven Doctor: Thanks for using Health Care Magic.Read your query.The bleeding in the gums suggest that you have gingivitis and periodontal inflammation which is usually the cause for bleeding in the mouth.In your query , you have not mentioned whether you have got your teeth professionally cleaned by your dentist , if not please get it done and then follow the correct method of brushing and flossing.Use chlorhexidine mouth wash for a while.If you have got your teeth cleaned by your dentist and still it bleeds more , I would advice you to consult your GP doctor to rule any systemic conditions which may be the cause.Hope this was helpful.Thanks and regards."
},
{
"id": 151683,
"tgt": "Saddle parasthesia, lateral sciatica, numbness, back pain, taking codeine",
"src": "Patient: I went to my local accident and emergency yesterday regarding saddle parasthesia. They said I don t have it but I have almost all the symptoms. They won t give me a scan but I feel like I should have one. I also have lateral sciatica . I do have numbness and an occasional burning sensation at my very lower back but I m not incontinent and I could feel when I had an examination of my rectum . Please help as I m taking codeine and I m still in pain. Doctor: Hello. Thanks for writing to us. The symptoms you are having are likely to be due to nerve compression at the level of sacral spine. A nerve conduction velocity and MRI of the lumbo sacral spine will help in proper diagnosis. For symptomatic relief you can take methylcobalamin and Vitamin B complex supplements regularly after consulting your physician. I hope this information has been both informative and helpful for you. Regards, Dr. Rakhi Tayal drrakhitayal@gmail.com"
},
{
"id": 123377,
"tgt": "Cause for swelling in the leg?",
"src": "Patient: so I cut my shin open around a year ago, it was pretty deep and I did not go to a doctor until after it healed. The Dr just gave me a tetnis shot. I noticed after the injury my foot on that same leg would swell up pretty good. Well it still swells up on me and it has been a year. Why would this be? Doctor: Hello, As you have not taken initial care there might be some microvasculature trauma to the capillaries. Which may have healed by now but due to the cut and malaligned healing there will be a disturbed circulation for now. I will advise going hot water fermentation, gentle massage over the injured area which will ease any knots leading to venous return trouble and then ankle toe movements. This should help you with avoiding the swelling. Also, getting it examined for any vascular insufficiency is not going to harm you in any way. Hope I have answered your query. Let me know if I can assist you further. Take care Regards, Jay Indravadan Patel, Physical Therapist or Physiotherapist"
},
{
"id": 139796,
"tgt": "What causes numbness on the head with pain at the bottom?",
"src": "Patient: I WOKE UP TOOK MY SHOWER AND AS I WAS BRUSHING MY HAIR I FELT MY RIGHT SIDE OF MY WHOLE HEAD NUMB AND IT STILL IS FOR THE PAST 2 HOURS WHY^/ ?? 25 YEARS OLD 5F9 130LB NO MEDICAL BESIDEMETHADONE ON A METH PROGRAM 150MG AND WHEN I TOUCH THE RIGHT BOTTOM OF MY HEAD IT REALY HURTS. BUT I DIDNT RERCEIVE NO SHOTS Doctor: Hello, It was probably a bad head and neck position during the sleep that may have compressed your nerves that supply the head. Normally, it should subside for a few days. If the symptoms persist for more than a few days you should see your doctor and get evaluated further. Hope I have answered your query. Let me know if I can assist you further. Take care Regards, Dr. Erion Spaho, Neurologist, Surgical"
},
{
"id": 154609,
"tgt": "Alternative cure for cancer?",
"src": "Patient: Dear Doctor, Surya tv broadcasted a story about Mr Harindranath, a chemical engineer hailing from Kerala, got complete cure of cancer by using ASHWAGANDHAM. What do you think about ASHWAGANDHAM? Where can I buy it from if it helps cure cancer? Do you have contact information for Mr Harindranath? Thanks Rajan, USA. Doctor: Hi,Thanks for writing in.Some medications like ashwagandha might help control cancer. However cancer should not be treated by this medicine alone and taking regular medical treatment is also advised.Ashwagandha is an ayurvedic preparation and has been in use for hundreds of years for its medicinal properties. Apart from suppressing certain cancer cells, it can also be used for conditions like arthritis, anxiety, trouble sleeping (insomnia), tumors, tuberculosis, asthma, a skin condition marked by white patchiness (leukoderma), bronchitis, backache, fibromyalgia, menstrual problems, hiccups, and chronic liver disease.Cancer can be due to many causes and all cancers cannot be cured with ashwagandha. You can purchase it from the retail store of well known ayurvedic medicine manufacturers in your place. Please take in recommended and discuss with your doctor if there are any potential side effects when taking chemotherapy treatment. Please do not worry."
},
{
"id": 119107,
"tgt": "Have hot flushes at night, itchy legs causing bleeding. Have high eusophil count, heavy periods. Related?",
"src": "Patient: Hi I keep getting hot flushes I'm only 20 but il be fine then get hot and clammy and calm down its getting especially worse as night I have no heating in my room and don't wear pjs anymore. I then get extremely itchy legs! Where I end up bleeding I itch so much! I have a high eusophil count as well. I also have extremely heavy periods if that's anything to do with it. Thanka Doctor: Hello, Your skin itching problem may be because of high eosinophil count.Your other symptoms may be due to thyroid problem .Get your thyroid profile done as thyroid disorder can cause all these type of problem."
},
{
"id": 169797,
"tgt": "What is the treatment for expressive language disorder?",
"src": "Patient: My son had an expressive language delay and was treated at 14 months. Now at age 10 I am noticing that socially he is struggling. He speaks about things that he feels confident about and I feel it is alienating him socially. If I were to have him evaluated again (he received speech, physical and currently is receiving Occupational therapy would speech therapy address these issues? We are correcting and redirecting and modeling for him but I am unsure if that is enough. Please advise. Doctor: Dear, Welcome to HCMWe understand your concernsI went through your details. Expressive language disorder is a communication disorder in which there are difficulties with verbal and written expression. Your son was treated at the age of 14 months. You did not mention for how many days.Treatment for expressive language disorder depends on its severity, but might include therapy with a speech pathologist. Most of the time, the treatment runs into adolescent periods. I suggest you should evaluate your son again for the problem. I am of the opinion that the problem has not been fully cured. I suggest you consult an ENT specialist and then speech language pathologist.If you require more of my help in this aspect, please use this URL. http://goo.gl/aYW2pR. Make sure that you include every minute detail possible. Hope this answers your query. Further clarifications are welcome.Good luck. Happy New Year. Take care."
},
{
"id": 184508,
"tgt": "What causes white bump inside mouth?",
"src": "Patient: Im 17 with no history of medical problems. Yesterday I found a small white bump on the left inside of my mouth. It doesnt hurt unless I scrape my teeth along it. And I'm really worried on what it could be. I was eating sunflower seeds the day before I found the bump and was wondering if one had maybe caught onto my lip. But why would that cause a bump? Doctor: Hello thanks for consulting at hcm..You may have inadvertently bit into your buccal mucosa on the inner side of the oral cavity.. resulting in microtrauma to the tissues ..hence the bump,.,.please blow air with mouth closed..so that the sharp teeth surfaces do not touch the buccal mucosa..please trim the sharp surfaces of the teeth by consulting a oral physician..if problem persists..nothing to worry as of now,,hope it helps,,take care,,,"
},
{
"id": 72074,
"tgt": "What causes pain in chest and shortness of breath?",
"src": "Patient: HI I am a 41 yr old female. I've been dealing with some chest pains and shortness of breath for the past 3 monthe. Two days ago something new has started. Under my left breast I am having a fluttering sensation. It doesn't last long but is happening more often. This is in the same spot where I used to feel a sharp pain. Now it is a burning pain, sometimes and the flutters have started. Is this heart related or could it be muscle spasmsS Doctor: Thanks for your question on Healthcare Magic.I can understand your concern. You are having chest pain and breathing difficulty since long (3 months). New symptom is fluttering sensations.So we should definitely rule out heart diseases (arrhythmia - rhythm disturbances in heart) first.So get done blood pressure monitoring, ecg, 2d echo, stress test and Holter monitoring (24 hours continuous recording of Ecg).If all these are normal then no need to worry for arrhythmia or other heart diseases. Sometimes muscular twitching can also cause similar symptoms. So apply warm water pad on affected areas. Take muscle relaxant drugs. Don't worry, you will be alright with all these. But first rule out heart diseases. Hope I have solved your query. I will be happy to help you further. Wish you good health. Thanks."
},
{
"id": 31843,
"tgt": "Suggest treatment for pain in the lower rectum",
"src": "Patient: Hello. I have been with pain in my lower rectum for the last three weeks. I thought it was caused by antibiotics I was taking for a tooth infection, as I don\u2019t usually have any rectum problems. Nothing like this anyway. About 8 days ago, while sleeping, the pain in my rectum was extreme. It was waking me up every 10 to 15 minutes. Sharp stubbing pain. The next day I got a fever (100). I visited the doctor and it was determine that I had a combination of issues: a) an external hemorrhoid and b) a virus was developing, perhaps the flu. As soon as I got home the sickness of whatever I had took hold. I had the chills, fever, and diarrhea. The first two lasted for a day. The diarrhea I still have it. It has been 8 days and I still have the diarrhea and the lower rectal pains. It\u2019s not diarrhea all the time; occasionally I have solid or loose stools. I\u2019m going to the bathroom very often, an average of 4 to 8 times per day. When I have to go I get a sense of urgency. The defecation is relatively small and is either diarrhea, soft stools, or very thin solid stools. The hemorrhoid is still there, if that is what it is. I\u2019m using creams and it is not going away. What do you think I\u2019m going through? I had stomach virus before and the diarrhea only lasted me couple days. Thanks. Doctor: HiI would suggest to replenish your salts and electrolytes which you are loosing through stools.use copious amount of ORS.For external haemorhhoid simple evacuation of the haematoma(blood)inside is adequate.If these are internal haemorrhoid which are coming out on defecation,then surgical treatments are needed such asInjection of sclerosant ie almond oilBandingStapled haemorrhoidectomyOr open haemorrhoidectomyThanx"
},
{
"id": 80632,
"tgt": "How to overcome my breathlessness problem?",
"src": "Patient: I am using a Symbicort Turbohaler - red packaging and when I inhale nothing seems to happen and my breathlessness is getting worse. Are there any other people having problems - I have used this for two months before this problem began with good results. Doctor: Hello dear, thanks for your question on HCM. I can understand your situation and problem. You are not improving with drug, so following are the possible causes. 1. Chronic lung infection. 2. Worsening of bronchitis. 3. Pulmonary fibrosis. So,In my opinion you should consult pulmonologist and get done1. Clinical examination of respiratory system. 2. Chest x ray. 3. PFT ( pulmonary function test )4. CT thorax if required. Chest x ray and CT thorax are needed to rule out lung infection and fibrosis. PFT is needed to rule out worsening of bronchitis. Lung infection is the most common cause for uncontrolled symptoms. So you may need antibiotic and other bronchodilators too.So consult pulmonologist and discuss all these."
},
{
"id": 182900,
"tgt": "Are blood borne diseases possible after denture implant injury?",
"src": "Patient: hello im a dental lab technician 3 weeks ago i cut myself witha duplicate of a denture, it is a copy of acrylic of the one the patient actually wears and it is made of an impression of the original denture, i trully fear getting any blood borne disease!!! im so anxious havent been sleeping, my muscles hurt, i feel nauseous, and my appetite has lowered.. i do suffer from hypocondria... risk? Doctor: Hello,Proper disinfecting techniques should be used by the laboratory and the dental office before sending you dental lab cases. It is unlikely that a patient blood borne disease has been transmitted to you. Some contamination may have entered your bloodstream from being cut from your tools or laboratory materials. You may have an infected cut due to poor healing. How deep was your cut? Did you clean out the cut sufficiently?Take some anti-inflammatory medication such as Tylenol or Motrin in. Check for signs of inflammation around the cut such as swelling, redness, or any type of drainage. Use some antibacterial medication and keep the area clean. See your physician if you continue to experience symptoms. You may need a prescription antibiotic. You can have simple blood tests performed to put your mind at ease.Thank you for your inquiry. I understand your concern because you do not know how the materials were handled prior to arriving at your lab or the history of the patient. You can always call the dental office to explain your concern and ask for some information on the particular patient. I hope you will start to feel better soon."
},
{
"id": 44820,
"tgt": "What medication is suitable to cure PCOD ?",
"src": "Patient: hi,m 33 years old suffering from pcod last 10 years,i m trying for another baby,started treatment for this,my follicular studt on 19 day shows small follicular size and endometrium size is 7.1mm, doctor has advised me to take duphastone tablet to conceive,for 8 days from 20 day of period and when period will start ,i should start letoval on 3 day pf period.my problem is that i m going out ofstation this month and will come back in june & my husband will not be there with me,can i take these tablet after one month,when i will be back and then only i can try for pregnancy . plz advice me. Doctor: Hello sonu; welcome to HealthcareMagic If you have regular periods then yes you should start the course of the medicines as advised from next cycle.The reason is even though you take those medicines if your husband is not with you there will be no result with the medicines.All said but still you should consult your treating Gynaec for opinion and advise as she/he will be expecting to see you after the course prescribed;so please consult and then follow the advice. Thanks"
},
{
"id": 189116,
"tgt": "Have hard, very small bump on the back of my hard palate. What could be it?",
"src": "Patient: I have a hard, very small bump on the back of my hard palate. you cant see it with the naked eye. its not an abnormal color but I can feel it with my tongue. I dont know how long it has been there but I just felt it the other day and it doesnt seem to be getting bigger. Im going to the dentist on Tuesday becuase I think I have a cavity also. Doctor: Dear friend.Thanks for sharing your concern.As your history suggests it can be torus palatinus,a bony swelling seen in the mid line of the palate.It can be well diagnosed with the help of an intra oral x ray.This lesion is usually not treated since it is a benign lesion.(size remains the same).Rarely if required it can be surgically reduced for giving prosthetic appliance (Dentures in older) to some patients.Regarding your cavity, it should be well diagnosed and treated with suitable filling.meanwhile maintain good oral hygiene .Let me know for further query.Thanks.Take care."
},
{
"id": 167317,
"tgt": "What causes hard lump below knee?",
"src": "Patient: I m wondering if this bump on my 3 year old daughter s leg is a calcium deposit? it s below the knee and feels hard, it doesn t hurt her, and it doesn t effect her in any way. i noticed it after some sort of allergic reaction, it went away and now it s back Doctor: Hi...by what you say it looks like a small area of fibrosis for which reason has to be worked up. It may be a calcified node also as you have rightly suggested. I suggest you consult your pediatrician regarding this.Regards - Dr. Sumanth"
},
{
"id": 14558,
"tgt": "What can be the itchy rashes below my belly during pregnancy?",
"src": "Patient: presently I am 10 weeks pregnant and about 7cm below my belly botton i've noticed a rash. It's red clusters of tiny buttons that group together in a rash. It's very itchy and blisters when I scratch it. Can you possibly diagnose and recommend something? Doctor: Hello. Thanks for writing to us at healthcaremagicThis seems like a Fungal Infection/ Tinea Cruris; patches of Tinea usually start as bumps which tend to expand and coalesce to form a peripheral raised margin with central clearing. Itching is common in fungal infections.The location too seems to suggest that it is most likely a fungal infection; fungal infections are common at sites such as waist and pubic region.I would suggest you to use an OTC topical antifungal clotrimazole 1% cream, twice daily for 4-6 weeks, regularly.Oral antifungals may be avoided during pregnancy.An oral antihistamine e.g hydroxyzine is safe in pregnancy and can be taken if itching is bothersome.Regards"
},
{
"id": 92687,
"tgt": "Abdominal pain, smelly discharge, fever. Taking medication for bacterial vaginosis. Suggestions?",
"src": "Patient: hi there i am 13 yrs old, i have abdominal pain, i have a bad smelling discharge down there, really uncomfortable pain i my abdomen its also swollen there, the discharge smells horrid, im not sexually active or anything, i also get high fevers alot im not sure if thats related, ive been to a doctor before and he said that i had bacterial vaginosis but the medicine isnt working and its not fungal either, my mum is a gynecologist and she refered me to an adolescent gyn clinic but its too far from now and i need to know what it could be as im worried i might not be able to have kids in the future? thanks for your time Doctor: Hi, You have bacterial vaginitis, with high grade fever & foul smelling vaginal discharge. You are already on antibiotics.& still there is no relief. you seem to be very much tensed. Please don't worry. You are not sexually active, so the chances of pelvic inflammation are lesser. If you get treatment on following lines in time, it wont hamper your capacity of child bearing! - Please get your vaginal swab testing for culture & sensitivity. It will guide about choice of effective antibiotic. Please start taking the antibiotic sincerely after an expert's advice. - Cleanliness & hygiene of vagina is of imence importance. Probably that's why you were refered to adolescent clinic by your mother. Please do visit it & know the health tips in time. _ you must stick to habbit of cleanliness & take a comlpeate course of medicines . Please take care."
},
{
"id": 184179,
"tgt": "How can a hard lump in the gums be treated?",
"src": "Patient: I had a hard lump in my gums after a root canal and dentist said lets wait to see what happens and it does not hurt. It was hard and I decided to put salt on it and the bump actually crawled up into my gums and the bump was gone but now I just feel the lump in where the cheek and top part of your gums meet above my teeth. Any ideas what it is and what I can do for it? Doctor: Thanks for using Health Care magic.Read your query.The swelling and the lump suggests that the abscess has not healed completely after the root canal treatment.It has to be excised or removed.I would suggest you to visit a good endodontist and have a radio graph done to see if the root canal treatment is done properly,if not retreat the tooth.You can also be advice to get a peri-apical surgery to remove the abscessed area depending upon how it is.Antibiotics like amoxicillin on prescription may be needful.Continue with salt water gargling.Hope this was useful.Thanks and regards."
},
{
"id": 217252,
"tgt": "What to do if having pain due to prostatis even after taking medicines?",
"src": "Patient: my father is suffering of prostatitis.In ultrasound enlarged prostate glands were detected.The treating doctor (urologist) says that it can be treated by medicines only..but after one month of treatment he is still in pain...patient is 79 yrs old.can we go for surgury ? suggest Doctor: Hi, . I can understand your concern. As you say pain is still persisting after one month of the treatment , then I think you should consult the urologist again or go for the second opinion. Before that get one more ultrasound done which will show if there are any changes in prostate during this one month. If the symptoms persist then it advisable to undergo surgery . This will also help to rule out the cancer by his histopathology reports . Take care and wish him all the best,"
},
{
"id": 108522,
"tgt": "Suggest treatment for severe back pain",
"src": "Patient: I had anx ray done on my spine and I have lived with severe back pain and daily problems with back muscles feeling cramped beyond relief to where I cannot even hold my upper body upright..its almost like a burning pain in my muscles. I was told my x ray revealed I have mild scoliosis. My doctor wanted me to go for an mri to diagnose the cause of my severe pain and siatic like symptoms. Acquiring scoliosis later in life..I'm 29..can that be a sign of anthing else medically to be aware of to come concerning my onset of mild scoliosis? I've learned scoliosis usually appears in childhood rather than later in adulthood so this has concerned me. Doctor: hiI agree with your doctor.An MRI is a must to asess the cause of scoliosis more so having developed now at this age which may be secondary to musculo-ligamentous inflammatory condition of spinal muscles or some condition of paraspinal muscle,ilio-psoas spasm due to vertebral attachment of ligaments/tendons inflammation or infection abscess of vertebrae etc.The scoliosis may be secondary flexible tilting of spine by pull push by inflamed group of spinal/paraspinal musculature.MRI shall givr position of muscles,ligaments,vertbral joint,disc disease etc which may be causing compensatory shifting to either sideby drag or push.The doctor shall asess your other abdominal organs like kidney,bowels etc in addition ,for inflammation,adhesions,banding with diffuse fibrosis near the spineI believe finding the cause shall make it treatable.You may be asked for fever,urinary and bowel symptoms also if any, h/o acute abdomen etcSo you may subject youself for further tests and evaluation, MRI etc for manageable condition"
},
{
"id": 182737,
"tgt": "Suggest treatment for canker sore",
"src": "Patient: I had braces brackets put up a week ago, and developed a canker sore the next day. however, i woke up today with my top left lip swelling, where the canker sore is. the swelling also seems to be spread to my cheek. i m a 25 year old female having her period now. could this be an allergic reaction? Doctor: Thanks for your query, I have gone through your query.According to your explanation, it looks like you have two different issues, one is aphthous stomatitis and another condition is cellulitis secondary to tooth infection.Consult a oral physician and get it evaluated.for, tooth infection, If i am your treating doctor, i would have prescribed a course of antibiotics like amoxicillin 500mg and metronidazole 400mg tid for 5 days (if you are not allergic). Then i would have suggested to go for RCT or extraction.If it is aphthous ulcer you can take topical anesthetic and analgesics like 2%lignocaine and choline salicylate gel (anabel gel) apply 3-4 times daily before food and topical steroid like triamcinolone acetonide 0.1% 4-5times daily after food.I hope my answer will help you, take care."
},
{
"id": 222000,
"tgt": "What are the chances of pregnancy through non-penetrative sexual activity?",
"src": "Patient: hi doctor, 3 days ago me and a partner kind of went crazy, we did not have sex but he rubbed his dick on me a few times. he did not put it inside and did not ejaculate but i am feeling guilty and am experiencing cramps in my lower amdomen, does that mean i am pregnant? ps. the only fluid that was there was mine. he did not cum nor ejaculate near me. Doctor: DearWe understand your concernsI went through your details. There is nothing to worry. If you are sure that your body friend did not ejaculate within your vagina, there is no chance of being pregnant. To become pregnant, the semen should be ejaculated within vagina, the semen should have enough count of healthy sperm cells, it should be the period of ovum traveling to the uterus etc. Otherwise, there is no chances of pregnancy. Stomach cramps should be due to the anxiety or physical stress. If you require more of my help in this aspect, please use this URL. http://goo.gl/aYW2pR. Make sure that you include every minute detail possible. Hope this answers your query. Further clarifications are welcome.Good luck. Take care."
},
{
"id": 42591,
"tgt": "Is fertisure safe to take for infertility?",
"src": "Patient: hi doctor, i hv been married for 5 years still now i didnt get pregnant one iui failed 2 months back and ivf failed 2 years back again before three days i did hysterolap, and my husband sperm count is 12 million the other doctor prescribed Tab. fertisure m for three months is it safe to take. Doctor: Haiwelcome to hcm you need dna fragmentation test to know about your husband sperm quality. For this sperm count you can go for in vitro fertilisation techniques using imsi where they are select morphologically good sperm to inject into egg .However good quality sperm and egg only produce good embryos. Consult your infertility specialist .Regards Dr .Vanitha devi"
},
{
"id": 70488,
"tgt": "What is a lump in the crease of my left arm?",
"src": "Patient: I had tyroid cancer and spread to lymph nodes in my neck. It has been a year. I feel a lump in the crease of my left arm.if a sleep on it or bend my arm for period of time i feel pressure and it gets numb. I never had a whole body cat scan. Could It be cancer? Or and enlarge, hardened lymph node from the radioactive radiation? Doctor: Hi. The location you described - the crease of arm has no nodes at all in a normal anatomy position. This may be something else , may be related to the joint or the bone. A simple x-ray may be helpful as is the opinion of an Orthopedic Surgeon."
},
{
"id": 119585,
"tgt": "Is muscle spams common post tethered cord surgery?",
"src": "Patient: I am a 39 year old male and I had tethered cord surgery in March of 2011. My symptoms were pain in my legs, buttocks, groin, back and feet with also having muscle spasms and weakness. My legs feel like I ran a marathon about one hour after I get out of bed. A cyst was also noticed from L4 to S2 but nothing was done to the cyst. I still have all the same symptoms and they are getting worse instead of better. I also take 1200 mg of gabapentin a day. Could this pain be from the cyst or is this what a tethered cord can leave a person with. Doctor: Hi, Symptoms in tethered cord occurs not only because of stretching of spinal cord but also because of degeneration in the cord, post surgery symptoms produced as result of stretching will gradually disappear but those of degeneration will not. Prognosis after the surgery is also explained prior to surgery. In case if the cyst persists as in your case there might be a chance that it still causes stretching of the cord. You need to thoroughly examined and investigated only after that prognosis can be ascertained. Take care. Hope I have answered your question. Let me know if I can assist you further. Regards, Dr. Rohan Shanker Tiwari, Orthopedic Surgeon"
},
{
"id": 70390,
"tgt": "Could the small raised bump on the thigh be due to Melanoma?",
"src": "Patient: Hi, my name is Julia. I'm concerned about a small, raised bump on my mothers leg, more like the thigh area. It also has a hole in the middle, black and hard around the sides. Many years ago, when I was a child, I remember her being able to constantly squeeze pus out of it. It was a black head that never went away and it was flat on the surface. A doctor a long time ago, said don't bother it, if it's not bothering you. But recent years it has changed. Nothing comes out of the hole now, it's hard and raised. I'm concerned about melanoma. No skin cancer runs in our family. Overall my mom is healthy, non-smoker, African American in get mid 60's. Doctor: Hi and welcome to HCm. Thanks for the query. This is not usual manifestation of melanoma or any other skin malignant disease. this is obviously some other benign skin lesion such as atheroma,changed black head or follicle inflammation. there is no rreasonn to be worried. it can be surgically removed if there is any concern but i dont think this is neccessary.Wish you good health. Regards"
},
{
"id": 64160,
"tgt": "Suggest medication for lump in the middle of left back of neck",
"src": "Patient: I am 22 years old, about 5'6\", and weigh 120, no major health issues in my past, and rarely even get colds. About a week ago, I noticed there is a lump in the middle of the left side of the back my neck. For the past few weeks, I have been getting pretty bad dizzy spells, in which case 1/3 times the hearing in my left ear becomes muffled, this whole process takes only a few minutes. I also have been getting light headed often, and I'm not one to get light headed or dizzy. The lump is visible and is clearly there to the touch, and when you feel it underneath, it's hard. It also hurts to move my neck, especially to the right and in fast movements. I have gotten blood work done, but the tests came back normal. What might be going on? Doctor: Hi, dear a- my opinionyou have sub acute on chronic tonsillitis with neck gland lymph adenitis with left eustachian salpingitis with its recurring blockage attacks.b- my adviseConsult ENT or Surgeon who would test it with antibiotics and needful drugs, after CBC ,CXR if need be.Hope and wish you to recover fast.Wellcome again for further query in this regard.Have a Gd Day."
},
{
"id": 98459,
"tgt": "How can allergic rhinitis be treated?",
"src": "Patient: Hello. I am allergie to almost all antibotics and have several ad health problems. Anything that is related to the Pencillin family is very deadly to me. I don't have much immune system at all. What type of old fashion home remedies would help a allergy cold? I stay away from as many people & public places as possible. But I have developed a winter allergy cold from being outside around my dogs & chickens. can you help me?I can't afford to pay the fee for your advise so thank you for your time. If I could have afforded it I would have when to the doctor or ER. Sorry I have wasted your time. Doctor: Regarding allergy the the old saying goes `anything below the sun including the sun can cause allergy'.The most important component in the management of allergy is avoidance of specific allergen.Maintain dust free your curtains,furniture,mats,carpets,avoid pets in your bedroom/house,winter/spring season can cause allergy by way of pollen release,wear mask while going out in dusty/polluted areas.These are non medical partof management and medical treatment starts with nasal sprays adjusted according to the clinical improvement"
},
{
"id": 122106,
"tgt": "What a feeling of tightness in the sides of the neck?",
"src": "Patient: I have been feeling a tightness in the sides of my neck, about where my carotid arteries would be. Sometimes it feels like a pain or pressure and in my jaw slightly. Just wondering if this means anything? Should I consider going for an ultrasound of my carotids? Doctor: Hi, No, it does not mean anything do not worry. It is probably a muscle strain or pain that will go away by time. Hope I have answered your query. Let me know if I can assist you further. Regards, Dr. Salah Saad Shoman, Internal Medicine Specialist"
},
{
"id": 74981,
"tgt": "What could swelling on chest suggest?",
"src": "Patient: I am 37 years old male and for the past 3 months I noticed a swelling (the size of a small cherry) on the left side of the chest, between two ribs. I am not sure, but I think everything started while going for physiotherapy and taking osteo-biflex for an issue I had with my knee. Any idea what could be? Thank you. Doctor: Hey there,From looking to your complains it seems nothing to worry at present.But get done USG of local part, if needed then do FNAC.Get done reported and show it to any physician."
},
{
"id": 63175,
"tgt": "Suggest treatment for a lump on the neck",
"src": "Patient: I am a 20 year old male and have had this grape sized lump on the right back die of my kneck and it has ben painful the last 2 years. i havent had it checked due to not having medicaide meedicae or any mone. it hurts alot and i have started to realize and so has man other people a difference in my mood, attitude. i get very mean very quick, tired all the time. and i pulled a knife on one of my loved ones yesterday which i would never of thought of doing. im a hristian and have always had a wonderful life. please help Doctor: Hi, dearI have gone through your question. I can understand your concern. You may have some enlarged lymphnode due to reactive hyperplasia, tuberculosis or lymphoma. Or it can be some soft tissue tumor, cyst or skin adenexal tumor. You should go for fine needle aspiration cytology or biopsy of that lump. It will give you exact diagnosis. Then you should take treatment accordingly. Hope I have answered your question, if you have doubt then I will be happy to answer. Thanks for using health care magic. Wish you a very good health."
},
{
"id": 18098,
"tgt": "What does this EKG report indicate?",
"src": "Patient: Thx. I am 58 year old female who 3 weeks ago had flu, but took Tamiflu and still has sob. But dramatically better. Started on prednisone. On ekg- sinus rhythm negative precirduak t-waves within normal limits. Do I need to worry? What does this mean? I m healthy, with hypothyroidism, take synthroid daily. Doctor: Hello and welcome to \u2018Ask A Doctor\u2019 service. I have reviewed your query and here is my advice. There is slight changes in few of the EKG waves, those are not significant and doesn\u2019t indicate any cardiac abnormalities or angina.If in case of symptoms like ches pain, you can go for blood tests like troponinI to rule out angina. Hope I have answered your query. Let me know if I can assist you further."
},
{
"id": 120250,
"tgt": "Is smoking marijuana safe in people with buerger s disease?",
"src": "Patient: Hello, I have a 32 year old male friend with buergers disease. He s already lost a partial foot from this disorder. He has quit smoking and chewing, however he has not quit smoking marijuana. He uses this because he fears he will become addicted to pain pills. Is marijuana bad to use in his situation? Doctor: Hello, Smoking in any form including marijuana can cause constriction and narrowing of blood vessels and worsen the symptoms. So Marijuana no way help in Buerger disease. Hope I have answered your query. Let me know if I can assist you further. Take care Regards, Dr. Shinas Hussain"
},
{
"id": 195988,
"tgt": "Suggest remedy for problem in left scrotum",
"src": "Patient: sir i m 30 year old and unmarried for last 2 week i have noticed some changes in my left scrotum. i consult to a doctor and he suggest me to go for a ultrasonic test. in the test result doctor said my left scrotum has some grade 3 changes of varicocele and shrinking of testicle. so does it needed surgery. i have no pain in my testicle. Doctor: Hi thank you for querry,As varicocele is much common in left side upto 90 % on left side due to some anatomical distribution.varicocele impair sperm structure,production and function and cause male infertility.semen analysis also required for sperm morphology and count.Ni medical treatment is effective in varicocele because its structural abnormality which only be corrected surgically.Following varicocelectomy, approximately 66-70% of patients have improved bulk semen parameters, and 40-60% of patients have increased conception rates. Because human spermatogenesis takes approximately 72 days, the first improvements in semen analysis results are typically not apparent until 3-4 months after surgery.So hope the answe will help you for making decision for surgery.Wish you a good health."
},
{
"id": 215490,
"tgt": "Suggest treatment for numbness, chills and pain from the jaw to the shoulder",
"src": "Patient: I am 77 years old I ve had many injuries in my younger life. I had by pass surgery in 1995, so far no problems. I had a fusion in my lower back 4 years ago. No further problem except for some discomfort in my lower back, upper hips. In the last few days I have developed some pain in my upper right should and neck. From time to time I have a numbing, chilled sensation from my should to my jaw and lower ear. My doctor in not available until next week. Should I seek other medical help or can this wait until by doctor is available? Doctor: Hello, As a first line management you can take analgesics like paracetamol or aceclofenac for pain relief. If symptoms persists better to consult a physician and get evaluated. We have to rule out possible causes like TMJ dysfunction. Hope I have answered your query. Let me know if I can assist you further. Regards, Dr. Shinas Hussain, General & Family Physician"
},
{
"id": 116163,
"tgt": "What causes high fever and high esr?",
"src": "Patient: had a high fever 103 degree F accompanied by shivering,...blood and urine tests done, malaria infection negative, no infection seen in the urine sample, but the blood report shows a high ESR of 60, and a Total Count(TC) of 15,800 which is above the range.. the patient is experiencing weakness and palpitation... please enlighten on this... Doctor: Hellohigh esr count indicates infection specially TUBERCULOSIS...get his detailed evaluation for tuberculosis i.e TB"
},
{
"id": 83172,
"tgt": "What are the side effects of Deviry with iron tablets?",
"src": "Patient: hi...i am suffering from endometoriosis since years. it fine since 2002.i am 41. now it got complicated again. i got heavy periods and abdominal pain.doctor gave me deviry 10 mg thrice a day with iron tabs. bleeding has stopped now. but i am worried about the side effects of the tablet Doctor: HiDeviry tablet contains hormonal progesterone and iron tablets are nutritional supplements for iron deficiency.These two drugs do not have major drug interactions and can be taken together without any issues.Iron tablets may cause constipation or diarrhoea in some patients.To avoid these side effects,Iron is usually advised to be taken after food.Black tarry stools also occur as a side effect of the drug.Deviry can cause headache,abdominal pain,dizziness,menstrual irregularities as some of the common side effects.Hope I have answered your query. Let me know if I can assist you further. RegardsDr.Saranya Ramadoss, General and Family Physician"
},
{
"id": 171656,
"tgt": "What causes low calcium levels and elevated liver enzymes in new born baby?",
"src": "Patient: Hi I have a question about my newborn nephew he was born yesterday morning at 9:14 he has low calcium levels and high liver enzyme levels and jaundace. They gave him IV calcium last night and it only brought his level up from 8.3 to8.4 so there giving him more this morning. The docotors won t tell us anything what we do know we ve had to make them tell us. Can you pls help me understand what s going? Doctor: Hi,Welcome to Hcm,Its a shame that your doctors dont seem to be explaining you what's happening. But don't worry, I am sure they must be waiting to get some more reports to finally give a complete talk to you. With the info you provided, child seems to be having low calcium which is not so uncommon thing seen in neonates. The most common cause is secondary to vitamin D deficiency which is very commonly seen in neonates these times. The best thing to do is to correct calcium whixh your doctors are doing. Having said that 8.3 calcium is not too low to be so worried about. The more of concern is the jaundice in the baby. However, I need more details regarding mother and child blood groups , gestational age of the baby and feeding status before I could comment more regarding that. Hope I was of some help to you. Take care."
},
{
"id": 49964,
"tgt": "Have hemangiolipoma in right kidney. Can there be a blister, possible lipoma on back?",
"src": "Patient: OK I have a 1cm hemangiolipoma in my right kidney (USN due in a month) I have what appears to be a small (2 cm?) blister like lipoma on my back. I had a frined take a picture of it beacuse I can t get a good look in the mirror. I am an RN and did some research and know that these lipomas can also show up in the brain and lungs so why not the skin? Isn t one question free? Doctor: Hi, Angiomyolipomas of kidneys are known to be associated with an entity called Tuberous sclerosis which can manifest with skin lesions. These are usually seen as crops on face and the angiomyolipomas also are seen in both the kidneys in such patients. Without actually having had a look at the lesion on your back it will be difficult to predict what it could be, but it does not sound like a tuberous sclerosis lesion at present. Your being an RN makes it easy for you to understand medical literature, and thus if at next USG, your doctor picks up an enlargement of the angiolipoma or if there are fresh ones, you should be concerned and discus with your urologist as to the next step. If there is no growth of the lesion, and it is less than 2cm, i suggest you can wait and watch if you have no symptoms."
},
{
"id": 159565,
"tgt": "Has UTI and vaginal bleeding. Is it cancer?",
"src": "Patient: my mother is 82 and is having her second uti in a couple of months....this time she presented with vaginal bleeding ....but the anitbiotic she is on now seems to be helping ie no more pain or bleeding......however her physician ordered an ultrasound and she has waited 2 days with no results back yet....she is very scared it may be something else.....was this test just done as a precautionary measure or do you think her dr. suspects cancer? Doctor: HI, Welcome to HCM, I am Dr. Das FIrst of all, bleeding from vagina may indicate some premalignant conditions in this age group. If, USG is ok then the patient should be examined for any lesion by using the speculum. What you will do? Wait for few days for any recurrence of bleeding and then if bleeding occur again go to gynaecologist for a thorough checkup."
},
{
"id": 220988,
"tgt": "Can masturbating in pregnancy cause harm?",
"src": "Patient: Hi, I'm 4 weeks away from my due date... I'm pregnant!And I have been occasionally masturbating but when I do I make sure not to be too rough. But after I orgasm I realize I bleed. At first it's small blood clots n then it's just light red blood. Could this be harmfull?? Doctor: Hallow Dear,Neither masturbation nor sexual intercourse is harmful during pregnancy if specifically not contraindicated. Deep insertion of penis or dildo is to be avoided to safeguard the pregnancy from irritating the uterus. The bleeding you are having after masturbation suggests:1. Deep insertion of dildo or2. Some trauma inside the vagina. To avoid any risk of infection, better report to your Obstetrician. If you are more than 36 weeks now, I would advise you to refrain from the masturbation now. Hope this helps you.Dr. Nishikant Shrotri"
},
{
"id": 188472,
"tgt": "Scheduled extraction of teeth, breastfeeding. Using lidocaine or articaine. Express concern for newborn?",
"src": "Patient: GOOD EVENING, I AM SCHEDULED FOR AN EXTRACTION OF SEVERAL TEETH I AM TOLD THE DENTIST WILL BE USING EITHER LIDOCAINE OR ARTICAINE I WAS ALSO SHOWN TO RECOMMEND LIDOCAINE OVER THE OTHER ANYWAYS I AM BREASTFEEDING AND I HAVE BEEN USING HERBS FOR OVER 20 YEARS I HAVEN'T HAD ANT CONVENTIONAL MEDICINE IN MY SYSTEM FOR ABOUT 25 YEARS NOW...I AM SERIOUSLY CONCERNED FOR MY NEWBORN YYYY@YYYY Doctor: Hello,Welcome to health care magic forum.I have read your concern.There is nothing to worry to extract teeth while breast feeding.The only concern is-you need to take rest after extraction of teeth.Certain antibiotics can cause diarrhea to the child while breast feeding.Please do inform your dentist regarding this.You can proceed with the required treatment to be done.Meanwhile, maintain oral hygiene well.Take a balanced diet.Hope this helps."
},
{
"id": 63867,
"tgt": "Suggest remedy for sore lumps on forearm",
"src": "Patient: I have a raise lump on the back of my right forearm. It is warm to the touch and red. I have noticed it to be getting bigger throughout the day. It is alittle sore but nothing to bad. I have had no recent injurys to my arm and it does not look like a insect bite. It is about 2\"x2\" and has a hard spot in the middle. Doctor: Hi dear this appears to be Sebaceous Cyst with infection. Antibiotics , Antiinflammatory under Er Surgeon guidance would resolve your issue.Hit thanks.Gd day."
},
{
"id": 155440,
"tgt": "Suggest treatment for aneroid carcinoma?",
"src": "Patient: Hi doctor , my dad is 81 yrs old has been recently diognized with anedoid systic carcinoma of the left nesal sinus tumor.we hv taken 2 opinions from renowned ent specialist who hv adviced total epilectamy as radiation does not work on this cancer . He has had a cbag done in 1996 , otherwise he's healthy n has no other medical complications. My mother is worried . We understand that this is a slow growing cancer but can b very painful if it can get into the nerves so we thought we should go ahead with surgery taking the heart risks involved .Can u advice if it would b worth taking the risk ? Doctor: You have been advised correctly. Adenoid cystic carcinoma is a tumor arising from salivary glands anywhere in the oral or nasal cavity. It is indeed a slow growing tumor but it can spread to other parts of the body when it becomes curable. If it hasnt spread, then the only curative option is surgical removal. Radiation has no role. However, I would suggest that you get the surgery done by a cancer surgeon and not an ENT specialist as complete removal is essential else the tumor will come back. CABG done 8 years back is not a problem with today's safe anesthesia."
},
{
"id": 37432,
"tgt": "What causes infection after belly button piercing?",
"src": "Patient: Hi my names brandy. I just have a question about a belly button piercing. I had it pierced like 8 years ago and then after 6 years it randomly got infected and ended up rejecting so I took it out and let it grow in. A piercer thinks it happened because it wasn t properly pierced in the first place and told me I could let it grow in and it would probably be fine to get it done again. So I waited a year and then got it repierced. Its been pierced for like a year now and at first wasn t getting better. I thought I was allergic to the metal (because some of my other piercings have done that and gold seems to be fine) so I put gold in it and I thought it was getting better. There was always a little bump from the scare tissue from the previous one but for the last couple months it was fine wasn t bothering me or anything. But then it randomly like a week ago it started oozing a bit and has been doing that ever since. Im not sure why that happened. I was wondering if that could be from the scare tissue from the previous one? Will the scar tissue affect the piercing or ever go away? I was just wondering what your thoughts were on it. Doctor: Hi,Give it having piercing ring permanently.Clean the wound with antiseptic lotion and get dressing done with antibiotic.Go for one oral antibiotic medicine course for 5 days.In future if you try for piercing ring again, there are all chances of getting infection again.Ok and take care."
},
{
"id": 44108,
"tgt": "Bilateral tubal block, removed through surgery. Taking metformin. Tips to conceive quickly?",
"src": "Patient: i am having bilateral tubal block so i done tubo plasty surgery on 28/8/12 blocks are cleared in this sugery.... doctor adviced me to take rest for 2 months strictly said dont make contact with ur husband..please tell when i make wiyh my husband give tips to getting conceive soon as possible and im taking oosure and metformin 500mg Doctor: Just confirm that your tubes are patent by HSG after 6-8 weeks after surgery. If patent you can try naturally/IUI/IVF."
},
{
"id": 40675,
"tgt": "When is conception possible after taking M2 Tone?",
"src": "Patient: Am 43 my doctor recommended m2tone , having started using it for the past two month , have started meet ing with my hubby during my fertile time but no pregnancy yet , how long doesn t it take m2tone work, should I continue use till I get pregnant, what can I do that can make me get pregnant quickly. Thank u Doctor: M2 tone is just a uterine tonic.It will help in better functioning of your uterus. U can expect conception only if all yours and husband\"s parameters are normal. So first do all your investigations."
},
{
"id": 91740,
"tgt": "What could cause swelling in spleen and abdomen pain?",
"src": "Patient: Hello, my wife is 26 years of age, she has upper abdomen pain ( left side ) her spleen is swollen. ( 14 cm) blood work is all normal. she has the pain that's now rolling to her back. she has fluid in around her spleen and left lower lung. she is very tender to the touch. no one can seem to help. Doctor: Hi.Gone through the history you have provided . It is possible to get such a finding due to trauma to the left side inadvertently or knowingly.Another possible reason can be typhoid or malaria or anemia and so on.You need to undergo all the relevant blood tests for co-relation of the symptoms with the signs the Doctors must have found. A possibility of acute pancreatitis is also there . Serum amylase and lipase levels can be done to confirm . Rush Her to the ER and get her admitted."
},
{
"id": 219891,
"tgt": "What causes lower abdominal pain during pregnancy?",
"src": "Patient: hi, im 11 weeks pregent. im having very little brown colour bleeding since 3 days. doctor did the scan and told that baby is fine. and the leakage is from outside the utrus. & she suggested me to take duphaston for 15 days. im having slight lower abdominal pain on my right side. is everything fine with my baby, pls help. Doctor: Hello dear,I understand your concern.In my opinion there are various causes for bleeding during pregnancy like :-1)Thyroid abnormalities .2)Progesterone insufficiency.3)Abnormal pregnancy like miscarriage or ectopic pregnancy.4)Bleeding from cervix.etc.So all the above causes should be ruled out by physical examination and necessary investigations.Ultrasound helps to know the well being of fetus.As ultrasound is normal you need not worry.Thyroid profile helps to rule out any thyroid abnormalities.As per your information the bleeding is from outside the uterus which might indicate the bleeding from cervix.Don't worry in some cases even though with the presence of bleeding everything will be normal and pregnancy continues till term and have good outcome.Relax.Avoid physical stress and intercourse.Take folic acid and progesterone supplements.Take iron rich diet and iron supplements to combat anaemia caused due to blood loss.Hope this helps .Best regards..."
},
{
"id": 65912,
"tgt": "What is the remedy for the lumps around the lip and nose in a 5 year old?",
"src": "Patient: I have a 5 year old son who has little white bumps around his top lip and near his nose. He also has a few lie bumps on his tongue. They look pimplish in nature, not blistery. He has also complained of a little bit of a sore throat? What could cause these symptoms? Doctor: Hi! Good evening. I am Dr Shareef answering your query. Even though it needs to be examined clinically for a diagnosis, from your history of sore throat and bumps around the lip and near his nose points to a viral infection like herpes. This would go off of its own in about a week's time. If I were your doctor, I would advise you for a multivitamin and some anti inflammatory drug for a symptomatic relief and if need be, an anti viral cream.I hope this information would help you in discussing with your family physician/treating doctor in further management of your problem. Please do not hesitate to ask in case of any further doubts.Thanks for choosing health care magic to clear doubts on your health problems. I wish you an early recovery. Dr Shareef."
},
{
"id": 6000,
"tgt": "Dermoid cyst removed. Left a strip of ovary on both sides. Trying to conceive. Which is the best time and still can I be pregnant?",
"src": "Patient: hi im debbie just got my dermoid cyst removed dec 13 of 2011. They left a almost stip of ovary on my right and part of left ovary. My husund and i have been trying for the last 2 months now and still no baby. when is the best time? can i still get pregnant? prior to that i got pregnant but become blighted and thats how i discoverd my cyst.please help Doctor: Hello. Thanks for writing to us. Since a part of ovarian tissue is left on both sides, chances of conception are present. You need to try during the fertile days- day 9 to day 16 in a regular 28 day cycle. You can also use LH surge kits to detect the time of ovulation with accuracy. I hope this information has been both informative and helpful for you. Regards, Dr. Rakhi Tayal drrakhitayal@gmail.com"
},
{
"id": 96257,
"tgt": "Cannot eat properly",
"src": "Patient: sometimes when i eat it feels liked it gets blocked half way down and it hurt pain under breastbone comes out my back try to drink water to push food down and that hurts stays for couple days then goes Doctor: Please be elaborative on some issues....habits, eating regimes, working style, previous medical history. We can help you once we know all these details vdchintamani@gmail.com"
},
{
"id": 62994,
"tgt": "What causes lump even after taking medication?",
"src": "Patient: I had celluitis on my jaw neck area a week ago it was swollen red and a big bump ER took a cat scan and said there was no puss or blood 3 days later it started draining but lump is still there im taking antibiotics theres no pain now the hole closed and i still have this lump what could be going on Doctor: hi.it is best if you consult with a doctor, preferably a general surgeon, for clinical evaluation and checkup.. it is possibly resolving and what you are feeling as a lump could be the healing/fibrotic tissue on the inside. complete your antibiotic regimen even if you're afebrile and not feeling the pain anymore. maintain a good hygiene and keep the area clean and dry as well.hope this helps.good day!!~dr.kaye"
},
{
"id": 140688,
"tgt": "Noticed a dip at back of head which is larger than the front",
"src": "Patient: My 7 year old son has a dip about halfway back on the top of his head and I am wondering if this is normal. The part of his head behind the dip also seems to be larger than the part in front of it. The back of his head does not appear to be uniform either. Doctor: Hello, If there are no symptoms, there is no need to worry, it could be normal. If any a headache is present, get a CT Brain done. Hope I have answered your query. Let me know if I can assist you further. Take care Regards, Dr Sudhir Kumar, Neurologist"
},
{
"id": 109576,
"tgt": "What causes excruciating lower back pain?",
"src": "Patient: I Have been having lower back pain on the right side. when I take a deep breath or cough it is very excruciating, The pain started today at around 10:30. It will not go away if I sit or lay down. Like three months ago my OBGYN doctor is testing me for endometriosis and PCOS. Doctor: HiWelcome to healhcaremagicAfter going through your query I came to know that you are probably suffering from lumbar disc prolapse. It can be confirmed by MRI scan. There are two treatment options.First option is rest in position of relief(generally lateral position with both knee bend),neurotopics such as mecobalamin and analgesics (Diclofenac three times a day after meals is effective). Sometimes strong analgesic(such as tramadol three times a day after meals) is required. Omeperazole before meals prevent acidity caused by analgesics .Second option is go for surgical operation(Discectomy).You can discuss your doctor about it. Hope your query get answered. If you have any clarification then please don't hesitate to write to us. I will be happy to answer your queries. You can also write to me directly on below link:https://www.bit.ly/askdrsudhirorthoWishing you good health. Take care."
},
{
"id": 181246,
"tgt": "What causes a white bump at the surgical site after tooth extraction?",
"src": "Patient: Hi, I had a upper tooth extracted 24 hours back. I am currently taking antibiotics. I just noticed there is some white stuff on the surgical site.Is it pus or infected? If yes then is it normal to have pus after 24 hours of surgery. FYI there is no stitching after surgery. Doctor: Hi,I have gone through your query and can understand your concern. As per your complain white stuff at the site of tooth extraction, of it is present inside the extraction socket then it does not seem to be healing tissue and not pus. In case if the fluid is draining from extraction socket then it can be pus.I would suggest you to consult an oral physician and get evaluated and a thorough clinical evaluation has to be done. In case if it is healing tissue then nothing has to be done as it will heal progressively. In case of pus antibiotics will help in its resolution.Hope this helps. Let me know if I can assist you further.Regards,Dr. Honey Arora"
},
{
"id": 23420,
"tgt": "What could be cause for death in sleep?",
"src": "Patient: Yes. My brother just died in his sleep. He was only 56 years old. He was a smoker and had high blood pressure, however, he had controlled his blood pressure with medication. I was told that he did not complain of any problems and he was very happy the night before he died. However, I believe that he also had sleep apnea because he was a very loud snorer, according to my sister in law. She made him sleep on his side. An autopsy has been performed and we won't find out for another 6 weeks. What are some factors that could cause someone to die in their sleep like that? There was no prior history of heart problems, no heart attacks, etc. Just the high blood pressure and smoking. He was not overweight, and did get excercise in his job, since he worked for shipping and receiving. Doctor: Hello,Thank you for using HCM to post your health query. I am really sorry about your brother.The most common cause of sudden death in patients his age is a myocardial infarction. From what I can see from your question, your brother had some risk factors for coronary artery disease (heart artery blockage). He was a male, had hypertension, he smoked smoking and probably had sleep apnea. A good guess could be that he had a myocardial infarction. Other causes of sudden death are arrhythmias, pulmonary embolism and stroke.The autopsy will give you a more accurate answer.Best regards,Dr. Fernando Batiz"
},
{
"id": 184792,
"tgt": "What causes pain and swelling in gums after a cavity fill?",
"src": "Patient: My friend constantly complains to tooth pain. He had cavity and had filled it recently. Now he is complaining of swollen gums though it doesn't pain and had weakness and nausea and felt feverish and weak. He smokes a lot and is in mid 20s. What could be the reason? I am bit worried for him. Doctor: Hello!Thank you for posting here.Please quit tobacco immediately.Swollen gums can be due to poor oral hygiene.Tobacco not only makes stains on tooth but also causes bad breath.He must get a sitting of professional scaling done.There is no problem with the filling as he has no pain in the tooth.Generalised gum health is poor.Use gum tone gel on the gums and a mild mouth wash thrice a week.Increase fiber food in diet. meal and snack.Regards.Have sugars as part of diet.Gargle after every"
},
{
"id": 134375,
"tgt": "Suggest remedy for discomfort in sternum after a hysterectomy",
"src": "Patient: i had a hysterectomy 8 weeks ago and ever since i have almost constant discomfort in my sternum which i believe is heartburn but i don t understand why it has started since the surgery. how do i get rid of it, its very uncomfortable. Antiacids don t seem to help. Doctor: hi,thank you for providing a brief history of your case. well, to look ahead of your discomfort in the sternal region it can be due to weakness of the respiratory muscles.try doing some breathing exercises and abdominal strengthening exercises. Abdominal muscles are the muscles for respiratory system and the strength in them are of due importance for the ease of breathing. Pilates is the form of exercises you can perform.suppose in case after doing what I told doesn't help then I Advice you to meet a physician and get yourself checked once.its never a bad idea to take a consultation from a specialist. a crst x-ray will be of input in future as well."
},
{
"id": 107383,
"tgt": "What causes mid-back pain radiating towards the ribs while having spinal stenosis?",
"src": "Patient: I have pain to the right of my mid back which feels like someone punched me in the back. It is worsened with standing or sitting upright and improves if I slouch in a chair. The pain is now radiating to my right ribs. I have had this for 6 months and the radiation is new. I have severe degenerative spine disease with spinal stenosis in my lumbar and cervical spine. Any suggestions? Doctor: Hi,I am Dr Gopal Goel Orthopedic Surgeon. Your pain seems as if one of the nerves in dorsal spine is being pressed. I suggest MRI of Dorsal spine , Pain killers , Pregabalin . If pain does not improve in few days time , I suggest you consult a pain specialist who may use nerve block to relieve pain."
},
{
"id": 197602,
"tgt": "How to treat penile rashes?",
"src": "Patient: hi I am male 26....I masturbate almost every day but 3-4 days back I did it back to back twice and got little rashes.yesterday u have a look inside my meatus it got few purple spots.please tell what it is and can I apply moisturiser to get rid of it......do I need to worry. Doctor: HelloThanks for the query.Apply a mild steroid like cortisone for 5 days. Let me know if you have any other doubts. Thank you"
},
{
"id": 216020,
"tgt": "Suggest treatment for persistent knee pain",
"src": "Patient: I suffer from knee pain which has expanded to include my thigh, hip and lower back. Had arthoscopic surgery on the knee end of year. Has not improved. Had a laminectomy 4 years ago. Pain gets worse when standing or walking, made worse when weather is damp. Doctor: Hello and Welcome to \u2018Ask A Doctor\u2019 service. I have reviewed your query and here is my advice. You can try analgesics like Tramadol or Gabapentin which shows good results in chronic pain conditions. Also get an MRI scan done to check whether a revision surgery needed or not. We have to rule out conditions like sciatica as the pain radiates to thigh region. Better to consult a physician for expert opinion. Wishing you good health. Thanks"
},
{
"id": 14517,
"tgt": "Suggest treatment for rashes on chest",
"src": "Patient: Hi, I have a dark red rash that just showed up on my chest and stomach. It is Hot, and it itches a little. I recently visited a(n) Hot Spring Resort (within the last 3 days). This is the only thing out of the ordinary. I have been living in China for 1.5 years. Doctor: Hello,Thank you for posting on HCM.It seems you are suffering from Tinea corporis, a kind of fungal infection.I would suggest you to consult your dermatologist for proper management of the condition.I usually recommend proper course of oral anti-fungal drugs like Fluconazole or Itraconazole along with anti-fungal cream like luliconazole for local application at bedtime. You can additionally use antifungal dusting powder containing sertaconazole during day time and a soap containing ketoconazole for rinsing of affected areas. Take oral antihistaminics for itching as required.Maintain hygiene over those areas and avoid wearing tight undergarments.Hope your queries are resolved and wish you best of health.Kindly spare some time to rate my answer and drop your valuable review at the following link:https://urldefense.com/v3/__http://doctor.healthcaremagic.com/doctors/dr-hardik-pitroda/67169If__;!!Mih3wA!SBzm6_kI6hCZ58EPH6N_05MFfiPbxWXT0a2TJCdFQObRWm5mV5ur7hW9rW0j8A$ you require any further assistance in future, you can reach me directly through the above mentioned link.Thank youDr Hardik Pitroda"
},
{
"id": 83135,
"tgt": "28 years old female suffering from autoimmune disorders",
"src": "Patient: I am a 28 year old female, who has experienced several symptoms of an autoimmune disorder for about 7-10 years with no treatment. I am going to see a Rheumatologist , but am curious if anyone knows about a Positive Sjogren s SS-B with a negative Sjogrens SS-A? I am curious what this means. My RA factor is 7 on a scale of 0-13.9. My TSH is between 1-2, which I read is good. I am ANA and ANA w/ reflex positive. Finally, in addition to negative SS-A and positive SS-B, my ENA panel shows below average slightly for ANTI-DNA, RNP Abs, and Smith ABs. With not seeing a Rheumatologist, my primary care physicians says I have a 10% chance of having SLE and a 30% chance of Sjogren s Syndrome. I have multiple symptoms of both conditions, however, no dry mouth . My symptoms include a long list, but it appears I have the following symptoms: Sinus Issues Reynauds Some joint discomfort in ankles, fingers, and knees My tonsils are slightly disfigured with particles that show up in them A history of weak and cavity ridden teeth Memory loss and confusion Dizziness Infrequent migraines w/ aura Frequent urination Fitigue Anxiety and Depression (Random) Rough/course and dry hair Acne Blurred Vision Trouble sleeping due to back pain Green mucus etc. No rashes or dry mouth Any help would be greatly appreciated in calming my anxiety over the blood test results. Thank you, Jade Doctor: Dear Jade welcome to healthcaremagic Ayurveda has better solution for autoimmune disorder. There is good herbal solution for building immunity in the body and protecting from the diseases GUDUCHI( Tinosporia cordifolia) is best herbs for immunity and its preparation are widely used.. Please consult your nearest ayurvedic doctor for details or write me info@rammani.com.np Thanks"
},
{
"id": 5351,
"tgt": "Headache, stomach ache, burning sensation, nausea, dizzy, lightheaded. On phenteramine. Get pregnant?",
"src": "Patient: im really stressed out, but i have had a continous headache since tuesday, my stomach ache and burns a little and its in the middle between my ribs, and under my brests~~ i get nausea later day to night time. I get dizzy and light headed, i am on phenteramine but iv been on it since feb and this is the first im experiencing it. could i be pregnant? Doctor: Hello. Thanks for writing to us. The symptoms that you are having could be related to the side effects of phentermine you are taking. Pregnancy is not likely but needs to be ruled out after a urine test of your periods are overdue.I hope this information has been both informative and helpful for you. Regards, Dr. Rakhi Tayal drrakhitayal@gmail.com"
},
{
"id": 182033,
"tgt": "What causes bad taste in mouth along with bad breath?",
"src": "Patient: Hi, I have a bad taste in my mouth most of the time & know that my breath smells too. My oral hygiene is really good- the dentist tells me so. He can't find any problem so I am in despair. I go through lots of chewing gum & quite often when I'm. chewing it I can taste a strange oniony sort if taste. Please help Doctor: HALITOSIS or bad breath can be also due to GI disturbances. If you are on medications, it could be side effect. If your dental hygene is good, it could be probably attributed to the above."
},
{
"id": 220963,
"tgt": "Does having previous abortion affect future pregnancies?",
"src": "Patient: please help me... i had contact with a guy ( 2 yrs back i abort also)... for last 3 years... now he s no more. now my periods are regular there s no issue s in that.if i married a new guy is he doubt ful me or any problem wil come ? if i pregnant any prob will come...? please guide me Doctor: HiDr. Purushottam welcomes you to HCM virtual clinic.I have gone through your query. I think I have understood your concern, I will try to suggest you the best possible treatment options.Please do not worry.As such if you have done abortions to terminate unwanted pregnancy, then it has no effect on future pregnancy.I will suggest to give proper history to your doctor at that point of time.During that pregnancy , please keep watch on length of the cervix , while doing USG for pregnancy. Sometimes with surgical or medical termination can impart weakness to cervix and you need to put stitch on the cervix.Otherwise you need not worry. Previous abortions (MTP) have no effects on the baby's growth.I hope my answer helps you.Thanks.Wish you good health.With regardsDr Purushottam"
},
{
"id": 110432,
"tgt": "What could the dull ache in back be?",
"src": "Patient: Hello, I am concerned about an dull aching right side, that moves more towards my back, but not low back. Will sometimes move from right hip inward. This does leave for a few days but returns. I have had an ultra sound and cat scan done, with results showing nothing of concern. Have wondered if it could be the start of an appendix or ovarian issue, but the symptoms I have leave and come back. Would appreciate some educated info as what I should do. Thank you Doctor: Hi, thanks for posting your concern in the HCM.Dull aching pain in the back may be due to several reasons, which need to be excluded by further evaluation.This may be due to1. Mechanical back pain. This may be due to faulty posture or injury to paraspinal muscles due to any reason. The former cause seems to be more likely because the pain is on and off.2. Any liver or pancreatic pathology. This seems to be less likely, taking into consideration that your imaging reports came out to be normal. Furthermore, they more commonly cause pain over both abdomen and back.3. Functional bowel disease. If you have irregular bowel habit and / or bloating, the pain may arise from the same. This is more common with constipation predominantly irritable bowel syndrome.4. Renal or ureteric pathology , stone or urinary tract infection may be associated with this. Usually, the pain is associated with dysuria, fever with chills etc. in these cases. However, renal pain may present with this alone. I think your imaging should have ruled out this also.5. Ovarian pathology e.g SOL, infected cyst or torsion may produce back pain. Even genital infection or PID may present with this.Appendicitis usually don't present with back pain as a sole complaint.I think except for mechanical back pain and IBS or FBD all others can be ruled out by imaging (usg or CAT scan).Therefore, you should consult your local physician for further evaluation.For the time being1. Maintain good oral hydration2. Avoid refined carbohydrate, spices and carbonated beverages. Have high fiber diet at frequent interval in small amount.3. Perform abdominal muscle strengthening exercise.For any further questions, please write back to me.Regards,Dr. Kaushik"
},
{
"id": 118461,
"tgt": "How can blood clot under eye after injury be removed?",
"src": "Patient: Hello Dr! I had a black eye injury over 10 years ago, there had been no fracture, no problems with vision but I still have slight swelling underneath my eye ever since, when its aggravated the swelling appears. I think the blood has clotted. Please tell me how I can remove the blood. I am desperate. Thank you Doctor: Hello and thank you for submitting your question.When there is trauma to an area, such as what you have described, part of the body's natural healing process can involve an increase in fibrous tissue in the area of injury. Because the swelling is still appreciable nearly a decade later and is causing you a degree of discomfort, I suggest that you visit your medical doctor for further examination, which may include imaging. Thanks again for sharing your health care concern with me.Sincerely,Dr. G"
},
{
"id": 226222,
"tgt": "Nausea with mild temperature after taking metformin. Side effect or drug interaction with nuva ring?",
"src": "Patient: I started taking Metformin 1000 mg last Sunday evening. I had no noticeable side effects. I replaced my nuva ring on Thursday and took the metformin as I had been after my evening meal. I started feeling very nauseated the next morning after breakfast and haven t been able to shake it since. I have had a slight temp at 99.5. Could this be an interaction between the two medications? If so, what should I do? Doctor: Hello thank you for your query. Normally, Metformin does have a side effect of nausea. However, there is no specific interaction with Nuvaring. Nuvaring itself is a hormonal substance, and can cause nausea. it could be the additive effect of both medications and not an interaction. You would do best to consult your health care provider, since nausea alongwith a mild temperature could herald the beginning of an infection. Take care."
},
{
"id": 225589,
"tgt": "On birth control pills. Continuous, heavy brown discharge after taking the pill late. Due for periods in a week. Normal?",
"src": "Patient: So I've been on the birth control pill (Ortho Tri Cyclen) for about a month now, and about 5 days ago I accidentally took my pill 4 hours late.That same day, I started getting brown discharge and i figured it was because of the late pill.Well I'm supposed to start taking my placebo pills and start my period in 5 days, but the brown discharge is still continuous and very heavy.Is the normal? Am I pregnant?I would \"call my doctor\" but i don't know how.Thanks in advance! Doctor: HelloThanks for your query.Taking just one pill 4 hours late would not affect the efficacy of the birth control pill.It is alright to spot when on the birth control pill.If this recurs every month, you are experiencing breakthrough bleeding.This occurs because the dose of hormones in the pill is not adequate for you, and you must be switched over to another brand with more hormone dose.ALso, during the first pack of pill use, a backup method like condoms is always advised.Take a pregnancy test after a week just to be sure.All the best."
},
{
"id": 197583,
"tgt": "Suggest remedy for pain in groin",
"src": "Patient: Hie. My problem is I have sharp pains in the lower part of my right abdomen. My groin area just above the penis below my stomach sometimes hurts as well. on days that I masturbate maybe some 5 hrs later I have this sharp shooting pain through the penis on the inside, like on the inside. It persists for a few seconds during which I cant even walk and I gotta maintain that posture until it passes. I masturbate an average one time every day, I don t have hair thinning issues and concentration problems though I think these groin issues are masturbation related. also recently I have started to feel this burning sensation at the tipof my penis. what could be my issue and how may I remedy it? thank you Doctor: Higreetings Your pain is generalised to the right groin and radiating to scrotum.It is not continuous and having burning sensation at the tip.Now I would like to get the urine tested for any infection or any other abnormalities. prostatitis or urinary tract infections can present like this.Also I have to rule out any renal or ureteric stones and either ultrasound scan or a contrast CT may be required.If both are negative a chronic appendicitis has to be ruled out.hope my answer helps you. Regards"
},
{
"id": 32953,
"tgt": "What causes breathing difficulty during treatment for helicobacter pylori?",
"src": "Patient: Hi, I m 25 years old and I have a positive helikobacter test before three months.I had my first therapy before two months and after that I d made a second test and I was positive for second time.Now a m taking my second therapy, but i had serious problems with breathing.I need more air.Please help me Doctor: Hello,I would like to tell you that the treatment for helicobacter pylori never causes breathing problems usually. I would suggest you visit your doctor and get yourself examined for respiratory problems. It could be side effects of medicines but visit a doctor to rule this out. Kindly mention what drugs you are on? So that I may help you better.Hope I have answered your query. Let me know if I can assist you further.Regards,Dr. Sheetal Verma"
},
{
"id": 36229,
"tgt": "Suggest remedy for fungal infections",
"src": "Patient: Hi i got infected with fungal infection on my body big size rings shaped on my stomach, chest and back.and lot of marks if bacteria on my private parts also.presently i am using propizole nf and closip and candid . but it is not doing any great help . plz advice as this is troubling for last 6months Doctor: HiThank you for asking HCMI have gone through your query. Your problem is most likely due to fungal infection Tinea corporis. Clotrimazole wont be effective in all. So i usually advise miconazole ointment for local application and fluconazole tablet for oral intake in my patients. I also advise ketoconazole soap in those who are having extensive lesions. Hope this may help you.Let me know if anything not clear.Thanks."
},
{
"id": 163118,
"tgt": "What causes yellowish discoloration and bad odor in feet?",
"src": "Patient: My son is 2 years old.we were at the doctor last week and he had mild pneumonia. He has finished his antibiotics.He has been tested and his iron has been low, the last few months .Today I noticed his feet are slightly yellow and have a very strange smell to them. Doctor: Hello,Due to pneumonia, antibiotics, the iron level may decrease, and so you need to take iron tablets for your child. Also, I will suggest a healthy diet with soups, vegetables, fruits, and liquids to help him recover.About your other problem I can say that the slightly yellow may be due to iron decrease and anemia but for the strange smell, I will suggest taking in consideration a fungal infection.Of course is just an opportunity that you can take into consideration in this case. So, first do not let his feet wet, dry them well, use cotton socks and leather shoes as well.You can also try powder with antifungal which helps in this case with the problem and with the smell.Hope I have answered your query. Let me know if I can assist you further.Regards,Dr. Elona Dashi"
},
{
"id": 26283,
"tgt": "Suggest treatment for heart palpitations and shortness of breath",
"src": "Patient: hello sir, i just wanna ask why im suffering palpitatation and sometimes my breathing is short.. I also have head ache often..my doctor said its just an acid.. But i suffering this almost four months im really concern about this..please help me.. I also felt tired often..i just started to felt this since i was at the hospital i was abort for just two months..and my doctor raspa me also please help me.. Thank you so much.. I will wait for your response.. Doctor: hello,I have gone through your query.Thanks for using HCM.your symptoms may be due to anemia.As per your history of abortion at 2 months suggests that you might have anemia.You should check your hemoglobin.If your hemoglobin is normal then you should go for further check up.My best wishesDr.Rajesh Teli,MD."
},
{
"id": 220760,
"tgt": "How to reduce stretch marks during pregnancy?",
"src": "Patient: hiiiii doctor, my 7th month is about to start after 2 days,,,,, can u suggest me the ways thru which i can control my stretch marks at my breast and stomach . also please suggest me some exercises which i cn do during dis period nd wat diet shld i tak nw.... my weight has been raised to 9 kgs yet.....is it fine??? please help me out thnks shilpi Doctor: weight gain is fine. For stretch marks olive oil is good . various creams are available but best time is to start message with oil or cream is 3rd month onwards. you can still start using them but results will be less."
},
{
"id": 113322,
"tgt": "Suffer middle back pain, x-ray shows minor marginal osteophytes and degenerative changes. What is the cause ?",
"src": "Patient: i have had middle back pain for a few years and over the last few months have had 2 episodes lasting about 9 to 14 days in agony, having to lay down for 95 per cent of the day, thats with pain releif and heat packs. i have had an xray which showed minor marginal osteophytes ( im not sure if thats the correct spelling), and degenerative changes. ive been seeing a doctor who says nothing is wrong, so i pushed to get a ct scan of my spine which i have right here with me on disc, i have a specialist appointmet on the 6th march, a long time away to be in agony. im 34 female 76 kgs, had a head on car accident in 07 with no injurys except sore for a few weeks from impact i guess, my car was going 100 km an hour and it totally wrote my car off. im not even sure how i came out of it with not even one scratch . i would just like to know what is wrong and what is causing my pain because ive just had enough, and i have a 3 year old and 13 year old i need to take care of. please help if you can. i cannot afford any payment and i understand if this stops me from getting help. thanks for your time. AAAAA Doctor: The changes to your vertebrae are due to wear and may be the result of your accident. This causes impingement of the nerve-roots as they leave your spine through the space between the vertebrae , this in turn can cause muscle spasm, which is a spinal cord reflex. It is often best treated conservatively, with pain management , postural care and correct exercise."
},
{
"id": 31477,
"tgt": "What are the chances of getting rabies through a stray dog bite?",
"src": "Patient: Hi,My husband's aunt was bitten by a stray dog yesterday on her hand. She has three puncture wounds on her hand (2 on the top and one near her thumb. The top of her hand was very swollen shortly after the bite happened. She went to her doctor and they gave her antibiotics and told her to contact the health department. They didn't even clean it.I am now concerned about the risk of rabies since the dog is a stray. Do you have any advice I can pass along? Doctor: Hi thanks for asking question.Yes through dog bite rabies can surely transmitted.Even your husband aunt has not clean the wound.Whenever there is dog bite bite area wash throughly with soap and water.She has to take anti rabies vaccine because according to history chances of infection is high in your aunt case.I hope i have solved your query.thanks."
},
{
"id": 89840,
"tgt": "Could the tumors in the abdomen be neurofibroma?",
"src": "Patient: hi sir i have more then 20 tumor in stomach and back side arm back of seat ..i discussed doctor yesterday he said this in nothing the name is neurofibroma ..this is look like a mole dont worry ..but i have feel uneasy please advice this is going any problem in my further life Doctor: Hi Thanks for posting your query. I think you have used tumor for describing bumps or swelling. Since how long you are having these ? Are they over your skin or inside abdomen? To confirm what it is you need to get a biopsy of these bumps and send for histopathological analysis. Revert back to me with the report."
},
{
"id": 35717,
"tgt": "How to treat cellulitis infection and high ggt?",
"src": "Patient: respected doctors,my age is 36 i have celulities infection in my left foot from 6 years for a couple of timenow i got my blood tests and the tests show that my ggt is high (65) i also have history of gestional diabetes nine years beforeplease can you kindly tell me any curation of my diseases Doctor: Hello dear,Thank you for your contact to health care magic.I read and understand your concern. I am Dr Arun Tank answering your concern.Cellulitis of the foot requires a antimicrobial therapy.I suggest you to take a cefixime 200 mg tablet for foot infection three times a day under your doctors guidance.You can also take paracetamol and diclofenac tablet under your doctors guidance for pain and fever.Please maintain good hygiene by frequent dressing and cleaning. This will equally cures you rapidly as do antibiotics do.Raised GGT indicates some liver pathology. I suggest you to get your all liver function test done. If you have this high GGT is because of hypertension than treating the hypertension will treat you with high GGT.Please control your sugar if your sugar is high than controlling the sugar also cures your infection.I will be happy to answer your further concern on bit.ly/DrArun.Thank you,Dr Arun TankInfectious diseases specialist,HCM."
},
{
"id": 46377,
"tgt": "How to treat calculus on upper region of left kidney?",
"src": "Patient: i had a kidney problem since 2005 usg reports shows a calculus on the upper region of left kindney ...... on 4 th.May 2011 when problem started doctor admited me to hospital and it was deducted that tuberclusis and i am now taking akt4. no claculus was found.today is the 13th day of taking akt4 . Now the problem is feeling very week from 3/4 days. blood pressure is 120/80. pules 70. taking thironums 75 Doctor: Hello and welcome to HCM.As an Urologist, i can understand your anxiety.You should write your age,sex and reports of the diagnosis.If the kidney stone is more than 6mm,take medicine for dissolution of stone.How was a diagnosis of renal t.b. made. Did you get urine-AFB c/s positive.If the stone is much larger and affecting the kidney,go for an ESWL or PCNL.Drink about 3 liters of fluids daily. Avoid dairy products in diet.If you've any doubts,send it to me,as a direct question with all reports.Dr.Matthew J. Mangat."
},
{
"id": 172748,
"tgt": "How to treat ear pain,cold in a 3 years old child?",
"src": "Patient: my 3 years daughter got ear pain cold n cough but no fever few days back..i gave her zifi 100 by pharmacist mistakes instead on cefoprox 100 prescribed for 7 days..today dr. diagnosed n prescribed cefoprox 100 for 7 more days.my daughter looks ok otherwise no fever n throat infection.shall i give or not? Doctor: Hi dear,I understand your concern, you can use Cyprofloxacin ear drops 4 drops 3 times in ear during 5 days.Drip Otrivin 2 drops 2 times to open Eustachian tube and improve ventilation of middle ear.You can treat without antibiotics. Give Kuka or Ambrodil syrup 1/2 teaspoon twice a day.Hope I answered your question. If you have more questions, then don't hesitate to write us,I am always happy to help.Wishing your daughter good health"
},
{
"id": 38398,
"tgt": "What does salmonella typhi O:1 in 40 dilution in widal test indicate?",
"src": "Patient: Hi I have under gone to widal test, and below are the test results: Salmonella typhi O: 1 in 40 dilution Salmonella typhi H: 1 in 80 dilution Salmonella para typhi AH: 1 in 40 dilution Salmonella para typhi BH: 1 in 20 dilution Is any one there to help on explaining me about the test results Balaji Doctor: Hello thank you for your contact to healthcare magic. If I am your treating doctor I would like to tell you that widal titre of 1:160 or above is considered positive. You have 1:80 titre, which I consider it to be negative. But ideal procedure is that you have to count the community titre and accordingly the cutoff is to be decided. If you have any thing else to know kindly contact me. Dr.Arun Tank and I am infectious disease specialist. Thank you"
},
{
"id": 139727,
"tgt": "What could cause problem in waking up?",
"src": "Patient: My grandma was fine this morning. She suffered a mild stroke about a year ago. She has been on bed rest and went through rehab. Like I said my grandma was fine this morning, but I went in her room and she wont wake up. She is breathing actually snoring. What do I do? Doctor: HiThanks for being on healthcaremagic.comAs it appears to be acute onset impairment in consciousness, you should take her immediately to a nearby hospital. She might need some blood tests to rule out any abnormality in electrolytes and blood sugar. Also, she might have to undergo a brain scan to rule out and brain lesion.Regards,Dr.Ajay Panwar,MD,DM(Neurology)"
},
{
"id": 124060,
"tgt": "What causes a lump and pain while lifting an arm?",
"src": "Patient: Hi, yesterday after playing 3 hours of basketball, I noticed a pain in my armpit. Everytime I try to lift my arm it starts to hurts, I also noticed that I have a lump in the area that hurts. Please tell me what s wrong and what I have to do to stop the pain. Doctor: Hi, This might be a case of frozen shoulder, tendon injury /inflammation, shoulder dislocation. You can get cold compression to reduce the pain and swelling and get an X-ray shoulder joint done for confirmation. Hope I have answered your query. Let me know if I can assist you further. Regards, Dr. Tanmoy Roy, General & Family Physician"
},
{
"id": 223781,
"tgt": "Can unwanted 72 cause delay in period?",
"src": "Patient: Me and my boyfriend had sex on April 21st. We used protection. The next day I ate unwanted 72. After 5 days I had my periods. The cramping was dreadful and it lasted for a longer time like 7 days. But this month there is no sign of my periods. I don t know what s wrong. I feel light pain in my lower stomach but it comes and goes. My last period was on April 28th. I am worried. Can it be the side effect of the pill? Or is it pregnancy? Help! Doctor: HelloHealth care magic user I understood your concern and My opinion is that yes unwanted 72 can cause delay in your period, pain and excessive bleeding but you should do first urine pregnancy test with velocit kit to rule out pregnacy first if it comes negative than please wait for few days your period will come If it does not come by than you require medicine to get periods I hope my answers help you Thanks"
},
{
"id": 55512,
"tgt": "Why does liver enlarge?",
"src": "Patient: My liver is minimally enlarged showing diffuse increased parenchymal echotexture. no focal sol is seen. Intra hepatic biliary radicals are not dilated. Portal vein measures 10mm in diameter at porta. in a USG of upper abdomain. My SGPT 98 U/L. SGOT 74 U/L. Bilirubin total 0.7nmg/dl. Lipid profile is normal. Doctor: HiDont worry.Increased echogenisity of liver means its fatty liver.it means accumulation of fat in hepatocytes.It causes increse in size of liver.its due to so many factors.increased intake of carbohydrate diet , diabetes , alcohol , overweight. Fatty accumulation in liver causes deranged liver function and thus abnormal liver function tests. Try to avoid risky life style.fatty liver is reversible.LFT comes to normal.Thank u With best regards"
},
{
"id": 88074,
"tgt": "Suggest treatment for pain in lower abdominal area",
"src": "Patient: I'm having a lot of pain near the lower abdominal area.There is more gas then normal, alot of cramping, and sharp pains above the genital area.When I perform a bowel movement, sometimes its relieving, but at the same time it's painful. It's painful to urinate too, as if I have a kidney stone.I'm beginning to catch some sort of fever/cold as well. I don't know how or what I should do, or what it could possibly be.. all I know is that I'm in an increasing amount of pain each day.Please, let me know of any thing you might think it is Doctor: Hi,From history it seems that you might be having urinary tract infection or kidney stone as history suggests.Go for routine and microscopic for pus cells and blood cells.Might require ultra sound abdomen.After reports go for treatment accordingly.Take plenty of water.Ok and take care."
},
{
"id": 29683,
"tgt": "Suggest diet to improve immunity after chickenpox infection",
"src": "Patient: About diet after Chickenpox and scars after chickenpoxIam 34 yrs aged and on march 4th i got infected by chickenpox .its very severe after 5days i consulted doctor and he prescribed Acyclovir 800 ,p750,betonin syrup ,vegam 500(some thing like that), it get cured on 20th March.after that iam eating normally i have eaten Bananas ,eggs,fish(Non veg and veg) items in april. I keep on hearing many people saying that diet should be maintained post chicken pox .They say like don't eat Egg, chicken,Banana,Fish and many more items now by eating all these in future you will suffer with many health disorders like body pains leg bone pains,sugar,Bp etc.,.So please let me know what to eat and what not. And what care i have to take to cure my scars on my face like holes .at starting i didn't recognise it as chickenpox and itched the pimples so i got holes on my face. So please tell me .thank you sir. Doctor: Hi, Thanks for posting in HCM. I understand your concern. What medications have been prescribed would help you to recover from infection. Infection itself would confer natural immunity. However, to recover from the illness and to gain good immunity, you need to take protein rich diet. Kindly take protein supplement powder like Proteinex twice daily with milk. Include in your diet pulses and legumes. Eating groundnuts, almonds and other dry fruits will provide good amount of proteins. Also, eating fruits like apple, banana and guava would help you to provide essential vitamins and minerals. With intake of these foods, you can gain good immunity. Hope the information provided would be helpful. All the best. Regards, Dr.Ashakiran.S. MBBS., MD."
},
{
"id": 68312,
"tgt": "What causes lumps on the back side of the head?",
"src": "Patient: hi i am 23 i ve had a lot of health problems over the last 3 an a half years and now i have to lumps on the back of my head leading in to the middle and one at the front which is not so big. One is very tender to touch and the other is not so bad. I have been having very bad head aches over the last 2 days with no sleep last night and feeling very dizzie. Doctor: Welcome to health care magic. 1.The symptoms suggest the features of infective aetiology with collection of pus with in, possibly hair follicular infection.2.Other possible cause could be lipoma and lymph nodal enlargement.3.I would recommend an ultrasound - ultrasound will help to evaluate the nature of the lesion, its source and extensions.4.In case of infective cause - a course of antibiotic treatment will be enough, if does not respond a small incision and drainage followed by antibiotics.5.maintain local hygiene, do not scratch, itch the area - as you will delay healing, apply some antiseptic liquids for aseptic conditions to heal better. Good luck.Hope i have answered your query.Any thing to ask do not hesitate. Thank you."
},
{
"id": 224848,
"tgt": "Is emergency pill effective to avoid unexpected pregnancy?",
"src": "Patient: The condom tore during the intercourse and he ejaculated inside me. I took the emergency pill after about an hour but I am still scared whether or not it will work. should I take any precautions regarding food or other drugs? how will it affect my monthly cycles? Doctor: Hi. emergency contraceptive pill is effective and you have taken it well in time. I think it will surely work. Take precautions for the rest of the month.You may get withdrawl bleeding after a few days due to pill and your cycle date may change this month rest nothing else. Thanks."
},
{
"id": 212303,
"tgt": "Suicidical tendencies. History of rape, kidnap and mental abuse. Help",
"src": "Patient: Hello im deployed over seas my gf has told me recently about wanting to kill herself off and on sense age of 7. Im not there to help her when shes sad or physically be there. She wont seek help and i dont know how to get her to go cause she doesnt have real friends or people i can get to take her. 11 months left here is there anything i can do to help her till i get back to get her medical help? Shes been in bad tramas mental abuse at home from her ex and kidnapped and raped. How can i help from this far away. I cant lose her. Doctor: Hi there ~ The only way I can think of that you can help is to alert social services over there by writing a very lengthy letter explaining what happened to your girlfriend. I am sure there would be suggestions that come from the other end and you could work on them. If this is urgent please call 911 and have the issue reported. I hope this helps. Take care and have a lovely day!"
},
{
"id": 152100,
"tgt": "What does brain back ground activity mean ?",
"src": "Patient: hi what is the mean of Brain back ground activity :alpha (8-14c/sec) activation :hyper ventilation abnormal waves :normal Doctor: hi there, please write your question in good way so that we can understand your problem and clarify it. thanks"
},
{
"id": 43093,
"tgt": "Suggest treatment for infertility with PCOD. EB test shows TB cells taking medications since 6 months",
"src": "Patient: My name is sania ,im hav completed my 3 yrs of my married life, i got pcod, my tretment is going at mumtha infertility centre frm last 8 mnths,bt no result.im very much eagar to conceive.Last mnth they hav done my EB Test which i came to know abt TB cells for which im taking medicine 4 that,its a 6 month. plz advise me. Doctor: Hi Sania. The course of anti-Tb medicines are almost finishing. You may go for further tests like Hysterosalpingography to know the status of the Fallopian tubes, if they are open or blocked. PCOD may cause a problem if there is no ovulation. Stop junk foods and have a healthy life-style to improve on PCOD along with the medicines your Gynecologist gives you. Also go for ovulation studies with ultrasonography. If the above tests along with the semen analysis of your husband are normal the chances of conceiving are more ."
},
{
"id": 186848,
"tgt": "Why am I having headache after removal of wisdom tooth?",
"src": "Patient: I got 3 of my wisdom teeth extracted yesterday. Today I feel very, very lightheaded. Also my I have a weird headache. Like lots of pressure in my head. Mostly across my forehead. And I keep getting blurred spots in my vision. Is this anything to be worried about? Doctor: Hello, Welcome Thanks for consulting HCM, I have gone through your query, as you have headache after extraction of wisdom tooth dont worry it can be due to weakness you are feeling pain it can be due to Malnutrition or stress , or due to effect of anaesthesia also. You take proper sleep of 8 hours atleast . Take proper antibiotic course prescribed by your dentist. Do warm saline gargle two - three times a day Consult your dentist for Vitamin supplement .Hope this will help you."
},
{
"id": 135686,
"tgt": "Suggest remedy for shooting back pain radiating towards knee",
"src": "Patient: I have a shooting back pain at the belt line above my right hip. At times it shots down to my knee & for a few minutes one day made my right foot numb. When I awoke this AM it was fine, but it seems the more gravity is in charge the more it hurts. I am a 65 year old male. I am not on any prescription meds and otherwise healthy Doctor: HiThank you for asking HCMI have gone through your query. Shooting back pain radiating towards knee or leg can be most likely due to Sciatica or lumbar radiculopathy. This usually occurs due to nerve compression from lumbar intervertebral disc bulge. A physical examination by orthopedician will be helpful to rule out this. X ray followed by MRI of lumbar spine will be required to confirm it. If you were my patient i would recommend NSAIDs like Aceclofenac , muscle relaxants like Tizanidine and neurotropics like Pregabalin and methylcobalamin. I also advise to take physiotherapy like TENS after consulting a physiotherapist.Back muscle strengthening exercises and wearing lumbar supporting belt will be helpful for this also. Avoid lifting heavy objects and bending.Hope this may help you. Let me know if anything not clear.Thanks."
},
{
"id": 207996,
"tgt": "What causes tenderness in one side of the body?",
"src": "Patient: Hi, may I answer your health queries right now ? Please type your query here... It seems to me that the left side of my body is more sensitive than my right. It also seems skinnier. The bones on my foot are not covered by as much soft tissue. My left shins and joints ache after working out. When I m sick, my left armpit is sensitive, and my left breast aches before my period. Also, if I have a tooth ache, it is on my left side. Am I imagining this? Doctor: Hello,Welcome to Healthcare Magic.First, you should get assessed and investigated by general physician to confirm above self observation of yourself. If general physician finds some problem then got treated for it.If there is no problem (or normal examination), then you can meet nearby psychiatrist, as such symptoms can happen in psychological conditions like anxiety and somatization.Wishing you good health and all the best.www.99doctor.com"
},
{
"id": 63830,
"tgt": "What causes small lump in salivary gland?",
"src": "Patient: I have a small stone in the salivary gland under my tongue. It's been there since I had surgery 18 months ago and had a very dry mouth while having an antibiotic drip - for appendicitis and partial bowel recession. The hard lump is hardly noticible now but has become more like a soft sac of fluid the last few days. It fills up and looks like it's got blood in it as it's purple/black in colour. It's not there all the time but is up and down during the course of the day. My GP suggested I wait and see if there's changes when I saw him 18 months complaining about the lump and sore gland under my jaw. I've had uterine cancer 15 yrs ago and have also had most of my thyroid removed due to a cyst and abnormal cells. Doctor: Hi..Welcome to HEALTHCARE MAGIC..The soft sac containing fluid is a MUCOCELE which is formed either due to trauma to any minor salivary gland or its ducts or any traumatic injury to the duct of major salivary gland and as it is under tongue it can be due to either submandibular or Submental salivary glands which are major salivary glands..You can also have a Sialolith or salivary gland stone and it can cause swelling under jawline and floor of mouth but a blue black swelling is a MUCOCELE most probably..You should consult a dentist and get evaluated..1. For Mucocele mostly treatment is not required but if it doesn't resolve surgical removal has to be done..2. For salivary gland stone take more of citrus foods and candies that simulate saliva flow and small stones can be flushed off with saliva flow..Large stones need either manual removal by an expert or surgical removal has to be done..Hope this information helps..If you find the answer helpful please give me a Five Star Review and click on found the answer helpful as a token of appreciation...Thanks and regards.Dr.Honey Nandwani Arora.."
},
{
"id": 80153,
"tgt": "What causes sharp pain under the rib cage while breathing?",
"src": "Patient: Having severe sharp pains in upper rib area under breast from coughing. Been on antibiotics for 9 of 10 dose to treat a severe sinus infection per doctor. Still highly congested both nasal & chest congestion. Concerned that still not better. What do I do for the sharp pains? Is it muscle strain from coughing? Doctor: Hi,Thanks for writing in to HCM.Kindly finish your entire course of antibiotics.The pain is mostly due to muscle straining due to coughing. The pain will reside slowly so don't worry.Kindly visit your physician because if chest congestion is still present on clinical examination of the chest then the course of antibiotics may be extended.Thanks."
},
{
"id": 185914,
"tgt": "What is the treatment of ache around wisdom tooth and cheek ?",
"src": "Patient: I had #19 molar removed on Monday, 1/30. I had no pain on Monday or Tuesday. Wednesday through today I have been experiencing aching around the wisdom tooth and my cheek. Also, the extraction site feels tight at times and the teeth in front of the site throb at times. The extraction was difficult and took a while to perform. It was removed because of external resorption. I even have a slight bruise on the outside of my chin. The aching goes to my ear, too. I saw the oral surgeon on Thursday....he said no dry socket or infection......I am still having pain and aching. What could be going on? Should I wait longer or call the dentist again? Doctor: Hello!Read your query.I strongly recommend you to call your dentist as pain after 4 days of extraction is unusual.This can be due to a dry socket (most common in tough extractions and lower jaw as in your case).I suggest you not to eat anything hot or spicy. Do not smoke.In other case, there can be another tooth which is causing the pain.Take Renerve plus once daily and analgesics.Regards."
},
{
"id": 139003,
"tgt": "Can falling down the stairs causes further injury to a labrum tear in hip?",
"src": "Patient: I have a labrum tear in my right hip which has hampered my mobility quite a lot and causes considerable pain usually helped by 250mg tramadol. However due to my mobility issue I fell down the stairs yesterday didn t feel too bad afterwards but from last night the pain is horrendous! Could I have further damage? Doctor: HiWelcome to healthcaremagicI have gone through your query and understand your concern.It may be possible that you may got further damage due to fall. You can get MRI to further assess the damage. You can get analgesic such as ibuprofen for pain relief. Rest will be useful. You can discuss with your doctor about it. Hope your query get answered. If you have any clarification then don't hesitate to write to us. I will be happy to help you.Wishing you a good health.Take care."
},
{
"id": 196125,
"tgt": "Suggest treatment for burning sensation during urination",
"src": "Patient: my boyfriend said when he goes to urinate it burns him before but mostly when he finishes. HE ALSO takes adsvil at least 4 to 5 a day & extra aspirin strenght because he has severe pain . HE needs to get opereated for his hips. he wants to know if it is ok & what else can he use ??? Thank YOU!!! Doctor: hii.welcome to our site.my opinion is that your friend has urinary tract infection.ask him to maintain good hygiene around pubic area.ask him to take about 3 litres of water a day.antibiotics like tablet norflox twice daily for 5 days will be helpful.ask him to undergo urine analysis test.hope my answer would have helped you.thank you."
},
{
"id": 102939,
"tgt": "Child having breathing problem since birth due to allergy. Prescribed Rigix, tendagyl, brintyl and nebulisation. Tips required?",
"src": "Patient: i really need ur answer as fast is it possible, my son has breathing problem since he has born every other dr said he has allergy i was giving rigix.tendagyl ,and brintyl syrup and nublize him as well for his flu problem then i went one of the best dr of our country and he suggest neublize as well and zatofen and myteka for a month and he said he ll carry all thx medicine till 3 months iam giving him these medicine since last wk and it feels he suffering more mean he drain more flu i can say and breathing problem at night occasionly pls advice me some tipsworried mother Doctor: hello dear thanks for your query at hcm my recommendations would be 1 give plenty of water2 avoid dust3 avoid pollution indoor and outdoor 4 avoid pet animals contact 5 avoid cold things like ice creams 6 good energy rich protein foods these things will improve child health regard"
},
{
"id": 197228,
"tgt": "What causes severe pain in scrotum?",
"src": "Patient: Hello , about 10 hours ago I started using Vimovo and Etoflam as prescribed by a GP for back pain, taken 2 tablets/applications so far . In the fast 4 hours I have quite severe pain in my scrotum where there is a large (16cm) epididymal cyst, is this pain connected to the new meds? i also take , warfarin , valsartan and bisop , the GP is aware of all my meds.Thanks Aidan Doctor: HelloThanks for query .Based on the facts that you have large 16 cm sized Epididymal Cyst in your scrotum the severe pain that you have since two days could be mostly due to either a) Sudden increase in size of the cyst b) Infection setting up in cyst It needs to be confirmed again by doing Ultrasound Scanning of scrotum .In the mean while start taking broad spectrum antibiotic like Cefotaxime along with anti inflammatory drug like Diclofenac twice daily .Continue rest of the medications that you are taking for other illness.Further treatment will depend upon result of this test.Dr.Patil."
},
{
"id": 166856,
"tgt": "Suggest treatment for cough",
"src": "Patient: Dear Dr. I have a baby with 3 yrs old and she is having cough at night. While she was in India (home country), she used to take Dilosyn for cough. Currently she is with me here in Saudi Arabia, I would like to know what medicines I can give to her. Or, is there any chance to get the same medicines here. Your prompt reply is highly appreciated. Regards - Feroz Doctor: Hi,Let me understand your little baby has been diagnosed with. The reason of nights cough? Ok! you are giving medicine, but for what? Cough its a sign not a disease. Example if you drink water quickly you can cough, right? Did you ask yourself why just tonight? Some disease can do that like respiratory allergies. Please let me understand better, thanks."
},
{
"id": 101137,
"tgt": "Suggest treatment for bronchitis",
"src": "Patient: I am being treated for bronchitis. I took a five course of arithromycin. I felt better and then worse again. My doctor prescribed Augmentin. After the third dose, I woke up with excessive saliva, eventually vomiting several times. Is this likely an intolerance to the Augmentin? Doctor: Hello.Thank you for asking at HCM.I went through your history. I would like to ask you a few more questions like: What is the total duration of your illness? Why did you need 5 courses of azithromycin? At what interval and for what duration did you take azithromycin? Did you have any blood reports? If yes, what were they? What are the other medications you are taking currently? Did you have fever? After antibiotic course, do you have fever at present? Did you have such complaints in past? etc etc.However, from your history, I would like to suggest you following:1. I would first think of gastritis as cause for excessive salivation and vomiting. It can be due to any antibiotics or some other medicines also. So it does not necessarily suggest intolerance to Augmentin. I would suggest you antacid like omeprazole or pantoprazole half-an-hour before meal for at least 7 days/as long as you are taking antibiotics/medications whichever later. 2. Please continue your other medications as prescribed by your doctor.3. If you have not had complete blood count, I would like to suggest to get it done. It may be helpful in deciding whether you need further antibiotics. Personally, if counts are normal and if you no longer have fever also, I would not prescribe antibiotics.4. Taking Augmentin after meals will also help. 5. Please try to avoid smoking, caffeinated beverages, hot & spicy meals.Hope this will be helpful to you.Wish you a quick recovery and the best of the health.Should you have any query, please feel free to ask at HCM.Regards."
},
{
"id": 101711,
"tgt": "What is the remedy for the rashes which are spreading?",
"src": "Patient: I have a allergy all over my body. This allergy has big phokas all around my body. I think this allergy is caused by dirty water I used to wash my hands. Moreover It is increasing in speed rate. Any suggestions, plz give me. I am really in difficult situation. (I am 18 years old.) Doctor: HI, thanks for using healthcare magicAllergic reactions are treated by avoiding future contact with the source if possible and medications.The medications normally include oral antihistamines, topical steroids and in more severe allergic reactions systemic steroids.I hope this helps"
},
{
"id": 73703,
"tgt": "What causes fluttering feeling in chest?",
"src": "Patient: I have noticed in the last week or so that after dinner, I feel a thump in my chest that you can actually feel and a fluttering right under my chest. I recently had a colectomy and am not sure if that has anything to do with it. I feel the fluttering after breakfast as well. I am 58, 5'5'' and weigh 126. I am a two time cancer survivor. Doctor: Thanks for your question on Healthcare Magic.I can understand your concern.Fluttering feeling in chest should be evaluated for arrhythmia (rhythm disturbances in heart).So get done ecg, 2d echo and Holter monitoring (24 hours continuous recording of Ecg).If all these are normal then no need to worry for arrhythmia and other heart diseases.Your fluttering is seen after food. So gastritis is more likely.So take pantoprazole tablet on empty stomach twice daily.You will mostly improve with this.Hope I have solved your query. I will be happy to help you further. Wish you good health. Thanks."
},
{
"id": 79593,
"tgt": "What causes burning sensation in the upper back and chest?",
"src": "Patient: Hi. Last Monday I was at work when all of a sudden my upper back area had a burning sensation and then I felt like my sides and chest had pressure. Mostly noticed in my sides and back. Then I felt a little dizzy. Lasted a couple of minutes. I am 51 years old. I quit smoking 10 months ago. I am about 30 lbs overweight. Doctor: Thanks for your question on Health Care Magic. I can understand your concern. We need to first rule out cardiac cause for your symptoms because you are smoker, obese and having age more than 50. So you are at risk for heart diseases. Get done ecg and 2d echo. If both are normal than no need to worry for cardiac diseases. You are mostly having GERD (gastroesophageal reflux disease). Smoking, obesity and old age are known risk factor for GERD. It is due to laxity of gastroesophageal sphincter. Because of this the acid of the stomach tends to come up in the esophagus and cause the symptoms of burning chest pain. So avoid stress and tension. Loose weight and quit smoking as soon as possible. Avoid stress and tension. Avoid hot and spicy food. Start proton pump inhibitors and prokinetic drugs. Don't worry, you will be alright. Hope I have solved your query. Wish you good health. Thanks."
},
{
"id": 331,
"tgt": "Is pregnancy possible after taking sicriptin for high prolactin and krimson 35?",
"src": "Patient: hi my age is 27 i ve irregular periods.i ve billateral polycystic ovaries and my prolactin is 48 and tsh is 7.4,lh is 9.54 doctor prescibed me to take eltroxin 50mg for hypothyroid.sicriptin 1.25 for high prolactin n krimson 35 for high in lh.iam married girl iam waiting to get pregnent.will it possible for pregnent after taking these tablets Doctor: yeah u can get pregnant...go to ur gynec and take clomiphene or letroze from day 2 ...den follicular study...if needed she will give inj gonadotropins. ...til follicle size 18 mm...then rupture...den keep relationship"
},
{
"id": 108796,
"tgt": "What causes lower back pain with appetite loss and loose stools?",
"src": "Patient: Hi. I woke-up this morning not feeling the greatest b/c at first my lower-back had a dull pain. I still have this mild ache 4hrs later which is kinda going down my right leg. Then as I moved around (and still) have a dull/mild pain in my mid to lower right abdomen. The last few days i have been very hungary but as soon as I get food infront of me I loose most of my appetite and have had some diarhea/loose stool. To include this morning. Any ideas? Thank you. Amanda Doctor: Hi,Going through your symptoms, I think the lower back pain radiating to your leg is a different entity from the pain in the mid to lower abdomen. The pain in your lower back is most probably a muscular strain and you need to initially treat it with hot pack application and over the counter pain medications. If that doesn't help fix an appointment with your physical therapist. Interferential therapy certainly helps in decreasing the pain meanwhile. Once the pain comes down discuss with your physical therapist about the need for exercises to strengthen core muscles. It will go a long way in helping you from getting the pain again. Swimming too is a very good exercise. But all these need to be started after the pain has receded."
},
{
"id": 866,
"tgt": "Can i take anti rabies vaccine if trying to get pregnant?",
"src": "Patient: last night I had tried to get pregnant for the first time for having a baby and last night only i was bitten by a rat i am worried that if I would take anti rabies vaccine then if I get conceive will it effect my child? Would i take any contraceptic pill to clear it out? Doctor: Hi, I think you can take the vaccine. It is safe during pregnancy and it will not have any effect on the bay. So, don't worry about it. Hope I have answered your question. Regards"
},
{
"id": 95344,
"tgt": "Is it because of infection that I am having cramps in my lower abdomen ?",
"src": "Patient: hi,i have been having some cramps to my lower abdomen ,i visited a OBGYN and he told me that it might be endometriosis or an infection,he prescribe some antibiotics which i already had taken.am not feeling the pain anymore so am wondering if there be any chance of me being pregnant? as he had told me that the pain might be the reason why am taking so long to get pregnant.i have had an pelvic abdominal ultra-sound and everything was normal. Doctor: hi, if you have got an ultrasound & its normal, there is no chance of you being pregnant. get a urine pregnancy test done if you feel so or a repeat ultrasound."
},
{
"id": 17305,
"tgt": "Suggest treatment for heart blockage",
"src": "Patient: Dear sir, My brother-in-law suffering with unknown dieseases. I will tell u the history. 9 months back he was got head-ack and got unconscious for 40 mins. All checkups are done.But nothing rulled out.(but his heart has 40% blocked ) After nine months the same problem he got infront of me. His eyes got red and himself told me go immediately to hospital as he is going to know he will get unconsious. And I himself get him to hospital mean time he got unconsius for 2 mins with pain in chest. EGC, EEG and other reports are normal. Kindly suggest the way to rull out the problem. Thanking you sir, Narendar Doctor: Hello, If he has all reports came back normal and everything is fine then finding the cause of unconsciousness is a task. Kindly consult a psychiatrist and a neurologist for proper evaluation and management. Hope I have answered your query. Let me know if I can assist you further. Take care Regards, Dr Bhanu Partap, Cardiologist"
},
{
"id": 161153,
"tgt": "Severe headache and low grade fever,dizziness.Treatment?",
"src": "Patient: HI. We are on Vacation in the Moutains. My six year old son complained yesterday with a tummy ache, which only lasted a little while. Last night as we come down out of the Moutains he complained with ear pressure, which was followed with a bad headache and low grade fever. This morning his only complaint is dizziness, otherwise he seems OK. We treated this with children s tylenol and benedryl. How should we treat this since we are many miles away from his doctor? Doctor: Hello, What you have done currently with Tylenol and Benadryl is perfectly normal. Please continue the same management, keep him well hydrated. Hope I have answered your query. Let me know if I can assist you further. Regards, Dr. Sumanth Amperayani, Pediatrician, Pulmonology"
},
{
"id": 62357,
"tgt": "Suggest treatment for large painful lumps on breasts",
"src": "Patient: I have large lumps in my breasts for 5 years now. I have went to a doctor and they did an ultra sound and I was told they were not anything to be worried about. They ache all the time and the left one is now weighing down my back and shoulders it is so big. Someone please help me. Doctor: hi.it is best if you consult with a doctor, preferably a general surgeon, for medical and physical examination. based from your description, it could be a cyst, a benign breast lesion. it could also be a fibroma or fibrocystic lesion. other tumor types, such as malignancy, must also be ruled out. these lesions are best evaluated clinically. further diagnostic examinations, such as breast ultrasound and mammography, will be requested as needed. regular breast examination and follow-up with your doctor is highly recommended in these cases. management (medical and/or surgical) will be directed accordingly.hope this helps.\u00a0\u00a0\u00a0\u00a0\u00a0good day!!~dr.kaye"
},
{
"id": 167106,
"tgt": "Suggest treatment for swelling in lymph glands",
"src": "Patient: My 13 month old daughter has been getting teeth and has had a fever on and off for about 4 days. Yesterday I noticed her lymph glands were terribly swollen and she was having a hard time swollowing. She also had a small white sore on her tongue. I took her to urgent care and was told she was teething. They said her nose, ears, and throat were all clear. Today I attempted to brush her teeth and blood went everywhere from her gums and she screamed. As much as I can see, her gums look very red and swollen. They look like they might be infected.. They look painful. Any help would be appreciated. Thanks. Doctor: hello ,Thank you for posting your query with healthcaremagic,First of all, I would like to appreciate you for taking your daughter to urgent care in order to make sure, things are under normal circumstances,Also, I would like you to know that during teething period it is quite common for kids to have fever, swollen/red gums, if you could offer you daughter cold teething toys(keep it in fridge for sometime) in order for her to chew on it which will sooth her gums down.Also, giving your daughter tylenol for fever will help her a lot. If she has any known allergies, kindly see the pediatrician before giving her tylenol.But if you feel, there is an infection in her gums, I will advise you to see pediatrician again , because kids are prone to infections when their immunity goes down ,as they are not eating well during teething period.Bleeding from gums while trying to brush , could be because they are sore/swollen right now. Don't worry, kids do go through this phase which is temporary,I wish your daughter healthy day,If you have any more questions feel free to right us back.thank youwith warm regards,Dr. Anshika Taneja"
},
{
"id": 39444,
"tgt": "What causes vomiting, cough and stomach pain inspite of normal urine ,ultrasound and blood tests?",
"src": "Patient: My daughter started with vomiting just over a week ago. That stopped by Wednesday but than early Thursday morning she woke up in extreme pain on her left side of her stomach. We took her to our family doctor who said she had a stomach virus but the pain in the stomach bothered him so he sent us to a specialist for stomachs. He examined her on Friday put her on liquid only and than saw her on Saturday. She seemed a little better saying the pain was about a 3 out of 10. We then saw him again on Monday and she was still having the pains so he sent us to emergency. They did blood work, a urine test and ultra sound which all came back normal. They told us to keep an eye on her that it could be just a strain on her stomach muscles from all the vomiting and coughing but she has now sent me a message that the pain is at 8. She has taken Advil to help with the pain but feel so helpless. Is there anything else we can try to do? Doctor: most common cause of generalised pain is indigestion and upset stomach , generalized mild pain or cramps pain lasting for several hours may be cause of obstruction . Localised pain may be coz of peptic ulcer disease , tubercular , celiac disease , even mild constipation can lead to the pain. My advice you you will be think of visiting a gastroenterologist and plan for endoscopy to rule the cause . Hope this helps ."
},
{
"id": 5077,
"tgt": "TTC. Off depo shots for a month. Prescribed folic acid. Pregnancy possible by being off depo shots?",
"src": "Patient: I BEEN ON THE DEPO FOR 10 YEARS MY LAST SHOT WAS IN MARCH I SUPOSSED TO GOT IT IN JUNE 7TH BUT I DIDNT MY HUSBAND WANT ANOTHER BABY I WANNA KNOW IS THE DEPO OUT MY SYSTEM I HAVENT HAD MY PERIOD YET IT WASNT NEVER NORMAL I WENT TO THE DOCTOR SHE GAVE ME FOLIC ACID TO START TAKING WEN I TOLD HER I WANNA HAVE ANOTHER BABY MY QUESTION IS CAN I GET PREGNANT BY BEING OFF THE DEPO FOR A MONTH Doctor: Hello, I would be happy to help you with your question. The depo shot is associated with more irregular cycles for the first few months after it has \"worn off\". It actually lasts longer than 3 months, so it probably is not completely out of your system. Your chance of pregnancy within the 6-12 months is slightly less than the average because of this cycle irregularity, but this should resolve over the year. I hope that this helps and good luck!"
},
{
"id": 221256,
"tgt": "Can withdrawal method cause pregnancy?",
"src": "Patient: I have been off the pill since the 26th of july and have been having unprotected sex since, however I have been using the withdrawal method of contraception, and about a week and a half ago I started feeling nauseous. What are my chances of being pregnant? I took a pregnancy test on Monday which was negative however the test says it is effective from 14 days of conception and it is most likely that I would have concieved between the 10th and the 13th of this months as they are my most fertile days. Doctor: HelloWithdrawal method is not completely safe.Pre ejaculation fluid may contain sperms and pregnancy is possible in withdrawal method.You should wait for next period.If period is missed then urine pregnancy test can be done.Two dark lines on the card confirms pregnancy.Ultrasound of pelvis can also be done for confirmation of pregnancy.It can detect G-sac earliest.Take CareDr.Indu Bhushan"
},
{
"id": 201599,
"tgt": "What causes slight back pain with pain in testicles?",
"src": "Patient: I have been experience slight back pain in the last few months. This morning out of nowhere I experience bad pain in my testicles. I have not been sexually active. I thought perhaps I should relieve the pressure on my own. Instead of semen, all that came out was sort of dark blood. Doctor: HIWell come to HCMThank for choosing HCM, if you found bleeding with semen with the history of back pain, this could be due to some infection with inflammatory process, and this would need some investigations, like ultrasonography, urine test, plain x-ray of lower abdomen, till then the symptoms could be managed with \"Tab Diclofenac 50 mg\" sustain release once in day, you have not mentioned your age else something would have been advised that is related with age, condition need to be taken seriously, hope this information helps."
},
{
"id": 67314,
"tgt": "Can itchy lump on calf be lipoma",
"src": "Patient: Not sure if its lipoma or what ..but I feel lumps in open places right around where my muscles are and I found one right on my right calf that is itching like crazy. It s inside right on the middle of my calf. It is not externally visible and there s no redness. deposit cyst, lipoma, or some other kind of cyst? Or what? Doctor: Hello and welcome to HCM,Lump on the calf could be due to cyst, lipoma or muscle swelling etc,However, itching is not associated with any of the above mentioned swellings.Itching is usually associated with allergic condition or any skin condition.A clinical assessment of the swelling is required.Imaging of the swelling is also required to assess the site and origin of the swelling.Aspiration cytology is the investigation which will determine the site and origin of the swelling.Thus, consult your primary health care provider for clinical assessment and relevant investigations.Thanks and take careDr Shailja P Wahal"
},
{
"id": 128842,
"tgt": "Feeling lifeless inside the knee after the removal of bursa sac",
"src": "Patient: I had infection in the bursa sac of my knee and had the bursa sac removed surgically. Since that time (March 2012) I've had cold feeling and lifeless feeling in my knee and sometimes my whole leg? Do you know if removal of the bursa sac could have caused this? Thank you so much! Doctor: Hi, I had gone through your question and understand your concernsWith such presentations in my clinic, I would first rule out any cause for neuropathy or cutaneous nerve injury that may be happened during removal of the bursa.This may take time to improve so i advice you to receive neurotronics to help rapid recovery."
},
{
"id": 216825,
"tgt": "Suggest treatment for jaw pain",
"src": "Patient: I am a 33 year old female. at 1030am i developed jaw pain, shooting pain from left ear to middle of chin and feeling syncope. i took bp with a result of 122/77 and then decided to check my cbg because i am hypoglycemic. it was 109 after a grande vanilla latte and a peach turnover 2 hours earlier...so i continued with my day. at 1230pm my feeling of like i was going to faint was still there, and the sharp jaw pain but then my left side of my chest developed a bad aching pain that radiated to my left shoulder and down my arm to right below my elbow. full deep breaths caused my left shoulder to hurt worse. So i took a 81mg aspirin and went to the ER. EKG showed normal and chest x ray was clear. this was on tuesday and it is now thursday ngiht and my chest still hurts. i f/u with my pcp on wednesday and he mentioned i needed to monitor this as i showed more discomfort when he pressed by my sternum then anywhere else. mentioned something of i could have a virus. his only suggestion for me was ibuprofen and time. what could be going on? should i just wait and see? should i request to speak with a cardiologist? I am 5 7 145lbs and live healthy lifestyle as i dont smoke and only social drink occasionally. any suggestions will be great. Thank you for your time in advance. Doctor: hiyou may also require checkup by a dentist for tooth issues and jaw pain,hope there is no external visible swelling in jaw.If EKG is normal, less chance for heart pain, but many a times early changes do not show on EKG, central sternal pain may have relation to heart, you may observe if physical climbing stairs or jogging or strnuous physical activity bring on any chest pain, if does, then a Tread mill test for heart and an echocardiogram aare neeeded and serum Treponin -T and I tests, for which a cardiologists consultation for all tests is necessaryInmeantime take ibuprofen as your doctor tells you, and arrange appointment with a cardiologist if chest pain is persistingbest wishes"
},
{
"id": 156043,
"tgt": "Do I need to remove scrotal cyst surgically?",
"src": "Patient: Hi, I have a scrotal cyst and it burst without me applying any pressure, I then squeezed all the puss out and wiped it to keep it clean and put a plaster on it.I don't think it's cancerous, should I get it surgically removed and what should I do if it gets infected?I've read about getting 'the pea' removed, what's this? And does this mean that the puss will come back? Doctor: Hi and welcome ot HCM. this wcan come bak but not necessariy. it can be removed surgically or you can wait and see progression. Wish you good health. Regards"
},
{
"id": 135031,
"tgt": "What causes redness and tingling sensation to the toes with numbness?",
"src": "Patient: I was hit by a scooter 3 days ago and my left foot is now broken. At the hospital they said I fractured my foot but I didn t get to see the x-rays but it s on the outside part of my foot-I can still move my toes. But every time I get up and move around my toes start tingling and feeling numb. They also get this red color to them as if all the blood has rushed to my toes and my break starts throbbing. My foots in a temporary cast until I got back to the doctor this week to make sure the swelling has gone down before they put on a harder cast. Is the break pinching a nerve or cutting off my blood circulation? My feet feel fine when they re elevated but hurt at any other time. Doctor: hello healthcare user, its completely normal for a fracture foot to pain.it is not advisable to walk on temporary cast..if you continue to walk or keep your foot down then swelling is going to increase and so tingling and numbness also...i would advice you to elevate your foot over pillows.this will reduce the swelling and will automatically reduce tingling and numbness.pain will also subside..just check if the plaster is too tight...if it is so then you need to consult our doctor immediately.....thank you"
},
{
"id": 132009,
"tgt": "How to get rid of the pain in both lower and upper back?",
"src": "Patient: To whom this may concern, My fianc\u00e9 experiences severe ( on a scale of 1-10; its a 9) pain that can be debilitating at times. He has had a surgical procedure done due to a herniated disk. He experiences pain in both upper and lower back, stiffness, sharp pain in the rib cage area (at times), a sharp pain his knee, leg, and the upper hamstring area (the buttock). We believe there is something wrong with his sciatic nerve. This pain is too the point where he has trouble sleeping and getting comfortable at night. He has trouble walking comfortably at times. I was wondering if you could provide any input as to what he may have and any methods that can be done to relieve his pain. Thank you Jennifer Doctor: Hi Jennifer. Your Fiancee had back surgery following which he experiences severe pain from rib cage down to hamstrings upto leg and stiffness in the back. Yes This pain is called Sciatica. With H/O Disc Surgery , In my opinion , I would like to start Pain killers, Muscle Relaxants and Physiotherapy for 10-15 days . If pain persists even after 15 days with the same intensity then I would go for another MRI and if gets relief , I would decrease the amount of medicines and increase his physiotherapy including muscle building exercises of the back. The aim is to relieve the pain and do something that pain doen not come back again."
},
{
"id": 183816,
"tgt": "What causes aches with swollen cheeks despite tooth removal?",
"src": "Patient: My 7 year old was in pain Tuesday all night we had a emergency visit to dentist they took 3 x rays well they make the choice to pull the tooth well thur and Friday he was in pain with swollen check n eye took back to dentist sat morning did exray again n looked and said took area fine...said if pain worsen n more swollen take to er ...what is going on n why Doctor: Thanks for your query, I have gone through your query.The possible causes of the swelling could be the infected tooth leading to cellulitis resulting in extension of the swelling to involve cheek and eye region. Consult a oral and maxillofacial surgeon and get Intravenous antibiotics injected. The infection will come down after that the infected teeth can be removed. do not delay, if the infection spreads then it leads to complications like difficulty in breathing and swallowing. I hope my answer will help you, take care."
},
{
"id": 179655,
"tgt": "Suggest treatment for diarrhea",
"src": "Patient: My daughter is 1 year and 7 months old, recently she had diarrhea before 17 days, she had given inocef 500 mg antibiotic twice a day for three days with zincat od syrup for 12 days from doctors.. Gradually she was comming to normal stool but i found her stool too much smelly for two days and from next day she had vomits and then she is having soft or loose stool ,she complainssome discomfort before passing stool,, now my doctor had suggested to go for a stool dr test which results wbc present 20 in report.. Now what can i do for my baby please i want help from you... What food is suitable and milk is good for her or not. Please reply Doctor: HIWell come to HCMAs long as the pediatric age group is concern most of the diarrhea are self limiting and the hydration is only matter, some time antibiotic could flair up the condition and this need to be avoided, in my opinion here the child need to be given \"lyophilized lactobacillus\" sachets three times in day, and be patience, hope this helps."
},
{
"id": 192795,
"tgt": "Could antiseptic lotion be used to clean penis after circumcision?",
"src": "Patient: Hi my name is Eldrin, i just had my foreskin cut off, and its early days but i was given pain killers and i was told to wash and dry it everyday. I only started washing/cleaning it yesterday, can I use the savlon antiseptic wash/cream to help with the hilling process or what can you advise me to use please thanks. Doctor: Hello, You can apply topical antibiotics like mupirocin in the initial post-op days. Antiseptic lotions can cause dryness and healing might get delayed. Once the wound is healed you can apply Savlon if required. Hope I have answered your query. Let me know if I can assist you further. Take care Regards, Dr Shinas Hussain, General & Family Physician"
},
{
"id": 129430,
"tgt": "Suggest treatment for numbness in the right leg",
"src": "Patient: Hi imwoke up with no feeling in my right leg and fell down like my leg was still sleeping I had to wait a while for the feeling to come back, what could this be? Is it a concern it s the first time this has happened Right now it feels like I strain on my leg but I m able to walk and when I walk it feels like I have pulled a muscle with no pain though Doctor: Hello and Welcome to \u2018Ask A Doctor\u2019 service.I have reviewed your query and here is my advice.This happens to normal people when you sleep with your arm or leg in an awkward position that compresses the nerve of the leg or arm. Usually, the tingling or numbness sensation goes way after few minutes when you change the position of the leg or arm.You should not be concerned unless it happens too often even during the daytime when you are not sleeping. If this is the case you should suspect for a discal hernia and see a neurologist.Hope I have answered your query. Let me know if I can assist you further.Regards,Dr. Edvin Selmani"
},
{
"id": 189492,
"tgt": "Heaviness in tongue, water in mouth, jaw pain, headache, warm palms and feet, brown spots on breast, stomach, hand",
"src": "Patient: i feel my tongue gets heavy and lots of water in mouth in every time,feeling bad pain in both side under jaw and have a bad headache in the right side .the pain under jawline increase due to the headache. and the area under jaw line becomes fatty.. there is another problem, my palm of hand and feet remains hot all the day. i have tiny brown and red spots on my breast,belly and hand,neck. Doctor: Hello dear Here i think you have mixed two conditions.your jaw pain , watering in mouth and heavy tongue and even your headache are most probably is due to bad oral hygiene and you might need regular professional cleaning because if there is excess of plaque in mouth then it can lead to such kind of symptoms.And second your increased body tmperature and brown red tiny spots can be because of disturbed hormonal level ( THYROID). So better consult with your dental hygienest and GP lateron. Best Dr. Amit k. Gaba"
},
{
"id": 149521,
"tgt": "Spinning sensation in head, reoccurring. Taking supplements. Occasional drinker. Recommendations?",
"src": "Patient: Hello DocYesterday I was doing Yoga in the morning - which I do regularly a posture called The Dhanurasana. After that when I was getting up to go for shower - I felt a reeling / spinning sensation in my head which was reasonably strong so I just went back to the lying position and this went away after some deep breathing. Then again in the night I woke up to go to the toilet and felt that there was a similar sensation but milder . I am male - 56, 176 cm, 70 kg, non smoker. occasional social drinker (wine only -no beer or spirits) till now do not have BP, Cholesterol or blood sugar problems go for swims and thrive mainly on low red meat diet. Yes during weekends some times there is an indulgence at get together parties with some oliy and spicy food. Take a few supplements like fish oil , Glucosamine Chonditrin - can you please help?Can you Doctor: Hi and thanks for the query,You are right to practice Yoga and take supplements. however, under such circumstances with such symptoms, it is important in my opinion to obtain a complete clinical review form an internist. A clinical review to exclude heart and neurological symptoms is important. Syncopal episodes should be excluded. A cardiac ultrasound and an electroencephalogramme could be necessary Based on associated symptoms such as fainting, history of epilepsy, history of heart disease in the family and breathlessness are important to be considered when doing your evaluation.I suggest you book an appointment with an internist for a complete clinical review. thanks and kind regards,Bain LE, MD."
},
{
"id": 199299,
"tgt": "What causes continous erection with yellow fluid discharge?",
"src": "Patient: Hi I have a problem now a days. My Penis is disturbing me. I feel erection most of the time also most of the time some yellow liquid in drop came out . I dont know how it happen but when i so piss or urine cme it i feel pain but my urine color is normal like transparent. I am really worried. there are thick drops on my underwear and trouser. Doctor: DearWe understand your concernsI went through your details. You have not mentioned your age here. The yellow liquid you are mentioning here could be pre-sperm-lubricator. Erection could be because you are an youngster and you are easily aroused. You need to talk to a physician regarding the pain you are experiencing. Don't worry and consult a physician.If you require more of my help in this aspect, please use this URL. http://goo.gl/aYW2pR. Make sure that you include every minute details possible. Hope this answers your query. Available for further clarifications.Good luck."
},
{
"id": 67905,
"tgt": "Suggest remedy for lumps in clitoris",
"src": "Patient: Hi, I have a lump above and to the right of my clitoris. The lump isn t really visible but you can feel it when you apply pressure to it. It is about the size of a pea. It is constantly hurting and it just started today. There is no drainage at all. It is very sore. J have no idea what it is!! Doctor: Hi,This is most likely to be a Bartholin's cyst that has become inflamed. Apply a warm compress to the area or take a warm bath to relieve the pain. Your may benefit from antibiotics so I suggest you see your doctor. If it does not improve with antibiotics, then it can be removed by a small surgical procedure. This is not a serious problem and is quite common. I hope this has reassured you. Regards,Dr K A Pottinger"
},
{
"id": 119049,
"tgt": "34 year old with leukopenia. Low WBC. Bone marrow biopsy negative. Pregnancy confirmed. Any danger to the fetus?",
"src": "Patient: My 34 daughter has had leukopenia for 18 months, with WBC counts ranging from 0.6 to 1.8. A bone marrow biopsy was negative and after numerous blood tests and seeing every kind of specialist, there still has been no cause found for her condition. She just found out she is pregnant and as her mother 3000 miles away I'm quite concerned. Any danger to her and the fetus? Doctor: Hi,A WBC count which ranges between 0.6-1.8 is quite low and there might be a risk of infection. Infections during pregnancy might complicate the pregnancy and affect the baby as well. Your Obstretician will be on the look-out for these complications and might even give your daughter some antibiotics to prevent infections. Hope that things turn out well for you.Regards."
},
{
"id": 187984,
"tgt": "Hard bumps behind teeth, slightly hurts, any advice?",
"src": "Patient: I have 3 hard bumps behind my front lower teeth, if I lay my tongue flat on the bottom of my mouth it's touching the end of my tongue, one is bigger and there's two smaller ones. I have another smaller one at the left side below my teeth that's also hard, they all can move around and it slightly hurts when I do, please help Doctor: Hello, As you are mentioning that there are 3 bumps behind your front lower teeth, there can be three possibilities-first it can be possible that these are normal salivary gland opening which is present there, and you have noticed it now. -second it can be possible that your front teeth are infected and now the infection is spreading in the apical region and forming pus because of which there is swelling. -third possibility is that there must be some salivary gland infection which has appeared like this.According to me you should visit a good dentist and get it examined completely with an x-ray which will give you the diagnosis and help to solve the problem. Hope it will help you."
},
{
"id": 106922,
"tgt": "How can backache be treated?",
"src": "Patient: Hi Doctor, I am a 40 years old women and have two kids. For the last ten years I am suffering from back pain which mostly goes to my leg. I have been trying physiotherapy, muscle strengthening exercises supported by physiotherapists, consulted Chiropractic clinics, etc. But so far the pain is till there and getting worse. Recent MRI result indicated that there is a problem on both sides of my hips( hip displacement) and recommended treatment (physiotherapy and compressor machine) for a month to fix them back and strengthen the muscles. Though there is improvement, I have a different pain on my left Thigh and Leg and it is getting worse. I can see a reddish color the side of thigh, it is not painful when I touch it but inside I feel pain and burning on thigh all the way to my leg. It gets worst when I walk. I don t know what to do and whom to consult (internist, bone specialists, etc.). and I am worried that this might not be a simple thing...I worried and not sleeping. That s why I am writing you to advise me. Awaiting for your response, Thank you so much! Email address: YYYY@YYYY Doctor: Hello, I have studied your case. As per your history there can be possibility of hip arthritis which can be degenerative or due to avascular necrosis of femoral head [AVN].I will advise you do X ray of hip and you can consult me with reports or with your doctor.After clinical examination IF AVN is suspected then you will need MRI again with spine screening.Till time take analgesic and cartilage protecting medication like glucosamine sulphate.For hip arthritis joint flexion, extension and rotation exercises will help.Physiotherapist will help you better to learn more exercises in proper protocol.If required intra-articular injection can be given.Diet rich in calcium and protein will helpHope this answers your query. If you have additional questions or follow up queries then please do not hesitate in writing to us. I will be happy to answer your queries. Wishing you good health.Take care"
},
{
"id": 13309,
"tgt": "What causes rash on legs after consuming alcohol?",
"src": "Patient: Hello. I ve noticed that every time I drink alcohol I end up w a rash on my lower legs. (Pin point like petichia) Now im starting to get a red tender inflamed area on the dorsal foot of both feet. It feels sore. Im on my feet all day and never get this rash, but as soon as i drink thats it. I ve done testing to check my platelets and their within normal limits. Please give me an idea of what it can be. Doctor: Hi, It is erythema nodosum. There are so many triggers responsible for the disease. The factors may vary patient to patient. In your case, alcohol may be the triggering factor for the development of erythema nodosum. Consult the dermatologist for the perfect diagnosis and proper treatment. To prevent the the lesions,you must stop to take alcohol. If the lesions are there, get proper treatment from a dermatologist. Hope I have answered your query. Let me know if I can assist you further. Regards, Dr. Ilyas Patel, Dermatologist"
},
{
"id": 104629,
"tgt": "Severe breathing problem, chest whistling, congestion, using asthalin, no relief. What tests do I need?",
"src": "Patient: i am 22 and weight 75 kg height 5-11. i am suffering from svere breathing problem.when ever there is change in climate specialy when its raining, problem get triggered like today. my chest is whistling nd having congestion. i used a lot of asthline now a days. i got relieve for short period of time and after dat same problem. help me out. what should i do. plz plz plz help me.... Doctor: hi, you have asthma related to pollen allergy can be dust mites also,since your having it most of the time. for your asthma i recommend you to start using steroid plus long acting bronco dilator inhaler. go to pulmonologist he will assess your lung function and also give you the inhalers also i suggest you go to immunologist to get allergy test done . then you will know what is causing your symptoms.once that is done you can start immunotherapy. which is very effective. it will control your symptoms by 80-90 percent atlest. thank you dr.mukesh"
},
{
"id": 82950,
"tgt": "Extreme pain in joints. Suffering from Lupus SLE & Discord, also had heart attack. Need help with the pain",
"src": "Patient: I have been been suffering with Lupus SLE and Discord and I also suffered a heart attack a year ago which a stint has been place and because of that I was seeing a phsychaitris for anxiety for fear of dying in my sleep. My Lupus causes extreme pain that affects my joints and I have been seeing a dr. in Tampa for 2 yrs and suddendly she is no longer in practice which cause me to be hospitilize on 10/15 for 4 days at Fawcett Memorial Hospital because of the extreme pain i had, and I also thought I was having another heart attack which ended up being a very bad anxiety attack, Im looking for a dr which can help me with my pain which I have a dr. that has referred me to pain management. The problem is no dr can see me until the end of Dec 2013 or Jan. 2014. What can I do? Doctor: HIThank for asking to HCM I really understand your pain but no need to worry it will going to alright soon first thing you need to do is just relax bring your confidence back, say that it will be alright keep your moral very high, this pain wont hurt you any more, I would advise you to start the corticosteroids start with 40 mg thrice in day and then tapered it gradually to 10 mg once in day within 12 days, you will get good result I would be happy to help you for further question or advise have nice day."
},
{
"id": 211675,
"tgt": "PTSD diagnosed, get disturbed by hearing past incidents of wife. Reason?",
"src": "Patient: I was diagnosed with PTSD upon returning to the states from Afghanistan. Can my wife be a factor by bringing up her past constantly. It drives me nuts and I am going to Loma Linda VA in CA. for one and one treatment but she drives me nuts about her past, I am thinking about seperating from her for a while! I want to get better for all of us, or is the wife exonerrated from this torment-these feelings I get and I wish I was back in Afghanistan Doctor: In fact, contrary to your expectation, talking about the issues that gave rise to PTSD, should help overcoming it. But, it should no be talked about all the time. It should be discussed half an hour daily one to two times. rest of the time, you mind other business.The fact that your wife does keep on speaking about this might indicate that she also has been perhaps traumatised, in the inner psychologial sense.Therefore, I would suggest both of you, to seek a psychiatrist / therapist who might put you on medication, too. This helpsSeparation is not the solution and might worsen the issues, in such cases"
},
{
"id": 116384,
"tgt": "Suggest treatment for low hemoglobin value",
"src": "Patient: My brother recently suffered from Loose Motion for which he took some medicine...post taking the medicine he did not have constipation for the 25 days. I finally took him to a physician who diagnosed pancytopenia...as of now his condition is much better however his haemoglobin is 5.9, WBC is 1.7, Platelet is 45K..Please suggest Doctor: HIWell come to HCMI really appreciate your concern, if this is iron deficiency anemia then this need to be treated with iron supplements drugs and iron containing diet, and the best option would be leafy vegetable, meat, some dry fruits, hope this information helps, every thing would be fine soon, take care."
},
{
"id": 36864,
"tgt": "What is the cause for sore throat after a typhoid fever?",
"src": "Patient: i felt giddiness, i was diagnosed typhoid, but i had no fever at that time.I got fever five days back. treated with oral medication for 5 days. i didnot get fever or giddiness again. doctor discontinue the medicines but i still feel ill, tired and i also have sore throat. is it normal or i am feeling ill becoz of sore throat. Doctor: HelloThank You for contacting HCM.Welcome to Health Care Magic.My name is Dr Muhammad Ahmad & i will look into your problem.I have gone through your query and would try to help you in the best possible way.Course of Typhoid: It's an infection of GUT Even if diagnosed early and treated well with best antibiotics it take 7-10 days to get cured ,some times it can also go to chronic stage which may not be symptomatic but bug stays there. Cough ,sore throat: Having Dry cough and rarely cough with sputum is a common sequellae of Typhoid feveer and patients often report it. Is feeling Ill normal: Yes almost hundred percent of patients report feeling ill ,lethargic and weak even after their treatment has been completed its because of high grade of fever which frequently have occured during course of illness and low diet and heavy duty meds given to patients. What to do: (1) Complete course of meds 2- Make sure infection is no more there by follow up exam and tests 3- Take good rest and food 4- In a month or so you will start getting to feel normal. Hope this answers your question. If you have additional questions or follow up questions then please do not hesitate in writing to us. Wishing you good health"
},
{
"id": 222772,
"tgt": "Is it considered safe to conceive while taking a malaria drug?",
"src": "Patient: Hi I am going to the Dominican Republic and have been advised to take Chloroquine. I want to conceive asap and have now been told i need to wait until after my course of malaria tablets. Is this right? A pharmacist told me it would be fine and im not sure which one to beleive! Websites give mixed reviews. If I have to wait, how long after i take my last tablet is safe to conceive? Doctor: Hi.There are no controlled studies to report and prove any serious contraindications towards taking chloroquine during pregnancy, but from what I had read somewhere, some teratogenic abnormalities were reported in one such case. Wait for a week after your course is complete and then start trying to conceive.Best wishes."
},
{
"id": 130603,
"tgt": "How to cure a muscle spasm?",
"src": "Patient: Hello, I took a bad fall on my back 48 hours ago. Didn't get to er for 10 hours. Emergency room diagnosed muscle spasms and prescribed 500 mg robaxin up to 3 x a day for pain with up to 8 ibupropgin/day. Worked well for 1day and now I feel no difference at all when I take it. Pain is getting worse. Pretty difficult to get out of chair or bed, unable to lift of open anything with right hand, pain/pressure on deep breathing, and coughing. Should I go back to er? Doctor: Hi,There is no need to hurry. Wait for one more day to see if body is responding to medication. Also you need to take rest. Try taking hot packs on the painful areas with each setting for upto 10 min even after doing this pain doesn't go away then you need to go and visit ER and explain him that last 3 days. I don't have any difference in my pain and would like to be checked up again for any fracture or other injury.Hope you find this helpful. Let me know if I can assist you further.Regards,Jenis Bhalavat"
},
{
"id": 64156,
"tgt": "What causes lump on the head?",
"src": "Patient: Hi Doc, Am 34 yr woman recently found while combing that there is a small ball like (lump) on the right side of my head (exactly on top right edge), begining i never felt it but now while i press it, it gives a soft pain...can anyone please help me to know what exacatly it is? Doctor: Hi,Dear ,Good Evening.Thanks for you query to HCM.This is Dr.Savaskar from India helping you out of your health query. I studied your query in-depth and understood your health concerns.In My opinion the your head lumps-is caused-as follows--You have infected scalp follicle with boil formation,which is felt lately by you -as it has grown over the right of the head-scalp on top right edge.Treatment - By your family doctor with antibiotics / anti-inflammatory for 3-5 days time would reduce and resolve it completely.Else a ER surgeon would advise proper surgical-drainage of the grown up boil.-Hope this would help you to plan treatment with your ER Surgeon and other TEAM doctors.-Wellcome for further queries for this issue to HCM.Wishing a fast recovery for you.Have a Good day.RegardsDr.Savaskar M.N.M.S.Genl-CVTSSuper -Specialist in Non-Curable Chronic Diseases(NCCD)and Expert in therapy for-Cancer/Asthma/Psoriasis / Autoimmune disorders/Rejuvenation protocols for tissue and organ failures- etc."
},
{
"id": 164916,
"tgt": "What causes continuous diarrhea and vomiting in children?",
"src": "Patient: Hi, may I answer your health queries right now ? Please type your que my 3 year ols daughter started having diareah and vomiting a week ago. she had the vomiting for 2 days and still had water stools. she went 2 days without vomiting then started again and had been for the past 3 days. im really concerned about dehydration, maybe she has salminella or maybe she swallowed a small magnet. please help! Doctor: Hi, thank you for the question. your child is suffering from gastroenteritis. we can treat it with probiotics, ORS , zinc, ondansetron for vomiting and an antibiotic like cefixime if indicated. you can avoid dehydration by giving your child ORS, coconut water in small sips."
},
{
"id": 109571,
"tgt": "What causes back pain?",
"src": "Patient: i have diffuse disc bulge at L5 S1 and dessicated disc.... i have been having back pain for the past 5-6 years now recently the pain increased and its giving me problem in my left leg. I now have problem even while turning in bed. I ve taken physiotherapy before..now tat is also not helping I am a 30 year old unmarried female. please help Doctor: Hello,Well, you have a disc problem. L5 S1 is most commonly involved area and symptoms like severe pain in the back, and this pain also radiates to the leg because the nerve supply is from here. You have spent six years with this condition and have disturbed your quality of life. It surgically treated before.No medical treatment will help you in reducing pain. The more you are late, the more it will affect your quality of life. Do an MRI scan now. My advice to you is that it is now time to go to the neurosurgeon. So that he can assess you, and consult you about the treatment option.Hope I have answered your query. Let me know if I can assist you further.Regards, Dr. Nizar Ali"
},
{
"id": 130609,
"tgt": "Suggest medication for fibromyalgia and neck pain",
"src": "Patient: I would like to know why I become nauseas and dizzy with severe tension in my neck and shoulders. Sometimes this happens even when the tension in those areas are not at their worst. I have to go and lay down. I have fibromyalgia and alot of neck pain. Could it be pinched nerve. Doctor: Hi,It actually could be a nervous injury, and don't worry it is treated by some NSAIDS like Advil tabs, Acetaminophen, hot fomentations with topical gels is helpful. Muscle relaxants are highly recommended in addition to some physiotherapy or even yoga is effective. Dietary strategies to keep in mind:- Eat fruits and vegetables. - Drink plenty of water.- Eat more plants than meat.- Reduce your intake of sugar.- Get regular exercise as best as you can.- Lose weight or maintain a healthy weight. Hope the above information helps you. Any further clarifications feel free to ask.Regards,Dr. Ahmed Aly Hassan"
},
{
"id": 133406,
"tgt": "What causes pain in the heel?",
"src": "Patient: For the past five days I ve had a bad pain in the left heel...the inside rear of the heel. It seems to be really bad in the morning and at night, when I m barefoot. I should tell you I m a former dance major and have been active as a dancer and dance teacher for most of my life. I don t remember having injured my foot, so I have no idea what this is. Doctor: Hello there,This complaint of heel pain indicates that you might be suffering from plantar fasciitis.Treatment comprises of if you are wearing high heelsfoot wear ,change it to soft chappals ,change of activities as you are dancer you might need rest for couple of weeks, anti inflammatory medicine.Get a consultation with orthopaedician ,you might also need an xray of foot for evaluation.Thank you"
},
{
"id": 221878,
"tgt": "What are the effects of having high glucose levels during pregnancy?",
"src": "Patient: Hello Dr, I\u2019m 33.5 years old, I\u2019m 152.5 and weigh 47kgs. LMP 16/5/11 and due date 26/2/12. I found out I was pregnant at 6 weeks about 4 weeks ago, I went to my GP at 7 weeks to get a few routine tests done and I had a ramdon test of high glucose reading of 8.7, I found out at week 8. Then the doctor requested I do the 1 hour one where I drank some glucose and waited around for an hour, it came back with 8.9. Then I was sent to do another test where I had to fast and it came back even worst, I couldn\u2019t remember what the first initial reading was but the second reading was 12 and the third reading was around there as well, I\u2019m now 10 weeks and found this out just Monday. I had a routine body check two years ago and my sugar levels were within the range. I\u2019m changing my diet now to low GI food and will see a dietician tomorrow 4/8/11. I\u2019ve read from a lot of sources where a high glucose reading the in first trimester will increase the changes of birth defects, stillbirth and premature baby. I\u2019ve read everywhere that the chances are increased. So what are the changes of having a healthy baby by controlling my diet starting now? I will be doing a nuchal translucency test on the 18th August and blood test to check the neck of the baby to see if there is risk of down syndrome, my OB also suggested that if it comes back bad, then he would recommend amniocentesis. I will be 34 when the baby is born, I want the baby to be in the best health, I\u2019m really scared if in the end of the 9 months it\u2019s a stillbirth. Would I even consider abortion at this stage and start again and get my sugars controlled and then try to conceive again? I know I should wait to do all those tests, but I know those tests don\u2019t pick up all the birth defects. I\u2019m impatient and I\u2019m getting old, I don\u2019t want to lose the change of trying to conceive for another baby. Scared Doctor: HiDr. Purushottam welcomes you to HCM virtual clinic!Thanks for consulting at my virtual clinic. I have carefully gone through your case, and I think I have understood your concern. I will try to address your medical concerns and would suggest you the best of the available treatment options.1] Please do not panic.and do not worry.2] I will suggest indulging in a healthy diet and regular exercise regime.3] Include plenty of fruits, salads, vegetables in the diet. 4] Avoid deep fried foods, bakery products, and refined sugars.5] please do not entertain any negative news and thoughts in your mind.6] Please follow the USG and blood tests as suggested by your doctor.Step by step testing of nuchal translucency, Double marker and triple marker test, amniocentesis if needed are the rightly advised tests.7] I will suggest you to focus on your diet and exercise and on sugar control.8] All things will sail smoothly.Wish you be blessed with healthy child.I hope my answer helps you.Thanks.Wish you great health."
},
{
"id": 58745,
"tgt": "Hepatomegaly, hepatosplenomegaly positive. Low hemoglobin, High serum Ferritin, low platelet count. Swollen lymphs. Sugar levels low. Further treatment?",
"src": "Patient: Reports indicated Hapatomegaly+++, and Hepatosplenomegaly positive.Low hemoglobin - 6.7, High sr.Ferritin - 644, and low platelet count - 69,000. No heart disease, No Cholestrol abnormal levels. There are Swollen Throat lymphs. Patient is losing her conscious for few minuts. Her sugar levels are low. Please advise for further investigation and treatment. Doctor: Hi and welcome to Healthcare magic.Thank you for the query.She neds to find the exact cause of liver damage and do liver biopsy if necessary. these are serious symptoms and many causes can be considered. Most common are infections, cirrhosis or viral hepatitis. Some metabolic diseases too. She needs to follow hepatoprotecgtive diet till symptoms subside and consider splenectomy if low patelet count persists.Wish you good health. Regards"
},
{
"id": 71811,
"tgt": "What causes upper chest pain?",
"src": "Patient: I have upper left chest pain since last Monday which I thought was due to muscular strain but has got worse.No fever pulse and bolo pressure normal as recorded by dentist during normal check up last Tuesday Took aspirin to dull pain Tuesday night and tonight.Plan to see my local doctor this morning. Am a male aged 85 in good health Doctor: Thanks for your question on Healthcare Magic.I can understand your concern. In my opinion, you should definitely rule out heart diseases for your left sided chest pain. So consult cardiologist and get done blood pressure monitoring, ecg, 2d echo and stress test (trade mill test).You are may need angiography if any of the above test is showing positive results for ischemia. Better not to wait and meet cardiologist as soon as possible. Hope I have solved your query. I will be happy to help you further. Wish you good health. Thanks."
},
{
"id": 218751,
"tgt": "Trying to get pregnant. Noticed brownish discharge on and off for few days. Implantation bleeding?",
"src": "Patient: hi I have been trying to get pregnant and my LMP was 5/26/13 and normally I have regular periods and I was supposed to have my period come on the 26th, however on the 20th I had bleeding which was pink to dark pink lasting few hours with brownish discharge on and off for few days and nothing now. I was wondering if this could be implantation bleeding or what. Doctor: Hello,First of all, you need to undergo one blood test for beta-hCG and TVS scan to rule out implantation bleeding. Other reasons are pelvic/ vaginal infection, old endometrial tissue, hormonal imbalance, uterine pathology etc.Following initial test reports, you consult with gynecologist to proceed next line investigation after clinical examination. Take suggestive measures accordingly.Maintain genital hygiene properly. Take healthy diet with folic acid supplement.Good luck."
},
{
"id": 185096,
"tgt": "What causes irritation and tastelessness to the tongue?",
"src": "Patient: I WAS AT A RESTURANT WITH A FRIEND HE ASKED ME TO TASTE HIS ICE TEA. IT TASTED LIKE DETERGENT OR SOME KIND OF CHEMICAL . HIS WIFE THINKS IT WAS INDUSTRIAL DISH SANITIZER, IT MADE MY TONGUE IRRATATED 7 SWOLLEN. WENT TO DOCTOR'S THEY GAVE ME PRETIZONE & A STEROID SHOT. I LOST SOME TASTE ABILITY WILL THIS EVER GO AWAY? Doctor: Hello,Severe chemical damage to the surface of the tongue where there are taste buds can be permanent. You were given anti-inflammatory medication, eliminated the chemical and now must keep the area clean for healing. Are you having difficulty healing with any ulceations? You need to watch for an additional oral infection such as thrush. Anti-fungal medication such as Nystatin would be necessary. Antibiotics could be required due to complicated healing. Gentle oral care along with warm salt water rinses will assist and soothe the tongue. Keep well hydrated and proper nutrition. Avoid vitamin deficiencies. Spicy and acdic foods may cause irritation. I suggest you have your dentist evaluate and monitor the surface of the tongue.I think you can have a positive outlook and realize with more time, more taste sensation should return. Your exposure sounds limited. Knowing the exact chemical would be helpful. Ruling out an allergic reaction would also be suggested in the case of a flavored tea, etc.Thank you for your inquiry. I am glad to provide additional assistance if you have more questions."
},
{
"id": 150207,
"tgt": "Dizziness, hot flashes, headaches, trembling sensation, lost consciousness, not a seizure. What is it?",
"src": "Patient: Hi, I am a 19 year old female. Yesterday, I was shopping and around 3:00 pm suddenly got very, very hot and dizzy and fell unconscious. If my Mother hadn't been with me, I would have slammed my head through a glass table. I was unconscious for about 45 seconds. Since then, I have felt dizzy, had a headache, felt very hot, and my hands have been trembling. It's really worrying me. I have a history of epilepsy, but I am fairly certain it wasn't a seizure, I am very familiar with the aura I get before a seizure, and this was completely different. (Ps- there's no chance of pregnancy.) Doctor: Hello,Thanks for the queryThis is not a seizure and it can happen to normal people and it is called syncope. When we stand for long and/or tensed some time the blood supply to the brain can decrease transiently and can have this type of manifestation. Just relaxBest wishes"
},
{
"id": 167575,
"tgt": "What causes allergic reaction to cow s milk and egg white in a 8 month old?",
"src": "Patient: baby of 8 months with allergies to cow s milk, egg white, egg yolk and possibly salmon. Has had skin problems since birth..severe cradle cap, dermatitis. Could she be allergic to spring greens? She suffered an all over body rash after eating a very small amount of spring greens... Doctor: cows milk protein allergy is a well known disease your childs history seems she/he s prone to allergens I would like to asked some questions whether anyone in your family have allergic histories or else family history of asthma family history of allergy also inherited to a childthe best management option is to identify the causes and avoid that food items in future because first exposure doesn't cause serious problem but second exposure does"
},
{
"id": 214037,
"tgt": "I always feel tired and not fresh in the morning",
"src": "Patient: I always feel tired and not fresh in the morning? What is the solution? Doctor: Dear Nisar, Tiredness and un-refreshing mornings could be due to lot of reasons like thyroid problem, stress in life, sleep problems and illnesses, problems with breathing called sleep apnea etc----all physical reasons or it could be due to psychological problems like depression, mania etc. Please consult a general physician first and if he rules out physical causes, then a psychiatric consultation MAY help. Good Luck"
},
{
"id": 87290,
"tgt": "Suggest medication for intense stomach pain & nausea",
"src": "Patient: I have been really sick for the past three or so days with varying symptoms. The first day I had cold-like symptoms - achy arms, headache, cough. That night after eating dinner I got intense stomach pains and ended up vomiting a little and having intense diarrhea several times. The next day the stomach pains were gone but I had an intense headache all day, as well as coughing and sneezing. That night I kept waking up every hour or so because the pain in my head was so intense. Then yesterday I again had an intense headache with mild coughing and sneezing and now nausea. Gradually my headache became isolated to the left side of my head only. I have also had two intense nosebleeds in the past 24 hours. Doctor: Hi.Thanks for your query.Your history of flu like symptoms, abdominal pain, vomiting, nausea, headache which has isolate to the left side and nose bleed is indicative of the following:Typhoid - one gets the similar problems with nose bleed. Or Flu with secondary infection of the GI tract and Upper respiratory tract. I would advise you the following:Get the vitals and general examination done by the Doctor.Investigations of the blood, urine , stool.CT scan of the Brain and sinuses to rule out any internal problem causing nose bleed and left sided headache.The treatment will be symptomatic to start with like an antibiotic, topical and oral decongestants, painkiller for headache.The specific treatment may be continuation of the same or changed as per the reports."
},
{
"id": 197922,
"tgt": "What does the sperm analysis report indicate regarding fertility issues?",
"src": "Patient: Dear Doctor,i am 37 year old male (recently married ) would like to know about male infertility. i am a chain smoker and this habit begun in the age 20. i heard that the person who has smoking habit, whose sperm count will be low and difficult to parenting. so i conducted a Sperm Anylisis Test and its report given below. please check this report and give your valuable suggestions if any deficiency found in this report.Anticipating your early replywith Regards,DKPSperm Analysis Test ReportTime Collected\u00a0\u00a0\u00a0\u00a0\u00a0: 6.30 PM, Time Received: 6.35 PM, Liquefaction Time: 30 MinutesVolume\u00a0\u00a0\u00a0\u00a0\u00a0\u00a0\u00a0\u00a0\u00a0\u00a0: \u00a0\u00a0\u00a0\u00a0\u00a03.2 mlColor \u00a0\u00a0\u00a0\u00a0\u00a0\u00a0\u00a0\u00a0\u00a0\u00a0: \u00a0\u00a0\u00a0\u00a0\u00a0Pale YellowpH \u00a0\u00a0\u00a0\u00a0\u00a0\u00a0\u00a0\u00a0\u00a0\u00a0\u00a0\u00a0\u00a0\u00a0\u00a0: \u00a0\u00a0\u00a0\u00a0\u00a0Alkaline Odor \u00a0\u00a0\u00a0\u00a0\u00a0\u00a0\u00a0\u00a0\u00a0\u00a0:\u00a0\u00a0\u00a0\u00a0\u00a0Musty Total Sperm Count \u00a0\u00a0\u00a0\u00a0\u00a0: 73.6 Millions / mlMotility Progressive \u00a0\u00a0\u00a0\u00a0\u00a050% \u00a0\u00a0\u00a0\u00a0\u00a0 36.80 Millions / ml Sluggish \u00a0\u00a0\u00a0\u00a0\u00a0 15% \u00a0\u00a0\u00a0\u00a0\u00a0 11.04 Millions / ml Non-Motile \u00a0\u00a0\u00a0\u00a0\u00a035% \u00a0\u00a0\u00a0\u00a0\u00a0 25.76 Millions / mlMorphology (%) \uf076\u00a0\u00a0\u00a0\u00a0\u00a075 % Normal Sperm\uf076\u00a0\u00a0\u00a0\u00a0\u00a025% Abnormal SpermRound Head \u00a0\u00a0\u00a0\u00a0\u00a0\u00a0\u00a0\u00a0\u00a0\u00a005% \u00a0\u00a0\u00a0\u00a0\u00a0\u00a0\u00a0\u00a0\u00a0\u00a0Long Tail \u00a0\u00a0\u00a0\u00a0\u00a005% \u00a0\u00a0\u00a0\u00a0\u00a0Squamous Head \u00a0\u00a0\u00a0\u00a0\u00a005%\u00a0\u00a0\u00a0\u00a0\u00a0\u00a0\u00a0\u00a0\u00a0\u00a0Short Tail \u00a0\u00a0\u00a0\u00a0\u00a005%Swollen Head \u00a0\u00a0\u00a0\u00a0\u00a0\u00a0\u00a0\u00a0\u00a0\u00a005%Others\uf076\u00a0\u00a0\u00a0\u00a0\u00a0PUS Cells \u00a0\u00a0\u00a0\u00a0\u00a0\u00a0\u00a0\u00a0\u00a0\u00a0\u00a0\u00a0\u00a0\u00a0\u00a0: \u00a0\u00a0\u00a0\u00a0\u00a06 \u2013 8 / HPE\uf076\u00a0\u00a0\u00a0\u00a0\u00a0Sperm Agglutination \u00a0\u00a0\u00a0\u00a0\u00a0: \u00a0\u00a0\u00a0\u00a0\u00a0Few Doctor: HelloThanks for your query and uploading report of your semen analysis ,Based on the report that you have posted I would analyse the report as follow 1) Viscosity ,Ph .Liqifaction time and volume of the semen are normal except colour of the semen(Yellow) which suggests infection 2) Sperm count of 73 millions /ml is also normal in comparison to WHO standards .3) Percentage of active progressive sperms 50% is normal for effective fertilization of female egg to result in to pregnancy 4) Morphology of the sperms is also normal Microscopic finding of presence of 6-8 pus cells and sperm agglutination is suggestive of infection of either seminal vesicle or prostate and needs to be treated by taking antibiotics like Doxicycline for one month .Repeat your semen analysis after completion of medication .To summarize I would state that there is no reason why you should not impregnate your wife .Dr.Patil."
},
{
"id": 190665,
"tgt": "Recurring mouth ulcers. On tonact-tg, vitamin-E, folic acid for high cholesterol, triglycerides. What is the cause?",
"src": "Patient: I am 47 years old.just two months before i had my lipid profile done and my cholestrol and triglycerids both were high.so i was advised tonact-tg for two months with vitamin e tablet also and miltivitamins too.I am pure veg.I also do exercise ( gym/ yoga ) thrice four times a week.When ever I have time I also go for long walks.But now i am facing problems with mouth ulcers frequently in very mild form.I take folic acid tablets too.But it gets cure and comes back again.So is this due to tonact or some other problem.Please advice some remedial measures.Shall I do my tests again.hope for the answer soon. Doctor: Hello, There are different types of mouth ulcers that can occur in different physical conditions. Practically higher cholesterol and triglycerides does not directly relates to any type of mouth ulcer. The anti-lipid drug that has been prescribed to you is also not related to formation of any mouth ulcers. But in suppressed physical condition some different mouth ulcers can occur. It is extremely difficult to diagnose such ulcers without seeing them. Your history of occurrence \"gets cure and comes back\" shows that the ulcer is most probably Recurrent Apthus ulcer. A picture of that ulcer will confirm the diagnosis. If the ulcer is painful you can use Zytee or Mucopain cream 3 to 4 times a day to reduce pain. The actual treatment of mouth ulcers can only be done after confirming the diagnosis. For that you have to contact some local dentist( preferably a specialist in oral medicine whom you can found in nearby Dental College or hospital) or send your details here including photograph of the lesion. Stay well. Thank you"
},
{
"id": 35431,
"tgt": "What causes fever,hot and cold flashes and vomiting?",
"src": "Patient: Hi, my friend has been really sick lately and has been experiencing, Hot and Cold Flashes, Cold sweats, Dizziness, Fever, Vomiting, a little bit of coughing with a little sore throat he is also experiencing light headed-ness, is there anything you can tell me from those symptoms? Any help would be appreciated Doctor: Hi,From history it seems that he is having viral infection giving all these symptoms.Give him symptomatic treatment for each symptom.Give him Ondem for vomiting.Give him analgesic medicine for pain, fever and malaise.Give him plenty of liquids.Give him complete rest.Ok and take care."
},
{
"id": 67565,
"tgt": "What causes lump towards adam s apple?",
"src": "Patient: Hello, For a couple of days now every time I yawn really big while my heads is Looking Forward I feel like there is a lump or something there, to the right of my Adams apple, then I feel like I need to cough. When I yawn and my head is Looking Down I do not feel anything. I guess the pain started on the right side of my lower jaw line and it seemed to work its way down to where it is now. Any suggestions? I do believe I do have bad acid reflux since I always have gas and keep burping all the time while an incredible burning comes back up to my throat. My chest burns and feels like someone is standing on it sometimes. Doctor: Hi! Good evening. I am Dr Shareef answering your query. Even though it needs a physical examination of the neck region to reach at a conclusion, from the history it seems to be a cervical lymph node swelling or a thyroid related swelling if it is seen and felt on the outside of neck while yawning. If I were your doctor, after a general physical examination with special attention to the head and neck region, I would advise you for some investigations like a complete blood count including an ESR, and a thyroid function test. Also an FNAC (fine needle aspiration cytology) of the lump and an x ray chest if need be could be helpful. Considering the symptoms of GERD (acid reflux) for a long time, I would not hesitate to refer you to a gastro enterologist for an endoscopic evaluation of your GI tract to rule out any organic cause of your feeling in the throat. In addition, you could do the following for getting some symptomatic relief from the symptoms of GERD :1) Elevate the head end of the bed with two blocks on each side, so that the regurgitation does not occur while lying down, and the contents of esophagus and stomach flow down with gravity.2) Some modification in your diet like more of roughage, enough liquids, avoidance of oily, fried, and fast food, and complete abstinence from alcohol and smoking if you do.3) Regular routine exercises like morning walk depending on your medical fitness.4) Prescription for a prokinetic agent and a proton pump inhibitor for symptomatic relief and to have a control on the acid production and facilitate movement of gastric contents downwards.Further management would depend on the reports of investigations.I hope this information would help you in discussing with your family physician/treating doctor in further management of your problem. Please do not hesitate to ask in case of any further doubts.Thanks for choosing health care magic to clear doubts on your health problems. I wish you an early recovery. Dr Shareef."
},
{
"id": 201026,
"tgt": "Suggest treatment for spots under foreskin of penis",
"src": "Patient: I think I may have Fordyce spots. They are diagnosed as warts. I have pictures. They are under my foreskin. I have pictures. Under normal magnification... they look like the spots but when you zoom in they look a bit suspicious. My there was one and I tried to pop it. A few days later there were much smaller ones around it. The one I tried to pop has gotten smaller. But it may have changed color. I really dont think they are warts. I also tested positive for herpies 2, but have never been symptomatic... and have only given and received oral sex since the last time I tested negative. Please help Doctor: Thanks for asking in healthcaremagic forumIn short: Sebaceous glands visible/your Fordyce spots are harmless and you need not worry about this. If you are not having vesicle with pain then it is not herpes. So, stop looking these lesions under magnification and if you are not convinced you may visit dermatologist for your problem. Good luck"
},
{
"id": 106990,
"tgt": "What causes severe lower back pain?",
"src": "Patient: I am a 58-year-old female in good health. Woke up fine this morning and then I had sudden onset of L lower back pain -waist level and slightly below. There is no radiating pain into my LE. With twisting pain almost feels like it is taking my breath away. What should I be concerned of. Doctor: welcome to hcm ..as there is no pain radiation ..it need not worry much ..it is just muscular spasm ..you need muscular relaxant ..get in touch with local doctor he will prescribe some muscle relaxant"
},
{
"id": 114057,
"tgt": "Constant feeling nausea for 11 weeks with back pain",
"src": "Patient: Hello, I am a 14 year old girl and been in and out of hospial in the past 11 weeks 3times, I hav been having propblem emptying my bowels so in on movicol an I am now still feeling sick constantly, and I have really ba back/sine pains and nothing takes it away.... I have no health problems I m normally fit and well, what could be causing this?I Doctor: Hello, Thanks for your query. you did not mention clearly why were you hospitalised for so many times. Was it due to continous fever or some other complaints. What was the main complaint. Get your complete blood count done, since anemia also presents vaguely. If all investigations were normal and still if you have fever for more than 3 weeks it is called pyrexia of unknown origin. Dont simply rely on medicines for constipation, try with fibre rich foods. Hope I have answered your query. I will be available for your followup queries if any. Wishing you good health. Regards,your query."
},
{
"id": 110868,
"tgt": "Is it safe to drive and climb steps for a lumbar spondylosis patient?",
"src": "Patient: Sir, I have been diagnosed as lumbar spondylosis. Problem is with L-5 and I had severe pain in the lower bottom for two days. Immediately consulted a Sr.Ortho and was hospitalised for pelvic treatment. I did undergo physiotherapy and traction for six days. Now the pain is considerably subsided. I have some queries regarding the dos and donts. 1. Is two wheeler riding considered a total ban in my case. I am wearing a belt on doctors recommendation. 2.Is climbing steps a ban in my case. I am working as Administrative Officer in an organisation. I used to carry my office bag in the shoulder. Is it allowed in my case. 3. I used to walk for about three miles in a day. For the last one month I am not going for a walk fearing that the pain may get aggravated. Can I walk. SATHISH KUMAR Doctor: two wheeler is considerably ban in your case reason is simple in bike we always are trunk forward and potholes or speed breaker can increase your pain and nerve compressionclimbing steps is ok but considering you lookout your posture.. too much stooping while climbing can cause paini would not suggest 3 miles a day... you can walk a mile or so if you wantabove all first get yourself relieved from pain via exercises with physiotherapist. then this precautions will helps for sure. for further queries you can contact anytime"
},
{
"id": 77057,
"tgt": "What causes shooting pain in chest and left arm?",
"src": "Patient: I have chronic pancreatitis and diabetes, for over a year now on and off every few months I get chest pains that shoot down the left arm. I am pretty used to pain suffering 4 a cute pancreas attacks. I would rate the chest pain as a 10. I has a stress test on the heart a year ago and the test was borderline for angina. They made me Waite for a secon opinion and the decision was it wasn't angina. I have been back to the gP and they are just dismissing it saying I had the test. I am getting pretty scared Doctor: Hi thanks for contacting HCM...Do you have gall stone?? Are you alcohol drinking ???Because these are two condition in which repeated acute pancreatitis can occur...So by through detailed history and investigation cause of acute pancreatitis searched for...And if possible treated accordingly....Meanwhile during acute pancreatitis ryles intubation might need to rest pancrease.It can lead reffered chest pain...While pain ECG can done to rule out angina.If angina present sorbitrate taken immediately.Take low fat diet and avoid alcohol....Take care..."
},
{
"id": 180689,
"tgt": "How can severe pain after tooth extraction be treated?",
"src": "Patient: Hi I had two teeth extracted last week last the first one top left and fine no pain the second the following day (Monday) was awful the dentist wasn't very pleasant on our first meeting and he'd cancelled my appointment on the Monday knowing the pain I was in so I called up the same day and got an appointment half an hour before my original with the same dentist, it seemed to me as he didn't want to do the extraction he said if I have a drink it could come out of my nose which I found unusual and many other things it took him 25 mins to get the tooth out and he kept taking breaks about half an hour after I'd left the dentist the pain I got was crippling I was screaming in pain and it wouldn't stop I had to go back to get another injection which seemed to help a little bit the pain was awful the whole was huge and it's been a week today and the side of my tooth the one next to the tooth that was removed I push my younger up and the gum is spilt in half it pulls away from the tooth is this normal and I have what looks like a long price of tooth comming out the side of my gum Doctor: Hello,If it is a week, the gum must have got attached to the tooth and also the pain must have reduced. You should consult an oral surgeon to get checked it there is any dry socket formation. Also, you should get an x-ray done to check if the tooth that is pulled has removed completely or is it still having some portion left in the extraction socket that is causing pain. For now, you should take soft meals, do warm saline gargles and take painkillers as and when necessary.Hope I have answered your query. Let me know if I can assist you further.Regards, Dr. Honey Arora"
},
{
"id": 5393,
"tgt": "Have irregular period. On Gynaecosidev. Was pregnant. Had premature delivery. Did folliculometry. Small follicles. Treatment?",
"src": "Patient: I am Larry, Age= 27+, Weight= 60kg, I have an irregular period history and I use Gynaecoside and Confidence before to correct the cycle, and it was quite regular.I am now married and I was pregnant the second month of my marriage but I have a premature rupture of membrane and a premature delivery at about 25 weeks and lost my daughter in December, 2012. Since then I've been trying to conceive and did folliculometry this month, it revealed that my follicles are of smaller size compared to the normal sizes. Pls help Doctor: HELLO YOU HAVE A HISTORY OF IRREGULAR CYCLES AND MIDPREGNANCY LOSS. FIRST OF ALL YOU HAVE TO FIND OUT THE ETIOLOGY BEFORE PLANNING FOR CONCEPTION. YOU SEEM TO BE OVERWEIGHT. CONCENTRATE ON WEIGHT REDUCTION ACCORDING TO YOUR HEIGHT. SCREEN FOR THE INFECTIONS THAT CAUSE PREGNANCY LOSS AND SHORTENED CERVICAL LENGTH. REGULARISE YOUR CYCLES IF STILL IRREGULAR. OVULATION INDUCTION AND HORMONAL INJECTIONS CAN BRING OUT A SUCCESFUL FOLLICULAR SIZE. GOODLUCK FOR A GOOD OUTCOME."
},
{
"id": 46726,
"tgt": "How to treat HCV,Quantitative real time PCR-338487 IU/ml with Genome Type-I?",
"src": "Patient: My brother is undergoing hemodialysis twice a week for last 8 months. He is diagonised Hepatitis C Viral( HCV), Quantitatitative Real Time PCR- 338487 IU/ml with Genome Type-I. He is planning to undergo Kidney Transplatation. please suggest treatment. Thanks R.N Bhargava Doctor: Hi..First of all don't be tense. Luckily now a days very effective oral medication is available. For genotype 1 your doctor will prescribe ledipasavir/sofsobuvir medication. Start medication as early as possible. Go for transplant after completion of course which is generally 12-24 weeks. All the best. if you have any questions feel free to contact me."
},
{
"id": 89079,
"tgt": "How to treat stomach ulcers?",
"src": "Patient: hai,, i need to make three test for my husband,,who is 35 yrs old.. with throat, stomach and gall bladder? with throat he had an operation 5 yrs before.. now when ever he keeps taking continously,,, he has pain.. his stomach has mild ulcer problems.. and there s a tiny 1mm polypolip in his gall bladder,, not grown since two yrs.. but we wish to get a complete health check up? Doctor: Hi ! Good morning. I am Dr Shareef answering your query.If I were your doctor, I would advise your husband for a planned and elective removal of gall bladder to avoid any complication out of the polyp in it.For the mild ulcer problem, I would advise him not to eat food from outside specially spicy, oily and fried food if he does. In addition to this I would advise him with a proton pump inhibitor along with a prokinetic agent which would help healing of his stomach ulcer.I would advise an ENT review for the problem in his throat.For a complete health check up, many hospitals run some programmes, and you could be in touch with any of them nearby you.I hope this information would help you in discussing with your family physician/treating doctor in further management of your husband's problem. Please do not hesitate to ask in case of any further doubts.Thanks for choosing health care magic to clear doubts on your health problems. I wish you an early recovery. Dr Shareef."
},
{
"id": 182202,
"tgt": "Suggest treatment for tooth abscess and lump under the eye",
"src": "Patient: Hello, This started 3 days ago and to me it seems like an absess bottom tooth because I can feel a round ballish (lymph node) type thing under it in my neck, and it hurts when I touch it. I don t have a toothache though. Since I first started with this I have to marks on my mouth near my lips (Look like bug bite marks) that are swollen and my lip is swollen and kind of numb. Yesterday I woke up with a red lump under my eye that s puffy (Like got punched in the eye) and the whole time I have a headache over my eye. All this is taking place on the right side of my body. I also feel like have the flu and very run down. I should probably go to hospital to get checked out but no insurance. I probably have a bad infection and pills will cur it I hope but figure would get your opinion. Thank you Bill Doctor: It is surely a periapical abscess. Lymph nodes are painfull due to infection, lymphadenitis. Fever is also due to this infection. Consult a doctor for appropriate antibiotics and antipyretics."
},
{
"id": 51212,
"tgt": "Calculus in left kidney, spread to vesico-ureteric junction. Surgical removal required?",
"src": "Patient: Hi Dr.. My wife (32 years) was diagnosed with a calculus in her left kidney last year. After a few painful episodes, the same calculus (6 mm) is now shown to be in the vesico-ureteric junction in USG KUB. She has had a urine infection, and currently on antibiotics . My question is does this type of calculus pass naturally, or should we go for surgical removal without any delay? Thanks Doctor: Hi Mr Raja Considering the size and location, your wife stands a good chance of passing it naturally. The problem is pain and time. Occasionally, it may obstruct - apart from size, surrounding swelling caused by its irritation may be responsible. Removal straight away is is the best option. It is not 'surgery' in the true sense of the word - a procedure! They put a tube from below (Ureteroscope) and extract the stone. It is fast but expensive... Drink plenty of water - doing nothing but drinking water and passing 'water' may hasten the process Wishing her a speedy recovery Good luck"
},
{
"id": 68529,
"tgt": "What does neck lumps imply?",
"src": "Patient: Hi Dr. Good afternoon, I am a survivor of Squmousa cell carcinoma of the tongue two years back. Recently there are two neck lumps visible and MRI scans confirms the enlargements. Is it a sign of metastates??? What should I do further to confirm it? Doctor: Welcome to Health care magic.1. Possibilities of metastases can not be ruled out ( in your case) with out an FNAC - fine needle aspiration cytology of an enlarged lymph nodes.2.Generally lymph nodal enlargement is seen after local or systemic, infection or inflammatory cause. And ease off after treatment.3.You have done MRI - it would have given you the status of the old treated tongue malignancy. A computer tomography with contrast will help to see the status of previous malignancy.4.Get an ultrasound abdomen done - to see the status of liver and abdominal lymph nodal enlargement.5.Get an FNAC of lymph node for confirmation and treatment to commence. Good luck.Hope it helps you. Wish you a good health.Anything to ask ? do not hesitate. Thank you."
},
{
"id": 62178,
"tgt": "How to remove bump inside the butt cheek?",
"src": "Patient: HelloI am 50 years old female, have marble size hard bump inside of butt cheek. I went to the dermatologist got antibiotics, a steroid shot. Still there will not pop. My dermatologist is now telling me to see a gynocologistCan they remove this...it seems she does not want to deal with it Doctor: hi.based from your description, it could be a cystic lesion (possibly a sebaceous or a keratinous type). it could also be a fibroma or a lipoma. it is best if you consult with a physician, preferably a general surgeon, for clinical evaluation. management (medical and/or surgical) will be directed accordingly.hope this helps.good day!!~dr.kaye"
},
{
"id": 42980,
"tgt": "Have low sperm count, will taking clomid make him sterile?",
"src": "Patient: Hi. My husband wants to start taking clomid. We are trying for a baby. His count is very low already, we are seeing reproductive specialist and a urologist. Hopes are low. Ivf prob only way. My question is will clomid make him totally sterile? Please tell me ty Doctor: Hi,Clomid does not make him sterile. Dont worry. Get him evaluated for the cause of low sperm count by ab andrologist.Regards"
},
{
"id": 95200,
"tgt": "What is the cause for my severe one sided abdominal pain that moves to my thighs ?",
"src": "Patient: I am a 33 yr old female .I have lower right sider abdominal pain which goes toward my inner thie since last 4-5 month also same sided back pain .month ago my laproscopicappendectomy is done but the pain haven t stopped infact increased(my abd ct andsonography and bld test are normal) pl. advize..it is kin of very much of tension coming on my nerves also my stomch feel very heavy and give me pain more when i m standing and walking although it is continuous even after taking pain killer .some relief i there only when i lie down. also i have motions 3-4 time a day i m really worried and feel like misguided .(what is this exactly ibs, or nerve or anything else)I would like to add some more points which according to me must be considered as I don\u2019t want to keep a single point to be neglected, yes this is my life and I have to be really concerned about it at least now looking into the past history. I had a small accident before 2 yrs back where I was collapsed from nearly 2ft high platform unknowingly ,that time I had ligament rupture in my right foot and ankle after 2 month of rest and physiotherapy I started walking gradually. I am really worried and want to seriously look into not only for present issue but for the future. Doctor: dear snehal first of all better consult your laproscopic surgeon who had done operation of your appendix to rule out no issues with that secondly as per your symptoms this could be chronic dysentery where pain and stool are very common and never diagnosed under any sonography better go for your stool examination about the accident if that accident injured your abdominal organs then and only then you need to worry about that accident otherwise just forget it one more thing i need to clear out is just check your urine if there is any burning or pain or hot feeling while passing the urine is there then it could be stone inside your kidney or ureter or bladder which might be very tiny in size so could not rule out during USG better go with urine and stool examination and come to us with the reports bye tc hope you got your query solved for more queries visit - www.homeotouch.com mail- drpremalshah@gmail.com call- +91 98254 44736"
},
{
"id": 222220,
"tgt": "Suggest possible tests to confirm pregnancy",
"src": "Patient: Hi, I just had my period from the 10th to the 15th. I had sex with my boyfriend protectively on 16th and unprotected on the 18th. On the 20th I went to get the morning after pill and the nurse told me I was pregnant. I told a ultrasounds today and nothing is there. Am I pregrant? I need help!!!! Doctor: Hallow Dear,There is a lot of misconception.Firstly, if you had menses from 10th to 15th, 18th is the day 8 of your cycle. The fertile window usually lies between day 10 to day 20 of the cycle (see below). So the chances of becoming pregnant are very remote. Moreover, it is impossible for the nurse to diagnose a pregnancy on day 10 of the cycle after menses. The egg is yet to be released. The egg is released (ovulation) 14 days before the next expected menses. Egg lives for 24 hours and sperms are active for 72 hours. Hence a period of about 8 to 10 days around this day of ovulation is considered as Fertile window. So on 20th your ovulation also had not taken place. How can you become pregnant? No test for pregnancy will reveal pregnancy as it is not there. Please wait for the menses. If you have few more intercourse in the fertile window, you will have chances to conceive and then 8-10 days after the missed period, your urine test for pregnancy will detect it. Of course, Beta hCG tests can give reliable results within 2-3 days of missed period. Best Wishes,Dr. Nishikant Shrotri"
},
{
"id": 84098,
"tgt": "What are the side effects of taking steroids?",
"src": "Patient: Hi My mother is on wysolon steroids since January 2012.The doctor has given her 60mg dose in the starting of the treatment for dysthyroid and now presently she is taking 20mg every alternate day.I would request you to please let me know the side effects of this streoid so that she can take precautions. Doctor: Hi,Steroids on long-term use can cause stomach ulcer, high blood sugar, high blood pressure, weight gain, thinning of the bones which may result in fractures, cataract, thinning of the skin, mood changes, increased risk of infection, and acute adrenal crisis if stopped abruptly. She must report to her doctor if she experiences these side effects.Hope I have answered your question. Let me know if I can assist you further. Regards, Dr. Mohammed Taher Ali, General & Family Physician"
},
{
"id": 23055,
"tgt": "What causes pain in chest and palpitations even while on BP medications?",
"src": "Patient: Hello, I am very concerned and confused I suffer from hypertension I an on three different meds for it twice a day my doctor saids i am fine yet I get dizzy sometimes and my heart when lying down pumps so hard the my chest hurts after a while, my heart rate while resting sometimes goes up to 104 and higher. What is going on? Does anyone know? Doctor: HIdizziness can be because of hypotension caused by anti htn medications , next time when you feel the same take your blood pressure from automatic machine, your dr needs to lowe the doses of the medicines hypotension causes tachycardia hence fast hear rate is because of lowering of blood pressure by anti htn medicines"
},
{
"id": 73300,
"tgt": "Can exposure to fuel vapours cause heavy chest and difficult breathing?",
"src": "Patient: I just had one of our technicians go to the hosptial - felt like someone was sitting on his chest and it was hard to breathe. the 17' Center Console boat he was working on does have a fuel leak, so he may have been around some fuel vapors a little bit. Doctor: Hello dear , hiWarm welcome to Healthcaremagic.comI have evaluated your query thoroughly .* The fuel vapors that obstruct the airway may be prime reason for the heavy chest and breathlessness .Hope this clears your query .Welcome for any further assistance .Regards dear take care ."
},
{
"id": 158386,
"tgt": "Lumps in lower back, offset from spine getting bigger and harder. U/S reposts normal. Is it Carcinoid cancer?",
"src": "Patient: My wife has found a few lumps in her lower back just offset from her spine on each side about 3-4 days ago, each day they have gotten bigger and harder. At first they would move slightly when pushed on but are going rock hard now. She had an ultra sound done yesterday but didn t seem to be too worried and was told they couldn t tell what they where. I am worried as her gp thinks she has carcinoid cancer or syndrome but won t do an MRI or ct scan. Please guide me as what we should do now! Doctor: Hi there, Are you sure its carcinoid? They usually arise in gut or lung and may be you mean to say carcinoma? Kindly rediscuss with your GP and confirm if there are some miscommunications. You can always take a second opinion with a surgeon if you are worried. Take care"
},
{
"id": 129189,
"tgt": "Suggest treatment for patchy haziness in ribs",
"src": "Patient: my medical examination in a GAMCA medical center is under process. they are repeating my X-rays. it is suggested by the doctor that there is no evidence of present or past disease but there is a patchy haziness on the ribs in upper right portion of chest which most most probably is due to ribs/bone injury. is there any treatment to remove those scars from the ribs? Please help me out as last chance of X-ray repetition remains. Doctor: Hello, thank for your question, your patchy haziness is due to trauma, x ray showing sign of callus formation, it is a good sign of bony healing, take calcium regularly and pain killer like diclofinac...thank you"
},
{
"id": 103849,
"tgt": "Itching on the body, rashes after eating expired food. Treatment?",
"src": "Patient: I ate a bunch of packaged cookies recently and I found out today that the expiration date of those cookies was 5 months ago. I slept through the night while itching myself constantly, and when I woke up this morning, I developed rashes in the areas where I scratched myself.. How do I make this rash go away as fast as possible? Doctor: helloAs you have told that you have rash over the body with itching and the reaction was delayed after eating expire foodthis reaction is known as allergic reaction type 1the best treatment will be to vomit it out , it its late you will have reaction till the food in inside your body and will finish when you will go to washroomtake a bath will cool down the itchingget any antihistamine like loratadine, diphenyhydramine (you can get this over the counter) will ease your lifewear loose clothesif severe get steroid injection from ERhope this helpstake care"
},
{
"id": 133448,
"tgt": "Need treatment for knee pain",
"src": "Patient: Hi I am just trying to work out what s wrong with my knee. A couple of weeks ago I was in my garden doing gardening, and since then I m not sure if I did something as I really cannot remember or aware if I did something but I m really struggling when I climb stairs. It s like this pain on my left knee on the left hand side of the knee at the top and as I take a step up, it s either just really sore with this shooting pain or sometimes depending on the angle I can almost just buckle over with the sudden sharp pain. It s not sore walking, lying in bed or anything else, just when I climb the stairs or bend down and try to stand up with that knee Doctor: hithank-you for providing the brief history of you.a thorough clinical examination is needed.As the sharp shooting pain will be indicative of the degenerative disc disease but may be less common as well which can be ruled out with clinical assessment.Also, this may be a ligament injury for which we may need an MRI of the knee to understand which knee ligaments are involved. I can suspect this more as there is pain while addressing full joint weight bearing. Also, post MRI and examination, in my clinical practice most cases are referred to physcial therapy and they respond well.RegardsJay Indravadan Patel"
},
{
"id": 46744,
"tgt": "Suggest treatments for kidney stones and hyperuricemia",
"src": "Patient: My mother is suffering from hyperuricemia and now she has two stones in each kidney.Her physician has prescribed her anti biotics and anti inflammatory drugs . kindly guide me does she have to proceed with anti biotics and what should be the best treatment Doctor: Hello and welcome to HCM.As an Urologist,i can understand your anxiety.She doesn't need any anti-biotics. You should write her age and medication.Also,size of the stones and the scan reports should've been sent.1. She needs Allopurinol or Febuxostat,only if her s. uric acid is above 7.2. She should drink about 12-15 glasses of fluids daily.3. Magnesium ammonium phosphate with B6, tablets twice daily is needed.If you've any doubts,send the reports to me,as a direct question.Dr.Matthew J. Mangat."
},
{
"id": 49975,
"tgt": "Discomfort in kidney area, feels like swollen, pain under both ribs. What could be this?",
"src": "Patient: Hi, I'm a 47 yr old female, experiencing discomfort on both sides, kidney area. All I can say is it feels like they are swollen. I ache in the front, right under both ribs, through to the back. No pain, just this chronic ache in this region. I don't have any insurance, so I've been putting off seeing a Doctor. This discomfort I've been expeciecing now on and off has been about a year in duration. Doctor: there are chances of gall bladder or urinary bladder or other organ of abdomen involvedyou need to diagnose it by ultrasound abdomen in which you will get confirmation of wahich organ is involved and you need to consult gastroenterologist to treat accordingly to the organ involved"
},
{
"id": 92395,
"tgt": "What might have gone wrong with results normal but having stomach bloated and pain in butt hole and pelvic bones?",
"src": "Patient: Hello I'm a 15 year old female and ive been having a constant beating in my lower abdomen. Im not sexually active and I have been to the doctor about this. For the past few months ive been having trouble pooping. I couldnt poop for 2 weeks straight at one point and took a enima and citromagnesium. Without taking both combined very little comes out. I'm gettin scared because it feels like something is moving inside of me. It hurts when I put pressure on my stomach or lay down with my chest to the ceiling. Whem my mom took me to urgent care we did the enima and citromagnesium the day before and I feel like that could have messed up the results. The results were nothing. Please help if you know anything. My stomach is getting bigger and bigger and there is a pain in my butt hole and my pelvic bones. Thank you Doctor: HI Thank for choosing HCMI can understand your problem, you do not have any disease, only thing need is to change your certain habit, like you have to go for defecation even if you do not have such reflexes but still you go in and sit there for at least 30 to 40 minuet do not apply any force during this time, just relax and take your time, next, drink lot of water even without feeling thirsty, have fibre diet stop taking medicine for constipation, try isabgol powder and very important, away from the stress, anxiety, depression, eat well sleep well, try all these, every thing will be fine soon good luck."
},
{
"id": 74838,
"tgt": "Suggest remedy for pain in left chest",
"src": "Patient: I am diabetic from last 5 years, some time i feel pain in the left chest but it last for only few minutes. i am taking medicin of glimmet 500 mg two tablets in a day. Also my burb is smelling too bad especially when i eat non veg. can u pls tell me the reason Doctor: Hellothanks for posting at HCM. Since your are diabetic, we must rule out a cardiac cause of this chest discomfort. I recommend an ECG, 2 d echo and a treadmill test. This the diagnostic modalities will give sufficient information about the heart function, previous damage to the heart, any leak between valves and can also predict the possibility of cardiac event in the near future. Also since you have got bad breath all this can also be connected to hyperacidity in the stomach. I recommend a course of antacids twice a day for 2 weeks. Please revert back with the above tests.wishing you good health. Regards."
},
{
"id": 209054,
"tgt": "Suggest treatment for panic attacks",
"src": "Patient: I became a widow on December 29, 2014 after 46 years of marriage. I suffer from panic attackes everynight. Waking up around 3:00 every morning in the middle of these attacks. I find it very hard to cry, is this my body s way of handling the grief or is this a more serious problem?l Doctor: HiThanks for using healthcare magicI think, you have panic disorder and that would be due to stress or grief reaction. In that case, you need antidepressant with low dose benzodiazepine. That would help you to control underline anxiety as well as keep you relax. You can also try some relaxation exercise like JPMR or deep breathing exercise. If you need further help, you can ask.Thanks"
},
{
"id": 64492,
"tgt": "What causes hard lump and pain in shoulder and arm?",
"src": "Patient: a hard lump after mastactomy with arm and shoulder pain what is it going for regular phisiotharapy and pain and pull in arm towards shoulder arm can put straight now visible hard kind of lump plese help alredy had surgery 1and half month back. Thank you Doctor: Hi,Dear,Good afternoon.Thanks for the query to my HCM online- clinic .I studied your query in-depth.I can understand your concerns for a recurrence after a surgery 1.5 mths back.-In my opinion the hard lump post mastectomy -is mostly - local-node-metastasis or could be recurrence of the breast tumour.I would advise -Pet scan with FDG-18 F-radiotracer which pinpoints lymphnodes mets , which may miss on CAT scan.FDG PET guided-FNAC Biopsy would fix the recurrence of cancer in local infra-clavicular-left out-nodes.-Alert-A review with Onco-Surgeon for the steps to be taken with his advise is a must and urgently advisable in your case.-Hope this would resolve your worry-some query.Wellcome again to HCM .Have a good day."
},
{
"id": 175272,
"tgt": "What causes frequent vomiting?",
"src": "Patient: My 11 month old is vomiting almost after every meal since yesterday afternoon. She acts normal before and after. Her last throw up had really dark brown stuff in it that I have no idea what it is. I m nursing her and she ll drink, but not very much. We are giving her pedia light as well. She doesn t have a fever or swollen tummy. What is wrong with her? Doctor: HI...by what you quote I feel that the kid is having GER (gastroesophageal reflux). Your daughter will need - 1. Antireflux therapy - Proton pump inhibitors + antiemetics2. Always make her lie down in a propped up position - as in the care seat. She should not be sleeping completely flatly.3. Avoid heavy feeding and feeding more times and in small aliquots.4. This type of GER is the root cause for the constipation she is having.She needs expert gastroenterology consultation.Regards - Dr. Sumanth"
},
{
"id": 134395,
"tgt": "What causes pain in buttocks and instability of leg after hip cupping?",
"src": "Patient: My bottoms hurts yet I don t exercise and on the skinny side, my legs don t hurt but I m very unstable on my feet. One more thing, I m seeing double of everything. Any help would be appreciated And I m 35. Thank you.my name is crystal, one last thing, I ve been hip cupping since all this pain has started. Thanx again! Doctor: Hi Crystalbeing unstable on your feet and seeing double of everything is very concerning and calls for a thorough medical examination to determine the cause of these symptoms which can be related to brain or brainstem problems. your bottoms pain can be alleviated with some over the counter pain medication but you need to see a doctor for your other symptoms"
},
{
"id": 175366,
"tgt": "What are the side effects of Xanax 2mg in a child?",
"src": "Patient: My 2 ur old grandson put a 2 mg tablet of Xanax in his mouth, he spit out most of it but there was residue of it in his mouth. I made him throw up, then gave him milk. He seems fine, it s been an hour and a half, it s way past his bedtime and seems sleepy. Doctor: Some of the most common side effects of Xanax (Alprazolam) included: drowsiness, fatigue, and dizziness, and disorientation, mental retardation and motor reactions.In individual cases, you can observe side effects in the digestive system as well as heartburn, nausea, diarrhea, vomiting, stomach pain. Its normal reaction that he started to sleep after xanax, but you should give him charkol 2 tab for absorbtion of this medicine 2- 3 days"
},
{
"id": 191465,
"tgt": "What causes muscle pain, dizziness and breathlessness?",
"src": "Patient: I can not lean forward to put on socks or tie shoes without dizziness and shortness of breath - also my muscles ache and lock up doing this - foot pain is extreme sharp jolts for days at time & then stops for days - blood sugar is normal - stress test 5 years ago was normal - can not exercise anymore - feel like will pass out just walking - getting worse and has been going on for several years - also retaining fluid - lower legs swollen daily Doctor: Welcome to Healthcare Magic, Muscle pain , dizziness, breathlessness and swelling in legs should be taken seriously . In such features I will suggest urine complete. After the tests, visit a general physician or come back to Healthcare Magic, I will help you . If you find my answer helpful then please mark my answer as helpful . With regards, Dr Varinder Joshi"
},
{
"id": 219478,
"tgt": "Will R Vac injection help to treat rubella igG during pregnancy?",
"src": "Patient: Hi i had abortion due to subchronic HGE of placenta..i have PCOD and i concieved by fertility treatment..my torch test shows high level of rubella igG and CMV igG..my doctor has given now r vac injection..i want to know whether this injection is proper treatment for rubella igG as my rubella igM is normal Doctor: Hi.R vac is a vaccine recommend to prevent rubella and is the right approach for now, there is nothing more your doctor can do at this point, so have faith and trust in him/her that they will do everything that is required at the right time.Best wishes."
},
{
"id": 84018,
"tgt": "Can the Metpure/Calaptin medication be stopped when BP is 90/60?",
"src": "Patient: Dear Sir, I had my MVR(star edward) in July 1998, followed by 2 TIA in 2004. Now I am on Dytor Plus, Metpure XL 25 and Calaptin 120 for last 3-4 years. For last a year or so my BP is 90/60 though I am feeling well. Shall I stop/reduce Metpure/Calaptin? I am 32 years old. Best Regards/Ravish Doctor: Hi,Advised dose adjustment. Based on the history you seem to have developed low blood pressure apparently due to a combination of various BP lowering medications. Consult your doctor for either stopping dytor plus or dose reduction of prescribed medications to control your BP.Hope I have answered your question. Let me know if I can assist you further. Regards, Dr. Mohammed Taher Ali, General & Family Physician"
},
{
"id": 145361,
"tgt": "Suggest treatment for a lumps on the base of the skull",
"src": "Patient: Hello there and thank for your time to read this. I m personally not sure how to ask a doctor about my situation. At the base of my skull I have lumps which have grown larger of the past few years. These lumps have be causing serious migraines which are unbearable. The pain will start from there location and travel around my head to my eyes. Sometimes during the migraines parts of my arm, face or leg will jerk and twitch. I also have other lumps as well one on my left forearm and left buttock. Any help would be appreciated. Thank you. Doctor: Hello. I have been through your question and understand your concern.I am not sure lumps can cause migraine, though you might need the appropriate treatment for migraine. Since you have different lumps you should consider getting seen by a dermatologist to rule out any genetic syndrome like neurofibromatosis.Hope this helps.Please feel free using MHC again"
},
{
"id": 127001,
"tgt": "What can cause pain from the ribcage to the shoulder blade?",
"src": "Patient: Hello, I have been having b/l rib pain which also goes into the base of each shoulder blade; also at times in between the shoulder blades. The rib pain has been going on for 2 months. Fatigue, too. Chest xrays indicated COPD, but I have not had any respiratory symptoms. No history of asthma, bronchitis, or emphysema. I have never smoked or worked at a worksite where I would have been exposed to substances which inflame the airways in the lungs. PFT is normal. Any ideas? What are my next steps? Thank you, Valerie Doctor: Hi The pain might be due to costochondritis or muscular cause and may not be related to COPD. As a first line of management you can take analgesics like Ibuprofen or Diclofenac for pain relief. If symptoms persist, better to consult a neurologist and get evaluated. Hope I have answered your query. Let me know if I can assist you further."
},
{
"id": 127810,
"tgt": "Sharp stabbing pain just below left breast and ribs with numbness and constipation",
"src": "Patient: Im having sharp stabbing pain just below my left breast and ribs. The pain in quite intense. The area is also numb to the touch. I have issues with constipation which is what everyone seems to think is causing it, however even when Im regulated it still hurts. I take miralax and sometimes milk of magnesia and it works for me. So obviously constipation isn t causing this. What else might it be and what kind of doctor should I see about it? Doctor: Hello,In my opinion, it would be wise to go see a cardiologist as chest pain on left side radiating to shoulder and increasing on exertion can be a heart symptom.Hope I have answered your query. Let me know if I can assist you further.Regards,Dr. Fahim Sheik"
},
{
"id": 12065,
"tgt": "How to get rid of black pigmentation?",
"src": "Patient: hello doctor my name is Anjana . I am from Bhopal M.P. I have black pigmentation ( JHAIN ) on both my cheek . Kindly suggest a solution . Doctor: hi you have melasma apply SHADOWS 30 sunscreen twice a day -apply TRIGLOW cream at night avoid sunexposure"
},
{
"id": 2277,
"tgt": "Will ovarian cysts cause problems in getting pregnant and suggest treatment?",
"src": "Patient: infertility problemsI been trying to concieve for two years nothing never happens...I once had a cyst removed off one of my ovaries..is there any fertility pills I can buy over the counter that will help me concieve??..do you think ovarian cyst is stoping me from getting pregnant...can somebody please help me????...it hurts to see other womens having kids with no problem Doctor: Hi.The cyst does indicate a possibility for or has led to a hormonal imbalance. Please understand that such issues cannot be treated with over the counter medication. Pregnancy and conception is no joke and so although your problem is minor, with proper guidance and treatment you will surely be able to conceive soon. A thorough work up followed by control of cycles along with the initiation of medication like clomiphene can help your conceive soon.Best wishes."
},
{
"id": 73907,
"tgt": "What causes acute breathing difficulties?",
"src": "Patient: hi doctor! I'm 23 years old.. i was suffering from breathing difficulty for 4-5 years. i went to ENT specialist. he suggest me the nose bone straightening surgery.. i have undergone the surgery on 7th jan 2011. but still my breathing difficulty continuing.. breathing through mouth.. i went to same doctor again and told the problem but he gave me some medicines to follow... but no use of that also...im very depressed...what should i do further? pls help me out to overcome this difficulty... pls Doctor: Thanks for your question on Healthcare Magic.I can understand your concern.Acute breathing difficulties can be due to acute bronchitis or asthma.So better to consult pulmonologist and get done clinical examination of respiratory system, chest x ray and PFT (Pulmonary Function Test).You may need inhaled bronchodilators (formoterol or salmeterol) and inhaled corticosteroid (ICS) (budesonide or fluticasone).Do warm water gargles and steam inhalation 4-5 times a day.Don't worry, you will be alright.Hope I have solved your query. I will be happy to help you further. Wish you good health. Thanks."
},
{
"id": 167710,
"tgt": "What causes cluster of bumps inside the buttocks of a child?",
"src": "Patient: I have a 2 yr old that has a cluster of bumps on the inside of his butt cheeks. They were itching him last night so I put AD ointment on them. He has not scratched or complained about them today. They don t appear to filled with any fluid. Any ideas? Doctor: if this diper rash is of a small size , red and sometimes can give a small blood tinged diper it's usually a contact Dermatitis and A&d ointment will protect the skin and moisturize it and will do the trick .but if these rashes have white satellite like slits , it's usually a candidal skin infection due to increased humidity at this area and candizole cream can treat them .if this rash is red raised papules that are very tender to touch then it could be a staph infection and fuscidin cream will be appropriate .I hope this helps"
},
{
"id": 125892,
"tgt": "What causes dizziness after taking St.John s Wort?",
"src": "Patient: I started taking St Johns Wort a week ago but I was only taking one a day for a few days . Then I started taking one twice a day until yesterday I started one 3 times a day . I noticed because I did a couple times take two at once that it made me dizzy ... My question is If I continue taking one 3 times a day for how long do I continue taking St Johns Wort? Doctor: Hello, It might be due to first dose effect. As of now drink plenty of water and take rest. If symptoms persist, better to consult a physician and get evaluated. Hope I have answered your query. Let me know if I can assist you further. Take care Regards, Dr Shinas Hussain, General & Family Physician"
},
{
"id": 115789,
"tgt": "Why are my WBCs and platelet count abnormal?",
"src": "Patient: Hi, i've been nauseas everyday for over 3 months now and i vomit sometimes too. I went to the doctor and they did blood work and my WBC was low i think 35.5 and My doctor found 2 swollen lymph nodes on the left side of my neck and one in my grion so she put me on amoxicilin and nothing happened and i got blood work done again and my WBC was normal but my platelets were high and they said my swollen lymph nodes and my neusea were too different issues should i be worried about something serious like cancer? Doctor: Hello and welcome to HCM,WBC count of 35.5 is high and not low.High WBC count is commonly seen in cases of infection.High WBC count along with lymph node enlargement further potentates presence of infection.Thus, you need to consult your doctor to look for focus of infection.The enlarged lymph node also needs to be investigated.An aspiration cytology of the lymph node needs to be done wherein cells are aspirated with a needle, spread on slide and stained with appropriate stains.The stained cells are studied to know the site of origin and nature of the lesion.Thanks and take careDr Shailja P Wahal"
},
{
"id": 34586,
"tgt": "Suggest remedy for severe itching and broken skin after being treated for staff infection",
"src": "Patient: I HAVE HAD STAFF INFECTION 10 YEARS AGO . COME BACK EVERY 2 TO 4 YEARS I THING IT COULD BE SOMETHING ELSE. WENT TO DOCTER GAVE ME ANTIBIOTICS CLAERED THE BAD AREA BUT SKIN BROKE OUT EVERY WHERE. WENT BACK TO DOCTOR SAID SEE DERMATALOGIST CAN NOT HELP. I AM UNINSURED THEY CHARGED ME, TO TELL ME THAT NEED ADVICE CAN YOU HELP MY ITCHING IS TERRIBLE NOW ON BENADRYL. HELP.... Doctor: Thanks for posting you query to health care magic.broaken skin is a predisposing factor for infection .As you are having only ithching goes in favour of some allergic etiology or could be due to some fungal infection .antibiotic will not be helpful .I need some more information about your complain to suggest you correct medication : is there any pigmentation , is itching generalized to whole body or only limited to lesion area only, is there any red pathches develop on ithching are , how ithching are looks like .after getting complete information i would suggest you correct medicin.presently you can use moisturizer to keep skin moist and calamine lotion to releive itching and Cetrizine 10 mg tablet once in a day for sever ithching under guidence of clinician .hope you would be satisfied with my answer . feel free to communicate if any query .regards,Dr.Manish PurohitInfectious disease specialist"
},
{
"id": 83014,
"tgt": "Using cremalax. Have blood sugar, high bp, thyroid preoblems. Any harm on prolonged use?",
"src": "Patient: i am using Cremalax (Sodium picosulphate) 10 mg tablets for the last four months, to relieve the acute difficulty in evacuating morrning stool. Does the prolong use of this medicine have any side effects. I am also suffering from High B.P.' Blood sugar and thyroid problems, but they are under control after medication. Kindly advice. Doctor: it is not advisable to take these medicines long term as they increase your dependance on them and gradually they may stop working all together or you may have to increase the dose of the medicine in order to get the desired result. i suggest it would be better to take natural supplements like isabgol and high fiber flax seeds.take plenty of water and fiber rich foods like salads and raw vegetables."
},
{
"id": 131534,
"tgt": "What could cause shoulder pain and numbness, have Tendonitis and Carpel tunnel?",
"src": "Patient: Hi I have carpual tunne (sp) l in both my wrists and tendinitous (sp) in both my elbows.. Now for about three weeks my right shoulder has been feeling like its bruised and is sending tingling sensation and numbness all the way down to my finger tips.. I went and saw a doctor she said it probably is tendinitous of my shoulder. But I cant stand the pain and hurting anymore. I called for a EMG and am waiting but that is for my carpual tunnel (sp). I am just tired of my hands being numb and tingly all the time. I have a one year son who I need to hold and care for and I can barely pick him up anymore.. I feel that I should go get seen and get an x-ray that is how bad it feels... Thank you Doctor: HiYou may take lyrica 75mg 1 BD,Ace -proxyvon 1BD, use arm sling, apply voltaren gel to shoulder after hot fomentation.For carpal tunnel, early decompression surgery must be done.consult orthopaedic surgeon for prescription meds if he agrees to suggestions and a hand surgeon for release surgery of carpal tunnel"
},
{
"id": 100164,
"tgt": "How can throat itching, acne causing skin holes and redness be treated?",
"src": "Patient: Hello,i'm 18 years old. i'm allergic from fruit and vegetable ,since last year ,i drink fresh juice, and i feel my throat itching me ,and comes up small acnes ,and gave my skin small holes on my cheeks,after the small acnes move, and my lips get red ,when i eat fresh fruit.... what can i do?! Doctor: Well...you are allergic to the fruit and vegetables..the symptoms you are having are due to allergic reaction.Here is some advice for you.Avoid these food.For the allergic reactions you can use antihistamines like cetrizine(generic name) once a day in the evening.your symptoms will be subsided then."
},
{
"id": 196548,
"tgt": "Is there any treatment other than surgery for gynocamestia?",
"src": "Patient: Hi I am 23 years old I am suffering from gynocamestia. Is there any treatment other than surgery that i can follow ? I had medicine for TB like five years back for about whole one year and i think it might be the side effects of the medicine i took as the condition is almost from that time. Please do recommend some medicine as i cant afford surgery for it. Kind regards Doctor: Dear sir, the only way to get your breasts without surgery is body building so that muscles will replace the fat content of your breasts and they will look better."
},
{
"id": 72945,
"tgt": "Are Rimactazid, Benadon, Mycobutal and PZA(1500 mg) correct medicine for tuberculosis?",
"src": "Patient: Hello, i have taken afb smear and afb culture and doctor diagnosed that i have tuberculosis in the lungs. i have been provided with rimactazid, benadon, mycobutal and pza(1500 mg) are these the correct medicines. how long am i supposed to take thse medicines. also doctor advised me to take liver function test after a month. Will i get any serious side effects since i am taking heavy dosages of medicines. i am 54 kgs. wanted to know wheter after taking thse medicines tb will be cured completely ? Doctor: Hello dear , HiWelcome to Healthcaremagic.comI have evaluated your query thoroughly .* Guidelines pertaining to your query - Your current medicine regimen is perfect as per your body weight and WHO guidelines for treatment of TB .- Liver function test is mandatory to get done at certain time intervals to check liver function when you are taking TB medicines .- These are not heavy doses , it is must to get cured , under medical supervision , it is not going to cause any serious side effects .- TB will be cured completely provided you finish all the doses as per advise of your doctor .Hope this clears your query .Wishing you fine recovery .Regards dear take care ."
},
{
"id": 13631,
"tgt": "Suggest remedies for itchy rash on the crease of my arm",
"src": "Patient: i have what i believe is a rash in the crease of my arm where you give/take blood from. it itches and is in a ring/circular shape with bumps. my skin there other than the bumps is smooth. it feels like it itches from the inside . i think it may be an allergy .. something im allergic to . maybe the black soap or detergent. i stopped using the black soap Doctor: Hi, As per your descriptions, the rash could be a Tinea infection (Ringworm). The possibilities for the rash being an allergic rash is also there. So, I recommend you to visit your Dermatologist to undergo proper examination and to initiate the treatment. Hope I have answered your query. Let me know if I can assist you further."
},
{
"id": 53166,
"tgt": "Suggest treatment for SGPT 57 and SGOT 63.80",
"src": "Patient: hello doctor my sister went to doctor for low fever she had tests with reports indicatingesr 85 rbc 307alkaline phosphatase iu/l 334.20s.g.p.t (alt) 57.70s.g.o.t(asp) 63.80 doctor said it is lever infection and gave antibiotics,can u please let me know is it more worrying thing Doctor: Hi.Thanks for posting query at HCM.antibiotic maybe prescribed for any other underlying infection or as prophylaxis. Usually ALT or AST values higher than \"two times the upper normal limit\", is considered abnormal ( in some countries, ALT or AST values of more than 100 are considered abnormal). Test should be done to rule out HBV,HCV.any further questions are welcomed.hope to answer your concern.wish you good health.regards,Dr Tayyab Malik"
},
{
"id": 79189,
"tgt": "Is it safe to take the medicine with empty stomach?",
"src": "Patient: Hi, I have been diagnosed with Kochs Effusion (pleural fluid in envelope of my left lung). 400ml of fluid has been taken out of my left lung but doctor says 1 ltr is still there. Also, I have gone through TB Gold test and Skin test and both have shown positive results. I am going to start my medication from tomorrow. I am a bit scared to take 2 Forex coated tablets empty stomach as it can be too heavy for me (my weight is 41 kgs, age 26 yrs)...please suggest if it's safe to take these tablets empty stomach everyday..Many thanks in advance for your help. Doctor: Thanks for your question on Health Care Magic. I can understand your concern. Forecox is having anti tubercular drugs. And it is best to take them on empty stomach in morning. These drugs show excellent therapeutic effect if taken on empty stomach. But at the same time these drugs are known to cause gastritis, nausea and vomiting. So in my opinion, you should take these drugs on empty stomach. If you are not developing side effects like nausea, vomiting, gastritis etc than continue taking them empty stomach. But if you develop sever gastritis and vomiting than take them after light breakfast or after proton pump inhibitors (antacid). Hope I have solved your query. Wish you good health. Thanks."
},
{
"id": 54934,
"tgt": "What causes elevated bilerubin levels?",
"src": "Patient: My brother-in-law is currently hospitalized because of pneumonia. He is 74 years old and when he entered the hospital his blood pressure and oxygen levels were very low. The doctors have also discovered elevated levels of bilirubin (the whites of his eyes are yellowish) and uncertain so far of the cause. What problems could cause this billirubin to remain elevated? Doctor: Hi thanks for asking question....Here mostly in your case it seems to be pneumonia infection with hemolysis...It can occur in some pneumonia infection like mycoplasma ....So hemolytic anemia can lead to elevated bilirubin level...For its detection serum LDH, retic count and PS examination helpful....Gradually with recovery from pneumonia bilirubin will comes to normal.Meanwhile take less fatty diet....Fruits taken more....If still bilirubin elevated then USG done...Take care....Dr.Parth"
},
{
"id": 197744,
"tgt": "How to get over masturbation ?",
"src": "Patient: Hi, Im 36 year old male. I have been trying to have a non masturbating relationship, but Im failing. When we dont have sex for a long time weeks I get discomfort and like semen drips. It almost like a short micro orgasm and I cant stand the feeling and the only way to get rid of the sensation is sex/ masturbation? Doctor: Dear, We understand your concernsI went through your details. Dripping of semen while you do not have sexual intercourse of masturbation is quit common. This happens because of a normal body process. Whenever you are sexually excited your body produces semen and it needs to be threw out. Body throws the semen through drippings and nocturnal emissions. Nothing to worry. Take care.If you require more of my help in this aspect, please use this URL. http://goo.gl/aYW2pR. Make sure that you include every minute detail possible. Hope this answers your query. Further clarifications are welcome.Good luck. Take care."
},
{
"id": 173680,
"tgt": "Suggest treatment for hole in heart and lung",
"src": "Patient: Dear Dr.Shetty, I am from bangladesh.my daughter she is only six month,but she have a problem in her heat and lungs.in sqare hospital in dhaka Dr.Abdus salam said there is a hole into her heat and lungs. please i want to contact you for better treatment. ATIQ YYYY@YYYY Doctor: HelloThere can be various type of heart defect and management depends on the type and location of hole.Whether it is VSD/ASD or complex heart disease.I request you to attach the report so that I better able to help you out.I can guide you whether he needs surgery or can be treated by medication only.Regards"
},
{
"id": 202458,
"tgt": "Itchy pinkish red bumps on the penis, mild burning on urination post unprotected sex. Partner negative for STD. Should I still go for a test?",
"src": "Patient: I am 17 and two days ago I had unprotected sex. I did not take a shower after and instead waited until the next morning. That night I noticed I had pinkish red bumps or a rash on the head of my penis . The girl I had sex with has been checked and has no STDs. I am uncircumcised I don t know if that matters of not. I have only experinced mild to no itching and a very slight burning sensation while I urinate. For the most part the almost feel as if they are not there. I am a little worried and wonder if I should consult a doctor or visit a STD clinic. I am asking here first to try and avoid an awkward conversation and visit. Thanks for your help, x Doctor: i think you need to visit, your partner being negative for STD doesnot mean any infection cant happen to you. it may be a simple fungal or bacterial infection. oral sex or not proper cleaning of the area might have caused. dont worry visit a dermatologist."
},
{
"id": 205121,
"tgt": "How can depression be treated?",
"src": "Patient: i am on effexor but is is not helping my depression at the moment -- works for awhile and then I m off and depressed again. Doctor want me to add lithium and the mixture scares me with all the side effects. Is there any other think you would suggest without knowing my history of depression. Doctor: Dear userWe understand your concernsI went through your details. Depression is a neuro chemical disorder and your treatment includes the maintenance of hormone / chemical level in your brain which helps in maintaining proper neural activity. Therefore do not fight with your depression. Learn to live with it for a cure. Your doctor is a capable person to treat you. Please trust him.If you require more of my help in this aspect, please use this URL. XXXX. Make sure that you include every minute details possible. Hope this answers your query. Available for further clarifications.Good luck."
},
{
"id": 187068,
"tgt": "What causes pain in gums after a root canal?",
"src": "Patient: I had some pain in my back molar a few weeks ago. I was told I had an infection and was given a course of antibiotics and told at some point I would need a root canal or an extraction.Fast forward to five days ago, my tooth was hurting at night and I was told that the nerve was dying and would need the root canal or extraction. I chose to just take it out. The Extraction was quick and easy and the Dentist said it should be fine. However, now five days there is slight discomfort where the tooth has been pulled. The tooth in front is quite sensitive and my jaw and cheek slightly aches. It is not horrendous pain at all, but more a dull ache. I am worried something could be wrong. On top of that I feel a little like I have the flu. Now this could be simply the fact that I was feeling run down a bit before I and the extraction and I have just picked up a common cold, but I am unsure. Is it normal to have any of the above after an extraction? Doctor: Hello, thank you for consulting with healthcaremagic. If you have infection in your tooth, it might have healed after extraction.Actually there is slight pain after extraction of molars, that is why dentist prescribes antibiotic and painkiller after the extraction. You should start with a muscle relaxant twice daily for 5 days. If it does not relieves after that also then you have to get an x - ray done of that region, which will show you the cause of pain. Hope it will help you."
},
{
"id": 110378,
"tgt": "Is bilateral hip pain related to migraine?",
"src": "Patient: Last night could not sleep because of bilateral hip pain. Also, for the past four mornings, I have woken up with migraines. Never got them before. This morning, vomiting a few times. Went to dr. They said migraine. No relation to hip pain last evening. No fever. I have started a new exercise program which I m lifting dumbells. I have two 5 lb. weights at home. This is a program designed to lose weight. Am 52 year old female, basically good health, clean eating and exercise. Is bilateral hip pain related to migraine - do you think? Or maybe hip pain last night, all evening is a separate issue? Thanks. Doctor: Hi, Welcome to healthcare magic. After carefully looking your query, it seems that hip pain and migraine is separate issue. I think your query has been answered."
},
{
"id": 165284,
"tgt": "Why does a child rub the buttocks aggressively on a hard surface?",
"src": "Patient: Hi, I have a daughter of age 6 plus, she has developed a wrong habit of scrathing or rubbing the 'ass' aggressively to the edge of hard surface. we have been hearing complaints from school and others regarding this since last 2-3 years we are helpless to understand this problem, can you help me to diagonise this case Doctor: Such complaint may be caused due to worm infestation (perianal itch), bacterial or fungal infections, allergies etc.Careful local examination my show fungal ring, infected wound, rash, urticaria, worms etc. And depending upon finding she may need deworming medications, antifungal, antibacterial or anti-allergic medications.For further queries regarding this disease or its medications you may take my appointment at Healthcare Magic.Dr. Khan Shoeb"
},
{
"id": 137691,
"tgt": "What causes sudden pain in stomach?",
"src": "Patient: The other day I was out shopping and suddenly got an extremely bad stomach ach and had to go home the next morning I tried going to work and about an hour into being there the pain was so unbearable I couldn t do anything and had to go home and the same thing happend Sunday morning i don t know what it is but it hurts extremely bad and I was just standing up giving my boyfriend a haircut and my stomach started hurting so bad I couldn t stand up Doctor: Hello.Thanks for writing to HCMI will advise to start antacids [proton pump inhibitors like] rabeprazole 20 mg twice a day.Sucralfate will protect gastric mucosa and will give protective covering.Gastric hyperacidity can lead to altered mouth tasteIf hyperacidity is corrected you will be normal.Hope this answers your query. If you have additional questions or follow up queries then please do not hesitate in writing to us. I will be happy to answer your queries. Wishing you good health.Take care."
},
{
"id": 49876,
"tgt": "Have CKD. Had chest pain and convulsion. Abnormal potassium. Why is face numb and tingling?",
"src": "Patient: Hi, I was diagnosed by my family doctor as having CKD , he did urine and blood work-he said a ratio between creatin (sp?) and I think it was potassium-it was abnormal. I was given potassium tablets when in the hospital 24 hours prior in hospital when they did my blood work because of chest pain and a convulsion because it was low...My family doctor said my potassium is now normal range 4.5 He said this has been ongoing as I had protienurea with my pregnancy 11 years ago and I ve been told numerous times I m fighting something because my potassium has always been low. My face is also always tingling numb-like a tightning when you have had too much sun. I have an ultrasound booked in a months time, and am being referred to a kidney specialist...a month is a long time, is there a chance he could be wrong? I m sorry for the spelling errors and I hope I was clear. Thank you for your time. Jenny Doctor: Hi Jenny,Welcome to HCM. From what you have mentioned in the question, you have longstanding low potassium, protenuria and have recently been diagnosed as CKD. There are various stages in CKD and it would be nice to know which stage you are in and what the latest actual eGFR reading is? Low potassium in blood could be due to various reasons and will need investigation especially if you are having symptoms. The tingling/ numbness may be due to another problem (Calcium and Magnesium levels). It is appropriate to see a kidney specialist for your problems and you will need quite a few blood and urine tests to clarify the reasons for your symptoms. BW. RB"
},
{
"id": 30915,
"tgt": "Is coming in contact with child having slap face during pregnancy infectious?",
"src": "Patient: I am 8 weeks pregnant and have come in contact this morning with a young child in my class who has slap face. I rang the womens hospital in liverpool for advice and they told me to go to my GP for a blood test. I was 10 minutes late for the blood test and they refused to see me and said they had no more appointments for me even though I told them my circumstances. I went to a walk in surgery elsewhere and they said I was to take a week off work and go for a blood test at the end of next week to see if I have the virus. I have returned home and been told by someone else that I need a blood test now to see if I am imune from the virus. I am very confused and don't know what to do. I had a miscarriage last year due to catching mumps from a child in my class so am very worried. Doctor: Hi, I really appreciate our concern about this delicate situation. As you have been advised by your GP and Women's Hospital you would have to take the Blood test suggested to you as soon as possible. I would suggest you to make another appointment and get that blood work done as soon as possible. As main aim of this blood test (IgG,IgM Titers against Parvovirus B19) is to evaluate if you had previous immunity against the disease. If you had immunity you will be fine but if you don't you will need to go under close obstetrical monitoring in following weeks including serial ultrasounds whil will keep an eye for any defects in your baby because of this infection. I hope I have answered your question correctly. I know you might be worried too much, but I would like to tell you to keep calm and reschedule that Blood wokup appointment and get the results done. Feel free to contact for any further queries. Have a nice day.Regards,Dr.Abhijeet"
},
{
"id": 223241,
"tgt": "Do I need to do a pregnancy test?",
"src": "Patient: Had sex on d 12th of december and the condom slipped,I took postinor 2 3 hours later and d 2nd pill 12 hours later.on the 17th of december I bled till the 21st and now I was meant to get my last period on the 2 of jan but had it on d 8th of jan with my normal blood flow.Do I still need 2 get a pregnancy test? Doctor: Hello,thanks for sharing your health concern with us. In the current scenario, there is no need to perform a pregnancy test since you attained your natural period, though delayed by a week which often happens when an emergency contraceptive pill is taken. The bleeding you observed soon after the pill intake was the withdrawal bleeding owing to the high hormone content in the pill. The pill offers 85-98 percent protection depending upon the time interval at which it is consumed after the unprotected intercourse. Please make sure that you restrict the use of the emergency pill to one or two occasions per year in order to avoid hormone disturbances. Take care."
},
{
"id": 68304,
"tgt": "What is the painful lump in my groin area?",
"src": "Patient: I have had a lump near my groin area and was told it was a swollen gland. It has almost completly gone away and come back to the size of a grape for a few years now. I found out I was pregnant this week and the lump is painful and is now the size of a walnut. What can I do? Doctor: Hi.Welcome to HCM.You may be having some infection and lymph node enlarged due to it. Sincw you are pregnant you should be more careful and take antibiotics in time after showing to your gynacologist and surgeon.Regards."
},
{
"id": 151908,
"tgt": "What are the symptoms of brain fever ?",
"src": "Patient: Hi my friends mother is suffering from brain tumor Dr hav i just wana konw that how person behaves and what are the symptoms of brain tumours? do it causes severe headache? because 1 of my friend often has this kind of headaches. Doctor: HI! Welcome to healthcaremagic.com.The symptoms of brain tumors depend on tumor size, type, and location. Symptoms may be caused when a tumor presses on a nerve or damages a certain area of the brain. They also may be caused when the brain swells or fluid builds up within the skull. These are the most common symptoms of brain tumors:- Headaches (usually worse in the morning) - Nausea or vomiting - Changes in speech, vision, or hearing - Problems balancing or walking - Changes in mood, personality, or ability to concentrate - Problems with memory - Muscle jerking or twitching (seizures or convulsions) - Numbness or tingling in the arms or legs.These symptoms are not sure signs of a brain tumor. Other conditions also could cause these problems.Only thorough examination of the patient,mri/ct scan of head coupled with biopsy of the tumour can finalize the diagnosis."
},
{
"id": 52932,
"tgt": "What causes swelling and pain in upper stomach after gall bladder removal?",
"src": "Patient: Female 54 years old. Had gall bladder removed 6 months ago. Pain now in upper stomach under bra area in center of chest. The size of swelling is palm size. Did a lot of gardening on weekend, so wondering if pulled muscle. Seems like no muscles in that area though. Doctor: Hello,This looks to me like so called postcholecystectomy syndrome which has all these symptoms and this occurs in about 10% of patients who undergo cholecystectomy, usually can be transient, and rarely persistent or lifelong. Symptoms include systemic symptoms such as sweating and heat sense but also dyspepsia, nausea and vomiting, flatulence, bloating, and diarrhea, and persistent pain in the upper right abdomen. Some individuals may benefit from diet modification, such as a reduced fat diet, following cholecystectomy since it may be more difficult for digestion of fatty foods. Postcholecystectomy syndrome treatment depends on the identified violations that led to it. Typically, the patient is recommended the dietary restriction table with fatty foods, enzyme preparations, antispasmodics, sometimes oral ursodeoxycholic acid can alleviate the condition.Hope I have answered your query. Let me know if I can assist you further.Regards,Dr. Ivan R. Rommstein"
},
{
"id": 100549,
"tgt": "What is the treatment for asthma?",
"src": "Patient: Dear Doctor, Nice to meet you and I stay in Bangalore and my sister is 28 year old and she is suffering from Asthma, many local doctors have said she is allergic to dust and certain smell etc. In the morning when she wakes up finds very much difficult to breath and it sounds a lot while breathing. So can you please which doctor should I consult. Please guide me. And we dont have health insrurance, but if i go to hospitals in bangalore they ask about insurance and charge based on it. So I am bit scared, due to my financial position. So please guide me with a good hospital where i can get a better service. Look forward to hear from you. My email ID is YYYY@YYYY and YYYY@YYYY Doctor: Hello.Thank you for asking at HCM.I went through your sister's history and would like to make suggestions for her as follows:1. To reply your main query, I can understand your concern about finance. I would suggest her to consult an Allergist-Immunologist who may suggest her some tests like spirometry, allergy testing, etc.There are many good allergists in Bangalore. I would suggest you to consult the one who is easily approachable and who is experienced. I cannot violate terms & conditions of HCM by naming a particular doctor, but I would suggest her to visit an institute like KIMS hospital, which has a good department dealing with allergies-asthma.2. For your sister, I would suggest her regular montelukast and levocetirizine for at least 2-3 months.3. I would also suggest her to use salbutamol inhaler on as-and-when-needed basis for asthma symptoms/wheezing/breathlessness.4. Allergy testing will help her identify the substances she may be allergic to and also to know the measures to avoid them. Common substances causing her early morning symptoms are house dust mites and indoor molds. 5. In general, I would suggest her avoidance of dusts, smokes and air pollution as much as possible.6. I would also suggest her regular breathing exercises like Yooga & Pranayam and a healthy diet rich in vitamins & minerals (adequate amounts of green leafy vegetables, fruits, sprouts, etc) which will improve her lung capacity and immunity respectively in a long run.Hope above suggestions will be helpful to her.Should you have any further query, please feel free to ask at HCM.Wish her the best of the health.Thank you & Regards."
},
{
"id": 114448,
"tgt": "How can polycythemia vera be treated?",
"src": "Patient: I am 56 y female, I was diagnosed with PV and at first the doc did only do phlebotomy and asperin. But then my platelets went up and now I have to take Hydroxyurea, 500 mg a day. Today he said I have to take 1000mg daily, because my HB is 16, hemoticrit 47 and platelet 500. WBC is normal. I don t want to take the high dose. What do you think? Doctor: Hi, dearI have gone through your question.I can understand your concern.You have polycythemia vera. In polycythemia vera there is high RBC and platelets count. it will increase the viscosity of blood and there is chance of blood clotting. Phlebotomy is the most cost effective treatment to reduce the RBC count. But aspirin and hydroxyurea is required the avoid any vascular complication like blood clot. So you should take hydroxyurea in prescribed dose. If you don't want to take it in high dosage then there are other alternative drugs like anagrelide. That is more costly. So consult your doctor and set the dosage accordingly.Hope I have answered your question.If you have any doubts then feel free to ask me.I will be happy to answer. Thanks for using health care magic.Wish you a very good health."
},
{
"id": 112456,
"tgt": "Sciatic nerve pain in lower back. Will surgery relive pain?",
"src": "Patient: Hi I suffer with sciatic nerve pain in my lower back and I've had this problem for ten years and just the last three weeks I've been in so much pain and are unable to work and walk properly due to the pain, I have a referel to see a pain specialist and I was wondering would they let me know if I need surgery to relieve my pain Doctor: Hello,Meet a spine specialist in your region. After proper clinical evaluation he will be able to tell whether you surgery or conservative management.Any try Physiotherapy by an expert before surgery."
},
{
"id": 18612,
"tgt": "How can low BP be treated?",
"src": "Patient: my husbands blood pressure was 67/45, he took it again and went to 77/55. He was feeling faint is why he took his pressure. He has daily injections of Lovenox for a blood clot in his groin, he also is on chemo every 2 weeks. How can he bring his pressure up? e-mail: YYYY@YYYY Doctor: Hello and Welcome to \u2018Ask A Doctor\u2019 service. I have reviewed your query and here is my advice. Your husband complaints are related to the low blood pressure values. Low blood pressure could be related to different possible causes: dehydration, anemia, electrolyte imbalance, etc. Coming to this point, I would recommend avoiding blood pressure medicine and increase salt and fluid intake. If these blood pressure values persist, I would recommend consulting with his doctor and performing some tests: - Complete blood count for anemia - Blood electrolytes for electrolyte imbalance - An abdominal ultrasound - Kidney and liver function tests - A cardiac ultrasound and a chest X-ray. You should discuss with his doctor on the above tests. Hope I have answered your query. Let me know if I can assist you further. Dr. Iliri"
},
{
"id": 91690,
"tgt": "Could the lower abdominal pain indicate a UTI or issues with my cyst/fibroid?",
"src": "Patient: I am not pregnant, but I have pain in the lower part of my abdomen. There's lots of pressure when I'm sitting, walking, cough, sneeze, or even hit a bump while I'm driving. I do have a history of ovarian cyst and fibroids. Could this be a UTI or issues with my cyst/fibroids? Doctor: Hi. All the history is very well related to the cyst more than a fibroid. UTI can be an associated symptom."
},
{
"id": 12468,
"tgt": "Why has psoriasis increased after childbirth?",
"src": "Patient: i am carrying now 7th month i am a psoriasis patient past 25 year's this is my 3rd delivery past 2 deliveries i didn't face this problem got increase of psoriasis but this pregnancy it got increase very much full body it got be spread is there any treatment for cuire Doctor: Hello and welcome to HCM,Psoriasis is known to aggravate during pregnancy.The treatment of psoriasis during pregnancy is slightly different from that during other times.There are some drugs like retinol, methotrexate etc which should not be used during pregnancy.These drugs can cause deleterious effect on the baby.You need to consult your dermatologist for prescription of drugs appropriate during pregnancy.Thanks and take careDr Shailja P Wahal"
},
{
"id": 58250,
"tgt": "Had ultrasound, result shows gall bladder blood sludge. What does it mean?",
"src": "Patient: Had ultrasound, result shows gall bladder blood sludge?..had hida test done 3days ago, still no results, have severe pain in l. u. q. have asked for pain medicine, and was given tramadol , this doesn t help. beginning to give up hope for getting help. not satisfied with the care I have received up to now. Doctor: Hi,Thanks for posting your query.I presume you are trying to say, your USG showed sludge in the gallbladder.Sludge is a sediment of bile that contains crystals of calcium and cholesterol.This most often remain asymptomatic. But sometimes it can become symptomatic due to complications like pancreatitis.In such cases gallbladder is removed to prevent recurrence and more complications.I hope that answers your question.Regards."
},
{
"id": 96012,
"tgt": "Bulge on right side below ribs",
"src": "Patient: I am 58 - 5 10 -male 230lbs bulge is 4 diameter appeared about 6 months ago and is a little tender Doctor: Hi, It is very difficult to say any thing without exact site of bulging as if it is in the chest it may be due to cold abscess and below the ribs on lower right side it may be in connection with liver. So without exact history like cough,temperature,abdominal symptoms it is difficult to say. Consult physician."
},
{
"id": 180508,
"tgt": "Should an infection in the wisdom tooth be completely cured before its extraction?",
"src": "Patient: I have been told that my wisdom tooth, which has a crown on it, has a cavity with infection and pain. I was told the tooth must be extracted. I have been prescribed a 7 day dosage of the antibiotic, amoxicillin, to clear up the infection first. This is the 3rd day of taking the prescription and the pain is still quite intense. Does my infection have to be completely cleared up before I go to an oral surgeon to have the infected tooth removed? Doctor: Hello and Welcome to \u2018Ask A Doctor\u2019 service. I have reviewed your query and here is my advice. As your wisdom tooth is having a cavity it is deep enough to cause infection and pain due to nerve irritation then taking antibiotics will clear up infection but it takes time.However, if there is associated severe pain then you need to take painkillers to reduce pain.Also if pain is so severe the crown must have been removed so that pressure is released that will also ease pain to some extent.As far as removal of tooth is concerned, the infection should be significantly reduced if not completely resolved and then extraction is done.In 7 days infection will be significantly reduced.For now to reduce pain you van take painkillers like Ibuprofen or Ketorolac.Hope I have answered your query. Let me know if I can assist you further. Regards, Dr. Honey Arora"
},
{
"id": 145575,
"tgt": "What causes tiny hole on back of head?",
"src": "Patient: I found a tiny hole in the back of my head,I found it when combing my hair and the comb kept getting stuck in the hole I begain picking at it with the comb trying to see if it was only dry skin,however it never itched,I use oil a lot after washing my hair, I have noticed I have lost quite a bit of hair about 10 inches or more in the past year.what could this be? Doctor: Hello ! I understand your concern. I would like to know if you have noticed any liquid to get out of that tiny hole?In my opinion it may be a skin problem of the scalp . It may be nothing, just a skin structural defect . But the hair loss may indicate a skin problem . I think you should be seen by a dermatologist. Hope to have been helpful!Thank you for using HCM!Best wishes, Dr. Abaz Quka"
},
{
"id": 164838,
"tgt": "What causes vomitting and diarrhea?",
"src": "Patient: On Tuesday of last week, my 10 month old son threw up a lot - no fever - he s had diarrhea ever since. This morning he finally woke up at 8am; he went to bed the night before at 6pm, his normal time. He was awake for one hour, had a 6 ounce bottle which he kept down and no diarrhea, but wet diaper . He was incredibly lethargic, really listless and fussy, just wanted to be held and cuddled. Once again, no fever. What do you think I should be doing? Doctor: Hi...Thank you for consulting in Health Care magic.It seems your kid is having viral diarrhoea. Once it starts it will take 5-7 days to completely get better. Unless the kid's having low urine output or very dull or excessively sleepy or blood in motion or green bilious vomiting...you need not worry.There is no need to use antibiotics unless there is blood in the motion. Antibiotics might worsen if unnecessarily used causing antibiotic associated diarrhoea.I suggest you use zinc supplements (Z&D drops 1ml once daily for 14 days) & ORS (Each small packet mixed in 200ml of potable water and keep giving sip by sip) as hydration is very important and crucial part of treatment. If there is vomiting you can use Syrup Ondansetron (as prescribed by your paediatrician).Regards - Dr. Sumanth"
},
{
"id": 162073,
"tgt": "What causes stomach pain, loss of appetite and stuck feeling in throat?",
"src": "Patient: My 8 yr. old daughter has been complaing of stabbing stomach pains she says it hurts more when she lays her head down has no fever and lost her appetite. She has not been sleeping at night says feels like she can breathe and feels like something is stuck in her throat and can t get it out? Doctor: Hello, But this is probably due to gastritis so please get a stool analysis to exclude h.pylori infection. Hope I have answered your query. Let me know if I can assist you further. Take care Regards, Dr Salah Saad Shoman, Internal Medicine Specialist"
},
{
"id": 85873,
"tgt": "Is this dangerous to have steroids for common cold like symptoms?",
"src": "Patient: I had open heart surgery 2 years ago and am on the normal meds a person with heart disease would be on. I went to the clinic today due to a serious cold, headaches, right chest aching inside (figured this was my lung), coughing up yellow junk, sneezing, etc. They gave me a shot in the hip that had steroids and an antibiotic in it. Is this dangerous? Doctor: Hello, While normally I wouldn't give steroids and antibiotics for cold symptoms since the side effects and risks probably outweigh the benefits, most of the time patients do not have any problems, and I don't see anything with your condition that would make me especially concerned. So while I probably wouldn't have given you this shot, you will probably be alright. The most common side effects from the steroids are stomach irritation and possibly ulcers, as well as problems with sleep, while with antibiotics you might get diarrhea or rash. There are many other side effects but these would be some of the more common ones. Hope I have answered your query. Let me know if I can assist you further. Regards, Dr. Aaron Branch, General & Family Physician"
},
{
"id": 64959,
"tgt": "What causes lump on testicle?",
"src": "Patient: I am a 5 foot 6\" 146 lbs very active male age 57. I weight train with lighter weights nowadays and i cycle vigorously. Yesterday i noticed a small lump on my right testicle that was about the size of a bb. I am in no pain and i do realize i need to get this checked out but i wanted to know is it possible that this is connected to my exercise. Also i must mention i had a vasectomy eleven years ago. Ken Boylan Doctor: Hi,Dear I went through your query.1-By the facts placed by your query,I feel that this lump appears to be vasectomy -fibrous lump-as its not painful and found co-incidentally by you during your examination of the testicle.2-Plz check your your ER- doctor for second opinion.3-Hope this would solve your query,4-Thanks for your query to the Health Care magic. 5-You are wellcome for any further treatments if required.6-I would love to help you out of YOur worry about your testicular lump. .Thnks.Wellcome."
},
{
"id": 10916,
"tgt": "Suggest remedy for hair loss",
"src": "Patient: Hi Doc,I am 33 yrs old.My hair is curly,recently I noticed lot of hair on my t-shirt was bit worried. I apply coconut hair oil thrice a week & wash my hair with Loreal total repair 5 shampoo.Kindly give some remedy.Please suggest some good doc in Pune. Doctor: Hi..Welcome to HEALTHCARE MAGIC..I have gone through your query and can understand your concerns..As per your complain hairfall can occur due to a number of causes like fungal infection, nutritional deficiency long standing illness, stress, side effect of medicines while androgenic aloepecia caused due to androgenic hormones and genetics can help..I would suggest you to consult a Trichologist and get evaluated and he can advise you to get investigations like blood tests, Dermoscopy, Trichogram etc done to rule out the exact cause of the problem and treat accordingly..You can be advised to take Biotin and multivitamin supplements..You can also be advised to apply Minoxidil lotion over the scalp..Keep massaging the scalp with warm coconut oil twice a week..You can also be advised to take oral Finesteride if Androgenic aloepecia is ruled out..Hope this information helps..Thanks and regards.Dr.Honey Nandwani Arora."
},
{
"id": 163582,
"tgt": "How can knee injury causing soreness be treated?",
"src": "Patient: my 11 year old daughter fell out of the car and down with her full weight on her right knee, it is very sore and she cannot straighten it out. She cries when she walks up and down stairs and any change of direction seems to make the pain worse. this happend a week ago and she does not seem any better infact i think it is getting worse. we went to a&e and an xray didnt show anything broken. Doctor: Hello, I can understand your problem. As X ray is normal, nothing to worry. So its a blunt trauma which takes more 4 to 5 days to cure. You can give painkiller like tab. Nimesulide 100 mg after consultation with orthopaedician. Apply DICLOFENAC GEL locally to injured knee. Go to physiotherapist and do regular exercises as advised. I think this is helpful to you. Review back. Thank you."
},
{
"id": 54549,
"tgt": "Why do I have pain at the gall bladder operation site?",
"src": "Patient: I have been having pain for 3 years where i had gall bladder surgery - xrays, etc show nothing - but just found out I have surgical clamps on the inside - they say that is okay - this weekend it hurts again there and I have been debating whether to go to the er - don't want to go if it is nothing - could the clamps slip and cause this pain or can a muscle make it hurt that bad around the ribs - I had low red cells for a few months but they are back to normal.Thanks for any advice - Candi Doctor: HiThe problem wouldn't be the surgical clips but you may have intestinal adhesions causing a partial obstruction so you need to get this evaluated at the ER"
},
{
"id": 223496,
"tgt": "Is there any chances of getting pregnant after taking an i-pill? What are its side effects?",
"src": "Patient: I got period on 22nd December,on 26,27th December I had sex and then on 27th morning I took an I-pill (Emergency Contraceptive Pill),then again on 30and 31st December had sex.then on 1st January morning I have taken an i-pill again.So it is safe for me? is there any chance to become pregnant or does the i-pill have any worse side effect? Doctor: Hello,I have gone through the query and understood your concern. Firstly, let me inform you that you had the intercourse during your safe period and did not need the emergency pill in the first instance. Presuming that you have a regular 28-30 day menstrual cycle, your ovulation would have fallen around the 7th of January. Since the sperm can remain active for up to 5 days after ejaculation and the ovum remains alive for 24 hours after ovulation, any unprotected intercourse during this period can cause conception, which you did not have. The emergency pill gives adequate protection when consumed within the stipulated time interval. As it contains high dose of hormone, menstrual irregularity can be expected. Your next natural period may get delayed by a week or two. Please restrict the use of the pill to one or two occasions per year. See a specialist for further support if you become overdue by ten days. Hope this helps."
},
{
"id": 96425,
"tgt": "How to force a bowl movement ? please help",
"src": "Patient: I am trying to help my cousin.. her daughter is 7 and has had chronic constipation for some time now.. her doctor just keeps telling my cousin to give her Miralax.. as we speak the poor child is screaming and crying in pain because she can't poop.. my cousin has tried everything we can come up with to help her.. a warm bath and even a glycerin enema.. is there anything we can do that isn't going to take hours to work.. Doctor: high fiber diet and regular exercise also the regularity of the food is what constitutes for the regular and smooth movement of the bowl, improper diet can lead to this. there is a service called ask for diet chat in this website, i will give you the link, this will help you resolve the problem in a month or so, http://www.healthcaremagic.com/diet-plans-from-nutritionist."
},
{
"id": 172498,
"tgt": "What could cause pain in anus?",
"src": "Patient: My Daughter is 2.5 yrs old. She has moved from chennai to chicago 2 weeks before. For the past 4 days, she is feeling pain in the anus area.She does not have any Constipation problemCan you please suggest the medication for this ?Thanks in advance Doctor: If there is no constipation or diarrhoea then u can wait on medications...give her warm water spongging over the anus...if she develops blood streaked stool, then consult a surgeon ,it might be anal fissure"
},
{
"id": 51872,
"tgt": "Give a diet chart for a person who has high micro albumin and diabetes",
"src": "Patient: My Father in law has high microalbumin in urine test of microalbumin 436mg/L? He is a high diabetic and blood pressure patient. Please share the deit chart to control the same apart from medicine that has been suggested by GP. Regards Sucharita Doctor: Hi Suchrita Welcome to HCM Read your prob, presence of microalbumin in urine shows high uncontrolled diabeteas of altered kidney function. you first you sure normal leval of blood sugur by drugs or insulin. n get one blood test for renal function. discuss it with nephrologist n start proper treatment. as diabetes n high blood pressure is cause of altered kideny function. Dr aseem 9982583020 aseemadhuri@gmail.com"
},
{
"id": 201835,
"tgt": "How to stop premature ejaculation and any solution for shrinked penis?",
"src": "Patient: hi , i am 27 years old man,my penis when not erected is half head out(shrinked all the time),when erected its 4.3'' mayb,but go closer to my partner for intercourse ,my erection finishes ,if i touch her body i ejaculated without my desire.all the time my penis is shrinks.my weight is 81.1 ,height mayb 5.2'',i m lazy also Doctor: DearWe understand your concernsI went through your details. I suggest you not to worry much. Erectile dysfunction and premature ejaculation are common problems attributable to anxiety and misconceptions about sex. Wrong knowledge leads to wrong perception. Shrunken penis in its lucid state is pretty common and is its actual state. Don;t worry about that. For Erectile dysfunction and premature ejaculation, please visit a psychologist who shall help you.If you require more of my help in this aspect, Please post a direct question to me in this website. Make sure that you include every minute details possible. I shall prescribe the needed psychotherapy techniques.Hope this answers your query. Available for further clarifications.Good luck."
},
{
"id": 174856,
"tgt": "Is tazocef injection, amikacin, kidped and monticope syrup right medication for bronchitis in an infant?",
"src": "Patient: Gud evening doctor,My son 7 months old. Recently he suffered with running nose, we went to hospital for treatment. They recommed us to admit my son, they diagnosised as bronchiolitis. Treatment advised take for five days tazocef injection 400 mg and amikacin 60 mg. Syrup - kidped, monticope Doctor: Bronchiolitis and bronchitis are 2 different conditions. Bronchiolitis is more common in a 7 month old baby and generally does not require too many medicines or a lot of antibiotics. But admission is often necessary for giving oxygen, IV fluids and maybe nebulisation."
},
{
"id": 192476,
"tgt": "What causes erectile dysfunction with coldness of penis and testicles?",
"src": "Patient: My penis and testes are constantly cold, my shaft feels retracted. I have stomach pains constantly, almost like a cramp or hunger pains all of the time. This has been going on for about 4-5 years. Ive been checked out by a urinologists and he's found nothing. My erections have been about 70-75% of what is normal. Any ideas? Doctor: Hello, If urologists were unable to find out anything or problems with your erection problems, it has to be due to psychological disorders like stress and anxiety. Therefore I suggest consulting a psychologist for physical examination, diagnosis and treatment. Hope I have answered your question. Let me know if I can assist you further. Regards, Dr. Sameen Bin Naeem, General & Family Physician"
},
{
"id": 43879,
"tgt": "On treatment for infertility. Prescribed Ovuclom, Gestin. Chances of pregnancy after medications?",
"src": "Patient: i am taking treatment to solve the infertality. I am suggested with ovuclom(will be taken at the 3 rd day of the last periods) tablet and Gestin Tablet (will be taken at 16th day of the last periods ) . From last month i am taking this treatment. But i got the periods at 22nd day itself.. some times i used to take the tablet late (like if i forgot morning i wil take at after noon ) is that the problem??? whether these two tablets solve my problem(leads to pregnancy) Doctor: Hi, Thanks for your query. I have read your query & I understand your concerns. It is fine if you have taken those tablets bit late. Not to worry. These tablets help you to get pregnant but chances are around 20 %. Please see an infertility specialist and get follicular growth scan done while you are on tablets. It helps in deciding best time to have intercourse ( during ovulation). I hope I answered your query. I will be available for any followup queries you have. Regards, Dr.Mahesh Koregol IVF & Infertility Specialist."
},
{
"id": 35070,
"tgt": "How to get rid of my fungal infection?",
"src": "Patient: i m having serious fungal infection , doc has given me floconazole tablet(1tab/week and its nearly 5 months now i m taking this tablet) and ocytetracycline injection IP for applying in infected area and then apply clobetasole cream. but it all hardly helps me ..plz help Doctor: Hello dear ,I read and understand your concern.I am Dr Arun Tank, infectious disease specialist answering your concern.Deep and severe fungal infection never treated in a easy way. It requires some time to cure. Example fungal infection of nail requires almost four to five months to heal. So in my advice you should continue your treatment with oral fluconazole and others. Please do not get discourage if result is not getting fast. You should wait and continue the treatment.If you stop the treatment than the fungal infection becomes resistant one and wont respond to your current antifungal treatment.Please avoid injury to infected site. Good hygiene maintenance with frequent cleaning is necessary. Please aboid excessive sweating to the site.Please wear cotton clothes as it is helpful for treating infection.You can ask me for further clarification here or you can ask me on bit.ly/DrArun.Thank you,Dr Arun TankInfectious Disease Specialist"
},
{
"id": 85552,
"tgt": "What could cause dizziness, fatigue and sleepiness after surgery using anesthesia?",
"src": "Patient: I had surgery oct 18 general ansethesia was used. I been feeling dizziness. The room is not spinning . But I feel like I am moving inside . What could this be ? I got mecilizine only for one day . I started yesterday but its makes me so tired and sleepy Doctor: Hello, Your symptoms are related to the operation you had under general anesthesia. However, to me, you should check your blood sugar and blood pressure levels, record the results for reference to rule out possibly increase or decrease in blood pressure/sugar levels. Keep well hydrated meanwhile. Hope I have answered your query. Let me know if I can assist you further. Take care Regards, Dr Albana Sejdini, General & Family Physician"
},
{
"id": 62153,
"tgt": "Suggest remedy for lumps in head with headache",
"src": "Patient: Hi, I am 19. I have a lump on the back off my head that comes and goes. I use to get it when I was younger, and it never bothered me unless i hit it while brushing my hair. Now, sometimes when it comes back it feels like it is progressing in pain. I also get a headache that follows but I get tension headaches so much I don t think it is connected. Should I go to a doctor to get it checked out?? Doctor: Hi, dearI have gone through your question. I can understand your concern. Your lump in back of head that comes and goes is reactive lymphnode hyperplasia. No need to worry about that. Just be relaxed. Your headache is not connected with that. For headache you should check other cause like hypertension, refractory error etc. If you don't find any causes then MRI can be fone if required. Consult your doctor and plan accordingly. No need to worry about lump. Hope I have answered your question, if you have doubt then I will be happy to answer. Thanks for using health care magic. Wish you a very good health."
},
{
"id": 140886,
"tgt": "What do moderate left and mild right foraminal narrowing and mild levocurvature at L2-L3 suggest?",
"src": "Patient: it s been 2-yrs now and finally got another MRI found L5-S1 5mm left central disc protusion with mass effect on the left sided nerves. There is underlying 3mm disc bulge and facet arthropathy causing moderate left and mild righ foraminal narrowing 2. mild bilateral foraminal narrowing at L3-L4 and L4-L5 3. mild levocurvature at L2-L3 4. Trace retrlisthesis at L4-L5 and L5-S1. What does this all mean???? Doctor: Hello, I would have ideally preferred if you had added your symptoms prior to telling us the report. I think, having read the report, that you must be having pain and/or sensory symptoms (like tingling and numbness) and/or weakness in your leg, predominantly the left leg more than the right leg. The report suggests that you have a disc problem. The disc is coming out of its normal position and pressing the nerves that come out of the spine and causing the symptoms. I would ideally need to examine you and need more details about your symptom before I can suggest a complete treatment. However, I can tell you that a simple prescription of Pregabalin or gabapentin with physiotherapy may be able to afford you some relief from your symptoms. You may need to meet a neurologist or neurosurgeon (dealing in spinal surgery, in case of latter) for a prescription. Hope I have answered your query. Let me know if I can assist you further. Take care Regards, Dr. Saumya Mittal , Diabetologist"
},
{
"id": 196020,
"tgt": "Why the head of the penis itches and tickles?",
"src": "Patient: Hi, i've been itching on my butt, thighs, and crotch area for several weeks now. I went to the doctor and they said it was probably an ingrown hair but i think it's something else. i have a weird red rash on the head of my penis, and my penis doesn't hurt. It more so itches and tickles. i have several bumps on my legs/butt as well. pleasee help Doctor: Hello Thank you for trusting HCM Dear since when you're suffering from this problem? Just several weeks??? Have started using any new soaps or moisturizing lotions?? It may be due to contact dermatitis due to new cosmetics, other causes are Jack itch due to fungal infection, it may be due to balanitis, or psoriasis etc. Use Candid b gel at local area. If symptoms not improved consult your doctor."
},
{
"id": 98887,
"tgt": "Suggest treatment for alpha gal",
"src": "Patient: I have recently been diagnosed with alpha gal. My Dr. told me to quit eating beef and pork with no known cure or treatment for it . Is there any and where might I go to seek help. You must also know that I am a southerner and enjoy my meats for bbq purposes, smoking a head of cabbage the rest of my life just won't cut it. Appreciate any help that you might be able to give. Doctor: Alpha gal allergy is known to decrease with time, may be over few years. Let me give you the best options available.1st - Avoid the allergy causing meat for sometime till your allergy abates.2nd - Avoid being bitten by the lone star tick because you are a southerner.3rd - Consult your doctor for regular followups.4th - Homeopathy can to certain extent give you symptom relief."
},
{
"id": 137364,
"tgt": "Suggest treatment for swollen and painful toe",
"src": "Patient: Hi, I m pretty sure I broke my baby toe last night, it s completely purple and swollen.. My baby toe use to be very close to the toe next to it, almost laying on top of it but now there is a space between them and I can t move it, the pain is horrible, shooting up my leg at times and then other times it feels nume... What do you think?? Doctor: your symptoms suggest that you have sever soft tissue injury over toe. icing ,anti-inflammatory analgesic such as diclofenac ,limb elevation over pillow and rest will decrease pain. strapping of injured toe with adjacent toe will give support to its tissue injury. if swelling persists for more then seven days then visit to your orthopedician and get an xray of injured toe.Hope this helps.Hope this answers your question. If you have additional questions or follow up questions then please do not hesitate in writing to us. I will be happy to answer your questions. Wishing you good health."
},
{
"id": 46168,
"tgt": "Suggest treatment for kidney problems",
"src": "Patient: My Mom's left kidney is shrunken(not working) due to overdose of pain killers during her treatment of arthrities and now after few years she's again having pain on the other kidney,n after few blood test reports doctor is saying that there is some swelling and infection going in the other kidney...so i am very tensed and worried that it can be cure through medicines or not????please help me out to solve my problem Doctor: It can't be cured but can be controlled with medicines.First of all make sure that she takes a plenty of water through out day.Start taking tab chadraprabha wati 2bd tab punarnava mandir 1bdsyp chamdanaasav 2 tsp with 3tsp water after meals.at bed time take 1/2 tsp of trifala with like warm water.Take this medicine for a week then contact me. we shall plan further treatment according to her response. we shall also give her diet charts for the same."
},
{
"id": 193379,
"tgt": "What can cause psoriasis?",
"src": "Patient: Hallo I got psoriasis from while using lithium. But at that time was not masturbating for 6 months or a 1 year, started masturbating because of bad sex eduction and teachers closing doors and teaching only certain kids, also maybe started a bit late and hear from other kid about the white stuff on your dick. Used the lithium for a year and the psoriasis started under my sperm bag with skin burning. Was first diagnose with eczema. I got it at 21 know it is burning and not good between my anus and sperm bag. My underwear was tight and my also played a roll and my Grand Mother had it. I just want to know can you cut it out or cut my private part that there is no pain and itching? But still usable. Also want to know if not masturbating and using lithium plays a roll in getting psoriasis? Doctor: Hello, Psoriasis is a common skin condition that speeds up the life cycle of skin cells. It causes cells to build up rapidly on the surface of the skin. The extra skin cells form scales and red patches that are itchy and sometimes painful. Psoriasis is a chronic disease that often comes and goes. Psoriasis is thought to be an immune system problem. Triggers include infections, stress and cold. Hope I have answered your query. Let me know if I can assist you further. Take care Regards, Dr K. V. Anand, Psychologist"
},
{
"id": 221882,
"tgt": "What causes pain in rib cage and back during pregnancy?",
"src": "Patient: I am 25 weeks pregnant and I have no energy and so drained and tired,also I have severe pain in my back on my right side where my rib cage is,it hurts so bad it throbbs,I can t sleep I toss and turn,is this my baby in my ribs or something more...please help me Doctor: Hello dear,I understand your concern.In my opinion the pain in the rib cage and back can be due to muscular pain or gastritis.Avoid spicy and oily diet.Take food in small quantities every 2 hourly.Take antacids.If not subsiding take panadol for pain and local irritants and hot fomentation can be applied.Dont worry It will subside.Some or other types of non specific pains are common in pregnancy.They will subside spontaneously.Best regards..."
},
{
"id": 119129,
"tgt": "Done abortion due to problem in baby s brain. Any trouble if couple has same blood group?",
"src": "Patient: i want to know whether couple with same blood group - have problem in pregnancy , as during my 8th month in ultrasound test i got the news there is problem in baby s brain and i had to terminate my pregancy. Please give me a solution doctor as i m very scared.if there is a problem wid same blood group (i.e me and husband having 0+ positve blood group) what is the solution Doctor: Hi, Sorry about the baby but don't worry, it is not related to the same blood group of the couple. Please check if genetic studies were done on the the abortus to look for chromosomal anomalies. Your gynaecologist will guide you regarding folic acid therapy from three months prior to conception next time and other relavent tests if needed.All the best. You have every chance of success next time."
},
{
"id": 216780,
"tgt": "Suggest treatment for pain in the calf",
"src": "Patient: I ve had an ache, like a Charlie horse in my left calf for about 3 weeks. I m concerned it may be a blood clot or DVT. I saw my Dr. last week, she ordered blood work and thinks my magnesium level is down. Now tonight the pain is bad again. Isn t there an ultra sound or MRI that should be done/ Doctor: Thanks for contacting healthcare magic. You have complain of pain in calf muscles that because of spasm of this calf muscle. There are many cause for spasm of muscle.Your magnesium level is low that because of this cause spasm of calf muscle. During sudden spasm of calf muscle you have to massage over this area.Try to strech your muscle so it become relax. Try to earn your muscle that decrease spasm.You can put some weight over muscle that may helpful. Eat foods that have more calcium and magnesium. Eat fress fruits and green vegetables. Drink more water.physiotherapy may helpful. Do some streching excercise. Avoid alcohol. less hydration may cause muscle spasm so be hydrated.Over exertions may worsen muscle cramps.put your leg against wall and strech. .. that type of excercise may helpful. Wish you a good health Take care.Thanks."
},
{
"id": 154159,
"tgt": "What causes Nausea,Vomiting and Muscle Spam after Chemotherapy?",
"src": "Patient: I am writing in regards to nausea/vomiting and muscles spasms as a side effect from chemotherapy. I am am enquiring about the physiology behind why these things are happening inside my body. Also, is there anything that can be done to reduce the seizure like spams and violent vomiting. Doctor: Hi, dearI have gone through your question. I can understand your concern. Nausea and vomiting is common side effects of chemotherapy. It is due to some toxic substance from cancer cells. Drug of choice for cancer or chemotherapy related vomiting is ondansetrone. You can take oral or injectable ondansetrone to manage this vomiting. Consult your doctor and take treatment accordingly. Hope I have answered your question, if you have doubt then I will be happy to answer. Thanks for using health care magic. Wish you a very good health."
},
{
"id": 175450,
"tgt": "What is the treatment for cough?",
"src": "Patient: Hi, my son is now 3 years old and gets frequently caugh. About almost every 15days.im giving him asthafin syrup 2 ml since almost a year as prescribed by the doctor.I want to know weather it is safe to give him so. Because I have noticed if I stop giving he starts falling sick again.his weight is 12. 5 kgs Doctor: Hi Dear Welcome to the HCM,Improve the immunity of the child with high protein diet.supplementation of iron and multivitamin syrups.Avoid contact of your baby coming in contact with persons suffering with viral infections.iNVESTIGAT THE CHILD IN TERMS OF SOME CHEST INFECTION.with CBC.ESR,CXR. mX test etc.Hope the query is answered.thanks"
},
{
"id": 118475,
"tgt": "Having cough and breathing problem. What is normochromic with eosinophilia?",
"src": "Patient: what is normochromic with eosinophilia? my age is 26, am having caugh from 6 months, now am facing problem to take breath freely,, is there is anything serious with me..please answer.. Doctor: Hi, In case you are having breathing problem alongwith eosinophilia, you need screening for presence of any fungal infection and allergic disease. Kindly get your Chest X Ray , sputum examination , ESR and ECG. In case you are having any fungal infection , you might need treatment with antibiotics.So kindly consult your physician and get yourself evaluated. Hope it helps."
},
{
"id": 113123,
"tgt": "Severe pain in lower back when applying pressure, on walking. How to reduce pain?",
"src": "Patient: Hi i am a female in the early twenties and I suddenly have this acute pain in my lower back near the spinal area at waist just above the tail bone area. I have not had any history for back problems so this is very sudden. I can t actually walk with the pain and it feels like whenever I actively apply any pressure or squeeze the muscles in the area especially when walking,the pain gets really bad. Doctor: Hello! Thanks for being with HCM. Sudden Onset of back pain In 20 years female could be due to 1- Coccydynia 2- Just a muscular back pain 3- Mechanical back pain 4- Vit-D deficiency 5- Prolapse intervertebral disc (less like in your case) in that generally patient has radiating pain in lower limb/s Due you have any sudden jerky movement while jimmying or have suddenly lift any heavy things? I would suggest to take an opinion of an Orthopedician for better clinical correlation and to get a X-ray sacrococcyx region and Vit-D levels. For pain relief you can apply hot fomentation and nise gel locally and mild anti inflammatory like Aceclofenac and paracetamol. For more queries you can write to us. Wish you good health."
},
{
"id": 17484,
"tgt": "How to cure raising heart beat with BP-151/103 after two open heart surgeries?",
"src": "Patient: my husband had two open heart surgeries last year in a moths time. he is 48 years old and has the stress of just losing his job he weighs around 238 his heart has been racing i took his bp it was 151/103 we just lost our insurance due to the loss of his job what do i do Doctor: Hello, I understand your concern and would explain that his high blood pressure and high heart rate could be related to anxiety. For this reason, I would recommend you to closely monitor his blood pressure values in the next days. If his blood pressure values are above the normal ranges (125/85mm Hg) most of the time, I would recommend consulting with his attending physician and discussing starting anti-hypertensive therapy and a beta blocker. Otherwise, I would recommend trying to focus on his anxiety. In this regard, Yoga and meditation can help. Hope I have answered your query. Let me know if I can assist you further. Regards, Dr. Ilir Sharka, Cardiologist"
},
{
"id": 201991,
"tgt": "What causes penis redness?",
"src": "Patient: My son is of 3.5 years and today morning I saw his Penis is little unusual as glans was totally out of foreskin and glans is red from one side. It seems if some insect has bitten. Please guide. Please let me know if we should visit Pediatrics. Can we give Avil. Doctor: HIWell come to HCMThis could be hypersensitive reaction could be due to smegma formation the condition can be best treated with, Syrup Levocetrizine and Beclomethasone cream for local application, if the symptoms persist then you could see the doctor, hope this helps."
},
{
"id": 49989,
"tgt": "Stones in left kidney, think it s severe ureter spasming, appetite loss, tight sore throat. What is it ?",
"src": "Patient: Had 2 stones in Left kidney 5 x 7 x 8. Used lots of water and lemons and potassium citrate etc. for 1.3 months and they don't show up on 2nd ultrasound any longer. Still have what doc thinks is severe ureter spasming and he ordered a CT today to see if any pieces are left in ureter. He said that even if nothing is found the irritation can continue for weeks before both ureters calm down. It hurts to evacuate in morning. He said that the irritated ureter can irritate the bowel but I'm not understanding that? How does that happen? Have never has any bowel issues at all. No diahhrea currently at all or IBS symptoms. Can't eat alot because when full and food moving downward the 2 lines going down become tight and sore. Can it take this long for the ureters to calm down? Having CT scan later today. Thanks. Doctor: hi..you had renal stones, probably it has come out with hydrotherapy... since it has not shown up in the earlier CT scan..you have sore throat, it is a completely different issue.. you have acute pharyngitis, probably due to water infection..see you ENT surgeon for this, for examination..and you need to start with antibiotics, analgesics, salt water gargling would help..your ureteric colic will be diagnosed with the scheduled ct scan today..see your urologist with the report for the further management..bye.."
},
{
"id": 36631,
"tgt": "Suggest treatment for filariasis",
"src": "Patient: hi, i m talking about my sister age 52 years. he suffering by filaria. she is complaining last about 4 years some thing is walking under skin. many doctor is treated by many medicine several time with high power antibiotic and other but no result. now a doctor said after diagnosis she suffering by filaria. she have no elephantiasis but leg is going to be black and complain something walking and bit in leg and her back side of arms. so please give me suggestion and treatment for her. thank you Doctor: HelloSince diagnosis is established , so I am advising accordingly.The drug of choice for microfilaria is DIETHYLCARBAMAZINE ( DEC).It has selective effect on microfilariae (MF).MF in peripheral blood are killed but those present in nodules and in transudate are not killed . So take this treatment because in your sister it seems that MF are in peripheral part .Doses :1 mg /kg body weight in 3 divided doses;thereafter increased gradually to 6 mg /kg body weight daily over 2-3 days for 21 days for 4-6 weeks .Treatment may be repeated after 6 months.Also give her albendazole 400 mg twice in day for 21 days .Concomitantly these both drugs can be given.So consult your physician and take a prescription as these drugs are prescription drugs.Good luck."
},
{
"id": 162893,
"tgt": "Suggest a multi vitamin supplement that can be taken by an obese child",
"src": "Patient: Is there one pill with essential vitamins for an overweight 14 year old male such as Vitamins A, C, D, Zinc, etc? Normally his diet is poor (FastFood, etc) but he needs to lose 50 pounds. At 14, his self-esteem is suffering. He doesn t want to take a multitude of pills, preferring to take just something akin to Once A Day. Doctor: Hello and Welcome to \u2018Ask A Doctor\u2019 service. I have reviewed your query and here is my advice.1. You can take Tab. A to Z daily one tab for 1 month2. Dietary restrictions: avoid fatty foods like fried items & sweets, ice creams. 3. Exercise regularly at least 1hr/day 4. Drink plenty of water.Hope I have answered your query. Let me know if I can assist you further."
},
{
"id": 67771,
"tgt": "What could inflamed lumps on arm pit suggest?",
"src": "Patient: I have a lump knot under my arm pit on right side. I m not sure if it is inflamed or an actual lump. It doesn t hurt when I push on it but at the same time when I lift both arms there is a noticeable difference in y right arm pit compared to my left arm pit. Should I be concerned? Doctor: Hi,It seems that you might be having chronic lymphadenitis giving this problem.It might be due to some infection in surrounding area.Consult your doctor and get examined and go for treatment accordingly.Ok and take care."
},
{
"id": 166135,
"tgt": "Suggest medication for fever and abdominal pain",
"src": "Patient: My almost 5yr old son had a fever of ranging from 103 - 104 this morning and when I felt his hand, it was kind of stiff. When I tried to straighten it, the hand would clench my hand back. Could this be a neurological problem that need to be addressed. A few months ago, when he was running a fever also, he passed out for a few minutes after complaining of stomach ache. Doctor: HI...this could be a viral fever associated myositis or inflammation of the muscles. I do not think this is a neurological problem. But I suggest you consult your pediatrician or nearest emergency room regarding this.Regards - Dr. Sumanth"
},
{
"id": 179357,
"tgt": "What causes fever with chills and shivering?",
"src": "Patient: My 2 yr old isnt feeling well. Fever / temp on and off. Giving tylenol to keep it down. She says she s cold, takes chills & shivers. She acts ok otherwise with playing & smiling. She seems a little shaky at times. Her head is warm, her temples, and under arms. At times her skin gets pink. . . It looks like shes swollen at the temples but not sure. Littlen puffiness in fact maybe. What can I do? Doctor: Which part of the globe do you belong to?Is her nose and throat clear? How are her bowel movements?In tropical areas this may be an initial presentation of Malaria.In any case tylenol only brings down fever for few hours. If she as fever for a few days, it is advisable to take her to a medical person/doctor."
},
{
"id": 34940,
"tgt": "What causes parasitic disease?",
"src": "Patient: parasites worms under skin in roof of mouth laying orange eggs also white wormlike creatures pulled out of chin.... pulled white fat wormlike out of ear pierce hole....74 yrs old.drs won't ackinowledge .... trying to find a parasitic desease ..please advise... Doctor: Hello dear,Thank you for your contact to health care magic.I read and understand your concern. I am Dr Arun Tank answering your concern.Most of the parasite resides in the either GIT or lungs. So it is seems to be other parasites. Most common parasites wandering in whole body are strongyloides stercoralis. It is seen most commonly in HIV reactive or immunocomprmised patient.But it is advised you to get the worm examined by near by laboratory.Treatment with albendazole will clear the infection. But to eradicate the infection you need to clear the background immunocomprmise state.I will be happy to answer your further concern on bit.ly/DrArun.Thank you,Dr Arun TankInfectious diseases specialist,HCM."
},
{
"id": 94794,
"tgt": "Recurring abdominal pain, loose stools post treatment, tests showed high SGPT, awaiting results for widal, blood culture, urine. Differentials?",
"src": "Patient: I have a patient who had abdominal pain and loose stools for 3 weeks.travelled from pakistan 10 year old.stool routine analysis ,urine analysis and ultrasound were normal.treated with metronidazolefor 7 days and he improved but after 5 days now he came now with 4 days of fever ,cough,loose motions,vomitting and abdominal pain.got his blood analysis done.CBC, CRP ,-normal.BUN-5,creat-normal,but SGPT-1820 raised.widal,blood cultureand urine analysis awaited.what should be the differentials Doctor: Hi Thanks for your query. Your patient has a highly elevated serum SGPT level. In this case you should go for total liver function test (LFT) which include total bilirubin, conjugated and unconjugated fraction, serum albumin, SGOT, SGPT and alkaline phosphatase level. Along with these you should also have hepatitisB surface antigen (HBSAg), anti-HBSAg, test for hepatitis c virus (ANTI-HCV), hepatitis A virus (Anti-HAV), Hepatitis E virus (Anti-HEV). Also wait for the other results to come which you have sent. It may be a case of viral hepatitis. If alkaline phosphatase and conjugated bilirubin is very high you should have an ultrasonography (USG) of abdomen. Keep the boy on bland diet, plenty of water per mouth and liver tonic and digestive enzymes until further diagnosis is reached. Thank you."
},
{
"id": 64018,
"tgt": "Does a lump located on the right tonsil pillar indicate cancer?",
"src": "Patient: I have a small lump located on the right tonsil pillar. It has been there quite sometime. I am guessing about a year. It isn t bothersome so I tend to forget about it. It is smooth and flesh in color, about 1 cm. I do have a picture if it would be helpful although I don t know how to attach it. I m just wwanting to know an opinion whether it may be a sign of cancer or pre cancer? I will be getting it officially evaluted. Thank you. Doctor: HI, Good Early morning- from INDIA.Thanks for the query to HCM. I studied it in all the details given from you and understood your health concerns.-Cause and Treatment -of lumps on the right tonsil pillar -could by Retention Cyst of the Tonsills and many benign tumors could be the reason.Wishing you fast recovery ASAP.Welcome for any further query in this regard.Have a good Day..!!With Regards,Dr.Savaskar M.N.M.S.Genl-CVTS,Super-specialist in NCCD-Non-Curable-Chronic and acute-Diseases and expert in Rejuvenation therapies for tissue and organ failures."
},
{
"id": 28342,
"tgt": "What causes fluttering in heart while having vasovagal syncope?",
"src": "Patient: It is about my daughter. She is 30 years old. she recently experienced what the doctor called a vassal vagal. Her blood pressure spikes for no apparent reason for short periods of time. When she goes to bed she feels like her heart is fluttering. Side note my brother had two major heart attacks at age 46. My Grandfather on my Dad s side died of a heart attack and my Grandmother on Dad s side had a pace maker. My grandfather on Mom s side had heart disease. what can I say or do to get my daughter to go to a specialist? Doctor: HIWell come to HCMI really appreciate your concern, vasovagal syncope can't be it self because there are many apparent reasons behind this and without that it may not be vasovagal attack and this need to be ruled out and this is only possible when patient clinically examined and history elicited but this is self limiting because this is malaise mediated which directly related with vagus nerve, vasovagal attack causes low blood and not high blood pressure, your sister may not be having any cardiac disease, still it is advisable to get done cardiac evaluation test, in short this is nothing to worry, take care."
},
{
"id": 162974,
"tgt": "Can molds cause systemic juvenile rheumatoid arthritis?",
"src": "Patient: I read your Can mold cause rheumatoid arthritis and I have also read toxic mold systemic juvenile rheumatoid arthritis - Google Search and a blog about a 14-year-old girl with juvenile rheumatoid arthritis, was found to have Stachybotrys growing in her bedroom walls... the reason I reiterate all this is because my son got so infected by Stachybotrys and other toxic molds in our apt that he developed systemic juvenile rheumatoid arthritis and it affects his heart, lung and joints - so does this type of mold cause it or not? Doctor: Hello and Welcome to \u2018Ask A Doctor\u2019 service. I have reviewed your query and here is my advice. Much research is being done to find the cause(s) of systemic juvenile rheumatoid arthritis, but to date, it is not known. One cannot say that living in a house with mold is the reason the person has systemic juvenile rheumatoid arthritis. I am sorry your son has this severe type of juvenile rheumatoid arthritis. Is he receiving the best possible medical care? What kind of doctor follows him? There are many new treatments for this condition and they are known to the pediatric rheumatologist. Do you live near a university medical hospital? Because this medical facility is the best place to find this type of doctor. Also, should your son need care for other body parts, at this same hospital will be pediatric cardiologist, pediatric lung specialist, pediatric bone specialist, pediatric eye specialist, and physical therapists who work with children. Hope I have answered your query. Let me know if I can assist you further."
},
{
"id": 199433,
"tgt": "What causes persistent itching at the base of scrotum with redness?",
"src": "Patient: I've been experiencing a persistant itch from the base of my scrotum to my anus. The itching is accompanied by redness( unsure whether this is a symptom or result of rubbing) and a moist clearish discharge from the area. The issue has persisted some time regardless of clenliness etc Doctor: Hi,It seems that there might be having some local skin infection like dermatitis or fungal infection on the part.This can be due to having more perspiration, less airy part and moist local part.Clean local part daily while taking shower.Apply triple action cream.Wear cotton inner wears and avoid inner wear during night.Ok and take care."
},
{
"id": 165888,
"tgt": "What causes high fever despite giving Paracetamol and Mefenamic acid?",
"src": "Patient: My daughter is 22 months old. Last 2 days, she is having high fever 101, 102 like that.Yesterday, after giving paracetamol her fever still remained high. Nearby dr gave mefenamic acid and paracetamol. So that her fever was go down. Today also after giving paracetamol her fever is still 99. Should I give her paracetamol or mefenamic acid? Doctor: Hello, I understand your concern over the health condition of your baby. First of all, are there any other symptoms apart from fever? Like cough/ cold/ recent infection? If yes kindly mention the details. It's good that you consulted a doctor for this rather than waiting or just giving OTC drugs. In children, it's very important to give proper medical attention on time or else their condition gets bad over the time. The paracetamol will help to bring the fever down but it's going to take time at least few days. Along with the paracetamol, the baby needs antibiotics treatment as well. Syrup Myclav would be a good option. For the proper dosage as per the baby's weight, I would suggest you consult a pediatrician soon.You have not given details regarding the baby's weight so it's better to let a pediatrician examine the baby in person and advise you proper dosage of the medicine I suggested. As children are very sensitive to anything, even the medicines they are given. So the best I can suggest you is visit a pediatrician. If there is need of any other medicines or advice as per the child's condition, that shall be taken care of better. For now, as per the doctor's advice give the baby paracetamol for fever but do not wait longer for making an appointment with a pediatrician. Also keep the baby away from cold and things that are cold. Make sure environment around the baby is warm, clean. It will take few days to get the fever down completely so don't worry about it. Proper treatment will help the baby to recover quickly. Hope this helps. If you have any further queries feel free to contact. Stay healthy."
},
{
"id": 14557,
"tgt": "Suggest treatment for rash on upper butt cheeks during pregnancy",
"src": "Patient: I have a rash on both upper butt cheeks. They are small red bumps, but the whole area is a large circle. I also have similar red bumps on the lOwer side of my thighs. I am pregnant. I am not sure if this has anything to do with it, but it won't go away. Doctor: Hello. Thanks for writing to us at healthcaremagicJudging from the description, this seems to me like a Fungal Infection/ Tinea Cruris; Annular or circular patches with central clearing and peripheral raised margins which may be broken at places or composed of bumps. I suppose it must be itchy too.The location too seems to suggest that it is most likely a fungal infection; fungal infections are common at sites such as waist and pubic region.Fungal infection is not specifically common in pregnancy but weight gain during pregnancy may be a contributory factor.I would suggest that you may use an OTC topical antifungal clotrimazole 1% cream, twice daily for 4-6 weeks, regularly.Oral antifungals may be avoided during pregnancy.An oral antihistamine e.g hydroxyzine is safe in pregnancy and can be taken if itching is bothersome.Regards"
},
{
"id": 41136,
"tgt": "How can follicular size be increased for the next IUI cycle?",
"src": "Patient: hello, had my 1st iui cycle cancelled due to follicle not growing.on day 17,follicle size was 11mm of which i was placed on gonadotropin injections for 15days after taking femara from day 2-6 of my cycle. what do i do to increase my follicle size on next iui process Doctor: Hello, you need to get your hormone profile tested to see if they are in line and normal then try ovulation inductionIn case you have any questions in future you can contact me directly on http://bit.ly/drmanishajain"
},
{
"id": 83550,
"tgt": "Can zanax be taken along with zinnat?",
"src": "Patient: i am 51 years old and was being treated for depression and anxiety disorder since 2009. i am not on any drugs right now for it but is taking zinnat for an infection. i am experiencing side effects and it is causing anxiety. can i take zanax while i am on zinnat. Doctor: Hi,There is no documented harmful interactions between the antibiotic zinnat and the anti-anxiety medicine zanax, hence they may be taken together.Hope I have answered your question. Let me know if I can assist you further. Regards, Dr. Mohammed Taher Ali, General & Family Physician"
},
{
"id": 70774,
"tgt": "Are suspicious densities on both upper lobes a sign of PTB?",
"src": "Patient: Hi ...I m sindy I m 30 years old way back 2015 I had my xray and was diagnosed with PTB ..so I have undergone 6mnths of medications..after it I was fit to work but still it leaved a trace ...now 2018 I got my new xray result and it says that there is a suspicious densities on both upper lobes..does it mean that my PTB GOT BACK?!.and do I need to undergo another medications?!. How about the previous ?!can I still consider it to be fit to my work .or do I have to take another findings and get back to medications..I need some advice. The results made me more stressed due to I couldn t got back to work!!.. I need answers.. Doctor: Hello and Welcome to \u2018Ask A Doctor\u2019 service. I have reviewed your query and here is my advice. X-rays are inconclusive you need sputum sample tests or bronchoscopy to confirm the diagnosis. Treatment will start only if they are positive. Hope I have answered your query. Let me know if I can assist you further. Regards, Dr. Achira Dilanka"
},
{
"id": 100278,
"tgt": "Why do I feel breathless and have nasal congestion?",
"src": "Patient: Sir, I am from Berhampur Odisha. I have been suffering from breathing difficulties since 3 months. I have difficulties in taking breah. I have tested the blood, urine, BP, thyroid etc and everything came normal. Here the chest specialist has also conducted the spirometry test which was normal. My p-flow is 400. My nose is always congested and when I use Nasivion spray I feel relief in my nose and my breathing difficultiy vanishes. One should not use a nasal spray for a long time but I am bound to use that one since the Inhaler prescribed by the chest specialist does not help me. Kindly help me. P,/y Odisha Doctor: Hello,Thank you for asking at HCM.I went through your history and would like to make suggestions for you as follows:1. It is good that your spirometry is normal, I assume it was done with a \"post-bronchodilator study\". In that case, I would think your cause of feeling breathing difficulties may be nose obstruction.2. I would agree that decongestant sprays (which \"open\" a congested nose) should not be used for long time, they can cause rebound symptoms upon discontinuation.3. Were I treating you, I would prescribe you intranasal corticosteroids (mometasone or fluticasone) for at least 14 days. I would also prescribe you montelukast and levocetirizine for 28 days.4. I would suggest you to use an inhaler like salbutamol for breathing difficulty symptoms on as-and-when-needed basis only.5. Allergy testing for common air-borne allergens like house dust mites, indoor molds, regional pollens, insect proteins and pet danders (if you have pet) will be useful to identify the substances causing allergies to you as well as to know the measures to avoid them.Based on allergy testing, an Allergist-Immunologist may prescribe you allergen specific immunotherapy which gradually improves allergy symptoms by working on immune system.6. Regular breathing exercises, physical activity and a healthy diet rich in vitamins & minerals will also help you in a long run by improving your lung capacity and immunity respectively.7. In general, I would suggest you to avoid exposure to dusts, smokes and air pollution as much as possible.Hope above suggestions will be helpful to you.Should you have any further query, please feel free to ask at HCM.Wish you the best of the health ahead.Thank you & Regards."
},
{
"id": 6579,
"tgt": "Will I conceive immediately after stopping birth control pills ?",
"src": "Patient: i stop taking my birth control pills ...my last menstruation was feb 24,2011, i had unprotected sex on march25,2011 and my husband did ejaculate ...i dont know when i am fertile is there a chance that i would get pregnant? please help.. many thanks Doctor: Hi,yes there is definitely a chance that you could be pregnant.You should get a pregnancy test. Good luck!"
},
{
"id": 38517,
"tgt": "What causes redness,swelling and itching on leg?",
"src": "Patient: Hi , Wednesday evening i had a tender groin and a little swelling at the groin/top of thhigh area with redness. By the next morning the redness had started to go down my leg and it has been itchy and warm with some swelling where it is red . It also feels like i have a thick hard vein? Or something like that going down from top of thigh to where the redness stops . It was still a little red this morning and i thought it was going but by late afternoon it s started to get itchy again . It is warn to the touch . I got bitten by something on Tue afternoon on my calf and by wed morning and my calf was very swollen but that seems to be less hard and swollen today . Wondered if it was to do with the bite . Thanks Angela Doctor: Hi, Angela. Certainly this could possibly be related to the bite, but there are 2 other possibilities that require prompt evaluation. One of these is a blood clot in the deep veins of your leg. This can create swelling, redness, heat, and pain. An infection in the skin can create similar symptoms and eventually spread into your blood. If you are still having any of these symptoms or the thick hard vein, get checked at an urgent care center or emergency right away. An untreated blood clot or infection like this could progress into a life-threatening situation. Hope this answers your query. If you have further questions, I would be happy to answer them."
},
{
"id": 116519,
"tgt": "Is Raynaulds causing numbness and tingling in my hands?",
"src": "Patient: I am 54 years of age female 5'2 and 9stone. I have SLE leading a good healthy life with mild symtoms. I have Raynaulds. Over the past few days I have noticed pain in both hands numbness tingling particularly in my right arm but occasionly in the left. I have been short of breath on and off for awhile. Do you it will pass or should I consult my GP? Doctor: Hi, dear. I have gone through your question. I can understand your concern. Your symptoms are very common in raynoud syndrome. Even SLE may produce this. No need to big worry. Just consult your doctor and take treatment of your raynoud syndrome. Hope I have answered your question, if you have doubt then I will be happy to answer. Thanks for using health care magic.Wish you a very good health.Don't forget to click thank you."
},
{
"id": 198354,
"tgt": "What causes erectile dysfunction at the age of 58?",
"src": "Patient: I AM 60 YRS AND I AM SUFFERING FROM E.D SINCE MORE THAN 10 YRS I HAD APPROACHED A SEX SPECIALITY WHEN I WAS 58 YRS OLD HE ADVISED ME TO TAKE M POWER CAPS TWICE DAILY AND TAZZLE 10 ONCE A WEEK I WAS SATISFIED FOR A MONTH OR TWO WITH GOOD ERECTION. BUT LATER SUFFERED WITH MY EYE WHICH WAS GETTING RED AND BLURED WHICH I HAD TO CONSULT AN EYE SECIALIST WHO HAD TO GIVE ME SOME EYE DROPS [STEROID TYPE] TO CLEAR MY REDNESS AND CLEAR MY VISION. AS SUCH I STOPPED TREATMENT. NOW AFTER A BREAK OF 2 YRS I CONSULTED AGAIN AND HE HAS ADVISED ME TO T AKE NANO LEO TWICE DAILY AND KUTUB 30X ONCE IN 10 DAYS. I DO GET AN ERECTION TOWARDS EARLY MORING SAY 3 O CLOCK FOR A SHORT PERIOD BUT I COULD GET A PROPER ERECTION ONLY WHEN I TAKE KUTUB 30X ONCE IN TEN DAYS WHERE I COULD PENETRATE AND CAN GET BEST SATISFACTION PLEASE ADVISE WHETHER I COULD TAKE KUTUB 30X AT FREQUENT INT ERVALS AS I AM A DIABETIC - BLOOD SUGUR IS UNDER CONTROL FBS 112 PPBS 144 Doctor: Kutub, if suits you may be taken but occasionally the doctor should be consulted.Alternatively as per Ayurveda principles, 'vrushya' group of herbs have vajikaran and Rasayana effect and are safe. In simple language some herbal groups responsible to increase semen, are aphrodisiacs and rejuvenators without side effects. Effect may be little slower but sustainable.If you agree, pl give little more info through direct question. Ideally seven weeks' treatment will suffice to maintain the erection on stimulus, provided you follow food protocol and your diabetes remain under control.Hope it helps."
},
{
"id": 95674,
"tgt": "Having loose motion from 2 days, what can i take ?",
"src": "Patient: hi doc,I am Garima,i have loose motion from 2 days,its my fifth mont is running,yesterday i had tkn radotril bt no relax then my doc has changed n advice me to tk lomotil ...still i am nt feeling well...gv me some advice wt sud i do...n wt sud i eat Doctor: Thanks for the query I have noticed u dint mention about fever so I am assuming u don't have it. Take probiotics like nutrolin b or sporolac. Dont take ritodril as it is not proven safe is pregnancy Have a healthy living"
},
{
"id": 192095,
"tgt": "Suggest treatment for fever, diarrhea, fatigue and vomiting in a diabetic",
"src": "Patient: Husband is diabetic, had a sudden onset of diarrhea and fatigue, vomited once (a lot of it), was running a fever of 100.7. Gave Tylenol every 4 hours for about 4 doses and temp is now around 96.6. Was thinking it was some type of bug but this is second full day of not feeling well. Says he feel some better after fever came down but still bad enough to stay in bed. Doctor: HIDon't worryGive ORS With Mineral water, 500 ml every half hourly-In diet-curd,porridge,Banana,coconut water to check dehydration-Avoid fruit juiceThanks"
},
{
"id": 151573,
"tgt": "Disc Degeneration, protrusion of disc, severe pain in the tailbone, difficulty in sitting, lower back pain. No improvement with pain killers. Effective treatment?",
"src": "Patient: Hi Doc, I am a manager working with MNC, I am working since 15 years in the field of Sales and Marketing. There is good amount of travelling. Also at office or dealers I am on sitting job. Lately since 1.5 years, I have been through tough carrer times and mostly on sittlng job . I am 32 and my weight is 94Kgs. Earlier I used to go to gym and my weight was 84 Kgs but since last 2 years I have left all excercies. Since last 1 year I had severe pain at the tail in between my buttox, It was getting difficult to sit and whenever I got up, I used to have a feeling that my lower back is cemented and hard it used to be very painful. Even smallest jerk on bike used to make me yell of pain. The pain has increased in last 1 year and still it is very difficult while sitting down and getting up.Last month I have come to know that I have Disc Denegration and protrution of disc. Doctors have taken MRI Scan and my L4 and L5 disc is damaged, the liquid and soft cartilage is totally damaged and also there is slip disc. they have given treatment and prescription of Pain Killers and Pain relieving gel to apply and also take treatment of (Traction, hotpads and physiotherepy). I have taken 8 days treatment, But again after 1 month the same problem started. Now I want permanent cure, Please suggest the best treatment in allopathy , Ayurvedic , Unani,Homeopathic or any other traditional treatments. Doctor: Hello welcome to health care magic please visit the best ayurveda and panchkarma center near you.No need to worry ,panchkarma can treat you problem for sure .\"kati basti\" has cured many people. Kati Basti is a procedure of applying heat to the sacral or lumber region by retaining warm medicated oils, which are specially formed on that area. mail me at payalrawatchauahan16@gmail.com for diet ptescription and the procedure of kati basti. take care thanks"
},
{
"id": 141554,
"tgt": "What causes fever and swelling in forehead?",
"src": "Patient: My brother has been very ill with a high fever and very weak. He thought he had the flu but what worries me is he told me his forehead swelled up and now the top and back of his head are swollen. Hurts to touch and feels like its on fire. What could that be? Doctor: Hello and Welcome to \u2018Ask A Doctor\u2019 service. I have reviewed your query and here is my advice. I would keep him generously hydrated with water and get him consulted by a doctor. They will need to check him out with some blood work and an examination as well as some urine testing. Quite likely some type of imaging study of the brain and maybe even a spinal tap. Hope I have answered your query. Let me know if I can assist you further."
},
{
"id": 147726,
"tgt": "Should an adolescent be prescribed Vinlep or Proleva after suffering from brain attack?",
"src": "Patient: My son (13 yrs , 37 kg weight) has attack on brain before week. We have gone thru MRI and other required test which are normal Out of opinion of two doctor s one has prescribed VINLEP 150 and other has prescribed TORLEVA(500 MG) WHICH ONE SHOULD PREFERRED ? Pl. guide as early as possible. Doctor: Hi. I'm so sorry to hear about your sons troubles. The first thing that i must try and understand before I can help you is what you mean by \"Brain Attack\". Some people use that term for Strokes and other people use that term for Seizures (Fits). Based on the medications you have mentioned here, I am going to assume you are speaking of a seizure.Both medications are good for the control of Seizures, though I prefer Torleva.When it comes to the management of seizure disorders, you must patiently wait and see which drugs work best for you. You might need to go through multiple combinations until you get to the one that works best. I hope this helps. Thanks for your question. If you would like you can Ask A Doctor on our website and they will be able to discuss this issue with you in detail."
},
{
"id": 72042,
"tgt": "How to cure sarcoidosis which is giving major problems to knees with trembling hands?",
"src": "Patient: I have sarcoidosis and it is giveing me major problems with my knees. My dr has me on methotrexate and methylprednisolone. I am starting to have severe pain pulsating in my right upper abdomen. This is happening 30-40 times each day. It last 20 to 30 seconds each episode. This is been happening for about a week. The last few days I also have started trembling in my hands. This does not happen at the same time. What do you think I should do? Doctor: HelloI agree with the current treatment but these might be side effects of medication discuss with your doctor for this.RegardsDr.Jolanda"
},
{
"id": 202537,
"tgt": "How to treat itching on all over outer penis skin?",
"src": "Patient: Dear Sir/Madam, I was having itching on all over my outer penis skin, so i used dettol on those parts to clean it, but nw all over my penis there is burning sensation and there are many burn patches, which is now hurting me a lot. pls advice medication for the same along with medication for itching. Doctor: Hi,It seems that you might be having local allergic reaction on your penis.Using undiluted Dettol precipitate the condition.Clean the part with running plane water.Apply some moisturizing cream locally.Take antihistamine like Cetrizine or Benadryl for itching for 2-3 days.Ok and take care."
},
{
"id": 217191,
"tgt": "How should acute spinal pain despite taking steroid epidural be treated?",
"src": "Patient: Hi I just recently got my second steroid epidural in my lower spine and for at least the past four hours any slight movement I make hurts like hell I am already on fentanyl patches Percocet Ibuprofen Tylenol gabbipentin Valium and cymbalta, and even with all of those it is barley taking the edge off of the pain and I don t know what I should do... Doctor: as per your explanation i realise you have a Severe back pain. I suggest not to treat the symptoms of pain but do proper evaluation for the reason of back pain. you already have taken enough of pain killers and steroid injections even after this if there is no relief you must go for mri of spine and consult again your ortho as well as physio doctor for further treatment and planning here ortho will give options of surgical and medical based on your mri report. suppose if there is no severe problem then you can consult a physio and take the physiotherapy treatment like ultrasound and IFT which will help you further for pain relief also take proper spinal exercise program to continue doing exercise at home this will help you to improve strength and stamina of your spinal muscles. you can also use hot and cold pack at home frequently and regularly on a daily basis to have some more pain relief by anti inflammatory as well as relaxing effect of this.for now if you have developed sudden pain then it can be a catch at your back and you need to take muscle relaxants. for this you can contact your doctor again. I am sure this will help you. take care."
},
{
"id": 180267,
"tgt": "What causes green loose bowel movements in a baby?",
"src": "Patient: Hi my daughter is 7 months old...... from the last few days she is suffering from loos motions.......n mostly her stool is of green in colour.... please suggest me possible ways to cure this problem.... am working mother.... when i was not at home we were giving formula milk but when i reached home from that time only i start breastfeeding. Doctor: Hi...Thank you for consulting in Health Care magic.i appreciate your eagerness to breast feed your baby though you are working.It seems your kid is having viral diarrhea. Once it starts it will take 5-7 days to completely get better. Unless the kid's having low urine output or very dull or excessively sleepy or blood in motion or green bilious vomiting...you need not worry. I suggest you use zinc supplements (Z&D drops 1ml once daily for 14 days) & ORS (Each small packet mixed in 200ml of potable water and keep giving sip by sip) as hydration is very important and crucial part of treatment. If there is vomiting you can use Syrup Ondansetron as prescribed by your pediatrician.Hope my answer was helpful for you. I am happy to help any time. Further clarifications and consultations on Health care magic are welcome. If you do not have any clarifications, you can close the discussion and rate the answer. Wish your kid good health.Dr. Sumanth MBBS., DCH., DNB (Paed).,"
},
{
"id": 83082,
"tgt": "Systemic lupus, sharp pain below collar bone, around ribs post eating. Taking plaquenil. Causes?",
"src": "Patient: I was recently diagnosed with systemic lupus and started plaquenil just over a month ago. Just today I started having very sharp pain above and below my left collar bone and around the lower left side of my ribs....just below my ribs on my side, every time I eat anything. It eased off some after taking advil and gas- x, but returns each time I eat. What could this be? Doctor: hi it could be gastritis since you are taking plauenil. so you could take an antacid and see if it subsides. the other could be a pleuritis since you suffer from lupus,but that seems less likely."
},
{
"id": 50134,
"tgt": "Blood in urine, frequent urge to urination, painful bladder spasms, lower back pain. How is lupus nephritis treated?",
"src": "Patient: How is lupus nephritis treated? I m a 36 yr female diagnosed with SLE 12 years ago. I also have fibromyalgia , migraines, fibercycstic breast disease , TMJ , chronic dry mouth and eyes, Siracusa of the lower, and scoliosis. I m currently taking 2000mg of cellcept, 400mg plaquenil, 100mg savella, 100mg ultram er, tramadol/aet, voltren gel, voltren, omega 3, vitamin e, multi vitamin pack, and calcium. For the past 3 month I ve to several doctors trying to find out why I still had a uti. I recently found out on Friday I don t have and never had a uti. I have blood in my urine, chronic feeling of having to go urinate only to have nothing come out, painful spasms of the bladder, and some lower back pain. Doctor: wht does your urine routine examination amd culture report say ?the symptoms you described are very much suggestive of a UTI, urinary tract infection.lupus nephritis treatment involves following :1) control of symptoms like joint pains fever etc by medicines like plaqnil you are taking.2) disease modifying drugs which control activity of disease and progression of complications like cellcept and wysolone3) treatment of complications like nephritis with control of swelling and anemia, and if itis fairly advanced then even dislysis and kidney transplant."
},
{
"id": 125457,
"tgt": "Suggest treatment for dystonia",
"src": "Patient: i am 35 years old woman . since 10 years i had found signs of dystonia.but only now doctors said this is dystonia.now i am completely bed ridden.can not write and hold things. doctor is there any treatment to improve my condition.if any body has time i can give my details.waiting for replay thank you Doctor: Hi, Please post the details in this forum and some doctor, with your disease specialization will try to help you. Hope I have answered your query. Let me know if I can assist you further. Take care Regards, Dr Gopal Goel, Orthopaedic Surgeon"
},
{
"id": 44199,
"tgt": "Trying to conceive. On follicular study, on day nineteen needs to take a flight. Safe?",
"src": "Patient: Hi doctor... Im XXXXXX. Married a year ago. No baby yet. Im taking treatment for the same.... From my second day im monitering my follicle and taking necessarysteps. My questio is : During my 19th day i have to fly to my native due to transfer... Is it safe to travel in flight or i can go via train (duration 2 days)... Pls help me...since i cant contact my doctor Doctor: Hi, you can travel either by flight or train. Both are safe. You have tried naturally only but with minimal medical assistance, so nothing to be so worried about. Make sure you take your medication as prescribed by your gynecologist and try avoiding too strenuous physical work. Pregnancy test can be done as advised by your doctor in future. All the best."
},
{
"id": 69333,
"tgt": "What is the lump on the neck causing no sickness?",
"src": "Patient: Please help..30 year old female 155 lbs, painful lump on left side of neck about two inches below earlove, begain a few days ago, supposed to get my period this weekend and did not, not sick and am taking LoEstrin 24 that is all. What is going on? Thank you! Doctor: Hi.Thanks for your query.The lump at a location you described is usually due to inflammed lymph node. You have to get a course of an antibiotic and anti-inflammatory medicines. This should regress in 5 to 7 days. Consult a Doctor id not regressing for further investigations ans treatment."
},
{
"id": 172005,
"tgt": "How long to use the movicol?",
"src": "Patient: My 14 mth old was prescribed movicol yesterday, he s got a follow up app next week to maybe change from movicol to something else but I wondered how long he could be on it for as I ve heard a few people say family members/ kids have been on it for years. Doctor: Hi...Thank you for consulting in Health Care magic. I think your kid is having habitual constipation. I have certain questions and suggestions for you.Questions:1. Did your kid pass motion or meconium on day one of life?2. Since how long is the kid constipated?3. Does the kid have any bleeding along with hard stools?4. How much milk does the kid consume per day?5. Does the kid eat fruits and vegetables (fibre diet) appropriately?You can get back with answers at the following link - www.healthcaremagic.com/doctors/dr-sumanth-amperayani/67696Suggestions:1. Natural methods are the best to relieve constipation.2. Constipation is a risk factor for UTI3. Maximum milk consumption per day should not exceed 300-400ml4. Minimum 3-4 cups of fruits and vegetables to be consumed per day5. Toilet training - that is - sitting in Indian type of lavatory daily at the same time will help a lot.Hope my answer was helpful for you. I am happy to help any time. Further clarifications and consultations on Health care magic are welcome. If you do not have any clarifications, you can close the discussion and rate the answer. Wish your kid good health.Dr. Sumanth MBBS., DCH., DNB (Paed).,"
},
{
"id": 80685,
"tgt": "Suggest treatment for chronic cough with phlegm",
"src": "Patient: My husband has had a chronic cough for about 15 years. He as taken hydrocodone syrup until about 5 years ago to calm the cough. Is there anything non narcotic that can be prescribed? he has gone through every test imaginable, thousands and thousands of dollars spent for a non diagnosis. He is healthy in every other way. He works outside as a general contractor and is in meetings daily...really needs help. some symptoms...not a dry cough. coughs up phlegm and sometimes a pea sized ball of hard phlegm. once the cough starts it can last 10 minutes or longer. Doctor: Just take syrups having contents like guiaphenesin, ambroxolol, ammonium chloride etc are non narcotic & will help remove phlegm. Tab. N acetyl cysteine also will help thinning secretions. But if your chest condition is not diagnosed properly ,then 1st consult for proper diagnosis, & if related to chronic bronchitis, then above treatment & other bronchodilators according to your consultant will suffice"
},
{
"id": 161432,
"tgt": "Suggest remedies for loose bowels and bloating in a child",
"src": "Patient: My 3 year old daughter has had problems with her bm s for a long time now. For a few months she was taking lansoyl every day to help her stay regular but sometimes she would still get bloated and gassy. We stopped using that to try her and see how she would do. We thought she might be lactose intolerant and we tried lactose free milk and she did ok but to see if that was the problem we quit that milk and started her on whole milk but she got really bloated and gassy, and sometimes going 3 times a day and sometimes runny and sometimes solid, and leaving stains or little nuggets in her pullups. For 3 days she has been back on lactose free milk but her bm s are still runny and really stinky, although now it s only once a day. Does this sound like lactose intolerance or could it be something else? Doctor: Hello,It may be due to stomach infection. For further assessment she may require complete hemogram, stool analysis, ultrasound abdomen sos after pediatrician consultation. She may require antibiotics after consultation. Give hygienic food and water. Keep her hydrated. Hope I have answered your query. Let me know if I can assist you further.Regards, Dr Shyam Kale Family and general physician"
},
{
"id": 41151,
"tgt": "What are the success rate of IVF treatment with donor eggs?",
"src": "Patient: I am a 33 yr old female-wt. 51kg-ht.157cm, asked to go in for IVF with donor eggs as my ovaries have shut down. Both my family and my husband's family(my mother has breast cancer, my mother in law had ovarian cancer) have history of cancer. Are there any serious side effects of this treatment. what will be the approx. cost for the IVF treatment with donor eggs. Since the baby will not be genetically mine, will I have a stranger for a baby? Will it still be mine? what is the success rate? Doctor: Hello, there will be no problem and difference in rearing of the child.Donor egg will eliminate the need of ovulation induction drugs on your body making it even more saferIn case you have any questions in future you can contact me directly on http://bit.ly/drmanishajain"
},
{
"id": 182381,
"tgt": "Suggest remedy for reoccurring mucous cyst on the inside of lower lip",
"src": "Patient: Hi,I have had a mucous cyst on the inside of my lower lip. I have had it for about 2 weeks in January. It is not painful and it decreases in size and then returns. So I went to see a doctor who recommended surgery. All went well for a week after surgery at the beginning of February. However on the second week the cyst appeared again and it has reached the same size as before. The doctor has recommended surgery again. I am afraid that this cyst may reappear after the second surgery. What should I do? Doctor: Thanks for your query, I have gone through your query.The causes for recurring mucous retension cyst is because of the improper removal of the cystic lining or repeated trauma to that region like lip biting.So consult a good maxillofacial surgeon and get it enucleated or surgically removed. Then we have to remove the cause like any trauma from the teeth to the lip. If any sharpness over the teeth are there, then it has to be rounded off. Nothing to get afraid. I hope my answer will help you, take care."
},
{
"id": 143326,
"tgt": "What is the cause of pulsations in the temple?",
"src": "Patient: i can feel a vein pulsate on the left side of my temple...it hurts..it hurts even more when i stand up..or look at my computer..i also wear glasses..but not all the time..only when i play video games because their to fragile to wear anywhere else..what does that mean? i did not know which specialty to choose. Doctor: Hi,Regarding your concern, you should know that your symptoms could be related to compression of the temporal artery from your glasses. Anyway, I would recommend performing inflammation tests (complete blood count, PCR, sedimentation rate) to exclude temporal arteritis which could mimic some of your symptoms. But if the above tests result normal, you should try to wear another pair of glasses, maybe more large, to avoid pressure on the temporal region. Hope to have been helpful. Let me know if I can assist you further. Kind regards, Dr. Aida"
},
{
"id": 219674,
"tgt": "Suggest remedies for healthy pregnancy without affecting with down syndrome",
"src": "Patient: Dr.I have got a child of 3 yrs with down syndrome and now got a pregnancy with down syndrome baby again which got aborted, now guide me how can I get a normal baby as there is no history of any genetic disorder in neither mine nor my husband s family Doctor: you and your husband make an appointment with a genetic counseller for the advice. He or she will be able to explain and give you the risk factors."
},
{
"id": 193639,
"tgt": "What is the treatment for swollen bumps in the groin and erectile dysfunction?",
"src": "Patient: Sir, my name is Prince ,am 22, i weigh 93kg. i have been having persistent bumps for the past 2 to 3 years now, bumps that come out whether i shave or not and produce puss. There are also internal swellings on the sides of the groin and noticeable skin irritations that itch. Currently, am having very weak erection, infact, i feel like my penis is shrinking. If my penis eventually erects, my performance is very poor. Please help me, i will love to know what is responsible for this, if it is an infection and what infection it is. Prince from Nigeria. Doctor: Hi, If you had unprotected sex - need to rule out sexually transmitted infection. If not it can be bacterial / fungal infection - share picture for conformation. Local application of antibiotics / fungal can help you. Other issue of your sex can be due to anxiety because of the groin bump. Hope I have answered your query. Let me know if I can assist you further. Take care Regards, Dr S.R.Raveendran, Sexologist"
},
{
"id": 78472,
"tgt": "What could be the reason for sharp stabbing chest pain?",
"src": "Patient: I HAVE A SHARP STABBING SEVERE CHEST PAIN THAT COMES AND GOES, IT LITERALLY MAKES ME STOP EVERYTHING IM DOING AND ITS VERY SCARY. ITS ON MY LEFT SIDE UPPER AREA. I HAD MULPITAL CLOTS IN MY LEFT LUNG AND I NOW TAKE 15 MG OF COUMADIN DAILY. I WENT TO SEE CARDIOLIGIST AND HE DID A D DIAMER TEST AND THE RESULTS WERE GOOD NO CLOTTING FIBERS, BUT THE PAIN IN MY CHEST IS STILL HERE. I WAS DIAGNOSED WITH PROTEIN S DEFIENCY WHEN THEY FOUND THE CLOTS 8 MONTHS AGO. I AM REALLY SCARED AND WHOULD APPRECIATE ANY ANSWERS. MY DEAREST RESPECT FOR THE DOCS HELPING PEOPLE WHO NEED THEM MAY GOD BLESS! Doctor: Thanks for your question on Health Care Magic. I can understand your concern. You are having protein S deficiency. It is hypercoaguable state. Intravascular clotting is common in this condition. So clotting and obstruction in the coronary arteries is also common. And this can cause sharp, stabbing, left sided chest pain. So better to get done 1. Ecg 2. 2d echo 3. Stress test (trade mill test) 4. Coronary Angiography if required. If all these are normal then no need to worry much. Musculoskeletal pain can cause similar symptoms. So avoid movements causing pain. Avoid heavyweight lifting and strenuous exercise. Take painkiller and muscle relaxant drugs. Apply warm water pad on affected areas. Don't worry, you will be alright, but first rule out heart diseases. Hope I have solved your query. Wish you good health. Thanks."
},
{
"id": 8115,
"tgt": "I want a cure for acne oily skin and remove the effects of stains, please help",
"src": "Patient: I want a cure for acne oily skin and remove the effects of stains and holes from the face I m female, I am 25 Snpeani of oily skin and pimples on the face of the top filled with white liquid Log in, and I have some brown spots and the effects of drilling. Previously, my skin is very normal for 4 weeks, suddenly became my skin greasy. Can you help me necessary medicines. Doctor: HI.......I I M ALSO SUFFERING FROM THIS PROBLEM FROM 1 YEAR BUT NOW FROM 2 MONTHS I HAVE SEEN A LOT OF DIFFERNCE ON MY FACE I USED TO HAVE ALL MY FACE WITH PIMPLES BUT NOW I CAN HARDLY SEE A PIMPLE ON MY FACE........U SHOULD ANY SKN PURIFIER DAILY BEFORE GOING TO BED N EAT 1 MINOZ TAB 50 DAILY N WASH UR FACE WITH HIMALAYA FACE WASH TWICE DAILY.FEEL THE DIFFERENCE IF IT WONT WORK I DNT THINK THN ANY THING CAN WORK THN,,,,,,,,,N ALSO U CAN USE HONEY WITH MILK ONCE IN A WEEK FOR 15 MINS,,,,,,DU REPLY ME THN...INSHALLAH IT WILL WORKOUT....THANK U."
},
{
"id": 99932,
"tgt": "What causes the split in nostrils and should I use saline to provide moisture?",
"src": "Patient: Every year I have a split in the skin (you can actually see the split with the two thicker sides) of my right nostril. I use to assume it was from dry air in the winter but here it is towards the end of April and even the Neosporine won t heal it. I do not have allergies and I happen to be a mouth breather (don t know why as if I concentrate I can breath through my nose). My question is, what actually causes the split? Should I be using saline to provide moisture? Thanks! Doctor: Bries answerAtriphic rhinitisDetailed answerHiGreetingsThanks for your query and welcome to the HCM what you are experiencing is just formation of crusts in your nostrilsExposure to dry and cold air can lead to atrophy of nasal mucosa and produces symptomsI would advise my patients to keep the nostrils well moisturised with normal saline and removal of crustshope this helps you, feel free to ask for follow up best luckREGARDSDr BR Hudda"
},
{
"id": 84111,
"tgt": "Is it safe to take doxycycline hyclate and rifampin together?",
"src": "Patient: Is it o.k to take doxycycline hyclate and rifampin at the same time? I have just been diagnosed with MRSA. i HAD A TOTAL KNEE REPLACEMENT IN JANUARY BUT THE INCISION HAS NEVER HEALED. FINALLY, MY DEMATOLOGIST TOOK A CULTURE AND I JUST GOR THE RESULTS NOW. Doctor: Hi,It is not safe to take these two antibiotics together. Rifampin is one of the most powerful liver enzyme inducers which if given along with doxycycline will increase its metabolism and increase its clearance from the body thus decrease the effects of doxycycline. Contact your doctor for a dose adjustment of these medications or to prescribed alternate effective antibiotics for the eradication of MRSA.Hope I have answered your question. Let me know if I can assist you further. Regards, Dr. Mohammed Taher Ali, General & Family Physician"
},
{
"id": 113871,
"tgt": "What can I do for severe back pain even after a surgery ?",
"src": "Patient: Hi had back surgery four weeks ago tomorrow s1 decompression still had pain after the surgery in left buttock and left leg and numbness in left foot had nerve block injection one week after surgery no relief still taking a lot of pain relief mainly endone also went back to Nero last week and I am now on neurtin but no relief as yet please help Hi had back surgery four weeks ago tomorrow s1 decompression still had pain after the surgery in left buttock and left leg and numbness in left foot had nerve block injection one week after surgery no relief still taking a lot of pain relief mainly endone also went back to Nero last week and I am now on neurtin but no relief as yet please help Doctor: Hello Mark, . You should be getting yourself Re-examined by your Orthopedist so that he can examine you thoroughly and suggest MRI or CT scans or if necessary Nerve Conduction Studies to know what exactly is causing your Pain. You History suggests, either you have Sciatica or Herniated Disc. These has to be ruled out so that Proper Treatment could be initiated or to know if the compression is still there. You are on right medications, just take them as directed by your Doctor and go for regular follow up. Do regular exercises which don't strain your Back, which could be low impact aerobic activities, if you smoke or Drink, QUIT. Maintain a healthy weight. Avoid high calorie foods and eat healthy nutritious balanced diet which should include fresh fruits and Green leafy vegetables. Avoid stress and Practice relaxation techniques like Yoga and Meditation. Wish you Good Health."
},
{
"id": 191622,
"tgt": "How can gastroparesis be treated while suffering from diabetes?",
"src": "Patient: Hi, my adult son is a type 1 diabetic, also a dialysis patient. He ALSO has gastroparesis, and his body is nothing but skin and bone. The endochrinologist remarked last week that we may have to consider a feeding tube for him! Are there different stages or categories of gastroparesis? If so, what are they? THANK YOU SO MUCH for any insights you can share! I m sorry, I ve only just realized I must pay for your answer. I don t have my card with me today. SORRY to have wasted your time. Doctor: HelloI have gone through your question and understood your concern.Gastroparesis is a condition that affects the normal spontaneous movement of the muscles in your stomach.If you have gastroparesis, your stomach's motility is slowed down or doesn't work at all, preventing your stomach from emptying properly.Diabetes is a major risc factor that increase the risc of gastroparesis.Many people with gastroparesis don't have any noticeable signs and symptoms.But depending to the cause ,the situation can aggravate and gastroparesis can produce lots of symptoms like nausea,vomiting,a feeling of fullness after eating just a few bites,acid reflux,lack of appetite,changes in blood sugar levels ,weight loss and malnutrition.My answer for you is:Yes,there are two grades of gastroparesis.Grade 1, or mild gastroparesis, is characterized by symptoms that come and go and can easily be controlled by dietary modification and by avoiding medications that slow gastric emptying. Grade 2, is characterized by moderately severe symptoms.Hope i have clarified all your doubts.If you have other questions,feel free and ask.Regards."
},
{
"id": 50796,
"tgt": "Pregnant. Diagnosed kidney stones. History of kidney stones in previous pregnancy as well. Reason and prevention?",
"src": "Patient: Hi,I am a 28yr old and I am 20-21 weeks pregnent for my second child. At the second trimister I have started to have severe left side back and abdominal pain. A urine test was done and there seems to be stones so a scane was given to be done later on.The scane shows that,In the down pole of the left kidney tiny calculus in size of -3.4 mm was detected.During my 1st pregnency also I faced the same problem. I had the same pain and 3 times small stones were passed while urinating.I face this problem only during my pregnency.I checked after the 1st delivery and there were no more stones. Y is that? and what can i do to prevent this hapening to me during my pregenecies? Doctor: Hi Welcome to the forum. I understand the history. What i feel is that proper evaluation is still to be done. Either a CT SCAN or a MRI is a must to completely evaluate the urinary tract. This can be done once you complete the pregnancy successfully. Stones are quite treatable and there should be no worry. best regards"
},
{
"id": 217463,
"tgt": "What causes pain in lower left side of back?",
"src": "Patient: I've been hurting down front of my body, where my vaginal is but on the inside for about past 2 months and it gets to the point where I can't sit because it hurts really bad but it's a sharp pain and it goes away after about 40 seconds. Now I've gone to a doctor and they found a cyst on my right ovaries and this was in February 2014.And now for some reason my lower left side of my back right next to my left hip has been hurting really bad for the last 4 days. It hurts to breath sometimes or move certain way. It's a really sharp pain. Then I started my period yesterday and it is really chunky. Why is that? And why is all this pain happening? ... And I've been trying to get pregnant for a while now. Been having unprotected sex for a year. have nothing happened yet. What can I do to help speed up the process? Doctor: Welcome to health care magic. 1.The history and symptoms and location of the pain suggest the possibilities of spinal disc pathology causing pressure effect on the exiting nerve - this will be the first possible cause.2.Treatment in this case will be back exercises - on daily basis will release the nerve compression and muscle spasms.3.The next possible cause could be the pelvic pathologies and cyst, depending on the size of the cyst and its nature and effect on the adjacent organs. 4.The reasons for not able to conceive after 1 year - need to get a HSG - A hysterosalpingogram (HSG) is an X-ray test that examines the inside of uterus and fallopian tubes and the surrounding area. 5.And a follicular study to monitor the process of rupture free fluid / following the ovulation calendar is needed. Good luck.Hope i have answered your query,any thing to ask ? do not hesitate to ask.http://doctor.healthcaremagic.com/doctors/dr-ganesh/62888"
},
{
"id": 38329,
"tgt": "Can prednisone cause itching?",
"src": "Patient: I was treated 6 days ago for poison ivy with a shot of solumedrol and then a tapered week of oral prednisone but over the last two days my whole legs, waist, and the backs of my arms have begun itching with no apparent rash. The itching does not allow me to sleep Doctor: Hello, thank you for your contact to healthcaremagic. If I am your doctor I advice you to that steroid won't cause itching, instead it relieves the itching. So there must be some other thing behind this itching. If you want to ask me anything you can ask me. Thank you. Dr Arun Tank. Infectious disease specialist."
},
{
"id": 109168,
"tgt": "Suggest treatment for frequent back pain",
"src": "Patient: hello doctor, i am 32 year old female. I am suffering from back pain very frequently.site is lumbo sacral, and around sacro iliac region. i am driving two wheeler daily for around 40 kilometers/day. I work for 6 hours sitting on the chair only. my height is 168 cm.weight is 69 kg. needs ur advise.thanks Doctor: Hi,From history it seems that this problem is due to your life style.Driving two wheeler long distance with sitting of six hours gives some postural strain giving this problem.Take some nap during your work and move about for some time.Do back extension exercises in the morning daily.Take some multivitamin supplements.Ok and take care."
},
{
"id": 67987,
"tgt": "What causes painful and reddish lump on butt cheek?",
"src": "Patient: I have this large reddish lump on my butt cheek it hurt really bad when I sat down. Also it was very hard to pop and when I did blood and puss came out. Afterwards it looked kinda black. It s been a few days now and there s still a lump there it s red and pealing around it. Can u please tell me what it is? Doctor: Hi,This is an abscess that may still be infected. I suggest you see your doctor for examination as you may need antibiotics. Regards,Dr K A Pottinger"
},
{
"id": 2056,
"tgt": "Is it safe to conceive if diagnosed with TB?",
"src": "Patient: Hi doctor, my wife age 30 is diagnosed with TB. due to which miscarriage happens. as baby was getting devloped at starting of right side tube. she was operated and tube is safe. She is taking MACOX Plus 5 month course right now. 2 month course of MACOX ZH is completed. Is it safe to conceive right now. as our doctor told that she can conceive. please suggest. Doctor: Hallow Dear, It seems your wife had ectopic (tubal) pregnancy which had to be managed surgically. It is a good sign that in spite of tuberculosis she had conceived. This proves that her tubes are patent. It is likely that her uterine lining has been damaged by tuberculosis. She is on an anti-tuberculosis treatment. Macorm Plus contains Rifampicin(600mg) and Isoniazid(300mg). These medicines are used for treatment of tuberculosis. Just 2 months treatment is not sufficient; she will have to take at least 6 to 9 months anti-tubercular treatment before she is completely cured of tuberculosis. Please do not think of pregnancy for her till she is declared completely cured of tuberculosis. She will be cured; Tuberculosis is a disease which can be completely cured when treated in time, scientifically and completely. Please follow your physician's instructions religiously. Dr. Nishikant Shrotri"
},
{
"id": 224295,
"tgt": "18 yr old, on birth control pills. Missed pills, having heavy brown bleeding. What to do ?",
"src": "Patient: Hi I am 18 years old and I m taking the birthcontrol pill . I take them everyday at the same time but recently with work I ve missed about 4 days. I started to bleed and I thoughtt or was my period. But it only lasted 4 days. So I took some pregnacy test and they all showed up negative. 2-3 days later I ve started to bleed again but brown and everyday its getting heavier and heavier Ans I need to take my new pack of pills. What should I do? Doctor: Hello,The bleeding you are experiencing is due to the hormone imbalance created by the missed pills. If you have had any sexual intercourse later on in the cycle, you are not protected also. Home pregnancy tests, however, can only read well 3-7 days after the missed period. You should therefore get a blood test for pregnancy and follow it up with a clinical exam and a trans-vaginal sonogram. Once pregnancy is excluded, you can start your new pack of pills. You may also need styptics simultaneously to help arrest bleeding faster. Hope this helps."
},
{
"id": 7030,
"tgt": "Does eating papaya affect pregnancy ?",
"src": "Patient: hi Doctor .. im 27 years old male. we are planning for pregnancy . will my diet (papaya fruit) have any negative impact on pregnancy chances? Doctor: hi welcome to hcm.no not a tall papya which is good for health onyhe contrary its helps in your planing.so without worry you can eat papya. thanks dr.dhara dhara.shah84@yahoo.in"
},
{
"id": 65657,
"tgt": "Is bump on leg because of a fall dangerous?",
"src": "Patient: my son two weeks ago feel off his scooter and hit his thigh on his handle bar now the skin is sunken in with a hard long lump above it but he does not seem to have any pain not sure if it was there prior it is close to his groin area and he is 12 years old Doctor: Hi, it could be a muscle break. If he has problem in walking around or running then you should consult a Surgeon who can examine you and let you know whether anything needs to be done. I suggest you see your Surgeon at the earliest as early intervention is far better in cases of trauma.Take care,Dr Rishi, New Delhi, India."
},
{
"id": 186651,
"tgt": "Can my broken teeth be treated urgently?",
"src": "Patient: I have shingles and testerday I broke my tooth. The tooth is very sharp and is cutting into my tongue and I cant eat anything as my tongue is very sore with rubbing on my tooth. Will the Dentidt be able to treat me if I ring in tomorrow for an urgent appointment Doctor: thanks for your query, you can get the sharp tooth rounded off but inform the dentist about your shingles...so that he can take special precautions..you can use topical anesthetic and analgesic for soreness of the tongue.. i hope my answer will help you..take care.."
},
{
"id": 199760,
"tgt": "What causes penis curved upwards on erection?",
"src": "Patient: Dr Grief, I suffered a stroke 3.5 years ago and from maybe a year ago, when I have an erection, it is curved upwards, toward my body, suggesting that something is restricting the blood flow to allow. Would you know what might be causing this restriction? Doctor: Hi, dearI have gone through your question. I can understand your concern.slight upward curve is normal at time of erection. You should not worry about that. It is totally normal condition. It has no relation with your stroke. So just be relaxed. Hope I have answered your question, if you have any doubts then contact me at bit.ly/Drsanghvihardik, I will be happy to answer you.Thanks for using health care magic.Wish you a very good health."
},
{
"id": 163741,
"tgt": "Suggest cause for death in the infant",
"src": "Patient: in last month February 2011, My baby was 1 year 4 months old with 1 day night fewer,cough and faint in early morning and during medical treatment dead within 2 hrs. Doctors sasy continue faint and with low heart rate and loss of platelate count and heamoglobin happened but they could not find the reason, says viral fever. Doctor: your baby may have severe septicemia , but you need to think of metabolic disorder also, ..Does your other child have any symptoms ? or do you have any family history of similar deaths ?"
},
{
"id": 191075,
"tgt": "Problem with projection of teeth, also I don't have four molars. If teeth is taken out then can dental implants be used there ?",
"src": "Patient: hello doctor , i am now 21 years old.i had done dental treatment done almost five years ago.i had a complicated problem.by birth my lower jaw was inside.When i went to treatment it was brought forward using twin blocks.Then braces was used to bring it back.It ended as okay.After one year settled teeth came forward.Then it was fixed using retainers.But the problem is there is still projection of teeth.It is not much visible from outside.I am not able to smile well or talk with confidence .I visited a dentist .He told me the only possibility is to take off four teeths .two from above and two from below and then put on braces.Is it possible. If teeth is taken out then can dental implants be used there.also i don t have four molars.Actually the teeth is still inside.i have asked about it to a doctor.he said it is better if it remains there since i dont have any pain or disturbance with it.will this make problem.Can you recommend any treatments.I am considering doing braces again.I am thinking of doing treatment after 2 years since I am doing my p.g.will that age be correct for treatment. Doctor: Hello friend. As per your symptoms you are having orthodontic relapse. After finishing the orthodontic treatment the teeth tend to move back to its original position, so a retainer is given to patient (removable or fixed) to hold the teeth in the new position. According to American standards it has to be given till the age of 22. At this age the growth of the body stops completely. Now that relapse has occurred in your case, you need to go again for orthodontic therapy that is fixed or removable depending on the severity. If minor correction has to be done better go for removable. If enough space is not there to align the teeth then to gain the space extraction of premolars (to gain more space) or proximal stripping (to gain less space) has to done. And yes, orthodontic treatment can definetely be done at this age. Its called Adult orthodontics. If you have photographs of your teeth plz send me to dentist@ravishah.co.in to recieve more advice."
},
{
"id": 23471,
"tgt": "Is it normal to have partial hiccups after CABG?",
"src": "Patient: Hi,My father had a CABG last week and after 2/3 days he has started showing partial hiccups that tend to cause abnormal movements of the stomach [noticeable] and thus affects his sleep. Also his voice breaks significantly during this. The hiccups are partial in the sense that the cause movements of the abdomen region. Is this common? Can u please suggest what I must do ? Is this something related to diaphragm inflation? Doctor: hi noted all his details dont worry hiccups can be because of slight irritation of the diaphragm (not inflation)a cabg surgery reqires opening of the chest wall our body is a closed system ,so if any new changes to the body enviorement is there ,body will take some time to cope with it and get adapted to itcomplete bed rest after cabg and limited mobilisation can also be a cause as gastritis hiocuups abnormal abdominal monents once as he will start to ambulate these problems will settle downthanks"
},
{
"id": 158340,
"tgt": "Taken vismodegib for Locally Advanced Basil Cell Carcinoma on ear. Total hair loss. Will it grow back?",
"src": "Patient: Hello. I took vismodegib (Erivedge) from December 2, 2012 to May 17, 2013 for Locally Advanced Basil Cell Carcinoma on my right ear that came back after two MOS surgeries. The cancer sore appears to be gone. My doctor says that obviously I am a responder to the drug. The side effects I had were bad leg muscle cramps, fatigue, weight loss, loss of taste, AND loss of hair (completely lost my facial hair and beard, nose hair, body hair, and my scalp hair has thinned greatly). I am a 55 year old man who had a full head of hair. WILL MY HAIR and BEARD COME BACK? If so, how long does it take? The manufacturer of the drug does not say if hair loss is temporary or permanent Doctor: Hi there, Unfortuntely there are no easy answers as people react differently to the drug and only time can tell if your hair start to grow and how much will it return to normal. Most of the cases it does start to regrow after 3- 6 months in most of the patients after chemotherapy (not specific to Vismodegib), Regards"
},
{
"id": 48051,
"tgt": "What causes dilation of kidney?",
"src": "Patient: Dear Doctor, My husband was diagnosed with mild left pelvicalceal dilation with normal uretic jets. So far his other laboratory test for kidney, liver and artery doppler test were normal. But my problem is he has microscopic hematuria. Does it has something to do with the dilation of the left kidney?What causes the dilation? His GP did not give him medicine cause according to him sometimes it is normal to have hematuria and laboratory were normal. Please enlighten me about this. My husband is 43 years old, non smoker/ non drinker and don t feel any pain when urinating.Thank you,Len Doctor: Good Day and thank you for being with Healthcare Magic! I would recommend doing a CTSCAN with IV contrast to check if there is obstruction. We have to rule out if the kidney is obstructed since this may cause kidney damage. other cause of dilatation is urinary reflux which is can be harmful Also. I hope I have succeeded in providing the information you were looking for. Please feel free to write back to me for any further clarifications at: http://www.HealthcareMagic.com/doctors/dr-manuel-c-see-iv/66014 I would gladly help you. Best wishes"
},
{
"id": 16027,
"tgt": "Small rashes appeared all over body. What is the cause and cure?",
"src": "Patient: HELP! I ve been having sudden (small) rashes popping up on random spots on my body, 3 under my armpit, 1 on each side of my arms, (close to the elbow ) and 2 on my stomach, I dont understand! I have a clean room, i shower daily, i have a normal diet , and i get all these rashes. then when i scratch then, they dry up and turn into a dark spot! Then when i try cleaning it off, it wont go away! Whats happening?!?!?! Doctor: Hello, You have said that you have rashes but whether it is associated with itching or fever?? Rashes may be of different types....It may also occur on exposure to sunlight....Anyway watch for some more time & if it spreads better consult a dermatologist... Take care...."
},
{
"id": 192898,
"tgt": "How to control early semen discharge while masturbating?",
"src": "Patient: Hi, may I answer your health queries right now ? Please type your query here...hello sir,'m 21 years old boy,past 6 years i use my hands for self sex.but now my penis look small and when i use to have self sex,the sperm was comes early.how can i control it? if any execise is there pls tell me? Doctor: Hi, It can be due to premature ejaculation caused by \u2022\u00a0\u00a0\u00a0\u00a0\u00a0Stressful work \u2013 night duties, 24-hour jobs \u2022\u00a0\u00a0\u00a0\u00a0\u00a0Diabetes, hypertension, thyroid issue has an effect on Premature Ejaculation. keeping your Blood sugar, blood pressure and thyroid hormones under control will help you \u2022\u00a0\u00a0\u00a0\u00a0\u00a0On long term medication related to hypertension and anti-psychiatric medications \u2022\u00a0\u00a0\u00a0\u00a0\u00a0Frequent masturbation / prone masturbation. Hope I have answered your query. Let me know if I can assist you further. Take care Regards, Dr S.R.Raveendran, Sexologist"
},
{
"id": 21937,
"tgt": "Suggest remedy to lower bp without causing water retention",
"src": "Patient: I am 39yrs old, and my blood pressure has hovered around 150/105 (pulse 61-75) for the past 3 days. I was on 20mg of Benicar HTC, 4 months ago, but stopped taking it due to excessive water retention. My DR prescribed water pills that lower BP and bloating, but that didn t do much for the water retention. To combat having to take pills for BP I have dropped 22lbs in the past 2 months and have a regular exercise regime, and eat practically a zero salt diet - but still don t see much of a difference for BP. High blood pressure is in my family, but never effected me until I gained 50lbs over the course of the last 5 years - I always had 120/80 prior. Doctor: Hi ThereI have read the details provided by you which gives a clear idea that hypertension runs in your family. Also in general one of the strongest etiology for high blood pressure is genetics which is there in your case.Its good to hear that you have a strong will power and you are already on life style modification from the last 2 months which is first line treatment of High bp although its has not benefited you much till now.So now what you can do is that just continue this life style for another one month and still your BP does not show any improvement then we need to start you on proper anti hypertensive meds along with the life style changes. And also get an ECHOCARDIOGRAPHY done in the mean time to check your LV function and to asses the thickness of LV musculature, so that the meds can be planned accordingly. And please dont make up your mind that you dont want to take meds as they are going to protect you against the harms of high BP (for eg Stroke, MI, Cardiac dysfunction etc.).One more important thing if you smoke please stop it as soon as possible.Good Luck"
},
{
"id": 73922,
"tgt": "Is chest pain while running a cause for concern?",
"src": "Patient: Hi, Im 23 and for the second time i have had chest pains while running, Im not a star runner but yesterday i completed 12 km, probably the most i have ever done, I have a fit physique and exercise regularily. the two pains occured months apart... should i be concerned? I had a test done at the hospital and they found no symptoms of a murmor or any other negative sign, could this be just a strained muscle? also i did not feel it this time during exercise it was several hours later after i had gone to bed. center of chest. Doctor: Thanks for your question on Healthcare Magic.I can understand your concern.No need to worry for cardiac chest pain as your work up for heart diseases in hospital was normal.Yes, possibility of muscular spasm related chest pain is more likely.So apply warm water pad and ice packs alternatively on painful area.Avoid heavyweight lifting.Don't worry, you will be alright.Continue exercise and running. No harm in it.Hope I have solved your query. I will be happy to help you further. Wish you good health. Thanks."
},
{
"id": 56318,
"tgt": "Need treatment for expanded liver",
"src": "Patient: Hi hope you can help me My 75 year old father has been diagonsed with an expanded liver He had a ultra scan and had liver function tests ( which were high) He has been for a CT scan but we have drawn a blank as both my GP and the hospital won t provide the results my dad has yellow eyes, has lost alot of weight, has pains in the base of his foot and has lot his appetite. He is also under morfin tablets due to regular dvt problems Would you be able to advise what steps we need to take next as we are very concerned Thanks Paul Jeffers Doctor: Hello Your father findings suggests raised liver enzymes and enlarged liver.Enlarged liver(hepatomegaly) may be due to many reasons like hepatitis,fatty liver,tumour etc.It is important to know which parameters of liver function test are raised.Hepatomegaly is most likely related to hepatitis in your father's case as he has yellow eyes.He may need few more investigations like routine hemogram,random blood sugar,viral markers.He should avoid fatty food.Proper treatment depend upon findings.Take CareDr.Indu Bhushan"
},
{
"id": 98870,
"tgt": "Suggest remedy for multiple chemical sensitivities",
"src": "Patient: Do you take multiple chemical sensitivities seriously and understand how debilitating fragrance chemicals are? Do you think there is always an under-lying cause for scent sensitivity/allergy (yes, I have allergic reactions to dryer sheets like Bounce and Scent Boosters)ie rash hot prickly skin, swelling of tongue, extreme thirst... do you think (my) immune system was already compromised or do you think the neurotoxic petro chemicals in fragrance products alone can cut a person s life down to the knees overnight? Doctor: It's a very good question and thousands of people who read it, I fact, can reap the benefits out of suggestions given herein, based on the principles of Ayurveda - the nascent science of life.First of all there is a principle cited in Ayurveda about how to prevent/ cure a disease. It's 'NIDAANPARIVARJAN' (in Sanskrit language) which literally means 'avoidance of the cause'.Now, in many cases people are not able to change jobs or circumstances are such that they have to continue to remain in such conditions. Here comes the principle of PRAANVAYU (richness of oxygen and fresh air) that is achieved by rythms of deep breathing exercises called PRAANAYAM.India's Yoga has spread to the World and people who are aware are enjoying the benefits. It enhances immunity, gives strength to fight allergens and extends life as also betters quality of life.Then there are herbs to naturally support the fight of body against allergens. Simple herbs like Turmeric, Neem, Manjishtha, Giloy, Tulasi (Holy basil) etc. not only counter allergens but detoxifies the blood and also raise natural immunity.The hitch among beginners is its slow pace of action. Yes naturals are slow as compared to man made synthetic and toxic medicines but then what advantage have these fast drugs given over a time period. No doubt modern synthetics have won in emergency situations and saved lives. But lifestyle disorders and the one you are suffering from needs naturals to support you. These do not have side effects, no interaction with drugs, are safe over prolonged use too. What else you need?Why not try it. For any further questions please write direct. To my own patients I would suggest to adopt natural Ayurveda medicine. Initially both your current medicine and Ayurveda may be used parallel with some gaps and gradually switch fully if you see advantage.Hope it helps."
},
{
"id": 108703,
"tgt": "Can 2/2 mm growth in pancreas cause severe back pain?",
"src": "Patient: Hi my father have heavy pain in back from 2 months and they have joindics too. They also lose their wait around 20 Kg.in 2 months. they had gone threw MRI . told them that you have a 2by2 m.m. growth in pancreas . what i should do Doctor: Thank you for the question.I would suggest that an ultrasound of liver, pancreas ,bile duct, duodenum should also be done.A CT-guided or ultrasound guided FNAC/biosy should be performed from growth in pancreas. serum amylase, ,complete blood count, liver function test and bone scan of spine for diagnosis .consult you gastroenterologist ketanov tablets 6hourly for pain relef for now."
},
{
"id": 18022,
"tgt": "Is it possible to experience severe chest pain after a CPR procedure?",
"src": "Patient: Hi my dad is in hospital after going into cardiac arrest twice on Monday. He was put into an induced coma and was wakened last night. The ecg came back clear after having a stent put in. Is it normal for him to have severe chest pain possibly after the cpr? Doctor: Hi, During CPR the ribs may get injured or even rib fracture can happen. If pain persists, consult a physician and get evaluated. Wishing all the best. Hope I have answered your query. Let me know if I can assist you further. Regards,\u00a0\u00a0\u00a0\u00a0\u00a0 Dr. Shinas Hussain"
},
{
"id": 39176,
"tgt": "How to cure itchy anus?",
"src": "Patient: i have a itchy & mucussy anus. i m not going to the bathroom like i should.been taking over the counter pinworm meds they dont seem to be working that good. what do you think this might be & is there any perscribable meds to cure.i have not gone to a doc. yet but maybe i need to. Doctor: Hello,Welcome to HCM,The history and symptoms suggests me that you are having some fungal infection which is causing all these tedious problems.The fungus are known to affect this region, because of the hygiene and the the presence of the hair which will favors the multiplication of this fungus which were the normal commensals of this region and produces these symptoms.For your symptoms I would suggest you to follow.1. Keep this area clean and dry by applying the neosporin powder over the affected area.2..Apply anti fungal cream over the affected area.3. Sinfgle dose of Tab Diflucan, 150 mg, will helps you to over come this symptoms.4. Oral Antihistamines will reduce the severity of itching.Thank You."
},
{
"id": 158799,
"tgt": "Lump on the hollow space of left clavicle. Could this be hematoma, cyst or lymphoma?",
"src": "Patient: Several days ago I noticed a very soft lump in the hollow space of my left clavicle. It is painless and measures about 3 cm by 5 cm. I went to the doctor today and he had blood drawn for a CBC and scheduled me for an ultrasound next week. he told me it could be a hematoma or a cyst or lymphoma. If I injured my shoulder in January could this soft tissue lump be from that ? I am very concerned since cancer runs in my immediate family. Doctor: Hello! Thank you for the query. First of all some good news for you. Most of such lumps are benign lesions and not a cancer for sure. Yes, it can be a hematoma caused by the trauma in January (sometimes blood collection can stay in the tissues for years). It can be also a cyst or lipoma (NOT lymphoma). All mentioned types of lumps are benign and can be easily diagnosed with an ultrasound. Hematoma should be drained with small incision. Cyst or lipoma can be removed and send for histopathology to make sure no cancer is present. Hope this will help. Regards."
},
{
"id": 185333,
"tgt": "Is smoothies right diet for dental work to extract the teeth?",
"src": "Patient: I am making smoothies due to dental work where teeth are being removed temporarily.And I am finding this working with a blender. And using vegetables of all kinds, as well as fruits for smoothies. Hummus basic is also taken in spoon fools for protein.This is ok? Doctor: HiIn regard to your query, I think you underwent or going for extraction I believe food after the extraction or during extraction is soft diet for 3-4 days is enough any how you are making softies with all the required nutrients so nothing to worry happily you can have it. Hope thus helps you"
},
{
"id": 82817,
"tgt": "How safe is using methmore for lupus?",
"src": "Patient: My wife and I have been separated since Aug of this year, we also have a2 year old son. My wife has lupus and and graves disease. I found out from her that she has been using meth ice , since Jan but really not sure how long. She has said it helps with her loss of energy and also with pain caused by her diseases. Is using methmore harmful to her with her diseases ? Also claims to be a controled addict. Doctor: Your wife should stop using it as it is not the standard treatment for this .And yes it can aggravate her lupus also.Dr. Shruti"
},
{
"id": 139918,
"tgt": "Suggest treatment for back pain radiating to the leg",
"src": "Patient: hi I am a 30yr old female and diagnosed for lumbar spine L5 S1 slip disc problem with the spine slightly pushing further. Also I was been treated for sciatica. I have recovered 90 % but still there is a lock on my lower back bcoz of it pains from the right side of my lower back or hip and also the same pain passes through the back of my right thy and caughfs. Please suggest me how do i get rid of this pain. I dont want to have pain killers just for the sake of having it. Doctor: Hello, Since you are feeling better, and you don't want to use painkillers, physical therapy may help you. In particular, lumbosacral traction may be more effective.Hope I have answered your query. Let me know if I can assist you further. Regards, Dr. Erion Spaho, Neurologist, Surgical"
},
{
"id": 129320,
"tgt": "What could painful lump in shin after getting kicked indicate, have Ehlers Danlos syndrome?",
"src": "Patient: I was kicked really hard in the shin last night, it now has a lump on it, very swore and painful to bear weight. Should I get this checked out or would it just be tissue damage? I also have ehlers danlos syndrome which means I am prone to worse injuries. Doctor: Hello,I think you should check your leg, have an x-ray to rule out a fracture. If no fracture then the lump is a hematoma (blood collection) that needs ice pack for 48 hours, bed rest, and elevation of the leg above heart level. Some doctors would give antibiotics to prevent infection of this hematoma.Hope I have answered your query. Let me know if I can assist you further.Regards,Dr. Edvin Selmani"
},
{
"id": 23545,
"tgt": "Suggest medication for fluctuating BP levels",
"src": "Patient: Hi I am 39 yrs old and 7 weeks ago I went to the doctors and bp was 150/100 and have cholesterol 6.3. I have been exercising daily, meditate and have changed my diet. I went to docs yesterday and bp was 126/90 but the diastolic fluctuates as it was 90, 88 and 80? What should I do as I have been told that the doctor will advise me whether to go on pills. I don't want to as I feel fine and bp has come right. I was told by the nurse that it is the diastolic which I should keep down??? Thanks. Doctor: firstly blood pressure is catagorised in these catagoriesnormal bp is less then 120/80pre hypertensive stage is 120 to 139/80 to 89 grade 1 hypertension (mild ) is 140 to 159/90to 99grade 2 hypertension (moderate) is 160 to 179/100 to 109grade 3 hypertension (severe) is 180 and above/110 and aboveyour daisyolic bp whether it is 90 or 88 or 82 falls under prehypertensive stage so right now for treatment of prehypertensive stage life style modification is advocateddo strenous exercise daily for atleast 45min a daytake low salt dietdrink plenty of wateradd fibres in your diet if not controlled in 3months then medication will be added but if your diastolic bp is more then 90mm of hg then you should add tab amlodipine 5mg once dailyand monitor bp every third day and keep a record"
},
{
"id": 120495,
"tgt": "What causes severe pain in collar bone while stretching and breathing?",
"src": "Patient: I ve have been having server pain in my left collar bone for about 5 and a half. The pain feels like someone is stretching it and it s hot to the touch. The pain really gets intense when ever I raise my right arm above my head. It also sometimes hurts when I breathe. Doctor: Hello,You may be suffering from spasm of muscles that attached to collar bone in like pectoral groups. Trauma or thoracic outlet syndrome or tumour may present like this. Please get it done X-ray chest and consult with your orthopedician he will examine and treat you accordingly.Take care. Hope I have answered your question. Let me know if I can assist you further. Regards, Dr. Penchila Prasad Kandikattu, Internal Medicine Specialist"
},
{
"id": 14587,
"tgt": "What causes rash between thighs and how to treat it?",
"src": "Patient: hi Doc!i have this rash between my thighs, very itchy and not sure what it is caused by. am told it could be heat rash but i am not considered big in any way. then i am told by family that it could be high protein allergy but i have not eaten anythig different lately. what coild be my issue and how can i treat it Doctor: Hello. Thanks for writing to us at healthcaremagicThis seems most likely to be a fungal infection of the groin (Tinea cruris). The location of the rash and itching goes in favour of this being a fungal infection.Groin folds are a common site of fungal infections; the anatomy being such that the folds trap moisture, which is conducive to fungal growth, specially in hot and humid weather of summers.I would suggest you to use an OTC topical antifungal e.g clotrimazole 1% cream, twice daily.An OTC oral antihistamine e.g cetrizine 10 mg once daily will provide you symptomatic relief from itching.General measures would include bathing twice daily, wearing loose fitting clothes and cotton undergarments.Regards"
},
{
"id": 139761,
"tgt": "What does this MRI report of brain indicate?",
"src": "Patient: HI, i just hada MRI . THIS IS WHAT THE RESULTS SAY .SEVERAL PUNCTATE FOCI OF T2/FLAIR SIGNAL ALTERATION ARE SCATTERED AT THE BILATERAL PREVENTRICULAR AND SUBCORTICAL WHITE MATTER, which are non-specific and probably reflect minimal chrocal microvascular ischemic gliosis. Doctor: Hi, These MRI findings are most commonly seen in people with migraine. However, conditions such as high BP, sugar or cholesterol can lead to brain ischemia (lack of blood flow to brain), which can result in similar changes. Hope I have answered your query. Let me know if I can assist you further. Regards, Dr. Sudhir Kumar, Neurologist"
},
{
"id": 135784,
"tgt": "Suggest remedy for leg pain caused by injury",
"src": "Patient: My husband was out walking and stepped hard onto a rock. Since then his heal and arch has been hurting. He went to his physician and was sent for xrays which came back negative. His physician is out of the office today. Is there something he can to do to relieve the pain? Doctor: he should try to get an MRI to confirm cause of pain. Motrin can be used for antiinflammation. otherwise he can get Voltaren gel, or Bengay cream. he should ice his foot and wrap it tight for immobilization. you may even benefit from crutches so he's non weightbearing."
},
{
"id": 172407,
"tgt": "Suggest treatment for blood in stool of a 2 year old child",
"src": "Patient: Hi. My daughter is two years old. For the past few days she is passing stool with blood. Got her stool test done n in the report occult blood is positive, pus cells are 2-4 and red blood cells are 1-2. All other values are nil. She complains of tummy ache n then passes stool which is semi formed n of weird texture. Please advise! Doctor: Thanks for asking I gone through your question, your 2 year old daughter passing stool with blood since few days, she having tummy ache. The stool with blood should be treated with course of antibiotics. If i was your treating pediatrician i would advised you Syp Cefexime 10 mg per kg per day in two divided dose for 5 days and tab zinc 20 per day . The blood content of stool start reducing in 2 day of therapy. The above medicine is not availble without precription so you should visit doctor. hope i answer your query have health family"
},
{
"id": 186082,
"tgt": "Whats the cause for gap between teeth gums?",
"src": "Patient: I am taking Eption tab 100 mg from 2 years, from past one year i am not facing problem with epilipsy but i am facing problem with my teeth , Getting gap between teeth with gums, i consulted Dental doctor , they told stop to use eption , can i stop or what , my gums increasing day by day between teeth.pls help me what to do Doctor: Thanks for posting your query to HCM.Eption consists of phenytoin which is associated with gum problems.I would suggest you to consult your physician regarding stopping the same and substitution with some other medication.Maintain good oral hygiene and do warm saline rinses 3-4 times a day.Hope my answer will help you."
},
{
"id": 56580,
"tgt": "What medication is suggested for mild hepatic steatosis ?",
"src": "Patient: yes I recently had a cat scan done on my liver it says that I have mild diffuse hepatic steatosis, the main portal vein is mildly prominent measuring 16,6 mm dia and the spleen is mildly enlarge at 13.4 cm and possible portal venous hypertension shold I be concern Doctor: Hi there,Thanks for using HCM.As per the scan report you are having features of portal hypertension.I would advise you to get a Doppler scan of the portal venous system and an upper GI endoscopy done to confirm portal hypertension.This might be due to cirrhosis of liver. You might be having early cirrhosis.A fibroscan or a liver biopsy may be done to see whether you are having cirrhosis.Liver panel may be done which may show numbers indicating liver damage and chronic liver disease.Alcohol intake, nonalcoholic fatty liver disease, Hepatitis B and C infection are some of the causes for chronic liver disease.You can consult a gastroenterologist for the above mentioned tests.Is this answer helpful?"
},
{
"id": 180705,
"tgt": "How can severe pain after a dental surgery be treated?",
"src": "Patient: I just had the first stage of dental implants didn't know there was more than one stage and in my case mygood dentist who is a good friend and person had to cut back my upper gums to expose the upper jaw boneI wondered about that and when i went in I told them I wanted them to give me all the pain meds and nitrousthey could legaly cause i have been told it would be painless, and i said basically i wanted the full Monty!He said it wouldn't be painful and he went about doing his think in a lawful and highly professional mannerAnd he was correct! I felt absolutely no pain ! best trip I've ever had at the dentist and i have had dentist thatwere literally sadists! now i've been prescribed Amoxicillin(Good!) Hydrocodone - Acetaminafin. I'm having pain now which the dental assistant said i would but i have taken a pill that i thought i remembered as a painmedicine but when i googled it they said it was a muscle relaxer! oh well that\"s the story of my life! I,m veryrelaxed of course but my mind and motor skills appear to be farily normal. I still have the pain from my surgery so my question is should I take the pain med now or not, sorry to drag this out and i know you have many eggs to fry so for that i am sorry. I am 66 in good health just had my check up and every good and normal have a good day and i be talking to you soon! Thanks in advance, Mike Reed in Clayton NC Doctor: Hello,Well, some amount of pain after implant placement or any dental surgical procedure is commonly seen. It is due to tissue trauma and inflammation caused due to the surgical procedure causing pain, swelling and soreness.The medicine that you are taking that is Amoxicillin and Hydrocodone-Acetaminophen combination, it is fine and Hydrocodone-Acetaminophen combination is a painkiller and will help in relieving pain.So, my suggestion is to continue taking this medication and it will help in reducing pain. If the pain continues to worsen, then you should consult an Oral Surgeon and Implantologist and get evaluated.Hope I have answered your query. Let me know if I can assist you further.Regards,Dr. Honey Arora"
},
{
"id": 28291,
"tgt": "Would stress cause a decrease in the BP level from 150/98 to 119/97?",
"src": "Patient: Hi, I am 44 year old female suffering from menorrhagia since 2011 subsequently I have low iron count and have now been advised to take 130mg of ferrous sulphate & take cerarette. I returned to GP with a slight tightness in chest after exercise - did spirometry inversion test & found my flow rate low ( 5ft 2 52kg). Also at this appointment my BP was 150 / 98 it has now decreased to 119/97. I am now concerned about my wellbeing, I am a dental technician & I am exposed to dusty working conditions. Could stress play a part in my blood pressure & how to reduce it ideally without medication. Doctor: Hi welcome to HCM.I understand your query and concern.High blood pressure should me monitored regularly to avoid complications like stroke in the brain.Best treatment for elevated blood pressure is through drugs like beta blockers and Ace inhibitors.Before that adequate management and work up is needed.I advise you to have a baseline 2 dimensional echocardiography,ECG and lipid profile to assess the basic cardiac reserve of your heart.Restrict the intake of salt to less than 6g/day.Regular physical exercise in the form of brisk walk for 20 min a day for 5 days a week is pretty useful.Drugs like antihypertensives and antiarrhythmics will help.Reduce the intake of fatty and fried food.One pomegranate a day will help . Stress management is most vital in your case.So Consult a Cardiologist for further expert management.Post your further queries if any.Thank you."
},
{
"id": 43540,
"tgt": "Done sperm analysis. Good result?",
"src": "Patient: spermogrammvolume 2.0 ph 7.5count71viscosity normal time of liquification 30 minabnormal form 35normal form 65activity after 30 min 60sluggish after 30 min 10immotile after 30 min 30activity after 1 hour 55sluggish after 1 hour 10immotile after 1 hour 35activity after 2hours 50sluggish after 2hours 10immotile after 2 hours 40activity after 3hours 50sluggish after 3hours 5immotile after 3 hours 45wbc/hpf 3-4sperm cultureorganism 1 negativeis the result is good or no? Doctor: Hi, Welcome to HealthcareMagic Your semen analysis report is normal in all parameters. According to WHO count should be above 15m/ml , Total Motility should be more than 40% ( Progressive motility should be more than 32%) and more than 4% sperms should be normal in morphology. Negative semen culture rules out genital tract infection. All the very best. Take care."
},
{
"id": 210116,
"tgt": "How long after consultation with psychiatrist for adult ADD will the medication be prescribed?",
"src": "Patient: I have adult add and has seen a pshycharist three sessions...they are 45=60 minutes each. I filled out multiple paperwork at home and returned to him. I wonder how long before he prescribes medication. Why so long ? 3-4 weeks in between appointments and this problem is not getting better on its own...what should I expect? Doctor: HiThanks for using healthcare magicUsually psychiatrists take hardly one session to diagnose ADHD and prescribe it on same session, but protocol of every psychiatrist is different from other. In your case, may be your psychiatrist is not convince with diagnosis or have any another doubt. Better to discuss with him about that.Thanks"
},
{
"id": 104818,
"tgt": "Suffering from Eosinophilia Allergy, during cold season. Any home remedy?",
"src": "Patient: Hello Doctor, my name is Vipul. I have been suffering from Eosinophilia Allergy since long and this problem differs from place to place and I usually witness it during cold seasons. I am having active lifestyle and never forget to go for some exercises or Yoga but still sometimes the problem emerges out. Could you please tell me the natural and home remedy to keep this allergy at bay? Doctor: hi, you have not mentioned your symptoms , but for all the allergy the first step is to find out the cause of allergy, eg dust allergy or food allergy. if you have food allergy the best thing is to avoid the food item. when allergen is airborne eg: dust or pollen , then you need to follow step wise treatment if you symptoms are mild and appears only in winter you can take anti histamines and control it. if you have for long duration in winter then you need to take steroid nasal spray and you will achieve very good control. last line of treatment immunotherapy and it is most effective. you seem to have allergy to pollen which is more during winter , i think you should go to immunologist or or good physician and he will guide you further. keep your self physically fit. home remedy usually do not work, but you can try jalneti , which may help you but no established study done on this thank you dr.mukesh"
},
{
"id": 152736,
"tgt": "Suggest treatment for squamous cell cancer",
"src": "Patient: Hello 3 years ago I found cancer in the lymph nodes on the left side of by neck. I was diagnosed with Squamous cell cancer they gave me 8 weeks of radiation and 2 sessions of chemotherapy. I had all the normal post stuff but now just recently I have nerve damage from scar tissue in my neck. This has affected my voice ,swallowing, eating, and my speech . I have been told this is permanent and the only thing I ca do is hyperbaric oxygen treatments to stop it where it s at. Is this true? Is any thing else I can do? A Doctor: Hello,Yes, the damage has been done and is almost permanent but some physiotherapist may help you better compared to HBOT.Hope I have answered your query. Let me know if I can assist you further.Regards,Dr. B.dinesh"
},
{
"id": 90570,
"tgt": "What causes pain on the sides of my stomach?",
"src": "Patient: Hi,doctor good evening In recent days I am getting a deep pain on left side of my stomuch as well as right side. If I take relax for 1 to 2 hours it is getting cured. I think i am getting this pain due to digestive problem. I am not getting hungry also in these days. Please help on this problem Doctor: Hi.Thanks for your query.Such type of pains are either due to colitis ( inflammation of large bowel, called as colon) or due to IBS, called as Irritable Bowel Syndrome.IF this is simple colitis due to an infection , you will be cured with 5 days of a course of antibiotics and anti-spasmodics. If IBS or diverticulitis or so, I would like to conduct all related investigations to rule out other problem and then a start a proper treatment for these ."
},
{
"id": 179959,
"tgt": "Need treatment for undescended testicles",
"src": "Patient: Hi..My newborn boy has the problem of undescended testes bilateral. How long we can wait before starting treatment? Is there any other sidda, Ayurvedic or homeo treatments for this problem? How many percentage of surety can be given for automatic descend in three to six months? Doctor: do not wait...alternative medicines will not help..undescended testis needs immediate surgical correction"
},
{
"id": 214840,
"tgt": "Scratches in penis head, severe pain when water is poured. History of masturbating using cucumber. Any natural remedy?",
"src": "Patient: hi i am male 18 yrs old i use to masturbate once in a weak .this time for a change i masturbate with a cucumber by making hole at the centre .after i masturbate i noticed that there are several scratches in my penis head head.its so painful when i pore water on it.can any one help me.say some natural medicine to cure it.... Doctor: Hi,Thanks for writing in.Looks like there has been small mucosal injury to your glans penis (head of penis). There is nothing to worry as it is due to friction between the inside of cucumber and the head of penis which has made scratches (physical injury).If left alone the mucosal injury in your penis head will repair by itself in 2 - 3 days. If it is bleeding, please consult a doctor and apply Neosporin ointment.Hope this helps."
},
{
"id": 195013,
"tgt": "What causes tenderness in the testicles?",
"src": "Patient: a few yrs ago i contracted chlamydia and took the medication and got rid of the std however i noticed immediately while flaccid my penis has shrunk i feel uncomfortable most of the time i am very sensitive even say a pat on the back unexpectedly(very jumpy) my testicles don't really hang much more and activity will make it shrink to about 1 in its a very depressing situation i found myself in and i would just like some closure on this if someone can help i was told by the clinic not to \"mechanically\" have sex (masturbate) while on the meds (but of course i didn't listen) as around my last day of the wk long process i gave in and masturbated was this the reason for all my sorrow? Doctor: Hi, It can be due to epididymitis or orchitis. You can consult a general surgeon and get an ultrasound scrotum to look for any signs of infection. You can also go for a urine routine also. You can empirically start on antibiotics preferably Ciprofloxacin. Hope I have answered your query. Let me know if I can assist you further. Regards, Dr Shinas Hussain, General & Family Physician"
},
{
"id": 162365,
"tgt": "How can fever, cold and blood blisters on the nose be treated in an 11-year-old?",
"src": "Patient: My 11 year old daughter has been having a fever and runny nose for a few days, which has led to a bloody nose, pretty bad....then this sac appeared in her nose, like a blood sac or something.....now its hanging out of her nose, but we are afraid to pull it out any further? Doctor: Hi, By what you say I feel that you are kid is having allergic rhinitis and nasal polyp. I suggest you not to pull it and just keep it that way and approach the nearest ENT surgeon so that it will be given appropriate management. Hope I have answered your query. Let me know if I can assist you further. Regards, Dr. Sumanth Amperayani, Pediatrician, Pulmonology"
},
{
"id": 50722,
"tgt": "Left renal calculus, left renal cortical cyst, bulky uterus. Recommendations?",
"src": "Patient: I am 50 years. I have a Calculus measuring 4.1 mm in the upper calyx. a simple cortical cyst measuring 3.0 cms. is noted in mud pole of left kidney. the uterus us bulky and measures 9.4 x 5.4 x 5.4 cms. the uterine myometrium show homogeneous echotexture. No evidence of focal lesion is noted. The endometrial echo is in the midline and measures 6 mm in thickness. The ovaries on the either side show normal echotexture. Impression left renal calculus. Simple left renal cortical cyst. Bulky uterus. Doctor: Hello.Uterus and ovaries are normal.Endometrial thickness is slightly higher.You must go Gynecologist to control.Simple cortical cyst is not important but must be controlled by renal ultrasounds.Calculus 4.1 mm,especially if you have frequent renal colic pain must be removed.There are conservative treatment,extracorporeal shock wave lithotripsy(ESWL),breaking the calculus into small fragments that are eliminated during urination.Go to urologist to evalue and decide that treatment.I wish you good health."
},
{
"id": 157183,
"tgt": "What could be the effect of prolonged exposure to kerosene fumes for a cancer patient?",
"src": "Patient: I have a boiler run by kerosene for heat and hot water, I have smelled fumes and the kerosene smell for a col yrs and told my landlord. The unit quits a lot even if there's fuel in it. And last weekend it got so bad in the house that I was sick and the ems came. I was brought to the hospital and they said that there was a density in my lungs and could possibly be chemical burns...what about prolonged exposure? I am also a cancer patient just starting chemo. I have also tried to contact city officials to report this ti no avail. Doctor: HIThank for asking to HCMI really appreciate your concern, the hazardous effects of kerosene would same for cancer or normal person this is kind of corrosive exerting effects on the tissue could be dangerous to the health (Occupational hazards ) better to keep away from this, before it is too late, take care and have nice day."
},
{
"id": 169901,
"tgt": "What causes cold and running nose in a child?",
"src": "Patient: Last 2 months my 2.5 Yrs kid is suffering with cold and running nose and we saw an ent specialist and completed one course of antibiotics now continuing with allegra syrip for cold and otrovin nasal drops .they said this is because of enlarged adenoids.Can you help me to identify the root cause.. Doctor: Hi.... by what you quote I feel that your kid is having a seasonal rhinitis or a moderate to severe persistent allergic rhinitis. This is definitely not an infection so no need to use antibiotics at all. Simple logic is that an infection cannot go on like this for 2 months. This is definitely allergy. Coming to your query about adenoids - you should consider adenoids only if the kid is having snoring and inadequate or disturbed night sleep and daytime sleepiness. If these are not there this could be simple allergic rhinitis. I suggest you consult a paediatric allergist who can assess the kid and prescribe intranasal steroid which is the drug of choice in such cases.Regards - Dr. Sumanth"
},
{
"id": 196361,
"tgt": "Suggest remedy for premature ejaculation and erectile dysfunction",
"src": "Patient: I'm suffering from premature ejaculation and erectile dysfunction as well as anxiety. I'm not sure but I think the anxiety is causing both other symptoms. I've tried all the methods to control my orgasm stop start techniques, pc muscle clenches, and squeezing the tip of the penis before cuming it doesn't help. I've also tried delay condoms and gels, which work sometimes but often cause me to loose my erection. If it is down to anxiety then I've also read that the anti anxiety causes impotence so that might solve the problem that causing it this time but create a new one, the same with anti depressants. This problem has lasted over a year before I had no problems in this area. Can you give me some advice as it is making me depressed and ruining my relationship. Doctor: Hi and welcome to Healthcaremagic. Thank you for your query. I am Dr. Rommstein, I understand your concerns and I will try to help you as much as I can.Premature ejaculation is uncontrolled ejaculation either before or shortly after sexual penetration. It happens with minimal sexual stimulation and before the person wishes. Most cases of premature ejaculation do not have a clear cause. With sexual experience and age, men often learn to delay orgasm. Premature ejaculation may occur with a new partner. It may happen only in certain sexual situations or if it has been a long time since the last ejaculation. Psychological factors such as anxiety, guilt, or depression can also cause it. In some cases, it may be related to a medical cause such as hormonal problems, injury, or a side effect of certain medicines. In many cases premature ejaculation gets better on its own over time. Treatment may not be needed. Practicing relaxation techniques or using distraction methods may help you delay ejaculation. For some men, stopping or cutting down on the use of alcohol, tobacco, or illegal drugs may improve how well they can control ejaculation.I hope I have answered you query. If you have any further questions you can contact us in every time."
},
{
"id": 20317,
"tgt": "Suggest medication for chest pain & palpitations",
"src": "Patient: Im 23 year old female with 2 children suffering with servere palpations that are gradually getting worse Ive suffered with them for 4 years now. I went to the gym this morning to do my normal routine when i began to get palpations i sat down for a few mins and they still didnt go i then began to go faint and dizzy i also had chest pain and tightning my partner drove me to a&e where they did an ecg which didnt show anything any ideas what it could be Doctor: If you have had the symptoms evaluated and found no underlying heart disease or dangerous rhythm, the primary concern is to relieve your symptoms and anxiety. Finding nothing dangerous does not necessarily mean that heart rhythm is not the source of what you feel. I usually recommend a long term monitor to try and catch the abnormal rhythm and tie it to the symptoms that you feel. Your doctor can arrange this.There are also tools that allow you to record your rhythm with the telephone. If a monitor finds nothing, you may want to look into one of these apps. If your symptoms only occur every few months, its great to try and catch it with the electronic device that goes every where that you go."
},
{
"id": 128396,
"tgt": "What causes engorged vein under the knee?",
"src": "Patient: I had an engorged vein pop up on the top backside of my right leg just under my knee. It was very painful, burned and stung. After a bit the engorged vein slowly went down and now I have a very large deep purple bruise. At the same time I also had the same thing happen to another vein on the outer side of that leg. Should I be concerned? Doctor: Hi. Thanks for reaching out. Based on the description you have mentioned, it sounds like you have varicose veins. These are seen because of faulty vein valves. This can cause the size of the veins to increase. Varicose veins are treated by using compression socks, raising the legs high, and rarely surgery may be suggested to treat this. Seeing a surgeon would be a good idea. I hope this helps! All the best."
},
{
"id": 23192,
"tgt": "Suggest treatment to control blood pressure while having hypertension",
"src": "Patient: Hi, I am suffering from Hypertension. Also My single kidney is working, other is shrunk. No Reason for the same whether it was from birth or due to hypertension. MY current Blood Pressure Reading are coming at 150/100. I am under current medication of Betaloc 50, Losar 50 & AMtas 5. My Blood pressure is not coming under control.What you suggest Doctor: hhirenal artery doppler should be done to rule out the blockage of renal artery if renal artery is normalbetaloc can be oncreased to 50mg twice daily and a diuretc like chlorthiazide can be added to lossrtan.a group of medicines called nitrates could be added to current meds ,nitrates are venodiltor and reduces the blood pressure45min of strenous exercise should be dine dailyadd fibres to your dietdont take excess butter or saturated fats"
},
{
"id": 36775,
"tgt": "Does dent on forehead with tight pressure and clogged nostril suggest sinus infection?",
"src": "Patient: Hello, I have a small dent in the middle of my forehead, kind of under the hairline, that sort of just appeared about 2 weeks ago. I have a tight pressure feeling around the front of my forehead. One nostril is clogged. Occasionally, I will cough, but it s pretty rare. I have been going to a chiropractor for about a year. After that physical therapy and now a osteopathic doctor. All for treatment for neck/shoulder pain from a vehicle accident back in 2013. I have asked 2 doctors about the dent and they both said don t worry about it. The symptoms seem to be increasing. Does this sound like a sinus infection? Doctor: dear,very possible a case of sinus infection.i suggest a course of tab amoxicillin+clavunic acid 625 mg bdcap omeprazole 20 mg bdtab diclopara tdsfor 5 daysanyways it appears to be an infection. and the above medicinces will cover it.thanks"
},
{
"id": 223332,
"tgt": "Is there any chance of pregnancy?",
"src": "Patient: Hi, I'm on the combined contraceptive pill and on day 16, I vomitted and did not take another pill. Then the next day I decided to take 2 pills, this has meant I have finished my pack a day early. During this time i was having unprotected sex, is my chance of pregnancy high? Emma Doctor: hello user,no, u wont get pregnant.because the day you vomited,you have repeated 2 pills as per the protocol..missing the last pill wont matter.do not worry."
},
{
"id": 157418,
"tgt": "Starting Sylatron injections for Myeloprliferative disorder. Is this the right medication and Is this a form of chemo?",
"src": "Patient: My husband has Myeloproliferative Dis.(type-essential Thromboytosis) very high platelets, he is going to start Sylatron injections, all I read about is this med is for Melonoma, or Hepetitis, nothing on platelets. Also is this a form of chemo? Thank-you for your time. Doctor: Hi and welcome to Healthcare magic.Thank you for the query.Sylatron is interferon and it is not typical chemotherapeutic,but it has effect on inflammatory response and cells growing. So it is used in some cases of malignant diseases including hematologic maligancies. There are no enough studies which confirm benefit of it in Myeloproliferative disorders,and it cant replace standard chemoth.However it is worth of try since side effects are minimal and tehre are some good results in other types of blood cancersWish you good heaIth. Regards"
},
{
"id": 19761,
"tgt": "What causes a strong chest pain mid-way through drinking?",
"src": "Patient: I started feeling a strong chest pain mid-way through taking a drink. I had no trouble breathing during the pain it was a very griping pain. While this was happing I thought that I might be having a heart attack but considering my age I realiest it was highly unprobable. After a few seconds the pain stoped then I was sure it was not an heart attack. What could this be? I am a type one diabetic with a vitamin D deficiency and slightly a rased cholesterol all of them are being treated. I hope that info helps at least somewhat. Thanks, Doctor: Chest pain could be due to many reasons .No age is immune for heart attacks .Since you are a diabetic and also have elevated cholesterol you have risk factor for developing Coronary artery disease.another probable cause could be oesaphagitis which is inflammation of the food pipe caused by your drinking.Please do an EKG and a stress testing and an echo to rule out any underlying heart disease ."
},
{
"id": 38229,
"tgt": "What causes swelling in arms?",
"src": "Patient: I was bitten by something maybe a spider on the inside of my wrist about 30 hours ago. There are two small bite marks about a quarter inch apart. There is some pain and a lot of swelling. My forearm is very swollen now and the swelling seems to continue spreading. I am a male in good health and I m 54 years old.Should I go to the ER and do you have any idea of what may have bitten me. Thank you for your time. Doctor: Hello, Thnx to contact us. If I am your treating doctor I would like to advice you that there is not worry. If the symptoms is very acute or life threatening than you have to think to go ER. But as it is already 30 hours there is no need to worry. You can start with antibiotics like cefixime, pain killer like aceclofenac, antihistaminic chlorpheniramine maleate, antiacidity agent pantoprazole. wound will recover in some week or so. If you have anything else to ask please contact me. Thanx. Dr. Arun Tank; Infectious Disease Specialist"
},
{
"id": 140961,
"tgt": "How can foraminal stenosis be treated?",
"src": "Patient: 63yr old male patient, vertigo both BPPV and Vestibular, cervical pain w/headaches The signal in the cord and partially imaged posterior fossa is unremarkable. Spinal canal stenosis and kinking of the cord is again noted at the C5 and C6 levels. Disc space narrowing and degenerative endplate signal changes are most apparent at the C5-6 level. No abnormal STIR signal is seen. The vertebral body heights are otherwise well maintained. No evidence of a marrow replacing process to suggest neoplasm or acute trauma is seen. The paraspinal soft tissues are unremarkable. At the C2-3 level, mild narrowing of the right neural foramen is seen. At the C3-4 level, mild narrowing of the neural foramina bilaterally is seen. At the C4-5 level, a left paracentral disc osteophyte complex or protrusion mildly effaces the anterior thecal sac. Narrowing of the neural foramina bilaterally is seen. At the C5-6 level, a mild diffuse disc osteophyte complex with protrusion of the inferior C5 and superior C6 end plates posteriorly is seen. Mild compression of the cord at this level is seen. No cord edema is seen. Narrowing of the neural foramina bilaterally is seen. At the C6-7 level, mild to moderate diffuse disc osteophyte complex effaces the anterior thecal sac and mildly narrows the neural foramina bilaterally. At the C7, T1 level, no significant disc osteophyte pathology is demonstrated. At the T1, T2 level, no significant disc osteophyte pathology is demonstrated. The partially imaged parotid and submandibular glands are unremarkable. The parapharyngeal spaces are fairly symmetric. The thyroid gland is not well visualized. No definite enlarged lymph nodes are seen. Facet hypertrophy is noted at multiple levels. Impression: Degenerative endplate signal changes with spinal canal stenosis at the C5-6 level. Associated mild disc osteophyte complex. See above discussion of each vertebral level. No cord edema. Suggestive possible treatments. Doctor: Hello, The main treatment for chronic degenerative cervical changes such as in your case is with physical therapy and painkillers. Pain killers consist first in over the counter anti-inflammatory medication such as Ibuprofen and if not enough medication for chronic pain such as Gabapentin or Duloxetin. Mind you that medication is only to alleviate symptoms, it is an only physical therapy which has a benefit in the long term. If symptoms persist in local shots of steroids and analgesics may be tried. If there are symptoms indicating cord compression and neurological deficits on physical examination then surgery might be considered. Hope I have answered your query. Let me know if I can assist you further. Take care Regards, Dr Olsi Taka, Neurologist"
},
{
"id": 93894,
"tgt": "Severe cold spells, lost appetite, cronic stomach pain. What is the treatment?",
"src": "Patient: Hi,For the past few days, I have been suffering severe cold spells (where I am wrapped in 2 quilts and still shaking) only then be extremely hot and botheres about 4 hours later.I have completely lost my appetite.I have cronic stomach pains which hurt when moving around.I live in a warm climate but I don't really like going to the doctors as they give you an array of brightly coloured pills which I have no clue as to what they are half the time.Thanks for your help Doctor: Hi welcome to Health care magic forum. Thanks for asking a question to H.C.M.Forum. Your symptoms strongly suggest a malaria fever, with accompanied hepatitis. You must be having a dark colored urine as well. You are having chills, and abdominal pain, loss of appetite,followed by extreme hot feeling. I advise you to consult a physician for diagnosis and treatment. You may have to go for a series of blood tests for confirmation. Acourse of anti malarial drugs will cure the chills, hot flush, stomach pain, and loss of appetite. Wishing for a quick and complete recovery. Best regards."
},
{
"id": 46477,
"tgt": "How to reduce creatinine level?",
"src": "Patient: Hi, my mother aged 60 had recentrly went both knee replacement. she got some hospital acquired bactirea infrection which leads to plumonary thrombosis. it was cured. our docotor was advised acitrom tablet to maintain PT INR value. due to local lab fault we are unknown that the INR Value is reached to 15.6 and the lab is projecting only 1.5. yesterday only i came to know this, and i rushed to hospital. they told it malena. today when we checked creatinen its 3.9 which was very high. can u pl suggest how to get down the creatinen value atonce. thank you Doctor: HelloThanks for query .Based on the facts that you have posted it appears that your mother landed in pulmonary thrombosis secondary to kidney infection and is on anticoagulants and her PT INR is being watched frequently .However her PT INR was raised to 15 and was wrongly reported to be 1.5 resulting into internal bleeding in GI tract which reflected as a malena (Occult blood in stools). and her serum creatinine has been detected raised to 3.9 mg .The raised serum creatinine is likely to be due primarily due to kidney infection and not due to raised PT INR.Consult qualified Nephrologist for assessment .Taking adequate fluids and antibiotics with diuretics should help to normalize her kidney function and lower down her creatinine levels .Dr.Patil."
},
{
"id": 182104,
"tgt": "What causes headaches after teeth removal?",
"src": "Patient: Hi,2 weeks ago I had all 4 wisdom teeth pulled , a moler and an extra tooth that I had in my upper pallet. I don't have any pain anymore but I do wake up every morning with a huge headache ever since the surgery. I feel the pain more on my temples . Will this pain ever go away? Doctor: HelloUsually there is never a side effect like head ache due to the removal 4-5 teeth.It could be due to the altered diet habits. Try to take more carbohydrates, protein and calcium based food.During extraction of so many teeth the tissue takes time to heal and needs correct nutrition for it.So kindly regularise your meals.If it isnt subsiding at all then show to a physician for possible migraine.Thank you."
},
{
"id": 212737,
"tgt": "Smoking weed, have thyroid disease, on inderal for rapid heart rate. Now?",
"src": "Patient: hi i have a question i know its bad but i smoke weed a lot it calms me down...and i am now recently diagnosed again with a thyroid disease its overactive..ive had it since i was 8..they said it was gone 6yrs later then i started smoking pot and it helped alot for me..now i cant see my dr till a few weeks from now for my meds for it..i went to the er and they gave me inderal 60mg for my heart rate cuz its too fast...i am in need of help please weed is the only thing that works for my anxiety and depression.. sincerely phil Doctor: Hello and welcome to Healthcare Magic. Thanks for your query. Inderal (Propranolol) is a medication which is prescribed for an increased heart rate as well as to reduce the physical symptoms of anxiety like palpitations, tremors, etc. Both thyroid disorders as well as smoking weed (cannabis) are known to trigger or worsen anxiety problems. You should not depend on smoking weed for tacking your anxiety and depression, as it can have long-term harmful health consequences. There are effective treatment options available - in the form of medication or counselling / psychotherapy which will help him overcome your problems. I would advise you to see a psychiatrist for a detailed psychological assessment and further treatment. Wish you all the best. Regards, Dr. Jonas Sundarakumar Consultant Psychiatrist"
},
{
"id": 125692,
"tgt": "How can pain and bruising in knuckles be treated?",
"src": "Patient: My right knuckle started hurting without hitting it and I looked down to see a bruise there all of a sudden and as time passed, not only this the bruise and pain spread, but the left knuckle started doing it. My right pinky is starting to tingle as well. Thoughts? Doctor: Hello, It might be due to minor trauma or abrasion that may have gone unnoticed. As of now, you can apply ice packs for symptomatic relief. If symptoms persist, it is better to consult a physician and get evaluated. Hope I have answered your query. Let me know if I can assist you further. Regards, Dr. Shinas Hussain, General & Family Physician"
},
{
"id": 193616,
"tgt": "How to treat erectile dysfunction?",
"src": "Patient: Hi, may I answer your health queries right now ? Please type your query here... sir im 21 yrs of age, my penis is 14cm recently i had sex with my gf but i was not able to penetrate, as my penis doesnt get hard enough, it was the first time i had sex . i dont smoke but drink but just occational drinker, sir is this a serious problem for my future life Doctor: Hi, Your erectile dysfunction could be due to psychological issues or performance anxiety for the first time. The erectile dysfunction problem is very rare at such a young age. However still you can investigate with serum testosterone estimation and get checked your sugar level and measure blood pressure. You can practice more foreplay for getting a hard penis and than only try to insert it in vagina. You can Consult sexologist for counseling. Hope I have answered your query. Let me know if I can assist you further. Take care Regards, Dr Parth Goswami, General & Family Physician"
},
{
"id": 223868,
"tgt": "What are the side effects of missed birth control pill?",
"src": "Patient: Hi, I missed a pill at the end of my second week/start third week and took it approximately 13hrs after I was meant to. I have since continued taking pills regularly and finished the pack. My pill-free week will start tomorrow. Should I just start a new pack and skip the pill-free week or will I be protected from now onwards since I have taken 7 pills after the delayed one? Thank you for your help! Doctor: HelloHealth care magic user I understood your concern and My opinion is that as such they are no serious side effects if you have missed your period and you have taken immediately with in 13hrs. so you are totally safe dont worry If you dont want to get your period and wants to continue protection than you can start your next pack todayI hope my answers help you Thanks"
},
{
"id": 111419,
"tgt": "What treatment is suggested for pain in the center of the lower back?",
"src": "Patient: Hi , I am only 18 years old and for two days now i have this pain in the center of my lower back . When I get up i feel the pain and if i sit for two long . Also when i walk i sorta have to bend my knees and make this strange arch with my body so i can move around . Doctor: Hi, welcome to our site. I am Dr Saumya Mittal, MD.Read your query. That is a very significant question and i appreciate your problem. I will try my best to answer your queryNow what you have described is an acute/sudden onset of pain in the back that is restricting the mobility. Since you have not mentioned it, I am assuming that you have not developed any symptoms in the legs yet.I would sincerely suggest an MRI of the LS spine. The MRI would be able to give us the information as to the cause of pain in the backThe medicines can be prescribed accordingly. So do let us know the report of MRI for further suggestionI hope this helps you. Inform the reports mentioned above/if any other so I can be of help further. I have given you the answer to the maximum considering the information provided. The results of the tests could further enhance my answer to you.Please do understand that some details could be extracted from a detailed history and examination.Please feel free to ask another query. I would be glad to help you. Looking forward to your return query with the details asked so that I can help you further. (If the answer has helped you, please indicate this)Best of luck."
},
{
"id": 171978,
"tgt": "What are the merits and demerits of Gemcal medication?",
"src": "Patient: want to know about merits and demerits of medicine called gemcal my younger brother who is now 11 years old is marked to be suffering with a disease called dmd he take this medicine with one more medicine called wyeth now he has become more fat than before and is having weight 47 . plz help Doctor: Hi,Thanks and welcome to HCM .Gemcal is a calcium preparation .Wyeth is not a medicine it is company's name.Children usually become fatty if they are taking steroids for a prolonged period.Hope this answer serves your purpose .Further queries invited .Dr.M.V.Subrahmanyam."
},
{
"id": 120022,
"tgt": "Is it safe to crack nose and chest?",
"src": "Patient: I can crack my nose where my nostrils are located and can also crack my sternum or chest. My nose is actually caving in at the nostrils more and more the older I get which is causing me to snore more at night and my chest is beginning to hurt every now and then as well. The chest cracks on it s own most of the time when I move a certain way...Is this safe or should I see someone about this? Doctor: Hi, It is safe, there is nothing to worry about. It's completely normal, no need to see a doctor for this cracking sound. Hope I have answered your question. Let me know if I can assist you further. Regards, Dr. Jaideep Gaver, Orthopedic Surgeon"
},
{
"id": 189645,
"tgt": "Incision made to drain the abscess after repeated root canal treatments, swelling reduced, but jaw hurts. Any treatment again ?",
"src": "Patient: Hello. I m a 26 year old female who is, overall, in very good health. I had a root canal done on tooth number 29 about ten years ago after an injury to the tooth. All seemed well until I noticed a swelling under this tooth a few weeks ago that hurt when pressed. I immediately called the dentist and was seen the next day. The dentist did an X-ray and I was told I had an abscess . She prescribed me 150mg Clindamycin over ten days for the infection and referred me to an endodontist. The endodontist retreated the root canal a couple weeks later and seemed happy with his work. A few hours after the retreated root canal tooth became horribly painful and I noticed some swelling. That night, swelling continued to intensify and spread to my cheek, but pain subsisted. I called my endodontist and he said that this was a flare-up of my old infection and I needed a called a 24-hour pharmacy to get me a prescription amoxicillin (500mg). The next day I went to the endodontist office and they had to make an incision to reduce some of the swelling and allow the abscess to drain. I was also given a prescription for metronidazole (500mg) to take in addition to the amoxicillin. Thankfully the swelling in my cheek subsisted gradually and now a week later my face looks normal, however I still have a fair amount of swelling and tenderness in my gum under the retreated root canal and my jaw hurts a little. Every time I call the endodontist the receptionist tells me to call back in a few days because the swelling should be gone and every few days when I call the swelling remains the same and yet I am told to call back again in a few days...very frustrating and worrisome. My questions are as follows, how long should it realistically take for the swelling and tenderness in my gum under my tooth to go away? Is there any chance of this infection spreading to my cheek again as I am now finished with my antibiotics? Should the tooth be treated again or possibly extracted? When they open the area again to permanently seal the root canal or do any other work is there a possibility of the infection spreading again? Thanks! Doctor: Hello and welcome to hcm forum, Since you got the re-rct for a traumatically injured tooth,chance of root fracture/spread of infection into adjacent tissue spaces has to be ruled out. Usually the pain and swelling after access opening and pulp extirpation lasts for 2 weeks. As the pain remains, in your case the tooth has to be checked for any fracture or infected pulp remaining within the pulp canal. If required,adjacent teeth also has to be root canal treated. If the symptoms persists,get it extracted. Hope this helps."
},
{
"id": 213197,
"tgt": "Head spinning, unclear vision, nausea. Blood phobia?",
"src": "Patient: Hello Doc. Im a 20 year old male having a healthy lifestyle no major diseases. Yesterday a young cousin of mine got injured pretty badly. I was the one who cleaned his wounds and applied antiseptic lotion to them. I even checked closely whether the wounds needed stitches or not. Till that instant was completely OK. But when i was walking him up to the Doctors Clinic, suddenly my head and my stomach started spinning badly and my vision became extremely unclear. I almost vomited a couple times and felt that i was gonna faint but finally somehow managed to reach the clinic where i became better after lying down. Everyone says that Im suffering from Blood Phobia but that cant be right because when i was cleaning his wound and checking them, i didnt feel any bit uneasy or disgusted. And i myself have had bad history of injuries in my childhood when i easily used to watch the doctor stitch up my skin. So what was this nausea all about? Doctor: Hi and welcome to Healthcare Magic. Thanks for your question...Phobia is a type of anxiety disorder where there is excessive and irrational fear of something, leading to avoidance of that fearful stimulus / object. In your case, the appearance of your symptoms seem to be corelating with the incident of closely obseving blood / injury and so is likely to have been the cause of your nausea. But still, in order to technically qualify for a diagnosis of blood phobia, you need to have had persistent such aversion to blood / injury, along with avoidance of such situations (...which you say is absent). It could be that the straining of your eyes to closely observe / clean his wounds along with fatigue may have caused the nausea. However, if these symptoms keep repeating again in similar situations, then it is more likely that you have a phobia.Wish you all the best.- Dr. Jonas SundarakumarConsultant Psychiatrist"
},
{
"id": 99718,
"tgt": "Can I take Aspirin and Simvastatin for allergy?",
"src": "Patient: m currently having a allergic reaction that started on September 20,2014. I don t know whats the cause I haven t eat something that I never eaten before or used. Im currently on aspirin 81g for at least 3 months know and im also on simvastatin 10mg. could it be the pills?? Doctor: hi, what kind of allergic reactions u are facing u didn't mention here, and also for which treatment u are taking Aspirin and Simvastatins?its important to know this two answers first.if u think that only after taking this drugs u started allergic reactions then u can stop this drugs and immediatly contact your physician for further advise."
},
{
"id": 50095,
"tgt": "Urge to urinate, distended abdomen. Had surgery to remove kidney stones. Taking stool softeners. Normal?",
"src": "Patient: I just had surgery to remove stones from both of my kidneys and also had stents placed in both of them. I am urinating but I feel like I am not going as much as I should. My abdomen is also extremely distended. I have been taking stool softeners 3x daily. Is this normal and is there anything I can do to relieve the swelling? Doctor: you may be having urinary tract infection which is common with ureteric stents in place.get a urine culture and sensitivity done and take a course of appropriate antibiotic as suggested by culture report."
},
{
"id": 218039,
"tgt": "Which is a good OTC medicine replacement for vicodin taken for fibromyalgia pains?",
"src": "Patient: Last year i was diagnosed with severe fibromyalgia and was put on vicodin 5/500 for pain and it helped. I just recently lost my job and cannot afford to go to my drs and they wont write a prescription for my vicodin unless i make an appointment....which i cant afford. Not sure what to do or where to turn next. Im back in severe pain once again! Doctor: HIThank for asking to HCMI appreciate your concern fibromyalgia is psychological disease may aggravate with the environment factor and functional elements, I generally advised NSIAD to my patients if ever comes with such disease, and the best option are \" Tab Acetaminophen with Aceclofenac and Tab Chlorodiazepoxide three times in day no need to take any potent analgesic, try to bear the pain as hard as you can this could give natural effects of analgesic, hope this information helps you, have nice day."
},
{
"id": 225795,
"tgt": "On birth control for a week, sexually active. Will taking one week of pills be effective? Could i still be pregnant?",
"src": "Patient: I have been taking birth control for a week now, same time every day, and haven't missed a dose and just recently became sexually active. Along with birth control, the first time we used a condom, but the second they were too small so instead he \"pulled out\" the other times. I'm still worried I could get pregnant. Even if I've only been on birth control for a week, will the effects still be as powerful? Could I still be pregnant? Doctor: Hello,Thanks for posting your query.If You started taking birth control pills from second day of your cycle & taking its regularly then you are safe & can have unprotected intercourse.However if you started its from mid of your cycle then you would have to use barrier contraceptive method along with oral pills for 2 weeks.I hope your query has been answered.Regards,"
},
{
"id": 139486,
"tgt": "Suggest remedy for loss of consciousness",
"src": "Patient: hi, my father 75yrs suffering from parkisons disease since 4 n half yrs on regular treatement, diabetic since 6yrs on hypoglycemic drugs regularly, after falldown 18monts back he got fracture neck humerus left side, since then he is almost bed redden, recently he became drowsi and unconscious on 19sep2011 for 8hrs, ct brain done but was doutful, so mri brain done shown 2 small ischemic infarcts, 8days later 0n 27sep2011 again same episode of loc, and again 3rd time on 1oct2011 loc for 10hrs.. doctors still investigatins,, can u plz help from ur valiabel advice Doctor: Hi, Based on the information provided, the loss of consciousness can not be explained by two small infarcts in the brain. EEG is required to rule out any subclinical seizures. CSF analysis should be done to rule out CNS infection. Metabolic derangements such low sodium or low sugar should also be excluded. Hope I have answered your question. Let me know if I can assist you further. Regards, Dr. Sudhir Kumar, Neurologist"
},
{
"id": 156601,
"tgt": "What are the symptoms of ovarian cancer?",
"src": "Patient: Hi i have numerous lumps/reactive lymph nodes but the ones in my groin relly hurt and my current period is almost 4 weeks late, been having back and tummy pains on and off and nausea for 4 weeks, i changed dr who has booked me for a chest x ray will this say whats going on down below? also booked for lletz as smear said servere dyskaryosis came back cin 3 after biopsy. Strong family history of cancer at a young age aunt had overian cancer at 28 and im now 29, could this be whats causing period problems, overy pain, reactive lymph nodes and pain deep inside vagina? Thank you Doctor: Ovarian cancer may cause one or more of these signs and symptoms\u2014\u2022Vaginal bleeding or discharge from your vagina that is not normal for you.\u2022Pain in the pelvic or abdominal area (the area below your stomach and between your hip bones).\u2022Back pain.\u2022Bloating, which is when the area below your stomach swells or feels full.\u2022Feeling full quickly while eating.\u2022A change in your bathroom habits, such as having to pass urine very badly or very often, constipation, or diarrhea.RegardsDR De"
},
{
"id": 123366,
"tgt": "How to cure broken bone at the tip of the right middle finger?",
"src": "Patient: broken finger I am a 46 year old male. I broke the tip of my right middle finger bone all the way across. When I went to urgent care at the beginning of May, they took an xray. I went to an orthopedic surgeon right away afterwards where the doctor applied a splint. I was supposed to keep it on for 6 weeks, but it was way too difficult, so I made another appointment. The same doctor removed the splint and recommended surgery to pin the bones together through my finger. It would take about an hour and I d be knocked out. I decided to get a second opinion and scheduled an appointment with a more senior doctor in the same practice. He reviewed another xray of the finger and gave me his take on the situation. His conclusion was that if this were him, he would not have the surgery. So, now after around two months since the injury, I m still in pain. My finger is still swollen and it is bent and slightly deformed. Every time I use this finger it hurts. Every day there s pain because anything I do with my hand involves putting pressure on the wound. The doctor s suggestion was to flex the tip of my finger, holding the majority of the finger, allowing the tip to be exercised. At work I just irritate the injury by typing the keyboard of my computer. Every time I write, open a door, lift my son, grab items, use silverware, tie a shoe, etc. it becomes painful. I don t know how to alleviate the pain and stop the swelling. What can I do? How can I prevent permanent damage? Should I see another doctor? Should I just accept the fact that it s never going to heal properly and the pain will continue? What other options do I have? I m tired of the non-stop dilemma I face consistently. Please help!! Thanks for any help you can provide! Doctor: Hello, As the history indicates there will be an avulsion fracture of the phalanges for which the splint was advised. as in most cases, the bone heals with the immobilization and doesn't need surgery. In case if the patient insists more for other options than surgery is guided. But as per my clinical practice of over 12 years, most of the cases with a fracture to phalanges have to be resolved well with conservative management. Hope I have answered your query. Let me know if I can assist you further. Regards, Jay Indravadan Patel, Physical Therapist or Physiotherapist"
},
{
"id": 144134,
"tgt": "Suggest treatment for vertebral collapse and lesion",
"src": "Patient: the patient has suffered D 11 vertebral collapse and a paravertabral lesion was seen in the mri so the Ct guided Fnac was done and the fnac report of the same sample from one labs shows lymphocytes, histiocytes, neutrophils, few multinucleated giant cells and necrosis in the background while the other report from second lab shows hemorrhagic paucicellular smears showing occasional osteoclastic giant cells on a background of inflammatory infiltrate. what is the cause? and what treatment she should be given. Doctor: Hi, I am Dr.Bruno. I have read your question and understand your concerns. Let me try to help you Question : What treatment she should be given.Answer : The Collapsed Vertebra should be fixed by a Neurosurgeon so that it does not lead to paraplegia . While fixing it the neurosurgeon will take a biopsy of the lesion. With Biopsy, you can have correct diagnosis. FNAC may not always give the full diagnosis since only cells are seen. In Biopsy the architecture of the lesion also will be seen Hope you found the answer helpful.If you need any clarification / have doubts / have additional questions / have follow up questions, then please do not hesitate in asking again. I will be happy to answer your questions.Let me know if I can assist you further.Take care."
},
{
"id": 214931,
"tgt": "What is the natural remedy to cure acne scars ?",
"src": "Patient: Doctor tell me How to remove dark spots of acne naturally? I am suffering from acne in my face since my puberty . It causes lots of dark spots in my face. Is there any natural way to reduce these dark spots? I am eagerly waiting for your reply. Doctor: Hi, , Go for facial wash 3-4 times a day, Avoid using cosmetics on face. Apply aloevera cream, Apply kojec cream Take plenty of water, Take green veggies and fresh fruits. Take vitamin A and D. Avoid stress. Ok and bye."
},
{
"id": 136051,
"tgt": "What causes swelling on legs and feet?",
"src": "Patient: mY HUSBANDS LEGS & FEET HAVE BEEN SWELLING REALLY BAD FOR ABOUT 2 TO 3 WEEKS. THE DR. SENT HIM FOR A TEST FOR A CLOT THE GUY DIDN T SEE ONE BUT TOLD HIM HIS LEG WAS REALLY FEVERED. NOW DR IS OFF WORK FOR THE WEEKEND & NO ANSWERS... wHAT COULD THAT MEAN? Doctor: HiWelcome to healthcaremagicI have gone through your query and understand your concern.That means his legs has raised temperature. It means he may be having cellulitis. Treatment of it antibiotics, analgesic such as ibuprofen and becomplex. You can discuss with your doctor about it. Hope your query get answered. If you have any clarification then don't hesitate to write to us. I will be happy to help you.Wishing you a good health.Take care."
},
{
"id": 213987,
"tgt": "my daughter has herpes and this is her first valentines",
"src": "Patient: my daughter has herpes and this is her first valentines day. what do i say to console her Doctor: hi; your daughter has herpes on which part of her body? anyway herpes is a painful condition most of the time ;if you could elaborate may be we can help better .If it is for sexual relation you are asking for it is absolute NO. thanks"
},
{
"id": 127965,
"tgt": "What causes rib pain post treatment for post nasal drip?",
"src": "Patient: Hi I have taken antibiotics and steroid medication for post nasal drip. I m ill for 5 weeks. The constant dripping in back if throat did not stop for 5 months. I m steaming and used the nebulizer. I now got an inhaler. The issue now is I have terrible pain on tue right rib and going towards the back. I went for xray and it indicates there is coarse bronchovascular pattern noted I both lung fields. I m told nothing wrong and the pain could be from coughing. I did not receive any medication at all? Please help I don t feel right internally.. . Doctor: Hello As per your history it suggests your respiratory system is weak , so you need extra care to maintain it.I will advise you bronchoscopy and HRCT chest to see for origin of symptoms.I will advise you to improve your immunity by herbs exercises and healthy diet.Also you can improve your lung capacity by yoga and pranayama.If you are having congestive heart failure due to hypertension then respiratory system can be involved.For tubercular infection you can do mantoux test, sputum culture, ESR and PCR all non invasive.Hope this answers your query. If you have additional questions or follow up queries then please do not hesitate in writing to us. I will be happy to answer your queries. Wishing you good health.Take care"
},
{
"id": 150703,
"tgt": "Have facial tics. Taken magnesium, calcium, done facial massage, EFT. How to cope with the problem?",
"src": "Patient: Im a 23 year old suffering from facial tic disorder . Usually happens (or threatens to happen) whenever I have excessive emotions (i have a mild anxiety disorder ). It first came about 6 years ago and it was a very embarassing moment. I seemed to have forgotten about it for a while but it never completely went away. Now its threatening my life again. I ve tried all sorts of things like magnesium and calcium intake, facial massage, EFT but to no avail. Its hurting my social life. What else could i possibly try to help cope with this problem? P/S: I have had numerous injuries on my head as i was growing up. Doctor: Hi, Thank you for posting your query. First of all, let me reassure you that good treatments are available to deal with facial tics. Medications likely to help include trihexiphenidyl, tetrabenazine, sodium valproate, etc. Please consult a neurologist for the choice of appropriate drug and its dose. In addition, reducing the anxiety would also help. Counseling and seeing a psychologist would help. Best wishes, Dr Sudhir Kumar MD (Internal Medicine), DM (Neurology) Senior Consultant Neurologist Apollo Hospitals, Hyderabad, My personal URL on this website: http://bit.ly/Dr-Sudhir-kumar My email: drsudhirkumar@yahoo.com"
},
{
"id": 59029,
"tgt": "Chest congestion and belching after gall bladder removal. Any suggestions?",
"src": "Patient: I am a 46 year old female, had ny gall bladder taken on March 5 2013, everything seemes to be a little better, for the last 2 -3 weeks I have been experiencing a \"stuck\" feeling in my chest and in the lower part pf my throat/ neck after eating and I am belching constantly when this happens, any suggestions please.. Thank You Doctor: Hi and welcome to HCM. Thank you for your query.This can be caused by so called postcholecystectomy syndrome which isnt uncommon condition. OR there is stomach issue such as gERD or gastritis. In every case you should change dietary habits. Eat more frequently but smaller meals,avoid fatty food, alcohol and smoking. Take antispasmic drugs when you have pain and take antiacid medicines. FOr further evaluation endoscopic and imaging tests should be done to see is there any pathology.Wish you good health."
},
{
"id": 163969,
"tgt": "What causes a pointed feeling on the skull of a child?",
"src": "Patient: Hello, When I feel the top of my 2 year olds head it feels as though the skull is pointed running from the back towards the front and down the center of his forhead, I know the bones arent fused yet but it feels almost like a roof peak would, any idea what this could be? Doctor: Hi..by what you say - the description is fitting into a medically described skull shape called brachycephaly. If the kid's development is normal, you need not worry. But if the developmental delay has been there, then I suggest you get him evaluated by the pediatrician.Regards - Dr. Sumanth"
},
{
"id": 9562,
"tgt": "Itchy red patches on jawbones, flaky and dry skin. Prescribed Zindaclin",
"src": "Patient: Good Day. I am a 41 years old Male, non-smoker. Over the past few weeks I have developed itchy red patches on my jawbones. The area is very flaky and is constantly dry. The condition is aggravated by shaving and even though I have been putting on different creams and lotions to hydrate the area, the itchiness and flaking continues. My physician has recommended applying Zindaclin 1% Gel and Eczmol 1% w/w cream none of which is proving beneficial. I do suffer from sebhoerric dermatitis on my scalp which I treat/manage with Alphosyl Shampoo but I have rarely had issues with my face. Please help. Thank you. H Doctor: Hi, Most probably the lesions over your face are that of seborrheic dermatitis only. The similar involvement of your scalp also supports the diagnosis. Though you can use a plain coal tar based shampoo, it would be more beneficial if you use a ketoconazole and coal tar based shampoo( for eg konaz ct). For your face lesions use a steroid anti fungal combination (like momoz t) twice in a day for a week. Hope that solves your query. Take care."
},
{
"id": 190165,
"tgt": "Enlarged papillae and white spots on tongue. Reasons?",
"src": "Patient: Hi, I am 21 years old and recently (for the last 2 months or so) I have had 2 enlarged circumvallate papilli on the back of my tongue and white spots at the back of my tongue as well. It is all clustered to the back and is not on any other surface in my mouth. I saw a doctor but they did not seem to think these symptoms were alarming, but they never went away. What are common reasons for this and what should I do? Should I see an ear nose and throat specialist? Doctor: Hello Welcome to HCM I would like to tell you that papilla enlargement can be seen in cases like:- -Iron & vitamin deficiency -Poor Oral hygiene -Smoking -Trauma First you need to rule out the cause & then get it treated accordingly. Consult your dentist for further query. Take Care Regards Dr.Neha"
},
{
"id": 97117,
"tgt": "Is tetanus vaccine needed in a iron nail injury?",
"src": "Patient: I got a scratch by the iron nail but that was not the direct contact with the skin , I was wearing a jeans at that point ...... I just want to know I dint get the tetanus injection because Of the scab formed on the wound or scratch.. It was not a deep cut just a scratch and scab formed on it with a slight pain around in the muscle. Please tell me is ok for now without getting injected with tetanus vaccine . Doctor: Hi,If you have been vaccinated against tetanus in the last 10 years then you do not need further vaccination. However, if you are not sure then I recommend you get vaccinated as the injury did cause a break in the skin. This would be the safest course of action.Regards,Dr K A Pottinger"
},
{
"id": 26600,
"tgt": "Does presence of cardiomyopathy along with ischemia affect the life expectancy?",
"src": "Patient: I have cardiomyopathy and sleep apnea. Four years ago, I was implanted with a bi-ventricular pacemaker with 3 leads to the heart. Recently, I have been diagnosed with ischemia. My beta blockers have been doubled to 25 mg. twice a day. What does that do to my life expectancy? Doctor: Thank you for choosing HCMHaving cardiomyopathy can significantly affect your life expectancy. The medication that you are on is correct and I do not think that you should worry. The best way to know the function and the effectiveness of the heart, you will need to get the heart checked up. You should see a Cardiologist to get a Echocardiogam and Technetium scan which will help find out if the heart is working properly.Please follow up after the results of the ECHO and the scan are back. All the best.Hope this helps you.Dr Vignan"
},
{
"id": 32499,
"tgt": "What is the treatment for a infection in the buttock?",
"src": "Patient: My two year old had an infection at the top edge of her butt around her pullup line. I've given her and sprayed silver biotics on it and the infection appears to be gone and the entire section that was swollen is getting better and now scar tissue is draining. It was very swollen in this area and although it looks better how much longer should it take for the scar tissue to drain or is it reproducing more? But she's walking better saying shes feeling better. I was trying to avoid going to the doctor because I prefer natural medicines. Doctor: Hi,As there is drain from the scar indicates that there is bacterial infection and might require one antibiotic medicine course to clear the infection.Clean the wound properly and apply antibiotic cream.Consult your doctor and discuss about antibiotic medicine.Ok and take care."
},
{
"id": 95706,
"tgt": "What is the purpose of using optalidon?",
"src": "Patient: What is the purpose of using optalidon? Doctor: hi welcome to HCM.optalidon contains caffiene,its given for severe migrain and severe head ache."
},
{
"id": 80266,
"tgt": "What is the cause of chest pain and blood in spit?",
"src": "Patient: I woke up unable to swallow. I have bad chest pain & have been seeing blood in my spit. I didnt eat all day & when i finally did I threw up & there was more blood in it. My throat is burning & my nose is stuffy. Is this just a cold or should i go to the hospital? Doctor: Thanks for your question on HCM. I can understand your situation and problem. You are having hemoptysis (blood in sputum). Chest pain with hemoptysis is seen in following conditions. 1. Tuberculosis 2. Pneumonia 3. Lung cancer 4. Chronic bronchitis So better to consult pulmonologist and get done 1. Clinical examination of respiratory system 2. Chest x ray 3. PFT (Pulmonary Function Test). Chest x ray is needed to rule out tuberculosis, pneumonia and lung cancer. PFT is needed to rule out chronic bronchitis. You are also having dysphagia (difficulty in swallowing), so upper GI scopy is also needed to rule out any esophageal pathology. So don't wait at home, better to consult doctor and discuss all these."
},
{
"id": 121583,
"tgt": "What to do for achilles tendinitis?",
"src": "Patient: Hi, I am 40 and suffering from Achilles tendinitis of portion just joining the heal on both leg. My weight is 80 Kg and height 175 cm. I some time go for running to remain fit but then heels pain for few days. What is the remedy and where can i get good medical shoes for me in Bangalore, India Doctor: Hello,Your symptoms could be related to chronic trauma. I recommend checking uric acid plasma levels and inflammation tests (complete blood count, PCR, ESR, calcium and vitamin D plasma levels). In the meantime, I would recommend using orthopedic shoes, which you can buy in the pharmacy.Hope I have answered your query. Let me know if I can assist you further. Regards, Dr. Ilir Sharka, Cardiologist"
},
{
"id": 151289,
"tgt": "Infant suffered with episodes of convulsion, CT report showed suspicious frontal lobes atrophy, advised MRI, on injection. Next step advise?",
"src": "Patient: dear sir, my 9 month female child got convulsion, she had fever of 38.9c. she had three episodes. In ct report there is a suspious frontal lobes atrophy ?. suggested for MRI . Inj Phenobarbitone started and 20 mg /day started. can i have your detailed opinion, we want to stop the medication . baby is active and everything is normal. please give an idea to whant to do? Doctor: good evening. by the problems u have posted , my first diagnosis offcourse not having the ct scan report in the hand, i would label it as a febrile seizures. but seeing the ct scan report and the treatment started , i would want to go for a further test -EEG, which helps in guiding the nature of seizure. i would suggest a detailed examination by a pediatric neurologist for ur baby and then decide upon the drug because these days many newer drugs with least side effects and maximum results are available.. ct scan report is only saying a suspicious frontal lobe atrophy, so do not worry , most of the times it would be normal variables. so do not worry. also i would stress upon getting a serum calcium serum magnesium assay for ur child. if the final diagnosis by ur attending doctor is FEBRILE SEIZURE, then continuing phenobarbitone is not necessary, but u have to take care that each time the child gets fever , u have to sponge her with warm water covering the whole body to prevent further attacks."
},
{
"id": 210961,
"tgt": "What is the smell that is disturbing my mind?",
"src": "Patient: I smell something....for the last couple days. No one else does. Not my husband or our kids. It smells like the cat maybe missed the liter box? It's making me nuts. Then today I started thinking maybe there's something wrong. I don't have a sinus infection. I'm not sick. I'm cleaned and showered for Pete's sake. This is driving me crazy. The smell is constant. Doctor: Hello,Thanks for choosing health care magic for posting your query.I have gone through your question in detail and I can understand what you are going through.You have an olfactory hallucination and the first suspicion that one should get on having this hallucinations are1. Local nasal pathology2. Space occupying lesion in the brain.You first have to get examined from a ENT surgeon to see if there is any local nasal pathology leading to this kind of symptoms. If it is normal then you should go ahead with CT scan brain to identify any SOL in the brain compressing in the olfactory pathway leading to the bad smell. The last possibility is a psychiatric illness causing bad smell. Hope I am able to answer your concerns.If you have any further query, I would be glad to help you.If not, you may close the discussion and if possible you may rate the answer for me, so that I get a good feedback.In future if you wish to contact me directly, you can use the below mentioned link:bit.ly/dr-srikanth-reddy\u00a0\u00a0\u00a0\u00a0\u00a0\u00a0\u00a0\u00a0\u00a0\u00a0\u00a0\u00a0\u00a0\u00a0\u00a0\u00a0\u00a0\u00a0\u00a0\u00a0\u00a0\u00a0\u00a0\u00a0\u00a0\u00a0\u00a0\u00a0\u00a0\u00a0\u00a0\u00a0\u00a0\u00a0\u00a0\u00a0\u00a0\u00a0\u00a0\u00a0\u00a0\u00a0\u00a0\u00a0\u00a0\u00a0\u00a0\u00a0\u00a0\u00a0\u00a0\u00a0\u00a0\u00a0\u00a0\u00a0\u00a0\u00a0\u00a0\u00a0\u00a0\u00a0\u00a0\u00a0\u00a0\u00a0\u00a0\u00a0\u00a0\u00a0\u00a0\u00a0\u00a0\u00a0\u00a0\u00a0\u00a0\u00a0\u00a0\u00a0\u00a0\u00a0\u00a0\u00a0\u00a0\u00a0\u00a0\u00a0\u00a0\u00a0\u00a0\u00a0\u00a0\u00a0\u00a0\u00a0\u00a0\u00a0\u00a0\u00a0\u00a0\u00a0\u00a0\u00a0\u00a0\u00a0\u00a0\u00a0\u00a0\u00a0\u00a0\u00a0\u00a0\u00a0\u00a0\u00a0\u00a0\u00a0\u00a0\u00a0\u00a0\u00a0\u00a0\u00a0\u00a0\u00a0\u00a0\u00a0\u00a0\u00a0\u00a0\u00a0\u00a0\u00a0\u00a0\u00a0\u00a0\u00a0\u00a0\u00a0\u00a0\u00a0\u00a0\u00a0\u00a0\u00a0\u00a0\u00a0\u00a0\u00a0\u00a0\u00a0\u00a0\u00a0\u00a0\u00a0\u00a0\u00a0\u00a0\u00a0\u00a0\u00a0\u00a0\u00a0\u00a0\u00a0\u00a0\u00a0\u00a0\u00a0\u00a0\u00a0\u00a0\u00a0\u00a0\u00a0\u00a0\u00a0\u00a0\u00a0\u00a0\u00a0\u00a0\u00a0\u00a0\u00a0\u00a0\u00a0\u00a0\u00a0\u00a0\u00a0\u00a0\u00a0\u00a0\u00a0\u00a0\u00a0\u00a0\u00a0\u00a0\u00a0\u00a0\u00a0\u00a0\u00a0\u00a0\u00a0\u00a0\u00a0\u00a0\u00a0\u00a0\u00a0\u00a0\u00a0\u00a0\u00a0\u00a0\u00a0\u00a0\u00a0\u00a0\u00a0\u00a0\u00a0\u00a0\u00a0\u00a0\u00a0\u00a0\u00a0\u00a0\u00a0\u00a0\u00a0\u00a0\u00a0\u00a0\u00a0\u00a0\u00a0\u00a0\u00a0\u00a0\u00a0\u00a0\u00a0\u00a0\u00a0\u00a0\u00a0\u00a0\u00a0\u00a0\u00a0\u00a0\u00a0\u00a0\u00a0\u00a0\u00a0\u00a0\u00a0\u00a0\u00a0\u00a0\u00a0\u00a0\u00a0\u00a0\u00a0\u00a0\u00a0\u00a0\u00a0\u00a0\u00a0\u00a0\u00a0\u00a0\u00a0\u00a0\u00a0\u00a0\u00a0\u00a0\u00a0\u00a0\u00a0\u00a0\u00a0\u00a0\u00a0\u00a0\u00a0\u00a0\u00a0\u00a0\u00a0\u00a0\u00a0\u00a0\u00a0\u00a0\u00a0\u00a0\u00a0\u00a0\u00a0\u00a0\u00a0\u00a0\u00a0\u00a0\u00a0\u00a0\u00a0\u00a0\u00a0\u00a0\u00a0\u00a0\u00a0\u00a0\u00a0\u00a0\u00a0\u00a0\u00a0\u00a0\u00a0\u00a0\u00a0\u00a0"
},
{
"id": 204248,
"tgt": "How to resolve communication issues in a long distance relationship?",
"src": "Patient: its complicated, I live in Texas and my girlfriend lives in Kentucky. I recently flew in and we went on a cruise. Started okay but then I absolutely couldn t talk to her, couldn t come up with conversation or anything. She is now frustrated and done with me. Giving me one last chance to tell her why I couldn t talk to her and hold a conversation, but I was able to with random people on the cruise. We were together for 10 days. I have no good answer to tell her as this has never happened before and I understand how and why it happened. I am scheduled to see a therapist to figure out somehow why I wasn t able to. Any help or insight in this would be helpful. Thanks, Brandon Doctor: Hello and Welcome to \u2018Ask A Doctor\u2019 service. I have reviewed your query and here is my advice. I think the incident happened only once. It may not happen again. You must have been stressed, may be having some deep seated troubles which help you up from starting a conversation. I suggest you not to worry much. this will be settled next time and you can now settle it through long distance video calls.Hope I have answered your query. Let me know if I can assist you further. Regards, Dr. K. V. Anand"
},
{
"id": 113907,
"tgt": "Does sexual inactivity cause lower back pain ?",
"src": "Patient: Does being sexually inactive cause back pain? Doctor: hi, welcome to health care magic forum, no not at all, who made u think this...exercise daily, keep your sitting posture up right.. it will help you in reliving your lower back pain.."
},
{
"id": 152902,
"tgt": "Suggest treatment for recurrent pituitary tumor",
"src": "Patient: my brother has a recurrent pitutary tumour which has now protruded to teh right eye orbit and teh lower portion of the eye has come out of the eye lids , He has removed the tumour in2008 but has grown after that , need a suitable doctor who can treat him Doctor: Hi,Thanks for writing in.It is probable that your brother has a recurrent pituitary tumor which has grown after surgery. It is 7 years after his surgery and therefore the tumor has been growing steadily to cause his eye to get protruded. It is important to see his pre operative MRI scan and and interval scans to know the extent of the tumor and the pressure it is causing on the adjacent structures. If the tumor has grown slowly in few years then it is possible to assess the condition and suggest a surgery.You should consult doctors in head and neck oncology and a neurosurgeon at a specialist cancer institute in your location. They will do imaging evaluation and know which approach of treatment is better for your brother. There might be a requirement of radiation treatment, chemotherapy and surgery which needs to be carefully planned. Many doctors can handle such cases and it is important to get urgent consultation because if the optic nerve gets damaged then vision will get affected. Please do not worry."
},
{
"id": 77180,
"tgt": "What does spot/hole in lung suggest?",
"src": "Patient: I have a CT scan they see Spot & a Hole on my lung but I had the spot for a long long time, I dont smoke, dont drink, my symptoms is sometimes I have shortness of breath, this is the only symptoms I have. What is that mean tomorrow I have to see a specialist. I am really scared Doctor: Thanks for your question on Healthcare Magic. I can understand your concern. Spot or hole on chest x ray can be due to 1. Benign cyst 2. Infective cavity like tuberculosis 3. Benign nodule 4. Malignant mass Possibility of benign lesion like (cyst or nodule) is more because you are non smoker and lesion is stable over years. But better to get done CT thorax to confirm the etiology. Also get done PFT (pulmonary function test) to search for the cause for your breathlessness. So when you see lung specialist, you should discuss about need of CT scan and PFT. Hope I have solved your query. I will be happy to help you further. Wish you good health. Thanks."
},
{
"id": 154030,
"tgt": "Is hysterectomy,right treatment for cervical cancer?",
"src": "Patient: i am 23 years old i was diagnoised with cerviacl cancer at 16 can i recommend a hysterctomy to my doctor? i am over the pelvic pain the feeling full after two bites not being able to eat, and the heavy bleeding every cycle i have. Just in general its taking over my life and daily functions. Doctor: Hi, dearI have gone through your question. I can understand your concern. You may have cervical cancer. But at age of 16 cervical cancer is very unlikely. So you should review your reports. If needed go four quadrant cervical biopsy. It will give you exact diagnosis. Then you should take treatment accordingly. Hope I have answered your question, if you have doubt then I will be happy to answer. Thanks for using health care magic. Wish you a very good health."
},
{
"id": 166796,
"tgt": "Does stomach acid cause hard bloated stomach and pains?",
"src": "Patient: Hi, may I answer your health queries right now ? Please type your query here... My daughter has a very swollen stomach and is hard she also complains of stomach pains. She is ten years old. She has been complaining of stomach pain for some weeks now. Doctor said she has stomach acid. But he didn t say why. Doctor: Hi,Whatever you are describing is called Chronic Pain Abdomen in medical terms. The differential diagnosis of abdominal pain in children varies with age, gender, genetic predisposition, nutritional exposure and many environmental factors. The causes are many including:- constipation / acid peptic disorders / inflammatory bowel disorders / irritable bowel syndrome / worm infestation. I suggest you see your pediatrician or a pediatric gastroenterologist for this.Hope my answer was helpful for you. I am happy to help any time. If you do not have any clarifications, you can close the discussion and rate the answer. Wish your kid good health.Hope I have answered your query. Let me know if I can assist you further. Regards,Dr. Sumanth MBBS., DCH., DNB (Paed)."
},
{
"id": 173766,
"tgt": "How treat gall bladder stone in a 8 years old child?",
"src": "Patient: sir my child suffering in gall bladder stone since 2 years ,it found in 15 days back after diegonised the size of the stone is 7mmx6mm and 6mmto4mm in gall bladder,so many dr adivace to take a remedies not foe surgery plz advice me he is 8 old son. Doctor: Hi,Thank you for asking question on health care magic.Oral Ursodeoxycholicacid for period of 2 years may dissolve the gall stones.But the may recur after stopping the treatment.Obstruction of bile ducts by gall stones may be relieved by ERS following ERCP (endoscopic proceedure) or Lithotrpsy to break stones into small particles so that they will be excreted in feces.Hope this answer will serve your purposePlease feel free to ask any more queries if requiredTake careDr.M.V.Subrahmanyam MD;DCHAssociate professor of pediatrics"
},
{
"id": 133204,
"tgt": "Need medicine for pain in bilateral thighs",
"src": "Patient: Have popped bilateral small bicep tendon doing noting at all recently. Then pain in bilateral thighs front. Again just started with no insighting event weakness. No cramping unless I am lifting or trying to lift 53 year old male no known muscular issues or orthopedic issues Doctor: hi,thank-you for providing the brief history of you.As per the history it looks a Neuromuscular reaction. Since your age is 53, it is expected to have some degenerative changes in the spine and the nerve may get pinch leading to abnormal symptoms.Having a thorough clinical examination is advised. Post which medication will be advised for pain relief and also doing exercises in the later stages should help further.Regards Jay Indravadan Patel"
},
{
"id": 166678,
"tgt": "What causes on and off blurry vision in children?",
"src": "Patient: My 7 year old daughter woke up saying things were blurry. Had her use warm washcloth in case any overnight goo was causing the blurriness. She seemed to be able to see OK up close, said things were a little blurry far away. She has complained of this a couple of times and then the blurriness goes away. I am going to make a doctor s appointment today but wondered if this is common (the on/off blurriness) or if this could be a sign of needing to have corrective lenses. Doctor: Hi,This could be due to an error of refraction. Errors of refraction may cause blurred vision when exhausted or when reading or on concentration for long time.Hope I have answered your query. Let me know if I can assist you further. Regards,Dr. Salah Saad Shoman"
},
{
"id": 189687,
"tgt": "Have severe tooth pain, swollen cheek, upset stomach and weakness. Abscessed tooth?",
"src": "Patient: Hi. I ve been having intense pain in my tooth . My cheek is swollen, I haven t slept in 2 nights, upset stomach , and I feel weak. I have no insurance at all, as far as dental goes. Debating on going to the ER tomorrow morning, beccause at this point I m in tears I hurt so bad. Is this an abscessed tooth? And if so can they help me until I can find an afordable dentist? Thank you. Doctor: Hi, Thanks for asking the query, Pain in the tooth and cheek swelling indicates severe tooth infection and abscess formation . there is spread of infection to the adjoining periodontal tissues leading to periodontal abscess . If not traeted in time it may lead to further spread of infection to the adjacent tissues . I would advice you to get a complete clinical evaluation done . Take an x-ray . At home take saline water gargles 3-4 times a day. Use 0.2% of chlorhexidine mouthwash gargles twice daily. Take tab flemiclav625 mg TDS , tab flagyl 400 mg TDS , tab dolonex TDS. Hope this helps out, Regards..."
},
{
"id": 139235,
"tgt": "How to reduce swelling in shin after been hit?",
"src": "Patient: i banged my shin on a coffee table over the weekend fairly hard, and didnt think much of it. the next day it was significantly swollen, but it didnt impair any mobility and i couldnt make the pain worse by flexing anthing or pushing on the bone or anything like that. the rest of the day i just took it easy, and iced when i wasnt busy, but it seemed to only get worse over time. today i went to work, and it felt about the same. i feel no pain in it, other than the pressure of the swelling. today after work, ive been keeping it elevated and icing it, but the swelling is just as bad or worse than it was a day ago. is this something i should be worried about or will it just take a few days to calm down? Doctor: Hello,I have studied your case. I think you have developed haematoma in the leg. It is collection of the blood due to injury and it is causing you trouble. I have following suggestions for you.1.Hot water fomentation2.Antibiotic daily.3.Elastocrape bandage 4.Bed rest and do not walk for long duration I hope these steps will help you.Let me know if there is any other followup questions.thanks"
},
{
"id": 174319,
"tgt": "How can one confirm frenulum breve in a child?",
"src": "Patient: Our 4 year old, but very tall grandson spends a lot of time grabbing his crotch. There is no rash or other sign of irritation. How would I check for frenulum breve? Is that a logical cause? Thanks, but we will try another source which we are already paying for. Another time perhaps. Doctor: It is clinically diagnosis.The diagnosis of frenulum breve is almost always confused with that of phimosis and a generally tight foreskin, since the symptom is difficulty retracting the foreskin. Most men with phimosis also have frenulum breve to a certain extent."
},
{
"id": 196189,
"tgt": "Will masturbation affect my married life?",
"src": "Patient: Dear Sir, I am doing handpractice on not daily basis but do twice a week, what will effect after marriage . I do this with my wife who is living in other city. What should I do for good health. My age is 28 year and Weight only 57 Kg. height is 5'10\". please tell me a proper guidance. Thanks & regards Suresh Doctor: doing masturbation twice a week is not a problem.it wont affect your sex life.dont be afraid.take healthy foods.avoid alcohol and smoking.have sound sleep.these factors helps to increase your sex drive."
},
{
"id": 187198,
"tgt": "Suggest treatment for Abscessed tooth",
"src": "Patient: I have an abscess tooth it is broken below my gum line and I am having a tingly sensation in my scalp, and stiffness in my neck. Two days ago it felt as of my brain were on fire. I am going to the dentist in February but I need to know if this is an emergency room trip situation and what will they do. My teeth are in horrible shape. 2 broken ones in the front and almost all of my molars are broken. I don't know if it is needed information but the hospital I will go to is a naval hospital. I know eh do things differently there. Doctor: hello thanks for the consult at hcm..u might be having a periapical r periodontal abscess,, u could go for incision and drainage at ur nearest dental office,, hp u have taken a course of antibiotics and analgesics..do salt water gargle thrice daily..u need a full mouth rehabilitation since most of the teeth r damaged, plz consult ur dentist asap if pain doesnt subside..hope t helps.tc..."
},
{
"id": 46694,
"tgt": "What does DTPA renal scan report Total GFR- 118.6 ml/min indicate",
"src": "Patient: Dr. i want to ask the result of my report of DTPA Renal Scan whose repot is as follows:-Total GFR- 118.6 ml/min. Rt. kidney:-75.97 ml/min. Lt. kidney:- 42.62 ml/min.Split Function: Rt. kidney:64.05% Lt. kidney: 35.95%Are these all nornal or not ?????? please sugest somthing if not...............!!!!!! Doctor: Overall your kidney function is normal. However Lt kidney function is little lower. It could be due to recurrent urine infections in the past or could be due to high BP damaging Lt kidney. But don't worry. Avoid pain killers,non vegetarian diet. All the best. If you have any questions feel free to contact me."
},
{
"id": 104418,
"tgt": "Itching skin, sore throat, coughing, breathlessness. Taking Rablet and Xaria",
"src": "Patient: I have a allergy of iching skin which gradually multilpies and create problems like sore throat and coughing and breatlessness. I have been taking Rablet 20 and Xaria tablets suggested by a ENT specialist for the last 4 months. But as soon as i stop it goes on multiplying again. Plz suggest a medication so that i can get rid of it. My email ID YYYY@YYYY Doctor: This is food allergy it can be caused by anything at any age and any time of life even if you are using the food for many years Most likely to be milk but can be potato,chana,rice,egg,nuts Get blood serum tested for milk and other foods i described above Mean time stop all milk and diary no use of mustard coconut walnut groundnut and soya use olive for cooking and application on skin Take Fexofenadine 120 mg once a day for 3 week till you get answer from the tests"
},
{
"id": 168629,
"tgt": "Suggest treatment for fever and cough",
"src": "Patient: Hi, I am glad for this service of consultation you are providing. My 18 months old son has been with feeling regular the past 4 days with not very high fever of 99 - 100 degrees. Today he started coughing as a barking old dog that sounds very scary. His coughing is getting more and more frequent during the night and sounds that he is having problems to breath. What should I do? Thank you so much for your help. Doctor: Hi,It seems that your son might be having viral infection producing productive cough and fever.Possibility of having bacterial infection is also there.In this situation one course of antibiotic medicine is advisable as there are chances of pneumonitis is there.Consult your doctor and get examined.Ok and take care."
},
{
"id": 160194,
"tgt": "Is this throat cancer?",
"src": "Patient: I have dry mouth from taking WellbutrinXL. I have had sores in my mouth on the right side for the past 10 weeks. I have never smoked or drank alcohol. I had this in November too but 10 days of Keflex cleared it up. I took Keflex again 6 weeks ago and the condition improved but didn t go away. Is it throat cancer? I m a 60 year old female. (My father died at age 58 from toncil cancer but he smoked heavily) Thank you! Tricia Doctor: Hi,Tricia,wellbutrin is known to cause dry mouth as you are aware.Multiple sores in mouth repeatably might be due to some nutritional deficiency.Keflex is an antibiotic which can help only in case of infection.It will not work in absence of infection.Please ge in tough with your doctor from regular vitamin supplementation which will take care of this current problem.From symptoms it does not seems to be cancer,but still after examination your doctor can throw more light on it.bye and take care."
},
{
"id": 155,
"tgt": "Does taking medication for TB cause difficulty in conception?",
"src": "Patient: I'm trying for a child and after the hysteroscopy and endometrial biopsy the result of TB PCR test was non Tb Micobactiria -detected and TB Micobactirial Infection as negative . Doctor adviced me to take medicine as if I have TB .Will there be a chance for me to concieve after treatment .Since I'm 43 years .I have to try with donor EGG .Can I get a baby after treatment Doctor: Hello,I would like to know why your doctor prescribed ATT, the anti-tuberculosis treatment, when you were diagnosed with non-tuberculous infection. I presume it was done as a safety precaution. You have to wait until the treatment is complete before you plan conception. If the endometrial tissue repair is good, your chances of successful implantation are increased. Though your ovarian reserve could be low considering your age, with ART, the assisted reproductive techniques, your chances are good.Hope I have answered your query. Let me know if I can assist you further.Regards,Dr. Shanti Vennam"
},
{
"id": 133609,
"tgt": "What causes sharp pain in rib cage?",
"src": "Patient: Hello I am a 32 year old male and about six months ago I noticed a sharp pain on my lower left rib cage when I laugh hard or breath heavy. Two weeks ago I noticed a fatty lump there and it was very sore but I could move it around about 1 any direction now the lump is harder but still moves and I m concerned as to what it may be Doctor: HIThe lump sounds like a lipoma--a benign fatty tumorIf it bothers you it can be surgically removed, usually under local anesthesia"
},
{
"id": 117915,
"tgt": "How to treat swollen lymph node in groin and neck?",
"src": "Patient: Hi my 16 year old daughter has 4 swollen lymph nodes in her left groin area and 1 in the right side of her neck. The largest one in her groin measured 2.3 by 2.2 by 1.8 about a week ago, but feels bigger now. She has had a10 day course of abt, but she has not been sick and they are growing and now she has one in her neck. CBC was normal, should I be concerned its been almost 4 weeks Doctor: yes generaly it it due to reactive lymphoid hyperplasia. it occurs due to infection in the draning area. but if it is not disappear after a course of antibiotics then definitly a subject of concer. you should consult phisician and if needed go for Fnac study of lymphnode. it will clear all doubts in most of the cases and if required go for biopsy. dont delay it can be anything."
},
{
"id": 215163,
"tgt": "Suggest treatment for hand and wrist pain",
"src": "Patient: hi my name is Antonio and about to years ago i got into a fight where i punch a guys head and after a short period of time my hand got numb and was hurt very much and got swollen, i decided not to go to the hospital to get it check out, today almost 3 years later on and off i feel pain in my wrist, for the past 2 weeks it been getting worse,it hurts to hyper extend my wrist and also to flex my wrist, also it hurt to turn my hand from a palm towards the floor to the right, what can it be? Doctor: Hello,I read carefully your query and understand your concern. Wrist pain is often caused by sprains or fractures from sudden injuries.I suggest to so an xray of the wrist for further evaluation. Meanwhile,I suggest using anti inflammatory medications such as Acetaminophen to relieve the pain. I also suggest to maintain a splint to maintain the wrist immobilized. Hope my answer was helpful.If you have further queries feel free to contact me again.Kind regards! Dr.Dorina Gurabardhi General &Family Physician"
},
{
"id": 84659,
"tgt": "What are the side effects of taking orofer Xt?",
"src": "Patient: Hi i am 28 years old and i feel like i should take orofer XT just to increase level of heamoglobin, cos last month my periods delayed. is it ok? if i take this tablet? or are there any side effect? I am good in health. not weak not fat i am just perfect. Doctor: Hi,Incomplete data like apart from delayed periods do you have any other symptoms or have you done any blood tests to confirm anemia? Orofer XT Tablet is commonly prescribed for the treatment and prevention of anemia associated with deficiencies of iron and folic acid. Its common side effects include metallic taste, nausea, abdominal pain, constipation and black colored stools. If your hemoglobin is low or if your delayed period is due to an early pregnancy, this medication may be taken.Hope I have answered your query. Let me know if I can assist you further. Regards, Dr. Mohammed Taher Ali, General & Family Physician"
},
{
"id": 108394,
"tgt": "What causes pain in lower back?",
"src": "Patient: Hi, I am a 25 year old male with Gastro oesophagal reflux, and have a beer or two drinks of whiskey on the rocks everyday. I also smoke 10-15 cigarettes a day. I have sharp pain in my lower back - somewhere between the last two ribs, which eventually also appears in my left shoulder. I have had this two times in the last 3 days during the night. Everything is fine when I wake up in the morning. Is it a gastric problem or something serious? the pain is quite sharp and increses when I breathe or move. Doctor: Hithank you for trusting HCMI read your query and understand your concernIt seems that your symptoms aren't related to your gastrointestinal problemsIn my opinion you are suffering from fibromyalgia, nothing to be worried to muchYou can discuss about my opinion with your doctors and use pain killer to avoid painTry analgesics as NSAID can worse refluxWish you good healthRegards"
},
{
"id": 170466,
"tgt": "How should internal injuries be checked due to a fall?",
"src": "Patient: My son fell off a bed that is about 4 ft high he is 9 months. Few days later ei saw blood in his crib and I took him in to the hospital. They checkedhim and said the blood was a mystrey because they didnt see trace of blood in his nose. He doesnt have any cuts or scrapes or bruising. But im worried he might have a hematoma or broken nose? How might I know should I bre worried if they dont look intrrnaly? Doctor: HiThanks for writing to us. A nasal endoscopy can sometimes identify bleeding spot.If he is otherwise active and cheerful with no abnormal behaviour don't worry.He should be fine.Xray nasal bone can.identify any minor cracking of nasal bone.Regards"
},
{
"id": 3329,
"tgt": "Can unprotected sex cause pregnancy?",
"src": "Patient: I had my last period July 7 . My husband and I had unprotected sex July 19 and I took Postinor 2 within 24 hrs. Had unprotected sex again July 25 and August 1. My period was due August 4 but as at August 10 my period has not come. Could I be pregnant? Doctor: Hello dearI understand your concernYour delayed period could be due to side effect of the postinor 2 or by pregnancy.Pregnancy is possible as you did sex on the 25th august without any protection.Postinor 2 pill cause delay in period by 10-14 days.If your period will delay by more than 14 days from the expected period date then go for urine pregnancy test and or blood HCG to confirm the pregnancy.If you will be pregnant then still you can terminate the intrauterine pregnancy by Abortion pill under the advice of gynecologist.Meanwhile avoid stress, take healthy diet, drink plenty of water and do regular exercise.Hope this may help youContact HCM for further health queryBest regardsDr.Sagar"
},
{
"id": 197795,
"tgt": "Why does flotral taken for enlarged prostate cause mouth dryness?",
"src": "Patient: I am a male , age 71, and have been taking flotral 10mg since 3 years. I take it at night after meal. The mouth is dry when I wake up. What is the best time to take it and is there any other medicine to reduce enlarged prostate? My PSA is 1.8Thank you for the immediate reply. Doctor: Thank you for being with healthcare magic! Flotral or alfusozin doesnt cause mouth dryness. If you are snoring when you sleep then that is the cause of your mouth dryness when you wake up. If you have a prostate size more than 40grams then you can take a medicine to reduce the prostate size (finasteride/dutasteride). I hope I have succeeded in providing the information you were looking for. Please feel free to write back to me for any further clarifications at: http://www.HealthcareMagic.com/doctors/dr-manuel-c-see-iv/66014 I would gladly help you. Best wishes."
},
{
"id": 217145,
"tgt": "Does physical therapy help in relieving the severe back pain?",
"src": "Patient: Denzel, a 46 year old man, is working with his physical therapist on a set of back exercises. Around four weeks ago, Denzel was helping a friend move and attempted to lift a heavy box. Since that incident, he has had severe pain in his lower back. His health care provider prescribed physical therapy three times per week to treat the condition. Doctor: Severe back pain need rest and medical treatment in the form of painkillers. After controlling severe pain slowly start physiotherapy. First you consult neuro surgeon and take opinion then do accordingly. You may have disc prolapse. There are so many type's of treatment modalities. Drug therapy. Epidural steroid injection. Surgical intervention. When you are having severe back pain, and if you try to lift the heavy objects with one side it will further exaggerate disc prolapse. Kindly avoid those work."
},
{
"id": 159102,
"tgt": "Breast tumor, painful. Did i suppose to go into surgery to find out if it cancerous or beginning?",
"src": "Patient: A female friend of mine 23-25 years old and she went to the doctor complaining of pain in both breasts and the doctor said to take an ultra mammogram , and found out there is a marble sized tumor . They gave her pain medication but it has not been working. Is she supposed to go into surgery to find out if it cancerous or benign? How long can she wait? Is cancer painful? And some information her mother and grandmother had colon cancer. She is about 110 or 115, and 5 3 or 5 2. Doctor: Hi, I can understand your concern. Generally breast cancer are not painful to start with. Infections are commonly painful. Better investigation will be bilateral mammogram of breast. Mammogram can predict the nature of the lesion. If any tumor found, then better to go for a core needle biopsy. Consult your surgeon."
},
{
"id": 60961,
"tgt": "What do lumps on the forehead indicate?",
"src": "Patient: I keep having these lumps under my skin on my forehead about the size of a quarter and it's almost like something has bitten me and it became swollen but nothing has bitten me. It started a few days ago. I have back pain really bad and I am very tiredr Doctor: Hello,The most possible issue for lumps on the forehead is benign soft tissue lesions as multiple neurofibroma or others. In our clinic, we recommend a fine needle aspiration cytology report of one of the lumps for pathological diagnosis and further management.Hope I have answered your query. Let me know if I can assist you further.Regards, Dr. Bhagyesh V. Patel"
},
{
"id": 9242,
"tgt": "Suggest cure for cracks in the leg",
"src": "Patient: my feet are badly cracked and splitting open. i have used cortisone creams, and i slather moisturizing creams on them continually. i always wear shoes and socks to bed. as soon as one heals a bit another split appears. i have even seen a podiatrist who couldn't help. i also gently pumice them but nothing works. i am on my feet all day at work and its really painful Doctor: Hi,It seems that most probably you may be having palmoplanter keratoderma.The skin of palms and soles become dry,hyperkeratotic,thick and fissured. You may feel discomfort during walking and working. You avoid frequent washing with water. Keep the affected area dry. Apply moisturising cream containing urea in morning after bath and rub salicylic acid with steroid oint at night on affected parts. Take vitamin E 400 mg daily for long time. You might improve after few weeks treatment.I hope you got my answer.Thanks.Dr. Ilyas Patel MD"
},
{
"id": 122509,
"tgt": "What causes pain and twisted veins on neck?",
"src": "Patient: My son has often awoke with pain in his neck. As I inspect further I feel twisted veins on the side of his neck 3 fingertips diagonal towards his chin from his earlobe. This twistedness seems to vary rather frequently and I can t seem to get a good measurement of the bumps and twists as they are many and appear to move. What is this? What should I do for him? Doctor: Hello, As first-line management, you can take analgesics like paracetamol or aceclofenac for pain relief. If symptoms persist, it is better to consult a physician and get evaluated. Hope I have answered your query. Let me know if I can assist you further. Regards, Dr. Shinas Hussain, General & Family Physician"
},
{
"id": 155554,
"tgt": "Is radiation chemotherapy advisable for Squamous Cell Cancer stage 4?",
"src": "Patient: My husband was diagnosed with Squamous Cell Cancerous Tumor in his left sinus cavity. Stage 4 we are scheduled for surgery Wednesday. It has all happened very fast. His PET scan came back clean. What do you recommend for him. We know that they want him to do radiation and chemo after the surgery. Doctor: Thanks for your question on HCM. In my opinion if PET SCAN is normal for distal metastases than you should get done enblock resection of sinus cancer.And post operative chemotherapy and radiotherapy are given to decrease the chances of local site recurrence .Chemotherapy will also useful in prevention of distal metastases. So better to take post operative chemotherapy and radiotherapy for better prognosis and life expectancy."
},
{
"id": 6112,
"tgt": "Trying to conceive. Husband takes morphine for pain, I'm hypothyroid. Any side effects of morphine ?",
"src": "Patient: My husband takes morphine for pain and we are. Trying for a second baby our first took 3 years and 1 miscarriage and we have been trying 3 and a half years so far. We had tests done and got all clear other then I have hypothyroidism which is being treated I m wondering what my husband can take ordo to counteract the effects of the morphine Doctor: Hi Sharon, Thanks for posting your query. You have had a healthy baby earlier. I do not think it is very difficult for you to conceive again. Continue your treatment for Hypothyroidism as it is very important for you to become pregnant. Morphine per say does not cause infertility. It may cause a decrease in the sex drive. Please repeat his semen analysis. Hope this helps. All the best."
},
{
"id": 48124,
"tgt": "Suggest medications for difficulty in urinating in cildren",
"src": "Patient: My granddaughter is 6 years old and cannot urinate. She has been to the doctor twice this week. He said she does not have kidney or any kind of infection, however she has been to the bathroom three times in the last hour, she is sitting here now crying. I let her sit in a warm bath for 30 minutes. Don t know what to do? Doctor: In my opinion she is having some urinary infection and needs urine test and ultrasound done.I advice my patients plenty of fluids painkiller in syrup forms and some antibiotics."
},
{
"id": 222637,
"tgt": "Suggest tests to confirm pregnancy",
"src": "Patient: We engage only in foreplay but it was my first time and I was afraid so I took the unwanted 72 pill on 8-01-2011 (within 6 hours). Today morning again we did the same thing but we were careful not to bring in contact his penis to my vagina. I think he had some pre-cum and we touched the parts with clothes on. Considering I am still a virgin are there any chances of me being pregnant because of this? Shall I take another dose today to make sure I dont get pregnant? Doctor: Hi dear, I have gone through your question and understand your concerns.Pregnancy can occur only when there is entry of semen in the vagina.If there was no direct contact of the penis or semen with the vagina, then there is nothing to worry about. You donot need to take any emergency contraception now.Hope you found the answer helpful.Wishing you good health.Dr Deepti Verma"
},
{
"id": 20717,
"tgt": "Suggest treatment for stress related chest pain",
"src": "Patient: Hi I m under a lot of stress at work while there the last few days I m expericing mild pain in the middle of my chest . I am prescribed Ativan and when I take one the pain subsides but when it wears off the pain is back can it just be stress related Doctor: Hello!Welcome on HCM!Your symptoms seem to be related to anxiety. Coming to this point, I would recommend consulting with a specialist of the field (psychotherapist or psychiatrist) to help you manage anxiety. An antidepressant would be better than Ativan as long term therapy for the management of anxiety. I would also recommend trying Omeprazole. If it helps, it would be indicative of gastro-esophageal reflux or gastritis underlying your symptoms. In such case a fibrogastroscopy would be necessary. Hope to have been helpful!Kind regards, Dr. Iliri"
},
{
"id": 162803,
"tgt": "What causes rashes on the chest of an infant?",
"src": "Patient: My grand baby is a year old. Last night her dad noticed she had some rash on her chest, then today it spread to her head. Shes not fussy, shes playing and being her happy as usual self. Shes been teething these last few days so she had been running a slight fever. How long do we wait before we panic. Doctor: Hello and Welcome to \u2018Ask A Doctor\u2019 service. I have reviewed your query and here is my advice. Skin conditions are best diagnosed only after seeing directly. I suggest you to upload photographs of the same on this website, so that I can guide you scientifically. Please revert back to me with images so that I can guide you better. You can approach me at the following link. Once the page opens there will be an option below my image as \u2013 ASK ME A QUESTION \u2013 click on it. Please find the link below - www.healthcaremagic.com/doctors/dr-sumanth-amperayani/67696 Regards."
},
{
"id": 169167,
"tgt": "What causes painful nipples after swimming?",
"src": "Patient: After swimming in the pool, my grandson complained about sore painful nipples and the tips of his nipples were very red. Two days later the second grandson was swimming in the pool, and later complained about the same problem,. what could be causing this weird conditions. Granddaugjters also swam and had no symtoms at all? Doctor: HiWelcome to the HCMI understand your concerns. It seems to be due to allergic reactions on mucosal surface of nipples due to some allergens in the pool. You may give antiallergic medicines for relief.Take care"
},
{
"id": 122402,
"tgt": "What is the best treatment for a compression fracture?",
"src": "Patient: My Dad just found out he has a Compression Fracture after falling out of a Hospital Bed after being admitted. He is going to be fitted for a Brace on Friday. He his slightly overweight, will he eventually require surgery or does the Brace usually correct the issue ? Doctor: Hello, Compression fractures usually don't require surgery. The pain usually improves significantly in a matter of days or weeks. Pain management may include analgesic pain medicines, bed rest, back bracing. Physical activity is recommended for recuperation. Hope my answer was helpful. If you have further queries feel free to contact me again. Regards, Dr. Dorina Gurabardhi General & Family Physician"
},
{
"id": 127084,
"tgt": "What causes pain and burning sensation in the upper back and shoulder?",
"src": "Patient: Hi for the past 9 months I ve been having random symptoms daily from upper back and shoulder pain to neck pain to a burning sensation in chest to a weird pain in chest followed by a sweat or getting really hot I ve had ecg and bloods and they keep saying I m fine I m not fine and the chest pains almost daily are really starting to bring me down they hurt they scare me I can t keep up like this Doctor: Hi, How old are you ? Male or female ? And do these symptoms appear during exertion ? I would recommend a treadmill test to check the endurance capacity of your heart as resting ECG can be normal. If the treadmill test is normal then you need not worry about it being cardiac. Can be early neck fatigue. Hope I have answered your query. Let me know if I can assist you further."
},
{
"id": 131261,
"tgt": "Suggest remedy for recurred pain due to fibromyalgia with no help from any medications",
"src": "Patient: I have a question. I am a 47-year old mother with 3 children who has suffered from fibromyalgia for over 15 years. You name a medication, I have tried it: Tylenol, Oxycodone, Advil, Oxycontin, Lyrica, (along with other anti-depressants and anti-convulsives). I am currently taking 250 mg of Depakote at night for pain relief and manic-depression. I also take 1 mg of Klonopin at bedtime for sleep and muscle pain. I am currently on Methadone, but coming off of it...however, I have noticed that the pain is returning with a vengeance. The only medication that has worked well for me (w/o putting me to sleep is the Duragesic patch). The problem is that I am on Medicaid and am having a very hard time finding a pain doctor in Colorado Springs. I have spoken to my family doctor about switching to the Fentanyl and he thinks it s a good idea, but does not feel pain management is his specialty. I simply cannot find a doctor in my area. Ideas? Thanks, Heidi Doctor: In my opinion you should explore 2 possibilities 1-kidney chronic failure 2-if it is stress related check kidney function first Good Luck"
},
{
"id": 58337,
"tgt": "High ALT and AST levels. Done blood test. Cause?",
"src": "Patient: ALT and AST raising from 95 to 160 to 260 in 3 months I had the flu in late October 2012, and one evening I suddenly fainted and passed out for about 30 seconds at work but regained conscious without help, and my blood pressure and blood sugar levels were normal according to emergency medical staff. Had my first bloody test a few days later, and my ALT/AST levels were 95 and everything else were normal. A new bloody test was done in mid December 2012 - everything was normal but the ALT and AST raised to 160. Tests for Hepatitis A B C were all negative. In early January, I had another bloody sample taken and also had an ultrasound examination of my abdominal area and nothing seemed to be wrong with my liver, gallbladder , kidney etc. The third blood test results showed today that my ALT and AST raised to 260. I still do not have any symptons such as pain or yellowing in the eyes. I am not on any medication (except some Paracetamol when had the cold, with Phenylephrine Hydrichoride for blocked nose - contained in Lemsip). I am 24, 6ft and 70kg. Generally very healthy. The doctor seems to be puzzled and will refer me to a Hematologist . Has anyone else experienced similar problems? What could be the cause of the slow but steady increase of my ALT AST? PLEASE HELP! Many thanks. Doctor: Hello there,Sometimes viral flu can also result in an elevation of the AST & ALT. The hepatitis viral markers were already negative as you have conveyed. I suggest you get the Liver function tests repeated 3-4 weeks after the flu has resolved. if the AST & ALT are still elevated at that point of time I recommend doing a lipid profile, iron and copper studies and then if indicated after seeing the above reports a liver biopsy may be indicated.Hope I have answered your query you get well soon. I will be available in case of any further queries.Regards,Dr. Prashantha S. Rao"
},
{
"id": 198788,
"tgt": "Suggest remedy for premature ejaculation",
"src": "Patient: hello dr.i m mr.rites from uganda,i have problem of early ejeculation since i started sex,thats why i m became gay ,i like to play sex with men, and play with penis of any budy,i have no children, i want to became a perfect man,please help me Doctor: DearWe understand your concernsI went through your details. I am sorry to say that you misunderstood the whole thing. You did not become gay because you had premature ejaculation. You are a gay and therefore , you had premature ejaculation whenever you tried sex with women. Nothing matters here. If you are happy with gay sex, enjoy it and feel it is normal. It is normal, for sure unless you think otherwise or become anxious. You are already a perfect person with natural sexual urge (your naturalism is gay). Don't worry and accept it as it is. If you require more of my help in this aspect, please use this URL. http://goo.gl/aYW2pR. Make sure that you include every minute details possible. Hope this answers your query. Available for further clarifications.Good luck. Take care."
},
{
"id": 7049,
"tgt": "What are the chances for me to conceive with a blighted ovum ?",
"src": "Patient: Does my urine pregnancy test will become positive even if i have a blighted ovum? I m 5 3 in height 37years old and I am taking aldomet 250mg 2x a day for my hypertension . I had an ultrasound yesterday and the doctor told me that my sac is irregularly shaped gs my lmp June 17, 2011 anembryonic pregnancy was the diagnosis Doctor: hi,thanks for query.Yes this is possible.Urine pregnancy test detects HCG in urine which is produced in case of blighted ovum or real pregnancy.Ultrasound gives clear objective evidence of pregnancy.Please go as per advise from your doctor. wishing you good health."
},
{
"id": 133022,
"tgt": "Suggestion of cure for severe pain in legs",
"src": "Patient: I have pain in my legs, and sometimes in my arms, that I can only describe as an intense aching in the long bones. Sounds a little strange, doesn t it? The pain is quite bad though. I can kind of equate the type of feeling to what we used to describe as growing pains as a kid, only much more severe. Doctor: Hello, I have studied your case. There are many reasons of leg pain.Due to spinal disc bulge there can be compression of nerve supplying your leg leading to pain and difficulty in walking. You may need to do MRI spine to see for nerve compression.Due to compression of nerve there is leg pain with it.For these symptoms analgesic and neurotropic medication can be started.Till time, avoid lifting weights. You can consult physiotherapist for help.Physiotherapy like ultrasound and interferential therapy will give quick relief.I will advise to check your vit B12 and vit D3 level.Another differential can be leg muscle cramps.Take plenty of water, fluid intake will dilute toxins.Physiotherapy with ultrasound and Tens will help you,Take potassium rich diet.Hot and cold fomentation to legHope this answers your query. If you have additional questions or follow up queries then please do not hesitate in writing to us. I will be happy to answer your queries. Wishing you good health.Take care"
},
{
"id": 122,
"tgt": "Need advice on ways to conceive after miscarriage",
"src": "Patient: I'm a 33yrs female that's want to have a child. The doctor tested my husband Sprem say that he have a antibiotic in his Sprem I try IFv once after three months I have a miscarriage know I don't know what to do I want To have a child so bad I need to know what advance can you give me thx Doctor: Hello,I have gone through the query and understood your concern. If your husband has an infection, he needs to get the same treated completely and semenalysis is repeated after 72 days. Progressive motility and morphology are the important factors that determine the adequacy of sperm count. Your ovulation status also should be regular. Please do not worry, patience is very important when trying to conceive. Your doctor is going to go for further procedures once you are ready. Hope your query has been clarified. Take care."
},
{
"id": 176496,
"tgt": "What causes red spots on lower spine like a bruise?",
"src": "Patient: My 5 years old has 2 red spots on his lower spine that I first noticed when he was around 2. At that time they looked like he just leaned against a chair and the vertebras got irritated, very faint pink spots. The pediatrician said it was nothing to worry about. Now I see them again in the same area, but red like he bruised his back, but the spots are perfectly round... Doctor: i think its better if u get an mri spine done for ur child and get it looked by an orthopedician and if needed a neurophysician too"
},
{
"id": 209556,
"tgt": "What are the symptoms of a mental disorder?",
"src": "Patient: hello ,i am prateek patel from nagpur.I think before some days my mother has been suffered from mental disorder .She create a new think whole day specially at night at 12:00 to 4:00 which is not possible in reality. so, can u suggest what is the actual problem with my mother and also please suggest me the name of good doctor in nagpur. Doctor: HiThanks for using healthcare magicI think, she is in psychosis. Usually in psychosis, patients do irrelevant talk and do not sleep at night. In that case, she needs antipsychotic agent. Rest, the treatment would depend upon her mental status evaluation. Better to consult a psychiatrist near by your location. Who is good psychiatrist in Nagpur, that i can't tell you. You can search it online. In case, you need further help, you can ask.Thanks"
},
{
"id": 50466,
"tgt": "Have UTI, kidney stones. Done urinalysis, indicates protein cells, bacteria, etc. What is wrong?",
"src": "Patient: i've just had a urinalysis and It indicates the ff:protein; trace/ WBC 5-8hpfrbc: 0-1hpf / E cells : manyA. Urates: few / M. Threads: manybacteria: many Ph is 7.0 and Specific gravity is 1.020i know i have problems with my urinary tract and i've had a history of kidney stones.. Could my kidney be malfunctioning? i am worried about my pregnancy and i am scheduled for the doctor next week... Doctor: Hi,You have a simple urinary tract infection. As such urinary tract infections are more common in females because of a short urethra or urinary tract. Infection is increased more in pregnancy because of decreased hygiene and immunity. So, all you need is a good dose of antibiotics. Do not take any antibiotics on your own because you need to consult a doctor to get antibiotics which are safe for your baby. I hope this will help you. All the best.Dr Vineet"
},
{
"id": 225739,
"tgt": "No periods after finishing Minigyon. Missed few pills. Reason for delayed period?",
"src": "Patient: It is my understanding that upon finishing the pack of 21 Minigynon 30 tablets, that my period would start. I took the last pill on Friday and today (Sunday) I am still periodless and worried. I had forgotten it on a few days but took the pills as soon as i remembered. :( This is my first set of oral contraceptive , could my body just be responding to it strangely?? Doctor: Hi Dear,Birth control pills have to be taken in a prescribed and correct manner in order to produce desired results. Missing few pills ( more than 2) during cycle exposes you to the risk of pregnancy even if you have taken them afterwards. Moreover, for maximum efficacy the pill acts best from 2nd cycle. However, you had your last pill on Friday and you can wait for1 week for periods to start. Ali the best"
},
{
"id": 225027,
"tgt": "Can taking antibiotics reduce the affect of depo shot?",
"src": "Patient: I have a question. I'm on the birth control shot, Depo, February 25th will be my third shot. I just recently been put on an antibiotic, \"Doxycycline\" for a UTI. 500mg, twice a day. Question is, is it possible that antibiotics lesson the affect of Depo? I know they do with the pill, just not sure about the shot. I was told yes, did research and was told no, and called the ER since my OB isn't open, & they said its possible. & What are the chances of getting pregnant while on depo and taking antibiotics? So, does anyone have any information about my questions? Doctor: Hello!Thanks for your query.There are no known drug interactions between doxycycline and depo provera.If you are on injections for birth control,there is no reason why you can not take Doxycycline. Take care of your self"
},
{
"id": 56757,
"tgt": "Can hepatitis cause swollen liver?",
"src": "Patient: I have had heppatis 3 different times , 2nd grade 9th grade and tenth grade. my blood work came back as positive for hep. My doctor wants me to see a gastroenology for this and because my liver is swollen. I ask if i should come and get a gama shot and let people know. he said we werent going to do nothing until i see the other doctor. is this contagious? what do you think? Doctor: hello,Welcome to Healthcare Magic.Hepatitis is highly contagious by sexual and injection route.It is more dangerous than HIV.Please consult any gastroenterologist for ultrasonography and liver function tests.Please avoid fatty diet.Take fresh fruits and green veg more.Treatment is available so don't worry.Hope this will helpful.Thanks for contacting.take care..."
},
{
"id": 118386,
"tgt": "Can differentiation in the platelets and ITP be cured?",
"src": "Patient: My daughter has low blood platelets she just got it tested and they are at 80000 she has had this in the past where she was diagnosed a year ago with ITP can this be cured? She was put on predizone then her platelets went up to 150000 however they have been going down and now they are at 80000 which I believe they should be between 150000 to 300000 Doctor: HiThanks for your query.if she is otherwise fine with no bleeds, platelets of 80000 is okay. It can be observed over time. if it falls below 40000, we may increase the dose of prednisolone or add another drug.often ITP gets better over time and may be cured.Hope this helps.Regards"
},
{
"id": 148484,
"tgt": "Pain on the left side of the brain while inhaling & also when pressed. 'Motirn' is not working. Is it a tumour?",
"src": "Patient: I've been having a small but questionable pain when I inhale through my nose regularly I feel a little pressure and pain behind my left eye along the left side of my head and temple a very faint lingering headahe followsand is still there. I've takin motirn and nuthing wrks? Is this a tumor of some sort? Also when I push on the left side of my head near tthe temple. There a big pain. Doctor: HIThank for asking to HCMYou are having some kind of ideas, in fact this is not the case as you are thinking about, if you are taking force full breathing then it may causes the headache and some time may induce the black out also, so just stop taking the air with extra force, and just forget about the tumour and every thing, asking you for any investigation will coast you more money, every thing will be alright just relax, and enjoy the life have nice day."
},
{
"id": 14097,
"tgt": "Are skin rashes a sign of HIV?",
"src": "Patient: I have skin rashes in my arms and back also I had pharengytis few weeks back and it got cured after consulting with doctor and taking medicine bt still my rashes r there and also have seen smegmas. My doctor said he dont think any serious illness I have. I asked him about HIV. But he said NO . I am still scared. Please suggest and help. Doctor: Hello and Welcome to \u2018Ask A Doctor\u2019 service. I have reviewed your query and here is my advice.Skin rashes can be a manifestation of HIV infection. If you are having doubt, please do visit voluntary testing center for HIV infection and get your HIV testing done.Hope I have answered your query. Let me know if I can assist you further."
},
{
"id": 4355,
"tgt": "Am I likely to conceive after starting clome?",
"src": "Patient: My doc prescribed me to take Clome 100mg Day 3 to Day 7.Follicular study scan On 12th day (today) showed one follicle of size 9mm on right overy.So she suggested me to take a injections ( INJ IVF -M) for 2 days and asked to go for another scan on 16th day .is there a possibility for conception this cycle??? Doctor: HelloThanks for writing to us with your health concern.Normally, in a 28 day cycle, ovulation occurs on day 14.By this time, the mature egg follicle, which is ready to rupture, should measure around 18 - 20 mm.The egg follicle grows at a rate of 1 - 2 mm per day so by day 12, it should be around 12 mm or more.In your case, it is not of the optimum size on day 12.However, some women have delayed cycles or delayed growth of the egg follicle.In that case, the egg will grow at a normal pace, only the day would be late.Continue having follicular monitoring for a few more days.If the follicle size becomes optimum, then there would be scope for ovulation.Ovulation ( rupture of the egg follicle ) also is confirmed via the follicular ultrasound scan.Once ovulation is confirmed, please remain sexually active to maximize the chances of pregnancy.Take regular folic acid supplements.Also, if 3- 6 such cycles of stimulation fail inspite of good ovulation, then go for laparoscopy to check the patency of your fallopian tubes.All the bestTake care."
},
{
"id": 60932,
"tgt": "What does a lump in the armpit indicate?",
"src": "Patient: Hi I'm 55 yrs man with a bluish sort of lump just below my armpit it never hurts me but time and again around it gets filled with pus with bad smell i squeeze that get some relief but time and again same procedure what do u suggest me surgery or else hope u suggest a cure Doctor: Hello,Since how many years or months, you are having a lump in the armpit? Is the lump soft or hard? Most of the times soft lump is usually lipoma ( a benign fat tumor). Presence of pus in it usually indicates sebaceous cyst which got infected. I advise you to consult a general surgeon and go for simple excision of the lump along with a course of antibiotics. Hope I have answered your query. Let me know if I can assist you further. Regards,Dr. Siddartha"
},
{
"id": 79329,
"tgt": "What could a spot in lungs on chest x-ray be?",
"src": "Patient: I have pain in my ribs next to my breast and a little lower...I lifted a couch in May that I should not have and it has been sore off and on for months now...what do you think it is...I went in to see about it in May and I was sent to have a chest CT scan because they took an X-ray of my ribs to make sure they were not broken and found a spot on my lungs (which turned out to be a calcification)...what could it be? Doctor: calcification a in chest X-ray or a cat scan are due to healed disease in the past nothing to worry if there are one or two calcified opacities in the chest X-ray healing of any infection in. the lung occurs either by fibrosis or calcification , mostly tuberculosis heals by calcificationconsult a Pulmonologist and take symptomatic medicines as advised thanksfeel free to ask more questions"
},
{
"id": 199967,
"tgt": "What causes a lump and soreness on the half circumcised fore skin?",
"src": "Patient: hi im 19 years old and ive had circumsised when i was 11 or 12 but I don t think the doctor did a very good job. It seems that despite being cut, I still have a lot of foreskin leftover. Below the head of my penis, there s a good bunch of skin that I am actually able to pull over my pee hole without any pain or effort. Also, when I want to, I can almost cover my whole head with skin when I roll the skin upward. then theres a little yellowish like pimple appeared i dont remember when it is first appeard but i ignore it so now one day when ive wake up it feels itchy then i look at it its swelling red ,then theres a two hole leaking a pus in the front and back of the skin .how did it get like this and how can i cure it immediatly sorry for my grammar im not that good in english s Doctor: Hi,Thank you for posting your query in healthcaremagic.comThis is Dr. Swarnava dattagupta answering your query.I understand your concern.If you are sexually active it can be some sexually transmitted disease. If you are not, there might be some fungal or bacterial infections leading to sore and discharge.It is impossible to diagnose without examining it. So i would ask you to visit a DERMATOLOGIST for further management. Hope i have answered your question.If you have any further questions i will be happy to help.Regards"
},
{
"id": 211923,
"tgt": "Extreme physical symptoms. Diagnsoed with some conversion disorder. Suggested phychiatric treatment. Unable to get appointment with anyone. Help",
"src": "Patient: I am 42 years old and have been diagnosed with a unique type of conversion disorder. I have had a MAJOR stress event in my life and have been trying to find a counselor, psychologist or psychiatrist to see me. Everyone I have contacted do not have appointments until end of July. Those I have spoken to say I am not \"sick\" enough or I am to complicated for their level of expertise. I am have extreme physical symptoms and my neurologist states I need to get in with some ASAP. I do not know what to do. Doctor: hi Tina, welcome to healthcare magic. i would like to know more about your symptoms, and what kind/subtype of convertion disorder you have been diagnosed with.for this condition you have to understand whats the primary gain/ secondary gain involved with the symptoms. and you have to learn coping up skills and relaxation techniques. consult your psychiatrist as soon as you can"
},
{
"id": 180931,
"tgt": "What causes constant headache after tooth extraction?",
"src": "Patient: I had three teeth extracted on thursday. they have since healed up nicely but since friday evening I have had a constant, horrible headache (moving my head, bending over, etc... is excruciating). When I am able to sleep for a few hours I am covered in sweat. Doctor: Hi..Thanks for the query..Headache after tooth extraction can be due to radiating pain from extraction site..But if there is no pain at the extraction site then the headache can be unrelated to the dental extraction..You can take painkillers to relieve pain but in case if there is no improvement or recurring episodes of headache then consult an Emergency room and get evaluated..Hope this helps..Regards.."
},
{
"id": 127189,
"tgt": "How can pain in the forearm from excessive typing be managed?",
"src": "Patient: My 24 year old son has been experiencing forearm pain and hand pain for several months - probably due to excessive typing of his master s thesis coupled with tension. He has been out of school for 1 month and resting has not helped. We have also tried heat treatments and cold treatments. Doctor: Hello and Welcome to \u2018Ask A Doctor\u2019 service. I have reviewed your query and here is my advice. It could be due to excessive stress on your hand and fingers due to typing which can be aggravated by improper posture.First you need to get correct posture for typing followed by hand strengthening exercises with hand gripper.Also take diet rich in fruits and vegetables, protein rich foods like milk,eggs,fish etc. Hope I have answered your query. Let me know if I can assist you further."
},
{
"id": 74316,
"tgt": "What causes excessive mucus in vomit while having severe cough?",
"src": "Patient: Hi, I am very sick. I have not eaten all day. I have been throwing up and it comes out all flemmy and gets stuck in my throat sometimes making me unable to breath. I have a really bad cough and every time I caugh it makes me want to throw up and it is also very flemmy. I want to know what to do about this, do I need to see a doctor or even go to a hospital? Doctor: Respected user , HiThanks for using Healthcaremagic.comI have evaluated your query thoroughly .* This indicates moderate to heavy bronchial infection most likely .* Strongly recommended to consult a doctor and get evaluated with x-ray and necessary laboratory tests so that it can be managed in a more precise manner , rather than taking empiric treatment .Welcome for further assistance .Regards ."
},
{
"id": 198660,
"tgt": "How to treat right testicular swelling?",
"src": "Patient: I suffered swelling of the right testicle.Medication was administered intra-mascular and was good.But my partner did not get same treatment. I have suffered repeated/ relapsing episode of UTI with possible presence of infection in the testicle , epidydmis and sperm delivery tube/ duct. What is the right medication to cure this? This is long term condition. Doctor: HelloThanks for query.Based on the symptoms that you have, I would state that you have Chronic infection of the Epididymis and Testicles (Chronic Epididymo-Orchitis) .You need to consult qualified Surgeon for clinical assessment and get following basic tests done to establish diagnosis .1) Routine urine test and Urine culture 2) Ultrasound Scanning of Scrotum.Taking appropriate antibiotics as per culture report along with anti inflammatory medicines like Diclofenac and Serropeptidase twice daily will help to eradicate infection .ensure to drink more water to keep your urine dilute .Dr.Patil."
},
{
"id": 115876,
"tgt": "What does abnormal WBC count and Lymphocyte count suggest?",
"src": "Patient: HELLO DOCTOR,I RECENTLY HAD A WEELL WOMEN'S CHECK AND FOUND THE RESULTS TO BE NORMAL EXCEPT MY BLOOD TEST THE WBC COUNT WAS 4.8 AND THE LYMPHOCYTE COUNT TO BE 1.3. THEN THE PCP HAD ADVISED ME TO REPEAT THE TEST AND THE NEXT TIME THE WBC COUNT WAS3.7 AND MY LYMPHOCYTE COUNT WAS 1.2 WITH THE PLATELETS COUNT 127 WHICH WERE NORMAL FOR THE FIRST TIME. I HAVE NO TOHER SMPTOMS LIKE LETHARGY, WEAKNESS ETC... THE PCP HAS SUGGESTED ME TO GO TO THE HEMATOLOGIST. WHAT DO YOU SUGGEST.THANK YOU Doctor: Hi, dearI have gone through your question. I can understand your concern. You have slightly low wbc count. Your platelet count is also at low normal range. Most common cause is viral infection. If you have no symptoms then no need to worry. No need to consult haematologist. Just go for repeat blood test after a week. If you have abnormal results then consult your doctor and take treatment accordingly. Right now no treatment is required. Hope I have answered your question, if you have doubt then I will be happy to answer. Thanks for using health care magic. Wish you a very good health."
},
{
"id": 63838,
"tgt": "How can a lump causing an asymmetrical face be treated?",
"src": "Patient: One side of my face/neck (kind of under the jowl area) seems to be quite larger than the other. It shows up as a large lump in all photos, but it is soft and it doesn't hurt. It is only on the right side. Is there a chance it would be the thyroid, or something like Metastatic Squamous Neck Cancer? What does that look like when someone has a problem with that? I am not overweight or underweight. Doctor: Hi, how long that lump exists?. If its since childhood, cuctyic hygroma may be the cause.CT would fix its diagnosis with biopsy HPR.if it worries get treated. Hit thanks.Gd day."
},
{
"id": 142817,
"tgt": "What causes numbness in both the feet and hands with pain?",
"src": "Patient: I have had numbness in both feet up to my mid section and in both hand for almost 2 months. now it is really bad in my hands to the point that I have to look to see if I have something in my hands. also getting bad cramps in my hands & legs. I don t now know to explain everything that is going on, but in a lot of pain. I have had a cat scan of the spinal and brain. Spinal came back good but they found a white matter in the front part of the brain. do you have any idea what could be going on with me. Thanks Tracey 33 year old female Doctor: Hi, Welcome to HealthCareMagic.com I am Dr.J.Mariano Anto Bruno Mascarenhas. I have gone through your query with diligence and would like you to know that I am here to help you.This can beedemaorleukodystrophyyou need to consult a Neurosurgeon at the earliest Hope you found the answer helpful.If you need any clarification / have doubts / have additional questions / have follow up questions, then please do not hesitate in asking again. I will be happy to answer your questions. In the future, for continuity of care, I encourage you to contact me directly in HealthCareMagic at http://bit.ly/askdrbruno Best Wishes for Speedy Recovery Let me know if I can assist you further.Take care."
},
{
"id": 173145,
"tgt": "What causes a blood filled cyst near the ear lobe in a child?",
"src": "Patient: The kid got a like a cyst blue mass under the skin(near the earlobe). The doctor said it moves around and looks like a cyst -trapped blood under the skin and sent us to ENT, what could it be? We went to ENT and they have no clue what it is and suggested MRI... then thought and figured ultrasound could be a smarter choice... Doctor: many thanks.mass below ear lobe are usually related with parotid gland These gland dome time swell up due to some infection.if you have a phtogragh then it is easier diagnosis.I will advised urgent ultrasound of the lesion and then may need MRI.if any solid lesion the need tissue biopsy will be good idea. pediatric surgeon or ENT specialist might help him."
},
{
"id": 131574,
"tgt": "How can i overcome low immune system and balance?",
"src": "Patient: Hi. Why do I feel spaced out, numb, or high when I look up to the left, forward or turn my neck? My tongue feels numb when this happens. I get sharp right side of face pain, eye pain. I have joint pain, trouble walking. I also get spine pain, middle pain and weakness. I have infections a lot. Uti s. Swollen glands. Inflammation in right eyes needing steroid. Neuropathy. Bad muscle cramping in legs and even torso. Balance and coordination issues. A poor neuro psych eval in info processing - cognitive decline, fluid buildup in body with mottled skin swollen lymph nodes maybe due to constant infections. Dr s agree low immune system high inflammation. Doctor: the reason could be sever spondylosis with osteophytes blocking your vertebro basilar artery that supply your head and brsin with blood also osteophytes could cause all your other neuropathy symptoms i suggest doing a CT scan on cervical vertebra to confirm i hope i my advice will be of much help Good Luck :)"
},
{
"id": 60314,
"tgt": "Is being hbsag positive (reactive) hereditary ?",
"src": "Patient: is it true that being hbsag positive( reactive ) is hereditary?i am a afraid because my older brother and some of my cousins from my mother side have this infection. Doctor: Hello/Hi Welcome to health care magic \u00a0\u00a0\u00a0\u00a0\u00a0HbsAg can be transmitted from one person of family to other ,chances of hereditary are not well known. \u2018Hope I have answered your query, I will be available to answer your follow up queries, \u201cWish you Good Health and speedy recovery\u201d Disclaimer"
},
{
"id": 147640,
"tgt": "Is numbness and tingling caused due to nerve pressure?",
"src": "Patient: Hi :) my boyfriend is being prescribed an antibiotic and pain medication for an abscesses tooth. He has been on both for about 6 days now. About 5 days ago the pain was terrible so he decided to use an over the counter remedy as we ll. I think the main ingredient was clove oil? And it had said not to touch to areas surrounding the tooth... Which I am sure he did... Since then he has had burning- numbness and tingling in that area. He describes it as like Novocain wearing off but also a burning sensation. Do U think this is caused by the over the counter remedy or maybe nerve pressure? He is on tour with a band and not even close to our doctor. (Things like this have a way of having amazing timing lol) I feel awful he is in so much pain and any advice you could offer would be amazing. Thanks!! Doctor: hello thanks for consulting at hcm..it could be due to clove oil on prolonged usage..ask him to discontinue using it ..since he has already taken a course of antibiotics and analgesics,,he shd get an incision and drainage done if there is an abcess,, kindly consult ur dentist once he finishes the tour..until then he could take analgesics to relieve his pain...hope it helps,,tc"
},
{
"id": 177002,
"tgt": "Suggest medication for pain,redness and lump on thigh due to a fall",
"src": "Patient: My daughter fell earlier yesterday and landed on her thigh, it hit the black edging they use to keep in mulch at playgrounds.she can hardly walk on it. It was red and hot to touch. She also heard a popping sound, it also hurts her to touch it and it now has a small lump. Should I take her in to get checked? Doctor: Hi Dear Welcome to the HCM,Antiinflammatory analgesic drug along with the muscle relaxant to be used.Hot fomentation.X ray of the area to be considered, to know any bony injury, which is quite likely.Hope the query is answered.thanks"
},
{
"id": 101348,
"tgt": "Does apple cider vinegar cause allergic reaction?",
"src": "Patient: I recently started to feel a cold sore coming on. I decided to try something new, so I applied a high quality apple cider vinegar directly to the cold sore in 15 minute intervals for about an hour. My lip is now 4 times its normal size. Allergic reaction? What should I do now? Doctor: Hi, thanks for using healthcare magicIt is possible that it is an allergic reaction though some persons may experience some swelling with cold sores (due to body's inflammatory response to the infection).You should consider the use of oral anti histamines, if it is allergic, this would reduce the swelling.These would be available over the counter, examples are: benadryl, claritine, allergra, zyrtecI hope this helps"
},
{
"id": 102596,
"tgt": "Suggest medication for asthma ,cold,shortness of breath,dizziness and palpitations",
"src": "Patient: im a women 35 recent ally getting over a cold about three weeks now\u2026sinus upper chest cold..i have asthma but my concern is as I m feeling better from the cold\u2026I m having a bit of shortness of breath heart seems to be pounding and i get a bit dizzy lightheaded..occasional heart palpations feels like heart fluttering.. Doctor: hi, are you on medication? as drugs used for asthma may cause palpitations. you may require steroid inhalers which can give good relief in short term use. hope this helped."
},
{
"id": 219626,
"tgt": "Will ingesting alcohol or taking Lexapro affect a pregnancy?",
"src": "Patient: I am pregnant. The father is a big pot smoker.... is there a chance of problems with baby from a mans sperm who is a pot smoker? Also I didnt know I was pregnant and I am 8 weeks. I have drank a few weekend ans took lexapro for a while, what affect can that have? Doctor: HelloWelcome here.I understand your concerns.Pot smoking in father will not affect the developing fetus.In your case, regular alcohol can definitely affect the baby's development. There is a risk of lexapro causing effect of baby development but only on regular use.Hope this answers your query.Thanks."
},
{
"id": 103465,
"tgt": "Sore throat, allergy, bump on roof of mouth, hurts to swallow, swollen neck",
"src": "Patient: I have had a sore throat for a couple of days. I thought it was just allergies starting for the year because i sneezed about 30 times yesterday. I could not sleep last night and i woke up to find a bump on the roof of my mouth near my front tooth . It hurts to swallow so I just looked at my throat with a flashlight and it is completely swollen in the back. Doctor: ACUTE ALLERGIC SINUSITISTHE POSTNASAL DRIP OF SINUSES CAUSES SWELLING AND INFLAMATION OF THROAT GIVING THE SIGNS YOU WROTEYPU NEED DOXYCYCLINE 100 MG BD FOR 5 DAYS FOR INFECTION AS THESE ARE FAST CLEARER OF INFECTION OF SINUSESTAKE FEXOFENADINE 120 MG BDTAB CPM NIGHTADD ANTACID GEL TDSAPPLY TRIAMSOLONE MOUTH PAINT IN MOUTH BEFORE EVERY MEALLOT OF WARM WATERAPPLY NEOMYCIN H EYE OINTMENT IN NOSE BDSEA WATER 2 DROPS IN EACH NOSE NIGHTCONTINUE 3 WEEK YOU WILL BE OK"
},
{
"id": 192278,
"tgt": "What causes bad odor to the semen?",
"src": "Patient: My boyfriend has very foul smelling semen...What could this mean? Also since we have gotten together(2 years ago) I have been struggling with bladder infections and I have also has a couple of vaginal bacterial infections. I am now wondering if he could possible have an infection and is spreading it to me through sex. I know that it isn't an STD because I have been tested (just recently). What's going on!? Doctor: Hi, infection in semen may be the case of foul smell. Go for semen c/s & urine c/s. Follow up with reports. Hope I have answered your query. Let me know if I can assist you further. Take care Regards, Dr. Ashish Kumar Khandelwal"
},
{
"id": 175904,
"tgt": "Can Ovol be given along with Amoxicillin and Ascoril?",
"src": "Patient: My 5wks old baby who weighed 8lbs when he was born...keep bringing up his milk and there was sumn like slime or cold came up off his chest with something brown in it.he sounds like they is a cold on his chest ...I was given Amoxicillin and ascoril.. .. while hes taking these can I give him ovol? And is these medications ok Doctor: Hi..you can give Ovol when using Amoxicillin and Ascoril. But 95% of kids illnesses are viral and do not require antibiotics like Amox.Regards - Dr. Sumanth"
},
{
"id": 167298,
"tgt": "What causes pulmonary heart infection in kids?",
"src": "Patient: Hello Sir My Son (2.5 Years) suffering with Right Pulmonary Heart. He is getting infection frequently, cough and fast breathing. Doctors said his right lung is small compared to Left Lung as blood is not passing to his right lung and there is no path also to unblock it. Also doctors said..there is no treatment for it. Could you please suggest is it something can be resolved ? Doctor: children usually dont have recurrent respiratory tract infection in pure right pulmonary heart but this may occur in mixed type of structural lesion. operability depends on complexiy of structural defect and experience of surgeon. so parents should seek second opinion."
},
{
"id": 100406,
"tgt": "Suggest treatment for breathing problems",
"src": "Patient: MY SON AGE ABOUT 11 SUFFERING FROM ASTHAMA FROM THE AGE OF 5, EARLIER BREATHING PROBLEM OCCURED FREQUENTLY 2 TO 3 TIMES IN A MONTH, RIGHT NOW IT IS ONCE IN ATWO MONTH, ACCORDING TO DOCTOR IT WILL DECREASE BY THE TIME, WHAT IS THE BEST ADVICE TO PREVENT BREATHING PROBLEM.CAN I DIAGONISE THRU HOMEOPATHY AND ELOPATHY TOO, PLS ADVICE BEST DOCTOR IN KOLKATA Doctor: HIWell come to HCMI really appreciate your concern, if this is the Asthma then in this age group this could be common some times but this comes around as the child grow old and chances of this would be pretty enough, no need to go for any other system of medicine continue the medicine must have been prescribed by his doctor, take care."
},
{
"id": 15722,
"tgt": "Rashes, on neck, breasts and back with yellow liquid. Why do they happen?",
"src": "Patient: Hello,I seem to be getting a particular kind of rash quite frequently nowadays. It occurred last year in the summer for the first time. Now again, I have red dots which quickly multiply only on my chest (area under my neck and above breasts), nowhere else. Happened again 3 weeks ago, in a week they disappeared, my skin was ok for 5 days and they are back again. I was thinking if it could be heat connected but I have always lived in a hot country and this never appeared before. I live in England now and it is just not that hot. I had an appearance in the winter too but maybe once. SO I have no idea what it could be. The rash does not itch at all. In a day or two after the spots appear (they come out as flat red dots), then you can see like whitish/yellowish liquid inside of them. Takes ages to heal and then appear again. I can not wear suitable for the season clothes now as it looks disgusting and I hide it which makes it worse. If it is \"sweat rash\", why doesn't it ever itch at all? Plus, I don't really sweat, I always even wear less clothes than anyone else I am so so confused. Please, help. Thank you so much! Doctor: Hi ,thanks for writing to health care magic.Rash is suggestive of fungal infection.It can occur in case you have dandruff.Use antifungal shampoo for dandruff.Keep area dry and clean.You can use antifungal powder at day time over affected area.Use loose cotton clothes.At night you can apply antifungal cream like miconazole.I hope this will solve your problem.Take care."
},
{
"id": 8554,
"tgt": "Is salicylic acid gel treatment effective for pimple spot and pores ?",
"src": "Patient: hi doctor ,m 22 yrs old n my skin is very oily n m getting pimles on my face n back which leaves dark spots n i hav large size pores which does nt look nice ..i wanna go for salicylic acid gel treatment for this problem ,pls guide me weither it is effective or not ? Doctor: Hello and welcome to healthcaremagicSalicylic acid is commonly used by dermatologist for chemical peeling in those whoc have acne and oily skin. Salicylic acid being lipophilic in nature easily dissolves in the lipid rich pores and breaks the comedones, which are the precursor lesions of acne. Salicylic acid also dries up existing pent up oil in the pores and is therefore also useful for oily skin.Along with salicylic acid peeling, you can also use topical antiacne medication like clindamycin phosphate 1% gel, twice daily.If your acne is inflammatory, you may also benefit from the use of oral antibiotic like either doxycycline or azithromycin or minocycline.regards"
},
{
"id": 60473,
"tgt": "5 year old kid suffering from bilirubin with jaundice and mild fever",
"src": "Patient: my 5 years old daughter has normal liver enzymes but high total and direct bilirubin with jaundice , dark urine and mild fever. she feels well and no other complaints, like no nausea or vomiting and has normal appetite and these symptoms started only yesterday. what this could be? Doctor: Hi, Thanks for query, Your daughter is suffering from infective hepatitis. Avoid fat and give more carbohydrate and protein diet. Give plenty of water and glucose. Give B.complex. Give her complete rest. ok and bye."
},
{
"id": 151496,
"tgt": "Stroke-like incidents, pins and needles sensation in head. Should I go to the doctor?",
"src": "Patient: Hello, When I was around 12, I was being a big baby and crying about something (don t remember what) when I felt something shoot up the right side up my neck into the right side of my head. It felt like the right side of my head suddenly got stuck with pens and needles. I didn t experience any other symptoms. I had no numbness anywhere in my body. I didn t tell anybody about it. Over the course of about the next 5 years, similar occurrences happened maybe about 5 or 6 times. These were never as profound as the initial incident, and only occurred during certain burst n movement. It happened once when I dunked a basketball, once when I turned my neck suddenly marching with the band in a parade, and once when I was about to run. I m 27 now, and NONE of this has happened within the last 9 or 10 years. I have to be unhealthier now. I ve been a drinker (weekends mostly) and cigarette smoker since I first went off to college at 18. I hadn t even attempted to drink or smoke once before 18. I was a little underweight in grade school. I m a litter overweight now. I m just confused how these stroke-like incidents occurred when I was healthier, but nothing has occurred in the last 10 years. Did these sounds like strokes? Or maybe something else could have caused it? Despite 10 years, should I still go tell a doctor and get checked out? For some reason out of the blue, this popped up in my mind a couple days ago and I m worried it will randomly happen again. Doctor: Dear friend, welcome to HCM, none of these symptoms point to any stroke. you need not worry much. stroke will come as local weakness, occassionally persistent altered sansation in well defined territories. take care and pl. keep me posted on the progress."
},
{
"id": 145350,
"tgt": "Suggest treatment for tingling in fingers after hand injury",
"src": "Patient: After a car accident (more than 8 months ago) my right wrist was having super sharp pains on the left and right sides when I picked up something. It also was locking up and felt like it needed to be cracked. The hand specialist had me wear a brace with top and bottom supports that I absolutely could not tolerate because it made the top of my hand and wrist hurting and got really tingly and cold. He gave me one without the top support and a cortisone injection. I wore it for 8 weeks. For two weeks I didn\u2019t wear anything when I then started getting constant tingling on the top of my thumb, pointer finger, top of my hand and wrist...no symptoms on the bottom. Since, I have had constant tingling in my right thumb and pointer finger. I have a brace on most of the time, otherwise it is much worse and gets very cold and hurts on the top of my wrist. You can physically feel the temperature difference between each hand. Today I had an EMG and the one where they put needles in your muscles. One of the shocks was supposed to go only down into my thumb, but it made horrible pain in my pinky finger. When he did my cercal spine area (I have spondylosis) it went up into my head and not down my arm. After this test he wrote the report and I saw the hand specialist. He said my EMG and tests were normal!!! ... as he was inspecting my hand, which was in a constant tingle and very cold. The report did say I may have a neuroma on the nerve above my wrist that goes into my thumb, but the hand doc said he didn\u2019t think this was the case, even though he could touch the area above my thumb up to above the inside of my wrist and it would produce strong tingles in my thumb on demand. He said then said there was nothing he could do for me. I wanted to cry; please help. Doctor: Hello ! I understand your concern 1 In my opinion the symptoms you describe may be related to a nerve injury . A neuroma is an abnormal overgrouth of the nerve in the region of the injury , after the nerve has been cut off from the trauma. It develops some time after the trauma. The changes in color and temperature in both hands are in favor of a nerve injury. Your symptoms also mimic the so called carpal tunnel, which is caused by the compression of the median nerve at the wrist. I would recommend you to go to a neurosurgeon specialist for the peripheral nervous system intervention . In both the possible situations I described above a small intervention would be very helpful to you. I recommend you not to wait too much , because these situations become worse and irreversible later if not treated. Hope to have been helpful. Best wishes, Dr. Abaz Quka"
},
{
"id": 170547,
"tgt": "What could red sores with loose stools in infants suggest?",
"src": "Patient: Hi my 13 month old child over the last 24 hours become very grogy and keeps appearing in wat looks like nettle stings in random places on his body, his bum is extremely red and sore his stool is very loose and mucusy and green, I m really concered but could it be part of teethin or something else? Doctor: Hi, your child has probably got stomach infection as the child has loose stools and rash over perianal region is present. In my opinion, you should give ORS solution to the child to prevent dehydration, and antibiotic like syrup oflox needs to be given for 3 days. Also, you need to give enterogermina ampule once a day for 3 days to increase gut immunity. Apply rashfree cream at the rash at bumps for 5 to 6 days. Take care."
},
{
"id": 122231,
"tgt": "Suggest correct medicine for ankylosing spondylities",
"src": "Patient: hello sir, this is lizon from dhaka. actually i am a patient of ankylosing spondylities. my hip joint (lt) space has a mild loss. now i can walk more or less fine, i have HLAB27 possitive. getting medicine (salfasalagine usp 500) in morning and night and also getting indomethacin bp 25 at morning-lunch-night. at present i am felling pain in my Lt hip, Doctor: Hi, There is a new medicine out for it that targets Interleukin 17A. It is a relatively narrow spectrum of action on the immune system, still, it's most worrisome side effects are from immuno-suppression. It is the only drug I know of that actually targets the disease process of ankylosing spondylitis. Hope I have answered your question. Let me know if I can assist you further. Regards, Dr. Matt Wachsman, Addiction Medicine Specialist"
},
{
"id": 179114,
"tgt": "What causes pinkish tint after urinating?",
"src": "Patient: I have a 2 1/2 year old daughter and once yesterday and once again just now, she urinated and everything seemed fine until we wiped her, then we noticed a pinkish tint on the toilet paper. She hasn t said it burns and the urine in the toilet looks normal too. Not sure what is going on. She has been drinking maybe a little more than normal today, other than that nothing out of the norm... Doctor: Thanks for posting on healthcaremagic. Urinary tract infections are often asymptomatic in children. The pinkish discoloration ofurine could be due to blood due to UTI. Please get her urine tested for routine examination and culture. If the results come negative, you can relax."
},
{
"id": 90604,
"tgt": "What is the dull aching pain on my right side?",
"src": "Patient: Hi i am a 44yr old female & i have a dull aching pain on my right side that seems to go from front to the back...it started when the skin on my back (just where you would put you hands on your hip) became sensitive to touch (& still is)....i also feel bloated and constipated...i've been awake with it all night...can you help please? I thought it might have been apendicitis but the pain has remained constant and not sharp!!! Doctor: Hello! Welcome to HCM.This seems to be urinary stone related pain.Stone in ureter causes obstruction to urine outflow & smooth muscles in ureter contract excessively to overcome this obstruction and cause pain.In my clinic, many such patients do come to whom I advise ultrasound abdomen & pelvis, X-ray KUB, urine routine, CBC, renal function tests.For temporary pain relief, I prescribe smooth muscle relaxants like drotaverine, pain killers like diclofenac, and tamsulosin.Definitive treatment is done according to the diagnosis.Also it is advisable to drink 2-2.5 litres of water in 24 hrs.Hope this helps.Wish you a good health.Thanks.Regards."
},
{
"id": 185214,
"tgt": "Can carmex be used on lips to soothe after having a dissolvable stitches?",
"src": "Patient: hi, i had a rugby injury where i fractured my dento-alveolar and split my top lip in half, they used dissolvable stitches on my gum and upper lip. Can i use carmex on my lips to soothe them? because they are getting very dry, but i do not want to affect the stitches. thanks steve Doctor: Thanks for using Health Care Magic .Read your query.Yes, you can apply it as far as you are not allergic to any of its products.Avoid applying pressure or rubbing it vigorously on the lip such that the sutures are disturbed.Thanks and regards."
},
{
"id": 215794,
"tgt": "Suggest a treatment for spasmatic torticollis involuntary",
"src": "Patient: I had fusion spine surgery on c-2 -c6 because I have spasmatic torticollis involentery my brain triggers chronic pain wore my protein between my spine bones . I had surgery March 17, this year 2and a half months ago. My spasms pulls my ncck down. They don t have a neck brass to fit me. Will this weaken my bone mass or wii my neck get strong like it was before my surgery? Very scared. My spasms never pulled my neck down and I know the first 12weeks your neck nots to pull down so the bone mass can fuse around my spine bone. Doctor: Hi, As a first line management, you can take analgesics/muscle relaxant combination for symptomatic relief. Acetaminophen/Baclofen combination is a good choice and has got good results. If symptoms are severe, you can consult a neurologist and get evaluated. Hope I have answered your query. Let me know if I can assist you further."
},
{
"id": 202674,
"tgt": "Any natural/herbal or allopathic medicine for erectile dysfunction?",
"src": "Patient: Dear Doctor, I am having erectile dysfunction for the past 10 years. i am 59 year old. Otherwise i am a very active guy. Please recommend good natural/herbal or alopathic medicines for this condition Vijay Doctor: Yohimbine and horny goat weed can make erections come back, but these medicines have cardiac side effects. Please rate 5 stars! I strive to provide you the best answer to your questions!"
},
{
"id": 82042,
"tgt": "What causes pain on the lungs while breathing with thyroid cancer?",
"src": "Patient: My 15 year old son was diagnosed with thyroid cancer in October. He had a bilateral neck dissection. The cancer metastasized to his lymph nodes and lungs. He is now complaining of left lung pain when he takes a deep breath with profuse sweating. Can it be related? Doctor: Thanks for your question on HCM. Since you are saying that he is having thyroid cancer (operated) with lung and lymphnodes metastases, his chest pain can be due to1. Pleural effusion2. Pneumonia3. Lung metastases Malignant pleural effusion is very common in thyroid cancers and if present can cause pain on inhalation. Pneumonia is another possibility. And if worsening of lung metastases is there than this can also cause pain.So better to get done chest x ray and if needed CT THORAX to rule out all these."
},
{
"id": 128800,
"tgt": "Does orbital fracture cause prolonged pain and numbness in the top jaw?",
"src": "Patient: Hi, a month ago i was jumped and had a non blowout orbital fracture, swelling went down wothin 2 weeks, my first week i wasnt able to eat anything because my jaw hurt so much it has gotten better but my top jaw still feels numb and it still hurts if i bite down to hard should i be worried Doctor: Hello,Yes this is cause for concern. See a doctor. You may need additional xrays or other imaging to determine the cause for your ongoing jaw pain.Regards"
},
{
"id": 192764,
"tgt": "Suggest medication for swelling of lymph nodes in groin area",
"src": "Patient: My lymph nodes near my groin are swollen. And I do have a bad case of poison oak on my penis and scrotum. Especially my penis became quite swollen but is slowly becoming less so in the last couple of days. Are the swollen lymph nodes a perfectly normal reaction to this or should I be more concerned? Doctor: Hello, It could be due to local inflammation. Consult a general practitioner and start a short course of antibiotics like amoxiclav for five to seven days. If symptoms persist better to consult a general surgeon and get evaluated. A fine needle biopsy is required to make a diagnosis. Hope I have answered your query. Let me know if I can assist you further. Take care Regards, Dr Shinas Hussain, General & Family Physician"
},
{
"id": 117622,
"tgt": "Suggest treatment for anemia",
"src": "Patient: hi.i am a student doctor.few months ago i found myself losing my weight and a pale color.then, on the internet i searched for the cause.noticing all the symptoms (hair loss, pale color, fatigue even after having a good sleep, headache etc) i found that i am suffering from iron deficiency anemia.for that i used \"Maltofer\" that is a tablet enriched with iron3.i found the dose quite effective but then i didn't took the dose properly.now again i am having the same problem.can u suggest me some better advice? Doctor: Hi,Thanks for asking.Based on your clinical history and query, my opinion is as follows:1. Check your hemoglobin for confirmation of anemia2. Check red cell indices to look for low MCV to diagnose iron deficiency anemia3. Start on iron therapy and after 15 days, check with reticulocyte count to evaluate for iron response.4. If responding well continue therapy until hemoglobin reaches normal level.5. Have a nutritious diet too.Hope it helps.Any further queries, happy to help again."
},
{
"id": 184460,
"tgt": "Suggest remedy for swelling of gums and difficulty in swallowing",
"src": "Patient: Hi my lower molar was already removed last Monday afternoon. As per my dentist the pus was also removed and I just need to take my antibiotics. But I noticed yesterday that my gums is swelling and pus is starting to be noticeable. I'm having hard time in swallowing and unable to open my mouth easily. Should I have x-ray for this one or a surgery because my left face is starting to swell. Thank You. Doctor: Hello,Thanks for consulting heathcare magic.As you said that the extraction is done one week ago and swelling appears just one day before and you can see the draining pus and swelling, it is a clear case of re-infection of the socket site.As the swelling appears on the face and difficulty in opening mouth is also present, it is a case of space infection in that region.Please visit your dentist immediatly and get the treatment done. Regards,Vishal JainVitaldent."
},
{
"id": 58387,
"tgt": "Liver panel levels are slightly off. Platelets count well below. US normal. Take seroquel and aspirin",
"src": "Patient: I am concerned about my liver. My annual liver panel numbers AST/ALT have been slightly off for at least the last five years. Plus, I have had well below normal platelets for at least the last four years. My doctor tested for hepatitis, and did an ultrasound of my liver/gall bladder. Everything was normal. Additionally, I have what appears to be classic Terry's nails. However, I have no symptoms that I am aware of - no weight loss, jaundice, fatigue. I am not overweight, don't smoke or drink any alcohol, an not diabetic, and I am extremely active - at least eight to ten hours of heavy cardio and weightlifting every week. I also take 100mg of seroquel (bipolar) and up to four aspirin or advil a day for joint pain - I was diagnosed with ankylosing spondylitis in 2000. Doctor: Dear friend,I am Dr. (Col) R Karanwal, M.D. (Internal Medicine), Consultant Physician, for your assistance.Thanks for reposing your faith in me and post your query on this website. I fully appreciate your concerns. 1. All drugs which are metabolized (detoxified) by the liver, can also be toxic to liver. Aspirin and Advil are known to be toxic for liver, which manifests as raised ALT/AST levels.2. Low platelet counts can either be a manifestation of ankylosing spondylosis or due to prolonged use of aspirin.3. If I were your treating doctor, I would have prescribed medicines which are quite effective in anklyosing spondylosis. rather than simply continuing with pain-killers. Consult your doctor and discuss specific treatment for your chronic condition.Please feel free to revert back to me in case of further clarifications, if any.I am sure that you will benefit from the advice.May I assure you that I am always available for your assistance. My endeavor would always be to give you a sound medical advice, to the best of my knowledge and vast experience.HAVE A NICE DAY AND A SPEEDY RECOVERY.Fond Regards,Dr.(Col) R KaranwalMBBS; M.D. (Internal Medicine)Consultant Physician(PLEASE NOTE :- My opinion/advice is based SOLELY on the details provided by you.)"
},
{
"id": 118268,
"tgt": "What to do for abnormal ESR?",
"src": "Patient: Just received my blood work from my doctor from my physical. I am 52 years old and have been having issues with inflammation for quite some time now. All my blood work is normal except my ESR.I exercise, and have been eating healthier. What do you suggest I can eat or take to get this rate to normal. Debbie Doctor: ESR is an investigation which indicates the status of inflammation in our body. High ESR means that chances of active inflammation are there in the body. In my opinion you should continue the treatment for inflammation, cure of inflammation will automatically normalize the ESR. There is nothing in particular to eat by which you can reduce the ESR. I will just advise you to continue eating healthy food and increase the amount of vagetables and fruits in your eatable items."
},
{
"id": 119086,
"tgt": "Have high esinophilia count. Have sneezing and water comes eye also. What is the remedy?",
"src": "Patient: sir, i have problem of having too much absolute esinophilia counts. and frequent sneezing problem at the early morning .maximum time waters comes in my eye .I have also pain in my sole .please guide me regarding the causes and how can i be relief from such types of problems.please also guide the foods that should be restricted...... Doctor: Hello, Cause of eosinophilia may be idiopathic ,allergic, hay fever,drug allergy ,parasitic, an some neoplastic disorder .Treatment of eosinophilia is aimed at cause of eosinophilia. As you have mentioned you have pain in sole your eosinophilia may be due to parasitic infection . Stool examination may be reqd to rule out parasitic infection . If it is due to parasitic infection it can be treated by antiworm medicines."
},
{
"id": 173988,
"tgt": "What causes itchy patches and Pityrosporum Orbiculare 99% in test?",
"src": "Patient: Hi my son has very itchy patches all over his body he has had these for approx 6 months now and the doctor has been treating him for Psoriasis. However we have just got results back from an Intolenrance test and on his results Pityrosporum Orbiculare is 99% which is very high. Can this be the answer to our problem? He has a lot in his head, chest, knees, arms, back just about everywhere. Doctor: HELLO GREETING FROM HCMI UNDERSTOOD YOUR PROBLEM PITYROSPORUM ORBICULARE IS SPECIES OF FUNGUS WHICH IS MAINLY RESPONSIBLE FOR DANDRUFF AND SEBORRHOEIC DERMATITIS. THIS MAINLY EFFECT OIL GLANDS OF BODY, SO THE MOST EFFECTED PARTS WILL BE HEAD, UPPER BODY, NASAL AREA.IN PSORIASIS PATCHES ARE PURPLE IN HUES.IN MY PATIENT I USUALLY START WITH ANTI FUNGAL TREATMENT LIKE KETOCANOZOLE SHAMPOO FOR HEAD.AVOID OILING IN HEAD."
},
{
"id": 176391,
"tgt": "What are the long term side effects of removing testicular torsion?",
"src": "Patient: My one year son have testical torsion and removed left testical and stitch the right testies . I am worried for his future he is only one year old is his pubirity come normal ? Does he live normal life ?Does he have children in future ?please help and reply does he have normal testestron ? Doctor: Hello. I just read through your question.With one testicle, your sin will have a normal life. He will go through puberty normally and will be to reproduce and have a normal sex life."
},
{
"id": 62415,
"tgt": "What causes tender knots behind the head?",
"src": "Patient: I have two large knots on the back of my head...behind my ear, first near top/behind ear, the next follows that down...I feel another one forming down past that one. They are tender to the touch and tend to give me headaches. I have had them for over 2 months now. I've taken one 3 day antibiotic and they went down in size but now that are enlarging again. What type of doctor should I see or type of medicine I may need to make them go away? Doctor: Hi,i can understand your problem.Tender nodule behind ear may be due to any infection.It require physical examination and evaluation like FNAC to make a proper diagnosis.You can start some broad spectrum antibiotics for few days along with painkiller like NSAIDS.Thanks"
},
{
"id": 142997,
"tgt": "How to treat cervical spondylitis?",
"src": "Patient: Sir, I am suffering from cervical spondylitis whose symtoms are pain in neck and also pain and sensation in my right shoulder and right arm. My doctor Dr. Amarendra Kumar MBBS. FRCS from Patna ( India ) have prescribed me a cap. Gabasafe of Aristo pharmaceuticals Doctor: HIWell come to HCMI really appreciate your concern, it seems to be cervical degenerative disease analgesic may not be the cure for disease but it requires complete evaluation for cure some time it may needs surgery or some time it may comes around with conservative line of treatment if you have not done the MRI Then get this done and have opinion from neurologist hope this information helps."
},
{
"id": 125812,
"tgt": "What causes intermittent grinding sensation in the shoulder joints?",
"src": "Patient: I am a college baseball pitcher.I have a constant gravel like feeling in my shoulder whenever I rotate my arm and my elbow is above or parallel to my shoulder and it has disabled my ability to throw a baseball completely but doesn t disable me in any other activities. Any thoughts? Doctor: Hello, It could be due to muscle strain or contusion due to repeated physical exertion. As a first line management, you can take analgesics like Paracetamol or Aceclofenac for pain relief. It is better to consult an orthopedician and get an MRI scan to rule out tendon or ligament involvement. Hope I have answered your query. Let me know if I can assist you further. Regards, Dr. Shinas Hussain, General & Family Physician"
},
{
"id": 165086,
"tgt": "Suggest cure for watery motions in infants",
"src": "Patient: 5 months old baby have watery motion 5 to 6 times a day for the past 7 days.watery some times seeds like in water stool. after checking doctor he say its normal and will be alright by itself have prescribed medicine twice a day and ORS powder, but still not cured, so concerned how to cure this Doctor: Dear Parent,By your description its not clear if the child is totally breast fed or has been started on top food/ weaning. But in any case if the child is feeding well, accepting fluids, passing motion and urine, just keep a watch, it will go on its own. if frequency and quantity of motions increases, then again visit your doctor."
},
{
"id": 116228,
"tgt": "How to reduce ALT levels in blood?",
"src": "Patient: i was immediately diagnosed with malaria after having no appetite n vommiting and alt levels raised to 210mg/dL. after a day the lft was repeated n it showed an increase to 1410mg/dl. after two days the test was repeated n it shows 1040mg/dl of alt. how can i help decrease it fast as itis causing severe nausea and vomitting. Doctor: Hi,Thanks for asking.Based on your query, my opinion is as follows.1. You are possibly suffering from malarial hepatitis, which is the reason for the increased ALT levels.2. The nausea and vomiting is at present due to liver damage and once the malaria or other cause for hepatitis is evaluated and treated, you nausea and vomiting will come down. 3. ALT level to fall will take 3-4 weeks. The symptoms of nausea and vomiting will reduce once the causative agent is removed. Liver biopsy is necessary. Avoid taking alcohol or other hepatotoxic drugs. Watch out for anti-malarial drugs affecting the liver.Hope it helps.Any further queries, happy to help again."
},
{
"id": 132686,
"tgt": "Does clindamycin cause joint pain?",
"src": "Patient: does clindamycin cause joint pain, i was prescribed it for tooth pain in november and two weeks later I develope joint pain in the kness which has since gone to my hips and shoulders.. but has gotten better in the knees and hips it know hurts the most in my shoulders and down my upper arms..I m taking advil for the pain and have been since December, it s the only thing I ve found to take the pain away..I ve been to the doctor three time and have had three blood test done and they don t know what it is, suggested a viral syndrome. I m thinking of contacting the FDA.. Doctor: Hello,I can understand your concern and feel sorry for the agony you have been through. Clindamycin is commonly given antibiotic for tooth and oral cavity infections. It does cause joint pain as a side effect. However, developing joint pain after 2 weeks of starting the drug and having it constantly for months cannot be related to the side effect of this medicine as the medicine has left your body by now.As your blood tests are also coming normal, infection and other blood related disorder can be ruled out. However, if you consult an orthopedic surgeon or pain management clinic, some light can be shed on what you are suffering from and it can be treated.I hope this information helps you. Thank you for choosing HealthcareMagic. I wish you feel better soon.Best,Dr. Viraj Shah"
},
{
"id": 111204,
"tgt": "What causes persistent back pain for weeks?",
"src": "Patient: My 15 year old son has has persistent back pain for over 2 weeks now and it's not improving, he says the pain feels like it's in his spine, the docs just said muscle strain but there's no improvement after 3 days of rest, he said the pain is changing from a dull ache to a sharp pain...please help!! Doctor: Hello,I had gone through the case and it might be slip disc or herniation of disc.So go for MRI of spine and then go for physiotherapy and treatment.Hope my answer will be effective for you.Thanks"
},
{
"id": 25864,
"tgt": "Suggest remedy for hypertension",
"src": "Patient: On blood pressure medication 50 mg.. blood pressure has been overly high last couple days have not been feeling myself. 181/111 took a extra Atenolol didn t do anything keeps going higher not sure if I should be concern about this? Pulse is only like 60. Do I need to go into a urgent care or wait till tomorrow and see If it goes down?? Doctor: Hello and welcome to HCMI have read and understood your queryI hope this will helpYou are having uncontrolled hypertension.. Which is a risk factor for heart ,RENAL and cerebrovascular diseases..I think you should keep on waiting for ATENOLOL 50MG to work for you ..you should consult your GP again and there are two options:Either dose of atenolol should be increased to 100mg per day (or)A second anti hypertensive drug should be added to it..Mean while try to lose weight, do regular exercise, eat less salty and fatty foods and avoid taking stress..I hope this was useful.. You can reach me any time with an otgeru questionRegardsDr faeza"
},
{
"id": 131409,
"tgt": "Is it OK to use spray when I am in plaster cast?",
"src": "Patient: I am in a plaster cast and have been for about a month and half. I think I have athletes foot. I have had it before. It itches and hurts pretty bad. I know I am not supposed to use lotion, but is it ok to use the spray? Should I call my doc? by the way, no, I am not shoving anything in my cast :) Doctor: I think you should go for a check of the cast and maybe replace it or remove it. you should check it with your doctor."
},
{
"id": 112200,
"tgt": "Upper back and shoulder blade area experienced popping sensation. Is that the reason why I have difficulty breathing?",
"src": "Patient: I was in the middle of sitting down and I sneezed. I heard a crack and now my upper back somewhat in line to shoulder blade hurts. I can t breathe right because it hurts bad. I m 15 and play basketball so it s important I please get a reply. Thank you Doctor: this may be the cause of your back pain and difficulty in breathing. for this type of symptom i prescibed the patients tablet acelofenac+thiocolchicoside twice daily after meal with pantoprazole for five day and investigate with chest xray PA view.i advise you to contact your physician to prescribe drug and investigathio"
},
{
"id": 205117,
"tgt": "Suggest medication that can be taken with lithium for bipolar disorder",
"src": "Patient: Hi My name is Oscar and I suffer from rapid cycling Bi-polar disorder and ADD. First of all, Thank you for your time. I weigh 275lbs and I am 40 years old. I ve been on lithium(300mg X 6/day) for a year and I also take Lexapro with it. I recently stopped taking the Lexapro because it was not working. In the past I have taken Wellbutrin with my lithium. I also used to do drugs I have been off for at least 5 years I don t know if maybe that factors in to why I am having trouble finding a medication that works. I don t have insurance and my primary physician is the one prescribing my medications and I wonder if the fact that he doesn t specialize in this field has something to do with it. If you could please recommend a drug that would work better with my lithium I would greatly appreciate it. Thank you again. Doctor: hi and thanks for question.u have bipolar disorder and u have take lithium than i will suggest u to take lurasidone which effective in bipolar disorder. u shall tell your physician to advice visit psychiatrist for problem. thanks"
},
{
"id": 109268,
"tgt": "Suggest remedy for back pain",
"src": "Patient: Hi, may I answer your health queries right now ? Please type your query here...hello, first of all thank you , i am mirzo now i am working in south korea and my work is to load sacks with 20 kg, when i am working i feel pain from my back and after the work i feel the same pain especially on the right side of the back when i sit down and bend over, some my friends say that it depends on the kidneys, please help me about it , what must i do , can i continue my work? Doctor: HiI think its lower back pain than kidney pain.Your pain is due to lifting weight and bending forward.You should change your work style.Get well soon."
},
{
"id": 148692,
"tgt": "Diagnosed with encephalopathy, Arteriovenous Malformation. Left side paralysed due to stroke. Having blood clot in brain. Chances of recovery?",
"src": "Patient: Hi. My nephew has been diagnosed with encephalopathy. He has AVM. the first attack bleeding on the brain. Stroke left side of his body paralized on 07/02/2013. 10/29/2013 blood clot in his brain from the AVM. We belived he was in a comma. after a eeg; dr's said he has encephalopathy. He is still paralized; however; he responds with his eyes. what is the chances of recovery? Doctor: HIThank for asking to HCMI am really sorry for your nephew's health issue, encephalopathy means the ill-health of brain and here the cause of this may the internal bleeding that have form the clot the entire process of recovery is depends upon the severity of disease and that include the size of clot and the damaged done by it to surrounding tissues although the chances of recovery said to be minimal because the time has gone away, as the date mentioned hare, but lets hope for the good, the perfects judgment demand the clinical examination, so this is just opinion based on history given here which could be differ after seeing the patient clinically, hope you can understand this have nice day."
},
{
"id": 191557,
"tgt": "Can injecting 200 units of Tresiba lead to over-dosage?",
"src": "Patient: Hi, I have been using tresiba 100 units/ml pens and reached the maximum dose. My Dr. Wrote my new script for the 200 units/ml pens. Will I reduce My dosage by half? My Dr prescribed me to inject the same units nightly but the pharmacist said that was double the amount! I m confused! Doctor: hi sirTresiba is a long acting human insulin suggested with personal of highly diabetic conditions it controls your blood sugar better but taking it once a day with be advisable no need of taking up at night time Since it's a longstanding human insulin it may lead to low sugar levels also better to consult the diabetologist about this and get a opinion. make sure that your HB a1c is under control and you have to take insulins according to that. thank you."
},
{
"id": 82690,
"tgt": "Could neck lump pain, muscle and joint pain be due to lupus?",
"src": "Patient: hi for a while now i ve been having many symptoms that first began with neck pain a while back (2 years ago ) with lump not visable (to the touch ) recently i ve been having ( ankle knee elbow back shoulder wrist finger neck hip joint pain and calf arm thigh etc. muscle pain mri cat scan and emg came back normal blood work came back abnormal liver functions insomia fatuige headaches ( doctor said about mid range ) tested ana came back postive then had a ra factor neg. then a ana profile (FANA was 1:80 speckled ) i think sedrate and crp was normal my question is what are the possiblities that i could have lupus or another autoimmune problem if its possible what should i consider to my doctor to test for more info : im a white male age 21 pain would be a 9 (chronic ) more than few months Doctor: Hi,Thank you for your query. I can understand your concerns.Antinuclear Antibody (ANA)-Primary disease\u00a0\u00a0\u00a0\u00a0\u00a0association (sensitivity, specificity):SLE(>95 %,low).often used as screening test; a negative test virtually excludes SLE; a positive test while non-specific, increases post-test probability of SLE. Titer does not correlate with disease activity.\u00a0\u00a0\u00a0\u00a0\u00a0Other disease association with positive ANA:rheumatoid arthritis(30-50 %),discoid lupus,scleroderma(60 % ),drug-induced lupus(100 %),Sjogren's syndrome(80 %),miscellaneous inflammatory disorders.Do test for Anti-double- stranded DNA/anti-ds-DNA (sensitivity, specificity):SLE ( 60-70 %,high )& Anti-Smith antibody/anti-Sm (sensitivity, specificity):SLE (30-40 %,high ).The specificity of Anti-smith & Anti-double stranded DNA for Systemic lupus erythemtosus (SLE )ranges from 55 -100%.Regards Dr. T.K. Biswas M.D.Mumbai"
},
{
"id": 157779,
"tgt": "On Alaxan for abdominal pain. Now diagnosed with ovarian cancer. Still continue Alalxan. Affect kidney?",
"src": "Patient: hi, i am taking alaxan for 4-5months already coz before i have a abdominal pain so every time i take alaxan the pain is gone...then i was just diagnosed that i have ovarian cancer so i still continue taking the alaxan for my pain, i am taking 3x a day..is it bad for me? is it true that it will affect my liver or kidney?d Doctor: Hi, Thanks for the query. Alaxan contains two drugs, paracetamol and ibuprofen. Potentially they can lead to kidney damage if used for prolonged periods in high doses. Pain of malignancies is severe and there is a need for pain killers. Are you on any chemotherapy? As chemo usually kills the cancer cells and decreases the pain so the need for painkillers decreases. Try to decrease the dose of Alaxan from 3x to 2x and than 1x. There are other group of drugs called opioid analgesics that decrease the pain and can be obtained on prescription by cancer patients. Try other measures like meeting friends and family that help distract from pain. Also other therapies like acupressure, massage etc that help to bring down the pain and need for pain killers. Thanks again and have a good day."
},
{
"id": 128460,
"tgt": "What causes recurring muscle spasms?",
"src": "Patient: I have been having muscle spasms for over a tear. Within the last 4 months, they have gotten real bad, several each day and night. They are not in the same location every time, and sometime I ll have them in 2 or 3 different places at the same time. I ve not changed any medications lately, and blood work shows everything is normal. None of my doctors know why. Any ideas? Doctor: hello, Make sure that your doctors have checked your magnesium levels and potassium levels. Those could be supplemented if they are low. Three things you may try to improve this situation is making sure you are well hydrated (that means avoiding things like coffee and alcohol that are dehydrating), take a multivitamin/multimineral supplement daily and do muscle stretching daily (yoga is a good option).Regards"
},
{
"id": 47612,
"tgt": "Is it normal to have bloating while on Cipro for kidney infection?",
"src": "Patient: I have been diagnosed with a rather serious kidney infection and have been prescribed 1000mg of cipro a day for ten days. My stomach is extremely bloated and it burns when I urinate. I have not had a bowel movement for 9 days. I have liver disease from hepatitis c and I can barely walk at this point and feel extremely emotional and acting strange. Is this normal? Doctor: Hi, welcome to HCM. Urine infection requires evaluation to determine site if infection and organism.Basic urine report with culture, S.creatinine and sonography help us in that.Ciprofloxacin generally is not associated with such severe constipation and gi trouble.Consult gastroenterologist for that.Maintain adequate hydration. I think this would be helpful to you. Best wishes. tc. Dr Jay Patel."
},
{
"id": 175985,
"tgt": "What causes sweet smelling urine and hair loss in a 5 year old?",
"src": "Patient: My 5 year old has a sweet smelling urine at times, and I don t think this is normal. Now I noticed a lock of her hair on the floor, and she said she didn t cut it.? I m not sure if something happened to her hair at school or if it would just fall out like that. I now wonder if the smelling urine and hair loss could be a symptom of something else. What could this be? Doctor: Hi...by what you quote I feel that this is fitting into a condition called Maple syrup urine disease. This is not a life threatening condition, but definitely needs evaluation. i suggest you get back to your pediatrician and get your daughter evaluated.regards - Dr. Sumanth"
},
{
"id": 57159,
"tgt": "Suggest treatment for liver cirrhosis and hepatitis c",
"src": "Patient: Hello Dr , am an Egyptian citizen and my 65 years old dad suffers from Hepatitis C , Liver cirrhosis and a two cm focus suggested to be Hc with past history of heart attack. We have been searching for the perfect procedure to treat him but doctors can't define which really suits his case . It would be very great if you checked the meassage and sent me back. If you Dr need more info or reports i'd be gratefully sending it to you by mail if you wrote it for me Thanks alot With regards Engy Sawah YYYY@YYYY Doctor: Dear Mr Engy,How are you? I am so sorry to hear about your father's diagnosis. Hepatitis C and Cirrhosis are both risk factors for development of cancer of the liver (HCC). The treatment options for someone with HCC depends on multiple factors 1) Patient condition (age, physical fitness) 2) whether the liver is healthy or not (normal liver vs cirrhotic liver) and 3) when patient has cirrhosis, the clinical stage of cirrhosis (early, asymptomatic vs late stage cirrhosis with fluid in belly). Normally upto 70% of liver can be surgically removed safely without any problems for the patient. But in a patient with cirrhosis, only 30-40% can be removed, that too, only if the patient is asymptomatic and his blood tests are normal. In a patient with abnormal blood tests or fluid in belly or portal hypertension (vomitting of blood), then surgical resection is totally contrindicated. In case of your father, I would like to know more about his blood tests, whether he has any symptoms and also the size of the HCC. Based on these the treatment options may include surgical resection, Radio frequency ablation (Burning of the tumor using probes), TACE (blocking the artery supplying the tumor) or even liver transplantation (age and previous heart attack make him a poor candidate.Please do contact me with his blood and imaging reports so I can guide you to the best possible treatment - rxsuresh@gmail.com.Hope this helps and hope your dad gets to feel better."
},
{
"id": 168667,
"tgt": "What causes bumps and swelling on ear?",
"src": "Patient: my 7 year old had an ear infection 4 wks ago, it has healed fine. She recently was bumped on the same ear and began to swell behind the ear. The swelling was getting better, but then again was bumped and the swelling came back. She has had no fever, no pain, unless it is dirrectly pushed on What is this? Doctor: Hi,It seems that due to ear infection there might be having post auricular enlargement of lymph node producing bump.As there are no signs of active infection wait for some time, it will be alright.Ok and take care."
},
{
"id": 19601,
"tgt": "Suggest treatment for high blood pressure and headache",
"src": "Patient: I am 63, in good health. I just got over the flu (finished taking Tamiflu yesterday) and perhaps strep. I am halfway thru my Amoxiccillin for strep. I thought I was well but noticed several times during the last week I had a headache. Tonight I couldn't sleep because of the headache and became suspicious maybe high blood pressure was the cause. Sure enough, the average of 3 successive readings over half an hour was 162/96. My pressure is usually in the normal range. Doctor: Hi, thank you for querry,As you mentioned you just complete your antiviral course and continuing the strep treatment.as well high blood pressure is concerened its higher reading according to age you must start calcium cahnnel blocker like amlodipine 5 mg at starting dose, titer your blood pressure and if its not in controll consult your dr.life style modification along woth diet modification reduce weight,daily exercise,less salt fat and sugar diet.hope the answer will help you to improve your health."
},
{
"id": 186454,
"tgt": "What is that small white clusters in my mouth?",
"src": "Patient: Hello? I have a question about very small white clusters if tiny bumps in the corner of my mouth. They have been there for over a month and dont hurt. Only now a notice larger pimple like bumps extending up my lip just on the line where the fry part of the lip meets the insider (moist) part of the lip. Doctor: Hi! Welcome to Healthcaremagic.I read your query. White cluster at the corner of mouth can be due to various reasons such as accidental biting, fordyce spots due to sebaceous glands, oral herpes, allergic reaction, or other severe conditions.Since you mentioned that there are no symptoms associated. Even then I suggest you to once visit a dentist and get them checked. If they change shape, size or texture, go for biopsy.In case of history of any medication causing allergy, contact your physician before stoping or changing drug. Herpes infection takes usually 3-4 weeks for complete healing.Hope the answer helps you. Thank you!"
},
{
"id": 139991,
"tgt": "What causes sudden onset of headache with leg numbness?",
"src": "Patient: I know someone is has just recently started getting a bad headache with a quick onset and when he gets them, his legs go numb and just give out. He has fallen once or twice because of this. He has seen his IM doctor and now is going for an MRI - they are looking at his pituitary gland. Can a sudden onset of a headache cause his leg numbness and weakness? His headaches do not last more that 10 - 15 minutes. Doctor: Hi, First of all, a headache is a symptom and the causes of the headache can cause leg numbness. For example atherosclerosis, smoking, drinking, endocrine disorders and kidney diseases can cause headache and leg numbness. Second, he should measure his blood pressure and if it were high, he should receive a medical treatment and stop eating too much fat and salt. Third, the women who take oral contraceptive pills or a hormonal replacement therapy can suffer from vascular problems, so they should ask a gynecologist to adjust the dose. Finally, the deficiency of vitamin B complex due to unhealthy lifestyle can cause headache and numbness. Hope I have answered your query. Let me know if I can assist you further. Regards, Dr. Mustafa, Neurologist"
},
{
"id": 58978,
"tgt": "Have gallbladder problems. Back pain, constipation and gas. Taken paracetamol. Suggestion?",
"src": "Patient: right side pain before 2 years.this pain increase last night 'back pain and gas problem immediately' sometime' i can face constipation .my weight 65 kg.my age 24.my individual problem.i will eat paracetamol 'plz suggest some medicine.first time or primary step u suggest me.i m a pharmacist.i can guess my problem is gallballader 'paincillar means?how much in the day?u suggest me.and give me good suggestion.plz helip me. i am pregnant women . Doctor: Hi, You have pain on right side and back, weight 65 kg, age 24, You are a pharmacist, and used to take paracetamol.you are pregnant. Is it necesory to take a pain killer regularly, and upto the maximum tolarance, being pregnant you should stop taking un necessory drugs, You have pains avoid taking potato, other tubers, tomato, egg, chicken, and sea foods. Have a nourishing and balanced diet. Thank you."
},
{
"id": 136434,
"tgt": "I RA causing pain in arm on extending?",
"src": "Patient: I m a 47 year old female,I was diagnosed with rheumatoid arthritis in 2012.About 6 months agoI began having problems extending my right arm.I do have an old injury,I broke this arm in 4 different places at age 7.Today I noticed a lump in the crease of the inner part of the same arm. What is going on,this is really beginning to frighten me. Doctor: Hello, I have studied your case with diligence.RA may lead to osteoporosis. Lump can be rheumatoid nodule leading to inflammation.Check your bone strength with DEXA SCAN and you can start medication according to level of osteoporosis,You need to start supplement of calcium with vitamin D, vit B12, I will advise you to do regular physiotherapy and exercises as my patients find much relief by these therapies.You need to take diet rich in calcium and vitamin D.Hope this answers your query. If you have additional questions or follow up queries then please do not hesitate in writing to us. I will be happy to answer your queries. Wishing you good health.Take care"
},
{
"id": 34957,
"tgt": "What could be the reason for having pain in shoulder even after taking medicines?",
"src": "Patient: My husband had an ecoli infection in his blood, his kidneys shut down, high fever. We went to the ER and spent 5 days in the hospital being treated with massive amounts of antibiotics. He has had left shoulder pain which goes into his upper back, up his neck and into the base of the skull. On a scale of 1-10 this is above a 10. This has been going on for 12 days now with no relief even when on pain meds. Any idea what this could be? Doctor: hellowelcome to HCM.if it is a post infectious artritis it would be gone after NSAID drugs.i think this may be from impingement syndrome..you should consult an orthopaedician for it.proper clinical examinations and investigations are needed.hope this is helpful.regards."
},
{
"id": 27961,
"tgt": "What is the next procedure based on the CT Coronary Angiography report?",
"src": "Patient: HI, i have undergone PTCA in Jan 2011, recently i had done C T Coronary angiography done and the report is as below:64 SLICE C. T. CORONARY ANGIOGRAPHYOBSERVATION :- Coronary CT Angiography was performed on 64 slice CT rendered images. Poor imaging due to poor breath hold, hence multiple step artifacts are noted.CALCIUM SCORE :- LM - 0, LAD - 0, LCx - 0, RCA - 0. Total = 0.LEFT CORONARY ARTERYLM :- NormalLAD :- Stent noted in proximal LAD. Stented segment show calcific plaque causing moderate narrowing. Soft plque noted in prestented LAD causing moderate narrowing. Post stented segment also shows soft plaque causing significant narrowing. Distal filling is not satisfactory.D1 :- NormalSEPTALS :- NormalCIRCUMFLEX ARTERY :- Soft plaque noted in LCx causing moderate narrowing. Rest of the LCx appear normal.OBTUSE MARGINAL BRANCHESOM1 :- Poor opacification.OM2 :- NormalRIGHT CORONARY ARTERY :- NormalPDA :- Normal and arises from RCA.scanner. Plain and post contrast images were obtained. Thestudy was reviewed in multiplanar reformatted and volumeCOMMENT :- Total Calcium score = 0 Moderate and significant disease in LAD. Soft plaque noted in LCx. Right dominant circulation. Suggest : Catheter Angiography.Can you please suggest what would be the next procedure. Doctor: This report suggests that you have instent restenosis.the very fact that u underwent CT angio\u00a0suggests that you have symptoms of angina.I strongly recommend that you should undergo cath angio and confirm degree of block in your LAD stent.depending on reports you may need further intervention or modification of medicines.my best wishes."
},
{
"id": 1699,
"tgt": "What causes difficulty in conceiving in spit of thyroid being normal?",
"src": "Patient: hi iam latha. iam 23 years old just i 11 months back i married after 4 months on that time iam conceiving 2nd month but unfortunately abortion to me on that 2nd month. on that time this time iam not conceivig after my docor check thyroid test it is is normal .so my doctor suggest letroze 2.5mg ,2nd day to 6th day of period now 25th day to me i check pregnency test today early morning it is negitive. so what happen to me? Doctor: Hi, it is very early to test for pregnancy. If your periods get delayed, then do a pregnancy test. If positive, consult a doctor. If negative wait for your periods. You can try naturally for 6 months to 1 year. Don't rush. The chance to conceive in a normal couple in one cycle is only 5 percent. So, it may take 1 year for you to conceive again. Hope I have answered your question. Regards Dr khushboo"
},
{
"id": 33980,
"tgt": "What causes clear discharge through navel with bad odor after a tummy tuck?",
"src": "Patient: I had a tummy tuck about 10 years ago. From time to time my belly button would get a clear discarge with a smell. I would give it air and clean it out. Went away no biggie. MY belly button has also been numbed since that surgery. Had my gallbladder out last year. Ok so now lots of pain inside the belly button and it seems like the pain goes into my stomach. If I push out or suck in the pain gets worse. Please help Doctor: i will advice you to get examined by a surgeon and let him see what is the wrong with it .some times it is just remnant latent slow infection which has to be treated ."
},
{
"id": 46004,
"tgt": "What are the consequences of donating a kidney?",
"src": "Patient: Hi.My name is Dhirender Bisht I AM HANDICAP PERSON I AM 46 YEAR YOUNG MAN .I HAD TRAIN ACCIDENT 5 YEAR AGO.SIR I DECIDE I GIVE MY KIDNEY TO KIDNEY FAILURE PARSON.SIR TELL ME IF I DO THIS WORK SHELL I FACING WITCH KINDS OF PROBLEM OR NOT .PLEAS GIVE ME RIGHT SUGGESTION.THANK YOU SIR Doctor: Hello Dhirrnder, I really appreciate your thought and intention. Successful Kidney transplants are carried out all over the world today. For any transplant program, the utmost priority is the healthy donor. For kidney donation, firstly you will be thoroughly examined and investigated to look for any subclinical kidney disease or any risk factors (diabetes, HTN) predisposing you to future kidney problems.Now lets assume that you are completely healthy and donate one of your kidneys.Following are the precautions you ought to take in your future1. Routine blood tests ( kidney function, urine)2. Regular monitoring of your blood pressure 3. To avoid any addictions (alcohol, smoking)4. To avoid taking any medications in excess that are harmful for your kidneys (pain killers, nephrotoxic drugs)5. Lastly, remember you will be having a single kidney, so in simple ways if at home you have a electricity shut down, you switch over to back up. In the same way if you have some infection in future, you will have only one kidney to fight. Overall, it is one of the safest organ donations with little surgical or post surger complications till you maintain a healthy lifestyleFeel free to post any further queries Thanks"
},
{
"id": 182060,
"tgt": "Suggest treatment for severe tooth pain",
"src": "Patient: I have to have two root canals but since I'm still in the wait zone with my dental insurance I have to live with this terrible pain for at least another six or so months. I've asked for pain medicine and was given one prescription that lasted ten days, no refills, and was told that their rules say they can't prescribe more until I have the surgery. What do I do about the pain? At first 800 mg ibuprofen helped a little, but it's gotten worse and nothing is helping. I'm allergic to tramadol which was their only alternative for me so that's out. Please help Doctor: hi if you are having severe pain with a due rct ,it it due to the pressure exerted by the pus formed ,which should be immediately treated upon because it has a direct risk of spreading towards your bony underlying tissues if not treated soon. it should'nt be delayed for the time you have mentioned . antibiotics and pain killers are taken for a while but even their usage cannot be extended for a prolonged period of time . besides they have their own side effects . I ADVICE YOU TO GET THE RCT DONE AS SOON AS POSSIBLE and not to wait for so long for the dental insurance . Remember health is wealth !!take care ."
},
{
"id": 214317,
"tgt": "Suggest natural treatment for the semen leakage",
"src": "Patient: Sir my name is Mohit.i m of 21.Sir i started doing masturbation when i was in mid 19,with twice a day for around half year or so.But now i left it completely.But i m facing the problem of Semen leakage..could u plzz guide me wht should i do to overcome it,can u plzz tell me ant natural treatment for it??Plzz sir i want ur help. Doctor: hello mohitthanks a lot for your query...dont be worried...there is nothing wrong that is happening to youit is quite natural for a man to have such a thing to happen,the sperms produced will be thrown out of the body on the regular basis...in your case as you np more masturbate, the sprems accumulate and cause its expulsion, which you are seeing as leakage...do be calm,dont be worried..have control on you..things will improve gradually..."
},
{
"id": 135943,
"tgt": "Suggest treatment for bone fracture in the legs",
"src": "Patient: Hello, my question is about detoxing off of Methadone. I have been on it for 5-6 years. I was on a pretty high daily dose of 160mgs, and tapered myself down to 40mgs a day. I then got into an accident at work resulting in double compound fractures in both legs. In the past year and 2 months I have had nine surgeries. I had to go up on my dose for pain because I could feel no other pain medications at all. I went from bedbound, to wheelchair, to crutches, but now i m finally walking. I still have a lot of pain. I received a disability ratting of 45percent for my left leg. That is my worst leg due to my bone pertruding through my skin, and lost 4 of bone on site. My right leg got a 15percent rating. I tapered back down to 40mgs, but missed a bottle recall, which meant I would go from having to go to the clinic from once a week to six days a week. I am not able to do that so I just stopped going four days ago. I wanted to know if there is anything I can do, or take to help with the symptoms of coming off of 40mgs, to nothing? My doctor is trying to figure something I can take for pain that s a non-narcotic, from my request. Is there any tips you could give me that might help from these painful side effects I am already experiencing ? Doctor: hiYour treating doctor is the best judge and i may suggest tramadol for pain may be tried.rely more on physical therapy instead of medicines aloneConsult ann orthopeic surgeon for reviewbest wishesthanks"
},
{
"id": 207886,
"tgt": "How to control extreme emotions causing anger and raise in heart rate?",
"src": "Patient: I've always been easy agrivated. If im doing something and someone intervenes on me doing what i was doing i get very angry. My heart races to the point to where i can feel it pounding in my chest. I get really hot. It happends all of the sudden, i dont even realize it sometimes. That lasts for about 15mins, if im by myself. If the person is still in the room it lasts much longer. Lately ive even become emotional over little things. I try then to talk about it but i cant explain. So then i get in an argument with that person because i cant explain and i get angry because i cant get them understand. I get angry to where im yelling, crying, shaking and my chest is pounding. When i calm down about 30 mins later, I can go on throughout my day like nothing ever happened. This isnt me, ive always been a naturally stressed out person but ive never been had these many emotions impact me so hard. My \"outbursts\" are happening more frequently. Help? Before i lose my love. Doctor: DearWe understand your concernsI went through your details. I suggest you not to worry much. This is what aptly called EGO. You think you are always right. Are you right? You think other person know less than you. Is it correct? You should think that your opinions are as good as others. You are right as others are right. You are wrong as others are wrong. You have no existence if others don't agree with you. Same is applicable to others also. That is why sages say \"destroy your ego\".If you require more of my help in this aspect, Please post a direct question to me in this website. Make sure that you include every minute details possible. I shall prescribe the needed psychotherapy techniques which should help you cure your condition further.Hope this answers your query. Available for further clarifications.Good luck."
},
{
"id": 3647,
"tgt": "Can i get pregnant again after abortion 7 years ago?",
"src": "Patient: Hi doctor I had an abortion 7 years back when I was 16 unfortunately ! and since then I get yellowish white discharge , now I am getting married and want to have a baby am I at risk ? nobody knows about this abortion of mine , I am upset because I love babies , can I get pregnant again? Doctor: Hi, Thanks for writing..If you have resumed your normal periods and had no infection following abortion, there is no reason for thinking that you will not conceive.. Medical abortion had lesser chance of infection as compared to surgical method. Since you have an yellowish discharge get a swab done to see for vaginal infection and take antibiotics according to the sensitivity.. Hope I have answered your query.. Good day.."
},
{
"id": 64562,
"tgt": "What causes lump on lower jaw?",
"src": "Patient: I have a hard lump on right side lower jaw just above where my lower teeth finish, I went to a dentist they said it was my glands, I then went to my doctor and he said it was a stone in my glands had antibiotics and still there, got refered to ent and she said she don t think it is a stone and I m currently waiting to have a ultra sound and is getting worrying now?? Doctor: Hi..Can understand your concern..As per your complain a lump on the jaw can be due to a number of reasons and also depends upon the time duration for which they are present..A lump can be due to an infected tooth leading to pus formation near the root tips leading to formation of a lump (Dental abscess) as pus accumulates in the soft tissues..In this case treatment of the tooth either by root canal treatment or extraction can be done..It can also be due to a blocked salivary gland duct leading to back flow of saliva into the gland and appears as a swelling just below the jawline as well as in the floor of the mouth..It can also be due to inflamed lymph nodes due to infection in any part of the body and the lymph carries the infection along with it to the distant lymph nodes.It can also be cyst (jaw cyst, sebaceous cyst) or a tumor.In few long standing cases it can be a cancerous lump too..I would suggest you to wait till the ultrasound report is given and need not to pain as the jaw lump can be most commonly due to non cancerous causes and can be nothing serious..Treatment can be planned only after the diagnosis is confirmed..Hope your query is solved..If you find the answer helpful please write a positive review and click on find this answer helpful as a token of appreciation..Thanks and regards..Dr.Honey Nandwani Arora."
},
{
"id": 216806,
"tgt": "Suggest treatment for the pain in my left side",
"src": "Patient: I have a 13% effective gallbladder and am on a low fat diet. Wednesday I started having an attack and I m still having it...only this time I have terrible pain on my left side which I haven t had before. Of course I also have back pain in addition. Is it something other than a gallbladder attack ...kidneys maybe? Doctor: Hi thanks for asking question in HCM.Here your pain could be by gall bladder inflammation like cholecystitis if fever also present as it can reffered to back pain.The pain could be by severe colitis like condition if abdominal cramp present ...USG done For it.Antibiotic need for it.If only abdominal pain present then ulcer could be cause.omeprazole taken for it.If it is stone in kidney then pain us constricting and intolerable..USG useful for it.....To rule out pancreatitis do serum lipase and amylase estimation if doubt regarding it.I hope your concern is solved ....Take care....Dr.Parth Goswami"
},
{
"id": 24550,
"tgt": "What could sensation of being shot in the heart and subsequent bruising indicate?",
"src": "Patient: My sister said while she was in bed last night, she felt like she received a shot into her heart, with the feeling of a warm fluid being injected. She awoke with a bruise this morning, and what appeared to be a red mark...like an injection site. She had a heart attack several years ago at age 38. She thinks this incident was strange, but she is not concerned. Should I alert her to call her cardiologist? What could it be? Thank You! Doctor: Thanks for your question on Healthcare Magic. I can understand your concern. Bruising, red spot on chest with shooting pain are suggestive of herpes zoster (virus) infection more. But since she had heart attack in past, better to rule out heart related chest pain first. So consult cardiologist and get done ecg and 2d echo to rule out heart diseases. If both these are normal then no need to worry for heart diseases. Herpes zoster infection causes similar kind skin lesion and chest pain. So she will need acyclovir tablet and pregabalin tablet for her symptoms. So consult dermatologist and discuss all these. But first rule out heart diseases. Hope I have solved your query. I will be happy to help you further. Wishing good health to your sister. Thanks."
},
{
"id": 161672,
"tgt": "What does red nose in a baby caused after falling down indicate?",
"src": "Patient: my baby fell off the bed onto his nose - he had a slight nose bleed which has stopped now but his nose is very red. should I go to the hospital to check it out. he s napping now so I can t check for symptons such as different behaviour and he didn t seem to have any other injuries. thanks. Doctor: Hello, You are quite correct. The most important feature to look for is any change in behavior: excessive sleepiness or fussiness, vomiting, eyes looking in different directions, or unequal size of pupils. It is normal for there to be redness at the site of injury. This will fade in several days. Unfortunately, this happens frequently and is no reflection on your parenting. Hope I have answered your query. Let me know if I can assist you further. Regards, Dr. Arnold Zedd, Pediatrician"
},
{
"id": 115719,
"tgt": "What could cause fatigue with a WBC count of 12,400?",
"src": "Patient: Female, 28y/o, 5'4\", 97 lbs (from 105 lbs mid-2010, in spite of good appetite). ER 3 days ago due to HR 136. BP 120/90. Fatigue for the last 3 weeks despite good sleep. EKG & CBC okay. WBC 12,400. Took 2 cycles of contraceptive pills without 7-day break. Last period 2 wks ago, after last pill. Doctor: Hi, dearI have gone through your question. I can understand your concern.You have slightly high WBC count. It may be due to some infection or may be as normal variation. Your hemoglobin and blood pressure is normal. No need to worry babout that. Just consult your doctor and take antibiotic treatment if required. Hope I have answered your question, if you have any doubts then contact me at bit.ly/Drsanghvihardik, I will be happy to answer you.Thanks for using health care magic.Wish you a very good health."
},
{
"id": 179679,
"tgt": "How to cure phobia to bugs in a child?",
"src": "Patient: My 5 year old daughter has become increasingly more afraid of bugs overtime. Within the past month, the episodes have resulted in her trembling and today, hyperventilating. How do we help her through this? And how do we get her out of a hyperventilating state? Doctor: Hi...this seems to be a phobia towards insects and bugs. As it is going to the level of hyperventilating, she needs a loving talk that bugs are not dangerous and if not better please approach a psychiatrist.Hope my answer was helpful for you. I am happy to help any time. Further clarifications and consultations on Health care magic are welcome. If you do not have any clarifications, you can close the discussion and rate the answer. Wish your kid good health.Dr. Sumanth MBBS., DCH., DNB (Paed).,"
},
{
"id": 2918,
"tgt": "Do tablets Endogest 200 and Lupi hcg 5000 help in conceiving?",
"src": "Patient: Hi...now i am 22...i get married at dec,13..then am getting period regularly..i am trying to get pregnancy....we consult the doctor she prescribed me to {Evion 400mg,Ebexid,M2 tone syrup}when i take this medicines i got 3 days later about my period...again we consult our doctor with my scan report......doctor says scan reports are very well..then she prescribed me to{Endogest 200 for 20 days,Lupi hcg 5000}now i get this tablet.its sure to give any positive results for us? Doctor: Hello and Welcome to \u2018Ask A Doctor\u2019 service. I have reviewed your query and here is my advice. The fact that your doctor has commented that your scan reports are good means that you are ovulating regularly. Endogest is a natural micronized progesterone preparation. After fertilization, progesterone is required to stabilize the pregnancy. This progesterone is released by Corpus luteum. If corpus luteum is weak, the pregnancy may not continue due to shortage of progesterone. Therefore you are given these pills. Moreover, Inj. hCG helps strengthening of corpus luteum. So it will be useful when the fertilization has taken place. If you miss the period, perform your pregnancy test on the overnight morning first urine sample, earlier the test may report false negative results. Alternatively you may perform Beta hCG test on the blood 10 days after the last unprotected intercourse. Now that the other medicines like Evion, Ebexid, M2 are not continued now, we need not discuss on them. Moreover, if you are ovulating naturally, you do not need these medicines. I hope this explains you about your medicines. Hope I have answered your question. Let me know if I can assist you further.Regards,Dr. Nishikant Shrotri"
},
{
"id": 177174,
"tgt": "Is it normal for the formation above the top surface where the tooth will pop?",
"src": "Patient: my son is almost 1 yr in 1 wk and he has ben teething since 3 mnths so now has alot. my quistion is is it normal for light pus to surface on top where tooth will pop? it is a bigger bottom middle tooth and it sure has ben giving trouble for awhile. it has shown signs of swolloness for 2 mnths and in that time 4 other teeth came through before this one. his appitite is not that great n he just seems out of sorts Doctor: Hi...what you say is not normal. it suggests that he has gingivitis or infection or inflammation of the gums. I suggest you see your pedo-dontist immediately. Regards - Dr. Sumanth"
},
{
"id": 182585,
"tgt": "Can an enlarged node in the neck relate to gums problems?",
"src": "Patient: for about 7 months ive had enlarged node in the left side of my neck and a slighty smaller one on the right side of my neck just abit above my adams apple and another one just under my ear on the left hand side ive been to me gp 4 times had 2 bloodtests and an ultrasound scan which have all been fine but i am still abit worried, also the dentist told me i have a gum diseaese, just wondered if you knew what the cause of the enlarged nodes could be. Doctor: Hello!As per your history and investigations reports the cause of your cervical lymphadenopathy is some benign condition. The primary location might be in mouth and throat. If you are some some long standing tooth or gum problem, it leads to the primary cause. Visit your doctor for treatment of primary cause and also go for check up for regular interval.Regards!"
},
{
"id": 141428,
"tgt": "What is mild generalized atrophy?",
"src": "Patient: Hello I just had a baby with full trissomy 18 and she was submited to a Transfontanellar ultrasound (is that the word in english?) that showed a mild generalized atrophy in the brain. What does that mean? Can she have an independent life? I imagine she ll have cognitive and motor problems becouse the syndrome that she has doenst have a good prognosis but with estimulation will she be able to do things by herself? Talk? walk? the douctor said, to her surprise, that she\u00b4ve seen worst... and kinda let us belive that that was really possible in her case. Thank you so much... Doctor: Hello and Welcome to \u2018Ask A Doctor\u2019 service. I have reviewed your query and here is my advice. Regarding your concern, I would explain that mild generalized brain atrophy, means that there is a mild shrinking of the brain. This is usually related to low development or aging in older patients. But her development does not really depend on this finding. I would encourage you to try to talk and search for professional help , which can help improve her development (there are many professional institutions who deal with patients like her). Hope I have answered your query. Let me know if I can assist you further."
},
{
"id": 37751,
"tgt": "What causes bump in pubic area?",
"src": "Patient: Hello I just noticed I have a bump in my pubic area. It s about the size of a kidney bean. I haven t shaved lately. My boyfriend and I just started having sex again after months of no sex because we had a miscarriage and D&C. Do you know what this might be? Doctor: Hello,Welcome to HCMThanks for posting your query in HCM. Bumps in pubic area are very common because of increased hair present in this region.It might be just a folliculitis that is infection of hair follicle.It may also be a enlarged lymph node or blocked sebaceous gland present enormously in this area.I would advice you to keep the area clean and give hot fomentation to the affected area as frequently as possible.Pain and swelling will come down.If it doesn't get relieved,you need to see your treating doctor for physical examination.Hope you are happy with the answer.Thank you."
},
{
"id": 107670,
"tgt": "What causes back and abdominal pain while eating?",
"src": "Patient: Hi, I am 28 years old and was diagnosed with a hiatus hernia 3 years ago. I acid rflux most days and have inherited a noise which is like a squark. Over the last 6 month I have been suffering back pain everytime I eat. I attended a&e last week as they thought I had kidney stones however nothing came back. They said there was blood in my urine and I had low palette count. I get serve pains when eating pasta bread etc which stays for 3o minutes on a good day which I can t walk because of the pain. I also get pains in my chest which I worry about however the dull ache in my back after eating which sometimes gets worse with shooting pains to the abdomen. Any ideas please ?? :-) Doctor: Dear-thanks for using our system and will try to help you with my medical advise.You have hiatal hernia, which makes you prone for esophageal reflux.The acid can irritate your esophagus and give you abdominal and chest pain that mimics heart pain. You said that pasta agravates the pain and it might be because it is producing a lot of gas. You may need to identify what gives you more pain and eliminate it from your diet. This problem can be chronic acid irritation producing gastritis or an ulcer too. It is important to have a checkup with your doctor.I hope that my advise has been helpfulDr.Sara"
},
{
"id": 119220,
"tgt": "Anemia, excessive bleeding during periods, low grade fever. Blood test, low haemoglobin levels. Cure?",
"src": "Patient: Hello, my name is XXXXXX, I recently experience problem with my health. About 2 weeks ago I developed anemia after loosing a lot of blood during my menstruation . I was very sick then. I know my hemoglobin was low so I couldn t even walk without help. Since that I have low fever (37.4) and don t know if it s related to that. Can you tell me what could it be? Doctor: do you have any sore throat or amenorrhoea before blood loss ? after blood loss you have developed low grade fever.so there is a possibility of septic abortion.you need some hematological test like cbc,platelet count to exclude ITP(low platelet),sepsis(elevated wbc,high esr).if these results are inconclusive then consult with hematologist."
},
{
"id": 45255,
"tgt": "Aazosparmia due to incomplete sparmatocytic arrest",
"src": "Patient: Dear Concern Doctor , I am 35 years old and got married in 3 years back. We are trying to have a baby since last 3 years. But my problem are as follows: \u2022 Semen analysis report: Aazosparmia, \u2022 Testicular biopsy report: Incomplete sparmatocytic arrest. I am seeking for your valuable suggestion how can I get rid of this problem and be a father. Doctor: dear Doctor I'm 33 years old trying for a baby 4 years now had testicular biopsy report shows :spermatocytic maturation arrest is there any treatment that can improve my chances of becomin e father please help thank you"
},
{
"id": 47621,
"tgt": "What does this ultrasound regarding right kidney indicate?",
"src": "Patient: I recently had a high viral (105 F) fever and preferred to get myself admitted to hospital. I was tested NS1 positive with platteletes count above 0.1 million always. No bacteri growth or any signs of infection observed in blood culture report. Medical history: I was diagnosed with type 2 diabetes a year back in a routine test. I had controlled it with regular exercise routine. For past few months ( around 3 months ) I was not following routine and the A1C came out as 8.4. After which I have been advised to take glimy 1mg. Towards the end of my stay I was asked to take an ultrasound of abdomen. In the result there were two findings.. 1. Fatty hepatomegaly 2. hyperechoic heteroechoic lesion measuring 4.6 \u00d7 4.2 cm, noted in the mid pole of right kidney. Please help me understand this second observation on right kidney. My Age: 35 years Weight : 86 kgs Food Habit: Non Vegeterian Thanks in advance A.K Doctor: HelloThanks for query .Based on the radiological(USG scan) findings that you have stated I would state that you have what is called as Angiomyolipoma of the kidney which needs further evaluation .Please consult qualified Urologist for clinical examination and get following basic tests done to confirm the diagnosis.1) Urine routine and culture.2) Ultrasound scanning of abdomen and pelvis3) CT scan of the abdomen and Pelvis 4) MRI (if required). Further treatment(if required ) will depend upon result of these tests and final diagnosis.Dr.Patil."
},
{
"id": 1160,
"tgt": "What are the chances of pregnancy if husband in on Lisinopril?",
"src": "Patient: My husband who is 52, recently had a heart attack is on many different pills including Lininopril, he drinks and smokes (we know it's bad he is trying to quit) but we are also trying to have a baby. What are our chances? Should I just give up and find someone else? Doctor: Hi , How are you doing ? That's lot of risk factors, age , drinking & smoking. Lisinopril will not significantly affect the pregnancy chances. But smoking & drinking is going to be risky for future repeat heart attack & will seriously affect pregnancy chances , due to poor semen quality.Once these avoidable risk factors are removed fully, there is no partial stopping, semen will definitely improve. Kindly see if the following checklist of investigations is doneHusband- Semen analysis at least 2- done 4 weeks apart preferably in an infertility center ( examination done by andrologist), Blood sugarsWife - Blood Sugars, Thyroid profile Ultrasound scans to evaluate Uterus , Ovaries & adnexa Confirm tubal patency- most important- Tube is the connection between uterus & ovaries( the bridge where sperm meets the egg & forms a baby) this is size of hair follicle & cannot be seen on scans. Being such a small opening tube can get blocked easily, So check with HSG( Xray with dye), or SSG ( Scan with dye) or LaparoscopyIf all this done & found normal , nothing can stop a pregnancy with God's willHope I have cleared your query, do write back if any more queriesAll the bestDr.Balakrishnan"
},
{
"id": 75502,
"tgt": "What causes skin peeling on hands after pneumonia?",
"src": "Patient: hi my dad was diagnosed with pneumonia about 1 months back. after the treatment, the doctor say that they are worried that he did not cough up much mucous. now the skin on the hands is peeling he is not under any medication. Can this be lung cancer ? my dad is 65 years old and is neither diabetic nor had blood pressure Doctor: The peeling of skin does not seem to be related to pneumonia . Similarly it`s difficult to say about lung cancer. Let the doctors decide the further course of action to find out the incomplete resolution of pneumonia."
},
{
"id": 196918,
"tgt": "Suggest effects of masturbation",
"src": "Patient: hello, I'm a female 31 of age and I'd like to ask about male masterbation. What are the possible disadvantages of frequent masterbation? My boyfriend who's 33 years old has masterbated over the last ten years every day and we're wondering whether this may affect his sexual health or sexual performance. P.S: We haven't had sex yet. Doctor: Hi welcome to the health care magic As such masturbation is pleasure giving practice and it is safe practice But according to few articles excess masturbation leads chronic fatigue, weakness, groin discomfort, social anxiety like problems So better to avoid excess masturbation Two to three times masturbation per week is safe Take care Don't worry If need further help psychiatrist can be consulted for examination and further discussion Hope this will help"
},
{
"id": 216551,
"tgt": "Suggest treatment for swelling and pain in face",
"src": "Patient: Hi Over the past 2 days I have had a sore jaw on the right side only. Doesn t feel sore on the outside of my face, but extremely tender on the tendon that attaches jaw together. Inside of cheek also appears swollen. Right side of tongue feels strange, and slightly sore to swallow also. I m 40 but my wisdom teeth never came in, could this be a cause? Doctor: Hello,I can understand your concern. Usually, wisdom teeth come in mouth at the age of 18 to 25 years. However, as you have never had them in mouth, it is possible that they might be impacted or locked in the jaw. As right side of jaw is causing you pain, let us focus on that only. Due to absence of physical space to come out in mouth or due to its improper position, it might be possible that the wisdom tooth of that side was locked in the jaw all this time and now might be impinging on the muscle of the mastication or nerve that supplies tongue causing you the pain and discomfort you are experiencing. It is also a possibility that a tooth might be decayed on the side of the jaw that is causing you problems, has developed an abscess that is causing these symptoms.In both cases, you need to visit a dentist to get an oral examination to find out if any tooth is decayed and taking x-ray of the wisdom tooth area to diagnose wisdom tooth related problem.By the time you et an appointment of the dentist, Ibuprofen 400 mg thrice a day will help you in getting relief from pain. Please refrain from hot water gargles or heat compression as if the pain would be abscess related, it would worsen.I hope this information helps you. Thank you for choosing HealthcareMagic. I wish you feel better soon.Best,Dr. Viraj Shah"
},
{
"id": 31579,
"tgt": "Could the discharge of green mucus after bursting of bertholin cyst be a matter of concern?",
"src": "Patient: I have had what I am sure is a Bertholin Cyst develop over last few days. This had now burst this morning to show a bright red discharge with green mucus. The pain I was suffering had now gone almost completley. Will I now be ok, or do I still need to see a doctor? Thanks Doctor: Hi, Thanks for posting in HCM. I understand your concern. Green mucus discharge following rupture of Bartholin cyst indicates that it would have been infected. It is good that it has ruptured and see to it that all the contents are completely drained out. Kindly take antiinflammatory medication like Tablet.Ibuprofen after food twice a day for 3 to 5 days. Continue with Sitz bath until the wound heals completely. Abstain from sexual activities till the lesions heal completely. Hope the information provided would be helpful. All the best."
},
{
"id": 156071,
"tgt": "Suggest treatment for ulcerative colitis",
"src": "Patient: I have a 25 year old daughter who is suffering from ulcerative colitis. She has just recently had a colonoscopy and has been taking humira shots for the last 6 mos for the illness. She is not getting any better and is struggling to go to work everyday. I don t know what to do to help her. Doctor: Ulcerative colitis can be controlled with diet changes to a significant level.Medication alone wont help.Try to observe which foods cause the flare and avoid them. Each person has different triggers.Reasearch on what diet will help control symptomsThere are other monoclonal antibodies available.Like cimzia .. Consult your doctor regarding a change in regimen."
},
{
"id": 76923,
"tgt": "Suggest diet plans after recovering from thyphoid",
"src": "Patient: hello,I am arpana of 21 yrs, i suffered from typhoid last month, though m putting weight but still i feel too weak, i am also having leg cramps. I live Hyderabad which is quite hot, but still i feel cold here, all day i feel sleepy and tired, i face has started looking too dull and aged.you please suggest me how to overcome with proper diet chart. Doctor: Hi thanks for contacting HCM...Noted you had typhoid infection...It lead to hyaline change in muscle and muscular weakness can be there....Gradually with proper diet it will recover...You have to take enough calories that is 2800 to 3000 calories per day....Milk , toast , mashed potato , lightly boiled eggs , boiled chicken , banana etc taken...Maintain hydration...Non veg and meat avoided.Small meal taken with frequent interval.In dinner include vegetable soup also...Avoid excess spicy and fried food....Hope your concern solved....Subsequentially HB estimation and electrolyte measurement done...Take care.Dr.Parth"
},
{
"id": 20625,
"tgt": "How safe is HCG drops in heart patients?",
"src": "Patient: 27 FEMALE CONGENITAL HEART PATIENT.135467856I HAVE PUT ON SOME WEIGHT AND IT S REALLY DIFFICULT FOR ME TO EXERCISE SINCE I ALSO HAVE A DISABILITY IN MY LEG THAT CAUSES SEVERE PAIN. I M ALSO A HEART PATIENT AND I HAVE A PACER. I WANT TO KNOW IF IT S SAFE FOR ME TO TAKE THE HCG DROPS FOR WEIGHT LOSS??? PLEASE HELP Doctor: Instead of taking drops why dont you start to decrease your food content intake slowly and start taking low caloric diet.there is a method of weightloss by calorie counting you can use that.download an android app for that which tells you the amount of calories in every single bite you take hence helping you to track your caloruc intake"
},
{
"id": 22046,
"tgt": "What causes heart palpitations whenever I bend down?",
"src": "Patient: i have had problems with my heart racing since i was 18. usually it seems to happen when i bend over suddenly & stand back up. it also happens for no apparent reason but mainly when bending down. is this a big deal? i feel like i am going to pass out when it happens, was watching my granddaughter yesterday & almost couldn't stop her from running in the street as my heart was doing it's racing thing & i was about to pass out. Doctor: hello, Hello, There are multiple possibilities like heart issues, positional due to sudden change in position or even anxiety related. But since symptoms are severe, we need to evaluate it. This can be a cardiac arrhythmia like psvt or atrial fibrillation, in which heart rate suddenly goes up and also suddenly decreases. Blood pressure falls during these episodes and one has dizziness, fainting, blackouts, sometimes syncope. Usually ecg is normal as it is usually done in between the episodes. ECG done during episodes will show abnormality. You will need 24 hour holter monitoring if symptoms are occurring in daily basis. So visit nearby cardiologist for these investigation.Also, hemoglobin and thyroid test should be done. ln order to abort such episodes you should try to cough hard.Is there any stress or anxiety associated. Till the time your evaluation is complete, you can ask your local doctor for tab ciplar LA 20 mg once a day. If none of the above is successful then last option is of electrophysiological studies which is invasive test which can diagnose and treat the arrhythmia permanently in most of the cases."
},
{
"id": 9564,
"tgt": "Dry rough itchy lips. Tried anti-itch creams and aloe vera. Need a permanent cure",
"src": "Patient: My upper lip is dry, rough, and itchy. Recently it has also been getting red. It has been like that on and off for almost a year now and I m not sure how to get rid of it. I ve tried anti-itch creams, aloe vera , petroleum jelly, etc. None of them seems to really moisturize it or get rid of it. I drink a lot of water and my diet consists of basically only fruits and veggies, so I m not sure what it is and how to get rid of it. Do you have any suggestions? Thank you! Doctor: Hi, Seems like you are having cheilitis(chapped lips). It could be due to frequent lip licking, cold weather, allergic condition, cosmetic induced,drug induced, nutritional, fungal infection, eczema etc. Use a fluticasone based cream along with a hydrating lip balm(neutrogena, blistex) over lips for few days. It will reduce dryness, itchiness and redness of your lips. Once you feel that your lips have become normal, stop fluticasone and continue with lip balm only. Use lip balms with good SPF.Continue with proper water intake and nutritious diet. Also do not forget to consult a dermatologist so that underlying cause of cheilitis could be elicited. Take care."
},
{
"id": 137879,
"tgt": "Can Diclofenac allergy cause body swelling with redness?",
"src": "Patient: I took 50 mg of diclofenac 2x a days for a year. No problems. My hip pain became severe and I resumed the diclofenac along with 4 g of voltaren gel one day x2. Went to bed & 6 hours later i awoke swollen, red all over. Clearly an anaphylactic reaction. Took benadryl which quickly subsided the symptoms. Now I m concerned that I may be allergic to diclofenac. Or could it be a reaction to just the voltaren? I called Novartis. They were no help & didn t really have any interest in recording my information regarding my reaction. Although I insisted they take it. My MD has just retired and I m in the process of finding another. Any suggestions? Thank you. kind regards, elaine Doctor: Dear Sir/MadamI have gone through your query and read your symptoms.In my opinion, you should be aware that you can report your symptoms, but unless a test has been done it cannot be proven, and finalized, that is the reason novartis was not interested in you, were your symptoms in the whole body or at the place of application of gel, a allergic test with the drug is a must to rule out this thing. so for the time being, stop diclofenac in all forms, gel tablets, injections, get allergy tests and ask your doctor to change to other options, like tramadol.I hope that answers your query. If you want any more clarification, contact me back.ThanksDr Narender Saini"
},
{
"id": 121115,
"tgt": "Suggest remedy for inability to walk",
"src": "Patient: my father is 62 years and 22 years as a diabitic patient his legs do not work, he can not walk, can not stand on his legs without support . we went to hospital Dr: suggested IVIG injections 14 in a day for 5 days .course is complete but he can not still walk or stand on his feet. TELL ME WHAT SHOULD I DO? WILL HE WALK EVER OR NOT? PLEASE REPLY Doctor: Hello,I read carefully your query and understand your concern. The symptoms of your father seem to be related to peripheral neuropathy due to diabetes. This is one of the common symptoms of diabetes.I recommend to maintain the glicemic level under control.I also to use prescription painkillers Gabapentin.This medication should be prescribed it by a neurologist.Hope my answer was helpful.If you have further queries feel free to contact me again.Kind regards! Dr.Dorina Gurabardhi General &Family Physician"
},
{
"id": 26969,
"tgt": "Suggest treatment for heart palpitations",
"src": "Patient: Having heart palpitations (feels like extra or skipped beat) for last two weeks straight everyday more than a few times a day. Never happened before. In the past only getting an occasional palpitation once in a blue moon. After first week of palpitations started getting chest pain or discomfort also in upper back ( like how it feels after your muscles hurt from exercise) and dwelling anxious from it all. EKG yesterday said it was normal except for slower heart rate (59 compared to last years EKG 60). Had a physical 3 months ago and everything seemed pretty normal. Doc suggested a chest X-ray eventually and Xanax. She thinks its anxiety or stress and I would have agreed with her normally only with the heart palpitations I don t feel like its that since I ve had a lot of past anxiety over the years but I ve never had these consistent palpitations and its pretty scary. I feel like the anxiety im experiencing now is just due to being scared about the palpitations and the fear and paranoia of something more serious? I m 36 5 1 103 lbs. normally healthy no history of heart disease in family. Doctor: Hello and thanks for writing.I can understand your concern and would try to help you in the best possible way. Your problems do not seem to be of cardiac origin as your EKG and other tests are normal. Your doctor may be right that it is due to anxiety or panic attacks. One more possibility could be an overactive thyroid which can be ruled out by a thyroid profile testing.I suggest you can speak to your GP regarding that plan the test accordingly. I would also advice you that you are in a good health right now. try to divert your mind away from illness. Start some exercise program like walking, gymming and also share quality time with your family. It would definitely help you stay fit and divert your mind of all unnecessary thoughts."
},
{
"id": 102519,
"tgt": "Should sharp abdominal pain after sneezing while on period evoke concern?",
"src": "Patient: Hello, I am 18 years old and I am on my period currently and I just sneezed and it resulted in a very sharp pain in my lower abdominal area. I am experiencing cramps, but I ve never felt anything this sharp and especially after a sneeze. Is this something that I should be concerned about? Doctor: Hello, It is my pleasure to help you.The sharp pain in the abdomen following sneezing during menstruation may be due to increased abdominal pressure during the act of sneezing. Sneezing put extra pressure on the menstruatiny uterus that may be the reason for the pain.If it continues to bother you consult your obstetrician for physical examination and find out the reason.Hope I answered your query.Thank you"
},
{
"id": 77788,
"tgt": "What causes bilateral pleural thickening with thoracic scoliosis?",
"src": "Patient: My husband is very hardworking he stands 5 feet and 11 inches but just 54 kilogram. He palys table tennis everyday. he already completed 6 months medication for PTB but his chest x ray result as of today says\"bilateral pleural thickening. consider PTB both upper lobes, subsegmental atelectasis and/or fibrosis, right upper lobe; thorasic scliosis. pls help. thanks marlyn Doctor: Thanks for your question on Health Care Magic. I can understand your concern. No need to worry for for his chest x ray report. His x ray findings are suggestive of healed, scarred lesions. Almost all lung lesions (tubercular or infective or injury) heal by either fibrosis or calcification or both. Your husband wad having tuberculosis and now completed the treatment. So tube lesions must be healed and showing fibrosis, pleural thickening etc.. Scoliosis is also due to healing sequel. So no need to worry for tuberculosis in your husband 's case. He is having old, healed, inactive, fibrotic scar lesions. Hope I have solved your query. I will be happy to help you further. Wishing good health to your husband. Thanks."
},
{
"id": 125862,
"tgt": "How can severe leg ache after a knee replacement surgery be treated?",
"src": "Patient: On January 23rd, I had a left total knee replacement. Since the surgery, especially at night, my left foot seems to drop forward and, as I m sleeping, my whole leg tends to fall inward causing intense pain in my left arch. I have wrapped an ace bandage around the arch, attaching it to the opposite headboard to hold my foot in place which seems to help. Is this a common problem and will it eventually go away as my knee heals. I usually sleep on my stomach and now need to sleep on my back because of the surgery. Thank you. Doctor: Hello, It is quite common after knee replacement. As a first line management, you can take analgesics like Tramadol or Aceclofenac for pain relief. If symptoms persist better to consult an orthopedician and get evaluated. Hope I have answered your query. Let me know if I can assist you further. Regards, Dr. Shinas Hussain, General & Family Physician"
},
{
"id": 91664,
"tgt": "What causes lower abdominal pain?",
"src": "Patient: Hello, my daughter has had a stomach pain for over three weeks, just below her belly button. It is a dull ache and sometimes the pain increases after eating or being active. She is not constipated nor dehydrated, She has been seen by her Doctor and the emergency room. All blood and urine test have come back fine. She also has a cat scan that came back fine. Please help. Thank you. Doctor: HI.3 weeks of pain with normal investigations, increases on food or activity. This indicates me a sub-clinical intestinal infection which may not be caught by any investigation. A 5 days- course of an antibiotic and metronidazole is therapeutic and diagnostic too. I hope Widal has already been done for typhoid."
},
{
"id": 186537,
"tgt": "Could the fleshy growth inside mouth be a canker sore?",
"src": "Patient: I have a small fleshy growth inside my mouth on the cheek . I had a canker sore near the gum on the same side that has healed. I have gotton canker sores due to stress but this the first time I have found what looks like a skin tag in my mouth thought it was due to stress but it didnt go away when the canker healed. what could it be? thank you Doctor: Hello, Welcome Thanks for consulting HCM, I have gone through your query, fleshy growwth inside mouth can be traumatic fibroma or painless swelling . Consult dentist for visual examination and investigations . If it is Fibroma then go for Excision and if it is swelling in gums then go for Oral prophylaxis . Hope this will help you."
},
{
"id": 75410,
"tgt": "What causes chest and shoulder pain with breathing difficulty?",
"src": "Patient: I am 42, overweight, smoker. I 'held in' a hiccup because they were painful- just as you would a sneeze you didn't want to occur. Then last night I had stabbing pain in my left shoulder blade. This morning I awoke with pain radiating to my left shoulder and to my left chest (parallel to my armpit). It hurts to breathe, pain is sharp and I can locate it. Went to dr, but dr said it is a muscle strain or a pinched nerve. She gave me 10 mg Flexeril. I still have severe sharp pain (8 on scale 1-10) in all 3 places and hurts to lift my arm. BP steady at 135/80. She said not heart issue. I am in house with my 7 year old and afraid to go to sleep fearing it could really be a heart attack. What could this be?? Doctor: Hi, The symptom along with shoulder pain and sharp pain when you breathe could be due to a lund or pleural problem . It can cause hiccups if the problem is lower down in the lung . I would suggest a chest x ray and blood counts.Can you tell me if you also have cough with sputum ? Will be happy to review .."
},
{
"id": 26001,
"tgt": "What causes heart burn?",
"src": "Patient: For the last few weeks, I have had the worst heartburn- heartburn isn't unusual for me, but it's been chronic and really bad. I also have been getting up in the middle of the night to pee, which is not normal at all. Do these two things correlate in any way? Doctor: Hello!Welcome on HCM!I carefully passed through your medical history and would explain that the heartburn seems to be related to a gastroesophageal reflux (possible hiatal hernia, esophagitis or gastritis). I recommend consulting with a gastroenterologist and performing a fibrogastroscopy. Regarding the frequent urination during the night, I don't thing that it is connected to the heartburn. It may be related to an urinary tract infection or any other disorders of the urinary tract. Do you drink water during the night? Are you a male or a woman? What is your age? Are you taking any medications (diuretics, etc.)?There are other different causes of frequent urination during the night (prostate hypertrophy, uterine prolapse, urinary incontinence, metabolic disorders, etc.). I recommend discussing with your GP on this situation and performing some lab tests (complete blood count, fasting glucose, liver and renal function tests, blood electrolytes, urine analysis, etc.). Hope you will find this answer helpful!Kind regards!Dr. Iliri"
},
{
"id": 153157,
"tgt": "Suggest management of unknown primary tumor in the stomach",
"src": "Patient: Dear doctor My father is diagnosed with uknown primary tumor in the lumph in the stomache. He got the diagnose last year sep. 1. 2010 after beeing sick in pain for 4 month. He was treated with kemo for about 6 month and i shrank, but nok the last time in the month April, so they started giving him a new type of kemo, and we just got the result yesterday that til got 20% bigger that in may, now they say that they have given him the only 2 types of different kemo treetment they have in Denmark, and the only thing they can offer him is to put him up as candidate for an experiment ( not kemo ) Do You have any experience with that type of disease or know anyone in the world that has? Regards Heidi Doctor: Good morning. Your father seems to be suffering from cancer of unknown primary and he has already taken two lines of chemotherapy. As the disease grows the chances of further chemotherapy working on the tumor become less. As your doctor rightly said, best for your father would be to put him on some experimental therapy. You should go ahead with this trial form of therapy as only this can lead to best results.regards"
},
{
"id": 131904,
"tgt": "What could be done for the pain in the left shoulder down to elbow after an allergy attack?",
"src": "Patient: Hi there. Couple of days ago I had a severe allergy attack. The allergy attack lasted roughly 2 days. During that time I blew my nose pretty hard. I noticed that I have intense pain in the left shoulder down to the elbow. Also the grip with my left hand is weaker. If I blow my nose, a relatively small amount of pain will be felt in the shoulder upper arm and elbow areas of the left arm. What is going on with my arm and what can I do about it? Thanks! David Doctor: Hi going by your description I am inclined to think in terms of cervical disc pressing on one of the nerve roots in neck. I suggest you go for a MRI to arrive at a diagnosis. I would recommend pain killers , muscle relaxants pregabalin and physiotherapy including neck exercises."
},
{
"id": 187778,
"tgt": "What could tooth growing at the back of mouth causing extreme pain mean?",
"src": "Patient: I think a tooth is growing at the back of my mouth, I doubt it's wisdom tooth because I'm only 14. I have so much pain at the back of my mouth at that back tooth. It hurts every time I swallow, eat, talk, everything Is hard. What is happening and what do I do? Doctor: Dear user,Thanks for using healthcare magic.I had gone through your query and understand your concerns.Yes wisdom tooth can be at 14yrs,because wisdom tooth start calcify at 8yrs,So don't worry about this,In mostly cases it is very pain-full also,pain can be radiated in neck,cheek,head etc,You should take a IOPA(x-rey) also,so can decide that it is erupt in proper line or not,So you need to visit a dentist,you should take some antibiotics and painkillers,so you can relief from all symptoms,Hope I have answered your query. If you have any further questions I will be happy to help\". Thanks."
},
{
"id": 74305,
"tgt": "What causes pain and pressure sensation in the chest and throat?",
"src": "Patient: i have been expierencing pain in my chest after comming down with an flu/cold today i expierenced pain in my left breats then around my shoulders and rib cage. then the pain became preesure on the center of my chest and throat what might be causing this discomfort the pain increased with stress i took two aspirins and now 3 hours latter i feel a little better lot less pressure 29 smoker 10yrs female Doctor: Respected user , HiThanks for using Healthcaremagic.comI have evaluated your query thoroughly .* This is in relation with pleural irritation through underlying bronchitis .Hope this clears your query .Welcome for further guidance .Regards dear ."
},
{
"id": 215272,
"tgt": "What can be the reason for frequent fever along with body ache?",
"src": "Patient: Trying to figure out if my 18 year old daughter has an underlying health issue. She has been spiking fevers and has been sick 4 times in the last 5 months. Twice in the month of February. As recent as last week after taking a swimming test to be a lifeguard (which she passed). So we ve noticed that it may be after exertion. At first we thought maybe because she was at college, exposed to a lot of bugs, but has happened now since she s been home too. Symptoms: fever, body aches, general sinus stuff, cough. Doctor: Hi, Without an exam, I can only give general information, but it's pretty good in this case. Fever implies inflammation. this can be from a generalized condition like an auto immune response or an infection which is generally pretty localized to one area and has symptoms that focus On That Area such as cough and sinus stuff. Chronic sinus/lung/ or infection anywhere along the respiratory pathway can give fever and might need just antibiotics or with chronic conditions often/usually has something bent out of shape like an abscess or stopped up sinus that needs (usually really minor) surgical correction. Looking for something bent out of shape, looking for a collection of pus would be the firs step and then antibiotics if small and surgery then antibiotics if big. Hope I have answered your question. Let me know if I can assist you further. Regards, Dr. Matt Wachsman, Addiction Medicine Specialist"
},
{
"id": 131706,
"tgt": "Is this a serious injury and what might have snapped or popped?",
"src": "Patient: I was holding a pole on the bus and the driver made a sudden stop that made me yank on the pole to keep front falling. I felt a snap or pop on the top of my hand and it s tender to just the area and hurts when I try to lift something with weight to it. Is this a serious injury and what might have snapped or popped?? Doctor: it could have caused a bone fissure that needs an xray done to rule it out..do not worry but start with ice packs and rest..and get an xray done."
},
{
"id": 185274,
"tgt": "Does tooth ache cause headache?",
"src": "Patient: For the past few years, I have had a headache in my right side, which only comes when I have a tingling in the same right molar. I have recently been to the dentist and they told me that I have no cavities or need for a root canal on that tooth. I am wondering if it could possibly be a really old filling that needs to be replaced, that is possibly triggering a nerve and causing a headache. Doctor: HiWith your query, as you mentioned you already had a filling did your dentist checked for any secondary caries or lesion under the already filled tooth? Moreover you have mentioned you are having headaches from years did you ever consulted your general physician for this problem? Yes sometimes toothache refers to headache especially when the upper teeth are affected. Whatsoever it would be to get it chracked by your dentist fir secondary caries and go to your physician as well to eliminate the cause. Hope thus helps you"
},
{
"id": 221437,
"tgt": "What are the early signs and symptoms of pregnancy?",
"src": "Patient: I have all signs of pregnancy. I was one week late for my period. Yesterday should have been the end of my period and I started to spot very light red blood. Not enough for a pad. Something happened today. Could I still be pregnant even though I am spotting? Doctor: Hi, I understand your concern. Pregnancy test ( home test after 8 days delay in periods ) or USG scan of uterus( after 5 weeks )are the confirmatory tests for pregnancy. You have just spotting after 8 days delay in periods .. This can be a sign in weak pregnancy so you should go for home pregnancy test ( with precaution.. not to mix vaginal blood in urine sample ).. to diagnose / exclude pregnancy. In case it 's +ve.. a hormone supplement ( with a gynecologist's advise ) should be started right away6. Thanks."
},
{
"id": 136571,
"tgt": "What causes shooting pain in the arms while coughing?",
"src": "Patient: What could cause pain shooting down arms while coughing or vomiting? It turns into numbness, tingling. I feel it like a string from yhe front of my armpit to the back of the elbow then down to my fingers. It makes me feel like I m dying and panicky after. I have a diagnosis of fibromyalgia and usually take an inhaler for asthma. I haven t taken it in about a month because I was concerned it was making my mouth and tongue sore. Doctor: Hi,Thanks for your query.Due to compression of nerve root in cervical spine there is tingling numbness and pain associated with it.I will advise you to MRI cervical spine for better diagnosis.For these symptoms analgesic and neurotropic like pregabalin and methylcobalaminmedication can be started consulting your doctor.Till time, avoid lifting weights, Sit with support to back. You can consult physiotherapist for help.I do hope that you have found something helpful and I will be glad to answer any further query.Take care"
},
{
"id": 214105,
"tgt": "Will you die if you take 30 150mg of Elavil & 40 over the counter sleeping pill ?",
"src": "Patient: Will you die if you take 30 150mg of Elavil & 40 over the counter sleeping pill ? Doctor: No, there is no fear of death but should not be done. It shall cause other side effects. You should go as per the prescription only. Taking an overdose does not help to solve the problem, whatever it is."
},
{
"id": 81893,
"tgt": "What does opacities in lower lobe on chest x-ray suggest?",
"src": "Patient: Hi i have gone through xray test and it says that streaky opacities are seen in the right lower lobe heart mot enlarge, diaphragm and costophrenic sulci are intact, bony throax is unfemarkable impresion basal pneumonitis. what does this mean. Please i need your reply.. Doctor: This means that either you have an infection of the airway that we commonly call as bronchitis or an infection of the lung i.e. pneumonia.Pneumonitis is a very vague term used by radiologist commonly and it signifies nothing. It could be just an increased bronchovascular markings on that part. May not be a pneumonia necessarily.See a good chest physician"
},
{
"id": 183959,
"tgt": "Suggest remedy for swollen tonsils",
"src": "Patient: Hi, 4 days ago I was smoking cigarettes with my friend then a day later I developed swollen tonsils. now only the right one is swollen and the gum behind the bottom wisdom tooth on the same side is soar. I feel that my groins hurt a little to. What could be the cause? Doctor: Hello and welcome to HCMi have gone through your query and i think that its a tonsillitis or inflammation of adenoids.Don have to worry about much. you can follow these steps:1. stop smoking (at least for few days)2. gargle with salty lukewarm water thrice daily3. rinse your mouth with chlorohexidine mouthwash4. take a course of antibiotic like Amoxycillin plus clavulanic acid5. if pain is more a painkiller can be taken.Following these steps will surely help.if the symtomps persist consult a doctor.Hope this ans your query. Take careRegardsDr. Shesh"
},
{
"id": 25002,
"tgt": "What causes high blood pressure and dull ache in chest?",
"src": "Patient: My blood pressure is 141/87. I have been getting dull aches in my chest for a very long time. Years now and I am now 45 years old. I saw a doctor about 6 years ago because I thought maybe i was having heart problems but they did a heart cath and found nothing. I am not sure what would cause dull aches in my chest. I do have a history of acid problems that also runs in my family. Could this be gas? Doctor: Hello and thank you for using HCM.I carefully read your question and I understand your concern. You should not worry. I'll try to explain you something and give you my opinion. You should know that chest pains might have different reasons.The origin might be cardiac, gastrointestinal problem, skelectic, muscular or neurotic problem.You should know that typical chest pains has some characteristics. They usually are retro-sternal pains like squizzing, tightness or heaviness. They may irradiate to both arms ,neck and jaw. They last from 5 to 20 minutes. To exclude ischemic heart disease, if I was your treating doctor I would recommend a strees test. The fact that you have done aniography is a good thing and probably the situation is still the same if you have the same complains, but it has passed a lot of time so a strees test is necesary.Ti exclude gastrointestinal problem you should do a gastrohepatologyst consulting and maybe a fibrogastroscopi. Gastic problems, gas presence, reflux or ulcers might be the cause of your symtoms. Also you should do a thorax xray to exclude any sceletic problem. Only after all this we can better understand what we are dealling whith and how to treat it. Hope I was helpfull. Best regards, Dr.Ervina"
},
{
"id": 144640,
"tgt": "Is diffuse posterior disc bulge, bilateral neural foramina serious?",
"src": "Patient: diffuse posterior disc bulge causing narrowing of central canal and bilateral neural foramina causing mild compression on bilateral exiting L4 nerve roots at L4-5 level. mild diffuse posterior disc bulge with right paracentral annular tear indenting on anterior thecal sac at L5-S1 level. MRI report Doctor: Seriousness is not fully dependent on MRI finding. Instead its more clinical symptoms and corresponding MRI findings decide further treatment."
},
{
"id": 81986,
"tgt": "Suggest treatment for cough and pain in the ribs",
"src": "Patient: I was sick with bronchitis for about 12 days then around the 15th day I started to have pain in my ribs on the right side below my breast. The pain has only gotten worse in the last 6 days. It hurts when I breath, cough or even move. I am coughing up thick white mucus. What can I do for this Doctor: Thanks for your question on HCM. In my opinion you should consult pulmonologist and get done chest x ray and clinical examination of respiratory system. Your complaints like pain on deep breathing and coughing suggest pleurisy (inflammation of pleura). And thick whitish expectoration suggest lower respiratory tract infection ( LRTI ). Pneumonia can cause both pleurisy and LRTI.So possibility of pneumonia is more in your case.So get done chest x ray first to rule out this. You may need antibiotics and bronchodilators too."
},
{
"id": 135377,
"tgt": "What causes localized pain in shoulder when diagnosed with fibromyalgia?",
"src": "Patient: I woke up yesterday with deep shoulder pain in what feels like the bone itself. It progressively got worse and by this morning the pain has become debilitating. I do suffer from fibromylagia, but this is localized and now giving me a headache and overall pain. It almost feels like a combo of the flu and fibro. I typed in some symptoms and thought it might be a bursitis, but it s hard to tell. Doctor: Do u have fever runny nose or sore throat if yes then it may be flu but since u hv fibromylagia since it can be aggravated by emotional stress if u dont take drugs for it it may aggravate pain (ssri are used) deep shoulder painis mostly due to rotator cuff injuries(muscle around shoulder ) whole bodily pain u have may be due to any systemic infection if u have behind eye pain it may also go in favour of dengue but dont worry u dont have fever i would suggest u to have some analgesics 1st & then see the difference if the pain goes early its rotator cuff injury also take good nutrition pain will go off follow up with more details"
},
{
"id": 185032,
"tgt": "How to treat an ulcer causing gum pain?",
"src": "Patient: Respected Dr., I (Tapan Thakur, 47y) live in West Bengal, India. I Suffered gum pain caused ulcer and also white patch (LP) found last one month. Local Dr. given Triamcinolone 0.1%, Chlorhexidine + Metronidazole (GEL). It used only 5 days but no improvement found. It\u2019s cancer or curable.\u00a0\u00a0\u00a0\u00a0\u00a0Today, Dental Dr. is given Tenovate \u2013M to use into mouth but I am afraid because it is external use only. Kindly advice what to do? Doctor: Hello!Thank you for posting here.Your doctor prescribed medication to reduce the symptoms,pain and burning sensation.5 days is a very small span and it is necessary to wait atleast a month to see results.Topical application means application on skin and mucous membranes.There is no issue on applying topically.However,make sure you do not ingest it.Avoid food intake after application for atleast half an hour.This is not cancer.This is a precancerous condition.However, you can ask your dentist for a biopsy if you insist.This is a precancerous lesion and potentially can become cancer if you continue smoking and other habits. Maintain a good oral hygiene.Regards."
},
{
"id": 42956,
"tgt": "Will I be ovulating egg normally and producing eggs if I have abdominal pain?",
"src": "Patient: hello doc, My name is Priya . we have been trying for a baby since 4 months. and doc had given me the cnseval ,folic acid tablets. On9th day she did scanning and given lupride injection to rupture the eggs. and gave progynova and clomipure tablets for egg development. but now my right ovary follicle is growing and its about 6cm and left is 3cm. but aim getting lower abdominal pain and sometimes it worsens. so planning to go to second opinion.when iam ovulating normally and producing eggs why they shud give the injection.???? now the eggs are not ruptures as did a scanning yesterday. it has grown very big. what are the methods what i shud do ...iam worried please help me Doctor: Hi,I read your query and I understand your concerns.Following is my reply:1) The pain during ovulation is sometimes perceived as abdominal pain.2) Injections are given to time rupture of eggs.Let me know if you have anymore questions.Regards,Dr. Mahesh Koregol"
},
{
"id": 210408,
"tgt": "Suffering from breathing,anemia and anxiety attacks",
"src": "Patient: Good Morning,Just a quick query, I have had an issue with my breathing recently, in this, I find it hard to take a full, deep, breath.I have been tested for anaemia in the past whilst having this problem, and my blood iron levels are always perfect, my blood pressure is also always perfect, as are my SP02 levels. I have found that this problem (which I can only describe as dyspepsia) is often worsened by alcohol.I drink frequently, but generally nothing stronger than Lager (which at most is 4% against my usual lager that is just 3.8%)I have found that meditation and relaxation techniques make the problem easier, but nothing seems to cure it completely. This is an issue that started literally overnight several months ago, and it is driving me crazy :(I have been told that it is anxiety, but this started before I suffered with anxiety, and is not cured by any anti anxiety medication... I think, I just want to know what it is, and how to make it go away. Kind Regards,Alexander Doctor: HiThanks for using healthcare magicI think, you have generalized anxiety disorder and you need continuous treatment with antidepressant and benzodiazepine. I would suggest you paroxetine, that is a antidepressant with very good anxiolytic effect. You can carry on with relaxation exercise and meditatation that would give you additional benefit.Thanks"
},
{
"id": 109948,
"tgt": "What is the treatment for lower back pain?",
"src": "Patient: I had 4 wisdom teeth removed on Friday. Shortly thereafter, the same day, I had excruciating pain in my lower back. I went to the ER and they said I did not have a kidney stone and that it was either muscular or a disc? But it happened like right after I woke up from anesthesia. Any thoughts about this? Does anyone else get similar symptoms? Doctor: Hi,Welcome to healthcare magic.After going through your query I think You are suffering from acute backache after your widome teeth removal.Treatment of it is rest and analgesics . Diclofenac three times a day after meals is effective. Sometimes strong analgesic(such as ultracet three times a day after meals) is required. OMEPRAZOLE before meals prevent acidity caused by analgesics . You can discuss with your treating Doctor .There are two possibility ,one is simple muscle pain,second is disc prolapse.I consider it to be muscular as this occur after your four wisdom teeth removal ,prolonged laying down on dental chair may have provoked it. To further investigate MRI of the region is advised. I think your query query answered.Welcome to any follow up query."
},
{
"id": 46460,
"tgt": "What causes frequent urination,back pain and puffy eyes?",
"src": "Patient: I was told by my ear surgeon that I have had kidney dicease for over a year. I was not told this by my primary doctor. I have no idea what it is. I have been urinating about 20 times a day and four or five times at night. I have chronic pain in my lower back, as well as significant swelling and bags around the sides and bottom of my eyes. What can I expect.? Doctor: R u a diabetic, if yes please check for protein in your urine. Don't take any drugs that harms kidney like ayurveda, homeo , unani, or siddha or pain killers."
},
{
"id": 112949,
"tgt": "Severe back pain, stinging pain while moving around. Treatment?",
"src": "Patient: ive had some heavby back pain over the last two years and have had several test done. When i start moving around much as in doing chores i get severe stinging pain in right to mid low back to the point that it make me stop what im doing for a few minutes. I have been to pt several times and had shots in my back but still no relief. Doctor: Hi Thanks for using Health Care Magic Forum, Back pain is more common after 40 years of age but as the pain is increasing in severity you should have complete bed rest for a week with mild anelgesics and antacids to prevent NSAIDS induced gastric ulcer ,. Physiotheraphy and hot compress can relieve pain spontaneously. Reduce weight if overweight as it put pressure to back bone too. take calcium tablets to prevent bony deformities. I Hope I Provided Useful Information. Do ask Furthur Questions to help You get well soon. Regards Dr.J.Kingson John David"
},
{
"id": 181257,
"tgt": "What causes a feeling of something stuck in between the teeth?",
"src": "Patient: I have a sore on my gums, above front tooth, it has been there for almost 5 weeks. I feel like there is always something stuck in my teeth and the sore hurts when touched. It makes the tooth it is above feel painful/ swollen sort of, its a very weird feeling. Doctor: Hi..Welcome to HCM..As per your complain feeling of something struck in between the teeth along with sore over the gum that is painful and also tooth hurting can be due to either foreign body impaction in between the gum and the tooth leading to infection and abscess formation..In case if tooth over which the gum has the sore is decayed then it can be a dental abscess..I would therefore suggest you to consult an Oral Physician and get evaluated and a thorough clinical evaluation and investigation like x ray can help in diagnosis and treated accordingly..In case if there is gum abscess then deep cleaning of the gums followed by a short course of antibiotics and gum paint application can help..In case of tooth abscess root canal treatment or extraction is advised..As of now apply numbing gel containing lignocaine over the sore and do warm saline gargles..Hope this helps..Regards."
},
{
"id": 203628,
"tgt": "Is it normal to have pain in the penis region after varicocele operation?",
"src": "Patient: hi dr, I have had undergone varicocele operation on both the testis. after the operation I have pain in the region connecting lower abdomen to legs on the way to penises. the pain is especially visible after masterbation. will that be an issue. I am going to get married in a month and this pain is bugging me often whether I should call off the marriage. please advise Doctor: HelloThanks for your query,based on the facts that you have posted it appears that you have undergone surgery for Varicocele very recently and getting pain in groin and penis .This is common due to inflammation of the spermatic cord which remains for 2-3 weeks after surgery.You need to take broad spectrum antibiotic like Cefexine along with anti inflammatory drug like Diclofenac twice daily.Ensure to drink more water., to keep your urine dilute .Dr.Patil."
},
{
"id": 178394,
"tgt": "Suggest treatment for vomiting in a child",
"src": "Patient: Hi . My son is 1.5 yrs .. From morning he is been vomiting . As suggested by doctor I have given olran syrup for three times in 24 hours .. He is having fever as well .. He has puked everything even eletrol ( O R S ) ,milk n water . Please suggest me some medicines and what food I can give . Doctor: Your child is probably suffering from acute gastroenteritis. Give him Ondem MD (ondansetron mouth dissolving) tablet 4mg 1/2 tablet. It will dissolve in the mouth and hence not be able to reduce vomiting without causing it. After 15 mins to half an hour of the dose, give ORS or electral and other oral food and the baby would be able to tolerate it. Give this medicine thrice a day before food. Avoid milk and wheat based food products for a week. You can give curd or yogurt. Other than these, you can give all sorts of foods that the baby is accustomed to. The episode should get controlled in 2-3 days."
},
{
"id": 99281,
"tgt": "How to get rid of itchy lumps on neck and forearm?",
"src": "Patient: i have small lumps that itch instantly they are random once on side of neck then on either forearm but today they on both pimafucort gives relief and its gone in a few days this time its been a week it feels like an instant sharp itchy tickle then to constant itch sometimes it goes away it has been a year or so since last time it was itchy then it can also be months it comes up within minutes can feel it start it wakes me up Doctor: HIWell come to HCMI really appreciate your concern such skin lesion could be chronic eczema some time may be associated with fungal infection, this can be managed with \"Miconazole and Betamethasone cream\" if this fails to response then clinical examination may be necessary because this is the only best way of diagnosis of skin lesion, hope this information helps, take care."
},
{
"id": 198095,
"tgt": "How to treat Orchitis?",
"src": "Patient: Diagnosed with Orchitis. I have an Enlarged and Indurated Left testicle. On Day 4 of Levofloxacin. No Pain...there never has been pain. Just the swelling and induration have been the symptoms. Ultrasound and a physical were taken before the diagnosis was made.However all my research indicates that pain is always associated with Orchitis...was I misdiagnosed? Doctor: HelloThanks for query .You have swelling and induration of Left testis and have done ultrasound scanning of scrotum and been diagnosed to have Orchitis .Orchitis is an infection of testicle and is always associated with sever pain and other urinary symptoms like burning urination,fever etc The other most common cause of swelling of the testicle without pain is Testicular Tumor .You need to consult qualified Urologist for clinical examination to confirm the diagnosis .He may advise for 1)Testicular Biopsy to rule out malignancy of testis 2) MRI of scrotum .Dr.Patil."
},
{
"id": 181956,
"tgt": "Suggest treatment for burning sensation in teeth after root canal treatment",
"src": "Patient: husband had root canal done behind front tooth eight years ago. has had a burning sensation behind front teeth since than. has taken antibiotics, Lyrica, capsaicin, new caps, had the nerves cut on top of the mouth, spent thousands on finding a cure. Nothing helps for long. Help Doctor: Thanks for your query, I have gone through your query.The burning sensation folllowing roo canal treatment can be because of the nerve injury while giving local anethesia or it can be a burning mouth syndrome. Nothing to be panic, consult a good oral physician and get it evaluated. Mean while you can take nerve regenerating drugs like Vitamin B6 and 12, Topical anesthetic like lignocaine and topical analgesics like choline salicylate.I hope my answer will help you, take care."
},
{
"id": 145216,
"tgt": "Suggest treatment for multiple sclerosis",
"src": "Patient: I have MS and fall often. Yesterday I fell forward landing very hard on my breasts. They hurt.Muscles under arms are hurting also. Should I go to http://d3kvn6nhl873a4.cloudfront.net/web/images/mashup/hpProcessV1.pnga doctor or just give them a couple days to heal? Doctor: Hello ! I read your question and understand your concern.The crash you have had may cause the so called crash syndrome , characterized by severe muscular traumas. I would recommend to consult with your GP and have a good physical examination, a full blood work, a CPK plasma level for crush syndrome. If everything is ok you will need just some days of bed rest.Hope to have been helpful! Best wishes, Dr. Abaz Quka"
},
{
"id": 94596,
"tgt": "Semen analysis report. Why is pregnancy an issue? What are the pin headed things noted?",
"src": "Patient: Hello Doctor, I took a semen analysis as my partner was not getting pregnant after trying to conceive for a year. I have mentioned the semen analysis report below. Please let me know if I will be able to help conceive natuarally? Are there any medications for the same? The results say : Colour: grey white Quantiy: 5ml Viscocity: Normal reaction: Alkaline Liquefication time: 30 mins Total spermatozoa count: 50millions/ml active motility: 20 to 30% Sluggish motility: 20 to 30% Non motile: rest Epithelial cells: 2 to 4 cells/hpf Pus cells: 4 to 6 cells/hpf RBC: Nil Morphology: 70% normal Impression: Asthenospermia (moderate) Remarks: Sperms appear normal in number and show moderate loss of active motility, Many pin headed, immature and swollen neck forms are noted. Doctor: there are chances that your partner may become pregnant but beware to have sex around ovulation time (midcycle) after 3-4 days of abstinence . the following tips increase the chances of pregnancy: 1.avoid heat exposure to your testes 2.take more nuts like cashew and badam 3.eat more veg and fruits 4.keep a thin pillow under buttocks of your wife in lying down position while having intercourse to increase the chances of sprems reaching ovum as your result shows asthenospermia and low active motile sperms"
},
{
"id": 48813,
"tgt": "Can my kidney condition be cured by medicine or should i undergo transplant?",
"src": "Patient: I am undergoing Dialysis for the last 2 months, started dialysis when creatnine level reached 9.7, now the predialysis creatnine level is 6.8 after 4 days, can this be cured with medicine or transplant is the only remedy? Jaskaran Bir Singh YYYY@YYYY Doctor: Thanks for using healthcaremagic. I understand your concern.Brief answer:Kidney transplantation is recommended.Detailed answer:I would say,kidney transplant as the best option for patients with chronic advanced kidney disease,to avoid long term morbidity associated with dialysis and improve the quality of life.Nowadays kidney transplant also considered to be cost-effective than lifelong dialysis. Also conservative treatment with drugs fails to help rather may worsen the kidney's function.Hope I have answered your query.If any further questions, happy to answer you.Thank you."
},
{
"id": 166541,
"tgt": "What is the dosage of iron syrup for babies?",
"src": "Patient: Hi. My daughter is 10 months old. I was told by her dr. that her iron was low and should give her enfamil iron drops. They have been very constipating even after eating fruit juice, prunes, and lots of vegetables. I bought liquid iron made with ferrochel that is supposed to be more gentle on the stomach. For 1 Tablespoon (15ml), contains 18 mg Iron (as Ferrochel amino acid chelate). It says for child 2 years and up 1/2 tablespoon. How do I convert the dosage for my daughter? Doctor: Dear parent, the dose for a 10 months old would be 1 and half ml which is equivalent to a quarter of a teaspoon"
},
{
"id": 156549,
"tgt": "Can cancerous tumor under the left breast cause swelling on the left midriff?",
"src": "Patient: I have a hard lump under my left breast on my bra line & I have been told it is a cancerous tumor however since my biopsy a week ago I am experiencing great discomfort on my left midriff which is swollen, (Smaller than my right midriff. Could this be part of the tumor? Doctor: Since you have a diagnosis of breast cancer on biopsy, your oncologist will now request for staging investigations including imaging of chest and an ultrasound scan of abdomen. Your midriff (abdominal) swelling can be due to cancer metastases and ultrasound scan will rule it out.Please discuss it with your oncologist."
},
{
"id": 13323,
"tgt": "Suggest remedy for skin rashes",
"src": "Patient: My daughter has a rash over her back arms and bum mainly her back I noticed a fine circular disc size under her right chin different to the one on her back I m giving an antihistamine she is not really complaining we are on holiday in Marbella a week now and the rash is here about 4 days but the disc is only since yesterday Doctor: Hi, It may be a fungal infection called Tinea corporis most probably. Consult the dermatologist for the perfect diagnosis and proper treatment. Antifungal like tab ketoconazole for long so weeks might cure the condition. And also give antihistaminics like hydroxyzine HCL to relieve the itch. Apply sertaconazole cream on the patches. Avoid bath with soap. She can use Dettol liquid. Keep the skin dry. Hope I have answered your query. Let me know if I can assist you further. Regards, Dr. Ilyas Patel, Dermatologist"
},
{
"id": 85340,
"tgt": "How to recover from side effects of mucinex",
"src": "Patient: I took mucinex d at 12 pm last night. I woke up at 5am and threw up and had diarrhea. I haven t gotten sick since then and it s 11:15 am, but my heart is beating fast and I feel like I have to throw up but can t. What can I do to get this out of my system? Doctor: Hello,I think diarrhea you had is not due to Mucinex. I think it might be due to respiratory infection for which you've been prescribed Mucinex. Thus, I think you are having acute gastroenteritis. Better check with your caring doctor.Hope I have answered your question. Let me know if I can assist you further. Regards, Dr. Albana Sejdini, General & Family Physician"
},
{
"id": 73400,
"tgt": "What causes dizziness, breathlessness, burning sensation in chest after respiratory infection?",
"src": "Patient: Hi... I have been suffering with an Acute Respiratory Infection for several weeks, and have taken a full course of Amoxycillan (500). My breathing is a lot better, albeit still a little strained. My concern is that I am still feeling a little dizzy and breathless at times, and very exhasuted and I have an intermittant burning sensation that keeps coming duting the day around my left chest area. I can still feel on the left side that things aren't as they should be, but there is a big improvement. The breast area on the left feels very bruised, particularly around the side and underside. If I press the underside it feels lumpy and as if I have been kicked. I have re called my Doctor and he has said that it is the aftermath of a bad infection. I just would like reassurance, and any advise as to how to speed up the healing process? The right breast is slightly sensitive at the right side, but nothing like the left. I am wondering whether the milk ducts in the breast area coulod have become blocked or infected? I finished breast feeding 12 months ago? I am also experiences intermittant tingling in my hands, which I have had since my first cold symptoms in January. Any advise is much appreciated. Thanks. Doctor: Hello dearWarm welcome to Healthcaremagic.comI have evaluated your query thoroughly .* This seems in relation with underlying lung inflammation still present , not blockage of mammary ducts or else .* Recommendations for better recovery - Ask for an x-ray chest with your doctor , and lab tests if not done . As you require further antibiotics and broncho dilators to cover the condition early .- Regular walking in fresh air , deep breathing exercises , YOGA will help tremendously .- Avoid smoking or alcohol if using .- Avoid exposure to excess cold , atmosphere with pollen , dust .- Balanced nutritious diet .Hope this will help you for sure .Regards dear take care ."
},
{
"id": 76267,
"tgt": "What does this PET scan of lungs indicate?",
"src": "Patient: My husband s report from a pet scan said the lungs are moderately inflated there is vascular crowding in the left lower lobe and poster segment of the left upper lobe presumable related to the patients absent pericardium. What does all this mean ? Thank you Doctor: PET scan indicates metabolic activity in the body.Areas which are infected or tumors are more metabolically active and are highlighted with PET Scan.The report you are indicating is from a PET/CT scan.And what you are reporting does not have any obvious abnormailty.Did your husband have a previous surgery?"
},
{
"id": 5558,
"tgt": "Difficulty conceiving, ovulating, sperm on fingers, put fingers into vagina. Pregnancy possible?",
"src": "Patient: Hi My wife and myself are desperate to get a baby. She has ovulated and yesterday egg has been released (Dr confirmed this via scan). Dr has asked us to have intercourse for the next 3-4 days. Today, however hard i tried, i couldn t ejaculate. So i masturbated and immediately (within a second), took the sperm in hand and put it in her vagina . Is there any chance of pregnancy by this way ? Doctor: Hello. Thanks for writing to us. If you have deposited the sperm in her vagina then the chances of pregnancy are there but not very high. With the natural method of conception, the sperms are deposited high up in the vagina near the uterine opening. I hope this information has been both informative and helpful for you. You can consult me again directly through my profile URL http://bit.ly/Dr-Rakhi-Tayal Regards, Dr. RakhiTayal drrakhitayal@gmail.com"
},
{
"id": 94317,
"tgt": "Stomach pain. Scan shows bilateral polycystic appearing ovaries. History of appendix surgery. Major issues?",
"src": "Patient: Past few days I got sever pain in my left side stomach. With doctor s advice taken scan. And the Impression is : 1) Bulky Uterus (? SUBSEPTATE) 2) Bilateral Polycystic Appearing Overies 3) Few Bowel Loops are seen adherent to the Anterior Abdominal wall in the Lower abdomen (SCAR ADHESION )4) Loading of the COLON + To consider COLITIS . Already appendix surgery done on 5 months back. Can you kindly tell me is there any major issues appear in the scanning result? Thanks Doctor: Scar adhesions needs to be addressed before it gets worse and you dont end up having complete adhesions. Colitis needs to be addressed. Polycystic ovaries usually doesn't cause any pain. please consult a surgeon for proper diagnosis and treatment at the earliest."
},
{
"id": 23012,
"tgt": "What does slight inferior repolarization disturbance in the ECG mean?",
"src": "Patient: ECG states slight inferior reploarization disturbance, consider ischemia, LV overload or aspecific change, small negative T ina VF with negative T in III; minor right-precordial repolarization disturbance, consider feminine pattern, small negative T in V2 with flat or low negative T in V3. I'm not understanding what this all really means, and what is a 'feminine pattern'? Doctor: ECG is showing some nonspecific changes which means there is some problem in your heart or sometimes these can be there normally. So I need to know your age, blood pressure ?, obesity? chest pain?, smoking or family history of heart disease.if any of these is present or even if not we should evaluate yourself with echocardiography and stress test if needed.overall not the matter of worry but should be investigated."
},
{
"id": 178565,
"tgt": "How can vomiting, diarrhea, throat pain and redness with dysphagia be treated?",
"src": "Patient: Hi. My daughter has been out of school since Monday because she has constintally thrown up and had diarrhea. But today she is complaining about her throat hurting when she swallows and her throat is red for what I see. Can you help us with these problems so she can get better to go to school?!? Thank you! Doctor: Thanks for putting up your query at HealthCareMagic. I understand the cause of your concern and have gone through the details carefully. Your daughter seems to have acute gastroenteritis or food poisoning. It generally occurs when some preformed bacterial toxin enters the body through some food item. It generally resolves on its own in a few days and the cause does not need to be treated. But the loss of salt and water from body may cause dehydration which needs to be managed with priority. Give her frequent sips of ORS. You may give Ondem MD (ondansetron mouth dissolving) tablet 15-30 mins before taking food and that will reduce the throwing up. Giving curd/yogurt could be helpful.Gurgle with lukewarm water will give her throat some relief. It is probably caused due to erosion caused due the gastric acids during her throwing up.She may skip her school for a day or two till she is stable enough to attend her classes.I hope that helps. Feel free to revert back with further queries if any."
},
{
"id": 30866,
"tgt": "How is secondary bacterial infection in kidney and liver treated?",
"src": "Patient: This is ,..fg. I have a serious problem. My husband Shyamsunder got a viral infection which later turned to a secondary bacterial infection and both his kidneys and liver have been affected. They have started dialysis for the kidney and put him on the ventilator. The problem is they are unable to control his liver infection. So I am sending you all the details.Can give advice on the line of treatment. Doctor: thanks for posting your query to health care magic.I want to know that how did you came to know about infection .have you undergone any kind of culture and sensitivity investigation .If yes then which organism is responsible for that infection and sensitive to which antimicrobial agent .After getting above information you can easily treat infection. I am sorry to say but as your patient is on ventilator it iself increases the chances of infection and further as I am understanding he immune status of patient is also decresed in stress condition . so you need to ake proper prophylactic antibiotic management as per your hospital protocol along with routine culture and sensitivity from recommended site.Thank you"
},
{
"id": 224407,
"tgt": "Have headaches after coil insertion for contraception. Is it normal?",
"src": "Patient: Hi There, i am 29 and i had the coil fitted july this year after the birth of my second child in January. I got it fitted when i was on my period and i have had a 28 day cycle since then with a period every month. my last period was beginning of oct and i have nothing since. I have done a test and its negative, I am getting concerned though as this is very out of character for me. I m tired and have had a few bad headaches but then again i am running about after 2 children. Any advice would be greatly appreciated. Thanks. Doctor: HIWell come to HCMHeadache may not be due to IUD but this could be due to your thought and confusion, not able to decide or taking right decision for device, if you are not comfortable with device then you can discuss this with your doctor as well as with your nearer and dearer, else this is nothing to worry, try to bring back your confidence, take care and have a nice day."
},
{
"id": 47529,
"tgt": "What does my ultrasound suggest?",
"src": "Patient: What does it mean when my ultrasound says I have a questionable bilateral extrarenal pelvis sees verus mild hydronephrosis bilaterally? The impressions says slight prominence of the collecting systems bilaterally left greater than right. What does this mean? Doctor: Hello and welcome to HCM.As an Urologist, i can fully understand your concern.Your scan report suggests there's urine collection, just outside the kidneys.It's minimal, and the significance of this can't be made out.It could be a normal variation or a possibility of very,mild, block between kidneys and bladder. Normally, urine drains out from kidneys to bladder.If you can send me your blood, urine and scan reports, as a direct question, i can give you advise expert management. Dr.Matthew J. Mangat."
},
{
"id": 184904,
"tgt": "What causes jaw pain during the growth of wisdom tooth?",
"src": "Patient: Hi. My top left wisdom tooth has began growing some more but for the last two days it is now rubbing against my cheek. At times my jaw hurts and I sometimes find it difficult to eat. My dentist told me that there was enough room for it in my last check up and is a healthy tooth and there is still plenty of room. Could you tell me what is wrong and what treatment I need? Doctor: Hello thanks for consulting at hcm..the flap covering is called pericoronal flap..you need to excise it..called operculectomy...since you are having pain and problem due to this...referred pain from tooth eruption... Food lodgement also in the area is possible...so practice good oral care regimen..blow air with mouth closed..so that teeth surface do not touch the buccal mucosa... Plz consult a oral surgeon for a check up and treatment.. Hope it helps.. Tc..."
},
{
"id": 50543,
"tgt": "Being treated for uti. Taken Bactrim, macrobid and diflucan. Have nausea, dizziness. Drug reaction or infection?",
"src": "Patient: Hi I'm being treated for a uti. Bactrim was ineffective and additional caused a yeast infection. I started macrobid and diflucan yesterday. I woke up an episode of coughing nausea cold sweat and dizziness that has since passed. However now I have slight flank pain, I'm not sure if this a drug reaction or an indication of an infection that has progressed to the kidneys? Doctor: HiThanks for the query.You have not mentioned your age/sex.If you have nausea, sweating and flank pain it may be a sign of infection in the kidney. You need to see a physician/nephrologist who may do further urine/blood tests and an Ultrasound of the kidneys to confirm the diagnosis.Based on the results a change in your antibiotic prescription may be required.Hope this helps.Good luck."
},
{
"id": 126502,
"tgt": "What causes tremors in the left arm?",
"src": "Patient: Trying to find out why my left arm jerks sometimes when I m doing nothing. I do have a LOT of arthritis in the neck, severe, and I also have mini seizures for which I take meds. I never lose consciousness, and only once was anyone but me aware that it happened. Can you read me, or am i supposed to enter this somehow. Oh, am I supposed to pay ? Lol. If so, then I ll get out of here now, and sorry for your time. Doctor: Hello, Consult a neurologist and get evaluated. We have to look for possible causes like Parkinson and thyroid related problems. Hope I have answered your query. Let me know if I can assist you further. Take care Regards, Dr Shinas Hussain, General & Family Physician"
},
{
"id": 85911,
"tgt": "How does one treat severe abdominal pain, nausea and breathing difficulty?",
"src": "Patient: My son is experiencing severe abdominal pain and nausea on the left side of his abdomen, to the point its gets hard for him to breath. He is currently in England and the doctors tell him there is nothing wrong. How can that be when he has changed his diet and still is doubled over in pain and barely able to live a normal life? Doctor: Hello, As what I understand from the history, the symptoms of breathing difficulty is due to stomach problem. May be he is not able to digest the food and the gastric issue is leading to pain. Nausea is a symptoms to feel in acidity. I will advice to consume slow sips of water with sugar in it. As fold water will ease the acid effect of the HCL produced in the stomach and sugar is a base which will help reduce the acidity. Having warm water post food and avoiding high protein food should help. In my clinical practice of over 12 years most cases of breathing issues are due to gastric issue. Once the gastric issue is resolved then the pain will come down and so does the breathing issue. Handling this kind of properly have always produced excellent results. Hope I have answered your query. Let me know if I can assist you further. Regards, Jay Indravadan Patel, Physical Therapist or Physiotherapist"
},
{
"id": 26669,
"tgt": "Can an SVT patient travel to higher altitude?",
"src": "Patient: My daughter has SVT and is going to Denver. She will be going up to a cabin at a higher altitude while she is there, about 2 days after arriving. She takes Metaprolol for her condition and I am wondering if the higher altitude will have a negative affect at all. Doctor: She can safely travel, acclimatisation is important and her staying for 2 days before ascending further up, would be good for her. Also just make sure he takes her metoprolol on time and carries enough stock of the drug. Per say if svt is well controlled and she is in normal sinus rhythm with the help of metoprolol , she can enjoy her vacation. Regards Dr Priyank Mody"
},
{
"id": 204632,
"tgt": "How can schizophrenia and bipolar disorder be treated?",
"src": "Patient: Our daughter is 37 years old and was diagnosed with schzofrenia when she was 13, then changed to atypical bipolar, but she doesn t feel she has a problem. She was treated with many different antipsychotic medications. She is very distrustful of people, including her parents and feel that we abused her as a child. She gets easily offended and she needs knee surgery. She has seen many orthopedic doctors for her knee problem but usually fires the doctor because he is unprofessional. She lives in California now; we live ib Maryland. She wants to have someone go with her to her next appointment, but usually wants us to help, but she does not want us to ask who is her doctor, when is her appointment, etc. She is very hard to work with. Is there any way we can persuade her to get on medication. She doesn t think she needs any, however, I don t know if she does taken anything now. She has been on many, including Seroquel, Closapine, Abilify, Geodon (for 1 day - it made her heart race), and others. As you know, there are many side effects. Any suggestions? Doctor: Hello,In my opinion, it is necessary to talk to her and find out why she is refusing medication. Also, assess her understanding of her illness.Also, have cognitive behavioral therapy sessions with psychologist depot injections are useful, in view of her reluctance to take the medications.Hope I have answered your query. Let me know if I can assist you further.Regards,Dr. Gayathri"
},
{
"id": 192586,
"tgt": "Why right testicle is larger than left?",
"src": "Patient: Why is my 6 wo sons right testical much larger than his left? Its doesnt seem to bother him when i palpate them, I just noticed about 2 weeks ago that his sac was much larger on the right so i tried to palpate the testical and thats when i noticed the difference. He has a check up in 2 weeks should i go sooner or wait until then? Doctor: Hello,It may be due to several causes. For detailed clinical examination, I would suggest you to consult with urologist. You may require ultrasound scrotum after surgeon consultation. You may require detailed evaluation to rule out any other possibilities for hydrocele or varicocele. Use scrotal support. Avoid heavy weight lifting Hope I have answered your query...Let me know if I can assist you further.Regards,Dr Shyam kaleFamily and general physician"
},
{
"id": 146739,
"tgt": "Can eczema be related to drainage around the nipple?",
"src": "Patient: Hi, I have a 17 yr old son with CP and a Baclofen Pump implant. I just recently noticed a somewhat clear drainage from the area around his right nipple. He is very cognitive, but unable to speak, however when I asked if it hurts he states no, but that it itches. I am also monitoring a low grade fever, yet he says he feels ok. Also he was diagnosed as having a form of exzema last year that I am constantly trying to control, could this be related? Doctor: unlikely. nipple exudate in a male is likely hormone related but this might be caused by a medication he is taking. consult your doctor that is treating the cp"
},
{
"id": 190177,
"tgt": "Got lower teeth filled. Having pain in the gums, jaw. Unable to eat, sleep due to sensitivity. Solution?",
"src": "Patient: I went to the dentist about a week ago to get a crown on both sides of my mouth at the top. I had 3 of my lower teeth filled in. It has been a week and yet one whole side of jaw has been aching. Also my gums are hurting as well. I can t eat on that side of my mouth or drink anything cold over there. It s stopping me from sleeping at night. Doctor: Hello there...there are two things to this...one being, pain owing to excessive material in the area of restoration (filling), if the decay is really deep enough it would cause pulpitis resulting in pain. Secondarily, senstivity could be from the region of your crowns (if you have got them)...there would have been a marginal exposure leading to sensitvity. Get your self examined on these lines...Dont worry your problem is treatable."
},
{
"id": 104256,
"tgt": "Have numb sinuses above palate towards back throat, choke. Had unsuccessful sinus surgeries, nerve damage causing, have asthma",
"src": "Patient: Good evening,I have this sensation of fullness/numbness in my sinuses just above my pallet and towards the back of my throat all the time. Sometimes it wakes me up at night because I feel like I am going to choke.I have had three sinus surgeries to repair this but all have been unsuccessful. My last ENT suggested nerve damage but I can't recall the name of the disease is or what may cause this.Just a note: I was involve in the search and rescue efforts at Ground Zero on 911 and was there for three months. I was diagnosed with severe Asthma in March of 2012. Doctor: chronic allergic sinusitis converting to allergic asthma in case of allergies if you operate sinuses manypatients land into asthma get allergy tests dne with special focus to foods milk wheat rice nuts potato chana and for dusts and pollens for foods elimination diet and for asthma and sinuses go for sublingual immunotherapy your sinuses and asthma will be controlled perfectly"
},
{
"id": 121277,
"tgt": "Suggest treatment for pain in knees",
"src": "Patient: My wife has pain in knees. I found two combination drugs. One with Glucosamine and chondroitin and another with Glucosamine and some Sulphonylmethyl salt. Which is better? And will this help her? She is 58 yrs, her height is 152 Cms and weight about 70Kgs. She is taking Thyroxin 100mcg for hypothyroid. Doctor: HiGlucosamine and chondroitin are structural components of cartilage, the tissue that cushions the joints. Both are produced naturally in the body. They are also available as dietary supplements. Researchers have studied the effects of these supplements, individually or in combination, on osteoarthritis, a common type of arthritis that destroys cartilage in the joints.Glucosamine is a naturally occurring compound that is chemically classified as an amino sugar.It serves as a building block for a variety of functional molecules in your body but is primarily recognized for developing and maintaining cartilage within your joints.So glucosamine and chondroitin can be used since both the compounds have cartilage protective activity.Hope that was helpful.Let me know if i can assist you further.RegardsDr.Saranya RamadossGeneral and Family Health Physician"
},
{
"id": 69104,
"tgt": "Suggest remedy for lumps",
"src": "Patient: my 6 year old complained of a bugbite on his head last week. i noticed today that he has a visible lump on his head. It doesnt appear to be a bite and it seems bigger than it was last week. he doesnt remember hitting it and he says it doesnt hurt. should i be overly concerned? Doctor: Hi.Yes , you should be concerned as the cause and effect is not known.I would advise you to consult a Doctor as he sees more of such atypical problems. IT may be a problem in a child of just 6 years. We have to confirm this is not infected and may need a treatment accordingly.Having a hematoma is definitely a possibility. But ruling out other problems is very important."
},
{
"id": 26794,
"tgt": "Could swelling of feet indicate heart problem?",
"src": "Patient: my husband is 77 years old. about ten years ago he litterly blew out the right side of his heart and never knew this until he had stint put in about 8 years ago. than the following year he had a ballon added. he has done well except that I am noticing that his left foot more than the right is quite swollen Doctor: Hi Sir,Welcome to HCM,I understand your query and concern.Your symptoms are suggestive of Congestive cardiac failure.I advise you to get a 2 dimensional echo of your heart to confirm the diagnosis.Drugs like diuretics,ionotropes will be helpful.Report to an emergency if you have symptoms like chest pain or shortness of breath or dizziness.Monitor blood pressure and heart rate regularly.Consult your cardiologist for expert management.Post your further queries if any,Thank you."
},
{
"id": 196643,
"tgt": "What is the treatment for premature ejaculation?",
"src": "Patient: I had unprotected sex 2 day after I developed symptoms of severe ilchy in both my penis and scrotum, buzzy and single bump in my left palm very small in size. I apply anti fungal cream and take one USA antibiotic LY MPH... I can t complete the name of the antibiotic everything come back to normal. 3 weeks later I masturbated rubbing pillow without lubricant and I ejaculated. I also did the samething 2 days after. The next day I had a sensation in my urethra towards the end of my penis and I have to squeez it for me to get relieve. After some hours I noticed discharge coming from the penis. This continues through the weekend. I went to Dr. On monday He prescribed some tablets which include metronidozole for 5 days. The discharge went away leaving the symptoms of buzzy and drop-like but when I check there will be nothing. I went back to Dr. He prescribed another antibiotic that include cipro for 4 days but nothing change. I went back for the third time he said it was normal but I know that there is something down there because I used feel like something is moving at the end of my urethra. One nurse suggested that I could be yeast infection that I should take Diflocan and I took but the discharge come back. Dr. Any idea about this? Can this be HIV? Thanks Doctor: Hi,Premature ejaculation is mainly treated with psychotherapy and proper training. If you are new to sex then it will automatically improve with time. In the resistant cases selective serotonin replace inhibitor is usually used. Hope it helps. If you have any other question please do not hesitate to contact us.Regards,Dr. Atishay Bukharia"
},
{
"id": 152982,
"tgt": "How often should colonoscopy be performed after removal of cancerous polyp?",
"src": "Patient: hi Dr. Grief I am a 70 yr. old male, how often should I be having a colonoscopy, I had a pre cancerous polyp removed about 12 yrs ago and have had three colonoscopy's since, I am scheduled for another in 2016, all procedures have been negative since original 12 yrs. ago. jim Doctor: ok, do u ave any family history. u have to undergo colonoscopy as it is very essential ans screeing meethod of detecting cancer at precancerous stage. and u have a history in the past. so i will suggest u to undergo colonoscopy."
},
{
"id": 99169,
"tgt": "Suggest remedy for allergy with rashes & hives",
"src": "Patient: hi...I am a student in dallas. I have some sort of allergy since march 2011...In April I visited the doctor at the university and they prescribed allegra and a skin cream for 15 days...The allergy seems to stay away for only 24 hours. I still continue to take the allegra tablet at the rate of one tablet per day. It has been more than 2 months now. I do not know if it is hives. But my rashes on the skin are bad. After 2 hours of taking the tablet they die out and pop up again the next day. Also, I have put on a lot of body weight and seem to gain weight really fast. Is it because of the allegra that I have been consuming everyday? Please let me know how can get rid of this allergy. THanks Doctor: HI, thanks for using healthcare magicYou may want to consider allergy testing to find out exactly what is causing your reaction so that it can be avoided if possible.This can be in the form of blood tests or skin prick testing.The allegra is not likely contributing to your rapid weight gain but there are other hormonal conditions that should considered eg thyroid disease, cortisol abnormalitiesYour doctor can also check these levels.I hope this helps"
},
{
"id": 56461,
"tgt": "What are the symptoms associated with enlargement of the liver?",
"src": "Patient: Sir, Recently I have gone medical checkup, it was noticed that Liver size is normal with increased echopattern. The IHBR aree dilated. No focal SOL is seen. Portal vein is normal in callibar What it means, what are the symptons normally we see in this regards. what do you suggest, whether is it serious in nature. Doctor: HelloIncreased echotexture of liver may indicate fatty changes in liver.It may be due to many reasons like obesity,sedentary life style,hepatitis,alcohol intake,altered lipid profile,diabetes mellitus,medicines etc.You may need few more investigations like routine hemogram,random blood sugar,liver function test,lipid profile.Other findings in liver are normal.Intrahepatic biliary radicles(IHBRs),portal vein are normal.You should increase your physical activity and avoid fried and fatty food.Fatty liver is a reversible condition.Get well soon.Take CareDr.Indu Bhushan"
},
{
"id": 217082,
"tgt": "What is the reason for the sharp pain in the bikini line?",
"src": "Patient: I have sharp pains in my bikini line. I tried asking my doctor but they said they will have to run tests. I m leaving the town soon so I cant do this. I m not pregnet, I know. This is driving me crazy. Maybe it s because of all the right pants, I am not a inner person but I have a good study in neurology so I was wondering if this could be something about the nerve. Anyway my final question is, Why do I have sharp pains in my bikini line? Doctor: Avoid tight fitted garments ....take an analgesic like tradmadol 50 mg sos. ..If it still persist consult your doctor again for further evaluation"
},
{
"id": 200523,
"tgt": "What causes difficulty reaching orgasm?",
"src": "Patient: I am a 47 year old healthy male. I takes me a long time to reach orgasm with my 52 year old girlfriend...at least 25-30 minutes...afterwards I am sore at the bottom of my penis and the vein that extends into my scrotum is sore as well...there is no redness or chafing on the penis. I am just sore. Doctor: Thanks for asking in healthcaremagic forum If intercourse is of long duration, soreness can be there naturally due to friction. Please send details regarding any medications(like viagra) and associated illnesses if any for further suggestions. All the best."
},
{
"id": 29487,
"tgt": "What causes itchy red rashes on the arms and headache despite using Ibuprofen?",
"src": "Patient: Hello. My daughter, age 7, has a low to moderate fever, complains of a headache, is fatigued. No stuffy nose of cough. Had an itchy throat yesterday, but today says her throat is fine. I also felt her glands were swollen yesterday. Today I noticed a rash on both of her arms. bigger on her right arm and small on her left. the skin is hard red and itchy, fever and headache get better with Ibuprofen but are coming back every 4-6 hours of so. this is second day she is not feeling well. Doctor: These rashes may to be due toViral illness small poxchicken poxAllergic reactionContinue feeding childproper rest is required leave from school( for 2-3 days at least)Anti allergic medication and other supportive treatment need to be prescribed after complete evaluation are required"
},
{
"id": 47910,
"tgt": "Why did blood appear in and around the mouth after stopping dialysis and cause death ?",
"src": "Patient: Family memeber, 85 and living alone, stopped her dialysis; did not have it on scheduled Saturday, talked to her son, Monday evening, then was found she had died in bed later that evening. Blood appeared in and around her mouth. Could you tell me why the blood? I understand she went peacefully. Doctor: Sorry to hear about the demise. In patients undergoing dialysis and having stopped there could be multiple reasons for bleeding. Theses patients have bleeding tendencies so she might have bled from any ulcer in stomach or esophagus. Or if due to stopping of the dialysis, she might have developed pulmonary hypertension due to accumulation of excessive fluids in lungs and cause bleeding through lungs which might have surfaced during excessive bought of cough containing blood. These could be the main reasons that i think of from what you have told me in your question. thank you"
},
{
"id": 216635,
"tgt": "What causes arm pain?",
"src": "Patient: I have had pain in my right whole arm starting from my bicep for few weeks and radiates to my little finger. Today the most pain is in the area between my elbow and hand. The only thing which relieves the pain is to keep my arm folded and a little higher and on the pillow. I have used rubs, but no relief. What could be causing this pain? Should I see a physician? Doctor: hi,thank-you for providing the brief history of you.A thorough clinical examination is advised.As the symptoms looks more of a radicular pain, it can be arising due to pinched nerve in the cervical spine.Undergoing an MRI of the cervical spine will help understand the soft tissue status.Treatment Physcial therapy which includes therapeutic ultrasound therapy and TENS therapy should help reduce the pain and inflammation. Exercise for neck and upper limb muscles should help regain the strength.In my clinical practice such cases are referred to physical therapy and they respond well.Regards Jay Indravadan Patel"
},
{
"id": 47936,
"tgt": "Suggest treatment for kidney stone removal",
"src": "Patient: My best Friend sufferd kidney stone from last two month he had done all treatment like Ayurvedik,Alopathic,Homiopathy as well but there is no effect still he has problem .In the Sonograpy report the kidney stone showing 9 mm wht is ur opinion please tell me.. Doctor: Hello, 9mm kidney stone can be reomved medically by a procedure called as hydrotherapy. In which lots of fluids is rushed in body and then diuretic medication is given which helps many patients of kidney stone to pass the stone naturally by urine. You can consult your doctor about the same. Thank you"
},
{
"id": 42314,
"tgt": "Suggest treatment for infertility",
"src": "Patient: I am Sukumar Khan from West bengal. We got married last 6 years. Last four years we trying for an issu. We visit many doctors. Last 2 years ago my wife effected by miss-caurage automatically. So avortion was done . After this issu is not coming. I want to last and final treatment by you. Please give me the way Doctor: HelloYou are trying for pregnancy from last 4 years ,but you mentioned that your wife had miscarriage 2 years back i.e. she got pregnancy .Mr Sukumar don't desperate do these measures . First of all get in SEMEN analysis . For wife , ovulation release date , dominant follicle size , rupture date , endometrium thickness ( size ) so for this get in ULTRASOUND of both ovaries . Either upload all these reports or consult an INFERTILITY specialist and get his opinion."
},
{
"id": 221822,
"tgt": "What causes lack of fetal heart beat during pregnancy?",
"src": "Patient: i m 22 years of age and today i had a vaginal scan. i m 6weeks and 5 days carrying. it says no cardiac at present and asked me to take another scan after 3 weeks. else everything is normal with growth of the fetus. i m scared and tensed wether it ll turn out a bad news after 3 weeks. please reply as soon as possible... Doctor: generally on tvs scan fetal heart beat can be detected as early as 6 week. so If on scan no fetal heart beat then go for repeat scan after 10 days. Don't worry everything will be fine if All other parameters are normal."
},
{
"id": 122552,
"tgt": "Should the doctor be consulted for complaints about left arm pain?",
"src": "Patient: Good morning, my mummy has been diabetic for 6 years now, her should is relatively controlled but not yet on target. Now she has been complain of pain in her left arm and down to the hand, and she no more raises the hand above the head level and associated sleepless night. What should we do? Doctor: Hello, The pain in the hand and arm with restricted movement is suggestive of a frozen shoulder that is commonly seen in diabetics. She needs to consult an orthopedician for proper management. Hope I have answered your query. Let me know if I can assist you further. Regards, Dr. Praveen Tayal, Orthopaedic Surgeon"
},
{
"id": 72764,
"tgt": "Does tightness in chest and shortness of breath indicate an asthma attack?",
"src": "Patient: OK this question is about asthma. I had an asthma attack last night continuing on to this morning with chest tightening, wheezing and shortness of breath. I went to my local doctor today and was nebulised. However this evening I am again having slight shortness of breath with chest tightness/pain and a faint feeling. My hands are also shaking however I'm not wheezing at all. Am I having another asthma attack or about to have one? Is there anything I should look out for as I'm home alone and don't really think I should go to sleep in case I AM having one. I've never experienced an asthma attack without wheezing before so I don't know if it's even possible. What should I do? Doctor: Thanks for your question on Healthcare Magic.I can understand your concern. Yes, you are having asthma attack most probably. Wheezing can be absent in early part of attack. To prevent such asthma attacks, you should consult pulmonologist and start controller medications Iike long acting inhaled bronchodilators (formoterol or salmeterol), inhaled corticosteroid (ICS) (budesonide or fluticasone) and oral combination of montelukast and fexofenadine.If you take these drugs, you won't get asthma attacks. At present, you need short acting beta 2 agonist (SABA) (Salbutamol or albuterol).Hope I have solved your query. I will be happy to help you further. Wish you good health. Thanks."
},
{
"id": 221954,
"tgt": "What are the early signs and symptoms of pregnancy?",
"src": "Patient: I just started the neuvaring this month, but before starting it me and my boyfriend had unprotected sex. I did use plan B the next morning but I took out the ring a week ago and I still havent started my period... I am also have some pregnancy symptoms. What do you think? Doctor: Hi dear, I have gone through your question and understand your concerns.Delay in the periods can be due to pregnancy or due to the effect of the emergency contraceptive pill you have taken, as it is known to delay the next expected periods by 2-3 weeks.I suggest you to get a urine pregnancy test done, if negative then you can safely wait for the periods for another2-3 weeks.Hope you found the answer helpful.Wishing you good health.Dr Deepti Verma"
},
{
"id": 109345,
"tgt": "What causes back pain, fever and stomach cramps?",
"src": "Patient: Hi doctor, please help me, i have being going through a pain in my lower back, and a fever of around 99 in some times of the day, and i feel some cramps in my right stomach, particularly after my meal, and recently i noticed a pain in my lower right abdomen but when i urinated it relaxed down, this thing just happened today! i have an appointment with my doctor on monday but till then i want some answer to relax down! Doctor: Back pain is not uncommon but according ur symptoms ..it may be urinary track infection..but it will confirm after urine test ..so try to take advise of doctor .."
},
{
"id": 148473,
"tgt": "Cerebral palsy, taking tablets. More information?",
"src": "Patient: Dr, In born time I am a cerebral palsy man both legs are not straghit. but now in 4 years iam a pixe patient .8 tablets are using night.and two tablets are using morning. but no fine.iam working internet cafe that time tension comming in some matter that disease comming in looking any property that time changes are comming and some time normal situation comming now iam using epsoval, escoplus. ab-dip, lopazesum, ab-nolpressure ,but no good fine doctor please give an information to this diseas? Doctor: Dear it is a progressive chronic illness.Unfortunatly no specific treatment exist.The treatment is just to reliaf symptoms but the desease is progressive.Wish you all the best"
},
{
"id": 100760,
"tgt": "What is the treatment for allergic rhinitis?",
"src": "Patient: Hello Doctor, Since childhood I have been suffering from continuous allergic rhinitis & sinus infection. for the last 3 years ihave chronic Urcaria . All the anti allergic and anti histamine medicatoion didn t give any cure to the disease only little relief to rhinitis only.can I be treated with Histaglobulin injections for a a year to get rid of this IGE mediated urticaria and rhinitis.Thanks alot doctor ,expecting reply. Varghese Innesent. Doctor: Hello,Welcome to HCM,Your history and symptoms are suggestive of allergy to dust mite. As you have undergone absolute eosinophil count (AEC), which is also evident of some allergic reactions in the body.These dust can stimulate the mast cells to release mediators like histamine which is producing all these symptoms.I would suggest you to undergo skin prick test to identify the proper dust causing all these symptoms. This will help to avoid the allergen otherwise you can go for immunotherapy.You can go for sub lingual immunotherapy (SLIT) or subcutaneous immunotherapy. For the present symptoms you can take Tab Montek LC, once daily in night for atleast 15 days.Thank you."
},
{
"id": 97727,
"tgt": "Is Dr. Ayurveda's Step Up formula good for increasing the height ?",
"src": "Patient: Sir, I am 32 yrs. old unmarried boy. My height is 5 feet 6 inches. I want my height to be increased. Is it any way to increase my height? Is Dr. Ayurveda's Step Up formula good for me. If not then what should I do regarding to increase my height. Please advice. Doctor: **If your growth stopped by itself at the age of 19 with no identifiable (pathological) cause, this means that growing ends of long bones are completely closed/fused and no possible further growth in terms of height is possible, 1. No Pharma company will ever describe his/her product not to be authentic, they are in market to sell ,earn and publicise their compound, and now even Actors have jumped into this marketing business (reason is simple-easy money and common people easily get influenced by their presence on show) and as you know 90% of knowledge is attained through visual (eyes) stimuli and this is the reason of showing their products on television.2. If height was a concern Sachin Tendulkar, Amitabh Bachhan would have been deeply depressed all their life.3. eat a healthy,balanced and nuritious diet and practice pranayam,yogasanas, back exercises that stretches the spine which helps in increasing height by 1\u201d-2\u201d inches.4. Contact with a yoga expert for various Yogasans which can be adopted and practiced in daily routine."
},
{
"id": 148584,
"tgt": "Increased pain with current shock type feeling in neck, shoulder. Have herniated disc, bulging disc. Any suggestions?",
"src": "Patient: I have have had documented herniated disc and bulging disc by MRI in neck. I have had two and the second showed some worsening of neck. Surgeon did not want to do surgery just for pain. I agree. Lately, I have had increase pain in the neck and left shoulder and an electrical current type feeling running down my neck and seems slightly to left. Can't find a comfortable position. Muscle relaxers and two aleve tablets do not help. Doctor: Dear you should urgently put cervikal collar,for at least three weeks and remove it during night.Help yourself with pregabalin 150 mg twice daily starting with 150 mg one week.If pain persist take one tablet Advil.I hope my advice with help you anyway I think the final solution of your problem will be surgery.Wish you all the best"
},
{
"id": 156561,
"tgt": "How long would magnesium citrate take to cure constipation in a person having daily doses of radiation for throat cancer?",
"src": "Patient: My boyfriend is going to take Magnesium Citrate, (10oz) for severe constipation. He is currently having daily doses of radiation for throat cancer. His Nurse did recommend this for constipation. He is 6ft 2in, 220 lbs. Can you estimate about how long before the laxative will take effect? Doctor: hi, mag.citrate is an osmotic purgative. it keeps water in the intestine so makes stools semisolid or liquid. we need to drink more water for better action. the dose can be increased according to need, too much dose will cause diarrhea or smaller dose will not be effective. along with that the intestinal movements should be adequate for progress of food bolus into gut further and expulsion of stools outside. another important fact is there should not be obstruction in the gut. the most common cause of such obstruction is 'fecoliths' in rectum. this is assessed by per-rectal examination. if fecoliths are present, they should be evacuated with the help of enema or may be manually. so consultation with a surgeon or a gastroenterologist will help u."
},
{
"id": 143514,
"tgt": "Suggest treatment for mild dementia",
"src": "Patient: I am the wife of a 91 year old man with mild dementia. He has a fungal infection on his penis which has not been cured with medications and his personal hygiene is difficult to manage. The urologist has suggest a dorsal slit. My husband does not want surgery but the doctor thinks it is necessary. Will myhusband, who is otherwise very healthy, be better off with a dorsal slit. Doctor: Hi, mild dementia isn't a problem. old age is like getting back to childhood so have to start in cooperating small mind games into your daily living Lego's, chess, snake & ladder, ludo, spellathon, etc. many such games will keep you occupied and also help you exercise your brain. it all depends on how you reintroduce these games into your life. CoMing to hygiene it's little tough to make them understand and also do. When we can toilet train a child we can also train and elderly to maintain hyeine unless he doesn't want to , it might that little help he needs to start learning and to maintain it.thank you"
},
{
"id": 107578,
"tgt": "Suggest treatment for back pain and fluttering sensation in the chest",
"src": "Patient: The past two or three days, I have randomly, not often, experienced this fluttering feeling in the right side of my chest. The fluttering last only less than 5 seconds but I was wondering if it meant something? and also, this could be connected so i ll add it, I have been having upper right back pain, again not often, for about a month or two and i m very unsure about that as well Doctor: Hi there.There could be degenerative changes in the spine and also chronic fatigue. Try to remain stress free. Eat healthy and exercise regularly. Keep well hydrated. Apply a hot water bag to the painful regions. Take pain killers. If no improvement, kindly consult your Orthopaedic surgeon for a proper evaluation."
},
{
"id": 97403,
"tgt": "Is there any treatment for chronic renal disease in Ayurveda or homeopathy?",
"src": "Patient: sirI had been operated for renal stones and hydronephrosis in right kidney in 2004 and 2007 and the kidney functioning was only 15% for right one .the creatinine level were normal even before operation.Since then i had been taking reckeweg renal drops R27last week i measured my BP which was 200/150 which is sure due to renal disease i had to go to check up for other tests next week so i wanted to know that what medicines should i take till my tests are over and is there any cure in homeopathy or ayurved for chronic renal disease Doctor: Hello Patient,Chronic Renal Disease is not a very easy disease to cure in any system of medicine. It is better to be honest than to give unrealistic expectations.However, what Homeopathy can definitely do is improve renal functioning, and in turn normalize the blood pressure, provided dialysis has not been started yet.Once renal function starts improving, then depending on the rate of improvement, the chances of cure can be gauged. But yes, Homeopathy has seen cases of chronic renal disease being cured.Let me know if you require any other help or medication.All the best!"
},
{
"id": 78747,
"tgt": "Suggest treatment for pulmonary embolism",
"src": "Patient: A few months, ago i was admitted to hospital with pain in my lower abdomen on my lift side, they found that i had some. Blood clots in my lungs, I went for a scan on my abdomen and pelvis, and went to my Doctors today for my results and was told that they was concerned that i had enlarged tubes to my liver, The Doctor was told to take another blood sample which he did. Don t under stand exactly what the problem is. Doctor: Thanks for your question on Health Care Magic. I can understand your concern.You probably are having pulmonary thromboembolism and pulmonary hypertension which can be treated after finding out the cause if any. You will require blood thinning medicine and medicines to control PHTN. Be under regular follow up for the same with your doctor.Don't worry, you will be alright. Hope I have solved your query. Wish you good health. Thanks."
},
{
"id": 180286,
"tgt": "Is increased blood pressure for a 7 year old something to worry ?",
"src": "Patient: Hello... I have a 7 year old daughter who went to her Pediatrician the other day and he stated that her blood pressure is rather high for a 52 pound female. I purchased a home blood pressure monitor and have been checking her several times a day. Her average is around 113/70. Sometimes when she is at rest it can be around 99/74. But when she is active during the day, it can be as high as 124/40. He is worried that this may be a sign of a kidney problem and I have set up an appointment with a specialist near the end of this month. If I read some of the publications, these numbers are not so far out of line. But, other government agencies have these numbers in the danger zone. She is in the 50% percentile of height, and since she is adopted, I don t have any family history. Are these blood pressure readings a reason for concern? What is the best source for blood pressure charts for children. Thank you, Jack Doctor: hi her blood pressure measurements are with in her age limits so no need to worry. you can get these charts online just google age vs blood pressure readings. you should worry if they are more than 95th percentile. for her age if serial resting blood pressure meant is greater than 119 you should show her to her doctor.hope this is helpful to you, rate it if you like it. take care"
},
{
"id": 197209,
"tgt": "How to stop masturbation?",
"src": "Patient: Hi I was 13-year-old boy before I engage the so called masturbation. Actually,it's not my intention to masturbate, it's my High school friend's influence. And now I am 18 now. I cant even stop it. Is it am I too sinner like that? I continually do it. need your advice. thank you Doctor: HelloThanks for query .It is natural and normal for every man to indulge in to masturbation at the age of adolescence .It is not harmful to help unless one gets indulged in habitual and frequent masturbation .Though it is harmless, excessive and frequent habitual masturbation does affect quality of erection and delays ejaculation .Please try to reduce the frequency of masturbation by keeping yourself busy in activities like sports ,reading ,social work etc so that you will not get free time to masturbate.Dr.Patil."
},
{
"id": 145337,
"tgt": "What causes fainting spell and blurring of vision?",
"src": "Patient: Hi. Recently I went to a concert with some friends. About alittle more than half way through I started to feel SUPER tired, and kind of nauseous. Then as I was standing in the crowd listening to music, my vision started to slowly go, and then I completely blacked out. Now this has happened once before, what does it mean? Doctor: Hello. I have been through your question and understand your concern.I think you experienced decreased blood pressure, which is quite common in overcrowded places.I think you should measure several times you blood pressure and if it tends to be low, you can increase your fluids uptake.Hope this helps. Please feel free using MHC again"
},
{
"id": 101305,
"tgt": "Suggest medication for asthma",
"src": "Patient: My son, 10 year old, is asthmatic. He is frequently getting sick these days. His asthma was diagnosed when he was only 8 months old. During these years he has improved. In last two years he was much better. For the last three months his frequency has increased. Doctor: Hello.Thank you for asking at HCM.I went through your son's history.Childhood asthma is a dynamic disease which has a tendency to improve and worsen over long time periods depending upon child's environmental changes, diet, medications, etc.If for last three months his frequency has increased, if his symptoms limits his daily activities/exercise or he has night-time cough more than once a week or he has to use his rescue inhaler (such as salbutamol or levosalbutamol) for more than twice a week, I would suggest him to start daily controller medications (or increase his controller medications if he is already taking them) for at least next 3 months. I will also suggest you to consult an Allergist-Immunologist who will test him for allergies and may suggest additional measures and allergen immunotherapy to improve asthma control.I want to emphasize on correct technique of using inhalers as this is crucial to ensure that adequate amount of drug is getting into his lungs.I would also suggest him regular breathing exercises, a healthy diet rich in vitamins and minerals and avoidance of dusts-smokes-air pollution as much as possible.Hope this will be helpful to you.Wish your son the best of the health.Regards."
},
{
"id": 50603,
"tgt": "Had peural fluid removed from lungs, creatinine level abnormal, urea level high. Is it organ failure?",
"src": "Patient: My uncle is 75 years old and had peural fluid removed from his lungs on the 14th April. He was given heavy doeses of antibiotics for the last 18 days to ensure that there is no recurrence of the fluid. When admitted his creatine level was 2.2 but today it is 3.4 and Urea has shot up to 288. No specific GFR reading was was done/calculated. Doctor feels that once the antibiotics are stopped his creatinine levels may drop. Waht do advise in the mean time .. is this a cause of major worry pointing to organ failure ? Doctor: Hello.Your uncle have 2 factors indicating poor prognosis in hisdisease, pneumonia: is over 65 years and has elevation of levels of urea.However their doctors will inform them about the current situation and prognosis.I wish you good health.(In case the answer would have been useful please indicate this)"
},
{
"id": 205142,
"tgt": "How can cognitive impairment be treated?",
"src": "Patient: I have recently had 4 major strokes which left me with cognizant deficits. I am having an extremely difficult time trying to live with a brain that does not work as it used to. I was formerly extremely independent and now have to ask for help, what would you suggest? I also lost my entire family in a 2 year period, I used to take care of them daily, now I don t know who I am or why I was allowed to live. I can t deal with it?? what do you suggest? I am no longer able to work and cannot afford to pay for your answer, I apologize for having wasted your time. I don t have any money for you to refund. I appreciate your time,but if looking into my predicament will help you deal with patients suffering from similar difficulties, I will answer any questions you may have. Doctor: Hi and thanks for question.i will suggest u that u have vascuhatlar dementia, in that case treatment will very few for memory loss. i will suggest u that u will start antidepressent which help u in mood elevation along with always keep diary with u and write down important thing in for daily activities so if u forget than u will remember from that. second don't be so hopeless and helpless because it's only due to your strokes.thanks"
},
{
"id": 156045,
"tgt": "How long a patient with gall bladder cancer and liver cancer survive?",
"src": "Patient: My mother was diagnosed with Gall Bladder cancer that spread to her liver 18 months ago. She had surgery at that time where they cut a section of the liver in an attempt remove it. Sadly that didn't work and the cancer is still there. She has recently had a stent put into her bile ducts to make her more comfortable. How much longer will we have her with us - weeks? months? Doctor: HiYour mother is suffering from stage 4 gall bladder cancer.Aim of treatment at this stage is palliative chemotherapy to improve survival and improve quality of life,As she has stent put in her bile ducts and already in stage 4 of the disease she will live on an average from 6 to 8 months.RegardsDR De"
},
{
"id": 37138,
"tgt": "What causes pus filled, itchy and red bumps on pubic region?",
"src": "Patient: I have noticed a small 2 or 3 red bumps on my pubic region and some pimple like bumps when I pop them they have a green/yellow tent to them. It is sometimes itchy they are minorly sore to the touch. Do I have an std or is it just ingrown hairs from shaving Doctor: Thankyou for the QuestionBrief...causes can be ruled out if bumps are seen by doctorDetail..STD can cause skin symptoms in pubic region but depends on sex life of a person,if sex is done with different partners and with out precautions .Also unhygenic measures( repeated shaving) can cause skin problems.My opinion would be ,if you notice increase number of same kind of bumps ,associated with fever and fatigue,you should consult your doctor.Mean while you can clean pubic area wilth antiseptic lotion daily and take good hygenic measures.Dr.Maheshwari"
},
{
"id": 55163,
"tgt": "Suggest treatment for gallbladder attack",
"src": "Patient: hello, I have had gall bladder attacks in the past and I do have scarring near the opening of the bile duct. I presently have very green stools over the last week. The last time this happened I ended up in emerg with a full gall bladder attack. I presently have right sided discomfort under my right shoulder blade (Muscular?). Should I be concerned? YYYY@YYYY Doctor: hi.noted history of right sided abdominal pain radiating to the shoulder area. it is possible that you have a gallstone disease. it is best if you consult with a doctor, preferably a gastroenterologist or a general surgeon, for physical examination and clinical evaluation. diagnostics (such as ultrasound, liver function tests, alkaline phosphatase, blood count, etc) and management (medical and/or surgical if indications are found) will be directed accordingly. if it is indeed a gallstone disease, definitive management is cholecystectomy (surgical removal of the gallbladder) which can be done laparoscopically (goldstandard) or thru open technique. low fat low salt diet is also recommended.hope this helps.good day!!~dr.kaye"
},
{
"id": 216906,
"tgt": "What causes painful rashes all over the chest,neck and arm pits?",
"src": "Patient: I have got some rash all over my chest neck under my arm pits and it feels like some one is puting a million pins in me i had it yesterday at the same time 630 and were gong by morning but this time thay hurt like a mother fu____ not itchy just pain Doctor: It's most probably a herpes lesion. Many a times post herpes infection, (which occurs pretty commonly during fevers) the virus gets lodged in the nerve plexus es. The distribution of lesion that you are speaking about may point to the localisation of the virus in your brachial plexus. However to confirm it we need to know1.a history of herpes infection. 2. A repeated similar event."
},
{
"id": 10480,
"tgt": "Suggest remedy for hair loss",
"src": "Patient: good evening sir/ madam.. i am suffering hair loss for about 6 months...intially it was abt 40-60 folicles a day...which i knew was usaul...but now i can observe my receding hair line...have gone to a tricology centre and the tricologist intially gave me just mineral supplements....and said that if it doesnt work...we will go with saw palmetto...but now i am in a worry that the treatment may have adverse side effects....and can u also tell me abt minoxidil....is it safe?? Doctor: Hello and Welcome to \u2018Ask A Doctor\u2019 service. I have reviewed your query and here is my advice. I do not know your gender. You must be having androgenetic alopecia if you are male. Take Finesteride daily for long time along with tab. Biotin thrice a day. Apply minoxidil solution twice a day on the scalp. If you are female, the disease may be telogen effluvium. Apply steroid lotion. Take biotin thrice a day with cap vitamin E for long time. If there is dandruff, Ketoconazole shampoo may be used. Almond oil may be used to oil the hairs. Hope this helps you. Dr.Ilyas Patel MD"
},
{
"id": 131646,
"tgt": "What cause soreness from neck to the fingers having diagnosed with Anxiety disorder?",
"src": "Patient: i have an anxiety disorder for the last 4 years but over the last year it has massively improved, I got the odd ache and pain now and again. But the last month i have had extremely store pain in my right arm which when sore a pain would shoot down to my fingers, But the last week it shot up to my neck and ever since ive had a sore next ( Right hand side ) and a sore head at the back. What should i do? Doctor: this is typically a problem with your cervical vertebra could be spondylosis or multiple cervical discs if pain increase in the morning then it is spondylosis if it increase through the day it is cervical discs also tension increase symptoms in both cases i recommend x ray or MRI if possible also start physical therapy as soon as possible Good Luck"
},
{
"id": 110474,
"tgt": "What causes lower back pain?",
"src": "Patient: My father has been having lower back pain, and after many trips to his chiropractor- he had xrays done- the chiro told him there was some darker shadowing he needed different angles of to ensure nothing was going on that would be concerning. What could this mean/be? He had stated if my dad ever had prostate problems he would be more concerned, but he never has...help? Doctor: Hi, thanks for posting your concern in the HCM.Back pain at this age may be due to several reasons. Commoner ones are- mechanical back pain, lumbar spondylosis, degenerative lumbar disease, UTI, Renal pathology or bowel problems.Therefore, a detailed history, physical examination and tests are needed for diagnosis.For the time being, my advice isXray LS Spine - Ap/lat (you haven't specified what Xray he has done. If the same was already done, you won't need to repeat)Complete blood count, urea, creatinine, Calcium, Vit D(25)Urine for routine examinationHe should have high fiber diet and Ispagulah husk 2 tsf in a cup of water at bedtime.Tab Paracetamol 500 mg twice daily for 3 days. Bed rest for 2 days.You should also consult your local physician for further evaluation. You may also need to consult an orthopaedic surgeon, if your physician feels that there is musculoskeletal problem.MRI of LS Spine may also be needed to rule out any spine abnormality.Regarding your concern about prostate, I would say that if there is any burning sensation during urination, difficulty in voiding, reduced flow, severe constipation he needs to have per rectal examination by your local doctor to rule out prostate enlargement. In that case you may need ultrasound examination and serum PSA level.For any further questions, please write back.Regards,Dr. Kaushik"
},
{
"id": 35944,
"tgt": "What causes typhoid during summer?",
"src": "Patient: hi, I stay in Gurgaon, India for last four years. Last three years i have had typhoid every year in summers. Even this year what started as a normal sunstroke fever later was diagnosed as typhoid. This too when i have taken booster dose just two months back. Doctor: Hello Typhoid fever is a systemic water born disease characterized by fever 7 abdominal pain and by dissemination of Salmonella typhi or paratyphi, for which humans are the only hosts .As you mentioned that you have fever in every summers ( so this may be due to infected water).Take care about that ( water boiled , cooled or sterile water to be used).Vaccinations are available as:Typhoid ty21, oral live attenuated 1 capsule every other day for 4 doses.Booster dose after 5 years .Typhoid Vi capsular polysaccharide , injectable (typhim ) 1 dose INTRAMUSCULAR .Booster dose after every 2 years.So this is a complete vaccination chart .The drug of choice in typhoid fever is ciprofloxacin 500 mg twice in day for 14 days .In few cases ceftrizone 2 gm for 7 days .Since all these are prescription drugs ,so consult your doctor and take a prescription.Good luck."
},
{
"id": 127314,
"tgt": "What causes pain in the knuckles?",
"src": "Patient: Hi. All of a sudden today I have had pain on two of my knuckles so bad I can t bend two fingers. It is throbbing from knuckles to almost tip of my fingers. It looks a little swollen but I didn t do anything to injure it Doctor: Hello and Welcome to \u2018Ask A Doctor\u2019 service. I have reviewed your query and here is my advice. The pain in the knuckles can be related to tendonitis. Taking a pain killer can help. Hope I have answered your query. Let me know if I can assist you further."
},
{
"id": 91739,
"tgt": "What could be the cause of discomfort in abdomen and tingling sensation in body?",
"src": "Patient: Hi doctor , I m having discomfort in lower right abdomen for few years . And for the last few months I have pain in the buttock and lateral side of the hip. I had blood test which showed elevated ESR of value 123, hb of 12.9 , mch 26.6 and MCHC 32.1 . I have got bit of tingling in my body especially in night time on feet . I m 33 years old male . Doctor: Hi.The pain in such a situation can be due to Psoas Abscess ( a muscle at the back of abdomen). You have a very high ESR indicating chronic problem.I would advise such a patient to undergo MRI of lower spine abdomen and hip area to confirm the diagnosis and go for further management. MY view goes into favor of ruling out the tuberculosis of the spine tracking down along the muscles.Please let us know the findings ."
},
{
"id": 78370,
"tgt": "Do i have PTB if my chest x-ray shows minimal PTB?",
"src": "Patient: I had 6 months treatment for my ptb and the last result of my sputum test is already negative, yesterday i went to the hospital for xray as requested by my employer and the result is still the same ptb minimal. Do i still have ptb and is it again contagious after 6 months of treatment Doctor: Thanks for your question on Health Care Magic. I can understand your concern. At present, you are mostly having old, healed, fibrotic lesion on chest x ray. After six months of treatment, active tb lesions on chest x ray heal. Healing takes place mostly in the form of fibrosis, calcification or both (fibro calcification). Complete resolution is rare in tuberculosis. So these healed areas appear as scar lesion on chest x ray. They will be seen as it is for lifetime. So presence of this fibrotic lesion is not suggestive of tuberculosis. Your sputum report is also negative. So you are mostly having healed lesion and not having active tb lesion. But if you really want to rule out active tuberculosis then get done CT thorax and bronchoscopy with BAL (bronchoalveolar lavage) analysis. If both these are negative for TB then you are not having active tuberculosis fir sure.Hope I have solved your query. I will be happy to help you further. Wish you good health. Thanks."
},
{
"id": 3058,
"tgt": "Suggest process to find out problem in conceiving",
"src": "Patient: Hi... i m 33 years old and not able to concieve from the past 9 - 10 months ... kindly suggest me the right procedure for treating with my problem as after all the check ups & tests, which are normal, my doctor has recommended me for IVF ... which is not affordable for me ... Kindly suggest me any other process with the help of which i can find out what the exact problem for not conceiveing ... Thanks Doctor: Hi welcome to the health care magic You have not mentioned which report has been done Following work up needed in infertility cases... -semen analysis(of husbsnd) -ESR -Serum TSH, prolactin, FSH, LH estimation -endometrial biopsy -Hysteroscopy-TVS USG etc If specific cause found than treated accordingly... Try to do frequent sexual intercourse around ovulation time If still can't conceive than assisted reproductive techniques used Take care Consult gynecologist with keeping these in mind"
},
{
"id": 9245,
"tgt": "Suggest treatment for dry skin and itching on body",
"src": "Patient: My son has continous itching all over his body. He use all various kind of body cream to help with his dry, hard and flaky skin, but nothing helps. Recently a doctor gave him pregnaone and benedryl and still that does not stop the itching. what do you advise. Doctor: Hello,Thank you for posting on HCM.It\u2019s very difficult to approach conclusive diagnosis unless for proper physical examination, anyways from your description it seems you are suffering from scabies.Scabies is a contagious disease caused due to mite and it spreads among people who are in close physical contact and by use of common household articles.Treatment is pretty straight foreword. Firstly, i would suggest treatment of your secondary infection (if any) with oral and topical antibiotics.The key to successful treatment of scabies lies in treating everyone simultaneously in the family and others who are in close physical contact. Your doctor would prescribe oral drugs like ivermectin which will kill the parasite. You might also be advised application of permethrin 5% cream and a corticosteroid with fusidic acid cream. Apply the medicated creams exactly as advised by your doctor. Take antihistaminics as advised to relieve itching and irritation. Avoid sharing of clothing and toiletries and launder all you bedding and clothes in warm water.Hope this will take care of your issue.Wish you best of health.Thank youDr Hardik Pitroda"
},
{
"id": 184324,
"tgt": "Suggest remedy for dental health problem",
"src": "Patient: Hi my name is Simone and about a month ago the inside of my bottom lip started to get raw. There are small bumps that I can't see but I can feel them in my mouth. Everytime I brush my teeth, the flouride burns my mouth a lot. I am only 15 and I'm really scared to know what is causing it. On my top lip on the inside is really sore, but I don't know why. The burning gets worse everyday. There are also cracks on the inside of my mouth and blood comes out of them. If I could have some help, that would be great. Thank you. Doctor: Thanks for your query, I have gone through your query.The possible causes for the burning sensation in the mouth could be because of lichenoiid reaction that is allergic reaction to the tooth paste or it can be because of angular chelitis secondary to anemia or it can be a aphthous ulcer. Consult a oral physician and get yourself examined to rule out these things.If it is a lichenoid reaction then you can change the tooth paste and use topical steroid like triamcinolone acetonide 0.1% 4-5times daily after food.If it is because of angular chelitis improve your nutritional status, consume lot of leafy vegetables.If it is aphtous ulcers then, you can take topical anesthetic and analgesics like anabel gel. apply 3-4 times daily before food and topical steroid like triamcinolone acetonide 0.1% 4-5times daily after food.I hope my answer will help you, take care."
},
{
"id": 88013,
"tgt": "Suggest medication for abdominal pain",
"src": "Patient: hey.. ive been feeling sick for a few months now quite bad but no vomiting.. but recently i have started wetting myself without feeling it but sometimes with lots of pain in my stomach and stomach pains.. and headaches..dizzyness..bleeding for about 3 weeks.. what could this be? Doctor: Hi.The wetting without feeling is a serious matter with a history of feeling sick for few months, stomach pains and headache, dizziness and bleeding for 3 weeks. This can probable be due to some Spinal cord problem and you need an MRI of the whole spine.Another cause can be local like cancer related to the uterus with wide spread. Hence a CT scan of the abdomen after a Gynecologist's internal check us is mandatory before it becomes late enough. You already are."
},
{
"id": 69921,
"tgt": "Suggest treatment for enlarged lymph node on head",
"src": "Patient: I discovered what I believe to be an enlarged lymph node at the back of my head.I discovered what I believe to be an enlarged lymph node at the back of my head.I discovered what I believe to be an enlarged lymph node at the back of my head.I discovered what I believe to be an enlarged lymph node at the back of my head. Doctor: Hi,If this is an enlarged lymph node then it is likely to reduce in size spontaneously. It is usually a common sign of infection. There is a possibility this is a sebaceous cyst. This is not serious and usually needs no treatment. If it becomes large or inflamed then see your doctor for examination. Regards,Dr K A Pottinger,MBChB FRCA"
},
{
"id": 100042,
"tgt": "What causes sneezing after exposure to sudden change in temprature?",
"src": "Patient: Hey, Doctor my husband sneezes badly after exposure to sudden change in temperature. Even if he enter in a warm area from the cold one, he gets irritation in the nose and starts sneezing. He is suffering from this problem from past 7-8 years. He has been using nasal decongestions which gives him temporary relief only. What should I do? Doctor: Hello,Thank you for asking at HCM.I went through your husband's history and would like to make suggestions for him as follows:1. I usually prescribe my such patients montelukast + levocetirizine for 14 days.2. Please wear personal protective device such as mask or simply cover nose by a cloth while anticipating change in temperature.3. I usually do not suggest nasal decongestants for such symptoms. Excessive use of nasal decongestants can cause persistent nose obstruction.Hope above suggestions will be helpful to you.Should you have any further query, please feel free to ask at HCM.Wish your husband a very good health ahead.Thank you & Regards."
},
{
"id": 79751,
"tgt": "Suggest treatment for Chronic Obstructive Pulmonary Disease",
"src": "Patient: I have been diagnosed with COPD emphaseamia since 2011..This year I am 70,but Iam an active 70 yr old female.. I quit smoking finally in 2013 (Dec) what should I expect in the breathlessness side in the following year, and should my flareups decrease? Doctor: thanks for asking your questioni completely understand your problemif you have shortness of breath while sitting or while walking a small distance , you have to take medications latest treatment recommendations are inhaled drugs like combination of long acting beta 2 agonists and steroids like formoterol and fluticasone plus antichlinergics like tiitropium . these are all inhaled drugs and their benefit is they do not have those threatning side effects like oral drugs and they act directly inthe lung. next and best add on management is pulmonary rehabilitation which improves each and every symptom of copd.pulmonary rehab means exercises that help lung to increase its endurance and decreases breathlessness. by using all these measures surely ur breathless ness will improvethanks/ regardsfeel free to ask more questionsmay god bless you with giod health"
},
{
"id": 97093,
"tgt": "Suggest remedy to remove tiny electronic device from larynx/epiglotis",
"src": "Patient: while i was sleeping i had an enemy put a tiny electronic device used for communication purposes into my mouth, i did not swallow it nor digest it. My research has shown that because i was sleeping it may have went down the wrong hole and is not stuck into my larynx/ epiglottis. Furthermore, It has been causing me severe pain every now and then, i have recently went for an X-ray and am scared if it does not show up on the screen that my doctor will not bring me for the surgery i need to remove it. if it is indeed stuck where i think it is,(when i speak through the remote i can hear it inside me) How & is it going to be possible for me to remove it myself? please help me as it has been an ongoing burden for months now and i fear for my life as every now and then i feel jolts of electricity and know it is causing me severe brain damage. The ones that did this to me are laughing and praying for it to damage my body enough to have brain damage for the rest of my life and end up in the psychiatric ward. please help me..HOW CAN I REMOVE IT FROM MY LARNYX/ EPIGLOTTIS MYSELF? Doctor: Hello, thanks for your query. any foreign body( electronic battery after ingestion , if not removed in time can cause breathlessness, lung infection, voiceful breathing, I feel you do not have any foreign body according to your symptoms. for confirmation u need to do xray or endoscopy by an ent surgeon. electronic devices after ingestion with liquid medium all around will not pass any current to any part. all the best. take care."
},
{
"id": 165373,
"tgt": "What could cause pain in left side of stomach?",
"src": "Patient: My Daughter has complained of stomach pain off and on for a about a year but now major problems, then at school she was bent over complaining of left side pain , She is hard to diagnosis cause she tolerates pain very well, she has had catoract surgery this year, teeth pulled due to infection in gums (who brushes her teeth well with moms help) not due to diet... complaining not one time right back to school with in a day. I also lost a child previous to her birth she lived for 3weeks with a dyphramitc hernia (spellings off sorry) i klnow that chances are rare for that and shes been to a heart doctor at birth and was fine but should i be concerned....? Also some one asked about diabetes since shes had some many issues with tummy pain eye problems , teeth problems and urinates alot....any answers Doctor: As your daughter is having abdominal pain since you should ask your doctor for detailed clinical examination and investigations.Ployurea, cataract, repeated infections can be caused due to Diabetes MellitusAsk your doctor for investigations like USG abdomen, blood investigations for metabolic diseases, A1C, GTT, LFT, KFT etc.to determine exact cause."
},
{
"id": 33433,
"tgt": "Suggest treatment for inflammation in inner groin",
"src": "Patient: Within a week ago I had inflammation in my inner groin. Then a mass appeared. It had the shape of a long cylinder. It then got worse until I finally burst. White puss, blood, and a cloudy maroon substance was produced. Now there is a clean hole where the mass broke. What is it? Doctor: It sounds like you had a fairly large abscess that finally ruptured. Now that it has spontaneously drained, you are left with the cavity or cyst that contained the pus and blood with its opening at the skin surface. If all the infection and inflammation is gone, the cavity and the hole should heal up in a week or two. Keep it clean and watch for the redevelopment of infection. If there is still inflammation or pus draining, you might need to see a doctor to be sure all infection is drained out and you may need antibiotics. Hope this helps."
},
{
"id": 36614,
"tgt": "Noticed a small hole which is itchy & painful above the anus",
"src": "Patient: I have a small hole above my anus. It's been there over a year now and itches and hurts every once and a while. It happened when a small bump started to itch and scratched it. Accidentally I scratched the top of the bump off and it has never healed back. Nothing oozes out and it doesn't bleed. Any ideas? Doctor: Hello,Welcome to HCMThanks for posting your query in HCM.Itching near anus may be due to variuos reasons.Skin conditions may affect the skin around the anus and cause itch - for example, eczema, psoriasis, lichen sclerosus and seborrhoeic dermatitis.Fungal infections, infections with bacteria, herpes infection, anal warts and some other sexually transmitted diseases may also cause this.Pilonidal sinus is also very common in this area.So,i would suggest you to keep the area clean and dry.Consult your family doctor and undergo physical examination for confirmation of diagnosis.Thank you"
},
{
"id": 55734,
"tgt": "Can I have gall bladder surgery with a hypo echoic structure on the pancreas?",
"src": "Patient: I am scheduled to have surgery on 5/22/14 to have my gall bladder removed, but I just received a note in the mail today from my gastroenterologist that the results from my 4/28/14 ultrasound showed a hypoechoic structure in my pancreas, and he wants me to make a follow-up appointment as soon as possible. Should I cancel the gall bladder surgery scheduled for 5/22? Doctor: Hi, dearI have gone through your question. I can understand your concern.You have hypo echoic structure in pancreas. Hypo echoic structure in head of pancreas can be normal variant, some cyst or other pathology. So it should be evaluated. If needed go for CT scan. It will clear all your doubts. If it is normal then you should proceed with your gall bladder surgery. If it is due to some pancreatic pathology then you should plan accordinglyHope I have answered your question, if you have any doubts then contact me at bit.ly/Drsanghvihardik, I will be happy to answer you.Thanks for using health care magic.Wish you a very good health."
},
{
"id": 175207,
"tgt": "What is the cause of red circles around the eyes?",
"src": "Patient: Afternoon,My 4 year old daughter has been looking very very pale over the past month & now has red circles around her eyes. She looks very drained & says that her back hurts. My wife has taken her to the doctors, urine tested & is clear. The doctor thinks its a viral infection, but would it go on for this long? Worried about her & just want to make sure nothing is seriously wrong.Thanks for your timeMark Doctor: Hi,From history it seems that she might be having severe anemia giving all signs and symptoms.Due to lack of iron and multivitamine deficiency one gets this type of problem and red circles around eyes.Go for Hemoglobin estimation.Go for stool test for ova and cyst of warms.After reports your doctor will go for treatment accordingly.Give her high nourishing protein food, fresh fruits and juices, green vegetables.Ok and take care."
},
{
"id": 166502,
"tgt": "Does head injury with no symptoms need medical attention?",
"src": "Patient: Hi, may I answer your health queries right now ? Please type your query here...My 2 year old banged his head on sunday I put a cold compress on the area and no bump came out just red spots. He did not vomit or show any symptoms of concussion, sleepiness etc. He has been well since. Should I still take him to a and e ? Doctor: Dear parent,I understand your concerns but you should monitor the child for a whole 24 hours after the injury. if the child experience dizziness or vomiting or headache then take the child immediately to the hospital for an urgent ct scan. if the child didn't experience any of these symptoms in the 24 hours following the injury then no need to worry. I hope I answered your question"
},
{
"id": 33777,
"tgt": "Suggest treatment for diarrhea",
"src": "Patient: Hi my 5 year old son is very warm and has a resting heart rate of 130 bpm looks very tired and has a cough my other son has dioreea and stomach cramps and my wife and i both have dioreea is it just a virus that will pass or should i be concerned.. Thankyou Doctor: Hi, dearI have gone through your question. I can understand your concern. You may have food poisoning. You should take plenty of water orally. ORS or lemon juice should be taken to maintain fluid and electrolytes. Drugs like ofloxacin plus ornidazole is useful. It is prescription based medicine so consult your doctor and take treatment accordingly. Hope I have answered your question, if you have doubt then I will be happy to answer. Thanks for using health care magic. Wish you a very good health."
},
{
"id": 16383,
"tgt": "What is the cause of recurring skin rash on the buttock ?",
"src": "Patient: Female, 34, i have skin rash on my right buttock it looks like blisters,it itches and hurts,i have used cream like betnovate n, tribotan,with ampiclox capsules but it keeps coming and goes after a while living the spot dark, sometimes it appears during my period or cold season, it frequently comes out around the same area. Doctor: Hi...dear nikky.., Thanks for choosing HCM.., Itchy blisters on buttock...is usually due to.., fungus...TENIA CORPORIS.., it is fungal..,contagious.., 1) Wash with ketaconazole soap.., 2) Terbinafine cream...(Terbicip-cipla).., daily 2 times..., 3) Tab fluconazole 150 mg weekly once for 8 weeks., 4) Thorough cleansing and dusting wih.., clotrimazole dusting powder...ok..thanQ"
},
{
"id": 134827,
"tgt": "Suggest treatment for swelling and bruises on finger",
"src": "Patient: 43 years old. Ring finger got caught somehow putting bike front tire on. Heard a snap sound from finger when finger dip hyperextended. Xray negative. Two weeks splint. Still swelling and bruising after 5 days ibuprofen 800. Do I still continue with the splinting? Doctor: Hello,I think you have developed mallet finger. In this condition there is tendon injury and that causes pain and swelling that lasts for some time three to four weeks. I would suggest you to continue wear splint for two more weeks. Burden needs to be continued 400 mg twice daily for seven to eight days.I hope these steps will be helpful for you.Let me know if there is any other followup questions.thanks"
},
{
"id": 106955,
"tgt": "Suggest treatment for severe back pain despite being treated with physical therapy",
"src": "Patient: I have been dealing with lower back pain for over two years now, and at first it was diagnosed as a leg descrepency and was treated with PT. In October the pain started coming back and spreading to my upper back, legs, flared up my arthritis and along with those i started getting major migranes. The lower back pain is sharp while the upper back pain (which spreads to my shoulders) and leg pain is more numb and tingly. I have had xrays whick say my back is fine and blood tests that say my SEDS are fine. Its just been a battle and no one seems to have any answers so im just hoping to find some here.F14 Doctor: hi sir/madam,Thanks for your question on Healthcare Magic.Low back pain may be found in mild form in case of anemia, sciatica, rheumatoid arthritis, hemorrhoids, urinary calculi, uterine disorders etc also.Line of treatment as per Ayurveda:-The general principles of treatment of vata dosha are adopted in case of katishoola (low back pain). It includes various measures to suit its varied clinical entities, stages and associated complaints.1. Snehana (oleation) \u2013 by sneha dhara (pouring oil), abhyanga (oil massage), avagaha (tub bath with oil or oleaginous medicaments), kati basti (retaining medicaments on the back) etc.2. Swedana (sudation) \u2013avagaha sweda (sweating treatment with tub bath), pizichil (kayaseka), nadi sweda (sudation through a tubular device \u2013 local sudation), panda sweda (sudation through medicated paste or powder) etc.3. Mridu Samshodhana -mild purgation.4. Basti (medicated enema) like eranda basti, vaitarana basti, pippalyadianuvasana basti etc.Formulations indicated in Low back ache as per Ayurveda:-1. Dashamoola kwatha.2.Maharasnadi kashaya.3. Rasnaerandadi kashaya.4. Sahacharadi kashaya.5. Gandharvahastadi kashaya.6. Trayodashanga guggulu.And for local application these oils will help:-1. Ksheerabala tail.2. Mahanarayana tail.3. Dhanwantaram tail.4. Maha narayan tail.Avoid these for better results:-1.Bitter, astringent and pungent food2.Cold water3.Fear4.Exhaustion5.Standing6.Driving7.Cold food and beverages etc.Yoga is also very helpful in treating your back ache.Now for your backache do these yoga aasana:-1. Ardha Matsyendrasana(Sitting Spinal Twist).2. Dhanurasana (Bow Pose).3. Marjariasana (Cat Stretch).4. Balasana(Child Pose).5. Halasana(Plow Pose).6. SUPTA MATSYENDRASANA(THE TWO KNEE SPINAL TWIST POSE).7.Adhomukha Shwanasana (Downward Facing Dog).8. Urdhva Mukha Svanasana(Upward Facing Dog Pose).9. Paschimottanasana(Seated Forward Fold).Follow these yogas you will get best results for your problems.Hope i was helpful.Have a healthy day."
},
{
"id": 218190,
"tgt": "Can Betnesol injection cause a sudden decrease in fetal movement at 35 weeks of pregnancy?",
"src": "Patient: Hello sir my wife is 35 week 2 days pregnant, yesterday we have 2 shots of betnesol injection at an interval of 12 hours dose 12mg but after that we noticed that baby is not moving frequently as he is doing before??My question is is it due to betnesol injection or at 35 week less movement are observed??? Doctor: Hi, Lesser fetal movements are observed when the baby grows bigger and starts settling in the uterus with the presenting part, usually the head, fixing in the pelvis. It can also happen when the baby is in distress. Since your wife has been advised steroids, it is likely that she is having a threatened preterm labor. In the scenario, fixation of the baby occurs earlier. The injection is not responsible for the situation. Instead, it helps to prevent respiratory complications should the baby be born preterm. Hope I have answered your query. Let me know if I can assist you further. Regards, Dr. Shanti Vennam, OBGYN"
},
{
"id": 13948,
"tgt": "Suggest remedy for itchy rash on body",
"src": "Patient: I have a rash on my stomach, back, hips. Recently also on my chest and tush. Currently on the end of a course of prednisone, which gave no relief and I feel miserable. Very itchy, worst in the warning. A biopsy on my back is positive for eczema. The rash on my front looks much different and is the itchiest. Doctors keep saying it s eczema, keep moisturizing. I have the moistest skin around, but the rash is growing and horribly itchy. Some spots feel like they are burning. Fabrics that rub against me all feel like sandpaper. Doctor: Hello, It can be eczema and can cause severe itching due to allergic reaction. So my suggestion is to consult an Emergency room and get evaluated and an allergy test can be done to rule out if it is due to allergic reaction. For now take anti allergic medicine like Levocetrizine to control itching. Apply steroid ointment over the rash and do i e packs. A steroid shot can help in improvement faster. Hope I have answered your query. Let me know if I can assist you further."
},
{
"id": 95709,
"tgt": "Why is there increasing pain while pressing near left kidney and abdomen ?",
"src": "Patient: 47 years and 5 9 with 76 kg weight having some kind of pain which increases with pressing and moving near left kidney and abdomene .wants to know the dose of metrogyl 400 Doctor: Hi the dose of metrogy 400 is 1tab thrice a day but plz tak the medication only if prescribed by doctor.first let a doctor examine u diagnose the cause of pain n then if he prescribes antibiotics u shud take it.plz do not selfmedicate thnx"
},
{
"id": 134808,
"tgt": "What causes numbness in the hip and the vagina?",
"src": "Patient: Hey today after sitting on the couch for about 20 mins I noticed the back of my right hip and the right side of my vagina felt numb. Almost like it was asleep. It s been 4 hours and it still feels numb. Also I have a pain in the right side of my pelvis that feels almost like gas pain but is getting increasingly worse. Any ideas what s going on? Doctor: Hello,I have studied your case and I think you have developed neuropraxia. In this condition there is transient pressure on the nerve. It is a reversible condition. I would suggest you to rub that part with mustard oil or ointment like volini gel.You need to take Pregabalin M tablet one time in night time. I hope this answer will be useful for you.Let me know if there is any other followup questions.thanks"
},
{
"id": 111464,
"tgt": "What causes pain in chest, shoulder and back?",
"src": "Patient: chest,shoulder,and backpain hi am david 22yrs male iam from kenya help me coz i am having pain in my chest shoulder and backpain i went to my doctor have an xray and she told me that my chest is clear so i was just wondering what is going on with me sory for my laguange Doctor: I had gone through the case and found that you must go for cervical vertebrae x- ray. It might be cervical spondylitis and pain radiate to shoulder and chest.Or it might be muscle spasm that trapped the nerveSo do neck and shoulder exercise, take mild painkiller and apply muscle relaxant gel.If pain is unbearable then go for physiotherapy.Hope my answer will be effective for you.Thanks ."
},
{
"id": 197662,
"tgt": "What causes stomach aches and purple spot on penis head?",
"src": "Patient: Hello! I have some questions about my well-being. Around 4 days ago, i went to go get tested for HIV because i was very concerned. Luckily, i came out negative but im still worrying nonetheless. Ive had 2 different sex partners in the past month and 1 few days after getting tested. Im still stressing this. Ive been having stomach aches lately, maybe since 2-3 days ago. They re not very bad aches, just those aches that will occur, then leave again, but sometimes i would go to the bathroom. Then i finished showering lately, and i found this dark-purple looking spot on the side of my penis head, it looks like something similar to a bruise, and i got very nervous & worried. My health anxiety became worst at this point. What could all this be? Im very nervous and i cant stop thinking about it! Help me! Thanks! Doctor: Hi. Welcome.It is not possible to comment on skin lesion without examination, so kindly see local doctor or upload the pictures in here to help you with specific advice.For stomach pain you can take capsule omeprazole 20 mg once daily empty stomach, before breakfast for week.This provisional advice provided by me is based entirely on the input provided to me. I would suggest examination by the doctor.I hope this suggestion might be of some help to you. Please accept my answer in case you do not have further queries.Take care."
},
{
"id": 175888,
"tgt": "Suggest treatment for depression in child",
"src": "Patient: My 16 year old daughter has been very depressed. We have taken her to the dr. and they gave her prozac for depression and anxiety. We also have her in counseling, although we have only went twice, and it s not very intense. She has also engaged in other risky behavior with drugs and alcohol and sneaking out while staying at other friends house. She is having a really rough time at school, with peers and teachers, and I just don t know what else to do.... Doctor: Hello,Welcome to HCM,Your son is of 16 years old and he is having lack of interest and feeling that he will blow up.There must be something that is bothering him. Another important point here is about his nature as a child and adolescent. Does he like to talk to people? Does he have any phobias? Does he have friends? Does he is less interactive as a child? whether he was outgoing and playful child? Does he say or do anything unusual recently? Like talking alone, seeing things, animals, people? does he say that somebody is behind him to harm him? Does he fearful?Important here is to rule any possibility of personality disorder and psychotic disorder. If these symptoms are not present then he might be having depression and he needs drugs and psychotherapy. So first go for consultation to child and adolescent psychiatrist and ask for help. He will guide you further about what can be done and what are the options. Thank you."
},
{
"id": 49373,
"tgt": "What are the precautions to be taken to avoid kidney failure?",
"src": "Patient: My sister is experiencing much loss of hair . She is 84 years old and is taking 50 mg of metoprolol for high blood pressure. She is also taking 75 mg of levoghiyoxine oxide for thyroid. She has lost most of her hair within several weeks. She states that she uriinates frequently ( about every 20 minutes). Her kidney count was 100 last month and is now 100.09. The doctor stated she is getting kidney failure - she gave her water pills because her feet and legs were swollen and gained 8 lbs in one week and also experienced shortness of breath. PLEASE let me know if you can help. Doctor: Thank you for your question.Kidney function usually decreases with age.Kidney failure can be due to several reasons but the most common ones are hypertension and Type 2 Diabetes mellitus.The symptoms which you mention are suggestive of generalised edema.Kidney failure progression can be slowed down by having stringent hypertension control measures and thyroxine level control.Antoxidants like acetylcysteine are prescribed in patients with renal failure but they cannot completely halt kidney failure.consult your physician for more details"
},
{
"id": 77486,
"tgt": "Should test to detect TB be done after its treatment?",
"src": "Patient: I am 34 year old female.i am unable to conceive.i diagnosed pcr+ve (Tb) last year. I have taken 9months medicine course. Now i want to know that is it necessary to diagnose for Tb again whether it is cured or not. Also tell me can i undergo for mantoux test for TB. Doctor: Hi thanks for asking question.You have taken full course of antituberculous treatment for 9 month.If you have decreasing symptoms and condition not deteriorating then no need for confirmation of tuberculosis.You have not mentioned the site of tuberculosis.If you have genital TB and still not conceiving then confirmation for negative test done.Mantoux test is not for diagnosis of tuberculosis.It is test for detection hypersensitivity to tuberculous antigen.I hope you understand my concern.Wish you good health.Dr.parth"
},
{
"id": 82225,
"tgt": "What does suspicious opacity in lower right lung field in chest X ray mean?",
"src": "Patient: I got my PA X ray chest on 26.06.2014. In X ray report all six parameters are normal in last Dr. commented suspicious opacity in lower right lung field? nipple shadow. What does it means. I am over 50 yrs of age and in very good healthy condition & still do running for 1.5 km daily and walking for 5/6 km daily. I have never any health problem except infections due to influenza. Occasionally cough. Pl ease advise me. Doctor: Thanks for your query on HCM.In my opinion you should not worry about the report.As nipple can give rise to rounded soft tisdue shadow in some cases.Better to go for lateral chest x ray as in this x ray if any shadow is there than it will appear inside the rib cage. And if it is nipple shadow than in lateral x ray it will appear outside the rib cage. So by this method we can differentiate. Another way is doing CT Thorax.But I don't think anything worrisome. It is just a nipple shadow nothing else."
},
{
"id": 154833,
"tgt": "Noticed a scar on the right lung and biopsied with Aden carcinoma",
"src": "Patient: Thanks you doctor -- I was just biopsied with Aden-carcinoma in a scar on my right lung, none small cell -- hopefully stage one -- need a good cancer doctor working out of St Joseph Mission Hospital in Mission Viejo Ca 92692 -- want the best latest schools and experienced who works out of that hospital or related hospital -- Thanks Jack Doctor: Hi, dearI have gone through your question. I can understand your concern. You have non small cell carcinoma of lung. It has good prognosis. Treatment options are surgery, chemotherapy or receptor antagonist therapy. It can be squamous cell carcinoma or adenocarcinoma. Treatment depends on the type. Consult your doctor and plan accordingly. Hope I have answered your question, if you have doubt then I will be happy to answer. Thanks for using health care magic. Wish you a very good health."
},
{
"id": 119572,
"tgt": "What causes restless joint pain and involuntary jerking?",
"src": "Patient: I am 22 years of age. For the last 7 to 8 months i have been going through a lot. relentless joint pain, random dizzy spells to the point of almost passing out. Shaking episodes that last for hours at a time. Involantary jerking. An overall feeling of being sick. My kidnyes seem to be hurting and acheing. Blured vision from time to time. back and neck pain that bring on migrains. memory loss and siver stomach problems. It is very over welming, and i don t know what is going on. When i try to sleep i eather wake up with siver pain or just seem to wake up every hour on the hour its like my body won t let me stay asleep. Do you have any idea what is going on? Doctor: Hi, Muscle spasm occurs in response to some painful pathology ranging from a herniated disc to a vertebral collapse...you need to get a thorough clinical examination along with an MRI done to know about the exact cause of the disease. Take care. Hope I have answered your question. Let me know if I can assist you further. Regards, Dr. Rohan Shanker Tiwari, Orthopedic Surgeon"
},
{
"id": 203290,
"tgt": "What do you suggest for back pain considering i masturbate twice a day?",
"src": "Patient: sir please tell elp me out this problem me if i can masturbate twice in a day & iam in age of 30+ so and also iam unmarried so help me out of ths problem that if i had low back pain problem last 10 years is only due to mastrbration or some other factors because i did masturbrate of 10years when i was sudied in school Doctor: HIThank for asking to HCMThis is real myth that masturbate cause the bad backache, your backache could be because of muscular pain due to more stress, and mental tension, keep the stress level low and try the following medicines,1) Tab Ibuprofen 400 mg three times in day2) Tab Chlorzoxazone three times in day no need to worry about this your back pain will get alright soon, take care and have good time."
},
{
"id": 167262,
"tgt": "What causes recurrent vomiting after consuming food?",
"src": "Patient: My four-year-old vomits very easily. This morning he ate food he eats all the time -- pancakes, grapes, milk -- was perfectly fine, threw up, was perfectly fine again. This happens every now and again. Is it a weak gag reflex? Could it be more serious stomach issues? Doctor: gastric outlet obstruction may b the cause of recerrent vomitting following feeding and the child has good appetite despite vomitting. u should go for a upper gi endoscopy to look for the cause."
},
{
"id": 160184,
"tgt": "I have been suffering from dull ache in the left lower abdomen",
"src": "Patient: Dear Doc/Friends I ve been suffering from a constant on and off dull ache in my lower abdomen ,especially on the left lower side. It sometimes also extends to my lower back and left leg,so much so, that this dull ache keeps me awake for a long time at night. I ve been following a treatment for a complicated UTI infection,and have been on oral courses of Augmentine,then to IM course of Amekacin 1gm for 5 days and now on to a 10day course of Melfur. My stomach seeems to be bloated and I feel fatigued and restless. Please advise, as I m also concerned on a chance of suffering from serious illnesses like Cancer,as Ovarian Cancer runs in our family. I m 30years old,married for the last 5 years with no Children/ pregnancy . I appreciate your time that you took out for me. Regards, Bhavana Doctor: hi bhavana, i'll say some ayurvedic medicines which u can take along with ur medication take gokshura guggulu-2tabs twice a day chandra prabha vati-1tab twice a day u take vaiswanara choornam -1tsp with luke warm water before food by using these medicines and rest u definitely get rid of ur problm any doubts contact me-09441294599"
},
{
"id": 126683,
"tgt": "What causes swollen feet while on Hydromorphone?",
"src": "Patient: My husband has mesothelioma cancer in his right lung. He was diagnosed a year and a half ago. He did 4 Chemo treatments over a year ago, but got too sick. He was experiencing severe pain 2 months ago, and MDAnderson did 10 days of radiation, which shrunk the tumors considerably, but he is still experiencing quite a bit of pain. He takes methodone every 12 hours, gabapentin 300 mg in the morning and again at 3, and 600 mg at 9 p.m. He can take hydromorphone 4 mg if needed, usually takes 1 in the afternoon, and sometimes 1-2 during the night. He prefers to take Tylenol instead, and was taking 8 per day, but Dr. told him to cut way back on that, but he is still taking 4-6 per day. Tonight his feet are swollen REALLY big, especially the right one. I have him soaking his feet in room temp water with some arthritus and sport cool blue ice soaking liquid with alcohol and capsium . what else should I do? Doctor: Hi, The swelling of feet may not be related to intake of Image result for Hydromorphone. This can be related to lymphedema caused by cancer itself or other cardiac or renal causes that need to be detected after proper investigations. Hope I have answered your query. Let me know if I can assist you further. Regards, Dr. Praveen Tayal, Orthopaedic Surgeon"
},
{
"id": 164135,
"tgt": "Suggest treatment for congenital growth hormone deficiency",
"src": "Patient: Hi, may I answer your health queries right now ? Please type your query here... My grandson (1 year 6 months) has been dignosed with congenital growth hormone deficiency. We are based at Mumbai, India. I wish to have an idea about the approximate cost of treatment and the duration of treatment. I shall be much obliged to receive this information, doctor please. My email id is YYYY@YYYY Thanking you and with regards R Vijaynathan Doctor: Hi... I am a consultant in Kanchi Kamakoti CHILDS Trust Hospital in Chennai. YOU NEED TO HAVE THE GROWTH HORMONE THERAPY UNDER THE GUIDANCE OF AN ENDOCRINOLOGIST. THE APPROXIMATE COST WOULD BE AROUND 10 LACS PER ANNUM.This is just an approximation. If you could come over to Chennai and meet me, I will put you on to our Endocrinologist who is very good and experienced at this. We might get the medicine also for a lower cost.Regards - Dr. Sumanth"
},
{
"id": 97813,
"tgt": "Itchy skin rashes, prescribed livocitrizen and cotosteroids, Homeopathy",
"src": "Patient: hai, facing itchy skin rashes from last 3 months...all test done tbc is normal, IgE is normal..nil haemoparasites. only globulin count is 3.7 in LFT while RFT is normal...dermotologist given livocitrizen for 1 month and one shot of 40 mg cotosteroids, she said it is just allergy or bug bite nothing elese but rashes are continue..2-3 each day and each substidized in 4-5 days..itching is much ... and at present I am on homeopathy... Doctor: Hello, Thanks for posting to HCM,Itching can be prevented only to the extent that the underlying cause(s) of the itching can be prevented. For example, careful use of sunscreen products can prevent the itching .Anti-itch creams and lotions like camphor.Use ice pak.Regards"
},
{
"id": 162614,
"tgt": "Should the formula milk of an infant suffering from cough be changed?",
"src": "Patient: Hi my 7month old doughter has a bad cough at night.it started when she was six months when she had to move to number two..at that time shd was taking infa care formula now we changed her to lactogen and she still have this cough at night and her tempereture is fine.should we change her to another formula?If yes please advise us which one will be best. Doctor: Hello and Welcome to \u2018Ask A Doctor\u2019 service. I have reviewed your query and here is my advice. It is very unlikely that the formula is the cause of the cough. This could, however, be a case of mild reflex. Add a teaspoon of baby cereal to the bottle. This will often solve the problem. If it doesn't, contact your doctor. Hope I have answered your query. Let me know if I can assist you further."
},
{
"id": 162205,
"tgt": "What causes prominent veins on body in a child after chiari malformation surgery?",
"src": "Patient: My daughter is 12 years old and she got out of the shower last night and her legs were blueish/purple and veins all over her body were highly noticable (Bright Colored). The color is still faintly there but it shows up more after a shower. She has not pain or tingling but she did have Chiari Malformation surgery a year ago. Could this be a complication or what is this?? Doctor: Hi, By what you say I feel that this could be ambient temperature induced reticular formation on the skin which is physiological and does not require any treatment at all. I do not think this is related to the surgery which has been performed 1 year back. This is medically called status marmoratus but at the same time to be 100% sure I would suggest that you upload an image. Hope I have answered your query. Let me know if I can assist you further. Regards, Dr. Sumanth Amperayani, Pediatrician, Pulmonology"
},
{
"id": 178150,
"tgt": "Suffering from fever,stomach flu & vomiting",
"src": "Patient: My ten year old granddaughter has had fevers for over two months off and on. The illness started with stomach flu, throwing up and fever. There seems to be a space of one to two weeks between the fevers. Sometimes she feels like throwing up and other times she doesn t. Today she has a fever of 101. We ve gone to her pediatrician, nothing showed up on blood tests, except an elevated c blood protein. Nothing on urine tests or fecal matter. More blood and urine tests are being done, we had them done on Friday, however I m getting concerned that she might have the beginnings of appendicitis. What do you think? Doctor: your graddaughtr seams to have chronic gastrointestinal infection . however it is diffuclt some times to treat such infections orally .it is better to treat with intravenous antibiotic .and IV fluids. regarding appendicits abdominal scan along with blood counts will click the diagnosis"
},
{
"id": 21479,
"tgt": "Can Norvasc cause headache?",
"src": "Patient: I was put on the blood pressure medication Norvasc 5mg almost 2 weeks ago.. After one week on the medicine, I began to have swelling in my my ankles, knees, and fingers, which I knew that was one of the side effects, but at the same time as the swelling began, I started having headaches every day... They last throughout the day. Usually by bedtime they have eased, and I do pretty good until the next morning after I have taken the medication.. Could the Norvasc be causing my headaches? I go back to the doctor in 9 more days, and I am wondering if I should try to deal with the headaches until my appointment... Also, would it be ok to take anything to help the headache? Thanks so much. Doctor: HI ThereNORVASC contains AMLODIPINE and its a very good drug to control high blood pressure but unfortunately like other medicines it also has its side effects which varies in intensity from patient to patient.Ankle swelling and Headaches are both common side effects of AMLODIPINE and if these side effects become bothersome then the anti hypertensives needs to be changed to some other class of drugs like ACE Inhibitors or ARB's Or Beta blockers.I would like to advise you to personally discuss this with your treating cardiologist. Till that time you can reduce the dose to half, it will relieve your symptoms upto some extent.I wish you good health"
},
{
"id": 173095,
"tgt": "Suggest treatment for a small black spot on the scalp of a child",
"src": "Patient: My six month old son has a very small black spot that appeared on his scalp a few days ago. It is about the size and shape of a strawberry seed. It is not raised, but looks as if it is something to be wiped off (like a speck of dirt). Is this something we should bring him into his pediatrician to check out? Doctor: Hi, I had gone through your question and understand your concerns. You should be worried but not too much.If so, you could have a fungus infection. Is the hair loss uniform or patchy? In short, you should see a dermatologist for a precise diagnosis and proper treatment. You can try to apply iodine 2 times a day for 5 days.Hope this answers your question. If you have additional questions or follow up questions then please do not hesitate in writing to us. I will be happy to answer your questions. Wishing you good health."
},
{
"id": 12584,
"tgt": "What is the permanent remedy for psoriasis ?",
"src": "Patient: i m the patient of PSOURIES from last 10 years., Do you have permanant solution of it and the main cause of it., what should i have do., ? Doctor: Hi..dear Manoj.., Thanks for choosing HCM.., PSORIASIS...it is inflammatory disorder with genetic predisposition.., So there is no permenant treatment for this.., only...manageable..., Cause is unknown...disturbances in the immune system... will play major role.., can control with these.., 1) SORVATE cream..(Glenmark)....it is non steroidal , safe and long term use..., good respond with this..,night times application 2) ACITRIN...it is Acetretin 25 mg daily for 1 month...expensive but safe.., good compliance.. 3) Mosturizing cream....CETRABEN, ELOVERA, MOISTURIZING SILK (Wallace) morning times... will give good result.., thanQ"
},
{
"id": 112999,
"tgt": "Lower back pain, nausea, dizziness. Getting black discharge with white particles in urine. CT scan advised. Suggestion?",
"src": "Patient: Hey, I have had this problem for a while. It occurs every 3 months or so. I have pain on my left side and down my lower back. My discharge is black and I have white pecks in my pee a lot. At one point the white specks were so big it looked like cottage cheese. I've had about every type of test except for a CT scan. Doctors keep sending me to the ER to get one but they wont give me one. My white blood cell count is very high. I feel dizzy and nauseated along with this process also. What is wrong? Doctor: Hello Thanks for the query. Thick white discharge in urine and increased WBC COUNT, FEVER and nausea suggest urinary tract infection. I suggest you to consult surgeon and get done Urine microscopy and culture sensitivity. Thank you"
},
{
"id": 188880,
"tgt": "Have abscessed tooth and throbbing pain. Taking Penicillin and Ibuprofen. Using heating pad. Suggestions?",
"src": "Patient: I have a abscessed tooth it has been going on for over a few days and the throbbing pain I had is gone but now my cheek where my lower jaw is its swollen only thing I have is pressure pain from the swelling. I am taking a antibiotic (Penicillin) and Ibuprofen also I am doing warm salt water rinses and using a heating pad at night when I sleep. I wanted to know if what I am doing is helping the situation or making things worse. I plan on getting the teeth removed but I can't until I am able to get money to pay for the extraction. Doctor: hiwarm saline rinses are fine.. consult your dentist for the dosage of your medication.. because medication is fine but if antibiotics (penicillin) dosage is not proper, its of no use and can be dangerous also..try to check out the tooth involved.. in case, its carious, it will require root canal treatment.. if not it might require periodontal treatment... sometimes both treatment parts are required... but dentist will proceed for above procedures after your swelling get subside i.e. after irrigation and proper medication...so take care and do not take it lightly... visit your dentist..."
},
{
"id": 100379,
"tgt": "Suggest medicine to get relief from swelling on skin",
"src": "Patient: I have some sort of allergy where my skin gets swollen like mosquito bites & it happen all over my body. it itches a lot &and goes away in about 3/4 hours. I'm not sure it is just only because of dust, as often it simply happened out of nowhere. please suggest me a medicine (capsule, tablet) for instant relief. I have taken Cetirizin tablets & seems to help a bit, but I'm not sure it has any real effect or side-effects. Doctor: You have urticaria .cetrizine does not have any long term side effects. But you need a blood work up and allergy evaluation."
},
{
"id": 145902,
"tgt": "Suggest treatment for vertigo",
"src": "Patient: Hi ! I just turned to my right side in bed and felt so very sick ! Felt the world was spinning horribly around me ! It happened for the first time and twice when I turned onto the same position. I am female, 59 yrs and overweight, and my height is 5 6 Doctor: You may be suffering from a condition known as Benign Paroxysmal Positional Vertigo (BPPV). Your symptoms are typical for this condition and so is your age. In order to solve this problem you should see a neurologist for advice on maneuvers designed to get rid of the symptoms. Medication is not a great option in my opinion since it really fails to address the real REASON behind the disorder whereas the maneuvers and exercises that the neurologist can show you really get to the root of the problem. Many ER physicians and internists will reach for a drug called Meclizine or trade name Antivert (in the U.S.). I find the majority of patients get little to no improvement in their condition so I don't recommend its use. If you would like more specific information on the maneuvers/exercises I'm referring to then, please contact me through my webpage on this site at: bit.ly/drdariushsaghafi and I can respond in more detail.If this information contained useful information for your question I would greatly appreciate some written feedback and a STAR RATING."
},
{
"id": 96658,
"tgt": "Suggest treatment for forehead injury",
"src": "Patient: My 6-yo daughter fell flat on her face on sat and have wounds on her forehead, side of her eyebrow and on the Sid elf her nose and nostril under. She seems fine and her paeditrician gave her ok to go ahead for school today. She came back 6 hours later with a slight swelling between her eyes and visible blue-black dark eye circle. Is this something to worry about and should I bring her to a & e? Doctor: HelloThank You for contacting HCM.After an injury blood accumulates under the skin. With the passage of time color of blood changes along with the overlying skin which is a normal process. I would suggest you not to worry. It will take some time to heal completely. Meanwhile you should take extra care and keep her in front of your eyes.> take a cloth and warm it over flame and put it over the forehead. It will give soothing effect to her> Continue to give analgesics as advised by your doctor.Report to hospital if:> There is loss of consciousness> Blurred vision> She becomes lethargic> She Vomits> Bleeding from wound or nose> Any unusual symptom Hope this answers your question. If you have additional questions or follow up questions then please do not hesitate in writing to us. Wishing her good health."
},
{
"id": 50814,
"tgt": "Has abscess formed near butt, kidney infection and jaundice symptoms. Diabetic. Recommendations?",
"src": "Patient: Hi, recently my uncle admitted in hospital to cure the abscess formed near the butt. Later by viewing the reports doctors reported he has infection in his kidney also a minor symptoms of jaundice. could you let me know what kind of infection it is and how the pus formed.he has diabetes since 20 years also to wound the heal and reduce the sugar level he rigorously used the insulin before he hospitalised Doctor: Hello, It is a known fact that diabetic patients\u00a0 are at higher risk for various infections than nondiabetic patients.\u00a0 Besides generalized impairments of immunity, other nonimmunologic, anatomically specific factors may contribute to an increased infection risk. Macrovascular \u00a0disease and microvascular dysfunction may result in compromised local circulation leading to delayed response to infection and impaired wound healing. Unawareness of lower extremity trauma due to sensory \u00a0neuropathy may result in inadequate attention to minor wounds and subsequent increased infection risk. \u00a0Incomplete bladder emptying due to autonomic neuropathy permits urinary colonization by microorganisms and high glucose concentration in the urine promotes the unhindered growth of bacteria resulting in various types of genito- urinary tract infections. Treatment lies in .... ....Adequate and meticulous control of blood sugar levels with injectable insulins. ....Proper anti biotic therapy,after culture and sensitivity report,for proper amount of time. ....In case of genitiourinary tract infection,prolonged anti biotic therapy. Thanks"
},
{
"id": 214041,
"tgt": "Anxiety get worse at night",
"src": "Patient: why does my anxiety get worse at night ? Doctor: During daytime usually people are busy in their work. Anxeity occurs in night due to exccsive thinking on same process. Hence worsens."
},
{
"id": 211800,
"tgt": "On medication for OCD. No improvement, worsened condition. Should smoking stopped while on medication?",
"src": "Patient: Hello sir, I am suffering from OCD since one and half years I'm taking medications as follows Clonil SR 75 mg at bed time, seranata 100 mg tablets three times a day and one riscon forte at bed time. I'm not responding well to this treatment and often conditions some times worsens, is it better to quit smoking while taking these medications. And suggest me the best treatment. Doctor: HiYou are getting a concoction of medicines . Neither of these is in full therapeutic dose so no wonder you have not shown improvement. If I were to treat you, I would taper off Clonil over a period of 2 weeks, then riscon forte over 1 week and see how you feel with 300 mg Sertraline ( your correct dose) . you should wait for 6-8 weeks before expecting any improvement because OCD needs higher doses for very long periods to initiate improvement. if you show some improveement, it can be hiked to 350 -400 mg for another 12 weeks before really thinking of change in medicines. It should be mixed with tab clonazepam as required and ERP DR SAATIISH JHUNTRRAA"
},
{
"id": 158996,
"tgt": "Has cancer and done chemotherapy. Has increased CEA levels. Reason?",
"src": "Patient: My father os a cancer patient( colon cancer stage b2), he is done with his 5 cycles of chemotherapy , but his recent tests shows that his CEA level is increased to 8.7 fron 3.7 within 2 months, but my concerned doctors dnt find it a reason to worry,, what may be the reason for such significant rise please Advise me if there issomething serious Doctor: Hello! Thank you for the query. CEA level is used to control possible colon cancer regrow or metastases appearance. If the person who has colon cancer history gets increased CEA level, possible metastases should be localized or ruled out. So this should be the reason of concern. Moreover, B2 stage gives high rate of survivals so it is necessary and worth to keep looking. The best way to do that is PET scan which will localize possible metastases. If this test is not available for your father, chest and abdominal CT with contrast should be performed. Hope this will help. Regards."
},
{
"id": 44615,
"tgt": "Can a male be fertile with a appendicitis infection ?",
"src": "Patient: can a male still be fertile if he had a appendisitis infection My husband and i have been trying to have a baby and have not been successful yet he had a appendisitis infection and had to go through scope surgery to get it taken out is it possible that was why we couldnt concieve? Very confused and frustrated want answers already Doctor: Hi Welcome to health care magic \u00a0\u00a0\u00a0\u00a0\u00a0appendicitis & infertility have no relation at all .Appendicitis don`t play any role in causing infertility.So get investigated to find the right reason & get it treated.Frustration is no solution/cure for infertility,it is right treatment that will help. \u2018Hope I have answered your query, I will be available to answer your follow up queries, \u201cWish you Good Health and speedy recovery\u201d Disclaimer"
},
{
"id": 160338,
"tgt": "Side effact of tobacco. How can I recover my mouth muscles ?",
"src": "Patient: hi i m 25 yr old, due to tobacco i cant open my mouth properly and my cheek look like a old man, now wht i do ? so how can i recover my mouth muscles Doctor: the first step is stop using tobacco and then consult a ent specialist who will phsically examine and assess and provide you with follow up treatment"
},
{
"id": 143341,
"tgt": "What causes tremor in hand after a shoulder joint replacement?",
"src": "Patient: My husband is 61 and had right shoulder joint replacement surgery six months ago, after a work-related injury he suffered five years ago. Immediately after surgery he noticed a tremor in his right arm. He will notice it when he is cooking and trying to slice something or when he uses his computer mouse or tries to play his guitar. He just saw a neurologist today and he said no Parkinson s or anything like that. He is thinking maybe something with the brain from anesthesia during the surgery and wants to do a brain MRI now. Can this tremor be caused from the surgery? He never had any tremor until right after the surgery and it s not in his other hand, legs, etc. Doctor: Hello and welcome to \u2018Ask A Doctor\u2019 service. I have reviewed your query and here is my advice. It might be possible during surgery any nerve might have been damaged at the level of brachial plexus (it a network of nerve). Due to which there has been weakness in his arm which shows it elf s a tremor while doing strenuous or any fine motor activity. You can contact physiotherapist as in physiotherapy there is a machine called as muscle stimulator which can help in strengthening of muscles and prevent muscle wasting. Along with exercises like (strengthening and active movements) of shoulder, this condition can be taken care of. Hope you will find the answer useful. Let me know if I can assist you further.Regards, Dr. Harsh Swarup"
},
{
"id": 200358,
"tgt": "What causes blood blisters on the penis foreskin after sex?",
"src": "Patient: Hello Doctor!! OK after a long day and night of sex with my Lady. woke up this morning with what appears to be a blood blister on the foreskin. I m 53 been tested for all diseases and all negative. like I said I was fine yesterday before the sex. question what can I do until I see my doctor. thank you Doctor: hello,i think this is a STD.i shall advise you not to do any intercourse before you consult your physician,dont worry.only antibiotic treaatment will help both you and your partner to get rid of it.hope this is helpful.regards."
},
{
"id": 189368,
"tgt": "Have salty feeling in mouth. Following a healthy diet. What could be the cause of salty saliva?",
"src": "Patient: 24 years old, healthy, active female. BMI normal. I ve been starting to have a salty feeling in my mouth which started about a week ago. It s presence is constant. I think that the saliva I secrete might be salty. It never does away even after brushing my teeth or chewing gum. I think that it may be caused by my diet but am not sure. I eat grains for lunch with vegetables and a protein for dinner with vegetables. In between I eat fruit and yogurt as snacks. I eat two eggs for breakfast with some salt added (but that is the only time I use salt). I think that I might be ingesting too much sodium. The vegetables I eat always come from cans. The only fresh thing that I eat apart from fruits (apples and bananas) are cherry tomatoes in my main meals. Recently, a cavity filling fell out of one of my molars leaving a big hole there. I try to clean it properly since I cannot get to a dentist where I currently am and will only be able to see one come May. Luckily, there s no pain or sensitivity. What could be the cause of this salty saliva? Doctor: Hello and welcome , The salty taste in mouth can be due to- Altered salivary secretion. Impacted wisdom tooth. Regressive changes in tooth. Sinus infections. I would suggest you to get a thorough examination by a dentist. Thorough scaling is adviced. Decayed teeth has to be restored soon to prevent progression of caries in exposed cavity. Take complete course of medicines too. Take balanced diet and plenty of fluid has to be taken. Hope this helps."
},
{
"id": 106812,
"tgt": "How can severe backache be treated?",
"src": "Patient: Sir,i am 37years old women.for past six months i had serious backpain in disk bulging(doctor told that disk water content level decresed).in amway can cure with this problem.use glucosamine, chondroition and msm complex? Any side effects?please answer me sir. Doctor: Hello,Back pain can definitely be cured, if we know the reason causing it. Back pain for 6 months may be due to disc degeneration or instability in the lower lumbar spine.Diagnosis needs to be confirmed by:1. An x-ray of the lumbosacral spine anteroposterior and lateral views to rule out bony abnormalities.2. MRI of the lumbosacral spine to check for soft tissue like disc and ligament pathology. Both can be done at the radiology center nearby you.If both are normal, then the pain may be due to muscle weakness, and you need physiotherapy. Visit expert an orthopedic spine specialist is recommended with reports.If there is disc degeneration and prolapse physiotherapy helps a lot except in case software nerve compression due to disc prolapse. Glucosamine and chondroitin are cartilage supplements and do not have major side effects.Hope I have answered your query. Let me know if I can assist you further.Regards,Dr. Jayesh Vaza"
},
{
"id": 46571,
"tgt": "Suggest medicine to improve serum creatinine",
"src": "Patient: My mother Serum Creatinine is 2.2 nad hb is 8.1 at the moment and doctor is giving one tablet renolog + one injection epocept 4000 U follwed with shelcal and olsar 40 (for BP) + matilda,, so i want to know how critical her disese is now...as i am not getting what is d best to get her improved or are we on right track with these medicines Mother age is 61 Doctor: Hi, I had gone through your question and understand your concerns. Looking at her creatinine level I can tell you that the disease is not that critical doest need dialysis at this time and medical management which you are taking is sufficient. If she is my patient I will definitely do the same and you are on the right track with those medications. Please keep an eye on the creatinine level do regular investigations and make sure the creatinine wont go high."
},
{
"id": 159504,
"tgt": "Polio, scoliosis, lipoma growing in legs and hands. Headache. Suggestions?",
"src": "Patient: Hi, I have several physical problem. My age is 26. 1. When I was 2 years old I got polio and still now i am carrying this. I can not walk. I use wheel chair. My full lower portion and left hand is affected by polio. Right hand is okay. for this polio my hips joint is also displaced. My both legs and left hand are thin. But I am used to with this problem and so far managed my life. 2. For last 5/6 years I have huge back pain. I thought as I can not stand out and walk may be for this reason i have this pain but when I x-ray my back then Doctor told me that I got Scoliosis . My upper spine is cured 30 degree and lower portion is curved 35 degree, so I have ahuge back pain. For this back pain I used a brace for 2 years but it was very uncomfortable and sometime it cause breathing problem, so leave it. But still living with this huge back pain and continuing my regular activity. 2. My another problem is Lipoma , Benign tumor . Now I am very much tensed for this. Though I got that first lipoma about 7/8 years ago in my left thigh and few grew up after that in my back but I wasn t concerned for these. But no a day these are growing in my both leg, thigh, hands, most of them are still small but 4/5 are big. I dont have any pain yet but very much tensed for this growing. I heard that there is a fewer chance to turn this lipoma as lipocircoma. I haven t take any medicine for this yet. But for reducing fat I am drinking Aloe Vera Juice 2 times a day and Apple Cider Vinegar once a day. More over frequently I have Headache. My eating habit is as usual. Please be kind to give me the best suggestions specially for the lipoma, I am very much worried for this. If any further information need about me then please be kind to as. I am waiting for the answers. Thank you. Jahid. Doctor: Hi Jahid, two things, lipomas in the tummy are prone for cancerous growth. Secondly there is nothing which can prevent them turning malignant. But if your sonogram of tummy shows lipoma, you need to do 3 monthly sonogram to watch on it. Rest other lipomas do not need much attention. Important ting is that you should be bothered of it all the times. Take care."
},
{
"id": 119567,
"tgt": "Should the titanium forearm plate be removed?",
"src": "Patient: hi i am 17 years old, 6 foot 1, 180 pounds, and was wondering if i would be able to get a titanium plate located in my forearm removed? i had surgery to correct a 6 degree over turn of my forearm while in the initial healing process. they inserted a titanium plate in my forearm to correct this, can or should i get this removed? Doctor: Hello, If implant was inserted more then 5 year back it would be safe to remove that. Although there are no any strong recommendation theoritically. But to prevent any future complications like infections or loosening of implant, implant can be removed after 5 years when they sub-served there purpose. Take care. Hope I have answered your question. Let me know if I can assist you further. Regards, Dr. Mukesh Tiwari, Orthopedic Surgeon"
},
{
"id": 71244,
"tgt": "What causes recurrent cough, cold and throat soreness?",
"src": "Patient: Sir, my son has a problem of frequent cold, cough and sore throat. this time he has cough for more than 40 days, sometimes acute, usually dry. Montaux test is normal (6X7 mm after 48 hours). Chest x-ray, abdomen USG, IgE and ESR etc are also normal. Kindly guide be what can be cause of this cough and what can be possible line of treatment. Doctor: Hello and Welcome to \u2018Ask A Doctor\u2019 service.I have reviewed your query and here is my advice.First of all no need to worry about lung infection or tuberculosis as his chest x-ray is normal. Cough variant asthma can also cause chronic coughing. And IgE can be normal in some asthma cases. So better to consult a pulmonologist and get done PFT (Pulmonary Function Test). If PFT is showing obstructive defect then it is due to asthma. He may need inhaled bronchodilators (formoterol or salmeterol) and inhaled corticosteroid (ICS) (budesonide or fluticasone). Hope I have answered your query. Let me know if I can assist you further.Regards, \u00a0\u00a0\u00a0\u00a0\u00a0Dr. Kaushal Bhavsar"
},
{
"id": 43102,
"tgt": "What are the chances of getting pregnant with IUI versus natural methods?",
"src": "Patient: Hi I m 25 n have pcos n I had fsh injection from 5th cd , now on my 21st cd I have 24,17,14mm and 2to3more small size follicles, now doc ask me to wait for natural rupture of follicles n then she'll do iui. I just wanted to know when will that possibly rupture . what are the chances of getting pregnant with iui n naturally? Doctor: HI, When you have intercourse the chances of pregnancy is around 4 to 8 percent. If you get IUI done it will increase to 15 to 20 percent. Regards"
},
{
"id": 214912,
"tgt": "Why has my baby's weight reduced drastically ? Is there a difference between Nan and Lactogen?",
"src": "Patient: Hello Doctor, My Baby boy is 1 month 22 days old. His birth weight was 3.1Kgs, but thereafter it reduced to 2.80 kg in next 10-12 days and after 1 month he was 3 Kgs. When we consulted our local Pediatrician , earlier he said theres nothing to worry, but after 1 month since he was only 3 Kg.. he suggested us to take Nan or Lactogen 1. At that point of time i was not aware about the difference in those two formulas. Now after 1st week of Lactogen (30ml, 3 times a day) he was weighing 3.2 kg. And now for last 2 weeks we are giving him 45ml, 4 times a day. He is passing stool only after 4-5 days, but he pee s atleast 10 times a day. Should i switch to Nan, as it is easily digestible..or should i continue with Lactogen 1. Doctor: Hi baby father, nothing to worry, some time babys wt. reduce for 1 or 2 week, after ward it increse slowly. I may like to suggest u to give him cow or befellow milk. I think baby will pass the motion regularly and wt. may also increase. If mother milk is there, it will be the best option."
},
{
"id": 193832,
"tgt": "What causes inflammation at penis tip?",
"src": "Patient: i have a slight rash inside my pee hole. I am 33 years old. I went to doctor 2 months ago because of itching when I urinate. He tested me for std's and tested my urine. My std test was negative. he stated i had a mild bladder infection and gave me antibiotics. The itching stopped after completion of medicine, but I still noticed discomfort. I looked really close last night at my penis and I noticed it was kinda red on the inside at the tip of my penis Doctor: Hello, Clotrimazole (brand name Lotrimin) is an antifungal medication commonly used in the treatment of fungal infections. In your case, it may be an STD (sexually transmitted disease). Chlamydia is a sexually transmitted disease sometimes causing painful urination, penile discharge and more. Gonorrhea is a sexually transmitted disease and can cause painful urination and penile discharge. In cases of infection, a diagnosis can be made by taking a urine sample or urethral swab, and can usually be cleared by a course of antibiotics. In the case of a sexually transmitted infection, it is recommended that sexual partners are also treated. Antibiotic treatment will be helpful for you. Continue applying antifungal ointment. The most secure way of prevention is definitely sustaining from sexual intercourse. Another option is to be in a long-term monogamous relationship with an uninfected partner. Hope I have answered your query. Let me know if I can assist you further. Take care Regards, Dr Iven Romic Rommstein, General Surgeon"
},
{
"id": 119941,
"tgt": "Is bp of 142/62 normal during peripheral spondylosis?",
"src": "Patient: My father has peripheral spondylosis (ankylosing spondylitis) and isolated systolic hypertension. The medicines that he is taking for AS are:1. Sulphasalazine 2. Methotrexate 3. Methyl prednisolone 4. Etoricoxib And for hypertension he is taking Telma-H.We have been monitoring his blood pressure for the past weeks, and the average has come to about 142/62. Is this status of systolic and diastolic pressure a cause to worry? What should we do at this point? We have just moved to Bangalore, India; and we are yet to find a doctor to consult on hypertension. Any suggestion of a doctor s name would be a great help. Doctor: Hi, hypertension and ankylosing spondylitis is not related to each other. Your father's blood pressure is on the borderline. It is not something to worry about but medicine dose has to be increased to decrease the blood pressure. Hope I have answered your question. Let me know if I can assist you further. Regards, Dr. Jaideep Gaver, Orthopedic Surgeon"
},
{
"id": 115227,
"tgt": "Why is my BP constant at 180-120?",
"src": "Patient: Doctor My Blood Pressure from last month remain constant at 180-120 while I dont know from how long I got this problem because most of the time it is asymptomatic and some time I feel tightness of abdomen. Due to some problem of skin I was used Anti histamin for more than 10 years injectabale Avil andv tab .Will you reply me If need any more history.Thanks Tnmnmbr> Doctor: Hi, dearI have gone through your question. I can understand your concern.Your blood pressure is very high. Life threatening events like myocardial infarction or stroke can occurs with this high blood pressure.You should consult your doctor and start anti hypertensive medicines.Losartan, amlodipin, atenolol are some of the drugs commonly used. You should consult your doctor and start them according to his advice.Hope I have answered your question, if you have any doubts then contact me at bit.ly/Drsanghvihardik, I will be happy to answer you.Thanks for using health care magic.Wish you a very good health."
},
{
"id": 26875,
"tgt": "Should i be concerned about the BP of 155/88 range?",
"src": "Patient: Good dayI apologize in advance for asking a question, after seeing a Dr at our local clinic. I am 29 and has had hypertension since my early teens. I was told that I am too young for medication amd was treated symptomatically.. ie . Headaches, dizziness etc.. I had my daughter on 28weeks due to pre eclampsia, which our local clinic and Drs missed, as my weight (123kg) apparently warrants high blood pressure.This week I nearly broke my neck due to an fainting episode, and since the fall, i still dont feel much better, in actual fact worse.Today i had a checkup at our clinic and the Dr said that 155/88 is normal, although i remember my previous Dr told me that from 150 + i should immediately consult a Dr, as this is dangerous.. I feel terrible, and they say everything is normal. Is there anything I might be missing or not understand about hypertension.Your feedback will be much appreciated. Kind regards Louise Doctor: Hello, Louise! Thank you for your question and welcome to HCM. I understand your concern. There are some things I would like to share with you in relationship with hypertension. Normal blood pressure ranges from 100-139/60-89 mmHg. Now, in certain moments in a 24-hours cycle, there are a lot of changes in psychological, physiological and hormonal peaks and nadirs in our organism, that can be detected. All of these factors determine our blood pressure and it is variable. Also, there is a phenomenon that is called ''white collar hypertension'', which is characterized by high blood pressure figures every time the patient sees the doctor and/or a sphygmomanometer gets attached to their arm. However, just to be thorough, I would recommend you to wear a 24 or 48-hour blood pressure monitor, to precisely determine whether this is your constant blood pressure or not, and, if it is, proper treatment should be installed. I can see that this figures come as worrisome events to you. The stress and/or anxiety generated by this state, produces high blood pressure figures. I recommend you to take the proper measure of immediately losing weight. You have to modify your eating and lifestyle habits: an hour a day of physical activity, no alcohol, no animal fat, no red meat but more fish and chicken, no fast food, more fruit and vegetables. With a combination of confidence and self-initiative, I believe you will succeed. I hope I was of help. Take care. Kind regards, Dr. Meriton"
},
{
"id": 45450,
"tgt": "I have nightfall after masturbation also",
"src": "Patient: I used to masterbate earlier 4 to 5 times a day ,past an year i am not doing it but still if i watch any sex sceen or if i have a dream of so i starts flowing from my pennis ,mainly in sleep how can i solve it....... Doctor: having sex is the only solution. once you get married you can have sex with your partner daily which will solve your most of the problems of flowing in the pennis at night"
},
{
"id": 146307,
"tgt": "Could lumbar scoliosis cause face tingling due to nerve pinching?",
"src": "Patient: For a few days, I have experienced intermittent tingling along the right side of my face. It usually starts in the neck, then goes up to my jaw and beyond, even to my ear lobe. No pain, just a tingling. At first I thought a spider was crawling all over the lower right side of my face, or that hair was swiping that part of my face (but my hair is very short, so that s not it!). I also have severe scoliosis (lumbar) with a compensatory hump on my right shoulder. Could this be related, maybe with nerves being pinched a little? Doctor: The compensatory hump COULD be part of the problem....COULD be....but the lumbar curvature.....no relationship to what's going on in the face. And it's not NERVES that are being pinched that's causing the face thing...it would specifically be the sensory portion of the trigeminal nerve (CN V) in which a small portion of it dips down into the upper cervical cord before making kind of a U-turn and coming back up to innervate structures in the neck and face. So that tingling in the face COULD be related to the hump in the right shoulder which could be twisting the upper spinal cord where that cranial nerve travels everyday to work and as a result causing your symptoms.If you'd like to write specific questions to my attention for personal answers please look me up at:bit.ly/drdariushsaghafiIf you like this answer and it gave you what you're looking for could you do me a favor by a STAR RATING as well as some written feedback on the answer I've given you."
},
{
"id": 97970,
"tgt": "Is it possible to cure hepatitis C with alternative medicines ?",
"src": "Patient: i am suffering from hep c for the last 6 yrs..tried combination treatment of ribavirin peg interferon for six mths..about 2 mths back i i had been detected wid the same virus and moreover the viral load has increased to 2.7 million. wat shd be the symptoms at this stage and how to cope with it.i dont want the treatment to repeat,as it is too much expensive and i vannot afford the same..Please guide if i can find a trustfull treatment in Ayurveda or Homeopathy... Doctor: 1. You haven't told about the MODE of Transmission by which you get infected, because it is not transmitted by Sexual Intercourse (very rare) 2.your place of origin, because more than 80% patients with HCV in India are due to blood transfusion or hemodialysis. 3. you haven't mentioned about the symptoms?? 4. even is the virus is removed from the blood, \"sustained virologic response\" will be there. 5. It will end either with Cirrhosis of Liver or Liver CA. 6. No Vaccination is available till date, and because we are measuring viral load in the blood and not in the liver cells thus HCV viral load doesn't tell you anything that how fast hepatitis is progressing. 7. Avoid spicy,oily,heavy food,take sweet,bitter vegetables and ripe fruits, Pranayam breathing is helpful. 8. In Ayurveda we have three classes of drugs: i. Anti-viral of which Bhoomyamalaki (Phyllanthus nirurii) and Kutki (Picrorrhiza kurroa) are best ii. Hepato protective of which Bhringraj (Eclipta alba) and Ghritkumari (Aloe vera) are best. iii. Liver Detoxifying of which chirayata (Swertia chirata) is best, you can consult an Ayurveda Physician there for the same."
},
{
"id": 174864,
"tgt": "Should I continue Econorm in a child if it has given some relief from vomiting?",
"src": "Patient: Hi...my 3 and 1/2 year old son has vomitted 4 times in the past 4 hours. He is throwing up everything that he is eating...I have given hime one sachet of econorm at around 7.30pm since then he has not vomitted. should i given hime another sachet now? Doctor: You can give another sachet now, no issues with the same. But I doubt whether econorm is the cause of stoppage of the vomiting. You can continue econorm for a few days. If the vomiting recurs, giving some antiemetic like ondansetron should be helpful. Giving ORS (oral rehydration solution) to prevent dehydration is important."
},
{
"id": 20343,
"tgt": "What causes palpitations in heart?",
"src": "Patient: i had palpitations on my heart. i went to the ER. i followed up with my doctor couple days later. he refered me to a cardiologist. i called the cardiologist but no open appont. til the end of feb. but my track season starts in 1 week. i feel great no sudden palpitations can i ask my doctor to clear me from my physical so i can participate? Doctor: You need to perform a rest and stress ECG before having the idoneity to partecipate to competition. Best regards"
},
{
"id": 189374,
"tgt": "Does morphine helps to numb the nerve ?",
"src": "Patient: would morphine be given to a patient that is difficult to numb the nerves would morphine be given to a patient that is difficult to numb the nerves Doctor: Hi Welcome to HCM I would like to tell you that morphine is very potent analgesic but its an opioid drug with many side-effects & also habit forming so its generally avoided. Secondly if you are talking about anesthesia then i would like to tell you that lignocaine is preferred as local anesthesia. Morphine is never given to anaesthetise. Consult your dentist before taking any medicine. Take Care Regards Dr.Neha"
},
{
"id": 201294,
"tgt": "How to get rid of semen leakage after urinating?",
"src": "Patient: my sperm count is 30 milion/ml,colour-pearly white,ph-alkaline,active motile-10%,sluggish motile-10%,non motile 80%,morphology norm form-95%,pus cells 08-10/HPF,epithelial cells 2-4/HPF, i tried 4 times for consive child but failure another problem -sperm comesout after urinate when i pressurized please help me............... Doctor: Hello Your report suggests semen infection with low sperm motility.Infection is indicated by presence of excessive pus cells in semen.You need proper clinical examination and further investigations.You need routine hemogram,RBS,culture and sensitivity of semen,colour doppler of scrotum.Proper antibiotics depend upon culture and sensitivity report.Motility is also low.Normally it should be at least 55 %.You have only 20 % motility.Sperm count is normal.Normally sperm count should be at least 20 millions/ml.Your sperm count is 30 million/ml,so count is normal.Motility is expected to improve with control of infection.If infertility persists then assisted fertilization techniques can be considered.It is normal that small amount of semen comes out during straining.Get well soon. Take Care Dr.Indu Bhushan"
},
{
"id": 187031,
"tgt": "How long does it take for the stitches to dissolve in mouth?",
"src": "Patient: I split my face between my nose and lip. I had 11 stitches outside and 8 inside. This happened on the 20th of December. I had the outside stitches taken out but the inside still remain. How long will it take for the stitches to dissolve in mouth and how should I care for the inside of my mouth? Doctor: Hello, Welcome Thanks for consulting HCM, I have gone through your query, as you are asking about stiches if it is absorable it will start dissolving in two to theee month Have you have confirmed this with your doctor is it absorable ? Consult your doctor and go removal if it is non absorable .Hope this will help you"
},
{
"id": 185878,
"tgt": "Does clindamycin help to treat tooth infection?",
"src": "Patient: Can I get Clindamycin on line for a tooth infection?I have an infected tooth and Clindamycin has helped me in the past. I live rurally and the few dentists in the area won't call in an antibiotic and it is hard to get in to be seen. It is the weekend and I am in great pain. Can you help me please? Doctor: Hello,Clindamycin is my choice to treat an infected tooth when a patient is allergic to Amoxicillin. Dentists cannot write a prescription or call in medication to a pharmacy without seeing a patient to determine a diagnosis that would require an antibiotic. I cannot tell you how to obtain Clindamycin without a presription. Do you have a dentist and would you be considered a patient of record? This is the only situation where I would refer to my chart information including your medical history and possibly be able to call the pharmacy.I am concerned that you indicate a great deal of pain. Please indicate that you have an emergency situation when calling an office. You could see a general dentist or specialist such as a oral surgeon or endodontist. You can go to a hospital emergency room or urgent care facility. You will most likely will be referred to a dentist, but can be given antibiotics and pain medication if needed. Infections can be serious and controlling the spread of an infection is necessary to your overall health.Please continue to attempt to be seen by a medical provider. Antibiotics will assist in controlling the spread of an infection, but you will need to address the source of the infection to eliminate the infection. Infections can reoccur and usually become worse. Over the counter anti-inflammatory medications may offer you some immediate relief, but this is temporary and you need to seek care.I hope this answer will help you. I am vailble to assist you further. Please do not hesitate to ask."
},
{
"id": 138344,
"tgt": "What causes swollen feet and ankles with numbness of toes?",
"src": "Patient: What causes swollen feet and ankles and numbness of toes. My feet kill me. I have been to doctors galore and have had every test done. I did however have 2 bands positive for lymes disease in the later stage. Now doc says neuropathy can come with lymes. I m still am not myself. Does neuropathy cause swelling in the feet and ankles and numbness of the toes? This is how it all started. I was tested for MS and sometimes I think that I have it with other unusal symtoms that I get like water dripping down my leg. However I was tested with MRI s of the brain and spine. Thanks Doctor: Hello, Thanks for writing to us, I have read your query,YES THIS CAN BE NEUROPATHY POST LYMES. Change your diet and remove inflammatory foods such as citrus fruits, dairy products and wheat that may aggravate peripheral neuropathy. Simultaneously increase your intake of potassium and calcium-rich foods such as bananas and avocados and green leafy vegetable.Vitamin B12 is important in the maintenance of health and normal functioning of the nerve tissue and deficiency of it can lead to numbness. You can increase B12 level by taking meat and fish, or you can take Inj methylcobalamine.Hope this answers your query. If you have additional questions or follow up queries then please do not hesitate in writing to us. I will be happy to answer your queries. Wishing you good health.Take care."
},
{
"id": 102822,
"tgt": "How long it takes to cure bronchitis and mild asthma?",
"src": "Patient: I have (usually) mild asthma. For several weeks I had a combination of deep chest infection, brinchitis and asthma. The infection eventually cleared up with the help of antibiotics but the asthma is not yet controlled. How long is this situation likely to continue? Many thanks for your help. Doctor: Hi,Usually respiratory tract infection precipitates asthmatic attack.Now as infection controlled gradually asthma attack will be alright.Continue with bronchodilator medicine.Add steroid for some time to control attack quickly.Do deep breathing exercises daily.Ok and take care."
},
{
"id": 16009,
"tgt": "Rashes spreading from the arm to shoulder. No pain. Is it Addison's disease?",
"src": "Patient: Hi...my name is Betsy. About...I want to say...5 years ago...I developed this odd rash on the inside of my upper arm...close to my underarm . It happened while I was at the beach. It went away...and I don t know if its linked or not...but I started to see bruise spots there. And then on the other side of my upper arm to the shoulder area. It spread to my upper thighs...and now I have them on my stomach . Some are big...some are small. I have had them for about 4 years I wanna say...and I am still starting to see more. I found 3 more. I have done blood work and they say that everything is fine. They do not hurt at all. I have been researching off and on and found Addison s Disease...but I am not too sure. Has anyone seen this? Doctor: hi Betsy..Thanks for your query. Addison\u2019s disease is a rare and chronic endocrine disorder in which the adrenal glands do not produce sufficient steroid hormones.The most common symptoms are fatigue, light headedness upon standing, muscle weakness, fever, weight loss etc.as of skin changes, hyper pigmentation is noted in sun-exposed areas, as well as darkening of the palmar creases, sites of friction, recent scars, the vermilion border of the lips, and genital skin.Are you noting any of these?The skin changes you have mentioned , do not point towards the above and i don't think that can be the only basis of the suspicion of addison's disease. However, you have not described the 'odd' rash you are having .But since it is a chronic disorder persisting since 4 years , it is necessary you visit a dermatologist and undergo a biopsy to know the exact nature of the eruptions. Hope this helps..Take care!"
},
{
"id": 120732,
"tgt": "Suggest remedy for rise in WBC count in RA patient",
"src": "Patient: Hi I am a patient of RA for 10 y.Now my Leukocyte count is around 12.5.for the last 3 months .my temp is 37.5 degree c .I have no pain or swelling now for the last 5 y except occasional pains .I am on methotraxate 2.5 mg weekly .I am worredbout my WBC count. Doctor: Hello, If you are not having any other problem like stress then this slightly raised leucocyte count is just the feature of high activity of your RA factor. You may better leave it on your treating physician to judge this .It will in no way going to harm you. Hope I have answered your query. Let me know if I can assist you further. Take care Regards, Dr. Mukesh Tiwari"
},
{
"id": 190596,
"tgt": "White tab under tongue, painful, bleeding. What is it?",
"src": "Patient: I have a white tab under my tounge that is not only painful when i touch it but bleeds as well. not sure what it is, or why I got it. Can you give me any advise? Doctor: Hi Thanks for posting your query There are all chances you have developed oral ulcers These could be due to constipation, lack of Viamin B Complex especially Folic Acid, Injury from dental braces etc The common remedy is treat the cause In any case a good Folic acid pill for 3-4 weeks should be ok Hope this helps Am available for any followup queries If there are no further doubts, do accept my reply and rate it Get Well Soon"
},
{
"id": 72675,
"tgt": "Suggest treatment for chest pain with shortness of breath",
"src": "Patient: Hello my friend Angela is experiencing chest pains when she breathes. She has felt this way for the past couple of days. She is 25 years old and does not smoke. Moderate drink. Maybe a glass of win here and there. She says she can breathe but it is hard when her chest is hurting. She has no fever. Doctor: Thanks for your question on Healthcare Magic.I can understand your concern. Chest pain with breathing difficulty in young individual (25 years old) is seen commonly with asthma.So get done clinical examination of respiratory system and PFT (Pulmonary Function Test).If PFT is normal then no need to worry for asthma. Sometimes undiagnosed stress and anxiety can also cause similar symptoms. So tell her not to take stress and tension, be relax and calm. Avoid alcohol. Consult psychiatrist for anxiolytic drugs (propranolol and flunarizine).Don't worry, she will be alright.Hope I have solved your query. I will be happy to help you further. Wishing good health to your friend. Thanks."
},
{
"id": 179561,
"tgt": "What causes ear pain with fever in 5 year old?",
"src": "Patient: Hi Doctor, my child aged 5 years was having ear pain before 2 days(Sunday night) and we went to one nearby hospital and there was only duty doctor who had suggested the Candibiotic drops for 3 days morning and night. now my daughter is telling no ear pain but still she is having temperature, is any insect would have gone inside the right ear ? how should i check for it or what should i need to do, can i take her to any pediatrician today. please advice and suggest. Hope your fast reply. Doctor: Hi dear welcome to the HCM.Infection in the middle ear causes such infection, Requires some antihistaminc decongestant medicines and broad spectrum antibioic with proper recommended doses..Candibiotic to be continued.Hope the query is answered.thanksThanks"
},
{
"id": 63992,
"tgt": "How to get rid of lump in the buttocks?",
"src": "Patient: A couple of years ago i was taking steroids and i was injecting into the upper region of the buttocks on both sides. After the course was over everthing was fine but 6 months after being off the steroids i lost a fair bit of weight due to the lazyness but i noticed a small lump in my right buttock. Overtime its gotten to a size which was noticeable outside the pants. It seems to have gone away but keeps comin back changing in size. No pain at all until it starts getting bigger. I get a throbbing pain round the lump if im sitting for a while. Had this for about 3 years nearly, ive been on ibilex 500 which shrunk it but never completely got rid of it. Im pretty sure it is just in the tissue not the muscle. Please if u could look into my query it would be much appreciated. Doctor: Hi,DEAR,Good Evening,Thanks for the query.I studied it in details and understood your health concerns regarding it.Lump caused on your right buttock-is a Chronic Injection Abscess.Treatment would depend on the cause-a-Your ER Surgeon would -i- Aspirate or could be drained and cavity wall scraped and repaired under antibiotic cover -i.e. by primary suturing of Chronic Abscess.or ii-could be treated by OPEN drainage after Incision and Drainage-to heal it by secondary intention with scar.Hope this would help you to plan the treatment with your doctor.Wellcome for any further query in this regard.Wishing you good health.Good Night.Dr.Savaskar M.N.From INDIA."
},
{
"id": 4053,
"tgt": "Can pregnancy happen when sex done during menstrual period?",
"src": "Patient: Is it possible to get pregnant if we have sex during her 4 days of mensturation and ejaculated inside the vagina, without any protection (pills,contraceptives, condoms, etc), or is it absolutely impoosible to get pregnant ? what about the 2 days before and after the 4 day mensturation period? Doctor: Hi, Welcome to HCM..Thanks for writing.. It's possible \u2014 but highly unlikely.What is the period length? women who has shorter menstrual cycle like 24 day cycle, ovulation occurs on day 10, so intercourse around 7 th day makes sperm available in the genital tract. If the woman have longer cycle, then ovulation occurs late. Sperms can survive in the female genital tract for 72 hrs but ovum can survive only for 24 hrs.Conception occurs when an egg and sperm meet in a fallopian tube. Sometime during the middle of your menstrual cycle, most likely between the 12th and 16th days, an egg reaches maturity in one of the two ovaries. The ovary releases the egg into the abdomen, where it's quickly sucked up by the tulip-shaped opening of the nearest fallopian tube.An egg can survive in your fallopian tube for about 24 hours after it's released from the ovary. So the only way you can get pregnant is if sperm are present in your fallopian tube during this window of opportunity. If the egg isn't fertilized, it's shed along with your uterine lining during your period.Fertile period in a 28 day cycle is 8-18 days. But periods can be mistaken for intermenstrual spotting, in that case time of ovulation varies. So its always better to take a contraceptive measure to prevent pregnancy. Hope i have answered your queries.. Good day."
},
{
"id": 42068,
"tgt": "How can PCOS causing infertility be treated?",
"src": "Patient: Hi.. I m 30 yrs old.. married 2 nd a half yrs back.. ws doing job so was on precaution for 2 yrs.. nw trying since few months bt unsucessful. Found PCO.. doctor suggested to take Fertisure F, Pregnistar and Uteen syrup for 3 months.. 2 months already over but no success.. wat should I .. Please suggest.. Doctor: in my opinion you should take clomiphene tablet for ovulation. it will help you alt to get pregnant. continue the other drugs too. i wish you will be mother soon. take care and hope my answer helped you. do not forget to rate this answer."
},
{
"id": 18341,
"tgt": "What causes palpitations, fatigue and confusion?",
"src": "Patient: So I ve been experiencing palpitations, confusion & fatigue for a while now, I thought It was because of iron deficiency but I ve overcome that according to doctor, he said that my liver function levels were slightly abnormal as I had a GGT of 34 when range was 30, I also had slight increase in bilirubin and albumin, I have never had alcohol and am active on most weekends, I m 19 years old and very skinny (have always been skinny), I was very sporty and athletic when I was younger, I also always experience weird feeling as if something is always in my neck. The biggest issue is the palpitations as they re really annoying and I ve also experienced them for a while, any idea with whats wrong with me as I m very very concerned Doctor: Hello and Welcome to \u2018Ask A Doctor\u2019 service. I have reviewed your query and here is my advice. I passed carefully through your question and would explain that palpitations could be related to many situations: anxiety, physical activity, anemia, thyroid gland dysfunction, electrolyte imbalance, etc. Coming to this point, I would recommend checking your thyroid hormone levels and performing a cardiac ultrasound and an ambulatory 24-48 hours ECG monitoring in order to investigate for possible cardiac arrhythmia. If all the above tests result normal, you should consider anxiety as the main cause of palpitations. Regarding high GGT, I would recommend repeating this test after a week just to be sure that it is OK. An abdominal ultrasound for gallbladder stones may be needed. Hope I have answered your query. Let me know if I can assist you further."
},
{
"id": 5265,
"tgt": "Thymectomy, taken gravitor, omnacortil 10 mg , xtracalCT, rabephex, trying to conceive, sperm count low",
"src": "Patient: Hi I am 31yr old gone through thymactomy last 8 months and I am under medication, medicine given are gravitor, omnacortil 10 mg ,xtracalCT and rabephex and mean while we are planning for first child we got married since 5yrs are these tablets effect in concieving. I have gone through HSG test and IUI for 4 times but no result and there is problem with my husband as sperm count and motility is low pl help what can be done Doctor: Hello. Thanks for writing to us. The medicines that he is taking are not likely to cause a low sperm count. The sperm count can be increased by taking a healthy diet, avoiding alcohol and smoking and taking proper supplements.I hope this information has been both informative and helpful for you. Regards, Dr. Rakhi Tayal drrakhitayal@gmail.com"
},
{
"id": 100618,
"tgt": "Suggest treatment for chest congestion and breathing problem",
"src": "Patient: hi,my dad was suffering from eosinophilia which is cured now but that is cured now. he recently had a tlc and dlc test and the results were:- tlc(11200),polymorph(79%),lymphocytes(19%),eosinophils(1%),monocytes(1%),basophils(0%).he is suffering with lot of chest congestion and faces problem while breathing.could you please suggest something Doctor: HelloThank you for contacting HCM.According to report mentioned, TLC count is at the higher level. Normal levels are less than 11,000. It shows that some infection element is going on in the body. It can be due to chest infection as well. I need to know some more questions before treating more efficiently. They are:1. had your father suffer such problem in past? If yes, then how did they settle?2. Is your father asthmatic? If yes, then is he taking any medication for it?3. Is your father suffering from any other chronic illness like diabetes or hypertension?4. Does your father has cough or fever?Apart from this i would recommend following basic management for the time being so that his symptoms could settle:a. Diphenhydramine + Dextromethorphan 1 table spoon 3 times a day ..for coughb. Acetaminophen 1 tablet after every 6 hrs if he has feverc. Take hot water in a pot, add 1 table spoon salt/glass into that pot, place a sheet over it and ask your father to take deep breaths in it. It will give soothing effect and improve the condition.d. If chest congestion continues then he would be needing \"nebulization with atrovent (ipratropium bromide) & clenic-C (Beclometasone dipropionate) which will improve it.Hope he will be well soon.Thank You"
},
{
"id": 129006,
"tgt": "What causes severe knee pain?",
"src": "Patient: I am getting moderate pain in the front of both, just below the knee on the nobbly bit below the patella, at the top of the tibia. It is a little sore when pressed on and made worse by moderate to strenuous exercise, such as cycling, or when carrying heady load Doctor: Hello,Thank you for using Healthcaremagic.I read your question and understood your concern.This is tendonitis of the patellar tendon, an overuse syndrom from your activities.I suggest you rest some, use ice therapy locally, nonsteroidal antiinflammatory medication and knee brace for protection.It will get better in few weeks.Dr. Selmani"
},
{
"id": 192498,
"tgt": "What causes pain in testicle?",
"src": "Patient: I am a 23 years old male who has started experiencing pain in the both testicle from last 3yr. I consulted many doctors and an ultrasound was done. They prescribed medicines and they told me that everything was fine. I am not able to concentrate on anything and always think about whether there is pain or not. I always touch right testicle & push it upwards so that right and left are at the same level. I always feel alone and think of the pain. actully this pain is start due to gym. and because of this i m also sufer from back pain. Please help me...Read more at: WWW.WWWW.WW Doctor: Hello, Since you have provided the specific history, this looks to be an inflamed part of the epididymis that is near the lower part of the testis. Since the skin is just lying in vicinity the pain may be felt in the skin and applying the frankincense oil can take care of the skin sensation and as it can not go through the skin to be effective at the inflamed part. Another possible causes can be varicocele. This can be diagnosed by the following: Clinical evaluation by a General Surgeon or preferably an Urologist. Color doppler with ultrasound can help further clarify. Tests of blood and urine can give further clue. Hope I have answered your query. Let me know if I can assist you further. Take care Regards, Dr. Iven Romic Rommstein"
},
{
"id": 70126,
"tgt": "Will the hardness after draining the abscess become normal?",
"src": "Patient: I have had an abscess on my lower inner thigh that was treated by a doctor (cut and drained and packed 3 separate times) the packing has now came out and the area is still hard.. I am on two antibiotics; will this hardness go away itself and will I be ok? Doctor: Hi! Good evening. I am Dr Shareef answering your query.If it was rightly diagnosed simply as an abscess, then the hardness/firmness would vanish gradually as this could be due to the inflammation of surrounding tissue caused by the abscess. Once the abscess is drained, and the source of inflammation is out, the induration of the tissues would go away of its own. So have patience.I hope this information would help you in discussing with your family physician/treating doctor in further management of your problem. Please do not hesitate to ask in case of any further doubts.Thanks for choosing health care magic to clear doubts on your health problems. Wishing you an early recovery. Dr Shareef."
},
{
"id": 132588,
"tgt": "Reason for swelling in the veins?",
"src": "Patient: Hey there! I have had a vein-related problem lately. I got my wisdom teeth taken out on Jan 1st and had an IV in my left hand. For 3 months (and it s only now just fading) the veins in my left hand and wrist were swollen and hard (like maybe hard because they were swollen with too much blood?) and very painful to touch. Over the past week or so, I have noticed that I have serious pain behind my left ear when I touch it, I think it s a vein, which is why I included the first part about the veins. I m not sure if it s an issue, not like I touch that part of my body that frequently, but my right ear feels absolutely fine and I don t feel any swollen veins, so I m not sure if it s something I should worry about Doctor: Hello,I can understand your concern. The veins that you were noticing as prominent for 3 months is not an issue anymore as you are suggesting that it is fading is returning to its normal texture. Now, the severe pain that you are feeling behind the left ear seems to be a separate problem than the veins of the wrist problem. In fact, what you are feeling behind the ear may not be a vein problem at all.The area behind the ear hurts when the person is suffering from either ear or oral infection. I would suggest to first clear yourself from a dentist as you have recently got a surgery done in the oral cavity and if the surgical site is not healed adequately and if it is having infection, then you can feel the pain you are having right now. If the oral surgical site is healed well, you should visit an ENT specialist to look for any infection in the ear. If any oral or ear area is infected, you will have to take antibiotics to eradicate the infection. If you are in pain right now, you can take Ibuprofen 400 mg up to thrice a day to have relief.I hope this information helps you. Thank you for choosing HealthcareMagic. I wish you feel better soon.Best,Dr. Viraj Shah"
},
{
"id": 25241,
"tgt": "What causes enlargement of heart?",
"src": "Patient: My son is 24 and healthy, however since high school his bp has been consistently around 137/77. He had Echocardiogram and the results were ok except the doctor mentionrd his heart was slightly enlarged but healthy. His w eight is normal, he eats right, avoids salt and is physically active. I do not know what to think about it. Should he seek cardiologist's advice or just watch his bp? I would appreciate an answer. Thank you Doctor: hi,thanks for the brief history of your son.since you mentioned that you have met a cardiologist and also the echo was done and all is Fine, I am sure he won't have much issue.137/77 is sometimes a normal blood pressure as the demands of the body and it's needs. you should only get worried if the 77 becomes 100-110 or else the 137 is normal as per my thinking.enlargement of the heart is the bodys mechanism to cope up. remember the proverb - \" Demand and supply \". so when body needs the body modifies it as per the need. by regular exercises and appropriate specific activities he will be more healthier.never worry with BP as 137/77 until you find any signs or symptoms complained. sometimes BP machine may show a big fluctuation without any signs and symptoms. And without having any signs and symptoms at 137/77 can never help a doctor to come to a proper diagnosis.due to the physical activity the body is having a BP of 137/77 which is I believe you should not worry.myself working out for more than 3-4 hours a day and without any sings and symptoms my BP will be 140//99 for which my body's demand and supply is high but I have no symptoms as well.so.to consider only the high BP one shouldn't get painic until one finds any signs and symptoms.with the grace.of God I wish you a speedy recovery"
},
{
"id": 23442,
"tgt": "What causes pain in the chest, rapid heart rate and shortness of breath?",
"src": "Patient: I have a neck and back injury, in the last year or I ve developed rhematoid arthritis and fibromygia, pain in my chest, rapid heart beat, shortness of breath, been to the er and doctor, nothing shows up. I m getting concerned also because of the fact that I m alone alot. also I have highbloodpressure which has been up the last few weeks. Doctor: Hi,Your pain may be a consequence of neck and back injury, usually there is a breathlessness along with such pain. See your doctor to prescribe a treatment for it.Take careCome back if you have any further questions"
},
{
"id": 135839,
"tgt": "Can Diazepam be taken for spasms?",
"src": "Patient: I am a 60 year old male in good health with the exception of bone related issues. Over 3 decades I have had several surgeries including 3 separate disc surgeries in lumbar area, one just 2 weeks ago. I have had 2 neck fusions, both shoulders fully replaced after numerous arthroscopic procedures, and 1 knee replacement with the other scheduled for late September, also after numerous scopes. I have been using the generic version of Norco 10-325 for 15 or more years at 3-4 tabs daily after trying every non drug remedy I know of. Recently my back surgeon had me try diazepam in case of spasms. I found that these were quite helpful with not only my back, but also the chronic pain I have been having in my neck & shoulders. I have decreased my Norco by half already. My question is, will this diazepam be effective as a long term treatment as I like the fact that my mental capacities are not as effected as the opiate drug. I still function clearly. That is my favorite part. I welcome any opinions. Thank you Doctor: hidefinitely, diazepam relaxes muscles spasms and long term intake will do no harm to memory. Only tabs of vitamin B6 B12 should be taken regularly to have mental alertness and make up for B6 deficiency of nerves playing good role in nerve coduction and thereby muscle contractions are also helpedthanks"
},
{
"id": 135386,
"tgt": "How is a recurring cyst in the buttock to be treated?",
"src": "Patient: my daughter has a recurring cyst on her buttox she is 2 1/2 now the doctors tell us its staff infection an give us antibiotic an a cream it goes away for a while then keeps returning 4 times in the last 8 months she drinks a lot of milk an eats a lot of dairy could this some how be effecting this if not what should I do I ve disinfected our home an we have all been taking bleach baths directed by the doctor Doctor: Hi Dear,Welcome to HCM.Understanding your concern. As per your query you have recurring cyst in the buttock. Well there can be many reasons for symptoms you mention in query like bacteria or fungus located on the surface of the skin can cause a boil. Boils are quite common and are usually caused by the bacteria Staphylococcus aureus, the most common bacteria found on the skin. When the hair follicle is damaged by a scratch or other break in the skin, the bacteria is able to move deeper into the follicular tissues, causing an infection and resulting in a boil . If cyst is appearing again and again then there must be other underlying problem like low immunity and diabetic condition . I would suggest you to consult dermatologist for proper examination . Doctor may prescribe antibacterial like cephalexin , clindamycin or erythromycin , anti fungal like fluconazole or vitamin supplement to boost immunity . For now apply warm compresses and eat healthy diet . Hope your concern has been resolved.Get Well Soon.Best Wishes,Dr. Harry Maheshwari"
},
{
"id": 1584,
"tgt": "Is it possible for my period to be an implantation bleed?",
"src": "Patient: I had a miscarriage at 14 weeks on March the 17 (about a month and a half now) after 4 weeks i got what i think is a period on April 17th\u2026i am currently trying to conceive again however my period was VERY light and only lasted about three days of spotting. i took 2 pregnancy tests which came up with a very faint line twice i took one at a clinic yesterday and they said it was negative was the lines evaporation lines or is it possible that my period was an implantation bleed and i am too early to test? what are my chance of being pregnant on this cycle? Doctor: Hi there , I have understood your concern. I will suggest you the best possible treatment options. Sorry to hear about your previous pregnancy loss. If you have delayed periods and your pregnancy test is negative, then you can simply wait for a week and test it again. I will suggest you to get morning's first sample of urine tested for pregnancy. In case of doubt blood beta HCG test and USG will be of help. In case you still have negative pregnancy test, you can simply wait for the onset of periods or induce the periods with progesterone tablets. Your treating doctor can guide you further about the same. As you might be aware that implantation bleeding occurs about 7 days after the ovulation day. So please wait for at least 10 days after you have suspected of the implementation bleeding, and then you can check for the pregnancy. Plesse start on Folic acid, Vitamin B 12 and Omega 3 supplements at least 3 months before you plan to get pregnant. This helps to prevent many problems during pregnancy and delivery. I hope this answer helps you. Thanks. Dr. Purushottam Neurgaonkar. ."
},
{
"id": 4415,
"tgt": "Can I have children at 41 years with tubes tied and burnt years ago?",
"src": "Patient: Hi there. I had my tubes tied and burned 19 years ago obviously thinking I was going to live with my childs father forever. Didn't turn out that way and now I am interested in having another child at 41 years old. My question is, is this possible? I am certain my tubes were burned at the top if that makes any kind of a difference. Price is a factor as well if this is possible, what kind of price rage would I be looking at, ballpark. Please help. Thank you for your time. Doctor: Dear member,Thanks for writing to healthcare magic.A diagnostic laparoscopy with dye test Will tell whether the tubes have been damaged. Laparoscopic. Recannulation can be done if tubal damage is minimal.otherwise I V F(invitro fertilisation) is recommended.As you are 41 it is also recommended that detailed investigation have to be done to check whether your body is fit for future pregnancy.Thanks.Dr Bhagyashree."
},
{
"id": 113403,
"tgt": "Pain in lower back, tightness in area, swelling. Due to back injury?",
"src": "Patient: I don t want to go to the er but I m scared. Not sure if my kidneys are hurting or my back. I injured my back a year ago and because of insurance problems I have been in able to get the right tests on my back. Anyways my lower thoracic spine hurts so bad and it wraps around to my stomach but I feel like a belt is wrapped around that area and everything is swelling and or inflammed over the belt it s very painful and scarey. Doctor: Hi, i hope you have got the right tests done when you had a injury, like x-ray of the thoracic spine to rule out fractures. Now that a constricting pain, swelling and inflammation its better you get physically checked by a doctor and not ignore it. Take care."
},
{
"id": 163461,
"tgt": "Suggest medication for fever and chills",
"src": "Patient: My 4 year old has a fever, chills, swollen glands and cold hands and feet and headache from time to time. Have given her calpol and ibuprofen, temp reduces then for a few hours but glands still making neck hurt. This has started since Friday afternoon. She seems fine in herself after medicine has taken effect but feels bad again when temp starts to rise. Doctor: Hello,A high-grade fever with chills required combination antipyretic to respond to it properly. Give it every six hourly, maintain hydration, and provide rest. Same time plan some test to search for the cause of fever, as a urine test, complete blood count (CBC), malaria after reports appropriate medication can start.Hope I have answered your query. Let me know if I can assist you further.Regards, Dr. Sachin Kumar Agarwal"
},
{
"id": 202103,
"tgt": "How many times can men do masturbation in a week?",
"src": "Patient: i m age of 23 and i suppose to do masturbation once in a week of average for past 3 years. is it good or bad of doing like this?also my penis goes towards left side, i m scared about this, is there any way to cure this?also please advise me how many times we can do masturbation? Doctor: Hi, masturbation is not wrong and as long as you keep the frequency to once or twice a week its alright. Excessive masturbation can lead to various problems and you should refrain from the same. As far as the curvature of your penis goes, it is a normal variant to have a mild curvature to one side and you can have normal sexual life with it. Dont worry. Take care."
},
{
"id": 169673,
"tgt": "Suggest treatment for chronic otitis media",
"src": "Patient: My 11 yr old daughter have been suffering from chronic otitis media since her 1.6 years. Eardrum of both ears, as said by an ENT Dr, have been eroded. He suggested for an instant operation, failing which, he opined, irreparable damage may cause to hearing nerve & CNS. Xray Mastoids says that Sclerosis of mastoid air cells on both sides of ear are detected. Kindly tell me about the treatment and the operation. Is it recoverable? Doctor: HIWell come to HCMI really appreciate your concern, if the condition being treated for infection, and allergy and no satisfactory result found and it is the investigations suggestive of surgical condition then it is to be given try and it may not have any bad consequences, and if infection and allergy not being treated then these has to be given try first, in investigations MRI is the test can is advisable, it is also advisable to have second opinion, hope this information helps."
},
{
"id": 216636,
"tgt": "Suggest remedy for severe pain in my left arm",
"src": "Patient: I have been having pain in my left arm and hand for several days now. I thought (and still do think) it is probably a pinched nerve since I have no other symptoms. But the pain is the worst today that it has ever been. Any suggestion on what I could do to help the pain? Doctor: hithank you for providing the brief history of you.As per the history it appears more of a pinched nerve and needs a thorough clinical examination.Also physical therapy like therapeutic ultrasound therapy and TENS therapy should help reduce inflammation and pain. Exercises to regain the muscle strength in further stages should work well.In my clinical practice such cases respond well to physical therapy.Regards Jay Indravadan Patel"
},
{
"id": 89419,
"tgt": "Why do I have stomach ache when I ear anything?",
"src": "Patient: I have been having stomach pains every time I eat something when it happens I don't necessarily have to use the restroom but most times I do and also recently I've notice that my poop has been green. Also my period is about 16 days late if that has anything to do with wrong with me. Doctor: Hi ! Good evening. I am Dr Shareef answering your query.Although your stomach problem might not be related to the delay in your periods, I would advise you to go for a serum HCG and an ultrasound abdomen if need be to rule out pregnancy in case you are sexually active. Your stomach problem might be due to some kind of intestinal infection which could come by eating outside which you should avoid if you do. If I were your doctor, I would get your stool test done for ova and cyst, and treat the infection which would relieve you. In case of no relief, I would refer you to a gastro enterologist for a possible endoscopic evaluation of your intestines.I hope this information would help you in discussing with your family physician/treating doctor in further management of your problem. Please do not hesitate to ask in case of any further doubts.Thanks for choosing health care magic to clear doubts on your health problems. I wish you an early recovery. Dr Shareef."
},
{
"id": 165839,
"tgt": "What is the treatment for a red rash in the forehead of a child?",
"src": "Patient: Hi there, my grandson developed a redish kind of rash on his forehead - we took him to the doctor last night who said that there was no fever and that it can be a viral infection - what bothers me is that it spreaded to his body now ? Must we take him back to the doctor or should we just see what s happening? It does not seem to bother my grandson much. Doctor: Hello dear.Yes the doctor is right.Its a viral infection and you need not to worry much.it will settle on its own in few days hoepfully.wishing your child good health :-)"
},
{
"id": 26750,
"tgt": "What is the BP range required for age 66?",
"src": "Patient: Hi Doc. I was in the hospital for CHF in March and was doing some reading up on the subject. I don t like to take drugs which are chemicals that affect the mind and body, but the Doctor here in Thailand has me on 5 different drugs, which do affect me a lot. I know I have to strengthen my heart with exercise, but I was wondering what my BP should be for me at 66. Is there a firm number today as to what my BP should be? Thanks for your time, Jay Hebert. Doctor: Hey Jay, the ideal for you should be maintained below 135 /85 if diabetic or you have issues of kidney or less than 140/90 if none. Also CHF the cause is important and you should treat reversible cause should be addressed, many meds which are used for blood pressure control have heart strength ING action, best among them are beta blocker group and across ace inhibitors group, fine titration of all the meds would be required to minimise the side effects and optimisation of benefit. Regards Dr Priyank Mody"
},
{
"id": 149399,
"tgt": "Brain tumor, removed, loss of abilities, possible melanoma. Will I feel better?",
"src": "Patient: Hello, my son had a stage 3 brain tumor on the left side 8 years ago....removed successfully except for losses in his abilities,however, he had surgery on his lip a few months ago and now this lump ( size of a tennis ball) under his R arm they are calling possible Melanoma. They are ( considering) taking it out........he is so worried that they might even take his arm off. His wife tries to console him ( and herself) that it is just the lump. His appointment is for next Thursday. Please help me understand...... Doctor: Hi,Thank you for posting your query.I can understand the personal and physical disabilities your son is going through at this stage. He needs all possible care and compassion to deal with this.The exact nature of the lump in the arm can only be ascertained after the surgery, when we receive the biopsy report.If it comes as melanoma, then, it would be considered as a more advanced stage disease than stage 3. however, if it is something unrelated to the melanoma, then, it would be a good news.Best wishes,Dr Sudhir Kumar MD DM (Neurology)Senior Consultant Neurologist"
},
{
"id": 28558,
"tgt": "What can cause itchy red spots on the scrotum while suffering from chicken pox?",
"src": "Patient: Hi! I'm 21 and my chickenpox started a few days back. About 6 days or so. I realised there were spots on my scrotum and it has been itching ever since. Today it gets worse.those on my scrotum are just red spots, no blisters and many holes. And spots are red and yellow in inside.it is so much painful and burning .I'm not sexually active. Doctor: Hi, You can safely take MS Contin while on OxyContin, it won\u2019t interact with each other and doesn\u2019t cause any adverse reactions. Hope I have answered your query. Let me know if I can assist you further."
},
{
"id": 144404,
"tgt": "Can multiple cranial palsy cause swallowing problem and chronic cough?",
"src": "Patient: I am having swallowing problems. this started dec. 25 1999. the dr. thought I had a stroke. I couldn t swallow, and my face showed signs of paralysis. I ended up being rush to Halifax. they thought I had a magor stroke. when I reached the hospital they did not have the same thoughts. after many tests they said I had multiple cranial palsy. a month later I was able to go home. yesterday, after another swallowing test they said because of the length of time there was not much they could do to improve things. I also have a chronic cough. Doctor: I read you question carefully and I understand your concern.There are 12 pairs of cranial nerves in each of us serving different purposes. Two of them the IX and the X are involved in commanding the muscles in the throat which make articulating words and swallowing possible.So if these nerves are affected in the setting of your multiple cranial nerve palsy it is well possible for swallowing to be affected. Cough is one common associated phenomenon with difficulty swallowing as food, fluids or secretions may pass in the airways due to the swallowing difficulty.I hope to have answered your question in a clear way."
},
{
"id": 86518,
"tgt": "Suggest treatment for severe abdominal pain and diarrhea",
"src": "Patient: Hi there, I was just wondering what happened to me last night. I went to bed and woke up with severe abdominal pain and cramps for diarrhea. I headed for the washroom, the cramps were worse and I began vomitting. Then, the palms of my hands and the bottom of my feet were really itchy. It lasted for an hour or so, with severe diarrhea and some more vomitting and very fatigued. I feel asleep and was just fine the next day.... why was I just fine the next day? and what caused this intense abdominal pain and the itchiness? Can you explain it? It s happened to me once before....so I knew I would be ok the next day. Doctor: HelloAs per the description of your clinical symptoms it seems that you have allergy to certain food. the condition you have faced is acute gastroenteritis probably triggered by some food allergen. This only leads to crampy pain abdomen with episodes of loose stool and often itching.I would suggest you to rule out the allergen ( Allergy test ) and avoid further exposure to it. If symptoms persists or aggravates you should consult a doctor to prescribe you some anti allergic and anti diarrheal drugs ( Antibiotics may be needed depending upon other clinical symptoms).Hope this helps you.Thank you."
},
{
"id": 212649,
"tgt": "Suffer from Remitting Relapsing MS, taking interferon, the test results are increase. How to hinder progression of MS ?",
"src": "Patient: I am writing because I would like to have a confirmation and I want an alternative method to the hinder the progression of MS. My investigations were motivated because I want to stop taking interferon beta and find an alternative method that is based on a natural basis. The long-term study results from last summer that were on interferon-beta, have not had a positive outcome: (MS Society - WWW.WWWW.WW / NY Times: WWW.WWWW.WW . After 4 years of taking Avonex, I still experience serious side effects that have been fought with Celebrex and Ritalin. There are also positive side effects, because I do not get ill as often as I did. After weeks of research, I reached the following conclusions: I understand that a vitamin D deficiency is a component for some diseases and one is MS. Vitamin D increases the interleukin (IL) 10, and simultaneously reduces the IL-6 and IL-17. It retains these two cytokines in balance. An increased IL-6 creates plaques in the brain and IL-17 is responsible for the autoimmune system. Should the IL-17 be too high, it upsets the immune system and it begins to attack its own cells. Interferon-beta is mainly focused on the IL-17 and neglects the IL-6. I also struggle with the herpes virus which is designated as vIL-10, and this fits into the same receptors that are intended for IL-10. Vitamin D is as you know, created by sunshine and after a 30 minute long sunbath in the summer time, you can have up to 20,000 IU in the body. So I think that a 10,000 IU per day cannot cause toxicity . Vitamin D is dependent on vitamin K , zinc, magnesium and boron, to activate it. I would like to take the following orally: - Vitamin D3 - 10,000 IU - Multivitamins per day which includes, among other things: o Vitamin K - 20 - 60 micrograms (mcg) o Zinc - 11 - 15 milligrams (mg) o magnesium - 100 - 120 mg o Boron - 150 mcg - Omega 3 to 1272 mg - Omega 6-928 mg - Omega 9-475 mg o A balance of Omega 3 and 6 is wanted If the above values are too high, or if there is something missing, please let me know? I will continue taking interferon, but I would like to stop it as soon as possible. I am 39, 150lbs, 5 8 and suffer from RemittingRelapsing MS Doctor: Hi there ~ I empathize with your situation and that you have been having a tough time with relapsing remitting multiple sclerosis. One thing that you may have noticed yourself is that you are focusing too much on labs. One way to reduce your anxiety is to distract yourself from thinking about your condition too much. If you start acting like you have no symptoms you probably will end up not having any problems. I know this is easier said than done but one thing that surely is treated is the anxiety which can make MS symptoms worse. Consulting a psychiatrist may help with getting you on anti anxiety medications and appropriate therapy referral. I hope this helps. Take care."
},
{
"id": 95598,
"tgt": "Details about Amrizole suspension",
"src": "Patient: Hi,I like to know more about Amrizole suspension..coz the doctor prescribe to my son,4 years old and I make the first course of treatment and after 20 days I am repeat again.And now he complain with abdominal pain ,so this is the same treatment of mebendazol??Thank you. Doctor: hello and wellcome to healthcare Magic, amrilzole suspension is same as mebendazole it is given in abdominal pain as an antibiotic/for infection ,it does not cause any pain in stomach but reduces if there is an infection in stomach due to infection ,please also add some antacid and have soft diet .take care"
},
{
"id": 34523,
"tgt": "Can Candida recurrence stop after pregnancy?",
"src": "Patient: I have a recurrent Candida problem and have tried everything - maintenance therapy with Diflucan, the progesterone only contraceptive pill, estrogen treatments. One doctor said that this problem is usually resolved with pregnancy. Is this true? What can I do? Doctor: Thanks for posting you query to health care magic.pregnancy do not cure Candida infection .There are many species of Candida and all are not pathogenic. some Candida spp. can be present in genitalia of some woman without causing any harm in a commensal form . what problem you are having with this organism as it is yeast like fungus and require long treatment and some precaution for complete cure .Hormonal pill only maintain vagianal Ph normal but dont have any effect on Candida.Diflucan is a oral tablet but it is better if you treat infection locally.you can use Mycostatin vaginal tablet 2 tab daily in vagina for 2 weeks .Maintain hygine of genitalia.usually such type of infections are mixed infection so go for microbiological examination for vaginal discharge and review me so that i can suggest you further treatment .hope you are satisfied with my answer . feel free to communicate if any query .regards,Dr.Manish PurohitInfectious disease specialist"
},
{
"id": 125196,
"tgt": "A fracture on clavicle bone has calcified.What is the remedy?",
"src": "Patient: hello doctor, i had fractured in my clavicle bone in 2009 and at that time doctor said it will be healed up with the time....but it is still fractured only its ends got calcified...i can easily feel its movement over each ends....what is the remedy for this...shd i leave it as it is or go for any surgery...plz advice Doctor: Hello, Consult an orthopaedician and get evaluated. If there is malunion you can go for surgical correction. Hope I have answered your query. Let me know if I can assist you further. Take care Regards, Dr Shinas Hussain, General & Family Physician"
},
{
"id": 198364,
"tgt": "Suggest remedies for ejaculation during sleep and thin semen discharge",
"src": "Patient: Hello Sir, My age is 26 Right now, Previously i masturbate alot because and than i was facing nightfall, and than i controlled masturbation but still my nightfall wasn't stop, i ate some herbal medicines than it stops for sometime. last december i masturbate. at that time everthing is fine than i am busy with my job and now this month accidentally i masturbate than i see my sperm flowing like water none of thickness in it, between this i also face sometime nightfall. What i have to do i am feeling scared. Doctor: Hi,Nocturnal Emissions are normal occasionally, say once a week if you had not have sex for some time or did not masturbate. However excess is bad. Thinning of semen is not regarded good health sign in Ayurveda while allopathic science doesnt bother about it.Neutrally saying, thinness of semen recovers if you do not lose your semen again n again through any route eg sex , masturbation etc. But to recover it fast and in a healthy way and without any bad effects, some group of herbo mineral combos have been prescribed in Ayurveda referral books which are very successful in enhancing libido, thickening the semen and checking excess nocturnal emissions.If you agree, you may take any ayurveda doctor in confidence, including me, to chose and select for you from among Vrushya, Vajikaran or Rasayana group or any other herbo mineral ayurveda medicine which is safe n without side effects.If you agree, pl write me direct question for further course of action."
},
{
"id": 28353,
"tgt": "Suggest treatment to control blood pressure",
"src": "Patient: I have had very high blood pressure in the mornings, could be as high as 180/95 and then normal to low in the afternoon. I think this started when taking Valsartan. Are there reports of problems like this? What can cause sometimes very high in the morning and then slowly back to normal or even low like 115/68 ? Doctor: Hi welcome to HCM.I understand your query and concern.Blood pressure can be best controlled through multimodal approach in the form of Drugs,life style modification and dietary restriction.A combination of antihypertensives like Beta blockers and Calcium channel blockers.Its always advisable to keep the blood pressure under 130/80 mm of hg.You are currently prescribed the right kind of drugs.Avoid overt treatment to prevent lightheadedness.,For this I advise you to have a baseline 2 dimensional echocardiography,ECG and lipid profile to assess the basic cardiac reserve of your heart.Restrict the intake of salt to less than 6g/day.Regular physical exercise in the form of brisk walk for 20 min a day for 5 days a week is pretty useful.Reduce the intake of fatty and fried food.One pomegranate a day will help to keep your heart at good pace without clot formation. Dizziness will go away away once the headache and blood pressure will subside.Consult a Cardiologist for further expert management.Post your further queries if any.Thank you."
},
{
"id": 178193,
"tgt": "How to deal with a toddler s accidental intake of diluted bleach?",
"src": "Patient: Hi, my 3 year old went to sink full of dishes that was soaking in bleach water. He put his sippie in and drank a very small amount. I believe I only out a teaspoon of bleach in a sink full of water. I made my son drink glasses of water and he s not acting strange at all. What should I do. I m very scared Doctor: hellothank you for contacting HCMyou have done very clever work for your baby by giving plenty of water. Quantity of bleach was very small so need not to worry. you can also use milk which is the first optionFor now take care of baby and don't scare just relaxAll the best"
},
{
"id": 223446,
"tgt": "Is rapid metabolism a side effect of Genette 35?",
"src": "Patient: Hi there. I have been using Genette 35 for 16 days now. I have observed that my metabolism is working exceptionally fast lately. My tummy starts running every afternoon. Is this by any chance one of the Genette 35 side effects? Also I would like to know what are my chances of getting pregnant if i take the pill between 7:00am and 7:30am every weekday and at 8:00am on weekends? How much does this time difference affect the effectiveness of the pills? Doctor: genette 35 causes breast tenderness,nausea,depression,rash as side effect..it doesnt increase metabolism...time difference of half an hour will not affect much...but yes..dese pill should b taken regularly at d same time.."
},
{
"id": 154713,
"tgt": "What causes loss of strength after radiation treatment?",
"src": "Patient: my sister has breast cancer she had chemo and radiation then they found lesions in her brain. She had 10 more radiation treatments on her head. That was 6 weeks ago. Her last Cat scan showed the lesions in her brain were significantly shrinking. Now she has no streength in her legs and her legs seem to be shrinling up?? Doctor: Hello dear, thanks for your question on HCM. I can understand your sister's situation and problem. Actually radiotherapy and chemotherapy breakdown the body tissue.These causes release many chemicals (cytokines, chemokines).These chemicals cause decrease appetite and loss of body mass.So patient feels weak. Better to start multivitamin and protien powder to build up body mass.Ask her to drink plenty of fluids."
},
{
"id": 170430,
"tgt": "What causes cracking sound in knees of a child while bending them?",
"src": "Patient: Hi, my son is 9 year old. I just noticed that his knees makes some cracking sounds when he bends them.... he expiriences no pain or discomfort. He is an active child,I was wondering if this is normal or are there any concerns I should worry about? Thanks Doctor: Hi, cracking sound at knee joints while bending in a 9 year old child can be normal or due to calcium deficiency. In my opinion, you should do calcium and phosphate levels in child with x ray wrist to look for rickets. If calcium deficiency is found than child will require calcium syrup for 3 months. Revert back with above reports. Take care."
},
{
"id": 14454,
"tgt": "What is the dry non itchy rash on the shoulder blades?",
"src": "Patient: Hi, I have a circular non itchy rash in between my shoulder blades, it started a few weeks ago as just 2 patches but is now spreading, it appears quite faint, darker around the edges and a little dry. My boyfriend also has a similar rash around his collar bone Doctor: Hello. Thank you for writing to usI suggest a possibility of Pityriasis Versicolor. It is a coincidence that both you and your boyfriend have the same condition.P. Versicolor is a superficial fungal infection caused by a yeast, Malassezia furfur.This yeast feeds on the oil produced by sebaceous glands.In hot and humid weather conditions this yeast may proliferate and form reddish brown, scaly lesions on sebaceous gland rich areas i.e chest, shoulder, back and proximal extremities.The diagnosis can be confirmed by KOH scraping and visualizing it under the microscope for fungal elements.P. versicolor lesions are asymptomatic.Treatment is either with Oral antifungals like fluconazole Or with Topical antifungals like ketoconazole.I usually suggest my patients to use a ketoconazole based lotion over the whole affected area once daily during shower, lather it well and left over for 5 minutes.This exercise should be repeated once daily for 5 days.Lesions may resolve with post-inflammatory hypopigmentation which resolves gradually over the next 2-4 weeks.I suggest you to visit a dermatologist for a confirmatory diagnosis as well as appropriate treatment.Regards"
},
{
"id": 70553,
"tgt": "Is growing, red lymphomas under the left breast on rib concerning?",
"src": "Patient: I have been diagnosed in the past with lympomas. They tend to be very painful at times. I seem to have one now under my left breast on my rib. Although my primary care doc agrees with the diagnosis, she has done nothing about it. It now seems to be growing and has become rather red. Should I be concerned? Doctor: Hi.I hope you are talking about lymph nodes ( Lymphoma is a cancer).By the way the area you mentioned- under left breast on your rib have no lymph nodes at all. This looks to be an infection or just inflamation of the muscles. Redness suggests an infection for which your Doctor should have given you an antibiotic and anti-inflammatory medicines."
},
{
"id": 96485,
"tgt": "My stomach gets bigger instead, What is the cause ?",
"src": "Patient: Dear Doctor, I have not been able to poo normally (which is at least every morning). I eat as usual, drink a lot of water and other liquids and it seems my stomach gets bigger instead. What is the cause. Thank you. Doctor: Your problem will need thorough examination and consultation with a doctor who will tell you few investigations to complete and then start treatment depending on the cause for it."
},
{
"id": 121187,
"tgt": "What causes swelling in right arm which has multiple hemangiomas ?",
"src": "Patient: My 25 year old son experiences swelling in his right arm for 3-4 days after lifting weights. The left never swells. There is no pain. He does have multiple hemangiomas (look like stork bites) in the right arm as well. He has had this swelling for 5-6 years since he began lifting weights. He does not lift weights consistantly. Any thoughts? Doctor: Hello,Her symptoms could be related to a blocked vein in this arm or Thoracic outlet syndrome. For this reason, I would recommend consulting with his doctor for a physical exam and performing a chest X-ray study and a Doppler ultrasound of the arm vessels.Hope I have answered your question. Let me know if I can assist you further. Regards, Dr. Ilir Sharka, Cardiologist"
},
{
"id": 106406,
"tgt": "I accidentally ate a Beclate Rotocap tablet",
"src": "Patient: I accidently ate a Beclate Rotocap tablet, Beclomethasone Dipropionate Inhaler, please tell me what shud I do ? Doctor: How can you accidentaly eat a tablet without seeing what it is for?? It will not do much harm i suppose but if you feel something going wrong, do ignore your symptoms and report to the doctor for a check-up."
},
{
"id": 208904,
"tgt": "What causes confusion and distress?",
"src": "Patient: sir I am a 21 yrs student, I had fight with my best friends now they don't talk to me as they were before. neither I am so confused about my masters too, I don't know whether i want to pursue my master in practical one or in theory. I think I like a guy in my college but I can't tell anyone about it...I dont know why I feels so but I feel strongly attraction towards him. please help me Doctor: Hi,It seems that you are going through a rough patch in life and I can understand your distress. You seem to be preoccupied with lots of stuff and that is leading to your being stressed. Stress further causes confusion and hampers our ability to think straight.Regarding your friends, remember that fights among friends do happen. You have to patch up. If you friends are not talking to you, you should go ahead and clear things up. Friendship is precious and you just need to put some effort and it will be back on track.Regarding your masters, i feel you should think about what you really like and pursue it. You can also discuss with your parents regarding the options. Parents are always helpful and give us advice with our best possible interests in mind.Being attracted to someone at this age is usual. You should be happy about it and go ahead and talk to the guy and see if it works out. Holding yourself back will only cause you more distress.You must also start exercising daily as that will help you relax and reduce you anxiety and stress. I do hope that you found the answer helpful. Best wishes for a great life ahead."
},
{
"id": 221375,
"tgt": "What are the chances of pregnancy through non-penetrative sexual activity?",
"src": "Patient: I and my gf are of 20 years. We reccently gave handjobs to each other around 16th Oct and lost control and my penis touched her vaginal walls for 2 secs. She has a menstrual cycle of 28 days and her date began on 9 and period lasts for 5 days. Can she get pregnant? Doctor: DearWe understand your concernsI went through your details. From the given information, the chances of your girl friend getting pregnant, is not possible. The penis should ejaculate within her vagina. Here the penis just touches the wall of vagina and not ejaculated. Secondly, the menstrual cycle period also confirms that the ovum is just in the formation state, where it cannot be inseminated even by the sperm. Don't worry, no chance of being pregnant.If you require more of my help in this aspect, please use this URL. http://goo.gl/aYW2pR. Make sure that you include every minute detail possible. Hope this answers your query. Further clarifications are welcome.Good luck. Take care."
},
{
"id": 29649,
"tgt": "How can lupus be treated?",
"src": "Patient: I found out 3 months ago that I have lupus. I don t know that much about lupus, and I don t really know what to expect. I Found Out 3 Months Ago That I Have Lupus. I Don t Really Know That Much About It Or What To Expect. My Name Is Donna. Doctor: Hello and Welcome to \u2018Ask A Doctor\u2019 service.I have reviewed your query and here is my advice.Lupus is a long-term (chronic) autoimmune disorder which can affect any part of the body like skin, bone or organs, etc. I will explain a little about what is an autoimmune disorder. Here immune system is overactive.In our body, we have our immune system which fights against any infection in the body exposed anytime. But in lupus affected patient the immune system gets disturbed where the body's immune system does not recognize the foreign body (infectious agent) and healthy tissue. So, it indirectly damages the healthy tissues of the body by autoantibodies causing inflammation, pain, and damage to organs.It is non-communicable, not even by sexual contact. Each year to 16,000 cases reported each year. In the US till now 1.5 million people are affected with lupus. People with age ranging from 15 to 44 years are more likely to develop the disease. Mostly, women develop lupus.Risk factors are excessive exposure to sunlight, UV light, exposure to Epstein-Barr virus infection.It is true that there is no specific cure to it, but the disease progression can be controlled with relieve to symptoms like fatigue, joint pain, rash, and fever, etc.Usually, doctor prescribes patients corticosteroids and immunosuppressant with lifestyle changes. If left untreated or poorly treated develops complications like anemia and blood clotting disorders. Also affecting lungs, kidneys, heart, and lastly brain and nervous system.So do not worry people with lupus live their full life, but with care and the advice followed properly with regular medications. So, follow all the advice your treating doctor has advised with medications.Hope I have answered your query. Let me know if I can assist you further.Regards,Dr. Anuradha Das"
},
{
"id": 25543,
"tgt": "What causes upper back pain after myocardial infarction and pericarditis?",
"src": "Patient: 33yo male. had Myocardial Infraction 2 months ago. Pain started in upper back and was diagnosed as Pericarditis. 2 days later chest pain then heart attack diagnosis. I am having upper back pain again about 3 of 10 on a pain scale. should I go back to the hospital or just schedule a normal appt Doctor: Thanks for your question on Health Care Magic. I can understand your concern. No need to urgently book an appointment because your pain is only 3/10 scale. It is not of great intensity. Pain associated with pericarditis is having great intensity and scale of 9-10/10. So you can wait and book normal appointment for your pain. Possibility of musculoskeletal pain is more. So in between time, you can take painkiller and muscle relaxant drugs for pain. Application of warm water pad on affected areas is also beneficial. Don't worry, you will be alright. Hope I have solved your query. I will be happy to help you further. Wish you good health. Thanks."
},
{
"id": 121619,
"tgt": "What causes numbness of hand with wrist pain?",
"src": "Patient: My MRI result is, small central & right paracentral disc protrusion at C4/C5 & C5/C6, but no significant central canal or neural foramen stenosis is demonstrated. Mild degenerative disc disease at C5/C6. Disc desiccation at C3/C4 & C4/C5. Osteodiscal protrusion at T12/L1 and mild annular disc bulge at L3/L4, L4/L5 & L5/S1. There is no significant neural foramen stenosis demonstrated on present study. Doctor, I m having mild numbness on my hand and mild pain on my wrist. Do I need a surgery, please need your doctor advice what to do, I really don t want to undergo surgery. Thank you. Doctor: Hello,It may be due to nerve compression at cervical spine level. For that you may require MRI neck after spine surgeon consultation. For pain you can take tablet acetaminophen. Do regular physical therapy. You may require tablet Pregabalin and multivitamin after consultation Hope I have answered your query.. Let me know if I can assist you further. Regards, Dr Shyam Kale Family and general physician"
},
{
"id": 68610,
"tgt": "How can an armpit lump with pain and redness be treated?",
"src": "Patient: I found a Pea size bump in my left armpit 4 days ago. It wasn't red at the time but it hurt. I have stopped using deodorant and was applying a hot compress to it. Now it is the size of a quarter red and more painful. What should I do I don't have medical insurance right now but It really hurts if I move just right in my sleep it will wake me up. Doctor: welcome to Health care magic.1.Possible cause of lump in the axillary region / armpit is due to bacterial infection ( infection / inflammation of the hair follicle)2.Good you stopped deodorant as it is an irritant in this case, stop shaving use trim until it heals completely.3.It can be treated with a course of antibiotic - if does not give relief.4.A small incision and drainage will be done in complicated / un healed cases.5.A pain killer for pain might be helpful.Hope it helps you. Wish you a good health.Anything to ask ? do not hesitate. Thank you."
},
{
"id": 122278,
"tgt": "Can an amputated finger be repaired at proskin?",
"src": "Patient: Hi Doc. my name is bjnkshi. one of my family member met an accident in which on of her hand finger has been removed, so i m searching a place where i could resolve this mater. so is there possibility. i heard about pro skin. what is that .could you pleas advise me. Doctor: Hello, Amputated finger can not be replaced. You can try prosthetic fingers which will look like normal finger and can be used for cosmetic purposes. But these are non-functional. Proskin does not help in this situation. Hope I have answered your query. Let me know if I can assist you further. Take care Regards, Dr Shinas Hussain, General & Family Physician"
},
{
"id": 30460,
"tgt": "What causes breast pain while breathing after having an ATV accident?",
"src": "Patient: I was in a ATV accident 4 days ago. Was pinned under a 750 lb ATV for approximately 20 seconds. Went to ER. Had a CAT scan. No internal bleeding & nothing broken. I have severe bruising on my stomach, upper part of my legs & on both breasts. Everytime i breathe i have pain on the left side of my left breast (where there is a severe bruise). Should i be concerned? Doctor: HiYou likely have one or more fractured ribs on that side.In general these just have to heal on their own but if you haven't had rib detail x-rays, I would order those"
},
{
"id": 112663,
"tgt": "Right back pain in an aged person. Alcoholic, smoker. Suggestion?",
"src": "Patient: 50 Year old male with complaints of right flank/back pain (like just beneath the ribs on the right side). Describes pain as deep, dull, constant, rated at 3/10 and sometimes a bit worse (pain is enough to be very annoying and produce grimacing at times). Tenderness when pressed. No fever, nausea, vomiting or diarrhea. Normal daily bowel movements. No significant health/medical history. Light smoker. Does drink beer daily. Doctor: Hi,Thanks for posting your query.Pain could be due to gall bladder or renal problem.Ultrasound abdomen is required to reach the diagnosis. I suggest you to consult your doctor for thorough examination of abdomen.Drink a lot of water daily.Hope this will helps you. Feel free to ask me if you have any further queries. Wish you good health. Take care.RegardsDr Saurabh Gupta"
},
{
"id": 159587,
"tgt": "Suffering from cancer, got food poisoning. Taking cefspan injection. Continue to take?",
"src": "Patient: hi this is i wanted to ask that i am a cancer patient and i am taking medicines as i got injected recently... one of it is a antibiotic (cefspan) during all this i got food poisoning (yesterday) shall i still take cefspan which i was taking for my cancer thankyou loads :D Doctor: Hi Dear thanks for your question,your a cancer patient and on medicine for that,mean while you have got food poisoning,Dear there is no harm in continuing your medicine ,you remain continue with your cancer medicine ,that will not going to change course of your food poisoning"
},
{
"id": 19712,
"tgt": "Suggest treatment for high BP",
"src": "Patient: hello, 4 hrs ago my husbands bp was 173/08, now we took it again and it is 136/93. High bp runs in the family and he has seen the dr within the last 6 mths concerned about it and they haven't done anything and I think its because it is a physicians assistant he has seen on a military base. What should we do? I'm worried. Doctor: if you're husband has persistent high blood pressure he has to get all his blood investigations and E.C.G done. And he has to be put on an antihypertensive as soon as possible depending on his test results"
},
{
"id": 49937,
"tgt": "Have excess urination. Blood test done. Feel increase of vitamins make frequent urination. Increased calcium?",
"src": "Patient: I take D3 5000 x 2 per day, citracal 250mg x 2 per day as my D strontium carbonate 340mg x 2 at night plus I for have excess urination in fact I feel I have become incontinente and I need to drink a lot of water. My blood test was over 200 the next time it was just 100 I thought that was high enough but my doctor said I should take more for the winter as no sun. I have not done much exercise lately and feel I would like to decrease these vitamins as I feel my kidneys and frequent urination is a problem and I am drinking much more water. could I have too much calcium in my blood regards cathy Doctor: hithanks for your question.I would recommend in addition random blood sugar tests for diabetes, blood pressure monitoring, glycosylated hemoglobin, complete blood, liver and renal function tests.you need to see doctor(endocrinologist) urgently so that the above tests can be performed.Hope I answered your question."
},
{
"id": 82878,
"tgt": "What is the pain in my chest with systemic lupus?",
"src": "Patient: I have pain in the middle of my chest which radiates around and under my left breast. It s worse when I lay on my left side and it wakes me up from my sleep or makes it difficult to sleep. What could the be? By the way I have been diagnosed with Systemic Lupus. Thanks Doctor: Dear Madam,Please consult your doctor to see if there is any evidence of myocarditis . A 2D ECHO is required.Dr. Shruti"
},
{
"id": 540,
"tgt": "Can pregnancy happen after taking I-pill?",
"src": "Patient: Hello Doctors,Me and my girlfriend had sex on 13 Jan and she took an i-pill after 3-4 hours of it, but now she feels tiredness, hunger, excess sleep, lower back pain, faints, absent mindedness, not feeling comfortable, nausea.... and she haven't had her periods yet. do i have anything to worry about? Doctor: Hello,Thanks for asking at healthcaremagic.i have gone through your query and I can understand your concerns.ipill is an emergency contraceptive pill and it is relatively safe. The side effects are minimal. I think the symptoms are due to unwanted anxiety and stress.Keep in mind that ipill is not 100 percentage safe and failures are seen rarely. Do a UPT( urine pregnancy test) to rule out the possibility of pregnancy even though it is rare. Because the symptoms you have mentioned also seen in early pregnancy.Hope i have answered your query.thanks.Dr.Shana"
},
{
"id": 105088,
"tgt": "Bumps appearing all over the body, allergic to fake metals. Related?",
"src": "Patient: My fiance is allergic to fake metals, so if she wears jewelry that is not real gold or silver she breaks out in these small white itchy bumps where the jewelry touched. She is so sensitive to it, that even the metal button on the inside of her jeans causes a breakout. However, recently these bumps have been appearing all over her body...trying to figure out if it is related to the metal allergy , or something different completely? Doctor: A person who is allergic to a partiular thing may be allergic to many other allergens and it may not be possible to find out all of them. She should take a montair lc and if allergy is very bothersome, a short course of oral steroids. She should get her IgE levels tested. She may go in for sensetivity testing where in an attempt can be made to find out various substances to which she is allergic. If required she take shots of anti IgE antibodies."
},
{
"id": 143933,
"tgt": "What causes CSF leak from nose?",
"src": "Patient: I m a Csf leak patient from left nasal. Doctors did endoscopy by my left nasal but after three month again Csf start from same left nasal. After doctor did open surgery . For 8 months Csf stop but now again Csf is waking from same place please guide me Doctor: csf rhinorrhoea have lot of causes, among these cribriform plate defect is most common. for findind these defect you have to go for mri brain alogwith csf cisternography."
},
{
"id": 100627,
"tgt": "What causes constant itching all over the body?",
"src": "Patient: Hi, Can u help? Please! Yesterday afternoon, I was drinking Pepsi, And I've always been never occuring to itching problems with it, and all the sudden I start itching like crazy! I hadn't eaten or drinking anything allergic to me. So I took this medicane I forgot what it was called and I rubbed it everywhere after I took a shower. Later that night I was fine until I woke up this morning. I'm now countiueing itching like mad. Everywhere! So you know what could've caused it? Doctor: Hello Patient,Although you may never have encountered any allergic reaction after drinking Pepsi, it is possible that you may have developed a hypersensitivity to it now. Another factor is that something completely different (and unknown to you) must have induced the response. For e.g. dust, smoke, any strong fragrance or fumes, etc. The effect of the cream you applied will last for not more than 8 hours or till you wash your body. I would advise you to go in for Homeopathy because it will give you a long-lasting solution. In the mean time, you can apply Calamine lotion to soothe the irritation.All the best!"
},
{
"id": 202174,
"tgt": "How to treat a swollen testicle with lower abdominal pain?",
"src": "Patient: My fiancee moves furniture for a living and came home 2 nights ago claiming he pulled something in his groin and balls area. Now, he is having sharp pains in his lower abdomen area...and his balls are real tender, and swollen. Is there anything I can do for him to help ease the pain he isnin??? Doctor: Hello welcome to Health Care magic.1.He need immediate medical help.2.The way you described looks like inguinal hernia for me.3.Tenderness and swelling abdominal pain - He need a scortal ultrasound scan including inginal region to rule out hernia.4.Do not delay.Like to ask ? Do not hesitate. Thank you."
},
{
"id": 207885,
"tgt": "How to cure lucid dreaming?",
"src": "Patient: This question may be more for a psychologist, but I am going to ask it away. I have a hard time sleeping because of my dreams. A while back I was reading on Lucid dreaming, because I tended to lucid dream a lot, and now I can't stop lucid dreaming. My mind is worn and tired from controlling and awakening from my dreams. What do you suggest I do to have regular dreams and just get some real sleep once in a while? Doctor: DearWe understand your concernsI went through your details. I suggest you not to worry much. As per Sigmund Freud dreams are subconscious activities while sleep. But there are no perfect research findings in that angle. So explanation is difficult. But, sound and perfect sleep reduces dreams. What you need to do is to keep yourself busy and work hard. Get tired by the time you reach the bed and I can assure you that you should get a dream less sleep. Also try to ignore these dreams. If you require more of my help in this aspect, Please post a direct question to me in this website. Make sure that you include every minute details possible. I shall prescribe the needed psychotherapy techniques which should help you cure your condition further.Hope this answers your query. Available for further clarifications.Good luck."
},
{
"id": 193991,
"tgt": "What causes bleeding in penis during intercourse?",
"src": "Patient: I have occassional bleeding in penis during intercourse. I guess it is the tear of the frenellum. Also I have erectrile dysfunction problem. Kindly suggest. My details areas follows Age - 33 yrs, WT - 87 kgs, height - 5'10'', Medical history - recently hypothyroidism detected Doctor: Hello, Erectile dysfunction can be due to bleeding. Bleeding can be due to phimosis or urinary infection. Sharing a picture will be helpful for diagnosis. Hope I have answered your query. Let me know if I can assist you further. Take care Regards, Dr S.R.Raveendran, Sexologist"
},
{
"id": 127418,
"tgt": "How can leg ache be treated?",
"src": "Patient: I had surgery for an absent at my anus and was perscribed hydro coding for pain management. After 4 days I stopped this medication because I did not like how it made me feel. Two days later I am experiencing nausea, some vomiting and severe pain in my lower legs. It is very painful and difficult to walk. Is there anything I can do to lessen the leg pain until I see my doctor on Monday? Doctor: Hello and Welcome to \u2018Ask A Doctor\u2019 service. I have reviewed your query and here is my advice. -Apply an ice pack in 20-minute intervals may also help. -Take OTC medications-Light stretching can help ease calf pain. After the symptoms subside slightly stretching can helpHope I have answered your query. Let me know if I can assist you further."
},
{
"id": 63259,
"tgt": "How to remove a pea sized lump on clitoris?",
"src": "Patient: i had a pea size lump on the left dry area of my clitoris. Its swelling alot more after i tried to squeeze it. There's no head and nothing comes out. It's painful and swollen like the size of 2 pills now.It's soft.. What should i do? I'm flying off tomorrow and have no time to see a doctor.. Please help! Doctor: Hi, dearI have gone through your question. I can understand your concern. You may have some benign cyst like batholin cyst or other cyst or some soft tissue tumor. You should go for gynecological examination. It will give you exact diagnosis. Then you should take treatment accordingly. Surgical excision is the treatment of choice. Consult your doctor and take treatment accordingly. Hope I have answered your question, if you have doubt then I will be happy to answer. Thanks for using health care magic. Wish you a very good health."
},
{
"id": 144180,
"tgt": "Suggest treatment for postlaminectomy syndrome",
"src": "Patient: Hi. I had a spinal cord stimulator fail and it had to be removed. My surgeon had performed a lamenectomy on my T10, 11 and 12 and now I have severe pain in thearea and cant function . They say post lamenectomy syndrom. What can be done? At present I m on hydrocodon and robaxin. I ve never had back issues ever. The stimulator was for my leg and ankle. Doctor: Hi, I am Dr.Bruno. I have read your question and understand your concerns. Let me try to help you Question : Suggest treatment for postlaminectomy syndromeAnswer : It depends on the time interval. How long ago was the surgery Hope you found the answer helpful.If you need any clarification / have doubts / have additional questions / have follow up questions, then please do not hesitate in asking again. I will be happy to answer your questions.Let me know if I can assist you further.Take care."
},
{
"id": 221169,
"tgt": "Could hot feeling in face/head with sweating infect amniotic fluid during pregnancy?",
"src": "Patient: Iam in 35th of pregnancy anabout from 2 weeks ago I fear ocasional hot feeling in my face and my head. Also I had rupture of amniotic membrane about 1 month ago, but at present time amniotic fluid is enough . My answer is that these hot feeling with sweating can contribute to infecion of amniotic fluid. Doctor: HI, I understand your concern. Hot feeling & sweating can not cause amniotic fluid infection.. actually amniotic fluid infection ( which might have started at rupture of amniotic membrane you had ) can cause feverishness & sweating. But that also has other signs of infection like fever/pains/ leaking with foul smell . You should get examined by a gynecologist to find possibility of infection & management accordingly. Thanks."
},
{
"id": 101399,
"tgt": "What causes itching at night?",
"src": "Patient: Basically, I have been itching mostly at night, (for about 6 months now non-stop) but sometimes in the daytime, and have been taking a Benadryl for it which 1) puts me to sleep 2) stops the itching. I have eliminated food allergies, mites, and dry skin. Now, I am wondering about autoimmune disorders. Are there any of those disorders or diseases that have itching as a symptom? Doctor: Hello.Thank you for asking at HCM.One most important thing I would like to know is whether you itch without hives/skin lesions/urticaria or with them. If you have hives/urticaria, I would say you are having chronic urticaria. I would suggest you to consult an Allergist who will perform some tests for physical urticaria and suggest some blood tests like total blood count, ESR, serum anti-thyroglobulin antibodies, anti-nuclear antibodies, anti-thyroperoxidase antibodies and urine & stool examination. Some of these tests are for autoimmune disorders. Then he can treat accordingly.But if you are not having skin lesions at all and only itching, I would not think of allergy at first. The causes are many like dry skin, nutritional deficiencies of vitamins & minerals, diabetes mellitus, thyroid disorders, chronic liver disease, chronic kidney disease, etc. So I would suggest you to consult a physician who may suggest you tests like CBC, ESR, blood sugar, SGPT, total proteins including albumin & globluin, urea, creatinnine and urine & stool routine and microscopic examination. Then he can treat you accordingly.Hope this will be helpful to you.Wish you the best of the health.Regards."
},
{
"id": 49356,
"tgt": "Is it a problem to avoid lasix and potassium for the kidneys to operate incorrectly?",
"src": "Patient: My doctor had me on Lasix and potassium for 3 years because of edema and the threat of pulmonary edema. I finally took myself off it nearly a year ago because I had lost nearly all of the swelling except that which appears towards the end of the day but is usually gone in the mornings when I wake up (probably from me being inclined for 6 to 8 hours ). My edema is probably the result of having bad airways to my lungs and simply not getting enough oxygen for my heart to pump properly, thus causing my kidneys to operate somewhat incorrectly. I think you ll know what I mean ? Am I okay to stay off the lasix and potassium ? Thanks, Thomy Doctor: HiThanks for your queryThe requirement and dose of lasix can be determined only by a thorough clinical examination which will tell us about your fluid status. If there are any signs of fluid overload we need to continue the lasix.There is a need to take potassium supplements only if your serum K is less than 3.5Hope this helpsGood luck"
},
{
"id": 114412,
"tgt": "What causes elevated WBC and lymphocyte levels?",
"src": "Patient: Hi good afternoon, I recently have had a series of blood tests done as my WBC and Absolute Lymphocytes have slowly increased over 4 mths. WBC went from 9.3 in Oct 2016 to 11.4 in Feb 2017, absolute Lymphocytes went from 32 to40 in the same time frame, I am being sent to a hematologist / oncologist and I am scared outta my mind.. my dad passed away from non Hodgkin's lymphoma and I know it is not hereditary. Im wanting to know what kinds of questions should I be asking when I see my hematologist for the first time next week... I am healthy other wise, stiffness in my neck and hip, and always tired however, I do get up at 4am and work until 330, then home for family stuff and life, til bed time about 10 pm - 11pm.. please let me know what I should be looking for and what I should be asking.. Please thank you, Melissa Doctor: Hello and Welcome to \u2018Ask A Doctor\u2019 service.I have reviewed your query and here is my advice.Leukemias and lymphomas rarely run in families, and most of the times, they are sporadic.Just mere increase of lymphocyte count doesn't mean that you got lymphoma. Lymphocytes are increased along with total WBC count in viral infections like flu or dengue or any other viral fevers. Don't worry about the small or minute elevation in the lymphocyte count.Hope I have answered your query. Let me know if I can assist you further.Regards,Dr. Siddartha"
},
{
"id": 201779,
"tgt": "What causes fatigue after intercourse?",
"src": "Patient: Dear Doctor , I am newly married, my wife and i have sex almost 3 times a day and I am getting tired . I assume following reasons. 1) My wife is very weak person she cannot eat one piece of mutton or chicken 2) she is very thin, 3) I feel that her weakness is coming into me. 4) Is this possible. Please advise me sir. Doctor: Thanks for contacting HCM with your medical questionsHaving sex 3 times a day is actually a very physical and exhausting activity. The reason you are tired and feel weak is because of having sex. The physical nature of your wife has nothing to do with your own physical strength and well being. I recommend that both of your eat a good balanced diet with plenty of protein. Your wife may benefit from additional vitamins and protein meals or supplements.Hope I answered your question. Please feel free to contact us again for you medical concerns and questions"
},
{
"id": 61500,
"tgt": "What causes a pea sized lump in the throat?",
"src": "Patient: I discovered a lump in my throat 2 months ago and still waiting for ENT. I do smoke and drink some, age 63. it is behind vocal chord and the size of a pea. I had my tonsils out as a child and had surgery for sleep apnea 10 years ago. Should I be pushing harder to see an ENT? Any idea what it could be? Of course fearing cancer. Doctor: Hi.Thanks for your query.Noted the history and understood your concerns.At the age of 63, cancer obviously is the major feared thing one should think about. It is wonderful to know how do you know that there is a pea sized lump behind vocal cords if endoscopy is not done. Well, with the history of surgery for tonsils and sleep apnea the lump in throat should be taken seriously. I would advise you hte following: Consult an ENT Surgeon for endoscopy and clinical evaluation.MRI of the neck is very important to see for the lump whether polyp or something else.Plan for surgical removal and histopathology. This will get the diagnosis clear and treatment plan manageable."
},
{
"id": 59060,
"tgt": "Getting UTI frequently. Urine test done, pus cells seen. Did uroflowmetry test, have high capacity bladder. Suggestions",
"src": "Patient: Hi doc,I am a 22 year old female,Past 2 months I have been getting UTI frequently. When I gave my urine for routine test it showed plenty or 15-20 pus cells 2-3 times. But all this times cutlture test didnt detect any. As per my doctor's advise I have done an ultrasound scan and a uroflowmetry test. Everything has been normal and scan result impression was UTI with insignificant post void volume. All my doctor could say after analysing the report was I have got a high capacity bladder of 1 litre and I have to void withing 2 -3 hrs which I am doing regularly even before visiting him. I am drinking almost 4 litres of water everyday. My doctor has now prescribed me Niftran and a citralka syrup. I have taken all these before and despite I have developed the symptomes of UTI again. But doctor asked me to take the same medicine again. Please help me with this Doctor: Hello,Urine should not be accumulated in the urinary bladder more time.This creates a good nides for the bacteria to grow and multiply rapidly.This is what that is causing you trouble by repeated UTI.It is basically cystitis which is nothing but infection of the urinary bladder.This infection may ascend or descend to cause infection to other organs of the urinary tract.Backflow of urine may cause hydronephrosis.Regular voluntary voiding of urine is advised.Alkalinize the urine so that bacterial growth is prevented."
},
{
"id": 204887,
"tgt": "Can Valium be taken prior to a hip arthrogram?",
"src": "Patient: I have to have a hip arthrogram done. No offices in my area offer iv sedation for the procedure. Instead the have given me two Valium 10mg to take before the procedure. I have SEVERE anxiety. The thought of being awake during this procedure is terrifying to me. I\u2019m 130lbs (female) will 10mg be enough to make me sleep through it!? Doctor: in my opinion it is important to take valour10mg would be enoughanxiety would be considerably less"
},
{
"id": 150188,
"tgt": "Pain in head when swallowing. Pressure behind eye. Have broken tooth. MRI shows mass. Should I be concerned?",
"src": "Patient: For the last few years I periodically get a sharp pain on the right side of my head when swallowing. It feels very similar to that of an intense brain freeze, however lasting seconds but only after I swallow. I feel slight pressure behind the right eye. I thought it was associated with a broken tooth, but after visiting the dentist he assured me that it could not be. I feel like it might be associated with allergies? I did have a MRi about 5 years ago indicating a small triangular mass, what I guess they felt was a small calcium deposit. Should I be concerned? Doctor: Hi,Thank you for posting your query.The description of your intermittent pain sounds like a neuralgic pain.It responds to medications such as carbamazepine.Calcium deposit in the brain in unlikely to be related to your current symptoms, however, if you could upload the MRI report here, it would be useful.Please get back if you require any additional information.Best wishes,Dr Sudhir Kumar MD (Internal Medicine), DM (Neurology)Senior Consultant NeurologistApollo Hospitals, Hyderabad,My personal URL on this website: http://bit.ly/Dr-Sudhir-kumar My email: drsudhirkumar@yahoo.com"
},
{
"id": 107081,
"tgt": "Suggest treatment for sharp pain in the back",
"src": "Patient: I get sharp pains in my lower back, usually on the left side. Starts when I go from sitting to standing. I have my husband press hard on the point where the pain is and if he presses hard enough the pain will stop as if it has moved. This happens often and has been going on several months to a year. I always bear the pressure on the point until the pain stops or moves from the pressure. My posture is declining. Doctor: HiA careful evaluation by orthopedist is required with X-rays andMRI .Acute sharp pain may be from some nerve impingement in a particular position of spinal or paravertebral small joints.Try hot packs and anti inflammatory gels local application"
},
{
"id": 148813,
"tgt": "Pulsing in head during sex. Why is this? Should I see a chiropracter or massage therapist?",
"src": "Patient: Hi, I am a healthy 30yr female. Recently, when my husband and I are having sex right before I am about to orgasm I get the worst stabbing , sharp, pulsing in my head at the base of my skull that makes me immobile and feeling like my head is going to explode. Why is this? Should I see a chiropractor or a massage therapist? Doctor: Hi,Thank you for posting your query.It is common after sex or during orgasm to experience headaches. These are vascular headaches, which are similar to migraines. You should consult a neurologist for evaluation.In rare cases, it could also mean an aneurysm (a small dilatation arising from a blood vessel in the brain).An MRI with angiogram of brain may be required to make a definite diagnosis.I hope my answer helps. Please get back if you require any additional information.Wishing you good health,Dr Sudhir Kumar MD (Internal Medicine), DM (Neurology)Senior Consultant NeurologistApollo Hospitals, Hyderabad, IndiaClick on this link to ask me a DIRECT QUERY: http://bit.ly/Dr-Sudhir-kumarMy BLOG: http://bestneurodoctor.blogspot.in"
},
{
"id": 214521,
"tgt": "Suggest home remedies for abortion",
"src": "Patient: I feel like I'm pregnant. I had sex 1 weeks ago but I have some symptoms. I can't tell my mom & I need some home remedies to abort the child. I'm on birth control but I was taking it 3 hours apart everyday & you aren't supposed to do that. No condom was used Doctor: HelloFirst of all Purchase a Urine Pregnancy Test kit and confirm the pregnancy, or lac of thereafter. For all you know , you might not be pregnant at all. That is after you have missed your period. Best time to test for pregnancy is 3 weeks after an unprotected sex for best and accurate results.Secondly, abortion is no joke to be done at home using home remedies. There are certain protocols to be followed. If you find the result positive then approach a Gynecologist for help. There are pills available, but this can not be purchased/ consumed without a doctors guidance. Follow what he/she advises you to.Thirdly, Try to relax. Stress delays period. Try to ensure discipline in your contraceptive practice to avoid such stress.Hope this was helpful.Warm RegardsDr Rashmi"
},
{
"id": 59954,
"tgt": "Have dizziness, nausea, mood swings. Had gall bladder surgery, ERCP to open the sphincter of ODI. Colonoscopy showing polyps, hemorrhoids and diverticulosis",
"src": "Patient: Male. 53. Complains of dizziness , nausea, experiencing mood swings (crying-rage); recently had gallbladder surgery, ERCP to open sphincter of odi and remove sludge following gallbladder surgery, gastric emptying tests, blood tests for thyroid, diabetes, celiacs - all were normal; recent colonoscopy revealed 6 small begnign pollyps, hemeroids, and diverticulosis; Symptoms are the same as before his gallbladder was removed. Mood swings are new symptom but may be attributed to feeling ill since November of 2011. Doctor: Hi, Mood swings have little to do with the GI history you have given. As far as nausea is concerned some patients do experience nausea after surgery. A simple anti - emetic drugs like zofer would be helpful. As far as the mood swings are concerned I would advice you to consult a psychiatrist for the same. The other GI reports seem okay. Colonoscopy report is grossly okay. Nothing to worry about is the polyps are benign. Hope I have answered your query. If you have any further questions I will be happy to help Regards Dr. Om Lakhani"
},
{
"id": 209978,
"tgt": "Suggest medication for mental health",
"src": "Patient: I have an aunt who thinks that someone is trying to make it appear that she is crazy. She thinks someone is switching things out in her house - tv's, pots, cloths, changing door locks, moving things, taking things. She lives alone (lost her husband 2 years ago). We the family don't know what to do. Doctor: Hello,Thanks for choosing health care magic for posting your query.I have gone through your question in detail and I can understand what you are going through.The symptoms that you are mentioning are suggestive of paranoid schizophrenia. She certainly needs proper psychiatric treatment. Risperidon, olanzapine, clozapine, aripiprazole are all good drugs for the treatment of such an illness. These medications need to be continued for a few years and the symptoms will mostly reduce by 2-3 weeks. If she is not willing to seek help then you may seek help of police and law who can guide her to proper medical help.Hope I am able to answer your concerns.If you have any further query, I would be glad to help you.In future if you wish to contact me directly, you can use the below mentioned link:bit.ly/dr-srikanth-reddy\u00a0\u00a0\u00a0\u00a0\u00a0\u00a0\u00a0\u00a0\u00a0\u00a0\u00a0\u00a0\u00a0\u00a0\u00a0\u00a0\u00a0\u00a0\u00a0\u00a0\u00a0\u00a0\u00a0\u00a0\u00a0\u00a0\u00a0\u00a0\u00a0\u00a0\u00a0\u00a0\u00a0\u00a0\u00a0\u00a0\u00a0\u00a0\u00a0\u00a0"
},
{
"id": 82359,
"tgt": "Suggest medication for Emphysema",
"src": "Patient: I have debilitating health problems that just seem to be getting worse and worse. I have 6 diffrent specialist that I see and they just come up with one medical problem after another. I am 38 and been and off and on smoker for 10 years. I was recently told I have emphysema. I have had medical and health problems my whole life and I asked to finally be tested for Alpha-1 because my sister tested positive but we have different fathers. I am a M1M1 and my antitrypsin is 161. But when looking at all symptoms I have all but 1. My question is with my health history should I be looking for something else to be the cause of can this still be a possibility Doctor: Thanks for your question on HCM.Being a smoker emphysema can occur at any age. And smoking make you more vulnerable to get emphysema at this antitrypsin level.So if you have chest x ray and PFT (Pulmonary Function Test) siggestive of emphysema than you are suffering from it only. No need to search for the cause.Better to start treatment. 1. Smoking cessation should be the atmost priority.2. Inhaled bronchodilators.3. Pulmonary rehabilitation with chest physiotherapy and deep breathing exercise. 4. Pulmonary vaccination with pneumococcal and h.influenzae vaccines."
},
{
"id": 64151,
"tgt": "Suggest remedy for small hard knot on the thumb side of wrist",
"src": "Patient: Hello, I am 49 years old, 5'6 180#, I have a small hard knot on the thumb side of my wrist, it is under the skin and sometimes causes numbness and pain. Once before when I was a kid I had a similiar knot, my dad thinks it was on the same hand.It has began to get larger. What could this be? Doctor: Hi,Dear,thanks for the query to HCM. After going through the query indetails,In my opinion--Your thumside of the wrist has -Ganglion Cyst.Treatment-a-Consult a orthopaedic doctor.b-Check Xray of the wrist would explain its articular relation and connectivity.C-Planned Surgical Excision by Orthopaedic Surgeon would help you to get rid of the growing Ganglion cyst of the Wrist.Hope this would help you to plan treats with your doctor.Wishing you fast recovery and Healthy life.Wellcome to HCM for further query to My HCM clinic.Have a Gd day.Regards,Dr.Savaskar M.S.Genl-CVTSSuper-specialist in NCCD, and Expert therapy for Cancer /Asthama/Tissue and organ-failures."
},
{
"id": 156447,
"tgt": "Is it necessary to remove cancerous lymph node on lung and clavicle bone?",
"src": "Patient: I have had adenocarsomonia in my right lung, the tumor was contained, had top lung removed on the right side a yr ago, now they tell me I have a lymp node that is cancerus in the area where the lung was and one on my clavical bone, largest bein 1.8cm what should I agree to as for treatment? I have a friend that had lu-177 and he is alive after 5 yrs...help me please ...I am scared to death of chemo Doctor: You have an advanced disease at present which requires systemic therapy.I would suggest that you have your EGFR (Epidermal growth factor receptor) mutation tested on the biopsy specimen. If mutation is positive, i would advise erlotinib which is a targeted therapy taken orally.If mutation is absent then chemotherapy is the systemic treatment option.Please discuss it with your oncologist"
},
{
"id": 4597,
"tgt": "Frequent urination, nausea, cramps, increased appetite, fatigue. Is this possible pregnancy?",
"src": "Patient: hi my names tristan um i believe i might be pregnant on the depo vera shot?i have frequent peeingnauseastomach pains ( kinda like cramps)trouble breathinghungry alot the smell of smoke suddenly makes me sick but my mom has smoked all my life headachestired alot Doctor: Hi,Thank you for choosing Healthcaremagic. If you have missed your periods kindly get a pregnancy test done, usually pregnancy is rare with depo. Kindly do not worry, if pregnancy test is negative then symptoms could be due to other reasons.Please ask if you have more questions. If you are satisfied, please make sure that your ACCEPT my answer so that I receive credit. Good luck!!"
},
{
"id": 109636,
"tgt": "Suggest remedy to relieve back pain due to multiple injuries",
"src": "Patient: Hi .. I've had back pain for a number of years.. But more so the last 2yrs I've had chiropractic , accupuntre , and phisio !! I take diclofenac and paracetamol .. For the last 4 weeks I've had lower back pain again I had a fall and then a week later I missed my footing and jarred my back I'm ok to get around but 3 times I went to drive the car it's pulled my back causing me to have pain , weakness in my legs and it affects my neck ! I'm booked in for phisio tomorrow !! Doctor: Longduration of back ache aggravated by injury with weakness in limb and you have tried many things.still you are not having relief.You should go see a spine specialist or atleast an orthopedic surgeon.They might give you medication, rest /exercise, and most important THE DIAGNOSIS.Further definitive treatment is only possible if there is confirmed diagnosis till then supportive treatment is the answer."
},
{
"id": 33998,
"tgt": "Suggest treatment for hbsag",
"src": "Patient: hello sir, hbsag is poitive and hbeag is negative also i have done the test of lft, usg abdomen afp all the reports are normal and my dna report is awaited . i am healthy and fine women , 25yers old, because of that i could not joint the work . what should do for the next steps. and what all diet i should take. please hep me out i\"ll be greatful. thank you. Doctor: Hello,you probably have chronic hepatitis B. The laboratory data you've provided are not sufficient to judge that. The hepatitis is considered chronic when the HBsAg is present for more than 6 months. Other markers can be used though to determine the diagnosis indirectly.Regarding treatment of chronic hepatitis, it depends on whether it is active or not. The serum transaminases and viral load (HBV-DNA) are crucial information to decide treatment initiation. Your diet has nothing to do with the outcome, so since you've no symptoms or obvious disease activity, there is no dietary change that you should try.You can contact me again, if you'd like more information about your condition or would like to discuss the results you're awaiting. I'll be glad to help!Kind Regards!"
},
{
"id": 202569,
"tgt": "How to reduce the tightness in the foreskin which is red and itchy?",
"src": "Patient: Hello i have taken Timodine for two weeks for tight foreskin and redness. But the foreskin is tight still and on a certain time itchiness i clean the penis with shower jets on a daily basis what other solutions are there Also time i use thrash cream Doctor: HelloThanks for your query,based on the facts that you have posted it appears that you have what is called as Phimosis.The itching in foreskin is a presenting symptom of Diabetes. Please get your blood sugar levels on empty stomach and two hours after lunch done to rule out Diabetes.Take anti fungal medications like Fluconozole twice daily along with broad spectrum antibiotics topical Fluconozone jelly for local application for a week.,If problem persists consult qualified General surgeon for clinical evaluation .You may need to get circumcised under local anaesthesia.Dr.Patil."
},
{
"id": 188595,
"tgt": "Painful swollen lymph nodes in my neck and armpit",
"src": "Patient: I am a 19 year old female. My wisdom teeth have been coming in for a while now. But haven t given me much trouble until recently. Sometimes I will get swollen lymph nodes in my neck and armpit. I can t really go to the dentist right now. Is this something to be worried about ? I brush my teeth a lot and am quite ocd about making sure the wisdom teeth and gums around them are very clean. Could this be an infection and if so how can I cure it because like I said I don t have the means to go to the dentist for another month or so. This has been going on for 2-3 months. The lymph node in my armpit is swollen as I type this and quite painful to the touch. Thanks in advance. Doctor: Hi, Wisdom teeth usually erupt between 18-25 years of age. A condition known as Pericoronitis results in pain and infection around the wisdom teeth and the infection can spread to other parts of the body. You should consult a Surgeon and get a OPG X-Ray taken to know the actual position of the tooth immediately. Remember Health is Wealth. Excuses for not visiting a dentist will only deteriorate your health."
},
{
"id": 115948,
"tgt": "Can ITP cause complications in marital life?",
"src": "Patient: Hello I want to marry a girl who has suffered from ITP previously .She has done her treatment but as doctors say that there is no reason why ITP happens so there is no surety that this will not happen in future. I want to know is there any kind of complications in our marriage life because of her ITP? Doctor: Hello and welcome to HCM,ITP stands for idiopathic thrombocytopenic purpura.In majority of cases, ITP is acquired and the cause is not known.Viral infection is the most probable cause of ITP.ITP can recur and it causes fall in platelet count. however, fall in platelet count will not affect your married life.Thanks and take careDr Shailja P Wahal"
},
{
"id": 139572,
"tgt": "What is the recovery time after undergoing Microdiscectomy?",
"src": "Patient: I undertook microdiscectomy for L-5 & S-1 in the year 2005.The stage was secondary. After surgery the then severe pain gone but I never become comfortable as before. Now often I feel pain & discomfort during the change of posture. It takes a considerable time to change my posture. My waist often gets bend towards left/right. I cannot also move my neck completely. Please give me best answer because I want to be fit completely like a normal man. My weight-75 kg., height-5 5 , age-43 years, gender-male, Resident- Kolkata, West bengal, India. Phone-033-0000, Mobile-0000, YYYY@YYYY .... Doctor: Hello,For 2-3 months after microdiscectomy, you should start exercises or physical therapy since you did not recover completely. This way you can achieve the best results. Discuss with your Surgeon about these issues.Hope I have answered your question. Let me know if I can assist you further. Regards, Dr. Erion Spaho, Neurologist, Surgical"
},
{
"id": 215469,
"tgt": "What causes sharp pain in quad after knee manipulation surgery?",
"src": "Patient: I had a knee manipulation last Wednesday after TKR surgery. Now I am having serious pain in my quads. Imam wondering if it s caused by the cpm machine which I have been using for 4 hours each day for 4hrs each day. My flex great a few days ago but now it is not good My flex is not good again. Doctor: Hello, As a first line management, you can take analgesics like paracetamol or aceclofenac for pain relief. If symptoms persists better to consult an orthopedician and plan for an MRI scan to rule out ligament or tendon involvement. Hope I have answered your query. Let me know if I can assist you further. Regards, Dr. Shinas Hussain, General & Family Physician"
},
{
"id": 94577,
"tgt": "Severe pain in lower back and nausea. Have been bedridden. Cause?",
"src": "Patient: I have a sever pain in my lower back (mainly on the left side) but also have feelings of nausea . I m I sure if I need to pass stools or be sick and the pain is becoming quite uncomfortable. I am drinking water to see if that will help but the pain originally started Wednesday then it dissipated and has since returned this evening rendering bedridden. Any advice would be welcomed. I also have moments of heartburn . Doctor: Hi Kristina, Thanks for writing your query. Heart burn is due to increased secretion of acid in the stomach. You should follow some precautions like :- 1. Avoid spicy and oily foods 2. Avoid mental stress 3. Donot take ciggarette and alcohol 4. Donot miss a meal 5. Donot self medicate especiaaly with pain killers and steroids Regarding your low back pain, it will be helpful if you tell more about it like - - Whether it increases on bending? - Any history of heavy weight lifting or trauma? - Whether any abnormality is noticed in urine like decrease in amount or pain or burning sensation while urinating? Though the pain can be muscular and can be relieved by giving rest to your body and applying local pain relieving gel after hot fomentation, still you should get yourself investigated for the function of the kidney. So get your blood tested for Renal Profile and your urine for Routine and Microscopic examination. Treatment can be planned according to the reports of the investigation. I hope tis is helpful to you. Thanks."
},
{
"id": 101237,
"tgt": "How to treat eosinophilia?",
"src": "Patient: My nephew aged 3 years is suffering from Eosinophilia and the doctors are unable to understand the reason why it happens and no proper cure is done since 2 to 3 months it is also effecting his brain and the doctors are suggesting his brain condition as Acute disseminated encephalomyelitis. Kindly advice what to do? Doctor: HelloThe patient is 3 years old having eiosinophilia . The normal count of eiosinophils in body is 2-5 % while more than 8 % is suggestive of eiosinophilia and the drug of choice is \"diethylcarbamazine\" 1mg / kg body weight in divide doses. Increase up to 6 mg / kg body weight . You can also add steroid , if not treated with above medicine .As far as concerned about ACUTE DISSEMINATED ENCEPHALOMYELITIS , I recommend you consult a \"NEUROLOGIST \" and get his opinion because this is a disease covered by a SUPER SPECIALIST or Neurologist .Hope this will help you."
},
{
"id": 197219,
"tgt": "What causes pressure in the testicles after workouts?",
"src": "Patient: I have an 11 year old boy who is starting to go into puberty. He is very active in Tai Jitsu and Judo. Tonight after Tai Jitsu (with lots of kicking tonight), he is complaining of a pressure in his left testicle. He denies getting hurt during practice tonight, has normal urination, and no other pain, nausea, etc. I gave him Tylenol and it has helped a little, but has not relieved it like Tylenol typically does for his aches from workouts or minor injuries. I examined him briefly and see no discoloration, swelling, or abnomality. Doctor: HelloThanks for query .In absence of any injury to testicle discomfort in Testicle that your son is having after Tai Jitsu is mostly due to pulled muscle and strain of cremastic muscle of the spermatic cord .He needs to take anti inflammatory medication like Diclofenac twice daily for 5 days .Refrain him from exercise for few days till pain subsides completely .Dr.Patil."
},
{
"id": 183440,
"tgt": "Suggest treatment for gum infection",
"src": "Patient: I had damage to my gum in a pocket, and maybe a vein in my gum that bled and was trapped in the gum. It was infected and swelled. After months of antibiotics and not knowing what it was I found it and cleaned it with Hydrogen peroxide. Its doing much better and no more pain but it seems like there is something still producing pus. The periodontist took xrays and says he doesn't know what it is and thought it was just healing. Since then a sliver of bone that was exposed has fallen off, but it keeps seeming like there is still an infection somewhere. What should I do? Doctor: hellooo...read thru ur query...accordingly i feel that the puss can be due to periodontal abscess....but nothing to worry...since u got it cleaned...u can do regular brushing twice daily .tongue scraping,use diluted listerine mouth rinse twice daily,do salt water gargle as many times as possible(min: twice)..do continue this for a week.. take antibiotics(amox+metronidazole)for 5 days if still persists take an opinion of another periodontist(just to know the cause)..be cool and calm..hope ur benefitted something from this reply..have a healthy day!!!"
},
{
"id": 48326,
"tgt": "What does the ultrasound report indicate?",
"src": "Patient: My father aged 69 few days ago got pain in left and right side kidneys. Doctor suggested urine and blood test and ultrasound. ultrasound report is .... Single echogenic focud with posterior acoustic shadowing seen in right kidney measuring 0.4 cm, resembling the picture of right renal stone. No renal mass lesions or cortical cyst noted in both kidneys..Fullness found in left kidney.Normal scan of urinary bladder.Please suggest what is it ? What to do next? Is there anything to worry?? Doctor: Hi,It looks like he has a small kidney stone on right which should comfortably pass off if he improves fluid intake.He might be on the process of passing another similar stone on the left side. I suggest he gets an X-ray KUB, and if required a CT scan of KUB if there looks to be a bigger problem."
},
{
"id": 201117,
"tgt": "Suggest treatment for phimosis",
"src": "Patient: Can I lead a normal healthy life&have a good sex life after marriage with tight foreskin problem(Phimosis)? I am 28 years old and I have a tight foreskin problem (Phimosis) of the penis. I cannot pull back my foreskin totally to reveal the head of the penis even during complete erection. I have seen the foreskin roll down completely in **** movies, but its just not possible for me to do the same even during complete erection. Moreover, the head of my penis (under the foreskin) is very very sensitive, so I am scared to pull it back completely. I am planning to get married soon, so I am even more worried now that it will affect my sex life after marriage. Is this normal for guys my age? Will I be able to lead a normal and healthy life without any complications with this problem? Doctor: Hi,From history it seems that you have tight foreskin.After marriage it can give rise some problem like para-phimosis.So it is advisable to go for Circumcision procedure before marriage.Afterwards you will have good marital life with full enjoyment.Ok and take care."
},
{
"id": 95680,
"tgt": "Abdominal pain and pain in penis",
"src": "Patient: hi doctor after meals or some time i feel my stomach is heavy full like gas and i feel laziness and feel heavyness in my penis also some time my age is 35 years ,male i don t have any medical history but did some checkup reports are fine and this problem is with me from long time. Doctor: Hi radianceexports, You may be suffering from chronic cholecystits which is an inflammatory condition of the Gall Bladder. You have not mentioned your weight, since this is common in obese individuals. Could you describe the sensation in your penis more clearly? Take care"
},
{
"id": 82615,
"tgt": "Suggest treatment for lupus",
"src": "Patient: I m an immune mess...lupus, crest, APS, etc. My creantin came back high 6 months ago, it came back at 1.12 today. I know that isn t horrible, but is it an indication of? Infection, stones, weakness, failure? I m not diabetic and my blood preasure is good. I have been on prednisolone for 10 years, so yes im overweight. thank you.... Doctor: We need your whole details to suggest you further , I think you are probably getting lupus kidney involvement which is a dangerous condition . Please get in touch with good rheumatologist ."
},
{
"id": 178977,
"tgt": "Suggest treatment for chest pain and shortness of breath in teens",
"src": "Patient: My 13 year old sister has been having symptoms of strange behavior that started with her having chest pains and saying she can t breathe. My parents took her to the er where a brain scan was done and also a look at her heart. Everything came back normal. Today she is acting very slow and strange. She can t sleep. She says she s too hot one minute and then she gets cold a little while later. She says her feet and hands feel like pins and needles and that light is too bright and that her hair is too heavy. She talks slowly and can t seem to think clearly and answers to questions are unorganized or not relevant to the question. I am very scared for her. I asked my parents to take her to a psychiatrist but they are waiting to see if she gets better. I don t know what to do. Doctor: Your sister is likely suffering from Conversion Reaction. This is common in this age group specially in girls. Something has happened which is sub conciously causing problem to her. She is not able share the problem to any one and that is increasing her distress. There may be any problem starting from onset of periods, any bulling or teasing in school, any love issues... any thing. Ask your parents to talk to her regarding any problem giving assurance that they will be by her side. Once she starts communicating things will resolve. . Tab clonazepam can give her some relieve. As weight is not mentioned cant give you the dose. Hope this helps."
},
{
"id": 121645,
"tgt": "What causes knee pain,elevated ESR and weakness?",
"src": "Patient: my mother is 74 years old. for the last six months she is suffering from knee pain, burning sensation in the stomach weight loss, palpitations, trembling in the hands, and severe weakness. her blood test shows her esr count is 80 where as normal is 20. what other tests should be done and what disease could she be having. Doctor: Hello, There are some reasons for the higher levels. It could be infection, inflammation tumors etc. I suggest you do a blood count, abdominal ultrasound and upper endoscopy. Hope I have answered your query. Let me know if I can assist you further. Take care Regards, Dr. Blerina Pasho, General & Family Physician"
},
{
"id": 180386,
"tgt": "How can oral thrush be treated while using a nebulizer with Albuterol and Medisone?",
"src": "Patient: my tongue is white covered with thrush) I am using a nebulizer with albuterol and medisone is there anything I can do now until I reach my doctor tomorrow morning ( I have been rinsing my mouth with biotene mouth wash Iris Jacobsen YYYY@YYYY Doctor: Hello and Welcome to \u2018Ask A Doctor\u2019 service. I have reviewed your query and here is my advice. Taking Nebulizer that contains Steroid is a potent cause for oral thrush.The first thing that you need to do is to rinse your mouth thoroughly with water after using Nebuliser.For now you can suck Clotrimazole lozenges and gargle with Nystatin mouthwash, it will help in clearing thrushTake more of probiotics like Yogurt and garlic.Drink plenty of water.Keep the tongue clean using a tongue scraper.Hope I have answered your query. Let me know if I can assist you further. Regards, Dr. Honey Arora"
},
{
"id": 195127,
"tgt": "What causes scrotal pain during defecation?",
"src": "Patient: My son is twenty years old and has been diagnosed with Crohn s Disease. He noticed that sometimes when he has a bowel movement he has a pain under his scrotum. He says the pain is not constant. It is intermittent and after he has a bowel movement. Could this be related to the Crohn s disease? Doctor: Hi, Yes, it is most likely related to Crohn's disease. The disease may involve the perianal region and may be presented with a pain in the area. You should contact your doctor. He may have perianal abscess or fistulas. These are complications of Crohn's disease and should be properly diagnosed. Sometimes the diagnosis of the fistulas or abscesses is difficult and he may be ordered MRI. Hope I have answered your query. Let me know if I can assist you further."
},
{
"id": 43871,
"tgt": "Prescribed Levocarnitine and Carnisure after varicocele surgery. Will this increase sperm count and mobility?",
"src": "Patient: Hi doctor, we got married 4 years ago and have no kids. My wife conceived 2times.but both leads to miscarriage after 4 wks as the baby was not growing.then I was diagnosed with vericocile vein and had undergone surgery last month. After that my doctor gave me levocarnitine tablets USPS carnisure 500 for 6 months to have twice a day.will this help me to increase sperm count and motility? Doctor: Hi, Thanks for your query. I have read your query & I understand your concerns. Your varicocele surgery has nothing to do with abortion. These tablets might help you to increase your count and motility. If your wife has conceived means you dont need to worry about count or motility. Please see an infertility specialist of fetal medicine specialist to evaluate reasons for abortion. I hope I answered your query. I will be available for any followup queries you have. Regards, Dr.Mahesh Koregol IVF & Infertility Specialist."
},
{
"id": 157830,
"tgt": "Endoscopy done and biopsies of throat and stomach taken. Nissen Fundoplication. Nausea and painful stomach. Concern?",
"src": "Patient: Hi..I had a endoscopy yesterday and he said he had to take numerous biopsies of my throat and stomach. I had a Nissen Fundoplication in 08 for extreme gerd. Lately my body has gone haywire after a brain tumor in 2010. It was taken out but everything is going wrong now. The latest is I am so nauseated that I am throwing up on the spot...for no reason...very painful stomach ache. How worried should I be. My brain tumor was not cancerous. Doctor: NamastheWelcome to Healthcare-MagicGreetings of the dayKindly get fundus exmaination from opthalmologist to rule out raised intra cranial pressure as cause of your symptoms. Kindly get back to me with complete medical history so that i can assist you further.Regards"
},
{
"id": 4766,
"tgt": "Thyroid patient, irregular periods, trying to conceive. Suggestions?",
"src": "Patient: Hii, I am 27 years old and trying to conceive from last 2 years but I am not. As I am a thyroid patient and mences arecompletely disturbed from last 8 months actually they use to appear very late and they are very slight even most of the times only gives spot and finish.kindly look towards my problem and revert back as soon as possible.Thanks a lot. Doctor: welcome to HCM!thyroid problem is to be solved first.get thyroid hormones level tested,accordingly the treatment by a physician.then consult a gynaec.all ur problems will be solved."
},
{
"id": 168062,
"tgt": "What causes sensitivity of eyes to sun in a 14 month old?",
"src": "Patient: Hi, My daughter is 14 months old, while riding in the car if sunlight gets in her eyes she sometimes crys, and covers or closers her eyes tightly. I have all different types of sun visors I use, but sometimes I can t block the sun at all. I know to use sunglasses, hat, etc.. But is this something I should be concerned about? Doctor: Photophobia is not an eye disease, but a symptom of many conditions such as infection or inflammation that can irritate the eyes. common causes of photophobia include corneal abrasion, uveitis ,central nervous system disorder such as meningitis,detached retina, sunburn.Photophobia often accompanies albinism (lack of eye pigment), total color deficiency (seeing only in shades of gray), botulism, conjunctivitis, keratitis and iritis.Certain rare diseases, such as the genetic disorder keratosis follicularis spinulosa decalvans (KFSD), are reported to cause photophobia. And some medications may cause light sensitivity as a side effect, including tetracycline and doxycycline."
},
{
"id": 94574,
"tgt": "Met with an accident. Started stomach pain and got bruise on it. Required treatment?",
"src": "Patient: Hi my brother in law has deep brusing in stomach from car accident four days ago. He went to hospital yesterday in lots of stomach pain and they gave pain killers but he just rang at 5.30 am to say he s in agony. They checked his urine , kidneys etc. and said all was fine. Is he likely to suffer this much pain or call an ambulance to go back to hospital? Thanks Tracy. Doctor: thank you.after starting pain killer the pain was aggravated.so there is possibility of NSAID induced acute gastritis though the possibility of internal organ injury could not be ignored.so, an endoscopy of upper GIT & USG of abdomen would be helpful.consult with a gastroenterologist."
},
{
"id": 98420,
"tgt": "What causes throat congestion along with pain in the eyes and face?",
"src": "Patient: for some years [ approximately 5 years] i awake up to clear bout Flem for an hour or so very morning . As of February the 8th I am constantly clearing my throat and spitting up flem and it is constant all day and night . My face and eyes hurt and I am totally worn out energy wise. I he tried to steam my self and anything that i think could work. also sneeze strongly bout 10 times in a row. Doctor: Hello,I have gone through your query regarding throat congestion along with pain in the eyes and face. These are the very common symptoms everybody suffers sometime or the other in life. The result of a post nasal drip irritating the throat, an infection by the whooping cough bacterium.Pollution is another major cause, when small particles set in our airways when we breathe in due to smoke and dust. Either cough can be triggered by cold air, movement, talking, change of temperature, or laughing.Expectorant may be clear, coloured, stringy, frothy or tacky. A nervous habit during times of anxiety, removing the cause will lead to recovery.In fact unpleasant though they are, coughs have a purpose, it is nature\u2019s boon in way of recovery. They protect our lungs from unwanted particles, infection, and the build-up of secretions that would otherwise lead to pneumonia.They also act as a warning signal for several serious diseases. Antibiotics can be used to treat bacterial infection with your physician\u2019s advice. Decongestants relieve nasal congestion by drying up the mucous.Care must be taken as decongestants can excessively dry the nasal mucous membranes, which leads to further swelling and congestion.Kunjal therapy with Salty Luke warm water, drink 4 to 5 glasses of this water and vomit it out helps cleansing all airways, lungs, chest congestion, paving the way to health.A pinch of Black pepper turmeric powder in spoon of honey thrice a day is soothing to throat and relieves from anxiety. Warm water with salt and few drops of lemon juice, will enhance fast recovery. Hope this helps solve your query. In case problem persists you must consult a specialist.Hope I have answered your query. Let me know if I can assist you further.Regards,Dr. Suchda"
},
{
"id": 186620,
"tgt": "What causes bad breath and excessive sleepiness?",
"src": "Patient: Bad smell in throat for the last 3 months even when washing and feel tired for more than half a year (Getting up after sleeping 9+ hours a day and still feeling like wanting to sleep). Could be just depression, but i don't know. What do you think it is? Doctor: thanks for your query, the halitosis could be because of throat infection or gum infection or pus discharge from gum or due to deposits or food lodgement due to decay...consult your oral physician to rule out these causes and manged acordingly. scaling can be done to prevent halitosis followed by maintaining the oral hygiene and rinsing the mouth with mouth washes. i hope my answer will help you. take care"
},
{
"id": 64083,
"tgt": "What causes lump with brownish pus on inner thigh?",
"src": "Patient: Hi my husband has had this big hard lump on his inner thigh. He says he s had it for years and it didn t cause him any pain. One day out of curiosity I pressed on it and this really foul smelling brownish pus came out. It frightened him and he never let me completely clean it out nor has he consulted a physician. What could this lump be and can ir effect his health if left untreated? Doctor: Hi,Dear,Thanks for the query to My HCM Clinic.I studied your query in details and understood your health concerns.-Treatment Advise and Cause of your lump in the inner thigh-a-In -my opinion-it is Chronic furucullitis-which is recurring.The pus that came after pressing suggests that abscess was present inside without much of the pain and soreness to him.b-Treatment I advise-is as follows-i-AS its long standing lump -?Chronic furuncle with abscess I would advise -FBS and PPS to rule out the diabetes in him-as a cause of it.ii-The Chronic inadequately drained abscess needs further incision and proper drainage.iii-Antibiotic Cover with Tb NSAID for 5 days would resolve the issue forever.Consult your ER Surgeon for this.This advise would help you to plan treatment with your doctor.Hope this resolves your query.Wellcome for further query in this regard. Have a Good Day...!!With Regards"
},
{
"id": 38755,
"tgt": "How to get rid of severe cough and chest congestion?",
"src": "Patient: Age 76, 5'11, 175# diabetic, in control, coronary artery disease. Getting head and chest cold with extreme coughing. Can't stop coughing. Gave vitamin C, home made chicken soup. Hot toddy, vicks on back, face and chest, throat.sucking on lemon. Nothing helping. Robitussin DM sugar free, sugar free cough drops. Can't stop. Anything else I can do? Doctor: It seems you have severe bronchitis,. Take acetaminophen with anti allergic and decongestant combination, Steam inhalation,warm saline gargle, broncho dilator like salbutamol or salmeterol puffs if not available may be oral and( I do not know which part of world you belong to but your area is bacterial infection prone you must start with a broad spectrum anti biotic) As such you are diabetic though under control you are more prone to infections which may have serious complications.See your doctor as soon as possible to avoid complications. Good Luck."
},
{
"id": 15080,
"tgt": "Have itchy, red rash on chest, spreading in nature. Negative blood test result for celiac",
"src": "Patient: I am a healthy 45 yowf. 5'9\", 190 lbs. I have a very itchy, bright red rash on my chest, around to my neck, progressively worse x 4 mo. Thickened, orange-peel skin. Over past 10 days severe itchy rash over forearms and ankle/shin area. These areas result in hundreds of small mosquito bite like hives. Initial blood test for Celiac was negative. Doctor: hi i think itchy rash u are suffering from is seborrhic dermatitis.it can present as erythematous rash with orange skin like peeling can be itchy over the hairy areas.it can be treated with atarax 25 mg at night,antibiotic twice daily and application of moisteriser along with momate s ointment and ketaconazole ointmernt in the night and use of flucanazole 150 mg weekly once for 4 weeks can cure it"
},
{
"id": 15106,
"tgt": "Swollen rash on hands, spreading. Applying betnovate cream. Is it safe to use betnovate?",
"src": "Patient: hi Doctor.. i suddenly had a small swollen rash with tiny dots aggrevating, on my hand(back side of palm),two week before.. i ignored it thinking it some temporary allergy ,, symptoms not gone even aftr 3-4 days, and it started increasing its size in circular form. now i have been applying betnovate morning/evening without consulting any doctor.. can you me telling what it is exactly and is it the right treatment or not.. ?? also i m worried that using betnovate may be harmful as it contains steroids.. Pls suggest and help..!! Doctor: Hi circular rash over the arm can be tinea corporis.it is a fungal infection caused by dermatophyte species.betnovate can cure it but again recurrences are more common as betnovate is a steriod .hence use sebifin ointment over the lesion and use of flucanazole 150 mg weekly once and atarax 25 mg at night can cure it"
},
{
"id": 167466,
"tgt": "What causes pimples on body?",
"src": "Patient: Hi, my two year old daughter always has one pimple with a white tip somewhere on her body at all times. Once the pimple pops and starts to heal another one shows up somewhere else on her body. She has had them on her face, legs, chest, and butt. This has been going on for about 7 months. There is no other rash or any other symptoms. Any idea what this could be from. Thanks, RC Doctor: dear parents,your baby must be suffering from infantile cane. infantile acne can be caused because of the sensitivity of the skin to male harmonies like testosterone.It can also be due to other skin conditions like impetigo. which is a bacterial infection.For infantile acne mild cleanser and if sever topical antibiotic like fusidic acid or clindamycin will do. If it's impetigo you require oral antibiotics."
},
{
"id": 103567,
"tgt": "Child having bad allergies, reacts to dust and cockroach, has swelling on eyes and nasal bridge, no pain. On zyrtec, singular and flonaise nose spray. What can be done?",
"src": "Patient: My four year old has had really bad allergies, we just visited an allergist 2 weeks ago, she had a allergy test done, the doctor said she had a reaction to dust and cockroach. she prescribed her zyrtec, singular, and flonaise nose spray. I ve been giving her these meds once a day. Anyways, the day before yesterday she woke up with her eyes swollen and the bridge of her nose. she says it doesnt hurt or bother her but its really noticeable. This has happened before but only for a short while, its been two days the meds arent helping it what should i do? Doctor: a child start getting many problems after adding milk and diarya child needs breast milk for 2 years and after that no milkif we add milk and proteins of diary they are non competible with human proteins so start giving problems in number of cases you withdraw milk and diary from diet add vegetables fruitd and cereals after 3 wk child will feel good and slowly all symptomps and diseases will go in 3 months he will be ninety percent oktill the effect continue same rx"
},
{
"id": 53345,
"tgt": "What is the normal range of bilirubin?",
"src": "Patient: i am 18 years old and today when i check the result it was found to be respectively>sgpt:-70 U/L. >total bilirubin:-0.5mg/dLdirect bilirubin:-0.2mg/dLNO PELVICALYCEAL SYSTEM DILATION AND NO EVIDENCE OF CALCULIRIGHT KIDNEY-->MEASURES 15.5*8 CM GROSS HYDRONEPHROSIS AND DILATED SYSTEM SHOWING ENDOLUMINAL POLYPOIDAL COMPONENTS,RIGHT URETER FILLED WITH ECHOGENIC MASS,PARENCHYMAL THICKNESS 8MM Doctor: Good Day and thank you for being with Healthcare Magic!Hydronephrosis means dilation of the kidney and urinary system usually due to an obstructing mass. The ureter is a tube that connects the kidney to the urinary bladder and where urine passes. The right ureter has a mass lesion inside thereby causing obstruction and hydronephrosis. The MASS lesion maybe cancer or blood clot and can only be confirmed by inserting a scope inside the ureter(ureteroscopy) and possibly do a biopsy. I hope I have succeeded in providing the information you were looking for. Please feel free to write back to me for any further clarifications at: http://www.HealthcareMagic.com/doctors/dr-manuel-c-see-iv/66014 I would gladly help you. Best wishes."
},
{
"id": 36089,
"tgt": "What causes swollen gums and ulcers in tongue while having typhoid?",
"src": "Patient: Hi, my sister, a 25 yrs female, 5ft 3inches, wt 51kgs is suffering from high fever since last week and is diagnosed of typhoid. She is having swollen gums and lots of ulcers in her tongue which is restricting her from eating and drinking. Are these the symptoms of typhoid? Doctor: Hello dear,Thank you for your contact to health care magic.I read and understand your concern. I am Dr Arun Tank answering your concern.No, this are not the symptoms of the typhoid.But it is probably arises because of the I'll effect of the typhoid on the body.I advice you to take the paracetamol, vitamin B complex and vitamin C under your doctors guidance. This can help heal the infection.Please take the treatment of the typhoid as your are taking above drugs are the add-on to your treatment. Ignore the drugs is you are already taking it.If you are having severe burn while eating you can take local anaesthetic cream to apply in the ulcers before you eat. It temporarily numbs the ulcers so that you can eat it.Please eat high fibre diet as it can clears the bowels and helps healing the ulcer.I will be happy to answer your further concern on bit.ly/DrArun.Please hit thank you for me if you are helped.Thank you,Dr Arun TankInfectious diseases specialist."
},
{
"id": 54633,
"tgt": "What causes elevated SGOT and SGPT levels?",
"src": "Patient: HI, this is Dhananjay. M 41 n half year old & hight 5.6 ft. Recently I had gone for medical test for immigration result. All the tests reports were normal except SGOT and SGPT. As per the report the normal range for SGOT was given 10 to 40 and mine is 54 and SGPT normal range was given 0 to 45 and mine is 102. I had gone to my family DR. who has given me 20 days medicinal course and assured me that I will be fine after this course. Firstly what is SGOT and SGPT? What are the causes of elevated level of these two? What could be the symptoms or illness caused by it? and how to cure it? what and what not should be eaten?If you could answer it then it will be great. Thanks. YYYY@YYYY Doctor: Thank you for posting your query.Lab results reveal hepatitis.SGOT (AST) and SGPT (ALT) are liver enzymes located inside the liver cell.these enzymes play an important role in the metabolic processes.an increase in the level of these enzymes reflect an INJURY to the Liver. medically, the condition is know as Hepatitis.Hepatitis maybe caused by- Virus (A, B, C, D, E) is one of the most common causes - Drugs - Alcohol - Increased Fat content in the body - certain bacteria, amoeba may also cause hepatitis/ abscessi would recommend you get a test done for HepB and HepC viral serologywish you good health.further queries are welcomed.Health professionals aim to diagnose properly and manage patients according to their limited knowledge. Cure is blessed by the ONE who Created us, whose power and knowledge is unlimited .wish you good health.regards,Dr Tayyab Malik"
},
{
"id": 196193,
"tgt": "What causes fordyce spots on penis and yellow colored pus spots on buttocks?",
"src": "Patient: Adter lookin at some pictures its clear to me that I have Some fordyce spots on mypenis and testes but there is also a few (2-3mm) hard yellowy puss coloured spots also on my balls, was just wondering if they were just another different looking version or whether I have something to worry about. Thanks. Doctor: fordyce spots are nothing but visible sebaceous glands.they usually dont need any treatment.they are usually yellow,black,brown in colour."
},
{
"id": 105866,
"tgt": "Which hospitals in Delhi specialize in asthma treatment ?",
"src": "Patient: hi doctor my mother facing a problem of asthma last one year kindly suggest me best hospital in delhi . Doctor: Hello, Thanks for posting your query. You can consult any pulmonologist nearby. Just see that he has specialised in pulmonology post M.B.B.s. Any doctors with this degree are good. Hope I have answered your query. Wishing you good health. Regards,"
},
{
"id": 178781,
"tgt": "Suggest treatment for my son s recurring cold and fever with cough",
"src": "Patient: My son is 2.11 years old and time and again almost once in 4 weeks time he is getting cold and fever... and cough is not going from last 4 to 5 months...sometimes is getting severe cough and coughing almost every minute. He also gets cough when he gets up in the morning. He also don t like to eat anything, and almost daily vomits before try to send him to school. We have taken him to our local paed, and lot of medicines are given. but cough is still there. Please suggest. Doctor: Hi thank you for choosing healthcare magic. There are many reasons for cough in kids most common being post viral, allergy mainly to dust, cold weather etc. Kindly get your child checked during cough to look for wheezing, nasal discharge, itching in nose etc. If he has those he is having allergic cough for which he will require anti allergic medicine regularily for atleast 3 months. If he is fine inbetween and getting cough during viral infections the cough will subside slowly in a week after the fever subsides. and remember that there is no effective cough medicine, we have to treat the cause and give them plenty of fluids as they tend to decrease the irritation in throat and decrease the cough.Hope this has been helpful to you, kindly feel free to contact again for any queries. Kindly rate the answer if so. Take care. Dr J Sravanthi"
},
{
"id": 735,
"tgt": "How to get pregnant?",
"src": "Patient: I have been diagnosed with small anterior intramural fibroid last month. I am planning for a kid for last 1.5 yrs still not successful. I m so worried don't know what should i do as my FSH,TSH reports are normal even Ultrasound of my pelvic region is normal too. pls help!! Doctor: Hi, Thanks for query.I understand your concern. * Consult your gynecologist .. if she find the size of fibroid would not interfere with pregnancy .. you can plan conception stat. - Get your husband's semen analysis to be normal. -confirm your ovulation day by LHSurge test &have unoritected sex for 5-6 days when you get tge test +ve. -start taking folic acid&vitamine E (both of you). -pregnancy testafter 8 days of delay in period would be +ve after conception. *In case you don't conceive in 6 months.. gi for detailed investigations of sterility. *If the fibroid is bigger you need to remove ot as per yiur dictor's advise. Thanks."
},
{
"id": 29753,
"tgt": "Suggest medication for facial eczema",
"src": "Patient: I can you please help .. I have eczema on my face . I take dicxycycline one a day .. I take it in the morning . I use a cream on my face called eucerin. I get it form Wal-Mart. . I feel it s not help I ve been on this medcation for about 2months .. I feel I need to be on my old one tetracycline.. Please help me.. What cream should I be using for my face . Doctor: Hi, my query, who duagnose for you eczema in face ??You are using different type of product in face, some are antibiotic.You should be carefull for using any medication in face , specially like steroid.dont use self medication.As soon possible you meet with a dermatologist about the diagnosis of your facial eczema and its proper treatment.thanks, best of luck."
},
{
"id": 17069,
"tgt": "What causes high BP?",
"src": "Patient: Hello Doctor, I am 34 years old, and have had a high BP since having my first child in 2000. It has been controlled with medication and once i am exercising regularly, i come off the medication. Recently it has been different, remaining high at 140/90 even with medication and daily aerobics exercise for 25 mins. what can be the probem Doctor: Hello, Regarding your concern, I would explain that high blood pressure is really uncommon at your age. For this reason, I would recommend performing some tests in order to investigate secondary causes underlying high blood pressure values: - complete blood count, PCR, ESR for inflammation - thyroid hormone levels for possible thyroid gland dysfunction - blood electrolytes for possible imbalance - kidney and liver function tests - a Doppler ultrasound of the renal arteries in order to investigate for possible narrowing - cortisol and aldosterone plasma levels for possible adrenal gland dysfunction. If all the above tests result normal, you should consider increasing the dose of the anti-hypertensive drugs that you are taking. Hope I have answered your query. Let me know if I can assist you further. Take care Regards, Dr Ilir Sharka, Cardiologist"
},
{
"id": 78138,
"tgt": "What causes pain in the left side of the chest?",
"src": "Patient: I often have a pain on the left side of my chest. It feels like its behind my breast. (I am female).. its not super sharp or intense. But it doesn't feel nice and it does hurt quite a bit. Any ideas? Its behind my left breast, lower left side of breast. Doctor: Hi. I can understand your concern. You will need a detailed evaluation with a detailed physical examination and chest x ray along with a mammogram. Consult a pulmonologist for the same.Don't worry, you will be alright. Hope I have solved your query. Wish you good health. Thanks."
},
{
"id": 18882,
"tgt": "What causes recurrent chest pain when I move?",
"src": "Patient: I have been to the doctor several times for recurring pain in my chest. Its right in the middle sternum and it feels tender to the touch and comes and goes for months now. It hurts when I move like standing up from sitting down. What else do I say to get some kind of response ? Doctor: Hello,Regarding your concern, I would explain that your symptoms are not typical of any cardiac disorders. A musculoskeletal pain (including costochondritis) seems to be the main cause of your complaints. Coming to this point, I would recommend taking ibuprofen for the pain. If the chest pain persists, I would recommend performing some tests: - a resting ECG and a cardiac ultrasound- a chest X ray study- complete blood count, PCR, ESR for inflammation. Hope I have answered your query. Let me know if I can assist you further.Regards,Dr. Ilir Sharka"
},
{
"id": 121986,
"tgt": "Suggest remedy for joint pain",
"src": "Patient: i am 33 years old. for the last three four months i am having pain in my joints, which lasts for two or three days and disappears. it may come any time and at anywhere in the body. the frequency of this pain is now gradually increasing. blood test reveals that i have inflammation in the blood. at present i have too much hair fall too. i am too much scared. kindly help me. Doctor: Hello, Your symptoms seem to be related to an infection. I suggest doing some further examinations to confirm the source of the infection. So, I suggest doing a urine test, a chest X-ray and a rheumatoid factor level for further evaluation. Meanwhile, I suggest using anti-inflammatory medication such as Ibuprofen to relieve the pain. Hope I have answered your query. Let me know if I can assist you further. Take care Regards, Dr Dorina Gurabardhi, General & Family Physician"
},
{
"id": 94333,
"tgt": "Have abdominal swelling. Had laparoscopic inguinal hernia repair surgery. Related?",
"src": "Patient: My husband had bilateral labrascopic inguinal hernia repair surgery two days ago performed by a physician with an excellent reputation for doing these surgeries in a city two and a half hour s drive from our home so the doctor is for all intents and purposes not available for help. He would probably say go to the local Emergency Room. My husband has developed quite severe abdominal swelling from below his breasts to just above the hernia surgery area. There is no swelling of the groin area or the scrotum or penis . This swelling is causing him much discomfort. What could be causing this, is this dangerous, should we seek help. Doctor: Hi After surgery,he has developed swelling extending from breasts to the surgery area. It is big one. Most probably,blood vessel got injured and there could be bleeding in the layers beneath the skin. As Mesh is placed in Hernia repair,it might have caused some reaction and lead to the swelling. Please do not waste time. Consult your nearest doctor and get him examined. Ultrasound examination will confirm the diagnosis. Till then,give him antibiotic,anti inflammatory drug. Wish him speedy recovery Regards"
},
{
"id": 124588,
"tgt": "Suggest remedy for depression due to chronic pain caused by bilateral clubbed foot",
"src": "Patient: I have Bilateral clubbed foot on both feet with all three symptoms. I m 16 years old and suffer from severe depression as a result from the chronic pain. I still play tennis and other sports but every day actions are getting harder and harder. Are there any new, innovative, methods to fix this? Doctor: Hi, Could you please send pictures of your feet since club foot is a different condition and playing tennis is very strenuous by nature and difficult for a person with club feet. Hope I have answered your query. Let me know if I can assist you further. Take care Regards, Dr Gopal Goel, Orthopaedic Surgeon"
},
{
"id": 219988,
"tgt": "What are the side effects of Meprate during pregnancy?",
"src": "Patient: hi I started with my period on 18-Jan-14 and I got my pregnancy test done on 23-Feb-14 it came negative then I hade meprate medicine on 25-Feb till 1st of March .I just want to known when you dont no about pregnancy and you take this medicine will it harmfull if I am pregnant. Doctor: Hello dear,I understand your concern.In my opinion meprate is not suggested to take during pregnancy.If the lady is on meprate and pregnancy is accidentally detected then the meprate is stopped.Meprate is associated with genetic abnormalities in fetus .But ofcourse there is no clear indication of damage caused by it.Your doctor might have prescribed only after the pregnancy is 100% ruled out.If you were pregnant the urine pregnancy test would have been positive.If you still want to confirm blood BHCG test can be done to check for pregnancy.But any doctor prescribes it only after confirming the absence of pregnancy.So relax.Hope this helps.Best regards...."
},
{
"id": 175742,
"tgt": "What causes fever after tonsillitis treatment?",
"src": "Patient: my son started cefalexin for tonsillitis and despite having had four doses still running a pyrexia of 39c. Paracetamol and ibuprofen (alternately) only bringing temp down to 38c.Is it normal to still run temps after 24hrs of antibiotics? He is ??sensitive to penicillin hence why he is on cefalexin. Doctor: HIWell come to HCMI really appreciate your concern, two possibilities are there one is this could be resistance case for \"cephalexin\" so antibiotic need to be changed and second this must be something else and not \"Tonsillitis\" in such situation it is better to get done the routine blood test, for differential diagnosis, \"Typhoid need to be ruled out\" hope this information helps, take care."
},
{
"id": 67892,
"tgt": "What does a hard lump on buttocks indicate?",
"src": "Patient: Hi I am female 24. I have a small hard lump at the top of my butt crack, it has been there around 1 month and hasnt changed, it doesnt hurt. I am very worried about it because my boyfriend has just recovered from polinidal sinus after three years and I really hope i dont have that. Doctor: Hi! Good evening. I am Dr Shareef answering your query.Yes, it could be a pilonidal cyst, which has not yet got infected to create problem. If I were your doctor, I would advise you not to sit on it so far as practicable to avoid friction and infection in the same. If it does occur causing pain and discharge of purulent material, I would advise you to see a general surgeon for a proper management instead of delaying a definitive treatment for the same.I hope this information would help you in discussing with your family physician/treating doctor in further management of your problem. Please do not hesitate to ask in case of any further doubts.Thanks for choosing health care magic to clear doubts on your health problems. I wish you an early recovery. Dr Shareef."
},
{
"id": 135238,
"tgt": "Suggest treatment for spondylolysis and bulge disk",
"src": "Patient: I have spondylolysis and bulge disk..I am getting lot of stabbing pain and abnormal sensations in my back....I have tried several natural therapies..but not helped at all...I am 53 years old and have severe pain...cannot sleep on my back...cannot sit....cannot lie down sideways......due to think surgery will cure me Doctor: HiWelcome to healthcaremagicI have gone through your query and understand your concern. You are likely to be get benefited by surgery of spinal fusion with or without bone grafting. However final decision depends upon surgeon on seeing your x ray and MRI report. Other methods which can be beneficial can be applicable of lumbosacral belt so that less movements occur at lumbar spine. Physiotherapy by back excercises are of use. You can discuss with your treating Doctor about it.Hope I have answered your query.If you have any further questions then don't hesitate to writing to us . I will be happy to help you.Wishing you good health.Take care."
},
{
"id": 102790,
"tgt": "What should I do for asthma and chest pain for which I have taken Ibrufen and is this Pericarditis?",
"src": "Patient: Hi, I'm a 18 year old male. I've been feelings pretty bad for months now. To make it short, I went into the ER because my breathing was really bad knowing that it was asthma. They had done a EKG (not sure if I said it right) because I said I had chest pain too. Now they are suspecting that I have pre- pericarditis. I was given ibuprofen for reduce swelling around the heart and also some inhalers with abuterol in them. Now I've been feeling super weak and very weird and also my stool is very very watery. I get even weaker when I use the bathroom.It almost feels as if I'm about to... die. I'm not too sure. I just feel that bad! What should I do? Doctor: hii am not sure about the treatment given to youyou see, ibuprofen should be avoided in asthamaticsyou can check the internet for the samei wish you would have uploaded the ecgthen we could have given a diagnosis betterbecause ecg also shows changes in asthmathat may resemble cardiac conditionsassuming that they were able to read the ecg correctlyyou should get an echocardiography and chest x ray done'this will help you evaluate pericarditis- whether you have it, and if you do- how severeas for your loosestools,you should try high fiber diet and take good amount of fluidsand get a cbc and stool routine testeda blood culture may sound frivolous, but may helpwith frequency of micturition and feeling of generalized weakness i would also suggest fbs and ppbs to be testedi am sorry i cant give you any direct answers nowdo let us know with the results of the tests aboveand we will surely try to help you further"
},
{
"id": 216075,
"tgt": "What causes pain in my left eye and left ear?",
"src": "Patient: hi, i have pain in my left eye socket and my left ear, the base of my head also hurts on the left side. i noticed that i have same symptoms whenever i am travelling or when i had alcoholic drinks. would there be a link between drinking/travelling and this sort of pain? Doctor: Hello and Welcome to \u2018Ask A Doctor\u2019 service. I have reviewed your query and here is my advice. Yes, there is a connection behind the ear down to the throat to normalize pressure on both sides of the ear drum. If one is changing pressure due to changes in altitude OR changing pressure in the throat by swallowing, then if the tube is a bit closed then it causes the ear drum to be painfully pulled. Hope I have answered your query. Let me know if I can assist you further."
},
{
"id": 110105,
"tgt": "Suggest cure for persistent pain in the lower back",
"src": "Patient: I am 37 yrs old male. I have undergone discectomy at L4-L5 and L5-S1 four weeks ago. I had recovered well after 2 weeks and doctor advised me slowly start normal activities.But after the 3rd week I am experiencing severe pain in my lower back. I have a twist towards my right above the hip area. Doctor has prescibed Medrol 4 mg 3 times a day for 5 days. But the pain is still persisting . Please advise. Sunil. Doctor: Hi Welcome to HCMI have gone thru your query regarding your persistant back painTroubles coming again & again is caused due to imbalaced matabolism and weakened immune system, and it depends on regular life sytyl proper regular exercise yoga pranayam healthy diet .Gulping on medicines will not help total cure if regulae regimen is not followed. Proper nutrition is an integral part of a healthy immune system if we take simple ,fat free nutritious diet including all essential nutrients as fiber vitamins minerals, veges ,fruit antioxidants as lemon water with water , garlic ,ginger onion -whole grain almonds ,walnut Amla , coconut water all help strengthening our immune system .and pave way for our disease free ,healthy life . Avoid fried , fast foods ,tea ,coffee alcohol meats colas , sodas as it can cause accumulation of toxins in the body .Take good night relaxed sleep & Meditation , helps further .Keep your bowels clean .Do exercise properly / regularly ,/ but don't do any exercise which involve front bending ,It will do more harm than good .Back bending slowly , without jerks , will give you instant relief . Stand straight keep your hands on your waist feet 2 ft apart .Bend body back gently as much as you can .Sit on the chair straight both nds clutch at the back on the hips . pull hands ,neck ,shoulders ,head as much as youcan gently help recovers your back ,spondilitis cervicleall back problems instantly . A cup of hot milk feewith a level tea spoon of turmeric or Shilajit - a herb will give relief from pain & strengthen bones .Hope this give you reply of your query .All the best .Take care .If any doubt pls mail at drsuchda@gmail.comDont hesitate to get back if have any further query"
},
{
"id": 37376,
"tgt": "Suggest suitable medication for lyme disease patient going through peri-menopause",
"src": "Patient: I have Lyme disease and going through peri-menopause at the same time. I was on Estrodial/Testosterone shots and they worked great, but became too expensive. Now I am trying Estroven weight management w/ Black Cohosh and Soy and Synetrim CQ. I also take Prozac, Ativan and Armour Thyroid. I am seem to be totally exhausted after taking the Estroven for a few days. Could I be having Serotonin and Synetrim CQ interactions/? Thank you, Alison Doctor: Hello,Welcome to HCM,As you were diagnosed to have lyme disease which is an infectious disease caused by Borrelia bacteria and it is a tick borne disease transmitted to humans by the bite of infected ticks.The symptoms are suggestive of the lyme disease.I would suggest oral doxycycline, if you have taken this and if there is resistant to this you can take either cefuroxime or azithromycin. These medications should be taken for atleast one to four weeks.You need a few tests including labs to check his kidney function and an ultrasound of the kidney area and an urine culture to rule out an infection. I think the best way to do this would be to go the primary doctor again and tell him or her the new symptoms.Thank you."
},
{
"id": 162110,
"tgt": "How can stomach skin discoloration with urinary infrequency be treated?",
"src": "Patient: my 2.5 year old has yellowish skin tone to her stomach area, she is prone to having episodes where she doesnt get to toilet for a wee in time lasts a few days with many accidents yet other times she can tell us with pleanty of notice this is the norm. she has had a temp of 37.6 - 37.8 with no other sypmtoms of illness this has happened a few times before and can last upto 5 days. I was thinking would a urine infection cause all of this or could it be more serious. Doctor: Hi, Yellowish discolouration of the skin might be a symptom of jaundice or hepatitis. I suggest that you get her evaluated by a paediatrician if she has got jaundice and then we can proceed, please. Even if it is an infection it could be a hepatotropic viral infection causing Viral hepatitis. Hope I have answered your query. Let me know if I can assist you further. Take care Regards, Dr Sumanth Amperayani, Pediatrician, Pulmonology"
},
{
"id": 146736,
"tgt": "What care should be taken for patient of brain hemorrhage?",
"src": "Patient: hi sir My father had a brain haemmorhage two years back and is paralytic with right side, post that he is bedridden and recent tests showed that his cholesterol level is v high. Since he does not do any physical exercise nor does he eat much what shall we do as we dont want any further problesm Doctor: brain hemorrhages are mainly a result of longstanding high blood pressure. cholesterol is a risk factor for packages of arteries which is a different disease. unfortunately there is nothing that can be done at this time for the strength"
},
{
"id": 209302,
"tgt": "Suggest management of mental problems",
"src": "Patient: Yes how do i get a relative some help, when she refuses any kind of help from me and other relatives? Her issues evolve around her mental problems. Is there anyway a doctor can speak with her face to face at her home? Because my sister refuses to go see a doctor about her mental issues. Doctor: All doctors can come for home visits. Kindly speak to a psychiatrist or a psychologist in your vicinity to see your sister."
},
{
"id": 219041,
"tgt": "Suggest tests to confirm pregnancy",
"src": "Patient: My doctor gave me promulgated to stop heavy bleed during the first few days of my period because I have endometriosis I started my period on the 1st Dec started taking primulot on 5th for about 4-5 days, had a bleed for about another 3days then had unprotected sex with my spouse on 25th & now Jan 1st & still don't have my period & am always regular every 28days could I be pregnant? Doctor: Hi, If you have missed your periods, you can do urine pregnancy test. If it is positive you can consult a doctor. If it's negative you can wait for your periods.You can also opt for an ultrasound to confirm the pregnancy. Hope I have answered your query. Let me know if I can assist you further. Regards,Dr. Khushboo Priya"
},
{
"id": 13439,
"tgt": "Suggest remedy for itchy skin rashes on the armpit and chest",
"src": "Patient: Itchy palms, then fingers...now have a rash on upper arms near armpits and around upper chest. The palms have gone away for the most part, now occasionally fingers itch. rash just noticed today. this all started about a week ago. Nothing unusual has changed in my lifestyle. Doctor: Hi, From what you are describing, the rash is due to fungal infection as long as nothing unusual has changed in your lifestyle (no garden work). Take anti-itch cream locally and orally for the next following days. If still no improvement, should consult with a dermatologist for further evaluation as might need prescription medications. Hope I have answered your query. Let me know if I can assist you further. Regards, Dr. Albana Sejdini, General & Family Physician"
},
{
"id": 20919,
"tgt": "What is the treatment for irregular heart beat?",
"src": "Patient: Hi I have recently started under the care of the cardiology department and have so far been diagnosed with an irregular heartbeat. I have had a 24hr monitor and am awaiting a tilt table test. I have dizziness on standing up and if standing still, overstretching or if in pain. This leads to passing out occasionally - about twice a week. I get pains in the middle of my chest and have until recently been out of breath after any physical exertion. In the last week this has got much worse and I am now struggling to breath normally when resting and I find it very difficult to sleep as it gets worse when laying flat. I don't want to bother the cardiologist if it's unneccessary but I am getting worried about the worsening of my problems. Do you have any adviceMany ThanksKatheryn Doctor: what was the holter report.you did not mention that.symptoms you are explaining are similar to heart failure symptoms.you got to bother your cardiologist ASAP"
},
{
"id": 131508,
"tgt": "Can any online doctor help me with the following results?",
"src": "Patient: How serious is this disk problem please ? L3/4 desiccation with broad based central and left inferior foraminal disk protrusion. left foraminal and anterolateral annular tear. facet and ligamentous hypertrophy and right facet joint effusion. relatively severe superior L4 lateral recess narrowing as L4 roots descend and subarticular narrowing which may contact the dorsal left foraminal L3 nerve root. Doctor: HiYour symptoms have to corelate with MRI findings..if low back pain and radiation is severe, then foraminotomy,disectomy and laminectomy and fusion of L4,5 may be decided upon by a spine surgeon.I f no radiation, pain confined to low back moderately,then wearing lumbosacral support brace, Myoril tab 1BD,p pregabelin 75mg 1BD,methycobalamin 1500 I.u. tab once daily,hot fomentation,application of voveran TPS gel,physical therapy may be advised .."
},
{
"id": 183066,
"tgt": "Suggest treatment for bleeding after teeth extraction",
"src": "Patient: 2 days ago, I had a teeth extraction. Right after I took the prescribed antibiotic, I noticed severe cramping. Hours later, I had heavy bleeding til now. I ve had my Mirena inserted on 10/26/13 and had a recent checkup on 08/02/14. In the shower, I noticed some fishing pole string look-alike sticking out of my vagina. I was too scared to pull or check on it from the inside. I heard from my sister that her friend had the Mirena and experienced the same thing and her friend had her uterus removed because of the IUD. Is the same thing going to happen to me? And tomorrow is Sunday so planned parenthood is closed. What do I do? I am freaking out right now. Doctor: Hi,Thanks for posting the query, I would suggest you to consult to an nearby OBGYN and get the appliance checked, i would like to tell you that tooth extraction will not have any affect on the appliance which you are wearing, get it checked.Hope you find this as helpful,Take care!"
},
{
"id": 114214,
"tgt": "I am 63 years old suffering from back pain,high BP and i am allergic to spicy food",
"src": "Patient: age 63 male operated for hydrocil / hernia before 30 years. had a slip disk (minor) treated and stayed in the hospital for 5 days ,cannot lift heavy load. high blood pressure for the last 6 years..medication atinol /aten 50 mg..one each day. I am a vegetarian and do consume a peg or two spirits occasionally and I do drink tea coffe atleast 4 cups a day with milk and sugar..dessert is a must after my meal .. allergic to spicy food (immidietley have to gargle mouth or I develop blisters on my toungue..was cosnuming tobacco and stopped completely since 2009...attimes I have back pain due to my past job..I was working at a gas station and a indian cash and carry shop....which involved daily loading unloading of various consummables. Now?please advise!!! Doctor: thanks for choosing health care majic.Need some minor adjustment in your life style.Keep alcohol in moderation.Avoid having too much of coffee and desert.take less oily diet more of fruits and vegetables.Do regular walking ,start with slow and short distance and then build time and speed.Keep your bp checked regularly.for back pain consult your doctor and take his advise about regular back exercise that will help strengthen your back muscles."
},
{
"id": 177528,
"tgt": "Suggest treatment for delayed myelination in infants",
"src": "Patient: Hello doctor, my daughter is 14 months old baby. she cant able to sit individually without any support. her pediatrician and neurologist suggest to take MRI for brain when she was 7 months old. her MRI report says that she has delayed myelination. As a result of this she had global developmental delay. Her thyroid report is normal. Her karyotype is normal. Finally her metabolism also normal. Kindly suggest me the procedure of treatment for my babies problem. Doctor: Hi,It seems that she might require Pediatric Neuro-physician or neo-natal neuro-physician opinion for this problem.After examination and seeing the MRI report will advice accordingly.Ok and take care."
},
{
"id": 106721,
"tgt": "How can nausea along with joint pain and lower backache be treated?",
"src": "Patient: Hello and thank you first . I have been suffering with nausea , severe joint pain and mid to lower back pain and sciatic is worsening , so much so that I can only sleep a couple of hours @ a time . I have pleurisy now as well and headaches . that being said do you think tizanidine may help me ? I currently am taking diclofenac and cyclobenzaprine . Doctor: Hello and Welcome to \u2018Ask A Doctor\u2019 service. I have reviewed your query and here is my advice. You can take Tramadol and paracetamol combination 2 times a day for five days take Omeprazole twice a weekly do exercise. Go for walk in morning \"physiotherapy is also helpful, apply hot water bag on affected site, apply Diclofenac spray at affected side. Hope I have answered your query. Let me know if I can assist you further."
},
{
"id": 135866,
"tgt": "Suggest treatment for severe rib cage fracture",
"src": "Patient: I have been diagnosed with three known closed fractures of ribs #5,6 and 7, last CT Scan showed no healing or new growth of bone tissue after 4 months, I suspect that another rib has fractured since then. I can feel movement and like a popping in ribs at times with breathing and certain movements. I understand that little can be done but there must be something that can be done. there is pain moderate almost all the time. What can be done and what specialty do I need to see? Doctor: Rib fractures are very slow to unite due to the constant movements during breathing, healing time varies with age and associated osteoporosis and may even take upto a year. U Could use a rib binder to restrict sudden rib cage Movement, analgesics and calcium. Advisable to see an orthopaedic surgeon"
},
{
"id": 83350,
"tgt": "Can Follic acid intake cause irreversible nerve damage?",
"src": "Patient: hi i am 24 year old women, i m planning to have my first baby. i visited a gynecologist she advised me to take biopreg-f tablets on a daily basis. it contains biotin 5mg and follic acid 5mg but i have read that follic acid intake must not exceed 1 mg for women and it might also have side effects such as irreversible nerve damage . how safe it is to continue this tablet. will it have any side effects? please help. Doctor: Hi,Folic acid is a very safe drug.It is highly beneficial to start the drug when planning for pregnancy. It prevents neural tube defects in the developing fetus. Take one bio-preg F tablet a day and it is completely safe and the drug may not be required by 12 weeks since the neural tube would have closed by that period.Hope I have answered your question. Let me know if I can assist you further. Regards, Dr. Saranya Ramadoss, General & Family Physician"
},
{
"id": 199100,
"tgt": "Suggest remedy for blood in urine followed by erectile dysfunction",
"src": "Patient: A few months ago, I waited too long to go the bathroom. When I finally relieved myself, I noticed blood in my urine. Almost as if I ruptured a vessel. Every since then, I have noticed some erectile dysfunction. What can I do to address this problem? Doctor: DearWe understand your concernsI went through your details. Blood drops in urine indicates inflammation of urinary tract and can be easily treated with the help of antibiotics and plenty of water consumption. It has nothing to do with erectile dysfunction directly. The erectile dysfunction you are talking could be due to psychological problem. The obsession about the incident and blood seen. Don't worry. Understand the physiology and you may consult a psychologist for counseling.If you require more of my help in this aspect, please use this URL. http://goo.gl/aYW2pR.Make sure that you include every minute details possible. Hope this answers your query. Available for further clarifications.Good luck."
},
{
"id": 12270,
"tgt": "Can psoriasis treatments result in cancer?",
"src": "Patient: I use lobate G cream intermittently for psoriasis for past 10years. Dermatologist prescribed it to me initially.And I use it ever since,when I get itching or a other psoriasis symptoms that come to me . it comes occasionally,especially when I am on stress.I heard psoriasis treatments can result in other diseases lime cancer. Is that true?I am 46 years old Doctor: Hello. Thank you for writing to us.Topical treatment for psoriasis is absolutely safe.Since you have been using only topical treatment therefore you need not worry about malignancy.Systemic treatments like Phototherapy and immunosuppressives like cyclosporin can result in skin cancer and other internal malignancies after years of treatment.Phototherapy involves exposing the body to UV A rays which are carcinogenic in the long run.Cyclosporin is an immunosuppressive and years of its use can lead to skin malignancies as well as other malignancies of the lymphoreticular system.You can talk to your doctor for more details regarding this.Regards"
},
{
"id": 187413,
"tgt": "What to do for bump in the gums, upper right cheek pain?",
"src": "Patient: I'm having some upper inside cheek pain on my right side just near my top row of teeth. It seems like the area might be inflamed near the gums but not where it hurts. There is also an invisible-like bump like thing around a half an inch long? I'm really confused and it's painful when I eat! Doctor: Hello, Welcome Thanks for consulting HCM, I have gone through your query, as you have mentioned that you have as you have upper painful inflamed sweling in gum it can be may be due to subgingival deposition of Calculus , may be there is formation of Periodontal Abscess or may be due to Root caries there is formation of Periapical Abscess , You should do is go warm salt gargle 2 -3 times a day Consult dentist and go for IOPAx ray then if periodontal abcess go for Scaling and root planning, If periapical abscess go for Root canal treatment .Hope this will help you. Wishing you good health Regards Dr. Priyanka tiwari"
},
{
"id": 49976,
"tgt": "Skin discoloration like pinpricks from toes to ankles, freckles. What could this be?s",
"src": "Patient: My husband has some skin discoloration that looks like pinpricks of brown running from the top of his toes to his ankles, on both feet. We at first thought they were freckles, because of the pigment color, but they have been spreading. My husband is 22 years old, in good health, so we are unsure on how we should proceed. What could this be? Doctor: these can be due to drugs used for other ailments even if off and on .ike paracetamo ibuprufen calcium vitamins and diet supplimentscan be soaps shampoo oils cooking oils or body application materials need to investigate on these lines and treat the cause"
},
{
"id": 200806,
"tgt": "What causes weakness after urination and pain in knees and elbows?",
"src": "Patient: Hey.I m Rameez Ahmad and my age is 22 years. Urine drops discharge for two years.For two month.....I m felling weakness after urine especially in morning..I m getting weak, Secondly, a small clot is along my right testicle.I feel pain on knees and elbows. My urine test is right now and range of Uric Acid is 4.5 Doctor: Thanks for asking in healthcaremagic forumIn short: Your Urine and knee problem may not be relatedExplanation: Your history is quiet confusing. Please send what is your problem at present. Knee pain and urine is not connected as such. For your clot in testis you have to visit a surgeon who can drain it if large and prescribe you some medication to prevent infection. Hope you understood, Please let me know."
},
{
"id": 176270,
"tgt": "Are Z&D drops,Entromax sachet and ORS drops the correct treatment for loose motions?",
"src": "Patient: Hiii my baby boy is 16 month old... He s suffering from loose motion since morning. I went to doctor s place. He prescribed Z & D drops, Entromax Sachet and ORS for loose motion. Is it ri8 to give all these medicines for same problem. Plz revert me soon. Doctor: Loose motion in a child could often be caused due to a gastrointestinal infection. The mentioned medicines are rightly prescribed and need to be given. Since it is most often caused due to viral cause, antibiotics are not required. Maintain proper hydration by giving ORS (oral rehydration solution) prepared by mixing a full sachet in a litre of plain water. Also give your child plenty of curd or yogurt since it contains beneficial bacteria."
},
{
"id": 165352,
"tgt": "Is diluted cows milk, safe in infants?",
"src": "Patient: hi doctor! i am worried about my 5 months old baby ...she is totally on breast feeding but my milk is not enough for her and i want her to be assisted with one doze of feeder ..but whenever i use any cereal milk she start doing loose motions(stools)...but in last few days i started her giving diluted cow s milk once a day she is quite fine with that but i am worried that is it safe for her or should i discontinue? Doctor: hi, cow milk can be given to child. But it should be given in undiluted manner. Nothing should be added to the milk. Take care."
},
{
"id": 35350,
"tgt": "Does having itch around vagina and passing white discharge, mean that azithromycin is working?",
"src": "Patient: I'm 19 weeks pregnant and I\"m diagnose with chlamydia my primary doctor prescribe me azithromycin it was only 2 pills which the label said take both pills as one dose it's day 2 and I have this really bad itch around my vagina it doesn't burn when I urinate but I do have white discharge my question is with these symptom can it mean that the treatment is working or do I need to go back to the clinic and get checked and prescribe me more antibiotics? I understand its my 2nd day going on 3 I need helpful answers please. Doctor: You most likely have developed a yeast infection as a result of the antibiotics. This requires a different treatment than chlamydia. It is best to go back to the clinic, get it evaluated, and confirm the diagnosis. The azithromycin most likely has worked, but you may have developed this secondary infection as a side effect. You also want to be sure you don't have another STD which wasn't initially detected. Trichomonas is a parasite that can also cause itchiness and discharge. Hope this helps."
},
{
"id": 6499,
"tgt": "I have a D&c done in 2/01/11 on 2/18/11",
"src": "Patient: .. I have a D&c done in 2/01/11 on 2/18/11 took a ovulation test and i was +.We have sex after that day could i be pregnant , the only sign i have now is heartburn . Doctor: Hello. Welcome to HealthcareMagic forum. Having heartburn is a very non-specific sign to comment about your pregnancy that too when your period is nit even overdue. You wait for at least one more week and if you dont get your periods by then, you go for home pregnancy test to confirm. Dr. Rakhi Tayal"
},
{
"id": 119081,
"tgt": "Have S in Hb. Negative for sickle cell trait. How to test partner?",
"src": "Patient: i did a sickle cell trait and themianasis test with my partner and the result is negative,now i go for hb electrophoresis and they detect S in our haemoglobin....and required for a further test...do we proceed.......while am doing all this test is because i dont want to give birth to a sickler...cos of AS(GENOTYPE)...but i dnt know my partners own here...and no such test to know if his AA OR AS here in pakistan...my mail i.d is YYYY@YYYY Doctor: Hi, Thanks for placing your query here. I can appreciate your concerns. All you & your partner need is an hemoglobin electrophoresis or HPLC (high performance liquid chromatography ) test. I am sure the test should be available in pakistan. If both are AS (sickle cell trait) ,the chances of baby being SS (sickle cell disease) are 25% with each child birth. If one is AA & the other is AS , the baby at the max can be AS only (no reason to worry) Hope I have answered your query. If you have any further questions I will be happy to help."
},
{
"id": 69332,
"tgt": "Suggest remedy for lumps in rectum",
"src": "Patient: I am taking metrogyl 400 and flucloxacillin 500 . I have a large lumpin my rectum which is quite sore it feels like a boil and doctor prescribe this medication for me what do you think he also said if it dosent help it would have to be drained. Please help me my name is maria Doctor: hello Maria,Its a boil, if it contains clear water. But when it gets infected, it gets filled with neutrophils also i.e. forms pus. Then, you call it abscess.Usually an abscess needs to be drained if it does not rupture. As it increases in size, the pain severity increases and slowly ruptures itself. You are having a good antibiotic cover and you can continue it for five days and it should rupture by itself. If it does not, an OPD procedure will be done maybe within ten minutes and after that you will require regular dressings until you heal completely.If the pain becomes severe, you can take painkillers like Diclofenac or aceclofenac. Keep the area clean and avoid touching it and simultaneously touching other areas. It could spread to other areas by touching as it is a bacterial infection. As it is a small localized lesion, you might not suffer from fever. Always try to keep the area dry.Hope I have answered your query. Any further queries, I am here to help."
},
{
"id": 92439,
"tgt": "Can sharp pains in upper abdomen be related to spleen issues?",
"src": "Patient: I have been having upper left abdominal pain for the past week or so. It isn't severe or sharp pain, more like a dull and uncomfortable pain. I know that this can be something related to my spleen so I am worried. Would you recommend going to an emergency room right away or should I just go to a regular doctor tomorrow? Doctor: Hi...go to a regular doctor tomorrow.... nothing urgent if you have not been hit in that area or you are not having fever...Dr. Ashish Verma"
},
{
"id": 131986,
"tgt": "How to treat severe knee pain that worsened after accidental twisting?",
"src": "Patient: I have had knee pain for several months and yesterday I twisted knee while stepping on my shoestring. I heard a popping noise and pain was so severe I thought I was going to pass out. I immediately sat down. I can stand and walk now but not without limping but pain is severe. It really hurts when sitting with leg extended straight. Doctor: Hi you had knee pain for several months and have now injured it during a fall. Knee is swollen and you can walk with limp. In my opinion you have probably damaged a meniscus / Cruciate ligament in your knee. Take pain killers , use knee support for walking and get an MRI to arrive at proper diagnosis."
},
{
"id": 105818,
"tgt": "Does furuncles affect one with weak immune system ?",
"src": "Patient: Doctor , i am being affected by furuncles. i am more worried whether my immune system is weak. also i have bronchitis .what to do?wether i have any serius health disease.once 3yrs back i has sex with women bu used condom. wether i have HIv infection Doctor: Hi! donot stay in doubt,otherwise you will always be thinking about that.Get HIV test done. It is an blood test routinely done in blood bank and even done for pregnant couples. Consult your Doctor for furuncles and bronchitis."
},
{
"id": 185496,
"tgt": "Suggest remedy for bad breath",
"src": "Patient: Hi! I'm having problems with my breath. I've tried licking my wrist & scraping the back of my tongue but I don't feel any particularly bad smell, it smells a bit sour, but nothing extreme. However whenever I'm in a room talking to people they discretely hold their noses. People smell my breath from 6 feet away. I've visited the dentist twice (my teeth are in perfect health, never had a cavity) and been to 3 ENTs, but they can't find anything wrong so I'm stumped. Doctor: Hello!Welcome to HCM.Gum infections can cause bad breath.As you saw your dentist and ruled out a dental cause, i suggest you to see your physician.Check if you have GERD.smoking and other tobacco habits also cause this.You did not mention anything about your general health. Diabetics,liver and kidney disease ,tonsillitis and other respiratory infections potentially cause halitosis.Check if you have a low salivary flow.If any of the above includes your general health, this has to be corrected immediately.Use saliva substitutes if you suffer low saliva flow.Keep away garlic,onion,tobacco products.Brush your teeth and tongue twice daily.Maintain a good oral hygiene.Regards."
},
{
"id": 178359,
"tgt": "Suggest remedy for persistent fever",
"src": "Patient: Sir, My son aged 8 is showing 1-2 Rbc, 5-6 WBC, Ph 6.0 and BF increased in stool examination earlier he was in fever 102-103F, since last5 days, all blood pathology in OK showing no ailment no urine infection but not getting fever down. Also showing weight loss now weighing only 20 kg . Need your help Kindly help. Thanks Doctor: hi,thanks for reaching ushe has fever of 5 days without any other issues..only urine test is done,which is normal...so, better way to go further would be to do few basic blood tests to rule out infectious causesstart with 1)complete blood count,peripheral smear for malarial parasite,ESR and see a pediatrician with the reports2)meanwhile, fever should be controlled with paracetamol syrup every 6 th to 8 th hrly to Keep his temperature less than 100*f , give adequate oral liquids to keep him well hydrated.regardsDr sowmya"
},
{
"id": 214495,
"tgt": "How to treat ruptured elbow bursa sac ?",
"src": "Patient: I believe I have ruptured the elbow bursa sac on my right elbow. Fluid has accumulated in the area about the size of a ping pong ball. There is little to no discomfort other than a slight tightness due to the fluid accumulation. Are there any home remedies for treating this condition? Doctor: HIThank for asking to HCMI really appreciate your concern and looking to the history given here I could say that at one hand you are talking about the ruptured bursa and at other hand you are asking for home remedy how come this could be possible, in my opinion it is advisable to consult the surgeon to get it confirm and if this is a same condition as you said then this would need to the surgical intervention, have a nice day."
},
{
"id": 79607,
"tgt": "Based on my chest x-ray, is this something a diet can fix?",
"src": "Patient: Hi, I had a chest x-ray/history Hypentension, chest discomfort Results;interstitial scarring in the upper lobes and the bases. No evidence of active pulmonary disease. Cardiomediastinal findings, associated with hypertension and suggestive or left ventriculat hypertrophy. Is this something a diet can fix? Any suggestions? Thanks for your help. Doctor: thanks for your questionI completely understand your problemthe ans to your question is yes ,in case of hypertension u need to take minimal salt ,or any sodium rich food and regular exercise. in diet you can have fruits and vegetables (preferably raw) food with lots of fibre are very benefitial like kidney beans, corn, pear, apple , broccoli, carrots, raddish, sprouts , spinach, legumes.regular execise is mustthanks /regardsfeel free to ask more questionsmay god bless you with good health"
},
{
"id": 167015,
"tgt": "What causes stomach pain and sticky clay coloured stool?",
"src": "Patient: Hi, may I answer your health queries right now ? Please type your query here...my son is 2.5 yrsold ispassing sticky clay colored stool. was having fever till yesterday and was also suffering forstomach pain 2 daysback are they symptomsof jaundice Doctor: Hi.... These are not symptoms of Jaundice. These are symptoms of viral or bacterial intestinal infection.Unless the kid's having low urine output or very dull or excessively sleepy or blood in motion or green bilious vomiting...you need not worry.Regards - Dr. Sumanth"
},
{
"id": 30922,
"tgt": "What is the right dosage of Macox plus tablet?",
"src": "Patient: Hello,I am suffering from Pulmonary TB from Jan'14. The doc has prescribed Forcox 2 tabs OD empty stomach x 2months. Post two months It has to be stopped and replaced with Macox Plus OD x 4months. The medicines were prescribed to me in a different city when I was travelling but now I am back to my city and cannot check with the Doctor who prescribed the medicines therefore I request you to guide me with the dosage of Macox Plus which will start in two weeks time. Since the doctor has not mentioned number of tablets how many should I take in a day. My age is 37, sex - male.Thanks,Gaurav Sharma Doctor: Hi & Welcome.Tablet Forecox contains Isoniazid, Rifampicin, Ethambutal & Pyrizinamide whereas Tablet Macox contains Isoniazid & Rifampicin.These are the first line drugs used in the treatment of pulmonary tuberculosis.As per the Revised National Tuberculosis Control Program, DOTS (Directly Observed Treatment - Short course) is recommended for the treatment of tuberculosis wherein the drugs are taken only thrice a week instead of daily. This is cost effective, results in reduced side effects and improves patient's compliance (capacity to take the tablet regularly). As per the DOTS you should take the medicines as follows. 1. Two tablets of Forecox to be taken once daily, thrice a week for 2 months & followed by 2. One tablet of Macox to be taken once daily, thrice a week for 4 months. All the medications should be taken early morning before meals as this helps in proper absorption of the drugs. Pyridoxine 10mg to prevent neuropathy (caused by Isoniazid) is added to this regime.You have been prescribed the correct treatment for TB. Since you have completed 2 months of treatment with tablet Forecox with Pyridoxine, now you need to take one tablet containing Isoniazid & Riframpicin (Macox) along with Pyridoxine for remaining 4 months.Hope this will help you in getting the proper treatment."
},
{
"id": 201886,
"tgt": "What does the sperm analysis with 70% motility indicate?",
"src": "Patient: Hi I'm a 42 year old male, hypertensive taking co-approvel. my liquefaction time is after 3 hours. with abnormal froms giant head 2%, constricted head 2% pin head 2% sperm count of 109,600,000 abstained from sex for 4 days and fructose positive. Motility is 70%, with 65% progressive, 5% non progressive. Pap's morphology is 15%.What medications can I take, we are trying to have a baby. Doctor: Hi. Were there any pus cells in the Semen? If yes did you get a culture done? Your count is slightly low. Otherwise the report is within normal limits. If there are pus cells then you should take antibiotics in consultation with your Doctor. You should get yourself evaluated for a Varicocele which is a common treatable cause for low sperm count. If all this is ok I usually prescribe my patients an Ayurvedic preparation from Himalya called Speman for 3 months which help in increasing the sperm count. Take care and feel free to contact me using the ' Ask a Specialist ' section of the site. Dr Rishi, New Delhi, India."
},
{
"id": 136299,
"tgt": "How can the implant be managed?",
"src": "Patient: I have an implant in my great toe which was done in 2009. 3 weeks ago I developed an infection in the toe with purulent matter being emitted from the base of the nail bed. I went to my primary care physician who took me down the hall to a podiatrist. He treated it with antibiotics for 3 days and then removed the lateral section of the nail bed for an ingrown toenail. I was treated with a 10 day treatment of antibiotics and have been going back weekly. The nail and tissue around the nail is better but still red and tender. I am on a 2nd round of antiobiotics and the physician wants a 2nd opinion and has sent me to an infectious control physician who I will see next Wed. Any thoughts, suggestions. He did not feel it was involving the implant which is in the proximal phalanx. Doctor: Answer: Hello,I have studied your case.I will advise x ray to see union status of your boneMedication like antibiotics and analgesic will reduce pain; you can take them consulting your treating doctor.if union has been achieved implant can be removed at a later stage because any new damage at implant site may decrease blood supply to nail bed damaging further so plan implant removal once your current problem subsidesHope this answers your query. If you have additional questions or follow up queries then please do not hesitate in writing to us. I will be happy to answer your queries.Take care."
},
{
"id": 163848,
"tgt": "Suggest treatment for fever ,cold and running nose",
"src": "Patient: Hello, my daughter has been having a fever off and on for 6 days now and a bad cold coughing runny nose and does not want to eat at all. I took her to see the doctor but all he said is to give her tylenol for the fever and that is all. I am afraind that she might be dehydrating what can i do? Doctor: dear userfrom your words i would like to tell you that your daughtet get viral infection that causes common cold and it was good behaviour to see the doctor to role out bacterial infection that may need antibiotics for treatment.....the treatment of this case is tylenol as antipyretic and may be cough sedative syrup helpful ...but you should know that viral infection causing common cold take course of few days and improvement occur gradually ...the treatment is only symptomatic...i feel your worry about fever so i want to say that fever can cause dehydration and we should manage it...tylenol has good effect against fever if the fever is mild but the most important step you should do is cold compresses.....cold compresses before any antipyretic treatment is very very important to manage fever ...cold compresses is the first step in lowering fever so you should do cold compresses for 10 minutes when the fever rise then give her tylenol...you can put her gradually under running water for relief of fever as the high grade fever doesnot reduced with out that even if you increased the dose of tylenol......so you can take this simple step and get out your worry and your daughter will be fine.......wish you and her ahealthy life"
},
{
"id": 33259,
"tgt": "What causes sore throat,headache,severe stiff neck?",
"src": "Patient: My 16 yr old daughter came home with a bad sore throat and a headache 2 nights ago. Now she also has a severe stiff neck and her throat is just getting more sore. She had diahhrea this morning and felt like she was going to vomit, although she didn't. She is not running a fever according to our thermometer, but she rarely does, even when she is sick. Any ideas? Doctor: Hi,Thanks for writing in to hcm.I understand your problem and will try my best to help you out.e haKindly consult a physician at the earliest for medications like1. Antibiotics - Tablet amoxycillin 500mg thrice a day for 5 days if she is not allergic to it.2. Antihistamines like - Cetrizine 10mg once at night for 5 days.Ask her to do salt water gargle/ betadine mouth gargle.Take plenty of fluids.It seems like a bacterial infection and since she is having neck stiffness it is important to consult a physician to rule out symptoms of meningitis by performing clinical examination. The stiffness in the throat may be also due to enlarged tonsils . hence kindly visit a doctor at the earliest. Hope this answered your query.Get well soon.Dr.Riyanka"
},
{
"id": 31133,
"tgt": "What is the treatment for bartonella infection?",
"src": "Patient: i got bartonella in 06 and still sick was in hospital 42 days its the after math that is so bad i have care givers 4 hrs a day pain is so bad on alot of things so i can walk and not cry husband left me over this need support group finiallypeople are talking about it on tv i was on dallas tx news in o6 dateline at one time wanted to get me on it is worse than they cdc saids i am in denver now alone cant work lost business and my life Doctor: Hi, Welcome to HCM.I have gone through your query and I understand your concern regarding the effects bartonella infection has had on your life.I empathise with you and I suggest you make some lifestyle changes to make things easier for you.You have not mentioned the treatment details you have received for this infection.Bartonellosis can be very debilitating and may cause severe fatigue in many individuals.Such individuals need counselling and you need to practice some relaxation techniques to decrease your anxiety.A regular low intensity exercise regime can be inculcated in your lifestyle slowly to elevate your mood and also make you stronger and fit.Get in touch with a nutritionist and plan a high protein diet.These tips will help gain your health and aide faster recovery.Wishing you fast recovery.Thanks."
},
{
"id": 137169,
"tgt": "Suggest remedy for pain in knuckles",
"src": "Patient: hello, i got hit in the hand by a baseball today while i was batting. my knuckles use to look identical but now two of them look indented/smashed down, there s slight pain and i can still move my fingers and open my hand but will my knuckles go back to normal? how long is the normal healing time Doctor: Hi there.You could have got your fingers dislocated at the knuckles. I advise you to go to an Orthopaedic surgeon as soon as possible and get evaluated for fracture or dislocation of the fingers."
},
{
"id": 163306,
"tgt": "What causes my daughter's recurring brief headache?",
"src": "Patient: Hi Doctor, I have a 2.5 year old daughter and she recently complained of a sharp head pain above her left year. This has happened all of a sudden and she cries a lot. After 2 minutes, she stops crying and says the pain. This has happened 6 times in one week. We have shown her to 2 doctors and they suggested if there are no other neurological sysmtoms, then nothing to worry. Please advise. Doctor: Hello,Go for eye checkup and blood pressure recording. It could be Migraine. If nothing positive, then do not take it lightly and go for brain CT scan.Hope I have answered your query. Let me know if I can assist you further.Regards,Dr. Hina Javed"
},
{
"id": 131059,
"tgt": "Ankle tender after bent inwards due to fall",
"src": "Patient: Fell over today and ankle bent inwards its now very tender when i move it side to side and i cant put my full weight on it and its quite stiff. I work as a cleaner for 4 hours a night but really don t feel i can! How long should i rest it as i think iv e sprained it. Doctor: These trauma are treated by topical and oral non steroidal antiinflammatory. Heat trauma is sripus and need CT brain to be done on it immediatly."
},
{
"id": 158256,
"tgt": "Took chemo radiation, surgery for esophageal cancer, CT showed nodes in mediastinum. Explanation ?",
"src": "Patient: my husband just went thru chemo radiation and surgery for esophageal cancer they took 3 inches of the lower esophagus and the top half of his stomach and all kinds of nodes and stuff in the surrounding area taken out he just had a ct with contrast that showed he has subcentimeter nodes in the mediastinum , noncalcified nodules in the interval , granulomatous disease , hepatic steatosis , right adrenal nodule adenoma , calcified granuloma , ill-defined subpleural density can someone please explain these findings to me Doctor: Hi,Thanks for writing in.Hope your husband is responding well to the chemoradiation for esophageal cancer.Let me explain the imaging findings in simple language.1. CT scan with contrast that showed he has subcentimeter nodes in the mediastinum: These are very small nodes and need to be observed for any change in size or number in the future. The esophagus can drain into these nodes and this is used in nodal staging of cancer.2. Noncalcified nodules in the interval , granulomatous disease: This signifies old infections in the region. Nothing serious.3. Hepatic steatosis: Fat accumulation in the liver. Not a serious finding.4. Right adrenal nodule adenoma: Non cancerous nodule is seen in the right adrenal gland ?hyperfunctioning.5. Ill-defined subpleural density: Some small thickening noted above the diaphragm, size and location need to be known to call it significant.Hope this helps."
},
{
"id": 188246,
"tgt": "Had a cracked tooth and numbness. Pain is severe and punched in face. Suggest the next step for painful teeth",
"src": "Patient: Hello, I had a cracked tooth number 4 that I had a crown prep done. The dentist had a hard time numbing the tooth. I did get root infection and was on clindamycin. Infection has resolved. Soon after I needed a root canal on number 4 since I still had temporary crown on the endodontist did the root canal and went back to doctor. At next appointment I was supposed to get crown placed on number 4 and a small cavity filled on another upper tooth on opposite side of mouth. Both teeth did no get numb and I felt so much pain. The dentist said that he does not understand why I don't get numb since he has not seen another case like this. Can this be related to TMJ or another nerurological disorder. Since then I have been having upper and lower tooth pain where I have no cavities or fractures, and pain where crown was placed on number 4 and pain where single cavity filled. This pain is severe. It feels like I have been punched in the face over and over. What would be the next step in care for my painful teeth. Not just 2 teeth several upper and several lower. NO cavities or previous pain associated with teeth until recently after Number 4 had crown prep. Doctor: HiThanks for writing in.If your tooth doesn't get anesthetized there can be several reasons:--abscess tooth-alcohol consumptionStill neurological condition like trigeminal neuralgia can also lead to severe pain unilaterally.That pain is so severe & unbearable.You need to rule out proper cause & get it treated.Consult dentist & get complete treatment.RegardsDr.Neha"
},
{
"id": 8019,
"tgt": "How to treat the pimples and its black spot ?",
"src": "Patient: sir i have problem of pimples &black mark . i tried more but cant give any better result sir plz help me Doctor: hi u need to take care of ur skin better to consult dermatologist u need to take medicine depend upon the grade of the acne there are many treatment option for acne and its spot and scar only thing u need to take regular medicine for it for 2-3 month drink more water avoid oily and spicy food avoid touch ,pick or popping avoid sun exposure use good face wash with salicylic peeling ,LASER ,microderma are best option for u"
},
{
"id": 41268,
"tgt": "What causes constipation and rectal pain while on infertility treatment?",
"src": "Patient: Hello.I'm 23, female. 6'0, 250lbs. I'm allergic to nothing. I've had my gallbladder and tonsils removed but other than that I'm fairly healthy.I am taking Femara this month for fertility. My worst fear is developing HOS (Hyper-ovulation Syndrome). Today I am suppose to be ovulating and I'm having constipation and mild rectal pain. No swelling that I can tell. Just uncomfy, and I'm obviously nervous. Nobody ever mentioned pain in my backbone while on Fertility drugs so I'm a little unnerved. Doctor: Hello,Welcome to Health Care Magic.Thanks for writing.I am Dr.Nitasha Khan. I have read you question completely, i understand your concern and will try to help you in best way possible.there is just no reason to worry about its nt related cnstipation might be because of food and rectal pain may be because of constipation just take luke warm milk and put 1 tea spoon of olive oil in it you wil se the miracle it wil go also dnt take stress you are on treatment and thy usualy work nthng to wory about the less you worry the more favourable it wil be for u and your family it wil help u cnciveve and dnt wory u wil nt hv hos doc has preacribed desired dose ....you wil be fine eat healthy and drink lots of fluidsI hope this answered your question.If you have more queries I am happy to answer.Otherwise rate and close the discussionRegards.Dr.Nitasha Khan"
},
{
"id": 161744,
"tgt": "How can fever in a child despite administering Imol syrup be treated?",
"src": "Patient: Hi doctor my daughter is 4 and half years as she had fever from 2days doctor has given imol syrup for 3times a day but after giving night dose again her fever has increased in mid night can i give imol syrup now I,m scared because its more than 3dose now please suggest me and i want to mention that I gave her last dose 7hours back pls suggest me doc Doctor: Hi Welcome to HealthcareMagic Ma'am, please note that you can safely repeat proper dosage of Paracetamo. As a rule, it should never be repeated before 4 hours. Other common anti-fever drug used in children is Mafenemic acid it can be repeated 8 hourly . Please increase fluid intake of child. If you have any further query then you can come back to HealthcareMagic. I will definitely help you . With Regards Dr Varinder Joshi"
},
{
"id": 189274,
"tgt": "Baby has fever, gums are swollen, foul smelling. Is he teething? Is it gingivitis?",
"src": "Patient: hi my 19 month old boy started with a fever a week ago which subsided after 2 days then i noticed a foul smell coming from his mouth and i thought he may be teething ,i looked into his mouth and his gums all appeared swollen there were no signs of thrush , i started to brush his teeth and he started to bleed on contact with the brush, this has been going on for three days . i started antibiotics today thinking this may be gingivitis bc of the foul smell swollen gums and bleeding with brushing , on two occasions he has cried and his gums started to bleed during the crying episode .. is this gingivitis ? i have been using chlorhexidine spray and the swelling has settled a little along with the antibiotics, but because of the bleeding i just wanted to make sure i wasnt missing anything Doctor: Hello there, Thanks for your query. At the age of 19 months swollen gums are sign of eruption of deciduous/milk teeth,pain is because of pressure exerted by erupting teeth on the gums. Therefore there is no point to get worried. But during this period children more prone to infection,so maintain good oral and general hygiene. Fever may be because of viral infection,so do not give any medicine yourself, consult with a child specialist at the earliest and take a proper treatment. Hope this will be helpful. Take care. Dr.Gunjan Gupta"
},
{
"id": 158909,
"tgt": "Pap smear shows superficial and intermediate squamous cells, intraepithelial lesion. Cancer?",
"src": "Patient: Doctor,My pap smear shows superficial and intermediate squamous cells in a background of mild inflammatory cells. There are no features of intraepitheilal lesion or viral change.andResult says Negative for intraepithelial lesion or malignancy .But my doctor have already done elctrocautery treatment . Do I need to worry about precancer/cancer in future. Doctor: Presence of HPV infection confirmation by DNA screening , if suspected,,colposcopy and biopsy of any abnormal spot are generally also carried out for determining pre disposition . if all these are negative, nothing to worry, I am sure your Doc.must have got it all done as per necessity,seek second opinion with other Gynecologist also if possible in an institution."
},
{
"id": 77259,
"tgt": "Is it safe to take Azithromycin for bronchitis?",
"src": "Patient: My daughter's pediatrician informed me that she has bronchitis and prescribed azithromycin. Is it ok for her to also take Hyland's homeopathic cold and cough with this? Active ingredients include allium cep 6x HPUS, hepar sulph 12x HPUS, natrrum muriaticum 6x HPUS, phosphorus 12X HPUS, pulsatilla 6x HPUS, sulphur 12x HPUS, and hydrastis 6x HPUS. Doctor: Thanks for your question on Healthcare Magic. I can understand your concern. Yes, you can give her Azithromycin. No harm in it.. Most common cause for bronchitis is infection. Azithromycin being a broad antibiotic, helps in infection. So you can give her Azithromycin. About homeopathic syrup you are talking about, in my opinion, you should avoid it. Better to give her allopathic syrupy for bronchitis. These homeopathic drugs are not tested under clinical trials so their safety is always in doubt. So better not to try them in children. Instead of this, you can safely give allopathic syrup as these drugs are time tested and proven safe in various clinical studies. Hope I have solved your query. I will be happy to help you further. Wish you good health. Thanks."
},
{
"id": 151256,
"tgt": "Stressed, nausea, pins and needles in hands and legs, blurred vision. Panic attack?",
"src": "Patient: I have been under an extreme amount of stress .....sometimes it feels like the air is not going into my lungs....I feel nausea ...my hands, mostly my left gets pins and needles and my hands shake. Last night, while having dinner, all of a sudden, I felt nausea....my vision went kindof weird, and I went in to lay down. I knew I wouldn t make it to the other room so I stopped and held onto he table to get my bearings. Next thing I new, I was waking up on the floor with my husband calling my name into my ear. I was terrified. I have taken my blood pressure a few times today and the readings are 130/84 pulse 108, 111/86 pulse 97, and just now 112/83 pulse 92. All day long I have this scared/nervous feeling. Is this a panic or anxiety attack. I am 48 years old, 5 ft 1 in, and 135 lbs. thank you for your prompt reply. Doctor: these symptomps are of some food allergies some food item may not be suiting you causing the effects narrated by you you get blood serum tested for specific antibodies for certain common foods you will defintely find answer"
},
{
"id": 20500,
"tgt": "What causes irregular heart beat, fatigue and hypertension?",
"src": "Patient: Good day dr!, I am ena san diego, 31 years old female, I have a resting heart rate of 105 rpm, it gets to 120-140 with a little physical activity. I tested negative for hyperthyroidism and pheochromocytoma. I also undergone stress test and 2D-echo, my cardiologist told me that I have normal heart with a fast heart rate. He prescribed ivabradine 7.5 but didn\u2019t give any diagnosis. Is this Inapropriate Sinus Tacychardia? Aside from a fast heart beat, I also feel tired most of the time, I am hypertensive, I have GERD, constipation, heat and cold intolerance. All of these symptoms came 3 years ago, and I have been to different doctors and no one can diagnose what I have, most of them will say everything is normal. How come it will be normal when I feel all these symtoms, help me doctor, thank you Doctor: if you have a hypertension it most probably is due to essential hypertension that means there is no specific reason for it but your physician muster roll out any underlying good causes which amounts to about 5% of patients with hypertensionIf the hypertension is the severe it can cause if regular heartbeats if it is a sinus tachycardia that means that is the main sinus when you have fast heart rate it may most probably be due to anxiety due to poor conditioning or due to dehydration and If you have fast heart rate up to 140 the rhythm has to be seen by Ekg if a sinus then again it can be due to poor conditioning and the treatment for these conditions can be for the underlying the cause like treating the hypertension if the EKG shows any rhythm like like it real flatter than it needs a specific and tie a rhythmic treatment so if you want to send me a copy of ekg I'll be glad to see and then advise you specificallyYour fatigue can be due to depression or anxiety order to uncontrolled blood pressureRead news and fast heart rate can be there without any organic heart disease you should pick up a reliable cardiologist and do not to visit different positions and then you'll get the answer or you can give me more details thank you very much"
},
{
"id": 40880,
"tgt": "How can amenorrhea and acne be treated for a successful pregnancy?",
"src": "Patient: Hi im 25 married girl i was diagnosed pcos due to amenorrhea 7 months back according to ultrasound i was having 8 cm simple cyst in my right ovary and many cysts in left ovary my gynecologist advised me to take neodipar 500mg TDS and wait as 8cm cyst may resolve while taking this medicine and no surgical intervention required.its been 5 months im taking this medicine but still no periods and im also facing acne problems.what can i do now?can i start diane 35 for 1 or 2 months and continue neodipar which im already taking bcoz of my acne or i should avoid any external hormonal stimulus and wait till my periods come naturally without any withdrawal medicine or diane? i dont know what to do as i realy want to resolve my acne amenorrhea and realy want to be pregnant Doctor: Hello , thanks for the query.Polycystic ovary is a very common problem which causes irregular periods. Let me help you understand few things.1. As you mentioned about your 8 cm right ovarian simple cyst is unlikely to resolve on its own as its size is bigger and it may need to be removed surgically. You may try to get ovarian drilling( about 4 punctures on each ovary) at the same time to help you ovulate( egg release process) regularly.2. Neodipar (metformin) is currently recommended in women who have intolerance to glucose (prediabetic like state) . But as you seem to be amenorrhoeic for a long time it helps if you continue it. For best results you need to maintain your weight by diet and exercise if you are on heavier side.3. If you are planning conception your doctor may suggest you ovulation induction drugs to help you release egg and conceive.4. Diane 35 will help you with acne but prevents pregnancy.5. For resuming your periods you may take T. Regesterone 5mg twice daily for 5 days . you can expect your periods once you discontinue medication. Hope this helps. Take care."
},
{
"id": 191007,
"tgt": "I brush 3 times a day but still my mouth smells",
"src": "Patient: hello sir, i think you can help me,so my problem is i brush my teeth three time a day but my mouth is smell still now.please kindly let mi know. thangtling tampa,fl usa Doctor: Hi, First of all it should be understood that malodor is caused by many reasons like decayed teeth, poor oral hygiene, gastric problems,Sinusitis, liver disorder, diabetis, smoking, alcohol consumption, tobacco chewing. You should consult a dentist to check for any problem in your oral cavity.If the problem is due to calculus and plaque it can be removed by scaling and polishing of your teeth. Flossing of teeth and tongue cleaning using tongue scraper should be done daily."
},
{
"id": 45127,
"tgt": "I am 20 yrs old, suffering from irregular periods, Is that i would not get pregnant because of this ?",
"src": "Patient: Hello, I AM 20 YEARS OLD I JUST GOT MARRIED IN MAY 7TH OF 2011. BUT I TEND TO SOMETIME GET IRREGULAR PERIODS. I HAVE BEEN GETTING MY PERIODS IRREGUARLY FOR QUITE SOME TIME ( EVEN BEFOR I GOT MARRIED). I CAN MISS MY PERIOD FOR 10 WHOLE DAYS AND NOW I THAT JUST HAPPENED AND MY PERIOD DID NOT COME SO I THOUGHT I WAS PREGNANT AND IM NOT. mY PERIOD JUST CAM TODAY WEN IT WAS SUPPOSED TO COME EXACTLY 13 DAYS AGO. wAT IS WRONG WITH MY BODY IM SO AFFRAID FOR MY HEALTH. wILL NI EVER BE ABLE TO HAVE CHILDREN?: ( Doctor: hi welcome, your problem related to hormonal imbalance only. for this , to make the hormone into normal level instead of going for hormone tablets i prefer you to go for alternate treatment,being an ayurveda doctor our system have so many medicines to make it regular and also we have ayurveda gynaec wing also for the details and better treatment write to us christyjoseph1@gmail.com sunithapoly124@gmail.com"
},
{
"id": 14453,
"tgt": "What causes red itchy widespread rash all over my body?",
"src": "Patient: i developed a very red itchy wide spread rash all over my body but especially on my torso after finishing my antobiotics ---is there a natural remedy that i can do or take to help the rash to get better please dont send me any attachments on my email if possible Doctor: Hello. Thank you for writing to us at healthcaremagicOne possibility that I can think of as the reason for widespread rash is a drug reaction probably to Oral antibiotics.Maculopapular drug reaction is the most common type of drug reaction to Oral antibiotics.If I was the treating doctor I would suggest you to try an OTC topical soothing lotion e.g calamine lotion, twice or thrice daily.An OTC oral antihistamine e.g cetrizine 10 mg once daily will provide you symptomatic relief from itching.If the rash does'nt respond as expected with this treatment, I suggest you to review it with a dermatologist.A severe drug rash may warrant a course of an Oral Or injectable steroids.Regards"
},
{
"id": 24470,
"tgt": "Can I ride a roller coaster after having catheterization?",
"src": "Patient: I am 52 yrs. old, 140 lbs., 5'71/2\". I had catherization 1 week ago, 2 stents and balloon. I still have a 30% blockage where they ballooned. I am going on vacation next week and would love to ride a roller coaster. Is this possible or what is the risk? Doctor: As such no contraindication for doing such things but being a heart patient ...avoid such strenuous or much anxiety producing activity...beside I don't know what is your baseline heart function. On echo...So best person to guide u is your cardiologist"
},
{
"id": 100099,
"tgt": "Any suggestion for scratchy throat, trouble breathing and heaviness in chest?",
"src": "Patient: I was cleaning my bathroom today and I think that I may have breathed in some chemicals. Right now my throat is scratchy. And I was having trouble trouble breathing. I did go outside and take a few deep breaths. No I do not feel so short of breathe. Still my throat is sore. And my chest feels heavy.p Doctor: HI, thanks for using healthcare magicIt is possible that your symptoms are related to a reaction to the chemicals.The reaction should decrease on its own, normally takes up to a few hours.It is important to remain in a well ventilated area.The use of anti inflammatory pain medication would help if any discomfort occurs.Eg- naproxen, cataflam, ibuprofenI hope this helps"
},
{
"id": 27516,
"tgt": "Suggest treatment for congestive heart problem in a 91 year old female",
"src": "Patient: My mom is 91 and just recently had a chest/heart x-ray which should some congestive heart problem with some water around her heart. Took her to cardiologist and he put her on Lasiks 1 per day and said to keep her feet and legs elevated. He wasn t real concerned, but did order an echo cardiogram to be done soon. Her home attendant just took her BP/pulse and reading was 128/77, pulse 100. She just started the Lasiks Thurs. May 7, 2015. She says she can breathe ok, but attendant says she s breathing heavy in a recliner chair w/feet up. She s adamant NO HOSPITAL. Do I need to worry? Doctor: Hi. Thank you for your query. I read it carefully.Now, the echocardiogram is really important in patients with congestive heart failure (CHF) in identifying the ejection fraction of the ventricle and in assessing the movement and compliance of the heart muscle. My opinion is that furosemide (Lasix) should be continued, but she should be put in a very low salt intake diet. Furthermore, she should be restricted to only performing routine activities of everyday life. The problem is that any moderate to vigorous effort, as well as other factors (e.g. salt) that increase the workload of a failing, insufficient heart, raise the possibility of decompensating the CHF and exacerbate severe symptoms, that may be even life-threatening. I hope I was helpful. Take care."
},
{
"id": 115748,
"tgt": "Does VDRL and TPHA level come to normal with medication?",
"src": "Patient: hi i am waheed from pakistan i had vdrl positive 1:8 then tpha negative i used tablets from skin specialist and after 1 month vdrl was 1:16 now i have used lycn and macrobac for 15 days and advised me to repeat vdrl and tpha tests after 2 months can it b negative Doctor: HIWell come to HCMHere the clinical symptom is important if there is not clinical symptoms then nothing to worry because VDRL may remain positive even after the complete course of disease, best is PCR test to know the active infection, take care."
},
{
"id": 46910,
"tgt": "Suggest treatment for right upper mid caliceal stone",
"src": "Patient: sirI am 21yrs. old ,75kg. wt.,182cms long and having a problem of right upper mid caliceal stone in kidney.As per CT uroscan picture it is locked in tight anterior middle calix.Attempts have been made to break it with ESWL 4 times it has not responded.Doctors are not advising going for PCNL, because it can be dangerous..So, what should i do now? Doctor: Hello and welcome to HCM.As an Urologist,i can understand your anxiety.Why did doctors not advise PCNL.Why dangerous? It's a routine procedure.The other treatment option is a flexible URS with laser lithotripsy.That's a routine procedure for all Urologists and done under spinal anesthesia.You're disharged in 24 hours. You can resume work in 2 days.If you've any doubts,send the reports to me,as a direct question.Dr.Matthew J. Mangat."
},
{
"id": 64872,
"tgt": "What causes lump in breast after started taking Novelon?",
"src": "Patient: Hello Dr, I had a period of 15 days and went to gynec for check up.(Im 28 years old, have a kid of 3 yrs) They had a scan and found a water cyst of size 5 cm in left ovary. Initially, she prescribed ovral g and i had it for 6 days. As i got vomitting and dizziness in morning i was askd to continue with novelon and discontinue ovral g. Now i had finished taking novelon for 21 days and already had ovral g for a week. So i totally took tablets for 28 days and had missed to take novelon for two days in between. And after started taking novelon(frm 3rd day), i had a breast lump in my left breast and it exists still...now it has been 4 days i completd taking the pills. But i havnt got the periods yet..wat shd i do now doctor for periods and breast lump? Doctor: Hi, dearI have gone through your question. I can understand your concern.You may have fibroadensis or fibroadenoma. You should go for mammography and if needed go for FNAC. It will give you exact diagnosis. then you should take treatment accordingly.Hope I have answered your question, if you have any doubts then contact me at bit.ly/Drsanghvihardik, I will be happy to answer you.Thanks for using health care magic.Wish you a very good health."
},
{
"id": 214491,
"tgt": "Sugest a home remedy for swelling of vein",
"src": "Patient: I was walking my dog an hour ago and her leash got caught on my hand and pulled downwards. The vein puffed up and I put ice on it right away. the vein is looking better but is still wider than normal and the swelling is still there. I can move my fingers and thumb. Just wondering if I should be doing anything different than just putting ice on it. Doctor: HIWell come to HCMBecause of the soft tissue inflammation the vein might have prominent otherwise this may not be anything wrong with the vein it self, you can keep the leg elevated for some time this would come around, if this is not then this need to be clinically examined, hope this helps."
},
{
"id": 144473,
"tgt": "What causes dent on top front of skull?",
"src": "Patient: I recently noticed I had a dent in the top front of my skull. I thought it was caused by my sunglasses sitting on my head too long but I ve suddenly realized that it isn t caused by my sunglasses. Today I have felt another one. Should I be alarmed and what could be causing this? Cyndie Doctor: Hello. Thanks for asking from HCM.I can understand your concern. The dent in skull could be due to various reasons: Bone moulding - It is a normal process which shapes skull and occurs during first 7 years age. During this, skull bone is formed from soft membrane cover and sutures between different skull bones fuse. No need to do anything for it. The dent due to it will be painless and present for long time.: Age related change - It is due to age causing bone sclerosis and areas of lysis which lead to irregular bone. It usually occurs after 45-55 years. It is also a normal process>No need to do anything for it. The dent is painless.: Post traumatic - Head injury can cause dent at impact site. It may be dent or depressed fracture. For confirmation, CT head is required. If there is any history of significant head injury before noticing dent, it could be the reason. You have to consult doctor for evaluation.Treatment will be decided after CT report. Depressed fracture may require surgery. Otherwise, normal dent is usually managed conservatively.: Bony lesion - it could be due to bony lesion like dermoid cyst/osetoma/fibrous dysplasia. It can be suspected if at dent site, you are feeling some kind of swelling. For confirmation, you have to consult doctor. He might take tissue from it to confirm diagnosis. Treatment usually requires surgical excision.Hope it will help you.Thanks. Take care."
},
{
"id": 60976,
"tgt": "What does a painful lump around the tailbone indicate?",
"src": "Patient: I don t really know how to explain my situation but I ll try my very best ,I have m.s ,fms,ra,Psorisis, psoratic arthritis, degenerated spine ,n alot of other health issues.I woke up n was hurting really bad on the side of my tail bone n lower back ,but I didn t check it out just got up n tryed to stay on my feet cause it hurt to sit down but my lower back got worse n worse n unbearable. Well I had to lay down cause I was done in severe pain so unbareable anyways I felt down my lower spine to my tail bone n this what I found out my lower back is swollen n between my lower back spine n my tail bone is a Knott (hard) on the edge of the spine .idk what it is or could be but its about a half inch n size or a little bigger n really hurts bad .thank you for your time. Ginger south Doctor: Hello,According to our clinical expertise, the lump around tailbone mostly infected pilonidal sinus or sebaceous cyst. Needs clinical confirmation with an expert consultant surgeon.Hope I have answered your query. Let me know if I can assist you further.Regards,Dr. Bhagyesh V. Patel"
},
{
"id": 104603,
"tgt": "Cold, running nose, productive cough, sneezing. Allergy symptoms?",
"src": "Patient: I have a cold \u2013 runny nose and a productive cough. No headache , chills or sweating. Feel OK apart from the runny nose and sneezing . Have had no need for bed-rest etc. But I did return from xxx on Feb 3. I couldn t have malaria , could i? As I say, I feel fine and have none of the other issues associated with the disease. Just being cautious! Doctor: Hello, Thanks for choosing health care magic for posting your query. From the description that you have given, it appears that you are having an upper respiratory tract infection,mostly a viral infection. In absence of fever it is highly unlikely that you may be having malaria. Regarding your running nose, you can start doing steam inhalation and salt water gargles. In case if symptoms are bad you can start yourself on OTC anticholinergics. Watch out for fever. If thefever comes then you should get yourself investigated for malaria as you have returned from endemic area. Hope I am able to solve your concerns. If you need any further help in detail, I will be more than happy to help you. Wish you good health. Regards, Dr. Pooja Reddy MD"
},
{
"id": 156640,
"tgt": "Should i be concerned about low grade prostate cancer inspite of having enlarged prostate?",
"src": "Patient: I had large bladder stone removed a few weeks ago. I also had an enlarged prostate that was only noticeable inside my bladder. My doctor did biopsy and the resulte show I have a very low grade of prostate cancer. What should I do. My doctor says its nothing to be concern about right now as my spa test was normal Doctor: Low grade prostate cancer with low serum PSA levels can be observed.You need to have regular serum PSA levels and digital rectal examinations for assessment of prostate. If PSA levels rise or findings of prostate change on examination,a re-biopsy will be required to see if tumor has progressed in grade. Till then close surveillance is an option."
},
{
"id": 64826,
"tgt": "Suggest treatment for a painful lump on the foot",
"src": "Patient: Hi, I stepped on a rusty nail about 2 weeks ago and went for a tetanus shot and antibiotics the foot continued to be a little painful, now this morning I woke up with a hard lump next to the puncture wound. This lump is only painful when I place pressure on it when trying to walk. Please assist me in what this could be and how to treat it? Doctor: HI,Dear,Thanks for the query to HCM.1-I have studied your query in depth.2-DEar,IN my opinion, your lump after 2 wk old nail injury,which proped up despite of antibiotics-is caused due to the infective indurated lump with anaerobic organism.3-Pain while walking confirms that-it is chronic-abscess in making4-This would need following treatment-NSAIDs, seropetidase tbs with changed antibiotic to cover anaerobic organisms,and drainage if it does not respond in 5 days time .For this you need to consult Surgeon and treat it under cover of surgeon.5-Hope this would solve your query to your satisfaction.6-Wellcome for more queries to me On HCM.7-Thnks once again for your inquisitive query.Have a Good Day..!!"
},
{
"id": 2982,
"tgt": "Are there chances of conceiving while taking norethisterone tablets?",
"src": "Patient: hi doctor,i have a history of irregular periods.i am 24,55kgs with 164 cms height.i want to conceive now.i took a clomifene tab for 5 days starting from 3rd day of my period and now i have been adviced to take 1 norethisterone tab daily for 10 days from 16th day of my cycle. do i have a chance of conceiving as these norethisterone tabs are supposed to be contraceptive,rt? Doctor: Hello and welcome to \u2018Ask A Doctor\u2019 service. I have reviewed your query and here is my advice. You are using correct drugs. I would suggest you to use medicine under the advice of the gynecologist. Clomiphene citrate will help in induction of ovulation in 80% cases and increase the chance of pregnancy to around 50%. So it should be taken from 3rd to 8th day of cycle. Noreethisterone contain progesterone and in your case it will not act as contraceptive. It should be taken in 2nd phase of cycle, from the 16th day.It helps to regularize the period by correcting hormone imbalance. It makes the uterine environment favorable for implantation and support pregnancy. You can also start folic acid tablet. Avoid stress, take healthy diet, drink plenty of water, do regular exercise. Do sex everyday or alternate day from 10th to 20th day of cycle. There may be a chance of pregnancy. Hope I have answered your query. Let me know if I can assist you further.Regards,Dr. Sagar"
},
{
"id": 206458,
"tgt": "Is inclination to play female characters in a drama, a disorder?",
"src": "Patient: Hi,I am from chennai. Before my marriage, i used to act as female role in dramas. After marriage, i left this because my wife does not want this. Past 6 months, i am having a thought of acting female role again in dramas. Whether this habit related to hormone problem? or is it some other personality? Doctor: DearWe understand your concernsI went through your details. I suggest you not to worry much. I think you are only inclined to play female role in dramas. Nothing more. There are several male bharathanatyam artists and how many you know have female behavior? I remind you of this because you are from Chennai. Please do not misunderstand things and put yourself in trouble. You have a passion for female role in dramas. That is all and that is good. just leave it at that and enjoy acting.Psychotherapy techniques should suit your requirement. If you require more of my help in this aspect, Please post a direct question to me in this URL. http://goo.gl/aYW2pR. Make sure that you include every minute details possible. I shall prescribe the needed psychotherapy techniques.Hope this answers your query. Available for further clarifications.Good luck."
},
{
"id": 67048,
"tgt": "What causes small lump on the outer wall of anus?",
"src": "Patient: hi I have a small lump on the outer wall of my anus next to, but not on the sphincter o the left side of it. What could it be? and, What can I do about it? It is not a hemorrhoid, I think, I had it there for a while and it seems that I did something that cut it or something because it feels like it s open, it does not hurt, it feels more like a sting, from a small cut. Doctor: Hi ! Good morning. I am Dr Shareef answering your query.Although it needs to be examined physically to reach at a diagnosis, I have got a strong feeling that it could be starting of a fistula in ano, and sooner or later you might get pain in it due to super added infection. I would advise you to get it clinically assessed by a general surgeon in your area for a proper diagnosis and appropriate management.I hope this information would help you in discussing with your family physician/treating doctor in further management of your problem. Please do not hesitate to ask in case of any further doubts.Thanks for choosing health care magic to clear doubts on your health problems. I wish you an early recovery. Dr Shareef."
},
{
"id": 201822,
"tgt": "Small sores spotted on veins during masturbation which opened into wound. How long will it take to heal?",
"src": "Patient: 38 years old enjoy masturbarting. Spotted small sores (about 5) on my veins (shaft), during masturbating. Sores opened up into wound after two days. Takes long to heal. What could b problem. Could it b a viral infection? It is itchy around the place. Another set has formed. Doctor: Hi. You should see a Doctor or a Dermatologist. This is not normal. Unless some rigorous activity has been done or exposure to some infecting agent this should not happen. I am refraining from advising any ointments as I have not examined you. But you would require some antibiotics to cover for secondary infection. Take care and see your Doctor whenever you can. Dr Rishi, New Delhi, India."
},
{
"id": 177688,
"tgt": "What causes vomiting sensation during meals?",
"src": "Patient: My 5 year old had an appendectomy over a month ago, and now complains that she feels like she s gonna throw up during every meal. Could this be a side effect of anasthesia? Could she have acid reflux? If so, can I give her something to help her while she heals? Doctor: HI...This is definitely not due to anesthesia. if it had not been for surgery I too would have considered the possibility of an acid reflux. Sometimes post surgery there is a possibility of band or fibrous formation and it might cause obstruction. I suggest you get back to the surgeon and if obstruction is ruled out then we can consider the possibility of acid reflux.Regards - Dr. Sumanth"
},
{
"id": 171624,
"tgt": "Is it safe to give Flucloxacillin with milk?",
"src": "Patient: Hi I d be very grateful if you d answer my question, please. My daughter was prescribed Flucloxacillin (& Amoxicillin) yesterday evening (for an infected post-surgery scar; the surgery was just over a year ago & this keeps happening so she s booked to her surgeon re: ? stitch left later this month) but I m finding it difficult to give all doses of the Flucox as directed. My question is: may I give Fluclox to my 16month old followed by (150ml ish) cows milk? She always has milk at 7am then goes back to sleep until nearer 9am (lucky me, I know!) when she then has breakfast. I can do the other 3 doses on empty stomach more easily. Many thanks Caroline Doctor: Hi Caroline,Welcome to Hcm,Yes its safe to give it with milk. There is no known interaction with the same. The intention is to make sure child gets the medicine. You are doing a great job. Take care."
},
{
"id": 210236,
"tgt": "Feeling presence of something moving around stomach",
"src": "Patient: I have been feeling odd for a while, but for the past couple of days, ive been feeling something inside of my stomach. it moves, so its not stationary, and ive taken several pregnancy test which all came back negative. What could this be? i feel this on the inside, as well as with my hand from the outside Doctor: Hello and welcome to Healthcare Magic. Thanks for your query.I understand that you are distressed due to the symptoms that you have mentioned. Sometimes, gastric problems can cause strange churning sensations in the tummy. However, from the limited information that you have provided, it is not possible to make a diagnosis at this time.A detailed evaluation including a proper physical examination is necessary to arrive at a definitive diagnosis. So, I would advise you to get yourself examined by a doctor to determine the cause of your symptoms.Wish you all the best.Regards,Dr. Jonas SundarakumarConsultant Psychiatrist"
},
{
"id": 140454,
"tgt": "What are the side effects of blood clot in the brain?",
"src": "Patient: My name is Shoib Hasan, and i am journalist in delhi. my father met with an accident about 25 days back, and had a small clot is his left side of brain, that problem is almos sorted out and he is physically fit now, however, their is some problem in his behaviour, he forgets things and is unable to sleep properly, he also halucinates, consulting a nurologist hwoever, i am not sure of his recovery please give me some advise Doctor: Hello, Symptoms you describe are possible after traumatic brain injury. These symptoms improve with time and in the first phase after the injury (a couple of months) symptomatic treatment (antipsychotics, antidepressants) may be indicated. Hope I have answered your query. Let me know if I can assist you further. Regards, Dr. Erion Spaho, Neurologist, Surgical"
},
{
"id": 52864,
"tgt": "Is it normal for Lipase to increase after undergoing surgery for pancreatitis?",
"src": "Patient: 47 year old son had surgery for pancreatitis. Lipase was 25,000.Months after surgery and removal of gall bladder, his lipase elevated to 500 and he is in pain. Doctor has review this and cannot find out why? This has happen several time since his surgery. Thoughts?He eats a great deal of baked fish. Elevated-lipase causes food poisoning.????? Thank you Doctor: Hi and welcome to Healthcaremagic. Thank you for your query. I am Dr. Rommstein, I understand your concerns and I will try to help you as much as I can.There are few possible causes and most common is postoperative pseudocyst formation or chronic pancreatitis. In most cases it can be regulated by medications which include insuline supplements and enzymes for digestion Also alcohol is forbidden in these cases and you should eat less fatty food, less sugar and more vegetables and boiled food. Also, avoid citotoxic medications. If there is no improvement then surgical therapy may be considered and include drainage procedures or pancreas resection. This is last option and you should consult your doctor about this.I hope I have answered you query. If you have any further questions you can contact us in every time.Kindly regards. Wish you a good health."
},
{
"id": 164790,
"tgt": "Suggest treatment for constipation",
"src": "Patient: My daughter just turned 2. A couple of weeks ago she did a very hard thick Poo and it hurt her doing it she cried. She then had some blood on it. I thought she just cut a little bit on be way out. The blood in stool lasted about 3 days. Then went away then this morning she started crying and she did a soft poo and visible red blood through it. She does bout 2/ 3 poos a day. Is she sick? Please give me some answers ! Doctor: Hello and welcome to healthcare magic.Blood with hard stools is often due to anal fissure (a cut in superficial part of anus) or rupture haemorrhide (blood vessels in superficial part of anus) both of which can be due to straining while passing stools later being uncommon.My advice is to take her to a paediatric surgeon who will thoroughly examine her anus+rectum and will give their expert advice.Meanwhile give her a lot of fluids,ispaghula husk.Make her sit in warm water for 10 mins twice a day.Hope you find this answer satisfactory.Good luck."
},
{
"id": 108275,
"tgt": "What causes bladder infection feelings, back pain, coldness, abdomen pain, stomach bloating?",
"src": "Patient: I am having some unusual problems. It starts off with the feeling of a bladder infection. Then I get a sharp pain in my left back. While the feeling of a bladder infection coming on I continously keep a cold. Then the pain leads to my left abdomen. My stomach then swells to the point I look about 6 months pregnant. The stomach remains this way for a period of time. I have been to so many doctors and they cannot figure this out. Have you ever heard of this? Doctor: Hi,from history it seems that you might be having chronic urinary tract infection giving rise back pain, abdominal pain and chills giving feeling of cold.Go for routine and culture of urine check up.After report go for long specific antibiotic medicine course.Feeling of stomach swelling might be due to deposition of fat in abdomen giving swelling and fullness.Take plenty of water.Ok and take care."
},
{
"id": 149767,
"tgt": "Episodes of unconsciousness, shivering, stomach pain at regular intervals. What is this ?",
"src": "Patient: Hi, I am a 23 year old female. I have had a few episodes of unconsciousness over the past few years. The first one was when I was 19, I fell unconscious for about a minute or two and also \"shivered\" during that time. Urine passed in that time too. I was told to have an EEG, which showed normal results.The second episode happened recently, about a week ago. I experienced a severe pain in my stomach, (which felt like gases) at regular intervals of time (about every 45 seconds) and suddenly I felt woozy, extremely nauseous and black spots appeared in front of me and I fainted. This time, stool was passed during the \"unconscious\" period. It lasted for about a minute and a half. I've asked a doctor, and he thinks its epilepsy. Please guide. Doctor: Hello,Thanks for using Healthcaremagic. Your clinical signs shows that you have Temporal lobe Epilepsy. Urinary or Fecal incontinence (urine or stool passed) in seizure is common. In some Temporal epilepsy patients, there is association of Epilepsy with Gastrointestinal Autonomic symptoms.Autonomic symptoms are usually nausea, vomiting, borborgymi (fell like gas in stomach), belching. Please repeat EEG once again and consult neurologist.Take care"
},
{
"id": 176067,
"tgt": "Does a bulging belly button indicate hernia?",
"src": "Patient: My son (28 mo) was seen Wednesday of last week because he was wheezing from a cough he s had for 3 weeks. I was told he was wheezing and had bronchitis and needed to be treated with albuteral. He s been taking the albuteral since last Wednesday and he s not better. During our visit I told the doctor that his tummy hurt- he complains everyday that it hurts. She didn t check it so I finally did yesterday because he wa crying and I think he has a hernia. The area next to his belly button is bulging and he points to it saying it hurts. Can I wait until the morning or should I take him to the ER? Thanks in advance! Doctor: welcome to HCM. As you son sufferred wheeze recently, it could be associated with cough, there is likely chances that he would have developed hernia due to cough (Increased abdominal pressure ). Just check out weather the swelling bulges out or increases in size on coughing...as your child is complaining of pain and swelling its better you visit Er as early as possible. only by physical examination one can tell its hernia or not as the location is near to belley button."
},
{
"id": 186911,
"tgt": "Can tiny grayish black painful spot on mouth be mold infection?",
"src": "Patient: Hi, my mom has a painful spot in her mouth that s been there for over a week. When I examined the spot more closely (near the back of the mouth, very hard to see), i noticed a tiny grayish black mark. Is it a mold infection, or something else? It hurts her a lot. Doctor: Hi,Thanks for posting the query, I would suggest you get her checkup done this could be due to either bacterial or viral infection. Are there any other symptoms associated?Are there lesions on other sites on her body.Take lukewarm saline rinses, antiseptic mouthwash gargles.Take care!"
},
{
"id": 191777,
"tgt": "Suggest treatment for swollen liver and infected kidneys",
"src": "Patient: Hello Doctor,My mother is suffering with high diabetes and now after few tests we got to know that she has a patches of TB in the lungs, liver is swollen and kidney is also impacted with the infection.Can you please help with the some diet or medication as she is hospitalized Doctor: Hello dearI have gone through your query and understood your concern.A healthy immune system often successfully fights TB bacteria, but your body can't mount an effective defense if your resistance is low.Diabetes ,especially uncontrolled diabetes can weaken the immune system.Thi is the reason why your mother's immune system could not prevent her from becoming sick. He is hospitalised and with her actual condition i'm sure that her doctors gave her insulin therapy.This therapy is absolutely necessary for her in this moment.About the diabetes diet my advice for your mother is to eat healthy foods in moderate amounts and sticking to regular mealtimes.This helps her body better use the insulin it produces or gets through a medicationA diabetes diet is a healthy-eating plan that's naturally rich in nutrients and low in fat and calories. Key elements are fruits, vegetables and whole grains and avoid procesed foods and saturated fats.Anyway this advices are for the time when she will return back home because now she is in hospital and i'm sure that she have the right diet and treatement for the moment.Hope this information is helpful for you.If you have other questions,feel free to ask.I will be happy to help.Take care."
},
{
"id": 103945,
"tgt": "Have asthma, shoulder blades pain, throbbing sensation below ear, nausea, extremely pain. What is going on?",
"src": "Patient: 48 yo female, no kids, mild GED, well-controlled Asthma, non-smoker. Female woke up (4) days ago with severe mid chest pain, radiating into right shoulder, mid shoulder blades/ upper back, right neck CA & into the right ear. (throbbing sensation just below right ear) Pain has been constant & (in)creases with deep breathing, or lying down. Greatly (in)creased pain when trying to rest on her right side though the pain remains persistent laying on her left side too. Holding her breath (de)creases the pain somewhat but it does not go away. Nausea is present with some mild loss of appetite. General fatigue has set in with no will to thrive. (mostly due to pain while breathing/ movement) Female's father died of a stroke at age 52 but he smoked heavily. Female's Mat. Grandmother died from Lung Cancer in her 70's but she was also a chronic smoker. No sweating, clammy skin, or shortness of breath while at rest as of yet. Only SOB with prolonged activity. (It feels like a thousand sharp knifes cutting into the wind pipe every time a breath is taken in with radiating pain into right chest, right shoulder, neck & lower right ear. It's like I can feel the air going through the vessels due to the extreme pain, easy to follow the line of the pain? Chest pain radiates across mid chest like a fan opening with each breath. No pain in Left Chest at all unless laying on left side. Doctor: Hello, Chest pain can be caused by various reasons, including minor problem such as indigestion or stress, to serious medical emergencies such as a heart attack. Investigations like an Electrocardiogram or Stress Echocardiography can be done to rule out any cardiac involvement. And a Chest X ray will rule out any pathology in the respiratory system. Other causes of chest pain include: 1. Strained chest muscles due to overuse 2. Chest muscle injury 3. Short-term, sudden anxiety with rapid breathing 4. Peptic ulcer disease 5. Pain from the digestive tract, such as esophageal reflux So, it will be better to consult a Doctor & get a physical examination done to rule out other causes. So that proper management steps could be taken. Wishing a Good Health. Thanks & take care."
},
{
"id": 75527,
"tgt": "Suggest treatment for cough with heavy phlegm",
"src": "Patient: okay my name is dylan i have bin getting really heavy phlegm for the past 2 days i am a smoker and i was just wondering if there is anything i should be concerned about . its even worse when i eat sumthin i am constantly spiting this discusting matter out of my throat HELP!! Doctor: Thanks for your question on Healthcare Magic. I can understand your concern. Since you are active smoker, you are at increased risk of lung diseases like lung infection (pneumonia), bronchitis etc. And both these can cause cough with excessive phlegm. So better to consult pulmonologist and get done clinical examination of respiratory system, chest x ray and PFT (Pulmonary Function Test). Chest x ray is needed to rule out lung infection. PFT is needed to rule out bronchitis. You may need antibiotics, inhaled bronchodilators and inhaled corticosteroid (ICS) on the basis of these reports. In any case, smoking cessation as soon as possible is needed. Don't worry, you will be alright but please quit smoking as early as possible. Hope I have solved your query. I will be happy to help you further. Wish you good health. Thanks."
},
{
"id": 193841,
"tgt": "Suggest treatment for the cut on the underside of the penis while shaving",
"src": "Patient: Hello,Sorry for the awkwardness of the situation but I was trimming my hair near my genitals and I slipped and cut the underside of my penis shaft. there is a 1/8 in cut that is slowly leaking blood. Would applying antiseptic cream be a recommended course of action? what do you suggest? Doctor: Hello, Please specify the antiseptic cream you are applying, because over the chemist counter cream buying could be dangerous for you. Please consult a general surgery specialist for the same. Hope I have answered your query. Let me know if I can assist you further. Take care Regards, Dr SHOBHIT KUMAR PRASAD, Psychiatrist"
},
{
"id": 13024,
"tgt": "What causes itchy rashes on my thigh and groin area?",
"src": "Patient: Ive been to the er twice, because I can't afford a doctor. I've had this really itchy rash on my right leg upper thigh and on my groin area. Now my urine has a funny smell, and the rash is spreading. The last doctor said it is a fungal infection and gave me cream. Any answers for me, I don't know if I believe the er here? Doctor: Hi,It may be fungal infection ..also called tines cruris. It is caused by dermatophyte. It may be exacerbated by perspiration,soap and wetness. Avoid excessive bathing. Avoid soap contact. Keep the area dry. You may take antifungal like itroconazole 100 mg twice a day along with antihistaminics like cetirizine.Apply antifungal cream like sertsconazole cream. Take bath in dettol mixed water. Consult the dermatologist.Hope this helps.Dr.Ilyas Patel,Dermatologist"
},
{
"id": 222093,
"tgt": "Could taking Zanocin cause any side effect during pregnancy?",
"src": "Patient: Hi, I am 29 years old and I took Zanocin 200mg for 4 days and half (9 tablets). Before this I did a pregnancy test and it was negative. I also went to a doctor and told me nothing about a pregnancy. After 4 days, while I was taking Zannocin I repeted the pregnancy test. And surprise: Pozitive. I went again to another doctor and he told me that I am pregnant in 5 weeks. My question: Could Zanocin affect my baby? Thank you very much . Doctor: HI, I understand your concern. You had taken Zanocan .. a short course.while you were in early weeks of pregnancy. No human tests are available for effect of Zanocin on fetus. ..In animal studies a ten fold doze showed miner effects of skeletal development of fetuses. I feel you should stop taking & avoid further use of medicine & ask for a safer substitute to your doctor. & Follow fetal development with USG scanning as advised by embryologist. Thanks."
},
{
"id": 187914,
"tgt": "How to cure lump on jawline?",
"src": "Patient: Having treatment for an infected broken tooth before extraction - terrible pain and jawline painful and now a lump on the right hand side on jawline has developed - can you explain why please and is this normal? By the way the antibiotics don t seem to be working too well as been taking them since Wednesday - thankyou Doctor: hi,thanks for your query. the condition you have been complaining might be abscess causing a lump or it might be a cyst. get an radiograph done in that region.by this the treatment can be done.visit oral and maxillofacial surgeon for this.hope this is helpful."
},
{
"id": 160608,
"tgt": "Suggest medication for loose motions & sleeplessness",
"src": "Patient: hi my baby is six months old .he has loose motions for lst two days.although he is active and doin normal things.but he has loose motions for 4 to 5 times a day.and also he is having sleeplesness problem.he gets up after 10 minutes after sleep.plz advice. Doctor: Hello, You may try streptoquin syrup or flagyl syrup in the first place, and as the stool become harder, you need to do stool analysis. Hope I have answered your query. Let me know if I can assist you further. Take care Regards, Dr. Salah Saad Shoman"
},
{
"id": 207841,
"tgt": "What can cause extreme mood swings?",
"src": "Patient: my mood goes from low to high to low again in a single day. do i have a bipolar? I can be really happy and full of excitment and i can get these great ideas in my head then my mood changes and all the happy things i was thinking before dont seem possable and dont interest me anymore. Doctor: HiThanks for using healthcare magicMood swing is part of bipolar disorder, but on basis on only mood swing, psychiatrist can not diagnose bipolar. If you have another symptoms like sleep disturbance, increased psychomotor activity, grandiosity etc, than you can have bipolar disorder. Better to consult a psychiatrist and get your mental status done. In case, you need further help, you can ask.Thanks"
},
{
"id": 3677,
"tgt": "Is it possible to get pregnant while taking Tetralysal?",
"src": "Patient: is it possible to get pregnant while taking tetralysal? I have the coil but my doctor told me it would not work with this acne medicine for three weeks. I have two days to go before completeing the three weeks. Could I get pregnant if I and my husband have done something? Doctor: Hello, and I hope I can help you today.If you have the coil, there is no reason that it would not be effective while taking an antibiotic. The interaction between antibiotics and birth control refer to oral contraceptives only (pills). The hormonal coil only delivers hormones to your reproductive organs, so it would not interact at all with any pills you are taking.So in summary, you are still protected from pregnancy with the coil, so you do not have to worry about any additional protection from pregnancy.I hope this information was helpful and best wishes, Dr. Brown"
},
{
"id": 108615,
"tgt": "What causes lower back pain going to the right ribs?",
"src": "Patient: Hi, I am suffering from lower back pain going to my lower right ribs. I've undergone blood test, urinary test, whole abdominal ultrasound and even kidney xray but found nothing on it. Also lately i feel thirsty all the time. Is there any other kind of test that I need to undergo in order to determine the cause of this?. Doctor: Hello, I have studied your case. Due to compression of this nerve root there is pain in your ribThere is another possibility of costochondritisI will advise you to do MRI thoracic spine, HRCT [CHEST] and EMG- NCV [nerve conduction study]For these symptoms analgesic and neurotropic medication can be started.Till time, avoid lifting weights, Sit with support to back. You can consult physiotherapist for help.Physiotherapy like ultrasound and interferential therapy will give quick relief.I will advise to check your vit B12 and vit D3 level.Hope this answers your query. If you have additional questions or follow up queries then please do not hesitate in writing to us. I will be happy to answer your queries. Wishing you good health.Take care"
},
{
"id": 206604,
"tgt": "Suggest treatment for AHAD",
"src": "Patient: Good Morning:Let me start off by saying that I have ADHD and I've been taking a combination of Adderall and Vyvanse for a few years now and it has done wonders with improving my focus and concentration at work and at home. I've had the same doctor for the length of time mentioned above; but I feel it's time to change physicians for a bunch of reasons. First off, it's a 2 hour transit time, there and back; so that in itself is a huge burden. Secondly, her office hours are during my work hours; so I always have to take time from work in order to see her. Lastly, she tends to forget things such as what scripts are due on which days: and required details on the scripts like the quantity and her signature. By doing this, I will have to return to her office to get a new script; which costs me time and money that I do not have.Furthermore, I get the feeling like I'm inconveniencing her when I have to contact her about these situations. It wasn't like this in the beginning; which is why I endured the transit times and her office hours. Prior to taking Adderall and Vyvanse, I did try other medications; but nothing worked. I don't want to go through a song and a dance when seeing a new doctor; so I guess my question would be how I should I approach the new doctor about taking Adderall and Vyvanse: and how I should go about screening doctors in my search. Doctor: DearWe understand your concernsI went through your details. I suggest you not to worry much. Many researches and researchers confirm that medicines alone cannot cure mental disorders. Life style changes, change in thinking pattern, relaxation etc are as essential as medicines. Psychotherapy can help you changing your lifestyle and thinking patterns. Yoga and meditation help you to streamline your metabolism and neurological balance. Please consult a psychologist for further information. It is also important that you work closely with your psychiatrist. Do trust him fully. If you do not trust the current psychiatrist, find a psychiatrist whom you can trust wholly. That trust makes cure easier and fuller.Psychotherapy techniques should suit your requirement. If you require more of my help in this aspect, Please post a direct question to me in this URL. http://goo.gl/aYW2pR. Make sure that you include every minute details possible. I shall prescribe the needed psychotherapy techniques.Hope this answers your query. Available for further clarifications.Good luck."
},
{
"id": 53812,
"tgt": "What causes low level of aspartate transaminase in blood?",
"src": "Patient: to be specific my blood test showed critically low level of aspartate transaminase level is 4iu/l ref range 16-56. my Alanine range is 13 (10-75). I go to the VA and am inbetween appts at the moment just curious as to whether i should bother my doctor about this or what it could possibly mean that i can correct. Doctor: Hi welcome to health care magic You have not mentioned your complaints If liver enzymes high than it suggest hepatocytes injury Here for low liver enzymes no need for much worry Better to do other work up regarding your complaints if having Take care Hope this will help"
},
{
"id": 212017,
"tgt": "Have memory issues. Fail to recollect incidents. When tested, came out age related normal. Treatment?",
"src": "Patient: Hi Dr. Grief,I am a healthy 66 yr old female with a happy marriage , 3 children , and 7 grandchildren. I'm retired. I have been having loss of memory issues for about (?) a year. Nothing horrible like getting lost, forgetting to take my Rxs , or how to get through my day. My recall of incidents is lacking strength. My husband forgets nothing which makes my lousy memory seem even more exaggerated and exasperating. I have had various degrees of anxiety since my early 20's\". I get in an \"overload\" status quite easily depending on my environment at that time (does that make sense to you?) A houseful of my kids, their spouses and my grandchildren does not overwhelm me. But what is upsetting is constant anxiety over my husband's Hep C contracted from blood transfusions due to a spontaneous ruptured esophagus occurring @17 yrs.ago although his only symptom is he tires easily; he has had several liver biopsies which indicate no cancer - he has been on 3 treatment plans, the first 2 lowered his liver viral load count to 0 but he is a \"reactor, not a sustainer\"; shortly after going off 2 treatments the viral load goes up again; the 3rd & last treatment was intolerable and he had to come off it. Now he has a cyst on one kidney discovered during a cat scan and they're pretty sure it's cancerous and will be operated on quite soon. No sign of cancer spreading as far as they can tell now. I feel like I can't be strong for him anymore and I'm always stressed inside. When someone (spouse or kids) say things like \"remember when we did such & such or why can't you remember we had a whole conversation about this or that. ?\" That's when I feel like something is wrong in my head; and they do not mind sharing their disbelief that I've forgotten whatever it is. \"You need to see someone about your memory\" is an often response. They don't beat me up about it and are well-meaning because they care about me, but I can sense that they are all getting more concerned. I went to a neurologist and everything came out age-related normal and that I should get cognitive remediation therapy. I do not have any dementia or any sign of early onset Alzheimer's. I've been searching the internet to find a therapist in my area (Frederick or Montgomery County Maryland) who deals with C.R.T. /memory function and anxiety issues as stated above. Would you be able to give me some direction on what I should be looking for in a professional mental health person? I prefer a female. Thank you and I'm sorry this is so lengthy. Barbara K. of Frederick Md. Doctor: Hi thanks for asking to HCM, the friend you talking about, has a sign depression. Need consultation from psychologist. don't be solely depend upon medicine. Give him support, do join a sport or exercise. Let him do his hobbies and give him challange in between. Take care. Soon things will be fine."
},
{
"id": 81805,
"tgt": "What is the treatmnet for pulmonary fibrosis?",
"src": "Patient: Hi my daughter has pulmonary fibrosis - she gets bad coughing fits - two to three times in 24 hours (other than mitts linctud codeine) nothing seems to work - she has recently developed shunting bet the Pulm artery and paul vein - she is 18yrs old - would it be advisable for her to fly? How do we find out living in India - she is on 2L/min O2 at all times - she used to use portable machines but now that doesn t work as her heart rate rises when oxygen is pulsed rather than continuous flow - she walks in the house but uses a wheelchair if she leaves home and also has raised BP after her coughing fit (144/106) Doctor: Sir, Its really rare to see pulmonary fibrosis with such a young age. She is really a brave girl to go through all this.To fly she will have to carry oxygen along however due to the decrease in the cabin pressures at heights, it is not advisable to fly without any risk.You can give her codeine 2-3 times daily however steroids have been shown to have good relief for the cough, but doubtful to slow down the progression of IPF.I hope she is on the medicines required, like steroids, azathioprine or pirfenidone.NOTE: Thalidomide is known to reduce cough in patients steroids dont help"
},
{
"id": 200356,
"tgt": "Is exercising advisable after masturbation?",
"src": "Patient: Is there any problem doing exercise in gym after mastrrbation. After my job I come home and masterbate daily and I am not able to control masterbation.and after that I will go to gym...I am a well built 25 yr old man. Once my friend told me that doing exercise after masterbation is not good for my health. Nowadays I started feeling that I am loosing my body power. Automatically before bathing I will masterbate and I am not able to control that .I masterbate 2 times a day..morning and evening...any problem .....??? Doctor: Hello dear,Thank you for your contact to health care magic.I read and understand your concern. I am Dr Arun Tank answering your concern.No, there is no problem in masturbation.There are false belief in people regarding masturbation. It will never cause any harm to you. Whether you do it single or twice a day.It has no effect on energy level or your performabce level.It is very common in today's life of doing masturbstion. If you don't do than you will get sweet dreams.Please do wash your hand before you masturbation and avoid injury for masturbation.I will be happy to answer your further concern on bit.ly/DrArun.Thank you,Dr Arun TankInfectious diseases specialist,HCM"
},
{
"id": 83720,
"tgt": "What causes bleeding through anus while taking fat burning medication?",
"src": "Patient: Hi there I recently started taking fat burners and at the end of the course when going to the toilet and wiping my bottom there was just pure blood there but no blood actually in my stool. But when not taking them for a couple of months have been absolutley fine I started taking them again yesterday and my first time going to the toilet exactly the same thing again nothing actually in my stool but just pure blood when wiping Doctor: HiThe name of the drug has not been mentioned.Drugs that increase the risk of constipation can cause bleeding.Since it is fresh blood it should be arising from lower gastrointestinal tract.If constipation is the cause,laxatives can be used and the symptoms can be checked.In the absence of constipation,it is better to avoid the drug.Hope I have answered your query. Let me know if I can assist you further. RegardsDr.Saranya Ramadoss, General and Family Physician"
},
{
"id": 86359,
"tgt": "What causes severe upper abdominal pain after total knee replacement surgery?",
"src": "Patient: Hi, I had total knee replacement a month ago and that is doing well. But since surgery I have had pain and discomfort in my upper abdomen particularly on the left side to just above the navel. It only hurts when I cough or sneeze or bend,etc. my doctor thought it could be from therapy, but I don t understand why it hurts when I cough, etc. my skin in that area is also sensitive. Doctor: Hi welcome to HCmI have gone thru your query regarding severe upper abdominal pain after total knee replacement surgery . It is a matter of great concern .Dear , I am sure , your pain has no connection with your knee surgery . Upper left abdominal pain is most often due to a condition afflicting one or more organs or structure in the LUQ (left upper quadrant) of the abdomen. It may also be due to conditions in the thoracic cavity or lower left abdominal area that either refers or radiates to the LUQ. Constipation ,Pancreatitis , enlarged SPLEEN , stone in kidney or ureter , conditions that may cause left side abdominal pain . Any of the above conditions may be cause of your present agony The severe pain Ultimately a diagnosis by a medical professional is necessary so that the appropriate treatment can be commenced as soon as possible. Your doctor may suggest abdominal ultrasound, x-ray, CT scan or MRI are often necessary for a definitive diagnosis .Meanwhile , Rest until you are feeling better.Drink plenty of fluids to prevent dehydration. You may find that taking small, frequent sips of a beverage is easier on your stomach than trying to drink a whole glass at once. Drink lemon juice in a glass of water with a pinch of salt and black pepper , thrice a day , is very energetic , good appitiser and alley pain very fast .Try eating several small meals instead of 2 or 3 large ones. Eat mild foods, such as rice - moong dal khichi , oats , porriage , crushed an inch of ginger with salt and black pepper .Do not use aspirin or medicines, may irritate your stomach and increase your pain.`avoidingAvoid fried fast foods too much of sugar, tea, coffee ,alcohol , smoking ,stress ,constipation . Pranayam - deep breathing , Vajjarasan , mandookasan , Bhujjangasan , lying on right side can help . If problem persists or worson ,rush to your ASAP .Hope this helps solves your query .Take care , All the best .Don't hesitate to get back if have any further query"
},
{
"id": 220522,
"tgt": "What are the early signs and symptoms of pregnancy?",
"src": "Patient: I had unprotected sex with my boyfriend a week ago, he pulled out at least 5 seconds before he came. We broke up two days ago due to him having to much on his plate to commit to a relationship, but we were talking for a year before we decided to date one another, yet he still really cares about me. My period is due in about a week and I have been having slight stomach cramps, some nausea at random times of the night, I started to have a white discharge yesterday (which i have had before but not anytime recent), I have been moody lately and I have had some slight headaches but thats nothing out of the norm. I have a good amount of stress in my life which has made me late on my period before. What are the possibilities of me being pregnant or am i over thinking this??? PLEASE HELP ASAP! Doctor: Don't be so stressed out.every problem has a solution.you may very well be pregnant.wait for a week or so and then u can get your pregnancy test done if you don't get periods.u may also get your blood test done if u can't wait for so long"
},
{
"id": 65390,
"tgt": "What causes a lump on the mid shoulder blade?",
"src": "Patient: I am 30 years old, 5'5\" and 138 lbs. I'm very healthy and active. No known skin problems in my family tree but I've always had oily skin. I tan very easily but generally put sunscreen on to prevent sun burns. Lately, I have noticed a bump on my back around my mid-shoulder blade. It's not red, itchy, or inflamed. If I squeeze the area a continuous line of thick white stuff (not smelly) comes out. I keep squeezing and squeezing and it keeps coming and coming. I generally relieve this area until the bump is no longer noticeable but then It fills up over time (months I would say) and the same process begins. Any explanations and should I worry? Doctor: Its is a sebaceous cyst. Usually in people with oily skin sebum forming glands in the skin produces lots of sebum which sometimes gets trapped and forms a sebaceous cyst. It is just below the skin and contains white to greyish cheese material. Treatment is excision of the cyst. Next time, don't squeeze it. Let it become prominent and once it is prominent, go to a general surgeon who will excise it completely. Its easier for a surgeon to take out the cyst when it is prominent, so that whole of the cyst is removed because if any remnant is left back, it will recur."
},
{
"id": 123016,
"tgt": "Can posterior annular tear cause bulging disc?",
"src": "Patient: Good day, I have 2 bulging discs (which I have known of for many years) and a posterior annular tear in the disc upwards. This concerns the L5/S1 and the two next discs. What implications does the tear have for my lifestyle? I run frequently and used to be a competetive fencer. Doctor: Hello, As due to muscle weakness and degenerative changes in the muscles of the spinal column there will be an injury to the ligaments and the intervertebral discs. Due to which there will be a pain, inflammation, tingling, etc. Recent research studies suggest that exercises have a positive role in the recovery post disc injury. Exercises like - core stability, spinal muscle strengthening exercises, hip muscle strengthening exercises, static hamstring, static quadriceps, straight leg raise will help improve the muscle strength and regain the stability of the spinal column. Hope I have answered your query. Let me know if I can assist you further. Regards, Jay Indravadan Patel, Physical Therapist or Physiotherapist"
},
{
"id": 118785,
"tgt": "Sweating, nausea, fainting, stomach pain after donating blood. Reason?",
"src": "Patient: I just gave blood. I was fine during and after and stayed at the canteena for a soda. I had to go to the wireless phone store and ended up being an hour on my feet. I began sweating terribly, felt nauseous, and almost fainted. After sitting for a while the dizziness passed but I double over with stomach pain. Can this be related to the blood donation? If so, how come? I have given blood many times and never had such a reaction.Thank you for your insight. Doctor: hi..you have donated your blood.. congratulations..after that you had fainting episode after long standing, it could be due to postural hypotension and pain abdomen due to drinking soda, might be due to gastric irritation..don't worry, you need to take some rest, take lot of fluids, oral rehydration solution... you will be all right.."
},
{
"id": 95171,
"tgt": "What is the remedy for mild abdominal pain due to wearing a tight belt ?",
"src": "Patient: i have mild intestine pain for 3 weeks now. it moves around in my lower abdomen . it started with my tight belt. i sat and moved forward. what is this Doctor: you need to tell ,the character of pain like catching,is it associated with vomiting or bloating any associated symptoms like constipation,diarrhea,burning micturition,redness of urine.any fever ,weight loss or blood in stool. you need to under go some basic blood investigation ,stool test and ultrasound of abdomen."
},
{
"id": 215754,
"tgt": "What causes pain in lower right side of body?",
"src": "Patient: Hi I have been having pain in my lower side, I also have had a metal taste in my mouth, this pain is sharp and it hurts when I get up or turn to the right or left I was wonder is it because I only have one kidney and low white cells my remaining kidney I have is on my right side where the pain is at can someone help me know what this could be I also have protein in my urine ad well but that has been like that for awhile Doctor: Hi, You can go for an ultrasound scan and renal function test to assess the function of kidneys. As of now you can take analgesics like Tramadol for pain relief. Hope I have answered your query. Let me know if I can assist you further."
},
{
"id": 163710,
"tgt": "Suggest treatment for bowel incontinence in kids",
"src": "Patient: Here is my concern. I have a 9yr old daughter that is still pooping in her pants. At first it started out that she refused to let it out because it was hurting to much. Then when I took her to see her doctor a pelvic ultrasound was ordered due to a mass on her left side. Her doctor told me that the mass was inpacted fecal matter. Then she was put on a strict diet, I was told to give her 1 ts of Micylax a day for a week, have her eat more fiber, eat an apple a day, 4 prunes a day and a green everyday. Well, I have done all that and now it has gotten to the point to where she has the runs so bad that she can not control it at all. She has been to see a Pediatric GI doctor, and they say that there is nothing wrong with her bowels. Please, help me, I can not handle throwing away anymore panties due to being stained so bad. What else can I do? Doctor: Hello! Welcome to HealthcareMagic! Your concern is right. But patience is required. In a few days time the child's bowel will learn how to process new kind of food. In the mean time you can tell her to do pelvic exercises, where she tries to hold and release muscles of genital area. She is now years old. She can go to the toilet as soon as she gets a feeling. Not to wait for school. You can discuss with teachers to be lenient for the time being. In the meantime, consult her doctor. Take care Regards, Dr. Ratna Mulay, Pediatrician"
},
{
"id": 48273,
"tgt": "Suggest treatment to remove kidney stones",
"src": "Patient: My husband is having kidney stone in both side 7mm & 6 mm and 1 stone 6.1 mm in upper ureter he is having pain and taking superspas tablet to avoid pain. Doctor said that it will take 2-3 months to clear all stones. kindly suggest some medicines or precautions so that stone can remove more fast. Doctor: HIWell come to HCMI really appreciate your concern, looking to the size of stone these need to be removed with lithotripsy or with ureteroscopy, this would be best option else waiting for the stone to come down in bladder with any other method beside this would be risky, hope this information helps, take care."
},
{
"id": 182028,
"tgt": "Can the amalgams used to fix cavity be repaired?",
"src": "Patient: I had an amalgam filling put into a lower first molar (2 surface) this morning. It broke on my way to work from the dentist. Is it possible the amalgam wasn't mixed properly? This office doesn't usually do amalgams, so maybe they're out of practice, so to speak? If it is replaced, does the entire filling have to be removed or can a mechanical bond be made by undercutting the prep? Doctor: Silver amalgam fillings are an outdated concept probably earlier days with no other alternative available. You must also understand it mechanically interlocks with the tooth unlike modern day composite restorations which bond to the tooth directly. Wise option would be to replace with a composite restoration."
},
{
"id": 205994,
"tgt": "How can unconsciousness be prevented in case of vasovagal syncope?",
"src": "Patient: I am an 18 year old college student with vasovagal syncope and have had three or four cases of fainting in my life. I have not had a case of fainting in a couple of years and have learned a little bit how to prevent vasovagal episodes by drinking Gatorade and tensing my leg muscles and sitting down as much as possible. However, many times I still feel dehydrated and/or dizzy. At my school, we have a 30 hour dancing event to raise money for charity and I would love to do this year. I just want to know if you think I would be able to get through 30 hours of dancing. There are some breaks and there is most likely plenty of water, however, I am afraid that I will have a vasovagal episode at this event. What is your advice? Doctor: dear,here are certain things that you can do to prevent unconsciousness-1) wear calf length tight stockings or high length socks during longstanding activities.2) carry a bottle of glucose with electrolyte drink with you, hav eit in case you start feeling dizzy.3) take breaks from physical activities at regular and scheduled time.4) avoid high altitude adventure or places.5) carry an identity card always with you.hope these are some lifestyle modification can help you. Besides, keep a regular consultation at your family doctor.thanks"
},
{
"id": 170429,
"tgt": "What causes fever with cold hands and feet?",
"src": "Patient: HI, My 4 year old daughter has had a fever of 103 for two days but she is shaking and her hands and feet are cold? She tells me she is cold and yet her temperature says she is running a fever I m confused. I m not sure if i should take her to the ER or not since i have been giving her medicine for the fever? Doctor: Hi, temperature at hands and feet is not reliable because these are exposed parts and due to cold air or washing hands can lower the temperature. Temperature at arm pit or mouth are reliable. Since, child has fever for two days, he needs to be investigated and tests like cbc, malarial antigen test should be done. Further treatment will depend on above reports. I hope this will help you. Take care."
},
{
"id": 126076,
"tgt": "Is bradycardia with abnormal leftward axis a concern before a knee replacement surgery?",
"src": "Patient: I am a 59 year old male I am having my knees replaced starting with my left next week. I went for my pre physical and my first ekg my results say i have a brady cardial leftward axis abnormal, but the dr said there is nothing to stop me from having the surgery I am wondering what this means I dont smoke and have lost close to 100 lbs in the last three years but i still weigh 305 Doctor: Hello and Welcome to \u2018Ask A Doctor\u2019 service. I have reviewed your query and here is my advice. It is not a serious issue. You may need proper cardiac evaluation.Hope I have answered your query. Let me know if I can assist you further."
},
{
"id": 144696,
"tgt": "What causes pain in upper left back?",
"src": "Patient: I ve been told I have Epstein barr virus (again, had it 20 yrs ago too) and that my numbers are off the charts (to quote dr!). My concern is I am having a sharp dull pain in my upper left back that will not go away. Have had back issues so can t tell if my spleen or not? Is it safe to just wait? Dr ordered a spleen ultrasound, but don t even have the prescription to book apt yet. Also having very generalized pain to touch everywhere around the area. Doctor: You could be having a muscle knot. You can get a steroid shot in that knot for relief. Consult your Pain Medicine Specialist."
},
{
"id": 166777,
"tgt": "What causes visible veins on skin in an infant when suffering from fever?",
"src": "Patient: I noticed my 10 month old s skin (weight is 18 lbs) has a splotchy veiny appearance. She seems to have a slight fever as well. We were at the physician s office today because she woke up with a croupy cough and she had a fever of 100.4 in the morning. The physician diagnosed an upper respiratory infection. Her skin didn t have this appearance until just now. Doctor: Hi,I understand your concern. By what you say I feel that you are a child is having cutis marmoratus. This is the skin condition where because of difference between the body temperature and the ambient temperature there will be blotched appearance of the skin with prominent veins. This is a self limiting condition and you need not worry about it. But if the child is extremely lethargic, please take her to the nearest Emergency room.Hope I have answered your query. Let me know if I can assist you further. Regards,Dr. Sumanth"
},
{
"id": 72833,
"tgt": "What causes shaky hands and tightness in the chest?",
"src": "Patient: My hands shake sometimes, and my chest is tight sometimes, i have to breathe in and out of my nose and mouth for the tightness to go, and sometimes the tightness stays for a little bit longer after breathing in and out, when i have a cold i cough for more than 2 weeks Doctor: Thanks for your question on Healthcare Magic.I can understand your concern. Undiagnosed stress and anxiety can cause chest tightness and tremors (shaking) in hands.So better to consult psychiatrist and get done counselling sessions. Try to identify stressor in your life and start working on its solution. You may need anxiolytic drugs too. Don't worry, you will be alright. Avoid stress and tension, be relax and calm. Hope I have solved your query. I will be happy to help you further. Wish you good health. Thanks."
},
{
"id": 131418,
"tgt": "What causes leg pain?",
"src": "Patient: Hi I am a 45 years old female from India.Since sometime I have been experiencing intense pain when I try to drive .The motion of the foot at the accelerator causes this pain which goes up to the calf and the muscle gets painfully tight.I have also have pain on my upper foot sometimes a slightly burning sensation .when walking too I get this discomfort and it feels better at times when I flex my foot upwards.It happens in both the feet but started with the right....any help? Doctor: This mostly symptoms of hypocalcemia ,so you should take mixture of vit.D and calcium tablets once daily for 6 weeks and take milk, cheese and meat daily."
},
{
"id": 107613,
"tgt": "What causes upper back discomfort?",
"src": "Patient: II have been bothered by upper back discomfort for approx 4 months ...2 years ago I had a Prostetectomy (Retropubic ) Prior to my surgery a bone scane / r/o mets ; MRI and all the other tests involved will this disease ... Psa was 4.5 ; After Surgery and for the past 2 years ( Non detectable ) .Tested every 6 months ... I had a gleason score of 6 , staging of t1c ,clean margins and was incapsulated ... Had lympth node dissection --- All was clean ...... Now I m thinking maybe Metasisis ---- Your thoughts --- (last PSA was in Oct / 2010 (undetectible ) ....... thanks, Dave ) Doctor: Dear patient You have not mentioned your age and occupation. Since prostate cancer usually affects elderly you must be around 60 years of age. at your age there are multiple causes of back pain and common is osteoporosis and degenerative spine disease. But possibility of metastasis should also needs to be ruled out. you need to be investigated with 1. Xray of dorsolumbar spine anteroposterior and lateral views 2. MRI of dorsolumbar spine to confirm diagnosis. You need to visit radiology center nearby you and get it done. you need to consult expert oncologist if report shows positive metastasis. all the best."
},
{
"id": 170957,
"tgt": "Suggest weight gain food for infants",
"src": "Patient: I recently visited a friend who has a 1 month old baby. He has not gained even 1 ounce of weight and he cannot keep any formula down. They have resorted to giving him mashed potatoes and gravy through a bottle but he constantly throws up. What can I do? Doctor: HiIt's not advisable to give anything apart from breastfeeding or formula feeds at this age. So please stop your friends from following such measures. There can be multiple reasons for poor weight gain and vomiting episodes. The most common reasons are improper feeds, hypertrophic pyloris stenosis, electrolyte disturbances, inborn errors of metabolism etc. So, kindly advise them for a complete check up be neonatologist and investigations as per examination.Hopefully this will help.Take care"
},
{
"id": 56509,
"tgt": "Treatment for mild hepatomegaly with grade 3 fatty infiltration",
"src": "Patient: Doctor sir my ultrasound report gives opinion that i haveMILD HEPATOMEGALY WITH GRADE 3FATTY INFILTRATION andENLARGED PERI PANCREATIC LYMPH NODE my blood sugar fasting is 156 SGPT (ALT) serum is 136 andSGPT( AST)serum is 92 kindly advise me its cure dos and donts my age is 35 yrs weightc86 kg waiting for ur response Thank u Doctor: HelloIncreased SGPT/ALT indicate liver injury.It may be due to many reasons like hepatitis,alcohol intake,altered lipid profile,medicines,auto immune causes etc.It may be related to fatty liver in your case.Excess fat damages liver cells.SGOT is non specific and it increases in many conditions.You may need few more investigations like routine hemogram,lipid profile,viral markers.I suggest tablet ursodeoxycholic acid 300 mg twice daily for three months to my patients.It helps in regeneration of liver cells.Fatty liver is a reversible condition.You need to change your diet and life style.You should avoid fatty food and go for brisk walk.Your fasting blood sugar is also raised.You may need to take oral anti-diabetic medicines like tablet metformin.You may need contrast CT scan of abdomen as peripancreatic lymph nodes are enlarged.Get well soon.Take CareDr.Indu Bhushan"
},
{
"id": 7608,
"tgt": "What is the permanent cure for recurring pimples ?",
"src": "Patient: I having pimples on my face from the past 6 years.I had used many lotions but i didn t get any releif from them.sometimes they are reducing and sometimes coming again.pls suggest me some permenent cure for it or any medicine? Doctor: Hi..dear Himabindu.., Thanks for choosing HCM., Pimples on face is common in this age.., it is due to hyperacivity of hormones.., So don't worry...go for good treatment to avoid.., further effects...ok.., 1) Frequent washes with Fash Bar.., 2) Tab..Isotretinoin 20 mg for 30 days.., 3) Clindamycin and Nicotinic acid combination.., cream 2 times daily till get good result.., 4) Take plenty of green leafy vegetables ...and fresh fruits..,ok..good luck..,"
},
{
"id": 122672,
"tgt": "What causes right lower quad pain, nausea and vomiting?",
"src": "Patient: I have had right lower quad pain for two weeks. I had an US and it show inflammed appendix. When they did the CT it was negative. The pain is now moving to the back and it comes and goes. I had colonscopy and everything looked great. Help I do not know what to do. WBC has been 15-19 and no temp. Have had nausea and vomit. Doctor: Hello, We have to rule out possible causes like appendicitis or renal stones. As a first line management, you can take analgesics like paracetamol or aceclofenac for pain relief. If symptoms persist better to consult a general surgeon and plan for an ultrasound scan. Hope I have answered your query. Let me know if I can assist you further. Take care Regards, Dr Shinas Hussain, General & Family Physician"
},
{
"id": 37008,
"tgt": "Suggest treatment for dengue fever, typhoid and pneumonia",
"src": "Patient: Hi, my aunt is suffering with dengue fever, typhoid, kidney effect and pneumonia. Please let me know her condition. She is in ICU. Doctors in that hospital giving treatment to dengue alone. Is it enough . she is not in a state to move to big hospital. Please give any suggestions. Thanks & Regards, Bhanuja Doctor: HIThanks for posting your query to Healthcaremagic. Very sorry to hear about your aunt . There is no specific treatment for dengue. Dengue may cause reduction is platelet count and if it goes to a very low level then Platelet transfusion has to be done. Sometimes dengue can cause breathing difficulties by causing fluid accumulation in lungs then treatment will be for that. For Typhoid Antibiotics has to be given and I think if she is in ICU Antibiotics for sure will be given . Just enquire again. Sometimes single antibiotic can cover for both Pneumonia and Typhoid so you might have felt that they are not treating for Typhoid . If really no Antibiotics is being given then she should be started on Antibiotics IMMEDIATELY. Hope this information was useful to you. Any clarifications feel free to ask ."
},
{
"id": 58035,
"tgt": "Can stones in gall bladder cause weakness and nausea?",
"src": "Patient: I have been told I have gall stones and need to have the gall bladder removed. my question is this: can this condition cause a non-specific feeling of weakness and unwellness? I am 76 years old and have just recently started feeling very uneasy and tired at the same time, along with borderline nausea. What do you think? Doctor: yes as the digestive enzyme bile is missing fron your juices there is difficulty in digesting fat and cholestrols causing the problem i may also suggest you to go for removal"
},
{
"id": 142613,
"tgt": "What treatment is suggested for pituatary adenoma and PCOS?",
"src": "Patient: Hello and Happy New Year! I have been diagnosed with a pituitary adenoma (very small, around 3mm), and PCOS back in September. I've been on meds for these and since the meds started I have had no side effects, and my symptoms went away (biggest one being headaches with blurred vision, nausea and vomiting). Beginning of December I got a headache that didn't go away for almost 3 weeks. Also accompanied with blurred/cloudy vision, dizziness, and nausea/vomiting. The headaches have gotten better, but the cloudy vision has stayed. I had an eye exam a few months ago which showed that I had blind spots in my left eye, leading the neuro and eye doc to believe the adenoma was slightly pressing on the optical nerve. Problem is when I went back to my endo and neuro this past week and told them about the headaches, neither seemed to really care. I of course am concerned because I do not want to end up with permanent vision issues. Not to mention the headaches are not pleasant. I am due to deploy in the very near future and don't want to get somewhere and have an issue down the road. Is this something I need to push with my primary care doc and possibly get a new neurologist, or am I worried about probably nothing? Thank you in advance! - Jill Doctor: Hi, Welcome to HealthCareMagic.com I am Dr.J.Mariano Anto Bruno Mascarenhas. I have gone through your query with diligence and would like you to know that I am here to help you.Your condition requires immediate surgery since vision is affected. please consult a Neurosurgeon at once Hope you found the answer helpful.If you need any clarification / have doubts / have additional questions / have follow up questions, then please do not hesitate in asking again. I will be happy to answer your questions. In the future, for continuity of care, I encourage you to contact me directly in HealthCareMagic at http://bit.ly/askdrbruno Best Wishes for Speedy Recovery Let me know if I can assist you further.Take care."
},
{
"id": 144846,
"tgt": "How to treat stenosis and herniation for a better quality of life?",
"src": "Patient: Dr. I had a epidural that led to paralysis of my right leg. A emergency laminectomy and decompression was performed the following day. There is little soft tissue between the L4/L5 and severe stenosis and herniation at the L5/S1. I m only 46 so nuro surgeons will not perform a disc replacement at my age. Use of my leg has returned over 6 moths of physical therapy. I am on percocet and baclofen to control spams and mask pain but neither bare overly effective. What other options may be available to me to return to a somewhat normal life? Doctor: Thank you for asking Healthcare majic. My name is Dr Ehsan Ullah & I have gone through your query. As the re-currence rate of disc prolapse is high,you need to take some general precautions in daily activities to avoid further distress.Well nice to know that your leg power are back so following lifestyle changes will help you: 1.Avoid heavy weight lifting 2.No more bike riding if you were riding before. 3.No stairs climbing 4.Apply lumbar coarset belt for three months.Pain killers,muscle relaxants and Gabapentin 150mg BD can be taken for few weeks and regular follow-up visits to neurosurgeon will be much helpful to you along with above mentioned precautions.Hope this may help you. Let me know if anything not clear. Thanks."
},
{
"id": 85260,
"tgt": "Could usage of medical marijuana cause lose of security clearance?",
"src": "Patient: I had ventricular tachycardia that went undiagnosed for 15 years. They finally caught it on a stress test and did an ablation. Now I am suffering from PVC s and no medicine has been able to help them. They are considering doing another ablation but my wife is afraid to have me go through this again. Two people at work told me that I should consider medical marijuana. I laughed them off but they were insistant I look into it. One, I don t know if it is something to consider and secondly, wouldn t I lose my security clearance?? Doctor: Hello,Medical marijuana is not recommended and it does not have any proven evidence for its efficacy. If the urine sample shows the traces it might lead to failure in security clearance. Consult a cardiologist and he will direct you accordingly.Hope I have answered your query. Let me know if I can assist you further. Regards, Dr. Shinas Hussain, General & Family Physician"
},
{
"id": 96380,
"tgt": "Can any one suggest me what should i do for motion Problem ?",
"src": "Patient: Hi, i had taken bear with chicken and spicy food on 17th Feb. On 19th Feb early morning continuos motions of 15 times with gap of 15 minutes happened. Than i consulted doctor on 19th Feb and explained him the same. The doctor gave me 3 medicines 1) Flagyl 400 - 15 Tablets (3 Times a day for 5 days) 2) Histac 150 - 10 Tablets (2 Times a day for 5 days) 3) Oflomac 200 - 6 Tablets (2 Times a day for 3 days) I have stared using the medicine from 19th Feb. Today is 21st Feb, But still after eating with in 20 minutes - 30 minutes i am getting motion. Can any one suggest me what should i do? - Thanks Doctor: cntnue the medicines and do not worry you are gonna be alright, i had similar situations last fall. it adjusts in couple of more days. if not then see the same doctor. these medicines are good i suppose."
},
{
"id": 215236,
"tgt": "How to get rid of the painful wound?",
"src": "Patient: Samuel Jin (14 Years Old... babyish though) I just went out on my bike, and my tires slipped and I got a 6 inch by 3 inch wound with concrete studs on it. I washed it (Bravely...Proudly too...mostly of myself) and applied painful iodine. I m going to bed and don t want it pussing plz answer Doctor: Hello,As of now you can use analgesics / anti inflammatory combination like aceclofenac / seratiopeptdase for symptomatic relief. There is risk of infection and you have to apply topical antibiotics like mupirocin after cleaning with soap and water. Oral antibiotics may also be required preferably amoxclav.Hope I have answered your query. Let me know if I can assist you further. Regards, Dr. Shinas Hussain, General & Family Physician"
},
{
"id": 171700,
"tgt": "Suggest remedy for frequent bowel movements",
"src": "Patient: My daughter is almost 2 several months ago she started having very frequent bowel movements. Like 30 a day. but they are incomplete bowel movements, more like a fart and a little poop comes out that it.. she has been diagnosed with strep on the anus although her bottom seems to bl clearing up its not helping the frequent bowel movements. she cries and has a straining stance with her legs locked stiff and really looks like it hurts coming out i dont know what it could be or how to help her Doctor: Hi,Welcome to HCM forum,I am sorry to hear about your child's condition. I am sure it must be really painful for her. The frequent bowel habits seems to be due to the pain she has in the anal region. You need to take here to a pediatric surgeon for a complete evaluation of her anal region and if needed a scopy study too.Hope I answered you query adequately. Let me know if there is any further clarification you need. Take Care."
},
{
"id": 36344,
"tgt": "How to treat the blood blister caused by a dog nip?",
"src": "Patient: I was nipped by a dog almost 2 months ago. It bruised and I think a blood blister formed. The swellinspg seems to have gone down but I think the blood blister is still there. Hard to see if it healed because it is right on top of a brown freckle/birth mark. Any insight u can provide would be appreciated. Doctor: Hello,I understand your concern.I am Dr. Arun Tank, infectious diseases specialist, answering your query.In my opinion it is normal nothing to worry.It will take some time to blood to get absorbed.If the wound is infected than you have yo worry. As there is no sign of infection there is nothing to worry As it is on birth mark it will take some times.Please maintain the cleanliness and local hygiene it will help in speed up healing.I will be happy to answer your further concern, you can ask me on bit.ly/DrArun. Thank you.Dr Arun TankInfectious diseases specialist."
},
{
"id": 67510,
"tgt": "What is the hard and red lump in my pubic region?",
"src": "Patient: Hello doc. I have a reddish bump in my pubic hair. About 1 inch away from base of shaft. I just noticed it today and am worried cause of the area. It s not an std I think cause I ve never participated in vaginal intercourse. It does not hurt unless pushed upon and when pushed upon I can feel a hardness to it. About the size of a pea. Just would like to know what s up. Thanks. Doctor: Hi,It seems that you might be having some ingrown hair follicle infection.If lump increases or pain develops, go for one antibiotic medicine course for 3days.Keep local hygiene clean and well shaved.Ok and take care."
},
{
"id": 201446,
"tgt": "Does reducing weight increase penis size?",
"src": "Patient: Hi, i am 22 yr and i am slightly obese my main concern is abt my penis size. When its flaccid only the glans or the head is visible and when erect its abt 3 and the shaft is really slim. Should i consult a doctor physically. Will reducing weight increase my size? Doctor: Hi,There is no change in penis size after loosing your weight but after loosing your abdominal fat there will be apparently penis size will look bigger as due to abdominal fat penis remains in fat gives very short penis.While erection having this much size will not create any problem in your sex life.So forget about size of penis and relax.Ok and take care."
},
{
"id": 45866,
"tgt": "Suggest alternative option for normal urine pass",
"src": "Patient: 11 year girl have a spina bifide (myclo meingocale) closed at birth.Doctors adviced bladder augmentation and Mitroffnof procedure.Please advice is their any other option from which she can pass her urine normally without any cathalitization process.???? Doctor: Hello and Welcome to \u2018Ask A Doctor\u2019 service. I have reviewed your query and here is my advice. Surgery is the only option available. You can also opt for life long catheterization which has got chances for complications like recurrent infection. So surgery is the preferred choice. Hope I have answered your query. Let me know if I can assist you further. Regards,\u00a0\u00a0\u00a0\u00a0\u00a0 Dr. Shinas Hussain"
},
{
"id": 79580,
"tgt": "What causes sharp pain in the left side of the chest?",
"src": "Patient: I have had really strong sharp pains in my left side of my chest. Sometimes they last a few seconds other times a few minutes and I can feel my hear beat faster and I get really hot. I ve also noticed more headaches frequently, stuttering and dizziness Doctor: Thanks for your question on Health Care Magic. I can understand your situation and problem. In my opinion, we should first rule out cardiac cause for your left sided chest pain. So get done ecg and 2d echo. If both are normal, than no need to worry for heart diseases. Sometimes, stress and anxiety can also cause similar symptoms. So better to consult psychiatrist and get done counseling sessions. Try to identify stressor in your life and start working on its solution. You may need anxiolytic drugs too. So avoid stress and tension, be relax and calm. Don't worry, you will be alright. Hope I have solved your query. I will be happy to help you further. Wish you good health. Thanks."
},
{
"id": 96362,
"tgt": "I am suffering from continuous urine passing problem",
"src": "Patient: dear mam' iam suffering from continious urine passing problem. more than 20 times a day. y is it so. not able to control urine just for 5 min also Doctor: hai"
},
{
"id": 155987,
"tgt": "Is cancer passed from mother to child?",
"src": "Patient: my mother found a lump on her breast at 32 years old right after and while nusring my now 17year old sister. obgyn did a physical exam and told her it was a cyst or a blocked duct. she then had another child and nursed her as well. 5 years later she was diagnosed with stage 2 ductalcatsonoma. she has since lost her battle with breast cancer almost 4 years ago. the issue now is that my now 15 year old sister has been diagnosed with osteoscarcoma of the spine. would having a baby and nursing with breast cancer have anything to do with the child developing cancer later in life? does the Brca2 cause bone cancer as well? all confusing for us right now. Doctor: Having a baby snd nursing them will not transfer the cancer.The baby will be at increased risk for cancer later on in life based on the fact t of family history.Osteosarcoma has never been linked to brca2 mutations .However li-fraumeni syndrome (p53 mutation) is know to cause breast cancers , sarcomas in the family.This syndrome is rare and osteosarcomas can occur sporadical also without mutations."
},
{
"id": 208544,
"tgt": "What to do for the weird feeling?",
"src": "Patient: sir,my brother is having some mental problem from past two years.we exactly cannot recognised what he has.he used to say that there is always some sound he heard in his ears.he always talk to invisible ,he said.he even ask from parents that from where he was brought,he belongs to some other family.he never believe any one.sometimes going to police station and told them to solve his case,and asks them if he might be suffer from ulcer,cancer or aids,what he has,he put questions to everyone whom he met.he is not doing his routine work .,leaveing his job from two years.sometimes he tells that he has twin brother who did some crime,and for that crime,he is punsihed,he dont know the reason why,saying how much delay remins,and said that he wants leave his body,want to die.he never sleep more than two hour,he is dreaming that we all are coming in his dream and do something abnormal to him.he become violent when we give him medicine,the doctor recommend,he refuse to take.he even not took food for no.of days.how we tackle him.please doctor help us to treat him.my father is unable to carry him to doctor.pls give me advice, Doctor: Hello,Your brother's symptoms are suggestive of psychosis, most probably schizophrenia.He needs medicine either he agrees or by putting it in his food..or by giving him injectable antipsychotics forcefully once a week.He will definitely get better with treatment..you might feel his improvement in few days of giving medicine. If he is accepting food/ liquid from your home, the medicine can be added in his food. You shoukd ask his doctor to write such liquid formulations.Please donot be dishearten, and also boost confidence of your father. I can undertand that your father is very stressed, but please tell him that with medicine your brother will get back to life and work too.It is quite usual in psychiatry that patient dosent accept treatment, so for sometime medicines are given smartly or many times force fully.Wish you good luck.Dr. Manisha GopalMD Neuropsychiatry"
},
{
"id": 14333,
"tgt": "What could be the itchy rashes on torso?",
"src": "Patient: My 4year old son had a sickness bug 4 days ago and this morning woke up with an itchy rash on his torso I've given him antihistamines to help with the itch. I've done the glass test and it disapered under it, he did have a mild temp of 37.3. Could this be a post viral rash? Doctor: HiYes it could be viral but it might also signal a streptococcal infection so it would be a good idea to have a doctor see him"
},
{
"id": 181789,
"tgt": "Suggest treatment for lumps on the side of jaws",
"src": "Patient: I have lumps either side of my jaw the size of a 50 pence piece they have been there over 3 months now. I have been to the doctors and had scans come and they come back clear. What could it be ? It's not glandular fever that came back clear too. I'm just woried. Doctor: Hello, Thank you for consulting with HCM.As you are mentioning that you have lump on either side of your jaws and scan showed nothing abnormal. It can be possible that these lumps are just lymph nodes which might have become enlarged.These lymph nodes get enlarged in cases of throat or dental infections. So better to visit your dentist and ENT specialist for diagnosis and complete treatment.Hope it will help you."
},
{
"id": 89843,
"tgt": "What causes cramps in c-section area?",
"src": "Patient: Dear Doc, I was done a c-section five years back but i over-bleed and taken back to theatre and i suppose another operation was performed. The following day after both surgeries I woke with a red patch in my right lower stomach area like I was internally bleeding. It faded away though but afer five months i felt a slight cramp in the area where i had red patch just before my perids. The cramps always come ever time i am about to be in my periods.....please help Doctor: This may be due to a condition called Endometriosis. Consult a Gynecologist or a General Surgeon and this operated. The lesion has to be excised in toto to get a good clearance from the surrounding tissues. You may request for histo-patho examination o confirm the diagnosis."
},
{
"id": 147069,
"tgt": "Head injury, bleeding in brain, spike in BP, confusion, disturbed vision. CT scan done. Prognosis?",
"src": "Patient: My 90 year old mother fell, hitting her head, and suffered a brain bleed on both sides of her brain. She has hypertension . We don t know if bleed occurred before the fall, or after the trauma to the head... Her bp was 280/160 immediately after the fall. She has been hospitalized for 12 days now. Ct scans have been done... She is very confused, speaking but making no sense... And her vision has been effected severely. She has not slept in 4 days... Continues to speak, move her arms, etc... What can we do and what can we expect? Doctor: Hello,I am Dr.Gaurav Baidya. Thank you for your query. I am happy to help you here.If your mother was an old hypertensive, then there was a chance of bleed and this can be billateral.And yes as you said it can be a cause of fall or may be ( not very likely) fall can be the cause of bleed.The kind of complaint you are talking usually apears after a lesion on wernickes area of brain that is in the temporal region. Specificaly there is no specific treatment if the tissue of brain is damaged. She is conscious means there are chances of improvement. Such cases of brain hamerroge have poor outcomes but still many pateints are found to return in far better state then what was expected. Begin hopeful may not be bad in this condition but it may take 30 to 60 days for final outcome state. The bleed in brain subsides slowly and as it starts subsiding the lesion might get better. Nothing specifically to be done just pray to god and be hopeful, its better to keep her in the hospital and take the best possible care from your side. Things may change to a far better state.Thank youRegardsDr.Gaurav Baidya"
},
{
"id": 221900,
"tgt": "Is induced labor advisable for having high uric acid and protein levels in urine?",
"src": "Patient: I am 37+4 days pregnent. i had my Bp last week at 130/90 which has now reduced to normal ie 110/80. i have trace of protien in my urine & my uric acid level is 6.6. this is my 2nd baby & my 1st one was a c sec as my son was breech. what is ur suggestion .....do the dr need to deliver me now?? Doctor: Hi,Since you have a previous caesarean, I would be more cautious before going for induction at 37 weeks, as if, the neck of the womb is not favourable, it is likely to fail and you could end up with a repeat caesarean.Also the complications and chances of failure are higher in previous casarean who are induced.I would advise you to get your blood pressure monitored regularly, and urine protein 3 times a week. If found to be high or if you develop any complication such as headache, epigastric pain then you should contact the doctor immediately.At 39 weeks the doctor should discuss if you could be induced, or a repeat caesarean would be appropriate. Meanwhile if you go in to labour yourself, you will have maximum chance of a normal delivery.Hope this helps.Regards.."
},
{
"id": 113599,
"tgt": "Pregnant. Given cortison shots for lower back pains. Myelogram, MRI and x-ray done. Should I proceed with the pregnancy?",
"src": "Patient: I just found out I am 10-11 weeks pregnant . On January 28,2012 I had lower back pains and I was give cortison shots for lower back pains. then on January 30, 2012 I was given x-ray for my lower back. Then on Februray 16, 2012 I had a MRI on my lower back Then March the 5, 2012 I had a Myelogram. And last I had a CT on March 9, 2012. What are the risk of my baby at this time and should I proceed with the pregnancy? Doctor: Good morning. I feel very sorry for u. U had underwent o lot of procedures. it is very dangerous if fetus is exposed to radiation in early weeks as most of internal organs start forming. Now u say that ur in 10-11weeks of pregnancy. U undergo detailed examination and ultrasound examination. If anyproblem is moted u can take opinion of your gynecologist. Thank u"
},
{
"id": 49742,
"tgt": "kidney stone, lower back pain, frequent urination, passed stone in urine",
"src": "Patient: I had a kidney stone on Feb 20 and about a week later started having lower back pain on opposite site where stone was. Also I am having frequent urination . I did pass the stone and it does not hurt to urinate. Doctor: Hi. Sorry I am not able to understand your query. Since you are aware that you have stones you need some investigations to decide further course of action. Frequent urination may be because of stone in the bladder, lower end of ureter or urinary tract infection. It might not be painful while passing small stones. Plain CT scan of the abdomen is the best investigation for stones. Please consult a urologist.Regards"
},
{
"id": 3354,
"tgt": "Chances of getting pregnant with an unprotected sex",
"src": "Patient: Hi, i am a singaporean. My last period started on the 10 december and ended on 15 december. My period are irregular. Am i possible to be pregnant if my boyfriend does not ejacualate inside me? The last time we had sex was on 10 december. He did not ejacualate inside me. Recently i have feel cramps but it had already stop. My breast are not really tender and my nipples did not change in colour. By the way does gastric problems delay period? Doctor: Hello dear.Understand your concerns.you are secured from pregnancy because you have sex in safe period, there is no chance of being a pregnant.During safe period, ovum is not released so no fertilization has been occur.But if you had sexula intercourse in unsafe period earlier with out ejaculation, then pregnancy is possible. As precum before ejaculation contain sufficient sperm and it enter into vagina unknowningly, so cause pregnancy.Gastrointestinal problem dose not cause delay in period.Your period could be dealyed by the hormonal imbalance, excessive stress, change in life style and dietary habit, ovarian cyst.Reproductive hormone analysis, USG scan, ovarian follicle study will help in the diagnosisPeriod can be induced by the progesteone pill under the advise of the hynecologist.I hope this will help you.ThanksDr.sagar"
},
{
"id": 91640,
"tgt": "What causes severe abdomen pain with cramps and nausea?",
"src": "Patient: I have been experiencing stomach cramps for over 24 hours, they are quite severe at times and fade away completely at other times, I am also experiencing nausea with them which appears to increase when I eat. The pain is in the upper abdomen and seemed to ease a little when sitting, although I find since eating this is not the case. Doctor: Hi ! Good evening. I am Dr Shareef answering your query.This could be due to some kind of food poisoning if you ate from outside your home. I am not aware if you do smoke or drink. In that case you should get an ultrasound done and some routine blood tests along with a serum lipase and amylase to rule out pancreatitis and gall stones. If I were your doctor, I would advise you for an ultrasound abdomen followed by a CT if need be. However, all this would be preceded by a physical examination and some other investigations if need arises. Till this time, you can go for a proton pump inhibitor drug, a probiotic, an anti spasmodic if advised by your treating doctor.I hope this information would help you in discussing with your family physician/treating doctor in further management of your problem. Please do not hesitate to ask in case of any further doubts.Thanks for choosing health care magic to clear doubts on your health problems. Wishing you an early recovery. Dr Shareef."
},
{
"id": 89978,
"tgt": "Please suggest a remedy for stomach pain",
"src": "Patient: hi,,i am suffering from stomach pain last 1 months and itake aurvedic medicin and remove the constipation problem but last 5 days take any diet even water will be drunk .presently also pain in stomch.please suggest me take help me what kind of precautions i take. Doctor: Hi.First of all you have undergo the tests as you are suffering from Abdominal pain for the last one month. Get the following tests done:UltrasonographyCT scan of abdomen to rule out Gastrointestinal Cancer.blood, urine and stool tests Enteroclysis to see if there is an Intestinal Obstruction.Once the diagnosis will be done the treatment will be acording to the cause ."
},
{
"id": 84241,
"tgt": "Is it safe to take expired liquid Pepto Bismol?",
"src": "Patient: I have a really bad stomach ache, and I can t throw up or use the bathroom though I feel I must;the only medicine for these sicknesses is Pepto Bismol, but that expired in May. It s June 10 right now. Is it still safe to take expired liquid Pepto Bismol(target brand Up&Up) Doctor: Hello,using expired products is not recommended unless it's a medical emergency - a matter of life or death, for example, an expired drug in case of an allergic shock when nothing else is available and the patient is dire condition). Nausea is not a medical emergency. Therefore taking expired Pepto-Bismol is strongly discouraged.I hope I've answered your questions. Please let me know if you need further assistance.Kind Regards,Dr. Panagiotis Zografakis,Internal Medicine Specialist"
},
{
"id": 180036,
"tgt": "What diet should be given for a 3 year old diagnosed with Frazier s syndrome?",
"src": "Patient: Hello, my name is Lisa. I just found out that my 3 yr old son has Fraziers Syndrome. He is also developmental delay, Autistic and has cardiac mumours. My question is all the research that I have done tonight shows me that it all boils down to the kidneys. what diet should I have him on to maintain them in Great health and can he play in sports without stressing out the body. Doctor: Thanks Lisa for putting up your query at HealthCareMagic. I am sorry that your 3 year old son is suffering from Frasier syndrome. It is a rare disease with improper development (dysplasia), underdevelopment (hypoplasia), or absence of one or both kidneys (unilateral or bilateral renal agenesis) caused due to a genetic defect. There are certain other associated abnormalities and I am not going into the details. Unless I can come to know the exact nature of kidney defect your child has, it is difficult to comment on the diet. An ultrasonography would make things clearer. That would also indicate whether the livers are affected as well. That too would play a role in determining the diet. It would also vary with the presence or absence of protein in urine which would indicate a defect at microscopic level. Please get back to me with the details and I would try to help you further."
},
{
"id": 100355,
"tgt": "Suggest therapy for asthma",
"src": "Patient: Dear Doctor, I am 31 years old, male living in Mumbai. I have had a brief history of heart diagnostic starting a year ago. I had applied for a life insurance policy with SBI for which I was asked to get a health check up done at a reputed hospital in Mumbai. The cardiologist over there took my ECG and said that he cannot do a stress test as I was suffering from a heart disease 'cardiomyopathy' and he said 'my heart is growing in size and I will need a heart transplant 10-15 years later'. He asked me to get an angiogram done. But I thought I was hale and health and I ignored his suggestion to get an angio done. A few months later, I started getting some pain in the chest - sometimes burning, squeezing etc on the left part of the chest. It was not severe ever but mild pain. I went to a cardiologist (different one from the earlier one) and got an echo test and a stress test done. As per the reports, the echo test was normal while the stress test said \"border line positive for ischemia\". The doctor however said this is likely to be a false positive as there was baseline abormality in my ECG and asked me to come after 3 months if the pain persists. As it so happened, the pain continued to come on and off. In the meantime, my father was diagnosed with liver cancer and I shifted base to Bangalore. Since I was getting these pains, I wanted to get them checked thoroughly and went to a cardiologist in Bangalore. He saw the reports and said the same thing that it is likely to be false positive. He did an echo test and said it was very likely false positive. He also said that if the pain continues, he will recommend a stress myoview. Unfortunately my father passed away within three months of diagnosis and I moved back to Mumbai. I visited the doctor again as I started getting palpitations and the pain continued to come. He said that it must be due to stress and he continues to think it is false positive. He however said that for my own comfort, I should get a CT-angiography done. I have read fair bit on the internet about all these tests and heart diseasees. I have read about all the risks involved in each of the procedures that have been recommended to me so far. I am not able to make out which is the right procedure for me. I am allergic to pollen and get asthma while in Bangalore. In Mumbai I am quite stable. I use siroflo inhaler occasionally. Apart from this I dont have any other health problems. My BP usually is 130-140/80-85. heart rate ranges 75-95. My cholesterol levels are not that great HDL -31, LDL -142, triglycerides - 199. Let me explain you about my pain: 1. It comes at various ponts on the left chest 2. Sometimes it is a burning sensation, sometimes squeezing, sometimes pains the way a wound would pain 3. The pain is off and on - average 4-5 times a week and lasts for a few seconds 4. Sometimes when I exhale, I get a pain along a column inside the chest. Feels as if something is rubbing 5. It has never been severe; mild pain and I continue to do my daily work as usual 6. I dont remember getting any chest pain while excercising. 7. At times when it pains, I feel like moving my chest to avoid pain. Can you please help me with what can be the cause for my pain and which test should I go for to know whether I have any heart disease or if yes, what is it? I have been an athletic individual for a long time. In the last 8-10 weeks, I feel my heart is pounding very hard and I can feel my heart beat a lot. I can see the left part of my chest shakes rhythmically with the heart beat. Please do let me know if you need any other information regarding my problem. Thanks. Doctor: Hello,Thank you for asking at HCM.I went through your history and would like to make suggestions for you as follows: (I will reply to you in a step-by-step manner)1. I should clarify that I should not suggest you any test for cardiac evaluation. These can be best suggested by cardiologists only. From your detailed workup, I can assume that at least two cardiologists believe that you are not having any significant heart disease at present. 2. I would like to know especially more about your allergies and asthma. This is because asthma can cause chest pains, an acute episode can cause even chest tightness. Not only that, an entity called \"GERD\" (gastroesophageal reflux disease) may be associated with asthma and it can cause burning type of chest pains.3. Hence, I would not interfere with your cardiac workup, but I would like to know - a. Since what age do you have asthma? What are the symptoms? What is the frequency of your symptoms? How you manage it? How was your asthma diagnosed?b. What are your nose/sinus symptoms? Do you have running nose, sneezing, nose obstruction, headache, facial heaviness, repeated throat clearing, etc? c. Do you have any symptoms like - regurgitation of sour fluids/foods after taking meals, especially after heavy/spicy meals or after lying down? Do you have sensation of nausea or vomit frequently?d. What are the medications that you frequently use or have used so far for allergies and asthma? What are any other medications that you use frequently/regularly?e. Is stress an important factor in your life?f. How is your digestion in general? DO you have repeated abdomen pains, constipation or diarrhoea?4. Above things are very important as - some of the asthma medications can cause palpitations, some of the asthma medications can worsen GERD. I would like to know whether you can correlate your symptoms with any particular drug/food intake.Hope above suggestions will be helpful to you.Should you have any further query, please feel free to ask at HCM.Wish you the best of the health ahead.Thank you & Regards."
},
{
"id": 33867,
"tgt": "Suggest better treatment for fungal infection",
"src": "Patient: I have had fungal infection for about two years now. The Doctor I have been consulting with prescribed advantan cream but this cream just makes the itching go away for a while and then it comes back. Now the itching has become much worse and I don't know what to do now. Please help with the solution. Doctor: Hi,You might require one course of oral anti-fungal medicine to get complete cure.Apply anti-fungal cream locally.Go for anti fungal medicine like Flucanozole 150 mg once in a week for 4 weeks.Keep affected part clean, dry and airy.Ok and take care."
},
{
"id": 101221,
"tgt": "Is fruit allergies causing irritation in tongue and throat?",
"src": "Patient: Hello, I have just found out about pollen and fruit allergies. I can t eat many fruits or nuts anymore as it causes my lips to burn and feel like they are swelling, my tongue gets raw and my thoart feels like it is staring to swell. I can t even drink sprite or any citrus drinks due to this. Also, when I eat something I shouldn t I get like a yeast infection but not a real one. I itch terribly in my private areas. Am I really allergic to fruit etc Doctor: Hello.Thank you for asking at HCM.I went through your history. I would particularly like to ask which fruits cause you symptoms and which fruits you can take safely.From your history, I would think first of Food-Pollen Syndrome (Oral Allergy Syndrome). It is a benign allergy syndrome with rare consequences/progression.It usually causes symptoms only after consumption of raw fresh fruits. So I would suggest you following:1. Please make a list of fruits that causes symptoms. You can take rest of the fruits without trouble.2. You can try taking cooked/steamed/baked fruits. This will alter allergenic proteins and may not cause you symptoms. Same way, some allergens are in peels of the fruit, so removal of peel also may not cause symptoms.3. If you have any symptom like itching, swelling, burning in lips/throat/oral cavity, please take an antihistamine like cetirizine immediately.4. I would suggest you to consult an Allergist-Immunologist who will ask your symptoms in great detail and may advise you allergy testing to confirm the diagnosis.5. Important: If fruits cause you symptoms like abdominal pain, vomiting, dizziness, severe throat swelling, suffocation, please do not take these fruits without confirmation of diagnosis.Hope this will be helpful to you.Wish you the best of the health.Regards."
},
{
"id": 126641,
"tgt": "What causes the recurrence of pain and poor blood circulation in the legs which resolved with Pletal the first time?",
"src": "Patient: My husband takes codeine, prescribed by sleep doctor at Winthrop Hospital on Long Island for serious sleep condition. 15 mg. Years ago he took Pletal (cilostazol) for pain/circulation in legs. He stopped when it got better. This week it is back. Can he take the two drugs or is there an interaction. Thanks. Doctor: Welcome to HCM community. Before elaborating on your husbnds problem, let me tell you that there are no reported interactions between codiene and Cilostazol. We prescribe Cilostazol in patients with claudiccation. Claudication occurs with narrowing of the arteries of the lower limbs. If the pain is back, it is better to visit your physician who will prescribe Cilostazol along with anti-platelet drugs like aspirin or clopidogrel. Cilostazol helps in improving the symptoms and increases the walking distance. Anti-platelet drugs are given to decrease the risk of stroke and cardiac problems.Exercise is the most crucial part of treatment. Before taking the medication, it is better to start exercising to the point where the pain starts. When he feels that the pain is about to start, he can stop exercising. This will help in the growth of new vessels which supplements the blood supply to the lower limb.If the patient cannot exercise or if the cilostazol does not improve symptoms, then your doctor will offer you re-vascularization procedures. In revascularization, the blood flow through the blocked artery is restored.I would recommend that you schedule an appointment with your GP. She/he will discuss all treatment options with you and make the necessary referrals. Thank you for sharing your story with us. Please feel free to ask if you have any more questions."
},
{
"id": 128753,
"tgt": "Could slight numbness in thumb and forefinger with tingling hand indicate neurologic disorder?",
"src": "Patient: i have slight numbness in thumb and forefinger and some tingling in hand. no injury no other symptoms. has lasted three days. numbness is quite slight so not sure if I am making more out of it than I should. worrried about ms or other neurologic disorder. Doctor: dear patient what is your age ? what is your profession?please always mention this when you are asking for treatment. your symptoms do suggest possibility of nerve compression . two possible diagnosis are 1.carpel tunnel syndrome 2. cervical disc prolapse.this can be diagnosis by various clinical tests. imaging required are 1. NCV of median nerve of involved side and 2. mri of cervical spine. please consult neurophyysician nearby you. meanwhile start tab pregalin x 75 one at bedtime. thanks."
},
{
"id": 142809,
"tgt": "What causes nausea, fatigue and headaches after having a concussion?",
"src": "Patient: I am 60 years old, three months ago I fell down on an icy floor, having a concussion. I had dizziness, fatigue, headaches... I did physical therapy for positive BPPV. Today I woke up with a strong spinning, lasting 1 minute and nauseated. I have it mostly when rising, lying down and rolling over in bed. My physical therapist did the nystagmus test with the chanalth repositioning maneuver, it didn t show positive. But I had ocular nystagmus from the vibration of the neck. She was not sure why. This is really frustrating...... Can I have some light on this? Doctor: Hi, Welcome to HealthCareMagic.com I am Dr.J.Mariano Anto Bruno Mascarenhas. I have gone through your query with diligence and would like you to know that I am here to help you.the fall can injure temporal bone and disturb the vestibular apparatus. this can cause dizziness and other symptoms you have told Hope you found the answer helpful.If you need any clarification / have doubts / have additional questions / have follow up questions, then please do not hesitate in asking again. I will be happy to answer your questions. In the future, for continuity of care, I encourage you to contact me directly in HealthCareMagic at http://bit.ly/askdrbruno Best Wishes for Speedy Recovery Let me know if I can assist you further.Take care."
},
{
"id": 5023,
"tgt": "Trying to conceive, following calendar method to match ovulation, sharp pains after ovulation day. Should i wait days after calender method shows?",
"src": "Patient: I've been trying to get pregnant dor the last year and a half, but no luck. I have been following the calandar method to match my possible days of ovulation, but I I for some reason I feel like I'm missing the ovulation day because I usually get sharp pains a few days after my ovulation day. Do you think I should wait a few days after what the calandar method shows? Doctor: Hi,It is better to go for ovulation study for which kit is available which gives exact day of having ovulation.When ovulation is there go for sex for 2-3 days daily.After sex do not go for toilet or stand up for half or one hour.After intercourse keep your both knees near your chest to prevent semen coming out of vagina.Ok and take care."
},
{
"id": 19787,
"tgt": "Is Ecosprin safe to be taken while on Olmetrach-H and Rozat?",
"src": "Patient: Hi, iam 40 years old...live a very hectic lifestyle...my cardiologist advised olmitrack h 40 in mornings and amlog mt50 and rozat f at night...hv been on hyperyension medication from 5 years...my bp is 140 by 90...IT job keeps me awake till 3 am most of the times....ECG, bloood report etc was normal....question: given my hectic life style should i hv ecosprin as well..if yes at what time...iam already having too many medicines i think...my pulse is usually in 90s during day time...resting bpm is normal...hv had issues of breathless ness too...dr said it could be due to weather but i dont like this feeling Doctor: dearI appreciate your concern for health.The medicines prescribed are perfect and should be continued uninterrupted.How ever my advise is that please dont add ecosprin without any indication .In stead try your best for rest and sleep for your body as it is a bare necessity now.Hope your Echo,Esg are within normal limits.Do also have an eye on a healthy,timely and regular diet .Hope this helpsmost welcome for further queries."
},
{
"id": 198693,
"tgt": "What causes penis ejecting fluid when applying pressure to pass stool?",
"src": "Patient: applying pressure when trying to pass stool in turn makes my penis eject out fluid and i also spend a lot of time trying to defecate. I allow the defecation to happen by nature and i don't put pressure trying to pass stool. I am suffering from constipation. Kindly help Doctor: HelloPassage of hard stool exerts pressure on urogenital organs which generally secrets and stores seminal fluid So that epells out due to pressure by stool loaded colon which further augments when you put strain during defecation.So you need not to worry, correct the constipation and it will disappear automatically You can take laxative or stool softeners in case of acute constipation you should take lots of fresh fruit and green vegetables with dietary finer supplements which will increase the roughage and facilitate smooth passage of stool.You should take plenty of fluids especially on getting up in morning have half to one liter other fluid mixed with lemon juice and honeyHope this helps you Please feel free to discuss further I will be happy to help you Best wishes"
},
{
"id": 136533,
"tgt": "What to do for locking of the joint near the palm and index finger?",
"src": "Patient: Hello, the joint nearest the palm on my index finger keeps locking up and is extremely painful. I do have small joint arthritis which is fairly well controlled by medication. This may be associated but I have never suffered from this before. The lock eases by itself after a period of time. I am concerned that it is happening quite frequently and is debilitating when it happens. Any help would be appreciated. Regards, May Richards. Doctor: HiIts condition called tenosynovitis. In this sheath of tendon get inflammed. Pain and swelling occurs. Start ice packs in affected area, anti inflammatory medicine. If it doent work we can offer injection at the site with success rate more than 95 percent.Thanks"
},
{
"id": 98055,
"tgt": "Old itchy keloid in chest, doesn't want surgery. Any ayurvedic cream ?",
"src": "Patient: the question is for my mother, she has keloid which is old and looks bad, it is itchy,in the chest .it is not a small one. she is afraid of surgery and prefers any cream to get relief, that may be ayurveda also. she is also suffering from asthama and also not in good mood. she is also recovering from an accident injury which she sustained 4-5 months back after falling from two wheeler. Doctor: 1. cause of occurrence of keloid, occupation? 2. because you say it is old so probably accident might not be the cause. 3. Kasisadi tail for external application and guggulu preparations for Internal. 4. Ayurvedic treatment is directed at reducing the size of the keloid"
},
{
"id": 216798,
"tgt": "Suggest treatment for numbness in left hand and lower back pain",
"src": "Patient: I m having a hard time typing. The thumb and first two fingers on both hands are numb right now. It comes and goes but has become more frequent. I ride a bike and that has become a problem too when I can t feel the handle bar. It was mostly the left side at first but now it s either one or both. I m also over weight and have been sitting on my ass taking computer classes for the past couple of years. Thats why getting back to riding the bike was important. About 1 month ago I hit a really bad pothole in the dark that jarred me really hard in the lower back. Since then I can t walk very far without the back pain starting. Doctor: hi,as you mentioned the brief history of you I will advice you to take an MRI of the cervical and lumbar spine to check the soft tissue Injury.the symptoms you are facing are looking more of like a pinched nerve in the spinal column. also this can be helped by a physical therapist who will use ultrasound therapy for muscle spasm and TENS therapy to reduce pain. both has a good research success.also, post that performing regular exercises under the guidance of physical therapist for neck and back.once you learn you will be back to normal. but, never stop this exercises in future. as a great saying, prevention is better than a cure.I have seen 100% of success rate in cases with pinched nerve.I with you a good health."
},
{
"id": 16844,
"tgt": "Suggest treatment for mitral prolapse valve",
"src": "Patient: I as told about 20 years ago that I have mitroprolapse valve. My mother had it and so do other members of my family. When I have a stressful day it really acts up. I had a heart monitor on yesterday and of course it was settled down and didn t do much. They took it off today and of course now it s really palpitating. And it makes me tired and makes me fell like I have to take deep breaths. Will some activity show even though it wasn t acting up. And if not should I ask for a different test Doctor: Hi, Mitral valve prolapse is a genetic disorder involving the Mitral valve. Treatment depends upon the severity of Mitral Regurgitation (leaking of the valve) and symptoms. If regurgitation is severe and the patient has symptoms, surgery (mitral valve repair) is indicated otherwise medical management is enough. Hope I have answered your query. Let me know if I can assist you further. Take care Regards, Dr Sameer Maheshwari, Cardiologist"
},
{
"id": 171129,
"tgt": "Suggest treatment for colic pain",
"src": "Patient: My child is 2 months old and she get assumingly a pain and she keep crying... We suspect its a colic pain.. and we give some drops of colicaid....she doesnt really get well after that n we keep swinging her for hours before she finally gets rest n sleeps. Let me know what i shd really do ? and how frequently this colicaid is okay to give. Doctor: Hi,Infantile colic is a common problem upto 3 months.You can give Colicaid or Colimax drops as and when required.Please see that child is not hungry and due to hunger she might be crying.Change the diaper frequently as wet diaper does cause inconvenience to a child.Ok and take care."
},
{
"id": 14232,
"tgt": "Suggest treatment for itchy rash behind and around ear",
"src": "Patient: Hi, l last week I had an itchy rash behind and around my ear, it then spread to the other ear. I went to the Dr s who prescribed me antibiotics and fungal cream.the rash is still there, but moving to my hair line and down the centre of my back. The rash is red and pimply and hot to touch Doctor: hi there.1. your concern may be seborrhoiec dermatitis.2. you can try lulifin lotion, once daily bed time for 30 consecutive days.3. apply linical lotion, as on require for relieving the pain and burning sensation.4.meanwhile consult another dermatologist for physical examination and confirmation of diagnosis.this may help you."
},
{
"id": 64832,
"tgt": "Suggest treatment for a painless lump on the head",
"src": "Patient: Hello, I was just googling lump on top of head and this site came up... I have a painless lump on the top of my head, and it feels like it is under the skull. It s in the center, and it s about an inch in diameter. Also, there is an indentation, about the size of a nickle, in front of it, directly where the fontanel would be, and another directly behind it. All three areas are perfectly aligned with the center of my skull. I m 43, female, about 175lb, and about 5 7 . I m perfectly healthy, and have been a very well-balanced vegetarian for over twenty years. Thanks so much. :) Doctor: HI,Luke,Dear,Thanks for the query to HCM.1-I have studied your query in depth.2-DEar,IN my opinion, the lump of 1 inch on top of your head,painless,with nickel size front indentation and one indentation on back of the head lump-all aligned with the center of your skull, is of multiple cysts? most probably-sebaceous cysts ,3 in no. in central line of your skull . and or/ it may be a dermoid cyst-with skull defects.This diagnostic puzzle would be clear on USG and Skull X-ray/CT and sos-MRI- of the head.3-Treatments-would be decided by a Surgeon-for sebaceous cyst or a Brain Surgeon-for dermoid /teratoid-cysts(congenital origin)-depending on the defects in the skull -if found.4-Hope this would solve your query to your satisfaction.5-Wellcome for more queries to me On HCM.6-Thnks once again for your inquisitive query.Have a Good Day..!!"
},
{
"id": 89966,
"tgt": "What is the treatment for entamoeba histolytica?",
"src": "Patient: Hi Doc, I am 33yrs old, 5.8inches tall, 55kg, I my laboratory test read Entamoeba Histolica three months back. Different antibiotics were precribed by doctors which I used, but eventually I am still having mild stomach ache and I gas and belch. Please what will I do? Doctor: HI ! Good morning. I am Dr Shareef answering your query.Amebiasis is a food born parasitic infestation, and so you should stay away from food from outside which is not infrequently cooked in unhealthy conditions. Go for high roughage diet, and plenty of liquids. If you already had a course of metronidazole in its proper dosage by your treating doctor, I would like to add a proton pump inhibitor, and a probiotic for six weeks. In case of no relief with these, I would not hesitate to refer you to a gastro enterologist for a possible endoscopic evaluation of your intestinal tract for a proper diagnosis, and appropriate management.I hope this information would help you in discussing with your family physician/treating doctor in further management of your problem. Please do not hesitate to ask in case of any further doubts.Thanks for choosing health care magic to clear doubts on your health problems. Wishing you an early recovery. Dr Shareef."
},
{
"id": 64427,
"tgt": "What causes itching bumps on clitoris?",
"src": "Patient: My clitoris has been uncomfortable in the past few months. I had my annual less than four months ago. And I had bacteria in my urine. Due to sexual activity I have become increasingly uncomfortable after sex in my clitoris area... thinking it was a yeast infection i get a cream... and find to irritated bumps of think skin on the lining of my clitoris. It is very itchy. it looks like two skin growths... I'm an extremely worried. what exactly could this be? Doctor: Hi,Dear,Good Morning from India.Thanks for your query to HCM Virtual Clinic.-I studied your query in depth.I understood your concerns.-Diagnosis-In my opinion-you may be suffering from-clitoral epidermal inclusion cysts with discharge-causing itching.-Cause-This could be due to the urinary infection detected in your last annual.-From the facts given from your query,I feel your itching bump is due to the cystitis secondary to urinary infection.-Clinical examination from Surgeon would fix yor worry.-Hope this would help you to discuss with your doctor on your visit.Wellcome to HCM again for any more queries.Have a Good day."
},
{
"id": 187563,
"tgt": "What are the other alternates for braces?",
"src": "Patient: Hi I've had braces on for more than 2 years now.. dealing with them has been beyond a struggle.. Many Cuts. Uncomfort every moment of every day. They break constantly and I have no control over it. I try my hardest to not break them but it's inevitable. When can I get them off with beneficial results Doctor: Hello, Thanks for your query.INVISILIGN It is the invisible way to straighten your teeth without braces. It uses a series of clear removable aligners to straighten your teeth without metal wires or brackets. It has been proven effective in clinical research and in orthodontic practices nationwide.I do hope that you have found something helpful and I will be glad to answer any further query.Take care"
},
{
"id": 24520,
"tgt": "What could cause heart palpitation?",
"src": "Patient: I am a 23 year old male. I started a running routine about a month ago. I never ran much before, but I am to the point where I enjoy it now. I also drink alot of coffee. Three days ago, I started to feel my heart palpitate, though I feel no other symptems which would lead me to worry about a heart attack. I worked out several times since then, and I do not feel abnormal while working out, though once I get calmed down, I feel fluttery at times. Should I be overly worried? I cut back on the caffeine since I started to feel the palpitations. Doctor: Thanks for your question on Healthcare Magic. I can understand your concern. Fluttering sensations, palpitations etc are commonly seen with arrhythmia (rhythm disturbances in heart). And if not treated promptly, arrhythmia can be life threatening. So better to first rule out arrhythmia in your case. So consult doctor and get done ecg, 2d echo and Holter monitoring (24 hours continuous recording of ecg). If all these are normal then no need to worry for arrhythmia or other heart diseases. Sometimes, overthinking of any thing can also cause similar symptoms. So don't think too much, relax and be calm. It's good that you cut down caffeine intake. Don't worry, you will be alright,but first rule out arrhythmia. Hope I have solved your query. I will be happy to help you further. Wish you good health. Thanks."
},
{
"id": 132871,
"tgt": "What is about the shearing nerve on the side which showed in the MRI?",
"src": "Patient: Yes I am 80 yrs old and currently was dx with Bulging Disk, also I have a tarlov cyst on my S-2, and MRI shows some shearing of my nerve on the left side. Also it shows spinal stenosis. I am waiting for an appt. with the specialist, however I would like some information on this. So I have an idea of what to come... Doctor: Thanks for your question on Healthcare Magic. I can understand your concern. This MRI report is not good. Your shearing of nerve is due to spinal stenosis you are having. Best treatment for this is surgical correction of this stenosis. You should first try non surgical measures like physiotherapy, warm water pad application, Avoid heavyweight lifting and strenuous exercise. Avoid sudden bending and change of posture. Loose weight if you are obese. If you still don't improve with all these after 1 month then surgical correction of stenosis is needed. Hope I have solved your query. I will be happy to help you further. Wish you good health. Thanks."
},
{
"id": 130939,
"tgt": "What causes pain under the left rib cage?",
"src": "Patient: I have had a pain under my left rib cage for three days now. It started a couple of days prior and just got worse. Depending on how I move the pain is more acute, and laying down makes it worse. I do have diverticulitis, and have been experiencing a lot of gas lately. Doctor: In my opinion you have splenomegaly i recommend CBC test to check your lymphocytes count Also another possibility in cracked ribso i suggest having a chest x ray as well Good Luck"
},
{
"id": 111772,
"tgt": "Have severe lower back pain due to missing part of bone from birth",
"src": "Patient: My son is 16 and he's been having lower back pain that is increasingly getting worse. I took him to an urgent care where they xrayed him. They said he's missing the very bottom part of his tail bone and the 1 at disc. They said he was probably born like that. Hes 6'1 and it's getting worse. He's starting to throw up and has bouts of diarrhea. What are your thoughts? We've been to a neurosurgeon and they've ordered an MRI that we are still waiting to have done. Thank youRebecca Doctor: Hi. Yes the defect explained by your Doctor on X-ray is congenital and usually have problems in the lower limbs.Diarrhea and throwing up do not look to be related to this problem. It can be due to an intestinal infection and needs different treatment.Please let us know the MRI reports, the clinical problems with lower limbs ."
},
{
"id": 26077,
"tgt": "Suggest medication for paracardiac pneumonitis",
"src": "Patient: what is the medicine for left paracardiac pneumonitis? I made and X Ray examination Chest PA and the results shows: \"There is slight haziness of the left paracardiac area. Heart is not enlarged. The rest of the study is unremarkable. Impression: CCONSIDER LEFT PARACARDIAC PNEUMONITIS. SUGGEST CLINICAL CORRELATION.\" Doctor: Paracardiac is just a region of lung which is next to the heart . One should try finding the cause of pneumonitis and the treatment remains the same as compared to any other pneumonitis. Some organism cause more commonly para cardiac pneumonitis one should order further tests based on your symptoms, where you stay and which organisms are more common in that area . Regards Dr Priyank Mody"
},
{
"id": 94443,
"tgt": "Done emergency surgery for perforated diverticulitis of cecum. Nausea, light headedness, blurred vision. Concerned",
"src": "Patient: Hi, I m a 30 year old female and on January 9 th I had emergency surgery for perforated diverticulitis of the cecum . They removed my cecum, ileocecal valve , and appendix, luckily I do not have a colostomy they were able to reconnect everything without any problems. I did have a general surgeon who actually specializes in vascular surgery do the procedure because he was on call. I have been experiencing some difficulty sleeping since surgery. I will doze off and wake up feeling nauseated and lightheaded with a weird vibratory sensation throughout my chest. Also when I stand up my vision goes dark for a few seconds. I am eating 4-5 small meals a day and drinking one Gatorade and about 2 bottled water a day. My urine is light yellow and I am going about 5-6 times a day. BM s daily to every other day, no diarrhea. I haven t been taking any pain meds since I left the hospital on the 12 th. I take Zantac and Prevacid once daily. It may just be anxiety from the whole experience but I m just a little concerned. Thanks for any advice. Doctor: Hi. I dont think it is related to surgery complication. It is procedure which can be well done by every general surgeon. It is always expected to feel some nonspecific gastrointestinal problems such as diarrhea or stomach cramps but symptoms you re saying dont seem to be serious. If you have fever associated with abdominal pain oe extensive vomiting then you should go to hospital. For other I suggest just to wait few more weeks. It is not so much time from surgical procedure so just be wait a bit more to see are these symptoms gonnna persist. Wish you good health."
},
{
"id": 218652,
"tgt": "Are contractions during 35 weeks of pregnancy a concern?",
"src": "Patient: I m 35 weeks and 3 days pregnant. When I turned 35 weeks my mucus plug fell out and I started having contractions every 6 mins apart consistently. I went to the hospital and was 1.5cms dilated. They couldn t do anything so they sent me home. I was wondering if it was okay to start trying to have him come out considering I m still having contractions and it s been consistent for the past 2 days straight. The doctor at the hospital told me it would be fine. But I wanted a second opinion. Is it okay if I try to speed the process up? Doctor: Hello,Dont speed up the process. Get a steroid cover for yourself and then only speed up your process but it's better once you cross 36 weeks and at the maximum 38 weeks to speed up the process.Hope I have answered your query. Let me know if I can assist you further.Regards,Dr. Mandavi Rai"
},
{
"id": 70343,
"tgt": "I feel a lump on the bone behind my right ear. Cancer?",
"src": "Patient: I was diagnosed with breast cancer in 2008. I had chemo, lumpectomy and radiation, in that order. Just hit the magic 5 year mark last year. Now, I feel a lump on the bone behind my right ear. Now, a little history...I've had hearing problems for the past 10 years and I've had ringing in my ears for about the past 7 years. Of course, I'm worried that this might be cancer. I've made an appt. with the family doctor to have this looked at but I'm a real worry kind of person. Doctor: Hi. Nice to know you have crossed 5 years- this itself makes that this swelling ma not be a cancer.Well, to the facts- Lump behind the ear is not really known to have a cancer. This is most probably a lymph node and develops due to some scalp problems like dandruff or so. You can confirm this by FNAC ASAP. To me this can not be a cancer."
},
{
"id": 37038,
"tgt": "Could the liquid-filled swelling in hand be an infection caused by metal plate and screws?",
"src": "Patient: I have swelling filled with liquid at the base of my right hand , just above the wrist , it was broken four months ago and repaired with a metal plate and screws , I seem to be losing movement too - tendon overuse or infection ? How can I tell the difference ? Doctor: Hello ,I understand your concern. I am Dr. Arun Tank, infectious disease specialist, answering your concern.I suspect infection of the hand.Tendon overuse never causes such a swelling. You should take the X ray out of the hand. Possibility of bone infection can not be neglected. Such infections can be treated as early as possible.I suggest you to take the antibiotics. Ampicillin and clavulanic acid is good antibiotic and can be taken with due consultation with your doctor.Glad to answer your further concern contact me on bit.ly/DrArunWe wish you a best health at healthcare magic. Thank you,Dr. Arun Tank"
},
{
"id": 32933,
"tgt": "Suggest opinion on taking ecstasy for mono",
"src": "Patient: Hi, I was diagnosed with mono a couple weeks ago and planning on taking ecstasy tomorrow. The only symptom I feel is slight fatigue. What will happen to me if I take the ecstasy? P.S I know its bad for me in general I'd like a real answer not just telling me its bad. Doctor: Hello,I would like to welcome you to HCM. If I were your treating doctor I would recommend you not take the Ecstasy, due to the possible dangerous side effects. The biggest danger is that MDMA (Ecstasy) , like most amphetamines, suppresses your immune system. You might find that your symptoms get worse or that it takes longer for you to get better - in extreme cases, it could cause your illness to escalate to a whole new level of severity. Your immunity, due to mono infection is really low. When you do Ecstasy, your heart, lungs, kidneys, liver, and neurological function are compromised.I would suggest you to get plenty of rest and eat healthy.Hope I have answered your question. If you have any further questions I will be happy to help"
},
{
"id": 133149,
"tgt": "How to overcome the pressure caused during opening and closing of my mouth?",
"src": "Patient: I have a problem with my left side of my jaw. When I open and when I close my mouth my jaw feels weird with a lot of pressure and sometimes even pops. When I make certain facial expressions the left side of my cheek indents more extremely than the right. (Makes me look boney.) Doctor: Hello,I can understand your concern. From the symptoms you have described, it is certain that you are suffering from temporomandibular joint problem, the joint near ear that helps in opening and closing of lower jaw along with side to side movements. When the joint has problems in the disc that acts as a cushion when the joint makes movements or if the muscles of mastication are affected, you can face such problems. have you had any accident that may have injured the side of the face? or are you having these troubles since long time? Injury to the joint can sometimes cause such problems.I would advise you to visit an oral surgeon who will examine the joint physically while you make movements, will take an x-ray to examine the associated structures with the joint. You might have to go for CT Scan and MRI, too. Based on the test results, specific diagnosis can be made and treatment can be advised. If the TMJ is affected, you might have to go for a surgical process to correct it. I hope this information helps you. Thank you for choosing HealthcareMagic. Take care.Best,Dr. Viraj Shah"
},
{
"id": 157066,
"tgt": "How to confirm blood cancer by laborotory tests?",
"src": "Patient: ok I went to my Dr. she did 9 different blood tests, when I went back I was told I had cancer in the blood. I was sent to hematology they did only ONE test basic cbc and told me I was fine. Should I request my Dr. do the tests again? If so how should I go about it. I am a 35 yo female with other medical issues such as epilepsy and heart problems just to name a few. Doctor: Good day! Blood cancer is a broad term and needs a complete hematologic profile assessment rather than a simple complete blood count. Let the doctor involve in to the matter and work up for thorough detailed examination.And see what comes out hoping for the bestregards"
},
{
"id": 54818,
"tgt": "Can Lipitor increase SGPT levels?",
"src": "Patient: hello Dr. today i did my blood test and i got the result as follow Cholesterol 224 S.G.P.T 87 SGOT is 41 the previuos blood test was Cholesterol 243 SGPT 46 sgot 36 im started to use a medicien to decrease the cholesterol since 45 days (LIPITOR-20) so , is it a cause to increase the SGPT level in the recent blood test ? if yes and if i have to stop the medicein what shall i do to adjust these readings ? Doctor: Hi thanks for asking question.Noted you are using lipitor that is atorvastatin tablet...You are taking correct drug for hyper cholesterol.Now with this drug your sgpt might raised.But just now lipid has to be controlled .so use atorvastatin...Regular follow up done for liver enzymes profile...Meanwhile take low fat diet with more fruits ....Avoid alcohol strictly. .If jaundice occurs immediately consult doc...Take care.Dr.Parth Goswami"
},
{
"id": 145531,
"tgt": "Suggest treatment for headaches when having degenerative disc disease",
"src": "Patient: I have degenerative disc disease and am suffering with severe headaches..I have neck devise that i did get and use every day...some exercises but lately, like 1 in the a.m. this morning, I have been awakened wtih severe pain my my head, especially behind the left eye.... Doctor: Hi,Thanks for writing in.It is possible that your symptoms of severe pain in the head and neck regions are due to irritation of nerve roots in the cervical spine. This is known as radiculopathy. It happens due to nerve root irritation due to slight movements including when you are asleep. This causes pressure on nerves roots and pinching of the nerves. This is followed by sudden shooting pain from the neck to the head and can also be experienced behind the eyes. Please avoid use of pillows when sleeping and sleep on a hard mattress. Avoid making sudden head and neck movements. If you feel severe pain at night then apply a little ointment containing pain reliever and muscle relaxant. Please discuss treatment available for degenerative cervical disc disease with your doctor and if surgery might help. Please do not worry."
},
{
"id": 211634,
"tgt": "Nauseated, dizziness, pressure sensation in head, sickness, heaviness in body. What does this suggest?",
"src": "Patient: Hi I was painting the ceiling of a kitchen a couple of ours ago with oil based killz primer and and became sick nauseous dizzy ,everything I went out side to get some fresh air n it didnt do much but was ok then went back in n thats when I felt Ilike I could even stand straight dizzy couldnt see right pressure on my head numerous feelings so I went outside fir 30min ate an apple n peanut s think it will make me feel better but not really so I tried to take a nap in my car for about 40 min n I felt better but when I went to drive I feel a bit sick again so I just ate a burger now its been 2hrs since I stopped painting n I still feel not right my body heavy I feel like im out if it not right like im zone out and dizzy n sick. Why what is wrong with me? Should I just sleep it out or take medicine ? Doctor: Hello, welcome to Healthcare Magic. You current problem due to poor neck posture during painting, fatigue and unhabitual inhalation of excessive paints vapor. It is better to take rest after some juice or milk. If symptom persist you should meet physician.Wish you good health and all the best. Regards, Dr Ashish Kumar Mittal www.99doctor.com"
},
{
"id": 222580,
"tgt": "How to ascertain that one can get pregnant?",
"src": "Patient: we had sex for the first time,n hymn broke n it started bleeding,v got scared n dint hv sex again,after having sex immediate next day i took meprate tablet do n no sperm got ejected as it was only two times we did n it bleeded,is there any chances of getting pregnent Doctor: Hallow Dear,The chances of pregnancy depend up on which day of the menstrual cycle the intercourse was performed. The egg is released 14 days prior to the next menses. The egg lives for 24 hours while the sperms are active for 72 hours. Hence a week around the day of egg release is a fertile period. If you had unprotected sex during this fertile window, you have got high chances of becoming pregnant. The best indicator of probable pregnancy is missing a period. All other symptoms appear a week or more after that. So if you miss your period, get your urine tested for pregnancy which will guide you. I hope this helps you."
},
{
"id": 215651,
"tgt": "Can Lyrica and Pamelor be taken together for pain in the legs?",
"src": "Patient: My son has Gillian barre AIDP pain in both legs and hands complicated by a fall on left leg resulting in broken patella and DVT. Now he has been diagnosed with RSD reflex sympathetic dystrophy in that foot. Pain specialist put him on a plan to manage pain but does not follow up or manage dosage or scheduling of meds. He has 4 doctors and no one seems to take lead on helping me implement the plan. He is on 175 mg Lyrica tid. which I try to give at 7 am 3 and 11 pm. I am to alternate 1000 mg of acetaminophen every 8 hours to prevent break through pain however if he is asleep I do not give him this but am afraid to give him this dose too close to Lyrica. I have weaned him off of oxycodone but when I introduced Robaxin 1000 mg every 6 hours and Cymbalta @ 30 mg to be increased to 60 mg daily and the palmelor 25 mg at bedtime he hit a brick wall. He\u2019s either sedated to the point he is dizzy tachycardic and short of breath when I try to wake him for physical therapy or transfer him from wheelchair or in terrible pain and unable to sleep. Right now I stopped all meds but Lyrica but just started to introduce others toward evening to promote his sleep at night so he is functional for therapy in daytime. I gave one dose of acetaminophen 4 hours after last Lyrica dose robaxin 2 hrs after that and want to give nortrytiline and Lyrica dose near 11 pm. Can I give Lyrica and Pamelor together? Could I try giving 30 mg cymbalta tonight with these two meds? We are desperate for pain relief and sleep. Do we go to ER for help with pain management? We have been 3 days trying to get answers from 4 doctor offices Doctor: Hello, You can safely take Lyrica and Pamelor together. They will not interact with each other or affect the efficacy of one another. Hope I have answered your query. Let me know if I can assist you further. Regards, Dr. Shinas Hussain, General & Family Physician"
},
{
"id": 51074,
"tgt": "Kidney stones, have fluttering feeling on the left of back. Cause?",
"src": "Patient: Hi. I am a 34 year old female, I m 5 5 , 120 lbs., active (especially since I have a 3 year old-haha). I have been having pain intermittently since before I was pregnant with my son, on both sides of my stomach in the kidney area. It is a sharp pain that comes in waves, no blood in urine . I have had a history of UTIs, but haven t had one in maybe two years. I went to a general practice doctor last year about the pain, and she ordered an ultrasound of my right side (I am not sure why she didn t ultrasound both sides), and she said that I had kidney stones , but they were small and they should pass, she didn t request that I follow up with her. I have continued to have pain on both sides, though not as often. I have recently been experiencing a fluttering feeling on the left side of my back, almost like a muscle spasm , but I know it s not a muscle spasm. I am fairly certain it is my left kidney. Why is this happening? Doctor: Hi, Welcome to HCM, After going through your history it is possible you had small stones on right side ,now you have some uriteric colic on left side as you suffered kidney stones ,if you have tendency to have stone formation . Please check with Uro surgeon and get your Ultra sonography repeated so as to find if you have it some times it is possible clot is there or stricture is there. You should also check Uric acid and calcium level Urine R&M. Please take plenty of water and avoid dairy products for time being till you get result of investigations. Please do not worry if any thing is there it can be treated easily. Take Care. Good Luck."
},
{
"id": 68708,
"tgt": "What causes lump behind left ear with fever?",
"src": "Patient: My 7mth old son has had a fever for 6 days now and eats but not very much. He also just this morning has a lump that is red behind his left ear and is tender. we have taken him to the doctor and his ears and chest and throat are fine. The doctor has put him on amoxicillin, but I'm concerned because he had the exact same symptoms 1 month ago with the lump and everything? what could we be dealing with? His fever and lump went away a month ago about a day and a half later. Doctor: HiFever is probably viral which will subside in a week or so, lump is probably enlarged lymph nodes which will get back to normal once the fever settles. Keep him well hydrated, give frequent feeds and antipyretics as prescribed and keep a watch on fever. Hope this helpsRegards"
},
{
"id": 205932,
"tgt": "What causes body tremors after decreasing the dosage of Prozac?",
"src": "Patient: My psychiatrist cut back on my Prozac to 20 ml and added effexor 37.5. I went to see him last Thursday for a follow up and he told me to cut back Prozac to 10 ml and to increase the effexor the problem is he only called in the 10 ml of Prozac to my pharmacy and I cant get in touch with him because its thanksgiving, I dont know what to do about the effexor, and today and yesterday I have been feeling not too good ? kinda shaky and agitated I do have some more effexor should I just increase it to 75ml? Doctor: HII had gone through your query.It is advisable to withdraw drug gradually while you increases doses of another medicines.So sudden cut down in prozac lead to withdrawal phenomena and caused tremors.Slowly increased doses of effexor are advisable. As per your doctors advise you have to stick on 10 ml of prozac till effexor starts working properly.Mild dose of benzodiazapine is also helpful to control withdrawal effect of prozac in initial few days.Take your treating doctors advise and follow it for better treatment.I hope i have answered your query.Thank you."
},
{
"id": 223609,
"tgt": "Can I be pregnant after taking Noriday contraceptive pill?",
"src": "Patient: Hi there, How long do you have to be taking Noriday contraceptive pill to be protected if you dont start taking it the first day of your period initially. I started taking it at another time in the month but took it for 7 days straight before I had unprotected sex, am I covered? Regards, Laura Doctor: Hello, and I hope I can help you today.Typically, women are advised to start birth control pills within seven days of their last menstrual period because it is unlikely that she would've already ovulated at that time. If you start the pill later in your cycle, the process initiates ovulation may have already begun and your body may produce an egg that is capable of being fertilized that cycle.However, even if you ovulated right when you started to take the pill, it is unlikely that the egg would still be viable to be fertilized after seven days. Therefore, waiting seven days before having unprotected intercourse most likely was enough time to ensure that you would not become pregnant.Modern pregnancy tests will become positive within 10 days to two weeks after conception, so if you do not have a menstrual period at the end of your pill pack, you should perform a urine pregnancy test to be safe. However, if you start your pill later than one week after the start of your period, you may not actually bleed the first month during the pill free week.I hope that I was able to adequately answer your question today, and that this information was reassuring.Best wishes,Dr. Brown"
},
{
"id": 176242,
"tgt": "What could cause a small lump on the jaw below skin?",
"src": "Patient: hi, we just had a baby. it s our first baby. we have just noticed a small lump below the baby s jaw and under the skin. the size of the lump is similar to a size of a small peanut (but soft) and it does not move from it s current position. do you know what that lump might be and what can we do about it? we want to try with the simplest process first to make it disappear. Vijay Doctor: Thanks for your query, I have gone through your query.the swelling on jaw can be because of the abscess secondary to tooth infection or it can be a garres osteomyelitis. nothing to be panic, consult a oral physician and get it evaluated. You need to take radiograph like OPG to confirm the diagnosis.,I hope my answer will help you, take care."
},
{
"id": 102940,
"tgt": "Body twitches, terrible taste in mouth, short term memory. Muscle in intestine is under stress. Spreads?",
"src": "Patient: I was a patient at Mayo Clinic untill last week, they could not determine what is wrong with me. They ruled out Parkinsons, ALS, and epilepsy. I have had MRI Brain shows one mass they assume is from a trauma, rest of spine is fine. All nerve and every test they could run showed normal. Now the process has turned to treating symptomatically, and when I lay down to take a nap (chronic fatigue) my body is beginning to quiver and twitch much more. Occasionally my entire body will make sudden twitch,,feels something like a sneeze building. I have a terrible taste in my mouth when things are going to start. I have intense loss of balance, short term memory as well. Blood tests were ran as well with elevated voltage gate potassium, ferratin, Kreatineine w/e gfr. Determined was not restless leg syndrome through sleep study. It started with intense intestinal pain, gastroenterologist determined the small muscles in my intestines were under stress and cramping causing the pain. Prescribed Hyoscamine that works very well. It seems to be spreading through my body. Any ideas?? Doctor: i suggest you to consult allergeologist to get blood serum tests for specific antibodies for food proteins like milk wheat potato and other foodas suddenly food proteins start reacting with body proteins to cause these types of thingsi advice to withdraw those food proteins which can be helpful to you"
},
{
"id": 124645,
"tgt": "What causes pain at the top of left calm muscle?",
"src": "Patient: Hi I am experiencing pain at the top of my left calm muscle. I play soccer, and I only experience the pain when I bend the knee with my weight on it especially when running backwards. When it happens I can feel my body collapsing upon the knee. Is it a muscle or tendon problem? Thanks. Doctor: Hello, As a first line management, you can take analgesics like paracetamol or aceclofenac for pain relief. If symptoms persist, it is better to consult a physician and get evaluated. Hope I have answered your query. Let me know if I can assist you further. Regards, Dr. Shinas Hussain, General & Family Physician"
},
{
"id": 58709,
"tgt": "Pain under left breast, started after having food, drank 7 cokes. Had gallbladder removed. Any ideas?",
"src": "Patient: started having pain// cramping after dinner last nite, I thought it was trapped gas , the pain is under my left breast ,today I drank 7 hot cokes and had little gas ,burp from that now when I drink or eat anything it hurts even swolling water it hurts if I take a deep breath in? I don't know what it could be , I had my gallbladder removed in oct 2012 so I don't know Doctor: Hello Good Evening,You are suffered from bloating, which may cause constipation due to decreased bowel movements and constipation may also caused bloating.Frequent burping may results of indigestion and bloating.There are many causes of bloating, including: diet, irritable bowel syndrome, lactose intolerance, re flux, and constipation and some more.Gas and bloating is a sign that food is not being digested correctly by the body. An inadequate intake of water will cause excessive stomach bloating.Water benefits the body by aiding with digestion because it supports a majority of the body's daily functions. Fatty foods cause a formation of fat cells to develop throughout the body and contribute to bloating as well. A build up of fat cells slows down the body's ability to empty the stomach.Dairy products also contribute to excessive cramps, gas, and bloating.Persons who are intolerant to lactose products experience this effect more than others.Following steps should follows to lessen the bloating:-Walk after eating a meal.-Exercise-Avoid coffee and chocolates.-Avoid Meals that are high in fat.-Avoid extremely hot or cold food.-Avoid bubble gum or bubbly beverages. If needed then medication should be included antacids, PPI, domperidone, Levosulpiride (prokinetic), digestive enzymes and some laxatives.Regards,Dr. Naresh Jain."
},
{
"id": 63521,
"tgt": "What is the small lump below the scar after lipoma removal?",
"src": "Patient: my 10 year old son had a lipoma removed 2 months ago, on his right leg, just below the knee. it was benign. the scar has healed perfectly, but i can feel a small lump under the scar line. i was hoping never to feel a lump there again, as i was worried for so long. what is this lump? Doctor: Hi,Dear I studied your query in all it details and I understood your concerns.Cause and Remedy - On whatever limited facts given from you about the lump under the scar ,it seems to be resolving post-surgical left out hematoma with progressive fibrosis .Hence To reduce your worry Please consult for opinion from ER Surgeon .Plz hit thanks and write excellent Reviews if this would resolve your query.Plz don't worry and do Welcome for any further query in this regard to me.Have a Good Day.Dr.Savaskar M.N."
},
{
"id": 131280,
"tgt": "Unable to use the right side of face. Is it a symptom of Bell's palsy?",
"src": "Patient: HI four days ago I thought I was getting a toothache but the more i looked at my sitituation the more worry i have , I cannot use the right side of my face .from being unable to wink or blink my eye waters up , i cannot flare my nostril and opening my mouth the right side cannot open as wide as left I have trouble drinking from a bottle I use a glass or straw. did i have a stroke or is it Belles palsey Doctor: it's Bell's palsy.get yourself checked for middle year infection(one of the common reason).on internet you will find a list of exercise for palsy,you need to do them by standing in front of mirror.take care of your eyes,place cotton cloth on the affected eye while sleeping to prevent dirt from entering.take physiotherapy treatment for faster recoveryHope I have answered your question. If you have any further questions I will be happy to help"
},
{
"id": 187684,
"tgt": "Is it normal to have sore, red tonsil after cavity filling?",
"src": "Patient: Hi, I recently had a cavity filled on the upper left part of my mouth. It's been two days since the procedure, and I still have sever pain and tenderness in my cheek where I'm guessing they put the novacain. I also am getting a burning sensation in my left ear and sore, red swollen tonsil on my left side. Again, it's been two days since the procedure. Is this normal? Doctor: Hi and thank you so much for this query.I am so sorry to hear about these symptoms that you have reported. It is not normal too have a swollen and painful tonsil after dental work. I will suggest that you return to the doctor for further assessment and management. This could represent tonsilitis that needs to be managed some what differently.I hope this addresses your query fully. Thank you so much for patronizing our services and please do feel free to ask for follow up clarifications and information if need be. I wish you the best of health.Dr. Ditah, MD."
},
{
"id": 140583,
"tgt": "How safe is taking Rericap tablets for improving concentration?",
"src": "Patient: hi i am 35..at present studing for my higher ranks..some days back some one suggest me to take rericap tablets to increase the memory...since due to age i cannot remenber much what i read. so should i take rericap tablets..if yes then how much in a day. Doctor: Hi, Raricap is a formulation of a medication produced by Bafna Pharma in India. It contains Calcium Citrate, Iron 50 mg, and Folic Acid 0.3 mg. It is indicated for use in lactating women as well as those women who have anemia while pregnant. I can see where someone may believe this to be a MEMORY supplement because of the folic acid element and possibly iron. I would caution you not to take too many of these capsules daily due to the iron content as this could change your bowel habits and you may become constipated. The calcium citrate can also cause side effects that are not pleasant if taken in excessive doses. If you feel your memory is not functioning at peak performance there may be a better solution than to take something like Raricaps though this appears to be relatively safe if taken as directed by a physician. Otherwise, see a doctor who can evaluate you for more specific causes of cognitive/memory dysfunction such as B12 or vitamin D deficiencies, underperforming thyroid gland, as well as other hormonal imbalances). Hope I have answered your query. Let me know if I can assist you further. Regards, Dr. Dariush Saghafi, Neurologist"
},
{
"id": 78355,
"tgt": "What causes chest pain,faster heart beat and arm pain?",
"src": "Patient: I am Mei, I have experience this couple of days after eating a very short while, I feel my heart beat faster chest pain, back side of left arm pain. Seem short of breath. it last about half a n hour to one hour and gone. Specially at lunch time. What can I do? Doctor: Thanks for your question on Health Care Magic. I can understand your concern. Your symptoms are seen after eating, so possibility of GERD (gastroesophageal reflux disease) is more. But better to rule out heart diseases first, because all these symptoms are commonly seen in heart diseases. So first get done ecg, 2d echo and stress test (trade mill test). If all these are normal then no need to worry for heart diseases. You are mostly having stress induced GERD. It is due to laxity of gastroesophageal sphincter. Because of this the acid of the stomach tends to come up in the esophagus and cause the symptoms of chest pain. So avoid stress and tension. Be relax and calm. Take proton pump inhibitors. Avoid large meals, instead take frequent small meals. Don't worry, you will be alright, but first rule out heart diseases. Hope I have solved your query. Wish you good health. Thanks."
},
{
"id": 142092,
"tgt": "What to do if having lightheadedness, occassional dizziness, imbalance, pain in the left arm?",
"src": "Patient: hello i seem to be rather light headed at the minute with occasional dizzyness which can also affect my balance at time but it takes a short while sometimes to focus on some things. at the same time i have been feeling an aching down my left arm which can be quite painful Doctor: Hello!Welcome on Healthcaremagic!The dizziness could be related to different possible causes: an inner ear disorder, cervical spondylarthrosis, orthostatic hypotension, etc.. It is not related to the pain in your left arm. The pain in the arm could be related to a pinched nerve. If the problem persists, a nerve conduction study may be necessary. I would also recommend consulting with an ENT specialist to exclude an inner ear disorder. Hope you will find this answer helpful!Kind regards, Dr. Aida"
},
{
"id": 107769,
"tgt": "Suggest treatment for severe back pain and fatigue",
"src": "Patient: I have left erector in constant spasm, causes severe pain/fatigue across lower back. When I try to walk, bend, or stairs becomes unbearable. Sitting/ lying relieves. Now I am experiencing sob when back is tight / painful, very sob, have to sit. I have limited scleroderma, why sob, can this have connection to scleroderma? Doctor: Dear- I reviewed your question with care and will try to help you with my medical advise.Scleroderma is a diffuse disease characterized for hardeness of the connective tissue and back. It is a systemic condition which means that can affect any part of the body,including your lungs. You might be having shortness of breath because of this too. The back pain and spasm is probably secondary to the Scleroderma too. I recommend you to see a rheumatologist to evaluate the status of the disease and have xray of the low back too .I hope that my advise has been helpfulDR.Sara"
},
{
"id": 157553,
"tgt": "Diagnosed with breast cancer. Hormonal therapies have no success. Suggested radiation therapy. Any other solution?",
"src": "Patient: 83 year old woman with locally advanced breast cancer.Diagnosed 2006.Tamoxifen given and was effective for 6 years.Since then two other hormonal therapies have been given without success.The doctor has said chemotherapy i s not an option but will give radiation therapy .In addition to radiation are there any other options which can be given.Should hormonal therapy be tried again. Doctor: Hi and welcome to HCM,thank you for your query.there are 4 possible treatment methods,surgery,chemo,radio and hormonal therapy. Surgery is obviously not an option. Unfortunately it seems like tehre wasnt good reaction on hormonal and chemo as well. So radiation is the last option and honestly,success rate is not very positive.If hormonal therapy wans succesfull once,it probably wont be successful if done again since it means that this is not cancer estrogen dependent. i hope that radioth will show good results.Wish you good health. Regards."
},
{
"id": 36964,
"tgt": "Will getting pricked by very sharp can cover cause infection?",
"src": "Patient: if I get pricked by a tuna can while picking it up, will I get infected?I am working at I found somebody left a couple of empty tuna cansI try to pick it up and it pricked on my pinky. The tuna can cover are sharp.Very sharp. Will I get infection? My pinky is not bleeding though Doctor: It is always possible to get an infection from a puncture injury. If it was not severe it is probably unlikely. It would depend on how contaminated the cans were. Clean the area well and keep an eye on it the next couple of days. If redness, heat, swelling, or tenderness develop, that might indicate infection. Also be sure you are up to date on your tetanus immunization. If you've had a tetanus shot in the last 10 years, you are okay, otherwise you should get one. Hope this helps."
},
{
"id": 58733,
"tgt": "Prescribed Udiliv, Nexium for fatty liver, had some red meat. Had palpitations, gas, pain in waist. What to do ?",
"src": "Patient: i have been prescribed udiliv 300 and neksium 40 as i am suffering from non-alcoholic fatty liver.tonight i ignored my doc's recommendation and had red meat for dinner.decent amount followed by ice cream.things were fin till 4:30 am after which i went off to sleep and woke up.as in the moment i would go into deep sleep i would wake up it happened twice followed by palpitations and breaking wind i had my meal roughly 7 hours earlier and i haven't slept.i am getting gas discomfort in my left side of the waist at times in the front and at times at the chest armpit.i took a rivotril .5mg to sleep.but am unable to do so as the discomfort is irritating me and making me turn sideways.can i take neksium as well as udiliv to relieve myself.Also for future what is the minimum time before i can take the dose of neksium and udiliv. Doctor: Hi, Thanks for choosing HCM You can take neksium everyday for one month as you are experiencing gastritis without it. Since you have fatty liver, try to avoid fatty foods like red meat. Take neksium twice daily once in twelve hours for one month. Take udiliv once daily for at least 3 months. Hope this helps you Take care"
},
{
"id": 181504,
"tgt": "Suggest remedy for oral thrush while on Flovent",
"src": "Patient: I have thrush, have been sick with The flu and a cold for ten days and using a Flovent inhaler twice a day. My throat has been sore but I was so miserable I didn't look in my mouth. I just started feeling better and realized this is going on. Is there a home treatment I can start tonight? Doctor: Hi..Welcome to HEALTHCARE MAGIC..I have gone through your query and can understand your concerns.As per your complain Flovent contains steroid Fluticasone and use of steroid inhalers can cause oral thrush as reduced immunity caused by steroids can cause overgrowth of opportunist organisms like fungus..You should consult an Oral Physician and get evaluated and he can evaluate you clinically and in case if thrush is confirmed he can advise you antifungal mouthwash like Nystatin Mouthwash and clotrimazole lozenges..You can take ample of probiotics like Yoghurt and garlic in diet..Drink plenty of water and maintain a good oral hygiene..For sore throat do warm saline gargles and steam inhalations..Take anti-inflammatory painkiller like Ibuprofen..Avoid spicy and chilled foods..Suck throat lozenges.. Hope this information helps..Thanks and regards.Dr.Honey Nandwani Arora."
},
{
"id": 127679,
"tgt": "Suggest treatment for severe arm pain despite a normal X-ray report",
"src": "Patient: Hey I went to the hospital and I got an x Ray coz I fell on my arm earlier and they went through the x Ray and said they couldn t find anything but there might be a broken bone and my arm really hurt but now it s better and I can move it is it a sprain or bruise c Doctor: Hello and Welcome to \u2018Ask A Doctor\u2019 service.I have reviewed your query and here is my advice.In my opinion, if the X-ray is normal you can ice the part with analgesic and give rest to the part with an arm pouch. Hope I have answered your query. Let me know if I can assist you further.Regards, \u00a0\u00a0\u00a0\u00a0\u00a0Dr. Fahim Sheik"
},
{
"id": 225887,
"tgt": "Polycystic Ovary Syndrome. Take Duphaston. Had unprotected sex. Recommendations on taking Duphaston, Nordette",
"src": "Patient: I'm a 17 year old girl with Polycystic Ovary Syndrome.I got my period last year, I used to take Duphaston to regulate it.I had sex yesterday for the first time and it was unprotected. I've been told I have low chances of being pregnant but I'm very worried. This seems dangerous and risky but would it be bad if I started taking Duphaston again? Take one pill a day like how I was prescribed so I can get my period? Or will that just increase my chances of being pregnant?And would it be alright to take the day-after-pill Nordette? Doctor: Hi,Thanks for posting your question here, I will try to answer it to the best of my abilities.Duphaston is given to women with fertility issues, it is a synthetic hormone designed to replace the one your body isnt producing properly.Yes, if you start taking it you will get your period regularly and that increases your chances of getting pregnant. But that is better than having irregular periods and having intercourse when you didnt know it could get you pregnant.So keep using it and ask your boyfriend to use a condom.Yes, you can use nordette as a contraceptive pill, but I recommend you talk it over with your doctor once.I hope this answered your question."
},
{
"id": 17713,
"tgt": "Suggest treatment for paralysis in left side of body after heart attack",
"src": "Patient: My brother is suffering from physical disorder. His left side was paralysed after severe heart attack, He can t walk properly. Recently he started physiotherapy but no significant improvement is observed. The heart attack happened six years back. Is there any solutions so that he may be normal. ? Doctor: Hello, After going through your medical query I understand your concern and I would like to tell you that for left-sided paralysis a proper evaluation is required to check if it is an Infarct or brain hemorrhage. Kindly get an MRI brain done and consult a neurologist for further management. Hope I have answered your query. Let me know if I can assist you further. Regards, Dr. Bhanu Partap, Cardiologist"
},
{
"id": 175732,
"tgt": "What causes on and off red colored rashes all over body?",
"src": "Patient: my 17 month old was prescribed distaclor and as a result of a side effect from it a rash broke out all over her body. (7 days ago). she was put on zirtec to control the rash. the rash is a lot better but after 7 days continues to re-appear and disappear(not as bad).one minute her face is completely rash free the next there are a few small red rash marks in different areas each time. is this common for the rash to disappear/reappear? Doctor: Hi...by what you quote I do not think this is drug allergy. Drug allergy doesn't come again and again like this even after discontinuation of medicine. By what you quote it should be an urticarial or a simple skin allergy. Most important thing to be remembered is that it has a propensity to recur (called as second crop) within 10-14 days. If this happens, you can start using the same medicine but I suggest you get the kid evaluated with your paediatrician.Hope my answer was helpful for you. I am happy to help any time. Further clarifications and consultations on Health care magic are welcome. If you do not have any clarifications, you can close the discussion and rate the answer. Wish your kid good health.Dr. Sumanth MBBS., DCH., DNB (Paed).,"
},
{
"id": 135244,
"tgt": "What causes pain, redness and swelling after flu shot?",
"src": "Patient: I got both the flu zone and the pneumonia shot on Sept.8,2014. I ve already been to the Drs. and was told to take Advil for a severe swelling and pain and redness had two shots one arm at local pharmacy. No fever Next day a large lump on left side of neck shoulder area appeared, has subsided. I have another appt. tomorrow,not better yet, completely out of sorts since shots, complete brain fog. The pain in left arm(intense), numbness, tingling some facial nerve pain and some loss of motion, left arm. Pain in back of right arm pins & needles in hands, weakness ,can t sleep. Was told years ago had Fibromyalgia. Doctor: Hi! Your problem requires a pain killer stronger than Advil. I suggest that you visit a doctor and ask for these 2 drugs, Desvenlafaxine and Tramadol. These two drugs are SNRI's that have a powerful effect on the psychological perception of pain, in addition to this Tramadol is a synthetic opiate that relieves the emotional aspects of the pain that you describe. You need a prescription for both these drugs. Kindly follow these suggestions. You'll get better. Thanks, Dr. Neel Kudchadkar"
},
{
"id": 70720,
"tgt": "Is Benzonatate more effective than Prednisone in treating severe cough with clear mucous?",
"src": "Patient: I have a deep cough that raises clear mucous. I have been xrayed and know there is no pneumonia or bronchitis. I do not have a history of asthma. My doc prescribed a week of prednisone which I have just started. But I wonder if Benzonatate might be helpful for this? Doctor: Hi, Benzonatate is a cough suppressant drug. Prednisone is an oral corticosteroid. Prednisone is effective only if it is an allergic etiology related cough. Since you don't have asthma or bronchitis, I don't think your cough will improve with prednisone. Better to take Benzonatate first as it is cough suppressant. It will definitely reduce your cough. Hope I have answered your query. Let me know if I can assist you further. Regards, Dr. Kaushal Bhavsar, Pulmonologist"
},
{
"id": 218362,
"tgt": "Does Primolut N affect a pregnancy test report?",
"src": "Patient: I was taking primolite n , 2 days from last pills i did pregnancy test , it was negative , shall i belive it or i need to repeat the test after while to make sure? Is this pills affect the the pregnancy blood test? Doctor: Hello and welcome to \u2018Ask A Doctor\u2019 service. I have reviewed your query and here is my advice. tablet primalut N dont have any effects on Your pregnancy test reports. You have to do pregnancy test one week after missed periods. Hope I have answered your query. Let me know if I can assist you further."
},
{
"id": 24011,
"tgt": "Is angiogram required if having a block in the heart?",
"src": "Patient: iam a heart patient having MI 2 years back. A few wweks ago i ave undergone stress test and the doctor opeined that ihave still block and recomended to take angeogram. my stress test shows that i have excellent exercise tolerance. I used to walk briskly for about 45 minutes daily and have no problem. please have your valuable advice Doctor: Hi,Thanks for writing to Health Care Magic, I am Dr Asad Riaz, I have closely read your question and I understand your concerns, I will hereby guide you regarding your health related problem.A block in the arteries supplying blood to the heart muscle usually manifests as having chest pain or ECG changes on exertion and heart attack as an ocassional grave consequenceIf your treating doctor saw sone hanges in your ECG upon exercise or if you developed any chest pain, it may predict upto 75-80% accuracy the existence of a blockage in those vessels.If you get it looked at, as your doctor advised, and get it managed by placement of stents, it may prevent the development of another heart attack. So yes, you should absolutely get it doneI hope this answered your question, if you have more feel free to ask.Regards.Dr.Asad Riaz.General and Family Physician.."
},
{
"id": 202385,
"tgt": "Will wound in glans cause permanent scarring after healing?",
"src": "Patient: hi there, 3 days ago i accidentally scratched my bf's glans while having sex. He has 2 little wounds, one of them was a superficial scratch (more like a skin irritation) and the other one bled when it happened. Both look red right now, and the wound that bled has a scab on top. I would like to know if glans wounds heal properly as any other wound on other parts of the body, and dont leave a scar. He is afraid this wounds would create permanent scarring. Also, do you recommend use of vaseline or any other moisturizer for the healing scab? Thank you! Doctor: Thank you for your question. I don't think this scratches and abrasions are deep enough. Glans is highly sensitive and vascular structure , so it bleeds on minor trauma. This superficial wounds heals predictably in 7-10 days. Don't peel off the scab already formed, use a antiboitic oinment preferably neomycin group in thin film until scab seperates itself bridging the wound with regenerating mucosa."
},
{
"id": 179904,
"tgt": "Suggest treatment and diet for dermatomyositis",
"src": "Patient: hi sir. my12 year doughter suffer dermeto myositis .cunsulted doctor says take long period medicin alonng 3 yers .i think thats make another side effects. iam vorried my dougters helth pleas give me compleat pictur of this desces and food selection. what type of avoid please helpme urs faithfully Abdul wahab Doctor: Thanks for putting your query up at HealthCareMagic. I am sorry that your daughter has dermatomyositis. Dermatomyositis (DM) is a connective-tissue disease that is characterized by inflammation of the muscles and the skin. While DM most frequently affects the skin and muscles, it is a systemic disorder that may also affect the joints, the esophagus, the lungs, and, less commonly, the heart. The mechanism is immune-mediated damage of microscopic vessels of skin and muscles and gives rise to muscle atrophy. The diagnosis can be confirmed by muscle biopsy, EMG,and blood tests. This disease has no known cure but the immune reactions that result in the disease, can be kept in control using certain medicines. She therefore needs the medicines and it is no need to worry about side-effects as without them the condition would worsen even further.Dermatomyositis is often associated with celiac disease and hence a gluten-free diet is recommended. In other words avoid wheat based products altogether. It is believed that certain dietary regimens like Paleo Diet are beneficial.Okay To Eat: Fruits, Vegetables, Lean Meats, Seafood, Legumes, Nuts & Seeds and Healthy Fats having omega-3 fish oil.Avoid: Dairy, Grains, Processed Food & Sugars, Starches and Alcohol.Eating your food as close to raw as possible is recommended.Hope that helps. Feel free to revert back with further queries if any."
},
{
"id": 116455,
"tgt": "Suggest remedy for a plastic anemia",
"src": "Patient: Age 58 Male,suffering from cold for past 4 weeks was having medicine prescribed by family doctor, after which we went for CBC from which we came to know that WBC had gone to 1000 and RBC was low & platelets were normal after which we consulted a hematologist who had done bone marrow test by which we came to know that it is aplastic anemia but after that he didn't gave medicine and after a week today we did CBC again which is showing WBC count 900 rest is ok...... now the doc. gave one capsule to take for 15 days and see if it is curing by that can someone help me with this case as i am really tensed and not getting what is this all about ???? can someone tell me what is this diesease ? what is the cure ?????? and how to cure it ????? please share your experience i am really in need of your advice Doctor: Hi,Thanks for asking.Based on your query, my opinion is as follows.1. Aplastic anemia is a bone marrow disorder commonly due to megaloblastic anemia.2. Vit B12 should show improvement if it is secondary to megaloblastic anemia. 3. If oral Vit B12 is not helping, Vit B12 shots will be helpful. Take a nutritious diet. If other cells are responding, this will also improve. If you have any infection, better to take a granulocyte infusion till WBC improves to avoid risking any critical infections. Hope it helps.Any further queries, happy to help again."
},
{
"id": 17266,
"tgt": "Is thrombolytic therapy as treatment a safe option?",
"src": "Patient: Q1.) What is the rationale for selecting PCI and thrombolytic therapy as treatment option? Q2.) What are the various co therapies you use and why? As in unfractioned heparin or enoxaparin? Q3.) What set of patients, conditions will benefit from pre hospital thrombolysis? Q4.) What facilities are required in ambulance to intiate pre hospital thrombolysis? Doctor: Hello, After going through your medical query I understand your concern and I would like to tell you that you can go for thrombolytic therapy but hypotension can be a risk, and an angiogram will be required after thrombolysis. Hope I have answered your query. Let me know if I can assist you further. Regards, Dr. Bhanu Partap, Cardiologist"
},
{
"id": 144729,
"tgt": "What is the time period for recovery from brain aneurysm?",
"src": "Patient: Hi my mother had a brain anneursym on jan 15 2914 she was showing improvement a month in mouthing words and moving around and responding when told to open or close eyes. she was put on a trach and they bounced back and fourth with the vent and trach and she is now in rehab and had phenomia and ulser bleeds. she is responding some and also had a craniotomy I waqs wondering since she isnt responding as much is this normal for what she has gone through. Doctor: Hello!Thank you for asking on HCM!I understand your concern and would explain that her symptoms seem to be related to the brain hemorrhage caused by the aneurysm rupture. You should know that it is needed a long time (months up to a year), to recover from such a disorder. After the first year, the possibility of improvement is too low, and the situation remains the same. This is because during the first months, the blood is being absorbed and the there is inflammation in the surrounding tissues. But after this long period, the deficits which remain are caused by the damaged brain cells, which do not regenerate. The situation, complicated with pneumonia and ulcer bleeds is really concerning and needs a careful treatment in specialized medical centers. They could be related to aspiration and the therapies she has been taking. Hope to have been helpful!Best regards, Dr. Aida"
},
{
"id": 63573,
"tgt": "What causes lump on top of calf?",
"src": "Patient: I have a dime size lump that came out of nowhere at the top of my calf 3 days ago. It is located on the outside edge of my calf (think of my calf as a tall rectangle and this would be in the far NorthWest part of that rectangle). It is sore only when I touch it; otherwise I tend to forget it's there. Are there lymph nodes on Doctor: Hi,Dear,Thanks for the query to HCM. I studied your problem in depth and I understood your concerns. Cause and plan of Treatment-In my opinion on the given data,you seem to have bug bite or may be a boil / or may be a lymphnode from wound in the leg.To fix the diagnosis you need to consult ER Surgeon.So don't build up wrong concepts and self medicate your self which would increase risks and costs to you.Instead just ask a query to HCM and be comfortable to resolve your health issues.Welcome for any more query in this regard to HCM.Write good reviews and Click thanks if you feel satisfied with my advise.Have a Good Day.Dr.Savaskar M.N."
},
{
"id": 206070,
"tgt": "Suggest treatment for personality disorder",
"src": "Patient: senior 66 210 5 7 with myriad of symptoms blood test muscle biopsy positive . for some type of muscle illness . .and all all many test about 15 with all specilalist even a hiv test negative i ve had test for everything neuro endo don t have symptoms of dermamyotosis .. positive for hypothyoid . hive looking rash near elbow i am diabetic with a personality disorder bipolar was taking 20 pills a day . stopped most except bipolar diabetic thyroid meds . i can not list all my symptoms about 19 what is your opinion is this in my head. lupus no no on everything Doctor: Hello,Thanks for choosing health care magic for posting your query.I have gone through your question in detail and I can understand what you are going through.Personlaity disorders require intensive psychotherapies like psychoanalytic psychotherapies or dialectical behavioural therapy. Hope I am able to answer your concerns.If you have any further query, I would be glad to help you.In future if you wish to contact me directly, you can use the below mentioned link:bit.ly/dr-srikanth-reddy\u00a0\u00a0\u00a0\u00a0\u00a0\u00a0\u00a0\u00a0\u00a0\u00a0\u00a0\u00a0\u00a0\u00a0\u00a0\u00a0\u00a0\u00a0\u00a0\u00a0\u00a0\u00a0\u00a0\u00a0\u00a0\u00a0\u00a0\u00a0\u00a0\u00a0\u00a0\u00a0\u00a0\u00a0\u00a0\u00a0\u00a0\u00a0\u00a0\u00a0"
},
{
"id": 152988,
"tgt": "Would CGI or TIP chemo regime be most effective for bladder cancer?",
"src": "Patient: I was recently diagnosed with squamous cell T4 bladder cancer. I m a 62 year old male paraplegic, 175 lbs and never a smoker. I also never had an indwelling catheter. I had a radical cystectomy and ileal diversion. The pathology report stated that all of the tumor was removed and all of the lymph nodes removed were negative. There was however evidence of lymphovascular involvement. I have been recommended both a CGI or TIP chemo regimen. Which would be most effective. I have no evidence of metastisis. Doctor: Hello you have good postop report except one high risk factor. in case of bladder cancer advanced stage-post cystectomy high risk features are present adjuvant chemotherapy can give survival benefit. Both regimens are effective but TIP is the more frequently used combination. toxicity profile of both regimens is almost same as two regimens differ in only one drug . CGI can cause more decrease in blood cell count and TIP can cause more of nerve toxicity. Hope this answers your question."
},
{
"id": 5635,
"tgt": "Trying to conceive. No positive result. HSG done. What does report suggest?",
"src": "Patient: i am 27 year old i done my HSG last week its (HSG reports shows that UTREUS :Normal, FALLOPIAN TUBES:Normal In Caliber .FREE PERITONEL SPILL OF CONTRAST:Bilateral spill is noted IMPRESSION: Normal study ) What does it indicate ? and I got married May 20th 2011 its completed 1year 9 month but till now no issues..no pregnancy, pls explain Doctor: Hi, Welcome to HCM, You must understand that HSG is done to find out patency of Fallopian tubes which is necessary for pregnancy to occur this is good that your tubes are patent. The cause for not able to conceive may be other then tubal block there are many causes of failure to conceive. You should consult infertility expert Gynecologist and under go tests for both the partner as many times there may be problem with the semen analysis of male partner. Complete examination and hormone tests like thyroid function and prolactin with TORCH, and other test will find out the cause. If every thing is fine then folicular study with proper ovulation and proper counselling it is possible to be successful in getting pregnant. Now a days assisted reproductive is very advance and with good success so do not worry be positive. Take care Good Luck."
},
{
"id": 136175,
"tgt": "What is the remedy for the swelling and pain in the shin area?",
"src": "Patient: I was hit in the shin with a softball and it swelled right away but I continued to play. The swelling hasn t gotten much better and now extends to my ankle. I have pain tolerance so not sure what to think might be causing it. I have been hit in the same spot several times before but never had the swelling go to my ankle. My insurance premiums are very high so prefer not to go to the er if I don t have too. Can you help me? Doctor: Hi,I have studied your caseThere can be bony contusion.As x ray may be essential to rule out fracture.I can not compare swelling, but if swelling is there, you need to take rest and use splint or compressive crepe bandage to reduce swelling.Watch for increase in redness, infection.Take precaution to avoid infection around hematoma.Do not use hot fomentation or massage.If required you may need to do blood investigation for prognosis and recovery.Hope this answers your query. If you have additional questions or follow up queries then please do not hesitate in writing to us. I will be happy to answer your queries. Wishing you good health.Take care"
},
{
"id": 43519,
"tgt": "Trying to conceive. Planning for test tube baby. Prescribed dronis 30. Why dronis 30? Suggestions?",
"src": "Patient: Hello Dr. I am 42 my menstrual cycle is normal... except for last month. last month it came after a month but not on 28th day like before... We are looking out for a child... n went to dr. and dr. prescribed for Dronis 30... what must b the reason for this capsules... we are planning for test tube baby? and this dr. is expert in that....Please suggest if we are going right... Doctor: Hi,Thanks for your query. I read your query and I understand your concerns.Following is my reply:1) It is prescribed to regularize your cycle and program it to fall on a particular day for starting treatment.2) It is also to synchronize follicles to get equal sized egg.3) You are on right track. Dont worryI hope I answered your query.Write back to me if you need more clarifications.Regards,Dr. Mahesh KoregolIVF & Infertility Specialist"
},
{
"id": 210541,
"tgt": "Are aspergers,adhd,bipolar mood disorder signs of schizaphrenia?",
"src": "Patient: Hi my son was diagnosed with aspergers ,adhd,bipolar mood disorder since he was 3 .he is now 15 and months and hearing voices hes fighting in school, he comes homes he doesnt like to socialize or go out he on alot off med is it possible hes schizaphrenic? sorry for speeling Doctor: HIThanks for using healthcare magicChances of getting mood and psychotic symptoms in autistic children are very high. If your son has hearing of voices, better to consult a psychiatrist. It may be schizophrenia or part of bipolar disorder, but in both cases, he should be on antipsychotic drugs.Thanks"
},
{
"id": 142439,
"tgt": "Should i be worried about the Cortical Brain Thrombosis?",
"src": "Patient: Namaste, uncle myself Rajeshwari bhavimath frm Bagalkot , karnataka.. Now I'am 17 years old .. From before 8 months I'am suffering from CORTICAL BRAIN THOMBOSIS & consulting Dr.Santosh kumar SAGAR hospital Bangalore .. But I'am stressed by my mentality very very very much more than 90 per cent .. I'am not leaving anyone to live in my house even me.. Troubling myself and everyone in my house.. I became like devil to everyone .. Day by day I'am going not a mental, devil.. I need your help doctor please help me.. Doctor: Hello Thank you for trusting HCM Dear cortical venous thrombosis may cause personality changes if frontal lobe involved. Don't worry about it. Continue meditation for it. Like anticoagulant and anticonvulsants. Take anti edema measures. Your may be having depression. Please consult psychiatrist opinion also. It will subside soon."
},
{
"id": 168516,
"tgt": "What causes sudden blood vomiting in kids?",
"src": "Patient: Hi, my daughter is 3 years old, and she started vomitting blood this evening, her father has her on post at the hospital, but his phone died and he can t tell me why she s vomitting blood. She has never had any major health problems, and is only allergic to a few things such as peanut butter and pineapple. She weighs 36 lbs Doctor: Hi...blood vomiting can occur in food allergies. It can occur due to some drugs like pain killers and steroids. But the quantity and history and clinical examination all have to be kept in mind before attempting to confirm a diagnosis.regards - Dr. Sumanth"
},
{
"id": 124677,
"tgt": "Suggest treatment for muscle pain and weakness",
"src": "Patient: I have viral fever from monday afternoon to tuesday whole day 104.5 F and some non digesion in the abdomen. now the fever is over but i fell extreme weakness and pain on my limbs and other muscles of my body. Beside this some times I would have vomiting feeling also. What to do, tell me please?? Doctor: Hi, With fever, back pain, body aches are common. I recommend plain paracetamol for this pain since other painkillers would enhance gastritis. In general, I suggest good hydration with an electrolyte solution to combat weakness. Hope I have answered your query. Let me know if I can assist you further. Take care Regards, Dr Gopal Goel, Orthopaedic Surgeon"
},
{
"id": 30002,
"tgt": "What causes severe cough in 23 month old baby despite medication?",
"src": "Patient: My 23 month old baby is having severe cough..had slight fever ystrday..bt fever is almst settled now..iam giving her taxim o syrup 5ml in the morning n 5ml at ngt..but still her cough is not reduced..im giving her syrup for the past 2 days.Plz guide me. Doctor: Hi, Thanks for your question. I do understand your pain and discomfort. As per your history is concerned please follow:-1)Give antibiotic Taxim-O as usual complete course2)Add [KOFAREST-PED oral drops]one ml 3 times per day3)Add[LABIFOS SACHET] twice a dayI hope I was able to address your query. If you have any further questions, please do not hesitate to write to me. Wishing you all the best. Thanks"
},
{
"id": 44287,
"tgt": "Done semen analysis test. Is my report normal?",
"src": "Patient: i need to know whether my semen report is normal and able to make pregnant Dear Dr., i am 26 years old man, 2 year ago i got married and we are expecting for baby. So, took semen analysis and got the report also in the below Liq. time: 25 min Quantity: 3.5ml Colour: pearl white PH: 7.5 FRUCTOSE: Present Viscosity: Normal Grade A (Rapid progressive) : 60% Grade B(Slow progressive): 10% Grade C (Non Progressive): 10% Non motile: 10% Total sperm count: 35.8 million/ml Abnormal sperms: 20% Microscopic: 3 to 4 Pus cells, Immature 3-4, Epithelial cells 1-2, Red cells-Absent So my report is normal or not Doctor: Hi Vikas Welcome to HealthcareMagic Your semen analysis report is absolutely normal. Get a semen culture done to rule out infection. You can be a father soon. Take care."
},
{
"id": 36030,
"tgt": "Suggest substitute of akurit 4 for treating TB",
"src": "Patient: HI sir, My mom is an TB patient and she use to take Akurit 4 tablets daily 4 with empty stomach however Akurit 4 has been stopped by the concerned company , so kindly suggest which tablets to take. Her weight is 59 kgs and she has used Akurit 4 tablets for 2.5 Months. Doctor: Dear I can understand your concern . I would like to inform you that you should consult your physician for another substitute without any delay , as discontinuation in treatment may cause treatment failure . There are so many pharmaceutical companies that make it one i would like to mention is AKT4. but before starting this discuss with your physician about this and if he is not available you can discus with any other physician but there should be no delay.Hope this will help you . Thanks for contacting HCM ."
},
{
"id": 172016,
"tgt": "What causes white crusty scabs on the scalp of a child?",
"src": "Patient: My three year old son has white crusty scabs on his scalp. I have noticed the dry patches before, but they were usually not attached. I have now found quite a few patches that are attached. They feel like lumps on his head but peel off. Can you tell me what they are? Doctor: Hi read ur question Ur child has crusty scab no the scalp Which was attche with scalp It's mostly seborrhetic dermatitis For this u have to use antifungal sampoo twice weekly To soften thick patch first , rub mineral oil onto the area and brush gently to help peel scales off .Use warm water and massage the affected area Avoid scratching of head . If not improve then show ur paediatrician or skin specialist"
},
{
"id": 154237,
"tgt": "What causes shortness of breath in a vaginal cancer patient?",
"src": "Patient: hello my wife has small cell primary vaginal cancer, advanced grade 3. she has sever shortness of breath edema in legs some memory problems she is on oxygen all the time. her hands and leg neck mouth area are red/blue. she get a coldness that she feels throught her body(deep chills0 she sleeps more about 12 hours a day sometimes more. Noone can tell me what to expect in the short run. Is she close to her time ? Thank you for your help Doctor: Thanks for your question on HCM. I can understand your wife's situation and problem. Small cell tumours of any organ are very notorious for distal organ spread. They are known to involve lung, liver, bones and brain. So possibility of lung metastasis as a cause for her breathlessness is high. Vaginal cancer tend to cause pelvic lymphnode involvement and lymphatic obstruction. So bilateral lower limb edema can be due to this. Brain metastasis can cause memory disturbance. So all her symptoms favour possibility of distal organ metastasis. And this is not good sign for prognosis. Sorry to say but she is having end stage cancer with life expectancy is less than 1 year."
},
{
"id": 94922,
"tgt": "Having lower abdominal pain, feels like period cramps. Not cured by paracetamol. What is going on?",
"src": "Patient: Hello, I have lower abdominal pain , it feels like period cramp but my period passed 2 weeks ago and I almost never suffer from it. It is worse on my right side but I can also feel this in my lower back. I am usually very healthy and am 37 year old mother of three. I have tried paracetamol but it has not eased it at all. It has also got slightly worse as the evening has progressed. Doctor: Hi, It will be better to have a check up with gynecologist /surgeon . The pain is not related to your period . There may be some other reason like pelvic inflammation . appendicitis to be excluded .A CLINICAL CHECK UP AND INTERNAL EXAMINATION WILL HELP REACHING A DIAGNOSIS .AN U S G WILL BE HELP FUL IF REQUIRED . For the pain you may take dicyclomin tab. Get a check up and be all right with treatment ."
},
{
"id": 220314,
"tgt": "Can I try for my second baby at this time?",
"src": "Patient: Heloo Doctor, Good Day to you. Iam 33 year old and mother of 7 year old son,(caesarin). My period cycle is very regular and its 26-28 days cycle. In feb 2010 i for my periods on 7th and in march i got in on 5th. As i was travelling in April i took tablets for 6 days ( 2 times a day) to postpone and i got my periods 3 days after i stopped taking tablets. I wanted to know whether we can try for 2nd baby after this april periods or we need to wait for one cycle to go. Also i would like to have twins this time Awaiting your valuable advice and suggestios. Doctor: You can definitely try .There may be some variation in ovulation time but you can go for it.its not going to harm you or your pregnancy in any way"
},
{
"id": 12821,
"tgt": "Suggest remedy for rash",
"src": "Patient: My 18 month old has a rash around her bcg site for the last two days. She is irritable but doesn't have a fever and isn't sleeping well. She appears to have a bit of a rash now around her lower trunk area as well.Should I be concerned. I came across KD on the Internet and now I wonder should I bring her to a physician? Doctor: Hello,I read carefully your query and understand your concern. The rash can be relate to a viral infection. I suggest to wait and check the body temperature regularly. Meanwhile,I suggest using Calamine lotion for local application. Hope my answer was helpful.If you have further queries feel free to contact me again.Kind regards! Dr.Dorina Gurabardhi General &Family Physician"
},
{
"id": 97724,
"tgt": "Is there any alternative medication for spine degeneration?",
"src": "Patient: I have a friend in his 60's who has degenerative spine and is in constant pain. This may not be the exact diagnosis but generally speaking. What can he do if anything holistically to treat??? He is over addicted to pain killers currently. I do not want to subscribe to the service I only need an estimated answer, please>>>> How unethical to charge money for someone who is critically ill. I thought your passion was to help sick people and save dying people. Shame, shame shame. Doctor: Hello friend,Thank you for posting your query through Healthcare magic. I am a Homoeopath and I can confidently say that Homoeopathy can help your friend. Degeneration of bones at this age is a genetic and constitutional problem and the treatment that is needed should help him to rebuild his bones. This can be done only by approaching his constitution genetically. Homoeopathy does that. No amount of pain killer drugs are going to help him in the long run. Not only that, those drugs will have their own bad effects that are going to make still worse problems in other systems.Therefore ask your friend to approach an expert Homoeopath at the earliest. Regarding the latter part of your query, every individual on earth normally works for his livelihood. The same work can be interpreted as a service directly or indirectly to other human beings. Medical men are experts in helping the people to regain their health. This does not mean that they should work without getting any remuneration from their clients. There is nothing unethical or shameful in charging money from their patients. If the patient cannot afford to pay for the treatment, every practitioner does it on a charity basis. Usually people's governments are expected to help such patients.Hope you are satisfied by my answer.With best wishes,Dr C. J. VargheseHomoeopathdrcjvarghese@gmail.com"
},
{
"id": 176083,
"tgt": "What causes constant runny nose and dry cough in child?",
"src": "Patient: My 6 year old daughter has a constant runny nose, dry croupyour cough and has run fever onot average 99 degrees for 2 days. She suffers from allergies and takes Allegra daily as well as a prescription nasal spray. We cannot seem to get her nose to clear up or her cought to go away. Are there any other treatments we can do at home to help? Doctor: HelloMost of the time this is allergic in origin. Sometimes it is really difficult to find out the culprit allergen.Some medications like montelukast on long term helps in controlling the symptoms.I would suggest you to observe for the allergens and consult a physician to start medication like montelukast or montelukast/cetrizine combination.Regards"
},
{
"id": 136973,
"tgt": "What causes pain in the lower back radiating to the spine?",
"src": "Patient: This morning I awoke with pain in my lower back (about level with my belly button). It felt like it was coming from my spine at first, but after adjusting positions in order to relieve the pressure, I realized that the pain was coming from a few inches to the left of my spine and I could not find a position to relieve the pain even slightly. After an hour the pain was unbearable and I had a bowel movement. During a quick shower the pain shifted to the my abdomen, to the left and below my belly button and the pain increased. My stomach muscles were tight making another bowel movement impossible though pushing made the pain tolerable for a moment. I then began to pour sweat even though I felt cold. Eventually I threw up a lot of water, though it appeared a bit grayish. Then, while laying on the bathroom floor, the pain gradually subsided over the next 5 to 10 minutes until all the symptoms were gone. A few hours later I feel a light pain in my back and I m worried that the episode might repeat before I have a chance to see a doctor. If I had normal insurance I would have gone to the ER, but I have only a VA medical plan. I apologize for the length of this, I m trying to provide all information up front. I m hoping you can tell me the cause of the pain and what to do to prevent it from reoccurring. Doctor: Hello, I have studied your case with diligence.As per your symptom and history there is possibility of acute muscle spasm in lower back and hamstring due to excessive work strainFor these symptoms muscle relaxant with analgesic and neurotropic medication can be started consulting your treating doctor.Till time take rest and continue analgesic available over the counter take rest, and hot fomentation. Physiotherapy like ultrasound and interferential therapy will give quick relief.You may need to do MRI spine if symptoms persist.I will advise to check your vit B12 and vit D3 level.Hope this answers your query. If you have additional questions or follow up queries then please do not hesitate in writing to us. I will be happy to answer your queries. Wishing you good health."
},
{
"id": 30721,
"tgt": "Suggest treatment for fever and blood shot eyes in case of cat scratch fever",
"src": "Patient: my son has cat scratch fever, saw a dr. and waiting for the cat scan results to come in, I am worried, he has fever, and is not eating and is sleeping for about 2 good days now, he has blood shot eyes, and feels faint. We are having to see the dr. on Friday morning, what are my concerns till the appt.? Doctor: thanks for posting your query to HCM .Cat scratch fever is a bacterial disease involve multiple system of body. red eyes is a manfestation of eye because of congestion . you dont need to be worried as disease if do not compicated resolve its own. you can give your child ANtipyretic : Paracetamol for releif of fever . Maintain hydration and give him plenty of elctrolytes.if still he is feeling weakness or faintness immediately addmited himtohospital .take care ."
},
{
"id": 25436,
"tgt": "What causes pain in thorax muscles after an angioplasty?",
"src": "Patient: my mum had angioplasty 2 months ago, she didnt go for physiotherapy as was advised by the doctor due to lack of money. She has been massaging her thoraxmuscles which was paining using hot water. a week ago she stopped to massage . now she compain of sharp pain on those muscle when she moves them. what could be the problem and whats the solution? Doctor: Thanks for your question on Health Care Magic. I can understand your concern. Possibility of muscular pain is more. It is better and more beneficial to apply ice packs than hot water to relieve muscular pain. So, start ice pack application on painful areas. Take painkiller and muscle relaxant drugs. Avoid movements causing pain. Avoid bad postures in sleep. Avoid heavyweight lifting and strenuous exercise. Don't worry, she will mostly improve with all these in 5-6 days. If not improving then get done ecg and 2d echo to rule out heart related causes for chest pain. Hope I have solved your query. I will be happy to help you further. Wishing good health to your mother. Thanks."
},
{
"id": 31987,
"tgt": "What causes liquid discharge with bad odor from the keloid on the back?",
"src": "Patient: I am a 52 year old woman who had an excision of cyst on the upper part of my back 4 years ago, my doctor left the site open and not stitched. Few months after that, a keloid was then developed. A year after the keloid has come out, i have noticed and smelled a foul smelling thing on my back which i believe is coming from the said keloid. i am wondering, what could this be? i have always kept my back and keloid dried and clean as possible, and yet the foul smelling still remains. Doctor: Hi..Welcome to HEALTHCARE MAGIC..I have gone through your query and can understand..As per your complain it seems that there is bacterial infection in the area of the scar where there is formation of keloid and the foul smelling discharge is due to infectious liquid known as Pus..You should consult a general physician or a general surgeon and get evaluated and in case of infection he can advise you oral antibiotics like Ciprofloxacin and Metronidazole, anti-inflammatory painkiller like Ibuprofen and topical treatment like cleaning the are with antiseptic solution and application of triple antibiotic ointment over it..The area should be kept clean and limit water exposure and do not cover it with a bandage..Many a times if there is recurring or persisting problem scar/keloid removal can be advised..Hope this information helps..Thanks and regards.Dr.Honey Nandwani Arora."
},
{
"id": 21658,
"tgt": "What is the treatment for aortic valve calcification?",
"src": "Patient: i am 72 yers old male normal blood presssure, no diabitis never had any surgery. per cardilogist i have aortic valve calsification (stenosis) jan. 2010. twice i visited. per dr's advice i need heart cath to check aerotiv valve calsification. what is your advice? should i go ahed for heart cath? Doctor: Hello,Yes definitely, because to guide for further treatment that is a must. In valvular disease, we have to know how much is pressure and velocity of blood at a particular point, based on that it decided you need surgery or medical management.Hope I have answered your query. Let me know if I can assist you further. Regards,Dr. Puneet Mahajan"
},
{
"id": 91501,
"tgt": "Is abdominal swelling related to eating?",
"src": "Patient: There seems to be swelling in my lower abdomen/ an area diagonal-right, above my penis. My stomach seems to be cramped/hurting as well. I ate an orange, a banana, an apple, a cup of oatmeal, two poptarts, a couple of carrots, and two leaves of lettuce all at once. Is this abnormal swelling somehow related to how much I ate? It s also strange because it s only swelling diagonal-right above my penis, not diagonal-left or above it in general. Doctor: Hi ! Good afternoon. I am Dr Shareef answering your query.From your history, the most likely diagnosis which could be thought of is an Inguinal hernia which causes such swelling. It might not be related to the food you ate. For a definitive diagnosis, you have to take an appointment with a general surgeon, and get your self examined for further management. Till that I would suggest you not to lift any heavy weight, or go for any heavy work which increases your intra abdominal pressure, and try to reduce your weight if overweight or obese.I hope this information would help you in discussing with your family physician/treating doctor in further management of your problem. Please do not hesitate to ask in case of any further doubts.Thanks for choosing health care magic to clear doubts on your health problems. Wishing you an early recovery. Dr Shareef."
},
{
"id": 32821,
"tgt": "Suggest medication for an infection in the buttocks",
"src": "Patient: I think my butt is leaking a fluid. I've had anal sex before but that was a yr ago, but I've noticed that after i've showered when I wake up in the morning after my morning pee, I wipe and my butt is dirty like I just pooped. Why is this happening to me? Doctor: Hi, Welcome to HCM.Looking at the description about the problems you are facing, it appears that you might have developed mucosal damage after you had anal sex. The problems you are facing must be due to anal fissures or fistula-in-ano. Kindly don't neglect the symptoms. You need to get examined by a Surgeon in this regard. Most likely, there aren't any medications to treat such disorders. You need to undergo surgical procedure, by means of cryosurgery. Hope the information provided would be of help. All the best."
},
{
"id": 12445,
"tgt": "What are the medications for psoriasis that are not harmful to the body?",
"src": "Patient: Hi, I am on oral steroids (Methotrexate) for servere psoriasis. They seem to be working because the psoriasis isnt as aggressive, but i am feeling worried about the long term use of the drugs. How long will i need to be on Methotrexate and what other medications could i be using that arent as harmful to my body or as depressing as this drug? Thanks Ben Doctor: Hello,I can understand your concern for severe psoriasis. It is an autoimmune disease where there is fast proliferation of skin within 4-5 days rather than 28 days of normal time.It depends on severity of the psoriasis lesions to give the proper treatment accordingly.Methotrxate is one of the best treatment but under dermatological supervision and proper regular testing of blood profile. it is not a steroid and there fore the side effects of steroids are not present.You can take further guidance form your dermatologist or from me. You can send pictures and take guidance accordingly at:bit.ly/drsanjaykanodiaTake care"
},
{
"id": 73337,
"tgt": "Does AV Node Ablation treat shortness of breath in elderly?",
"src": "Patient: Is AV Node Ablation a good treatment for my 81 yr old mother (she's a young 81!) with shortness of breath (worsening over the past 90 days)? She had a heat attack with 5-way bypass 11 yrs ago and already has an ICD (defib and packmaker) implanted. Has been very independent up until last 90 days. Now very low energy and worsening shortness of breath. Her heart rate is good, blood pressure good and oxygen content in blood all good. Tried multaq and amiodarone briefly but sides effects too strong for her. She's still with coreg. Just wondering.....? Thanks. Doctor: I am assuming that your mother was offered this procedure for atrial fibrillation (an irregular heart rhythm)? If that is the case, the procedure itself would prevent the irregular and sometimes fast electrical activity from the atria from reaching the ventricles. This has the potential to improve shortness of breath that is caused by ventricles that are beating too quickly. However, your mother may also have other causes of shortness of breath that are directly related to the efficiency of the pumping function of the heart. I hope she feels better."
},
{
"id": 128717,
"tgt": "How to treat heel pain while having back pain caused due to herniated disc?",
"src": "Patient: Hi, I am a 51 year old lady, who has heirnated disc, I have started physio at the hospital and my back pain seems to have subsided so is not as painful, but now I have pain on the bottom of my heel/ arch, hobbling around like some old women! (Been told by physio it neves and got excises to do). Just wondered if I can get any pain relief, shouldn t take ibuprofen as I also have Crohn!My other issue is about three years ago I had my carpool tunnelling done on my right hand and at the time I had a bit of a trigger finger, last year was re referred back to consultan, long story cut short eventually gave me an injection which was great, he left a appointment for me three months later, which I cancelled because everything was ok, which was last September! Then it came back with a vengeance! I went back to Gp in December and he has now sent me to a Rumatologis but the first appointment is not till the 27th February, and I don t know what to do with the pain! Worse in the mornings as I can t even bend mid knuckle to joint knuckle doses get a bit better during day, but opening and closing hand u can see that their is something stopping it opening smoothly like the others or how it should be! Any advise would be appreciated as both issues are affect my every day life! If don t work I don t get paid! Thanks you Sara Doctor: Dear patient uou have got two conditions 1. trigger finger 2. plantar fascitis. For trigger finger you need to take tab paracetamol 500 mg thrice a day for pain relief. Do hot water fomenting with water bag. if not relieved local steroid injection is very helpful. For for plantar fascitis Do ankle stretching exercises , wear soft cushion heel both sides , do hot water fomenting. If not relieved Local steroid injection gives complete relief in almost all cases.Tablet paracetamol also works for plantar fascitis. If both conditions are significantly hampering with work please consult your orthopaedic surgeon as soon as possible."
},
{
"id": 35723,
"tgt": "Is Nuforce good for vaginal infection?",
"src": "Patient: My wife has some kind of vaginal infection for last two months, now doctor prescribed me Nuforce 3 Kit, why he prescribed me that and what is the usage of that can u please explain me? I am having some stomach problems after taking that so is it okay for me to continue it? Doctor: vaginal infection is mostly combined infection(fungal, protozoal and bacterial), rarely single infection. so, nuforce 3 kit given you to remove infection. use of the drug is to control and remove the infection. combination of three anti infection agent can cause gastric irritation, so it is advised to take antacid till the treatment is prescribed."
},
{
"id": 39574,
"tgt": "What are the side effects of typhoid?",
"src": "Patient: i am detected double typhoid 1:160 and 1:20 25 days back i was on heavy antibiotics after 10 days i got checked then it was 1:80 and 1:40. for 15 days i am having oflomac 400mg and minmin (vitamin), i m a male, can it be safe for baby if i have sex for conception of baby. Doctor: Dear Friend.Welcome to HCM. I am Dr Anshul Varshney. I have read your query in detail. I understand your concern.You should avoid conception till you complete the course of antibiotics.As far as typhoid id concerned, it is not going to cause any problem in conception.Only problem is that if you concieve now and you continue with antibiotics, they may harm the baby.This is my best opinion for you based on available details. If you have any further query please ask me.Stay Healthy.Dr Anshul Varshney, MD"
},
{
"id": 11380,
"tgt": "Suggest remedy for regrowth of lost hair",
"src": "Patient: Hi, I have lost a lot of hair over past one year. i was going through a very stressful situation. My Doctore prescribed Ipca s Kera XL, no oral medication though. I do see a the hair fall has stopped, however, I am worried about the regrowth of hair. Please advise, as the hair growth around the front portion of my head is not very significant and my scalp can be seen through. Doctor: Hello. Thanks for writing to us at healthcaremagicCauses of hair fall vary in males and females.Androgenetic alopecia is the most common reason for frontal hair loss in both males as well as females.I usually start my patients of androgenetic alopecia on a minoxidil preparation, twice daily. Minoxidil is US FDA approved for this type of hair loss.Oral treatment includes tablet finasteride 1 mg once daily in males. Females can be started on either spironolactone or finasteride.These ae prescription drugs therefore, I would suggest that you seek another appointment with your dermatologist for the needful.Regards"
},
{
"id": 79816,
"tgt": "Suggest treatment for burning skin, coughing, mucus",
"src": "Patient: Hey my eyes hurt, my skin and body feels fragile, my eyesHurt when i turn them, and burns at times, ive been voughing with mucus. I have bronchitis and live in carpeted house, i also have really bad allergies. Should i be worried? What meds should i take? Ive been deinking vick and zyrtec Doctor: Thanks for your question on Health Care Magic. I can understand your situation and problem. Since you are having severe allergies, your all symptoms are suggestive of allergic etiology only. The best treatment of allergy is identification of allergen and if possible complete avoidance of it. You are living in carpeted house. And carpet fur is common allergen. So if possible remove the carpets. Avoid dust, animal contact, pollen exposure etc. Antihistamine drugs are needed to suppress cough and eye problems. Anti allergic drug is also beneficial in your case. Since all these drugs are prescribed medicines, you need to consult doctor for prescription. Hope I have solved your query. I will be happy to help you further. Wish you good health. Thanks."
},
{
"id": 165405,
"tgt": "Is it advisable to go for ureter implant?",
"src": "Patient: sir hi have 6 months old boy. there was a mild hidronephrosis in left kidney that time when my wife was pregnant. now i consult to my pediatrician in jaipur. they advised me sonography. i got it done. there was again mild hidronephrosis. then i consult a urologist in jaipur. they said to me to go for DTPS scan. i again got it done. there was the problem in ureter that ureter is significantly diluted. then my doc advised me MCU. i got the MCU done. it was fine. now my doc is saying me to ureter implanment .kindly sugest me what should i do. Doctor: Hello,Hydronephrosis is a condition in which the kidney gets swollen due to accumulation of fluid in kidney due to obstruction or reverse flow of urine. Hydronephrosis can lead to loss of function and permanent damage to kidney. Significantly dilated ureter with hydronephrosis in your case is ideal indication for ureter reimplantation surgery which will prevent future damage to kidney.Hope I have answered your query. Let me know if I can assist you further.Regards,Dr. Khan Shoeb Mohammad Sher Mohammad"
},
{
"id": 142412,
"tgt": "Does sore jaw bone after head injury indicates concussion?",
"src": "Patient: I fell and hit my head over the weekend. There was initial swelling near my temple, but after a day or 2 it is almost gone. My jaw bone still a little sore, and it feels like water is in my ear. My vision is fine. Could that be a sign of a concussion or something internal? Doctor: From your description of the injury and your symptoms- you do not appear to have suffered a concussion. You may write to me with more questions to: www.bit.ly/drdariushsaghafi Please rate as a 5 STAR ENCOUNTER."
},
{
"id": 158400,
"tgt": "Having fibro, hirschspjprungs, post breast cancer and dysthymia. Decreased the dosage of Ambien. Suggestion?",
"src": "Patient: I have an appt With my doc next week but for the av last 2.5 weeks the np has cut my ambien from 10mg to half that I have been on for ten yrs leaving me very agitated and awaking every few minutes. I have fibro, hirschspjprungs, 4 yrs post breast cancer and dysthymia . I am retired and have worked very hard to limit the stress in my life. Lack of sleep aggravates meverything in my world. Help! Doctor: Hi there, Ambien (zolpidem) is advised for short term relief of insomnia only and one should not be on for like 10 years, You need expert help and plenty of will power to get out of its dependence. See your GP and see experts like psychiatrists if referred to. Take care"
},
{
"id": 201430,
"tgt": "Suggest treatment for low libido, erection problem and mood swings",
"src": "Patient: My husband and I are in our late 60's. Up until two years ago we had a reasonably healthy sex life but even then my husband had difficulty keeping an erection. His depression mood swings have increased over the last two years, he's become disinterested in intimacy, he lacks drive and is always tired. He refuses to take any natural supplements to improve the situation. He was also diagnosed with Type 2 diabetes which he has managed to reverse with cutting sugar out of his diet altogether. We can't go on living like this. Doctor: Thanks for contacting HCMI am sorry to hear that you husband is having problems with depression and erectile dysfunction. These two problems go together. As long as he feels depressed he is not wanting to be engaged with any sexual intimacy. He needs to get control of his depression first before he can feel better. I recommend that he be seen by his doctor and have him screened for low testosterone. A low testosterone level is very common in men with diabetes and this could also be at the root of his problem. If he is not having low testosterone I then recommend for him I recommend either celexa or wellbutrin. Once these medication start working and he still has erectile dysfunction he should then try Viagra or CialisHope this answers your question. Please contact us again with your health care concerns and questions"
},
{
"id": 160153,
"tgt": "What is the chances of having cancer from a burst tumor ?",
"src": "Patient: hi my father was 56 yrs old,last march 2010 he was suffering from gingivities and he thought it was just simple pain,hr suffered for almost 9mos untill he realized to see a doctor last early of december.the doctor diagnose was a tumor in his mouth was a malignant neoplasm and he was advised for operation. My father refused because we cannot afford of the medical and hospitalization expenses.On the mid of march 2011 the tumor burst in his jaw until now he suffer the pain,Now he is taking medicine for the pain,he is taking also a supplement which is called mx3 capsule?i just wanna ask if this capsule can help my fathers illness? Doctor: welcome to healthcaremagic as per report your father is having cancer mx3 as claimed may be rejuvenating but it is not anti cancer drug it bis better to have opinions of cancer surgeon and cancer chemotherapist"
},
{
"id": 25743,
"tgt": "Suggest treatment for BP related to stress",
"src": "Patient: i am aged 25years. i am having high BP related to stress-tension, not related to food. last week , I checked my whole body , everything was normal ,even Iodine content also normal ...wont smoke , drink also....daily doing exercise for 45 mins jogging.....I having high BP as 130,140,150 varies....please suggest Doctor: Thanks for your question on Health Care Magic. I can understand your concern. Stress and tension are the biggest risk factors for hypertension in young individuals. And your blood pressure readings are not very high for starting antihypertensive drugs. Only stress management will mostly work for you. For this, better to consult psychiatrist and get done counselling sessions. Try to identify stressor in your life and start working on its solution. You may need relaxation exercises. You may need anxiolytic drugs too. Along with drugs, counselling and relaxation exercise are very useful in controlling stress. Avoid stress and tension, be relax and calm. Strict salt restriction in diet and exercise should be continued. Don't worry, you will be alright. Hope I have solved your query. I will be happy to help you further. Wish you good health. Thanks."
},
{
"id": 155152,
"tgt": "What are the symptoms of ovarian cancer?",
"src": "Patient: I am a 38 year old female. I've been vegan for 4 years. I eat a whole foods diet of fruits, vegetables, fresh juices, legumes and some grains. I recently cut out all sugar and have cut down on carbs from pasta and bread. Still, I am 35 pounds overweight and almost all of it is in my lower and now upper abdomen. I'm worried that I have some fluid build up in my stomach that is causing such severe bloating and weight problems. My mother had breast cancer at age 37. Should I be checked for ovarian cancer? Doctor: HiThanks for your queryBased on your query, my opinion is1. Distension of abdomen if present needs to be evaluated for exact cause. 2. Ovarian cancer can cause abdominal distention, pain and dragging sensation in the lower abdomen.3. Moreover it causes systemic symptoms like loss of weight, appetite, easy fatiguability, tiredness, anemia.4. Genetic causes have been found for ovarian and breast cancer have been found, so chances are there for you to get.5. You can get that doubt cleared by getting a ultrasound examination of abdomen. 6. You can go for genetic counselling and testing for ovarian cancer and breast cancer as wellHope this helpsWish you good health"
},
{
"id": 170871,
"tgt": "Suggest remedy for constant passage of gas and painful discomfort",
"src": "Patient: Hi Doctor, I have a 45 days old baby girl who is on 70% breast feeding and Nan Pro 1 for the top feeds. She has been having irritation in the stomach and passes gas continuously through the day and night and as a result has not been able to sleep well. She gets utter discomfort at every passage of gas which happens almost every two mins. She passes stool once a day and it s not watery. She urinates normally. Her weight gain is also normal per the pediatrician. We tried simethicone drops but nothing is relieving her from discomfort. We try to burp every pre and post feed and has not helped.we massage regularly also to see if it relieves the discomfort.. It does not. We really need a solution / medicine which will resolve this constant pasage of gas and the painful grunting /discomfort the baby feels as a result. Please help. Doctor: Hi...Thank you for consulting in Health Care magic.This is called evening colic and is quite common in this age group. This happens when the baby sucks at the breast very fast and in eagerness to drink milk will gulp in air too. Unless the air comes out like burping or flatus this discomfort will be there and next time check if the baby is sucking too fast and gulping in air too. You will be more convinced. Usually I don't advice any medicines for this as they give only temporary relief.The two best ways to relieve this distress is - 1. Do not put the baby in lying position after feeding till the baby burps out the swallowed air.2. If still crying - put the baby in prone position and keep patting the back gently so that the baby passes off the flatus and gets relieved.Hope my answer was helpful for you. I am happy to help any time. Further clarifications and consultations on Health care magic are welcome. If you do not have any clarifications, you can close the discussion and rate the answer. Wish your kid good health.Dr. Sumanth MBBS., DCH., DNB (Paed).,"
},
{
"id": 92419,
"tgt": "Could the hernia repair done in groin area be tearing lose?",
"src": "Patient: My name is Bonnie i am 69 years old I had surgery done about i will say 7 years ago ! I had i believe two Herieas repaired in my stomace and one in the groin area ,I can not remenber the Doctors name that did the surgery ,he was in the doctors building next to St Elizabeth hospital on Turfway . I feel like the one in my groin area is tearing lose I get very upset and afraid of being put to sleep even more now i have a few illnesses that make me more afraid of being put to sleep anyway I need to see if it is tearing lose if you could help me find a doctor that i can see thank you sorry about the spelling bonnie Doctor: Hi..Thanks for writing in.I would suggest that you consult a General Surgeon and get your self examined.In case there is a recurrence or any other problem that needs to be rectified at the earliest.Regards"
},
{
"id": 197254,
"tgt": "How to cure painful bumps near the genitals?",
"src": "Patient: Dear Doctor, I am a 21 year old male, recently I've been getting under the skin bumps near my genitals. They usually appear in the area either in-between the thigh and the beginning of the testicles or on the buttox. The bumps are painful to the touch and painful to sit on. I've popped one of them and they secrete a liquid similar to that when you pop a pimple. They go away and appear one at a time every now and then. I am growing concerned because this started happening all of a sudden. It is not possible that it may be an STD, I'd appreciate any advice you can give me. Doctor: HelloThis sounds like folliculitis--hair shaft infections--and at this point you'll need a course of antibiotics like Septra DS or Augmentin to resolve the problem"
},
{
"id": 188118,
"tgt": "Nausea, throbbing pain, blood and yellow fluid on gauze pad after wisdom teeth extraction. Is the infection spreading?",
"src": "Patient: It's has been 8 1/2 days since wisdom teeth extraction. 1 lower left. 2 upper ones as well as the two extra teeth beside the wisdom teeth. I have been feeling very nausea since that day. Throbbing pain came around the fourth day and stayed until the 6th day. around day four I was feeling dizzy as if I was going to pass out. The wooziness feeling came back the 6th day. I notice a white sac thing on my cheek and it was sitting on top of my gums beside extraction site around day 5. It deflated and pain eased up. 6th and 7th day I noticed yellow fluid then brown fluid. Now today the 8th day, I feel very sick and blood was on gauze pad I use to check for odd fluids. my question is this an infection spreading throughout my body. Should go to oral surgeon, dentist, or hospital. Doctor: Hello,Fluid discharge post extraction of wisdom tooth may be due to-Development of dry socket at the site of extraction socket.Irrigate the socket with betadine solution and dressing with ZOE pack has to be done.Complete course of antibiotics-analgesics has to be administered.Maintain oral hygiene well.Take adequate rest and avoid anxiety.Take care."
},
{
"id": 30813,
"tgt": "Can measles cause sore throat and spots?",
"src": "Patient: I had measles over a week ago.. I did not finish my Antibiotics, but the spots and sore throat had gone away.. My boyfriend who I live with broke out with them yesterday... I am as of yesterday getting my sore throat back and getting a few spots this morning.. Is it the same thing again? Doctor: thanks for posting you query to HCM . I want to know how did you make diagnosis of measles and what is your age . Measles virus is usually infect achildren .As I understand you are having some other kind of viral infection . for which you need to undergo complete blood count examination to know infection .but no need to worry as even if it is a infection it will be recover by taking approriate care .you can take 1.Aceclophenac 100mg two times a day .for pain and fever 2. Loratidine 10mg once a day for rash .3. maintain good hydration drink plenty of fluid snd juice.s4. Take good nutritious dite .take care ."
},
{
"id": 30189,
"tgt": "What causes scabies?",
"src": "Patient: We were just moving into a new apartment and my girlfriend's daughter's boyfriend was helping us move things in for several days. About 76 hours ago, before my girlfriend took him home for the night I remember him telling me he was itchy. about 15 hours after that, 67 hours ago she saw him at a local store picking up medication. When she asked what it was he apparently said, \"oh, my doctor said I've got chiggers or something\" and she looked at the medicine he was prescribed and called me at work to look it up. The medication was Permethrin Cream, 5%. I looked it up and found out it was for scabies treatment. Within the next day, he was apparently very badly broken out; within the next day and a half following that his girlfriend, my girlfriend's daughter with whom he's had considerable contact with, was broken out and my girlfriend had complained the night before had an itchy bump on her hip. That evening my girlfriend and her daughter went to the doctor and were tentatively diagnosed with also being infested with scabies. A skin scrape was done on each but the labs haven't come back yet as it was a Thursday and they were told it would be at least 3-4 days before the lab results came back. Both have lived in the same apartment since the news and have been in contact with many things in the apartment. Since it was immediately after/during the move-in many thing were still yet unpacked. A lot of the large furniture had been moved around (many times to see how it looked) by the boyfriend and two other people at the same time, as well as me and my girlfriend separately on the last occasion (the one he told me he itched badly as he left). For three nights I slept with my girlfriend there. The night before they went to the doctor we had about 20 minutes contact where she manually stimulated me with lubrication and held each other off and on throughout the night. I was home very little after the initial move to help move things around and touched very few things after I got home late at night after waking up, but I did move the furniture (suede-like cover) and only wore clothes that the original infested person did not touch. There were, however, things (clothes and otherwise) in our bedroom he did carry that were on the carpet. I showered in a hot shower, as I always do, every morning before and after this became evident. As soon as I found out I have never returned to the apartment, have not had any physical contact with my girlfriend or anyone else for that matter. I am still showing no signs and used about a quarter tube of Permethrin cream on myself the day we all found out. I'm staying at my parents' house at the time, as the doctor my girlfriend saw advised, for a week or longer. I still can't really tell any signs but am getting ready to give myself a second treatment of the Permethrin cream as I was not aware I should wear full covering clothing and not wash it off for 24 hours. I applied the cream fairly thinly except my hands, arms, feet, ankles and genitals (as we had had prolonged contact at that point the night before). My girlfriend has sprayed things in the apartment down with anti-lice spray after vacuuming but has just nearly three days later gotten to wash all the clothes that were on the floor in bunches of hangers because they were not yet put away. Mine have been washed in hot water and dried on high heat, as hers have (considering neither of all have been done yet); mine have been delivered to me without her ever touching them (had her wear long rubber gloves) and I have also washed and dried them at my parents' house without me ever touching them. I have never had scabies before and read that symptoms may take 4-6 weeks to manifest. My girlfriend and her daughter have both had them within the last year for the first time and have manifest symptoms soon after exposure. What are my chances of being infested, infesting my immediate family (with which I've had fairly minimal contact now) and what is my best course of action? -- Thank You for Your Advice -- Travis Doctor: Hello ,Thank you for your question . (Scabies )Scabies is a condition of very itchy skin caused by tiny mites that burrow into your skin. The itching is caused by an allergic reaction to the mites.Scabies spreads very easily from person to person. It can affect people of all ages and from all incomes, social levels, and living situations.With treatment, the scabies mites die and the itching goes away over a period of days to weeks. Without treatment, the mites continue to reproduce under the skin, causing more sores and itching.Scabies has two main symptoms:Severe itching that is usually worse at night. Small children and older adults tend to have the worst itching.A rash with tiny blisters or sores . Children tend to have worse skin reactions than adults.How is it treated?you need to use a special cream,These products contain permethrin.Permethrin 5% cream (Elimite) is the treatment of choice for scabies. Permethrin 5% cream is applied to clean, dry skin. For best results, clip and clean all fingernails and toenails. Permethrin is usually left on the skin for 10-14 hours and then washed off in the shower. It is best to apply permethrin at bedtime and then wash it off in the morning.Lindane 1% cream or lotion is an older medication that is rarely used because it is potentially toxic to the nervous system (leading to symptoms such as dizziness or seizures). Some scabies have become resistant to Lindane.Ivermectin pill(s) (Stromectol) is an oral medication that is active against several parasites. It is not FDA-approved for use in scabies but has been used in cases with very heavy infestations. Ivermectin is not used in small children or in women who are pregnant or breastfeeding.Malathion 0.5% lotion (Ovide) is usually used for head lice and is not approved by the U.S. FDA for the treatment of scabies. It is irritating to the skin. It must be used only as directed and should be kept out of the reach of children because ingestion may cause organophosphate poisoning.Benzyl benzoate lotion is an older treatment for scabies. It can be irritating to the skin, especially in people who have eczema.Crotamiton lotion or cream (Eurax) is approved for use in adults with scabies. Treatment failures with this drug are more common than with permethrin.Sulfur-based lotions, creams, or soaps have been used but are less effective than other options. They should not be used in people who are allergic to sulfa.Most creams or lotions are applied to the entire body from the neck down. In most cases, you leave the medicine on for 8 to 14 hours and then wash it off. Be sure to read and follow all instructions that come with your medicine.To make sure that all the mites are killed:Wash all clothes, bedding, and towels that you used in the 3 days before you started treatment. Use hot water, and use the hot cycle in a dryer. Another option is to dry-clean these items. Or seal them in a plastic bag for 3 to 7 days.Clean and carefully vacuum the room or rooms used by the person who had scabies.After treatment, the itching usually lasts another 2 to 4 weeks. It will take your body that long to get over the allergic reaction caused by the mites. Antihistamines (such as Benadryl), steroid creams, or, in severe cases, steroid pills can help relieve itching.If you still have symptoms after 4 weeks, you may need another treatment. and visit doctor.I wish was helpful for youBest regardsDr. Ahmed Habib"
},
{
"id": 18500,
"tgt": "What causes fluctuating BP readings in an elderly person?",
"src": "Patient: Hello, My 78 yr old brother-in-law has been having Hypertensive episodes. He is on meds for this and recently has been 179/91 when up and about when resting 128/83. He has had a past cardiac event and at that time had a defibrillator placed in case of another life-threatening rhythm. He has been very stable with this and it has never activated. He has had a shingles outbreak and it has affected an eye, he uses a steroid eye drop? What could be causing his irregular BP readings? Doctor: Hello and Welcome to \u2018Ask A Doctor\u2019 service. I have reviewed your query and here is my advice. You have not mentioned the BP reading here, little fluctuation is there if the BP being reading by non medico person it is the error count, and just to rule this out it has to be read by physician, if this is the hypertensive patient then it can be the cause of irregular blood pressure, age is also one factor, actually diastolic pressure matters the great, if the fluctuation of blood pressure is within normal limit then it is nothing to worry. Hope I have answered your query. Let me know if I can assist you further."
},
{
"id": 171866,
"tgt": "Suggest treatment for red bruises , fever and sore throat in a child",
"src": "Patient: Hi my 6 year old has received about 10 bites on her legs from mosquitoes in poland..some are red and angry but one has very large bruising surrounding the bite.she also has a fever, sore throat,no appetite and very lethargic..she is also complaining of her leg being very sore.she has no rash nor any joint pains Doctor: You need to be sure that those bites are due to mosquitoes and not due to ticks, as Lyme disease, caused by a tick, is found regularly in mountainous territories. The latter illness is more problematic. The kind of large rash you are describing is called the herald patch in medical language. Better show a pediatrician as soon as possible.Dr. Taher"
},
{
"id": 143750,
"tgt": "Feeling dizzy while resting",
"src": "Patient: Is there an over-the-counter drug or medication that I can purchase - I have had for the past couple of days dizziness when lying on my back in bed or when turning over in bed - also a little when watching tv - feelings of nausea but haven t thrown up? Also right ear sometimes seems to fill up or echo, particularly when I am singing in the choir. Doctor: Your dizziness can be due to -BPPV ENT problemsTab stugeron forte can be used sos for dizziness. Thanks"
},
{
"id": 59743,
"tgt": "Child having jaundice, cholelithiasis and hyperechoic liver lesion. Should we do endoscopy or surgery?",
"src": "Patient: hello doctor, MY wife is 25 years old and in july 2012 she delivered a boy baby by surgery.after three months she has been identified for jaundice and on further investigation doctor identified she has cholelithiasis and hperechoic lesion on liver.Kindly advise whether endoscopy is enough/ or surgery is the option? very urgent .plz reply Doctor: Hello! Thank you for the query. First of all you should know that cholelithiasis does not cause jaundice directly as mostly it means that you have gallstones (the correct name for it is cholecystolithiasis). Jaundice is caused by stones in bile ducts what is called choledocholithiasis. Your wife should have this stones removed right away as it can result with acute pancreatitis. To do that endoscopy with ERCP should be performed (surgery for bile duct stones can be done if the stones are really big). After this procedure, you need a break for bile ducts to get healed (at least two weeks) and after that, laparoscopic removal of the gallbladder should be performed as such stones can migrate from gallbladder to bile ducts again. Hope this will help. Regards."
},
{
"id": 177486,
"tgt": "Can neopeptine be given to a child who is a picky eater?",
"src": "Patient: Hi, I am 33 years old. I have a son who is a very picky eater. He is now 20 months old and still didn't know how to drink from the bottle. I use the syringe to feed hime since he is 3 months old. He didn't like to eat much. I gave him 21 oz of whole milk a day. I am thinking of giving him neopeptine to try, should it it OK? Doctor: Neopeptine is an enzyme supplement that is supposed to help in digestion in case there are related problems. Although it is ok to be given and can be safely given to a child of 20 months age, I doubt whether it would do any good and I would not suggest giving it. It is not unusual for children to be picky eaters. But milk is now supposed to be an optional food component with cereal based semi solid diet supposed to constitute the bulk of the feeds. Basically the child now needs a mashed version of adult food low on spices and oil."
},
{
"id": 207306,
"tgt": "Suggest remedy for mental health problem",
"src": "Patient: I m so horny sir that my Slung hangs out and talks to me while I sleep Yesterday it told me I better fuck the 27 year old girl next door Otherwise it would jump on her either way oh dear doc I m scared Of going to sleep I can fill Her coming into my room sometimes calling My name Jasper she says I m Home oh Bloody Mary I m sorry I called You Home please Doc Help and older man out She s coming back again she sees the light from my computer screen dear baby jesus help Me oh no shes right outside my bedroom door oh Jesus noooo ahhhhhhhhhhhh Doctor: DearWe understand your concernsI went through your details. I suggest you not to worry much. From the description given, you are hallucinating. Hallucination could be due to overuse of some substance, in psychiatry we call it substance abuse. Another possibility is schizophrenia. Don't worry because worrying cannot cure you. Consult a psychiatrist ASAP for treatment.If you require more of my help in this aspect, Please post a direct question to me in this URL. http://goo.gl/aYW2pR. Make sure that you include every minute details possible. I shall prescribe the needed psychotherapy techniques.Hope this answers your query. Available for further clarifications.Good luck."
},
{
"id": 82882,
"tgt": "Could the tingling on my legs an onset of lupus?",
"src": "Patient: My legs have been tingling everyday now for 2 weeks. 3 years ago my md said I have a very high Ana and referred me to a rheumatologist and they said I don't belong there because at that time I had no symptoms. Could this be an onset of lupus at age 49? Female Doctor: Dear Madam,Yes there is a possibilty but a NCV of your limbs is necessary to look for what kind of neuropathy is there.Dr. Shruti"
},
{
"id": 127439,
"tgt": "What causes severe pain in the legs?",
"src": "Patient: I have recently began experiencing significant pain in my glutes and down my legs. The pain is a dull ache that keeps escalating until I have to change the position I m currently in. The pain is most severe on my left side. I started physical therapy but was not having any relief. I thought it was more muscular than actually my back. However, an MRI of my lower back revealed a herniated disc at L5-S1 with associated cyst causing left side impingement and severe spinal canal narrowing at L3-L4 & L4-L5. I see a spine specialist in a few days, this being new to me, do you have any suggestion as to a few educated questions I would want to ask the doctor. What can I expect regarding remedies and future abilities. severI ve recently started having This is all new to me but I ve Doctor: Hello and Welcome to \u2018Ask A Doctor\u2019 service. I have reviewed your query and here is my advice. Your MRI report shows herniated disc which your suffering from sciatica. your spine doctor will first start you on pain killers and would ask you to do some exercise daily specially back exercise. if those pain killers don't work, then he/she might start you on muscle relaxants or drugs like Pregabalin or Gabapentin. Surgery will be the last option if all these remedies fail. Hope I have answered your query. Let me know if I can assist you further."
},
{
"id": 105756,
"tgt": "Swollen lymph glands, sneezing tendency, dust allergy, azithral with vizilac, gland not visible",
"src": "Patient: My daughter (13 years) is having one lymph gland swollen on left neck , from outside not visible but if touch by hand feeling is there . She is having sneezing tendency & do sneezing continuously 8/10 times . Probably have dust alergy . Recently local doctor prescribes Azithral 500 : 1 time in a day --total 6 days alongwith vizilac . Before medication gland was visible , now not visible but can be felt by hand . Please suggest that will it be automatically normalise or some more medication required. Doctor: Hello. Thanks for writing to us. Any neck gland less than 1cm in size is not significant and does not require any treatment. It seems that the glandular swelling in your daughter's case is due to the allergy she is having. It will subside on its own once the allergy subsides. I hope this information has been both informative and helpful for you. Regards, Dr. Praveen Tayal drtayal72@gmail.com"
},
{
"id": 168880,
"tgt": "What causes fever and fast breathing in a child?",
"src": "Patient: My 8 month old has had a low grade fever all day today. We thought he was possibly teething. He now is asleep and looks to be breathing a little faster than normal. We checked his respirations and is between 60 and 64 for the past hour. Is this normal? We have been giving him motrin and Tylenol all day. Can that effect respiration? Doctor: Hi...fast breathing in kids of this age could be due to respiratory infection or due to high fever it self. But an 8 month old with a respiratory rate of 60-64 is not to be left with out medical attention. I suggest you you take him to a pediatrician right away.Regards - Dr. Sumanth"
},
{
"id": 123321,
"tgt": "How to treat a swollen ankle?",
"src": "Patient: I hit my ankle with an outside chair, it hit my ankle bone. I swelled badly, but I iced it. It is 2 weeks now and i have bruising on both sides of ankle, and the part I hit is sore to the touch. I am an avid walker, and am trying to do less, and ice after I walk. I have full range of motion, and can walk with no problem. What could I have done? Doctor: Hello, As this will be a bony hurt on the malleolus. Do hot water fermentation and use crepe bandage. This should help provide stability to the muscle and joint. Simple ankle stabilization exercises post this should help. Hope I have answered your query. Let me know if I can assist you further. Regards, Jay Indravadan Patel, Physical Therapist or Physiotherapist"
},
{
"id": 222514,
"tgt": "What are the chances of pregnancy through pre-ejaculatory fluid?",
"src": "Patient: Hello i was just wondering if there is anychance i could be pregnant. Basically me and my oh had unprotected foreplay and he had quiet abit of precum when he was rubbing his penis around my vigina. Then we had protected sex. I have been having some signs of early pregnancy. Sore breasts, brown spotting, bloating and extreme tiredness. So is there a chance i could be? Thank you. Very confused. Doctor: Hi dear, I have gone through your question and understand your concerns.Pregnancy is possible only when there is entry of sperms in the vagina.Sore breasts, brownish discharge are symptoms of early pregnancy, however the pregnancy can be confirmed only when the periods are delayed and the urine pregnancy test is positive.I will suggest you to wait for the periods, if delayed then do get a urine pregnancy test done to confirm the diagnosis and get treated accordingly.Hope you found the answer helpful.Wishing you good health.Dr Deepti Verma"
},
{
"id": 215317,
"tgt": "What causes throbbing, radiating pain from foot to hip region?",
"src": "Patient: Hello, I was hit in the back of my right leg about 10 days ago by the chair lift that I was loading myself onto (Skiing)... the entire 4 person chair lift hit my leg before making contact with anyone... it was very painful on impact. I am writing because I just went to the ER because the pain has not subsided... it s actually gotten worse. No bruise ever appeared - which amazes me. But I have a throbbing, radiating achy pain from my foot to my hip... but mainly located behind my right knee and along my right calf and in my right foot. It is painful to walk... They took an X-ray - no break. And they did an ultrasound to look for a blot clot.... they saw nothing. But the pain is just getting more noticeable... and it is not located only where I got hit. It s around my knee. I am limping a bit now... and nervous. I feel like I might have a clot and they missed it... what would you suggest as my next move? Doctor: Hello, Most probably it will be a neuropathic pain due to a prolapsed disc and subsequent nerve impingement. As a first-line management drug like gabapentin can be tried. If symptoms persist better to consult a neurologist and get evaluated. An MRI scan is required for further assessment. Hope I have answered your query. Let me know if I can assist you further. Take care Regards, Dr Shinas Hussain, General & Family Physician"
},
{
"id": 73818,
"tgt": "Suggest remedy for cough and runny nose followed by chest pain",
"src": "Patient: HI, im 17 years old and ive been sick for about 1.5 weeks now it started with runny nose and coughing and has now become a deep flemmy sound in my chest wen i breath heavily in and out and mi have a pain in the left side of my chest around my pec and wen i cough Doctor: Hello! you seem to have developed a respiratory tract infection. I would suggest you take a course of Tab. Amoxyclav for 10-14 days. Chest pain is quite common when we constantly cough. So once your Infection is controlled , your cough will reduce , so will your chest pain . Until then you can take simple analgesic like Tab. Paracetamol if you have too much pain.You can also get a Chest Xray done to ensure there s nothing else causing your chest pain . Hope my reply helps. Do drop in your valuable feedback."
},
{
"id": 151771,
"tgt": "Do these findings indicate any damage to my vertebra and what is the appropriate treatment ?",
"src": "Patient: Findings: Maintained lumbar lordosis and hight oflumbar vertebral bodies Normal sagittal dimeter of lumbar bony spinal canal Slightly reduced bright T2 singnal intensity of L4/L5 disc subtance,yet preserved hight. Doctor: hi welcome no need to worry about this, no slip disc ,no degeneration to bones except a little bit to the disc between 4th and 5th vertebrae you might have suffrein gfrom pain and radiating to back of thigh i think you no need to worry about this you go for ayurvedic treatment like abhyangam,pps,kateevasthy ,yogavasthy kerala style if it available you do at your area or i am specialised in this field contact in detail christyjoseph1@gmail.com 09895313720"
},
{
"id": 52417,
"tgt": "What does the following LFT report suggest?",
"src": "Patient: My son has been sick and just received blood tests back showing a Lipase result of 5, AST of 319 and ALT of 145. He cannot get back into the doctor until next week. So, we are trying to learn what these values mean and if he has liver disease. And if he should all doctor back to ask for additional blood tests before his next apt. (Hept A, B, C, etc) Doctor: Hello, Is he alcoholic? how old is he?? Yes, elevated enzymes may be called significant, if elevated more than 2-3 times above than normal. Normal levels of AST ranges from 5 to 43 or 45 units per liter and of ALT from 7 to 56 or 60 units per liter. Elevated enzymes may be seen in hepatitis due to alcohol or drugs or infection or storage disorders or alcohol, etc.He may advice tests according to the clinical presentation and age. Hope I have answered your query. Let me know if I can assist you further. Regards, Dr. Penchila Prasad Kandikattu, Internal Medicine Specialist"
},
{
"id": 111244,
"tgt": "Is the bulging disc in my vertebrae causing back discomfort?",
"src": "Patient: HI I HAVE BEEN HAVING DISCOMFORT ALL ALONG MY RIGHT SIDE OF MY BACK FOR A FEW MONTHS, I HAVE BEEN GOING TO A CHIROPRACTOR THAT HAS BEEN DOING ADJUSTMENTS EVERY WEEK. I THOUGH BY NOW IF IT WAS BECAUSE OF BULGING OF DISC IN MY VERTEBRAE IT WOUND BE GONE. COULD THIS BE STRESS OR ANXIETY? Doctor: Hello the bulging disc can cause discomfort leading to back Pain. The disc causes compression on the nerve roots n back pain. I suggest you to continue the back physiotherapy with hot fomentation and short wave diathermy. And then if still nonrelife get mri of lumbar spinal region n then consult again"
},
{
"id": 1233,
"tgt": "When can next pregnancy be planned after having an abortion?",
"src": "Patient: Hi, iam 29 yrs old. Iam having PCOD. I was pregnant and it got aborted in the 20th week . I am undergone hysteroscopy due incomplete abortion and i was advised to take Progynova and Orgamed for 2 cycles. Could u plz tell me when i can try for a pregnancy and scope for it? Doctor: Hi.Usually we recommend at least a 6 month gap from the time of an abortion or miscarriage before you can start trying to conceive, this would be most beneficial for the woman trying to get pregnant, and also gives the treating doctor enough time to rectify the issue within his/her best capabilities.Best wishes."
},
{
"id": 127123,
"tgt": "What causes tingling sensation and numbness in the right arm and fingers?",
"src": "Patient: for the past 3 weeks I get pins and needles a tigling feeling in my right arm that starts at my shoulder and runs down to my fingers and then my fingers fel numb and my entire arm feels heavy. it happens more when I lean with my elbow but it also happened out of no where as well? Doctor: Hello, The pain & tingling that you are experiencing in your arm is due to compression, irritation, damage of the peripheral nerves. Peripheral nerves can get affected due to diabetes or any other developing pathology. Visit ER or see a Neurologist, get physically examined and get all the tests for the developing Peripheral Neuropathy done. In the meantime take vitamin B with folic acid, take diet rich in calcium, iron and minerals. For pain take Acetaminophen (Tylenol)when needed. Hope I have answered your query. Let me know if I can assist you further."
},
{
"id": 119598,
"tgt": "Is wrist drop with numbness worrisome?",
"src": "Patient: I have wrist drop, and can move last three fingers more, side to side if not up and down. My thumb and pointer have lost some of their nunbness, starting at their tips first. My fear is that I still cannot move my wrist at all..how long before I should get worried? Doctor: Hello,I would explain that your symptoms are caused by a pinched nerve (possible radial nerve). For this reason, I recommend consulting with a neurologist for a physical exam and performing nerve conduction studies. If you have slept in a wrong position, this situation may last up to some weeks, depending on the degree of nerve compression.Take care. Hope I have answered your question. Let me know if I can assist you further. Regards, Dr. Ilir Sharka, Cardiologist"
},
{
"id": 185705,
"tgt": "Why does my tongue swell up at night?",
"src": "Patient: my tongue swells up at night. It has been going on for some time. But last night I had a burning sensation that kept me awake. My upper pallette is affected also. I suffer from long term heartburne also, are these related. I am a 15 year survivor of breast cancer. Doctor: thanks for your query, i have gone through your query. the swelling over the tongue could be because of the growth secondary to chronic irritation. it can be fibroma or hyperplastic candidiasis or fissured tongue. the burning sensation could be because of the fissured tongue. on the palate the lesion could be a kissing lesion that is spread of candidal infection from the tongue to the palate region because of contact of tongue with palate. consult a oral physician for further management. you can use topical antifungal like candid mouth paint and topican analgesic and anesthetic like anabel gel. i hope my answer will help you. take care."
},
{
"id": 84298,
"tgt": "Will dytor 5 help in getting rid of water retention?",
"src": "Patient: hi i had gone through a treatment for back pain 3 yrs ago nd the medicines had steroids which caused severe water retention in my body. since then i have been trying to get rid of it through diet control and exercise but all in vain. will dytor 5 help me in getting rid of this water weight?? Doctor: HiDiuretics such as dytor increases salt and water excretion thereby reducing edema.But it should not be used without ruling out other causes of edema such as cardiac or renal dysfunction.A thorough physical and clinical examination is required before taking the drug.Complete hemogram,renal,cardiac and liver function tests should be done.Hope I have answered your query. Let me know if I can assist you further. RegardsDr.Saranya Ramadoss, General and Family Physician"
},
{
"id": 152778,
"tgt": "Can a bulging stomach with a malodorous pus-filled mass be cancer?",
"src": "Patient: my mother had a fall in march 2011, since then she started having problem,incontinence,not eating properly,not sleeping well,and white blood cells more than red blood cells that blood transfusion has to be done in the hospital,doctor said she had myoma,suspected been there for 15 years, have also minor liver cyst(2008- doctor said she had shingle) shingle cause her to not eating the right food because of fear that shingle cause more pain when trigger by wrong food, thats why she was malnourishred, when she was hospitalized had lot of medication to treat potassium low level, urine antibacterial medications, has iron supplement to Keep the blood,had immucell to support immune system, after a week in the hospital my mom came home but continued to have problems such as gastric problem, swelling feet, stomach pain because there is a mass inside so big the size of a fist, oosing pus and smelly, the skin hole the size of a looney. where is this mass came from? THERE IS MYOMA IN THE UTERUS AND OUTSIDE THE UTERUS IS ALSO A FORM OF GROWTH, BUT THIS MASS IS DIFFERENT AND CAUSE PUS TO COME OUT OF THE SKIN TUMMY, my question is the bulging in the tummy develop a pus and ruptured that made a bigger hole the size of a looney, smelly, is it cancer or what is the cause? Doctor: HiPus samples are to be collected along with the specimen of uterine samples ,and microsco0pic investigations are to be done to find out if its cancerous or not.A mri abdomen should be done.markers of uterine carcinoma should be checkedthanks."
},
{
"id": 140832,
"tgt": "What causes numbness in head with headache and nausea?",
"src": "Patient: I have the feeling of numbness in my head along with a constant headache, nausea and sudden feelings of being on fire. The heat starts from my back to my chest and head. I don t have a fever. When sitting still I get a feeling like I m on a boat. Dizzy spells and shaky hands even though I ve had a meal and I usually need to eat something sweet to stop the dizziness. Doctor: Hi, Your description of constant headache along with vertigo or constant feelings of dizziness or other movements could be interpreted as a type of headache we refer to as vertiginous migraine which can also be accompanied by nausea and dysautonomias as you are describing of feeling sudden flashes of heat, etc. On the other hand, such a headache is usually episodic and may last for a few hours then, go away, later coming back. Other things to consider would be metabolic imbalances in blood chemistry or even hormonal imbalances in the thyroid profile or other glandular systems. A good physical examination and appropriate blood work may reveal the underlying source of your condition. Hope I have answered your query. Let me know if I can assist you further. Regards, Dr. Dariush Saghafi, Neurologist"
},
{
"id": 166196,
"tgt": "How to stop blood color motions in a child?",
"src": "Patient: dr.my son 1 year 3 months old he is suffering from loose motions past 1week. (loose motion lost one or two drops blood colour motion). what is the best way to stop blood colur motions until we go to the doctor for consultation. please help me dr. by. Mother P.Arulmani. Doctor: hi, blood colour stools in a child occurs due to acute gastroenteritis due to stomach infection, worm infestation. At this age, child usually puts everything in mouth and as a result, it results in infection in intestine and stomach. prebiotic like enterogermina ampule should be given once a day for 3 days to increase intestine eliminator. If bloody stools persist then antibiotic needs to be started. Kindly do stool for routine microscopic test and stool for occult blood test. This will help us in confirming the diagnosis. An examination by doctor should be done so that we may not miss any important finding. Review with reports. Take care."
},
{
"id": 50762,
"tgt": "Low GFR, high creatine, high BP. What type of diet and medicine should be followed?",
"src": "Patient: My general practicianer just informed me that my blood test shows a GFR level of 40 and it should be 60 and my Creatinine is high at 1.3. I have high blood pressure and am on Losartan. 135/75. I see by the web that I need a special diet, but that wasn't discussed. I feel I should get a doctor that deals with kidney disease but I don't know what type doctor I should get. Any advise? Doctor: Hello,GFR is a number based on your blood test for creatinine.It tells how your kidneys are working.A GFR of 40 suggests moderate kidney disease.There should be more tests performed on you to see the extent of damage to kidneys.These tests include various kidney function tests and 24 hour urine estimation of protein.You must keep your BP below 120/80 and avoid taking nephrotoxic medicines.Keep your sugar levels normal if you are diabetic.Eat healthy food which is less in fats and salt.Exercises to be done to keep a healthy weight.You must consult a nephrologist and get yourself thoroughly investigated and treated.Thanks"
},
{
"id": 44205,
"tgt": "Trying to conceive. History of recurrent miscarriages, fibroids, diabetes. Treatment?",
"src": "Patient: I am a 41yr old female with a history of recurrent miscarriages 5 before my successful pregancy and two after. She will be three in December. I have fibroids , diabetes which was diagnosed in my successful pregnancy. I have been trying to concieve for over a year now and have not been successful. Can fibroids contribute to a miscarriage and/or infertility? The only symptom I have is a bloated tummy but otherwise my periods are fine. Doctor: Hi, There are multiple problems concerned in your case. I would have liked to know your previous pregnancies were assisted or natural conception. There can be multiple causes for infertility. Yes, fibroid can be one of the causes of infertility but rarely by itself can cause recurrent miscarriages. However its important to know the size and location of fibroids for considering its treatment. I hope you have checked your sugars after your pregnancy, as it may persist even after pregnancy in few people. I would advise you to meet your gynecologist and consider starting the infertility treatment at the earliest, specially considering your age. All the best."
},
{
"id": 84817,
"tgt": "Is it okay to take i pill while on primolut?",
"src": "Patient: Hi Doctor, I have been taking Primolut-N to delay my period for the past few days (one tablet, twice a day). I had unprotected sex yesterday and want to take the I-pill. Please tell me if I should or should not take it, and if there will be any side-effects? Doctor: Hi,It may lead to side effects. Primolut-N contains the hormone progesterone, which prevents your womb lining from shedding and hence used to delay the periods. It is not a contraceptive hence and cannot prevent pregnancy following an unprotected sex. Barrier methods such as condoms or diaphragm are recommended following an unprotected sex while taking Primolut-N.Now, since you have had unprotected sex yesterday, the emergency contraceptive pill (I pill) may be taken as early as possible in order to achieve maximal efficacy. When you are taking pill along with Primolut-N it can affect your menstrual cycles causing irregular bleeding or spotting, stomach upset, vomiting, breast discomfort, changes in sexual desire, edema, etc. Hope I have answered your question. Let me know if I can assist you further. Regards, Dr. Mohammed Taher Ali, General & Family Physician"
},
{
"id": 83657,
"tgt": "Are flucos 150 tablet and doxy -1 safe?",
"src": "Patient: i hav infection in my urinary part frm lst 1 mnth i consulted dr.she gav me flucos 150 tablet twice in week and doxy -1 twice in a day .....n betadine oinement to apply on part.............betadine oinement i cn aaply....their......is ther any harm to me Doctor: Hi, Based on the history your doctor has prescribe flucos-150, doxy-1 and betadine ointment for the treatment of mixed vaginal infection caused by fungi and bacteria. Betadine is generally well tolerated however it may cause mild burning sensation at the site of application. Hope I have answered your query. Let me know if I can assist you further. Take care Regards, Dr. Mohammed Taher Ali"
},
{
"id": 90675,
"tgt": "What causes abdominal pain?",
"src": "Patient: I am a 42 year old healthy female and have experienced a severe sustained increasing pain in my upper right side abdomen in the past few days, I shift positions and that helps but the pain does not seem to go away. I was hoping it was gas or diet related so I kicked up fiber intake to flush my system and notice black pellets in my stool today with no blood present and I am in less pain at the moment, what does that mean ? Doctor: Hello! Welcome to HCM.Right upper abdominal pain suggests gall bladder related pain.As you said black pellets, there can be occult blood positive if you do stool exam for it.This can be due to bleeding somewhere in upper gastrointestinal tract.This can also be due a stone in ureter causing obstruction to urine outflow.In my clinic, many such patients do come to whom I advise ultrasound abdomen & pelvis, urine & stool routine, CBC.Treatment is done accordingly.Hope this information is useful to you.Wish you speedy recovery.Thanks. Regards."
},
{
"id": 16033,
"tgt": "Recurring rashes on the waist line, chest and genital area. No itching. What are these?",
"src": "Patient: I have a rash that started on my waist line....it goes away completely than comes back...I thought it was my laundry detergent....it spread all over my chest and genital area now....it did not itch or breakout all day than started late in the day...a shower...hot water makes it feel better...right now I am broken out but not hitching...redness and hive live stymptoms Doctor: Hello.There are quiet a few possibilities here-1)Probably seborrheic dermatitis, or dandruff of the non scalp area.2)Psoriasis 3)Contact dermatitis-allergy to something that you have come in contact with.Please see a dermatologist, because only after seeing one would ascertain the cause.It definitely seems curable."
},
{
"id": 123707,
"tgt": "What is the use of etody,zevit,prothiaden and rabekind in knee pain?",
"src": "Patient: My Mother is 80 Years old and currently as inspected by an Orthopadeic surgeon She is having no space is knee joints, and sever pain. He prescribed -- ETODY 90 ZEVIT PROTHIADEN 25 RABEKIND 20 Need to know whether these drugs are OK and sued for whic purpose Doctor: Hello, The medicine is used for this purposes since your mother is 80 and have reduced joint space. Etody - anti-inflammatory drug - used for reducing inflammation in the jointZevit prothiaden - is an anti-depression drug which will help her have good sleep in the night which may have got affected due to painRabekind - is for any gastric issues caused due to the medicine mentioned above. As the knee joint space is reduced, I will advise undergoing physiotherapy treatment like therapeutic ultrasound therapy and TENS therapy for reducing inflammation and pain. This works well in combination with drug therapy. Also, exercises will be implemented by the physiotherapist like static hamstring, static quadriceps and straight leg raise to improve the muscle strength and help improve the joint space. In my clinical practice of over 12 years, geriatrics respond well to a combination of conservative therapy. Hope I have answered your query. Let me know if I can assist you further. Regards, Jay Indravadan Patel, Physical Therapist or Physiotherapist"
},
{
"id": 195294,
"tgt": "What causes redness in area where sperm was spread during masturbation?",
"src": "Patient: Hi, im 28 male 5.8 and 128. I was pleasing my self sexually and I came over my chest and stomach. After shower, I notices redness in my skin where the sperm was? this is the second time i notice that. What is the reason for that, and is there any treatment. Thanks Doctor: Hello and Welcome to \u2018Ask A Doctor\u2019 service. I have reviewed your query and here is my advice. Is there any itching? Does it disappear immediately.? Possibility of allergy has to be kept in mind. Need further evaluation. Hope I have answered your query. Let me know if I can assist you further."
},
{
"id": 196867,
"tgt": "What causes penile discharge and swollen lymph nodes in the groin?",
"src": "Patient: Hi im 14 years old.. My penis still looks like the same size from when i was 11 i have started puberty and i also have Penile Discharge And Swollen lymph nodes in my groin and also my foreskin is still attached to my penis please help me.. im starting to hate myself :'( Doctor: Hi, stop worrying first. This won't solve your problems. You are just 14, and much age to go to get into problems. Cheers up. I hope there is no history of unprotected sex from your side. About your penile discharge, this can be due to poor hygiene, as your foreskin has remained attached to the glans. Try consulting a urologist for the same. The reason for discharge needs to be evaluated along with treating the discharge. Also if your care takers are having the same symptoms, that needs to be addressed first."
},
{
"id": 93374,
"tgt": "Foul-smelling discharge, watery, abdominal pain. Swollen cervix with red ring",
"src": "Patient: Hi, it s actually very embarrassing but for the about 5 weeks now I have what started of as a lot more normal discharge , it then turnt really badly smelling and now it s very watery and pretty much poaring out of me and I have a aching kind of pain around my left abdomin/hip area. I seen a doctor about 10 weeks ago who did some checks due to I was constantly having yellow discharge but I d had STI checks and they all came back clear so the doctor did further checks and said that my cervix was swollen with a red ring around it, but did no more checks.. Someone please help... Doctor: Hi ! You have not mentioned if you are also running fever or not. Also if you are sexually active, then an unnoticed pregnancy and a partial miscarriage might have to be ruled out. At home you can go for a urine preg-test. If this is not conclusive, you can discuss with your family physician and go for serum HCG test and an ultrasound abdomen. If all these things are clear, then you can go for a culture and sensitivity test of the discharge and then for appropriate antibiotics. If everything is normal and the problem persists, then you have to visit a gynaecologist, who may advise you some medications, and a dilatation and curettage, which possibly will relieve you of your condition.Wishing you an early revcovery."
},
{
"id": 191136,
"tgt": "TINY RED DOTS AND ITCHY LESION ON LOWER LIP. ANY THOUGHTS ?",
"src": "Patient: Hi. I am a 45 YO female 1/2 pack a day smoker with an itchy lesion on my lower lip where my cigarette rests. I ve had it for 2.5 weeks. Thought it was a cold sore at first as it started with a couple of tiny blisters. I ve never had a cold sore before. Saw dentist two days ago. He pointed out tiny red dots on my lips as well (bursted blood vessels). He thought it may be an allergic reaction to the Blistex I had recently used and to take a wait and see approach. I have not used Blistex for a week now and had never had an issue before. The dentist also took pictures of my lips. Worth mentioning, my teenage daughter is currently being treated for active MRSA on her buttocks . Although, because I don t have the pain associated with MRSA, I don t believe this is what is on my lips. I have light skin and have had severe sun burns in the past but not for several years now. I do not spend a lot of time in the sun and I live in the Northeast. Any thoughts ? Doctor: Hi, this may be a serious problem. plz dont wait, consult an ent surgeon n do biopsy done of that area. in the mean time apply some steroid ointment locally like kennacort mouth paint on ur lips, 3 TIMES A DAY."
},
{
"id": 4610,
"tgt": "Took siphene. Body has got over simulated. Keen on getting pregnant. Worried if this is normal",
"src": "Patient: I took siphene 50mg for 5 days. My body has got over simulated with the tablet and in response, on the 14th day of my cycle I have 5 follicles of size 2.6x1.9 , 2.5x1.9 , 2.4x1.7, 2.3x 2.0 and 2.0x1.4 in left ovary and 4 follicles in right ovary with the major one being 2.2x1.9. The endo reaction is 10.3.I am worried if this is normal and will the follicles rupture soon as I am keen on getting pregnant. Thanks alot in advance Doctor: Hello. Thanks for writing to us. You have overgrown follicles which are less likely to rupture to release an egg. These follicles will gradually regress before your periods. Please do not worry. Avoid taking siphene for next few cycles.I hope this information has been both informative and helpful for you. Regards, Dr. Rakhi Tayal ,drrakhitayal@gmail.com"
},
{
"id": 9924,
"tgt": "Suggest treatment for pain in scalp due to hair loss",
"src": "Patient: Hi my name Loma ,26 years old I have a pain in my scalp and hair pain .Also my hair fall i have baldness in my left side of my hair and the side i have pain What can be the cause..since i went to see dermatologist and they told me (Lichen planus) and some doctors alopecia ,,nothing works for me ...and these days i have pain every night specially night time ...can you help me with treatment of something can help me to regrow the hair and scalp pain. Doctor: Hi Dear,Understanding your concern. As per your query you have symptoms of pain in scalp due to hair loss which seems to be due to alopecia areata. Stress is becoming additional factor. I would suggest you to consult dermatologist for proper examination. Doctor may order skin patch test, blood test and physical examination. Doctor may prescribe you medications like finpecia at least for 3-4 months and can prescribe anti fungal treatment along with minoxidil spray. You should avoid wearing hat/cap for long time and use almond oil for regular head massage.Hope your concern has been resolved.Best Wishes,Dr. Harry Maheshwari"
},
{
"id": 121625,
"tgt": "What could cause severe pain in foot, recently suffered from Chikunguniya?",
"src": "Patient: Goodevening.. My name is manoj.I have been suffereing from lot of pain in my foot since last 3 months.I suffered from Chikanguniya 3 months back,but after that the pain in foot remained,making it difficult for me to stand evenly and walk properly. Can u suggest the reasons for this pain? Doctor: Hello,It may be due to chickengunia. For pain you can take tablet acetaminophen. As it's viral infection, there is no need for antibiotics. Keep your self hydrated. Take balanced diet. Hope I have answered your query.. Let me know if I can assist you further. Regards, Dr Shyam Kale Family and general physician"
},
{
"id": 205365,
"tgt": "Can medication for anxiety relief in these amounts be considered as low dosage?",
"src": "Patient: Thank you. I have been taking Klonopin under the care of my doctor for several years. Because I cut a .50 mg in half and take on this schedule: through the day I take a total .75 and at night ,.50. Is that what you call a low dosage? I m 78 and in good health. This is for anxiety relief. Doctor: If you are on this dose since long then it is ok, as long as your anxiety is under control. for all benzodiazepine, you will develop tolerance. so to get the same effect you will have to increase the dose. But does is ok."
},
{
"id": 144593,
"tgt": "Suggest treatment for microangiopathic ischemic disease",
"src": "Patient: have a hx acoustic neuroma on rt. 1996. x2 surgery. and small piece of tumor still present and unchanged. lately tingling hands feets and around mouth. which thought was from steroid that had been on for uri. and had larygoscope showed rt side vocal cord paralysis. had recent mri neuroma unchanged but again considered was chronic microangiopathic ischemic disease. can you explain microangiopathic ischemic disease to me and if it is something to worry about or could this be causing the tingling and also forgot to mention have had chronic headaches for a long time too? Doctor: HelloMRI findings suggests chronic microangiopathic ischemic changes( due to compromised blood supply).These are generally due to involvement of small blood vessels.It is generally due to condiotions like hypertension,diabetes mellitus,altered lipid profile etc.So,you may need clinical correlation and screening for these conditions.These are not very serious findings and it only need treatment of underlying cause.Get well soon.Take Care"
},
{
"id": 96072,
"tgt": "What should be the normal range of pus cells and R.B.C ?",
"src": "Patient: If a stool microscopic examination shows 1-2 Pus cells and 2-3 R.B.Cs, is that within the normal range ? Doctor: Hi; welcome to HealthcareMagic If you don't have any dysentery;diarrhoea or any other complaints then the result of stool test showing 1-2 pus cells;2-3 RBC's is normal no need to worry. Thanks"
},
{
"id": 25034,
"tgt": "What causes swelling on leg in a person with high blood pressure?",
"src": "Patient: My boyfriend is 53 years old he s got swelling in his left leg down the bottom below the knee it s red radish looking and stuff but everything is down their spouse his growing aches and doesn t feel right and he s super tired even Empire a referee in for sports has high blood pressure you take flomax and take high blood pressure pills but he hasn t had them for the last month and also his leg has a burning sensation to it with along with the swelling Doctor: Thanks for your question on Healthcare Magic. I can understand your concern. In my opinion, we should first rule out deep venous thrombosis (DVT) in his case. Uncontrolled hypertension can damage blood vessels especially lower limb vessels. And this damaged vessels can cause thombus formation in it. When this thombus occlud the vein, it hampers venous return from that limb. So fluid and blood tend to accumulate in lower limb. So possibility of DVT of left lower limb is more in his case. So get done venous doppler study of left lower limb to confirm DVT. Control of blood pressure is must. If DVT is there then he may need surgical removal of thrombus.Hope I have solved your query. I will be happy to help you further. Wishing good health to your boyfriend. Thanks."
},
{
"id": 147210,
"tgt": "Numbness in little finger, tingling in other fingers, have ebstein's anomaly. Treatment?",
"src": "Patient: I consulted my cardiologist about this, I have been passing out I expected it to be from my ebstein s anomaly but I was told my heart beat was normal when having a few episodes, I have recently been having numbness in my pinky and tingling on the rest of my fingers on my left hand . I am not sure what to do from here can you please you please guide me in the correct direction? Doctor: HIThank for asking to HCMI really appreciate your concern looking to the history given here I could say that your tingling sensation complaint of little finger is nothing to with the hear congenital valve disease these are two different conditions, the tingling must be due to the cervical spine disease, and for this you need to go for x-ray of cervical spine, take care and have a nice day."
},
{
"id": 190791,
"tgt": "Why are my lower gums dark and strange looking?",
"src": "Patient: My lower gums down below lower lip , are very dark and strange looking. The upper gums around my upper geeth are very pink and healthy looking. What is going on with my lower gums? I am 77 years old and weigh about 200 lbs. There is no pain involved in this gum change, buy I am in the dark about what has happened.Thank you Doctor: Hi Your lower gums have been of the same color and nature from starting or have they started appearing like this from few months or years like this ? if they are from starting like this, they cant be pathological, so nothing to worry about. but if they have recently appeared like this , then it can be due to gingivitis or periodontitis or some blood disorder or due to some drug reaction. if so id the case , then go for a oral check up to your nearest dentist and rule out all possibilites. if its the gingivitis or perodontitis, scaling ( cleaning of teeth ) will help, if required gum surgery can be done also. if other reasons, then you have to go for their treatment. if its just from starting and you want them removed for esthetic purposes, go for laser treatment and your gums will be normal in no time. wishing you good health with best regards Dr Ankit"
},
{
"id": 139747,
"tgt": "Suggest remedy for damage in autonomic nervous system",
"src": "Patient: I took a fall in 08 about 9 ft landed on buttocks, did some damage to my back, since then my autonomic system has been have issues, from forgetting how to swallow to breathing, my digestive system was shutting down, I have constant ringing in my ears, my vision is blurred, my skin and nails are getting bad, dizzy all the time with no balance,all my extremities were numb. My blood work says I am hydrated. I had a MRI, Mra both were fine. They thought I might have colon issues, but my colonoscopy was fine. I did feel better after the peg solution and gatorade. But slowly going right back down. also I for the first time in three years I was having normal bowel movements, now I am having the loose or diarrhea. I cannot sleep without pills never had a problem before the fall. I dont know if I have everthing here, but if you have any Ideas it would be helpful ty Doctor: Hi, Without an exam, I can only give general information. The problems occur in basically every part of the nervous system. A first part would be to actually document how bad the worst parts actually are. Actual measurement of the not sleeping, the not breathing, and other autonomic problems can all be documented with a sleep study. It is quite possible to have something needing artificial ventilation or.... more commonly... just some mild anti-anxiety medicines. Cannot say in your particular situation.. which is why, the tests. Hope I have answered your query. Let me know if I can assist you further. Regards, Dr. Matt Wachsman, Addiction Medicine Specialist"
},
{
"id": 165886,
"tgt": "What causes testicle redness with irritation?",
"src": "Patient: I don t believe either of my son s testicles have dropped. He is 3 and has been complaining of having a rock in his pee pee. His testicle area is marble size, and has been turing bright red. Is this serious? Could there be other causes for the irritation? How do you know what to look for? Doctor: Dear parent , undescended testis is a serious problem and needs correction as soon as possible as any further delay can lead to infertility . the redness could be due to a urinary tract infection"
},
{
"id": 175685,
"tgt": "What is the treatment for productive cough?",
"src": "Patient: Dear Doctor,My bhanja is 7 years old. He had developed productive cough but he can't out his sputum. Sometimes headache. After that he is treated with clavam ( Amoxycillin + Clavulanic Acid) Now, Hi is better than earlier but his report shows ESR = 25 What may be the differnetial diagnosis? Please help me. Doctor: Ho...by what you quote I feel that he might have been having sinusitis and this has gotten better with the appropriate antibiotics. But this sinusitis is known to have recurrences and I suggest you get back to your paediatrician if the same symptoms recur again.regards - Dr. Sumanth"
},
{
"id": 38147,
"tgt": "What causes rash on body after eating burger?",
"src": "Patient: My husband ate a burger from burger king with onions and became real sick and almost passed out but with the help of cold wet wash clothes, Zantac and then Benadryl after a rash on his feet under arms & upper part of leg and privacy part is feeling much better. Is there any thing more that he can do for the rash. Doctor: Hello, Thnx to contact us. I understand your concern. If I am your treating doctor I advice you that it is a allergic reaction, and good thing is that you had taken the benadryl. I add to this your regime take a 2 day short course of Prednisolone. With this two tablet all of your body rash disappears. In future take due care for not coming in contact with such a allergen to prevent the future allergic reaction.If you have anything else to ask, kindly contact me.Wish you a very good health at health care magic. Dr. Arun Tank. Infectious Disease."
},
{
"id": 194534,
"tgt": "What causes pain in testicles?",
"src": "Patient: i'm 19yrs old. for the last 3 days my testicles have been hurting on and off, there are no lumps and i have not been sexually active for the last 2 years. only thing i have done is masturbated. i'm not feeling sick or anything like that. just that they hurt. sometimes the pain goes down my leg. but that's about it. they are normal color and everything Doctor: Hi, You may have a condition called varicocele. This is the dilation of the veins in the scrotum and this causes pain during physical activity and ejaculation. There is also a chance that you may have the pain because of frequent ejaculation. If that is the case then I suggest you abstain from is relation until the pain goes away. Hope I have answered your query. Let me know if I can assist you further. Regards, Dr. Manuel C See IV, Urologist"
},
{
"id": 194419,
"tgt": "What causes red spot on the forehead of the penis",
"src": "Patient: i am 27 yrs old weight 5'ft 65kilo, i have never on hospitallized before i have sex with my gf that was 8 months ago when i get vacation then this february i was wondering i have small red spot on my forehead of my penis, i think that that is nothing then disappears then it comes again and its becoming a small line no can you tell me what exactly is this and how can i cure this one Doctor: Hello, As per your history, it may be dermatitis or penile irritation. For that, you can apply hydrocortisone ointment locally. Take tablet loratadine for itching. Avoid harsh cosmetic products. Maintain proper hygiene. Along with that watch for symptoms like fever, penile discharge, open sore, testicular swelling. In that case, you may require a detailed evaluation by a physician. Hope I have answered your query. Let me know if I can assist you further. Regards, Dr. Shyam B. Kale, General & Family Physician"
},
{
"id": 196723,
"tgt": "Suggest treatment for premature ejaculation",
"src": "Patient: Hi. Dr. Dear Sir i had master basin since my school life approx. last 10 years on daily basis. i got married in Nov. 2009. I am suffering from early ejaculation or Premature ejaculation. I took a Homeopathy drops to reduce ED Problem, but it remains same. Please help me out to do sex for a long time. And Improve the time of intercourse. Doctor: Dear user,Thanks for writing to healthcare magic.I have gone through your question and understand your concerns to delay the ejaculation for prolonging the duration of sexual intercourse.The following methods are helpful to delay the ejaculation and orgasm.1. Stop-start technique of behavior therapy.2. Female on top with active role and movements.3. Double condom4. Kegel exercise also helps in some cases. You can learn the same from internet. (As if try to hold urine intermittently during voiding)5. Penile squeeze technique applied at the point of ejaculatory inevitability.6. Applying a local anesthetic ointment to glans penis prior to penetration.7. Leading partner to orgasm by exploring and stimulating other erogenous zones. (Other than vaginal containment movements only)8. You can do masturbation before sex (at least 2 hours before). It will make seminal vesicles somewhat empty. So, at the time of sex, semen will take more time to come out.Remember that do not evaluate the sexual satisfaction and success ONLY in terms of TIC TIC OF TIME, SEX STROKES , NUMBER OF THRUSTS OR HOURS. It is the PLEASURE (touching and being touched) NOT THE TIME is the key to healthy sex life.There are medications like Paroxetine20 mg or Dapoxetine 30 mg with Sildenafil 50 mg which helps to delay orgasm with improving erectile function. These medicines must be taken under medical supervision.Wish you good health."
},
{
"id": 92444,
"tgt": "Pain in lower back, lower abdomen. What could it be?",
"src": "Patient: my lower back and lower abdomen has been hurting on an off for about 6 days now what could it be? I drink a lot of water so I don t think it could be a kidney infection. my girlfriend had a uti not to long ago and we have unprotected sex is there anyway I could of picked it up? what else could it be? Doctor: HiPlease go for ultrasound abdomen to rule out stones in kidneysUrine culture is also advisable Drinkig lot of water does not prevent Urinary tract infectionTAKE CAREDr Lal Psychiatrist"
},
{
"id": 175194,
"tgt": "What causes constant vomiting despite having antibiotics for fever?",
"src": "Patient: My son (3 and half years) had febrile seizure due to high fever on 27.11.2014 was in hospital for 2 days. Since 30.11.2014, temperature above 100 F has not occured. But now he is vomiting whatever he eats. Has little bit cough also. We took him again to doctor and he said its due to gastric problem because of antibiotics (cefdinir) and asked to stop it. He prescribed Emeset (thrice daily) and Nexpro junior sachet (twice daily) for two days. But even at the end of two days vomiting is still happening. He had rice yesterday night and did not vomit. But today he had oats and vomited. Otherwise he looks okay. Better active than before. Earlier he used to turn away face from food. Now he eats but vomits after some time. The vomit is mainly watery and occasionally undigested food goes. He has loose motion at the time of giving antibiotics but has now decreased and seems to be okay. Should I continue Emeset and Nexpro till he vomits no more ? Doctor: Hi...looks like your son is on the recovery path from a viral illness. It is very common for kids to vomit during recovery period and in convalescence. Fever of few days without any localizing signs could as well a viral illness. Usually rather than fever, what is more important is the activity of the child, in between 2 fever episodes on the same day. If the kid is active and playing around when there is no fever, it is probably viral illness and it doesn't require antibiotics at all. Once viral fever comes it will there for 4-7 days. So do not worry about duration if the kid is active.Regards - Dr. Sumanth"
},
{
"id": 9319,
"tgt": "Suggest creams for dry scrotum skin",
"src": "Patient: Hi, I have had a very dry scrotum which I tried to treat with moisturising cremes but it is quite bad now, bleeding and obviously very uncomfortable. I had this for the last 6 months at least, on and off but hasn t gone away for at least a month now. Do I have to worry? And what sort of Creme would be best to use? Doctor: HIWell come to HCMChances of fungal infection may be likely and this can be well treated with \"Terbinafine cream\" and Beclomethasone Cream\" continue this for some time, this would come around soon, take care and have a nice day."
},
{
"id": 140420,
"tgt": "Need treatment for brain problem",
"src": "Patient: sir i have a problem regarding my brain and i m in undercover of Dr sanantan rath since 9 months .now its time to again 4 checkup as dr told ,due to my annual examination i cant go there...my question is sometime in my mind 1 thought is coming but when it is over i cant remember it but after that my head starts paining..what is this sir Doctor: Hello, The history is incomplete but there are many causes for what you describe where a person thinks of something which may not completely be able to be pulled up to full consciousness followed by a headache. For example, stress, anxiety, fear to name a few highly emotional states that can block thoughts and result in headaches. Organic causes of you're describing could be some type of aborted seizure disorder, migraine headache aura, or even medication side effect. I hope you are able to find a way to see your doctor for your regular checkup. In the mean time, try practice deep breathing exercises along with visual imagery and physical relaxation while quietly lying on your back and you may be able to reduce or eliminate some of these episodes and improve your memory or concentration at the same time. Hope I have answered your query. Let me know if I can assist you further. Take care Regards, Dr Dariush Saghafi, Neurologist"
},
{
"id": 164617,
"tgt": "Suggest remedy for regaining fair complexion in an infant",
"src": "Patient: Hi, may I answer your health queries right now ? Please type your query Hi my 5month old baby was born fair. He got jaundice when he was two days old. His complexion became tan. Now his face has gone back to the fair complexion but his legs are getting tan day by day. What s wrong? Doctor: HiTanning is not associated with jaundice. skin colour depends on genetic inheritance , sun exposure & hormones. See if babies legs are more exposed to sun . If it is unusual hyperpigmentation get him evaluated by pediatrician for hormonal imbalance ( Excess of ACTH hormone causes hyperpigmentation).continue breastfeeding if on exclusive breastfeeding. start complimentary food after 6 months. Best wishes"
},
{
"id": 99077,
"tgt": "Suggest remedy for pain in shoulders, chest and back",
"src": "Patient: My son has had pains in his shoulders, chest and back (rib area) for the last year. His Dr said it was probably growing pains. They are very painful and getting worse as time goes on. He describes them as sharp, bubble like pains like they are trying to burst out of him. He has also gotten them on his hips. He also gets a tingly pain in his left elbow. The chest pains are just sharp with no bubble feeling to them. Any ideas as to what this could possibly be? More info on him, he is 11 yrs old, asthmatic (less than 2 inhalers per month), active, otherwise healthy. His diet is mostly veggies, fruit, wild game (deer, turkey, etc) and very little sugars and milks...he gets his calcium and other nutrients from vegetables. I am not looking for a diagnosis, just trying to make sure that it is just growing pains and not something more serious.Thank you for your advice! Doctor: Hi,Welcome to health care magic,I have read your query and understand your concern. As you have mentioned you have pain in your shoulder to chest and back rib area which is growing but does not seem to be serious.Here, I would like to know that whether you have any history of diabetes or thyroid illnesses or any injury to that joint.There could be several causes of shoulder pain such as rotator cuff injury or tear or rotator cuff tendinitis (inflammation of its tendons).The initial treatments for all these are, 1. Avoid repetitive overhead activities 2. Do not carry any heavy object or bag on affected shoulder or arm.3. Apply ice packs for about 15 minutes twice or thrice daily. 4. If you have learned how to do shoulder exercises then continue with that it's could be effective . 5. Apply diclofenac based topical ointment which is more effective than paracetamol based.(available over the counter)6. Can take oral pain killers if pain is very severe. 7. Take Re-nerve Plus,as it is just a multivitamin supplement which is good for nerves (more useful in diabetic patients).If still pain is persisting then proper examination of shoulder joint is required to find the exact cause and if needed X-ray and MRI should be done. You can consult nearby doctor or orthopedician who can write X-ray and MRI of the shoulder after taking complete history and thoroughly clinical examination. Thanks and Regards,"
},
{
"id": 193104,
"tgt": "Can hypospadias affect male fertility?",
"src": "Patient: My husband and I are ttc for 6 months I have three kids from a first marriage but my new husband has hyposdias is this causing us not to become pregnant, we also did a sperm count test he tested with normal sperm count? I'm getting dissapointed and feeling hopeless Doctor: Hi, Yes, hypospadias can affect the fertility in males.Hope I have answered your question. Let me know if I can assist you further. Regards, Dr. S. R. Raveendran, Sexologist"
},
{
"id": 42047,
"tgt": "How can amenorrhea with cramping be treated?",
"src": "Patient: HI, im a 22 yr old who hasent been sexually active for the past 11 months. my period has stopped completely. Im not on birth control nor is there any possible way im pregnant. Any thoughts of while my cycle has stopped. Also, for the past few months i have been having serve cramps. any thoughts? Doctor: Hi, Welcome to HealthcareMagic . The cause are many like hypothroidism , sex hormone imbalance , block in the cervix, Polycystic ovarian disease etc. There are no chance of pregnancy . I suggest you to get your thyroid and sex hormone profile done after consulting gynacologist. And also ultrasound of abdomen. These will tell the cause and it may be treated accordingly. Hope I have answered your query. RegardsDr.Deepika Patil"
},
{
"id": 128600,
"tgt": "What causes leg pain, back pain, joint pain and tiredness?",
"src": "Patient: I ve had leg pain and back pain . My body full of fluid . Pain inside .bloated stomach . Swelling joint pain . Very tired no energy. Went to dr . Said take 3 fluid pills . I ve gained 10 pounds in last 4 days . They gave me steroids last Thursday ..been having regular bowel movements . Last nite lots of gas and I used bathroom n looked like I shit @-10pds. . Tomr goin for a CT scan , been goin thru this fir months , and they say they don t know and are puzzled , my stomach is blowed up like imma 9 months pregnant .!!!!!! What can I do can u give me any idea ??????? Please . Doctor: I still don't understand why you took steroids.steroids are generally taken for inflammatory conditions like autoimmune disorders , any inflammatory bowel disease etc.Gaining weight and flushing feeling, bloated feel might be due to steroids only and stools might be stopping disease activity.consult internal medicine specialist for work up and management"
},
{
"id": 148030,
"tgt": "Can dilo BM be given to a person with slight block in his brain?",
"src": "Patient: hi sir, DILO BM is suggested to my dad by a dotor, he has slight block in his brain and has been taking medication for that and has undergone engeo plastry. can he take DILO BM? he has been given SAFECET and Asthlin with DILO BM Waiting for your response... Regards Hema Doctor: hi HemaI am not sure exactly what you mean by \"slight block\" in brain. i am assuming you mean he has had a stroke. and an ischemic stroke at that. So you might want to confirm that assumption when reading the further suggestionHe is on safecet, asthalin and dilo bm has ambroxol, so i am assuming he ahs an associated respiratory condition like asthma, copd etcI will therefore suggest that you can give it to him,However do talk to your neurologist, because i am unaware from your query as to the stage of block. occasionally ambroxol has a sedative action or may cause vertigohence neurologist may avoid this medicineBest of luckDr Saumya Mittal"
},
{
"id": 188032,
"tgt": "What is the treatment for sore in back of mouth after teeth cleaning?",
"src": "Patient: Yes, I had a teeth cleaning two days ago. The Hygienist was rough while scaling around my gum line that she pricked my gum near #21 tooth and had position the Saliva Ejector in my mouth while cleaning my teeth. Now the back of mouth and has a sore from the vacuum. What to do? Doctor: Hello,It looks like that the hygienist has done an improper scaling because of which your gums have now become sore. You apply a topical ointment named mucopain or pansoral gel and even a gum paint which will relieve your symptoms.But still i think once you should visit the dentist and explain him the problem as in future it never happen.Thank you"
},
{
"id": 1809,
"tgt": "Are there pregnancy chances with non penetrative sex?",
"src": "Patient: doctor,me and my girlfriend were doin foreplay yesterday and we started having oral sex ..i was fingering and took the hand out and adjusted my penis and again i started fingering her...i neither cummed at that tym nor cummed on her .. i fear she might get pregnant ..can u pls help us out? Doctor: Hi, if there was no contact with precum or the semen, then there is no chance of pregnancy. So dont worry. Hope I have answered your question. If you have any other query I will be happy to help. Regards Dr khushboo"
},
{
"id": 76866,
"tgt": "What is the reason for an easy lung surgery with no severe bleeding?",
"src": "Patient: a surgeon removed three adjacent bronchopulmonary segmants from the left lung of a patient with TB. Almost half of the lung was removed, yet there was no severe bleeding and relatively few blood vessels had to be cauterized. Why was the surgery so easy to perform? Doctor: HiIn general there isn't much in the way of blood loss connected with this kind of surgery so there isn't anything unusual or unexpected here"
},
{
"id": 114073,
"tgt": "I still have back pain after undergoing operation. I took a nerve block injection",
"src": "Patient: Hi I had surgery on the 19/5/2011 it was s1decompression and I still have a lot of pain in left buttock and left leg also 7days after op had a nerve block injection and still have pain and sometimes I think I have worst pain now then before op please help Doctor: I do not know the reason for the back surgery. I suppose due disc pathology or spinal stenosis in lower level of the back(L4-5 and L5-S1). During surgery there might be touching of surgical instruments of the nerves which leads to edema and more pain which can last up to 3-4 weeks and treated by fysio and pain killers.A more serious situation might be nerve damage due to edema or trauma to the nerve by the surgeon.To solve this an NMR after a periode of 4 weeks needs to be done. Important is : do you have loss of power (standing on heel or toes)or sensory disturbance on the frontal or posterior part of your leg."
},
{
"id": 208524,
"tgt": "What causes confusion and anger?",
"src": "Patient: Hi I m 22 female having 2 children since last years I have been feeling some problem on my head.I feel too much confusion and I try to get some one message my head and I feel releaf also feel too much angary for every small bit and I can t control my self some times I abuse for every body Doctor please advice me Doctor: Hi,From what you have mentioned, it seems that you are preoccupied with something or feeling low which is making you very irritable. As a result, you get angry frequently. You confusion can also be due to the same. Such symptoms are highly likely to be due to an underlying depressed state.I would suggest that you consult a psychiatrist for complete evaluation of your symptoms. You will respond well to treatment with antidepressants like escitalopram. It will help you get rid of the depressive symptoms, which in result will help control your anger and feel better.I do hope that I was able to answer your query. Best wishes."
},
{
"id": 39469,
"tgt": "What causes lower abdominal pain during yeast infection?",
"src": "Patient: Last week while on my period i had developed a yeast infection. After it went away i felt a little pressure in my lower abdomen, then during intercourse with my husband I experienced severe pain that felt like my ovaries were swollen. I haven't had intercouse since then but im still experiencing lower abdominal pressure and it feels more like severe cramping. What do you think it is? Doctor: Hi,Welcome to HCM!From what you have described, you are likely suffering from Pelvic Inflammatory Disease. It is an infection of your pelvis organs and are characterized by lower abdominal pain, cramps, discharge, fever etc. It is a treatable condition. Visit a gynaecologist. With regards,Dr. Sridhar"
},
{
"id": 161733,
"tgt": "Suggest remedy for mucus in stool",
"src": "Patient: my baby is almost six months old by this january 4, these past days, she began to get slimy stools and i got stool test for her and it came out that she had amoeba, then the pediatrician prescribed methanidazole for her,after taking 6 ml, she began to get swollen gums and she was crying and didn t want to drink milk. i bought her to see another doctor and got her stool examined again and found out that she didn t have amoeba but she had bacteria because there was mucus in her stool, the doctor prescribed chloramphenicol and she took 7 ml already out of the 28 ml that she must take. and the doctor asked me to reduce the scoop of formula milk. now, the stool always comes out every time she farts and the stools that come out are just in small quantity and the stool is still the same, still have mucus. what is wrong with my baby, should i continue the chloramphenicol palmitate. please help me Doctor: Hi, Please be informed that amoebic cysts are common in the stool routine and it needs treatment only if there are trophozoites in the stool routine. Treating it with metronidazole might have induced antibiotic-associated diarrhoea and I suggest you stop all antibiotics and give only Saccharomyces boulardii probiotic. Hope I have answered your query. Let me know if I can assist you further. Regards, Dr. Sumanth Amperayani, Pediatrician, Pulmonology"
},
{
"id": 110208,
"tgt": "Suggest remedy for back pain",
"src": "Patient: Hi, I have had a backache on the right lower side of my back for a few days but for the past two nights I have felt sick and on both occasions have woke up at 4 am the same time each night to be sick after that I am fine. What is wrong with me ? Help! Doctor: Hi,Welcome to healthcare magic.After going through your query I think You are suffering from acute backache. Treatment of it is rest and analgesics . Diclofenac three times a day after meals is effective. Sometimes strong analgesic(such as ultracet three times a day after meals) is required. OMEPRAZOLE 20 MG before meals prevent acidity caused by analgesics .Sometimes vitamin D deficiency is the cause so get your vitamin D checked .If it is low then vitamin D supplementation(Bon DK 60K weekly with milk) can be taken.To further investigate MRI of the region is advised. I think your query answered.Welcome to any follow up query"
},
{
"id": 87654,
"tgt": "Suggest medication for severe abdominal pain & depression",
"src": "Patient: My name is Elli and i am 16 years old, i have been ill with something that the gyneacologyst in my town cannot recognise. My illness includes excrutiating pain in the lower-abdominal area, and i also have servere depression. I have been taking 'Yasmin', a contraceptive drug to ease the pain of my periods, I used this for 3 months with a 5-day gap, as you would normally take it. For the past 3 months, i have used this as she ordered me to, to take it once a day with no gap in the month. So, I have not been having a period at all for 3 months, and now through the past week, i have had a dark-brown substance which has no smell, i am guessing that it is blood, although when i used to have periods, they were very heavy, and this is not.I do not know why this is happening to me, please can you provide me with an answer or advice please? P.S. i am also taking several prescribed pain-killer drugs to ease my pain, ibuprofen, codiene phosphate, St. John's Wurt (for my depression), and audmonal. Doctor: Hi Elli...Thanks for your query and specific information and understanding... Pain in lower abdomen and severe depression and inability of the gynecologist to understand your problem can be due to :-Yasmin has altered your hormonal environment; so no periods for 3 months; I would advise you the following:Ultrasonography of the abdomen and the pelvis to see for PCOS and / or any other problem.Hormonal assay of the blood.Second opinion of the General Surgeon to rule out any other possibility like appendicitis, colitis or so.Proper diagnosis will lead to a proper treatmentTry to take least possible medicines as there is likelihood of addiction to the medicines you mentioned like Ibuprofen, Codeine, St John's Wurt."
},
{
"id": 222840,
"tgt": "Does pain in back and thighs due to SPD increase during pregnancy?",
"src": "Patient: Hi I am 29 weeks pregnant and been told by physio that I have SPD. I experience pain in my left bum check, lower back and inside of my thighs. She gave me a support belt, exercises and I have been perscribed some pain killers. She has now discharged me and told me to deal with the pain. However I feel it is just getting worse. Will it get worse the further my pregnancy goes? I know I am suppossed to rest more but find it hard as work 3 full days and have a 16month old. Doctor: Hi,Symphysio pubic dysfunction will get worse as pregnancy advances ,and the treatment is rest and if its too bad braces to help you move around. Pain killers are also helpful,but you have to realize that all these are of only limited help. This pain will get away once you deliver.It is not a serious condition and doesn't have any long term effects but it is a very painful situation to be in.You will recover completely by 3-6 months post delivery.Labor can also be affected and please try for a normal delivery as, caesarean does not benefit the condition in any way.Try to take as much rest as possible and also do the exercises prescribed by the physio.Hope this helps.Regards."
},
{
"id": 150362,
"tgt": "Randomly passed out. No convulsions. Felt outrageously, uncomfortably stoned. Recommendations",
"src": "Patient: Hello! 26 yo female. Last night I randomly passed out. I also wet myself. My boyfriend said when this happened, I had my eyes open and wasn't responding to him. There were no convulsions. I had eaten 3 hours before that, wasn t feeling dehydrated, and I wasn't particularly hot. All of the sudden I felt outrageously, uncomfortably stoned and within 15 seconds passed out. When I came to 30 second later, I felt fine. This is the first time this has ever happened. Thanks! Doctor: Hi, Thank you for posting your query. Based on your symptoms, you seem to have suffered a generalized seizure (an epileptic attack). You would require evaluation by a neurologist and investigations such as MRI brain and EEG to confirm the same. Then, treatment would consist of anti epileptic drugs. Please get back if you require any additional information. Best wishes, Dr Sudhir Kumar MD (Internal Medicine), DM (Neurology) Senior Consultant Neurologist Apollo Hospitals, Hyderabad, My personal URL on this website: http://bit.ly/Dr-Sudhir-kumar My email: drsudhirkumar@yahoo.com"
},
{
"id": 45625,
"tgt": "Suggest treatment for stage 5 chronic kidney disease",
"src": "Patient: ok I am a diabetic a1c averages 7 to 7.5 I am in stage 4 to stage 5 kidney disease my doctor says my blood acids are higher than he wants he has said to use sodium bicarbonate to ease the acid. I have been told to avoid any salt is this not contradictory? Doctor: Hello, You are right. The hypertensive patient should avoid sodium in the diet and from medication as well. But if acid level very high means metabolic acidosis present than it can be counteracted by bicarbonate. So I suggest you follow your doctor advice to treat your condition. You should get treatment by nephrologist only. Hope I have answered your query. Let me know if I can assist you further. Regards, Dr. Parth Goswami, General & Family Physician"
},
{
"id": 135041,
"tgt": "Is it safe to take Crisanta L S for lumbar spondylosis?",
"src": "Patient: I took crisanta L S for 8 months.Now, i am having a problem of lumbar spondylosis.Is it because of crisanta L S ? I got acne on my face in august 2013.Then my dermatalogist prescribed me this medicine.then on,I took 8 packets of this medicine.my acne are treated but problem of lumbar spondylosis has come out in spinal X-ray. Doctor: hello healthcare user, crisanta LS is a oral contraceptive..it is not related with lumbar spondylosis..crisanta LS has its set of complications and side effects,,,but lumbar spondylosis is not associated with crisanta...thank you"
},
{
"id": 160322,
"tgt": "I have a little bit fear that it would be cancer or not ?",
"src": "Patient: Hi, I have a small wound in between my back buttocks for the past two years. some times yellow color fluid and some time blood also come in that wound. still now I didn't consult any doctor. would you please give your suggestions whether it is a cancer or not. I have a little bit fear that it would be cancer or not Doctor: First of all consult a doctor, let tht doctor examine what exactly is the problem..get it treated, get a biospy done from that wound to see if there is any cancer changes in that..if there is any cancer changes, the type of cancer will direct further management steps.."
},
{
"id": 53770,
"tgt": "What causes grade II fatty liver?",
"src": "Patient: My husband is 33 years old and does not smoke and drink. Mostly non vegetarian and doesn't have a balanced diet with vegetables and fruits.Tests end of December 2010 results are Cholesterol is 180, Triglycerides 308, Cholesterol HDL 31,Direct LDL Cholesterol 93,CHOL/HDL RATIO 5.8,LDL/HDL RATIO 3.0 And Very Low Density Lipoprotein 61.6.While the results as on 22.5.10 were Cholesterol 179,Triglycerides 201,HDL - Chosterol 31, LDL Cholesterol 112, VLDL Cholesterol 40.2, LDL /HDL RATIO RATIO 3.61 while CHOL/HDL Ratio is 5.77.On 23.2.11 LFT was done and it was as follows : - SGOT/AST = 55, SGPT/ALT = 108, GGT = 41,Alk.Phophate=88,Total Protein =7.3,Albumion = 4.7,Globulin =2.6;Bilirubin (Total ) =0.6; Bilirubin (Direct ) = 0.2, Bilirubin (Indirect ) = 0.4.His ultrasound says he has Fatty Liver Grade II. His mother had primary billiary cirosis and has passed away.Stomach x ray shows no abnormality.He has been given Golbi 300 thrice daily, with Zevit one a day and Pantocit twice daily.Please help us know what is his disease and is it very serious . What diet can I give him to be healthy ? Doctor: Hi and welcome to Healthcaremagic. Thank you for your query. I am Dr. Rommstein, I understand your concerns and I will try to help you as much as I can.There are two main types of fatty liver disease: Alcoholic liver disease and Nonalcoholic fatty liver disease. You can get alcoholic liver disease from drinking lots of alcohol. It can even show up after a short period of heavy drinking, other causes are obesity and hepatitis B and C.If you have alcoholic liver disease and you are a heavy drinker, quitting is the most important thing you can do. If you don't stop you could get complications like alcoholic hepatitis or cirrhosis. Even if you have nonalcoholic fatty liver disease, it can help to avoid drinking. If you are overweight or obese, do what you can to gradually lose weight -- no more than 1 or 2 pounds a week.Eat a balanced and healthy diet and get regular exercise. Limit high-carb foods such as bread, grits, rice, potatoes, and corn. And cut down on drinks with lots of sugar like sports drinks and juice.I hope I have answered you query. If you have any further questions you can contact us in every time."
},
{
"id": 200471,
"tgt": "What does reddish color semen indicate?",
"src": "Patient: Hi, I noted today that my semen was mixed with some reddish color. The background with this is that 1.5 days ago I ate a very large salad with a lot of beets. I rarely eat beets and when I do it is only a few small pieces so this was the only time I had so much. I never had this issue and I don\u2019t have any other symptoms. My urine is fine plus I drink a lot of water. Also, I\u2019m 56. Thanks for your time. Doctor: Thanks for asking in healthcaremagic forum May be because of that. But is not normal to have red colour semem. Please visit a doctor for semen analysis and further management. All the best."
},
{
"id": 194288,
"tgt": "What does greenish yellow sperm and soreness in testicle indicate?",
"src": "Patient: My left testicle has been sore for the last 24 hours. Ive also had weird feelings in my lower stomic and back. It doesn't hurt to urinate. I have taken aspirin and have put a cold pack down there to relieve the pain. I do play hockey and I was thinking that maybe I hit it some how and didn't realized it? I don't know. I have also noticed my sperm is greenish yellow. I've had the same sexual partner for the last year and a half. Please help. Doctor: Hi, Considering the color, abdominal pain it can be urinary tract infection. Still, we can't exclude Sexually Transmitted infection (STI). You gave the history of sex. If it is unprotected sex we can think of STI and run a panel of STI investigation. Also, need to do urine culture sensitivity to rule out urinary tract infection. A pelvic ultrasound will help you diagnose the prostate related issue. Hope I have answered your query. Let me know if I can assist you further. Regards, Dr. S.R.Raveendran, Sexologist"
},
{
"id": 113778,
"tgt": "Had back operation. Started pain in back, stiff neck and headache. Not getting relief from medicine. Looking for advice on what to do?",
"src": "Patient: I am a 59 year old women who has had 5 back operations in Pa. I only live in Plant City Florida 6 months out of the year. About a month ago I started with severe pain in my lower baack that goes all the way down my right leg into my foot . And a very stiff neck with awful headaches . Since I don t live here I don t know where to go for treatment . The last time I saw my neurosurgeon was about 4 years ago and he said that I have a failed back. I have bad times and go through alot of pain and usually a couple trips to the chiropacter helps me, this time the pain meds. don t even help. Can you advise me where to go and what to do? Doctor: Ayurvedic panchakarma therapy like greeva vasthy,kadi vashi,pichu and ayurvedic medicine like prasarinyadi kashayam,prasarinyadi tailam,karasasyadi tailam....etc....."
},
{
"id": 187076,
"tgt": "Suggest remedies for white stuff and bleeding from gums below braces",
"src": "Patient: can you answer a question on braces? I have braces. for an year now. Where they placed the band, the gum swelled. Now, I squeezed it and white, yellowish stuff came put. it's was like liquid. Then blood came. What should I do? Should I ask my ortho to adjust my band? Doctor: Hi! Welcome to healthcaremagic.I read your query. As you mentioned you are having orthodontic treatment right now. The gum swelling after band placement is traumatic periodontitis. The band or the braces might be impinging in your gums leading to irritation and abscess. I suggest you visit your dentist and get adjustment of band and braces done in dat area.Also, dont pinch or squeeze the swelled area as it can cause infection further.Maintain good oral hygiene. You can use orthodontic brushes. Use mouthwash or warm saline rinses. Do rinses after eating or drinking.Massage gums with gum paint for increasing blood supply and soothing.Maintain good diet. And get oral prophylaxis done at regular intervals of 3-6 months.Hope the answer helps you. Thank you!"
},
{
"id": 163423,
"tgt": "Is Vomikind the right medicine for vomiting in infants?",
"src": "Patient: Hi dear..... My baby s 11 month old... he spit or u can say vomitting ... take out milk with very bad smell.. same his digestive system not able to digest... it\u2019s a pure cow organic milk... Dr prescribed VOMAKIND.. but it\u2019s only for stop vomitting ... what about if the infection start Waiting thx Doctor: Hello,Yes, Vomikind is right medicine for vomiting in infant. But have to give as per his weight. For infection, you can give syrup Cefixime as per his weight after consultation.Maintain proper hygiene and give him pure water. I will suggest you to follow up with detailed history or consult with pediatrician for detailed examination.Hope I have answered your query. Let me know if I can assist you further.Regards,Dr. Shyam B. Kale"
},
{
"id": 88703,
"tgt": "Suggest remedies for chronic radiating abdominal pain",
"src": "Patient: for last few months i am having severe abdominal pain. sometimes on the right side of abdomen while most of the time on the left hand side of abdomen... but this pain only last for few seconds or a minute.. i am very much afraid of Doctors... please suggest me any herbal medicine by using which my kidneys start to function properly... my skin has become dry and pale.... please reply very very very sooon.. i am so tensed up these days.. Doctor: You can try these solutions:1. Drink green tea regularly twice daily.2. Also you may try Himocid (which is a herbal preparation by Himalaya drug company) 10 ml twice daily.3. For your kidney's function, drink plenty of fluids. Also you may have tablet CYSTONE (which is usually used in prevention of kidney stone, which will improve your kidney's function, and also it is herbal.)Avoid alcohol and smoking if you do, reduce spicy and oily food.Hope this resolves your problemTC"
},
{
"id": 210776,
"tgt": "Will olanzapine sort out the repetition and anxiety?",
"src": "Patient: I have totally dominating repetitive thoughts, which are preventing me from being able to concentrate on anything, I can't focus on anything else for more than a minute or 2. I have been put on olanzapine, will it sort out the repetition and anxiety? Doctor: HiThanks for using healthcare magicOlanzapine is a antipsychotic and it helps to control psychotic symptoms like delusion, hallucination or aggression. In low dose, it could be used for sleeping or anxiety, but it does not control repetitive thoughts. Repetitive thoughts are part of OCD in which treatment of choice is antidepressant. Consult a psychiatrist for proper treatment and diagnosis.Thanks"
},
{
"id": 174510,
"tgt": "Is a surgery needed for the mild caliactesis for a 3 month old?",
"src": "Patient: my childs left kidney has mild caliactesis , before born it was 12 mm . after birth 10mm . doctors suggested to repeat scan after 3 months . if persists go for surgery . Is this surgery risky ? if we ignore surgery any troubles ? baby all activiteis are normal like passign urine and all Doctor: Hi,Thank you for asking question on health care magic.The calculus in the kindney needs observation as it i not causing any symptoms.if the stone comes into the uterus and cause obstruction, it causes pain and back pressure leading to hydronephrosis.At that time some times surgery may be needed.Hope this answer will serve your purposePlease feel free to ask any more queries if requiredTake careDr.M.V.Subrahmanyam MD;DCHAssociate professor of pediatrics"
},
{
"id": 22679,
"tgt": "What causes pain in the middle of my chest?",
"src": "Patient: Hello doctors very recently i have had a pain in the middle of my chest, it feels like a very small lump but the pain occurs when i touch it, stretch and sometimes when i'm breathing, i'm a male 17 year old with no history of asthma etc. i weigh 73.4 kg and have a very active lifestyle but i dont believe this pain was created through me practising my sports as i noticed this when i just woke up one day Doctor: Hello and welcome to \u2018Ask A Doctor\u2019 service. I have reviewed your query and here is my advice. This is likely a musculoskeletal pain, and there is no need to worry much. Use diclofenac gel for local application and tab Diclofenac whenever needed with tab pantoprazole. If it doesn't resolve in a week then visit a local doctor for examination.Hope I have answered your query. Let me know if I can assist you further.Regards,Dr. Sagar Makode"
},
{
"id": 205469,
"tgt": "Suggest remedy for severe depression",
"src": "Patient: I m a girl of 20, my boyfriend broke up with me 2 yrs back.My childhood friend hates me the most. One of my closest friend hurted me. I smoke & often drink. Stay awake whole night & then avoid normal daily routine. I generally avoid going out. Feel very lonely when alone. Sucidal thoughts are very common. Have attempted before. I stay annoyed modt of the times & get angry very easily. Zero concentration in studies. Lost all intrests in my hobbies. All day i just sleep, browse random things on google to spend my day & to avoid friends and people.Planned a number of times to visit a psychiatrist but I do not. Doctor: DearWe understand your concernsI went through your details. From the given symptoms it is almost clear that you have depressive disorder. Lack of interest, low motivation, lonely feeling, suicidal ideation are all clear cut symptoms of depression. You must consult a psychiatrist at the earliest. Early diagnosis and treatment is best for complete cure.If you require more of my help in this aspect, please use this URL. http://goo.gl/aYW2pR. Make sure that you include every minute detail possible. Hope this answers your query. Further clarifications are welcome.Good luck. Take care."
},
{
"id": 190641,
"tgt": "Lump around the broken tooth, persistent ever after extraction. Had an injection on the lump during extraction. What could it be?",
"src": "Patient: My tooth broke off around two years ago It wasn t the front one but three from the front right hand side. I had no money so I left it. The pain went away after a few weeks then out of the blue I got a lump which was around an inch up from my tooth on the cheek near the side of my my nose which was a wee bit sore then the pain went away but the lump stayed. My right hand side did look a bit swollen but no one ever commented on it. On Monday I went to the dentist and had the remainder removed. I never mentioned the lump as I wanted what was left of the tooth out as it was starting to get sore. The tooth is gone plus it was a bit of a battle to get what was left out and very painful, also one of the injections went right into the lump. I did have swelling after it came out and put ice on it, the swelling has gone but this lump is still there. I have the usual pain from an extraction which I can handle but would like some advice on this lump. Thanks. Doctor: hi...the lump arised as a result of fracture of the tooth.the fracture caused periapical infection of the tooth.pus has accumulated in that region.so it appeared as a lump.after removal of tooth,the pus will drain through the socket.thus lump will get cured..you should take the complete course of antibiotics.if the lump persist even after 2-3 weeks,curettage of that region should be done.thank you."
},
{
"id": 66395,
"tgt": "What treatment to be taken for the lump in my hand?",
"src": "Patient: I had a car accident a few years back and I have nerve damage in my arm still, which I guess is normal. the concern I have is a soft mass on the outer part of my forearm. it s squishy but about the size of 1/2 a medium egg sticking out. it doesn t always hurt there to touch but it s hypersensitive today, just brushing my finger on it brings localized pain to the lump. Doctor: Hi! Good evening. I am Dr Shareef answering your query.Although it requires a personal physical examination of the lump to say what this could be, from your history it seems that this could be an organised hematoma (solidified blood clot) caused due to the trauma in case this appeared from the date of the trauma. If the swelling occurred later, it might be an independent entity, and could be benign swelling like a lipoma, or more probably a neurofibroma considering the hypersensitivity of the lump. If I were your doctor, after an initial clinical assessment I would advise you for a FNAC of the swelling to have a cytological diagnosis followed by the definitive treatment depending on its report. Till then, you could go for an anti inflammatory and a proton pump inhibitor drug for a symptomatic relief.I hope this information would help you in discussing with your family physician/treating doctor in further management of your problem. Please do not hesitate to ask in case of any further doubts.Thanks for choosing health care magic to clear doubts on your health problems. I wish you an early recovery. Dr Shareef."
},
{
"id": 139749,
"tgt": "Suggest treatment for spinal cord pain",
"src": "Patient: My mother is suffering with spinal cord pain since last 6 months. If she take rest, she feels better, once she started working in kitchen continuously then again she gets the pain. If she take rest and sleep for 2 hours she feels better. What kind of treatment she need? Doctor: Hello, I will ask you a few questions. What age is your mother? How are her sun exposure and intake of food rich in calcium and vitamin-D, like fish milk eggs? In old age, you must get an X-ray done of exactly the point at which she points for pain. Also, do her calcium, phosphate , alkakine phoshatase and vitamin d levels checked. She should at moment take some light analgesic like paracetamol with orphanedrine (nuberol forte) and very light exercise of only walking almost 10 to 15 minutes daily.Hope I have answered your query. Let me know if I can assist you further. Regards, Dr. Muhammad Faisal Bacha, Internal Medicine Specialist"
},
{
"id": 193347,
"tgt": "What is the treatment for wet dreams?",
"src": "Patient: I have left masturbationg about months and now I am completely fine.I am having wet dreams after a period of 1 month.how to stop thisandI am a regular pushup-er.Is it likely that masturbation cause veins of hands and legs to pop out?and does mstrbtion cause popping out of veins in penis?Is it Bad? Doctor: Hello,The masturbation is safe procedure and full fill sexual desire. It doesn't cause pop out of any vein over penis. However, excess masturbation can cause groin discomfort. So masturbation is not harmful as far as sexual life concenerd Hope I have answered your question Let me know if I can assist you further Regards"
},
{
"id": 186525,
"tgt": "Suggest treatments for shadowing of tooth",
"src": "Patient: My 16 year old son had three teeth knocked out five weeks ago. They were put back into his mouth about 40 minutes after getting knocked out by our friend who is a dentist. A splint was put in place to keep the teeth in place. He has not been able to chew using these teeth (front upper, number 8-10). Today, the number 10 tooth has some shadowing and he has been advised to see an Endodontics. When should we do this and is it ok for him to use his front teeth now? Doctor: Thanks for posting your query to HCM.After going through the history of trauma to your son, his tooth number 10 should be treated by root canal therapy for which you should visit an endodontist to stop the progress of infection underneath it which is seen in cases of trauma commonly.And he should not start using his front teeth as of now to promote faster bone healing.Hope my answer will help you.Thanks"
},
{
"id": 24779,
"tgt": "What causes high blood pressure spikes occasionally?",
"src": "Patient: I have occasional episodes of high blood pressure spikes (160+/100+ with pulse 120+) combined with shivering through my whole body for 15-30 minutes. I am taking 2 meds to control hypertension and 2 for diabetes as well as Pradax blood thinner. I did have stroke 1.5 years ago caused by atrial frib. which I have fully recovered from. One month ago, I had a seizure like episode lasting 3 minutes which my whole face and voice muscles were frozen. These shivering/high bp are increasing in frequency.What could be casuing thise events? Doctor: Thanks for your question on Healthcare Magic. I can understand your concern. Since you had seizures in the past, we should definitely rule out recurrence of pulmonary this seizure episodes in your case. Shivering of whole body can be due to seizure activity. And diabetes and hypertension can precipitate seizure activities. So better to consult neurologist and get done clinical examination, MRI brain and EEG (electro encephalo gram). You may need regular anti epileptic drugs on the basis of above results. If all these reports are normal then get done vitamin B12 level as deficiency can also cause similar symptoms. And if deficiency then you will need supplements too. Don't worry, you will be alright. But first rule out seizures. Hope I have solved your query. I will be happy to help you further. Wish you good health. Thanks."
},
{
"id": 207261,
"tgt": "How can OCD with irrational fears be treated?",
"src": "Patient: Sir I am Gaurav from Lucknow Sir I am sufferring from ocd I have following sysmptoms 1)Used to wash hands excessively when I was youn 10-12 yrs 2)fear of God(In 2003 when we were doin some puja) 3)Fear of exam(in 2008 I was having my final sem MCA exam a strange feeling came to my mind If I throw my roll no then would be debared from exam) 4)What happen If I slap someone in public Sir I dont know why these thoughts came to me I am mentally sound also sir I am very very shy and soft spoken,and also have feeling of fear of getting into new environment..thats why I am unable to get job.pls help me Doctor: In my opinion your problem is easily treatable by drugslije fluoxetine,fluvoxamine, chlomiprine. Visit a lical psychiatrist or kgmc lucknow.So dont worry. Hope my suggestions helpful and thankful to you. Take care. Don't forget to give ratings."
},
{
"id": 4879,
"tgt": "Under fertility treatment, given sifai for egg rupture, egg not ruptured. Symptoms to find egg rupture?",
"src": "Patient: Hi My wife is in her 19th day and under the fertility treatment; she was given Sifai 5000 on 16th Sep 2013 and we were advised that egg will rupture in 36 hours and to have intercourse; egg has reached 22 - 23mm but still the egg has not ruptured till this morning when she was scanned. Please let me know if there are any symtoms can be found when egg ruptures so that we would have intercourse Doctor: Hello,When egg ruptures, slight pelvic pain is observed in some women. If you track her body temperature daily, 0.5 to 1 degree less temperature is observed on the day of egg rupture. Best thing is using ovulation prediction kit or LH kit. It is available in market. It is useful to predict ovulation. Good luck."
},
{
"id": 201265,
"tgt": "What causes bump near penis?",
"src": "Patient: I have had a bump near my penis, not on it, that i thought was just an ingrown hair. However, while i was inspecting it, it burst and mostly blood and a little puss came out. I wasnt worried but earlier tonight it reappaered and burst again with the same results Doctor: Hi,It seems that there might be having ingrown hair infection or some local skin condition like dermatitis or fungal infection.Apply triple action cream.Keep local hygiene well shaved and clean.Ok and take care."
},
{
"id": 181684,
"tgt": "What causes numb sensitive front teeth post injury?",
"src": "Patient: I'm not sure if you can help me, but here goes, a week ago a crock pot lid fell of shelf hitting me on my 2 front teeth, now they are numb and some what sensitive, the gums behind these 2 teeth are irritated.My general dentist took x-rays which were negative for crack/fractures. Doctor: Hi.. Thanks for the query..As per your complain it is common to have numbness of teeth post trauma..Sometimes the numbness gets better with time if it is only due to the soft tissue injury around the tooth and not due to death of the nerves which are present inside the tooth..If you have sensitivity of teeth you can use anti sensitivity tooth paste for a week time..If you do not fine any betterment in the numbness or there is any change in colour of the tooth you should visit your dentist again...All this is a sign of nerve and blood vessel damage.In this condition you will have to get a ROOT CANAL TREATMENT done which will cure your problem..The irritated gums can be treated by using a gum paint.. Slowly the gums will recover..Hope your query is solved..Thanks and regards..Dr.Honey Nandwani Arora.."
},
{
"id": 15234,
"tgt": "Recurring rash. Diagnosed as wring worm, given ointment. Suggestions?",
"src": "Patient: My daughter 34 years old has been dealing with a rash, she saw a doctor who said she has wring worms, and gave her some ointment, but, it will heal some, but new ones seem to surface. I informed her to use some chlorine bleach because I heard it works. If her wring worm continually comes back what can she do to stop it? Doctor: Hi,As your doctor said it is tinea infection, continue anti-fungal cream or lotion locally.Go for one course of oral anti-funngal tablet like Flucanozole 150 mg once in a week for 4 weeks.Ok and take care."
},
{
"id": 181367,
"tgt": "What causes shooting pain at the site of tooth extraction?",
"src": "Patient: Hi i had a root canal/stainless steel crown put on a molar tuesday using cement, i also had an extraction of a top tooth on the same side and 3 fillings....i waited until the numbness stopped after the dental work to start eating and i ate soft foods for 3 days, i have avoided chewing on that side of my mouth, ive been brushing my teeth softly, ive been gargling with warm salt water, i have been putting a icepack on the side of my face and i have been taking the ibuprofen prescribed to me....the swelling on the side of my face and soreness was improving..but just a little bit ago i was gargling like i have been doing and severe shooting pain started up for about a minute and a half then eased up until i gargled some more then it started up again for another minute so i stopped gargling, took ibuprofen and put the icepack on, now the pain has eased up again...so im wondering why the pain all of a sudden returned in the form of shooting pain while gargling...because i was doing better and following the instructions from my dentist, i know its only the beginning of the 4th day after the dental work im just afraid i might be having a set back when all i want to do is get back to eating normally...is the random shooting pain while gargling a good reason to worry that i might be having a set back or is my dental work phobia getting the best of me. Thank you in advance for taking time to read my concern and i very much appreciate any feedback. Doctor: Hi..Welcome to HEALTHCARE MAGIC..I have gone through your query and can understand your concerns..As per your complain sudden shooting pain after dental extraction during gargling that too after 4 days is not normal and can be due to nerve related pain..In case if the nerve is injured during the extraction procedure the a slight stimulation can lead to firing of impulses and pain..But it will gradually subside as the healing will proceed..But in case if there are recurring episodes of same problem consult an Oral Physician and get evaluated and investigations like x ray or MRI scan can help in diagnosis..As of now continue to take painkiller like Ibuprofen and follow up with your dentist when you complete 7 days to check for proper healing is taking place or not..Hope this helps..Regards."
},
{
"id": 161901,
"tgt": "Are worsening handwriting, spastic movements, confusion and lack of logical reasoning after a head injury serious concerns?",
"src": "Patient: My GF s daughter (10 years old) is having all these symptoms for the last 3 weeks and we think it s due to a fall. She hit her head at school and still has some tenderness (3 weeks later). Her handwriting is getting worse, she has been making these sort of spastic movements (similar to someone on drugs), she is not thinking clearly (apparently couldn t get her pants on this morning because she was only using one of her hands while the other fist was closed. She didn t grasp the concept of using both hands and gave up. Her school work has dropped off drastically and she was previously a good student. Simple projects that would take her 10 mins previously now take 2 hours. We re afraid she might have some sort of swelling on the brain or something? Do these symptoms go along with some head injury? For the record - her mother is taking her to Urgent care right now. Doctor: all of the symptoms you described are not definitely determine head injury, on my opinion she must ambulatory admitted by a psychiatrist."
},
{
"id": 98430,
"tgt": "How can allergic rhinitis be treated?",
"src": "Patient: Hey my son is 17 years old mainly early morning or he wakes up late night constantly sneezing, blowing his nose itchy eyes and sinus -pressure/ mucus. I don't know what kind of over the counter medicine to buy. He's been taking cold medicine for the last two days it hasn't worked. Doctor: Hello,Kindly, investigate with CBC and AEC count.If AEC count more than 450, then fexofenadine plus montelukast combination drug can be prescribed. Otherwise decongestant like Levocetirizine, OD or phenylephrine for five days can be prescribed.Steam inhalation can be done three times a day. Salt water gargle can also be done with warm water thrice a day.Hope I have answered your query. Let me know if I can assist you further.Regards,Dr. Parth Goswami"
},
{
"id": 7211,
"tgt": "PCOS and completed medication. When should I have intercourse?",
"src": "Patient: hi im shilpa 19 and weigh 90 height 5.6 daignosed and confirmed pcos and after taking arbonnid capsules i got periods on 10th july and doctor is told to take siphene 50 mg 5 tablets yesterday i finishd all 5 now when shuld i have sex and when wil i get pregnant i want to get pregnant fst plz help me im confused Doctor: You need to have sex during your fertile days which is usually between day 12-17 of the cycle.Along with that you should also undergo ovulation study to monitor the growth of the follicle."
},
{
"id": 55354,
"tgt": "Does an increase in the liver enzymes anyway linked to tumor?",
"src": "Patient: I am 19 years old and just had a 4 cm tumor including a lymph node removed from my neck near the jaw. there is another tumor higher up and more difficult to remove as it will involve muscle. The tumor was sent for biopsy and await results. In the meantime thru blood work they found my liver enzymes to be over 800 and a fatty liver. Can these 2 things be connected and if so, what might the diagnosis be? Doctor: hi.it is best if you consult with a doctor, preferably a general surgeon, for physical examination and clinical evaluation. it may or may not be related (as such in multiple endocrine syndromes). await your neck biopsy results. as for your liver enzymes, further evaluation of your abdomen must be done. a consult with your doctor is best. diagnostics (such as ultrasound & other blood exams) and management (medical and/or surgical) will be directed accordingly.hope this helps.good day!!~dr.kaye"
},
{
"id": 153528,
"tgt": "Suggest treatment for colon cancer spreading to liver and lungs",
"src": "Patient: My son (47) just recently diagnosed with terminal cancer (tumor in colon spread to liver and lungs). Hospice stepping in tomorrow but in the meantime he has no physician due to no insurance. At the moment he is expericing the hiccups and sore throat. Hiccups is hurting him. He is on oxycodone as he is in much pain when not. Is there an over the counter rememdy I can get him to relieve the hiccups? Doctor: Hi,Thanks for writing in.Sorry to know that your son is having an advanced stage of colon cancer. At this time, he should be given complete pain relief and emotional support. Arrangements should be made to take care of him all the time and hospice will do that I hope.Hiccups in cancer might be due to acid reflux or as a side effect to chemotherapeutic agents. If his hiccup is from acid reflux then please give him over the counter remedies to prevent acid reflux. Also keep him in seated position for 30 minutes after meals to prevent the food in stomach from returning back in to the esophagus. Many chemotherapy hiccups might also respond to steroid treatment and if he is being given steroids for his condition then that will also treat the hiccups.FDA has approved a medicine called chlorpromazine to treat hiccups in cancer patients but you require a prescription to get it."
},
{
"id": 111656,
"tgt": "What is the name of the alternative back surgery where they inject fluid between disks ?",
"src": "Patient: I have been told that there is a alternative back surgery method a doctor in Texas is trying which consists of injecting fluid between the disks. Do you know the name of this procedure? I am facing back surgery and would like to explore other options. Thank You. Glenn Doctor: Hello, I have studied your case. There are various procedure which include chemonucleolysis ,ozone therapy and injection hyglan.MRI spine will show any disc bulge is there or not.And when muscle sprain is confirmed muscle relaxant, analgesic and neurotropic medication can be started.Till time, avoid lifting weights, Sit with support to back. You can consult physiotherapist for help.Physiotherapy like ultrasound and interferential therapy will give quick relief.Hope this answers your query. If you have additional questions or follow up queries then please do not hesitate in writing to us. I will be happy to answer your queries. Wishing you good health.Take care"
},
{
"id": 109093,
"tgt": "Suggest treatment for back pain",
"src": "Patient: I am a 21 year old female and i have to lift on my grandmother because she cant walk and my back has been hurting bad for 1 week now and it mostly starts when i bend down i have tried a heating pad tylenol for pain bengay and resting it on pillows but so far nothings easing the pain out of it what could i have done to it and what kind of medicine is good for so i wont have to go to my doctor Doctor: Dear patient Weight lifting puts strain on your back. Your pain increases on bending that also indicates back strain. So you have to give rest to your back for few days and avoid weight lifting and bending. Hire on person to attend your grandma start taking diclofenac plus thiocolchicoside combination twice a day for 5 day's. Apply volini gel to your back. All the best.and after doing this if you don't get relief do not forget to visit your doctor."
},
{
"id": 29997,
"tgt": "Are bumps on urethra, related to fungal infections?",
"src": "Patient: Hey,I am 24 years (virgin) old and have had these white bumps extern near my urethra for as long as I remember (2 left and 7 right). They don t itch, irritate...Could they be penile bumps? I have had myself tested for std s nothing came out...I do have these bumps on my penis itself, foreskin and under my glans which is normal.Could this be some kind of rash, fungal infection, Trichomonias? Next month I m seeing my dermatologist as I also want to remove my fordyce spots.In attachment you find 2 pictures... as this is what they look like.WWW.WWWW.WW WWW.WWWW.WW Doctor: Thanks for your query .As I understand it could be sebaceous cyst .because such cyst do not produce any symptom ,these are multiple under the skin layer. I suggest you to consult a general surgeon for examination and correct management ."
},
{
"id": 52045,
"tgt": "It hurts when i pee and fell to pee again, what could be the problem ?",
"src": "Patient: It hurts when i pee & i feel like i have to again & lil tiny chunks of something comes out , what is wrong with me ? Doctor: hi; from what you describe it is a urinary track infection ;also if chunks come out please consult a doctor who will advise investigation & treatment. thanks"
},
{
"id": 211223,
"tgt": "What medicines should be taken to come out of depression?",
"src": "Patient: Meri relative kalpana apni pdhai ko le ke depressed ho chuki h. She always tell that mai kuch nhi kar paungi , mai barbad ho gyi, maine bahut ghamand kiya isliye bhagwan ne mujhe saja di. Aur fir rone lagti h. Kya karna chahiye use normal krne k liye Doctor: DearWe understand your concernsI went through your details. Please ask your friend not to worry. She is not depressed. She is confused. She is distressed. She tries but unable to reap the benefits, or that is what she understands about herself. To get correct understanding of her problem, she needs to talk to a psychologist. Psychologist will talk to her and will diagnose the correct problem. He may give initial counseling to her. But nothing to worry here. Studies are problem to every teen and they overcome the problem when they mature.Your friend might need psychological counseling and or psychotherapy. Please visit http://psychocure.webs.com for behavior modification, mental health assistance, Cognitive Behavior Therapy and other psychotherapies.Hope this answers your query. Available for further clarifications.Good luck."
},
{
"id": 25072,
"tgt": "Can I have mirena iud insertion when having blood pressure?",
"src": "Patient: Hi, i am 43 yrs old and considering getting a IUD to control my hormones and anxiety. However, my blood pressure goes up and down. My doctor put me on a low dosage birth control pill estrogen only. Do you think that I could be a good candidate for Mirana IUD? Doctor: Thanks for your question on Healthcare Magic. I can understand your concern. Hypertension is not a contraindication for Intra Uterine Device (IUD). So you can definitely go for Mirena IUD placement even if you are hypertensive. This will relieve your anxiety and hormonal imbalance. This inturn help in control of blood pressure. So no need to worry. You can definitely put Mirena IUD without any fear and worry. Hope I have solved your query. I will be happy to help you further. Wish you good health. Thanks."
},
{
"id": 30820,
"tgt": "How to get rid of cold, ear infection and weakness?",
"src": "Patient: ongoing undiagnosable severe suffering , despretely need helphey i am 20 years old, always been abit ill wit longer lasting general colds, throat & ear infections , general weakness or alot more tireder then others, tonsilitious, but iv had ears operated on when i was younger for gromets which one ear took the other ear drum had deflated 3 bones hilden ear drum had dispeared, at same time had adnoids removed, 14 fracted elbow, 15 broken shattered dislocated finger along wit 5 bavy teeth removed for braces put on as ugly teeth for mouth to smsll crowded and higher then normal palit, 17 tonsils removed, 18 rhinoplasty as few bumps to nose n previous problems causing bteathing blockage , telling u all that could b pointless but since 18 after all those minor problems i had bern fine, now nearly 21 i have mever been so confused and let down by drs, i fell ill 5 months ago started wit sevete eye irratation discharge of clearish thickish jelly like mucous, swelling and lashes fallen off, vomiting witvor without eating rare bowel habit, loss of appetite, coma like sleep wit eatimg constant all over weakness tenderness, eye and hradache leading to sever migrains, lack of body blood circulation, paleness, abdomanel pain moving up right to side then behind back up middle to right should, throbed aches cramping, allergies constsnt sniffling or snrezing fits, point of unable to move by random back belly uterus pain, late period, unbearable migrains, skin spots like blidters rashes red roar hot rashes coming n going black wholes in hands, dry skin in early stages now constant bodily thick staining clothes oil like suvstsnce, lost apoetite but still eat a good diet cos in.2 months i lost 25 kgs told i had sever sinus infection, then i was told depressed then giardia then blastocystis hominos now nearly bald on my head unheslthy weak ugly 29 kggs lighter unknown body bruising cuts welts like or wholes stiff painful hands in morning ganglene in wrist skin irartation eye irratation chest heart pain cracking. hurtung hating lifd, i was a girgeous healthy enjoyed life type girl i need answer not re directed to diffrrent drs Doctor: hello there..completely miserable as i can see..would like to know your blood reports ..especially thyroid functions.."
},
{
"id": 37990,
"tgt": "What causes bleeding in the stools?",
"src": "Patient: Hi. I have had bright red blood in my stool for many years now. Sometimes there is a great deal of pain, while other times I don t feel any pain. Over the past year or so, the amount of blood has increase a lot. The toilet water will be completely red by the time I am finished going to the bathroom. Also, I had a blood clot (about the size of a grape) on the toilet paper recently. I am a 29 year old female. What could these symptoms indicate? Doctor: hello madam,thanx for your query on HCM,dont be panicked.looks like there is some bleeding point in your gastrointestinal tract.its very hard to correctly diagnose your case without examining you.better do visit a surgeon.get examined by him.take proper treatment recommended by him..things will improve."
},
{
"id": 183940,
"tgt": "What causes small flap of gum between teeth?",
"src": "Patient: Hi there i have recently developed some kind of small flap of gum inbetween my two front top teeth which at first felt a bit puffy and slightly painful so i gently brushed my teeth and a little blood came out from behind this small flap of gum, now the pain is mostly gone but this strange little piece of loose gum is still here. Doctor: Helloo..regarding your query i can say that there is nothing to worry its just a gum inflammation.it can be either due to some food debris like small bit of shellfish or some small bit of some debris..it will heal by its own naturally.you can do nice salt water gargle.and can do listerine mouth wash twice daily for 1-2 wks..absolutely nothing to worry..be happy and smile.have a nice day.."
},
{
"id": 177124,
"tgt": "What are the side effects of Motrin?",
"src": "Patient: my grandson has been running a temperature in the range of 102 to 103.7 since Tuesday. He s seen the doctor 3 times who believes the reason for his elevated temperature is viral. I am alternating between tylenol and motrin as per doctor s advice. I am concerned about what the tylenol may do to his liver and the motrin to his digestive system....your comments please Doctor: tyelenol(PCM) and ibuprofen causes diarrhoea, gastric upset and even nausea , vomiting. higher doses given for longer duration can cause hepatic dysfunction. tyelenol can be given at a dose of 15 mg/kg/dose 4-6 hourly. along with giving medications, try doing tepid sponging of the child's body with cold water to reduce the fever. gastric symptoms are transient and will resolve spontaneously. worry more about his high grade fever rather than the side effects of these drugs.thanks"
},
{
"id": 198603,
"tgt": "Could tingling in right leg, heavy testicle, pain behind knee after taking amoxicillin course suggest varicocele?",
"src": "Patient: 26 year old male. Evaluated by Urologist to have varicocele. Symptoms started to appear after finishing Amoxicillin course (5 days 3x a day). Now I'm experiencing tingling sensation on right leg accompanied by heaviness on right testicle, with episodes of pain behind the knee going all the way down foot. Sometimes present on both legs. Doctor: HelloThanks for query .You have been diagnosed to have Varicocele and your Dr prescribed Amoxicillinn for five days .You have noticed pain in in Rt leg and heaviness in testicle and pain behind knee joint .The pain in leg and behind knee joint is a referred pain along the innervation of Genito Femoral nerve and could be mostly due to heaviness of the testicle or Varicocele .You need to take anti inflammatory medicine like Diclofenac twice daily ..Using scrotal support to prevent sagging of scrotum and testicle will reduce hanging of testicle there by help to reduce pain .Dr.Patil."
},
{
"id": 177958,
"tgt": "Possibility of suffering from ADHD",
"src": "Patient: I think I have ADHD I am eleven, but my parents won t listen because I have high grades. Everyone thinks I have it. My dad has it and he almost got 100% on his state tests when he was a kid, so what does that have to do with anything? I guess I have just used what I know because I get numbers. I fidget almost all the time to help me focus, I am SUPER disorganized, I forget things constantly, and my grades are slipping. Believe me, there is much more. I also have synesthesia and most websites say that increases the chance of ADHD. I just moved, so I don t have a doctor to ask and I REALLY want to get into the Gifted and Talented program in my school. WHAT DO I DO?!?!?!?!?! Doctor: Thanks for the query dear.his level of awareness at your age is really commendable.As per your description it seems you have almost decided that you are suffering from ADHD. You have not mentioned what your doctors said regarding your condition, that is before you shifted to a new place.You may be quite simply a disorganized adolescent so to speak, then you will need support of a psychologist.But again it is best to find a good doctor in your new place, take your parents in confidence and take help from the doctor. He shall be able to guide you or prescribe medication if needed, in better way.Don't suffer unnecessarily by obsessing over something that has not been diagnosed as yet (you have not mentioned that its a confirmed diagnosis).Take professional help and you shall be better and improve on your grades."
},
{
"id": 43288,
"tgt": "Delayed periods. History of hysterectomy due to infertility, endometrium lining being stretched and giving progynova and modus. Possible pregnancy?",
"src": "Patient: Hi , i had hysteroscopy last month for infertility , said my endometrium lining is only 5 mm. They stretched the lining and gave progynova and modus . my periods wil be around 28 days. But now stil i didnt get my periods, i m worried of whether my periods wil get irregular.. or am i concieved,, it was extended for abt 6 days.. pls reply .... Doctor: Hi Dear,Endometrium of 5mm is very thin and progynova is an estrogen preparation given to increase endometrial thicknesss and progynova and modus a progesteron preparation given for the support in later half.If you have stopped progesterons for over a week and still not getting your periods, than kindly do a pregnancy test. There are chances that you might have conceived"
},
{
"id": 132087,
"tgt": "What to do for pain and swelling i the ankle?",
"src": "Patient: I have had a job as a hostess for the last year on the week ends. After a week end it takes me a couple of days to feel like I can walk pain free again. My ankles swell up considerably and it feels as if a bone is coming through on my left upper foot in front of my ankle. My mother has neuropathy as well as her mother (my grandmother). Will this eventually take over my ability to walk? Also how should I be diagnosed and what can be done about the pain. I feel like someone has a vice grip around of my legs at the bottom by the end of each working day. Doctor: Hello, Thanks for writing to us, I have studied your case with diligence.There is possibility of ankle arthritis.You may need to do X ray and blood investigation.If required you may need MRI ankle for soft tissue involvement.Till time you can use supportive splint or brace and take analgesic.Hope this answers your query. If you have additional questions or follow up queries then please do not hesitate in writing to us. I will be happy to answer your queries. Wishing you good health.Take care"
},
{
"id": 26545,
"tgt": "Suggest treatment for rapid heart rate",
"src": "Patient: Hello . I am 6 5 slender build . I have been noticing that when I lie down at night my heart will miss beats . Then when the next beat comes its much harder almost like I m getting punched . It happens every 10-50 beats and is starting to scare me . It almost feels like my heart is stopping then starting very violently. Doctor: Hello!Welcome and thank you for asking on HCM!Your symptoms seem to be related to ectopic heart beats or possible cardiac arrhythmia. Do these symptoms occur only at night? Have you noticed any similar troubles during the day? Are your symptoms associated with difficulty breathing, changes in blood pressure or chest pain?Anxiety could also mimic this clinical situation and would explain the fact that you notice this only at night. I recommend consulting with your GP for a careful physical examination, a resting ECG, a chest x ray and some lab blood tests (complete blood count, fasting glucose, kidney and liver function tests, blood electrolytes, thyroid hormone levels, etc.). An ambulatory 24-48 hours ECG monitoring and a cardiac ultrasound would be necessary to rule in/out possible cardiac arrhythmia if suspicions are raised. But, if all the above mentioned tests result normal, the main cause of this clinical symptomatology would be just pure anxiety. In such a case, you should discuss with a specialist of this field to help you manage your anxiety. I would also advise to reduce caffeine intake and perform a lot of physical activity. Hope to have been helpful!Feel free to ask any other questions, whenever you need. Greetings!Dr. Iliri"
},
{
"id": 6800,
"tgt": "What causes dizziness and sweaty palms and how can these symptoms be treated ?",
"src": "Patient: hi, i m 19 year old female i been feeling dizzy, alot of shaking, weak, and sweaty hands at times. what can be the cause of it please help! i dont have insurance right now do you think i should have to go see a doctor? thanks Doctor: Hi,nancy thanks for posting query.Do not worry,drink plenty of fluids,take light diet.Get your blood pressure checked.If it is normal there is not need for concern.This could be due to some ear problem.Avoid drinking very cold liquids,take steam inhalation,.You should be fine,if not the please see your doctor.bye."
},
{
"id": 138988,
"tgt": "What causes tingling in right hand and arm?",
"src": "Patient: Often when I am sleeping or first wake up, I get a tingling in my right arm and hand. This has only started occurring less than 4 weeks ago. What might be causing it, and how can it be stopped? I am sorry to have wasted your time. I thought it was free to ask the question or that I would be directed to a webpage where I might read about things like this. Doctor: hiYou are welcome to seek help.The symptoms are probably due to neuro-vascular impingement i neck or axilla or elbow.frquently faulty postures like lying on shoulder with arm under the body may give pressure to nerves or blood supply temporarily cutting off - or there may be cervical spondylosis, cervical rib,shoulder outlet syndrome which can be ruled out by x ray and consultation/examination by an orthopedic surgeonIn medical conditions, Vitamin B6,B12 deficiency,diabetic neuropathy are common causes which can be ruled out by blood levels testsRegards"
},
{
"id": 163412,
"tgt": "Why is my 22 week old refusing to take more than 5 ounces?",
"src": "Patient: I have a 22 week old boy who is refusing to take any more than 5 ounces at each feed he had previously been taking 7 or 8 but now point blank refuses anymore and cry\u2019s if i try to feed him more than that. this has been going on for a week and I\u2019m wondering if it is something to worry about. he is in good form wetting and soiling nappies. he has recently starting teething i thought that might be a cause or perhaps he wants solid\u2019s ?? Doctor: Hello,After 4 month of age, any baby on top feeding can be started with supplementary diet.If baby is active, passing urine regularly, having normal stools, having adequate weight gain, then it is normal.Hope I have answered your query. Let me know if I can assist you further.Regards,Dr. Sachin Kumar Agarwal"
},
{
"id": 112700,
"tgt": "Central spinal stenosis, burning sensation in hip, painful ankles, ruptured discs. Is this an emergency?",
"src": "Patient: i am a 48 year old female with central spinal stenosis, i have one surgery already to free up the sciatic nerve on my left side,that was in 2002 and now if i am on my feet for any length of time (like to do the dinner dishes) i have a burning sensation in my right hip area that skips to my outside ankle and becomes VERY painful. is it the stenosis? ive been told my lowest two discs are ruptured also. should i go to the ER because i dont have insurance at this time and cant afford a doctor Doctor: Welcome to HCM.It wold be prudent to see a doctor/ER as your pain is probably related to compression of lumbosacral spinal nerve roots.Drugs for neurogenic pain like gabapentin,steroids to reduce the nerve inflammation and analgesics will probably help.This may be secondary to herniated discs or spinal stenosis causing nerve compression."
},
{
"id": 99782,
"tgt": "What could a lump on leg with bruising after sardine intake indicate?",
"src": "Patient: Hi I first went to hospital after suffering an allergic reaction what I thought was from sardines, turned bright red and couldn't stop itching, eyes swollen etc but the following day I noticed I had a lump on my leg which has a include marked red and swollen surrounded by a bruise? Culd this be related? Doctor: Hi,any medicine might cause allergic reactions by any type.What u had is lump with bruise on leg with itching, redness etc.See this is a skin or surgical problem,doctor has to examine this,only by descriptions one cannot diagnose. So if its lump,then u should consult general surgeon for that rather than seeking medical help."
},
{
"id": 81075,
"tgt": "Is it safe to have shoulder surgery while lung infection?",
"src": "Patient: I have test result of iga 72. I am being treated for a cough and wheezing, with symbicort inhaler, for seven months now. I am preparing for a shoulder replacement, and wonder if, because of my lung infection, if it is safe to have surgery? Thank you, Barbara Doctor: Thanks for your question on HCM.I can understand your problem and situation.I will not advice you to go for surgery, until your lung Infection is completely cured.Because surgery is stressful condition. Body is in tremendous stress during and after surgery due to anaesthesia and procedure itself.As a rule, infections are flare up in stress.So possibility of flaring up of lung infection is high in your case.So it is not advisable to go for surgery with lung Infection.Better to consult pulmonologist and first treat your lung infection and wheezing properly, once all are stabilized then plan for shoulder surgery."
},
{
"id": 44924,
"tgt": "Is it safe to insert Guaifenesin Tablets into vagina ?",
"src": "Patient: hello doctor . The doctor has prescribed my wife to insert guaifenesin tablet into vagina from day12 to day 18 for infertility problem. is this safe to insert this tablet into vagina. Doctor: Welcome to Healthcare Magic This tablet needs to be taken by mouth which will loosen the secretions of the thick cervical mucous. It makes the mucous less thick and allows sperm to penetrate this mucous better. You need to confirm as I am not aware of a vaginal tablet."
},
{
"id": 178650,
"tgt": "Will taking benadryl help treat cough?",
"src": "Patient: our 9 month old daughter has been coughing for a few days now. we called her dr. today and they told us to give her Benadryl so we did. well tonight before bed she was coughing very hard which she has don t a couple times lately. she then coughed up some clear plem. this seemed to make her fell better. she only coughted a few more times and went to bed. she woke up 3 hours later and started to cry ( which she does bout every night now mostly wanting to get into bed with us not so much as wanting a bottle) we were getting her bottle and I guess the crying triggered the coughing and she started back more than before. you can her the stuff in her throat and chest. we started to give her more Benadryl but she was coughing so much. we were sucking stuff out her nose too. we started gived her the bottle and she was coughing so much we stopped. she has went back to sleep now after only one ounce maybe of milk. what does this sound like to you? Doctor: Dear Sir/ Madam,Thank you for posting your query at healthcaremagic.comIt appears that the nine month old child is suffering from bronchitis. Child requires mucolytic agents and bronchodialators, which will help in loosing of sputum and easing the cough. Along with this, if the child is febril, I would advice you a course of antibiotics treatment, to control the infection. You can also administer the paracetamol dose (SOS, if there is fever) depending age and weight.Saline nasal drop is highly recommended for nasal block, to help in to breath better. Apart from this, warm fluids will give soothing effect to child. Percurssion on the back: when the child stomach is empty, make it lie on your shoulder and give gentle tap.Hope with this advice your child will feel better, still if the condition worsen let pediatrician see him.Your thank you note will be very much appreciated!With best wishesDr. Vishwanath Patil"
},
{
"id": 88268,
"tgt": "What causes pain on right side under ribs radiating to kidney area?",
"src": "Patient: after going out to eat chinese food . i stared having alot of pain on my right side under my rib and radiating into my kidney...it started about 2hours ago and hasnt let up yet. it hurts to push on the side...i have a head ache and just hurt? I took my blood pressure and its 150 over 100 Doctor: HelloThank You for contacting HCM.Welcome to Health Care Magic.My name is Dr Muhammad Ahmad & i will look into your problem.I have gone through your query and would try to help you in the best possible way.it seems to be an intestinsal spasm Kindly take some anti spasmodic like Drotaverin 40mg or mebeverine it will hopefully be relievedI don/t wana jump onto bigger issues which can be gall bladder issue which is always occuring after meal ...but if things don't settle down kindly get a detailed ultrasound of yourself KUB with full bladder though hopefully it will come ot clear and you will endup with slight infection and intestinal spasm ...some raw chinee veges have amoebae in them which pain so try two tablets of metronedazole if you canHope this answers your question. If you have additional questions or follow up questions then please do not hesitate in writing to us. Wishing you good health"
},
{
"id": 15020,
"tgt": "Can vaseline be sufficient to help with dark rashes and soreness on elbows and knees?",
"src": "Patient: Just want Diagnosis Of Skin problem it's around My Elbow Joints and Above My Knee Joints at Present Time . On My Elbows It Feels Like a Carpet Abrasion where It's Sore and Tender When I rest my Elbows on mY lap or on Arm of Sofa Chair. Real Slight Soreness on top Of Knee Caps , just Noticed very small dark rash. I put Petroleum Jely Vaseline on it now , seems to help , but it comes Back after a Day or two . Can You Diagnois What it Could Be ? Thanks Doctor: HiYou seem to be developing a frictional dermatitis over the elbows and knees.It can lead to pigmentation as is happening in your case.You need to avoid friction to the above areas as far as possible.Keep the elbows and knees moisturized.Using Vaseline is a good idea.But make sure you prevent any further friction.For pigmentation, using a kojic acid based cream would be helpful.Dr Geetika Paul"
},
{
"id": 18192,
"tgt": "What is the prognosis while suffering from leaky valves?",
"src": "Patient: My mother is 80 and this past sept had 3 vessel disease heart surgery. She didn t do well and didn t come home until dec. Now they say she has 2 leaky valves (one is the main valve) and they don t think its wise to do surgery. They can hear the leaky valves with their stephoscope. How long can she live with leaky valves? Months, years? Doctor: Hello and Welcome to \u2018Ask A Doctor\u2019 service. I have reviewed your query and here is my advice. It is all depends upon the lesion and which valve is affected and how severe is the condition, severe condition prognosis is very poor but it mild condition of valvular heart disease patient can complete the normal life span, clinical examination is the only way that could tell exactly about the prognosis. Hope I have answered your query, Let me know for further assistance."
},
{
"id": 221067,
"tgt": "What causes spotting during 6th week of pregnancy?",
"src": "Patient: I am 6 weeks pregnant. Started spotting dark a week ago after an exam. Sat. morning started spotting heavier and brighter red blood, have done it off and on each day since then. Some clotting. Cramping off and on also, headaches, still positive pregnancy test. My ob office aware but tell me to do nothing til appt on Fri and it isn t anything to worry about. I miscarried in Nov 09 and also in May 09. What should i do? Waiting on HCG levels to come back. HELP! Doctor: Hello,I have gone through your query and understood your concern. If I were to treat you, I would ask for a sonogram of the pelvis to look for any hemorrhages and fetal viability, get serial serum beta-hCG titres, put you to absolute bed rest and manage based on the results. In view of your previous history of abortions, it becomes all the more important to suspect another missed pregnancy with your persistent spotting now. Doubling serum beta-hCG titres are a positive sign of a healthy pregnancy. If there is no fresh bleeding now, it is likely that the bleeder may have arrested and hopefully, the pregnancy will continue. Hope you find this information useful. Wish you good health."
},
{
"id": 57988,
"tgt": "Is it serious having SGPT and SGOT raised and having fatty liver?",
"src": "Patient: Hi, I got my Test Done for pre employement helath check up... I got my SGPT RAISED (72), SGOT RASISED (189)... The test is done after I resgined... Now they are saying I am not fit so I can not continue... Which is unfair... Please let me know how serious this is... As I don't even feel any symptoms of this like acidity, gas etc. After consulting I was aked to do the sonography of the upper abdomen wherein they told me everything is fine but I have got a fatty lever... Please help. Doctor: HIThank for asking to HCMI really understand your problem, the only option remained before you and that is go for a cross checking with some other lab and simultaneously go to see the physician but as long as the lab test are concern this can not taken in to account alone the other supportive test has to be there and the the clinical finding is the prime and crucial one have nice day."
},
{
"id": 59550,
"tgt": "Liver function tests done. Am I safe?",
"src": "Patient: Hello I am 23 years old Pakistani male the other day i went in for a Liver function test on 19/01/2013 on 2pm (because i want to use Whey protein isolate for amateur use not professional ) I have an athletic body but due to loads of work I can t find reasonable time to hit the gym but recently I have started jogging running and mild gym routine so I wanted to see if my Liver is functioning well to use the supplement without any complications LFT (LIVER FUNCTION TEST) Result(s) NORMAL RANGE Total Bilirubin 0.6 mg/dl 0-10 Direct Bilirubin 0.11 mg/dl 0-0.25 Alkaline Phosphatase 217 u/l 80-270 SGPT 93 u/l 0-43 Gamma GT 41 u/l 10-50 Now I just want know if this is alarming or not ? because I have heard that one should not perform hard physical routine like weightlifting,body building ....Mostly bodybuilders have high SGPT because of muscle fiber breakage and recovery ...... Doctor: Hi, thanks for writing in. Increased intake of protein can cause an increase in BUN and creatinine which are functions of the kidney. A marginal increase in SGPT may not be worrisome at all . However ensure that you gradually increase the duration of work out and not suddenly. With regards"
},
{
"id": 153160,
"tgt": "What is the treatment for CML in a 58 year old?",
"src": "Patient: My father is a C.M.L. patient in T.M.H. Mumbai. he is 58 years old. he is continue having medicine \"Veenat 600mg\" Since 2005. Right now he has side effect (asicitic fluid in abdomen) and for this he is having \"Aldactone 25mg\" and regular tapping.But he doesn't have any relax with this medicine. Kindly suggest me some thing for him..... Doctor: Good morning. Your father is getting right treatment for CML but now since he has got ascitis, we need to check whether there is accelerated phase disease or blast crisis. he need to undergo bone marrow examination and BCR ABL transcript evaluation and ascitic fluid examination for that and accordingly a change in therapy may be required. Aldactone will not help much in that case. You need to discuss all these with your treating doctor.regards"
},
{
"id": 8552,
"tgt": "How to restore damaged facial skin due to prolonged tenovate usage?",
"src": "Patient: Hello I am a 17 year old African American girl with dry, sensitive skin. I have been using African black soap for my face and it works wonders to help keep my skin nice. I was also using the cream tenovate plus to treat occurrences of acne on my face. I thought the cream worked so well that I began to mix it with my daily moisturizer and apply it to my face every morning. I have been doing this for at least 1-2 years now. Well, the store that I bought tenovate from stopped selling it so I no longer use tenovate. Not too long after I stopped using the cream my face broke out severely and my skin darkened and it is really itchy. It has been 2 weeks and my skin has not improved. I just recently read online that tenovate is bad for your skin and now I am scared I have damaged my face. What should I use to help restore my beautiful face? Doctor: Hello and welcome to healthcaremagicYes, indeed tenovate should not be used on face, infact using it on face and that too for acne is an absolute NO for me! It contains a potent steroid, clobetasol propionate which can cause steroid related side effects like Acne, photosensitivity, hirsutism, skin thinning, telangiectasis etc.I would suggest that you use a topical broad spectrum sunscreen throughout the day, that which offers both UV B as well as UV A protection, till your skin returns back to normal, which might take a few days.An OTC antihistamine e.g cetrizine 10mg once daily would help you symptomatically with the itch.If I was the treating doctor I would have started you on oral antibiotics for acne e.g either doxycycline or azithromycin, would be good for acne flare.Topically you could use clindamycin phosphate 1% gel twice daily over active acne.Use a gentle cleanser for face wash e.g cetaphil cleansing lotion, twice daily.regards"
},
{
"id": 1759,
"tgt": "How to detect embryo implantation?",
"src": "Patient: Hi! :) I am in my 1st IVF cycle. I have had 5 failed IUIs and a negative laparoscopy. Today is my day 13th after embryo transfer. Three embryos were transferred. I feel no symptoms in my body, no new changes or implantation bleeding or cramping. My LMP is 5/6/14 and I am on Utrogestan 200mg (thrice a day), My Estra (thrice) and Inj. Lonopin (once). My Beta HCG is due on 11/7/14. Does the absence of symptoms indicate that the embryos have not implanted? Kindly guide, Thanks! Doctor: Hi, sometimes you may have no symptoms of pregnancy. So you have to get it confirmed on blood test. Only that will confirm that you are pregnant or not. Hope I have answered your question. If you have any other query I will be happy to help. Regards Dr khushboo"
},
{
"id": 198460,
"tgt": "How to treat lump under penile skin?",
"src": "Patient: hi i lookd undernearth the head of my penis n notice like a lump formin under my skin see no small pumps around it or discolor ration and doesnt hurt when i touch it but when i look in side penis it kind of grayish n notice a small lil bump that u can barely see Doctor: HelloI had gone through your question and I really appreciate your concern Here I would like to know about your age and relevant medical history like diabetes or other metabolic disease, any recent exposure to unprotected sex?If possible upload a photographIn my opinion you should wash it properly with antiseptic soap or solution and keep it clean and moist.You should apply anti bacterial ointment like mupirocin or soframycin to prevent superadded infectionPlease write back to me with these details so that I will be able to tell you more accurately.Best wishes"
},
{
"id": 225584,
"tgt": "Feeling sick, stomach pain, tasteless mouth after taking unwanted 72 pill. What should be done?",
"src": "Patient: I had an unprotective sex with my bf on 18sept than I took unwanted 72 on the very next day (19sept). Late Night we had sex again & I took one another Unwanted 72 next day (20sept). After consuming it I felt short period of sickness, but from yesterday I'm having stomach aches, tasteless mouth & mouth watering. Suggest me what to do & what not? Doctor: Hi, unwanted 72 is a harmone preparation, so there will be gastric irritation, and more over you have used two packs in 72 hours. So you had gastric irritation, and cramps. I to my patient with such symptoms prescribe omeprazole, domperidone, and antacid gel for relief. Thank you."
},
{
"id": 25703,
"tgt": "What does this ECG report indicate?",
"src": "Patient: ecg report vent rate63 bpmPR interval 152 msQRS duration 148 msQT/QTC454/464 msP-R-T axes 26/26/244RR/PP956/PP interval 956/950 msSystem Evaluationnormal sinus rhythm left bundle blockHDL Colestral 35swear pain during fast walking & Eye sight intruption/looses for fifteen mts once Doctor: Thanks for your question on Health Care Magic. I can understand your concern. Your ecg (electro cardio gram) is suggestive of left bundle branch block. There are two conduction systems in heart by which pulse is generated and heart beats. One is right bundle branch and other is left bundle branch. Left one supplies left side of heart which is main part of heart which supplies blood to whole body. So any conduction defect in this part will cause reduced blood supply to body organs like brain, eyes, lungs, liver, kidneys etc. So your symptom of sight inturption is due to reduced blood supply to eyes. So you should immediately consult cardiologist and start treatment. You may need pacemaker insertion. Hope I have solved your query. I will be happy to help you further. Wish you good health. Thanks."
},
{
"id": 52088,
"tgt": "Is it serious to have some dark urine?",
"src": "Patient: I had some dark urine on Tuesday. I started drinking water and cranberry. I even had a beer on Tuesday night. Wednesday morning my urine was light yellow. After coffee it was a little darker. I keep drinking water and my urine is clear to yellow. Doctor: Hi. Welcome To HealthcareMagic Urinary tract infections, stones can cause the dark urine. Even the diet , if colored cause change in urine color. I recommend Urine Analysis and culture. Continue Drink plenty of water. Also Drink Cranberry Juice. Avoid sitting for long periods. Avoid stress. Some times it resolves after 2-3 days. Regards, Dr. Jagdish"
},
{
"id": 56162,
"tgt": "What does liver of metastatic adenocarcinoma indicate?",
"src": "Patient: I m 39 yrs old, 4 11 in height and 113.8 pounds with biopsy result in my liver of metastatic adenocarcinoma and positive of hepatitis B. recieved the result of my LDH is 371.0 and CA 125 is 16.80. what is your empression about my LDH and CA 125 result? Doctor: Hi there,Thanks for posting in HCM,Your LDH is on the higher side. The CA 125 is not significantly raised.Metastatic adenocarcinoma in liver means, you have a cancer elsewhere and it has spread to liver. Since the cancer is an adenocarcinoma, the most likely site is colon from where it would have spread to liver. You can get a colonoscopy test done to look for cancer in the colon.I hope that answers your query.Regards,"
},
{
"id": 29093,
"tgt": "What precautions should be taken while suffering from tuberculosis?",
"src": "Patient: I got a TB test today and it gets read on Wednesday. I forgot to ask the nurse wat wash and care instructions there are. Can I shower and is there anything I need to do before I shower so I dont mess the test up? Also should I leave a bandage on it or no? Doctor: Hello,Firstly, if you have been diagnosed with Tuberculosis (TB) then you will be started on DOTS regime and will be asked to cover up wearing a mask.But, if you have just gotten a test done wherein you are injected and a circle is drawn around (Monteux Test) you may carry on with your daily activities including your daily hygiene. You may however leave the bandage on.Hope I have answered your query. Let me know if I can assist you further.Regards,Dr. Vignan Rachabattuni"
},
{
"id": 151742,
"tgt": "What treatment should one take for subdural hematoma in a 5 month old ?",
"src": "Patient: My son born on 13th September 2011 and next day he got some problem for which we took him into Government hospital where we were told that he got hypoglycemia and later his heart beat stopped. The heart beat they recovered and gave us manual ambu to pump. We did for 5 hours and when enquired how long we have to do that, we were told that they do not have any working ventilator and then we shifted to different hospital where he treated on Octroid at first to control insulin and then diazoxide for controlling sugar and saved the life but because of hypoglycemia and non supply of oxygen to brain , his brain has a gross damage. The second MRI reflected many pot holes at different place and in the left brain there was a clot also (subdural haemotoma) which later drained out surgically and placed a shunt there. Now as were told there is no chance of recovery of brain and he surely be moving towards spastic. I want to know if any thing we can do to revert such damage?? If required, I will post all the reports (MRI, CT-SCAN, Discharge summary etc). The baby boy is now 5 months old. Doctor: Since the convenctional mode of treatment is of no help, Homeopathy can help you if you believe in the science. You can give you child the following treatment, since thesee medicines have npo side effects it can safely be given to the baby. 1 dose of Nat phos 200. Either 1 drop on the tongue of rub the medicine in the web of baby's finger Arnica 200 repeated every six hours to be given in the form of drop on the baby's tongue or can be rubbed on the web of the baby's finger."
},
{
"id": 84448,
"tgt": "Am I having side effects from Eption 3*100mg?",
"src": "Patient: i am taking eption 3*100mg tablet daily..some red spoits are occured on my head, looks as some skin allergy, do it due to tablets ...looks as dandruff but if scrach then red skins spots occur..do i stop this medicine and start some other medicine.. Doctor: HiSkin rashes,itching,peeling of skin and other dermatological side effects can occur with phenytoin.In case of severe side effects ,the drug should be stopped.Other alternate anticonvulsants should be started immediately after physician consultation to avoid risk of seizures.Hope that was helpful.Let me know if i can assist you further.RegardsDr.Saranya RamadossGeneral and Family Health Physician"
},
{
"id": 34641,
"tgt": "What is the treatment for dog bite?",
"src": "Patient: Dog bite to my husband two days back he has taken tetnuis & dob bite injection on the same day but yesterday night he is having some fever. and his next injection on tomarrow. but where dog has bite there is already some skin infection. is it create any problem Doctor: hello..do not worry..if antirabies vaccine and TT shot taken already..if the bite site is getting infected a course of antibiotics and local ointment would be sufficient.if the bite is bigger and the dog is infected dog then he has to take immunoglobulin injection..hope my answer helps youif you have any more queries i would be happy to answer.."
},
{
"id": 206363,
"tgt": "What causes hair loss, night tremors/sweats and anxiety?",
"src": "Patient: Hello! I have been struggling for a few years now on and off with my symptoms. I am gluten/dairy free and grain free 80% of the time. I struggle with hair loss, night sweats, waking up with tremors, fatigue yet high cortisol levels, anxiety, and candida(my naturopath said I had it). I m just wondering what the heck is going on. I can barely tolerate any type of sugar now without having the shakes the next morning. Help! Doctor: HiI had gone through your query.It is features of a stress and anxiety.Proper psychological evaluation will give confirm diagnosis.Physical examination with blood investigation is also necessary to rule pit any underlying physical illness that precipitate psychological stress.medicinesI have treat so many patient with mild dose of benzodiazapine and mild dose of SSRI,in SSRI mild dose of escitalopram and paroxetine like drug is very usefulBut before taking this medicines you have to consult psychiatrist and evaluate your self.Besides this some natural ways to treat anxiety and daily stress areExercise meditationdeep breathing and hyperventilationproper sleepstress free activitydo pleasurable activitymusic and hobbiesdiscuss with friendssee all thing with different anglepositive outlookconfidence all this will help you to counter daily stress.Still if you have query then feel free to ask.You can directly ask me on http://doctor.healthcaremagic.com/doctors/dr-vishal-garala/68046Thank you.Get well soon."
},
{
"id": 130175,
"tgt": "Suggest treatemnt to make leg look normal after polio",
"src": "Patient: heyi'm 41, a woman, got polio since 1 year old, my right leg is weaker and thinner that left one, it bothers me a lot. is there any way, like plastic surgery to make them look like together.I live in Vancouver, British Columbia.thanks for your time and considerationNarciss YYYY@YYYY 1-604-360-2049 Doctor: Hello,This is a difficult problem. It is unlikely that a plastic surgeon could make the legs look identical. However, intensive physical therapy could improve the strength and may increase the muscle mass of the weak leg.Regards"
},
{
"id": 79423,
"tgt": "Is chest tightness a sign of brain problem?",
"src": "Patient: Hi doctor, My son is a doctor and he 36 years now. He had burping and tightness of the chest. Cardiologist did stress test and blood tests. Non sure indication of blockage. But suspicious and have decided to do angigram and said that if that is negative he wants to do MRI on the brain. What is the connection between chest tightness and brain MRI? The last 2 procedures are planned for next few days. Saravana Doctor: Thanks for your question on Health Care Magic. I can understand your son's situation and problem. I don't find any relevant connection between chest tightness and brain pathology. By his history and description, possibility of GERD (gastroesophageal reflux disease) is more. It is due to laxity of gastroesophageal sphincter. Because of this the acid of the stomach tends to come up in the esophagus and cause the symptoms of Central burning chest pain, burping, chest tightness, pressure etc. Better to rule out cardiac diseases first. So no harm in doing Angiography. But before MRI brain, discuss with doctors about possibility of GERD. He may improve with proton pump inhibitors and prokinetic drugs. Ask him to avoid stress and tension. Avoid hot and spicy food. Don't worry, he will be alright. Hope I have solved your query. Wishing good health to your son. Thanks."
},
{
"id": 12700,
"tgt": "Suggest remedy for rashes in elbow",
"src": "Patient: hi.. i am a 26yr old male..work as a barista and recently devolped a rash on my inner left elbow...it kinda burns and is in lines going up my arm? i noticed it after being sick the other day ... it doesnt go away when under a glass and seems to get more violent as the day goes on Doctor: Hello, I have gone through your query and it could be either herpes zoster or eczematous rash. I would recommend you to consult a doctor and get examined. Hope I have answered your query. Let me know if I can assist you further. Take care Regards, Dr. Asmeet Kaur Sawhney"
},
{
"id": 151591,
"tgt": "Multilevel degenerative disc disease, disc bulging, herniated discs, central spine stenosis. What type of activities should I avoid?",
"src": "Patient: hii I have multilevel degenerative disc disease with multiple bulging and herniated disc from C3-4 down through C6-7 with mild central canal spinal stenoses at C4-5 and C5-6 levels. Multiple herniated disk in the upper thoracic spine, worse at the T3-T4 level. Multilevel foraminal stenoses. what type of activities or jobs should I avoid Doctor: Avoid intense exercise and physical activity, particularly heavy lifting and trunk twisting, if there is acute back pain. Try Rhustox, Ruta, Colchicum, Arnica remedys in 200 potency . These medicines often provide significant benefits. Try alternating between hot and cold packs. Some doctors recommend changing from hot to cold every 3 minutes and repeating this sequence three times. Others believe ice packs should be applied first. This routine should be done two or three times during the day. Supportive back belts, braces, or corsets may help some people temporarily, but these products can reduce muscle tone over time and should be used only briefly. Exercise, diet, stress, and weight all have a significant influence on back pain. Changing certain lifestyle factors can help reduce, and possibly prevent, backaches.Yoga relieves low back pain better than conventional exercise Dr Rajashekar"
},
{
"id": 144205,
"tgt": "Need treatment for bulging and herniated disc with dizziness and blurred vision",
"src": "Patient: im 52 old ,i have a lot of buldiging disc and hernated disc also fused vertebre in neck and lower 5 disc are dried out .im having dicey spells with overwhelmingly weakness ,dizziness and pressure in my headand at times disoridianted with blury vision Doctor: Hi, I am Dr.Bruno. I have read your question and understand your concerns. Let me try to help you From What you have described, i find that you are suffering from Cervical Spondylosis. The Vertebral Artery (Blood Vessel supplying your brain stem) is likely to be compressed and this can cause the dizziness and blurred vision I would suggest you to consult a Neurosurgeon at the earliest Hope you found the answer helpful.If you need any clarification / have doubts / have additional questions / have follow up questions, then please do not hesitate in asking again. I will be happy to answer your questions.Let me know if I can assist you further.Take care."
},
{
"id": 138756,
"tgt": "What is the cause of loss of balance?",
"src": "Patient: I have trouble with balance-I don t have painful feet as some acquaintenances who felt I have neuropathy: At times I do have a lack of feeling in my feet. Took a several week treatment with a foot vibrating machine and also a rebuilder(water to stimulate feet for nerve activation) which may have helped balance somewhat. In addition an electro-magnetic delivery system connected from my knees to the bottom of my feet; which resulted in no perceptable change. Doctor: HelloThank you for posting your query. I have gone through your query and understand your concern. You may be having neuropathy.For which you may require to undergo lumbar spine MRI, thyroid function test (T3, T4 and TSH), Blood sugar test to search for the cause. Mecobalamine will be useful in reducing numbness and tingling caused by neuropathy.Hope your query get answered.Wishing you a good health. Take care."
},
{
"id": 225862,
"tgt": "On birthcontrol. Red pink discharge, Left leg numb. Pills reaction?",
"src": "Patient: I'm on birthcontrol . I've been on it for an entire packet & already on my 2 week on the 2nd packet I had sex on Saturday and this morning I used the bathroom red pink was on the toilet paper . I'm wondering if its still regulating my cycle or something is wrong also my left leg has been numb for a couple days do you think the pills have something to do with that ? Doctor: Hi and thank you so much for this query.Congrats for adopting and using a birth control method. Nothing puts you more in control of your reproductive life than this.The above symptoms are very non specific and cannot be attributed to birth control pills. They are often transient and never really represent any real underlying disease. My advice will be that you continue the pills and monitor whether the numbness and red pink discharge is continuing. If it should continue for more than a week or becoming even more worsening, then you will need to seek expertise advice. For now, stay calm and monitor.Hope you find this helpful and provides guidance on how to approach this worry. If you should have more questions, please return to us and we shall provide answers. I wish you well and hope this gets sorted out soon.Dr. Ditah, MD."
},
{
"id": 82741,
"tgt": "What is the treatment for lupus and vacuities in the thighs?",
"src": "Patient: HI DRS, I HAVE LUPUS AND VASCULITIS TO MY LEGS AND THIGHS. PAIN, SWELLING, CRAMPING, FATIGUE, LOW GRADE FEVERS, RAYNAUD SYNDROME ARE MY SYMPTOMS. WHAT DO YOU KNOW ABOUT GLYCOSYLATION DEFECTS IN AUTOIMMUNE DISEASES SUCH AS LUPUS AND IF AYURVEDIC THERAPY CAN RELIEVE OR CURE THIS DEBILATING DISEASE. i AM 43 YEARS OLD AND HAVE SUFFERED FOR 11 YEARS. I AM NOT TAKING ANY DRUGS BUT HAVE TRIED ALLOPATHIC-NO RESULTS. IVE DONE ONLY ALTERNATIVE MEDICINE,BUT FEEL STAGNANT. NEEDING TO TAKE MY TREATMENT TO THE NEXT LEVEL. WHAT CAN YOU DO FOR ME? I AWAIT FOR YOUR RESPONSE. REGARDS, THELMA TABIOLO Doctor: You should stick with the allopathic treatment as lupus with vasculitis can be a life threatening condition and needs treatment . Steroids and HCQS will be the mainstay of treatment . Belimumab can also be tried."
},
{
"id": 164156,
"tgt": "Is Advil and Tylenol recommended for flu and fever?",
"src": "Patient: My 4 year old daughter caught a cold/flu she gets a high fever 104 and I give her Advil or Tylenol to take it down,it will take one or two doses for it to go away, but it returns after a few hours . Then she gets better for a few days only to relapse, she gets sick ( throwing up) loss of appetite, but will drink fluids, Her doc said it sounds good, good urine etc... says it is a cold that is not rested... I keep her in to rest as much as possible... Doctor: Hi...Thank you for consulting in Health Care magic.Fever of few days without any localizing signs could as well a viral illness. Usually rather than fever, what is more important is the activity of the child, in between 2 fever episodes on the same day. If the kid is active and playing around when there is no fever, it is probably viral illness and it doesn't require antibiotics at all. Once viral fever comes it will there for 4-7 days. So do not worry about duration if the kid is active.Paracetamol can be given in the dose of 15mg/kg/dose (maximum ceiling dose of 500mg) every 4-6th hourly that too only if fever is more than 100F. I suggest not using combination medicines for fever, especially with Paracetamol.Hope my answer was helpful for you. I am happy to help any time. Further clarifications and consultations on Health care magic are welcome. If you do not have any clarifications, you can close the discussion and rate the answer. Wish your kid good health.Regards - Dr. Sumanth MBBS., DCH., DNB (Paed).,"
},
{
"id": 219763,
"tgt": "What causes fluid discharge after masturbation during pregnancy?",
"src": "Patient: Hi, I am 37 weeks and 1 day pregnant and I was masturbating using only one finger. I was also laying on my back. All of a sudden I felt fluid run down in two pulses and didn t feel like it qas coming out of the pee hole. It was maybe the volume of about 1/2 cup total. When I smelled it, it was hard to tell if it smelled like urine or not. I have no had any contractions yet. Do you think this was waters breaking?? Should I wait and see if more comes out or if contractions start? Doctor: Hello,Thanks for writing to us.It doesn't suggestive of water breaking. If your bladder is entirely empty during masturbation, then chances are that the sensation of having to urinate has more to do with pressure on your bladder, or potentially a precursor to ejaculation.Also, if your bladder is empty, then you know that if any fluid does come out of your urethra during masturbation or orgasm is not urine. It is similar to male type ejaculation, seen among women during orgasm.Urinating after masturbation is also a good idea, because it flushes out any bacteria from your urethra and can help prevent urinary tract infections.Hope, it helps for your information. Good luck and take care."
},
{
"id": 125774,
"tgt": "What causes pain in the left arm?",
"src": "Patient: I have had this pain in my left arm for the last week. It started to feel like I pulled a muscle or maybe pinched a nerve in my shoulder blade area. Now the pain runs down my whole left arm and it feels sore like someone was punching my arm over and over again. Its a bit hard for me to give a tight grip and my fingers are a bit numb/tingly. Doctor: Hello, Most probably it will be a simple musculoskeletal pain. As a first line management, you can take analgesics like Paracetamol or Aceclofenac for pain relief. If symptoms persist better to consult an orthopedician and get evaluated. Hope I have answered your query. Let me know if I can assist you further. Regards, Dr. Shinas Hussain, General & Family Physician"
},
{
"id": 210787,
"tgt": "What could cause hallucinations, sleepiness, lethargy and tremors in arms?",
"src": "Patient: What illnesses causes someone to start hearing voices, talking to themselves, hallucinations, withdraw from normal activities and friends, sleeps more than awake. Later they develop an arm tremor and constantly rubs their fingers together. Sometimes they sit alone with blank expression. Doctor: Itappears you are having anxiety neurosis It requieres consul tation with psychiatrist fordiagosis management and councelling."
},
{
"id": 83527,
"tgt": "Does intake of telma 40 cause side effects in respiratory tract?",
"src": "Patient: i was taking telma 40 to control my BP which is in the range of 130/~90. i also have sinus problem and suffer from upper respiratory infaction if i do not take precaution specially in winter. as i have read that one of the side effects of telma 40 is upper respiratory infaction, is it safe for me to use it. Doctor: Hello, Telma (telmisartan) has got no relation with upper respiratory tract infection. If the cough or sinus problem persists, consult a general practitioner and get evaluated. Hope I have answered your question. Let me know if I can assist you further. Regards, Dr. Shinas Hussain, General & Family Physician"
},
{
"id": 84723,
"tgt": "Are stomach pain and back pain caused by side effects of duphaston tablets?",
"src": "Patient: I had duphaston tablet for 10 days twice a day....ie from 14 days of my cycle to 24 day...usually my cycle is of 26 days..this month i got periods on 29th day and it vey painful...i hav sharp stomach and back pain....i am worried to continue duphaston for next month... Doctor: Hi, Oh, I cannot say in your particular case, but generally, it will change the cycle. AND inhibiting the cycle ending and having it build up is pretty common. There are many adjustments of this or a different drug that would be another roll of the dice to see if it worked out better. BUT mostly you'd get the same results every month with the same pill. Hope I have answered your query. Let me know if I can assist you further. Regards, Dr. Matt Wachsman, Addiction Medicine Specialist"
},
{
"id": 73489,
"tgt": "Suggest treatment for severe lung infection",
"src": "Patient: hello , my grandmother has uti problem since 1982, now she has been admitted with severe lung infection and is on ventilators since last 4 days.. 1) she took sugar medicines a week back.. 2) oxygen level in her body was 40%..light bluish finger tips... 3) she had fever 100degrees Doctor: Hello dearWarm welcome to Healthcaremagic.comSorry to hear about the condition of your grandmother .* As per the narration she is in terminally ill condition .* Only supportive care can be possible with ventilator , IV antibiotics , IV fluids and controlling electrolytes , BP and other parameters to support her vital systems , outcome depends upon the severity of the condition as well as many more factors to be evaluated by in person examination only .Wishing her peaceful life ahead .Regards ."
},
{
"id": 219537,
"tgt": "Can a hair dye cause allergies, headache and tiredness in a pregnancy?",
"src": "Patient: I colored my hair yesterday. I am 29 week pregnant. Today im having awful allergies I can breath I have a headache and I feel extremely tired. is this because of the hair color? am I possible getting sick? or could it be from something else. I did get hit in the face over the weekend and my nose is still kinda sore but its not swollen or anything. Doctor: Hi.For the allergies, speak to your doctor to prescribe a safe anti-histamine (e.g. cetrizine or loratadine are considered safe during pregnancy); as for the headache and breathing difficulties they could be due to the injury to your nose which should heal in given time.Best wishes."
},
{
"id": 58916,
"tgt": "Had gall bladder removed. History of pancreatitis. Vomiting, red marks on abdomen, itching. Your thoughts?",
"src": "Patient: Hopefully this is a non-fee site here. email: YYYY@YYYY Just had my gall bladder removed. I had a severe case of Pancreatitis a year ago, but after several tests nothing was found. The only thing that revealed anythingwas a simple blood tests, which revealed a severe level of an Pancreotic enzymeknown as Lypace, which was at 26,000 rather than below 300. Fast forward to this past Wednesday. I had been again vomiting after eating food with acid and some fat (spaggetti with tomato sauce, hamburger and later the chips. I say allthis to provide background information. This past Wednesday, 6/5/2013, the gallbladder was removed. The next day, and here is why I'm writing, I have an across my abdomen with acouple of off shaped red marks on my right side--NO not the surgery incisions.The rash does itch a little bit, sometimes more so than others. I will be keepingthe checkup type routine post surgical appointment with the surgeion this week, but wanted to know your thoughts. Doctor: Hello,i went through your statement. Acute pancreatitis is confirmed by elevated serum Amylase. The indication for removal of gall bladder is not mentioned as well as were you jaundiced. Any patient with their gall bladder removed must take for certain period fat free diet and life long with minimum fat.The off shaped red marks may be due to multiple reasons. Unless a Dr sees them personally it is very difficult to comment on it. Sorry.With best wishes"
},
{
"id": 225860,
"tgt": "Was on Diane, stopped recently. PCOS, high testosterone levels, partner s semen fell near vagina, took an ipill. Will it be effective ?",
"src": "Patient: hi, i am a 19 year old i hav had symptoms of pcos..wen i did the tests only the testosterone level was high in my blood where as the other hormones were within the normal range and there were no cysts found in my sonography..so my doctor prescribed me Diane 35 which i was having for one year...now my doctor has asked me to discontinue it so its been over 4-5 months i am not having it. today i had an i pill. will it be effective? will it work? i did not hav unproctected intercourse but i felt my partners semen fell near my vagina..so to be on the safer side i took an i pill in a few hours. will it work? Doctor: Hi,Thank you for choosing Healthcaremagic. As you took I pill within few hours , it is very effective in preventing the pregnancy. But, watch out if you miss your periods.I hope this information has been both informative and helpful for you. In case of any doubt, I will be available for follow-ups. If you like my answers kindly rate it, and write a review as well. Please do not forget to accept it.Thank you,Wish you good health.Regards,Dr ArifYou can consult me again directly through my profilehttp://www.healthcaremagic.com/doctors/dr-arif-n-khan/65133"
},
{
"id": 85434,
"tgt": "What is the dosage and side effects Susten VT 200?",
"src": "Patient: HI Team, i had a question related to use of Susten VT 200 tablets. My wife is pregnent now (6 weeks 3 days) and doctor has asked us to use these tablets. just want to know the use of these tablets and any side effects of using it. Thanks for your help Doctor: Hi, Side effects are: Hot flushes Chest Pain Fever Difficulty In Passing Urine Breast Pain Muscle Or Joint Pain White Or Brownish Discharge From The Vagina Headache Dizziness Depression Viral Infections Breast Lumps Loss Of Vision Or Blurred Vision Unexpected Vaginal Bleeding Convulsions Swelling Of Face, Lips, Eyelids, Tongue, Hands And Feet Constipation Hope I have answered your query. Let me know if I can assist you further. Regards, Dr. Ajeet Singh, General & Family Physician"
},
{
"id": 173584,
"tgt": "Suggest treatment for white tongue and bad breath in a child",
"src": "Patient: Hi. My 3yearold daughter has a white tongue, inflamed swollen gums that bleed when i clean her teeth with a wash cloth. she ran a fever about 102 give or take for almost5 days.it lookslike maybe a ulcer, blister or lesion or coldsore onherlip(not sure). she is in pain. bad breath,maybe teething. whatshould i do??? Doctor: Hi,Thanks and welcome to healthcare magic.Oral ulcers,bad breath,swelling of gums,bleeding from gums and bad breath may be due to vitamin deficiency with superadded secondary bacterial infection .Prolonged antibiotic therapy may also lead to above condition .Treat with vitamin B complex and vitamin C in high doses will facilitate healing.Listerene mouth gargle as well as local application of Zytee liquid will lessen the suffering.Judicious use of antibiotic may cure this condition .Good food and oral hygiene will prevent further infection .Hope this answer is OK for you.Please feel free to ask further queries if any.Dr.M.V.Subrahmanyam."
},
{
"id": 136478,
"tgt": "Suggest treatment for severe ankle pain",
"src": "Patient: Hi Sir, My mom have severe pain at angle. As instructed by an orthopedic surgeon we took x ray & then scan of ankle. it shows hyper intence latyc lesion at ankle bone & Doctor told us needed surgery & then need to biopsy. Kindly suggest us any other option than surgery and kindly let us know what is the actual problem or disease. Kindly give your valuable advise Doctor: hi,i appreciate your concern about ur mother.pain and lytic lesion in bone may be the cause of pathological fracture.among so manytypes of lytic lesions,most worrisome may be the multiple myeloma which needs tobe confirmed by biopsy,followed by other relevent investigations.if talus bone is affected,being a wt bearing bone,chances of bone collapce is higher.hence it is advisable to follow treatment advice of your treating orthoexpert.this will help to establish diagnosis.thanks.[will be happy to answer any further quary."
},
{
"id": 20797,
"tgt": "Suggest treatment for heart palpitation,diarrhea and fatigue",
"src": "Patient: Hello. I had to call for an ambulance on Monday and was diagnosed with Tachycardia. They couldn't find anything else wrong in the ER and sent me home. Since Friday I have been experiencing heart palpitations, diarrhea, shakes and fatigue. I try to lay down and sleep but can't because the symptoms come in waves. The episodes last for about an hour, then I might have a half hour symptom free. Am I just having panic attacks? Doctor: Get your holter monitoring done so that your whole day profile of heart rythm can be considered before labelling you to have panic attacks"
},
{
"id": 115060,
"tgt": "What causes reduction of blood count with loss of hair, skin rashes and swollen neck?",
"src": "Patient: i am a 44yr woman, i am suffering from reducing blood count.now i am having wyslone 20mg ,azoran 40mg,folvite 5mg ,thyronorm 100 and gemfos.does this affect my health?now i am facing problems like losing hair ,rashes on face,swelling on neck and face.I am having the medicine for almost 2 months. Doctor: Hi,Thanks for asking.Based on your query, my opinion is as follows.1. Wysolone medication can cause bone marrow suppression leading to reduction in blood counts along with hair thinning and hair loss. Associated nutritional deficiency also requires evaluation.2. Hypothyroidism has caused goiter which has resulted in swollen neck. If there is any compression symptoms due to thyroid, surgery maybe necessary.3. Vit B12 supplements along with steroid dose reduction is necessary. Also skin moisturizers with good hydration will necessary for skin rashes. Once overall nutrition improves, you will start recovering. Hope it helps.Any further queries, happy to help again."
},
{
"id": 124346,
"tgt": "How to cure an indentation that is noticed on the left hip ?",
"src": "Patient: Hi, im 33 years old and ok physical health,5 6 and 152 lb. I notice about one week ago a big indentation on the front part of my left hip, almost at the level of the troque. i have also been sore in that area when I walk and now most of the time, the pain is not hard but is constant. What do you think it could be? Doctor: Hello, As you mentioned the indentation over the high region, is it on the anterior aspect of the posterior aspect. If it is on the anterior aspect, I mean over the waist bone front side then it may or may not be an iguinal hernial. We can rule this out by doing an ultrasound scan over the area and find out. If that indentation is troubling and giving pain while walking then it may need surgical removal. In case the indentation you mentioned is on the posterior aspect, I mean over the back side then it should be get examined first by a general physician that it is any lump or some notch. We come to a conclusion if the diagnosis is made properly based on the proper clinical complaints. As your history is little, in general, it is difficult for me to give you a precise diagnosis. But in general, I have given an opinion about what it could be so please take a good call. Hope I have answered your query. Let me know if I can assist you further. Take care Regards, Jay Indravadan Patel, Physical Therapist or Physiotherapist"
},
{
"id": 214166,
"tgt": "What techniques can one use to reduce paranoia ?",
"src": "Patient: What techniques can one use to reduce paranoia in paranoid personality disorder? Doctor: People with PPD (paranoid personality disorder) usually do not seek treatment, as they do not see themselves as having a problem. The focus should be increase his general coping skills, improve social interaction, communication and self esteem. Trust is an important factor in treatment because the patients have a tendency to distrust and refuse the treatment . Psychotherapy is the mainstay of treatment . Medication generally is not used to treat PPD. However, medications, such as anti-anxiety, antidepressant or anti-psychotic drugs, might be prescribed if the person's symptoms are extreme, or if he or she also suffers from an associated psychological problem, such as anxiety or depression. Consult the psychiatrist for these."
},
{
"id": 91798,
"tgt": "Is severe abdominal pain after taking perssery following severe thrush episode concerning?",
"src": "Patient: Hi I took a pessery following a severe thrush episode and experienced significant abdominal pain four hours later and have experienced right abdominal pain all of today. I would appreciate your advice. Is this something I need to worry about and could this cause pelvic inflammatory disease? Thank you. Doctor: Hi. This sort of a problem is not really encountered , but in medical practice anything is possible.It would be wise to consult a Gynecologist for clinical examination , ultrasonography and additional investigations which may be thought by Her.It is possible that thrush only caused the pelvis inflammation and co-incidentally pessary was also used. Of course , stop using pessary. Continue other medicines as advised by your Gynecologist. You are free to take second opinion of a Surgeon in case you are not satisfied."
},
{
"id": 154612,
"tgt": "What is the prognosis for nasopharyngeal cancer with metastasis without treatment?",
"src": "Patient: Hello! My husband was diagnosed with Staged 3 Nasopharyngeal Cancer with EBB Metastasis & lymph nodes in the neck in December 2010. He was on chemo treatment for 3 mons. & decided to stop NO treatment at all. He rejected the radiotherapy. He is 49 yrs., in good health, non smoker & he does not drink. What is his life expectancy. Please advice. Thank you for the help. Doctor: Hi, dearI have gone through your question. I can understand your concern.Your husband has nasophryngeal cancer with metastasis. So surgery is not possible at this stage. Treatment of choice remains chemotherapy and radiotherapy. Despite of treatment prognosis is not good. Moreover there are many side effects of chemotherapy. SO without treatment he can avoid side effects. Life expectancy is not good with or without treatment. There is not much difference in life expectancy. Consult your doctor and plan accordingly.Hope I have answered your question, if you have any doubts then contact me at bit.ly/Drsanghvihardik, I will be happy to answer you.Thanks for using health care magic.Wish you a very good health."
},
{
"id": 189598,
"tgt": "Had molar extracted, dental implant inserted. Having muscle spasms on left side of face. Unable to open mouth. Suggestions?",
"src": "Patient: i need advice form a neurologist . I had the left lower 3rd molar extracted and dental implant inserted at the same time on 13 March 2013. 7 days later i started getting messeter muscle spasms and pain in L side of face. Dentist removed implant on 26 March 2013 and said it is most likely a dry socket. i can not open my mouth at all now since 24 March and if I try I get the most excrutiating pain in the masseter muscle . I am taking anti-inflammatories, Flagyl antibiotics, muscle relaxents but still no improvement. The muscle is in constant spasm but how can i prevent the acute spasms when trying to open my mouth Kathleen Doctor: Hi, Thanks for aksing the query, Pain can be because development of dry socket , dental implant should be placed after healing of the wound . Take complete course of antibiotic and analgesic . Use saline water gargles 3-4 times daily , use antiseptic mouthwash gargles twice daily this will prevent infection and promote healing. Avoid eating hot and spicy food . Maintain a good oral hygiene . Your Dentist can even clean the socket to remove the debris and place a dressing of zinc oxide eugenol to promote healing . You probably have to come back to the dentist periodically to change the dressing . Dry socket ususlly takes a time upto two weeks to heel. Hope this helps out. Regards..."
},
{
"id": 206236,
"tgt": "What causes hair fall with insomnia & fatigue?",
"src": "Patient: since december, i've been having issues with my hair falling out, insomnia, breakouts, general lack of energy, and a strange greasy pinkish tint and almost ham like smell to my urine. i have a doctor's appointment with my family doctor on the 14th, but i'm hoping that perhaps i can get a rough idea what i should be suggesting she looks for... Doctor: Hi.I understand your concern .Fatigue , weakness and insomnia may suggest underlying stress . Further detail evaluation may yield depression or anxiety disorder.So proper psychological assessment is must.Hair fall may be part of stress or may be due to some hair disorder. Need to consult dermatologist for underlying causes.Urine routine and micro examination is necessary to reach proper conclusion or rule out infection.So consult physician and go for basic investigations that wil help to reach proper diagnosis.I hope I have answered your question .Still if you have a query then feel free to ask .Thank you ."
},
{
"id": 197923,
"tgt": "Will daily masturbation affect the consistency of semen?",
"src": "Patient: hi doctor,i just wanted to ask something about semen consistency, because my fiance is having clear and liquid semen after ejaculation for how many months already, thou were not sexualy active,, but he admits that almost evry day he is masturbating, does it affect its consistency. because im worried maybe he has infertility problem. Doctor: helloThanks for query .Your fiance is indulged in frequent masturbation almost every day and you have noticed hi semen to be watery in consistency .Normally it takes 2 days for seminal vesicle .prostate and testicle to produce semen of normal consistency and volume hence it is natural to have semen to be watery in a person who ejaculates frequently over a short period of time .You can ask him to get his semen analysis done after observing abstinence for 3 days (No ejaculation for 3 days either by sexual intercourse or masturbation ) to be rest assured about his fertility .Dr.Patil."
},
{
"id": 141374,
"tgt": "What causes sudden stiffness in the body after a seizure attack?",
"src": "Patient: my brother 57 had what appeared to be a seizure as his body became very stiff, eyes fixed and he fell backwards fracturing his skull causing internal bleeding that took his life 24 hours later. I want to know if the seizure caused the stroke or a stroke caused him to have a seizure? he was stiff for almost 10 minutes and until medics arrived. Doctor: Hello and Welcome to \u2018Ask A Doctor\u2019 service. I have reviewed your query and here is my advice. I would explain that we cannot know if the trauma has caused the intracranial bleeding or the opposite. Anyways, the autopsy would have helped to exclude a possible brain aneurysm, which could have lead to intracranial bleeding followed by the seizure and head trauma. Hope I have answered your query. Let me know if I can assist you further."
},
{
"id": 211957,
"tgt": "Lower back pain, painful to sit or stand. Difficulty sleeping, feeling hungry. Suggestions?",
"src": "Patient: I am having severe pain in my lower back it is extremely painful to sit, stand or lay down. i hurt so much I can't sleep! I feel hungry but can't eat! I have a Dr. @ Kaiser in Lakewood, Colorado but she won't give me anything for pain. This is very depressing, It hurts sooo BAD I'm about to locate other means of something for my pain!! What would you suggest for me too do??? Doctor: HI, thanks for using healthcare magicIf the pain is so severe that it is affecting the quality of your life then you may want to consider consulting a pain specialist.These are doctors that specialise in the management of any type of pain and would be to prescribe therapy that may be able to help you.This would include medication as well as physical therapy.Your doctor may be able to refer you to one of these specialist.I hope this helps"
},
{
"id": 194341,
"tgt": "Why is it burning while urinating after an unprotected anal sex?",
"src": "Patient: I had unprotected anal sex with this girl about a year ago, and I am now noticing that I have a ear ache, and I have white patches of skin in the inside of my mouth and throat, I went to the doctor when it was just in my throat ,and he said it was possibly just food stuck in the pockets of my throat , but now its on my cheek right next to mollers I really think I have somthing wrong with me. I also notice that after ejaculation it really kinda burns to pee is this normal as well ? Doctor: Hello, Since its an unprotected sex, you need to take Sexually transmitted panel test to rule out infections. Burning sensation during urination can be due to STI or a simple Urinary tract infection. Hope I have answered your query. Let me know if I can assist you further. Take care Regards, Dr S.R.Raveendran, Sexologist"
},
{
"id": 104900,
"tgt": "
Suffering from cough, asthma. When walking, turns leg, falls down. Medicine for asthma?
",
"src": "Patient: Hi Sir, I have to queries.. 1. My dad is suffering from cough & ashthmatic problem from long time...sometimes it goes worst. doctor says its due to allegry & one doctor has suggested deriphyllin drug & one has suggested doxyphyllin drug.. so my question is that which should we continue for treatment...also tell me the side effects of it. 2. there is a problem with his one leg..during walk whenever he tries to turn that leg stops moving suddenly & just fall down..it happens so frequently & he gets injuries on body..brain MRI has been done..but doctor could not diagnos the cause. & his voice is also shivering .. Please suggest on this.. Thanks in advance, Krati Doctor: Hi krati ! Doxyphillin will be better but like all drugs it also have side effect but less. About leg try contrast x ray of pelvic . But at present do massage , warm compress, and most of the time lay down or if sit make leg straight. and last do diabetic test too.. Take care and get well. Bye."
},
{
"id": 57964,
"tgt": "What is the reason for pancreatitis and stomach bloating when the gall bladder removal being scheduled?",
"src": "Patient: My brother is in the hospital in Nova Scotia and I am in Ontario trying to get some infoabout his condition. I am told he will be having his gall bladder removed but they are treating himright now for pancreitis and his stomach is so swollen and he is in so much pain that they are keepinghim sedated and waiting for the swelling to go down before they can do anything.Can you give me any ideas about what causes this and what I can expect Doctor: Hi...the abdominal swelling could be due to ileus following pain and pancreatitis as he must be kept nil by mouth...or could be due to acsites (reactionary fluid)... both will resolve with treatment... you need not worry...Dr. Ashish Verma"
},
{
"id": 52145,
"tgt": "Do hypertension causes kidney failure?",
"src": "Patient: my nephwe is suffering from hypertension since 16 years and recentely i came to know that he is advised dialysis for kidney failyre. do hypertension cause kidney failure? Doctor: Yes, hypertension which is not controlled or not treated properly can treat kidney failure. So it is very much necessary for your blood pressure to be controlled."
},
{
"id": 121114,
"tgt": "Is Xerina effective for foot fissures?",
"src": "Patient: I had a lot of fissures on my foot, and was advised to use Xerina creme. It was helpful and the foot fissures vanished. I stopped using it since then. Now i am in my second trimester and the foot fissures have reappeared, can i use the same creme again? Doctor: Hello,I read carefully your query and understand your concern. Apply Xerina cream for treatment of feet fissures at night. It should be used for application once daily. Apply Fucidin cream over cuts.If the symptoms continue, I suggest to see a dermatologist and get evaluated. Hope my answer was helpful.If you have further queries feel free to contact me again.Kind regards! Dr.Dorina Gurabardhi General &Family Physician"
},
{
"id": 83876,
"tgt": "What are the side effects of postinor?",
"src": "Patient: HI, I took postinor 2 double dose on the 31 march at 8:15p.m and the other 01 April at 8:15 a.m. Howevr, I had unprotected sex on the 4th April, so i would like to know if i take another postinor how harmful will it be. I know its not for contraception. Thanks Doctor: Hi, Postinor-2 is an effective form of emergency contraception. Its common side effects include vomiting, headache, breast pain and bloating. You should use positnor-2 only in emergencies and not as a regular method of contraception. If it is used more than once in a menstrual cycle it is more likely to upset your menstrual cycle causing irregular or heavy bleedings. Hope I have answered your query. Let me know if I can assist you further. Take care Regards, Dr. Mohammed Taher Ali"
},
{
"id": 34377,
"tgt": "Suggest remedy for persistent chronic cough",
"src": "Patient: My mother was suffering from Chronic Cough for a period of 6 months. She is a Chronic Diabetic and she is having insulin injection. Her age is 66 and her height is 155cms and weight is 60kg. She has labile hypertension. She has been prescribed Dilzem by her Cardiologist and she has been given Seroflo 100 and Flouhale 100 rotocaps one capsule two times a day for 3 wks. I would like to know whether Seroflo inhalation for 3 wks is good to her health? As I went through the prescription catalog I found that long administration of Seroflo will lead to rise in blood sugar levels? I would also like to know wheteher Dilzem30 mg can be combined with the administration of Seroflo rotocaps? Doctor: Hi,From the symptoms that you mentioned,it seems that your mother might be having lower respiratory tract infection(COAD/Bronchial asthma/Emphysema).If i were her treating doctor,i would have suggested her to undergo some tests like complete blood count,spirometry,chest x-ray.I would also have liked to rule out the possibility of cough due to cardiac origin as she is a cardiac patient and a known diabetic on insulin.And ECG and Echo would have been helpful in such a situation.Seroflo can be used for the given time period in accurate dosage for symptomatic relief along with dilzem 30 which is a cardioprotective drug.Consult your cardiologist/GP for carrying out the tests and a further review.Take care"
},
{
"id": 128426,
"tgt": "What causes persistent pain in the elbows?",
"src": "Patient: Hi I am a very healthy active woman who has a 3 to 4 day a week work out regimen. Recently over the past 2 weeks my forearms, the surface around my elbows, as well as my hands hurt. Making a fist especially after waking in the morning is pin full I don t have much strength to even squeeze my fist closed. I also have a little lump on my right forearm that s tender to the touch. Not much discomfort in my lower body but mainly upper body.. Doctor: Dear patientyour symptoms points towards possibility of tennis elbow. There is inflammation at the lateral epicondyle of the humerus with pain on making fist and weight lifting in the involved forearm. Treatment is rest and anti inflammatory medications like ibuprofen 400 mg thrice a day. Avoid weight lifting and twisting movements. If not relieved in 5 days please consult expert orthopedic surgeon nearby you."
},
{
"id": 201492,
"tgt": "What could testosterone levels 320 suggest?",
"src": "Patient: Recent blood work returned t levels of 320 and free of 4.04. My doctor is in a rush to put me on trt.I still have great strength and muscle mass is good. I can perform in bed but I will say libido is minimal. I am a 47 yo male. Is trt necessary and what else can I try first? Doctor: Hi,Thanks for writing in to us.Testosterone is an important sexual hormone in the body. At 47 years of age the average testosterone level is 606 and free testosterone is 9.1 and therefore your levels are below the mean. You might be able to perform in bed and the low libido is because your testosterone is about half that of an average male of 45 years age.You can try and improve your testosterone by following methodsMaintain a healthy weightExercise regularly.Take multivitamins and minerals.Sleep for 7 -8 hours every day.Eat healthy foods."
},
{
"id": 68535,
"tgt": "What are the small hard lumps arising on the vagina?",
"src": "Patient: Hi. Please help! I have suddenly noticed lots of small hard lumps on the entrance of my vagina. There is also a lump on my left labia and near my anus. They seem to have come out of nowhere! They are not sore, they do not itch and there is no nasty smell. I am sexually active. Please help as I'm terrified! Doctor: Hi. The painless lumps on the area you have specified are mostly due to a rash or furunculosis. Apply liquid Povidone iodine locally, may get alright. If not consult a Gynecologist as visual impression is very important in diagnosis of such problems."
},
{
"id": 79051,
"tgt": "Should I worry about cough in an infant being an indication of croup?",
"src": "Patient: My 6 month old has been waking up with a barking cough for 6 weeks. The cough sounds just like croup, but not upon breathing- just coughing. It does not keep him up, he just \"barks\" when he wakes up to feed or for the day. Ocassionally, he will cough at naptime or during the day.No other symptoms. Doctor: Hello, thanks for posting on HCM.If you could make out that cough was barking type it could be possible he has croup. It will be confirmed on chest x-ray where we see narrowing in trachea.It will be advisable to consult a pediatrician for the same and treatment to be started as soon as possible. If it stays for long, it may result in breathing difficulty as airway gets narrowed. So I would like to advise to consult as early as possible before difficulty in breathing occurs.I hope I have given satisfactory answer to your query, if you still have any more query kindly write back to me and I will be happy to help.Thank you"
},
{
"id": 191808,
"tgt": "Suggest treatment for knee pain while having diabetes",
"src": "Patient: My mother is a diabectic patient also having hypertension and thyroid also having uric acid.... since from last couple of days she is having knee pain.... its very difficult for her to bend her knee.... our doctor is out of town will be available form next week, so my question is can I give her Hifenac P for the interim period. Doctor: Hi, yes you can give her Hifenac P--but with Omeprazole so that there is no acidity. Also massage knees with any warm oil and do fomentation. Regards"
},
{
"id": 84202,
"tgt": "How to withdraw spasmocip plus?",
"src": "Patient: i am taking spasmocip plus or parvonspa for last 5 years, taking 15 to 20 capsules daily. i want to get rid of this medicine at any cost, when i dont get it i cant sleep and my legs pain, somewhat restlessness is there. kindly tell me the way to get rid of it. Doctor: Hello, You can gradually reduce the dose and frequency of the tablet. Rapid withdrawal is not recommended as it will result in withdrawal symptoms. You can seek help from a psychiatrist if required.Hope I have answered your question. Let me know if I can assist you further. Regards, Dr. Shinas Hussain, General & Family Physician"
},
{
"id": 70037,
"tgt": "What could a blister like hard lump on buttocks indicate?",
"src": "Patient: Hi, this is quite embarrassing but i have a swollen bump on my butt. it's really hard all around the outter part and then has a blister like center. I had one before on my leg and when i went to the hospital they said it was a spider bite. this was a year ago. what would it be? Doctor: Hello!Thank you for the query.It is really hard to give precise answer not being able to see it. Hard lump usually indicates some inflammation. If there is a redness and pain in the lump area, some small abscess can be the reason.If it is only hard, but painless, it may be allergic reaction to some bite indeed.The best way is to consult dermatologist or surgeon with this lump. If it is an abscess, you should have it incised and drained. As it is similar to the lump you have had before, swab test of it should be also done.Hope this will help.Regards."
},
{
"id": 217358,
"tgt": "What cause pain and red mark in my leg?",
"src": "Patient: I had a baby two weeks tomorrow. Since the labour I have had pains in my right leg. This leg I had petherdean in. Mentioned it to Midwife and she advised this is due to injection. The last couple of days Ive had a big red mark appear which is sore to touch and I can hardly move my leg. Doctor: Hello dear,Thank you for your contact to health care magic.I read and understand your concern. I am Dr Arun Tank answering your concern.Congratulations for the baby.If the pain is arising is because of the injection, than it probably is infected.I advice you to take good care of the local wound by cleaning and applying antibiotics ointment over it.You can take the aceclofenac tablet under your doctors guidance this will relieve the pain. If the leg is immovable and can cause pain muscle relaxant like Chlorzoxazone can also be used.I will be happy to answer your further concern on bit.ly/DrArun.Thank you,Dr Arun TankInfectious diseases specialist,HCM."
},
{
"id": 225775,
"tgt": "Feeling nauseous, diarrhea, sweating, heartburn after unprotected sex, on Tri-Sprintec. Could I be pregnant ?",
"src": "Patient: my boyfriend and i had unprotected sex last wednesday afternoon around 1. that night i went to dinner with my parents at souplantation and took my first pill of my fifth month of tri sprintec at 6pm. At 3am the next morning i woke up feeling nauseous and threw up a few times. I had diarrhea, sweating, and shaking/chills. I thought it had to be food poisoning. The next few days I had been unusually gassy. But for the most part felt fine. Then last night, I ate rice with a lot of sliced garlic in it, and got really bad heartburn. I have acid reflux disease so ive had heartburn before, but never this bad. I couldnt sleep much due to the feeling that i would throw up. I got maybe four or five hours of sleep total. Today I also dont feel that great, but Ive also been thinking about the possibility of being pregnant. Ive been very thirsty, but still have a dry mouth when i drink. I have diarrhea and occasionally constipation. I took a clear blue pregnancy test today and it came up negative however I am worried that it is not accurate because it is too soon. Could I be pregnant, or could I just be over thinking and worrying too much? Or is it possible to still have food poisoning, if thats what it was? Doctor: Hi,Thank you for choosing Healthcaremagic. As you had unprotected sex, you should take emergency contraceptive pill like Plan B, rather than taking Tri-sprintec. Yes it is too early for a pregnancy test.I hope this information has been both informative and helpful for you. In case of any doubt, I will be available for follow-ups. If you like my answers kindly rate it, and write a review as well. Please do not forget to accept it.Thank you,Wish you good health.Regards,Dr ArifYou can consult me again directly through my profilehttp://www.healthcaremagic.com/doctors/dr-arif-n-khan/65133"
},
{
"id": 88968,
"tgt": "What causes stomach & lower back pain with rapid heart beat?",
"src": "Patient: i have inconsistant pain in my right side of my stomach and lower back pain as well. i pee frequent. the pain in tolerable unless i ampeeing then there is a sharp pain in my abdomen IF i put pressure on my abdomen. i have a slight 99 fever and a fast heart beat as well. Doctor: Hi Thanks for using HCM.You seem to be suffering from urinary tract infection.Get your urine cultured and sensitivity done.Mean while you can start a broad spectrum antibiotic with urine alkalizer. You can take tab acetaminophen if pain is severe, it will help in fever as well.Get ultrasound abdomen also done if pain is severe and radiating, to rule out any chances of stone, as it may also cause recurrent urine infection.Good luck."
},
{
"id": 23850,
"tgt": "What causes a pinching feel and pain at the center of the heart?",
"src": "Patient: i am 38 years old, weight is 74 kgs and height is 5.9 and there is no previous medical history. I am feeling some kind of pain in the center of the heart like someone pinching or touching with finger and sometimes heart area becomes hard. I had been to doctor for checkup and told me that its a gastric problem but still to make sure i had been for ECG and report was excellent. I use lemon, orange juices and milk frequently. does it produce acidity? Doctor: Hi! I had gone through your question and I understand your concern.Yes lemon , Orange juice and fatty milk is not good for your stomach problem.Lemon and Orange juice have acid in it , fatty milk has a lot of fat and it's heavy for your stomach .Also if I were your treating doctor I would give your some advise :-Avoid alcohol , caffeine ,fatty food ,spicy food ,fried food you can drink milk but with low day in it ( less than 1.5%)-Use boil vegetables (bananas , yogurt ,broccoli, white rice , youghurt -Use plenty of water (2-3l per day).- You can consume juice but diluated juice If you can't manage the pain with diet, you can start the medication to low the stomach acid ( such as Omeprazol .For that you have to decide with your local doctor Wish you good health Thank you for using HCM ."
},
{
"id": 59110,
"tgt": "Diagnosed with hepatitis A. My recovery seems to go pretty slow, am I doing anything wrong? Concerned?",
"src": "Patient: Hi,I was diagnosed with hepatitis A five weeks ago, at that time my SGPT 2063 u/l, SGOT 455 u/l, total bilirubin 7.2mg/dl. I have been checking my blood every 5 days and there has been improvement. currently SGPT 105u/l, SGOT 56, bilirubin 9 mg/dl. my concern is why my bilirubin going up? I would expect the number to go down while I am recovering. Also my SGPT and SGOT have been fluctuate in the past three weeks. Last week my SGPT 65 and SGOT 40, is this something alarming? I have checked with three specialist (Gastroenterologist and Hepatologist) and they all didn't seem too worry about my condition, they said I will fully recover. My family are so scared because I am so yellow (jaundice) and they don't let me near my child, afraid of contagious. It has been very challenging these past few weeks for me and my family. My family have been watching my food diet, I can only eat soup, porridge, and steam fish and vegetables. In the meantime I lost 15lb and still feel nausea with no appetite. My recovery seems to go pretty slow, am I doing anything wrong? Please help... Doctor: Hello, There is no effective drug treatment of hepatitis A since it is a viral disease. So as with any other viral diseases,the affected individuals should avoid contacts with others,take plenty of fluids,and take rest. Since the stool remains very contagious for several weeks into the illness, they should be extra careful with bathroom hygiene. If possible, a separate bathroom should be set aside until recovery. Separating eating utensils may be of benefit. Most of he people suffering from hepatitis A recover completely in 5 to 6 weeks time however around 10% people may have prolonged waxing and waning illness that may last as long as 6 months. There is no chance of this problem becoming chronic and once a person recovers from it,he never gets it again in his life. Your temporary increase in bilirubin levels is Normal and should not be worried. Relax,soon you shall be all right.You are not doing anything wrong.Nausea and loss of appetite shall be allright soon. Avoid alcohol. Thanks"
},
{
"id": 55131,
"tgt": "Could itching in right side of groin and red bumps be due to liver problem?",
"src": "Patient: I HAVE ITCHING ON RIGHT SIDE OF GROWING THEIR R NO BUMPS OR RASH ITS IN THE CRESE OF MY LEG ITS LIKE IT MAKING ME ITCH FROM UNDER MY SKIN ABOUT THE SAME TIME I GOT RED BUMPS ON MY BUT IT WOULD 1 OR 2 OR SOMTIMES MORE COULD THIS BE A PROBLEM WITH MY LIVER OR WHAT PLEASE HELP THAN U Doctor: hi.noted history of right sided itching associated with bumps on your groin area. it may not be related to a liver pathology or problem. the symptoms you are experiencing may be of dermatologic or inflammatory/infectious in origin. for confirmation, it is best if you consult with a doctor for complete physical examination and clinical evaluation. diagnostics and management will be directed accordingly. avoid manipulating the affected area. maintenance of a good hygiene is also advised.hope this helps.good day!!~dr.kaye"
},
{
"id": 191514,
"tgt": "How can high blood sugar levels be treated?",
"src": "Patient: My husband has Type 2 diabetes and received a steroid injection yesterday and his blood sugars have been over 500 today. He's having no problems with his eyesight, no frequent urination and he says he feels fine. He has been battling a sinus infection since last week and that is why he received a steroid shot. He doesn't want to call his doctor. Doctor: You should consult the doctor and good that a work up is done for any complications of hyperglycemia.Since you didn't mention his regular blood sugar levels, I would like to rule out complications. Some complications occur without much symptoms in people with diabetes."
},
{
"id": 95555,
"tgt": "My ALT is 41. Should I be concerned ?",
"src": "Patient: I just got a high AST of 57 blood result. What does this mean ? My ALT is 41. Should I be concerned? Doctor: hello, both your ast and alt are on the higher side. it can be caused to various drugs, alcohol or any disease involving the liver. look out for the drugs you are taking. in case you develop symptoms like pain abdomen and fever, i would advise you to visit a physician regarding the same. avoid smoking and alcohol. i hope i have answered your query. take care and contact us at HCM in case of further query"
},
{
"id": 57761,
"tgt": "What is the cause for dark yellow coloured urine and back pain with the history of gallstones when on prenatal vitamins?",
"src": "Patient: For the past three days I have noticed my urine is a dark yellow and it stains the toilet paper when I wipe. My lower back also has hurt and I have thrown up twic so far. I haven't eaten much because when I do I don't feel well.I have a history of gallstones but have never experienced this before. I also started taking prenatal vitamins two weeks ago because my fiance and I are trying to have a baby. My stool has ranged in color from a tanish light drown to a dull green. Doctor: Hi and welcome to HCM Thanks for the query. it sounds like gallbladder attack and this is the first thing to rule out. your urine color may be changed due to bilirubin elevation which can be seen if your biliary ducts are obstructed by gallstones. vitamins shouldnt be related to your pain. so you need to do ultrasound and check your liver enzymes for start.Wish you good health. Regards"
},
{
"id": 36045,
"tgt": "What causes a dark yellow spot that has appeared little above the thumb & pointer finger?",
"src": "Patient: I have a dark yellow spot that appeared a little above the webbing of my thumb and pointer finger. It has faded since the first day I noticed it (3 days ago) but hasn't changed since then. I havent really changed my diet or lifestyle. I did hit my hand, but in a different area. Doctor: Good day and thanks for your question. Is the spot in any other part of the body? Is this the first time you are noticing the spot? Any history of insect bite or allergy? Is it itchy? Kindly visit a dermatologist to have a look at the yellow spot.Hope it helps. Thank you."
},
{
"id": 40919,
"tgt": "Can OHSS recur and how to avoid it? Is the body ready for an IVF?",
"src": "Patient: Hi, My wife underwent IVF on 10th December 2009. ER( Egg Retrieval) was successful on that day .18 oocytes were recovered .but after two days pain started in her lower abdomin area and started increasing .The ART Specilist stopped the procedure of ET(Egg transplant) as ultrasound confirmed OHSS(Ovarian Hyper Stimulation Syndrom) .Ovaries size had increased (B/L Ovaries enlarged and cystic ( rt ovary - 8 cm, lt ovary - 7 cm, free fluid ++ve)) . She was hospitalised for 3 days.Specilist suggested Periodic Ultrasound . FET(Frozen Embryo Transplant) was planned after 2 months.After two montys no ET was done, as the thawed embryos were not of good quality as per the specialist (Embryos frozen : 4 (Grade 1A/2B)) . Medicine Suggested presentlly is Metformin- 1500mg, folic acid, colsprin - 75 mg. (These continued through out). Second cycle of IVF suggested from next day 2 . Some details: 1. Female Age : 31 2. Height : 173cm 3. Weight : 70 4. Reason of Infertility : PCOD (Polycystic Ovarian Disease), 5. Have you had any pelvic surgery? yes 6. Surgery Name : Adhesiolysis( Ovarian Drilling Done on both sides) (Year : 2006) 7. Findings : Both ovaries polycystic, fallopian tubes open 8. Medical problems and current medications of female partner: Suspected uterine Tuberculosis, had ATT (AKT - 2 / 4)medicines for 6 months. 9. In vitro fertilization (IVF) : yes Date : 10 Dec 2009 Stimulation Protocol : D21 onwards : Lupride 1/2 cc (14/11/09 to 8/12/09 (23 inj)), GonalF 225 ml (1/12/09 to 4/12/09 (4inj)), HCG ( 8/12/09 at 1 o clock at night) Dose of Hmg :300 Days of Hmg :D5,6,7 (5/12/09 to 7/12/09) Follicles grown :18-20 Eggs collected :15 (10/12/2009) Embryos formed :4 Embryos transferred :0 Embryos frozen :4 (Grade 1A/2B) My Question : 1. Is it possible that OHSS may reoccur? Spl suggests that if procedure starts on D2, probability reduces by 20%. 2. Is the body ready for 2nd IVF so soon? 3. I want to reduce chances of OHSS. Please suggest how to go about it. Doctor: higreetings.In PCOS there is always a chance of OHSS ,however drs can take precautions in preventing it by modifying the hormones used.but difficult to predict that it won't happen.since she has taken antitubercular treatment was hystroscopy done to see inside uterus? if so is it normal.you have to decide on that before IVF.please provide these details privately .can suggest more.regards"
},
{
"id": 185021,
"tgt": "Suggest treatment for a lump near the tooth in a child",
"src": "Patient: My 7 yr old son had a bad toothache, went to Dentist, dx with a cavity and was put on antibiotics. Several days after antibiotics, he was playing and jumping and he fell and his knee came up and hit him on the same side as the bad tooth (upper) Now it has a pea size lump there. What to do?? Been there about 4-5 days now. He says does not hurt now. Doctor: Hello!Thank you for posting here.This can be a abscess formed there.A x-ray must be taken to assess if the tooth can be saved with a root canal.Extraction of the tooth or a root canal may be necessary.I suggest you to see your dentist immediately to prevent any further spread of the infection.Excessive usage of antibiotics is also not recommended.Antibiotics only subside the infection.A complete treatment is necessary to eliminate infection.Also, the age and exfoliation of the concerned tooth must be considered.A clinical and radiological examination is necessary.Regards."
},
{
"id": 5068,
"tgt": "Delayed period followed by light brown to pink spotting, severe back pain. Pregnancy test negative. Possible pregnancy?",
"src": "Patient: Today is 5 days late with my period. I took a test and it said negative, which was in the afternoon. Shortly after, I began to spot. It is light brown to sometimes light pink. I am having excruciating back pain for the past 5 days also. My period is like clockwork so I'm wondering if I could possibly be pregnant?.... Doctor: Hello, I would be happy to help you with your question. It is always possible for a test to be falsely negative, so a confirmation blood test is indicated in your case. In all likelihood, you are not pregnant, and this could represent a persistent ovarian cyst. It would be best to get into your doctor as soon as possible for a blood test and an ultrasound. I hope that this helps and good luck!"
},
{
"id": 104219,
"tgt": "Taking high to medium doses of steroids, prednisone, rash across stomach, buttocks, upper thighs. Is there a cause for this?",
"src": "Patient: I have been taking high to medium doses of steroids for hives over a four month period in conjunction with antihistamines. I am currently taking 15mg of prednisone per day and 100mg of atarax. I am trying to ween off the prednisone for the fifth or sixth time. Everytime I do I develop a rash mainly across my stomach, buttocks and upper thighs. Is there a cause for this? I have pretty much every side effect from the prednisone, eye problems, weight gain (30 pounds), buffalo hump, etc. Should I be tested for Cushings Syndrome? Will these side effects go away after treatment if I can ever stop the steroids?Thank you. Doctor: Hi, thanks for posting to H.C.M.C. The main problems are hives for which you are taking tt. No doubt prednisolone is a best remedy for urticaria ( hives) but only in emergency that too for one or two days ( even more days, if condition). The side effects of this drugs are intolerable & very dangerous. Buffalo hump, eye problems , swelling over face , gain of weight all are related to pred. Your's outlook resembles like a patient of Cushing's Syndrome , but you haven't this disease because your's symptoms are due to pred's side effects. You need not to be checked for Cushing's Syndrome. Now \"Q\" is treatment of hives , get a blood examination , stool examination, urine . If eosinophils are if high take tt accrdingy. If worm infestation tt accordingly. If urine inf accordingly. You can change antihistaminic medicine also, more potent. I think you will get benefits from these things. Good luck . Dr HET"
},
{
"id": 224941,
"tgt": "Why did I experience cramps after consuming jadelle contraceptive after having unprotected intercourse?",
"src": "Patient: I am on the jadelle contraceptive. I had unprotected intercourse in January, (after having the jadelle in 3 weeks) then got my period 2 days later. I did not have intercourse for a month, then got cramps when I was due to have my next period, however it never came. I had unprotected sex after this, however he did not cum inside, then a condom was used. Since then I have had severe cramps twice, and I thought I was getting my period but haven't. it is about 20 days late and I am beginning to panic that I am pregnant :/ could I be? If I did get pregnant in January then I would be almost 3 months and would show other signs? Please help :( Doctor: Hi and thank you so much for this query.It would be highly unlikely that you are pregnant. It is not uncommon to have missed periods while on these pills. however, it would all sort itself as the months get by. If it should still prove difficult even after 3months of continued use, then I will suggest that you get evaluated for any other possible causes. This is for sure not pregnancy considering all the information you have provided. Stay calm and relaxed.I hope this helps. I wish you well. Thanks for using our services and do feel free to ask for more information or clarification if need be.Dr. Ditah, MD."
},
{
"id": 99057,
"tgt": "Suggest remedy for asthma and gastritis",
"src": "Patient: Hello sir I am from india I am suffering from gastritis and asthma problem what is best remedy for me because if i take medicine for asthma then it effect my stomach and after some days problem increase and if i take medicine for gastritis then asthma problem increase Doctor: HI, thanks for using healthcare magicGastritis is inflammation/irritation of the stomach. You may need to change the medication that you are using for it if it worsens your asthma.Also monitor your diet to see if there are any particular foods that may be upsetting your stomach.Asthma control is very important so your doctor may also need to look at the best medications that you would be able to tolerate.I hope this helps"
},
{
"id": 188610,
"tgt": "Burning sensation after doing root canal. Taking medicines for acid reflex. Remedy?",
"src": "Patient: I got this burning feeling a while back after getting a root canal the dentist said it would go away it did but took a long time.Now it is back, my gp gave me meds for acid reflux and told me to see a dentist, I have neighborhood health ins and no dental ins and can,t get an appointment with a dentist without paying cash and everywhere I call has a 3wk waiting time and want 125.00 just. for visit.I will pay what ever I have to,to get rid of this feeling now not in 3wks.Please help me out here,this is making me feel very down and I am not a depressed person. Doctor: Dear friend.Thanks for sharing your concern.Burning sensation could be due to vitamin deficiency,any allergy to particular food stuff or chemicals that you are using.It can also be due to gastritis and acid reflux.Have you seen your gastroenterologist before?Are you taking any medications presently?please reply to suggest you towards better treatment.Meanwhile please start taking vitamin B COMPLEX for a month.Hope it helps.Thanks.Take care."
},
{
"id": 226177,
"tgt": "Had depo shot. Heavy bleeding. Tried premarin, vitamins, maca root pills. Not helping. Switch to pills, stop bleeding?",
"src": "Patient: Hi, I have only used the depo shot two time. My first time was nov15 2012 and for the first three months i really didnt bleed. my second shot was jan 31 2013 and a month after that about march 8 i was spotting and now apr 23 it is heavy bleeding and no signs of stopping. i tried premarin for 3 days and slowed down but started back. I also take vietamins and maca root pills.I suposed to have gotten a shot on apr 18 but decided to do birth control pills to help with the bleeding. Will this stop all of this bleeding? Doctor: Hi, Thanks for the query. Yes, birth control pills can help in stopping the bleeding. Better to take two pills a day with your doctor's advice to stop the bleeding. Along with the pills you go for anti-fibinolytics initially. Once the bleeding stops you can continue one tablet a day for the purpose of contraception. For more details you can ask me through: http://www.healthcaremagic.com/doctors/dr-sree-gouri-sr/63429 Take care."
},
{
"id": 116193,
"tgt": "What causes elevated CA in the blood test?",
"src": "Patient: During an annual exam my CA 125 came back at 60. I have the test because I was adopted and have no family history. My blood sugars were also elevatedt his year. I've had an MRI, Ultrasound, CT Scan, Lung Xray, Mammogram and colonoscopy and all are normal. What are potential causes for the elevated CA 125? Doctor: Hello,I understand your concern.I am Dr. Arun Tank, infectious diseases specialist, answering your query.In my opinion it is raising suspicion over the ovarian cancer.CA 125 or carbohydrate antigen 125 is specific for the ovarian cancer. Its value increases whenever patient has ovarian cancer.I suspect it can be a small tumor so you have not detected in radiography.Constant check over marker and radiography will help you for good management. For good prognosis make look over the cancer.I will be happy to answer your further concern, you can ask me on bit.ly/DrArun. Thank you.Dr Arun TankInfectious diseases specialist."
},
{
"id": 175794,
"tgt": "Suggest treatment for constipation in children",
"src": "Patient: My 4year old is on a full cap of miralax twice a day as directed by his doctor and it s been a week and he s still struggling to poop, is there something else I can do to help his poop come out? It is causing him to have a horrible rash cause all he is doing is leaking a little throughout the day. Doctor: Hi...Thank you for consulting in Health Care magic. I think your kid is having habitual constipation. I have certain questions and suggestions for you.Questions:1. Did your kid pass motion or meconium on day one of life?2. Since how long is the kid constipated?3. Does the kid have any bleeding along with hard stools?4. How much milk does the kid consume per day?5. Does the kid eat fruits and vegetables (fibre diet) appropriately?Suggestions:1. Natural methods are the best to relieve constipation.2. Constipation is a risk factor for UTI3. Maximum milk consumption per day should not exceed 300-400ml4. Minimum 3-4 cups of fruits and vegetables to be consumed per day5. Toilet training - that is - sitting in Indian type of lavatory daily at the same time will help a lot.Kindly get back to me with answers to the above questions.You can approach me at the following link. Please find the link below.www.healthcaremagic.com/doctors/dr-sumanth-amperayani/67696"
},
{
"id": 11010,
"tgt": "Suggest solution for excessive hair fall problem",
"src": "Patient: i am 19 years old female and suffering from excessive hairfall problem ...please suggest me some good doctor who can help me to prevent it at affordable rates ..kindly also inform me about skin specialists at aiims,delhi who can provide medicine to prevent this problem Doctor: Hello, as you are suffering from excessive hair loss and havenot mentioned the area of the scalp from which it is occuring, i assume it is diffuse and not from a localised area..The most common diffuse hair loss is a condition known as telogen effluvium. I would adv you to get a serum ferritin level and thyroid function tests, to rule out common treatable causes of diffuse hair loss (iron deficiency and thyroid disorders respectively). If these are normal, start on hair supplements like tab Follihair 1 tab daily for atleast 3 months and apply Proanagen lotion on the dry scalp daily. Also, for washing scalp, use Xgain shampoo thrice weekly and use a good conditioner for your hair (apply conditioner on wet hair, leave for 10 min and then, rinse with water).With this, your hairloss will improve, have patience, in case you notice any other complaints, then visit a dermatologist for evaluation. At AIIMS, there are many dermatologists viz. Dr.Ramam to whom you may consult.ThanksTake care"
},
{
"id": 27967,
"tgt": "Could calcium & lysine supplements aid blocked coronary artery?",
"src": "Patient: I have 1 completely blocked coronary artery. Calcium deposit in the artery is too hard that roto blast could not penetrate and bypass surgery is too risky as I already have had that twice. I am medication (calcium channel blocker to relieve me from angina. I am trying other means to help my condition. My question is that: would high dose of calcium and lysine supplements help. Art Doctor: Well there is no role of calcium or lysine dose increment for blocked arteries.I see your case suitable for a procedure called EECP ,which will relieve your symptoms ."
},
{
"id": 216987,
"tgt": "Suggest treatment for mild dull ache in left leg",
"src": "Patient: I have a continuous throbbing in my shin area of my left leg. Is not painful. Went to the doctor who suggested Magnesium 2 - 3 times per day. I have been taking magnesium but is not helping. I have only been taking twice a day. Do you think the 3rd dose may do the trick? or can you suggest something else I might try? Doctor: Thanks for keeping trust with us. ..as per your history u r taking magnesium twice a day to controll symptoms. ..In my opinion more than that can increase serem level of magnesium that can create additional problems ....so better to consult your physcion before you increase the doses. ...at present to releve symptoms u can a start a calcium supplement and a analgesic like tab tradmadol 50 mg sos basis. .."
},
{
"id": 196634,
"tgt": "Could the Depo testosterone cause burning in nipples and increase in size?",
"src": "Patient: I m a 50 year old male. My doctor has me on Depo Testosteone 200 mg every two and half weeks. It has really made me feel better, almost life changing. I don t want him to lower my dose. I think I have started to develope what they call bicth tit. My neeples burn and itch and they are getting larger. I order mamofen 20 mg and now I m scared to take it. Your advice would be helpful. Doctor: Hi,Yes it can occur as a side effect of testosterone as sometimes testosterone get converted into estrogen due to activity of an enzyme which leads to these side effects. Hope it helps. If you have any other question please do not hesitate to contact us.Regards,Dr. Atishay Bukharia"
},
{
"id": 45449,
"tgt": "Is semen viscosity curable?",
"src": "Patient: Is semen viscosity curable? Doctor: it is not natural and it is not a disease to cure."
},
{
"id": 54375,
"tgt": "Suggest risk with high SGPT levles",
"src": "Patient: Hi,I just done a LFT ,Total bilurubin is 1.2,direct bilirubin is .24 ,sgot ->40,sgpt->110,alkaline phosphates->150,total prtein ->6.4,albumin->4.5,globulin->1.9 and A/G ratio is 2.37. I can see that my sgpt is high. Do i need to consult a doctor .Is this values alarming ? Doctor: thank you for posting query.yes, you should visit your treating physician as your suffering from acute hepatitis. usually the cause is not worrysome but better to exclude certain causes of hepatitis.you need to be tested for HBsAg, Anti HCV and if possible anti HAV.after the results of these test, further management maybe adviced.HAV is a common cause of acute hepatitis and self limited.meanwhile:- abstinence from Alcohol - LOW fat diet should be followed- NO red meat- vegetables should be ingested daily- REST more, Less activities- use lemon juice (lemonade) once in a day- Liv52 maybe used after visiting a hepatologist and taking his advice-\"recheck liver enzymes after 4 to 6 weeks\".if liver enzymes still high, You may visit hepatologist as soon as possible.if any further questions, feel free to ask.Health professionals aim to diagnose properly and manage patients according to their limited knowledge. Cure is blessed by the ONE who Created us, whose power and knowledge is unlimited .wish you good health.regards,Dr Tayyab Malik"
},
{
"id": 191566,
"tgt": "What causes blurred vision in a diabetic?",
"src": "Patient: Earlier today upon going outside my vision was very blurry, I was slightly confused periodically when talking and sometimes I saw double. I m a diabetic and My sugar is 226 which is low for me being that it s usually always high. Now when coming in my house I feel okay what may be my problem. Doctor: Hi,Thank you for asking your question on HealthcareMagic.I have gone through your query. Considering you are a long-term diabetic, it is very important to take care of other body parts which diabetes affect, commonly being heart, eyes, kidney and nerves. Blurred vision in a diabetic person can be a sign of Diabetic retinopathy and should be evaluated at the earliest. I would suggest you visit your ophthalmologist. He will be checking your eyes under a slit microscope to see any signs of Diabetic retinopathy. Also, kindly get your sugar levels checked including HbA1C to see the diabetic control. All these levels must be within normal levels. Let me know in case of any further query or to show the reports.Best,Dr Vishesh"
},
{
"id": 148793,
"tgt": "Have Complex Regional Pain Syndrome(CRPS). Tried Gabbapenton, nerve blocks and others, no relief. How to treat this?",
"src": "Patient: im looking for info on treatments for (crps) complex regional pain syndrome in my left hand iv had ketamine infushions lidnocane nerve blocks pain pump and spinal cord stimulater been on lyrica and gabbapenton tried all the natural therapies physiology relaxation but it keeps getting worse iv had to have 1 finger and part of my hand amputated because of compartment syndrome caused by flare ups and im in so much pain im having problems just moving im gaining wight becausei cant do anything Doctor: Hi,I would like to advise that you must put your best bet on non pharmacologic measures for pain relief. Available options are prolotherapy, biofeedback, progressive muscular relaxation exercises (consult your doctor for these) in conjunction with meds like Gabapentin (you have tried), Pregabalin etc. You must engage in at least some physical activity as weight gain can further worsen your condition & thus pain. Good Luck"
},
{
"id": 40578,
"tgt": "What causes a missed period after getting a Menogon injection?",
"src": "Patient: Hi I had taken inj menogon every nine days from 7th day of my cycle and dr said me your egg only two so we are not going for ivf but my period is not coming till date 14th and my last lmp date is 16th Sept but always my period came before 4 days of last lmp can I m pregnant Doctor: Hello,Injection Menogon is used in the treatment of infertility. It has the combination of 2 hormones and it helps in egg formation.If you have undergone follicle study after Menogon injection, then you can expect your next menses 14 days after the date of ovulation.If you still don't get your periods, then please get morning's first sample of urine tested for pregnancy. In case of any doubt, blood beta HCG test and USG will be of help.Hope I have answered your query. Let me know if I can assist you further.Regards,Dr. Purushottam Neurgaonkar"
},
{
"id": 101434,
"tgt": "What causes red hives on the leg and arms with pain after bee stung?",
"src": "Patient: My 6 year old daughter was stung by a bee today ( I think it was a bee). I didn t actually see it. She broke out into hives on her leg and arm. I gave her benadryl immediately. The itching went away but the area is still red and tender to touch and now she is complaining of severe pain when walking. Should I take her to the urgent care for additional treatment or is this normal. She has no known allergies to bees, only food allergies. Doctor: Hi, thanks for using healthcare magicThe pain would be due to the inflammation from the bee sting. If you use an anti inflammatory pain killer such as ibuprofen (Advil for children) , then it may help ( provided she has not history of reactions to this medication).She should also be given another dose of the benadryl when the appropriate dosing time has passed.I hope this helps"
},
{
"id": 102166,
"tgt": "How to treat allergic symptoms caused by fire in the fireplace?",
"src": "Patient: This weekend, I had a fire in the fireplace, since then me and my daughter have had allergy symptoms (congestion watery eyes, clogged sinuses, how long will these symptoms continue to persist, and is there a way to use the fireplace without having these reaction. Thanks Vic Doctor: Hello,Welcome to HCM,The symptoms are suggestive of some allergic reactions following exposure to triggering factor like ashes created by the fire.These allergens can stimulate the mast cells to release the mediators of allergy like histamine, which will produce all these symptoms. If you are able to find out the cause for your symptoms it will be easy to avoid the triggering factor as for as possible.To control these symptoms, I would suggest you to take Tab Montek LC, for 3 days which will reduces the symptoms but it is not going to cure the allergy completely.Thank you."
},
{
"id": 208854,
"tgt": "What are the signs of stress?",
"src": "Patient: All this stuff is happening to me at one time I ve been really stressed and im not sure if my health is failing from it. I m nine days late for my period and I have taken 3 HPTs but they were all negative. I was also coughing up blood after crying and I have red spots in my eyes like my eyes are bleeding. Could this all be from stress? Doctor: DearWe understand your concernsI went through your details. I suggest you not to worry much. I sincerely do not think that the symptoms are due to stress. Please consult a general medicine practitioner and proper diagnosis. Self diagnosis and self medication can be dangerous.If you require more of my help in this aspect, Please post a direct question to me in this website. Make sure that you include every minute details possible. I shall prescribe the needed psychotherapy techniques which should help you cure your condition further.Hope this answers your query. Available for further clarifications.Good luck."
},
{
"id": 132528,
"tgt": "What led to top of my hand being sore and stiff?",
"src": "Patient: Hi, this morning I woke up with the top of my hand being sore and stiff. I thought maybe I slept on it wrong. It didn t get any better and now it is swollen and warm so I m thinking that maybe I was bitten by a spider. I don t see any bite marks. Is there some way to treat this? Doctor: Hello. thank you for writing us here.it could be because of insect bite or maybe you hit yourself at night.You can put ice on it and take ibuprofin for getting relief from pain.there's nothing to worry about okay? it would be alright in 5 to 6 days. best regards,Dr Gunjan"
},
{
"id": 185665,
"tgt": "How to prevent the wound in the gums from getting infected?",
"src": "Patient: Hi, I have a wishdom tooth coming down and it is cutting into my gums and causing it to bleed and swell. I also can't open my mouth wide as it is stretching my inflammed gums. I am going to see a dentist about it, however I can't be seen for a few days, is there anything I can do to stop my open-wounded gums getting infected? Doctor: Hello,Please keep up with your dental home care. Rinsing, gentle brushing and flossing to reduce plaque buildup is important. Warm salt water rinses can cleanse the area. Make sure nothing gets lodged in the gum area. Do not chew on that side and consider a softer diet. keep hydrated and take vitamins. Anti-inflammatory medicine such as acetaminophen can offer relief. If pain or swelling increases this indicates the spread of an infection and antibiotics may be prescribed by your dentist. It is then more urgent and you should contact your dentist to be seen as soon as possible and be placed on an antibiotic. I hope this advice is useful and that you will have relief soon."
},
{
"id": 96330,
"tgt": "I havent had a poo in a week",
"src": "Patient: i havent had a poo in a week im desperate but when i try to go its painful and gets stuck ... Doctor: Tab. dulcolax 2 tab at night for week or tab. herbolax 2 tab at night"
},
{
"id": 173946,
"tgt": "Does swelling of hand after an injury require an X-Ray?",
"src": "Patient: My child hit his hand against coffee table, cried for a bit and then afterwards I noticed his hand was swollen. He didn't seem to be in too much discomfort and move fingers and bend wrist without much complaint. Put a cold compress on it but should I take him in for X-rays? Doctor: Hi,As there is no pain and easy movement at hands, fracture is less likely.There might be having small hematoma on the part producing swelling.Apply ice pack 2-3 times a day for 2-3 days.Gradually swelling will be subside.Ok and take care."
},
{
"id": 113832,
"tgt": "Extreme back pain in the mid back, treatment options ?",
"src": "Patient: I am suffering from extreme pain in mid back, taking %0mg fentenyl patch , 8 oxycocet, 2 10mg oxy ir and 15 mg statex everyday sometimes more. The pain intensified to more than ever before yesterday, what can I do. I have also had blood work done due to calf pain, muscle enzynes very high. I can no longer due anything due to pain. I am 56 year old female. Please help, sometimes I feel I would be better off dead. Doctor: Hello there, Welcome to HealthCare Magic Forum. For the pain in back you may take following Homeopathic medicines which are capable to reduce severe muscular pain in back. These medicines are safe and can be used for a longer duration without any side effects:- 1)\u00a0\u00a0\u00a0\u00a0\u00a0RHUS TOX 200 Three times in a day. 2)\u00a0\u00a0\u00a0\u00a0\u00a0ARNICA MONTANA 200 One dose every day at bed time. 3)\u00a0\u00a0\u00a0\u00a0\u00a0MAGNESIA PHOS 6X Two times a day. You need not to stop medicines you are taking. Only you have to make about half hour gap between these medicines. If you are suffering from constipation, use any mild laxative. Do light exercise or the exercise advised by your Doctor. Get well soon."
},
{
"id": 141775,
"tgt": "What causes difficulty in talking post Acitrom withdrawal?",
"src": "Patient: My dad suffering problem of blood clotg due to which his blood circulation of brain was reduced and he unable to talk . so Dr suggested acitrom 4 mg . After inr report came normal he suggested us to use ward 3 mg but now again he find difficulty to talk. Plz suggest Doctor: Hello,It seems from your message that your father has had a stroke. Therefore, there may have been the need for Acetrom drug. However, this medicine can cause bleeding and so INR needs to be maintained.There are 2 possibilities in view of the difficulty in speaking:1. He has had a minor stroke again- this is usually quite common in stroke patients (the development of the 2nd stroke).2. There is edema developing in the brain that is leading to the recurrence of the symptoms. There remains a possibility, however this I have not considered chiefly because you have not mentioned any suggestive symptoms, that he may have a metabolic/infectious cause going on which may be leading to the recurrence of the symptom.Having said all that, I don't think it is related directly to the dose of Acetrom- if the doctor had increased the dose, I would have considered a risk for hemorrhage. But in this situation, it does not seem to be likely.I would suggest admission, and testing with a repeat scan and blood tests. Continued speech therapy may help him.Hope I have answered your query. Let me know if I can assist you further.Regards,Dr. Saumya Mittal"
},
{
"id": 33462,
"tgt": "Suggest treatment for dog bite",
"src": "Patient: Hellow, I have bitten by a street dog.Very smll scratch was noticed , As advised by the doctor I have taken 1st injection of 5 injections of Rabivax. Now the same is not available at Hyderabad and 'Rabicure' is available .Can I take remaining 4 doses of Rabupur? Doctor: HI, thanks for using healthcare magicThese are 2 different vaccines but they compare in terms of effectiveness.You should be able to complete the course with the rabipur vaccine.You should also receive tetanus vaccination as well.I hope this helps"
},
{
"id": 99806,
"tgt": "What causes pressure in head/neck/nose and around forehead?",
"src": "Patient: Hi, I am getting weird feelings in my head. Some kind of pressure in head , back of neck and nose. This is something that comes and go (or is bearable at least for some time), but when it is there I feel like there is constanly some pressure around my forehead, eyes, sometime at the back aswell and in my neck. Doctor: Hi, What u said its heaviness in head in medical terms.There can be many reasons behind that like tension headache, middle ear infection, hypertension, viral infection etc but the main thing is to be diagnosed first.In many cases no diagnosis made then one is prescribed symptomatic treatment after routine clinical examination. Consult your physician first."
},
{
"id": 9874,
"tgt": "How to treat chronic alopecia universalis?",
"src": "Patient: My son is 9 and has had Alopecia universis for 3 years. It started with spots and elevated to 90% hair loss. He is vegan and have consulted 2 doctors. We have changed diet to remove most processed foods and 5 months ago removed gluten. his hair began to regrow and then stopped and now is thinning again. What is the next step? Doctor: Hi,It is alopecia areata... not alopecia universalis, as the the some hairs are there on the scalp. Consult the expert dermatologist for the perfect diagnosis and proper treatment. I would recommend intradermal inj triamcinolone 10 mg/ml at various spots on alopecia patch,trioxsalen lotion application and tab biot 10 mg per day ,..etcAfter few months of treatment, I hope he may get good result.Thanks.Dr.Ilyas Patel,Dermatologist"
},
{
"id": 95853,
"tgt": "What treatment should i go for ovarian cyst ?",
"src": "Patient: i m on my period and i put a tampon in and layed back down and my ovaries hurt really bad now! i m 18 years old and a female, obviously. could this maybe be an ovarian cyst? Doctor: Welcome to Healthcare Magic Good Day If you have severe pain while inserting a tampon it could be due to a vaginal and/pr cervical infection. Are you having any fever. Is there any whitish or yellowish discharge from your vagina. Are you sexually active. Are you bleeding heavily. You need to show to your Gynaecologist as soon as you can for proper diagnosis and management of the pain and other problems."
},
{
"id": 94768,
"tgt": "Lower stomach pain after periods. Reason?",
"src": "Patient: 1 am 37 year s my proplem is :: i am suffering from lower stomach pain after periods . pain starts with in one week after periods , it occurs mainly in left side but sometimes in right side and comes in my left or right side of that abdomin.please tell me the reason and diagnosis .i have visited several doctor but dosent help me Doctor: HELLO SUFIA, What i gather from the info is that you have got low abdominal pain mostly after the periods. I think you should do an ab domino pelvic ultrasound to rule out some gynecological pathology most commonly endometriosis. These are diseases and you need not have to worry much.bye."
},
{
"id": 104212,
"tgt": "Recurring burning in nose and eyes, throat dryness after taking Zpack treatment. Allergy?",
"src": "Patient: I have a burning sensation in my nose as well as my eyes and my throat hurts and feels very dry. I experienced this about a month and my doctor gave me a z pack ( Azithromycin ). I have felt great the past few weeks, but now it s back. Do you think this is just allergies? I am puzzled. I do have terrible allergies and take medication for my athsma whisabarbera1ch works great. Thank you! Doctor: yes allery symptomps no use of antibiotics oly to take anti allergics start montair fx(combination of montelucast 10mg and fexofenadine 120 mg apply neosporin h eye ointment in nose bd put sea water drops 2 drops at niht in each nostril chromal forte ey drops in eyes syp tossex 1/2 tsf night to protect throat can take for 2 week and off and on in recurence withut side effects"
},
{
"id": 206253,
"tgt": "How to get rid of homosexual feelings?",
"src": "Patient: Sir, i am a medical student from Pune. I am going through a big problem. Since i am 12, i've always had a feeling of sexual attraction towards the same sex. i used to think that as the age progresses i'll be normal. I am 21 now, but still there are no changes. Infact my body has shaped out in quiet a feminine pattern, onto it i've got a crush over one of my best friend of same sex. Now all these things make me feel so conjusted. I couldnt expect my parents to understand this. Many times i feel like getting my sex changed. But what about my parents then, they wont be able to cope up with it. Also the boy whom I am in crush with is straight and he doesnt feel anything like this. All these worries and excessive sexual desperation for that person, not only deprive me off the concentration in my studies but also I am unable to live happily!! Please help i'll really be thankful to you!! Warm reguards.. Doctor: DearWe understand your concernsI went through your details. I suggest you not to worry much. Sexual fantasies are common but they are against social values. You must understand that there is nothing wrong in feeling attracted sexually with the same sex. Everybody has to go through such a state in their life. But why you are worrying about it? Worrying and escapism makes that thought to be present always and makes you anxious and stressed, as you are now. Learn to ignore such thoughts. Accept what you are now and believe everything will change in future. You are just 21 and there could be another 5 years before your marriage. Things settle within 5 years. Psychotherapy techniques should suit your requirement. If you require more of my help in this aspect, Please post a direct question to me in this URL. http://goo.gl/aYW2pR. Make sure that you include every minute details possible. I shall prescribe the needed psychotherapy techniques.Hope this answers your query. Available for further clarifications.Good luck."
},
{
"id": 54613,
"tgt": "What causes a solid mass in liver and normal liver enzymes?",
"src": "Patient: I am 64 years old, they just did a Catscan on my liver and found a mass about 1.7 in. The liver enzeme test on my liver came back normal. If it is normal could or would that rule out cancer. Also, it is a solid mass and was wondering if it could be fatty liver or a cyst? Doctor: HiIf it's a solid mass then it isn't a cyst and cancer isn't ruled out by normal liver function tests. You need a PET scan of the liver and possibly a fine-needle biopsy for diagnosis"
},
{
"id": 64656,
"tgt": "Should I be worried for painful lump on chest?",
"src": "Patient: I have a lump on my chest, more or less central but a little to the left side, just above the nipple line. It is sore to the touch. It is kind of like a big pimple but a little bigger. Noticed it a few days ago. Is it something I should be worried about. About 1-2 months ago I had shingles, which seemed to heal pretty quick with medication. I am 42yr old male, overweight and smoker. Should I try to pop it? Thanks Doctor: Hi,Dear,Good afternoon.Thanks for your query to my HCM-virtual clinic.AFter indepth study of your query,In my opinion-lump on chest is -Furuncle with abscess.Treatment -I would advise -Tb-NSAIDs , with Antibiotics ,if it responds and reduces its OK.If it grows still-Incision and proper drainage would be needed.Hope this would relieve your worrysome query.Wellcome to HCM for more queries"
},
{
"id": 32730,
"tgt": "Suggest treatment for cold , cough and fatigue",
"src": "Patient: I came down with a cold last Thursday that included fatigue, chills, aches & pains, and a bad cough. I lost my voice two days ago as well which seems to be returning. Today is the following Thursday and all symptoms have subsided now except a deep cough. I have been taking an expectorant, but don t seem to have any major mucus coming up. My cough seems to be a deep, dry one. Am I on the verge of healing or should I have to seek medical treatment for antibiotics? Doctor: Hi. I understand your concern. The symptoms you had 1 week ago suggest allergic cold along with secondary infection( you had chill, aches, pains.. ) It seems with your resistance & natural subsidence of the problem has occurred.. still some apt of problem remains in for of dry/ deep cough.You are taking cough syrup.. without much effect. In this respect, you should get your blood TLC/DLC done.. to exclude infection in lungs & to exclude Eosinophilia ( raised number of eosinophils which are increased in response to allergy & in turn cause dry cough .. So such cough does not respond to cough syrup)According to result, you will need suitable antibiotic & Diethylcarbamazine pills in proper dosage. You should consult a physician & get specific advise. Nutritious diet & vitamin B complex supplements would take away your fatigue. Thanks,"
},
{
"id": 86553,
"tgt": "Suggest remedy for abdominal pain and swelling",
"src": "Patient: I havepain in my abdoman and feel swelling inside my right side below my rib cage. I have had bad problems with having a bowel movement. I can feel the need to go but only minimal amount comes out and it feels like something blocking up inside of anus, No Pain No blood just swollen abdomen and tender in right side below rib cage. Noises coming from gut like growling stomac. Have had growling since 2002 when I had gastric by pass. kept weight off all these years until problem with bowels. Have put on 22 lbs in last two years. Lost 128 I am 59. Doctor: Hi. Thanks for your question. You have a typical history of Gastric Bypass Surgery in 2002 and starting the growling since then.Now you have pain in right side of the abdomen and feeling of swelling under right rib cage. Feeling of blocking of something in the anus along with passage of small amount of feces. You also have borborygmi (noises in the abdomen), This indicated that the Colon on the affected side has some problems. This can be detected well by:CT scan of the abdomen and - Barium studies (Enteroclysis) and colonoscopy will diagnose the Problem and give perfect guideline for medical and / or surgical management. I hope this answer helps you."
},
{
"id": 3829,
"tgt": "Is conceiving possible while having irregular periods with history of TB?",
"src": "Patient: i am 28 yrs old, i got married before 6 year, i was infected by pulmonary tuberculosis before 6 years, i took treatment for 6 months after that i statred gainig weigt, my periods was irredulat, i increased nearlt 20 kgs after that, now my weight is 75 kgs, i trying to conceive, i took tsh, other blood test too, but everything is normal, do i have the chances to conceive Doctor: Hi, thank you for asking healthcare magicI understand your concernsNo need to worry so much. The chances of conceiving still exist with irregular periods. I don't know if you have been trying to conceive but to no avail. If that is the case, then I think both of you have to be assessed. Infertility within a couple can be linked to a man, woman or both. If everything is fine with you except irregular periods, then I would advise that both of you should see a doctor for assessment and counseling.I hope that helpsI wish you well"
},
{
"id": 225654,
"tgt": "On Lupin contraceptive with no problems. Have had light period that does not stop. No pain or clots",
"src": "Patient: I am a 49 year old, very healthy and fit woman. I have been on Lupin contraceptive tablets (Levonorgestrel and Ethinyl Estradiol & Ethinyl Estrdiol Tablets) for the last year, with no problems. However, for the last four weeks, I have had an ongoing menstrual period (extremely light) that does not seem to want to stop. There is no discomfort or pain, nor clots. I had two healthy, normal births (23 & 19 years ago), and never experienced any menstrual cycle problems or irregularities. I have made an appointment to see my Gyn, but it's not for several weeks. Grateful any advice. Doctor: Hi, the cause of the irregular periods may be anemia, harmones, infection etc. I advise you to take more of green leafy vegetables, pulses, sprouts, and proteine rich foods. I to my patient with such symptoms prescribe fluconazole, cifixime, iron preparations. Thank you."
},
{
"id": 173164,
"tgt": "What causes difficulty in breathing after food poisoning?",
"src": "Patient: My child had good posioning and threw up in the night but then started struggling to breathe. This has not happened often but has happened in the past when she has needs to vomit due to food aliment or illness. It scares me and I am not sure what to do Doctor: \"Struggling to breath is\" this is need urgent medical help. so I would advise you to take help of your nearest health providing facility . Difficulty in breathing after food poisoning can be occurs due to aspiration of content of vomiting . Any way may be other reason but it need URGENT MEDICAL HELP.Do not delay hope speedy recovery thanks"
},
{
"id": 64819,
"tgt": "What causes blue lump on leg due to hitting?",
"src": "Patient: My husband has a brain tumor. Healthy otherwise. Playing ball w the kids and hit his leg on her toe, a big blue lump appeared. He has has a blood clot 2 times so Im unsure if this is dangerous. Hubby will say no its fine. Trying to be safe. Surgery in 2 weeks Doctor: HI,Dear,Thanks for the query to HCM.1-I have studied your query in depth.2-DEar,IN my opinion, this blue lump on the leg ,is mostly due to the hematoma-traumatic cause in your husbands case .3-REcurring 2 blood clots is worrysome event and needs -investigation if-2 blood clots are creating consumption Coagulopathy-or the clotting factor defitiancy ,leading to the blue-leg hematoma ,needs to investigated.3-Hence I would advise prothrombin time and other clot tests 3-a- and team consultation of the brain surgeon and the hematologist doctor to treat our husband effectively.Dont plan hasty surgery in 2 wks for the saftey,this would risk your husband more and leave it to expert doctors.4-Hope this would solve your query to your satisfaction.5-Wellcome for more queries to me On HCM.6-Thnks once again for your inquisitive query.Have a Good Day..!!"
},
{
"id": 1806,
"tgt": "Having one D&C and taken unwanted 72 pill 3 times will I be able to conceive?",
"src": "Patient: hello i have 2 yr baby i was took unwanted 72 pill 3 time in this duration first time when my baby has been 9month then after 3or 4 month but oct 10 2014 i was conceived but after ultra sound report baby had no hart beat 10 jan 2014 i had got DNC .But march 13 was my period date but in between i pill took unwanted 72 so i want ask u its harmful if i got also DNC in future will i conceive again without face this type of problem. Doctor: Hi, there is generally no problem in conceiving with one d n c. Unwanted 72 also don5 have effect on future pregnancy. So, if you have regular periods and no other problems like less periods flow, pain during menses or any pain and discharge per vagina, there is nothing to worry about. Hope I have answered your question. if you have any other query I will be happy to help. Regards Dr khushboo"
},
{
"id": 169363,
"tgt": "What causes constant hunger in a child?",
"src": "Patient: My 5 year old daughter says she is hungry constantly. As soon as well leave the restaurant or get up from the table she wants a snack. I am afraid of her becoming obese some day. Is this constant hunger normal? She is 4 foot tall and weighs about 65 lbs. She is solid. Doctor: Dear, Welcome to HCMWe understand your concernsI went through your details. When children of 5 year old are always hungry, it means they are deprived of some nutrients. You must be able to identify which food the child is always craving for? Is there any association or patterns etc. Identify the food and include that food in everyday meal. Second reason could be, that they are making food association. TV and food, video game and food are great association they make. When they sit there, they ask for food.Some children require more fat, because of the child's physical requirement. Such a child will be thin or skinny. Sometime you fail to provide those foods they enjoy. They eat less and then ask for more. You can easily identify the child's habit and pinpoint the problem.If you require more of my help in this aspect, please use this URL. http://goo.gl/aYW2pR. Make sure that you include every minute detail possible. Hope this answers your query. Further clarifications are welcome.Good luck. Take care."
},
{
"id": 82154,
"tgt": "What causes chest and ear pain with syncope and sweating?",
"src": "Patient: I had pain while lying in bed .It was around the heart but no pain in the centre. Also in the ears but not in the neck or arms. I spayed nitro glycerine spray twice and waited ten minutes and sprayed twice again. Still same pain. I moved slowly to get up to prepare to call the ambulance and I felt very faint and was perspiring. I lay down again and waited and the pain left after a while and I felt better. I had not been able to sleep that night and it was 5 a.m I have had pressure in the centre of my chest in the past and have had an echocardiogram last October and it was normal.I have high blood pressure and on medication. Thanks for your reply Mary Doctor: Thanks for your question on HCM.In my opinion you should immediately get done ECG and 2D Echo again. As your symptoms like early morning chest pressure, perspiration and high blood pressure are suggestive of cardiac disease only most probably angina. So consult cardiologist immediately and get done ECG, 2D ECHO and control of blood pressure. If above things are normal than better to get done coronary angiography to rule out coronary blockage."
},
{
"id": 115091,
"tgt": "Suggest treatment for haemochromatosis and anaemia",
"src": "Patient: I have no stomach and have hemocromatois,I am tired all the time. My D 17 is very low. My bone marrow is not making mature red cells , my iron binding capac 195,wbc 4.17,rbc 3.60, hct 34,hgb 11,ferritin 98, iron 79,% iron saturation 41. They have done a phlebotomy on me but it made me more anemic. They want to do another blood test and a possible phlebotomy on me the end of july. There has to be something I can do . Doctor: Hi, dearI have gone through your question. I can understand your concern.You should confirm the diagnosis of hemochromatosis. If you have not gone through liver biopsy then go for it. No need to do phlebotomy with anemia. consult your doctor and take treatment of anemia.Hope I have answered your question, if you have any doubts then contact me at bit.ly/Drsanghvihardik, I will be happy to answer you.Thanks for using health care magic.Wish you a very good health."
},
{
"id": 126625,
"tgt": "What causes burning sensation in between the fingers?",
"src": "Patient: I have an odd burning sensation/pain that occurs between my index finger and middle finger near where they join the main part of my hand. There is nothing on the skin surface, but occasionally when something pulls on this are, it feels like it was burned under the skin. What could cause this? Doctor: Hi, This area that supply between index and middle finger extensor surface is supplied by superficial branch of radial nerve. In case of any compression at radial ridge or neuropthy or radiculopathy pain or burning sensation will be there. local Infection or scabies may can cause this problem. Use tablet Pregabalin once daily for five days. Use Vitamin B complex tablets once a day one tablet for one week. If symptoms not improved please consult your neurophysician/orthopedician they will examine and treat you accordingly. Hope I have answered your query. Let me know if I can assist you further. Regards, Dr. Penchila Prasad Kandikattu, Internal Medicine Specialist"
},
{
"id": 153823,
"tgt": "What are the chances of recovery from stomach cancer?",
"src": "Patient: Dear Sir, My dad is 63 yrs old and from past 15-20 days he lost weight of about 4-5 kgs and facing problems like indigetion and not feeling hungry, tigedness. consulting doctor suggested to get admitted to hospital and sucpecting a Stomach Cancer. my worrey is the doctor also told like there is no cure for this and usally hospitals will do some surgery just for the money sacke.. i am really worried please help me suggesting good hospital for this desise and also is there any hop that my dad will recover Doctor: Hi, dearI have gone through your question. I can understand your concern. Your father may have stomach cancer. Treatment depends on type and stage of cancer. In early stage surgery will be helpful. In later stage surgery will not helpful and treatment options are chemotherapy and radiotherapy. These treatment is obviously very costly. You can go to government hospital where this treatment is done freely. In late stage prognosis is very poor. Consult your doctor and plan accordingly. Hope I have answered your question, if you have doubt then I will be happy to answer. Thanks for using health care magic. Wish you a very good health."
},
{
"id": 20880,
"tgt": "What causes sudden rise in heart beat with dizziness and sweating?",
"src": "Patient: hi, im 29 yrs old and today i experience something that i never had, my hrt started beating really fast, i got dizzy, and sweating. I tried to sit down and take a deep breath but did not work. It lasted about 20 mins, this happened after a cup of coffee(which i have everyday)...should i see a dr asap and can it be that im anemic. Doctor: Hello, This can be a cardiac arrhythmia like psvt or atrial fibrillation, or just rapid heart rate both precipitated by the coffee, in which heart rate suddenly goes up and also suddenly decreases. Blood pressure falls during these episodes and one has dizziness, fainting, blackouts, sometimes syncope. Usually ecg is normal as it is usually done in between the episodes. ECG done during episodes will show abnormality. You will need 24 hour holter monitoring. So visit nearby cardiologist for these investigation. Avoid coffee and other stimulant if any.Also, hemoglobin and thyroid test should be done. ln order to abort such episodes you should try to cough hard.Is there any stress or anxiety associated. Till the time your evaluation is complete, you can ask your local doctor for tab ciplar LA 20 mg once a day. If none of the above is successful and episodes recurs then last option is of electrophysiological studies which is invasive test which can diagnose and treat the arrhythmia permanently in most of the cases."
},
{
"id": 164767,
"tgt": "Does persistent cough and congestion after treatment for pnuemonia need medical attention?",
"src": "Patient: My 2 year old was diagnoised with pneumonia last Thursday. He got a shot of rociphine in the office and 7 days of antibotic. Today is day 4 and he is still running slight fever at night and his cough is terrible. I have listened to his breathing with stethescope and can still hear a wheezing sound. Should I take him back to the dr.? Doctor: hi, cough and breathing problem should not purchased after 7 days of antibiotic. If it is persisting then we have to look for the cause of cough and breathing problem. Kindly do a x-ray chest. An examination by doctor should be done so that we may not miss any important finding. Take care."
},
{
"id": 27299,
"tgt": "Suggest medication for slow functioning of heart as per stress test",
"src": "Patient: I have a history of orthostatic hypotension, shortness of breath, and dizziness. I had a echo stress test done yesterday. I was told my heart was lazy and very slow. He said my normal doctor is out of town and will get back with me next week. He said he is not familiar with my history but it looks like I could be in the beginning stages of heart failure. I am a very active 45 female. I am not over weight, do yoga 4 times a week and eat well. What could this mean and what will be the next steps? Doctor: If you are doing yoga and deep breathing exercise, please add one herbal support to your treatment.My three decades standing in Ayurveda says that you should add ARJUNA i.e. Terminalia Arjuna powder. You can get capsule of Arjuna from Himalaya pure herb series 1 bid for 8 weeks and assess yourself. Purely SAFE. No side effects. It is known to increase the efficiency of heart. Cardiac Asthma and hypotension are also widely treated by it. If you can, please additionally take Liquid Arjunarishta 15 ml bid after meals by adding water to it.If do not feel benefit, you may stop it. But I know you won't stop it.No side effects, no counteraction with other medicine. Do it now please.Hope it helps you. Yes it will. Wishes."
},
{
"id": 10225,
"tgt": "How many days I have to use mx 2% to stop falling hair?",
"src": "Patient: hi,siri am facing hairfall problem since 3 months. i had consulted a doctor she advised to use mx solution usp 2% i want treatment only for hair fall .. not for hair regrowth because i am interesting to use \"hair growth injections\" for hair growth. doctor said that he will give injection only after stopping hair fall. I want to know how many days i have to use mx 2% to stop falling hair. Doctor: Hello and Welcome to \u2018Ask A Doctor\u2019 service. I have reviewed your query and here is my advice. Minoxidil should be applied life long. It will take a minimum of 6 months for initial improvement and again hair falls if you stop medication. Please do continue Minoxidil. Hope I have answered your query. Let me know if I can assist you further."
},
{
"id": 131474,
"tgt": "What causes painful swollen knee?",
"src": "Patient: i sat all afternoon watching the PHOENIX OPEN Got up to leave stood up, turned to walk to the left, at the very first step I KNEW something was wrong. to right of my knee cap and down a little ---PAIN. some swelling no bruising can bend it, can straighten it but hurts Doctor: in my opinion you have minor injury to your lateral meniscusi advice the use of NSAIDs like alphinturn also restto be safe do an MRI to confirmif pain persist then you should see a physical therapistGood Luck"
},
{
"id": 89763,
"tgt": "What causes nausea, dehydration and dizziness after hitting the stomach?",
"src": "Patient: hi, yesterday i was hit extremely hard in the stomach to the point i could feel everything inside shake and ache. I fell down the stairs on concrete floor today and right after i got up i had severe nauseous, dehydration, and dizziness. i have no broken bones just a bruised back and jamed fingers, im curious to know if this could be something severe or just something that will go away over time? Doctor: Hi.The blunt trauma over the abdomen can have many consequences.You have already developed a few symptoms suggestive of some internal problems. This can be a contusion to any internal organ, perforation of the bowel and so on.The best way to get a proper diagnosis is to have a clinical / physical examination by a Surgeon to see the abdomen and vitals like temperature, pulse , BP etc. You have to go for the tests of the blood, urine and stool as well as ultrasonography and X-ray of the abdomen in a standing position.CT scan to confirm any positive findings. I would advise such a patient to get admitted to the hospital for observation and management."
},
{
"id": 213484,
"tgt": "Depression, inferiority complex, social withdrawal. Treatment?",
"src": "Patient: I have been getting into deep depressed states which over time are getting worse and longer. they are about a week long now and I can tell people are noticing a difference in me. when it happens it feels like my whole would is crashing down on me. when I get depressed I start to question my own judgement. what can I do? I am starting to get scared. When I try to talk about it with friends I can tell they do not want get involved. Doctor: HI nb5257. THANK YOU FOR YOUR QUESTION. YOU HAVE DEPRESSED MOOD BUT TO DIAGNOSE ITS A CASE OF DEPRESSION SOME OTHER FEATURES ARE NEEDED FIRST OF ALL ALL SYMPTOMS MUST PERSIST ABOVE 2 WEEKS,HOPELESSNESS,LETHARGY,LOSS OF PLEASURE FROM ALMOST ALL JOYFUL ACTIVITY,EASY NAVIGABILITY, REDUCED SLEEP AND APPETITE,GUILTY FEELING,SUICIDAL IDEATION OR THOUGHT. DO YOU HAVE ALL THIS SYMPTOMS? IF ITS SO THEN YOU ARE SUFFERING FROM MAJOR DEPRESSIVE DISORDER.YOU SHOULD CONSIDER A PSYCHIATRIST,TELL HER EVERYTHING,HE WILL SUPPORT YOU GIVE TREATMENT IN FORM OF ANTIDEPRESSANT OR COGNITIVE BEHAVIOR THERAPY SO YOUR PROBLEM WILL BE SURELY SOLVED BUT REMEMBER IT TAKE SOME TIME ATLEAST 4-6 WEEKS TO GETTING INITIAL RESPONSE TO DRUGS. THANK YOU DR SOURAV GANGULY"
},
{
"id": 63274,
"tgt": "Suggest remedy for lumps chin",
"src": "Patient: I am 26 years old, 5ft 2in and 110lbs. I have just noticed a small lump, about the size of a dime, just above my windpipe under my chin. It is slightly tender if I press on it. I have had nodules on my thyroid in the past and have had them biopsied - all results were negative for anything cancerous. This lump is a little higher than I remember them being in the past and also is directly in the middle of the underside of my chin. I have no other symptoms that I am aware of. What is your advice? Should I just monitor it for a while? This morning is the first time that I have ever noticed it. Doctor: hi.a consultation with a physician, preferably a general surgeon, would be of help to you. you could ask for a neck ultrasound to check on that \"lump\" and your previous thyroid nodules.. biopsy and/or surgical management will be directed accordingly. but all those lesions need regular monitoring. you have to observe them, and know if they are increasing in number, size or if they have change in characteristics/ behavior. hope this helps.good day!!~dr.kaye"
},
{
"id": 60524,
"tgt": "query abt jaundice",
"src": "Patient: hi, i m 24 yrs old i have a jaunidice from 20 days, the jaudice is diagnose in early stage. after consulting the doctor they recommend to avoid oily, fatty & protiens diet & take carbohydrate diet, fresh fruits etc. after 20 days- my billurubin level at this time 10.5 from a long time but enzime is decreasing dr. allows protein diet, tea, little bit fat in food but i feel my appetite is decrease what the reason behind it plz give ur suggestion Doctor: Your body is recovering from a illness. it will take some time to return to normalcy. IN the mean time maintain the diet as it is important for a complete recovery."
},
{
"id": 211529,
"tgt": "Imagining things that never happened. Could not deal with friends and people. Mental disorder?",
"src": "Patient: i am having some kine of mantle disorder .i am working in my work place for 9 months but still couldn't understand my duties well .actually even though i learn things dally from my job . i cannot apply them to my work,all the time i make mistakes and i have lost my common sense also .my supervisor used to blame me all the time now .some days i could work like a genus but some days i start my work as a beginner. i was having this thing since 12 years those days i could not even deal with my friends and public and also my body was very lazy and sleepy.and i have to mention that i had very bad habits since i was 12 years .it is masturbating.Now i m 30 years old but still doing it all the time more than my fiends do.i had another bad hobby.it was thinking of things never happen .i always liked to live in movies and imagine about miracles. A lot of things like this all the time .2 years ago i took some medicine from a psychiatrist doctor (Alprazolam USP 0.25mg and Escitalopram Oxalate Tablets 5mg ) after taking this medicine i was recovered well and could deal with friend and people.but there is still some thin wrong in my mined which effect to my job function as i mentioned at the start of this massage .please doctor i wont to be a normal person.how can i be a normal person.help me i am helpless now. Doctor: Hello I have read your question and I can understand your problem. You have symptoms like poor work performance, poor learning, poor concentration some times. You have also complaints like imagining things or living in miracles. You are also concerned about masturbation habit since last 12 years.See first of all I would like to tell you that masturbation is completely a healthy practice and it is not associated with any mental abnormality. There is nothing wrong with masturbation. The symptoms you have mentioned are most likely due to some depressive illness with some co-morbid obsessive ideations. I would advise you to consult a Psychiatrist for expert evaluation. Medicines like SSRI (Escitalopram you have taken already) are very useful in such condition.Thanks"
},
{
"id": 112570,
"tgt": "Lower belly and lower back pain, worsens on deep breathing, tightness in tummy. Cause?",
"src": "Patient: Hi, my lower belly and lower back hurt at times, mostly when I am in bed, and if I breathe in deeply they hurt worse. I feel like my tummy is tense and tight and its miserable. Keeps me from sleeping. Sometimes hurts bad when I roll over so I have to keep adjusting my sleep position . I am awake now and its still hurting the same but tends to go away during the day, come back when I lay down at night.. Doctor: Hello, Thanks for writing to us, Are you obese ? do you drink alcohol regularly?You may be suffering from lumbar muscle sprain ,or spinal disc bulge which can aggravate on lying down.If you tummy is large increased pressure may lead to pain .You need muscle relaxant with analgesic and neurotropic .I f you are drinking alcohol then these symptoms can be due abdominal fluid collection . you may need USG abdomen to further confirm. I hope that I have answered your query. Let me know if I can help you further. Take care"
},
{
"id": 40570,
"tgt": "Is Menogon 75 IU effective in the treatment of infertility?",
"src": "Patient: Am 39years old. I have 3 kids already and my last son is 11years now. My husband and I wish to have another baby, preferably twins and our doctor place me on menogon 75iu for 6 days. Is this save for me, can it increase my chances of having twins? I do not have any medical issues, just that it s been 11years since I have a child and will just want to have twins and stop. Doctor: Hello,That is a hormonal IUD to balance the internal milieu of uterus for pregnancy and it cannot definitely increase chances of twins for that Clomiphene Citrate should be tried.Hope I have answered your query. Let me know if I can assist you further.Regards,Dr. Bhadresh Lakhani"
},
{
"id": 209342,
"tgt": "Suggest remedy for irrational fear and lack of confidence",
"src": "Patient: I am 48 years old man, peace loving person. I am very well aware that Self medication is extremely dangerous & hence I would like to take your advise before taking any medicine. I will be highly obliged if you prescribe/advise medicine for me. Opponents (not really enemy) when he express anger or use threatening words or somebody stares at me (even minors) take away my confidence & fear immediately creeps into mind. If I see the same person again & again same situation arises, it takes away my courage even to see him. One must have basic courage in ones life to deal any problems in ones life. So this is my problem Sir, fear & lack of confidence. We all know living with fear means living in hell. Jo dar gaya woh mar gaya! Kindly help me. I read psychological books but It is of no use. Verbally I can understand & even convince others but when we face actually, words defeated. As you Doctors explain how brain works, the chemical variation in the brain leads to such type of problems. If it is so then it must be because of physical factors may be due to lack of nutrients? I dont want to play with Mind. If anything goes wrong then life is gone, thats sure. Hence I honestly request you to help me to solve my FEAR PROBLEM (Fear Propensity). NOTE : CAN I GET REPLY THRU EMAIL PLEASE. Thanks n regards Ramesh Mumbai My email id: YYYY@YYYY Doctor: HIThanks for using healthcare magicI think, you have social phobia and in that case, you need antidepressant as well as low dose benzodiazepine that would help to control anxiety symptoms. You can also try some relaxation exercise and exposure therapy to decrease anxiety symptoms. Better to consult a psychiatrist for proper treatment. In case, you need further help, you can ask.Thanks"
},
{
"id": 216883,
"tgt": "Suggest treatment for stump pain",
"src": "Patient: I am an amputee and have been so quite a long time. My problem is severe pain in the stump. On a scale of 1to10 it is a 10. These pains come on rather quickly about every few months and last at least a day. The pains come on every few minutes and last 10 or 15 seconds. The are very severe. I have talked to my Doctors about it but never a satisfied answer. I desperate and hope you can help. Thank you, Bobby Doctor: quite a lot. Any 'nerve pill' is likely to work:lidocaine patchanti-epileptics such as gabapentinamitryptilineand many others. A neurologist or pain managment physician, heck a regular doc too, should be able to treat this without need of narcotics."
},
{
"id": 105103,
"tgt": "Child has periodic bout of tonsillitis, suspicion of allergy. Recommended Romilast, Pneumonococcal vaccine. Advise?",
"src": "Patient: My 14 year old daughter has been suffering from tonsilitis- off & on since child hood. Recently her doc told me that she could be allergic to certain foods which are causing the problem. He recommended Romilast 10 mg for 4 months & Pneuococcal Vaccine & Influenza vaccine should I go ahead with the treatment . I searched on Google but Romilast seems to be a drug for asthma - please advise. Doctor: Tadvice is correct now a days it has also been recomended in allergies it started with used for asthma but now is recommended for other allergies the vaccine increases resistence you can go ahead investigations blood serum testing for specific antibodies for milk egg potato wheat rice and chana"
},
{
"id": 195422,
"tgt": "Suggest treatment for erectile dysfunction when suffering from diabetes",
"src": "Patient: Hello doctor! I am a male 55 yrs old and a diabetic patient using glycinorm M40 twice and Janumet 50/500 at night and Volibo 0.2 in the afternoon. Off late for the last 4 months i am suffering from erectile disfunction. please advise me the suitable medicine. my sugar level is fasting between 104 and 115 and PP is 225 and 250 Doctor: Hello and Welcome to \u2018Ask A Doctor\u2019 service. I have reviewed your query and here is my advice. Erectile dysfunction in men has multiple causes which include diabetes, hypertension, uric acid and high cholesterol. In your case diabetes mellitus is the chief cause of erectile dysfunction as it leads to neuropathy, your post prandial glucose levels are still high you need to keep them below 180, more over I would advise you to get your Hba1c levels, serum uric acid levels and fasting lipid profile. If any of the above tests are abnormal then get it treated first in most of the cases treating these condition leads to better results and if your labs come out to be fine then start tablet Tadalafil half daily night 1 hour before going to bed. Hope I have answered your query. Let me know if I can assist you further."
},
{
"id": 27767,
"tgt": "What are the symptoms of heart attack?",
"src": "Patient: like a week in a half ago right under my boobs but like the top of my stomach start hurting i never had these pains before i thought i was having a heart attack so i called my sister and described the feeling and she said it was gas so she came and gave me a zantac it made the pain go away but then i start feeling like i had to burp then nothing would come up sometimes it feels like i have to vomit but it hurts when it trys to come up but nothing come up and when i do burp it seems like it relieves a little pressure Doctor: DearWelcome to HCMWe understand your concernsI went through your details. There are many possibilities that acidic symptoms create pain across chest. It happens so because the gas accumulated in the stomach area pushes the diaphragm up. The chest cavity is tightly filled with its organs like lungs, heart. So every upward push of the diaphragm congests the chest cavity and we feel constriction and pain. In all possibilities, your problem could be acidic. Still, you should consult a physician and get your EKG taken and analyzed to be safe. If you still need my assistance in this regard, please use this link. http://goo.gl/aYW2pR. Please remember to describe the whole problem with full detail.Hope this answers your query. Available for further clarifications.Good luck."
},
{
"id": 216937,
"tgt": "What to do for pain in the back due to a fall?",
"src": "Patient: Hello, I slipped and fell on some ice on Mondaybut I felt fine that evening. On Tuesday and Wednesday my lower back and left buttock was quite sore. Since yesterday evening my right leg has been very uncomfortable with tingling. The General discomofort goes from my foot to my buttock (almost like restless leg). I have no bruising and no swelling. Is it possible that I have pinched a never somewhere? Doctor: Hi there, thanks for your question. Yes you are right. Such Patten of radiating pain with tingling sensation after fall on your back , would suggest a nerve picnched in your back. If the pain is not severe and there is no weakness or numbness, rest and analgesics will help relieve pain over a week But if there is very severe pain, area of numbness or weakness u should see your physician earliest for further work up, whicj might include X Ray and if need be an MRI. Hope this helps. All the best. Regards. Dr SBK"
},
{
"id": 167819,
"tgt": "Is it safe to use microgynon 30 while breastfeeding?",
"src": "Patient: Am 33 years with a 14 month old baby. weigh 80kg. about to under go ivf and have to use microgynon 30. still breast feeding. is it okay to use while breast feeding? would like to continue breastfeeding .How long does it stay in the blood stream? can i also continue breast fedding while using suprefact injection? How long does this stay in the blood stream? Doctor: using of microgyron can affect quality and quantity of breast milk , small amount of steroids is passed in the breast milk , American Academy of Pediatrics says it's safe with nursing .half life 12-50 hours a very few sources talks about suprefact , some of them suggest not to take it if you are breast feeding , since it passes in the breast milk .half life is 80 minutes I hope this helps"
},
{
"id": 67587,
"tgt": "Suggest treatment for lump on collar bone",
"src": "Patient: Yes hello, 2 was ago I felt a soft lump bigger than grape size right of of my neck above coller bone, above v shaped area on coller bone, not painful. SHould I be worried, blood test came back normal, going to see gen surgeon recommended by my primary doc Doctor: Welcome to health care magic. 1.The features and history suggest possibility of lipomatous - fat content lump. 2.Next possible cause could be lymph nodal enlargement.3.In this case i would have request for a ultrasound lump - which will help to detect the nature of the lump, its source and extensions.4.Since its soft in nature the bony source is excluded - how ever chest x-ray will help to see any bony involvement. 5.In case of lipoma - no active treatment needed, lymph nodal enlargement will be seen in case of any infection / inflammation - it will ease off once the infection treated.Good luck.Hope i have answered your query,any thing to ask ? do not hesitate to ask.http://doctor.healthcaremagic.com/doctors/dr-ganesh/62888"
},
{
"id": 93599,
"tgt": "Recurring abdominal pain, in different place, occasional sharp sting. Treatment?",
"src": "Patient: I have been having abdominal pains off and on for the last week. They are always in a different place...right upper, back, left upper, lower back right, once on awhile in middle but not much just always in a different place. Appetite is fine and actually usually feel better after eating. Pretty regular with BM's once a day. Occasionally a split second sharp sting but most just achy side pain, again in all different spots. Doctor: Hi welcome to Health care magic forum. Thanks for choosing H.C.M.Forum. You have got abdominal pain, it not constant and mot in one spot. Appetite and bowel moments are normal, feel better after eating. It appears to be gastric irritation due to peptic ulcer, or irritant foods, or due to worms in the intestine. I advise you to consult a gastroenterologist, for diagnosis and treatment, You may need to have gastroscopy, besides other routine tests for confirmation. Wishing for a quick and complete recovery. Best regards."
},
{
"id": 96463,
"tgt": "Burning stomach ache from last two days",
"src": "Patient: i am suffering from burning stomach ache from last two days any help. I take spicy food. i always have a problem of pinple on my face especialy on nose, how to control it how to control pimple Doctor: take mucaine gel thrice daily 15ml each time. drink cold milk at night and morning stop hot beverages for time being. mucaine will help to reduce the burning pain and soothes the stomach wall reduce the spicyness in the food. I sincerely suggest you to see a cosmetologist as pimples get physical examination done it is not related to food. take these measures along with consulting a cosmetologist never touch the pimples with the hand it is usual that many people keep on titillating them so dont do that wash your face with mild soap atleast 4 times a day please consult the cosmetologist"
},
{
"id": 31378,
"tgt": "How to cure plantar warts on the foot?",
"src": "Patient: Hi... I have planters warts on my right foot. It started out as a splinter a while ago then it progressed to be worse..It's been there for about 5 or 6 months. i know, a long time...But we went to the doctor to get it checked out & they said its planters warts...They are also saying that theres nothing inside, it is just infected or whatever...I think theres glass inside but we aren't for sure. We can't afford to go to the doctors right now so I was just consulting you guys. It's hard to walk & sometimes when it gets really bad(so bad it stings and numbs my whole foot) I soak it in a big bowl of water with epsom salt. Thanks (: Doctor: Hi thanks for asking question.Wart sometime can be go away without treatment within few month.But if it is severe and spreading then treatment has to be done.The best treatment is its removal through cryotherapy.For that apply liquid nitrogen with cotton swab or as spray over affected area.Laser therapy also available.Occlusive bandage available for it.Wear comfortable sock and shoes.While walking put some cushion or pad in shoes for comfort.I hope my suggestion will help you."
},
{
"id": 71036,
"tgt": "How can an old Koch s scar in the right lung be treated?",
"src": "Patient: hello doctor..i am having old koch scar in my right lung ..as of now i want to work abroad .so is there any way to hide these scars before i go for a x-ray? pls suggest me if is there any way to hide ... Doctor: Hello and Welcome to \u2018Ask A Doctor\u2019 service. I have reviewed your query and here is my advice. Scarring is a natural process after any infection in lung specially tuberculosis. There is no remedy for it. All these become lighter with time. You must do deep breathing exercises to improve the capacity of remaining lung. Hope I have answered your query. Let me know if I can assist you further."
},
{
"id": 51796,
"tgt": "Masturbating while fighting off a UTI with antibiotics",
"src": "Patient: Is it advised that males refrain from masturbating while fighting off a UTI with antibiotics? Doctor: Hi Dave,Welcome to HCM.Masturbation is better avoided, when you have urinary tract infection. However there is no hard and fast rule that masturbation should never be done during UTI. If you feel like masturbating and if you are confortable with it, you might as well masturbate.Primarily the attempt should be made to find out the source of infection, and that sorce should be avoided.Wish you all the best."
},
{
"id": 9396,
"tgt": "How can dry knuckles with itchy palms be treated?",
"src": "Patient: I have what looked like extremely dry knuckles and very itch palms for some time now. I soaked my hand and arms with hydrogen peroxide, it did it's job and foam where it started burning. I had a look with a lighted magnifying sight and saw what looked like worms. I worried should I be? Doctor: HIWell come to HCMThe symptoms are suggestive of some allergic condition and this can be best treated with Tab Hydroxyzine 10 mg three times in day and this is nothing to worry this would come around, hope this helps."
},
{
"id": 96016,
"tgt": "Suggest me drugs to abort 1 month fetus",
"src": "Patient: please suggest me drugs to abort 1 month fetuswhich should easy to use and safe I am suffering from abdominal pain and missed my period upto 7 days and and pregnancy test kit is showing positive result, so I wants to know the names of drugs-to-abort-1-month-fetus which should safe and easy to use Doctor: hi welcome to health care magic there are drugs which help in preventing pregnancy if they are taken before results show pregnancy.In order to abort the foetus ..there are abortion short procedures which are painless....please consult an gynaecologist. she will help you out with this.. do not panic.. take care"
},
{
"id": 207733,
"tgt": "How can mental illness causing headaches, palpitations, tremor, chest pains be treated?",
"src": "Patient: I have been suffering from mental illness from four years and symptoms of this are headache, palpitations, abnormal heartbeat, trembling, pain in chest and palms and not able to sleep. I consulted to many doctors and they prescribed me medicines but that does not work. Now i am very worried about my health and not able to do anything. Please tell me a solution about my problem. Thanks and Regards. Doctor: DearWe understand your concernsI went through your details. I suggest you not to worry much. From the description, I can say that you are troubled with anxiety disorder. Possibly Generalized Anxiety Disorder (GAD). For GAD, is a disorder, and not a disease, medicinal treatment may not work.Many researches and researchers confirm that medicines alone cannot cure mental disorders. Life style changes, change in thinking pattern, relaxation etc are as essential as medicines. Psychotherapy can help you changing your lifestyle and thinking patterns. Yoga and meditation help you to streamline your metabolism and neurological balance. Please consult a psychologist for further information.If you require more of my help in this aspect, Please post a direct question to me in this website. Make sure that you include every minute details possible. I shall prescribe the needed psychotherapy techniques which should help you cure your condition further.Hope this answers your query. Available for further clarifications.Good luck."
},
{
"id": 174925,
"tgt": "Is constant stomach and chest pain in a child with no help from zantac be serious?",
"src": "Patient: My 12 year old daughter complains with constant stomach pain and chest pain. A couple weeks ago her pediatrician put her on zantac. It doesn't seem to help. Should I be worried that it is something other than heartburn or reflux and take her to a different doctor? Doctor: HI...by what you quote I feel that the infant is having severe form of GER (gastroesophageal reflux). Your daughter will need - 1. Antireflux therapy - Proton pump inhibitors + antiemetics2. Always make her lie down in a propped up position - as in the care seat. She should not be sleeping completely flatly.3. Avoid heavy feeding and feeding more times and in small aliquots.Regards - Dr. Sumanth"
},
{
"id": 89365,
"tgt": "Suggest treatment for pain and excess sweating in lower abdomen",
"src": "Patient: I am 51 years old, and have become intolerant of heat. I perspire very easily, but mostly in the coccyx area. I will literally become dripping wet, looking as though I've wet my pants, which as you can imagine is extremely uncomfortable and embarrassing. I am 130 pounds, 5'5\", have been dealing with a strange fatigue and pain in my upper abdomen that radiates to my back. I am also chronically thirsty. Doctor: Hi there..Thanks for writing in.You haven' t mentioned if you are a male/ female.If female, then menopausal period might explain the cause of your symptoms.Other possibilities I can think of are Thyroid disorders, Diabetes, which can also present with increased sweating, thirst, fatigue. To rule out these medical conditions get a TSH, FBS and PPBS done.Acidity related symptoms are also possible with upper abdominal pain radiating to the back. If the pain is on the right upper abdomen radiating to the back, gall stones might be the cause. Get an ultrasound abdomen done to rule out gall stones and other pathologies.Hope this helps.Regards,Dr. Divya Kuttikrishnan"
},
{
"id": 8964,
"tgt": "Is gluthathione useful to make skin whiter ?",
"src": "Patient: im a female 31 years of age and married but not yet had a child,. i just want to ask gluthathione is really makes the skin whiter? what is really their benefits? and also the side effect.? is it ok to take like me that planning to conceive? Doctor: Hi welcome to HCM Read your prob, gluthaion is noyhing but a antioxidant which reduse free radicals from our body n keep our skin ,organ healthy. along with its antioxidant effect it reduse synthesis of melanin in skin n reduse darkness of skin with in a limit. it only reduse darkness not make skin white or fair. its no side effect in pregnancy so you can take it during pregnancy. Dr aseem saraswat aseemadhuri@gmail.com 9982583020"
},
{
"id": 41566,
"tgt": "Suggest treatment for infertility",
"src": "Patient: Hello doctor pls help me, my name is rinku n I am 40yrs old, I HV 2child age 20, n19 from my first husband, I HV my left tube blocked n bulky uterus now I am trying to conceive from my 2nd husband who is 34yrs old but could not, All r medical reports r normal except my problem. I hv done 2 ivf but failed can u advised me whether is it to possible to conceive again. Doctor: HelloYou didn't mention complete history.Your left tube is blocked and uterus is bulky.If right tube is patent then there is no problem in getting normal pregnancy .Tow IVF done but failed , need not to worry.First of all right down complete menstruation history.Study of size of dominant follicle is important .If follicle dominant ( however once in two months because of left tube is blocked hence ovulation only from right side ) then date and time of ovum release . Endometrium thickness .So please upload all these reports and thereafter it will be possible to say something about pregnancy.Good luck."
},
{
"id": 126154,
"tgt": "What does this blood report post a knee replacement surgery indicate?",
"src": "Patient: Having replacement knee surgery 10/02. Had blood work yesterday...get phone call today, but just a message to call dr office. I can see the results on line...only thing that looks odd is wbc is 8 and squam epithel is 19. What does this mean and will it affect my having surgery? Doctor: Hi, It may be urine routine results. It will not affect the prognosis or surgery. Hope I have answered your query. Let me know if I can assist you further. Regards, Dr. Shinas Hussain, General & Family Physician"
},
{
"id": 87714,
"tgt": "What causes pain in abdomen while driving?",
"src": "Patient: Left abdominal pain that gets aggrivated when driving, sometimes when very active or during sex. I have discussed this with physicians off and on for 2 years. Had an ultrasound recently: no findings. Colonscopy last year unearthed 3 polyps (removed). Pain still exists. Recenty painful when I press on this area. Doctors write letters: see you soon. Your in good health. Frustrating. Was hoping it was a hernai but ultrasound came back negative. In menopause but zero to no symptoms. Was told 2 years ago that ovary is very shrunk on left side. Any suggestions? Doctor: Hi.Thanks for your query and an elucidate history.You are menopausal without any symptoms related to it - getting pain in the left side of the abdomen- aggravated by driving, in activity or during sex - normal ultrasonography - 3 colonic polyps removed but pain still persists - pain on pressure. All these history is suggestive that you have colitis and more probably of a IBS type hence the reports are normal.The inflammation causes the pain as the body moves. The shrunken ovary has nothing to do with the type of pain you have. Get antispasmodic with an anti-anxiety pills regularly and you may be fine. '"
},
{
"id": 203824,
"tgt": "What could be the reason for unconscious masturbating at night?",
"src": "Patient: hi sir...i am a student..i am masturbating for past few years during night time without my knowledge sometimes..i don't know how is this happening..i fear that it will create a problem for my wife in future...when i feel like ejaculating during night i will go to rest room...that time when i pass it then it will be urine that time..i don't know what prob is this.. Doctor: Hi student. You are a student of this problem too. This is very natural process and do not worry about it any more . Carry on the way it is. Masturbating has no bad effects on your future married life , neither for you nor for your would-be wife."
},
{
"id": 157985,
"tgt": "Stage III melanoma. In phase II interferon treatment, melanoma returned on left thigh. Treatment not effective?",
"src": "Patient: I am a Stage III melanoma patient. During Phase II of interferon treatments (self-injected 3 times a week treatment), my melanoma returned on my upper left thigh. I am very discouraged by this news. Is this revealing that the interferon treatments are not effective on me? My oncologist told me that interferon does not work on everyone. Doctor: ya interferons not work on every patient.you may have go for change of chemotherapy.your disease need to be restaged and investigated before proceeding to change of treatment."
},
{
"id": 22420,
"tgt": "Does heart failure cause breathlessness annd excertion?",
"src": "Patient: 2 1/2 years ago I was in ICU with toxic shock. at one point my vital organs started to shut down but was brought back from the \"brink\". But now on my records it says I had heart failure. Could my heart have been damaged and would this account for the breathlessness on exertion? It soon goes when I sit down. I am 81 weigh 11 1/2 stone and am taking Amlodipine 5mg for BP and otherwise feel reasonabley fit and intend to lose more weight Doctor: Hi,Certainly heart failure can cause symptoms like breathlessness, swelling of legs, and weakness. For confirmation of the same, you should get one echo done which will confirm the diagnosis. Also have low salt and water diet for sometime and look for the response. Maintain healthy lifestyle like avoid fatty, oily and high calorie diet. Have low salt diet and monitor blood pressure regularly. Regular exercises like brisk walking, jogging according your capacity at least 30 min a day and 5 days a week. Eat lots of green leafy vegetables, fruits, fish once or twice a week, and avoid meat. Avoid smoking and alcohol if any. There shouldn't abdominal fat deposition or obesity. Get your lipid profile and sugars tested once.Hope I have answered your query. Let me know if I can assist you further. Regards,Dr. Sagar Makode"
},
{
"id": 21896,
"tgt": "What is the treatment for nausea , rapid heart beat and severe headache?",
"src": "Patient: I was seen in the er last night after I vomitted and passed out. squad took me in to er and I was unresponsive in squad and I don't remember any of this, I start having full body tremors in the squad and er and during the tremor my heart rate was as high as 163 (per my mom) they did blood and urine tests all came back normal. I developed a severe headache and they treated me for a virus. Stated that I could have a virus in my blood stream that would cause the tremors. sent me home with pain pills and nausea medication and told me to follow up with my PCP. I don't think the diagnosis was correct? No head ct was done and they did not seem concerned about the tremors at all. This is a new symptom for me. any ideas? Doctor: Hello!Welcome and thank you for asking on HCM!I carefully passed through your question and would explain that your symptoms could be related to different causes: - anxiety- electrolyte imbalance- epileptic seizures- fever or an infection. Do you have fever? This would indicate a possible infection (probably viral as all the tests resulted normal). Coming to this point, I would recommend performing an EEG to exclude seizures and thyroid hormone levels to exclude possible dysfunction. Hope to have been helpful!Kind regards, Dr. Iliri"
},
{
"id": 171415,
"tgt": "What causes abdominal pain and mucus in stools of a child?",
"src": "Patient: 8 yr old child has had intermittent abdominal pain, and heavy mucus in stool for approx 2 weeks. 3 wks ago, child finished med for ear infection. has allergies, tendency to snuff. also had severe constipation when very young, appears to stretched colon, as when he passes, it is unusually large. Should i be concerned about ibs, or possible blockage, or might it be just mucus running through his system Doctor: Hi, you need to do a stool R/M, ultrasound abdomen. This could be infection and child should be put on antibiotics. Revert back with exact weight of child. Are there other associated symptoms like fever, cough?I hope this has helped you. If you have any more questions, i will be very happy to help you."
},
{
"id": 17637,
"tgt": "Are migraine problem and fluctuating BP inter related?",
"src": "Patient: Hi...I was just wondering what numbers is concern (if any) with blood pressure. I just checked mine ant it s 93 over 70....altough I do have migraine....I m wondering how much closer they can get to each other before I notice something is worng? Thx. Doctor: Hi, After going through your medical query I understand your concern and I would like to tell you that migraine has many triggering factors and high/low BP can be one out of all reasons. Generally BP under 140/90 is normal and 90/60 is low. Your BP stays in lower side for which you should take extra liquids. Take Nexdom during migraine. Hope I have answered your query. Let me know if I can assist you further. Regards, Dr. Bhanu Partap, Cardiologist"
},
{
"id": 140671,
"tgt": "Suggest treatment for neck pain and numbness due to disc desiccation",
"src": "Patient: I had a MRI and have been in so much pain in my neck lower back also my right keeps going numb. the MRI said that i a degree of disc desiccattion,and cervical straightening of the cervical lordotic curve. please tell me more about this and how is it treated cause the pain is BAD. Doctor: Hello, The reading of the MRI is rather nonspecific with respect to your symptoms and in simpler terms suggests that you may be suffering from normal articular degenerative changes that accompany the ageing process. Cervical straightening of the cervical lordosis is often taken as a sign of muscle tightening or spasming that could account for some of the pain. Your best approach to treating this problem would be to start with some form of cervical physical therapy and even AQUATHERAPY itself may be very helpful given the MRI findings. Your doctor would best be able to counsel you on the use of acute pain management medications since they know your history and other medications you may already be taking. Surgery to correct this problem does not seem to be a possibility given the MRI findings. Hope I have answered your query. Let me know if I can assist you further. Take care Regards, Dr Dariush Saghafi, Neurologist"
},
{
"id": 82795,
"tgt": "What to do for swelling in the stomach due to lupus?",
"src": "Patient: I JUST FOUND I HAVE LUPUS AND I AM FEELING EXSTREME DISCOMFORT IN MY BELLIE WHICH HAS SWELLED TO LIKE 5MOS PREGNANT IS THIS NORMAL I AM TAKING PLANIGUINIL BUT I ONLY BEEN TAKING IT FOR 2WKS SO I DONT THINK ITS THAT MY ABDOMIN IF VERY FULL FEELING AND ACHING TO THE PRESSING TOUCH WHAT CAN I DO. Doctor: Hi! It could be just fat deposition, sedentary lifestyle but as per your query it seems to be rapid so advised to see your primary physician & get investigated & diagnosed for intra abdominal disorder."
},
{
"id": 98936,
"tgt": "What causes swelling in eyes and lips after smoking marijuana?",
"src": "Patient: Hi, I was wondering if marijuana causes your face to swell. After smoking recently my eyes and my lips swelled up severely. I have only had my eyes swell up just a bit in my past smoking experiences. I am a chronic smoker. I took a detox pill and have been working out all day in result my swelling has gone down quite a bit. Doctor: HI, thanks for using healthcare magicIt is possible that you are allergic to it. Allergies can occur to any chemical agent including normal traditional medications and also to marijuana.It would be best to avoid future use since continued use of an allergen would cause the allergic response to progressively worsen.An oral antihistamine would help decrease the allergic response.I hope this helps"
},
{
"id": 46213,
"tgt": "What is the prognosis for kidney failure patient needing dialysis?",
"src": "Patient: Hello,My aunt is 84 years old, is diabetic, has had a heart attack, stroke, and mastectomy. Last October a malignant tumor was found in her kidney and the kidney was removed. The procedure was to have been robotic, but the surgeon cut into vein and her kidney was removed surgically. My aunt almost died. Since then, she has continued to decline in strength, the incision never healed from inside and she has been going to a would center weekly for months now. Two weeks ago, she became very ill and was taken to the hospital where she was told she had a UTI. Unfortunately, it turns out she has MRSA, and an ulcer on her ankle that had been there for months and wouldn't heal. Her remaining kidney is not functioning and she has just been told she will have to go on lifetime dialysis. She is having difficultly breathing and is very weak. Can you offer an approximate prognosis based on this information. She has been through so much, and always with a wonderful attitude, but she is understandably very depressed and her family is very worried.Thank you. Doctor: Hello,As an urologist and transplant surgeon, I can understand your anxiety. Your aunt's survival through so many illnesses is itself god's miracle. Only he can prognosticate about her survival. But as you know, all our illness and recovery is according to God's plan. As we age, our systems get weaker and survival is therefore decreasing. How long each system continues to function, no man on earth can say. If she survives for another year, it'll truly be another miracle.Hope I have answered your query. Let me know if I can assist you further.Regards,Dr. Matthew J. Mangat"
},
{
"id": 190291,
"tgt": "I have gum recession pain. Will my tooth need to be extracted ?",
"src": "Patient: Hello, I have some gum recession around an upper back tooth that has never given any trouble until today. When I brushed my teeth this morning, I experienced sharp pain in the recessed area. It does not hurt to bite or eat. The earliest my dentist can see me is 3 weeks from now. Is there anything I can do until then? Will my tooth need to be extracted? Doctor: Your gum recession may be either due to a faulty brushing technique (if you are young), or due to bone-loss (if you are past middle age), or both. Do warm saline rinses 6- 10 times a day, use a desensetising mouthwash or toothpaste, a do gentle gum message (with finger) 3-4 times a day for 5 minutes each,till you get to see your dentist. A proper treatement can be adviced by a dentist, once he/she sees you in his/her office, and gets to a proper diagnosis."
},
{
"id": 20468,
"tgt": "What is the treatment for tachycardia?",
"src": "Patient: I am 50 years old have tachycardia, but lately all I want to do is sleep. I injured my back in 2000. Get headaches alot, can go 2 to 3 days not eating. Get dizzy standing up. am on Cymbalta 60mg once a day,Metoprolol ER 100mg once a day,Lisinopril 10mg once a day, Morphine 30mg 1 every 8 hours, Methadone 10mg eight times a day. Also am 5 6 and weigh 232. I know over the winter I got lazy. Sometimes walking just wears me out and my heart beats out of my chest, yesterday my pulse was very high and I was just sitting. Doctor: Hello,You may be having some condition here, a tachycardia while sitting and getting effortless may be signs of heart failure. You need to get checked by a cardiologist to do EKG, echocardiography, and some labs. Some thyroid dysfunction may be a differential here. So, I suggest to do a thyroid profile. Also, check your blood pressure regularly as it may be lowered too much with you medications.Hope I have answered your query. Let me know if I can assist you further.Regards,Dr. Mahmmad Gamal"
},
{
"id": 103245,
"tgt": "Suffering from allergy, cold, itchy ears, ringing noise in ears. Prescribed ciprodex ear drops, candidbiotic. Suggestions?",
"src": "Patient: i have allergies and cold problems. my ears are always itchy and sometimes sticky . occasionally it hurts i use ear buds to clean with hydrocortise doc prescribed ciprodex /oflaxcin eardrops i also have ringing noise in my ears , now i have this candidbiotic eardrops from india is it the same kinda drops is it advisiable to use .thank you Doctor: drops are not advisable as there is eustachuan tube blockage leading to symptomps it can damage druminstead ou can use anti allergics and local antiu allergic ointment in external aditory canal and nasal drops to clear tubesad when blockage is clear every symptom will be oks"
},
{
"id": 223362,
"tgt": "Why are my birth control pills only for three weeks instead of four?",
"src": "Patient: I have a question about my birth control pills. My first two packs had brown pills but now my third pack does not and where it doesn't have the brown pills it only has pills for 3 weeks instead of 4. After I take the last pill for the third week do I go 7 days without taking a pill and then start back on the new pack? Doctor: hello user,birth control pills are hormonal pills meant for 3 weeks..one week is placebo pills which are just vitamins or iron tablets..some companies gives placebo pills,some not..it does not matter wheather u take placebo pills or not..thanks.."
},
{
"id": 225778,
"tgt": "Is it safe to take nordette twice a week as emergency contraceptive?",
"src": "Patient: I'm Miss Rivera, I just want to confirm if it's safe to take nordette twice in a week as an emergency contraceptive. I know its a regular birthcontrol pill and it is not an emergency contraceptive and yet is it used by few people in high dose like 4 pills with in 120hours after intercourse but I was just told to take it as a precaution. Doctor: Hi thanks for your question.Yes you can take regular contraceptive pills in high doses for emergency contraception (EC)theoretically speaking taking EC twice a week is safe ,but it should not be made as routine as EC causes lot of irregular and heavy bleeding sometime to the dangerous level along with lot of gastric upset and menstrual cycle get disturbed. It should be reserved for emergency use only.Hope this answers your question."
},
{
"id": 12771,
"tgt": "What do red bumps around buttock in 23 month old indicate?",
"src": "Patient: My 23 month old son has tiny red bumps that look a little like pimples on his butt and a little on the back of his left thigh. They're not getting better or worse. They started about 3 days ago and haven't gone away. We thought the cause was that we changed the brand of diaper he was wearing, so we switched it back. I haven't been using baby wipes when he needs to be changed. Instead I have been using water and a cloth, and soapy water when he poops. I can't seem to find anything anywhere online that can tell me what these little red bumps are. Doctor: H Dear,Understanding your concern. Condition seems to be mainly infection of follicles leading to folliculitis and it could be due to wetness in that area. I would suggest you to keep area clean and apply antibacterial powder to avoid red bumps. Apply mupirocin ointment topically twice a day. If symptoms still persist, visit dermatologist once and get it examined and start treatment accordingly.Hope your concern has been resolved.Get Well Soon.Best Wishes,Dr. Harry Maheshwari"
},
{
"id": 182364,
"tgt": "Can a tooth infection cause sinus pain?",
"src": "Patient: I had an infected tooth that was causing me severe pain up into my head, jaw, and behind my eye. I got a root canal 2 days ago and immediatly had sinus issues after. Today my skin has been very itchy/burning and my stool is black. Could the tooth infection/root canal have caused this or is it a reaction to the drugs I was prescribed (Ibuprofen 800mg and amoxicillan)? Doctor: yes it does, you will have sinus issues due to infection with your upper molar teeth, and black stools may be due to hyperacidity caused by your amoxicillin and ibuprofen. itching may be due to allergy to amoxicillin, so please conact ur health care provider and see if something can be done about it."
},
{
"id": 30810,
"tgt": "Suggest remedy for infection inside mustache",
"src": "Patient: Hi, last night I felt a little dump on my mostache so I started to scratch. It. Quicly became infected, to me it looks like its a haair ingrown inffection. I put vasaline, then I put triple antibiotic oiment. Is there any over the counter medecine I can buy to get rid of this?Thank you. Doctor: Hi,Welcome to Healthcare Magic Forum.I have gone through your Query & I really appreciate your concern about the problem you are facing.As a physician I would like to tell you that according to the symptoms you are prescribing you may be suffering from Folliculitis in Mustache. I would suggest that you continue using Triple antibiotic cream as you were using.Additionally I would like to tell you that if the infection still persist for more than5 days or if Pus starts coming out from it, you may need to visit a physician to get prescribed a Antibiotic course.Thanks for Trusting & Posting your Query on this forum.Hope this will help you. Feel free to ask any further question.Have a nice Day.Regards,Dr.Abhijeet"
},
{
"id": 26370,
"tgt": "What causes dizziness and nausea?",
"src": "Patient: For the past month to month and a half I get severe dizzy spells, I'm nauseous , I go pale my Bp goes up along with my pulse and my eyes do this fluttery thing. They put me on a holter monitor and blood pressure monitor which came back normal. What do you think it could be? Doctor: Hello!Welcome and thank you for asking on HCM!I understand your concern and would explain that your symptoms may be related to an inner ear disorder. A partial epileptic seizure can not be excluded, based on your symptomatology. So I recommend consulting with the ENT specialist and perform a careful physical examination and labyrinthine tests. If the above tests result normal, an EEG would be the next examination to be done. This would exclude possible partial seizures, which could mimic this clinical scenario. Hope to have been helpful!Best regards, Dr. Iliri"
},
{
"id": 189422,
"tgt": "What could be the reason for mouth ulcer with bleeding ?",
"src": "Patient: my husband has been suffering with mouth ulcers and they have also been bleeding very heavy but just today he pulled out this big thing it was full of blood out of his ulcer can you please explain what this could be thankyou. Doctor: Dear friend. Thanks for sharing your concern. According to the history given,it appears that your husband has apthous ulcer.It is caused due to vitamin deficiency, especially vitamin B. Also stress is the aggravating factor here. Do not worry .Ask him to take tablet becasolues for a month. Locally apply dologel for relief. Avoid stress and hot and spicy food stuff. Hope it helps. Thanks. Take care."
},
{
"id": 45666,
"tgt": "What causes fatigue and edema in the feet and lower legs?",
"src": "Patient: Mrs. Smith is a 46 year old woman who has a history of hypertension and obesity. She has recently been feeling extremely fatigued and has noticed edema in her feet and lower legs. She also reports shortness of breath and abdominal pain. She says she has not been sleeping and is suffering from polyuria. Doctor: Hi, As you say she has edema in the feet and legs indicates she has developed kidney disorder and also she has hypertension which indicates that there is also some cardiac problem. Shortness of breath is due to pulmonary edema from retention of fluids. All it needs an emergency intervention to check the functions of vital organs like kidneys and heart. I suggest you take him to the emergency room as soon as possible for proper clinical examination and other necessary tests. Hope I have answered your query. Let me know if I can assist you further. Regards, Dr. Soheel Hussain Zargar, Dentist"
},
{
"id": 168311,
"tgt": "What causes frequent vomiting, with high temperature followed by nose bleed?",
"src": "Patient: My baby (1yr) is suffering from what seems to be a Vomitting Bug with a temperature, he has just tried to be sick again but there is nothing left in him to come up, however, his nose has started to bleed too - could this just be down to the wretching? Doctor: Hi, Welcome to HCM. Can understand your concerns. Fever, vomiting followed by nose bleed is suggestive of food poisoning. Other causes could be upper respiratory tract infection. I suggest you to get the child examined by a pediatrician so that we may not miss important finding. If the child is feeling so sick then a admission may be required. I hope this will help you. Wishing your child good health. Take care."
},
{
"id": 83449,
"tgt": "Does trapic 500 help stoppage of bleeding?",
"src": "Patient: I am 35 and i have been bleeding profusely for the past couple 5 days and i am travelling to the USA tomorrow is there any medications i can take just to stop the bleeding like trapic 500 or something. i will visit a doctor as soon as i reach the US. Doctor: Hi, Yes, it helps in reducing bleeding. Trapic-500 (tranexamic acid) is commonly prescribed to stop or reduce the bleeding associated with menstruation and other conditions. It may be taken for short term control of bleeding associated with menstruation. Hope I have answered your query. Let me know if I can assist you further. Take care Regards, Dr. Mohammed Taher Ali"
},
{
"id": 57564,
"tgt": "Suggest ways to lower SGOT and SGPT levels with no help from Udiliv, Bevon and antoxide tablets",
"src": "Patient: sir,I AM A NON ALCOHOLIC FATTY LIVER PATIENT SINCE LONG.TAKE TAB. UDILIV-150MG AND THEN 300 MG DAILY SINCE LONG.BUT SGOT AND SGPT LEVEL DOES NOT DECREASED.THEN ANOTHER DR. RECOMEND TO TAKE CAP.EVION-400MG/CAP.BEVON AND ANTOXIDE DAILY.I AM TAKING THESE MEDICINE SINCE LAST 3 YEARS. AND THEN MY LIVER'S SGOT AND SGPT LEVEL COMES DOWN BUT STILL ITS IS HIGHER THEN NORMAL LEVEL.SHOULD I CANTINUE THESE MEDICINE OR TAKE A BRAKE. MY WEIGHT IS NORMAL IN COMPARE TO HEIGHT.THANKS WAITING FOR YOUR ADVICE Doctor: HIThank for asking to HCMI really appreciate your concern no medicine will work in SGPT and SGOT this is kind of enzymes it rises in inflammatory process of liver parenchymal cells once the inflammatory process reverts the enzymes also start declining slowly and gradually some times this takes months hope this information helps you, take care and have nice day."
},
{
"id": 49100,
"tgt": "Spells of dizziness, headache, severe back and chest pain. Had pancreatitis, kidney problem. Taking iron tablets. Serious?",
"src": "Patient: hello my na,e is cindy and i am 39 yr old and i wass told about 3 yrs ago i hade pantirtis and my kidneys were not working well and now i have dizziness and faiting spells and head aches and serve back pain and chest pain what could be going on if i go to er how long will they keep me there i have kids that why i dont go and i take 3 iron pills a day and have over 100 refills is there something life treating Doctor: hi, As you have pancreatitis, your symptoms like chest pain, back pain are likely to be due to it. dizziness and fainting spells are seen in kidney diseased patients due to uremic autonomic neuropathy, lead by sudden changes in posture. your problems are understandable and i regret them. you may manage conservatively and as long as you stick on to the prescribed drugs and check your kidney status regularly, life threatening issues can be avoided to the best.hope this helps."
},
{
"id": 183374,
"tgt": "Suggest treatment for decayed or broken teeth",
"src": "Patient: Hi Doc, My name is Nitin. I am a 27 year old male. I have some serious problems with my teeth and I need some advice, I have only 14 teeth in my lower jaw and 13 in the upper one. Almost all these teeth are either broken or decayed. Could you please give me a suggestion of what can be done to get back my smile? Doctor: Hello, thank you for consulting with healthcaremagic. As you are mentioning that many of your teeth are missing and now the rest are decaued or broken, it looks that you might be having a defect in enamel of dentin of the tooth, these are the component of tooth structure. In this case just treating the decayed teeth is not the solution, you have to get an OPG done of your jaw, which will show the cause of this, then get the rest teeth restored with topical flouride application on the teeth and then at the end get a full mouth rehabilitation, means get crowns on the remaining teeth.Hope it will help you."
},
{
"id": 81704,
"tgt": "Suggest treatment for sharp chest pain after panic attack",
"src": "Patient: Hi , I had went to a haunted house last night and I had what seemed to be a panic attack. I had chest pajns afterwards and this morning they werent very bad but they keep occur occuring and some of them are very sharp pains. Do I need to go get checked out? Doctor: Hellowelcome to hcmwell the chest pain you had was most probably due to ACUTE ANXIETY AND PANIC ATTACK...but to be on the safer side kindly get done an ECG and XRAY OF CHEST and then send me a scanned copy of your reports so that i can make a precise diagnosis and guidd you furthertake caredr rahul"
},
{
"id": 203349,
"tgt": "Experiencing burning sensation on penis and anus",
"src": "Patient: Hello Sir / Madam,I have burning sensation on the top of the penis and the anus i.e \u2022\u00a0\u00a0\u00a0\u00a0\u00a024 hours burning sensation of the Meatus top peripheral part of the Glans and the upper part of the Corona \u2022\u00a0\u00a0\u00a0\u00a0\u00a024 hours burning sensation of the peripheral regions of the Anus and the upper part of the Rectum Doctor: HiThank you for asking HCMI have gone through your query.Burning sensation can be due infection like Sexually transmitted.Usually genital herpes gives such symptoms. But if you dont have any sexual history or exposure then we can exclude that.Other possibility is urinary tract infections.That you can rule out by doing a urine routine test.If its urinary infection then it can be treated by antibiotics.For STDs according to infected agent treatment may vary. For herpes antiviral treatment like acyclovir or valacyclovir is given.Hope this may help you.Let me know if you have any further query."
},
{
"id": 74877,
"tgt": "What causes upper respiratory infection and severe congestion?",
"src": "Patient: I have an upper respiratory infection that seems to be lingering. I have been taking amoxicillin, 500mg. for 4 days. I do feel better but still have a lot of congestion. The nasal mucus has gone from being quite thin and clear to much thicker and greenish/yellow. I am concerned I am not getting rid of the infection. What do you think? Doctor: Respected user , hi I evaluated your query thoroughly.* Usually upper respiratory tract infections are airborne through atmospheric bacteria / virus / pollens / tiny dust particles .* Recovery part depends upon the causative organisms , host immune response , type of treatment , associated comorbid conditions etc.* Better to get consulting with your doctor to get early rid of infection & congestion.Hope this clears your doubts.Thanks for using Health care magic & keeping trust in our medical services.Wishing you fast and speedy recovery.Welcome for any further issues.Regards dear take care."
},
{
"id": 85624,
"tgt": "Should Banocide Forte be continued despite it causing vomiting?",
"src": "Patient: my mother,aged 72 suffering from eosinophilia and also a patient of rheumatoid arthritis has been prescribed Banocide Forte. 2 days after taking the medicine, she showed symptoms of vomitting. would you recommend her to continue taking the medicine? If yes then why? Doctor: Hi, She may need to stop it. Eosinophilia can be due to drugs or any other auto immune conditions. If any signs of systemic worms infestation or stool analysis reveal so, than alternative like Albendazole may be started. If not tolerated better to stop a medicine which is given prophylactically. Hope I have answered your question. Let me know if I can assist you further. Wishing you a speedy recovery. Regards, Dr. Saddiq Ulabidin, General & Family Physician"
},
{
"id": 17165,
"tgt": "What causes unusual racing & pounding of the heart?",
"src": "Patient: i have a question 3 days ago my heart started to racs and started pounding really hard and i wasnt moving around alot or doin any activities i usually wouldnt do? and still today it pounds or races at random times what should i do or what could it mean ? Doctor: Hi, You should see a doctor and do some examinations: routine blood tests, TSH, FT3, an ECG, a cardiac sonography, and a 24 h ECG Holter monitoring. These are to see where is the problem and if it needs medications. Hope I have answered your query. Let me know if I can assist you further. \u00a0\u00a0\u00a0\u00a0\u00a0 Regards, Dr. Anila Skenderi, General & Family Physician"
},
{
"id": 113743,
"tgt": "Chronic back pain, nerve root damage to L5-S1. Treatment options?",
"src": "Patient: hi been suffering with back pain for about 12 years the last 5 worse chronic pain .got told it was l5s1 with nerve root damage.i got referred to a cosultant who for 2 yrs said he didnt know what was wrong with me.got referred back to the first place i went to and they said its ddd but not worth operating as the nerves are damaged.just wish someone would tell me what it is exactly Doctor: hi, your history is inadequate to answer about your query correctly, You show to a good orthopaedic surgeon or a neuro surgeon."
},
{
"id": 17006,
"tgt": "Can lack of second stent increase the chances of a heart attack?",
"src": "Patient: My husband just had a stent put in his coronary artery - he has had several before including a 3 vessel bypass and a ballloon to open the first blockage he had (which produced a heart attack and damage to his left heart muscle. The doctor also told me that my husband has an artery , the mammary artery, that remains blocked . He said the thought of putting two stints in but they would likely close as it is too long - my question is what will this mean to his day-to-day functioning and is it likely to cause another heart attack? the doctor said the stent he put in should give him a good improvement. Doctor: Dear Mrs,To give you a complete answer I need all his discharge papers, his last echocardiography answer and the loops of his last coronary angiography and stenting.However, based on provided information, if his Mammary artery is completely blocked, then there is no risk of another heart attack, and sometimes, when an artery is blocked the other one starts supply the region of that artery, or if he already has had a heart attack in that region in the past, then it will cause him no further harm.If his doctor told you that he will improve after the stent he has put, most probably it was an important artery and he will feel the difference soon.Wish a good health to your husbandIn case of any further questions, don't hesitate to contact me with more detailed information.Regards,Dr Baghdasaryan"
},
{
"id": 180466,
"tgt": "What causes bad breath?",
"src": "Patient: I came home today and my 70 year old healthy husbands breath smells like pickles even after brushing/rinsing his mouth should this be a worrythis is a new odorMy husband has a problem with bad breath I chalk it up to his Marilyn Bridge on his front teeth. He is very good about brushing. He is not aware of the odor either tonight or any day unless I tell him. Any suggestions Doctor: Hello and Welcome to \u2018Ask A Doctor\u2019 service. I have reviewed your query and here is my advice. As per your complain bad breath also known as Halitosis can occur due to a number of causes like poor oral hygiene, oral and dental infections,post nasal drip, sinusitis, acid reflux,respiratory infections, diabetes etc..You should consult an Oral Physician or an Otolaryngologist and get evaluated and a thorough clinical evaluation and investigations depending upon suspected cause can help in diagnosis..Treatment of underlying cause will relieve the symptoms..As of now you should start gargling with a flavoured mouthwash twice a day..You should chew flavored sugarfree chewing gum..Do warm saline gargles twice a day..Limit the intake of high aroma foods like onion and garlic..Drink plenty of water and maintain a good oral hygiene..Hope I have answered your query. Let me know if I can assist you further. Regards, Dr. Honey Arora"
},
{
"id": 11768,
"tgt": "Pigmentation in neck, breast and lower stomach, have PCOS. Treated with Tretinoin Gel. Treatment for pigmentation?",
"src": "Patient: Hi, My name is Angela Chapman and I have hyper pigmentation around my neck, thighs and some under my breast, lower stomach. I am obese, and I also have a disease of PCOS ( Poly cystic ovarian syndrome ) The doctor told me the only way to get rid of the dark spots is by losing weight, offered me some treatment of Tretinoin Gel, 0.01%. I have used the Gel before around my neck, I have never gotten a reaction from it! It actually lighten up the skin around my neck, made some of the dark spots go away. I have been working out, trying to lose weight by taking Metformin to prevent the high level of insulin resistance on my neck. I just recently saw a doctor again bc I moved out of state, my prescription was over due. I had to get another prescription note, when I went to pick up my medication from the clinic. They screwd up and gave me Tretinoin Cream, it s 40 g. It some how made a reaction on my skin, made me break out in a rash, it itches around my neck, thighs. I have really really bad dark spots in between my thighs, and I m trying to get good skin. But the doctors I have been seeing in the past have been screwing up on my body. I had a cyst in my right ovary before, it was huge. Instead of going through the surgery they decided to put me on different birth control pills to help dry out the cyst in my right ovary. The cyst was to big to dry out, so I had surgery done. They did a larparascopy on me, and a DNC with getting the polyps in my body, they also stuck a IUD ( merina ) inside of me which made me gain a lot of weight. Luckily, after that happen I was okay but after that the IUD popped out of my uterus wall because of being heavy weight. After being on so many different birth control pills I had no idea that the oral contraceptives causes discoloration in the skin. I was also on the Yaz, Microgestin FE, IUD, Depo prover shot, Prover pills and some other that I forgot. The abnormal bleeding stopped, and I stopped with taking the oral contraceptives. But it damaged the skin cells in my body, and I wanted to know how to get rid of it. I have been diagnoised with Ancanothis Nigerican which is related to my pcos. Do you have any suggestions of how to get rid of the dark spots on my neck? I never had it this bad but after using the tretinioin cream it made me break out in a rash, it itches. The itching in between my thighs went away but after I stopped using it. It gotten a lot worse in between my thighs. Any suggestions of what to use to get rid of the itchy, the hyper pigmentation in between my thighs? I have taking fish oil supplements, that has been helping a lot as well! Doctor: Hi, Thanks for the detailed information provided by you . The pigmentation around the neck, thighs , breast abdomen is Pseudo acanthosis nigricans.There is no damage of skin cells after taking contraceptives. It is related to obesity. So the best way to reduce this pigmentation is weight reduction. Creams like tretinoin may be used to reduce pigmentation .Tretinoin can cause hypersensitivity reaction in the skin in the form of itching and rashes as has happened in your case. You may stop applying tretinoin cream and apply a moderate steroid like momatosone for a few days, this will help in reducing redness and itching can be taken care of with an antihistaminic. You may continue taking fish oil supplements. After the rash disappears you may restart tretinoin gel 0.01 % for 2hours , slowly increasing the duration if no rashes appear Hope this helps you"
},
{
"id": 141528,
"tgt": "What is the treatment of vertigo attacks ?",
"src": "Patient: hello, I suffer from vertigo attcks with severe sickness and am taking Serc for this. I seem to be having more attacks lately and in between attacks have a very dizzy head most of the time. I do not know whether the dizziness is due to the vertigo or if the high blood pressure tablets that I take also are causing this.I have had these attacks tfor about 7 years now but the dizziness in between attacks is more recent. Should I be trying another treatment? Doctor: Hello and Welcome to \u2018Ask A Doctor\u2019 service. I have reviewed your query and here is my advice. Chronic vertigo can occur due to ear pathology. Drugs like Betahistine, Dimenhydrinate and Cinnarizine can be helpful when used in combination. Additional usage of Vitamin D and Ginkgo biloba may be tried. Vertigo exercises are also helpful. Regards, Dr N Kumar Neurologist"
},
{
"id": 96626,
"tgt": "Can feeling uncomfortable in chest and nose be caused by poisoning from Clorox?",
"src": "Patient: I poured Clorox down my sink both sides of sink . I have a bug problem that's why I did this. I live with my small dog. My window and front door is open. My friend said run cold water down the drains so I did. I also put Armen Hammer baking soda down drain. It's about 2 hrs now. Still smells. I'm worried. I feel funny in my chest and alittle cough nose feels funny. Do I have poisoning . Should I be worried ? My dog I'm worried. He seems fine. Doctor: Hello and welcome to HCM,Yes, bleach can be very irritating to the membranes of the nose and airways. If you can, run a fan near the bathroom, and stay away from the bathroom for a few hours.Don't pour anything further down the drain other than cold water (if necessary) as it might react with bleach, although at this point there probably is not a lot of bleach left in there except for what is in the air.If you continue to have discomfort or it is worsening, then it would be wise to go see a doctor.Otherwise, consider going for a walk with the dog and getting some fresh air right now, while a fan runs and helps dilute the bleach in the air.I hope this information helps."
},
{
"id": 174081,
"tgt": "What causes stomach ache?",
"src": "Patient: my child of 6 years is having frequent tummy aches and passes stool which relieves the pain and the stool has some mucus and the food that she ate recently. She had fever yesterday but no fever today. We thought she is recovering but the aches are back again. can we give her anafortan? or what else? . Doctor: HiWelcome to the HCMIn my opinion your child is suffering from a bacterial or parasitic infestation of the gut.It seems to be more like an intestinal amoebiasis.It is usually caused by ingestion of unhygeinic food or water. To support the diagnosis you can go for a stool examination and immunological test for amoebiasis. For management you should give her the following:1. Keep her well hydrated by regular oral rehydration therapy such as lemonade, Buttermilk, ORS etc.2. Start her on Ofloxacin- ornidazole syrup to treat the infection.3. For fever, you can give her anafortan.4. Also give her 2 doses of albendazole at 15 days interval.For further questions, do contact us"
},
{
"id": 33725,
"tgt": "Suggest treatment for dry cough and chest congestion",
"src": "Patient: i've developed a dry cough it hurts a little when i cough and a very slight dry throat. the worst part is that my lungs hurt and when i breathe my back aches. i also feel a shortness of breath. i feel pin pricks in my lungs. i've had back pain and chest congestion before, but this feels a bit different, it's constant and no matter what i do it doesn't seem to relent. i've stretched, had my son walk on my back, nothing takes the ache away. i've also been feeling a bit warm at times. no fever no cold. i do suffer from a bit of asthma and i've had bronchial pneumonia before. i'm also very sensitive to the cold. i've been feeling very fatigued. i can't seem to shake it off. it all started with a slight pressure in one particular part of my lung. this was a few moths ago. i didn't think much it, as i've felt my lungs congested before. it feels like it's where my brochial tube connects to my lung, right below the sternum. now it feels like it has grown and my other lung also feels the same. my ribs hurt and a little bit of my chest muscle and skin also feel sensitive Doctor: HI, thanks for using healthcare magicThe most common cause for persistent dry cough is post nasal drip. This can sometimes be silent.It can cause the slight dryness that you are experiencing.It most often occurs in response to an allergen.Treatment involves avoidance of the allergen, topical steroid nasal sprays (Eg flonase, rhinocort, nasonex, nasocort) and oral antihistamines.Other causes of persistent cough are: GERD (reflux) and asthma.I hope this helps"
},
{
"id": 82618,
"tgt": "How to treat lesions on face,body and scalp?",
"src": "Patient: Dear Dr I am a patient of PV for the last 3 years with hypothyroidism for the last 6 years. I am presently on 15mg Predesolone+100 mg Azithroprine+188mcg thyroxine+1 shellcal + Multivitamin+ Vitamin E + VIT D weekly. 3 years before when I was diagnosed PV I was given 32mg Methylpredesolone which was tapered from time to time over about a year and when completly stopped it relapsed after a fortnight. Again I was restarted with 48 mg Methylpredesolone which is now tapered to 12mg + 100mg Azithroprine (50mg morning& evening). At 8mg methylpresolone+ 100mg Azron my I did not get any lesions for about a month but all of sudden there were several lesions on face, body & scalp. Then I consulted other Doctor who administred me 140g IVIG. After IVIG my lesions were reduced, my skin health improved and itching disappeared altogather. After about a month of IVIG again 7-8 lesions appeared and was put on 30mg predesolone {instead of Methyl)+100mg Azron which is now tapered to 15mg Predesolone+100Azron. I am not getting any lesions now for about 2 weeks and my Dr is suggesting to taper it to 10 mg which I am doing from tomorrow. He is also suggesting 2 infusions of Retumbix 0.5g each over a gap 15 days each & thereafter 1 more infusion of IVIG ne month thereafter. Now I request you to please suggest the best course of treatment. I am 60 yrs old male with 5 -2 height & 72 kg body weight, with hypothyroidism. With regards skk Doctor: Dear SKK, its really a pressure upon you to gulp so much of medicine, which I would not like my patients to continue for such long, lifetime.You can't stop it either. I would rather suggest my own such patients to get support of pure herbals to gain some resistance against disease. To start with Turmeric, Holy basil leaves, Manjishtha, Neem powder, Amalaki and Guduchi alone would help you a lot. But may be its slower in action. But what did you gain after getting so many fastest acting medicine. Even if you have to take slow medicine for life, what's the harm when there's no side effect, no interaction with drugs. But you will find in few weeks what benefits you will gain in few months ahead.If you are in India, you get all the herbs at some Pansari shop i.e. herbal grocery shop. Get all these - clean, grind and mix in equal ratio. Takw with pure honey if not diabetic. Otherwise with water. We may also help you in getting these if you so desire. Globally you may get capsules of many cos containing extracts or powder of these herbs. I prefer Himalaya's pure herbs series.For any other query, pl write direct question. I want that you must start taking support of these herbs. In few weeks you will start realizing the benefits. These all will not only detoxification, cure but also raise your immunity."
},
{
"id": 155741,
"tgt": "What is the treatment for cancer?",
"src": "Patient: My mom 60 years old obese women had a car accident and she was in severe back pain. The back CT revealed there is nodule in the lung. So she had chest CT and the report says there is 8 mm nodule in each lower zone along with D12 compression fracture indicating lung and dorsal metastases. The question, the fracture is a result of cancer or oestoporosis or the accident. Is chemotherapy the treatment for her cancer? Doctor: Thanks for your question on HCM.Lung cancer is very notorious in spread. It can spread by both, blood and lymphnodes.Bone involvement is very common especially thoracic vertebrae. And bone metastases from lung cancer are usually osteolytic in nature. So they produce demineralisation and osteoporosis both. So these bones are highly prone to fracture even by slightest force.So your mother had osteolytic bone metastases in thoracic vertebrae and fracture after accident.Bone metastases made lung cancer stage 4 disease (inoperable). Only treatment option is palliative chemotherapy and nutritional support in the form of multivitamins, calcium and vitamin d."
},
{
"id": 8481,
"tgt": "How to treat dark spots and uneven skin surface?",
"src": "Patient: I m male 20,My dermatologist suggested me to use benzoyl peroxide gel bp for acne, and was very useful but now and now I m having dark spots and non even skin surface. So what should I use to remove those dark spots and have even skin surface and I m having oily skin too Doctor: Hello,Welcome to healthcare magic,I understand from your query that you are having marks left behind from acne.You could apply a gel containing 2-4% kojic acid at bedtime for 2-3 months. Apply a gel based sunscreen every 4 hours in the day time.You could also get glycolic acid based chemical peels done by a dermatologist.Hope this helps you.Take care."
},
{
"id": 84084,
"tgt": "What are the side effects of tenovate m cream?",
"src": "Patient: i have some fungal infection may b a ringworm. my doctor has prescribed tenovate m and i am using it. my skin has stopped itching and now it looks dry and scaly. Could you please share your experience with this cream. is this one of the stages of getting cured. thank u Doctor: Hello,I read carefully your query and understand your concern. Clobetasol cream is used to treat inflammation and itching from various\u00a0skin\u00a0conditions. It can cause dry skin as a side effect.I suggest to use a gentle moisturizer to help the dry skin. I also suggest to drink liquids.Hope my answer was helpful.If you have further queries feel free to contact me again.Kind regards! Dr.Dorina Gurabardhi General &Family Physician"
},
{
"id": 134742,
"tgt": "Suggest remedy for severe pain caused by knee replacement surgery",
"src": "Patient: My left leg knee replacement took place hardly month and half back. Presently I am going thro lot of pain. I also also have my back disc L4 and L1 L2 damage. TIred of eating Oxycrocent and other pain killers. Hot water salt bath helps temp. Can U. Suggests to avoid severe pain I go thro my leg and back pain. Night can not sleep et sleep disturbed Doctor: Hi Thanks for your query and Welcome to Healthcare Magic. I am Dr Akshay from Fortis Hospital, New Delhi.You have mentioned that you had a knee replacement surgery around 6 weeks back and even after that you are having a lot of pain.Normally pain after a knee replacement surgery should start settling 6-8 weeks from the date of surgery and should be a bare minimum at 3 months , and there should be no pain from 6 month onwards.But some patients can have pain for a slightly prolonged periods of time. Anyways if there is only pain, but no other s/s of inflammation or infection like Skin redness, increasing swelling, local warmth etc , then it can be managed by :- Ice packs for few more days- Change in the medications you are taking , maybe a combination of Tramadol + Paracetamol can be given if it is routinely used by your Orthopaedic surgeon.- Physical therapy can be modified according to your pain tolerance as aggresive physical therapy can sometimes increase pain after knee replacement surgery.Also you can initially limit your ambulation and other activities as per your pain tolerance, or as guided by your physical therapist.As far as your spine problem goes, i can advise you in detail if i have a detailed information about your problem including neurological examination and relevant MRI images and report.Do not hesitate to contact me if you need any further assistance. Thanks & RegardsDr Akshay Kumar Saxena"
},
{
"id": 43823,
"tgt": "Done semen analysis. Can my wife conceive?",
"src": "Patient: i have undergone a semen analysis and have the below results Count= 23.5 million Motility= 35% progressive motile,20% non progressive motile, 45% immotile Morphology= 70% normal, 30% abnormal Me and my wife is trying to have a child for almost a year now but were not having any luck please advise if base on this results am i more than capable to contribute for her to conceive? thanks! Doctor: Hello, Welcome to HCM, I am Dr. Das The sperm count is absolutely normal. Sperm motility is also within normal limit. Morphology is also within normal limit. So, your semen analysis report is totally normal. Reference - WHO 2010. Regards."
},
{
"id": 159867,
"tgt": "Am I suffering from tumor in stomach or cancer as I have experienced dehydration but the scan shows a normal report ?",
"src": "Patient: sir i have been taking excess doses of evion-600 ranging from 6-7 capsules daily and 7-8 capsules of b-complex daily from past 6 yrs........recently i had dehydration and the doctors confirred infection i nthe abdomen area then they asked for ultrasound but it was all okkk but the pain is still persistent in my stomach a slight one not very high intensity what is the reason do i have a tumor in my stomach or cancer ???................ravinder Doctor: hi welcome, hi ravinder if you are having tumor or cancer definitely that will seen as abnormality in scan. any way stop the excessive intake of vitamine e for the pain you have to explain in detail write to me we can make you better christyjoseph1@gmail.com thanking you"
},
{
"id": 13668,
"tgt": "What causes rashes on my elbow?",
"src": "Patient: I have a rash on my left arm near my elbow. Under the rash is muscle that feels very sore. It appears to look bruised, blue almost. It feels as though its hot & burning, doesn t really itch. When I took a picture and zoomed in I could see little blisters. Doctor: Hi, Perhaps it is shingles. The rash in shingles is red, blistery and associated with symptoms like aching pain and burning. It is usually distributed along a dermatome i.e along the distribution of a spinal nerve. I suggest you to kindly visit a dermatologist in your region for a confirmation of diagnosis. If it is indeed shingles, you would need oral antivirals like either Acyclovir and Valacyclovir for a week. You would also require an OTC painkiller for symptomatic relief from pain. Hope I have answered your query. Let me know if I can assist you further."
},
{
"id": 30168,
"tgt": "Suggest treatment for recurring MRSA",
"src": "Patient: I am a 42 year old female, I have been suffering with recurring MRSA for the past 3 and a half years. I am in the hospital every 3 months with out breaks, usually on my legs, but I have had them on my eyebrows, chin, armpit, stomach, and right now I have 4 on my right butt cheek an one large one on my left butt cheek. The left side I have developed severe sciatic pain, and was wondering how I can tell if the mrsa has gone into my sciatic nerve. I am on vancomyacin twice a day since last sunday. My doc has given me Lyric to see if it would stop the nerve pain but it has not. as well I am takiing naprisin sups for inflamation and that is not helping either. I just want to get better and get home, this is my 8th day in the hospital and I don t know when I am getting out. Doctor: As per yur history please check these drugs whether you are given or not -Clindamycin 300-450mg PO three times a day.TMP/SMX 1-2 DS tabs PO twice daily.Doxycycline 100mg PO twice daily.Minocycline 100mg PO twice daily.Linezolid 600mg PO twice daily.if not given tell your doctor to add it.Thanks"
},
{
"id": 109347,
"tgt": "Suggest remedy for severe pain in the lower back and right leg",
"src": "Patient: Hi Doctor, Am 25 yr old,i got sever pain in my lowerback and right leg (i.e)L4 & L5,I went to doctor,he asked me to take MRI,X-ray and finally asked me to do surgery on jan 2011.so i went under the surgery,was in bed rest for 2 months,after that if i do work continously,spinal cord where they operated is getting strained.so when can i come to normal without pain whatever i do? Doctor: helloIf i would be your treating doctor i would suggest you to go for panchkarma therapy ( kati basti)-for pain take tablet mahayogaraja guggulu 1 tab 2-3 times a day after food.- for this therapy & medicine i suggest you to consult doctor at ayurvedic hospital or panchkarma center as it is not given without prescription.Hope this will help you."
},
{
"id": 68791,
"tgt": "What causes lumps on the jawbone of a baby?",
"src": "Patient: Hi My newly born grandson has a lump on left side of jaw bone. It is hard and feels like bone. I dont think its painful and he is feeding well. He is 5 days old and we have just discovered it. The midwife said its probably nothing and to see the doctor. What could this be please Doctor: Hi.This can be a Dentigerous cyst or rare tumors. Needs X-ray to see what it is . Consult a Oral Surgeon or a Dentist specialized in Pediatric dentistry ."
},
{
"id": 62106,
"tgt": "What causes lump in stomach, green urine, back ache and queasiness?",
"src": "Patient: Hi iam 31 female for past 2weeks I ve got lump on my left hand side of my stomach its the size of a tennis ball and from saturday I ve noticed my urine is green getting stomach pains sometimes,getting a lot of back ache and noticed my stomach getting bigger also felt sick and quizzy,what can this be plz help Doctor: Hi,Dear,Thanks for your query to HCM.Studied your query in full depth of its details.Reviewed it in context of your health concerns.I understood your needs for your health concerns.Based On the facts,You seem to suffer from-Renal Lump with Backache with Bloating of your stomach,with green urine.Many causes need to be assessed.In this scenario, with limited data on the Renal lump,I would suggest your to have Second opinion from Surgeon / Urologist and get it evaluated and investigated for the possible causes in your case.I would suggest to get -urine micro/ KUB -X-ray abdomen/USG Abdomen/CT abdomen studies to assess your stomach lump.YOur doctor would reduce your sick feeling by -Painkillers like-Tab Meftal Spas / with plenty of fluids/ Tab Ciplox -500 mgh x 2 times per day x 14 days and would investigate the case and would treat accordingly,as per the need of your case.Hence I would suggest to take Second opinion from Surgeon / Urologist.Hope this reply would satisfy you and would help you to plan further treatment with your doctor there.Welcome any further query in this regard,which would be replied in next session.Wish you fast and early recovery.Good Day!!Dr.Savaskar M.N.Senior Surgical SpecialistM.S.Genl-CVTS"
},
{
"id": 99013,
"tgt": "Are headaches and cough considered allergy symptoms?",
"src": "Patient: hellomy name is RyanI'm 15and I'm not sure if I have allergy symptoms or if I have something else.On Wednesday night, what I think is my allergies started to kick in. I started to have a 3 part cough (cough, cough, a little pause and then another cough). I live in northern NJ and trees are blooming everywhere. Since I am allergic to pollen, dust, mold, and ragweed, I thought that this was definitely the problem. On Thursday, it got a little worse, so I closed my windows and instead put on the A/C even though it was quite cool out. However, the cough stayed the same. I drank (and still am) lots of water and had a cough drop every other hour or two. Friday, I thought was a bit better since I went down to a 2 part cough, but the occasional cough would hurt my throat. Shockingly, I did not have a runny nose, headache, anything else, but the cough. However, I changed my mind on Saturday what I thought. In the morning, I went to help out at an outdoor fundraiser and there were trees everywhere. I knew this would be bad and went in and out consistently. Then, I started to get headaches and back aches, but no usual runny nose. Afterward, I went to my uncle's house and I felt much better. I thought that maybe the different trees had an effect on me, but as soon as we got in the car, it started up again. I took a cough medicine and slept, but it didn't do anything. I then saw I had a fever of 102. I took medicine and went back to sleep. This morning (Sunday), the fever was \"gone\" to me and I just had the cough again. When I took my temperature afterward, I was at 99 something. Now, I am at normal temperature, have a cough, now a slight runny nose, and every once in a while get a chill.After this too long story, is it allergies or something else? What should I do? Doctor: Surely it is allergic symptoms. It is not necessary that all chest allergy is associated with nasal allergy.Repeated coughing induce headache.mile fever is also there during allergic attack."
},
{
"id": 207881,
"tgt": "What causes panic attacks and anxiety during public speaking and seminars?",
"src": "Patient: Hi, I have problems with anxiety with regards to public speaking and giving presentations. The problems are physical as I shake quite a lot, to the point where others notice and comment on it. It's becoming a problem as I am in university and my course requires me to give presentations on a regular basis. Is there anything that could be prescribed to me to treat my physical symptoms or is a doctor unlikely to do this? Doctor: DearWe understand your concernsI went through your details. I suggest you not to worry much. I think you are refusing to learn from your experiences. You are pointing to your shaking while doing presentation after you experienced it. You must understand that you also experienced you standing there and delivering. that should build your confidence. The only remedy available is \"do it again and again\". Your anxiety should be eased.If you require more of my help in this aspect, Please post a direct question to me in this website. Make sure that you include every minute details possible. I shall prescribe the needed psychotherapy techniques which should help you cure your condition further.Hope this answers your query. Available for further clarifications.Good luck."
},
{
"id": 120315,
"tgt": "What is causing my left knee pain?",
"src": "Patient: I am a 29/F, 5 3 and about 170lbs. I have given birth to two children with no complications and have no significant medical history. For the last 6 months whenever I sit with my legs crossed for an extended period of time and I go to straighten my legs, I have significant pain and stiffness in my left knee. What could be causing this? Doctor: Hello,I read carefully your query and understand your concern. Stiffness of the knee can result from poor posture.I suggest :-Switch sitting positions often.-Avoid crossing your legs or ankles.-Stand up frequently.-Take short walks.Hope my answer was helpful.If you have further queries feel free to contact me again.Kind regards! Dr.Dorina Gurabardhi General &Family Physician"
},
{
"id": 94613,
"tgt": "Having cyst on the left ovary, horrific pain in the right side of the abdomen, not appendicitis. Reason?",
"src": "Patient: Hi there. I m 38 years old. Very active and fit. Tubal ligation and abblasion (spelling?!) done at age 33. Two full term pregnancies, youngest child is 14. Two years ago I had a cyst on left ovary found with vaginal ultrasound which burst without complications. Last week I saw my OB and he found another on same left ovary. He said not to worry but to call if complications. Two nights ago I started having horrific pain on right side for about ten hours which woke me atb1am. Worst pain I ve had in a while. Went to ER as it was in the right location for appendicitis and I was queasy. A CT scan with contrast showed appendix fine. The Dr. Said I had an enormous amount of fecal matter and that i was constipated. This was not constipation. I had been having bowel movements. Now I m spotting and it does in fact feel a bit like menses. Do CT scans pick up ovarian cysts or is that something that could have been missed? Thanks forbyourbhelp. Doctor: Hi,Welcome to healthcaremagic.From the symptoms it is most likely a gynecological condition giving rise to acute pain . It will be better to get a check up with gynecologist and get an u s g done for proper diagnosis and treatment . Hope you be all right soon ."
},
{
"id": 58364,
"tgt": "Pancreas stone, prescribed oxycontin 20mg, oxycodone 15mg, braked meds and given exalgo 8mg, pain. Proper procedure?",
"src": "Patient: i passed a stone though my pancrease,i have been in pain management four 15 years .had a morphine pump put in so i could keep working.they also give me oxycontin 20mg three times a day and oxycodone 15mg 6 times a day.the doctors all quit,now i see i nurse and the first thing she did was cut my breakthough meds in half gave me exalgo 8mg twice a day. i ended up in hospital with pain after 15 years.they have two offices and the owner is the only doctor there.before i left i asked her what she is treating me for and she told me she would have to look it up.asked for doctor has no openings.have to see nurse is this a proper proceedure. Doctor: Hi,Thanks for writing in.You need to undergo MRCP to evaluate the exact status of the pancreas. Depending upon the findings a treatment plan needs to be formulated.Regards"
},
{
"id": 92372,
"tgt": "What else besides bifilac and nexpro can help in curing stomach bloating and gastric issues?",
"src": "Patient: Hi, may I answer your health queries? Ple i am suffering from stomach bloating ..gas problem since 10 yrs ...since three months i am taking bifilac hp...nexpro 40..and dulane 20...but recently i devoloped a sort of spreding pain under left rib ..near to stomach..still there are gas symptoms and lot of discomfort in stomach ..can u advice any further treatment Doctor: Hi. Gas problem for 10 years need to be investigated. Many times it can be because of Gall Bladder stones. Now the associated pain you are describing needs further investigations under Doctor's supervision and appropriate treatment."
},
{
"id": 120324,
"tgt": "What causes red bump on leg after using T-Zone vibrator?",
"src": "Patient: I went jogging which I do all the time, but then after that I used a different kind of a machine. A T-Zone vibrator for 10 minutes. After which my calf muscle in my leg tensed up and got very hard. I tried to stretch it out. It didn t work. It was not very painful. The next morning a bump the size of a quarter rose on my leg in the same place where the muscle was. The next day, the bump went down, but left a red spot on my leg. Thank you for your help, Sue Doctor: Hello,I read carefully your query and understand your concern. The symptoms seem to be related to a muscle strain. I suggest using a muscle relaxant such as Baclofen three times a day. I also suggest using magnesium supplement for muscle relaxation. Warm compresses can also be helpful. Hope my answer was helpful.If you have further queries feel free to contact me again.Kind regards! Dr.Dorina Gurabardhi General &Family I Physician"
},
{
"id": 91967,
"tgt": "Can sciatica cause abdominal pain, nausea, and constipation?",
"src": "Patient: Hi,Lately my sciatica has been more severe and when it is at it's worst I been having abdominal pain in the area of my appendix. During this time I am nauseous and become constipated that seems to last for 2-3 days at a time. I've been to the Emergency dept. and my GP trying to figure this out because it has been going on for the last 3 months.Can my sciatica be causing these other symptoms? Thanks, Bruce Doctor: Hi,Sciatica cannot directly cause these abdominal symptoms. If very severe pain due to autonomic nervous system imbalance where in you try to resist pain there may be chance of variation in bowel movement, constipation.It is very difficult to tell the cause without complete examination. Since abdominal pain in appendix area, nausea with constipation I advice you to get done USG abdomen and pelvis once to rule out appendicitis or any bowel obstruction. For pain you can take NSAID like ibuprofen and get evaluated for cause of sciatica and get treated for it. This may relieve all your symptoms.Regards"
},
{
"id": 97120,
"tgt": "Suggest treatment for coin removal",
"src": "Patient: I just swallowed a quarter! Will I be okay? It went down okay, but now it hurts in my chest. Kind of on the right side and middle? It was on the bed and i was laying on the floor, and my sister jumped on the bed and it went down my throat. I drank a whole bunch of water. Also I m a 13 year old girl if that helps. But I m big for my age, I m 5 foot 9 and 120 lbs. Doctor: Hi,If you are sure you swallowed it then you do not need to go to the ER. Continue to drink plenty of water. The pain in your chest will be caused by the edges of the coin causing a small amount of trauma to your esophagus as it when down. This is not serious and will improve. Take paracetamol for any pain. You may wish to eat a soft diet for the next day.Regards,Dr K A Pottinger"
},
{
"id": 179646,
"tgt": "Does augmentin cause elevated blood platelets?",
"src": "Patient: My 3.5 years daughter (weight 16.5Kg) has dengue fever. Platelet count is 2.1 lakh. how often should I test her platelet count. She has infection in her right leg s mother toe and is on antibiotic too (Syp Augmentin DDS 3 ml dose for 5 days) Will this antibiotic have any impact on her platelet? Doctor: Hi Platelet count in dengue fever is usually low (less than 1.5 lakh ) . I think 2.1 lakh count is OK if her condition is other wise well . No augmentin will not have any effect on plateket count. Regarding testing for platelet it dependents on condition of baby .thanks"
},
{
"id": 114216,
"tgt": "My sister is suffering from back pain and stomach ache. What could be the possibilities?",
"src": "Patient: My sister has had a back pain last month, without any specific treatment because she insist on not having an X-Ray. The pain has eases, but she is now suffering with stomach ache and diarrhea . She also said that her tummy skin aches , although she is not vomiting. My sister is 30 years old, and she live in another country. Could anyone please advise on the possibilities of the above mentioned symptoms? Could the stomach ache be in anyway related to the back pain? Thanks in adv. for answering the question. Doctor: hi; back pain is common in people around 30 but that was different .now what your sister has is stomach infection;as you said she is not vomiting still she may be having amoebic dysentery where there is complaint of loose stools with mucus & stomach pain.she will require antibiotic for her please consult a doctor. thanks"
},
{
"id": 92432,
"tgt": "How to overcome the sharp pain in lower left abdomen and in buttocks?",
"src": "Patient: Hi I'm having left lower abdomen pain, very sharp and painful. I'm also experiencing pain in my buttocks. The pain comes and goes, often in the middle of the night or earlier morning. Please help my doctor has done MRI's Cat scans, ultra sounds. I need help please the pain is killing me:(( Doctor: Hello!Thank you for the query.If you are male, such pain can be caused by prostate inflammation. The pain is very sharp, can be present in the rectum area and lower abdomen. Can come and go suddenly. Sometimes appears during urinating.Lower left abdominal pain can be also caused by diverticulitis. This is possible if you have had chronic constipation. The pain can aggravate while passing the stool. Colonoscopy is necessary to diagnose it. If you are older than 50, colon cancer should be also ruled out. To do that, colonoscopy is also necessary.Hope this will help.Regards."
},
{
"id": 79488,
"tgt": "What causes constant pains in left side of chest?",
"src": "Patient: Hlo I am 21 year old and I am suffering from a pain that is on my left side of my chest . When I first got that pain I feel like burning like sensation on my left side of chest and also a pain that sometimes was on right side of chest (little), and also the pain on the back, one day I feel that my left arm was also involved then I concerned a doctor he gave me some medicine and I feel that I am recovering but after few days I feel that when I strech my neck i feel the same pain on my left side of chest and in the arm pit I searched on net what causes this type of pain and that came to many results and the most dangerous result which was also the cause of chest pain was cancer now I thought all the day about this and pray to god that it should not be cancer so I am in tension I also had ECG which came normal so plz tell me is it cancer or something Doctor: Thanks for your question on Health Care Magic. I can understand your concern. No need to worry for cancer as cancer is extremely uncommon at your age of 21 years. Your ecg is also normal, so no need to worry for cardiac disease as well. In my opinion, you are mostly having stress and anxiety related symptoms. So better to consult psychiatrist and get done counseling sessions. Try to identify stressor in your life and start working on its solution. You may need anxiolytic drugs too. So avoid stress and tension, be relax and calm. You are not having cancer or cardiac diseases. Don't worry, you will be alright. Hope I have solved your query. Wish you good health. Thanks."
},
{
"id": 145566,
"tgt": "Suggest treatment for cervical spine injury",
"src": "Patient: i have been diagnose with cervical spine injury specifically;compression fracture on c5 for five years ago.since then, i have been going through a difficult to have a deep breath,tight back,a small meal will bloat my stomach,difficult to urinate,bowel movement,chest pain and above all, difficult to sit still what is going on docs, Doctor: Hi,Thanks for writing in.A cervical injury causing compression of C 5 vertebra might cause significant pain and discomfort. It is important to get the affected vertebra stabilized and pressure on nerve roots decompressed. In a vertebra that is compressed, it can be stabilized by screws and plates. Please check with your doctor if your compression of C 5 vertebra requires surgical treatment.There are many nerves originating at various levels in the cervical spine and there might be irritation of nerves that requires attention.The bloating sensation, bowel discomfort, chest pain and inability to sit still might be treated with medication and this also needs to be evaluated by doing a MRI screening of whole spine. Each symptoms might be due to certain nerves getting irritated at different levels. Please do not worry."
},
{
"id": 107784,
"tgt": "Suggest treatment for stress and lower back pain",
"src": "Patient: I have been under alot of stress this passed year both at home and work. I have been having a loose yellow stool for at least 6 months and I also have lower back pain. I am eating properly but just feel tired at the end of the day. Not sure why I have these stools. Doctor: Since you are at stress and pain both and having pain for the last six months, it is prudent you get your BP checked and also microscopic stool examination. You haven't mentioned sleep pattern too. It could be disturbed.For the time being please try to de stress by deep breathing and meditation twice a day. May start for 5 minutes initially and raise to 15 minutes in a week.Ayurveda medicine, safe n side effect less and no counteraction with allopathy med, if started too, would bring drastic positive changes along with deep breathing and meditation.Brahmi capsules, Ashwagandha capsules each 1 bid for destressing, Tab Rumalya Forte 1 bid for pain and Tab Diarex 1 TDS for lose motions. May take Lavanbhaskar Ayurveda powder 1 tsp dissolved in buttermilk will be beneficial for controlling stools and strengthen intestines.Hope you get these and it helps."
},
{
"id": 5356,
"tgt": "Trying to conceive, have cramps after ovulation. Normal?",
"src": "Patient: I've been trying to conceive for 3 months now. This month a couple days after ovulation which was a week ago I started getting cramps, normal period like cramps but now it's moved up to both sides about equal with my belly button. This morning it was rather sharp and woke me up. Should I be worried or is something normal? Doctor: Hi,Thanks for the query.If you had unprotected intercourse around the time of ovulation, then there is possibility of pregnancy.And the pains you are having could be due to implantation etc.In the absence of pregnancy the pains can be due to hormonal changes or there could be some possibility of infection.And now it is early to comment about the pregnancy.So you wait till your expected date of periods and if you miss the periods go for pregnancy test once. Start taking folic acid tablets from now-onwards.For more details you can ask me through: http://www.healthcaremagic.com/doctors/dr-sree-gouri-sr/63429Take care."
},
{
"id": 5176,
"tgt": "Delayed second menstrual cycle after having unprotected sex. Any chances of pregnancy?",
"src": "Patient: hi,i had a sex with my gf on 23 april n gave her ipill n she took it within 7-8 hrs..den on 25 i again had a sex bt dat time i didnt gave her the pill n we were also not dat the sperm has ejected inside or not..later on 28-29 she got her menstural cycle bt aftr dat til date she didnt hav got her scnd cycle..expcted date was 2 days agois der anythng to worry..plss help me Doctor: Hi and thanks for using this panel.If she has seen her menses after this unprotected sex episodes, then it is unlikely that she is again pregnant. However, have you had any unprotected sex after her last menses?Doing a rapid pregnancy test is the standard of care when you have not seen your menses. This will give an instant answer. I will not be particularly worried at this time because menses may come earlier or later than expected. But if it should delay for more than a week, then please run a rapid test.If you are afraid of unwanted pregnancy, i suggest you both meet and talk with your physician on what other options are available if you don't want to use protection during sex. Hope this helps...feel free to return to us.We wish you good health"
},
{
"id": 12695,
"tgt": "Suggest treatment for pruritic urticarial papules & plaques",
"src": "Patient: Hello! I suffered with eczema and with what I believe was PUPPs (Pruritic Urticarial Papules and Plaques of Pregnancy) when I was pregnant with my son almost 5 years ago. It is recurring now even if I am not pregnant. I'm not sure of the cause but it started around 2 weeks ago and is getting worse, it's starting to spread all-over my body like before. What can I take/do to make this go away? I would like to stay away from steroids and antibiotics 'cause I used topical I think it is not helping anymore but instead making it worse. Doctor: Hello, I would recommend you to apply moisturizing cream containing white soft paraffin on the entire body 2-3 times in a day and take antihistaminic like tablet cetrizine 10 mg twice daily. Hope I have answered your query. Let me know if I can assist you further. Take care Regards, Dr. Asmeet Kaur Sawhney"
},
{
"id": 27860,
"tgt": "Suggest treatment for rapid heart rate",
"src": "Patient: My girlfriend is running a 111 heart rate. She is a diabetic and has a doctor appointment for the 17th and an appointment with cardio on the 19th. Should we be concerned about her waiting for the appointments with her heart rate running so high? Also has had a few chest pains on occasion but they go away. Doctor: hello,I have gone through your question.Thanks for using HCM.As such i need more information on symptoms of your girl friend.But if she has chest pain and heart rate of 111,you should get her ECG as early as possible,because in diabetics, heart disease may be deceptive and may have milder symptoms with high heart rate.so i suggest you to go for early check up,however it may turn out to be minor condition,but first we must rule out heart disease.My best wishes.Dr.Teli,MD"
},
{
"id": 15397,
"tgt": "Has periodic spells of skin sores, come as blisters, itch badly. What could be the cause ?",
"src": "Patient: My brother-in-law has perioditic spells of skin sores. They come on as a blister and change to a quarter size-3 inch sore overnight. They itch insanely and stay for awhile then go away. He feels bad physically beside the discomfort. Has been to 10 dr and noone has an answer. His health is deteriorating. What could be the cause? I should have known this is not what it seems to be. Just forget it, he is going to more doctors himself. Doctor: Hello i think it is pemphigus vulgaris it is autoimmune blistering disorder.it causes itching which causes recurrent blisters.it can be cured by oral antihistamine.antibiotic.steriods and application of gentian violet on the blisters and lobate gn ointment on the sores.if not subsided or recurrences are more go for dexamethasone pulse therapy which consists of dexamethazone inj about 12 vials for 3 continous days along with cyclophospamide monthly once for one year"
},
{
"id": 201018,
"tgt": "What causes foul smelling urine and white discharge?",
"src": "Patient: hi doctor myself yogesh from India, I have masturbating from 13 year of age. now I am 25 , I ejaculate quickly and when go to pee it smells very bad even I don t smoke, drink or had any sexual intercourse. and also when I am urinating some white stuff come off my penis.... please tell me whether I am facing any health problems and please do give remedies to it... Doctor: Thanks for asking in healthcaremagic forumIn short: Urinary tract infection can cause thisExplanation: Masturbating and this is not related. May be you are having a urinary tract infection in the form of urethritis/prostatitis and it may be your urethral discharge what you are saying about white stuff. So, I would like to suggest you to go for a doctor and get yourself and your urine examined for further investigation and treatment. Good luck"
},
{
"id": 141565,
"tgt": "What does my CT scan of the brain suggest?",
"src": "Patient: my ct scan of the brain ? ischaemic focus in parietal lobe i don't have any known symptoms of stroke yet except maybe memory recall what does this mean for me long term does it mean a stroke is eminent for me i can't have mri for few months yet public thanks Doctor: Hello and Welcome to \u2018Ask A Doctor\u2019 service. I have reviewed your query and here is my advice. Ischemic changes can be due to age, diabetes, hypertension, stroke or vitamin deficiencies. MRI of brain and blood investigations are required. Hope you found the answer helpful. Let me know if I can assist you further."
},
{
"id": 106139,
"tgt": "How do you treat inflamation of the sclera and a swollen cornea ?",
"src": "Patient: How do you treat inflamation of the sclera and a swollen cornea? My eye doctor has put me on a steriod eye drop , but I still have some inflamation and my left upper eyelid is swollen too.I suffer from allergies and am experiencing sinus pressure , my upper left forhead and my lower cheek, below left eye area, too! Doctor: hi welcome, for the allergies the better treatment can provide by either ayurveda or homeopathy is better. you consult any one belongs to this at your area ,he will make you better being ayurveda doctor i prefer ayurveda contact christyjoseph1@gmail.com"
},
{
"id": 113932,
"tgt": "Can having a fall cause anal bleeding combined with back pain ?",
"src": "Patient: I am a 16 year old female and have yet to tell my parents this, but while fishing withy mother on a muddy slope, I slipped and went down HARD landing on a large rock right on my bum and back. A few days go by and no anal bleeding just minor back pains. Now, the back pains are gone, but every time I go number two it s always bloody. Is this a problem, or should I just wait it out? Doctor: Hi welcome to healthcare magic Before the injury, everything was normal... but after injury bleeding from rectum suggestive of injury to rectum and anal canal... do not neglect , inform your parents and consult a general surgeon...."
},
{
"id": 142262,
"tgt": "What causes small dent on the head with addisons disease?",
"src": "Patient: i have a small dent located to the left of the crown on my head. it has been there for approximately a month and has gotten deaper/bigger. it is sensitive. I had an xray and the nurse practitioner told me that the X ray showed no abnormal lesions. I did Have some cortisone injections into what were some ingrown hairs around the same area. also some other information I recently in the past 2 months have started a very low carb diet I also have Addison s disease and take replacement steroids daily Doctor: Hello!Welcome on Healthcaremagic!I would explain that this small dent is related to the changes in bone structure (also called bone remodulation), due to steroid replacement therapy. It does not indicate any serious medical condition!I would just recommend checking calcium and vitamin D plasma levels. Hope you will find this answer helpful!Wishing all the best, Dr. Aida"
},
{
"id": 151348,
"tgt": "Severe pain in the back. Difficulty inn walking and coughing. Prescribed with antacid and diclofenac. Pain radiating to legs. Is this sciatica pain?",
"src": "Patient: hi there i been having a massive pain starting from the lower lumbo saccral region i cant sit for long since sunday night all of a sudden. neither i can walk properly and micturation is very difficult including coughing. i have seen a surgeon he has given me antacid and diclofenac twice daily...the pain has subsidized as off now till the effect of painkillers but if i stop taking them the pain resurfaces back again although not like the one i had 3 days before but its like a chronic pain and it goes down the legs?has been advised a week of complete bed rest . just want to know is it a sciatica pain? Doctor: Hi, Thanks for posting your query. You are most likely suffering from prolapsed intervertebral disc or cauda conus syndrome. Do you have radiating pain in lower limbs/ weakness in lower limbs? You should consult with orthopedic surgeon/ neurosurgeon and should go for x ray lumbo-sacral spine as preliminary investigations. You should also go for CT/ MRI scan lumbo-sacral spine for better clarification. You should do strict bed rest. You should take pain killers with serratiopeptidase and chymoral forte for reducing swelling. You should stop sitting & lying down on floor. If not treatable with medicines then you should go for surgery. Take care, Dr. Mayank Bhargava"
},
{
"id": 166794,
"tgt": "Suggest treatment for vomiting and skin rash in a child",
"src": "Patient: My 5 year old daughter complained she wasn t feeling good at dinner, then threw up before bed. She had a rash on her inner thighs and arms. We gave her children s benedryl. Her temp was 95 but we thought we just had a bad thermometer. We just checked on her and she threw up in her sleep and feels very cold. Any suggestions? Doctor: Hi,By what you say I feel that your daughter might be going through a viral illness. Unless she is extremely lethargic or having severe diarrhea with blood or having altered sensorium, you need not worry.Hope I have answered your query. Let me know if I can assist you further. Regards,Dr. Sumanth"
},
{
"id": 147861,
"tgt": "Suggest remedy for a teenager with constant migraine and seizures with being on topomax",
"src": "Patient: hello, My daughter is 17 and has suffered with migraine type headaches seems all her life. she has been to many doctors and has been on various medicaitons. a couple years ago she did start to have seizures and is on topomax for those. has had 3. anyway she has had a continuous headache for the last 3 weeks. medication no help. i looked online researching out types of headaches and the one that most closely resembles what she has is occipital neuralgia. she has had mri s, eeg no evidence of anything that would cause these. I want to find a physician to find the correct diagnosis and treatment for her. kids should not have to suffer like this im wondering should i suggest to her neuro botox injections? chiropractic intervention? i am at wits end here and need guidance. thank you , sheila Doctor: Hi, Welcome to Health care magic forum. It appears to be the tension head ache, or head ache due to refractive error of the eyes. As there are seizures, it may be secondary to the epilepsy, or there may be some underlying cause like T.B, etc. I advise you to consult a neurologist for diagnosis and treatment. She may need to have C.T. scan with contrast. I usually prescribe to my patient with such symptoms alprazolam,ranitidine, valporic acid, and paracetamol when ever required,till the diagnosis is established. Wishing for a quick and complete recovery. Thank you."
},
{
"id": 122289,
"tgt": "How can one cure an air pocket under the shoulder blade?",
"src": "Patient: I feel like I have a air pocket under my left shoulder blade, yet I tilt my head I feel pain @ that spot and tried lifting a gallon of milk with my right hand and it also affected the pain. How do I remedy this issue if that is what it my be...just an airpocket? Doctor: Hello, It could be a ligament or tendon related problem. Consult an orthopedic and plan for an MRI scan. Hope I have answered your query. Let me know if I can assist you further. Wishing you all the best. Regards, Dr. Shinas Hussain, General & Family Physician"
},
{
"id": 26361,
"tgt": "Could heart palpitations be due to gastric problems?",
"src": "Patient: Hi. I am a healthy 32 yr old female and I have just been hospitalized for severe dehydration after a terrible bout of gastro. I have since been battling with bad heart palpitations. Is this because of the gastro or should I return to the doctor? Is there anything I can do to stop it? Thank you Doctor: Dear Ms,Palpitations can be caused due to dehydration and electrolyte disbalance after it.I would advise you to take much fluids, eat more fresh fruits and vegetables, and palpitations will disappear gradually.Wishing you good healthIn case of further questions don't hesitate to askRegards,"
},
{
"id": 112450,
"tgt": "Back pain. Epidural nerve block. Lumbar puncture. When will I feel better?",
"src": "Patient: I am going for a series of tests for my back. They sent me a paper saying guidelines for nerve block epidural block causl block sacro-iliac block facet block lumbar puncture do they do all these exams all together how long does it usaully take what is the procedure ? i am getting a little scared . can you pls inlighten me of what i should expect. ty Doctor: Hi, Thanks for using HCM.All these blocks are given to block nerve conduction so that sensation are lost in lower limbs.I feel your doctor may conduct any one of the block not all at a time, they would have just explained different options available. Depending on situation they may decide one procedure.Lumbar puncture can also be done along with block in a same prick.Need not have to worry you can discuss with your doctor about which procedure he is going to perform and indication for it.Hope this answered your question. Consult your doctor for further management.Feel free to ask me if you have any further queries or doubts and I shall be glad to help you out.Wish you good health. Take care.RegardsDr. Vidya"
},
{
"id": 11715,
"tgt": "Have some ache scar marks, blackheads ,dark pigmentation on my face. Using lumnaskin cream. Safe remedy?",
"src": "Patient: i am 26yrs old grl and i hve some ache scar marks ,blackheads ,dark pigmentation on my face.my face skin looks vry dull n loose.plz tell me lumnaskin creame is effective for my skin problem?can i use this creame without any specialist precription.?its sideeffects or not. Is lumnaskin is better to improve skin complexion ? Doctor: Hi, welcome to HCM.You are having acne scars and hypermelanosis on face.Do have active acne lesions?If yes, apply clindamycine or adapalene gel at night on lesions.If no active acne lesions, you can uselumnaskin cream. It is herbal product containing hyaluronic acid , grape seed extract, swiss plants extracts and biowhite serum. It is not irritant as it does not contain hydroquinone. However , one can not say that you will get perfect result.To get perfect result, you apply kojic acid and vitamin C containing cream.Take vitamin C, glutathion and nicotinic acid in proper dose.Have patience, avoid tension.I hope you will be improved.Regards,Dr. Ilyas Patel MD,DVD"
},
{
"id": 217070,
"tgt": "Suggest treatment for joint pain in body",
"src": "Patient: My mother aged about 64 years facing pain in joints, wrist, back shoulder, fingers and other parts since last one year. i have consulted with neurologists in my area near by asansol but after a long treatment the same is exists.please give me proper guidance. Doctor: Should consult a rheumatologist. ..RA factor. ..ESR ...and xray just sh be done.....by the time u can start analgesic like PCM 500 MG SOS"
},
{
"id": 139625,
"tgt": "Suggest treatment for cellebellar vermis lacunar",
"src": "Patient: my father who has been daibetic past 18 yrs always under control .he also has parkinson s from last 12 yrs .He had dizziness and was unable to walk ,Doctors said that there is a a small clot at vermis it is called as Cellebellar vermis lacunar acute infarct .Doppler test showed 50% blokage in the nerve of his neck . He was unable to walk for 2 days but now he does with support .Will he be able to walk /balance and will that clot improve with medication. Your help is appreciated. Doctor: Hello, Physical therapy is necessary for further improvement after brain infarction. Blood pressure should be maintained under control besides the diabetes. Blood thinners also should be used to prevent recurrence of infarction and to reduce the clot.Hope I have answered your query. Let me know if I can assist you further.Regards,Dr. Erion Spaho, Neurologist, Surgical"
},
{
"id": 119210,
"tgt": "Suffering from hemophilia type a. Had abdominal pain, vomitting. Given bolus of Novoseven medicine. Suggestions?",
"src": "Patient: Hello Sir/ Madam I am Ravi Patel from Indore, Madhya Pradesh, India. I am facing a very serious problem in my family. My uncle Mr. Paresh Patel is Hemophilia Patient having \u201cHemophilia Type A\u201d. Presently he is in very serious condition and so I need your help or opinion. Here with giving brief history of Mr. Paresh Patel (Age 43) by his Doctor. On 20th morning at around 2.00 AM he had a pain in abdomen and vomiting .he was infused with 2000 i.u of AHF immediately.after 6 Hrs. pain is increased and he was uncomfortable. Radio logically it was diagnosed with retro peritoneal hemetoma. he was infused regularly doses of factor at regular interval.but coagulation was not achieved for 24 Hrs.by the time he had a bleed in lungs and he was intuubated and supported with respirator. His Haemoglobin was falling and maitained by RCC transfusions same evening we received the report of presence of inhibitors.started FEIBA and his renal function was impaired and creatinine was moree than 7.his weight is 60 Kg.and he is maintain on FEIBA accordingly. on 2nd December he had a nasal bleed which was controlled by FEIBA. Today his creatienine 2.5mg/dl , APTT 96 sec. and Hemoglobin 9.2 Again he had same nasal bleed at around 5 AM.and he is applied in neutral nasal packs. at present his blood pressure is maintained and hemoglobin is maintained and heis given first bolus of Novo seven 5.2 mg Infusion Details FACTOR Date Time Factor- VIII I.U 19.11.2012 12;30 am 2000 2011.2012 09;00 am 2000 20.11.2012 08;00 pm 3000 21;11;2012 11;30 pm 2000 21.11.2012 09;00 am 750 21.11.2012 11;00 am 716 21.11.2012 03;30 am 1074 21.11.2012 06;15 am 716 21.11.2012 08;30pm 716 22.11.2012 03;30 am 2000 22.11.2012 07;30 am 2000 21.11.2012 11;30 pm 2000 22.11.2012 03.30 am 2000 22.11.2012 07;30 am 2000 22.11.2012 11;30 am 2000 23.11.2012 03;30 am 2000 23.11.2012 7;30 am 2000 23.11.2012 10.00 am 2000 FEIBA Date Time FEIBA I.U 23.11.2012 10;15 am 6000 23.11.2012 11;00 pm 3000 24.11.2012 06;00 am 3000 24.11.2012 04;00 pm 6000 25.11.2012 04;00 am 6000 25.11.2012 01;30 pm 6000 26.11.2012 01.30 am 6000 26.11.2012 03.30 pm 6000 27.11.2012 03.00 am 6000 27.11.2012 03 00.pm 5736 28.11.2012 03.00 am 4500 28.11.2012 05.00 pm 4500 29.11.2012 5.00 am 3000 29.11.2012 5.00 pm 3000 30.11.2012 3.00 am 3000 30.11.2012 3.00 pm 3000 1.12.2012 4.oo am 3000 1.12.2012 7.30 pm 3000 2.12.2012 10.00 am 4500 2.12.2012 10.00pm 4500 3.12.2012 10.00 am 4500 3.12.2012 10.00 pm 4500 4.12.2012 4..00 am 3000 4.12.2012 5.00 am 3000 NOVO SEVEN 5.4 mg on 4 dec.2012 pl. opine thanks. Mo no. 91 0000 Doctor: start porcine factor VIII or recombinant factor VIIa or start immune tolerance therapy by infusing large dose (25-100 iu/kg/day) of factor VIII daily/alternate days for months till the inhibitor level become un detectable, it can be combined with immunosuppressive agents, keep on doing factor assay to monitor the treatment and to prevent any further bleeding episode and also keep on checking inhibitor status whether its concentration is decreasing or not."
},
{
"id": 99981,
"tgt": "What causes burning sensation in nose and black phlegm?",
"src": "Patient: Hi I am currently deployed overseas as a civilian contractor. The pollution in the air is extreme. I got what would consider extremely ill my lungs burned my nose was runing constantly tong was numb i was hacking green and black phelgm. I was not wearing a mask. I have been here on and off for several years. When I go home I want to see and dr and see if I have caused any permanent damage to my lungs but am not sure what type of Dr. I should see or what to explain Doctor: Hello Whatever you will inhale ( if polluted ) that if not fresh air will intermingled with mucus ( a long procedure due to phagocytic activities) and will come out in the form of phlegm . If polluted material is some allergen they will cause bronchoconstriction ( if bacteria then infection and phlegm i.e black or green or yellow depending on the type of allergen or bacteria).In your case you should consult a \"PULMONOLOGIST \" or chest specialist .Good luck."
},
{
"id": 208491,
"tgt": "Suggest remedy for retaining memory",
"src": "Patient: hi ....my height is 5-11,weight is around 72, my age is 17 and my queries are I am unable to memorise properly i am not able to recapture things exactly ... althought i was a bright student soring 80 s and have come down suddenly to 60 ... i feel very pathetic to face people outside please guide me Doctor: I appreciate your concern and asking on HCM.I would suggest the following:1. It is important to look into the cause of the academic decline. If there are some associated stress factors or emotional disturbance that needs to be addressed.2. For memory improvement you can take the following measures.a. Improve your attention. Only those things go to memory to which full attention is given.b. Rehearse the information for some time to get it into short term and then long term memory.c. Study in short spans.d. Store information in chunks and memorize the main concept first and attach the associated concepts.e. Use example, flow charts, diagrams to improve the retention of the particular concept.f. Use mnemonics for lists.g. Recall after you have read.h. Make questions of headings and read to find the answers.I hope you find this helpful."
},
{
"id": 38276,
"tgt": "Why do I wake up everyday with a weird face?",
"src": "Patient: Hey I wake up everyday with a weird face but I didn t have this before... I started getting it after I kissed some girl... My eyes get lower and my upper lip changes... There s a line in the middle of my upper lip and every time I look at a pretty girl they seem very unattracted and even when I look at ugly girls Doctor: HIWell come to HCMThis is could be due to the illusion and this may be possible due to functional condition you might be having, you really need to control your thought to get rid of this condition, take care and forget about this."
},
{
"id": 27193,
"tgt": "Can Clomid be safely resumed for low testosterone levels?",
"src": "Patient: Hi there, I am a 21 year old male and two weeks ago I had a week s stint in hospital with myocarditis or inflammation of the heart muscle, and was discharged without any long term damage - expecting a full recovery. However, I was taking clomifene (clomid) for low testosterone and was advised to come off it. I was in the middle of a year s course and the drug had been very beneficial and had taken my T to normal levels. Now unfortunately, my testosterone has returned to a low level and is affecting my quality of life - and my andrologist can t seem to find any evidence of heart problems being caused or exacerbated by the drug. So I would very much like to go back on clomid as soon as possible, however, I need an opinion from a heart specialist to know it s safe to do so. Please could you offer me some help? Thank you Doctor: clomid can rarely cause and side effect on heart. so if your andrologist wants he can restart your treatment for low testosterone. no harm in restarting the clomid."
},
{
"id": 57402,
"tgt": "Has portal vein thrombosis, cirrohsis, liver tumor, had peritonitis infection. Solution?",
"src": "Patient: My mother was just told by doctors, that she has pvt (portal vein thrombosis ), cirrohsis, and a baseball size tumor in her liver . she was in the hospital with peritonitis infection...she s hemorraged # times in 1 month..has had 3 transfusions.....How long can this go on?????? she s looked 9 months pregnant and has went down some..... How long can she live like this? Doctor: Hi and welcome to HCM. Thanks for the query.Unfortunately this is very serious codnitions with poor prognosis and nothing cant be done to prevent the worst. it is hard to predict how long can she live but i am sure that doctor will give all to prevent significant hemorrhage and pain. Wish you good health. Regards"
},
{
"id": 124519,
"tgt": "Should i take X ray after hitting hard on rock and getting bruised?",
"src": "Patient: Hit a rock really hard the other day while rafting...seriously thought I broke something in my hip/buttocks area. Goose egg swollen and bruised under my left butt cheek. Can still walk, but hard to sit down. Should I get an x-ray, or did I just bruise it very badly? Doctor: Hello, It could be a contusion probably. Fracture is unlikely. As of now you can use analgesics/anti-inflammatory combination like aceclofenac/serratiopeptidase for symptomatic relief. You can apply icepacks for faster recovery. If symptoms persists better to consult an orthopedician and get an X-ray done. Hope I have answered your query. Let me know if I can assist you further. Regards, Dr. Shinas Hussain, General & Family Physician"
},
{
"id": 59419,
"tgt": "Suffering with end stage liver disease, vomiting, nausea, had general malaise, abdominal pain. Help?",
"src": "Patient: my husband suffers with end stage liver disease , he has fluid removed every Wednesday 8 litres to be exact, he got up about 2 0 clock this morning with vomiting and nausea , had general malaise during the day with chills, vomited around 4 pm, presently is feeling very cold and covered with a blanket has a temperature and is complaining with abdominal pains for the first time. What do i make of this condition? Doctor: Hello, Ascites is the most common complication of end stage liver disease.This a very important mile stone in the patients suffering from ESLD,as this makes the survival chance very poor. The most important complication in such patients is bacterial peritonitis. The symptoms and signs of your husband suggest that your husband has developed this infection. Must consult your doctor urgently,get the required investigations done and start broad spectrum antibiotics as per the advise of your doctor. I hope it helps. Thanks"
},
{
"id": 4157,
"tgt": "an I get pregnant even under birth control?",
"src": "Patient: I have been on the Implanon birth control since February 2012 I haven't had a period since then and around July 10, 2012 i got my period on the 16th it was spotty and by the 17th it was gone I had unprotected sex with my partner on the 18 and i was bleeding. Can I get pregnant? Doctor: HIThank for asking to HCMI really appreciate your concern looking to the history given here as I say nothing is hundred per cent protect you form the conception and there is little chance of conception even after having the oral contraceptive medicine so if you have any doubt get done the test and the best option is ultrasonography test, hope this information helps you, have a nice day."
},
{
"id": 193030,
"tgt": "What could forehead skin scarring be?",
"src": "Patient: hi...my parents are worried about a guy they arranged for arrange married..this guy is good looking but has alot of cut scars on forehead akin to one on harry potter...his father died of depression...he too is very serious and pestering only on one point..!! could just help me understand if this good match and he is not suffering from depression...??? Doctor: Hi, You can ask him directly about the cause of the scar, if the reply is not link-able, it can be due to self-cut. Hope I have answered your query. Let me know if I can assist you further. Regards, Dr. S. R. Raveendran, Sexologist"
},
{
"id": 167695,
"tgt": "What could red itchy patches, with high temperature & low appetite suggest in children?",
"src": "Patient: My daughter aged 2 and a half years developed small red spots on her body (both hands and legs) 3 days ago- it almost appeared like a bug bite at first. Associated with that she also had very high temperature for 12 hours which subsided. The red spots have gradually grown to red patches, some are itchy some arent. Some patches have a clear skin in the middle. Her appetite is low and has alos developed a cold since this morning. What could this be? Doctor: Hi...Thank you for consulting in Health Care magic.By what you quote it should be an urticarial or a simple skin allergy. You can use Hydroxyzine at 1-2mg/kg/dose every 6th to 8th hourly for 7-10 days. Most important thing to be remembered is that it has a propensity to recur (called as second crop) within 10-14 days. If this happens, you can start using the same medicine but I suggest you get the kid evaluated with your paediatrician.But...Skin conditions are best diagnosed only after seeing directly. I suggest you to upload photographs of the same on this website, so that I can guide you scientifically.Hope my answer was helpful for you. I am happy to help any time. Further clarifications and consultations on Health care magic are welcome. If you do not have any clarifications, you can close the discussion and rate the answer. Wish your kid good health.Dr. Sumanth MBBS., DCH., DNB (Paed).,"
},
{
"id": 74062,
"tgt": "What is the reason of chest pain?",
"src": "Patient: my 14 year old experiences severe chest pain as soon as she swallows rice. I have felt this while eating but maybe not to the same severity. Even drinking water doesn't help. It feels even more painful and doesnt go down. Have had her blood tested for celiac but it came out negative. What else could it be? Doctor: Hi there,This sounds like achalasia, which is failure of the lower esophageal sphincter to relax. The lower esophageal sphincter is a valve that separates the stomach from the esophagus, and it prevents food and stomach acid from going back into the esophagus from the stomach. Sometimes, the sphincter is too tight and doesn't relax very well, which is called achalasia. The usual diagnosis for this is something called an upper GI, in which the patients drinks some contrast and then x-rays are taken, and you'll see a pattern like a bird's beak at the lower esophageal sphincter. Another test she should have is manometry, which tests all the pressures throughout the esophagus.The treatment for achalasia is something called a Heller Myotomy, where the muscle fibers near the lower esophageal sphincter are cut in order to allow it to relax. This can be done laparoscopically with small incisions. Some medications can be used, but the best treatment, especially in patients who are healthy and of young age is a Heller Myotomy. I hope I have answered your questions. Please feel free to contact me if you have any further questions or if you feel I haven't addressed your question adequately."
},
{
"id": 193081,
"tgt": "What causes large chest compared to arms in 21 year old?",
"src": "Patient: Hi i am a 21 year old engineering student in hyderabad. i weigh 85 kgs. But my chest is very large compared to my arms i feel. I think i might be suffering from gynecomastia but i am not sure. What do you suggest? Should i go some tests or consult a doctor?? Doctor: Hello,It might be a part of obesity. If the excess breast tissue as in females it can be considered as gynecomastia. In that case you can consult a general surgeon and get evaluated. Various treatment options are available like liposuction and mastectomy.Hope I have answered your query. Let me know if I can assist you further. Regards, Dr. Shinas Hussain, General & Family Physician"
},
{
"id": 71330,
"tgt": "Suggest treatment for recurrent bronchitis",
"src": "Patient: Yes I have been treated for bronchitis twice within a 4 month period and my symptoms get better but just keep coming back. I get a lot of phlegm buildup in my throat, and an annoying cough, sometimes no matter what I eat or drink the slightest burp a little bit of food or fluid comes back up, It doesn t burn at all. And at times i have problems swallowing the air I breath, I was wondering if this could be a concern? Awhile back a couple Drs. Has asked if I have trouble swallowing and I didn t bother to ask the reason for that question, and I tried thinking about it and I ended up saying no Doctor: Hello,A recurrent bout of cough and expectoration are also seen with pneumonia. This is the reason why the doctor was asking about the trouble with swallowing.In my opinion, you should get done upper GI (gastrointestinal) scope and CT thorax. Upper GI scope is needed to rule out aspiration. CT thorax is needed to rule out pneumonia as a cause for a recurrent cough with expectoration.Hope I have answered your query. Let me know if I can assist you further.Regards,Dr. Kaushal Bhavsar"
},
{
"id": 21641,
"tgt": "What causes rapid heart rate with diarrhea in the mid of night?",
"src": "Patient: I awoke from a nightmare last night and realized that I had a rapid heart rate. I also experienced the need to use the restroom, which resulted in diarrhea. Can this be considered to be a normal occurrence as this has happened before? I have had all the tests for heat condition, which were normal. Doctor: Hi,Welcome.This is Dr Sameer.The condition you are telling seems abnormal i.e. Palpitation, tachycardia followed by diarrhoea.This type of scenario generally occurs in a arrhythmia called as supra ventricular tachycardia.So you need to evaluate it further.If all the heart tests have been done, than its fine, but I'll still recommend you for a 24hr ECG monitoring called as Holter Monitoring. This may catch any abnormal palpitation if that occurs in that 24hr period. Otherwise there are additional tests called ELR(Elective Loop Recorder) which can be put for 7 days or a month for ECG monitoring. Come back if you need any more help.Thanks"
},
{
"id": 123423,
"tgt": "Suggest remedies delocated knee pain",
"src": "Patient: hi, im 19 & diclocated my knee 3 years ago. My knee seems to be clicking for the past few weeks when I bend it, it is also vert painfull if I am on it for A long period of time, when my knee is bent it seems like the top pant of my knee is being pulled Doctor: Hello, I will advise you to do balance board training for improving muscle imbalance & joint stability, and pilates for quadriceps muscle. This will you regain full functions of the knee. Hope I have answered your query. Let me know if I can assist you further. Regards, Jay Indravadan Patel, Physical Therapist or Physiotherapist"
},
{
"id": 127100,
"tgt": "What causes cramps and stiffness in the fingers?",
"src": "Patient: my left hand gets cramps; my fingers separate into a frozen state. I can t move them; it s as if they separate and gointo a frozen and locked position It may last 2 to 8 minutes. now, starting to happen more frequently, and last longer; I don t know if it is a spasm, or what to think Doctor: Hi, It may be due to ulnar nerve compression at elbow or wrist lead temporary numbness in the little and ring fingers may go into lock in state. Other possibilities like Raynaud's phenomenon or arthritis of joints etc. Until examination is done it is difficult to say what it is. Avoid reading/sleeping on sitting position with both arms resting position. Use vitamin B12 tablets, if symptoms not improved please consult your doctor he will examine and treat you accordingly. Hope I have answered your query. Let me know if I can assist you further."
},
{
"id": 125338,
"tgt": "What is the solution for AVN of hip joint other than replacement surgery?",
"src": "Patient: Hi! I have been diagonised with AVN of the hip joint stage 2. I have been advised by my doctor to have calcium and physiotherapy. Its been 10 days since I have been on physio and the pain has reduced quite a bit. I want to know what is the actual solution to this problem apart from hip replacement as the docs are advising. Will the AVn spread to other joints, will it go to stage 3. Will my joint/ bone break if I fall as I feel very weak in my legs and feel that I have no strength in my hip joint. Will physiotherapy help me in the long run. What chance do I have. Doctor: Hi, AVN stage 2 is middle of stages and is likely to progress further. It does not spread to other joints as such. Hip replacement is the only answer but wait for it, until you are not able to do your activities of daily living. Physiotherapy will strengthen your muscle and keep them in good shape for future. There are no statistics to suggest how and when will AVN progress further. Hope I have answered your query. Let me know if I can assist you further. Regards, Dr. Gopal Goel, Orthopaedic Surgeon"
},
{
"id": 62181,
"tgt": "What does a lump near ear bone indicate?",
"src": "Patient: Hi, I had a small lump near ear bone, just next to the sideburn. Ive had this lump maybe a year and sometimes the lump did decrease in side but most of the time it stayed the same size. it doesnt hurt or anything but then today i noticed that on the lump there is a dark black mark on the lump. It was a black dark round mark on the lump, it still doesnt hurt or anything. Should i be worried? why is there suddenly a dark black mark on the lump? Doctor: Hi,Welcome to HCM. Based on the facts of your query,You seem to suffer from very tricky side ear lump,and the black dark mark on the lump,indicates Nevus developing in to Melanoma. A wide Margin Surgical Excision Biopsy of this lump would resolve this mystery about the diagnosis and treatment of this side ear lump.Hence I would suggest you to go to Surgeon / or Dermatologist to rule out the Risk Melanoma in such a lump by Excision Biopsy .HOpe this reply would help you to plan further treatment with your doctors.Contact with a Followup Premium question to ME. Will appreciate your Hitting thanks and writing excellent review comments to help needy patients like you. Good Day!! Dr.Savaskar, Senior Surgical Specialist M.S.Genl-CVTS"
},
{
"id": 127497,
"tgt": "What causes hand pain and numbness?",
"src": "Patient: I have had type2diabetes for more than 35 years. have had periphial neur opothy forabout 25 of those years bilaterly in my feet. However it only manifests in making my feet feel coldBut about a yearago, my hands fbegan to fall asleep. but about six weeks ago, I awoke to find my hands holding a porcupine struggling to get free, accompanied by excruciating pain as if an ice pick was being thrust through my hands. I frequently drop small items from my hands. pages in printed matter are difficult to turn. Doctor: Hello and Welcome to \u2018Ask A Doctor\u2019 service. I have reviewed your query and here is my advice. The pain and numbness in the hand can be related to neuropathy or nerve root compression at the level of cervical vertebra. Hope I have answered your query. Let me know if I can assist you further."
},
{
"id": 132738,
"tgt": "Does twitching fingers mean nerve damage?",
"src": "Patient: I HAVE TWITCHING FINGERS RING ,INDEX,LONG,GETS WORSE WITH EXSERCISE , INCREASED SENSATIVITY THREW ENTIRE HAND NUMBNESS ON UNDER AND RADIAL FOREARM FROM INNER ELBOW DOWN TO HAND I AM 7 WEEKS POST SURGICAL REPAIR OF PARTIAL RUPTURE OF DISTAL BICEP TENDON . FROM INJURY TO REPAIR WAS 3 MONTHS.IS THERE POSSABLE NERVE DAMAGE Doctor: Hi Hope this message finds you in good health.I have gone through your complaints and understand your concern.U seem to have compressed a nerve in the neck somewhere that might be causing radiating pain. Possibility of a cervical disc prolapse,or an injured nerve is likely.I suggest you get an Xray of neck and that region as well as an MRI and Nerve conduction velocities to find out the exact cause.Physiotherapy,analgesics,anti-inflammatory drugs should help.Nothing to worry about.I hope your question has been answered.If you have any follow-up queries,feel free to consult me anytime.Thanks,Take care,God bless."
},
{
"id": 74816,
"tgt": "What is the treatment for blurry vision and chest soreness?",
"src": "Patient: Hi, I am 26 yrs old, 5'4 and 115lbs. I have 3 children and a prothrombin gene mutation. I have had a cough for about a week. For about 2 days now my vision has become blurry. And this morning since Ive been up, my chest is sore when I eat or breath deeply. Is this something to see a doctor about? Doctor: Hello there,Yes definitely you must consult a doctor on a urgent bases.get done consulted by a physician and ophthamologist soon."
},
{
"id": 87863,
"tgt": "Suggest treatment for pain below the rib cage",
"src": "Patient: sspeciailists, please mention your problem in as much detail as possible. Do not forget to highlight your Age, Gender, Medical History and Treatment History of the problem you are facing. I recently fell and hurt my knee now when I cough or spit it hurts just below my rib cage on both sides Doctor: Hi.Thanks for your query and an elucidate history. You have a typical history of fall on the knees followed by pain bellow the rib-cage if you cough or spit.This has happened due to the muscles of the upper abdomen start from the lower ribs and it looks they has got sprained a lot while your body tried to balance the fall. I would advise you the following:Take a full bed rest.Take anti-inflammatory medicine like Ibuprofen at 8 hourly duration after food and with Ranitidine.Apply a rubrafacient liniment locally.You can apply an elasto-crepe to the lower chest to support the movement of the ribs. Take laxative to avoid constipation. Let the room be warm. You will be fine soon."
},
{
"id": 169546,
"tgt": "Why is my baby spitting all the feed?",
"src": "Patient: My Daughter who is 4 months old keeps spitting out milk mostly after every feed. I took her to her doctor and he prescribed junior lanzol 15mg for 15days . The symptom seemed to slightly reduce but not fully ok so he has asked to continue for another 15days. I am worried please help me out with this whole thing. She has a twin brother and my daughter who was born 1kg.940gms now weighs 3.760. Doctor: Hi,From history it seems that this problem might be due to over feeding or due to inhaling air while feeding and afterwards when feeding is over with ingested air milk might have come out.Inspite of vomiting she is gaining weight nothing to worry.After feed keep her stand position for some time so that all ingested air will come out.Ok and take care."
},
{
"id": 217162,
"tgt": "Suggest treatment for sharp pain in the left pectoral region",
"src": "Patient: im a young 31 year old male, extremely active male in the fitness industry...over the last several weeks I ve had sharp yet very brief pain in the left pectoral region at random moments. These pains are not accompanied by sweat or any strenuous activity. Likewise, there is no change in breathing; in fact the only thing i notice is I m leaning over to a directon or simply breathing sitting in the evening while working on planning revisions. Ironically the latest incident happened roughly 3 minutes ago prior to me searching, yet again, online as a means of being overly cautious for some reason or another. I also check my blood pressure and it is extremely healthy not even borderline high. Thanks in advance Doctor: Looks like you live a very healthy life. I suggest to check with your cholesterol and electrocardiogram. If there is any problem then take treatment accordingly. But if that is also normal then nothing to worry use hot pack onnyoir painful area and it should be fine in a day or two. Here if it happens once again and if you feel it is gradually increased in intensity then do consult your doctor to have proper check. Take care."
},
{
"id": 123704,
"tgt": "Why does my leg become cold after the therapy?",
"src": "Patient: Had knee arthroscopy and excision of medial plica 2 weeks ago. Just started PT four days ago and am finding that in the evening when resting, both in sitting and laying down positions, the leg gets cold from the knee down. There is no swelling or pain in the calf or foot. It is just cold. Could this be from the movements involved in the therapy? Doctor: Hello, This has nothing to do with surgery or therapy. You start using crepe bandage and see if things improve. Hope I have answered your query. Let me know if I can assist you further. Take care Regards, Dr Anuj Gupta, Spine Surgeon"
},
{
"id": 40663,
"tgt": "How can infertility be treated?",
"src": "Patient: I am married since past 7 months me and my husband are trying since then to have baby. I was fortunately found pregnant in the month of July. But i do not what happened after 5 days i started bleeding as a normal period. The after 10 day i started bleeding heavily with full pain in my abdoman for 15 days. I consulted to Gynaecologist then in UAE (Dubai) she said it was just chemical pregnancy. Nothing worry. Since then i have irregularity in my cycle. My haemoglobin count is also decreasing. Now its more than week i didnt get periods, yesterday i went to doctor for B HCG test it showed negative. I am really stressed for my pregnancy. Please help me out with good suggestions so that I can conceive as soon as in a month. Thanks hoping for your best solution in this case Doctor: Hi there,I have understood your concern and I will suggest you the best possible treatment options.I will suggest you to get USG done to know if there is PCOS- polycystic ovarian syndrome as a cause for your irregular cycles.If so opt for a healthy diet and regular exercise regime. Include more portions of fruits, vegetables and salads in daily diet. Avoid refined sugars , deep fried foods and bakery products. Start on Folic acid and Vitamin B 12 supplements.Please get postmenstrual HSG - Hysterosalpingography done to know about the patency of Fallopian tubes.Please seek assistance from your treating doctor for ovulation induction medicine like Clomiphene, and get follicle study done to know the day of ovulation.You can opt for natural intercourse on and around the day of ovulation or opt for IUI- intrauterine insemination on the day of ovulation.Think positive.Meditate regularly.May God bless you with a bundle of Joy.I hope this answer helps you.Thanks Dr. Purushottam Neurgaonkar"
},
{
"id": 44197,
"tgt": "Have endometriosis. Trying to conceive. On Luprin. Next?",
"src": "Patient: I AM TRYING TO CONCIEVE SINCE LAST TWO YEARS THEN 3 MONTHS BACK MY DOCTOR DIAGONISED ME HAVING ENDOMETROSIS AND STARTED INJECTION LUPRIN , I AHVE TAKEN THE INJECTIONS FOR TWO MONTHS BUT WHEN I GOT MY ULTRASOUND DONE THERE WAS EFFECT ON THE SIZE OF THE CYST , POST THE SAME I WENT TO ONE INFERTILITY CENTER AND THEY ADVISED TO GET SOME TEST LIKE AMH AND FSH , IN THE SAME IT WAS DIAGONISED THAT MY AMH LEVEL IS LOW 0.49, MY DOCTOR HAS ADVISED ME TO TAKE EEMA-D & FOLIC ACID FOR THREE MONTHS AND ADTER THE SAME I NEED TO VISIT HER IF INBETWEEN I DONOT GET PREGNANT, KINDLY ADVISE WETHER I AM GOING UNDER CORRECT TREATMENT OR NOT Doctor: Hi, Endometriosis is the most commonly seen condition, more so in women trying to concieve. Yes it can be one of the reason for infertility. AMH( ante mullerian harmone) indicates your egg quantity and FSH is an harmone for predicting the quality of egg. However these test are not the definitive test. AMH decreases with increasing age. Any treatment is unlikely to increase your AMH level significantly. Your medications are appropriate and do continue with the treatment with your doctor. My advice is to start treatment by end of 3rd month and not loose much time as endometriosis can recur. Hope i have answered your question."
},
{
"id": 176181,
"tgt": "Suggest treatment for cough and cold in child",
"src": "Patient: Hi, my son is 3 and a half years old. He has cough and cold problems every winter. Hence he was prescribed Montec LC kid and Foracort 100 pump by his pulmonologist last week. However since last 2-3 days he has running nose and cough(wet cough) inspite of being on medication. Can these drugs treat his current problem. If not, what do you suggest? Doctor: He is more than likely experiencing a cold right now. Give it a few days and it should resolve on it's own. If he develops fever or the symptoms worsen over the next week, have his doctor check him."
},
{
"id": 52899,
"tgt": "How to treat horse voice after allergic reaction to gall bladder surgery?",
"src": "Patient: I had gall bladder surgery on 3/12th and had a reaction to the anesthesia, my asthsma was triggered and wasn t breathing well. I didn t come around in the 2hrs that they told my husband I would. After about 4hrs they admitted me. I was in the hospital for 3 days/2nights. It has been 2 weeks since the surgery. Since then, which also coinsided with the tree pollen, I have had a raspy/hoarse voice. Last week I also had the asthsma symptoms and called the allergiest, they told me to increase the Flovent. Some of the shortness of breath and coughing has subsided but I still have a very raspy voice. Doctor: Hello,Continue medicines told to you for allergy. Besides some home remedies like steam and breathing exercises can help you.Hope I have answered your query. Let me know if I can assist you further.Regards,Dr. B.dinesh"
},
{
"id": 81342,
"tgt": "Suggest remedy for cough with phlegm in chest",
"src": "Patient: i am a male 43 years old, 5 6 in height and 73 kg in weight..i was suffering cough caused by phlegm in my chest for more than a week..i was taking ambroxol tablets for three days and cefalexin 500mg antobiotic for a week. But still nothing happened..please advise. Doctor: Thanks for your question on HCM.In my opinion you should consult pulmonologist and get done1. Chest x ray2. PFT (pulmonary function test) Since you are taking antibiotic (3rd generation cephalosporin) and not improving even after 7 days, so we need to rule out lung infection and bronchitis.So chest x ray is needed to rule out infection.PFT is needed to rule out bronchitis.So better to consult pulmonologist and first diagnose yourself and than start treatment accordingly.You will be alright. No need to worry much."
},
{
"id": 25946,
"tgt": "Suggest treatment for the numbness in the leg after stroke",
"src": "Patient: My husband is 42 and had a stroke a few weeks ago (a broken blood vessel, not a clot). He came out of it pretty good, no paralysis, he can speak and function fine. He has numbness on his left side and his leg is especially bothering him. I was wondering if physical therapy could help that. Also, since the stroke, whenever he takes a deep breath he has tremors. Is that an effect of the stroke? Is there anything he can do about it? Doctor: Dear Mrs,Your husband needs a rehabilitation course. It will help greatly. He needs special exercises. Usually they help, but it may take long time to recover, you should be patient and everything will be fine.Hope I could help youWishing a good health to your husbandIn case of further questions don't hesitate to askRegards,"
},
{
"id": 136272,
"tgt": "How to cure RSD on right arm?",
"src": "Patient: hi I had dequervainnes syndrome of right wrist. also extensor origin tears to right forearm/elbow. also had two rotator cuff reconstructions to right shoulder including detached long head of biceps which was tenodesed. the shoulder surgery failed and have new tear to superior labrum of right shoulder. recently diagnosed with complex regional pain syndrome (RSD) to right arm after being in pain for 3 years. also had drop attacks recently although brain/spinal scans came back clear. shoulder is irrepairable and CRPS is spreading (right hand and left foot turn blue). what am I supposed to do other than take medication for pain relief? Doctor: Hello, I have studied your case.Reflex sympathetic dystrophy is common after injury which can lead to hardness.RDS needs prolonged multimodal treatment, Tryptomer, and calcium channel blockers will help you.Take them consulting your doctor.If you are not getting relief with medication then sympathetic block can be done.Many of my patients find relief with yoga, pranayama, meditation.So you can also do that to improve your treatment results.Do continue physiotherapy.Hope this answers your query. If you have additional questions or follow up queries then please do not hesitate in writing to us. I will be happy to answer your queries. Wishing you good health.Take care."
},
{
"id": 206064,
"tgt": "Suggest treatment for stress and depression",
"src": "Patient: my husband don't love me again, but he still live with me I don't know why. But, I can feel there's a compulsion by doing living together. He is often doing sexual good with his wife. I know, I realized he has someone in his heart. Consequently I had 'crucifixion' because I can't get a needs for sex that God given. In the final I get suffer, my body were sick, headache, and broke down feeling. But I try to survive facing my life. So, what should I do about this relationship? Let him choose woman that he want, or what? Doctor: DearWe understand your concernsI went through your details. I suggest you not to worry much. I do not know for how many years, you are married and living together. Family life is purely mutual understanding and mutual trust. If you do not trust your husband, there is every possibility that your emotions will be hurt. It is also possible that your husband is purely innocent and all these are just your imaginations. thus happens due to possessive attitude.In every case, talking to your husband should be of great help. You may also consult a psychologist for marital / family counseling if need be. Psychotherapy techniques should suit your requirement. If you require more of my help in this aspect, Please post a direct question to me in this URL. http://goo.gl/aYW2pR. Make sure that you include every minute details possible. I shall prescribe the needed psychotherapy techniques.Hope this answers your query. Available for further clarifications.Good luck."
},
{
"id": 79871,
"tgt": "What could chronic chest pain indicate, taken antibiotics for pneumonia and crackles?",
"src": "Patient: I have two questions. The first one is on March 3rd I went to the dr s about chest pain because I had a bad cold and I couldn t breath in deep without it hurting. They told me I had Pneumonia and prescribe me Mylan-Levofloacin 500mg took 1 1/2 tablet once by mouth for 7 days and 200 DOS Ventolin HDA 100mcg 2 puffs every 4hrs. I did that and I still had pain so I went back and the dr said my pneumonia was gone but still had crackles in my lungs so he said I had pleurisy and prescribed me Naproxin 375mg 1 tablet by mouth 3 times a day I got them on march 18th. It s now march 23rd and I still have that sharp pain on my left side when I breath or cough and still have crackles. What do I do? And the other question is can all these antibiotics make my period late or skip it? Thanks! Doctor: Thanks for your question on Health Care Magic. I can understand your concern. Answer to your 1st question. In my opinion you should get done CT thorax to rule out bronchiectesis. It is type of healed lung tissue common after pneumonia treatment. In this part of lung tissue dilates and remain as it is. In the dilated part, secretions tend to accumulate and cause crackles and pain on coughing. So get done CT thorax to rule out bronchiectesis. Answer to your 2nd question. Delay in periods is seen in acute infection and antibiotic use both. But infection is the likely cause in your case. Infection anywhere in the body causes stress and hormonal imbalance. Hope I have solved your query. I will be happy to help you further. Wish you good health. Thanks."
},
{
"id": 29338,
"tgt": "What causes itchy welts all over the body while on Eliquis?",
"src": "Patient: My husband has been taking Eliquis for 2 years..(for all the right reasons)! Suddenly he has a rash although I cannot call it a rash..It looks like welts.. The itching is agonizing..He just came from the dermatologist who is baffled.. Tomorrow I will call out cardiologist.We are all puzzled..before the welts came out it itched all over his body.. He is ingesting NOTHING new..No new laundry products...etc. etc.. If you would like a picture of what they look like I can cell it to some cell phone!! What do you people at Eliquis think????How do U send you this message?? Doctor: HI, thanks for using healthcare magicIt is possible to have an allergic reaction to eliquis like any medication or supplement.This means that is no other cause is found and there are no other medications that he is taking that may be responsible, it may be the eliquis causing the problem.He may need blood tests to correctly determine the cause of the rashI hope this helps"
},
{
"id": 77127,
"tgt": "What causes chest pain on right side and shoulder blade pain?",
"src": "Patient: I have been sick for a few days now with cold/flu-like symptoms. Yesterday I woke up with chest pain on the right side. It felt as if I had been punched and it went through to the back at an angle feeling it at the right shoulder blade. What could this be? Is it the flu? Doctor: Hi ....I can understand your concern.You have viral flu....So yes it can lead to muscle pain like constitutional symptoms....Symptomatic treatment done...Take rest...For cold you can take oxymetazoline nasal drop as decongestent...Cetrizine tablet taken to relieve inflammationDrink more water...Steam inhalation with eucalyptus good one.Avoid excess spicy and fried food for few days....If still symptom present your auscultation done....and if need chest xray done...Take care.Dr.Parth"
},
{
"id": 44459,
"tgt": "CT scan showed multiple follicles on both ovaries, suspicion of PCOS, blood test showed elevated FSH and LH levels. Taken progesteron tp stop bleeding. Does this mean infertility problems?",
"src": "Patient: Hi, I had a period that lasted from April 13-June 1. I was put on progesteron which helped it to stop. I got an u/s on May 24 and it came back that I have multiple small peripheral follicles on both of my ovaries. It says I may have PCOS . My FSH number was 5.3 and my LH was 13.6 when I had blood work on May 26. Do I really have PCOS? I m scared that I have infertility problems. Doctor: hi, as per your reports you are having normal FSH and high LH, and your scan report showing multiple small follicles on both ovaries, both are indicating the possibility of poly cystic ovarian disease. infertility can occur in PCOD because of anovulation, but by taking treatment for PCOD you can over come this problem. you follow the treatment given by your doctor, if having over weight try to reduce, don't take excessive stress about anything, as it can increase the severity of PCOD. take care."
},
{
"id": 218207,
"tgt": "Is pregnancy a possibility when periods is late despite experiencing only outercourse?",
"src": "Patient: Hai ...tis s Mathan from tamilnadu ..I am 23 ...me and my girlfriend not done intercourse ..but that time.. periods near time ...periods not came ....only touching of men s reproductive and women s reproductive..is there Any possibility to get pregnant..please help us ... Doctor: Hi, By touching private parts, she won't be pregnant. Ask her to get a gynecologist check up to find out the cause of delay especially hormones. Hope I have answered your query. Let me know if I can assist you further. Regards, Dr. B. Radhakrishnan. Nair, OBGYN"
},
{
"id": 55747,
"tgt": "What causes bloated stomach with blood traces in urine and stools?",
"src": "Patient: I am a 24 year old male. I had my gall bladder removed 5 years ago and I have acid reflux and Celiacs disease. My stomach has been swelling for the past week with pain and nausea and extreme fatigue. Two days ago my urine started turning pinkish red. Today I my feces is pale with blood. Doctor: Hello, thank you for posting your query in HMC. I understand your concern. The symptoms you display are consistent with liver disease. As the gall bladder is already removed there are chances of developing liver diseases. I would suggest you to consult your gastroenterologist . He or she will be able to carry out further tests and clinical examinations before arriving at a diagnosis.Hope i have answered your question. If you have any further questions i will be happy to help."
},
{
"id": 212519,
"tgt": "Dent on skull, been stressed. Can they be related?",
"src": "Patient: Hi I I been having a lot of stress lately and I cut my hair last week and I saw a lil dent on my skull and I m petty sure I did t hit my head with anything, so I was wondering I if the reason why is becouse the stress I been having or if it was becouse I just also got done with migrant and allergies my mouse was plugged very bad and I will blow my nose out to clean them.. I m a lil worry becouse I didn t have this dents before... Any answers will help thank you!!!! I had to Edit my question and all I wanna say is that thanks but I don t have any money if I did I will go to the doctor thanks anyways Doctor: Hello abe-89 There is nothing to worry. The dent is pretty normal, due to development impressions are formed on skull and the dent is probably that. So nothing to worry. Regarding the allergy in not and blocked nose please take steam 2-3 times a day, do warm saline gargles and you can take OTC antiallergic drugs to get symptomatic relief. Don't think about the dent again and again as it is a normal incidental finding. Take Care Thanks"
},
{
"id": 142177,
"tgt": "Suggest treatment for severe wrist pain after ganglia cyst removal",
"src": "Patient: Hello, six months ago I had a ganglia cyst removed from my inner wrist.I woke that night in severe pain as i scratched my back in my sleep ,it hurt so bad i nearly passed out and vomited.I was on pain killers for a month,when I saw my doctor again, he suggested opening it up to have a look,he gave me antidepressants and sent me on my way.. I am in constant pain,its not right. please advise me on what my options are.I think he has damaged my nerves in my skin, it hurts like sore pins and needles, and long stretch movements hurt alot,. I have endured alot of pain.Im scared now too cause my other wrist is sore,but its a lump on the upper side of my wrist. I need help now, but i dont trust my local doctor anymore. He gave me no warning of the damage that could have happened,also no options. Im a mother of four young children please advise me. Doctor: Unfortunately, chronic pain from the surgery itself cannot be ruled out. Have you been tried on standard NSAID therapy, warm compresses, and wrist/hand exercises. There are also other types of pain relief agents that could be used depending on whether the pain is neuropathic or not in nature. If your doctor who performed the surgery cannot seem to find the right combination of medications to work then, I would ask the primary for a referral to a pain management specialist."
},
{
"id": 137183,
"tgt": "Suggest treatment for constant pain under the shoulder blade",
"src": "Patient: I suffer from severe anxiety get constant pain behind my ribs and up the sides of my ribs through to my back under my shoulder blade and across my back but today have been getting pain in my left shoulder and down the top of my arm starting to worry can you help. Doctor: Hi there.You could be suffering from a pinched nerve in the back of the neck. Avoid bending neck forwards or carrying weight over the shoulders. Apply hot water bottle over the painful areas. Consult an Orthopaedic surgeon and also your Psychiatrist to help you in this regard further."
},
{
"id": 112030,
"tgt": "How to cure severe back pain?",
"src": "Patient: i am on tramal hydrochloride 100 mg 2ml injections. I have server back pain. I was on oxicotin 60 mi for 3 years. My problem is nothing is working anymore. I was told by a friend to inject the tramal strait into my vein and I would get better releaf is that true. Doctor: Hi, thanks for writing to HCM.Simply, increasing the dose of an analgesic medication or changing the mode of administration (by I.V.route) will only masks or suppresses the pain but that is not going to solve your problem as the root cause for your back pain is not being addressed.So, unless proper diagnosis for your back pain is made and the root cause is treated, your symptoms will stay with you.I suggest you to consult a spine surgeon for a detailed clinical examination and further workup (xrays & MRI scans).Hope this information is helpful. Good day"
},
{
"id": 16223,
"tgt": "Itchy rashes around bikini area,thighs and pubic area. What should I do?",
"src": "Patient: I have extremely itchy rashes around bikini area,thighs and pubic area. Had been to my GYN, he did tests for east infection, it came out clear. Blood tests for diabetes and thyroid also came clear. He said it might be a cause from detergent, but i have nat changed any brands in the detergent or toileteries. This is the second time I m getting this. First time was last month which ended with painful boils. Please help. Doctor: hello, im dr satya. itchy rash around pubic area, thigh, and bikini area, common causes of itch aroud this area is candidiasis, fungal infection or also called tenia, alleric dermatitis, id reactions, parasitic infestations such as scabies, and some rare disorders. before concluding that you have something serious i would suggest that you follow a good hygiene and consult a dermatologist for a better evaluation. thank you dr satya ,"
},
{
"id": 216494,
"tgt": "What causes constant pain in my big toe?",
"src": "Patient: Hi my name is Sarah and the side of my big toe hurts terribly. I ve been experiencing similar pains in both feet. the side hurts and it looks as if 3 veins alongside the bone start to stick out. At first I thought it was from some shoes that had never bothered me before but I haven t worn them for some time and the pain starts randomly, such as when I m sleeping and then the veins start bulging. Movement or pressure on the foot doesn t have any effect on the pain and it s a sharp hot constant pain Doctor: I think you might be having problem with some deep veins. kindly visit the surgeon for further tests ."
},
{
"id": 186435,
"tgt": "Suggest treatment for painful lesions in the gums",
"src": "Patient: I have a few painful holes in my gums around my teeth, and today a new one appeared, rather large on my bottom back gum. Im quite worried, and i have no insurance. I practice good oral hygiene twice a day, and floss before i go to sleep, i brush lightly, and try to get all teeth for the equel amont of time front and back, bottom and top. Do you know what this may be? Doctor: Hello, Welcome Thanks for consulting HCM, I have gone through your query, as you have painful lesion in gums that can be due to periapical infection or periodontal infection . For this you should consult dentist and go for investigations done IOPAxray if it is periapical infection then go for Root canal treatment and if due to periodontal problem then go for Oral prophylaxis .Do warm saline gargle two - three times a day"
},
{
"id": 173702,
"tgt": "Could cluster of small wart like non itchy spots be juvenile spring eruption?",
"src": "Patient: I took my child to gp because of a cluster of small wart type spots on her knee. He diagnosed juvenile spring eruption but after looking into this online , these cases present themselves with red itchy spots. My daughters spots are skin coloured and non itchy. Could it be anything else? Doctor: Hi,Thank you for asking question on health care magic.Non itchy cluster of small eruptions specially over knees and elbows may be due to vitamin A deficiency.It is called xeroderma or toad's skin.Try AD caps as well as Vitamin A containing foods like fish ,liver, green,orange ,red and yellow fruits.Hope this answer will serve your purposePlease feel free to ask any more queries if requiredTake careDr.M.V.Subrahmanyam MD;DCHAssociate professor of pediatrics"
},
{
"id": 121372,
"tgt": "What causes swelling & sores in chin?",
"src": "Patient: hi my mother has a acutely swollen face under chin and along jaw line.pussy boil like sores have developed out of the swelling and she has a black line down the middle of her tongue. the swelling directly under the chin is the size of a golf ball and similarly shaped. doctor says it may be a form of herpes, symptoms getting worse, finding it hard to breath Doctor: Hello!Welcome to Ask a Doctor service!Regarding your concern, I would explain that her symptoms could be related to a hematoma in this region (bleeding) or local infection. For this reason, I would recommend consulting with an ENT specialist and performing a CT scan or MRI of this region (neck and head). Hope you will find this answer helpful!Kind regards, Dr. Ilir Sharka, Cardiologist"
},
{
"id": 48398,
"tgt": "Are renal cysts a matter of concern?",
"src": "Patient: I am a 54 yr old feamale with mildly high blood pressure 140-150 over 95- 100 even with medication sometimes I do have 2 renal cyst in my right and would like to know if I should really be concerned with my kidneys , there is kidney disease in my family. Doctor: HiThanks for your query.You have not mentioned the size of the cyst or its appearance.Renal cysts are often diagnosed incidentally. If they are not too big and look benign on the ultrasound then no active treatment is required.All we need to do is monitor them by doing an ultrasound every 1-2 years.Hope this helpsGood luck"
},
{
"id": 19897,
"tgt": "Could pain in hands, fingers and breasts be related to heart problem?",
"src": "Patient: Hello I am Sravani age 29. I working in private firm. I am having pain in my left hand and sometimes in right hand, even in the fingers of the hand. Even some sought of pain in my left breast area not exactly in the chest and disturbing mild pain in my neck, back, inside the arms or shoulders. I am facing this problem from past 6 months. My ECG , pulse rate, BP everything is normal. Even my echocardiogram is also normal. I just want to know whether the symptoms that have relates to heart related problem ? or pls advise me how to rule out my problem that I have. Doctor: Hi SravaniAfter going through your query I understand your concern.I would like to tell you that pain you have in arm, chest and back that can be musculoskeletal pain. If all cardiac work is being already done and is normal then you should see an Orthopedeation for further evaluation and treatment in a proper way.Hopefully this will answer your query.Kind Regards Dr Navneet Mahajan"
},
{
"id": 73912,
"tgt": "What causes breathing difficulty?",
"src": "Patient: Hi. Am 42 years old. Six years ago I was diagnosed with mild scoliosis. Lately I have experienced worsening breathing difficulties and notice a small hump in my upper spine just where my neck starts. I also feel a lump in my throat. I also notice my chest points left of center when standing.I was told by my spine doc six years ago that adult mild scoliosis doesn t worsen. He also did not give me a brace for my back, or a lift for my shoe or check me for vitamin d deficiency which I recently found I am deficient in.Is it possible for my scoliosis to worsen? What can I do to slow it down or stop it? I am very active with sports and exercise. Is it true that post surgery for scoliosis means no more sports? Thank you Doctor: HiIt depends on the severity of the scoliosis.Given the fact that surgery is recommended for you means that yes after it you should be careful with the effort.Dr.Jolanda"
},
{
"id": 52733,
"tgt": "What is the life expectancy if small percentage of liver, heart is functioning, presence of fluid around lungs and heart?",
"src": "Patient: My brother had his stomach removed almost two years ago. He is a type one diabetic. He cannot keep any weight and currently is on dialysis. Only a small percentage of his liver is functioning. His nutrition level is at a 17 and the doctor told us today that he has fluid around his lungs and heart. Only 10% of his heart is working. They told us that there was nothing more they could do because he has refused a feeding tube....and his body is too fragile for any other treatment or surgery. I m just wondering what is his life expectancy at this point....and what can we do to increase that 10%? Doctor: hi 10% is too low only possible soluition is Intra ventricular cardiac device..FOLLOWED BY HEART TRANSPLANTATION.iTS A BIG OPERATION AND WILL REQUIRE LOTS OF MONEY.10% can't be managed by medicines patient would land into cardiac arrest."
},
{
"id": 216705,
"tgt": "Suggest remedy for pain in head,neck,shoulder,shoulder blade and forearm",
"src": "Patient: I have been having a lot of pain in my neck, shoulder and shoulder blade area on left side. I also have pain in my left forearm and sometimes my wrist on left side. I have had a couple of instances where my hand has gone cold while sleeping. I have a lot of pain in front of my neck above clavicle, and it is extremely sore when I press on the area as ell as base of my skull. I have had headaches along with these symptoms. What is going on? Doctor: hi,thank-you for providing the brief history of youAs this symptoms looks more of a pinched nerve in the cervical spine and needs a thorough clinical assessment along with MRI of cervical spine to visualize the level of nerve compression.Also, due to the pinching of the nerve in the cervical spine, the pain radiates to the head and lower below it like - shoulder blade, shoulder, arm, forearm and hand.Physical therapy like - Therapeutic ultrasound therapy and TENS therapy will help to reduce pain and inflammation. Also, exercises on later stages will help improve the strength in the muscles of the neck, upper back, shoulder, arm, forearm and hand.In my clinical practice such cases respond well to physical therapy.RegardsJay Indravadan Patel"
},
{
"id": 122838,
"tgt": "Why has stiffness persisted after undergoing undergone surgery for distal femur fracture?",
"src": "Patient: Hello doctor, i have undergone surgery for distal femur fracture 3 months ago. I am doing exercises regulary as adviced by my doctor, but still stiffness persist. i can only bend my leg upto 60 degree. Kindly let me know when i will resume my normal activites Doctor: Hello, It is quite common after major trauma and fracture. You might require physiotherapy. Consult a physiatrist and he will direct you accordingly. Hope I have answered your query. Let me know if I can assist you further. Regards, Dr. Shinas Hussain, General & Family Physician"
},
{
"id": 25883,
"tgt": "What causes foul body odor when on cholesterol medication?",
"src": "Patient: my husband has gained wt.he takes cholesterol meds...heart meds...and bp lowering meds. He has a sour smell that is offensive now when he never did before. Its awful and increases with sweating. something is being emitted from his pores because he actually will cause the white paint of a wood toilet seat to turn a pinkish...and the linens in our bed are all stained where his body touches...it even removes the color of very expensive leather furniture. I am about ready to scream with this.... Doctor: hello,I have gone through your query.Thanks for using HCM.Basically sour smell in perspiration is not related to any medicine.I suggest you to check his serum Uric acid level ,sometimes patients of gout have such odour in perspiration.My best wishesDr.Rajesh Teli,MD."
},
{
"id": 65488,
"tgt": "Is it necessary to operate the calculus at ureto vesical junction found through sonography?",
"src": "Patient: Sir I suffered sudden pain in my right side stomach I consulted doctor and they suggest sonography in which they said they is about 8 mm calculus at right ureto vesical junction. They had given medicine for 3 days and then said we need to operte does it is curable without operation Doctor: hi dear thanks for query on HCMyou need to meet urologist and get the calculus removed at the earliest.it is a large calculi hence wont come by medical line of treatment.it is simple procedure of cystoscopy and ureteroscopin retrival of stone.if delayed will affect the rt kidney permanently.thank you please get it removed at the earliest"
},
{
"id": 106063,
"tgt": "8 year old boy has swollen lips",
"src": "Patient: My 8 year old has swollen lips I have given him 1 allergex pill how soon can I give him another 1 Doctor: Hi! If this pill is having 2 mg (weight), you can give it to your son/daughter every 4 hourly. If this pill is having 4 mg , you can repeat it after 8 hour. Have a speedy recovery."
},
{
"id": 100253,
"tgt": "What causes dizziness and breathing difficulty in an asthmatic patient?",
"src": "Patient: Hi there i have a friend that just went to the doctors today and they told her that she has asthma. Her symtoms are severe i looked it up. She also has some other problems like she is freezing cold an has two of everything on an still freezing. She also feels like she is going to pass out if she stands up and when she does stand up everybod goes black and its hard for her to breathe. How serious is this for my friend? im very concerned. Doctor: Hello,I appreciate your concern for your friend.How was her asthma diagnosed? Did she have a lung function test ( spirometry) or was it only on clinical grounds. There are other causes of breathlessness. Feeling like freezing is not a usual symptom of asthma, a black out on standing up again is not a feature of asthma. These suggest a low blood perfusion in the body like in certain conditions of the heart. Your friend is quite ill and I would recommend that you take her to the ER.You are welcome to contact me if you have further queries.Take careDr. Noble Zachariah"
},
{
"id": 174924,
"tgt": "What should I do as my child has purplish bump on buttox?",
"src": "Patient: My 1 year old has a purplish bump on her butt-ox. When my wife pushed down on it last night white/red puss came out of it and it went down a little. Well it filled back up and won't release again? Should we wait until tomorrow and try again and then schedule a Dr. Appointment or go ahead and take her to the ER? Doctor: Hi...it could be an abscess. I suggest you not to wait and get the kid to the nearest physician or nearest emergency room as this might require antibiotic prescription. If it spreads it can become a skin and soft tissue infection.Regards - Dr. Sumanth"
},
{
"id": 81860,
"tgt": "How safe is taking Amoxicillian for upper respiratory infection?",
"src": "Patient: Hi, I believe I have a upper respiratory infection and took Amoxicillian. I was going to take a Rite Aid brand Nyquil equivalent that contains Acetaminophen, Dextromethorphan HBr and Doxylamine Succinate. Is this an okay to take with the Amoxicillian? Doctor: Yes you can take amoxycillin and it is safe but why take any drug without a doctors prescription.If you have taken amoxycillin earlier then fine otherwise sometimes even amoxycillin can give rise to severe anaphylactic reactions"
},
{
"id": 202275,
"tgt": "What is the cause for swelling of the foreskin with red patches after sex?",
"src": "Patient: Hello dr a few days ago i had unprotected sex with a new partner 2 days later my foreskin swelled up and i have red patches on the head of my penis. It doesnt burn when i pee but i do have a burning sensation on the head of penis and foreskin. Is this a std or just a yeast infection please help Doctor: HelloThanks for your query,based on the facts that you have posted it appears that you have developed red patches and swelling of foreskin and glans after unprotected sex .This is most likely to be due to infection of foreskin and glans (Ballano-Posthatitis).However in view of having unprotected sex possibility of STD has to be ruled out.Please get basic screening test for STD done to be rest assured about free from acquiring STD.You need to take broad spectrum antibiotic like Cefexine along with anti inflammatory drug like Diclofenac twice daily.along with topical antibiotic ointment like Neosporin twice daily.Ensure to wash your genitals with warm water twice dailyEnsure to avoid sexual encounters till it heals up completely.Dr.Patil."
},
{
"id": 114736,
"tgt": "What causes inconsistency in blood pressure?",
"src": "Patient: hello...i take beta blockers for my hypertension. when my blood pressure goes high(146/95) i feel dizzy, but at resting my blood pressure goes as low as 101/48 50pulse...i do cardio training 5 times a week mostly jogging but not a hardcore trainer...i m concerned about the diastolic, is it normal? Thanx Doctor: Hi, dearI have gone through your question. I can understand your concern. You have hypertension but with beta blockers your diastolic blood pressure becomes very low. You need to change the drug to other group like ace inhibitors losartan or change the dosages. Consult your doctor and take treatment accordingly. Hope I have answered your question, if you have doubt then I will be happy to answer. Thanks for using health care magic. Wish you a very good health."
},
{
"id": 123245,
"tgt": "What causes painful bruise on lower rib cage and back pain?",
"src": "Patient: Bruise with appearred on my right side at lower rib cage. Had been having pain on that side and in my back yesterday and some this morning. Now I notice what appears to be bruising of the skin and it is painful to touch. I haven t fallen or beenhit by anything that would have caused it. Doctor: Hello, As the bruised area is painful to touch I will advise doing icing which will desensitize the skin and reduce the pain. Post that please undergo some laboratory tests for blood routine which can help understand if any infection is causing this issue. As the skin is only painful to touch when there is some infection. Also for back pain do regular simple exercises like breathing exercises and simple flexibility exercises which will help improve the muscle property. Hope I have answered your query. Let me know if I can assist you further. Regards, Jay Indravadan Patel, Physical Therapist or Physiotherapist"
},
{
"id": 123050,
"tgt": "What causes knee pain while running and climbing stairs?",
"src": "Patient: My knee started popping everytime I move about a week ago. It pops when I stand up, walk up stairs, or ride a bicycle. I had twin boys in November so I recently started running to tone up but had to stop due to my knees hurting. I am just wondering if this is normal or may be some sort of problem. Doctor: Hello, As this is happening due to the weakness of the muscles of the knee. Doing knee stability exercises will help avoid such situations to occur again. A properly guided exercise like - core stability, spinal muscle strengthening exercises, hip muscle strengthening exercises, static hamstring, static Quadriceps and straight leg raise will help regain the muscle strength. Hope I have answered your query. Let me know if I can assist you further. Regards, Jay Indravadan Patel, Physical Therapist or Physiotherapist"
},
{
"id": 95526,
"tgt": "Blood disorders, please give any suggestion",
"src": "Patient: DR. My mom got blood disorders and she is tahing siddha medicine, for the past two days, she could not eat, gas coming out of mouth repeatdily, any solution Doctor: Thanks for the query I could have helped u better if u had told me which blood disorder she is suffering from For the gaseous distention of the stomach and burping let her take Tab pantoprazole with domperidone before food once a day before food for week. She will definitely feel better Have a healthy life"
},
{
"id": 61143,
"tgt": "What does a lump between the vagina and anus indicate?",
"src": "Patient: I have a single lump in-between my vagina and anus, it feels a lot bigger than it looks and is barely noticeable to the eye, it is a little itchy and sore to touch, its hard and it's come back about 3 times so far, the first time it was smaller but that was a few years ago. I have been getting some bad pains around my stomach on and off for a few months, I want to get it checked but I'm scared it will be something embarrassing. Doctor: Hello dearWarm welcome to Healthcaremagic.comI have evaluated your query in details .* This is possible soft tissue lesion as - sebaceous cyst - boil - wart - others .Hope this will help you for sure .Wishing you fine recovery .Welcome for any further guidance .Regards ."
},
{
"id": 102798,
"tgt": "Is it safe to take benadryl for allergies with BP meds?",
"src": "Patient: Is it safe to take benedryl for allergies with BP meds? I am on Losartin 50 mg. also, I have fluid behind my ears, my dr. Gave me a steroid shot & prescribed antibiotics - no decongestant to relieve pressure behinds the ears. Is there something I can do for that, or do I just have to wait it out? Doctor: Unfortunately there is no medication to reduce the fluid and pressure behind your ears. Sometime with allergies you can get these problems. If you are having nose blocks very often you can try steroid Nasal spray (like Flixonase or Nasonex). For allergies it is best to use regular antihistamines. Best to avoid long term use of nasal decongestants as this can cause rebound swelling of the lining in your nose and worsen nose blocks.If you are getting frequent symptoms, use antihistamines only. Losartin can cause nasal congestion."
},
{
"id": 202496,
"tgt": "Should masturbation habit be stopped after actively involving in sex?",
"src": "Patient: am a 24 yrs guy, i started masturbating at the age of 13 and doing the same till today everyday. i have a girlfriend,we are having sex thrice a week and i m fully satisfied with this. but the main problem is that the bad thinking always knocks the door on my mind and with these i masturbate usually thrice a day. I think it is bad habit but i have not been able to control myself. i want to ask you that is that extent of masturbating harmful for our mental and physical health? What can be done to minimize the sexual desire? Doctor: HelloThanks for your query,based on the facts that you have posted it appears that you have been indulged in masturbation since many years.Masturbation is one of the means to get sexual satisfaction in absence of female partner.Since you have a girlfriend and enjoying sex thrice a week I would suggest you to refrain yourself from masturbation.Though it is harmless, excessive and frequent habitual masturbation does affect quality of erection and delays ejaculation.Please try to reduce the frequency of masturbation by keeping your self busy in activities like sports,reading ,social work etc so that you will not get free time to masturbate. Dr.Patil."
},
{
"id": 211018,
"tgt": "What could cause nighmares?",
"src": "Patient: I have nightmares since starting Hydroxyurea (Oral) for 4 years.....Doctors don't think its from the drug. Never had nightmares before. Could it be a interaction of Acetaminophen or Fiorinal (for headaches). I have essential thrombocytocsis and taken Hydrox 500, 1000 alternating each day.....Thank you so much Doctor: Hello,Thanks for choosing health care magic for posting your query.I have gone through your question in detail and I can understand what you are going through.Unlikely for the Hydrox to cause nightmares. Nightmares are horryfying dreams and dreams occur cause of mixing of fantasies with the real life events. Many times life stressors can cause night mares and even depression can cause nightmares. You may visit a psychiatrist who can evaluate you ones for depression or stress. Hope I am able to answer your concerns.If you have any further query, I would be glad to help you.In future if you wish to contact me directly, you can use the below mentioned link:bit.ly/dr-srikanth-reddy"
},
{
"id": 41628,
"tgt": "What causes non conception despite normal partners?",
"src": "Patient: Hello , My name is indu. i got married before 4yrs yet i did'n conceive. I checked lot of Gynaecologist but all are said \"Both of u normal\" but don know wt's the reason.Recently i visit new Doctor she said uterus is small v hav try to make large u take tablets like Progynova 2 mg, Loprin 75mg,Natoz ,Duphaston and C-Rich tonic. i feel very sad.pls suggest me when i will get pregnant. I have any severe probs ? Doctor: Hi welcome to healthcaremagic.I have gone through your question.As your both reports are normal then it's called in explained infertility, as now 10-20% couple suffering from this.Uterus size doesn't matter in fertilization, if endometrium is not sufficient then it will not make favorable condition for implantation.According to me no medicine are there to increase uterine size. One can increase endometrial thickness and make it favorable for implantation.IUI intra uterine insemination IVF in vitro fertilization are also better options if not conceived naturally.Hope i answered your question.Would be happy to help you further.Take care."
},
{
"id": 100137,
"tgt": "Suggest treatment for seezing caused by change in temprature",
"src": "Patient: Hi, My name is Ayush and iam 29 years old...i am suffering from sneezing problem. whenever i come under fan from outside i start sneezing and its opposite too, whenever i go outside from house i start sneezing due to temperature change. please advice, now a days it frequently happening with me, iam sitting in one room under fan and if i go to other room where fan is off, i start sneezing due to sudden change of temperature. Doctor: Hello Sneezing due to change of temperature is a characteristic feature of allergic rhinitis .Inflammation of nasal lining membrane is the main reason of such symptoms.Predisposing factors for sneezing are :Major allergens found in houses , so try to avoid them , these are dust, mite , pollen , fumes , sudden increase or decrease in temperature ( as in your case ),pests smoking ( active or passive).Use vacuum cleaner and dehumidifier in house . If possible avoid the use of fan and install air conditioner .When such patient visit I prescribe them to take montelukast=levoceterizne tablet once in the night .Hot coffee and steam inhalation also provide relief.Since this medicine is a prescription medicine , so consult your physician and get a prescription.Good luck."
},
{
"id": 168241,
"tgt": "Why 14 year old still urinate in bed?",
"src": "Patient: I have a 14 year old girl who still wets the bed ,I have taken her to the doctors and she has had ultrasounds ,xrays and there is nothing wrong all tests came back normal ,She has juvenile arthritis has had it since the age of one I was wondering if there was any advice on what to do about this situation,I know it sounds bad but is she doing it on purpose because she was at aunties house and admitted to laying there and peeing because she was to lazy to get up exact words that were said Please help all other doctors say she will grow out of it but when ? Doctor: Hi...Thank you for consulting in Health Care magic. I have few questions for you -1. Has your daughter stopped bed wetting at certain age and again started having it2. If so at what age she stopped and when did it startIf he had never stopped bed wetting form birth...you can try some behavioural changes which I can elaborate after proper history and interaction with you , but the effect will not be immediate, but it will be permanent. If its a recurrence after attaining night time control, I suggest you see your paediatrician to rule out pathological causes like urinary infections or Voiding dysfunction.Hope my answer was helpful for you. I am happy to help any time. Further clarifications and consultations on Health care magic are welcome. If you do not have any clarifications, you can close the discussion and rate the answer. Wish your kid good health.Dr. Sumanth MBBS., DCH., DNB (Paed).,"
},
{
"id": 128309,
"tgt": "What is the cause of pressure in head, eyeballs and swelling of shoulder and neck?",
"src": "Patient: Am 29 female and have been suffering from cronic pain that no doctor has been able to find out whats causing it or what it is. When this pain occurs is usually at least 2-3 times a months for anywhere from 2-7days or longer. i have extreme pressure in head and eyeballs also swelling of shoulder and neck. Is disabling, i end up sick to my stomach, sweats, diarreia...and the weirds thing is, the sound of gurgiling in the back of neck and head. I am desperate to find answers. i don t know where to turn? Doctor: HiIt should be step by step exploring by MRI scan of head,neck and shoulder.Be more vigilant as to what is the CO relation to events bringing it on,like any particular stress,diet or physical activity.Whats the emotional status and what subsides swelling and pain..please consult a neurologist or orthopedist"
},
{
"id": 114967,
"tgt": "How to cure acute anemia causing respiratory problems?",
"src": "Patient: My mother, aged 69, shows signs of acute anaemia and we are trying to figure out the cause. She was hospitalised and treated a month ago for severe respiratory infection and given strong antibiotics. Side effects included tachycardia and significant drop in potassium levels, the latter of which was addressed with medication. She was discharged after 10 days, improved and regained her appetite. But for the last about 3-4 days she has been extremely tired and weak, has lost her appetite, finds it hard to swallow, is occasionally feverish, finds it a bit difficult to talk, and her feet and face are beginning to show signs of puffiness, all of which lead us to believe it might be anaemia. As she was thoroughly tested in hospital for kidney and lung problems, and given the all-clear, we believe this might be drug-related acute anaemia. Any advice/information would be much appreciated. Thanks. Doctor: Hi, dearI have gone through your question. I can understand your concern.She may have anemia. We should search for the cause of her anemia first. You should go for complete blood count, peripheral smear examination, anemia profile and ultrasound abdomen. this will give you exact cause of her anemia. It may be due to iron deficiency, vitamin b12 deficiency, hemolysis, blood loss or other cause. Then she should take treatment according to cause.Hope I have answered your question, if you have any doubts then contact me at bit.ly/Drsanghvihardik, I will be happy to answer you.Thanks for using health care magic.Wish you a very good health."
},
{
"id": 120593,
"tgt": "Suggest remedy for pain in knee",
"src": "Patient: Good afternoon. My dog (a boxer with a giant head) ran into my left medical side knee. The injury happened about 4 weeks ago. I have to put ice on my knee 3-4 times a day, and it is also warm to the touch. The pain has gotten worse since the injury even with the home care.I have an appointment with an Orthopaedic Surgeon on Tuesday. I was told that the results showed a hairline fracture to my femur at the knee. She also said something else but I m not sure of what she was saying.I m so tired of the pain, cant wait to see what can be done. What other questions should I ask at the doctor s office.Thanks,Barry Doctor: Hello,I read carefully your query and understand your concern. Most people who receive specialized treatment for a\u00a0femur fracture need a\u00a0long-term nursing or rehabilitation facility. Full recovery\u00a0can take\u00a0anywhere from 12 weeks to 12 months. You may also need a physical therapist.You should discuss with your doctor about the treatment and the recovery process. Meanwhile,I suggest using anti inflammatory medications such as Ibuprofen to relieve the pain. Hope my answer was helpful.If you have further queries feel free to contact me again.Kind regards! Dr.Dorina Gurabardhi General &Family Physician"
},
{
"id": 102222,
"tgt": "Does hiatal hernia cause coughing, rattling in throat and heavy breathing?",
"src": "Patient: Hi! I have a hiatal hernia.When I exert myself or am in high humidity I experience asthma like symptons. Coughing and rattling in my throat. Heavy breathing,like it s hard to catch my breath. Is this due to my hernia? I have been checked for asthma and copd. Doctor: Thank you for your question!The symptoms explained by you are due to hiatal hernia. Hiatal hernia is condition where upper part of stomach enter into thoracic cavity to some extent through the weakness in diaphragm.As a result, lower esophageal sphincter tone (which protects esophagus and throat from acid reflux) is lost and the acids secreted in the stomach reflux into esophagus which causes burning sensation in the chest and rattling in throat.Take Pantoprazole tablets twice daily for 30 days. If symptoms persist and too severe, consult the physician. Further evaluation may be required to decide about surgical correction of hiatal hernia.Avoid smoking, alcohol, caffeine, lifting heavy weights. All these things will aggravate your symptoms. Keep your head in elevated position during sleep."
},
{
"id": 158902,
"tgt": "Have pelvic, lower abdomen, vaginal pain, numb bottom, urinating urge, vaginal bleeding. Cervix removed after cervical cancer",
"src": "Patient: Pain in hips and pelvic area, it goes into my lower back and causes numbness in my bottom and backs of legs. Fullness in my abdomen all the time, constantly need to pee, heaviness in vagina and back passage with constant bleeding from vagina, pressure and pain in this area when I sit, constant pain in lower abdomen area, vagina and back passage when I walk, had cervical cancer, cervix was removed still waiting on all clear from this, CIN3 in womb Doctor: Hi and welcome to HCM. Thank you foryour querry. This is definitely something that should be evaluated especially considering your previous surgery and cancer. After surgical procedure in pelvis usually adhesions develops which can cause bowel compression,bloating and cramps. I think that ultrasound it at least what should be done to rule out cancer relapse and other serious diagnosis. But I would do CT scan too in your case. You shouldnt ignore these signs. ALso, do basic lab ant urine analysis to exclude anaemia and uroinfection. WIsh you good health."
},
{
"id": 34295,
"tgt": "What is the treatment for chicken pox in stomach , neck and back?",
"src": "Patient: my boyfriend has a pox on his stomach, neck & back that look like molluscipox. The dr said it would go away on its on but it has been over six months & getting more of them not less. he takes anti-rejection drugs for an organ transplant that weakens his immune system. what can we do to mke the pox go away? Doctor: Hello dear,Thank you for your contact to health care magic.I read and understand your concern. I am Dr Arun Tank answering your concern.Your infection arises because of the weak immune system.First of all you should diagnose yourself by ELISA testing. If it's showing positive reaction for the pox than you can start the appropriate antiviral drugs.This will cure the infection at earliest.As the organ has received we cant do of the drugs which is taken by us to withdraw the rejection.I will be happy to answer your further concern on bit.ly/DrArun.Thank you,Dr Arun TankInfectious diseases specialist,HCM."
},
{
"id": 170091,
"tgt": "Suggest treatment for choking and swelling on tonsil",
"src": "Patient: My sons tonsils have bee enlarged for a while, and now they are so swollen they are almost touching. My daughter had to have hers removed when she was three(my son is 2). I am making an appointment with my ent. Is there anything I can give him(clarion, or advil) to help relive some of the swelling and ease his chocking and gagging fits? Doctor: Hi...you can give Paracetamol or Ibuprofen at the appropriate dose. Paracetamol can be given in the dose of 15mg/kg/dose (maximum ceiling dose of 500mg) every 4-6th hourly that too only if fever is more than 100F. I suggest not using combination medicines for fever, especially with Paracetamol.Regards - Dr. Sumanth"
},
{
"id": 107623,
"tgt": "What causes severe back pain, discoloration at the point of pain with sporadic sharp nerve pain?",
"src": "Patient: When I woke up this morning I had killer back pain. So I rubbed my back like I always do with morning back pains and it hurts so bad in one spot. So I quit and went on with my day. When my husband got home I asked him to look at it. There s a 3inch diameter circle of discoloration where the pain is and a small painful lump in the center of it. It s up against the right side of my spine. When touched it sends pain through my spine and hip. Now that I ve touched it so much I keep getting sporadic sharp nerve pains that goes to my upper back and hips like an epidural shot that I recieved during labor. What could this be?? There is no holes from a bite nor does it look like an ingrown hair. Doctor: Its an Inter vertebral disc problem of spine where some dislcoloration has taken place causing spot pain, lump and radiating pain.Management can be done by knowing exact spot of such dislocation or reduction in gap or where radiation of nerve pain is originating from by having an x ray of AP and Lateral view of concerned region of spine or USG etc at Radiologist.Medicine wise only ant inflammatory and analgesics is the solution for temporary relief. Traction, lumbar, sacral mspine band etc would be the right direction for long term management and some vitamines and healers.At home front, you may start taking Turmeric powder 1/2 tsp in warm milk or water giving relief in pain and inflammation by natural properties of this herb.Hope it is helpful"
},
{
"id": 93306,
"tgt": "Bad cough, nauseous, pain in lower left abdominal area. What is it ?",
"src": "Patient: I have a really bad cough , I am sort of nauseous and I have a weird pain in my lower left abdominal area when I lay on my back. I have had this cough for about a week but the abdominal pain just started happening about 4 hours ago. My head hurts when there is a sudden noise and I get dizzy When I stand for a long period of time. What do you think it is? Doctor: Hello,First please take medications for you cough.Since you have been coughing for a week, there has been a lot of repetative abdominal wall contractions.This is the cause of your pain.I suggest you take cough reducing medicines and do not stress yourself while coughing.Hope this helps."
},
{
"id": 147437,
"tgt": "What causes tight feeling in head with numbness in body?",
"src": "Patient: My son falls asleep while he is having a conversation with someone, while driving, and sitting straight up. He says he can feel it coming on. The top of his head feels like a band is tightening up and he goes numb all over. He is 36 years old and I can't get him to go the doctor because he says he has no insurance. Please helpp me because I don't want to lose my son. Doctor: HIThank for asking to HCMI really appreciate your concern for your son looking to the history given here I could say that such case need to be kept under the observation because this could be central nervous system problem (Cerebral) and this possibility need to be ruled out and for that the CT san is the better way of investigation, hope this information helps you, have a nice day."
},
{
"id": 138507,
"tgt": "What causes swollen hand and fingers after a hand fracture?",
"src": "Patient: my 77 year old mother broke her radious and luna bone three weeks ago...she is in a nursing home waiting to see if she needs surgury...she is a diabetic....her hand and fingers have been swollen for three weeks...is this normal for it to be swollen for this long? Doctor: Dear patient Swelling of hand and forearm occurs after every fracture of forearm bones and persists for 3 to 4 days in normal cases if treated properly. If she is waiting for surgery due to diabetes following should be done 1. Above elbow slab should be given to splint the fracture 2. Forearm should be kept elevated on saline stand 3. Fingers should be moved actively 4. Medication like chymoral forte needs to be taken for 5 days initially. If all this things are not followed swelling persists. Onother reason may be tight slab application as it hampers venous drainage. So please check all this things and follow this properly swelling will reduce in few days. All the best."
},
{
"id": 65384,
"tgt": "Suggest treatment for lump on upper arm",
"src": "Patient: I have a hard lump on my upper arm right beneath the skin. It started off as the same color of my skin, but has now turned black. It itches if my shirt sleeve is too tight or if I carry a purse, the strap irritates it. Other than that, it is painless. I would like to have it removed. My doctor recommended to have it removed by a plastic surgeon. Doctor: Hi. Thanks for the query. The lump you have noticed can be one of many including a sebaceous cyst, lipoma (soft in nature), a parasitic cyst, a lymph node, etc. So the need for removal depends purely on the diagnosis. But if its really bothering you in your day to day life, I suggest you to get it excised."
},
{
"id": 25238,
"tgt": "Suggest treatment to control blood pressure",
"src": "Patient: My blood pressure has been extremely high 170/100 to 134/94. My psychiatrist feels it is an adverse reaction to Wellbutrin 300XL ( I have been taking it for approx 8 mo) He has weaned me off and my PCP has ordered dyazide. I have only been taking the dyazide for 5 days, pressures are still high no matter what time of day. Do you know how long the hypertension from the Wellbutrin will continue. I also take Fetzima which they feel may be attributing to the hypertension, the dose has been cut in half and will wean off by the end of the week. I have been unable to find any info on the internet. Thankyou. Doctor: Hi dear thanks for asking question.You are previously taking drug bupropion which is used for psychiatric problem.Usually not single factor lead to hypertension.You might have drug and stress induce hypertension.So first thing try to avoid stress.Avoid smokingEarly in morning do yoga.Do regular 1 hour exercise.Take low salt diet.Low fatty diet taken....Drink more water....If still with life style measures bp not comes down and remain above 160/90 consult doctor for starting antihypertensive medication.Hope your concern solved.Take care.Dr.Parth"
},
{
"id": 125382,
"tgt": "Suggest medication for increasing height",
"src": "Patient: Hi.My daughter have a diagnosos SED congenita-short stature.She is 24 years.I search method for increase height of statur with gen ingeneering-growth plate of bons/the work of prop=fessor Hellen Heber Katz of Wistar institut USA/There is a drug for growth taller.We are from Europe.I am Lyubomir. Doctor: Hello, Height is a genetically determined factor and can not be changed by drugs. The only possible option is complex orthopaedic surgery. Consult an orthopaedician and he will direct you accordingly. Hope I have answered your query. Let me know if I can assist you further. Take care Regards, Dr Shinas Hussain, General & Family Physician"
},
{
"id": 221749,
"tgt": "What causes no fetal detection in sonography after positive UPT?",
"src": "Patient: IM PREGNANT AND IVE HAD THREE SONOGRAMS, THE FIRST AND LAST SONOGRAM I RECEIVED THE BABY WAS DETECTED BUT A HEART BEAT WAS NOT. THE SECOND TIME NO BABY WAS SEEN ON THE SONOGRAM. WHAT COULD BE THE CAUSE? I WAS TOLD THAT ID BE SCHEDULED A D&C BUT WAS NOT TOLD WHEN. Doctor: You didn't mention about your gestation week. If it is more than 7 week and no fetal heart beat then it'd missed abortion and in that case you need d&c."
},
{
"id": 193296,
"tgt": "Why did I see blood drop after masturbating ?",
"src": "Patient: i masturbated last night and in the morning i woke up with blood in my briefs. i had a drop of brown blood on my brief and a big spot of bright red. i also have hemroids and i did eat hot peppers but the blood was in the front of my briefs not the back part. so where is this blood coming from ? and why? Doctor: Hi,You can neglect episodic single episode of blood in semen which is called hematospermia. If blood in semen persists than seminal vesicle inflammation or prostate infection could be possible. You need to investigate with trans rectal ultrasonography and semen culture for further work up. You can revert back with reports once done.Hope I have answered your query. Let me know if I can assist you further. Regards, Dr. Parth Goswami, General & Family Physician"
},
{
"id": 22812,
"tgt": "What causes weakness, dizziness, nausea, chest pains,shortness of breath and headaches?",
"src": "Patient: i have had 7 abnormal ekg's and failed my stress test. I suffer from weakness, dizziness, nausea, chest pains,shortness of breath and headaches. Swollen feet and ankles. My blood pressure is 95 over 55 on a good day and 83 over 41 on bad days. Nitro didn't help with the chest pains and Morphine helped me walk about with almost no problems. However there is no diagnosis. Im still in pain and no answers what do you think? Doctor: Hithis is most likely congestive cardiac failreadviceNT-pro bnp testSerum electrolytesStress thallium test to rule out cardiac functions , if stress thallium not available go for a 2 d echo As soon as possible .2d echo will reflect the functions of the heart , the left ventricle ejection fraction .after that copronary angiogram would be planned.most likely you are in congestiver cardiac failure due to blockage of coronary vessels.urgently contact a catdiologist he would start you on diuretics , nitrates, beta blockers and calium channel blockers.you low blood pressure is most likely because your lvef is low , so go for a echo asap.sorry but if you will neglect this , you can have a cardiac arrest soon"
},
{
"id": 83584,
"tgt": "Can Maxoza and fertomid cause any problem if used for long run?",
"src": "Patient: Hi I am taking Maxoza 5mg and fertomid twice a day to increase my sperm count and mortility level. Is there any side effects. I am also suffering from hypertention since the age of 17. Now I am 31 years old and married three years back and have no isuues. I am a bit worried. Please help me and suggest. Doctor: HiThese drugs should definitely not to be used in the long run due to the side effects.Monitor your sperm count and motility after a month and on improvement,you can slowly stop the drug.Hope I have answered your query. Let me know if I can assist you further. RegardsDr.Saranya Ramadoss, General and Family Physician"
},
{
"id": 203681,
"tgt": "What are the symptoms of adult phimosis and frenulum breve?",
"src": "Patient: Hello Dr. I am hoping i am speaking to a urologist with experience in adult phimosis and frenulum breve. This is about my boyfriend who is 42 years old. We started to make out recently-not sex yet-however i noticed that i never see the head of the penis. It is covered with the foreskin. I tried pulling it back but he had a lot of pain. He said he can retract the foreskin when flaccid. He uses oil. However i never saw the skin retracting when he is erect. He said he has been like this from childhood. My initial thought was that this is phimosis. He said his problem is a short frenulum. He also looks really small down there-like 3-4 inches when erect. So 1) does he have a frenulum breve or phimosis 2) Can a short frenulum cause the head of the penis not to be exposed at all? the photos i have seen of this condition show the glans pointing downwards. With this man it is not like that-instead the glans is not exposed at all. 3) could his small size be due to problems like phimosis or frenulum breve? Can these conditions limit erections? 4) if he gets treated for will there be any increase in size at least up to an inch? I am not looking for a big guy it is just his small size worries me. I am very much looking forward to your answers. I am very inexperienced myself so i have no idea what is going on and i really want to understand this issue. Doctor: HelloThanks for your query,based on the facts that you have posted it appears that your boyfriend has nothing but Phimosis from childhood .Please consult qualified general surgeon or Urologist for clinical assessment.He need to get circumcised .This is a simle surgery carried ut under local anaesthesia as a out patient procedure.The results of the surgery are gratifying and he will have normal erection and enjoyable sexual life through out rest of his life.Dr.Patil."
},
{
"id": 13949,
"tgt": "Do measles look like rashes with tiny spots?",
"src": "Patient: hi, i always thought that measles had a rash that was big patches of tiny spots, this photo is opposite, i have some friends that are roomates, with similar rash on their trunks somewhat like this photo, sporadic and larger, they are late 20 s early 30 s Doctor: Hello, There are multiple probable diagnosis for rash on trunk. It could be viral exanthem (Measles or Parvovirus infection) or drug reaction. If you have additional symptoms such as fever or itching, do consult your Dermatologist immediately. Hope I have answered your query. Let me know if I can assist you further."
},
{
"id": 24561,
"tgt": "What causes difficulty in breathing after an angioplasty?",
"src": "Patient: sir,My husband is 56 yeras old.weight 80kg.undergone angeoplasty in 2004.medical stent put. His stress test always shows ecg changes ,but doctor says it will show it even after plasty and it is okay. He is taking medicines for heart,high bloodpressure and cholestrol.From say 1.6 years , he is having some problems which I notices....1)while chanting *aarti* that is ,*mantras* when u have to chant long lines in one breath, he has to stop in between.2)Sometimes in his sleep, exhales through mouth3)unsound sleep4)feels like eating every now n then5)for last 6 moths has started chewing paan-parag,with tobacco and cannot resist it,though in limit6)last 4 moths gets irritated every now and then at anything .gets angry very fast which is contrary to his nature7)his teeth r really bad with this chewing tobacco habit.someone also suggested me to get ENT check done for him as oral hygiene is very important .also his father died of CANCER and mother of HEART ATTACK. he knows that chewing paanparag with tobacco is bad but cant control.8)his postlunch blood sugar is less than fasting sugar level.Please advise.regards Doctor: Pan parag and tobacco are very unhealthy things and need to avoid . If his ecg and echo shows some. New changes then need to under go angiography....if some blood test done ???.. if these are OK?!!if he can climb two floors comfortably ???"
},
{
"id": 196514,
"tgt": "Is it safe to take viagra while on Bisoprolol fumarate?",
"src": "Patient: I take Bisoprolol Fumarate (5mg) daily for mild hyper tension and control of congenital SVT. I am 48 years old, in good heath, other than smoking 10 small cigarettes per day and I consume 2-6 oz of alcohol nightly. Is it safe for me to take Viagra when needed, minimal dosage? Thank you! Doctor: Good day\u200b and thank you for being with healthcare magic!!!If you are in a good health and can climb 3 flights of stairs without chest pain then I don't see any reason why you can't take Viagra. Bisprolol has no interaction with Viagra and won't be a problem. I hope I have answered your question satisfactorily. Please consider a 5 Star rating. Thank you."
},
{
"id": 118089,
"tgt": "What causes elevated ESR levels and suggest diet?",
"src": "Patient: HiMy mum has just conducted an ESR test. Her results is 39.00.It is mentioned that normal range should be between 0.00 - 15.00.Please advise what could be the cause? And if there are any foods that she should stop eating? And what foods should be recommended for her to consume more so as to lower her ESR?Please advise.Many thanx. Doctor: HIThank for asking to HCMI really appreciate your concern looking to the history given here I would like to say that ESR is just indication of some infection and shows that subject is ill this included in the routine test of blood and some time the ESR carried out alone to confirm the doubt of some infection, one can not go and get done this test without any suspected pathology, hope this helps you, have a nice day."
},
{
"id": 189170,
"tgt": "Partial plate on top needs replacement and bottom teeth are loose. Need dentures?",
"src": "Patient: Thank you! I am 66 years old and retired. Although I have medicare, it doesn't cover dental. I live on a very limited income, (below poverty level) and I need some major dental work done. Can you guide me inthe right direction? I have a partial plate on top that's old and needs replacing and my bottom teeth are very loose. I'm thinking Imneed dentures. Doctor: Hello and welcome to hcm, If your dentures are ill-fitting,please get it replaced as soon as possible. The ill-fitting dentures can interfere with your speech,occlusion,chewing food etc. Removable dentures are not costly,please go for it. Always keep your denture clean . Fungus can grow,if denture is not cleaned properly. Always maintain good oral hygiene too. Chew the food equally with both the sides. hope this helps."
},
{
"id": 117788,
"tgt": "Advise on the blood report with hemoglobin 11.4% for fever",
"src": "Patient: Hello,My wife getting fever his body temperature in thermometer showing 98.9 to 99.9 degree appox. 15 days.blood report tlc-7900. hemoglobin is 11.4%. urine microscopic exam. red blood cells 2-3, crystals- calcium oxalate detected(+), blood-detected(trace). Doctor: Hi,Thanks for asking.The given blood and urine report are almost near normal.The temperature is appearing variable.Details are limited to suggest anything.However, for your query, Hemoglobin of 11.4% is normal and you need not worry about it. Evaluation of fever is necessary. If you can provide more details, i will try to help you.Any further queries, happy to help again."
},
{
"id": 21442,
"tgt": "What causes dizziness and chest pains?",
"src": "Patient: i am 20 and i get dizzy when i turn on my bed or get up.. i ve fallen twice, once when i was getting up off the floor and another time when i was standing near my car. I also had a chest pain when i at a friends but i don t know if it had anything to do with that. Doctor: Hello, I understand how worried ou are about your condition. Your fainting can be caused by lots of factors, could be underlying heart diseases, hypoglycemia etc. You have to visit a physician to have a total examination to be able to determine the cause of your problem. Warm regards"
},
{
"id": 103527,
"tgt": "Have swollen eyes, runny nose and congestion.Was attacked by bees a year ago. Remedy?",
"src": "Patient: I WAS ATTACKED BY BEES ABOUT A YEAR AGO (ABOUT 20 STINGS AND DIDNT GO TO THE DR. ,IT DID NOT HURT THAT MUCH SO I JUST USED BENADRILE SPRAY) OVER THE LAST 9 MONTHS I HAVE DEVELOPED SEVERE ALLERGIES( SWOLLEN EYES,RUNNY NOSE CONGESTION). SAW AN OPTHOMOLIGST AND EVERYTHINGS GOOD.SHOULD I SEE AN ALLERGIST TO TRY TO HELP WITH THESE SYMPTOMS? Doctor: Hello,Bee stings are known the produce allergic reactions.The allergic reaction may be associated or may not be associated to the bee string.If it is associated with bee sting doctors administer immunotherapy for the long term management.Consult your doctor for more information."
},
{
"id": 184918,
"tgt": "What causes sensitive tongue and palate?",
"src": "Patient: what is the cause of a sensitive tongue/palate I am 63 years old, generally good health just been prescribed crestor for high cholesterol. I am 5'4\" and weigh 8st4oz. I regularly go to a dentist and attended at periodonitis for 8 years as I have gum disease. I also get a lot of blood blisters in my mouth. Doctor: hi,Good evening, I went through your question would be answering it precisely,1. Due to age sometimes a patient develops \" Candidiasis \" ( a Fungal Growth )This usually happens on palate or tongue or corners of mouth due to humid conditions there.2. Other conditions which we call it as \" Red & White \" lesions can not be rules.3. Blisters are water filled cavities which causes more hypersensitive palate or tongueI suggest u to immediately contact the Specialist dentist ( Oral Medicine & Radiology) who would help u out in this regardsAlternatively in the mean time u can apply OINTMENT TESS Twice Daily on affected area.Hope my answer helps u to get information u required. Do get back without any hesitation for further questions"
},
{
"id": 129024,
"tgt": "Will taking Clexane be safe for vein stripping in left leg ?",
"src": "Patient: Hello. I started taking Clexane 40mg on the 25 October, 9 days ago. I had vein stripping in me left leg on the 24 October. I'm supposed to inject myself with 40mg once a day/once every 24 hours. I just cant remember if I injected one today or not. Should I inject one now just in case I forgot or should I wait till tomorrow. I dont know what to do? Please advice. Thank you Doctor: Hello,Thank you for using healthcaremagic.I read your question and understood your concern.I think it is better to inject one right now than to miss a dose.The risk of missing a dose is greated than the risk of doing double dose of Clexane.Dr. Selmani"
},
{
"id": 219358,
"tgt": "Suggest treatment to get rid of morning sickness",
"src": "Patient: i am 9 weeks pregnant and i wanted to know if it would be harmful for me to smoke pot for morning sickness? i have heard this worked for some people but i am afraid to do it because i am high risk and have had cramping and bleeding. they said everything is ok now i just need to rest alot but i just wanted to know if i did smoke when i felt sick if it would have any harmful effects on the baby? Doctor: Hi, Thanks for your query. I understand your concern. Yes, smoking marijunia pot in pregnancy has tremendous side effects in fetus & mother both. It may seize morning sickness vomiting, but the side effects it causes are hazardous.( name physical / mental/ intellectual / cognitive side effects in new born.. making it's life miserable. To control morning sickness you can- Chew dry snacks slowly before getting out of bed( water or liquids stimulate vomiting., be in bed for half hour & then get up to start your routine. Never starve. Take small/ multiple servings( 5-6 in a day ) of nutritious food you relish. Avoid spicy, oily or left over food.Avoid aerated drinks, _ get prescription from your doctor for preventing morning sickness.( like Doxinate pills 2 at bed time ) - Consult your doctor regularly for pregnancy check up. - Low blood pressure increases sickness.. Some times IV fluids can control such excessive vomiting. - Think that it's pregnancy hormones that cause the symptoms.. so your baby is getting good hormones & is growing well. This will stop when your body gets adjusted to the hormones.( usually by 3 months of pregnancy ) It's helpful to be positive. Thanks."
},
{
"id": 95745,
"tgt": "What treatment should i take for Stomach ulcer for the past 8 years ?",
"src": "Patient: I am a 26 year old lady and have been suffering from stomach ulcer for the past 8 years.I tried so many medication and even tried injections, i dont get better and they work for a short period.Im worried because now started to feel as if its swollen in the stomach and also feeling pains especially when I eat after swallowing food.I ve lost food appetite but i always force myself to eat to avoid hunger.Are the any chances of getting Cancer from Ulcer?Can it be cured?What treatment can i take? Doctor: Hello, Thanks for posting your query. Since you did not find relief with medicines you can go for surgical management. There are many surgical procedures for gastric ulcers. Chances of ulcer transforming to cancer are there though rare. Consult a general surgeon, he is the right person. You can also directly consult a surgical gastroenterologist. Hope I have answered your query. Wishing you good health. Regards,"
},
{
"id": 66579,
"tgt": "Is it to be concerned about the lump on the heel?",
"src": "Patient: Hi i recently discovered a fairly big triangular white lump on the ball of my heel. its kind of hard and it doesn t hurt, though its uncomfortable when i walk or put pressure on it. i play baseball so i think thats where it came from but i would like to know what it is. its also on my other foot but smaller and more circular. do you know what it is? Doctor: Hi, thanks for sharing your health concerns with HCM!If I were your treating Doctor for this case of heel swellings, I would come up with three possibilities, these include: 1.\u00a0\u00a0\u00a0\u00a0\u00a0wart-like benign squamous lesion 2.\u00a0\u00a0\u00a0\u00a0\u00a0The second possibility is of a bursa or ganglion cyst \u00a0\u00a0\u00a0\u00a0\u00a03.\u00a0\u00a0\u00a0\u00a0\u00a0The last possibility is of some gouty tophus or some infection /trophic changes! I suggest you to go for a physical examination by some surgeon and to relieve your concerns!Hope this answers your question. If you have additional questions or follow up questions then please do not hesitate in writing to us. I will be happy to answer your questions. Wishing you good health."
},
{
"id": 101992,
"tgt": "Any suggestion for having seasonal allergies and sinus congestion?",
"src": "Patient: Seasonal. Allergies and sinus congestion Hey.Im 31 years old..My allergies seem to be out of control u this year..I stay congested all year around and nothing seem to help ..Im on nose spray, asthma medicines and still have trouble..Any ideas that will help me stay uncongested? Doctor: Hi, first of all go for a head and neck X ray in caldwel luc view and water's view to rule out other pathology of sparanasal sinuses like polyp, mucocele or chronic sinusitis, then meet an ENT specialist to have a through examination including rhinoscopy to assess your condition .let them diagnose the type of rhinitis you have , go for complete pannel allergen test to get the list of all substances which are allergic to you, also go for pulmonary function test . proper se of levocetrizine , montelukast , oral or topical steroids and antibiotics can manage your symptoms perfectly for which you should follow your ENT specialist. take care."
},
{
"id": 63345,
"tgt": "What causes lump on neck of a 3 weeks old ?",
"src": "Patient: Hi, my brother's son is 3 weeks old and yesterday we noticed a lump on his neck. We took him to the doctors today and did an ultrasound and we still dont know if it is from the muscle or the veins. We will do another and detailed ultrasound tomorrow. what could it possibly be? and what is the difference if it was a muscle or vein lump? Doctor: hi.which part of his neck is affected? does the lump go with fever and are there signs of infection?there are varied possibilities. but with his age, it could be a cystic hygroma, hemangioma, an arteriovenous malformation, a lipoma.. it is best if you consult with a pediatric surgeon."
},
{
"id": 184648,
"tgt": "What causes burning sensation and white patches on tongue?",
"src": "Patient: hello my husband had been having this burning sensation and white patches on tongue..doc thought candida gave meds for it helped some but same time ive been making him gargle with lemon juice....has helped a bit but now doc thinks its not candida....i also make him scrape it off after he swishes....doc also said doesnt look like cancer at all pharm. said same thing he said looks like dry mouth.....he also does smoke Doctor: Thanks for your query, I have gone through your query.The burning sensation along with the white patches occurs in candidal infection and erosive lichen planus. or it can be two separate lesions like leukoplaki(white patch since you are giving history of smoking) and erythematous candidiasis(redness and burning sensation).consult a good oral physician to rule out these two conditions. If it is candidiasis then you have to take topical antifungal like candid mouth paint.If it is leukoplakia, you have to stop the habit of smoking and take antioxidants like capsule lycopene.if it is lichen planus then you have to take topical steroids like triamcinolone acetonide (0.1%) 4-5times daily for 1 week. You can also take topical anesthetic and analgesics like anabel gel for the pain. I hope my answer will help you, take care."
},
{
"id": 223830,
"tgt": "Does Mirena insertion cause back pain with bleeding slow down?",
"src": "Patient: I just had Mirena put in last week. The day it was put in I was still bleeding heavily from my period. By the time I got home I was not bleeding at all. I have had constant lower back pain and now have stringy blood and small blood particles in my urine. I have no blood flow on a pad. Is this normal? And will the back pain stop? Doctor: Hallow Dear, Getting Mirena inserted during phase of bleeding is not a very good idea. You should have got bleeding controlled by progesterone or waited for stoppage of bleeding before getting it inserted. Mirena in an intrauterine device which contains Progesterone. Slow release of this progesterone does bring down the uterine bleeding. However, because of intrauterine intervention while inserting it during the phase of bleeding, there are chances of infection. Now your stingy bloody discharge along with low back ache is highly suggestive of infection. Please report to the Gynaecologist. You will require a course of antibiotics and may be you might have to get the device removed for infection. I hope this helps you. Dr. Nishikant Shrotri"
},
{
"id": 216722,
"tgt": "Suggest treatment for fibromyalgia",
"src": "Patient: I was diagnosed with Fibromyalgia 8 years ago. I am now experiencing tenderness to the touch on my skin which is different then my normal symptoms from fibro. When I touch my skin if feels as if I am touching a tender bruise and this is all over my body. What could this be indicitive of? Doctor: hi,thank you for providing the brief history of you.A thorough neuromuscular assessment is advised.As you have fibromyalgia - fibro-myal-gia is a condition where there is inflammation in the fibres of the muscles. The muscles contains numerous fibers and based on the contraction of the numerous muscle fibers the muscle performs the movement.Since there is inflammation there is also a pain in the muscles which later happens every now and then. Now muscle is connected to the nerve and also the skin.As the circuit is enclosed in a most complicated fashion transformation of the pain to the nerve , muscle fiber and the skin are vice versa.In my clinical practice patients with fibromyalgia will have not one but numerous issues. solving each queries slowly one by one helps them to come out of the painful like a provide a better functional life.We start slowly with a combination of 3 major therapies along with drug therapy. 1. meditation2. physical therapy - ultrasound, TENS and exercises3. Breathing exercisesnow this combination along with drug therapy works in a uniform pattern. Like meditation help to avoid anxiety & aggression, Physical therapy - like therapeutic ultrasound therapy, TENS therapy and exercises helps to reduce - pain , inflammation and metabolic waste. The last is about respiration - having good Oxygen content in blood will help improve the metabolism and immune system.Majority of cases seen by this methods in my clinical practice have provided better results.RegardsJay Indravadan Patel"
},
{
"id": 103580,
"tgt": "Having Asthma, on treatment. Is this curable?",
"src": "Patient: hi ,this is rabia. I had a frequent symptoms of bad cough from almost a year.I was coughing(with mucous) a lot from last few days so i visited doctor, he suspected that i have asthma so he asked me to get x-rays done.From the reports he confirmed that and prescribed me an inhaler and an antibiotic. I work as a early childhood educator in a daycare.i was never sick before i joined daycare.Is it from working with kids .Should i continue working in daycare because i am really fed up from being sick all the time and coughing ?Is this curable or i have to live with inhaler and these antibiotics throughout my life? Doctor: Hi, Asthama is a breathing disorder which is mostly allergic in origin. there is difficulty in exhalation producing a weezing sound.sometimes infections produce same symptoms which is labelled as Bronchitis. I feel you need more investigations apert from x ray. like Hb%, CBC, Mx test, allergy tests,culture of sputum if needed. this wii confirm your diagnosis &also the line of treatment. Do you have family H/O asthama or allergies? Evevn if it turns out to be asthama -pl. dont be depressed-there are wonder drugs which along with few life style changes -make patient's life life normal."
},
{
"id": 174021,
"tgt": "What causes red rash on thighs and legs?",
"src": "Patient: My nephew has a red rash covering his thighs, back of legs, and stomach, there were patches on his shoulders an back but those went away(rest of his skin tanned, those stayed white). One Dr said dermititis and another said heat rash. Neither cream they gave has worked and It has spread across and down his legs more. Doctor: Hi...Thank you for consulting in Health Care magic.By what you quote it should be an urticarial or a simple skin allergy. You can use Hydroxyzine at 1-2mg/kg/dose every 6th to 8th hourly for 7-10 days. Most important thing to be remembered is that it has a propensity to recur (called as second crop) within 10-14 days. If this happens, you can start using the same medicine but I suggest you get the kid evaluated with your paediatrician.But skin conditions are best diagnosed only after seeing directly. I suggest you to upload photographs of the same on this website, so that I can guide you scientifically. Hope my answer was helpful for you. I am happy to help any time. Further clarifications and consultations on Health care magic are welcome. If you do not have any clarifications, you can close the discussion and rate the answer. Wish your kid good health.Dr. Sumanth MBBS., DCH., DNB (Paed).,"
},
{
"id": 135559,
"tgt": "What causes numbness & muscle pumping during workouts?",
"src": "Patient: HI,Yesterday while working out I started to go a little numb and tingly in my forearms and hands. My muscles were really pumped in those areas so I thought it could be the reason for the slight numbness. They were so pumped possibly that they were cutting off blood flow I guess. Anyway, is there a certain reason for this? Should I hydrate more, eat better, or possibly just take it easier when working out. I workout frequently and stay healthy, I have just never had this happen before. Thanks Doctor: HiU seem to b having some sort of vitamin or calcium deficiency U shud get an detailed blood profile done n let me know Otherwise everything should b fine n nothing to worry bout Take care"
},
{
"id": 53790,
"tgt": "Need treatment for cyst on the liver, enlarged pancreas and gallstone",
"src": "Patient: Following numerous scans it has been confirmed that I Have a large retroperineal cyst lying on my liver, enlarged pancreas ducts and a 6.5mm gallstone. I am waiting for a follow up appointment with my consultant. In the mean time what do u think his course of action may be? Doctor: Hi and welcome to HCMThe only possible solution would be surgical procedure and I suggest to do it as soon as possible before cyst starts to compress surrounding structures.This may be benign or malignant one and from this it looks like more to be benign, but in every case it should be surgically removed.Wish you good health. Regards"
},
{
"id": 31028,
"tgt": "Suggest treatment for high fever, cough & pain in the limbs",
"src": "Patient: I Mrs. Bandana Som, age - 43, is suffering from symptoms like high fever at every evening, coughing, pain at hands and legs. Then I met with my family doctor had few tests. The reports are :- 1) Chest PA view shows patchy opacities at Rt.lung upper lobe. Few tiny calsifications are noted at Lt. upper lobe. Both costophrenic angles are clear.Likely to be Reactivation of koch's lesion at Rt. side.Plasma Glucose( fasting ) = 130.00 mg%Plasma Glucose( P.P.) = 157.00 mg%Now I had been told I got T.B.Now, I Want to be get treatment get well soon. Can you help me ? Doctor: Dear Ma'am, Hi & Welcome.I can understand your concern. Tuberculosis is 100% curable disease. To treat active TB, it is necessary to take several antibiotics at the same time for at least 6 to 9 months depending upon the severity.For the 6 months regimen;You are required to take 4 antibiotics like Isoniazid, Rifampicin, Ethambutol and Pyrazinamide ONCE daily for the first TWO months (Initial phase). This is followed by administration of 2 antibiotics like Isoniazide & Rifampicin ONCE daily for remaining 4 months (continuation phase). There after you will be rechecked again by certain lab tests such as Sputum for AFB, ESR, Chest Xray, etc to check the response to the treatment.Along with medications you need to take a high protein diet, increase the intake of fruits and vegetables and Vitamin B6 (Pyridoxine) to prevent the side effect of inflamed nerves caused by Isoniazide.Your fasting blood glucose level is high, so you also need to take medications like Metformin with or without Sulfonylurea like Glimepride to control the high blood glucose after consulting a physician.Hope the suggestions given above would be useful."
},
{
"id": 203141,
"tgt": "What is a painful large bump in my pubic area?",
"src": "Patient: I have quite a large bump situated in my pubic area near my penis and testicles. It was initially small, but it has become bigger over the past week or two. It is slightly painful, and it is soft. What could it be and would Kenacomb be a good cream to apply? Doctor: Hi,From history it seems that you might be having some hair follicle infection giving rise this problem.Go for one antibiotic medicine course for 3-5 days.There is another possibility of having inguinal hernia.Consult surgeon and get examined.Ok and take care."
},
{
"id": 147792,
"tgt": "What is the cause for swelling, numbness in the right arm in the middle of the night?",
"src": "Patient: the last 2 or 3 days I have been waking up in the middle of the night with my right arm swollen and numb they feel like they are ready to explode and after I shake them for a while they go back to normal except that I am still sensitive in my fingers Doctor: numbness and tingling during night time is characteristics of carpel tunnel syndrome.B/C OF COMPRESSION OF NERVE at wrist NOT IN NECK.seen in middle age female.with diabetes,thyroid,hormonal disturbances.take advice to local doctor,may required couple of tests-EMG/NCVMRI cervical spineblood sugarTSH"
},
{
"id": 218103,
"tgt": "Cause for pain in hip area radiating to leg causing numbness, tingling and burning",
"src": "Patient: The pain in my right hip area started about a month ago. It came on unexpectedly and without an injury or accident happening. It started as just an ache that I thought was arthritis. After two weeks the pain then started affecting my leg. The pain will radiate down my right leg and cause numbness, tingling and burning. My doctor has guessed that I have sciatica and has treated me for that. I haven t noticed any changes or improvements with the medications she has prescribed to me (Naproxen, Flexaril and Prednisone) X-Ray s showed no abnormalities. In the past three or four days I haven t been able to straighten my back. When I try to straighten it I feel slight pain in my lower back, but the pain isn t very severe. It s more that my back simply will not straighten, not due to pain, just due to inability to. What could be causing this? Doctor: Hi there, Thanks for your query. You have not mentioned your age, as there may be spontaneous collapse of vertebra in osteoporotic elderly people, causing 'pinching' of the nerve supplying the lower limb. Even a trivial event (which the individual may not even remember) may cause it. MRI of lower spine will confirm the diagnosis, as it may be seen in a normal X-ray. Other causes can be spasm of Piriformis muscle (called Piriformis Syndrome), or compression over the nerve at the hip joint. Nerve-pain-killers, such as, carbamazepine- used for treating epilepsy) will relieve the pain. Consult your doctor and apprise him of my opinion. I am certain that he will agree with me, get MRI done and will prescribe the advised drugs in appropriate doses. If you find my response helpful and informative, do not forget an \u201cexcellent\u201d (5-star rating) to my answer, to ENCOURAGE ALL doctors- engaged in social service- to render sound advice to the FREE queries. Take care Dr. Rakesh Karanwal"
},
{
"id": 165243,
"tgt": "What causes shoulder pain in toddler after a fall when x-ray is normal?",
"src": "Patient: How long should a child of three years of age have shoulder pain for following a fall 3 weeks ago which the G.Psaid noacute injuries seen on x-rays. Straigth & apical x-rays taken. NB Child born following shoulder dystocia.could this fall have aggravated anything. Concerned grandparents Doctor: X-ray shoulder can detect dislocation or fractures at shoulder joint. Ligament and cartilage injuries are not detected by X-ray. As he is having pain since three weeks, probably he had suffered some ligament or cartilage injury.Give paracetamol and ibuprofen for pain relief. Consult orthopedic surgeon who will advise USG shoulder to detect ligament or cartilage injuries."
},
{
"id": 33964,
"tgt": "What causes shortness of breath?",
"src": "Patient: I have copd, presently I am on sprivia and advair. my shortness of breath seems to be getting worse since the advair was added. I also have had a number of times where I got what appears to me is a cold or flu, I get the chills, ackes and piaings etc, I Doctor: Thanks for posting you query to health care magic.Advair do not causes shortness of breath as it is a steroidal drug and usfull in releiving inflammtion of respiratory tract given to releive asthmatic attack.your shortness of breath may be due to some other causes .as you are presently suffering from flu like symptom so there may be possibility of some kind of infection and for which you need to contact to local physician for examination .you can take Paracetamol in 750mg adult dose two times a day to releieve pain and body ache .furhter treatment will be decided after examination by physician .Hope you would be satisfied with my answer . Feel free to communicate if any query .regards,Dr.Manish PurohitInfectious disease specialist"
},
{
"id": 157552,
"tgt": "Smoker. Having sharp pain in lungs, testical and penis. Are these symptoms a sign of cancer?",
"src": "Patient: For a very long time, I have had sharp pains on the sides of my lungs and now the pain doesn t go away. I used to be a heavy marijuana smoker and when I did smoke, my testicles/ penis would be in a lot of pain. I have confirmed this by not smoking for awhile then I smoked once more when the pain came back. I am honestly not a healthy person. I am 23 years old and I weigh around 240lbs. Are these symptoms a sign of any type of cancer? Doctor: Hi and welcome to HCM,thank you for your query.Cancer is not the first thing to think of and cancers,especially lung cancers are very rare in your age. Testicle cancer i more common but also I dont think this is likely to be malignant disease. however you should do some basic tests such as lung xray and testicular ultrasound just to rule out some other diseases.Wish you good health. Regards."
},
{
"id": 46246,
"tgt": "Suggest treatment for blood in urine and pain in kidney",
"src": "Patient: i take meow almost every weekend and i am gettin worried as i have been having problems with my kidneys and weeing blood.... i have constant water infections and sometimes end up in hospital on morphine coz of the pain in my kidney, i also have noticed my personality has changed and i have lost almost 5 stone in a year....please could you help me or reeassure me that its not the meow ?? Doctor: Pain and blood in urine is commonly due to stones . So a CT kub is advised and after the evaluation furthur plan should be made for surgery like pcni, ursl rirs which is appropiate."
},
{
"id": 116675,
"tgt": "What is the target level of INR before undergoing general surgery?",
"src": "Patient: sir, for a patient on t.acitrom for post mvr ( ttk chitra valve).assymptomatic at present .now posted for hysterectomy.inr is 1.33.how long should i stop acitrom.should i give heparin.what is the target for patient undergoing general surgery Doctor: Hi, dearI have gone through your question. I can understand your concern. You should stop acitrom before surgery. INR should be around 1 at the time of surgery. After surgery you should start your anticoagulant. INR should be maintained between 2 and 3 afterwards. Consult your doctor and take treatment accordingly. Hope I have answered your question, if you have doubt then I will be happy to answer. Thanks for using health care magic. Wish you a very good health."
},
{
"id": 189043,
"tgt": "On dental anesthesia. Full teeth decayed. How to prevent from side effects of dental anesthesia?",
"src": "Patient: i have been on dental anesthesia for 11 months now as i have fully teeth decay and i need to fill all my teeth and crown most of them !! i previously asked about the side effects of long term dental local anesthesia and the doctors here recommend that i should stop it however i could not because my teeth hurt and my doctor said that i can neither stop procedures now nor can i bare the pain without anesthesia .... actually i am terrified form the side effects you mentioned like syncope, nervous and cardiac problems i do not know what to do !!! any help ?? Doctor: hi,Thanks for asking the query,I would suggest you to take local anesthesia during the Dental only when it is needed, if you cannot bear the pain you can take an analgesics prior the procedure.Avoid anxiety and stress.If you are having any systemic conditions like hypertension, cardisc deisease consult a Physician before the procedure.Take care."
},
{
"id": 49058,
"tgt": "Is it normal to have left flank pain post stone and urethral stent removal?",
"src": "Patient: I had a left urethral stent place on 6/7 for a 7mm kidney stone. i never passe the stone despite the stent being in and medication. I had the stone and stent removed 7/1 and continue to have left flank pain. is this normal and for how long does it usually last thanks Doctor: Good Day and thank you for being with Healthcare Magic! I would suggest getting another CT Stonogram to check the cause of the left flank pain and if the stone is still there then I would suggest a ureteroscopic procedure to remove the stone definitely. I hope I have succeeded in providing the information you were looking for. Please feel free to write back to me for any further clarifications at: http://www.HealthcareMagic.com/doctors/dr-manuel-c-see-iv/66014 I would gladly help you. Best wishes.Regards, Manuel C. See IV, M.D. DPBU FPUA"
},
{
"id": 114647,
"tgt": "What could cause monocytosis and weight loss?",
"src": "Patient: I am a family medicine practitioner. One of my patients, aged 23, weighing 55kg. shows monocytosis, but total count is normal. What investigation is relevant , the patient c/o loss of weight and inability to gain weight.My e-mail i.d. is YYYY@YYYY Doctor: Hi, dearI have gone through your question. I can understand your concern.It can be due to thyroid condition or some infection like tuberculosis or due to some other cause. Please tell me about your other symptoms and reports.Hope I have answered your question. If you have any doubts then feel free to ask me. I will be happy to answer. Thanks for using health care magic.Wish you a very good health."
},
{
"id": 1862,
"tgt": "Will Hiberix affect the process of IVF?",
"src": "Patient: Hi. Im Soni, 32 years old Femaie, Currently Im in process of IVF & soon i will be having egg collection day. Right now im using Decapeptel injection & Gonal-f also. What i would like to know is that can i take Haemophilus type b conjugate vaccine IP (Hiberix)? if i take this will it affect the process of IVF & eggs ? Doctor: Hi, you should avoid taking it now, because it may need harmful for your baby. If embryos are going to be frozen and will need transferred after 2 months or more, then you may think of taking it. But if embryo transfer is planned this cycle, don't take it. Hope I have answered your question. if you have any other query I will be happy to help.Regards Dr khushboo"
},
{
"id": 75741,
"tgt": "What causes sharp chest pain while climbing stairs?",
"src": "Patient: hello iam 21 years old and of average size. Iam experiencing sharp chest pain and find myself out of breath just from going up the stairs. Im worried because heart disease does run on my family. my aunt was only 26 when she passed and my mother had an aorict disection in her early 30s. I could really use some help! thank you Doctor: Thanks for your question on Healthcare Magic. I can understand your concern. Climbing stairs is considered as heavy exertion. So you are having chest pain and breathing difficulty on heavy exertion. You are also having strong family history of heart diseases. So better to first rule out heart diseases in your case. So get done ecg, 2d echo and stress test. If all these are normal then no need to worry for heart diseases. Sometimes lung disease like asthma or bronchitis can also cause similar symptoms. So get done clinical examination of respiratory system and PFT (pulmonary function test). Don't worry, with treatment, you will be alright. First diagnose yourself and then start appropriate treatment. Hope I have solved your query. I will be happy to help you further. Wish you good health. Thanks."
},
{
"id": 75763,
"tgt": "What causes weakness and breathlessness while walking?",
"src": "Patient: I get out of breath and weak during daily hot shower. Then I have trouble standing up to shave at the sink after the shoer. Seems like lifting my arms is a burden so I have to lean over the sink. I had by-pass surgery more than ten years ago.Some days I also am very weak during my daily walk and have to find a place to sit down before returning home. This is particular to humid weather. Could I be having blockage problems again? Doctor: Hi thanks for contacting HCM..You had history of cardiac bypass surgery ...And this time complaint of weakness and dyspnea on walking present ..So cardiac cause like ventricular failure has to be ruled out by .....1.EKG 2.Stress test 3.ECHO (if needed )Your blood pressure also should be checked ...As history of weakness present rule out anemia also ...by HB estimation....According to cause present further treatment guided ..Ex.if heart failure present than diuretic like drug prescribed ....Consult physician or cardiologist with keeping this in mind. ...Take care ...."
},
{
"id": 218261,
"tgt": "Is a quad test showing trisomy and NTD a serious concern?",
"src": "Patient: hello mam i have done the Quad test but in the report Ultra Slund Details are like........the reportable range for trisomy 21, trisomy 18 and NTD is from 1:50 to The reported value which is either 1:50 or below 1:250, 1:100 and 1:50 for trisomy 21, Trisomy 18 and NTD respectively, indicates high risk please suggest Doctor: Hi, The risk of Trisomy 21 and Trisomy 18 increases when the ratio is less than 1 in 250. 1 in 50 indicates that the risk of Trisomy is high, but quad test is a screening test. To confirm I would suggest amniocentesis. Amniocentesis will tell us the baby's karyotyping thereby confirming whether baby is affected or not. Hope I have answered your query. Let me know if I can assist you further. Regards, Dr. Swathi Kamidi, OBGYN"
},
{
"id": 83855,
"tgt": "Is Indocap safe for treatment of back pain?",
"src": "Patient: HI I AM DINESH AGE-28 SUFFERING FROM SACROLITIES FROM LAST 4 YEAR ,THIS DESEASE MAKE ME AWRE ONLY WHEN I HAD ACCIDENT BY BIKE. AS PER DOCTOR ADVICE THIS CANT BE TREATED IN FULL MODE. I AM USING INDOCAP SR WHEN I FEEL LOWER BACK PAIN,PLEASE SUGGEST IS IT HARMFUL IF I WILL TAKE IT ON REGULAR Doctor: Hello, This is an NSAID , long term use may lead to renal failure,peptic ulcer disease and pedal edema etc. So take only if pain is severe. You may use tramadol to relive pain which may have less side effects on kidney. I think I answered your questions still have queries please feel free to ask. Hope I have answered your query. Let me know if I can assist you further. Take care Regards, Dr. Penchila Prasad Kandikattu"
},
{
"id": 162278,
"tgt": "What causes vomiting and diarrhea in a toddler?",
"src": "Patient: Hi, my 4 year old son has been vomitting and had diarrhea for well over a month. He has no fever with it at all. He is ok for a couple of days and then starts it all over for 2 to 5 days. He has seen the doctor and they can t figure anything out. He has lost 10 lbs in like 3 weeks. He is already so little for his age. Please help me, I am running outta options. Doctor: Hello, You tell me your toddler is having intermittent vomiting and diarrhoea. The intermittent aspect of his illness suggests that he might has cyclic vomiting syndrome(CVS). About one-third of the child who have vomiting also have diarrhea. He is the right age. Usually, the diagnosis is more likely if there has been a longer history of illness. However, he has had a significant weight loss and it would not be in his best interest to wait any longer. I want you to take him for evaluation to a pediatric gastroenterologist AS SOON AS YOU CAN. Hope I have answered your query. Let me know if I can assist you further. Take care Regards, Dr Arnold Zedd, Pediatrician"
},
{
"id": 94449,
"tgt": "Pain in belly button, on left side of stomach, gets worse on walking. Cure for pain?",
"src": "Patient: Hi, I have pain in my belly button and my left side of stomach around the belly button. I ve noticed it for a few days when running up stairs. Last night I lifted a heavy load of washing and the pain was very intense and made me feel sick. Since then I ve been in more pain - it comes and goes but is worse when walking etc. I don t feel well in myself I couldn t get an appointment today and am a bit worried. Doctor: Hi. Welcome to Health Care Magic. It seems that you might be having recti muscles catch. Take rest and give rest to abdominal muscles. Apply some local analgesic cream. take some analgesic medicine like Diclo-para combination with antacid. Within a day or two you will be alright. Ok and bye."
},
{
"id": 41248,
"tgt": "What causes infertility?",
"src": "Patient: Hi,i am 28 yrs old, I am married,my wife has children,I was five yrs ago in a relationship i was 19 yrs old and we didn't use protection she was 18 yrs old and that relationship no baby it lasted 3 yrs after that i stayed single..I just months ago found out i have a nine yr old son. From some girl i had a sexual relationship in 2001,I been trying to get my wife preganic is there a reason why i haven't got my wife preganic,i want so much to have a baby with my wife its been a stressfull ride,Can you help me, Angel Doctor: Hi, I have gone through your question and understand your concerns. Infertility can be due to many causes, like problems in semen, or disturbances in ovulation, tubal function or uterine environment.Both the partners need to undergo investigations to find out the reason for giving appropriate further management.I would suggest you to consult an infertility specialist to get properly examined, investigated and treated accordingly.Hope you found the answer helpful. Wishing you good health.Dr Deepti Verma"
},
{
"id": 164664,
"tgt": "Suggest treatment for loose stools and diaper rash in a child",
"src": "Patient: My baby has been pooing 5-6 times a day for a week. In the last few days it has increased to 7 and they are very watery with green stringy bits in them. He does not seem unwell i.e no temperature and still eating. He now has a very bad nappy rash from the pooing ...help! Doctor: Hi...Thank you for consulting in Health Care magic.It seems your kid is having viral diarrhoea. Once it starts it will take 5-7 days to completely get better. Unless the kid's having low urine output or very dull or excessively sleepy or blood in motion or green bilious vomiting...you need not worry.There is no need to use antibiotics unless there is blood in the motion. Antibiotics might worsen if unnecessarily used causing antibiotic associated diarrhoea.I suggest you use zinc supplements (Z&D drops 1ml once daily for 14 days) & ORS (Each small packet mixed in 200ml of potable water and keep giving sip by sip) as hydration is very important and crucial part of treatment. If there is vomiting you can use Syrup Ondansetron (as prescribed by your paediatrician).Regarding diet - You can use cerelac...any flavour will do. Avoid fruit juices as they might aggravate diarrhea. You can give zinc supplements & ORS apart from normal vegetarian porridges & soups.You can use siloderm ointment for diaper rash.Hope my answer was helpful for you. I am happy to help any time. Further clarifications and consultations on Health care magic are welcome. If you do not have any clarifications, you can close the discussion and rate the answer. Wish your kid good health.Regards - Dr. Sumanth MBBS., DCH., DNB (Paed).,"
},
{
"id": 42918,
"tgt": "What should i do to become pregnant?",
"src": "Patient: siri am facing problem with pregnancy, i could not get conceive from last one year. my doc suggest me oosure plus, fertisure f & duphaston. bt no postive result. i have big size ovarian & pcod. my periods is not regular. pls suggest me what to do? now i am loosing every hope. Doctor: haiIT IS better to do laparoscopic ovarian drillin in case of severe pcod.before that you need to check you AMH LEVEL TO know your ovarian reserve.you can try ovulation induction after surgery."
},
{
"id": 211508,
"tgt": "Fearing to lose friends with my attitude and behaviour. Looking for solution?",
"src": "Patient: hi doctor. i m like sufferin a lot like very scared weather i ll loose my frnd s with my attitude and behaviour. eve though my frnd is true to me and trust worthy i m not able to believe her. every time its creating a problem not only with m frnd but my family members to Doctor: Hello and welcome to Healthcare Magic. Thanks for your query. It's good that you have taken the initiative to seek help. You need a detailed psychological assessment to find out why you have such maladaptive behaviour. I would advise you to consult a psychiatrist for a detailed psychological assessment and further treatment. There are effective treatment options - in the form of medication or counselling / psychotherapy which will help you overcome your problems. Wish you all the best. - Dr. Jonas Sundarakumar Consultant Psychiatrist"
},
{
"id": 194576,
"tgt": "How to treat pain during masturbation?",
"src": "Patient: my name is matshepo i was dignosed with endometriosis in 2008 and had a laparotomy done in june of same year.my problem is that the operation didn't help i even had to quit my job,i was told that my bowel is adherent to uterus.i'm 29 and everytime i menstruate is so painful and it becomes worse when a flesh like thing comes out.what can i do to help myself? Doctor: Hi, Endometriosis will cause such pain. You may have to be on regular oral contraceptive pills if not married. If married planning for pregnancy is the best solution. Hope I have answered your query. Let me know if I can assist you further. Regards, Dr. B. Radhakrishnan. Nair, OBGYN"
},
{
"id": 69313,
"tgt": "Suggest remedy for lumps in thigh",
"src": "Patient: Greetings, I have a painful red golfball sized lump on the outside of my thigh. I have already seen a doctor who prescribed an antibiotic, but Im not sure if its working. The lump has gotten less hard, and is very fleshy in the middle now, although that might be fluid. Ive also been heating it too. Doctor: Hi.This is an abscess under resolution but with an antibioma around it . This is giving you the classical feeling.Since the lesion is large , request your Doctor for a regular incision and drainage if you need a fast and early recovery. Continue the medicines ."
},
{
"id": 160166,
"tgt": "Blood count comes down due to spleen enlarged, please help",
"src": "Patient: I have a friend who has an enlarged Spleen it has enlarged to such an extent as if to burst. He also has a problem that due to this his blood count comes down to 5.9 6.3 or so can you help me with the right advise Doctor: Hi I'm french so please don't be too tough with my english... My mother is ill : she has the same trouble : very big spleen and very bad blood count. I would be very pleased if you can explain what you finally discover and help us to help her... My email : sand.blr@gmail.com Thanks Sandrine"
},
{
"id": 67324,
"tgt": "What causes a small lump behind the left ear?",
"src": "Patient: Ok so i ve had this small lump behind my left ear for a couple of months, maybe even longer than that and it won t go away. When i asked someone they said it just might be my glands or something like that. And others said lumps behind the ear in the middle on the bone part just happens and goes away in time but it is still worrying me to death, it isn t painful at all though just bothering when i touch it. Hopefully it is nothing serious at all Doctor: Hi,From history it seems that you might be having non specific lymphadenitis in that area.It shows that there might be having some minor infection in near by area.If there is no pain or increase in size, nothing to worry.Ok and take care."
},
{
"id": 44895,
"tgt": "Suggest me a doctor to get conceived after an ectopic pregnancy ?",
"src": "Patient: hi,.i am 35 years old.i have actopic pregnency in 2003,after that i am trying for baby but i am no cnceive.in tht case i want to come here.please can u suggest which doctor best for my case.thnx. Doctor: Get through infertility check up of you & your husband.Consult some good center dealing in infertility ,near your place,as infertility treatment needs multiple visits.IVF is one option if you can afford.Disclaimer"
},
{
"id": 145888,
"tgt": "Is there any treatment for brain shrinkage?",
"src": "Patient: My sonis30 tears old and was born with Asbergers syndrome not diagnosed until 21....then developed seizures ..now they have said his brain is shrinking and it is the size of a 65/70 year old man. My son is in the uk AND i AM IN usa SO i AM UNDECIDED WHETHER TO MOVE BACK TO uk YET Doctor: Hi,Thanks for writing in.It looks like your son who is 30 years of age has been confirmed to have mild brain atrophy on brain CT scan or MRI scan. It will help to know if this brain volume loss happened in a few months or years. Any sudden decline in neurocognition is also important. The brain is such an organ that it does not regrow by itself. There might be metabolic changes happening and therefore the shrinkage of brain. It is important to restore as much cognitive functions by applying neuro rehabilitation techniques. There can be a possibility of brain stem cell techniques in the future by which certain areas of the brain can be restored and the cells regrown by using special methods. These might take some time and are still under research."
},
{
"id": 75387,
"tgt": "Suggest treatment for blood clot in lung and high BP",
"src": "Patient: My husband had phenomena in July with a small blood clot in his right lung, he recovered great, until his primary increased his dosage of warfarin, since that time he has had stomach issues, his liver is now swollen, the primary then told us he had lung and liver cancer, biopsy s have been done and show no cancer, his primary then put him on pain meds for his stomach, now he does not want to eat, has lost about 30 lbs in the last 6 weeks, sleeps all the time, he also has high blood pressure and needs to eat to take those meds, which I have to fight with him to do because he does not want to eat, he is now having dizziness and muscle problems, I NEED HELP PLEASE Doctor: thank you and very pleased to answer you,your husband suffering from high blood pressure has done firstly a pulmonary embolism (clot in the right lung), after you found an hepatic enlargement and stomach issues. I suspect a right heart failure, and I urge you to take your husband to a cardiologist.right heart failure usualy complicate with pulmonary embolism, hepatic enlargement and stomach issues.may this help you, and best wishes"
},
{
"id": 148082,
"tgt": "How to treat Parkinson s disease mediated hallucination in a 71 years old female while on syndopa plus 125 mg,syndopa CR 250mg and pramipex 1?",
"src": "Patient: Hello Doctor, My mother suffers from Parkinson s disease since last 10-12 years and she is 71 years old nor. Since last 2-3 months the tremor has increased and she has hallucination. Also, she has pricking sensation on her legs, sometime in the chest, arms and scalp, especially in the night. Which she thinks are larva like insects crawling on the bed and she keeps on scratching and cleaning the bed to get rid of them. To the normal eye nothing is visible. Whether it can be due to hallucination? She is on Syndopa Plus 125 mg thrice a day and Syndopa CR 250 mg in the night along with Pramipex 1 thrice a day. Pls advise. Regards Xyz Doctor: Dear I am sorry for the health problem your mother is experiencing.You should take contact with your neurologist and see the possibility of starting an antipsychotic.Anyway this are medications which can be started after an neurologist exam your mother.Don't worry, it's treatable.Wish you all the best"
},
{
"id": 10976,
"tgt": "Suggest treatment for hair loss",
"src": "Patient: Hello! I am just 17 and i am having severe hairfall issues. I am losing hair by leaps and bounds, and hair thinning is even a major problem. I had really beautiful long hair now they are short just till the neck and the volume even gone. Please suggest me some solution ASAP. Harshita Doctor: HIWell come to HCMI really appreciate your concern, you need to take some care, like avoid applying too much oil in long air this could cause the hair fall and increase the chances of infection, pediculosis need to be ruled out, hormone imbalance could be one of the cause, be carful in diet, no more spicy food, no more calorie contain diet, else this is nothing to worry, hope this information, helps takes care and have a nice day."
},
{
"id": 122038,
"tgt": "What could cause sore & enlarged cubital nodes?",
"src": "Patient: I went to the dr. last night for paranychia. They drained it and gave me an antibiotic and antibiotic cream. I woke up this morning and the inside of my elbow hurt. I figured out that it s probably the cubital nodes swollen. They are very sore to the touch and pretty large. Should I be concerned? Doctor: Hello, Your symptoms seem to be related to swollen lymph nodes. It is a common condition after an infection. I suggest using anti-inflammatory medications such as Acetaminophen to relieve the pain. I also suggest to continue the antibiotic to treat the infection. Hope I have answered your query. Let me know if I can assist you further. Regards, Dr. Dorina Gurabardhi, General & Family Physician"
},
{
"id": 154324,
"tgt": "What is the normal size of abdomen nodes?",
"src": "Patient: iam suffering gastric problem.indigestion.last 3 year.i went asian institute gastroentrology.they suggest ultra sound and endoscope all reports normal.but i feel gas problem and indigestion.i am using omez 20 mg daily morning.recently i went meet another doctor he suggest ultrasound and sgpt sgot,b12,bloodtest.all are normal.in ultrasound test hypoecoic lobuler lesion in the mid mesentery 4.2*1.9 node suggest ct scan.i didnt get any pain in my abdomen,no vomthing.if i press my stomach i listen sounds.no motion problem.only gas and indigestion.why doctor suggest ct scan. is this cancer.2years back i had viral infection.what is the normal size of abdomen nodes. Doctor: Hi, dear I have gone through your question. I can understand your concern. You have enlarged lymphnode in abdomen. You sshould go for ct scan abdomen to rule out any cancer or major pathology. Lymphnode more then 1 cm in size is significant. So go for ct scan and take treatment accordingly. Hope I have answered your question, if you have doubt then I will be happy to answer. Thanks for using health care magic. Wish you a very good health."
},
{
"id": 189994,
"tgt": "Sore throat, white spot on the middle of roof since onset of denture. Advice?",
"src": "Patient: I just got top denture for the first time 4 days ago & 2 days ago I started with a sore thorat and today I see a white spot in the middle of the roof of my mouth right at the back edge of the denture, I would think if it were sore from the denture it would be red or could this white (small about 1/8 x 1/4 ) spot be from the new denture, otherwise they seem to fit really good. Doctor: hello, the white spot may be due to denture irritation. this can be due to low quality of denture/irregular borders/ill-fitting denture. apply mucopain gel on the affected site. get your denture borders smoothened to remove the irregularities. visit your dentist as prolonged use of such denture can cause denture hyperplasia. regards dr.bindiya"
},
{
"id": 71980,
"tgt": "Treatment for chest pain when resting, anxiety, panic attacks?",
"src": "Patient: I have been experiencing a mild pain in my chest when resting. It has caused anxiety and panic attacks. I am 22 years old and an active sportsman. The pain itself is not intolerable but it is alarming and I find it hard to break the train of thought that it could be serious, causing a cyclical increase in anxiety and heart rate. Doctor: Hello dearWarm welcome to Healthcaremagic.comI have evaluated your query thoroughly .* It is the underlying stress responsible for every symptom herewith .* Guidelines for better outcome- Drink plenty of liquids .- Balanced nutritious diet .- Deep breathing exercises , concentration techniques , relaxation adaptations , YOGA are the corner stone of treatment .- Manage to have sound regular sleep of 8 hours .- Avoid smoking , alcohol if consuming .Hope this clears your doubt .Wishing you fine recovery .Welcome for any further assistance .Regards take care ."
},
{
"id": 63647,
"tgt": "How can a head lump on scar tissue be treated?",
"src": "Patient: Hi, I have a small lump that has appeared on some scar tissue on the front of my head. When i press it it seems to cause a tingling sensation in my nose! Ive had a bad flu and pneumonia over the last month but am feeling better now. The lump feels very hard and is growing. Doctor: Hi,Dear,Thanks for the query to HCM.I studied your problem in depth and I understood your concerns.Treatment of the head lump on the scar-You seem to have -nerve involved in fibrous scar.So dont worry and attend to ER Surgeon , who would treat it -by Surgical Excision -as its causing pain on pressure and is increasing also-causing cosmetic issues for you.This advise is based on the facts from the history you give and needs further clinical check.Hence I would advise you to check with ER Surgeon.So dont build up wrong concepts and create more psychic complications in you which would increase risks and costs to you.Hope this would relieve your problem.Welcome for any more query in this regard to HCM.Write good resume and Click thanks if you feel satisfied with my advise.Have a Good Day.Dr.Savaskar M.N."
},
{
"id": 124356,
"tgt": "What causes tiredness, achy joints, swollen tonsil, headache and frequent urination?",
"src": "Patient: i started feeling sick very suddenly last night, i was very tired, i had very achy joints, my left tonsil is swollen, i had a heachache, i ve been urinating a lot more than i feel like i should be, and i had the worst shivers last night. what could these symptoms be linked to? Doctor: Hello, As mentioned by you about the symptoms in history I feel before coming to a conclusion we need you to undergo a blood and urine routine. As this might be due to some viral infection. Also, the part missing in history is about your age, any past medical history of high blood pressure or diabetes? Even if we give you a provisional diagnosis it will unnecessarily panic you. What I would advice is to get the blood & urine get assessed at a close by laboratory, post which it will be easy for any specialist to come to a conclusion and provide the required medicine. Hope I have answered your query. Let me know if I can assist you further. Take care Regards, Jay Indravadan Patel, Physical Therapist or Physiotherapist"
},
{
"id": 217277,
"tgt": "Suggest treatment for body pain and nausea",
"src": "Patient: My whole body aches right now from my feet to my head, i feel like i am burning up and feel nauseous. Started riding my bike to and from work with swimming with the kids for 2 hours lately on top of it everyday. I almost feel like i have a full body sunburn the way it tingles but it dont not hurt a lot pain wise. Doctor: I strongly suggest to check your blood report different parameter with also calcium and vitamin d test. And as per the result take medication with your doctor advice. I also suggest to take multi vitamin and multilingual supplement along with omega 3 which will help as anti inflammatory and support for your body's normal functioning as well for internal strength or immunity development. And if you feel too much of pain in body then can take pain killer medication for time being. Hope this was useful for you. Take care"
},
{
"id": 188193,
"tgt": "What is the cause of black, painless lines on gums?",
"src": "Patient: hi, doctor ... i am 22 years old ... i have a crown in my front tooth(probably when i was 13 or 14 years old) .... but just noticed that through out my gums there are black lines .... even in the lower part ... i have not really noticed since when .... i have no pain .... but this really got me scared .... hoping for a reply,sincerely Doctor: Hi,Thanks for asking the query,Gingival pigmentation is he main cause of black gums around teeth.Gums with poor oral hygiene may also cause gums to turn black.Smoking and use of certain medication can also cause blackish gums.Visit to a Dentist and get the checkup done.Some Periodontist now offer cosmetic bleaching procedures to lighten the appearence of your gingiva.This can also be improved by some surgical procedures in which the outer layer of gums is removed surgically.Hope you find this as helpful,Regards..."
},
{
"id": 141047,
"tgt": "What causes dizziness, acid reflux and severe back pain?",
"src": "Patient: I found out i was pregnant 2 weeks ago and a week later I had Brown stringy discharge, the next day I had heavy dark red blood with clots for 2 days then Brown the next day. I still have pregnancy symptoms nearly a week later including bad back, dizziness, a lot of gas, feeling exsausted, heaving when brushing my teeth etc and my vagina is a purple colour. I had no pain when passing blood at all either. Could I still be pregnant it was this a misscaraige? Doctor: Hi, Given the symptoms, there is certainly still a possibility that you are pregnant if you had a definitive urine test 2 weeks ago which was positive. The only way to truly prove the lack of a pregnancy is to visit a gynaecologist and have an ultrasound performed at the appropriate time to discover the presence or lack of an embryonic sac along with appropriate blood tests. Hope I have answered your query. Let me know if I can assist you further. Take care Regards, Dr Dariush Saghafi, Neurologist"
},
{
"id": 173507,
"tgt": "Suggest treatment for anger disorder and seizure in a child",
"src": "Patient: I have a 4 yr old grandaughter who has to rock herself to sleep at night. This consists of the same pattern. She puts her left hand over her chest, tilts her head back and sometimes it looks like her eyes don,t close all the way she rolls her eyes. the rocking is very aggressive an can last from a couple of minutes to 15 mins, until she goes to sleep. The parens said they have checked with a dr and they should not be concerned. I am conserned because it looks more like a programmed seisure. Doctor: Thanks for queryThe posture you are describing is typically body rocking. This is extremely common in children of age 6 months to 3 years. It is not uncommon to see children using body rocking as a means to soothe themselves and typically at bed time to fall asleep, as described in your description. The only worry could be that you have to make sure that the child does not hurt himself while doing body rocking. Only rarely it is associated with developmental anomalies like autism. So, be assured. Its definitely not a seizure as per your description.Hope this helps."
},
{
"id": 44482,
"tgt": "Semen analysis showed low sperm count and motility. Modes to increase count and motility?",
"src": "Patient: I am a 29 years old man , married since two and a half years. My semen analysis shows a sperm count of 30 million and motility score of 130. How can i increase my sperm count and motility score to increase chances of fertility? Doctor: Hello. Thanks for writing to us. To enhance your sperm count and its motility, some simple lifestyle changes that can help you are: 1) Have a healthy nutritious diet comprising fruits, veggies, salads etc. Include bananas, avocados, nuts, tomatoes, oysters as they help in increasing sperm count. 2) Take a Zinc, selenium, l-carnitine, vitamins C and E supplementation once daily for few months. 3) Abstain from smoking and alcohol. 4) Do some form of exercise daily. 5) Avoid stress and practice meditation. Consider visiting an Infertility Specialist for thorough evaluation. I hope this information has been both informative and helpful for you. Regards, Dr. Rakhi Tayal drtayalrakhi@gmail.com"
},
{
"id": 120372,
"tgt": "What medication is suggested for pain on the right side of the neck?",
"src": "Patient: My fiance woke up this morning with a pain on the right side of his neck. He first thought he slept wrong, then realized there is what appears to be a vein behind his right ear that is swollen. Pain goes down his neck and up the side of his head. Is this something to be concerned about? Doctor: Hi,If he do not have fever and there is no local redness of overlying skin, we need not worry now. There is a normal prominence behind our ear- this is formed by mastoid bone and tendons attached on that. If he is able to feel a similar swelling on the opposite side also, probably he is dealing with this.Just ask him to take paracetamol three time as a day(ibuprofen if pain is severe) for 3 days. Hopefully pain will disappear by then. Otherwise, kindly see a doctor to look for any local infection or lymphadenitis.Hope I have answered your question. Let me know if I can assist you further. Regards, Dr. Muhammed Aslam T. K., Pediatrician"
},
{
"id": 212495,
"tgt": "Panic attack with numbness in face, chest, legs and arms, could not breathe, swollen lips. What should I do ?",
"src": "Patient: Hello, my name is AAAAA I have a question about panic attacks. The last panic attack was the worst I have had, my face, chest , legs and arms got numb. I could not breath and afterwards its usually my lips the only thing getting swollen, but this time my eyelids and my lips got swollen and they are taking a long time to go down. I am just concerned because I am scared that I have some kind of anxiety disorder and not even know it. What should I do and am I over reacting? Doctor: Hello and welcome to Healthcare Magic. Thanks for your query. It is common to have a lot of strange sensations in the body during a panic attack. Numbness of the hands and legs, difficulty in breathing and chest discomfort are well known physical sympoms of anxiety. However, swelling of the lips and eyelids, which is persistent, is very atypical and not a symptom of panic attacks. So, I suggest that you have it checked with your doctor, to rule out any medical problems which could be causing this. Wish you all the best. Regards, Dr. Jonas Sundarakumar Consultant Psychiatrist"
},
{
"id": 67368,
"tgt": "What does big lump on shin indicate?",
"src": "Patient: My boyfriend all of a sudden has a lump bigger than a half dollar and about an inch protruding out from his left shin. It is almost a bruised color but appeared suddenly with no injury that we noticed. I work in an urgent care and it worries me. It is not hot to the touch and I have his leg elevated above his heart with ice on it. He says it hurts Doctor: Hello and welcome to HCM,A bruise appearing on leg after with no history of trauma suggests internal bleeding.Spontaneous bleeding occurs due to platelet disorders or coagulation disorders.However, platelet disorders usually cause petechial hemorrhages.Coagulation disorders cause hematoma formation or collection of blood.You need to get some investigations to know the cause of the mentioned symptoms.I suggest you to get bleeding time, clotting time, complete hemogram, platelet counts and coagulation tests- PT, INR and aPTT.The cause of bleeding will be determined by these investigations.Thanks and take careDr Shailja P Wahal"
},
{
"id": 20256,
"tgt": "What is the daily intake of fluids in an congestive cardiac failure patient?",
"src": "Patient: what should be the total daily fluid intake in a 86 year pt with congestive cardiac failure and ejection fraction 28%.bp90/60. My grand mother is on LAsix 4o,solotrate 30 aspirin 75 atorva 20.losartan 25 creatine is 1.9 when i giove more than a littre a day she becomes brethleass and her pressure goes down and she feels distressed.\\ cardiologist havr resufused to do angio for as she is old i dont know why cant they just put a balloon cath with a stent? whatis your opinion sir Doctor: A balloon and stent is not always the answer. They may not be able to help her in that way. Since her kidney function is abnormal, just trying could make her worse.I usually recommend that daily water intake stay below 1-1.5L/d. However, it is not the water that gets you into trouble. Ideal sodium intake is <2000mg within 24 hours. She should weigh daily if possible. That is the best way to keep up with her fluid level. A gain of more than 5 kg means trouble is coming and you may want to notify her doctor."
},
{
"id": 155195,
"tgt": "Suggest remedy for bitter taste and dark patches on tongue while on chemo",
"src": "Patient: Hello Doctor I am on chemotherapy for stage 1b breast cancer. My treatment regimen includes Docetaxol, Cyclophophamide and Herceptin. I am having dark patches on surface of my tongue and on sides. It is scattered. I feel bitter taste in my tongue . What is the remedy for this? Nothing taste good. I finished second chemo and two more to go once in three weeks cycle. Then I have to go for 21 cycles of radiation. Will this continue even during radiation. Thanks Padma Doctor: The problems that you are facing are the side effects of this chemotherapy and will go after the chemo is topped. During chemotherapy you can a multivitamin supplement and maintain a healthy nutritious diet in order to ameliorate these symptoms somewhat. Radiotherapy will not lead to any such symptoms as it only lead to problems localized to the chest wall."
},
{
"id": 193813,
"tgt": "What does this semen analysis report indicate?",
"src": "Patient: HI I AM 26 YEAR OLD HEIGHT IS 5 FEET 2 INCH 80 KG MY SEMEN REPORT IS BELOWTOTAL QUANTATY 2mlCOLOR OFF WHITEVISCOSITY THINPH 8.0RED CELL NILLPUSS CELL 6-8ACTIVE SPERMATOZA 30%SLUGGISH 20%NONMOTILE 50%TOTAL SPERUM COUNT 30/MILLION/ML (NORMAL 60-150) Doctor: Hello, Your active motile sperm is 30% which should be at least 32%. So it is a borderline value. All other finding including count, viscosity, pH are normal. However, sperm morphology is not mentioned here which is also important to know. So kindly mention sperm morphology report as well. You can take a zinc tablet to increase sperm motility. Forte is one of such tablet which contains zinc as well l arginine. Don't take a hot shower. Don't do smoking and drinking to increase active motile sperm. Hope I have answered your query. Let me know if I can assist you further. Take care Regards, Dr Parth Goswami, General & Family Physician"
},
{
"id": 100477,
"tgt": "Suggest remedy for allergic infection",
"src": "Patient: I ve recently become highly allergic to all paint and chemical smells, as well as cigarette smoke. I start feeling sick and have to immediately get away from the smell. If not, I will feel sick the rest of the day. I used to be able to spend hours in a smokey room with no effects . Any thoughts about what has changed in my body? I am a 53 yr old female. Thank you. Doctor: HelloThank You for contacting HCM.Your body became sensitized to that particular odor so whenever you smell it then it causes allergic symptoms. I would recommend you following things:> Take montelukast one daily for one month. Research has shown that it decreases the allergic response.> If condition remain same then I would also suggest you to under go allergy testing at allergy clinic. It will tell that you are allergic to what specific thing. The results will help an allergist to prescribe you immunotherapy to that specific allergen and it will improve the problem considerably.Hope this answers your question. If you have additional questions or follow up questions then please do not hesitate in writing to us. Wishing you good health."
},
{
"id": 135736,
"tgt": "How to cope with excruciating pain from CRPS?",
"src": "Patient: Hi, I have recently been diagnosed with CRPS. I ve had a few flares now that are absolutely excruciating, and that s putting it mildly. I take breakthrough pain meds when needed, but I m in a flare now that I can t cope with. What would YOU recommend I take to get through a flare? I really need another opinion, please. Thank you in advance, Alyssa Doctor: hiFor flare ups oxycodone may be taken after prescription and consult a neurologist if he thinks spinal cord stimulation shall help, may be tried along with physical therapy and anti-inflammatory medications.Amytryptaline tabs may also be taken with methycobalamin.Stay calm and relaxed and do not panicthanks"
},
{
"id": 173307,
"tgt": "Suggest treatment for painful lumps in the anus of a child",
"src": "Patient: my grandaughter is two, she has two lumps by her anus that bleed when she scratches them , they are obviously itchy. The specialist said he is going to burn them off but has not told the parents what it is, what caused this etc, they need to know to stop it happening in the future Doctor: Hi... By what you quote I feel that they could be viral warts. Probably that's the reason why the physician has advised cauterization. But if possible try to upload an image of the area. So that I can guide you more appropriately. I am telling this because, skin conditions are best diagnosed after directly seeing them.Regards - Dr. Sumanth"
},
{
"id": 208560,
"tgt": "What causes wounds on glans penis?",
"src": "Patient: hi I had sex during the last six month but after that I had oral sex only about a month ago. I masturbated at night and next day suddenly I saw wounds below the penis head open wounds not circular shaped and these wounds are there from 3 days ago im cleaning them with betadine Doctor: HIThank for asking to HCMI really appreciate your concern and looking to the history given here it could be said that the chances of infection is very likely and possibility of sexually transmitted infection need to be ruled out and for this you need to see the physician for clinical examination and test, take care and have a nice day."
},
{
"id": 200177,
"tgt": "What causes fluid filled lump on the testicle?",
"src": "Patient: Hi I hv irratation near the upper left side of one testical. I can also feel fluid like lump. The itch goes when I apply anti fungal cream but the jelly like lump doesn t go. Though its not painful but am worried as it s been there since few months. Uppon feeling the lump properly, i dont think its atrached to the scrottum Please let me know what coulb be the reason Doctor: Hii appreciate your concernLooking at your history it seems to be a varicocoele please visit your doctor for detailed examination and ultrasound testisHope i have answered your questionthanks for using HCMgood luck"
},
{
"id": 191875,
"tgt": "Is a black dot under the nail a symptom of diabetes?",
"src": "Patient: Hi Im a 39 y/o female & I noticed during my last pedicure a small black dot under neath my middle toe nail...I m very freightened because I work at a nursing facility & I know a black dot ( necrosis)usually deals with diabetes/ poor circulation.. There s no pain/ swelling to the are & it has not grown in size... Doctor: hi therethank you for using HCMThe black dot on toenail is not a definitive sign for diabetes, so u should not be scared of it. It can appear due to multiple causes like sometimes because of trauma, sometimes it occurs due to bacterial or fungal infection, or in a very small percentage it can be a reason for skin cancer.But since for the ist time u hve noticed, if it has not change its size or colour or there are no visible changes on your toe nail you do not need to be worried as it seems to be harmless.if u think it to b a complication of diabetes and feel anxious,the best advice I can give you is to get a test of your blood sugar fasting.if result is normal, stay relaxed and enjoy your life.hope this answer is helpful.kind regards"
},
{
"id": 100097,
"tgt": "What causes allergy in condoms?",
"src": "Patient: I am allergic to some common condom ingredient, but haven t determined which one yet. It isn t latex, and it isn t N-9, based on the fact that I have the same painful allergic reaction to Skyn condoms (Lifestyles product), which are non-latex condoms without spermicide. My symptoms include redness, itchiness, dryness and lesions of the labia. Symptoms appeared within an hour of product use during sexual contact. What else in condoms might I be allergic to? Doctor: Hello,Welcome to Healthcare magic.It may be due to polymer used in condoms.Please use latex condoms and try that it happened or not.You can also use other method of contraception like copper-T or pills.For allergy, take tablet levocetirizine 5 mg twice ad day for 3-5 days.You can also use oils.Please consult your dermatologist if problem persist.Hope this will helpful to you.Take care...."
},
{
"id": 41826,
"tgt": "Does Krimson 35 cause mid cycle bleeding?",
"src": "Patient: hello doctor i have been taking krimson 35 for a month now for pcod.after taking 21 pills for 21 days my periods starts on 23 day.my periods usually end by the seventh day but while taking krimson 35 little bleeding continued for 15 days.is it normal. how can i stop this? Doctor: HiWelcome to healthcaremagic.I have gone through your question.Krimson contains estrogen and progestin is contraceptive drugs prescribed for pcod polycystic ovarian disease.As spotting/ intermittent bleeding may halpen with it.Consult gynecologist for this spotting to counter this.Hope i answered your question.Would be happy to help you further.Take care."
},
{
"id": 126901,
"tgt": "Why does it hurt in my left humerus bone above the elbow?",
"src": "Patient: I have been having pain in my left humerus bone about 2-3 inches above my elbow. It is not my heart. An orthopedic dr I was seeing for my knee noticed I had it wrapped with an ace bandage one day. Without any xrays he claimed it was rotator cuff tear and gave me a cortisone shot. The shot helped a lot and I have had it 3 more times since. It helps each time. However, it continues to hurt. Mostly when I am quiet or trying to sleep. I have to have a large pillow to lay it across when I sleep at night. Any idea what this could be? My email is YYYY@YYYY Thank you for your time and thoughts! Doctor: Hi, You might be having tennis elbow on the left side. Tennis elbow is also known as lateral epicondyle inflammation and tenderness over the lateral aspect of the elbow is usually a sign. This is usually due to inflammation at the extensor muscle of elbow on the outer aspect of humerus. It is different from rotator cuff tear. Diagnosis is clinical examination and treatment is local Cortisone injection. If three injections are not helping you MRI of the elbow should be done. Local injection of platelet rich plasma is a very useful treatment. Hope I have answered your query. Let me know if I can assist you further."
},
{
"id": 120175,
"tgt": "What exercises relieves the pain due to writer's cramp disease?",
"src": "Patient: I am Amit. i have suffer from writer's cramp diseases in my right hand finger. I am suffering in writing so much, specially in small characters. So please tell me exercise for this treatment for quick well. My e-mail i.d. is YYYY@YYYY Doctor: Hello, I would recommend performing nerve conduction studies in order to exclude possible pinched nerve in this hand. Some treatment to consider would be trihexyphenidyl (Artane) and benztropine (Cogentin). Another treatment option would be botulinum injection. Unfortunately besides relaxing, there are not many exercises to help. Hope I have answered your query. Let me know if I can assist you further. Take care Regards, Dr. Ilir Sharka"
},
{
"id": 202932,
"tgt": "Body building, hemo-rage, loss of erection",
"src": "Patient: Hello I recently began body building again and I don t smoke or drink but have tried a pre workout supplement called hemo-rage. Since I have, I have been experiencing loss of erection which is unusual for me as I usually can t stop getting them, I have a very high sex drive. It s very upsetting, as when intimate with my partner I cannot sustain or sometimes achieve an erection. It isn t anxiety , I am very comfortable with her but I think it might be this supplement. After training my penis is smaller too. Doctor: DearWe understand your concernsI went through your details. You started body building training and that is the correct answer to your question. Also you cannot rule out anxiety. Body needs energy to do its day to day functions. You started body building and the energy is being spent on that front and therefore your body struggles to adjust energy for other activities such as sex or masturbation. Almost every body builder faces this problem. Your supplement has nothing to do with this. Your penis has not become smaller. The training schedule made you anxious about your sexual performance and now you feel all these. Means all these problems are just in your mind. Forget it. But body building and sex go hand in hand only if you have enough energy. Distribute your energy wisely.Hope this answers your query. Available for further sexology clarifications.Good luck."
},
{
"id": 177620,
"tgt": "What causes predominant veins on abdomen?",
"src": "Patient: My daughter went swimming today. She swallowed a little water coughed for a short period of time. She appeared o.k. after that. My daughter then took about a 3 hour nap and woke up and there was no noticeable change in her behavior or condition. My wife notice that her veins were prominently showing on her abdomen. We are a little concerned at this point probably because we never noticed this. One issue is that our daughter has not had a bowel movement since yesterday morning. Is there anything to worry about other than constipation? Doctor: HI...prominent veins due a pathological condition wont appear like that so fast and acutely. I feel that they should have been there since long and wasn't noticed. If her daily activities are normal, you need not worry. 1-2 days of constipation also would not cause this. But prominent veins are tell tale markers of portal hypertension and I suggest you consult your pediatrician regarding this.Regards- Dr. Sumanth"
},
{
"id": 69178,
"tgt": "What causes painful lumps on stomach and neck?",
"src": "Patient: i have two lumps growing on the right side of my stomach and also one on the left side of my neck and one on the left side of my groin they are very painful and the more activity that i do the more they hurt. should i be worried? does anyone know what they could be? Doctor: Hi.Thanks for your query.The lumps at different places occurring simultaneously in the body are more suggestive of an infection.They are very painful. There are conditions wherein one gets lumps at different sites in the body. I would advise you to first of all check for blood sugar as diabetes is a known reason.Please go for blood tests particularly for blood culture and sensitivity before the starting of antibiotic. You can start antibiotics and anti-inflammatory medicines the moment you give blood sample. Do not wait till the reports.Get checked by the Surgeon as they may need to be aspirated or incised to get an early recovery."
},
{
"id": 11994,
"tgt": "What treatment should be taken for hyper pigmentation ?",
"src": "Patient: Hi I am 35 married female suffering from hyperpigmentation on cheeks and uneven spots all over face . Have tried every treatement except laser and the problem doesnt go. I am not on contraceptives either. Please help You answer doesnt help since diagnosis might be wrong. Please note I dont have a baby yet nor I am on contraceptive... nope nothing wrong with me in this area, its just that I just got married Doctor: Hi!, Welcome to HealthcareMagic forum, According to your history ,it seems that you have MELASMA,since you quoted that you are not on oral contraceptive pills and you have not had any issues either.These questions you would have faced by many doctors . It is true that such pigmentation is often seen after pregnancy or one who is on OCPs.But we have been seeing such pigmentation on those who have not had such a history either.So,even if you never had a baby ,one can have such pigmentation problem ,just because of slight shift in the hormones in the body.It is the estrogen level in the blood which is more responsible for such pigmentation. If this is melasma,we are dealing with a very chronic problem.It can wax and vain also.Sunlight & stress also has a big role to play. The best advise here would be to use a triple combination cream to begin with ,and then gradually taper it off.Don't use it for more than 2 weeks .As you taper it gradually start using hydroquinone based cream at night .Along with this also start using kojic acid & vit c based creams in day and evening.But please don't forget to use a good spf sunscreen daily thrice.Sunscreen is mandatory. You can even under go chemical peeling from a experienced dermatologist. Results of this all might vary from patient to patient. Now being a laser expert I've myself done number of cases with lasers too.The best of the laser for this is Q-switched Nd:yag laser .But one cannot guarantee you any great results with this either.The results are variable here too.And its costly also. Melasma has always been a very frustrating problem both for the patient and the treating doctor.It is one thing which can destroy the reputation of most reputed doctor. So,have patience and kindly consult a dermatologist for this .Please do not take any beauty parlor treatment for this . This is one possibility what came in my mind. Secondly it might be freckles too since you said you have spots all over your face.Lasers is best for treating freckles .You may require only one sitting for this. Kindly consult a experienced dermatologist who can diagnose and treat your problem. I hope this answers your query,if you any further questions,kindly write back to us. regards, Dr.Bharat chawda"
},
{
"id": 144767,
"tgt": "Suggest treatment for pain and spasms in left leg",
"src": "Patient: hi I have herniated disc and nerve pain tingling and numbness in my left leg, a year ago July I have 4 steroid shots in my lower back, I think they are wearing. the past new weeks Ive noticed NEW Pains in back and hips. Last night and the night before Ive been waking up all night with spasms/tremmors and excruciating pain in my left leg, and now in my right leg too. I am taking Norco 10/325 and Neurantin. Last night at 2:30 the Pain was so bad I was shaking and almost throwing up. I needed muscle relaxers and fast, I took a valium and at 4:30 I was still having the pain and spasms, I took the Norco and Neurantin because it was time again and at 6am I ended up getting out of bed and walking on the old cement garage floor to get some relief. What can I take along with Norco and Neurantin? I have valium but it didn t help, I also have tramidol, but not sure if these are save to take with the others.... Doctor: Hello!Thank you for asking on HCM!I read carefully your question and understand your concern. Regarding your concern, your symptoms seem to be related to a worsening of your situation ( which may be a higher grade of herniated disc, or a stenosis of the spinal canal, compressing both the nerve radix and the spine). This is an urgent situation, which can lead to serious complications if not treated immediately. I recommend consulting with your attending physician for a careful physical examination and a lumbar spine MRI or CT scan to exclude possible huge hernia or compression of the spine. Even if the situation of the herniated disc results to be the same, this can be considered a case abbortive to therapy, which is an indication for surgery. You should discuss with your attending physician and the neurosurgeon on this possibility. Hope to have been of help!Best regards, Dr. AidaI"
},
{
"id": 93827,
"tgt": "Sharp shooting pain in stomach and appendix. Is it nerves?",
"src": "Patient: I suddenly got a sharp almost electric shock like pain at both sides of the area around my stomach and appendix (where love handles should be) and a litle bit at the back part of both sides too. It felt like something with nerves maybe? My mother said it was because i ve been sitting for a few hours and i was on the computer too much. It repeated twice and i suddenly also couldn t breathe but maybe that part was because of fear. Doctor: Hi and welcomet to HCM, It can be neuropathic pain due to abdominall wall or spine nerves impact. But also it sounds like ureteral or gallbladder colic which can be felt on this side too. If you feel it again you should do abdominal ultrasounds to rule out some intraabdominal pathology or inflammation. This kind of pain is often experienced and it shouldnt worry you if this last for second and doesnt repeat. WIsh you good health."
},
{
"id": 21774,
"tgt": "What are the symptoms of heart attack?",
"src": "Patient: Hi, my name is Adam. I m 29. The only medication i ve been on for the past 2.5 years is methadone.. I was on 90MG but I ve dropped 1 a week. I m currently on 27MG. the last 5-6 days,..mainly at night.. my heart aches really bad.. Especially if I lay flat on my back. I ve had to sit up in a recliner and go to sleep that way.. About every 3-4hours my heart is aching and I wake up and my arm(s) are tingling and numb.. I m afraid of a stroke or heart attack.. Please give me some advice.. I have no insurance. Thanks for your time, Adam. Doctor: Hi,Welcome.The symptoms you are telling does not seem to be a heart issue. Cardiac chest pain is diffused, squeezing type & increase on exertion or stress. It can radiate to left arm or shoulder. It does not occur in specific position. But still I'll advice you for certain tests to be completely sure about it. I'll recommend you to do a ECG, 2D Echo & Lipid Profile. You can follow up after the above reports.Thanks"
},
{
"id": 119840,
"tgt": "What causes excruciating pain in feet and hands?",
"src": "Patient: my father is suffering an excruciating pain in his feet & hand for almost 1 week & he can bear the pain anymore, he was hospitalized. He also have fever on & off for more than 3 days. He was tested for Blood Chem but everything was normal. We worry much about the situation of our father... pls give us answer doctor what was the problem,.tnx very much Doctor: Hello,I read carefully your query and understand your concern. The symptoms seem to be related to a viral infection. I recommend to use anti inflammatory medications such as Acetaminophen to relieve the pain. I also suggest using magnesium supplement for muscle relaxation. I recommend to rest and drink liquids. Hope my answer was helpful.If you have further queries feel free to contact me again.Kind regards! Dr.Dorina Gurabardhi General &Family Physician"
},
{
"id": 97410,
"tgt": "Are the Chinese herbal medications effective for infertility problem?",
"src": "Patient: hi I'M 25, I HAVE A 3 YEAR OLD SON, WE HAVE BEEN TRYING FOR ANOTHER BABY FOR 1YEAR & A HALF GOT PREGNANT 13MONTHS AGO BUT LOST THE BABY AT 11 WEEKS, HAVE BEEN TRYING SINCE BUT NOTHINGS HAPPENING, HAVE NOW STARTED USING CHINESE HERB MEDICINE NUAN GONG YUN ZI WAN, IS IT EFFECTIVE & SAFE? THANKS Doctor: Hi Welcome to HCM Dear your anxiety about your pregnancy is matter of concern .As you have started Chinese treatment for your problem , there is no harm in it All alternative therapies have successfull treatments for this and in Chinese treatment there is no doubt about its success . It is one of the best .So complete your treatment under right guidance . you will be surely blessed . Hope your query is replied . Take care .All the best . Mail at drsuchda@gmail.com if any doubtDon;t hesitate to get back for any further query"
},
{
"id": 134386,
"tgt": "What causes deep muscular pain in the chest?",
"src": "Patient: I have this deep muscular pain in my left shoulder chest area. I can t sleep on my left side. It hurts when I breathe deeply. Also I cant hold a cup of ice water too long on with that arm....I had simular pain in my right arm for 10 years before it went away... Doctor: hi,thanks for mentioning little history about you. you should meet a cardio or a physician if it is a left sided pain. if they give thumps up that all is fine then you should do some regular breathing exercises and also some simple strengthening exercises for hand and legs. it will help you. but if the symptoms are bothering than get checked once. there is no harm in getting checked.thank you"
},
{
"id": 175940,
"tgt": "Can Phexin dry syrup be given to a child?",
"src": "Patient: Sir,The Phexin Dry Syrup, which i bought for my daughter has slight bitter taste after reconstituting.In fact i checked a couple of bottles and there is difference in the intensity. Is it safe to give her this syrup? The bottles are kept in refrigerator. Doctor: Hi...cough and cold are viral in origin 95% of the times in kids. usually they do not warrant antibiotics.All the dry syrups once reconstituted should be kept in fridge. If not the efficacy will be altered. I suggest you use a fresh bottle if prescribed by your pediatrician. Do not use antibiotics on your own.Regards - Dr. Sumanth"
},
{
"id": 222970,
"tgt": "Does presence of Epi cells 18-20/HPF Bacteria in urine cause pre mature delivery?",
"src": "Patient: Hi i am 13th week Pregnant women and recently i have done with the urine test , microscopic examination ,puss cells 8-10/HPF and Epi cells 18-20/HPF Bacteria is present and my concern is is this problem leads to pre mature delivery? i have been treated with the tab Azithrol 500mg . Please help me Doctor: yes it is quite possible so be very careful and always get treated for your uti and be careful to take correct according to sensitivity"
},
{
"id": 9922,
"tgt": "Suggest remedy for hairloss",
"src": "Patient: hi sir i am abhishek i am suffring from hair loss problem from 1 year and i started treatment 8 months back my doctor prescribed me finax and hair for u after using that for 5 months i got my hairs back but after 5 months my doctor told me to take a brake for two months but its been only one month and i lost most of my new born hairs so i started medicine again but its not working i am continuously lossing my hairs and i am getting bald pls suggest me should i continue the medicine or stop it one more thing i want to say is i takes No xplode supplement for bodybuilding do it has any side effect for hair Doctor: Hello, I have gone through your query and would recommend you to continue with the medicines as they take time to act. You need to continue these for 5-6 months to see significant results. You can continue with your no explode as it doesn\u2019t cause hair loss. Hope I have answered your query. Let me know if I can assist you further. Take care Regards, Dr Asmeet Kaur Sawhney, Dermatologist"
},
{
"id": 206537,
"tgt": "Suggest treatment for hallucination about harm and body pain",
"src": "Patient: am I abused? it is not like most abused wife knew their husbands did to(beat-up) them. I got nightmares about needle shot on my arms and legs, some one twisted my right feet, hit forehead 3 times,...all happened after I slept without knowing. over 17 years can t catch anyone did to me with security systems bought by my husband. in the meanwhile frequently things disappeared/stolen, some of them would return... we did visit family doctor when our marriage worse and worse but she didn t believe anything I said such as showing knife-cut my finger and wanted me take depression-related medicine and to visit mental doctor (both I refused)... we invited my 2nd brother lives with us (over a year) but making worse that he believed I should see mental doctor 9himself didn t have job over 15 years). I didn t lose my confidence in myself and knew bad things won t stop. I thought there is 3rd person involved to abuse me because something disappeared I m pretty sure is not my husband did(he may know/agree but not all of them). in the beginning I thought my husband seemed was abused and wanted me to go thru what he suffered.. all over my body was hurt except my left front ribs/head/foot not suffer heavy painful yet. the 1st time I felt right arm was needle shot we went to emergence room the doctor can t tell but a warning... becareful in private short seconds with me. am I in danger? Doctor: Hello I have read your history and I can understand your problem. You have mentioned that you get nightmares about needles shot on your arms and legs, some one hit you on forehead. Things have disappeared and appeared again. You have visited your family doctors who advised to consult psychiatrist. Even you didn't consult psychiatrist and again started having complaints. You called your brother and after one year he also advised you to consult psychiatrist. See although I need more details of symptoms but from the summary it appears that you have delusions that some one is harming you. The delusion is held despite evidence to contrary. The delusions are causing development of other symptoms. Consult a Psychiatrist for expert opinion.Thanks"
},
{
"id": 40272,
"tgt": "Is it okay to take less dosage of azithromycin?",
"src": "Patient: Today is the first day I took AZITHROMYCIN Formy sinus infection. I took 250mg tab at lunch time, and 250mg tab before bed =500mg tab total. . I did not realize that the first dosage should have been = 500mg total. DO I NEED TO BE CONCERNED ABOUT EFFECTIVENESS? Doctor: Hello,Welcome to HCM,Daily requirement of tab Azithromycin is 500mg, it can taken in divided dose but you should take recommended daily dose of 500 mg.It will have same effectiveness if you take the drug in divided dose,but first dose of 500 mg of azithromycin maintain bacteriostatic levels in the infected tissue for several days.I would suggest you to complete the course of antibiotics as advised by your treating doctor, your symptoms of sinus infection will improve with this excellent antibiotics.Thank you."
},
{
"id": 81793,
"tgt": "What causes heaviness and discomfort on the Xiphoid?",
"src": "Patient: Hi. I have been experiencing a really full feeling below my xiphoid process.it feels as if there is a baseball sitting there. It is very discomforting to me because of me being short chest & waisted. Just started like 3 days ago. I m very concerned but I wanted to see if I could out answers first before taking the next step. Doctor: Thanks for your question on HCM. In my opinion we should first rule out cardiac cause for your symptom. As cardiac issue can present with problem, below xiphoid process. So get done ECG to rule out this. If ECG is normal than no need to worry much. This can be due to gaseous distention of stomach. So try to follow these steps. 1. Avoid hot and spicy food. 2. Avoid large meals, instead take frequent small meals. 3. Avoid stress and anxiety. 4. Start proton pump inhibitor. 5. Go for walk after meals. 6. Keep 2 - 3 pillows under head in the bed. 7. Avoid smoking and alcohol if you have this habits.8. Loose weight if you are obese."
},
{
"id": 11173,
"tgt": "Could hair loss be the side effect of mepture xl?",
"src": "Patient: Hi, I am 23.5 years male, 5ft 11 inches tall and 97 kg weight. I was diagnosed with High BP 140/100 in may 2011.Since then I am on Metpure XL 25, One pill a day. Recently I am suffering from hair loss. The hair that falls is mostly thick strands along with some thin ones. Say around 30-40 on an average.My question is, Is this rate of hair fall a part of the normal hair growth cycle or can i be a side-effect of the medicine. Doctor: HIWell come to HCMI really appreciate your concern, hair loss could be due to the Male-pattern-Baldness, and not due to medicine (Mepture xl) but of course some time medicine could facilitate the condition, but in most of the cases the cause would be genetic usually seen in male parson, hope this information helps, take care."
},
{
"id": 100385,
"tgt": "What causes asthmatic skin allergy?",
"src": "Patient: My son has an asthma skin allergy since he was 5 years old. Today he is already 16 years old. During this years we had consulted to diffirent doctors and even to dermatologist. I want to know what causes his allergy? What medication he needs? Is it the climate since we here in the Philippines? And, what the foods that he should not take. Doctor: Hello.Thank you for asking at HCM.I went through your history and would like to make following suggestions for your son:1. If he as both asthma & skin allergy, I would suggest him allergy testing for air-borne and food allergens. This will help you identify the substances that he may be allergic to and also to know the measures to avoid them.2. Common air-borne allergens causing allergies are - house dust mites, molds, pollens and animal danders. Common foods causing allergies are - milk, peanut, wheat, soy, eggs, tree nuts, fish, sea foods, etc.He should avoid only those foods to which he is allergic to.3. The medicine that he needs would depend upon severity and frequency of his diseases. So more details would require to advice about medications.In general, I would suggest him to use salbutamol inhaler for wheezing/breathing difficulty symptoms and he as asthma symptoms very frequently, I would suggest him daily montelukast.For skin allergy, I would suggest him regular application of lotion like calamine immediately after bath to retain moisture in skin. If he still has symptoms, he might require corticosteroid creams.4. Climate is important for air-borne allergens like house dust mites, molds and pollens. So, yes, it does have effect. Especially humidity increases both house dust mites and mold concentration.Hope above suggestions will be helpful to you.Should you have any further query, please feel free to ask at HCM.Wish your son the best of the health.Thank you & Regards."
},
{
"id": 78805,
"tgt": "What could cause problem while exhaling?",
"src": "Patient: Hello. My wife is suddenly having difficulty exhaling. She has never smoked, nor does she have asthma. She hasn't been ill or had any type of cold. She says it feels like she \"can't get her air out, like it's stuck.\" She's not complaining of any chest pain. Quite literally her only problem is in exhaling. What's going on??Thanks. Doctor: Thanks for your question on Health Care Magic. I can understand your concern. breathlessness on exhalation can be seen in anxiety attack or in asthma.So better to consult pulmonologist and get done clinical examination of respiratory system and PFT (Pulmonary Function Test). PFT is needed for the diagnosis of asthma. It will also tell you about severity of the disease and treatment of is based on severity only. A detailed examination is definitely required.Don't worry, she will be alright. Hope I have solved your query."
},
{
"id": 66913,
"tgt": "What causes small painless lump in my vagina?",
"src": "Patient: I had an abortion 2 weeks ago, I have took the pregnancy test to make sure the procedure was successful which came back negative. I thought it would as all the blood passed after 1 week. I examined myself today and inside my vagina on the wall is a very small hard pea size lump. It has never been there before. Should I be concerned and how do I get rid of it? Doctor: It is possibly a benign cyst could be Bartholin's cyst or a minor hematoma related to trauma relating to the procedure!wait for few days and if it persists then get a physical examination not to worry much!all the best............."
},
{
"id": 419,
"tgt": "How long does it take for pregnancy symptoms to begin?",
"src": "Patient: I am 16 years old and I'm afraid I might be pregnant. The last time I've had intercourse was two weeks ago and I can't wait until my next period because I only get my period every three months. I am not able to go to a free clinic. How long does it take for me to notice if I am pregnant or not? Doctor: Hi welcome to the health care magic You had history of unprotected sexual intercourse.... So pregnancy should be ruled out here... After three week of last LMP you can detect pregnancy by blood hcg estimation... It is somewhat costly If you can wait then after five week of last LMP pregnancy can be detected by urine hcg card test which is not costly Choose any one of the above method. If urgent detection needed then serum hcg estimation done If hcg positive then for further confirmation USG can be done Take care Hope your concern solved Consult gynecologist for examination for your irregular delayed period"
},
{
"id": 70804,
"tgt": "What causes chest pain and fever after altitude travel?",
"src": "Patient: Hello, I recently traveled up to Mammoth (elevation between 7900 - 8900 ft) for the Memorial Day Weekend. We arrived on Friday evening. I felt fine all day Saturday and Sunday. After going in to bed on Sunday night I was restless and had a hard time sleeping. I started shivering and had chills. I think I started to scare myself and started having some anxiety which was making me feel like I wasn t breathing right - though I know I wasn t really breathless, didn t have shortness of breath nor was my heart rate irregular. I ended up taking my temperature at it was just under 100 degrees (normal for me is in the 97-low 98 degrees). I could tell my temperature rose a bit more as I got really warm then sweaty. I felt very dehydrated and kept drinking a lot of fluids. I was somewhat dizzy and felt a slighly nauseated. I couldn t sleep for most of the night. I took some ibuprofin and finally dosed off around 4 in the morning. I woke up around 6/7 am and didn t feel any better. My fever had come back (still low grade) and then I had a headache and body aches. I did start having some chest tightness and aches in my chest area too - similar to when my asthma acts up. As the morning went on I felt a little bit better but still had a headache, body aches and low grade fever. We left the area around noon and went back home (which is close to sea level). I felt much better still had some of the symptoms. My fever rose to almost 101 during the night (last night) but dropped after taking ibuprofin this morning. I still have had a slight headache but it and the aches have improved. Temperature hasn t risen since the night again. Question - does this sound more like a flu or virus or could it have been altitude sickness? or something else? Some of the symptoms sound like it could have been but not sure since I did have a fever and it came on after 48 hrs of being at that elevation (we slept at the lower elevation). Doctor: Hi, Your chest pain and fever look more like flu than altitude sickness, when fever is present body aches will be there and heart activity will be increased so that chest tightness and pain you have felt. Your headache also favours more towards flu, just if you get another episode of same just consult a physician. Hope I have answered your query. Let me know if I can assist you further. Take care Regards, Dr Pusarla Naga Sai Lakshmi, Cardiac Surgeon"
},
{
"id": 125929,
"tgt": "Suggest alternative to Mestinon and CellCept for myasthenia gravis",
"src": "Patient: I am a 73 year old woman. I was diagnosed with MG six years ago. I am on Mestinon and Cellcept. I get apheresis every other week, I am not getting any worse and I certainly my not getting any better. I heard that there may be some chemotherapy drugs that doctors are trying now on some MG patients and they are doing fairly well with it. Have you hear anything regarding any new drugs that are being offered to MG patients? Doctor: Hello, Immunosuppressive drugs in treatment of Myasthenia Gravis (MG): Mycophenolate mofetil has become one of the most widely used drugs in the long-term treatment of MG, because of its effectiveness and relative lack of side effects. Hope I have answered your query. Let me know if I can assist you further. Regards, Dr. Tushar Kanti Biswas, Internal Medicine Specialist"
},
{
"id": 121033,
"tgt": "What causes pain in hip and shoulder joint?",
"src": "Patient: i am having pain in the right hip joint has been taking glucosamine tablets and shellcal 500 so far no relief, now I am also getting pain in my left shoulder joint, I am a female /Asian origin , have high cholesterol and taking atorin 10 mg daily. please let me know the reason for this pain. Doctor: Hello,The pain in the joints that you have can be related to spondylottic changes or ankylosis. You need to consult an orthopedician and get an MRI scan done to find the exact cause. Hope I have answered your question. Let me know if I can assist you further. Regards, Dr. Praveen Tayal, Orthopedic Surgeon"
},
{
"id": 60012,
"tgt": "Had gall bladder removal surgery after finding a polyp in ultrasound. Itching, discoloration and bubbles on the incision site. What to do?",
"src": "Patient: I had my gallbladder removed 3 weeks ago. About a weeks after surgury I starteditchin bad at incision sites and area got pink and bubbly. Now a week later looking a bit better in those areas but starting to itch all over . My gallbladder was removedbe pause a polyp was seen on a ultrasound when doing ultrasound for something else. I feel like I am going crazy with so much itching. I have had 2 csections and never felt like this Doctor: Hi, Thanks for posting your query. There is high chances of developing fistula at operative site. Presence of air bubble with itching (because of irritation) suggest this diagnosis. You should consult with the treating surgeon. You may require another episode of surgery for getting relief from fistula. Thanks, Dr. Mayank Bhargava"
},
{
"id": 85232,
"tgt": "What causes headache after taking Postinor?",
"src": "Patient: after having sex about 3 days ago i took postinor 2 on the 3rd day and that was in the night i then took one this morning which kinda went on to the fourth day, i then experience headache in the right side of my head and i started to feel upset (nausea) are these feelings all in responds to the pill Doctor: Hi,Suggestive of self limiting side effects. The emergency contraception like positnor-2 is generally well tolerate by most women however nausea, vomiting, breast tenderness, tiredness, headache and migraine can occur. The hormone present in the pill positnor-2 is responsible for such side effects which can. These side effects usually subside within a day or two.Hope I have answered your query. Let me know if I can assist you further. Regards, Dr. Mohammed Taher Ali, General & Family Physician"
},
{
"id": 183401,
"tgt": "How to stop bleeding from gum?",
"src": "Patient: I had deep cleaning scaling down yesterday and I have one tooth that the gum keeps bleeding and it bled all night. I've washed with warm salt water, listerine mixed with peroxide. what else can i do? i have gause in my mouth now to absorb the blood but it's crazy right now. How do I know if it's clotting properly? Doctor: Hi,Thanks for posting the query, If the bleeding is cot stopping i would suggest you to consult the concerned Dentist and get a checkup done usually this does not occur after normal scaling and polishing. Also i would suggest you to get your blood pressure and routine blood examination done. Continue iwth lukewarm saline gargles and antiseptic mouthwash rinses, take multivitamin suplements take something cold like ice cream, juice, avoid hot spicy foodstuffs.Hope you find this as helpful,Take care!"
},
{
"id": 101079,
"tgt": "What causes pain in body after eating salmon?",
"src": "Patient: I ate Salmon from Longhorn on Tuesday around noon. Around 4 my body started acheing like a ton of bricks were pressing on me. I went home, at 6 took nyquil slept till 11 got up with diarriah, took more nyquil, all day wed, terrible aches, and took dayquil hoping the aches would go away, now it's the 3 day, and my body is still hurting terrible. what does this sound like? Doctor: Hi, thanks for using healthcare magicIt sounds like gastroenteritis which is infection and inflammation of the gastrointestinal tract resulting in all or some of the following symptoms: nausea, vomiting, abdominal pain, diarrhoea, loss of appetite, muscle aches and pain, weaknessThe most important aspect of treatment is preventing dehydration by replacing the fluids loss during vomiting and diarrhoea. It would be best to use a oral rehydration fluid from your local pharmacy or a sports drink like gatorade.These would give your back both the fluids and the electrolytes that you are losing. Loss of electrolytes would contribute to your muscle aches.There are medications that you can use to decrease the vomiting, diarrhoea and muscle pain that should be available over the counter at your local pharmacy. eg peptobismol, paracetamol, dramamineI hope this helps"
},
{
"id": 54305,
"tgt": "What is the medication for loose stools after gall bladder removal?",
"src": "Patient: I no longer have a gallsbladder. But the result I observed is my frequent diarrhea. It is said that the cause being the liver can no longer control the amount of bile supplied to the stomach. this cause the diarrhea. will Livolin Forte or the Essentiale forte can help resolve my problem? thanks Doctor: Hi I can understand your concern....See let me know you after post cholecystectomy bile acid enter in intestine constantly in low amount....As reservoir for bile that is gall bladder not present....So fatty food digestion not proper....This is called bile acid diarrhea....Sometime diarrhea can be by leakage of bile to adjacent structures like stomach , intestine....Now for treatment of diarrhea in some pateint dietary modification is helpful....Like take low fat foods....Fruits more....Heavy meal restricted....If still no benefit then we can use bile acid sequestrant like cholestyramin and colestipol...Antispasmodic drug also can be used....Besides this no other drug tried....Consult gastroentetologist with keeping this in mind....Take care.Dr.Parth"
},
{
"id": 134233,
"tgt": "Suggest remedy for swelling and pain in head after an accident",
"src": "Patient: Hello, My daughter is 12 years old and two days ago, she and her sister were fighting and her sister pushed her and she banged her head on a cupboard. I looked to see if she was okay and she said there was pain but it was fine. Today, she said her head hurt so I looked at it and I have realized that her head has now swelled up on the side and is bruised. What should I do? Doctor: hi,thanks for providing the mechanism of injury.as you said the next day the swelling took place and it is painful. swelling is a protective mechanism of human tissue.incase if you find and chances in the texture of the skin color, like black circles near eye, near the ear or jaw etc you should visit an ER and get checked once. If your daughter has any bleeding from nose or redness of eyes, or bleeding from ear then a neurologist opinion will count.or else most of the time the swelling subsides on its own when there is no major issue.still it is recommended on a safer note to get checked once.regards"
},
{
"id": 41359,
"tgt": "Suggest treatment for infertility",
"src": "Patient: HiDoctor.....I am 28 yrs old and m married for 5 yrs now and live in the netherlands......its been more than 2 yr i have been trying to get pregnant.....have gone thru all kinds of tests related to infertility in Holland and all our test are positive and everything is fine with me and my husband...am not allergic to any medicines till now... i lead a normaly life...i took a second opinion from my doctor is india and she suggested me to take some medicines for superovulation as i get my regular periods and mostly ovulate on the 18th day from my period .......she prescribed Letroz - 5mg or Naturagest and lupigest - 300. is it ok to take these medicines? do they have any side effects? As i am in a colder country am a bit over weight (though my doctor said its fine) and am not able to reduce my weight inspite of being on proper diet and cycling, will this medicine have any effect in ganning weight? Normally when am in India I easily reduce weight and also am able to maintain it as I sweat a lot. please do advise me as I really dont want to miss out on any option cause of which i can get pregnant as its more than 2 years of trying and its sometimes depressing inspite of evrything being ok with me and my husband. Please help :) Doctor: Hi, I have one through your problem. When all the tests of the couple are normal and still they are unable to conceive naturally, then the condition is known as unexplained infertility. It is responsible for approximately 30-40% cases of infertility. In such cases, the stepwise treatment protocol is followed, the next step is taken when the previous one fails:1. ovulation induction drugs (clomiphene, letrozol)2. Intrauterine insemination (IUI)3. In- vitro fertilization (IVF)Letroz is an ovulation induction drug, while naturagest and lupigest are progesterones to make uterine environment more conducive for pregnancy.These are hormones which do cause increase in weight and very minor side effects. I would suggest you to go for IUI if you have failed to conceive after 5-6 cycles of the above drugs.Hope you found this answer helpful. Wishing you good health."
},
{
"id": 76697,
"tgt": "What causes sharp pain in chest with tingling pains in legs and arms?",
"src": "Patient: I have had chest pains for about a month, my halter mon. was ok, I'm waiting to hear back about my chest x-ray.However yesterday morning I had a different SHARP chest pain that only lasted for a second, and was followed by about 1-2 min of sweating like I had just run a mile. I am also currently experiencing tingling pains in my legs, feet, wrists, and arms on and off all day today. Doctor: Thanks for your question on Healthcare Magic. I can understand your concern. Sudden onset sharp chest pain, sweating and tingling in arms are commonly seen with heart diseases like coronary artery disease (CAD). Holter monitoring (24 hours continuous recording of ecg) is for arrhythmia (rhythm disturbances in heart). CAD can note be picked up by this. So despite of normal Holter report, your should definitely get done 2d echo and stress test (trade mill test) and coronary angiography (if required). If all these are normal then no need to worry for heart diseases. Sometimes stress and anxiety can also cause similar symptoms. So avoid stress and tension, be relax and calm. Don't worry, you will be alright. But better to first rule out CAD. Hope I have solved your query. I will be happy to help you further. Wish you good health. Thanks."
},
{
"id": 120005,
"tgt": "What causes numbness around forearm after shoulder surgery?",
"src": "Patient: just had shoulder surgery last wed. i due suffer from periphial neropathies due to a c3/c4 fusion done in 2005. i am experiencing increased nunbess around left forearm up to the wrist and back of hand.also my left eye feels watery and itchy any advice to this symptoms will be greatly appreciated. Doctor: Hello,The increased numbness in the forearm can be related to the peripheral neuropathy that you suffer. I suggest using medications such as Gabapentin to relieve the symptoms. The symptom of the watery eyes is not related to the first symptom. It can be related to a cold, flu or allergy. You can use artificial tear to help with the eye inflammation.Hope I have answered your question. Let me know if I can assist you further. Regards, Dr. Dorina Gurabardhi, General & Family Physician"
},
{
"id": 216725,
"tgt": "What causes sharp pain on the left side of my head?",
"src": "Patient: I have pain and soreness on the left side of my head, neck, and shoulder. It started out as sharp pain on the left side of my head last spring off and on. Now it has progressed to pain and soreness constantly. I get relief only when I do Yoga or light aerobics. What could be causing this? Please help. Doctor: hi,thank-you for providing the brief history of you.A thorough clinical neuromuscular assessment is advised.As you have pain in neckz, shoudler and upper back, It should be because of the nerve impingement in the cervical spine. Also, an MRI will help understand the soft tissue status.Since you are getting relief by Yodg and aerobics this activities helps in improving the blood circulation and muscle stretching. Undergoing physical therapy for the same and getting a specific muscle Strengthening will help ease the factor.In my clinical practice such cases respond well to physical therapy.RegardsJay Indravadan Patel"
},
{
"id": 138846,
"tgt": "Suggest treatment for numbness and pain on the ankle after falling down",
"src": "Patient: Yesterday I was running with friends and rolled my ankle causing me to fall to the ground I heard a pop as my ankle rolled right after I couldn t put any weight on it and was in terrible pain I put ice on it and I took Norco for pain since I ve been able to limp on it but it still hurts bad to walk on it as well as my foot and ankle goes numb all the way to my knee weather I m sitting as well as walking on it also I think I sprained it badly but not sure Doctor: Here you need to take x-ray of your ankle and check on it for hairline fracture. If there is no hairline fracture then it can be due to soft tissue strain. It could be ligament sprain or muscular strain. How ever it can be conformed with only MRI scan.Here based on report you need to take treatment after consulting orthopedic doctor. If there is a hair line crack then need to keep plaster for 2-3 weeks. And if there is no crack in bone then can be treated with conservative management like physiotherapy treatment and use of cold pack and hot pack. You can also use crep bandage over the ankle in figure of eight way to stabilize the ankle and support the ankle muscles and ligaments.I am sure if you follow all above steps then will have good relief in pain. But have patience for treatment because either its hair line crack or soft tissue damage it takes 2-3 weeks to get good relief.Take care"
},
{
"id": 218000,
"tgt": "What is the pain above my belly button?",
"src": "Patient: I have had pain above my belly button for 5 days now and it keeps getting worse... the sikin is pink in color warm to the touch and feels like there is a lump under the skin... I just bumped it and now there is clear drainage from it... what is wrong with me? Doctor: Dear Friend.Hi , I am Dr Anshul Varshney , I have read your query in detail , I understand your concern.It seems you hhave got local infection of hat area, causing a pus formation there.You might require antibiotic therapy for the same.I advise you to see a doctor soon, and to get antibiotics prescribed after local examination.In case pain is severe, you may take Acetaminophen in moderated doses.I hope this would help you.This is my personal opinion based on details available here. If you still have any query you may please ask me.Stay Healthy. Dr Anshul Varshney , MD"
},
{
"id": 84564,
"tgt": "Does nurstre injection cause pregnancy again?",
"src": "Patient: hey..i am 24 years old nd i startd using the nurstre injection the nxt day after i had my bby boy...so iv been on nurstrue for 10 mnth now ....i feel lik thers sumtng moving in my tummy went o the clinic they took a test nd it was negitive....is it possilbe that i culd be pregnt??? Doctor: Hello, Pregnancy is unlikely. If you are worried you can simply go for a urine pregnancy test(UPT) to confirm whether you are pregnant or not. Hope I have answered your query. Let me know if I can assist you further. Take care Regards, Dr Shinas Hussain, General & Family Physician"
},
{
"id": 104031,
"tgt": "Have allergy to food. No medication as off now. What is the cure for this?",
"src": "Patient: I ave become allergic to a lot of foods and things over the past 5/6 yrs and take benadryl to relieve the allergic reactions; hives, itch and such. I always take one then two then three to help it settle down and a shower. The other night I had to take 5 and it was horrible, took a while before I was okay. Is there somthing else I should be doing? I had an epi pen twice but it expired and then I wasnT sure if I really needed it. Doctor: Hello and Welcome to HCM, Thanks for writing to us. The only way to avoid an allergic reaction is to avoid the foods that cause signs and symptoms. oral immunotherapy (OIT) as a treatment for food allergy. If you have any suspicion at all that a food may contain something you're allergic to, steer clear. visit the allergist and get the treatment according to that. Hope this helps you. Wishing you good health... Regards."
},
{
"id": 204866,
"tgt": "How can schizophrenias and anxiety be treated?",
"src": "Patient: All indicates my mom has schizophrenia paranoia. I ve talked to her about going to doc but she says we are the crazy ones. I tell her she can at least get medication to stop the anxiety from what she hears. She says the only way she won t hear is that she s dead and then accused me of wanting her dead. This is hard. Don t know what to do.abby Doctor: Hello there I have read through your and understand your concern. Schizophrenia is a type of Psychosis and needs to be addressed as soon as possible. The way you have described her symptoms she seems to be suffering from paranoid schizophrenia. If treated properly she can lead a Normal life. It is very important to take her to a psychiatrist and get her treated as soon as possible as symptoms will worsen if not treated immediately. You have to be very patient as whatever she is doing and saying is because of the disease. I hope that answers your question. If you have any further questions please feel free to write. Thank you for choosing healthcare magic."
},
{
"id": 175897,
"tgt": "Suggest treatment for developmental delay in a child",
"src": "Patient: My son is having a hard time with staying on track in school. I not sure what is going on with him ,but I know he is slow at doing everything. He is very good in school, but he is always forgetting something. I also notice that it took a long time for him to learn how to tie his shoes. He also never would use both legs to go up or down the stairs. He is 7 now and just recently started using both feet to walk down the stairs and sometimes you still have to remind him. Doctor: Hello thanks for asking from HCM for your health related queriesI can understand your concern. He is seven years old and has started showing symptoms like hard time in keeping track at school, forgetting things, difficulty in learning the tasks etc. These symptoms can occur due to some intellectual disability or developmental delay. I will advise you to get him evaluated by a child psychiatrist for evaluation of IQ and for treatment. If IQ comes in borderline to mild range then medicines like Piracetam, Multivitamines etc can help. These medicines help to improve his cognition and he would feel better. Thanks, hope this helps you. Take care"
},
{
"id": 74557,
"tgt": "Is it advisable to take R-Cinex for cold and cough?",
"src": "Patient: Hi, 27 year old male, I had got treatment for TB, First two months AKT-4 was adviced and next 4 months R-cinex (450 MG Capsul), Completed the both the courses, The course was ended on december 2009. Now, If I get sever cold and cough, I do take R-Cinex without any prescription by doctor, for one or two days, the only side effect is Urine get coloured, is it okay to take the tablet in these situation? please advice. Doctor: Respected user , hi Thanks for using Healthcaremagic.comI have evaluated your query thoroughly .* R-Cinex is an anti tuberculosis drug , has to be used as a regimen drug when under treatment of tuberculosis ( pulmonary or extra pulmonary ) * It can never ever be taken for simple cold and cough on OTC basis without medical guidance .Hope to clarify your query .Welcome for further assistance .Thanks for using Healthcaremagic.comRegards dear take care ."
},
{
"id": 126876,
"tgt": "What can cause swelling and itching on the legs after twisting the knee?",
"src": "Patient: I TWISTED MY KNEE WHEN MY KANE SLIPPED WHEN I HIT A WET SPOT ON CARPORT. FELT LIKE MY WHOLE BODY TWISTED. LEGS KEEP SWELLING AND FEEL MORE LIKE A CARPET BURN AND ITCHES. I DID NOT FALL BUT CAUGHT MYSELF ON POLE ON CARPORT. ALL MY WEIGHT WENT ON LEFT LEG. Doctor: Hi, It may be due to minor contusions or ligament injury associated with twisted knee. As a first line management, you can take analgesics like Acetaminophen or Tramadol for pain relief. Apply ice packs also. If symptoms persist better to consult an orthopedician and get evaluated. Hope I have answered your query. Let me know if I can assist you further."
},
{
"id": 92400,
"tgt": "What could on and off sharp pain under belly button with severe cold indicate?",
"src": "Patient: I have been suffering from a really bad cold these past few days and however tonight I was just sitting there and I go this really bad pain in the lower part of my stomach just below my belly button, then around half an hour later I go another really sharp pain in the same place and I couldn t walk cause it was giving me that much pain what could it mean? Doctor: Hi,From history it seems that you might be having abdominal colic giving rise this problem.Take antispasmodic tablet like Dicyclomine ( Cyclopam ) as and when required.Take light diet.Take plenty of water.Ok and take care."
},
{
"id": 58446,
"tgt": "Had elevated SGPT, started Silliver and Silymarine. After month still not in normal range. Suggest?",
"src": "Patient: hi.My SGPT level was 350+ couple of months ago.a retest after a week to that showed 230.another test a week later showed 280 again.during this period I restricted myself from oily or fried stuff.then I started Silliver a Silymarine tablet.. after a month a re tested to find it 145.the Silymarine wasn't used daily.cant figure out how to go for it now.I need a quick remedy to bring it to normal range.is there a 1 week miravle formula..regards Doctor: HelloThanks for writing to HCMYou didn't mentioned your age.You need proper clinical examination and full investigations.Investigation includes routine hemogram,RBS,Lipid profile,LFT,RFT,GGT,Ultrasound of abdomen, Viral markers, CT Scan of abdomen can be done if needed.You need proper history taking like alcohol history etc.Treatment depend upon reports.Probably you need medicines other than silymarine.Get well soon.Take CareDr.Indu Bhushan"
},
{
"id": 143856,
"tgt": "How effective is L-Serine for Amyotrophic Laterals Sclerosis (ALS)?",
"src": "Patient: My husband thinks he has ALS because he has many symptoms that emulate this disease such as muscle atrophy and fasciculations. He has had neurological testing from to specialists which came up inconclusive. I have been researching and discovered a physiologist Paul Alan Cox who has been testing L-Serine. What do know about this suppliment for ALS. Doctor: As of now there is no defiant I've cure for ALS....L serine is of doubtful benefit. . but as it dint cause any harm...it can be given.Rilutor is an only drug approved for ALS.regards."
},
{
"id": 106272,
"tgt": "Continuous cough problem. Is it any chest infection ?",
"src": "Patient: sir my name is farooq ahmed mughal and i m from karachi,sir cousin is facing alot with cough problem?is it chest infection?what r the main causes of this problem?and what do u suggest that we may consult with?waiting for ur kind reply Doctor: Continuous cough can be due to various causes. Among the most common are Asthma, Allergic rhinitis, Acidity with acid regurgitation, Smokers Bronchitis, or Lung cancer. Get your cousin checked-up with a Chest Physician."
},
{
"id": 50356,
"tgt": "Work out daily, take weight gainer, suffer lung problem, left renal litheasis. What to do ?",
"src": "Patient: Hiii m AAAA.. m gym work out daily nd get early morning drink whey protein after work out drink weight gainer and repeat also before 1 hour sleep again drink weight gainer . 2 days before m getting scanning stomach. M suffering from lungs probelm- left renal litheasis(small) c hydro nefluois c ureteru obstruction c small coveretions sylt- kidney c fally liver Doctor: Hi, many thanks for the query!Stone in the ureter causes obstruction to urine outflow, so urine accumulates in kidneys causing so called swelling of kidneys i.e. Hydronephrosis.You need to do certain investigations- USG (Abdo-Pelvis), X-ray KUB,RFT, LFT, Sr. Uric acid, Urine (Routine & Microscopy).You haven't mentioned, exactly what Lung problem you have.If you are drinking alcohol, stop it gradually.For time being, you may take anti-spasmodics, pain killers withyour doctor's opinion.Drink plenty of water so that at least 2 litres of urine is voided in 24 hrs.Wish you a good health. Take care. Regards."
},
{
"id": 21163,
"tgt": "Suggest remedy for rapid heart beat and pain in chest",
"src": "Patient: My hearts been Beatin extremely loud have been coughing some some but chest hurts bad last night it was awful I was tryin 2 sleep but my heart was pounding so hard I was scared I'm only 20 i do smoke cigs It felt as if I was goin to die was waiting for the pain to ease but , unfortunately this morning the center of my chest was so sore. It's felt like a big bruise and it feels like it races Doctor: Hello, thanks for posting a question.You seem to be having issues with palpitations. The can be caused emotional issues, smoking, underlying heart issues. Most of the time they are not harmful but scary. I advise you exercise at least 3 times a week, eat healthily, avoid alcohol or smoking and sleep 6-7 hours a day, and hopefully, you will be fine. Please see a physician if you do not get relief in 2-3 weeks for a physical examination. Best regards"
},
{
"id": 98216,
"tgt": "Having pain after a gunshot which was done 7 months before",
"src": "Patient: Hi Doc, About 7 Months Ago I Was Shot twice once in my thigh and once in my buttocks a exit wound for each, its been semi pain over the months but i dont know if its because of the cold weather (A freind who was also shot before suggested) but the pain has worsened, how can i fix this matter? Im sure its not infected but my leg gets soar out of nowhere sometimes. Doctor: Hi Welcome to healthcare magic forum The bullets could have severed muscles and nerves. So the pain is being experienced. You need medical attention. For pain relief, you can take homeopathic remedies safely. Takecare For further enquires- dranshita.rathore@yahoo.co.in"
},
{
"id": 90472,
"tgt": "What causes abdominal pain after consuming ipill?",
"src": "Patient: gud mrng dr ,i did sex with my hubby on 6 th july , from that day i having gud pain in my abdomen , and on 6 th evening i took ipill , from that pain is there , today (11 07 12) i having gud bleeding which passing urine and pain is server , can u pls guide me wat i have to do Doctor: Pain in abdomen after consuming ipill is due the referred pain due to strong uterine contractions arising as a result of ipill ......so don't worry .just chill its spasm of uterus causing you pain in abdomen....tc care"
},
{
"id": 125211,
"tgt": "What causes swelling in the ankle for a person who is under medication for BP?",
"src": "Patient: I have been under medication for BP for more than 5 years. Recently I have changed the table from ATEN -25 to Amlodac -5. I take ecospirin av 75 at night. I have swelling in the right ankle after a day s work. I teach English at an engineering college. Doctor: Hello, It is due to long-standing hypertension. Consult a physician and switch to drugs like losartan. If the swelling persists additional drugs like Lasix might be required. Hope I have answered your query. Let me know if I can assist you further. Regards, Dr. Shinas Hussain, General & Family Physician"
},
{
"id": 221777,
"tgt": "What are the symptoms of pregnancy after having unprotected sex?",
"src": "Patient: my boyfriend finished inside of me without a condom on or anything sunday the 26th of september. i also want to point out that before we had sex that sunday we haven't seen or done anything with eachother for an entire week so i am sure he had more you know what then usual. i have birth control & only take it sporadically because it makes me feel sick. so because i have messed up my cycle from not taking birth control correctly i have been spotting a lot lately...i dont even know when i am expected to get my period its that messed up..i also have a track record for taking plan b, which is causing my period to be crazy too. so what i am concerned about is that i am only 20, he's 22 & we are nowhere near ready to have a kid. i haven't take a pregnancy test..what symptoms would i be having to confirm that i could possibly be pregnantmelanie Doctor: Hello,Your description indicates that your menstrual pattern is totally disrupted and in such a situation, it is difficult to predict whether you have ovulated or not. The only way out to know if conception took place due to that sexual intercourse is to get an estimation of the serum beta-hCG titres 2 weeks after the act. This is the earliest and most predictable way. You can also get a HPT not earlier than 3 weeks after the incident. A trans-vaginal sonogram at the same time also helps. If negative, make sure your hormonal balance is returned to normal to ensure proper planning of contraception. Hope this helps."
},
{
"id": 47652,
"tgt": "What does my USG report suggest?",
"src": "Patient: In my USG report, it is written that there is heperechoic one SOL (37x 35 x 31) mm in the mid and lower pole of right kidney with peripheral increased vascularity (? character) . n the imression it is written right renal SOL (? character), coarse liver parenchyma and prostatomegaly (grade I). What does it mean ? ReagrdsSubhasis Doctor: Hello Subhasis and welcome to HCM.As an Urologist, let me try to explain the report.Normally,sound or\"echo\"waves, pass uniformly through the kidney tissue.But, in your case, they are 'hyper'echoic, meaning, of increased penetration. SOL,means\"space occupying lesion\",so there's an abnormal area there.Again, liver should show uniform,sound penetration. Report is coarse.Early, prostate gland enlargement is also seen. If you can send the scan, with urine and blood reports, as a direct question to me, i can give an expert opinion.Dr.Matthew J. Mangat."
},
{
"id": 217440,
"tgt": "Suggest remedy for severe arm pain",
"src": "Patient: My Husband had an angioplasty in June 2010 from the wrist he has been getting servere arm pain since he had the procedure we have found out through a sonologist that the radial artery is blocked due to the angioplasty what can we do to releave the pain he is all ready on clopidogrel ramipril diltizem rousavastatin and nicorandial Doctor: pain from lack of blood flow has particular characteristics... feels like severe fatigue.. either it occurs with use OR there is very bad blood flow indeed ! and there is tissue damage (gangrene).Or.... other causes of arm pain....certainly neck problems like a pinched nerve in the neck can cause pain when the nerve that is pinched has pain along its pathway (neck, shoulder, arm, hand). This common condition is either painful All the time, or with moving the neck.Uncommonly, there can be pain from the angioplasty. It's location mostly is in the chest but can be in the left arm (like angina).cannot say in your particular case without a direct examination."
},
{
"id": 11692,
"tgt": "Hyper pigmentation, acne problem, skin oily in summer and dry in winter. Suggest good sunscreen lotion?",
"src": "Patient: I am 33 years of age and a female .I have a very oily skin in summer and dry in winter.very sensitive too. Having a hyperpigmentation and acne/pimples problem, I need a very good sunscreen which can get rid of both pigmentation and pimples. Couldn't use most of the sunscreen because it makes my skin more oily. Thank you. Doctor: Hi,You are having facial melanosis and acne vulgaris.Facial melanosis may be due to pollution, cosmetics, vitamin and mineral deficiencies, anaemia,internal diseases,stress,excessive sun exposure and so on..Acne vulgaris is due to tendency ... and you have oily skin..it may be acneform eruptions...acneform eruptions may be due to stress, anxiety, oily food,milk intake, ghee,excessive sun exposure, drug like steroid, INH,iodine in cough syrup..You take vitamin C, nicotinic acid and glutathion containg tablets for melanosis.Kojic acid and vitamin C containing cream may be applied at night on melanotic patches.For acne, you take antibiotics like azithromycine or minocycline.And you apply clindamycine gel on acne lesions.I do not advise to apply sun screen at this stage ..After control of acne lesions ,you may apply sunscreen. No sunscreen can cure acne. It only protects the skin from sun rays , which cause melanosis.You avoid tesion , worries, foods which aggravate acne lesions.Take healthy food and proper sleep.I hope you may got proper advice..Regards..Dr. Ilyas patel MD"
},
{
"id": 134176,
"tgt": "Suggest treatment for pain in back and hand after an injury",
"src": "Patient: I m a female in my 40 s and very active. i fell hard on my back (during my step arobics/zumba workout at home), my main back bone hit the corner of my brick fireplace and my hand hit the floor hard. Now my back bone hurts when i bent my back backwards its been 24 hrs since fall. My doctor cant see me until 30 more hours, should I be worry ? I love to dance latin music so Im worry that i wont be able to move as before the fall. My hand also hurts when I open my fingers. Any advice while I wait to see my doctor? Doctor: hi,as you mentioned about the pain and the mechanism of injury. you need to first take a bed rest as your back is still hurting. Also using hot water fermentation for the hand and back will be good to soothe it. taking an x-ray of hand and MRI of spine will help further. depending upon the extent of the pain and injury the next treatment is planned, but for time being you should be resting as it is unclear what injury it is.also post all these, a good exercise programme will help to the deepest extent to make you back in Latin dancing.with the grace the God I wish you a speedy recovery."
},
{
"id": 81071,
"tgt": "Suffering from shortness of breath specially while doing exercise",
"src": "Patient: Hi my name is Frankie and am from PNG.Theres this cold and numb thing on my feet and hands and makes me feel sick every afternoon after i go home from work. Before that i was frequenting hospital due to viral infections/ URHT and also chickungunya malaria. Malaria blood test is negative. Am also suffering from shortness of breating especially when doing exercises. Doctor: Thanks for your question on HCM.I can understand your problem and situation.Breathlessness on exercise is seen commonly in cardiac and pulmonary causes.So we need to rule out these first.So get done1. ECG to rule out cardiac cause.2. Chest x ray and PFT (pulmonary function test) to rule out pulmonary causes.If all of the above are normal then get done CBC (complete blood count) to rule out anaemia and thyroid function test to rule out thyroid diseases, because anaemia and thyroid diseases can also cause breathlessness on exercise.If these are also negative then you are mostly having anxiety.So avoid stress and tension. Be relax and calm."
},
{
"id": 193397,
"tgt": "What causes penile pain during intercourse?",
"src": "Patient: i have been having a lot of sex with my girlfriend 2 time a day sometimes 3 but the other day it was painfull i see looks like a few cut arond the bottom of the head on the shaft and wow does it sting when i get erect or pull the skin back its like very painful what can i do for it Doctor: Hi, You can have sexual intercourse about 3 to 4 times a week. If you have more than that you will feel fatigue and pain. If you still want to increase the intercourse times - you need to be like a professional by doing regular exercise and balanced diet. Hope I have answered your query. Let me know if I can assist you further. Regards, Dr. S. R. Raveendran, Sexologist"
},
{
"id": 221419,
"tgt": "How should a painful headache during a pregnancy be treated?",
"src": "Patient: hi my name is srinivas from hyd i am midil class family my wife is some kind painfull problum she have a headec it's very painfull to her. some time she crying like i can't say that she have a pain the medile up said of the hed we r going many more hospitals but no results now she is pregent she can't take any tablet's please kind of me what can i want to do? Doctor: Hello dear,I understand your concern.In my opinion the cause of headache needs to be evaluated.The headache can be due to stress,high BP,migraine etc.The high BP needs to be ruled out.If it is normal then it is ruled out.Avoid tension or stress.Take paracetamol tablet which is safe in pregnancy.Take good rest and healthy nutritional diet.Take adequate fluids upto 3 litres per day.Migraine also presents with headache on one side with giddiness and vomitings.I suggest you to consult a neurophysician and get the opinion.Best regards..."
},
{
"id": 14393,
"tgt": "What causes itchiness skin rash on nose?",
"src": "Patient: skin rash on my nose, skin comes out and its very itchy. comes and disappears when i apply broad spectrum antifungal creams.. male, 24. Is it an allergy or what? im confused, been to all kinds of hospitals in my town, no one knows what it is. please help Doctor: Hello. Thank you for writing to usI suggest a likely possibility of seborrheic dermatitis.Seborrheic dermatitis presents as red, itchy, scaly patches on sides of nose commonly.Other facial areas like glabella, eyebrows, chin etc may also be involved with these dry, scaly patches.Topical antifungal creams do help partially but the condition can recur after seemingly complete cure.If I was the treating doctor I would suggest you to apply a moderately potent topical steroid e.g fluticasone propionate 0.05% cream twice daily for a few days.For mild involvement and for recurrent disease a topical immunomodulator e.g tacrolimus Or pimecrolimus is suitable.I suggest that you visit a dermatologist for a confirmatory diagnosis and for the needful.Regards"
},
{
"id": 140028,
"tgt": "What causes head spinning, hot and cold flashes?",
"src": "Patient: hi i am verry worried about my mom, she says her hed feels like it is spinning and that she feels hot and cold, its been like this since this morning. so I am trying to find out how sirous this thing is. sorry for any spelling mistakes! :) and if i haveto pay for an ans srry bt then you wont need to answor because i cant do that. but i would highly apreshate it if you could help me. thnx Doctor: Hello, What you're describing happens commonly in people with viral infections of the ears, sinuses, or upper respiratory tract. Since you didn't say anything about nausea or vomiting I'm assuming that's not an issue and that is good news. However, if she is unable to stand or move due to the dizziness and she continues to have these symptoms even when lying down then, I think it best she be seen by a doctor in an Emergency Room. Make sure you give her plenty of fluids to drink if she is not able to easily get up, call her doctor and then, follow their instructions. If she starts to have nausea, vomiting, or has trouble with her speech or doesn't seem to understand things because she's zoning call 911 immediately. Hope I have answered your query. Let me know if I can assist you further. Take care Regards, Dr Dariush Saghafi, Neurologist"
},
{
"id": 123430,
"tgt": "What causes pain and swelling on toe?",
"src": "Patient: I bumped my foot to a chair last night. Now my right fourth little toe is painful that it kept waking me up all night. I can t curl my toes on my right foot like I do my left. It hurts to walk on it. I did ice it last night but do i still need to see a doctor for the ongoing pain? It does look a little swollen compared to my left toes. Doctor: Hello, On a safer side, an x-ray is advised. Firstly stabilize the injured joint. As the movement will happen and also due to weight bearing the inflammation at the ligament will not subside. So stabilizing is of utmost importance. Do hot water fermentation and slow passive movements of the toes. Don't perform the active ones. Within 6-7 days of the time, the pain will subside if there is no fracture. Hope I have answered your query. Let me know if I can assist you further. Regards, Jay Indravadan Patel, Physical Therapist or Physiotherapist"
},
{
"id": 147565,
"tgt": "What is the cause of whole body trembling?",
"src": "Patient: Hi, I just drank coffee (I'm not normally a coffee drinker) and now I'm feeling really shakey - my whole body is trembling, my stomach feels shakey and I can't relax. I feel like I want to curl up and cry. Is this a normal reaction to coffee if you haven't had it before? How can I make it go away? Doctor: coffee might increase anxiety or may cause panic attack but only in those who are predisposed to these disorders. Please get your evaluated by a physician and rule out these possibilities. Hope this helps.."
},
{
"id": 14402,
"tgt": "Suggest treatment for tender rash on right eye lid and scalp which is spreading",
"src": "Patient: I have a tender rash on my right eye lid, by my hair line, and a tender bump on my scalp in my hair approximately 2 inches from my hairline on to of my head. I have not hit my head on anything lately. I have been under a tremendous amount of stress. My concern is that it's spreading. It's been a little over 24 hours. Doctor: Hello. Thank you for writing to usI would like to keep a possibility of shingles/ herpes zoster.Shingles usually presents as painful, fluid filled lesions that are limited on one side of the body, in a particular area/ dermatome.The cause is re-activation of the same virus that causes chicken pox at a young age i.e varicella zoster virus/ vzv.Diagnosis is obvious on clinical examination. Therefore I suggest you to visit a doctor for a confirmatory diagnosis.Treatment of confirmed shingles is with oral antivirals e.g acyclovir Or valacyclovir. Treatment duration is 7-10 days.An OTC oral anti-inflammatory/ painkiller e.g ibuprofen can be taken twice or thrice daily for symptomatic relief from pain.Regards"
},
{
"id": 205109,
"tgt": "How can aggressive behaviour be controlled while suffering from autism?",
"src": "Patient: Caregiver for a 38 year old young man , Mike , having retardation with autism. 6 mos. ago he developed bahavorial disorders , puts his arms over his head and wiggles his fingers , face turns reddish and he often screams very loud while rambling on verbally. he has also recently began hitting / tapping me , Joe , with his fist. just this morning he hit his mother while she was trying to calm him down during one of his episodes. I love him dearly. His mother has some dimentia so my hands are full. plus I m on kidney dialysis :( Can you recommend a lite drug for Mikie. He has no other current health issues. I removed him from a pre-diebetic problem by lowering his body weight from 225 lbs. down to 115 lbs. in just a year. his primary says he s one healthy guy. THX , Joe Gerst Doctor: Hi and thanks for question.in autism due to disturbance in order list they become violent, and behavior disturbance occur. i will suggest u start aripiprazole or risperidone at low dose than gradually build up its dose. if behavior disturbance more than we add mood stabilizer also.along with that u can consult psychaitrist.thanks"
},
{
"id": 80975,
"tgt": "Suggest medication for cough,congestion and breathing problem",
"src": "Patient: My husband, has continous coughing , bringing up pllegn , to the point that he almost chokes and becomes very breathless , this happens constatnly sometimes 6 =8 times a day. His doctor says is reflux and has put him on vEXION . He has had this for many months now , and he feels awlful . the Vexion is not doing anything to help do you have any suggestions. Doctor: Thanks for your question on HCM.I can understand your husband's situation and problem.In my opinion you should consult pulmonologist first for your husband and get done1. Chest x ray2. PFT (pulmonary function test)Chest x ray is needed to rule out lung infection.PFT is needed to rule out bronchitis.Both lung infection and bronchitis can cause cough, congestion and breathlessness.So better to consult pulmonologist for your husband, first diagnose him and then start appropriate treatment.Don't worry, he will be alright."
},
{
"id": 217602,
"tgt": "Suggest treatment for sciatic nerve pain",
"src": "Patient: i m female 37. 5.5 ft. i m suffering from sciatic nerve pain since last 9 yrs. followed physiotherapy, yoga, took non asteroid medicine, hydrotherapy, but no improvement at all. pain is severe while i stand . there is swelling on L 4-5. at the same time there is scattered black and brownish blob on my back may be notalgia paresthetica? for this also i consulted many doctors they all sugsted dermatological medicine but of no use. is this problem related to the sciatic pain? My major problem is pain . plz suggest me what i need to do . i dont want to go for surgery, Doctor: HiiI am a Physiotherapist. Dear I don't think that, this blob is notalgia paresthetica as it usually occurs with pruritis/ itching and at mid back region.Please describe more about your blob like location, itching/ painful, nodules, any discharge etc.Secondly if your sciatica pain is not improving even despite of taking all the conservative and allied treatment from last 9 years. Then you should go for extensive examination and meet Spine specialist/ Surgeon before it is getting too late.It does not mean that you should go for surgery but now you should know the current condition of your problem.Thank You"
},
{
"id": 10219,
"tgt": "Are there medications to cover the grayness of hair?",
"src": "Patient: hi, i had to undergo hair transplantation, as i had baldness at front and middle...i had hair on sides and back but my hair is not so strong and having white hair....can u please suggest me any medicines to get rid of this white hair and to make my hair stronger...and also the diet wht to be taken? Doctor: Hello and Welcome to \u2018Ask A Doctor\u2019 service. I have reviewed your query and here is my advice. There isn't medical treatment for grey hair. I can only suggest you dye them with a good brand hair dye. You can start applying topical Minoxidil 5% to 10% once or twice a day for at least a year to see optimal results as the Minoxidil promotes new hair growth. Along with that I suggest you start PRP (platelet rejuvenation plasma) treatments where plasma from your blood is injected into your scalp once every 4 to 5 weeks for a minimum of 8 to 10 sessions. PRP will reduce your hair fall, strengthen your hair follicles and also promoted new hair growth. Start having multivitamin tablets which have biotin daily. Start consuming foods which will further strengthen your hair such as eggs, green leafy vegetables, nuts, lentils, avocado, to name a few. Hope I have answered your query. Let me know if I can assist you further. Regards, Dr. Smruti Pevekar"
},
{
"id": 20119,
"tgt": "Is the ASD defect of the heart operable based on the report?",
"src": "Patient: Dear Doctor, I am 45 year female from Lucknow need advise through your section as a cardiologist told me that my herat is inoperable condition. I am suffering from Cardiac problem since last 20 years like pain in chest, dizziness, swelling in feet, breathing problem etc. then i visited a cardiologist and he performed echocardiogram, the results are following - 1. Large Ostium, Secundum ASD with severe PAH (ASD Defect 25mm) 2. Mindely Dilated RA & RV 3. Good LV, Systolic function 4. LVEF =77% 5. Trival MR 6. Mild TR 7. Mild PR 8. Normal Pulmonary connection 9. No clot/vegetation Therefore I want to know that is there surgery is possible in this case or not. Please advise me. Best Regards, suman Doctor: HelloIn patients with ASD with the passage of time the shunt become reversed (right to left) due to increased pulmonary vascular resistance. These patients develop severe PAH due to which its become impossible to close the defect. Its Called Eisenmengerisation of the shunt.This could be the possible reason in your case keeping in mind SEVERE PAH reported in your ECHO report.See in the report if it's written Reversal of shunt or right to left shunt across ASD then surgery is not possible Take Care"
},
{
"id": 70739,
"tgt": "What does the following test result of lung indicate?",
"src": "Patient: I need help please This is some reports about my wife She can t make more testing like open lung biopsy it s impossible. I d like to know if my wife has lymphoma, and if so, which type We need to take appropriate actions. Please reply to me. My wife can t walk 5 steps at the same room and she live with 7 liters oxygen 24 hour a day if she move about 5 steps the oxygen level will be less than 50 . X- RAY REPORT 9-11-2009 Chest Radiograph ( PA View ) Revealed :- Both lung fields show multiple reticulonodular infiltration Giving honey comb appearance with tendency to cystic changes for correlation with clinical data end previous films. Pathology Report Material: C.T. guided fine needle aspiration cytology (FNAC) of a lung nodule & liver focal lesions. Date of aspiration: 24/11/2009 CLINICAL PICTURE: C.T. guided fine needle aspiration cytology (FNAC) of a lung nodule & liver focal lesion smeared over eight slides. MICROSCOPY: Cellular lung smears formed of many groups of epitheloid cells having large nuclei and abundant esinphilic cytoplasm. Many groups of hyperplastic bronchial cells exhibiting mild-moderate atypia were detected. The background was infiltrated with chronic inflammatory ceels mainly lymphocytes with moderate number of uniuncleated histiocytes. Few multinucleated giant cells were observed. The liver aspirate showed many groups of liver cells having bland cntral nuclei over background having infiltrated with lymphocytes. DIAGNOSIS: LUNG NODULE & LIVER FOCAL LESION: C.T. guided Fine Needle Aspiration Cytology (FNAC) Lymphoroliferative disorder suggestive for granulomatous inflammation for immunophenotyping confirmation. PATHOLOGY REPORT Microscopic: Examination of the films received many red cells, many scattered small, intermediate and large lymphoid cells with very few polmonuclear leucocytes. Diagnosis: Lung nodule and liver focal lesions, FINDING ARE IN FAVOUR OF MALIGNANT LYMPHOMA. -CASE FOR TISSUE BIOPSY FOR IMMUNOPHENOTYPING. PATHOLOGY REPORT GROSS: - Referred 8 FNAC smear slides, labeled by patient s name. four of the slides are labeled for site as liver and the other 4 as lung. All slides are examined and re-enclosed. - Three of the slides are de-stained after routine examination, then immunostained for LCA. CK & NSE. MICROSCOPY: - Liver and lung FNAC smears show blood and scattered groups of small, medium and large round cells. The FNAC smears also rich in neutrophils. - The immunostained smears show negative staining for CK & NSE and positive staining for LCA. Conclusion: Cytology slides ; re-staining for immunophenotyping: FINDINGS ARE CONSISTENT WITH NON-HODGKIN S LYMPHOMA ; MIXED SMALL AND LARGE CELL. PATHOLOGY REPORT One of the referred FNAC smear slides is de-stained after routine Examination, then immunostained for CD 20 as requested MICROSCOPY The immunostained smear shows a majority of CD 20 \u00e2??megative cells and few \u00e2?? likely reactive- positively stained small lymphocytes. CONCLUSION: Cytology slides ; restining for CD 20 Most cells are CD 20 negative. Scattered positive (likely reactive) small lymphocytes. Complete Blood Count Ref. Range Haemoglobin 5.6 gm/dl 12.1-16.6 Haematocrit 20.0% 30-40 Red cells count 3.37 mil/cmm 4.2-5.4 Red cell indices MCV 59.3 fl 78-94 MCH 16.6 pg 26-32 MCHC 28.0 gm/dl 31-34 RDW 20.9% Total leucocytic count 11,900 /cmm 4,000-11000 Differential Count Basophils 0 % 0-1 Eosinophils 1 % 1-4 Staff 2 % 1-7 Segmented 68 % 40-75 Lymphocytes 23 % 20-45 Monocytes 6 % 1-8 Others Platelet Count 800,000 /cmm 150,000-450,000 All the best, Doctor: Hello, As you explain the history she has non-Hodgkin lymphoma which needs chemotherapy. I am sorry about your wife's situation. Hope I have answered your query. Let me know if I can assist you further. Take care Regards, Dr Jnikolla, Pulmonologist"
},
{
"id": 96768,
"tgt": "Does electric shock from a cooking grill requires attention?",
"src": "Patient: hi. I got electrocuted at work the other day from a cooking grill that had a frayed cord. My arm hurt for the rest of the day after it happened. And my eyes look like they have dark circles under them. I also feel kind of nervous and gittyery but I think that might be because I am paranoid that something is wrong. Do you think I am okay? I seem to be able to function okay though maybe my mind is a bit on edge or something. Do you think I should go in to see a doctor? Doctor: hiif i was your treating doctor then electric shock needs attention and i would have subjected you to an EKG and also for some electrolytes. if those tests are normal then there is no need to fear and you are alright.if there are any abnormalities then treatment depends on the type of abnormalities. so my advice is to consult a doctor."
},
{
"id": 162486,
"tgt": "What could possibly cause pain in the legs and neck along with a migraine attack?",
"src": "Patient: My 10 yer old son woke up saying he was having a hard time walking on one of his legs, his nexk was really sore (upper back), and hard to turn huis head. 1/2 way through the day he said he started to feel a migrane coming on, and now he has lines / steaks on his back. What could this possibly be? Doctor: Hi, I have gone through your question and I understand your concerns but please head to the hospital for a ct scan on his brain to exclude increased intracranial pressure or subdue all haemorrhage. Hope I have answered your query. Let me know if I can assist you further."
},
{
"id": 204292,
"tgt": "Is inability to focus on more than one thing at a given moment normal?",
"src": "Patient: I m really unable to do more than one thing at a moment. I stop paying attention to absolutly everything around me when i m occupied. I m not able to drive and talk to somebody. I often don t see people juste in front of me when i m thinking or talking on the phone. Is it normal? Doctor: Hello and Welcome to \u2018Ask A Doctor\u2019 service. I have reviewed your query and here is my advice. As you mentioned that you will stop paying attention to absolutely everything around you when you are occupied it means that you are able to fully concentrate on the work that you are doing. I think it's a very good sign that you are able to concentrate on the hope that you are doing so that results will be definitely better as per your concentration levels. And it's always tries to focus on a single task each time then to dilute your focus on multiple tasks. Unable to do more than one thing at the moment is really not a disease. If you are not able to focus on even one task then that should be evaluated further. As you are not suffering from a particular disease there is no need of any medications or investigations. Going for a morning walk daily, exercising for 2 to 3 days in a week, listening to music, spending time with your friends and family members definitely improve your concentration further. So no need to worry about your condition. Hope I have answered your query. Let me know if I can assist you further."
},
{
"id": 60517,
"tgt": "I am 36 year old female with irregular bowel movements. What is the solution ?",
"src": "Patient: my bowel habits are very diferent some times i have alot of trouble moving my bowels an dont for a week or more ori will be awful smelling runny black stool with mucus the pain is so bad im homebound 36yr female had a pancratic duct prob i only have 1 duct instead of 2has been going on a long time far 2 long cant get no one to help some one out there please help tks stomach pain girl Doctor: Hi, Endoscopy, Barium meal Xray, USG abdomen may be helpful for proper diagnosis. Black stool may be due to ulcer. Avoid oilly spicy food. Drink more water. Take food in small quantity and frequent interval. Do regular exercise and jogging. Milk of magnesia and isapgol (pergative) may be helpful."
},
{
"id": 209230,
"tgt": "How to get rid of stress and depression?",
"src": "Patient: I m 16 years old, almost 17 and i deal with stress and depression almost on a daily basis. Ive been bullied in the past, and that has bulit up a huge amount of hate inside me. And also it has changed my personality complitely.(im still a good kid though)Unlike 5 years ago, i get angry easily, anything makes me want to swear, i often catch my self in deep hatred towards humanity BUT I LOVE LIFE, i definetly have my future in mind but like i said, i hate most people. Anywhoo, i also am very strange, i can feel what people are thinking about me, its like my brain and heart automatically read people s feelings about me... MY PARENTS DONT LISTEN, THEY THINK IM JUST BEING PARANOID, but i can guarantee you that i am NOT paranoid. ALSO, i can sense people s pain, and make it my own.. like, i can feel other people s pain and understand them even without ever being in their situation. One more thing. MUSIC, it hightens my ability to live, example, if i aint feeling pretty, or if im tired i can just plog in my ears with some music and my mood lights up automatically. I cant go out without music. (THE REASON IM WRITING ALL THIS, IS TO LET THE READER KNOW EXACTLY WHAT MY PERSONALITY IS). (The other night, i was calmly watching a movie on my phone, very relaxed without a care in the world, and out of nowhere MY HEART SHOOCK TO THE SIDES SOOO HARD THAT I FELT EVERY SINGLE BIT OF IT.) (It actually jumped around) I get really bad heat waves alot, (could be stress, or because i shower with really hot water) and it always happens when im relaxed playing with my phone. My heart speeds every now and then. (Mom says that for stuff like that to happen, a person must be either excited or ancious, but when these things happen i am always relaxed watching dramas on my phone.) When i was little i was taken to the doctor and they checked my heart, and they said that i was 100% healthy, my heart rate was very normal. Infact, every time i go to the doctor my heart pulse is always calm. So i NEVER get to prove that these things really happen. 3 NIGHTS AGO MY LEFT EAR BLED, BUT THEN I CLEANED UP THE BIT OF BLOOD AND IT STOPED COMPLITELY.. NO PAIN, NO NOTHING, INFACT, I GRABBED AN EAR THINGY TO CLEAN OUT THE BLOOD, BUT THE BLOOD WAS GONE. IT ONLY TOOK ONE EAR THINGY TO CLEAN MY EAR, THEN THE BLOOD DISSAPEARED. Is this normal? Are my heat waves harming my ears? Idk, (PERSONALLY I DONT THINK THAT THERE IS ANYTHING WRONF WITH ME, BUT THEN AGAIN, I SHOULD LET THE PROS DECIDE THAT.) Doctor: Hey. First of all I must appreacite you for understanding your inner conflicts so well, and for seeking help. Now coming to the queries you have pit, a detailed interaction with you is needed to get an hold of your personality, which needs proper assessment. Visit a psychiatrist\\psychologist. Don't loose hope. There so many help options available for you. Right now try to engage yourself in meaningfull activities, spend time with your hobbies, keep yourself busy and do some sports or any other physical activityRegards, Dr Arun"
},
{
"id": 41487,
"tgt": "Suggest remedy for infertility problem",
"src": "Patient: im 32yrs old married for 5yrs having pcod with f.tubes normal in hsg. undergoing fertility treatment since 4yrs starting wit tabs first then with ovlation inducing inj n iui. no result. went thru ov. drilling n endo biopsy for pcr n culture. tested tb pcr positive, AFB neg, culture negative. taken Akt4 2mnths,Akt3 1mnth . planning for ivf suggest if to continue tabs for another 3 mnths or goahead with ivf. Doctor: Hi, first complete your ATT course then go for IVF. mean while try to reduce weight so IVF drugs work well. AS you are taking treatment along with that you go for IVF test also so your time is not getting waste . Good luck."
},
{
"id": 81642,
"tgt": "What causes tightness feeling in chest?",
"src": "Patient: 39 yr old female, very healthy, vegetarian, exercise regular, 124 lbs, 5 8 . Having tighnes in my chest, no pain on left side. tightness comes and goes, lasts for about 3 seconds, but hurts. makes me catch my breath. Non-drinker, non-smoker. Weird. Has been happening for a few days now. Should I see someone? Doctor: Hello Tightness in chest for few seconds need proper evaluation.It may be due to cardiac reasons.You need clinical correlation and investigations.Investigations include routine hemogram,renal function test,liver function test,renal function test,ECHO,ECG in all leads,chest X-ray(PA view).Further investigations can be done if needed.Proper treatment depend upon findings.You should consult cardiologist.It is good that you are non-drinker and non-smoker.Get well soon.Take CareDr.Indu Bhushan"
},
{
"id": 7635,
"tgt": "Acne on face, back, forehead, cheeks, sensitive skin. Taking doxt-s, minoz. Treatment?",
"src": "Patient: Hello Doctor, I am male age 22. weight 52. As of now I am facing acne problem. exaclty 1 year back the problem is ver severe, I got acne in my back , and on face. after that I consulted a doctor, he suggested me minoz od tablet and face clin gel, doxt s - for night . These worked a lot. my acne marks gone and skin became normal. But after use of the above for medicines , acne marks reduced and almost no marks for over 5 monts.But now they started again and the sevearity is meidum.. they are on my forehead and cheeks.with a light reddish. I have been using doxt-s for past 13 days. More over my skin became so sensitive. I stopped using oils and junk foods. If I take any oily food immediately I used to get 2/3 acnes. and Skin become sensitive to sunlight also. Doctor: Hi, Thanks for choosing HCM, the problem of acne on face and back generally occurs when you have dandruff in your hair so get rid of dandruff use selsun lotion for shampooing your hair take all necessary measures to avoid dandruff like regular hair wash regular combs wash dont use the same caps ,they carry dandruff after you wash the hair also take care against pollution.....use scarf to cover hair as well as face completely clear out your dandruff take less fatty foods, go for good diet and exercise keep your skin away from pollution(use a scarf etc) try to control cause of acne frequent facewash as well as hand wash stop fast foods seriously have fresh fruits,green leafyveggies reduce spicy foods dont scrub the area inorder to get rid of pimples because it makes your pimples spread dont squeeze them ,it leaves pores and dark patches clear your pimples by following the above measures or meet a doctor... once your pimples get cleared completely ,then only the treatment of pores and black patches can be cleared succesfully . this happens naturally after your pimples clear off or you may need medication also remember dont go for facials when you have pimples it worsens the condition also go for natural remedies like applying tulsi paste or neem leaves paste apply sunscreens like sunban with spf factor more than 25 so that your skin gets protected ya males need to be extra careful while shaving the beard...it causes the pimples to spread use a new razor or blade each time you shave and see that you dont rupture the lesions hope i answered your query feel free to contact me for further queries thank you"
},
{
"id": 112153,
"tgt": "What is the reason for having lower back pain, pain in hip and pain in left side?",
"src": "Patient: I HAVE BADE PAIN IN MY LOWER BACK AND HIP IN TO MY LEG RIGHT SIDE AND I ALSO HAVE PAIN IN MY LIFE SIDE ALLSO SOMETIMES . I HAVE HADE A XRAY AND I WILL NOT HEAR FROME MY DOC UNTILL MY NEX APP WHAT COUND IT BE .itSTARDED WHEN I WAS IN TH ARMY AND THEY DID NOTHEN. Doctor: what is your age and duration of symptom. i will suggest you now take rest,warm compress and mucle relaxtant tablet like thiocolchicoside and pregabalin tablets and consult with your phyisican for prescibing drugs and other investigation for further followup like xrays of LS SPINE and MRI of spine if your symptom ouare for more then 12 wks.i my opinion you are having disc disease required further follow up."
},
{
"id": 172593,
"tgt": "Can formula feed be introduced in 3 weeks old baby?",
"src": "Patient: I have a 3weeks old baby who only feeds on breast milk. Am worried because she looks dehydrated, tiny and malnourished unlike when she was born. She cries often and seems not to be getting enough breast milk. What do I do, do I introduce formula feed. Doctor: Hi, I had gone through your question and understand your concerns. You should be worried but not too much. You should introduce NaN or other if you would like and if baby don't gain weight. Start from 30 ml,add every day 30 ml. Give 90-100ml for 1 feeding. You can give NaN 1-2 times .Hope this answers your question. If you have additional questions or follow up questions then please do not hesitate in writing to us. I will be happy to answer your questions. Wishing you good health."
},
{
"id": 187272,
"tgt": "What is the remedy for the Novocain with pain and welling in one cheek?",
"src": "Patient: Hi I had couple feeling done today and had a novocaine on both sides of my mouth.After I left the dentist office both sides were numbed and one side start swelling and hurting. After 3 hours one side feels normal the other side is really swollen and hurting . What should I do? Thanks Doctor: Hi,Thanks for posting the query, I would suggest you to get the filling checked, there could be presence of infection on the affected side.Take tab brufen BD .Take lukewarm saline and antiseptic mouthwash gargles.Take care!"
},
{
"id": 134225,
"tgt": "What causes general weakness and muscle pain while having seronegative arthritis?",
"src": "Patient: Hi, I am Milind, diagnosed sero negative arthritis,with Rh factore positive.ANA TEST IS NEGATIVE.I am on celecoxib 200 1bd,Leflunomide 20 1 od & Hydroxychloroquine 200 1 bd. My overall condition is stable & without pain episode ever since medicine started 5 month back. HOWEVER, I am fealing weakness & mascular pain on stress. My physician has given Myoril 4mg 1 bid for week. HOW long these medicine will continue? Do you suggest any other reasone for weakness & mascular pain. I am non diabitic, no history of hypertension,lipid profil, LFT, RFT & BIOOD PICTURE REPORT NORMAL. AGE 53Y MALE, WT 90KG HT 165 CMS. Doctor: Hi,Thank you for providing the brief history of you,Since you mentioned about you have been diagnoses with Sero negative arthritis and on medication which is helping is a good news.As you mentioned about your symptoms of muscular weakness and weakness, this are a part of the RA symptoms.What you need to come out of this symptoms is indulging yourself under a physical therapist who will help you learn how to make the muscle stronger and balance your functional activities without losing more energy.Since you have a pathology, the body mechanism will be running out of energy to support the needs of the cellular mechanism.Undergoing physical therapy session will help your muscles to become stronger, improve the metabolism, improve the fatigue levels to lower down, and overall body conditioning .After all, exercise is the natural way of healing the body as post exercise the body produces its own medicine to heal the body.I recommend you to learn the exercises from a physical therapist.Important note :RA should not travel towards the joints of the ribs. As it is painful and may lead to trouble. For which i will highly recommend to perform regular breathing exercises.With the grace of God i wish you a Good health.RegardsJay Indravadan Patel"
},
{
"id": 98463,
"tgt": "How long does it take to recover from an allergic reaction to gold?",
"src": "Patient: How long does it take to recover from a severe chronic reaction to Gold. I am only allergic to Gold. I have wore it all my life. I had an allergy test done and Gold is all I was positive for. I know the reaction went on at least two years before I could get answers I have had reaction pretty much on all parts of my body. I am currently going through healing process I do know when the reaction of the salt grains come out it itchs and then burns like you are on fire. Doctor: hi sir/madam,Thanks for your question on Healthcare Magic.Treat your reaction by removing your earrings and washing the area with plenty of water to remove any remaining irritants. You can also apply an over-the-counter topical cortisone cream such as hydrocortisone cream. If your symptoms do not improve in five to seven days, you may need to speak with a doctor.hope this was helpful.have a healthy day."
},
{
"id": 124216,
"tgt": "Having noticed sharp pain & cracks on legs while doing splits",
"src": "Patient: I was doing the splits on top of my friends couches that are across from eachother about 4 feet apart and my right leg cracked 3 times right below my right buttcheek. Sharp pain and numbness now going down my right leg. Did I pull something or could it be serious? Doctor: Hello, What I understand from the history that this could be related to a muscle pull or nerve pull. You need to get it checked with the orthopedist and take some x-ray if you have damaged the hip joint. Having taken an extra precaution is better rather leaving the injury just idle. Hope you will take this matter to the orthopedist at the earliest for the better outcome of the treatment. Hope I have answered your query. Let me know if I can assist you further. Take care Regards, Jay Indravadan Patel, Physical Therapist or Physiotherapist"
},
{
"id": 9760,
"tgt": "Suggest ways of enhancing hair growth",
"src": "Patient: Hi I am Carlos. I suffered from a sculp ringworm when i was young, and that caused my hair 2 disappear. After I recieved a treatment I had my hair grown BUT far cry from my beautiful fine soft hair to un ugly strong roots. how i can get my hair back?? I reallly need help!! I always shave my head incase my old hair to come back. I am even afriad of dating! my life is falling apart. Doctor: Hi Dear,Understanding your concern. As per your query you have alopecia problem which is mainly due to fungal infection , covering head with cap and hormonal changes. Stress is an additional factor.Need not to worry. I would suggest you to consult dermatologist once for proper examination . Doctor may order skin patch test , blood test and physical examination . Doctor may prescribe you drug like finpecia at least for 3-4 months or anti fungal treatment along with minoxidil spray . Doctor may also prescribe supplements like Vitamin A ,Omega-3 and Vitamin E . Doctor may also recommend hair transplant with HUE technique . Take Indian gooseberry powder ( half spoon ) with water once a day and use almond oil to massage your hairs.Hope your concern has been resolved.Best Wishes,Dr. Harry Maheshwari"
},
{
"id": 210928,
"tgt": "What do you suggest for depression?",
"src": "Patient: My mother is depressed with her prrsonal probs with my father... as my father is having an affair....she is very unstable w.....doc asked her to tKe shicalm h. ...she was takin it for lSt 8 months but now refuses to take....n symptoms r already vlear dT she is angry Doctor: Hi, Apart from medications, Your mother needs a psychotherapy/counselling to have a control on her emotions./ to have interest in life by adopting few hobby activities/ have meditation session for getting balanced & positive attitude. Thanks."
},
{
"id": 105322,
"tgt": "Taking cetrizine, avil and allegra for allergy. Need permanent cure",
"src": "Patient: hi i am having a problem of allergy ( hives ) is there any medicine for this, if this can u tell me the name of this medicine bcoz i am pitched off from this problem please help me i have taken cetrizine, avil, allegra bt there is no result since childhood i dont hav a problem bt after marriage i am facing this problem last 3 years please help me Doctor: TAKING YOUR MEDOCINES IS NO ANSWER IN PREGNANCY BECAUSE OF USE OF CALCIUM,IRON AND VITAMINS ,ALLERGY STARTS I TELL YOU SOME PRECAUTIONS DONT USE MUSTARD,COCONUT,ALMONDS,SOYABEAN OIL USE AMLA,OLIVE FOR APPLYING SUNFLOWER,OLIVE FOR COOKING SOAP USED IS NIKKO NIZRAL SHAMPOO AND NUT FREE CREAMS FOR BODY USE TAB METHOTREXTATE 2.5 MGM ONCE WEEKLY DOSE(LATEST TREATMENT) TAB ATARAX 10 MGM AT NIGHT TAB EBASTINE 20 MGM DAILY IF RESULTS ARE THERE YOU CAN POST AGAIN OTHERWISE YOU MAY GO FOR ALLERGY TESTS AND SUBLINGUAL LOW DOSE IMMUNOTHERAPY FOR PERMANENT CURE"
},
{
"id": 4192,
"tgt": "What are the chances of pregnancy after having unprotected sex during fertile period and taking plan B after 23 hours?",
"src": "Patient: Hi. I had unprotected sex with my boyfriend last january 29. And then, after having sex, he told me he cummed inside of me and then I went home and saw a blood spot on my under wear with a white color liquid. And then after 23 hours, I took a plan b...and then I also found out that last january 29 I was fertile. what are the chances of me getting pregnant? Doctor: Hi, You had unprotected sex during fertile period and within 24 hr took emergency Progestin only contraceptive pill so the chances of pregnancy becomes least. So don't worry just wait till one weeklater to the expected date of menstruation .Your normal menstruation ll come(then no chance of pregnancy ) if it doesn't then do a Urine pregnancy test to confirm status of pregnancy .Hope this ll help THANKS N REGARDS"
},
{
"id": 115005,
"tgt": "What causes reduction in level of eEFGR?",
"src": "Patient: My creatine level in February 2013 was o.53 with eEGFR Rate of 99. All other baseline tests were normal except for microscopic hematuria. I am 68 now with pseudomonas bacterial UTI that hasn t gone away even with ciprofloxasin. June 2014 creatine level was 0.9. Using same formula as used in 2013, the eEGFR rate is now 66!!! Same lab says Less than 60 indicates chronic renal disease. I know of NO risk factors (drugs, lifestyle, toxins, hospital care) that would make this happen. I look and feel very good.But the UTI isn t going away. Just saw my doctor, cystoscopy scheduled. He was not concerned.Any reason eEFGR could go down so fast??? (I am doing heavy contructioin work in my house.) Doctor: Hi, dearI have gone through your question. I can understand your concern. You should not worry about that. You may have urinary tract infection. Repeated urinary tract infection may be due to diabetes. You should check your blood sugar level. Heavy exercise can also cause this. So don't worry about that. Repeat your creatinine level and eGFR level after a month. Hope I have answered your question, if you have doubt then I will be happy to answer. Thanks for using health care magic. Wish you a very good health."
},
{
"id": 55829,
"tgt": "When will bilirubin level come to normal in case of jaundice?",
"src": "Patient: Hi. I m 36 yrs male, suffering with jaundice. On 2.01.2014 bilirubin total count was 6.32mg/dl. And on 13.01.2014 bilirubin total count was 11.78mg/dl. I m having yellow in eyes & urine. I m having complete rest & good diet (normal home cooked food). When will the bilirubin level come to normal? How much time Will it take for normal color of eyes & urine? Doctor: Thank you for choosing HCMI understand the anguish that you must be facing. You seem to have jaundice which is the yellowish discoloration of skin. This will cause the yellow colour of the skin.To get to the cause you first need to understand which type of Jaundice you have. You need to see a Doctor and get a lab test which will distinguish conjugated bilirubin from unconjugated type. Based on this, you will be able to idenitfy the cause and then get the appropriate treatment. Blood disorders can take a longer time to treat than Hepatitis. Both will present with similar symptoms, however. Please follow up after you have the results. All the best.Hope this helps you.Dr Vignan"
},
{
"id": 86015,
"tgt": "Could frequent stomach pain and vomiting in the morning be a sign of appendicitis or issues with the gallbladder?",
"src": "Patient: My son is 16, he has had on and off stomach pain for the last year. He also throws up in the morning and once he does he feels better. Today he says he has middle to right side pain and is very tired. Our general practitioner has proscribed him antibiotics. And his pediatrician in the past just told us to take antacids. Could this be his appendix or gallbladder? Look forward to hearing back from you. Thank you, JT Doctor: Hello and Welcome to \u2018Ask A Doctor\u2019 service. I have reviewed your query and here is my advice. As per my clinical experience, your son can be having different possible conditions which can present as stomach pain and vomiting as - gastritis - indigestion - appendix inflammation - mesenteric Lymphadenitis - recurrent intestine infection - gall bladder, biliary drainage system derangement - others To find out the underlying cause, I recommend basic lab tests with CT scan of abdomen & pelvis during the period of acute pain episode. Hope I have answered your query. Let me know if I can assist you further. Regards, Dr. Bhagyesh V. Patel"
},
{
"id": 40407,
"tgt": "Can anal itching indicate worm infection, also have prostatitis?",
"src": "Patient: hello, i have had prostatitis accompanied by anal itching for several years. I have taken cipro and bactrim but neither have cured the condition. I have no known food sensitivities. is it possible that I have a pinworm infection as evidenced by the anal itching? Doctor: Hello,Welcome t HCMAnal itching may be due to various reasons like spicy foods, hot sauces, and peppers.It may also be due to yeast infection, pinworm infection,hemorrhoids,tears of the anal skin (fissures), and skin tags (abnormal local growth of anal skin). Prostatitis doesn't cause anal itching.Also you don't have food sensitivities,So, most probably you are suffering from pinworm infection.I would suggest you to take antiparasitic medications like mebendazole, pyrantel pamoate after consulting your treating doctor.Thank you."
},
{
"id": 20284,
"tgt": "What causes high heart rate with sinus trachea?",
"src": "Patient: Hello I have a question I took a physical for the job I going for The ekg was bad sinus trycarca I was drinking 4 cups of coofee about 2 days before the exam The doctor there took my heart rate pulse again it ddidin't go down She stated to go to my doctor qith a note stating that I have a high heart rate. So far I haven't had any coffee since then Doctor: Your heart rate is usually a sign of something else. It goes up if you are worried, excited or scared. It may also change when you are sick. Rarely, the normal leader of the heart has lost control and the rate signals that. Coffee changes heart rate very little and for a very short time. If the doctor observed something abnormal, its best to have your doctor follow up to see if looking further is worthwhile."
},
{
"id": 141881,
"tgt": "Suggest treatment for TIA",
"src": "Patient: II was recently put on a higher dose of atervastain(20) due a TIA.Then a couple months later Forteo because of severe ostoporosis. I have been having extreme leg pain,much worse at night. I also had kyoplastic on my back, due to vertabra fractures. Could the atervastain possible be a cause? Doctor: Hello!My name is Dr. Aida and I am glad to attend you on Healthcaremagic!I agree with you that atorvastatin may lead to increased muscle cramps, as this is a common adverse effect of atorvastatin. For this reason, I would recommend checking liver function tests and muscular enzyme levels. Considering the fact that you suffer from osteoporosis, I would recommend checking also calcium and vitamin D plasma levels. It may be necessary stopping atorvastatin for a short period of time, especially if your liver or muscle enzymes are elevated. Hope you will find this answer helpful!Best regards!"
},
{
"id": 25653,
"tgt": "What causes heart palpitations with a pulse rate 51?",
"src": "Patient: husband h1s 58 has chronic hep c for 20 years never did treatment. Drinks two cocktails daily,wine or beer. His is getting heart palpatations at night and his tongue looks purple? His pulse tonight was 51 bp 155/95 he will not see a dr. Are any of these things to worry about Doctor: Thanks for your question on Health Care Magic. I can understand your concern. Yes, there are many things in your husband's case times worry about. He is taking alcohol. So possibility of alcohol induced heart damage is more. His blood pressure is high, pulse rate is low, so possibility of heart block is more. He is also having purple tongue at night. This might be due to cyanosis. Cyanosis is due to reduced blood supply to the organ. And it is commonly seen in heart block. So all these are danger signs in him and these need evaluation by cardiologist. So consult cardiologist and get done ecg, 2d echo and Holter monitoring (24 hour continuous monitoring of ecg). Ask him to quit alcohol as soon as possible. Hope I have solved your query. I will be happy to help you further. Wishing good health to your husband. Thanks."
},
{
"id": 13490,
"tgt": "What causes red, itchy and circular rashes in the feet, lower back, and arms?",
"src": "Patient: I have circular rash on top of both feet, central lower back, back of arms, outside of legs and thumb joints. Low numbers on blood tests for Lupus and hodgekins lynphoma has come up at doctors offices. no one seems to know what it is. Rash is red, itchy and scaley. Sometimes painful. Even cool showers make them flare up. I ve had it for 1 1/2 years. Doctor: Hi, The rash could most probably a fungal infection (Tinea infection). But there are many other differential diagnoses such as urticaria, contact dermatitis etc. If you are too busy to consult your doctor, you could take an antihistamine tablet and apply antifungal cream. If you do not get any improvement or if your rash worsens, you should consult your Dermatologist for further management. Hope I have answered your query. Let me know if I can assist you further. Regards, Dr. Siva Subramanian, Dermatologist"
},
{
"id": 27513,
"tgt": "How to treat palpitating heart?",
"src": "Patient: have woken up around 5am a few times with my heart racing like I have been running/aerobics in my sleep. I do not remember dreaming of running. My breath is not labored. when I notice my heart is racing it comes back down, quickly just as it would while cooling down from an Aerobic workout. 30 years teaching fitness, aerobics, and yoga. Doctor: Hi. Thank you for your question. Heart racing during sleep is a known phenomenon. It happens during the REM (rapid eye movements) phase of sleep, a phase characterized by a flush of catecholamines (e.g. adrenalin) in the system, which causes heart rate to \"race\". I am assuming it is a regular beat. My opinion is that no further investigation is warranted. I hope I was helpful. Greetings."
},
{
"id": 159777,
"tgt": "Overweight,lump under armpit,no family history of cancer",
"src": "Patient: Hey.. I m a girl aged 17, 85kgs (:( overweight ), approximately 165-175cm tall, no previous breast cancer or other diseases in my immediate family (that i know of) and i ve found a lump underneath my armpit.. It s not sore most of the time, only occassionally and sometimes if i use roll-on deoderant.. I don t know what it is or what it could mean.. Could you help?? Thanks... Doctor: hi... please try to feel the lump urself and assess whether it is hard, non mobile, any change in color size and the surrounding skin of the lump... the best investigation is Fine needle aspiration cytology(FNAC) .. it ll give a clear cut reason behind the lump... hope u get well soon"
},
{
"id": 164615,
"tgt": "Suggest treatment for optic nuerities in a child",
"src": "Patient: Comments = My son 7 \u00c2\u00bd yr suffered Optic Nuerities.He was prescribed in first time on 1.6.2009 injection Metypred/IVpred 250mg & Tab Omna Cortil by which he recovered from such disorder.Again after 8 month same disease has recovered & same treatment was given. Again after 12 months same disease has recovered the same treatment was given. Pl.suggest what precautions/treatment should be given in future. Kindly please inform me as to why this disorder is recurring. , Address = 15, Wright Town, Ganjipura, zip Code = 482002, City = Jabalpur, State = Madhya Pradesh, Country = India, Phone = 0000, E-Mail = YYYY@YYYY . At present he is complaining Headache. It is worthwhile to bring to your notice that twice Brain MRI and other medical tests and checkups as suggested by neurologist and ophthalmologist have been done for the aforesaid purpose. However no defect was found and the reports were normal. Doctor: Hello and welcome to healthcare magicRecurring optic neuritis in a child can be part of two demyelimating condition (which means nerve sheath destruction.The sheath called myelin protects and feeds the nerve).One is multiple sclerosis (MS) and other ADEM-on (acute dissiminated encephalomyelitis-optic neuritis).But MRI has to be abnormal for diagnosis of these conditions.So probably your child is suffering from a condition Recurrent Idiopathic Optic Neuritis as MRI is normal and no other system is involved.The recurrence depends upon previous dose of steroids.High enough dose of IV steroids prevents recurrence.The number one precaution is vigillent watching If other systems like sensation movement hearing and consious level is affected take him immediately to neurophysician to rule out Ms and ADEM-ON.If future attack occurs talk to your neurophysician about high dose IV steroids.Moreover gastrointestinal infections and chest infection can also trigger Recurrent Idiopathic Optic neuritis.So precautions can be practiced to avoid such infections like use of boiled water hand sanitation before eating and preparing meals after using toilet.Yearly influenza vaccination.Keeping Nose and throat moistened during cold and dry weather.Hope you find this answer helpful.Good luck!"
},
{
"id": 172750,
"tgt": "What causes headache in 10 years old as the tests were normal?",
"src": "Patient: My Daughter aged 10 years and 6 months is suffering from headache for last 1 month. Consulted eye, ENT, neuropsychiatrist, neuro surgeon and 3 paediatricians. Did CT of sinuses and brain, all blood investigations, MRI, MRA and MRV of brain and everything is normal. Taken a lot of analgesics, but still she complains of headache. SO want a opinion on her headache. Doctor: Hi dear,I understand your concern. With such presentation in my clinic, I advice Nootropil syrup or tablets for 1 month.Consult neurologist to rule out migraine ,high intracranial pressure and other possibilities and check BP.When does she have headache? If she has after school, then it is due noice in the school, overload of information, tiredness, heat wave,increasing of uric acid due lack of consumption of water. During episodes of headache you can give 1 tablet of Diclofenac and 1 tablet of Domperidon.Hope it helps"
},
{
"id": 65467,
"tgt": "What causes sore lump on the calf muscle?",
"src": "Patient: I have a lump at the top of my calf muscle in one leg. It is sore like a bruise. I've been working out but do not recall any pain at any point indicating I pulled or strained it. It's not floating like a nodule and is about the size of a large shooter marble or small golf ball. Doctor: Hi, dearI have gone through your question. I can understand your concern. You may have some soft tissue tumor like lipoma, neurofibroma or dermatofibroma. You should go for biopsy. It will give you exact diagnosis. Then you should take treatment accordingly. Hope I have answered your question, if you have doubt then I will be happy to answer. Thanks for using health care magic. Wish you a very good health."
},
{
"id": 27320,
"tgt": "How to treat breathing difficulty?",
"src": "Patient: i have been working out for years, but in the last few months i noticed i am losing my breath really quickly i thought it might be that im not breathing right but i have been watching that while i lift really close. i do a low carb diet and i do drink caffine, and i have been suffering with alergies Doctor: Allergic bronchitis may be the reason in your case for getting breathless. Get chest xray and pulmonary function test done, it will help to diagnose lung problem. Stay away from allergens, if severe you may take anti allergen medicine with doctors advice. Get cardiac check up too."
},
{
"id": 87426,
"tgt": "Suggest treatment for abdominal pain",
"src": "Patient: Hi... I have a pain in my vagina when I pee like cystitus... also there is blood in it... I often have abdominal pain.. I have been to the doctors over 10 times in the past 3 month... I feel they thob me off... also I have been to the gu and my result were ok... can you help me please ?? Doctor: Hi.Thanks for your query and an elucidate history.If you have visited Doctors for more than 10 times, this indicates that you have problems. The possible causes of the pain in the vagina which passing urine and having blood too can be :Cystitis Stone in the urinary system or some obstructive uropathy. I would advise you the following:::Ultrasonography of the whole abdomen including trans-vaginal route.CystoscopyIntravenous pyelographyFull Urological evaluation by the Urologist in assistance with a Gynecologist so that the root-cause can be found and treated accordingly."
},
{
"id": 148617,
"tgt": "Suffering with MND, no improvement with medicine, now have fully paralyzed right hand, affected throat. Help?",
"src": "Patient: He is suffering with MND from last 2 and Half year and now his age is 72 Yrs. 5 Month back, he was having only issue with throat(Speaking problem ) and left hand palm was not working proper but when we took all reports and doctor found MND then we start medicine of MND ( Riluzole ) but after that he is having more problem. Now his right hand is fully paralysis and throat is more affected and left hand is as it is. Doctor: Hi,Thank you for posting your query.It is unfortunate that your relative has MND- motor neurone disease. At present, there is no cure for MND. Riluzole does not stop the disease progression, it may only help little bit in improving the muscle strength. At present, please start Ryle's tube (nose-to-stomach food pipe) feeds, to prevent aspiration pneumonia.Regular physiotherapy and exercises should also be done.I hope my answer helps. Please get back if you require any additional information.Wishing you good health,Dr Sudhir Kumar MD (Internal Medicine), DM (Neurology)Senior Consultant NeurologistApollo Hospitals, Hyderabad, IndiaClick on this link to ask me a DIRECT QUERY: http://bit.ly/Dr-Sudhir-kumarMy BLOG: http://bestneurodoctor.blogspot.in"
},
{
"id": 139640,
"tgt": "How to cure silent migraine,numbness,dizziness and nausea?",
"src": "Patient: I have been diagnosed with silent migraines which i seem to get on a daily basis,numbness inpart of my face, dizziness, and nausea and my vision is not as good since i first had my first episode of vision disturbance when it started 5 weeks ago. Today my left hand felt weak and shaky for about 5 mins and was just woindering if this is still part of the migraine Doctor: Hi, First of all, there are many causes to your symptoms other than migraine, so you should measure your blood pressure and if it was high, you should receive a medical treatment, take aspirin 160 mg twice per day after meals, decrease salt and fat intake, do lipid profile in the blood and do an ECHO and an ECG on the heart. Second, the hormonal therapy for women such as oral contraceptive pills and replacement therapy can cause similar symptoms, so the woman should ask a gynecologist for help. Third, you should take vitamin B complex once per day after meal and Benfotiamine 40 mg twice per day after meals and you should ask an ophthalmologist for help. Hope I have answered your query. Let me know if I can assist you further. Regards, Dr. Mustafa, Neurologist"
},
{
"id": 83691,
"tgt": "Suggest alternative for Coravdil A",
"src": "Patient: i am 53 yrs old lady, taking coravdil A since last 8 yrs. my weight is 82 kgs n my height is 5 ft.my sugar level is within the normal limits. My problem is that my both legs are swollen since last 5 yrs, is this the side effect of corvadil A ? if yes is there any alternate medication ? thank u Doctor: Hi,Corvadil-A consists of two medicines a calcium channel blocker (amlodipine) and a heart-selective beta receptor blocker (atenolol) which is commonly prescribed to treat high blood pressure (BP).Swollen legs appears to be the side effect of amlodipine. Other alternate medicine that can be combined with atenolol is hydrochlorothiazide which is equally effective in controlling the high BP.Hope I have answered your question. Let me know if I can assist you further. Regards, Dr. Mohammed Taher Ali, General & Family Physician"
},
{
"id": 198161,
"tgt": "Suggest treatment for excess shrinking of penis and scrotum",
"src": "Patient: Basically for a while now I ve had a problem with my penis and scrotum. When ever I get the slightest bit nervous my scrotum shrivels up inside my body and my penis gets incredibly small. Very uncomfortable. This also happens in cold weather. I understand they shrink but this is worse than normal. My gential are very cold all the time. Freezing. I also have a problem with shrinkage when I go to the toilet Doctor: DearWe understand your concernsI went through your details. I am unable to find your age from your question. Assuming that you are a healthy person in his young adulthood or middle age, I answer this question.Penis and the scrotum do shrink when people are nervous, anxious, during cold weather etc and that is quite normal. You say that it is beyond normal. That is where you have to understand the normal shrinkage. As age advance, the penis actually shrinks. But that proportion is very minute.Shrinkage happens when the body pulls out blood from that part because some other part of your body needs more blood. Therefore the channelization. this is a natural procedure and you cannot control it. My suggestion would be you don't have to worry unless you have any sexual erection or ejaculation problems. If you worry about this too much, obsession and anxiety shall result and which in turn can be reason for ED or PE.If you require more of my help in this aspect, please use this URL. http://goo.gl/aYW2pR. Make sure that you include every minute detail possible. Hope this answers your query. Further clarifications are welcome.Good luck. Take care."
},
{
"id": 132550,
"tgt": "What causes the bone sticks out from under the stomach?",
"src": "Patient: Hi, I m a 15 year old male. There s a bone that sticks out from just under my stomach, and it s been there for as long as I remember. It sticks out a lot when I m in the deep end of a swimming pool or when I m doing sit ups. Is there a way to stop this? Doctor: Hello. This sounds like an umbilical hernia and you need to have a doctor look at it since it may need surgical repair"
},
{
"id": 170904,
"tgt": "Suggest treatment for scar on nose after a fall in child",
"src": "Patient: Yesterday, my 14-month granddaughter fell on her sidewalk and got a bad sidewalk burn on her nose and a small on her forehead.Her nose is swollen some.Her Mom is putting children s neosporin on it and I told her to be sure and 100% sunblock to keep from tanning.Are these correct and what would you recommend to speed healing and to prevent scarring?My son and wife are so worried about scarring.It s really burned badly on her nose.Thanks, Janet Steele Doctor: Hi....USUALLY THE BLOOD SUPPLY TO THE FACIAL REGION IS VERY RICH AND THE HEALING WITHOUT MUCH SCARRING CAN BE EXPECTED.But....Skin conditions are best diagnosed only after seeing directly. I suggest you to upload photographs of the same on this website, so that I can guide you scientifically.Hope my answer was helpful for you. I am happy to help any time. Further clarifications and consultations on Health care magic are welcome. If you do not have any clarifications, you can close the discussion and rate the answer. Wish your kid good health.Dr. Sumanth MBBS., DCH., DNB (Paed).,"
},
{
"id": 155995,
"tgt": "What is the red spot on the right side of my right breast?",
"src": "Patient: I noticed a little red spot on the right side of my right breast. I pushed on the red spot and it was like a whole with skin covering it up. It does not hurt. I had a Mamogram this past Tuesday, just as part of my yearly exam. I am 50 years old. Good health. Is this a sign of cancer? I am concerned Doctor: Welcome to Healthcare Magic.. The lesion you described seems to be something in the skin.. It is unlikely to be breast cancer especially if your mammogram was normal.... However given your age it is better to show it to your surgeon and get his opinion... Hope this answered your query.."
},
{
"id": 16541,
"tgt": "Suggest treatment for high BP and palpitation after treated pneumonia",
"src": "Patient: Hi dr samuel, firstly thankyou for your time, iam a male my name is ben 42 years of age 6 months ,ago i caught pneumonia, which in short gave me gave me heart arrythmia, take anitbiotics, i.v. which fixed the pneaumonia , but then had cardioversion , heart back into normal rythym now, never had high blood pressure , but doctor put me on beta blocker and bisosprol and coversyl, iam experiencing side effects from , the tablets anxiety , impotence, and just not feeling myself, iam wondering why i need them if my heart is in normal rythm and dont have blood pressure , he said just as a preventive , to keep your heart healthy, so it never happens again , i was very active and it has just slowed me right down , lost alot of drive and enthusiasam , is there any natural alternatives , should i seek a 2nd oppinion , he said we dont have any journals or references we can look up to c how people go off them , so just keep taking them , Doctor: Hi, Arrhythmia especially Atrial Fibrillation (irregular heartbeat) is pretty common, especially during Pneumonia. The episode is generally paroxysmal i.e. it occurs during that time only and mostly doesn't recur back. If it is treated at that time we generally do not give any medication but if there are certain risk factors like dilated heart chamber (Left Atrium) or heart dysfunction we continue it for prophylaxis. You can stop the medication if you don't have risk factors or the arrhythmia has never recurred again. If it has and you are feeling side effects of beta-blockers than for such patients we can start on another type of anti-arrhythmic called calcium channel blockers (Tab Diltiazem). For BP you have to check your BP daily and if it is normal on medication than you definitely need them if the BP is lower than 110/70 you can try tapering the dose to half tablet and gradually stop it. Hope I have answered your query. Let me know if I can assist you further. Take care Regards, Dr Sameer Maheshwari, Cardiologist"
},
{
"id": 70123,
"tgt": "What is the cause of lumps on the butt region?",
"src": "Patient: rash I have a bunch of red bumbs on my buttcheeks and there s a cluster of big bumps.on the lower left buttcheek and that part hurts to touch. I can t see them but my boyfriend looked at them and he says some of them.have pus it them what could this be? Doctor: HI. This may be a condition called Pyoderma, meaning pus filled swellings in this area. Apply liquid povidone iodine locally to make the skin sterile and you may need a prescription based antibiotic to be written by a Doctor. He will examine to decide if this is pyoderma or infected scabies."
},
{
"id": 99792,
"tgt": "What causes tightness to breath, cough and neck pain?",
"src": "Patient: hello, i was diagnosed with hypothyroid and very high level of allergy last september 2010. since then im an medication including thyroup, and telekast-L. the problem is that if i dont take telekast-L even for two days consecutively, there is tightness in breathing,persistant cough for about 1/2 hour with thick white discharge, some sweating, some pain in neck. it also leaves me with some pain in the lower ribcage area. what can be the real cause of the breathing difficulty? allergy or the thyroid? what should i be doing to correctly address this problem? need ur advice urgently. thanks in advance.T.R.O Doctor: Hi, u have hypothyroidism and u are taking treatment for that , its okay.but for which treatment u started taking telekast- L for so long u have not mentioned in spit of this u are saying withdrawal symptoms of that drug.may be its due to chronic bronchitis, allergic asthma but should be evaluated by pulmonologist.go to the doctor meanwhile u can take that."
},
{
"id": 20626,
"tgt": "Can Coumadin increase risk of heart attack?",
"src": "Patient: So is it true that if i'm taking coumadin then i am at risk of a heart attack or possible blood clots. You see i was diagnosd with congestive heart failure as well as kidney failure. Somehow I didn't die. Also I have poor circulation in both of my legs, again will the hcg be helpfull in my weight loss rejamond. Doctor: Hello, Coumadin is given if there is already a blood clot or you have high probability of blood clot formation due to slow circulation, poor heart function or change in heart rhythm. As for your answer. NO by no means does your risk for heart attack increases with Coumadin, rather there will be benefit in preventing future heart related issues in your case. The only increased incidence is of bleeding. But than also the risk benefit ratio is towards benefit. Regrads Dr. Priyank Mody"
},
{
"id": 167260,
"tgt": "Suggest remedies for high histamine level and recurrent hive breakout",
"src": "Patient: Hi, I was just told by my daughter s doctor, she is 10 yrs old, that she has high histamine levels and swollen abdomen. She breaks out in hives from head to toe, swollen joints and slight fever for 10 days on a pretty regular basis. I asked what to do and he said nothing it should balance out in 6mths to a year. But I want to know if I should change her diet or add more vitamins. Please advise. Doctor: its an allergic disorder. release of histamine from mast cells causes manifestation of allergy. there are certain safe drugs to stablize mast cells and to prevent effect of released histamine. such as monteleukast. u can ask yr pediatrician for this option"
},
{
"id": 196090,
"tgt": "What could cause penile bruising?",
"src": "Patient: hallow doctor,how ar u?i have a problem with my health,when i sex then my penis get bruise.this start just two years now and i went to hospital but what they told me is prepare someone whom i sex with but that is my ususally business before having sex,but still the problem exist.what wrong with my penis skin is it so soft or what actually i fail to enjoy my sex.i had reach a time i run way of my wife when i discover tht she want me fuck her .help pls Doctor: Hello Thank you for trusting HCM Many causes for bruising of the penis some of them mentioned below. 1.Entrapment in clothing- appear as a simple cause and many of times neglected. 2.Thrombosed lymphatic vessel - Lymphatic vessels can be found all over the body - they are responsible for delivering lymphatic fluid. Masturbation, sex, and the use of erectile enhancement devices such as vacuum pumps can lead to bruising of this tissue.3.Trauma to the penis- Any type of trauma, or sudden injury, to the penis may cause bruising, which is characterized by discoloration of the skin, soreness, and in some cases, inflammation.4.Fractures- Unfortunately, the penis really can be fractured; this occurs when the erectile tissue in the penis is ruptured, usually during sexual activity where the partner is on top. 5.Injection of drugs- Injecting drugs into the penis - whether for treating erectile issues or for other reasons - can result in bruising and soreness around the injection site.Vigorous Masturbation, priarpism etc... Please consult your doctor (sexologist he will guide you in right way)."
},
{
"id": 26223,
"tgt": "What could cause severe night sweats?",
"src": "Patient: Every single night my fiance has been having severe night sweats. This has been going on for quite some time. I have been with him for over 2 and a half years and it's been getting worse. Around 2 to 3 am I wake us up, completely drenched in sweat. Every night we have to take our bed sheets off, take a shower (again), and get cleaned up. The entire bed is soaked and we feel like we just got out of a swimming pool. I talked him into going to his doctor last month to see what the problem is, but she said there is nothing wrong. The doctor did several blood tests and everything came back as normal. He is on a very low prescription of antidepressants and has been for several years, but nothing has been changed lately. Other than that, he is very active, healthy, perfect weight, etc. He never gets sick and seems perfectly normal otherwise.Not only is this an annoyance to have to wash bed sheets everyday and get cleaned up at 3 in the morning, I am really beginning to get scared something is wrong with him! :( I'm not sure what it could be or what steps to take next. I am even having a hard time of sleeping at night because I am constantly waking up or checking him to see if he is getting close to having a night sweat. HELP!!!! I am desperate for answers....it's 3 am and it happened again for the 6th time this week! We are currently down stairs in the couch and recliner because our bed is soaked. I usually have two sets of plastic protectors on, but I can't keep up with the laundry due to the frequency of these night sweats. Doctor: Hello!Welcome and thank you for asking on HCM!I understand your concern and would explain that these symptoms may be related to many disorders (inflammatory disease, metabolic disorders, infection diseases, etc.). What are the blood tests that he has performed? Have you measured his body temperature, blood pressure and heart rate during the nights? I recommend consulting with the GP for a careful physical exam, a resting ECG, a chest X ray study, an abdominal ultrasound and some lab blood tests : -complete blood count-PCR, sedimentation rate-blood electrolytes-kidney and liver function tests -thyroid hormone levels (thyroid gland function)-cortisol plasma levels (adrenal gland function)- immunological tests (ANA, ANCA, RF, ENA, etc)- infectious disease tests (HIV, HBV-Ac, Lyme tests, Wright tests, gama INF for tuberculosis and Mantoux test). Hope to have been helpful!Best wishes!Dr. Iliri"
},
{
"id": 166923,
"tgt": "Suggest remedy for diarrhea and fever in 6 year old",
"src": "Patient: Our six year old grandaughter has slight diarreah yesterday and a fever which spiked to102 at night. She has ot eaten and only drinks sips of water,not often. This started yesterday, can we let it go through tomorrow and take her to her pediatrician on Monday? Doctor: for fever give acetaminophen or paracetamol every 8 hours give oral rehydration solution as five tea spoons for every pound of her weightfor diarrhea give motiliuim or any over the counter antidiarrheal drug"
},
{
"id": 202709,
"tgt": "Problem of erectile dysfunction and difficulty in urinating. Renal cyst found. How to rectify problems?",
"src": "Patient: Hello, i am 36 yrs old married male. i am suffering from ED problem for one year and also suffering difficulty urinating. So, i follow up a local urologist about my problems. He gave some test and found that a right renal cyst in the mid cortical region measuring 34.1 mm * 24.6 mm, urine bladder is normal, PVR is 28 ml, prostate is normal in size with uniform echopattern (T*base-apex * AP= 27.6 mm * 37.1 mm * 30.0 mm). weight of the prostate is 16.9 grams, Volume of the prostate is 16.1 cc.outline is well defined. capsule is intact. No indentation at the bladder base is noted, urine color is straw, apperrance is clear, RBC not found, Pus cell 0-2 / HPF, sugar nil, uroflowmetry, peak flow, 22ml, mean flow 10 ml, voiding time 104 sec, flow time 95 sec, time to peak flow 8 sec, voided volume, a00a ml. After reporting, he prescribe of medicine maxrin 0.4 mg, ocarnix, e-gel 200, forgest 20. mg. so iam taking this medicine for two month. i mean to say you that my difficulty urinating problem has somewhat better than previous condition, but ED problem has not improved. in view of above, is it my ED problem is any serious something and how can i get rid of ED problem completely. please clarify about my contions. Thanks Mr. Haasan Doctor: Renal cyst is likely unrelated. Would recommend medicines like viagra to help with ED Please rate 5 stars! I strive to provide you great answers to your questions."
},
{
"id": 32184,
"tgt": "What causes red colored hole under crease of armpit?",
"src": "Patient: I have a hole just under the crease of my arm pit I have tried putting band aids on it but we re it is and bending all the time makes them fall off . It s been here for about 1 week and it has a really bad sense to it like really bad. I have been cleaning it but now it is getting red around it and starting to look gross Doctor: It could be an abscess, maybe minor or major, depending on the size of the bump. Keep your armpits dry and clean, avoid sweaty places, and consult a surgeon if discomfort persists. You could also use a Magnesium sulphate ointment thrice a day till it develops a pus point and bursts out all the contents, which you could then easily clean with an antiseptic solution like Dettol. Follow hygienic practices and use talcum powder if need be. Avoid anti-Perspirants as they tend to clog pores and stuff infection-causing bacteria inside them producing such infections."
},
{
"id": 51873,
"tgt": "Is wheatgrass juice good for Chronic kidney disease ?",
"src": "Patient: my brother is suffering from CKD stage 3, his creatinine level i think its 56-64% .someone told us to drink wheatgrass juice? Does wheatgrass juice helps to patients of age 34years with CKD stage 3, who is undergoing chemotherapy too? are the side effects of chemotherpy really scarry...! please advice. Doctor: Wheat grass should not cause per se any problem with either chemo or to CKD. . However wheat grass juice is high in potassium and CKD patients are unable to remove this potassium and develop hyperkalemia ( high potassium) which is dangerous. Some chemotherapy medications can be stressful for kidneys should be monitored during and after chemo. The concept of everything natural is safe is so obviously fallacious - as if there are no poisonous plants or fruits in nature. All plants are also chemicals - simplest example is sugar (actually a chemical C6H12O6) extracted from sugarcane (natural)"
},
{
"id": 155249,
"tgt": "Chapped lips can be considered as a side effect of chemotherapy?",
"src": "Patient: I am a cancer patient being treated for rectal cancer. Last summer I went thru radiation concurrent with chemotherapy, both of which ended somewhere between the end of August and the middle of September. In October my rectum was removed and I was fitted with a colostomy. Since about mid-January I have gone back on chemo, three days per week, every other week, with my last treatment coming mid-April. My problem is that recently I have been suffering from what feels like chapped lips on the INSIDE of my lips. I have to say my oral hygiene hasn t been up to par due to my simply feeling crappy from the chemo. So, does the problem inside my lips sound like a result of the chemotherapy or could it be some sort of a periodontal disease caused by my intermittent brushing? \u2014 John in Sacramento Doctor: Chappy lips is a very common side effect of most chemotherapy drugs and is a result of poor oral intake and vitamin deficiencies. Poor oral hygiene adds to this situation. So while there is nothing to worry about, you can get rid of the situation by trying to eat as much as possible (whatever suits your altered taste), and to take a vitamin supplement."
},
{
"id": 210067,
"tgt": "Suggest alternative of mirtazapine for depression",
"src": "Patient: Hi, my name is Camilla, I would like to know of any alternative to mirtazapine, as I am gaining too much weight from it. It has to be something that works as good as mirtazapine in order to get a good night sleep, since my depression symptoms really triggers this. Bst rgds Camilla Doctor: Hi Camilla,Mirtazapine is an effective anti-depressant with sedation and increase in weight as its significant side effects. As an antidepressant, there are many alternatives to mirtazapine. Selective serotonin reuptake inhibitors like escitalopram, sertraline, etc are effective and safe. It causes lesser weight gain than mirtazapine but is not as effective sedative. Bupropion has least association with weight gain. However, is also not sedating.As you have mentioned that your sleep problems are triggered by depression, I think starting these anti-depressants with short term sedatives will solve your problem.Sedating anti-depressants include agomelatine and tricyclic antidepressants like amitryptilline. Tricyclics also cause weight gain and also have much more side effects.I do hope this information was useful. You can discuss these drugs with your treating psychiatrist and select the best suited for you.Best wishes."
},
{
"id": 147996,
"tgt": "Is it safe to take Celexa and Tramadol while suffering from seizures and GERD?",
"src": "Patient: Hi, I am a 64 yo woman with a history of gastroparesis, GERD, and degenerative L4-L5 disc with back pain and bilateral weakness in thighs and tingling and numbness in feet. Also have sleep apnea. A fantastic GI Doc put me on Celexa at a very low dose - 15-20 mg/d - to increase GI motility for the gastroparesis, and it has worked like a charm. An Ortho Doc put me on Tramadol at a very low dose - 50 mg/d - for the back pain (also getting steroid injections for the back pain). I ve only needed 25 mg of the Tramadol to help with the pain, and it has helped me sleep and even helps with GERD pain. Have been on both for about two months with no problem. I needed my Celexa refilled, and the nurse practitioner has refused on the basis of the interaction between the two drugs and concerns of seizures. I have no history of sz disorder, and the Ortho Doc has no problem with the combo. Do you feel this combination at these doses is of concern; and, if not, how can I work around the script problem. The nurse practiioner has in the past been a fantastic assist with the GI practice, but I feel in this instance that she is being a bit over zealous. This combo is the only thing that has made me feel like myself in a long time. Doctor: Hi,Thank you for posting your query.Though I agree with the concern that a person who takes tramadol either alone or in combination with Celexa, has a higher chance of getting seizures.However, the risk is higher in a person with history of seizures, or with injections of tramadol, or with higher doses of tramadol (typically more than 100 mg dose). So, in your case, I feel you have a very low risk of getting seizures, if at all, and I would have no problem in prescribing you the combo, especially as, these two drugs make you feel so comfortable.I hope my answer helps. Please get back if you have any follow up queries or if you require any additional information.Wishing you good health,Dr Sudhir Kumar MD (Internal Medicine), DM (Neurology)Senior Consultant NeurologistApollo Hospitals, Hyderabad, IndiaClick on this link to ask me a DIRECT QUERY: http://bit.ly/Dr-Sudhir-kumarMy BLOG: http://bestneurodoctor.blogspot.in"
},
{
"id": 145985,
"tgt": "Could brain aneurysm be possibly causing major bilateral temple pain?",
"src": "Patient: Hi, I had a subarachnoid bleed and coiling of brain aneurysm last year. I am having some major bilateral temple pain along with numbness that comes and goes in my left arm/hand/fingers. I have had headaches since the aneurysm, but this is different. Also, my moods have been horrible for the past few weeks. Doctor: aneurysm in brain causes headache due to compression of pain sensitive structure in brain. but pain is usually unilateral not bilateral. this pain will not radiate in both upper limb. bilateral upper limb pain may be due to cervical spondylosis or other causes. you have anxiety of aneurysm, so due to anxiety your mood is horrible because aneurysm does not cause horrible mood change. so your headache may be chronic tension type headache. you may relieved by antianxiety drug, or amytryptyline or benzodiazeoine"
},
{
"id": 50088,
"tgt": "Pain around ovaries, ultrasound clear. History of repaired bladder mush. Recommended tests?",
"src": "Patient: Hi, I m having pain on my right side about where the right ovaries is. I had a bladder mush put in 2008 that fall out and was repaired in 2009. Nov 18, 2012 that part of my body started giving my problems again. I have had two ultra sounds and the bladder and kidney are clear. I m still having pain and I m going to a Gyn doctor today to see what else can be done? What test would you recommend? Doctor: Hi, there may be few reasons for pain on right side. i would suggest you to get done few test like transvaginal sonography ( after gynacologist opinion), ct scan abdomen as well as urologist consultations. That may be able to find out cause. take care."
},
{
"id": 22196,
"tgt": "What causes sharp diffuse chest pain?",
"src": "Patient: my first hour esr was 30 mm and second was 55 mm. i went to the doctor for sharp diffuse chest pain and i am a smoker for 10 years, 20 cigarettes a day. my chest x-ray was normal except for carbon formation as doctor said. is there is anything to worry or further tests are required, please help me with my question. my age is 29. Doctor: DEAR USER,THANKS FOR CONSULTING WITH HCMI UNDERSTAND YOUR CONCERN..DIFFUSE SHARP PAIN IN A SMOKER CAN BE BECAUSE OF SUPERADDED INFECTION IN THE LUNG. THAT CAN BE RULED OUT WITH A CHEST XRAY.U NEED TO ANSWER SOME QUESTIONS SO THAT I CAN HELP YOU...WHAT IS THE EXACT SITE OF PAIN..?WHEN DID IT START?IS IT ASSOCIATED WITH COUGH OR EXPECTORATION?ANY HISTORY OF SWEATING..?ANY SIMILAR EPISODE IN THE PAST?YOU ALSO NEED TO GET A ECG DONE AND GET A CONSULTATION FROM A PHYSICIAN..WAITING FOR YOUR DETAILSHOPE I ANSWERED YOUR QUERY. YOU CAN MESSAGE ME FOR ANY FURTHER CONCERNS"
},
{
"id": 220735,
"tgt": "What causes lack of movement in baby during pregnancy?",
"src": "Patient: Hi my name is Tara, I'm actually pregnant.. a little over 4 months. 17 weeks and 1 day today, a few days ago I started not being able to feel the baby move. But lately, me and my boyfriend have been arguing alot.. I didn't have a miscarriage. I was wondering what else could have happened. Or if I'm just really freaking out??? Doctor: if you take my opinion then i would suggest you not to get worried as this can happen quite commonly ,try drinking some glucose water and take left lateral position and you can feel the movement of baby"
},
{
"id": 202922,
"tgt": "Why am I having bubble feeling in my lower abdomen and lump on my epididymis?",
"src": "Patient: Hello, umm I have a small bubble feeling on my lower right abdomen when I breath deeply and also a heavy feeling, and I think in my right testicle I have found a lump on my epididymis , it is not on the testicle so I know it s not TC. What is it and should I be worried ? Doctor: HIThank for asking to HCMI can understand your problem this could be your ideas because you must have examine your testis, but this may not be the thing as you have noticed to get it confirmed better to go for clinical examination, the bubble feeling is nothing to worry, hope this information helps you have nice day."
},
{
"id": 203090,
"tgt": "What causes thick white liquid discharge from penis after urinating?",
"src": "Patient: I have a white liquid after urinating come out of my penis that is thicker than the urin. I am a young teenager that doesn't want too go too the doctor because I don't want to be embarrassed what is it?? And what causes it??? (I have never had sexual activities) Doctor: HelloThis is the secretions of paraurethral and bulbourethral glands. This is a normal secretion , meant to clear the pathway for passage of sperms and to lubricate the area for normal sex so relax, there is nothing to worry.DR SAATIISH JHUNTRRAA"
},
{
"id": 154262,
"tgt": "Does blood clot indicate cancer?",
"src": "Patient: My friends daughter has a blood clot in her arm and the drs can t dissolve it. She is being sent to a pulmonary specialist in Boston. They are also are checking her for cancer. What happens if they can t dissolve it? Are there any cancers this could be. She is only 30 years old. I am not paying for an answer. I will keep researching. Thank you Doctor: Hi, dearI have gone through your question. I can understand your concern. She may have some clotting diorder or clotting tendency. So there is blood clot. It may be due to cancer or other disorder. So she should search the cause. For blood clot she should take heparin injection. Later on she should take warferin for long time. These all are prescription based medicine so consult your doctor and take treatment accordingly. Hope I have answered your question, if you have doubt then I will be happy to answer. Thanks for using health care magic. Wish you a very good health."
},
{
"id": 214255,
"tgt": "How to eat durian fruit for conception?",
"src": "Patient: can i eat durian fruit for child how far its true and what is the procedure to eat this fruit.accourding to period cycle which date i should start the fruit.i got married on 2010 i had a cyst and it was punchured and i did take treatment it was failed please suggest natural treatment and how to eat this durian fruit Doctor: , durian fruit has to be taken daily and there is no restriction. durian fruit helps in increasing the libido and thereby increases the chance of conception. otherwise it has natual vitamins like vit c in it it helps in delaying the aging process.hope this explainswith regardsDr.Amarnath"
},
{
"id": 167254,
"tgt": "What causes constant accidents in a child suffering from hashimoto s thyroiditis?",
"src": "Patient: My 7 year old was recently diagnosed with Hashimoto s thyroiditis. Her TSH was 103.I originally took her to the dr. because of constant accidents . Are the two related? She is 4 1 and weighs appx. 100 lbs. I also have Hashimoto s so is it genetic? Doctor: yes hashimoto is genetic disease which produce symptoms of hypothyroidism after a brief period of hyperthyroidism.hypothyroidism cause sluggishness of body and increased reflex response time. so this may b a cause of recurrent accidents."
},
{
"id": 106710,
"tgt": "Should one visit the ER for severe pain after a back sprain?",
"src": "Patient: I fell off my horse Wednesday night and Thursday morning the doctor said I have a sprained back and I have bone bruising all over my pelvis. I was told to go back if the pain gets worse, and it has. I am now in so much pain that I could vomit and I'm struggling at work. I already take ibuprofen (800mg/3x a day) and diazepam (5mg) at night, which doesn't help much. Is it time to go back to the doctor? Doctor: Hello and Welcome to \u2018Ask A Doctor\u2019 service. I have reviewed your query and here is my advice. Yes, you should go back, and perhaps change medications as well, seems like your pain is not well controlled, you will likely benefit from physical therapy. Hope I have answered your query. Let me know if I can assist you further."
},
{
"id": 133286,
"tgt": "What causes pain in leg muscles after having coeziday?",
"src": "Patient: i am taking co eziday but since 3 months i have severe muscular pain specially with my leg muscles and thigh musvles smtimes i couldnt even unable to stand after 15 min . i have stomach prb history also . the muscles shrink of my legs smtimes pain is very sweet Doctor: Hello there,The tablets you are using is for hypertension.Which is 2drug combination.Out of which one is diuretics, which might cause electrolyte change causing muscle cramps or painKindly visit your treating doctor with this complain he might change the dose at minimum or might change the drug that will reduce your problem.Thank you."
},
{
"id": 146403,
"tgt": "What causes lightheadedness?",
"src": "Patient: I have been feeling light headed today and when bending over to tee a golf ball, nearly passed out within 5-10 seconds. It would take me 20-30 seconds to recover. BP 123/68 pulse 80 and oxygen level at 98. What could the problem be. Currently on Qsymia 7.5 mg per day and blood pressure meds. 50 yr old male. Non smoker non drinker. Doctor: Hi,Thanks for asking.Based on your query, my opinion is as follows.1. It could be postural hypotension or dehydration.2. Get your blood pressure evaluated with change in posture. If its normal, take good amount of water.3. Blood pressure and oxygen levels are good. Possible dehydration. Take good amount of fluids. Continue with your healthy routine.Hope it helps.Any further queries, happy to help again."
},
{
"id": 208453,
"tgt": "Suggest remedy for mental health problem",
"src": "Patient: I ve had a reoccurring dream for as long as I can remember. The dream is that the objects and people around me are shrinking and growing quickly. Usually in the dream, I ll be holding something, usually a toothpick or a stick, and it grows or shrinks right in front of me. The people around me are speaking to me but I do not understand. The people will grow and proportionally. I feel as if I can t breath in the dream because of what going on. Just thinking about the dream makes me feel like my throat is closing. I ve researched it and found that there is a condition called Micropsia. also known as Alice In Wonderland Syndrome. Those who have the condition have hallucinations that the things around them are growing and shrinking, or seeming far away or very close. I only have dreams about things growing and shrinking, but I do have horrible depth perception. Thank you for your time, and please respond as soon as you can. Doctor: DearWe understand your concernsI went through your description. I can categorically rule out micropsia. You are just having dreams and dream analysis is a complex analysis. Only 5 % of dream analysis came true. But what am I worried is that why are you worried about a dream which is not at all worrysome. And I think that is your problem. You are worried about that dream and that worry makes you to expect the dream. Once you expect something, body gives you (that is the essence of positive thinking). Once you get it you again start worrying and again expect it. Such cycle goes on and you become worried nd anxious. We call it obsessive thinking. Ignore the dream and concentrate on your education / profession. Within a week you shall be cleared. You can also consult a psychological counselor for further advice.If you feel that I can be of help to you, feel free to post a direct question to me and am obliged to serve your cause. I shall prescribe Cognitive Behavioral Therapy techniques tailor made for you to clear your problems.Hope this answers your query. Available for further clarifications.Good luck."
},
{
"id": 30967,
"tgt": "Suggest treatment for pain and liquid filled bump on knee",
"src": "Patient: Hello, I have this pain in my knee and I went to the doctor about a month ago and they drained it, it was an abscense, with cellulitis in my leg. But now my knee is starting to hurt again, not near as much but I tried to squeeze it and nasty stuff came out of it. Doctor: HI, thanks for using healthcare magicThis may mean that the abscess has recurred.This is a possible complication of an abscess. It would be best to see your doctor for assessment which would include examination and likely incision and drainage of the area.A course of antibiotics may also be needed as well.I hope this helps"
},
{
"id": 35034,
"tgt": "Suggest treatment for infection on the incision after hysterectomy",
"src": "Patient: 19 days post op with full abdominal hysterectomy and lymph node removal. Some staples were removed this past Friday, hopefully others this Friday the 25th. I have an infection along part of the incision line and have been on Keflex for about four days now with daily cleaning with 1/2 peroxide and 1/2 water but there is still redness and very sore spots along the line. Considering the length of time the staples have been there, could these be part of the problem. Also is there great sucess with RT & Chemo with Stage 1B Grade 3 (serous features) in preventing reoccurence. Doctor: Hello there,I am dr.milan an infectious disease specialist answering your question.For your case what i feel you is you may have infection which is acquired during hospital stay. So now you need to go for culture examination from that wound area specimen. THough antibiotic is running so might be culture comes negative, but we should go once at least. And also that specimen is need to be screen for atypical mycobacteria & fungus. Because sometimes this type of organism is found in such a case. Than if any organism is found you can go with sensitivity pattern under guidance of your surgeon.For RT and chemotherapy opinion you need to follow guidance of your surgeon, it can be advised only after reviewing all the reports including radiological findings and local examination.Hope i have given appropriate guidance to you.if you have any query you can consult me anytime.Give me star rating according to your satisfactory level.Thanking you."
},
{
"id": 123940,
"tgt": "What causes fainting and pain after hit at the back?",
"src": "Patient: hello i have a friend who hurt her back last night working in a nursing she took avild for the pain she ate find but she fainting but she was not breathing until i put a cold wet rag on her and started to talk to her and put a fan on her too is that the avild did that to her Doctor: Hello, Your friend suffered from vasovagal attack due to injury or had an allergic reaction to the medicine that she took. In either case, make the patient lie down. Losen the necktie & belt around the waist. Turn the face on one side so that breathing is possible. Lift the legs. Be ready for artificial respiration. Get the ambulance service informed. Hope I have answered your query. Let me know if I can assist you further. Take care Regards, Dr Nirmal Chander Gupta, Orthopaedic Surgeon"
},
{
"id": 170546,
"tgt": "What causes rashes while having rice and wheat for my baby?",
"src": "Patient: Hello doc, my baby is 6 months old. I am giving her both rice and wheat cereal. I am seeing some rashes on her right cheek. Earlier she used to have rashes . Is this food allergy? Should I discount ue wheat cereal and give only rice. She does not like gerber cereal. Doctor: Hi! thank you for consulting with healthcare magic!Is the rashes just on her right cheek? Have you noticed rashes on other parts of her body? Babies have very sensitive skin. It can be that she always sleeps on the right side and this part of the face is always in contact with the bed sheet or the cloth where she sleeps. It is best to avoid strong scents like perfumes, lotions and powder for both babies and caregivers. Also, when you start feeding the baby, introduce food one at a time. Give one kind of food for 3 days before introducing a new food to her. In that way you can observe if she has any reaction to that particular food. For example give wheat cereals for 3 days. Observe if she has a reaction like rashes, swollen eyes or in worst cases difficulty of breathing. If none, you may give mashed vegetable like potato for the next 3 days. Again observe for a reaction. If none, then you know she can eat wheat cereals and potato. Then again you can add another kind.Hope I was able to answer your question. Have a good day!Sincerely, Hannalae Dulay-See, M.D.Pediatrics"
},
{
"id": 193514,
"tgt": "What causes recurring pain in penis?",
"src": "Patient: My penis has hurt on and off for several years. I have been screened for STD and have only had sex once in the last 8 years *sep. 2009* the pain goes away, then comes back and can stay for a few days at a time. it is not quite the head of my penis, but more along the underside of the shaft, and I have had some discomfort in the right side of my groin but have had hernia repair there before. I am wondering if chronic back pain could be causing the symptoms? Doctor: Hi, Since you had surgery done it can be due to neuropathic. Yes, it can be due to spinal nerve compression. Doing blood sugar can help you. Hope I have answered your query. Let me know if I can assist you further. Take care Regards, Dr S.R.Raveendran, Sexologist"
},
{
"id": 180535,
"tgt": "What does a bump on the tip of the tongue indicate?",
"src": "Patient: I really need an answer to whats going on with me. Im a 28 year old male and i have basically scared myself to death trying to google what the bump on the tip of my tongue is. Its a fleshy small bump thats sort of hanging on the tip of my tongue. I can move it around and it has a white-ish looking head. Not like a pimple. I tried to pull it off last night with a pin needle and tweezers last night and it wouldnt come off. All i did was irritate it and it bled a tiny bit. Everything i look up leads to cankors and i ate something hot or spicy etc. The things is, its been on there for over a month now and everything i look up is now leading to a HPV bump or early sigs of mouth cancer.. i have a history of bad teeth on my dads side of the family. Ive never had braces always brush my teeth but lately my teeth have gotten bad. Im worried myself to death thinking i have some kinda tongue disease or early signs of cancer now. I dont have a lrimary dentist to go to. I cant find a single picture online that looks like mine. Its the only bump on my tongue its painless its just in the way bcuz its on the top tip of my tongue and moves around in my mouth all day. Its embarrassing bcuz i feel it when i kiss my wife. I had an ex gf about 8 years ago that had hpv and had to get anal warts removed from her. Im wondering im starting to get a wart on the tip of my tongue it looks sorta like a fish egg. It cant be popped or pulled off. Any answers or opinions would help becuz im a nervous wreck.. i do drink i do smoke.. ive literally been researchin this for 48 hours strate and am freakin out. A professional opinion would really be nice insyead of the same websotes poopin up leading to cancer!! Doctor: Hello and Welcome to \u2018Ask A Doctor\u2019 service. I have reviewed your query and here is my advice. I have viewed the query and here is my advice. From your explanation it looks like an overgrown tongue papillae or tongue wart. Without clinical examination it is not possible to tell you what exactly it is. So please send me a picture of the tongue growth so that I can guide you better. Hope I have answered your query. Let me know if I can assist you further."
},
{
"id": 64768,
"tgt": "What causes lump on knee after falling?",
"src": "Patient: I fell over a year ago and the area directly after my knee remained with a now noticeable lump on each of my legs, in the exact same spot.I do not have diabetes,high blood pressure or any health issues currently. The swelling does reduce when i took water pills,prescribed after the doctor saw the swelling ,He said it was nothing and could be from veins, they are also permanently showing since I the fall..I am 30 years old. I currently lost 19 lbs but it hasn't helped. I walk every morning and use my treadmill in between . Can you kindly give me some advice as to what the problem may be. Doctor: Hi,DEar,thanks for your query to HCM.I studied your query in depth.1-In My opinion the painless lumps on both the knees -post trauma a year back,which reduces on the water-pills(Tb Lasix-?) is mostly a -MB Cyst or the Patellar Bursa-Cyst.2-I dont think it to be from the vein,for which I would advise Surgeons consultation, who would fix the cause with more detailed study.3-Hope this would solve your query.4-Wellcome for any more questions on new or this subject also...Have A Good Day..!!"
},
{
"id": 22788,
"tgt": "What can be done to control BP without medicines?",
"src": "Patient: Hi.. My fiance is 22 years old. He has high bp.. Normally 140/90 without any medicines.. He has this problen since the last two years.. He is on medication.. betacap tr40.. His bp sometimes goes upto 160/110 or 168/110. He weighs 82kgs and is 5'10\". He has severe headaches and body ache.. What should he do to control his bp withou medications? If he quits medications.. his bp again goes upto around 140/90 in 2-3 days.. Doctor: Hi,first we should try to find cause why he high bp in young age.get hisrenal artery Doppler test,Renal function test 24hr Urinary metanephrines serum cortisol.Start him on low salt diet and regular exercise, including yoga..Bp should be measured at rest of atleast 10 min."
},
{
"id": 213023,
"tgt": "Depression, worried, convincing of being pregnant after taking microgynon 30. How to get relief?",
"src": "Patient: I ve been on microgynon 30 for over a year, around beginning of december 2010, never had any problems with it, was able to take pills back to back wouldn t have any issues what so ever. Since december 2011, i began to get all depressive , started worrying about everything, like worrying my boyfriend didn t love me, my friends didn t really like me, and also started convincing myself i was pregnant , even though i had all my periods on time, and they were normal (partly thing this was because some girl at college got pregnant for using no protection, auntie was pregnant, it was just always mentioned) so i got it in my head that i could be and got myself all worked up. I even managed to get things like sore breasts, pains in my stomach , tiredness, everything, but only when i think about it, anything i get wrong with me i am convinced is a symptom of pregnancy because i sat there looking up symptoms and scared myself big time. I ve always been quite an emotional person but i feel since december i have become worse and all i can think of is my pill. I take it at 7am everyday unless the odd day my alarm doesn t go off and i ll take it 8:30am latest, never miss any, i m VERY conscious about it, i ll always think i ve forgotten to take it when i haven t and panic. I m 17, nearly 18 and just want to be able to sort out this issue as it s genuinely effecting my life now, We don t always use condoms as in the leaflet it says you don t have to as long as you take the pills correctly and I am fully aware you aren t protected against sti s but me and my long term boyfriend are each clean. Doctor: Hello and welcome to Healthcare Magic. Thanks for your query. You need professional help to deal with your depressive symptoms and the false belief that you could be pregnant (which could be anxiety-related or sometimes, indicative of a delusion). I would advise you consult a psychiatrist for a detailed psychological assessment and further treatment. There are effective treatment options - in the form of medication or counselling / psychotherapy which will help you overcome your problems. Wish you all the best. - Dr. Jonas Sundarakumar Consultant Psychiatrist"
},
{
"id": 175658,
"tgt": "What causes fever, loss of appetite and tiredness?",
"src": "Patient: My daughter is almost 2 yrs old. She had a fever for 3 days about 2 weeks ago. She currently has not had an appetite for nearly a week, cries way more than usual, staying tired, has a bad cough, looks like she is losing weight and looks pale.. What could this be? Doctor: Hi...by what you quote I feel that your daughter is in convalescence period of a viral illness. She's recovering from viral illness. All the symptoms suit. So do not worry. She will get better in a couple of days.Regards - Dr. Sumanth"
},
{
"id": 112141,
"tgt": "What is the cure for chronic back pain and neck pain?",
"src": "Patient: I have chronic back and neck pain, along with Thyroid edema (chronic) and I have a hard time finding a doctor in my area that will treat it with pain medicine. Its the only thing that works - I have tried every over the counter medicine and they do not work. What can I do? YYYY@YYYY Doctor: Hi,Chronic neck and back pain may be due to spine problems. Most common is spondylosis but a complete clinical examination is required. You may require MRI spine depending on the type of pain. If analgesics are not responding then pain may be due to nerve compression. You have to consult a local orthopedic doctor to find out the cause and treatment accordingly.Do consider a positive feedback as a credit to my work. Let me know if you have any further questions."
},
{
"id": 49656,
"tgt": "Bloating, sore lower abdomen, pain in kidney area. History of kidney stones",
"src": "Patient: Hi. For a while I have been bloated, and sore in the lower abdomen area. Lately, it's been in my back area with sharper pains in the kidney area. I am always in pain either dull or sharp pains. It's uncomfortable to sit, or where pants because of the pressure. I did pass a kidney stone about 5 years ago, but this pain has been going on for a while unlike the quicker pain I had when I went in for stones. Doctor: Hi,From history it appears that you might be having kidney stone problem again.Go for urine check up and ultra sound.after report go for treatment accordingly.Take light diet and plenty of water.ok and take care."
},
{
"id": 30353,
"tgt": "What causes elevated eosinophil count and fever?",
"src": "Patient: My GI diagnosed me with IBS in April. However, everytime I experience an \"Attack\" a fever is always associated with it. Last night I had a fever of 102 along with my attack. I am suffering severe joint and muscle pain which is also not associated with IBS. I am mostly concerned because while I going through the diagnostic process with my GI for the month of March, repeated blood tests shown my eosinophil levels were elevated. And I have been dealing with these on and off again fevers. I spoke with my GI today and they did not offer much help. I know my primary care physician is not going to be of much help either. What do I do? What is with the repeated fevers? Doctor: Hello,The elevated eosinophil count could be a sign of parasite infection. Please have one of your doctor do a stool culture and an ova and parasite panel to make sure that this is not the cause.Regards"
},
{
"id": 96052,
"tgt": "I am having abdominal pain,lump in rectum and blood in urine. What can i do ?",
"src": "Patient: MY HUSBAND HAS A BIG PAINFUL LUMP IN HIS RECTUM BLOOD IN THE URINE IS UNABLE TO AMBULATE DUE TO THE PAIN WHAT CAN BE WRONG MY HUSBAND IS 35 YEARS OLD, COMPLAINING FOR THE LAST 3 DAYS OF ABDOMINAL PAIN , PELVIC PAIN , AND ALSO LUMP IN THE RECTUM. HE ALSO NOTICED BLOOD WHEN URINATING. HE IS UNABLE TO AMBULATE AND HAS NOT BEEN ABLE TO HAVE A BOWEL MOVEMENT DUE TO THE PAIN. WHAT CAN BE WRONG? Doctor: Hi; welcome to HealthcareMagic The painful lump in rectum which is preventing bowel movement as well as ambulatation could be a blind or obstructed piles.Please consult a Surgeon as soon as possible so that he can examine him and give treatment to reduce the pain & help him be comfortable.Don't wait consult soon to avoid further complication. Thanks"
},
{
"id": 21701,
"tgt": "Is angiogram required for episodes of chest pain?",
"src": "Patient: I'm a 45 year old male with mild-to-moderate restrictive lung issues (cause unknown). I've had a few episodes of chest pain radiating in left arm - went to the emerg once - they said there was something in ecg but enzymes were normal, so no heart attack. They referred me to a cardiologist who did stress test - was normal, nuclear scan - again normal but now he wants to do an angiogram. If all the tests are normal, should I still go through this test? As he explained there are some risks involved. Doctor: Hi,Usually when stress test is normal we don't go for further examinations, such as angiography. Rarely, there are cases, when suspicion remained, despite tests and the patient has additional risk factors, such as familly hystory of heart attacks, strokes, heavy smoker, very high cholesterol.In such cases we may order angiography to be in the safe side.Take care"
},
{
"id": 161894,
"tgt": "Is bathing in chlorinated water safe for kids?",
"src": "Patient: I have a friend who has a 2 yo and a 4 yo. The tap water in their home has a strong smell of chlorine. Even though they don t drink the tap water, they use it to rinse fruits and vegetables, shower, and bathe in it. I have read articles on the internet that a 10-minute shower with this water is like drinking eight glasses of it since the chlorine is absorbed into the skin. Is this water dangerous to this family, especially the kids? Doctor: Hi, Usually, chlorinated water from the tap is safe for bathing as the supply would usually be met with the ppm criteria regulations. Nothing to worry about this. Hope I have answered your query. Let me know if I can assist you further. Regards, Dr. Sumanth Amperayani, Pediatrician, Pulmonology"
},
{
"id": 48300,
"tgt": "Can I take Tradjenta Duo for kidney damage?",
"src": "Patient: I have been taking Trajenta Duo 2/500 mg two times a day and Glimestar M3 Forte also two times a day. The total quantity of Metformin is 4000 mg per day. My homeopathy doctor says that too much Metformin can cause kidney damage. What is your advise ? Doctor: Hi,Thanks for writing in.Trajenta Duo contains Linagliptin and metformin which is give to control diabetes. It should not be given if there is renal failure or dysfunction (CrCl <60 mL/min). Creatinine clearance is a test that measures the functioning of the kidneys. If your creatinine clearance is more than 60 ml per min then you can take Trajenta Duo safely. This medication is to be taken under doctors supervision and in recommended doses. Take this medication after food. Please do not worry."
},
{
"id": 10783,
"tgt": "How to treat hair fall?",
"src": "Patient: hi sir,my age is 26(M) years & am suffering from hair fall since 6 month.i have consult to a doctor.he suggest me to apply MorrF & folifast solution daily,it 45 days passed but there is no remedy.can i continue this medicine or go to some other doctor. Doctor: hey , i can understand your concern,you can get your hemoglobin, serum ferritin and thyroid levels tested.sometimes hairfall is due to dysbalance of all these,if all the parameters turns out to be normal than minoxidil application and biotins once a day for a long period of time can be helpful.but its a long therapy, you will get the evident results after three months , so be patient.good luck"
},
{
"id": 208690,
"tgt": "Suggest treatment to adjust new environment with all normal activity",
"src": "Patient: Dear sir My brother haveing problem daily activity everything ok behaviour with people communiction with family people ok but is not interested to go job,if he join duty he will work six month after he blame others if some one says any thing with loudervoice he will don,t like immediately he stop going for job. Doctor: Hi dear,might be you brother is not adjusted at particular environment and due to that he feels irritation or depressive and anxiety feeling and after left job.so he might have adjustment disorder.for better work adjustment at your work site is important for that you must visit at psychiatrist and do counseling about your problem.Not to worry much.there are psychotherapy and medication by which all is good happen.Thank you"
},
{
"id": 101075,
"tgt": "What causes burning sensation on the hands after taking bath?",
"src": "Patient: Hi, sometimes when I shower my hands swell up and feel like they are on fire. My fingers and palms of my hands visibly swell. It's not every time I shower, just occasionally. Also running my hands under ice cold water then raising my hands high into the air seems to make better. Any idea what this could be or mean? Doctor: HI, thanks for using healthcare magicThere is a condition called cold urticaria where persons experience an allergic response to cold temperature.The allergy would cause the swelling and abnormal sensations.It is possible that this is the cause of your symptoms. You may want to consider visiting a dermatologist for review.The most important aspect of treatment is to stay warm.I hope this helps"
},
{
"id": 93312,
"tgt": "Have gas and pain in belly. Excessive farting. Left side belly is hard. Suggestions?",
"src": "Patient: Good day doctor,I am 43 years man. I experienced much air/gas on my belly last night that resulted on excessive farting, but I discovered pain all over my belly when I work up this morning, tried to feel my belly, I discovered that the left side of my belly is harder than the right side. I went to my doctor and he requested I should go for sonar and report back tomorrow. I am worried and wish to know if any advice from you doctor.Thanks and regards,Tony Osita Email: YYYY@YYYY Doctor: Hi, Thanks for writing your query. After reading your post, it seems that you are having your symptoms due to gastritis caused by excessive acidity and gas production in your stomach. You can take an antacid like Omeprazole 40 mg along with Domperidone once a day before breakfast. Following precautions will be helpful :- 1. Do not miss a meal. 2. Avoid spicy and oily foods. 3. Avoid smoking/alcohol, if any. 4. Avoid emotional/mental stress. 5. Do not self medicate, especially with pain killers and steroids. 6. Avoid gas producing foods like peas, legumes, etc. I hope this is helpful to you. Thanks."
},
{
"id": 146095,
"tgt": "What does this MRI report of spine indicate?",
"src": "Patient: Hello doctor,I am a 34 year old male suffering from severe spine,shoulder and hand pain. I have taken a MRI spine today and the report is as follows.1)Broad based disc protrusion of C6/7 with peropheral annular tear with compression of thecal sac and compromise of exiting neural foramina with disc bulges at C4/5 and C7/D1 with no canal compromise.2) C5 and C6 vertebral bodies suggest block with narrow waist /no marrow oedema with rest of the study normal - congenital blocked bodies with no post elemental involvement is possible.3) Loss of bright signal on T2 WI suggest diffuse degenerative disc disease at all the cervical levels.4) All the rest of vertebral bodies suggest heterogenous fatty marrow changes.5) Pedicles,laminae,spinous processes,facet joints,craniovertebral junction,spinal cord is normal.6) Screening of dorsal spine,lumbar spine as seen in the whole spine scout normal.Please tell me what this means and what treatment should I be taking now.I am very nervous.Please help. Doctor: I read your question and I understand your concern.The MRI shows degenerative changes of your spine which are to be expected to a certain extent in an old age but are fairly premature for a 34 year old, I don't know your medical history, you should be investigated for some chronic joint inflammatory condition.What specifically causes your pain spreading to the limb though, is the herniated disc between C6 and C7 vertebrae compressing the nerve root.The first options are antiinflammatory over the counter drugs like ibuprofene, analgesics like acetaminophene, physiotherapy and the use of a cervical collar.If those measures don't work then local injections with steroids and anesthetics near the nerve root are the second option.The last option if symptoms persist is surgery.I hope things work out for the best."
},
{
"id": 118629,
"tgt": "Diagnosed with severe anemia. Treatment period? Recovery period?",
"src": "Patient: Hi! I was just diagnosed this week with a severe anemia . Its weekend and I will be able to see my Internal Medicine doctor after my endoscopy and colonoscopy procedures which will be done next week, but I m so anxious to know How long will be the entire treatment for severe anemia and how long will be the recovery process? . Your immediate reply is greatly appreciated. PLEASE REMOVE MY POSTING NAME. THANK YOU, Doctor: Severe anemia,if Hb is less than 5%,you need immediate blood transfusion ,even before the procedures under emergency ICCU.The procedures are to evaluate and find out the cause.If the cause is found,the relevant appropriate management may be done to cure you from the disease."
},
{
"id": 149245,
"tgt": "Numbness in fingers and arm. Serious?",
"src": "Patient: Hi. I woke up last week and the outside skin between my right thumb and forefinger had become slightly numb. I noticed also that the upper part of my right arm also had a similar sensation. However I often wake up with pins and needles in my hand / arms as if I have pinched a blood vessel or vein in my sleep. The numbness is slowly diminishing after a week. Is this something I should take seriously? I am 53 years old and otherwise healthy. My BMI is below average, by blood pressure is 130 / 80, my heart rate is ~80 / minute and my blood sugar is 4 on a scale of 2 ~ 6. I also walk 1 to 2 hours a day, 7 days a week and generally feel fine. I don t think I have any high risk factors but I wonder about other potential degenerative diseases such as Alzheimers as I find that my short term memory sometimes turns off lately. Again, any suggestions are welcome. Doctor: Namaste Welcome to HCM Greetings of the day There seems nothing wrong with you. Pins and needles is due to nerve compression, also Vitamin deficiency especially B12 will aggravate the symptoms. Start taking Multi vitamin supplements which will definitely help you. As You have a good life style like walking for 1 - 2hours which is really good indeed. Try meditation and yoga which will help your memory. If problem persists get a routine assessment done from your physician. Take care Regards"
},
{
"id": 17291,
"tgt": "What causes skipping of heart beat,rapid heart rate and panic attack?",
"src": "Patient: heart feels like it skips a beat then that makes my throat tickle and i cough,gets worse when falling asleep. i usually end up waking up feeling as if i stopped breathing for a minute and my heart stopped.then my heart starts beating rewal fast like a panic attack and i try to lay down and drift off to sleep again.this will happen a few more times then i eventually fall asleep Doctor: Hello, Your symptoms could be related to a sleep disorder. Anxiety or sleep apnea cannot be excluded either. For this reason, I would recommend performing a polysomnogram in order to investigate for a possible sleep disorder. Hope I have answered your query. Let me know if I can assist you further. Take care Regards, Dr Ilir Sharka, Cardiologist"
},
{
"id": 167526,
"tgt": "What causes hard stools with mucus in a 2 year old?",
"src": "Patient: I just found two bean like balls in my 2 year olds, nappy. She had diarrhea earlier today but when i changed her nappy just now it was just two little balls with a bit of mucuos. I have thought about her diet over the past few days and I know she hasnt eaten anything like this Doctor: Hi...Thank you for consulting in Health Care magic.Unless the kid's having low urine output or very dull or excessively sleepy or blood in motion or green bilious vomiting...you need not worry. Regards - Dr. Sumanth"
},
{
"id": 200281,
"tgt": "What causes pain in penis while masturbating?",
"src": "Patient: Hi, I am 30 year old. It s been 3 months am having a dull pain in the center pipe of my penis. It started when I masturbated a bit roughly. Even I speak on phone with my gf, my penis gets a half erection & a white sticky substance comes out. The erection also doesn t last long when I maturbate now& post I feel a light pain in my pipe day long. Please help Doctor: HiI appreciate your concernLooking at your history it seems to be urinary tract infectionif such a case comes to my clinic I would advise him to get urine and urethra discharge swab test for culture and sensitivitya course of antibiotics is recommended in your casehave plenty of fluids maintain good hygienealkalisation of urine Hope this helps youthanks for your questionwish you a very good health take care"
},
{
"id": 113259,
"tgt": "Have back pain and lump on abdomen. Had tumor removed from abdomen and thyroid. Cure?",
"src": "Patient: I have had many tests done and no real answers, lower back pain lower pelvicarea pain and large lump on my lower right abdomen, pressure almost constant that feels like labour pains right before pushing. have had hysterectomy 10 years ago. tumors removed 6years ago from the right side of abdoman and tumor removed from thyroid 2 years ago. checked for chrones, colitis , and many other things and all negetive. am at the point i am at wits end from the pain, pass out it gets so bad at times Doctor: Hello. Thanks for writing to us. Since no pathology has been detected after various investigations, it seems to be a nerve related pain. This can happen due to nerve root irritation or compression. You can consult a neurologist for proper evaluation. I hope this information has been both informative and helpful for you. Regards, Dr. Praveen Tayal drtayal72@gmail.com"
},
{
"id": 98877,
"tgt": "What causes bruising, shortness of breath and headaches with red sots under the skin?",
"src": "Patient: Hey,right, I ve been coming out with random bruising,shortness of breath ( I ve also got asthma ),I wake up in the morning, with my body aching all the time? And I don t think this is normal? I ve been getting headaches at least 3 times a day everyday,but they are really painful? And it s also been effecting my eyes? I ve also noticed a few red/pinkish spots under the skin, I just wanted you option,and I just wanted to know if this was normal . I ve also been freaking out as I seen on Facebook that this little girl had all of these issue, and they then found that she has leukaemia? I ve also read that these are some of the symptoms? What do you recommending me doing? Thank you so much,I look forward from hearing from you. From Emily ,age 14 Doctor: Do not panic. Simply opt for getting safest on earth - Ayurveda anti allergic treatment for your immunity to enhance and correcting the current problem. Asthma is also not curable by allopathy. In Ayurveda it may cure, it may not cure fully but constant herbal medicine which are simple ones, will raise immunity agaist side effects and ailment both.In my clinical career of three decades, many complicated cases have been solved using alternative approach. If you agree pl ask direct question with all the advise you need to know.Hope it helps. Pl write direct question."
},
{
"id": 108117,
"tgt": "Can the reduction of the size of buttocks reduce back pain?",
"src": "Patient: I am a male 21 years of age. Im 5 10, 245. I lift weigts 4 to 5 days a week. Recently i have been having alot of back pain, and i think its due to my oversize buttox. I am not fat, i have no problem running miles and i dont think its due to lifting. I do have a job where i sit in a chair all day, but i do get up and stretch. But i recently while im running my back has been hurting, or just little things have been causing pain. I have a 38 waist but around my butt is 44, so jean shopping is difficult. I just would like to know if a buttox reduction would help, and before i go to the doctor what are some question that i should ask while there. Doctor: HiYou appear to be physically fit from weights and exercise but your back issue could be due to lower spine problem. prominence of the buttocks can be the result of a lordosis which causes the buttocks to protrude.Long hours of desk work and poor work posture contributes to this.Your doctor needs to Xray the low back to look for spinal deformity and for muscle spasm and also see if reduction of buttock size can help resolve this.Buttock reduction is a plastic surgery procedure and needs evaluation by that speciality.so your questions will be:do I have spinal deformity or muscular spasm?do I have a lordosis and how do I correct this?"
},
{
"id": 207825,
"tgt": "What causes hair loss and fatigue while having major depression?",
"src": "Patient: I m a 43yr old female and for the past 3 or 4 months my hair has been falling out, fatigue all the time to the point I don t do anything on my days off and have to push myself out of bed to go to work, and have begun gaining weight eventhough I have started walking. I do have been diagnose with severe Major Depression for most of my life but I feel like this is more than just my depression Doctor: You may be right. At your age being a woman the first possibility is hypothyroidism. Therefore, you may get your Serum T3, T4, TSH levels checked. Other causes include nutritional deficiency, Vit D deficiency, cardiac problem, hormonal problems surrounding menopause if you are going through menopause. You may try Vitamin supplements.I hope this answers your question."
},
{
"id": 108551,
"tgt": "Suggest remedy for painful lower back spasms",
"src": "Patient: Extremely painful lower back spasms started on left hand side and now have passed to the right hand side. Weight lifting belts now make matters worse. I run a farm, cycle and walk every day. Probably caused by very heavy lifting and sawing up of fallen trees, 58 years old. Help please. Doctor: Hi,from history it seems that there might be having heavy strain of back muscles and even acute prolapse disc giving this problem due to lifting very heavy trees.Take analgesic medicine with muscle relaxant medicine for 2-3 days.If problem persisted then consult orthopedic surgeon and get examined.X ray of lower spines will give exact cause of your problem.Take complete rest.Ok and take care."
},
{
"id": 45250,
"tgt": "Is there any food supplement to reduce high pro lactine ?",
"src": "Patient: I m 27 yrs women. My prolactine is 27.5. Is there any food supplyment to reduce high prolactine? Doctor: Hello Welcome to health care magic forum there is no food supplement to reduce serum prolactine levels as it is a hormone Wish you good health Disclaimer"
},
{
"id": 164570,
"tgt": "What causes cough, stomach flu and fever in a 6 year old?",
"src": "Patient: Hi, may I answer your health queries right now ? Please type your query here...my almost 6 year old daughter has coughing, stomach flu and fever goes upto 103.8..i think she also pink eye on her left eye..if i give her any fever reducer medicine, she throws up. the only way i am tryinh to keep her temp don to 102 is by putting cold wet strips of cloth on her fore head..please tell me what can i do? Doctor: you can use rectal paracetamol suppositories. try to paracetamol patches also if available in your drug stores."
},
{
"id": 211536,
"tgt": "Suffering from inferiority complex, always get teased, overshadowed since childhood. Difficult to take initiative. How to overcome?",
"src": "Patient: I am suffering frm inferiority complexon, an am smallest in the family and been always ovrshadowd by my bigger brother, my opinion is always givn last or no priority, i was teased by my classmates for being fat, frequently abused by my brother , now i am 19 yr old, and maintained good physic and height but i still have left over of my childwood mayhem, i always find difficulty to take initiative, make new frnds, gal frnds.. Pls adviss e me, i am going to persue charterd accntancy intrnship nxt month, and i am vry nervous.. Doctor: Hi,You need to focus on your strengths rather than your weaknesses and the traumatic past. Look at your achievements and feel proud of them. When you like yourself for what you are, it shows in your demeanor and self confidence and attracts people towards you. So make a list of your successes and appreciate yourself. Also start practicing relaxation exercises to get rid of your nervousness. Slow, deep breathing and progressive muscular relaxation are both good techniques for relaxation. You can get detailed instructions online for these techniques.Hope this helps you. Best wishes."
},
{
"id": 192150,
"tgt": "How to cure Erectile dysfunction?",
"src": "Patient: hi, i got married by 4 months ago, after marriage i had good sex with my partner, afterwards, one day my penis was not got enough tightness to have sex, however i have managed to had sex with her, she too enjoyed the same...however am feeling fear till now to have sex not having that much interest in that campare to how i was in before my marriage...plz..guide me Doctor: Hello, You are not having erectile dysfunction since you getting enough erection to have sex and satisfy your partner. Just maintain enough gap between having sex like 4-5 days so that this problem will not occur. Take care. Hope I have answered your question. Let me know if I can assist you further. Regards, Dr. Pramod Kokare, General & Family Physician"
},
{
"id": 213490,
"tgt": "Poor mental health, disturbed, no proper response to treatment, two years to completely cure mental disease. Ways to improve mental health faster?",
"src": "Patient: sir I have a Sister. She was disturb till 1 year back. We went to Several doctor but responses were not good. Atlat we went to pshychiatrist, Now she is looking ok .Doctor is Saying it will take more than two years to be treated well and also saying that disease is cured but treatment is required for the disease no to come. please advice. We want it to be ok within 6 to 9 months Doctor: Hello Narayan Thank you for getting in touch with us. From what I make out, your sister was diagnosed with a psychiatric disorder sometime back. It would have been better if you had mentioned the diagnosis as we would have been able to provide more information. Then also, we would like to inform you that for many psychiatric disorders, treatment carries on for a long time. The medicines available in the market these days are really effective but they have to be taken on a regular basis. Missing your doses in few psychiatric disorders can lead to complications. For most psychiatric patients, family support also is very crucial as these patients are not stable and need someone around them all the time. Please get back to us with the diagnosis and we will provide further details. Hope this helped. Dr. Gagandeep Dhillon"
},
{
"id": 70858,
"tgt": "What can cause pain and heaviness in the chest?",
"src": "Patient: Hello doctor my name is Masarat .. age ..29.. during my pregnancy se I get high Ths level ... Bt knw the problem is ... My tsh level is zero n my baby is 14 months old .... Frm sum days I HV uper chest pain n inching ... N heaviness ... Can u tell me pzz ... What's the problem ... Is it due to low tsh or other issue Doctor: Hi, TSH alteration will not give rise to solo chest pain or heaviness. As per my clinical experience, it is the underlying stress with gastritis responsible for the pain and heaviness in the upper chest. Hope I have answered your query. Let me know if I can assist you further."
},
{
"id": 165085,
"tgt": "What signs should be checked out in a 13 month old having whooping cough?",
"src": "Patient: Our 13 month old just got her first year shots on Tuesday. They gave her the whooping cough and MMR shot along with the Dtap. She developed a cough the next day and it has progressivly gotten worse. What are some signs that we need to get her checked out? Doctor: you must do chest x ray for follow up any complication and you must keep taking antibiotic and cough suuppressant"
},
{
"id": 78696,
"tgt": "Suggest medication for chronic cough",
"src": "Patient: trying to figure out why I am coughing, had 3 courses of antibiotics, don't smoke, chesk xray that was clear, blood tests that were clear but the cough is still here and worse in the mornings and at night time, I can't speak but I cough and also worse the day after I have had alcohol. I had sarcodosis 15 years ago, on my skin though, my lung tests were always clear. I could go back to my doctor but what else can we do? Doctor: Thanks for your question on Health Care Magic. I can understand your concern. Cough with breathlessness are commonly seen in bronchitis and lung infection. Since your chest x ray is normal, no need to worry for lung infection. Possibility of bronchitis is more in your case. So better to consult pulmonologist and get done clinical examination of respiratory system and PFT (Pulmonary Function Test). PFT is needed for the diagnosis of bronchitis. It will also tell you about severity of the disease and treatment of bronchitis is based on severity only. You may need inhaled bronchodilators and inhaled corticosteroid (ICS). Sarcoidosis is a disease having remissions and exacerbations. So you will benefit with inhalers for it as well.Don't worry, you will be alright. Hope I have solved your query. Wish you good health. Thanks."
},
{
"id": 85394,
"tgt": "Will it be safe to take dispirin with soda?",
"src": "Patient: I am planning hangover with my friends. As alcohol is banned in our country, so as we have heard that if we take 2 tablets of disprin with pepsi we will be having hang over for a day. Is that true? Any side affects for this? I will be grateful to you, if you will answer this query of mine. Thankyou. Doctor: Hi, It may increase your risk of liver damage while taking disprin. Avoid coffee, tea, cola, energy drinks or other sources of caffeine while taking this medication. They can add to the side effects of the caffeine in the medication. Hope I have answered your query. Let me know if I can assist you further. Regards, Dr. Ajeet Singh, General & Family Physician"
},
{
"id": 85108,
"tgt": "What are the side effects of taking Coricidin while on Celexa & saphris?",
"src": "Patient: I am currently prescribed Celexa (citalopram) 20 mg daily and Saphris (asenapine) 10 mg daily. Would it be safe to take Coricidin HBP Cough and Cold (dextromethorphan HBr and chlorpheniramine maleate) in conjuction with these two medications? Are there any possible drug interactions? Doctor: Hi, If taken together these medications may cause the following harmful interactions: 1. Celexa with Dextrometharphan can increase the risk of a serious condition called the serotonin syndrome, which is characterized by confusion, hallucination, seizure, increased blood pressure and heart rate, fever, excessive sweating, shivering, blurred vision, muscle stiffness, tremor, stomach cramps, nausea, vomiting, and diarrhea. In its severe form it may result in coma and even death. 2. Celexa with Saphris can increase the risk of an irregular heart rhythm that may be serious and potentially life-threatening. 3. Celexa with Chlorpheniramine maleate may increase the side effects such as drowsiness, blurred vision, dry mouth, decreased sweating, difficulty urinating, abdominal cramping, constipation, irregular heartbeat, confusion, and memory problems. 4. Saphris with Dextromethorphan may increase side effects such as dizziness, drowsiness, confusion, and difficulty concentrating. Hope I have answered your query. Let me know if I can assist you further. Take care Regards, Dr. Mohammed Taher Ali, General & Family Physician"
},
{
"id": 36717,
"tgt": "Suggest treatment for red bump filled with liquid?",
"src": "Patient: Hi I have a little red bump whith liquid in It right above my wrist I cleaned the liquid in then returned the next day so I cleared it out again now it is very sore feels swollen like hurts when I apply pressure what is it now. Its red. But look a lil bruised Doctor: Hi,It seems that there might be having some insect bite giving some allergic reaction giving soreness and blister.Clean the part and apply triple action cream.Take some antihistamine like Cetrizine or Benadryl for 2-3 days.Ok and take care."
},
{
"id": 92985,
"tgt": "Facial area puffy, abdomen sore, tender. Are symptoms related. Possible cause?",
"src": "Patient: Two days ago my husband woke up and noticed his entire facial area was puffy. Although it is less puffy today, it is still noticeable today, it seems to be better. To add to this, today he has felt some soreness in his upper left abdomen just under the rib cage. If he applies pressure to the area it is somewhat tender to the touch. Do the two have anything to do with each other and what may be causing this? Doctor: Hello,Welcome to Healthcaremagic.Both Puffiness of face and left upper quadrant abdominal pain may or may not be relates.because puffiness of face may be seen in allergies and kidney diseases.Left upper abdominal may be either from heart cause like Heart attack, angina etc.Some other causes are Splenic causes ie. ruptured spleen, splenic abscess.Kidney causes are renal infection, stones or chronic or acute renal failure etc.Even gastric problem can be a cause.Since the facial puffiness decreases in one or two days most probably allergy or kidney problem could be the cause.Please do an ECG and X ray chest to ruleout Heart and lung disease.We have to do Ultrasound abdomen, blood urea , creatinine, and urinalysis to rule out kidney disease.As tenderness is in left upper quadrant, apleen may be involved. Check for hemoglobin and other RBC parameters and Peripheral smear for RBC details.Hope this helps.Take care.Regards,Dr.Manjeth."
},
{
"id": 64679,
"tgt": "What causes lumps in knee?",
"src": "Patient: hi after sport i always seem to get a lump on the outside of my right knee that sticks out quite abit, it feels quite hard. ive only really had this since i had water on the knee nearly 3 years ago now, i constatly play football and it only flares up during the game and is very distinctive after. thanks scott Doctor: Hi,dear ,Scotts,thanks for the query to my HCM virtual online clinic.Its my pleasure to help you .After the indepth study of your query My diagnosis -is -MB Cyst in rt knee.Cause of this lump-is the repeated miniscal and ligamental traumas / with resultant arthritis -is most probably the cause of this MB Cyst.Increase in it size after or during the football play-is due to the pressure changes while playing and increased joint fluid pressing it out more and flaring it up more.Still on safer side I advise you to Consult a ER Ortho-Surgeon.Dont worry and act fast.This would solve your worrysome query.Hope this would solve your query.Wellcome to my HCM clinic once again.Have a Good Day."
},
{
"id": 8794,
"tgt": "How should I apply cialo lotion on my face ?",
"src": "Patient: my doctor has recommended the cialo lotion when should i apply this. Today was my first day for application on my face . But it appearance remains on the face, i can t go out after this application of lotion. Advice me how to apply this cialo lotion. Doctor: Hi..dear Bond.., Thanks for choosing HCM., Cialo lotion.., It is calamine lotion..., It appears on face like this only.., It is soothing lotion and gives cooling effectg.., Slowly absorbs on the skin.., So don't worry about this.., Morning times apply on fqace and leave it off.., ok ..good luck..,"
},
{
"id": 130169,
"tgt": "What causes severe groin and thigh pain?",
"src": "Patient: Hi my mom ask sevre groin pain and pain in her thights. She has had bot hip replaced and the pain is still there. The same pain in her groin before she had the surgery. She has lost all of her upper motor skills in her legs. She cannot tie her shoes, put on shoes. go up stairs, roll over in bed. Doctor: Hello,She most likely is getting the groin pain from hip disease. Pain in the hips gets referred (transferred) into the groin area. She needs to see her orthopedic doctor to have her hips checked.Regards"
},
{
"id": 177168,
"tgt": "What are the symptoms of kippel trenaunay syndrome in children?",
"src": "Patient: What can you tell me about kippel Trenaunay Syndrome? My granddaughter was born width this. Her left arm is twice its size with pock type markings, as well as a large strawberry type area on the torso front and back. Where would something like this come from, and is there a cure? Doctor: KTS is a cutaneous vascular malformation caused by ARteriovenous malformation.its present since birth. it can cause a finger, a part or a whole limb to enlarge.child might present with cellulitis, pain and swelling of limbs along with a strawberry or a cavernous venous dilatations.complications that can occur include thrmboflebitis, heart failure, gangrene of hand, sepsis and infection.treatment includes percutaneous sclerotherapy guided by colour doppler ECHO, compression bandages for vericosities and corrective orthopedic surgeries for significant leg length discrepancy. treatment is palliative and symptomatic. no definite treatment available."
},
{
"id": 89818,
"tgt": "What causes severe abdomen cramps?",
"src": "Patient: hello, i m 18 and from yesterday morning i am having stomach cramps like that of what i go through during my periods but last week only my periods got over....from today morning 12:50am i am also having diarrhea, i am feeling very pukish though the puke is not coming and also my head is paining and at times i feel dizzy... can you please help me out. Doctor: HI.These are the symptoms of gastroenteritis. May be due to Waterborne bacterial infection. Go to ER get your vitals checked and get a prescription of antibiotics , metronidazole and supportive medicines, '"
},
{
"id": 194857,
"tgt": "Why is a biopsy required for blood in urine when the CT scan was negative 4 months ago?",
"src": "Patient: Risks with prostrate biopsy through the rectum. If my CT was negative 4 months ago and then had surgery for a very large prostate Discovered in the er due to blood in my urine and inability to urinate Why would they go in and take 18 punch biopsies when no mass has been seen. I m 60 and my psa went from 6.6 to 7.3. In 3 months.. Infection has been r/o and I want to be informed of the most up to date information. Thanks Doctor: Hello and Welcome to \u2018Ask A Doctor\u2019 service. I have reviewed your query and here is my advice. The PSA level indicates that something is going on in your prostate. In lieu of radiological evidence, transrectal biopsies are more sensitive in finding cancer. The most important risks of the biopsies are bleeding and prostatitis. Hope I have answered your query. Let me know if I can assist you further."
},
{
"id": 186448,
"tgt": "What causes neuropathic sensations to upper front teeth?",
"src": "Patient: I'm experiencing neuropathic sensations to my four upper front teeth and upper palate ( teeth feel brittle,thin,cracking, and loose). Dental exams, brain MRI and facial CT are normal. I'm on neurontin 600mg tid without much help. A dental mouth guard helps decrease sensations when in place, but symptoms immediately return when I take it off. The symptoms are there all the time and effect my sense of speech and chewing. Any ideas or suggestions for referral Doctor: hello thanks for consulting at hcm..u might be having deep overbite and overjet,,that the reason u might be having brittle teeth and palate,,since all ur examination including dental, ct and mri is normal,,either ways check for any gum pocket since ur feeling loose teeth,,also check for blood work up also ,,plz consult a orthodontist/ neurologist..hope it helps,,tc"
},
{
"id": 186841,
"tgt": "Having noticed a pea sized lump inside the mouth",
"src": "Patient: I am having a pea size swelling lump in between check and last teeth. Painful on chewing or pressure for chewing. If i press with finger or rub the check some blood comes out and got some relief from pain. But very short time it will again built up. what will the cause and treatment. Doctor: Hello, Welcome Thanks for consulting HCM, I have gone through your query, as you have small pea sized awelling between wisdom tooth and cheek dont worry ot can be due to formation of Pericoronal Abscess . Dont worry consult dentist and go for Currettage. Do warm saline gargle two - three times a day . Do betadine gargle two times a day. Consult dentist and go for treatment. Hope this will help you."
},
{
"id": 51256,
"tgt": "Got kidney transplantation. Why bleeding? Can I get pregnant?",
"src": "Patient: hello doctor...im 23 yrs old..i was a ckd patient and i got kidney transplantation on jan 2012...after that i was not having menstural cycle..on this may 15 it started bleeding for 1 month after that it get stopped for 5 or 6 days...during those 5 or 6 i will have discharges in red colour...and this is continously happening till now..in the scanning report it is noted that internal OS is dilated which measures 8.5mm..what does it mean sir?Is it possible for me to become pregnenet?I take following medicines...Myfortic 360mg,wysolone 10,carloc 10,prazopress XL 5mg,Tacrolimus capsules of .25 mg at night and .5 in the morning, bactrim DS.. Doctor: Welcome to HealthcareMagicForum Hi..fasmin.. the os dilated means , there is dilation of cervical canal, this is associated with bleeding..you are treatment with drugs, which is teratogenic to fetus.. hence if you plan for pregnancy, you need to be changed for the treatment of CKD.. Take care.."
},
{
"id": 56010,
"tgt": "What is the treatment for Hepatitis B infection?",
"src": "Patient: Good After Noon Dr.5 years back I have donated blood to my cousin test results were HbSAg positive. What should I do for this if I want it to be Negative.please advise me, is there any treatment for this how should I get relief from this.My Liver Kidneys & other test are Negative & don't have any kind of symptoms.Thanking You Kashif Doctor: Welcome at HCM i have gone through your query and being your doctor I completely understand your health concerns.. Yes you should immediately meet your doctor so that treatment can be initiated... Meanwhile get your family members screening done and particularly your wife if you are married. Get precautions during sexual intercourse Treatment usually varies from 6, months to one year . your doctor will decide about your case. Get well soon Regards Dr saad sultan"
},
{
"id": 96473,
"tgt": "Stomach ache and feeling of vomiting all day?",
"src": "Patient: Hello doctors and communities. last day i had a fever with head ache like... i cured from it with some medicine now i have no fever or head ache but i have a stomach ache is not a stomach ache because it hurts like very little and i feel like vomoting all day at school and i dont eat at lunch because im scared that i vomit at school so i just drink chocolate milk in there. last night at dinner when i finished eating i threw up but i didnt make myself throw up by the way.. so tonight it happened again..=( after i finished eating i felt like vomting for a while then i vomited again :( it felt horrible i hate it. am i going to die? do i have swine flu please help me! Doctor: Swine flu will have symptoms of flu with symptoms that you have and diarrhea. You will need to test yourself for swine flu to confirm/rul it out. YOu will also need to have a gastroenterology check up and endoscopy to see wht is causing the immediate vomiting.."
},
{
"id": 15069,
"tgt": "Persistent reddish brown, scaly, extremely itchy rash on shin with swollen and stretchy skin, used lotramin. Effective solution?",
"src": "Patient: I have had a reddish brown, scaley, extremely itchy rash on my shin for almost 4 months. It is located just below a scar from a previous car accident. The skin tends to swell and stretch in that area from edema making the itching worse. I ve tried lotramin for 4 months and it s still not gone. Doctor: Hi,You seem to have developed an allergic skin disease.You should avoid applying any kind of skin irritants like dettol, spirit over it.You might need to apply a topical steroid cream over the affected area twice a day for few days.Take some oral antihistamines along.See a nearby dermatologist if the condition tends to persist despite this therapy.You might need a course of oral steroids.Hope it helps,Dr Geetika Paul.MD(Dermatology)"
},
{
"id": 21343,
"tgt": "Does taking nicotine patch cause high bp?",
"src": "Patient: My mom suffers from high blood pressure but takes medication and that keeps her in check. From time to time it gets real high but not in a while. She was a 2 pack a day smoker but hasn,t smoked like that for almost 2 months but started back over a week ago having maybe 4 - 8 a day. Hasn'y smoked at all for a whole day and a half. She put a 21 mg patch on this a.m and her pressure is real high, like 184/119?? Is it from the patch? Doctor: Hello, I understand your concern. Your mom probably had high blood pressure unknowingly to her before she started using the nicotine patch. Using it when you have high blood pressure will worsen your blood pressure. I advise you to talk to your physician about this. Warm regards"
},
{
"id": 59821,
"tgt": "Weight loss, have calculus in gallbladder. What is the best treatment?",
"src": "Patient: of my father has a calcus in gal blader up to 1 cm .he looses his weight from 55 kgs to 45 kgs .say the best treatment Doctor: Hi, Weight loss without any specific cause is a dangerous sign. Your father should undergo immediate investigation to rule out any malignancy and there after get operated. With regards"
},
{
"id": 123174,
"tgt": "Should i be concerned as i am having spasmodic pain under my left shoulder blade?",
"src": "Patient: I have been having spasmodic pain under my left shoulder blade for several days now--non-muscular. I had a lap-cholecystectomy many years ago. It gets slightly better if I lean forward. No increase on movement or with inspiration, no coughing. Any cause for concern? Doctor: Hello, As pain around the shoulder blade is muscular. Due to inflammation of the insertion of the Trapezius muscle there will be pain. For which we call it Trapezitis. Along with this there will be a inter-scapular muscle spasm. This happens due to poor posture. Firstly need correction of the posture, next, we need to have hot water fermentation, gentle kneading massage, myofascial release, IASTM, neck static exercise, shoulder stabilization exercises, inter-scapular muscle strengthening exercises, breathing exercises. All of this will help reduce pain and spasm. Hope I have answered your query. Let me know if I can assist you further. Regards, Jay Indravadan Patel, Physical Therapist or Physiotherapist"
},
{
"id": 65150,
"tgt": "Suggest treatment for a lump on the mons pubis area",
"src": "Patient: Hello, just a few days ago I started to notice that I have a lump on my mons pubis. I first noticed it because it was a bit painful, though its not visible at all, but I can feel it. Now I almost don t feel anything anymore, but when I press around that area, it feels like theres a big something underneath my mons pubis. If it helps, I m a virgin, so I m not sexually active or anything like that. Doctor: Thanks for your question on HCM. I can understand your situation and problem. Mons pubis area is having a lot of hair. So possibility of skin infection is more in your case. You are also having pain, this also suggest skin infection. Possibility of infected sebaceous cyst or boil is more. So better to consult dermatologist and get done clinical examination and ultrasound examination of local part. You may need antibiotics. Better to avoid moisture and keep proper hygiene of private parts. Don't worry, you will be alright."
},
{
"id": 102807,
"tgt": "Red, itchy bumps on shoulders, elbow, arm. Not caused by clothes. Could it be food allergy?",
"src": "Patient: About a week ago, a cluster of red, itchy bumps appeared on my shoulders. I thought I might possibly be allergic to the sweaters I was wearing, or the chemicals on them, but after washing all of them, and not wearing them for a few days, another cluster appeared on my right arm near the crease of my elbow. The bumps are still on my shoulders and are more prominent on my left shoulder (always have been), and they are now on my right arm. I am concerned that this could be an allergic reaction to something I am eating, especially since my mother has food allergies, many of which did not appear until she was in college (as I am now). Doctor: HelloThanks for the query to H.C.M. Forum.May be due to .1 Scabies , diagnosis can be confirmed by scratch test from skin.2 Dermatitis , diagnosis can be confirmed by physical examination by a dermatologist .3 Urticarial rash , diagnosis can be confirmed by blood examination for eiosinophils count .In my opinion consult a dermatologist and get his opinion .Good luck. Dr. HET"
},
{
"id": 131532,
"tgt": "Is hospitalization required for non-displaced inferior pubic ramus?",
"src": "Patient: My mother is 92 petite-95# with carpal tunnel and osteoporosis, who fell laterally to the left and sustained a non-displaced L inferior pubic ramus Fx 3 days ago. She is unable to weight bear on her left leg, has mild pain in her left deep groin at rest, and a resolving bruise over her left femoral greater tuberosity. Assuming non-surgical treatment, does she need to be hospitalized for bedrest, nursing care - transfer from bed to chair etc, and initial PT, and then SNF for ongoing PT etc? Doctor: HiPubic ramus fracture may take 6 weeks to unite,seeing osteopenia,i t may take longer.If good nursing care can be provided,would be good.rest in bed is mainstay of treatment.Supportive treatment with calcium, vitamin D3, mild pain meds is all that is required"
},
{
"id": 173621,
"tgt": "Would taking 15mg of methotrexate safe for psoriatic juvenile idipathic arthritis?",
"src": "Patient: My 7 year old daughter has psoriatic juvenile idiopathic arthritis and is currently taking 15mg of methotrexate a week. Even though I she has had her chickenpox immunisation, her blood test show she has no immunisation to the virus. One of her friends has just developed chickenpox and my daughter has been exposed to the virus. She has no symptoms but she is booked into have injections into her joints tomorrow. Should I cancel the injections and do I need to take her to the doctor Doctor: those person who are on methotrexate therapy have risk of bad chicken pox. you can talk with your pediatrician as chicken pox can be prevented by varicella antibodies injection. In my opinion you should defer the next dose of methotrexate for 21 days from the exposure date of your daughter by her friend. best wishes for health"
},
{
"id": 132854,
"tgt": "What does the meaning of black spot on the leg x-ray?",
"src": "Patient: Hello my cousin had some leg pain and after some time she went to the dr.. She had xrays done and it showed a black spot on the bone. What does a black spot on the x ray mean? She is 28 with no kids and in very good shape and takes very good care of her health. the spot is close to her right leg joint. Doctor: hihope this msg finds u in good health. i have gone through the complaints and under stand your concern 1i feel the black spot seen on xrays in the bones are generally small benign cysts which might be congenital. Such cysts do not cause any harm over lifetime and u shud not worry .Hope this answers your question. if u have any follow up queries, feel free to consult me anytime. thanks Take care god bless"
},
{
"id": 217126,
"tgt": "Suggest therapy for neck & shoulder pain",
"src": "Patient: Hello I am 2.5months post partum. I had an epidural and got a spinal headache from it however do to the sudden onset (within 1hr from epidural placement) my neck was aching and hurting, after the epidural was removed it got much worse. Then the next day a little better and so forth. I have been back to work for 3wks (lots of walking) and I have developed the neck and shoulder pain again, along with foggy headed, and a throbbing headache, and neck is sore and stuff, vision feels more blurry. No fever and I can turn my head from side to side. Doctor: Please do a mri brain if possible contrast enhanced.immediately consult a neuromedicine doctor.epidural may nit be cause for this headache"
},
{
"id": 37155,
"tgt": "Suggest treatment for typhoid",
"src": "Patient: Hi Doctor, my wife is suffering from Typhoid for past 2 days, as prescribed by doctor we are taking tablets OFLOX-200 + Dolo + Cifran 250. Also she continue to take WOMENS HORLICKS with milk to feed 2 months old baby. Please advise : 1. Whether good to take womens Horlicks during typhoid. If no, advise which drink is good to increase mothers milk. 2. Best diet food to take during typhoid. 3. Is it ok to take CIFRAN 250 tab ? Doctor: Hello,I understand your concern.I am Dr. Arun Tank, infectious diseases specialist, answering your query.In my opinion you should continue with the women's Horlicks. There is no harm in taking it.You can take either tab. Cifran or oflox. Taking both is not essential.In typhoid high protein diet and fat free diet is adviced. Never dump intestine with heavy eating. It can cause a perforation in typhoid ulcer. Taking a small but frequent diet is adviced. Also do not eat spicy foods.I will be happy to answer your further concern, you can ask me on bit.ly/DrArun. Thank you.Dr Arun TankInfectious diseases specialist."
},
{
"id": 213535,
"tgt": "Episode of tingling in hands and feet with pressure and warmness in head, hot flashes during a bout of itch or stat of hunger. Panic attack?",
"src": "Patient: Hi, I m 23 yrs old and I have been expierencing tingling in feet and hands on a dime. Not the kind where they fall asleep. Also my head feels hot (inside) as rediculous as it sounds like my brain is melting. Start feeling pressure in my head. Hot flashes and frankly I feel like I m dying. Every time my body has a feeling i.e empty tummy or itch , these symptoms start. My fiance said panic disorder but it comes on so randomly everyday. Its unbarable! Doctor: yes it is panic disorder. consult a psychatrist. take tab librium. one tablet in night. your symptoms will be reduced."
},
{
"id": 92280,
"tgt": "What could be the reason for having bloating, abdominal pain and weight gain?",
"src": "Patient: Hi, I'm 28yr old woman. I start feeling sick to my stomach at night sometimes, I've gained weight, feel bloated and sometimes get pain in my lower abdomen and sometimes right side and right back side, and my urine is cloudy and smells kind of like coffe? Really weird Doctor: Hello dear,Pain in lower abdomen associated with cloudy urine can be due to Urinary tract infection. Investigations like Urine examination (routine & microscopy) and Ultrasound of Abdomen & KUB (Kidney, Ureter, Bladder) will be helpful in clinching the diagnosis.Regarding the sensation of bloating, it can be attributed to gas formation in the stomach.Symptomatic relief can be obtained with Pantoperazole preparations (to be taken half an hour before meals)And to prevent weight gain, it is also needed that you take certain precautions regarding your diet & lifestyle, such as:1. Take soft, plain, non-spicy foods2. Drink lots of water to maintain adequate hydration3. Avoid fast foods & gas producing food products like cabbage, etc.4. Avoid smoking & alcohol.5. Keep away stress, think positive.6. Exercise regularly & practice meditation.There is no need to worry, you will be fine.Wishing a good health.Take care."
},
{
"id": 221671,
"tgt": "What is the best medicine to avoid pregnancy?",
"src": "Patient: im 22. i havent been with a man for last 18 months. i got my last period in august . i missed my september period. i consulted dr, and took deviry 10mg 2 per day for 5 days. i took the last pill 2 days back , didnt get my periods yet. is there any chance that im pregnant even if didnt had any contact with a man for last 18 months? Doctor: Hi No chance of pregnancy, with out intercourse. Don't worry. One's tab deviry is over with in 7 to 10 days you will get periods. Not immediately after stopping tablets."
},
{
"id": 175795,
"tgt": "Suggest treatment for fever, digestion issues and sleeplessness",
"src": "Patient: my 2 year old son had very smelly poop 2 weeks ago. Now he has a high fever and farts a lot.....we called a doctor but he has no symptoms to be seen ( throat,ears, breathing ok). it s been 2 days we give him Tylenol to bring the fever down....he does normally though , does not look sleepy or else. What could he have that creates the fever and the digestion issues ? could it be related ? like a gluten issue ? Doctor: Hi...this looks like viral illness. This definitely doesn't look like a gluten problem.Once it starts it will take 5-7 days to completely get better. Unless the kid's having low urine output or very dull or excessively sleepy or blood in motion or green bilious vomiting...you need not worry. I suggest you use zinc supplements (Z&D drops 1ml once daily for 14 days) & ORS (Each small packet mixed in 200ml of potable water and keep giving sip by sip) as hydration is very important and crucial part of treatment.Regards - Dr. Sumanth"
},
{
"id": 104891,
"tgt": "Severe stomach pain near the belly button, pain near the rib. Have asthma. Headache, sneezing. Pneumonia?",
"src": "Patient: Hi, I have asthma . Tuesday night I ate a a restaurant and became sick that night. For the next three days I experienced severe stomach pain near the belly button that would come and go. Friday night I developed sinus symptoms that I believe has lead to a sinus infection . Headache , fever, stuffy nose, sneezing , yellow snot, etc. I think it s starting to go to my chest. I have a dry cough but it s not constant. I do have to clear my throat. I m also experience an ache in my lower right rib when I touch it. Could these all be symptoms of pneumonia or something else? I m not sure if the vomiting was related to my sinus issue. Doctor: Hello Melissa, Your initial symptoms of a stomach bug and the ongoing symptoms of sinusitis, headache, sneezing, cough could all be connected with a viral gastroenteritis followed by a chest infection. The pain in the rib is usually an inflammation of the lining surrounding the lungs, called pleura, and hence referred as pleuritis. I hope the stomach or gastrointestinal symptoms have now cleared up and if not, you will need a stool c/s followed by a Chest X Ray to ensure there is no pneumonia; and urine c/s (if there are urinary symptoms as well). Additional blood tests would be a complete blood count, liver function tests, CRP. The treatment will then depend on the results. Typically a viral gastroenteritis does not require any therapy, but you must ensure you drink a lot of water, take rest, have some anti-emetics (anti-sickness drug) if nausea is severe. Antihistamines such as cetirizine and paracetamol will help with the sinus symptoms. Hope that helps. Best Wishes."
},
{
"id": 9741,
"tgt": "Does amphetamine salts cause addiction?",
"src": "Patient: I have been using amphetamine salts 10mg 2X a day and I noticed hair shedding and breaking off about a week after. I was previously taking concerta and noticed the hair loss also. Is this normal? Will my body become immune to it eventually and the hair loss stop? When I dont take it even for a day my hair does not shed that day. Doctor: Hello, Yes, amphetamines are habit-forming. Hope I have answered your query. Let me know if I can assist you further. Take care Regards, Dr Kakkar S., Dermatologist"
},
{
"id": 33808,
"tgt": "Suggest remedy for cold and sore throat",
"src": "Patient: Good morning, I had a cold and my throat is sore. I feel like im getting over it. This morning I blew my nose an a big piece of mucus and a tiny blood sac Attached to it. Instantly my throat no longer hurt. My question is , is it possible to blow the bacteria/virus out of your nose? Doctor: Hello. Thanks for writing to Healthcaremagic.I give you some suggestions, by which you can get relief in cold and sore throat.Try to drink plenty of water( 10- 12 litre per day) in a day.Take Tab. Diamonic -DCA tablet for morning and evening for 5 days.If symptoms not relieve than you have to start antibiotics.I don't preffer antibiotic at starting.Thanks and Regards.Dr. Vijay"
},
{
"id": 218447,
"tgt": "What causes stomach upset and vaginal bleeding during pregnancy?",
"src": "Patient: thank you. I am 11 weeks pregnant and will be 12 weeks from tomorrow. I took a long walk on Friday with my husband for about 30 minutes. U noticed a little stomach upset and much later, some light bleeding in my vagina. What do i do. This is my second baby after 10 yrs and I am 42 yrs old. Doctor: Hello and Welcome to \u2018Ask A Doctor\u2019 service. I have reviewed your query and here is my advice. The bleeding you saw was due to over exertion due to long walk. It does not seem to be connected with stomach upset. You should taken rest. Have light diet & plenty of water with a pinch of salt & sugar to maintain hydration. Consult your gynecologist earliest for possible threatened abortion &needful treatment. Needful hormone replacement in time helps to control bleeding. Hope I have answered your query. Let me know if I can assist you further."
},
{
"id": 96212,
"tgt": "Should I be worried about my ultra sound results of lower abdomen ?",
"src": "Patient: Hello, I just got my results from an ulstra sound I had on my lower abdmonin the findings were as follows: 1) My right Kidney is 11.1 c.m is that normal size? 2) There is prominence of the left pelvocalyceal system, what does this mean? and what is it? 3)There is a post void residual of 9mls in the distended bladder measures 168mls, what is this? and what is a CT KUB to exclude ureteric calcus? please can you help me understand any of the above? thank you :) Kind Regards SOnja Doctor: Dear sonja the first two queries are just normal, the third means that urine that remains in bladder after voiding is 168ml. and the last is his advice that ct scan should be done of kidney ureter and bladder to rule out any stone. Hope things are clear now."
},
{
"id": 40562,
"tgt": "How can infertility be treated?",
"src": "Patient: Hii.. I have 23 yrs old and have PCOD.. got married 15 months before.. taking treatment for PCOD and doctor took follicular study(on 12th day my egg was 1.5 and 14th day it was ruptured) and had contact with my husband. My husband s semen analysis also good. My weight is 66.7/156 cm. My last period date is 4.2.2017 and taking ovarshield DS,folpris(folic acid), APCOD-Obis from 2nd day of my period and taking optogest SR 300mg from 14th day..Now I m feeling some pain in abdomen I don t know I have possibility to conceive or not. Becoz I it looks like period pain.. can you please help me to decide on this??? Doctor: Hello,The best way to decide, is to do an out for yourself and best is to get a blood Beta HCG done for yourself and if you have a missed period get ultrasound pelvis done for yourself after six weeks.Hope I have answered your query. Let me know if I can assist you further. Regards,Dr. Mandavi Rai"
},
{
"id": 75104,
"tgt": "Suggest treatment for cough and wheezing",
"src": "Patient: chest infection for so long now.. hi im a girl of 27yrs. 6wks ago i started gettin a really bad cough and bringing up green mucus stuff and was very wheezy went to doctors was given amoxicillin 500mg 3 times a day for a week and steriods. that didnt help so went back to doctors and told i had bronchitis so given another course of tablets and an inhaler as breathing was hard and still wheezy i dont smoke not around people that do and ive now finished all my tablets and still have this horrible couch, wheezyness and now having chest pains. im thinking of a visit to my a&e department as surley this is not right. please help... Doctor: Hello and welcome in HCMi cn understand your concernI would recommend you to do as follows:Chest X rayHemogramCRPExamination of the sputumAfter having the results the pulmonologist will advice you further for the treatment and the follow up.DR.Jolanda"
},
{
"id": 163754,
"tgt": "What is the prognosis of calcium deposit on a new born s brain?",
"src": "Patient: My neices newborn has calcium deposits on her brain caused by a herpes type infection contracted in the first trimester. The doctors are saying she probably has a syndrome beginning with ??C I wanted to look it up but she cannot give me any more information at present Doctor: Hello... probably your doctors are suggesting that your baby might have got congenital rubella syndrome.It is one of the many possibilities but as you have told that it starts with C I am trying to give a clue to you.Regards - Dr. Sumanth Amperayani"
},
{
"id": 122046,
"tgt": "What causes burning cold feeling in the arm and shoulder?",
"src": "Patient: I have been having positional (woke up with a twisted arm and it never went back) and pain problems in my left arm and shoulder for 2 yrs... I ve had all the scans done and nothing is found. I know have a new symptom, it feels cold on the inside? Almost a burning cold feeling, do you know what that could be? Could it be connected and is there something it could be that can t be seen in ct MRI or nerve conduct test on arms? Doctor: Hello, Your symptoms are suggestive of a possible pinched nerve. For this reason, I recommend consulting with a neurologist and performing nerve conduction studies. Hope I have answered your query. Let me know if I can assist you further. Take care Regards, Dr Ilir Sharka, Cardiologist"
},
{
"id": 85537,
"tgt": "Is second dosage of unwanted 72 dangerous?",
"src": "Patient: HAD SEX WITH MY GIRLFRIEND ON SUNDAY .. THEN she had taken unwanted 72 within 48hr after 7hr we again had sex .... i have told her not to take the second doze as its harmful and secondly with in 72hr there is no need of it .... is it correct plz tell me .. Doctor: Hello, I think she has overused, but not dangerous. She might be having irregular periods for the following months, but nothing else. Try not to repeat the dose again. Hope I have answered your query. Let me know if I can assist you further. Wish you the best recovery. Regards, Dr. Albana Sejdini, General & Family Physician"
},
{
"id": 53404,
"tgt": "What is the treatment for enlarged liver?",
"src": "Patient: My mom has been diagnosed with fatty liver, and she is up all night with dry heaves and no energy to move. She ends up in the hospital every other month or so, feels better for awhile and then it starts all over again. Is this normal for her condition Doctor: Hi,Alcohol ingestion and obesity are common causes of fatty liver disease. Advice for fatty liver:- abstinence from \"Alcohol\" - Low fat diet should be followed, avoid junk food and beverages- decreased oil consumption (oily food)- NO red meat- green vegetables should be ingested daily- use lemon juice (lemonade) once in a day- reduce weight if overweight/obese- \"recheck liver enzymes after 6 to 8 weeks\" and/or ultrasound.Hope I have answered your query. Any further questions are welcome. Wish you good health.Regards,Dr. Tayyab Malik"
},
{
"id": 126787,
"tgt": "What causes pain in the upper arm?",
"src": "Patient: I have pain and sore muscles in my upper left arm. No chest pressure or pain. Arm, muscle pain started out about 2 days ago, slight arm muscle pain and has increased so that it s hard to lift my left arm. Taking advil helps, but it s pretty sore currently. Doctor: Hi, It may be a simple musculoskeletal pain. As of now, you can take analgesics like Acetaminophen or Tramadol for pain relief. If symptoms persist, you can consult an orthopedician and get evaluated. Hope I have answered your query. Let me know if I can assist you further. Regards, Dr. Shinas Hussain, General & Family Physician"
},
{
"id": 65379,
"tgt": "Suggest remedy for lumps",
"src": "Patient: I have a small raised hard lump of skin on the surface of my left nostril - I saw a doc who said it looked like a corn and prescribed Fucidin cream. But can you get a corn inside your nostril? And how can I treat it? (I'm male 33, no other medical history) Doctor: Hi ! Good morning. I am Dr Shareef answering your query. If I were your doctor, I would advise you to avoid pricking your nostril if you do this. If it does not heal with a course of antibiotic/antiseptic, I would refer you to an ENT specialist for a proper clinical examination along with related investigations for a proper diagnosis and further management.I hope this information would help you in discussing with your family physician/treating doctor in further management of your problem. Please do not hesitate to ask in case of any further doubts.Thanks for choosing health care magic to clear doubts on your health problems. I wish you an early recovery. Dr Shareef."
},
{
"id": 55457,
"tgt": "What causes mouth blisters, swollen lymph nodes in neck with elevated liver enzymes?",
"src": "Patient: my son had been diagnosed with mono even though multiple blood test were negative. he has blisters in mouth and swollen lymph nodes in neck. all other symptoms such as vomiting, head aches, fatigue and loss of apatite have went away. it has been 10 days since first symptoms began. could this may have just been the flew. also consider that his liver enzymes were slightly elevated 7 days from today during wich blood test were administered. Doctor: Hi, dear I have gone through your question. I can understand your concern. He has some viral infection or flu. It leads to fever, fatigue, headache, vomiting and elevated liver enzymes. Every symptoms are common in viral infection. Nothing to worry about that. Just drink plenty of water. Your liver enzymes will come back to normal within few days. Hope I have answered your question, if you have doubt then I will be happy to answer. Thanks for using health care magic. Wish you a very good health."
},
{
"id": 14128,
"tgt": "What causes itchy rashes on the arms and legs?",
"src": "Patient: Rash red lines one inch or dotted line of rash itchy, prickly, hot at times. On arms, legs and neck. It started with blister looking now dried up. I thought it was poison ivy at first but then the rash has since started ( after first week ) on both arms and legs and it has been 3 weeks now. Doctor: Hello and Welcome to \u2018Ask A Doctor\u2019 service. I have reviewed your query and here is my advice. It seems that probably you may be having allergic dermatitis. - Levocetirizine daily at night till the control of itching - Azithromycine once a day for a week - Oral steroids in tapering dose - Avoid contact with suspected allergens Hope I have answered your query. Let me know if I can assist you further."
},
{
"id": 71125,
"tgt": "What causes shortness of breath after implantation of a pacemaker?",
"src": "Patient: My Dad had a pacemaker implanted two weeks ago. He was fine for a while but is now experiencing a dry cough and shortness of breath. He saw a doctor today who said he is not concerned and sent my dad home with cough medicine. The cough medicine has helped a little but the shortness of breath is getting worse. Any ideas? Doctor: Hello and Welcome to \u2018Ask A Doctor\u2019 service. I have reviewed your query and here is my advice. Your father has developed cough and breathing difficulty immediately after pacemaker insertion. So we should definitely rule out heart failure and Pulmonary embolism. Both of them are seen if pacemaker is not functioning properly. So get done 2D echo and CT pulmonary angiography. If both these are normal then no need to worry for these complications. Sometimes bronchitis can also cause similar symptoms. So get done PFT (Pulmonary Function Test). Hope I have solved your query. I will be happy to help you further. Wish you good health. Thanks."
},
{
"id": 207055,
"tgt": "Suggest treatment for panic attacks and insomnia",
"src": "Patient: I've been having major panic attacks for almost a year now, but they are getting worse. It started when I was trying to stop drinking every night to go to sleep b/c I have major insomnia. I have had 911 dialed and just about to push call in case I was going to pass out, but having a drink or a glass of wine seems to be the only thing helping me.... what can I do to help me get over this? Doctor: Hi dear. I had gone through your query. I understand your concern. Panic attack define as sudden rise in anxiety and it can be last for few minutes to hours. It can be treated with medicine like combination of SSRI and benzodiazapine. Peroxetine/ sertraline/ escitalopram can be beneficial. Insomnia can be treated with medicine like lorazepam and clonazepam.But do take advise from your treating doctor before taking the medicine. Still if you have a query then feel free to ask. Happy to help you further. Thank you."
},
{
"id": 8784,
"tgt": "Is there any treatment to improve complexion of a 1.5 year old a?",
"src": "Patient: hi My relative is one and a half years old (baby girl ). Her complexion is pretty dark. Face is wheatish but rest of the body is dark since birth. Her mother is fair by complexion and father is dark. Is there any way by which we can improve complexion of her body permanently? How long is the procedure? What are the side effects? How much does it cost? At what age can we go for it? Doctor: No except plastic surgery with too much cost and time."
},
{
"id": 153717,
"tgt": "Suggest possible treatment for throat cancer",
"src": "Patient: Hello Doctor my uncle who is from kerala he has throat cancer & he of 65 years Always remember, if pain is severe or incapacitating, please see your doctor / professionalHope this helps solves your query .Take care .All the best .Get well soonDon't hesitate to come back in case of any further query ."
},
{
"id": 2512,
"tgt": "When should i take a pregnancy test after missing a period?",
"src": "Patient: Hi ive been ttc for to months now, just have started my montly cyle but havent. Over the last week ive had dizzy spells feeling sick but havent ive got cold like symptoms but no cold lower backache but no stomach ache like what i get with monthly cycle. ive done hpt about 3 to 4 days ago but all came back negative. If i have concived when would be the best time to take a hpt and get a positive?Thanks. Doctor: Hi & Welcome.You can do most home pregnancy tests 3 to 4 days before your missed period but if you test too early, you are more likely to get a false negative, where the test says you are not pregnant but you really are.Doing the test at least a week after a missed period will give you the most accurate result. Results may also be more accurate if you do the test first thing in the morning, when your urine is more concentrated.Hope this answers your query."
},
{
"id": 135350,
"tgt": "Suggest treatment for persistent numbness in back caused by an accident",
"src": "Patient: Please tell me first, how much is your fees? The Drs online Always charge at least $25-100 payment. I can t afford it(sorry). I got hurt in car accident few yrs ago & lost job-No pension, nothing! Lay on back numbness, but no pains. I can(old) move easily,but do so..Sleep is 3-4 hrs/night. Lost some wt I went to Int.Med=Nml lab, but MRI l/s, thorax=5+ HNP ( old) now. It takes effort to walk,etc.. Thanks for helping me..MEDI-CAL Drs don t help here... Doctor: HiWelcome to healthcaremagicI have gone through your query and understand your concern. This question of your is posted in free question section so I want to clear you that you have to pay nothing for this. You got accident due to which you may have got injury to spinal cord due to which you may be having numbness in the part lower to injury and back. Mecobalamine is helpful in reducing numbness. Multivitamin and trace elements supplements are of use.Further medical advice can be given by seeing your investigation such as x rays and MRI. You can You can discuss with your treating Doctor about it.Hope I have answered your query.If you have any further questions then don't hesitate to writing to us . I will be happy to help you.Wishing you good health.Take care."
},
{
"id": 208669,
"tgt": "What is the treatment to be followed in case of ADD?",
"src": "Patient: I want to seek treatment for add. I and a 62 year old male who has suffered from being unable to focus in sports activities. For many years I ve tried every supplement I could find and have been unable to find anything that helps. Do you know of any doctor in the Sacrament are that specializes in this area ? Doctor: DearWe understand your concernsI went through your details. I suggest you not to worry much. Attention Deficit Disorder is a disorder and not a disease. Disorder becomes health or life problems if you take them seriously. Ignore them and be aware of them. Find your attention deficit areas and be aware. Never try to overpower them. Ignore them. Think that they are normal and are causing no problems to you. Slowly and steadily the effect on life reduces. You learn to live with it effectively. If you require more of my help in this aspect, Please post a direct question to me in this website. Make sure that you include every minute details possible. I shall prescribe the needed psychotherapy techniques which should help you cure your condition further.Hope this answers your query. Available for further clarifications.Good luck."
},
{
"id": 218483,
"tgt": "Is vaginal spotting while on birth control pills a symptom of pregnancy?",
"src": "Patient: I have been on birth control for 4 years now. I switched birth control brand three months ago. Last month I didn t get my period. This month I just spotted for a day. Does that mean I could be pregnant? I took a pregnancy test last month when I didn t get my period and it came back negative. Or is that normal? Doctor: Hello and Welcome to \u2018Ask A Doctor\u2019 service. I have reviewed your query and here is my advice. Some contraceptives ( progestrin only contraceptive ) have effect to cause irregular or absent periods . So absence & spotting at/ in between period can be explained. Please keep on taking the pills regularly at same time of a day- as advised.If you were on pills without any unadvised break in between , there are least chances of conception. Please be cool.Pregnancy test done after 8 days delay after expected date of periods would give reliable result.( In case you get + ve result, consult your treating gynecologist). Hope I have answered your query. Let me know if I can assist you further."
},
{
"id": 215457,
"tgt": "What causes pain in left side of abdomen extending down into the groin?",
"src": "Patient: i have a pain in my left side abdominal and extending down into groin. Hurts mostly when I move or sit in one place too long. It is not debilitating pain, just uncomfortable. Had a bowel obstruction 11 years ago and my last colonscopy was Nov 2014. Some slight rebound tenderness. Doctor: Hello, Most probably it could be a renal stone. As a first line management you can take analgesics like paracetamol or aceclofenac for pain relief. If symptoms persists better to consult a general practitioner and plan for an ultrasound scan. Hope I have answered your query. Let me know if I can assist you further. Regards, Dr. Shinas Hussain, General & Family Physician"
},
{
"id": 54619,
"tgt": "Can fnac liver biopsy cause fever and body aches?",
"src": "Patient: i had undergone fnac liver biopsy test three days back. i m having pain in my shoulders, headache,possibly on right side . i m having tooth ache and joint pain and fever. joints in my wrest,knee and ankle is paining more.i m taking 250 mg paracetamol for fever. after the test they didnt made me to remain in observation and they didnt check my BP . few days before they took blood test to know my blood clotting time etc. after the biopsy i came home and now i m suffering from fever and full body pain. is this because of the side effects of the biopsy or something else? please answer my question and make me to comfort to come out of my fear like anything. i will give my handfull of thanks to u if i get an answer. Doctor: HIIt's possible that you've developed an infection in your liver after the biopsy so you need to see a doctor right away for an ultrasound of the liver and a course of antibiotics"
},
{
"id": 145381,
"tgt": "Is it normal to have dent in head?",
"src": "Patient: Hi, I have a dent in the crown of my head. I can t remember when I first noticed it but I was wondering if it s normal and will it just be the shape of my head? I am 16, and I haven t came across with anyone that has it before. It s quite a deep dent. Thank you. Doctor: Hello. I have been through your question and understand your concern.The indentations are quite common and are not problematic at all. They tend to grow sometimes. The only problem is aesthetical. In such case a little surgery can solve permanentely this problem.Hope this helps.Please feel free using MHC again"
},
{
"id": 221883,
"tgt": "Is it safe to take Duphaston during 6 week pregnancy?",
"src": "Patient: Hello! I happened to be 6 weeks pregnant and according to my transvaginal ultrasound, my baby has a small sac. My Ob gave me Duphaston, 30pcs, to be taken 2X a day. Is ot safe for the baby? I heard that Duphaston was banned in the US because accoding to waht I read in the net, it causes fetal abnormalities. Is it true? I am deeply worried but I have no choice but to take it because it was prescribed by my OB. Many many many thanks! Doctor: Hi,Duphaston is given as it a progesterone hormone containing drug, this progesterone is required to maintain the lining of the womb, especially when there is a doubt about survival of the pregnancy. Which is why the doctor must have prescribed you the Duphaston.No, Duphaston does not cause any fetal defects. And it does not harm the baby. It causes abdominal bloating, breast tenderness in the mother when she is taking it.Hope this helps.Regards."
},
{
"id": 47214,
"tgt": "What are the risks of having dialysis?",
"src": "Patient: Doctor, I'm suffering from ESRD from 1 year. The ESRD was caused due to NSAID. Right now my creatinine is 11. However all other parameters (calcium, phosphorus erc) are within range. I do not have blood pressure or diabetes. I have been advised dialysis. What are the risks with dialysis and what stage should I opt for kidney transplant Doctor: helloThanks for query .You have been diagnosed to have End Stage Renal Disease (ESRD) with your serum creatinine being 11 which is very high compared to normal value of serum creatinine (1.2 ).ESRD is a progressive disease and persistent high levels of creatinine will lead to hypertension and hepatorenal failure .You should consult qualified Nephrologist for assessment and start Dialysis at frequent and regular intervals to keep your creatinine levels at acceptable minimum levels .There is no risk involved in dialysis and has to carry on regularly till you are prepared for Renal Transplant .With creatinine level of 11 you can opt for transplant now itself if matching donor is available .Dr.Patil."
},
{
"id": 195866,
"tgt": "Suggest treatment for low sperm motility",
"src": "Patient: I am 42 years old. Last five years me and my wife has been trying for baby. We did all tests and my wife's tests came out normal. My semen analysis showed low motility. My wife did get pregnant in March of this year after taking ayurvedic medicines, but had a miscarriage when only one and half month pregnant. Doctor said the baby's growth did not happen as required. Doctor: Hello,Genetic causes like the chromosomal problem are a common cause for repeated abortion. Hypothyroidism, diabetes, lupus are some other causes. Low sperm motility can be from excess stress, excess heat exposure, vitamin and mineral deficiency, smoking, alcohol etcetera.Hey Forte like a tablet for increasing sperm motility can be prescribed. Consult a gynecologist for examination and discuss your case.Hope I have answered your query. Let me know if I can assist you further.Regards,Dr. Parth Goswami"
},
{
"id": 134738,
"tgt": "Suggest remedy for severe leg pain",
"src": "Patient: My sister asked me to go here and try to find out what s going on with her . She is in constant pain. It doesn t matter where you touch her it causes her extreme pain. She says when someone touches her legs it feels like her bones are breaking. She hurts from head to toe. When she stands she says that it talks a few moments for her legs to get the message they have to move. She would have done this herself, but it s too painful. Thank you. Kathy Doctor: Hi Thanks for your query and Welcome. I am Dr Akshay from Fortis Hospital, New Delhi.Actually pain at calf/Leg region can be due to many reasons , commonly it is muscular in nature but it can also be due to Tendons, Nerves, Blood vessels etc.For me to actually answer your question in detail, i would have wanted a thorough clinical examination as there could be many reasons for your sister's leg pain and other associated symptoms.My advise to you for initial 2-3 weeks will be : - Ice packs application intensively with elevation of your limbs at night- Take a short course of anti-inflammatory medication as per your body's suitability- Stop your physical activity till pain completely subsides- Intensive physical therapy, training and proper muscle conditioning once pain has subsided- Slow resumption of physical activities with proper pre activity warm up and stretching of muscles and short increments in duration of physical activity each day as per pain tolerance.- Some blood tests like Serum Vitamin D and serum Potassium levels, ESR and CRP help in better diagnosis of such condition.Do not hesitate to contact me if you need any further assistance/SOSDr Akshay Kumar SaxenaConsultant Orthopaedics Fortis Hospital, New Delhi"
},
{
"id": 158178,
"tgt": "Non-hogkins lymphoma diagnosed. Had biopsy, bronchoscopy, gastroscopy on ears, shoulders for growth. Chemo required? Possible treatment?",
"src": "Patient: my husband has been diagnosed w non-hogkins lymphoma-had biopsy procedures-bronchoscopy,gastroscopy,and seeprolavicular, also both ears and by ear on face growth starting.we are not sure what stage yet.also biopsy done on upper rt shoulder area for growth removed,had this growth since childhood but gotten larger-has no smyptoms and has been told will have chemo.I guess we are not sure what to expect and until we know stage,what are your thoughts or suggestions re this matter. Doctor: HelloAs per the details that you have provided it seems he has NHL but stage requires a bone marrow report and also abdominal imaging ie.e is there disease below the diaphragmHe requires chemotherapy and overall amongst the cancers diagnosed and treated chemotherapy has a potential to cure NHLs and the patients genrally tend to do well but further details are required in your case as i already mentioned so also LDH levels and endoscopy reports,do forward the same ,take cared"
},
{
"id": 218221,
"tgt": "Is pregnancy possible without intercourse or orgasm?",
"src": "Patient: So I masturbated at 9am and I got my right hand slightly covered in sperm..I washed my hand after multiple times. Then later at 6pm my girlfriend came over and I rubbed her clitoris. I looked up that sperm usually die within 20 mins when there not in the right environment but I just want to make sure that she is safe from getting pregnant Doctor: Hello and Welcome to \u2018Ask A Doctor\u2019 service. I have reviewed your query and here is my advice. Since you say you washed your hands multiple times after ejaculation; ideally it should not happen. However, if semen from a recent ejaculate are in the fingers & the sperms are still alive & fingers are inserted into the vagina, the sperms can swim into the vaginal canal & cause pregnancy. If sperms from a recent ejaculate are on some toys or other objects & these are inserted into the vagina, pregnancy can occur if the sperms are still alive. The normal sperm life inside the vagina is 72 hrs tough it can survive longer in the right cervical fluid. Take care. Hope I have answered your query. Let me know if I can assist you further. Regards, Dr. Nupur K."
},
{
"id": 28017,
"tgt": "Should I continue Clavix and Acitrom having blood in urine and high INR values?",
"src": "Patient: My AORTIC VALVE was replaced 10 years ago twice in a span of 17 days since the first open heart surgery had gone bad on account of one of the suitchers had opened very next day. Since then, I am on CLAVIX 75 and ACITROM 1mg/2mg per day and the INR Values are ranging between1.15 to 1.58 and at times it shoots up to 13/14 when I start urinating blood. I am taking the following medicines besides CLAVIX 75 and ACITROM : - Remistar 5 mg - 1 BD,Lanoxin 6/7, Frusalac DS 1 OD, Amtas 5 1 OD, Arkamin 1 BD and Derriphylin Retard 150 mg 1 BD. Kindly advise if I should continue with CLAVIX 75 and ACITROM? Doctor: i presume that you have metallic prosthetic valve,i feel you may discontinue clavix .and stop temporarily acitrom if INR is above 13,also evaluate yourself for any urological abnormality causing hematuria.once INR returns around 2 you should restart Acitrom."
},
{
"id": 78223,
"tgt": "Suggest treatment for chest infections",
"src": "Patient: I have had this reccuring chest infectinno for the last 6 months..the first time I was prescribed Ammoxicillin... Second time i was given steriods and Co-Amoxiclav. I have had this irritating cough on and off for months and now have a full blown Infection infection again, with sore throat and sneezing, and for months have been having Angina type pain on my left side, I use a noitrolingual GTN spray but it never goes for more than a few minuites. I just feel so ill. Doctor: Thanks for your question on Health Care Magic. I can understand your concern. You might be having pleuritic chest pain and most common cause for this is lung infection. Your lung infection (pneumonia) is not 100% resolved. You are suffering since 6 months. So possibility of chronic lung disease like tuberculosis, fungal infection or atypical pneumonia is more. So better to consult pulmonologist and get done 1. CT scan of thorax 2. Sputum culture and sensitivity. 3. Bronchoscopy and BAL (bronchoalveolar lavage) analysis. All these are needed to diagnose the type of infection you are having. On the basis of this, you may need anti tubercular drugs, anti fungal drugs or higher antibiotics. Don't worry, you will be alright. Hope I have solved your query. I will be happy to help you further. Wish you good health. Thanks."
},
{
"id": 209916,
"tgt": "What to do if having bad memories of life like rape, drugs etc.?",
"src": "Patient: just want advise on my life as a child and everything I went through trying to write a book don't know how to start it about being raped by my father and shots fired to a bad merrage to drugs I went though more I did not do drugs but dealing with this in the family I have a book please help me maybe even a movie nancybeeder grandisland nebr nancy YYYY@YYYY 0000 and even more Doctor: DearWe understand your concernsI went through your details. I suggest you not to worry much. Basic things first. We all come across so many experiences about our life. Some less and some more traumatic. We experience in different ways according to circumstances and age and development. Trauma and emotions about bereavement of a loved one may not be the same for a 5 yr old, 10 yr old, 15, 25, 35, 45 etc etc. The difference in emotional attributes are called development. Secondly we gain positive lessons from these experiences. So far so good.The emotions run high when we are at the middle age. Because we think that we are matured. Actually we are not. We can't even understand ourselves.At your age, we start seeing things happened earlier critically. Unfortunately, we all see negative things and batter over them. Just go back. Count every positive things happened in your life from age 1. You will be amazed to see, those events shall be at least 100ws of pages. Instead of using these, we start lamenting handful of negative experiences. Funny, huh..Take my word for it. Enough is enough. Understand whatever happened was for your benefit. Everything benefited you positively. Live life today. Forget yesterday and tomorrow.Please post a direct question to me in this website. Make sure that you include every minute details possible. I shall prescribe some psychotherapy techniques which should help you cure your condition further.Hope this answers your query. Available for further clarifications.Good luck."
},
{
"id": 114959,
"tgt": "Suggest treatment to increase the hdl cholesterol",
"src": "Patient: Good after noon sir..I am working at UK. My hdl cholesterol is very low sir. I am use Rozavel10 tablets.....sir please give me answer and your suggestion... Thank you. Lalitha. Doctor: Hi, dearI have gone through your question. I can understand your concern. Your hdl cholesterol is low. It is a risk factor for coronary heart disease. You should take low fat diet with high amount of polyunsaturated fatty acids. You can also take drugs like atorvastatin or pravastatin to control your cholesterol level. Hope I have answered your question, if you have doubt then I will be happy to answer. Thanks for using health care magic. Wish you a very good health."
},
{
"id": 93237,
"tgt": "Taking cerazette birth control, missed pill, unprotected sex, ECP taken. Continue cerazette?",
"src": "Patient: . Doctor: Hello,Thanks for writing in.Nice to know that you have taken ECP after having missed your birth control when you had unprotected sex. I assume you have taken ECP within stipulated time period of your unprotected sex.You may please continue taking your regular cezarette birth control pill but please use another (preferably physical barrier method) for next one week if required.Hope this helps."
},
{
"id": 122094,
"tgt": "Is it normal to have stabbing pain after spraining my ankle?",
"src": "Patient: I sprained my ankle 2 weeks ago, today. I got xrays done and they say no fracture. I did the RICE treatment and when I felt ready, i did moderate excercise at the gym, with an ankle brace. I am getting stabbing pains in my ankle, and in the middle of the night, I get sharp pains. Is this normal? I am able to walk on it with minor pains. Doctor: Hello, Your syndrome seem to be related to the sprain. I suggest using anti-inflammatory medications such as Ibuprofen to relieve the pain. I also suggest using cold compresses for local applications. I recommend continuing using the brace to prevent further damage and rest. Hope I have answered your query. Let me know if I can assist you further. Take care Regards, Dr Dorina Gurabardhi, General & Family Physician"
},
{
"id": 110145,
"tgt": "What causes sharp pain on lower back and waist?",
"src": "Patient: I had a very sharp pain on my lower back/ waist were i coulndt walk anymore and my sromach was tendee and rwal bloated it look like if i was pregnant but i had a massage and im barely gettn better after 5 days of pain im doin a lil better i can bend now n stretch wat was the cause for that Doctor: Dear friend, we have a got a nerve which starts from our lower back area to the legs.so if we cause our back bones or pelvic bones to work or bend more then it causes sudden severe pain.also we have a cushion like structure between our vertebral bones.if this structure gets damaged due to overwork of backbone , nerve in between this vertebral bones feels pain.so that is precisely what happened in your case."
},
{
"id": 184871,
"tgt": "What causes a deep tear at the end of the jawline?",
"src": "Patient: Hi Doctor, There is a deep tear on the inside of my mouth at the end of the jaw where the upper & lower jaw meet. It has some discomfort to offer me on opening the jaw. My mouth doesn't open fully. Please tell me if there is anything to worry. My age is 49yrs. I am 5 ft 5 inches tall. My weight is about 75kgs. I am otherewise fine having no concerns. Regards, Mamta Doctor: Hello,I have read your inquiry. I am concerned that you describe a deep tear and do not tell me the circumstances that caused this condition. If you have had trauma to the jaw and it does not function properly, please consult an oral surgeon. You may have damage to the TMJ joint.If you have a cut or ulcerated conditon with inflammation, you can take steps to reduce inflammation and reduce discomfort. If the area is infected or gets worse, please seek professional care with your dentist or oral surgeon.Rinse with warm salt water to cleanse and soothe the area. Avoid any irritants such as smoking, alcohol, acidic foods and liquids, spicy, crusty and hard foods. Do not stess the jaw and keep a soft diet. Keep well hydrated to assist healing. Follow proper nutrition with balanced vitamins. Vitamin B complex is helpful to healing. There are gels available with benzocaine or other anesthetic medications to offer relief if you experience great discomfort. Take some anti-inflammatory medication such as Tylenol, Advil or Benedryl. Follow good daily dental hygiene.Plan to visit your dentist if area does not improve in afew days or condition becomes worse. You may need prescription medication such as an antibiotic to treat an infection. You may have an undiagnosed dental condition contributing to this damaged area. A prescription rinse combination of pain relief, anti-inflammatory medication and anesthetic may be very helpful with the location of your damaged area. Anti-fungal medicine can be included in the rinse if needed.Thank you for consulting Healthcare Magic. I am available for additional questions. I hope you find my recommendations helpful and will feel better soon."
},
{
"id": 219807,
"tgt": "What are the symptoms of potential pregnancy?",
"src": "Patient: Hi, My name is Noemi. Well I had my last regular period the 21 of april and had unprotected sex the 25 of april and then got an unsual pinkish redish period it lasted for 4 days but not like regular period...could i be pregnant? and if not why did i have the unusual spotting/period Doctor: Please do get the pregnancy test done to know whether you are pregnant or not. Spotting can occur during early pregnacy sometimes."
},
{
"id": 177521,
"tgt": "Suggest treatment for abdominal pain and vomiting",
"src": "Patient: granddaughter woke up 3am with abdominal pain and vomiting together. After vomiting for an hour still had fairly high abdominal pain. had 2 liquid stools at approx. 5am without vomiting. slept till 7am with abdominal pain and 1 liquid stool. slept till 10am. noticed light red blood in stool today approx. 10 times today small amounts. still complains of generalized abdominal pain. taking a few sips of water. Just had another liquid light red watery stool. no fever. granddaughter is 8 years old. Had a cochlear transplant last Friday. Had taken ibuprofen every 6 hours Saturday only. then 1 time sunday and 1 time Monday. none today. Doctor: Hi....It seems your kid is having acute watery\u00a0 diarrhea. Unless the kid's having low urine output or very dull or excessively sleepy or blood in motion or green bilious vomiting...you need not worry. I suggest you use zinc supplements & ORS...This is not related to cochlear implant. But red color in stools could be blood. If this is the case, she might need antibiotic therapy.Hope my answer was helpful for you. Happy to help anytime. Further clarifications and consultations on Health Care Magic are welcome.Dr. Sumanth MBBS., DCH., DNB (Paed)."
},
{
"id": 56617,
"tgt": "Suggest methods to control high AST levels",
"src": "Patient: Hello. I am a 24 year old Male that was diagnosed with Mononucleosis one week ago. My initial AST levels were 620, ALT at 655, Alcphos 263. They have dropped to AST 206, ALT 512, Alcphos 233. I would like to know how long it will take for my liver enzymes to go back to normal, and to make a full recovery, and when can i have a glass of wine? Doctor: HelloWelcome to HCM Thanks for posting your query. I understand your concerns. As you are recovering from mononucleosis infection, it would take 3 months for the complete recovery. The enzyme like AST and All are elevated whenever there is injury or damage to liver cells. So it would take around 10 to 12 weeks for the liver tissue to recover. Till then beware of habits that cause injury to liver. Hope you are happy with the answer. Thank you"
},
{
"id": 112925,
"tgt": "Have back pain. MRI shows diffuse annular bulge indenting on thecal sac. Have flat foot. Need advice",
"src": "Patient: hello dr, my son is 14 yrs old and was suffering from backpain for many years and his mri report says 1. diffuse annular bulges of l4-5, l5-s1 iv discs, indenting on thecal sac, l5&s1 perineural fat.2. diffuse annular bulge of l3-4 iv disc indenting on thecal sac and Schmorl's nodes in l2-3 vertebral end plates. Also, he is having flatfoot.my family physician suggests some exercise and ir fomentation at back lumbar region. needs correct advise Doctor: hallo my friend i read your complete history you need some neurotropics like pregabalin and calcium suppliment with vit D take rest , take some analgesic , hot or cold fomentation avoid lifting weights sit with taking support to your back continue spine extension exersizes for flat foot there are specific exersizes you can contact me for that will need arch support also"
},
{
"id": 19565,
"tgt": "What causes fluttering in head and dizziness while on treatment for cholesterol/BP?",
"src": "Patient: Yesterday I had a fluttering feeling in the left side of my head followed quickly by dizziness and nausea; I almost passed out. When the fluttering sensation subsided I noticed that my vision was blurry. The dizziness and nausea lasted several minutes and the blurred vision lasted several hours. I m 54, male being treated for high cholesteral and blood pressure and I m over-weight. Do I need to see my doctor? Should I be concerned? Doctor: Hello!Welcome and thank you for asking on HCM!I understand your concern and would explain that your symptoms could be related to a cardiac arrhythmia. For this reason, I would recommend consulting with your attending physician for a careful physical exam and some tests: - a resting ECG a cardiac ultrasound- a chest X ray study- complete blood count for anemia- blood electrolytes for possible electrolyte imbalance- thyroid hormone levels to exclude thyroid gland dysfunction. An ambulatory 24-48hours ECG monitoring would help examine your heart rhythm for a prolonged time and exclude possible cardiac arrhythmia. Hope you will find this answer helpful!Kind regards, Dr. Iliri"
},
{
"id": 181414,
"tgt": "Is taking an Amoxicillin course necessary prior to tooth extraction?",
"src": "Patient: Hi my name is master I'm down here in tampa Florida and I've been going to the dentist beaches I have two tooths that I have to get pulled so they prescribed me. Amoxicillin and I just took a bc powder and. Its about time for me to take the amoxicillin but I have second thoughts on taking it Doctor: Thanks for contacting healthcare magic. Before extracting a tooth antibiotics are given to decrease infection.During extraction time....you have lack pain and our anesthesia work properly.In extraction procedure dentist use a local anesthesia to anesthetise a tooth.In infection condition anesthesia is not work properly. so during procedure you may feel pain or discomfort. This amoxicillin antibiotics are given to decrease infectious condition.Take proper course of antibiotics and consent a dentist.I hope my guidance is helpful to you. Take care.Thank you very much. Dr Rachana"
},
{
"id": 61039,
"tgt": "What does a lump on the shin indicate?",
"src": "Patient: This morning after doing some therapy for my injured knee (my trainer said i had tendonitis and my doctor said i have tissue damage so im not sure what is exactly wrong with my knee) i realized that i had a large bump on my upper shin or right under my knee. there was no bruising, redness, or sensitivity to touch but hurt pretty badly and the pain got worse throughout the day. The lump is over an inch long, is circular, and stiff/hard. as i lay in my bed right now, it hurts a significant amount more than this morning. there is also a tingling feeling or the area feels like it \u201cfell asleep\u201d. The swelling has gone down but the pain keeps increasing. what might this mystery lump be? Doctor: Hello dearWarm welcome to Healthcaremagic.comI have evaluated your query in details .* Possible hematoma after injury .* Needs ultrasound examination for further management .Hope this clears your doubt .Wishing you fine recovery .Welcome for any further assistance .Regards take care .Dr. Bhagyesh ( MS , FMAS - consultant surgeon )"
},
{
"id": 104043,
"tgt": "Recurring nose bleed. Coughed blood. Allergic to pollen. Normal after nose bleed?",
"src": "Patient: Hi, I had a recurring severe nose bleed for about a week. Lost a lot of blood. Was examined by an ENT, who found a bleeding anterior artery in my right nostril, and it was cauterized. It stopped the bleeding, but obviously, I couldn't blow my nose for a week. It's allergy season, and I'm very allergic to pollens. Started to wheeze. I've never wheezed before this. Yesterday I was able to gently blow my nose, and the wheezing stopped. This morning, I coughed up clotted blood. It wasn't much, but it alarmed me. Is that a natural occurrence after such a large nose bleed? Is it just blood that went to my lungs from the nose? Should I ignore it and just wait and see. Otherwise, I feel fine, just congested from allergies. Thank you. Doctor: Hi and welcome to HCM. Yes, this is normal for nose bleeding because usually some amount of blood comes into tho mouth so this isnt actually coughing. YOu should take antiallergic medicines in seasons of pollens. There is no specific prevention of nose bleeding but it usually stops spontaneously. If this persists you should check blood clotting factors and blood pressure to see is there any disorder. Wish you good health."
},
{
"id": 113074,
"tgt": "Back pain on lower part of spine, lump on spine, pain on touching, sitting. Cure for pain?",
"src": "Patient: Hi I ve had a very bad back pain recently. It s a lower and middle part of my spin. Basically really hurts in the mornings when I wake up it hurts when I breath or yawn can t lie on my back only on side. And found a lump on my spine in the middle part that extremely hurts when you touch it even when I sit and touch the chair it hurts. Any ideas please? Doctor: Hello. Thanks for writing to us. The painful lump on your back could be due to a local infection in the area like abscess. You need to get the area examined by an orthopedician for a proper diagnosis so that proper treatment can be started. I hope this information has been both informative and helpful for you. Regards, Dr. Praveen Tayal drtayal72@gmail.com"
},
{
"id": 64957,
"tgt": "Suggest remedy for lumps in armpit",
"src": "Patient: I have a grandson that has had a history as his father and grandfather has with lumps under their armpits from certain deodorants but recently my grandson, just turned 18 has developed lumps like ones under his pits on his chest. He's had Mono but supposedly is over it. Can the lumps be caused by the mono? Doctor: Hi,Dear I went through your query.1-By the facts placed by your query,I feel that this lump under the armpit appears to be due to the mono.2-This lump is not like the ones like his father and grand father-after de-0derants. and needs FNAC/ biopsy of the armpit lump. -It could be due to even TB,but needs FNAC biopsy to to give any reasoning opinion on it.3-Plz check your your ER- doctor for second opinion . 4-Hope this would solve your query,5-Thanks for your query to the Health Care magic. 6-You are wellcome for any further treatments if required.7-I would love to help you out of YOur armpit lumps. .Thnks.Wellcome."
},
{
"id": 155879,
"tgt": "Should I wait for 3 weeks to undergo Thyroid cancer?",
"src": "Patient: Hi Doctor yesterday my doctor told me that i have Thyroid cancer but i have to wait about 3 weeks for the operation and about 3 years ago i had breast cancer too but it was just a small lump i am ok about that what you think is 3 weeks to long to wait thanks doctor Joyce. Doctor: HIWell come to HCMI really appreciate your concern, either this is secondary or primary malignancy if this is not severe enough or does not have any metastasis, and other vital organs are intact then you can wait for 3 weeks its okay, malignancy is very slow progressing disease, sudden onset of emergency is very rare, hope this information helps you, take care."
},
{
"id": 75110,
"tgt": "What causes tingling in forearms/legs after having steroid injection for pneumonia?",
"src": "Patient: I went to the doctor today and found out that I have pneumonia. I got a steroid injection and breathing treatment in the office and was sent home with levaquin, albuterol, and cough medicine. My forearms and legs are very tingly and almost numb and heavy feeling. Is this from the steroid injection? I am 23 years old and I have juvenile rheumatoid arthritis so I am also taking humira, methotrexate, plaquenil, and folic acid for the RA. Is this a side effect or a drug reaction or nothing to be concerned about? Doctor: Tingling in extremities may be a sign of peripheral neuropathy, wich is most of the cases caused by inflammatory injury to the nerves. Anti-TNF drugs, such as Humira, have been reported to be associated to this kind of injury, that can be potentialized when associated to steroids. You should visit your attending rheumathologist for physical evaluation and apropriate management."
},
{
"id": 146016,
"tgt": "What does the following MRI findings mean?",
"src": "Patient: What does the MRI findings here mean? FINDINGS: . Alignment: Normal. . Vertebrae: No marrow signal abnormalities to suggest neoplasm. . Spinal cord: Normal signal and contour. . Cervicocranial junction: No significant focal abnormality. . C1-C2: No significant focal abnormality. . C2-C3: No significant focal abnormality. . C3-C4: No significant focal abnormality. . C4-C5: No significant focal abnormality. . C5-C6: At C5-C6 there is eccentric disc bulging and disc osteophytic ridge complex formation eccentric to the left which likely narrows the left lateral foramen and may impinge upon the anterior nerve roots at that level.. . C6-C7: At C6-C7 there is a similar appearance of eccentric disc bulging and ridge formation eccentric to left narrowing the left neural foramen and impinging upon the left ventral nerve roots.. . C7-T1: No significant focal abnormality. . Upper Thoracic Spine: No significant focal abnormality. . Contrast: No abnormal enhancement within the spinal canal to suggest neoplasm or infection. As described in the concurrent brain study there is impingement and indentation on the left lateral surface of the medulla by the left vertebral artery . Doctor: Hello. I have been through your question and understand your concern.It is very important to know the indication why you had the spinal MRI. Whereas the findings suggest foraminal narrowing on C6 and C7 nerve roots. The symptoms one can expect from such findings are neck pain which irradiates in the medial part of the left hand and forearm w/o sensation problems like tingling and numbness in the first three fingers.You should also know that these findings in the MRI are very common and not necessarily related to the above symptoms. If you have no such complain you should consider a normal MRI and not worry about.Still, a neurosurgical consult would be appropriate to see if there is any correlation between the MRI findings and your complaints. Hope this helps, please feel free for further question."
},
{
"id": 96919,
"tgt": "Is it necessary to take anti-rabies shots after stray cat bite?",
"src": "Patient: is it necessary to take anti-rabies shots? I was fishing in a pier and a stray cat ate my bait(hook and line), I pulled the line until it broke -off. Unknowingly, I tied another hook on the line and licked the line to moisten it before cinching the hook. I remembered too late and I spat out for a few seconds. Do I need an anti-rabies shot?.. Thanks.. 39yrs old, 5 6 ,250lbs, asthmatic(seasonal) but with an anti-tetanus vaccine for 5 years(Got the shots recently). Doctor: hi iam dr.nallammaiwelcome to hcmanti rabies vaccine are advisable fr both dog and cat bitesas it is a stray cat i would advise u take IDRV, which is harmless, doesnt cause any adverse reactionsprevention is better than cure, so take full course of IDRV without fail"
},
{
"id": 132227,
"tgt": "Need treatment for pain in the shin",
"src": "Patient: I smashed my shin about 2 weeks ago & now that the swelling is going down, i can feel what I would describe as a dent in the bone. It doesnt hurt when I stand on it, but if I walk on it by the end of the day it is sore. If I elevate my legs horizontally on a table & try to let the heel support the weight, I can feel alot of soreness in the spot where I feel the dent. Was hoping just a deep bone bruise. Doctor: hi What you have probably is a bruise in the soft tissues covering the bone or bone. Shin being sub cutaneous bone is more prone to bruised .The dent you are feeling is not a dent in the bone but in the soft tissues and the swelling must be increasing in the evening causing more soreness. Use cold compresses and a creppe bandage to support the bruised area . It might take 3-6 weeks for full recovery"
},
{
"id": 168849,
"tgt": "Suggest remedy for abdominal pain in a child",
"src": "Patient: My 6 yr old son has been complaining off & on for a couple weeks that his belly hurts. He now has a couple rash looking spots on his belly & also on his cheeks. No fever or any other symptoms. Wondering what it could be & if we should take him to see our dr or let it run it s course Doctor: Hi Dear,Welcome to HCM.Understanding your concern. As per your query your son is having symptoms of abdominal pain. Well the symptoms you mention in query can occur due to many reasons like food poisoning , food allergy , infection of digestive system, acid reflex , stomach flu or colic. Need not to worry much. I would suggest you to have quietly sips of water or other clear fluids. You should give child oral rehydrating solution as well. If condition doesn't get better then consult pediatrician for proper examination. Doctor may prescribe antiemetic drug , antibiotics ,analgesic and antacid. Avoid solid foods for a few hours try small amounts of mild foods such as rice, applesauce, or crackers and do not give your child foods or drinks caffeine, carbonated beverages, citrus and dairy products.Hope your concern has been resolved.Get Well Soon.Best Wishes,Dr. Harry Maheshwari"
},
{
"id": 135663,
"tgt": "Is it normal to get cramps in leg after chemo treatment?",
"src": "Patient: My husband recently received a chemo treatment for CLL. He now has low energy which is understandable but when he stands he has cramps in his legs from his knees down the back of his legs. He has never had that before with his first chemo treatment 20 months ago. What does this mean? Doctor: HiWelcome to healthcaremagicI have gone through your query and understand your concern.It may be possible to have cramps following chemotherapy because it can cause vitamin deficiencies leading to generalised pain. Vitamin supplements and analgesic are of use. You can discuss with your doctor about it. Hope your query get answered. If you have any clarification then don't hesitate to write to us. I will be happy to help you.Wishing you a good health.Take care."
},
{
"id": 126939,
"tgt": "What causes stiffness in the ankles?",
"src": "Patient: Hi Dr. Ry I am a diabetic for about 15 years. Recently I have noticed that the bottom of my have been number which is neuropathy. Any way I have been getting a tightness in my left ankle and on the right side of the same foot. Also when at tI move my ankle, the joint hurts. Any idea what it may causing it. Doctor: Hi, The joint pain is not related to diabetes. You may have diabetes neuropathy if you have pain in the foot or feel cold or hot the extremities but not joint pain. A detailed clinical examination is needed. Hope I have answered your query. Let me know if I can assist you further."
},
{
"id": 171648,
"tgt": "What to do if child is diagnosed with astigmatism?",
"src": "Patient: hi.just recently my 2 year old son was diagnosed with astigmatism in both eyes as what as the pedia optha said to us.doctor said 500 and 400 in both eyes,is it too high?we went to other doctor for second opinion and he said that my son can see well,he observed his activities and it seems that it is not blurred.now,we are confused on what to do.the first doctor prescribed eyeglasses for him..the second one said that he cannot get the accurate grade because my son does not cooperate on other tests and eye exams coz he just cries when being force to do it.please help me answer my questions.tnx Doctor: its difficult to do detailed eye examination in a 2 year old but many tests can be done in modern centres .eye glasses for a 2 year old needs cajoling and using methods to make them keep it on .It is not easy .Moreover the glasses for astigmatism are different type and can cause other symptoms Astigmatism the grades that you mention i take it moderate.My advise will be to keep regular appointments at ophthalmology centre for children and have a single centre periodic evaluation .May be modern methods of Laser treatment suggexted as he gets a little older or complications of squint or short sightedness develop."
},
{
"id": 10680,
"tgt": "Suggest shampoo that can be used for daily head wash",
"src": "Patient: Hi sir, I Had a hair fall for the long time.. Initially i got frustated a lot, but right now i am not bothering about it much as falling becomes as part of my life.. But the thing is i am maintaining the hair with some extra care.. I have used 1.Renocia Hair revitalising shampoo 2. Sebamed re-vitalizing shampoo . Both are seems good but i am confused which to prefer. Coming to hair type.. Mine is purely silky, and had a habbit of daily head wash.. With the information i provided could you please suggest me a good shampoo which i can use it for daily bath for ever... As i feel renocia shampoo is making my hair more silky... And i want one more information.. I am planning to apply oil in the nights and to have head eash in the morning.. Will it improves hair strength? The above shampoos are good for oil head wash...? Can i use any shampoo which is available in the market for daily use and use the above shampoo for twice in a week???? Doctor: Renocia hair shampoo is a good choice...For the dandruff you may use an antidandruff shampoo like sebowash or 8x shampoo once/twice a week...Applying oil on to the hairs will only improves the texture of the hairs... massage with fingers with or without oil will be more beneficial as it increase the blood circulation and in turn hair growth...Frequency of hair wash depends on the individual type of hairs, the sole purpose is to keep your hair dirt and oil free which may lead to hairfall..."
},
{
"id": 194450,
"tgt": "What are the side effects of masturbation for a long time?",
"src": "Patient: dear doctor i m 15 year old and i m watching sex videos due to biology internet links . and that makes my penis to grow up and hard and then i cant stop my self from doing masturbating . and due to a lot of masterbating my penis is becoming a little tword my left side like its turning left . so how can i make it straight again Doctor: Hello, Masturbation is never unhealthy and it is normal and natural. There are no long term side effects for masturbation if you do it moderately. Hope I have answered your query. Let me know if I can assist you further. Take care Regards, Dr K. V. Anand, Psychologist"
},
{
"id": 58051,
"tgt": "Diagnosed with 2 liver hemangiomas. Any danger in hemangioma bursting and hemorrhaging?",
"src": "Patient: I have been diagnosed with 2 liver hemangiomas. The biggest one is 3X3X2 CM in the right lobe. I have had pain for 3 to 4 years now, and my Dr is finally coming around after shrugging his shoulders for any questions that I ask him. Now the pain is radiating to my right back flank. Lately I have also been getting these pin prick sharp pains directly under my right rib. With 4 ultrasounds and one CT scan showing everything else is okay. Last night I thought I almost had to go to the ER, but I took a Bentyl that helped.Any danger in the hemangioma bursting and hemorraging? Doctor: hi it depends on the size of hemangioma if big it come compress surrounding leading pain if taking alcohol kindly stop it"
},
{
"id": 107472,
"tgt": "Suggest treatment for severe lower back spasms",
"src": "Patient: I have a lower back injury, and do not know how it happened. I have had lower back spasms in the past. About every few months that will last only a few hours. This time i felt fine, until i bent over and it felt like a knife was stuck into my lower spine. Paint at 9/10. At that point it was very painful to stand back up. Throughout the rest of the night movement in any way was painful, and it was impossible to get comfortable in bed to sleep. The pain is not as severe today, about a 7/10. But movement in any way is very hard and painful. The pain is located on my lower spine, and the muscles right around the area. Occasionally, the upper part of the butt muscles feels like it is pulled. Doctor: Hi,Low back pain is sometimes very painful and may interfere with your daily activities. Rest with hot compresses on your back and the avoidance of lifting heavy objects is the first line you may do. Taking some NSAIDs, muscle relaxants, whether topical or in tab forms are very helpful, topical gels with yoga is highly recommended at your stage.Sometimes you may need intramuscular injections of steroids to decrease the inflammation around your nerve root, pregabalin and B12 supplements are effective with the neuropathic pain. If all these fail you may ask your specialist for more investigations or even minor endoscopic interventions or even spinal surgeries are needed in some late cases so I insist you go to your specialist for better management of your case.Hope I have answered your query. Let me know if I can assist you further.Regards, Dr. Ahmed Aly Hassan"
},
{
"id": 192193,
"tgt": "What causes stinging in penis during and after urination/ejaculation?",
"src": "Patient: Hi, may I answer your health queries right now ? Please type your query here...Ok, here's the deal, my penis stings but only the passage inOk, here's the deal, my penis stings but only the passage in which urine or semen exit from. It stings during and after urinating and ejaculation. It started very subtle but gradually after a few days hurt so bad it sent a shock wave through my body. Any ideas? Doctor: Hello,It may be due to urinary tract infection. For that you may require complete hemogram, urine analysis and ultrasound abdomen after urologist consultation. Further treatment mainly depends upon the underlying condition. You may require antibiotics after consultation.Take care. Hope I have answered your question. Let me know if I can assist you further. Regards, Dr. Shyam B. Kale, General & Family Physician"
},
{
"id": 123929,
"tgt": "How to treat pain in my right leg below knees and burning sensation?",
"src": "Patient: Hello Doctor, I had a herinated disc and got operated L4/L5 with endoscopic dorsal lateral entry ( Left side ). Even after surgery I am having pain in my right leg below knees and burning sensation. Why I do still have apin after surgery? Is it normal? Thanks and Greets Doctor: Hello, It's not common to have pain post the surgery for the herniated disc. It is only in few due to muscle weakness of the spinal column. In surgery the herniated part is removed which belongs to the inter vertebral disc . I will advice you to start the physiotherapy so by use of therapeutic ultrasound therapy the inflammation of will be reduced and TENS therapy to reduce the radiating pain. Once the pain falls below the level 5 in Visual analogue scale of 0-10 then exercise should be initiated with lower intensity. Initially focussing on the core stability and later on the lower limb muscles as whole. If the desired strength is not achieved then it might lead to trouble to the intervertebral disc of the upper or the lower levels. In my clinical practice of over 12 years, most cases post op are advised with Physiotherapy to regain the full functions of the muscles. Hope I have answered your query. Let me know if I can assist you further. Take care Regards, Jay Indravadan Patel, Physical Therapist or Physiotherapist"
},
{
"id": 210885,
"tgt": "Is craziness caused due to zoloft medication?",
"src": "Patient: Why is it when I take my mental meds such as Zoloft 100mg I ve been on it for a while I do have a lot of issue at hope child with disabilities and other things I put everyone first , I have seizure and no more ca. But I feel like my mind is still frakin raicing and talking 100miles per hr that I have to put head phone on with music to tune it out . What s up with that, am I going crazy? Doctor: Hello,Thanks for choosing health care magic for posting your query.I have gone through your question in detail and I can understand what you are going through.Zoloft is an SSRI and the moment you start this medication there is a serotonin boost in the body. It may cause this confusional state in some but over a period of timeif you are tolerating the side-effect then there is no need to worry. However I fail to understand why does it happen this often that you have to restart the medicine. It is not one of the wise things that you should be doing. Its always wise to stick to your medicines regularly.Hope I am able to answer your concerns.If you have any further query, I would be glad to help you.In future if you wish to contact me directly, you can use the below mentioned link:bit.ly/dr-srikanth-reddy\u00a0\u00a0\u00a0\u00a0\u00a0\u00a0\u00a0\u00a0\u00a0\u00a0\u00a0\u00a0\u00a0\u00a0\u00a0\u00a0\u00a0\u00a0\u00a0\u00a0\u00a0\u00a0\u00a0\u00a0\u00a0\u00a0\u00a0\u00a0\u00a0\u00a0\u00a0\u00a0\u00a0\u00a0\u00a0\u00a0\u00a0\u00a0\u00a0\u00a0"
},
{
"id": 102001,
"tgt": "What causes static feeling of nose?",
"src": "Patient: Hi my nose is feeling static often and I would like to know why?I suffer from vasamotor rhinitis (minute temperature change, air pressure and stress influence) and allergic rhinitis, and wake up with congested nose, headache, dizzyness and nausea about once a week. Is it to do with this? Doctor: Hi, you are quite write that vssomotor rhinitis can be aggranvated by several stimuli including changes ig temperature , humidity, barometric pressure , smeel of fumes, pollutant, exitement, emotional stress , anxiety, spicy food, alcohol intake etc. it can be present in mixed form with allergic rhinitis. you should try to avoid all thhose factors as much as possible. you can use nasal spray of any one of these for symptomatic relief like azelastine , olopatidine, fluticasone propionate or budesonide nasal spray. wish you all the best, thanks."
},
{
"id": 111563,
"tgt": "What is the remedy for reddish discharge with back pain?",
"src": "Patient: Am sexuaky active am on family planning g pills but I don t take them everyday after I stopped taking them I had my periods they are finishing today but I have a pinkish reddish discharge with slight back pains and cramps and for two months now my periods have been irregular please help Doctor: May you have renal stone or dysfunctional uterine bleeding.Go for USG abdomen and urine routine let me know"
},
{
"id": 71946,
"tgt": "Suggest treatment for painful bruise on chest",
"src": "Patient: Since Sunday morning I notice that at the upper left chest is like a bruise of 2\" is like green and when I touch is painfull, I went to the walking clinick and the doctor gave me antiinflamatory Today is Tuesday the pain is still there when I touch Doctor: Hello Probably it is an inflammation of that area which needs antibiotics. Regards Dr.Jolanda"
},
{
"id": 195086,
"tgt": "What does this semen analysis report indicate?",
"src": "Patient: hi how are you .following is my semen analysis report please check is it normal. volume --2.0ml reaction(ph)---8.0 liquification time --- 30 mins sperm count ---70 million viability (live forms)---75% rapid linear progressive (grade 3 & grade4)----60% sluggish linear progressive(grade 2)---- 10% non progressive---5% dead---- 25% normal morphology---85% WBC----- 5-7% RBC----- nil Doctor: Hi, Your semen analysis report is within limit. For 3-5 white blood cells in semen no need to worry. Morphology, sperm count and viability all within limit. kindly mention about your any clinical complaint if present. Hope I have answered your query. Let me know if I can assist you further."
},
{
"id": 156487,
"tgt": "What does swelling and redness of pelvic area indicate?",
"src": "Patient: Hi! I recently went to the health dept for a pelvic exam and pap. I was told I had swelling and redness at the left of a previous scar for pre cancer and they did cryotherapy twice to get rid of cin1 cells. I recently had also been diagnosed with Diverticulitis. I have noticed when I strained to have a bowle movement occassionally i saw spotting blood. I have had a complete hysterectomy and have nothing inside anymore but those cervical cells showed up on the vaginal wall which I have been told is very rare. My question is does the red spots and swelling and light spotting mean I have cancer now? Doctor: These symptoms and examination findings need to be evaluated further with a punch biopsy of the affected area to rule out malignancy (cancer).Please discuss it with your doctor."
},
{
"id": 211841,
"tgt": "Insomnia and OCD after taking citalopram and trazadone for severe anxiety. History of intake of several medications for anxiety and depression. Suggest",
"src": "Patient: I have been on several meds for insomnia, depression, anxiety, from the amitriptyline, nortripylines and other tryptylines to klonopin with klonopin being the longest and the most difficult to get off of. Since weaning myself of it nearly 18 months ago I have had terrible anxiety and OCD started shortly afterwards. I am currently on citalopram and trazadone. The trazodone, I am still experiencing insomnia and OCD. What can I do? Doctor: Hello.... Thanks for your query. The treatment options for OCD include:1. Anti-obsessive agents like fluoxetine2. Psychotherapy interventions like ERP/ Exposure and response preventionI would suggest that a combination of the above modalities might suit your case. Clonazepam is long acting sedative and needs to tapered gradually over 2-3 months. Contact your psychiatrist at the earliest to discuss these options. Wish you speedy recovery.RegardsDr SundarPsychiatrist"
},
{
"id": 21806,
"tgt": "What causes soreness in upper breast and shoulder?",
"src": "Patient: Hi, may I answer your health queries right now ? Please type your query here... I am 61 years old - white female. I woke up this morning very sore in the upper left breast (chest) and shoulder blade. I really think I slept wrong because I slept with my left arm up over my head almost all night. I just want to make sure it is not my heart. If I take a really deep breath it is sore and when I turn to the right real far, it is sore. That's why I think it is just a muscle strain or something. Doctor: Dear Mrs,It is just a neuromuscular pain, may be a muscle strain. Don't worry, take Ibuprofen 400 mg. If the pain doesn't disappear after a single dose, then take it three times a day after the meal for 3 days.Take careCome back if you have any further questions"
},
{
"id": 124075,
"tgt": "What is the treatment for swollen and painful knee?",
"src": "Patient: I fell on my knee last night and the inside of it is extremely swollen and painful but there is no bruise. I can barely move my leg at the knee now and have a lot of trouble walking, especially up and down stairs. Is this something that I should be worried about or should I be fine to wait it out? Doctor: Hello, As mentioned about not able to move the knee and pain while climbing up the stair's I would recommend you to take up and MRI of the knee as what I suspect is the ligament sprain. Usually, in certain cases, we get degenerative changes in the ligaments and joints and can be treated conservatively. An MRI will help us understand the extent of the soft tissue injury and guide exercise as accordingly which can help regain the functions of the knee back. In my clinical practice of 12 years, most cases have been recovered with only executed exercises for both the limbs equally to get the desired outcome score. Hope I have answered your query. Let me know if I can assist you further. Regards, Jay Indravadan Patel, Physical Therapist or Physiotherapist"
},
{
"id": 50056,
"tgt": "Had a kidney infection. On antibiotics. Has low fever. Lost weight, appetite. Tired. Had met with an accident. Suggestions?",
"src": "Patient: young lady in her early 30s works for me she has sean dr for a year can't find whats wrong. she went to californis san diago for her honeymoon came back and had a kidney infection went on antibiotics got better however has run a low grade fever since now its been a year lots of tests and no answers she has lost alot of weight, loss of appetite is very tired but can't sleep, has some good days lately then plunges cant work a whole day. she was in a car accident in high school had to cut her out of the car broken back they wrapped it and let it heal so is pain most of time and has been on pain meds since she was 18 now pain patches no insurance so cant afford back surgery and also scared ro do it anyway what do you think should be looked into? Doctor: Hi,Thank you for your query.With multiple issues the young lady needs to be investigated thoroughly for:1. Low grade fever.2. Back ache.There must be a nidus of infection which needs to be found and managed accordingly. At times the kidney infection can linger on and cause both the complaints mentioned above. Hence a simple urine routine and culture examination become mandatory to start with.I hope I have answered your query to your satisfaction.Wishing the young lady all the best.Regards,Rajiv K Khandelwalhttp://goo.gl/SuCjl"
},
{
"id": 38701,
"tgt": "Suggest treatment for pyoderma",
"src": "Patient: What is treatment for piaderma? I am 61 yoa, 5\"6, 155, had ulcerative colitis for approx 20 yrs. Recently had surgery for colon removal and developed piaderma. Have been referred to a dermatologist but until then, is there any remedy I can do? Is getting progressively worse, ulcers increasing and bleeding. Doctor: Hello,Welcome to HCM,Pyoderma means pus in the skin and can be caused by infections, inflammation and neoplastic etiologies. Any condition that results in the accumulation of neutrophilic exudate can be termed a pyoderma. But in general practice most commonly pyoderma refers to bacterial infections of the skin. Normal resident bacteria of skin such as staphylococci, streptococci, Micrococcus and Acinetobacter can lead to this condition.The treatment of superficial pyoderma is with appropriate antibiotics for not less than 20 days. Fluoroquinolone group of drugs helps in treating this condition.Topical antibiotics may be helpful in superficial pyoderma. Bacitracin and polymyxin B are more effective against gram negative bacteria, if it is the cause of infection.Thank you."
},
{
"id": 93505,
"tgt": "Severe left abdominal pain, increases when taking deep breaths. Advise?",
"src": "Patient: HI,Suffer from severe sharp pain in my @@ left stomach at around the height of my belly button. It hurts more severe when tensing or breathing in deeply.I have a hard time standing straight as the sharp pain hurts even then. The pain does not come in intervals, it is pretty much constant. I do not have fever, have been drinking and eating enough.I would appreciate any help. Doctor: Hi and thanks for the query,Other parameters are required for the complete evaluation of this pain. causes could range from acute gastritis, gastroesophagal reflux, gastroenteritis, and gall bladder disease or liver disease. The intensity, duration, irradiation, aggravating and relieving factors, associated sympoms like fever, vomiting, cough, family history, medical and drug history are very important in the diagnostic process.Suspicion of peptic ulcer disease or gastroesophagal reflux might require a fibroscopy. An abdominal ultrasound could be required at some point in time. A stool examination, liver function tests could asked as shall be guided by a clinical examination.I suggest you book an appointment with an internist or gastroenterologist for a proper examination and appropriate care.Thanks and best regards,Luchuo, MD."
},
{
"id": 8034,
"tgt": "Which is the best medicine available in India for treatment of acne?",
"src": "Patient: best medicine available in india for treatment of acne??? Doctor: there are many good medicines available. Go to a skin doctor. Let he decide what kind of acne you have got and accordingly he will give medicine."
},
{
"id": 134445,
"tgt": "Suggest treatment for protrusion of eyebrow bones post upper eyelid surgery",
"src": "Patient: I had a 2nd upper eyelid surgery but too much skin and fat was taken out, 10mm of skin on the side. Only 1 to 2 mm on the side should be excised but too much skin and fat was taken out. My first eyelid surgery was 2011 to remove the skin sagging blocking my peripheral vision but there was 1-2 mm skin left and for more than 2 years I have been putting makeup in my eyes to cover that 1 to 2 mm that was left and have just decided to have it corrected this January 2014 and didn t know it would cause me more problem now that it deformed my face. Instead of correcting, the doctor did another blepharoplasty again. It was overdone and very aggressive where I lost a lot of skin and fat in my eyes.The bones in the side of my eyes and cheek bone got too close which I can feel which resulted in pulling my eyes down and my cheekbone moved up, because it is pulling each other my nose stretch too and the side of my forehead pulled down too. I can feel my eyes compressed or shrunk. It is too unfortunate to say that it really deformed my face. The most worrying part is my cheekbone moved up and the bones in my brows protruded due to the fat that was taken out. This is my main concern that I want to address. What is the best approach to this problem? How can the cheekbone that moved up and eyes that got pulled down go back to its normal position? Doctor: the best approach to this complicated case is to visit a plastic and facial reconstruction surgeon who will examine you properly and suggest a couple of things. there is still hope of deformity correction. difficult to say what needs doing without actually seeing your face."
},
{
"id": 202838,
"tgt": "What could cause recurred blisters on penis after having sex after being treated for pus cells in urine?",
"src": "Patient: Hi, my husband complained of having blisters on his penis after we have sex. So we went to a lab and they took urine samples and said they found pus cells and we were placed on antibiotics for a week. 3 days ago we had sex and this morning he has a blister again on his penis....what should we do? Doctor: Hi,For a married man having genital ulcers, the most likely cause is that his wife is carrier of the organism and having asymptomatic infection, if his only sexual partner is his wife. You and your husband both should be tested for the typical bacteria causing genital ulcers chlamydia, and as well as PCR for herpes simplex virus and genital sectreations for fugal infections. Syphilis doesn't cause recurrent genital ulcers, so in my opinion it is not prudent going for syphilis testing.Regards"
},
{
"id": 215816,
"tgt": "Suggest remedy for pain in abdomen spreading towards rib cage",
"src": "Patient: I had a stomach start at the bottom of my stomach, then I ate some lunch and it started to hurt even worse and then the pain moved up to my diaphragm area. After school, I went out with my friend for food and the pain got way worse and moved up to my lungs and rib cage. Doctor: Hi, It may be due to costochondritis or acid reflux. As a first line of management, you can take analgesics like Tramadol and antacids like Pantoprazole for symptomatic relief. If symptoms are severe you can consult a physician and get evaluated. Hope I have answered your query. Let me know if I can assist you further."
},
{
"id": 122576,
"tgt": "Could bump on shoulder indicate internal bleeding?",
"src": "Patient: I m on pradaxa and I was horse playing with my girl and she threw a cup at me which hit me in the front of my shoulder around where the clavicle meets the upper arm. it is very painful but there is a large bump that has formed. I know that I ve been warned about a head injury but should i be alarmed that this could be a serious injury like internal bleed. Doctor: Hello, It could be a contusion only. You can go for an ultrasound to confirm the diagnosis. Hope I have answered your query. Let me know if I can assist you further. Regards, Dr. Shinas Hussain, General & Family Physician"
},
{
"id": 177322,
"tgt": "Suggest baby formula to be given in place of milk",
"src": "Patient: my 1.5 month son has been diagnosed with cow milk protien intolerance. mother has been asked to stop consuming dairy products. we have been asked to give the baby isomil or zerolac formula in addition to breast feed as his weight gain is inadequate.. birth wieght 3.1kg and wight after 44 days 3.7kg.isomil is giving him constipation and painful stools. plz help. shpuld we switch to zerolac or wt can we do? Doctor: Hi SirI feel your son's weight gain is almost near his expected for his age(according to our calculations)..you can continue isomil as it is a perfect substitute in kids with CMPI.But i would recommend you the following dilution...jus check if you are correct...Dissolve 1 scoop of powder(the measuring spoon given by the product manufacturers) in 30 ml of leukwarm water.You can give 100 to 120 ml per feed for your son's age...which means 3 1/2 to 4 scoops in 100 to 120 ml of water respectively...you can give 6 to 8 feeds per day in addition to breastfeeds as and when required.But make sure your wife is off milk products in all forms after having a confirmation with your doctor.If the country which you reside is tropical or hot at present,you can give 2 to 3 spoons of boiled and cooled water 4 times a day.Normally we dont advise so in exclusively breast fed kids but because your son has already been started on formula feeds I'm advising you this way just to make sure his water content is also adequate.I would be glad to help you further if needed.Dr.Sri Vidhya . V.SPediatrician"
},
{
"id": 190906,
"tgt": "Do I have to go for another root canal treatment as the temporary crown from the previous treatment has faded away ?",
"src": "Patient: wobbling first molar with pain after 13 months of a root canal procedure Hi , my name is Joe .. i had a root canal procedure done on my first inferior molar 13 months ago ... the crown i got was a temporary one and it has faded down a little bit already, im 34 yo and that piece is wobbling and have some pain with a slightly swollen exterior gum...looks like an infection problem ... root canal again??? Doctor: Hi Joe, A swelling around the root canal treated tooth looks like there is an infection. An Xray of the tooth would give an idea of the problem .Consult your dentist at the earliest. An infection of the tooth can result in mobility of the tooth. If required a re-treatment of the tooth should be done to solve the problem."
},
{
"id": 112903,
"tgt": "Suffer mild back pain, minor protrusion of disc which presses on the L5 S1 nerve route on the left side. Treatment?",
"src": "Patient: i suffer mild back pain .I have a minor protrusion of the Disc which presses on the L5 S1 nerve route on the left side.I have full mobility and have ran the dublin marathon with this ailment in 4 hrs 27 . However I have tried physio, the advice and exercize routine given in Robin MC kenzies book etc but I still have the slight pain which very often extends down my left leg and causes slight numbness to my left foot.My quality of life is affected.I am 50 yrs old, and would really like to correct this for once and all Doctor: Hello , Welcome at HCM. I have read details of your question. It looks that you are having prolapsed disc at L5-S1 & it is compressing over nerve root.This is reason for numbness & pain in foot. Although with mild type of prolapse conservative treatment is prefeerred but if it si leading to numbness & pain which is aanoying you & affecting your life then you should go for surgery( for dissectomy). There are various methods for this which include opening through large incision to opening through minimal incision.You should opt for surgery through minimal incision.For this you should consult to a neurosurgeon. Hope this all will help you. With best wishes."
},
{
"id": 1121,
"tgt": "How to overcome COPD, bronchitis and asthma during conception?",
"src": "Patient: Hi, I am a 24 year old female. I have been diagnosed with COPD (I know I m young, but I ve gotten multiple diagnoses from multiple doctors-I have chronic bronchitis, Asthma, and I was born 6 weeks premature), PSVT, and an arrhythmia (I m not sure what type, but it showeds up on the EKG as an extra and irregular beat that appeared at regular intervals). I have a Mirena IUD, and have had it for four years now. Recently I have been experiencing symptoms of pregnancy, though I took two tests that came back negative. I plan to take another test in a few weeks since I have amenorrhea due to the IUD. I am wondering if I am pregnant, what risks do my other health problems pose to the pregnancy? Thank you! Doctor: Hi I think as you are having mirena still inside you it is very rare to get pregnant with it. So if you want pregnancy mirena has to be removed first. Then you can try for pregnancy. Your health issues won't have any effect on your baby but you need to be in constant touch with your doctor and get regular checkups done.Some medicines may need to be changed if you are planning pregnancy. Talk to your doctor regarding this."
},
{
"id": 65746,
"tgt": "What is the cause and treatment for lump on the butt region?",
"src": "Patient: I recently noticed a lump in my buttocks crack. It doesn t hurt, but it is large and hard. It doesn t feel like an ingrown hair. I initially thought it may be a pilonidal cyst, but it s about halfway between the top of my buttocks crack and my anus, so I don t know. Doctor: Hi, thanks for sharing your health concerns with HCM! If I were your treating Doctor for this case of painless butt-crack lump, I would come up with three possibilities, these include: 1.\u00a0\u00a0\u00a0\u00a0\u00a0skin or hair follicle infection due to some friction injury or sebaceous cyst2.\u00a0\u00a0\u00a0\u00a0\u00a0The second possibility is of lipoma, neurofibroma or skin tag \u00a0\u00a0\u00a0\u00a0\u00a03.\u00a0\u00a0\u00a0\u00a0\u00a0The last possibility is of some infected cyst or benign tumor / tumor like condition \u00a0\u00a0\u00a0\u00a0\u00a0Overall, it is benign or inflammatory and not to worry about this!I suggest you to see a surgeon for an assessment and if possible, surgical excision of the lesion to cure it!Hope this answers your question. If you have additional questions or follow up questions then please do not hesitate in writing to us. I will be happy to answer your questions. Wishing you good health."
},
{
"id": 1990,
"tgt": "How to get pregnant in case of secondary amenorrhea and sinusitis?",
"src": "Patient: I have been suffering from secondary amenorrhea for about 4 years now. I am 38 years and had abortion (no grwoth) from1 IUI and then had two more IUIs with no pregnancy. My tube test came normal. I am not responding to the IVF-M injections to produce more follicles. My weight is more and i am not able to reduce it. I have sinusitis which increases when i exercise and sweat and due to that i am not able to exercise more as well. Can you please guide me as what should I do to get follicles and subsequently get pregnant? Is this the end of the road for me? Doctor: Hi I think you should go for evaluation first. Do a day 2 FSH , LH and AMH level and a so a ultrasound for follicles count . It will show how many eggs are left in your ovaries. If these reports are abnormal then you will have to go for IVF with your eggs or with donor eggs to get pregnant. Talk to your doctor regarding this."
},
{
"id": 204295,
"tgt": "Suggest treatment for suicidal thoughts despite taking Duloxetine",
"src": "Patient: I take duloxetine Hc Dr 30 caps. 2 a day. But, I still feel like the world would be better off without me. Or maybe I would be better off with out the world. I lost my job about 4 months ago because the director of the nursing home I worked at told me he had to let me go because someone told I was crazy. This person wanted my job, even I knew that. But, I have not been able to get over it. I saw the director 2 weeks ago and he asked if I had lost my small ranch yet. When I said no he was very disturbed. I am having a hard time adjusting to being without a job. But, what he said about losing my ranch really bothers me. That is when started taking antidepressants. I just can t seem to get back to my old self. I am going to be 66 the 18 of this month and to tell you the truth it scares me. The last place I want to be is a nursing home at my age. Doctor: Hello and Welcome to \u2018Ask A Doctor\u2019 service. I have reviewed your query and here is my advice. It's very disheartening to know about your condition at that age. As you rightly expressed about your suicidal thoughts inspite of taking Tab Duloxetine at adequate dosage, it's better to add on Tab Lithium low dose which effectively works to reduce suicidal thoughts and ideation. Consult your psychiatrist and discuss this with him and take a decision. Morning walk daily exercise for 2 to 3 days which makes you feel better, interact with your friends, watch movies, listen to songs, spend time with your family members. If possible, attend to cognitive behavior therapy sessions from clinical psychologist. Hope I have answered your query. Let me know if I can assist you further."
},
{
"id": 102346,
"tgt": "What is the remedy for skin allergy when on cetrizinefrom?",
"src": "Patient: Hello dr., tamanna this side I hv a problem of skin allergy.i m facing itching problem , my lips get swollen n itchinh in whole body.This problem is from the last 5 yrs.I knw its starts if I eat something sour like curd, pickle etc..bt after few months of my marriage it get stop I think bcoz of harmonal changes.bt nw ihas started again m taking medicine for this that is cetrizinefrom the last 5 yrs.plzz tell me solution for this allergy Doctor: Hi, thank you for posting at HCM.From what you refer I can understand that you have identified allergic agents.The solution is simple, you should avoid those foods that cause allergy and by avoiding those foods the allergy is going to disappear.However, to determine the diagnosis correctly you need:- Complete blood test(leukocyte rate).- Skin allergy test.- Stool parasites test(parasitic diseases can cause a skin allergy).You should contact a allergologist.Dr. Behar."
},
{
"id": 139037,
"tgt": "Suggest treatment for cervical degenerative disc disease",
"src": "Patient: I have a degenerative neck condition, but recently, my head has an intense pressure-feeling, and I want to know if this is directly related or something else. I also have a swollen area behind my left ear, right at the bottom of my hairline, and that lymph node feels a little swollen as well. The head pressure makes me feel disconnected and disoriented, and makes my head hurt. Thank you. Doctor: Hi, I value your concern regarding the symptoms. I have gone through your symptoms, and in my opinion your symptoms are not probably due to your degenerated discs, but the swelling behind ear is definitively a thing to be concerned about, please visit an ENT surgeon in your area to see what that swelling is all about.Hope this answers your question. If you have additional questions or follow up questions then please do not hesitate in writing to us. I will be happy to answer your questions. Wishing you good health.Special note- Any medication prescribed needs to be taken after consultation with your personal doctor only."
},
{
"id": 108596,
"tgt": "Suggest treatment for back pain and headache",
"src": "Patient: I am having back pain on the right side at waist height. Some days it s worse than others but I have had it quite a few months. It causes me to get headaches nearly everyday and neckache. I am nearly 50, is it just my age or should I get it investigated further Doctor: Hi welcome to HCMI have gone thru your query regarding treatment for back pain & headache .I can understand your concern .Dear , your age 50 has nothing to do with your pains . It seems that your problem may be due to your inadequate posture , inactive life style , and faulty food habits . and in turn , your weakened Immunity Pain can occur due to many reasons , is often caused by a over exersion , in wrong posture , weakening & over- stretching of the muscles between the shoulder blades. This is often as a result of spending too long, too often with your head in the position akin to reading a book that is on your lap.Every day, we are bombarded with toxins in our environment and in our food which is why detoxification is such a critical process in your body. If the detox process is hindered, toxins will build up in your body eventually causing acidity and toxemia. These are the root causes of nearly all diseases.I would like to suggest you to Avoid processed foods carbs , sugar , fried ,fast foods , Tea , coffee , alcohol and smoking , sleeplessness stress and Constipation .All produce biochemical and metabolic conditions in your body that will decrease your immunity, So avoiding is definitely first step in the right direction to detoxing the body, boosting the immune system ..There is another prominant factor iS, gas in our body which causes pains and other symptoms as vomiting etc. To alley pains ,Modification in your life style , food habits ,is neccessary -Include healthy diet as fiber,vitamins minerals ,cheese, milk , supplements full of antioxidents , in natural form --fruit ,green veges , lemon juice with water and salt and pepper before meals , ginger ,garlic , Aloe vera 30 ml , Amla ,Turmeric powder a level spoon + almond oil a Tspoon with hot milk ,at bed time, with required work out ---- There is no better option than physical exercise. Walking , yoga - pranaym is the best exercise to begin with , also gives you well shaped body & sound mind and increases your immunity giving resistance from diseases.Albove regimen helps gain resistance from nearlyall health problems assuring you disease free life Take Homeo pathic Nux Vom 30 a dose dose in the evening and Chammomila 30 / 1 dose at bed time .Always remember, if pain is severe or incapacitating, please see your doctor / professionalHope this helps solves your query .Take care .All the best .Get well soonDon't hesitate to come back in case of any further query ."
},
{
"id": 221611,
"tgt": "What are the symptoms of pregnancy?",
"src": "Patient: how to identify if i m pregna? my peroid date is 11th, now i m in middle cycle, my collegues asked me to intercourse now with my husband....to get more changes to pregenant.for past 3 days i intercourse with my husband. but i have heavy stomatch pain...pls advice what shall i do? request your help. Doctor: Hello dear,I understand your concern.In my opinion the chances of pregnancy are more during the fertile period.Fertile period is the one with 3 days on either side of the predicted ovulation date.For example if the expected period is on 20 th then 14 days should be substracted ie.,it comes upto 6th.So 6 th and 3 days on either side of it comes around fertile period.The pain abdomen might be muscular pain.Take rest and avoid physical strain.Dont worry.Take panadol if pain is severe.In order to confirm pregnancy do a urine pregnancy test a week after missed period.Best regards..."
},
{
"id": 138844,
"tgt": "What causes low calcium levels in body?",
"src": "Patient: I have a calcium level of 52 in my urine test results and had pancreatic cancer with whipple surgery 5 years ago, why is my levels so low, I had a kidney stone in april and possible ones in my kidney now. I was prescribed calcium supplement but most thing do not digest and absorbe in my system due to the surgery Doctor: Calcium is one of the most difficult mineral to get absorb from digestive system. And for this you need to have enough of vitamine D also in your body. Vitamine D helps for absorption as well as for metabolism of calcium in body. I suggest to take a calcium from natural organic supplement source so that bio availability will be good comparing to allopethic calcium tablets.Also have a good vitamine D with either direct exposure to sun light early morning or as a supplement again.I also suggest to have enough of milk, egg, banana in your diet to have good calcium availability from your food.One most important thing is do not take much of allopathic calcium because you already have kidney stone issue also. In today's fast life we do not concentrate much on what we eat as well food quality is also not so well. Along with this we are not regularly expose to morning sun light. All this together will result in low calcium in body.I am sure this all information will be useful for you if you follow properly.Take care."
},
{
"id": 205594,
"tgt": "What is the best medicine for depression?",
"src": "Patient: My husband is 40 years old, and we have been in hospital almost a week now...hopefully tomorrow they release him. He is very depressed, and didnt know if antidepressant would be good for him. Also, im totally lost on what I can cook for him now? ?? No salt at all? O\u0155 just lower the take of salt . Thanks ashannon Doctor: Thanks for writing.The information you are providing is not sufficient.More information needed in terms of episodes,severity,reason for admission etc.In general antidepressant takes approx.15 days to start its effect so one week is not sufficient to see the outcome of medicine.You can serve him anything he likes .Salt is not restricted in depression."
},
{
"id": 199115,
"tgt": "What causes brown semen with pain in testicle and abdomen?",
"src": "Patient: hi. im 24 yr old ,male, single and have not been with any one yet. but the last 2 times i have masturbated i found brown semen and today my right testicle started hurting, and when i masturbated a very small amount of blood was in the semen as well. pain also spread a little bit to my right lower abdomen. Doctor: DearWe understand your concernsI went through your details. Brown coloured semen indicates blood stain and blood in semen surely requires clinical intervention because that is the first step of prostate problems. please consult a physician immediately.If you still need my assistance in this regard, please use this link. http://goo.gl/aYW2pR. Please remember to describe the whole problem with full detail.Hope this answers your query. Available for further clarifications.Good luck."
},
{
"id": 125719,
"tgt": "What causes severe pain in the heels after an achilles tendon injury?",
"src": "Patient: I have torn both of my Achilles previously, not from the heel but complete rupture in between the calf and heel. For the past week Ive been having pain on the lower half of my heel. Today while flexing foot upward extreme pain at heel. The Achilles in not torn now. Ideas? Doctor: Hello, Most probably it will be an inflammatory pain. As of now, you can use analgesics/anti-inflammatory combination like Aceclofenac/Serratiopeptidase for symptomatic relief. If symptoms persist you can consult an orthopedician and plan for an MRI scan. Hope I have answered your query. Let me know if I can assist you further. Regards, Dr. Shinas Hussain, General & Family Physician"
},
{
"id": 122322,
"tgt": "What causes shaking and difficulty in holding objects?",
"src": "Patient: i didnt eat anything yesterday nor today. i also didnt drink much yesterday, and i havent drunk anything today either. when i was in a music class today i couldnt hold my instrument still and when i stood up i was shaking really bad too. But ive had anorexic off an on for two years though. what should i do? Doctor: Hello,Thanks for the query.Consult a neurologist and get evaluated.Conditions like Parkinson disease has to be considered.If there is diagnostic confusion an MRI brain may be needed to look for central causes.wishing all the best.Thanks"
},
{
"id": 120242,
"tgt": "Can surgery cure uneven gait?",
"src": "Patient: sir i`m a Medical graduate,i`ve a polio affected left limb.bulk of my left limb is comparatively little smaller than right.power of my limb is grade 3/5,i`m having pes cavus.my limb when i lie down in prone position measures equal with right.i`m able to walk without support.will any surgery give cure for my gait? i`m doing regular exercises.pls suggest me a cure Doctor: Hello, Surgical correction like tendon transfer may be effective. Consult an orthopedic and he will direct you accordingly. Hope I have answered your query. Let me know if I can assist you further. Take care Regards, Dr. Shinas Hussain"
},
{
"id": 56264,
"tgt": "What causes elevated amylase and lipase levels in case of pancreatitis?",
"src": "Patient: I have chronic pancreatitis and have had for several years (I had my gall bladder removed in 2001 and CP diagnosed in 2003). My most recent blood test results show my triglycerides low (they have been steadily high in prior years) and cholesterol levels in a good range. My weight is down with the help of garcinia cambogia(?), but both my amylase and lipase are elevated ( I frequently have elevated amylase, but not very often lipase). My last ERCP showed minor pancreatitis, but the pain has continued. Is the garcinia a foolish thing to take with my pancreatitis history? Any other explanations for increased amylas/lipase? My family has bad heart history (I am 57), so the good cholesterol is good to see, but the sudden rise in both amylase and lipase is concerning (though very low triglycerides for me now . . . 800s 10 yrs ago, 400 s 7 years ago, 150-200 recent years, 71 now). I have been under the care of a pain specialist and am prescribed opiates for the pain, but I have lowered my dose about 33% over the past six months and hope to lower it as far as possible (eliminate if pain can be reasonably contained through healthy eating. Any thoughts besides see a pancreas specialist (which I do)? Doctor: Hi, I had gone through your question and understand your concerns.Amylase and lipase are enzymes. Enzymes are proteins produced by the body to do a particular job. The pancreas produces amylase to break down carbohydrates in food into simple sugars. Lipase is produced by the pancreas to digest fats into fatty acids. Sugars and fatty acids can then be absorbed by the small intestine. Some amylase and lipase can also be found in saliva and in the stomach, but most is made in the pancreas and released into the small intestine.Amylase and lipase are enzymes produced by the pancreas that help to digest food. If the pancreas is damaged, high levels of these enzymes can be detected in the bloodstream.Do not worry about the low triglycerides. They won't harm you.Stop taking Garcinia.I would suggest you to-\u00a0\u00a0\u00a0\u00a0\u00a0Drink loads of water-\u00a0\u00a0\u00a0\u00a0\u00a0Avoid oily,spicy food-\u00a0\u00a0\u00a0\u00a0\u00a0Have a diet rich in fibres-\u00a0\u00a0\u00a0\u00a0\u00a0I would start tablet Pantoprazole . Your doctor can guide on the exact dosage- Get a CECT (contrast enhanced CT scan) done . This will help to look at the pancreas , the calcifications and ducts.Visit a gastro-enterologist. He/she can examine you and correlate clinically.Do not worry.Hope this answers your question. If you have additional questions or follow up questions then please do not hesitate in writing to us. I will be happy to answer your questions. Wishing you good health."
},
{
"id": 162024,
"tgt": "What causes my son s recurring fever?",
"src": "Patient: Hi, may I answer your health queries right now ? my son is 2 years i month old he is having light infection of typhoid fever. from dec 23 to dec 31 he had fever and after that 12 days he was normal. again now from jan 13 to today he is having fever every five hours he is having fever what is the reason for this. Doctor: Hi, It's very common for children to get infection easily from their surroundings. Since your child was well for almost two weeks prior to this illness, it looks more like a newer infection rather than the older one. There are many causes for recurrent fever in children, but I don't think your son is having any serious or worrying condition. I would suggest you give plenty of oral fluids and paracetamol (dose as advised by your paediatrician). Hope I have answered your query. Let me know if I can assist you further. Take care Regards, Dr Nirubhan Bharathy, Pediatrician"
},
{
"id": 183297,
"tgt": "Suggest medications for tooth pain",
"src": "Patient: went to dentist on wednesday for broken tooth. HE did nothing, said root was infected gave me amoxicillin and 3 vicodin & said pain will be gone is 48 hours. now i am in more pain than before, constant, hurts to even touch the tooth, orajel has no effect, how can i stop the pain, my vicodin are gone, not that they even made a dent in the pain? Doctor: helloo..read thru ur query ...accordingly i must say that u continue with the amoxicillin and u can take diclofenac painkiller...do salt water gargle ...try not to touch that area...have soft diet..have lots and lots of water...be cool...and meet the dentist after a week..mostly u may have to undergo rootcanal treatment and crown...dont panic be cool..happy mind can relieve all pain....hope ur benefittrd something from this reply..have a healthy day!!!"
},
{
"id": 42304,
"tgt": "Suggest treatment for infertility",
"src": "Patient: hello doctor. I got marry 3 years before. we are planning for a baby from last 3 months. My wife s period is almost regular. we mostly meet every alternate day from 10th to 20th day. In last 3 months, my wife has not concieved. please suggest what should we do. Doctor: Hi,I have gone through your query.Both of you have to go for some tests. You will go for semen analysis and your wife will need ultrasound along with HSG. There could be some cause in both of you or any one. You have to consult a infertility expert for this. You can share your result on my page for further queries.Hope i answered you.You can contact me on my page http://bit.ly/DrGauravWish you a good health.RegardsDr. G.Katiyar"
},
{
"id": 19088,
"tgt": "Can electrical cardioversion cause excessive fatigue in a heart attack survivor?",
"src": "Patient: Three weeks ago I was in the hospital emergency room and had a heart attack. I flat lined and they had to shock me to get me back. They had to shock me four more times to get my heart beating like it should. I m 82 and this is my third heart attack (first was 1999 and second in 2000). After going through rehab in 2000, I got to where I was running 6 miles in an hours time, three times a week until my knee got bad. My question...is it normal to feel this weak like I do? It wasn t like this in 2000. Doctor: Hello,Since it is the third time you have a\u00a0heart attack, you need to be more careful about your health and take the necessary precautions. Your body needs time to recover since you had a major cardiac event. You will feel tired weak, and you should not be in a hurry to start back running or walking like before. Kindly take your medicines as advised by your cardiologist, follow dietary precautions, include fruits and vegetables in your diet. Watch the color of your urine, if it's dark yellow means you are dehydrated, which can be also one of the cause for your weakness.\u00a0Hope I have answered your query. Let me know if I can assist you further.Regards,\u00a0Dr. Aparna Amritsagar"
},
{
"id": 174044,
"tgt": "What causes high fever with stomach ache and heeadache?",
"src": "Patient: My 6 year old son has had a fever of around 102.7 for the last two days. He hasn't complained of a stomach ache...just a headache and just now said it hurts a little when he swallows. Not sure if I should try to contact the physician on call from our ped. office. Doctor: HI having high grade fever with headache most commonly occurs due to viral infection. Give him paracetamol syrup 15 mg/kg and soft diet with adequate fluids.Observe him and showing any symptoms like continuing high fever, neck pain , headache and vomiting, show to a child specialist immediately."
},
{
"id": 7205,
"tgt": "Is there any other medicine instead of Proxeed Plus Powder for the treatment of semen liquefaction ?",
"src": "Patient: We are trying since a year for a baby hence doctor said to have semen test reported that After waiting for about 30 minutes after ejaculation, to allow the semen to liquefy, the doctor checked the semen report said that even after 3 hours not liquefy. Please suggest me to get normal since the doctor prescribed that to have Proxeed Plus Powder at least 3 months but still same condition from new report. Doctor: Hello Thanks for your query. \u00a0\u00a0\u00a0\u00a0\u00a0An abnormally long liquefaction (more than 30 minutes at 37 \u00b0C) time indicate an infection. Take a full course of broad spectrum antibiotics Take plenty of liquids along with antioxidants to bring the liquification time to normal. It may take 3-6 months to become normal. \u2018Hope I have answered your query, I will be available to answer your follow up queries, \u201cWish you Good Health and trouble free speedy recovery\u201d"
},
{
"id": 50404,
"tgt": "Delayed periods, white discharge, frequent UTIs. Do I have to be worried about a kidney infection? Chances of pregnancy?",
"src": "Patient: I have tons of problems with my period, it is always late, sometimes more than a week/two weeks late. I am sexually active, I have been for a steady year now, and my partner & I rarely use protection. Every month, I have a different problem. Sometimes white, stringy discharge will come out, (assuming I m ovulating, but it happens 2 weeks before my period is scheduled to start), sometimes it will be completely clear discharge. I also have a lot of UTI s. I have been to the doctor and the only thing they can tell me is I have a UTI or kidney infection . They treat me for it, and it comes right back the next month. Is there a chance it could be serious, or is this just normal? Could I be infertile? Doctor: Hello. Recurrent urinary tract infections can lead to kidney problems in long run. Hence it needs proper investigations. You should consult nephrologist (kidney specialist) for further evaluation. You also need thorough gynaecological evaluation for your irregular periods. Take care."
},
{
"id": 4452,
"tgt": "Can pink bleeding and breast tenderness represent pregnancy associated symptoms?",
"src": "Patient: Hi, I've been on depo consistently for 4 Years now. My last injection was Oct.1 I was due Dec.17-Dec.31for a follow up but I missed it...And I've been having unprotected sex with my boyfriend since November. I used the bathroom yesterday, and it was light pink blood on the tissue.. and my breast have been tender the past 2weeks. Could I be pregnant? Doctor: Hi It is just possible that you are pregnant if you did not take your next inj. in time. But these can also be the side effects of depo inj.Breast tenderness is quite common and there is usually amenorrhoea (no periods) after the use of this inj.and it takes some time in conceiving after discontinuing it.This inj. should not be used for more than 2 years continuously for birth control method.So, I will advise you to get a scan done to rule out pregnancy and then use some other method of contraception. Probably you might not be pregnant.Thanks."
},
{
"id": 97013,
"tgt": "How to treat pelvic and rib fracture?",
"src": "Patient: Had a patient with p wave inversion and 1mm depression in t waves only in septal leads. Any thoughts? 65yo female Presented unresponsive, needed airway managment and ventilation, rr shallow and uneffective, hypotensive 90/60, varying bglu, small pupils, not pinpoint. 4 mg narcan given, 12.5g d50 given bglu initially 121, then 20 min later bglu was 20, then after d50 back up to 200, no response, no improvement. She is sp 2 weeks fall down a flight of steps with pelvic fracture and rib fracture and no significant pmh. Doctor: Have you checked her for pulmonary embolism and collapsed lung? Either of these could result from the injuries she had received earlier, particularly if she'd been immobile. Changes in the EKG would also result. The concern now is not to treat the pelvic and rib fractures, but to diagnose and treat the acute life-threatening problems. Has she had a blood gas? Chest xray? Cardiac enzymes? I would be obtaining these while giving the supportive treatment needed. Could she have had a stroke? The EKG can also be affected with stroke. My first thought would be PE but I would be checking for other cardiopulmonary problems as well."
},
{
"id": 49666,
"tgt": "Multiple UTI, urter repositioned, kidney removal, thickening bladder. Reason?",
"src": "Patient: My daughter has had multiple UTI and has had urter repositioned and then removal of her kidney due to it dying and still causing infections. now she has been told she has a thickening bladder and a segment of her small intestine looks abnormal. is this connected to UTI or just 2 separate problems. what does a thickening bladder mean? Doctor: Hi, it appears that your daughter had kidney infection, at its end and so there might be a spill of the scrap while removing the kidney. or it may be a metastasis of the kidney pathalogy, if there is malignancy. Unless it is examined personally and having a biopsy, it not possible to judge the diagnosis. Thank you."
},
{
"id": 161920,
"tgt": "What causes abdomen pain in a toddler with constipation?",
"src": "Patient: 4 year old son stomach aches whenever he eats, his bowel movements are every other day and sometimes even 2 days, this has been going on for about 2 weeks. He recently had a benign tumor removed from his brain which was located in the pineal gland area. Currently, he is taking both 4ml of keppra and 5ml of B6 twice a day. Doctor: Hello, Looks like your child has constipation. Maybe pain abdomen after every feed is due to constipation or sometimes may be due to duodenal ulcer. kept causing abdominal symptoms is rare. Hope I have answered your query. Let me know if I can assist you further. Take care Regards, Dr Lingamurthy Itha, Pediatrician"
},
{
"id": 104940,
"tgt": "Frequent cold, sneezing and cough. Took Montek LC. Suffering from stomach pain and digestion problems. Side effects?",
"src": "Patient: hello sir/mam.... i have an allergy since a year... frequent cold, sneezing ... so i went to ent specialist , he prescibed me tab montek-LC, so after 2-3 days my cold , cough, sneezing is gone.... but after a week, i have having severse stomach pain, digestion problem and even getting more and more sleep day time.... so is it the side effect of tab Montek LC or wat... ?? Doctor: Hi Thanks for your query Yes gastric irritation occurs in case of motelukast after prolonged use and probably that is causing your stomach pain and digestion problem and sleepiness probably is occuring from levocetrizine component of the montek-LC medicine. So if your symptoms are well controlled now you can omit this drug now. Better to go for local application of azelastine fluticasone nasal spray now after consulting your doctor. For stomach pain you need to take local anesthetic gel like mucaine gel for symptomatic releive and pantoprazole tablets for healing of the erosions if any by montek-lc. Thank you."
},
{
"id": 5743,
"tgt": "Trying to conceive, tests all normal. Prescribed siphene, folicullar study suggested. What to do?",
"src": "Patient: hello doctor before 1 nd half year i got married bt still i dint conceive we hv consultd a gynocologist they have tested evythng is kk they askd me to use SIPHENE 50mg first their was no result so they asked me to use 100mg and now its already 10th day of my period their is still bleeding i dnt knw why today i have consultd the doctor they have tested and told me nw itsss kk come on 12th day for follicular study again so please suggest me what should i do? Doctor: Hi, As you have been trying to conceive for the past one-and-a-half years, you should see an infertility specialist for a specific work up which includes couple evaluation in several aspects. The cause of your present bleeding could be a possible metropathic cycle and this needs to be corrected first before going for follicular study and further procedures. Ask your gynecologist for further management. Take care."
},
{
"id": 99791,
"tgt": "Does swollen lower lip once a day indicate allergy?",
"src": "Patient: in the past three days my lower lower lip has swollen approximately once each day. the swelling only lasts for minutes, about 15 - 20 minutes. i have been smoking cigarets for 10 years and i suspect smoking causes the lip swelling. am i allergic to cigarets? or something else? Doctor: Hi, since u are smoking since 10 years u had not any allergic reaction towards it but u are facing swollen lips since 3 days and in addition it lasts for just 15 minutes as stated by you,its nothing to do with cigarette smoking.these problems usually subsides by itself if problem persists u can contact dermatologist. No need to worry"
},
{
"id": 79293,
"tgt": "What is the cause of chest discomfort?",
"src": "Patient: Was told I ended up with a nasty virus which caused explosive diarreah and vomiting. Was at the hospital to get rehydrated. Now last night I ended up with a high fever, then extreme chills, then high fever again to where Tylenol wasnt helping. Now all day to again I've been dealing with chills and fevers. But now it hurts to take a deep breath or cough. Could it be from the chills and shakes I've had? Doctor: Hi Welcome to HCMto have diarrhoea followed by fever and then chest pain on deep breath and coughing could because of either fluid outside lung or musculoskeletal pull. get and Xray Chest done."
},
{
"id": 150624,
"tgt": "CT scan shows calcium in brain. Have dystonia. Headache, memory loss, tingling face. Treatment?",
"src": "Patient: Hi ive just been told that my ct scan shows quite a bit of calcium in front left lobe of my brain & that i need an mri now to see in more detail. I suffer from dystonia to which affects my neck shoulders arms hands. I have been getting a lot of headaches lately with some memory loss & lso tingling in my face they dont think at moment that theres any tumors behind the deposits but just wondering what treatment i could expect Doctor: Hello, Thanks for the query, I understand your problem Let me tell you that If the CT scan shows calcium in the brain, you continue to do MRI as advised by your doctor. There e is a possibility that you may be having parathyroid problem and the whole symptoms that you have could be related to disorder of calcium metabolism and it is treatable and you will improve. Patients with dystonia generally require neurological evaluation and exact classification of dystonia so that treatment becomes easy. Medications that are used to treat are Tetrabenazine, clonazepam, trihexyphenidyl, botulinum toxin etc. Which medicine is to be used will depend upon the classification of dystonia. Please consult a local neurologist who will guide you through clinical evaluation and investigations and treatment Best wishes Dr Gopal K Dash MD, DM, Post-doctoral fellowship (Epilepsy) Consultant Neurologist and Epilepsy specialist Nrayana Hrudayalaya Hospital, Bangalore My Blog in the Web site: http://www.healthcaremagic.com/doctors/dr-gopal-krishna-dash/64344"
},
{
"id": 152646,
"tgt": "What causes low testosterone level while treating prostate cancer?",
"src": "Patient: I had prostate cancer 7 years ago but after radiation and hormonal shots it receded and my PSA has been normal eversince. As a consequence the treatment left me impotent. My testosterone level is very low.I am in excellent physical shape and exercise 4 or 5 times a week. Would a testosterone treatment be adviceable or could it bring the cancer back? I am 84 years old but look 60. Doctor: The main driver behind Prostrate cancer is testosterone and all treatment first aims at bringing testosterone levels to castrate levels. This leaves impotency as a consequence. But you are fortunate that the disease is under control. Taking testosterone injections is not advisable. Remember life is more important than anything else. Make use of this life to help and serve others."
},
{
"id": 226379,
"tgt": "Had unprotected sex. Taken ipill. Blood clots, excessive bleeding, abdominal pain. Taken Trenaxa. What is the indication?",
"src": "Patient: i had unprotected sex on 3 November but wasn t sure of conceiving so took an i pill on 5 November.after 5 days blood clotts came, but it was just one or two clots a day. But from yesterday excessive bleeding started with little bit of abdominal pain . So i took trenaxa 500mg yesterday night and i have two more doses to take.. But bleeding did not stop. Please help !! What is the indication? Will trenaxa help? Does ut have any side effect? Doctor: Hi thanks for your question. You have taken I pill 48 after unprotected sex and passed blood clots after 5 days. Now you\u2019re having excessive bleeding. Passing clots and excessive bleeding is normal after I pill use. Trenax is antifibrinnolytic that competitively inhibits the activation of plasminogen to plasmin and prevents the body from breaking down blood clots and thus decreases bleeding Thus trenaxa decreases the overall blood loss. It is used for different medical conditions, like: In hemophilic to prevent bleeding G. I hemorrhage, Treatment of heavy menstrual bleeding. Side effects are very few and include: Nausea, headache, nervousness and vomiting Hope this answer your question"
},
{
"id": 201116,
"tgt": "Does excessive masturbation affect penis size?",
"src": "Patient: Hi, doctor.... im doing a hand job from last 15 years. i am 29yr old now. my penis size is too small now. now i am going to get married next year. if this can effect my married life like(to make my wife pregnant)? and is there treatment or madicine for this. please help me about that as soon as possible .... Doctor: Hello Masturbation doesn't lead to shrinkage of penis size.It is purely psychological.There shouldn't be any problem in your married life but you should try to avoid excessive masturbation.Its obsession is bad.If you find any difficulty in getting rid of it the you may seek psychologist counselling.Take CareDr.Indu Bhushan"
},
{
"id": 50623,
"tgt": "Tiny stone in left kidney, pain in sides below ribs. Have gastric problem. Advised alkacitron. Will this dilute stones?",
"src": "Patient: Hii have a small tiny concretion of stone in my left kidney and having pain in both side just below to ribs.i have gastric problem too,found excess gas in ultra sound.i have been advised to take alkacitron.please advise,how it wil help me to dilute stone.i am taking pantop-d in morning i don't have digestion well too,every time i found a bubble of gas in my stomach.need your view . Doctor: Hello,You have 2 problems1) Tiny stone in the left kidney2) You have gastric problem - which may be hyper acidity - leading indigestion and gas formation. Moreover Gas inside the stomach is a normal one. Only excessive gas is an abnormal finding and mostly diagnosed by X-ray.The tiny kidney stone is not a problem at all. It may produce pain from the kidney area to the lower middle of the abdomen on that particular side and not on both sides.The urine alkalising agents is useful, but along with that Antibiotic, pain killers and diuretic as per your treating Dr's choice Intake of plenty of water + barley rice water and not holding the urine for a long time is advised.Take treatment for your Hyper - acidity, That isTimely bland diet, avoiding hot drinks, gas forming food items and chilly. Antacid + Antispasmodic + Anti flaultuants are advisedGenerally your problem is nothing to worry. Watch for descend of stone.Best wishes"
},
{
"id": 177325,
"tgt": "What could cause the blood tests to be elevated in my child?",
"src": "Patient: My child s blood tests came in today. He is 10 years old. His ALT is 96 and his AST is 43. The Dr s said there wasn t anything wrong with his blood work but when I saw the H listed beside both of these tests and to see the ALT 4 times the norm really has me concerned. What could cause these tests to be elevated in my child? Doctor: Hello and thanks for writing in.AST normal value ranges from 10 to 40 units per literALT normal value ranges from 7 to 56 units per literif any of the value or both values increases more than 2-3 times from normal than it is considered as a mild elevation.so in your case the reports are within normal range and you don't have to worry. take care.RegardsDr. Shesh"
},
{
"id": 205366,
"tgt": "Will an autistic child behave like a regular child after getting treated?",
"src": "Patient: My 18th month old grandson was just diagnosed with being on the autism spectrum. We does not chew food, hold a bottle or stand up. If put in a walker he is a toe walker and was told not to put him in the walker. Does he have a chance of growing out of this disease with the proper treatment mentioned?They live in Palm beach Florids in Florida. They are considering a program in Lakeworth ,Florida. Are youfamiliar with it? Any suggestions are welcomed. We want to catch this early with the hopeof him improving to normalcy. Please help. Thank you. Doctor: Well,Autistic symptoms can be improved with careful training and practice. How much degree it can be improved depend on the degree of the symptoms and intellectual ability of the kid.But be hopeful and keep trying.They do tend to have certain exceptional abilities and that can be flourished. Sometimes medications may be required especially if the child have behavioural problems or Hyperactivity.Regards"
},
{
"id": 49523,
"tgt": "What is the treatment for mild fullness of renal pelvis with history of bilateral kidney?",
"src": "Patient: Hi! I m arlyn I have a kidney problem 41 yeard old with history of bilateral kidney.Left and right moiety, I had my partial nephrectomy at right upper moiety 3 years ago due to hydrophronosis.and now I have my check up last February 2013 U/S done with mild fullness of the renal pelvis and my Dr. advice me to do follow up u/s after 4 months then u/s done 24/11/3 the result is mild fullness of the renal pelvis with residual in the bladder.What is the plan for mu case and what is the treatment and I did my renal profile the result with normal and I have pain in the rigth side bellow the ribs. Doctor: HIThank for asking to HCMI appreciate your concern but let me tell you that the clinical sign and symptoms are great matter if you are asymptomatic then such finding of ultrasonography has nothing to do unless you have some serious problem more over your renal profile you said is also normal then you need not worry about it in chronic renal disease such findings are part of disease, that does not cause the routine disturbances, have a nice day."
},
{
"id": 216160,
"tgt": "What causes breakthrow pain?",
"src": "Patient: I have been on methadone for 6 years and I started seeing a PM Dr. that wants to switch me to oxycontin. At the moment I take 60mg of methadone with 4-6 oxycodone 5 mg for breakthrow pain. My question is what is the correct amount of MG daily of oxycontin I should be switched too? and will I go throw any withdrawl during the change? Doctor: Ok, there are a wide variety of websites with estimates of the conversion rate. The actual effect is within 50% of the estimate and a large number of factors including the genetic effect of particular opioids on opiate receptors and the genetics of how fast the drugs are metabolized and the genetic effects of how the drugs feel that are NOT taken into account in the conversion rates.https://www.YYYY.com/pin/0000/here are a bunch of charts.basically, one might start with the oxycontin regardless of the previous methadone dose provided the methadone dose isn't silly high. THere is a huge risk of overdose more than withdrawal because methadone stays in the system and if you use oxycontin the same day as the methadone you are basically doubling the dose."
},
{
"id": 95446,
"tgt": "My thyroid level was 4.86 , Is that bad ?",
"src": "Patient: Hi, I'm a 16 year old girl and I recently went to the doctor and had blood tests done. He said that my thyroid level was 4.86. Is that bad? Doctor: In thyroid profile, there are 3 hormones tested, T3, T4 and TSH. Could you specify which one you are talking about?"
},
{
"id": 134917,
"tgt": "Suggest remedy for severe leg pain",
"src": "Patient: My legs ache and it s quite bad in the calf muscles...I can t stand them being touched let alone massaged...I walk a lot but even if not walking the terrible ache wakes me at night sometimes...it s been like this for years,ni font work so it s not like I m in my legs ...it s not varicose veins as I have had them done years ago...I take magnesium at night with my calcium bug it doesn t help...I am 62 and aware of it all the time... Doctor: Hi.thanks for writing to healthcaremagic.you seem to have muscle weaknessyou should:drink lots of water everyday (min 10-12 glasses per day)take adequate rest, avoid any strenous activities for sometimedrink coconut water everyday if possible eat 3-4 bananas everydaytake vitamin E regularlydip your legs in a bucket of warm water 2-3 times a dayif possible, try and get your levels of vitamin D.feel free to get back if any queryThanks and take care"
},
{
"id": 172457,
"tgt": "Suggest treatment for frequent colds,snoring and breathlessness in a child",
"src": "Patient: My daughter is 10 yrs old, she has frequent colds from childhood and since her 4th year she has heavy snoring, some times she feels breathless and some times feels disturbed sleep. Last year when she developed same symptoms even it accompanied with headache for almost 45 days with light swelling beside nose (under the eyes-sinus cavity). Doctor has suggested oxymetazoline hydrochloride nasal solution USP (0.05%) for a week, and then to use triamcinolone acetonide nasal spray for almost 3 months + montelukast & levocitrizine hyrochloride dispersible tablets also for 3 months. We have used for almost 3 months but problem still persists.Please help me in my daughter's treatment and also advice me should i consult a pediatrician or a chest specialist. Doctor: Hi,Thanks and welcome to healthcare magic.It looks like allergic rhinitis.You may try ebastine syrup twice daily for 1-2 weeks.You have to exclude adenoids also.Better consult ENT specialist.Hope this answer is OK for you.Further questions invited.Dr.M.V.Subrahmanyam."
},
{
"id": 192617,
"tgt": "What causes the growth of white film on the glans penis?",
"src": "Patient: I have a white film growing on my glans penis, it is painless, itch-less, and can be easily take off by just gently robing it. I have tried washing it thoroughly, but it keeps returning; and after the white firm been washed out, my glans looks extremely sleek when it is erected. Also after i masturbated, the white film seems growing faster. I have no other any obvious symptoms, like painful urination, aching testicles, etc. and there is no other abnormal symptoms on my genitals either. I don't have circumcision and my foreskin is normal length. I'm worried I may have an sexually transmitted disease or candidiasis., may I answer your health queries right now ? Please type your query here... Doctor: Hello, It could be a smegma. Poor personal hygiene is the main cause for its development. As of now clean the foreskin with soap and water and apply topical antibiotics like neosporine. If lesion persists better to consult a dermatologist and get evaluated. Hope I have answered your query. Let me know if I can assist you further. Take care Regards, Dr. Shinas Hussain, General & Family physician"
},
{
"id": 28523,
"tgt": "How long does it take for a nose piercing infection to go away with sea salt soaks?",
"src": "Patient: I got my nose pierced about 5 weeks ago and it was perfectly fine, producing no crusties at all, wasn t hurting, and i thought it was healed so i decided it was time to change it since i really didn t like the stud. I had my piercer change out my jewelry for a hoop i had bought at a store that sells lots of body jewelry. Two days later, I have a smallish, red bump right above my piercing with tenderness around my nostril. I don t know the material of the hoop but I am not allergic to anything but mosquitos so it can t be that. I tried a hot sea salt soaks today and it reduced the redness and tenderness but now it s forming green crusties around the piercing. So here are my two questions: 1. Is that good that i m forming green crusties after the sea salt soak since maybe it s maybe pushing the infection out or is that making it worse? 2. How long does it take for a nose piercing infection to go away with sea salt soaks? Doctor: Hi, No, the green discharge can imply that the infection is unfortunately worsening. There is no specific time interval that it may take for an area to heal with the soaks though it normally takes 4 to 7 days. If there is no improvement or any worsening then you may need a course of topical or oral antibiotics. While an infection is present, the foreign object or jewellery, that is the cause of the infection, may need to be removed till everything is improved. Hope I have answered your query. Let me know if I can assist you further. Regards, Dr. Michelle Gibson James, General & Family Physician"
},
{
"id": 200876,
"tgt": "What causes pain in groin?",
"src": "Patient: Hello, My husband was complaining earlier of a painful boil in the groin, also he had pain higher up in the groin, away from the boil. He s gone to bed, and was only there 10 mins when he said he felt hot, but was shivering visibly. The feeling of unwell has come on him so quickly, and that s my concern.Can you help? Doctor: Hi,It seems that there might be having ingrown hair follicle infection giving rise enalrged inguinal lymph node in the groin.Due to bacterial infection he got shivering, feeling hot and unwell.Go for one antibiotic medicine like Amoxicilin for 3-5 days.Give analgesic like Ibuprofen as needed.Within 2-3 days he will be alright.Ok and take care."
},
{
"id": 206425,
"tgt": "Suggest treatment for mental problems",
"src": "Patient: Hi, i am amit, my mother is suffering from mental problem she is taking rispid and betacap medicine for last 15 years. She always thinks someone is going to harm her family. Her age is 49 now. She seems to have very little common sense too.She is upset today too.I would deeply appriciate any help or guidence.Thanks in advance, Doctor: Hello She always thinks that some one is going to harm the family. Such type of symptoms are seen with suspiciousness and paranoid ideation. Most likely she is having some psychotic disorder. She is on risperidone and propranolol (betacap).Risperidone is a good anti-psychotic drug and this drug help in decreasing the symptoms of psychotic disorder. As she is continuously taking the drug and you have also mentioned that she has vey less common sense or in other terms she can reason logically and abstractly. These type of symptoms are seen in negative symptoms of psychosis. Propranolol helps in improving anxiety symptoms.As per my opinion consult her psychiatrist again and explain the negative symptoms she is showing. Medicine like Amisulpiride is antipsychotic drug with good effect in negative symptoms. Discuss with her psychiatrist for any dose adjustments or any substitution of drug if needed.Hope this helps you, THANKS"
},
{
"id": 159422,
"tgt": "In last stage of pancreatic cancer, has bad breath, in severe pain, weakness. What can make her comfortable?",
"src": "Patient: My mom is in the last stages of pancreatic cancer , we have hospice now and are just trying to keep her comfortable. We know that her organs are starting to shut down, can that be the reason for her bad breath? She only gets out of bed once or twice a day to use the bathroom but it takes her awhile since she is so weak and has shortness of breath. She has all the signs that death is approaching but is still hanging on and is in pain, is there anything we can do to keep her comfortable? Doctor: Hello! Thank you for the query. Unfortunately, there is nothing to be done for terminal pancreas cancer. Bad breath can be caused due to acid reflux, lack of proper digesting or some inflammation process. The only thing you can do is to ease her pain with as much morphine as possible. And stay with her with love and hope till the last day. Hope this will help. All the best for you. Regards."
},
{
"id": 77099,
"tgt": "What causes cramps in hands while on Clarithromycin for gurgling in lung?",
"src": "Patient: I have been ill with chest infection and was on antibiotics 500mg x 3 times a day + cortisone (8 a day for 5 days) and then because i had still gurgling in left lung was put on clarithromycin 250mg every 12 hours and 3 days x 8 cortisone tablets.(predisolone). I am still on clarithromycin 250mg for a few more days but i am getting lots of cramps in my hands why is this? Doctor: Hi and thank you for choosing HCM to post your question.Cramps in your hands are a side effect of cortisone(prednisone) and are due to the electrolyte disbalance that cause cortisone.Clarithromycine in combination with cortison increases the effect of cortison.A combination of foods that contain Magnesium and Potasium or supplements can help reduce cramps.I would recommend plenty of rest and hydration.Hope my opinion is helpful.Wish you good health.Kind regardsDr. Dushi"
},
{
"id": 211222,
"tgt": "Can tingling legs, fainting feeling, low blood pressure and headache be signs of panic attack ?",
"src": "Patient: I am female, age 69, weight 200 lbs., all afternoon have been experiencing what seems to be a panic attack; feeling that I'm going to faint, now legs are tingling, took blood pressure, but seems to be lower than normal; had headache and took aspirin; headache seems to be trying to return. What's going on?? Doctor: Thank you for asking on HCM.If I am your consulting physician I would suggest the following:1. Check your blood sugar.2. Check your Blood Pressure.3. Get your ECG done.4. If your heart status is fine, encourage you to increase your fluid intake.First we need to focus on other causes of such episodes and the common are low blood sugar ( or high as well) , changes in blood pressure and hydration status of the body. Panic attack is unlikely but if the above tests are cleared we need further evaluation for panic attack confirmation.I hope you find this helpful."
},
{
"id": 183409,
"tgt": "Is it advisable to wear braces instead of RPD?",
"src": "Patient: hi.... sir i wanna ask you regarding my teethsir, in an accident i lost my one of the front teeth now i used to wear RPD but i dont like my curve shape teeth.so i just wanna ask you that if i wear braces it can be ok or not.i will send you my pic... suggest me Doctor: Hello,Braces are recommended if you are unhappy with your bite and position of your teeth. You may notbe able to eliminate the space of a missing tooth and will not change the shape of your teeth.Your options for tooth replacement include fixed bridges and implants for a solution that does not require removal daily. A clinical exam with X-rays will give you the best ability to diagnose your options. Plan a visit to the orthodontist to discuss braces and the expected results.I am glad to look at a photo and assist you further. Thank you for your inquiry."
},
{
"id": 118112,
"tgt": "What is the remedy for hypochromic anemia?",
"src": "Patient: Iam 28 years old ,159cm height, 69kg weight, Iwas done cholesistectomy 2month ago, Ihave ahistory of mild microstatic hypochromic anemia, menstraution regural but its sever amount, still 3days, my skin colour still pallor although treatment taken, what can ido? Doctor: Thanks for contacting HCM. It appears that excess blood loss during menstruation is causing you to loose blood and thus anaemia. Please get yourself examined by a gynaecologist and get a ultrasound done to find out cause of heavy menstruation and then get it cured. Also start an Iron and folic acid supplement which should give you atleast 100mgs of elemental iron 2-3 times a day, till the hemoglobin becomes normal. Regards"
},
{
"id": 73380,
"tgt": "What causes low BP while treating neuroendocrine tumor?",
"src": "Patient: Hi, my blood pressure has decreased over the last few months and has been quite low in the last few days. My personal trainers were concerned and did an ecg to check heart and blood pressure. Blood pressure was 88/68, ECG was fine. Saw my GP yesterday and it was 110/?? but he said it wasn t too low and recent blood tests showed normal results. I m currently being treated for neuroendochrine tumour so have had lots of diarrhoea in the last few months. I think that may be the cause of the low blood pressure. Oncologist said tumours would not have affected my blood pressure. I am drinking plenty of fluids. The diarrhea is not constant so wondering if I should take some medication to stop the diarrhoea. I normally suffer from constipation and have haemorrhoids. Doctor: Although the blood pressure reading is low, it may not be dangerous depending on the symptoms that you had during that time and what your baseline blood pressure is. Some people walk around with a blood pressure of 80/60 and feel fine, this is just their normal. If this is dramatically lower than your normal blood pressure, then you should be evaluated for this. Diarrhea can cause dehydration, which can certainly drop your blood pressure."
},
{
"id": 210385,
"tgt": "Suggest treatment for delusional disorder",
"src": "Patient: hi, my son seems to have the delusion that he is an extra-terrestrial acting through my son,s earthly bodywith the purpose of helping mankind to higher levels. He has a history of drug taking but has recently become obsessive with health and raw food. He has difficulties being among people, especially it seems among those who woulld threaten his beliefs. He has recently moved in with me in the remote countryside. . cutting himself off from many friends; although he still visits family members and those close to him. He is becoming more critical of me with accusations he has lately written down in the form of letters to me. He is 26 and unemployed and has alot of freedom to spend his time writing a fantasy novel, meditating and preparing his raw food recipes. There is obviosly no way I could even suggest a treatment option as he sees himselfaloof and beyond my advice, and would,nt contemplate the idea that he was 'ill'. Is there anything I can do to help him, other than simply being around for him ? Andrew Doctor: DearWe understand your concernsI went through your details. I suggest you not to worry much. Instead of suggesting he does not have any super human abilities, suggest he has some and be with him. That is the way you should have followed in the first hand itself. Also try to get hold of some people around him to agree with him on some points which are not harmful to anyone. Gain his trust and then we can go further in the treatment. Please post a direct question to me in this website. Make sure that you include every minute details possible. I shall prescribe some psychotherapy techniques which should help you cure your condition.Hope this answers your query. Available for further clarifications.Good luck."
},
{
"id": 143911,
"tgt": "Is it correct to have the disc fusion between C6 and C7 causing severe pain?",
"src": "Patient: Hi. I had my c5 and c6 fused last year. I now have severe pain down my neck shoulder and down my arm and into my fingers. I had am MRI and it showed that I now have a problem between c6 and c7. Do u think it s a good idea to have all these discs fused. Doctor: HelloIt is fairly common to develop a disc problem above or below where a fusion has been done.As for whether you need a second fusion, that would depend on the actual MRI report and results of nerve conduction studies and a cervical myelogram"
},
{
"id": 160223,
"tgt": "pain under teeth can be cancer or not?",
"src": "Patient: I have a pain under one of my teeth and I am concerned that it may be cancer. It is on the gums where my bottom front tooth begins. Doctor: i have the same problem i have this squishy gum on the back of my mouth on a molar and it hurts. on the top layer though"
},
{
"id": 130845,
"tgt": "What could cause numbness all over the body?",
"src": "Patient: I went to a chiropractor after having some back pain then he adjusted my back and on another visit he pulled my legs. Ever since then I have experienced numbness on my feet now recently I m having numbness throught my body and muscle weakness. Had MRI of spine normal;EMG normal; but no answer yet. Still trying to figure out what s wrong. Had one neurologist mention Transerve Myelitis but no sure. Would anyone know what could be wrong? Doctor: In my opinion first start by checking your kidneys for kidney failure , i recommend a urine analysis if kidney is clear then you can suspect viral infections and autoimmune disease like (transverse myelitis, MS , Gallein baree , Lyme disease) Immunoglobulin tests and spinal tab are recommendedGood Luck"
},
{
"id": 7062,
"tgt": "How to know the time of ovulation ?",
"src": "Patient: trying to get pregnant . i got 30 days cycle. now am undergoing follicular study in follicular study today is my 14 th day my right ovary is about 15*15 mm. when will be my ovulation . Doctor: Hello Welcome to HealthcareMagic Exact time of ovulation can be told after serial USG monitoring throughout cycle. Generally ovulation is about 14th day but your gynae doctor by examintion tell you specifically. Have a Healthful Day.."
},
{
"id": 118484,
"tgt": "When can I stop the medicine, Wysolon, given for low platelet count and what are its side effects?",
"src": "Patient: Hi sir, am 22years old,,,,Daily am using 20mg of wysolon tablet at morng and nit times,because til 2years before itself I having less platelate count in my body..but I don't know what purpose to use this wysolon tablet..sometimes can I stop this tablet, immediately it showing teeth germ bleeding and clotting the blood in my body...what I do sir its my very biggest problem then how it possible to cure then how long to use this tablet??.anyother side effect will happen??? plz kindly ans me YYYY@YYYY this is my email id if passible wil u send answer to this id sir..... Doctor: Hi. Welcome to HCM.I beleive that u r suffering from ITP (idiopathic thrombocytopenic purpura) a condition of low platelet count.Idiopathic means unknown cause. In medical field we have something called Autoimmune disorder, means your body's own immune system is targeting and destroying your platelets. The treatment option in such conditions is giving immunosuppresant, that is what i guess your doctor is doing. This low platelet count is the reason y u r getting gum bleeding also. By taking wysolone (it contains a steroid prednisolone) usually u will be symptom free. If i am correct that u r suffering from ITP then dear there is no cure for this, but your doctor can stop the progression of the disease by giving steroids. But remember this dont ever stop wysolone tablet on your own, it is too dangerous to do so. Consult your doctor and then stop. If u still have query pls feel free to ask. Regards and wish u good health."
},
{
"id": 48336,
"tgt": "Suggest treatment for kidney infection",
"src": "Patient: Hello, I have a kidney infection and have been on ciprofloxacin for 8 days now. I had severe pain on left side. As things have cleared up, I still have pain coming and going. This is the third kidney infection I have had. I am 34 years old. I am worried about the pain I am still feeling in my left kidney. Can you please let me know if it s normal to have residual pain this long a after?I am on the antibiotic for 21 days. Doctor: Yes my dear, pain may persist even after resolution of the infection. But if im ur treating doc, then I'll do ur urine examinatin to confirm and document that infection get resolved or change antibiotic accordingly if not."
},
{
"id": 64207,
"tgt": "Suggest treatment for lumps on leg & knee",
"src": "Patient: I am 31 yrs old, in great health.I just noticed a lump on the outside of my lower left leg , right below the knee area. It's about the size of a half of lemon, but I just noticed it a few days ago. Not sure if I just didn't notice it, or if it is rapidly getting bigger. It is not painful and there is no skin discoloration, it is not sensitive to touch either, Doesn't feel hard Doctor: Hi Thanks for your querry to HCM.You mostly suffer from - Morant Bakers cyst of left knee.?Sebaceous Cyst- Treatment 1-Orthopaedic Surgeon would Excise it and would treat it after confirmation2 If its Sebaceous Cyst a Surgeon would b needed to Excise it.Hope this would help to treat painless soft left knee Lump.Wellcome for any note help from me.Have a Good Day."
},
{
"id": 54759,
"tgt": "How to increase the liver enzymes with crohn's disease?",
"src": "Patient: Hi. I suffer with chronic joint pain was recently diagnosed with Crohn's disease. Also after routine lab test found liver enzymes were 750. Don't drink or use any tobacco products or illegal drugs. Liver enzymes have come down to 168 and are holding at that. Any suggestions? Doctor: Hi thanks for contacting health care magic.Noted you have crohn disease...with elevated liver enzymes....Here you have to continue with drugs for crohns disease...And take care to rest liver by low fat diet...Avoid strenous exercise....Fruit juice more.....Avoid excess fried and fatty food.Non veg totally prohibited....You can take adequate protein diet to regain muscle mass and fast healing....Avoid alcohol....If your enzymes remain elevated still then I want to rule out secondary amyloidosis induce by chronic inflammatory disease like you have crohn ...Biopsy might need.Take care....Dr.Parth goswami"
},
{
"id": 33895,
"tgt": "Suggest treatment for cough and cold in a child",
"src": "Patient: Hello sir I have 2 son 1 is 3.5 & 2nd is 7 the month both are regular suffering from cold & caugh and taking prescribed medicine by doctor getting normal for 2 or 3 day s but next again suffer by cold & cough so sir what would be ur suggestion ? Pl se ansr me ----- Doctor: Hello dear,Thank you for your contact to health care magic.I read and understand your concern. I am Dr Arun Tank answering your concern.Cough and cold in child's are because of the viral infection and need not requires antibiotics therapy every time.It will be given only once. Since both the child's are kids the infection is keep rotating in both of them. So symptomatic treatment is required for both the person.You can give both of them paracetamol suspension and chlorpheniramine suspension. The combination is available in the market. Please take the advise of your doctor before you take the drug.Please give them good hygiene practice. Good hand washing and keep the handkerchief while sneezing can prevent the cross infection to one another. You can also keep both of them separate while sleeping this also prevent cross infection.Give them one glass milk or one egg daily. This will help to boost up their immunity.I will be happy to answer your further concern on bit.ly/DrArun.Thank you,Dr Arun TankInfectious diseases specialist,HCM"
},
{
"id": 27215,
"tgt": "What causes pain in the breast & chest?",
"src": "Patient: Hello Doctor I have a pain in my left chest and breast since yesterday. Even earlier, i have experienced this pain and i got a breast sonography done,results were normal And doctor said it was muscle pain only. I have also got ECG done, that too shows normal. Now again for the third time, I am experiencing this pain. I think it could be chest muscle again, I have 3 gels with me , which one should i apply to check if there is relief. - Infen Gel, AR5 Gel and Volitra . Thanks a lot Doctor: Hi,It seems you have muscle pains or neuralgic pains, which can be caused also by spine problems.You can use Volitra gel, it will relieve the pain.Wishing you good healthRegards,"
},
{
"id": 15671,
"tgt": "Have rashes. Had ouncher wound before by pitbull. Wound is not healing fast. What to do?",
"src": "Patient: I was attacked on Valentines day this year by a pitbull and sufferedpuncture wounds that have been carefully treated. Yet this week I have developed a rash. Why? And what amount of time allows for true lightening from these types of puncture wounds?I am so worried because the doctor fears my healing is not well.But my physical therapists agreed it was. Just feeling unsure and wishto be educated on why this is happening?Thank you,Shelley G Doctor: Hello,Thanks for the query.Deep wounds generally take time to heal.As it was a bite injury there is high possibility of infection.you require treatment with proper antibiotic.If possible get a culture of material from wound,to see what is the infective organism.Let me know if you have any other doubt.you can ask a direct question to me on this forum, following the below link.https://urldefense.com/v3/__http://www.healthcaremagic.com/doctors/dr-rahul-kumar/64818Wishing__;!!Mih3wA!SBzm6_kI6hCZ58EPH6N_05MFfiPbxWXT0a2TJCdFQObRWm5mV5ur7hUOMa8clQ$ you a good health.Thank you"
},
{
"id": 122798,
"tgt": "Suggest treatment for neck & head pain",
"src": "Patient: I ve been experiencing neck and head pain. It feels as if something was tugging down on my neck muscles. My head feels as if it was too heavy for my neck to carry. It gets better with rest and by using a neck pillow. I wake up without pain, but it comes back later during the day. Doctor: Hello, It could be a musculoskeletal pain. As first-line management, you can take analgesics like paracetamol or aceclofenac for pain relief. If symptoms persist better to consult an orthopedic and plan for an MRI scan. Hope I have answered your query. Let me know if I can assist you further. Regards, Dr. Shinas Hussain, General & Family Physician"
},
{
"id": 10002,
"tgt": "Can Trassgain lotion, Androfol-M and K2 lotion help in treating severe hair loss?",
"src": "Patient: Hello, I am 27 year old male and I am suffering from hair loss. I consulated dermatologist and they told you are in initial stage of AGA - C1. They have prescribed Trassgain lotion, Androfol-M and K2 lotion. Is it safe to use these drugs and Can I increase my density of hair with these solutions. None of my ancestors have been bald and can you also prescribe some diet from which I can increase my hair. Doctor: Hello, The medications you are taking are very useful for hair growth. There doesn't exist a special diet to improve hair growth but some vitamins can help. Use B complex vitamins, biotin and folic acid can help somehow. Hope I have answered your query. Let me know if I can assist you further. Take care Regards, Dr Olgeta Xhufka , General & Family Physician"
},
{
"id": 223514,
"tgt": "What causes headache and vomiting after taking i pill?",
"src": "Patient: sir i am taking unwanted 72 at the 7th day of my mensens time after one week bleeding will be shown and i am suffering from headacqe and also vomting sensation since i using ipll is it is any side effect of that pill please give right solution my problem Doctor: Hi, I think it is a side effect of I pill. If nausea is severe, you can take some anti-emetic. For headache, you can take paracetamol 500 mg. Consult a doctor if symptoms persist. Hope I have answered your question."
},
{
"id": 88378,
"tgt": "What causes abdominal pain and dark stools?",
"src": "Patient: this morning i woke up eith a very sharp pain in my stomach. i didnt know if i was just hungry or in just in pain. i tried avoiding it as i thought it was because i was hungry but then i felt like i needed to vomit. so i went to the toilet and as i was about to poo, instead if the normal my poo came out as water and little dark stuff Doctor: Hi ! Good morning. I am Dr Shareef answering your query.A few more information from your history if such pain occured earlier also and if you had any pain killers recently would have helped me in planning your management. Anyways, in case I was your doctor, I would send your stool for a routine test for ova and cyst and for occult blood test, and prescribe you with a proton pump inhibitor, an anti emetic, and a liquid antacid. On the dietary front, I would advise you for a bland diet, and avoid any kind of spices, fried or fast food, along with avoidance of alcoholic beverages and smoking. If the symptoms continue, and the occult blood test was positive, I would consider referring you to a gastro-enterologist for a possible endoscopic evaluation of your intestinal tract to rule out any active pathology.I hope this information would help you in discussing with your family physician/treating doctor in further management of your problem. Please do not hesitate to ask in case of any further doubts.Thanks for choosing health care magic to clear doubts on your health problems. I wish you an early recovery. Dr Shareef."
},
{
"id": 41180,
"tgt": "Is IVF the only option to concieve if husband has oligoasthenoteratozoospermia and wife has PCOS?",
"src": "Patient: Dear doctor , I m 36 and my wife is 33 and her weight is 70kg . we ve been married for past 1.5 years. I have oligoasthenoteratozoospermia and my wife has PCOS. My urologist said that my FSH,testesterone , luteinizng hormone and prolactin level is normal. the chromosome culture and karyotyping culture report says that I have a polymorphic variant on the chromosome 13 . My US scan report of scrotum says no features of varicoceole, inflammation or a neoplasm. Is IVF the only option for us to concieve a child? can my sperm count be improved? Doctor: Hello, your sperm count cannot be improved and rather you might not get success in IVF either so in my opinion go for IVF and along with that get PGD testing done too.In case you have any questions in future you can contact me directly on http://bit.ly/drmanishajain"
},
{
"id": 143291,
"tgt": "What causes tongue twitching and muscle movement problems?",
"src": "Patient: I ve been having a problem with muscule movement and tongue twitching and my neurologist do not give a straight answer about what it could be. I know I have an abnormal EEG and also I have epilepsy since I am Seven years old, I would appreciate if you can at least tell me what do you think it could be happening in my brain Doctor: Hi,Regarding your concern, I would explain that your symptoms could be related to epilepsy or any possible adverse effects caused by your current therapy. I would like to know your actual therapy and doses. The abnormal EEG is related to the seizures. But, I would recommend performing a brain MRI and some lab blood tests to investigate for the possible causes (complete blood count, blood electrolytes, thyroid hormone levels, kidney and liver function tests). Also the plasma levels of your antiepileptic drugs may need to be monitored (based on the type of the drug). You should discuss with your neurologist on the above issues. Hope to have been helpful. Let me know if I can assist you further.Best wishes, Dr. Aida"
},
{
"id": 12100,
"tgt": "What should i do for loss of pigmentation on legs ?",
"src": "Patient: , I am 25, 9stone and 5 6 with no health issues orther than low iron sometimes. I am noticing increasing patches of whiter skin on my legs. I am quite pale anyway but these patches seem to be growing and even if I am in the sun the patches dont gain any colour to match the rest of my skin. It makes the rest of my skin seem dirty. Could you please advise what this is or if there is something I should do? Doctor: Hi Thanks for using Health Care Magic, This may be due to a skin infection,allergy or autoimmune disorder. If it is a fungal infection like tinea versicolor there might be itching and scaling of the skin.This can be treated by antifungals.It can be due to vitamin deficiency and can be treated by b complex tablets. Sometimes rarely it might turn out to be vitiligo,or any precancerous skin lesion.It is advisable to consult a skin specialist and undergo skin tests for further treatment."
},
{
"id": 142894,
"tgt": "Why do I have hot and cold sensation at the same time?",
"src": "Patient: Good day doctors, my GBS patients, after treatment and medication for nearly four months now I still feel very cold like ice cubes hold out, and at the same time I also feel the heat hit the water as hot as chili ... I asked why what and whether the heat and cool it off? What therapinya? Doctor: Hi, Welcome to HealthCareMagic.com I am Dr.J.Mariano Anto Bruno Mascarenhas. I have gone through your query with diligence and would like you to know that I am here to help you.Q : Why do I have hot and cold sensation at the same time?A : You are having this sensation because of the the damage cause to the nerve roots by GBS. This may take 6 to 18 months to be completely curedHope you found the answer helpful.If you need any clarification / have doubts / have additional questions / have follow up questions, then please do not hesitate in asking again. I will be happy to answer your questions. In the future, for continuity of care, I encourage you to contact me directly in HealthCareMagic at http://bit.ly/yyyyyyyyyBest Wishes for Speedy Recovery Let me know if I can assist you further.Take care."
},
{
"id": 176461,
"tgt": "Suggest diet for a 7 month old child",
"src": "Patient: Hi, i have a seven month old daughter. She doesnt really seem to like baby foods as such. She likes to eat what we eat with all those spices n seasonings but in a mashed, easy to swallow consistency. Can i add cornflakes in her diet with whole milk. She is still on breastfeed. She loves apple but apples, pears n potatoes constipate her. Also which other dals can i add in her diet besides the normal yellow moong dal? Doctor: Hi...At this age you can start giving Cerelac stage one rice or Nestum rice stage 1 etc. All are same in calories and energy. Only thing is palatability which is subjective for each kid. Other home-made food options will be - 1. Daal and rice well cooked and made into a porridge along with ghee2. Boiled apple or banana and mashed into a paste.3. Any made cereal porridge preparation with additional ghee added.Remember certain principles like -1. Add one food material / cereal or fruit per week. This will be useful because if he develops vomiting or diarrhoea, we will know what to avoid.2. Add vegetables and fruits made into a paste surely to avoid constipation.3. Do not add too much sugar as it may cause diarrhoea.Hope my answer was helpful for you. I am happy to help any time. Further clarifications and consultations on Health care magic are welcome. If you do not have any clarifications, you can close the discussion and rate the answer. Wish your kid good health.Dr. Sumanth MBBS., DCH., DNB (Paed).,"
},
{
"id": 179498,
"tgt": "Suggest treatment for child suffering from cold and cough",
"src": "Patient: Dear Doctor, My daughter aged 4 year old has cold and cough, I have used cetrizine & ascoril-D syrup, but no result.. kindly request you to suggest me the best syrup which cures both cold&cough as it is tough to give medicine. Thanks.. Nagendra Arveti - 0000 Doctor: Hi Dear welcome to the HCM,Antihistaminc decongestant medicine along with the paracetamol (like sinarest)to be given on 8 hourly basis with proper recommended doses.Cough expectorant with bronchodiltors to also be added. Steam inhalation.Some times if cough is sever then nebulized form of bronchodilators also will be of help.Antibiotics may also be considered if symptoms are gradually increasing with the consultation of pediatrician.Hope the query is answered.thanks"
},
{
"id": 219670,
"tgt": "Suggest treatment for absent fetal heart beat during pregnancy",
"src": "Patient: i am a seizure disorder patient presently on oxycabazepine.i am pregnant but the growth is slow. my LMP was on 19/12/2009 (have irregular periods).according to that my baby shud be 9 weeks 06 d.but the recent usg shows 6 weeks 6 days.last week also had a usg which showed 6 weeks 2 days.fetal pole has appearedthis week but no heart beat.moreover i culd have conceived only after 6 of jan since had intercourse only after that.my doctor says to go in for D&C.please advice. Doctor: Hi there,,Since you have irregular periods its possible that you must have ovulated late and that's why the pregnancy is 6-7 weeks at present. So do not worry about the dating of the pregnancy.More importantly since the fetal pole has appeared when compared to the previous report it would mean that there is some progress in the pregnancy.I would advise you to wait and get another scan done to check for fetal heart after 10 days of this one.This will tell for sure if the pregnancy is viable or not.Also since you have a seizure disorder and taking medications for the same , you need to consult your neurologist and see if the seizure disorder is under control.It is very important that your medical condition is under control, do not stop taking the medication as seizures during pregnancy can have an adverse effect on the baby.You are also more likely to have a child with birth defect a slightly more chances about 35 more. So it is very important that you get regular scans and tests done to check for neural tube defects, heart defects .This risk can be decreased to a certain extent by taking high dose 5 mg folic acid.I would advise you to visit a neurologist and repeat the scan after 10 days before deciding on discontinuing the pregnancy.Hope this helps.Regards."
},
{
"id": 161016,
"tgt": "How to treat stomach pain and intermittent rise in temperature?",
"src": "Patient: Hi, My 2yr old son is having a tummy ache with slight on/off rise in temperature for 3 days now. What might be the problem? Me and my wife are getting really worried and decided to take him to the doctor later as soon as i get off work. Please I need your help. TIA Doctor: Hi, Fever of a few days without any localizing signs could as well a viral illness. Usually, rather than fever, what is more, important is the activity of the child, in between 2 fever episodes on the same day. If the kid is active and playing around when there is no fever, it is probably viral illness and it doesn't require antibiotics at all. Once the viral fever comes it will there for 4-7 days. So do not worry about duration if the kid is active. Paracetamol can be given in the dose of 15mg/kg/dose (maximum ceiling dose of 500mg) every 4-6th hourly that too only if fever is more than 100F. I suggest not using combination medicines for fever, especially with Paracetamol. Hope I have answered your query. Let me know if I can assist you further. Take care Regards, Dr. Sumanth Amperayani, Pediatrician, Pulmonology"
},
{
"id": 166259,
"tgt": "What causes stomach pain and loose motion?",
"src": "Patient: Hi, may I answer your health queries right now ? Please type your query here... my son is 4 years old he is not prooerly going motion daily now its three days he did not sit in toilet and he is having pain when motion passes.motion is also very thick Doctor: hi, most common cause thick stools and not passing motions clearly is stomach infection. Other cause could be worm infestation. Deworming in child should be done. Give syrup Albendazole 10 ml once at night before sleep. if problem persists, then antibiotic like metronidazole should be started. Prebiotic like enterogermina ampule should be given once a day for 3 days to increase gut immunity. if no improvement in 24 hours then get child examined by a doctor because antibiotic needs to be started and so that we may not miss any important finding. Take care."
},
{
"id": 17774,
"tgt": "What can cause irregular heart beats and excessive weight loss?",
"src": "Patient: My son is 21 yrs old with a near syncope episode and irregular heart rate yesterday. He lost 40 lbs without trying within a couple months. The Dr. said his blood tests xrays and EKG were okay and sent him home. As a baby and until his teenage yrs he suffered from severe asthma and had to receive IM Epinephrine frequently. His maternal grandfather is in remission from Lymphoma. Any suggestions as to cause? Doctor: Hello!Welcome to Ask a Doctor service!I passed carefully through your question and would explain that his excessive loss in the weight is really concerning and should be investigated for a possible metabolic disorder. For this reason, I would recommend consulting with his attending physician for a physical exam and some tests: -a cardiac ultrasound- an ambulatory 24-48 hours ECG monitoring for possible cardiac arrhythmia- thyroid hormone levels for possible thyroid gland dysfunction- cortisol plasma levels for possible adrenal gland dysfunction- blood electrolytes for possible electrolyte imbalance. Hope you will find this answer helpful!Wishing all the best, Dr. Ilir Sharka, Cardiologist"
},
{
"id": 67872,
"tgt": "What causes painful, itchy growing lumps on back of head?",
"src": "Patient: Hello, this morning i noticed two lumps on the back of my head, one on the very bottom of my skull. They feel hard but are painful if i push them too much, i wasn t concerned until my entire scalp started to each and being tender in other places, i ve also noticed they ve grown over the day, or at least feel like they re bigger. I m a 15 year old female about 5.75 and about 65kg Doctor: hi and welcome to HCMthis is probably ateroma and this is benign subcutaneous lesions which needs to be treated surgically if there is inflammation or grow to fast. so see your suregon about further therapy Wish you good health. Regards"
},
{
"id": 187500,
"tgt": "What is the remedy for a runny nose after the removal of wisdom teeth?",
"src": "Patient: I had all four of my wisdom teeth removed in late December of 2013. I have been sick for the majority of the time after the surgery, with a constant runny nose among the most annoying symptoms. I was wondering why this is, and if there is a corrective surgery to make it stop or if I will be like this forever. Doctor: Hello,Thank you for consulting with HCM.As you are mentioning that after removal of your wisdom tooth you are experiencing these kind of symptoms, it looks that you are having some sinusitis problem.Actually these wisdom tooth are associated closely to the sinus, in case of surgery there can be accidental trauma to the sinus.Better once you should visit an ENT specialist to get complete examination of the sinus done with x-rays, as it will confirm the condition.Hope it will help you."
},
{
"id": 126227,
"tgt": "What causes pins and needles sensation in the heel when I stretch?",
"src": "Patient: I am having pins and needles pain on the back of my left heel when i stretch too far, such as when I do downward facing dog in yoga. I have never had this pain before. Should I see a foot doctor? I think it has to do with my achiles tendon. The muscle is very tight and it only hurts when I stretch too far. Doctor: Hello, Consult an orthopaedician and get evaluated. The possibilities are plantar fasciitis and neuropathic causes. Hope I have answered your query. Let me know if I can assist you further. Take care Regards, Dr. Shinas Hussain, General & Family Physician"
},
{
"id": 85853,
"tgt": "Does Fluconazole interfere with Flomax?",
"src": "Patient: I am currently taking flomax for urine retention issues. I also have a yeast infection in my groin area and my doctor is recommending i also take an oral fluconazole. I read that fluconazole can interact with flomax and that a reduced level may be required. But it didn t say if I need to reduce the flomax or reduce the fluconazole. What is the recommendation course of action here? Doctor: Hello, Fluconazole and Flomax do not have a known interaction. You can continue taking both medications the same. You should not be worried. Hope I have answered your query. Let me know if I can assist you further. Regards, Dr. Dorina Gurabardhi, General & Family Physician"
},
{
"id": 105914,
"tgt": "Which hospitals in Maharashtra treat asthma and allergies ?",
"src": "Patient: SIR... CAN U SUGGEST ME ANY ALLERGY HOSPITAL IN AHMEDNAGAR (MAHARASHTRA) OR AURANGABAD (MAHARASHTRA) Doctor: Thanks for the query What exactly do u mean by an 'allergy hospital'? what allergy are u suffering from? Is it asthma? Most allergies are treated by a good internal medicine MD Have a healthy living"
},
{
"id": 107604,
"tgt": "Suggest treatment for lower back pain and constipation",
"src": "Patient: hello im a 49 y/o woman with no history of back pain yet 10 days ago i developed a stiffness and pain in my lower back more prominent on my left side. i saw a chiropractor for sim treatments and adjustments and it improved but i noticed constipation symptoms and increasing soreness in lower left. could this be a gastro issue- over last two days i've incresesd water really pushed it today and got somwe walking in but only produced liquid stool, pain is increasing. Doctor: Hello I have gone through your question and appreciate Your concern. For constipation u can try some laxatives like cremmaffin syrup.Back pain and constipation doesn't seems to be related.For back pain ucan use local Analgesic gel. Analgesic tablet can also be tried. Muscle relaxants and analgesics will help you. Lie on flat hard bed. Hot fomentation thrice a day. Avoid bending forward and lifting heavy Weights. Thanks. U can write me back for any query."
},
{
"id": 186975,
"tgt": "How to fill the gap in my front teeth?",
"src": "Patient: how i feel the gap in my front teeth ? i am 19year girl actually i have an accident case it happen when i am 18year so give me the suggestion which is the best option for filling the gap in my front teeth in minimum cost becz i am belonging to the middle class family swee Doctor: Hi. Welcome to Healthcaremagic.I read your query. The gap in front tooth can be closed via numbers of ways but firstvit require proper check up.If its only this gap and rest of the dention is perfect, and the gap is small, you can get crown or laminates to close it. Since you mentioned trauma as its cause, you need to get an x-ray done of these teeth to check if there is any pulp damage or not. You may require RCT as well. Else you need to consult an orthodontist, who will study your dentition, OPG and tell if you require fixed braces or clasps and springs can help too. Hope the answer helps you. Thank you!"
},
{
"id": 31192,
"tgt": "Suggest remedy for a gas bubble appearing near the groin",
"src": "Patient: I have had a 'gas bubble' near my groin for almost 2 years now. It hasn't gotten any larger, and is not painful. It comes and goes, and I am able to 'push it back in' when it protrudes. It typically protrudes when I 'push', for instance when I am going to the bathroom. What is it, and what can I do about it? Doctor: Hello,Thank you for your query.Your description is suggestive of an Inguinal Hernia.When there is a defect in the abdominal muscle wall, the internal organs can push through it and can be felt & seen on the body. The intestine is usually involve but can include other structures too.A hernia can be asymptomatic as yours but can become complicated if a too much of an organ is pushed out and is strangulated by the opening defect in your abdominal wall. This is a life threatening complication.I strongly recommend you see a general surgeon who could assess the size of the defect and advise a proper plan of action.I would do a physical exam, assess your medical /personal history and risk factors. I would also request an Ultrasound or a CT scan of the Abdomen.Depending on the findings, a treatment plan will be advised.Surgery is the only definitive treatment. a wait and watch plan may be an option if defect is very small.Avoid 'pushing' or straining. don't lift heavy objects or do any activity that makes the hernia appear. This can enlarge the defect and cause strangulation as mentioned aboveIf you develop abdominal pain, blood in the stools, fever, see your doctor right away or go to the emergency.Hope this helps. Please get back to us if you need further clarification.Wish you good health"
},
{
"id": 211129,
"tgt": "Can ashwagandha brahmi combo cause anxiety?",
"src": "Patient: I've tried ashwaganda brahmi combo and the first week I felt so much better,had more energy ...etc,then when I got my period had very intense anxiety simptoms.Do you think this might be do to those supplements.I started those to help with stress!Thank you Doctor: Dear User,Thanks for using health care magic.Generally Ashwaganda and brahmi are used for anxiety symptoms and there is no evidence that these itself can cause anxiety symptoms.Thanks"
},
{
"id": 145452,
"tgt": "What causes seizures, blurring of vision, weakness and severe headaches?",
"src": "Patient: A boy of 13 years, never had history of epilepsy, started getting Absence Seizures , since 2 month. It starts with eye blurring, cannot stand, drops down without conscious. It lasts for 1 to 2 minutes. When he regains conscious, he complaint of severe head ache and weakness. No other symptom like body stiffening or stroking is observed. It started once in several days, now increased the frequency to 3 per day. He has changed his school to Chaitenya IIT sylubus for 8 th class from this June. A neuro physician prescribed TOPAZ 25. Can you advice on above. MRI, CT, EEG are found to be normal. Doctor: I think there is a mistake in your description, because if your son has absences the EEG can't be normal. The main drug to treat absence seizures is ethosuximide 500mg/day at first,but of course you have to always consult his doctor!"
},
{
"id": 63979,
"tgt": "What causes pain in armpit and lump in nipple?",
"src": "Patient: hi i have recently been getting a pin in my arm pit and i felt around adn i have a lump thats not very big but hurts a lot to toutch and i also have the same thing in my left nipple.please help is this serious. i am a 21 year old male very activethanks you Doctor: Hi, dearI have gone through your question. I can understand your concern. You may have fibroadenoma, fibroadenosis or fibrocystic disease or some other benign breast lesion. Chances of cancer is very rare at your age. Still you should go for examination and if needed go for biopsy of that lump. It will give you exact diagnosis. Then you should take treatment accordingly. Hope I have answered your question, if you have doubt then I will be happy to answer. Thanks for using health care magic. Wish you a very good health."
},
{
"id": 101279,
"tgt": "Is it asthma or valve disease if inhaler is giving me relief?",
"src": "Patient: sir, i am suffering from shortness of breath. doctor ask that i am suffering from asthama as well as valve disease, advised me replacement of valve. one week ahead, i started taking inhaller and thereafter i am feeling much better from the disease of shortness of breath. there is no other problem with me. i am suffering only from shortness of breath. pleased advised, if inhaler is giving me relief then is it asthama or valve disease. Doctor: Hello dear,As per the case history mentioned in your post, the symptoms can be attributed to Asthma.Asthma is caused due to broncho-constriction (obstruction of smaller airway passages) which is indicative of Hyper-responsiveness of air passages.Management consists of:1. Asthalin+ Seroflo inhaler- provide symptomatic relief by causing broncho-dilation (dilating the smaller airway passages, relieving the obstruction & increasing airflow to lungs)So, you can use it whenever you are having an acute attack.2. Montelukast preparations- used as a maintenance therapy to relieve symptoms of asthma.3. Antihistamines like Cetrizine can also be used to provide symptomatic relief.4. Need for anti biotics can arise if there is a secondary infection. But this should be taken only under the guidance of your Physician.5. Also protect yourself from exposure to cold, dust or other allergens.Wish you a good health.Take care."
},
{
"id": 180822,
"tgt": "How can severe toothache be treated?",
"src": "Patient: My husband is on day 6 on antibiotics for an infection at the root of a tooth. Today his pain has increased to an intolerable level and his face is swollen, temp is 99.2. It s 10:45 p.m., so can t call his regular dentist. I m trying to decide if he go to the ER or wait till tomorrow. Doctor: Hello and Welcome to HCM, inhabe gone through your query.The history suggests that your husband\u2019s tooth is infected with a periapical lesion (most probably). As you said he took antibiotics for 6days now. I dont think he needs to take them anymore. I would suggest you to make an appointment with the nearest dentist, as the tooth will more likely undergo Root Canal Treatment. The dentist will take an X-Ray of the involved tooth and will accordingly come to conclusion as if it might need a RCT or Extraction. Meanwhile take analgesic medicine only to relieve the pain and not antibiotics as he has completed that course."
},
{
"id": 213069,
"tgt": "Same sex relationship, one partner depressed, angry, suicidal, frustrated. Treatment?",
"src": "Patient: hi i have been with my partner for three years this is my first same sex reltionship and can honestly say i am totally happy though we have some problems my girlfriend lost her mum at a young age and blames herself she has a son who is fifteen and talks to her like crap i have four children myself that live by rules though are not strict however we have a few good months then all of a sudden her son plays up and talks to her like crap she tells him off but then feels bad goes into depression and starts cutting herself i keep helping her through but am t loss because then she starts on me and my kids about silly things like washing etc i work thirty nine hours a week cook take kids too and from school plz help Doctor: Hello and welcome to Healthcare Magic. Thanks for your query. You partner may be having emotionally unstable or borderline personality traits which may be the reason for her mood instabilities, poor coping skills, low frustration tolerance and self-harming behaviour. I would advise you take her to a psychiatrist for a detailed psychological assessment and further treatment. There are effective treatment options - in the form of medication or counselling / psychotherapy which can help can handle these emotional problems and depressive symptoms. Wish you all the best. - Dr. Jonas Sundarakumar Consultant Psychiatrist"
},
{
"id": 89119,
"tgt": "Suggest treatment for severe stomach pain",
"src": "Patient: my son has been experiencing severe stomach pains since he had colostomy surgery. He has had several CT scans and colostomy but they have all been negative. Drs think he has scar tissues and had prescribed pains meds but doesnt want to any longer so they suggest he takes an acidophilus supplement. Do you agree or do you have any other suggestions. Thank you Doctor: Hi ! Good morning. I am Dr Shareef answering your query.The pain could be due to some kind of sub acute intestinal obstruction due either to adhesion formation inside the abdomen or due to partial herniation of part of the gut nearby the colostomy site. This can be judged only by a clinical examination at the time of pain abdomen, and also a plain X ray of the abdomen in upright posture could possibly help in it. However, the pain could also be due to the original lesion for which a colostomy was performed. Ingesting acidophilus supplement would not do any harm to him and so could be tried. If I were your family physician, I would refer either to your original surgeon, or would not hesitate to take a second opinion for a judgmental decision by the doctor.I hope this information would help you in discussing with your family physician/treating doctor in further management of your son's problem. Please do not hesitate to ask in case of any further doubts.Thanks for choosing health care magic to clear doubts on your son's health problems. I wish him an early relief. Dr Shareef"
},
{
"id": 91828,
"tgt": "How to stop pain and sensitivity around navel when pressed?",
"src": "Patient: Hi. I have had some pain or sensitivity around my navel area for 2-3 days now. It seems to hurt, when I press in that area or lay on my stomach. I have had bowel movements but not everyday. I m not sure what it is. Any ideas or solutions on how to make it stop being so sensitive? Doctor: Hi. There is a possibility of an infection in the naval area or the intestines beneath.You may need to take a course of an antibiotic and see a Gastro Surgeon for further evaluations on clinical examination.Sometimes the mild infection is the cause for such tenderness."
},
{
"id": 212715,
"tgt": "Severe OCD, depression, anxiety, get flu like symptoms with viibryd. Medicines without side effects?",
"src": "Patient: i have severe ocd ( germs ) and severe depression and anxiety..i have been put on just about every psych med there is..my dr. had just put me on viibryd and it made me very ill, flu like symptoms, i take meds for my stomach issues nexuim and promethazine and was wondering if there are any meds out there that would not upset my stomach nor have alot of side effects? Thank-you Doctor: Hello...... Thanks for your query. Most of the psychotropic medication, particularly the anti-obsessional agents have abdominal side effects. Dividing the medication and using controlled release tablets is one option. The other option is psychotherapy (eg cognitive behavioural therapy/ exposure & response prevention) can be tried for OCD and they are devoid of side-effects since they are non-pharmacological methods. Wish you speedy recovery. Regards Dr Sundar Psychiatrist"
},
{
"id": 141458,
"tgt": "Does tremor in the finger indicate early signs of Parkinson\u2019s disease?",
"src": "Patient: I am 73yrs. old female, with controlled b/p and glaucoma. for last 6yrs since my retirement, I have noticed a tendency to walk or pull to my left after taking 5 steps My PF took an Mri of the brain, normal. two days ago while sitting I observed a sl. tremor on my right finger, next to the thumb. yesterday, the same thing happen, today I am having the same sl. tremor. Could it be early signs of Parkkinsons? Doctor: Hello and Welcome to \u2018Ask A Doctor\u2019 service. I have reviewed your query and here is my advice. Regarding your concern, I would explain that your symptoms could be related to different possible causes: thyroid gland dysfunction, drug adverse effects, parkinson's etc. Tremor is not a specific sign of Parkinson disease. Coming to this point, I would recommend consulting with a neurologist for a physical exam and some tests: - complete blood count - thyroid hormone levels - blood electrolytes. A brain MRI may be needed. Hope I have answered your query. Let me know if I can assist you further."
},
{
"id": 166589,
"tgt": "What is the treatment for loose motion in a baby?",
"src": "Patient: Hi my daughter is 9th month old and having loose motion from 2 days and i consulted doctor they said its due to food infection .. And my family members had given her little cake, sweet and fried items to her tongue to taste... can u please suggest me is this the reason for her loose motion? any home remedies to control? Doctor: Yes your daughter seems to have had a gut infection due to the food.You can give her some probiotics for 3 days and keep her hydrated with coconut water and butter milk. Not to worry."
},
{
"id": 72977,
"tgt": "What causes tight chest and back pain?",
"src": "Patient: hi Last week i was experiencing flu like symptoms and felt quite achey and tired runny nose etc however since the wkend I have been experiencing tightness in my chest and a sharp stabbing pain in my upper back when i breath..also a few migraines which i never get? 2 is seen in alcoholic liver disease.but have other symptoms too..i shall advise you to get a repeat LFT 1 month later.hope this is helpful.regards."
},
{
"id": 116421,
"tgt": "Can alcohol be consumed if platelet count is low?",
"src": "Patient: I just had a CBC incl. a platelet count checked, and the platelet count was low 118,000. I had some kind of flu virus that started showing symptoms last Sunday. I had a fever of 101-102 for 4 days with body aches and chills. All other CBC results were normal. My Creactive Protien was high and my band cells were high. I've never had a low platelet count before and my doctor could not explain it. I am feeling better and just tired, and have a graduation party to attend later today. Is it okay for me to drink alcohol when my platelet count is slightly low? Please advise. Doctor: Hi,Thanks for asking.Based on your query, my opinion is as follows.1. Alcohol is not advisable during low platelet count as any fall can lead to severe bleeding episodes - both internal and external.2. Increased C-reactive protein along with band cells indicate acute infection with inflammation. The viral could be turning to bacterial infection.3. You need to take rest, antibiotics and anti-pyretics. Avoid alcohol as it could lead to injuries and also possibly react with medications. Hope it helps.Any further queries, happy to help again."
},
{
"id": 224642,
"tgt": "Taking Previfem, history of ovarian cysts. Ultrasound showing ruptured ovarian cyst. Suffering from heavy bleeding",
"src": "Patient: I have a history of ovarian cysts , but have been on Previfem for over a year (a Mirena before that, several other pills throughout the years and at one point Depo-Provera ) . . . . had an ultrasound last week. The nurse said it was unremarkable, except for some free fluid. She said it could be from a ruptured ovarian cyst. How common are ovarian cysts if one is on hormonal contraceptives? The U/S was ordered for sequential breakthrough bleeding . I take my pill continuously and have 4 scheduled breaks a year. Didn t know I had to pay for this . . . cannot afford what I would pay for a co-pay . . . thanks anyway. Doctor: Hello. Thanks for writing to us. The heavy bleeding that you are having could be due to a hormonal disturbance caused by the ovarian cysts. Instead of regular combined pills higher dose pills will help you.I hope this information has been both informative and helpful for you. Regards, Dr. Rakhi Tayal ,drrakhitayal@gmail.com"
},
{
"id": 173731,
"tgt": "What causes bad breath?",
"src": "Patient: i know of a child that i take care and has a very bad breath the parents said that he eat a corner of ha foam bed and that is why he has a horrible smell that linger in the room what is his problem or what should the parents be worry about that smell. the boy is 2yr Doctor: Hi,Thank you for asking question on health care magic.Bad breath in a small boy is usually due to bad oral hygiene.Washing the mouth after each feed and use of listerine mouth gargle solution 2-3 times daily will cure the prroblem.A condition known as atrophic rhinitis may give rise to bad breath.Consult ENT specialist if not cured by above treatment.Hope this answer will serve your purposePlease feel free to ask any more queries if requiredTake careDr.M.V.Subrahmanyam MD;DCHAssociate professor of pediatrics"
},
{
"id": 209856,
"tgt": "What causes getting bright red when talk in front of people?",
"src": "Patient: Hi, in class when I've been asked questions by the teacher or asked to read something out in front of the class I go instantly bright red, or even when people come over to talk to me I can feel myself going warm inside and my eyes water.. It's getting really embarrassing and I'm a shy person Doctor: HelloIt looks like you are having performance anxiety. It is a type of anxiety disorder. All the symptoms you are having are seen in this condition. You will be benefitted by anti anxiety medications with psychotherapy. Please consult a psychiatrist who can help you with this.RegardsDr. SushmaConsultant psychiatrist"
},
{
"id": 33231,
"tgt": "What causes fever, sweating and breathing difficulty?",
"src": "Patient: i have been suffering from a fever for the last 5 days, i have been taking a course of AB but today seems to be the worst i have felt, my temp has gone from 37.2c one hour ago to 38c and i am sweating extreamly, my headache has worsened in the past 5 hours even after taking panadol and my breathing has become laboured, my sinuses are blocked and i am unable to sleep, i am wondering if i should go to the hospital or if this is the last of the fever trying to leave my body? Doctor: Hi, I had gone through your question and understand your concerns. Most probably you have upper respiratory tract infection.Headache is due to nose blockage.Take steam inhalation with added menthol or if available KARVAL PLUS TABLET , it will be very effective.Take one anti-allergen like LEVOCETRIZINE 5mg,use paracetamol for 3 times daily.Consult an ENT specialist soon.There is a possibility of malaria fever because you have fever,chills, headache rule out it by diagnostic test(rapid malaria kit method) if you not get relieved with in 2 days.Continue same antibiotic minimum of 6 doses(3 days). Hope this answers your question. If you have additional questions or follow up questions then please do not hesitate in writing to us. I will be happy to answer your questions."
},
{
"id": 7684,
"tgt": "Face full of pimples with pus and dark scar. Is it due to dust and pollution and remedy to remove marks?",
"src": "Patient: Hi, i am 26 yrs old, married, and a mother of 2 yr old daughter. Recently i hv developed pimples on my face. i dnt knw if it is pimples or acne . it starts like a small bubble, and turns to puss formation and when broken a white puss comes out. i hv got so many like that and at the end leaves a dark scar on my face which is very odd. Now that my face is full of marks and pimples, please suggest me some good medicines. my periods are regular. I feel it is because of the dust and pollution since i ride my bike to office. Please help me. Thank you. Doctor: Hallow Sushma how are you ? thanks for trusting on Health care Magic .You have to keep few things with you those are as follows 1. never try to squeeze your acne , it helps in spreading and delays the healing . 2. Squeezing results in scaring . 3. Wash your face multiple time with 2% salicylic acid containing face wash 4. Never apply oily creams over face . 5. Use Retino A 0.025 % at night .Clindamycin gel in day times twice daily . 6. Use fresh fruits ,proper exercise ,helps in early healing . You may also opt for IPL treatment for your ailment or Glycolic acid peeling by dermatologist It works wonderfully ,ok take care bye . Dr Prakash Roy"
},
{
"id": 86467,
"tgt": "Suggest treatment for painful bloating with abdominal pain",
"src": "Patient: I have had bloating and pain at times extreme in my abdomen and anus to the point of making it difficult to urinate over the pas week the bloating is to the point that my abdomen is tight and distended. I have been having regular bowel movements but have quite a lot of pain in my lower abdomen when having a bowl movement. I also have pain when I sit down that radiated from my rectum up into my lower abdomen. Doctor: Hello, a consultationen gastroenterology is n\u00e9c\u00e9ssaire to examine your bowel it is necessary to make a colonoscpie"
},
{
"id": 147479,
"tgt": "Suffering from movement problem and memory loss. Diagnosed as tumor in the brain or inflammation within the brain. Need some advice",
"src": "Patient: my husband s brother suffers from something in the brain that affects his movement and memory as he can not remember his children and he is always confused. he makes several kinds of x-rays some doctors diagnose that as tumor in the brain while the others say that it might be the inflammation within the brain because the tumor never exists in two halves of the brain as the x-rays show. i have a copy of x-rays and i can send it if you wish Doctor: Your doctor seems to be right.Chances of tumour in both the halves of brain simultaneously are very rare. He may be suffering from some inflammation which may be bacterial,tubercular or due to cysticercosis(A type of worm infestation of brain)It may be no active lesion,but may be just healed scar.I think a further evaluation by a physician/neurologist is required.treatment is required according to cause."
},
{
"id": 29193,
"tgt": "What does this widal test report indicate?",
"src": "Patient: Hello doctors, i m 29 years old M from India.i have typhoid since 2.5 months i have take medicines Oflox Oz for 15 days. I have a widal test that show that non reactive. Now after stop the medicines , i have again widal test and its shows Typhi-O is 1:160 and Typhi-H is 1:80. AH & BH are non reactive. I have no fever but i feels like fever in my body two or three times daily. Feel body heat from inside, legs and hand pain, weakness etc . But thermometer shows only 98.4 or98. .6 temps. Please tell me that am i typhoid positive or its gone? Doctor: HI SIR....SIMPLY WE WOULD LIKE TO INFORM U THAT WIDAL TEST IS NOT CONFIRMATORY FOR DIAGNOSIS ....IT JUST SAY THAT U HAVE ABS FOR TYPHOID BACTERIA....U NEED TO DO STOOL CULTURE TO BE TRUSTED OF DIAGNOSIS ....MAY BE U HAVE SOME STRESS OF TYPHOID FEVER"
},
{
"id": 52589,
"tgt": "Is surgery necessary for removal of non-cancerous cyst on liver?",
"src": "Patient: Yes. A friend of mine who is 74 who is in good shape and works out three times a week, and Yoga twice a week, had a CT scan, which shows she has a non-cancerous cyst on her Liver. Her doctors are not overly concerned, but she is very upset and is afraid that she might have to surgery. Can you suggest anything that I can said to her, to possible make her feel more at ease? Doctor: Hello and Welcome to \u2018Ask A Doctor\u2019 service. I have reviewed your query and here is my advice. Benign (non cancerous) cyst in the liver are generally harmless and does not require surgery. Hepatic surgery is an extensive procedure and is not recommended for a non-cancerous lesion especially for this age group. However frequent follow-ups are required and we a have to check whether the size is increasing or not. If the tumor grows rapidly or in case of symptoms like bleeding, we have to operate. Hope I have answered your query. Let me know if I can assist you further. Regards, Dr. Shinas Hussain"
},
{
"id": 78461,
"tgt": "Suggest treatment for hashimoto s thyroiditis and COPD",
"src": "Patient: 67yo woman recently diagnosed with Hashimotos and COPD. Quit smoking 15 yrs ago have been very active biking, hiking, walking. About 9 months ago I sarted noticing a slight shortness of breath. Now I have severe trouble breathing when I Stand up and try to walk. Two months ago I was walking around a little slower but still had quality of life. due to feeling tired I went to a dr who did a blood test and discovered the Thyroid problem. I was put on 50mcg of Levothyroxin. I had two stents put in ten years ago.EKG 2mos ago showed a normal heart for my age. 81% on the Bruce Protocol. Over the last two months my breathing has worsened to the point that I can t take more than 2 steps without severe loss of breath. As soon as I lie down I can breathe again. Have very little appetite and have lost 25 lbs since May. Doctor: Thanks for your question on Health Care Magic. I can understand your situation and problem. You are having weight loss and breathlessness on minimal exertion. All these indicate worsening of underlying COPD in you. So better to consult pulmonologist and get done 1. Clinical examination of respiratory system 2. PFT (Pulmonary Function Test). PFT will tell you about severity of the disease and treatment of COPD is based on severity only. You should start inhaled bronchodilators and inhaled corticosteroid (ICS). Enroll yourself in Pulmonary Rehabilitation center where chest physiotherapy and deep breathing exercises are done. All these are important in COPD patients. They will improve lung functions and reduce symptoms. So consult pulmonologist and discuss all these. Hope I have solved your query. I will be happy to help you further. Wish you good health. Thanks."
},
{
"id": 150671,
"tgt": "Mild indentation of the thecal sac, multilevel spondylosis, idiopathic scoliosis small air in sacroiliac joints . Indications?",
"src": "Patient: what does mild indentation of the thecal sac at l2-l3 and l3-l4 mean? multilevel spondylosis? scoliosis (kyphoscoliosis) ( idiopathic )? osteitis condensans ilii bilaterally? small air in sacroiliac joints bilaterally? reactive bone at the iliac margins of the sacroiliac joints bilaterally? conus ends at l1-l2? levoscoliosis (lumbar) with apex at l3? sacroiliitis? I am 32 year old female with all these things wrong with me. I had spine surgery from t11-l5. and MRI shows quoting notes there are postsurgical changes of the left lateral rod and vertebral body screw fixation from t11-l3 without change in hardware alignment Doctor: Mild indentation of thecal sacs are no problem unless some underlying pathology is active and may further produce selling or indentation as a result of inflammatory/space occupying selling.In your case since spine has been fixed internally by plate/screw it shall prevent mechanical torsion in that area.Under cover of anti-inflammatory drugs,the swelling isunlikely to increase and also control thec arthritic element in sacro-iliac joint.My only advice would be you keep on some form of medication for arthritis and avoid violent bending or jerk to back.Your physician should evaluate the nature of arthritis viz--osteoarthritis-infective arthritis,rheumatoid or septic or secondary to trauma-- all said and done your doc.should keep you covered up with periodical (3 monthly ) evaluation to keep underlying pathology contained'.He should evaluate neurological power of limbs from time to time"
},
{
"id": 37881,
"tgt": "Does viral meningitis spreads quickly?",
"src": "Patient: Hi, I am a teacher of 6 year olds. One of my students was hospitalized last week with viral meningitis and returned to school today. What are the chances of me or other students in class getting meningitis? How contagious was she 3-5 days prior to being diagnosed/hospitalized. My sister has a 3 month old, I am concerned about getting her sick. Doctor: Hello, Thnx to contact us. I understand your concern. If I am your doctor I advice you that it is not much contagious. To get entry into the central nervous system virus must enter into the body first. So no need to worry , if one host is compromised than and than virus will attack into central nervous system of the body. So no point of worry for your 3 month old baby.I will be happy to answer more of your concerns, kindly know me,Wish you a very good health at health care magic. Dr. Arun Tank. Infectious Disease."
},
{
"id": 161236,
"tgt": "What causes brown color urination in a child?",
"src": "Patient: My 4 years old son looks healthy, no fever, no pain, but yesterday he called me to look at his urine and I was so terrified when I saw the urine which looks like a strong coffee. Actually I am going to take him to a doctor this evening but what could this be. Doctor: Hi, Cola colored urine in a well thriving child could mean post streptococcal glomerulonephritis following upper respiratory tract infection or skin infection. Don't worry, it is a self resolving condition. Please go to a pediatrician for blood and urine tests and blood pressure check-up. Hope I have answered your query. Let me know if I can assist you further. Regards, Dr. Prasanna Lakshmi, Pediatrician"
},
{
"id": 21992,
"tgt": "What does slightly dilated right ventricle mean?",
"src": "Patient: Hi, may I answer your health queries right now ? Please type your query here...son has been to doctor twice with chest pain. had an echocardiogram yesterday and it show a slight dilated right ventricle or a murmur. what does this mean. Go to see doctor tommorrow and talk about results Doctor: hi therethere are lot of things to explain in this but ill try to explain you in short so that you can understand. heart has two sides right and the left, right side of the heart recieves used blood that returns from the body parts and right ventricles pump this blood to lungs to supply it with oxygen again and to make it reusable to the tissues now this oxygenated blood is returned from the lungs to left heart which pumps it back to whole body. thats how circulatation is completed.now coming to your sons case, right ventricle (rv) can dilate due to various reasons and we summaries them in two scenarios Pressure and Volume Overload. pressure overload happens when there is some sort of resistance in the normal blood flow as some lung patholohy and volume overload is due to extra amount of blood entering the right side of the circulation from some abnormal pathway like some congenital heart defect or some valve incompetency of right heart.Your son had episodes of chest pain which is not a good sign, he probably needs a bilateral lower limb doppler to rule out any sort of blood clot which could have embolised from there to heart and lungs leading to pain and a dialted RV and murmur.Good Luck"
},
{
"id": 176931,
"tgt": "What causes protruding lower rib cage in seven month old baby?",
"src": "Patient: Hey, my seven month old boy has a protruding lower rib cage. He is also usually restless. I give him five drops of Livol D a day but could it be that he just isn t swallowing it properly and has a difficiency? Went to see a doctor about this and there was an x ray taken of his chest area and because it didn t show anything abnormal the doctor said he wasn t worried, we would just have to keep an eye on this so to speak. I would really like a second opinion becouse his rib protrudes quite a bit. Thank you very much. Doctor: Hello Ur babay had protruding rid cage whichis is normal in x ray then u have to not worry about itLivol D is vitamin D drop which help to ibcresase calcium in the body If vit D deficiency is present then there is other sympton occre with itIt is normal. So do not worry"
},
{
"id": 183002,
"tgt": "Suggest remedies for infection and pain in wisdom teeth",
"src": "Patient: I have a few problems. The first I have had for many months, my bottom left wisdom tooth has come in, but didn't come all the way out. It allows food to get under the gums and get infected, its pusses alot. The second problem is my bottom and top back molars are turning black and starting to hurt, despite good brushing. My wisdom teeth are also coming in right behind both of those teeth. What can I do with no insurance? I'm in pain everyday. Doctor: Thanks for your query, I have gone through your query.The painand infection could be because of the gum infection over the wisdom tooth (pericoronal infection leading to space infection). The discoration can be secondary to the decay.Consult a oral physician and get a radiograph done to rule the amount of space present for the tooth to erupt if space is there for the tooth to erupt then the gum covering over the wisdom tooth can be removed.If the space is not available then you have to get the wisdom tooth removed. If I am your treating doctor, Mean while I would have prescribed a course of antibiotics like amoxicillin 500mg and metronidazole 400mg tid for 5 days (if you are not allergic). Do saline gargling. i hope my answer will help you, take care."
},
{
"id": 85230,
"tgt": "Is there any side effect of taking nexpro?",
"src": "Patient: Hi, I am Soumyak, I am taking nexpro rd 40 daily basis for acidity problem from last couple of month. I am suffering this acidity problem from last 7-8 years. Now a days without nexpro rd 40 I getting more problem. My query is, will this create any side effect in my future life. Doctor: Hi,Nexpro RD (esomeprazole + domperidone) is combination of a proton pump inhibitor and an anti-emetic drug commonly prescribed in the treatment of stomach ulcers, gastro-esophageal reflux disease (GERD) and other acidity related disorders. Generally it is well tolerated however on prolonged use it may cause osteoporosis ('bone thinning'), vitamin B12 deficiency and increased risk of intestinal infections.Hope I have answered your query. Let me know if I can assist you further. Regards, Dr. Mohammed Taher Ali, General & Family Physician"
},
{
"id": 18421,
"tgt": "Is Aspirin an effective alternative for Plavix?",
"src": "Patient: In 2013 and 1 yr later, in 2014, I had a TIA. As a result, after the carotid artery check, my doctor put me on plavix 75 mg and lipitor 10 mg. My GI doctor asked me why I was on the medication, and when I told him, he suggested I could accomplish the same thing with a daily aspirin. Is that correct? Secondly, while applying to change my Medicare Supplement coverage, I was turned out because I take Plavix, which I m told is new and industry-wide. Can you advise on this issue? Doctor: Hello and Welcome to \u2018Ask A Doctor\u2019 service. I have reviewed your query and here is my advice. In case of TIA Aspirin is the best medicine, it is the alone is enough for that, still the underlying cause of TIA would be matter of concern Aspirin can only prevent the further even but not for sure, some time the TIA might not be due to disturbances in hemodynamic system but it could be due to other etiology. Hope I have answered your query. Let me know for further assistance if needed."
},
{
"id": 81851,
"tgt": "What causes green thick mucus discharge and chest congestion?",
"src": "Patient: I cough up green thick mucous almost daily - have for many years. Was told it was from allergies manifesting differently - but have alsi been depressed and drinking - dies this affect mucous. Sometimes I feel like an elephant ia sitting on my chest and have no energy. And yes I smoke. My husband loves smoking and I hate it but not a heavy smoker. So tired of coughing up snot! How much will this question cost? Doctor: This question may not cost u much however the habits that u have formed may cost u your life. It is quite obvious that u will be having problems with your chest if u have allergies as well as smoking habits. Kindly quit and get your lung function tested so that a chest physician can put u on proper drugs"
},
{
"id": 191105,
"tgt": "Suffering from constant headache. My wisdom teeth have cut through just recently",
"src": "Patient: Hi! I have had a constant headache for several months now. It is literally always there, night and day. I have taken excedrine migraine and ibuprofen and asprin free tylenol . I have tried just about everything to make it go away, and it just won t. I have noticed that a couple of my wisdom teeth have cut through just recently, but I don t have any pain whatsoever inside of my mouth. My dentist gave me a z-pak just to prevent infection, and he wants me to get them removed, but I can t afford it...even with insurance. Do you think my headache is caused by my teeth or could it be something else that I need to get checked out by my physician? Like I said, I have had constant, never ending headache for probably about 5 months now. HELP! Doctor: hello, Constant, never-ending headache indicates that there is some thing grossly wrong... i will advice you to go for a second opinion to some other dentist so that you can rule out dental cause for it... if there is no dental cause then better arrange a visit to a neurophysician... take care."
},
{
"id": 117323,
"tgt": "What causes increased level of monocytes during medication for high BP?",
"src": "Patient: My absolute monocytes are out of range, high. What causes this. I am 73 years old, weigh 190, and am 5 foot, 5 1/2 inches tall. I take medicine for high blood pressure, diabetes, arthritis, heart burn, high cholesterol and allergies (Diovan/HCT,Janumet,Tyenol for arthritis with Ultram, Lipitor, and Allegra). Doctor: Hi,Thanks for asking.Based on your clinical history and query, my opinion is as follows:1. Chronic inflammations and infections are usually the cause of increased monocytes specifically tuberculosis, brucellosis, syphilis.2. Also it can be seen in autoimmune diseases, lymphomas and certain myeloproliferative disorders.3. You need not worry about it at present as any chronic inflammation can also induce. You would have arthritis, which could be the cause. Unless a new symptom develops, not to worry at present. Hope it helps.Any further queries, happy to help again."
},
{
"id": 108120,
"tgt": "Suggest treatment for chronic lower back pain",
"src": "Patient: i suffer from chronic lower back pain. i saw two different back specialists about 5 yrs. ago. one thought it was spinal stenosis and the other said arthritis. i have retired and play a lot of golf now, and the pain is getting worse. giving up golf is not an option. is there anything i can do to reduce the chronic lower back pain? (i have already done the steroid injections twice, and they were not very effective). Doctor: I read your question carefully and I understand your concern.Whether it has resulted in spinal stenosis or not you obviously have degenerative changes of the spine. Medication effect is mainly in alleviating the pain but they do not do much to slow down the process. The best means to address that is through physical therapy with stretching and strengthening exercises. As for the pain itself, apart from over the counter pain killers like Ibuprofene I wouldn't add anything else, in advanced rebel cases drugs for chronic pain like antidepressants or anticonvulsants may be considered.I hope to have been of help."
},
{
"id": 187996,
"tgt": "Can taking clindamycin and metronidazole for tooth infection lead to dark brown stools?",
"src": "Patient: I finished a course of antiobiotics clyndamiacin 300mg. 4x/day and metronidazole 500 mg 4x/day for 7 days for a tooth infection. I now have very dark or brown stools that are very loose. I have also been taking a probiotic during this time. Is this something I should be worried about. I noticed the stool get progressively darker as the week went on Doctor: Hello and welcome.Thanks for sharing your concern.Loose stools and mild darker stool is expected with such antibiotic.Therefore do not worry and wait for two more days.Loose stools should stop once your antibiotic course gets over.Therefore if the symptoms persists then you can see your physician for further investigation regarding this.until then do take good care of yourself.Thanks.Take care"
},
{
"id": 213191,
"tgt": "Feeling of bugs in skin, taking medication for mental illness. Can you help?",
"src": "Patient: hi my partner as got a mental illness she takes medication every day and nite for it but she keeps saying she as got bugs in her skin and keeps trying to get them out but i keep telling her there is nothing there but she keeps saying the people in her head are telling her they are i dont know what to do she wont go to a and e with me can u help me please Doctor: This condition is Delusional perceptive disorder if there is no skin infection. Please take one antipsychotic in low dose like olanzapine, trifluperazine&trihexiphenidyl combination and antidepressant like fluoxetine,escitalopram with low dose anxiolytic like clonazepam."
},
{
"id": 69210,
"tgt": "What causes a lump site to be sensitive to touch?",
"src": "Patient: this afternoon i saw what appeared to be a pimple on my leg, so i popped it. It hurt like hell, and soon after (maybe an hour so),. it formed a solid red ring around it and is really sensitibve to the touch...can you tell me the posibilties of what im working wuth please... thank you Doctor: Hi.Thanks for your query.You have tried to pop a lump which was at the height of inflammation with all its characteristics. The blood supply at this stage is at the highest- the body's mechanism to control the infection by pouring more blood / blood cells particularly the soldiers- white blood cells.You have disturbed this - pressed on the already most ailing stage. So as a reaction of the body there is a red ring around it. The nerve endings too get very sensitive as they are the part of the inflammatory process. You have instigated the to be more sensitive.Do not worry ; take a proper course of an antibiotic and anti-inflammatory medicines till this gets settled under the prescription of a Doctor."
},
{
"id": 53983,
"tgt": "What causes the chest incision site to be hard?",
"src": "Patient: hello, im 30 years old female who had undergone laparoscopic cholecystectomy about ten days ago..the doctors had removed the bandages today and removed the sutures..i noticed when i get home that the muscle in the incision area in the chest part is hard but it is not painful when touched..what could be the reason? im worried that i might be operated again.please help me with your answers.thank you Doctor: Hi and welcome to Healthcaremagic. Thank you for your query. I am Dr. Rommstein, I understand your concerns and I will try to help you as much as I can.There is no reason for repeated surgery since this is probably fibroma or formed hematoma and this is commonly seen and should pull back in few weeks.I hope I have answered you query. If you have any further questions you can contact us in every time.Kindly regards. Wish you a good health.DR. Ivan Rommstein"
},
{
"id": 168197,
"tgt": "What causes headache in a child at night?",
"src": "Patient: Dear Doctors My 4.5 years old kid is complaining and crying saying he is having headache at night. It is not on regular intervals, but happens frequently sometimes. He had hit the head on the floor once while falling from 6 foot height and another minor head accidents. Is there anything to be worried or it could be a a minor issue? Doctor: Hello. I just read through your question.In the vast majority of cases, these things are minor and not of any concern. However, anytime a child complains of headache for a prolonged period of time, it's a good idea to be evaluated by a pediatric neurologist for reassurance."
},
{
"id": 139721,
"tgt": "What causes numbness in the face while having seizures?",
"src": "Patient: Hi, my names amber my sister had just started having seizures about a week ago. now they are getting worse and last night she had a new symptom during her most recent seizure, she said her face had went numb and as well as her tounge, the doctors said they are stress seizures ... is this a symptom from stress seizures? Doctor: Hello Dr Faisal here,I have read and understood your query, I will suggest you to work up for root cause of these increasing symptoms. numbness of face and tongue can be an integral part of a seizure or it can be a manifestation of cause of seizure. (a brain stem lesion or hypercalcemia can lead to a numb face and tongue and a seizure).a stress occasionally can leads to start of seizures . i would like you to go for an MRI brain and rest of basic chemistry profile including electrolytes calcium and magnesium.Hope I have answered your query. Let me know if I can assist you further.Take CareRegards.Dr. FAISAL BACHA, MEDICAL SPECIALIST (FCPS INTERNAL MEDICINE)"
},
{
"id": 166045,
"tgt": "How long will typhoid fever last in children?",
"src": "Patient: My grandson is 21months old and he is treated for typhoid in Melbourne from last Friday.he was treated with cefataxime for one week ,he had severe diahorrea and high fever ,now he is better but in evening his temperature goes to 38 :8 still and goes down after 3hours,he is still on injections . I am worried .pl guide me it wii take how much time or fever last. Thanks Pratima Doctor: Hello,Typhoid fever is treated normally for ten to fourteen days. But I can understand your concern, fever should go down by one week. In some children, it takes more time than usual, but I would recommend that you should go for a repeat blood culture of your child, get its preliminary report that comes in three days (the usual time is seven days). Also, consult your treating physician again whether to change antibiotics or not depending on the report and the general condition.Hope I have answered your query. Let me know if I can assist you further.Regards, Dr. Musfirah Tahir"
},
{
"id": 219149,
"tgt": "Been on implanon. Spotted after sex. What is going on?",
"src": "Patient: I have been on implanon for about 15 months. The first 9 months i had my period pretty much the whole time with 1-2 day break about once a month. After that till now they have been pretty regular. Well 4 days ago i started spotting and it started the day after my regular period should have started. My husband and i have been sexually active with no protection. First pregnancy test came out negative the first day of spotting but im just wondering if it could just be the birth control . Doctor: hello,Thanks for writing to us.You have gotten negative pregnancy test with spotting 4 days ago.Implanon is birth control intradermal implant and efficacy rate is more than 99%. Spotting or irregular bleeding may be due to side effect of the implant or hormonal imbalance caused by stress/ emotional upset etc.Don't be much worried about this and try to avoid stress. If further irregular spotting appears, then consult with your gynecologist and need some investigations to rule out pathology.Maintain genital hygiene. Be well."
},
{
"id": 159085,
"tgt": "Have stage 3 rectum cancer. Taking Coumadin for blood clots. Can hemp oil be taken for rectum, kidney and urethral canal?",
"src": "Patient: My 82 yr old dad has stage 3 rectum cancer and suffers from high cholesterol , high blood pressure ,diabetes , gall bladder and is taking Coumadin because he had a problem with blood clots recently when he was in the hospital for 3 weeks since the two chemotherapy treatments he had almost killed him.Could he juice and or take hemp oil to try to prevent taking his rectum out and bad kidney as well as his urethra canal? Doctor: Hi, Hempseed oil is natural antioxidant with essential fatty acids, there are some studies regarding its role on EGF positive cancer specially in lung cancers, in rectal cancer no such study is there. Anyway this has got no adverse effects . Your father can take this drug after consultation with his oncologist."
},
{
"id": 133573,
"tgt": "Suggest solution for pain from degenerative disk disease",
"src": "Patient: I m on oxycodone 15mg. and missed a doctors appointment because he is in OR and I need them since Monsy is a holiday and Tuesday he is in OR and I m going to be sick is there anything I can take or a way to get prescibed anything because I have Degenertive disk disease and in alot of pain and does not want to get sick from not having my meds what can I do? Doctor: Hi,thank you for providing the brief history of you.A thorough neuromuscular assessment is advised. As yo have a degenerative disc disease, an MRI will be recommended to see the level of the injury. Post this, you can undergo physical therapy which will help to regain the muscle strength. Also, by therapeutic ultrasound therapy and TENS therapy, there will be a instant relief of the pain. It is a non-invasive therapy and most physicians recommend physical therapy.Research also supports that physical therapy provides better results with core strengthening and also with TENS therapy.In my clinical practice, majority of patients respond well to physical therapy and in long run with exercises the spine can be made stronger.RegardsJay Indravadan Patel"
},
{
"id": 155055,
"tgt": "Does COPD bronchi and COPD emphysema cause cancer?",
"src": "Patient: Hi ,my name is Linda ,I live in Myrtle Beach SC. today I had an attack of not being able to breath I started coughing and couldn t get my breath called 911 was told I had COPD emphysema I had already been told I had COPD bronchi. and they found 0.4cm noncalcified nodule inferiorly in the right middle lobe. tell me if you can what may happen and do I stand a greater chance of cancer? I did smoke for 30yrs and quite 15 yrs ago Doctor: Hi, dearI have gone through your question. I can understand your concern. You are smoking since many years. So you have high chance of cancer. Copd per se not increasing the risk but it is also caused by smoking so it increased the risk of cancer. Consult your doctor and take treatment accordingly. Hope I have answered your question, if you have doubt then I will be happy to answer.Thanks for using health care magic. Wish you a very good health."
},
{
"id": 184653,
"tgt": "What does a hard bump underneath the tongue indicate?",
"src": "Patient: HI i went to a dentist and he said i have a little lukoplakia (excuse my spelling), but thats not what im asking about. On my left side, very back muller, on the inner gum, underneath but not on my tongue, i have a small hard bump, well kinda hard, im afraid to push on it. when i touch it , it doesnt feel like a circle bump, but a oblong bump attached to a line. to get a better picture, think of the shape of a single sperm, or a shooting star. a bump with a tail. Nothing hanging of the gums or anything just underneath. The tail part of the bump is a bump also and it heads towards my throat but is only about a pinky nail long and about a centimeter big. A side note, I smoke marijuana, and ten cigs a day. Doctor: Thanks for your query, I have gone through your query.The hard bump on the inner surface of your gum could be because of a bony overgrowth like mandibular tori or exostosis or osteoma or garres osteomyelitis.Consult your oral physician and take a radiograph like mandibular occlusal and OPG to rule out above mentioned conditions. If it is mandibular tori or exostosis or osteoma it doesnot require any treatment unless it interfere with denture fabrication. If it is garres osteomyelitis, then cause has to be treated like any tooth or gum infection along with a course of antibiotics.I hope my answer will help you, take care."
},
{
"id": 154994,
"tgt": "Suggest treatment for Pancreas cancer",
"src": "Patient: My wife is suffering from Pancreatic cancer. It has metastasised and effected the liver, due to which she has got jaundice. Her bilirubin has gone very high (18) due to blockages (BC 10.4, BU1.53). She has also developed Ascites and Edena with swellings in her abdomen and legs and her weight is increase at the rate of 1kg per day. Is there any way other than PTBD to remove the blockages in the liver? Doctor: Hi,Thanks for writing in.Pancreatic cancer is one of the most feared diseases and gives no time to the patient for survival. Almost every patient of pancreatic cancer will survive for months or few years at most unless it is detected at early stage and resectable.The main concern right now for your wife is to control the rising bilirubin levels. What has happened is that the tumor in pancreas might be causing block in the biliary channels. This leads to collection oof wbile within the liver and biliary pathway. PTBD is a procedure by which the blocked bile will be drained through the skin and the bilirubin level is controlled. This is a safe and easy technique but might affect patient mobility due to insertion of tubes. There is another way called intrahepatic biliary stent placement. In this the biliary channels are approached with catheters and a stent is put in the area of maximum block. Following the procedure bile is drained through the normal opened passage and this reduces the symptoms."
},
{
"id": 93005,
"tgt": "Abdominal pain after workouts. Any insights?",
"src": "Patient: My 16 year old is very active. She plays tennis, runs, works out, on a regular basis. You would think after a while she would build stamina, but can not work out without resulting abdominal pain that doubles her over and incapacitates her for hours afterward. She describes them as feeling the same as menstrual cramps. Its very frustrating to her to not be able to keep up with her peers. Maybe stamina was the wrong word. She has plenty of that. It's the cramps that stop her before she is done with her workouts.Any insights? Doctor: Hello Thanks for writing to HCM She needs Proper clinical examination and investigations. Ultrasound of abdomen should be done as it will exclude many conditions. Investigations like routine hemogram should also be done. Further investigations like CT Scan can be done if needed. Treatment depends upon clinical findings and investigations reports. Get well soon. Take Care Dr.Indu Bhushan"
},
{
"id": 38879,
"tgt": "What causes fever that causing death due to ARDS and ARF?",
"src": "Patient: In KANKAVALI a coastal area of south maharashtra a fever has occured which is of short duration causing death due to ARDS & ARF fever is of only 4 to 6 days of duration , blood investigation lepto , dengue , malaria , negative but low platelets count Doctor: Kindly get immediately checked for H1N1 and plague.. Also please immediately notify the health ministry regarding this issue.."
},
{
"id": 20451,
"tgt": "Suggest treatment for dizziness and palpitations",
"src": "Patient: Hello, Yesterday I went to the DR because i got dizzy and was having palpatations, All the blood work came out fine, but they told me if the problem continued to go to the ER. Last night I only slept for about two hours I had a constant twich in my forehead and everytime i went to fall asleep i would jump up it was actually kind of scary. Now I feel like my head has pressure in ears and behind eyes not really any palpitations this morning compared to last night. My EKG and blood work came back fine, BP and Heart Rate was also good. Could this be Sinusitis? Could these type of Palpitations come from Sinus Issues? I rarely have any stress so hopefully I have not thrown myself into the realm of Panic Attacks. Should I go to the ER? Doctor: Hello,Your symptoms could be related to a sinus infection. Do you have any other symptoms like running nose or congestion? For this reason, I would recommend consulting with your attending physician for a careful physical exam and some tests: \u2022 A sinus x-ray study\u2022 A sinus discharge bacterial culture \u2022 PCR and ESR for inflammation\u2022 Thyroid hormone levels for thyroid dysfunctionYou should discuss with your doctor on the above issues.Hope I have answered your query. Let me know if I can assist you further.Regards,Dr. Ilir Sharka"
},
{
"id": 184458,
"tgt": "What causes cavities in every tooth at the age of 33?",
"src": "Patient: My son is 33years old he takes Coumadin 7.5mg for mechanical aortic valve and abilify for bipolar disorder ,and his teeth are extremely prone to tooth decay.I know low saliva precipitates this condition,although ,when I look up these meds side effects no indication of this.we went to dentist three months ago,and now same dentist says he has cavities in almost every tooth!! Doctor: Hello,There can be many reasons for the tooth decay like poor oral hygiene, faulty brushing technique, poor eating habits, low immunity, less salivary secretions, sticky foods, etc.As low salivary flow is the factor present in your case, you can use oral salivary suplements to enhance the salivary flow.Please visit to dentist to rule out other factors also and get the prescription for the oral suppliments for salivar y flow.Regards,Vishal Jain"
},
{
"id": 183133,
"tgt": "What causes swelling on tongue and gums?",
"src": "Patient: For a couple days I've noticed an increasingly painful happening in my mouth. My tongue is extra pink and swollen, has \"gullies\" in it and this sensation includes the roof of my mouth and my gums as well - I wear a top denture and bottom partial. By tonight all I can eat is a popsicle. What might this be? Doctor: Hello, thank you for consulting with healthcaremagic. It can be allergic reaction due to denture that is why your tongue and gums are swollen, so once you should visit your dentist and get proper examination of the denture doneHope it will help you."
},
{
"id": 190026,
"tgt": "Sore mouth. Blisters on the gums. Hard lump on the salivary gland. Any opinions?",
"src": "Patient: Hi i am a 43 year old female i have 2 year history of sore mouth its extremely painful and raw, there is nothing can be seen to the naked eye, but when photographed by dentist it showed up as brown freckle like areas over gums roof of my mouth etc. i now have a hard lump to salivory gland area under my chin to right side! I am waiting to see a specialist for oral problems. Please could you give me your opinion on these symtoms Avril Doctor: Hello there...there are two things to this...one could be a viral infection and the second one could be because of major apthous ulcerations...tantum oral rinses can be used to avoid pain...this is a topical anesthetic solution that would give you transient pain relief... use the solution everyday atleast 3- 4times a day and gargle the solution for 1- 2 minutes...Dont dilute the solution..... You can use this 15 minutes before consuming anything orally.... Submandibular salivary gland is the only salivary gland present below the jaw especially in the posterior aspect... salivary glands are swollen in various conditions... there are lymph nodes as well that swell in case of any inflammation or infections in the oral cavity...dont get confused for these swellings as salivary gland swelling...Clinical evaluation is mandatory before i could confirm your diagnosis..... Consult a Oral and Maxillofacial Surgeon only for expert opinion and management...."
},
{
"id": 81760,
"tgt": "Suggest treatment for wheezing after swallowing ice",
"src": "Patient: I was watching some youtube videos online while eating some Ice Breakers and I had a piece in my mouth then accidently swallowed it whole and I m now wheezing and I feel this weird stinging feeling in my thoat. I feel like it might be lodged in my lungs. How do I get it out?!?!?(I also have Asthma) Doctor: Use ventorlin nebulization or inhaler sos for wheezing. If things don't get better then along with ventorlin, also start advair inhalers twice daily. See a pulmonologist if you aren't fine after 2 days"
},
{
"id": 71463,
"tgt": "What causes pinching sensation in chest?",
"src": "Patient: For the past 3 months about 2 days before i start my period i get a weird sensation(doesn't really hurt)its more like someone pinched my chest and let go lasts only a second....what could this be? it happens a few times until i actually start my period then goes away. Doctor: Hello,Since these sensations are typically seen before menstrual bleeding, possibility of Premenstrual Syndrome (PMS) is more likely. It is very common in women to have PMS.Most common cause for this is hormonal changes during menstrual cycle, so no treatment is required. Just assurance and counseling are needed.Sometimes multivitamin tablets containing folic acid, vitamin b 12, vitamin d3, niacin and pyridoxine are beneficial. Avoid stress and tension, be relaxed and calm.Hope I have answered your query. Let me know if I can assist you further.Regards,Dr. Kaushal Bhavsar"
},
{
"id": 38385,
"tgt": "What causes fever, fatigue, and loss of appetite?",
"src": "Patient: Fever of 101.9 past two days, extremely weak and tired, can t sit up longer than 30 minutes from wanting to pass out, consistent headache for three days now and extremely light handed. I haven t ate in three days, I don t have an appetite. Not too sure what s wrong Doctor: HI, thanks for using healthcare magicIt is possible that you have a viral infection though bacterial infection may need to be ruled out if the symptoms persist.Viral infections can present with fever, loss of appetite, fatigue, headache and dizziness.Your doctor can examine you to determine the likely infection causing your symptoms.Treatment depends on infection present but general management would consist of: rest, fluids, paracetamol for fever and pain.I hope this helps"
},
{
"id": 167329,
"tgt": "What causes headache along with fever after taking pain reliever medicine?",
"src": "Patient: my 4 year old daughter was out in the sun all day, and she woke up later that night burning up with a fever. The day after she got sick 3 times in a row. I put some cooling lotion on her, and gave her a pain reliever for the pain. She hasn t got sick since then. She has been drinking water and eating dry cereal, and she is keeping it down fine. She told me she has a headache and is still very hot, it has been a while since i gave her a pain reliever. Should i be worried. What else can i do for her? Doctor: syrup ibugesic pluc can be considered however you should visit Pediatrician personally for further evaluation"
},
{
"id": 187809,
"tgt": "How to treat tooth pain due to stitching of the gums ?",
"src": "Patient: I had 3 teeth pulled on one side in a row....they used 2 stetches to hold the gums together. where one of the stetches was there was a big whole and lots of pain where it was in the gum line area. This pain has been there at that same spot for 10 days now and is not lighting up any....is that normal? Doctor: Hello, thank you for consulting with HCM.As you are mentioning that stitching of gums were done 10 days back and now also you are having pain , it looks that there is some infection still left there as 10 days is a quite long time healing starts till this time and pain subscides.You should visit your dentist again and get an x-ray done of that region, to get confirmed about the infection.Hope it will help you.Regards"
},
{
"id": 9075,
"tgt": "How can I take out these stretch marks from my chest ?",
"src": "Patient: hello i am 15 years old and i have strech marks on my chest as i get older and the chest is getting bigger the strech marks are getting bigger and more visiable will they ever go as i get older and what should i do to help them go Doctor: welcome to hcm dont worry it will take some time..definately it will go..keep your body fit and active and have healthy life..eat protein rich foods..take care"
},
{
"id": 193583,
"tgt": "How to get rid of premature ejaculation?",
"src": "Patient: I am 36 year old and i got married in April 2010. I am 6ft and I weigh 65 Kgs. I never had sex prior to my marriage with anyone or even my wife ( then girlfriend ). Its been an year now and since the first time, i am facing this Premature Ejaculation. That is, i ejaculate within a minute while intercourse.Before getting married, I used to Mastrubate. I am not sure if Mastrubation has anything to do with Premature Ejaculation. ( Let me know, if it is related in some way). Moreover, I do smoke and I had been smoking for almost 15+ years now.Also, my foreskin is tight and it doesn't retract while the penis gets erected. For past couple of days I am manually retracting it on daily basis on a doctor's advice ( locallly ). During the first month of my married life ( sex life, to be precise ), I even had kind of erection problem. I couldn't hold an erection for even a minute. The penis goes flacid right when I try to insert ( probably tight foreskin, not sure though ). I would like to confirm, if tight foreskin could cause Premature Ejaculation.?My wife gave birth to a Baby Boy on March 12th 2011. Please advice on how to get rid of this Premature Ejaculation as we will be resuming sexual life again shortly. I want to satisfy my partner. She doesn't have anything to complain but still... Doctor: Hi, Prone masturbation can cause this issue. What type of masturbation do you follow - regular hand in penis or penis on bed? Regular masturbation won't cause premature ejaculation. Foreskin tight can be due to phimosis. Share a picture for confirmation. Yes, phimosis can cause premature ejaculation. Circumcision is a option if it is phimosis. Finding the cause for your issue is also important. Share your blood sugar value. Hope I have answered your query. Let me know if I can assist you further. Regards, Dr. S. R. Raveendran, Sexologist"
},
{
"id": 195899,
"tgt": "Suggest treatment for male sore nipples with white discharge",
"src": "Patient: Hi, I'm a 21 year old male, and my nipple has being extremley sore and itchy for a long time, my doctors have been perscrbing hydrocortisone cream, which works fine until the tube runs out, then it comes right back, just as severe. It also leaks white liquid, that solidifes to an orangy substance. Doctor: Hello,If I would be your physician, then I will do through examination for any lump palpable, any cyst, breast skin condition, warm, or not. In those cases, I would suggest warm compressions along with that tablet Amox clav, tablet Sumo.In this case, I will advise for regular follow up. As these are prescription medicines, so consult with surgeon and wear loose dress.Hope I have answered your query. Let me know if I can assist you further.Regards,Dr. Shaym B. Kale"
},
{
"id": 105924,
"tgt": "Can asthma create a problem in sex life ?",
"src": "Patient: Dear sir, i am 25 years married female, i have astham but i never used inhaler, when i was having a sex with my hasbend my chest is tightness and my : tongue fold inside and i was unconscious up to 3/5 min then i relax what happent me plz give me answer Doctor: Welcome to HCM. It happens due to your excitement.To avoid the collapse during sex ,Take your medicine one hour before it. Use inhaler just before sex. Spend some more time in foreplay."
},
{
"id": 85365,
"tgt": "Could blood in urine be due to medication?",
"src": "Patient: i am women of age 52 taking nurotroy ,glucosamine methylsulfonylmethane diacerein(drug name glucocerin) and niser drug for oesteoartheritis and after intake of 7 days i can see the blood while i urine pls help is it due to theses drugs or something else ? Doctor: Hi, Cannot say in your particular case and blood in the urine is a serious symptom that can mean a lot of serious possibilities including cancer. Arthritis medicines are commonly non-steroidal pain drugs like Aspirin that cause bleeding. Hope I have answered your query. Let me know if I can assist you further. Regards, Dr. Matt Wachsman, Addiction Medicine Specialist"
},
{
"id": 1643,
"tgt": "Can i get pregnant while on birth control implant?",
"src": "Patient: hi may i be pregnant whislt on the implant and when would be the best time to take a home test to be sure as i dont have regular periods because of the implant but can feel a huge change in myself like i did when i found out i was pregnant with my daughter... Doctor: Hi, it is very rare to get pregnant while on implant. So, even then if you feel symptoms of pregnancy, you can check urine pregnancy test at home any day after 2 weeks of your last contact. Hope I have answered your question. Regards Dr khushboo"
},
{
"id": 171404,
"tgt": "What causes abnormal teeth growth from the gums in child?",
"src": "Patient: My 10 week old son has what looks like a tooth coming through on the top left side of his gum. Its not in the normal spot where a tooth should be coming through. My sister said it could be a pocket. Should i take him to the dentist? should i worry about this? Doctor: My deear 10 weeks is too early to teeth----You must take him to a dentist .It may be one of early erupting milk tooth we call it the natal tooth.and this may become loose and can cause suffocation. may need extraction .the dentist will be the best to help your child."
},
{
"id": 169943,
"tgt": "Suggest ideal time lines to give booster vaccine",
"src": "Patient: My son is 1.5 years old. He is due to get 1st booster \u2013 OPV + DPT and HIB booster vaccine today. But only last week, he got mesporin 1 GM IV shots - 5 days continously ending on 30th June 11 since he had fever. Is it OK to give him these booster vaccine today. But now, he does not have fever...only little cough sometimes. Doctor: Yes mam it is better and advisable to give the booster now mam..antibiotics don't interfere with this immunity for dpt vaccine..so kindly go on and put the vaccine.."
},
{
"id": 185478,
"tgt": "What causes sudden appearance of small gaps between the teeth with filling?",
"src": "Patient: Hi, I m 21 and suddenly noticed small gaps appearing in my front teeth. I had a filling in the front that chipped and prior to this my teeth were perfectly straight. Now they re becoming crooked and gapped. Could it be because of this? It did create a gap that is almost nonexistant now. And is this fixable by braces or invisalign? Doctor: Hello, Read your query, as you have small gap between front teeth dont be worried so much this gap in front tooth is created when filling is chipped , this gap is midline diastema treatment is done by orthodontic treatment by fix treatment and for this you should go for composite restoration also. Consult dentist and go for oral examination and go for its treatment. Hope this will help you. Regards, Dr. Priyanka tiwari"
},
{
"id": 91636,
"tgt": "What is the treatment for pain in stomach?",
"src": "Patient: Since last night I have a dull ache on the middle of my stomach I don't feel sick I'm going toilet normally but this gas been constant it can't be period as I have the implant in but every month gets worse and this is worse than its been help no painkillers help at all haven't eaten I'm hungry but can't eat cause of my stomach Doctor: Hi ! Good evening. I am Dr Shareef answering your query.If I were your doctor, I would advise you for a routine complete blood count, urine routine and culture sensitivity test, an ultrasound abdomen, followed by a clinical examination by a surgeon/gynaecologist to rule out any intra abdominal pathology or a pelvic inflammatory disease. I would also give you a dose of deworming drug. I would suggest you not to eat from outside your home if you do to prevent some intestinal infections from unhygienically cooked food. My further management would depend on the reports of investigation and clinical examination.I hope this information would help you in discussing with your family physician/treating doctor in further management of your problem. Please do not hesitate to ask in case of any further doubts.Thanks for choosing health care magic to clear doubts on your health problems. Wishing you an early recovery. Dr Shareef."
},
{
"id": 168064,
"tgt": "Suggest treatment for fever and chills in a baby while teething",
"src": "Patient: Hi , I have a 1 year 3 month old baby girl , she is teething , for the last 2 nights and days she has had fever with cold chills , I have been giving her Iburopen Fever meds for kids , which breaks the fever , however I only give it on the onset of the fever and chills , please advise Doctor: Hello,I can understand your concern. It is right that she might be teething at this age and might be little cranky and annoyed due to sore gums. However, teething process is not associated with fever and chills. The symptoms she is presenting indicates that she might be having infection which is not related to oral cavity and she should be seen by a pediatrician. Thus, my advise to you is to do not rely just on Ibuprofen as it might control the symptoms for the time being while not treating the underlying infection or disorder. You should consult a pediatrician for the clinical examination of your daughter and further investigations, if needed.I hope this information helps you. Thank you for choosing HealthcareMagic. I wish your daughter feels better soon.Best,Dr. Viraj Shah"
},
{
"id": 147373,
"tgt": "What causes continuous pulsation on the lower lip?",
"src": "Patient: I have had been having a pulsation in my lower lip on the right hand side for the last 24H. I am worried because I do not know what it meeans.Could you please enlighten me on the ccause of a continuous pulsation in my lower lip - on the right hand side? Doctor: HIThank for asking to HCMI really appreciate your concern here the given history of lip problem actually suggestive of twitching and this is very common phenomenon seen and this is mostly happens on facial muscle one possible cause would be involuntary muscular conduction could be due to the disturbances in oxygen supply this is assumption, in short this is not to worry at all, the symptom disappear soon, have nice day."
},
{
"id": 151656,
"tgt": "paralysis on right side of body, difficulty walking and talking, cure ?",
"src": "Patient: My nice was sent for a CT scan as she has recently had paralysis take over the right side of her body. She is under 10 and now cant use her right hand or arm , drags her right leg and is having trouble talking because of the right side of her mouth. What do you think is the issue and how could this have come on in a matter of a week? Doctor: Hello Thanks for using Healthcaremagic.com From the clinical details provided it looks like she has a stroke on the left side of the brain causing weakness of the right hand and leg. Other causes like brain tumor, infection and demyelination can also cause similar problem. I feel MRI of the brain would be a better option than CT scan for better details. Sincerely, Dr Shiva Kumar R Consultant Neurologist & Epileptologist Bangalore"
},
{
"id": 140053,
"tgt": "What causes seizure with unconsciousness and jerky limbs?",
"src": "Patient: My 16 year old niece had a seizure 2 weeks ago. Witness advised that she has fainted and that her arms an legs were jerking. this apparently did not las t long. She was immediately hospitalised, EEG, CT & MRI scans were done. EEG was good, and MRI showed a shadow on the brain. She does not have a history of any illness, no symptoms, never had a headache. For a month she was on Roaccutane tablets for severe case of pimples. Could you please advise what could be the problem Doctor: Hello, In such case is necessary to clarify the \"shadow on the MRI\". For this is necessary to have a brain MRI with contrast enhancement and vascular protocol. Discuss with the Neurologist for these issues. Hope I have answered your query. Let me know if I can assist you further. Take care Regards, Dr Erion Spaho, Neurologist, Surgical"
},
{
"id": 49820,
"tgt": "Can you help in reading my liver and kidney function test reports?",
"src": "Patient: Please find below the recent result of my Liver function and kidney function tests and help me with the meaning and implication of this. Electrolytes, Urea & Creatinine: S-CO2 31mmol/1 S-Urea 2.4mmol/1 Liver functions: S-Unconjugated Bilirubin 15umol/1 Liver Enzymes: S- ALT 43u/1 37 c My mail addresss is YYYY@YYYY Doctor: hi, Both your liver function test and kidney function tests are normal.Blood urea and serum creatinine are measured for evaluating kiney functions. both are breakdown products of protein and if they are excreted normally by the kidneys, the blood levels of these will be within normal limits. Serum bilirubin is a byproduct of breakdown of old red blood cells in the body. Elevated unconjugated bilirubin and ALT means even though liver might be functioning normally, there is obstruction in the excreting system of liver. It could be because of stones, tumour, or strictures.regards"
},
{
"id": 48609,
"tgt": "What causes dull pain in back near the kidney?",
"src": "Patient: Hi Iam having a back pain from a week, So I thought it as a normal pain,from past 3 days I could feel that it is not back pain but actually my kidney is paining a bit.I unable to concentrate on my work because i feel that I am a bit dull , a bit fever and body pains as well, What to do?? Doctor: Hi,Thanks for writing in to us.It is important that you consult a doctor and get an ultrasound scan done immediately. Fever, pain and discomfort in the kidney area can be due to many conditions including that involving kidneys. Any associated fever is a sign of infection and inflammation and must be treated at the earliest. As you have not mentioned the side, it is difficult to discuss all possibilities but if it is on the right side, a gall bladder or liver problem must also be considered.Please take paracetamol 500 mg every 6 hours after food to control fever and body ache till you consult your doctor. Please do not delay more than 24 hours to meet your doctor."
},
{
"id": 224323,
"tgt": "Will taking Augmentin effect the birth control pill intake?",
"src": "Patient: I had sex 13 days ago and am worried because I am taking birth control medication but during the time I had the sexual intercourse I was also taking an antibiotic, Augmentin, which can affect birth control pills. I used a condom during the sexual act as a means of back up and I am almost positive that nothing went wrong, no breakage or tears. I took an at home pregnancy test yesterday, 12 days after the intercourse, and it came out negative. Is that accurate or should I be worried I am pregnant? Doctor: Hi,Birth control pills sometimes is affected by antibiotics. It was right of you to use a condom during the act. If the condom wasnt breached, then you can safely rule out pregnancy from that particular time."
},
{
"id": 81819,
"tgt": "Is chest pain related to osteoarthritis?",
"src": "Patient: i am taking levofloxacin for four days.today I have been experiencing bad chest pain which I am attributing to indigestion which I have not had in years., mainly because I have gas and burping. I am chronic copd and currently taking only free inhalers I get from dr. breo ellipta 100mcg/25mcg. I also take synthroid and 5mg amlodipine. I am 66 years young 5ft 3 180 lbs. I have switched to wheat free diet last 3 weeks, which have totally helped swelling with osteoarthritis, , bone on bone (need knee replaced), bursitis.do you think it is unlikely that this pain is related to heart? it has been over an hour with chest pain. nothing in my arm or jaw but some headache pain(never get) Doctor: The chest pain is because of your acidity and gastritis due to drugs. Not because of COPD or any cardiac problem. Kindly start syrup gelusil 2 tsf 4 times daily"
},
{
"id": 100773,
"tgt": "Suggest remedy for barrel chest in asthma patients",
"src": "Patient: Hi, my 14 year old son has had chronic asthma since he was 2. As he has grown he has developed a barrel chest. He has seen 2 doctors and neither of them seem too concerned about it, he is a skinny boy so it is quite obvious. As he gets older and puts on weight, will it become less noticeable and is it caused by asthma? Doctor: often thin individuals have barrel shaped chest, however, id asthma is there, some changes in lung tissue (emphysema) may be associated.see a chest speciaist for lung function test as obstructive lung disease may be ensueing.give him adequate fats and protein rich diet for some filling up and also mild gym or home exercise for chest build up. Slow deep breathing exercise shuld also be included."
},
{
"id": 1291,
"tgt": "Suggest ways to conceive while having PCOD and on Metformin",
"src": "Patient: HI, i am 22 yrs old married girl, i am trying for pregnency,but i have a background of PCOD,I AM Taking Metformin 1500 daily , mensuration witll be every two months once but this time now it is 2 and half month and i am not getting pregnent , feeling hungry,saliva is coming from mouth and sour voimiting from 2 days. Doctor: Hi, I think you should do a urine pregnancy test at home. If positive consult a doctor. If negative, you can take some medicines for inducing periods. In pcod, ovulation doesn't happen. Thus, pregnancy doesn't occur and periods get delayed. You have to take medicines for growth of your follicles and track your follicles growth by repeated ultrasound. When follicles reach a size more than 17 to 18 mm, take injection for rupturing the follicles. Be in contact with your husband for 2 to 3 days after injection. Take progesterone for next 2 weeks. Do a urine pregnancy test at home after that. You can try like that for 3 to 6 months. Also do a thyroid profile and prolactin levels for yourself and a semen analysis of your husband. If it doesn't work, then you have to go for tubal patency test and thereafter IUI. Metformin alone will not help you in conceiving. Hope I have answered your question. Regards Dr khushboo"
},
{
"id": 119903,
"tgt": "Do pap smear result have any effect on meniscus repair?",
"src": "Patient: hello, i was wondering im getting a meniscus repair on wednesday and i just got blood work done and a pap smear because i have had vaginal discharges but no stds. if my pap comes back bv will i have to postpone my surgery or will that not affect anything Doctor: Hello, Ideally no invasive procedure is advised till there is any sort of infection going on in the body. As meniscus repair is not a emergency procedure and vaginal infections have high chances to cause urinary tract infection,better is to wait till your report becomes negative. Hope I have answered your query. Let me know if I can assist you further. Take care Regards, Dr. Rohan Shanker Tiwari"
},
{
"id": 169728,
"tgt": "What could improper bowel movement with slimy feces and bloating indicate?",
"src": "Patient: Hi doctors, I have a two year old daughter. when I gave her solid food on 9 mos old, she had sticky, slimy poop, n then bleeding stool. so we stop giving her solid food until she was a year. she can eat solid food now, but she often gets stomac2 upset. In a few days, she poops well, then she can t poop for 2-3 days, then poop the sticky, slimy feces.She complains about having stomach ache quite often. I have paid attention to her food, guessing which is suitable n wihich is not, but cant find a conclusion bcoz she shows different reaction towards the same food, sometimes fine, sometimes not.What to do, doc? She gets lot of wind in her tummy. these last 3 weeks, she sometimes poops 6-7 times in 24h. She even sleeps in fear, rejects to wear trousers, n rush to training pot once she feels hurt or wants to fart. Doctors have given meds, vitamin, n What to do, doc? Doctor: hai.according to me first test for stool examination , culture for any cyst are worms and any increased fat and ultrasound abdomen liver function and blood picture are there according to that we will start treatment .mostly she suffering with dysentry or fat indigestion we should rule out and we will give treatment.according to that.still problem there go for colonoscopy and biopsy"
},
{
"id": 180320,
"tgt": "What causes bulging of the navel?",
"src": "Patient: Hello, My newborn, 5 weeks, has a bulging out his navel and have taken him to Dr and he is on amoxil drops 3 times a day and betadine cream on the area twice a day. Is this an infection? What are the risks? Should I see a specialist? He also has quite a bit of yellowy discharge that continues to leak too. Be glad of your help. Doctor: hi if your baby is having reddish or pinkish mass like thing coming out of her umbilicus with yellowish discharge, it is mostly a granuloma which is quite common and regresses on its own. clean the area if the skin around the umbilicus is red and tender. I advice you to put rock Salt in the umbilical area as it absorbs the moisture and shrinks the granuloma. you can do this twice one week apart. if it's still there consult your doctor for copper sulphate crystal application as it decreases the mass, but be careful it should be applied by a medical person. hope this helps you, take rate"
},
{
"id": 224300,
"tgt": "Can i take 'levonelle' morning after pill after missing a period?",
"src": "Patient: I have PCOS and never had a period last month. I had sex on wednesday and i am scared there could be a chance that i am pregnant as the condom kept slipping off, my question is am i safe to take Levonelle morning after pill even when i missed a month's period? Doctor: Hi,Since you have PCOS, you may be having irregular ovulation. I hope you were not sexually active last month. If you were, then there is a small possibility of that missed period being due to pregnancy.Levonelle is safe and you should take it even if you have missed last month's period. Levonelle although can be used within 72 hours of sex, it is more effective the earlier after sex you take this medicine.If you don't get your period in two weeks time,you should do the pregnancy test. (Levonelle is 95% effective in avoiding pregnancy. There is a small possibility of method failure).I hope I have answered to your satisfaction."
},
{
"id": 20255,
"tgt": "What causes high rate of BPM?",
"src": "Patient: Hi my name is kevin, 3 months ago i started to have what my first doctor called an anxiety attack, they told me to take lorazepam when it happend and i would feel better, i was having 1 every 2 days. finally i was doing better and having 1 every 6 days. I couldnt work or anything and havnt since, after 1 month i got a bad cold and started having 3 a day. After my cold stopped i continued having alot so i went to another doctor which told me to take blood tests and take this xanex 3 times a day at.25mg then i went back to another doctor and he said the other doctor was wrong i took more blood tests and had .5mg xanex for a month. Thats when i really had troubles breathing through out the day and so i went to see a chyropractor/kinsiologist and he told me that i would be fine its just i had too much acidity in my body so i should eat right. Soon after i went back to my doctor who perscribed me .5mg xanex and told him i wanted off , then he told me after 3 months its weird that your heart is still going fast , i thought it was cause u took too much redbull but now u need to go see a cardioligist (heart doctor) and strap a machine on u for 24 hours to see ur heart . he lowered my xanex to .25mg and said in two weeks to go lower and then off , he also told me to take more blood tests. My heartbeat stays at about 100bpm sometimes its 80 bpm and somtimes 120 bpm , he told me if it goes over 140bpm i should go to the Emergency room, but since the last 6 days it hasnt gone over 120 that i know of. Can someone just please help me its been 3 months i havnt worked and i feel like sh*t. i went to go see a cardiologist but they said i have to wait till march 2012 ....... What am i suppose to do. Can i even do exercize if my heart cant go over 140? my doctor seems weird. My chyropractor/kinsiologist seemed more aware of the situation, the other doctor i see for 2 minutes and he gets me out right away. Doctor: Kevin, a normal heart rate is 60-100 beats/minute. It is less if you exercise and more if you don\u2019t. Your heart rate variation may be anxiety related. If so, I agree that xanax is a poor long-term treatment option. You should try to wean from it if possible. The monitor is a good idea. I will usually ask for long-term (3-7days) monitoring to judge. It is good to keep of diary of symptoms with heart rate along with blood pressure. Sometimes abnormal control of blood pressure and heart rate go together to make you feel tired."
},
{
"id": 218741,
"tgt": "How to confirm if pregnancy test result is false positive?",
"src": "Patient: I took almost two weeks ago the morning after pill because the condom broke.Yesterday I did just one test because I was feeling nautious and had stomache. It was positive. Could I be pregnant or could it be a false positive? Should I try another test? Doctor: Hello,Thanks for sharing your health concern with us. If you tested positive with a standard test kit, you need not repeat the pregnancy test. Instead, see a specialist for a proper clinical evaluation and go for a sonogram of the pelvis, preferably in the trans-vaginal route, to confirm the gestation sac, the location, the age and the viability. Emergency contraceptive pill offers 85-98 percent protection depending on the time duration elapsed after unprotected intercourse. Please plan further management after review. Hope this helps."
},
{
"id": 173805,
"tgt": "How to treat for amoeba in children?",
"src": "Patient: hi my 1 yr old daughter had her amoeba back, and i am worried because everytime she drinks her milk, she poops, watery stools. what would be the best thing to do for that? she was prescribed with metronidazole 3 times a day 5ml for 1 week, she had a history of amoeba, she was 6 mos when she had it and it came back again. the dr. advised to give her lactose free milk, Doctor: some things i like to know about her . since when she is suffering from loose stools? how frequenly she has loose stools ? do u give her purified water ? what her weigh and lenth? amoeba is not so common cause of diarrhea. commnest cause of diarrhea is viral that is self limiting in 5-7 days . did she underwent stool exam so that you saying it is amoeba.we give ors, Zinc and lactobaccilus in loose stools"
},
{
"id": 183106,
"tgt": "What causes shoulder pain with stiffness in jaw?",
"src": "Patient: Fell on sidewalk face first about 8 hrs ago. Swollen, cut lip bruised and cut knee, and hurt pride initially. Now have muscle and shoulder pain, stiff jaw(think this is from the really swollen lip),pain in wrist, and headache. I assume this is all normal but are there specific things I should look for or will I just be in pain for a while given the hard fall? Doctor: Hello and welcome to HCM,The injuries caused on various body parts have to be managed.The cut and swollen lip has to be managed by sterile dressing.The body pains, pain on injured site and jaw pain should be managed by pain killer and muscle relaxant.I suggest you to consult your primary healthcare provider for clinical assessment.In case there is evidence of infection in any of the injured sites, a course of antibiotics may have to be started to control the infection.Thanks and take careDr Shailja P Wahal"
},
{
"id": 164678,
"tgt": "Suggest treatment for severe mood swings with outbursts",
"src": "Patient: I have a 9 year old grandchild who suffers from severe mood swings. He becomes angry and agressive for no apparent reason and will scream abuse at whoever is around. He is uncontrollable especially if told No to something. His behaviour is starting to wear off on his younger siblings. Help his parents and grandparents are at wits end. He seems to suffer from anxiety. He can have these uncontrollable outbursts up to 3 or 4 times a day. Please help. Doctor: Dear Grandparent,I am sorry for the situation you are in. Given that the problem is disrupting his daily activities and family life, your grandchild definitely needs medical help. Has your grandson been evaluated for this problem before?. If not, I suggest that you see a pediatrician who can evaluate ypur grandchild and if needed refer him to a child psychologist/ psychiatrist. The treatment options would vary according to diagnosis and evaluation and may involve counselling, behavioral therapy and medications."
},
{
"id": 195065,
"tgt": "Suggest treatment for epididymitis despite taking Ciprofloxacin",
"src": "Patient: I have been smoking meth for 2 months straight everyday. My testicles hurt extremely bad but come in episodes. The doc told me I have epidipimitus and gave me ciperoroflaxin. I have been taking it for a good 6-7 days and nothing has changed. Can someone explain this to me? Is it cuz if the meth or what can someone break it down to me. Doctor: Hi, You can get a urine culture and change the antibiotics. You can preferably consider cephalosporins like Cefixime. Hope I have answered your query. Let me know if I can assist you further."
},
{
"id": 34334,
"tgt": "Should a vaccination be taken for the dog's scratch?",
"src": "Patient: I was lightly bitten by a dog (that appeared to be a house pet) and the bite only left two scratches. It did NOT penetrate the skin. There was some saliva (the dog wasn't foaming), but I cleaned it immediately. This happened 24 hours ago. I don't know if the dog had its vaccinations Doctor: HiThanks for posting your query on HCM.I appreciate your concern for the suspected dog bite.The rabies virus is predominantly found in the brain and the salivary secretions of the rabid animals.In view of the history of dog bite with some salivary secretions on the scratches, you are required to take the anti rabies vaccine after the thorough cleansing of the wound, a booster dose of Injection Tetanus toxoid.The anti rabies vaccine has to be injected into the deltoid muscle of the arm on the following days 0,3,7,14, and 30 days.If the dog happens to be healthy after 10 days, you may discontinue the remaining doses of anti rabies vaccine,Hope this answers your query."
},
{
"id": 89880,
"tgt": "What causes sudden and severe pain in right lower abdomen?",
"src": "Patient: yesterday when i got up everything was fine. Later in the day when I was standing I suddenly had a severe pain on my right side lower ab area. I curled up in a ball and it subsided in about 10 min. I then had sweats and carried on with what I was doing. Then about 1/2 hr later same thing. I went home from work and layed down. The pain is less when I'm laying down but when I get up it hurts. I'm 45 male no prior pain. It does not hurt when I push on my stomach area. I feel like I have to go have a bowel movement but couldn't. Sine last night I did have a loose one. Any idea Doctor: welcome to Health care magic.1.The symptoms you are describing are mixed - seems like gastritis and enteric causes.2.Mean while you can change your diet into a basic form, avoid junk food alcohol, take excessive of salads and fruits, with multiple portions ( for gastritis and enteric pathology) for at least 2 weeks and see.3.If any change and better condition you can continue.4.If not you need to gat an ultrasound to rule out negative findings and get in to conclusion.5.A CT might be needed for better evaluation of the bowel.6.Get an appointment with your GP and get the needful investigations done.Hope it helps you. Wish you a good health.Anything to ask ? do not hesitate. Thank you."
},
{
"id": 133249,
"tgt": "Does broken bones heal individually?",
"src": "Patient: I had my 3rd and 4th metatarsal broken in the center of the bone and had to have surgery. Number 4 healed fine, but the 3rd is not healing. The doctor after 7 weeks said I could start to walk without my boot/cast around the house to help with keeping the range of motion ok. I returned again at 9 weeks and he say s the bracket is all twisted, the screws are coming out, and the bone is not healing. He wants to do surgery again, or wear the boot for 3 more weeks with no weight bearing at all. Is this normal since one bone healed fine and the other one didn t? Could it have been that when they put the bracket on #3 bone they didn t have the bone close enough to promote healing of bone between the break? Thank you, Chris Doctor: hi,thank-you for providing the brief history of you.As per your history I agree with the surgeon, that the healing has not occurred and might require another surgery. This is the commonest thing which occurs in foot injuries. Also, the bone healing is totally individual physiology induced, so a surgeon will not be able to do much except to regain the alignment of the bone by which there will be a normal physiological healing. If the healing doesn't occur there might be a need bone graft by which healing occurs.Regards Jay Indravadan Patel"
},
{
"id": 191051,
"tgt": "Tissue under tongue turning white",
"src": "Patient: Hi, the tissue on the floor of my mouth, behind my bottom teeth has changed from rounded tissue to ridgelike, with the top of the ridge turning white. I think it might be my sublingual papilla, from photos I have seen online. How concerned should I be? Doctor: hello, welcome to healthcare magic, rather than self examination through internet, go to a dentist and have a check up before it complicates.. take care.."
},
{
"id": 144548,
"tgt": "What is the relation between sinusitis and epilepsy?",
"src": "Patient: EPILEPSY 1 is it any relation with sinusitis and epilepsy 2 early menstural cycle which prolonged and stopped by a tablet 3 Tablets for sinusitis,throat infection taken before meal. 4 sun stroke 5 All above causes cumulatively. 6 whether first time seizure conforms epilepsy, EEG shows abnormal( may be due to fall on head during first seizures) PLEASE REPLY Doctor: Hi,Thanks for writing in.There is electrical activity going on in the brain at all times. This is mapped by doing electroencephalogram which records the waves on the surface of the brain. When there is abnormal brain waves from a hyperactive area then it causes seizures and this is called epilepsy. Epilepsy is another name for seizure disorder.You have got seizures and the EEG is abnormal, therefore a detailed investigation using MRI scan brain is required. The abnormal EEG area should be studied in detail. The answers to your questions are the following.1. Sinusitis and seizures is not directly related. However an infection from sinuses can rarely reach the brain and cause seizures. This is very rare.2. Menstrual cycles can be affected by medicines, illnesses and stress. The details of tablet are important to discuss this point3. It depends on which medicine is there in the tablet. Most medicines are taken after food but some might require to be taken before food.4. Sun stroke is an environmentally influenced condition and there should be sufficient history of going out in the hot sun. Sun stroke is a metabolic disorder and can lead to seizure or epilepsy.5. Epilepsy can be from many causes and if your EEG us abnormal then a cause in the brain must be looked for.6. Epilepsy and seizures are almost the same. Seizure is a single episode and epilepsy is usually for multiple episodes. A fall during first seizure is less likely to cause abnormal EEG and the seizure caused it."
},
{
"id": 112972,
"tgt": "Severe lower back pain, abdominal pain, leg pain, loss of appetite, lack of energy. On pain killers. Help?",
"src": "Patient: Hey mia. I m a 22 year old girl. During my periods i suffer from severe pain in lower abdomen, back and my legs(thighs). I have not taken any pain killers yet for the same. But now the pain is becoming untolerable for me. I want to ask that is taking pain killers safe in the long run?? During my periods i usually lose my apetitte and energy. Doctor: This is common in females. These symptoms happen due to hormonal changes. There is no harm in taking pain medicine for short period. Pain killers for weeks to months is not advisable. I would suggest to consult a gynecologist and rule out any serious issues related to uterus such as uterine fibroid or endometriosis. Thanks."
},
{
"id": 33273,
"tgt": "Is MRSA infection contagious?",
"src": "Patient: My father (who has myeloma/secondary PCL) came home from the hospital with an MRSA infection on his back. It responded to antibiotics and cleared up, but he still has the bacteria in his nose. We've been practicing good handwashing and disinfecting surfaces. This morning I woke with several mouth ulcers. Could this be MRSA acquired somehow? e.g., not using hot enough water to wash cutlery or something? We don't share food. I am also under a lot of stress, having been his solo caregiver through his cancer battle for the past 6 months. Thank you, Doctor(s). Doctor: Hi & Welcome to HCM.I appreciate your concern for the MRSA infection.Yes, it is contagious infection, the bacteria (staphylococci) which are normally present in the nose can be transmitted by sneezing, coughing or shaking hands, direct contact with an infected wound or by sharing personal items, such as towels or razors, that have touched infected skin, etc.MRSA infection usually occurs in the skin with pus formation.Mouth ulcers are unlikely to have been caused by MRSA. Hope this answers your query.Best regards."
},
{
"id": 112762,
"tgt": "Have chronic back pain. Now have hip pain. Difficulty driving or turning to the right. Cause?",
"src": "Patient: I have chronic back pain of 20 yrs & have managed itbquitebwell by, walking, daily exercise regime. I now have developed a very severe pain on my r) hip & get pain when I attempt to turn to the right. I even get pain when driving, cannot turn to the right without pain. I am a very healthy, fit, 68 year old, weighing 9 st. What could cause this? Doctor: Hi,From history it seems that you might be having osteoartheritis changes in your vertebral column.Due to this changes there might be stretching or irritation of nerves giving you this problem of back pain and restricted movement.Go for x-ray lumbo sacral region and find out the changes of osteo- artheritis.Go for physiotherapy and back extension exercise after consulting orthopedic surgeon.Ok and take care."
},
{
"id": 122053,
"tgt": "What causes pain below the knee in the calf muscles?",
"src": "Patient: Hi my son is 12. He just injured his leg in a basketball game. He jumped up high and landed awkwardly. He felt immediate pain and remained in severe pain for almost an hour. The location is the lower left left just below the knee. The pain extends down to about mid calf. It is in the front of the leg just right of his shin. What has he injured and how soon should he be evaluated? Doctor: Hello, The pain in the calf muscle is related to an injury in the area. This can be a muscle tear or a ligament injury also. You need to get an MRI scan done to find the cause. Hope I have answered your query. Let me know if I can assist you further. Take care Regards, Dr Praveen Tayal, Orthopaedic Surgeon"
},
{
"id": 4764,
"tgt": "Used noriday for 1 month heavy bleeding changed to femulen, no periods. Am i be pregnant?",
"src": "Patient: hi.i am currently on femulen contaceptive pils.i have been using it for 2weeks.previously i was on noriday for a month and half but my periods were long for 3weeks.so my dr had to change my pills to femulen.my worries are last month i got ma periods on 2/09/2013 which lasted for 3weeks until now no traces or sign of it.could i be pregnant??pls help Doctor: welcome to HCM!can be pregnant.get urine pregnancy done with morning sample.get gynaec check-up too."
},
{
"id": 100645,
"tgt": "Suggest remedy for asthamatic attack with breathing difficulty",
"src": "Patient: i'm 24 yrs old male from bangalore.. i get asthmatic attack or breathing difficulty specially during winter.. i'm not taking any medication as if now. I just started hitting the gym very recently which inturn showed a very dramatic change in my breathing problem. But from past 3-4 days i've stopped going to the gym n i'm facing the same problem again. I've wheezing problem after a continous sneeze which i often have in the morning and accompanied with that i'l be left with running nose the whole day, if i dont go out in the sun.. Please help Doctor: Hello.Thank you for asking at HCM.I went through your history and would like to make suggestions for you s follows:1. It s really good that you started going to Gym. In fact, I have seen many patients with asthma whose complaints dramatically improve by regular exercise. Personally I would suggest you to continue exercises.2. Were I treating you, I would suggest you montelukast and levocetirizine for at least 2-4 weeks. I would also suggest you to use salbutamol inhaler on as-and-when-needed basis for asthma symptoms.3. I would suggest you allergy testing which will help you identify the substances you may be allergic to, especially I would think of allergy to dust mites and indoor molds.Based on the testing report, an Allergist-Immunologist may suggest you avoidance measures and allergen specific immunotherapy which works on immune system to gradually improve your allergy symptoms.4. In general, I would suggest you to avoid exposure to dusts, smokes and air pollution.5. Regular breathing exercises including Yoga & Pranayama and healthy regular diet will also help you in a long run by improving your lung capacity and immunity respectively.Hope above suggestions will be helpful to you.Should you have any further query, please feel free to ask at HCM.Wish you the best of the health.Thank you & Regards."
},
{
"id": 60784,
"tgt": "What does a lump under my xiphoid process indicate?",
"src": "Patient: I have a lump under my xiphoid process. Not painful. It's hard. You can see it when I lean back through my skin. No weight loss or loss of appetite. I feel like it's been there a while. But just paying more attention to it now. It's not my xiphoid process. I'm 32 years old. 190 lbs. Been gaining weight. Not very active. Doctor: Hi, Most possible issue is lipoma involving the xiphoid region. A soft tissue ultrasound of high resolution is best way to get it perfectly diagnosed. I would like to admire your further doubts anytime. Hope I have answered your query. Let me know if I can assist you further. Regards, Dr. Bhagyesh V. Patel"
},
{
"id": 188650,
"tgt": "Why do I quench my teeth all day?",
"src": "Patient: why do I quench my teeth all day Doctor: Hello,Thanks for choosing HCM,Quenching teeth is medically termed as bruxism..Its mainly related to the stress,anxiety,bad dreams.etc...Do you quench vigorously in a particular moment?If its related to the above mentioned then you have to decrease the stress..but it may be due to other various reasons..like malocclusion..Consult with Dentist for whole mouth check-up...Don't neglect this...bruxism can lead to temporomandibular joint problem...Get your proper treatment done...Wish you good health...Regards..."
},
{
"id": 187981,
"tgt": "Why is the jaw swollen on the left side under the ear which causes pressure and tingling sensation while eating?",
"src": "Patient: swollen jaw under my left ear. It's only on the left side and doesn't hurt but causes pressure. I have a tingling sensation when i eat something on that side. my teeth do not hurt. the issue comes and goes. sometimes I get a salty saliva taste on the left side sometimes with the pressure and sometimes just the taste Doctor: HiThanks for writing in.There is some pus formation in any tooth in that region.You need to consult your dentist & get it diagnosed & treated accordingly.Maintain proper oral hygiene.Take a course of analgesic & antibiotic for 3 days.RegardsDr. Neha"
},
{
"id": 123578,
"tgt": "What causes pain in the butt cheek?",
"src": "Patient: hi Friday night i played ball, and when i ran to first base my left buttock cheek hurt like i was going to be sick and could hardly get back to the bench the pain was so Bad. It felt like i pulled a muscle in my buttock and shot down the back and inside of my leg. I have been icing and tylenol and advil when i can. got any ideas how i can get better next week for ball. Doctor: Hello, As the pain was radiating in nature this is a clear indication of the nerve entrapment. Taking up an MRI of the lumbar spine to see the soft tissue anatomy is important. With an exercise like core stability, spinal muscle strengthening, hip muscle strengthening, and lower limb strengthening are advised to avoid the symptoms to aggravate. Hope I have answered your query. Let me know if I can assist you further. Take care Regards, Jay Indravadan Patel, Physical Therapist or Physiotherapist"
},
{
"id": 192945,
"tgt": "What could cause increasing drowsiness?",
"src": "Patient: Dear doctors, Day by day, my drowziness is going from bad to worse. This morning I got up very, very drowzy. I am 65, of Indian decent from Malacca, Malaysia. I am diabetic. This morning's reading is 6.0 (fasting). On other days it my be up to 10.00 (fasting). I am on insulin ( 14.0) every night, and take a Metformin tablet at night and in the morning. My parents were diabetic. My blood pressure this morning (20.2.2011) is 122/78. Pulse 79. In June 2010 a doctor at the General Hospital told me that I had slightly low blood pressure (106/66), and advised me to increase my salt intake. I have been adding very tiny amounts of salt to my drinks. I do not excercise, except cut the grass once a week, have never been a sportman in my life. That did not prevent me from learning to fly microlight aircraft. I flew for three years(1995-1998). At that time I was very normal and didn't even know that I had diabetes. I am worried that the increased frequency of drowzniness may have something to do with a decreased flow of blood to my brains and that I may have a stroke soon. I can't afford to undego angiograms, by-pass surgery or any expensive treatment. I need advise. I live alone. Jeyapaul S. YYYY@YYYY Doctor: Hello, You need to check your blood sugars regularly in the early morning and midnight as well. You might be having low blood sugar levels during your sleep that is causing the drowsiness. I suggest you to visit your doctor for expert consultation and review your sugar chart properly and adjust your treatment according to your needs. Hope I have answered your query. Let me know if I can assist you further. Take care Regards, Dr SAMEEN BIN NAEEM, General & Family Physician"
},
{
"id": 70548,
"tgt": "What could be the hard lump on left side of rib cage?",
"src": "Patient: Hi I have a lump on the left side of my rib cage its the size of a golf ball but instead of being round it overal, it feels hard but I can more it slightly, its also very achey and if I press on my ribs just below the lump there very tender! Any ideas? Doctor: Hi,It can be a lymph node or a sebaceous cyst or can be even a malignant tumor.... you need to show it to a surgeon for a physical examination and a needle biopsy, if he feels so, to know exactly what it is.Dr. Ashish Verma"
},
{
"id": 152604,
"tgt": "What does this CT scan report indicate?",
"src": "Patient: My husband has had prostate cancer and had radiation 7 years ago....now he has regular cystscopes for bladder cancer..Dr. has found a few tumors and zapped them.. a couple were cancerous but very small..Dr. ordered a CTScan and the results from that showed a spot on his pancreas...Could this be in anyway related to the prostate & Bladder problems he s had ? He is having an MRI tomorrow morning to have it investigated.. Doctor: HiSince your husband is having a spot in his pancreas in CT scan and he is already having history of prostate cancer i would advice you to do a blood test for CA 19-9 along with the MRI as sometimes it can spread to the pancreas.Warm RegardsDR DE"
},
{
"id": 52939,
"tgt": "What causes dryness on the hands and high SGOT levels?",
"src": "Patient: Hi I m 21 yrs old suffering for acute dryness on palms of hands leading to deep cuts.3 months ago I took a test to be I my liver and count out I had low globulin At 2.4 and high leukocytes count . I was taking medication at that point for the skin disease I have done the test 3 months later and now although globulin levels continue to be low My sgot and sgpt levels have shot up to levels250. What cud this ran ? Need urgent reply Doctor: Hello and Welcome to \u2018Ask A Doctor\u2019 service.I have reviewed your query and here is my advice.If this is 250 for SGOT and SGPT then this is a serious indicator of liver damage. Since SGOT and SGPT are sensitive indicators of liver injury from different types of diseases or conditions, and collectively they are termed liver tests or liver blood tests. Globulins are also produced in the liver. So the next step is to look for certain liver damage and the most common cause is the fatty liver disease. It should be confirmed by ultrasound.Other causes are viral hepatitis, cirrhosis, medications, alcohol intake, or autoimmune diseases. To verify the exact cause, USG, bilirubin levels, and tumor markers should be additionally done then appropriate treatment can be started.I have answered your query. Let me know if I can assist you further.Regards,Dr. Ivan R. Rommstein"
},
{
"id": 184796,
"tgt": "What causes tooth ache?",
"src": "Patient: I am taking Combiflam, twice a day ,for last 10 days for tooth ache.I underwent RCT which was not effective.I had taken Novamax for 5 days too.The toothache is still there.When should I stop taking Combiflam? Is it OK to take for more than 10 days? Doctor: Thanks for your query, i have gone through your query. You can stop taking the course of antibiotics after taking for 5 or 7 days. consult a endodontist and take a radiograph to rule out whether RCT has been done properly and also to check any infection in the neighbor toooth. if it is not done properly then it has to be retreated. I hope my answer will help you, take care."
},
{
"id": 18172,
"tgt": "Is Stamlo a better option than Minipress XL in the treatment of high BP?",
"src": "Patient: I have high BP for last many years, I had been taking Stamlo 5 along with Nebistar SA for last 5 years, I have been having swelling of my ankles and of my legs if traveled insitting position for long hours. Last month my Physician prescribed Minipress XL 5 twic a day in place of Stamlo 5. But my ankle swelling has not gone, having severe scratches of scalp and frequent urination which I feel started after taking Minipress. Iwould like to switch over to Stamlo 5, can You advise if that would be a better option? Doctor: HelloWelcome to Ask a Doctor service.I have reviewed your query and here is my advice.Underlying cause of lower limb swelling matters, if this is not due to high blood pressure then swelling will not subsides, if with the change of medicine swelling is still there and newer prescribed medicine does not suits you then you can discuss this with your physician and ask for switch over to old medicine, anti-hypertensive medicine has to be taken for long time so it has to be taken under your cardiologist or physician always discuss the issue with them before changing the medicine or adjusting the dose.Hope I have answered your query, let me know for further assistance,"
},
{
"id": 42445,
"tgt": "Suggest treatments to become pregnant",
"src": "Patient: Good morning Doctor,I am 25 years old...Weight 73...I have been having PCOS, and trying to get pregnant for past 3 years . My periods are irregular and last time i had my periods was sep 19th 2011. I have tried lot of treatments and nothing seems to work. Please suggest some ways which can help me get pregnant. Doctor: Thank you for posting Q on HCMFirsty you will need to undergo hormonal tests for checking ovarian & thyroid function, then you will need to have Ultrasound examination to look at ovarian size.Depending on results of the test one may need to get them repeated for tracking.There are medications for treatment of hormonal imbalance associated with PCos.Also it will be important to achieve healthy weight which will have positive impact on getting ovarian function back to normal.Hope this helps"
},
{
"id": 43205,
"tgt": "Suffering from right varicocele pain, tolerable. Can I skip surgery as of now?",
"src": "Patient: right varicocele pain, but tolerable, can i skip surgery as of now? :) Hello, i'm 22 years old, i am suffering right varicocele pain 3 years from now, since i'm 19 yrs old.. the pain is tolerable , about 4 out of 10 (10 is most painful), my doctor advised me that surgery IS NOT NECESSARY at this time because the pain is tolerable and i don't have wife at this time, he told me that i can have surgery after marriage when fathering a child is delay.. sometimes while playing basketball the pain goes away.. My question is, is my doctor right? Is his suggestion correct? He told me that i can have surgery after 5 more years or more :) thanks! Doctor: Hello dear,The main cause of concern in case of varicocele is the risk associated with infertility.In varicocele, there is swelling/dilatation of the veins around testis.As a result the temperature around testis is increased.Increased temperature inhibits sperm formation, which in long term leads to infertility (inability to father a child)But management depends on the severity of the varicocele.If your doctor has advised you not to undergo surgery, then kindly follow his advise.However, if the symptoms still persist, then it will be better to consult a Urologist & then take a decision.Take care"
},
{
"id": 102389,
"tgt": "Suggest medication for cold and cough",
"src": "Patient: I am suffering from cough n cold from 3 days. I took cold tablets but did not get relief. I am tired of sneezing for 3 days and my throat also started paining now. Since morning I have sarted coughing more. Just now my friend have offered me cold tablet. Shall I take it or wait till evering to meet a doctor ? Doctor: Hi,I have read your query.You should take medicine only by discussion with your doctor and by you friend. So you should take antibiotic like capsule amoxicillin+cloxacillin 250mg 3 times a day provided you are not sensitive to penicillin. Take tab. Levocitrizine 3 times a day. Do beta-dine gargles in morning nd evening and steam inhalation in morning and night.I hope you have got your answer.In future if you wish to contact me directly, you can use the below mentioned link: http://doctor.healthcaremagic.com/doctors/dr-chander-mohan-singh/67013 Let me know if you have any concerns. Thanks."
},
{
"id": 156032,
"tgt": "Will Moh's surgery be recommended for squamous cell skin cancer?",
"src": "Patient: I have been diagnosed with squamous cell skin cancer. Only .2 cells are identified with the other as pre-cancerous. I have had basal cell 5 times and had them removed. The squamous cell is on my nose which causes me to rethink the treatment. My dermatologist reccomended Moh's surgery, but ,my plastic surgeion said to try the topical cream, first. I am trying to research the best way for treatment and don't want a huge scar on my nose. 2 of the basal cells were on my forehead so I know the aftermath. You can't even see the place where the biopsy was taken; only a tiny red dot. Should I try the topical cream? Will I see signs if this does not work; such as, the scaly spot on my nose returning? I just don't want to be cut on if I can treat this less aggressively. Doctor: Mohs surgery offers tissue sparing , facilitating small minimally disfiguring reconstructions of the resulting defects. Its time consuming and largely dependent on the technique. Topical chemotherapy creams are a first Option if all the cells were precancerous.Since you had basal cell carcinoma previously, you are a high risk candidate and surgical approach is best suited for you.Regards"
},
{
"id": 34615,
"tgt": "What causes thread like substance in mouth?",
"src": "Patient: am 30 years, 5'9 and 75kg. every morning i pick thread like substance in my mouth and i can pick like five every morning sometimes none. it is posible this are worms. i have gone the the dentist and he said i have no decay or infection in my mouth.thank you Doctor: Hello i appreciate your concern this can't be worms it could be mucus thread if you can upload me a photograph or complete medical history that will be a great help to diagnose itin my opinion you should consult a ENT surgeon for examination and a battery of tests for confirmation of diagnosisyou can do gargles with antiseptic mouth wash before sleepwaiting for your responsebest wishes"
},
{
"id": 212653,
"tgt": "Painful period, mood swings, depression. What is going on?",
"src": "Patient: Hi, I m 46 years old and I have had a regular monthly cycle every 4 weeks on the dot. The last 6 months my periods have been very painful with terrible mood swings and almost depression . The last period I had was on November 16th 2012 and I have had not had a period since. I would describe myself as healthy and I have no apparent health issues and I have not been in a relationship for a while so I know for a fact its not pregnancy. Should I be concerned its a relief not to have them..... Doctor: Hi, Welcome to HealthcareMagic Forum As you are 46 years old, you are in peri menopausal age. Menstrual cycle stop around 45-50 years of age. Before that women used to have irregular and prolong gap between the periods which is frequently associated with anxiety, depression, mood swings and hot flushes. You can meet gynecologist and psychiatrist to rule out other pathology. Anti depressants and hormone replacement therapy provide relief in symptoms. Wish you Good Health and all the best. Regards, Dr. Ashish Mittal www.99doctor.com"
},
{
"id": 173426,
"tgt": "Is lactodex n m w and lactodex same?",
"src": "Patient: Baby is 3 months old and motherndoesnt have sufficent milk to feed the baby so doctor advised us to give lactodex n m w . We have started giving the baby s the feed,while we br8ught the second set of tin in it its simply written lactodex only is there any difference between the latter Doctor: Hi,Thanks and welcome to healthcare magic.Lactodex nmw as well as lactodex are almost same .Both are starter formulas.You can use one in place of other upto 6 months age.After 6 months age you need to change ti lactodex-2 or Lactogen-2.Hope this answer serves your purpose .Please feel free to ask further queries if any.Dr.M.V.Subrahmanyam."
},
{
"id": 46131,
"tgt": "What does shadow on kidney signify?",
"src": "Patient: My husband found out he has a large kidney stone. They did a cat scan in the ER yesterday. Today the doctor called and said he has a shadow on the kidney. What could this be? He is scheduled for an ultra sound. Is this cancer or could it mean something else. Doctor: Dear User, may be he would simply indicate the shadow that is observed after a stone is under an ultrasound beam (which is diagnostic for stone). Otherwise, please post the US report. Take care,"
},
{
"id": 110837,
"tgt": "Causes of constant lower back pain with pelvic and abdomen pain",
"src": "Patient: pain I have constant lower back pain,pain in the sides of my lower abdomen,& strong pain & pressure in my pelvic area. painful ovulation & menstral cycles are very abnormal. hurts a lot during intercourse. I'm also suffering from anxiety a lot lately. main concern is the constant pain its getting worse. this has been going on for awhile. what could it be? Doctor: Hi, thanks for posting your concern in the HCM. From the history, I think you might have got pelvic inflammatory disease or adnexal pathology. Conditions mimicking similar pain (renal or ureteral pathology or spine problem ) should be ruled out. You should consult your gynaecologist at the earliest. Detailed clinical examination and investigations including blood and urine tests and imaging may be needed to come to a final diagnosis and you will be treated accordingly. Regards. Dr. Kaushik"
},
{
"id": 196150,
"tgt": "Does paterina help to increase penis size?",
"src": "Patient: my husband is 31 years old heightis 5,11 inch he has he dont want to do intercourse because his penis size is not increased before 6month everything is alright he was done sperm test that report is normal than doctor suggest him to take paterina tablet please tell me pterina tablet help me in this case n what to do help me Doctor: hiii.natural methods are good for improving penis size.oil massage your penis daily.apply coconut oil over penis and stroke penis for 20 to 40 times a day.each stroke should last for atleast 7 seconds.as you exercise,muscles are bulging na.likewise,by doing this penis also become big to some extent.thanks."
},
{
"id": 106078,
"tgt": "Why am i still suffering from allergic rhinitis even after using inhalers ?",
"src": "Patient: i have allergic rhinitis and asthma . doctors give me to use a inheler name seretide and a nosol spray name beconase aqueous nasal spray to use everyday . i am using it everyday. asthma is no more causing problem. but i am suffering allergic rhinitis. my eyes and nose are itching, eyes look like red eyes, it is hard to breath . water dicharge from eyes and nose. what i do ??? Doctor: Hi Mohsin, . Kindly don't worry as it will take some time for the symptoms to subside and depends on the weather also. Get yourself examined by a ENT Specialist, so that he may examine you and Prescribe medications. Stick to your treatment plan and go for regular follow up. Follow these home remedies: 1.) Stay indoors 2.) Use a Air conditioner at home 3.) wear a face mask when you move out 4.) Remove pets if you have any from home 5.) exercise regularly for an hour 6.) eat healthy nutritious balanced diet which should include fresh fruits and green leafy vegetables. Wish you Good Health."
},
{
"id": 32545,
"tgt": "IS it possible to contact chlamydia from a guinea causing swollen vagina?",
"src": "Patient: I am 32 and have never had sexual intercourse. I masterbate regularly, because I'm waiting for marriage for intercourse. My pet (guinea) has an eye infection, and I think I may have wiped myself, or masterbated, once without washing my hands last night. My left labia and outer vaginal areas now feel swollen, but don't look it. Last week I had pink eye and strep throat (probably both strep) and took a z-pack finishing last Monday. I am waiting on the vet's okay to give my guinea antibiotics. I am concerned that I could have gotten chlamydia from him. Is that possible? I don't have any other symptoms other than that feeling down there, and extreme fatigue the past 3 weeks. Doctor: HiThank you for asking HCM.I have gone through your query. Your problem can be most likely bartholinitis which bartholin gland infection. It can be from any bacteria. It can be from fecal side or from your pet. Hot fomentation will be helpful for this. If there is no improvement then antibiotics will be required. I usually suggest combination of antibiotics like ofloxacin and ornidazole for such cases. Chlamydia is most unlikely as you never has intercourse. And z pack will cover chlamydia also. You can use intimate wash daily to normalize vaginal bacterial flora and PH which will help you to prevent getting such infections.Hope this may help you. Let me know if anything not clear.Thanks."
},
{
"id": 151408,
"tgt": "Toddler with periventricular leukomalacia, has affected vision, limps and eyes not in tune, taking stramonium. Can I give cicuta virosa with stramonium?",
"src": "Patient: Hello, I have a son 2 years old with periventricular leukomalacia . Among others affected his vision . He has taken stramonium 12 ch for one month now and he will continue for another one. It seems that his perception is doing better but his limps and eyes seem to be detuned. So can I give him cicuta virosa together with stramonium? Doctor: PVL is sequle of hypoxic/ischemic insult to brain during birth and I disagree with the improvement you had noticed after using stramonium - the process of delayed myelination in your child seems non progressive and has tendency to improve in course of time, by itself, little by little although not totally. Moreover, datura is relatively a poisonous substance and I had seen in India doctors prescribe in higher doses to patients which we neurologists strongly disagree . A regular physiotherapy with combined occupational therapy programme is all that is needed and don't ever go for non-scientific ways of treatment which are not yet proved to be beneficial."
},
{
"id": 155518,
"tgt": "Does Provera cause swelling in arms and legs?",
"src": "Patient: My sister has secondary breast cancer in the liver and lungs. Her primary cancer was in the right breast. She had right mastectomy and partial removal of the lymph nodes in the left armpit, chemotherapy X2, and other hormone treatments with no success. She is now on Provera 500 mg. Recently, she has developed swelling in her right arm (the opposite side of the primary cancer) and both legs/ankles/feet. Can Provera 500 mg be the possible cause of the swelling? Should she stop the tablets? Thanks. Doctor: Hi welcome to HCMI have gone thru your query . I can understand your concern about swelling after taking Provera and for your info it causes many side effects swelling is one of them . But you should not stop using it abruptly on your own . Discuss this issue with you dotor .Dear in fact,Your body is always fighting something -- an infection, a toxin, an allergen, a food or the stress response -- and somehow it redirects its hostile attack on your joints, your brain, your thyroid, your gut, your skin, or sometimes your whole body. Your immune system is your defense against invaders .But when the immune system becomes weak, the disease cells can grow out of control , the disease overpowersSo you should not ignore other possibilities , that can be constipation , mental stress , worry , anger . All affect our health badly . Our health depends on 'what we eat ,when we eat and how we eat '.Gulping on only tablets and medicines is not enough and not going to help anyway ,unless strict diet control and regular exercise to control weight is made part of the regimen . Meanwhile to srengthen your immune system , Regular exercise yoga ,pranayam - deep breathing sufficient rest , meditation and Balanced diet containing essential nutrients as fiber , fruit green veges vitamins and minerals in natural form - coconut water ,lemon juice with water , garlic, ginger , nuts , almonds , and supplements full of antioxidants as Aloe vera juice 35 ml + Amla juice 2 spoons , A level spoon Turmeric powder + Almond oil juice of bitter gourd is very important for our sound mental and physical health and strengthen immune response naturally . This regimen if followed regularly will helps in detoxifying and strengthening the immune system to enhance resistance from diseases and pave way for healthy , happy ,disease free life , It is to be taken as life long commitment for overall health .It\u2019s important to check with your physician before adding herbs to your existing medications,regarding the herbs that are antioxidant ,have no side effects and go with any therapy of treatment .Take Homeopathic Carcinocin 200 / fortnightly / 3 doses .Apply north pole Magnets to hands morning & feet in evening for 5 min to start with & drinking Water,30-35 ml every 3 hours , prepared on north pole magnets also plays as a potent tool in treating cancer by killing bacteria on mass basis. slower the chances of reccurance .HOPE THIS HELPS SOLVE YOUR QUERY Take care All the best , wish her early recovery . If any doubt mail at drsuchda@gmail.comDont hesitate for futher query if any"
},
{
"id": 131168,
"tgt": "Reason for hard bump between knuckles on index finger?",
"src": "Patient: Hard bump between the 2 knuckles on index finger. It appeared suddenly last week & not going away. No pain and I just worked the 24 Hours of Daytona-I joke about it being a muscle developed working so hard for a week without doing anything like that for a long time. Doctor: bump near the knuckles that is painless3 factors1)knuckles is near a joint its called mcp joint2)its painless3) and you have a history of strenuous activitypoints out to a ganglion cystits a non cancerous fluid filled mass occuring near joints after trauma or strenuous activityif its not painful treatment is just plain observationit usually resolves on its ownaspiration of the cyst can be an option but theres a 50% chance itll go back"
},
{
"id": 80285,
"tgt": "Will Macrobid dose leads to bronchitis?",
"src": "Patient: Hi, I am being treated for a UTI with macrobid, which is working..symptoms gone. However, I also developed bronchitis and have a low grade fever. My cough is productive, as they say, but I didn t know whether I should see my internist for an antibiotic that could cure both. Doctor: Thanks for your question on HCM. Macrolide is group of antibiotic. It includes azithromycin, clarithromycin, erythromycin. So it is not the cause for your bronchitis. Bronchitis is infection of airways. So possibility of same infection as a cause for bronchitis which is responsible for UTI (Urinary tract infection) is more. Better to consult pulmonologist and get done 1. Clinical examination of respiratory system 2. Chest x ray 3. PFT (Pulmonary Function Test). Chest x ray is needed to rule out lung infection. PFT is needed for the diagnosis of bronchitis. It will also tell you about severity of the disease. And treatment of bronchitis is based on severity only. You may need antibiotics, inhaled bronchodilators and antihistamines drugs. So consult pulmonologist and start appropriate treatment."
},
{
"id": 66663,
"tgt": "What could bumps/lumps on thigh suggest?",
"src": "Patient: Hello, im 18 years old and have had about 10 bumps all over that look and feel like ant bites, however I have not seen any ants in my house and I dont spend much time outside. Today I noticed a very large lump underneath some stretch marks on my outer thigh, it doesn t hurt or itch, any ideas on what it may be? Doctor: Hi, you got some allergic reaction and folliculitis or panniculitis like conditions associated with that!it could turn into boil therefore take cetirizine like anti-allergics and get to a dermatologist for physical examination of the lesions!all the best!"
},
{
"id": 150101,
"tgt": "MRI scan shows vertebral congenital fusion. Treatment without surgery?",
"src": "Patient: Impression C3 and c4 vertebral congenital fusion L4 right hemivertebra Lumbarisation of s1 and s2 (or two supernumery lumbar vertebra)C5-c6 and c6-c7 left uncoverterbral hypertrophy and disk protrusion causing significant foraminal narrowing and indentation of the cord L5-s1 and s1-2 dick dessication bulge with facet arthrosis indenting the the cal sac with mass effect on the traversing nerve rootsRight foraminal narrowing at s2-3 indenting the right existing s2 nerve root(this is my mri report pleas tel me the treatment for my problem without surgery) Doctor: Hi,Thank you for posting your query.Initial treatment in your case is always without surgery.This includes physiotherapy such as neck and back extension exercises, IFT, ultrasound, traction, etc.Medications such as pregabalin or gabapentin would also help in reducing pain.In addition, please avoid forward bending and lifting heavy weights.Please get back if you require any additional information.Best wishes,Dr Sudhir Kumar MD (Internal Medicine), DM (Neurology)Senior Consultant NeurologistApollo Hospitals, Hyderabad,My personal URL on this website: http://bit.ly/Dr-Sudhir-kumar My email: drsudhirkumar@yahoo.com"
},
{
"id": 136402,
"tgt": "Suggest treatment for toe injury",
"src": "Patient: Yes, I smashed my toe the other day from skateboarding & it started bleeding immediately from under my name. It turned purple right away & I couldn t walk on it the first day. It s been 4 days later & it s still purple & juice is now coming out if I put pressure on it. Any advice? Doctor: Thanks for your query. I have gone through your query.The toe injury can be treated with a course of antibiotics like amoxicillin and metronidazole (if you are not allergic). The purple colour can be because of the ecchymosis or bleeding secondary to injury. The liquid coming out from the wound can be pus secondary to infection. Nothing to be panic, consult a general physician and get it evaluated. I hope i have answered your query. Take care"
},
{
"id": 167185,
"tgt": "Can urticaria on the stomach turn into bruising?",
"src": "Patient: Hi, my 2 year old daughter has had urticaria for the last 3 days. i started orapred this morning---it was a prescription that was a few months old but I called the pharmacy and they checked the manufacturer and it has a 1 year expiration date. my concern it that the urticaria on her stomach has turned into what looks like bruising. is that normal? Doctor: urticaria may become dark during the stage of resolution. dark color may also b due to scratching of these lesions. these will fade away with time. no need to worry."
},
{
"id": 164643,
"tgt": "What could thick mucus discharge in genitals of infants suggest?",
"src": "Patient: my 8 months old baby girl has a thick mucus discharge. she has been having fever all day...up and down, throwing up and lost to appetite. went to the dr and he said it must be a viral infection. two hours ago, i called the dr back because i saw a thick yellowish// green mucus in my baby s private area. ps: i did use a new shampoo on her today. can u please tell me what it may be??? Doctor: Hi, let's not jump to any conclusion... Let's wait and observe... Clean the area with lots of water and leave it open so that if there is any discharge you can see it immediately and the give the sample for testing... If the baby is otherwise normal we can definitely wait and not panick.TC."
},
{
"id": 193445,
"tgt": "What causes swelling in testicle with a lump inside?",
"src": "Patient: Hi, may I answer your health queries right now ? Please type your query here... my husband is 60 yrs old. last week he notice that his right testicle was swollen and had a hard lump inside. very painful we had sex one night and he pushed himself a little to much could this be the reason his testicle is now swollen Doctor: Hello, It can be inflamed testis due to sex at that time. I suggest you apply ice wrapped cloth over his swollen testis to sooth it. He can be prescribed analgesics and anti inflammatory drug for five days. Don't do sex for at least one week. If still lump not resolved than he need to investigate with ultrasonography to check for any mass. Hope I have answered your query. Let me know if I can assist you further. Regards, Dr. Parth Goswami, General & Family Physician"
},
{
"id": 68596,
"tgt": "Suggest treatment for lump on my abdomen",
"src": "Patient: I have a small lump on my back since over five years.It has not grown in size since and never bothered but recently I noticed a small hard thing under skin on my abdomen on left side must of less than 1 cm not visible but can be felt under skin its very hard and a small same thing pea sized under skin on front of my thigh. What it may be am worried? Doctor: Hi ! Good afternoon. i am Dr Shareef answering your query.If I were your doctor, after doing a general physical examination of yours with special attention to the lesions described by you, I would go for some routine tests like a CBC, ESR, and depending on the physical findings would consider referring you to a general surgeon for a possible biopsy of the lesion for a histo pathological diagnosis. Further management would depend on the results of the physical examination and investigations including the biopsy.I hope this information would help you in discussing with your family physician/treating doctor in further management of your problem. Please do not hesitate to ask in case of any further doubts.Thanks for choosing health care magic to clear doubts on your health problems. Wishing you an early recovery. Dr Shareef."
},
{
"id": 170908,
"tgt": "Is bottle feeding leading to freezer burnt lips in an infant?",
"src": "Patient: baby is 7 weeks, last week his lower lip except the middle part are darker like they are freezer burn, i say that because my husband used to have his lips look like this when working in the cold storage warehouse. should i be concerned or is it from him sucking so hard on his bottle? Doctor: Hi, this is infection and you should apply antibacterial lotion like Zytee lotion for 5 to 6 days. Take care."
},
{
"id": 49672,
"tgt": "Renal function, effects of medical marijuana on kidney",
"src": "Patient: Hi, I have a question regarding mmj and renal function . I have 20 percent of renal function but I also have a prescription for mmj for another issue. I was just wondering what the effects of marijuana are on the kidneys and if it damages it? I am not on Dialysis yet and I understand that I need to be clean for 6 months before getting on a transplant list. Doctor: HelloThanks for your query.Based on the facts that you have posted it appears that you have Chronic Renal Paranchymal Disease and expected to need Renal Transplant. You havebeen taking Marijuana for some other illness and worried about the side effects of Marijuana on kideneys.There are clear evidence of negative effect of Marijuana on kidneys neither any clinical studies have been done on this subjects .You can take marijuana after consulting your Nephrologist.Dr.Patil."
},
{
"id": 114511,
"tgt": "How can low blood iron levels be treated?",
"src": "Patient: I am showing lower blood iron level over the last year or so and consensus of my doctor is that it is due to Arthritis. It was always over 13 and is now just over 11, I play golf without pain, walk and mover around without pain, I am stiff when wakening in the morning, but some movement takes care of it I am not taking anything for the arthritis and so far have not taken any thing for the anemia. what do you think? Doctor: Hello. Thanks for asking on Healthcaremagic. In case you are not taking any pain killers for arthritis then it is unlikely that the low iron is due to arthritis. Low iron can occur due to gastritis caused by pain killers. It is advisable to start taking iron tablets daily so that iron levels increase to normal. Kindly follow up with your serum iron tests and start taking iron rich foods. In case it does not normalise after intake too then it might be malabsorption and it's better to take injectable then. Hope I helped. Thanks."
},
{
"id": 123671,
"tgt": "Suggest treatment for pain in the ankle",
"src": "Patient: I fell yesterday and now my foot feels like it needs to crack, kind of like it s stiff or stuck. I can walk but it hurts mostly on the inside top area. No bruising or swelling. Could I have dislocated something? It hurts near that bone on the inside near my ankle. Doctor: Hello, An x-ray is always good on a safer side to rule out any bony Injury. Looks like by history is a ankle sprain. But x-ray can reveal more details. First of all immobilizing the ankle in the crepe bandage or ankle brace will be good. Doing icing or hot water fermentation will be good for pain relief. In case x-ray turns out to be normal and no fracture than post 3 weeks exercise will good to regain strength. For ankle stabilization balance board training will be good. Hope I have answered your query. Let me know if I can assist you further. Take care Regards, Jay Indravadan Patel, Physical Therapist or Physiotherapist"
},
{
"id": 29557,
"tgt": "What causes a wound on the abdomen oozing clear liquid?",
"src": "Patient: HELLO, PLZ FORGIVE THE CAPS, I HAVE A DECENT SIZE BELLY AND UNDER THE OVERHANG IT HAS COME TO MY ATTENTION THAT I HAVE A SMALL WOUND WITH VERY LITTLE BLOOD LOSS - IT S MORE OF A SEEPING CLEAR LIQUID AND IT DOESN T SEEM TO WANT TO HEAL UP. PLEASE KNOW, THERE IS ZERO PAIN ANYWHERE AND I SHOW ZERO SYMPTOMS OF AN ILEUS - IF I M REMEMBERING THAT BLOCKAGE S FORMAL OR MEDICAL NAME. ALSO THERE HAS BEEN NO INJURY TO THE AREA - IS BETWEEN MY BELLY BUTTON AND MY SIDE WHERE MY APPENDIX IS (MY RIGHT SIDE) - I AM AWAITING HIP REPLACEMENT SURGERY AND I AM NEAR 95% DISABLED WITH CONSTANT PAIN ALTHO I TAKE 30MG OXYCODONE 4X DAILY. CURRENTLY I SUPPRESS THE AREA WITH A FOLDED PAPER TOWEL TUCKED UNDER THE BELLY AND CHANGED ABOUT TWICE DAILY WITH A RELATIVELY MOSTLY CLEAR DISCHARGE. ONLY A SMATTERING OF BRIGHT RED BLOOD UPON A CLEANSING WIPE TWICE A DAY. I WOULD APPRECIATE ANY ADVICE TO CALM MY NERVES AS I CANNOT GET TO THE HOSPTAL WITHOUT CALLING AN AMBULANCE. THANK YOU, DAVE HANIFIN - TEL 413.525.4824 - EMAIL - YYYY@YYYY Doctor: It seems there is some infection in the wound leading to oozing.You can apply some skin cream or Skin dusting powder with anti bacterial.Also you can consult your Surgeon who will be performing Hip Surgeryto prescribe you relevant Skin Cream / Dusting Powder after examination."
},
{
"id": 162645,
"tgt": "Is a bump below the jaw a matter of medical emergency?",
"src": "Patient: Hello, my 13 year daughter was home today with a soar throat and just had a slight fever today 99 being the highest. She now has a bump right below the right side of her jaw. I cannot get in to see a doctor right now, I am now sure if this would be an emergency or not. Doctor: Welcome to ' Ask a doctor ' service .I have reviewed your query and here is my answer.Madam , your child has developed mumps .Its not an emergency and you can see your doctor in 2 or 3 days .Just maintain oral hygiene , give medicine for cold as levocetrizine plus phenylephrine ..Paracetamol for fever .Donot massage the swelling .The enlarged gland will come to normal size in 2 or 3 weeks .I hope I have solved your query .Let me know if I can assist you further in this query .WITH REGARDS , DR VARINDER JOSHI ."
},
{
"id": 33750,
"tgt": "Suggest remedy for paralysis in left side of face on getting sinus attack",
"src": "Patient: Hi..I have a friend aged 23 years with height of 5'10, who gets cold, cough and headaches with pain in left side of face..When I asked him he told that he has Sinusitis and it has become incurable as per his doctor..And now if he gets a sinus attack, left side of his face will be paralyzed..He will not be able to speak even..It can only be suppressed not cured.. He is taking homeopathic medicines..Is there really no cure available??? Doctor: Hello,Welcome to Health Care Magic.Thanks for writing.I am Dr.Saddiq-ul-Abidin. I have read your question completely, I understand your concern and will try to help you in best way possible.As far your friends history is suggesting, he is suffering from chronic sinusitis or recurrent sinusitis. This can be allergic or bacterial, but mostly caused by nasal septal deformities or any other pathology which interfere with drainage of bony air sinuses lading to infections. As far as pain is concerned that can occur when any division of Facial nerve or Maxillary division of Trigeminal nerve is involved on the same side due to mass lesions or viral infections. But most of these need proper evaluation and ENT consultants review, because most of such conditions are reversible, and also that a sinusitis is less likely to present like this unless there is any other other underlying abnormality exacerbating it.Your friend may need a detailed workup including complete blood picture, CRP, ES\\R as well as Xray Paranasla sinuses, and CT or MRI if the problem aggravates. He may need IV antibiotics if not cured with oral antibiotics but in neither case the solution is there. He just needs to consult the right person.As far as his story is suggestive, it seems your friend is in wrong hands and being cured by a wrong person who is not licensed physician, and is just arousing doubts and fears in your friend's mind, so that he should follow him repeatedly without consulting a proper ENT surgeon.I hope this answered your question.If you have more queries I am happy to answer.Otherwise rate and close the discussionRegrads.Dr.Saddiq ul AbidinM.B.B.S(Licensed Family Physician)Resident Medicine."
},
{
"id": 172462,
"tgt": "What is causing fever and stomach pain to 3 year old?",
"src": "Patient: hi my son is 3 and has been unwell. he keeps burning right up and calpol is not helping. he is complainibg his head his hurting and also his stomach. he vomited earlier but is off his food and im kind of forcing him 2 sip fluids. what could be wrong. does he need to see somebody? Doctor: in viral fever all these symptoms come.give me plenty of fluids and try to give him small meals at frequent intervals. give me syp onden half an hr before his meals as per his weight. u can give syp ibugesic also if his fever or headache doesn't go.this needs to b given as per his weight too"
},
{
"id": 42760,
"tgt": "Is prescribed dosage of Susten compulsory for Frozen embryo transfer?",
"src": "Patient: Hi I just underwent a Frozen embryo transfer on the 10 of December.My doc advised me Susten Vaginal Tablets 400 mg 4 times a day.I dont recollect if I have taken the 3rd dose today or not. Will that be a problem? I did day 3 transfer.Pls help as I am really worried.thanks,Hema Doctor: Hi Hema,Thanks for writing to HCM .It is ok if you have missed one or two doses of susten. Ideally you shouldn't miss but in circumstances it is ok. No need to worry. Normally body start secreting progesterone once conception occurs. Susten is progesterone given to help in maintenance of early pregnancy. It increases local progesterone level. Also it prevents early abortions. So if you miss one dose your body will cover up. Just don't miss again and again. Be stress free. Hope I have been helpful .RegardsDr.Deepika Patil"
},
{
"id": 223423,
"tgt": "Does Nordette usage within 12 hours prevent pregnancy?",
"src": "Patient: hi doctor. i used Nordette as emergercy contraceptive Levonorgestrel 0.15 mg + ethinyl estradiol 30 ug... i drunk 4pcs within 120 hrs of unprotected sex, and drunk again 4pcs after 12hrs of 1st dosage. what are the signs (without using a pregnancy test kit) that the drug was effective? thank you Doctor: Hello,I have gone through the query and understood your concern. Nordette is often used as an emergency contraception and is shown to be effective in the dose as mentioned. You can see withdrawal bleeding 3-7 days after use of the medication. This is suggestive of the efficacy of the drug and protection from pregnancy. Your next natural period may get delayed by a week. Please see your health care provider for further help if you become overdue by ten days. You may need to get a sonogram of the pelvis and a blood test for pregnancy at that stage to help plan further management. Hope this helps."
},
{
"id": 77257,
"tgt": "Could chest infection be due to paint allergy?",
"src": "Patient: I have been painting the house Friday and Saturday with gloss - solvant based paint - Saturday was about 8 hrs. Now have a chest infection, I don't feel I'll just lost my voice, very sore throat tight chest and have catarrh. Could the paint be the reason ? Doctor: Thanks for your question on Healthcare Magic. I can understand your concern. In my opinion, you are not having chest infection. It is mostly bronchitis. You had exposure of chemical (paint material). On inhalation of this chemical, inflammatory reaction is seen trachea and bronchi. This causes bronchitis which in turn causes coughing, tightness in chest, catarrh, sore throat etc. So better to get done PFT (pulmonary function test). It will not only diagnose bronchitis but it will also tell you about severity of the disease. Inhaled treatment is the best treatment for bronchitis. Don't worry, you will improve with inhaled bronchodilator and inhaled corticosteroids. Hope I have solved your query. I will be happy to help you further. Wish you good health. Thanks."
},
{
"id": 140544,
"tgt": "Suggest treatment for tiredness,confusion and eyesight problems",
"src": "Patient: i am tired all the time with widespread flu like pain, was diagnosed with both migraine and hypothyroidism and given medication. but I am still very tired, easily confused, eyesight problems. What should I ask my doctor so that I can get the right help Doctor: Hello, Problems you are facing may be due to your wavering thyroid levels. Regular monitoring of thyroid levels along with practise of ujjayi pranayam and kapalbhati pranayam is advisable. Triphala ghritham (medicated ghee) 1tsp with warm milk or water at night after food. Hope I have answered your query. Let me know if I can assist you further. Take care Regards, Dr Akshatha Kp, Ayurveda Specialist"
},
{
"id": 61685,
"tgt": "Suggest remedy for painful lump on collar bone with excessive sweating and nausea",
"src": "Patient: I had an accident skiing a week ago and landed hard on my collar bone. Large lump developed over collar bone, very painful, restricted movement in arm, a lot of bruising and swelling which has moved down my chest as the week progressed, I had x-rays on the day of fall which found no broken bones. I just today ( a week later) had a moment where I felt nauseas, nearly fainted and sweated a lot. I sat down, drank water and almost immediately felt better and now feel fine, except y hands are cold. Is this connected to me fall a week ago and is it anything to worry about? thanks Doctor: Sorry to hear that you met an accident and an ill effect due to lump formed after accident.If your xray shoulder is within normal limit then i suggest local swelling of muscle , which in medical term can be said as inflammation of local area due to injury.This swelling has mass effect which compress the vital area over neck region which may lead to nauseating feeling and your upper extremity appear cold.I advice you , again do the xray of shoulder and USG local part which allow us to corelate the extent of swelling and nearby vulnerable structures.Mean while you can performe hot fomention over local region along with pain killer to relief the pain .I advice you meet orthopaedic surgeon in person for better understanding.I hope your queries would have clear.Thank you."
},
{
"id": 139087,
"tgt": "What causes a sharp pain in the knee in bent position?",
"src": "Patient: Hi I was bedding down in a squatting position to plug in my fone charger and got a sharp elastic band snapping type of pain in ny right knee, the pain was quite servere and lasted abiut 20/30 mins after my knee felt tight but no pain, abd again today i was vleaning and my knee did the same, sane right knee, its not painfull after but only for a 20/30mins. Should i go to the loal a&e and consult my GP??? Ik confused as thier is no pain but when i bend down this happens. Thankyou Doctor: Your symptoms are more in favor of menisci injury in the knee, if it feels inside the knee if outside, then sprain of the muscles, in both cases you will need the help of your GP."
},
{
"id": 89814,
"tgt": "What causes abdominal pain?",
"src": "Patient: Hi. Im a 25 year old male 232 pounds who has been experiencing abdominal pain sporadically for over a week. The pain is generally dull almost like a cramp but at times briefly sharp. Typically in my lower abdomen but also in my upper left abdomen and side. Today pain in lower abdomen was continuous and not sporadic. Thank you in advance! Doctor: HiThanks for posting your query. There are many possibilities. Depending on the site i will enumerate few . 1. Pancreatitis 2. Colitis. 3. Indigestion 4. Spleenic abscess. You need certain investigation to see what's wrong with you . I advise you to get Ultrasound abdomen pelvis scan ,serum lipase, urine routine . Kindly revert back with the reports."
},
{
"id": 127367,
"tgt": "How can severe pain in the legs be treated?",
"src": "Patient: I\u2019m 50 trails I have a very difficult time walking coordination T times not good also I get severe where I\u2019m paralysis in my leg muscle looks like a shark took a bite out of my leg pain is so bad I can\u2019t move but cry and deep breath lasts about 45min please help. Doctor: Hello and Welcome to \u2018Ask A Doctor\u2019 service. I have reviewed your query and here is my advice. Many causes for leg pains like restless leg syndrome or infection or anemia or peripheral vascular disease or anxiety or excessive intake of coffee and tea etc. Until examination is done it is difficult to say what it is. please consult your physician get Doppler of limbs or hemoglobin status or serum electrolytes or sugar levels or X-ray of both lower limbs etc. Please consult your doctor he will examine and treat you accordingly. Hope I have answered your query. Let me know if I can assist you further."
},
{
"id": 91793,
"tgt": "Does pepto bismol cure abdominal pain?",
"src": "Patient: my husband has just been diagnosed with a hiatal hernia. After eating holiday foods yesterday he has had abdominal pain and nausea all day today. He takes omeprazole. Will Pepto bismol give him any relief. Is there anything wrong with doing that. Doctor: Hi,Thanks for writing to HCM.He seems to be having gastritis along with the hernia. Holiday foods are rich in spices and fat, which increases gastric acid production, leading to pain.Ask him to switch to a combination of prokinetics like levosulpride along with omeprazole, in consultation with his doctor, that will provide better relief.And next time when he is having holiday food, ask him to take those medicines twice a day, or better to avoid them.Dr. Ashish Verma"
},
{
"id": 218185,
"tgt": "Is pregnancy possible through dry sex?",
"src": "Patient: Dear Doctor, 3 weeks ago I was having dry sex without condom with my partner. It was the 7th day of her menstrual cycle. There was no intercourse, only clitoral stimulation. I also did not ejaculate. I don t think I released precum either. How big is the risk of pregnancy here? Doctor: Hi If you have not inserted your penis need not worry about pregnancy. Hope I have answered your query. Let me know if I can assist you further. Regards, Dr. B. Radhakrishnan Nair, OBGYN"
},
{
"id": 156974,
"tgt": "What apart from anastrozole trastuzumab can be used post surgical removal of a stage 1 cancer?",
"src": "Patient: If a tumor has been surgically removed what cell would hormone drug therapy attack. My oncologist is thinking of treating with anastrozole trastuzumab. Cancer has been removed margins are clean however he said it was estrogen dominant. I am a 71 year old woman with diabetis II and I do not like the side effects of these drugs. AS my cancer was 1 and found very early the 4x.1 centimeter tumor was removed without any chemo or radiation therapy. Is there any other drug that would not have these many side effects. Recommended length of time is 5 years on drug. The onka test was 15, very low risk of reoccurence. Doctor: Thanks for query.any tumour >1cm in size develops its own blood supply, even if margins areclear tumour cells might have already disseminated.thus it is always better to go for targeted treatment as you have hormone receptor positive.coming to onka test Women with a score of 17 or smaller. You have a low risk of having cancer return, if you get a hormone treatment like tamoxifen or an aromatase inhibitor drug. With this score, chemotherapy probably won't do much to prevent a recurrence.so my advise to go with your doc.regards"
},
{
"id": 74162,
"tgt": "Suggest treatment for weird sensation in chest and shortness of breath",
"src": "Patient: Hi, Basically I was on a plane earlier this week travelling home, when I got a funny feeling in my chest. At first i thought it was a heart attack, but as I didn't collapse or get any ther symptoms, I presummed not. The problem has persisted, and it feels like a shortness of breath and tightness in my chest. I read up on the problem, and it says it could be a collapsed lung, or Primary spontaneous pneumothorax. I read the cause, and it says it is common in males, young, smokers and can start from a change in atmospheric pressure. I am 21, male, a smoker (sorry, I know, trying to give up), and the plane would explain this. I was wondering if you could give me some advice, as I don't really have lots of free time to visit a doctors, as I have a disserttation in in 2 weeks. Thanks! Doctor: Hello ! Yes pneumothorax is definitely one of the possibilities . However it is not the only possiblity. Since you mentioned shortness of breath and chest tightness, even bronchial asthma is a possibility. Have your ever had similar episodes in the past ? Problems related to your heart cannot be ruled out just coz you din't collapse. The oxygen levels drop in the higher altitude so your shortness of breath could have been a response to that as well.You need to be examined by a physician in person . You will need a Chest X ray , ECG , Spirometry (pre and post bronchodilator) for basic cardiorespiratory evaluation. Treatment needs to be decided based on your final diagnosis . Do not miss to consult coz if it's a pneumothorax(air getting collected around your lungs) it needs to be drained. So do book an appointment with your physician at the earliest.Do quit smoking too ! Your health needs priority."
},
{
"id": 78129,
"tgt": "What causes congestion in chest and pain in lower groin area?",
"src": "Patient: i had some sharp pains in my lower groin area and now it feels like a fluttering or something. like if you have a cold and there is congestion in your chest, when I breathe I can feel it in my lower groin area. I have never felt or heard of anything like this before. what can it be? Doctor: Hi. I can understand your concern. Your description of pain is vague.You will need a detailed examination and also you will need to undergo a chest x ray to look for any lung infection if your pain is in the chest.Don't worry, you will be alright. Hope I have solved your query. Wish you good health. Thanks."
},
{
"id": 189877,
"tgt": "Got sucker punched. Bottom molars hurting, numbness in chin, lips. Solution?",
"src": "Patient: I got sucker punched about a week ago on the left side of my jaw and it didn t swell up or anything but my bottom molars on that side and cheek none ache on and off and when they ache there is a slight numb feeling on the left side of my chin and lips , it kinda hurts to chew but not really and when I move my jaw around I can hear/feel a grainy feeling in one of my left bottom molars. What is wrong and when should it heal? Should I see a doctor? Doctor: hi and welcome,Physical Trauma to the jaws and teeth may lead to inflammation of the tissues causing pain and discomfort.I would suggest you to get an OPG radiograph done .This examination will confirm the damage of tissue if any , condition of jaws and teeth , presence of hematoma or blood clot may be caused by impact injury.Accordingly treatment can be carried out by a dentist.I suspect , Molar teeth may have developed irreversible pulpitis due to impact which needs a root canal treatment for permanent pain relief.pain in jaws and muscular aches can be cured by analgesics like TRAMADOL. twice daily along with anti inflammatory drugs.i hope this helps,take care."
},
{
"id": 159010,
"tgt": "Spots in the retina. Taking tamoxifen after cancer treatment. Is that the reason. Will Anabrez work?",
"src": "Patient: my wife was diagonised with breasr cancer 2 stage and she underwent 6 chemo ,surgery for removal of breast and 25 radiation ,er was positive,aince 2 years she was taking tomoxofin and recently when she had a eye check up there were 3 spots on her retina and after her consulation with her oncolist he has changed tomoxofin toi ANABREZ.is it the right drug for her Doctor: Tamoxifen at high doses given for 2 years or more causes retinal deposits, reduced visual acuity & cataracts in some patients. Anabrez (Anastrazole) is Aromatase inhibitor is used in the management of ER positive early stage and advanced breast cancer. It blocks aromatase enzyme & prevents the conversion of androgen to estrogen. More effective than Tamoxifen in early stage in preventing tumor recurrence & in late metastatic stage in preventing tumor progression. Results in lower incidence of side effects like vaginal bleeding, vaginal discharge, hotflushes & thromboembolic manifestations when compared with Tamoxifen. But it is associated with higher incidence of side effects like fractures & musculoskeletal disorders compared to Tamoxifen."
},
{
"id": 136803,
"tgt": "Suggest treatment for pain and soreness in the ankle",
"src": "Patient: My ankle has been hurting for months now, I rolled my ankle in May and it is on and off sore every now and then. I wear an ankle sleeve when it starts hurting or acting up. My mom has also told me I don t walk right when I wear flip flops, and I wear them quite often, she says I point my toes inside when I walk. Both ankles tend to hurt but I ve only actually hurt the one. Could it be my walking or something else? Even before I rolled my ankle it hurt both to run . Doctor: HIWell come to HCMI really appreciate your concern, rheumatoid arthritis could be the reason for such symptoms of pain and this can be well treated till it is being diagnosed finally and the best drug would be Tab Diclofenac 50 mg once in day, hope this information helps, take care and have a nice day"
},
{
"id": 120379,
"tgt": "What causes hand and shoulder pain and high BP?",
"src": "Patient: Hi Dr. My age is 26 year,my weight is 93.5 KG & my height is 190cm. i have faced a mild chest pain and dizziness about 15 days ago.i am also facing lower back pain,pain in left hand and shoulder also.I have monitored my BP from last 15 days, it went upto 160/110 but on regular basis it is 90/140 from last few days.I have done all the test including TMT,ECHO test reports are normal except HDL and LDL.my HDL is 30 and LDL is 100.Doctor has recommended me HOPACE H5(Ramipril).I have started it from last 4 days,today my BP is 120/76.I want to know the reason of pain and high BP.Should i continues the BP medicine ? please suggest me alternative so that i can quit the medicine because doctor is saying that i have to continue this medicine life long. Doctor: Hello,High blood pressure do cause multiple pain spots in body as well as chest pain due to increase in cardiac work. At present I agree with your doctor in starting anti hypertensive medicines. With this you should also do following to have relief and with time following will help in decreasing dose of your medicines:- Start doing meditation and yoga under supervision of an expert.- Start doing exercises and morning walk.- Avoid high fat diet and spicy food items.- Get frequent blood examination and health check up for your cholesterol. Hope I have answered your question. Let me know if I can assist you further. Regards, Dr. Mukesh Tiwari, Orthopedic Surgeon"
},
{
"id": 119121,
"tgt": "Taking iron supplements for anemia. Mass in the uterus diagnosed. Cramps in the back, heavy menstruation and leg pain",
"src": "Patient: I was diagnosed with anemia on march 5 and they prescribed me iron which I taken3 times a day. They determined it was due to my periods. I went to obgyn and pap smear is normal but they did a pelvic ultrasound and saw a mass on my uterus and they said my uterus was enlarged. They have referred me to obgyn oncologist which my appt is monday. I have cramps my back and leg hurt every time I stand and my ca 125 level was 300. Is there a good chance this is cancer. I want a hysterectomy because for the past year I have dealt with my periods being so heavy I have to sit on toilet Doctor: Hello, thanks for using health care magic. Ca 125 as you already know is a marker for cancer. It increases in cancers of the digestive system and those of the reproductive system including the ovaries, fallopian tubes, endometrium of uterus etc. It is highly used in the diagnosis and follow-up of ovarian cancer. Ca 125 is not specific to a particular disease and does not necessarily indicate a malignant disease. It could be elevated in benign conditions such as uterine fibroids as well. From your history; heavy menses, anemia, enlarged uterus etc, there is a good chance that what you have is a uterine fibroid. The only problem is that the ca 125 level is far above the normal which is 35 (300 in your case). The appearance of uterine fibroid on ultrasound is particular and you should see your ultrasonographer to say if the mass was not well round and with borders and characteristic of fibroid. If its a fibroid or none metastasized cancer, hysterectomy will be just fine. Best regards and hope this helps. Dr Achuo"
},
{
"id": 129242,
"tgt": "What could cause sporadic on and off pain in armpit and around neck?",
"src": "Patient: I have sporadic pains that come and go in my arm pit and around my neck. They don't last long but occur most days. I have seen my doctor and he said it was probably muscular. . I don't think i have any lumps in my neck or armpits. It has been around for a few months now and is worrying me Doctor: Hello, Thank you for using healthcare magic . I read your question and understand your concern . Based on the information that you provide it looks like the origin of the problem is on your neck. The first diagnosis that comes in my mind is cervical spondyloarthropathies in your neck where the nerves on your neck impinges and causes pain in the neck and arm. So the pain in the arm is a referred pain or radiating pain from the neck. This hypothesis can be confirmed or excluded through a neck MRI. If that is the case you should do a consultation with a neurology specialist and depending on the grade or stage of the disease you can treat with physical therapy and medication in the beginning. Other less frequent diagnosis might be triggerpoints or muscular pain. I wish you quick recovery.Dr.Selmani"
},
{
"id": 93926,
"tgt": "Done abdominal hysterectomy. Have intestinal pain, loose bowel movements, painful urination, nausea",
"src": "Patient: I am 36 years old, on Jan 4th 2013 I had an abdominal hysterectomy , after a couple days of being discharged I was readmitted for ileus , I am still having issues with intestinal pain same area that was bothering me before. upper large intestine close to stomach, when I eat I can feel material going through intestine, BM are loose no form but intense pain(almost like labor contraction pain) also feel this pain when urinating (all urinalysis comes back normal) Im still nauseated dont eat alot, feels full all the time when I do eat it feels like its pushing on diaphram and causes breathing coughing spells Doctor: Hi welcome to Health care magic forum. Thanks for choosing H.C.M.Forum. You had abdominal hysterectomy on 4-1-2013. and in 2 days after discharge got readmitted for ileus. It must be a secondary to obstruction. So it may be a side effect of the surgery. Of course if it is ileus it must be there since long before. I advise you to consult a gastroenterologist for diagnosis and treatment.For ileus long term and continuous treatment may be recommended. If it is secondary to the surgery if the problem is corrected , there is a possibility of complete correction . Wishing for a quick and complete recovery. Best regards."
},
{
"id": 165079,
"tgt": "What causes infant to vomit after first meal?",
"src": "Patient: My daughter is 8 months old. She is mostly on semi-solids now. She throws up after taking her food. This happens especially in the morning, after her first meal. Doctor said that this might be because of reflex. She was on Junior lanzol for 15 days. The problem reduced slightly after that. But her vomitting in the morning still continues. I am not sure if this is because of blockage in her food pipe. In case there is a blockage what are the solutions available? Doctor: Thanks for consulting at Healthcare Magic.If your baby is vomiting during feeding or immediately after feeding then try feeding baby in small sips rather than big boluses, follow right procedure for preparing formula, position properly while feeding, keep baby upright over shoulder for 10-15 minutes after feeding.Along with Lansoprazole use antiemetic medications like Domperidone or Ondansetaron for vomitings.If no relief even after adding antiemetic medications, consult pediatrician who will advice investigations like USG Abdomen, CBC etc. to find out exact cause of such complaints."
},
{
"id": 16742,
"tgt": "Is a TMT test not performed during heart blockage?",
"src": "Patient: hey,, today my dad n me went to a clinic lab for performing TMT and ECHO CARDIOGRAPHY test for heart.the doctor performed ECHO CARDIOGRAPHY test but refused to perform TMT test.I dont know the reason but probably he ws talking about some blockage in heart.could you please tell me the reason why he didnt perform the TMT test and is there any fear for my father ??plz reply as soon as possible.vi Doctor: Hello There After going through your medical query I understand your concern and I would like to tell you that TMT is an extersion to check if a patient has inducible ischemia. In state of hypokinetic echo, TMT is not advisable and patient should go for a CART.Hopefully this information will guide you properly.Kind Regards Dr Bhanu Partap"
},
{
"id": 154080,
"tgt": "What causes severe leg pain after chemotherapy?",
"src": "Patient: Soon I will be cancer free for 2 years after chemo for breast cancer. I love to walk briskly for exercise. However, I notice that after walking a block or two my legs begin to feel very heavy and tired & as a result ,my walking feels very labored and uncomfortable. No more brisk walking. I am 70 years old and feel quite healthy otherwise. No medications. Doctor: Hi,Thanks for writing in.Good to know that you have conquered cancer and won your battle against it. Chemotherapy has helped you a lot and made you cancer free. Chemotherapy medicines are strong and might cause a number of side effects that includes nerve damage.This is known as peripheral neuropathy and I guess that you hare having symptoms of peripheral neuropathy. Unfortunately this might become a chronic condition in many patients and they have to take medications to feel better and make their lives comfortable. There are medicinces to help in peripheral neuropathy and I suggest you consult your neurologist and get your condition confirmed and take medicines. Please do not worry."
},
{
"id": 113269,
"tgt": "Lower back pain, moving to legs. MRI shows central disc protrusion causing thecal sac indentation. Cure?",
"src": "Patient: I am suffering from lower backache and the pain travels through out the legs which swtiches between left and right leg, spcieally from hips to knees. I got my MRI & X-ray done, the x-ray report says it s a muscle spasms and the MRI report says Focal posterior central disc protrusion at L5-S1 level causing mild thecal sac indentation . Kindly suggest me what kind of treatment / cure I need? Doctor: Hi, As long as there are no sensory problems, give absolute bed rest for 3 wks, have good muscle relaxants and Physiotherapy. If persists, you might have to think about micr discectomy . Regards"
},
{
"id": 13204,
"tgt": "What causes an itchy painful rash on ankles, thighs, hands and elbows?",
"src": "Patient: My Wife has Mottled a Rash appeared on her angles then overnight moved all the way up to her thighs, back of her Elbows also on her hands with slight swelling in the Wrist, Hand and Fingers. Rash was itchy at first now Pain also, walking with slight difficulty Doctor: Hi, As per your query you have symptoms of itchy painful rash on ankles, thighs, hands and elbows which occur because of allergic reaction and could be due to the outbreak of viral infection. Need not to worry. I would suggest you maintain complete hygiene and apply a cream containing azelaic acid which is good for use. Apply aloe vera gel at night. If the condition doesn't get well in a week then consult a dermatologist for proper examination and treatment. The doctor may prescribe antibiotics like minocycline, steroid ointment. You should apply Melalite XL cream which is a combination of hydroquinone, tretinoin and steroid. Hope I have answered your query. Let me know if I can assist you further. Regards, Dr. Harry Maheshwari, Dentist"
},
{
"id": 80874,
"tgt": "Can AZITHRAL be used as a precaution for lung infection ?",
"src": "Patient: can AZITHRAL 500 used as a precaution of lung infection ? My dad admitted due to infection and my family visit him daily worried we will be infected , dad got the infection from hospital . Can I give this medicine to my mum who is 85 years old ? Thank You Regards Doctor: Hello dear, thanks for your question on HCM. I can understand your situation and problem. Yes, you can give azithromycin to your mother to prevent cross infection. Azithromycin is given routinely for prophylaxis purpose.Also ask her to wear mask during hospital visit. And avoid unnecessary hospital visits. Ask her to avoid fasting, eat healthy and drink plenty of fluids. Along with the drug, she should also follow these measures to prevent cross infection."
},
{
"id": 68232,
"tgt": "What causes a lump in the middle of the chest?",
"src": "Patient: I HAVE A LUMP IN THE MIDDLE OF MY CHEST IT'S SORE TO TOUCH BUT OTHER WISE DOESN'T HURT WHAT COULD THIS BE.i HAVE BEEN HAVING TROUBLE EATING CARBS AND STARCHES,WHEN I EAT THE MY CHEST HURT REALLY BAD ALL MOST LIKE I SWALLOWED SOMETHING WHOLE. IT'S BEEN LIKE THIS FOR A LONG TIME BUT JUST LATELY GETTING WORSE. SO I'M NOT SURE IF THESE THINGS ARE RELATED. Doctor: Welcome to health care magic. 1.The possible cause could be infective lump which causing pain on touch and chest movements. Hair follicle infection will be in the first place and then the infective collection / abscess.2.In this case i would have examined the patent and ask for an ultrasound examination - which is investigation of choice.3.Ultrasound will assess the nature of the lump its source and extensions.4.In case of mass lesion an FNAC - fine needle aspiration cytology needed to assess the nature of the cells involved.5.in case of infective collection, a small incision and drainage followed by a course of antibiotics. good luck.Hope i have answered your query,any thing to ask do not hesitate to ask.http://doctor.healthcaremagic.com/doctors/dr-ganesh/62888"
},
{
"id": 5789,
"tgt": "Wants to conceive through IVF treatment, previous medical abortion done. Chances of success?",
"src": "Patient: Hi, About two years ago I got pregnant and had an medical abortion at age 25. Decided later that we want to try again at age 26 and could not get pregnant after a year trying. I was diagnosed as unexplained since all the tests came back great. I am considering doing IVF since nothing is working, please advise if I have a chance of getting pregnant with IVF? Doctor: Hello Unexplained infertility is a big cause of infertility, and unfortunately, a frustrating one, as you have nothing to work upon. IVF would definitely help you, if the basics such as semen analysis and your uterine anatomy are normal. In the best of centres, worldwide, the success rates vary from 30 - 35 % percent. SO, there is no reason for you to believe that you would not benefit from IVF. Prognosis is better for your young age. take care and all the best."
},
{
"id": 98014,
"tgt": "Snake bites. spots became very painfully and lickage some liquied water",
"src": "Patient: me female 48 years from india,after 14 years a snake bitten me and every year red spots & itch appear on my left leg under knee in the month august-september now these spots became very painfully and lickage some liquied water type anything from these spots.plz. suggest me any allopathic medicine. thanking u in anticipation Doctor: 1. any cracking of skin,pus or any other secretion apart from water? 2. getting around a particular month...could be eczema with fungal infection? 3. cortisone and oral anti-histamine will help with itching and other symptoms. 4. consult with a dermatologist for further necessary medical care."
},
{
"id": 138443,
"tgt": "Suggest treatment for swelling of knuckle",
"src": "Patient: I punched a mirror about 2 weeks ago, and the wound on my knuckle is now healed but under the wound is very warm, puffy, and swollen. I m 99 percent sure it s infected. It hurts more everyday and gets a little more swollen everyday. It has been goin on now for about 4 days. Any advice on what I should do? Doctor: HiWelcome to healthcaremagicI have gone through your query and understand your concern.You are having collection of hematoma. You are sure that 99 percent it is infected. It is better to clinically examined it and if needed it can be drained under aseptic condition. You can discuss with your doctor about it. Hope your query get answered. If you have any clarification then don't hesitate to write to us. I will be happy to help you.Wishing you a good health.Take care."
},
{
"id": 75090,
"tgt": "What causes pins and needles sensation in upper chest with asthma?",
"src": "Patient: Hello Doctor, as of the past couple of days when i cough, there is pins and needles sensation in my upper chest. I have asthma and history lung infections although i have never felt this sensation before. I am 26 yars old. I use albuterol sulfate in the liqued form with a nebulizer. Doctor: Hello and thanks for asking in HCM I can understand your concern Sometimes this might be a side effect of the treatment with inhalers, not so common with albuterol but it happens. So please discuss with your doctor for your complain and probably he will change the treatment. RegardsDr. Jolanda"
},
{
"id": 172664,
"tgt": "Should i consult a doctor as my daughter fell off from her bed?",
"src": "Patient: my 8 year old daughter fell off her bed and her shin has a very large bump on it. She has stopped crying but says that is still hurting very much. She wont stand on it. And when i was feeling around her leg she says pressure starts hurting about an inch and a half away from the bump. also the bruise appears to be almost grey... any advice? Doctor: HiWelcome to the HCMI have gone through your question and it seems that your daughter has suffered a severe musculoskeletal injury most likely a fracture. You should immediately take her to an orthopedic surgeon for proper clinical examination and management accordingly. They may advice you for the X Ray leg to look for injury or fracture.I would recommend you to splint the part by a board to help in transportation to hospital.You may give her acetaminophen or ibuprofen tablet for pain relief.Hopefully this will help you. I would be happy to help you out in any future health related questions.Take care"
},
{
"id": 98812,
"tgt": "What causes muscle cramps with cough and wheezing?",
"src": "Patient: Hello I have had a recent bout of asthma, not improved with serevent and Spiriva yesterday my MO put me on prednisone 25mgs bd for 5 days asthma improving, less coughing, minimal wheeze, however... now I am experiencing muscle cramps feet legs hands pleura I am on an aged pension do I still need to pay? you there sam? Doctor: HI, thanks for using healthcare magicSteroids can cause muscle cramps in some persons mainly due to changes in certain electrolyte levels.Electrolyte abnormalities can be confirmed with a blood test done by your doctor.When the steroids are discontinued, if they are the only cause of the symptoms, it is possible that they may decrease.I hope this helps"
},
{
"id": 76403,
"tgt": "What causes constant chest pain during the post partum period?",
"src": "Patient: I m 37 year old female who is 8 months post-partum. For about a month now I ve had almost constant chest pain-sometimes on the left, sometimes on the right. The pain does not feel deep and there is no change in intensity with activity or rest. At times my left shoulder burns and now feel icy-cold sensations in center-top of ribcage. Initial visit to ER - EKG, blood work-2 sets 6 hours apart and chest xray all normal. ER doctor said it was not my heart and probably pulled muscle strain from lifting baby. Worried as father had large heart attack at 47...should I see a cardiologist, request more tests? Doctor: Thanks for your question on Healthcare Magic. I can understand your concern. No need to worry for heart attack or consult cardiologist because you are young (37 years) and having normal ecg. Possibility of musculoskeletal pain is more in your case. So follow these steps for better symptomatic relief. 1. Avoid movements causing pain. 2. Avoid heavyweight lifting and strenuous exercise. 3. Avoid bad postures in sleep. 4. Take painkiller like ibuprofen. 5. Apply warm water pad on affected areas. Don't worry, you will be alright with all these in 1-2 weeks. If not improving then consult cardiologist. Hope I have solved your query. I will be happy to help you further. Wish you good health. Thanks."
},
{
"id": 180919,
"tgt": "What causes chronic migraine headaches post wisdom teeth extraction?",
"src": "Patient: i had all four of my wisdom teeth removed 3 weeks ago, after I stopped taking pain pills I suddenly have migraines for days!! Also My back and thighs hurt so much I can't bend down it I can only sit in a chair for about 10 mins before my legs beg me to lay down it's getting bad I'm 17 years old. Should I go see a doctor or just wait It out? Doctor: Hi..Thanks for the query..Well, migraine can be due to wisdom teeth extraction, in case if there s any infection in the extraction site or there is any unhealed socket..In that case also the pain will begin at the extraction site and will radiate to head..But the pain in thighs and back is not related to extraction , it needs evaluation and proper treatment can be planned accordingly..So better consult an Emergency room and get examined..For now to relieve pain you can take painkillers and muscle relaxants..Hope this helps..Regards."
},
{
"id": 116680,
"tgt": "Could having Marfan s cause anemia?",
"src": "Patient: Is it possible Marfan s could cause anemia? I have severe anemia (iron: 11; hemoglobin: 8.7; and I do not have any external bleeding issues that would cause such a drastic anemia. I do have a heart condition (Marfans) with very poor circulation and would like Doctor: Welcome to H.C.M.I am Dr Krishna Dubey.Dear friend , severe anemia can be seen in Marfan syndrome but this is not common. I advise you to repeat complete blood count with peripheral blood filmto rule out type of anemia and further management.Some cases are reported of Marfan syndrome with systemic vasculitis presenting as severe anemia. No need to worry because anemia can be treated.Thanx for query."
},
{
"id": 84313,
"tgt": "Does penicillin lead to cramps?",
"src": "Patient: Hi I finished my clomid last sunday and my expected ovulation was this monday or tuesday that just passed according to my doctor. I m now taking penicillin for an infection in a tooth. I ve been having sharp cramps and light pink spotting. Is this normal? Doctor: Hello, The symptoms you mentioned are less likely related to penicillin, as you were already taking Clomid for some pre-existing issue and the cramps and spotting can be related to the same issue. Although if the symptoms have exclusively started after taking penicillin then I suggest you see your doctor. Till then stop taking the medicine and seek medical advice from your current doctor. If he asks you to continue with the medicine then there should not be any reason to panic as he is aware of all your medical history. Hope I have answered your query. Let me know if I can assist you further. Take care Regards, Dr. Manisha Auradkar, General & Family Physician"
},
{
"id": 91415,
"tgt": "What causes pain under the belly button?",
"src": "Patient: hi i am 24/f been having constant pain directly under my belly botton for several days now. back pain which i think may just be my job. no appitite which is making me tiard and am unsure wither just to take ani inflamitries and pain killers and hope for the best. what would you recommend. Doctor: Hi! Good evening. i am Dr Shareef answering your query.If I were your doctor, I would go for a routine complete blood count, and a urine routine and culture examination for you. If you are sexually active, I would also keep in my mind the chances of Pelvic inflammatory disease, and refer you to a gynaecologist. Also I would consider a Liver Function test for you. The management would depend on the result of physical examination and reports of investigations.I hope this information would help you in discussing with your family physician/treating doctor in further management of your problem. Please do not hesitate to ask in case of any further doubts.Thanks for choosing health care magic to clear doubts on your health problems. Wishing you an early recovery. Dr Shareef."
},
{
"id": 120711,
"tgt": "Suggest remedy for pain in armpit post lung surgery",
"src": "Patient: I m 16, 5 9 , 105 pounds, and I have had lung surgery to fix spontaneous pnemothorax. I am having pains right below my armpit and it hurts to breathe in but only sometimes and the pain got worse throughout the day. It hurts really bad but not bad enough for my lung to have collapsed again. I was just wondering what u think is wrong under my ARM. Doctor: Hello, It may be an inflammatory pain at the chest tube site. As a first line management you can take analgesics like paracetamol or aceclofenac for pain relief. If symptoms persists a CT thorax is required to look for any inflammatory collection at the surgical site. Hope I have answered your query. Let me know if I can assist you further. Take care Regards, Dr. Shinas Hussain"
},
{
"id": 53824,
"tgt": "What could gallbladder with multiple calculi suggest?",
"src": "Patient: iam deepika 27 yr old in my pcod scan it is found that iam having gall-bladder- contracted E/o multiple calculi of average sizes 4-5 mm noted wall thickness normal what do mean by this is it only sugery the solution is not curable by medicines plz help me iam very much woried Doctor: Hi welcome to health care magic... If you have no any gall bladder calculi symptoms than no need to much worry In asymptomatic gall bladder stone only clinical follow up done Udiliv tablet available for it's treatment... But if you develops repeated symptoms like abdominal pain from that than it needs to removed known as cholecystectomy. . Avoid excess fried and fatty foods Fruits and green leafy vegetables more and regular exercise helpful You consult surgeon for examination and further work up"
},
{
"id": 10267,
"tgt": "Will taking cansel b help in reducing hair fall?",
"src": "Patient: Hi, may I answer your health queries right now ? Please type your query here... My age 29 year.hight 5 feet 6 inch wight 50 kgs my problem is hair fall. This is not genetic in frunt side of head. Can cansel-b is help for strong helty hair and stop hair fallen. Doctor: Hello and Welcome to \u2018Ask A Doctor\u2019 service. I have reviewed your query and here is my advice. I have gone through your complaints and Kansel B won't be effective in preventing hair loss. I would recommend you to take hair supplements like tablet follihair A once daily for 3-4 months. Hope I have answered your query. Let me know if I can assist you further."
},
{
"id": 221929,
"tgt": "Why does one feel drowsy and tired with loss of appetite after sex?",
"src": "Patient: I VE BEEN HAVING SEX EVERY OTHER DAY FOR THE PAST 2 WEEKS & I HAVE BEEN FEELING REALLY DROUSY & TIRED, I DONT HAVE A APPETITE FOR ANYTHANG AND IM ALWAYS SLEEPING ON MY LEFT SIDE. SOMETIMES I MY MOUTH IS WATERY AND I FEEL LIKE THROWING UP BUT IT DOESNT COME OUT. MY PERIOD SHOULDNT BE HERE UNTIL ABOUT THE 10TH OR 11TH OF FEBUARY CAUSE MY LAST PERIOD WAS JANUARY 8-13. I GO TO THE RESTROOM CONSTANLY & I CAN SMELL MY PEE. PEOPLE KEEP TELLING ME THESE ARE SIGNS OF PREGNANCY AND IM GETTING NERVOUS & ITS MAKING ME SO ANXIOUS TO FIND OUT IF I AM OR NOT CAN SUM 1 HELP ME PLEASE. Doctor: Hello dear,I understand your concern.In my opinion pregnancy is confirmed by urine pregnancy test .This test gives accurate results a week after missed period.And the symptoms of pregnancy are nausea,vomitings,increased urination,sore breasts start 2 weeks after missed period.As you had unprotected sex there might be risk of pregnancy.Dont worry.The first sign of pregnancy is missed period.So wait and check for the period.Ofcourse your symptoms are similar to pregnancy symptoms but it should be associated with positive pregnancy test.Best regards..."
},
{
"id": 156654,
"tgt": "Is it normal to bleed while urinating after having D and C to remove polyp?",
"src": "Patient: I had a D&C last month. A polyp was removed. No cancer found. My period returned about a month laster. I recently masterbated twice and had bleeding when i peed. Enough to put a liner on. I'll be 40yo next week is this normal or the start of perimenopause.I'm afraid to have intercourse with my husband. Doctor: No its not normal to bleed one month after D and C. You need to see your gynaecologist again for pelvic examination"
},
{
"id": 28041,
"tgt": "Could the enlargement in the heart be due to viral infection?",
"src": "Patient: My sister in law has just been told she seems to have an enlarged heart. She has had a virus recently which affected her throat and chest. They seem to be saying that it could be due to the virus and she is to have hospital tests done soon to investigate it. Could a virus cause this - or is it something she may have had for years? How serious could it be? Doctor: What's her age? Viral myocarditis can cause cardiomegaly.. Cardiomyopathy can be due to virus also...but not so common...need to know excatly what she has and examine to give ur answer. If u can send me her 2D echo report and tell more history , probably o can help. I hope this is helpful reply to u."
},
{
"id": 205167,
"tgt": "How can depression be treated?",
"src": "Patient: I went off of mimvy about a month ago to see if I could handle premenopausal fog depression and sleeplessness without it. I was doing ok for about a month and now I feel despondent and not enjoying life and am exhausted. I am 51 and in good health. Does mimvey help depression? Doctor: u start taking desvanlafaxine 50mgs half tablet for 6days and then increase to one tablet after 6 days after lunch"
},
{
"id": 156589,
"tgt": "What does a pap smear test for?",
"src": "Patient: In my yearly checkup following were found My pap smear result says smear shows sheets of superficial and intermediate squamous cells. Inflammatory cells, degenerated cells and few clusters of endocervical cells are present. No dysplastic or malignant cell seen. its there any problem doctor.. Doctor: This is a normal pap smear report. No cancer (malignant) or dysplastic cells seen. So there is nothing to worry about. Enjoy your healthy life and keep following your doctor's advice for followup pap smear."
},
{
"id": 37099,
"tgt": "Suggest treatment for typhoid",
"src": "Patient: Doctor i am from Calgary Canada I visited India and sick over there when I came back I was very sick my family doctor sent my blood to lab for test and it was found that I was attacked by salmonella para typhi (a kind of thypoid ) I was hospitalized for 9 days and given 2gm of ciftraxione I feel ok so I send home where nurse gave this antibiotics for further 9 days intra venous so total 14 days I am better than before but I still sometimes feel cold in my body, mild pain in intestine and back and in my legs . According to Candian doctors I do not need any diet restrictions and i can eat anything like roti and other stuff just take care of spicy food and oit ghee and butter . As per doctor I am started going to my work and started eating every thing and try to avoid outside food as long as possible My question is this can this fever come again as I am working now and is it ok to eat roti and cooked vegetable other than soft food and how long it will take to heal every thing after your doctor say that your fever is fixed Can I do any kind of work after 15 days of cure pls let me know my email is YYYY@YYYY Thanks Parmined Gill Doctor: Hello For typhoid fever ceftriazone 2 gm is drug of choice and you took treatment . If possible take CIPROFLOXACIN 500 mg twice in day for 4 weeks (orally ) . As in India most of typhoid strains responded to this drug . As for as concerned about diet and work , you can take any type of diet and resume your work at once but if possible avoid hard physical work.In India a patient of typhoid advised to avoid chilled items ( freeze items ) , I don't know why?I don't advise such things .Since there are 4 type of typhoid strains , so I advise ciprofloxacin.But this is a prescription drug so take prescription from your doctor and consultation.Good luck."
},
{
"id": 174105,
"tgt": "Suggest treatment for dry cough",
"src": "Patient: my son is 6 yrs old.Always we all got irritated because of his caughing sound particularly at night-doc suggest us for a x-ray. in his chest x-ray we got a report of a well defined homogenous opacity in lower lung zone in paracardial location is this a matter of problematic or this is just a mild infection.thanx Doctor: HI YOUR KID BASED ON THE XRAY IS HAVING BACTERIAL LUNG INFECTION-PNEUMONIA WHICH NEEDS TO BE TREATED ALONG WITH OTHER BLOOD TESTS.PLEASE FOLLOW THE ADVISE OF YOUR CHILD SPECIALIST"
},
{
"id": 130024,
"tgt": "Suggest treatment for numbness in hands caused by an electric shock",
"src": "Patient: I was shocked by a 220volt AC outlet three days ago and the numbness in my arm has been getting progressively worse. It is now difficult to lift my left arm. I am in a foreign country and would like to avoid treatment if possible but is this likely to be serious? I m concerned about blood clots. Doctor: Hi , I have gone through your question and understand your concerns, Its likely that you had an nerve injury (electric shocks mostly damage muscles and nerves) , If there is no swelling of arm or if there is no bluish/blackish discoloration of arm than you need not to worry , numbness will settle down. If any of above described features appear than you really need to visit your physician to look for any obstruction/damage in your veins/arteries. You can take paracetamol for pain and duloxetine for neuropathic pains and numbness if it is not settling with paracetamol.Hope this answers your question. If you have additional questions or follow up questions then please do not hesitate to ask. I will be happy to answer your questions. BesafeWishing you good health."
},
{
"id": 16809,
"tgt": "What causes sudden dizzy spells followed by weird taste/smell?",
"src": "Patient: I HAVE BEEN EXPERIENCING SUDDEN DIZZY SPELLS, FOLLOWED BY A WEIRD TASTE AND SMELL.. SORT OF LIKE ONION.. WHICH I DONT EVEN EAT. AFTER THAT I FEEL LIKE MY BODY GOES HEAVY AND I FEEL LIKE I CANT FOLLOW MY TRAIN OF THOUGHT. EVEN TO SPEAK, I FEEL LIKE IT S IN SLOW MOTION. THEN AFTER THAT I GET A BIT NAUSEATED AND A HEADACHE. PLEASE ADVICE.. Doctor: Hello, I would explain that your symptoms could be related to focal epilepsy seizures. For this reason, I would recommend consulting with a neurologist and performing an EEG and a brain MRI. Further lab blood tests may be needed. Hope I have answered your query. Let me know if I can assist you further. Regards, Dr. Ilir Sharka, Cardiologist"
},
{
"id": 192266,
"tgt": "Suggest treatment for yellow jelly like semen",
"src": "Patient: Hi, I am forty years old and have had a very high semen viscosity for about the last eight years. Why has my semen changed from a white low viscosity semen of ten years ago to a yellow very thick (jelly like) semen that really does not come out of my urethra very well during ejaculation? I have been to a urologist and really did not get a good answer other than \"its just my bodies way of changing. I would really like to look into doing something to change this if possible. Thanks for your help Doctor: Hello, Go for a semen analysis to look for any signs of infection. Infective causes can lead to similar clinical picture. Hope I have answered your query. Let me know if I can assist you further. Take care Regards, Dr. Shinas Hussain"
},
{
"id": 51182,
"tgt": "High creatinine levels. Dialysis suggested. Taking renolog and Cudo forte. Right medication?",
"src": "Patient: Hi ... My mom has been diagnosed with high cretinine levels. Since last 3 months she is on medicines. She has no problem in passing urine and feels hungry too but even with medicines the creatinine level is shooting up. She started with 4.5 level and now it is around 7. The doc says with med it increases but should come down then. In between it came down too a couple of times. But since last 2-3 days it has shoopted up to 6-7 levels. Earlier it was between 5 and 5.5 10 days back. Now doc has suggested renolog med for 3 weeks and see if it comes down else suggested dialysis . Please help as if the med is ok or not. Before renolog he even gave Cudo Forte for 10 days which helped in geeting the levels down from 5.8 to 5.1 Doctor: Hello saurabh, greetings from healthcaremagic.com I am Dr Sourav Chakraborty and i have seen the case scenario of your mother and will help you out with your query. Your mother hasbeen diagnosed with high creatinine levels recently less than a year , you have not mentioned her blood urea and uric acid levels which is very important in your mother's case. Also i would like to enquire about her co morbid conditions like diabetes , because we have to rule out any medicinal cause of increase in creatinine. You have not mentioned the age of your mother but if she is less than 50 years then please get ultrasonography of whole abdomen with renal doppler study , which will rule out any structural abnormality. We have to be sure it is primarily a kidney disease and not secondary to other cause, as if your other have kidney disease then she has to lead very disciplined life also need a lot of support from family members. If she is an established case of chronic kidney disease i ask you to restrict protein in her as it may lead to increase in creatinine. The medicines you mentioned about are new supportive medicine which is used to renal health . Although no substantial trials have been publishes , small trials with some patients have revealed it to be beneficial in preserving kidney function and prevent further damage. There is lot to discuss about kidney disease and i have tried to give some idea relevant to your mom. For more information discuss with her doctor. Hope io have been lucid in her approach. Take care"
},
{
"id": 53320,
"tgt": "Suggest treatment for constipation/pain near kidney area while having fatty liver",
"src": "Patient: Dear doctor, since last 4 weaks aim having pain in the back righ side , ten inches above waist and not in the spine area, may be near kidney area. I got scanned my kidneys and are OK. scan says my liver is fatty. and I got belly 4 inches over my chest. my abdomen is not in perfect condition these days. constipation problem is experienced. pl guide me Doctor: Hi,Its difficult to figure out the cause of constipation. Constipation definition varies from person-to-person. Stools may be too hard or too small, difficult to pass, or infrequent (less than three times per week). Some people with constipation may feel a sense that the bowels are not empty advice:- stop any medication being used for constipation for a week - thrice daily, take two tea spoon (or one big spoon) isphagula husk with 200 to 300 mL water- increase warm water( atleast 8 glass/day) intake- do not drink water during and after meals- eat small meals (rich in fiber)- eat plentiful \"fresh oranges\" and \"pear \"- avoid COLD milk and and AVOID junk food(NO burger, pizza-even if its veg.) altogether- mix few drops of \"lemon juice\" in warm water. Drink on empty stomach, early in the morning, to aid in bowel movement- avoid beverages / alcohol/ soft drinks- Do some exercise (push-ups / squats etc) early in the morning- avoid carbonated drinks. Take 2-3 spoon edible virgin olive oil in the morning. If no resolution after 10 days, then better to visit treating physician. besides constipation, if your weight is increasing and/or you feel fatigue, then have a thyroid profile test. Advise for fatty liver: Usually ALT or AST values higher than \"two times the upper normal limit\", is considered abnormal (in some countries, ALT or AST values of more than 100 are considered abnormal). Value of AST or ALT greater than 85 or above maybe investigated further. Alcohol ingestion and obesity are common causes of fatty liver disease.- abstinence from \"Alcohol\" - LOW fat diet should be followed, AVOID junk food and beverages- Decreased oil consumption (oily food)- NO red meat- green vegetables should be ingested daily- Use lemon juice (lemonade) once in a day- Reduce weight if overweight/obese- \"recheck liver enzymes after 6 to 8 weeks\" and/or ultrasound.Hope to answer your concern.Hope I have answered your query. Let me know if I can assist you further. Wish you good and sound health.Regards,Dr. Tayyab Malik"
},
{
"id": 97747,
"tgt": "What is the homeopathic or ayurveda treatment for palmo plantar keratosis?",
"src": "Patient: Hi I am anit from india.I suffer from palmo plantar keratosis.my mother and father had no it but it has been to me. I am marred.while my daughter in affected. Is there any cure or treatment available in homeopathy or ayurved. Thanks for your support. Doctor: **1. since skin of hands/feet are thick [keratoderma], they can affecting your ADL [activities of daily living] since unusual appearance can be stressful and lead to psychological difficulties.2. In Ayurveda, we generally treat the disease on the lines of 'ek kushtha' and/or 'charum kushtha', so in best Interest kindly consult an Ayurveda Physician in your locality for further treatment options or Homeopathy Doctor for Management.PS. Since disease is hereditary [as is with daughter] thus treatment is time consuming, since Rasayan Chikitsa [Immunomodulation] is required."
},
{
"id": 22715,
"tgt": "How to treat a catheterization site knot?",
"src": "Patient: the sight where the cath entered for my heart catherization was not sore, but this morning it is very sore. Last Friday, I saw the cardioolgist---he said there was a small knot. At that time what I felt was more of an annoyance. Today it hurts more. I do not know if the knot is bigger, as I did not feel it last Friday. The heart cath was July 10. Doctor: Hello and Welcome to \u2018Ask A Doctor\u2019 service. I have read your question closely, I understand your concern and will be helping you with your health related problem. This can happen, a local scar can be there and inflammation can cause pain and swelling. I would suggest you not to panic just take ibuprofen for pain and inflammation. Ice the area thrice a day this will help with the knot and inflammation. This will get better with time. Local pain killer cream or balm can help too.I hope this answered your question, if you have more queries I am happy to answer you.Regards,Dr. Muhammad Ahmad"
},
{
"id": 186480,
"tgt": "Is there any treatment for aligning teeth without braces?",
"src": "Patient: hello doctor.. im a student of 18 yrs old.. my teeth is arrnged in improper way so im not able to smile confidently.. is there any treatment without using clips and which do not align any problms further in future?? if so how much it costs to the treatment and also the time taken to it?? Doctor: hello thanks for consulting at hcm..kindly take a consult ur orthodontist/dental orthopeadics for alignment of teeth and for surgical option with oral surgeon..either ways these specialist will take a call on what to do after doing a series of radiograph such as opg, lat ceph, hand wrist etc,,so plz take a consult and money depends on what procedre ..hp it helps..tc"
},
{
"id": 61958,
"tgt": "How to treat small hard lump causing pain around the breast?",
"src": "Patient: hi i am 25 from last week i am feeling a little bit pain in my right breast, just below the armpit. After a week when i crosschecked i xould feel a small hard lump inside the breast and the pain is from that area only. can you please suggest me what should i do? Doctor: Hi, dearI have gone through your question. I can understand your concern. You have breast lump with pain. You may have fibroadenoma, fibroadenosis or fibrocystic disease or some other mass lesion. You should go for fine needle aspiration cytology or biopsy and mammography. It will give you exact diagnosis. Then you should take treatment accordingly. Hope I have answered your question, if you have doubt then I will be happy to answer. Thanks for using health care magic. Wish you a very good health."
},
{
"id": 202596,
"tgt": "How can I increase sperm quantity?",
"src": "Patient: Hi Sir, I am Mohan 23 years old. I have spermed up for 7 years staring from 2007 to till now. I am student with rank holder. But now on wards I don t wanted to such things. My question is that, Will it effect to my remaining sperms during giving birth after marriage. What I should do now to strong and increase my sperm quantity. Please reply as soon as posible. Doctor: HIThank for asking to HCMI can understand your problem I think you are under the wrong impression, did you send you semen for analysis, if yes then you have to see the total number of sperm in that, of this is low then this need to be considered and if you have not dome the test then you need not worry about this, hope this information helps you, have a nice day."
},
{
"id": 74546,
"tgt": "Suggest treatment for allergic bronchitis",
"src": "Patient: I am suffering fr. Chronic Breathig Problem. Allergic Bronchitis Causing running nose, dificulty in stepping stair case. Previously I used to take Cipla Asthalin Inhaler since last 25-years. Now my doctor says stop using Asthalin Inhaler & start using Esiflo 100 Transcap Twice daily. Use Asthalin Inhaler only in SOS condition. Doctor: Respected user , HiThanks for using Healthcaremagic.comI have evaluated your query thoroughly .* Best treatment for allergic bronchitis is prevention only .* Stay away from the component you know which one is exacerbating .* Once attack is there , you have to take symptomatic relief medicines according to your doctor`s advise .Hope to clarify your query Welcome for further assistance Thanks for using Healthcaremagic.com & giving opportunity to assistWishing fine recovery Regards dear take care"
},
{
"id": 216279,
"tgt": "How to stop using norco as pain reliever?",
"src": "Patient: Hello yes just a question a friend of mine has been going to this pain management place and they have been giving him norco s... The problem is that he recently has been crushing them and sniffing them and I am a little concerned about ... What is the best way of dealing with this i wanted to call the pain management place and tell them but do you think they will take me seriously and can i do it anomonisly Doctor: They will take you seriously. However if the person is not taking other drugs of abuse nor seeing other doctors nor having other sources of medication, there is precious little to be able to find that they are snorting the medicine. It gets in significantly faster that way and is more addictive. BUT, the overall amount taken is not really changed. Direct overdose is not much of a risk with the drug alone if it is taken at the amounts prescribed by whateve route of administraation."
},
{
"id": 218793,
"tgt": "Are Folic acid and Zinc bisglycinate drugs effective for postpartum anemia?",
"src": "Patient: Dear sir/mam I am a nursing mother( 84kg weight and ) 15days baby,may I take a tablet with following contents to beat Post delivery anemia?Elemental iron 100 mg Zinc bisglycinate 91.3mg Eqv to elemental zinc 27.5mg Folic acid IP 500mcg Adenosylcobalamine 500mcg Excipients qs????Thanks Doctor: hello user.You can take the tablets to deliver post delivery anaemia.thanks.you can ask if any further query."
},
{
"id": 105474,
"tgt": "Coughing, blood in ears, neck pain. History of asthma. Can coughing cause ear injury?",
"src": "Patient: I have asthma and had a coughing bout which lasted a good hour. I m coughing because I cannot stand the cigarette smoke/odor coming in thru my air-conditioning vents, but this is another issue with the landlords. However, shortly thereafter I used my nedi-pot and took a shower. Upon cleaning my ears, I had blood in my right ear showing at then end of the Q-tips. My right ear is hurting pretty bad now. Also, my neck on the right side where my cervical spine incision is hurting and pain radiating upward towards ear. I m not so sure how serious to take this. And could my coughing caused some injury to the ear and/or incision? Doctor: Hello. Coughing normally will not cause bleeding in ears. It can however cause problem to an extent if your eustachean tubes (connecting nose to middle ear) were blocked due to cold. From you history, there seems to be some injury or boil to your ear canal. Your hearing seems to be ok since you have not mentioned about any loss. Please get your ear examined. There is no need to suffer from a cough bout for 1 hour! Whenever your asthmatic airways trigger a cough bout, it is a good idea to take inhaled salbutamol 1-2 puffs and that should abort the cough bout. Bad prolonged cough bout can cause neck/ abdominal pain and can also cause fainting. -Dr. Tushar"
},
{
"id": 116192,
"tgt": "What does the blood test with cholesterol 264 indicate?",
"src": "Patient: I gave blood last week and my cholesteral level was 264. It has been running around 210 to 220 but my HDL was over 75 and my LDL was 130. My ratio is 2.9 should I get it rechecked. My last Dr. Blood test was in Jan. or do you thing the 264 was a fluke. I eat very healthy and exercise almost on a daily basis Doctor: Hi, dearI have gone through your question. I can understand your concern.Your total cholesterol is high. But you have very high HDL cholesterol. You should go for repeat complete lipid profile including LDL, VLDL, HDL, Triglyceride. Then you should take treatment accordingly.Hope I have answered your question, if you have any doubts then contact me at bit.ly/Drsanghvihardik, I will be happy to answer you.Thanks for using health care magic.Wish you a very good health."
},
{
"id": 168767,
"tgt": "What causes distended abdomen in a 11 month old?",
"src": "Patient: my 11 month old daughter has over the last 3 weeks got a large smooth non tender liver, her abdomen is distorted by it.She is otherwise well except over last few days she occasionally appears to posset some food again. She was given azithromycin for a presumed mycoplasma infection for 2 months atopped in may and had a tonsillitis in may with high fevers for 10 days. At present she is well but this mass is very concerning. We are seeing a paediatrician soon do you have any thoughts. Doctor: in infancy liver is normally palpable. its pathological only if the liver span or its actual size is increased. as u said mycoplasma infection which can cause respiratory symptoms can cause pushed down liver which can be why its palpable. so unless there is visible jaundice or high colored stools or clay stools you need not worry. repeated vomiting and loss of appetite can be symptoms of hepatitis. i would suggest a ultrasound abdomen and liver function tests to rule out liver pathology"
},
{
"id": 98653,
"tgt": "Can intake of Tamiflu worsen asthma and cough-related issues?",
"src": "Patient: My 12 yr old son has been having problems with his asthma. We are following his yellow zone protocols and he just finished up with an oral steroid for 5 days. He is still coughing a lot and his doctor recommended we up his maintance inhaler yesterday. My daughter was diagnosed with influenza A so he was put on Tamiflu as a precautionary measure. I just gave him the first dose at 7 and it seems like his cough/asthma issues have gotten worse since. Could this be a reaction to the Tamiflu? He has no other symptoms of a run Doctor: Cough is not a known side effect of taking Tamiflu.The cough could be due to the influenza. I would recommend an over the counter anti tussive like Dextrometharphan. Also water vapor inhalation might be helpful.Hope this answers your query. Thank you"
},
{
"id": 160655,
"tgt": "Suggest treatment to gain weight",
"src": "Patient: hai, my son (11yrs) tested mantoux test postive (28mm) , his chest xray and liver test was done which had no sign of tuberculosis. he is not gaining any weight , at present weight 35 kgs, he had primary complex when he was four years old. can you advise me Doctor: Hi,35 kg for a 11 year old boy is pretty normal, nothing to worry. As he had received TB treatment in the past, a positive Mantoux test alone need not be taken as significant, provided he don't have symptoms like recurrent fever, anorexia, weight loss or chronic cough.Just give him a healthy diet, that includes enough green leafy vegetables, fruits, pulses, nuts and dairy products. Also let him engage in regular physical exercise / play 40-45 minutes a day.Hope I have answered your question. Let me know if I can assist you further. Regards, Dr. Muhammed Aslam T. K., Pediatrician"
},
{
"id": 182142,
"tgt": "What causes jaw pain while chewing post molar tooth filling?",
"src": "Patient: I had an old filling replaced in my back molar. New onlay was too short causing the tooth in front to bite hard and causing jaw pain. Dentist made several adjustments to the healthy tooth by grinding it down. Now my front top and bottom teeth are hitting first and causing excruciating pain in gums and slight sensitivity. What should I do? Doctor: Hello,It is not uncommon to need a bite adjustment after a restoration such as onlay, crown or filling. Usually the new restoration is causing an interference. This often occurs on a movement from side to side called excursive movement. Please make sure this is checked and adjusted. From your description, this may involve other teeth than the tooth recently restored and still require further adjustment.Currently, you should stick to a soft diet. Give your mouth a chance to adjust. Take some Tylenol or Motrin, if necessary. Long dental procedures with local anesthesia can cause stress on the jaw. Do you grind your teeth? If so, do you wear a nightguard? You could consider purchasing a soft nightguard to reduce pressure on your teeth.. These are available we are dental supplies are sold such as your pharmacy.This may offer you some temporary relief.If your discomfort does not begin to decrease with time, please consider having another clinical exam with a second opinion. This will determine if the restoration was done properly. If you have no contact with the tooth and your bite continues to feel off, the onlay may need to be redone. Do you have any other dental treatment planned that may affect your bite? This may be a factor concerning the current restoration as well.Thank you very much for your inquiry. I hope my suggestions are helpful and that you feel better soon."
},
{
"id": 213238,
"tgt": "Masturbation, depression, overweight, less erection, no semen discharge. Cure for condition?",
"src": "Patient: Hi, i know a young man of 29 years who had been musterbating for a long time. He is ovewight and under stress of dipression due to various reasons. He has told me, in confidence that for the last 3 to 4 years he has getting full to slightly lazy erection but on doing musterbating no material ( serum ) is discharged. He is worried that this may not be a serious impediment to his marriage. Sir, can you give your advice. Regards Doctor: Hi, thanks for using healthcare magic Reduction in semen volume can be due to:(1) psychological reasons (2) short abstinence period (3) retrograde ejaculation (the semen goes in the wrong direction) (4) blockage of the ejaculatory duct (5) decrease in testosterone (6) problems or disease of the seminal vesicle which make the majority of the ejaculate.The seminal vesicle are glands located near the bladder The psychological stress that your friend is experiencing and his weight (excess weight is associated with hypogonadism which is when the sex glands produce little or no sex hormones such as testosterone) may be contributing to his problem. Your friend should consult a urologist who will be able to investigate him appropiately. I hope this helps"
},
{
"id": 187804,
"tgt": "Could my tooth infection be the cause of the extreme fatigue and constant achy feeling?",
"src": "Patient: Hi there. I have had extreme fatigue and weakness for a few months now. I was sleeping more than necessary and at times, sleeping up to 15-20 hours on weekends in addition to achy joints. I recently went to the Dr. because of a sore tooth. I scheduled a root canal and at that time, I was given an antibiotic, Clindamycin HCL 300mg, taking it every 6 hours for 10 days. I have completed the medication and feel amazingly better. I feel very awake and the extreme fatigue I was feeling has subsided substantially. Could the tooth infection be the cause of the extreme fatigue and constant achy feeling? Doctor: Hi,Thanks for asking the query,No this could not be the reason.I would suggest you to get the root canal treatment of the tooth completed.Also consult to a Physician and get the checkup done.Take care!"
},
{
"id": 119686,
"tgt": "What causes swelling,bruise and pain in the arm after donating blood?",
"src": "Patient: after giving 7 vials of blood my 90 year old mothers arm from shoulder to elbow is swollen, bruised, and fluid filled (like the ankles get when she forgets her diuretic) . Could the strap they use cause such damage to her arm, she complained of pain and it being too tight the entire time it was on. There is no bruise where the needle went in, just severe swelling above the site and red blotchy bruised. Doctor: Hi, It seems there is leakage of blood from the vein that was used for the blood transfusion. This happens quite often in very elderly age group patients. I think she should be taken to a hospital and attended by vascular surgery team. Take care. Hope I have answered your question. Let me know if I can assist you further. Regards, Dr. Nirmal Chander Gupta, Orthopedic Surgeon"
},
{
"id": 48151,
"tgt": "Suggest treatment methods for kidney transplant",
"src": "Patient: What is the procedure to start the process if some one from western world needs a kidney transplant urgently and come to Vellore to do it? What might be the ball park expenses for whole procedure? Any way the finding and matching the organ could be fast tract. Doctor: HelloThanks for query.Following procedure is followed for Kidney transplant in India.1) Normally we prefer related donor like brother mother ,son wife etc having same blood group of recipient(Patient).2) Written consent from the donor in front of legal expert (Magistarte) on stamp paper is mandatory.3) In case no related matching related donor is available a friend ,distant relative can be accepted as a donor .It should be voluntary There should not be any money transaction involved .4) Once the donor is confirmed to be matching all the necessary tests like blood test ,Renal angiography are done for recipient and donor for technical aspects of surgery of transplant.5) Once all the formalities are completed which normally takes 2-3 weeks The surgery is carried out by minimal invasive technique The cost involved ranges from 4-5 lakhs of Indian Rupees.( This is variable). Dr.Patil."
},
{
"id": 40704,
"tgt": "How can infertility be treated?",
"src": "Patient: I\u2019m struggling to fall pregnant after a miscarriage earlier this year. I have normal Periods that varies from 34 to 36. I\u2019m 36 years old. Dr subscribed Clomid but the medical aid do not pay for fertility meds so I asked for the Generic that cost less than Clomid. The Pharmacist gave me Fertomid 50. When should I start taking Fertomid 50? The Pharmacist said on the first day of the period, but the package insert said that I should start on the 5th day. I\u2019m confused Doctor: Hello,Thanks for choosing healthcare magic for your query...I understand your concern...Fertomid 50 is a tablet taken for ovulation induction, egg formation. It is taken from day two of menses to day six of menses , that means for five days in the evening.But it is highly recommended to go for follicular study when you are using tablets for ovulation induction. Looking at your age my suggestion to you is go for proper Gynaecologist guided treatment....I hope this helps...Feel free to ask any further queries...Regards,Dr Tejashree BhorGynaecologist,Dhanori, Pune"
},
{
"id": 117815,
"tgt": "Can WBC counts are higher than normal cause any complication during the surgery ?",
"src": "Patient: Hi Dr. I am 39 yrs old man and have severe breathing trouble due to deviated septum, planning to go for a surgery. Recently i was diagonized with Myloproliferative disorder so my platelet and WBC counts are higher than normal (750000 and 32000) respectively and Hemoglobin is 17.5, my question is can this blood condition cause any complication during the surgery ? Doctor: hi, you should take medicine for your myeloproliferative disease. it can cause complication in surgery. however it all depends on the risk benefit ratio, because if you have intolerable breathlessness then without wait you have to go for surgery there is no option, so consult your physician and anesthetist for your problem and take decision accordingly."
},
{
"id": 86045,
"tgt": "What causes lower abdominal pain and bowel incontinence?",
"src": "Patient: What is causing my lower abdomen to cramp and cause urgent loose stools for over a week straight? I am a female with PCOS. My last period was May 13 2018 and the one before that was March of 2016. (Pregnant and breastfed) I am NOT currently on my period. This is getting in the way of my job and my ability to parent. I feel like I can\u2019t eat anything without it causing me pain. Help Doctor: Hello and Welcome to \u2018Ask A Doctor\u2019 service. I have reviewed your query and here is my advice. * As per the narration and my clinical experience co relation, the current condition is most likely due to intestine infection from some sort of bacteria or virus which may be ingested via contaminated food, liquid or uncleaned hands. Hope I have answered your query. Let me know if I can assist you further."
},
{
"id": 29821,
"tgt": "Do abdominal pain, cough and nausea with rashes on hand indicate HIV infection?",
"src": "Patient: Hello sir.I had lymph, cough, body pain, burning sensation, nausea pain in the stomach and two small rashes on my hand.I took 4th genertion test and it was negative. But still the pain exist. After 72 days I took rapid test it s too negative. Can I rely on the tests. Doctor: Hi ThereI read details provided by you. If Your test for HIV is negative twice then you can rely and relax you are not infected. You might be going through some stomach infection or urine infection. Get done complete blood count, Urin routine analysis and chest X-Ray done and consult with a physician available near to you.Good Luck"
},
{
"id": 185411,
"tgt": "Can severe enamel dysplasia cause major heart issues?",
"src": "Patient: My son has severe enamel dysplasia. He was brushing his teeth and his front tooth broke off at the gum line. This is more than just a cosmetic issue and it effects all of his teeth. His teeth seems to be rotting away, even though he brushes and eats a healthy diet. My nephew also had the condition and had to have all of his teeth pulled at age 17. His pediatrician said the severity of his enamel dysplasia could cause major health issues, namely with hiws heart if it wasn't taken care of. is this true? Doctor: Hello,I would think the pediatrician is warning you that there is a high risk of infection without the shield of strong enamel. Decay can advance quickly in the dentin layer. Once into the pulp, bacteria has a chance to travel in the blood stream. Describing a 17 year old needing to pull teeth is a very severe situation. Placement of bonding or crowns to replace enamel and protect the teeth might be an option. If the recurrent decay rate is high, continued restorative care may be problem better treated with extractions and implants placed. If the condition is part of a syndrome with other organs affected or deficiencies, then treatment of or inability to treat the condition would affect dental treatment choice options.I hope this is helpful. I am glad to answer additional question if you have additional details to provide or more questions. Thank you for your inquiry."
},
{
"id": 221636,
"tgt": "What causes rupture of amniotic sac?",
"src": "Patient: My wife s amneotic sac has ruptured on 20/112/10 but we only found out on a scan on Tuesday, since then I have been looking at frrums for similar stories hoing for some hope that the baby will survive. One person said that drinking lots of water will help restore some of the amneotic fluid lost, is this true? Steve Doctor: Hi, I understand your concern The exact cause of rupture membranes is not known. mostly it is associated with history of fall/ incompetant os/ undernourished & weak mother/ twin pregnancies/ hydramnios/ some fetal abnormalities. Rupture of membranes mostly ends in delivery/ complications. A normal delivery is possible, in cases of rupture of membranes at term/ those who approach doctor & get treated meticulously. Rupture of membranes is mostly felt by the lady as a sudden gush of warm fluid out of uterus.,.. which continues to dribble out, changes it' s color .The discharge increases on coughing/ straining. Your wife did not have such symptoms.& the diagnosis was while the scanning . I doubt weather it is really a ruptured membrane or reduction of amniotic fluid.. which is a totally separate condition , can be treated by specific treatment. * I would advise you to consult a gynecologist, get thoroughly examined / investigated ( as per need ) & start with treatment advised strictly. Thanks."
},
{
"id": 206997,
"tgt": "How to overcome the feeling of jealousy?",
"src": "Patient: Hi, I introduced my fiance (girl) to my friend and made her feel confident and my friends too. In the other way, she doesn't have much friends but the only two she is hanging out with are boys who are interested in her. She told me that she loves me and it is just a meeting with friends but I cannot stop feeling bad about it. We spoke about it, she said that she understand my point of view and would not like if it was the contrary but she is still doing it. I am torn because I want her to have some her time but it with boy who are interested in more with her. I trust her but I know that a man is a man... if he has a chance he would take it! Am I wrong to feel that way? Doctor: DearWe understand your concernsI went through your details. I suggest you not to worry much. I can see your possessiveness towards her. Possessiveness is normal with everyone in the case of girl and boy friends. But nothing to worry in that aspect. In due course, you will learn how not to be jealous about these happenings. You are a versatile person. God made us like that. We know how to adjust with every situation. Therefore, be confident, you will change., If you require more of my help in this aspect, Please post a direct question to me in this URL. http://goo.gl/aYW2pR. Make sure that you include every minute details possible. I shall prescribe the needed psychotherapy techniques.Hope this answers your query. Available for further clarifications.Good luck."
},
{
"id": 173322,
"tgt": "Suggest treatment for seizure and panic attack in a child",
"src": "Patient: Hi my son is 2.5 year old he got is first seizure in Jan 2015 following a panic shock he had seeing a balloon burst before him. Initially he used to go still and collapse, we meet a neurologist who said it is seizure and started with Valproic acid without eeg and MRI, After taking valproic acid he started getting tonic clonic seizure, so they started increasing it, but it became over dose in the blood and he was admitted at sick kids in canada, their doctors veined valproic acid at this time his MRI and EEG was normal and doctors started with keppra and clobazam, which again increased his Tonic clonic seizure so he was re-admitted , this time his EEG showed slow movement in the left side and they also said it showed some seizure activities. by this time he started getting absence seizure and drop seizures, now he is in ketogenic diet+ Keppra 500mg twice a day, lemotrogine 7.5 mg twice a day, clobazam 10 mg twice a day, still he has more than 30 drop seizure per day and more than 100 staring episodes(absence seizures). we are not sure how to control currently he is treated at sick kids in Canada. we are worried over dose of medication has causing him a problem. can you help us with how we should proceed as we are planning to bring him to Chennai India for treatment. NOTE: there is no family history of seizures and some of his genetic results are negative. Doctor: Thanks for your concern shown here. Let us be clear that some time there use to be refreactory eplilepsy which is difficult to control.Now in the same time kindly check these things :One should do video event and show exactly the same to doctor.Also have an EEG when such seizures occur . By this I mean real time EEG .Take a closer look at seizures, if seizures are of similar type or they vary every time.well there must be some thing in EEG when seizures occures .If everything is ok consult child neurologist and children psychologist to rule out seizures from pseudo seizures. It will seem ackward but had to be ruled out if everything is normal."
},
{
"id": 98501,
"tgt": "How can pollen allergy and allergic rhinitis be treated?",
"src": "Patient: My name is Joe. I am 28 years old. For years I have developed allergies. At first, I thought them to be normal allergies caused by seasonal pollen and whatnot.. just taking medications like Benadryl and nasal spray seemed to be helpful. However, It became an everyday thing. Everyday, as soon as I open my eyes, my sinus starts burning and I start sneezing. My nose constantly runs, which can be embarrassing in public. The not so normal part is I started developing sores inside my nose and they cause me pain and irritation which makes me sneeze even more frequently. Another symptom is a foul smelling odor that kind of smells like a wet dog. I have been dealing with this issue for years and I can t seem to find a proper medication that works and brings relief. Can anyone shed some light on this issue so I can treat it better? Thanks Doctor: hi sir/madam,Thanks for your question on Healthcare Magic.As per Ayurveda, Allergic rhinitis is compared with Vata- Kaphaja pratishaya. The concept of Ama, Asatmya and virudh aahara also predict the allergic conditions.Ama is the product of impaired digestion and metabolism. It affects Rasa and Rakta Dhatu leading to manifestation of Pratishyaya. Wrong food combinations (Virudh aahar) having antagonist properties like fish with milk, fruit juice with milk, clarified butter with honey, ice cream after night meals, etc. leads to allergy.Ayurvedic line of treatment:-If the symptoms are severe and in chronic conditions, Vamana Panchakarma treatment, followed by Virechana is administered. This helps to relieve Ama and balance Tridosha.In some cases, where the patient has normal digestion strength, Nasya treatment (nasal drops therapy) is administered with herbal oils such as Anu taila or Shadbindu Taila. After the above Panchakarma treatments, Ayurvedic medicines are administered to improve respiratory immunity and strength of upper respiratory tract.The medicines given for allergic rhinitis are:-1. Nimbarajanyadi tablet \u2013 Contains neem and turmeric as main ingredients.1-0-1 after food.2. Chavikasavam \u2013 Fermented Ayurvedic liquid, also used in urinary tract disorders.10 to 15ml with water 3. Mahalaxmi Vilas Ras \u2013 Also useful in the treatment of skin diseases, diabetes, Sinus, non healing wounds, etc.1-0-1 after food.Diet to be followed are:-1. Take Light food, lukewarm water, Kapha nashaka foods like little spicy food, saindhav salt, legumes, soups etc.2. Avoid heavy food, fermented food, congestive food, too hot or too cold food, soar food, non vegetarian food, sweets, banana, curd, yogurt, ice creams, deserts, cakes, yellow grams, etc.3. Avoid Alcohol and cold beverages.Lifestyle improvement:-Use mask while driving vehicle or going through public places.Always take warm water bath.Do not expose to hot sunrays.Regularly take inhalation of steam.Regularly go for morning walk in warm weather.Do sufficient exercise.Do not reside in dark or dump places. Get fresh air.Do not get sleep at day time or immediate after dinner.Take sufficient rest.Yoga for rhinitis:-1.Ardha chandrasana2.Veerabhadrasana3.Salamba sarwangasana4.Setu bandhasanaHome remedies:-1.Take half liter water, add 20 gm salt, one teaspoon turmeric, boil it for 10 minutes and take it repeatedly.2.Take six cloves add six black pepper and one teaspoon of turmeric make powder and take it with honey.3.Take half cut lemon, spread a pinch of turmeric and salt on its inner surface, heat the lemon for one minute, and choose it.4.Take tea of basil leaf and mint leaf.5.Chew a piece of garlic and ginger.6.Take roasted horse gram \u2013 one teaspoon once or twice a day.7.Take extract off ajwain, black pepper, poppy seeds.8.Chew clove and black pepper through a day.9.Try coriander and cumin boiled in water.hope i was useful helping you.Have a healthy day."
},
{
"id": 57558,
"tgt": "Any suggestion for blood test showing high amylase in gallbladder?",
"src": "Patient: Hi I went to my gp because of pain in my right upper stomach under my ribs. I also constantly have an upset stomach. She order a ultrasound which showed sludge in my gallbladder and blood tests showed high amalyse levels. One min she thinks it s my gallbladder and next my pancreas. I have to go for another blood test and so confuse at the point. Any guidance would b appreciated. Doctor: Dear Friend.Welcome to HCM.I understand your concern. Raised Amylase with pain abdomen usually occurs on Pancreatitis.You require :1. CECT Abdomen2. MRCP3. Liver Function tests.You need to get workup to rule out stone in Common Bile Duct which can cause pancreatitis.Review with reports so that we can help you better.Stay Healthy."
},
{
"id": 15010,
"tgt": "Any medication for itchy, raised rash on the points of elbows, knees and armpits ?",
"src": "Patient: I have a itchey, raised rash on the \"points \" of my elbows, a spot behind one knee and now I have it on my upper arms right above where my arms meet.. Just outside of my armpit .. What is it ?? I am trying Cortisone cream, it helps but it's not getting better !!! Please help !!llk Doctor: Hi,I can understand your concern for the itchy, raised rash on the points of elbows, knees and armpits.After reading your history I can make out the possibility of Fungal infection of the area. If this is raised and causing flaking with maceration of skin along with map like appearance then this is the best possibility.For fugal infection, I suggest you to apply plane anti-fungal cream such as clotrimazole or ketoconazole. Take fluconazole 150 mg alternate days or tablet terbinafine 250 mg daily for 15 days. Do not apply any mixed cream or mixed ointments as it causes further aggravation of the problem.Do not pinch, pop or scratch the area. Keep the area dry and wear cotton clothes. Change the clothes two time a day. Do not get exposed to extremes of environment and protect yourself from heat and humidity.Take care."
},
{
"id": 108276,
"tgt": "Suggest treatment for lower back pain",
"src": "Patient: I am suffering with low backpain during past 3 years.. after I gave birth to child it got worsen and some times cannot even stand right with pain..I am a s/w engineer who sits in chair daily for 8 hours continuously...I consulted spin specialist and he suggested to do exersize, but when i start doing excersizes getting more pain..and now getting sciatic pain also in right leg and also from past 6 months getting left side upper back pain also Doctor: Hi,From history it seems that due to lack of daily physical exercise and increasing weight during pregnancy might give this problem.Not having habit of regular exercises and sitting in chairs for longer time gives stiffness of back muscles produces strain on back muscles.Go for regular exercises like running, walking, swimming daily for 30-35 minutes a for 5 days in a week.Reduce weight if you gained the weight.Take milk and high protein diet.Ok and take care."
},
{
"id": 185306,
"tgt": "Suggest remedy for cold & canker sores",
"src": "Patient: since april i've been getting colds at lease monthly and canker sores. I think it's because my immune system is low, but i can't stay well enough long enough to exercise. I'm a flight attendant and I think being exposed to a lot of germs doesn't help. I'm only 31 but i feel like i'm falling apart. I just don't know if I'm having a streak of unlucky health?I've been married over a year and have gained 30 lbs so I'm 168lbs. I'm 5'5 and have always been healthy. However since moving to a colder climate a few years ago I'm not as physically active anymore. Doctor: Thanks for using Health Care Magic.Read your query.I can understand your predicament due to the sores. I would suggest you some points which will be of help.* Stress is a factor which will aggravate the canker sores. Try to lead a stress free life.*Apply mucogel/ dentogel on the ulcers for two to three times daily .(keep it handy when you fly also).* Have a nutritious food including egg and you can start with a multi vitamin tablet after consulting a GP .* Avoid spicy food.*Use a betadine mouth wash for a while.Hope this was useful.Thanks and regards .Welcome for any further queries."
},
{
"id": 129844,
"tgt": "What causes right knee pain after a 10 mile race?",
"src": "Patient: Right knee painI am a runner an this past weekend i had a 10mile race afterwards my right knee had some pain in the upper outer side of it. I haven't had any swelling an its only bothering me on long distance runs any advice on how to solve this issue im having.Thank You Doctor: most of athletes suffer as ur suffering. . I,.etc, meniscus injuries of knee might be associated ligaments (structures around knee joint) I juries. . get MRI of ur knee joint and consult arthroscopic surgeon for further evaluation and management. thank u"
},
{
"id": 143758,
"tgt": "Suggest remedy for brain tumour",
"src": "Patient: hello my nephew just found a mass on his brain . they told him if he got dx by biopsy it may grow very rapidly. he chose not to have this done just following growth rate by pet scanning. how long can he survive if nothing is done. i know you can not give a specific time but estimate please? Doctor: Hi, brain mass should be operated by decompression and biopsy in superficial tumors and by steriotactic biopsy in deep seated tumors .Nothing can be told about survival time without tissue diagnosis.Thanks"
},
{
"id": 83165,
"tgt": "Why did increasing Eptoin dosage cause dizziness and visual problems?",
"src": "Patient: I am 30 years old boy and have seizure problem from last 3 years. Doctor suggest me Eptoin 100 mg (3 tablets) per day. Since last 2 years I have no problem and do my regular jobs and doctor also suggest me to take 200 mg daily. But recently before 15 days I have another attack and doctor suggest me to take 300 mg daily sir, after increasing the dosages I have some problems like shades on my left side of eye and feel weekness and i feel the attack will repeat. Sir please suggest me what I have to do. Doctor: Hello,You may experiencing the aura (symptoms before attack). Eptoin intake for long term may lead to encephalopathy, psychosis, locomotor dysfunction, hyperkinesia, megaloblastic anemia, decreased serum folate level, decreased bone mineral content, liver disease, IgA deficiency, gingival hyperplasia, and a lupus-like hypersensitivity syndrome etc. Hence, please consult with your neuro-physician he will examine and take EEG then treat you accordingly.Take care. Hope I have answered your question. Let me know if I can assist you further. Regards, Dr. Penchila Prasad Kandikattu, Internal Medicine Specialist"
},
{
"id": 18825,
"tgt": "Suggest treatment option for multiple sclerosis in elderly males",
"src": "Patient: My primary care physician is sending me to a rheumatologist for evaluation. I have tested negative for all the markers. M.S. is also being considered. What are the symptoms for M.S. in an otherwise healthy 66 year old white male? Treatment options and prognosis? Doctor: Hello and Welcome to \u2018Ask A Doctor\u2019 service.I have reviewed your query and here is my advice.60 is not a typical age for presentation of multiple sclerosis (MS). MS usually presents in early life around 20-30 years and is defined as multiple neurological deficits spread in space and time. Symptoms include optic neuritis, bladder dysfunction, muscle spasticity and ataxic gait. It is relapsing and remitting in nature most of the times. Treatment includes: 1. High dose steroids in acute replace to decrease the intensity. 2. Disease-modifying drugs (interferon, Natalizumab) 3. Bladder relief (catheterization, anticholinergics) 4. Baclofen, Dantrolene for muscle spasticity. Hope I have answered your query. Let me know if I can assist you further.Regards, \u00a0\u00a0\u00a0\u00a0\u00a0Dr. Muhammad Adnan Iqbal"
},
{
"id": 208920,
"tgt": "Can depression and bipolar disorder be symptoms of PTSD?",
"src": "Patient: I realized recently I don t react to being yelled at well. I suffer from depression and bipolar disorder. I got in an arguement with my mom over something simple and for the next few hours I couldn t function or stop crying. Is it possible I have PTSD? Doctor: HiThanks for using healthcare magicIt is not PTSD. In PTSD, patient should have very severe traumatic experience. In your case, it is an adjustment disorder. You should try some relaxation exercise after such kind of incident. That would help you to control such symptoms. You can also consult a psychologist for such technique. In case, you need further help, you can ask.Thanks"
},
{
"id": 208392,
"tgt": "Why do I feel vertigo while walking or changing head position?",
"src": "Patient: hello doc, i am 33 year old male , nowadays i am going through vertigo,whenever i change my head position in bed , or situp in bed ,or walk i feel vertigo,pls advice me what to do and thus is cause of concern ,let me brief u liitle more ,i used to have prolonged siiting in fron of my laptop in very awkward position u can say not comfortable ,also max times i used to have stiff neck and pain in nech while waking up.' Doctor: Hello,Your symptoms look like to be due to cervical spondylosis. Please get an XRay Neck, sr. Vit. D levels, sr. clacium and phosphorous levels.If there are even early cervical changes then you should ideally see an orthopedician and get the needed drugs. He would also teach you neck exercises which are to be done everyday.Donot bend neck or make sudden neck movements for sometime. Sleep using a normal small cotton stuffed mild cushioned pillow and avoid soft big mattresses.Pease get the above tests and proceed further. Dont worry, you wil surely get better.Good luck.Dr. Manisha GopalMD Neuropsychiatry"
},
{
"id": 76724,
"tgt": "Suggest treatment for respiratory infections caused after cardiovascular exercise",
"src": "Patient: I frequently get lower respiratory infections after cardiovascular exercise. I believe the infections are viral, but they do occur often; generally lasting 7-10 days. I produce thick, copious, and distinctly smelling phlegm ranging in color from yellow to green, depending on the infection. Sometimes I will produce clear mucus from the lungs for up to a month post the acute phase of the infection. It is frustrating and concerning that I have this issue when trying to get back into a cardiovascular exercise routine. I do use Symbicort for mild asthma and have albuterol for when these infections occur. I am a 40 y/o male with a 10 year pack/week smoking history (age 22-32). What do we have hear? Doctor: Hi Dear !! Thanks for your query to HCM.Read and reviewed your query and health concerns. You seem to suffer from Smoker's Bronchitis with Asthmatic Bronchitis-for which you use Symbicort and albuterol,to control athamatic attacks.When Cadio-respiratory exercises are done-the severity of exercises-leads to aggravation of Asthmatic Bronchitis, which later on complicates with secondary infection / or re-activation of subclinical infection in ?bronchectatic ? lung/bronchial tissues.And hence you get the respiratory infections after Cardio-respiratory exercisesDo's Suggested-You need to stop Smoking,due to which your lung/bronchial system is sensitive and prone to undergo re-infections.Plenty of Fluids before exercises and during and after exercises-to keep lung passages sufficiently humid and hydrated, would reduce such respiratory infection attacks, by timely removal of infected sputum-lot from the bronchial and lung tissue.High Protein/high carb bland diet/plenty of fruits /Tab Zincovit, would keep you away from- Symbicort and albuterol-bronchodilators and dependence on them, for long.Proper timely antibiotics support would be needed to reduce number of attacks from chronic lung tissue damage.Proper Bronchial physiotherapy would keep your lung tissues free from bronchiectatic residual infection.Hope this reply would help you to resolve the health issues with help of doctors attending on you.If need be, update any health issue 24 x 7 by a direct question to ME, at following HCM link-Dear, if satisfied,Don't forget to close this query with YOUR pleasing feedback comments to rate this reply and service, to boost the morale of incoming Emergency patients like YOU, at HCM services.If you want to update more details and ask more update queries ,You are most Welcome herewith !!Good Day!!Wishing Good Healthy Life in time to come!!Dr.Savaskar M.N.Senior Surgical SpecialistM.S.Genl-CVTS"
},
{
"id": 226198,
"tgt": "Had discontinued yaz, started again. No period, pregnancy test negative. What is it?",
"src": "Patient: Hi I have been taking yaz for 6 months. After the 6th month I stopped for one month and continued taking yaz on the 8th month. I have been spottin for almost a month now and have not yet gotten my period as normal, it should occur on the 2nd white pill of the Yaz conticeptive. I have taken a pregnancy test and it is negative. Any explanations for this? Doctor: Hi, Thanks for the query. After stoppage of the pill it will take sometime for regularization of the periods. As you again started pills after one month, it might have led to some hormonal imbalance leading to inter-menstrual spotting. If you had any unprotected intercourse during the pill free period there is possibility of pregnancy. So you once go for blood test for pregnancy. If the test comes negative, you continue with Yaz, the spotting can subside soon. For more details you can ask me through: http://www.healthcaremagic.com/doctors/dr-sree-gouri-sr/63429 Take care."
},
{
"id": 80460,
"tgt": "What is the cause of persistent cough?",
"src": "Patient: Thank you.I have a persistant cough which seems to have got worse over time.I do not have any cold or flue symptoms but a few weeks ago I noticed that I couldn't sing anymore as my voice keeps cracking and my throat has become dry and now feels a little swollen.I don't have any mucus or phlegm.What do you think could be the cause of this? Doctor: Hello dear, thanks for your question on HCM. Chronic dry cough can be due to1. Chronic lung infection like tuberculosis and fungal infection. 2. Bronchitis3. Vocal cord pathology. So better to consult pulmonologist and get done1. Clinical examination of respiratory system. 2. Chest x ray. 3. PFT ( pulmonary function test ). Chest x ray is needed to rule out lung infection. PFT is needed to rule out bronchitis. If all are normal then consult ENT doctor to rule out vocal cord pathology. Since you are singer, vocal cord damage can be there.So rule out this."
},
{
"id": 190094,
"tgt": "Blister on the gum after crowning a tooth. No pain. What to do?",
"src": "Patient: I have recently had a crown fitted on a lower tooth . It took a while to settle down and is now pain free. However, I now have a blister on the outer gum alongside the crowned tooth which when squeezed oozes blood and pus . I have tried rinsing with medicated mouthwash but the blister still remains and now smells like it is infected. i have no pain with this but am concerned that it is not going. What can I do? Doctor: if there is pus discharge & foul smell is cuming then, there is definite infection. get an x-ray done, localized cleaning & antibiotics can help sumtimes but if not then get the cap removed & after the infection subsides place a new crown"
},
{
"id": 104172,
"tgt": "Allergic to cold temperatures, get cold, popping ears, herpes on nose. Any ideas?",
"src": "Patient: Analizing my last 8 years or so I came into conclusion that I am alergic to the cold tempretures in surrounding environment or have somekind of virus that gets activated when I am exposed to cold weather, last 3-4 years I became even more sensitive. Basically if I get exposed to cold temperature - I get cold too easerly, also years pop, get herpes on the nose , other cold symptoms - too often!! any ideas? Doctor: yur immune system is low when you have allergies your immune system goes down and it cannot toletate external temprature change it increases with age keep your ext temp between 25 to 35 degree even if you are allergic to pollens and foods the body reacts to cold and very hot tempertures so get your allergy diagnosed by allergy tests and treat to increase your immunity"
},
{
"id": 62023,
"tgt": "Is the hard lump on inner thigh due to chafing?",
"src": "Patient: I have a hardened lump on the inside of my thigh, located within a stretch mark. Very tender but not extremely painful. I am prone to chafing - 10 hour shift days at work on my feet all day. Just healed up an infected chafe boil on my other thigh...but this is a hard mass. Related to chafing? Need to be concerned? Doctor: Hi,Dear,Welcom with your query to HCM.Studied your query in full depth of its details.Reviewed it in context of your health concerns.Based On the facts,You seem to suffer from-repeat boil with abscess in it-which is causing chafing of the supreficial skin.Treatment-You need to drain it to reduce risk of growth in its size, to be done under antibiotic cover from your Surgeon.Check for DM-diabetes by blood sugar test with your doctors advise.This reply would help you to plan further treatment soon with your treating doctors.Best of Luck and early recovery.Welcome any further query in this regard,which would be replied in next session.Good Day!!Dr.Savaskar M.N.Senior Surgical SpecialistM.S.Genl-CVTS"
},
{
"id": 34221,
"tgt": "Does rusty nail puncture on foot cause persistent pain in the leg?",
"src": "Patient: my husband stepped on a rusty nail exactly a month ago and immeditely went for the tetnus shot but, since that has happened he has pain in his leg all the way up to his waist. Pain that causes him to twitch and almost cry- Is this from the rusty nail? Doctor: HI Thanks for posting your query . Yes it could be due to the rusty nail . Post injury from the nail it might have caused infection in the foot which is slowing spreading up. Other possibility is enlargement of lymph node in the groin region due to infection restricted only to foot . Whatever the case is he needs to get examined by the doctor to check for infection in the foot or in the leg spreading to the thigh. If so then he needs to be started on Antibiotic course . Hope this information was useful to you. Any clarifications feel free to ask."
},
{
"id": 38408,
"tgt": "What causes gastric and weakness after taking malaria treatment?",
"src": "Patient: Hi docter, my mother has been recognized with TB few weeks ago and mecidines were going but she was feeling very weak and suddently she recognized with malaria and again hospitalized now.. She was doing lots lf vomiting but since yesterday vomiting has stopped and today dr is releasing her from hospital but she feels lots of gas when she eat something.why can u please help me to understand what should we do Doctor: Hello, thank you for your contact to healthcare magic. If I am your doctor I suggest you that malaria treatment causes vomiting by causing intense gastric irritation. As your mother is already taking the drug irritation will be some of more. As the vomiting is stopped still stomach is releaseing acid and body is try to neutralize it. In process on neutralization it releases the gas, so your mother is feeling gas. Kindly continue the treatment as suggested by doctor. If you have to ask for anything you can contact me. Dr Arun Tank. Infectious disease specialist."
},
{
"id": 151934,
"tgt": "Is it possible to come out of 30 days in coma ?",
"src": "Patient: My mother suffered a severe intraventricular haemorrhage post craniotomy of a pituitary adenoma . She is in a state of coma for the last 30 days and had severe pneumonia .She has been under course of antibiotic which is over now. The neurosurgeon and the other doctors attending to her has advised us to take her out of the hospital to prevent further infection and give care at home. Is there any proper treatment Doctor: Hello and welcome to healthcare magic forum! I can give you one contact. It's the institute of neurosciences, circus venue. But its still a good advise to take her home and get her taken care of at home."
},
{
"id": 218805,
"tgt": "Is pregnancy possible while on birth control pills?",
"src": "Patient: Good night! I have a question: if I took a doxycycline 50 mg yesterday around 4pm, for how long will it stay in my system? I m also taking a birth control (every day, same time)and tonight I had sex with my husband and he came in. Any chances of pregnancy? Thank you! Doctor: hello user.dont worry.birth control pill provide 99 percent protection..doxycycline does not decrease the effectiveness of birth control pill.thanks."
},
{
"id": 50884,
"tgt": "Back pain after bariatric surgery, ultrasound shows mild right hydronephrosis. Any other test needed ?",
"src": "Patient: hello doctor i had bariatric surgery done one year back. since two days back i had back pain when consulted my doctor suggeste ultrasound. results showed mild right hydronephrosis . i immediately cons ulted urologist he suggested plain ct kub and he said that the results were okay. my doubt is CT scan enough to find out kidney disease or should i take any other tests to confirm kidneys are woking normal. please help Doctor: Hi Hydronephrosis is enlargement of kidney.It can be due to many causes. Please take IVP(Intra venous pyelogram) to know the kidney's function before going for CT scan. Get Blood urea and Serum creatinine. Nothing to worry IF you come back with IVP report,i will clarify further. Till then,please take Norfloxacin tablet along with Dicyclomine and mefenamic acid tablets. Wish you good health Regards"
},
{
"id": 211286,
"tgt": "Is it possible to have ADD at 60 years of age?",
"src": "Patient: I'm wondering if it is possible to have ADD considering I am 66 years old. I've always had difficulty with concentration, struggled in school and inability to focus. I've compensated for it by applying myself diligently to any task, and in spite of it have a strong work ethic, but nonetheless, it was/is still difficult. I'm also a great procrastinator, always have been. Otherwise, I'm in great health, always have been, take no medications other than a daily vitamin and am 5' 9\" and weight 150 lbs. and have yearly checkups. Thank you. Doctor: Hello,welcome to Healthcare Magic.Symptoms of attention deficit disorder (ADD) usually start around 5 years of age. Its symptoms persist throughout the life. But with time severity of symptoms decreases.If at 66 years of age, there is aggravation of symptoms then there is need to rule out neurological problem, thyroid imbalance and vitamins deficiency.Regards,Dr Ashish Kumar Mittalwww.99doctor.com"
},
{
"id": 161383,
"tgt": "Suggest remedy for acid reflux and enlarged tonsils while having chronic lung disease",
"src": "Patient: my daughter was born 3 months early has chronis lung diease.. Came home with no cough and has improved about 3 months at 5 months corrected developed a runny nose then a cough which she still has now 3 months later she coughs till she vomits especially in the morning.She has been tested for all bugs which came back negative and She has huge tonsils which she had been given steroids to redude the size..The doctors i have been too say it is a mixture of her bad reflux and still or enlarged tonsils could this be correct its very frustrating as she is not getting better Doctor: Hi, What is the age of the child? There is no need for steroids to reduce the size of the tonsils. Reflux may cause these symptoms. Hope I have answered your query. Let me know if I can assist you further. Regards, Dr. Rajmohan, Pediatrician"
},
{
"id": 54589,
"tgt": "How to recover from liver cirrhosis?",
"src": "Patient: and liver cirrhosis in dec 2011 modified akt staarted because of cirrhosis i.e inh 300, ithabutol 1000, and oflox 800. and steroid . and liver cirrhosis in dec 2011 modified akt staarted because of cirrhosis i.e inh 300, ithabutol 1000, and oflox 800. and steroid . steroid... Doctor: Hi thanks for contacting HCM..Noted you have cirrhosis of liver.AKT drugs can damage liver more.So you have started modified AKT to minimize the liver damage....Continue using drug for tuberculosis...As you have cirrhosis take life style measures....Take low fat diet....Green leafy vegetables more....Early in morning take grined spinach and carrot juice....After completing drug for TB , for cirrhosis TIPSS like invasive procedure done if portal hypertension associated.....Ultimate treatment is liver transplantation...Junk foods and non veg avoided.Take care...Dr.Parth"
},
{
"id": 135207,
"tgt": "Suggest remedy for swelling and bleeding from a sore toe",
"src": "Patient: I had a soar toe and it was swollen and then one day started to bleed. When I applied pressure something came out from under my toe nail that looked like a tiny grub? What could this have been and what do I do with my toe now? Soak it, put antibiotic ointment on it? Doctor: Hi thanks for asking question in HCM.here according to history it is most likely that you have ingrown toenail.Here is some suggestion for you.Start treatment at home as follow.Sock your toe in warm water for 4 to 5 times a day.Cotton ball that is soked in oil used to separate toe nail edge from skin.For pain relief take simple analgesic.To prevent infection apply topical antibiotic over affected area.If still you not get relief then surical treatment needed.sometime partial nail removed.I hope my suggestion will definitely help you.thanks"
},
{
"id": 36420,
"tgt": "Suggest treatment for yeast infection",
"src": "Patient: Have urine smell.sounds weird but smells like food. Had yeast infection and was given fluconazole right before this.Also had feeling like something was pushing out of me no pain. Actually thought I d might be coming out. Doc sent me for vaginal sonogram all okay and said might be kidney stone.Small stone came out one day.No more uti symptoms. Only problem food smell in urine. Took amoxicilen for sinus for 2 days those days no smell.Color is fine .I m freaking out.Oh go doc did a dip. Had blood work done for general checkup and kidney function was good. Doctor: Hello,Welcome to HCM,As you are having fungal infection for which you have visited your doctor and got some treatment.The affected region is more prone for fungal infection, as it is a normal commensals of this area which is kept under control by the helpful organisms, whenever there is a change in the environment either by the change in the temp, other infection or poor hygiene will invoke the growth of fungus and produces the symptoms.For your condition I would like to suggest1.Keep the area clean and dry by applying neosporin powder.2.Topical antifungal cream like ketaconazole3.Single oral dose of Diflucan 150mg.These measures will help you to control the infection and makes you comfortable.Thank you."
},
{
"id": 143690,
"tgt": "What is the cause of fever and dizziness,and are they related to each other?",
"src": "Patient: Hi, I fell down the stairs 9 days ago and now i am starting to feel dizzy and I have also had a fever, however the fever is now gone and my dizzyness still remains, i am wondering weather these two might be in relation to each other and if i need to go and see a doctor, or if i am just feeling dizzy because of the fever i have had, Doctor: Hi, you had fall from stairs and fever after that.Most likely dizziness is due to fever .Fever with vomitting and diarrhoea which cause dehydration can cause dizziness.Common causes of dizziness are-AnemiaAllergies.Illnesses such as the flu or colds.Vomiting, diarrhea, fevers, and other illnesses that cause dehydration.Very deep or rapid breathing (hyperventilation).Anxiety and stress.The use of tobacco, alcohol Kindly consult the physician for workup. Thanks"
},
{
"id": 92939,
"tgt": "Pain in abdominal area, low grade fever, feeling cold, fatigue. What could be this?",
"src": "Patient: Woke up yesterday morning with pain in abdominal area from underneath my breast to my pelvic floor. Very hard to get up and down from sitting and lying position. By late afternoon I was freezing and had low grade fever. Slept from 6pm til 7 am this morning. Abdominal pain is now from about belly button to pelvic floor. Have had bowel movements and my kidneys are working. Not as hard to get up today but still can't get around very well. Fatigued, no fever but still I'm cold. Have high pain threshold but this is very painful. No gas but feels like I'm full of it. Doctor: Good eveningIs pain associated with food intake, what relieves the pain?I would advice you to see doctor and have your blood checked for infections, complete blood count and other tests.I would also recommend abdominal ultrasound to be done.Hope I answered your question, will be happy to answer more questions from you."
},
{
"id": 147609,
"tgt": "What causes seizures during sleep?",
"src": "Patient: Dear Dr., I had two seizure during sleep in 2012 and I lost my passenger service licence. Neurologist said that it is due to sleep deprivation as I was driving cab at nights and sleep only 3 or 4 hours during day time. He done my EEG and CT and found nothing. Dr. prescribed me epilim 200 ec two times in a day. I am worried about what is this? Is it epilepsy or something else? Should I take this medication as this is anti-psychotic and has huge side effects. Can I get back my passenger service licence? Doctor: Hi,Thank you for posting your query.I have noted your symptoms, investigation reports and current medications.Most probably, you are suffering from epilepsy. In many cases of epilepsy, CT scan of brain and EEG may come out as normal. It may be worthwhile doing an MRI brain- epilepsy protocol to find out more details.Epilim is also an anti-epileptic drug and is reasonably safe. You should continue it as per your doctor's advice. Regarding the license, you need to check the local laws.I hope my answer helps. Please get back if you have any follow up queries or if you require any additional information.Wishing you good health,Dr Sudhir Kumar MD (Internal Medicine), DM (Neurology)Senior Consultant NeurologistApollo Hospitals, Hyderabad, IndiaClick on this link to ask me a DIRECT QUERY: http://bit.ly/Dr-Sudhir-kumarMy BLOG: http://bestneurodoctor.blogspot.in"
},
{
"id": 40134,
"tgt": "Suggest treatment for paraplegic",
"src": "Patient: I'm 29 and a paraplegic I can't feel anything from my nipples down I can still use my arms i have had a lump on my left testicle for at least 12 years now it appeared when I was about 16 or 17 before I became a paraplegic sometimes it is soft and sometimes it is hard 2hat could it be Doctor: Dear Friend.Hi , I am Dr Anshul Varshney , I have read your query in detail , I understand your concern.It might be:1. Cervical compression2. Transverese myelitis3. Pott's spine.4. GBS.You should immediately consult your doctor.This is my personal opinion based on details available here. If you want to discuss your issues further, you may please ask usStay Healthy. Dr Anshul Varshney , MD"
},
{
"id": 91615,
"tgt": "Suggest remedy for abdominal pain after taking antibiotics for chest and sinus infection",
"src": "Patient: My sister (63 yrs old) has had flu synptoms for about 10 days. She did go to the clinic after experiencing a fever of 104 and bouts of coughing. She was diagnosed as having chest infection and sinus infection. She seemed to get better after having antibiotics but she has gotten worse after finishing her script. She started with diareah which has now stopped but was also vomiting whch is now become just a nausea but she has alot of abdominal pain. She is taking immodeum and gingerale but not able to eat. She is extremely tired but not lethargic. Doctor: the hyper acidity after medicines used can be causeyou can use ppi agents weith antacid gels 3-4 times a day with plenty of fluids and nin oily non greesy and non fatty food for few days"
},
{
"id": 23254,
"tgt": "What are the symptoms of supra ventricular arrhythmia?",
"src": "Patient: age 7, 4'5\", 50 lbs, at 1 wk old, was in hospital with super vintricular / arithemia. (ck spelling.) what does it mean when you are born with a small hole in your ear. Is there a learning disability? He is very smart has a large vocabulary and very out going,but, difficulty reading. Doctor: HiSupraventricular tachycardia (SVT) is an abnormally fast heart rhythm arising from improper electrical activity in the upper part of the heart. There are four main types: atrial fibrillation, paroxysmal supraventricular tachycardia (PSVT), atrial flutter, and Wolff-Parkinson-White syndrome. Symptoms may include palpitations, feeling faint, sweating, shortness of breath, or chest pain.MechanismThe main pumping chamber, the ventricle, is protected (to a certain extent) against excessively high rates arising from the supraventricular areas by a \"gating mechanism\" at the atrioventricular node, which allows only a proportion of the fast impulses to pass through to the ventriclesHOLE IN EAR HAS NOTHING TO DO WITH SV AYTHMMIAits defect in conductionsystemof heart ,it not a learninng deformity"
},
{
"id": 35327,
"tgt": "How to confirm tb?",
"src": "Patient: Before 4 monthes i got typhiod and after that i have gaind 3 kg .Recently i went to doc for my delayed periods who told me for tb test myreports shows tb IgA is negative,tb IgG is 160.00 and tb IgM is 1.49. Am i effected by tb and only blood test enough to onfirm tb? Doctor: Thanks for asking in healthcaremagic forum Usually Tuberculosis is diagnosed by sputum smear and X-ray tests not antibody tests. I think you have confused typhoid with tuberculosis. Typhois also diagnosed with widal test. So, please send me the details regarding the test report for further suggestions. All the best."
},
{
"id": 52411,
"tgt": "Why is Lipitor prescribed for fatty liver?",
"src": "Patient: Yes my total cholesterol is114 , ld l is 29 hdl is 61. I am 71. They tell me I have fatty liver . I have been put on lo Iipator since this test but I do not understand why. Can you tell me. I had chemo for breast cancer 4years ago but otherwise healthy and active and .cancer has not returned. Doctor: Hi, Your total cholesterol level is within limit as per mentioned history. So according to my opinion no need to give any anti lipidemic drug like statin. You can Consult good physician for examination and discuss accordingly. You only need to take healthy diet and exercise more. Loose weight if obese. Hope I have answered your query. Let me know if I can assist you further. Regards, Dr. Parth Goswami, General & Family Physician"
},
{
"id": 65684,
"tgt": "Is it to be concerned about a lump in the thigh?",
"src": "Patient: 5 mo ago I noticed that the side of my thigh had noticible caved in. In the center of this indention was a tiny hard lump. My mother told me not to worry it will go away. So now 5 mo later it has very quickly getting larger but only about pea size but in addition to that lump there are two others. One being pea sized and one like a large knotty like mass that seems to stretch about 3\" down my leg starting right after the second knot I found. I have no previous injury or anything and im not sure what it is. They are hard and virtually painless, they do not move around they seem to be attached to my thigh muscle. Any ideas? I have a doc appt on monday but just wanting to see if I am going to be wasting their time if I go lol. Doctor: Hi! I would like to summarize your description of the lumps as 'spontaneous, hard, painless and fixed/attached to underlying muscles...'and I generally come to following possibilities as per the final diagnosis after fine needle biopsy like:1. neurofibormas2. lipomas3. dermatofibromas4. lymph nodes5. parasitic nodules6. muscular nodules/neuromasI think there is nothing to worry about these but you need to confirm the diagnosis therefore please see a physician and go for a needle biopsy;regards,"
},
{
"id": 183680,
"tgt": "What are the symptoms and treatment for Irritable Bowel Syndrome?",
"src": "Patient: I have a situation. I don t know if it s IBS or what, but my stomach is messed up. My bowel movements are irregular and when I do go, it is almost always a loose stool accompanied by constipation/pain on my left side (intestine/colon). I find myself on the toilet 6-7 times a day. I also have issues ingesting food/drinks - as it doesn t want to go down sometimes - what I would suspect to be heartburn or acid indigestion. It s all really embarrassing. I m 29 years old and otherwise in good health. I need some guidance as to what it could be and who I should go see? Doctor: Hello.You can go to gastroenterologist or physician for diagnosis of your condition. You can consult for dental diagnosis here.Wish you good health"
},
{
"id": 165074,
"tgt": "What causes lower abdominal pain and nausea in children?",
"src": "Patient: Yes. My son is 10. He has been sick for 7 months. He complains of pain in his lower abdomen (colon area). He has nausea but not really throwing up. They have him on amitriptyline(anit depressant) to help dnumb the muscles & nerves. But all his blood work & tests came back good. But they did not run a colonoscopy. Is it possible for him to have colon problems and it not show up in blood work. My mom has crohns since she was small. Her Gi dr says colon trouble can be silent & my sons drs could be missing it. Without a colonoscopy they cant rule it out. Is this possible? Doctor: Dear Parent,Your child's long lasting lower abdomen (10 months) pain is a cause of concern. Given a family history of Chrohs disease it is pertinent that Crohn's disease is ruled out in your child. The most definitive test for Crohn's disease is to take a small sample of tissue from his colon( biopsy) during colonoscopy and study it under a microscope. It is surely possible that your child can have Crohn's disease with a normal blood work up. I suggest that you consult a gastroenterologist as soon as possible who can help diagnose your child's condition and initiate appropriate treatment."
},
{
"id": 192850,
"tgt": "What causes pain in left kidney after nocturnal emission?",
"src": "Patient: Hello doctor,I've been having nocturnal emissions once every two or three months for the last year. I'm 23 years old. Today I woke up after I had one and I feel a very very slight pain in my left kidney. Also I want to mention I just had a nocturnal emission earlier this same week. Should I masturbate or often or less? I do it once a week.Appreciate your help.Thanks in advance,Joe Doctor: Hello, I don't think so if it is related to nocturnal emissions. It might be due to some other reasons like stones or renal tract infections. Nocturnal emissions are normal and masturbation is likely going to lessen the frequency of nocturnal emission. I suggest you to visit your doctor to run some tests like urine routine examination and culture studies along with ultrasound renal tract. Hope I have answered your query. Let me know if I can assist you further. Take care Regards, Dr SAMEEN BIN NAEEM, General & Family Physician"
},
{
"id": 149020,
"tgt": "Have plaque build in one or more arteries of brain, CATT scan shows pneumonia. Prevent stroke?",
"src": "Patient: Hi- I am trying to gather as much information to send to my Foster sister on Atherosclerosis of brain arteries. Her Father has some plaque build up of one or more arteries of the brain, but I do not know which are affected. I explained some to her this afternoon, but I am trying to get more explicit information to give to her, to prevent a stroke in her father. She has durable power of attorney for him. They just did a CATT scan, as he gave them a bad scare on Friday night, but he has pneumonia. The CATT Scan did show he has atherosclerosis of at least one brain artery. He fell off a ladder helping a neighbor a few weeks ago and broke his pelvis and some lumbar vertebrates. Doctor: Hi, thanks for using healthcare magicStroke prevention involves controlling risk factors such as high blood pressure, diabetes, high cholesterol, smoking.By controlling these factors the risk of a stroke is reduced.This means that all of the above should be maintained within normal range.Different blood vessels are involved in supplying different aspects of the brain and signs and symptoms of stroke would vary somewhat according to the area affected.I hope this helps"
},
{
"id": 186693,
"tgt": "Why am I having pain on the left side of my mouth?",
"src": "Patient: The left half of the roof of my mouth is very sore -- almost like I burned with hot food (only I didn't); there are painful red bumps on my gums and even on the outside & left side of my nose. It's only on one side of my mouth and one side of my nose -- but they are quite painful. Doctor: thanks for your query, the feature you are presenting looks like herpes zoster virus infection or it can be herpes simplex virus infection..consult your oral physician and get it examined immediately and treated..you can take topical analgesics and anesthetics.. after confirming the diagnosis you can start with antiviral drugs..i hope my answer will help you..take care.."
},
{
"id": 159627,
"tgt": "Suffering with anxiety, straining of eyes in front of the light, neck stiffness, nausea, weakness, pressure in the leg, family history of cancer. Brain tumor?",
"src": "Patient: Hey i have been suffering with anxiety for a long time my eyes started to become strained by the light i would get a stiff neck and feel a little nauseous it worsens in front of the tv and bright light. My docter checked my eyes and sent me to the opticians who gave me glasses and said my eyes werent working great together. Lately it has been mainly in one eye pressure and my pupils vary from each other sometimes one looks bigger. I also have been having weakness in one leg and pressure in my foot but this comes and goes. Im worried its a brain tumor , but as my step dad died of cancer this year could i be building all these symptoms up in my head out of worry, im terrified and self diagnose daily?? i want to get tests but the doctor has always diagnosed it as anxiety. Doctor: it is better you should take MRI brain scan"
},
{
"id": 115410,
"tgt": "What causes neck pain/heavy headedness while having elevated uric acid levels?",
"src": "Patient: dear Sir,my uric acid levels is slioghtly elevated , came to know in the recenet preventive jhealth checkups.i am having neckpain and also heavy headness.. is this becoz of uric acid. i am having HTN also .is there any relation between HTN and elevated uric acid Doctor: Hello,uric acid has nothing to do with either of your symptoms. The same applies for hypertension. Hypertension and elevated uric acid levels are sometimes interconnected but not causally. That is hypertension is not caused by uric acid and uric acid elevation is not caused by hypertension. They just happen to exist in combination particularly in overweight or obese individuals along with increased triglycerides, blood glucose, waist perimeter, low HDL (high density cholesterol) levels etcAn isolated uric acid elevation (that is without the other findings that I mentioned, particularly when lower than 9-10mg/dL) is nothing to worry about unless you've suffered gout attacks, have had uric acid renal stones or have had uric acid skin nodules.If you're overweight, keep in mind that weight loss will make uric acid levels decline. Alcohol consumption is detrimental regarding uric acid levels control.I hope you find my answer helpful!If you'd like further information, please contact me again.Kind Regards!"
},
{
"id": 215073,
"tgt": "How to get rid breathing problem and his lungs are weak ?",
"src": "Patient: hello doctor , my friend,age 19,has frequent breathing problems and his lungs are weak. i would like to know what kind of food will strenthen his lungs? also, he goes to a gym, is it advisable for him to continue doing so? Doctor: Hi Welcome to Healthcare Magic Forum Please mention his symptoms. Breathing problem can be- asthma, bronchitis, a condition of the heart etc... Also what treatment he/she is taking for this. How long has he/she had it? Good nutritious diet is must here, to keep the immunity up. Exercise and gym can be suggested only if the name of condition is known. Takecare For further enq- dranshita.rathore@yahoo.co.in"
},
{
"id": 112191,
"tgt": "Severe constant upper back pain, taking Ibuprofen, no relief. How to get cured?",
"src": "Patient: I have had severe upper back pain for over a month. It was combinded with severe spasms but the spasms have subsided with taking Ibuprofen 800 that was prescribed The pain is always in the same place and radiates out. Ibuprofen 800, icing, heat, and rest, have not helped! It s constant severe pain. Doctor: Hiplease do not indulge in self medication Pain killers have adverse effect on kidneys.Hot formations may do no good if it is spinal pain GET MRI of the part done and consult ortho ot physio doctor do not perform spinal exercises without advice of ortho or physio doctorDr Lal Psychiatrist"
},
{
"id": 182881,
"tgt": "Can phlegm while coughing be indicative of jaw infection?",
"src": "Patient: Hello, I wanted to ask a question regarding myself coughing up mucus/phlegm almost every morning that is black in color or is clear with tiny blackish/grey specs in it. I tried doing some research online but with little results. I do smoke marijuana on daily basis pretty much but I do not smoke cigarettes at all. On some of the sites I was reading on suggested that it is the tar/resin being expelled from your lungs and is the bodies natural cleansing method for getting the toxins out of the body. But I've also read that it could be caused by a jaw infection which oozes out from in between the teeth and gums and trickles down the throat to the lungs and that the infection could be caused by having wisdom teeth removed which I had done a few years ago. I was hoping to get a little insight before scheduling a doctors and/or dentist appointment. Doctor: Thanks for using Health Care Magic.Read your query .Removal of wisdom tooth followed by what is mentioned is not a relevant thing and if read somewhere ,there are no such theories as such.Marijuana smoking is harmful .Black mucus and phlegm can includes the tar from the burning process caused marijuana.The longer you wait, the worse it will get. It will effect the whole body and once addicted ,it is difficult to quit. I suggest you to stop using it completely.Visit your general physician and have yourself evaluated for any lung infection and have it treated. Hope this was useful.Thanks and regards."
},
{
"id": 46584,
"tgt": "Can tritace 10 mg and avapro hct 300 mg be taken during kidney problem?",
"src": "Patient: hi yes asking about been on meds is it good to take tritace 10 mg and avpro hct 300mg i have blood pressure and kidney problem . There s been a change in meds still takiing avpro and amiodipine 5 mg not that happy about it as i see its not to treat the kidneys . thanks rina Doctor: ramipril ( ACE INHIBITOR) ( titrace) is good to retard kidney disease with proteinuria. irbesartan (avpro)also does same. but hydrochlorothiazide can cause volume depletion and renal damage."
},
{
"id": 219870,
"tgt": "Suggest treatment for missed periods",
"src": "Patient: Am having a missed period. My last periods first date is 23-12-2013. I had sex with my boyfriend on Jan 15 and this is my first time and we had unprotected sex. I don t feel any changes in my body condition but not getting my period is starting to worry me. I have a regular period cycle.. I did a urine pregnancy test couple of days back and it was negative. Am afraid to consult a doc, as they mite embarass us, so me and my bf are confused and stressed.. Please advice.. I live in chennai Doctor: Hello, and I hope I can help you today. Skipping a menstrual cycle is common to many women at least a few times in their life. Your cycle can be affected by factors such as travel, weight changes, and stress... Even stress of worrying about pregnancy. So I think you can be reassured that your negative pregnancy test is accurate. To be extra sure, you may want to retest in one week. I hope I was able to adequately answer your question today and that my advice was helpful. Best wishes,Dr. Brown"
},
{
"id": 2574,
"tgt": "Are there pregnancy chances post intercourse a day after periods has ended?",
"src": "Patient: hi i had unprotected sex with my boyfriend for less than a minute. he had pre cum but never came. i am getting some symtoms but i feel like it is paranoia. i had sex the day after my period ended. which was last monday. i started ovulation today. am i pregnant? Doctor: Hi,It is very less likely to get pregnancy after periods, after periods 10-20 days will be the most fertile period. But there may be chances. The symptoms will be less likely to be related to the pregnancy. Wait till your next period and get an urine pregnancy test if it is delayed.Hope I have answered your query. Let me know if I can assist you further. Regards,Dr. Vidya Jyothi B"
},
{
"id": 105166,
"tgt": "Ears pierced, swelling, pus, pain, small lumps in ears, calcium deposits. Allergic reaction?",
"src": "Patient: i had my ears pierced when I was five years old and had zero problems until my late twenties. I ve always had a nickel allergy , but even gold or neobium earring backs caused my left ear to swell horribly after wearing earrings for a day (really badly with pus and pain). There are small lumps in my ears I ve always had (I assume they re calcium deposits or the like), but have no idea why I am reacting so horribly to wearing earrings now. Doctor: Hello Lisa.. Thanks for your query. From your words it appears that you are allergic to certain metals, so its best to avoid them.As for the allergic episodes you can use antiallergy medications like Levocetrizine 5mg tablet twice daily for 5 days and for pain and inflammation you can use NSAIDS like Paracetamol-Aceclofenac combination.If it gets infected and forms pus then you may need antibiotic ointment too.Do consult a Dermatologist and do the allergen-specific IgE antibody testing to screen for an allergy to a specific substance or substances and avoid contact to such substances in future.And dont worry too much. You just need to learn which things suit you best . Take care :) ."
},
{
"id": 120765,
"tgt": "Suggest treatment for chondromalacia patellae",
"src": "Patient: HI, I am Durga Prakash. I am suffering from chondromalacia patella, The patella is dislocated & is located upward & laterally outward,The tibia torsion is also high, I am presently doing some exercises as prescribed by Orhopaedician for the past 3 months. Doctor: Hello,I read carefully your query and understand your concern. The damage caused by\u00a0chondromalacia can\u00a0often\u00a0heal. Conservative treatment is usually recommended first since rest and physical therapy may eliminate the symptoms. The goal of\u00a0treatment\u00a0is to reduce the pressure on your kneecap and joint. Resting, stabilizing, and icing the joint may be the first line of\u00a0treatment. If conservative treatment for several months fails, surgery may be recommended.Hope my answer was helpful.If you have further queries feel free to contact me again.Kind regards! Dr.Dorina Gurabardhi General &Family Physician"
},
{
"id": 111552,
"tgt": "What to do for back pain after fall from stairs?",
"src": "Patient: Hi I had a fall down some concrete stairs last night. Sense the accident iv had lower back pain on the right hand side and all so have stomach pain on the right side. Iv have a bad bad on my left hand side just under my rib cage. I can't go for a wee. Doctor: Hello,I have gone through the case and found that it might be simple muscular spasm or any fracture. So I will advise if there is inflammation increases then immediatego for X-ray of thoracic and lumbar vertebrae.Meanwhile take Necoxia 120 mg for two times a day and apply muscle relaxant gel and hot water bag for relief.Hope my answer will be effective for you.Thanks"
},
{
"id": 192292,
"tgt": "What causes pain in lower thigh above knee during intercourse?",
"src": "Patient: I tend to have this pain in my lower thigh, above my left knee sometimes when I'm having intercourse with my wife (at the point of ejaculation). It occurs quite often - enough to be concerned. Could there be some sort of obstruction or circulatory issue to be concerned about? Doctor: Hi, It is likely because of muscle spasm. Try changing position and you can take tablets for muscle cramp. It is unlikely to be serious issue. However, if the problem persists, please see doctor. Hope I have answered your question. Let me know if I can assist you further. Regards, Dr. Sujoy Dasgupta, OB & GYN Specialist"
},
{
"id": 86447,
"tgt": "Could there be a connection between lower abdominal pain and neuropathy?",
"src": "Patient: I have been experience pain in my lower left abdomen since mid-December. I have seen numerous doctors, had several tests and have not received any help except to be passed on to other doctors. I recently saw a medical professional for another reason and she asked if anyone ever mentioned neuropathy ( I am a breast cancer survivor and I had a mastectomy of my left breast in April, 1998). Could there be a connection? Doctor: Hi. Thanks for your query and an elucidate history. Since you had a breast surgery long ago, having neuropathy now is not possible due to this particular reason. Yes neuro problems like prolapsed disc in th lower thoracic region can cause pain in lower abdomen, but has a specific character like cutting or burning pain. Please go for MRI of the spine and CT scan of the abdomen to find the cause."
},
{
"id": 73625,
"tgt": "What causes pain in chest and difficulty in breathing?",
"src": "Patient: i am having chest pain its happend be4 and it hurts to breath in i am 29 yr old female pretty healthy normal weight . once be4 i got admited when i was 19 for chest pain and they said my heart had been inflamed and didnt knw why? i have had alot of times i hVE WOKE UP WITH CHEST PAIN but a physch doctor a long time ago thought i might have panic attacks in my sleep i am starting to get more frequent pain in my chest and it hurts to breathe in i am noticing that most of the time i have these pains are when i wake up and im sleeping on my back shld i be really worried what are the things it cld be and what shld i do? Doctor: Respected user , HiWarm welcome to Healthcaremagic.comI have evaluated your query thoroughly .* Provided other causes are ruled out from EKG and x-ray chest , this seems to be psychosomatic manifestation of underlying stress or anxiety disorder .Hope this will help you .Wishing you fine health ahead .Regards ."
},
{
"id": 111486,
"tgt": "What is the treatment suggested for disc prolapse causing pain in legs and back?",
"src": "Patient: hello doctor, i am having disc prolapse between l5 s1 and i am having pain in my right leg from buttack till down and while sleeping i feel after sometime crams in my foot? so pls advise me what should to reduce this problem, shall i go for traction lumbar? pls advise Doctor: Hello, I had gone through the case and found that it might be sciatica pain. So if pain is severe then take mild painkiller and go for physiotherapy.If pain is mild then take painkiller and do exercise under good supervision.Avoid long time sitting and standing in one position. Avoid high heel and bending forward. Hope my answer will be effective for you.Thanks"
},
{
"id": 205832,
"tgt": "How to deal with abnormal behaviour of a teenager?",
"src": "Patient: My 20 year old daughter does not communicate well with us. She says we think she is an idiot. She was raised in a functional, Christian home, and is a third year college student with no major. when we try to talk to her about college grades and majors, she becomes irrational and has outbursts of unkind remarks toward us, her parents. what have we done wrong, and what can we do to change her attitude. She is very pretty. Doctor: Hello thanks for asking from HCMShe is 20 and is showing complaints like not communicating well with you, she has ideas that you people consider her idiot, she becomes irrational and she show outburst of unkind remarks or anger. Such symptoms should not be ignored. She is in twenties and this can be considered as normal teenage behaviour but her anger outbursts and irrational thinking points more towards some other cause.Underlying conflict and depression may also present with such symptoms. Her ideas that she is an idiot could be a feature of depression. Anger outbursts, irrational thinking etc can occur due to bipolar disorder or some psychotic disorder. From the symptoms you have mentioned it is very difficult to make a proper diagnosis. I would advise you to consult a psychiatrist and get her evaluated to make final diagnosis.Thanks, hope this helps you. Take Care"
},
{
"id": 112551,
"tgt": "Head and back injury, back pain, severe discomfort. Cause of worry?",
"src": "Patient: Hi, I fell a week and a half ago going out my garage. I slipped on the first concrete step and missed the other one and landed on my bottom ( feet going out) and went back and hit my back really hard on the concrete step and then my head. My head is fine, but my back is still very sore and some days I cannot get comfortable. I don t think anything is out of place, but just really sore still. Should I be concerned? Doctor: Hi,From description , it seems you might developed muscular or ligamentous strain over back.Following measures will help with the pain\u2022 Lie on a hard bed.\u2022 Anti -inflammatory drugs like Tablet Motrin 1 tablet as and when required \u2022 Avoid forward bending.\u2022 Avoid strenuous activity & lifting heavy weights.\u2022 Apply diclofenac gel on the affected area. Warm compresses will also help.If it persist even after these measures,I would suggest getting this evaluated by an orthopedician for an accurate diagnosis and appropriate management. You may get the MRI of the area affected under his/her guidance."
},
{
"id": 81665,
"tgt": "Suggest medication for severe chest congestion and sore throat?",
"src": "Patient: I have severe chest congestion and sore throat. When I lie down I get dry caugh with little mucus. I also having issues with my sleep. If I take wikroyl does it effect my sleep? Because it contains caffeine. I also have cetzine. Which one should I take? Doctor: Instead of wikoryl plain if you take wikoryl-L, it will be helpful as it has Levocetirizine.And I would advise you to take steam inhalations three times daily.Also if the cough is pretty bad and disturbing at night then you need to start a codeine syrup at night and sos"
},
{
"id": 136009,
"tgt": "What does sharp pain & burns in back muscles suggest?",
"src": "Patient: I fell and hit my back left side on the arm of a chair. I have constant aching/burning pain like a pulled muscle and sharp pain when I move/stand/try to sit up\u2026and sneezing/coughing causes pretty severe pain. I also feel like something is popping in and out of place about 4-5 inches from my spine. I thought maybe I dislocated something or is it more likely a fracture? Doctor: hiit may be muscle contusion or sprain, take advil tabs after meals daily, apply anti-inflammatory gels locally after hot fomentation.it should settle, if doesnt then consult orthopedic doctor and if needed x ray may be advisedbest wishes"
},
{
"id": 216103,
"tgt": "How to relieve the pain in the burned inner palm and thumb?",
"src": "Patient: i lifted an aluminum pan that was sitting next to a gas burner on high. it was extremely hot and instantly burned my inner palm and thumb. i put ice on it immediate an d now hold a glass full of ice water to ease the extreme pain. the skin on inner thumb is pure white. i took 2 advil and took 2 more 2 hours later. the pain is extreme. what else should i do besides keep holding this glass of ice water?? Doctor: Hello and Welcome to \u2018Ask A Doctor\u2019 service.I have reviewed your query and here is my advice.The dermatological conditions are hard to discuss without seeing them. So I can only give general advice.The area is going to get worse before it gets better, and it is likely to have some skin falling off.Medical intervention is going to be helpful at that point. For pain acutely, Aspirin-like drugs and the ice is about the only things likely to do anything.Hope I have answered your query. Let me know if I can assist you further."
},
{
"id": 72157,
"tgt": "What are the symptoms of potential chest infection?",
"src": "Patient: I got home from work tonight and my chest from my heart to the top of my throat started to feel very uncomfortable, also my neck and back are in pain and i hear popping noises on different parts of my head/neck like a vein or something. should i go to the hospital or is this just the start of an extremly uncomfortable chest cold? Doctor: Hello dearWarm welcome to Healthcaremagic.comI have evaluated your query thoroughly .* The symptoms of potential chest infection include variety of symptoms in co relation with each other or separately as :- Difficulty in breathing which may be moderate to severe- Tightness feeling- Excess perspiration , lethargy- High grade fever- Severe weakness - Others * In your case , this seems respiratory infection with manifestations from underlying anxiety .Hope this clears your doubt .Wishing you fine recovery .Welcome for any further assistance .Regards dear take care ."
},
{
"id": 99587,
"tgt": "What causes chest/stomach pain, coughing orange mucus and headache?",
"src": "Patient: Yes I have had massive pressure like chest pains for about 4 days now. I have been trying to couch things up but it has been a dry cough. The symptoms with this is headaches, fatigue and stomach ache, and hard to take deep breaths. As of this am I was able to cough up some bright orange mucas but the chest pain is still there. Extremely painful to take deep breathes. Doctor: Hi,It can be due to upper respiratory tract infection like pharyngitis or laryngitis or sinusitis, may be acute bronchitis,pleurisy etcYou can go for routine blood and X- ray chest with sputum ( if still coming) examination, it will be helpful to make clear diagnosis.You should take antibiotics ( quinolones or macrolides group) with analgesics and antiinflammatory, antihistamines in combination with mast cell stabilizers to combat dry cough.Don't worry about stomach ache and fatigue, it will subside with the infection.You need to see pulmonologist for further advise.Thanks."
},
{
"id": 154680,
"tgt": "What does transitional cell cancer suggest?",
"src": "Patient: my son is having a uteroscopy for a mass in the ureter. Kidney stones have been ruled out ( he did have blood in the urine and pain in his Right side). can a doctor know if it is cancer or does he have to wait for the biopsy? The doctor mentioned Transitional cell cancer. My son is 32 and 6'2'' Doctor: Hi, dearI have gone through your question. I can understand your concern. If he has mass in ureter with bleeding and no any stones then there is high chance of transitional cell carcinoma. However confirmed diagnosis can be given only after biopsy and histopathological examination. So go for biopsy and plan accordingly. Hope I have answered your question, if you have doubt then I will be happy to answer. Thanks for using health care magic. Wish you a very good health."
},
{
"id": 62786,
"tgt": "Suggest treatment for a lump on the back",
"src": "Patient: my husband has a large lump on his back we have not squeezed on it but it has now become sore like a pimple I looked at it it has a small red spot like a pimple I reminds me of a lump mom had on her back that when squeezed there would be white smell stuff when i feel the lump it feels firm he has had it for four years and it really has mot gotten much bigger, when he showed to the doctor he was told not to worry but if it began to hurt he would send him to somelse the lump is about two inches long what type of doctor should he see Doctor: Hi,Welcome with your query to HCM virtual Clinic.I reviewed all the details of your query,in context of the of the large lump on your husband's back.Based on the facts of your query,You seem to have infected Sebaceous Cyst,which though firm/soar and existing there for 4 yrs without any increase in size,has been confirmed by your doctor as not emergency problem as off now.This ambiguity in the diagnosis by your doctor,indicates confusion in its assessment and to resolve this I would suggest Second Opinion from ER Surgeon,who would confirm its cause and diagnosis by physical examination and by USG if need be and would treat it in the best possible way.So you need to have Second Opinion from Er Surgeon.Hope this reply would help you to plan treatment with your doctors and resolve your worries.Will appreciate writing excellent review comments to help the needy patient visitors like you at HCM.Welcome any further query and information in this regard.Good Day!!Dr.Savaskar M.N.M.S.Genl-CVTS"
},
{
"id": 167022,
"tgt": "How long can Montair-LC be taken without any side effects?",
"src": "Patient: Hi, may I answer your health queries right now ? Please type your query here.. My son who has just turned 4 yrs, has been advised to take Montair - LC Kid for allergies, does this have any side effects ? ALso for how long can it be taken at one time ? Doctor: Hi....it can be taken at a time for 4-6 weeks. It is usually a safe drug. Sometimes hyperactivity, drowsiness, vomiting and stomach discomfort have been reported.Regards - Dr. Sumanth"
},
{
"id": 163608,
"tgt": "What causes cough and runny nose in a child?",
"src": "Patient: Hi, may I answer your health queries right now ? Please type your query here... I have a 19 month old son with runny nose, congestion and coughing for the past five days. He s not eating and drinking very little fluids. The good thing is their s urine in his diapper probably same amount of his intake. He s cough is getting worst and congestion. No fever but weak probably due to congestion and no food and small amount of fluids. Should I be alarmed? Doctor: Hi, welcome to HCM. This is most probably lower respiratory tract infection. You need to get the child examined by a doctor as soon as possible. Take care."
},
{
"id": 8960,
"tgt": "How can this black spot on my cheek be removed without any surgery ?",
"src": "Patient: hi doctor ,i have a a permanent black spot on my cheek n i am 18yrs old girl n it feel very imbrassing so i wanna get rid of it so wil u suggest me some method except surgeory Doctor: Blackhead are a result of open skin pores on face. Its a common problem, but sadly pores cannot be closed. For blackhead removal, best way is to remove them mechanically in a salon/beauty parlour. Then keep skin clean, by regular washing and scrubbing. Also avoiding steam during facial/clean up is helpful, as steam futher enlarges the pores. Keeping body healthy by good diet, more fluids, keeping fit by exercise and minimizing stress is good for whole body including skin & hair."
},
{
"id": 86645,
"tgt": "Suggest treatment for lower abdominal pain",
"src": "Patient: Hi my friend has contacted the nhs he has lower abdominal pain below the naval if he presses on it the pain eases he has a strong pain threshold and he says the pain is excrutiating he ate today out but his friend who also ate is fine he has also been sick twice any suggestions if he raises his legs the pain eases Doctor: Hi.Thanks for your query. If the pain is easing on local pressure or raises the legs and increases on eating indicates that the pain below the naval may be due to :Lymph node mass or diverticulitis.It is also possible that it may be of he sensory nervous system, the nerves in the back being pressed by the compressing inter-vertebral discs.I would advise your friend to do the following:First of all get a clinical evaluation and physical examination done by a general Surgeon as the history is not going with any of the settled diagnostic character, in fact is contradictoryIt is possible that he is malignaring , telling the lies for whatever the reasons may be. TO get the CT scan of the abdomen, colonoscopy and Endoscopy and MRI of the spine to confirm or rule out any probable diagnosis. Symptomatic treatment may be required."
},
{
"id": 49958,
"tgt": "Kidneys- Parenchymal echogenicity, pelvocalyceal fullness, no lithiasis.. What does this mean?",
"src": "Patient: both kidney are normal in size and parenchymal echogenicity . there is pelvocalyceal fullness,both kidneys . no lithiasis . no focal solid or cystic mass ... ureters-not visualized urinary bladder floating fine internal echoes.. pre void urine - 173 cc . post void urine 11 cc ... wanna know whats the really meaning of this im confused Doctor: Hi,Thanks for writing in to us.I will explain your ultrasound findings in simple terms:1. Both kidney are normal in size and parenchymal echogenicityBoth kidneys are normal in size and structural appearance as seen on ultrasound.2. There is pelvocalyceal fullness,both kidneysThe kidney - ureter junction and calyces in the kidney are well seen.3. No lithiasisNo kidney stones.4. No focal solid or cystic massNo abnormal areas seen in kidneys5. Ureters-not visualizedThis is normal as ureters are visualised on ultrasound scan only when obstructed.6. Floating fine internal echoesThis may indicate mild bladder infection.7. Pre void urine - 173 cc . post void urine 11 ccNo obstruction to flow of urine and no retention also.From the above it looks like a mild bladder infection which can be completely treated by drinking lots of water and with a course of antibioticsHope this helps"
},
{
"id": 197931,
"tgt": "What causes erection problem while taking hair loss treatment?",
"src": "Patient: I am 26 Years old taking finax 1 mg & Minoxidal Topical Solution 5 % since last 14 months for hair loss treatment. But from last 2 months I am facing problem in achieving an erection. Whats the cause. ??? How to recover back to from this problem. ??? How much time will it take ??? Doctor: HelloThanks for query .You have been taking Finax (Finasteride) and Minoxidal for hair loss and facing problem of Erectile Dysfunction since last 2 months .Fineatsride is drug that belongs to a class of drug called 5 alpha reductase inhibitors which prevents conversion of Testosterone in to Dihydro Testosterone which is the main hormone involved in sexual functions like libido ,erection and Ejaculation Finasteride taken for a long period of time is known to have side effect of Erectile Dysfunction .loss of libido and reduced ejaculation .There are plenty of research papers on this subject and it has been proved that in majority of patients Erectile functions will resume to normal within 3-6 months after discontinuation of drug...Dr.Patil."
},
{
"id": 72148,
"tgt": "What causes right sided chest pain with vomiting?",
"src": "Patient: I have had pain in my right side of chest, above breast but pain goes down right breast and into my right side under arm. I have had lap banding (vbg) about 6 years ago and have kept the weight off. I have trouble keeping food down and have been vomiting it up to relieve some of the pain. I also have a large hernia (from the surgery), which took over 200 stitches to close. I am just wondering if I have caused some problem to my chest or throat from all the throwing up I have done. I do this at least once a day if not more. Doctor: Thanks for your question on Healthcare Magic.I can understand your concern. Since you had banding surgery possibility of spasm of lower esophageal sphincter is more likely. This sphincter plays very important role in swallowing. If this sphincter doesn't work properly, chest pain, vomiting, pain relieved by vomiting is seen.So better to consult gastroenterologist and get done upper GI (gastrointestinal) scopy with manometry (pressure monitoring).Hope I have solved your query. I will be happy to help you further. Wish you good health. Thanks."
},
{
"id": 171371,
"tgt": "Suggest treatment for west syndrome in a child",
"src": "Patient: Dear all, my baby is suffering from West syndrome which was confirmed by EEG(hypsarrythmia) and visual symptoms. He is now 7 months old. It all started when he was 6 months and we took him for immunisation. After the Vaccine he had fever and was crying incessantly. though the fever was gone in 1-2 days but these infantile seizures started. He is taking Sabril 500mg ( Vigabatrin) two tablets per day, valparin and Blong (B vitamin). since last 3-4 weeks but not much relief as yet. Your suggestions and feedback awaited. Rupesh Doctor: Hi, you are taking correct drugs for west syndrome. You have to wait for atleast 3 months before going for another drug. There are very few drugs for treatment of west syndrome, sometimes steroids are also tried. Keep patience, everything will be alright."
},
{
"id": 14920,
"tgt": "What is the treatment for rash under arm which is burning and itching?",
"src": "Patient: I have a rash under my arm, it is like red welts, they burn and itch, i have had them for 3 weeks now and they are spreading. i have tried gold bond powder and a itch medicine that our dr. gave me for our son who had several bad spider bites. my arm pit kind of smells like yeast. Doctor: hello,Your symptoms point towards these diagnosis :1. tinea corporis characterized by itchy, red coin shaped large lesion over your body.2. candidiasis (intertrigo) characterized by itchy red macerated lesion over body folds with some pus filled lesion also present.3. erythrasma characterised by less itchy, red- brown smooth lesion over your body folds.You should start with oral antifungal like fluconazole , antihistamine (levocetrizine, loratidine), and a cream containing antifungal ( azole like clotrimazole) and steroid (like betamethasone) to apply over the affected area. This treatment cover tinea, intertrigo and erythrasma.However, I would suggest you to consult a dermatologist to start your treatmentThanksDr Jyoti Gupta"
},
{
"id": 20492,
"tgt": "Why am I hearing my heart pounding very heard?",
"src": "Patient: Hi I'm a 19 year old girl and I have recently just been hearing my heart pound very hard. I'm worried thinking it might be because I don't drink water at all. I'm 5'7\" and weigh 115 pounds. I normally eat fast food and sometimes even skip meals throughout the days. So at times I don't eat anything at all. I also drink Pepsi every single day about 2-3 times a day. Can you please tell me if this heart thumping is normal for me? Doctor: The condition and the state that you're describing a sign of anxiety neurosis if you pay attention to your heartbeat or your breathing you can hear the is a vital signs sometimes if the heartbeat is faster and you are nervous and you are paying attention to your heart beat you can hear heart pounding this can be can we do worsened by Pepsi because it has a caffeine taking a girl Pepsi or coffee or tea and excessive amounts more than 2 to 3 cups a day can cause a fast heart rate and if you are and and nervous you can hear the founding so the best thing inThe best thing is to do regular exercise keep yourself hydrated and avoid excessive caffeine in your life I hope this helps you think"
},
{
"id": 201417,
"tgt": "Does Lexapro leads to lack of sex drive?",
"src": "Patient: Hi, Im taking lexapro,concerta and I just started abilify.. I do not want to take lexapro anymore,because of the fact I have no sex drive. can I just take concerta and abilify? concerta hepls me the most with focus ,energy and mood, not sur about abilify yet. just started it. I want to give it time to work. Doctor: Hi dear . I had gone through your query .Lexpro is antidepressant and it helps in mood features .Yes lexpro has some sexual indifference.Mostly erectile dysfunction is common than decrease in sex drive.Decrease sex drive can be part of depression and need to take proper consideration between these two.There are some medicines that improve sex drive if it is caused by lexpro or related medicines.Dose reduction can also help.Better to consult with your doctor and get advise from him .Detail evaluation of psychological stress and thought process will help for better treatment .I hope I have answered your concern .Happy to help you further .Thank you ."
},
{
"id": 14143,
"tgt": "What causes red, blotchy, rash type patch near to breast?",
"src": "Patient: On Sunday i noticed a red blotchy rash type patch on the outside of my right breast. It hasnt dsappeared yet. Should i seek medical asvice? No lumps. I have had gallactorea bilaterally for 30 plus years with raised prolactin due to a micro adenoma on the pituitary gland. Have regular mammograms and bannual MRI scans . What do you advise? Thank you Doctor: Hello,Red patch on breast may be either allergic rash or fungal. Kindly consult the dermatologist for the perfect diagnosis and proper treatment.Hope I have answered your query. Let me know if I can assist you further.Regards,Dr. Ilyas Patel"
},
{
"id": 180350,
"tgt": "What is the time duration for healing after an operculectomy?",
"src": "Patient: Hey! I\u2019ve had operculectomy performed last week but now even after a week,i dont feel much better. I mean my right lower side of mouth hurts so much and sometimes sudden pain arises in lower tooth or at the back soft tissue of mouth. Does it mean that operculectomy wasn\u2019t performed well enough? I\u2019m unable to figure out why isn\u2019t my tissue healing already because my dentist left it open to heal it on its own. Doctor: Hi, It takes some time to heal. Depends on each person's body, how it reacts to surgery. Wait for a week. Take the antibiotic and anti-inflammatory dosage as prescribed by your dentist and also do salt water gargle. Have lots of water. Hope I have answered your query. Let me know if I can assist you further. Regards, Dr. Rohit S Menon, Dentist"
},
{
"id": 127689,
"tgt": "Suggest treatment for severe pain in the extremities",
"src": "Patient: 42 year old caucasian male. PMH kidney stones, low vitamin D (mildly low). On allopurinol, vitamin D. shooting pain to all extremities, feels like electrical shocks. Off and on pain difficult to sleep and to work but working full time. No exposure to heavy metals. endocrinology tests are otherwise normal. No diabetes. No alcohol use. No obvious reason for polyneuropathy Doctor: Hello and Welcome to \u2018Ask A Doctor\u2019 service.I have reviewed your query and here is my advice.For which disease you are on Allopurinol? Did you ever ask your physician for it?As per my opinion you have gouty arthritis, because of which you are having pain in distal joints. And for gout only you are on Allopurinol. Although, it reduces uric acid level in blood, for pain relief you have to take Ibuprofen / Mefenemic acid / Diclofenac (NSAIDS) or Colchicin according to physician\u2019s advice.I hope I have answered your query. Let me know if you have any further questions. Regards, Dr. Hiren Hirpara"
},
{
"id": 31492,
"tgt": "What can cause prolonged fever?",
"src": "Patient: i am 30 years having fever since 4 days i consult to a doctor and having one antiboitic and cetamol... but the fever is still there and today i went for blood test ,,, the test was WBC count which was low than normal... is it typhoid as malaria test was negative Doctor: Hi thanks for asking question.Here your CBC count reveal low WBC count , so yes it might be typhoid if present with abdominal pain.For confirming it blood culture or widal test can be done.But if typhoid comes negative then search for other causes has to be done according to complete history and physical examination.If in severe infection bone marrow is involved then it get suppressed and present with low wbc cpunt.Certain drug can also lead to low WBC count.TB, HIV, Aplastic anemia, radio or chemotherapy also lead to low wbc .So detailed work up necessary.I hope you can understand my concern."
},
{
"id": 93537,
"tgt": "Have stomach pain. Unable to sleep. Took Vicodin. Treatment?",
"src": "Patient: for the past couple years I have been getting stomach pains so horrible it keeps me from working sleeping anything. it comes in waves the pain is right above and below my belly button. it gets so bad that last night I took two vicodin that I had left over from having my gallbladder removed (thinking that the pain would stop after having it removed) and it still didn't even help one bit. it comes on suddenly and gets worse I feel every time it comes back. the docs cant seem to figure out what the fuck it is but its getting to the point where its lasting longer and I cant keep going on like this and missing work and sleep, I am only 23 and I need this to be figured out...... Doctor: Hello,You're having frequent episodes of severe abdominal pain.You have undergone cholecystectomy in the past.The pain may be due to adhesions in the abdomen following surgery.This is a common occurrence in many postsurgical patients.If this is the case than medication can do little help in controlling or curing it.Due to Gallbladder involvement in the past it may any pathology to the level which may be causing you trouble.Consult a gastroenterologist."
},
{
"id": 132721,
"tgt": "How to treat pain in the hip and knee joint?",
"src": "Patient: Hi, I had a vascular problem with my arm. The surgeon removed a veian from my leg and did a branchail to ulner bypass to try to savethe arm, ehich later had to be amputated. Now I have quite a bit of pain in the hip and knee joint of that leg, especially when walking. Is there anything that I can do to alleviate the pain and help me with walking? Doctor: Hi Hope this message finds you in good health.I have gone through your complaints and understand your concern.The pain you are having right now may be due to straining of some sort of soft tissues,nerves.Avoid strenous work,take adequate rest.Physiotherapy,analgesics,anti-inflammatory drugs should help.Get an Xray to find out whats wrong. Nothing to worry about.I hope your question has been answered.If you have any follow-up queries,feel free to consult me anytime.Thanks,Take care,God bless."
},
{
"id": 154913,
"tgt": "Are the rashes on chest, stomach after chemotherapy due to iron deficiency?",
"src": "Patient: My sister has been on chemo for nearly 20 months... pill form for 18 months and IV every two weeks for the last two months. She recently finished radiation as she has an inoperable brain tumor that is cancer. She broke out into a rash that is non itchy, non raised, and almost looks like flat blood blisters. They cleared up some and came back this morning, only now much bigger on her chest and all around her stomach, back and shoulder. Could this be more than just an iron deficiency? Doctor: Hi, dearI have gone through your question. I can understand your concern. Rash on chest and other part is not due to iron deficiency. Iron deficiency never cause rashes. It may be due to radiotherapy or coagulation problems. You should go for platelet count and complete coagulation profile test. Then you should take treatment accordingly. Hope I have answered your question, if you have doubt then I will be happy to answer. Thanks for using health care magic. Wish you a very good health."
},
{
"id": 51914,
"tgt": "SEX DURING JAUNDICE",
"src": "Patient: IS IT SAFE TO DO SEX WITH A LADY SUFFERING FROM JAUNDICE OR HAVING KIDNEY PROBLEMS. AGE -24 GENDER-FEMALE Doctor: Hi,Welcome to HCM.It depend on what is the cause of Jaundice. If it is infective hepatitis then there is a possibility that you may get the infection. In other cases if you have minor abrasion and her blood comes in contact with your wound then again the risk is more.Risk involved in Kidney problem also depends on what kind of kidney problem she has. If it is pyelonephritis or urinary tract infection you run a risk of getting infected."
},
{
"id": 41385,
"tgt": "Does chemotherapy cause infertility?",
"src": "Patient: Sir, My wife had undergone chemotherapy four years before (when she was 26) she got married at 29. now she is 30. We have been trying to get preganant for last one year, but no result. her periods are normal. the only abormal symptom is that her body use to get hot (for seconds) quite often and come b ack to normal temeprature. Can she become pregnant. is it due to chemotherapy. I think she did chemotherapy for six months and did not had periods for i year. Doctor: Thanks for posting your query to HCM.To get pregnancy you require:1 Normal semen analysis report .2. normal uterus with patent tubes tested by Histosalpingography .3.normal hormonal status of your wife :FSH,LH,TSH .I want to know the cause for which your wife had taken chemotherapy for long time because genital tuberculosis is a common cause of infertility and require long chemotheray .So you should check for above parameter and then consult gynecologist .best wishes for both of you .Feel free to communicate if any query.regardsDr.Manish Purohit"
},
{
"id": 10433,
"tgt": "Suggest remedy for hair fall",
"src": "Patient: I have a hairfall problem since last two years.. And I workout in the gym regularly..from the past few months my hairfall reduced almost to nil., recently I started taking tribulus supplement.. And my hairfall started again.. Is that due to tribulus? Does tribulus causes hairloss? Doctor: Hello and Welcome to \u2018Ask A Doctor\u2019 service. I have reviewed your query and here is my advice. As per your case history of hair fall, my treatment advice is - 1. Use a good herbal hair oil and shampoo for routine use. 2. Take good nutritious diet full of green leafy vegetables and milk. 3. Take an iron supplement and vitamin B12 supplement. 4. Other treatment options are oral multivitamins and mesotherapy done by a dermato-surgeon. Hope I have answered your query. Let me know if I can assist you further."
},
{
"id": 84250,
"tgt": "Are novaclox lb and otek ac right medication for ear pain?",
"src": "Patient: There has been intermittent pain deep in my left Ear. I have started OTEK AC Ear Drops yesterday (once at night before sleep) and also started Novaclox LB yesterday. It is right medication for this problem ? Also, what should be the dose of Novaclox LB. Is it thrice a day ? Doctor: Hi,Yes, these medications are commonly prescribed to treat ear infection. Novaclox-LB is a combination of two antibiotics (amoxicillin and dicloxacillin) is usually taken three times daily. Yes, both Novaclox-LB and Oteck AC ear drops are rightly prescribed to treat ear infection. Continue taking them as prescribed.Hope I have answered your question. Let me know if I can assist you further. Regards, Dr. Mohammed Taher Ali, General & Family Physician"
},
{
"id": 183370,
"tgt": "What causes bleeding gums and pain in roof of the mouth?",
"src": "Patient: Hello my name is joseph, lately my gums have been bleeding and hurting when I brush them. And they hurt when I eat and the food touchs the roof of my mouth. What is causing the roof of my mouth to hurt. And also I have red dots above my teeth on my top jaw whats that? Doctor: Thanks for your query, I have gone through your query.The pain and bleeding in the gums could be because of the gum infection secondary to deposits or pulp polyp or any bleeding disorders like idiopathic thrombocytopenic purpura. Consult a oral physician and get these things ruled out. Get a complete blood investigation along with INR to rule out the bleeding disorders.If there is any gum infection then get your teeth cleaned and use topical gum astringents like stolin gum paint.if pulp polyp is there then get it root canal treated.If bleeding disorder is there then consult a general physician.I hope my answer will help you, take care."
},
{
"id": 215974,
"tgt": "Suggest treatment for cramping and pressure on my left lower side",
"src": "Patient: hi, im Jocelan ... I guess I am experiencing cramping / pressure on my left lower side. i first noticed them about three weeks ago a week after unprotected sex with boyfriend when it started 3 weeks ago it hasn t stopped or went away i notice it everyday. i thought i was pregnant because i was 4 days late but i don t understand what my body is trying to tell me Doctor: Hello and Welcome to \u2018Ask A Doctor\u2019 service. I have reviewed your query and here is my advice. It could be due to genital infection caused by unprotected intercourse. Do not panic. I would suggest you to take gynecologist opinion and get few tests done to evaluate the cause. I hope I have answered your question. If you have further queries always feel free to ask me."
},
{
"id": 3184,
"tgt": "Is a second pregnancy possible despite PCOS?",
"src": "Patient: Hi, I had taken OCPs for three months and then stopped a month ago. We were actively trying to conceive for a second one. my LMP was 7/7/2014 i tested for Pregnancy through UPT kit on 10/8/2014 as my periods had not come. The test was negative so delay could be due to what reasons, may be PCODs. As i was diagnosed with PCOD before my first delivery but i conceived normally without any medications. So negative UPT means for sure that I am not pregnant. Rgd VS Doctor: Hello, after taking oral contraceptives it takes time For the return of fertility depending on the composition of the OCP used. Mostly women concieve after 6-12months. Have patience you have left ocp just a month ago. Your body needs time to adjust its proper hormone levels for fertility. Well controlled hypothyroidism and treated PCOD do not cause any problem in conceiving. Moreover first normal delivery without medication is a very good sign for next successful pregancy. If you are overweight for your age and height then in the meanwhile you can join a gym for 6months to reduce weight."
},
{
"id": 119497,
"tgt": "What causes swelling in the ankle?",
"src": "Patient: The bottom part of my ankle is swollen and when I set for more then 5 minutes I can hardly walk on it,also if I touch it it feels as I have a big bruise on my ankle this has been going on for 4 months I ve been put in a walking boot for 3months so far its been wrapped and iced almost everyday now it is starting to do the same on my other ankle its becoming very painful to walk r do anything Doctor: Hello,Swollen ankles are commonly due to obesity, water retention, venous insufficiency, sprains/strains, ankle injury, lymphedema, heart, kidney or liver issue, side effects of certain medicines, RA or OA, etc. You are suggested to cut on salt intake, reduce weight if obese, be careful about what medicines you take, if diabetic maintain your blood glucose level at all cost.You are suggested to get clinically examined by a GP. Get blood tests (anti - CCP, ESR, CRP) and Imaging tests done to detect the pathology causing you discomfort. Stay physically active to improve circulation and tones muscles in your legs. Stretching your calf and thigh muscles can also help improve blood flow. Take the help of a physical therapist. Stay well hydrated. Your treatment will depend on your clinical condition and reports of the the investigations obtained.Take care. Hope I have answered your question. Let me know if I can assist you further. Regards, Dr. Nupur K., General & Family Physician"
},
{
"id": 134003,
"tgt": "What causes severe fatigue, left knee pain and lump on back of neck?",
"src": "Patient: Hi, I m 37 and a female...I have my own cleaning business. For the past couple weeks my arms and legs feel really tired/heavy, and my left knee hurts. Every night I ve been in bed before 10:00 pm, which is not like me at all. Also last month I found a lump on the back of my neck. My doctor ran white blood cell test and to see if I d been bit by a tick. All came back normal or negative. Not sure what the problem could be. Should I go see my doctor? Thank you. Doctor: Hi,thank you for providing the brief history of you.As you mentioned there is pain in the neck region, back region and left knee a thorough neuro-muscular assessment is guided for further assistance. As due to heavy work loads, over a period of time, there is a strain in the muscles of the neck and back region. Also, the knee is a weight bearing joint and may have some wear and tear in the joint and can lead to stress on the ligaments of the knee joint.Post assessment of the neck region, back region and knee, a provisional diagnosis is made. Based on which the next treatment plan can be designed.Based on the symptoms, i will advice you to take hot water fermentation on the affected region . Also, undergoing a physical therapy , the therapist will help with Therapeutic ultrasound therapy and TENS therapy for inflammation and pain. Post which the exercises will be planned to make the muscles stronger in the back region, neck region and knee. Also, by performing the exercises the fatigue issue of your will be solved. In my personal practice i have seen cases with same issues and have found 99% recovery in them. I wish the same from you.RegardsJay Indravadan Patel"
},
{
"id": 176630,
"tgt": "What causes white stuff deposition in ears?",
"src": "Patient: I just noticed that my 7 week old daughter has white stuff in her ear. It is beyond what I can reach with a damp cloth, but visible when looking in ear. She doesn t seem uncomfortable laying down, has no fever, and is mostly breatsfed. Any thoughts? My first thought was maybe some of that white gunk from when she was born? Maybe soap from baths? Doctor: Hi...the possibilities could be the soap or powder or it could even be a fungal infection of the external ear. I suggest you consult your paediatrician and he will the best person to directly visualize and decide what it is and if any treatment is required.Regards - Dr. Sumanth"
},
{
"id": 10566,
"tgt": "Why is there a poor growth of the mustache?",
"src": "Patient: sir i have asked with one skin specailist about not having mustache but i have beared..he said if pubic is hair is correct then dont worry it may be gene..yes my father too have thin hair abut he has musatche..so can i go into nay treatement to get mustache and the skin specailsit told me that some may get mustavhe after the age of 29 is it true sir? Doctor: hi there.1. moustache is related to male hormone, testosterone.2. usually by adolescence , pattern hair should start growing due to influence of hormones.3.it's your body tendncy for sparse moustache.4. you can try using tugain 2% gel, once daily bed time for 3 consecutive months along with adgain capsules daily one after breakfast.5. plan for hair transplant on moustache area in the future, it is safe and promising procedure."
},
{
"id": 209330,
"tgt": "Suggest remedy for suicidal tendencies",
"src": "Patient: Hello Doctor,I am having too much expectation from my lover. And now it is turned after long time to very much possiveness and now I could find myself more on doubting her unnecessarily and fighting with her. This has created a lot of problem in us and forced me to attempt for suicide, I want to get out of this problem of being possive on anybody. Please let me know a reply for my problem. Doctor: DearWe understand your concernsI went through your details. I suggest you not to worry much. I am very glad that you diagnosed your problem as possessiveness. i shall go a step further as obsessive behavior. Obsessive behavior makes you to think about whatever you want to forget. If you try to forget something, due to obsessive behavior, that thinking returns again and again. The reason behind obsessive thinking is possessiveness. As now you have understood, the problem shall come down in gravity. Time knows the procedure to heel you and it shall take the course. In the mean time, you should concentrate on your activities such as education, career and games to keep you engaged. If you are able to keep you engaged for a week or so, your obsessive behavior problem is solved, for sure.If you require more of my help in this aspect, Please post a direct question to me in this website. Make sure that you include every minute details possible. I shall prescribe some psychotherapy techniques which should help you cure your condition further.Hope this answers your query. Available for further clarifications.Good luck."
},
{
"id": 14862,
"tgt": "Why had body rashes after changing medication form Diltiazem ER to Verapamil ER?",
"src": "Patient: I changed from Diltiazem ER 360mg to Verapamil ER 360mg, after 7 days I broke out in a rash covering my whole body. Now I'm back on Diltiazem. They are both channel blockers so I want to know why the verapamil did that, it is a tier 1 drug for me with no co-pay, the diltiazem is tier 3 & costly. Thank you Doctor: Hi,I can understand your concern for skin rashes with itchiness. There can be multiple possibilities of the red rashes and it requires a close physical examination to diagnose the exact entity of the problem.In my view correlating with the facts you have provided it appears to be either allergic dermatitis or simple hives of the area due to multiple causes.I suggest you to meet a dermatologist to for proper therapy. In between you should avoid contact of dust to the area properly. If I were your dermatologist then presumptively would have preferred to apply fluticasone with mupirocin cream. You can take an antihistamine such as loratadine for twice a day if itching is present.Do not try to pinch, pop or squeeze the area as it may cause secondary infection of the area.All the best"
},
{
"id": 115663,
"tgt": "What causes frequent blacking out with a history of anemia?",
"src": "Patient: I am 13 and have had anemia since I was 4 years old and lately if I stand up I just collapse to the ground because my vision blacks out and I get so lightheaded and can't catch my breath I lay there for a few seconds then get up and feel in shock. Why does this happen? Doctor: Hi, dearI have gone through your question. I can understand your concern. weaknesses, blackout, difficulties in breathing are common in anemia. Because your oxygen carring capacity os reduced. You should go for complete blood count, peripheral smear examination and anemia profile. Find out the cause of anemia and take treatment accordingly. As your hb level improve you will get releif from your symptoms. Hope I have answered your question, if you have doubt then I will be happy to answer. Thanks for using health care magic. Wish you a very good health. Don't forget to click thank you."
},
{
"id": 200399,
"tgt": "Suggestion of remedy for a normal prostate",
"src": "Patient: Hi I m Michael Cowan I had a bad prostate infection in 2011 and got bad inflamation.since being treated I used smokless tobacco and it got worse it feels hard and I cannot feel sexual pleasure but can still ejaculate but it is not enjoyable have not had normal prostate since did I dammage my prostate Doctor: Thanks for asking in healthcaremagic forum Your pleasure and prostate need not be related. Prostate infection takes time to heal. Please visit an urologist for your problem and further management. All the best."
},
{
"id": 155600,
"tgt": "Suggest treatment for breast cancer",
"src": "Patient: I am undergoing testing for breast cancer. On 01-27-14 the doctor removed my nipple, areola and some tissue under the skin. The results showed cancer. Another surgery on 03-10-14 resulted in removal of lymph nodes and a lumpectomy and removal of additional tissue. The surgeon thinks it is early stage 2. I saw an oncologist today, and we are planning a Pet-scan. She said I have lobular breast cancer. I also discovered a small lump on my left breast, and 2 different colleagues concurred that there was a lump. Depending on the outcome, maybe mastectomies on bone breasts followed by chemotherapy. A lot to take in one sitting. Doctor: Thanks for your question on HCM. Lobular carcinoma of breast is notorious to spread in opposite breast. So better to get done biopsy from left breast lump too, to rule out malignancy. And yes PET scan is needed to rule out distal metastases. As bones are most common site to metastases. In early stage 2 breast cancer treatment of choice is enblock resection of breadt tissue with resection of involved lymphnodes.Post operative chemotherapy and radiotherapy is necessary if lymphnode s are positive for malignant spread.Also get done ER (Estrogen Receptor) and PR (Progesterone Receptor) study to guide hormone therapy (tamoxifen). So discuss all these with oncologist."
},
{
"id": 43526,
"tgt": "Trying to conceive. Had PCOD. Do patients who have suffered from endometrial tuberculosis face problems while conceiving through IVF?",
"src": "Patient: Hello Doctor. I am a 31 year old female trying to conceive since the last 1 year. After starting my infertility treatment, my doctor suggested IUI. We did two 2 IUI but they did not turn positive. Since I had PCOD, my doctor advised for laproscopic ovarian drilling and hysteroscopy. The endometrial biopsy post hysteroscopy revealed endometrial tuberculosis. I underwent anti tubercular treatment for 6 months. Have now been trying to conceive with IVF.HCG number post 1st IVF was 77 which later dropped.Awaiting result for 2nd IVF cycle.My question to you is that do patients who have suffered from endometrial tuberculosis face problems while conceiving through IVF? Doctor: Hi, Endometrial tuberculosis affects the uterine lining essential for implantation and as such, spontaneous conception is affected. This is why IUI also had perhaps failed. Added to this, you suffer from PCOD, a deficient ovulation condition. Tuberculosis usually affects the fallopian tubes first, so tubal blocks should also be assessed. Endometrial tuberculosis may lead to endometrial or uterine adhesions and in such a case, IVF also may not give good results. A hysteroscopy is necessary to diagnose this. Please wait for your next cycle and follow your doctor's instructions. Take care."
},
{
"id": 98163,
"tgt": "Is acupuncture therapy helpful in increasing height ?",
"src": "Patient: i am 23 years old.is there any possibilty of hieght increment by acupuncture therapy i am 23 years old.is this possible to grow height by acupuncture therapy Doctor: hi and welcome to healthcare magic!! well i am sorry to disappoint you but the age at which height increment stops is 18-20 years after which all the secondary centers in the bones are fused so to answer your question no acupuncture or any other method cannot increase your height at the age of 23. take care."
},
{
"id": 210779,
"tgt": "What could cause a feeling of having cancer?",
"src": "Patient: I have normal cholesterol tri-gly LDL/HDL, insulin/BP, don't smoke, drink, I exercise and eat a v. healthy diet but for last 18 mo my crp went from 0.5-1.0 to between 4 and 6.. Dr can't figure it out-all tests normal;ANA and rheumatoid factor and SR normal. I'm worried some occult cancer is lurking!Thanks for any help. Doctor: Hello,Thanks for choosing health care magic for posting your query.I have gone through your question in detail and I can understand what you are going through.CRP is a very non-specific predictor and the levels may increase even with mild fever or stress. There is nothing to worry. The fact its only the CRP that has come high suggests that it may not be a cancer.Hope I am able to answer your concerns.If you have any further query, I would be glad to help you.In future if you wish to contact me directly, you can use the below mentioned link:bit.ly/dr-srikanth-reddy\u00a0\u00a0\u00a0\u00a0\u00a0\u00a0\u00a0\u00a0\u00a0\u00a0\u00a0\u00a0\u00a0\u00a0\u00a0\u00a0\u00a0\u00a0\u00a0\u00a0\u00a0\u00a0\u00a0\u00a0\u00a0\u00a0\u00a0\u00a0\u00a0\u00a0\u00a0\u00a0\u00a0\u00a0\u00a0\u00a0\u00a0\u00a0\u00a0\u00a0"
},
{
"id": 169144,
"tgt": "Can formula feeding cause dark green bowel movement in babies?",
"src": "Patient: my 6 month old baby has been excusivly breast feed for the past 6 month but now he is formula feed during the day and breast feed at night, but now he is pooping dark green solid poop and some times a bit lighter dark green but wetery poop what is going on? Doctor: hello. passage of greenish stools is due to incomplete digestion of milk sugars. vegetable oil in formula milk can also give rise to loose stools. if baby is gaining weight properly and is comfortable paying urine adequately, be relaxed."
},
{
"id": 166268,
"tgt": "What is the normal hemoglobin level of a 18 month baby?",
"src": "Patient: Hi, may I answer your health queries right now ? Please type your query here... hi, my kid is 18months old and is not well past few days. he has cough,cold and also lack of Hemoglobin(8.7). what should be normal count if Hemoglobin for a kid of 18months.. Doctor: the Normal haemoglobin in 18 month old child should be around 12 milligram per litre. the hemoglobin level is 8.7 in your child which is less for his age. to know the cause of low hemoglobin you should to some blood test like complete blood count with peripheral smear, iron studies, hemoglobin electrophoresis. An examination by doctor is also necessary so that we may not miss any important finding. Review with report. Take care."
},
{
"id": 144648,
"tgt": "Suggest treatment for seizure",
"src": "Patient: yes hello doctor I had sex recently of 3 nights ago going onto the third day of being away from this person I felt this very big weight on my back up into my left should to feel I have a huge weight of feeling something crawling into my skin waking up from a neurological seizure unsure of what the cause was jus need as many answers as possible sharp pains located down inside of left anus along with ovary pain referring to the left ovary concerning my body will go into some type numbing shock almost like needing out on dopamine I always feel my tattoo which to say in cursive, Never A Failure Always a Lesson I a m 21 year sols as to all I want to do is be a doctor or a teacher who can teach to speak about sexual despises knowing I have brand new disease I am not aware of yet. please doctor feed me some answers doctors are the best all I can trust to know thank you scincerly your networking patient sara Doctor: The description you giving about your symptom, does not seem like a seizure. If you had a seizure, its need to investigated and if needed, should be treated.Doing sex is not related to getting seizure."
},
{
"id": 216886,
"tgt": "Suggest treatment for pain after disc prolapse surgery",
"src": "Patient: i got disc prollapse surgery l4 l5 in 2002 , Recently i am getting pain at l4 l5. based on recent MRI reports doctors told me some swelling is appearing at l4 l5 and he prescribed me tablets. tolflex 150 and doloneuron100 for 10 days but no result , still getting pain .. please sugest me i am 40years old Doctor: Hi there Thanks for your question at HCMIt is not uncommon for disc problem to recur again. It is advisable for to try 3 to 4 months of non-operative treatment in the form of physiotherapy, analgesic, injections when indicated , activity modification before advising surgery for pain that is manegeable. IF you have tried these and you are able to manage with your pain, there is no need of urgent surgery. On the other hand surgery is warranted if 1.Your daily activates of daily living are affected due to the pain. 2. Your pain is unresponsive to the non-operative measures over 3 to 4 months. 3. You have new onset weakness or sensory loss. 4. Severe stenosis or pressure on your nerves seen on your MRI which correlates with your pain pattern. 5. Associated with back pain which is getting worse over time.If you are working on a computer, there are several posture related adjustments, which can help you to fasten your recovery and help you maintain your lumbar spine strength over time .Ergonomics is the field of science which deals with such adjustments. I would suggest you to go through websites which deal with ergonomics for working on a computer and adopt those postures. These minor modifications can help you maintain good lumbar spine postures.Hope this helps All the best Regards,Dr.SBK"
},
{
"id": 52435,
"tgt": "What causes stomach pain and bloating with diarrhea after a hysterectomy?",
"src": "Patient: Hi my name is Jill I had a hysterectomy done on August 3 2009. I had every thing taken out except my gallbladder and my appendex. I'm having the same symptome that I'm having are the same symptome before my hysterectomy gas bloating and diarrhea the really bad pains in my stomach is in the upper part of my stomach around where my gallbladder before my surgery I had every test done out there all of test came back negative for my gallbladder. Any feed back would be great. I'm also wondering because all the problem that I was having with my period and they thought it might be my gallbladder. Since having a hysterectomy I'm really still wondering if it still might be my gallbladder. any feed back would be greatly apperciated. Thanks Jill Closs Doctor: Hello, Probably you may be suffering from irritable bowel syndrome or food Intolerance or malabsorption syndromes or inflammatory bowel problems or postcholecystectomy syndrome. Use tablet Pantoprazole before breakfast for one week. Avoid spicy food and alcohol avoid smoking and caffeine drinks. Avoid lactose and gluten-containing food. Use tablet Normaxin twice a day for five days. If symptoms not improved please consult your doctor he will examine and treat you accordingly. Hope I have answered your query. Let me know if I can assist you further. Take care Regards, Dr Penchila Prasad Kandikattu , Internal Medicine Specialist"
},
{
"id": 106833,
"tgt": "What does backache on the right side indicate?",
"src": "Patient: I have been having injections for back pain which has been working, within the last couple of day I am having right side back pain and a egg sized lump in the back right above the hip that is making it hard for me to walk and my leg is aching terrible Doctor: Hello,This can be due to impinged nerve in the lumbar spine. Get MRI of the affected area done.Take analgesics and rest. Also, use a lumbar belt.Hope I have answered your query. Let me know if I can assist you further.Regards,Dr. Mahaveer Patil"
},
{
"id": 194993,
"tgt": "How can erectile dysfunction and premature ejaculation be treated while on Losartan and Amlodipine?",
"src": "Patient: Hi for the last one month i am suffering from erectile duysfunction and premature ejeculation . I do not get hard erection and do not sustain it too. Iam a hypertensive patient for the last 3 year amd i am taking one tablet of losatan 50 mg + amlodipone 5mg everyday in the morning after breakfast for the lasy 2 year. Doctor: Hi,As per your clinical history, please follow like this: 1) Do a clinical examination by your Doctor[endocrinologist]and to rule out other causes do few investigations like this [Urine RE/ME, Fasting blood sugar,TSH, S testosterone and ultrasound of penis]. 2) Do take regular medicines for your high blood pressure and take treatment for any underlying pathological cause detected in investigation reports. Do follow lifestyle modifications like this:1) Work with your doctor to manage diabetes, heart disease or other chronic health conditions. 2) Visit your doctor for regular checkups and medical screening tests. 3) Try to stop smoking, limit or avoid alcohol, and don't use illegal drugs. 4) Do exercise regularly. 5) Take steps to reduce stress. 6) Get help for anxiety, depression or other mental health concerns. Hope I have answered your query. Let me know if I can assist you further.Regards,Dr. Uday Nath Sahoo"
},
{
"id": 88710,
"tgt": "What causes abdominal pains and anal bleeding?",
"src": "Patient: when i get up, before and after i eat and throughout the day my stomach has severe pains it feels like hunger pains but its not because it does this after i eat to and then i go to the bathroom and there is alot of blood in the toilet and throughout my bowel movements what could this be? Doctor: Hi.The blood throughout he the bowel movement and the history you have noted , this can be due to ulcerative colitis or IBS with severe colitis. I would recommend you to undergo colonoscopy to see the reason, get a biopsy done and institute appropriate treatment.Get stared on an antibiotic, probiotics, metronidazole and supportive treatment. Get investigated : blood, urine and stool tests, ultrasonography, colonoscopy."
},
{
"id": 10722,
"tgt": "Suggest remedy for hairfall post a typhoid infection",
"src": "Patient: Hi , I had suffered from typhoid symptoms for some 2-3 weeks. Now i have recovered from it. But now i am facing lot of hair fall. I have lost almost more than half of the hair i had. i am worried about this. Please suggest if any measures are there. also does the hair grow back and how much time approx it may take to grow back. Thanks a lot Doctor: hi, thank you for putting up your query on HCM.as per your history, you have telogen effluvium which usually occurs after a illness.-- it is a self limiting disease.-- you need not to worry much, as stress aggravates the hair fall.-- it will take around 3-6 months for disease to cease.--you may take cap zevit once a day.-- adequate dietary intake in the form of fresh fruits and green leafy vegetables and applying hair oil like coconut oil twice a week ill help.-- avoid hair cosmetic products, use herbal shampoos twice a week.-- avoid external trauma to hair in the form of straightening, curling and hair spa.-- topical minoxidil can be tried, but it has to be started only after consulting a dermatologist.wising you good health.thank you"
},
{
"id": 183972,
"tgt": "Can wisdom tooth be removed if causing pain and swelling in gums and face?",
"src": "Patient: I am 24, 162cm, 55kg, having eruption of wisdom tooth now but not all 4, only at the right lower part. The pain has been for days and now I have swollen gum and face. Do I need to have this tooth being taken out since it's causing pain? Is it alright to be absent from work due to this and consult the dentist once it's available? Doctor: Hello, thank you for consulting with healthcaremagic. As you are mentioning that your wisdom tooth erupting, then it is normal to have some pain while eruption, but if the pain is for many days then it means that there is less space in the jaw for the tooth that is why it is causing difficulty, better that you should visit your dentist early and get the tooth removed. Hope it will help you."
},
{
"id": 150193,
"tgt": "Brain scan showing altered signals, intense lesion in frontal region. Have episodes of fits. Is this curable?",
"src": "Patient: plz doctor help me just before 3 days I have attacked with fits 2 times . I found my left hand was not working at all at the time of fits but i have recovered from it after 5 minutes . I had done a mri brain scan and the report shows that \" Small altered signals intense lesion seen in right frontal region S/O Healing Granuloma \" . It also shows Hypersignal intensity on T2W and FLAIR images and iso -hypo signal intensity on T1W images. The doctor prescribed me to take valprol -cr- 300 tablet once a daily for 2 years . So plz tell me whether it is curable or i have to take medicine regularly to avoid fits. Doctor: Hi,Thank you for posting your query.Your symptoms are suggestive of partial or focal seizures affecting one side of body. The cause seems to be a tapeworm infection of the brain, which is healing. MRI reports are suggestive of same.You would need to be on anti-epileptic medications (valprol CR) for now to prevent seizures. In many patients, the granuloma would completely heal, then, medications may be stopped. However, in some cases, healing is incomplete, they may need to take medicines for long time.Please get back if you require any additional information.Best wishes,Dr Sudhir Kumar MD (Internal Medicine), DM (Neurology)Senior Consultant NeurologistApollo Hospitals, Hyderabad,My personal URL on this website: http://bit.ly/Dr-Sudhir-kumar My email: drsudhirkumar@yahoo.com"
},
{
"id": 192745,
"tgt": "Would nightfall lead to reduced height and health?",
"src": "Patient: Hi, may I answer your health queries right now ? Please type your query here...Dear Sir My son is 14 Years old and have the 5 -8 . He is gd 9 student. He has frequent nightfall. His final exams are coming.We are worried that his height may not remain same either he may not become weak or achieve poor gds. XXXX Doctor: Hello, Nightfall does not affect height or general health. It is a hoax that does not have any scientific evidence. Hope I have answered your query. Let me know if I can assist you further. Take care Regards, Dr Shinas Hussain, General & Family Physician"
},
{
"id": 121836,
"tgt": "Suggest treatment for jaw pain",
"src": "Patient: Since September 11, my jaw on right side, just below my ear lobe has been very sore. It hurts my jaw at that location when I chew or yawn. There is unnoticeable swelling, if any. VERY tender and painful when I push on the muscles just below where the ear and jaw meet. I have no molars on that side, upper or lower. I had an ultra-sound on the lymph node in November 11, result was normal. I have a light constant pain w/o touching the area. I clean my ears several times a day w/ q-tips, going deeper than I know I should, but it feels so good! Is it possible some cotton from a swab could be lodged in my ear canal and causing the pain? Doctor: Hello,Your symptoms seem to be related to local inflammation or infection of the temporo-mandibulary joint. For this reason, I recommend consulting with a dentist for a physical exam and an X-ray of the joint. If an infection is present in the ear, it should be treated too. Hope I have answered your query. Let me know if I can assist you further. Regards, Dr. Ilir Sharka, Cardiologist"
},
{
"id": 5301,
"tgt": "Trying to get pregnant. Took ova-mit. Suggestions",
"src": "Patient: hello.my name is michaela and i am trying for mor than 6 months to get pregnant,i have 26 years and my husband is 28.We have been to the dr she gave us some vitamines like:fertilcare for him,and for me pregnant care and wellness .after that i took ova-mit for 5 days ,twice a day,and we have been trying to get pregnant for about one month but nothing yet!what i can do?the dr said we have to make some analysises to see if we are ok,but is very expensive to us i feel some cramps in abdomen down in the night only it can be somtingh or is it my imagination only ?thank you very much Doctor: Dear Michaela, welcome to healthcare magic.The good thing is that you are still young, so don't hurry. Six months are not sufficient to make you seek medical advice. I suggest that you wait another 3-6 months and make intercourse at the time of ovulation that is days 11,13,15 of your cycle.If no pregnancy occurs after this period, the first thing we want to check is your husband's semen analysis after 3 days of sexual abstinence. If the semen parameters are ok, at that time you can do whatever investigations your doctor asks you to do.I hope you get pregnant soon and good luck.Dr. Ahmed Bahaa."
},
{
"id": 171727,
"tgt": "Suggest treatment for frost bite of a 12 year old",
"src": "Patient: Hello doctor, We ve been 2 years now here in Saskatchewan and this is my daughter first time to experience a frostbite on her lip.She is 12 years old, 5 2 in height and 80 lbs in weight. May we know what should we do. Thank you very much for your advice. doris Doctor: HiWelcome to the HCMI understand your concerns but don't worry. The most important thing is prevention of any further cold exposure. So, move to warmer climate if possible. For treating frost bite which has occurred, you may take the following measures:1. Warm lip by applying cloth or gauze dipped in warm water. Avoid repeated cold - thaw cycles as they are more damaging.2. Give a pain killer such as acetaminophen or ibuprofen every 6-8 hourly to relieve pain.3. In case the injury is deep, then consult a plastic surgeon immediately for care and management required as per the intensity of damage.Hopefully this will help you. I would be happy to help you any further questions.Take care"
},
{
"id": 26142,
"tgt": "What causes dizziness and heart palpitation after taking Metoprolol 50 mg?",
"src": "Patient: I am suffering from un rythmic heart for almost an year and take metoprolol 50 mg o.d. 2 days ago i had terrific pulse rate variation and I felt very dizzy my bp became 110/70 and pulse rate 60/min was hospitalised now i am out of hospital but feel very weak and still have irregular heart...in the hospital they adminstered normal saline with Cordarone.. what should be my next course? Doctor: Hello Sir / Madam,Thanks for writing in to HCM.Un rhythmic heart (Atrial Fibrillation) is usually treated with Amiodarone (Cordarone) . Hence , you have been given the right treatment. The dose shall change in due course as an when the heart beats stabilize.However, the reason of atrial fibrllation need to be investigated hence you need to consult a cardiologist regarding further cardiac evaluation. Sometimes infections do cause atrial fibrillation which comes down as the infection subsides.What needs to be taken care of is incidences of stroke increase in un rythmic heart beat so kindly take rest and follow your doctors instructions. donot do stressful work. Continue with the treatment.Regards,Dr.Riyanka"
},
{
"id": 159690,
"tgt": "Having chest pain, no lumps found. Ultrasound done. Could it be breast cancer?",
"src": "Patient: Hi, i have had some dull chest pain on my left side. I become aware of the pain when a carry one of my children, i have a 3 year old an 7month old twins. I feel the dull pain on the left side of my chest/breast area. It feels to be under the breast area. No lumps . Could this be something serious? such as breast cancer . I have had an ultrasound performed on my left breast and, thankfully nothing was found however the dull pain remains. Would a mammogram rule out any possibility of cancer? Doctor: hello dear mastalgia (breast pain ) without lump is unlikely related to cancer. hence need not to worry. mastalgia are of two type 1)CYCLICAL: starts one or two days prior to menstruation and relieves on onset of menstruation 2)NON-CYCLICAL: is not related to menstrual cycle. cyclical needs only counselling, evening primerose 3gm/day in divided doses, avoid coffee and chocolate intake. noncyclical needs only pain killer. hence classify your pain into cyclical and noncyclical. and take decision. hope this is helpful to you."
},
{
"id": 92263,
"tgt": "Why there is pain in solar plexus area every morning which goes away later?",
"src": "Patient: Hi, for maybe a couple of moths most mornings I wake up with a pain in my solar plexus area, not terrible pain, more like hunger. But when i get up and have a shower or anything it usually goes without the need to eat. Sometimes its accompanied by a feeling in my throat. Doctor: Hi...you are having gastritis with reflux...take any ppi with levosulpride combination for a few weeks... you may also get an endoscopy done if it dosent resolves...Dr. Ashish Verma"
},
{
"id": 100592,
"tgt": "Suggest medication for asthma with severe headache",
"src": "Patient: hello doc,how are you?i am 47 years old woman.having asthama and sinustis.i have severe and continuos headache for past 5-6 years.i am using all the pain releives.my vision are also blurr.is this headache is related to sinus and asthama.please suggest me a better solution or remedy to overcome this.i willbe very grateful to you and thanks Doctor: Hello.Thank you for asking at HCM.I went through your query and would like to make suggestions for you as follows:1. Were I treating you, I would suggest you regular montelukast and levocetirizine. I would also suggest you to use salbutamol inhaler on as-and-when-needed basis for asthma symptoms.2. For sinusitis, I would suggest you regular steam inhalation which will also help.3. Sinusitis is a common cause of headache, but as you are having headache for last 5-6 years, I would suggest you to consult a physician to rule out other possible causes of headache. In general, I would suggest you to take paracetamol for headache.4. Please consult an Ophthalmologist for blurring vision as it is usually not related to asthma and allergies. 5. I would suggest you allergy testing which will help you identify the substances causing allergies to you as well as the measures to avoid them.6. I would also suggest you regular breathing exercises and a healthy diet rich in vitamins and minerals (adequate green leafy vegetables, fruits, sprouts, etc) which will improve your lung capacity and immunity in a long run.Hope above suggestions will be helpful to you.Should you have any further query, please feel free to ask at HCM.Wish you the best of the health.Thank you & Regards."
},
{
"id": 6538,
"tgt": "Within how many hours one should take ipill to avoid pregnancy ?",
"src": "Patient: i had one ipill an hour after sex,am i safe plz help iam very scared hi, had sex 8 days after my periods and am scared of getting pregnant ,i took ipill after an hour of sex, am i safe,when will my periods occur again,is takin one ipill enough,i had sex on 14th june Doctor: Hi dear Welcome to HealthcareMagic. Taking ipill an hour after sex would definitely be efficient in achieving contraception.You would not be pregnant , but you can have menstrual irregularities like intermittent bleeding , delay of periods.Do not worry and it is better to avoid taking i pills frequently as they are high dosage hormones and can cause other health related complications. Take care."
},
{
"id": 221917,
"tgt": "Is triderm ointment safe for boil during pregnancy?",
"src": "Patient: i have a boil on my upper tummy and im 8months pregnant... is it a cause of alarm? I have been putting triderm ointment on the reddened area.. is it safe... its painful... i covered it up with gauze so it would not rub on my shirt...it s a relief but is it ok to cover it up? Doctor: Hi,Triderm is a steroid cream, and it could increase the size boil if it is infectious, so I would advise you to discontinue it.If the boil is present for a longtime and is causing pain and itching please see a gynaecologist and she will examine you and decide the nature of it. Certain skin conditions are more common in pregnancy and these need to be identified by a doctor before treatment.Triderm is not contraindicated in pregnancy, but should be used with caution and the skin where it is applied should not be covered by anything as this can increase the absorption the drug, and cause more side effects.Do not use it without a doctors advise.Hope this helps.Regards."
},
{
"id": 127158,
"tgt": "What causes discoloration on the calves and numbness in the legs?",
"src": "Patient: I have discoloration of the calf muscle bilaterally. Tingling and numbness down both legs to the middle toe on each side. The peripheral numbness has been the most important and disturbing symptom. Past history of high sedimentation rate(100-120 consistently) and splenomegaly. I weigh 140 lbs,. and its starting to worry me about my future ability to walk/ Do you have any idea? Thank you so very much! Noreen Doctor: Hello and welcome to \u2018Ask A Doctor\u2019 service. I have reviewed your query and here is my advice. The discoloration with numbness can be related to blood clot or a spontaneous bleeding inside due to enlarged spleen. Hope I have answered your query. Let me know if I can assist you further."
},
{
"id": 192108,
"tgt": "What causes tiredness with sweating and lack of stamina?",
"src": "Patient: Hello Sir, I Amrat i am 28(M) - 6 ft - weight 104kgs. I am a beginner weight lifter. My problem is that i get tired very quickly,i got heavy sweating after getting some work done, and after that i feels totally exhausted. No diabetic background, but i am fond of sweets. My mother have thyroid but it arises after giving birth to my younger sister, way back in 1987.till now she is having her medicine on. please guide me that how can i increase my stamina naturally? Doctor: Hello, As you have told, you are a beginner in the exercises. it is natural that you need to improve your stamina by consistent physical activities. Slowly and steadily you will be able to perform tough tasks. Therefore I suggest talking to your weight trainer in this aspect. Take care. Hope I have answered your question. Let me know if I can assist you further. Regards, Dr. K. V. Anand, Psychologist"
},
{
"id": 95891,
"tgt": "I have puss cells in my urine, with this how can i gain weight ?",
"src": "Patient: HI?ANYONETHERE? i m 19 years old female ..i jxt have my urine tst and i found puss cells 10-12/hpf and epithellil cells 6-8/hpf ..i usuaally have pain near my ribs...and i suddenly loose my weight ..i am 5 FOOT 3 INCHS and m afraid my weight is 39 KG.........i want to gain weught and i want to know about my problem i pension above wAT SHD I DO? Doctor: Hi Welcome to health care magic \u00a0\u00a0\u00a0\u00a0\u00a0Get the urine culture sensitivity done & find the effective drugs .Take proper & timely treatment ,along with plenty of fluids daily.Also improve your diet to add weight to your body \u2018Hope I have answered your query, I will be available to answer your follow up queries, \u201cWish you Good Health and speedy recovery\u201d Disclaimer"
},
{
"id": 125848,
"tgt": "How to fix glute imbalance?",
"src": "Patient: Hi I ve been having problems with my glutes for months now (well, since September ish) both sides of my butt feel different when I walk, it wasn t a huge deal at first, then I started thinking about it more and felt like my butt might be uneven looking when it probably isn t. The creases under my butt cheeks are slightly different but that s probably the only real difference I can tell in their appearance. Anyways, I used to like doing squats and riding my bike but now I can t stand doing either as the frustration with my butt drives me to tears (it sounds really stupid, I know). Like you know how your butt feels when you do a fire dude any exercise? That s kind of hous it feels on my right side when I squat or ride my bike (which is how it used to feel on both sides prior to this issue) and on my left it just doesn t feel the same at all. This is really bothering me and I know it is probably a stupid issue but I really appreciate any help I can get. Doctor: Hello, Consult an orthopaedician and get evaluated. An MRI scan is required for further assessment. Hope I have answered your query. Let me know if I can assist you further. Take care Regards, Dr Shinas Hussain, General & Family Physician"
},
{
"id": 179954,
"tgt": "Suggest remedy for diarrhea with vomiting",
"src": "Patient: My 6 year old daughter came home from the first day of school with an upset stomach. She started having diarrhea and vomiting at 8 am this morning. I kept her home from school, gave her some crackers and sprite then she said she felt better. She had loose stools off and on all day then it seemed to finally subside. She began eating and drinking regularly. She went to bed at 10 pm then woke at midnight with poop in her panties then puked everything up she had eaten throughout the day. What do I do? No fever but gets chilled after she pukes and she said she is extremely shaky. Should I take her to the ER? Doctor: Thanks for putting up your query at HealthCareMagic. Your daughter probably has acute gastroenteritis. It might have ingested some preformed toxin through her food. Give her ORS. Prepare ORS by mixing a packet in a litre of water and give it sip by sip. This is important for maintaining the salt and water balance in her body. No need to give her antibiotics as of now. Subsequently you can definitely consult a doctor who would be able to guide your further.I hope that helps. Feel free to revert back with further queries if any."
},
{
"id": 225634,
"tgt": "On ortho patch. Experiencing pregnancy symptoms. Chances of pregnancy?",
"src": "Patient: Im on ortho patch ,I always change my patch on mondays and one monday on the first week a start a new patch bc my period ended I forgot to put on the patch bc it was a holiday and i forgot to bring it with me .so i didnt get a chance to put in on in about 24 hours .My mom said to just put it on right away and i would be fine .but about 2 weeks later my period was very irregular it was very light pink spotting then it turned a little brown ,I was a little confused bc my period is always a normal flow since ive been on the patch ....I do have some pregnancy symptoms but im not sure if i should just take a pregnancy test i just wanted someones opinion Doctor: Hi,The present symptoms of irregular spotting could be either due to an early pregnancy or the hormonal disturbance created by the delayed application of the patch. You can test for pregnancy if it has been more than three weeks after your last cycle through the blood test as urine pregnancy test may not give you any definite conclusions right away. I would ask you to see a gynecologist for an examination as well to correct if it happens to be a hormonal disturbance or further management if it is a pregnancy. Hope this helps."
},
{
"id": 50532,
"tgt": "Have frequent urination, unable to sit, memory loss, absent minded. Have crf, kidney paranchymal disease. Curable?",
"src": "Patient: sir, my mother is 78 years old and she is suffering with crf and kidney paranchymal disease, at present she is unable to sit, frequent urination(small quantity) without any sensation. absent mind, lost her memory. since 2 years she became stout. we are worrying about her, is it curable at this age. we are not in a position to spent more money for her. kindly guide us for her improvement.regardsvenkatarao YYYY@YYYY Doctor: Hello, Thanks for the query to H.C.M. Forum. It is very difficult to say any thing ,without knowing the condition of patient. Chronic renal failure and parenchymal disease are there, but what is the exact general condition of patient , what is the ultrasound reports, Blood reports of urea, serum creatinine, C B C , E S R .Condition of swelling over body. Dementia is a curable but first we should know ,what is the reason of patient in general. Hope I have answere dyour question. If further any question I will help. Good luck. Dr. HET"
},
{
"id": 182524,
"tgt": "Suggest remedies for pain due while treating a cavity",
"src": "Patient: hi,well my question is i think that i have a cavity and im super nervouse that i dont want to go to a dentist to get a fixed beacuse i know the pain.So either way i have to go to go get it fixed,my question is how to you repair a cavity and is there any way my dentist can give me something that can complety not make me feel the pain? Doctor: Generally cavity restoration is a painless procedure but if the caries is extended to your pulp chamber than dentist can do pulpotomy or in severe condition root canal treatment this treatments are performed under local anesthesia you will not feel any pain during and after procedure so don't fear meet your dentist and get it restored."
},
{
"id": 56079,
"tgt": "What are symptoms of having twisted liver and right kidney?",
"src": "Patient: Hello I have a possible liver and right kidney twisted inside my body just got tested for creastinine and sometimes have blood in urine also just had a hernia in my right rib cage as well as on my left lung and my spiter broke got 40 plus staples taken out not to long ago Doctor: Hi,Thanks for writing in.A twisted liver and kidney is unlikely to occur in a human being. The liver and kidney are fixed with ligaments and supports. They have a capsule and also surrounded by fat in some areas. This holds them in place and it cannot get displaced spontaneously. Bowel is a tubular structure and might get twisted or displaced through a hernia which is a defect in the abdominal wall or inside.There is a history of hernia in your right rib cage and this needs to be evaluated in detail for any complications. A X ray chest and if required a CT scan chest might be done to know completely on it. Please do not worry."
},
{
"id": 144071,
"tgt": "Suggest remedy for dizziness and discomfort in ear",
"src": "Patient: I feel dizzy when I am up moving around. I can hear fluid in my head at the back of my skull and neck. I have a heavy feeling in my left ear. My ears will often pop and ring but not continuously. When I bend over clear liquid drainage will come out of my nose sometimes. Doctor: Hello,Your symptoms may be due to ear infection etc.Besides clinical evaluation you may need routine hemogram,random blood sugar.CT scan may be advised after evaluation.Proper treatment depend upon findings.Get well soon. Take CareDr. Indu Bhushan"
},
{
"id": 202586,
"tgt": "What causes poor sperm motility and abnormal morphology?",
"src": "Patient: Me and my husband are trying to conceive but are having trouble.He had a semen analysis and it showed he ad a normal cunt however the mobility was poor and had abnormal morphology or shape.I was wondering if certain antibodics could cause this since my husband took antibodics recently for a lung infection.He has to have another test done to confirm his results.One of the antibodics he was taking was ciprofloxacin could this cause abnormalities in sperm Doctor: IT could be abmornal veins near his testicles called varicocele.If he has it, then he can have surgery to fix it and the sperm.Have a doctor examine him for this.Please rate 5 stars! I strive to provide the best answer for your question."
},
{
"id": 155260,
"tgt": "What causes 4th stage of metastatic cancer?",
"src": "Patient: my father is suffering from metastatic cancer,poorly differentiated(4th stage).can you plaese elaborate about this cancer type? is it curable? If yes what should be his best treatments.If not then what is his survival rate?His age is 57 and his weight is 62kg. Doctor: With this information it is impossible to answer your questions. Where is the metastatic cancer and where did it start from. These are the 2 main factors that will determine his outcome. Yes, it is the last stage technically, but in certain cancers, the patient can survive for years with proper treatment. e.g. prostate cancer patients can live as long as 5 years in the last stage. The treatment also depends on the primary site but is mostly based on chemotherapy."
},
{
"id": 197084,
"tgt": "What caused sudden syncope?",
"src": "Patient: My son who is in good health ..age 48 passed out the other night. My daughter in law called 911 and they checked him ...B/P normal, Blood sugar normal, .....later had profuse diaphoresis and vomited. Did not want to go to the hospital.... The next morning felt fine and did his usual 2 miles of jogging with no ill effects. It is recommended he have a stress test.....do you have any suggestions? Thank you for your help. Angela Doctor: HelloThanks for query .Feel very sorry for tragic death of your son at young age .Based on the facts that you have narrated your son though apparently in good health had coronary heart disease and initially got mild attack of myocardial infarction which was not confirmed by ECG and neglected due to normal blood pressure readings .This attack got aggravated in to massive fatal heart attack due to vigorous exercise which caused him his life .Dr.Patil. ."
},
{
"id": 14698,
"tgt": "Suggest treatment for a red rash on the legs",
"src": "Patient: from one week i have suffaring from some kinds of rash in my legs its like some redish and pain around it I have taken wyeth (medicine) as droctor prescribed but its not clear and wghenever i used cold water for bath in morning this symtomps come back..(as my assumption) could you please help me! Doctor: Hello,Thank you for posting on HCM.The condition you have referred to is called Urticaria or hives. Its an allergic manifestation of skin, where an allergen leads to release of certain substances from your blood, leading to itchy skin rash and swelling over soft tissues. Its proper management requires thorough history, clinical and laboratory work-up.You may have to go for specific tests like patch test, food prick test, IgE antibody levels etc. As for treatment part, best would be the avoidance of allergen as far as possible. Try to eliminate possible triggering foods from diet. I would also advise you various anti-histaminics for long duration( atleast 3 months) with or without oral corticosteroids. For non- responding cases there are many other drugs like dapsone, cyclosporine, montelukast, omalizumab etc which can be used in certain selected cases. Hope this will help you in resolving your query.Thank you Dr Hardik Pitroda"
},
{
"id": 89341,
"tgt": "What can be the cause of severe swelling in one side of stomach?",
"src": "Patient: hi - Im a 48 year old woman with severe stomach swelling and buttock on one side and pain. I stopped my period in november 2011. have had xray and pevic scans and have small fibroid in womb (about 7mmdiameter) My concern is about swelling and especially on one side Doctor: Hi,Thanks for writing hereFirstly i would like to know whether fibroid was only the positive finding in your x rays and pelvic scans?if is it so then only 7mm fibroid wont cause a swelling in abdomen i.e. stomach as you say.It is also necessary to do a USG abdomenIf all these tests show nothing and swelling is local,get it checked from a general surgeon.From the information you provided this is what i can comment.With regards,Dr.Akash Jain."
},
{
"id": 28898,
"tgt": "Which specialist should one consult for shingles?",
"src": "Patient: My husband was told that he has shingles and has an appointment with a neurologist tomorrow. This was at the suggestion of his chiropractor who has been treating him for severe back pain. He has had bladder cancer and also part of his kidney removed because of a malignant tumor. Is a neurologist the specialist he should be seeing for shingles? He's 75 years old Doctor: Hi,Consult a Neurologist if he is suffering from post herpetic neuralgia.Based on the history given, your husband seems to be suffering from post herpetic (post shingles) neuralgia for which he may need evaluation by a neurologist.Hope I have answered your query. Let me know if I can assist you further.Regards,Dr. Mohammed Taher Ali"
},
{
"id": 7994,
"tgt": "Boil/pimple on the crown of my anus, what should i do ?",
"src": "Patient: Hello i am a 16 year old male and have a boil/pimple on the crown of my anus and its very painful when touched. Im not sure what to do. I asked around and some people told me to burst it and some others told me to leave it alone and it will go away. what would be the best course of action out of those two (2) options? Remember i can t see a doctor because i have no money and no medical plan. Doctor: Hello and welcome to healthcare forum. After reading your problem it seems that you are suffering from perianal abscess, which may sometimes progress to anal fistula. So, i am sorry to say that you have to visit a doctor for your problem and after proper examination he will advice you either antibiotic treatment or incision and drainage of abscess. Best wishes for your health."
},
{
"id": 185030,
"tgt": "What causes white stringy mucous in the gum after brushing the teeth?",
"src": "Patient: I complained to a dentist about some white stringy mucous dat develops in ma gum after brushing my teeth in the morning, she prescribed Stolin for me and I don't know whether to rinse it out after applying it for sometime. secondly, will it help my predicament? Doctor: Hello!Thank you for posting here.Stolin gum paint helps improve your gum health.You did not mention if you got scaling done.I suggest you to get it done.You can apply stolin gum paint twice daily on the gums on the outer side and inner side.Leave it for 5 minutes.Rinse it off.Also use Doxy-1 in the night for 3 days.Do not take milk after the tablet.Take it on plain stomach.Also, rinse warm saline gargle thrice daily.Hope this helps."
},
{
"id": 166746,
"tgt": "Suggest treatment for nail marks on face of a child",
"src": "Patient: I ve a 2.5 years old daughter. While playing with other children, one of the child scratch her face with nails. Now she got marks of nails on her face which are very prominent and they seems deep in skin. How can I remove those? Need your opinion. Thanks Doctor: Hello,I just read through your question. Apply any over the counter antibiotic ointment to the scratches 3 times per day. This will prevent infection and speed the healing process.Hope I have answered your query. Let me know if I can assist you further. Regards,Dr. Eric Goldstein"
},
{
"id": 36865,
"tgt": "What causes abdominal pain and swollen lymph nodes along with chest infection?",
"src": "Patient: my 6 year old son has been unwell with a chest infection and nasty cough, this seems to happen at least one week out of every month. i took him to the dr to have him examined as i also was concerned that my son was complaining of a sore lower abdomen when coughing, (not urinating) and a sore to touch upper back (lung area). the dr did a urine test which showed positive for blood, no infection present, he ordered a blood test and second urine test for FBE; ESR;E/LFTS - M/C/S; ANA; CRP;. i have also noticed his inguinal lymph nodes, Retroauricular lymph nodes, and Retroauricular lymph nodes appear inflamed. what in your opinion are the concerning factors regarding the information i have provided. Doctor: HI, thanks for using healthcare magicLymph nodes respond to infection or inflammation in the area where they are responsible for drainage.The enlargement of the cervical nodes (retroauricular) is common with an upper respiratory tract infection (common cold or flu).They should return to normal size by 4 to 6 weeks after the infection provided he does not become unwell again during this period.The enlarged inguinal lymph nodes may indicate an infection in the thigh or groin area eg even a small cut.They can be monitored for any changes.The FBC would likely show an increase in WBC since he currently has an infection.It may help to repeat when he is well to see if it is in normal range.I hope this helps"
},
{
"id": 182793,
"tgt": "Would braces benefit in treating crooked and spaced out teeth?",
"src": "Patient: Hi, I'm just wondering if I could really benefit from braces or if you think that they would be good for me. Well about twelve months ago at my family dentist appointment I was told that they recommend getting braces to fix a couple of things, but at my recent appointment they didn't say anything about it. I'm most likely scheduling an appointment at a local orthodontics soon but I want to get an opinion before going in. The reason I want to get braces is because I just hate how my teeth look. I have a slight overbite and some crooked and spaced out teeth. I think braces could fix it, what do you think? Thanks Doctor: Thanks for using Health Care Magic.Read your query.Yes an orthodontist will be in a better position to help you evaluate whether you need an orthodontic treatment or not.Correction of overbite along with spacing of teeth will usually require a orthodontic correction but planned properly. Once the treatment is started ,completion and maintaining it well is very important.Hope this was useful.Thanks and regards."
},
{
"id": 165832,
"tgt": "What is the reason of stomach pain?",
"src": "Patient: My 7 year old daughter keeps complaining of stomach ach and has been for a few week, she always has wind in the morning which we give her gripe water and she also drinks a lot, but she has never complained of her stomach hurting her like she is now ,its normally in the evening when she starts complaining. My daughter says the pain is in the lower part of her stomach, can you help with any ideas of what it might be or should i take her to the doctors. thank you Doctor: Hello deari would like to have a complete history.Does your daughter has perianal itching also?if yes then its worm infestation.Does she has increase frequency of urination also?Does she has painful/burning micturition?.if yes then its probably urinary tract infection.I would advise a urine routine examination of your daughter.Give her plenty of fluids and syp. vermox 5 ml twice a day for 3 days.Avoid constipation by adding fibre to the diet.Hope it will help.Wishing your child good health :-)"
},
{
"id": 177056,
"tgt": "What causes rash on stomach, knees, arms and cheek?",
"src": "Patient: Hello, my 9 month grandson all of sudden has a rash on his stomach, knees, arms and on his cheek...should I be concerned or could it be he came into contact with something that caused this? He did not have this rash earlier today...now, he has it this evening??? Doctor: it may be allergic to something .it may be resolve itself or if it may causing itching and baby cried then u have to take cetrizine 2 ml two times a day for 1 day"
},
{
"id": 137095,
"tgt": "Suggest treatment for temporomandibular joint disorder",
"src": "Patient: Hello, just recently I have been told I have tmj. With this my jaw has bothered me but its ok because the pain is not severe. Over the past few months my chest hurts when breathing but this happens every now and then. Just today I discovered that the top part of my gums are a very dark color and was thinking maybe that can be a reason why my face has been hurting me. Also within the past two weeks I have gotten really dizzy and feel like I need to puke. This may be nothing but is this something I should take a little more serious? Doctor: TMJ dislocation or dysfunction is not very common like regular knee or back pain. Resting (Jaw movement) should be more than enough for little problem to resolve. However, if rest isn't solving the pain or dysfunction then you must get one X-Ray and consultation of Orthopaedician done.Meanwhile, avoid hard food to chew and speak minimal to give proper rest.'Shitali' pranayama/breathing can help to reduce excessive heat in mouth or chest region."
},
{
"id": 158517,
"tgt": "Diagnosed with Hyperplasia Endometrium w Atypia. On Medroxyprogesterone Acetate. Now is diagnosed Breast Cancer. Side effect of pill?",
"src": "Patient: My concern is about my sister .She was diagnosted with Hyperplasia Endometrium w Atypia.Doctor prescribe for her : Medroxyprogesterone Acetate .She was taking this pills for 2.5 week ( 2 times a day - 10 mg).The bleeding was stoped and she was scheduled for surgery.But at this time she foud out lump in her breast - and after research it was diagnosted as Breast Cancer ( but her mammogram this spring 2013 was normal).Question is : Can this hormonal pills effected for her breast cancer growing to fast or not? Doctor: HiResearch shows that Medroxyprogesterone acetate may increase the risk of breast cancer risk in post-menopausal women. Its use for more than 12 months caused the increase of risk by 2.2 fold in a trial. The risk is lower in younger women. The risk appears to dissipate after discontinuation of treatment.Dr Vaishalee"
},
{
"id": 1227,
"tgt": "Is she pregnant as period is late for one day?",
"src": "Patient: i had fun with my girl friend not a complete sex just touched my penis to her vagina some movements..... she told that she didnt get her periods its already one day late ..!! does it means she is pregnant..? what to do if she come pregnant..!?? could you please help us with this the more problematic issue it ,ight atleast take 2 years to get married with her..?? please help with this problem sir/madam.. Doctor: Hi.No, if there was no penetration art all, then there is nothing to worry about at all. Some women experience still changes in their cycle just by sexual arousal or sexual activities, so I do not think there is anything to worry about.If you need further information, write to me using our premium service.Best wishes."
},
{
"id": 152128,
"tgt": "My C5 was factured and caused a hairline spiltner close to my spinal cord. Should I continue to wear the the neck brace ?",
"src": "Patient: I slipped on the bathroom rug and went head first into the tub. I don t know if I passed out. I had no feeling in my left arm. I finally managed to get out of the tub by pulling my self with my feet. I pulled myself to the living room by the heels of my feet. Called 911 and even pulled myself to unlock the door. Xrays were taken and I had to wear a neck brace. I still had no feeling in my arm. After about 4 hours in the ER. they told me C5 was factured and caused a hairline spiltner close to my spinal cord. By then the feeling in my arm was coming back, but for at least a week no one could touch my left arm because of shockin pain. Today my neck constantly pops with pain all day. Should I continue to wear the the neck brace ? Doctor: thanks for choosing healthcare magic forum, your neural tissue structures are recovering from injury.the symptom which you explained are common sometimes.There are some tests : E.M.G. and n.c.v. tests,which are required to be done to evaluate current status of your neural tissue recovery. it is advisable to wear your neck brace for few more weeks."
},
{
"id": 58758,
"tgt": "Suffering from jaundice. What could be done to cure this? Cause?",
"src": "Patient: band has jaundice and want to know what i can do for himand he is 66 yrs oldwhat causes jaundice?he's eyes and skin are very yellow and he's stools are very dark and has been vomiting quiet a lot and lost a lot of weight and loss of appetite as well Doctor: Hello, Thanks for the query to H.C.M. Forum. Your main query is , causes of jaundice , there are so many causes as, Direct jaundice, Indirect jaundice, drug induced ( rifampicin, probenecid),inherited, hemolytic disorders, Viral hepatitis as A , B , C. D, & E In jaundice liver enzymes level increases so get in blood examination for Serum bilirubin, AST, ALT, AMYLASE, . If still cause is not found then LIVER BIOPSY. Consult physician and get his opinion. Good luck. Dr. HET"
},
{
"id": 208191,
"tgt": "Suggest treatment for learning disability with insomnia",
"src": "Patient: I have had a learning disability ever since I was a toddler. I also believe, I have ADD. I am a very mellow (very low blood pressure) person. I do stress out a lot when I struggle to learn. But I keep it private. I do not want to go through a battery of tests. I would like to take a drug to help me to relax. I do not have any addictions. I do suffer from severe insomnia. Any suggestions for me? I am currently not taking any meds. I am a 55 yr. old female. I would like to be able to have better focus on what I am doing or learning as I mature. Thanks for any advice. Doctor: Hi, Thanks for writing to us, If you do not want to undergo a battery of paper and pencil tests to get yourself checked, psychiatrists will not be able to help you find ways of coping with the stress of learning. There is no drug to treat learning disability. For relaxing, try relaxation techniques and yoga or meditation to help you sleep. Getting a psychiatric evaluation is important to find the cause of insomnia. Simply taking sleeping pills to make the problem go away is not the solution here. Please visit a psychiatrist at the earliest. Hope this helps,Dr A Rao"
},
{
"id": 104560,
"tgt": "Have congestion, watery eyes, cough, runny nose, sneezing, low grade fever. What can be done?",
"src": "Patient: I have what I think to be allergies. Congestion. Lots of drainage which seem to have attached to vocal chords. Have terrible laryngitis for four days now. Watery eyes. Cough . Runny nose . Sneezing . Pressure in ears. Ran a low grade fever of 99.3. Do I need to see a doctor or is there at home remedy. Have stayed home from workbforvtwo days ne. Doctor: Hi and thanks for the query, The symptoms you describe are symptoms of a normal common cold or a probable viral rhinopharyngitis. It is possible that bacteria could cause similar symptoms, but viruses are more common causes. treatment is mainly symptomatic. Draining the nasopharyngeal passages, giving local anti histamines of general if necessary to reduce congestion and simple analgesics or non steroidal anti inflammatory drugs would be sufficient to control the pain of fever. However, a clinical evaluation might be useful to exclude a bacterial cause which might require antibiotics. We cannot talk of allergies in this case, for it needs to be repeated, acute, almost permanent, should have a past history of family history of allergies or other atopic conditions. I suggest you visit your primary care physician for a proper clinical evaluation and management. Thanks and best regards, Luchuo, MD."
},
{
"id": 196039,
"tgt": "Any suggestion for having molluscum contagiosum on penis?",
"src": "Patient: I'm pretty sure i have molluscum contagiosum on my penis. i've started to use salicylic acid on it as of yesterday. to early to tell if it is working but i have noticed that around the papules my skin is darkening where the salicylic acid was applied. will this dissapear after i am done using it? Doctor: Hello Thank you for trusting HCM molluscum contagiosum a common, benign, usually self-limited viral disease of the skin marked by the formation of firm, rounded, translucent, crateriform papules containing caseous matter and intracytoplasmic inclusions (molluscum bodies), which contain replicating virions. The disease is spread by contact and is common in young children. In adults, lesions in the pubic area indicate sexual transmission.This infection is fade out on its own, but you developed allergic reactions to salisilate cream so avoid it. Consult your doctor he will advice other topical agents like imiquimod, canthradin or tretinoin cream."
},
{
"id": 128271,
"tgt": "What causes numbness and tingling sensation in the feet?",
"src": "Patient: Hi, my name is Rick. Last night walking into my bedroom, I was stung on the side of my foot. The pain was like a wasp sting, I tried to wipe off what ever bite me but nothing was there. I could not find what bite or stung me. After inspection nothing was at the site, pain was still there. After a couple of minutes the pain which feels like numbness, tingling and skin soreness started to spread. It went from the side of my foot, to the surface of my foot, toe to ankle, then five minutes later it went up my shin and stopped just below my knee. I have no other symptoms, no sight marks, no swelling or redness what so ever, just what I described above. It has been about fifteen hour since the sting, should I be concerned and what do you think it was that bite/stung me? I live in Arizona and I can only guess it was a scorpion. Doctor: Take Advil anti allergic and cortisone tab one each once a day for 3 days.keep leg and foot elevated.consult a physician .what ever sting is there can be controlled by above meds.your physician shall prescribe"
},
{
"id": 164001,
"tgt": "What causes 6-yearold to drink lot of water?",
"src": "Patient: Hi there. I have a question about my baby cousin (6 year old boy) who apparently has got a rare sickness and he drinks a lot of water throughout the day (3-4 litres) I don't know any other symptoms because they won't say. What sickness (rare) could this be and how serious? Doctor: Hi...This could be an initial symptom of diabetes insipidus. This is called polydypsia in medical terms. I suggest you get him evaluated with his pediatrician. The other possibility is a renal tubular disorder.Regards - Dr. Sumanth"
},
{
"id": 197642,
"tgt": "What does a cluster of brown spots on penis indicate?e",
"src": "Patient: I am very scared as I recently discovered a small cluster of brown wart-like spots on the underside of my penis. I have been tested extensively before my last relationship (as did she) and we were both 100% clean. I have not had sex or any form of it since. I often take baths in a tub that could use a cleaning (it's not filthy - but it's hardly sparkling clean). On rough nights, I have passed out in them from drinking. I wake up hours later and only after doing that recently did I notice these things. I am honestly very scared. Any help would be appreciated. I did my \"homework\" online and the closest I could find people described itching, burning, redness, swelling, or discharge. I have none of those symptoms. Please help. Doctor: HiWelcome.I have gone through your query.It is difficult to comment on it without examination, so I would either suggest you to see doctor for examination and for exact diagnosis or upload the pictures here, I'll go through them, and provide you with specific advice.Hope this helps.Take care"
},
{
"id": 170359,
"tgt": "Suggest treatment for intussusception",
"src": "Patient: My child (9 nd half month)is recurrently suffering from intussusception.sonoguided saline reduction done thrice with in one and half months.how can we protect the child from again occurring this?his diets are cerelac biscuit (aroroot),rice and other streamed food..will dietry modifications help to avoid the condition?The Child is extremely playful at other times will it cause the disease come again? Doctor: Hello, thank you for using Health Care Magic. To amswer your question, no dietary modifications will prevent recurrence of intussuception. The main question.is why is he having it so often? He.needs a colonoscopy to determine what is the.main.focus that is triggering the intussuception. It.could be a polyp or lipoma and seeing a GI specialist will help ."
},
{
"id": 114180,
"tgt": "I have pain on left side of the lower back",
"src": "Patient: I have pain on left side of the lower back , 2 months ago i was diagnosed with kidney infection and was treated for it in USA now iam back in india and have noticed pain in the lower back from 2 days and not able to move or walk properly does it have to do anything with the infection Doctor: i suggest to take South Korean acupressure therapy CERAGEM. phone to their toll free number 18001020203 and find out the nearest center for you. it is free all over the world."
},
{
"id": 206609,
"tgt": "Suggest treatment to overcome fear of men",
"src": "Patient: I am a female age 32 due some reasons I am not able to face people especially male ....I am scared of male ... i have cultivated a strange behavouir of looking at their private parts that I dont do intentionally.... i cannot resist.... I hve gone 2 or 3 molestations in my childhood may b that is the reason.... I am not able to Live peacefully now Doctor: DearWe understand your concernsI went through your details. I suggest you not to worry much. From the given symptoms, this is just a case of obsessive thinking or a compulsive behavior. In short OCD, but not clinically manifested. If you try to resist it, the behavior pattern will try to over whelm you. Therefore, the best treatment method is ignorance. That can be easily achieved by busying yourself for a period of around 30 days. Make yourself so engaged that you have no spare time to think about this. Ignorance sometimes is a bliss.Psychotherapy techniques should suit your requirement. If you require more of my help in this aspect, Please post a direct question to me in this URL. http://goo.gl/aYW2pR. Make sure that you include every minute details possible. I shall prescribe the needed psychotherapy techniques.Hope this answers your query. Available for further clarifications.Good luck."
},
{
"id": 95546,
"tgt": "While doing latrine,i feel that something is blocked",
"src": "Patient: i have excretion problem,while doing laterine,i feel that something is blocked and i can not do laterine further,after 3-4 hours i feel pressure again,please tell me about this disease give me suggestions . Doctor: Hi, Thanks for query, It seems that you are having chronic constipation. Take more green leafy veggies and morefibre containing salads. Take plenty of water. Do daily regular exercise. Take mild laxative as and when required. Ok and bye."
},
{
"id": 23645,
"tgt": "What is the prognosis of a person having coronary artery disease with quadruple bypass surgeries?",
"src": "Patient: My fiance has coranary artery disease. Has had quadruple bi-pass surgery. Three stents inserted with one failure and replaced. He is a white 50 year old. High blood pressure. Also, needs knee joint replacement, but they keep discouraging him from having the surgery. He keeps telling me he is not going to live long and I need to be prepared for this. His heart surgery was in 2007. Please give me an average life expectancy of people who have been through what he has. It s uncomfortable talking to him about it. I m going to be married to him soon, I need answers. Thank you Kelli Doctor: Hi, Kelli. That is a difficult and complicated question to answer. He has severe premature coronary artery disease. There are 2 important pieces of information I would want in order to help answer your question--the most important is: What is his Ejection Fraction (EF)? That is the \"muscle strength number\" of his heart. Has he just had blocked arteries treated with surgery and stents? Or has he had heart attack damage to his heart muscle? It is the latter, not the former, that greatly reduces a person's life expectancy. The 2nd question is: At what age did his parents die? Cardiac disease and cardiac mortality seems to be hereditary, although there are many factors involved, and many more treatment advances in medicines, cardiac interventions, and cardiac surgeries between his generation and his parents' generation that have improved survival for this generation vs. previous ones.One study from the Mayo clinic showed that in patients with 3-vessel cardiac disease (blockages in all 3 major coronary arteries) and a normal EF (normal is usually considered between 60-75%), the 5-year survival rate was about 90%. With 3 vessel disease and an EF between 35-50%, it was about 70%. With 3-vessel disease and an EF under 35%, it drops to about 50%.Of course, much depends on luck and medical care. Does he see his doctors regularly? Is he on \"standard-of-care\" medications that are proven to help reduce future cardiac events and/or decrease his mortality rate? These include beta blockers, statins, ACE Inhibitors (especially if he's diabetic or has had heart attacks or reduced EF), aspirin, maybe other anti-platelet drugs. Has he stopped smoking (or even better, never smoked)? Does he have diabetes (diabetics have higher death and complication rates in general and for many specific complications)? Is it well-controlled if he does? Does he have a defibrillator (AICD) if his EF is at or below 30%?As much as we know about mortality risks from chronic ischemic heart disease and chronic left ventricular dysfunction, there is much we don't understand. Two people can have the same combination of diseases, the same cardiac history, the same heart surgeries, the same EF's--and one can have severe symptoms while the other is \"well-compensated\" and has no symptoms and a much better quality of life. Even with those survival percentage graphs, we may know a percentage number, but we never know for sure who will wind up in which percentage group. All we can do is make sure the people take care of themselves, exercise regularly under the supervision of their doctor, lose weight, refrain from smoking, take the correct combination of mortality-improving medications regularly, and give themselves the best chance of doing well and surviving. The rest is up to G-d or fate or whatever.Congratulations, Kelli, on your engagement, and good luck to you and your fiancee. While these conversations are always uncomfortable, they are important, and, since you will soon be graduating from his fiancee to his wife, you have every right to open a conversation with him about it. Chances are, if he loves you enough to warn you as he did, he'd love you enough to be honest about sharing with you what his doctors have told him. You could also ask him if he'd mind if you accompany him to his next doctor's visit. I have plenty of patients who bring their fiancees or spouses with them. As a matter of fact, I find out most of my honest information from my male patient's wives!"
},
{
"id": 44278,
"tgt": "Spotting between periods, post sex. Pelvic exam shows cervical erosion. Can this affect fertility?",
"src": "Patient: I was told I had a raw cervix during my last pelvic exam several weeks ago. I ve been spotting between periods and after sex. Results are negative for disease/ Hpv . I m married and have never used douche or spermicides. I am looking up cervical erosion online and there is conflicting info about infertility. We aren t actively trying but I am not using bc and we aren t tracking ovulation. I have been off the pill since last September and haven t gotten pregnant but again we aren t actively trying. Should I be concerned about the diagnosis of cervical erosion? Doctor: Hello. Thanks for writing to us. Cervical erosion does not usually cause problem in conception or delivery unless there is deep scarring of cervix present. These cervical lesions can be easily treated with cryotherapy. You can get the proper treatment done at your gynecologist's. I hope this information has been both informative and helpful for you. Regards, Dr. Rakhi Tayal drrakhitayal@gmail.com"
},
{
"id": 22241,
"tgt": "Suggest treatment for high blood pressure",
"src": "Patient: Two weeks ago I had to go for a physical exam for work and my BP was 170/110. The doctor asked me to go back three consecutive days and check if it was still that high. I went three other times, not consecutively, and the result was about the same. I haven't done anything about that yet and tomorrow I am having some blood work. Am I waiting too long? Should I just go somewhere else immediately? I am not feeling my best lately. No exercise either. I don't drink or smoke which is a good sign, I guess. Doctor: Hi,It is good that you don't drink or smoke, I would advise you to restrict the amount of salt in your diet. Ask your doctor to prescribe some medication before you get the results of your analysis, as blood pressure 170/110 is quite high and you might not feel well.Take careCome back if you have any further questions"
},
{
"id": 83632,
"tgt": "Suggest alternative for Clingen forte?",
"src": "Patient: Hi, my doctor prescribe to use 4pkt of clingen forte caps 2pkt this month before menstruction period and other one after mens period for 3 months. I get only one pkt on phamacy which I m using I have try to look for another pkt without any success. Is thre any medication I can use insteady? Doctor: Hello,You may use clima forte pessary for infection. If you developed any allergy means you may consult with your treating doctor he will examine and treat you with alternative medication.Take care. Hope I have answered your question. Let me know if I can assist you further. Regards, Dr. Penchila Prasad Kandikattu, Internal Medicine Specialist"
},
{
"id": 207850,
"tgt": "Suggest treatment for abnormal and suspicious behaviour",
"src": "Patient: dear sir, my wife age is 42, she has short sleep,high pb, and became as a suspecious patients,thinking always about my son s study,some time she behaives very well and time not and she forgot her last speaking means ,no similiarity with her talking,and always thinking some body would takes her flats key or some time she wanted to go for asking keys of our flat.Now some time midnight she try to get down ground floor, watch some thing and come back . so sir what should i do ,please advice? my email id is -- YYYY@YYYY Doctor: Hello and thanks for your query.I understand that you are concerned about your wife's abnormal and suspicious behaviour. From your description of her symptoms, it appears that she is having delusional beliefs and this seems to be the cause for her suspiciousness. This and the other symptoms of irrelevant talk and abnormal behaviour are indicative of a mental illness called Psychosis.Your wife needs a detailed psychological assessment by a Psychiatrist in order to conform the diagnosis and plan further treatment. Effective treatment is available with antipsychotic medication. So, please take her to a Psychiatrist as early as possible.Regards,Dr. Jonas SundarakumarConsultant Psychiatrist"
},
{
"id": 149676,
"tgt": "History of bone fracture. Now CT scan shows bleeding in brain. Mood swings, low testosterone. How to help?",
"src": "Patient: Five years ago, my husband was in a terrible accident where a young lady hit the dump truck he was driving. She died at the scene even though he tried to help her. He himself had a broken wrist, a huge lump on his head with multiple cuts and scrapes all over his body. Two months later, after he just got the cast off his wrist, my husband fell outside during a major snow storm. I quickly drove him to the emergency room. I thought he was having a stroke. The Doctor ordered a catscan and found out he had bleeding in the brain. Fortunately for my husband, it stopped just in time before they had to do surgery. Since then, my husbands moods have been changing to the point that one moment he is crying about something sad that happened to someone we don't even know and the next moment he would yell and curse out me or our kids. Very Bipolar like symptoms. He has been diagnosed with very low testosterone but refuses to take therapy. He says there is nothing wrong with him and it is all me and the kids. How can I help him? Doctor: Hi,Thank you for posting your query regarding your husband's health. It is unfortunate that your husband is not willing to accept the therapy.He definitely has psychiatric symptoms, which could be related to his brain injury. However, he needs medications and counseling under the care of a psychiatrist.He would benefit from anti depressants and antipsychotics. Best wishes,Dr Sudhir Kumar MD DM (Neurology)Senior Consultant Neurologist"
},
{
"id": 203187,
"tgt": "Could itchy, recurring bumps near anus with history of genital wart on penis be just cysts?",
"src": "Patient: I had a genitial wart on my penis 3 months ago that was cauterized repeatedly till it mas removed. One month before my last treatment I noticed a bump near my anus, more on the taint region. The bump which was red would grow and shrink in size dramatically but never goes away. The dermatologist who was treating my genital warts said that bump was just a cyst. This morning when I woke up I noticed another bump on the opposite buttock near my taint. It is very itchy. Could this be genitial warts? Is it possible the doctor misdiagnosed me? Doctor: Hi,Thanks for writing in.You are having genital warts lesions around your anus. Genital warts cannot be treated completely, only the lesions in a particular area can be cauterised. This does not rule out appearance of new lesion in another area as mentioned by you."
},
{
"id": 8182,
"tgt": "Should i take some other medicine along with DIane-35 in hormonal imbalance?",
"src": "Patient: I have been suffering from hormonal imbalanced problem since 10-15 yrs. I get pimples after period becoz i have high progesterone . I have been on Diane-35 for the past 10 months but still im not cured completely. Should i take some other medicine along with DIane-35 or just continue with the same. if i need to continue with same medicine then for how long?? Doctor: Hi! Welcome to healthcareMagic forum, If you have a hormonal imbalance then diane 35 usually works if you have taken it for more than 6 months or so.If diane alone doesn't work then you need to get all your hormonal profile done along with serum insulin level.Drugs like spironolactone or metformin can be added to your prescription if needed.If your acne is sever then even retinoids can be added.All these needs a consultation with both an endocrinologist and dermatologist together.Along with this topical medicines too should be added.Drink plenty of water and have a good 7 hr sleep at night. take care, Dr.Chawda"
},
{
"id": 47287,
"tgt": "What is the treatment for pain?",
"src": "Patient: I have increase of cortical echoes according to my reportThis is likely to be vascular in aetiology unilateral Excess fluid in the bowe which is overlying the kidney Can u please let me know what to do As I am having pain everyday after eating anythingThanks Doctor: Hello , The increased fluid in the bowels is because proper bowel preparation was not done before sonography. I mean they didn't give you any laxative to clear the bowel . It's normal finding . The increased cortical echo texture of kidneys per say may suggest some kidney injury . Bilateral renal artery doppler ( one kidney which has increased echo texture may have some blockage in flow ) and S. Creatinine and maybe comprehensive kidney function test should be considered. However I would like to point out that none of the above may cause pain post meals . A plain antacid like neksium 40 mg once a day in morning before breakfast should be taken for 6 weeks . Antibiotic metronidazole can be taken for 5 days if you are in India / Africa . Let me know if any further clarification is needed . Regards Dr Priyank Mody"
},
{
"id": 23830,
"tgt": "Does normal breathing after exercise indicate a healthy heart?",
"src": "Patient: I'm 72 years old and I've checked my heart rate a number of times this evening. I haven't been getting a lot of exercise the last couple of monthsbut I did challenge my 9 year old to a foot race without a warm up and did not get left behind or breathe too hard after a few yards. I do a few miles on the treadmill, my chief source of exercise. How would this rate on a scale of 1 to 10? Doctor: Hello!Thank you for asking on HCM!Based on your later physical activities, I am glad to confirm that you do not seem to suffer from any cardiac disorder. I would rate this as 10 for your age. I recommend continuing on a healthy lifestyle (Mediterranean diet, a lot of physical activity, avoid smoking contacts and alcohol). You have nothing to worry about!Kind regards, Dr. Iliri"
},
{
"id": 103128,
"tgt": "Child has asthma and allergy issues with breathing problem. Albuterol is not helping, chiropractic helped. Suggest?",
"src": "Patient: my 11 year old daughter who has asthma and allergy issues is all of a sudden struggling to breathe normally. Albuterol isn't helping, she is contstantly taking long deep breathes. She just had her back adjusted this evening by the chiro it only helped her for a bit. Does she need to be taken in or just monitored? I have also just gave her benedryl about an hour ago Doctor: the allergies usually increases with time and age as the medicines working already can loose control over disease nontelucast bd dosage can give benefit along with other medicines you are givingbenedryl candry up the mucusif there is lot of problem consult allergy specialist doctor as this has t adjust the dosage and medicines depending on the situation of the lungs and breaths"
},
{
"id": 183911,
"tgt": "Suggest remedies for tightness in jaw after a tooth extraction",
"src": "Patient: i got all four of my wisdom teeth out almost a month ago and i still cant open my mouth all the way and ive been exercising it everyday. i get more pressure on my right side where it hurts and i feel like my jaw when i yawn goes more to the left side then to the normal up and down what do i do to make it open all the way? Doctor: Dear sir/ Madam, I understand your problem.Since you have removed all four teeth at the same time , you would have kept your mouth wide open for atleast 45 minutes to one hour (both in local and general anesthesia) Resulting in jaw muscle fatique.Normally we must start mouth opening exercise from the fouth day of extraction.Since you have restricted mouth opening after a month, take some analgesic with muscle relaxant and try forceful mouth opening.your jaw deviation while yawning will also be corrected.Dont worry, restricted mouth opening is very common problem after wisdom tooth extraction.It is absolutely solvable.Hope your query is answered.For further queries please dont hesistate to contact back."
},
{
"id": 154739,
"tgt": "What are the side effects of chemo therapy?",
"src": "Patient: my father is under chemo for Ca Rectal metastatic stage....he is having burning sensation after this medication and also have an sevear itching pain in the stomach....please give the medicine for this ..........he is getting chemo since last 10 months but only now he is having this problem....... Doctor: Hi, dearI have gone through your question. I can understand your concern. Chemotherapy has many side effects like nausea, vomiting, acidity, hair loss, weight loss, immunosuppressive actions etc.He should take Proton pump inhibitors like pentoprazol or rabeprazole and ondansetrone to avoid gastrointestinal side effects.Consult your doctor and take treatment accordingly. Hope I have answered your question, if you have doubt then I will be happy to answer. Thanks for using health care magic. Wish you a very good health."
},
{
"id": 80357,
"tgt": "What is the cause of chest tightness and abdominal tightness?",
"src": "Patient: Hello, I am a healthy 18 year old female but when I take a deep breath I feel a tightness around my upper abdomen/lower chest sort of underneath my right breast . Last week I had tightness in my chest but I am almost positive it was my anxiety. Any idea of what this feeling is? Doctor: Thanks for your question on HCM. I can understand your situation and problem. In my opinion, your symptoms are mostly related to anxiety. But better to get done ecg, to rule out cardiac cause for your chest tightness. If ecg is normal then no need to worry for cardiac cause. Anxiety seems more likely cause for your symptoms. So better to consult psychiatrist and get done counseling sessions, try to identify stressor in your life and start working on its solution. Avoid stress and tension, be relax and calm. You may need anxiolytic drugs too. Don't worry, you will be alright."
},
{
"id": 38314,
"tgt": "What causes bump on butt cheek?",
"src": "Patient: hello, im 26 year old male and i have this bump inside my lowerleft\u00a0butt cheek, I noticed it this afternoon when I was have a bowel movement. I looked between my cheeks and didnt see the bump but when i placed my finger where the sensation was, i felt it. When i press on it i do feel some mild discomfort, on pain scale id say its a 3. During my first bowel movement (aprox 10 or 10:30am today (06.15.14)) I didn t feel any bump when whipping myself. It wasn t until aprox 6pm today after my second bowel movement, did I notice the bump when i was whipping. So im figuring that the bump would have had to form between? Please help me figure it out Doctor: HI, thanks for using healthcare magicIt is likely that it formed during this interval since you did not notice it 3 days ago.You would have to monitor it to see if it progresses.It may be a small infected 'hair bump' (folliculitis) or it may be revolving into an abscess (collection of pus).Since it would be difficult for you to see it clearly you may want to visit your doctor for an assessment.I hope this helps"
},
{
"id": 114431,
"tgt": "What causes high cholesterol levels?",
"src": "Patient: I have just had a blood test as I have changed doctors and she told me I have high cholesterol(she wouldn\u2019t tell me the level just that it is high) she also put me on cholesterol lowering medication. I am vegetarian and really eat any dairy products, oils ect. She wouldn\u2019t say anything about how this may have happened just that I probably have a family history. I am currently on sertralin for severe depression and anxiety as well as the pill(levelen ed) I have been on the pill about 10 years. This spike in cholesterol has happened quickly as it was normal about 2 months ago when I had my last blood test before then. I\u2019m a 21 year old female. Doctor: Hello,Cholesterol is mainly supplied from the diet, and our body can also synthesize it in a small amount. Cholesterol is mainly in oils, cheese, fatty food et cetera.As you have depression and you are taking medication, it may alter metabolism and cause side effect.Hope I have answered your query. Let me know if I can assist you further.Regards,Dr. Bhadresh Lakhani"
},
{
"id": 199000,
"tgt": "What does the sperm count on semen analysis report state?",
"src": "Patient: Hello Dr. my husband's semen analysis report..Liquefication time 30min, vol 2ml, sperm count 78million, active motile 30%, sluggish motile 35%, non motile 35%, abnormal morphology 20%, pus cells 2-3 /hpf.his age is 29.can u please suggest me the results are normal or not.thank you Doctor: HelloYour semen analysis findings may indicate semen infection.Infection is indicated due to presence of few pus cells in semen.Normally semen shouldn't contain any pus cells.You may need investigations like routine hemogram,random blood sugar,semen culture and sensitivity,ultrasound of scrotum.You may need to take antibiotics based on culture and sensitivity report.Motility is normal.Normally it should be at least 55 %.Your findings suggests over all 65% motile sperms,so it is normal.Other factors like sperm morphology,count etc are normal.Fertility is expected to increase with control of infection.Get well soon.Take CareDr.Indu Bhushan"
},
{
"id": 97297,
"tgt": "How to treat subaniod hematoma?",
"src": "Patient: it has been 3 month since I fell at work had a subanoid hematoma, concussion and blood behind ear drum having problems with hearing loss and shoulder problems my question is should my head where I hit still be sore and have a knot on it also yes I was unconcusis for a few minutes and don t remember anything and have also lost sense of smell Doctor: Hi ! Good evening. I am Dr Shareef answering your query.If I were your doctor, with your kind of history, I would advise you for an MRI head and brain to rule out any chance of a residual hematoma specially with a history of loss of consciousness and amnesia. Further management would depend on the reports of the MRI.I hope this information would help you in discussing with your family physician/treating doctor in further management of your problem. Please do not hesitate to ask in case of any further doubts.Thanks for choosing health care magic to clear doubts on your health problems. Wishing you an early recovery. Dr Shareef."
},
{
"id": 38581,
"tgt": "Suggest treatment for swelling due to insect bite",
"src": "Patient: Yes. A large branch fell on a small box wood plant last Saturday ( two days ago). After removing large branch, took off my gloves to remove the broken box wood branches and reach into the bush and received a sharp hot sting on end of my little finger. Could not see or find the insect, there was no marks or stinger in the skin. Treated with ice and desoximetasone .05%, which reduced the stinging. Went back to work. Use the desoximetasone every 3-6 hours and stinging reduced. Finger did not swell much until now. End of finger is swollen about 50% in size, hard, hot, and pink. Swelling is spreading down finger. Doctor: Hello, Thnx to contact us. If I am your treating doctor I would like to advice you that insect sting has foreign protein that has injected in the screen. The reaction in terms of swelling is because of reaction of body to that foreign protein. I think you discuss with your treating doctor with antihistaminic (chlorpheniramine maleate), oral steroid (prednisolone), analgesic (aceclofenac) & antibiotic (Amoxycillin). Also take good local care in terms of cleaning, dressing with betadine etc. If you have anything else to ask please contact me. Thanx. Dr. Arun Tank"
},
{
"id": 126677,
"tgt": "Is my ankle sprained or fractured?",
"src": "Patient: I have rolled my ankle many times in the last two years but we haven t gone to get it checked out yet. Today during soccer practice I heard a crack and my ankle immediately became numb. I can walk on it mostly normal but when I touch in certain places on the bone it hurts. I can t tell if it s a fracture or a sprain. Doctor: Hello, If you can walk, then it's not fracture. Fracture is very painful to walk. It could be sprain. My advice is to have an X-ray. Hope I have answered your query. Let me know if I can assist you further. Regards, Dr. Phanindra Dulipala, Diabetologist"
},
{
"id": 62040,
"tgt": "What is the painful blood filled lump on my vagina?",
"src": "Patient: About a week or two ago I noticed a lump on my vaginal area. Got scared. Then the next day it seemed to have grown considerably. It became painful to walk or even wipe and shower. I got a mirror to see it and it was big. I tried squeezing it several times to see if it would pop. I couldn t get it to pop any until tonight in the shower. It only squeezed a little bit of blood. It has a dark spot in it as of looking in the mirror tonight. What could this be? Thanks. PS I had my Pap smear in August but there were no abnormalities and I ve been celebate for over a year. Doctor: hi.is it located internally or externally? based from your description, it could be folliculitis if it is on the outer surface. this usually occurs after shaving or when a hair is accidentally pulled out and it gets infected. it could also be a cyst, or an inflammatory reaction which is starting to form an abscess. if it is located internally, a consult with your gynecologist is best for clinical evaluation. other tumor types must also be ruled-out, such as malignancy. medical and/or surgical management will be directed accordingly.hope this helps.good day!!~dr.kaye"
},
{
"id": 187442,
"tgt": "Is it normal for redness and swelling to remain in cheeks after removal of wisdom teeth?",
"src": "Patient: I had my wisdom teeth removed last Thursday and my left side is swollen and their was hardness in my cheek next to the incision spot. It was extremely painful and I call my doc and he prescribed 2 antibiotics and drained it. He said it was an infection. Most of the pain is gone and but the swelling and hardness remains. It was drained yesterday.Is this normal ? should i get a 2nd opinion? Doctor: Hello, Welcome Thanks for consulting HCM, I have gone through your query, as you have mentioned that you have undergone wisdom tooth extraction you have now swelling and pain dont worry some times Dry socket occurs as your dentist has drained and given you antibiotics , you take it properly on time it swelling will resolve by time and visit your dentist regularly for Oral examination , Do warm salt gargle two - three times a day Hope it will help you. Wish you good health Regards Dr. Priyanka tiwari"
},
{
"id": 922,
"tgt": "What are the chances of pregnancy with fibroid and heavy cycles?",
"src": "Patient: I'm 46 and I was diagnosed with several fibroids, one the size of a fist, my doc said that there was nothing that could be done, because it's not really a problem, but I have one day of very heavy bleeding and I look and feel like I'm about 3 months pregnant. Is he correct? I also have hypothyroidism and taking synthroid. But other than those two issues im fine. And what are the chances of me having another child without fertility drugs at my age? Doctor: Hi, I think you should wait for 2 to 3 months and see the size of fibroid on scan. You may need surgery for removal of fibroid if size is same or increasing. You can get pregnant again but you will need some fertility treatment. Hope I have answered your question. Regards Dr khushboo"
},
{
"id": 60369,
"tgt": "What is the cause for unconjugated hyperbilirubinemia and high ALT ?",
"src": "Patient: My father has been admitted to hospital with jaundice . He has unconjugated hyperbilirubinemia and high ALT. He is also severely anaemic and in one day his haemoglobin levels dropped from 13 to 5. He had 4 units of blood to cater for this. He has high iron levels but otherwise blood tests are normal. He also has sciatica . Nobody can figure out what the problem is. He does not have enough energy to stand up and walk. He has had an xray and ultrasound both of which were normal. The next step is CT scan and bone marrow biopsy. Do you have any ideas what may be causing this? Doctor: Hi, Welcome to Health Care Magic, Thanks for your query. It is hemolytic jaundice which has many reason.Clinical examination,lab investigations are required to find out the cause. Kindly consult a Physician or a gastroenterologist for better management. Thank you"
},
{
"id": 128668,
"tgt": "What causes pain in shoulder and loss of appetite?",
"src": "Patient: my right arm hurts when ever i try to do simple things like put a blanket on at night, and my head always hurts, it started happening a while ago and im not sure whats wrong . i havent been eating so much meat lately because i hate how it tastes. maybe im not getting enough iron? Doctor: why don't you visit a psychiatrist. So much unrelated problems and dear iron deficiency don't typically affects selective body parts. why you eating so much meat? I will suggest report to a physician and certainly to a psychiatrist"
},
{
"id": 221358,
"tgt": "What causes blood spitting during pregnancy?",
"src": "Patient: I am 3 1/2 months pregnant, and this morning, I threw up a small amount of blood (2ish Tbsp). This was right after I ate a huge meal that made me overfull, and I only threw up about a cup total. I haven t thrown up for weeks, and it was rare even with morning sickness. Any ideas what caused it? Doctor: HiI have gone through your complaint.You are three and half month pregnant withblood spotting .the pissibilities can be placenta previa,placenta is covering internal os.its imp to exclude this.go for ultrasound .asses the fetus , placenta.if placenta previa is there best is absolute bed rest to avoid further bleeding.if placenta is not related to birthcanal nothing to worry.Thank youRegardsVasundhara"
},
{
"id": 205820,
"tgt": "Suggest treatment for auditory hallucinations",
"src": "Patient: MEDICAL HISTORY Dear Doctor, I am writing this letter for our daughter (23 years now). We started our USA life in Florida on 2001. She started her study at 5th grade. We have two children (Son and daughter). She is our youngest one. 1)\u00a0\u00a0\u00a0\u00a0\u00a0When she was studying in 10th grade (2005 year), A big hurricane hit our house window glass and burst. At that time we were praying in another bedroom. That wind burst our bed room window where we were praying burst outward. She feared very lot at that time but she has no problem at that time. 2)\u00a0\u00a0\u00a0\u00a0\u00a0One day she was taking shower after her school some boys hit her bath room. She feared, ran to sitting room and could not speak some. My wife asked so many times, after some times she told her this incident. Our house was on ground floor.This was the second incident. 3)\u00a0\u00a0\u00a0\u00a0\u00a0After one year, when she was studying in 11th grade, some boys bothered her so much. She told me that no one never touched her but they teased her with words like prostitute etc. when she travel by school bus or she was studying in school because she is nice, beautiful. So they tried to attract but failed. She needs to live holy, doesn\u2019t want American people life. She didn\u2019t know who is working in background. At last she found told us that one of boys is living above our apartment house. Her tension increased, one day we noticed that she is smiling and talking herself. This was starting point. We admitted in hospital and discharged from hospital after 10 days. They gave Seroquel tablet. We found that our daughter will not get peace if she stays in our apartment house. So I sent daughter and my wife to India in February 2007. We admitted her in hospital, Kerala India and they doubted, it may be Schizophrenia. They gave medicines Night: Quetiapine (Ingredients: Substituted for Seroquel) Morning: RIZ LS - Risperidone and Trihexyphenidyl Hydrochloride Orodispersible tablet (Ingredients: Risperidone BP 2 mg, Trihexyphenidyl Hydrochloride IP 2 mg) When she was taking, she told us that she cannot look naked body when she bath or change her dress and she is hearing voice. My wife told this problem then doctor told her that ignores it. My wife didn\u2019t know that it was medicine side effects. 7 years she suffered these problems and she studied 11th, 12th and graduated B.Sc. in year 2013 with very good marks. I got a new job in Texas, USA on June 2012. So I moved from Florida to Houston. So we decided to bring my wife and our daughter to here (Texas, USA) and our daughter can continue her education. They came here on July 2013. At that time she was taking medicines as follows. Night: Quetiapine 50 mg (Ingredients: Substituted for Seroquel 50 mg) Morning: RIZ LS - Risperidone and Trihexyphenidyl Hydrochloride Orodispersible tablet (Ingredients: Risperidone BP 2 mg, Trihexyphenidyl Hydrochloride IP 2 mg) As per our doctor s instruction india, we reduced medicine from Quetiapine 50 mg to Quetiapine 25 mg (bed time). On 2014 March 2nd night (after one month), She couldn t sleep, so I brought her to Hospital Emergency Room on March 3rd morning. They tested her blood but they didn t give any medicine and discharged from hospital. Then I called our doctor in India, he told me that increase medicine Quetiapine 25 mg to Quetiapine 50 mg. So we gave Quetiapine 50 mg on March 3rd night but she couldn t sleep again. So I brought her to Emergency Room (Mid night). At that time she was very nice to talk to us. They admitted her on March 4th evening on patient care. So doctors stopped all medicine then she has no problem and she can sleep without any issue. Also she can look her naked body when she was bathing and she has no hearing voices etc after stopping these medicines. Everything happened because of these medicines side effect. Next day they discharged from hospital and prescribed started Risperidone 2 mg and Benadryl tab 25 mg bed time only for safety. They didn t check prolactin level before giving Risperidone tab. I was watching her and found that something is going on her but she has no issue. Her period (menses) started March 3rd, 2014. After taking this medicine 18 days, her period was started again. Then I identified, it is an issue of this medicine. We got appointment after 5 days as per hospital referral doctor. March 26, 2014, we consulted the doctor and I told her the problem. Doctor didn t believe my words. Because of my request, doctor wrote prescription to test prolactin level. We gave her blood on March 28, 2014 Friday and Lab told, they will send result to doctor on Monday March 31, 2014. Friday after noon, she was completely changed and she was throwing all old dress to garbage, tore old photos, documents and etc. Next day again worst, she throws all disliked items to garbage. I contacted doctor but she didn t change medicine. Two times I brought her to hospital, they didn\u2019t believe our words. I am not able to stop medicine without right help. At last again brought her to our hospital Emergency Depart, there duty doctor stopped this medicine and prescribed a tablet to sleep and discharged. The doctor told us to contact our follow up doctor on morning. We gave this sleeping tab at midnight and then she slept on that night. Morning, we went to doctor s office, at that time, doctor got prolactin result and result was abnormal (124.5) which is 6 times more than the maximum (18). She changed medicine quetiapine 50 mg in night (bed time) and Quetiapine 25 mg morning only. Doctor didn t give any medicine for Risperidone internal reaction. One week, we suffered very lot and found that our daughter\u2019s tendency \u201cto throw old dress\u201d which was not completely cured. We found another good doctor (Neuro Psychiatrist) and changed appointment to him. During this period, we tested Neuro transmitter test and found that Dopamine is normal but DOPAC level is low. We tested neurotransmitters (after her periods) from urine (They told give 2nd urine to avoid any medicine trace). Neurotransmitters: Serotonin (5 HT), Catecholamine (Dopamine, Norepinephrine, Epinephrine), GABA (Gamma- Amino butyric acid level), Glutamate, R/E Ratio, Glycine, PEA (Beta-phnylethylamine level), Acetylcholine, Endorphin, Melatonin. A test result, DOPAC is low and others are fine. The new doctor told Quetiapine is also allergic to her. He told us that Risperidone damaged her brain which affected her thyroid gland function and hormone. He gave Latuda 20 mg in night only. After some blood test we found that her Vitamin D is low. Doctor told that it will affect her mood. So doctor prescribed vitamin D. After giving 3 days vitamin D medicine, our daughter told that she feels snake crawling on her body. Next day we watched and gave vitamin D to her, after one hour our daughter was alerted and she told snake strangling her neck also. So we contacted doctor and he looked her problem. Doctor told it is hormone sensitivity which was caused by Risperidone. He stopped vitamin D and prescribed a tablet to decrease prolactin level and prescribed a medicine to increase thyroid function. After 2-3 days giving this medicine, our daughter\u2019s mood were changed, the doctor stopped these medicines and referred to hospital. One week, our daughter was admitted this hospital. This hospital told us that her disease may be some kind of Psychosis. So the doctor prescribed the medicine Abilify 5mg(aripiprazole) and Activan (Lorazepam 0.5 mg) twice a day. We told doctor that she is going back to home country India to take master degree or other professional degree. Her mood is changing in her period\u2019s time. Also if she eats any item with ingredients Corn her mood will change (eg: Oats). As per our watching, something is connected to her hormone. They left to India again. One doctor in India, increased Ablify and gave another tablet \u2018pacitane tablet\u2019 to remove side effects but she fell down unconsciously two times. Also we give quetiapine tab before, same thing happened in USA. Suddenly my wife take her to another doctor in cochin, she explained all allergies etc. Doctor told , Abilify is not the right medicine, he changed medicine to Sizodon LS tab half morning and half evening, Activan 10mg night. My wife told doctor that Risperidone is severe allergic to her. Two days before we found that her behavior is changing, we tested prolactin which is elevated from 5.5 to 45 within 10 days. My wife told but doctor is not considering these changes. If prolactin increase Here is the list of behavior change happened in USA 1) The habit of doing/telling lies (Same thing happened in kerala today when my wife observed Our daughter, she pretend sleep on bed tonight and also she covered her body with blanket {which used in cold weather} when the atmosphere was very hot now in kerala.) 2) She sits alone on a room and anyone comes to room, she will get out from there. (This happened in kerala one day before). \u00a0\u00a0\u00a0\u00a0\u00a0 3) She took important items and throws them in trash. 4) She tore photos and valuable documents. 5) She loses touch with reality and starts talking non-sense After all these behavior changes, we suffered 3 months to bring back our daughter at least 50% with her right behavior. Doctor in USA told, Risperidone tab affected her pituitary gland and prolactin level based on blood test results. So we cannot continue this medicine. We need right treatment. Now she is allergic to Corn, Soy, Pistachio, Walnut, Corn Oil and Quetiapine tab, Risperidone tab, Vitamin D tab, RIS-LZ tab. We need proper help to bring back our daughter with her nice behavior with minimum medicine. All these issues happened after giving Risperidone. What is will be alternate medicine for Sizodone. If we give aripiprazole tab, only issue is, some mood changes (not always). So what is the right medicine for her Regards Doctor: Hello thanks for asking a question from HCM for your health related queriesYou have mentioned detailed history and it can help in making a provisional diagnosis. From the history you have mentioned and age of the onset of her symptoms it appears that her diagnosis of Psychosis is correct and probably she is suffering from Schizophrenia. A lot of factors affected her psychological development like migrating from India to USA and then due to fear from boys and change in culture she was migrated again to India. Over protective parenting also resulted in increased pressure on her like her decision to remain \"holy\" even you were aware of this and this is not a expected behaviour. Her symptoms started with muttering and later progressed to violent episodes. Initially she was well with quetiapine but later due to shifting to USA her symptoms relapsed. She developed hyper prolactenimia due to risperidone and this has resulted in galactorrhoea. She failed to respond to Quetiapine again. Aripiprazole has cause intolerance and mood changes and Extra pyramidal side effects. Neurotransmitter studies revealed abnormal receptor functionsAs per my opinion she should underwent one MRI scan of brain with special target to pituitary ares. Porlactin increase to such levels with such doses of Risperidone is no so common. MRI will rule out any prolactinoma or other tumour. Once it is ruled out then we can think about other medicines. Olanzapine is relatively better tolerated and is safer. Clozapine can also be considered as a good option. The drug is useful for resistant symptoms and have excellent efficacy. You can discuss with your psychiatrist regarding prescription of these drugs. Both drugs will help in sleep also.Thanks hope this helps you. If you have some more doubts then you can contact me directly via that link-http://doctor.healthcaremagic.com/Funnel?page=askDoctorDirectly&docId=65151Dr. Seikhoo BishnoiConsultant Psychiatrist"
},
{
"id": 154658,
"tgt": "Suggest treatment esophagus cancer",
"src": "Patient: My father aged 74 years has been suffering from esophagus cancer since past 3 months and on ayurvedic treatment from the beginning. Not taking anything due to low apetite and today his there is swelling in his red eyelid. Is there any relation between this swelling and cancer? He lost nearly 10 kgs in these 3 months. Doctor: Hi,Thanks for writing in.Cancer is a difficult condition to treat and it is best to follow standard treatment practices for esophagus cancer. Ayurvedic treatment might not work always with cancer as it can spread fast. As there is weight loss in last 3 months, it appears that cancer is spreading and causing his body to become weak.The swelling in eyelid might be due to fluid retention in the body which is due to cancer. Please take him to a cancer treatment center immediately and get him treated for his general condition. Get his complete blood investigation done and provide nutritious diet. Cancer patients can survive for a long time if early treatment is given. Please do not worry."
},
{
"id": 14123,
"tgt": "How can leg rashes with blisters be treated?",
"src": "Patient: I have type 2 diabetes. I have leg rash some are like water blisters, my doctor said it was called veena but I can t remember the rest of the name. She told be to get compression socks. I did and it busted the blisters. I m washing my legs with Band Aid antiseptic wash and them I but Neosporin on the broken blisters. Is there something else I can do? Doctor: Hello and Welcome to \u2018Ask A Doctor\u2019 service. I have reviewed your query and here is my advice.Your symptoms can be related to Impetigo witch is a bacterial infection.The treatment that you're doing is correct.Some advice are:-Make sure to wash your hands thoroughly after treating the impetigo sores.- Gently clean the sores with warm water and soap and then remove the crusts-You can also soak the affected area in warm soapy water before removing the crusts.Hope I have answered your query. Let me know if I can assist you further"
},
{
"id": 210737,
"tgt": "Suggest remedy for panic disorder, fatigue, weightloss, memory problems, cold hands and feet",
"src": "Patient: I m 37 years old, have panic disorder and I m very tired all the time.I can t gain weight , I ve lost alot of muscle and have problems eating. I can t think right have memory problems. I m always cold especially my hands and feet. I ve had tests done for my heart and diabetes and every thing was fine. I ve been to different doctors and therapists and can t figure out what s wrong. I m so tired all the time and it has made me lose jobs it s hard for met ork 8hrs cause I have no energy. please help Doctor: HIThanks for using healthcare magicI think, you have underline depression and anxiety symptoms, that you have, are part of depression. In that case, better to take antidepressant like miratazapine. It would help to control depressive symptoms as well as increase the appetite. For that, you have to consult a psychiatrist. This drug is not available over the counter. Thanks"
},
{
"id": 215747,
"tgt": "Suggest treatment for chronic pain",
"src": "Patient: i have tic doularoux and migraines. i am at my wits end. i use hydrocodone, gabapentine, clonepin and blood pressure meds plus levoxl for hypothyroidism. i am sensitive to most drugs and can t take many i have been prescribed. since i have lived with this since 1999, i now push myself to drive a car, gardening, housework , shopping i am not able to hold a job, but my husband supports me and provides insurance, my question is, should i stop pushing myself to do all these things and just stay in my rocking chair. or keep this up which feels like torture to me and makes me cry out in pain everyday. Doctor: Hi, You can do all your routine activities. Keep engaged and avoid unwanted straws and anxiety. You can take analgesics like Tramadol for pain relief. If symptoms persist, better to consult a neurologist and get evaluated. Hope I have answered your query. Let me know if I can assist you further."
},
{
"id": 113342,
"tgt": "Back injury due to fall, numbness, pain after exercising. Prescribed anti inflammatory, muscle relaxer. Cure?",
"src": "Patient: Hello doctor, My name is XXXXX, 22 year old female, recently injured my back falling down stairs. I worked out a couple days later and immediately got numbness and excruciating pain radiating through my hips and down my legs. Went to urgent care and the physician said in my x rays everything looked normal except maybe a slight tilt in coccyx which the radiologist ruled as normal. It s very painful to walk sit or stand up but the physician just prescribed a muscle relaxer and anti inflammatory. Should I still go see an orthopedic or wait to see if the pills fix the issue? Thank you! Doctor: Thanks for the query. Do you still have numbness and pain along the leg? Back muscle sprain is usually cause back pain but not the leg pain or numbness in the leg. Do you have specific tenderness along the spine? It may be simple muscle sprain but if you still having leg complaints then I would investigate further. Get examined by a orthopedic doctor. I would like to rule out the disc prolapse.Thanks."
},
{
"id": 130769,
"tgt": "What causes sudden loss of strength in hand with vision problem?",
"src": "Patient: hi thanks i am very worried. the other day i woke up and couldnt use my right arm and hand the way i normally do. i found i couldnt grip things with my normal strength and have trouble doing normal things like doing buttons up etc. i did ask my pharmacist what the symptoms might be and he asked about my vision to which i answered it was fine. however when i got home i realised that the past few weeks i have started having episodes where it feels like my eyes are not focusing together and the room is hard to focus properly. i have also had minor bouts of nausea and some quite minor headaches to which i had taken aspirin for and had put it down to a toothache . i am scheduled to work today and am wondering wether i should go to the hospital _ it is sunday here and my doctors is shut, or if i should be okay to wait until tomorrow. thankyou very much am waiting on your answer. Emz. oh, i am a 35 year old woman , i dont smoke but did heavily in the past (2 years off smokes) and i dont drink or do drugs, . i did heavily in the past however. Doctor: Hello and welcome to \u2018Ask A Doctor\u2019 service. I have reviewed your query and here is my advice. You should definitely go to a hospital and get it checked as what you are experiencing could be symptoms of stroke. History of smoking and drug abuse in the past can be a strong factor. I urge you to seek immediate medical attention and at least get basic labs done. Better safe than sorry.Hope I have answered your query. Let me know if I can assist you further.Regards, Dr. Samuel Parmar"
},
{
"id": 168077,
"tgt": "What causes fever and sore throat?",
"src": "Patient: my 6 year old has a fever of 102.8, is this normal, or should I be concerned he says he feels sick and has been laying down sleeping all afternoon. His throat s sore and I am concerned because I don't remember him ever having a fever that high, he rarely gets fevers. Doctor: Hi...by what you quote I feel that your kid might be having a viral sore throat or a streptococcal infection. I suggest you see your pediatrician and get a rapid streptococcal antigen test done. If it is positive then the kid will require antibiotics. If it is negative then it might be viral illness and doesn't require antibiotics at all.Cough and cold are viral 95% of the times in children. For cold you can use anti-allergics like cetirizine and for nose block, saline nasal decongestants will do. Paracetamol can be given in the dose of 15mg/kg/dose (max ceiling dose 500mg) every 4-6th hourly, that too only if fever is more than 100F. I suggest not using combination medicines for fever, especially with Paracetamol.Warm saline gargles will be soothing and will give a lot of relief if the kid can do that.Regards - Dr. Sumanth"
},
{
"id": 211193,
"tgt": "Are being aggressive when provoked, blackouts part of diagnosed schizophrenia?",
"src": "Patient: A friend of mine was diagnosed with schizophrenia. He seems to get aggressive when provoked, and at times goes into blackouts (he took the end of a pew ledge out of place during a meeting, then went into a blackout)t Is this part of the disease? We are part of a Christian church where people are concerned about the collective body, but I want to know how to stand by the individual and help church leaders understand him. Some are almost afraid of him, the pastor doesn't know how to help. He reacts to the aggression.. Doctor: Hi Schizophrenia is a disease where a person gets aggression with or without provocation. community psychiatry is one important aspect of management of the disease. You can take help of psychiatry social workers. These people will help you and your community to understand the disease, so that every one around you would know the type of behavior your friend needs. Finally your friend seems to have some florid symptoms.Make sure that he is taking medication regularly.Hope this answers your query. If you have additional questions or follow up queries then please do not hesitate in writing to us. I will be happy to answer your queries. Wishing you and your friend good health."
},
{
"id": 71897,
"tgt": "What causes multiple nodules at lung base after left kidney removal?",
"src": "Patient: My father has gone through left kidney removal last week. In CT diagnosis it was found that he is suffering from metastsis and multiple nodules were found in Lungs bases. where as in biopsy report Renal cell carcinoma was found with stage of pTb3 pN0 pMo. Mo means it is not metastsis. I want to confirm still there are chances that it is not spread in Lungs. Doctor: HelloAccording the history its not clear the biopsy of the lung nodules was taken or notRegardsDr.Jolanda"
},
{
"id": 21267,
"tgt": "Is swollen arms with passing out common in rhematic fever?",
"src": "Patient: My godson was diagnosed with Rheumatic fever in July 2009. He went into cardiac arrest and had open heart surgery to replace one of his severely damage heart valves. Since this time he has been in and out of the hospital. He is on a blood thinning medicine for the artificial valve but has been unable to maintain stable INR levels. He constantly gets swollen arms that require casts. He has passed out and now they believe he had a stroke. He is a 12 year old boy who best friends are the nurses in the ER. Is this typical for this disease? Doctor: Hello, thanks for posting a question.Yes, people with rheumatic fever have swelling around their ankle or knee joints. It is a common symptom amongst other symptoms in these patients. I really hope your Godson gets better as soon as possible. Best regard"
},
{
"id": 149335,
"tgt": "Get dizzy while standing and bending, all test normal. BP dropping and told to be vitamin shortage. Advise?",
"src": "Patient: all my test have came back normal, but sometimes when I stand up from sitting or bending way over I get a little dizzy to standing. we know my BP has been dropping 30 points when it happened in doctors office, he now thinks its a vitamin shortage. any idea's? also I am on blood thinners for I have both gene's, and my chronic condition is familial amyloidosis. but life is good, jon Doctor: when nothing comes in any test for any disease it is food linkedthe major component in our diet start reacting with normal body proteins and cause these types f problems i have tretd thousands of patients like thisget blood serum test for specific antibodies for milk wheat potato chana and oher food you take withdraw according to results and after food withdrawl you will be benefitted in few wk time"
},
{
"id": 69407,
"tgt": "How to get rid of hard lump inside my lower lip?",
"src": "Patient: Dear doctor,I seem to have developed a hard lump inside my lower lip (left side). It is about the same size and shape as a pea. I am male, 30, non-smoker, but I do enjoy a beer or a glass of wine from time to time.I cannot tell when this lump first developed. I discovered it by accident, when I noticed (in front of a mirror) that this part of my lip appeared ever so slightly swollen.Please advise if this is something I should be worried about.Thank you Doctor: Hi.Thanks for your query and an elucidate history.Nothing much to worry about this. This looks to be a simple sub-mucus cyst of the lower lip and a small surgery under local anesthesia can cure this. There is no medicines for such a lump so do not try and do not puncture as this will give you only temporary relief ."
},
{
"id": 94554,
"tgt": "Pain in abdomen, pelvic area. Taking doxycline. Negative STD tests. Causes for pain?",
"src": "Patient: Im a 22 year old male have moderate pain at times in lower abdomen and sometims in pelvic area on movememts like ggetting up from chair or doing a sit up. I have had a 8 panel screen for stds 4 months post exposure I was placed on doxycline from doctor because my original symtoms were an itch in mornings and night in lower scrotum and anus . All my test results came out negative for stds. Was an igg blood test with urine. Wondering what else could be causing my pain?? Doctor: thank you.have you any burning sensation during micturition?your pelvic pain is probably due to mechanical pain. do rutine exam. of urine & usg of kub region."
},
{
"id": 139610,
"tgt": "Suggest treatment for parietal granuloma",
"src": "Patient: i had two epilepsy attack in a day, 1 month ago. doter advised me to hav ct scan and said i hav parietal granuloma. i want know wht is the best teatment of it. and how long it will take to cure. i also smoke, 5 cigarette a day and also use zarda often. from last 2 year i was feeling that of my body limbs is not under my control(1-2% control) or say weekness in limbs. medicine i had from 1 month is waysolone, mycobutol, p-zide, epsoline, multivitmin, rabmaye d. there is no genetic problem. Doctor: Hello,If your neurologist or neurosurgeon thinks that the granuloma is responsible for the symptoms, it should be removed surgically. Anti-epileptics should be used long term too. Discuss with a Neurosurgeon for these issues.Dr. Erion Spaho, Neurologist, Surgical"
},
{
"id": 140306,
"tgt": "What causes fainting while urination?",
"src": "Patient: i woke up after having a short nap in the afternoon and went to the urinal. as i started urinating i fell down fainting. within seconds i resumed and got up. this is the second time i met with such an incident. can u tell why this happened and how to avoid in future Doctor: Hello, It is probably the lowering of your blood pressure after you get up quickly that is causing the faint. Next time, try to sit for a couple of minutes from laying in order to avoid that. If the condition repeats after such measures, get an evaluation by your Doctor. Hope I have answered your query. Let me know if I can assist you further. Take care Regards, Dr Erion Spaho, Neurologist, Surgical"
},
{
"id": 173590,
"tgt": "Is frequent bowel movement normal to a 1 year old?",
"src": "Patient: my 1 year old daughter is passing stools 7-8 times per day. is it normal? last week she got fever with diarrhoea. she was on bactrim for 5 days( tues to sat). but since yesterday she is passing stools 8 times a day but there is no fever.. please advice Doctor: HiWelcome to the HCMI completely understand your concerns but don't worry. Many a times, children develop hyperactive gastrocolic reflex after an episode of gastroenteritis. Due to this they may poop immediately after any oral intake. This is not associated with any other symptoms and the child looks otherwise well. It doesn't require any further treatment of antibiotics. So my opinion is to wait and watch. Hopefully, it will stop in the next few days."
},
{
"id": 62758,
"tgt": "Does lump under chin and muscle/bone indicate sign of cancer?",
"src": "Patient: Hey fellas,I have recently discovered a small, soft-ish lump under my chin, which is slightly moveable but visible from the outside and upon further inspection I found another hard unmoveable lump on the hard muscle/bone (I don't know much about anatomy) at the very top of my throat. I'm quite the hypochondriac as a couple of weeks ago I found a hard lump on the roof of my mouth which I immediately assumed to be cancer but that has since dissappeared. Anyway my question is... do the above symptoms sound like some form of cancer? Freaking out a little... Doctor: Hi,From history it seems that there might be having enlarged, palpable sub mandibualr lymph nodes produced movable and palpable lumps.This indicates that there might be having some chronic infection in the mouth, teeth, tonsils etc.Nothing to worry about cancer.Consult your doctor and get examined.Keep oral hygiene clean and healthy.Ok and take care."
},
{
"id": 141709,
"tgt": "Suggest treatment for neck stiffness and tingling in both arms",
"src": "Patient: tightness in neck and shoulders can not lie down or tilt backwards standing and sitting is fine.for last 2 years.chiropractor gives a bit of relief. any slight move muscles spasm. both arms are numb tingling and burning sensation mild is always there. Doctor: I would get a MRI of the cervical spine ordered to evaluate for possible \"cervical stenosis\" or a pinched nerve in your neck.The arms being numb and tingling with some discomfort suggests you have a issue with nerves in your neck."
},
{
"id": 101138,
"tgt": "What causes breathing problem at the time of season changes?",
"src": "Patient: .22years,5.8f,73kg,asthma since childhood,every year facing breathing problem at the time of change of season and in the month of april.i take asthalin inhaler 2or 3times a day.since last 1 year taking foracort inh.(200) twice daily. some times i take nabulizer(levolin).am i taking right treatment. Doctor: Hello.Thank you for asking at HCM.I went through your history. Please consider following points, they will be helpful to you:1. From your history, I would agree with your doctor that you are taking regular controller inhaler, that is, Foracort 200 twice a day. It is important for you to know that it is a preventer/controller type of inhaler and you have to take it regularly regardless of your asthma symptoms. Please review your treatment at least every 3 months with your doctor, who may increase or decrease your controller medications.2. Asthma inhaler or levolin nebulizer - both are similar type of drugs (rescue inhalers) for asthma patient. You can use either of them, whichever you are comfortable with.However, if you have to use them more than twice a week, it suggests your asthma is not well controlled and you should upgrade your regular(controller/preventer) medicines.3. Most important thing - please make sure your technique of using inhaler is perfect. If all the necessary steps of using inhalers are not followed, adequate amount of drug might not reach the lungs.4. Were I treating you, I would suggest add daily montelukast to above treatment, which may reduce your foracort dose. 5. Personally I would suggest you to get allergy testing done, which may identify the substances you are allergic to and guide you how to avoid them to improve asthma symptoms.6. In general, please try to avoid exposure to dusts, smokes and air pollution as much as possible. 7. Regular breathing exercises and a nutritive diet rich in vitamins and minerals (adequate amounts of green leafy vegetables, fruits, sprouts, etc) will also help you in a long run.\\Hope above points solves your query and will be helpful to you.Wish you the best of the health.If you have any further query, please feel free to ask at HCM.Thank you & Regards."
},
{
"id": 89466,
"tgt": "What causes stomach pain below rib cage?",
"src": "Patient: my stomach hurts, right below my rib cage. it's been hurting on and off for 5 days now. it's not a gasy, bloaty pain, but it is a sharp pain. and it feels like someone is squeezing my stomach really really hard. the pain comes and goes, but when it does hurt, it hurts REALLY BAD...and i don't have a fever, nor have i thrown up, so I don't know what's wrong. Doctor: Hello and WelcomeI appreciate your concern.I would recommend an ECG first, then an Ultrasound abdomen to rule out MI and Gall stones. Please fix an appointment with your doctor for the necessary workup.I would empirically recommend a two week course of aPPI such as Pantoprazole and a pro kineticlike itopride and an anti spasmodic to see for improvementwishing you best of healthThanks"
},
{
"id": 222985,
"tgt": "What causes blood discharge from the vagina during pregnancy?",
"src": "Patient: I m 37 weeks pregnant and today i went to use the bathroom and when i wiped myself it was little drops of blood on the tissue but not enough to fill the tissue and when i looked in the toilet there was no blood in the stool . What is going on? I m worried. Doctor: Dear Mumma,Any amount of bleeding at any stage of pregnancy is not a good sign. At this stage of pregnancy, if the bleeding is without any pain, first get the ultrasonography done for low lying placenta. If it is placenta praevia, since you are already 37 + weeks, there is no need to wait further with placenta praevia. The baby at 37 weeks is sufficiently mature enough. You can go for delivery - may be caesarean section if the position of the placenta demands for. First episode of placenta paevia bleeding is with little bleeding and the subsequent episodes are alarmingly heavy. Continuing the pregnancy with placenta praevia is like sitting on a sleeping volcano!May be it could be a spotting from the cervix which is eroded during pregnancy. In such case no intervention is needed. Please report to your Obstetrician ASAP to sort out the matter."
},
{
"id": 64267,
"tgt": "Suggest treatment for a lump on the forearm",
"src": "Patient: Yes, I have a lump below the skin on my right forearm. It is painful, swells up and not hard moveable. I had it checked by two different Doctors. I had surgery on it to get it removed, but the Doctor did a biopsy which came back negative. He said it was fat and if I lose weight it would go down. The second Doctor said it was fat also. I am still concerned that it is a cyst causing me pain. I want it removed, it is 1 inch in diameter, approximately 1/4 inch deep, guessing. Help don t know what to do. Don t have any images. Doctor: Hi, dearI have gone through your question. I can understand your concern. You may have some soft tissue tumor like lipoma or some benign cyst like sebaceous cyst. No need to worry. If it is painful then you should go for excision biopsy. It will cure your lump and give definite diagnosis. Consult your doctor and go for excision. Hope I have answered your question, if you have doubt then I will be happy to answer. Thanks for using health care magic. Wish you a very good health."
},
{
"id": 6615,
"tgt": "Why do I have a light coloured discharge even after pregnancy test shows negative results ?",
"src": "Patient: \\ hello i have a question, i had abortion dec 29 i bleed until sumtime in feb. then i been haven periods its seem twice amonth MARCH I GOT IT ONE DAY WHICH WAS THE 16TH .in apirl my period came on the 17th. last month i got my period on the 21th and then again on the 30th the one on the 3OTH WAS BROWN/BLACK DISCHARGE NOT ENOUGH FOR A PAD OR A TAMPOND I TOOK A PT TEST AFTER AND IT WAS NEGATIVE. THIS MONTH ON THE 7TH I GOT LIGHT PINK DISCHARGE FOR TWO DAYS CAN U PLEASE HELP... Doctor: hello miracle, welcome to hcm generally it is common and normal to have intermittent bleeding or brown discharge for weeks to months after the Abortion .but its above 5 months now.is it associated with foul smell and pain? you need a detail medical evaluation by your gynecologist. take care"
},
{
"id": 95696,
"tgt": "Will i get my period after taking vdm kit?",
"src": "Patient: hello doctor ,i m 24 yrs old.in this month my periods is 10 days delay.and also suffer from white discharge.doctor prescribed me VDM-KIT.m2 tone,or also suggest for thiroid testing..my question is that after taking the vdm kit my periods is start or not??my height is 5.6ft and weight 68kg.. Doctor: Hi Madhumati, Welcome to HCM. VDM kit is for the treatment of white discharge not for your periods. M 2TONE will help you for regularising your periods.Wiat for sometime still if you dont get your periods then you need to check your hormone levels and if required probably a scan. Wish you good health."
},
{
"id": 125835,
"tgt": "What causes discomfort under the small toes?",
"src": "Patient: On my right foot only, under the two small toes, I have a sensation that my socks wrinkle in this area. It is not painful. Since I like to walk a lot, it is just annoying. I don t have this sensation wearing sandals as I don t use socks while walking on them. Could you please tell me what causes this feeling? Should I consult a doctor? What type of specialty doctor can advise me on it? Doctor: Hello, It could be an early sign of arthritis like gout. Consult an orthopedician and plan for an MRI scan. As of now, analgesics like Tramadol or Aceclofenac can be taken for pain relief. Hope I have answered your query. Let me know if I can assist you further. Regards, Dr. Shinas Hussain, General & Family Physician"
},
{
"id": 166944,
"tgt": "Is the phototherapy harmful to the baby?",
"src": "Patient: Hello Doctor...I am very happy that I have been blessed with boy baby on 2nd April 5:37 pm.Mother and the baby are fine.I wanted to have few clarification on the treatment provided to the baby.The doctor had advised to take blue fin test on 3rd april and test are as follows Free T3 Sample Serum Result 1.03 freet T4- 1.09 and TSH 9.67.Today morning when the pediatrician has advised to keep the baby in the photo therapy and has told us take one more blood test tomorrow morning.My question and want a suggestion from your end the following 1.Is the phototherapy harmful to the baby in any way ? what is the caution to be taken 2.is it fine to to keep taking blood test within the two days of the birth of the baby All i really understand its for the traetment and i am very much confused because i dont have any clarity on this from the doctor side Doctor: hi! I understand your concern. the baby had been started on photo therapy for jaundice. the baby may develop a rash due to it but that goes in a few days.also using protective covering for the eyes and genitals is required. but treating jaundice is more important than these side effects as high jaundice may cause brain damage. regarding blood tests whatever important investigation is required need to be done for proper treatment."
},
{
"id": 127601,
"tgt": "What causes pain in the right shoulder?",
"src": "Patient: I have been having right shoulder pain for about 36 months when raising it beyond 90 degrees. Diagnosed with clavicular tendonitis and got injections. No difference to the pain. Have pain when layimng on my right side and some sharp numbing pain sometimes when holding my cup of coffee with it tending to fall Doctor: Hello and Welcome to \u2018Ask A Doctor\u2019 service. I have reviewed your query and here is my advice. You may have frozen shoulder or cervical spondylosis, first go for MRI Scan. Hope I have answered your query. Let me know if I can assist you further."
},
{
"id": 190246,
"tgt": "Intense pain at the back of mouth, small lump at the palate side, no relief with medicine. Worrisome?",
"src": "Patient: I had an intense pain in the right side at the back of the mouth. The first dentist I saw said it is an infection. Took antibiotics for the 3 day course. The pain was the same. After 2-3 days more, the pain subsided on its own. Now I have a small lump on the palate side at the back of the mouth. It is not painful but is not going away or getting bigger for the past 3 weeks. the OPG xray showed that the wisdom tooth is impacted but it s not hurting right now. I am worried what that bump could be Doctor: Hello As per information you have given this lump is a thing of concern. If it is due to wisdom tooth then you should go for removal of this impacted tooth.For the time being you can do warm saline rinses four times a day and should avoid starchy food.This will certainly reduce the swelling and provide you relief. Best Dr. Amit K. Gaba"
},
{
"id": 52370,
"tgt": "How can jaundice be treated when there isn't much improvement in the SGPT, SGOT and alkaline phosphatase counts?",
"src": "Patient: Dear Sir/ Madam, I have been suffering from Jaundice since last one month.My Bilirubin count is now 0.9mg/dl however there is not much improvement in my SGPT /SGOT/Alkaline Phospatase count.Please find below my last 15 days Report. Dt-17/6/2017-SGPT-132/SGOT-121/Alkaline Phospatase-152 Dt-8//6/2017-SGPT-146/SGOT-112/Alkaline Phospatase-319 Please advise as how long it will take to come to a normal level or if i have to take some additional steps as i am strictly having home boiled food without any spices and also having only boiled water. Doctor: you are improving subject yourself to clinical examination look for enlarged liver icterus and appetite level they are signs of progress exclude hepatitis b and c since you have to take specific treatment and confirm improvement are you imunised against hepatitis b. otherwise you are improving. have you got nausea and vomiting. take bland diet. less oil and spices keep watch on your improvement. repeat liver function test."
},
{
"id": 220299,
"tgt": "What are the early signs and symptoms of pregnancy?",
"src": "Patient: i recently had unproteced sex with my boyfriend...i am supposed to start my period on the 13th... i was reading on the internet and it said that it should take 7-10 days b4 implantation occurs so why are my nipples are already sensitive to the touch && what are the chances of me being pregnant? Doctor: Hi, Dr Purushottam Neurgaonkar here. I welcome you to HCM VIRTUAL CLINIC. I have gone through your question, and I think I have understood your concern. I will suggest you the best possible treatment options. 1 ) First of all do not panic. 2 ) in a woman with regular cycles day 10 to 20 of the cycle is the most fertile period of cycle. If you had unprotected sex in this period, then You Stand chance to get pregnant. 3 ) To confirm Pregnancy, I will suggest you to get mornings first sample of urine tested for pregnancy. If at all you miss your periods, please do urine pregnancy test. 3 ) There are many other symptoms of early pregnancy. Missing periods is the most common symptom. Nausea , vomiting, breast pain, frequent urination, dizziness are some other symptoms. 4 ) Even If you are pregnant, you can opt for termination with pills safely till 9 weeks of the pregnancy. I hope this answer helps you. Thanks Dr Purushottam Neurgaonkar"
},
{
"id": 16685,
"tgt": "Could rapid heart beat be related to blood sugar levels?",
"src": "Patient: When my husband goes to bed, some nights he wakes up after about an hour with a rapid heart beat. He eats right and exercises. His doctor says he is very healthy. Some nights he sleeps very well. He is wondering if this has something to do with blood sugar. He is on a low carb diet and is at his ideal weight. Thank you. Sue Doctor: Hello, His symptoms could be related to a sleep disorder. I don't think it is hypoglycemia, as it does not cause tachycardia. Coming to this point, I would recommend taking melatonin at bedtime and see if this improves his situation. If this does not help, I would recommend consulting with his attending physician for a physical exam, cardiac ultrasound and an ambulatory 24/48 hours ECG monitoring in order to examine his heart rhythm trends during the night. Hope I have answered your query. Let me know if I can assist you further. Take care Regards, Dr Ilir Sharka, Cardiologist"
},
{
"id": 53949,
"tgt": "Suggest treatment for jaundice and itching on skin",
"src": "Patient: Doctor I am having a disease call BRICK Iam getting Itching and after that jaundice iam 36 years old I got this more than 8 times doctor suggest only Udilive tablet and its getting cured within few days? they said its may be herritory is there any medicine to cure this permanently?there is no block or stones in my liver ultra sound report says,liver biopsy also normal please help me Doctor: Hi and welcome to Healthcaremagic. Thank you for your query. I am Dr. Rommstein, I understand your concerns and I will try to help you as much as I can.Jaundice is sign of some pathology and exact cause should be done. Then it can be treated. In most cases this is caused by bile duct obstruction and you need to do ct scan or ultrasound at least to see is there stone or some other blockage.I hope I have answered you query. If you have any further questions you can contact us in every time.Kindly regards. Wish you a good health.DR. Ivan Rommstein"
},
{
"id": 17933,
"tgt": "Suggest remedy for heart ailments",
"src": "Patient: My mother is 67 over weight , has a pace maker ,st jude valve. defib . micro valve defib ( ? ) saying right? taking BP meds Thyroid, Blood thinners ( boy does that number go up and down.) Her BP is 106/56 her defib was not put in correctly had to redo, went off 40 times. her BP has changed from what is Norm for her . Should I be on stand by? Doctor: Hello, After going through your medical details I understand your concern and I would like to tell you that there is no remedy which can help her for her set of diseases and is advisable not to do any herbal experiments in such conditions. Hope I have answered your query. Let me know if I can assist you further. Take care Regards, Dr Bhanu Partap, Cardiologist"
},
{
"id": 98286,
"tgt": "How can asthma be managed when uncontrollable cough leads to respiratory failure?",
"src": "Patient: I am a 60 yr old female diagnoised with asthma as a child. had no asthma problems for 50 yrs. 10 months ago i was in a departmemnt store and something triggered it. Uncontrolled coughing the ability to take air in but not expell it caused my lungs to constrict and close causing complete resp. failure. I had to be intubated, was unable to breathe on my own. was on life support for 2-3 days. couple of weeks after discharge i was suppose to see a lung Dr. but before that could happen my husband was let go at work causing use to loose our ins. he was off work 7 mths. It was a struggle to pay for medications alot of them we couldn t. i quess what i need to know is how to posssibly keep myself safe until i can afford to see a lung doctor Doctor: HiThanks for positng your query on HCM.I have read and undewrstood your concern.Sorry to hear what you have gone through.Asthma is an illness where prevention and long term control are very important to avoid attacks.While waiting to see the Lung specialist, i will advise you try the following:-Get a Long-term asthma control medication like Seretide (fluticasone and salmeterol) for daily use.-Quick-relief (rescue) medication like albuterol for use when attack or when you feel symptoms.-Use your air conditioner-Decontaminate your decor-Maintain optimal humidity-Get regular exercise-Maintain a healthy weight-Monitor your breathing-Identify and treat attacks earlyThese may help till you see your Lung specialist.You may write back if more concerns.Wish you good healthDr Mbuomboh"
},
{
"id": 117512,
"tgt": "Can furosemide be used to induce diuresis?",
"src": "Patient: hi,Iam a medical student.I want to ask if i can use furosemide to induce diuresis incase of oliguria in a patient that had hypovolaemic shock due to abruptio placenta after she was managed with shock therapy inaddition to 1000cc blood.N.B pt. is preeclamptic Doctor: If hypovolemic shock is corrected and the blood pressure is back to normal and there is normal JVP and no dehydration then maybe frusomide may be tried for oliguria,hopefully there may be no urine retention and bladder is empty on percussion or catheterisation been tried. It will be safer to start simulataneous iv fluids for electrolyte and fluid balaance.If your system permits take consultation from a physician if available in premises or nearby for additional information and guidance"
},
{
"id": 212996,
"tgt": "Frequent urge to urinate, on clonazepam, effexor, levothyroxin for depression and anxiety. What is the cause?",
"src": "Patient: hello, just turned 57 130lbs 5 5 Iv been taking levothyroxin for about 3wks,( i get the urge to urinate often), if not near a bathroom im skrewd, its 25mcg. I had a complete hystorectomy 35 yrs ago not on any hormones, i was giving this drug to see if it would help w/servere out burst of depression an anxiety , also taking clonazepam 0.5mg and effexor 75mg 2in morning and 1 in afternoon, thankyou Doctor: Hi, Frequent urination is a symptom of anxiety and depression too. There is also need to rule urinary tract infection, if it is associated with burning and fever. If problem is very severe then for review of treatment, you can visit your psychiatrist. I hope this information has been both informative and helpful for you. Wish you Good Health. Regards, Dr. Ashish Mittal www.99doctor.com"
},
{
"id": 175854,
"tgt": "Suggest treatment for severe cough, wheezing and fever in an infant",
"src": "Patient: Hi my 5 month old and 7 year old were taken to the dr last tuesday for coughing and wheezing they both had same symptoms but the cough has been there forever it seems like but now they both have raspy coughs with flem and the dr said there was wheezing he put them both on a steroid and a inhaler til cough is gone he said it would be about 4 to 5 days ...well the cough is still there and now my 5 month old is running a fever of 101.8 what do I do Doctor: Hi...it seems to me like both your kids are down with the same virus and this is causing wheezing and it is viral associated wheeze. Your pediatrician was right in staring steroid and inhaler. Fever of few days without any localizing signs could as well a viral illness. Usually rather than fever, what is more important is the activity of the child, in between 2 fever episodes on the same day. If the kid is active and playing around when there is no fever, it is probably viral illness and it doesn't require antibiotics at all. Once viral fever comes it will there for 4-7 days. So do not worry about duration if the kid is active.Paracetamol can be given in the dose of 15mg/kg/dose (Maximum ceiling dose 500mg per dose) every 4-6th hourly that too only if fever is more than 100F. I suggest not using combination medicines for fever, especially with Paracetamol.Regards - Dr. Sumanth"
},
{
"id": 212648,
"tgt": "Start stammering in the mid of seminar, moral lowers on reminding the situation. How can I fix this?",
"src": "Patient: hello ,is anyone here? i am a student of computer science field and pursuing m.tech in it. so in my field generally we have to talk to persons and have to give presentation..but whenever i am giving any presentation i start to stammer,not only that whenever i think of those situation that i have to give presentation on that day my moral goes down...suggest me some way to get free from it.i am facing this problem for the last 3 years and thus loosing my confidence. mail YYYY@YYYY Doctor: Hi there ~ Clearly your stammering is related to your anxiety of performing in a social situation. This is also known as performance anxiety and is easily treated with beta blockers. Also anti anxiety medications can help. Therapy can help regulate your stammering and boost your confidence and self esteem. Please consult a psychiatrist as soon as you can to find your remedies and referrals. I hope this helps. Take care."
},
{
"id": 110,
"tgt": "Can pregnancy happen after having protected sex?",
"src": "Patient: yes me and my girlfriend had sex last night and we used a condom. but she is not on birth control. so when when i released i kept going a little bit not much. but when i pulled out the condom like un rolled more, so the end was longer but it looked like everything was sucked out of it. is there a chance she will get pregant. Doctor: ya she can get pregnant if u have ejaculated inside...if semen entered inside..she wil ge pregnant provided u have done sex on unsafe period...between day 11 to day 18 on a regular 28 days cycle..."
},
{
"id": 205928,
"tgt": "Suggest treatment for anxiety and tiredness",
"src": "Patient: My roommate just started on lorazepam (0.5mg) exactly one week ago and has been taking 4x/day for the past week. He has been incredibly stressed over work and recent death in the family and started feeling depressed and anxious. The first couple of days he took lorazepam, the anxiety level seemed to have gone down and he seemed more relaxed. Then a few days into it, I started noticing that he seemed more anxious, more unhappy and completely depressed. He just wants to sleep all day and cannot seem to function at all. He seems worse than before he started his meds. The other medication he is taking is Lexapro 10mg once a day. I think he may be taking the wrong meds for him, especially the lorazepam. His doctor told him to take 2-3 lorazepam per day vs 4 when my roommate called him to tell him that he is feeling worse than ever before. He also takes antabuse which he has been taking for a year now, everyday, without missing one single dose. I am not sure lorazepam is the correct anti anxiety pill for an alcoholic taking antabuse. Should he start weaning off lorazepam? How? He's only been on it for a week. He only took one dose today and he's a big mess. He said he feels so anxious and tired. He looks like he's about to have a breakdown and physically he just looks miserable. Should he be weaning off Lexapro (escitalopram) as well? If so, how? Please help. I want to help my friend. He will be seeing another doctor early next week (earliest he can see one due to the holiday) but would love you know how I can help in the meantime. Doctor: HelloWelcome here.I can understand your concerns for your friend.I have some suggestion for you:Antabuse drugs are usually not prescribed for longer durations. As soon as the person feels that he has string enough will power, there is no role of antabuse drugs further.Lorazepam, at doses which your friend is taking, should decrease his anxiety.Now if his anxiety has increased even after takibg lorazepam, this is most probably due to interaction between antabuse and lorazepam.Lexapro is an anti-depressant and anti-anxiety drug. However in some cases it can increase anxiety for initial few days.My advice for your fruend would be just to take single drug lorazepam till he sees his doctor. Antabuse can be tapered off by reducing one-third dose every three days and lexapro can be stopped for now.Hope this helps.Thanks"
},
{
"id": 109236,
"tgt": "What causes itchy spine/back during pregnancy?",
"src": "Patient: My wife is 4 months pregnant and has been having an itchy spine/back for a while, when she takes pre-natal supplements it is worse. We can't afford to see a doctor and get tests. What could be causing her back to itch? She says it could be that the baby has a blood type that she is allergic too, is this possible? Doctor: HIGreetings from Dr.Divakara.PThanks for posting your query. No that is not the cause of Itchy back . Most likely it is due to an allergy to one of her pre natal supplements. I advise you stop one of the supplements for a week and observe , if there is a relief from itching then ask your doctor for an alternative for that supplement. If there is no relief then restart that supplement and stop the other supplement and observe. Till then she can apply a cream called Betnovate over her back . Ask your doctor for the prescription.Hope this information is useful to you. Any clarification feel free to ask."
},
{
"id": 154071,
"tgt": "What causes breathlessness and weakness when having cancer?",
"src": "Patient: I was recently diagnosed with colon and liver cancer. Had a bowel resection 6 weeks ago. I was going to start chemo, but came down with shortness of breath and feeling weak. Went to emergency room and was tested for blood clots. Had EKG's and wore a heart monitor. That was normal. I started feeling better and was discharged. Symptons are back. Wondering what it might be. Doctor: Hi,Thanks for writing in.Chemotherapy is treatment with strong medicines which kill the cancer cells. Since you are having colon cancer with liver involvement therefore its possible that you are taking combination chemotherapy. This involves more than one chemotherapy agent and might be aggressive in action. When chemotherapy works then the cancer cells get destroyed in large numbers. These deal cells are mobilized in circulation and cause side effects like weakness and fever as there is release of low amounts of toxin.Some chemotherapy agents also cause toxicity to the lungs and muscles and please discuss this with your doctor. You must also get liver and kidney function tests repeated to know if there is any abnormal function due to taking chemotherapy. Please do not worry."
},
{
"id": 161150,
"tgt": "Suggest treatment for nausea, dry-heaves and diarrhea in an infant",
"src": "Patient: Hi my 21 month old toddler has had diarrhea the past 3 - 4 days. He has been refusing to ingest anything besides milk and rice cereal. If I try to get him to eat anything else he gags and on one instance puked. His poops have been gray/white/tan in colour as well. He has been generally happy and playful though. Does this just sound like a stomach bug or could it be something more serious? Doctor: Hello, If a similar patient comes to my old as u have mentioned that child is playful Nd happy, it's nothing serious. you can give Ors sachet mixed In 1 liter of boiled water Lukewarm within a day, Sporolac sachet two times a day and zinc 5 ml at night. as far as the motions don't contain blood you don't have to worry. Hope I have answered your query. Let me know if I can assist you further. Take care Regards, Dr Jilu Joseph, General & Family Physician"
},
{
"id": 57478,
"tgt": "Can one with liver cirrhosis take anti-allergic medicines ?",
"src": "Patient: ATIF PAKISTAN Sir CAN ANTI ALLERGY SUCH AS LORIN NSA AND MYTEKA BE TAKEN IN LIVER CHORRISS OR LIVER PROBLEM IF THIER AN ALLERGY AND MUTLI VITAMIN HAVING ZINC BE TAKE IN THE ABOVE PROBLEM Doctor: HIThank for asking to HCMI really appreciate your concern if you do have liver pathology then medicine should be taken with all the precaution because in liver pathology all the drug are highly contraindicated, only in highly indicated situation it has to be taken with the great care otherwise try to avoid the medicine as long as it is possible, hope this information helps you have a good day."
},
{
"id": 214128,
"tgt": "Is this a learning disability?",
"src": "Patient: Hello, I have a question about my learning capabilitys. Lately, i have been having the most terrible time trying to comprehend the material everyone is understanding . I feel as if i am the only one having the problem. When i am in class, i try to..lets say.. adsorb the information and material we are trying to learn. I can't seem to get engaged in learning. I feel helpless and going home or getting help isn't helping much either. what would you recommend? Doctor: Counselling could be recommended ince it will be easy to understand what exactly are the problems you are facing. An appropriate modality of therapy could be decided if something is unusual."
},
{
"id": 96245,
"tgt": "Totally health loss in ulcerative colitis",
"src": "Patient: hi docter i m 23 yrs old girl n dignosed wid ulcerative colitise.i have problem of loos motions daily 7-10 since i was 13 yr old but my docter could nt find the particular reason but before two year my doc told me that i have ibs then aftr one doc find that i have have ulcerative colitis as my bloody stool is.n mucus also n lower left side pain in my abdomen but now my docter say that they have givven me all medicines,but i have no relief by my pain i m on treatmnt.n i cant eat any thing except daal n roti,pllzz suggest me what shud i do?i m really tierd n i m anemic too Doctor: thanks for choosing health care magic.Ulceration colitis is problem of intestine in which there are ulcers in your intestine.This leads to pain and other problems that you have.But this disease usually have remissions in between means the disease will be active for some time and then the symptoms will be resolved for some time and appear again.Usually medicines are effective along with diet control so do not get dishearten.Take medicines and diet as advised."
},
{
"id": 60544,
"tgt": "My protein level is 7.3g/dl and my age is 28 years .What is my stage ?",
"src": "Patient: hello doctor ..im 28 yrs age frm nepal..my current report is Hbeab+ve, Hbeag-ve, Total brilium=0.5mg/dl, c brilium=0.10mg/dl, ALT=19.0 iu/ml, AST=23.5iu/ml, Total protin=7.3g/dl, Albumin=4.8g/dl...and HBV DNA Viral copies/ml plasma=750...so doctor i want to know my stage of disease and condition of liver ....should i start medicitions..?..if i start how long i ll take an what is it cost per month..? Doctor: Welcome to Healthcare Magic Good Day You do not have significant liver damage. Since your HbeAb has appeared, your infection is being contained and you should recover soon. Your HBV DNA Viral copies are not clinically significant. Don't worry. You should be fine soon. Eat fat free food for 2-3 weeks. Eat plenty of fresh fruits and vegetables. Drink enough water. Get the test repeated after 2-3 weeks or as your Doctor has asked."
},
{
"id": 58081,
"tgt": "Abdominal pain, vomitings after having fatty meal. Can gall bladder disease cause elevated liver level?",
"src": "Patient: what s the pattern and level of elevation of lipase, amylase, alkaline phosphatase, AST,. ALT, and bilirubin in primary pancreatitis, vs gall bladder disease, with Day 1 being the acute onset of abdominal pain, and eventual vomiting 1-2 hrs after consumption of a fatty meal (2 hot dogs), in a patient who was well 2 hrs before Doctor: hi, in gall bladder desease may rise alkaline phosphatase, AST, ALT, and bilirubin, its depend on which condition or type of gall bladder desease.in acute pancreatitis rise both amylase and lipase level but lipase level is most remarkable."
},
{
"id": 134188,
"tgt": "Suggest remedy for pain in ribs and shoulder blade",
"src": "Patient: Hi My partner has had pain just below and under ribs on left side which is worse when lying down. Now some pain below left shoulder blade. This has gone on for 4 nights, he has had blood and urine tests all fine. X-ray and ultrasound all fine except for steatisis and a 9mm paraplegic cyst middle third of right kidney. Bowel motion 4 times a day. Thanks Doctor: hi,thank you for providing the brief history.as you mentioned there is a pain under the ribs and shoulder blade it is more of a muscular since the x-ray and ultrasound are normal. Also using hot water fermentation initially will help soothing and later on exercises which includes the breathing exercises , in particular the costal breathing exercises and later on the Strengthening exercises. as the practice will continue the muscles will become stronger and the pain will come down to a greater extent. Also, correction of posture is also important.with the grace of God I wish your partner a good health.regards"
},
{
"id": 98958,
"tgt": "Can tuna fish with herb seasoning cause red rashes all over the body?",
"src": "Patient: We finished eating our meal about one and a half hours ago and my partner has just come to see me because he felt rather hot and noticed a bright red rash on his body (chest, back and neck). He says it doesn't itch and doesn't appear to be swollen. This has never happened to him before. It is quite an even rash, not blotchy just quite red, like sunburn. We had tuna fish steaks (cooked), peppers, mushrooms, sugar snap peas and potatoes cooked in olive oil with an italian garlic and herb seasoning. This has never happened before when we have had tuna fish. Any ideas would be appreciated - thank you, Valerie, U.K Doctor: Nothing to worry. It's mild local reason. If persists for few days, may take own herbal med in kitchen. Turmeric powder + Sugarcandy powder will address it. Additionally, for better benefit, get Neem leaves powder or Neem capsules of Himalaya co. easily available in UK. Even turmeric capsules are manufactured by Himalaya co in the name of Haridra.These are natural anti allergics and clear toughest of rashes like urticaria also. No side effects. May be slow in action.Hope it helps."
},
{
"id": 109800,
"tgt": "What causes pain in neck/shoulder while having sciatica in lower back?",
"src": "Patient: hi my names emma coxon iam 27yrs old .i have been diagnosed with siatica in lower bk. but 4 last 2years ive been aceing first thing on a morning it seems to be all my musels in my boady that ace. this is verry painfull i find it realy hard to get out ov bed. an have allot ov neck and sholder pain 2. and bad head aces thay are only from time to time . i would b verry grate full if u could give me an idea whats wrong with me . thanks E COXON. Doctor: Hello, Thanks for your query. After going through your query I think you are having muscle generalized pain all over the body. In my opinion these pains are likely to be due to vitamins and trace element deficiency specially that of vitamin D. I will suggest you to got following test to see cause (if these are not done earlier).1.Serum Vitamin D2.Serum Vitamin B12 3.T3,T4,TSH(Thyroid profile) 4.Blood sugar fasting and PP 5.Hb Furthermore Empirically treat my such patients by:-1.EVION LC 2.VITAMINE D3(if it is low) 3.VITAMIN B COMPLEX AND VITAMIN C 4.CALCIUM 5.Mecobalmine 6.ANALGESICS such as diclofenac for pain relief.You can discuss with your treating Doctor about it.I do hope that you have found something helpful.If you have additional questions or follow up queries then please do not hesitate in writing to us. I will be happy to answer your queries. Wishing you good health.Take care."
},
{
"id": 210673,
"tgt": "How to treat Bipolar disorder and addictions in a person?",
"src": "Patient: I believe I mean I know my husband is abusive .he doesn't hit me but he is verbal he blames me for thing so he can feel better about himself. He is bipolar and blames his lashing out on the desease that I don't understand he is an acholic, drug addict, gambler, sex addict . He always has to have some addiction of some kind all the time. Doctor: HIThanks for using healthcare magicI think, he has underline borderline personality disorder. Number of patients of borderline personality misdiagnosed as bipolar disorder. Better to consult a psychiatrist for proper diagnosis. In that case, you need some mood stabilizer that would help to control his mood. You can also take help of psychologist for personality assessment.Thanks"
},
{
"id": 91201,
"tgt": "What causes pains in my lower abdomen near my ovaries on the LHS?",
"src": "Patient: I am having pains in my lower abdomen near to my ovaries on the left hand side. These come and go over time, but tonight they are very painful. The pain is a sharp pain that doesnt go away. Sometimes it is in my ribs as well. I am also frequently urinating. Doctor: hello madam, thanks for writing to us, as per your above mentioned symptoms the pain site can be a cause of concern. the reason for pain in that area could be either for some pathology in the ovary or in the kidney of left side. but this can only be confirmed after an USG of abdomen done,therefore i would request you to go for the above mentioned investigation along with other regular investigations. thank you and wish you good health"
},
{
"id": 185150,
"tgt": "What causes swelling in gums under tooth?",
"src": "Patient: I have as willed left cheek and vertical raised lines running down it. The gums around my left bottom wisdom teeth are also sore and swollen and it's been like this for 2/3 days. I'm 19 and as far as I know I'm healthy. Anything I can do? Or should I make a doctors/dentist appointment? Doctor: Thanks for using Health Care Magic.Read your query.The swelling in relation to your wisdom tooth can be because of pericoronitis which occur during it eruption and when it gets inflamed .I would advice you to take a pain killer like ibuprofen ( if you are not allergic to any medicine) and amoxicillin .Do warm saline gargling.Keep the area clean and avoid food debris.If still persistent visit your dentist and get a X-Ray done to know the exact location if the wisdom tooth.Hope this was useful.Thanks and regards."
},
{
"id": 22768,
"tgt": "Suggest treatment for tachycardia",
"src": "Patient: Dr . my age is 28 , from past 10 years i am facing sinus problem , i am taking medicine from ENT specialist , last month i faced breathless ness and abnormal heartbeats , it was checked by Cardiologist and went for ECHO everthing is normal he written trachycardia seen , and in blood content eosinophill content is 8 % it is due to allergy he suggest and given prisciption . is any problem please suggest me to maintain good health. Doctor: Hitachycardia is a condition in which the heart beats faster then the normal rate , it can be because of anxiety ,anaemia, medicines and a number of causes, infection can also be a cause of tachycardia.mild tachycardia is not a problem but if heart rate is more then 120 in that case low dose propranolol should be taken with ent medicines.exercise daily for 45miutes, walk for 30 minutes,avoid too much stress,dont eat heavy foodthank you"
},
{
"id": 172819,
"tgt": "What causes headache in a child after using Cetirizine hydrochloride?",
"src": "Patient: My 8 year old has a runny nose and sore throat - thought it was hay fever and gave him 5ml of cetirizine hydrochloride. He now says he has a headache and can t sleep - been like this for 1 1/2 hours - can I give calpol or calprofen 1 hour after piriteze? Doctor: HiSure you can give him Colpol 15mg/kg/dose . headache can be because of the common cold, and is very common symptom.Thanks"
},
{
"id": 190544,
"tgt": "Back molar pain, painful blister on molar. X-ray done. What could be the reason?",
"src": "Patient: Hello, I have seen a dentist for my back molar being in a lot of pain. There were no restorations on the tooth and the dentist did xrays and said there was nothing wrong and dismissed me, three days later I went to the emergency room and they found nothing to be wrong that they could find without a dentist on staff, I have now dwveloped a very large and painful blister stemming from the tooth that had origionally caused me pain and it concerns me a lot what could be wrong?! Doctor: Hi, After reading your symptoms i understand that it is related to your wisdom tooth and there is no caries(bacteria in tooth) as your dentist didnt find anything on xrays. Abnormal position of the wisdom tooth and the gum covering it can cause sever pain. assuming it is the wisdom tooth, it is not unusual or rare so you should not worry too much. Due to the tooth being at back and malpositioned food lodgement occurs resulting in infection and swelling of the gum tissue in the area, which leads to ulceration due to improper biting while eating resulting in pain. Warm saline gargles(3-4 times/day) along with an antibiotic course with a painkiller will help you temporarily. I would strongly advice you to visit your dentist to identify the problem and treat it accordingly. Regards"
},
{
"id": 107924,
"tgt": "What causes constant shooting pain in my lower back?",
"src": "Patient: why do i have constant pain in my mid to lower back that sometimes shoots up to the upper part of my neck. it started when i was about 7 months pregnant my ob told me it was just the baby laying on a nerve its been 6 months since i had my baby and i still have this problem Doctor: such types of symptoms some times can be seen in post pregnancy period.the pain persist due to laxity of back muscle and improper physiotherapy after delivery . lack of rest also aggravates these symptoms. back muscle strengthening and pelvic floor exercises will be beneficial to you.For this you need expert's guidance by rehab specialist and physical therapist .oral calcium and analgesics such as ibuprofen will give instantly relief.Hope this answers your question. If you have additional questions or follow up questions then please do not hesitate in writing to us. I will be happy to answer your questions. Wishing you good health."
}
]
}